{"question": "\uc601\uad6d\uc758 \ucd5c\uace0\ubd09\uc740?", "paragraph": "\uc9c0\ud615\uc801\uc73c\ub85c\ub294 \ub3d9\ub0a8\ubd80\uc758 \uc800\uc9c0\uc640 \ubd81\uc11c\ubd80\uc758 \uace0\uc9c0\ub85c \ud06c\uac8c \ub098\ub25c\ub2e4. \uc774 \ub458\uc758 \uacbd\uacc4\ub294 \ub3d9\ubd80 \ud574\uc548 \uc911\uc559\ubd80\uc758 \ud2f0\uc2a4 \uac15 \ud558\uad6c\uc640 \ub0a8\ubd80 \ud574\uc548\uc758 \ub77c\uc784\ub9cc\uc744 \uc5f0\uacb0\ud558\ub294 \uc120\uc774\ub2e4. \uace0\uc9c0\ub77c\uace0 \ud574\ub3c4 \ub300\ubd80\ubd84\uc740 \uace0\ub3c4 1,000m \uc774\ud558\uc758 \ub0ae\uc740 \uc0b0\ub9e5 \ubc0f \uad6c\ub989\uc73c\ub85c\uc11c \uc601\uad6d\uc758 \ucd5c\uace0\ubd09 \ubca4\ub124\ube44\uc2a4 \uc0b0\ub3c4 1,343m\uc5d0 \uc9c0\ub098\uc9c0 \uc54a\ub294\ub2e4. \uace0\uc9c0\ub294 \uc9c0\ud615\uacfc \uc9c0\uc9c8\uc774 \ub2e4\ub978 \ub2e4\uc74c\uacfc \uac19\uc740 \uc9c0\uad34(\u5730\u584a)\ub85c \ub098\ub25c\ub2e4. \uc2a4\ucf54\ud2c0\ub79c\ub4dc \uace0\uc9c0\uc640 \ubd81\uc544\uc77c\ub79c\ub4dc\ub294 \uc8fc\ub85c \uce84\ube0c\ub9ac\uc544\uacc4\uc758 \uac00\uc7a5 \uc624\ub79c \ubcc0\uc131\uc554\uc73c\ub85c \ub418\uc5b4 \uc788\uc73c\uba70, \uc2a4\ucf54\ud2c0\ub79c\ub4dc \ub0a8\ubd80 \uc0b0\uc9c0, \uce84\ube0c\ub9ac\uc544 \uc0b0\uc9c0(\ud638\uc218 \uc9c0\ubc29) \ubc0f \uc6e8\uc77c\uc2a4\ub294 \uc8fc\ub85c \uace0\uc0dd\ub300 \uc2e4\ub8e8\ub9ac\uc544\uacc4\uc758 \ubcc0\uc131\uc554\uc73c\ub85c \ub418\uc5b4 \uc788\ub2e4. \ud398\ub098\uc778 \uc0b0\ub9e5\uacfc \uc2a4\ucf54\ud2c0\ub79c\ub4dc \uc911\uc559 \uc9c0\uad6c\ub300\ub294 \uace0\uc0dd\ub300\uc758 \uc11d\ud0c4\uacc4\uc640 \uc911\uc0dd\ub300 \uc0bc\ucca9\uacc4\ub85c \uc774\ub8e8\uc5b4\uc838 \uc788\uc73c\uba70 \uc5ec\uae30\uc5d0 \uc11d\ud0c4\uce35\uc774 \uc788\uc5b4 \uc601\uad6d\uc758 \ud0c4\uc804(\u70ad\u7530)\uc744 \ud615\uc131\ud55c\ub2e4. \uc6e8\uc77c\uc2a4 \ub0a8\ubd80\uc640 \ub370\ubc88 \ucf54\ub978\uc6d4 \uc9c0\uad34\ub294 \uace0\uc0dd\ub300 \ub370\ubcf8\uacc4\ub85c \ub418\uc5b4 \uc788\ub2e4.\ud55c\ud3b8 \uc800\uc9c0\ub294 \uc789\uae00\ub79c\ub4dc\uc758 \ub300\ubd80\ubd84\uc744 \ucc28\uc9c0\ud558\uba70 \ud574\ubc1c 300m \uc774\ud558\uc758 \ub0ae\uc740 \uad6c\ub989\uacfc \ud3c9\uc57c\ub85c \ub418\uc5b4 \uc788\ub2e4. \uad6c\ub989\uc740 \ubc31\uc545\uacc4\uc640 \uc81c3\uacc4\uc758 \ub290\ub9ac\uac8c \uc8fc\ub984\uc7a1\ud78c \uc0c8 \uc9c0\uce35\uc73c\ub85c \uc774\ub904\uc838 \uac01\uc9c0\uc5d0 \ucf00\uc2a4\ud0c0 \uc9c0\ud615\uc774 \ubcf4\uc778\ub2e4. \uc601\uad6d\uc740 \uc81c4\uae30 \ud64d\uc801\uc138\uc5d0 \uc720\ub7fd \ub300\ub959\uc5d0\uc11c \ubed7\uce5c \ub300\ub959 \ube59\ud558\uc5d0 \ub36e\uc5ec \uc788\uc5c8\uc73c\ubbc0\ub85c \uc0b0\uc9c0\uc5d0\ub294 \ube59\uc2dd(\u6c37\u8755) \uc9c0\ud615\uc774 \ub9ce\uc73c\uba70, \ud669\ud1a0\uc5d0 \ub4a4\ub36e\uc778 \ud669\ubb34\uc9c0\uc640 \uc2b5\uc9c0\uac00 \ub9ce\ub2e4. \ub354\uad6c\ub098 \ubd81\ucabd\uc758 \uc2a4\ucf54\ud2c0\ub79c\ub4dc \uc81c\ub3c4\uc5d0\ub294 \ud53c\uc624\ub974\ub4dc \ud574\uc548\uc774 \ubc1c\ub2ec\ud574 \uc788\ub2e4. \ud15c\uc2a4 \uac15(\uc804\uccb4\uae38\uc774 350km)\uc744 \ube44\ub86f\ud558\uc5ec \uc774 \ub098\ub77c\uc758 \ud558\ucc9c\uc740 \ubaa8\ub450 \uc9e7\uc73c\uba70 \ub610 \uc0b0\uc9c0\ub3c4 \ub0ae\uc73c\ubbc0\ub85c \ud558\ucc9c\uc758 \ucda9\uc801(\u6c96\u7a4d)\uc791\uc6a9\uc740 \ud06c\uc9c0 \uc54a\ub2e4. \uadf8\ub798\uc11c \ud15c\uc2a4 \uac15 \ud558\uad6c\ubd80, \ube0c\ub9ac\uc2a4\ud2c0 \ud574\ud611, \ud3ec\uc2a4\ub9cc \ub4f1\ucc98\ub7fc \ubb3b\ud600 \ubc84\ub9b0 \uc0bc\uac01\uac15\uc774 \ub9ce\uc73c\uba70, \ud574\uc548\uc120\uc740 \uad74\uace1\uc774 \ub9ce\uace0 \ud558\ucc9c\uc758 \uacbd\uc0ac\uac00 \uc644\ub9cc\ud558\ubbc0\ub85c \ud5a5\ud56d\uacfc \uac00\ud56d\ud558\ucc9c(\u53ef\u822a\u6cb3\u5ddd)\uc758 \uc870\uac74\uc744 \uac16\ucd94\uace0 \uc788\ub2e4.", "answer": "\ubca4\ub124\ube44\uc2a4 \uc0b0", "sentence": "\uace0\uc9c0\ub77c\uace0 \ud574\ub3c4 \ub300\ubd80\ubd84\uc740 \uace0\ub3c4 1,000m \uc774\ud558\uc758 \ub0ae\uc740 \uc0b0\ub9e5 \ubc0f \uad6c\ub989\uc73c\ub85c\uc11c \uc601\uad6d\uc758 \ucd5c\uace0\ubd09 \ubca4\ub124\ube44\uc2a4 \uc0b0 \ub3c4 1,343m\uc5d0 \uc9c0\ub098\uc9c0 \uc54a\ub294\ub2e4.", "paragraph_sentence": "\uc9c0\ud615\uc801\uc73c\ub85c\ub294 \ub3d9\ub0a8\ubd80\uc758 \uc800\uc9c0\uc640 \ubd81\uc11c\ubd80\uc758 \uace0\uc9c0\ub85c \ud06c\uac8c \ub098\ub25c\ub2e4. \uc774 \ub458\uc758 \uacbd\uacc4\ub294 \ub3d9\ubd80 \ud574\uc548 \uc911\uc559\ubd80\uc758 \ud2f0\uc2a4 \uac15 \ud558\uad6c\uc640 \ub0a8\ubd80 \ud574\uc548\uc758 \ub77c\uc784\ub9cc\uc744 \uc5f0\uacb0\ud558\ub294 \uc120\uc774\ub2e4. \uace0\uc9c0\ub77c\uace0 \ud574\ub3c4 \ub300\ubd80\ubd84\uc740 \uace0\ub3c4 1,000m \uc774\ud558\uc758 \ub0ae\uc740 \uc0b0\ub9e5 \ubc0f \uad6c\ub989\uc73c\ub85c\uc11c \uc601\uad6d\uc758 \ucd5c\uace0\ubd09 \ubca4\ub124\ube44\uc2a4 \uc0b0 \ub3c4 1,343m\uc5d0 \uc9c0\ub098\uc9c0 \uc54a\ub294\ub2e4. \uace0\uc9c0\ub294 \uc9c0\ud615\uacfc \uc9c0\uc9c8\uc774 \ub2e4\ub978 \ub2e4\uc74c\uacfc \uac19\uc740 \uc9c0\uad34(\u5730\u584a)\ub85c \ub098\ub25c\ub2e4. \uc2a4\ucf54\ud2c0\ub79c\ub4dc \uace0\uc9c0\uc640 \ubd81\uc544\uc77c\ub79c\ub4dc\ub294 \uc8fc\ub85c \uce84\ube0c\ub9ac\uc544\uacc4\uc758 \uac00\uc7a5 \uc624\ub79c \ubcc0\uc131\uc554\uc73c\ub85c \ub418\uc5b4 \uc788\uc73c\uba70, \uc2a4\ucf54\ud2c0\ub79c\ub4dc \ub0a8\ubd80 \uc0b0\uc9c0, \uce84\ube0c\ub9ac\uc544 \uc0b0\uc9c0(\ud638\uc218 \uc9c0\ubc29) \ubc0f \uc6e8\uc77c\uc2a4\ub294 \uc8fc\ub85c \uace0\uc0dd\ub300 \uc2e4\ub8e8\ub9ac\uc544\uacc4\uc758 \ubcc0\uc131\uc554\uc73c\ub85c \ub418\uc5b4 \uc788\ub2e4. \ud398\ub098\uc778 \uc0b0\ub9e5\uacfc \uc2a4\ucf54\ud2c0\ub79c\ub4dc \uc911\uc559 \uc9c0\uad6c\ub300\ub294 \uace0\uc0dd\ub300\uc758 \uc11d\ud0c4\uacc4\uc640 \uc911\uc0dd\ub300 \uc0bc\ucca9\uacc4\ub85c \uc774\ub8e8\uc5b4\uc838 \uc788\uc73c\uba70 \uc5ec\uae30\uc5d0 \uc11d\ud0c4\uce35\uc774 \uc788\uc5b4 \uc601\uad6d\uc758 \ud0c4\uc804(\u70ad\u7530)\uc744 \ud615\uc131\ud55c\ub2e4. \uc6e8\uc77c\uc2a4 \ub0a8\ubd80\uc640 \ub370\ubc88 \ucf54\ub978\uc6d4 \uc9c0\uad34\ub294 \uace0\uc0dd\ub300 \ub370\ubcf8\uacc4\ub85c \ub418\uc5b4 \uc788\ub2e4.\ud55c\ud3b8 \uc800\uc9c0\ub294 \uc789\uae00\ub79c\ub4dc\uc758 \ub300\ubd80\ubd84\uc744 \ucc28\uc9c0\ud558\uba70 \ud574\ubc1c 300m \uc774\ud558\uc758 \ub0ae\uc740 \uad6c\ub989\uacfc \ud3c9\uc57c\ub85c \ub418\uc5b4 \uc788\ub2e4. \uad6c\ub989\uc740 \ubc31\uc545\uacc4\uc640 \uc81c3\uacc4\uc758 \ub290\ub9ac\uac8c \uc8fc\ub984\uc7a1\ud78c \uc0c8 \uc9c0\uce35\uc73c\ub85c \uc774\ub904\uc838 \uac01\uc9c0\uc5d0 \ucf00\uc2a4\ud0c0 \uc9c0\ud615\uc774 \ubcf4\uc778\ub2e4. \uc601\uad6d\uc740 \uc81c4\uae30 \ud64d\uc801\uc138\uc5d0 \uc720\ub7fd \ub300\ub959\uc5d0\uc11c \ubed7\uce5c \ub300\ub959 \ube59\ud558\uc5d0 \ub36e\uc5ec \uc788\uc5c8\uc73c\ubbc0\ub85c \uc0b0\uc9c0\uc5d0\ub294 \ube59\uc2dd(\u6c37\u8755) \uc9c0\ud615\uc774 \ub9ce\uc73c\uba70, \ud669\ud1a0\uc5d0 \ub4a4\ub36e\uc778 \ud669\ubb34\uc9c0\uc640 \uc2b5\uc9c0\uac00 \ub9ce\ub2e4. \ub354\uad6c\ub098 \ubd81\ucabd\uc758 \uc2a4\ucf54\ud2c0\ub79c\ub4dc \uc81c\ub3c4\uc5d0\ub294 \ud53c\uc624\ub974\ub4dc \ud574\uc548\uc774 \ubc1c\ub2ec\ud574 \uc788\ub2e4. \ud15c\uc2a4 \uac15(\uc804\uccb4\uae38\uc774 350km)\uc744 \ube44\ub86f\ud558\uc5ec \uc774 \ub098\ub77c\uc758 \ud558\ucc9c\uc740 \ubaa8\ub450 \uc9e7\uc73c\uba70 \ub610 \uc0b0\uc9c0\ub3c4 \ub0ae\uc73c\ubbc0\ub85c \ud558\ucc9c\uc758 \ucda9\uc801(\u6c96\u7a4d)\uc791\uc6a9\uc740 \ud06c\uc9c0 \uc54a\ub2e4. \uadf8\ub798\uc11c \ud15c\uc2a4 \uac15 \ud558\uad6c\ubd80, \ube0c\ub9ac\uc2a4\ud2c0 \ud574\ud611, \ud3ec\uc2a4\ub9cc \ub4f1\ucc98\ub7fc \ubb3b\ud600 \ubc84\ub9b0 \uc0bc\uac01\uac15\uc774 \ub9ce\uc73c\uba70, \ud574\uc548\uc120\uc740 \uad74\uace1\uc774 \ub9ce\uace0 \ud558\ucc9c\uc758 \uacbd\uc0ac\uac00 \uc644\ub9cc\ud558\ubbc0\ub85c \ud5a5\ud56d\uacfc \uac00\ud56d\ud558\ucc9c(\u53ef\u822a\u6cb3\u5ddd)\uc758 \uc870\uac74\uc744 \uac16\ucd94\uace0 \uc788\ub2e4.", "paragraph_answer": "\uc9c0\ud615\uc801\uc73c\ub85c\ub294 \ub3d9\ub0a8\ubd80\uc758 \uc800\uc9c0\uc640 \ubd81\uc11c\ubd80\uc758 \uace0\uc9c0\ub85c \ud06c\uac8c \ub098\ub25c\ub2e4. \uc774 \ub458\uc758 \uacbd\uacc4\ub294 \ub3d9\ubd80 \ud574\uc548 \uc911\uc559\ubd80\uc758 \ud2f0\uc2a4 \uac15 \ud558\uad6c\uc640 \ub0a8\ubd80 \ud574\uc548\uc758 \ub77c\uc784\ub9cc\uc744 \uc5f0\uacb0\ud558\ub294 \uc120\uc774\ub2e4. \uace0\uc9c0\ub77c\uace0 \ud574\ub3c4 \ub300\ubd80\ubd84\uc740 \uace0\ub3c4 1,000m \uc774\ud558\uc758 \ub0ae\uc740 \uc0b0\ub9e5 \ubc0f \uad6c\ub989\uc73c\ub85c\uc11c \uc601\uad6d\uc758 \ucd5c\uace0\ubd09 \ubca4\ub124\ube44\uc2a4 \uc0b0 \ub3c4 1,343m\uc5d0 \uc9c0\ub098\uc9c0 \uc54a\ub294\ub2e4. \uace0\uc9c0\ub294 \uc9c0\ud615\uacfc \uc9c0\uc9c8\uc774 \ub2e4\ub978 \ub2e4\uc74c\uacfc \uac19\uc740 \uc9c0\uad34(\u5730\u584a)\ub85c \ub098\ub25c\ub2e4. \uc2a4\ucf54\ud2c0\ub79c\ub4dc \uace0\uc9c0\uc640 \ubd81\uc544\uc77c\ub79c\ub4dc\ub294 \uc8fc\ub85c \uce84\ube0c\ub9ac\uc544\uacc4\uc758 \uac00\uc7a5 \uc624\ub79c \ubcc0\uc131\uc554\uc73c\ub85c \ub418\uc5b4 \uc788\uc73c\uba70, \uc2a4\ucf54\ud2c0\ub79c\ub4dc \ub0a8\ubd80 \uc0b0\uc9c0, \uce84\ube0c\ub9ac\uc544 \uc0b0\uc9c0(\ud638\uc218 \uc9c0\ubc29) \ubc0f \uc6e8\uc77c\uc2a4\ub294 \uc8fc\ub85c \uace0\uc0dd\ub300 \uc2e4\ub8e8\ub9ac\uc544\uacc4\uc758 \ubcc0\uc131\uc554\uc73c\ub85c \ub418\uc5b4 \uc788\ub2e4. \ud398\ub098\uc778 \uc0b0\ub9e5\uacfc \uc2a4\ucf54\ud2c0\ub79c\ub4dc \uc911\uc559 \uc9c0\uad6c\ub300\ub294 \uace0\uc0dd\ub300\uc758 \uc11d\ud0c4\uacc4\uc640 \uc911\uc0dd\ub300 \uc0bc\ucca9\uacc4\ub85c \uc774\ub8e8\uc5b4\uc838 \uc788\uc73c\uba70 \uc5ec\uae30\uc5d0 \uc11d\ud0c4\uce35\uc774 \uc788\uc5b4 \uc601\uad6d\uc758 \ud0c4\uc804(\u70ad\u7530)\uc744 \ud615\uc131\ud55c\ub2e4. \uc6e8\uc77c\uc2a4 \ub0a8\ubd80\uc640 \ub370\ubc88 \ucf54\ub978\uc6d4 \uc9c0\uad34\ub294 \uace0\uc0dd\ub300 \ub370\ubcf8\uacc4\ub85c \ub418\uc5b4 \uc788\ub2e4.\ud55c\ud3b8 \uc800\uc9c0\ub294 \uc789\uae00\ub79c\ub4dc\uc758 \ub300\ubd80\ubd84\uc744 \ucc28\uc9c0\ud558\uba70 \ud574\ubc1c 300m \uc774\ud558\uc758 \ub0ae\uc740 \uad6c\ub989\uacfc \ud3c9\uc57c\ub85c \ub418\uc5b4 \uc788\ub2e4. \uad6c\ub989\uc740 \ubc31\uc545\uacc4\uc640 \uc81c3\uacc4\uc758 \ub290\ub9ac\uac8c \uc8fc\ub984\uc7a1\ud78c \uc0c8 \uc9c0\uce35\uc73c\ub85c \uc774\ub904\uc838 \uac01\uc9c0\uc5d0 \ucf00\uc2a4\ud0c0 \uc9c0\ud615\uc774 \ubcf4\uc778\ub2e4. \uc601\uad6d\uc740 \uc81c4\uae30 \ud64d\uc801\uc138\uc5d0 \uc720\ub7fd \ub300\ub959\uc5d0\uc11c \ubed7\uce5c \ub300\ub959 \ube59\ud558\uc5d0 \ub36e\uc5ec \uc788\uc5c8\uc73c\ubbc0\ub85c \uc0b0\uc9c0\uc5d0\ub294 \ube59\uc2dd(\u6c37\u8755) \uc9c0\ud615\uc774 \ub9ce\uc73c\uba70, \ud669\ud1a0\uc5d0 \ub4a4\ub36e\uc778 \ud669\ubb34\uc9c0\uc640 \uc2b5\uc9c0\uac00 \ub9ce\ub2e4. \ub354\uad6c\ub098 \ubd81\ucabd\uc758 \uc2a4\ucf54\ud2c0\ub79c\ub4dc \uc81c\ub3c4\uc5d0\ub294 \ud53c\uc624\ub974\ub4dc \ud574\uc548\uc774 \ubc1c\ub2ec\ud574 \uc788\ub2e4. \ud15c\uc2a4 \uac15(\uc804\uccb4\uae38\uc774 350km)\uc744 \ube44\ub86f\ud558\uc5ec \uc774 \ub098\ub77c\uc758 \ud558\ucc9c\uc740 \ubaa8\ub450 \uc9e7\uc73c\uba70 \ub610 \uc0b0\uc9c0\ub3c4 \ub0ae\uc73c\ubbc0\ub85c \ud558\ucc9c\uc758 \ucda9\uc801(\u6c96\u7a4d)\uc791\uc6a9\uc740 \ud06c\uc9c0 \uc54a\ub2e4. \uadf8\ub798\uc11c \ud15c\uc2a4 \uac15 \ud558\uad6c\ubd80, \ube0c\ub9ac\uc2a4\ud2c0 \ud574\ud611, \ud3ec\uc2a4\ub9cc \ub4f1\ucc98\ub7fc \ubb3b\ud600 \ubc84\ub9b0 \uc0bc\uac01\uac15\uc774 \ub9ce\uc73c\uba70, \ud574\uc548\uc120\uc740 \uad74\uace1\uc774 \ub9ce\uace0 \ud558\ucc9c\uc758 \uacbd\uc0ac\uac00 \uc644\ub9cc\ud558\ubbc0\ub85c \ud5a5\ud56d\uacfc \uac00\ud56d\ud558\ucc9c(\u53ef\u822a\u6cb3\u5ddd)\uc758 \uc870\uac74\uc744 \uac16\ucd94\uace0 \uc788\ub2e4.", "sentence_answer": "\uace0\uc9c0\ub77c\uace0 \ud574\ub3c4 \ub300\ubd80\ubd84\uc740 \uace0\ub3c4 1,000m \uc774\ud558\uc758 \ub0ae\uc740 \uc0b0\ub9e5 \ubc0f \uad6c\ub989\uc73c\ub85c\uc11c \uc601\uad6d\uc758 \ucd5c\uace0\ubd09 \ubca4\ub124\ube44\uc2a4 \uc0b0 \ub3c4 1,343m\uc5d0 \uc9c0\ub098\uc9c0 \uc54a\ub294\ub2e4.", "paragraph_id": "6458616-7-0", "paragraph_question": "question: \uc601\uad6d\uc758 \ucd5c\uace0\ubd09\uc740?, context: \uc9c0\ud615\uc801\uc73c\ub85c\ub294 \ub3d9\ub0a8\ubd80\uc758 \uc800\uc9c0\uc640 \ubd81\uc11c\ubd80\uc758 \uace0\uc9c0\ub85c \ud06c\uac8c \ub098\ub25c\ub2e4. \uc774 \ub458\uc758 \uacbd\uacc4\ub294 \ub3d9\ubd80 \ud574\uc548 \uc911\uc559\ubd80\uc758 \ud2f0\uc2a4 \uac15 \ud558\uad6c\uc640 \ub0a8\ubd80 \ud574\uc548\uc758 \ub77c\uc784\ub9cc\uc744 \uc5f0\uacb0\ud558\ub294 \uc120\uc774\ub2e4. \uace0\uc9c0\ub77c\uace0 \ud574\ub3c4 \ub300\ubd80\ubd84\uc740 \uace0\ub3c4 1,000m \uc774\ud558\uc758 \ub0ae\uc740 \uc0b0\ub9e5 \ubc0f \uad6c\ub989\uc73c\ub85c\uc11c \uc601\uad6d\uc758 \ucd5c\uace0\ubd09 \ubca4\ub124\ube44\uc2a4 \uc0b0\ub3c4 1,343m\uc5d0 \uc9c0\ub098\uc9c0 \uc54a\ub294\ub2e4. \uace0\uc9c0\ub294 \uc9c0\ud615\uacfc \uc9c0\uc9c8\uc774 \ub2e4\ub978 \ub2e4\uc74c\uacfc \uac19\uc740 \uc9c0\uad34(\u5730\u584a)\ub85c \ub098\ub25c\ub2e4. \uc2a4\ucf54\ud2c0\ub79c\ub4dc \uace0\uc9c0\uc640 \ubd81\uc544\uc77c\ub79c\ub4dc\ub294 \uc8fc\ub85c \uce84\ube0c\ub9ac\uc544\uacc4\uc758 \uac00\uc7a5 \uc624\ub79c \ubcc0\uc131\uc554\uc73c\ub85c \ub418\uc5b4 \uc788\uc73c\uba70, \uc2a4\ucf54\ud2c0\ub79c\ub4dc \ub0a8\ubd80 \uc0b0\uc9c0, \uce84\ube0c\ub9ac\uc544 \uc0b0\uc9c0(\ud638\uc218 \uc9c0\ubc29) \ubc0f \uc6e8\uc77c\uc2a4\ub294 \uc8fc\ub85c \uace0\uc0dd\ub300 \uc2e4\ub8e8\ub9ac\uc544\uacc4\uc758 \ubcc0\uc131\uc554\uc73c\ub85c \ub418\uc5b4 \uc788\ub2e4. \ud398\ub098\uc778 \uc0b0\ub9e5\uacfc \uc2a4\ucf54\ud2c0\ub79c\ub4dc \uc911\uc559 \uc9c0\uad6c\ub300\ub294 \uace0\uc0dd\ub300\uc758 \uc11d\ud0c4\uacc4\uc640 \uc911\uc0dd\ub300 \uc0bc\ucca9\uacc4\ub85c \uc774\ub8e8\uc5b4\uc838 \uc788\uc73c\uba70 \uc5ec\uae30\uc5d0 \uc11d\ud0c4\uce35\uc774 \uc788\uc5b4 \uc601\uad6d\uc758 \ud0c4\uc804(\u70ad\u7530)\uc744 \ud615\uc131\ud55c\ub2e4. \uc6e8\uc77c\uc2a4 \ub0a8\ubd80\uc640 \ub370\ubc88 \ucf54\ub978\uc6d4 \uc9c0\uad34\ub294 \uace0\uc0dd\ub300 \ub370\ubcf8\uacc4\ub85c \ub418\uc5b4 \uc788\ub2e4.\ud55c\ud3b8 \uc800\uc9c0\ub294 \uc789\uae00\ub79c\ub4dc\uc758 \ub300\ubd80\ubd84\uc744 \ucc28\uc9c0\ud558\uba70 \ud574\ubc1c 300m \uc774\ud558\uc758 \ub0ae\uc740 \uad6c\ub989\uacfc \ud3c9\uc57c\ub85c \ub418\uc5b4 \uc788\ub2e4. \uad6c\ub989\uc740 \ubc31\uc545\uacc4\uc640 \uc81c3\uacc4\uc758 \ub290\ub9ac\uac8c \uc8fc\ub984\uc7a1\ud78c \uc0c8 \uc9c0\uce35\uc73c\ub85c \uc774\ub904\uc838 \uac01\uc9c0\uc5d0 \ucf00\uc2a4\ud0c0 \uc9c0\ud615\uc774 \ubcf4\uc778\ub2e4. \uc601\uad6d\uc740 \uc81c4\uae30 \ud64d\uc801\uc138\uc5d0 \uc720\ub7fd \ub300\ub959\uc5d0\uc11c \ubed7\uce5c \ub300\ub959 \ube59\ud558\uc5d0 \ub36e\uc5ec \uc788\uc5c8\uc73c\ubbc0\ub85c \uc0b0\uc9c0\uc5d0\ub294 \ube59\uc2dd(\u6c37\u8755) \uc9c0\ud615\uc774 \ub9ce\uc73c\uba70, \ud669\ud1a0\uc5d0 \ub4a4\ub36e\uc778 \ud669\ubb34\uc9c0\uc640 \uc2b5\uc9c0\uac00 \ub9ce\ub2e4. \ub354\uad6c\ub098 \ubd81\ucabd\uc758 \uc2a4\ucf54\ud2c0\ub79c\ub4dc \uc81c\ub3c4\uc5d0\ub294 \ud53c\uc624\ub974\ub4dc \ud574\uc548\uc774 \ubc1c\ub2ec\ud574 \uc788\ub2e4. \ud15c\uc2a4 \uac15(\uc804\uccb4\uae38\uc774 350km)\uc744 \ube44\ub86f\ud558\uc5ec \uc774 \ub098\ub77c\uc758 \ud558\ucc9c\uc740 \ubaa8\ub450 \uc9e7\uc73c\uba70 \ub610 \uc0b0\uc9c0\ub3c4 \ub0ae\uc73c\ubbc0\ub85c \ud558\ucc9c\uc758 \ucda9\uc801(\u6c96\u7a4d)\uc791\uc6a9\uc740 \ud06c\uc9c0 \uc54a\ub2e4. \uadf8\ub798\uc11c \ud15c\uc2a4 \uac15 \ud558\uad6c\ubd80, \ube0c\ub9ac\uc2a4\ud2c0 \ud574\ud611, \ud3ec\uc2a4\ub9cc \ub4f1\ucc98\ub7fc \ubb3b\ud600 \ubc84\ub9b0 \uc0bc\uac01\uac15\uc774 \ub9ce\uc73c\uba70, \ud574\uc548\uc120\uc740 \uad74\uace1\uc774 \ub9ce\uace0 \ud558\ucc9c\uc758 \uacbd\uc0ac\uac00 \uc644\ub9cc\ud558\ubbc0\ub85c \ud5a5\ud56d\uacfc \uac00\ud56d\ud558\ucc9c(\u53ef\u822a\u6cb3\u5ddd)\uc758 \uc870\uac74\uc744 \uac16\ucd94\uace0 \uc788\ub2e4."} {"question": "\uae40\uc5f0\uc544\uac00 2009\ub144 3\uc6d4 \uc138\uacc4 \uc120\uc218\uad8c \ub300\ud68c \uc1fc\ud2b8\ud504\ub85c\uadf8\ub7a8\uc5d0\uc11c \uc5ec\uc790 \ucd5c\ucd08\ub85c \uc138\uc6b4 \uae30\ub85d\uc740?", "paragraph": "2009\ub144 3\uc6d4, \ubbf8\uad6d \ub85c\uc2a4\uc564\uc824\ub808\uc2a4\uc5d0\uc11c \uac1c\ucd5c\ub41c 2009\ub144 \uc138\uacc4 \uc120\uc218\uad8c \ub300\ud68c \uc1fc\ud2b8 \ud504\ub85c\uadf8\ub7a8\uc5d0\uc11c\ub294 \ubaa8\ub4e0 \uc810\ud504\uc758 \uc131\uacf5, \uc2a4\ud540\uacfc \uc2a4\ud30c\uc774\ub7f4\uc5d0\uc11c \ub808\ubca8 4, \uc2a4\ud15d \ub808\ubca8 3\uacfc \uac00\uc0b0\uc810 1\uc810\uc744 \ubc1b\ub294 \ub4f1 76.12\uc810\uc774\ub77c\ub294 \uc0ac\uc0c1 \uc720\ub840\uc5c6\ub294 \ub192\uc740 \uc810\uc218\ub97c \ubc1b\uc558\uace0, \uc774\ub85c\uc368 4\ub300\ub959 \uc120\uc218\uad8c \ub300\ud68c\uc5d0\uc11c \uc790\uc2e0\uc774 \uc138\uc6b4 \uc138\uacc4\uae30\ub85d\uc744 \ub2e4\uc2dc \uacbd\uc2e0\ud558\uac8c \ub41c\ub2e4. \uc774\ub0a0 \uc5f4\ub9b0 \uc1fc\ud2b8\ud504\ub85c\uadf8\ub7a8\uc5d0\uc11c \uc5ec\uc790\uc120\uc218 \ucd5c\ucd08\ub85c \uad6c\uc131\uc810\uc218 8\uc810\ub300\ub97c \ubc1b\ub294 \ucf8c\uac70\ub3c4 \uc774\ub8e8\uc5c8\ub2e4. \ub2e4\uc74c\ub0a0 \uc5f4\ub9b0 \ud504\ub9ac\uc2a4\ucf00\uc774\ud305\uc5d0\uc11c\ub294 \uc0b4\ucf54\uc640 \uc2a4\ud540 \uc2e4\uc218\uc5d0\ub3c4 \ubd88\uad6c\ud558\uace0 131.59\uc810\uc744 \ubc1b\uc544 \ucd1d\uc810 207.71\uc744 \uae30\ub85d\ud558\uba70 \uc885\ud569 1\uc704(\uae08\uba54\ub2ec)\uc744 \ucc28\uc9c0\ud55c\ub2e4. \ub610\ud55c \ud504\ub9ac\uc2a4\ucf00\uc774\ud305\uc5d0\uc11c \uad6c\uc131\uc810\uc218 68.40\uc810\uc744 \uae30\ub85d\ud558\uba70 \uc5ec\uc790 \uc120\uc218 \ucd5c\ucd08\ub85c \ud504\ub9ac\uc2a4\ucf00\uc774\ud305\uc5d0\uc11c 8\uc810\ub300\ub97c \ub9c8\ud06c\ud55c \uc120\uc218\uac00 \ub418\uc5c8\ub2e4. \uae40\uc5f0\uc544\uc758 \ud504\ub9ac\uc2a4\ucf00\uc774\ud305\uc758 \uc810\uc218(131.59)\ub294 \uc790\uc2e0\uc774 \uc138\uc6b4 \uc138\uacc4 \ucd5c\uace0 \uae30\ub85d\uc778 2007-2008 \ucef5 \uc624\ube0c \ub7ec\uc2dc\uc544(133.70)\uc640 2007-2008 \uadf8\ub791\ud504\ub9ac \ud30c\uc774\ub110(132.21)\uc5d0 \uc774\uc5b4 3\ubc88\uc9f8\ub85c \ub192\uc740 \uc810\uc218\uc774\ub2e4. 4\ub300\ub959 \uc120\uc218\uad8c \ub300\ud68c\uc640 \ub9c8\ucc2c\uac00\uc9c0\ub85c \uc1fc\ud2b8 \ud504\ub85c\uadf8\ub7a8\uacfc \ud504\ub9ac\uc2a4\ucf00\uc774\ud305\uc5d0\uc11c 3\ud68c\uc804+3\ud68c\uc804 \uc5f0\uc18d \uc810\ud504\ub97c \uc2dc\ub3c4\ud574 \uc1fc\ud2b8, \ud504\ub9ac \ubaa8\ub450 \uc131\uacf5\ud55c \uac83\uc740 \uae40\uc5f0\uc544\ub9cc\uc774 \uc720\uc77c\ud558\ub2e4. \ub610\ud55c \ud53c\uaca8 \uc2a4\ucf00\uc774\ud305 \uc5ec\uc790 \uc2f1\uae00 \uc120\uc218 \uc0ac\uc0c1 \ucd5c\ucd08\ub85c 200\uc810\uc744 \ub118\uc740 \uac83\uc774\uc5c8\ub2e4. \ub2e4\ub978 \uacbd\uae30\uc5d0\uc11c \ub17c\ub780\uc774 \ub418\uc5b4\uc628 \ud50c\ub9bd \uc810\ud504\uc5d0 \ub300\ud574\uc11c\ub294 1992\ub144 \uc62c\ub9bc\ud53d \uc5ec\uc790 \uc2f1\uae00 \uae08\uba54\ub2ec\ub9ac\uc2a4\ud2b8 \ud06c\ub9ac\uc2a4\ud2f0 \uc57c\ub9c8\uad6c\uce58\ub294 \ud50c\ub9bd\uc5d0 \uc5b4\ud150\uc158\uc774 \ubd99\uc5c8\ub2e4\uba74 \uc2ec\ud310\uc740 \ubc18\uc131\ud574\uc57c\ud588\uc744\uac83\uc774\ub77c\uace0 \ub9d0\ud558\uace0, \uc77c\ubcf8\uc758 \uc62c\ub9bc\ud53d \uc740\uba54\ub2ec\ub9ac\uc2a4\ud2b8 \uc774\ud1a0 \ubbf8\ub3c4\ub9ac \ub610\ud55c \uc774\uac83\uc740 \uba85\ubc31\ud55c \ud50c\ub9bd\uc774\ub77c\uace0 \ub9d0\ud588\ub2e4. \uadf8\ub7ec\ub098 \uc774 \uacbd\uae30\uc5d0\uc11c\ub3c4 \uc2ec\ud310\ub4e4\uc740 \uae40\uc5f0\uc544\uc758 \ud50c\ub9bd\uc5d0 \uc5b4\ud150\uc158\uc744 \uc918 \ud3b8\ud30c \ud310\uc815\uc758 \ub17c\ub780\uc774 \uc77c\uc5c8\ub2e4.", "answer": "\uad6c\uc131\uc810\uc218 8\uc810\ub300", "sentence": "\uc774\ub0a0 \uc5f4\ub9b0 \uc1fc\ud2b8\ud504\ub85c\uadf8\ub7a8\uc5d0\uc11c \uc5ec\uc790\uc120\uc218 \ucd5c\ucd08\ub85c \uad6c\uc131\uc810\uc218 8\uc810\ub300 \ub97c \ubc1b\ub294 \ucf8c\uac70\ub3c4 \uc774\ub8e8\uc5c8\ub2e4.", "paragraph_sentence": "2009\ub144 3\uc6d4, \ubbf8\uad6d \ub85c\uc2a4\uc564\uc824\ub808\uc2a4\uc5d0\uc11c \uac1c\ucd5c\ub41c 2009\ub144 \uc138\uacc4 \uc120\uc218\uad8c \ub300\ud68c \uc1fc\ud2b8 \ud504\ub85c\uadf8\ub7a8\uc5d0\uc11c\ub294 \ubaa8\ub4e0 \uc810\ud504\uc758 \uc131\uacf5, \uc2a4\ud540\uacfc \uc2a4\ud30c\uc774\ub7f4\uc5d0\uc11c \ub808\ubca8 4, \uc2a4\ud15d \ub808\ubca8 3\uacfc \uac00\uc0b0\uc810 1\uc810\uc744 \ubc1b\ub294 \ub4f1 76.12\uc810\uc774\ub77c\ub294 \uc0ac\uc0c1 \uc720\ub840\uc5c6\ub294 \ub192\uc740 \uc810\uc218\ub97c \ubc1b\uc558\uace0, \uc774\ub85c\uc368 4\ub300\ub959 \uc120\uc218\uad8c \ub300\ud68c\uc5d0\uc11c \uc790\uc2e0\uc774 \uc138\uc6b4 \uc138\uacc4\uae30\ub85d\uc744 \ub2e4\uc2dc \uacbd\uc2e0\ud558\uac8c \ub41c\ub2e4. \uc774\ub0a0 \uc5f4\ub9b0 \uc1fc\ud2b8\ud504\ub85c\uadf8\ub7a8\uc5d0\uc11c \uc5ec\uc790\uc120\uc218 \ucd5c\ucd08\ub85c \uad6c\uc131\uc810\uc218 8\uc810\ub300 \ub97c \ubc1b\ub294 \ucf8c\uac70\ub3c4 \uc774\ub8e8\uc5c8\ub2e4. \ub2e4\uc74c\ub0a0 \uc5f4\ub9b0 \ud504\ub9ac\uc2a4\ucf00\uc774\ud305\uc5d0\uc11c\ub294 \uc0b4\ucf54\uc640 \uc2a4\ud540 \uc2e4\uc218\uc5d0\ub3c4 \ubd88\uad6c\ud558\uace0 131.59\uc810\uc744 \ubc1b\uc544 \ucd1d\uc810 207.71\uc744 \uae30\ub85d\ud558\uba70 \uc885\ud569 1\uc704(\uae08\uba54\ub2ec)\uc744 \ucc28\uc9c0\ud55c\ub2e4. \ub610\ud55c \ud504\ub9ac\uc2a4\ucf00\uc774\ud305\uc5d0\uc11c \uad6c\uc131\uc810\uc218 68.40\uc810\uc744 \uae30\ub85d\ud558\uba70 \uc5ec\uc790 \uc120\uc218 \ucd5c\ucd08\ub85c \ud504\ub9ac\uc2a4\ucf00\uc774\ud305\uc5d0\uc11c 8\uc810\ub300\ub97c \ub9c8\ud06c\ud55c \uc120\uc218\uac00 \ub418\uc5c8\ub2e4. \uae40\uc5f0\uc544\uc758 \ud504\ub9ac\uc2a4\ucf00\uc774\ud305\uc758 \uc810\uc218(131.59)\ub294 \uc790\uc2e0\uc774 \uc138\uc6b4 \uc138\uacc4 \ucd5c\uace0 \uae30\ub85d\uc778 2007-2008 \ucef5 \uc624\ube0c \ub7ec\uc2dc\uc544(133.70)\uc640 2007-2008 \uadf8\ub791\ud504\ub9ac \ud30c\uc774\ub110(132.21)\uc5d0 \uc774\uc5b4 3\ubc88\uc9f8\ub85c \ub192\uc740 \uc810\uc218\uc774\ub2e4. 4\ub300\ub959 \uc120\uc218\uad8c \ub300\ud68c\uc640 \ub9c8\ucc2c\uac00\uc9c0\ub85c \uc1fc\ud2b8 \ud504\ub85c\uadf8\ub7a8\uacfc \ud504\ub9ac\uc2a4\ucf00\uc774\ud305\uc5d0\uc11c 3\ud68c\uc804+3\ud68c\uc804 \uc5f0\uc18d \uc810\ud504\ub97c \uc2dc\ub3c4\ud574 \uc1fc\ud2b8, \ud504\ub9ac \ubaa8\ub450 \uc131\uacf5\ud55c \uac83\uc740 \uae40\uc5f0\uc544\ub9cc\uc774 \uc720\uc77c\ud558\ub2e4. \ub610\ud55c \ud53c\uaca8 \uc2a4\ucf00\uc774\ud305 \uc5ec\uc790 \uc2f1\uae00 \uc120\uc218 \uc0ac\uc0c1 \ucd5c\ucd08\ub85c 200\uc810\uc744 \ub118\uc740 \uac83\uc774\uc5c8\ub2e4. \ub2e4\ub978 \uacbd\uae30\uc5d0\uc11c \ub17c\ub780\uc774 \ub418\uc5b4\uc628 \ud50c\ub9bd \uc810\ud504\uc5d0 \ub300\ud574\uc11c\ub294 1992\ub144 \uc62c\ub9bc\ud53d \uc5ec\uc790 \uc2f1\uae00 \uae08\uba54\ub2ec\ub9ac\uc2a4\ud2b8 \ud06c\ub9ac\uc2a4\ud2f0 \uc57c\ub9c8\uad6c\uce58\ub294 \ud50c\ub9bd\uc5d0 \uc5b4\ud150\uc158\uc774 \ubd99\uc5c8\ub2e4\uba74 \uc2ec\ud310\uc740 \ubc18\uc131\ud574\uc57c\ud588\uc744\uac83\uc774\ub77c\uace0 \ub9d0\ud558\uace0, \uc77c\ubcf8\uc758 \uc62c\ub9bc\ud53d \uc740\uba54\ub2ec\ub9ac\uc2a4\ud2b8 \uc774\ud1a0 \ubbf8\ub3c4\ub9ac \ub610\ud55c \uc774\uac83\uc740 \uba85\ubc31\ud55c \ud50c\ub9bd\uc774\ub77c\uace0 \ub9d0\ud588\ub2e4. \uadf8\ub7ec\ub098 \uc774 \uacbd\uae30\uc5d0\uc11c\ub3c4 \uc2ec\ud310\ub4e4\uc740 \uae40\uc5f0\uc544\uc758 \ud50c\ub9bd\uc5d0 \uc5b4\ud150\uc158\uc744 \uc918 \ud3b8\ud30c \ud310\uc815\uc758 \ub17c\ub780\uc774 \uc77c\uc5c8\ub2e4.", "paragraph_answer": "2009\ub144 3\uc6d4, \ubbf8\uad6d \ub85c\uc2a4\uc564\uc824\ub808\uc2a4\uc5d0\uc11c \uac1c\ucd5c\ub41c 2009\ub144 \uc138\uacc4 \uc120\uc218\uad8c \ub300\ud68c \uc1fc\ud2b8 \ud504\ub85c\uadf8\ub7a8\uc5d0\uc11c\ub294 \ubaa8\ub4e0 \uc810\ud504\uc758 \uc131\uacf5, \uc2a4\ud540\uacfc \uc2a4\ud30c\uc774\ub7f4\uc5d0\uc11c \ub808\ubca8 4, \uc2a4\ud15d \ub808\ubca8 3\uacfc \uac00\uc0b0\uc810 1\uc810\uc744 \ubc1b\ub294 \ub4f1 76.12\uc810\uc774\ub77c\ub294 \uc0ac\uc0c1 \uc720\ub840\uc5c6\ub294 \ub192\uc740 \uc810\uc218\ub97c \ubc1b\uc558\uace0, \uc774\ub85c\uc368 4\ub300\ub959 \uc120\uc218\uad8c \ub300\ud68c\uc5d0\uc11c \uc790\uc2e0\uc774 \uc138\uc6b4 \uc138\uacc4\uae30\ub85d\uc744 \ub2e4\uc2dc \uacbd\uc2e0\ud558\uac8c \ub41c\ub2e4. \uc774\ub0a0 \uc5f4\ub9b0 \uc1fc\ud2b8\ud504\ub85c\uadf8\ub7a8\uc5d0\uc11c \uc5ec\uc790\uc120\uc218 \ucd5c\ucd08\ub85c \uad6c\uc131\uc810\uc218 8\uc810\ub300 \ub97c \ubc1b\ub294 \ucf8c\uac70\ub3c4 \uc774\ub8e8\uc5c8\ub2e4. \ub2e4\uc74c\ub0a0 \uc5f4\ub9b0 \ud504\ub9ac\uc2a4\ucf00\uc774\ud305\uc5d0\uc11c\ub294 \uc0b4\ucf54\uc640 \uc2a4\ud540 \uc2e4\uc218\uc5d0\ub3c4 \ubd88\uad6c\ud558\uace0 131.59\uc810\uc744 \ubc1b\uc544 \ucd1d\uc810 207.71\uc744 \uae30\ub85d\ud558\uba70 \uc885\ud569 1\uc704(\uae08\uba54\ub2ec)\uc744 \ucc28\uc9c0\ud55c\ub2e4. \ub610\ud55c \ud504\ub9ac\uc2a4\ucf00\uc774\ud305\uc5d0\uc11c \uad6c\uc131\uc810\uc218 68.40\uc810\uc744 \uae30\ub85d\ud558\uba70 \uc5ec\uc790 \uc120\uc218 \ucd5c\ucd08\ub85c \ud504\ub9ac\uc2a4\ucf00\uc774\ud305\uc5d0\uc11c 8\uc810\ub300\ub97c \ub9c8\ud06c\ud55c \uc120\uc218\uac00 \ub418\uc5c8\ub2e4. \uae40\uc5f0\uc544\uc758 \ud504\ub9ac\uc2a4\ucf00\uc774\ud305\uc758 \uc810\uc218(131.59)\ub294 \uc790\uc2e0\uc774 \uc138\uc6b4 \uc138\uacc4 \ucd5c\uace0 \uae30\ub85d\uc778 2007-2008 \ucef5 \uc624\ube0c \ub7ec\uc2dc\uc544(133.70)\uc640 2007-2008 \uadf8\ub791\ud504\ub9ac \ud30c\uc774\ub110(132.21)\uc5d0 \uc774\uc5b4 3\ubc88\uc9f8\ub85c \ub192\uc740 \uc810\uc218\uc774\ub2e4. 4\ub300\ub959 \uc120\uc218\uad8c \ub300\ud68c\uc640 \ub9c8\ucc2c\uac00\uc9c0\ub85c \uc1fc\ud2b8 \ud504\ub85c\uadf8\ub7a8\uacfc \ud504\ub9ac\uc2a4\ucf00\uc774\ud305\uc5d0\uc11c 3\ud68c\uc804+3\ud68c\uc804 \uc5f0\uc18d \uc810\ud504\ub97c \uc2dc\ub3c4\ud574 \uc1fc\ud2b8, \ud504\ub9ac \ubaa8\ub450 \uc131\uacf5\ud55c \uac83\uc740 \uae40\uc5f0\uc544\ub9cc\uc774 \uc720\uc77c\ud558\ub2e4. \ub610\ud55c \ud53c\uaca8 \uc2a4\ucf00\uc774\ud305 \uc5ec\uc790 \uc2f1\uae00 \uc120\uc218 \uc0ac\uc0c1 \ucd5c\ucd08\ub85c 200\uc810\uc744 \ub118\uc740 \uac83\uc774\uc5c8\ub2e4. \ub2e4\ub978 \uacbd\uae30\uc5d0\uc11c \ub17c\ub780\uc774 \ub418\uc5b4\uc628 \ud50c\ub9bd \uc810\ud504\uc5d0 \ub300\ud574\uc11c\ub294 1992\ub144 \uc62c\ub9bc\ud53d \uc5ec\uc790 \uc2f1\uae00 \uae08\uba54\ub2ec\ub9ac\uc2a4\ud2b8 \ud06c\ub9ac\uc2a4\ud2f0 \uc57c\ub9c8\uad6c\uce58\ub294 \ud50c\ub9bd\uc5d0 \uc5b4\ud150\uc158\uc774 \ubd99\uc5c8\ub2e4\uba74 \uc2ec\ud310\uc740 \ubc18\uc131\ud574\uc57c\ud588\uc744\uac83\uc774\ub77c\uace0 \ub9d0\ud558\uace0, \uc77c\ubcf8\uc758 \uc62c\ub9bc\ud53d \uc740\uba54\ub2ec\ub9ac\uc2a4\ud2b8 \uc774\ud1a0 \ubbf8\ub3c4\ub9ac \ub610\ud55c \uc774\uac83\uc740 \uba85\ubc31\ud55c \ud50c\ub9bd\uc774\ub77c\uace0 \ub9d0\ud588\ub2e4. \uadf8\ub7ec\ub098 \uc774 \uacbd\uae30\uc5d0\uc11c\ub3c4 \uc2ec\ud310\ub4e4\uc740 \uae40\uc5f0\uc544\uc758 \ud50c\ub9bd\uc5d0 \uc5b4\ud150\uc158\uc744 \uc918 \ud3b8\ud30c \ud310\uc815\uc758 \ub17c\ub780\uc774 \uc77c\uc5c8\ub2e4.", "sentence_answer": "\uc774\ub0a0 \uc5f4\ub9b0 \uc1fc\ud2b8\ud504\ub85c\uadf8\ub7a8\uc5d0\uc11c \uc5ec\uc790\uc120\uc218 \ucd5c\ucd08\ub85c \uad6c\uc131\uc810\uc218 8\uc810\ub300 \ub97c \ubc1b\ub294 \ucf8c\uac70\ub3c4 \uc774\ub8e8\uc5c8\ub2e4.", "paragraph_id": "6269375-9-0", "paragraph_question": "question: \uae40\uc5f0\uc544\uac00 2009\ub144 3\uc6d4 \uc138\uacc4 \uc120\uc218\uad8c \ub300\ud68c \uc1fc\ud2b8\ud504\ub85c\uadf8\ub7a8\uc5d0\uc11c \uc5ec\uc790 \ucd5c\ucd08\ub85c \uc138\uc6b4 \uae30\ub85d\uc740?, context: 2009\ub144 3\uc6d4, \ubbf8\uad6d \ub85c\uc2a4\uc564\uc824\ub808\uc2a4\uc5d0\uc11c \uac1c\ucd5c\ub41c 2009\ub144 \uc138\uacc4 \uc120\uc218\uad8c \ub300\ud68c \uc1fc\ud2b8 \ud504\ub85c\uadf8\ub7a8\uc5d0\uc11c\ub294 \ubaa8\ub4e0 \uc810\ud504\uc758 \uc131\uacf5, \uc2a4\ud540\uacfc \uc2a4\ud30c\uc774\ub7f4\uc5d0\uc11c \ub808\ubca8 4, \uc2a4\ud15d \ub808\ubca8 3\uacfc \uac00\uc0b0\uc810 1\uc810\uc744 \ubc1b\ub294 \ub4f1 76.12\uc810\uc774\ub77c\ub294 \uc0ac\uc0c1 \uc720\ub840\uc5c6\ub294 \ub192\uc740 \uc810\uc218\ub97c \ubc1b\uc558\uace0, \uc774\ub85c\uc368 4\ub300\ub959 \uc120\uc218\uad8c \ub300\ud68c\uc5d0\uc11c \uc790\uc2e0\uc774 \uc138\uc6b4 \uc138\uacc4\uae30\ub85d\uc744 \ub2e4\uc2dc \uacbd\uc2e0\ud558\uac8c \ub41c\ub2e4. \uc774\ub0a0 \uc5f4\ub9b0 \uc1fc\ud2b8\ud504\ub85c\uadf8\ub7a8\uc5d0\uc11c \uc5ec\uc790\uc120\uc218 \ucd5c\ucd08\ub85c \uad6c\uc131\uc810\uc218 8\uc810\ub300\ub97c \ubc1b\ub294 \ucf8c\uac70\ub3c4 \uc774\ub8e8\uc5c8\ub2e4. \ub2e4\uc74c\ub0a0 \uc5f4\ub9b0 \ud504\ub9ac\uc2a4\ucf00\uc774\ud305\uc5d0\uc11c\ub294 \uc0b4\ucf54\uc640 \uc2a4\ud540 \uc2e4\uc218\uc5d0\ub3c4 \ubd88\uad6c\ud558\uace0 131.59\uc810\uc744 \ubc1b\uc544 \ucd1d\uc810 207.71\uc744 \uae30\ub85d\ud558\uba70 \uc885\ud569 1\uc704(\uae08\uba54\ub2ec)\uc744 \ucc28\uc9c0\ud55c\ub2e4. \ub610\ud55c \ud504\ub9ac\uc2a4\ucf00\uc774\ud305\uc5d0\uc11c \uad6c\uc131\uc810\uc218 68.40\uc810\uc744 \uae30\ub85d\ud558\uba70 \uc5ec\uc790 \uc120\uc218 \ucd5c\ucd08\ub85c \ud504\ub9ac\uc2a4\ucf00\uc774\ud305\uc5d0\uc11c 8\uc810\ub300\ub97c \ub9c8\ud06c\ud55c \uc120\uc218\uac00 \ub418\uc5c8\ub2e4. \uae40\uc5f0\uc544\uc758 \ud504\ub9ac\uc2a4\ucf00\uc774\ud305\uc758 \uc810\uc218(131.59)\ub294 \uc790\uc2e0\uc774 \uc138\uc6b4 \uc138\uacc4 \ucd5c\uace0 \uae30\ub85d\uc778 2007-2008 \ucef5 \uc624\ube0c \ub7ec\uc2dc\uc544(133.70)\uc640 2007-2008 \uadf8\ub791\ud504\ub9ac \ud30c\uc774\ub110(132.21)\uc5d0 \uc774\uc5b4 3\ubc88\uc9f8\ub85c \ub192\uc740 \uc810\uc218\uc774\ub2e4. 4\ub300\ub959 \uc120\uc218\uad8c \ub300\ud68c\uc640 \ub9c8\ucc2c\uac00\uc9c0\ub85c \uc1fc\ud2b8 \ud504\ub85c\uadf8\ub7a8\uacfc \ud504\ub9ac\uc2a4\ucf00\uc774\ud305\uc5d0\uc11c 3\ud68c\uc804+3\ud68c\uc804 \uc5f0\uc18d \uc810\ud504\ub97c \uc2dc\ub3c4\ud574 \uc1fc\ud2b8, \ud504\ub9ac \ubaa8\ub450 \uc131\uacf5\ud55c \uac83\uc740 \uae40\uc5f0\uc544\ub9cc\uc774 \uc720\uc77c\ud558\ub2e4. \ub610\ud55c \ud53c\uaca8 \uc2a4\ucf00\uc774\ud305 \uc5ec\uc790 \uc2f1\uae00 \uc120\uc218 \uc0ac\uc0c1 \ucd5c\ucd08\ub85c 200\uc810\uc744 \ub118\uc740 \uac83\uc774\uc5c8\ub2e4. \ub2e4\ub978 \uacbd\uae30\uc5d0\uc11c \ub17c\ub780\uc774 \ub418\uc5b4\uc628 \ud50c\ub9bd \uc810\ud504\uc5d0 \ub300\ud574\uc11c\ub294 1992\ub144 \uc62c\ub9bc\ud53d \uc5ec\uc790 \uc2f1\uae00 \uae08\uba54\ub2ec\ub9ac\uc2a4\ud2b8 \ud06c\ub9ac\uc2a4\ud2f0 \uc57c\ub9c8\uad6c\uce58\ub294 \ud50c\ub9bd\uc5d0 \uc5b4\ud150\uc158\uc774 \ubd99\uc5c8\ub2e4\uba74 \uc2ec\ud310\uc740 \ubc18\uc131\ud574\uc57c\ud588\uc744\uac83\uc774\ub77c\uace0 \ub9d0\ud558\uace0, \uc77c\ubcf8\uc758 \uc62c\ub9bc\ud53d \uc740\uba54\ub2ec\ub9ac\uc2a4\ud2b8 \uc774\ud1a0 \ubbf8\ub3c4\ub9ac \ub610\ud55c \uc774\uac83\uc740 \uba85\ubc31\ud55c \ud50c\ub9bd\uc774\ub77c\uace0 \ub9d0\ud588\ub2e4. \uadf8\ub7ec\ub098 \uc774 \uacbd\uae30\uc5d0\uc11c\ub3c4 \uc2ec\ud310\ub4e4\uc740 \uae40\uc5f0\uc544\uc758 \ud50c\ub9bd\uc5d0 \uc5b4\ud150\uc158\uc744 \uc918 \ud3b8\ud30c \ud310\uc815\uc758 \ub17c\ub780\uc774 \uc77c\uc5c8\ub2e4."} {"question": "\ubbf8\uad6d \uc815\ubd80\ub294 '\uacac\uc81c\uc640 \uade0\ud615 \uccb4\uc81c\uc5d0 \ub530\ub77c \ud1b5\uc81c\ub41c\ub2e4'\ub77c\ub294 \uac83\uc740 \ubbf8\uad6d \ucd5c\uc0c1\uc704 \ubc95\uc804\uc778 \uc5b4\ub514\uc5d0 \uaddc\uc815\ub41c \ub0b4\uc6a9\uc778\uac00?", "paragraph": "\ubbf8\ud569\uc911\uad6d\uc740 1776\ub144 \uc131\ub9bd\ub418\uc5b4, \uc804 \uc138\uacc4\uc5d0\uc11c \ud604\uc874\ud558\ub294 \uac00\uc7a5 \uc624\ub798\ub41c \uc5f0\ubc29 \uad6d\uac00\ub2e4. \ubbf8\uad6d\uc740 \"\ubc95\uc5d0 \uc758\ud558\uc5ec \uc18c\uc218\uc790\uc758 \uad8c\ub9ac\ub97c \ubcf4\ud638\ud558\uc5ec, \ub2e4\uc218\uacb0\uc758 \uc6d0\uce59\uc744 \uc870\uc815\ud558\ub294\" \ub300\uc758 \ubbfc\uc8fc\uc8fc\uc758 \uc785\ud5cc \uacf5\ud654\uad6d\uc774\ub2e4. \ubbf8\uad6d \uc815\ubd80\ub294 \ubbf8\uad6d \ucd5c\uc0c1\uc704 \ubc95\uc804\uc778 \ud5cc\ubc95\uc5d0 \uaddc\uc815\ub41c \uacac\uc81c\uc640 \uade0\ud615 \uccb4\uc81c\uc5d0 \ub530\ub77c \ud1b5\uc81c\ub41c\ub2e4. \ubbf8\uad6d\uc758 \uc5f0\ubc29 \uc81c\ub3c4\uc5d0\uc11c \uc2dc\ubbfc\uc740 \ubcf4\ud1b5 \uc138 \ub2e8\uacc4\uc758 \uc815\ubd80 \uc989 \uc5f0\ubc29, \uc8fc, \uc9c0\uc5ed \ub2e8\uc704\uc5d0 \uc18d\ud55c\ub2e4. \ubbf8\uad6d \ud5cc\ubc95\uc740 \uc5f0\ubc29 \uc815\ubd80\uc758 \uad8c\ud55c\uc5d0 \uad00\ud55c \uac83\uc744 \uad6c\uc131 \uc8fc\uc5d0\uc11c \uc704\uc784\ubc1b\uc558\ub294\ub370, \uadf8 \uac00\uc6b4\ub370 \uad6d\ubc29\u00b7\uc678\uad50 \uc815\ucc45\u00b7\ub300\uc678 \ubb34\uc5ed\uc758 \uc870\uc815 \uae30\ub2a5, \ud1b5\ud654, \ucd5c\uace0 \ubc95 \uc9d1\ud589\uae30\ub2a5, \uc8fc\uac04 \ud1b5\uc0c1\uc758 \uc870\uc815, \uc774\ubbfc \ub4f1\uc740 \uc5f0\ubc29\uc815\ubd80\uc758 \uac00\uc7a5 \uc911\uc694\ud55c \ucc45\uc784\uad8c\uc5d0 \uc18d\ud55c\ub2e4. \uc8fc \uc815\ubd80\uc758 \uc8fc\uc694 \uae30\ub2a5\uc740 \uad50\uc721, \ub18d\uc5c5, \uc790\uc5f0\ubcf4\ud638, \uace0\uc18d\ub3c4\ub85c \uad00\ub9ac, \ucc28\ub7c9 \uac10\ub3c5, \uacf5\uacf5\uc548\ub155, \uad50\ub3c4\ud589\uc815, \uc8fc\ub0b4 \ud1b5\uc0c1\uc758 \uc870\uc815, \uad50\uc721\u00b7\ubcf4\uac74\u00b7\ubcf5\uc9c0 \uc815\ucc45\uc758 \uc2dc\ud589 \ub4f1\uc774\ub2e4. \uc9c0\uc5ed \uc815\ubd80\uc758 \uc5c5\ubb34\ub294 \ubcf4\ud1b5 \uac01 \uad70\uacfc \uc2dc \uc815\ubd80\uc5d0 \ub098\ub220 \ub9e1\uae30\uace0 \uc788\ub2e4. \uac70\uc758 \ubaa8\ub4e0 \uacbd\uc6b0\uc5d0 \ud589\uc815\ubd80\uc640 \uc785\ubc95\ubd80 \uacf5\ubb34\uc6d0\uc740 \uad6c\uc5ed\ubcc4\ub85c \uc2dc\ubbfc\uc758 \ub2e4\uc218 \ud22c\ud45c\uc5d0 \ub530\ub77c \uc120\ucd9c\ub41c\ub2e4. \uc5f0\ubc29 \ucc28\uc6d0\uc758 \ube44\ub840\ub300\ud45c\uc81c\ub294 \uc5c6\uc73c\uba70, \ud558\uc704 \uc815\ubd80 \ub2e8\uc704\uc5d0\uc11c\ub3c4 \uc774 \uc81c\ub3c4\ub294 \ub4dc\ubb3c\ub2e4.", "answer": "\ud5cc\ubc95", "sentence": "\ubbf8\uad6d \uc815\ubd80\ub294 \ubbf8\uad6d \ucd5c\uc0c1\uc704 \ubc95\uc804\uc778 \ud5cc\ubc95 \uc5d0 \uaddc\uc815\ub41c \uacac\uc81c\uc640 \uade0\ud615 \uccb4\uc81c\uc5d0 \ub530\ub77c \ud1b5\uc81c\ub41c\ub2e4.", "paragraph_sentence": "\ubbf8\ud569\uc911\uad6d\uc740 1776\ub144 \uc131\ub9bd\ub418\uc5b4, \uc804 \uc138\uacc4\uc5d0\uc11c \ud604\uc874\ud558\ub294 \uac00\uc7a5 \uc624\ub798\ub41c \uc5f0\ubc29 \uad6d\uac00\ub2e4. \ubbf8\uad6d\uc740 \"\ubc95\uc5d0 \uc758\ud558\uc5ec \uc18c\uc218\uc790\uc758 \uad8c\ub9ac\ub97c \ubcf4\ud638\ud558\uc5ec, \ub2e4\uc218\uacb0\uc758 \uc6d0\uce59\uc744 \uc870\uc815\ud558\ub294\" \ub300\uc758 \ubbfc\uc8fc\uc8fc\uc758 \uc785\ud5cc \uacf5\ud654\uad6d\uc774\ub2e4. \ubbf8\uad6d \uc815\ubd80\ub294 \ubbf8\uad6d \ucd5c\uc0c1\uc704 \ubc95\uc804\uc778 \ud5cc\ubc95 \uc5d0 \uaddc\uc815\ub41c \uacac\uc81c\uc640 \uade0\ud615 \uccb4\uc81c\uc5d0 \ub530\ub77c \ud1b5\uc81c\ub41c\ub2e4. \ubbf8\uad6d\uc758 \uc5f0\ubc29 \uc81c\ub3c4\uc5d0\uc11c \uc2dc\ubbfc\uc740 \ubcf4\ud1b5 \uc138 \ub2e8\uacc4\uc758 \uc815\ubd80 \uc989 \uc5f0\ubc29, \uc8fc, \uc9c0\uc5ed \ub2e8\uc704\uc5d0 \uc18d\ud55c\ub2e4. \ubbf8\uad6d \ud5cc\ubc95\uc740 \uc5f0\ubc29 \uc815\ubd80\uc758 \uad8c\ud55c\uc5d0 \uad00\ud55c \uac83\uc744 \uad6c\uc131 \uc8fc\uc5d0\uc11c \uc704\uc784\ubc1b\uc558\ub294\ub370, \uadf8 \uac00\uc6b4\ub370 \uad6d\ubc29\u00b7\uc678\uad50 \uc815\ucc45\u00b7\ub300\uc678 \ubb34\uc5ed\uc758 \uc870\uc815 \uae30\ub2a5, \ud1b5\ud654, \ucd5c\uace0 \ubc95 \uc9d1\ud589\uae30\ub2a5, \uc8fc\uac04 \ud1b5\uc0c1\uc758 \uc870\uc815, \uc774\ubbfc \ub4f1\uc740 \uc5f0\ubc29\uc815\ubd80\uc758 \uac00\uc7a5 \uc911\uc694\ud55c \ucc45\uc784\uad8c\uc5d0 \uc18d\ud55c\ub2e4. \uc8fc \uc815\ubd80\uc758 \uc8fc\uc694 \uae30\ub2a5\uc740 \uad50\uc721, \ub18d\uc5c5, \uc790\uc5f0\ubcf4\ud638, \uace0\uc18d\ub3c4\ub85c \uad00\ub9ac, \ucc28\ub7c9 \uac10\ub3c5, \uacf5\uacf5\uc548\ub155, \uad50\ub3c4\ud589\uc815, \uc8fc\ub0b4 \ud1b5\uc0c1\uc758 \uc870\uc815, \uad50\uc721\u00b7\ubcf4\uac74\u00b7\ubcf5\uc9c0 \uc815\ucc45\uc758 \uc2dc\ud589 \ub4f1\uc774\ub2e4. \uc9c0\uc5ed \uc815\ubd80\uc758 \uc5c5\ubb34\ub294 \ubcf4\ud1b5 \uac01 \uad70\uacfc \uc2dc \uc815\ubd80\uc5d0 \ub098\ub220 \ub9e1\uae30\uace0 \uc788\ub2e4. \uac70\uc758 \ubaa8\ub4e0 \uacbd\uc6b0\uc5d0 \ud589\uc815\ubd80\uc640 \uc785\ubc95\ubd80 \uacf5\ubb34\uc6d0\uc740 \uad6c\uc5ed\ubcc4\ub85c \uc2dc\ubbfc\uc758 \ub2e4\uc218 \ud22c\ud45c\uc5d0 \ub530\ub77c \uc120\ucd9c\ub41c\ub2e4. \uc5f0\ubc29 \ucc28\uc6d0\uc758 \ube44\ub840\ub300\ud45c\uc81c\ub294 \uc5c6\uc73c\uba70, \ud558\uc704 \uc815\ubd80 \ub2e8\uc704\uc5d0\uc11c\ub3c4 \uc774 \uc81c\ub3c4\ub294 \ub4dc\ubb3c\ub2e4.", "paragraph_answer": "\ubbf8\ud569\uc911\uad6d\uc740 1776\ub144 \uc131\ub9bd\ub418\uc5b4, \uc804 \uc138\uacc4\uc5d0\uc11c \ud604\uc874\ud558\ub294 \uac00\uc7a5 \uc624\ub798\ub41c \uc5f0\ubc29 \uad6d\uac00\ub2e4. \ubbf8\uad6d\uc740 \"\ubc95\uc5d0 \uc758\ud558\uc5ec \uc18c\uc218\uc790\uc758 \uad8c\ub9ac\ub97c \ubcf4\ud638\ud558\uc5ec, \ub2e4\uc218\uacb0\uc758 \uc6d0\uce59\uc744 \uc870\uc815\ud558\ub294\" \ub300\uc758 \ubbfc\uc8fc\uc8fc\uc758 \uc785\ud5cc \uacf5\ud654\uad6d\uc774\ub2e4. \ubbf8\uad6d \uc815\ubd80\ub294 \ubbf8\uad6d \ucd5c\uc0c1\uc704 \ubc95\uc804\uc778 \ud5cc\ubc95 \uc5d0 \uaddc\uc815\ub41c \uacac\uc81c\uc640 \uade0\ud615 \uccb4\uc81c\uc5d0 \ub530\ub77c \ud1b5\uc81c\ub41c\ub2e4. \ubbf8\uad6d\uc758 \uc5f0\ubc29 \uc81c\ub3c4\uc5d0\uc11c \uc2dc\ubbfc\uc740 \ubcf4\ud1b5 \uc138 \ub2e8\uacc4\uc758 \uc815\ubd80 \uc989 \uc5f0\ubc29, \uc8fc, \uc9c0\uc5ed \ub2e8\uc704\uc5d0 \uc18d\ud55c\ub2e4. \ubbf8\uad6d \ud5cc\ubc95\uc740 \uc5f0\ubc29 \uc815\ubd80\uc758 \uad8c\ud55c\uc5d0 \uad00\ud55c \uac83\uc744 \uad6c\uc131 \uc8fc\uc5d0\uc11c \uc704\uc784\ubc1b\uc558\ub294\ub370, \uadf8 \uac00\uc6b4\ub370 \uad6d\ubc29\u00b7\uc678\uad50 \uc815\ucc45\u00b7\ub300\uc678 \ubb34\uc5ed\uc758 \uc870\uc815 \uae30\ub2a5, \ud1b5\ud654, \ucd5c\uace0 \ubc95 \uc9d1\ud589\uae30\ub2a5, \uc8fc\uac04 \ud1b5\uc0c1\uc758 \uc870\uc815, \uc774\ubbfc \ub4f1\uc740 \uc5f0\ubc29\uc815\ubd80\uc758 \uac00\uc7a5 \uc911\uc694\ud55c \ucc45\uc784\uad8c\uc5d0 \uc18d\ud55c\ub2e4. \uc8fc \uc815\ubd80\uc758 \uc8fc\uc694 \uae30\ub2a5\uc740 \uad50\uc721, \ub18d\uc5c5, \uc790\uc5f0\ubcf4\ud638, \uace0\uc18d\ub3c4\ub85c \uad00\ub9ac, \ucc28\ub7c9 \uac10\ub3c5, \uacf5\uacf5\uc548\ub155, \uad50\ub3c4\ud589\uc815, \uc8fc\ub0b4 \ud1b5\uc0c1\uc758 \uc870\uc815, \uad50\uc721\u00b7\ubcf4\uac74\u00b7\ubcf5\uc9c0 \uc815\ucc45\uc758 \uc2dc\ud589 \ub4f1\uc774\ub2e4. \uc9c0\uc5ed \uc815\ubd80\uc758 \uc5c5\ubb34\ub294 \ubcf4\ud1b5 \uac01 \uad70\uacfc \uc2dc \uc815\ubd80\uc5d0 \ub098\ub220 \ub9e1\uae30\uace0 \uc788\ub2e4. \uac70\uc758 \ubaa8\ub4e0 \uacbd\uc6b0\uc5d0 \ud589\uc815\ubd80\uc640 \uc785\ubc95\ubd80 \uacf5\ubb34\uc6d0\uc740 \uad6c\uc5ed\ubcc4\ub85c \uc2dc\ubbfc\uc758 \ub2e4\uc218 \ud22c\ud45c\uc5d0 \ub530\ub77c \uc120\ucd9c\ub41c\ub2e4. \uc5f0\ubc29 \ucc28\uc6d0\uc758 \ube44\ub840\ub300\ud45c\uc81c\ub294 \uc5c6\uc73c\uba70, \ud558\uc704 \uc815\ubd80 \ub2e8\uc704\uc5d0\uc11c\ub3c4 \uc774 \uc81c\ub3c4\ub294 \ub4dc\ubb3c\ub2e4.", "sentence_answer": "\ubbf8\uad6d \uc815\ubd80\ub294 \ubbf8\uad6d \ucd5c\uc0c1\uc704 \ubc95\uc804\uc778 \ud5cc\ubc95 \uc5d0 \uaddc\uc815\ub41c \uacac\uc81c\uc640 \uade0\ud615 \uccb4\uc81c\uc5d0 \ub530\ub77c \ud1b5\uc81c\ub41c\ub2e4.", "paragraph_id": "6514860-27-1", "paragraph_question": "question: \ubbf8\uad6d \uc815\ubd80\ub294 '\uacac\uc81c\uc640 \uade0\ud615 \uccb4\uc81c\uc5d0 \ub530\ub77c \ud1b5\uc81c\ub41c\ub2e4'\ub77c\ub294 \uac83\uc740 \ubbf8\uad6d \ucd5c\uc0c1\uc704 \ubc95\uc804\uc778 \uc5b4\ub514\uc5d0 \uaddc\uc815\ub41c \ub0b4\uc6a9\uc778\uac00?, context: \ubbf8\ud569\uc911\uad6d\uc740 1776\ub144 \uc131\ub9bd\ub418\uc5b4, \uc804 \uc138\uacc4\uc5d0\uc11c \ud604\uc874\ud558\ub294 \uac00\uc7a5 \uc624\ub798\ub41c \uc5f0\ubc29 \uad6d\uac00\ub2e4. \ubbf8\uad6d\uc740 \"\ubc95\uc5d0 \uc758\ud558\uc5ec \uc18c\uc218\uc790\uc758 \uad8c\ub9ac\ub97c \ubcf4\ud638\ud558\uc5ec, \ub2e4\uc218\uacb0\uc758 \uc6d0\uce59\uc744 \uc870\uc815\ud558\ub294\" \ub300\uc758 \ubbfc\uc8fc\uc8fc\uc758 \uc785\ud5cc \uacf5\ud654\uad6d\uc774\ub2e4. \ubbf8\uad6d \uc815\ubd80\ub294 \ubbf8\uad6d \ucd5c\uc0c1\uc704 \ubc95\uc804\uc778 \ud5cc\ubc95\uc5d0 \uaddc\uc815\ub41c \uacac\uc81c\uc640 \uade0\ud615 \uccb4\uc81c\uc5d0 \ub530\ub77c \ud1b5\uc81c\ub41c\ub2e4. \ubbf8\uad6d\uc758 \uc5f0\ubc29 \uc81c\ub3c4\uc5d0\uc11c \uc2dc\ubbfc\uc740 \ubcf4\ud1b5 \uc138 \ub2e8\uacc4\uc758 \uc815\ubd80 \uc989 \uc5f0\ubc29, \uc8fc, \uc9c0\uc5ed \ub2e8\uc704\uc5d0 \uc18d\ud55c\ub2e4. \ubbf8\uad6d \ud5cc\ubc95\uc740 \uc5f0\ubc29 \uc815\ubd80\uc758 \uad8c\ud55c\uc5d0 \uad00\ud55c \uac83\uc744 \uad6c\uc131 \uc8fc\uc5d0\uc11c \uc704\uc784\ubc1b\uc558\ub294\ub370, \uadf8 \uac00\uc6b4\ub370 \uad6d\ubc29\u00b7\uc678\uad50 \uc815\ucc45\u00b7\ub300\uc678 \ubb34\uc5ed\uc758 \uc870\uc815 \uae30\ub2a5, \ud1b5\ud654, \ucd5c\uace0 \ubc95 \uc9d1\ud589\uae30\ub2a5, \uc8fc\uac04 \ud1b5\uc0c1\uc758 \uc870\uc815, \uc774\ubbfc \ub4f1\uc740 \uc5f0\ubc29\uc815\ubd80\uc758 \uac00\uc7a5 \uc911\uc694\ud55c \ucc45\uc784\uad8c\uc5d0 \uc18d\ud55c\ub2e4. \uc8fc \uc815\ubd80\uc758 \uc8fc\uc694 \uae30\ub2a5\uc740 \uad50\uc721, \ub18d\uc5c5, \uc790\uc5f0\ubcf4\ud638, \uace0\uc18d\ub3c4\ub85c \uad00\ub9ac, \ucc28\ub7c9 \uac10\ub3c5, \uacf5\uacf5\uc548\ub155, \uad50\ub3c4\ud589\uc815, \uc8fc\ub0b4 \ud1b5\uc0c1\uc758 \uc870\uc815, \uad50\uc721\u00b7\ubcf4\uac74\u00b7\ubcf5\uc9c0 \uc815\ucc45\uc758 \uc2dc\ud589 \ub4f1\uc774\ub2e4. \uc9c0\uc5ed \uc815\ubd80\uc758 \uc5c5\ubb34\ub294 \ubcf4\ud1b5 \uac01 \uad70\uacfc \uc2dc \uc815\ubd80\uc5d0 \ub098\ub220 \ub9e1\uae30\uace0 \uc788\ub2e4. \uac70\uc758 \ubaa8\ub4e0 \uacbd\uc6b0\uc5d0 \ud589\uc815\ubd80\uc640 \uc785\ubc95\ubd80 \uacf5\ubb34\uc6d0\uc740 \uad6c\uc5ed\ubcc4\ub85c \uc2dc\ubbfc\uc758 \ub2e4\uc218 \ud22c\ud45c\uc5d0 \ub530\ub77c \uc120\ucd9c\ub41c\ub2e4. \uc5f0\ubc29 \ucc28\uc6d0\uc758 \ube44\ub840\ub300\ud45c\uc81c\ub294 \uc5c6\uc73c\uba70, \ud558\uc704 \uc815\ubd80 \ub2e8\uc704\uc5d0\uc11c\ub3c4 \uc774 \uc81c\ub3c4\ub294 \ub4dc\ubb3c\ub2e4."} {"question": "2001\ub144 3\uc6d4 23\uc77c \ucd9c\uc804\ud55c \uacbd\uae30\uc758 \uc0c1\ub300\ud300\uc740?", "paragraph": "2000\ub144, 11\ub144 \ub9cc\uc5d0 2\uad70 \uc2a4\ud504\ub9c1 \ucea0\ud504\uc5d0\uc11c \uc2dc\uc98c\uc744 \ub9de\uc774\ud558\uac8c \ub410\uace0 \ucc98\uc74c\uc73c\ub85c 1\uad70 \ub4f1\ud310\uc744 \ud558\uc9c0 \uc54a\uc740 \ucc44 \uc2dc\uc98c\uc744 \ub9c8\ucce4\ub2e4. \uc2dc\uc98c \uc885\ub8cc \ud6c4 \uacc4\uc57d \uac31\uc2e0\uc5d0\uc11c\ub294 \uc5f0\ubd09\uc774 \ud070 \ud3ed\uc73c\ub85c \uc0ad\uac10\ub3fc \uc774\ub4ec\ud574 2\uc6d4\uae4c\uc9c0 \uac78\ub838\uace0 \ud638\uc2dc\ub178 \uac10\ub3c5\uc5d0\uac8c\uc11c\ub294 \u2018\uacc4\uc57d\ud558\ub294 \uac83\ub9cc\uc73c\ub85c\ub3c4 \uac10\uc0ac\ud558\uac8c \uc0dd\uac01\ud560 \uac83\u2019\uc774\ub77c\ub294 \ucd5c\ud6c4 \ud1b5\uace0\ub97c \ubc1b\uc558\ub2e4. \ud55c\ud3b8 2001\ub144 1\uc6d4\uc5d0\ub294 \uc774\ub77c\uace0\uacf6\uc5d0\uc11c\uc758 \ud569\ub3d9 \uc790\uc8fc \ud2b8\ub808\uc774\ub2dd\uc5d0 \uc801\uadf9\uc801\uc73c\ub85c \ub7f0\ub2dd\uc5d0 \uc784\ud558\ub294 \uc790\uc138\ub97c \uc57c\ub9c8\ub2e4 \ud788\uc0ac\uc2dc \ud22c\uc218 \ucf54\uce58\ub85c\ubd80\ud130 \ud3c9\uac00\ub97c \ubc1b\uc544 2\ub144 \ub9cc\uc5d0 1\uad70 \uc2a4\ud504\ub9c1 \ucea0\ud504\uc5d0\uc11c\uc758 \uc2dc\uc98c\uc744 \ub9de\uc774\ud588\ub2e4. \uc2a4\ud504\ub9c1 \ucea0\ud504 \uc9c1\uc804\uc778 1\uc6d4 30\uc77c\uc5d0\ub294 \uc694\ubbf8\ud0c4 \uad6c\uc7a5\uc5d0\uc11c\uc758 \ubd88\ud39c\uc5d0\uc11c 100\uac1c\uc758 \uacf5\uc744 \ub358\uc9c0\uace0 \uc2a4\ud504\ub9c1 \ucea0\ud504 \uc911\uc5d0\ub3c4 \ud22c\uad6c \uc5f0\uc2b5\uc758 \ud22c\uad6c \uc218\ub97c \uc18c\ud654\ud574\ub0c8\uc9c0\ub9cc \uc81c\uad6c\uc5d0 \ubb38\uc81c\uac00 \uc788\uc5c8\ub2e4. \uc2dc\ubc94 \uacbd\uae30\uc5d0\uc11c\ub294 \uad6c\uc18d\uc774 \ucd5c\uace0 131 km/h\uc5d0 \uba38\ubb3c\ub800\uc9c0\ub9cc \ud3ec\ud06c\ubcfc\uc774\ub098 \uc288\ud1a0\ub97c \uad6c\uc0ac\ud574\uc11c 3\uc6d4 3\uc77c \uc9c0\ubc14 \ub86f\ub370\uc804\uc5d0\uc11c\ub294 4\uc774\ub2dd\uc744 1\uc2e4\uc810\uc73c\ub85c \ub9c9\uc558\ub2e4. \uadf8\ub7ec\ub098 \uc57c\uc218\ub4e4\uc758 \uc0c1\ud0dc\uac00 \ud638\uc804\ub428\uc5d0 \ub530\ub77c\uc11c \uc548\ud0c0\ub098 \ud648\ub7f0\uc744 \ub9de\ub294 \uc0c1\ud669\uc774 \uc18d\ucd9c\ud558\uc790, 3\uc6d4 23\uc77c \uad50\uc721 \ub9ac\uadf8\uc5d0\uc11c\uc758 \ud55c\uc2e0 \ud0c0\uc774\uac70\uc2a4\uc640\uc758 \uacbd\uae30\uc5d0\uc11c\ub294 5\uc774\ub2dd \ub3d9\uc548 106\uac1c\uc758 \uacf5\uc744 \ub358\uc838 8\ud53c\uc548\ud0c0 5\ubcfc\ub137 3\uc2e4\uc810\uc774\ub77c\ub294 \ucd5c\uc545\uc758 \ud22c\uad6c \ub0b4\uc6a9\uc73c\ub85c \uac1c\ub9c9\uc804 1\uad70 \uc5d4\ud2b8\ub9ac \ub4f1\ub85d\uc5d0\ub294 \uc2e4\ud328\ud588\ub2e4. \uadf8 \ud574\uc5d0\ub294 7\uacbd\uae30\uc5d0 \ub4f1\ud310\ud588\uc9c0\ub9cc \uccb4\ub825\uc758 \ud55c\uacc4 \ub4f1\uc744 \uc774\uc720\ub85c 10\uc6d4\uc5d0 \ud604\uc5ed \uc740\ud1f4\ub97c \uacb0\uc2ec\ud588\ub2e4. \uc57c\ub9c8\ub2e4\ub294 \uc911\uac04 \uacc4\ud22c\ub85c\uc11c\uc758 \uc7ac\uc0dd\uc744 \uc0dd\uac01\ud588\uace0 \ud0c0 \uad6c\ub2e8\uc73c\ub85c\ubd80\ud130\ub3c4 \uc601\uc785 \uc81c\uc758\ub97c \ubc1b\uc558\ub2e4\uace0 \ud55c\ub2e4. 11\uc6d4 10\uc77c\uc5d0 \uc740\ud1f4 \uae30\uc790\ud68c\uacac\uc744 \uac00\uc84c\ub2e4.", "answer": "\ud55c\uc2e0 \ud0c0\uc774\uac70\uc2a4", "sentence": "\uadf8\ub7ec\ub098 \uc57c\uc218\ub4e4\uc758 \uc0c1\ud0dc\uac00 \ud638\uc804\ub428\uc5d0 \ub530\ub77c\uc11c \uc548\ud0c0\ub098 \ud648\ub7f0\uc744 \ub9de\ub294 \uc0c1\ud669\uc774 \uc18d\ucd9c\ud558\uc790, 3\uc6d4 23\uc77c \uad50\uc721 \ub9ac\uadf8\uc5d0\uc11c\uc758 \ud55c\uc2e0 \ud0c0\uc774\uac70\uc2a4 \uc640\uc758 \uacbd\uae30\uc5d0\uc11c\ub294 5\uc774\ub2dd \ub3d9\uc548 106\uac1c\uc758 \uacf5\uc744 \ub358\uc838 8\ud53c\uc548\ud0c0 5\ubcfc\ub137 3\uc2e4\uc810\uc774\ub77c\ub294 \ucd5c\uc545\uc758 \ud22c\uad6c \ub0b4\uc6a9\uc73c\ub85c \uac1c\ub9c9\uc804 1\uad70 \uc5d4\ud2b8\ub9ac \ub4f1\ub85d\uc5d0\ub294 \uc2e4\ud328\ud588\ub2e4.", "paragraph_sentence": "2000\ub144, 11\ub144 \ub9cc\uc5d0 2\uad70 \uc2a4\ud504\ub9c1 \ucea0\ud504\uc5d0\uc11c \uc2dc\uc98c\uc744 \ub9de\uc774\ud558\uac8c \ub410\uace0 \ucc98\uc74c\uc73c\ub85c 1\uad70 \ub4f1\ud310\uc744 \ud558\uc9c0 \uc54a\uc740 \ucc44 \uc2dc\uc98c\uc744 \ub9c8\ucce4\ub2e4. \uc2dc\uc98c \uc885\ub8cc \ud6c4 \uacc4\uc57d \uac31\uc2e0\uc5d0\uc11c\ub294 \uc5f0\ubd09\uc774 \ud070 \ud3ed\uc73c\ub85c \uc0ad\uac10\ub3fc \uc774\ub4ec\ud574 2\uc6d4\uae4c\uc9c0 \uac78\ub838\uace0 \ud638\uc2dc\ub178 \uac10\ub3c5\uc5d0\uac8c\uc11c\ub294 \u2018\uacc4\uc57d\ud558\ub294 \uac83\ub9cc\uc73c\ub85c\ub3c4 \uac10\uc0ac\ud558\uac8c \uc0dd\uac01\ud560 \uac83\u2019\uc774\ub77c\ub294 \ucd5c\ud6c4 \ud1b5\uace0\ub97c \ubc1b\uc558\ub2e4. \ud55c\ud3b8 2001\ub144 1\uc6d4\uc5d0\ub294 \uc774\ub77c\uace0\uacf6\uc5d0\uc11c\uc758 \ud569\ub3d9 \uc790\uc8fc \ud2b8\ub808\uc774\ub2dd\uc5d0 \uc801\uadf9\uc801\uc73c\ub85c \ub7f0\ub2dd\uc5d0 \uc784\ud558\ub294 \uc790\uc138\ub97c \uc57c\ub9c8\ub2e4 \ud788\uc0ac\uc2dc \ud22c\uc218 \ucf54\uce58\ub85c\ubd80\ud130 \ud3c9\uac00\ub97c \ubc1b\uc544 2\ub144 \ub9cc\uc5d0 1\uad70 \uc2a4\ud504\ub9c1 \ucea0\ud504\uc5d0\uc11c\uc758 \uc2dc\uc98c\uc744 \ub9de\uc774\ud588\ub2e4. \uc2a4\ud504\ub9c1 \ucea0\ud504 \uc9c1\uc804\uc778 1\uc6d4 30\uc77c\uc5d0\ub294 \uc694\ubbf8\ud0c4 \uad6c\uc7a5\uc5d0\uc11c\uc758 \ubd88\ud39c\uc5d0\uc11c 100\uac1c\uc758 \uacf5\uc744 \ub358\uc9c0\uace0 \uc2a4\ud504\ub9c1 \ucea0\ud504 \uc911\uc5d0\ub3c4 \ud22c\uad6c \uc5f0\uc2b5\uc758 \ud22c\uad6c \uc218\ub97c \uc18c\ud654\ud574\ub0c8\uc9c0\ub9cc \uc81c\uad6c\uc5d0 \ubb38\uc81c\uac00 \uc788\uc5c8\ub2e4. \uc2dc\ubc94 \uacbd\uae30\uc5d0\uc11c\ub294 \uad6c\uc18d\uc774 \ucd5c\uace0 131 km/h\uc5d0 \uba38\ubb3c\ub800\uc9c0\ub9cc \ud3ec\ud06c\ubcfc\uc774\ub098 \uc288\ud1a0\ub97c \uad6c\uc0ac\ud574\uc11c 3\uc6d4 3\uc77c \uc9c0\ubc14 \ub86f\ub370\uc804\uc5d0\uc11c\ub294 4\uc774\ub2dd\uc744 1\uc2e4\uc810\uc73c\ub85c \ub9c9\uc558\ub2e4. \uadf8\ub7ec\ub098 \uc57c\uc218\ub4e4\uc758 \uc0c1\ud0dc\uac00 \ud638\uc804\ub428\uc5d0 \ub530\ub77c\uc11c \uc548\ud0c0\ub098 \ud648\ub7f0\uc744 \ub9de\ub294 \uc0c1\ud669\uc774 \uc18d\ucd9c\ud558\uc790, 3\uc6d4 23\uc77c \uad50\uc721 \ub9ac\uadf8\uc5d0\uc11c\uc758 \ud55c\uc2e0 \ud0c0\uc774\uac70\uc2a4 \uc640\uc758 \uacbd\uae30\uc5d0\uc11c\ub294 5\uc774\ub2dd \ub3d9\uc548 106\uac1c\uc758 \uacf5\uc744 \ub358\uc838 8\ud53c\uc548\ud0c0 5\ubcfc\ub137 3\uc2e4\uc810\uc774\ub77c\ub294 \ucd5c\uc545\uc758 \ud22c\uad6c \ub0b4\uc6a9\uc73c\ub85c \uac1c\ub9c9\uc804 1\uad70 \uc5d4\ud2b8\ub9ac \ub4f1\ub85d\uc5d0\ub294 \uc2e4\ud328\ud588\ub2e4. \uadf8 \ud574\uc5d0\ub294 7\uacbd\uae30\uc5d0 \ub4f1\ud310\ud588\uc9c0\ub9cc \uccb4\ub825\uc758 \ud55c\uacc4 \ub4f1\uc744 \uc774\uc720\ub85c 10\uc6d4\uc5d0 \ud604\uc5ed \uc740\ud1f4\ub97c \uacb0\uc2ec\ud588\ub2e4. \uc57c\ub9c8\ub2e4\ub294 \uc911\uac04 \uacc4\ud22c\ub85c\uc11c\uc758 \uc7ac\uc0dd\uc744 \uc0dd\uac01\ud588\uace0 \ud0c0 \uad6c\ub2e8\uc73c\ub85c\ubd80\ud130\ub3c4 \uc601\uc785 \uc81c\uc758\ub97c \ubc1b\uc558\ub2e4\uace0 \ud55c\ub2e4. 11\uc6d4 10\uc77c\uc5d0 \uc740\ud1f4 \uae30\uc790\ud68c\uacac\uc744 \uac00\uc84c\ub2e4.", "paragraph_answer": "2000\ub144, 11\ub144 \ub9cc\uc5d0 2\uad70 \uc2a4\ud504\ub9c1 \ucea0\ud504\uc5d0\uc11c \uc2dc\uc98c\uc744 \ub9de\uc774\ud558\uac8c \ub410\uace0 \ucc98\uc74c\uc73c\ub85c 1\uad70 \ub4f1\ud310\uc744 \ud558\uc9c0 \uc54a\uc740 \ucc44 \uc2dc\uc98c\uc744 \ub9c8\ucce4\ub2e4. \uc2dc\uc98c \uc885\ub8cc \ud6c4 \uacc4\uc57d \uac31\uc2e0\uc5d0\uc11c\ub294 \uc5f0\ubd09\uc774 \ud070 \ud3ed\uc73c\ub85c \uc0ad\uac10\ub3fc \uc774\ub4ec\ud574 2\uc6d4\uae4c\uc9c0 \uac78\ub838\uace0 \ud638\uc2dc\ub178 \uac10\ub3c5\uc5d0\uac8c\uc11c\ub294 \u2018\uacc4\uc57d\ud558\ub294 \uac83\ub9cc\uc73c\ub85c\ub3c4 \uac10\uc0ac\ud558\uac8c \uc0dd\uac01\ud560 \uac83\u2019\uc774\ub77c\ub294 \ucd5c\ud6c4 \ud1b5\uace0\ub97c \ubc1b\uc558\ub2e4. \ud55c\ud3b8 2001\ub144 1\uc6d4\uc5d0\ub294 \uc774\ub77c\uace0\uacf6\uc5d0\uc11c\uc758 \ud569\ub3d9 \uc790\uc8fc \ud2b8\ub808\uc774\ub2dd\uc5d0 \uc801\uadf9\uc801\uc73c\ub85c \ub7f0\ub2dd\uc5d0 \uc784\ud558\ub294 \uc790\uc138\ub97c \uc57c\ub9c8\ub2e4 \ud788\uc0ac\uc2dc \ud22c\uc218 \ucf54\uce58\ub85c\ubd80\ud130 \ud3c9\uac00\ub97c \ubc1b\uc544 2\ub144 \ub9cc\uc5d0 1\uad70 \uc2a4\ud504\ub9c1 \ucea0\ud504\uc5d0\uc11c\uc758 \uc2dc\uc98c\uc744 \ub9de\uc774\ud588\ub2e4. \uc2a4\ud504\ub9c1 \ucea0\ud504 \uc9c1\uc804\uc778 1\uc6d4 30\uc77c\uc5d0\ub294 \uc694\ubbf8\ud0c4 \uad6c\uc7a5\uc5d0\uc11c\uc758 \ubd88\ud39c\uc5d0\uc11c 100\uac1c\uc758 \uacf5\uc744 \ub358\uc9c0\uace0 \uc2a4\ud504\ub9c1 \ucea0\ud504 \uc911\uc5d0\ub3c4 \ud22c\uad6c \uc5f0\uc2b5\uc758 \ud22c\uad6c \uc218\ub97c \uc18c\ud654\ud574\ub0c8\uc9c0\ub9cc \uc81c\uad6c\uc5d0 \ubb38\uc81c\uac00 \uc788\uc5c8\ub2e4. \uc2dc\ubc94 \uacbd\uae30\uc5d0\uc11c\ub294 \uad6c\uc18d\uc774 \ucd5c\uace0 131 km/h\uc5d0 \uba38\ubb3c\ub800\uc9c0\ub9cc \ud3ec\ud06c\ubcfc\uc774\ub098 \uc288\ud1a0\ub97c \uad6c\uc0ac\ud574\uc11c 3\uc6d4 3\uc77c \uc9c0\ubc14 \ub86f\ub370\uc804\uc5d0\uc11c\ub294 4\uc774\ub2dd\uc744 1\uc2e4\uc810\uc73c\ub85c \ub9c9\uc558\ub2e4. \uadf8\ub7ec\ub098 \uc57c\uc218\ub4e4\uc758 \uc0c1\ud0dc\uac00 \ud638\uc804\ub428\uc5d0 \ub530\ub77c\uc11c \uc548\ud0c0\ub098 \ud648\ub7f0\uc744 \ub9de\ub294 \uc0c1\ud669\uc774 \uc18d\ucd9c\ud558\uc790, 3\uc6d4 23\uc77c \uad50\uc721 \ub9ac\uadf8\uc5d0\uc11c\uc758 \ud55c\uc2e0 \ud0c0\uc774\uac70\uc2a4 \uc640\uc758 \uacbd\uae30\uc5d0\uc11c\ub294 5\uc774\ub2dd \ub3d9\uc548 106\uac1c\uc758 \uacf5\uc744 \ub358\uc838 8\ud53c\uc548\ud0c0 5\ubcfc\ub137 3\uc2e4\uc810\uc774\ub77c\ub294 \ucd5c\uc545\uc758 \ud22c\uad6c \ub0b4\uc6a9\uc73c\ub85c \uac1c\ub9c9\uc804 1\uad70 \uc5d4\ud2b8\ub9ac \ub4f1\ub85d\uc5d0\ub294 \uc2e4\ud328\ud588\ub2e4. \uadf8 \ud574\uc5d0\ub294 7\uacbd\uae30\uc5d0 \ub4f1\ud310\ud588\uc9c0\ub9cc \uccb4\ub825\uc758 \ud55c\uacc4 \ub4f1\uc744 \uc774\uc720\ub85c 10\uc6d4\uc5d0 \ud604\uc5ed \uc740\ud1f4\ub97c \uacb0\uc2ec\ud588\ub2e4. \uc57c\ub9c8\ub2e4\ub294 \uc911\uac04 \uacc4\ud22c\ub85c\uc11c\uc758 \uc7ac\uc0dd\uc744 \uc0dd\uac01\ud588\uace0 \ud0c0 \uad6c\ub2e8\uc73c\ub85c\ubd80\ud130\ub3c4 \uc601\uc785 \uc81c\uc758\ub97c \ubc1b\uc558\ub2e4\uace0 \ud55c\ub2e4. 11\uc6d4 10\uc77c\uc5d0 \uc740\ud1f4 \uae30\uc790\ud68c\uacac\uc744 \uac00\uc84c\ub2e4.", "sentence_answer": "\uadf8\ub7ec\ub098 \uc57c\uc218\ub4e4\uc758 \uc0c1\ud0dc\uac00 \ud638\uc804\ub428\uc5d0 \ub530\ub77c\uc11c \uc548\ud0c0\ub098 \ud648\ub7f0\uc744 \ub9de\ub294 \uc0c1\ud669\uc774 \uc18d\ucd9c\ud558\uc790, 3\uc6d4 23\uc77c \uad50\uc721 \ub9ac\uadf8\uc5d0\uc11c\uc758 \ud55c\uc2e0 \ud0c0\uc774\uac70\uc2a4 \uc640\uc758 \uacbd\uae30\uc5d0\uc11c\ub294 5\uc774\ub2dd \ub3d9\uc548 106\uac1c\uc758 \uacf5\uc744 \ub358\uc838 8\ud53c\uc548\ud0c0 5\ubcfc\ub137 3\uc2e4\uc810\uc774\ub77c\ub294 \ucd5c\uc545\uc758 \ud22c\uad6c \ub0b4\uc6a9\uc73c\ub85c \uac1c\ub9c9\uc804 1\uad70 \uc5d4\ud2b8\ub9ac \ub4f1\ub85d\uc5d0\ub294 \uc2e4\ud328\ud588\ub2e4.", "paragraph_id": "6545142-11-2", "paragraph_question": "question: 2001\ub144 3\uc6d4 23\uc77c \ucd9c\uc804\ud55c \uacbd\uae30\uc758 \uc0c1\ub300\ud300\uc740?, context: 2000\ub144, 11\ub144 \ub9cc\uc5d0 2\uad70 \uc2a4\ud504\ub9c1 \ucea0\ud504\uc5d0\uc11c \uc2dc\uc98c\uc744 \ub9de\uc774\ud558\uac8c \ub410\uace0 \ucc98\uc74c\uc73c\ub85c 1\uad70 \ub4f1\ud310\uc744 \ud558\uc9c0 \uc54a\uc740 \ucc44 \uc2dc\uc98c\uc744 \ub9c8\ucce4\ub2e4. \uc2dc\uc98c \uc885\ub8cc \ud6c4 \uacc4\uc57d \uac31\uc2e0\uc5d0\uc11c\ub294 \uc5f0\ubd09\uc774 \ud070 \ud3ed\uc73c\ub85c \uc0ad\uac10\ub3fc \uc774\ub4ec\ud574 2\uc6d4\uae4c\uc9c0 \uac78\ub838\uace0 \ud638\uc2dc\ub178 \uac10\ub3c5\uc5d0\uac8c\uc11c\ub294 \u2018\uacc4\uc57d\ud558\ub294 \uac83\ub9cc\uc73c\ub85c\ub3c4 \uac10\uc0ac\ud558\uac8c \uc0dd\uac01\ud560 \uac83\u2019\uc774\ub77c\ub294 \ucd5c\ud6c4 \ud1b5\uace0\ub97c \ubc1b\uc558\ub2e4. \ud55c\ud3b8 2001\ub144 1\uc6d4\uc5d0\ub294 \uc774\ub77c\uace0\uacf6\uc5d0\uc11c\uc758 \ud569\ub3d9 \uc790\uc8fc \ud2b8\ub808\uc774\ub2dd\uc5d0 \uc801\uadf9\uc801\uc73c\ub85c \ub7f0\ub2dd\uc5d0 \uc784\ud558\ub294 \uc790\uc138\ub97c \uc57c\ub9c8\ub2e4 \ud788\uc0ac\uc2dc \ud22c\uc218 \ucf54\uce58\ub85c\ubd80\ud130 \ud3c9\uac00\ub97c \ubc1b\uc544 2\ub144 \ub9cc\uc5d0 1\uad70 \uc2a4\ud504\ub9c1 \ucea0\ud504\uc5d0\uc11c\uc758 \uc2dc\uc98c\uc744 \ub9de\uc774\ud588\ub2e4. \uc2a4\ud504\ub9c1 \ucea0\ud504 \uc9c1\uc804\uc778 1\uc6d4 30\uc77c\uc5d0\ub294 \uc694\ubbf8\ud0c4 \uad6c\uc7a5\uc5d0\uc11c\uc758 \ubd88\ud39c\uc5d0\uc11c 100\uac1c\uc758 \uacf5\uc744 \ub358\uc9c0\uace0 \uc2a4\ud504\ub9c1 \ucea0\ud504 \uc911\uc5d0\ub3c4 \ud22c\uad6c \uc5f0\uc2b5\uc758 \ud22c\uad6c \uc218\ub97c \uc18c\ud654\ud574\ub0c8\uc9c0\ub9cc \uc81c\uad6c\uc5d0 \ubb38\uc81c\uac00 \uc788\uc5c8\ub2e4. \uc2dc\ubc94 \uacbd\uae30\uc5d0\uc11c\ub294 \uad6c\uc18d\uc774 \ucd5c\uace0 131 km/h\uc5d0 \uba38\ubb3c\ub800\uc9c0\ub9cc \ud3ec\ud06c\ubcfc\uc774\ub098 \uc288\ud1a0\ub97c \uad6c\uc0ac\ud574\uc11c 3\uc6d4 3\uc77c \uc9c0\ubc14 \ub86f\ub370\uc804\uc5d0\uc11c\ub294 4\uc774\ub2dd\uc744 1\uc2e4\uc810\uc73c\ub85c \ub9c9\uc558\ub2e4. \uadf8\ub7ec\ub098 \uc57c\uc218\ub4e4\uc758 \uc0c1\ud0dc\uac00 \ud638\uc804\ub428\uc5d0 \ub530\ub77c\uc11c \uc548\ud0c0\ub098 \ud648\ub7f0\uc744 \ub9de\ub294 \uc0c1\ud669\uc774 \uc18d\ucd9c\ud558\uc790, 3\uc6d4 23\uc77c \uad50\uc721 \ub9ac\uadf8\uc5d0\uc11c\uc758 \ud55c\uc2e0 \ud0c0\uc774\uac70\uc2a4\uc640\uc758 \uacbd\uae30\uc5d0\uc11c\ub294 5\uc774\ub2dd \ub3d9\uc548 106\uac1c\uc758 \uacf5\uc744 \ub358\uc838 8\ud53c\uc548\ud0c0 5\ubcfc\ub137 3\uc2e4\uc810\uc774\ub77c\ub294 \ucd5c\uc545\uc758 \ud22c\uad6c \ub0b4\uc6a9\uc73c\ub85c \uac1c\ub9c9\uc804 1\uad70 \uc5d4\ud2b8\ub9ac \ub4f1\ub85d\uc5d0\ub294 \uc2e4\ud328\ud588\ub2e4. \uadf8 \ud574\uc5d0\ub294 7\uacbd\uae30\uc5d0 \ub4f1\ud310\ud588\uc9c0\ub9cc \uccb4\ub825\uc758 \ud55c\uacc4 \ub4f1\uc744 \uc774\uc720\ub85c 10\uc6d4\uc5d0 \ud604\uc5ed \uc740\ud1f4\ub97c \uacb0\uc2ec\ud588\ub2e4. \uc57c\ub9c8\ub2e4\ub294 \uc911\uac04 \uacc4\ud22c\ub85c\uc11c\uc758 \uc7ac\uc0dd\uc744 \uc0dd\uac01\ud588\uace0 \ud0c0 \uad6c\ub2e8\uc73c\ub85c\ubd80\ud130\ub3c4 \uc601\uc785 \uc81c\uc758\ub97c \ubc1b\uc558\ub2e4\uace0 \ud55c\ub2e4. 11\uc6d4 10\uc77c\uc5d0 \uc740\ud1f4 \uae30\uc790\ud68c\uacac\uc744 \uac00\uc84c\ub2e4."} {"question": "\ubcc4\uc77c \uc5c6\uc774 \uc0b0\ub2e4\ub97c \ud2b9\uc774\ud55c \uc74c\ubc18\uc774\ub77c\uace0 \ud3c9\uac00\ud55c \ub300\uc911\ubb38\ud654 \ud3c9\ub860\uac00\uc758 \uc774\ub984\uc740?", "paragraph": "\ub300\uc911\ubb38\ud654 \ud3c9\ub860\uac00 \ucd5c\uaddc\uc131\uc740 \"\ucc98\uc74c\ubcf4\ub2e4\ub294 \ubc18\ubcf5\ud574 \ub4e4\uc744\uc218\ub85d \ub178\ub798\ub9c8\ub2e4 \ubb18\ud55c \uc911\ub3c5\uc131\uc744 \ubc1c\ud718\ud558\uba70 \uac10\uce60\ub9db\uc744 \ub354\ud558\ub294 \ud2b9\uc774\ud55c \uc74c\ubc18\"\uc774\ub77c\uace0 \ud3c9\uac00\ud588\ub2e4. \ub300\uc911\uc74c\uc545 \uc6f9\uc9c4 \u300a\ubcf4\ub2e4\u300b\ub294 \ub9ac\ubdf0\uc5d0\uc11c 10\uc810 \ub9cc\uc810\uc5d0 7\uc810\uc744 \uc8fc\uc5c8\uace0, \"\uadf8\ub4e4\uc740 \uc18d\uce6d \uc18c\ubab0\uc774 \ucc3d\ubc95\uc758 \uac10\uc815 \uacfc\uc789, \uadf8 \ucc9c\ud3b8\uc77c\ub960\uc774\ub098 \uc190\uc27d\uac8c \uc774\ud574\ud560 \uc218 \uc5c6\ub294 \uc608\uc220\uc131\uc744 \ubaa8\ub450 \ud53c\ud558\uace0 \ub204\uad6c\ub098 \uc27d\uac8c \ud765\uc5bc\uac70\ub838\ub358 \uac00\uc694\uc758 \ub300\uc911\uc801 \uc990\uac70\uc6c0\uc744 \uc7ac\ud604\ud558\ub294\ub370 \uc131\uacf5\"\ud588\ub2e4\uba70 \ud638\ud3c9\ud588\uc73c\ub098 \"\uc7a5\uae30\ud558\uc640 \uc5bc\uad74\ub4e4 1\uc9d1\uc758 \ub9e4\ub825\uc801\uc778 \ubcf5\uace0\ub294 \ucd08\ubc18\ubd80\uc758 \uba87 \uace1\uc73c\ub85c \uadf8\uce5c\ub2e4. \uac8c\ub2e4\uac00 \uc55e\uc11c \uc5b8\uae09\ud588\ub358 \uace1\ub4e4 \uc5ed\uc2dc \uc7a5\uae30\ud558\uc640 \uc5bc\uad74\ub4e4\uc758 \uace0\uc720\ud55c \uc9c8\uac10\uc774\ub77c\uae30\ubcf4\ub2e4\ub294 \uc774\ubbf8 \uc120\ubc30 \ubba4\uc9c0\uc158\ub4e4\uc774 \ucc3d\uc870\ud55c \uc0ac\uc6b4\ub4dc\uc758 \ud754\uc801\uc744 \uac15\ud558\uac8c \ud48d\uae40\uc73c\ub85c\uc368 \uc644\ubcbd\ud55c \uc624\ub9ac\uc9c0\ub110\ub9ac\ud2f0\ub97c \ud68d\ub4dd\ud558\uc9c0\ub294 \ubabb\ud55c\ub2e4.\"\uace0 \uc9c0\uc801\ud588\ub2e4.", "answer": "\ucd5c\uaddc\uc131", "sentence": "\ub300\uc911\ubb38\ud654 \ud3c9\ub860\uac00 \ucd5c\uaddc\uc131 \uc740 \"\ucc98\uc74c\ubcf4\ub2e4\ub294 \ubc18\ubcf5\ud574 \ub4e4\uc744\uc218\ub85d \ub178\ub798\ub9c8\ub2e4 \ubb18\ud55c \uc911\ub3c5\uc131\uc744 \ubc1c\ud718\ud558\uba70 \uac10\uce60\ub9db\uc744 \ub354\ud558\ub294 \ud2b9\uc774\ud55c \uc74c\ubc18\"\uc774\ub77c\uace0 \ud3c9\uac00\ud588\ub2e4.", "paragraph_sentence": " \ub300\uc911\ubb38\ud654 \ud3c9\ub860\uac00 \ucd5c\uaddc\uc131 \uc740 \"\ucc98\uc74c\ubcf4\ub2e4\ub294 \ubc18\ubcf5\ud574 \ub4e4\uc744\uc218\ub85d \ub178\ub798\ub9c8\ub2e4 \ubb18\ud55c \uc911\ub3c5\uc131\uc744 \ubc1c\ud718\ud558\uba70 \uac10\uce60\ub9db\uc744 \ub354\ud558\ub294 \ud2b9\uc774\ud55c \uc74c\ubc18\"\uc774\ub77c\uace0 \ud3c9\uac00\ud588\ub2e4. \ub300\uc911\uc74c\uc545 \uc6f9\uc9c4 \u300a\ubcf4\ub2e4\u300b\ub294 \ub9ac\ubdf0\uc5d0\uc11c 10\uc810 \ub9cc\uc810\uc5d0 7\uc810\uc744 \uc8fc\uc5c8\uace0, \"\uadf8\ub4e4\uc740 \uc18d\uce6d \uc18c\ubab0\uc774 \ucc3d\ubc95\uc758 \uac10\uc815 \uacfc\uc789, \uadf8 \ucc9c\ud3b8\uc77c\ub960\uc774\ub098 \uc190\uc27d\uac8c \uc774\ud574\ud560 \uc218 \uc5c6\ub294 \uc608\uc220\uc131\uc744 \ubaa8\ub450 \ud53c\ud558\uace0 \ub204\uad6c\ub098 \uc27d\uac8c \ud765\uc5bc\uac70\ub838\ub358 \uac00\uc694\uc758 \ub300\uc911\uc801 \uc990\uac70\uc6c0\uc744 \uc7ac\ud604\ud558\ub294\ub370 \uc131\uacf5\"\ud588\ub2e4\uba70 \ud638\ud3c9\ud588\uc73c\ub098 \"\uc7a5\uae30\ud558\uc640 \uc5bc\uad74\ub4e4 1\uc9d1\uc758 \ub9e4\ub825\uc801\uc778 \ubcf5\uace0\ub294 \ucd08\ubc18\ubd80\uc758 \uba87 \uace1\uc73c\ub85c \uadf8\uce5c\ub2e4. \uac8c\ub2e4\uac00 \uc55e\uc11c \uc5b8\uae09\ud588\ub358 \uace1\ub4e4 \uc5ed\uc2dc \uc7a5\uae30\ud558\uc640 \uc5bc\uad74\ub4e4\uc758 \uace0\uc720\ud55c \uc9c8\uac10\uc774\ub77c\uae30\ubcf4\ub2e4\ub294 \uc774\ubbf8 \uc120\ubc30 \ubba4\uc9c0\uc158\ub4e4\uc774 \ucc3d\uc870\ud55c \uc0ac\uc6b4\ub4dc\uc758 \ud754\uc801\uc744 \uac15\ud558\uac8c \ud48d\uae40\uc73c\ub85c\uc368 \uc644\ubcbd\ud55c \uc624\ub9ac\uc9c0\ub110\ub9ac\ud2f0\ub97c \ud68d\ub4dd\ud558\uc9c0\ub294 \ubabb\ud55c\ub2e4. \"\uace0 \uc9c0\uc801\ud588\ub2e4.", "paragraph_answer": "\ub300\uc911\ubb38\ud654 \ud3c9\ub860\uac00 \ucd5c\uaddc\uc131 \uc740 \"\ucc98\uc74c\ubcf4\ub2e4\ub294 \ubc18\ubcf5\ud574 \ub4e4\uc744\uc218\ub85d \ub178\ub798\ub9c8\ub2e4 \ubb18\ud55c \uc911\ub3c5\uc131\uc744 \ubc1c\ud718\ud558\uba70 \uac10\uce60\ub9db\uc744 \ub354\ud558\ub294 \ud2b9\uc774\ud55c \uc74c\ubc18\"\uc774\ub77c\uace0 \ud3c9\uac00\ud588\ub2e4. \ub300\uc911\uc74c\uc545 \uc6f9\uc9c4 \u300a\ubcf4\ub2e4\u300b\ub294 \ub9ac\ubdf0\uc5d0\uc11c 10\uc810 \ub9cc\uc810\uc5d0 7\uc810\uc744 \uc8fc\uc5c8\uace0, \"\uadf8\ub4e4\uc740 \uc18d\uce6d \uc18c\ubab0\uc774 \ucc3d\ubc95\uc758 \uac10\uc815 \uacfc\uc789, \uadf8 \ucc9c\ud3b8\uc77c\ub960\uc774\ub098 \uc190\uc27d\uac8c \uc774\ud574\ud560 \uc218 \uc5c6\ub294 \uc608\uc220\uc131\uc744 \ubaa8\ub450 \ud53c\ud558\uace0 \ub204\uad6c\ub098 \uc27d\uac8c \ud765\uc5bc\uac70\ub838\ub358 \uac00\uc694\uc758 \ub300\uc911\uc801 \uc990\uac70\uc6c0\uc744 \uc7ac\ud604\ud558\ub294\ub370 \uc131\uacf5\"\ud588\ub2e4\uba70 \ud638\ud3c9\ud588\uc73c\ub098 \"\uc7a5\uae30\ud558\uc640 \uc5bc\uad74\ub4e4 1\uc9d1\uc758 \ub9e4\ub825\uc801\uc778 \ubcf5\uace0\ub294 \ucd08\ubc18\ubd80\uc758 \uba87 \uace1\uc73c\ub85c \uadf8\uce5c\ub2e4. \uac8c\ub2e4\uac00 \uc55e\uc11c \uc5b8\uae09\ud588\ub358 \uace1\ub4e4 \uc5ed\uc2dc \uc7a5\uae30\ud558\uc640 \uc5bc\uad74\ub4e4\uc758 \uace0\uc720\ud55c \uc9c8\uac10\uc774\ub77c\uae30\ubcf4\ub2e4\ub294 \uc774\ubbf8 \uc120\ubc30 \ubba4\uc9c0\uc158\ub4e4\uc774 \ucc3d\uc870\ud55c \uc0ac\uc6b4\ub4dc\uc758 \ud754\uc801\uc744 \uac15\ud558\uac8c \ud48d\uae40\uc73c\ub85c\uc368 \uc644\ubcbd\ud55c \uc624\ub9ac\uc9c0\ub110\ub9ac\ud2f0\ub97c \ud68d\ub4dd\ud558\uc9c0\ub294 \ubabb\ud55c\ub2e4.\"\uace0 \uc9c0\uc801\ud588\ub2e4.", "sentence_answer": "\ub300\uc911\ubb38\ud654 \ud3c9\ub860\uac00 \ucd5c\uaddc\uc131 \uc740 \"\ucc98\uc74c\ubcf4\ub2e4\ub294 \ubc18\ubcf5\ud574 \ub4e4\uc744\uc218\ub85d \ub178\ub798\ub9c8\ub2e4 \ubb18\ud55c \uc911\ub3c5\uc131\uc744 \ubc1c\ud718\ud558\uba70 \uac10\uce60\ub9db\uc744 \ub354\ud558\ub294 \ud2b9\uc774\ud55c \uc74c\ubc18\"\uc774\ub77c\uace0 \ud3c9\uac00\ud588\ub2e4.", "paragraph_id": "6657153-12-1", "paragraph_question": "question: \ubcc4\uc77c \uc5c6\uc774 \uc0b0\ub2e4\ub97c \ud2b9\uc774\ud55c \uc74c\ubc18\uc774\ub77c\uace0 \ud3c9\uac00\ud55c \ub300\uc911\ubb38\ud654 \ud3c9\ub860\uac00\uc758 \uc774\ub984\uc740?, context: \ub300\uc911\ubb38\ud654 \ud3c9\ub860\uac00 \ucd5c\uaddc\uc131\uc740 \"\ucc98\uc74c\ubcf4\ub2e4\ub294 \ubc18\ubcf5\ud574 \ub4e4\uc744\uc218\ub85d \ub178\ub798\ub9c8\ub2e4 \ubb18\ud55c \uc911\ub3c5\uc131\uc744 \ubc1c\ud718\ud558\uba70 \uac10\uce60\ub9db\uc744 \ub354\ud558\ub294 \ud2b9\uc774\ud55c \uc74c\ubc18\"\uc774\ub77c\uace0 \ud3c9\uac00\ud588\ub2e4. \ub300\uc911\uc74c\uc545 \uc6f9\uc9c4 \u300a\ubcf4\ub2e4\u300b\ub294 \ub9ac\ubdf0\uc5d0\uc11c 10\uc810 \ub9cc\uc810\uc5d0 7\uc810\uc744 \uc8fc\uc5c8\uace0, \"\uadf8\ub4e4\uc740 \uc18d\uce6d \uc18c\ubab0\uc774 \ucc3d\ubc95\uc758 \uac10\uc815 \uacfc\uc789, \uadf8 \ucc9c\ud3b8\uc77c\ub960\uc774\ub098 \uc190\uc27d\uac8c \uc774\ud574\ud560 \uc218 \uc5c6\ub294 \uc608\uc220\uc131\uc744 \ubaa8\ub450 \ud53c\ud558\uace0 \ub204\uad6c\ub098 \uc27d\uac8c \ud765\uc5bc\uac70\ub838\ub358 \uac00\uc694\uc758 \ub300\uc911\uc801 \uc990\uac70\uc6c0\uc744 \uc7ac\ud604\ud558\ub294\ub370 \uc131\uacf5\"\ud588\ub2e4\uba70 \ud638\ud3c9\ud588\uc73c\ub098 \"\uc7a5\uae30\ud558\uc640 \uc5bc\uad74\ub4e4 1\uc9d1\uc758 \ub9e4\ub825\uc801\uc778 \ubcf5\uace0\ub294 \ucd08\ubc18\ubd80\uc758 \uba87 \uace1\uc73c\ub85c \uadf8\uce5c\ub2e4. \uac8c\ub2e4\uac00 \uc55e\uc11c \uc5b8\uae09\ud588\ub358 \uace1\ub4e4 \uc5ed\uc2dc \uc7a5\uae30\ud558\uc640 \uc5bc\uad74\ub4e4\uc758 \uace0\uc720\ud55c \uc9c8\uac10\uc774\ub77c\uae30\ubcf4\ub2e4\ub294 \uc774\ubbf8 \uc120\ubc30 \ubba4\uc9c0\uc158\ub4e4\uc774 \ucc3d\uc870\ud55c \uc0ac\uc6b4\ub4dc\uc758 \ud754\uc801\uc744 \uac15\ud558\uac8c \ud48d\uae40\uc73c\ub85c\uc368 \uc644\ubcbd\ud55c \uc624\ub9ac\uc9c0\ub110\ub9ac\ud2f0\ub97c \ud68d\ub4dd\ud558\uc9c0\ub294 \ubabb\ud55c\ub2e4.\"\uace0 \uc9c0\uc801\ud588\ub2e4."} {"question": "\ubbf8\uad70\uc815\uccad \uccb4\uc721\ubd80\uc7a5\uacfc \uc870\uc120\uccb4\uc721\ud68c \ud68c\uc7a5\uc744 \uacb8\uc9c1\ud558\uace0 \uc788\ub358 \uc0ac\ub78c\uc740 \ub204\uad6c\uc600\ub098?", "paragraph": "1947\ub144 7\uc6d4 19\uc77c IOC\uac00\uc785 \ucd95\ud558\uae30\ub150\uc73c\ub85c \ud55c\uad6d\uacfc \uc601\uad6d\uc758 \uce5c\uc120\ucd95\uad6c\uacbd\uae30\uac00 \uc11c\uc6b8\uc6b4\ub3d9\uc7a5(\ud604\uc7ac \ub3d9\ub300\ubb38 \uc6b4\ub3d9\uc7a5)\uc5d0\uc11c \uc5f4\ub9b0 \ub0a0, \ub2f9\uc2dc \ubbf8\uad70\uc815\uccad \uc870\uc120 \uccb4\uc721\ubd80\uc7a5\uc774\uc790 \uc870\uc120\uccb4\uc721\ud68c(\ub300\ud55c\uccb4\uc721\ud68c\uc758 \uc804\uc2e0) \ud68c\uc7a5\uc73c\ub85c \uc88c\uc6b0\ud569\uc791\uc704\uc6d0\ud68c\ub97c \uc774\ub04c\uc5c8\ub358 \uc5ec\uc6b4\ud615\uc740 \uacbd\uae30 \ucc38\uad00\uc744 \ud558\uae30 \uc804\uc5d0 \uc637\uc744 \uac08\uc544\uc785\uaca0\ub2e4\uace0 \ucc28\ub97c \ud0c0\uace0 \uc9d1\uc73c\ub85c \uac00\ub358 \uc911, \ud61c\ud654\ub3d9 \ub85c\ud130\ub9ac \uadfc\ubc29\uc5d0\uc11c \ubc30\ud6c4\ub97c \uc54c \uc218\uc5c6\ub294 \ud55c\uc9c0\uadfc(\u97d3\u667a\u6839, \ubcf8\uba85\uc740 \uc774\ud544\ud615(\u674e\u5f3c\u70af))\uc5d0 \uc758\ud574 \ud749\ud0c4\uc5d0 \ub9de\uc544 \ud53c\uc2b5 \uc808\uba85\ud558\uc600\ub2e4. \uc5ec\uc6b4\ud615\uc774 \uc8fd\uae30 \uba87 \uc2dc\uac04 \uc804\uc5d0 \ub610\ud55c \uae40\uaddc\uc2dd\ub3c4 '\uc8fd\uc774\uaca0\ub2e4'\ub294 \ud611\ubc15\ud3b8\uc9c0\ub97c \ubc1b\uc558\ub2e4\uace0 \ud55c\ub2e4. \uae40\uaddc\uc2dd\uc740 \uc5ec\uc6b4\ud615\uacfc \uc0ac\uc801\uc73c\ub85c \ud615\ub2d8\u00b7\uc544\uc6b0\ub2d8 \ud558\ub358 \uc0ac\uc774\uc600\uace0, \uc624\ub798\uc804 \uc0c1\ud574 \uc784\uc2dc\uc815\ubd80 \uc2dc\uc808\ubd80\ud130 \ud574\ubc29\uc774\ud6c4 \uc88c\uc6b0\ud569\uc791\uc6b4\ub3d9\uae4c\uc9c0 \ud568\uaed8\ud574\uc628 \ub3d9\uc9c0\uc600\uae30\uc5d0 \uc5ec\uc6b4\ud615\uc5d0 \ub300\ud55c \uadf8\uc758 \ud2b9\ubcc4\ud55c \ub3d9\uc9c0\uc560\ub294 1947\ub144 12\uc6d4, \uc88c\u00b7\uc6b0\ud569\uc791\uc774 \uc2e4\ud328\ub85c \ub05d\ub09c \ub4a4\uc5d0 \ucd2c\uc601\ud55c \uae30\ub150\uc0ac\uc9c4\uc5d0 \uc190\uc218 \uc5ec\uc6b4\ud615\uc758 \uc5bc\uad74\uc774 \ub4e0 \uc0ac\uc9c4\uc744 \uc624\ub824\uc11c \ubd99\uc774\ub294 \uac83\uc73c\ub85c\ub3c4 \ud45c\ud604\ub418\uc5c8\ub2e4.", "answer": "\uc5ec\uc6b4\ud615", "sentence": "1947\ub144 7\uc6d4 19\uc77c IOC\uac00\uc785 \ucd95\ud558\uae30\ub150\uc73c\ub85c \ud55c\uad6d\uacfc \uc601\uad6d\uc758 \uce5c\uc120\ucd95\uad6c\uacbd\uae30\uac00 \uc11c\uc6b8\uc6b4\ub3d9\uc7a5(\ud604\uc7ac \ub3d9\ub300\ubb38 \uc6b4\ub3d9\uc7a5)\uc5d0\uc11c \uc5f4\ub9b0 \ub0a0, \ub2f9\uc2dc \ubbf8\uad70\uc815\uccad \uc870\uc120 \uccb4\uc721\ubd80\uc7a5\uc774\uc790 \uc870\uc120\uccb4\uc721\ud68c(\ub300\ud55c\uccb4\uc721\ud68c\uc758 \uc804\uc2e0) \ud68c\uc7a5\uc73c\ub85c \uc88c\uc6b0\ud569\uc791\uc704\uc6d0\ud68c\ub97c \uc774\ub04c\uc5c8\ub358 \uc5ec\uc6b4\ud615 \uc740 \uacbd\uae30 \ucc38\uad00\uc744 \ud558\uae30 \uc804\uc5d0 \uc637\uc744 \uac08\uc544\uc785\uaca0\ub2e4\uace0 \ucc28\ub97c \ud0c0\uace0 \uc9d1\uc73c\ub85c \uac00\ub358 \uc911, \ud61c\ud654\ub3d9 \ub85c\ud130\ub9ac \uadfc\ubc29\uc5d0\uc11c \ubc30\ud6c4\ub97c \uc54c \uc218\uc5c6\ub294 \ud55c\uc9c0\uadfc(\u97d3\u667a\u6839, \ubcf8\uba85\uc740 \uc774\ud544\ud615(\u674e\u5f3c\u70af))\uc5d0 \uc758\ud574 \ud749\ud0c4\uc5d0 \ub9de\uc544 \ud53c\uc2b5 \uc808\uba85\ud558\uc600\ub2e4.", "paragraph_sentence": " 1947\ub144 7\uc6d4 19\uc77c IOC\uac00\uc785 \ucd95\ud558\uae30\ub150\uc73c\ub85c \ud55c\uad6d\uacfc \uc601\uad6d\uc758 \uce5c\uc120\ucd95\uad6c\uacbd\uae30\uac00 \uc11c\uc6b8\uc6b4\ub3d9\uc7a5(\ud604\uc7ac \ub3d9\ub300\ubb38 \uc6b4\ub3d9\uc7a5)\uc5d0\uc11c \uc5f4\ub9b0 \ub0a0, \ub2f9\uc2dc \ubbf8\uad70\uc815\uccad \uc870\uc120 \uccb4\uc721\ubd80\uc7a5\uc774\uc790 \uc870\uc120\uccb4\uc721\ud68c(\ub300\ud55c\uccb4\uc721\ud68c\uc758 \uc804\uc2e0) \ud68c\uc7a5\uc73c\ub85c \uc88c\uc6b0\ud569\uc791\uc704\uc6d0\ud68c\ub97c \uc774\ub04c\uc5c8\ub358 \uc5ec\uc6b4\ud615 \uc740 \uacbd\uae30 \ucc38\uad00\uc744 \ud558\uae30 \uc804\uc5d0 \uc637\uc744 \uac08\uc544\uc785\uaca0\ub2e4\uace0 \ucc28\ub97c \ud0c0\uace0 \uc9d1\uc73c\ub85c \uac00\ub358 \uc911, \ud61c\ud654\ub3d9 \ub85c\ud130\ub9ac \uadfc\ubc29\uc5d0\uc11c \ubc30\ud6c4\ub97c \uc54c \uc218\uc5c6\ub294 \ud55c\uc9c0\uadfc(\u97d3\u667a\u6839, \ubcf8\uba85\uc740 \uc774\ud544\ud615(\u674e\u5f3c\u70af))\uc5d0 \uc758\ud574 \ud749\ud0c4\uc5d0 \ub9de\uc544 \ud53c\uc2b5 \uc808\uba85\ud558\uc600\ub2e4. \uc5ec\uc6b4\ud615\uc774 \uc8fd\uae30 \uba87 \uc2dc\uac04 \uc804\uc5d0 \ub610\ud55c \uae40\uaddc\uc2dd\ub3c4 '\uc8fd\uc774\uaca0\ub2e4'\ub294 \ud611\ubc15\ud3b8\uc9c0\ub97c \ubc1b\uc558\ub2e4\uace0 \ud55c\ub2e4. \uae40\uaddc\uc2dd\uc740 \uc5ec\uc6b4\ud615\uacfc \uc0ac\uc801\uc73c\ub85c \ud615\ub2d8\u00b7\uc544\uc6b0\ub2d8 \ud558\ub358 \uc0ac\uc774\uc600\uace0, \uc624\ub798\uc804 \uc0c1\ud574 \uc784\uc2dc\uc815\ubd80 \uc2dc\uc808\ubd80\ud130 \ud574\ubc29\uc774\ud6c4 \uc88c\uc6b0\ud569\uc791\uc6b4\ub3d9\uae4c\uc9c0 \ud568\uaed8\ud574\uc628 \ub3d9\uc9c0\uc600\uae30\uc5d0 \uc5ec\uc6b4\ud615\uc5d0 \ub300\ud55c \uadf8\uc758 \ud2b9\ubcc4\ud55c \ub3d9\uc9c0\uc560\ub294 1947\ub144 12\uc6d4, \uc88c\u00b7\uc6b0\ud569\uc791\uc774 \uc2e4\ud328\ub85c \ub05d\ub09c \ub4a4\uc5d0 \ucd2c\uc601\ud55c \uae30\ub150\uc0ac\uc9c4\uc5d0 \uc190\uc218 \uc5ec\uc6b4\ud615\uc758 \uc5bc\uad74\uc774 \ub4e0 \uc0ac\uc9c4\uc744 \uc624\ub824\uc11c \ubd99\uc774\ub294 \uac83\uc73c\ub85c\ub3c4 \ud45c\ud604\ub418\uc5c8\ub2e4.", "paragraph_answer": "1947\ub144 7\uc6d4 19\uc77c IOC\uac00\uc785 \ucd95\ud558\uae30\ub150\uc73c\ub85c \ud55c\uad6d\uacfc \uc601\uad6d\uc758 \uce5c\uc120\ucd95\uad6c\uacbd\uae30\uac00 \uc11c\uc6b8\uc6b4\ub3d9\uc7a5(\ud604\uc7ac \ub3d9\ub300\ubb38 \uc6b4\ub3d9\uc7a5)\uc5d0\uc11c \uc5f4\ub9b0 \ub0a0, \ub2f9\uc2dc \ubbf8\uad70\uc815\uccad \uc870\uc120 \uccb4\uc721\ubd80\uc7a5\uc774\uc790 \uc870\uc120\uccb4\uc721\ud68c(\ub300\ud55c\uccb4\uc721\ud68c\uc758 \uc804\uc2e0) \ud68c\uc7a5\uc73c\ub85c \uc88c\uc6b0\ud569\uc791\uc704\uc6d0\ud68c\ub97c \uc774\ub04c\uc5c8\ub358 \uc5ec\uc6b4\ud615 \uc740 \uacbd\uae30 \ucc38\uad00\uc744 \ud558\uae30 \uc804\uc5d0 \uc637\uc744 \uac08\uc544\uc785\uaca0\ub2e4\uace0 \ucc28\ub97c \ud0c0\uace0 \uc9d1\uc73c\ub85c \uac00\ub358 \uc911, \ud61c\ud654\ub3d9 \ub85c\ud130\ub9ac \uadfc\ubc29\uc5d0\uc11c \ubc30\ud6c4\ub97c \uc54c \uc218\uc5c6\ub294 \ud55c\uc9c0\uadfc(\u97d3\u667a\u6839, \ubcf8\uba85\uc740 \uc774\ud544\ud615(\u674e\u5f3c\u70af))\uc5d0 \uc758\ud574 \ud749\ud0c4\uc5d0 \ub9de\uc544 \ud53c\uc2b5 \uc808\uba85\ud558\uc600\ub2e4. \uc5ec\uc6b4\ud615\uc774 \uc8fd\uae30 \uba87 \uc2dc\uac04 \uc804\uc5d0 \ub610\ud55c \uae40\uaddc\uc2dd\ub3c4 '\uc8fd\uc774\uaca0\ub2e4'\ub294 \ud611\ubc15\ud3b8\uc9c0\ub97c \ubc1b\uc558\ub2e4\uace0 \ud55c\ub2e4. \uae40\uaddc\uc2dd\uc740 \uc5ec\uc6b4\ud615\uacfc \uc0ac\uc801\uc73c\ub85c \ud615\ub2d8\u00b7\uc544\uc6b0\ub2d8 \ud558\ub358 \uc0ac\uc774\uc600\uace0, \uc624\ub798\uc804 \uc0c1\ud574 \uc784\uc2dc\uc815\ubd80 \uc2dc\uc808\ubd80\ud130 \ud574\ubc29\uc774\ud6c4 \uc88c\uc6b0\ud569\uc791\uc6b4\ub3d9\uae4c\uc9c0 \ud568\uaed8\ud574\uc628 \ub3d9\uc9c0\uc600\uae30\uc5d0 \uc5ec\uc6b4\ud615\uc5d0 \ub300\ud55c \uadf8\uc758 \ud2b9\ubcc4\ud55c \ub3d9\uc9c0\uc560\ub294 1947\ub144 12\uc6d4, \uc88c\u00b7\uc6b0\ud569\uc791\uc774 \uc2e4\ud328\ub85c \ub05d\ub09c \ub4a4\uc5d0 \ucd2c\uc601\ud55c \uae30\ub150\uc0ac\uc9c4\uc5d0 \uc190\uc218 \uc5ec\uc6b4\ud615\uc758 \uc5bc\uad74\uc774 \ub4e0 \uc0ac\uc9c4\uc744 \uc624\ub824\uc11c \ubd99\uc774\ub294 \uac83\uc73c\ub85c\ub3c4 \ud45c\ud604\ub418\uc5c8\ub2e4.", "sentence_answer": "1947\ub144 7\uc6d4 19\uc77c IOC\uac00\uc785 \ucd95\ud558\uae30\ub150\uc73c\ub85c \ud55c\uad6d\uacfc \uc601\uad6d\uc758 \uce5c\uc120\ucd95\uad6c\uacbd\uae30\uac00 \uc11c\uc6b8\uc6b4\ub3d9\uc7a5(\ud604\uc7ac \ub3d9\ub300\ubb38 \uc6b4\ub3d9\uc7a5)\uc5d0\uc11c \uc5f4\ub9b0 \ub0a0, \ub2f9\uc2dc \ubbf8\uad70\uc815\uccad \uc870\uc120 \uccb4\uc721\ubd80\uc7a5\uc774\uc790 \uc870\uc120\uccb4\uc721\ud68c(\ub300\ud55c\uccb4\uc721\ud68c\uc758 \uc804\uc2e0) \ud68c\uc7a5\uc73c\ub85c \uc88c\uc6b0\ud569\uc791\uc704\uc6d0\ud68c\ub97c \uc774\ub04c\uc5c8\ub358 \uc5ec\uc6b4\ud615 \uc740 \uacbd\uae30 \ucc38\uad00\uc744 \ud558\uae30 \uc804\uc5d0 \uc637\uc744 \uac08\uc544\uc785\uaca0\ub2e4\uace0 \ucc28\ub97c \ud0c0\uace0 \uc9d1\uc73c\ub85c \uac00\ub358 \uc911, \ud61c\ud654\ub3d9 \ub85c\ud130\ub9ac \uadfc\ubc29\uc5d0\uc11c \ubc30\ud6c4\ub97c \uc54c \uc218\uc5c6\ub294 \ud55c\uc9c0\uadfc(\u97d3\u667a\u6839, \ubcf8\uba85\uc740 \uc774\ud544\ud615(\u674e\u5f3c\u70af))\uc5d0 \uc758\ud574 \ud749\ud0c4\uc5d0 \ub9de\uc544 \ud53c\uc2b5 \uc808\uba85\ud558\uc600\ub2e4.", "paragraph_id": "5991262-36-2", "paragraph_question": "question: \ubbf8\uad70\uc815\uccad \uccb4\uc721\ubd80\uc7a5\uacfc \uc870\uc120\uccb4\uc721\ud68c \ud68c\uc7a5\uc744 \uacb8\uc9c1\ud558\uace0 \uc788\ub358 \uc0ac\ub78c\uc740 \ub204\uad6c\uc600\ub098?, context: 1947\ub144 7\uc6d4 19\uc77c IOC\uac00\uc785 \ucd95\ud558\uae30\ub150\uc73c\ub85c \ud55c\uad6d\uacfc \uc601\uad6d\uc758 \uce5c\uc120\ucd95\uad6c\uacbd\uae30\uac00 \uc11c\uc6b8\uc6b4\ub3d9\uc7a5(\ud604\uc7ac \ub3d9\ub300\ubb38 \uc6b4\ub3d9\uc7a5)\uc5d0\uc11c \uc5f4\ub9b0 \ub0a0, \ub2f9\uc2dc \ubbf8\uad70\uc815\uccad \uc870\uc120 \uccb4\uc721\ubd80\uc7a5\uc774\uc790 \uc870\uc120\uccb4\uc721\ud68c(\ub300\ud55c\uccb4\uc721\ud68c\uc758 \uc804\uc2e0) \ud68c\uc7a5\uc73c\ub85c \uc88c\uc6b0\ud569\uc791\uc704\uc6d0\ud68c\ub97c \uc774\ub04c\uc5c8\ub358 \uc5ec\uc6b4\ud615\uc740 \uacbd\uae30 \ucc38\uad00\uc744 \ud558\uae30 \uc804\uc5d0 \uc637\uc744 \uac08\uc544\uc785\uaca0\ub2e4\uace0 \ucc28\ub97c \ud0c0\uace0 \uc9d1\uc73c\ub85c \uac00\ub358 \uc911, \ud61c\ud654\ub3d9 \ub85c\ud130\ub9ac \uadfc\ubc29\uc5d0\uc11c \ubc30\ud6c4\ub97c \uc54c \uc218\uc5c6\ub294 \ud55c\uc9c0\uadfc(\u97d3\u667a\u6839, \ubcf8\uba85\uc740 \uc774\ud544\ud615(\u674e\u5f3c\u70af))\uc5d0 \uc758\ud574 \ud749\ud0c4\uc5d0 \ub9de\uc544 \ud53c\uc2b5 \uc808\uba85\ud558\uc600\ub2e4. \uc5ec\uc6b4\ud615\uc774 \uc8fd\uae30 \uba87 \uc2dc\uac04 \uc804\uc5d0 \ub610\ud55c \uae40\uaddc\uc2dd\ub3c4 '\uc8fd\uc774\uaca0\ub2e4'\ub294 \ud611\ubc15\ud3b8\uc9c0\ub97c \ubc1b\uc558\ub2e4\uace0 \ud55c\ub2e4. \uae40\uaddc\uc2dd\uc740 \uc5ec\uc6b4\ud615\uacfc \uc0ac\uc801\uc73c\ub85c \ud615\ub2d8\u00b7\uc544\uc6b0\ub2d8 \ud558\ub358 \uc0ac\uc774\uc600\uace0, \uc624\ub798\uc804 \uc0c1\ud574 \uc784\uc2dc\uc815\ubd80 \uc2dc\uc808\ubd80\ud130 \ud574\ubc29\uc774\ud6c4 \uc88c\uc6b0\ud569\uc791\uc6b4\ub3d9\uae4c\uc9c0 \ud568\uaed8\ud574\uc628 \ub3d9\uc9c0\uc600\uae30\uc5d0 \uc5ec\uc6b4\ud615\uc5d0 \ub300\ud55c \uadf8\uc758 \ud2b9\ubcc4\ud55c \ub3d9\uc9c0\uc560\ub294 1947\ub144 12\uc6d4, \uc88c\u00b7\uc6b0\ud569\uc791\uc774 \uc2e4\ud328\ub85c \ub05d\ub09c \ub4a4\uc5d0 \ucd2c\uc601\ud55c \uae30\ub150\uc0ac\uc9c4\uc5d0 \uc190\uc218 \uc5ec\uc6b4\ud615\uc758 \uc5bc\uad74\uc774 \ub4e0 \uc0ac\uc9c4\uc744 \uc624\ub824\uc11c \ubd99\uc774\ub294 \uac83\uc73c\ub85c\ub3c4 \ud45c\ud604\ub418\uc5c8\ub2e4."} {"question": "\uc601\ud654 \ud504\ub80c\uc988 \uc704\ub4dc \ubca0\ub124\ud54f\uc740 \uba87 \ub144\ub3c4\uc5d0 \uacf5\uac1c\ub418\uc5c8\ub294\uac00?", "paragraph": "2011\ub144\uc5d0\ub294 \uc70c \uae00\ub7ed \uac10\ub3c5\uc758 \u300a\ud504\ub80c\uc988 \uc704\ub4dc \ubca0\ub124\ud54f\u300b\uc5d0\uc11c \uc800\uc2a4\ud2f4 \ud300\ubc84\ub808\uc774\ud06c\uc640 \ubc00\ub77c \ucfe0\ub2c8\uc2a4\uc640 \ud568\uaed8 \uc5f0\uae30\ud558\uc600\ub2e4. \uc601\ud654\ub294 2010\ub144 7\uc6d4 \ub274\uc695\uacfc \ub85c\uc2a4\uc564\uc824\ub808\uc2a4\uc5d0\uc11c \ucd2c\uc601\uc774 \uc2dc\uc791\ub418\uc5b4 2011\ub144 7\uc6d4 22\uc77c\uc5d0 \uacf5\uac1c\ub418\uc5c8\ub2e4. \uac19\uc740 \ud574, \uc2a4\ud2f0\ube0c \ucee4\ub810, \ub77c\uc774\uc5b8 \uace0\uc2ac\ub9c1\uacfc \ud568\uaed8 \ucd9c\uc5f0\ud55c \u300a\ud06c\ub808\uc774\uc9c0, \uc2a4\ud22c\ud53c\ub4dc, \ub7ec\ube0c\u300b\uac00 \uacf5\uac1c\ub418\uc5c8\ub2e4. \uc601\ud654\ub294 \uc88b\uc740 \ud3c9\uc744 \ubc1b\uc558\uace0, TV \uac00\uc774\ub4dc\uac00 \uc120\uc815\ud55c 2011\ub144 \ucd5c\uace0\uc758 \uc601\ud654 \ubaa9\ub85d\uc5d0 \uc774\ub984\uc744 \uc62c\ub838\ub2e4. \ud765\ud589\ub3c4 \uc131\uacf5\ud55c \ud3b8\uc774\uba70, \uc2a4\ud1a4 \ub610\ud55c \uc5ec\ub7ec \uc2dc\uc0c1\uc2dd\uc5d0 \ud6c4\ubcf4\ub85c \uc624\ub974\uac70\ub098 \uc218\uc0c1\ud558\uc600\ub2e4. \ub610\ud55c, \uce90\uc11c\ub9b0 \uc2a4\ud1a0\ud0b7\uc758 \ub3d9\uba85\uc758 \uc18c\uc124\uc744 \uc6d0\uc791\uc73c\ub85c \ud55c \u300a\ud5ec\ud504\u300b\uc5d0 \ucd9c\uc5f0\ud558\uc600\ub2e4. \uc601\ud654\ub294 1960\ub144\ub300 \ubbf8\uc2dc\uc2dc\ud53c \uc8fc \uc7ad\uc2a8\uc744 \ubb34\ub300\ub85c \ud558\uace0 \uc788\uc73c\uba70, \uc2a4\ud1a4\uc740 \uc791\uac00 \uc9c0\ub9dd\uc0dd \uc2a4\ud0a4\ud130 \uc5ed\ud560\uc744 \ub9e1\uc544 \ub0a8\ubd80 \ubbf8\uad6d \uc601\uc5b4\ub97c \uc5f0\uae30\ub97c \ud588\ub2e4. \uc601\ud654\ub294 \uc88b\uc740 \ud3c9\uc744 \ubc1b\uc558\uc73c\uba70, \ud765\ud589\uc5d0\uc11c\ub3c4 \uc131\uacf5\ud558\uc600\ub2e4. \uc774 \uc791\ud488\uc740 \uc81c84\ud68c \ubbf8\uad6d \uc544\uce74\ub370\ubbf8\uc0c1 \ucd5c\uc6b0\uc218 \uc791\ud488\uc0c1 \ud6c4\ubcf4\uc5d0 \uc62c\ub790\ub2e4. \uc601\ud654\uc758 \uc6d0\uc791\uc790 \uce90\uc11c\ub9b0 \uc2a4\ud1a0\ud0b7\uc740 \uc2a4\ud0a4\ud130 \uc5ed\uc5d0 \uc5e0\ub9c8 \uc2a4\ud1a4 \uc774\uc678\uc758 \ub2e4\ub978 \ubc30\uc6b0\ub97c \uc0dd\uac01\ud558\uc9c0 \uc54a\uc558\ub2e4\uace0 \ud55c\ub2e4. \ub610, \uc2a4\ud1a4\uc740 \uc790\uc2e0\uc774 \uc5f0\uae30\ud55c \uc2a4\ud0a4\ud130 \uc5ed\ud560\uc5d0 \ub300\ud574 \uc790\uc2e0\uc758 \uc131\uaca9\uc744 \uc5b4\ub290\uc815\ub3c4 \ubc18\uc601\ud558\uc600\ub2e4\uace0 \ub9d0\ud588\ub2e4.", "answer": "2011\ub144", "sentence": "2011\ub144 \uc5d0\ub294 \uc70c \uae00\ub7ed \uac10\ub3c5\uc758 \u300a\ud504\ub80c\uc988 \uc704\ub4dc \ubca0\ub124\ud54f\u300b\uc5d0\uc11c \uc800\uc2a4\ud2f4 \ud300\ubc84\ub808\uc774\ud06c\uc640 \ubc00\ub77c \ucfe0\ub2c8\uc2a4\uc640 \ud568\uaed8 \uc5f0\uae30\ud558\uc600\ub2e4.", "paragraph_sentence": " 2011\ub144 \uc5d0\ub294 \uc70c \uae00\ub7ed \uac10\ub3c5\uc758 \u300a\ud504\ub80c\uc988 \uc704\ub4dc \ubca0\ub124\ud54f\u300b\uc5d0\uc11c \uc800\uc2a4\ud2f4 \ud300\ubc84\ub808\uc774\ud06c\uc640 \ubc00\ub77c \ucfe0\ub2c8\uc2a4\uc640 \ud568\uaed8 \uc5f0\uae30\ud558\uc600\ub2e4. \uc601\ud654\ub294 2010\ub144 7\uc6d4 \ub274\uc695\uacfc \ub85c\uc2a4\uc564\uc824\ub808\uc2a4\uc5d0\uc11c \ucd2c\uc601\uc774 \uc2dc\uc791\ub418\uc5b4 2011\ub144 7\uc6d4 22\uc77c\uc5d0 \uacf5\uac1c\ub418\uc5c8\ub2e4. \uac19\uc740 \ud574, \uc2a4\ud2f0\ube0c \ucee4\ub810, \ub77c\uc774\uc5b8 \uace0\uc2ac\ub9c1\uacfc \ud568\uaed8 \ucd9c\uc5f0\ud55c \u300a\ud06c\ub808\uc774\uc9c0, \uc2a4\ud22c\ud53c\ub4dc, \ub7ec\ube0c\u300b\uac00 \uacf5\uac1c\ub418\uc5c8\ub2e4. \uc601\ud654\ub294 \uc88b\uc740 \ud3c9\uc744 \ubc1b\uc558\uace0, TV \uac00\uc774\ub4dc\uac00 \uc120\uc815\ud55c 2011\ub144 \ucd5c\uace0\uc758 \uc601\ud654 \ubaa9\ub85d\uc5d0 \uc774\ub984\uc744 \uc62c\ub838\ub2e4. \ud765\ud589\ub3c4 \uc131\uacf5\ud55c \ud3b8\uc774\uba70, \uc2a4\ud1a4 \ub610\ud55c \uc5ec\ub7ec \uc2dc\uc0c1\uc2dd\uc5d0 \ud6c4\ubcf4\ub85c \uc624\ub974\uac70\ub098 \uc218\uc0c1\ud558\uc600\ub2e4. \ub610\ud55c, \uce90\uc11c\ub9b0 \uc2a4\ud1a0\ud0b7\uc758 \ub3d9\uba85\uc758 \uc18c\uc124\uc744 \uc6d0\uc791\uc73c\ub85c \ud55c \u300a\ud5ec\ud504\u300b\uc5d0 \ucd9c\uc5f0\ud558\uc600\ub2e4. \uc601\ud654\ub294 1960\ub144\ub300 \ubbf8\uc2dc\uc2dc\ud53c \uc8fc \uc7ad\uc2a8\uc744 \ubb34\ub300\ub85c \ud558\uace0 \uc788\uc73c\uba70, \uc2a4\ud1a4\uc740 \uc791\uac00 \uc9c0\ub9dd\uc0dd \uc2a4\ud0a4\ud130 \uc5ed\ud560\uc744 \ub9e1\uc544 \ub0a8\ubd80 \ubbf8\uad6d \uc601\uc5b4\ub97c \uc5f0\uae30\ub97c \ud588\ub2e4. \uc601\ud654\ub294 \uc88b\uc740 \ud3c9\uc744 \ubc1b\uc558\uc73c\uba70, \ud765\ud589\uc5d0\uc11c\ub3c4 \uc131\uacf5\ud558\uc600\ub2e4. \uc774 \uc791\ud488\uc740 \uc81c84\ud68c \ubbf8\uad6d \uc544\uce74\ub370\ubbf8\uc0c1 \ucd5c\uc6b0\uc218 \uc791\ud488\uc0c1 \ud6c4\ubcf4\uc5d0 \uc62c\ub790\ub2e4. \uc601\ud654\uc758 \uc6d0\uc791\uc790 \uce90\uc11c\ub9b0 \uc2a4\ud1a0\ud0b7\uc740 \uc2a4\ud0a4\ud130 \uc5ed\uc5d0 \uc5e0\ub9c8 \uc2a4\ud1a4 \uc774\uc678\uc758 \ub2e4\ub978 \ubc30\uc6b0\ub97c \uc0dd\uac01\ud558\uc9c0 \uc54a\uc558\ub2e4\uace0 \ud55c\ub2e4. \ub610, \uc2a4\ud1a4\uc740 \uc790\uc2e0\uc774 \uc5f0\uae30\ud55c \uc2a4\ud0a4\ud130 \uc5ed\ud560\uc5d0 \ub300\ud574 \uc790\uc2e0\uc758 \uc131\uaca9\uc744 \uc5b4\ub290\uc815\ub3c4 \ubc18\uc601\ud558\uc600\ub2e4\uace0 \ub9d0\ud588\ub2e4.", "paragraph_answer": " 2011\ub144 \uc5d0\ub294 \uc70c \uae00\ub7ed \uac10\ub3c5\uc758 \u300a\ud504\ub80c\uc988 \uc704\ub4dc \ubca0\ub124\ud54f\u300b\uc5d0\uc11c \uc800\uc2a4\ud2f4 \ud300\ubc84\ub808\uc774\ud06c\uc640 \ubc00\ub77c \ucfe0\ub2c8\uc2a4\uc640 \ud568\uaed8 \uc5f0\uae30\ud558\uc600\ub2e4. \uc601\ud654\ub294 2010\ub144 7\uc6d4 \ub274\uc695\uacfc \ub85c\uc2a4\uc564\uc824\ub808\uc2a4\uc5d0\uc11c \ucd2c\uc601\uc774 \uc2dc\uc791\ub418\uc5b4 2011\ub144 7\uc6d4 22\uc77c\uc5d0 \uacf5\uac1c\ub418\uc5c8\ub2e4. \uac19\uc740 \ud574, \uc2a4\ud2f0\ube0c \ucee4\ub810, \ub77c\uc774\uc5b8 \uace0\uc2ac\ub9c1\uacfc \ud568\uaed8 \ucd9c\uc5f0\ud55c \u300a\ud06c\ub808\uc774\uc9c0, \uc2a4\ud22c\ud53c\ub4dc, \ub7ec\ube0c\u300b\uac00 \uacf5\uac1c\ub418\uc5c8\ub2e4. \uc601\ud654\ub294 \uc88b\uc740 \ud3c9\uc744 \ubc1b\uc558\uace0, TV \uac00\uc774\ub4dc\uac00 \uc120\uc815\ud55c 2011\ub144 \ucd5c\uace0\uc758 \uc601\ud654 \ubaa9\ub85d\uc5d0 \uc774\ub984\uc744 \uc62c\ub838\ub2e4. \ud765\ud589\ub3c4 \uc131\uacf5\ud55c \ud3b8\uc774\uba70, \uc2a4\ud1a4 \ub610\ud55c \uc5ec\ub7ec \uc2dc\uc0c1\uc2dd\uc5d0 \ud6c4\ubcf4\ub85c \uc624\ub974\uac70\ub098 \uc218\uc0c1\ud558\uc600\ub2e4. \ub610\ud55c, \uce90\uc11c\ub9b0 \uc2a4\ud1a0\ud0b7\uc758 \ub3d9\uba85\uc758 \uc18c\uc124\uc744 \uc6d0\uc791\uc73c\ub85c \ud55c \u300a\ud5ec\ud504\u300b\uc5d0 \ucd9c\uc5f0\ud558\uc600\ub2e4. \uc601\ud654\ub294 1960\ub144\ub300 \ubbf8\uc2dc\uc2dc\ud53c \uc8fc \uc7ad\uc2a8\uc744 \ubb34\ub300\ub85c \ud558\uace0 \uc788\uc73c\uba70, \uc2a4\ud1a4\uc740 \uc791\uac00 \uc9c0\ub9dd\uc0dd \uc2a4\ud0a4\ud130 \uc5ed\ud560\uc744 \ub9e1\uc544 \ub0a8\ubd80 \ubbf8\uad6d \uc601\uc5b4\ub97c \uc5f0\uae30\ub97c \ud588\ub2e4. \uc601\ud654\ub294 \uc88b\uc740 \ud3c9\uc744 \ubc1b\uc558\uc73c\uba70, \ud765\ud589\uc5d0\uc11c\ub3c4 \uc131\uacf5\ud558\uc600\ub2e4. \uc774 \uc791\ud488\uc740 \uc81c84\ud68c \ubbf8\uad6d \uc544\uce74\ub370\ubbf8\uc0c1 \ucd5c\uc6b0\uc218 \uc791\ud488\uc0c1 \ud6c4\ubcf4\uc5d0 \uc62c\ub790\ub2e4. \uc601\ud654\uc758 \uc6d0\uc791\uc790 \uce90\uc11c\ub9b0 \uc2a4\ud1a0\ud0b7\uc740 \uc2a4\ud0a4\ud130 \uc5ed\uc5d0 \uc5e0\ub9c8 \uc2a4\ud1a4 \uc774\uc678\uc758 \ub2e4\ub978 \ubc30\uc6b0\ub97c \uc0dd\uac01\ud558\uc9c0 \uc54a\uc558\ub2e4\uace0 \ud55c\ub2e4. \ub610, \uc2a4\ud1a4\uc740 \uc790\uc2e0\uc774 \uc5f0\uae30\ud55c \uc2a4\ud0a4\ud130 \uc5ed\ud560\uc5d0 \ub300\ud574 \uc790\uc2e0\uc758 \uc131\uaca9\uc744 \uc5b4\ub290\uc815\ub3c4 \ubc18\uc601\ud558\uc600\ub2e4\uace0 \ub9d0\ud588\ub2e4.", "sentence_answer": " 2011\ub144 \uc5d0\ub294 \uc70c \uae00\ub7ed \uac10\ub3c5\uc758 \u300a\ud504\ub80c\uc988 \uc704\ub4dc \ubca0\ub124\ud54f\u300b\uc5d0\uc11c \uc800\uc2a4\ud2f4 \ud300\ubc84\ub808\uc774\ud06c\uc640 \ubc00\ub77c \ucfe0\ub2c8\uc2a4\uc640 \ud568\uaed8 \uc5f0\uae30\ud558\uc600\ub2e4.", "paragraph_id": "6523785-5-0", "paragraph_question": "question: \uc601\ud654 \ud504\ub80c\uc988 \uc704\ub4dc \ubca0\ub124\ud54f\uc740 \uba87 \ub144\ub3c4\uc5d0 \uacf5\uac1c\ub418\uc5c8\ub294\uac00?, context: 2011\ub144\uc5d0\ub294 \uc70c \uae00\ub7ed \uac10\ub3c5\uc758 \u300a\ud504\ub80c\uc988 \uc704\ub4dc \ubca0\ub124\ud54f\u300b\uc5d0\uc11c \uc800\uc2a4\ud2f4 \ud300\ubc84\ub808\uc774\ud06c\uc640 \ubc00\ub77c \ucfe0\ub2c8\uc2a4\uc640 \ud568\uaed8 \uc5f0\uae30\ud558\uc600\ub2e4. \uc601\ud654\ub294 2010\ub144 7\uc6d4 \ub274\uc695\uacfc \ub85c\uc2a4\uc564\uc824\ub808\uc2a4\uc5d0\uc11c \ucd2c\uc601\uc774 \uc2dc\uc791\ub418\uc5b4 2011\ub144 7\uc6d4 22\uc77c\uc5d0 \uacf5\uac1c\ub418\uc5c8\ub2e4. \uac19\uc740 \ud574, \uc2a4\ud2f0\ube0c \ucee4\ub810, \ub77c\uc774\uc5b8 \uace0\uc2ac\ub9c1\uacfc \ud568\uaed8 \ucd9c\uc5f0\ud55c \u300a\ud06c\ub808\uc774\uc9c0, \uc2a4\ud22c\ud53c\ub4dc, \ub7ec\ube0c\u300b\uac00 \uacf5\uac1c\ub418\uc5c8\ub2e4. \uc601\ud654\ub294 \uc88b\uc740 \ud3c9\uc744 \ubc1b\uc558\uace0, TV \uac00\uc774\ub4dc\uac00 \uc120\uc815\ud55c 2011\ub144 \ucd5c\uace0\uc758 \uc601\ud654 \ubaa9\ub85d\uc5d0 \uc774\ub984\uc744 \uc62c\ub838\ub2e4. \ud765\ud589\ub3c4 \uc131\uacf5\ud55c \ud3b8\uc774\uba70, \uc2a4\ud1a4 \ub610\ud55c \uc5ec\ub7ec \uc2dc\uc0c1\uc2dd\uc5d0 \ud6c4\ubcf4\ub85c \uc624\ub974\uac70\ub098 \uc218\uc0c1\ud558\uc600\ub2e4. \ub610\ud55c, \uce90\uc11c\ub9b0 \uc2a4\ud1a0\ud0b7\uc758 \ub3d9\uba85\uc758 \uc18c\uc124\uc744 \uc6d0\uc791\uc73c\ub85c \ud55c \u300a\ud5ec\ud504\u300b\uc5d0 \ucd9c\uc5f0\ud558\uc600\ub2e4. \uc601\ud654\ub294 1960\ub144\ub300 \ubbf8\uc2dc\uc2dc\ud53c \uc8fc \uc7ad\uc2a8\uc744 \ubb34\ub300\ub85c \ud558\uace0 \uc788\uc73c\uba70, \uc2a4\ud1a4\uc740 \uc791\uac00 \uc9c0\ub9dd\uc0dd \uc2a4\ud0a4\ud130 \uc5ed\ud560\uc744 \ub9e1\uc544 \ub0a8\ubd80 \ubbf8\uad6d \uc601\uc5b4\ub97c \uc5f0\uae30\ub97c \ud588\ub2e4. \uc601\ud654\ub294 \uc88b\uc740 \ud3c9\uc744 \ubc1b\uc558\uc73c\uba70, \ud765\ud589\uc5d0\uc11c\ub3c4 \uc131\uacf5\ud558\uc600\ub2e4. \uc774 \uc791\ud488\uc740 \uc81c84\ud68c \ubbf8\uad6d \uc544\uce74\ub370\ubbf8\uc0c1 \ucd5c\uc6b0\uc218 \uc791\ud488\uc0c1 \ud6c4\ubcf4\uc5d0 \uc62c\ub790\ub2e4. \uc601\ud654\uc758 \uc6d0\uc791\uc790 \uce90\uc11c\ub9b0 \uc2a4\ud1a0\ud0b7\uc740 \uc2a4\ud0a4\ud130 \uc5ed\uc5d0 \uc5e0\ub9c8 \uc2a4\ud1a4 \uc774\uc678\uc758 \ub2e4\ub978 \ubc30\uc6b0\ub97c \uc0dd\uac01\ud558\uc9c0 \uc54a\uc558\ub2e4\uace0 \ud55c\ub2e4. \ub610, \uc2a4\ud1a4\uc740 \uc790\uc2e0\uc774 \uc5f0\uae30\ud55c \uc2a4\ud0a4\ud130 \uc5ed\ud560\uc5d0 \ub300\ud574 \uc790\uc2e0\uc758 \uc131\uaca9\uc744 \uc5b4\ub290\uc815\ub3c4 \ubc18\uc601\ud558\uc600\ub2e4\uace0 \ub9d0\ud588\ub2e4."} {"question": "\ud55c\uc885\uc6b0 \uad50\uc218\uac00 \uae40 \uc804 \ub300\ud1b5\ub839\uc774 \uac00\uc7a5 \ud070 \uacf5\ud5cc\uc744 \ud55c \ubd84\uc57c\ub77c\uace0 \ud3c9\uac00\ud55c \uac83\uc740?", "paragraph": "\ub9ac\uc624\ub12c \uc870\uc2a4\ud33d \uc804 \ud504\ub791\uc2a4 \ucd1d\ub9ac\ub294 \"\uae40 \ub300\ud1b5\ub839\uc740 \ub098\uc5d0\uac8c \uc0b4\uc544\uac00\uc57c \ud560 \ud798, \uc0b4\uc544\uac00\uc57c \ud560 \ub3c4\ub355\uc801 \uc2a4\uc2b9\uc774\uc790 \uae38\uc7a1\uc774\ub2e4\"\ub77c\uace0 \uadf9\ucc2c\ud588\ub2e4. \uc694\ud558\ub124\uc2a4 \ub77c\uc6b0 \uc804 \ub3c5\uc77c \ub300\ud1b5\ub839\uc740 \"\uae40 \ub300\ud1b5\ub839\uc5d0 \ub300\ud55c \uc874\uacbd\uc2ec\uc774 \ub3c5\uc77c\uc774 \ud55c\uad6d\uc758 \uae08\uc735\uc704\uae30 \ub54c \ud55c\uad6d\uc744 \ub3d5\ub294 \ub3d9\uae30\uac00 \ub410\ub2e4\"\ub77c\uace0 \ubc1d\ud614\ub2e4. \ube4c \ud074\ub9b0\ud134 \uc804 \ubbf8\uad6d \ub300\ud1b5\ub839\uc740 \ud55c\ubbf8 \uc815\uc0c1 \ubc31\uc545\uad00 \uae30\uc790\ud68c\uacac\uc5d0\uc11c \ub300\ubd81\uc815\ucc45\uc5d0 \ub300\ud574 \"\uae40 \ub300\ud1b5\ub839\uc740 \uc9c0\uae08 \ud55c\ubc18\ub3c4\uc758 \uc815\uc138\ub97c \ubcf8\uc9c8\uc801\uc73c\ub85c \ubcc0\ud654\uc2dc\ud0a4\ub294 \uc804\uc8fc\uace1\uc744 \uc5f0\uc8fc\ud558\uace0 \uc788\uc2b5\ub2c8\ub2e4. \uae40 \ub300\ud1b5\ub839\uc758 \uc77c\uad00\ub41c \ube44\uc804\uacfc \uac15\uc778\ud55c \uc758\uc9c0\ub294 \uc774\ub97c \uc131\uacf5\uc791\uc73c\ub85c \ub9cc\ub4e4\uac83\uc774\ub77c \ubbff\uc2b5\ub2c8\ub2e4. \uadf8\ub807\uac8c \ub418\uba74 \uad6d\ubc29\uc608\uc0b0\uc744 \uc904\uc5ec \uc0ac\ud68c\ubcf5\uc9c0\ub97c \ub298\ub9b4 \uc218 \uc788\uaca0\uc9c0\uc694\"\ub77c\uace0 \ud3c9\uac00\ud588\ub2e4. \ud074\ub9b0\ud134 \uc804 \ub300\ud1b5\ub839\uc774 \uae40\ub300\uc911\uc5d0\uac8c \"\ub098\uc5d0\uac8c 1\ub144\uc774\ub77c\ub294 \uc2dc\uac04\ub9cc \ub354 \uc788\uc5c8\ub2e4\uba74 \ud55c\ubc18\ub3c4\uc758 \uba85\uc6b4\uc774 \ub2ec\ub77c\uc84c\uc744 \uac83\"\uc774\ub77c\uba70 \uc544\uc26c\uc6cc\ud588\ub2e4\uace0 \ubc1d\ud614\ub2e4. \ud55c\uc885\uc6b0 \uc2dc\ub7ec\ud050\uc2a4 \ub300\ud559\uad50 \uad50\uc218\ub294 \"\uae40 \uc804 \ub300\ud1b5\ub839\uc740 \ubbfc\uc8fc\ud654\uc5d0 \uac00\uc7a5 \ud070 \uacf5\ud5cc\uc744 \ud558\uc168\uace0, IMF \uacbd\uc81c\uc704\uae30\ub97c \uadf9\ubcf5\ud558\uc2dc\uace0, \ubd81\ud55c\uacfc\uc758 \uad50\ub958\ub97c \ud0c4\ud0c4\ub300\ub85c\uc5d0 \uc62c\ub824\ub193\uc73c\uc168\ub2e4\"\ub77c\uace0 \ud3c9\uac00\ud588\ub2e4.", "answer": "\ubbfc\uc8fc\ud654", "sentence": "\ud55c\uc885\uc6b0 \uc2dc\ub7ec\ud050\uc2a4 \ub300\ud559\uad50 \uad50\uc218\ub294 \"\uae40 \uc804 \ub300\ud1b5\ub839\uc740 \ubbfc\uc8fc\ud654 \uc5d0 \uac00\uc7a5 \ud070 \uacf5\ud5cc\uc744 \ud558\uc168\uace0, IMF \uacbd\uc81c\uc704\uae30\ub97c \uadf9\ubcf5\ud558\uc2dc\uace0, \ubd81\ud55c\uacfc\uc758 \uad50\ub958\ub97c \ud0c4\ud0c4\ub300\ub85c\uc5d0 \uc62c\ub824\ub193\uc73c\uc168\ub2e4\"\ub77c\uace0 \ud3c9\uac00\ud588\ub2e4.", "paragraph_sentence": "\ub9ac\uc624\ub12c \uc870\uc2a4\ud33d \uc804 \ud504\ub791\uc2a4 \ucd1d\ub9ac\ub294 \"\uae40 \ub300\ud1b5\ub839\uc740 \ub098\uc5d0\uac8c \uc0b4\uc544\uac00\uc57c \ud560 \ud798, \uc0b4\uc544\uac00\uc57c \ud560 \ub3c4\ub355\uc801 \uc2a4\uc2b9\uc774\uc790 \uae38\uc7a1\uc774\ub2e4\"\ub77c\uace0 \uadf9\ucc2c\ud588\ub2e4. \uc694\ud558\ub124\uc2a4 \ub77c\uc6b0 \uc804 \ub3c5\uc77c \ub300\ud1b5\ub839\uc740 \"\uae40 \ub300\ud1b5\ub839\uc5d0 \ub300\ud55c \uc874\uacbd\uc2ec\uc774 \ub3c5\uc77c\uc774 \ud55c\uad6d\uc758 \uae08\uc735\uc704\uae30 \ub54c \ud55c\uad6d\uc744 \ub3d5\ub294 \ub3d9\uae30\uac00 \ub410\ub2e4\"\ub77c\uace0 \ubc1d\ud614\ub2e4. \ube4c \ud074\ub9b0\ud134 \uc804 \ubbf8\uad6d \ub300\ud1b5\ub839\uc740 \ud55c\ubbf8 \uc815\uc0c1 \ubc31\uc545\uad00 \uae30\uc790\ud68c\uacac\uc5d0\uc11c \ub300\ubd81\uc815\ucc45\uc5d0 \ub300\ud574 \"\uae40 \ub300\ud1b5\ub839\uc740 \uc9c0\uae08 \ud55c\ubc18\ub3c4\uc758 \uc815\uc138\ub97c \ubcf8\uc9c8\uc801\uc73c\ub85c \ubcc0\ud654\uc2dc\ud0a4\ub294 \uc804\uc8fc\uace1\uc744 \uc5f0\uc8fc\ud558\uace0 \uc788\uc2b5\ub2c8\ub2e4. \uae40 \ub300\ud1b5\ub839\uc758 \uc77c\uad00\ub41c \ube44\uc804\uacfc \uac15\uc778\ud55c \uc758\uc9c0\ub294 \uc774\ub97c \uc131\uacf5\uc791\uc73c\ub85c \ub9cc\ub4e4\uac83\uc774\ub77c \ubbff\uc2b5\ub2c8\ub2e4. \uadf8\ub807\uac8c \ub418\uba74 \uad6d\ubc29\uc608\uc0b0\uc744 \uc904\uc5ec \uc0ac\ud68c\ubcf5\uc9c0\ub97c \ub298\ub9b4 \uc218 \uc788\uaca0\uc9c0\uc694\"\ub77c\uace0 \ud3c9\uac00\ud588\ub2e4. \ud074\ub9b0\ud134 \uc804 \ub300\ud1b5\ub839\uc774 \uae40\ub300\uc911\uc5d0\uac8c \"\ub098\uc5d0\uac8c 1\ub144\uc774\ub77c\ub294 \uc2dc\uac04\ub9cc \ub354 \uc788\uc5c8\ub2e4\uba74 \ud55c\ubc18\ub3c4\uc758 \uba85\uc6b4\uc774 \ub2ec\ub77c\uc84c\uc744 \uac83\"\uc774\ub77c\uba70 \uc544\uc26c\uc6cc\ud588\ub2e4\uace0 \ubc1d\ud614\ub2e4. \ud55c\uc885\uc6b0 \uc2dc\ub7ec\ud050\uc2a4 \ub300\ud559\uad50 \uad50\uc218\ub294 \"\uae40 \uc804 \ub300\ud1b5\ub839\uc740 \ubbfc\uc8fc\ud654 \uc5d0 \uac00\uc7a5 \ud070 \uacf5\ud5cc\uc744 \ud558\uc168\uace0, IMF \uacbd\uc81c\uc704\uae30\ub97c \uadf9\ubcf5\ud558\uc2dc\uace0, \ubd81\ud55c\uacfc\uc758 \uad50\ub958\ub97c \ud0c4\ud0c4\ub300\ub85c\uc5d0 \uc62c\ub824\ub193\uc73c\uc168\ub2e4\"\ub77c\uace0 \ud3c9\uac00\ud588\ub2e4. ", "paragraph_answer": "\ub9ac\uc624\ub12c \uc870\uc2a4\ud33d \uc804 \ud504\ub791\uc2a4 \ucd1d\ub9ac\ub294 \"\uae40 \ub300\ud1b5\ub839\uc740 \ub098\uc5d0\uac8c \uc0b4\uc544\uac00\uc57c \ud560 \ud798, \uc0b4\uc544\uac00\uc57c \ud560 \ub3c4\ub355\uc801 \uc2a4\uc2b9\uc774\uc790 \uae38\uc7a1\uc774\ub2e4\"\ub77c\uace0 \uadf9\ucc2c\ud588\ub2e4. \uc694\ud558\ub124\uc2a4 \ub77c\uc6b0 \uc804 \ub3c5\uc77c \ub300\ud1b5\ub839\uc740 \"\uae40 \ub300\ud1b5\ub839\uc5d0 \ub300\ud55c \uc874\uacbd\uc2ec\uc774 \ub3c5\uc77c\uc774 \ud55c\uad6d\uc758 \uae08\uc735\uc704\uae30 \ub54c \ud55c\uad6d\uc744 \ub3d5\ub294 \ub3d9\uae30\uac00 \ub410\ub2e4\"\ub77c\uace0 \ubc1d\ud614\ub2e4. \ube4c \ud074\ub9b0\ud134 \uc804 \ubbf8\uad6d \ub300\ud1b5\ub839\uc740 \ud55c\ubbf8 \uc815\uc0c1 \ubc31\uc545\uad00 \uae30\uc790\ud68c\uacac\uc5d0\uc11c \ub300\ubd81\uc815\ucc45\uc5d0 \ub300\ud574 \"\uae40 \ub300\ud1b5\ub839\uc740 \uc9c0\uae08 \ud55c\ubc18\ub3c4\uc758 \uc815\uc138\ub97c \ubcf8\uc9c8\uc801\uc73c\ub85c \ubcc0\ud654\uc2dc\ud0a4\ub294 \uc804\uc8fc\uace1\uc744 \uc5f0\uc8fc\ud558\uace0 \uc788\uc2b5\ub2c8\ub2e4. \uae40 \ub300\ud1b5\ub839\uc758 \uc77c\uad00\ub41c \ube44\uc804\uacfc \uac15\uc778\ud55c \uc758\uc9c0\ub294 \uc774\ub97c \uc131\uacf5\uc791\uc73c\ub85c \ub9cc\ub4e4\uac83\uc774\ub77c \ubbff\uc2b5\ub2c8\ub2e4. \uadf8\ub807\uac8c \ub418\uba74 \uad6d\ubc29\uc608\uc0b0\uc744 \uc904\uc5ec \uc0ac\ud68c\ubcf5\uc9c0\ub97c \ub298\ub9b4 \uc218 \uc788\uaca0\uc9c0\uc694\"\ub77c\uace0 \ud3c9\uac00\ud588\ub2e4. \ud074\ub9b0\ud134 \uc804 \ub300\ud1b5\ub839\uc774 \uae40\ub300\uc911\uc5d0\uac8c \"\ub098\uc5d0\uac8c 1\ub144\uc774\ub77c\ub294 \uc2dc\uac04\ub9cc \ub354 \uc788\uc5c8\ub2e4\uba74 \ud55c\ubc18\ub3c4\uc758 \uba85\uc6b4\uc774 \ub2ec\ub77c\uc84c\uc744 \uac83\"\uc774\ub77c\uba70 \uc544\uc26c\uc6cc\ud588\ub2e4\uace0 \ubc1d\ud614\ub2e4. \ud55c\uc885\uc6b0 \uc2dc\ub7ec\ud050\uc2a4 \ub300\ud559\uad50 \uad50\uc218\ub294 \"\uae40 \uc804 \ub300\ud1b5\ub839\uc740 \ubbfc\uc8fc\ud654 \uc5d0 \uac00\uc7a5 \ud070 \uacf5\ud5cc\uc744 \ud558\uc168\uace0, IMF \uacbd\uc81c\uc704\uae30\ub97c \uadf9\ubcf5\ud558\uc2dc\uace0, \ubd81\ud55c\uacfc\uc758 \uad50\ub958\ub97c \ud0c4\ud0c4\ub300\ub85c\uc5d0 \uc62c\ub824\ub193\uc73c\uc168\ub2e4\"\ub77c\uace0 \ud3c9\uac00\ud588\ub2e4.", "sentence_answer": "\ud55c\uc885\uc6b0 \uc2dc\ub7ec\ud050\uc2a4 \ub300\ud559\uad50 \uad50\uc218\ub294 \"\uae40 \uc804 \ub300\ud1b5\ub839\uc740 \ubbfc\uc8fc\ud654 \uc5d0 \uac00\uc7a5 \ud070 \uacf5\ud5cc\uc744 \ud558\uc168\uace0, IMF \uacbd\uc81c\uc704\uae30\ub97c \uadf9\ubcf5\ud558\uc2dc\uace0, \ubd81\ud55c\uacfc\uc758 \uad50\ub958\ub97c \ud0c4\ud0c4\ub300\ub85c\uc5d0 \uc62c\ub824\ub193\uc73c\uc168\ub2e4\"\ub77c\uace0 \ud3c9\uac00\ud588\ub2e4.", "paragraph_id": "6576830-63-1", "paragraph_question": "question: \ud55c\uc885\uc6b0 \uad50\uc218\uac00 \uae40 \uc804 \ub300\ud1b5\ub839\uc774 \uac00\uc7a5 \ud070 \uacf5\ud5cc\uc744 \ud55c \ubd84\uc57c\ub77c\uace0 \ud3c9\uac00\ud55c \uac83\uc740?, context: \ub9ac\uc624\ub12c \uc870\uc2a4\ud33d \uc804 \ud504\ub791\uc2a4 \ucd1d\ub9ac\ub294 \"\uae40 \ub300\ud1b5\ub839\uc740 \ub098\uc5d0\uac8c \uc0b4\uc544\uac00\uc57c \ud560 \ud798, \uc0b4\uc544\uac00\uc57c \ud560 \ub3c4\ub355\uc801 \uc2a4\uc2b9\uc774\uc790 \uae38\uc7a1\uc774\ub2e4\"\ub77c\uace0 \uadf9\ucc2c\ud588\ub2e4. \uc694\ud558\ub124\uc2a4 \ub77c\uc6b0 \uc804 \ub3c5\uc77c \ub300\ud1b5\ub839\uc740 \"\uae40 \ub300\ud1b5\ub839\uc5d0 \ub300\ud55c \uc874\uacbd\uc2ec\uc774 \ub3c5\uc77c\uc774 \ud55c\uad6d\uc758 \uae08\uc735\uc704\uae30 \ub54c \ud55c\uad6d\uc744 \ub3d5\ub294 \ub3d9\uae30\uac00 \ub410\ub2e4\"\ub77c\uace0 \ubc1d\ud614\ub2e4. \ube4c \ud074\ub9b0\ud134 \uc804 \ubbf8\uad6d \ub300\ud1b5\ub839\uc740 \ud55c\ubbf8 \uc815\uc0c1 \ubc31\uc545\uad00 \uae30\uc790\ud68c\uacac\uc5d0\uc11c \ub300\ubd81\uc815\ucc45\uc5d0 \ub300\ud574 \"\uae40 \ub300\ud1b5\ub839\uc740 \uc9c0\uae08 \ud55c\ubc18\ub3c4\uc758 \uc815\uc138\ub97c \ubcf8\uc9c8\uc801\uc73c\ub85c \ubcc0\ud654\uc2dc\ud0a4\ub294 \uc804\uc8fc\uace1\uc744 \uc5f0\uc8fc\ud558\uace0 \uc788\uc2b5\ub2c8\ub2e4. \uae40 \ub300\ud1b5\ub839\uc758 \uc77c\uad00\ub41c \ube44\uc804\uacfc \uac15\uc778\ud55c \uc758\uc9c0\ub294 \uc774\ub97c \uc131\uacf5\uc791\uc73c\ub85c \ub9cc\ub4e4\uac83\uc774\ub77c \ubbff\uc2b5\ub2c8\ub2e4. \uadf8\ub807\uac8c \ub418\uba74 \uad6d\ubc29\uc608\uc0b0\uc744 \uc904\uc5ec \uc0ac\ud68c\ubcf5\uc9c0\ub97c \ub298\ub9b4 \uc218 \uc788\uaca0\uc9c0\uc694\"\ub77c\uace0 \ud3c9\uac00\ud588\ub2e4. \ud074\ub9b0\ud134 \uc804 \ub300\ud1b5\ub839\uc774 \uae40\ub300\uc911\uc5d0\uac8c \"\ub098\uc5d0\uac8c 1\ub144\uc774\ub77c\ub294 \uc2dc\uac04\ub9cc \ub354 \uc788\uc5c8\ub2e4\uba74 \ud55c\ubc18\ub3c4\uc758 \uba85\uc6b4\uc774 \ub2ec\ub77c\uc84c\uc744 \uac83\"\uc774\ub77c\uba70 \uc544\uc26c\uc6cc\ud588\ub2e4\uace0 \ubc1d\ud614\ub2e4. \ud55c\uc885\uc6b0 \uc2dc\ub7ec\ud050\uc2a4 \ub300\ud559\uad50 \uad50\uc218\ub294 \"\uae40 \uc804 \ub300\ud1b5\ub839\uc740 \ubbfc\uc8fc\ud654\uc5d0 \uac00\uc7a5 \ud070 \uacf5\ud5cc\uc744 \ud558\uc168\uace0, IMF \uacbd\uc81c\uc704\uae30\ub97c \uadf9\ubcf5\ud558\uc2dc\uace0, \ubd81\ud55c\uacfc\uc758 \uad50\ub958\ub97c \ud0c4\ud0c4\ub300\ub85c\uc5d0 \uc62c\ub824\ub193\uc73c\uc168\ub2e4\"\ub77c\uace0 \ud3c9\uac00\ud588\ub2e4."} {"question": "\uc774 \uace1\uc5d0\uc11c A\ub2e8\uc870\uc758 \ub538\ub9bc\ud654\uc74c \ud6c4 E\uc74c\uc744 \ud2b8\ub808\ubab0\ub85c\ub85c \uc5f0\uc8fc\ud558\ub294 \uc545\uae30\ub294?", "paragraph": "\uc54c\ub808\uadf8\ub85c \ucf58 \ube0c\ub77c\uc624\uac00 \ub418\ub294 55\ub9c8\ub514\ubd80\ud130 \uc81c\uc2dc\ubd80\uac00 \uc2dc\uc791\ub41c\ub2e4. A\ub2e8\uc870\uc758 \ub538\ub9bc\ud654\uc74c\uc744 \uc774\uc5b4\ubc1b\uc544 \ubc14\uc774\uc62c\ub9b0\uc774 E\uc74c\uc744 \ud2b8\ub808\ubab0\ub85c\ub85c \uc5f0\uc8fc\ud558\uc5ec \uc2dc\uc791\ub41c\ub2e4. \uc774\uac83\uc740 A\ub2e8\uc870\uc758 \ub538\ub9bc\ud654\uc74c\uc774 \uc544\ub2c8\ub77c C\uc7a5\uc870\uc758 \uc81c3\uc74c\uc744 \ud558\uae30 \uc2dc\uc791\ud55c\ub2e4. \uc774 \ud2b8\ub808\ubab0\ub85c\ub97c \ubc14\ud0d5\uc73c\ub85c \ud638\ub978\uacfc \ud604\uc774 \uc800\uc74c\ubd80\uc5d0\uc11c \uc81c1\uc8fc\uc81c\ub97c \uc81c\uc2dc\ud55c\ub2e4. \uc774 \uc8fc\uc81c\ub3d9\uae30\uac00 C\uc74c\uc758 \uc9c0\uc18d\uc800\uc74c \uc704\uc5d0\uc11c \ub300\ub2f4\ud55c \ubd88\ud611\ud654\uc74c\uc744 \uc774\ub8e8\uba74\uc11c \uc81c5\ub3c4\uc74c\uc744 \ud558\ud589\uc2dc\ud0a8 G\ub2e8\uc870\uc640 B\u266d\ub2e8\uc870\uc801\uc778 \uc6b8\ub9bc\uc744 \uac70\uccd0 \uac15\uc870\ub418\uba74, 78\ub9c8\ub514\ubd80\ud130 \ud22c\ud2f0\uc758 ff\ub85c \uba85\ud655\ud55c \ud615\ud0dc\uc758 \uc8fc\uc81c\uac00 \ud655\ubcf4\ub41c\ub2e4. \uc8fc\uc81c\uc758 \ud6c4\ubc18\uc740 \ud2b9\uc815\uc801\uc778 \ub9ac\ub4ec\uc744 \uac00\uc9c4 \ub3d9\uae30\ub97c \uc911\uc2ec\uc73c\ub85c \ud06c\uac8c \ubc1c\uc804\ub41c\ub2e4. \ud558\ud589 \uc74c\uacc4\uc758 \uc131\uaca9\uc744 \uac00\uc9c4 \uacbd\uacfc\uc74c\ud615\uc774 \uc810\ucc28 \ud655\uc2e4\ud55c \ud615\ud0dc\uac00 \ub418\uc5b4 110\ub9c8\ub514\ubd80\ud130 ff\ub85c \ub098\ud0c0\ub098\uba74, 112\ub9c8\ub514\uc5d0 G\uc74c\uc758 \uc9c0\uc18d\uc800\uc74c\uc774 \ub193\uc774\uace0 \uadf8 \uc704\uc5d0 \ubc14\uc774\uc62c\ub9b0\uc774 \uc81c2\uc8fc\uc81c\ub97c \uc81c\uc2dc\ud55c\ub2e4. \uc774\ubbf8 \uc554\uc2dc\ub41c \ud558\ud589\uc74c\ud615 \ub3d9\uae30\uac00 \uc644\uc804\ud558\uac8c \ubc1c\uc804\ub41c\ub2e4. \uadf8 \uc18d\uc5d0 \uc5d0\ud54f\ub4dc\uc640 \uac19\uc740 \uc545\uad6c\uac00 \ucc98\uc74c\uc5d0\ub294 \ud50c\ub8e8\ud2b8 \uc774\uc5b4\uc11c \ubc14\uc774\uc62c\ub9b0\uc73c\ub85c \ub098\ud0c0\ub098\uace0, \ub2e4\uc2dc \uc81c2\uc8fc\uc81c\uc758 \ubc1c\uc804\uc801 \uacbd\uacfc\uad6c\uac00 \ub418\uc5b4 133\ub9c8\ub514\uc5d0\uc11c \ud22c\ud2f0\uc758 \ud655\ubcf4\ubd80\uc5d0 \uc774\ub978\ub2e4.", "answer": "\ubc14\uc774\uc62c\ub9b0", "sentence": "A\ub2e8\uc870\uc758 \ub538\ub9bc\ud654\uc74c\uc744 \uc774\uc5b4\ubc1b\uc544 \ubc14\uc774\uc62c\ub9b0 \uc774 E\uc74c\uc744 \ud2b8\ub808\ubab0\ub85c\ub85c \uc5f0\uc8fc\ud558\uc5ec \uc2dc\uc791\ub41c\ub2e4.", "paragraph_sentence": "\uc54c\ub808\uadf8\ub85c \ucf58 \ube0c\ub77c\uc624\uac00 \ub418\ub294 55\ub9c8\ub514\ubd80\ud130 \uc81c\uc2dc\ubd80\uac00 \uc2dc\uc791\ub41c\ub2e4. A\ub2e8\uc870\uc758 \ub538\ub9bc\ud654\uc74c\uc744 \uc774\uc5b4\ubc1b\uc544 \ubc14\uc774\uc62c\ub9b0 \uc774 E\uc74c\uc744 \ud2b8\ub808\ubab0\ub85c\ub85c \uc5f0\uc8fc\ud558\uc5ec \uc2dc\uc791\ub41c\ub2e4. \uc774\uac83\uc740 A\ub2e8\uc870\uc758 \ub538\ub9bc\ud654\uc74c\uc774 \uc544\ub2c8\ub77c C\uc7a5\uc870\uc758 \uc81c3\uc74c\uc744 \ud558\uae30 \uc2dc\uc791\ud55c\ub2e4. \uc774 \ud2b8\ub808\ubab0\ub85c\ub97c \ubc14\ud0d5\uc73c\ub85c \ud638\ub978\uacfc \ud604\uc774 \uc800\uc74c\ubd80\uc5d0\uc11c \uc81c1\uc8fc\uc81c\ub97c \uc81c\uc2dc\ud55c\ub2e4. \uc774 \uc8fc\uc81c\ub3d9\uae30\uac00 C\uc74c\uc758 \uc9c0\uc18d\uc800\uc74c \uc704\uc5d0\uc11c \ub300\ub2f4\ud55c \ubd88\ud611\ud654\uc74c\uc744 \uc774\ub8e8\uba74\uc11c \uc81c5\ub3c4\uc74c\uc744 \ud558\ud589\uc2dc\ud0a8 G\ub2e8\uc870\uc640 B\u266d\ub2e8\uc870\uc801\uc778 \uc6b8\ub9bc\uc744 \uac70\uccd0 \uac15\uc870\ub418\uba74, 78\ub9c8\ub514\ubd80\ud130 \ud22c\ud2f0\uc758 ff\ub85c \uba85\ud655\ud55c \ud615\ud0dc\uc758 \uc8fc\uc81c\uac00 \ud655\ubcf4\ub41c\ub2e4. \uc8fc\uc81c\uc758 \ud6c4\ubc18\uc740 \ud2b9\uc815\uc801\uc778 \ub9ac\ub4ec\uc744 \uac00\uc9c4 \ub3d9\uae30\ub97c \uc911\uc2ec\uc73c\ub85c \ud06c\uac8c \ubc1c\uc804\ub41c\ub2e4. \ud558\ud589 \uc74c\uacc4\uc758 \uc131\uaca9\uc744 \uac00\uc9c4 \uacbd\uacfc\uc74c\ud615\uc774 \uc810\ucc28 \ud655\uc2e4\ud55c \ud615\ud0dc\uac00 \ub418\uc5b4 110\ub9c8\ub514\ubd80\ud130 ff\ub85c \ub098\ud0c0\ub098\uba74, 112\ub9c8\ub514\uc5d0 G\uc74c\uc758 \uc9c0\uc18d\uc800\uc74c\uc774 \ub193\uc774\uace0 \uadf8 \uc704\uc5d0 \ubc14\uc774\uc62c\ub9b0\uc774 \uc81c2\uc8fc\uc81c\ub97c \uc81c\uc2dc\ud55c\ub2e4. \uc774\ubbf8 \uc554\uc2dc\ub41c \ud558\ud589\uc74c\ud615 \ub3d9\uae30\uac00 \uc644\uc804\ud558\uac8c \ubc1c\uc804\ub41c\ub2e4. \uadf8 \uc18d\uc5d0 \uc5d0\ud54f\ub4dc\uc640 \uac19\uc740 \uc545\uad6c\uac00 \ucc98\uc74c\uc5d0\ub294 \ud50c\ub8e8\ud2b8 \uc774\uc5b4\uc11c \ubc14\uc774\uc62c\ub9b0\uc73c\ub85c \ub098\ud0c0\ub098\uace0, \ub2e4\uc2dc \uc81c2\uc8fc\uc81c\uc758 \ubc1c\uc804\uc801 \uacbd\uacfc\uad6c\uac00 \ub418\uc5b4 133\ub9c8\ub514\uc5d0\uc11c \ud22c\ud2f0\uc758 \ud655\ubcf4\ubd80\uc5d0 \uc774\ub978\ub2e4.", "paragraph_answer": "\uc54c\ub808\uadf8\ub85c \ucf58 \ube0c\ub77c\uc624\uac00 \ub418\ub294 55\ub9c8\ub514\ubd80\ud130 \uc81c\uc2dc\ubd80\uac00 \uc2dc\uc791\ub41c\ub2e4. A\ub2e8\uc870\uc758 \ub538\ub9bc\ud654\uc74c\uc744 \uc774\uc5b4\ubc1b\uc544 \ubc14\uc774\uc62c\ub9b0 \uc774 E\uc74c\uc744 \ud2b8\ub808\ubab0\ub85c\ub85c \uc5f0\uc8fc\ud558\uc5ec \uc2dc\uc791\ub41c\ub2e4. \uc774\uac83\uc740 A\ub2e8\uc870\uc758 \ub538\ub9bc\ud654\uc74c\uc774 \uc544\ub2c8\ub77c C\uc7a5\uc870\uc758 \uc81c3\uc74c\uc744 \ud558\uae30 \uc2dc\uc791\ud55c\ub2e4. \uc774 \ud2b8\ub808\ubab0\ub85c\ub97c \ubc14\ud0d5\uc73c\ub85c \ud638\ub978\uacfc \ud604\uc774 \uc800\uc74c\ubd80\uc5d0\uc11c \uc81c1\uc8fc\uc81c\ub97c \uc81c\uc2dc\ud55c\ub2e4. \uc774 \uc8fc\uc81c\ub3d9\uae30\uac00 C\uc74c\uc758 \uc9c0\uc18d\uc800\uc74c \uc704\uc5d0\uc11c \ub300\ub2f4\ud55c \ubd88\ud611\ud654\uc74c\uc744 \uc774\ub8e8\uba74\uc11c \uc81c5\ub3c4\uc74c\uc744 \ud558\ud589\uc2dc\ud0a8 G\ub2e8\uc870\uc640 B\u266d\ub2e8\uc870\uc801\uc778 \uc6b8\ub9bc\uc744 \uac70\uccd0 \uac15\uc870\ub418\uba74, 78\ub9c8\ub514\ubd80\ud130 \ud22c\ud2f0\uc758 ff\ub85c \uba85\ud655\ud55c \ud615\ud0dc\uc758 \uc8fc\uc81c\uac00 \ud655\ubcf4\ub41c\ub2e4. \uc8fc\uc81c\uc758 \ud6c4\ubc18\uc740 \ud2b9\uc815\uc801\uc778 \ub9ac\ub4ec\uc744 \uac00\uc9c4 \ub3d9\uae30\ub97c \uc911\uc2ec\uc73c\ub85c \ud06c\uac8c \ubc1c\uc804\ub41c\ub2e4. \ud558\ud589 \uc74c\uacc4\uc758 \uc131\uaca9\uc744 \uac00\uc9c4 \uacbd\uacfc\uc74c\ud615\uc774 \uc810\ucc28 \ud655\uc2e4\ud55c \ud615\ud0dc\uac00 \ub418\uc5b4 110\ub9c8\ub514\ubd80\ud130 ff\ub85c \ub098\ud0c0\ub098\uba74, 112\ub9c8\ub514\uc5d0 G\uc74c\uc758 \uc9c0\uc18d\uc800\uc74c\uc774 \ub193\uc774\uace0 \uadf8 \uc704\uc5d0 \ubc14\uc774\uc62c\ub9b0\uc774 \uc81c2\uc8fc\uc81c\ub97c \uc81c\uc2dc\ud55c\ub2e4. \uc774\ubbf8 \uc554\uc2dc\ub41c \ud558\ud589\uc74c\ud615 \ub3d9\uae30\uac00 \uc644\uc804\ud558\uac8c \ubc1c\uc804\ub41c\ub2e4. \uadf8 \uc18d\uc5d0 \uc5d0\ud54f\ub4dc\uc640 \uac19\uc740 \uc545\uad6c\uac00 \ucc98\uc74c\uc5d0\ub294 \ud50c\ub8e8\ud2b8 \uc774\uc5b4\uc11c \ubc14\uc774\uc62c\ub9b0\uc73c\ub85c \ub098\ud0c0\ub098\uace0, \ub2e4\uc2dc \uc81c2\uc8fc\uc81c\uc758 \ubc1c\uc804\uc801 \uacbd\uacfc\uad6c\uac00 \ub418\uc5b4 133\ub9c8\ub514\uc5d0\uc11c \ud22c\ud2f0\uc758 \ud655\ubcf4\ubd80\uc5d0 \uc774\ub978\ub2e4.", "sentence_answer": "A\ub2e8\uc870\uc758 \ub538\ub9bc\ud654\uc74c\uc744 \uc774\uc5b4\ubc1b\uc544 \ubc14\uc774\uc62c\ub9b0 \uc774 E\uc74c\uc744 \ud2b8\ub808\ubab0\ub85c\ub85c \uc5f0\uc8fc\ud558\uc5ec \uc2dc\uc791\ub41c\ub2e4.", "paragraph_id": "6479950-4-1", "paragraph_question": "question: \uc774 \uace1\uc5d0\uc11c A\ub2e8\uc870\uc758 \ub538\ub9bc\ud654\uc74c \ud6c4 E\uc74c\uc744 \ud2b8\ub808\ubab0\ub85c\ub85c \uc5f0\uc8fc\ud558\ub294 \uc545\uae30\ub294?, context: \uc54c\ub808\uadf8\ub85c \ucf58 \ube0c\ub77c\uc624\uac00 \ub418\ub294 55\ub9c8\ub514\ubd80\ud130 \uc81c\uc2dc\ubd80\uac00 \uc2dc\uc791\ub41c\ub2e4. A\ub2e8\uc870\uc758 \ub538\ub9bc\ud654\uc74c\uc744 \uc774\uc5b4\ubc1b\uc544 \ubc14\uc774\uc62c\ub9b0\uc774 E\uc74c\uc744 \ud2b8\ub808\ubab0\ub85c\ub85c \uc5f0\uc8fc\ud558\uc5ec \uc2dc\uc791\ub41c\ub2e4. \uc774\uac83\uc740 A\ub2e8\uc870\uc758 \ub538\ub9bc\ud654\uc74c\uc774 \uc544\ub2c8\ub77c C\uc7a5\uc870\uc758 \uc81c3\uc74c\uc744 \ud558\uae30 \uc2dc\uc791\ud55c\ub2e4. \uc774 \ud2b8\ub808\ubab0\ub85c\ub97c \ubc14\ud0d5\uc73c\ub85c \ud638\ub978\uacfc \ud604\uc774 \uc800\uc74c\ubd80\uc5d0\uc11c \uc81c1\uc8fc\uc81c\ub97c \uc81c\uc2dc\ud55c\ub2e4. \uc774 \uc8fc\uc81c\ub3d9\uae30\uac00 C\uc74c\uc758 \uc9c0\uc18d\uc800\uc74c \uc704\uc5d0\uc11c \ub300\ub2f4\ud55c \ubd88\ud611\ud654\uc74c\uc744 \uc774\ub8e8\uba74\uc11c \uc81c5\ub3c4\uc74c\uc744 \ud558\ud589\uc2dc\ud0a8 G\ub2e8\uc870\uc640 B\u266d\ub2e8\uc870\uc801\uc778 \uc6b8\ub9bc\uc744 \uac70\uccd0 \uac15\uc870\ub418\uba74, 78\ub9c8\ub514\ubd80\ud130 \ud22c\ud2f0\uc758 ff\ub85c \uba85\ud655\ud55c \ud615\ud0dc\uc758 \uc8fc\uc81c\uac00 \ud655\ubcf4\ub41c\ub2e4. \uc8fc\uc81c\uc758 \ud6c4\ubc18\uc740 \ud2b9\uc815\uc801\uc778 \ub9ac\ub4ec\uc744 \uac00\uc9c4 \ub3d9\uae30\ub97c \uc911\uc2ec\uc73c\ub85c \ud06c\uac8c \ubc1c\uc804\ub41c\ub2e4. \ud558\ud589 \uc74c\uacc4\uc758 \uc131\uaca9\uc744 \uac00\uc9c4 \uacbd\uacfc\uc74c\ud615\uc774 \uc810\ucc28 \ud655\uc2e4\ud55c \ud615\ud0dc\uac00 \ub418\uc5b4 110\ub9c8\ub514\ubd80\ud130 ff\ub85c \ub098\ud0c0\ub098\uba74, 112\ub9c8\ub514\uc5d0 G\uc74c\uc758 \uc9c0\uc18d\uc800\uc74c\uc774 \ub193\uc774\uace0 \uadf8 \uc704\uc5d0 \ubc14\uc774\uc62c\ub9b0\uc774 \uc81c2\uc8fc\uc81c\ub97c \uc81c\uc2dc\ud55c\ub2e4. \uc774\ubbf8 \uc554\uc2dc\ub41c \ud558\ud589\uc74c\ud615 \ub3d9\uae30\uac00 \uc644\uc804\ud558\uac8c \ubc1c\uc804\ub41c\ub2e4. \uadf8 \uc18d\uc5d0 \uc5d0\ud54f\ub4dc\uc640 \uac19\uc740 \uc545\uad6c\uac00 \ucc98\uc74c\uc5d0\ub294 \ud50c\ub8e8\ud2b8 \uc774\uc5b4\uc11c \ubc14\uc774\uc62c\ub9b0\uc73c\ub85c \ub098\ud0c0\ub098\uace0, \ub2e4\uc2dc \uc81c2\uc8fc\uc81c\uc758 \ubc1c\uc804\uc801 \uacbd\uacfc\uad6c\uac00 \ub418\uc5b4 133\ub9c8\ub514\uc5d0\uc11c \ud22c\ud2f0\uc758 \ud655\ubcf4\ubd80\uc5d0 \uc774\ub978\ub2e4."} {"question": "\ub098\uc7a0 \uc218\ub294 \ubb34\uc2a8 \uadf8\ub8f9\uc758 \uba64\ubc84\uc778\uac00?", "paragraph": "\uc7a5\uae30\ud558\uc758 \ub9d0\uc5d0 \ub530\ub974\uba74, \ub300\ubd80\ubd84\uc758 \ub179\uc74c \uc791\uc5c5\uc740 \uc11c\uc6b8\ud2b9\ubcc4\uc2dc \uc465\uace0\uac1c\uc5d0 \uc704\uce58\ud55c \uc220\ud0c4 \uc624\ube0c \ub354 \ub514\uc2a4\ucf54 \uba64\ubc84 \ub098\uc7a0 \uc218\uc758 \ubc29\uc5d0\uc11c \uc9c4\ud589\ub418\uc5c8\uace0 \uc608\uc678\uc801\uc73c\ub85c \ud1a4 \uc2a4\ud29c\ub514\uc624\uc5d0\uc11c \ub4dc\ub7fc \uc18c\uc2a4\uc640 \uc77c\ubd80 \ubcf4\uceec \uc18c\uc2a4\ub85c \uba87 \uace1\uc744 \uc791\uc5c5\ud588\ub2e4\uace0 \ud55c\ub2e4. \ub4dc\ubb38\ub4dc\ubb38, \uadf8\ub9ac\uace0 \uc7a5\uae30\ud558\uc758 \ud45c\ud604\ub300\ub85c \ud6c4\ubc18\ubd80\uc5d0\uc11c '\ube61\uc138\uac8c' \uc774\ub904\uc9c4 \ub179\uc74c\uc740 \ub300\ub7b5 \ubc18\ub144 \uc815\ub3c4\uc758 \uc2dc\uac04\uc774 \uac78\ub838\ub2e4. \ud648 \ub808\ucf54\ub529\uc744 \ud558\ub294 \uc774\uc720\uc5d0 \ub300\ud574\uc11c \uc7a5\uae30\ud558\ub294 \"\ud2b9\ubcc4\ud788 \ubc29\uc758 \uc5b4\ub5a4 \ud2b9\uc131 \ub54c\ubb38\uc740 \uc544\ub2c8\"\uba70 \"\uc544\ubb34\ub798\ub3c4 \ub204\uad70\uac00\uc640 \uc788\uac70\ub098 \uc0ac\ub78c\uc774 \ub9ce\uc73c\uba74 \uc2e0\uacbd \uc4f0\uc5ec\uc11c.\"\ub77c\uace0 \uc124\uba85\ud588\ub2e4. \ub610\ud55c \ud504\ub85c\ub4c0\uc11c\ub85c \ud611\uc5c5\ud55c \ub098\uc7a0 \uc218\uc5d0 \ub300\ud574\uc11c\ub294 \"... \ub9d0\uc774 \uad49\uc7a5\ud788 \uc798 \ud1b5\ud588\ub2e4. \ub0b4\uac00 \uc774\ub7f0 \uc810\uc774 \ubc14\ub00c\uc5c8\uc73c\uba74 \uc88b\uaca0\ub2e4\uace0 \ud558\uba74 \uadf8\uac78 \uae08\ubc29 \uce90\uce58\ud55c\ub2e4. \ub098\ub294 \ud504\ub85c\uadf8\ub7a8\uc774\ub098 \uc774\ub7f0 \uac78 \uc870\uc791\ud558\ub294 \uac78 \ubaa8\ub974\ub2c8\uae4c \ucd94\uc0c1\uc801\uc73c\ub85c '\ubb34\uc5b8\uac00 \ube44\uc5b4\uc788\ub294 \ub290\ub08c\uc774\ub2e4'\ub77c\ub294 \uc2dd\uc73c\ub85c \ub9d0\uc744 \ud558\uba74 \ub098\uc7a0\uc218\ub294 \uadf8\uac8c \uc774\ub860\uc801\uc73c\ub85c \uc5b4\ub5a4 \ubd80\ubd84\uc744 \uac74\ub4dc\ub824\uc57c \ud558\ub294\uc9c0 \ucc3e\uc544\uc11c \uc218\uc815\ud588\ub2e4. \ud53c\ub4dc\ubc31\uc774 \uad49\uc7a5\ud788 \ube68\ub790\ub2e4.\"\uace0 \ub9d0\ud588\ub2e4.", "answer": "\uc220\ud0c4 \uc624\ube0c \ub354 \ub514\uc2a4\ucf54", "sentence": "\ub300\ubd80\ubd84\uc758 \ub179\uc74c \uc791\uc5c5\uc740 \uc11c\uc6b8\ud2b9\ubcc4\uc2dc \uc465\uace0\uac1c\uc5d0 \uc704\uce58\ud55c \uc220\ud0c4 \uc624\ube0c \ub354 \ub514\uc2a4\ucf54 \uba64\ubc84 \ub098\uc7a0 \uc218\uc758 \ubc29\uc5d0\uc11c \uc9c4\ud589\ub418\uc5c8\uace0 \uc608\uc678\uc801\uc73c\ub85c \ud1a4 \uc2a4\ud29c\ub514\uc624\uc5d0\uc11c \ub4dc\ub7fc \uc18c\uc2a4\uc640 \uc77c\ubd80 \ubcf4\uceec \uc18c\uc2a4\ub85c \uba87 \uace1\uc744 \uc791\uc5c5\ud588\ub2e4\uace0 \ud55c\ub2e4.", "paragraph_sentence": "\uc7a5\uae30\ud558\uc758 \ub9d0\uc5d0 \ub530\ub974\uba74, \ub300\ubd80\ubd84\uc758 \ub179\uc74c \uc791\uc5c5\uc740 \uc11c\uc6b8\ud2b9\ubcc4\uc2dc \uc465\uace0\uac1c\uc5d0 \uc704\uce58\ud55c \uc220\ud0c4 \uc624\ube0c \ub354 \ub514\uc2a4\ucf54 \uba64\ubc84 \ub098\uc7a0 \uc218\uc758 \ubc29\uc5d0\uc11c \uc9c4\ud589\ub418\uc5c8\uace0 \uc608\uc678\uc801\uc73c\ub85c \ud1a4 \uc2a4\ud29c\ub514\uc624\uc5d0\uc11c \ub4dc\ub7fc \uc18c\uc2a4\uc640 \uc77c\ubd80 \ubcf4\uceec \uc18c\uc2a4\ub85c \uba87 \uace1\uc744 \uc791\uc5c5\ud588\ub2e4\uace0 \ud55c\ub2e4. \ub4dc\ubb38\ub4dc\ubb38, \uadf8\ub9ac\uace0 \uc7a5\uae30\ud558\uc758 \ud45c\ud604\ub300\ub85c \ud6c4\ubc18\ubd80\uc5d0\uc11c '\ube61\uc138\uac8c' \uc774\ub904\uc9c4 \ub179\uc74c\uc740 \ub300\ub7b5 \ubc18\ub144 \uc815\ub3c4\uc758 \uc2dc\uac04\uc774 \uac78\ub838\ub2e4. \ud648 \ub808\ucf54\ub529\uc744 \ud558\ub294 \uc774\uc720\uc5d0 \ub300\ud574\uc11c \uc7a5\uae30\ud558\ub294 \"\ud2b9\ubcc4\ud788 \ubc29\uc758 \uc5b4\ub5a4 \ud2b9\uc131 \ub54c\ubb38\uc740 \uc544\ub2c8\"\uba70 \"\uc544\ubb34\ub798\ub3c4 \ub204\uad70\uac00\uc640 \uc788\uac70\ub098 \uc0ac\ub78c\uc774 \ub9ce\uc73c\uba74 \uc2e0\uacbd \uc4f0\uc5ec\uc11c. \"\ub77c\uace0 \uc124\uba85\ud588\ub2e4. \ub610\ud55c \ud504\ub85c\ub4c0\uc11c\ub85c \ud611\uc5c5\ud55c \ub098\uc7a0 \uc218\uc5d0 \ub300\ud574\uc11c\ub294 \"... \ub9d0\uc774 \uad49\uc7a5\ud788 \uc798 \ud1b5\ud588\ub2e4. \ub0b4\uac00 \uc774\ub7f0 \uc810\uc774 \ubc14\ub00c\uc5c8\uc73c\uba74 \uc88b\uaca0\ub2e4\uace0 \ud558\uba74 \uadf8\uac78 \uae08\ubc29 \uce90\uce58\ud55c\ub2e4. \ub098\ub294 \ud504\ub85c\uadf8\ub7a8\uc774\ub098 \uc774\ub7f0 \uac78 \uc870\uc791\ud558\ub294 \uac78 \ubaa8\ub974\ub2c8\uae4c \ucd94\uc0c1\uc801\uc73c\ub85c '\ubb34\uc5b8\uac00 \ube44\uc5b4\uc788\ub294 \ub290\ub08c\uc774\ub2e4'\ub77c\ub294 \uc2dd\uc73c\ub85c \ub9d0\uc744 \ud558\uba74 \ub098\uc7a0\uc218\ub294 \uadf8\uac8c \uc774\ub860\uc801\uc73c\ub85c \uc5b4\ub5a4 \ubd80\ubd84\uc744 \uac74\ub4dc\ub824\uc57c \ud558\ub294\uc9c0 \ucc3e\uc544\uc11c \uc218\uc815\ud588\ub2e4. \ud53c\ub4dc\ubc31\uc774 \uad49\uc7a5\ud788 \ube68\ub790\ub2e4. \"\uace0 \ub9d0\ud588\ub2e4.", "paragraph_answer": "\uc7a5\uae30\ud558\uc758 \ub9d0\uc5d0 \ub530\ub974\uba74, \ub300\ubd80\ubd84\uc758 \ub179\uc74c \uc791\uc5c5\uc740 \uc11c\uc6b8\ud2b9\ubcc4\uc2dc \uc465\uace0\uac1c\uc5d0 \uc704\uce58\ud55c \uc220\ud0c4 \uc624\ube0c \ub354 \ub514\uc2a4\ucf54 \uba64\ubc84 \ub098\uc7a0 \uc218\uc758 \ubc29\uc5d0\uc11c \uc9c4\ud589\ub418\uc5c8\uace0 \uc608\uc678\uc801\uc73c\ub85c \ud1a4 \uc2a4\ud29c\ub514\uc624\uc5d0\uc11c \ub4dc\ub7fc \uc18c\uc2a4\uc640 \uc77c\ubd80 \ubcf4\uceec \uc18c\uc2a4\ub85c \uba87 \uace1\uc744 \uc791\uc5c5\ud588\ub2e4\uace0 \ud55c\ub2e4. \ub4dc\ubb38\ub4dc\ubb38, \uadf8\ub9ac\uace0 \uc7a5\uae30\ud558\uc758 \ud45c\ud604\ub300\ub85c \ud6c4\ubc18\ubd80\uc5d0\uc11c '\ube61\uc138\uac8c' \uc774\ub904\uc9c4 \ub179\uc74c\uc740 \ub300\ub7b5 \ubc18\ub144 \uc815\ub3c4\uc758 \uc2dc\uac04\uc774 \uac78\ub838\ub2e4. \ud648 \ub808\ucf54\ub529\uc744 \ud558\ub294 \uc774\uc720\uc5d0 \ub300\ud574\uc11c \uc7a5\uae30\ud558\ub294 \"\ud2b9\ubcc4\ud788 \ubc29\uc758 \uc5b4\ub5a4 \ud2b9\uc131 \ub54c\ubb38\uc740 \uc544\ub2c8\"\uba70 \"\uc544\ubb34\ub798\ub3c4 \ub204\uad70\uac00\uc640 \uc788\uac70\ub098 \uc0ac\ub78c\uc774 \ub9ce\uc73c\uba74 \uc2e0\uacbd \uc4f0\uc5ec\uc11c.\"\ub77c\uace0 \uc124\uba85\ud588\ub2e4. \ub610\ud55c \ud504\ub85c\ub4c0\uc11c\ub85c \ud611\uc5c5\ud55c \ub098\uc7a0 \uc218\uc5d0 \ub300\ud574\uc11c\ub294 \"... \ub9d0\uc774 \uad49\uc7a5\ud788 \uc798 \ud1b5\ud588\ub2e4. \ub0b4\uac00 \uc774\ub7f0 \uc810\uc774 \ubc14\ub00c\uc5c8\uc73c\uba74 \uc88b\uaca0\ub2e4\uace0 \ud558\uba74 \uadf8\uac78 \uae08\ubc29 \uce90\uce58\ud55c\ub2e4. \ub098\ub294 \ud504\ub85c\uadf8\ub7a8\uc774\ub098 \uc774\ub7f0 \uac78 \uc870\uc791\ud558\ub294 \uac78 \ubaa8\ub974\ub2c8\uae4c \ucd94\uc0c1\uc801\uc73c\ub85c '\ubb34\uc5b8\uac00 \ube44\uc5b4\uc788\ub294 \ub290\ub08c\uc774\ub2e4'\ub77c\ub294 \uc2dd\uc73c\ub85c \ub9d0\uc744 \ud558\uba74 \ub098\uc7a0\uc218\ub294 \uadf8\uac8c \uc774\ub860\uc801\uc73c\ub85c \uc5b4\ub5a4 \ubd80\ubd84\uc744 \uac74\ub4dc\ub824\uc57c \ud558\ub294\uc9c0 \ucc3e\uc544\uc11c \uc218\uc815\ud588\ub2e4. \ud53c\ub4dc\ubc31\uc774 \uad49\uc7a5\ud788 \ube68\ub790\ub2e4.\"\uace0 \ub9d0\ud588\ub2e4.", "sentence_answer": "\ub300\ubd80\ubd84\uc758 \ub179\uc74c \uc791\uc5c5\uc740 \uc11c\uc6b8\ud2b9\ubcc4\uc2dc \uc465\uace0\uac1c\uc5d0 \uc704\uce58\ud55c \uc220\ud0c4 \uc624\ube0c \ub354 \ub514\uc2a4\ucf54 \uba64\ubc84 \ub098\uc7a0 \uc218\uc758 \ubc29\uc5d0\uc11c \uc9c4\ud589\ub418\uc5c8\uace0 \uc608\uc678\uc801\uc73c\ub85c \ud1a4 \uc2a4\ud29c\ub514\uc624\uc5d0\uc11c \ub4dc\ub7fc \uc18c\uc2a4\uc640 \uc77c\ubd80 \ubcf4\uceec \uc18c\uc2a4\ub85c \uba87 \uace1\uc744 \uc791\uc5c5\ud588\ub2e4\uace0 \ud55c\ub2e4.", "paragraph_id": "6657153-5-0", "paragraph_question": "question: \ub098\uc7a0 \uc218\ub294 \ubb34\uc2a8 \uadf8\ub8f9\uc758 \uba64\ubc84\uc778\uac00?, context: \uc7a5\uae30\ud558\uc758 \ub9d0\uc5d0 \ub530\ub974\uba74, \ub300\ubd80\ubd84\uc758 \ub179\uc74c \uc791\uc5c5\uc740 \uc11c\uc6b8\ud2b9\ubcc4\uc2dc \uc465\uace0\uac1c\uc5d0 \uc704\uce58\ud55c \uc220\ud0c4 \uc624\ube0c \ub354 \ub514\uc2a4\ucf54 \uba64\ubc84 \ub098\uc7a0 \uc218\uc758 \ubc29\uc5d0\uc11c \uc9c4\ud589\ub418\uc5c8\uace0 \uc608\uc678\uc801\uc73c\ub85c \ud1a4 \uc2a4\ud29c\ub514\uc624\uc5d0\uc11c \ub4dc\ub7fc \uc18c\uc2a4\uc640 \uc77c\ubd80 \ubcf4\uceec \uc18c\uc2a4\ub85c \uba87 \uace1\uc744 \uc791\uc5c5\ud588\ub2e4\uace0 \ud55c\ub2e4. \ub4dc\ubb38\ub4dc\ubb38, \uadf8\ub9ac\uace0 \uc7a5\uae30\ud558\uc758 \ud45c\ud604\ub300\ub85c \ud6c4\ubc18\ubd80\uc5d0\uc11c '\ube61\uc138\uac8c' \uc774\ub904\uc9c4 \ub179\uc74c\uc740 \ub300\ub7b5 \ubc18\ub144 \uc815\ub3c4\uc758 \uc2dc\uac04\uc774 \uac78\ub838\ub2e4. \ud648 \ub808\ucf54\ub529\uc744 \ud558\ub294 \uc774\uc720\uc5d0 \ub300\ud574\uc11c \uc7a5\uae30\ud558\ub294 \"\ud2b9\ubcc4\ud788 \ubc29\uc758 \uc5b4\ub5a4 \ud2b9\uc131 \ub54c\ubb38\uc740 \uc544\ub2c8\"\uba70 \"\uc544\ubb34\ub798\ub3c4 \ub204\uad70\uac00\uc640 \uc788\uac70\ub098 \uc0ac\ub78c\uc774 \ub9ce\uc73c\uba74 \uc2e0\uacbd \uc4f0\uc5ec\uc11c.\"\ub77c\uace0 \uc124\uba85\ud588\ub2e4. \ub610\ud55c \ud504\ub85c\ub4c0\uc11c\ub85c \ud611\uc5c5\ud55c \ub098\uc7a0 \uc218\uc5d0 \ub300\ud574\uc11c\ub294 \"... \ub9d0\uc774 \uad49\uc7a5\ud788 \uc798 \ud1b5\ud588\ub2e4. \ub0b4\uac00 \uc774\ub7f0 \uc810\uc774 \ubc14\ub00c\uc5c8\uc73c\uba74 \uc88b\uaca0\ub2e4\uace0 \ud558\uba74 \uadf8\uac78 \uae08\ubc29 \uce90\uce58\ud55c\ub2e4. \ub098\ub294 \ud504\ub85c\uadf8\ub7a8\uc774\ub098 \uc774\ub7f0 \uac78 \uc870\uc791\ud558\ub294 \uac78 \ubaa8\ub974\ub2c8\uae4c \ucd94\uc0c1\uc801\uc73c\ub85c '\ubb34\uc5b8\uac00 \ube44\uc5b4\uc788\ub294 \ub290\ub08c\uc774\ub2e4'\ub77c\ub294 \uc2dd\uc73c\ub85c \ub9d0\uc744 \ud558\uba74 \ub098\uc7a0\uc218\ub294 \uadf8\uac8c \uc774\ub860\uc801\uc73c\ub85c \uc5b4\ub5a4 \ubd80\ubd84\uc744 \uac74\ub4dc\ub824\uc57c \ud558\ub294\uc9c0 \ucc3e\uc544\uc11c \uc218\uc815\ud588\ub2e4. \ud53c\ub4dc\ubc31\uc774 \uad49\uc7a5\ud788 \ube68\ub790\ub2e4.\"\uace0 \ub9d0\ud588\ub2e4."} {"question": "\uc9e7\uc740 \uc1fc\ud2b8\uc758 \uc5f0\uc18d\uc73c\ub85c \ub41c \ucd2c\uc601 \uc6a9\ubc95\uc740 \ubb34\uc5c7\uc778\uac00?", "paragraph": "\uc774\uc640 \uac19\uc740 \ub3c5\uc77c\uc758 \uc0c8\ub85c\uc6b4 \uc601\ud654 \uc6b4\ub3d9\uc5d0 \ub300\ud558\uc5ec \uc9c0\ud06c\ud504\ub9ac\ud2b8 \ud06c\ub77c\uce74\uc6cc\ub294 \ud504\ub791\uc2a4\uc758 \uc2e0\uc9c4 \uc601\ud654\uc791\uac00\ub4e4\uacfc \ub73b\uc744 \ubaa8\uc544\uc11c, 1920\ub144\ub300\uc758 \uc544\ubc29\uac00\ub974\ub4dc \uc601\ud654\uc758 \ud575\uc2ec\uc744 \ud615\uc131\ud558\uae30\uc5d0 \uc774\ub974\ub800\ub2e4 \ud558\uc5ec \uadf8 \ubbf8\ud559\uc801(\u7f8e\u5b78\u7684) \ubd84\uc11d\uc744 \ub2e4\uc74c\uacfc \uac19\uc774 \uc694\uc57d\ud558\uace0 \uc788\ub2e4. \uc774 \uc2dc\ub300\uc758 \uc601\ud654\uc758 \ud45c\ud604\ub825\uc740, \uc2a4\ud1a0\ub9ac\ub97c \ud615\uc131\ud558\uae30 \uc704\ud574\uc11c\ub294 \uc5f0\uadf9\uc774\ub098 \ubb38\ud559\uc5d0 \uba40\ub9ac \ubbf8\uce58\uc9c0 \ubabb\ud55c \uac83\uc774 \ud070 \uc57d\uc810\uc774\uc5c8\uc73c\ubbc0\ub85c \uc2a4\ud1a0\ub9ac\ub97c \uadf9\ub3c4\ub85c \uc0c9\ub2e4\ub978 \uc18c\uc7ac\ub85c \ub9cc\ub4e4\ub4e0\uc9c0, \ud639\uc740 \uc804\ud600 \uc2a4\ud1a0\ub9ac\uac00 \uc5c6\ub294 \uc21c\uc218\uc601\ud654\ub97c \uc9c0\ud5a5\ud588\ub2e4. \uc774\ub9ac\ud558\uc5ec \uc601\uc0c1\uc758 \uc21c\uc218\ud45c\ud604\uc5d0 \ub530\ub978 \uc601\ud654\uc5b8\uc5b4(\u6620\u7575\u8a00\u8a9e)\uc758 \ud655\ub300\ub97c \ud0d0\uad6c\ud588\ub2e4. \uadf9\ub2e8\uc801\uc778 \uce74\uba54\ub77c \uc575\uae00, \uc9e7\uc740 \uc1fc\ud2b8\uc758 \uc5f0\uc18d\uc73c\ub85c \ub41c \ucee4\ud305 \uc18c\ud504\ud2b8 \ud3ec\ucee4\uc2a4 \ucd2c\uc601, \uadf8 \ubc16\uc758 \uc0c8\ub85c\uc6b4 \uc6a9\ubc95\uc774\ub77c\ub4e0\uac00 \uadf8\ub85c\ubd80\ud130 \uc2ec\ub9ac\uc801\uc778 \ub9ac\uc5bc\ub9ac\ud2f0\uac00 \ucd94\uad6c\ub418\uc5c8\ub2e4.", "answer": "\ucee4\ud305 \uc18c\ud504\ud2b8 \ud3ec\ucee4\uc2a4 \ucd2c\uc601", "sentence": "\uadf9\ub2e8\uc801\uc778 \uce74\uba54\ub77c \uc575\uae00, \uc9e7\uc740 \uc1fc\ud2b8\uc758 \uc5f0\uc18d\uc73c\ub85c \ub41c \ucee4\ud305 \uc18c\ud504\ud2b8 \ud3ec\ucee4\uc2a4 \ucd2c\uc601 ,", "paragraph_sentence": "\uc774\uc640 \uac19\uc740 \ub3c5\uc77c\uc758 \uc0c8\ub85c\uc6b4 \uc601\ud654 \uc6b4\ub3d9\uc5d0 \ub300\ud558\uc5ec \uc9c0\ud06c\ud504\ub9ac\ud2b8 \ud06c\ub77c\uce74\uc6cc\ub294 \ud504\ub791\uc2a4\uc758 \uc2e0\uc9c4 \uc601\ud654\uc791\uac00\ub4e4\uacfc \ub73b\uc744 \ubaa8\uc544\uc11c, 1920\ub144\ub300\uc758 \uc544\ubc29\uac00\ub974\ub4dc \uc601\ud654\uc758 \ud575\uc2ec\uc744 \ud615\uc131\ud558\uae30\uc5d0 \uc774\ub974\ub800\ub2e4 \ud558\uc5ec \uadf8 \ubbf8\ud559\uc801(\u7f8e\u5b78\u7684) \ubd84\uc11d\uc744 \ub2e4\uc74c\uacfc \uac19\uc774 \uc694\uc57d\ud558\uace0 \uc788\ub2e4. \uc774 \uc2dc\ub300\uc758 \uc601\ud654\uc758 \ud45c\ud604\ub825\uc740, \uc2a4\ud1a0\ub9ac\ub97c \ud615\uc131\ud558\uae30 \uc704\ud574\uc11c\ub294 \uc5f0\uadf9\uc774\ub098 \ubb38\ud559\uc5d0 \uba40\ub9ac \ubbf8\uce58\uc9c0 \ubabb\ud55c \uac83\uc774 \ud070 \uc57d\uc810\uc774\uc5c8\uc73c\ubbc0\ub85c \uc2a4\ud1a0\ub9ac\ub97c \uadf9\ub3c4\ub85c \uc0c9\ub2e4\ub978 \uc18c\uc7ac\ub85c \ub9cc\ub4e4\ub4e0\uc9c0, \ud639\uc740 \uc804\ud600 \uc2a4\ud1a0\ub9ac\uac00 \uc5c6\ub294 \uc21c\uc218\uc601\ud654\ub97c \uc9c0\ud5a5\ud588\ub2e4. \uc774\ub9ac\ud558\uc5ec \uc601\uc0c1\uc758 \uc21c\uc218\ud45c\ud604\uc5d0 \ub530\ub978 \uc601\ud654\uc5b8\uc5b4(\u6620\u7575\u8a00\u8a9e)\uc758 \ud655\ub300\ub97c \ud0d0\uad6c\ud588\ub2e4. \uadf9\ub2e8\uc801\uc778 \uce74\uba54\ub77c \uc575\uae00, \uc9e7\uc740 \uc1fc\ud2b8\uc758 \uc5f0\uc18d\uc73c\ub85c \ub41c \ucee4\ud305 \uc18c\ud504\ud2b8 \ud3ec\ucee4\uc2a4 \ucd2c\uc601 , \uadf8 \ubc16\uc758 \uc0c8\ub85c\uc6b4 \uc6a9\ubc95\uc774\ub77c\ub4e0\uac00 \uadf8\ub85c\ubd80\ud130 \uc2ec\ub9ac\uc801\uc778 \ub9ac\uc5bc\ub9ac\ud2f0\uac00 \ucd94\uad6c\ub418\uc5c8\ub2e4.", "paragraph_answer": "\uc774\uc640 \uac19\uc740 \ub3c5\uc77c\uc758 \uc0c8\ub85c\uc6b4 \uc601\ud654 \uc6b4\ub3d9\uc5d0 \ub300\ud558\uc5ec \uc9c0\ud06c\ud504\ub9ac\ud2b8 \ud06c\ub77c\uce74\uc6cc\ub294 \ud504\ub791\uc2a4\uc758 \uc2e0\uc9c4 \uc601\ud654\uc791\uac00\ub4e4\uacfc \ub73b\uc744 \ubaa8\uc544\uc11c, 1920\ub144\ub300\uc758 \uc544\ubc29\uac00\ub974\ub4dc \uc601\ud654\uc758 \ud575\uc2ec\uc744 \ud615\uc131\ud558\uae30\uc5d0 \uc774\ub974\ub800\ub2e4 \ud558\uc5ec \uadf8 \ubbf8\ud559\uc801(\u7f8e\u5b78\u7684) \ubd84\uc11d\uc744 \ub2e4\uc74c\uacfc \uac19\uc774 \uc694\uc57d\ud558\uace0 \uc788\ub2e4. \uc774 \uc2dc\ub300\uc758 \uc601\ud654\uc758 \ud45c\ud604\ub825\uc740, \uc2a4\ud1a0\ub9ac\ub97c \ud615\uc131\ud558\uae30 \uc704\ud574\uc11c\ub294 \uc5f0\uadf9\uc774\ub098 \ubb38\ud559\uc5d0 \uba40\ub9ac \ubbf8\uce58\uc9c0 \ubabb\ud55c \uac83\uc774 \ud070 \uc57d\uc810\uc774\uc5c8\uc73c\ubbc0\ub85c \uc2a4\ud1a0\ub9ac\ub97c \uadf9\ub3c4\ub85c \uc0c9\ub2e4\ub978 \uc18c\uc7ac\ub85c \ub9cc\ub4e4\ub4e0\uc9c0, \ud639\uc740 \uc804\ud600 \uc2a4\ud1a0\ub9ac\uac00 \uc5c6\ub294 \uc21c\uc218\uc601\ud654\ub97c \uc9c0\ud5a5\ud588\ub2e4. \uc774\ub9ac\ud558\uc5ec \uc601\uc0c1\uc758 \uc21c\uc218\ud45c\ud604\uc5d0 \ub530\ub978 \uc601\ud654\uc5b8\uc5b4(\u6620\u7575\u8a00\u8a9e)\uc758 \ud655\ub300\ub97c \ud0d0\uad6c\ud588\ub2e4. \uadf9\ub2e8\uc801\uc778 \uce74\uba54\ub77c \uc575\uae00, \uc9e7\uc740 \uc1fc\ud2b8\uc758 \uc5f0\uc18d\uc73c\ub85c \ub41c \ucee4\ud305 \uc18c\ud504\ud2b8 \ud3ec\ucee4\uc2a4 \ucd2c\uc601 , \uadf8 \ubc16\uc758 \uc0c8\ub85c\uc6b4 \uc6a9\ubc95\uc774\ub77c\ub4e0\uac00 \uadf8\ub85c\ubd80\ud130 \uc2ec\ub9ac\uc801\uc778 \ub9ac\uc5bc\ub9ac\ud2f0\uac00 \ucd94\uad6c\ub418\uc5c8\ub2e4.", "sentence_answer": "\uadf9\ub2e8\uc801\uc778 \uce74\uba54\ub77c \uc575\uae00, \uc9e7\uc740 \uc1fc\ud2b8\uc758 \uc5f0\uc18d\uc73c\ub85c \ub41c \ucee4\ud305 \uc18c\ud504\ud2b8 \ud3ec\ucee4\uc2a4 \ucd2c\uc601 ,", "paragraph_id": "6541198-3-0", "paragraph_question": "question: \uc9e7\uc740 \uc1fc\ud2b8\uc758 \uc5f0\uc18d\uc73c\ub85c \ub41c \ucd2c\uc601 \uc6a9\ubc95\uc740 \ubb34\uc5c7\uc778\uac00?, context: \uc774\uc640 \uac19\uc740 \ub3c5\uc77c\uc758 \uc0c8\ub85c\uc6b4 \uc601\ud654 \uc6b4\ub3d9\uc5d0 \ub300\ud558\uc5ec \uc9c0\ud06c\ud504\ub9ac\ud2b8 \ud06c\ub77c\uce74\uc6cc\ub294 \ud504\ub791\uc2a4\uc758 \uc2e0\uc9c4 \uc601\ud654\uc791\uac00\ub4e4\uacfc \ub73b\uc744 \ubaa8\uc544\uc11c, 1920\ub144\ub300\uc758 \uc544\ubc29\uac00\ub974\ub4dc \uc601\ud654\uc758 \ud575\uc2ec\uc744 \ud615\uc131\ud558\uae30\uc5d0 \uc774\ub974\ub800\ub2e4 \ud558\uc5ec \uadf8 \ubbf8\ud559\uc801(\u7f8e\u5b78\u7684) \ubd84\uc11d\uc744 \ub2e4\uc74c\uacfc \uac19\uc774 \uc694\uc57d\ud558\uace0 \uc788\ub2e4. \uc774 \uc2dc\ub300\uc758 \uc601\ud654\uc758 \ud45c\ud604\ub825\uc740, \uc2a4\ud1a0\ub9ac\ub97c \ud615\uc131\ud558\uae30 \uc704\ud574\uc11c\ub294 \uc5f0\uadf9\uc774\ub098 \ubb38\ud559\uc5d0 \uba40\ub9ac \ubbf8\uce58\uc9c0 \ubabb\ud55c \uac83\uc774 \ud070 \uc57d\uc810\uc774\uc5c8\uc73c\ubbc0\ub85c \uc2a4\ud1a0\ub9ac\ub97c \uadf9\ub3c4\ub85c \uc0c9\ub2e4\ub978 \uc18c\uc7ac\ub85c \ub9cc\ub4e4\ub4e0\uc9c0, \ud639\uc740 \uc804\ud600 \uc2a4\ud1a0\ub9ac\uac00 \uc5c6\ub294 \uc21c\uc218\uc601\ud654\ub97c \uc9c0\ud5a5\ud588\ub2e4. \uc774\ub9ac\ud558\uc5ec \uc601\uc0c1\uc758 \uc21c\uc218\ud45c\ud604\uc5d0 \ub530\ub978 \uc601\ud654\uc5b8\uc5b4(\u6620\u7575\u8a00\u8a9e)\uc758 \ud655\ub300\ub97c \ud0d0\uad6c\ud588\ub2e4. \uadf9\ub2e8\uc801\uc778 \uce74\uba54\ub77c \uc575\uae00, \uc9e7\uc740 \uc1fc\ud2b8\uc758 \uc5f0\uc18d\uc73c\ub85c \ub41c \ucee4\ud305 \uc18c\ud504\ud2b8 \ud3ec\ucee4\uc2a4 \ucd2c\uc601, \uadf8 \ubc16\uc758 \uc0c8\ub85c\uc6b4 \uc6a9\ubc95\uc774\ub77c\ub4e0\uac00 \uadf8\ub85c\ubd80\ud130 \uc2ec\ub9ac\uc801\uc778 \ub9ac\uc5bc\ub9ac\ud2f0\uac00 \ucd94\uad6c\ub418\uc5c8\ub2e4."} {"question": "\ud504\ub77c\uac00 \uc804\ud22c \uc804 \ub7ec\uc2dc\uc544\uad70\uc5d0\uac8c \uc57d\ud0c8\uacfc \ubc29\ud654\ub97c \uae08\uc9c0\ud55c \uc7a5\uad70\uc758 \uc774\ub984\uc740?", "paragraph": "\uc804\ud22c\uac00 \ub05d\ub09c \ud6c4 \ub7ec\uc2dc\uc544\uad70\uc740 \uc804\ud22c \uc804 \uc218\ubcf4\ub85c\ud504\uc5d0\uac8c \ubc1b\uc740 \uba85\ub839\uc744 \uc5b4\uae30\uace0 \ubc14\ub974\uc0e4\ubc14 \uc678\uac01\uc5d0\uc11c \uc57d\ud0c8\uacfc \ubc29\ud654\ub97c \ubc8c\uc774\uae30 \uc2dc\uc791\ud558\uc600\ub2e4.(\ub7ec\uc2dc\uc544 \ubcd1\uc0ac\ub4e4\uc740 1794\ub144 \ubc14\ub974\uc0e4\ubc14 \ubd09\uae30 \ub54c \ud3f4\ub780\ub4dc \uc778\ub4e4\uc774 \ubc14\ub974\uc0e4\ubc14\ub97c \uc9c0\ud0a4\ub358 \ub7ec\uc2dc\uc544 \uc7a5\ubcd1\ub4e4 2,000\uba85\uc744 \ud559\uc0b4\ud558\uace0 \uad6c\uae08\ud55c \uac83\uc5d0 \ub300\ud55c \ubcf5\uc218\ub77c\uace0 \uc8fc\uc7a5\ud558\uc600\ub2e4.) \uac70\uc758 \uc804\uc5ed\uc774 \uc57d\ud0c8\ub2f9\ud558\uace0 \ubd88\ud0d4\uc73c\uba70 \ud504\ub77c\uac00\uc758 \uac70\uc8fc\uc790 \ub300\ubd80\ubd84\uc740 \uc0b4\ud574\ub2f9\ud588\ub2e4. \uc774\ub54c \uc0b4\ud574\ub2f9\ud55c \uc0ac\ub78c\ub4e4\uc758 \uc22b\uc790\ub294 \ubd88\uba85\ud655\ud558\ub098, \uc801\uc5b4\ub3c4 2\ub9cc \uba85\uc5d0 \uc774\ub97c \uac83\uc73c\ub85c \ucd94\uc815\ub418\uace0 \uc788\ub2e4. \uc5ec\uc790\uc640 \uc544\uc774\ub4e4\ub3c4 \uc6a9\uc11c\uc5c6\uc774 \uc0b4\ud574\ub418\uc5c8\ub2e4. \uc218\ubcf4\ub85c\ud504\ub294 \ud6d7\ub0a0 \ub2e4\uc74c\uacfc \uac19\uc774 \uae30\ub85d\ud558\uc600\ub2e4. \"\ud504\ub77c\uac00 \uc804\uc5ed\uc5d0 \uc2dc\uccb4\ub4e4\uc774 \ub110\ubd80\ub7ec\uc84c\uace0, \ud53c\uac00 \uac15\ubb3c\uc744 \uc774\ub8e8\uc5c8\ub2e4.\" \ud1b5\uc81c\uac00 \ub418\uc9c0 \uc54a\uc558\ub358 \ucf54\uc0ac\ud06c \ubd80\ub300\uac00 \uc774 \ud559\uc0b4\uc5d0 \uc8fc\ub3c4\uc801\uc778 \uc5ed\ud560\uc744 \ud588\ub2e4\ub294 \ube44\ub09c\uc744 \ubc1b\uace0 \uc788\ub2e4.", "answer": "\uc218\ubcf4\ub85c\ud504", "sentence": "\uc804\ud22c\uac00 \ub05d\ub09c \ud6c4 \ub7ec\uc2dc\uc544\uad70\uc740 \uc804\ud22c \uc804 \uc218\ubcf4\ub85c\ud504 \uc5d0\uac8c \ubc1b\uc740 \uba85\ub839\uc744 \uc5b4\uae30\uace0 \ubc14\ub974\uc0e4\ubc14 \uc678\uac01\uc5d0\uc11c \uc57d\ud0c8\uacfc \ubc29\ud654\ub97c \ubc8c\uc774\uae30 \uc2dc\uc791\ud558\uc600\ub2e4.", "paragraph_sentence": " \uc804\ud22c\uac00 \ub05d\ub09c \ud6c4 \ub7ec\uc2dc\uc544\uad70\uc740 \uc804\ud22c \uc804 \uc218\ubcf4\ub85c\ud504 \uc5d0\uac8c \ubc1b\uc740 \uba85\ub839\uc744 \uc5b4\uae30\uace0 \ubc14\ub974\uc0e4\ubc14 \uc678\uac01\uc5d0\uc11c \uc57d\ud0c8\uacfc \ubc29\ud654\ub97c \ubc8c\uc774\uae30 \uc2dc\uc791\ud558\uc600\ub2e4. (\ub7ec\uc2dc\uc544 \ubcd1\uc0ac\ub4e4\uc740 1794\ub144 \ubc14\ub974\uc0e4\ubc14 \ubd09\uae30 \ub54c \ud3f4\ub780\ub4dc \uc778\ub4e4\uc774 \ubc14\ub974\uc0e4\ubc14\ub97c \uc9c0\ud0a4\ub358 \ub7ec\uc2dc\uc544 \uc7a5\ubcd1\ub4e4 2,000\uba85\uc744 \ud559\uc0b4\ud558\uace0 \uad6c\uae08\ud55c \uac83\uc5d0 \ub300\ud55c \ubcf5\uc218\ub77c\uace0 \uc8fc\uc7a5\ud558\uc600\ub2e4. ) \uac70\uc758 \uc804\uc5ed\uc774 \uc57d\ud0c8\ub2f9\ud558\uace0 \ubd88\ud0d4\uc73c\uba70 \ud504\ub77c\uac00\uc758 \uac70\uc8fc\uc790 \ub300\ubd80\ubd84\uc740 \uc0b4\ud574\ub2f9\ud588\ub2e4. \uc774\ub54c \uc0b4\ud574\ub2f9\ud55c \uc0ac\ub78c\ub4e4\uc758 \uc22b\uc790\ub294 \ubd88\uba85\ud655\ud558\ub098, \uc801\uc5b4\ub3c4 2\ub9cc \uba85\uc5d0 \uc774\ub97c \uac83\uc73c\ub85c \ucd94\uc815\ub418\uace0 \uc788\ub2e4. \uc5ec\uc790\uc640 \uc544\uc774\ub4e4\ub3c4 \uc6a9\uc11c\uc5c6\uc774 \uc0b4\ud574\ub418\uc5c8\ub2e4. \uc218\ubcf4\ub85c\ud504\ub294 \ud6d7\ub0a0 \ub2e4\uc74c\uacfc \uac19\uc774 \uae30\ub85d\ud558\uc600\ub2e4. \"\ud504\ub77c\uac00 \uc804\uc5ed\uc5d0 \uc2dc\uccb4\ub4e4\uc774 \ub110\ubd80\ub7ec\uc84c\uace0, \ud53c\uac00 \uac15\ubb3c\uc744 \uc774\ub8e8\uc5c8\ub2e4. \" \ud1b5\uc81c\uac00 \ub418\uc9c0 \uc54a\uc558\ub358 \ucf54\uc0ac\ud06c \ubd80\ub300\uac00 \uc774 \ud559\uc0b4\uc5d0 \uc8fc\ub3c4\uc801\uc778 \uc5ed\ud560\uc744 \ud588\ub2e4\ub294 \ube44\ub09c\uc744 \ubc1b\uace0 \uc788\ub2e4.", "paragraph_answer": "\uc804\ud22c\uac00 \ub05d\ub09c \ud6c4 \ub7ec\uc2dc\uc544\uad70\uc740 \uc804\ud22c \uc804 \uc218\ubcf4\ub85c\ud504 \uc5d0\uac8c \ubc1b\uc740 \uba85\ub839\uc744 \uc5b4\uae30\uace0 \ubc14\ub974\uc0e4\ubc14 \uc678\uac01\uc5d0\uc11c \uc57d\ud0c8\uacfc \ubc29\ud654\ub97c \ubc8c\uc774\uae30 \uc2dc\uc791\ud558\uc600\ub2e4.(\ub7ec\uc2dc\uc544 \ubcd1\uc0ac\ub4e4\uc740 1794\ub144 \ubc14\ub974\uc0e4\ubc14 \ubd09\uae30 \ub54c \ud3f4\ub780\ub4dc \uc778\ub4e4\uc774 \ubc14\ub974\uc0e4\ubc14\ub97c \uc9c0\ud0a4\ub358 \ub7ec\uc2dc\uc544 \uc7a5\ubcd1\ub4e4 2,000\uba85\uc744 \ud559\uc0b4\ud558\uace0 \uad6c\uae08\ud55c \uac83\uc5d0 \ub300\ud55c \ubcf5\uc218\ub77c\uace0 \uc8fc\uc7a5\ud558\uc600\ub2e4.) \uac70\uc758 \uc804\uc5ed\uc774 \uc57d\ud0c8\ub2f9\ud558\uace0 \ubd88\ud0d4\uc73c\uba70 \ud504\ub77c\uac00\uc758 \uac70\uc8fc\uc790 \ub300\ubd80\ubd84\uc740 \uc0b4\ud574\ub2f9\ud588\ub2e4. \uc774\ub54c \uc0b4\ud574\ub2f9\ud55c \uc0ac\ub78c\ub4e4\uc758 \uc22b\uc790\ub294 \ubd88\uba85\ud655\ud558\ub098, \uc801\uc5b4\ub3c4 2\ub9cc \uba85\uc5d0 \uc774\ub97c \uac83\uc73c\ub85c \ucd94\uc815\ub418\uace0 \uc788\ub2e4. \uc5ec\uc790\uc640 \uc544\uc774\ub4e4\ub3c4 \uc6a9\uc11c\uc5c6\uc774 \uc0b4\ud574\ub418\uc5c8\ub2e4. \uc218\ubcf4\ub85c\ud504\ub294 \ud6d7\ub0a0 \ub2e4\uc74c\uacfc \uac19\uc774 \uae30\ub85d\ud558\uc600\ub2e4. \"\ud504\ub77c\uac00 \uc804\uc5ed\uc5d0 \uc2dc\uccb4\ub4e4\uc774 \ub110\ubd80\ub7ec\uc84c\uace0, \ud53c\uac00 \uac15\ubb3c\uc744 \uc774\ub8e8\uc5c8\ub2e4.\" \ud1b5\uc81c\uac00 \ub418\uc9c0 \uc54a\uc558\ub358 \ucf54\uc0ac\ud06c \ubd80\ub300\uac00 \uc774 \ud559\uc0b4\uc5d0 \uc8fc\ub3c4\uc801\uc778 \uc5ed\ud560\uc744 \ud588\ub2e4\ub294 \ube44\ub09c\uc744 \ubc1b\uace0 \uc788\ub2e4.", "sentence_answer": "\uc804\ud22c\uac00 \ub05d\ub09c \ud6c4 \ub7ec\uc2dc\uc544\uad70\uc740 \uc804\ud22c \uc804 \uc218\ubcf4\ub85c\ud504 \uc5d0\uac8c \ubc1b\uc740 \uba85\ub839\uc744 \uc5b4\uae30\uace0 \ubc14\ub974\uc0e4\ubc14 \uc678\uac01\uc5d0\uc11c \uc57d\ud0c8\uacfc \ubc29\ud654\ub97c \ubc8c\uc774\uae30 \uc2dc\uc791\ud558\uc600\ub2e4.", "paragraph_id": "5926617-2-0", "paragraph_question": "question: \ud504\ub77c\uac00 \uc804\ud22c \uc804 \ub7ec\uc2dc\uc544\uad70\uc5d0\uac8c \uc57d\ud0c8\uacfc \ubc29\ud654\ub97c \uae08\uc9c0\ud55c \uc7a5\uad70\uc758 \uc774\ub984\uc740?, context: \uc804\ud22c\uac00 \ub05d\ub09c \ud6c4 \ub7ec\uc2dc\uc544\uad70\uc740 \uc804\ud22c \uc804 \uc218\ubcf4\ub85c\ud504\uc5d0\uac8c \ubc1b\uc740 \uba85\ub839\uc744 \uc5b4\uae30\uace0 \ubc14\ub974\uc0e4\ubc14 \uc678\uac01\uc5d0\uc11c \uc57d\ud0c8\uacfc \ubc29\ud654\ub97c \ubc8c\uc774\uae30 \uc2dc\uc791\ud558\uc600\ub2e4.(\ub7ec\uc2dc\uc544 \ubcd1\uc0ac\ub4e4\uc740 1794\ub144 \ubc14\ub974\uc0e4\ubc14 \ubd09\uae30 \ub54c \ud3f4\ub780\ub4dc \uc778\ub4e4\uc774 \ubc14\ub974\uc0e4\ubc14\ub97c \uc9c0\ud0a4\ub358 \ub7ec\uc2dc\uc544 \uc7a5\ubcd1\ub4e4 2,000\uba85\uc744 \ud559\uc0b4\ud558\uace0 \uad6c\uae08\ud55c \uac83\uc5d0 \ub300\ud55c \ubcf5\uc218\ub77c\uace0 \uc8fc\uc7a5\ud558\uc600\ub2e4.) \uac70\uc758 \uc804\uc5ed\uc774 \uc57d\ud0c8\ub2f9\ud558\uace0 \ubd88\ud0d4\uc73c\uba70 \ud504\ub77c\uac00\uc758 \uac70\uc8fc\uc790 \ub300\ubd80\ubd84\uc740 \uc0b4\ud574\ub2f9\ud588\ub2e4. \uc774\ub54c \uc0b4\ud574\ub2f9\ud55c \uc0ac\ub78c\ub4e4\uc758 \uc22b\uc790\ub294 \ubd88\uba85\ud655\ud558\ub098, \uc801\uc5b4\ub3c4 2\ub9cc \uba85\uc5d0 \uc774\ub97c \uac83\uc73c\ub85c \ucd94\uc815\ub418\uace0 \uc788\ub2e4. \uc5ec\uc790\uc640 \uc544\uc774\ub4e4\ub3c4 \uc6a9\uc11c\uc5c6\uc774 \uc0b4\ud574\ub418\uc5c8\ub2e4. \uc218\ubcf4\ub85c\ud504\ub294 \ud6d7\ub0a0 \ub2e4\uc74c\uacfc \uac19\uc774 \uae30\ub85d\ud558\uc600\ub2e4. \"\ud504\ub77c\uac00 \uc804\uc5ed\uc5d0 \uc2dc\uccb4\ub4e4\uc774 \ub110\ubd80\ub7ec\uc84c\uace0, \ud53c\uac00 \uac15\ubb3c\uc744 \uc774\ub8e8\uc5c8\ub2e4.\" \ud1b5\uc81c\uac00 \ub418\uc9c0 \uc54a\uc558\ub358 \ucf54\uc0ac\ud06c \ubd80\ub300\uac00 \uc774 \ud559\uc0b4\uc5d0 \uc8fc\ub3c4\uc801\uc778 \uc5ed\ud560\uc744 \ud588\ub2e4\ub294 \ube44\ub09c\uc744 \ubc1b\uace0 \uc788\ub2e4."} {"question": "\ube14\ub808\uc774\ud06c \ub77c\uc774\ube14\ub9ac\uac00 \ucd9c\uc5f0\ud55c \uc601\ud654 <\uccad\ubc14\uc9c0 \ub3cc\ub824\uc785\uae30>\uc5d0\uc11c \ub9e1\uc740 \uc5ed\ud560\uc758 \uc774\ub984\uc740?", "paragraph": "\ub77c\uc774\ube14\ub9ac\uc758 \uc5b4\ub9b0 \uc2dc\uc808, \uadf8\ub140\uc758 \ubd80\ubaa8\ub2d8\uc740 \uadf8\ub140\ub97c \ubca0\uc774\ube44\uc2dc\ud130\uc640 \ud568\uaed8 \uc9d1\uc5d0 \ub0a8\uaca8\ub450\uace0 \uc2f6\uc9c0 \uc54a\uc558\uae30 \ub54c\ubb38\uc5d0 \ubd80\ubaa8\ub2d8\uacfc \ud568\uaed8 \uadf8\ub140\uc640 \ud568\uaed8 \ud589\ub3d9 \uac15\uc2b5\ud68c\uc5d0 \ub370\ub824\uac14\uc2b5\ub2c8\ub2e4. \ub77c\uc774\ube14\ub9ac\ub294 \uc5b4\ub838\uc744 \ub54c\ubd80\ud130 \uadf8\ub140\uc758 \ubd80\ubaa8\uac00 \uc5f0\uae30 \uc218\uc5c5\uc744 \uc9c0\ub3c4\ud558\ub294 \uac83\uc744 \ubcf4\uace0 \uc5f0\uae30\uc5d0 \ub300\ud574 \ubc30\uc6b0\uac8c \ub418\uc5c8\uace0, \ucc28\uce30 \uadf8\ub140\uac00 \ub098\uc774\uac00 \ub4e4\uba74\uc11c \uc5f0\uae30\ud65c\ub3d9\uc5d0 \ub300\ud55c \ud655\uc2e0\uc744 \uac16\ub294\ub370 \ub3c4\uc6c0\uc774 \ub418\uc5c8\ub2e4\uace0 \ud68c\uc0c1\ud558\uc600\ub2e4. \uadf8\ub140\ub294 \ub2f9\ucd08 \ucc98\uc74c\uc5d0\ub294 \uc5f0\uae30\uc5d0 \uad00\uc2ec\uc774 \uc5c6\uc5c8\uace0, \uc2a4\ud0e0\ud3ec\ub4dc \ub300\ud559\uc5d0 \uc785\ud559\ud558\uae30 \ud76c\ub9dd\ud558\uace0 \uc788\uc5c8\ub2e4. \uadf8\ub140\uac00 \uc911\ud559\uad50 1\ud559\ub144\uacfc \uace0\ub4f1\ud559\uad50 2\ud559\ub144 \uc0ac\uc774\uc758 \uc5ec\ub984\uae30\uac04 \ub3d9\uc548 \uadf8\ub140\uc758 \uc624\ube60 \uc5d0\ub9ad\uc740 \uc5d0\uc774\uc804\ud2b8\uc5d0 \uba87 \ub2ec \ub3d9\uc548 \uba87 \ucc28\ub840\uc758 \uc624\ub514\uc158\uc744 \ubcf4\ub0c8\ub2e4. \uc774 \uc624\ub514\uc158\uc73c\ub85c \uadf8\ub140\ub294 \uc601\ud654 \u300a\uccad\ubc14\uc9c0 \ub3cc\ub824 \uc785\uae30\u300b\uc758 \ube0c\ub9ac\uc9d3 \ube0c\ub9b4\ub79c\ub4dc \uc5ed\ud560\ub85c \uce90\uc2a4\ud305 \ub418\uc5c8\ub2e4. \ub77c\uc774\ube14\ub9ac\ub294 \uadf8\ub140\uac00 \ubc84\ubc45\ud06c \uace0\ub4f1\ud559\uad50 2\ud559\ub144\uacfc \uc878\uc5c5\uc744 \uc55e\ub454 \uc2dc\uae30 \ub3d9\uc548 \uc774 \uc601\ud654\uc758 \ucd2c\uc601\uc744 \ud588\ub2e4. \ubc84\ubc45\ud06c(Burbank)\uc5d0\uc11c \ub77c\uc774\ube14\ub9ac\ub294 \uc2dc\ub2c8\uc5b4 \ud074\ub798\uc2a4\uc758 \ud68c\uc7a5, \uce58\uc5b4\ub9ac\ub354 \ubc0f \ucc54\ud53c\uc5b8 \uc27d \ud569\ucc3d\ub2e8 \uba64\ubc84\ub85c \ud65c\ub3d9\ud588\ub2e4.", "answer": "\ube0c\ub9ac\uc9d3 \ube0c\ub9b4\ub79c\ub4dc", "sentence": "\uc774 \uc624\ub514\uc158\uc73c\ub85c \uadf8\ub140\ub294 \uc601\ud654 \u300a\uccad\ubc14\uc9c0 \ub3cc\ub824 \uc785\uae30\u300b\uc758 \ube0c\ub9ac\uc9d3 \ube0c\ub9b4\ub79c\ub4dc \uc5ed\ud560\ub85c \uce90\uc2a4\ud305 \ub418\uc5c8\ub2e4.", "paragraph_sentence": "\ub77c\uc774\ube14\ub9ac\uc758 \uc5b4\ub9b0 \uc2dc\uc808, \uadf8\ub140\uc758 \ubd80\ubaa8\ub2d8\uc740 \uadf8\ub140\ub97c \ubca0\uc774\ube44\uc2dc\ud130\uc640 \ud568\uaed8 \uc9d1\uc5d0 \ub0a8\uaca8\ub450\uace0 \uc2f6\uc9c0 \uc54a\uc558\uae30 \ub54c\ubb38\uc5d0 \ubd80\ubaa8\ub2d8\uacfc \ud568\uaed8 \uadf8\ub140\uc640 \ud568\uaed8 \ud589\ub3d9 \uac15\uc2b5\ud68c\uc5d0 \ub370\ub824\uac14\uc2b5\ub2c8\ub2e4. \ub77c\uc774\ube14\ub9ac\ub294 \uc5b4\ub838\uc744 \ub54c\ubd80\ud130 \uadf8\ub140\uc758 \ubd80\ubaa8\uac00 \uc5f0\uae30 \uc218\uc5c5\uc744 \uc9c0\ub3c4\ud558\ub294 \uac83\uc744 \ubcf4\uace0 \uc5f0\uae30\uc5d0 \ub300\ud574 \ubc30\uc6b0\uac8c \ub418\uc5c8\uace0, \ucc28\uce30 \uadf8\ub140\uac00 \ub098\uc774\uac00 \ub4e4\uba74\uc11c \uc5f0\uae30\ud65c\ub3d9\uc5d0 \ub300\ud55c \ud655\uc2e0\uc744 \uac16\ub294\ub370 \ub3c4\uc6c0\uc774 \ub418\uc5c8\ub2e4\uace0 \ud68c\uc0c1\ud558\uc600\ub2e4. \uadf8\ub140\ub294 \ub2f9\ucd08 \ucc98\uc74c\uc5d0\ub294 \uc5f0\uae30\uc5d0 \uad00\uc2ec\uc774 \uc5c6\uc5c8\uace0, \uc2a4\ud0e0\ud3ec\ub4dc \ub300\ud559\uc5d0 \uc785\ud559\ud558\uae30 \ud76c\ub9dd\ud558\uace0 \uc788\uc5c8\ub2e4. \uadf8\ub140\uac00 \uc911\ud559\uad50 1\ud559\ub144\uacfc \uace0\ub4f1\ud559\uad50 2\ud559\ub144 \uc0ac\uc774\uc758 \uc5ec\ub984\uae30\uac04 \ub3d9\uc548 \uadf8\ub140\uc758 \uc624\ube60 \uc5d0\ub9ad\uc740 \uc5d0\uc774\uc804\ud2b8\uc5d0 \uba87 \ub2ec \ub3d9\uc548 \uba87 \ucc28\ub840\uc758 \uc624\ub514\uc158\uc744 \ubcf4\ub0c8\ub2e4. \uc774 \uc624\ub514\uc158\uc73c\ub85c \uadf8\ub140\ub294 \uc601\ud654 \u300a\uccad\ubc14\uc9c0 \ub3cc\ub824 \uc785\uae30\u300b\uc758 \ube0c\ub9ac\uc9d3 \ube0c\ub9b4\ub79c\ub4dc \uc5ed\ud560\ub85c \uce90\uc2a4\ud305 \ub418\uc5c8\ub2e4. \ub77c\uc774\ube14\ub9ac\ub294 \uadf8\ub140\uac00 \ubc84\ubc45\ud06c \uace0\ub4f1\ud559\uad50 2\ud559\ub144\uacfc \uc878\uc5c5\uc744 \uc55e\ub454 \uc2dc\uae30 \ub3d9\uc548 \uc774 \uc601\ud654\uc758 \ucd2c\uc601\uc744 \ud588\ub2e4. \ubc84\ubc45\ud06c(Burbank)\uc5d0\uc11c \ub77c\uc774\ube14\ub9ac\ub294 \uc2dc\ub2c8\uc5b4 \ud074\ub798\uc2a4\uc758 \ud68c\uc7a5, \uce58\uc5b4\ub9ac\ub354 \ubc0f \ucc54\ud53c\uc5b8 \uc27d \ud569\ucc3d\ub2e8 \uba64\ubc84\ub85c \ud65c\ub3d9\ud588\ub2e4.", "paragraph_answer": "\ub77c\uc774\ube14\ub9ac\uc758 \uc5b4\ub9b0 \uc2dc\uc808, \uadf8\ub140\uc758 \ubd80\ubaa8\ub2d8\uc740 \uadf8\ub140\ub97c \ubca0\uc774\ube44\uc2dc\ud130\uc640 \ud568\uaed8 \uc9d1\uc5d0 \ub0a8\uaca8\ub450\uace0 \uc2f6\uc9c0 \uc54a\uc558\uae30 \ub54c\ubb38\uc5d0 \ubd80\ubaa8\ub2d8\uacfc \ud568\uaed8 \uadf8\ub140\uc640 \ud568\uaed8 \ud589\ub3d9 \uac15\uc2b5\ud68c\uc5d0 \ub370\ub824\uac14\uc2b5\ub2c8\ub2e4. \ub77c\uc774\ube14\ub9ac\ub294 \uc5b4\ub838\uc744 \ub54c\ubd80\ud130 \uadf8\ub140\uc758 \ubd80\ubaa8\uac00 \uc5f0\uae30 \uc218\uc5c5\uc744 \uc9c0\ub3c4\ud558\ub294 \uac83\uc744 \ubcf4\uace0 \uc5f0\uae30\uc5d0 \ub300\ud574 \ubc30\uc6b0\uac8c \ub418\uc5c8\uace0, \ucc28\uce30 \uadf8\ub140\uac00 \ub098\uc774\uac00 \ub4e4\uba74\uc11c \uc5f0\uae30\ud65c\ub3d9\uc5d0 \ub300\ud55c \ud655\uc2e0\uc744 \uac16\ub294\ub370 \ub3c4\uc6c0\uc774 \ub418\uc5c8\ub2e4\uace0 \ud68c\uc0c1\ud558\uc600\ub2e4. \uadf8\ub140\ub294 \ub2f9\ucd08 \ucc98\uc74c\uc5d0\ub294 \uc5f0\uae30\uc5d0 \uad00\uc2ec\uc774 \uc5c6\uc5c8\uace0, \uc2a4\ud0e0\ud3ec\ub4dc \ub300\ud559\uc5d0 \uc785\ud559\ud558\uae30 \ud76c\ub9dd\ud558\uace0 \uc788\uc5c8\ub2e4. \uadf8\ub140\uac00 \uc911\ud559\uad50 1\ud559\ub144\uacfc \uace0\ub4f1\ud559\uad50 2\ud559\ub144 \uc0ac\uc774\uc758 \uc5ec\ub984\uae30\uac04 \ub3d9\uc548 \uadf8\ub140\uc758 \uc624\ube60 \uc5d0\ub9ad\uc740 \uc5d0\uc774\uc804\ud2b8\uc5d0 \uba87 \ub2ec \ub3d9\uc548 \uba87 \ucc28\ub840\uc758 \uc624\ub514\uc158\uc744 \ubcf4\ub0c8\ub2e4. \uc774 \uc624\ub514\uc158\uc73c\ub85c \uadf8\ub140\ub294 \uc601\ud654 \u300a\uccad\ubc14\uc9c0 \ub3cc\ub824 \uc785\uae30\u300b\uc758 \ube0c\ub9ac\uc9d3 \ube0c\ub9b4\ub79c\ub4dc \uc5ed\ud560\ub85c \uce90\uc2a4\ud305 \ub418\uc5c8\ub2e4. \ub77c\uc774\ube14\ub9ac\ub294 \uadf8\ub140\uac00 \ubc84\ubc45\ud06c \uace0\ub4f1\ud559\uad50 2\ud559\ub144\uacfc \uc878\uc5c5\uc744 \uc55e\ub454 \uc2dc\uae30 \ub3d9\uc548 \uc774 \uc601\ud654\uc758 \ucd2c\uc601\uc744 \ud588\ub2e4. \ubc84\ubc45\ud06c(Burbank)\uc5d0\uc11c \ub77c\uc774\ube14\ub9ac\ub294 \uc2dc\ub2c8\uc5b4 \ud074\ub798\uc2a4\uc758 \ud68c\uc7a5, \uce58\uc5b4\ub9ac\ub354 \ubc0f \ucc54\ud53c\uc5b8 \uc27d \ud569\ucc3d\ub2e8 \uba64\ubc84\ub85c \ud65c\ub3d9\ud588\ub2e4.", "sentence_answer": "\uc774 \uc624\ub514\uc158\uc73c\ub85c \uadf8\ub140\ub294 \uc601\ud654 \u300a\uccad\ubc14\uc9c0 \ub3cc\ub824 \uc785\uae30\u300b\uc758 \ube0c\ub9ac\uc9d3 \ube0c\ub9b4\ub79c\ub4dc \uc5ed\ud560\ub85c \uce90\uc2a4\ud305 \ub418\uc5c8\ub2e4.", "paragraph_id": "6509731-1-0", "paragraph_question": "question: \ube14\ub808\uc774\ud06c \ub77c\uc774\ube14\ub9ac\uac00 \ucd9c\uc5f0\ud55c \uc601\ud654 <\uccad\ubc14\uc9c0 \ub3cc\ub824\uc785\uae30>\uc5d0\uc11c \ub9e1\uc740 \uc5ed\ud560\uc758 \uc774\ub984\uc740?, context: \ub77c\uc774\ube14\ub9ac\uc758 \uc5b4\ub9b0 \uc2dc\uc808, \uadf8\ub140\uc758 \ubd80\ubaa8\ub2d8\uc740 \uadf8\ub140\ub97c \ubca0\uc774\ube44\uc2dc\ud130\uc640 \ud568\uaed8 \uc9d1\uc5d0 \ub0a8\uaca8\ub450\uace0 \uc2f6\uc9c0 \uc54a\uc558\uae30 \ub54c\ubb38\uc5d0 \ubd80\ubaa8\ub2d8\uacfc \ud568\uaed8 \uadf8\ub140\uc640 \ud568\uaed8 \ud589\ub3d9 \uac15\uc2b5\ud68c\uc5d0 \ub370\ub824\uac14\uc2b5\ub2c8\ub2e4. \ub77c\uc774\ube14\ub9ac\ub294 \uc5b4\ub838\uc744 \ub54c\ubd80\ud130 \uadf8\ub140\uc758 \ubd80\ubaa8\uac00 \uc5f0\uae30 \uc218\uc5c5\uc744 \uc9c0\ub3c4\ud558\ub294 \uac83\uc744 \ubcf4\uace0 \uc5f0\uae30\uc5d0 \ub300\ud574 \ubc30\uc6b0\uac8c \ub418\uc5c8\uace0, \ucc28\uce30 \uadf8\ub140\uac00 \ub098\uc774\uac00 \ub4e4\uba74\uc11c \uc5f0\uae30\ud65c\ub3d9\uc5d0 \ub300\ud55c \ud655\uc2e0\uc744 \uac16\ub294\ub370 \ub3c4\uc6c0\uc774 \ub418\uc5c8\ub2e4\uace0 \ud68c\uc0c1\ud558\uc600\ub2e4. \uadf8\ub140\ub294 \ub2f9\ucd08 \ucc98\uc74c\uc5d0\ub294 \uc5f0\uae30\uc5d0 \uad00\uc2ec\uc774 \uc5c6\uc5c8\uace0, \uc2a4\ud0e0\ud3ec\ub4dc \ub300\ud559\uc5d0 \uc785\ud559\ud558\uae30 \ud76c\ub9dd\ud558\uace0 \uc788\uc5c8\ub2e4. \uadf8\ub140\uac00 \uc911\ud559\uad50 1\ud559\ub144\uacfc \uace0\ub4f1\ud559\uad50 2\ud559\ub144 \uc0ac\uc774\uc758 \uc5ec\ub984\uae30\uac04 \ub3d9\uc548 \uadf8\ub140\uc758 \uc624\ube60 \uc5d0\ub9ad\uc740 \uc5d0\uc774\uc804\ud2b8\uc5d0 \uba87 \ub2ec \ub3d9\uc548 \uba87 \ucc28\ub840\uc758 \uc624\ub514\uc158\uc744 \ubcf4\ub0c8\ub2e4. \uc774 \uc624\ub514\uc158\uc73c\ub85c \uadf8\ub140\ub294 \uc601\ud654 \u300a\uccad\ubc14\uc9c0 \ub3cc\ub824 \uc785\uae30\u300b\uc758 \ube0c\ub9ac\uc9d3 \ube0c\ub9b4\ub79c\ub4dc \uc5ed\ud560\ub85c \uce90\uc2a4\ud305 \ub418\uc5c8\ub2e4. \ub77c\uc774\ube14\ub9ac\ub294 \uadf8\ub140\uac00 \ubc84\ubc45\ud06c \uace0\ub4f1\ud559\uad50 2\ud559\ub144\uacfc \uc878\uc5c5\uc744 \uc55e\ub454 \uc2dc\uae30 \ub3d9\uc548 \uc774 \uc601\ud654\uc758 \ucd2c\uc601\uc744 \ud588\ub2e4. \ubc84\ubc45\ud06c(Burbank)\uc5d0\uc11c \ub77c\uc774\ube14\ub9ac\ub294 \uc2dc\ub2c8\uc5b4 \ud074\ub798\uc2a4\uc758 \ud68c\uc7a5, \uce58\uc5b4\ub9ac\ub354 \ubc0f \ucc54\ud53c\uc5b8 \uc27d \ud569\ucc3d\ub2e8 \uba64\ubc84\ub85c \ud65c\ub3d9\ud588\ub2e4."} {"question": "\uc81c\uc784\uc2a4 \uc624\uc5b8 \uc124\ub9ac\ubc88\uc758 \uc0ac\ub9dd\ub144\ub3c4\ub294?", "paragraph": "\uc81c\uc784\uc2a4 \uc624\uc5b8 \uc124\ub9ac\ubc88(James Owen Sullivan, 1981\ub144 2\uc6d4 9\uc77c ~ 2009\ub144 12\uc6d4 28\uc77c)\uc740 \ubbf8\uad6d\uc758 \uc74c\uc545\uac00\uc774\uc790 \uc791\uace1\uac00\ub85c, \ub354 \ub808\ubc84\ub7f0\ub4dc \uc18c\ub864\ubba4 \ud50c\ub808\uc774\uadf8(The Reverend Tholomew Plague) \ub610\ub294 \uac04\ub2e8\ud788 \ub354 \ub808\ube0c(The Rev)\ub77c\ub294 \uc774\ub984\uc73c\ub85c \uc798 \uc54c\ub824\uc838 \uc788\ub2e4. \uadf8\ub294 \ubbf8\uad6d\uc758 \ud5e4\ube44 \uba54\ud0c8 \ubc34\ub4dc \uc5b4\ubca4\uc9c0\ub4dc \uc138\ube10\ud3f4\ub4dc\uc758 \uc1a1\ub77c\uc774\ud130, \uacf5\ub3d9 \ub9ac\ub4dc\ubcf4\uceec, \ucc3d\ub9bd \uba64\ubc84\uc600\ub2e4. \uadf8\ub294 \uc5b4\ubca4\uc9c0\ub4dc \uc138\ube10\ud3f4\ub4dc\uc758 \uc568\ubc94\uc5d0 \uc9c0\ub300\ud55c \uacf5\ud5cc\uc744 \ud588\ub2e4\uace0 \ud3c9\uac00\ub418\uba70 \"Afterlife\", \"A Little Piece of Heaven\", \"Almost Easy\"\uac19\uc740 \uace1\ub4e4\uc744 \uc791\uace1\ud558\uc600\ub2e4. \uadf8\ub294 \ub610\ud55c \uac19\uc740 \uc5b4\ubca4\uc9c0\ub4dc \uc138\ube10\ud3f4\ub4dc\uc758 \uba64\ubc84\uc778 \uc2dc\ub2c8\uc2a4\ud130 \uac8c\uc774\uce20(Brian Elwin Haner Jr.)\uc640 \ud568\uaed8 \uadf8\uc758 \uc0ac\uc774\ub4dc \ud504\ub85c\uc81d\ud2b8 \ubc34\ub4dc \ud551\ud074\ub9ac \uc2a4\ubb34\uc2a4\uc5d0\uc11c \uc608\uba85 \ub7ab\ud5e4\ub4dc(Rathead)\ub85c \ubcf4\uceec\uacfc \ud53c\uc544\ub178\ub97c \ub2f4\ub2f9\ud588\uc5c8\ub2e4. \ub610\ud55c \uadf8\ub294 1998\ub144\ubd80\ud130 1999\ub144\uae4c\uc9c0 \uc11c\ubc84\ubc18 \ub808\uc804\uc988\uc758 \ub4dc\ub7ec\uba38\uc600\ub2e4.", "answer": "2009\ub144", "sentence": "\uc81c\uc784\uc2a4 \uc624\uc5b8 \uc124\ub9ac\ubc88(James Owen Sullivan, 1981\ub144 2\uc6d4 9\uc77c ~ 2009\ub144 12\uc6d4 28\uc77c)\uc740 \ubbf8\uad6d\uc758 \uc74c\uc545\uac00\uc774\uc790 \uc791\uace1\uac00\ub85c, \ub354 \ub808\ubc84\ub7f0\ub4dc \uc18c\ub864\ubba4 \ud50c\ub808\uc774\uadf8(The Reverend Tholomew Plague) \ub610\ub294 \uac04\ub2e8\ud788 \ub354 \ub808\ube0c(The Rev)\ub77c\ub294 \uc774\ub984\uc73c\ub85c \uc798 \uc54c\ub824\uc838 \uc788\ub2e4.", "paragraph_sentence": " \uc81c\uc784\uc2a4 \uc624\uc5b8 \uc124\ub9ac\ubc88(James Owen Sullivan, 1981\ub144 2\uc6d4 9\uc77c ~ 2009\ub144 12\uc6d4 28\uc77c)\uc740 \ubbf8\uad6d\uc758 \uc74c\uc545\uac00\uc774\uc790 \uc791\uace1\uac00\ub85c, \ub354 \ub808\ubc84\ub7f0\ub4dc \uc18c\ub864\ubba4 \ud50c\ub808\uc774\uadf8(The Reverend Tholomew Plague) \ub610\ub294 \uac04\ub2e8\ud788 \ub354 \ub808\ube0c(The Rev)\ub77c\ub294 \uc774\ub984\uc73c\ub85c \uc798 \uc54c\ub824\uc838 \uc788\ub2e4. \uadf8\ub294 \ubbf8\uad6d\uc758 \ud5e4\ube44 \uba54\ud0c8 \ubc34\ub4dc \uc5b4\ubca4\uc9c0\ub4dc \uc138\ube10\ud3f4\ub4dc\uc758 \uc1a1\ub77c\uc774\ud130, \uacf5\ub3d9 \ub9ac\ub4dc\ubcf4\uceec, \ucc3d\ub9bd \uba64\ubc84\uc600\ub2e4. \uadf8\ub294 \uc5b4\ubca4\uc9c0\ub4dc \uc138\ube10\ud3f4\ub4dc\uc758 \uc568\ubc94\uc5d0 \uc9c0\ub300\ud55c \uacf5\ud5cc\uc744 \ud588\ub2e4\uace0 \ud3c9\uac00\ub418\uba70 \"Afterlife\", \"A Little Piece of Heaven\", \"Almost Easy\"\uac19\uc740 \uace1\ub4e4\uc744 \uc791\uace1\ud558\uc600\ub2e4. \uadf8\ub294 \ub610\ud55c \uac19\uc740 \uc5b4\ubca4\uc9c0\ub4dc \uc138\ube10\ud3f4\ub4dc\uc758 \uba64\ubc84\uc778 \uc2dc\ub2c8\uc2a4\ud130 \uac8c\uc774\uce20(Brian Elwin Haner Jr.)\uc640 \ud568\uaed8 \uadf8\uc758 \uc0ac\uc774\ub4dc \ud504\ub85c\uc81d\ud2b8 \ubc34\ub4dc \ud551\ud074\ub9ac \uc2a4\ubb34\uc2a4\uc5d0\uc11c \uc608\uba85 \ub7ab\ud5e4\ub4dc(Rathead)\ub85c \ubcf4\uceec\uacfc \ud53c\uc544\ub178\ub97c \ub2f4\ub2f9\ud588\uc5c8\ub2e4. \ub610\ud55c \uadf8\ub294 1998\ub144\ubd80\ud130 1999\ub144\uae4c\uc9c0 \uc11c\ubc84\ubc18 \ub808\uc804\uc988\uc758 \ub4dc\ub7ec\uba38\uc600\ub2e4.", "paragraph_answer": "\uc81c\uc784\uc2a4 \uc624\uc5b8 \uc124\ub9ac\ubc88(James Owen Sullivan, 1981\ub144 2\uc6d4 9\uc77c ~ 2009\ub144 12\uc6d4 28\uc77c)\uc740 \ubbf8\uad6d\uc758 \uc74c\uc545\uac00\uc774\uc790 \uc791\uace1\uac00\ub85c, \ub354 \ub808\ubc84\ub7f0\ub4dc \uc18c\ub864\ubba4 \ud50c\ub808\uc774\uadf8(The Reverend Tholomew Plague) \ub610\ub294 \uac04\ub2e8\ud788 \ub354 \ub808\ube0c(The Rev)\ub77c\ub294 \uc774\ub984\uc73c\ub85c \uc798 \uc54c\ub824\uc838 \uc788\ub2e4. \uadf8\ub294 \ubbf8\uad6d\uc758 \ud5e4\ube44 \uba54\ud0c8 \ubc34\ub4dc \uc5b4\ubca4\uc9c0\ub4dc \uc138\ube10\ud3f4\ub4dc\uc758 \uc1a1\ub77c\uc774\ud130, \uacf5\ub3d9 \ub9ac\ub4dc\ubcf4\uceec, \ucc3d\ub9bd \uba64\ubc84\uc600\ub2e4. \uadf8\ub294 \uc5b4\ubca4\uc9c0\ub4dc \uc138\ube10\ud3f4\ub4dc\uc758 \uc568\ubc94\uc5d0 \uc9c0\ub300\ud55c \uacf5\ud5cc\uc744 \ud588\ub2e4\uace0 \ud3c9\uac00\ub418\uba70 \"Afterlife\", \"A Little Piece of Heaven\", \"Almost Easy\"\uac19\uc740 \uace1\ub4e4\uc744 \uc791\uace1\ud558\uc600\ub2e4. \uadf8\ub294 \ub610\ud55c \uac19\uc740 \uc5b4\ubca4\uc9c0\ub4dc \uc138\ube10\ud3f4\ub4dc\uc758 \uba64\ubc84\uc778 \uc2dc\ub2c8\uc2a4\ud130 \uac8c\uc774\uce20(Brian Elwin Haner Jr.)\uc640 \ud568\uaed8 \uadf8\uc758 \uc0ac\uc774\ub4dc \ud504\ub85c\uc81d\ud2b8 \ubc34\ub4dc \ud551\ud074\ub9ac \uc2a4\ubb34\uc2a4\uc5d0\uc11c \uc608\uba85 \ub7ab\ud5e4\ub4dc(Rathead)\ub85c \ubcf4\uceec\uacfc \ud53c\uc544\ub178\ub97c \ub2f4\ub2f9\ud588\uc5c8\ub2e4. \ub610\ud55c \uadf8\ub294 1998\ub144\ubd80\ud130 1999\ub144\uae4c\uc9c0 \uc11c\ubc84\ubc18 \ub808\uc804\uc988\uc758 \ub4dc\ub7ec\uba38\uc600\ub2e4.", "sentence_answer": "\uc81c\uc784\uc2a4 \uc624\uc5b8 \uc124\ub9ac\ubc88(James Owen Sullivan, 1981\ub144 2\uc6d4 9\uc77c ~ 2009\ub144 12\uc6d4 28\uc77c)\uc740 \ubbf8\uad6d\uc758 \uc74c\uc545\uac00\uc774\uc790 \uc791\uace1\uac00\ub85c, \ub354 \ub808\ubc84\ub7f0\ub4dc \uc18c\ub864\ubba4 \ud50c\ub808\uc774\uadf8(The Reverend Tholomew Plague) \ub610\ub294 \uac04\ub2e8\ud788 \ub354 \ub808\ube0c(The Rev)\ub77c\ub294 \uc774\ub984\uc73c\ub85c \uc798 \uc54c\ub824\uc838 \uc788\ub2e4.", "paragraph_id": "6486786-0-0", "paragraph_question": "question: \uc81c\uc784\uc2a4 \uc624\uc5b8 \uc124\ub9ac\ubc88\uc758 \uc0ac\ub9dd\ub144\ub3c4\ub294?, context: \uc81c\uc784\uc2a4 \uc624\uc5b8 \uc124\ub9ac\ubc88(James Owen Sullivan, 1981\ub144 2\uc6d4 9\uc77c ~ 2009\ub144 12\uc6d4 28\uc77c)\uc740 \ubbf8\uad6d\uc758 \uc74c\uc545\uac00\uc774\uc790 \uc791\uace1\uac00\ub85c, \ub354 \ub808\ubc84\ub7f0\ub4dc \uc18c\ub864\ubba4 \ud50c\ub808\uc774\uadf8(The Reverend Tholomew Plague) \ub610\ub294 \uac04\ub2e8\ud788 \ub354 \ub808\ube0c(The Rev)\ub77c\ub294 \uc774\ub984\uc73c\ub85c \uc798 \uc54c\ub824\uc838 \uc788\ub2e4. \uadf8\ub294 \ubbf8\uad6d\uc758 \ud5e4\ube44 \uba54\ud0c8 \ubc34\ub4dc \uc5b4\ubca4\uc9c0\ub4dc \uc138\ube10\ud3f4\ub4dc\uc758 \uc1a1\ub77c\uc774\ud130, \uacf5\ub3d9 \ub9ac\ub4dc\ubcf4\uceec, \ucc3d\ub9bd \uba64\ubc84\uc600\ub2e4. \uadf8\ub294 \uc5b4\ubca4\uc9c0\ub4dc \uc138\ube10\ud3f4\ub4dc\uc758 \uc568\ubc94\uc5d0 \uc9c0\ub300\ud55c \uacf5\ud5cc\uc744 \ud588\ub2e4\uace0 \ud3c9\uac00\ub418\uba70 \"Afterlife\", \"A Little Piece of Heaven\", \"Almost Easy\"\uac19\uc740 \uace1\ub4e4\uc744 \uc791\uace1\ud558\uc600\ub2e4. \uadf8\ub294 \ub610\ud55c \uac19\uc740 \uc5b4\ubca4\uc9c0\ub4dc \uc138\ube10\ud3f4\ub4dc\uc758 \uba64\ubc84\uc778 \uc2dc\ub2c8\uc2a4\ud130 \uac8c\uc774\uce20(Brian Elwin Haner Jr.)\uc640 \ud568\uaed8 \uadf8\uc758 \uc0ac\uc774\ub4dc \ud504\ub85c\uc81d\ud2b8 \ubc34\ub4dc \ud551\ud074\ub9ac \uc2a4\ubb34\uc2a4\uc5d0\uc11c \uc608\uba85 \ub7ab\ud5e4\ub4dc(Rathead)\ub85c \ubcf4\uceec\uacfc \ud53c\uc544\ub178\ub97c \ub2f4\ub2f9\ud588\uc5c8\ub2e4. \ub610\ud55c \uadf8\ub294 1998\ub144\ubd80\ud130 1999\ub144\uae4c\uc9c0 \uc11c\ubc84\ubc18 \ub808\uc804\uc988\uc758 \ub4dc\ub7ec\uba38\uc600\ub2e4."} {"question": "\uc6e8\uc2a4\ud2b8 \ud584\uc774 1\ub77c\uc6b4\ub4dc\uc5d0\uc11c 3:2\ub85c \uaca9\ud30c\ud55c \uc0c1\ub300\ud300\uc740?", "paragraph": "\ubcfc\ud134\uc758 \uc218\ube44 \uc911\uc2ec \uc804\uc220\uacfc \ub2ec\ub9ac, \uc6e8\uc2a4\ud2b8 \ud584\uc740 \ub300\ub2e4\uc218 \ud32c\ub4e4\uc774 \uc88b\uc544\ud558\ub294 \ube60\ub978 \uc6c0\uc9c1\uc784\uacfc \uacf5\uaca9\uc801\uc778 \ud50c\ub808\uc774\ub97c \uc120\ubcf4\uc600\ub2e4. \ub7f0\ub358\uc744 \uc5f0\uace0\uc9c0\ub85c \uc0bc\uace0 \uc788\ub294 \uc6e8\uc2a4\ud2b8 \ud584\uc740 1\ub77c\uc6b4\ub4dc\uc5d0\uc11c 2\ubd80 \ub9ac\uadf8 \uc18c\uc18d\uc778 \ud5d0 \uc2dc\ud2f0\ub97c 3:2\ub85c \uaca9\ud30c\ud588\ub2e4. 2\ub77c\uc6b4\ub4dc\uc5d0\uc11c 3\ubd80 \ub0a8\ubd80 \ub9ac\uadf8\uc758 \ube0c\ub77c\uc774\ud134 \uc564 \ud638\ube0c \uc568\ube44\uc5b8\uc744 \ub9de\uc544 \ubb34\uc2b9\ubd80\ub97c \uae30\ub85d\ud55c \uc6e8\uc2a4\ud2b8 \ud584\uc740 \uc7ac\uacbd\uae30\uc5d0\uc11c 1:0\uc73c\ub85c \uc2b9\ub9ac\ud558\uace0 \ub2e4\uc74c \ub77c\uc6b4\ub4dc\uc5d0 \uc9c4\ucd9c\ud588\ub2e4. \uc6e8\uc2a4\ud2b8 \ud584\uc740 3\ub77c\uc6b4\ub4dc\uc5d0\uc11c 3\ubd80 \ub0a8\ubd80 \ub9ac\uadf8\uc758 \ub610\ub2e4\ub978 \ud300 \ud50c\ub9ac\uba38\uc2a4 \uc544\uac00\uc77c\uc744 \uc27d\uac8c \uaebe\uace0 4\ub77c\uc6b4\ub4dc\uc5d0 \uc9c4\ucd9c\ud588\uc9c0\ub9cc, 4\ub77c\uc6b4\ub4dc\uc5d0\uc11c\ub294 \uae4c\ub2e4\ub85c\uc6b4 \uc0c1\ub300\uc778 \uc0ac\uc6b0\uc0d8\ud504\ud134\uc744 \ub9cc\ub098\uac8c \ub418\uc5c8\ub2e4. \uc6e8\uc2a4\ud2b8 \ud584 \ud648\uad6c\uc7a5 \ubd88\ub9b0 \uadf8\ub77c\uc6b4\ub4dc\uc5d0\uc11c \ud3bc\uccd0\uc9c4 \uccab \uacbd\uae30\ub294 1:1 \ubb34\uc2b9\ubd80\ub85c \ub05d\ub0ac\uace0, \uc0ac\uc6b0\uc0d8\ud504\ud134\uc758 \ub378 \uacbd\uae30\uc7a5\uc5d0\uc11c \uc5f4\ub9b0 \uc7ac\uacbd\uae30\ub3c4 \ubb34\uc2b9\ubd80\ub85c \uc885\ub8cc\ub418\uc5c8\ub2e4. \ub450 \ubc88\uc9f8 \uc7ac\uacbd\uae30\ub294 \uc911\ub9bd \uacbd\uae30\uc7a5\uc778 \uc544\uc2a4\ud1a4 \ube4c\ub77c\uc758 \ud648\uad6c\uc7a5, \ubc84\ubc0d\uc5c4\uc758 \ube4c\ub77c \ud30c\ud06c\uc5d0\uc11c \uc5f4\ub838\uace0, \uc6e8\uc2a4\ud2b8 \ud584\uc774 \ube4c\ub9ac \ube0c\ub77c\uc6b4\uc758 \uacb0\uc2b9\uace8\uc5d0 \ud798\uc785\uc5b4 \uc0ac\uc6b0\uc0d8\uc2a4\ud134\uc744 1:0\uc73c\ub85c \uaebe\uc5c8\ub2e4. \ube4c\ub9ac \ube0c\ub77c\uc6b4\uc740 70\ubd84 \uacbd\uc5d0 \"\uc808\ubb18\ud55c \ud504\ub9ac\ud0a5\"\uc73c\ub85c \uc0ac\uc6b0\uc0d8\ud504\ud134 \uace8\ud0a4\ud37c \ud5c8\ubc84\ud2b8 \ub77d\uc744 \uaf3c\uc9dd \ubabb\ud558\uac8c \ub9cc\ub4e4\uba70 \uacb0\uc2b9\uace8\uc744 \ubf51\uc544\ub0b4\uba70 \ud300\uc758 \uc900\uacb0\uc2b9\uc804 \uc9c4\ucd9c\uc744 \uc774\ub04c\uc5c8\ub2e4. \ucc9c\uc2e0\ub9cc\uace0 \ub05d\uc5d0 \uccbc\uc2dc\uc758 \ud648\uad6c\uc7a5\uc778 \uc2a4\ud0ec\ud37c\ub4dc \ube0c\ub9ac\uc9c0\uc5d0\uc11c \uc5f4\ub9ac\ub294 \uc900\uacb0\uc2b9\uc804\uc5d0 \uc9c4\ucd9c\ud55c \uc6e8\uc2a4\ud2b8 \ud584\uc740 \ub354\ube44 \uce74\uc6b4\ud2f0\ub97c \ub9de\uc544 5:2 \ub300\uc2b9\uc744 \uac70\ub450\uba70 \uacb0\uc2b9\uc804\uc5d0 \uc9c4\ucd9c\ud588\ub2e4. \ube4c\ub9ac \ube0c\ub77c\uc6b4\uc774 2\uace8\uc744 \ud130\ub9ac\ub294 \ud65c\uc57d\uc744 \uc120\ubcf4\uc600\uace0, \ube4c\ub9ac \ubb34\uc5b4\ub3c4 2\uace8\uc744 \ud130\ub728\ub824 \ud300\uc758 \uc2b9\ub9ac\ub97c \uc774\ub04c\uc5c8\ub2e4. \uc6e8\uc2a4\ud2b8 \ud584\uc774 \uc6f8\ube14\ub9ac\uc5d0\uc11c \uc5f4\ub9ac\ub294 \uacb0\uc2b9\uc804\uae4c\uc9c0 \uc62c\ub77c\uac00\uba74\uc11c \ub9cc\ub09c \ud300\ub4e4\uc740 \ubaa8\ub450 2\ubd80 \ub9ac\uadf8 \uc774\ud558\uc758 \ud300\ub4e4\uc774\uc5c8\ub2e4. \uadf8\ub807\uac8c\ud574\uc11c \uc6e8\uc2a4\ud2b8 \ud584\uc740 \uc789\uae00\ub79c\ub4dc\uc5d0\uc11c \ub9ac\uadf8 \uc2dc\uc2a4\ud15c\uc744 \uc6b4\uc601\ud558\uae30 \uc2dc\uc791\ud55c \uc774\ub798\uc5d0 \ud55c \ubc88\ub3c4 \ucd5c\uc0c1\uc704\ub9ac\uadf8\uc5d0 \uc18d\ud55c \ud300\ub4e4\uacfc \ubd99\uc9c0 \uc54a\uace0 FA\ucef5 \uacb0\uc2b9\uc804\uc5d0 \uc9c4\ucd9c\ud55c \uccab \ud300\uc774 \ub418\uc5c8\ub2e4.", "answer": "\ud5d0 \uc2dc\ud2f0", "sentence": "\ub7f0\ub358\uc744 \uc5f0\uace0\uc9c0\ub85c \uc0bc\uace0 \uc788\ub294 \uc6e8\uc2a4\ud2b8 \ud584\uc740 1\ub77c\uc6b4\ub4dc\uc5d0\uc11c 2\ubd80 \ub9ac\uadf8 \uc18c\uc18d\uc778 \ud5d0 \uc2dc\ud2f0 \ub97c 3:2\ub85c \uaca9\ud30c\ud588\ub2e4.", "paragraph_sentence": "\ubcfc\ud134\uc758 \uc218\ube44 \uc911\uc2ec \uc804\uc220\uacfc \ub2ec\ub9ac, \uc6e8\uc2a4\ud2b8 \ud584\uc740 \ub300\ub2e4\uc218 \ud32c\ub4e4\uc774 \uc88b\uc544\ud558\ub294 \ube60\ub978 \uc6c0\uc9c1\uc784\uacfc \uacf5\uaca9\uc801\uc778 \ud50c\ub808\uc774\ub97c \uc120\ubcf4\uc600\ub2e4. \ub7f0\ub358\uc744 \uc5f0\uace0\uc9c0\ub85c \uc0bc\uace0 \uc788\ub294 \uc6e8\uc2a4\ud2b8 \ud584\uc740 1\ub77c\uc6b4\ub4dc\uc5d0\uc11c 2\ubd80 \ub9ac\uadf8 \uc18c\uc18d\uc778 \ud5d0 \uc2dc\ud2f0 \ub97c 3:2\ub85c \uaca9\ud30c\ud588\ub2e4. 2\ub77c\uc6b4\ub4dc\uc5d0\uc11c 3\ubd80 \ub0a8\ubd80 \ub9ac\uadf8\uc758 \ube0c\ub77c\uc774\ud134 \uc564 \ud638\ube0c \uc568\ube44\uc5b8\uc744 \ub9de\uc544 \ubb34\uc2b9\ubd80\ub97c \uae30\ub85d\ud55c \uc6e8\uc2a4\ud2b8 \ud584\uc740 \uc7ac\uacbd\uae30\uc5d0\uc11c 1:0\uc73c\ub85c \uc2b9\ub9ac\ud558\uace0 \ub2e4\uc74c \ub77c\uc6b4\ub4dc\uc5d0 \uc9c4\ucd9c\ud588\ub2e4. \uc6e8\uc2a4\ud2b8 \ud584\uc740 3\ub77c\uc6b4\ub4dc\uc5d0\uc11c 3\ubd80 \ub0a8\ubd80 \ub9ac\uadf8\uc758 \ub610\ub2e4\ub978 \ud300 \ud50c\ub9ac\uba38\uc2a4 \uc544\uac00\uc77c\uc744 \uc27d\uac8c \uaebe\uace0 4\ub77c\uc6b4\ub4dc\uc5d0 \uc9c4\ucd9c\ud588\uc9c0\ub9cc, 4\ub77c\uc6b4\ub4dc\uc5d0\uc11c\ub294 \uae4c\ub2e4\ub85c\uc6b4 \uc0c1\ub300\uc778 \uc0ac\uc6b0\uc0d8\ud504\ud134\uc744 \ub9cc\ub098\uac8c \ub418\uc5c8\ub2e4. \uc6e8\uc2a4\ud2b8 \ud584 \ud648\uad6c\uc7a5 \ubd88\ub9b0 \uadf8\ub77c\uc6b4\ub4dc\uc5d0\uc11c \ud3bc\uccd0\uc9c4 \uccab \uacbd\uae30\ub294 1:1 \ubb34\uc2b9\ubd80\ub85c \ub05d\ub0ac\uace0, \uc0ac\uc6b0\uc0d8\ud504\ud134\uc758 \ub378 \uacbd\uae30\uc7a5\uc5d0\uc11c \uc5f4\ub9b0 \uc7ac\uacbd\uae30\ub3c4 \ubb34\uc2b9\ubd80\ub85c \uc885\ub8cc\ub418\uc5c8\ub2e4. \ub450 \ubc88\uc9f8 \uc7ac\uacbd\uae30\ub294 \uc911\ub9bd \uacbd\uae30\uc7a5\uc778 \uc544\uc2a4\ud1a4 \ube4c\ub77c\uc758 \ud648\uad6c\uc7a5, \ubc84\ubc0d\uc5c4\uc758 \ube4c\ub77c \ud30c\ud06c\uc5d0\uc11c \uc5f4\ub838\uace0, \uc6e8\uc2a4\ud2b8 \ud584\uc774 \ube4c\ub9ac \ube0c\ub77c\uc6b4\uc758 \uacb0\uc2b9\uace8\uc5d0 \ud798\uc785\uc5b4 \uc0ac\uc6b0\uc0d8\uc2a4\ud134\uc744 1:0\uc73c\ub85c \uaebe\uc5c8\ub2e4. \ube4c\ub9ac \ube0c\ub77c\uc6b4\uc740 70\ubd84 \uacbd\uc5d0 \"\uc808\ubb18\ud55c \ud504\ub9ac\ud0a5\"\uc73c\ub85c \uc0ac\uc6b0\uc0d8\ud504\ud134 \uace8\ud0a4\ud37c \ud5c8\ubc84\ud2b8 \ub77d\uc744 \uaf3c\uc9dd \ubabb\ud558\uac8c \ub9cc\ub4e4\uba70 \uacb0\uc2b9\uace8\uc744 \ubf51\uc544\ub0b4\uba70 \ud300\uc758 \uc900\uacb0\uc2b9\uc804 \uc9c4\ucd9c\uc744 \uc774\ub04c\uc5c8\ub2e4. \ucc9c\uc2e0\ub9cc\uace0 \ub05d\uc5d0 \uccbc\uc2dc\uc758 \ud648\uad6c\uc7a5\uc778 \uc2a4\ud0ec\ud37c\ub4dc \ube0c\ub9ac\uc9c0\uc5d0\uc11c \uc5f4\ub9ac\ub294 \uc900\uacb0\uc2b9\uc804\uc5d0 \uc9c4\ucd9c\ud55c \uc6e8\uc2a4\ud2b8 \ud584\uc740 \ub354\ube44 \uce74\uc6b4\ud2f0\ub97c \ub9de\uc544 5:2 \ub300\uc2b9\uc744 \uac70\ub450\uba70 \uacb0\uc2b9\uc804\uc5d0 \uc9c4\ucd9c\ud588\ub2e4. \ube4c\ub9ac \ube0c\ub77c\uc6b4\uc774 2\uace8\uc744 \ud130\ub9ac\ub294 \ud65c\uc57d\uc744 \uc120\ubcf4\uc600\uace0, \ube4c\ub9ac \ubb34\uc5b4\ub3c4 2\uace8\uc744 \ud130\ub728\ub824 \ud300\uc758 \uc2b9\ub9ac\ub97c \uc774\ub04c\uc5c8\ub2e4. \uc6e8\uc2a4\ud2b8 \ud584\uc774 \uc6f8\ube14\ub9ac\uc5d0\uc11c \uc5f4\ub9ac\ub294 \uacb0\uc2b9\uc804\uae4c\uc9c0 \uc62c\ub77c\uac00\uba74\uc11c \ub9cc\ub09c \ud300\ub4e4\uc740 \ubaa8\ub450 2\ubd80 \ub9ac\uadf8 \uc774\ud558\uc758 \ud300\ub4e4\uc774\uc5c8\ub2e4. \uadf8\ub807\uac8c\ud574\uc11c \uc6e8\uc2a4\ud2b8 \ud584\uc740 \uc789\uae00\ub79c\ub4dc\uc5d0\uc11c \ub9ac\uadf8 \uc2dc\uc2a4\ud15c\uc744 \uc6b4\uc601\ud558\uae30 \uc2dc\uc791\ud55c \uc774\ub798\uc5d0 \ud55c \ubc88\ub3c4 \ucd5c\uc0c1\uc704\ub9ac\uadf8\uc5d0 \uc18d\ud55c \ud300\ub4e4\uacfc \ubd99\uc9c0 \uc54a\uace0 FA\ucef5 \uacb0\uc2b9\uc804\uc5d0 \uc9c4\ucd9c\ud55c \uccab \ud300\uc774 \ub418\uc5c8\ub2e4.", "paragraph_answer": "\ubcfc\ud134\uc758 \uc218\ube44 \uc911\uc2ec \uc804\uc220\uacfc \ub2ec\ub9ac, \uc6e8\uc2a4\ud2b8 \ud584\uc740 \ub300\ub2e4\uc218 \ud32c\ub4e4\uc774 \uc88b\uc544\ud558\ub294 \ube60\ub978 \uc6c0\uc9c1\uc784\uacfc \uacf5\uaca9\uc801\uc778 \ud50c\ub808\uc774\ub97c \uc120\ubcf4\uc600\ub2e4. \ub7f0\ub358\uc744 \uc5f0\uace0\uc9c0\ub85c \uc0bc\uace0 \uc788\ub294 \uc6e8\uc2a4\ud2b8 \ud584\uc740 1\ub77c\uc6b4\ub4dc\uc5d0\uc11c 2\ubd80 \ub9ac\uadf8 \uc18c\uc18d\uc778 \ud5d0 \uc2dc\ud2f0 \ub97c 3:2\ub85c \uaca9\ud30c\ud588\ub2e4. 2\ub77c\uc6b4\ub4dc\uc5d0\uc11c 3\ubd80 \ub0a8\ubd80 \ub9ac\uadf8\uc758 \ube0c\ub77c\uc774\ud134 \uc564 \ud638\ube0c \uc568\ube44\uc5b8\uc744 \ub9de\uc544 \ubb34\uc2b9\ubd80\ub97c \uae30\ub85d\ud55c \uc6e8\uc2a4\ud2b8 \ud584\uc740 \uc7ac\uacbd\uae30\uc5d0\uc11c 1:0\uc73c\ub85c \uc2b9\ub9ac\ud558\uace0 \ub2e4\uc74c \ub77c\uc6b4\ub4dc\uc5d0 \uc9c4\ucd9c\ud588\ub2e4. \uc6e8\uc2a4\ud2b8 \ud584\uc740 3\ub77c\uc6b4\ub4dc\uc5d0\uc11c 3\ubd80 \ub0a8\ubd80 \ub9ac\uadf8\uc758 \ub610\ub2e4\ub978 \ud300 \ud50c\ub9ac\uba38\uc2a4 \uc544\uac00\uc77c\uc744 \uc27d\uac8c \uaebe\uace0 4\ub77c\uc6b4\ub4dc\uc5d0 \uc9c4\ucd9c\ud588\uc9c0\ub9cc, 4\ub77c\uc6b4\ub4dc\uc5d0\uc11c\ub294 \uae4c\ub2e4\ub85c\uc6b4 \uc0c1\ub300\uc778 \uc0ac\uc6b0\uc0d8\ud504\ud134\uc744 \ub9cc\ub098\uac8c \ub418\uc5c8\ub2e4. \uc6e8\uc2a4\ud2b8 \ud584 \ud648\uad6c\uc7a5 \ubd88\ub9b0 \uadf8\ub77c\uc6b4\ub4dc\uc5d0\uc11c \ud3bc\uccd0\uc9c4 \uccab \uacbd\uae30\ub294 1:1 \ubb34\uc2b9\ubd80\ub85c \ub05d\ub0ac\uace0, \uc0ac\uc6b0\uc0d8\ud504\ud134\uc758 \ub378 \uacbd\uae30\uc7a5\uc5d0\uc11c \uc5f4\ub9b0 \uc7ac\uacbd\uae30\ub3c4 \ubb34\uc2b9\ubd80\ub85c \uc885\ub8cc\ub418\uc5c8\ub2e4. \ub450 \ubc88\uc9f8 \uc7ac\uacbd\uae30\ub294 \uc911\ub9bd \uacbd\uae30\uc7a5\uc778 \uc544\uc2a4\ud1a4 \ube4c\ub77c\uc758 \ud648\uad6c\uc7a5, \ubc84\ubc0d\uc5c4\uc758 \ube4c\ub77c \ud30c\ud06c\uc5d0\uc11c \uc5f4\ub838\uace0, \uc6e8\uc2a4\ud2b8 \ud584\uc774 \ube4c\ub9ac \ube0c\ub77c\uc6b4\uc758 \uacb0\uc2b9\uace8\uc5d0 \ud798\uc785\uc5b4 \uc0ac\uc6b0\uc0d8\uc2a4\ud134\uc744 1:0\uc73c\ub85c \uaebe\uc5c8\ub2e4. \ube4c\ub9ac \ube0c\ub77c\uc6b4\uc740 70\ubd84 \uacbd\uc5d0 \"\uc808\ubb18\ud55c \ud504\ub9ac\ud0a5\"\uc73c\ub85c \uc0ac\uc6b0\uc0d8\ud504\ud134 \uace8\ud0a4\ud37c \ud5c8\ubc84\ud2b8 \ub77d\uc744 \uaf3c\uc9dd \ubabb\ud558\uac8c \ub9cc\ub4e4\uba70 \uacb0\uc2b9\uace8\uc744 \ubf51\uc544\ub0b4\uba70 \ud300\uc758 \uc900\uacb0\uc2b9\uc804 \uc9c4\ucd9c\uc744 \uc774\ub04c\uc5c8\ub2e4. \ucc9c\uc2e0\ub9cc\uace0 \ub05d\uc5d0 \uccbc\uc2dc\uc758 \ud648\uad6c\uc7a5\uc778 \uc2a4\ud0ec\ud37c\ub4dc \ube0c\ub9ac\uc9c0\uc5d0\uc11c \uc5f4\ub9ac\ub294 \uc900\uacb0\uc2b9\uc804\uc5d0 \uc9c4\ucd9c\ud55c \uc6e8\uc2a4\ud2b8 \ud584\uc740 \ub354\ube44 \uce74\uc6b4\ud2f0\ub97c \ub9de\uc544 5:2 \ub300\uc2b9\uc744 \uac70\ub450\uba70 \uacb0\uc2b9\uc804\uc5d0 \uc9c4\ucd9c\ud588\ub2e4. \ube4c\ub9ac \ube0c\ub77c\uc6b4\uc774 2\uace8\uc744 \ud130\ub9ac\ub294 \ud65c\uc57d\uc744 \uc120\ubcf4\uc600\uace0, \ube4c\ub9ac \ubb34\uc5b4\ub3c4 2\uace8\uc744 \ud130\ub728\ub824 \ud300\uc758 \uc2b9\ub9ac\ub97c \uc774\ub04c\uc5c8\ub2e4. \uc6e8\uc2a4\ud2b8 \ud584\uc774 \uc6f8\ube14\ub9ac\uc5d0\uc11c \uc5f4\ub9ac\ub294 \uacb0\uc2b9\uc804\uae4c\uc9c0 \uc62c\ub77c\uac00\uba74\uc11c \ub9cc\ub09c \ud300\ub4e4\uc740 \ubaa8\ub450 2\ubd80 \ub9ac\uadf8 \uc774\ud558\uc758 \ud300\ub4e4\uc774\uc5c8\ub2e4. \uadf8\ub807\uac8c\ud574\uc11c \uc6e8\uc2a4\ud2b8 \ud584\uc740 \uc789\uae00\ub79c\ub4dc\uc5d0\uc11c \ub9ac\uadf8 \uc2dc\uc2a4\ud15c\uc744 \uc6b4\uc601\ud558\uae30 \uc2dc\uc791\ud55c \uc774\ub798\uc5d0 \ud55c \ubc88\ub3c4 \ucd5c\uc0c1\uc704\ub9ac\uadf8\uc5d0 \uc18d\ud55c \ud300\ub4e4\uacfc \ubd99\uc9c0 \uc54a\uace0 FA\ucef5 \uacb0\uc2b9\uc804\uc5d0 \uc9c4\ucd9c\ud55c \uccab \ud300\uc774 \ub418\uc5c8\ub2e4.", "sentence_answer": "\ub7f0\ub358\uc744 \uc5f0\uace0\uc9c0\ub85c \uc0bc\uace0 \uc788\ub294 \uc6e8\uc2a4\ud2b8 \ud584\uc740 1\ub77c\uc6b4\ub4dc\uc5d0\uc11c 2\ubd80 \ub9ac\uadf8 \uc18c\uc18d\uc778 \ud5d0 \uc2dc\ud2f0 \ub97c 3:2\ub85c \uaca9\ud30c\ud588\ub2e4.", "paragraph_id": "6489739-2-0", "paragraph_question": "question: \uc6e8\uc2a4\ud2b8 \ud584\uc774 1\ub77c\uc6b4\ub4dc\uc5d0\uc11c 3:2\ub85c \uaca9\ud30c\ud55c \uc0c1\ub300\ud300\uc740?, context: \ubcfc\ud134\uc758 \uc218\ube44 \uc911\uc2ec \uc804\uc220\uacfc \ub2ec\ub9ac, \uc6e8\uc2a4\ud2b8 \ud584\uc740 \ub300\ub2e4\uc218 \ud32c\ub4e4\uc774 \uc88b\uc544\ud558\ub294 \ube60\ub978 \uc6c0\uc9c1\uc784\uacfc \uacf5\uaca9\uc801\uc778 \ud50c\ub808\uc774\ub97c \uc120\ubcf4\uc600\ub2e4. \ub7f0\ub358\uc744 \uc5f0\uace0\uc9c0\ub85c \uc0bc\uace0 \uc788\ub294 \uc6e8\uc2a4\ud2b8 \ud584\uc740 1\ub77c\uc6b4\ub4dc\uc5d0\uc11c 2\ubd80 \ub9ac\uadf8 \uc18c\uc18d\uc778 \ud5d0 \uc2dc\ud2f0\ub97c 3:2\ub85c \uaca9\ud30c\ud588\ub2e4. 2\ub77c\uc6b4\ub4dc\uc5d0\uc11c 3\ubd80 \ub0a8\ubd80 \ub9ac\uadf8\uc758 \ube0c\ub77c\uc774\ud134 \uc564 \ud638\ube0c \uc568\ube44\uc5b8\uc744 \ub9de\uc544 \ubb34\uc2b9\ubd80\ub97c \uae30\ub85d\ud55c \uc6e8\uc2a4\ud2b8 \ud584\uc740 \uc7ac\uacbd\uae30\uc5d0\uc11c 1:0\uc73c\ub85c \uc2b9\ub9ac\ud558\uace0 \ub2e4\uc74c \ub77c\uc6b4\ub4dc\uc5d0 \uc9c4\ucd9c\ud588\ub2e4. \uc6e8\uc2a4\ud2b8 \ud584\uc740 3\ub77c\uc6b4\ub4dc\uc5d0\uc11c 3\ubd80 \ub0a8\ubd80 \ub9ac\uadf8\uc758 \ub610\ub2e4\ub978 \ud300 \ud50c\ub9ac\uba38\uc2a4 \uc544\uac00\uc77c\uc744 \uc27d\uac8c \uaebe\uace0 4\ub77c\uc6b4\ub4dc\uc5d0 \uc9c4\ucd9c\ud588\uc9c0\ub9cc, 4\ub77c\uc6b4\ub4dc\uc5d0\uc11c\ub294 \uae4c\ub2e4\ub85c\uc6b4 \uc0c1\ub300\uc778 \uc0ac\uc6b0\uc0d8\ud504\ud134\uc744 \ub9cc\ub098\uac8c \ub418\uc5c8\ub2e4. \uc6e8\uc2a4\ud2b8 \ud584 \ud648\uad6c\uc7a5 \ubd88\ub9b0 \uadf8\ub77c\uc6b4\ub4dc\uc5d0\uc11c \ud3bc\uccd0\uc9c4 \uccab \uacbd\uae30\ub294 1:1 \ubb34\uc2b9\ubd80\ub85c \ub05d\ub0ac\uace0, \uc0ac\uc6b0\uc0d8\ud504\ud134\uc758 \ub378 \uacbd\uae30\uc7a5\uc5d0\uc11c \uc5f4\ub9b0 \uc7ac\uacbd\uae30\ub3c4 \ubb34\uc2b9\ubd80\ub85c \uc885\ub8cc\ub418\uc5c8\ub2e4. \ub450 \ubc88\uc9f8 \uc7ac\uacbd\uae30\ub294 \uc911\ub9bd \uacbd\uae30\uc7a5\uc778 \uc544\uc2a4\ud1a4 \ube4c\ub77c\uc758 \ud648\uad6c\uc7a5, \ubc84\ubc0d\uc5c4\uc758 \ube4c\ub77c \ud30c\ud06c\uc5d0\uc11c \uc5f4\ub838\uace0, \uc6e8\uc2a4\ud2b8 \ud584\uc774 \ube4c\ub9ac \ube0c\ub77c\uc6b4\uc758 \uacb0\uc2b9\uace8\uc5d0 \ud798\uc785\uc5b4 \uc0ac\uc6b0\uc0d8\uc2a4\ud134\uc744 1:0\uc73c\ub85c \uaebe\uc5c8\ub2e4. \ube4c\ub9ac \ube0c\ub77c\uc6b4\uc740 70\ubd84 \uacbd\uc5d0 \"\uc808\ubb18\ud55c \ud504\ub9ac\ud0a5\"\uc73c\ub85c \uc0ac\uc6b0\uc0d8\ud504\ud134 \uace8\ud0a4\ud37c \ud5c8\ubc84\ud2b8 \ub77d\uc744 \uaf3c\uc9dd \ubabb\ud558\uac8c \ub9cc\ub4e4\uba70 \uacb0\uc2b9\uace8\uc744 \ubf51\uc544\ub0b4\uba70 \ud300\uc758 \uc900\uacb0\uc2b9\uc804 \uc9c4\ucd9c\uc744 \uc774\ub04c\uc5c8\ub2e4. \ucc9c\uc2e0\ub9cc\uace0 \ub05d\uc5d0 \uccbc\uc2dc\uc758 \ud648\uad6c\uc7a5\uc778 \uc2a4\ud0ec\ud37c\ub4dc \ube0c\ub9ac\uc9c0\uc5d0\uc11c \uc5f4\ub9ac\ub294 \uc900\uacb0\uc2b9\uc804\uc5d0 \uc9c4\ucd9c\ud55c \uc6e8\uc2a4\ud2b8 \ud584\uc740 \ub354\ube44 \uce74\uc6b4\ud2f0\ub97c \ub9de\uc544 5:2 \ub300\uc2b9\uc744 \uac70\ub450\uba70 \uacb0\uc2b9\uc804\uc5d0 \uc9c4\ucd9c\ud588\ub2e4. \ube4c\ub9ac \ube0c\ub77c\uc6b4\uc774 2\uace8\uc744 \ud130\ub9ac\ub294 \ud65c\uc57d\uc744 \uc120\ubcf4\uc600\uace0, \ube4c\ub9ac \ubb34\uc5b4\ub3c4 2\uace8\uc744 \ud130\ub728\ub824 \ud300\uc758 \uc2b9\ub9ac\ub97c \uc774\ub04c\uc5c8\ub2e4. \uc6e8\uc2a4\ud2b8 \ud584\uc774 \uc6f8\ube14\ub9ac\uc5d0\uc11c \uc5f4\ub9ac\ub294 \uacb0\uc2b9\uc804\uae4c\uc9c0 \uc62c\ub77c\uac00\uba74\uc11c \ub9cc\ub09c \ud300\ub4e4\uc740 \ubaa8\ub450 2\ubd80 \ub9ac\uadf8 \uc774\ud558\uc758 \ud300\ub4e4\uc774\uc5c8\ub2e4. \uadf8\ub807\uac8c\ud574\uc11c \uc6e8\uc2a4\ud2b8 \ud584\uc740 \uc789\uae00\ub79c\ub4dc\uc5d0\uc11c \ub9ac\uadf8 \uc2dc\uc2a4\ud15c\uc744 \uc6b4\uc601\ud558\uae30 \uc2dc\uc791\ud55c \uc774\ub798\uc5d0 \ud55c \ubc88\ub3c4 \ucd5c\uc0c1\uc704\ub9ac\uadf8\uc5d0 \uc18d\ud55c \ud300\ub4e4\uacfc \ubd99\uc9c0 \uc54a\uace0 FA\ucef5 \uacb0\uc2b9\uc804\uc5d0 \uc9c4\ucd9c\ud55c \uccab \ud300\uc774 \ub418\uc5c8\ub2e4."} {"question": "\ud601\uba85 \ub4f1\uc758 \uc0ac\uc720\ub85c \ud5cc\ubc95 \uc81c\uc815 \uad8c\ub825\ub9c8\uc800 \ubc30\uc81c\ud558\ub294 \uac83\uc744 \ubb34\uc5c7\uc774\ub77c \ud558\ub294\uac00?", "paragraph": "\ud5cc\ubc95\uc758 \uac1c\uc815\uc740 \ud5cc\ubc95\uc758 \ubcc0\ucc9c\uacfc \uad6c\ubd84\ub41c\ub2e4(\uc774\uc5d0 \ub300\ud574\uc11c\ub294 \ub4a4\uc5d0 \uc11c\uc220). \ub610\ud55c \ud5cc\ubc95\uc758 \uac1c\uc815\uc740 \ud5cc\ubc95\uc758 \uacbd\uc6b0\uc5d0 \ubc18\ud558\ub294 \uc870\uce58\ub97c \ucde8\ud558\ub294 \ud5cc\ubc95\uc758 \uce68\ud6fc(\u4fb5\u6bc1, \ub3c5\uc77c\uc5b4: Verfassungsdurchbrechung)\uc640 \uad6c\ubd84\ub418\uba70(\ud5cc\ubc95\uc758 \uce68\ud6fc\ub294 \ubb3c\ub860 \uc704\ud5cc\uc774\ub2e4), \ud601\uba85 \ub4f1\uc73c\ub85c \ub9d0\ubbf8\uc554\uc544 \ud5cc\ubc95 \uc81c\uc815 \uad8c\ub825\uae4c\uc9c0\ub3c4 \ubc30\uc81c\ub418\ub294 \ud5cc\ubc95\uc758 \ud30c\uad34 \ub610\ub294 \ud30c\uae30(\u7834\u68c4, Verfassungsvernichtung)\uc640\ub3c4 \uad6c\ubd84\ub41c\ub2e4. \uadf8\ubfd0\ub9cc \uc544\ub2c8\ub77c \uae30\uc874\uc758 \ud5cc\ubc95\ub9cc \ubc30\uc81c\ub420 \ubfd0, \ud5cc\ubc95 \uc81c\uc815 \uad8c\ub825\uc740 \ubcc0\uacbd\ub418\uc9c0 \uc54a\ub294 \ud5cc\ubc95\uc758 \ud3d0\uc81c(\u5ee2\u9664) \ub610\ub294 \ud3d0\uc9c0(\u5ee2\u6b62, Verfassungsbeseitigung)\uc640 \ud2b9\uc815 \uc870\ud56d\uc758 \ud6a8\ub825\uc774 \uc77c\uc2dc\uc801\uc73c\ub85c \uc0c1\uc2e4\ub418\ub294 \ud5cc\ubc95\uc758 \uc815\uc9c0(Verfassungssuspension)\uc640 \uad6c\ubd84\ub41c\ub2e4.", "answer": "\ud5cc\ubc95\uc758 \ud30c\uad34 \ub610\ub294 \ud30c\uae30", "sentence": "\ub610\ud55c \ud5cc\ubc95\uc758 \uac1c\uc815\uc740 \ud5cc\ubc95\uc758 \uacbd\uc6b0\uc5d0 \ubc18\ud558\ub294 \uc870\uce58\ub97c \ucde8\ud558\ub294 \ud5cc\ubc95\uc758 \uce68\ud6fc(\u4fb5\u6bc1, \ub3c5\uc77c\uc5b4: Verfassungsdurchbrechung)\uc640 \uad6c\ubd84\ub418\uba70(\ud5cc\ubc95\uc758 \uce68\ud6fc\ub294 \ubb3c\ub860 \uc704\ud5cc\uc774\ub2e4), \ud601\uba85 \ub4f1\uc73c\ub85c \ub9d0\ubbf8\uc554\uc544 \ud5cc\ubc95 \uc81c\uc815 \uad8c\ub825\uae4c\uc9c0\ub3c4 \ubc30\uc81c\ub418\ub294 \ud5cc\ubc95\uc758 \ud30c\uad34 \ub610\ub294 \ud30c\uae30 (\u7834\u68c4, Verfassungsvernichtung)\uc640\ub3c4 \uad6c\ubd84\ub41c\ub2e4.", "paragraph_sentence": "\ud5cc\ubc95\uc758 \uac1c\uc815\uc740 \ud5cc\ubc95\uc758 \ubcc0\ucc9c\uacfc \uad6c\ubd84\ub41c\ub2e4(\uc774\uc5d0 \ub300\ud574\uc11c\ub294 \ub4a4\uc5d0 \uc11c\uc220). \ub610\ud55c \ud5cc\ubc95\uc758 \uac1c\uc815\uc740 \ud5cc\ubc95\uc758 \uacbd\uc6b0\uc5d0 \ubc18\ud558\ub294 \uc870\uce58\ub97c \ucde8\ud558\ub294 \ud5cc\ubc95\uc758 \uce68\ud6fc(\u4fb5\u6bc1, \ub3c5\uc77c\uc5b4: Verfassungsdurchbrechung)\uc640 \uad6c\ubd84\ub418\uba70(\ud5cc\ubc95\uc758 \uce68\ud6fc\ub294 \ubb3c\ub860 \uc704\ud5cc\uc774\ub2e4), \ud601\uba85 \ub4f1\uc73c\ub85c \ub9d0\ubbf8\uc554\uc544 \ud5cc\ubc95 \uc81c\uc815 \uad8c\ub825\uae4c\uc9c0\ub3c4 \ubc30\uc81c\ub418\ub294 \ud5cc\ubc95\uc758 \ud30c\uad34 \ub610\ub294 \ud30c\uae30 (\u7834\u68c4, Verfassungsvernichtung)\uc640\ub3c4 \uad6c\ubd84\ub41c\ub2e4. \uadf8\ubfd0\ub9cc \uc544\ub2c8\ub77c \uae30\uc874\uc758 \ud5cc\ubc95\ub9cc \ubc30\uc81c\ub420 \ubfd0, \ud5cc\ubc95 \uc81c\uc815 \uad8c\ub825\uc740 \ubcc0\uacbd\ub418\uc9c0 \uc54a\ub294 \ud5cc\ubc95\uc758 \ud3d0\uc81c(\u5ee2\u9664) \ub610\ub294 \ud3d0\uc9c0(\u5ee2\u6b62, Verfassungsbeseitigung)\uc640 \ud2b9\uc815 \uc870\ud56d\uc758 \ud6a8\ub825\uc774 \uc77c\uc2dc\uc801\uc73c\ub85c \uc0c1\uc2e4\ub418\ub294 \ud5cc\ubc95\uc758 \uc815\uc9c0(Verfassungssuspension)\uc640 \uad6c\ubd84\ub41c\ub2e4.", "paragraph_answer": "\ud5cc\ubc95\uc758 \uac1c\uc815\uc740 \ud5cc\ubc95\uc758 \ubcc0\ucc9c\uacfc \uad6c\ubd84\ub41c\ub2e4(\uc774\uc5d0 \ub300\ud574\uc11c\ub294 \ub4a4\uc5d0 \uc11c\uc220). \ub610\ud55c \ud5cc\ubc95\uc758 \uac1c\uc815\uc740 \ud5cc\ubc95\uc758 \uacbd\uc6b0\uc5d0 \ubc18\ud558\ub294 \uc870\uce58\ub97c \ucde8\ud558\ub294 \ud5cc\ubc95\uc758 \uce68\ud6fc(\u4fb5\u6bc1, \ub3c5\uc77c\uc5b4: Verfassungsdurchbrechung)\uc640 \uad6c\ubd84\ub418\uba70(\ud5cc\ubc95\uc758 \uce68\ud6fc\ub294 \ubb3c\ub860 \uc704\ud5cc\uc774\ub2e4), \ud601\uba85 \ub4f1\uc73c\ub85c \ub9d0\ubbf8\uc554\uc544 \ud5cc\ubc95 \uc81c\uc815 \uad8c\ub825\uae4c\uc9c0\ub3c4 \ubc30\uc81c\ub418\ub294 \ud5cc\ubc95\uc758 \ud30c\uad34 \ub610\ub294 \ud30c\uae30 (\u7834\u68c4, Verfassungsvernichtung)\uc640\ub3c4 \uad6c\ubd84\ub41c\ub2e4. \uadf8\ubfd0\ub9cc \uc544\ub2c8\ub77c \uae30\uc874\uc758 \ud5cc\ubc95\ub9cc \ubc30\uc81c\ub420 \ubfd0, \ud5cc\ubc95 \uc81c\uc815 \uad8c\ub825\uc740 \ubcc0\uacbd\ub418\uc9c0 \uc54a\ub294 \ud5cc\ubc95\uc758 \ud3d0\uc81c(\u5ee2\u9664) \ub610\ub294 \ud3d0\uc9c0(\u5ee2\u6b62, Verfassungsbeseitigung)\uc640 \ud2b9\uc815 \uc870\ud56d\uc758 \ud6a8\ub825\uc774 \uc77c\uc2dc\uc801\uc73c\ub85c \uc0c1\uc2e4\ub418\ub294 \ud5cc\ubc95\uc758 \uc815\uc9c0(Verfassungssuspension)\uc640 \uad6c\ubd84\ub41c\ub2e4.", "sentence_answer": "\ub610\ud55c \ud5cc\ubc95\uc758 \uac1c\uc815\uc740 \ud5cc\ubc95\uc758 \uacbd\uc6b0\uc5d0 \ubc18\ud558\ub294 \uc870\uce58\ub97c \ucde8\ud558\ub294 \ud5cc\ubc95\uc758 \uce68\ud6fc(\u4fb5\u6bc1, \ub3c5\uc77c\uc5b4: Verfassungsdurchbrechung)\uc640 \uad6c\ubd84\ub418\uba70(\ud5cc\ubc95\uc758 \uce68\ud6fc\ub294 \ubb3c\ub860 \uc704\ud5cc\uc774\ub2e4), \ud601\uba85 \ub4f1\uc73c\ub85c \ub9d0\ubbf8\uc554\uc544 \ud5cc\ubc95 \uc81c\uc815 \uad8c\ub825\uae4c\uc9c0\ub3c4 \ubc30\uc81c\ub418\ub294 \ud5cc\ubc95\uc758 \ud30c\uad34 \ub610\ub294 \ud30c\uae30 (\u7834\u68c4, Verfassungsvernichtung)\uc640\ub3c4 \uad6c\ubd84\ub41c\ub2e4.", "paragraph_id": "6571295-4-0", "paragraph_question": "question: \ud601\uba85 \ub4f1\uc758 \uc0ac\uc720\ub85c \ud5cc\ubc95 \uc81c\uc815 \uad8c\ub825\ub9c8\uc800 \ubc30\uc81c\ud558\ub294 \uac83\uc744 \ubb34\uc5c7\uc774\ub77c \ud558\ub294\uac00?, context: \ud5cc\ubc95\uc758 \uac1c\uc815\uc740 \ud5cc\ubc95\uc758 \ubcc0\ucc9c\uacfc \uad6c\ubd84\ub41c\ub2e4(\uc774\uc5d0 \ub300\ud574\uc11c\ub294 \ub4a4\uc5d0 \uc11c\uc220). \ub610\ud55c \ud5cc\ubc95\uc758 \uac1c\uc815\uc740 \ud5cc\ubc95\uc758 \uacbd\uc6b0\uc5d0 \ubc18\ud558\ub294 \uc870\uce58\ub97c \ucde8\ud558\ub294 \ud5cc\ubc95\uc758 \uce68\ud6fc(\u4fb5\u6bc1, \ub3c5\uc77c\uc5b4: Verfassungsdurchbrechung)\uc640 \uad6c\ubd84\ub418\uba70(\ud5cc\ubc95\uc758 \uce68\ud6fc\ub294 \ubb3c\ub860 \uc704\ud5cc\uc774\ub2e4), \ud601\uba85 \ub4f1\uc73c\ub85c \ub9d0\ubbf8\uc554\uc544 \ud5cc\ubc95 \uc81c\uc815 \uad8c\ub825\uae4c\uc9c0\ub3c4 \ubc30\uc81c\ub418\ub294 \ud5cc\ubc95\uc758 \ud30c\uad34 \ub610\ub294 \ud30c\uae30(\u7834\u68c4, Verfassungsvernichtung)\uc640\ub3c4 \uad6c\ubd84\ub41c\ub2e4. \uadf8\ubfd0\ub9cc \uc544\ub2c8\ub77c \uae30\uc874\uc758 \ud5cc\ubc95\ub9cc \ubc30\uc81c\ub420 \ubfd0, \ud5cc\ubc95 \uc81c\uc815 \uad8c\ub825\uc740 \ubcc0\uacbd\ub418\uc9c0 \uc54a\ub294 \ud5cc\ubc95\uc758 \ud3d0\uc81c(\u5ee2\u9664) \ub610\ub294 \ud3d0\uc9c0(\u5ee2\u6b62, Verfassungsbeseitigung)\uc640 \ud2b9\uc815 \uc870\ud56d\uc758 \ud6a8\ub825\uc774 \uc77c\uc2dc\uc801\uc73c\ub85c \uc0c1\uc2e4\ub418\ub294 \ud5cc\ubc95\uc758 \uc815\uc9c0(Verfassungssuspension)\uc640 \uad6c\ubd84\ub41c\ub2e4."} {"question": "\ub3d9\uc11c \uad50\ud68c \ubd84\uc5f4\ub85c \uc778\ud574, \uc7ac\uc77c\uce58\ub97c \uc2dc\ub3c4\ud558\uae30 \uc704\ud57412-13\uc138\uae30 \uad50\ud669\ub4e4\uc740 \ubb34\uc5c7\uc744 \uc18c\uc9d1\ud588\ub294\uac00\uc694?", "paragraph": "\ub3d9\ubc29 \uad50\ud68c\uc640 \uc11c\ubc29 \uad50\ud68c\ub294 1054\ub144\uc5d0 \uacb0\uc815\uc801\uc73c\ub85c \uc11c\ub85c \uac08\ub77c\uc11c\uac8c \ub418\uc5c8\ub2e4. \uc774 \ubd84\uc5f4\uc740 \uad50\ub9ac\uc801 \uc774\uacac\uc774\ub77c\uae30 \ubcf4\ub2e4\ub294 \uc815\uce58\uc801\uc778 \uc774\ud574\uad00\uacc4 \ub54c\ubb38\uc5d0 \ubc1c\uc0dd\ud55c \uac83\uc774\ub2e4. \uad50\ud669\ub4e4\uc740 \ub77c\ubca4\ub098 \ucd1d\ub3c5\ubd80\ub97c \ud1b5\ud574 \uc0ac\uc0ac\uac74\uac74 \uac1c\uc785\ud558\ub824 \ub4dc\ub294 \ub3d9\ub85c\ub9c8 \ud669\uc81c\uc758 \uac04\uc12d\uc73c\ub85c\ubd80\ud130 \ubc97\uc5b4\ub098\uae30 \uc704\ud574 \ud504\ub791\ud06c \uad6d\uc655\uc744 \ud669\uc81c\ub85c \ucd94\ub300\ud558\uc5ec \ub9de\uc11c\ub3c4\ub85d \ud558\uc600\ub2e4. \uc591\uce21\uc758 \uac08\ub4f1\uc740 8\uc138\uae30 \uc131\uc0c1 \ud30c\uad34 \uc6b4\ub3d9 \uc774\ub798 \ud45c\uba74\ud654\ub418\uc5c8\uc73c\uba70, 9\uc138\uae30\uc5d0\ub294 \ud544\ub9ac\uc624\ucf00 \ubb38\uc81c\uc640 \uadf8\uc5d0 \ub530\ub978 \ud3ec\ud2f0\uc6b0\uc2a4\uc758 \ubd84\ub9ac \ub4f1\uc5d0\uc11c \ub9cc\uc131\uc801\uc778 \ubd88\ud654\uac00 \uc787\ub2ec\uc544 \uc77c\uc5b4\ub0ac\ub2e4. \uadf8\ub9ac\uace0 1054\ub144\uc5d0\ub294 \ud574\ubb35\uc740 \uad50\ud669 \uc218\uc704\uad8c\uc758 \uc2dc\ube44\ub85c \ub85c\ub9c8 \uad50\ud68c\uc640 \ucf58\uc2a4\ud0c4\ud2f0\ub178\ud3f4\ub9ac\uc2a4 \uad50\ud68c\uac00 \uc11c\ub85c\ub97c \ud30c\ubb38\ud558\uae30\uc5d0 \uc774\ub974\ub800\ub2e4. \ub3d9\uc11c \uad50\ud68c\uc758 \ubd84\uc5f4\uc774\ub77c\uace0 \ubd88\ub9ac\ub294 \uae30\ub3c5\uad50 \uc138\uacc4\uc758 \ucd5c\ub300 \ubd84\uc5f4 \uc0ac\uac74\uc73c\ub85c \uacb0\uad6d \uad50\ud68c\ub294 \ub3d9\uc11c \uc9c0\uc5ed\uc73c\ub85c \ub2e8\uc808\ub410\ub2e4. \uc774\ud6c4 12~13\uc138\uae30 \uad50\ud669\ub4e4\uc740 \uacf5\uc758\ud68c\ub97c \uc5ec\ub7ec \ubc88 \uc18c\uc9d1\ud558\uc5ec \ub3d9\uc11c \uad50\ud68c\uc758 \uc7ac\uc77c\uce58\ub97c \uc2dc\ub3c4\ud588\uc73c\ub098 \ud070 \uc131\uacfc\ub97c \uac70\ub450\uc9c0\ub294 \ubabb\ud588\ub2e4.", "answer": "\uacf5\uc758\ud68c", "sentence": "\uc774\ud6c4 12~13\uc138\uae30 \uad50\ud669\ub4e4\uc740 \uacf5\uc758\ud68c \ub97c \uc5ec\ub7ec \ubc88 \uc18c\uc9d1\ud558\uc5ec \ub3d9\uc11c \uad50\ud68c\uc758 \uc7ac\uc77c\uce58\ub97c \uc2dc\ub3c4\ud588\uc73c\ub098 \ud070 \uc131\uacfc\ub97c \uac70\ub450\uc9c0\ub294 \ubabb\ud588\ub2e4.", "paragraph_sentence": "\ub3d9\ubc29 \uad50\ud68c\uc640 \uc11c\ubc29 \uad50\ud68c\ub294 1054\ub144\uc5d0 \uacb0\uc815\uc801\uc73c\ub85c \uc11c\ub85c \uac08\ub77c\uc11c\uac8c \ub418\uc5c8\ub2e4. \uc774 \ubd84\uc5f4\uc740 \uad50\ub9ac\uc801 \uc774\uacac\uc774\ub77c\uae30 \ubcf4\ub2e4\ub294 \uc815\uce58\uc801\uc778 \uc774\ud574\uad00\uacc4 \ub54c\ubb38\uc5d0 \ubc1c\uc0dd\ud55c \uac83\uc774\ub2e4. \uad50\ud669\ub4e4\uc740 \ub77c\ubca4\ub098 \ucd1d\ub3c5\ubd80\ub97c \ud1b5\ud574 \uc0ac\uc0ac\uac74\uac74 \uac1c\uc785\ud558\ub824 \ub4dc\ub294 \ub3d9\ub85c\ub9c8 \ud669\uc81c\uc758 \uac04\uc12d\uc73c\ub85c\ubd80\ud130 \ubc97\uc5b4\ub098\uae30 \uc704\ud574 \ud504\ub791\ud06c \uad6d\uc655\uc744 \ud669\uc81c\ub85c \ucd94\ub300\ud558\uc5ec \ub9de\uc11c\ub3c4\ub85d \ud558\uc600\ub2e4. \uc591\uce21\uc758 \uac08\ub4f1\uc740 8\uc138\uae30 \uc131\uc0c1 \ud30c\uad34 \uc6b4\ub3d9 \uc774\ub798 \ud45c\uba74\ud654\ub418\uc5c8\uc73c\uba70, 9\uc138\uae30\uc5d0\ub294 \ud544\ub9ac\uc624\ucf00 \ubb38\uc81c\uc640 \uadf8\uc5d0 \ub530\ub978 \ud3ec\ud2f0\uc6b0\uc2a4\uc758 \ubd84\ub9ac \ub4f1\uc5d0\uc11c \ub9cc\uc131\uc801\uc778 \ubd88\ud654\uac00 \uc787\ub2ec\uc544 \uc77c\uc5b4\ub0ac\ub2e4. \uadf8\ub9ac\uace0 1054\ub144\uc5d0\ub294 \ud574\ubb35\uc740 \uad50\ud669 \uc218\uc704\uad8c\uc758 \uc2dc\ube44\ub85c \ub85c\ub9c8 \uad50\ud68c\uc640 \ucf58\uc2a4\ud0c4\ud2f0\ub178\ud3f4\ub9ac\uc2a4 \uad50\ud68c\uac00 \uc11c\ub85c\ub97c \ud30c\ubb38\ud558\uae30\uc5d0 \uc774\ub974\ub800\ub2e4. \ub3d9\uc11c \uad50\ud68c\uc758 \ubd84\uc5f4\uc774\ub77c\uace0 \ubd88\ub9ac\ub294 \uae30\ub3c5\uad50 \uc138\uacc4\uc758 \ucd5c\ub300 \ubd84\uc5f4 \uc0ac\uac74\uc73c\ub85c \uacb0\uad6d \uad50\ud68c\ub294 \ub3d9\uc11c \uc9c0\uc5ed\uc73c\ub85c \ub2e8\uc808\ub410\ub2e4. \uc774\ud6c4 12~13\uc138\uae30 \uad50\ud669\ub4e4\uc740 \uacf5\uc758\ud68c \ub97c \uc5ec\ub7ec \ubc88 \uc18c\uc9d1\ud558\uc5ec \ub3d9\uc11c \uad50\ud68c\uc758 \uc7ac\uc77c\uce58\ub97c \uc2dc\ub3c4\ud588\uc73c\ub098 \ud070 \uc131\uacfc\ub97c \uac70\ub450\uc9c0\ub294 \ubabb\ud588\ub2e4. ", "paragraph_answer": "\ub3d9\ubc29 \uad50\ud68c\uc640 \uc11c\ubc29 \uad50\ud68c\ub294 1054\ub144\uc5d0 \uacb0\uc815\uc801\uc73c\ub85c \uc11c\ub85c \uac08\ub77c\uc11c\uac8c \ub418\uc5c8\ub2e4. \uc774 \ubd84\uc5f4\uc740 \uad50\ub9ac\uc801 \uc774\uacac\uc774\ub77c\uae30 \ubcf4\ub2e4\ub294 \uc815\uce58\uc801\uc778 \uc774\ud574\uad00\uacc4 \ub54c\ubb38\uc5d0 \ubc1c\uc0dd\ud55c \uac83\uc774\ub2e4. \uad50\ud669\ub4e4\uc740 \ub77c\ubca4\ub098 \ucd1d\ub3c5\ubd80\ub97c \ud1b5\ud574 \uc0ac\uc0ac\uac74\uac74 \uac1c\uc785\ud558\ub824 \ub4dc\ub294 \ub3d9\ub85c\ub9c8 \ud669\uc81c\uc758 \uac04\uc12d\uc73c\ub85c\ubd80\ud130 \ubc97\uc5b4\ub098\uae30 \uc704\ud574 \ud504\ub791\ud06c \uad6d\uc655\uc744 \ud669\uc81c\ub85c \ucd94\ub300\ud558\uc5ec \ub9de\uc11c\ub3c4\ub85d \ud558\uc600\ub2e4. \uc591\uce21\uc758 \uac08\ub4f1\uc740 8\uc138\uae30 \uc131\uc0c1 \ud30c\uad34 \uc6b4\ub3d9 \uc774\ub798 \ud45c\uba74\ud654\ub418\uc5c8\uc73c\uba70, 9\uc138\uae30\uc5d0\ub294 \ud544\ub9ac\uc624\ucf00 \ubb38\uc81c\uc640 \uadf8\uc5d0 \ub530\ub978 \ud3ec\ud2f0\uc6b0\uc2a4\uc758 \ubd84\ub9ac \ub4f1\uc5d0\uc11c \ub9cc\uc131\uc801\uc778 \ubd88\ud654\uac00 \uc787\ub2ec\uc544 \uc77c\uc5b4\ub0ac\ub2e4. \uadf8\ub9ac\uace0 1054\ub144\uc5d0\ub294 \ud574\ubb35\uc740 \uad50\ud669 \uc218\uc704\uad8c\uc758 \uc2dc\ube44\ub85c \ub85c\ub9c8 \uad50\ud68c\uc640 \ucf58\uc2a4\ud0c4\ud2f0\ub178\ud3f4\ub9ac\uc2a4 \uad50\ud68c\uac00 \uc11c\ub85c\ub97c \ud30c\ubb38\ud558\uae30\uc5d0 \uc774\ub974\ub800\ub2e4. \ub3d9\uc11c \uad50\ud68c\uc758 \ubd84\uc5f4\uc774\ub77c\uace0 \ubd88\ub9ac\ub294 \uae30\ub3c5\uad50 \uc138\uacc4\uc758 \ucd5c\ub300 \ubd84\uc5f4 \uc0ac\uac74\uc73c\ub85c \uacb0\uad6d \uad50\ud68c\ub294 \ub3d9\uc11c \uc9c0\uc5ed\uc73c\ub85c \ub2e8\uc808\ub410\ub2e4. \uc774\ud6c4 12~13\uc138\uae30 \uad50\ud669\ub4e4\uc740 \uacf5\uc758\ud68c \ub97c \uc5ec\ub7ec \ubc88 \uc18c\uc9d1\ud558\uc5ec \ub3d9\uc11c \uad50\ud68c\uc758 \uc7ac\uc77c\uce58\ub97c \uc2dc\ub3c4\ud588\uc73c\ub098 \ud070 \uc131\uacfc\ub97c \uac70\ub450\uc9c0\ub294 \ubabb\ud588\ub2e4.", "sentence_answer": "\uc774\ud6c4 12~13\uc138\uae30 \uad50\ud669\ub4e4\uc740 \uacf5\uc758\ud68c \ub97c \uc5ec\ub7ec \ubc88 \uc18c\uc9d1\ud558\uc5ec \ub3d9\uc11c \uad50\ud68c\uc758 \uc7ac\uc77c\uce58\ub97c \uc2dc\ub3c4\ud588\uc73c\ub098 \ud070 \uc131\uacfc\ub97c \uac70\ub450\uc9c0\ub294 \ubabb\ud588\ub2e4.", "paragraph_id": "6475662-8-1", "paragraph_question": "question: \ub3d9\uc11c \uad50\ud68c \ubd84\uc5f4\ub85c \uc778\ud574, \uc7ac\uc77c\uce58\ub97c \uc2dc\ub3c4\ud558\uae30 \uc704\ud57412-13\uc138\uae30 \uad50\ud669\ub4e4\uc740 \ubb34\uc5c7\uc744 \uc18c\uc9d1\ud588\ub294\uac00\uc694?, context: \ub3d9\ubc29 \uad50\ud68c\uc640 \uc11c\ubc29 \uad50\ud68c\ub294 1054\ub144\uc5d0 \uacb0\uc815\uc801\uc73c\ub85c \uc11c\ub85c \uac08\ub77c\uc11c\uac8c \ub418\uc5c8\ub2e4. \uc774 \ubd84\uc5f4\uc740 \uad50\ub9ac\uc801 \uc774\uacac\uc774\ub77c\uae30 \ubcf4\ub2e4\ub294 \uc815\uce58\uc801\uc778 \uc774\ud574\uad00\uacc4 \ub54c\ubb38\uc5d0 \ubc1c\uc0dd\ud55c \uac83\uc774\ub2e4. \uad50\ud669\ub4e4\uc740 \ub77c\ubca4\ub098 \ucd1d\ub3c5\ubd80\ub97c \ud1b5\ud574 \uc0ac\uc0ac\uac74\uac74 \uac1c\uc785\ud558\ub824 \ub4dc\ub294 \ub3d9\ub85c\ub9c8 \ud669\uc81c\uc758 \uac04\uc12d\uc73c\ub85c\ubd80\ud130 \ubc97\uc5b4\ub098\uae30 \uc704\ud574 \ud504\ub791\ud06c \uad6d\uc655\uc744 \ud669\uc81c\ub85c \ucd94\ub300\ud558\uc5ec \ub9de\uc11c\ub3c4\ub85d \ud558\uc600\ub2e4. \uc591\uce21\uc758 \uac08\ub4f1\uc740 8\uc138\uae30 \uc131\uc0c1 \ud30c\uad34 \uc6b4\ub3d9 \uc774\ub798 \ud45c\uba74\ud654\ub418\uc5c8\uc73c\uba70, 9\uc138\uae30\uc5d0\ub294 \ud544\ub9ac\uc624\ucf00 \ubb38\uc81c\uc640 \uadf8\uc5d0 \ub530\ub978 \ud3ec\ud2f0\uc6b0\uc2a4\uc758 \ubd84\ub9ac \ub4f1\uc5d0\uc11c \ub9cc\uc131\uc801\uc778 \ubd88\ud654\uac00 \uc787\ub2ec\uc544 \uc77c\uc5b4\ub0ac\ub2e4. \uadf8\ub9ac\uace0 1054\ub144\uc5d0\ub294 \ud574\ubb35\uc740 \uad50\ud669 \uc218\uc704\uad8c\uc758 \uc2dc\ube44\ub85c \ub85c\ub9c8 \uad50\ud68c\uc640 \ucf58\uc2a4\ud0c4\ud2f0\ub178\ud3f4\ub9ac\uc2a4 \uad50\ud68c\uac00 \uc11c\ub85c\ub97c \ud30c\ubb38\ud558\uae30\uc5d0 \uc774\ub974\ub800\ub2e4. \ub3d9\uc11c \uad50\ud68c\uc758 \ubd84\uc5f4\uc774\ub77c\uace0 \ubd88\ub9ac\ub294 \uae30\ub3c5\uad50 \uc138\uacc4\uc758 \ucd5c\ub300 \ubd84\uc5f4 \uc0ac\uac74\uc73c\ub85c \uacb0\uad6d \uad50\ud68c\ub294 \ub3d9\uc11c \uc9c0\uc5ed\uc73c\ub85c \ub2e8\uc808\ub410\ub2e4. \uc774\ud6c4 12~13\uc138\uae30 \uad50\ud669\ub4e4\uc740 \uacf5\uc758\ud68c\ub97c \uc5ec\ub7ec \ubc88 \uc18c\uc9d1\ud558\uc5ec \ub3d9\uc11c \uad50\ud68c\uc758 \uc7ac\uc77c\uce58\ub97c \uc2dc\ub3c4\ud588\uc73c\ub098 \ud070 \uc131\uacfc\ub97c \uac70\ub450\uc9c0\ub294 \ubabb\ud588\ub2e4."} {"question": "\ub204\ubca8\uce7c\ub808\ub3c4\ub2c8\uc758 \ub3c5\ub9bd\uc744 \uc704\ud574 \ud22c\uc7c1\ud558\ub294 \ub2e8\uccb4\uc758 \uc774\ub984\uc740?", "paragraph": "\ub274\uce7c\ub808\ub3c4\ub2c8\uc544\ub294 1986\ub144\ubd80\ud130 \uc720\uc5d4\uc758 \ube44\uc790\uce58 \uc9c0\uc5ed \ub9ac\uc2a4\ud2b8\uc5d0 \uc62c\ub77c\uc788\ub2e4. \uce74\ub098\ud0a4 \ubbfc\uc871\ud574\ubc29\uc804\uc120(Front de Lib\u00e9ration Nationale Kanak Socialiste, FLNKS)\uc758 \ub3c5\ub9bd\uc744 \uc704\ud55c \ud22c\uc7c1\uc774 1985\ub144\uc5d0 \uc2dc\uc791\ub418\uc5c8\ub2e4. 1989\ub144\uc5d0 \uc554\uc0b4\ub41c \uc7a5 \ub9c8\ub9ac \ud2f0\ubc14\uc6b0(Jean-Marie Tjibaou)\uac00 \uc774\ub04c\uace0 \uc788\ub358 \uce74\ub098\ud0a4 \ubbfc\uc871\ud574\ubc29\uc804\uc120\uc740 \"Kanaky\"\uc758 \ub3c5\ub9bd\uc744 \uc8fc\uc7a5\ud558\uc600\ub2e4. \uc774 \ubd84\uc7c1\uc740 \uc6b0\ubca0\uc544\uc5d0\uc11c \uc77c\uc5b4\ub09c \uc720\ud608 \uc778\uc9c8\uadf9\uc73c\ub85c 1988\ub144\uc5d0 \uc808\uc815\uc5d0 \ub2ec\ud558\uc600\ub2e4. \uc0ac\ud68c\uc801 \ubd88\uc548\uac10\uc740 1988\ub144\uc758 \ub9c8\ud2f0\ub18d \ud611\uc57d\uacfc 1998\ub144\uc758 \ub204\uba54\uc544 \ud611\uc57d\uc5d0\uc11c \uc790\uce58\uad8c\uc744 \ud5a5\uc0c1\ud558\ub294 \ud569\uc758\ub97c \uc774\ub04c\uc5b4\ub0c8\ub2e4. \uc774 \ud611\uc57d\uc740 \ucde8\uc18c\ubd88\uac00\ub2a5\ud55c \uad8c\ub825\uc758 \uc774\uc591\uacfc \ub274\uce7c\ub808\ub3c4\ub2c8\uc544 \uc2dc\ubbfc\uad8c, \uad6d\uae30\uc640 \uac19\uc774 \ub274\uce7c\ub808\ub3c4\ub2c8\uc544\uc758 \uc815\uccb4\uc131\uc744 \uc0c1\uc9d5\ud558\ub294 \uacf5\uc2dd\uc801\uc778 \uc0c1\uc9d5, 2014\ub144 \uc774\ud6c4\uc5d0\ub294 \uc5b8\uc81c\ub098 \uac00\ub2a5\ud55c \ud504\ub791\uc2a4\ub85c\ubd80\ud130\uc758 \ub3c5\ub9bd\uacfc \uad00\ub828\ud55c \ucca8\uc608\ud55c \ubb38\uc81c\uc5d0 \ub300\ud558\uc5ec \uad6d\ubbfc\ud22c\ud45c\uc5d0 \uc0c1\uc815\ud558\ub294 \uad8c\ud55c\uc758 \uc704\uc784 \ub4f1\uc744 \uaddc\uc815\ud558\uace0 \uc788\ub2e4.", "answer": "\uce74\ub098\ud0a4 \ubbfc\uc871\ud574\ubc29\uc804\uc120", "sentence": "\uce74\ub098\ud0a4 \ubbfc\uc871\ud574\ubc29\uc804\uc120 (Front de Lib\u00e9ration Nationale Kanak Socialiste, FLNKS)\uc758 \ub3c5\ub9bd\uc744 \uc704\ud55c \ud22c\uc7c1\uc774 1985\ub144\uc5d0 \uc2dc\uc791\ub418\uc5c8\ub2e4.", "paragraph_sentence": "\ub274\uce7c\ub808\ub3c4\ub2c8\uc544\ub294 1986\ub144\ubd80\ud130 \uc720\uc5d4\uc758 \ube44\uc790\uce58 \uc9c0\uc5ed \ub9ac\uc2a4\ud2b8\uc5d0 \uc62c\ub77c\uc788\ub2e4. \uce74\ub098\ud0a4 \ubbfc\uc871\ud574\ubc29\uc804\uc120 (Front de Lib\u00e9ration Nationale Kanak Socialiste, FLNKS)\uc758 \ub3c5\ub9bd\uc744 \uc704\ud55c \ud22c\uc7c1\uc774 1985\ub144\uc5d0 \uc2dc\uc791\ub418\uc5c8\ub2e4. 1989\ub144\uc5d0 \uc554\uc0b4\ub41c \uc7a5 \ub9c8\ub9ac \ud2f0\ubc14\uc6b0(Jean-Marie Tjibaou)\uac00 \uc774\ub04c\uace0 \uc788\ub358 \uce74\ub098\ud0a4 \ubbfc\uc871\ud574\ubc29\uc804\uc120\uc740 \"Kanaky\"\uc758 \ub3c5\ub9bd\uc744 \uc8fc\uc7a5\ud558\uc600\ub2e4. \uc774 \ubd84\uc7c1\uc740 \uc6b0\ubca0\uc544\uc5d0\uc11c \uc77c\uc5b4\ub09c \uc720\ud608 \uc778\uc9c8\uadf9\uc73c\ub85c 1988\ub144\uc5d0 \uc808\uc815\uc5d0 \ub2ec\ud558\uc600\ub2e4. \uc0ac\ud68c\uc801 \ubd88\uc548\uac10\uc740 1988\ub144\uc758 \ub9c8\ud2f0\ub18d \ud611\uc57d\uacfc 1998\ub144\uc758 \ub204\uba54\uc544 \ud611\uc57d\uc5d0\uc11c \uc790\uce58\uad8c\uc744 \ud5a5\uc0c1\ud558\ub294 \ud569\uc758\ub97c \uc774\ub04c\uc5b4\ub0c8\ub2e4. \uc774 \ud611\uc57d\uc740 \ucde8\uc18c\ubd88\uac00\ub2a5\ud55c \uad8c\ub825\uc758 \uc774\uc591\uacfc \ub274\uce7c\ub808\ub3c4\ub2c8\uc544 \uc2dc\ubbfc\uad8c, \uad6d\uae30\uc640 \uac19\uc774 \ub274\uce7c\ub808\ub3c4\ub2c8\uc544\uc758 \uc815\uccb4\uc131\uc744 \uc0c1\uc9d5\ud558\ub294 \uacf5\uc2dd\uc801\uc778 \uc0c1\uc9d5, 2014\ub144 \uc774\ud6c4\uc5d0\ub294 \uc5b8\uc81c\ub098 \uac00\ub2a5\ud55c \ud504\ub791\uc2a4\ub85c\ubd80\ud130\uc758 \ub3c5\ub9bd\uacfc \uad00\ub828\ud55c \ucca8\uc608\ud55c \ubb38\uc81c\uc5d0 \ub300\ud558\uc5ec \uad6d\ubbfc\ud22c\ud45c\uc5d0 \uc0c1\uc815\ud558\ub294 \uad8c\ud55c\uc758 \uc704\uc784 \ub4f1\uc744 \uaddc\uc815\ud558\uace0 \uc788\ub2e4.", "paragraph_answer": "\ub274\uce7c\ub808\ub3c4\ub2c8\uc544\ub294 1986\ub144\ubd80\ud130 \uc720\uc5d4\uc758 \ube44\uc790\uce58 \uc9c0\uc5ed \ub9ac\uc2a4\ud2b8\uc5d0 \uc62c\ub77c\uc788\ub2e4. \uce74\ub098\ud0a4 \ubbfc\uc871\ud574\ubc29\uc804\uc120 (Front de Lib\u00e9ration Nationale Kanak Socialiste, FLNKS)\uc758 \ub3c5\ub9bd\uc744 \uc704\ud55c \ud22c\uc7c1\uc774 1985\ub144\uc5d0 \uc2dc\uc791\ub418\uc5c8\ub2e4. 1989\ub144\uc5d0 \uc554\uc0b4\ub41c \uc7a5 \ub9c8\ub9ac \ud2f0\ubc14\uc6b0(Jean-Marie Tjibaou)\uac00 \uc774\ub04c\uace0 \uc788\ub358 \uce74\ub098\ud0a4 \ubbfc\uc871\ud574\ubc29\uc804\uc120\uc740 \"Kanaky\"\uc758 \ub3c5\ub9bd\uc744 \uc8fc\uc7a5\ud558\uc600\ub2e4. \uc774 \ubd84\uc7c1\uc740 \uc6b0\ubca0\uc544\uc5d0\uc11c \uc77c\uc5b4\ub09c \uc720\ud608 \uc778\uc9c8\uadf9\uc73c\ub85c 1988\ub144\uc5d0 \uc808\uc815\uc5d0 \ub2ec\ud558\uc600\ub2e4. \uc0ac\ud68c\uc801 \ubd88\uc548\uac10\uc740 1988\ub144\uc758 \ub9c8\ud2f0\ub18d \ud611\uc57d\uacfc 1998\ub144\uc758 \ub204\uba54\uc544 \ud611\uc57d\uc5d0\uc11c \uc790\uce58\uad8c\uc744 \ud5a5\uc0c1\ud558\ub294 \ud569\uc758\ub97c \uc774\ub04c\uc5b4\ub0c8\ub2e4. \uc774 \ud611\uc57d\uc740 \ucde8\uc18c\ubd88\uac00\ub2a5\ud55c \uad8c\ub825\uc758 \uc774\uc591\uacfc \ub274\uce7c\ub808\ub3c4\ub2c8\uc544 \uc2dc\ubbfc\uad8c, \uad6d\uae30\uc640 \uac19\uc774 \ub274\uce7c\ub808\ub3c4\ub2c8\uc544\uc758 \uc815\uccb4\uc131\uc744 \uc0c1\uc9d5\ud558\ub294 \uacf5\uc2dd\uc801\uc778 \uc0c1\uc9d5, 2014\ub144 \uc774\ud6c4\uc5d0\ub294 \uc5b8\uc81c\ub098 \uac00\ub2a5\ud55c \ud504\ub791\uc2a4\ub85c\ubd80\ud130\uc758 \ub3c5\ub9bd\uacfc \uad00\ub828\ud55c \ucca8\uc608\ud55c \ubb38\uc81c\uc5d0 \ub300\ud558\uc5ec \uad6d\ubbfc\ud22c\ud45c\uc5d0 \uc0c1\uc815\ud558\ub294 \uad8c\ud55c\uc758 \uc704\uc784 \ub4f1\uc744 \uaddc\uc815\ud558\uace0 \uc788\ub2e4.", "sentence_answer": " \uce74\ub098\ud0a4 \ubbfc\uc871\ud574\ubc29\uc804\uc120 (Front de Lib\u00e9ration Nationale Kanak Socialiste, FLNKS)\uc758 \ub3c5\ub9bd\uc744 \uc704\ud55c \ud22c\uc7c1\uc774 1985\ub144\uc5d0 \uc2dc\uc791\ub418\uc5c8\ub2e4.", "paragraph_id": "6484441-2-0", "paragraph_question": "question: \ub204\ubca8\uce7c\ub808\ub3c4\ub2c8\uc758 \ub3c5\ub9bd\uc744 \uc704\ud574 \ud22c\uc7c1\ud558\ub294 \ub2e8\uccb4\uc758 \uc774\ub984\uc740?, context: \ub274\uce7c\ub808\ub3c4\ub2c8\uc544\ub294 1986\ub144\ubd80\ud130 \uc720\uc5d4\uc758 \ube44\uc790\uce58 \uc9c0\uc5ed \ub9ac\uc2a4\ud2b8\uc5d0 \uc62c\ub77c\uc788\ub2e4. \uce74\ub098\ud0a4 \ubbfc\uc871\ud574\ubc29\uc804\uc120(Front de Lib\u00e9ration Nationale Kanak Socialiste, FLNKS)\uc758 \ub3c5\ub9bd\uc744 \uc704\ud55c \ud22c\uc7c1\uc774 1985\ub144\uc5d0 \uc2dc\uc791\ub418\uc5c8\ub2e4. 1989\ub144\uc5d0 \uc554\uc0b4\ub41c \uc7a5 \ub9c8\ub9ac \ud2f0\ubc14\uc6b0(Jean-Marie Tjibaou)\uac00 \uc774\ub04c\uace0 \uc788\ub358 \uce74\ub098\ud0a4 \ubbfc\uc871\ud574\ubc29\uc804\uc120\uc740 \"Kanaky\"\uc758 \ub3c5\ub9bd\uc744 \uc8fc\uc7a5\ud558\uc600\ub2e4. \uc774 \ubd84\uc7c1\uc740 \uc6b0\ubca0\uc544\uc5d0\uc11c \uc77c\uc5b4\ub09c \uc720\ud608 \uc778\uc9c8\uadf9\uc73c\ub85c 1988\ub144\uc5d0 \uc808\uc815\uc5d0 \ub2ec\ud558\uc600\ub2e4. \uc0ac\ud68c\uc801 \ubd88\uc548\uac10\uc740 1988\ub144\uc758 \ub9c8\ud2f0\ub18d \ud611\uc57d\uacfc 1998\ub144\uc758 \ub204\uba54\uc544 \ud611\uc57d\uc5d0\uc11c \uc790\uce58\uad8c\uc744 \ud5a5\uc0c1\ud558\ub294 \ud569\uc758\ub97c \uc774\ub04c\uc5b4\ub0c8\ub2e4. \uc774 \ud611\uc57d\uc740 \ucde8\uc18c\ubd88\uac00\ub2a5\ud55c \uad8c\ub825\uc758 \uc774\uc591\uacfc \ub274\uce7c\ub808\ub3c4\ub2c8\uc544 \uc2dc\ubbfc\uad8c, \uad6d\uae30\uc640 \uac19\uc774 \ub274\uce7c\ub808\ub3c4\ub2c8\uc544\uc758 \uc815\uccb4\uc131\uc744 \uc0c1\uc9d5\ud558\ub294 \uacf5\uc2dd\uc801\uc778 \uc0c1\uc9d5, 2014\ub144 \uc774\ud6c4\uc5d0\ub294 \uc5b8\uc81c\ub098 \uac00\ub2a5\ud55c \ud504\ub791\uc2a4\ub85c\ubd80\ud130\uc758 \ub3c5\ub9bd\uacfc \uad00\ub828\ud55c \ucca8\uc608\ud55c \ubb38\uc81c\uc5d0 \ub300\ud558\uc5ec \uad6d\ubbfc\ud22c\ud45c\uc5d0 \uc0c1\uc815\ud558\ub294 \uad8c\ud55c\uc758 \uc704\uc784 \ub4f1\uc744 \uaddc\uc815\ud558\uace0 \uc788\ub2e4."} {"question": "\ubcfc\ub799\ud640\uc740 \uc774\ub860\uc0c1\uc758 \ud638\ud0b9\ubcf5\uc0ac\ub97c \uc81c\uc678\ud55c, \uc9c1\uc811\uc801\uc778 \ubb34\uc5c7\uc744 \ubc29\ucd9c\ud558\uc9c0 \uc54a\ub098\uc694?", "paragraph": "\ube14\ub799\ud640\uc740 \uadf8 \uc131\uc9c8\uc0c1 \uc774\ub860\uc0c1\uc758 \ud638\ud0b9 \ubcf5\uc0ac\ub97c \uc81c\uc678\ud558\uba74 \uc9c1\uc811\uc801\uc778 \uc804\uc790\uae30 \ubcf5\uc0ac\ub97c \ubc29\ucd9c\ud558\uc9c0 \uc54a\uc73c\uba70, \ub54c\ubb38\uc5d0 \ube14\ub799\ud640\uc744 \ucc3e\ub294 \ucc9c\uccb4\ubb3c\ub9ac\ud559\uc790\ub4e4\uc740 \uac04\uc811\uc801 \uad00\uce21\uc218\ub2e8\uc5d0 \uc758\uc874\ud574\uc57c \ud55c\ub2e4. \uc608\ucee8\ub300 \ube14\ub799\ud640 \uc8fc\uc704\uc758 \uc911\ub825\uc801 \uc0c1\ud638\uc791\uc6a9\uc744 \ud1b5\ud574 \ube14\ub799\ud640\uc758 \uc874\uc7ac\ub97c \uc54c\uc544\ub0b4\ub294 \uac83\uc774 \uadf8\ub7ec\ud55c \uac04\uc811\uc801 \uad00\uce21\uc218\ub2e8 \uc911 \ud558\ub098\uc774\ub2e4. \uadf8\ub7ec\ub098 MIT \ud5e4\uc774\uc2a4\ud0dd \ucc9c\ubb38\ub300\uc758 \uc0ac\uac74\uc758 \uc9c0\ud3c9\uc120 \ub9dd\uc6d0\uacbd(EHT)\ub294 \uc6b0\ub9ac\uc740\ud558 \uc911\uc2ec\uc758 \uad81\uc218\uc790\ub9ac A* \ube14\ub799\ud640\uc5d0 \ub300\ud55c \uc9c1\uc811 \uad00\uce21\uc744 \uc2dc\ub3c4 \uc911\uc774\uba70, 2016\ub144 \ucd08\uc5d0\ub294 \ucd5c\ucd08\ub85c \uc0ac\uac74\uc758 \uc9c0\ud3c9\uc120\uc744 \ud3ec\ucc29\ud55c \uc0ac\uc9c4\uc744 \uc5bb\uc744 \uc218 \uc788\uc744 \uac83\uc73c\ub85c \uc608\uc0c1\ud558\uace0 \uc788\ub2e4. \uad81\uc218\uc790\ub9ac A*\uc758 \uc0ac\uac74\uc758 \uc9c0\ud3c9\uc120 \ubc14\ub85c \ubc14\uae65\uc5d0 \uc790\uae30\uc7a5\uc774 \uc874\uc7ac\ud560 \uac83\uc774\ub77c\ub294 \uc774\ub860\uc801 \uc608\uc0c1\uc740 2015\ub144 EHT\uc5d0 \uc758\ud574 \uc0ac\uc2e4\ub85c \ubc1d\ud600\uc84c\ub2e4.", "answer": "\uc804\uc790\uae30 \ubcf5\uc0ac", "sentence": "\ube14\ub799\ud640\uc740 \uadf8 \uc131\uc9c8\uc0c1 \uc774\ub860\uc0c1\uc758 \ud638\ud0b9 \ubcf5\uc0ac\ub97c \uc81c\uc678\ud558\uba74 \uc9c1\uc811\uc801\uc778 \uc804\uc790\uae30 \ubcf5\uc0ac \ub97c \ubc29\ucd9c\ud558\uc9c0 \uc54a\uc73c\uba70, \ub54c\ubb38\uc5d0 \ube14\ub799\ud640\uc744 \ucc3e\ub294 \ucc9c\uccb4\ubb3c\ub9ac\ud559\uc790\ub4e4\uc740 \uac04\uc811\uc801 \uad00\uce21\uc218\ub2e8\uc5d0 \uc758\uc874\ud574\uc57c \ud55c\ub2e4.", "paragraph_sentence": " \ube14\ub799\ud640\uc740 \uadf8 \uc131\uc9c8\uc0c1 \uc774\ub860\uc0c1\uc758 \ud638\ud0b9 \ubcf5\uc0ac\ub97c \uc81c\uc678\ud558\uba74 \uc9c1\uc811\uc801\uc778 \uc804\uc790\uae30 \ubcf5\uc0ac \ub97c \ubc29\ucd9c\ud558\uc9c0 \uc54a\uc73c\uba70, \ub54c\ubb38\uc5d0 \ube14\ub799\ud640\uc744 \ucc3e\ub294 \ucc9c\uccb4\ubb3c\ub9ac\ud559\uc790\ub4e4\uc740 \uac04\uc811\uc801 \uad00\uce21\uc218\ub2e8\uc5d0 \uc758\uc874\ud574\uc57c \ud55c\ub2e4. \uc608\ucee8\ub300 \ube14\ub799\ud640 \uc8fc\uc704\uc758 \uc911\ub825\uc801 \uc0c1\ud638\uc791\uc6a9\uc744 \ud1b5\ud574 \ube14\ub799\ud640\uc758 \uc874\uc7ac\ub97c \uc54c\uc544\ub0b4\ub294 \uac83\uc774 \uadf8\ub7ec\ud55c \uac04\uc811\uc801 \uad00\uce21\uc218\ub2e8 \uc911 \ud558\ub098\uc774\ub2e4. \uadf8\ub7ec\ub098 MIT \ud5e4\uc774\uc2a4\ud0dd \ucc9c\ubb38\ub300\uc758 \uc0ac\uac74\uc758 \uc9c0\ud3c9\uc120 \ub9dd\uc6d0\uacbd(EHT)\ub294 \uc6b0\ub9ac\uc740\ud558 \uc911\uc2ec\uc758 \uad81\uc218\uc790\ub9ac A* \ube14\ub799\ud640\uc5d0 \ub300\ud55c \uc9c1\uc811 \uad00\uce21\uc744 \uc2dc\ub3c4 \uc911\uc774\uba70, 2016\ub144 \ucd08\uc5d0\ub294 \ucd5c\ucd08\ub85c \uc0ac\uac74\uc758 \uc9c0\ud3c9\uc120\uc744 \ud3ec\ucc29\ud55c \uc0ac\uc9c4\uc744 \uc5bb\uc744 \uc218 \uc788\uc744 \uac83\uc73c\ub85c \uc608\uc0c1\ud558\uace0 \uc788\ub2e4. \uad81\uc218\uc790\ub9ac A*\uc758 \uc0ac\uac74\uc758 \uc9c0\ud3c9\uc120 \ubc14\ub85c \ubc14\uae65\uc5d0 \uc790\uae30\uc7a5\uc774 \uc874\uc7ac\ud560 \uac83\uc774\ub77c\ub294 \uc774\ub860\uc801 \uc608\uc0c1\uc740 2015\ub144 EHT\uc5d0 \uc758\ud574 \uc0ac\uc2e4\ub85c \ubc1d\ud600\uc84c\ub2e4.", "paragraph_answer": "\ube14\ub799\ud640\uc740 \uadf8 \uc131\uc9c8\uc0c1 \uc774\ub860\uc0c1\uc758 \ud638\ud0b9 \ubcf5\uc0ac\ub97c \uc81c\uc678\ud558\uba74 \uc9c1\uc811\uc801\uc778 \uc804\uc790\uae30 \ubcf5\uc0ac \ub97c \ubc29\ucd9c\ud558\uc9c0 \uc54a\uc73c\uba70, \ub54c\ubb38\uc5d0 \ube14\ub799\ud640\uc744 \ucc3e\ub294 \ucc9c\uccb4\ubb3c\ub9ac\ud559\uc790\ub4e4\uc740 \uac04\uc811\uc801 \uad00\uce21\uc218\ub2e8\uc5d0 \uc758\uc874\ud574\uc57c \ud55c\ub2e4. \uc608\ucee8\ub300 \ube14\ub799\ud640 \uc8fc\uc704\uc758 \uc911\ub825\uc801 \uc0c1\ud638\uc791\uc6a9\uc744 \ud1b5\ud574 \ube14\ub799\ud640\uc758 \uc874\uc7ac\ub97c \uc54c\uc544\ub0b4\ub294 \uac83\uc774 \uadf8\ub7ec\ud55c \uac04\uc811\uc801 \uad00\uce21\uc218\ub2e8 \uc911 \ud558\ub098\uc774\ub2e4. \uadf8\ub7ec\ub098 MIT \ud5e4\uc774\uc2a4\ud0dd \ucc9c\ubb38\ub300\uc758 \uc0ac\uac74\uc758 \uc9c0\ud3c9\uc120 \ub9dd\uc6d0\uacbd(EHT)\ub294 \uc6b0\ub9ac\uc740\ud558 \uc911\uc2ec\uc758 \uad81\uc218\uc790\ub9ac A* \ube14\ub799\ud640\uc5d0 \ub300\ud55c \uc9c1\uc811 \uad00\uce21\uc744 \uc2dc\ub3c4 \uc911\uc774\uba70, 2016\ub144 \ucd08\uc5d0\ub294 \ucd5c\ucd08\ub85c \uc0ac\uac74\uc758 \uc9c0\ud3c9\uc120\uc744 \ud3ec\ucc29\ud55c \uc0ac\uc9c4\uc744 \uc5bb\uc744 \uc218 \uc788\uc744 \uac83\uc73c\ub85c \uc608\uc0c1\ud558\uace0 \uc788\ub2e4. \uad81\uc218\uc790\ub9ac A*\uc758 \uc0ac\uac74\uc758 \uc9c0\ud3c9\uc120 \ubc14\ub85c \ubc14\uae65\uc5d0 \uc790\uae30\uc7a5\uc774 \uc874\uc7ac\ud560 \uac83\uc774\ub77c\ub294 \uc774\ub860\uc801 \uc608\uc0c1\uc740 2015\ub144 EHT\uc5d0 \uc758\ud574 \uc0ac\uc2e4\ub85c \ubc1d\ud600\uc84c\ub2e4.", "sentence_answer": "\ube14\ub799\ud640\uc740 \uadf8 \uc131\uc9c8\uc0c1 \uc774\ub860\uc0c1\uc758 \ud638\ud0b9 \ubcf5\uc0ac\ub97c \uc81c\uc678\ud558\uba74 \uc9c1\uc811\uc801\uc778 \uc804\uc790\uae30 \ubcf5\uc0ac \ub97c \ubc29\ucd9c\ud558\uc9c0 \uc54a\uc73c\uba70, \ub54c\ubb38\uc5d0 \ube14\ub799\ud640\uc744 \ucc3e\ub294 \ucc9c\uccb4\ubb3c\ub9ac\ud559\uc790\ub4e4\uc740 \uac04\uc811\uc801 \uad00\uce21\uc218\ub2e8\uc5d0 \uc758\uc874\ud574\uc57c \ud55c\ub2e4.", "paragraph_id": "6583541-11-0", "paragraph_question": "question: \ubcfc\ub799\ud640\uc740 \uc774\ub860\uc0c1\uc758 \ud638\ud0b9\ubcf5\uc0ac\ub97c \uc81c\uc678\ud55c, \uc9c1\uc811\uc801\uc778 \ubb34\uc5c7\uc744 \ubc29\ucd9c\ud558\uc9c0 \uc54a\ub098\uc694?, context: \ube14\ub799\ud640\uc740 \uadf8 \uc131\uc9c8\uc0c1 \uc774\ub860\uc0c1\uc758 \ud638\ud0b9 \ubcf5\uc0ac\ub97c \uc81c\uc678\ud558\uba74 \uc9c1\uc811\uc801\uc778 \uc804\uc790\uae30 \ubcf5\uc0ac\ub97c \ubc29\ucd9c\ud558\uc9c0 \uc54a\uc73c\uba70, \ub54c\ubb38\uc5d0 \ube14\ub799\ud640\uc744 \ucc3e\ub294 \ucc9c\uccb4\ubb3c\ub9ac\ud559\uc790\ub4e4\uc740 \uac04\uc811\uc801 \uad00\uce21\uc218\ub2e8\uc5d0 \uc758\uc874\ud574\uc57c \ud55c\ub2e4. \uc608\ucee8\ub300 \ube14\ub799\ud640 \uc8fc\uc704\uc758 \uc911\ub825\uc801 \uc0c1\ud638\uc791\uc6a9\uc744 \ud1b5\ud574 \ube14\ub799\ud640\uc758 \uc874\uc7ac\ub97c \uc54c\uc544\ub0b4\ub294 \uac83\uc774 \uadf8\ub7ec\ud55c \uac04\uc811\uc801 \uad00\uce21\uc218\ub2e8 \uc911 \ud558\ub098\uc774\ub2e4. \uadf8\ub7ec\ub098 MIT \ud5e4\uc774\uc2a4\ud0dd \ucc9c\ubb38\ub300\uc758 \uc0ac\uac74\uc758 \uc9c0\ud3c9\uc120 \ub9dd\uc6d0\uacbd(EHT)\ub294 \uc6b0\ub9ac\uc740\ud558 \uc911\uc2ec\uc758 \uad81\uc218\uc790\ub9ac A* \ube14\ub799\ud640\uc5d0 \ub300\ud55c \uc9c1\uc811 \uad00\uce21\uc744 \uc2dc\ub3c4 \uc911\uc774\uba70, 2016\ub144 \ucd08\uc5d0\ub294 \ucd5c\ucd08\ub85c \uc0ac\uac74\uc758 \uc9c0\ud3c9\uc120\uc744 \ud3ec\ucc29\ud55c \uc0ac\uc9c4\uc744 \uc5bb\uc744 \uc218 \uc788\uc744 \uac83\uc73c\ub85c \uc608\uc0c1\ud558\uace0 \uc788\ub2e4. \uad81\uc218\uc790\ub9ac A*\uc758 \uc0ac\uac74\uc758 \uc9c0\ud3c9\uc120 \ubc14\ub85c \ubc14\uae65\uc5d0 \uc790\uae30\uc7a5\uc774 \uc874\uc7ac\ud560 \uac83\uc774\ub77c\ub294 \uc774\ub860\uc801 \uc608\uc0c1\uc740 2015\ub144 EHT\uc5d0 \uc758\ud574 \uc0ac\uc2e4\ub85c \ubc1d\ud600\uc84c\ub2e4."} {"question": "\uc5f0\uadf9\uc73c\ub85c\ub294 \ud074\ub77c\uc774\ub9e5\uc2a4\uc774\uc9c0\ub9cc \uc74c\uc545\uc73c\ub85c\ub294 \uc5b4\ub5a4 \ubd80\ubd84\uc5d0 \uc788\uc5b4\uc11c \uc791\uace1\uc790\ub098 \uc5f0\ucd9c\uac00\ub4e4\uc774 \uc624\ub958\ub97c \ubc94\ud560 \uc218 \uc788\ub294\uac00?", "paragraph": "\ub300\uc0ac\u00b7\ucc3d\u00b7\ub3d9\uc791\uc73c\ub85c \uc5ee\uc5b4\uc11c \uc77c\uad00 \uc5f0\uc2b5\uc5d0 \ub4e4\uc5b4\uac00 \ud558\ub098\uc758 \uc5f0\uc18d\uc801\uc778 \uc5f0\uadf9\uc758 \ud750\ub984\uc774 \uc774\ub8e8\uc5b4\uc9c8 \ub54c \ub300\ubcf8\uc774 \uac16\ub294 \uc5f0\uadf9\uc73c\ub85c\uc11c\uc758 \ud15c\ud3ec\u00b7\ub9ac\ub4ec\uacfc \ucc3d\uadf9\uc73c\ub85c\uc11c\uc758 \ud15c\ud3ec\u00b7\ub9ac\ub4ec\uc758 \ubd80\ubd84\uc801\uc778 \ucc28\uc774\uac00 \uc5ec\uae30\uc800\uae30\uc11c \ub178\ucd9c\ub41c\ub2e4. \uc5f0\uadf9\uc801\uc778 \ud15c\ud3ec\uac00 \uc74c\uc545\uc131\uc744 \uc555\ubc15\ud558\uac70\ub098 \uc74c\uc545\uc801\uc778 \ud45c\ud604\uc758 \uac15\uc870\uac00 \uc5f0\uadf9\uc801\uc778 \uae34\ubc15\uac10 \ub0b4\uc9c0 \uc0ac\uc2e4\uc131\uc744 \uc774\uc644\uc2dc\ud0a4\ub294 \uc774\uc728\ubc30\ubc18\uc774 \ub530\ub974\ub294 \uc218\uac00 \uc788\ub2e4. \uc774\ub54c\uc5d0 \uc5b4\ub290 \uac83\uc744 \ud0dd\ud558\uc5ec \ud558\ub098\ub97c \uc6b0\uc704\uc5d0 \ub458 \uac83\uc778\uc9c0 \ud639\uc740 \uc591\uc790\ub97c \uc808\ucda9\ud560 \uac83\uc778\uc9c0\ub294 \uc5f0\ucd9c\uc790\uc758 \uc7ac\ub7c9\uc774\uaca0\uc9c0\ub9cc, \uc885\ud569\uc608\uc220\ub85c\uc11c\uc758 \ucc3d\uadf9\uc758 \uc608\uc220\uc801 \ucc3d\uc870\ub825\uc774 \ud310\ub2e8\uc758 \uae30\uc900\uc774 \ub41c\ub2e4. \uc21c\uc218 \uc5f0\uadf9\uc5d0\uc11c\ub294 \ub2e8\uc21c\ud55c \ub3d9\uc791\uc758 \ub418\ud480\uc774\uac00 \uc9c0\ub8e8\ud55c \uc0ac\uc871\uc774 \ub420 \uc218 \uc788\uc73c\ub098 \uc5ec\uae30\uc5d0 \uc74c\uc545\uc744 \ub3c4\uc785\ud560 \ub54c\ub294 \uc810\uc99d\uc801\uc778 \uace0\uc870\uc758 \uc218\ub2e8\uc774 \ub418\uae30\ub3c4 \ud558\ub294 \uac83\uc774 \uc885\ud569\uc608\uc220\uc758 \ub9c8\ub825\uc774\ub2e4. \uc989 \ucc3d\uadf9\uc758 \ud750\ub984\uc740 \uc5f0\uadf9\uad6c\uc131\uc801\uc778 \ud750\ub984\uacfc \uc74c\uc545\uad6c\uc131\uc801\uc778 \ud750\ub984\uc784\uc744 \uc5fc\ub450\uc5d0 \ub450\uc5b4\uc57c \ud55c\ub2e4. \uc5f0\uadf9\uc73c\ub85c \ubcf4\uc544\uc11c\ub294 \ud074\ub77c\uc774\ub9e5\uc2a4\uc778\ub370, \uc74c\uc545\uc73c\ub85c \ubd10\uc11c\ub294 \uc774\uc644\ub418\ub294 \ubd88\uade0\ud615\uc740 \uc791\uace1\uc790\ub098 \uc5f0\ucd9c\uac00\ub4e4\uc774 \ubc94\ud558\uae30 \uc26c\uc6b4 \ucc3d\uadf9\uc758 \ud568\uc815\uc774\ub2e4.", "answer": "\uc774\uc644\ub418\ub294 \ubd88\uade0\ud615", "sentence": "\uc5f0\uadf9\uc73c\ub85c \ubcf4\uc544\uc11c\ub294 \ud074\ub77c\uc774\ub9e5\uc2a4\uc778\ub370, \uc74c\uc545\uc73c\ub85c \ubd10\uc11c\ub294 \uc774\uc644\ub418\ub294 \ubd88\uade0\ud615 \uc740 \uc791\uace1\uc790\ub098 \uc5f0\ucd9c\uac00\ub4e4\uc774 \ubc94\ud558\uae30 \uc26c\uc6b4 \ucc3d\uadf9\uc758 \ud568\uc815\uc774\ub2e4.", "paragraph_sentence": "\ub300\uc0ac\u00b7\ucc3d\u00b7\ub3d9\uc791\uc73c\ub85c \uc5ee\uc5b4\uc11c \uc77c\uad00 \uc5f0\uc2b5\uc5d0 \ub4e4\uc5b4\uac00 \ud558\ub098\uc758 \uc5f0\uc18d\uc801\uc778 \uc5f0\uadf9\uc758 \ud750\ub984\uc774 \uc774\ub8e8\uc5b4\uc9c8 \ub54c \ub300\ubcf8\uc774 \uac16\ub294 \uc5f0\uadf9\uc73c\ub85c\uc11c\uc758 \ud15c\ud3ec\u00b7\ub9ac\ub4ec\uacfc \ucc3d\uadf9\uc73c\ub85c\uc11c\uc758 \ud15c\ud3ec\u00b7\ub9ac\ub4ec\uc758 \ubd80\ubd84\uc801\uc778 \ucc28\uc774\uac00 \uc5ec\uae30\uc800\uae30\uc11c \ub178\ucd9c\ub41c\ub2e4. \uc5f0\uadf9\uc801\uc778 \ud15c\ud3ec\uac00 \uc74c\uc545\uc131\uc744 \uc555\ubc15\ud558\uac70\ub098 \uc74c\uc545\uc801\uc778 \ud45c\ud604\uc758 \uac15\uc870\uac00 \uc5f0\uadf9\uc801\uc778 \uae34\ubc15\uac10 \ub0b4\uc9c0 \uc0ac\uc2e4\uc131\uc744 \uc774\uc644\uc2dc\ud0a4\ub294 \uc774\uc728\ubc30\ubc18\uc774 \ub530\ub974\ub294 \uc218\uac00 \uc788\ub2e4. \uc774\ub54c\uc5d0 \uc5b4\ub290 \uac83\uc744 \ud0dd\ud558\uc5ec \ud558\ub098\ub97c \uc6b0\uc704\uc5d0 \ub458 \uac83\uc778\uc9c0 \ud639\uc740 \uc591\uc790\ub97c \uc808\ucda9\ud560 \uac83\uc778\uc9c0\ub294 \uc5f0\ucd9c\uc790\uc758 \uc7ac\ub7c9\uc774\uaca0\uc9c0\ub9cc, \uc885\ud569\uc608\uc220\ub85c\uc11c\uc758 \ucc3d\uadf9\uc758 \uc608\uc220\uc801 \ucc3d\uc870\ub825\uc774 \ud310\ub2e8\uc758 \uae30\uc900\uc774 \ub41c\ub2e4. \uc21c\uc218 \uc5f0\uadf9\uc5d0\uc11c\ub294 \ub2e8\uc21c\ud55c \ub3d9\uc791\uc758 \ub418\ud480\uc774\uac00 \uc9c0\ub8e8\ud55c \uc0ac\uc871\uc774 \ub420 \uc218 \uc788\uc73c\ub098 \uc5ec\uae30\uc5d0 \uc74c\uc545\uc744 \ub3c4\uc785\ud560 \ub54c\ub294 \uc810\uc99d\uc801\uc778 \uace0\uc870\uc758 \uc218\ub2e8\uc774 \ub418\uae30\ub3c4 \ud558\ub294 \uac83\uc774 \uc885\ud569\uc608\uc220\uc758 \ub9c8\ub825\uc774\ub2e4. \uc989 \ucc3d\uadf9\uc758 \ud750\ub984\uc740 \uc5f0\uadf9\uad6c\uc131\uc801\uc778 \ud750\ub984\uacfc \uc74c\uc545\uad6c\uc131\uc801\uc778 \ud750\ub984\uc784\uc744 \uc5fc\ub450\uc5d0 \ub450\uc5b4\uc57c \ud55c\ub2e4. \uc5f0\uadf9\uc73c\ub85c \ubcf4\uc544\uc11c\ub294 \ud074\ub77c\uc774\ub9e5\uc2a4\uc778\ub370, \uc74c\uc545\uc73c\ub85c \ubd10\uc11c\ub294 \uc774\uc644\ub418\ub294 \ubd88\uade0\ud615 \uc740 \uc791\uace1\uc790\ub098 \uc5f0\ucd9c\uac00\ub4e4\uc774 \ubc94\ud558\uae30 \uc26c\uc6b4 \ucc3d\uadf9\uc758 \ud568\uc815\uc774\ub2e4. ", "paragraph_answer": "\ub300\uc0ac\u00b7\ucc3d\u00b7\ub3d9\uc791\uc73c\ub85c \uc5ee\uc5b4\uc11c \uc77c\uad00 \uc5f0\uc2b5\uc5d0 \ub4e4\uc5b4\uac00 \ud558\ub098\uc758 \uc5f0\uc18d\uc801\uc778 \uc5f0\uadf9\uc758 \ud750\ub984\uc774 \uc774\ub8e8\uc5b4\uc9c8 \ub54c \ub300\ubcf8\uc774 \uac16\ub294 \uc5f0\uadf9\uc73c\ub85c\uc11c\uc758 \ud15c\ud3ec\u00b7\ub9ac\ub4ec\uacfc \ucc3d\uadf9\uc73c\ub85c\uc11c\uc758 \ud15c\ud3ec\u00b7\ub9ac\ub4ec\uc758 \ubd80\ubd84\uc801\uc778 \ucc28\uc774\uac00 \uc5ec\uae30\uc800\uae30\uc11c \ub178\ucd9c\ub41c\ub2e4. \uc5f0\uadf9\uc801\uc778 \ud15c\ud3ec\uac00 \uc74c\uc545\uc131\uc744 \uc555\ubc15\ud558\uac70\ub098 \uc74c\uc545\uc801\uc778 \ud45c\ud604\uc758 \uac15\uc870\uac00 \uc5f0\uadf9\uc801\uc778 \uae34\ubc15\uac10 \ub0b4\uc9c0 \uc0ac\uc2e4\uc131\uc744 \uc774\uc644\uc2dc\ud0a4\ub294 \uc774\uc728\ubc30\ubc18\uc774 \ub530\ub974\ub294 \uc218\uac00 \uc788\ub2e4. \uc774\ub54c\uc5d0 \uc5b4\ub290 \uac83\uc744 \ud0dd\ud558\uc5ec \ud558\ub098\ub97c \uc6b0\uc704\uc5d0 \ub458 \uac83\uc778\uc9c0 \ud639\uc740 \uc591\uc790\ub97c \uc808\ucda9\ud560 \uac83\uc778\uc9c0\ub294 \uc5f0\ucd9c\uc790\uc758 \uc7ac\ub7c9\uc774\uaca0\uc9c0\ub9cc, \uc885\ud569\uc608\uc220\ub85c\uc11c\uc758 \ucc3d\uadf9\uc758 \uc608\uc220\uc801 \ucc3d\uc870\ub825\uc774 \ud310\ub2e8\uc758 \uae30\uc900\uc774 \ub41c\ub2e4. \uc21c\uc218 \uc5f0\uadf9\uc5d0\uc11c\ub294 \ub2e8\uc21c\ud55c \ub3d9\uc791\uc758 \ub418\ud480\uc774\uac00 \uc9c0\ub8e8\ud55c \uc0ac\uc871\uc774 \ub420 \uc218 \uc788\uc73c\ub098 \uc5ec\uae30\uc5d0 \uc74c\uc545\uc744 \ub3c4\uc785\ud560 \ub54c\ub294 \uc810\uc99d\uc801\uc778 \uace0\uc870\uc758 \uc218\ub2e8\uc774 \ub418\uae30\ub3c4 \ud558\ub294 \uac83\uc774 \uc885\ud569\uc608\uc220\uc758 \ub9c8\ub825\uc774\ub2e4. \uc989 \ucc3d\uadf9\uc758 \ud750\ub984\uc740 \uc5f0\uadf9\uad6c\uc131\uc801\uc778 \ud750\ub984\uacfc \uc74c\uc545\uad6c\uc131\uc801\uc778 \ud750\ub984\uc784\uc744 \uc5fc\ub450\uc5d0 \ub450\uc5b4\uc57c \ud55c\ub2e4. \uc5f0\uadf9\uc73c\ub85c \ubcf4\uc544\uc11c\ub294 \ud074\ub77c\uc774\ub9e5\uc2a4\uc778\ub370, \uc74c\uc545\uc73c\ub85c \ubd10\uc11c\ub294 \uc774\uc644\ub418\ub294 \ubd88\uade0\ud615 \uc740 \uc791\uace1\uc790\ub098 \uc5f0\ucd9c\uac00\ub4e4\uc774 \ubc94\ud558\uae30 \uc26c\uc6b4 \ucc3d\uadf9\uc758 \ud568\uc815\uc774\ub2e4.", "sentence_answer": "\uc5f0\uadf9\uc73c\ub85c \ubcf4\uc544\uc11c\ub294 \ud074\ub77c\uc774\ub9e5\uc2a4\uc778\ub370, \uc74c\uc545\uc73c\ub85c \ubd10\uc11c\ub294 \uc774\uc644\ub418\ub294 \ubd88\uade0\ud615 \uc740 \uc791\uace1\uc790\ub098 \uc5f0\ucd9c\uac00\ub4e4\uc774 \ubc94\ud558\uae30 \uc26c\uc6b4 \ucc3d\uadf9\uc758 \ud568\uc815\uc774\ub2e4.", "paragraph_id": "6557859-9-2", "paragraph_question": "question: \uc5f0\uadf9\uc73c\ub85c\ub294 \ud074\ub77c\uc774\ub9e5\uc2a4\uc774\uc9c0\ub9cc \uc74c\uc545\uc73c\ub85c\ub294 \uc5b4\ub5a4 \ubd80\ubd84\uc5d0 \uc788\uc5b4\uc11c \uc791\uace1\uc790\ub098 \uc5f0\ucd9c\uac00\ub4e4\uc774 \uc624\ub958\ub97c \ubc94\ud560 \uc218 \uc788\ub294\uac00?, context: \ub300\uc0ac\u00b7\ucc3d\u00b7\ub3d9\uc791\uc73c\ub85c \uc5ee\uc5b4\uc11c \uc77c\uad00 \uc5f0\uc2b5\uc5d0 \ub4e4\uc5b4\uac00 \ud558\ub098\uc758 \uc5f0\uc18d\uc801\uc778 \uc5f0\uadf9\uc758 \ud750\ub984\uc774 \uc774\ub8e8\uc5b4\uc9c8 \ub54c \ub300\ubcf8\uc774 \uac16\ub294 \uc5f0\uadf9\uc73c\ub85c\uc11c\uc758 \ud15c\ud3ec\u00b7\ub9ac\ub4ec\uacfc \ucc3d\uadf9\uc73c\ub85c\uc11c\uc758 \ud15c\ud3ec\u00b7\ub9ac\ub4ec\uc758 \ubd80\ubd84\uc801\uc778 \ucc28\uc774\uac00 \uc5ec\uae30\uc800\uae30\uc11c \ub178\ucd9c\ub41c\ub2e4. \uc5f0\uadf9\uc801\uc778 \ud15c\ud3ec\uac00 \uc74c\uc545\uc131\uc744 \uc555\ubc15\ud558\uac70\ub098 \uc74c\uc545\uc801\uc778 \ud45c\ud604\uc758 \uac15\uc870\uac00 \uc5f0\uadf9\uc801\uc778 \uae34\ubc15\uac10 \ub0b4\uc9c0 \uc0ac\uc2e4\uc131\uc744 \uc774\uc644\uc2dc\ud0a4\ub294 \uc774\uc728\ubc30\ubc18\uc774 \ub530\ub974\ub294 \uc218\uac00 \uc788\ub2e4. \uc774\ub54c\uc5d0 \uc5b4\ub290 \uac83\uc744 \ud0dd\ud558\uc5ec \ud558\ub098\ub97c \uc6b0\uc704\uc5d0 \ub458 \uac83\uc778\uc9c0 \ud639\uc740 \uc591\uc790\ub97c \uc808\ucda9\ud560 \uac83\uc778\uc9c0\ub294 \uc5f0\ucd9c\uc790\uc758 \uc7ac\ub7c9\uc774\uaca0\uc9c0\ub9cc, \uc885\ud569\uc608\uc220\ub85c\uc11c\uc758 \ucc3d\uadf9\uc758 \uc608\uc220\uc801 \ucc3d\uc870\ub825\uc774 \ud310\ub2e8\uc758 \uae30\uc900\uc774 \ub41c\ub2e4. \uc21c\uc218 \uc5f0\uadf9\uc5d0\uc11c\ub294 \ub2e8\uc21c\ud55c \ub3d9\uc791\uc758 \ub418\ud480\uc774\uac00 \uc9c0\ub8e8\ud55c \uc0ac\uc871\uc774 \ub420 \uc218 \uc788\uc73c\ub098 \uc5ec\uae30\uc5d0 \uc74c\uc545\uc744 \ub3c4\uc785\ud560 \ub54c\ub294 \uc810\uc99d\uc801\uc778 \uace0\uc870\uc758 \uc218\ub2e8\uc774 \ub418\uae30\ub3c4 \ud558\ub294 \uac83\uc774 \uc885\ud569\uc608\uc220\uc758 \ub9c8\ub825\uc774\ub2e4. \uc989 \ucc3d\uadf9\uc758 \ud750\ub984\uc740 \uc5f0\uadf9\uad6c\uc131\uc801\uc778 \ud750\ub984\uacfc \uc74c\uc545\uad6c\uc131\uc801\uc778 \ud750\ub984\uc784\uc744 \uc5fc\ub450\uc5d0 \ub450\uc5b4\uc57c \ud55c\ub2e4. \uc5f0\uadf9\uc73c\ub85c \ubcf4\uc544\uc11c\ub294 \ud074\ub77c\uc774\ub9e5\uc2a4\uc778\ub370, \uc74c\uc545\uc73c\ub85c \ubd10\uc11c\ub294 \uc774\uc644\ub418\ub294 \ubd88\uade0\ud615\uc740 \uc791\uace1\uc790\ub098 \uc5f0\ucd9c\uac00\ub4e4\uc774 \ubc94\ud558\uae30 \uc26c\uc6b4 \ucc3d\uadf9\uc758 \ud568\uc815\uc774\ub2e4."} {"question": "\ubbf8\uc8fc\ub9ac \uc8fc \uc138\uc778\ud2b8 \ub8e8\uc774\uc2a4\uc5d0\uc11c \uc911\uc0b0\uce35\uc774 \ub9ce\uc774 \uac70\uc8fc\ud588\ub358 \uc774 \uc9c0\uc5ed\uc740?", "paragraph": "\ubbf8\uc8fc\ub9ac \uc8fc \uc138\uc778\ud2b8 \ub8e8\uc774\uc2a4\uc5d0\uc11c \ud0dc\uc5b4\ub09c \ubca0\ub9ac\ub294 \ubd80\ubaa8\uc758 \uc5ec\uc12f \uc790\uc2dd \uc911 \ub137\uc9f8\uc600\ub2e4. \ubd81 \uc138\uc778\ud2b8 \ub8e8\uc774\uc2a4 \uadfc\uad50\uc758 \uc77c\uba85 '\ub354 \ube4c\ub808(The Ville)'\uc5d0\uc11c \uc790\ub790\ub2e4. \uc774\uacf3\uc740 \ub2f9\uc2dc \uc911\uc0b0\uce35 \uc0ac\ub78c\ub4e4\uc774 \ub9ce\uc774 \uac70\uc8fc\ud558\ub358 \uacf3\uc774\uc5c8\ub2e4. \uadf8\uc758 \uc544\ubc84\uc9c0\uc778 \ud578\ub9ac\ub294 \uacc4\uc57d\uc790\uc774\uc790 \uadfc\ucc98 \uce68\ub840\uad50\ub3c4 \uad50\ud68c\uc758 \ubd80\uc81c\uc600\ub2e4. \uadf8\uc758 \uc5b4\uba38\ub2c8 \ub9c8\uc0ac\ub294 \uacf5\uc778\ub41c \uc0ac\ub9bd\ud559\uad50\uc758 \uad50\uc7a5\uc774\uc5c8\ub2e4. \uc544\ubc84\uc9c0\uc758 \ud6c8\uc721\uc5d0 \uc758\ud574 \ubca0\ub9ac\uc758 \uc74c\uc545\uc5d0 \ub300\ud55c \uad00\uc2ec\uc740 \uc5b4\ub9b4 \uc801\ubd80\ud130 \ucee4\uc838\uc654\uc73c\uba70, \uadf8\ub294 1941\ub144 \ubca0\ub9ac\uac00 \uc12c\ub108 \ud558\uc774 \uc2a4\ucfe8 \uc7ac\ud559 \uc2dc\uae30\uc5d0 \uacf5\uc5f0\uc744 \ud560 \uc218 \uc788\ub3c4\ub85d \ud588\ub2e4. 1944\ub144 \uc12c\ub108 \ud558\uc774 \uc2a4\ucfe8\uc5d0 \uc7ac\ud559\ud558\uace0 \uc788\uc744 \ub2f9\uc2dc \ubbf8\uc8fc\ub9ac \uc8fc\uc758 \uce94\uc790\uc2a4 \uc2dc\ud2f0\uc5d0 \uc788\ub294 \uc0c1\uc810 \uc138 \uacf3\uc5d0\uc11c \ubb34\uc7a5 \uac15\ub3c4\ub97c \ubc8c\uc774\uace0 \ucd1d\uc744 \uaca8\ub204\uc5b4 \uce5c\uad6c\ub4e4\uacfc \ud568\uaed8 \ucc28\ub97c \ud6d4\uce5c \ud610\uc758\ub85c \uccb4\ud3ec\ub418\uc5c8\ub2e4. \ubca0\ub9ac\uc758 \uc790\uc11c\uc804\uc5d0\uc11c \uadf8\uac00 \ubc1d\ud78c \ubc14\ub85c\ub294, \uadf8\uc758 \ucc28\uac00 \uace0\uc7a5\ub098\uc11c \uc9c0\ub098\uac00\ub294 \ucc28\ub97c \uc791\ub3d9\ud558\uc9c0 \uc54a\ub294 \ucd1d\uc73c\ub85c \uc704\ud611\ud558\uc5ec \ud6d4\ucce4\ub2e4\uace0 \ud55c\ub2e4. \uc720\uc8c4\ub97c \uc120\uace0\ubc1b\uc740 \uadf8\ub294 \ubbf8\uc8fc\ub9ac \uc8fc\uc758 \uc81c\ud37c\uc2a8 \uc2dc\ud2f0 \uadfc\uad50\uc758 \uc54c\uace0\uc544\uc5d0 \uc788\ub294 \uc911\uae09 \uc18c\ub144\uc6d0\uc5d0 \ub4e4\uc5b4\uac00\uac8c \ub418\uc5c8\ub2e4. \uc5ec\uae30\uc11c \ubca0\ub9ac\ub294 4\uc911\uc8fc\ub85c \ub178\ub798\ub97c \ubd80\ub974\uac70\ub098 \uad8c\ud22c\ub97c \ud588\ub2e4. \uadf8\uac00 \uc788\ub358 \uadf8\ub8f9\uc740 \ub2f9\uad6d \uce21\uc5d0\uc11c \uc2dc\uc124 \ubc16\uc5d0\uc11c \uacf5\uc5f0\uc744 \ud574\ub3c4 \uc88b\ub2e4\ub294 \ud5c8\ub77d\uc774 \ub098\uc654\uc744 \uc815\ub3c4\ub85c \uc2e4\ub825\uc774 \ub2a5\uc219\ud574\uc84c\ub2e4. \ubca0\ub9ac\ub294 \uc790\uc2e0\uc758 21\ubc88\uc9f8 \uc0dd\uc77c\uc5d0 \uc18c\ub144\uc6d0\uc5d0\uc11c \ucd9c\uc18c\ub418\uc5c8\ub2e4.", "answer": "\ub354 \ube4c\ub808(The Ville)", "sentence": "\ubd81 \uc138\uc778\ud2b8 \ub8e8\uc774\uc2a4 \uadfc\uad50\uc758 \uc77c\uba85 ' \ub354 \ube4c\ub808(The Ville) '\uc5d0\uc11c \uc790\ub790\ub2e4.", "paragraph_sentence": "\ubbf8\uc8fc\ub9ac \uc8fc \uc138\uc778\ud2b8 \ub8e8\uc774\uc2a4\uc5d0\uc11c \ud0dc\uc5b4\ub09c \ubca0\ub9ac\ub294 \ubd80\ubaa8\uc758 \uc5ec\uc12f \uc790\uc2dd \uc911 \ub137\uc9f8\uc600\ub2e4. \ubd81 \uc138\uc778\ud2b8 \ub8e8\uc774\uc2a4 \uadfc\uad50\uc758 \uc77c\uba85 ' \ub354 \ube4c\ub808(The Ville) '\uc5d0\uc11c \uc790\ub790\ub2e4. \uc774\uacf3\uc740 \ub2f9\uc2dc \uc911\uc0b0\uce35 \uc0ac\ub78c\ub4e4\uc774 \ub9ce\uc774 \uac70\uc8fc\ud558\ub358 \uacf3\uc774\uc5c8\ub2e4. \uadf8\uc758 \uc544\ubc84\uc9c0\uc778 \ud578\ub9ac\ub294 \uacc4\uc57d\uc790\uc774\uc790 \uadfc\ucc98 \uce68\ub840\uad50\ub3c4 \uad50\ud68c\uc758 \ubd80\uc81c\uc600\ub2e4. \uadf8\uc758 \uc5b4\uba38\ub2c8 \ub9c8\uc0ac\ub294 \uacf5\uc778\ub41c \uc0ac\ub9bd\ud559\uad50\uc758 \uad50\uc7a5\uc774\uc5c8\ub2e4. \uc544\ubc84\uc9c0\uc758 \ud6c8\uc721\uc5d0 \uc758\ud574 \ubca0\ub9ac\uc758 \uc74c\uc545\uc5d0 \ub300\ud55c \uad00\uc2ec\uc740 \uc5b4\ub9b4 \uc801\ubd80\ud130 \ucee4\uc838\uc654\uc73c\uba70, \uadf8\ub294 1941\ub144 \ubca0\ub9ac\uac00 \uc12c\ub108 \ud558\uc774 \uc2a4\ucfe8 \uc7ac\ud559 \uc2dc\uae30\uc5d0 \uacf5\uc5f0\uc744 \ud560 \uc218 \uc788\ub3c4\ub85d \ud588\ub2e4. 1944\ub144 \uc12c\ub108 \ud558\uc774 \uc2a4\ucfe8\uc5d0 \uc7ac\ud559\ud558\uace0 \uc788\uc744 \ub2f9\uc2dc \ubbf8\uc8fc\ub9ac \uc8fc\uc758 \uce94\uc790\uc2a4 \uc2dc\ud2f0\uc5d0 \uc788\ub294 \uc0c1\uc810 \uc138 \uacf3\uc5d0\uc11c \ubb34\uc7a5 \uac15\ub3c4\ub97c \ubc8c\uc774\uace0 \ucd1d\uc744 \uaca8\ub204\uc5b4 \uce5c\uad6c\ub4e4\uacfc \ud568\uaed8 \ucc28\ub97c \ud6d4\uce5c \ud610\uc758\ub85c \uccb4\ud3ec\ub418\uc5c8\ub2e4. \ubca0\ub9ac\uc758 \uc790\uc11c\uc804\uc5d0\uc11c \uadf8\uac00 \ubc1d\ud78c \ubc14\ub85c\ub294, \uadf8\uc758 \ucc28\uac00 \uace0\uc7a5\ub098\uc11c \uc9c0\ub098\uac00\ub294 \ucc28\ub97c \uc791\ub3d9\ud558\uc9c0 \uc54a\ub294 \ucd1d\uc73c\ub85c \uc704\ud611\ud558\uc5ec \ud6d4\ucce4\ub2e4\uace0 \ud55c\ub2e4. \uc720\uc8c4\ub97c \uc120\uace0\ubc1b\uc740 \uadf8\ub294 \ubbf8\uc8fc\ub9ac \uc8fc\uc758 \uc81c\ud37c\uc2a8 \uc2dc\ud2f0 \uadfc\uad50\uc758 \uc54c\uace0\uc544\uc5d0 \uc788\ub294 \uc911\uae09 \uc18c\ub144\uc6d0\uc5d0 \ub4e4\uc5b4\uac00\uac8c \ub418\uc5c8\ub2e4. \uc5ec\uae30\uc11c \ubca0\ub9ac\ub294 4\uc911\uc8fc\ub85c \ub178\ub798\ub97c \ubd80\ub974\uac70\ub098 \uad8c\ud22c\ub97c \ud588\ub2e4. \uadf8\uac00 \uc788\ub358 \uadf8\ub8f9\uc740 \ub2f9\uad6d \uce21\uc5d0\uc11c \uc2dc\uc124 \ubc16\uc5d0\uc11c \uacf5\uc5f0\uc744 \ud574\ub3c4 \uc88b\ub2e4\ub294 \ud5c8\ub77d\uc774 \ub098\uc654\uc744 \uc815\ub3c4\ub85c \uc2e4\ub825\uc774 \ub2a5\uc219\ud574\uc84c\ub2e4. \ubca0\ub9ac\ub294 \uc790\uc2e0\uc758 21\ubc88\uc9f8 \uc0dd\uc77c\uc5d0 \uc18c\ub144\uc6d0\uc5d0\uc11c \ucd9c\uc18c\ub418\uc5c8\ub2e4.", "paragraph_answer": "\ubbf8\uc8fc\ub9ac \uc8fc \uc138\uc778\ud2b8 \ub8e8\uc774\uc2a4\uc5d0\uc11c \ud0dc\uc5b4\ub09c \ubca0\ub9ac\ub294 \ubd80\ubaa8\uc758 \uc5ec\uc12f \uc790\uc2dd \uc911 \ub137\uc9f8\uc600\ub2e4. \ubd81 \uc138\uc778\ud2b8 \ub8e8\uc774\uc2a4 \uadfc\uad50\uc758 \uc77c\uba85 ' \ub354 \ube4c\ub808(The Ville) '\uc5d0\uc11c \uc790\ub790\ub2e4. \uc774\uacf3\uc740 \ub2f9\uc2dc \uc911\uc0b0\uce35 \uc0ac\ub78c\ub4e4\uc774 \ub9ce\uc774 \uac70\uc8fc\ud558\ub358 \uacf3\uc774\uc5c8\ub2e4. \uadf8\uc758 \uc544\ubc84\uc9c0\uc778 \ud578\ub9ac\ub294 \uacc4\uc57d\uc790\uc774\uc790 \uadfc\ucc98 \uce68\ub840\uad50\ub3c4 \uad50\ud68c\uc758 \ubd80\uc81c\uc600\ub2e4. \uadf8\uc758 \uc5b4\uba38\ub2c8 \ub9c8\uc0ac\ub294 \uacf5\uc778\ub41c \uc0ac\ub9bd\ud559\uad50\uc758 \uad50\uc7a5\uc774\uc5c8\ub2e4. \uc544\ubc84\uc9c0\uc758 \ud6c8\uc721\uc5d0 \uc758\ud574 \ubca0\ub9ac\uc758 \uc74c\uc545\uc5d0 \ub300\ud55c \uad00\uc2ec\uc740 \uc5b4\ub9b4 \uc801\ubd80\ud130 \ucee4\uc838\uc654\uc73c\uba70, \uadf8\ub294 1941\ub144 \ubca0\ub9ac\uac00 \uc12c\ub108 \ud558\uc774 \uc2a4\ucfe8 \uc7ac\ud559 \uc2dc\uae30\uc5d0 \uacf5\uc5f0\uc744 \ud560 \uc218 \uc788\ub3c4\ub85d \ud588\ub2e4. 1944\ub144 \uc12c\ub108 \ud558\uc774 \uc2a4\ucfe8\uc5d0 \uc7ac\ud559\ud558\uace0 \uc788\uc744 \ub2f9\uc2dc \ubbf8\uc8fc\ub9ac \uc8fc\uc758 \uce94\uc790\uc2a4 \uc2dc\ud2f0\uc5d0 \uc788\ub294 \uc0c1\uc810 \uc138 \uacf3\uc5d0\uc11c \ubb34\uc7a5 \uac15\ub3c4\ub97c \ubc8c\uc774\uace0 \ucd1d\uc744 \uaca8\ub204\uc5b4 \uce5c\uad6c\ub4e4\uacfc \ud568\uaed8 \ucc28\ub97c \ud6d4\uce5c \ud610\uc758\ub85c \uccb4\ud3ec\ub418\uc5c8\ub2e4. \ubca0\ub9ac\uc758 \uc790\uc11c\uc804\uc5d0\uc11c \uadf8\uac00 \ubc1d\ud78c \ubc14\ub85c\ub294, \uadf8\uc758 \ucc28\uac00 \uace0\uc7a5\ub098\uc11c \uc9c0\ub098\uac00\ub294 \ucc28\ub97c \uc791\ub3d9\ud558\uc9c0 \uc54a\ub294 \ucd1d\uc73c\ub85c \uc704\ud611\ud558\uc5ec \ud6d4\ucce4\ub2e4\uace0 \ud55c\ub2e4. \uc720\uc8c4\ub97c \uc120\uace0\ubc1b\uc740 \uadf8\ub294 \ubbf8\uc8fc\ub9ac \uc8fc\uc758 \uc81c\ud37c\uc2a8 \uc2dc\ud2f0 \uadfc\uad50\uc758 \uc54c\uace0\uc544\uc5d0 \uc788\ub294 \uc911\uae09 \uc18c\ub144\uc6d0\uc5d0 \ub4e4\uc5b4\uac00\uac8c \ub418\uc5c8\ub2e4. \uc5ec\uae30\uc11c \ubca0\ub9ac\ub294 4\uc911\uc8fc\ub85c \ub178\ub798\ub97c \ubd80\ub974\uac70\ub098 \uad8c\ud22c\ub97c \ud588\ub2e4. \uadf8\uac00 \uc788\ub358 \uadf8\ub8f9\uc740 \ub2f9\uad6d \uce21\uc5d0\uc11c \uc2dc\uc124 \ubc16\uc5d0\uc11c \uacf5\uc5f0\uc744 \ud574\ub3c4 \uc88b\ub2e4\ub294 \ud5c8\ub77d\uc774 \ub098\uc654\uc744 \uc815\ub3c4\ub85c \uc2e4\ub825\uc774 \ub2a5\uc219\ud574\uc84c\ub2e4. \ubca0\ub9ac\ub294 \uc790\uc2e0\uc758 21\ubc88\uc9f8 \uc0dd\uc77c\uc5d0 \uc18c\ub144\uc6d0\uc5d0\uc11c \ucd9c\uc18c\ub418\uc5c8\ub2e4.", "sentence_answer": "\ubd81 \uc138\uc778\ud2b8 \ub8e8\uc774\uc2a4 \uadfc\uad50\uc758 \uc77c\uba85 ' \ub354 \ube4c\ub808(The Ville) '\uc5d0\uc11c \uc790\ub790\ub2e4.", "paragraph_id": "6203196-2-0", "paragraph_question": "question: \ubbf8\uc8fc\ub9ac \uc8fc \uc138\uc778\ud2b8 \ub8e8\uc774\uc2a4\uc5d0\uc11c \uc911\uc0b0\uce35\uc774 \ub9ce\uc774 \uac70\uc8fc\ud588\ub358 \uc774 \uc9c0\uc5ed\uc740?, context: \ubbf8\uc8fc\ub9ac \uc8fc \uc138\uc778\ud2b8 \ub8e8\uc774\uc2a4\uc5d0\uc11c \ud0dc\uc5b4\ub09c \ubca0\ub9ac\ub294 \ubd80\ubaa8\uc758 \uc5ec\uc12f \uc790\uc2dd \uc911 \ub137\uc9f8\uc600\ub2e4. \ubd81 \uc138\uc778\ud2b8 \ub8e8\uc774\uc2a4 \uadfc\uad50\uc758 \uc77c\uba85 '\ub354 \ube4c\ub808(The Ville)'\uc5d0\uc11c \uc790\ub790\ub2e4. \uc774\uacf3\uc740 \ub2f9\uc2dc \uc911\uc0b0\uce35 \uc0ac\ub78c\ub4e4\uc774 \ub9ce\uc774 \uac70\uc8fc\ud558\ub358 \uacf3\uc774\uc5c8\ub2e4. \uadf8\uc758 \uc544\ubc84\uc9c0\uc778 \ud578\ub9ac\ub294 \uacc4\uc57d\uc790\uc774\uc790 \uadfc\ucc98 \uce68\ub840\uad50\ub3c4 \uad50\ud68c\uc758 \ubd80\uc81c\uc600\ub2e4. \uadf8\uc758 \uc5b4\uba38\ub2c8 \ub9c8\uc0ac\ub294 \uacf5\uc778\ub41c \uc0ac\ub9bd\ud559\uad50\uc758 \uad50\uc7a5\uc774\uc5c8\ub2e4. \uc544\ubc84\uc9c0\uc758 \ud6c8\uc721\uc5d0 \uc758\ud574 \ubca0\ub9ac\uc758 \uc74c\uc545\uc5d0 \ub300\ud55c \uad00\uc2ec\uc740 \uc5b4\ub9b4 \uc801\ubd80\ud130 \ucee4\uc838\uc654\uc73c\uba70, \uadf8\ub294 1941\ub144 \ubca0\ub9ac\uac00 \uc12c\ub108 \ud558\uc774 \uc2a4\ucfe8 \uc7ac\ud559 \uc2dc\uae30\uc5d0 \uacf5\uc5f0\uc744 \ud560 \uc218 \uc788\ub3c4\ub85d \ud588\ub2e4. 1944\ub144 \uc12c\ub108 \ud558\uc774 \uc2a4\ucfe8\uc5d0 \uc7ac\ud559\ud558\uace0 \uc788\uc744 \ub2f9\uc2dc \ubbf8\uc8fc\ub9ac \uc8fc\uc758 \uce94\uc790\uc2a4 \uc2dc\ud2f0\uc5d0 \uc788\ub294 \uc0c1\uc810 \uc138 \uacf3\uc5d0\uc11c \ubb34\uc7a5 \uac15\ub3c4\ub97c \ubc8c\uc774\uace0 \ucd1d\uc744 \uaca8\ub204\uc5b4 \uce5c\uad6c\ub4e4\uacfc \ud568\uaed8 \ucc28\ub97c \ud6d4\uce5c \ud610\uc758\ub85c \uccb4\ud3ec\ub418\uc5c8\ub2e4. \ubca0\ub9ac\uc758 \uc790\uc11c\uc804\uc5d0\uc11c \uadf8\uac00 \ubc1d\ud78c \ubc14\ub85c\ub294, \uadf8\uc758 \ucc28\uac00 \uace0\uc7a5\ub098\uc11c \uc9c0\ub098\uac00\ub294 \ucc28\ub97c \uc791\ub3d9\ud558\uc9c0 \uc54a\ub294 \ucd1d\uc73c\ub85c \uc704\ud611\ud558\uc5ec \ud6d4\ucce4\ub2e4\uace0 \ud55c\ub2e4. \uc720\uc8c4\ub97c \uc120\uace0\ubc1b\uc740 \uadf8\ub294 \ubbf8\uc8fc\ub9ac \uc8fc\uc758 \uc81c\ud37c\uc2a8 \uc2dc\ud2f0 \uadfc\uad50\uc758 \uc54c\uace0\uc544\uc5d0 \uc788\ub294 \uc911\uae09 \uc18c\ub144\uc6d0\uc5d0 \ub4e4\uc5b4\uac00\uac8c \ub418\uc5c8\ub2e4. \uc5ec\uae30\uc11c \ubca0\ub9ac\ub294 4\uc911\uc8fc\ub85c \ub178\ub798\ub97c \ubd80\ub974\uac70\ub098 \uad8c\ud22c\ub97c \ud588\ub2e4. \uadf8\uac00 \uc788\ub358 \uadf8\ub8f9\uc740 \ub2f9\uad6d \uce21\uc5d0\uc11c \uc2dc\uc124 \ubc16\uc5d0\uc11c \uacf5\uc5f0\uc744 \ud574\ub3c4 \uc88b\ub2e4\ub294 \ud5c8\ub77d\uc774 \ub098\uc654\uc744 \uc815\ub3c4\ub85c \uc2e4\ub825\uc774 \ub2a5\uc219\ud574\uc84c\ub2e4. \ubca0\ub9ac\ub294 \uc790\uc2e0\uc758 21\ubc88\uc9f8 \uc0dd\uc77c\uc5d0 \uc18c\ub144\uc6d0\uc5d0\uc11c \ucd9c\uc18c\ub418\uc5c8\ub2e4."} {"question": "\uac8c\uc784\uc740 \ubb34\uc5c7\uc5d0 \ub300\ud55c \uc0ac\uc2e4\uc801\uc778 \ubb18\uc0ac\ub85c \ud638\ud3c9\uc744 \ubc1b\uc558\ub294\uac00?", "paragraph": "\uc774 \uac8c\uc784\uc740 \uada4\ub3c4\uc5ed\ud559\uc5d0 \ub300\ud55c \uc0ac\uc2e4\uc801\uc778 \ubb18\uc0ac\ub85c \ud638\ud3c9\uc744 \ubc1b\uc558\ub2e4. \ub610\ud55c \uace0\uc815\ub41c \uada4\ub3c4\ub97c \ub3c4\ub294 \ucc9c\uccb4\ub4e4\uc744 \uc81c\uc678\ud558\uba74, \ubaa8\ub4e0 \ubb3c\uccb4\ub4e4\uc740 \ub274\ud134 \uc5ed\ud559 \uc2dc\ubbac\ub808\uc774\uc158\uc758 \uc601\ud5a5 \uc544\ub798\uc5d0 \ub193\uc778\ub2e4. \ub85c\ucf13\uc758 \ucd94\ub825\uc774\ub098 \ube44\ud589\uccb4\uc758 \uc591\ub825\uc740 \uae30\uccb4\uc758 \uc5b4\ub290 \uc704\uce58\uc5d0 \uc5d4\uc9c4 \ubc0f \ub0a0\uac1c\uac00 \ubd99\uc5b4\uc788\ub294\uc9c0\uc5d0 \ub9de\ucdb0\uc11c \uc815\ud655\ud788 \uc791\uc6a9\ud55c\ub2e4. \uc870\ub9bd\ud574\ub193\uc740 \ubd80\ud488\uac04\uc758 \uacb0\ud569\ub825 \ub610\ud55c \ud55c\uacc4\uac00 \uc788\uc5b4\uc11c \ub108\ubb34 \uc2ec\ud558\uac70\ub098 \uc5b4\uae0b\ub09c \ubc29\ud5a5\uc758 \ud798\uc774 \uae30\uccb4\uc5d0 \uac00\ud574\uc9c8 \uacbd\uc6b0, \uae30\uccb4\uac00 \uc0b0\uc0b0\uc774 \ubd80\uc11c\uc838\ubc84\ub9ac\uae30\ub3c4 \ud55c\ub2e4. \uadf8\ub9ac\uace0 \uac8c\uc784 \ub0b4 \ucc9c\uccb4\ub4e4 \uc911, \ucee4\ube48\uc744 \ud3ec\ud568\ud574 \uac01\uae30 \ub2e4\ub978 \uc870\uac74\uc758 \ub300\uae30\uac00 \uc874\uc7ac\ud558\ub294 \ucc9c\uccb4\ub4e4\uc5d0\uc11c\ub294 \ub192\uc774\uc640 \ubc00\ub3c4\uc5d0 \ub530\ub77c\uc11c \ub0a0\uac1c\uc758 \uc591\ub825 \ubc0f \ub099\ud558\uc0b0 \uc804\uac1c \ud6a8\uacfc, \ud56d\ub825\uc774 \uc601\ud5a5\uc744 \ubc1b\ub294\ub2e4. \uc2dc\ubbac\ub808\uc774\uc158\uc740 \uc0c1\ub2f9\ud788 \uc815\uad50\ud574\uc11c, \u2018\ud638\ub9cc \uc804\uc774 \uada4\ub3c4\u2019\ub098 \u2018\uc5d0\uc5b4\ub85c \ube0c\ub808\uc774\ud0b9\u2019\ub4f1 \ud604\uc2e4 \uc138\uacc4\uc758 \uc6b0\uc8fc\ube44\ud589\uae30\uc220\ub4e4\uc774 \ucee4\ubcfc\uacc4 \ud0d0\uc0ac\uc5d0\uc11c\ub3c4 \uc0ac\uc6a9 \uac00\ub2a5\ud558\ub2e4. \ub2e4\ub9cc \ud604\uc7ac \uac8c\uc784\ubc84\uc804\uc5d0\uc11c\ub294 \uada4\ub3c4 \uc5f0\uc0b0\uc744 \uc644\uc804\ud55c N\uccb4 \uc2dc\ubbac\ub808\uc774\uc158 \ub300\uc2e0 Patched conic approximation\uc744 \uc0ac\uc6a9\ud558\uace0 \uc788\uae30 \ub54c\ubb38\uc5d0, \ub77c\uadf8\ub791\uc8fc \uc810\uc774\ub098 \ud5e4\uc77c\ub85c \uada4\ub3c4\uc640 \uac19\uc740 \ud604\uc0c1\uc740 \ubb18\uc0ac\ud558\uc9c0 \ubabb\ud55c\ub2e4.", "answer": "\uada4\ub3c4\uc5ed\ud559", "sentence": "\uc774 \uac8c\uc784\uc740 \uada4\ub3c4\uc5ed\ud559 \uc5d0 \ub300\ud55c \uc0ac\uc2e4\uc801\uc778 \ubb18\uc0ac\ub85c \ud638\ud3c9\uc744 \ubc1b\uc558\ub2e4.", "paragraph_sentence": " \uc774 \uac8c\uc784\uc740 \uada4\ub3c4\uc5ed\ud559 \uc5d0 \ub300\ud55c \uc0ac\uc2e4\uc801\uc778 \ubb18\uc0ac\ub85c \ud638\ud3c9\uc744 \ubc1b\uc558\ub2e4. \ub610\ud55c \uace0\uc815\ub41c \uada4\ub3c4\ub97c \ub3c4\ub294 \ucc9c\uccb4\ub4e4\uc744 \uc81c\uc678\ud558\uba74, \ubaa8\ub4e0 \ubb3c\uccb4\ub4e4\uc740 \ub274\ud134 \uc5ed\ud559 \uc2dc\ubbac\ub808\uc774\uc158\uc758 \uc601\ud5a5 \uc544\ub798\uc5d0 \ub193\uc778\ub2e4. \ub85c\ucf13\uc758 \ucd94\ub825\uc774\ub098 \ube44\ud589\uccb4\uc758 \uc591\ub825\uc740 \uae30\uccb4\uc758 \uc5b4\ub290 \uc704\uce58\uc5d0 \uc5d4\uc9c4 \ubc0f \ub0a0\uac1c\uac00 \ubd99\uc5b4\uc788\ub294\uc9c0\uc5d0 \ub9de\ucdb0\uc11c \uc815\ud655\ud788 \uc791\uc6a9\ud55c\ub2e4. \uc870\ub9bd\ud574\ub193\uc740 \ubd80\ud488\uac04\uc758 \uacb0\ud569\ub825 \ub610\ud55c \ud55c\uacc4\uac00 \uc788\uc5b4\uc11c \ub108\ubb34 \uc2ec\ud558\uac70\ub098 \uc5b4\uae0b\ub09c \ubc29\ud5a5\uc758 \ud798\uc774 \uae30\uccb4\uc5d0 \uac00\ud574\uc9c8 \uacbd\uc6b0, \uae30\uccb4\uac00 \uc0b0\uc0b0\uc774 \ubd80\uc11c\uc838\ubc84\ub9ac\uae30\ub3c4 \ud55c\ub2e4. \uadf8\ub9ac\uace0 \uac8c\uc784 \ub0b4 \ucc9c\uccb4\ub4e4 \uc911, \ucee4\ube48\uc744 \ud3ec\ud568\ud574 \uac01\uae30 \ub2e4\ub978 \uc870\uac74\uc758 \ub300\uae30\uac00 \uc874\uc7ac\ud558\ub294 \ucc9c\uccb4\ub4e4\uc5d0\uc11c\ub294 \ub192\uc774\uc640 \ubc00\ub3c4\uc5d0 \ub530\ub77c\uc11c \ub0a0\uac1c\uc758 \uc591\ub825 \ubc0f \ub099\ud558\uc0b0 \uc804\uac1c \ud6a8\uacfc, \ud56d\ub825\uc774 \uc601\ud5a5\uc744 \ubc1b\ub294\ub2e4. \uc2dc\ubbac\ub808\uc774\uc158\uc740 \uc0c1\ub2f9\ud788 \uc815\uad50\ud574\uc11c, \u2018\ud638\ub9cc \uc804\uc774 \uada4\ub3c4\u2019\ub098 \u2018\uc5d0\uc5b4\ub85c \ube0c\ub808\uc774\ud0b9\u2019\ub4f1 \ud604\uc2e4 \uc138\uacc4\uc758 \uc6b0\uc8fc\ube44\ud589\uae30\uc220\ub4e4\uc774 \ucee4\ubcfc\uacc4 \ud0d0\uc0ac\uc5d0\uc11c\ub3c4 \uc0ac\uc6a9 \uac00\ub2a5\ud558\ub2e4. \ub2e4\ub9cc \ud604\uc7ac \uac8c\uc784\ubc84\uc804\uc5d0\uc11c\ub294 \uada4\ub3c4 \uc5f0\uc0b0\uc744 \uc644\uc804\ud55c N\uccb4 \uc2dc\ubbac\ub808\uc774\uc158 \ub300\uc2e0 Patched conic approximation\uc744 \uc0ac\uc6a9\ud558\uace0 \uc788\uae30 \ub54c\ubb38\uc5d0, \ub77c\uadf8\ub791\uc8fc \uc810\uc774\ub098 \ud5e4\uc77c\ub85c \uada4\ub3c4\uc640 \uac19\uc740 \ud604\uc0c1\uc740 \ubb18\uc0ac\ud558\uc9c0 \ubabb\ud55c\ub2e4.", "paragraph_answer": "\uc774 \uac8c\uc784\uc740 \uada4\ub3c4\uc5ed\ud559 \uc5d0 \ub300\ud55c \uc0ac\uc2e4\uc801\uc778 \ubb18\uc0ac\ub85c \ud638\ud3c9\uc744 \ubc1b\uc558\ub2e4. \ub610\ud55c \uace0\uc815\ub41c \uada4\ub3c4\ub97c \ub3c4\ub294 \ucc9c\uccb4\ub4e4\uc744 \uc81c\uc678\ud558\uba74, \ubaa8\ub4e0 \ubb3c\uccb4\ub4e4\uc740 \ub274\ud134 \uc5ed\ud559 \uc2dc\ubbac\ub808\uc774\uc158\uc758 \uc601\ud5a5 \uc544\ub798\uc5d0 \ub193\uc778\ub2e4. \ub85c\ucf13\uc758 \ucd94\ub825\uc774\ub098 \ube44\ud589\uccb4\uc758 \uc591\ub825\uc740 \uae30\uccb4\uc758 \uc5b4\ub290 \uc704\uce58\uc5d0 \uc5d4\uc9c4 \ubc0f \ub0a0\uac1c\uac00 \ubd99\uc5b4\uc788\ub294\uc9c0\uc5d0 \ub9de\ucdb0\uc11c \uc815\ud655\ud788 \uc791\uc6a9\ud55c\ub2e4. \uc870\ub9bd\ud574\ub193\uc740 \ubd80\ud488\uac04\uc758 \uacb0\ud569\ub825 \ub610\ud55c \ud55c\uacc4\uac00 \uc788\uc5b4\uc11c \ub108\ubb34 \uc2ec\ud558\uac70\ub098 \uc5b4\uae0b\ub09c \ubc29\ud5a5\uc758 \ud798\uc774 \uae30\uccb4\uc5d0 \uac00\ud574\uc9c8 \uacbd\uc6b0, \uae30\uccb4\uac00 \uc0b0\uc0b0\uc774 \ubd80\uc11c\uc838\ubc84\ub9ac\uae30\ub3c4 \ud55c\ub2e4. \uadf8\ub9ac\uace0 \uac8c\uc784 \ub0b4 \ucc9c\uccb4\ub4e4 \uc911, \ucee4\ube48\uc744 \ud3ec\ud568\ud574 \uac01\uae30 \ub2e4\ub978 \uc870\uac74\uc758 \ub300\uae30\uac00 \uc874\uc7ac\ud558\ub294 \ucc9c\uccb4\ub4e4\uc5d0\uc11c\ub294 \ub192\uc774\uc640 \ubc00\ub3c4\uc5d0 \ub530\ub77c\uc11c \ub0a0\uac1c\uc758 \uc591\ub825 \ubc0f \ub099\ud558\uc0b0 \uc804\uac1c \ud6a8\uacfc, \ud56d\ub825\uc774 \uc601\ud5a5\uc744 \ubc1b\ub294\ub2e4. \uc2dc\ubbac\ub808\uc774\uc158\uc740 \uc0c1\ub2f9\ud788 \uc815\uad50\ud574\uc11c, \u2018\ud638\ub9cc \uc804\uc774 \uada4\ub3c4\u2019\ub098 \u2018\uc5d0\uc5b4\ub85c \ube0c\ub808\uc774\ud0b9\u2019\ub4f1 \ud604\uc2e4 \uc138\uacc4\uc758 \uc6b0\uc8fc\ube44\ud589\uae30\uc220\ub4e4\uc774 \ucee4\ubcfc\uacc4 \ud0d0\uc0ac\uc5d0\uc11c\ub3c4 \uc0ac\uc6a9 \uac00\ub2a5\ud558\ub2e4. \ub2e4\ub9cc \ud604\uc7ac \uac8c\uc784\ubc84\uc804\uc5d0\uc11c\ub294 \uada4\ub3c4 \uc5f0\uc0b0\uc744 \uc644\uc804\ud55c N\uccb4 \uc2dc\ubbac\ub808\uc774\uc158 \ub300\uc2e0 Patched conic approximation\uc744 \uc0ac\uc6a9\ud558\uace0 \uc788\uae30 \ub54c\ubb38\uc5d0, \ub77c\uadf8\ub791\uc8fc \uc810\uc774\ub098 \ud5e4\uc77c\ub85c \uada4\ub3c4\uc640 \uac19\uc740 \ud604\uc0c1\uc740 \ubb18\uc0ac\ud558\uc9c0 \ubabb\ud55c\ub2e4.", "sentence_answer": "\uc774 \uac8c\uc784\uc740 \uada4\ub3c4\uc5ed\ud559 \uc5d0 \ub300\ud55c \uc0ac\uc2e4\uc801\uc778 \ubb18\uc0ac\ub85c \ud638\ud3c9\uc744 \ubc1b\uc558\ub2e4.", "paragraph_id": "6561662-3-0", "paragraph_question": "question: \uac8c\uc784\uc740 \ubb34\uc5c7\uc5d0 \ub300\ud55c \uc0ac\uc2e4\uc801\uc778 \ubb18\uc0ac\ub85c \ud638\ud3c9\uc744 \ubc1b\uc558\ub294\uac00?, context: \uc774 \uac8c\uc784\uc740 \uada4\ub3c4\uc5ed\ud559\uc5d0 \ub300\ud55c \uc0ac\uc2e4\uc801\uc778 \ubb18\uc0ac\ub85c \ud638\ud3c9\uc744 \ubc1b\uc558\ub2e4. \ub610\ud55c \uace0\uc815\ub41c \uada4\ub3c4\ub97c \ub3c4\ub294 \ucc9c\uccb4\ub4e4\uc744 \uc81c\uc678\ud558\uba74, \ubaa8\ub4e0 \ubb3c\uccb4\ub4e4\uc740 \ub274\ud134 \uc5ed\ud559 \uc2dc\ubbac\ub808\uc774\uc158\uc758 \uc601\ud5a5 \uc544\ub798\uc5d0 \ub193\uc778\ub2e4. \ub85c\ucf13\uc758 \ucd94\ub825\uc774\ub098 \ube44\ud589\uccb4\uc758 \uc591\ub825\uc740 \uae30\uccb4\uc758 \uc5b4\ub290 \uc704\uce58\uc5d0 \uc5d4\uc9c4 \ubc0f \ub0a0\uac1c\uac00 \ubd99\uc5b4\uc788\ub294\uc9c0\uc5d0 \ub9de\ucdb0\uc11c \uc815\ud655\ud788 \uc791\uc6a9\ud55c\ub2e4. \uc870\ub9bd\ud574\ub193\uc740 \ubd80\ud488\uac04\uc758 \uacb0\ud569\ub825 \ub610\ud55c \ud55c\uacc4\uac00 \uc788\uc5b4\uc11c \ub108\ubb34 \uc2ec\ud558\uac70\ub098 \uc5b4\uae0b\ub09c \ubc29\ud5a5\uc758 \ud798\uc774 \uae30\uccb4\uc5d0 \uac00\ud574\uc9c8 \uacbd\uc6b0, \uae30\uccb4\uac00 \uc0b0\uc0b0\uc774 \ubd80\uc11c\uc838\ubc84\ub9ac\uae30\ub3c4 \ud55c\ub2e4. \uadf8\ub9ac\uace0 \uac8c\uc784 \ub0b4 \ucc9c\uccb4\ub4e4 \uc911, \ucee4\ube48\uc744 \ud3ec\ud568\ud574 \uac01\uae30 \ub2e4\ub978 \uc870\uac74\uc758 \ub300\uae30\uac00 \uc874\uc7ac\ud558\ub294 \ucc9c\uccb4\ub4e4\uc5d0\uc11c\ub294 \ub192\uc774\uc640 \ubc00\ub3c4\uc5d0 \ub530\ub77c\uc11c \ub0a0\uac1c\uc758 \uc591\ub825 \ubc0f \ub099\ud558\uc0b0 \uc804\uac1c \ud6a8\uacfc, \ud56d\ub825\uc774 \uc601\ud5a5\uc744 \ubc1b\ub294\ub2e4. \uc2dc\ubbac\ub808\uc774\uc158\uc740 \uc0c1\ub2f9\ud788 \uc815\uad50\ud574\uc11c, \u2018\ud638\ub9cc \uc804\uc774 \uada4\ub3c4\u2019\ub098 \u2018\uc5d0\uc5b4\ub85c \ube0c\ub808\uc774\ud0b9\u2019\ub4f1 \ud604\uc2e4 \uc138\uacc4\uc758 \uc6b0\uc8fc\ube44\ud589\uae30\uc220\ub4e4\uc774 \ucee4\ubcfc\uacc4 \ud0d0\uc0ac\uc5d0\uc11c\ub3c4 \uc0ac\uc6a9 \uac00\ub2a5\ud558\ub2e4. \ub2e4\ub9cc \ud604\uc7ac \uac8c\uc784\ubc84\uc804\uc5d0\uc11c\ub294 \uada4\ub3c4 \uc5f0\uc0b0\uc744 \uc644\uc804\ud55c N\uccb4 \uc2dc\ubbac\ub808\uc774\uc158 \ub300\uc2e0 Patched conic approximation\uc744 \uc0ac\uc6a9\ud558\uace0 \uc788\uae30 \ub54c\ubb38\uc5d0, \ub77c\uadf8\ub791\uc8fc \uc810\uc774\ub098 \ud5e4\uc77c\ub85c \uada4\ub3c4\uc640 \uac19\uc740 \ud604\uc0c1\uc740 \ubb18\uc0ac\ud558\uc9c0 \ubabb\ud55c\ub2e4."} {"question": "\ud55c\ubbf8FTA \uc7ac\ud611\uc0c1\uacfc \uad00\ub828\ud558\uc5ec \uc6cc\uc2f1\ud134\uc5d0\uc11c \uc9c4\ud589\ub41c \uc81c1\ucc28 \ud611\uc0c1\uc774 \uc5f4\ub9b0 \uac83\uc740 \uc5b8\uc81c\uc778\uac00?", "paragraph": "\uc720\uba85\ud76c \uc0b0\uc5c5\ubd80 \ud1b5\uc0c1\uc815\ucc45\uad6d\uc7a5\uacfc \ub9c8\uc774\ud074 \ube44\uba3c \ubb34\uc5ed\ub300\ud45c\ubd80 \ub300\ud45c\ubcf4\uac00 \uc218\uc11d\ub300\ud45c\ub85c \ucc38\uc11d\ud55c 1\ucc28 \ud611\uc0c1\uc774 2018\ub144 1\uc6d4 5\uc77c \uc6cc\uc2f1\ud134\uc5d0\uc11c \uc9c4\ud589\ub418\uc5c8\ub2e4. \ud55c\uad6d\uc740 \ud22c\uc790\uc790 \uad6d\uac00 \ubd84\uc7c1 \ud574\uacb0(ISDS)\uacfc \ubb34\uc5ed \uad6c\uc81c \ub4f1\uc744 \uad00\uc2ec\ubd84\uc57c\ub85c \uc81c\uc2dc\ud588\uc73c\uba70 \ubbf8\uad6d\uc758 \uad00\uc2ec\ubd84\uc57c\ub294 \ubc1d\ud788\uc9c0 \uc54a\uc558\uc9c0\ub9cc \uc790\ub3d9\ucc28 \ubd84\uc57c\uc758 \ube44\uad00\uc138 \uc7a5\ubcbd \ud574\uc18c\uc640 \uc790\ub3d9\ucc28\u00b7\ucca0\uac15 \ubd80\ubb38\uc758 \uc6d0\uc0b0\uc9c0 \uae30\uc900 \uac15\ud654 \ub4f1 \uc790\ub3d9\ucc28 \ubd84\uc57c\ub97c \uc9d1\uc911 \uac70\ub860\ud588\uc744 \uac83\uc73c\ub85c \ubcf4\uace0 \uc788\ub2e4. 1\uc6d4 31\uc77c\uc5d0\ub294 \uc11c\uc6b8\uc5d0\uc11c 2\ucc28 \ud611\uc0c1\uc5d0 \ub4e4\uc5b4\uac14\ub294\ub370 \uc8fc\uc694 \ub0b4\uc6a9\uc744 \uc678\ubd80\ub85c \uc54c\ub9ac\uc9c0 \uc54a\uae30\ub85c \uc57d\uc18d\ud558\uc5ec \uad6c\uccb4\uc801\uc778 \uc774\uc57c\uae30\ub294 \uacf5\uac1c\ub418\uc9c0 \uc54a\uc558\uc9c0\ub9cc \ud55c\uad6d\uc740 \uc0c8\ub85c\uc6b4 \ud604\uc548\uc73c\ub85c \ub5a0\uc624\ub978 \uc138\ud0c1\uae30\u00b7\ud0dc\uc591\uad11 \uc138\uc774\ud504\uac00\ub4dc \ub4f1 \ubb34\uc5ed\uad6c\uc81c \ub0a8\uc6a9 \ubb38\uc81c\ub97c, \ubbf8\uad6d\uc740 \ub300\ud55c \ubb34\uc5ed\uc801\uc790\uc758 \uac00\uc7a5 \ud070 \ubd80\ubd84\uc744 \ucc28\uc9c0\ud558\ub294 \uc790\ub3d9\ucc28\uc640 \uc790\ub3d9\ucc28 \ubd80\ud488 \uad50\uc5ed \ubb38\uc81c\ub97c \uc9d1\uc911\uc801\uc73c\ub85c \uac70\ub860\ud55c \uac83\uc73c\ub85c \uc804\ud574\uc84c\ub2e4. \ub450 \ucc28\ub840\uc5d0 \uac78\uce5c \ud611\uc0c1\uc744 \ud1b5\ud574\uc11c\ub3c4 \ud569\uc758\uc810\uc744 \ucc3e\uc9c0 \ubabb\ud574 3\ucc28 \ud611\uc0c1\uc774 \ub2e4\uc2dc \uc9c4\ud589\ub420 \uc608\uc815\uc774\uc9c0\ub9cc \ubbf8\uad6d \uc6b0\uc120\uc8fc\uc758\ub97c \ub0b4\uc138\uc6b0\ub294 \ud2b8\ub7fc\ud504 \ud589\uc815\ubd80\uac00 \"\ub098\uc05c \ubb34\uc5ed\ud611\uc815\uc744 \uace0\uce58\uace0 \uc0c8\ub85c\uc6b4 \ud611\uc815\ub4e4\uc744 \ud611\uc0c1\ud558\uae30 \uc704\ud574 \ub178\ub825\ud560 \uac83\"\uc774\ub77c\uace0 30\uc77c \uad6d\uc815\uc5f0\uc124\uc5d0\uc11c \ubc1d\ud600 \ud55c\ubbf8 FTA \ud3d0\uae30\uc640 \uac19\uc740 \ucd08\uac15\uc218\uac00 \ub098\uc62c \uac00\ub2a5\uc131\ub3c4 \ubc30\uc81c\ud560 \uc218 \uc5c6\ub294 \uc0c1\ud669\uc774\ub2e4.", "answer": "2018\ub144 1\uc6d4 5\uc77c", "sentence": "\uc720\uba85\ud76c \uc0b0\uc5c5\ubd80 \ud1b5\uc0c1\uc815\ucc45\uad6d\uc7a5\uacfc \ub9c8\uc774\ud074 \ube44\uba3c \ubb34\uc5ed\ub300\ud45c\ubd80 \ub300\ud45c\ubcf4\uac00 \uc218\uc11d\ub300\ud45c\ub85c \ucc38\uc11d\ud55c 1\ucc28 \ud611\uc0c1\uc774 2018\ub144 1\uc6d4 5\uc77c \uc6cc\uc2f1\ud134\uc5d0\uc11c \uc9c4\ud589\ub418\uc5c8\ub2e4.", "paragraph_sentence": " \uc720\uba85\ud76c \uc0b0\uc5c5\ubd80 \ud1b5\uc0c1\uc815\ucc45\uad6d\uc7a5\uacfc \ub9c8\uc774\ud074 \ube44\uba3c \ubb34\uc5ed\ub300\ud45c\ubd80 \ub300\ud45c\ubcf4\uac00 \uc218\uc11d\ub300\ud45c\ub85c \ucc38\uc11d\ud55c 1\ucc28 \ud611\uc0c1\uc774 2018\ub144 1\uc6d4 5\uc77c \uc6cc\uc2f1\ud134\uc5d0\uc11c \uc9c4\ud589\ub418\uc5c8\ub2e4. \ud55c\uad6d\uc740 \ud22c\uc790\uc790 \uad6d\uac00 \ubd84\uc7c1 \ud574\uacb0(ISDS)\uacfc \ubb34\uc5ed \uad6c\uc81c \ub4f1\uc744 \uad00\uc2ec\ubd84\uc57c\ub85c \uc81c\uc2dc\ud588\uc73c\uba70 \ubbf8\uad6d\uc758 \uad00\uc2ec\ubd84\uc57c\ub294 \ubc1d\ud788\uc9c0 \uc54a\uc558\uc9c0\ub9cc \uc790\ub3d9\ucc28 \ubd84\uc57c\uc758 \ube44\uad00\uc138 \uc7a5\ubcbd \ud574\uc18c\uc640 \uc790\ub3d9\ucc28\u00b7\ucca0\uac15 \ubd80\ubb38\uc758 \uc6d0\uc0b0\uc9c0 \uae30\uc900 \uac15\ud654 \ub4f1 \uc790\ub3d9\ucc28 \ubd84\uc57c\ub97c \uc9d1\uc911 \uac70\ub860\ud588\uc744 \uac83\uc73c\ub85c \ubcf4\uace0 \uc788\ub2e4. 1\uc6d4 31\uc77c\uc5d0\ub294 \uc11c\uc6b8\uc5d0\uc11c 2\ucc28 \ud611\uc0c1\uc5d0 \ub4e4\uc5b4\uac14\ub294\ub370 \uc8fc\uc694 \ub0b4\uc6a9\uc744 \uc678\ubd80\ub85c \uc54c\ub9ac\uc9c0 \uc54a\uae30\ub85c \uc57d\uc18d\ud558\uc5ec \uad6c\uccb4\uc801\uc778 \uc774\uc57c\uae30\ub294 \uacf5\uac1c\ub418\uc9c0 \uc54a\uc558\uc9c0\ub9cc \ud55c\uad6d\uc740 \uc0c8\ub85c\uc6b4 \ud604\uc548\uc73c\ub85c \ub5a0\uc624\ub978 \uc138\ud0c1\uae30\u00b7\ud0dc\uc591\uad11 \uc138\uc774\ud504\uac00\ub4dc \ub4f1 \ubb34\uc5ed\uad6c\uc81c \ub0a8\uc6a9 \ubb38\uc81c\ub97c, \ubbf8\uad6d\uc740 \ub300\ud55c \ubb34\uc5ed\uc801\uc790\uc758 \uac00\uc7a5 \ud070 \ubd80\ubd84\uc744 \ucc28\uc9c0\ud558\ub294 \uc790\ub3d9\ucc28\uc640 \uc790\ub3d9\ucc28 \ubd80\ud488 \uad50\uc5ed \ubb38\uc81c\ub97c \uc9d1\uc911\uc801\uc73c\ub85c \uac70\ub860\ud55c \uac83\uc73c\ub85c \uc804\ud574\uc84c\ub2e4. \ub450 \ucc28\ub840\uc5d0 \uac78\uce5c \ud611\uc0c1\uc744 \ud1b5\ud574\uc11c\ub3c4 \ud569\uc758\uc810\uc744 \ucc3e\uc9c0 \ubabb\ud574 3\ucc28 \ud611\uc0c1\uc774 \ub2e4\uc2dc \uc9c4\ud589\ub420 \uc608\uc815\uc774\uc9c0\ub9cc \ubbf8\uad6d \uc6b0\uc120\uc8fc\uc758\ub97c \ub0b4\uc138\uc6b0\ub294 \ud2b8\ub7fc\ud504 \ud589\uc815\ubd80\uac00 \"\ub098\uc05c \ubb34\uc5ed\ud611\uc815\uc744 \uace0\uce58\uace0 \uc0c8\ub85c\uc6b4 \ud611\uc815\ub4e4\uc744 \ud611\uc0c1\ud558\uae30 \uc704\ud574 \ub178\ub825\ud560 \uac83\"\uc774\ub77c\uace0 30\uc77c \uad6d\uc815\uc5f0\uc124\uc5d0\uc11c \ubc1d\ud600 \ud55c\ubbf8 FTA \ud3d0\uae30\uc640 \uac19\uc740 \ucd08\uac15\uc218\uac00 \ub098\uc62c \uac00\ub2a5\uc131\ub3c4 \ubc30\uc81c\ud560 \uc218 \uc5c6\ub294 \uc0c1\ud669\uc774\ub2e4.", "paragraph_answer": "\uc720\uba85\ud76c \uc0b0\uc5c5\ubd80 \ud1b5\uc0c1\uc815\ucc45\uad6d\uc7a5\uacfc \ub9c8\uc774\ud074 \ube44\uba3c \ubb34\uc5ed\ub300\ud45c\ubd80 \ub300\ud45c\ubcf4\uac00 \uc218\uc11d\ub300\ud45c\ub85c \ucc38\uc11d\ud55c 1\ucc28 \ud611\uc0c1\uc774 2018\ub144 1\uc6d4 5\uc77c \uc6cc\uc2f1\ud134\uc5d0\uc11c \uc9c4\ud589\ub418\uc5c8\ub2e4. \ud55c\uad6d\uc740 \ud22c\uc790\uc790 \uad6d\uac00 \ubd84\uc7c1 \ud574\uacb0(ISDS)\uacfc \ubb34\uc5ed \uad6c\uc81c \ub4f1\uc744 \uad00\uc2ec\ubd84\uc57c\ub85c \uc81c\uc2dc\ud588\uc73c\uba70 \ubbf8\uad6d\uc758 \uad00\uc2ec\ubd84\uc57c\ub294 \ubc1d\ud788\uc9c0 \uc54a\uc558\uc9c0\ub9cc \uc790\ub3d9\ucc28 \ubd84\uc57c\uc758 \ube44\uad00\uc138 \uc7a5\ubcbd \ud574\uc18c\uc640 \uc790\ub3d9\ucc28\u00b7\ucca0\uac15 \ubd80\ubb38\uc758 \uc6d0\uc0b0\uc9c0 \uae30\uc900 \uac15\ud654 \ub4f1 \uc790\ub3d9\ucc28 \ubd84\uc57c\ub97c \uc9d1\uc911 \uac70\ub860\ud588\uc744 \uac83\uc73c\ub85c \ubcf4\uace0 \uc788\ub2e4. 1\uc6d4 31\uc77c\uc5d0\ub294 \uc11c\uc6b8\uc5d0\uc11c 2\ucc28 \ud611\uc0c1\uc5d0 \ub4e4\uc5b4\uac14\ub294\ub370 \uc8fc\uc694 \ub0b4\uc6a9\uc744 \uc678\ubd80\ub85c \uc54c\ub9ac\uc9c0 \uc54a\uae30\ub85c \uc57d\uc18d\ud558\uc5ec \uad6c\uccb4\uc801\uc778 \uc774\uc57c\uae30\ub294 \uacf5\uac1c\ub418\uc9c0 \uc54a\uc558\uc9c0\ub9cc \ud55c\uad6d\uc740 \uc0c8\ub85c\uc6b4 \ud604\uc548\uc73c\ub85c \ub5a0\uc624\ub978 \uc138\ud0c1\uae30\u00b7\ud0dc\uc591\uad11 \uc138\uc774\ud504\uac00\ub4dc \ub4f1 \ubb34\uc5ed\uad6c\uc81c \ub0a8\uc6a9 \ubb38\uc81c\ub97c, \ubbf8\uad6d\uc740 \ub300\ud55c \ubb34\uc5ed\uc801\uc790\uc758 \uac00\uc7a5 \ud070 \ubd80\ubd84\uc744 \ucc28\uc9c0\ud558\ub294 \uc790\ub3d9\ucc28\uc640 \uc790\ub3d9\ucc28 \ubd80\ud488 \uad50\uc5ed \ubb38\uc81c\ub97c \uc9d1\uc911\uc801\uc73c\ub85c \uac70\ub860\ud55c \uac83\uc73c\ub85c \uc804\ud574\uc84c\ub2e4. \ub450 \ucc28\ub840\uc5d0 \uac78\uce5c \ud611\uc0c1\uc744 \ud1b5\ud574\uc11c\ub3c4 \ud569\uc758\uc810\uc744 \ucc3e\uc9c0 \ubabb\ud574 3\ucc28 \ud611\uc0c1\uc774 \ub2e4\uc2dc \uc9c4\ud589\ub420 \uc608\uc815\uc774\uc9c0\ub9cc \ubbf8\uad6d \uc6b0\uc120\uc8fc\uc758\ub97c \ub0b4\uc138\uc6b0\ub294 \ud2b8\ub7fc\ud504 \ud589\uc815\ubd80\uac00 \"\ub098\uc05c \ubb34\uc5ed\ud611\uc815\uc744 \uace0\uce58\uace0 \uc0c8\ub85c\uc6b4 \ud611\uc815\ub4e4\uc744 \ud611\uc0c1\ud558\uae30 \uc704\ud574 \ub178\ub825\ud560 \uac83\"\uc774\ub77c\uace0 30\uc77c \uad6d\uc815\uc5f0\uc124\uc5d0\uc11c \ubc1d\ud600 \ud55c\ubbf8 FTA \ud3d0\uae30\uc640 \uac19\uc740 \ucd08\uac15\uc218\uac00 \ub098\uc62c \uac00\ub2a5\uc131\ub3c4 \ubc30\uc81c\ud560 \uc218 \uc5c6\ub294 \uc0c1\ud669\uc774\ub2e4.", "sentence_answer": "\uc720\uba85\ud76c \uc0b0\uc5c5\ubd80 \ud1b5\uc0c1\uc815\ucc45\uad6d\uc7a5\uacfc \ub9c8\uc774\ud074 \ube44\uba3c \ubb34\uc5ed\ub300\ud45c\ubd80 \ub300\ud45c\ubcf4\uac00 \uc218\uc11d\ub300\ud45c\ub85c \ucc38\uc11d\ud55c 1\ucc28 \ud611\uc0c1\uc774 2018\ub144 1\uc6d4 5\uc77c \uc6cc\uc2f1\ud134\uc5d0\uc11c \uc9c4\ud589\ub418\uc5c8\ub2e4.", "paragraph_id": "6580768-85-0", "paragraph_question": "question: \ud55c\ubbf8FTA \uc7ac\ud611\uc0c1\uacfc \uad00\ub828\ud558\uc5ec \uc6cc\uc2f1\ud134\uc5d0\uc11c \uc9c4\ud589\ub41c \uc81c1\ucc28 \ud611\uc0c1\uc774 \uc5f4\ub9b0 \uac83\uc740 \uc5b8\uc81c\uc778\uac00?, context: \uc720\uba85\ud76c \uc0b0\uc5c5\ubd80 \ud1b5\uc0c1\uc815\ucc45\uad6d\uc7a5\uacfc \ub9c8\uc774\ud074 \ube44\uba3c \ubb34\uc5ed\ub300\ud45c\ubd80 \ub300\ud45c\ubcf4\uac00 \uc218\uc11d\ub300\ud45c\ub85c \ucc38\uc11d\ud55c 1\ucc28 \ud611\uc0c1\uc774 2018\ub144 1\uc6d4 5\uc77c \uc6cc\uc2f1\ud134\uc5d0\uc11c \uc9c4\ud589\ub418\uc5c8\ub2e4. \ud55c\uad6d\uc740 \ud22c\uc790\uc790 \uad6d\uac00 \ubd84\uc7c1 \ud574\uacb0(ISDS)\uacfc \ubb34\uc5ed \uad6c\uc81c \ub4f1\uc744 \uad00\uc2ec\ubd84\uc57c\ub85c \uc81c\uc2dc\ud588\uc73c\uba70 \ubbf8\uad6d\uc758 \uad00\uc2ec\ubd84\uc57c\ub294 \ubc1d\ud788\uc9c0 \uc54a\uc558\uc9c0\ub9cc \uc790\ub3d9\ucc28 \ubd84\uc57c\uc758 \ube44\uad00\uc138 \uc7a5\ubcbd \ud574\uc18c\uc640 \uc790\ub3d9\ucc28\u00b7\ucca0\uac15 \ubd80\ubb38\uc758 \uc6d0\uc0b0\uc9c0 \uae30\uc900 \uac15\ud654 \ub4f1 \uc790\ub3d9\ucc28 \ubd84\uc57c\ub97c \uc9d1\uc911 \uac70\ub860\ud588\uc744 \uac83\uc73c\ub85c \ubcf4\uace0 \uc788\ub2e4. 1\uc6d4 31\uc77c\uc5d0\ub294 \uc11c\uc6b8\uc5d0\uc11c 2\ucc28 \ud611\uc0c1\uc5d0 \ub4e4\uc5b4\uac14\ub294\ub370 \uc8fc\uc694 \ub0b4\uc6a9\uc744 \uc678\ubd80\ub85c \uc54c\ub9ac\uc9c0 \uc54a\uae30\ub85c \uc57d\uc18d\ud558\uc5ec \uad6c\uccb4\uc801\uc778 \uc774\uc57c\uae30\ub294 \uacf5\uac1c\ub418\uc9c0 \uc54a\uc558\uc9c0\ub9cc \ud55c\uad6d\uc740 \uc0c8\ub85c\uc6b4 \ud604\uc548\uc73c\ub85c \ub5a0\uc624\ub978 \uc138\ud0c1\uae30\u00b7\ud0dc\uc591\uad11 \uc138\uc774\ud504\uac00\ub4dc \ub4f1 \ubb34\uc5ed\uad6c\uc81c \ub0a8\uc6a9 \ubb38\uc81c\ub97c, \ubbf8\uad6d\uc740 \ub300\ud55c \ubb34\uc5ed\uc801\uc790\uc758 \uac00\uc7a5 \ud070 \ubd80\ubd84\uc744 \ucc28\uc9c0\ud558\ub294 \uc790\ub3d9\ucc28\uc640 \uc790\ub3d9\ucc28 \ubd80\ud488 \uad50\uc5ed \ubb38\uc81c\ub97c \uc9d1\uc911\uc801\uc73c\ub85c \uac70\ub860\ud55c \uac83\uc73c\ub85c \uc804\ud574\uc84c\ub2e4. \ub450 \ucc28\ub840\uc5d0 \uac78\uce5c \ud611\uc0c1\uc744 \ud1b5\ud574\uc11c\ub3c4 \ud569\uc758\uc810\uc744 \ucc3e\uc9c0 \ubabb\ud574 3\ucc28 \ud611\uc0c1\uc774 \ub2e4\uc2dc \uc9c4\ud589\ub420 \uc608\uc815\uc774\uc9c0\ub9cc \ubbf8\uad6d \uc6b0\uc120\uc8fc\uc758\ub97c \ub0b4\uc138\uc6b0\ub294 \ud2b8\ub7fc\ud504 \ud589\uc815\ubd80\uac00 \"\ub098\uc05c \ubb34\uc5ed\ud611\uc815\uc744 \uace0\uce58\uace0 \uc0c8\ub85c\uc6b4 \ud611\uc815\ub4e4\uc744 \ud611\uc0c1\ud558\uae30 \uc704\ud574 \ub178\ub825\ud560 \uac83\"\uc774\ub77c\uace0 30\uc77c \uad6d\uc815\uc5f0\uc124\uc5d0\uc11c \ubc1d\ud600 \ud55c\ubbf8 FTA \ud3d0\uae30\uc640 \uac19\uc740 \ucd08\uac15\uc218\uac00 \ub098\uc62c \uac00\ub2a5\uc131\ub3c4 \ubc30\uc81c\ud560 \uc218 \uc5c6\ub294 \uc0c1\ud669\uc774\ub2e4."} {"question": "\ubc38\ub7ec\ub9ac \uc194\ub77c\ub098\uc2a4\uc640 \uc6cc\ud640\uc758 \ub9cc\ub0a8\uc744 \uadf8\ub9ac\uc2a4 \ube44\uadf9\ud48d\uc73c\ub85c \ud574\uc11d\ud55c \uc791\ud488\uc740?", "paragraph": "\uc194\ub77c\ub098\uc2a4\uc758 \uc0b6\uc744 \ub2e4\ub8ec \uc5f0\uadf9\uc774 \uc138 \uac1c \uc788\ub2e4. \ud558\ub098\ub294 2001\ub144\uc5d0 \uce74\uc2a8 \ud06c\ub808\uc774\ucc98\uac00 \uc4f4 Valerie Shoots Andy\ub85c, \ubc30\uc6b0 \ub450 \uba85\uc774 \uc80a\uc740 \uc194\ub77c\ub098\uc2a4\uc640 \ub299\uc740 \uc194\ub77c\ub098\uc2a4\ub97c \uac01\uac01 \uc5f0\uae30\ud588\ub2e4. 2003\ub144 \uce74\ub80c \ud638\uc6b0\ud37c\ud2b8\uac00 \uc4f4 Tragedy in Nine Lives\ub294 \uc194\ub77c\ub098\uc2a4\uc640 \uc6cc\ud640\uc758 \ub9cc\ub0a8\uc744 \uadf8\ub9ac\uc2a4 \ube44\uadf9\ud48d\uc73c\ub85c \ud574\uc11d\ud558\uace0 \uc788\uc73c\uba70, \uc904\ub9ac\uc544\ub098 \ud504\ub79c\uc2dc\uc2a4\uac00 \uc194\ub77c\ub098\uc2a4 \uc5ed\uc744 \ub9e1\uc558\ub2e4. 2011\ub144\uc5d0\ub294 \ub9e4\uae30\ucf00\uc774\uc2a4 \ucf5c\uba3c\uacfc \uc548\ub098 K. \uc81c\uc774\ucef5\uc2a4\uac00 \uc4f4 \ubba4\uc9c0\uceec Pop!\uc774 \uc788\ub2e4. \uc774 \uc791\ud488\uc740 \uc194\ub77c\ub098\uc2a4\ubcf4\ub2e4\ub294 \uc6cc\ud640\uc5d0 \uc9d1\uc911\ud55c \uc791\ud488\uc73c\ub85c\uc11c, \ub808\uc774\uccbc \uc7a0\ud3a0\ub9ac\uac00 \uc5f0\uae30\ud55c \uc194\ub77c\ub098\uc2a4\ub294 \"Big Gun\"\uc774\ub77c\ub294 \uc790\uae30 \ub118\ubc84\uac00 \uc788\ub2e4. \u300a\uc6cc\uc2f1\ud134 \ud3ec\uc2a4\ud2b8\u300b\ub294 \uc774 \uace1\uc744 \u201c\uacf5\uc5f0\uc758 \uac00\uc7a5 \uac15\ub825\ud55c \ub118\ubc84(evening's strongest number)\u201d\ub77c\uace0 \ud588\ub2e4.", "answer": "Tragedy in Nine Lives", "sentence": "2003\ub144 \uce74\ub80c \ud638\uc6b0\ud37c\ud2b8\uac00 \uc4f4 Tragedy in Nine Lives \ub294 \uc194\ub77c\ub098\uc2a4\uc640 \uc6cc\ud640\uc758 \ub9cc\ub0a8\uc744 \uadf8\ub9ac\uc2a4 \ube44\uadf9\ud48d\uc73c\ub85c \ud574\uc11d\ud558\uace0 \uc788\uc73c\uba70, \uc904\ub9ac\uc544\ub098 \ud504\ub79c\uc2dc\uc2a4\uac00 \uc194\ub77c\ub098\uc2a4 \uc5ed\uc744 \ub9e1\uc558\ub2e4.", "paragraph_sentence": "\uc194\ub77c\ub098\uc2a4\uc758 \uc0b6\uc744 \ub2e4\ub8ec \uc5f0\uadf9\uc774 \uc138 \uac1c \uc788\ub2e4. \ud558\ub098\ub294 2001\ub144\uc5d0 \uce74\uc2a8 \ud06c\ub808\uc774\ucc98\uac00 \uc4f4 Valerie Shoots Andy\ub85c, \ubc30\uc6b0 \ub450 \uba85\uc774 \uc80a\uc740 \uc194\ub77c\ub098\uc2a4\uc640 \ub299\uc740 \uc194\ub77c\ub098\uc2a4\ub97c \uac01\uac01 \uc5f0\uae30\ud588\ub2e4. 2003\ub144 \uce74\ub80c \ud638\uc6b0\ud37c\ud2b8\uac00 \uc4f4 Tragedy in Nine Lives \ub294 \uc194\ub77c\ub098\uc2a4\uc640 \uc6cc\ud640\uc758 \ub9cc\ub0a8\uc744 \uadf8\ub9ac\uc2a4 \ube44\uadf9\ud48d\uc73c\ub85c \ud574\uc11d\ud558\uace0 \uc788\uc73c\uba70, \uc904\ub9ac\uc544\ub098 \ud504\ub79c\uc2dc\uc2a4\uac00 \uc194\ub77c\ub098\uc2a4 \uc5ed\uc744 \ub9e1\uc558\ub2e4. 2011\ub144\uc5d0\ub294 \ub9e4\uae30\ucf00\uc774\uc2a4 \ucf5c\uba3c\uacfc \uc548\ub098 K. \uc81c\uc774\ucef5\uc2a4\uac00 \uc4f4 \ubba4\uc9c0\uceec Pop!\uc774 \uc788\ub2e4. \uc774 \uc791\ud488\uc740 \uc194\ub77c\ub098\uc2a4\ubcf4\ub2e4\ub294 \uc6cc\ud640\uc5d0 \uc9d1\uc911\ud55c \uc791\ud488\uc73c\ub85c\uc11c, \ub808\uc774\uccbc \uc7a0\ud3a0\ub9ac\uac00 \uc5f0\uae30\ud55c \uc194\ub77c\ub098\uc2a4\ub294 \"Big Gun\"\uc774\ub77c\ub294 \uc790\uae30 \ub118\ubc84\uac00 \uc788\ub2e4. \u300a\uc6cc\uc2f1\ud134 \ud3ec\uc2a4\ud2b8\u300b\ub294 \uc774 \uace1\uc744 \u201c\uacf5\uc5f0\uc758 \uac00\uc7a5 \uac15\ub825\ud55c \ub118\ubc84(evening's strongest number)\u201d\ub77c\uace0 \ud588\ub2e4.", "paragraph_answer": "\uc194\ub77c\ub098\uc2a4\uc758 \uc0b6\uc744 \ub2e4\ub8ec \uc5f0\uadf9\uc774 \uc138 \uac1c \uc788\ub2e4. \ud558\ub098\ub294 2001\ub144\uc5d0 \uce74\uc2a8 \ud06c\ub808\uc774\ucc98\uac00 \uc4f4 Valerie Shoots Andy\ub85c, \ubc30\uc6b0 \ub450 \uba85\uc774 \uc80a\uc740 \uc194\ub77c\ub098\uc2a4\uc640 \ub299\uc740 \uc194\ub77c\ub098\uc2a4\ub97c \uac01\uac01 \uc5f0\uae30\ud588\ub2e4. 2003\ub144 \uce74\ub80c \ud638\uc6b0\ud37c\ud2b8\uac00 \uc4f4 Tragedy in Nine Lives \ub294 \uc194\ub77c\ub098\uc2a4\uc640 \uc6cc\ud640\uc758 \ub9cc\ub0a8\uc744 \uadf8\ub9ac\uc2a4 \ube44\uadf9\ud48d\uc73c\ub85c \ud574\uc11d\ud558\uace0 \uc788\uc73c\uba70, \uc904\ub9ac\uc544\ub098 \ud504\ub79c\uc2dc\uc2a4\uac00 \uc194\ub77c\ub098\uc2a4 \uc5ed\uc744 \ub9e1\uc558\ub2e4. 2011\ub144\uc5d0\ub294 \ub9e4\uae30\ucf00\uc774\uc2a4 \ucf5c\uba3c\uacfc \uc548\ub098 K. \uc81c\uc774\ucef5\uc2a4\uac00 \uc4f4 \ubba4\uc9c0\uceec Pop!\uc774 \uc788\ub2e4. \uc774 \uc791\ud488\uc740 \uc194\ub77c\ub098\uc2a4\ubcf4\ub2e4\ub294 \uc6cc\ud640\uc5d0 \uc9d1\uc911\ud55c \uc791\ud488\uc73c\ub85c\uc11c, \ub808\uc774\uccbc \uc7a0\ud3a0\ub9ac\uac00 \uc5f0\uae30\ud55c \uc194\ub77c\ub098\uc2a4\ub294 \"Big Gun\"\uc774\ub77c\ub294 \uc790\uae30 \ub118\ubc84\uac00 \uc788\ub2e4. \u300a\uc6cc\uc2f1\ud134 \ud3ec\uc2a4\ud2b8\u300b\ub294 \uc774 \uace1\uc744 \u201c\uacf5\uc5f0\uc758 \uac00\uc7a5 \uac15\ub825\ud55c \ub118\ubc84(evening's strongest number)\u201d\ub77c\uace0 \ud588\ub2e4.", "sentence_answer": "2003\ub144 \uce74\ub80c \ud638\uc6b0\ud37c\ud2b8\uac00 \uc4f4 Tragedy in Nine Lives \ub294 \uc194\ub77c\ub098\uc2a4\uc640 \uc6cc\ud640\uc758 \ub9cc\ub0a8\uc744 \uadf8\ub9ac\uc2a4 \ube44\uadf9\ud48d\uc73c\ub85c \ud574\uc11d\ud558\uace0 \uc788\uc73c\uba70, \uc904\ub9ac\uc544\ub098 \ud504\ub79c\uc2dc\uc2a4\uac00 \uc194\ub77c\ub098\uc2a4 \uc5ed\uc744 \ub9e1\uc558\ub2e4.", "paragraph_id": "6535070-12-0", "paragraph_question": "question: \ubc38\ub7ec\ub9ac \uc194\ub77c\ub098\uc2a4\uc640 \uc6cc\ud640\uc758 \ub9cc\ub0a8\uc744 \uadf8\ub9ac\uc2a4 \ube44\uadf9\ud48d\uc73c\ub85c \ud574\uc11d\ud55c \uc791\ud488\uc740?, context: \uc194\ub77c\ub098\uc2a4\uc758 \uc0b6\uc744 \ub2e4\ub8ec \uc5f0\uadf9\uc774 \uc138 \uac1c \uc788\ub2e4. \ud558\ub098\ub294 2001\ub144\uc5d0 \uce74\uc2a8 \ud06c\ub808\uc774\ucc98\uac00 \uc4f4 Valerie Shoots Andy\ub85c, \ubc30\uc6b0 \ub450 \uba85\uc774 \uc80a\uc740 \uc194\ub77c\ub098\uc2a4\uc640 \ub299\uc740 \uc194\ub77c\ub098\uc2a4\ub97c \uac01\uac01 \uc5f0\uae30\ud588\ub2e4. 2003\ub144 \uce74\ub80c \ud638\uc6b0\ud37c\ud2b8\uac00 \uc4f4 Tragedy in Nine Lives\ub294 \uc194\ub77c\ub098\uc2a4\uc640 \uc6cc\ud640\uc758 \ub9cc\ub0a8\uc744 \uadf8\ub9ac\uc2a4 \ube44\uadf9\ud48d\uc73c\ub85c \ud574\uc11d\ud558\uace0 \uc788\uc73c\uba70, \uc904\ub9ac\uc544\ub098 \ud504\ub79c\uc2dc\uc2a4\uac00 \uc194\ub77c\ub098\uc2a4 \uc5ed\uc744 \ub9e1\uc558\ub2e4. 2011\ub144\uc5d0\ub294 \ub9e4\uae30\ucf00\uc774\uc2a4 \ucf5c\uba3c\uacfc \uc548\ub098 K. \uc81c\uc774\ucef5\uc2a4\uac00 \uc4f4 \ubba4\uc9c0\uceec Pop!\uc774 \uc788\ub2e4. \uc774 \uc791\ud488\uc740 \uc194\ub77c\ub098\uc2a4\ubcf4\ub2e4\ub294 \uc6cc\ud640\uc5d0 \uc9d1\uc911\ud55c \uc791\ud488\uc73c\ub85c\uc11c, \ub808\uc774\uccbc \uc7a0\ud3a0\ub9ac\uac00 \uc5f0\uae30\ud55c \uc194\ub77c\ub098\uc2a4\ub294 \"Big Gun\"\uc774\ub77c\ub294 \uc790\uae30 \ub118\ubc84\uac00 \uc788\ub2e4. \u300a\uc6cc\uc2f1\ud134 \ud3ec\uc2a4\ud2b8\u300b\ub294 \uc774 \uace1\uc744 \u201c\uacf5\uc5f0\uc758 \uac00\uc7a5 \uac15\ub825\ud55c \ub118\ubc84(evening's strongest number)\u201d\ub77c\uace0 \ud588\ub2e4."} {"question": "\ub0a8\uc545\uc2e0\ub3c4\uc2dc\ub294 \ubb34\uc548\uacfc \uc5b4\ub290 \uc9c0\uc5ed\uc758 \uacbd\uacc4\uc5d0 \uac78\uccd0\uc11c \ud615\uc131\ub418\uc5b4 \uc788\ub294\uac00?", "paragraph": "\ub0a8\uc545\uc2e0\ub3c4\uc2dc\ub294 \ubb34\uc548\uacfc \ubaa9\ud3ec\uc640\uc758 \uacbd\uacc4\uc5d0 \uac78\uccd0 \ud615\uc131\ub418\uace0 \uc788\ub2e4. \uadf8\ub7ec\ub2e4 \ubcf4\ub2c8 \uac19\uc740 \uc0dd\ud65c\uad8c\uc778\ub370 \ud589\uc815\uc11c\ube44\uc2a4\ub294 \ub2e4\ub974\ub2e4. \uc625\uc554\uc9c0\uad6c\ub294 \ubaa9\ud3ec\uc2dc \ud589\uc815\uad6c\uc5ed, \ub0a8\uc545\uc9c0\uad6c\ub294 \ubb34\uc548\uad70 \ud589\uc815\uad6c\uc5ed\uc774\ub2e4. \ud559\uad70, \ud589\uc815\uc11c\ube44\uc2a4, \ub300\uc911\uad50\ud1b5\ub4f1\uc774 \ub300\ud45c\uc801\uc778 \ubb38\uc81c\ub2e4. \uc11c\ub85c \ub2e4\ub978 \ud589\uc815\uad6c\uc5ed\uc73c\ub85c \uc778\ud574 \ub0a8\uc545\uc2e0\ub3c4\uc2dc \uc625\uc554\uc9c0\uad6c \ud559\uc0dd\ub4e4\uc774 \ubaa9\ud3ec\uc2dc \uad6c\ub3c4\uc2ec\uc73c\ub85c \ubc30\uc815\ub2f9\ud558\uae30\ub3c4 \ud55c\ub2e4. \uc774\uc5d0 \ubaa9\ud3ec\uad50\uc721\uc9c0\uc6d0\uccad\uc740 \uc625\uc554\uc9c0\uad6c \ud559\uc0dd\uc744 \ubaa8\ub450 \uadfc\uac70\ub9ac\uc5d0 \ubc30\uc815\ud558\uba74 \uad6c\ub3c4\uc2ec \uc911\ud559\uad50 \ud559\uc0dd \uc218\uac00 \ubd80\uc871\ud574\uc9c4\ub2e4\uba70 \ud559\uc0dd \uc218\uac00 \uc801\uc740 \uc6d0\ub3c4\uc2ec \ud559\uad50\ub97c \uc704\ud574 \uac15\uc81c\ubc30\uc815\uc774 \ubd88\uac00\ud53c\ud558\ub2e4\ub294 \uc785\uc7a5\uc774\ub2e4. \uad50\ud1b5\ub3c4 \ub9c8\ucc2c\uac00\uc9c0\ub2e4. \ud0dd\uc2dc\ub3c4 \ubaa9\ud3ec\uc2dc \uad00\ud560\uc778 \uc625\uc554\uc9c0\uad6c\uc5d0\uc11c \ub098\uc640 \ubb34\uc548\uad70 \uad00\ud560\uc778 \ub0a8\uc545\uc9c0\uad6c\uae4c\uc9c0 \uc774\uc6a9\ud558\uba74 \uc2dc\uc678\ud560\uc99d\uc694\uae08\uc774 20%\ub97c \uac00\uc0b0\ud588\uc5c8\ub2e4. \ud558\uc9c0\ub9cc \uc804\ub77c\ub0a8\ub3c4\uac00 \ubaa9\ud3ec\uc2dc\uc640 \ubb34\uc548\uad70\uc758 \ud611\uc758\ub97c \uac70\uccd0 \ub2e8\uc77c\ud654\ub97c \uc801\uc6a9\ud574 2008\ub144 4\uc6d4\ubd80\ud130 \ub0a8\uc545\uc2e0\ub3c4\uc2dc \uc804\uc5ed\uc5d0\uc11c \ubaa9\ud3ec \uc2dc\ub0b4\uc694\uae08\uc73c\ub85c \uc774\uc6a9\ud558\uac8c \ub418\uc5c8\ub2e4. \ubc84\uc2a4\ub3c4 \ub9c8\ucc2c\uac00\uc9c0\uc600\ub2e4. \ub0a8\uc545\uc8fc\ubbfc\ub4e4\uc740 \uac19\uc740 \ub3d9\ub124\uc5d0 \uc0b4\uba74\uc11c \ubc84\uc2a4 \uc815\uac70\uc7a5 \ud55c \ub450 \uacf3 \ub354 \uac04\ub2e4\uace0 \ud574\uc11c \ub354 \ub9ce\uc740 \uc694\uae08\uc744 \ub0b4\ub294 \uac8c \ub9d0\uc774 \ub418\ub0d0\uba74\uc11c \ud56d\uc758\ud558\uc5ec, \uae30\uc874 \ubaa9\ud3ec \uc2dc\ub0b4\ubc84\uc2a4 \uc694\uae08\uc740 1200\uc6d0\uc774\uc600\uace0 \ubb34\uc548\uad70 \ub0a8\uc545\uc9c0\uad6c\uae4c\uc9c0 \uc774\uc6a9\ud558\uac8c \ub418\uba74 1350\uc6d0\uc744 \ub0b4\uc57c \ud588\uc9c0\ub9cc 2014\ub144 1\uc6d4 1\uc77c\ubd80\ud130 \ub2e8\uc77c\ud654\uac00 \ub418\uc5b4 \uae30\uc874 \uc694\uae08\uc73c\ub85c\ub3c4 \ub0a8\uc545\uc9c0\uc5ed\uc744 \uc774\uc6a9\ud560 \uc218 \uc788\uac8c \ub418\uc5c8\ub2e4. \uadf8\ub807\uc9c0\ub9cc \ub2e4\ub978 \ubb38\uc81c\ub3c4 \ub9ce\ub2e4. \uc4f0\ub808\uae30 \ubb38\uc81c\uc640 \ud3ed\uc124\uac19\uc740 \uc7ac\ud574 \ub300\ube44 \ub2a5\ub825\ub4f1\ub3c4 \ubb34\uc548\uad70 \ud589\uc815\ub825\uc774 \ubaa9\ud3ec\uc2dc \ud589\uc815\ub825\uc640 \uac19\uc740 \ub3c4\uc2dc\ud589\uc815\uc744 \ub530\ub77c\uac00\uc9c0\ub97c \ubabb\ud558\ub294 \uac83\uc774 \ud604\uc2e4\uc774\ub2e4.", "answer": "\ubaa9\ud3ec", "sentence": "\ub0a8\uc545\uc2e0\ub3c4\uc2dc\ub294 \ubb34\uc548\uacfc \ubaa9\ud3ec \uc640\uc758 \uacbd\uacc4\uc5d0 \uac78\uccd0 \ud615\uc131\ub418\uace0 \uc788\ub2e4.", "paragraph_sentence": " \ub0a8\uc545\uc2e0\ub3c4\uc2dc\ub294 \ubb34\uc548\uacfc \ubaa9\ud3ec \uc640\uc758 \uacbd\uacc4\uc5d0 \uac78\uccd0 \ud615\uc131\ub418\uace0 \uc788\ub2e4. \uadf8\ub7ec\ub2e4 \ubcf4\ub2c8 \uac19\uc740 \uc0dd\ud65c\uad8c\uc778\ub370 \ud589\uc815\uc11c\ube44\uc2a4\ub294 \ub2e4\ub974\ub2e4. \uc625\uc554\uc9c0\uad6c\ub294 \ubaa9\ud3ec\uc2dc \ud589\uc815\uad6c\uc5ed, \ub0a8\uc545\uc9c0\uad6c\ub294 \ubb34\uc548\uad70 \ud589\uc815\uad6c\uc5ed\uc774\ub2e4. \ud559\uad70, \ud589\uc815\uc11c\ube44\uc2a4, \ub300\uc911\uad50\ud1b5\ub4f1\uc774 \ub300\ud45c\uc801\uc778 \ubb38\uc81c\ub2e4. \uc11c\ub85c \ub2e4\ub978 \ud589\uc815\uad6c\uc5ed\uc73c\ub85c \uc778\ud574 \ub0a8\uc545\uc2e0\ub3c4\uc2dc \uc625\uc554\uc9c0\uad6c \ud559\uc0dd\ub4e4\uc774 \ubaa9\ud3ec\uc2dc \uad6c\ub3c4\uc2ec\uc73c\ub85c \ubc30\uc815\ub2f9\ud558\uae30\ub3c4 \ud55c\ub2e4. \uc774\uc5d0 \ubaa9\ud3ec\uad50\uc721\uc9c0\uc6d0\uccad\uc740 \uc625\uc554\uc9c0\uad6c \ud559\uc0dd\uc744 \ubaa8\ub450 \uadfc\uac70\ub9ac\uc5d0 \ubc30\uc815\ud558\uba74 \uad6c\ub3c4\uc2ec \uc911\ud559\uad50 \ud559\uc0dd \uc218\uac00 \ubd80\uc871\ud574\uc9c4\ub2e4\uba70 \ud559\uc0dd \uc218\uac00 \uc801\uc740 \uc6d0\ub3c4\uc2ec \ud559\uad50\ub97c \uc704\ud574 \uac15\uc81c\ubc30\uc815\uc774 \ubd88\uac00\ud53c\ud558\ub2e4\ub294 \uc785\uc7a5\uc774\ub2e4. \uad50\ud1b5\ub3c4 \ub9c8\ucc2c\uac00\uc9c0\ub2e4. \ud0dd\uc2dc\ub3c4 \ubaa9\ud3ec\uc2dc \uad00\ud560\uc778 \uc625\uc554\uc9c0\uad6c\uc5d0\uc11c \ub098\uc640 \ubb34\uc548\uad70 \uad00\ud560\uc778 \ub0a8\uc545\uc9c0\uad6c\uae4c\uc9c0 \uc774\uc6a9\ud558\uba74 \uc2dc\uc678\ud560\uc99d\uc694\uae08\uc774 20%\ub97c \uac00\uc0b0\ud588\uc5c8\ub2e4. \ud558\uc9c0\ub9cc \uc804\ub77c\ub0a8\ub3c4\uac00 \ubaa9\ud3ec\uc2dc\uc640 \ubb34\uc548\uad70\uc758 \ud611\uc758\ub97c \uac70\uccd0 \ub2e8\uc77c\ud654\ub97c \uc801\uc6a9\ud574 2008\ub144 4\uc6d4\ubd80\ud130 \ub0a8\uc545\uc2e0\ub3c4\uc2dc \uc804\uc5ed\uc5d0\uc11c \ubaa9\ud3ec \uc2dc\ub0b4\uc694\uae08\uc73c\ub85c \uc774\uc6a9\ud558\uac8c \ub418\uc5c8\ub2e4. \ubc84\uc2a4\ub3c4 \ub9c8\ucc2c\uac00\uc9c0\uc600\ub2e4. \ub0a8\uc545\uc8fc\ubbfc\ub4e4\uc740 \uac19\uc740 \ub3d9\ub124\uc5d0 \uc0b4\uba74\uc11c \ubc84\uc2a4 \uc815\uac70\uc7a5 \ud55c \ub450 \uacf3 \ub354 \uac04\ub2e4\uace0 \ud574\uc11c \ub354 \ub9ce\uc740 \uc694\uae08\uc744 \ub0b4\ub294 \uac8c \ub9d0\uc774 \ub418\ub0d0\uba74\uc11c \ud56d\uc758\ud558\uc5ec, \uae30\uc874 \ubaa9\ud3ec \uc2dc\ub0b4\ubc84\uc2a4 \uc694\uae08\uc740 1200\uc6d0\uc774\uc600\uace0 \ubb34\uc548\uad70 \ub0a8\uc545\uc9c0\uad6c\uae4c\uc9c0 \uc774\uc6a9\ud558\uac8c \ub418\uba74 1350\uc6d0\uc744 \ub0b4\uc57c \ud588\uc9c0\ub9cc 2014\ub144 1\uc6d4 1\uc77c\ubd80\ud130 \ub2e8\uc77c\ud654\uac00 \ub418\uc5b4 \uae30\uc874 \uc694\uae08\uc73c\ub85c\ub3c4 \ub0a8\uc545\uc9c0\uc5ed\uc744 \uc774\uc6a9\ud560 \uc218 \uc788\uac8c \ub418\uc5c8\ub2e4. \uadf8\ub807\uc9c0\ub9cc \ub2e4\ub978 \ubb38\uc81c\ub3c4 \ub9ce\ub2e4. \uc4f0\ub808\uae30 \ubb38\uc81c\uc640 \ud3ed\uc124\uac19\uc740 \uc7ac\ud574 \ub300\ube44 \ub2a5\ub825\ub4f1\ub3c4 \ubb34\uc548\uad70 \ud589\uc815\ub825\uc774 \ubaa9\ud3ec\uc2dc \ud589\uc815\ub825\uc640 \uac19\uc740 \ub3c4\uc2dc\ud589\uc815\uc744 \ub530\ub77c\uac00\uc9c0\ub97c \ubabb\ud558\ub294 \uac83\uc774 \ud604\uc2e4\uc774\ub2e4.", "paragraph_answer": "\ub0a8\uc545\uc2e0\ub3c4\uc2dc\ub294 \ubb34\uc548\uacfc \ubaa9\ud3ec \uc640\uc758 \uacbd\uacc4\uc5d0 \uac78\uccd0 \ud615\uc131\ub418\uace0 \uc788\ub2e4. \uadf8\ub7ec\ub2e4 \ubcf4\ub2c8 \uac19\uc740 \uc0dd\ud65c\uad8c\uc778\ub370 \ud589\uc815\uc11c\ube44\uc2a4\ub294 \ub2e4\ub974\ub2e4. \uc625\uc554\uc9c0\uad6c\ub294 \ubaa9\ud3ec\uc2dc \ud589\uc815\uad6c\uc5ed, \ub0a8\uc545\uc9c0\uad6c\ub294 \ubb34\uc548\uad70 \ud589\uc815\uad6c\uc5ed\uc774\ub2e4. \ud559\uad70, \ud589\uc815\uc11c\ube44\uc2a4, \ub300\uc911\uad50\ud1b5\ub4f1\uc774 \ub300\ud45c\uc801\uc778 \ubb38\uc81c\ub2e4. \uc11c\ub85c \ub2e4\ub978 \ud589\uc815\uad6c\uc5ed\uc73c\ub85c \uc778\ud574 \ub0a8\uc545\uc2e0\ub3c4\uc2dc \uc625\uc554\uc9c0\uad6c \ud559\uc0dd\ub4e4\uc774 \ubaa9\ud3ec\uc2dc \uad6c\ub3c4\uc2ec\uc73c\ub85c \ubc30\uc815\ub2f9\ud558\uae30\ub3c4 \ud55c\ub2e4. \uc774\uc5d0 \ubaa9\ud3ec\uad50\uc721\uc9c0\uc6d0\uccad\uc740 \uc625\uc554\uc9c0\uad6c \ud559\uc0dd\uc744 \ubaa8\ub450 \uadfc\uac70\ub9ac\uc5d0 \ubc30\uc815\ud558\uba74 \uad6c\ub3c4\uc2ec \uc911\ud559\uad50 \ud559\uc0dd \uc218\uac00 \ubd80\uc871\ud574\uc9c4\ub2e4\uba70 \ud559\uc0dd \uc218\uac00 \uc801\uc740 \uc6d0\ub3c4\uc2ec \ud559\uad50\ub97c \uc704\ud574 \uac15\uc81c\ubc30\uc815\uc774 \ubd88\uac00\ud53c\ud558\ub2e4\ub294 \uc785\uc7a5\uc774\ub2e4. \uad50\ud1b5\ub3c4 \ub9c8\ucc2c\uac00\uc9c0\ub2e4. \ud0dd\uc2dc\ub3c4 \ubaa9\ud3ec\uc2dc \uad00\ud560\uc778 \uc625\uc554\uc9c0\uad6c\uc5d0\uc11c \ub098\uc640 \ubb34\uc548\uad70 \uad00\ud560\uc778 \ub0a8\uc545\uc9c0\uad6c\uae4c\uc9c0 \uc774\uc6a9\ud558\uba74 \uc2dc\uc678\ud560\uc99d\uc694\uae08\uc774 20%\ub97c \uac00\uc0b0\ud588\uc5c8\ub2e4. \ud558\uc9c0\ub9cc \uc804\ub77c\ub0a8\ub3c4\uac00 \ubaa9\ud3ec\uc2dc\uc640 \ubb34\uc548\uad70\uc758 \ud611\uc758\ub97c \uac70\uccd0 \ub2e8\uc77c\ud654\ub97c \uc801\uc6a9\ud574 2008\ub144 4\uc6d4\ubd80\ud130 \ub0a8\uc545\uc2e0\ub3c4\uc2dc \uc804\uc5ed\uc5d0\uc11c \ubaa9\ud3ec \uc2dc\ub0b4\uc694\uae08\uc73c\ub85c \uc774\uc6a9\ud558\uac8c \ub418\uc5c8\ub2e4. \ubc84\uc2a4\ub3c4 \ub9c8\ucc2c\uac00\uc9c0\uc600\ub2e4. \ub0a8\uc545\uc8fc\ubbfc\ub4e4\uc740 \uac19\uc740 \ub3d9\ub124\uc5d0 \uc0b4\uba74\uc11c \ubc84\uc2a4 \uc815\uac70\uc7a5 \ud55c \ub450 \uacf3 \ub354 \uac04\ub2e4\uace0 \ud574\uc11c \ub354 \ub9ce\uc740 \uc694\uae08\uc744 \ub0b4\ub294 \uac8c \ub9d0\uc774 \ub418\ub0d0\uba74\uc11c \ud56d\uc758\ud558\uc5ec, \uae30\uc874 \ubaa9\ud3ec \uc2dc\ub0b4\ubc84\uc2a4 \uc694\uae08\uc740 1200\uc6d0\uc774\uc600\uace0 \ubb34\uc548\uad70 \ub0a8\uc545\uc9c0\uad6c\uae4c\uc9c0 \uc774\uc6a9\ud558\uac8c \ub418\uba74 1350\uc6d0\uc744 \ub0b4\uc57c \ud588\uc9c0\ub9cc 2014\ub144 1\uc6d4 1\uc77c\ubd80\ud130 \ub2e8\uc77c\ud654\uac00 \ub418\uc5b4 \uae30\uc874 \uc694\uae08\uc73c\ub85c\ub3c4 \ub0a8\uc545\uc9c0\uc5ed\uc744 \uc774\uc6a9\ud560 \uc218 \uc788\uac8c \ub418\uc5c8\ub2e4. \uadf8\ub807\uc9c0\ub9cc \ub2e4\ub978 \ubb38\uc81c\ub3c4 \ub9ce\ub2e4. \uc4f0\ub808\uae30 \ubb38\uc81c\uc640 \ud3ed\uc124\uac19\uc740 \uc7ac\ud574 \ub300\ube44 \ub2a5\ub825\ub4f1\ub3c4 \ubb34\uc548\uad70 \ud589\uc815\ub825\uc774 \ubaa9\ud3ec\uc2dc \ud589\uc815\ub825\uc640 \uac19\uc740 \ub3c4\uc2dc\ud589\uc815\uc744 \ub530\ub77c\uac00\uc9c0\ub97c \ubabb\ud558\ub294 \uac83\uc774 \ud604\uc2e4\uc774\ub2e4.", "sentence_answer": "\ub0a8\uc545\uc2e0\ub3c4\uc2dc\ub294 \ubb34\uc548\uacfc \ubaa9\ud3ec \uc640\uc758 \uacbd\uacc4\uc5d0 \uac78\uccd0 \ud615\uc131\ub418\uace0 \uc788\ub2e4.", "paragraph_id": "6511917-2-0", "paragraph_question": "question: \ub0a8\uc545\uc2e0\ub3c4\uc2dc\ub294 \ubb34\uc548\uacfc \uc5b4\ub290 \uc9c0\uc5ed\uc758 \uacbd\uacc4\uc5d0 \uac78\uccd0\uc11c \ud615\uc131\ub418\uc5b4 \uc788\ub294\uac00?, context: \ub0a8\uc545\uc2e0\ub3c4\uc2dc\ub294 \ubb34\uc548\uacfc \ubaa9\ud3ec\uc640\uc758 \uacbd\uacc4\uc5d0 \uac78\uccd0 \ud615\uc131\ub418\uace0 \uc788\ub2e4. \uadf8\ub7ec\ub2e4 \ubcf4\ub2c8 \uac19\uc740 \uc0dd\ud65c\uad8c\uc778\ub370 \ud589\uc815\uc11c\ube44\uc2a4\ub294 \ub2e4\ub974\ub2e4. \uc625\uc554\uc9c0\uad6c\ub294 \ubaa9\ud3ec\uc2dc \ud589\uc815\uad6c\uc5ed, \ub0a8\uc545\uc9c0\uad6c\ub294 \ubb34\uc548\uad70 \ud589\uc815\uad6c\uc5ed\uc774\ub2e4. \ud559\uad70, \ud589\uc815\uc11c\ube44\uc2a4, \ub300\uc911\uad50\ud1b5\ub4f1\uc774 \ub300\ud45c\uc801\uc778 \ubb38\uc81c\ub2e4. \uc11c\ub85c \ub2e4\ub978 \ud589\uc815\uad6c\uc5ed\uc73c\ub85c \uc778\ud574 \ub0a8\uc545\uc2e0\ub3c4\uc2dc \uc625\uc554\uc9c0\uad6c \ud559\uc0dd\ub4e4\uc774 \ubaa9\ud3ec\uc2dc \uad6c\ub3c4\uc2ec\uc73c\ub85c \ubc30\uc815\ub2f9\ud558\uae30\ub3c4 \ud55c\ub2e4. \uc774\uc5d0 \ubaa9\ud3ec\uad50\uc721\uc9c0\uc6d0\uccad\uc740 \uc625\uc554\uc9c0\uad6c \ud559\uc0dd\uc744 \ubaa8\ub450 \uadfc\uac70\ub9ac\uc5d0 \ubc30\uc815\ud558\uba74 \uad6c\ub3c4\uc2ec \uc911\ud559\uad50 \ud559\uc0dd \uc218\uac00 \ubd80\uc871\ud574\uc9c4\ub2e4\uba70 \ud559\uc0dd \uc218\uac00 \uc801\uc740 \uc6d0\ub3c4\uc2ec \ud559\uad50\ub97c \uc704\ud574 \uac15\uc81c\ubc30\uc815\uc774 \ubd88\uac00\ud53c\ud558\ub2e4\ub294 \uc785\uc7a5\uc774\ub2e4. \uad50\ud1b5\ub3c4 \ub9c8\ucc2c\uac00\uc9c0\ub2e4. \ud0dd\uc2dc\ub3c4 \ubaa9\ud3ec\uc2dc \uad00\ud560\uc778 \uc625\uc554\uc9c0\uad6c\uc5d0\uc11c \ub098\uc640 \ubb34\uc548\uad70 \uad00\ud560\uc778 \ub0a8\uc545\uc9c0\uad6c\uae4c\uc9c0 \uc774\uc6a9\ud558\uba74 \uc2dc\uc678\ud560\uc99d\uc694\uae08\uc774 20%\ub97c \uac00\uc0b0\ud588\uc5c8\ub2e4. \ud558\uc9c0\ub9cc \uc804\ub77c\ub0a8\ub3c4\uac00 \ubaa9\ud3ec\uc2dc\uc640 \ubb34\uc548\uad70\uc758 \ud611\uc758\ub97c \uac70\uccd0 \ub2e8\uc77c\ud654\ub97c \uc801\uc6a9\ud574 2008\ub144 4\uc6d4\ubd80\ud130 \ub0a8\uc545\uc2e0\ub3c4\uc2dc \uc804\uc5ed\uc5d0\uc11c \ubaa9\ud3ec \uc2dc\ub0b4\uc694\uae08\uc73c\ub85c \uc774\uc6a9\ud558\uac8c \ub418\uc5c8\ub2e4. \ubc84\uc2a4\ub3c4 \ub9c8\ucc2c\uac00\uc9c0\uc600\ub2e4. \ub0a8\uc545\uc8fc\ubbfc\ub4e4\uc740 \uac19\uc740 \ub3d9\ub124\uc5d0 \uc0b4\uba74\uc11c \ubc84\uc2a4 \uc815\uac70\uc7a5 \ud55c \ub450 \uacf3 \ub354 \uac04\ub2e4\uace0 \ud574\uc11c \ub354 \ub9ce\uc740 \uc694\uae08\uc744 \ub0b4\ub294 \uac8c \ub9d0\uc774 \ub418\ub0d0\uba74\uc11c \ud56d\uc758\ud558\uc5ec, \uae30\uc874 \ubaa9\ud3ec \uc2dc\ub0b4\ubc84\uc2a4 \uc694\uae08\uc740 1200\uc6d0\uc774\uc600\uace0 \ubb34\uc548\uad70 \ub0a8\uc545\uc9c0\uad6c\uae4c\uc9c0 \uc774\uc6a9\ud558\uac8c \ub418\uba74 1350\uc6d0\uc744 \ub0b4\uc57c \ud588\uc9c0\ub9cc 2014\ub144 1\uc6d4 1\uc77c\ubd80\ud130 \ub2e8\uc77c\ud654\uac00 \ub418\uc5b4 \uae30\uc874 \uc694\uae08\uc73c\ub85c\ub3c4 \ub0a8\uc545\uc9c0\uc5ed\uc744 \uc774\uc6a9\ud560 \uc218 \uc788\uac8c \ub418\uc5c8\ub2e4. \uadf8\ub807\uc9c0\ub9cc \ub2e4\ub978 \ubb38\uc81c\ub3c4 \ub9ce\ub2e4. \uc4f0\ub808\uae30 \ubb38\uc81c\uc640 \ud3ed\uc124\uac19\uc740 \uc7ac\ud574 \ub300\ube44 \ub2a5\ub825\ub4f1\ub3c4 \ubb34\uc548\uad70 \ud589\uc815\ub825\uc774 \ubaa9\ud3ec\uc2dc \ud589\uc815\ub825\uc640 \uac19\uc740 \ub3c4\uc2dc\ud589\uc815\uc744 \ub530\ub77c\uac00\uc9c0\ub97c \ubabb\ud558\ub294 \uac83\uc774 \ud604\uc2e4\uc774\ub2e4."} {"question": "\ub274\uc695\uc758 \ubaa9\uc870 \uac74\ucd95 \uac74\uc124\uc774 \uc81c\ud55c\ub41c \ubc30\uacbd\uc740?", "paragraph": "\ub274\uc695\uc758 \ub300\uaddc\ubaa8 \uc8fc\uac70 \uc9c0\uc5ed\uc740 \uac08\uc0c9 \uc0ac\uc554\uc73c\ub85c \ub41c \ud14c\ub77c\uc2a4 \ud558\uc6b0\uc2a4, \ud0c0\uc6b4\ud558\uc6b0\uc2a4, \uadf8\ub9ac\uace0 1870\ub144\ubd80\ud130 1930\ub144\uae4c\uc9c0\uc758 \uae09\uc18d\ud55c \uac1c\ubc1c\uae30\uc5d0 \uc9c0\uc5b4\uc9c4 \ud5c8\ub984\ud55c \uc8fc\ud0dd \ub2e8\uc9c0\ub85c \uacbd\uacc4\uac00 \ub098\ub258\ub294 \uacbd\uc6b0\uac00 \ub9ce\ub2e4. 1835\ub144 \ub274\uc695 \ub300\ud654\uc7ac \ud6c4, \ubaa9\uc870 \uac74\ucd95\uc758 \uac74\uc124\uc774 \uc81c\ud55c\ub418\uc5c8\uc73c\uba70, \ub3cc\uacfc \ubcbd\ub3cc\uc744 \uc8fc\ub85c \uac74\ucd95 \uc790\uc7ac\ub85c\ud558\uc5ec \uc9d1\uc744 \uc9c0\uc5c8\ub2e4. \uba87 \uc138\uae30\uc5d0 \uac78\uccd0 \ub9c8\uc744\uc758 \uc11d\ud68c\uc554 \uc9c0\ubc18\uc5d0\uc11c \uac74\ucd95 \uc790\uc7ac\ub97c \uc5bb\uc5c8\ub358 \ud30c\ub9ac\uc640 \ub2ec\ub9ac, \ub274\uc695\uc740 \ub113\uc740 \ucc44\uc11d\uc7a5\uc5d0\uc11c \uac74\ucd95 \uc790\uc7ac\ub97c \uc5bb\uc5b4 \uc11d\uc870 \uac74\ubb3c\uc744 \uc9c0\uc5c8\uc73c\uba70, \uadf8 \uc11d\uc870 \uac74\ubb3c\uc744 \uad6c\uc131\ud558\ub294 \ub3cc\uc758 \uc885\ub958\ub294 \ub2e4\uc591\ud558\ub2e4. 1800\ub144\ub300\uc758 \ub274\uc695\uc740 6\uce35\ubcf4\ub2e4 \ub192\uc740 \uac74\ubb3c\uc740 \uae09\uc218\ud0d1\uc744 \uc124\uce58\ud558\uc9c0 \uc54a\uc73c\uba74, \ub0ae\uc740 \uce35\uc5d0\uc11c \uc9c0\ub098\uce58\uac8c \ub192\uc740 \uc218\uc555\uc744 \uac78\uc5b4\uc57c\ud558\uace0, \uadf8\ub807\uac8c \ub418\uba74 \uc218\ub3c4\uad00 \ud30c\uc5f4\uc758 \uc704\ud5d8\uc774 \uc788\uc5c8\ub2e4. 1920\ub144\ub300\uc5d0\ub294 \uc911\uc2ec\ubd80\uc5d0\uc11c \ub5a8\uc5b4\uc9c4 \uc9c0\uc5ed\uc5d0\uc11c \uc804\uc6d0\ub3c4\uc2dc\uac00 \uc131\ud589\ud588\uc73c\uba70, \ud038\uc2a4 \uc9c0\uc5ed\uc758 \uc7ad\uc2a8 \ud558\uc774\uce20\ub294 \uc804\uc6d0\ub3c4\uc2dc \uc911 \ud558\ub098\uc774\ub2e4.", "answer": "1835\ub144 \ub274\uc695 \ub300\ud654\uc7ac", "sentence": "1835\ub144 \ub274\uc695 \ub300\ud654\uc7ac \ud6c4, \ubaa9\uc870 \uac74\ucd95\uc758 \uac74\uc124\uc774 \uc81c\ud55c\ub418\uc5c8\uc73c\uba70, \ub3cc\uacfc \ubcbd\ub3cc\uc744 \uc8fc\ub85c \uac74\ucd95 \uc790\uc7ac\ub85c\ud558\uc5ec \uc9d1\uc744 \uc9c0\uc5c8\ub2e4.", "paragraph_sentence": "\ub274\uc695\uc758 \ub300\uaddc\ubaa8 \uc8fc\uac70 \uc9c0\uc5ed\uc740 \uac08\uc0c9 \uc0ac\uc554\uc73c\ub85c \ub41c \ud14c\ub77c\uc2a4 \ud558\uc6b0\uc2a4, \ud0c0\uc6b4\ud558\uc6b0\uc2a4, \uadf8\ub9ac\uace0 1870\ub144\ubd80\ud130 1930\ub144\uae4c\uc9c0\uc758 \uae09\uc18d\ud55c \uac1c\ubc1c\uae30\uc5d0 \uc9c0\uc5b4\uc9c4 \ud5c8\ub984\ud55c \uc8fc\ud0dd \ub2e8\uc9c0\ub85c \uacbd\uacc4\uac00 \ub098\ub258\ub294 \uacbd\uc6b0\uac00 \ub9ce\ub2e4. 1835\ub144 \ub274\uc695 \ub300\ud654\uc7ac \ud6c4, \ubaa9\uc870 \uac74\ucd95\uc758 \uac74\uc124\uc774 \uc81c\ud55c\ub418\uc5c8\uc73c\uba70, \ub3cc\uacfc \ubcbd\ub3cc\uc744 \uc8fc\ub85c \uac74\ucd95 \uc790\uc7ac\ub85c\ud558\uc5ec \uc9d1\uc744 \uc9c0\uc5c8\ub2e4. \uba87 \uc138\uae30\uc5d0 \uac78\uccd0 \ub9c8\uc744\uc758 \uc11d\ud68c\uc554 \uc9c0\ubc18\uc5d0\uc11c \uac74\ucd95 \uc790\uc7ac\ub97c \uc5bb\uc5c8\ub358 \ud30c\ub9ac\uc640 \ub2ec\ub9ac, \ub274\uc695\uc740 \ub113\uc740 \ucc44\uc11d\uc7a5\uc5d0\uc11c \uac74\ucd95 \uc790\uc7ac\ub97c \uc5bb\uc5b4 \uc11d\uc870 \uac74\ubb3c\uc744 \uc9c0\uc5c8\uc73c\uba70, \uadf8 \uc11d\uc870 \uac74\ubb3c\uc744 \uad6c\uc131\ud558\ub294 \ub3cc\uc758 \uc885\ub958\ub294 \ub2e4\uc591\ud558\ub2e4. 1800\ub144\ub300\uc758 \ub274\uc695\uc740 6\uce35\ubcf4\ub2e4 \ub192\uc740 \uac74\ubb3c\uc740 \uae09\uc218\ud0d1\uc744 \uc124\uce58\ud558\uc9c0 \uc54a\uc73c\uba74, \ub0ae\uc740 \uce35\uc5d0\uc11c \uc9c0\ub098\uce58\uac8c \ub192\uc740 \uc218\uc555\uc744 \uac78\uc5b4\uc57c\ud558\uace0, \uadf8\ub807\uac8c \ub418\uba74 \uc218\ub3c4\uad00 \ud30c\uc5f4\uc758 \uc704\ud5d8\uc774 \uc788\uc5c8\ub2e4. 1920\ub144\ub300\uc5d0\ub294 \uc911\uc2ec\ubd80\uc5d0\uc11c \ub5a8\uc5b4\uc9c4 \uc9c0\uc5ed\uc5d0\uc11c \uc804\uc6d0\ub3c4\uc2dc\uac00 \uc131\ud589\ud588\uc73c\uba70, \ud038\uc2a4 \uc9c0\uc5ed\uc758 \uc7ad\uc2a8 \ud558\uc774\uce20\ub294 \uc804\uc6d0\ub3c4\uc2dc \uc911 \ud558\ub098\uc774\ub2e4.", "paragraph_answer": "\ub274\uc695\uc758 \ub300\uaddc\ubaa8 \uc8fc\uac70 \uc9c0\uc5ed\uc740 \uac08\uc0c9 \uc0ac\uc554\uc73c\ub85c \ub41c \ud14c\ub77c\uc2a4 \ud558\uc6b0\uc2a4, \ud0c0\uc6b4\ud558\uc6b0\uc2a4, \uadf8\ub9ac\uace0 1870\ub144\ubd80\ud130 1930\ub144\uae4c\uc9c0\uc758 \uae09\uc18d\ud55c \uac1c\ubc1c\uae30\uc5d0 \uc9c0\uc5b4\uc9c4 \ud5c8\ub984\ud55c \uc8fc\ud0dd \ub2e8\uc9c0\ub85c \uacbd\uacc4\uac00 \ub098\ub258\ub294 \uacbd\uc6b0\uac00 \ub9ce\ub2e4. 1835\ub144 \ub274\uc695 \ub300\ud654\uc7ac \ud6c4, \ubaa9\uc870 \uac74\ucd95\uc758 \uac74\uc124\uc774 \uc81c\ud55c\ub418\uc5c8\uc73c\uba70, \ub3cc\uacfc \ubcbd\ub3cc\uc744 \uc8fc\ub85c \uac74\ucd95 \uc790\uc7ac\ub85c\ud558\uc5ec \uc9d1\uc744 \uc9c0\uc5c8\ub2e4. \uba87 \uc138\uae30\uc5d0 \uac78\uccd0 \ub9c8\uc744\uc758 \uc11d\ud68c\uc554 \uc9c0\ubc18\uc5d0\uc11c \uac74\ucd95 \uc790\uc7ac\ub97c \uc5bb\uc5c8\ub358 \ud30c\ub9ac\uc640 \ub2ec\ub9ac, \ub274\uc695\uc740 \ub113\uc740 \ucc44\uc11d\uc7a5\uc5d0\uc11c \uac74\ucd95 \uc790\uc7ac\ub97c \uc5bb\uc5b4 \uc11d\uc870 \uac74\ubb3c\uc744 \uc9c0\uc5c8\uc73c\uba70, \uadf8 \uc11d\uc870 \uac74\ubb3c\uc744 \uad6c\uc131\ud558\ub294 \ub3cc\uc758 \uc885\ub958\ub294 \ub2e4\uc591\ud558\ub2e4. 1800\ub144\ub300\uc758 \ub274\uc695\uc740 6\uce35\ubcf4\ub2e4 \ub192\uc740 \uac74\ubb3c\uc740 \uae09\uc218\ud0d1\uc744 \uc124\uce58\ud558\uc9c0 \uc54a\uc73c\uba74, \ub0ae\uc740 \uce35\uc5d0\uc11c \uc9c0\ub098\uce58\uac8c \ub192\uc740 \uc218\uc555\uc744 \uac78\uc5b4\uc57c\ud558\uace0, \uadf8\ub807\uac8c \ub418\uba74 \uc218\ub3c4\uad00 \ud30c\uc5f4\uc758 \uc704\ud5d8\uc774 \uc788\uc5c8\ub2e4. 1920\ub144\ub300\uc5d0\ub294 \uc911\uc2ec\ubd80\uc5d0\uc11c \ub5a8\uc5b4\uc9c4 \uc9c0\uc5ed\uc5d0\uc11c \uc804\uc6d0\ub3c4\uc2dc\uac00 \uc131\ud589\ud588\uc73c\uba70, \ud038\uc2a4 \uc9c0\uc5ed\uc758 \uc7ad\uc2a8 \ud558\uc774\uce20\ub294 \uc804\uc6d0\ub3c4\uc2dc \uc911 \ud558\ub098\uc774\ub2e4.", "sentence_answer": " 1835\ub144 \ub274\uc695 \ub300\ud654\uc7ac \ud6c4, \ubaa9\uc870 \uac74\ucd95\uc758 \uac74\uc124\uc774 \uc81c\ud55c\ub418\uc5c8\uc73c\uba70, \ub3cc\uacfc \ubcbd\ub3cc\uc744 \uc8fc\ub85c \uac74\ucd95 \uc790\uc7ac\ub85c\ud558\uc5ec \uc9d1\uc744 \uc9c0\uc5c8\ub2e4.", "paragraph_id": "6383054-6-0", "paragraph_question": "question: \ub274\uc695\uc758 \ubaa9\uc870 \uac74\ucd95 \uac74\uc124\uc774 \uc81c\ud55c\ub41c \ubc30\uacbd\uc740?, context: \ub274\uc695\uc758 \ub300\uaddc\ubaa8 \uc8fc\uac70 \uc9c0\uc5ed\uc740 \uac08\uc0c9 \uc0ac\uc554\uc73c\ub85c \ub41c \ud14c\ub77c\uc2a4 \ud558\uc6b0\uc2a4, \ud0c0\uc6b4\ud558\uc6b0\uc2a4, \uadf8\ub9ac\uace0 1870\ub144\ubd80\ud130 1930\ub144\uae4c\uc9c0\uc758 \uae09\uc18d\ud55c \uac1c\ubc1c\uae30\uc5d0 \uc9c0\uc5b4\uc9c4 \ud5c8\ub984\ud55c \uc8fc\ud0dd \ub2e8\uc9c0\ub85c \uacbd\uacc4\uac00 \ub098\ub258\ub294 \uacbd\uc6b0\uac00 \ub9ce\ub2e4. 1835\ub144 \ub274\uc695 \ub300\ud654\uc7ac \ud6c4, \ubaa9\uc870 \uac74\ucd95\uc758 \uac74\uc124\uc774 \uc81c\ud55c\ub418\uc5c8\uc73c\uba70, \ub3cc\uacfc \ubcbd\ub3cc\uc744 \uc8fc\ub85c \uac74\ucd95 \uc790\uc7ac\ub85c\ud558\uc5ec \uc9d1\uc744 \uc9c0\uc5c8\ub2e4. \uba87 \uc138\uae30\uc5d0 \uac78\uccd0 \ub9c8\uc744\uc758 \uc11d\ud68c\uc554 \uc9c0\ubc18\uc5d0\uc11c \uac74\ucd95 \uc790\uc7ac\ub97c \uc5bb\uc5c8\ub358 \ud30c\ub9ac\uc640 \ub2ec\ub9ac, \ub274\uc695\uc740 \ub113\uc740 \ucc44\uc11d\uc7a5\uc5d0\uc11c \uac74\ucd95 \uc790\uc7ac\ub97c \uc5bb\uc5b4 \uc11d\uc870 \uac74\ubb3c\uc744 \uc9c0\uc5c8\uc73c\uba70, \uadf8 \uc11d\uc870 \uac74\ubb3c\uc744 \uad6c\uc131\ud558\ub294 \ub3cc\uc758 \uc885\ub958\ub294 \ub2e4\uc591\ud558\ub2e4. 1800\ub144\ub300\uc758 \ub274\uc695\uc740 6\uce35\ubcf4\ub2e4 \ub192\uc740 \uac74\ubb3c\uc740 \uae09\uc218\ud0d1\uc744 \uc124\uce58\ud558\uc9c0 \uc54a\uc73c\uba74, \ub0ae\uc740 \uce35\uc5d0\uc11c \uc9c0\ub098\uce58\uac8c \ub192\uc740 \uc218\uc555\uc744 \uac78\uc5b4\uc57c\ud558\uace0, \uadf8\ub807\uac8c \ub418\uba74 \uc218\ub3c4\uad00 \ud30c\uc5f4\uc758 \uc704\ud5d8\uc774 \uc788\uc5c8\ub2e4. 1920\ub144\ub300\uc5d0\ub294 \uc911\uc2ec\ubd80\uc5d0\uc11c \ub5a8\uc5b4\uc9c4 \uc9c0\uc5ed\uc5d0\uc11c \uc804\uc6d0\ub3c4\uc2dc\uac00 \uc131\ud589\ud588\uc73c\uba70, \ud038\uc2a4 \uc9c0\uc5ed\uc758 \uc7ad\uc2a8 \ud558\uc774\uce20\ub294 \uc804\uc6d0\ub3c4\uc2dc \uc911 \ud558\ub098\uc774\ub2e4."} {"question": "\uac00\ubb3c\uce58\ub294 \ube44\uac00 \uc62c \ub54c\ub294 \uc5b4\ub514\ub97c \uae30\uc5b4 \ub2e4\ub2c8\ub294\uac00?", "paragraph": "\uac00\ubb3c\uce58\ub294 \ubbfc\ubb3c\uace0\uae30\ub85c\uc11c, \uc5fc\ub3c4\uac00 \ub192\uc740 \ubb3c\uc5d0\uc11c\ub294 \uc0b4 \uc218 \uc5c6\ub2e4. \uc801\uc751\ub825\uc774 \ub6f0\uc5b4\ub098 \ud55c\ubc18\ub3c4\uc5d0\uc11c\ub294 \ub2aa\uc774\ub098 \ubed8, \uc5f0\ubabb, \ud638\uc218, \ud558\ucc9c \ub4f1 \ucda9\ubd84\ud55c \uacf5\uac04\uacfc \uc218\uad8c, \uba39\uc774\uac00 \ud655\ubcf4\ub41c \uacf3\uc5d0\uc11c\ub77c\uba74 \uc5b4\ub514\uc5d0\uc11c\ub77c\ub3c4 \uc790\ub784 \uc218 \uc788\uc73c\uba70, \uc218\uc628\uacfc \uc624\uc5fc\ub3c4\uc5d0 \uac70\uc758 \uad6c\uc560\ubc1b\uc9c0 \uc54a\uc73c\ub098 \uadf8 \ub54c\ubb38\uc5d0 \ubab8 \uc18d\uc5d0 \uac01\uc885 \uc0b0\uc5c5 \ud3d0\uae30\ubb3c, \ub610\ub294 \uce58\uba85\uc801\uc778 \uc77c\ubd80 \uc911\uae08\uc18d\uc774 \ucd95\uc801\ub418\uae30\ub3c4 \ud55c\ub2e4. \ud2b9\ud788 \uc218\uc740\uc740 \ub300\ubd80\ubd84\uc758 \ubbfc\ubb3c\uace0\uae30\ubcf4\ub2e4 \uac00\ubb3c\uce58\uc5d0\uac8c\uc11c \ubcf4\ub2e4 \ub9ce\uc740 \uc591\uc774 \uac80\ucd9c\ub418\uc5c8\ub2e4. \ub300\uac1c\ub294 \uc595\uc740 \ubb3c\uc744 \uc120\ud638\ud55c\ub2e4. \uacf5\uaca9\uc131\uc774 \uc544\uc8fc \uac15\ud55c \uc721\uc2dd\uc131 \uc5b4\uc885\uc73c\ub85c \uc131\uc9c8\uc774 \uc0ac\ub0a9\uace0 \ud798\uacfc \uc545\ub825\uc774 \uac15\ud558\uc5ec \ub2aa\uc758 \ubb34\ubc95\uc790\ub77c\uace0 \ubd88\ub9ac\uba70, \uce58\uc5b4 \uc2dc\uae30\uc5d0\ub294 \uc8fc\ub85c \ubb3c\ubcbc\ub8e9\uc744 \uba39\uc9c0\ub9cc \uc131\uccb4\uac00 \ub418\uba74 \uc8fc\ub85c \uac1c\uad6c\ub9ac\u00b7\ub3c4\ub871\ub1fd \ub4f1\uc758 \uc591\uc11c\ub958\uc640 \uac00\uc7ac\u00b7\ubbfc\ubb3c\uc0c8\uc6b0 \ub4f1\uc758 \uac11\uac01\ub958, \ubd95\uc5b4\u00b7\ub0a9\uc790\ub8e8\u00b7\ubbf8\uafb8\ub77c\uc9c0 \ub4f1\uc758 \uc18c\ud615 \uc5b4\ub958\ub97c \uba39\uc9c0\ub9cc, \uba54\uae30 \ub4f1\uc758 \ub300\ud615 \uc5b4\ub958\ub97c \uc7a1\uc544\uba39\uac70\ub098 \ub3d9\uc871\uc744 \uacf5\uaca9\ud558\ub294 \uc77c\ub3c4 \uc788\ub2e4. \ube44\uac00 \uc62c \ub54c\ub294 \ub2aa \ubc11\uc744 \uae30\uba74\uc11c \ub2e4\ub2c8\uae30\ub3c4 \ud55c\ub2e4.", "answer": "\ub2aa \ubc11", "sentence": "\ube44\uac00 \uc62c \ub54c\ub294 \ub2aa \ubc11 \uc744 \uae30\uba74\uc11c \ub2e4\ub2c8\uae30\ub3c4 \ud55c\ub2e4.", "paragraph_sentence": "\uac00\ubb3c\uce58\ub294 \ubbfc\ubb3c\uace0\uae30\ub85c\uc11c, \uc5fc\ub3c4\uac00 \ub192\uc740 \ubb3c\uc5d0\uc11c\ub294 \uc0b4 \uc218 \uc5c6\ub2e4. \uc801\uc751\ub825\uc774 \ub6f0\uc5b4\ub098 \ud55c\ubc18\ub3c4\uc5d0\uc11c\ub294 \ub2aa\uc774\ub098 \ubed8, \uc5f0\ubabb, \ud638\uc218, \ud558\ucc9c \ub4f1 \ucda9\ubd84\ud55c \uacf5\uac04\uacfc \uc218\uad8c, \uba39\uc774\uac00 \ud655\ubcf4\ub41c \uacf3\uc5d0\uc11c\ub77c\uba74 \uc5b4\ub514\uc5d0\uc11c\ub77c\ub3c4 \uc790\ub784 \uc218 \uc788\uc73c\uba70, \uc218\uc628\uacfc \uc624\uc5fc\ub3c4\uc5d0 \uac70\uc758 \uad6c\uc560\ubc1b\uc9c0 \uc54a\uc73c\ub098 \uadf8 \ub54c\ubb38\uc5d0 \ubab8 \uc18d\uc5d0 \uac01\uc885 \uc0b0\uc5c5 \ud3d0\uae30\ubb3c, \ub610\ub294 \uce58\uba85\uc801\uc778 \uc77c\ubd80 \uc911\uae08\uc18d\uc774 \ucd95\uc801\ub418\uae30\ub3c4 \ud55c\ub2e4. \ud2b9\ud788 \uc218\uc740\uc740 \ub300\ubd80\ubd84\uc758 \ubbfc\ubb3c\uace0\uae30\ubcf4\ub2e4 \uac00\ubb3c\uce58\uc5d0\uac8c\uc11c \ubcf4\ub2e4 \ub9ce\uc740 \uc591\uc774 \uac80\ucd9c\ub418\uc5c8\ub2e4. \ub300\uac1c\ub294 \uc595\uc740 \ubb3c\uc744 \uc120\ud638\ud55c\ub2e4. \uacf5\uaca9\uc131\uc774 \uc544\uc8fc \uac15\ud55c \uc721\uc2dd\uc131 \uc5b4\uc885\uc73c\ub85c \uc131\uc9c8\uc774 \uc0ac\ub0a9\uace0 \ud798\uacfc \uc545\ub825\uc774 \uac15\ud558\uc5ec \ub2aa\uc758 \ubb34\ubc95\uc790\ub77c\uace0 \ubd88\ub9ac\uba70, \uce58\uc5b4 \uc2dc\uae30\uc5d0\ub294 \uc8fc\ub85c \ubb3c\ubcbc\ub8e9\uc744 \uba39\uc9c0\ub9cc \uc131\uccb4\uac00 \ub418\uba74 \uc8fc\ub85c \uac1c\uad6c\ub9ac\u00b7\ub3c4\ub871\ub1fd \ub4f1\uc758 \uc591\uc11c\ub958\uc640 \uac00\uc7ac\u00b7\ubbfc\ubb3c\uc0c8\uc6b0 \ub4f1\uc758 \uac11\uac01\ub958, \ubd95\uc5b4\u00b7\ub0a9\uc790\ub8e8\u00b7\ubbf8\uafb8\ub77c\uc9c0 \ub4f1\uc758 \uc18c\ud615 \uc5b4\ub958\ub97c \uba39\uc9c0\ub9cc, \uba54\uae30 \ub4f1\uc758 \ub300\ud615 \uc5b4\ub958\ub97c \uc7a1\uc544\uba39\uac70\ub098 \ub3d9\uc871\uc744 \uacf5\uaca9\ud558\ub294 \uc77c\ub3c4 \uc788\ub2e4. \ube44\uac00 \uc62c \ub54c\ub294 \ub2aa \ubc11 \uc744 \uae30\uba74\uc11c \ub2e4\ub2c8\uae30\ub3c4 \ud55c\ub2e4. ", "paragraph_answer": "\uac00\ubb3c\uce58\ub294 \ubbfc\ubb3c\uace0\uae30\ub85c\uc11c, \uc5fc\ub3c4\uac00 \ub192\uc740 \ubb3c\uc5d0\uc11c\ub294 \uc0b4 \uc218 \uc5c6\ub2e4. \uc801\uc751\ub825\uc774 \ub6f0\uc5b4\ub098 \ud55c\ubc18\ub3c4\uc5d0\uc11c\ub294 \ub2aa\uc774\ub098 \ubed8, \uc5f0\ubabb, \ud638\uc218, \ud558\ucc9c \ub4f1 \ucda9\ubd84\ud55c \uacf5\uac04\uacfc \uc218\uad8c, \uba39\uc774\uac00 \ud655\ubcf4\ub41c \uacf3\uc5d0\uc11c\ub77c\uba74 \uc5b4\ub514\uc5d0\uc11c\ub77c\ub3c4 \uc790\ub784 \uc218 \uc788\uc73c\uba70, \uc218\uc628\uacfc \uc624\uc5fc\ub3c4\uc5d0 \uac70\uc758 \uad6c\uc560\ubc1b\uc9c0 \uc54a\uc73c\ub098 \uadf8 \ub54c\ubb38\uc5d0 \ubab8 \uc18d\uc5d0 \uac01\uc885 \uc0b0\uc5c5 \ud3d0\uae30\ubb3c, \ub610\ub294 \uce58\uba85\uc801\uc778 \uc77c\ubd80 \uc911\uae08\uc18d\uc774 \ucd95\uc801\ub418\uae30\ub3c4 \ud55c\ub2e4. \ud2b9\ud788 \uc218\uc740\uc740 \ub300\ubd80\ubd84\uc758 \ubbfc\ubb3c\uace0\uae30\ubcf4\ub2e4 \uac00\ubb3c\uce58\uc5d0\uac8c\uc11c \ubcf4\ub2e4 \ub9ce\uc740 \uc591\uc774 \uac80\ucd9c\ub418\uc5c8\ub2e4. \ub300\uac1c\ub294 \uc595\uc740 \ubb3c\uc744 \uc120\ud638\ud55c\ub2e4. \uacf5\uaca9\uc131\uc774 \uc544\uc8fc \uac15\ud55c \uc721\uc2dd\uc131 \uc5b4\uc885\uc73c\ub85c \uc131\uc9c8\uc774 \uc0ac\ub0a9\uace0 \ud798\uacfc \uc545\ub825\uc774 \uac15\ud558\uc5ec \ub2aa\uc758 \ubb34\ubc95\uc790\ub77c\uace0 \ubd88\ub9ac\uba70, \uce58\uc5b4 \uc2dc\uae30\uc5d0\ub294 \uc8fc\ub85c \ubb3c\ubcbc\ub8e9\uc744 \uba39\uc9c0\ub9cc \uc131\uccb4\uac00 \ub418\uba74 \uc8fc\ub85c \uac1c\uad6c\ub9ac\u00b7\ub3c4\ub871\ub1fd \ub4f1\uc758 \uc591\uc11c\ub958\uc640 \uac00\uc7ac\u00b7\ubbfc\ubb3c\uc0c8\uc6b0 \ub4f1\uc758 \uac11\uac01\ub958, \ubd95\uc5b4\u00b7\ub0a9\uc790\ub8e8\u00b7\ubbf8\uafb8\ub77c\uc9c0 \ub4f1\uc758 \uc18c\ud615 \uc5b4\ub958\ub97c \uba39\uc9c0\ub9cc, \uba54\uae30 \ub4f1\uc758 \ub300\ud615 \uc5b4\ub958\ub97c \uc7a1\uc544\uba39\uac70\ub098 \ub3d9\uc871\uc744 \uacf5\uaca9\ud558\ub294 \uc77c\ub3c4 \uc788\ub2e4. \ube44\uac00 \uc62c \ub54c\ub294 \ub2aa \ubc11 \uc744 \uae30\uba74\uc11c \ub2e4\ub2c8\uae30\ub3c4 \ud55c\ub2e4.", "sentence_answer": "\ube44\uac00 \uc62c \ub54c\ub294 \ub2aa \ubc11 \uc744 \uae30\uba74\uc11c \ub2e4\ub2c8\uae30\ub3c4 \ud55c\ub2e4.", "paragraph_id": "6548393-1-1", "paragraph_question": "question: \uac00\ubb3c\uce58\ub294 \ube44\uac00 \uc62c \ub54c\ub294 \uc5b4\ub514\ub97c \uae30\uc5b4 \ub2e4\ub2c8\ub294\uac00?, context: \uac00\ubb3c\uce58\ub294 \ubbfc\ubb3c\uace0\uae30\ub85c\uc11c, \uc5fc\ub3c4\uac00 \ub192\uc740 \ubb3c\uc5d0\uc11c\ub294 \uc0b4 \uc218 \uc5c6\ub2e4. \uc801\uc751\ub825\uc774 \ub6f0\uc5b4\ub098 \ud55c\ubc18\ub3c4\uc5d0\uc11c\ub294 \ub2aa\uc774\ub098 \ubed8, \uc5f0\ubabb, \ud638\uc218, \ud558\ucc9c \ub4f1 \ucda9\ubd84\ud55c \uacf5\uac04\uacfc \uc218\uad8c, \uba39\uc774\uac00 \ud655\ubcf4\ub41c \uacf3\uc5d0\uc11c\ub77c\uba74 \uc5b4\ub514\uc5d0\uc11c\ub77c\ub3c4 \uc790\ub784 \uc218 \uc788\uc73c\uba70, \uc218\uc628\uacfc \uc624\uc5fc\ub3c4\uc5d0 \uac70\uc758 \uad6c\uc560\ubc1b\uc9c0 \uc54a\uc73c\ub098 \uadf8 \ub54c\ubb38\uc5d0 \ubab8 \uc18d\uc5d0 \uac01\uc885 \uc0b0\uc5c5 \ud3d0\uae30\ubb3c, \ub610\ub294 \uce58\uba85\uc801\uc778 \uc77c\ubd80 \uc911\uae08\uc18d\uc774 \ucd95\uc801\ub418\uae30\ub3c4 \ud55c\ub2e4. \ud2b9\ud788 \uc218\uc740\uc740 \ub300\ubd80\ubd84\uc758 \ubbfc\ubb3c\uace0\uae30\ubcf4\ub2e4 \uac00\ubb3c\uce58\uc5d0\uac8c\uc11c \ubcf4\ub2e4 \ub9ce\uc740 \uc591\uc774 \uac80\ucd9c\ub418\uc5c8\ub2e4. \ub300\uac1c\ub294 \uc595\uc740 \ubb3c\uc744 \uc120\ud638\ud55c\ub2e4. \uacf5\uaca9\uc131\uc774 \uc544\uc8fc \uac15\ud55c \uc721\uc2dd\uc131 \uc5b4\uc885\uc73c\ub85c \uc131\uc9c8\uc774 \uc0ac\ub0a9\uace0 \ud798\uacfc \uc545\ub825\uc774 \uac15\ud558\uc5ec \ub2aa\uc758 \ubb34\ubc95\uc790\ub77c\uace0 \ubd88\ub9ac\uba70, \uce58\uc5b4 \uc2dc\uae30\uc5d0\ub294 \uc8fc\ub85c \ubb3c\ubcbc\ub8e9\uc744 \uba39\uc9c0\ub9cc \uc131\uccb4\uac00 \ub418\uba74 \uc8fc\ub85c \uac1c\uad6c\ub9ac\u00b7\ub3c4\ub871\ub1fd \ub4f1\uc758 \uc591\uc11c\ub958\uc640 \uac00\uc7ac\u00b7\ubbfc\ubb3c\uc0c8\uc6b0 \ub4f1\uc758 \uac11\uac01\ub958, \ubd95\uc5b4\u00b7\ub0a9\uc790\ub8e8\u00b7\ubbf8\uafb8\ub77c\uc9c0 \ub4f1\uc758 \uc18c\ud615 \uc5b4\ub958\ub97c \uba39\uc9c0\ub9cc, \uba54\uae30 \ub4f1\uc758 \ub300\ud615 \uc5b4\ub958\ub97c \uc7a1\uc544\uba39\uac70\ub098 \ub3d9\uc871\uc744 \uacf5\uaca9\ud558\ub294 \uc77c\ub3c4 \uc788\ub2e4. \ube44\uac00 \uc62c \ub54c\ub294 \ub2aa \ubc11\uc744 \uae30\uba74\uc11c \ub2e4\ub2c8\uae30\ub3c4 \ud55c\ub2e4."} {"question": "\ud601\uc2e0\uc548 \ubd80\uacb0 \uc774\ud6c4 \ub450\ub2ec \ub4a4\uc778 4\uc6d4\uae4c\uc9c0 \ubbfc\uc8fc\ub178\ub3d9\ub2f9\uc744 \ud0c8\ub2f9\ud55c \ub2f9\uc6d0\uc740 \uba87 \uba85\uc778\uac00?", "paragraph": "\uacb0\uad6d 2008\ub144 2\uc6d4 8\uc77c, \uc13c\ud2b8\ub7f4\uc2dc\ud2f0 9\uce35 \ub300\ud68c\uc758\uc2e4\uc5d0\uc11c \uc9c4\ud589\ub41c \ubbfc\uc8fc\ub178\ub3d9\ub2f9 \uc784\uc2dc\ub2f9\ub300\ud68c\uc5d0\uc11c \uc790\uc8fc\ud30c \ub300\uc758\uc6d0\ub4e4\uc758 \ubc18\ub300\ub85c \ud601\uc2e0\uc548\uc758 \ud575\uc2ec \ub0b4\uc6a9\uc740 \ubd80\uacb0\ub418\uc5c8\uc73c\uba70 \uc790\uc8fc\ud30c\uc758 \uc218\uc815\uc548\ub300\ub85c \ud1b5\uacfc\ub418\uc5c8\ub2e4. \ud601\uc2e0\uc548\uc774 \ubd80\uacb0\ub418\uc790 \uc2ec\uc0c1\uc815 \uc704\uc6d0\uc7a5 \ub4f1 \ube44\ub300\uc704\uc6d0 \uc804\uc6d0\uc774 \uc0ac\ud1f4\ud558\uc600\uace0, \ubbfc\uc8fc\ub178\ub3d9\ub2f9 \uc81c1\uae30\uc640 \uc81c2\uae30 \uc9c0\ub3c4\ubd80\ub97c \uad6c\uc131\ud558\uc600\ub358 \uc778\ubb3c\ub4e4 \uac00\uc6b4\ub370 \uae40\ud61c\uacbd, \uc8fc\ub300\ud658, \uae40\uc885\ucca0, \uc2ec\uc7ac\uc625, \ud64d\uc2b9\ud558, \uae40\uae30\uc218 \ub4f1\uc774 \ud0c8\ub2f9\ud558\uc600\uc73c\uba70, \ubbfc\uc8fc\ub178\ub3d9\ub2f9 \uc18c\uc18d \uad6d\ud68c\uc758\uc6d0 \uc2ec\uc0c1\uc815, \ub178\ud68c\ucc2c, \uc870\uc2b9\uc218, \ub2e8\ubcd1\ud638\ub3c4 \ud0c8\ub2f9\ud558\uc600\ub2e4. \uc784\uc2dc\ub2f9\ub300\ud68c \uc774\ud6c4 2008\ub144 4\uc6d4\uae4c\uc9c0 \ubbfc\uc8fc\ub178\ub3d9\ub2f9\uc744 \ud0c8\ub2f9\ud55c \uc0ac\ub78c\uc740 16,904\uba85\uc5d0 \uc774\ub974\ub800\uace0, 2008\ub144 1\uc6d4 \ub2f9\uc2dc 11\ub9cc\uc5ec \uba85\uc5d0 \uc774\ub974\ub358 \ubbfc\uc8fc\ub178\ub3d9\ub2f9\uc758 \ub2f9\uc6d0 \uc218\ub294 9\ub9cc 4\ucc9c\uc5ec \uba85\uc73c\ub85c \uc904\uc5c8\ub2e4. \ubbfc\uc8fc\ub178\ub3d9\ub2f9\uc744 \ud0c8\ub2f9\ud55c \uc0ac\ub78c\ub4e4 \uc911 \uc77c\ubd80\ub294 2008\ub144 2\uc6d4 21\uc77c '\uc9c4\ubcf4\uc2e0\ub2f9\uc5f0\ub300\ud68c\uc758' \uc900\ube44\uc704\uc6d0\ud68c\ub97c \uacb0\uc131\ud558\uc600\uc73c\uba70, 2008\ub144 3\uc6d4 \uc9c4\ubcf4\uc2e0\ub2f9 \ucc3d\ub2f9\uc774 \uacf5\uc2dd \uc120\uc5b8\ub418\uc5c8\uace0, \ubbfc\uc8fc\ub178\ub3d9\ub2f9\uc740 \ubd84\ub2f9\ub418\uc5c8\ub2e4.", "answer": "16,904\uba85", "sentence": "\uc784\uc2dc\ub2f9\ub300\ud68c \uc774\ud6c4 2008\ub144 4\uc6d4\uae4c\uc9c0 \ubbfc\uc8fc\ub178\ub3d9\ub2f9\uc744 \ud0c8\ub2f9\ud55c \uc0ac\ub78c\uc740 16,904\uba85 \uc5d0 \uc774\ub974\ub800\uace0, 2008\ub144 1\uc6d4 \ub2f9\uc2dc 11\ub9cc\uc5ec \uba85\uc5d0 \uc774\ub974\ub358 \ubbfc\uc8fc\ub178\ub3d9\ub2f9\uc758 \ub2f9\uc6d0 \uc218\ub294 9\ub9cc 4\ucc9c\uc5ec \uba85\uc73c\ub85c \uc904\uc5c8\ub2e4.", "paragraph_sentence": "\uacb0\uad6d 2008\ub144 2\uc6d4 8\uc77c, \uc13c\ud2b8\ub7f4\uc2dc\ud2f0 9\uce35 \ub300\ud68c\uc758\uc2e4\uc5d0\uc11c \uc9c4\ud589\ub41c \ubbfc\uc8fc\ub178\ub3d9\ub2f9 \uc784\uc2dc\ub2f9\ub300\ud68c\uc5d0\uc11c \uc790\uc8fc\ud30c \ub300\uc758\uc6d0\ub4e4\uc758 \ubc18\ub300\ub85c \ud601\uc2e0\uc548\uc758 \ud575\uc2ec \ub0b4\uc6a9\uc740 \ubd80\uacb0\ub418\uc5c8\uc73c\uba70 \uc790\uc8fc\ud30c\uc758 \uc218\uc815\uc548\ub300\ub85c \ud1b5\uacfc\ub418\uc5c8\ub2e4. \ud601\uc2e0\uc548\uc774 \ubd80\uacb0\ub418\uc790 \uc2ec\uc0c1\uc815 \uc704\uc6d0\uc7a5 \ub4f1 \ube44\ub300\uc704\uc6d0 \uc804\uc6d0\uc774 \uc0ac\ud1f4\ud558\uc600\uace0, \ubbfc\uc8fc\ub178\ub3d9\ub2f9 \uc81c1\uae30\uc640 \uc81c2\uae30 \uc9c0\ub3c4\ubd80\ub97c \uad6c\uc131\ud558\uc600\ub358 \uc778\ubb3c\ub4e4 \uac00\uc6b4\ub370 \uae40\ud61c\uacbd, \uc8fc\ub300\ud658, \uae40\uc885\ucca0, \uc2ec\uc7ac\uc625, \ud64d\uc2b9\ud558, \uae40\uae30\uc218 \ub4f1\uc774 \ud0c8\ub2f9\ud558\uc600\uc73c\uba70, \ubbfc\uc8fc\ub178\ub3d9\ub2f9 \uc18c\uc18d \uad6d\ud68c\uc758\uc6d0 \uc2ec\uc0c1\uc815, \ub178\ud68c\ucc2c, \uc870\uc2b9\uc218, \ub2e8\ubcd1\ud638\ub3c4 \ud0c8\ub2f9\ud558\uc600\ub2e4. \uc784\uc2dc\ub2f9\ub300\ud68c \uc774\ud6c4 2008\ub144 4\uc6d4\uae4c\uc9c0 \ubbfc\uc8fc\ub178\ub3d9\ub2f9\uc744 \ud0c8\ub2f9\ud55c \uc0ac\ub78c\uc740 16,904\uba85 \uc5d0 \uc774\ub974\ub800\uace0, 2008\ub144 1\uc6d4 \ub2f9\uc2dc 11\ub9cc\uc5ec \uba85\uc5d0 \uc774\ub974\ub358 \ubbfc\uc8fc\ub178\ub3d9\ub2f9\uc758 \ub2f9\uc6d0 \uc218\ub294 9\ub9cc 4\ucc9c\uc5ec \uba85\uc73c\ub85c \uc904\uc5c8\ub2e4. \ubbfc\uc8fc\ub178\ub3d9\ub2f9\uc744 \ud0c8\ub2f9\ud55c \uc0ac\ub78c\ub4e4 \uc911 \uc77c\ubd80\ub294 2008\ub144 2\uc6d4 21\uc77c '\uc9c4\ubcf4\uc2e0\ub2f9\uc5f0\ub300\ud68c\uc758' \uc900\ube44\uc704\uc6d0\ud68c\ub97c \uacb0\uc131\ud558\uc600\uc73c\uba70, 2008\ub144 3\uc6d4 \uc9c4\ubcf4\uc2e0\ub2f9 \ucc3d\ub2f9\uc774 \uacf5\uc2dd \uc120\uc5b8\ub418\uc5c8\uace0, \ubbfc\uc8fc\ub178\ub3d9\ub2f9\uc740 \ubd84\ub2f9\ub418\uc5c8\ub2e4.", "paragraph_answer": "\uacb0\uad6d 2008\ub144 2\uc6d4 8\uc77c, \uc13c\ud2b8\ub7f4\uc2dc\ud2f0 9\uce35 \ub300\ud68c\uc758\uc2e4\uc5d0\uc11c \uc9c4\ud589\ub41c \ubbfc\uc8fc\ub178\ub3d9\ub2f9 \uc784\uc2dc\ub2f9\ub300\ud68c\uc5d0\uc11c \uc790\uc8fc\ud30c \ub300\uc758\uc6d0\ub4e4\uc758 \ubc18\ub300\ub85c \ud601\uc2e0\uc548\uc758 \ud575\uc2ec \ub0b4\uc6a9\uc740 \ubd80\uacb0\ub418\uc5c8\uc73c\uba70 \uc790\uc8fc\ud30c\uc758 \uc218\uc815\uc548\ub300\ub85c \ud1b5\uacfc\ub418\uc5c8\ub2e4. \ud601\uc2e0\uc548\uc774 \ubd80\uacb0\ub418\uc790 \uc2ec\uc0c1\uc815 \uc704\uc6d0\uc7a5 \ub4f1 \ube44\ub300\uc704\uc6d0 \uc804\uc6d0\uc774 \uc0ac\ud1f4\ud558\uc600\uace0, \ubbfc\uc8fc\ub178\ub3d9\ub2f9 \uc81c1\uae30\uc640 \uc81c2\uae30 \uc9c0\ub3c4\ubd80\ub97c \uad6c\uc131\ud558\uc600\ub358 \uc778\ubb3c\ub4e4 \uac00\uc6b4\ub370 \uae40\ud61c\uacbd, \uc8fc\ub300\ud658, \uae40\uc885\ucca0, \uc2ec\uc7ac\uc625, \ud64d\uc2b9\ud558, \uae40\uae30\uc218 \ub4f1\uc774 \ud0c8\ub2f9\ud558\uc600\uc73c\uba70, \ubbfc\uc8fc\ub178\ub3d9\ub2f9 \uc18c\uc18d \uad6d\ud68c\uc758\uc6d0 \uc2ec\uc0c1\uc815, \ub178\ud68c\ucc2c, \uc870\uc2b9\uc218, \ub2e8\ubcd1\ud638\ub3c4 \ud0c8\ub2f9\ud558\uc600\ub2e4. \uc784\uc2dc\ub2f9\ub300\ud68c \uc774\ud6c4 2008\ub144 4\uc6d4\uae4c\uc9c0 \ubbfc\uc8fc\ub178\ub3d9\ub2f9\uc744 \ud0c8\ub2f9\ud55c \uc0ac\ub78c\uc740 16,904\uba85 \uc5d0 \uc774\ub974\ub800\uace0, 2008\ub144 1\uc6d4 \ub2f9\uc2dc 11\ub9cc\uc5ec \uba85\uc5d0 \uc774\ub974\ub358 \ubbfc\uc8fc\ub178\ub3d9\ub2f9\uc758 \ub2f9\uc6d0 \uc218\ub294 9\ub9cc 4\ucc9c\uc5ec \uba85\uc73c\ub85c \uc904\uc5c8\ub2e4. \ubbfc\uc8fc\ub178\ub3d9\ub2f9\uc744 \ud0c8\ub2f9\ud55c \uc0ac\ub78c\ub4e4 \uc911 \uc77c\ubd80\ub294 2008\ub144 2\uc6d4 21\uc77c '\uc9c4\ubcf4\uc2e0\ub2f9\uc5f0\ub300\ud68c\uc758' \uc900\ube44\uc704\uc6d0\ud68c\ub97c \uacb0\uc131\ud558\uc600\uc73c\uba70, 2008\ub144 3\uc6d4 \uc9c4\ubcf4\uc2e0\ub2f9 \ucc3d\ub2f9\uc774 \uacf5\uc2dd \uc120\uc5b8\ub418\uc5c8\uace0, \ubbfc\uc8fc\ub178\ub3d9\ub2f9\uc740 \ubd84\ub2f9\ub418\uc5c8\ub2e4.", "sentence_answer": "\uc784\uc2dc\ub2f9\ub300\ud68c \uc774\ud6c4 2008\ub144 4\uc6d4\uae4c\uc9c0 \ubbfc\uc8fc\ub178\ub3d9\ub2f9\uc744 \ud0c8\ub2f9\ud55c \uc0ac\ub78c\uc740 16,904\uba85 \uc5d0 \uc774\ub974\ub800\uace0, 2008\ub144 1\uc6d4 \ub2f9\uc2dc 11\ub9cc\uc5ec \uba85\uc5d0 \uc774\ub974\ub358 \ubbfc\uc8fc\ub178\ub3d9\ub2f9\uc758 \ub2f9\uc6d0 \uc218\ub294 9\ub9cc 4\ucc9c\uc5ec \uba85\uc73c\ub85c \uc904\uc5c8\ub2e4.", "paragraph_id": "6580881-1-2", "paragraph_question": "question: \ud601\uc2e0\uc548 \ubd80\uacb0 \uc774\ud6c4 \ub450\ub2ec \ub4a4\uc778 4\uc6d4\uae4c\uc9c0 \ubbfc\uc8fc\ub178\ub3d9\ub2f9\uc744 \ud0c8\ub2f9\ud55c \ub2f9\uc6d0\uc740 \uba87 \uba85\uc778\uac00?, context: \uacb0\uad6d 2008\ub144 2\uc6d4 8\uc77c, \uc13c\ud2b8\ub7f4\uc2dc\ud2f0 9\uce35 \ub300\ud68c\uc758\uc2e4\uc5d0\uc11c \uc9c4\ud589\ub41c \ubbfc\uc8fc\ub178\ub3d9\ub2f9 \uc784\uc2dc\ub2f9\ub300\ud68c\uc5d0\uc11c \uc790\uc8fc\ud30c \ub300\uc758\uc6d0\ub4e4\uc758 \ubc18\ub300\ub85c \ud601\uc2e0\uc548\uc758 \ud575\uc2ec \ub0b4\uc6a9\uc740 \ubd80\uacb0\ub418\uc5c8\uc73c\uba70 \uc790\uc8fc\ud30c\uc758 \uc218\uc815\uc548\ub300\ub85c \ud1b5\uacfc\ub418\uc5c8\ub2e4. \ud601\uc2e0\uc548\uc774 \ubd80\uacb0\ub418\uc790 \uc2ec\uc0c1\uc815 \uc704\uc6d0\uc7a5 \ub4f1 \ube44\ub300\uc704\uc6d0 \uc804\uc6d0\uc774 \uc0ac\ud1f4\ud558\uc600\uace0, \ubbfc\uc8fc\ub178\ub3d9\ub2f9 \uc81c1\uae30\uc640 \uc81c2\uae30 \uc9c0\ub3c4\ubd80\ub97c \uad6c\uc131\ud558\uc600\ub358 \uc778\ubb3c\ub4e4 \uac00\uc6b4\ub370 \uae40\ud61c\uacbd, \uc8fc\ub300\ud658, \uae40\uc885\ucca0, \uc2ec\uc7ac\uc625, \ud64d\uc2b9\ud558, \uae40\uae30\uc218 \ub4f1\uc774 \ud0c8\ub2f9\ud558\uc600\uc73c\uba70, \ubbfc\uc8fc\ub178\ub3d9\ub2f9 \uc18c\uc18d \uad6d\ud68c\uc758\uc6d0 \uc2ec\uc0c1\uc815, \ub178\ud68c\ucc2c, \uc870\uc2b9\uc218, \ub2e8\ubcd1\ud638\ub3c4 \ud0c8\ub2f9\ud558\uc600\ub2e4. \uc784\uc2dc\ub2f9\ub300\ud68c \uc774\ud6c4 2008\ub144 4\uc6d4\uae4c\uc9c0 \ubbfc\uc8fc\ub178\ub3d9\ub2f9\uc744 \ud0c8\ub2f9\ud55c \uc0ac\ub78c\uc740 16,904\uba85\uc5d0 \uc774\ub974\ub800\uace0, 2008\ub144 1\uc6d4 \ub2f9\uc2dc 11\ub9cc\uc5ec \uba85\uc5d0 \uc774\ub974\ub358 \ubbfc\uc8fc\ub178\ub3d9\ub2f9\uc758 \ub2f9\uc6d0 \uc218\ub294 9\ub9cc 4\ucc9c\uc5ec \uba85\uc73c\ub85c \uc904\uc5c8\ub2e4. \ubbfc\uc8fc\ub178\ub3d9\ub2f9\uc744 \ud0c8\ub2f9\ud55c \uc0ac\ub78c\ub4e4 \uc911 \uc77c\ubd80\ub294 2008\ub144 2\uc6d4 21\uc77c '\uc9c4\ubcf4\uc2e0\ub2f9\uc5f0\ub300\ud68c\uc758' \uc900\ube44\uc704\uc6d0\ud68c\ub97c \uacb0\uc131\ud558\uc600\uc73c\uba70, 2008\ub144 3\uc6d4 \uc9c4\ubcf4\uc2e0\ub2f9 \ucc3d\ub2f9\uc774 \uacf5\uc2dd \uc120\uc5b8\ub418\uc5c8\uace0, \ubbfc\uc8fc\ub178\ub3d9\ub2f9\uc740 \ubd84\ub2f9\ub418\uc5c8\ub2e4."} {"question": "2014\ub144\uc5d0 IOC\ub294 \uba87 \ud68c\ucc28\uc758 \ucd1d\ud68c\ub97c \uc5f4\uc5c8\ub098?", "paragraph": "2014\ub144 12\uc6d4 7\uc77c \ud1a0\ub9c8\uc2a4 \ubc14\ud750 IOC \uc704\uc6d0\uc7a5\uc740 \u201c8\uc77c\ubd80\ud130 \uc774\ud2c0\uac04 \uc5f4\ub9ac\ub294 IOC \ucd1d\ud68c\uc5d0\uc11c \u2018\uc5b4\uc820\ub2e4 2020\u2019\uc774 \ud655\uc815\ub418\uba74 2018\ub144\uacfc 2020\ub144 \ub3d9\u00b7\ud558\uacc4 \uc62c\ub9bc\ud53d\uc744 \uce58\ub974\ub294 \ud55c\uad6d\uacfc \uc77c\ubcf8\uc774 \uc77c\ubd80 \uc885\ubaa9\uc744 \ubd84\uc0b0 \uac1c\ucd5c\ud560 \uc218 \uc788\ub2e4\u201d\uace0 \ub9d0\ud588\ub2e4. IOC(\uad6d\uc81c\uc62c\ub9bc\ud53d\uc704\uc6d0\ud68c)\ub294 2014\ub144 12\uc6d4 8\uc77c(\ud55c\uad6d \uc2dc\uac04) \uc81c 127\ud68c \ucd1d\ud68c\uc5d0\uc11c \uc0c8\ub85c\uc6b4 \uac1c\ud601\uc548\uc774 \ub2f4\uae34 'Olympics Agenda 2020(\uc5b4\uc820\ub2e4 2020)'\uc744 \ucc44\ud0dd\ud558\uc600\ub2e4. \uc0c8\ub85c\uc6b4 \uac1c\ud601\uc548\uc740 \uae30\uc874\uc758 \uc62c\ub9bc\ud53d \uac1c\ucd5c \ubc29\uc2dd\uacfc \ub2ec\ub9ac \uad6d\ub0b4 \uc5ec\ub7ec \ub3c4\uc2dc\ub4e4\uacfc \ubd84\uc0b0 \uac1c\ucd5c\uac00 \uac00\ub2a5\ud558\uba70, \ub2e4\ub978 \uad6d\uac00\uc758 \ub3c4\uc2dc\uc5d0\uc11c\ub3c4 \ubd84\uc0b0 \uac1c\ucd5c\ub97c \ud560 \uc218 \uc788\ub2e4\ub294 \uac83\uc774\ub2e4. \ub610 \uc2e0\ucd95 \uacbd\uae30\uc7a5\uc774 \uc544\ub2cc \uae30\uc874\uc758 \uc2dc\uc124\ubb3c\uc744 \ubcf4\uc644\ud558\uac70\ub098 \ucca0\uac70 \uc608\uc815\uc778 \uc2dc\uc124\ubb3c\ub4e4\uc5d0\uc11c\ub3c4 \uacbd\uae30\ub97c \uce58\ub97c \uc218 \uc788\ub2e4.", "answer": "127\ud68c", "sentence": "IOC(\uad6d\uc81c\uc62c\ub9bc\ud53d\uc704\uc6d0\ud68c)\ub294 2014\ub144 12\uc6d4 8\uc77c(\ud55c\uad6d \uc2dc\uac04) \uc81c 127\ud68c \ucd1d\ud68c\uc5d0\uc11c \uc0c8\ub85c\uc6b4 \uac1c\ud601\uc548\uc774 \ub2f4\uae34 'Olympics Agenda 2020(\uc5b4\uc820\ub2e4 2020)'\uc744 \ucc44\ud0dd\ud558\uc600\ub2e4.", "paragraph_sentence": "2014\ub144 12\uc6d4 7\uc77c \ud1a0\ub9c8\uc2a4 \ubc14\ud750 IOC \uc704\uc6d0\uc7a5\uc740 \u201c8\uc77c\ubd80\ud130 \uc774\ud2c0\uac04 \uc5f4\ub9ac\ub294 IOC \ucd1d\ud68c\uc5d0\uc11c \u2018\uc5b4\uc820\ub2e4 2020\u2019\uc774 \ud655\uc815\ub418\uba74 2018\ub144\uacfc 2020\ub144 \ub3d9\u00b7\ud558\uacc4 \uc62c\ub9bc\ud53d\uc744 \uce58\ub974\ub294 \ud55c\uad6d\uacfc \uc77c\ubcf8\uc774 \uc77c\ubd80 \uc885\ubaa9\uc744 \ubd84\uc0b0 \uac1c\ucd5c\ud560 \uc218 \uc788\ub2e4\u201d\uace0 \ub9d0\ud588\ub2e4. IOC(\uad6d\uc81c\uc62c\ub9bc\ud53d\uc704\uc6d0\ud68c)\ub294 2014\ub144 12\uc6d4 8\uc77c(\ud55c\uad6d \uc2dc\uac04) \uc81c 127\ud68c \ucd1d\ud68c\uc5d0\uc11c \uc0c8\ub85c\uc6b4 \uac1c\ud601\uc548\uc774 \ub2f4\uae34 'Olympics Agenda 2020(\uc5b4\uc820\ub2e4 2020)'\uc744 \ucc44\ud0dd\ud558\uc600\ub2e4. \uc0c8\ub85c\uc6b4 \uac1c\ud601\uc548\uc740 \uae30\uc874\uc758 \uc62c\ub9bc\ud53d \uac1c\ucd5c \ubc29\uc2dd\uacfc \ub2ec\ub9ac \uad6d\ub0b4 \uc5ec\ub7ec \ub3c4\uc2dc\ub4e4\uacfc \ubd84\uc0b0 \uac1c\ucd5c\uac00 \uac00\ub2a5\ud558\uba70, \ub2e4\ub978 \uad6d\uac00\uc758 \ub3c4\uc2dc\uc5d0\uc11c\ub3c4 \ubd84\uc0b0 \uac1c\ucd5c\ub97c \ud560 \uc218 \uc788\ub2e4\ub294 \uac83\uc774\ub2e4. \ub610 \uc2e0\ucd95 \uacbd\uae30\uc7a5\uc774 \uc544\ub2cc \uae30\uc874\uc758 \uc2dc\uc124\ubb3c\uc744 \ubcf4\uc644\ud558\uac70\ub098 \ucca0\uac70 \uc608\uc815\uc778 \uc2dc\uc124\ubb3c\ub4e4\uc5d0\uc11c\ub3c4 \uacbd\uae30\ub97c \uce58\ub97c \uc218 \uc788\ub2e4.", "paragraph_answer": "2014\ub144 12\uc6d4 7\uc77c \ud1a0\ub9c8\uc2a4 \ubc14\ud750 IOC \uc704\uc6d0\uc7a5\uc740 \u201c8\uc77c\ubd80\ud130 \uc774\ud2c0\uac04 \uc5f4\ub9ac\ub294 IOC \ucd1d\ud68c\uc5d0\uc11c \u2018\uc5b4\uc820\ub2e4 2020\u2019\uc774 \ud655\uc815\ub418\uba74 2018\ub144\uacfc 2020\ub144 \ub3d9\u00b7\ud558\uacc4 \uc62c\ub9bc\ud53d\uc744 \uce58\ub974\ub294 \ud55c\uad6d\uacfc \uc77c\ubcf8\uc774 \uc77c\ubd80 \uc885\ubaa9\uc744 \ubd84\uc0b0 \uac1c\ucd5c\ud560 \uc218 \uc788\ub2e4\u201d\uace0 \ub9d0\ud588\ub2e4. IOC(\uad6d\uc81c\uc62c\ub9bc\ud53d\uc704\uc6d0\ud68c)\ub294 2014\ub144 12\uc6d4 8\uc77c(\ud55c\uad6d \uc2dc\uac04) \uc81c 127\ud68c \ucd1d\ud68c\uc5d0\uc11c \uc0c8\ub85c\uc6b4 \uac1c\ud601\uc548\uc774 \ub2f4\uae34 'Olympics Agenda 2020(\uc5b4\uc820\ub2e4 2020)'\uc744 \ucc44\ud0dd\ud558\uc600\ub2e4. \uc0c8\ub85c\uc6b4 \uac1c\ud601\uc548\uc740 \uae30\uc874\uc758 \uc62c\ub9bc\ud53d \uac1c\ucd5c \ubc29\uc2dd\uacfc \ub2ec\ub9ac \uad6d\ub0b4 \uc5ec\ub7ec \ub3c4\uc2dc\ub4e4\uacfc \ubd84\uc0b0 \uac1c\ucd5c\uac00 \uac00\ub2a5\ud558\uba70, \ub2e4\ub978 \uad6d\uac00\uc758 \ub3c4\uc2dc\uc5d0\uc11c\ub3c4 \ubd84\uc0b0 \uac1c\ucd5c\ub97c \ud560 \uc218 \uc788\ub2e4\ub294 \uac83\uc774\ub2e4. \ub610 \uc2e0\ucd95 \uacbd\uae30\uc7a5\uc774 \uc544\ub2cc \uae30\uc874\uc758 \uc2dc\uc124\ubb3c\uc744 \ubcf4\uc644\ud558\uac70\ub098 \ucca0\uac70 \uc608\uc815\uc778 \uc2dc\uc124\ubb3c\ub4e4\uc5d0\uc11c\ub3c4 \uacbd\uae30\ub97c \uce58\ub97c \uc218 \uc788\ub2e4.", "sentence_answer": "IOC(\uad6d\uc81c\uc62c\ub9bc\ud53d\uc704\uc6d0\ud68c)\ub294 2014\ub144 12\uc6d4 8\uc77c(\ud55c\uad6d \uc2dc\uac04) \uc81c 127\ud68c \ucd1d\ud68c\uc5d0\uc11c \uc0c8\ub85c\uc6b4 \uac1c\ud601\uc548\uc774 \ub2f4\uae34 'Olympics Agenda 2020(\uc5b4\uc820\ub2e4 2020)'\uc744 \ucc44\ud0dd\ud558\uc600\ub2e4.", "paragraph_id": "6469594-2-0", "paragraph_question": "question: 2014\ub144\uc5d0 IOC\ub294 \uba87 \ud68c\ucc28\uc758 \ucd1d\ud68c\ub97c \uc5f4\uc5c8\ub098?, context: 2014\ub144 12\uc6d4 7\uc77c \ud1a0\ub9c8\uc2a4 \ubc14\ud750 IOC \uc704\uc6d0\uc7a5\uc740 \u201c8\uc77c\ubd80\ud130 \uc774\ud2c0\uac04 \uc5f4\ub9ac\ub294 IOC \ucd1d\ud68c\uc5d0\uc11c \u2018\uc5b4\uc820\ub2e4 2020\u2019\uc774 \ud655\uc815\ub418\uba74 2018\ub144\uacfc 2020\ub144 \ub3d9\u00b7\ud558\uacc4 \uc62c\ub9bc\ud53d\uc744 \uce58\ub974\ub294 \ud55c\uad6d\uacfc \uc77c\ubcf8\uc774 \uc77c\ubd80 \uc885\ubaa9\uc744 \ubd84\uc0b0 \uac1c\ucd5c\ud560 \uc218 \uc788\ub2e4\u201d\uace0 \ub9d0\ud588\ub2e4. IOC(\uad6d\uc81c\uc62c\ub9bc\ud53d\uc704\uc6d0\ud68c)\ub294 2014\ub144 12\uc6d4 8\uc77c(\ud55c\uad6d \uc2dc\uac04) \uc81c 127\ud68c \ucd1d\ud68c\uc5d0\uc11c \uc0c8\ub85c\uc6b4 \uac1c\ud601\uc548\uc774 \ub2f4\uae34 'Olympics Agenda 2020(\uc5b4\uc820\ub2e4 2020)'\uc744 \ucc44\ud0dd\ud558\uc600\ub2e4. \uc0c8\ub85c\uc6b4 \uac1c\ud601\uc548\uc740 \uae30\uc874\uc758 \uc62c\ub9bc\ud53d \uac1c\ucd5c \ubc29\uc2dd\uacfc \ub2ec\ub9ac \uad6d\ub0b4 \uc5ec\ub7ec \ub3c4\uc2dc\ub4e4\uacfc \ubd84\uc0b0 \uac1c\ucd5c\uac00 \uac00\ub2a5\ud558\uba70, \ub2e4\ub978 \uad6d\uac00\uc758 \ub3c4\uc2dc\uc5d0\uc11c\ub3c4 \ubd84\uc0b0 \uac1c\ucd5c\ub97c \ud560 \uc218 \uc788\ub2e4\ub294 \uac83\uc774\ub2e4. \ub610 \uc2e0\ucd95 \uacbd\uae30\uc7a5\uc774 \uc544\ub2cc \uae30\uc874\uc758 \uc2dc\uc124\ubb3c\uc744 \ubcf4\uc644\ud558\uac70\ub098 \ucca0\uac70 \uc608\uc815\uc778 \uc2dc\uc124\ubb3c\ub4e4\uc5d0\uc11c\ub3c4 \uacbd\uae30\ub97c \uce58\ub97c \uc218 \uc788\ub2e4."} {"question": "\uc0ac\ub4dc \ubc30\uce58\uc758 \uc77c\ucc28\uc801 \ubaa9\ud45c\ub294?", "paragraph": "\uc870\uc9c0\ud0c0\uc6b4 \ub300\ud559\uad50 \ube45\ud130 \ucc28 \uad50\uc218\ub294 \u201c\ud55c\uad6d\uc740 \ubbf8\uad6d\uacfc \uc804\ub7b5\uc801\uc73c\ub85c \ubc00\uc811\ud55c \uad00\uacc4\ub97c \ub9fa\uc5b4\uc57c \uc911\uad6d\uc758 \uc874\uc911\uc744 \ubc1b\uace0 \ub354 \ub098\uc740 \ub300\uc6b0\ub97c \ubc1b\uc744 \uc218 \uc788\ub2e4\u201d\uba70 \ud55c\ubbf8\ub3d9\ub9f9\uc5d0 \ubb38\uc81c\uac00 \uc0dd\uacbc\uc744 \uacbd\uc6b0 \ud55c\uad6d\uc774 \uc911\uad6d\uc73c\ub85c\ubd80\ud130 \uc874\uc911\ubc1b\uc9c0 \ubabb\ud558\uace0 \ub300\uc6b0\ubc1b\uc9c0 \ubabb\ud560 \uc218 \uc788\ub2e4\uace0 \ud588\ub2e4. 2017\ub144 \uae30\uc900\uc73c\ub85c 200\uac1c\uc758 \ubd81\ud55c \ubbf8\uc0ac\uc77c\uacfc 600\uac1c \uc774\uc0c1\uc758 \uc911\uad6d \ubbf8\uc0ac\uc77c\uc774 \ud55c\uad6d\uad70\uacfc \uc8fc\ud55c\ubbf8\uad70 \uae30\uc9c0\ub97c \ud5a5\ud574 \uc870\uc900\ub418\uc5b4 \uc788\ub2e4. \uc8fc\ud55c\ubbf8\uad70\uc774 \ud55c\ubc18\ub3c4\uc5d0 \uc8fc\ub454\ud568\uc73c\ub85c\uc11c \ubbf8\uad6d\uc774 \uc5bb\uace0 \uc788\ub294 \uac00\uc7a5 \uc8fc\ub41c \uc804\ub7b5\uc801 \uac00\uce58\ub294 \uc911\uad6d\uc744 \uacac\uc81c\ud560 \uc218 \uc788\ub2e4\ub294 \uac83\uc774\uace0 \uc0ac\ub4dc \ubc30\uce58\uac00 \ucca0\ud68c\ub418\uba74 \uc624\ud788\ub824 \ubc18\ub300\ub85c \uc8fc\ud55c\ubbf8\uad70\uc774 600\uac1c \uc774\uc0c1\uc758 \uc911\uad6d \ubbf8\uc0ac\uc77c\ub85c \uc778\ud574 \uc704\ud611\uc744 \ubc1b\uac8c \ub418\uae30 \ub54c\ubb38\uc5d0 \ubbf8\uad6d\uc740 \ud55c\ubc18\ub3c4\ub97c \ub354 \uc774\uc0c1 \uc804\ub7b5\uc801 \uc694\ucda9\uc9c0\ub85c \ud65c\uc6a9\ud560 \uc218 \uc5c6\uac8c \ub41c\ub2e4. \uc0ac\ub4dc\ub294 \uc8fc\ud55c\ubbf8\uad70\uc744 \ubcf4\ud638\ud558\ub294 \uac83\uc774 \uc77c\ucc28\uc801 \ubaa9\ud45c\uc774\uace0 \ud55c\ubc18\ub3c4\uc5d0 \uc8fc\ub454\ud558\uace0 \uc788\ub294 \uc8fc\ud55c\ubbf8\uad70\uc774 \ubd81\ud55c\uacfc \uc911\uad6d\uc758 \ubbf8\uc0ac\uc77c\ub85c \uc778\ud574 \uc704\ud611\uc744 \ubc1b\uace0 \uc788\ub294 \uc0c1\ud669\uc5d0\uc11c\ub294 \ud55c\ubc18\ub3c4\ub97c \ub354 \uc774\uc0c1 \uc804\ub7b5\uc801 \uc694\ucda9\uc9c0\ub85c \ud65c\uc6a9\ud560 \uc218 \uc5c6\uc744 \ubfd0\ub9cc \uc544\ub2c8\ub77c \uc624\ud788\ub824 \ud55c\ubc18\ub3c4\uc5d0 \uc8fc\ub454\ud558\uace0 \uc788\ub294 \uc8fc\ud55c\ubbf8\uad70\uc774 \uc911\uad6d \ubbf8\uc0ac\uc77c\uc758 \uc778\uc9c8\ub85c \uc7a1\ud600 \uc788\ub294 \ud615\uad6d\uc774 \ub418\uc5b4 \ubc84\ub9b0\ub2e4. \ud55c\ubc18\ub3c4\uc5d0 \uc8fc\ub454\ud558\uace0 \uc788\ub294 \uc8fc\ud55c\ubbf8\uad70\uc744 \ubcf4\ud638\ud558\uae30 \uc704\ud574 \ubc30\uce58\ub41c \uc0ac\ub4dc\uac00 \ucca0\ud68c\ub41c\ub2e4\uba74 \uc8fc\ud55c\ubbf8\uad70\uc740 \uc911\uad6d \ubbf8\uc0ac\uc77c\uc5d0 \ubb34\ubc29\ube44 \uc0c1\ud0dc\ub85c \ub178\ucd9c\ub418\uc5b4 \uc704\ud611\uc744 \ubc1b\uac8c \ub418\uace0 \uc774\ub85c \uc778\ud574 \uc8fc\ud55c\ubbf8\uad70\uc774 \ud55c\ubc18\ub3c4\uc5d0 \uc8fc\ub454\ud568\uc73c\ub85c\uc11c \ubbf8\uad6d\uc774 \uc5bb\uc744 \uc218 \uc788\ub294 \uc804\ub7b5\uc801 \uac00\uce58\uac00 \uc18c\uba78\ub418\uae30 \ub54c\ubb38\uc5d0 \ubbf8\uad6d\uc815\ubd80\uac00 \uc8fc\ud55c\ubbf8\uad70\uc744 \ucca0\uc218\uc2dc\ud0ac \uc218\ubc16\uc5d0 \uc5c6\uac8c \ub41c\ub2e4. \uc911\uad6d\uc740 \ubbfc\uc8fc\ud601\uba85\uc744 \uac70\uce58\uc9c0 \uc54a\uc740 \uacf5\uc0b0\ub2f9 1\ub2f9\uccb4\uc81c \uacf5\uc0b0\uc8fc\uc758 \uad6d\uac00\uc774\uba70 \uc911\uad6d\uc740 \"\ubbf8\uad6d\ub9cc \uc5c6\uc5c8\uc73c\uba74 \ud55c\uad6d\uc740 \uc9c4\uc791\uc5d0 \uc190\ubd24\uc744 \ub098\ub77c\"\ub77c\uace0 \ud558\uba70 \ud55c\uad6d\uc744 \uad70\uc0ac\uc801\uc73c\ub85c \uc704\ud611\ud558\ub294 \ubc1c\uc5b8\uc744 \ud558\uc600\uace0 \ub9cc\uc57d\uc5d0 \uc8fc\ud55c\ubbf8\uad70\uc774 \ud55c\ubc18\ub3c4\uc5d0\uc11c \ucca0\uc218\ud55c\ub2e4\uba74 \uc911\uad6d\uc774 \uc774 \ubc1c\uc5b8\uc744 \uc2e4\ud589\uc73c\ub85c \uc62e\uae38 \uac00\ub2a5\uc131\uc774 \uc788\ub2e4. \uc911\uad6d\uc740 2025\ub144\uae4c\uc9c0 \uc81c1\ub3c4\ub828\uc120\uc744 \uc810\ub839\ud558\uaca0\ub2e4\ub294 \uacc4\ud68d\uc744 \uac16\uace0 \uc788\ub294\ub370, \uc81c1\ub3c4\ub828\uc120\uc5d0\ub294 \ud55c\ubc18\ub3c4\uac00 \ud3ec\ud568\ub41c\ub2e4.", "answer": "\uc8fc\ud55c\ubbf8\uad70\uc744 \ubcf4\ud638", "sentence": "\uc0ac\ub4dc\ub294 \uc8fc\ud55c\ubbf8\uad70\uc744 \ubcf4\ud638 \ud558\ub294 \uac83\uc774 \uc77c\ucc28\uc801 \ubaa9\ud45c\uc774\uace0 \ud55c\ubc18\ub3c4\uc5d0 \uc8fc\ub454\ud558\uace0 \uc788\ub294 \uc8fc\ud55c\ubbf8\uad70\uc774 \ubd81\ud55c\uacfc \uc911\uad6d\uc758 \ubbf8\uc0ac\uc77c\ub85c \uc778\ud574 \uc704\ud611\uc744 \ubc1b\uace0 \uc788\ub294 \uc0c1\ud669\uc5d0\uc11c\ub294 \ud55c\ubc18\ub3c4\ub97c \ub354 \uc774\uc0c1 \uc804\ub7b5\uc801 \uc694\ucda9\uc9c0\ub85c \ud65c\uc6a9\ud560 \uc218 \uc5c6\uc744 \ubfd0\ub9cc \uc544\ub2c8\ub77c \uc624\ud788\ub824 \ud55c\ubc18\ub3c4\uc5d0 \uc8fc\ub454\ud558\uace0 \uc788\ub294 \uc8fc\ud55c\ubbf8\uad70\uc774 \uc911\uad6d \ubbf8\uc0ac\uc77c\uc758 \uc778\uc9c8\ub85c \uc7a1\ud600 \uc788\ub294 \ud615\uad6d\uc774 \ub418\uc5b4 \ubc84\ub9b0\ub2e4.", "paragraph_sentence": "\uc870\uc9c0\ud0c0\uc6b4 \ub300\ud559\uad50 \ube45\ud130 \ucc28 \uad50\uc218\ub294 \u201c\ud55c\uad6d\uc740 \ubbf8\uad6d\uacfc \uc804\ub7b5\uc801\uc73c\ub85c \ubc00\uc811\ud55c \uad00\uacc4\ub97c \ub9fa\uc5b4\uc57c \uc911\uad6d\uc758 \uc874\uc911\uc744 \ubc1b\uace0 \ub354 \ub098\uc740 \ub300\uc6b0\ub97c \ubc1b\uc744 \uc218 \uc788\ub2e4\u201d\uba70 \ud55c\ubbf8\ub3d9\ub9f9\uc5d0 \ubb38\uc81c\uac00 \uc0dd\uacbc\uc744 \uacbd\uc6b0 \ud55c\uad6d\uc774 \uc911\uad6d\uc73c\ub85c\ubd80\ud130 \uc874\uc911\ubc1b\uc9c0 \ubabb\ud558\uace0 \ub300\uc6b0\ubc1b\uc9c0 \ubabb\ud560 \uc218 \uc788\ub2e4\uace0 \ud588\ub2e4. 2017\ub144 \uae30\uc900\uc73c\ub85c 200\uac1c\uc758 \ubd81\ud55c \ubbf8\uc0ac\uc77c\uacfc 600\uac1c \uc774\uc0c1\uc758 \uc911\uad6d \ubbf8\uc0ac\uc77c\uc774 \ud55c\uad6d\uad70\uacfc \uc8fc\ud55c\ubbf8\uad70 \uae30\uc9c0\ub97c \ud5a5\ud574 \uc870\uc900\ub418\uc5b4 \uc788\ub2e4. \uc8fc\ud55c\ubbf8\uad70\uc774 \ud55c\ubc18\ub3c4\uc5d0 \uc8fc\ub454\ud568\uc73c\ub85c\uc11c \ubbf8\uad6d\uc774 \uc5bb\uace0 \uc788\ub294 \uac00\uc7a5 \uc8fc\ub41c \uc804\ub7b5\uc801 \uac00\uce58\ub294 \uc911\uad6d\uc744 \uacac\uc81c\ud560 \uc218 \uc788\ub2e4\ub294 \uac83\uc774\uace0 \uc0ac\ub4dc \ubc30\uce58\uac00 \ucca0\ud68c\ub418\uba74 \uc624\ud788\ub824 \ubc18\ub300\ub85c \uc8fc\ud55c\ubbf8\uad70\uc774 600\uac1c \uc774\uc0c1\uc758 \uc911\uad6d \ubbf8\uc0ac\uc77c\ub85c \uc778\ud574 \uc704\ud611\uc744 \ubc1b\uac8c \ub418\uae30 \ub54c\ubb38\uc5d0 \ubbf8\uad6d\uc740 \ud55c\ubc18\ub3c4\ub97c \ub354 \uc774\uc0c1 \uc804\ub7b5\uc801 \uc694\ucda9\uc9c0\ub85c \ud65c\uc6a9\ud560 \uc218 \uc5c6\uac8c \ub41c\ub2e4. \uc0ac\ub4dc\ub294 \uc8fc\ud55c\ubbf8\uad70\uc744 \ubcf4\ud638 \ud558\ub294 \uac83\uc774 \uc77c\ucc28\uc801 \ubaa9\ud45c\uc774\uace0 \ud55c\ubc18\ub3c4\uc5d0 \uc8fc\ub454\ud558\uace0 \uc788\ub294 \uc8fc\ud55c\ubbf8\uad70\uc774 \ubd81\ud55c\uacfc \uc911\uad6d\uc758 \ubbf8\uc0ac\uc77c\ub85c \uc778\ud574 \uc704\ud611\uc744 \ubc1b\uace0 \uc788\ub294 \uc0c1\ud669\uc5d0\uc11c\ub294 \ud55c\ubc18\ub3c4\ub97c \ub354 \uc774\uc0c1 \uc804\ub7b5\uc801 \uc694\ucda9\uc9c0\ub85c \ud65c\uc6a9\ud560 \uc218 \uc5c6\uc744 \ubfd0\ub9cc \uc544\ub2c8\ub77c \uc624\ud788\ub824 \ud55c\ubc18\ub3c4\uc5d0 \uc8fc\ub454\ud558\uace0 \uc788\ub294 \uc8fc\ud55c\ubbf8\uad70\uc774 \uc911\uad6d \ubbf8\uc0ac\uc77c\uc758 \uc778\uc9c8\ub85c \uc7a1\ud600 \uc788\ub294 \ud615\uad6d\uc774 \ub418\uc5b4 \ubc84\ub9b0\ub2e4. \ud55c\ubc18\ub3c4\uc5d0 \uc8fc\ub454\ud558\uace0 \uc788\ub294 \uc8fc\ud55c\ubbf8\uad70\uc744 \ubcf4\ud638\ud558\uae30 \uc704\ud574 \ubc30\uce58\ub41c \uc0ac\ub4dc\uac00 \ucca0\ud68c\ub41c\ub2e4\uba74 \uc8fc\ud55c\ubbf8\uad70\uc740 \uc911\uad6d \ubbf8\uc0ac\uc77c\uc5d0 \ubb34\ubc29\ube44 \uc0c1\ud0dc\ub85c \ub178\ucd9c\ub418\uc5b4 \uc704\ud611\uc744 \ubc1b\uac8c \ub418\uace0 \uc774\ub85c \uc778\ud574 \uc8fc\ud55c\ubbf8\uad70\uc774 \ud55c\ubc18\ub3c4\uc5d0 \uc8fc\ub454\ud568\uc73c\ub85c\uc11c \ubbf8\uad6d\uc774 \uc5bb\uc744 \uc218 \uc788\ub294 \uc804\ub7b5\uc801 \uac00\uce58\uac00 \uc18c\uba78\ub418\uae30 \ub54c\ubb38\uc5d0 \ubbf8\uad6d\uc815\ubd80\uac00 \uc8fc\ud55c\ubbf8\uad70\uc744 \ucca0\uc218\uc2dc\ud0ac \uc218\ubc16\uc5d0 \uc5c6\uac8c \ub41c\ub2e4. \uc911\uad6d\uc740 \ubbfc\uc8fc\ud601\uba85\uc744 \uac70\uce58\uc9c0 \uc54a\uc740 \uacf5\uc0b0\ub2f9 1\ub2f9\uccb4\uc81c \uacf5\uc0b0\uc8fc\uc758 \uad6d\uac00\uc774\uba70 \uc911\uad6d\uc740 \"\ubbf8\uad6d\ub9cc \uc5c6\uc5c8\uc73c\uba74 \ud55c\uad6d\uc740 \uc9c4\uc791\uc5d0 \uc190\ubd24\uc744 \ub098\ub77c\"\ub77c\uace0 \ud558\uba70 \ud55c\uad6d\uc744 \uad70\uc0ac\uc801\uc73c\ub85c \uc704\ud611\ud558\ub294 \ubc1c\uc5b8\uc744 \ud558\uc600\uace0 \ub9cc\uc57d\uc5d0 \uc8fc\ud55c\ubbf8\uad70\uc774 \ud55c\ubc18\ub3c4\uc5d0\uc11c \ucca0\uc218\ud55c\ub2e4\uba74 \uc911\uad6d\uc774 \uc774 \ubc1c\uc5b8\uc744 \uc2e4\ud589\uc73c\ub85c \uc62e\uae38 \uac00\ub2a5\uc131\uc774 \uc788\ub2e4. \uc911\uad6d\uc740 2025\ub144\uae4c\uc9c0 \uc81c1\ub3c4\ub828\uc120\uc744 \uc810\ub839\ud558\uaca0\ub2e4\ub294 \uacc4\ud68d\uc744 \uac16\uace0 \uc788\ub294\ub370, \uc81c1\ub3c4\ub828\uc120\uc5d0\ub294 \ud55c\ubc18\ub3c4\uac00 \ud3ec\ud568\ub41c\ub2e4.", "paragraph_answer": "\uc870\uc9c0\ud0c0\uc6b4 \ub300\ud559\uad50 \ube45\ud130 \ucc28 \uad50\uc218\ub294 \u201c\ud55c\uad6d\uc740 \ubbf8\uad6d\uacfc \uc804\ub7b5\uc801\uc73c\ub85c \ubc00\uc811\ud55c \uad00\uacc4\ub97c \ub9fa\uc5b4\uc57c \uc911\uad6d\uc758 \uc874\uc911\uc744 \ubc1b\uace0 \ub354 \ub098\uc740 \ub300\uc6b0\ub97c \ubc1b\uc744 \uc218 \uc788\ub2e4\u201d\uba70 \ud55c\ubbf8\ub3d9\ub9f9\uc5d0 \ubb38\uc81c\uac00 \uc0dd\uacbc\uc744 \uacbd\uc6b0 \ud55c\uad6d\uc774 \uc911\uad6d\uc73c\ub85c\ubd80\ud130 \uc874\uc911\ubc1b\uc9c0 \ubabb\ud558\uace0 \ub300\uc6b0\ubc1b\uc9c0 \ubabb\ud560 \uc218 \uc788\ub2e4\uace0 \ud588\ub2e4. 2017\ub144 \uae30\uc900\uc73c\ub85c 200\uac1c\uc758 \ubd81\ud55c \ubbf8\uc0ac\uc77c\uacfc 600\uac1c \uc774\uc0c1\uc758 \uc911\uad6d \ubbf8\uc0ac\uc77c\uc774 \ud55c\uad6d\uad70\uacfc \uc8fc\ud55c\ubbf8\uad70 \uae30\uc9c0\ub97c \ud5a5\ud574 \uc870\uc900\ub418\uc5b4 \uc788\ub2e4. \uc8fc\ud55c\ubbf8\uad70\uc774 \ud55c\ubc18\ub3c4\uc5d0 \uc8fc\ub454\ud568\uc73c\ub85c\uc11c \ubbf8\uad6d\uc774 \uc5bb\uace0 \uc788\ub294 \uac00\uc7a5 \uc8fc\ub41c \uc804\ub7b5\uc801 \uac00\uce58\ub294 \uc911\uad6d\uc744 \uacac\uc81c\ud560 \uc218 \uc788\ub2e4\ub294 \uac83\uc774\uace0 \uc0ac\ub4dc \ubc30\uce58\uac00 \ucca0\ud68c\ub418\uba74 \uc624\ud788\ub824 \ubc18\ub300\ub85c \uc8fc\ud55c\ubbf8\uad70\uc774 600\uac1c \uc774\uc0c1\uc758 \uc911\uad6d \ubbf8\uc0ac\uc77c\ub85c \uc778\ud574 \uc704\ud611\uc744 \ubc1b\uac8c \ub418\uae30 \ub54c\ubb38\uc5d0 \ubbf8\uad6d\uc740 \ud55c\ubc18\ub3c4\ub97c \ub354 \uc774\uc0c1 \uc804\ub7b5\uc801 \uc694\ucda9\uc9c0\ub85c \ud65c\uc6a9\ud560 \uc218 \uc5c6\uac8c \ub41c\ub2e4. \uc0ac\ub4dc\ub294 \uc8fc\ud55c\ubbf8\uad70\uc744 \ubcf4\ud638 \ud558\ub294 \uac83\uc774 \uc77c\ucc28\uc801 \ubaa9\ud45c\uc774\uace0 \ud55c\ubc18\ub3c4\uc5d0 \uc8fc\ub454\ud558\uace0 \uc788\ub294 \uc8fc\ud55c\ubbf8\uad70\uc774 \ubd81\ud55c\uacfc \uc911\uad6d\uc758 \ubbf8\uc0ac\uc77c\ub85c \uc778\ud574 \uc704\ud611\uc744 \ubc1b\uace0 \uc788\ub294 \uc0c1\ud669\uc5d0\uc11c\ub294 \ud55c\ubc18\ub3c4\ub97c \ub354 \uc774\uc0c1 \uc804\ub7b5\uc801 \uc694\ucda9\uc9c0\ub85c \ud65c\uc6a9\ud560 \uc218 \uc5c6\uc744 \ubfd0\ub9cc \uc544\ub2c8\ub77c \uc624\ud788\ub824 \ud55c\ubc18\ub3c4\uc5d0 \uc8fc\ub454\ud558\uace0 \uc788\ub294 \uc8fc\ud55c\ubbf8\uad70\uc774 \uc911\uad6d \ubbf8\uc0ac\uc77c\uc758 \uc778\uc9c8\ub85c \uc7a1\ud600 \uc788\ub294 \ud615\uad6d\uc774 \ub418\uc5b4 \ubc84\ub9b0\ub2e4. \ud55c\ubc18\ub3c4\uc5d0 \uc8fc\ub454\ud558\uace0 \uc788\ub294 \uc8fc\ud55c\ubbf8\uad70\uc744 \ubcf4\ud638\ud558\uae30 \uc704\ud574 \ubc30\uce58\ub41c \uc0ac\ub4dc\uac00 \ucca0\ud68c\ub41c\ub2e4\uba74 \uc8fc\ud55c\ubbf8\uad70\uc740 \uc911\uad6d \ubbf8\uc0ac\uc77c\uc5d0 \ubb34\ubc29\ube44 \uc0c1\ud0dc\ub85c \ub178\ucd9c\ub418\uc5b4 \uc704\ud611\uc744 \ubc1b\uac8c \ub418\uace0 \uc774\ub85c \uc778\ud574 \uc8fc\ud55c\ubbf8\uad70\uc774 \ud55c\ubc18\ub3c4\uc5d0 \uc8fc\ub454\ud568\uc73c\ub85c\uc11c \ubbf8\uad6d\uc774 \uc5bb\uc744 \uc218 \uc788\ub294 \uc804\ub7b5\uc801 \uac00\uce58\uac00 \uc18c\uba78\ub418\uae30 \ub54c\ubb38\uc5d0 \ubbf8\uad6d\uc815\ubd80\uac00 \uc8fc\ud55c\ubbf8\uad70\uc744 \ucca0\uc218\uc2dc\ud0ac \uc218\ubc16\uc5d0 \uc5c6\uac8c \ub41c\ub2e4. \uc911\uad6d\uc740 \ubbfc\uc8fc\ud601\uba85\uc744 \uac70\uce58\uc9c0 \uc54a\uc740 \uacf5\uc0b0\ub2f9 1\ub2f9\uccb4\uc81c \uacf5\uc0b0\uc8fc\uc758 \uad6d\uac00\uc774\uba70 \uc911\uad6d\uc740 \"\ubbf8\uad6d\ub9cc \uc5c6\uc5c8\uc73c\uba74 \ud55c\uad6d\uc740 \uc9c4\uc791\uc5d0 \uc190\ubd24\uc744 \ub098\ub77c\"\ub77c\uace0 \ud558\uba70 \ud55c\uad6d\uc744 \uad70\uc0ac\uc801\uc73c\ub85c \uc704\ud611\ud558\ub294 \ubc1c\uc5b8\uc744 \ud558\uc600\uace0 \ub9cc\uc57d\uc5d0 \uc8fc\ud55c\ubbf8\uad70\uc774 \ud55c\ubc18\ub3c4\uc5d0\uc11c \ucca0\uc218\ud55c\ub2e4\uba74 \uc911\uad6d\uc774 \uc774 \ubc1c\uc5b8\uc744 \uc2e4\ud589\uc73c\ub85c \uc62e\uae38 \uac00\ub2a5\uc131\uc774 \uc788\ub2e4. \uc911\uad6d\uc740 2025\ub144\uae4c\uc9c0 \uc81c1\ub3c4\ub828\uc120\uc744 \uc810\ub839\ud558\uaca0\ub2e4\ub294 \uacc4\ud68d\uc744 \uac16\uace0 \uc788\ub294\ub370, \uc81c1\ub3c4\ub828\uc120\uc5d0\ub294 \ud55c\ubc18\ub3c4\uac00 \ud3ec\ud568\ub41c\ub2e4.", "sentence_answer": "\uc0ac\ub4dc\ub294 \uc8fc\ud55c\ubbf8\uad70\uc744 \ubcf4\ud638 \ud558\ub294 \uac83\uc774 \uc77c\ucc28\uc801 \ubaa9\ud45c\uc774\uace0 \ud55c\ubc18\ub3c4\uc5d0 \uc8fc\ub454\ud558\uace0 \uc788\ub294 \uc8fc\ud55c\ubbf8\uad70\uc774 \ubd81\ud55c\uacfc \uc911\uad6d\uc758 \ubbf8\uc0ac\uc77c\ub85c \uc778\ud574 \uc704\ud611\uc744 \ubc1b\uace0 \uc788\ub294 \uc0c1\ud669\uc5d0\uc11c\ub294 \ud55c\ubc18\ub3c4\ub97c \ub354 \uc774\uc0c1 \uc804\ub7b5\uc801 \uc694\ucda9\uc9c0\ub85c \ud65c\uc6a9\ud560 \uc218 \uc5c6\uc744 \ubfd0\ub9cc \uc544\ub2c8\ub77c \uc624\ud788\ub824 \ud55c\ubc18\ub3c4\uc5d0 \uc8fc\ub454\ud558\uace0 \uc788\ub294 \uc8fc\ud55c\ubbf8\uad70\uc774 \uc911\uad6d \ubbf8\uc0ac\uc77c\uc758 \uc778\uc9c8\ub85c \uc7a1\ud600 \uc788\ub294 \ud615\uad6d\uc774 \ub418\uc5b4 \ubc84\ub9b0\ub2e4.", "paragraph_id": "6546295-24-1", "paragraph_question": "question: \uc0ac\ub4dc \ubc30\uce58\uc758 \uc77c\ucc28\uc801 \ubaa9\ud45c\ub294?, context: \uc870\uc9c0\ud0c0\uc6b4 \ub300\ud559\uad50 \ube45\ud130 \ucc28 \uad50\uc218\ub294 \u201c\ud55c\uad6d\uc740 \ubbf8\uad6d\uacfc \uc804\ub7b5\uc801\uc73c\ub85c \ubc00\uc811\ud55c \uad00\uacc4\ub97c \ub9fa\uc5b4\uc57c \uc911\uad6d\uc758 \uc874\uc911\uc744 \ubc1b\uace0 \ub354 \ub098\uc740 \ub300\uc6b0\ub97c \ubc1b\uc744 \uc218 \uc788\ub2e4\u201d\uba70 \ud55c\ubbf8\ub3d9\ub9f9\uc5d0 \ubb38\uc81c\uac00 \uc0dd\uacbc\uc744 \uacbd\uc6b0 \ud55c\uad6d\uc774 \uc911\uad6d\uc73c\ub85c\ubd80\ud130 \uc874\uc911\ubc1b\uc9c0 \ubabb\ud558\uace0 \ub300\uc6b0\ubc1b\uc9c0 \ubabb\ud560 \uc218 \uc788\ub2e4\uace0 \ud588\ub2e4. 2017\ub144 \uae30\uc900\uc73c\ub85c 200\uac1c\uc758 \ubd81\ud55c \ubbf8\uc0ac\uc77c\uacfc 600\uac1c \uc774\uc0c1\uc758 \uc911\uad6d \ubbf8\uc0ac\uc77c\uc774 \ud55c\uad6d\uad70\uacfc \uc8fc\ud55c\ubbf8\uad70 \uae30\uc9c0\ub97c \ud5a5\ud574 \uc870\uc900\ub418\uc5b4 \uc788\ub2e4. \uc8fc\ud55c\ubbf8\uad70\uc774 \ud55c\ubc18\ub3c4\uc5d0 \uc8fc\ub454\ud568\uc73c\ub85c\uc11c \ubbf8\uad6d\uc774 \uc5bb\uace0 \uc788\ub294 \uac00\uc7a5 \uc8fc\ub41c \uc804\ub7b5\uc801 \uac00\uce58\ub294 \uc911\uad6d\uc744 \uacac\uc81c\ud560 \uc218 \uc788\ub2e4\ub294 \uac83\uc774\uace0 \uc0ac\ub4dc \ubc30\uce58\uac00 \ucca0\ud68c\ub418\uba74 \uc624\ud788\ub824 \ubc18\ub300\ub85c \uc8fc\ud55c\ubbf8\uad70\uc774 600\uac1c \uc774\uc0c1\uc758 \uc911\uad6d \ubbf8\uc0ac\uc77c\ub85c \uc778\ud574 \uc704\ud611\uc744 \ubc1b\uac8c \ub418\uae30 \ub54c\ubb38\uc5d0 \ubbf8\uad6d\uc740 \ud55c\ubc18\ub3c4\ub97c \ub354 \uc774\uc0c1 \uc804\ub7b5\uc801 \uc694\ucda9\uc9c0\ub85c \ud65c\uc6a9\ud560 \uc218 \uc5c6\uac8c \ub41c\ub2e4. \uc0ac\ub4dc\ub294 \uc8fc\ud55c\ubbf8\uad70\uc744 \ubcf4\ud638\ud558\ub294 \uac83\uc774 \uc77c\ucc28\uc801 \ubaa9\ud45c\uc774\uace0 \ud55c\ubc18\ub3c4\uc5d0 \uc8fc\ub454\ud558\uace0 \uc788\ub294 \uc8fc\ud55c\ubbf8\uad70\uc774 \ubd81\ud55c\uacfc \uc911\uad6d\uc758 \ubbf8\uc0ac\uc77c\ub85c \uc778\ud574 \uc704\ud611\uc744 \ubc1b\uace0 \uc788\ub294 \uc0c1\ud669\uc5d0\uc11c\ub294 \ud55c\ubc18\ub3c4\ub97c \ub354 \uc774\uc0c1 \uc804\ub7b5\uc801 \uc694\ucda9\uc9c0\ub85c \ud65c\uc6a9\ud560 \uc218 \uc5c6\uc744 \ubfd0\ub9cc \uc544\ub2c8\ub77c \uc624\ud788\ub824 \ud55c\ubc18\ub3c4\uc5d0 \uc8fc\ub454\ud558\uace0 \uc788\ub294 \uc8fc\ud55c\ubbf8\uad70\uc774 \uc911\uad6d \ubbf8\uc0ac\uc77c\uc758 \uc778\uc9c8\ub85c \uc7a1\ud600 \uc788\ub294 \ud615\uad6d\uc774 \ub418\uc5b4 \ubc84\ub9b0\ub2e4. \ud55c\ubc18\ub3c4\uc5d0 \uc8fc\ub454\ud558\uace0 \uc788\ub294 \uc8fc\ud55c\ubbf8\uad70\uc744 \ubcf4\ud638\ud558\uae30 \uc704\ud574 \ubc30\uce58\ub41c \uc0ac\ub4dc\uac00 \ucca0\ud68c\ub41c\ub2e4\uba74 \uc8fc\ud55c\ubbf8\uad70\uc740 \uc911\uad6d \ubbf8\uc0ac\uc77c\uc5d0 \ubb34\ubc29\ube44 \uc0c1\ud0dc\ub85c \ub178\ucd9c\ub418\uc5b4 \uc704\ud611\uc744 \ubc1b\uac8c \ub418\uace0 \uc774\ub85c \uc778\ud574 \uc8fc\ud55c\ubbf8\uad70\uc774 \ud55c\ubc18\ub3c4\uc5d0 \uc8fc\ub454\ud568\uc73c\ub85c\uc11c \ubbf8\uad6d\uc774 \uc5bb\uc744 \uc218 \uc788\ub294 \uc804\ub7b5\uc801 \uac00\uce58\uac00 \uc18c\uba78\ub418\uae30 \ub54c\ubb38\uc5d0 \ubbf8\uad6d\uc815\ubd80\uac00 \uc8fc\ud55c\ubbf8\uad70\uc744 \ucca0\uc218\uc2dc\ud0ac \uc218\ubc16\uc5d0 \uc5c6\uac8c \ub41c\ub2e4. \uc911\uad6d\uc740 \ubbfc\uc8fc\ud601\uba85\uc744 \uac70\uce58\uc9c0 \uc54a\uc740 \uacf5\uc0b0\ub2f9 1\ub2f9\uccb4\uc81c \uacf5\uc0b0\uc8fc\uc758 \uad6d\uac00\uc774\uba70 \uc911\uad6d\uc740 \"\ubbf8\uad6d\ub9cc \uc5c6\uc5c8\uc73c\uba74 \ud55c\uad6d\uc740 \uc9c4\uc791\uc5d0 \uc190\ubd24\uc744 \ub098\ub77c\"\ub77c\uace0 \ud558\uba70 \ud55c\uad6d\uc744 \uad70\uc0ac\uc801\uc73c\ub85c \uc704\ud611\ud558\ub294 \ubc1c\uc5b8\uc744 \ud558\uc600\uace0 \ub9cc\uc57d\uc5d0 \uc8fc\ud55c\ubbf8\uad70\uc774 \ud55c\ubc18\ub3c4\uc5d0\uc11c \ucca0\uc218\ud55c\ub2e4\uba74 \uc911\uad6d\uc774 \uc774 \ubc1c\uc5b8\uc744 \uc2e4\ud589\uc73c\ub85c \uc62e\uae38 \uac00\ub2a5\uc131\uc774 \uc788\ub2e4. \uc911\uad6d\uc740 2025\ub144\uae4c\uc9c0 \uc81c1\ub3c4\ub828\uc120\uc744 \uc810\ub839\ud558\uaca0\ub2e4\ub294 \uacc4\ud68d\uc744 \uac16\uace0 \uc788\ub294\ub370, \uc81c1\ub3c4\ub828\uc120\uc5d0\ub294 \ud55c\ubc18\ub3c4\uac00 \ud3ec\ud568\ub41c\ub2e4."} {"question": "\ub3cc\uc5f0\ubcc0\uc774\uc758 \ub300\ud45c \uc0ac\ub840\ub294?", "paragraph": "\ub3cc\uc5f0\ubcc0\uc774\uc758 \ub300\ud45c\uc801\uc778 \uc0ac\ub840\ub85c\ub294 \uc720\uc804\uc790 \uc911\ubcf5\uc774 \uc788\ub2e4. \ub3d9\ubb3c\uc758 \uacbd\uc6b0 \ub9e4 \ucc9c \ub144\ub2f9 \ud55c \ubc88 \uaf34\ub85c \ubc1c\uc0dd\ud558\ub294 \uc720\uc804\uc790 \uc911\ubcf5\uc740 \uc5fc\uc0c9\uccb4\uc758 \uc77c\uc815 \uad6c\uac04\uc774 \uc2e4\uc218\ub85c \ubc18\ubcf5\ud558\uc5ec \ubcf5\uc81c\ub418\ub294 \uac83\uc774\ub2e4. \ub300\ubd80\ubd84\uc758 \uc720\uc804\uc790\ub4e4\uc740 \uc0c1\ub3d9\uc131\uc744 \uac16\ub294 \uc720\uc804\uc790\uc871\uc5d0 \uc790\ub9ac\uc7a1\uace0 \uc788\ub2e4. \uc6d0\ubcf8 DNA\ub97c \ubcf5\uc81c\ud558\ub294 \uacfc\uc815\uc5d0\uc11c \ubc1c\uc0dd\ud558\ub294 \uc720\uc804\uc790 \uc911\ubcf5\uc740 \uc0c1\ub3d9\uc131\uc744 \uae68\ub728\ub9ac\uace0 \uc720\uc804\uc790\uac00 \ub2e4\ub978 \uae30\ub2a5\uc744 \ud558\ub3c4\ub85d \ub9cc\ub4e4 \uc218 \uc788\ub2e4. \uc989, \uc720\uc804\uc790 \uc911\ubcf5\uc5d0 \uc758\ud574 \uc0c8\ub86d\uac8c \ud615\uc131\ub41c DNA \uc5fc\uae30\uc11c\uc5f4\uc740 \ub2e8\ubc31\uc9c8 \ub3c4\uba54\uc778\uc758 \uc21c\uc11c\ub97c \ubc14\uafb8\uace0 \uc774 \uacb0\uacfc \uc6d0\ubcf8 DNA\uc640\ub294 \ub2e4\ub978 \ub2e8\ubc31\uc9c8\uc744 \ud615\uc131\ud558\uac8c \ud55c\ub2e4. \uc774\ud574\ub97c \ub3d5\uae30 \uc704\ud574 \uc778\uac04\uc758 \ub208\uc744 \uc608\ub85c \ub4e4\uba74 \ub9dd\ub9c9\uc5d0\ub294 \uc138 \uc885\ub958\uc758 \uc6d0\ucd94 \uc138\ud3ec\uc640 \uac04\uc0c1 \uc138\ud3ec\uac00 \uc788\uc5b4 \ube5b\uc744 \uac10\uc9c0 \ud55c\ub2e4. \uc138 \uc885\ub958\uc758 \uc6d0\ucd94\uc138\ud3ec\ub294 \uac01\uac01 \uace0\uc720\ud55c \uc8fc\ud30c\uc218 \ub300\uc5ed\uc758 \uac00\uc2dc\uad11\uc120\uc5d0 \ubc18\uc751\ud558\uc5ec \uc0c9\uc744 \ubcfc \uc218 \uc788\uac8c \ud55c\ub2e4. \uc6d0\ucd94 \uc138\ud3ec\ub97c \ud615\uc131\ud558\uac8c \ud558\ub294 \uc720\uc804\uc790 \uc911\uc758 \ud558\ub098\uac00 \uc720\uc804\uc790 \uc911\ubcf5\uc73c\ub85c \uc774\uc0c1\uc744 \uc77c\uc73c\ud0a4\uba74 \ud574\ub2f9\ud558\ub294 \uc6d0\ucd94\uc138\ud3ec\uac00 \uc815\uc0c1\uc801\uc778 \uae30\ub2a5\uc744 \ud560 \uc218 \uc5c6\uac8c \ub418\uace0 \uc0c9\uac01 \uc774\uc0c1\uc774 \ubc1c\uc0dd\ud560 \uc218 \uc788\ub2e4.", "answer": "\uc720\uc804\uc790 \uc911\ubcf5", "sentence": "\ub3cc\uc5f0\ubcc0\uc774\uc758 \ub300\ud45c\uc801\uc778 \uc0ac\ub840\ub85c\ub294 \uc720\uc804\uc790 \uc911\ubcf5 \uc774 \uc788\ub2e4.", "paragraph_sentence": " \ub3cc\uc5f0\ubcc0\uc774\uc758 \ub300\ud45c\uc801\uc778 \uc0ac\ub840\ub85c\ub294 \uc720\uc804\uc790 \uc911\ubcf5 \uc774 \uc788\ub2e4. \ub3d9\ubb3c\uc758 \uacbd\uc6b0 \ub9e4 \ucc9c \ub144\ub2f9 \ud55c \ubc88 \uaf34\ub85c \ubc1c\uc0dd\ud558\ub294 \uc720\uc804\uc790 \uc911\ubcf5\uc740 \uc5fc\uc0c9\uccb4\uc758 \uc77c\uc815 \uad6c\uac04\uc774 \uc2e4\uc218\ub85c \ubc18\ubcf5\ud558\uc5ec \ubcf5\uc81c\ub418\ub294 \uac83\uc774\ub2e4. \ub300\ubd80\ubd84\uc758 \uc720\uc804\uc790\ub4e4\uc740 \uc0c1\ub3d9\uc131\uc744 \uac16\ub294 \uc720\uc804\uc790\uc871\uc5d0 \uc790\ub9ac\uc7a1\uace0 \uc788\ub2e4. \uc6d0\ubcf8 DNA\ub97c \ubcf5\uc81c\ud558\ub294 \uacfc\uc815\uc5d0\uc11c \ubc1c\uc0dd\ud558\ub294 \uc720\uc804\uc790 \uc911\ubcf5\uc740 \uc0c1\ub3d9\uc131\uc744 \uae68\ub728\ub9ac\uace0 \uc720\uc804\uc790\uac00 \ub2e4\ub978 \uae30\ub2a5\uc744 \ud558\ub3c4\ub85d \ub9cc\ub4e4 \uc218 \uc788\ub2e4. \uc989, \uc720\uc804\uc790 \uc911\ubcf5\uc5d0 \uc758\ud574 \uc0c8\ub86d\uac8c \ud615\uc131\ub41c DNA \uc5fc\uae30\uc11c\uc5f4\uc740 \ub2e8\ubc31\uc9c8 \ub3c4\uba54\uc778\uc758 \uc21c\uc11c\ub97c \ubc14\uafb8\uace0 \uc774 \uacb0\uacfc \uc6d0\ubcf8 DNA\uc640\ub294 \ub2e4\ub978 \ub2e8\ubc31\uc9c8\uc744 \ud615\uc131\ud558\uac8c \ud55c\ub2e4. \uc774\ud574\ub97c \ub3d5\uae30 \uc704\ud574 \uc778\uac04\uc758 \ub208\uc744 \uc608\ub85c \ub4e4\uba74 \ub9dd\ub9c9\uc5d0\ub294 \uc138 \uc885\ub958\uc758 \uc6d0\ucd94 \uc138\ud3ec\uc640 \uac04\uc0c1 \uc138\ud3ec\uac00 \uc788\uc5b4 \ube5b\uc744 \uac10\uc9c0 \ud55c\ub2e4. \uc138 \uc885\ub958\uc758 \uc6d0\ucd94\uc138\ud3ec\ub294 \uac01\uac01 \uace0\uc720\ud55c \uc8fc\ud30c\uc218 \ub300\uc5ed\uc758 \uac00\uc2dc\uad11\uc120\uc5d0 \ubc18\uc751\ud558\uc5ec \uc0c9\uc744 \ubcfc \uc218 \uc788\uac8c \ud55c\ub2e4. \uc6d0\ucd94 \uc138\ud3ec\ub97c \ud615\uc131\ud558\uac8c \ud558\ub294 \uc720\uc804\uc790 \uc911\uc758 \ud558\ub098\uac00 \uc720\uc804\uc790 \uc911\ubcf5\uc73c\ub85c \uc774\uc0c1\uc744 \uc77c\uc73c\ud0a4\uba74 \ud574\ub2f9\ud558\ub294 \uc6d0\ucd94\uc138\ud3ec\uac00 \uc815\uc0c1\uc801\uc778 \uae30\ub2a5\uc744 \ud560 \uc218 \uc5c6\uac8c \ub418\uace0 \uc0c9\uac01 \uc774\uc0c1\uc774 \ubc1c\uc0dd\ud560 \uc218 \uc788\ub2e4.", "paragraph_answer": "\ub3cc\uc5f0\ubcc0\uc774\uc758 \ub300\ud45c\uc801\uc778 \uc0ac\ub840\ub85c\ub294 \uc720\uc804\uc790 \uc911\ubcf5 \uc774 \uc788\ub2e4. \ub3d9\ubb3c\uc758 \uacbd\uc6b0 \ub9e4 \ucc9c \ub144\ub2f9 \ud55c \ubc88 \uaf34\ub85c \ubc1c\uc0dd\ud558\ub294 \uc720\uc804\uc790 \uc911\ubcf5\uc740 \uc5fc\uc0c9\uccb4\uc758 \uc77c\uc815 \uad6c\uac04\uc774 \uc2e4\uc218\ub85c \ubc18\ubcf5\ud558\uc5ec \ubcf5\uc81c\ub418\ub294 \uac83\uc774\ub2e4. \ub300\ubd80\ubd84\uc758 \uc720\uc804\uc790\ub4e4\uc740 \uc0c1\ub3d9\uc131\uc744 \uac16\ub294 \uc720\uc804\uc790\uc871\uc5d0 \uc790\ub9ac\uc7a1\uace0 \uc788\ub2e4. \uc6d0\ubcf8 DNA\ub97c \ubcf5\uc81c\ud558\ub294 \uacfc\uc815\uc5d0\uc11c \ubc1c\uc0dd\ud558\ub294 \uc720\uc804\uc790 \uc911\ubcf5\uc740 \uc0c1\ub3d9\uc131\uc744 \uae68\ub728\ub9ac\uace0 \uc720\uc804\uc790\uac00 \ub2e4\ub978 \uae30\ub2a5\uc744 \ud558\ub3c4\ub85d \ub9cc\ub4e4 \uc218 \uc788\ub2e4. \uc989, \uc720\uc804\uc790 \uc911\ubcf5\uc5d0 \uc758\ud574 \uc0c8\ub86d\uac8c \ud615\uc131\ub41c DNA \uc5fc\uae30\uc11c\uc5f4\uc740 \ub2e8\ubc31\uc9c8 \ub3c4\uba54\uc778\uc758 \uc21c\uc11c\ub97c \ubc14\uafb8\uace0 \uc774 \uacb0\uacfc \uc6d0\ubcf8 DNA\uc640\ub294 \ub2e4\ub978 \ub2e8\ubc31\uc9c8\uc744 \ud615\uc131\ud558\uac8c \ud55c\ub2e4. \uc774\ud574\ub97c \ub3d5\uae30 \uc704\ud574 \uc778\uac04\uc758 \ub208\uc744 \uc608\ub85c \ub4e4\uba74 \ub9dd\ub9c9\uc5d0\ub294 \uc138 \uc885\ub958\uc758 \uc6d0\ucd94 \uc138\ud3ec\uc640 \uac04\uc0c1 \uc138\ud3ec\uac00 \uc788\uc5b4 \ube5b\uc744 \uac10\uc9c0 \ud55c\ub2e4. \uc138 \uc885\ub958\uc758 \uc6d0\ucd94\uc138\ud3ec\ub294 \uac01\uac01 \uace0\uc720\ud55c \uc8fc\ud30c\uc218 \ub300\uc5ed\uc758 \uac00\uc2dc\uad11\uc120\uc5d0 \ubc18\uc751\ud558\uc5ec \uc0c9\uc744 \ubcfc \uc218 \uc788\uac8c \ud55c\ub2e4. \uc6d0\ucd94 \uc138\ud3ec\ub97c \ud615\uc131\ud558\uac8c \ud558\ub294 \uc720\uc804\uc790 \uc911\uc758 \ud558\ub098\uac00 \uc720\uc804\uc790 \uc911\ubcf5\uc73c\ub85c \uc774\uc0c1\uc744 \uc77c\uc73c\ud0a4\uba74 \ud574\ub2f9\ud558\ub294 \uc6d0\ucd94\uc138\ud3ec\uac00 \uc815\uc0c1\uc801\uc778 \uae30\ub2a5\uc744 \ud560 \uc218 \uc5c6\uac8c \ub418\uace0 \uc0c9\uac01 \uc774\uc0c1\uc774 \ubc1c\uc0dd\ud560 \uc218 \uc788\ub2e4.", "sentence_answer": "\ub3cc\uc5f0\ubcc0\uc774\uc758 \ub300\ud45c\uc801\uc778 \uc0ac\ub840\ub85c\ub294 \uc720\uc804\uc790 \uc911\ubcf5 \uc774 \uc788\ub2e4.", "paragraph_id": "6557516-5-0", "paragraph_question": "question: \ub3cc\uc5f0\ubcc0\uc774\uc758 \ub300\ud45c \uc0ac\ub840\ub294?, context: \ub3cc\uc5f0\ubcc0\uc774\uc758 \ub300\ud45c\uc801\uc778 \uc0ac\ub840\ub85c\ub294 \uc720\uc804\uc790 \uc911\ubcf5\uc774 \uc788\ub2e4. \ub3d9\ubb3c\uc758 \uacbd\uc6b0 \ub9e4 \ucc9c \ub144\ub2f9 \ud55c \ubc88 \uaf34\ub85c \ubc1c\uc0dd\ud558\ub294 \uc720\uc804\uc790 \uc911\ubcf5\uc740 \uc5fc\uc0c9\uccb4\uc758 \uc77c\uc815 \uad6c\uac04\uc774 \uc2e4\uc218\ub85c \ubc18\ubcf5\ud558\uc5ec \ubcf5\uc81c\ub418\ub294 \uac83\uc774\ub2e4. \ub300\ubd80\ubd84\uc758 \uc720\uc804\uc790\ub4e4\uc740 \uc0c1\ub3d9\uc131\uc744 \uac16\ub294 \uc720\uc804\uc790\uc871\uc5d0 \uc790\ub9ac\uc7a1\uace0 \uc788\ub2e4. \uc6d0\ubcf8 DNA\ub97c \ubcf5\uc81c\ud558\ub294 \uacfc\uc815\uc5d0\uc11c \ubc1c\uc0dd\ud558\ub294 \uc720\uc804\uc790 \uc911\ubcf5\uc740 \uc0c1\ub3d9\uc131\uc744 \uae68\ub728\ub9ac\uace0 \uc720\uc804\uc790\uac00 \ub2e4\ub978 \uae30\ub2a5\uc744 \ud558\ub3c4\ub85d \ub9cc\ub4e4 \uc218 \uc788\ub2e4. \uc989, \uc720\uc804\uc790 \uc911\ubcf5\uc5d0 \uc758\ud574 \uc0c8\ub86d\uac8c \ud615\uc131\ub41c DNA \uc5fc\uae30\uc11c\uc5f4\uc740 \ub2e8\ubc31\uc9c8 \ub3c4\uba54\uc778\uc758 \uc21c\uc11c\ub97c \ubc14\uafb8\uace0 \uc774 \uacb0\uacfc \uc6d0\ubcf8 DNA\uc640\ub294 \ub2e4\ub978 \ub2e8\ubc31\uc9c8\uc744 \ud615\uc131\ud558\uac8c \ud55c\ub2e4. \uc774\ud574\ub97c \ub3d5\uae30 \uc704\ud574 \uc778\uac04\uc758 \ub208\uc744 \uc608\ub85c \ub4e4\uba74 \ub9dd\ub9c9\uc5d0\ub294 \uc138 \uc885\ub958\uc758 \uc6d0\ucd94 \uc138\ud3ec\uc640 \uac04\uc0c1 \uc138\ud3ec\uac00 \uc788\uc5b4 \ube5b\uc744 \uac10\uc9c0 \ud55c\ub2e4. \uc138 \uc885\ub958\uc758 \uc6d0\ucd94\uc138\ud3ec\ub294 \uac01\uac01 \uace0\uc720\ud55c \uc8fc\ud30c\uc218 \ub300\uc5ed\uc758 \uac00\uc2dc\uad11\uc120\uc5d0 \ubc18\uc751\ud558\uc5ec \uc0c9\uc744 \ubcfc \uc218 \uc788\uac8c \ud55c\ub2e4. \uc6d0\ucd94 \uc138\ud3ec\ub97c \ud615\uc131\ud558\uac8c \ud558\ub294 \uc720\uc804\uc790 \uc911\uc758 \ud558\ub098\uac00 \uc720\uc804\uc790 \uc911\ubcf5\uc73c\ub85c \uc774\uc0c1\uc744 \uc77c\uc73c\ud0a4\uba74 \ud574\ub2f9\ud558\ub294 \uc6d0\ucd94\uc138\ud3ec\uac00 \uc815\uc0c1\uc801\uc778 \uae30\ub2a5\uc744 \ud560 \uc218 \uc5c6\uac8c \ub418\uace0 \uc0c9\uac01 \uc774\uc0c1\uc774 \ubc1c\uc0dd\ud560 \uc218 \uc788\ub2e4."} {"question": "\uc0b4\ub85c\uacf5\ud654\uad6d\uc774 \uc704\uce58\ud55c \ub3c4\uc2dc\ub294?", "paragraph": "1943\ub144 \uc81c2\ucc28 \uc138\uacc4 \ub300\uc804 \uc911\uc5d0 \uc774\ud0c8\ub9ac\uc544\uac00 \ud328\ubc30\ud558\uba74\uc11c \ubca0\ub2c8\ud1a0 \ubb34\uc194\ub9ac\ub2c8\ub294 \ub098\uce58 \ub3c5\uc77c\uc758 \ubcf4\ud638\uc544\ub798 \uc0b4\ub85c\ub77c\ub294 \ub3c4\uc2dc\uc5d0 \uc774\ud0c8\ub9ac\uc544 \uc0ac\ud68c\uacf5\ud654\uad6d \ub9dd\uba85 \uc815\ubd80\ub97c \uc138\uc6b0\uace0 \uc0b4\ub85c \uacf5\ud654\uad6d\uc774\ub77c \uce6d\ud55c\ub2e4. \ubb34\uc194\ub9ac\ub2c8\ub294 \uadf8\uacf3\uc5d0\uc11c \ud788\ud2c0\ub7ec\uc640 \uc0c1\uc758\ub97c \ud558\uac70\ub098 \uc774\ud0c8\ub9ac\uc544 \ud68c\ubcf5 \ud6c4\uc758 \uc0ac\ud0dc \uc815\ub9ac\uc5d0 \ub300\ud574 \uc758\ub17c\ud558\uba70 \ub9dd\uba85 \uc815\ubd80\uc758 \uc5ed\ud560\uc744 \ud588\ub2e4. \ub098\uce58 \ub3c5\uc77c, \ubd88\uac00\ub9ac\uc544, \uc2ac\ub85c\ubc14\ud0a4\uc544, \ud06c\ub85c\uc544\ud2f0\uc544 \ub3c5\ub9bd\uad6d, \ub8e8\ub9c8\ub2c8\uc544, \ud5dd\uac00\ub9ac, \uc77c\ubcf8 \uc81c\uad6d, \ud540\ub780\ub4dc, \ud0dc\uad6d, \ub9cc\uc8fc\uad6d \ub4f1 \uc5ec\ub7ec \ucd94\ucd95\uad6d\ub4e4\uc774 \uc774\ud0c8\ub9ac\uc544 \uc0ac\ud68c\uacf5\ud654\uad6d\uc758 \uc8fc\uad8c\uc744 \uc778\uc815\ud588\uc744 \ubfd0, \uc2e4\uc9c8\uc801\uc73c\ub85c\ub294 \ub098\uce58 \ub3c5\uc77c\uc758 \uad34\ub8b0 \uc815\ubd80\uc778 \uc148\uc774\ub2e4. \uc989, \uc774\ub984\ub9cc \uc815\ubd80\uc600\uc9c0, \uc0ac\uc2e4\uc0c1 \uc54c\ub824\uc9c0\uc9c0\ub3c4 \uc54a\uc558\uc73c\uba70 \ub300\ubd80\ubd84\uc758 \ub0b4\uac01\uc758 \uc7a5\uad00\uac10\uc778 \uc778\uc0ac\ub4e4\uc774 \ub300\ubd80\ubd84 \ubca0\ub2c8\ud1a0 \ubb34\uc194\ub9ac\ub2c8\uc758 \uc2dc\uae30\uc5d0 \uc219\uccad\ub418\uac70\ub098 \ubbf8\uad70\uc758 \uc774\ud0c8\ub9ac\uc544 \ubcf8\ud1a0 \uc0c1\ub959 \uc774\ud6c4 \uc0ac\ub9dd\ud588\uae30\uc5d0 \uc81c\ub300\ub85c \ub41c \uad6d\uac00 \uad6c\uc2e4\uc744 \ud558\uc9c0 \ubabb\ud55c \ucc44 \ub098\uce58 \ub3c5\uc77c\uc758 \uaf2d\ub450\uac01\uc2dc \uc815\ubd80\ub85c \ub0a8\uc740 \uac83\uc774\ub2e4.", "answer": "\uc0b4\ub85c", "sentence": "1943\ub144 \uc81c2\ucc28 \uc138\uacc4 \ub300\uc804 \uc911\uc5d0 \uc774\ud0c8\ub9ac\uc544\uac00 \ud328\ubc30\ud558\uba74\uc11c \ubca0\ub2c8\ud1a0 \ubb34\uc194\ub9ac\ub2c8\ub294 \ub098\uce58 \ub3c5\uc77c\uc758 \ubcf4\ud638\uc544\ub798 \uc0b4\ub85c \ub77c\ub294 \ub3c4\uc2dc\uc5d0 \uc774\ud0c8\ub9ac\uc544 \uc0ac\ud68c\uacf5\ud654\uad6d \ub9dd\uba85 \uc815\ubd80\ub97c \uc138\uc6b0\uace0 \uc0b4\ub85c \uacf5\ud654\uad6d\uc774\ub77c \uce6d\ud55c\ub2e4.", "paragraph_sentence": " 1943\ub144 \uc81c2\ucc28 \uc138\uacc4 \ub300\uc804 \uc911\uc5d0 \uc774\ud0c8\ub9ac\uc544\uac00 \ud328\ubc30\ud558\uba74\uc11c \ubca0\ub2c8\ud1a0 \ubb34\uc194\ub9ac\ub2c8\ub294 \ub098\uce58 \ub3c5\uc77c\uc758 \ubcf4\ud638\uc544\ub798 \uc0b4\ub85c \ub77c\ub294 \ub3c4\uc2dc\uc5d0 \uc774\ud0c8\ub9ac\uc544 \uc0ac\ud68c\uacf5\ud654\uad6d \ub9dd\uba85 \uc815\ubd80\ub97c \uc138\uc6b0\uace0 \uc0b4\ub85c \uacf5\ud654\uad6d\uc774\ub77c \uce6d\ud55c\ub2e4. \ubb34\uc194\ub9ac\ub2c8\ub294 \uadf8\uacf3\uc5d0\uc11c \ud788\ud2c0\ub7ec\uc640 \uc0c1\uc758\ub97c \ud558\uac70\ub098 \uc774\ud0c8\ub9ac\uc544 \ud68c\ubcf5 \ud6c4\uc758 \uc0ac\ud0dc \uc815\ub9ac\uc5d0 \ub300\ud574 \uc758\ub17c\ud558\uba70 \ub9dd\uba85 \uc815\ubd80\uc758 \uc5ed\ud560\uc744 \ud588\ub2e4. \ub098\uce58 \ub3c5\uc77c, \ubd88\uac00\ub9ac\uc544, \uc2ac\ub85c\ubc14\ud0a4\uc544, \ud06c\ub85c\uc544\ud2f0\uc544 \ub3c5\ub9bd\uad6d, \ub8e8\ub9c8\ub2c8\uc544, \ud5dd\uac00\ub9ac, \uc77c\ubcf8 \uc81c\uad6d, \ud540\ub780\ub4dc, \ud0dc\uad6d, \ub9cc\uc8fc\uad6d \ub4f1 \uc5ec\ub7ec \ucd94\ucd95\uad6d\ub4e4\uc774 \uc774\ud0c8\ub9ac\uc544 \uc0ac\ud68c\uacf5\ud654\uad6d\uc758 \uc8fc\uad8c\uc744 \uc778\uc815\ud588\uc744 \ubfd0, \uc2e4\uc9c8\uc801\uc73c\ub85c\ub294 \ub098\uce58 \ub3c5\uc77c\uc758 \uad34\ub8b0 \uc815\ubd80\uc778 \uc148\uc774\ub2e4. \uc989, \uc774\ub984\ub9cc \uc815\ubd80\uc600\uc9c0, \uc0ac\uc2e4\uc0c1 \uc54c\ub824\uc9c0\uc9c0\ub3c4 \uc54a\uc558\uc73c\uba70 \ub300\ubd80\ubd84\uc758 \ub0b4\uac01\uc758 \uc7a5\uad00\uac10\uc778 \uc778\uc0ac\ub4e4\uc774 \ub300\ubd80\ubd84 \ubca0\ub2c8\ud1a0 \ubb34\uc194\ub9ac\ub2c8\uc758 \uc2dc\uae30\uc5d0 \uc219\uccad\ub418\uac70\ub098 \ubbf8\uad70\uc758 \uc774\ud0c8\ub9ac\uc544 \ubcf8\ud1a0 \uc0c1\ub959 \uc774\ud6c4 \uc0ac\ub9dd\ud588\uae30\uc5d0 \uc81c\ub300\ub85c \ub41c \uad6d\uac00 \uad6c\uc2e4\uc744 \ud558\uc9c0 \ubabb\ud55c \ucc44 \ub098\uce58 \ub3c5\uc77c\uc758 \uaf2d\ub450\uac01\uc2dc \uc815\ubd80\ub85c \ub0a8\uc740 \uac83\uc774\ub2e4.", "paragraph_answer": "1943\ub144 \uc81c2\ucc28 \uc138\uacc4 \ub300\uc804 \uc911\uc5d0 \uc774\ud0c8\ub9ac\uc544\uac00 \ud328\ubc30\ud558\uba74\uc11c \ubca0\ub2c8\ud1a0 \ubb34\uc194\ub9ac\ub2c8\ub294 \ub098\uce58 \ub3c5\uc77c\uc758 \ubcf4\ud638\uc544\ub798 \uc0b4\ub85c \ub77c\ub294 \ub3c4\uc2dc\uc5d0 \uc774\ud0c8\ub9ac\uc544 \uc0ac\ud68c\uacf5\ud654\uad6d \ub9dd\uba85 \uc815\ubd80\ub97c \uc138\uc6b0\uace0 \uc0b4\ub85c \uacf5\ud654\uad6d\uc774\ub77c \uce6d\ud55c\ub2e4. \ubb34\uc194\ub9ac\ub2c8\ub294 \uadf8\uacf3\uc5d0\uc11c \ud788\ud2c0\ub7ec\uc640 \uc0c1\uc758\ub97c \ud558\uac70\ub098 \uc774\ud0c8\ub9ac\uc544 \ud68c\ubcf5 \ud6c4\uc758 \uc0ac\ud0dc \uc815\ub9ac\uc5d0 \ub300\ud574 \uc758\ub17c\ud558\uba70 \ub9dd\uba85 \uc815\ubd80\uc758 \uc5ed\ud560\uc744 \ud588\ub2e4. \ub098\uce58 \ub3c5\uc77c, \ubd88\uac00\ub9ac\uc544, \uc2ac\ub85c\ubc14\ud0a4\uc544, \ud06c\ub85c\uc544\ud2f0\uc544 \ub3c5\ub9bd\uad6d, \ub8e8\ub9c8\ub2c8\uc544, \ud5dd\uac00\ub9ac, \uc77c\ubcf8 \uc81c\uad6d, \ud540\ub780\ub4dc, \ud0dc\uad6d, \ub9cc\uc8fc\uad6d \ub4f1 \uc5ec\ub7ec \ucd94\ucd95\uad6d\ub4e4\uc774 \uc774\ud0c8\ub9ac\uc544 \uc0ac\ud68c\uacf5\ud654\uad6d\uc758 \uc8fc\uad8c\uc744 \uc778\uc815\ud588\uc744 \ubfd0, \uc2e4\uc9c8\uc801\uc73c\ub85c\ub294 \ub098\uce58 \ub3c5\uc77c\uc758 \uad34\ub8b0 \uc815\ubd80\uc778 \uc148\uc774\ub2e4. \uc989, \uc774\ub984\ub9cc \uc815\ubd80\uc600\uc9c0, \uc0ac\uc2e4\uc0c1 \uc54c\ub824\uc9c0\uc9c0\ub3c4 \uc54a\uc558\uc73c\uba70 \ub300\ubd80\ubd84\uc758 \ub0b4\uac01\uc758 \uc7a5\uad00\uac10\uc778 \uc778\uc0ac\ub4e4\uc774 \ub300\ubd80\ubd84 \ubca0\ub2c8\ud1a0 \ubb34\uc194\ub9ac\ub2c8\uc758 \uc2dc\uae30\uc5d0 \uc219\uccad\ub418\uac70\ub098 \ubbf8\uad70\uc758 \uc774\ud0c8\ub9ac\uc544 \ubcf8\ud1a0 \uc0c1\ub959 \uc774\ud6c4 \uc0ac\ub9dd\ud588\uae30\uc5d0 \uc81c\ub300\ub85c \ub41c \uad6d\uac00 \uad6c\uc2e4\uc744 \ud558\uc9c0 \ubabb\ud55c \ucc44 \ub098\uce58 \ub3c5\uc77c\uc758 \uaf2d\ub450\uac01\uc2dc \uc815\ubd80\ub85c \ub0a8\uc740 \uac83\uc774\ub2e4.", "sentence_answer": "1943\ub144 \uc81c2\ucc28 \uc138\uacc4 \ub300\uc804 \uc911\uc5d0 \uc774\ud0c8\ub9ac\uc544\uac00 \ud328\ubc30\ud558\uba74\uc11c \ubca0\ub2c8\ud1a0 \ubb34\uc194\ub9ac\ub2c8\ub294 \ub098\uce58 \ub3c5\uc77c\uc758 \ubcf4\ud638\uc544\ub798 \uc0b4\ub85c \ub77c\ub294 \ub3c4\uc2dc\uc5d0 \uc774\ud0c8\ub9ac\uc544 \uc0ac\ud68c\uacf5\ud654\uad6d \ub9dd\uba85 \uc815\ubd80\ub97c \uc138\uc6b0\uace0 \uc0b4\ub85c \uacf5\ud654\uad6d\uc774\ub77c \uce6d\ud55c\ub2e4.", "paragraph_id": "6590443-0-1", "paragraph_question": "question: \uc0b4\ub85c\uacf5\ud654\uad6d\uc774 \uc704\uce58\ud55c \ub3c4\uc2dc\ub294?, context: 1943\ub144 \uc81c2\ucc28 \uc138\uacc4 \ub300\uc804 \uc911\uc5d0 \uc774\ud0c8\ub9ac\uc544\uac00 \ud328\ubc30\ud558\uba74\uc11c \ubca0\ub2c8\ud1a0 \ubb34\uc194\ub9ac\ub2c8\ub294 \ub098\uce58 \ub3c5\uc77c\uc758 \ubcf4\ud638\uc544\ub798 \uc0b4\ub85c\ub77c\ub294 \ub3c4\uc2dc\uc5d0 \uc774\ud0c8\ub9ac\uc544 \uc0ac\ud68c\uacf5\ud654\uad6d \ub9dd\uba85 \uc815\ubd80\ub97c \uc138\uc6b0\uace0 \uc0b4\ub85c \uacf5\ud654\uad6d\uc774\ub77c \uce6d\ud55c\ub2e4. \ubb34\uc194\ub9ac\ub2c8\ub294 \uadf8\uacf3\uc5d0\uc11c \ud788\ud2c0\ub7ec\uc640 \uc0c1\uc758\ub97c \ud558\uac70\ub098 \uc774\ud0c8\ub9ac\uc544 \ud68c\ubcf5 \ud6c4\uc758 \uc0ac\ud0dc \uc815\ub9ac\uc5d0 \ub300\ud574 \uc758\ub17c\ud558\uba70 \ub9dd\uba85 \uc815\ubd80\uc758 \uc5ed\ud560\uc744 \ud588\ub2e4. \ub098\uce58 \ub3c5\uc77c, \ubd88\uac00\ub9ac\uc544, \uc2ac\ub85c\ubc14\ud0a4\uc544, \ud06c\ub85c\uc544\ud2f0\uc544 \ub3c5\ub9bd\uad6d, \ub8e8\ub9c8\ub2c8\uc544, \ud5dd\uac00\ub9ac, \uc77c\ubcf8 \uc81c\uad6d, \ud540\ub780\ub4dc, \ud0dc\uad6d, \ub9cc\uc8fc\uad6d \ub4f1 \uc5ec\ub7ec \ucd94\ucd95\uad6d\ub4e4\uc774 \uc774\ud0c8\ub9ac\uc544 \uc0ac\ud68c\uacf5\ud654\uad6d\uc758 \uc8fc\uad8c\uc744 \uc778\uc815\ud588\uc744 \ubfd0, \uc2e4\uc9c8\uc801\uc73c\ub85c\ub294 \ub098\uce58 \ub3c5\uc77c\uc758 \uad34\ub8b0 \uc815\ubd80\uc778 \uc148\uc774\ub2e4. \uc989, \uc774\ub984\ub9cc \uc815\ubd80\uc600\uc9c0, \uc0ac\uc2e4\uc0c1 \uc54c\ub824\uc9c0\uc9c0\ub3c4 \uc54a\uc558\uc73c\uba70 \ub300\ubd80\ubd84\uc758 \ub0b4\uac01\uc758 \uc7a5\uad00\uac10\uc778 \uc778\uc0ac\ub4e4\uc774 \ub300\ubd80\ubd84 \ubca0\ub2c8\ud1a0 \ubb34\uc194\ub9ac\ub2c8\uc758 \uc2dc\uae30\uc5d0 \uc219\uccad\ub418\uac70\ub098 \ubbf8\uad70\uc758 \uc774\ud0c8\ub9ac\uc544 \ubcf8\ud1a0 \uc0c1\ub959 \uc774\ud6c4 \uc0ac\ub9dd\ud588\uae30\uc5d0 \uc81c\ub300\ub85c \ub41c \uad6d\uac00 \uad6c\uc2e4\uc744 \ud558\uc9c0 \ubabb\ud55c \ucc44 \ub098\uce58 \ub3c5\uc77c\uc758 \uaf2d\ub450\uac01\uc2dc \uc815\ubd80\ub85c \ub0a8\uc740 \uac83\uc774\ub2e4."} {"question": "GPU\uc640 CPU\ub97c \uac19\uc774 \uc0ac\uc6a9\ud560 \uc218 \uc5c6\ub294 \uc6b4\uc601\uccb4\uc81c\ub294?", "paragraph": "\uc778\ud130\ub137 \uc775\uc2a4\ud50c\ub85c\ub7ec 9\ub294 2011\ub144 3\uc6d4 14\uc77c\uc5d0 \uc815\uc2dd \ucd9c\uc2dc\ub418\uc5c8\ub2e4. \uc6f9 \ud45c\uc900\uc744 \ubcf4\ub2e4 \uc900\uc218\ud558\uc5ec Acid 3\uc5d0\uc11c\ub294 95/100\uc73c\ub85c \uc774\ub294 \uc804\uc758 \uc778\ud130\ub137 \uc775\uc2a4\ud50c\ub85c\ub7ec 8\ubcf4\ub2e4 \ub192\uc740 \uc218\uce58\uc774\ub2e4. \uc77c\ubd80\ub294 \ud55c\uad6d\uc5d0\uc11c \ud638\ud658\uc131 \ubb38\uc81c\uac00 \ubc1c\uc0dd\ud558\uc9c0 \uc54a\uc744\uae4c \ud558\ub294 \uc6b0\ub824\ub3c4 \uc788\uc5c8\uc9c0\ub9cc \uadf8\ub807\uac8c \ud070 \ubb38\uc81c\ub294 \uc5c6\ub294 \uac83\uc73c\ub85c \ubcf4\uc778\ub2e4. \ub2e4\ub9cc \uc77c\ubd80 \uc0ac\uc774\ud2b8\uc5d0\uc11c \ubb38\uc81c\uac00 \ubc1c\uc0dd\ud560 \uc5ec\uc9c0\ub294 \uc788\ub2e4. \uadf8\ub9ac\uace0 \uc778\ud130\ub137 \uc775\uc2a4\ud50c\ub85c\ub7ec 9\ubd80\ud130\ub294 GPU \uac00\uc18d\uc758 \uc9c0\uc6d0\uc73c\ub85c CPU \ubfd0\ub9cc \uc544\ub2c8\ub77c GPU\ub3c4 \ud568\uaed8 \uc4f0\uace0 \uc774 \uae30\uc220\uc774 \uc708\ub3c4\uc6b0 XP, \uc708\ub3c4\uc6b0 \uc11c\ubc84 2003, \uc708\ub3c4\uc6b0 \uc11c\ubc84 2003 R2\uc5d0\uc11c\ub294 \uc9c0\uc6d0\ud558\uae30 \ud798\ub4e4\ub2e4\ub294 \ubb38\uc81c\ub85c \uc708\ub3c4\uc6b0 XP, \uc708\ub3c4\uc6b0 \uc11c\ubc84 2003, \uc708\ub3c4\uc6b0 \uc11c\ubc84 2003 R2\ub294 \uc9c0\uc6d0 \ub300\uc0c1\uc5d0\ub294 \uc81c\uc678\ub418\uc5c8\ub2e4. \uc708\ub3c4\uc6b0 \ube44\uc2a4\ud0c0, \uc708\ub3c4\uc6b0 \uc11c\ubc84 2008, \uc708\ub3c4\uc6b0 7, \uc708\ub3c4\uc6b0 \uc11c\ubc84 2008 R2\ub9cc \uc9c0\uc6d0\ud558\uba70, \uc708\ub3c4\uc6b0 \ube44\uc2a4\ud0c0, \uc708\ub3c4\uc6b0 \uc11c\ubc84 2008\uc744 \uc9c0\uc6d0\ud558\ub294 \ub9c8\uc9c0\ub9c9 \uc6f9 \ube0c\ub77c\uc6b0\uc800\ub2e4. \uc708\ub3c4\uc6b0 7, \uc708\ub3c4\uc6b0 \uc11c\ubc84 2008 R2\uc6a9\uc740 \uc708\ub3c4\uc6b0 XP \uc784\ubca0\ub514\ub4dc, \uc708\ub3c4\uc6b0 8\uacfc \uc708\ub3c4\uc6b0 \ube44\uc2a4\ud0c0\uc6a9 \uc778\ud130\ub137 \uc775\uc2a4\ud50c\ub85c\ub7ec 7\uacfc \uc708\ub3c4\uc6b0 \ube44\uc2a4\ud0c0, \uc708\ub3c4\uc6b0 \uc11c\ubc84 2008, \uc708\ub3c4\uc6b0 7, \uc708\ub3c4\uc6b0 \uc11c\ubc84 2008 R2\uc6a9 \uc778\ud130\ub137 \uc775\uc2a4\ud50c\ub85c\ub7ec 8 \uadf8\ub9ac\uace0 \uc708\ub3c4\uc6b0 7, \uc708\ub3c4\uc6b0 \uc11c\ubc84 2008 R2, \uc708\ub3c4\uc6b0 8\uc6a9 \uc778\ud130\ub137 \uc775\uc2a4\ud50c\ub85c\ub7ec 10\uacfc \uac19\uc774 \uc9c0\uc6d0\uc774 2016\ub144 1\uc6d4 12\uc77c \uc885\ub8cc\ub418\uc5c8\uace0, \uc708\ub3c4\uc6b0 \ube44\uc2a4\ud0c0\uc6a9\uc740 \uc708\ub3c4\uc6b0 \ube44\uc2a4\ud0c0\uc640 \uac19\uc774 \uc9c0\uc6d0\uc774 2017\ub144 4\uc6d4 11\uc77c \uc885\ub8cc\ub418\uc5c8\uc73c\uba70, \uc708\ub3c4\uc6b0 \uc11c\ubc84 2008\uc6a9\uc740 \uc708\ub3c4\uc6b0 7, \uc708\ub3c4\uc6b0 \uc11c\ubc84 2008, \uc708\ub3c4\uc6b0 \uc11c\ubc84 2008 R2 \uadf8\ub9ac\uace0 \uc708\ub3c4\uc6b0 7, \uc708\ub3c4\uc6b0 \uc11c\ubc84 2008 R2\uc6a9 \uc778\ud130\ub137 \uc775\uc2a4\ud50c\ub85c\ub7ec 11\uacfc \uac19\uc774 \uc9c0\uc6d0\uc774 2020\ub144 1\uc6d4 14\uc77c \uc885\ub8cc\ub420 \uc608\uc815\uc774\ub2e4.", "answer": "\uc708\ub3c4\uc6b0 XP, \uc708\ub3c4\uc6b0 \uc11c\ubc84 2003, \uc708\ub3c4\uc6b0 \uc11c\ubc84 2003 R2", "sentence": "\uadf8\ub9ac\uace0 \uc778\ud130\ub137 \uc775\uc2a4\ud50c\ub85c\ub7ec 9\ubd80\ud130\ub294 GPU \uac00\uc18d\uc758 \uc9c0\uc6d0\uc73c\ub85c CPU \ubfd0\ub9cc \uc544\ub2c8\ub77c GPU\ub3c4 \ud568\uaed8 \uc4f0\uace0 \uc774 \uae30\uc220\uc774 \uc708\ub3c4\uc6b0 XP, \uc708\ub3c4\uc6b0 \uc11c\ubc84 2003, \uc708\ub3c4\uc6b0 \uc11c\ubc84 2003 R2 \uc5d0\uc11c\ub294 \uc9c0\uc6d0\ud558\uae30 \ud798\ub4e4\ub2e4\ub294 \ubb38\uc81c\ub85c \uc708\ub3c4\uc6b0 XP, \uc708\ub3c4\uc6b0 \uc11c\ubc84 2003, \uc708\ub3c4\uc6b0 \uc11c\ubc84 2003 R2\ub294 \uc9c0\uc6d0 \ub300\uc0c1\uc5d0\ub294 \uc81c\uc678\ub418\uc5c8\ub2e4.", "paragraph_sentence": "\uc778\ud130\ub137 \uc775\uc2a4\ud50c\ub85c\ub7ec 9\ub294 2011\ub144 3\uc6d4 14\uc77c\uc5d0 \uc815\uc2dd \ucd9c\uc2dc\ub418\uc5c8\ub2e4. \uc6f9 \ud45c\uc900\uc744 \ubcf4\ub2e4 \uc900\uc218\ud558\uc5ec Acid 3\uc5d0\uc11c\ub294 95/100\uc73c\ub85c \uc774\ub294 \uc804\uc758 \uc778\ud130\ub137 \uc775\uc2a4\ud50c\ub85c\ub7ec 8\ubcf4\ub2e4 \ub192\uc740 \uc218\uce58\uc774\ub2e4. \uc77c\ubd80\ub294 \ud55c\uad6d\uc5d0\uc11c \ud638\ud658\uc131 \ubb38\uc81c\uac00 \ubc1c\uc0dd\ud558\uc9c0 \uc54a\uc744\uae4c \ud558\ub294 \uc6b0\ub824\ub3c4 \uc788\uc5c8\uc9c0\ub9cc \uadf8\ub807\uac8c \ud070 \ubb38\uc81c\ub294 \uc5c6\ub294 \uac83\uc73c\ub85c \ubcf4\uc778\ub2e4. \ub2e4\ub9cc \uc77c\ubd80 \uc0ac\uc774\ud2b8\uc5d0\uc11c \ubb38\uc81c\uac00 \ubc1c\uc0dd\ud560 \uc5ec\uc9c0\ub294 \uc788\ub2e4. \uadf8\ub9ac\uace0 \uc778\ud130\ub137 \uc775\uc2a4\ud50c\ub85c\ub7ec 9\ubd80\ud130\ub294 GPU \uac00\uc18d\uc758 \uc9c0\uc6d0\uc73c\ub85c CPU \ubfd0\ub9cc \uc544\ub2c8\ub77c GPU\ub3c4 \ud568\uaed8 \uc4f0\uace0 \uc774 \uae30\uc220\uc774 \uc708\ub3c4\uc6b0 XP, \uc708\ub3c4\uc6b0 \uc11c\ubc84 2003, \uc708\ub3c4\uc6b0 \uc11c\ubc84 2003 R2 \uc5d0\uc11c\ub294 \uc9c0\uc6d0\ud558\uae30 \ud798\ub4e4\ub2e4\ub294 \ubb38\uc81c\ub85c \uc708\ub3c4\uc6b0 XP, \uc708\ub3c4\uc6b0 \uc11c\ubc84 2003, \uc708\ub3c4\uc6b0 \uc11c\ubc84 2003 R2\ub294 \uc9c0\uc6d0 \ub300\uc0c1\uc5d0\ub294 \uc81c\uc678\ub418\uc5c8\ub2e4. \uc708\ub3c4\uc6b0 \ube44\uc2a4\ud0c0, \uc708\ub3c4\uc6b0 \uc11c\ubc84 2008, \uc708\ub3c4\uc6b0 7, \uc708\ub3c4\uc6b0 \uc11c\ubc84 2008 R2\ub9cc \uc9c0\uc6d0\ud558\uba70, \uc708\ub3c4\uc6b0 \ube44\uc2a4\ud0c0, \uc708\ub3c4\uc6b0 \uc11c\ubc84 2008\uc744 \uc9c0\uc6d0\ud558\ub294 \ub9c8\uc9c0\ub9c9 \uc6f9 \ube0c\ub77c\uc6b0\uc800\ub2e4. \uc708\ub3c4\uc6b0 7, \uc708\ub3c4\uc6b0 \uc11c\ubc84 2008 R2\uc6a9\uc740 \uc708\ub3c4\uc6b0 XP \uc784\ubca0\ub514\ub4dc, \uc708\ub3c4\uc6b0 8\uacfc \uc708\ub3c4\uc6b0 \ube44\uc2a4\ud0c0\uc6a9 \uc778\ud130\ub137 \uc775\uc2a4\ud50c\ub85c\ub7ec 7\uacfc \uc708\ub3c4\uc6b0 \ube44\uc2a4\ud0c0, \uc708\ub3c4\uc6b0 \uc11c\ubc84 2008, \uc708\ub3c4\uc6b0 7, \uc708\ub3c4\uc6b0 \uc11c\ubc84 2008 R2\uc6a9 \uc778\ud130\ub137 \uc775\uc2a4\ud50c\ub85c\ub7ec 8 \uadf8\ub9ac\uace0 \uc708\ub3c4\uc6b0 7, \uc708\ub3c4\uc6b0 \uc11c\ubc84 2008 R2, \uc708\ub3c4\uc6b0 8\uc6a9 \uc778\ud130\ub137 \uc775\uc2a4\ud50c\ub85c\ub7ec 10\uacfc \uac19\uc774 \uc9c0\uc6d0\uc774 2016\ub144 1\uc6d4 12\uc77c \uc885\ub8cc\ub418\uc5c8\uace0, \uc708\ub3c4\uc6b0 \ube44\uc2a4\ud0c0\uc6a9\uc740 \uc708\ub3c4\uc6b0 \ube44\uc2a4\ud0c0\uc640 \uac19\uc774 \uc9c0\uc6d0\uc774 2017\ub144 4\uc6d4 11\uc77c \uc885\ub8cc\ub418\uc5c8\uc73c\uba70, \uc708\ub3c4\uc6b0 \uc11c\ubc84 2008\uc6a9\uc740 \uc708\ub3c4\uc6b0 7, \uc708\ub3c4\uc6b0 \uc11c\ubc84 2008, \uc708\ub3c4\uc6b0 \uc11c\ubc84 2008 R2 \uadf8\ub9ac\uace0 \uc708\ub3c4\uc6b0 7, \uc708\ub3c4\uc6b0 \uc11c\ubc84 2008 R2\uc6a9 \uc778\ud130\ub137 \uc775\uc2a4\ud50c\ub85c\ub7ec 11\uacfc \uac19\uc774 \uc9c0\uc6d0\uc774 2020\ub144 1\uc6d4 14\uc77c \uc885\ub8cc\ub420 \uc608\uc815\uc774\ub2e4.", "paragraph_answer": "\uc778\ud130\ub137 \uc775\uc2a4\ud50c\ub85c\ub7ec 9\ub294 2011\ub144 3\uc6d4 14\uc77c\uc5d0 \uc815\uc2dd \ucd9c\uc2dc\ub418\uc5c8\ub2e4. \uc6f9 \ud45c\uc900\uc744 \ubcf4\ub2e4 \uc900\uc218\ud558\uc5ec Acid 3\uc5d0\uc11c\ub294 95/100\uc73c\ub85c \uc774\ub294 \uc804\uc758 \uc778\ud130\ub137 \uc775\uc2a4\ud50c\ub85c\ub7ec 8\ubcf4\ub2e4 \ub192\uc740 \uc218\uce58\uc774\ub2e4. \uc77c\ubd80\ub294 \ud55c\uad6d\uc5d0\uc11c \ud638\ud658\uc131 \ubb38\uc81c\uac00 \ubc1c\uc0dd\ud558\uc9c0 \uc54a\uc744\uae4c \ud558\ub294 \uc6b0\ub824\ub3c4 \uc788\uc5c8\uc9c0\ub9cc \uadf8\ub807\uac8c \ud070 \ubb38\uc81c\ub294 \uc5c6\ub294 \uac83\uc73c\ub85c \ubcf4\uc778\ub2e4. \ub2e4\ub9cc \uc77c\ubd80 \uc0ac\uc774\ud2b8\uc5d0\uc11c \ubb38\uc81c\uac00 \ubc1c\uc0dd\ud560 \uc5ec\uc9c0\ub294 \uc788\ub2e4. \uadf8\ub9ac\uace0 \uc778\ud130\ub137 \uc775\uc2a4\ud50c\ub85c\ub7ec 9\ubd80\ud130\ub294 GPU \uac00\uc18d\uc758 \uc9c0\uc6d0\uc73c\ub85c CPU \ubfd0\ub9cc \uc544\ub2c8\ub77c GPU\ub3c4 \ud568\uaed8 \uc4f0\uace0 \uc774 \uae30\uc220\uc774 \uc708\ub3c4\uc6b0 XP, \uc708\ub3c4\uc6b0 \uc11c\ubc84 2003, \uc708\ub3c4\uc6b0 \uc11c\ubc84 2003 R2 \uc5d0\uc11c\ub294 \uc9c0\uc6d0\ud558\uae30 \ud798\ub4e4\ub2e4\ub294 \ubb38\uc81c\ub85c \uc708\ub3c4\uc6b0 XP, \uc708\ub3c4\uc6b0 \uc11c\ubc84 2003, \uc708\ub3c4\uc6b0 \uc11c\ubc84 2003 R2\ub294 \uc9c0\uc6d0 \ub300\uc0c1\uc5d0\ub294 \uc81c\uc678\ub418\uc5c8\ub2e4. \uc708\ub3c4\uc6b0 \ube44\uc2a4\ud0c0, \uc708\ub3c4\uc6b0 \uc11c\ubc84 2008, \uc708\ub3c4\uc6b0 7, \uc708\ub3c4\uc6b0 \uc11c\ubc84 2008 R2\ub9cc \uc9c0\uc6d0\ud558\uba70, \uc708\ub3c4\uc6b0 \ube44\uc2a4\ud0c0, \uc708\ub3c4\uc6b0 \uc11c\ubc84 2008\uc744 \uc9c0\uc6d0\ud558\ub294 \ub9c8\uc9c0\ub9c9 \uc6f9 \ube0c\ub77c\uc6b0\uc800\ub2e4. \uc708\ub3c4\uc6b0 7, \uc708\ub3c4\uc6b0 \uc11c\ubc84 2008 R2\uc6a9\uc740 \uc708\ub3c4\uc6b0 XP \uc784\ubca0\ub514\ub4dc, \uc708\ub3c4\uc6b0 8\uacfc \uc708\ub3c4\uc6b0 \ube44\uc2a4\ud0c0\uc6a9 \uc778\ud130\ub137 \uc775\uc2a4\ud50c\ub85c\ub7ec 7\uacfc \uc708\ub3c4\uc6b0 \ube44\uc2a4\ud0c0, \uc708\ub3c4\uc6b0 \uc11c\ubc84 2008, \uc708\ub3c4\uc6b0 7, \uc708\ub3c4\uc6b0 \uc11c\ubc84 2008 R2\uc6a9 \uc778\ud130\ub137 \uc775\uc2a4\ud50c\ub85c\ub7ec 8 \uadf8\ub9ac\uace0 \uc708\ub3c4\uc6b0 7, \uc708\ub3c4\uc6b0 \uc11c\ubc84 2008 R2, \uc708\ub3c4\uc6b0 8\uc6a9 \uc778\ud130\ub137 \uc775\uc2a4\ud50c\ub85c\ub7ec 10\uacfc \uac19\uc774 \uc9c0\uc6d0\uc774 2016\ub144 1\uc6d4 12\uc77c \uc885\ub8cc\ub418\uc5c8\uace0, \uc708\ub3c4\uc6b0 \ube44\uc2a4\ud0c0\uc6a9\uc740 \uc708\ub3c4\uc6b0 \ube44\uc2a4\ud0c0\uc640 \uac19\uc774 \uc9c0\uc6d0\uc774 2017\ub144 4\uc6d4 11\uc77c \uc885\ub8cc\ub418\uc5c8\uc73c\uba70, \uc708\ub3c4\uc6b0 \uc11c\ubc84 2008\uc6a9\uc740 \uc708\ub3c4\uc6b0 7, \uc708\ub3c4\uc6b0 \uc11c\ubc84 2008, \uc708\ub3c4\uc6b0 \uc11c\ubc84 2008 R2 \uadf8\ub9ac\uace0 \uc708\ub3c4\uc6b0 7, \uc708\ub3c4\uc6b0 \uc11c\ubc84 2008 R2\uc6a9 \uc778\ud130\ub137 \uc775\uc2a4\ud50c\ub85c\ub7ec 11\uacfc \uac19\uc774 \uc9c0\uc6d0\uc774 2020\ub144 1\uc6d4 14\uc77c \uc885\ub8cc\ub420 \uc608\uc815\uc774\ub2e4.", "sentence_answer": "\uadf8\ub9ac\uace0 \uc778\ud130\ub137 \uc775\uc2a4\ud50c\ub85c\ub7ec 9\ubd80\ud130\ub294 GPU \uac00\uc18d\uc758 \uc9c0\uc6d0\uc73c\ub85c CPU \ubfd0\ub9cc \uc544\ub2c8\ub77c GPU\ub3c4 \ud568\uaed8 \uc4f0\uace0 \uc774 \uae30\uc220\uc774 \uc708\ub3c4\uc6b0 XP, \uc708\ub3c4\uc6b0 \uc11c\ubc84 2003, \uc708\ub3c4\uc6b0 \uc11c\ubc84 2003 R2 \uc5d0\uc11c\ub294 \uc9c0\uc6d0\ud558\uae30 \ud798\ub4e4\ub2e4\ub294 \ubb38\uc81c\ub85c \uc708\ub3c4\uc6b0 XP, \uc708\ub3c4\uc6b0 \uc11c\ubc84 2003, \uc708\ub3c4\uc6b0 \uc11c\ubc84 2003 R2\ub294 \uc9c0\uc6d0 \ub300\uc0c1\uc5d0\ub294 \uc81c\uc678\ub418\uc5c8\ub2e4.", "paragraph_id": "6548255-4-1", "paragraph_question": "question: GPU\uc640 CPU\ub97c \uac19\uc774 \uc0ac\uc6a9\ud560 \uc218 \uc5c6\ub294 \uc6b4\uc601\uccb4\uc81c\ub294?, context: \uc778\ud130\ub137 \uc775\uc2a4\ud50c\ub85c\ub7ec 9\ub294 2011\ub144 3\uc6d4 14\uc77c\uc5d0 \uc815\uc2dd \ucd9c\uc2dc\ub418\uc5c8\ub2e4. \uc6f9 \ud45c\uc900\uc744 \ubcf4\ub2e4 \uc900\uc218\ud558\uc5ec Acid 3\uc5d0\uc11c\ub294 95/100\uc73c\ub85c \uc774\ub294 \uc804\uc758 \uc778\ud130\ub137 \uc775\uc2a4\ud50c\ub85c\ub7ec 8\ubcf4\ub2e4 \ub192\uc740 \uc218\uce58\uc774\ub2e4. \uc77c\ubd80\ub294 \ud55c\uad6d\uc5d0\uc11c \ud638\ud658\uc131 \ubb38\uc81c\uac00 \ubc1c\uc0dd\ud558\uc9c0 \uc54a\uc744\uae4c \ud558\ub294 \uc6b0\ub824\ub3c4 \uc788\uc5c8\uc9c0\ub9cc \uadf8\ub807\uac8c \ud070 \ubb38\uc81c\ub294 \uc5c6\ub294 \uac83\uc73c\ub85c \ubcf4\uc778\ub2e4. \ub2e4\ub9cc \uc77c\ubd80 \uc0ac\uc774\ud2b8\uc5d0\uc11c \ubb38\uc81c\uac00 \ubc1c\uc0dd\ud560 \uc5ec\uc9c0\ub294 \uc788\ub2e4. \uadf8\ub9ac\uace0 \uc778\ud130\ub137 \uc775\uc2a4\ud50c\ub85c\ub7ec 9\ubd80\ud130\ub294 GPU \uac00\uc18d\uc758 \uc9c0\uc6d0\uc73c\ub85c CPU \ubfd0\ub9cc \uc544\ub2c8\ub77c GPU\ub3c4 \ud568\uaed8 \uc4f0\uace0 \uc774 \uae30\uc220\uc774 \uc708\ub3c4\uc6b0 XP, \uc708\ub3c4\uc6b0 \uc11c\ubc84 2003, \uc708\ub3c4\uc6b0 \uc11c\ubc84 2003 R2\uc5d0\uc11c\ub294 \uc9c0\uc6d0\ud558\uae30 \ud798\ub4e4\ub2e4\ub294 \ubb38\uc81c\ub85c \uc708\ub3c4\uc6b0 XP, \uc708\ub3c4\uc6b0 \uc11c\ubc84 2003, \uc708\ub3c4\uc6b0 \uc11c\ubc84 2003 R2\ub294 \uc9c0\uc6d0 \ub300\uc0c1\uc5d0\ub294 \uc81c\uc678\ub418\uc5c8\ub2e4. \uc708\ub3c4\uc6b0 \ube44\uc2a4\ud0c0, \uc708\ub3c4\uc6b0 \uc11c\ubc84 2008, \uc708\ub3c4\uc6b0 7, \uc708\ub3c4\uc6b0 \uc11c\ubc84 2008 R2\ub9cc \uc9c0\uc6d0\ud558\uba70, \uc708\ub3c4\uc6b0 \ube44\uc2a4\ud0c0, \uc708\ub3c4\uc6b0 \uc11c\ubc84 2008\uc744 \uc9c0\uc6d0\ud558\ub294 \ub9c8\uc9c0\ub9c9 \uc6f9 \ube0c\ub77c\uc6b0\uc800\ub2e4. \uc708\ub3c4\uc6b0 7, \uc708\ub3c4\uc6b0 \uc11c\ubc84 2008 R2\uc6a9\uc740 \uc708\ub3c4\uc6b0 XP \uc784\ubca0\ub514\ub4dc, \uc708\ub3c4\uc6b0 8\uacfc \uc708\ub3c4\uc6b0 \ube44\uc2a4\ud0c0\uc6a9 \uc778\ud130\ub137 \uc775\uc2a4\ud50c\ub85c\ub7ec 7\uacfc \uc708\ub3c4\uc6b0 \ube44\uc2a4\ud0c0, \uc708\ub3c4\uc6b0 \uc11c\ubc84 2008, \uc708\ub3c4\uc6b0 7, \uc708\ub3c4\uc6b0 \uc11c\ubc84 2008 R2\uc6a9 \uc778\ud130\ub137 \uc775\uc2a4\ud50c\ub85c\ub7ec 8 \uadf8\ub9ac\uace0 \uc708\ub3c4\uc6b0 7, \uc708\ub3c4\uc6b0 \uc11c\ubc84 2008 R2, \uc708\ub3c4\uc6b0 8\uc6a9 \uc778\ud130\ub137 \uc775\uc2a4\ud50c\ub85c\ub7ec 10\uacfc \uac19\uc774 \uc9c0\uc6d0\uc774 2016\ub144 1\uc6d4 12\uc77c \uc885\ub8cc\ub418\uc5c8\uace0, \uc708\ub3c4\uc6b0 \ube44\uc2a4\ud0c0\uc6a9\uc740 \uc708\ub3c4\uc6b0 \ube44\uc2a4\ud0c0\uc640 \uac19\uc774 \uc9c0\uc6d0\uc774 2017\ub144 4\uc6d4 11\uc77c \uc885\ub8cc\ub418\uc5c8\uc73c\uba70, \uc708\ub3c4\uc6b0 \uc11c\ubc84 2008\uc6a9\uc740 \uc708\ub3c4\uc6b0 7, \uc708\ub3c4\uc6b0 \uc11c\ubc84 2008, \uc708\ub3c4\uc6b0 \uc11c\ubc84 2008 R2 \uadf8\ub9ac\uace0 \uc708\ub3c4\uc6b0 7, \uc708\ub3c4\uc6b0 \uc11c\ubc84 2008 R2\uc6a9 \uc778\ud130\ub137 \uc775\uc2a4\ud50c\ub85c\ub7ec 11\uacfc \uac19\uc774 \uc9c0\uc6d0\uc774 2020\ub144 1\uc6d4 14\uc77c \uc885\ub8cc\ub420 \uc608\uc815\uc774\ub2e4."} {"question": "\uc870\uc120 \uc2dc\ub300 \ub54c\ubd80\ud130 \uc591\ubc18 \uc0ac\ub300\ubd80\ub4e4\uc758 \ubb38\ud654 \ubc1c\uc804\uc774 \uae30\ubc18\ud55c \uac83\uc740?", "paragraph": "\ud55c\uad6d \ubbf8\uc220\uc740 \uc57d \uae30\uc6d0\uc804 7\ucc9c\uc5ec\ub144\uc804 \uc2e0\uc11d\uae30 \uc2dc\ub300\ubd80\ud130 \uc2dc\uc791\ub418\uc5c8\ub2e4. \uace0\uc870\uc120 \uc2dc\uae30\uc5d0\ub294 \uc81c\uc758\ub97c \uc704\ud574 \ub9cc\ub4e4\uc5b4\uc9c4 \ube44\ud30c\ud615 \ub3d9\uac80, \ub3d9\uacbd (\uac70\uc6b8), \ubc29\uc6b8\uc5d0\uc11c \uadf8 \uc608\ub97c \ucc3e\uc544\ubcfc \uc218 \uc788\ub2e4. \uc0bc\uad6d \uc2dc\ub300\uc5d0\ub294 \uc655\uc871\uacfc \uadc0\uc871\uc744 \uc704\ud55c \uc608\uc220\uc774 \ub4f1\uc7a5\ud558\uc600\ub294\ub370, \uace0\uad6c\ub824\uc758 \uace0\ubd84\ubcbd\ud654, \ubc31\uc81c\uc758 \uae08\ub3d9\ub300\ud5a5\ub85c, \uc2e0\ub77c\uc758 \uae08\uad00\uc774 \ub300\ud45c\uc801\uc774\ub2e4. \uc774\ud6c4 \uace0\ub824\uc2dc\ub300\uc5d0\ub294 \ub3c4\uc790\uae30\uc758 \uc77c\uc885\uc778 \uace0\ub824 \uccad\uc790\uc640 \uba39\uc73c\ub85c \uadf8\ub9ac\ub294 \ubb38\uc778\ud654\uac00 \ubc1c\ub2ec\ud588\ub2e4. \uc870\uc120 \uc2dc\ub300 \ub54c\ubd80\ud130 \uc591\ubc18 \uc0ac\ub300\ubd80\ub4e4\uc744 \uc131\ub9ac\ud559\uc5d0 \uae30\ubc18\ud55c \ubb38\ud654\ub97c \ubc1c\uc804\uc2dc\ucf30\uc73c\uba70, \uc774\ub4e4\uc740 \ubb38\uc778\ud654\uc640 \ubc31\uc790\ub97c \uc120\ud638\ud558\uc600\ub2e4. \uc774\ub4e4 \ubb38\ud654\ub294 \ub2e4\ubd84\ud788 \uc0ac\ub300\uc8fc\uc758\uc801\uc774\uc5c8\uc73c\ub098, \uc601\uc870\uc640 \uc815\uc870 \uc2dc\ub300\uc5d0\ub294 \uc815\uc120, \uae40\ud64d\ub3c4, \uc2e0\uc724\ubcf5 \ub4f1\uc758 \ud654\uc6d0\ub4e4\uc774 \ub098\ud0c0\ub098 \ud2b9\uc0c9 \uc788\ub294 \ubbf8\uc220\uc744 \ub9cc\ub4e4\uc5c8\ub2e4. \ub3d9\uc2dc\uc5d0 \uc870\uc120\uc2dc\ub300\uc5d0\ub294 \uc11c\ubbfc\uc801\uc778 \ubbf8\uc220\uc778 \ubbfc\ud654\uac00 \ubc1c\ub2ec\ud588\ub2e4.", "answer": "\uc131\ub9ac\ud559", "sentence": "\uc870\uc120 \uc2dc\ub300 \ub54c\ubd80\ud130 \uc591\ubc18 \uc0ac\ub300\ubd80\ub4e4\uc744 \uc131\ub9ac\ud559 \uc5d0 \uae30\ubc18\ud55c \ubb38\ud654\ub97c \ubc1c\uc804\uc2dc\ucf30\uc73c\uba70, \uc774\ub4e4\uc740 \ubb38\uc778\ud654\uc640 \ubc31\uc790\ub97c \uc120\ud638\ud558\uc600\ub2e4.", "paragraph_sentence": "\ud55c\uad6d \ubbf8\uc220\uc740 \uc57d \uae30\uc6d0\uc804 7\ucc9c\uc5ec\ub144\uc804 \uc2e0\uc11d\uae30 \uc2dc\ub300\ubd80\ud130 \uc2dc\uc791\ub418\uc5c8\ub2e4. \uace0\uc870\uc120 \uc2dc\uae30\uc5d0\ub294 \uc81c\uc758\ub97c \uc704\ud574 \ub9cc\ub4e4\uc5b4\uc9c4 \ube44\ud30c\ud615 \ub3d9\uac80, \ub3d9\uacbd (\uac70\uc6b8), \ubc29\uc6b8\uc5d0\uc11c \uadf8 \uc608\ub97c \ucc3e\uc544\ubcfc \uc218 \uc788\ub2e4. \uc0bc\uad6d \uc2dc\ub300\uc5d0\ub294 \uc655\uc871\uacfc \uadc0\uc871\uc744 \uc704\ud55c \uc608\uc220\uc774 \ub4f1\uc7a5\ud558\uc600\ub294\ub370, \uace0\uad6c\ub824\uc758 \uace0\ubd84\ubcbd\ud654, \ubc31\uc81c\uc758 \uae08\ub3d9\ub300\ud5a5\ub85c, \uc2e0\ub77c\uc758 \uae08\uad00\uc774 \ub300\ud45c\uc801\uc774\ub2e4. \uc774\ud6c4 \uace0\ub824\uc2dc\ub300\uc5d0\ub294 \ub3c4\uc790\uae30\uc758 \uc77c\uc885\uc778 \uace0\ub824 \uccad\uc790\uc640 \uba39\uc73c\ub85c \uadf8\ub9ac\ub294 \ubb38\uc778\ud654\uac00 \ubc1c\ub2ec\ud588\ub2e4. \uc870\uc120 \uc2dc\ub300 \ub54c\ubd80\ud130 \uc591\ubc18 \uc0ac\ub300\ubd80\ub4e4\uc744 \uc131\ub9ac\ud559 \uc5d0 \uae30\ubc18\ud55c \ubb38\ud654\ub97c \ubc1c\uc804\uc2dc\ucf30\uc73c\uba70, \uc774\ub4e4\uc740 \ubb38\uc778\ud654\uc640 \ubc31\uc790\ub97c \uc120\ud638\ud558\uc600\ub2e4. \uc774\ub4e4 \ubb38\ud654\ub294 \ub2e4\ubd84\ud788 \uc0ac\ub300\uc8fc\uc758\uc801\uc774\uc5c8\uc73c\ub098, \uc601\uc870\uc640 \uc815\uc870 \uc2dc\ub300\uc5d0\ub294 \uc815\uc120, \uae40\ud64d\ub3c4, \uc2e0\uc724\ubcf5 \ub4f1\uc758 \ud654\uc6d0\ub4e4\uc774 \ub098\ud0c0\ub098 \ud2b9\uc0c9 \uc788\ub294 \ubbf8\uc220\uc744 \ub9cc\ub4e4\uc5c8\ub2e4. \ub3d9\uc2dc\uc5d0 \uc870\uc120\uc2dc\ub300\uc5d0\ub294 \uc11c\ubbfc\uc801\uc778 \ubbf8\uc220\uc778 \ubbfc\ud654\uac00 \ubc1c\ub2ec\ud588\ub2e4.", "paragraph_answer": "\ud55c\uad6d \ubbf8\uc220\uc740 \uc57d \uae30\uc6d0\uc804 7\ucc9c\uc5ec\ub144\uc804 \uc2e0\uc11d\uae30 \uc2dc\ub300\ubd80\ud130 \uc2dc\uc791\ub418\uc5c8\ub2e4. \uace0\uc870\uc120 \uc2dc\uae30\uc5d0\ub294 \uc81c\uc758\ub97c \uc704\ud574 \ub9cc\ub4e4\uc5b4\uc9c4 \ube44\ud30c\ud615 \ub3d9\uac80, \ub3d9\uacbd (\uac70\uc6b8), \ubc29\uc6b8\uc5d0\uc11c \uadf8 \uc608\ub97c \ucc3e\uc544\ubcfc \uc218 \uc788\ub2e4. \uc0bc\uad6d \uc2dc\ub300\uc5d0\ub294 \uc655\uc871\uacfc \uadc0\uc871\uc744 \uc704\ud55c \uc608\uc220\uc774 \ub4f1\uc7a5\ud558\uc600\ub294\ub370, \uace0\uad6c\ub824\uc758 \uace0\ubd84\ubcbd\ud654, \ubc31\uc81c\uc758 \uae08\ub3d9\ub300\ud5a5\ub85c, \uc2e0\ub77c\uc758 \uae08\uad00\uc774 \ub300\ud45c\uc801\uc774\ub2e4. \uc774\ud6c4 \uace0\ub824\uc2dc\ub300\uc5d0\ub294 \ub3c4\uc790\uae30\uc758 \uc77c\uc885\uc778 \uace0\ub824 \uccad\uc790\uc640 \uba39\uc73c\ub85c \uadf8\ub9ac\ub294 \ubb38\uc778\ud654\uac00 \ubc1c\ub2ec\ud588\ub2e4. \uc870\uc120 \uc2dc\ub300 \ub54c\ubd80\ud130 \uc591\ubc18 \uc0ac\ub300\ubd80\ub4e4\uc744 \uc131\ub9ac\ud559 \uc5d0 \uae30\ubc18\ud55c \ubb38\ud654\ub97c \ubc1c\uc804\uc2dc\ucf30\uc73c\uba70, \uc774\ub4e4\uc740 \ubb38\uc778\ud654\uc640 \ubc31\uc790\ub97c \uc120\ud638\ud558\uc600\ub2e4. \uc774\ub4e4 \ubb38\ud654\ub294 \ub2e4\ubd84\ud788 \uc0ac\ub300\uc8fc\uc758\uc801\uc774\uc5c8\uc73c\ub098, \uc601\uc870\uc640 \uc815\uc870 \uc2dc\ub300\uc5d0\ub294 \uc815\uc120, \uae40\ud64d\ub3c4, \uc2e0\uc724\ubcf5 \ub4f1\uc758 \ud654\uc6d0\ub4e4\uc774 \ub098\ud0c0\ub098 \ud2b9\uc0c9 \uc788\ub294 \ubbf8\uc220\uc744 \ub9cc\ub4e4\uc5c8\ub2e4. \ub3d9\uc2dc\uc5d0 \uc870\uc120\uc2dc\ub300\uc5d0\ub294 \uc11c\ubbfc\uc801\uc778 \ubbf8\uc220\uc778 \ubbfc\ud654\uac00 \ubc1c\ub2ec\ud588\ub2e4.", "sentence_answer": "\uc870\uc120 \uc2dc\ub300 \ub54c\ubd80\ud130 \uc591\ubc18 \uc0ac\ub300\ubd80\ub4e4\uc744 \uc131\ub9ac\ud559 \uc5d0 \uae30\ubc18\ud55c \ubb38\ud654\ub97c \ubc1c\uc804\uc2dc\ucf30\uc73c\uba70, \uc774\ub4e4\uc740 \ubb38\uc778\ud654\uc640 \ubc31\uc790\ub97c \uc120\ud638\ud558\uc600\ub2e4.", "paragraph_id": "6490879-40-1", "paragraph_question": "question: \uc870\uc120 \uc2dc\ub300 \ub54c\ubd80\ud130 \uc591\ubc18 \uc0ac\ub300\ubd80\ub4e4\uc758 \ubb38\ud654 \ubc1c\uc804\uc774 \uae30\ubc18\ud55c \uac83\uc740?, context: \ud55c\uad6d \ubbf8\uc220\uc740 \uc57d \uae30\uc6d0\uc804 7\ucc9c\uc5ec\ub144\uc804 \uc2e0\uc11d\uae30 \uc2dc\ub300\ubd80\ud130 \uc2dc\uc791\ub418\uc5c8\ub2e4. \uace0\uc870\uc120 \uc2dc\uae30\uc5d0\ub294 \uc81c\uc758\ub97c \uc704\ud574 \ub9cc\ub4e4\uc5b4\uc9c4 \ube44\ud30c\ud615 \ub3d9\uac80, \ub3d9\uacbd (\uac70\uc6b8), \ubc29\uc6b8\uc5d0\uc11c \uadf8 \uc608\ub97c \ucc3e\uc544\ubcfc \uc218 \uc788\ub2e4. \uc0bc\uad6d \uc2dc\ub300\uc5d0\ub294 \uc655\uc871\uacfc \uadc0\uc871\uc744 \uc704\ud55c \uc608\uc220\uc774 \ub4f1\uc7a5\ud558\uc600\ub294\ub370, \uace0\uad6c\ub824\uc758 \uace0\ubd84\ubcbd\ud654, \ubc31\uc81c\uc758 \uae08\ub3d9\ub300\ud5a5\ub85c, \uc2e0\ub77c\uc758 \uae08\uad00\uc774 \ub300\ud45c\uc801\uc774\ub2e4. \uc774\ud6c4 \uace0\ub824\uc2dc\ub300\uc5d0\ub294 \ub3c4\uc790\uae30\uc758 \uc77c\uc885\uc778 \uace0\ub824 \uccad\uc790\uc640 \uba39\uc73c\ub85c \uadf8\ub9ac\ub294 \ubb38\uc778\ud654\uac00 \ubc1c\ub2ec\ud588\ub2e4. \uc870\uc120 \uc2dc\ub300 \ub54c\ubd80\ud130 \uc591\ubc18 \uc0ac\ub300\ubd80\ub4e4\uc744 \uc131\ub9ac\ud559\uc5d0 \uae30\ubc18\ud55c \ubb38\ud654\ub97c \ubc1c\uc804\uc2dc\ucf30\uc73c\uba70, \uc774\ub4e4\uc740 \ubb38\uc778\ud654\uc640 \ubc31\uc790\ub97c \uc120\ud638\ud558\uc600\ub2e4. \uc774\ub4e4 \ubb38\ud654\ub294 \ub2e4\ubd84\ud788 \uc0ac\ub300\uc8fc\uc758\uc801\uc774\uc5c8\uc73c\ub098, \uc601\uc870\uc640 \uc815\uc870 \uc2dc\ub300\uc5d0\ub294 \uc815\uc120, \uae40\ud64d\ub3c4, \uc2e0\uc724\ubcf5 \ub4f1\uc758 \ud654\uc6d0\ub4e4\uc774 \ub098\ud0c0\ub098 \ud2b9\uc0c9 \uc788\ub294 \ubbf8\uc220\uc744 \ub9cc\ub4e4\uc5c8\ub2e4. \ub3d9\uc2dc\uc5d0 \uc870\uc120\uc2dc\ub300\uc5d0\ub294 \uc11c\ubbfc\uc801\uc778 \ubbf8\uc220\uc778 \ubbfc\ud654\uac00 \ubc1c\ub2ec\ud588\ub2e4."} {"question": "\uace0\uc5f0\uc804\uc740 \ub9e4\ub144 \uba87\uc6d4\uc5d0 \uc9c4\ud589\ub418\ub294\uac00?", "paragraph": "\uace0\uc5f0\uc804\uc740 \uc560\uad50\uc2ec\uc744 \uace0\uc591\ud558\uace0 \uace0\ub824\ub300\ud559\uad50\uc640 \uc5f0\uc138\ub300\ud559\uad50 \uc591\uad50\uc758 \uce5c\uc120\uc744 \ub3c4\ubaa8\ud558\uae30 \uc704\ud55c \ubaa9\uc801\uc73c\ub85c \uc2a4\ud3ec\uce20 \uacbd\uae30 \ub97c \ube44\ub86f\ud55c \uac15\uc5f0, \uc628\ub77c\uc778 \uac8c\uc784, \uc0ac\ud68c\uacf5\ud5cc\ud65c\ub3d9 \uacfc \uac19\uc740 \ub2e4\uc591\ud55c \ubd84\uc57c\uc5d0\uc11c \uc5f0\uc138\ub300\ud559\uad50\uc640 \uc2b9\ubd80\ub97c \uaca8\ub8e8\ub294 \ud589\uc0ac \uc77c\uccb4\ub97c \uc758\ubbf8\ud55c\ub2e4. \uc601\uad6d\uc758 \uc625\uc2a4\ud3ec\ub4dc \ub300\ud559\uad50\uc640 \ucf00\uc784\ube0c\ub9ac\uc9c0 \ub300\ud559\uad50\uc758 \u2018\uc625\uc2a4\ube0c\ub9bf\uc9c0(Ox-Bridge)\uc804(\u6230)\u2019\uacfc \uc77c\ubcf8\uc758 \uc640\uc138\ub2e4 \ub300\ud559\uacfc \uac8c\uc774\uc624\uae30\uc8fc\ucfe0 \ub300\ud559\uc758 \u2018\uc18c\ucf00\uc774\uc13c\u2019(\u65e9\u6176\u6226) \ub4f1\uacfc \uc720\uc0ac\ud55c \ud615\ud0dc\uc774\ub2e4. \uace0\uc5f0\uc804\uc740 \ub9e4\ub144 9\uc6d4\uc5d0 \uc5f0\uc138\ub300\ud559\uad50 5\uac1c \uc885\ubaa9\uc758 \uc6b4\ub3d9\ubd80 \uc120\uc218\ub4e4\uacfc \ub9de\ubd99\ub294 \u2018\uc815\uae30 \uace0\uc5f0\uc804\u2019 \uacfc \uc815\uae30 \uace0\uc5f0\uc804 \uc774\uc678\uc758 \uae30\uac04\uc5d0 \uc5f4\ub9ac\ub294 \u2018\ube44\uc815\uae30 \uace0\uc5f0\uc804\u2019 \uc73c\ub85c \ud06c\uac8c \ub098\ub204\uc5b4\ubcfc \uc218 \uc788\uc73c\uba70, \uc774\uc640 \uad00\ub828\ud558\uc5ec \uad50\ub0b4\uc5d0 \ucd95\uad6c\ubd80, \ub7ed\ube44\ubd80, \uc57c\uad6c\ubd80, \ub18d\uad6c\ubd80, \uc544\uc774\uc2a4\ud558\ud0a4\ubd80 \ub4f1 5\uac1c \uc885\ubaa9\uc758 \uc2a4\ud3ec\uce20\ub2e8\uccb4\uac00 \ud65c\ub3d9\ud558\uace0 \uc788\ub2e4. \ud754\ud788 \uc77c\uceec\uc5b4\uc9c0\ub294 \u2018\uace0\uc5f0\uc804\u2019\uc774\ub77c\ub294 \uba85\uce6d\uc740 \u2018\uc815\uae30 \uace0\uc5f0\uc804\u2019\uc744 \uc758\ubbf8\ud558\uba70, \uc544\ub798\uc5d0 \uc774\uc5b4\uc9c0\ub294 \uc124\uba85\ub3c4 \u2018\uc815\uae30 \uace0\uc5f0\uc804\u2019\uc5d0 \uad00\ud55c \uac83\uc774\ub2e4.", "answer": "9\uc6d4", "sentence": "\uace0\uc5f0\uc804\uc740 \ub9e4\ub144 9\uc6d4 \uc5d0 \uc5f0\uc138\ub300\ud559\uad50 5\uac1c \uc885\ubaa9\uc758 \uc6b4\ub3d9\ubd80 \uc120\uc218\ub4e4\uacfc \ub9de\ubd99\ub294 \u2018\uc815\uae30 \uace0\uc5f0\uc804\u2019 \uacfc \uc815\uae30 \uace0\uc5f0\uc804 \uc774\uc678\uc758 \uae30\uac04\uc5d0 \uc5f4\ub9ac\ub294 \u2018\ube44\uc815\uae30 \uace0\uc5f0\uc804\u2019 \uc73c\ub85c \ud06c\uac8c \ub098\ub204\uc5b4\ubcfc \uc218 \uc788\uc73c\uba70, \uc774\uc640 \uad00\ub828\ud558\uc5ec \uad50\ub0b4\uc5d0 \ucd95\uad6c\ubd80, \ub7ed\ube44\ubd80, \uc57c\uad6c\ubd80, \ub18d\uad6c\ubd80, \uc544\uc774\uc2a4\ud558\ud0a4\ubd80 \ub4f1 5\uac1c \uc885\ubaa9\uc758 \uc2a4\ud3ec\uce20\ub2e8\uccb4\uac00 \ud65c\ub3d9\ud558\uace0 \uc788\ub2e4.", "paragraph_sentence": "\uace0\uc5f0\uc804\uc740 \uc560\uad50\uc2ec\uc744 \uace0\uc591\ud558\uace0 \uace0\ub824\ub300\ud559\uad50\uc640 \uc5f0\uc138\ub300\ud559\uad50 \uc591\uad50\uc758 \uce5c\uc120\uc744 \ub3c4\ubaa8\ud558\uae30 \uc704\ud55c \ubaa9\uc801\uc73c\ub85c \uc2a4\ud3ec\uce20 \uacbd\uae30 \ub97c \ube44\ub86f\ud55c \uac15\uc5f0, \uc628\ub77c\uc778 \uac8c\uc784, \uc0ac\ud68c\uacf5\ud5cc\ud65c\ub3d9 \uacfc \uac19\uc740 \ub2e4\uc591\ud55c \ubd84\uc57c\uc5d0\uc11c \uc5f0\uc138\ub300\ud559\uad50\uc640 \uc2b9\ubd80\ub97c \uaca8\ub8e8\ub294 \ud589\uc0ac \uc77c\uccb4\ub97c \uc758\ubbf8\ud55c\ub2e4. \uc601\uad6d\uc758 \uc625\uc2a4\ud3ec\ub4dc \ub300\ud559\uad50\uc640 \ucf00\uc784\ube0c\ub9ac\uc9c0 \ub300\ud559\uad50\uc758 \u2018\uc625\uc2a4\ube0c\ub9bf\uc9c0(Ox-Bridge)\uc804(\u6230)\u2019\uacfc \uc77c\ubcf8\uc758 \uc640\uc138\ub2e4 \ub300\ud559\uacfc \uac8c\uc774\uc624\uae30\uc8fc\ucfe0 \ub300\ud559\uc758 \u2018\uc18c\ucf00\uc774\uc13c\u2019(\u65e9\u6176\u6226) \ub4f1\uacfc \uc720\uc0ac\ud55c \ud615\ud0dc\uc774\ub2e4. \uace0\uc5f0\uc804\uc740 \ub9e4\ub144 9\uc6d4 \uc5d0 \uc5f0\uc138\ub300\ud559\uad50 5\uac1c \uc885\ubaa9\uc758 \uc6b4\ub3d9\ubd80 \uc120\uc218\ub4e4\uacfc \ub9de\ubd99\ub294 \u2018\uc815\uae30 \uace0\uc5f0\uc804\u2019 \uacfc \uc815\uae30 \uace0\uc5f0\uc804 \uc774\uc678\uc758 \uae30\uac04\uc5d0 \uc5f4\ub9ac\ub294 \u2018\ube44\uc815\uae30 \uace0\uc5f0\uc804\u2019 \uc73c\ub85c \ud06c\uac8c \ub098\ub204\uc5b4\ubcfc \uc218 \uc788\uc73c\uba70, \uc774\uc640 \uad00\ub828\ud558\uc5ec \uad50\ub0b4\uc5d0 \ucd95\uad6c\ubd80, \ub7ed\ube44\ubd80, \uc57c\uad6c\ubd80, \ub18d\uad6c\ubd80, \uc544\uc774\uc2a4\ud558\ud0a4\ubd80 \ub4f1 5\uac1c \uc885\ubaa9\uc758 \uc2a4\ud3ec\uce20\ub2e8\uccb4\uac00 \ud65c\ub3d9\ud558\uace0 \uc788\ub2e4. \ud754\ud788 \uc77c\uceec\uc5b4\uc9c0\ub294 \u2018\uace0\uc5f0\uc804\u2019\uc774\ub77c\ub294 \uba85\uce6d\uc740 \u2018\uc815\uae30 \uace0\uc5f0\uc804\u2019\uc744 \uc758\ubbf8\ud558\uba70, \uc544\ub798\uc5d0 \uc774\uc5b4\uc9c0\ub294 \uc124\uba85\ub3c4 \u2018\uc815\uae30 \uace0\uc5f0\uc804\u2019\uc5d0 \uad00\ud55c \uac83\uc774\ub2e4.", "paragraph_answer": "\uace0\uc5f0\uc804\uc740 \uc560\uad50\uc2ec\uc744 \uace0\uc591\ud558\uace0 \uace0\ub824\ub300\ud559\uad50\uc640 \uc5f0\uc138\ub300\ud559\uad50 \uc591\uad50\uc758 \uce5c\uc120\uc744 \ub3c4\ubaa8\ud558\uae30 \uc704\ud55c \ubaa9\uc801\uc73c\ub85c \uc2a4\ud3ec\uce20 \uacbd\uae30 \ub97c \ube44\ub86f\ud55c \uac15\uc5f0, \uc628\ub77c\uc778 \uac8c\uc784, \uc0ac\ud68c\uacf5\ud5cc\ud65c\ub3d9 \uacfc \uac19\uc740 \ub2e4\uc591\ud55c \ubd84\uc57c\uc5d0\uc11c \uc5f0\uc138\ub300\ud559\uad50\uc640 \uc2b9\ubd80\ub97c \uaca8\ub8e8\ub294 \ud589\uc0ac \uc77c\uccb4\ub97c \uc758\ubbf8\ud55c\ub2e4. \uc601\uad6d\uc758 \uc625\uc2a4\ud3ec\ub4dc \ub300\ud559\uad50\uc640 \ucf00\uc784\ube0c\ub9ac\uc9c0 \ub300\ud559\uad50\uc758 \u2018\uc625\uc2a4\ube0c\ub9bf\uc9c0(Ox-Bridge)\uc804(\u6230)\u2019\uacfc \uc77c\ubcf8\uc758 \uc640\uc138\ub2e4 \ub300\ud559\uacfc \uac8c\uc774\uc624\uae30\uc8fc\ucfe0 \ub300\ud559\uc758 \u2018\uc18c\ucf00\uc774\uc13c\u2019(\u65e9\u6176\u6226) \ub4f1\uacfc \uc720\uc0ac\ud55c \ud615\ud0dc\uc774\ub2e4. \uace0\uc5f0\uc804\uc740 \ub9e4\ub144 9\uc6d4 \uc5d0 \uc5f0\uc138\ub300\ud559\uad50 5\uac1c \uc885\ubaa9\uc758 \uc6b4\ub3d9\ubd80 \uc120\uc218\ub4e4\uacfc \ub9de\ubd99\ub294 \u2018\uc815\uae30 \uace0\uc5f0\uc804\u2019 \uacfc \uc815\uae30 \uace0\uc5f0\uc804 \uc774\uc678\uc758 \uae30\uac04\uc5d0 \uc5f4\ub9ac\ub294 \u2018\ube44\uc815\uae30 \uace0\uc5f0\uc804\u2019 \uc73c\ub85c \ud06c\uac8c \ub098\ub204\uc5b4\ubcfc \uc218 \uc788\uc73c\uba70, \uc774\uc640 \uad00\ub828\ud558\uc5ec \uad50\ub0b4\uc5d0 \ucd95\uad6c\ubd80, \ub7ed\ube44\ubd80, \uc57c\uad6c\ubd80, \ub18d\uad6c\ubd80, \uc544\uc774\uc2a4\ud558\ud0a4\ubd80 \ub4f1 5\uac1c \uc885\ubaa9\uc758 \uc2a4\ud3ec\uce20\ub2e8\uccb4\uac00 \ud65c\ub3d9\ud558\uace0 \uc788\ub2e4. \ud754\ud788 \uc77c\uceec\uc5b4\uc9c0\ub294 \u2018\uace0\uc5f0\uc804\u2019\uc774\ub77c\ub294 \uba85\uce6d\uc740 \u2018\uc815\uae30 \uace0\uc5f0\uc804\u2019\uc744 \uc758\ubbf8\ud558\uba70, \uc544\ub798\uc5d0 \uc774\uc5b4\uc9c0\ub294 \uc124\uba85\ub3c4 \u2018\uc815\uae30 \uace0\uc5f0\uc804\u2019\uc5d0 \uad00\ud55c \uac83\uc774\ub2e4.", "sentence_answer": "\uace0\uc5f0\uc804\uc740 \ub9e4\ub144 9\uc6d4 \uc5d0 \uc5f0\uc138\ub300\ud559\uad50 5\uac1c \uc885\ubaa9\uc758 \uc6b4\ub3d9\ubd80 \uc120\uc218\ub4e4\uacfc \ub9de\ubd99\ub294 \u2018\uc815\uae30 \uace0\uc5f0\uc804\u2019 \uacfc \uc815\uae30 \uace0\uc5f0\uc804 \uc774\uc678\uc758 \uae30\uac04\uc5d0 \uc5f4\ub9ac\ub294 \u2018\ube44\uc815\uae30 \uace0\uc5f0\uc804\u2019 \uc73c\ub85c \ud06c\uac8c \ub098\ub204\uc5b4\ubcfc \uc218 \uc788\uc73c\uba70, \uc774\uc640 \uad00\ub828\ud558\uc5ec \uad50\ub0b4\uc5d0 \ucd95\uad6c\ubd80, \ub7ed\ube44\ubd80, \uc57c\uad6c\ubd80, \ub18d\uad6c\ubd80, \uc544\uc774\uc2a4\ud558\ud0a4\ubd80 \ub4f1 5\uac1c \uc885\ubaa9\uc758 \uc2a4\ud3ec\uce20\ub2e8\uccb4\uac00 \ud65c\ub3d9\ud558\uace0 \uc788\ub2e4.", "paragraph_id": "6474998-14-0", "paragraph_question": "question: \uace0\uc5f0\uc804\uc740 \ub9e4\ub144 \uba87\uc6d4\uc5d0 \uc9c4\ud589\ub418\ub294\uac00?, context: \uace0\uc5f0\uc804\uc740 \uc560\uad50\uc2ec\uc744 \uace0\uc591\ud558\uace0 \uace0\ub824\ub300\ud559\uad50\uc640 \uc5f0\uc138\ub300\ud559\uad50 \uc591\uad50\uc758 \uce5c\uc120\uc744 \ub3c4\ubaa8\ud558\uae30 \uc704\ud55c \ubaa9\uc801\uc73c\ub85c \uc2a4\ud3ec\uce20 \uacbd\uae30 \ub97c \ube44\ub86f\ud55c \uac15\uc5f0, \uc628\ub77c\uc778 \uac8c\uc784, \uc0ac\ud68c\uacf5\ud5cc\ud65c\ub3d9 \uacfc \uac19\uc740 \ub2e4\uc591\ud55c \ubd84\uc57c\uc5d0\uc11c \uc5f0\uc138\ub300\ud559\uad50\uc640 \uc2b9\ubd80\ub97c \uaca8\ub8e8\ub294 \ud589\uc0ac \uc77c\uccb4\ub97c \uc758\ubbf8\ud55c\ub2e4. \uc601\uad6d\uc758 \uc625\uc2a4\ud3ec\ub4dc \ub300\ud559\uad50\uc640 \ucf00\uc784\ube0c\ub9ac\uc9c0 \ub300\ud559\uad50\uc758 \u2018\uc625\uc2a4\ube0c\ub9bf\uc9c0(Ox-Bridge)\uc804(\u6230)\u2019\uacfc \uc77c\ubcf8\uc758 \uc640\uc138\ub2e4 \ub300\ud559\uacfc \uac8c\uc774\uc624\uae30\uc8fc\ucfe0 \ub300\ud559\uc758 \u2018\uc18c\ucf00\uc774\uc13c\u2019(\u65e9\u6176\u6226) \ub4f1\uacfc \uc720\uc0ac\ud55c \ud615\ud0dc\uc774\ub2e4. \uace0\uc5f0\uc804\uc740 \ub9e4\ub144 9\uc6d4\uc5d0 \uc5f0\uc138\ub300\ud559\uad50 5\uac1c \uc885\ubaa9\uc758 \uc6b4\ub3d9\ubd80 \uc120\uc218\ub4e4\uacfc \ub9de\ubd99\ub294 \u2018\uc815\uae30 \uace0\uc5f0\uc804\u2019 \uacfc \uc815\uae30 \uace0\uc5f0\uc804 \uc774\uc678\uc758 \uae30\uac04\uc5d0 \uc5f4\ub9ac\ub294 \u2018\ube44\uc815\uae30 \uace0\uc5f0\uc804\u2019 \uc73c\ub85c \ud06c\uac8c \ub098\ub204\uc5b4\ubcfc \uc218 \uc788\uc73c\uba70, \uc774\uc640 \uad00\ub828\ud558\uc5ec \uad50\ub0b4\uc5d0 \ucd95\uad6c\ubd80, \ub7ed\ube44\ubd80, \uc57c\uad6c\ubd80, \ub18d\uad6c\ubd80, \uc544\uc774\uc2a4\ud558\ud0a4\ubd80 \ub4f1 5\uac1c \uc885\ubaa9\uc758 \uc2a4\ud3ec\uce20\ub2e8\uccb4\uac00 \ud65c\ub3d9\ud558\uace0 \uc788\ub2e4. \ud754\ud788 \uc77c\uceec\uc5b4\uc9c0\ub294 \u2018\uace0\uc5f0\uc804\u2019\uc774\ub77c\ub294 \uba85\uce6d\uc740 \u2018\uc815\uae30 \uace0\uc5f0\uc804\u2019\uc744 \uc758\ubbf8\ud558\uba70, \uc544\ub798\uc5d0 \uc774\uc5b4\uc9c0\ub294 \uc124\uba85\ub3c4 \u2018\uc815\uae30 \uace0\uc5f0\uc804\u2019\uc5d0 \uad00\ud55c \uac83\uc774\ub2e4."} {"question": "\ub3c5\uc77c\uc758 \uc0ac\uc6b0\ub098\ub294 \ud63c\ud0d5\uc774\uc9c0\ub9cc, \ud63c\ud0d5\uc774\ub77c\ub294 \uc6a9\uc5b4\uac00 \uc544\ub2cc \uc5b4\ub5a4 \uc6a9\uc5b4\ub97c \uc0ac\uc6a9\ud558\ub294\uac00?", "paragraph": "\ub3c5\uc77c\uc758 \ud63c\uc695 \uc0ac\uc6b0\ub098\ub294 \uc77c\ubcf8\uacfc\ub294 \ub2ec\ub9ac \uaddc\ubaa8\uac00 \ud06c\uba70 \ub0a8\ub140\ub178\uc18c\uac00 \ub9ce\uc774 \uc774\uc6a9\ud55c\ub2e4. \ud2b9\ud788 \ubc14\ub374\ubc14\ub374\uc758 \ud504\ub9ac\ub4dc\ub9ac\ud788 \uc628\ucc9c\uacfc \uce74\ub77c\uce7c\ub77c \uc628\ucc9c\uc774 \uac00\uc7a5 \uc720\uba85\ud55c \ub0a8\ub140 \ud63c\ud0d5 \uc0ac\uc6b0\ub098\uc774\ub2e4. \ub3c5\uc77c\uc758 \uc0ac\uc6b0\ub098\ub294 \ud63c\ud0d5\uc774\uc9c0\ub9cc, \ud63c\ud0d5\uc774\ub77c\ub294 \uba85\uce6d\ubcf4\ub2e4 \u2018\uc0ac\uc6b0\ub098\u2019\ub77c\ub294 \uc6a9\uc5b4\ub97c \uc0ac\uc6a9\ud55c\ub2e4. \uc218\uac74 \ub300\uc5ec\ub8cc\ub294 \ub530\ub85c \uc9c0\ubd88\ud574\uc57c\ud558\uba70 \uc544\uc6b0\ud504\uad6c\uc218(Aufguss)\ub77c\uace0 \ud574\uc11c \ub9e4\uc2dc \uc815\uac01 \ub610\ub294 \ub9e4 30\ubd84, 15\ubd84\ub9c8\ub2e4 \uc0ac\uc6b0\ub098 \ub0b4\ubd80\uc758 \uc628\ub3c4\ub97c \ub192\uc778\ub2e4. \uc77c\ubc18\uc801\uc778 \uac00\uaca9\uc740 2\uc2dc\uac04\ub3d9\uc548 \uc57d 10~12\uc720\ub85c, 4\uc2dc\uac04\ub3d9\uc548 15~17\uc6b0\ub85c, \ud558\ub8e8 \uc774\uc6a9\uad8c\uc740 18~20\uc720\ub85c \uc815\ub3c4 \ucc45\uc815\ub41c\ub2e4. \uac00\uaca9\uc740 \uac1c\ubcc4 \uc0ac\uc6b0\ub098\uc5d0 \ub530\ub77c \ucc28\uc774\uac00 \uc788\uc9c0\ub9cc \ud070 \ucc28\uc774\ub294 \uc5c6\ub2e4. \ub300\ubd80\ubd84\uc758 \ud63c\uc695 \ub204\ub4dc \uc0ac\uc6b0\ub098\uc758 \uacbd\uc6b0\uc5d0\ub294 16\uc138\uc758 \ub098\uc774 \uc81c\ud55c\uc744 \ub454\ub2e4. \uc774\ub294 \uc0ac\uc6b0\ub098\ub97c \ud734\uc2dd \uadf8 \uc790\uccb4\ub85c \uc5ec\uae30\ub294 \uc778\uc2dd\uc774 \ud06c\uae30 \ub54c\ubb38\uc774\ub2e4. \uae30\ubcf8\uc801\uc778 \uc900\ube44\ubb3c\uc5d0\ub294 \uc138\uba74\ub3c4\uad6c\uc640 \ub450 \uc7a5\uc758 \uc218\uac74 \uc815\ub3c4\uc774\ub2e4. \uc774\ub54c \uc218\uac74\uc758 \uc6a9\ub3c4\ub294 \ud55c \uc7a5\uc740 \ubab8\uc744 \ub9d0\ub9ac\uae30 \uc704\ud55c \uac83\uc774\uace0 \ub2e4\ub978 \ud55c \uc7a5\uc740 \uc0ac\uc6b0\ub098 \ub0b4\ubd80\uc5d0\uc11c \ucc29\uc11d\ud560 \ub54c\ub9c8\ub2e4 \uc0ac\uc6a9\ud560 \uae54\uac1c\uc6a9\uc73c\ub85c \uc4f0\uc778\ub2e4.", "answer": "\uc0ac\uc6b0\ub098", "sentence": "\ub3c5\uc77c\uc758 \ud63c\uc695 \uc0ac\uc6b0\ub098 \ub294 \uc77c\ubcf8\uacfc\ub294 \ub2ec\ub9ac \uaddc\ubaa8\uac00 \ud06c\uba70 \ub0a8\ub140\ub178\uc18c\uac00 \ub9ce\uc774 \uc774\uc6a9\ud55c\ub2e4.", "paragraph_sentence": " \ub3c5\uc77c\uc758 \ud63c\uc695 \uc0ac\uc6b0\ub098 \ub294 \uc77c\ubcf8\uacfc\ub294 \ub2ec\ub9ac \uaddc\ubaa8\uac00 \ud06c\uba70 \ub0a8\ub140\ub178\uc18c\uac00 \ub9ce\uc774 \uc774\uc6a9\ud55c\ub2e4. \ud2b9\ud788 \ubc14\ub374\ubc14\ub374\uc758 \ud504\ub9ac\ub4dc\ub9ac\ud788 \uc628\ucc9c\uacfc \uce74\ub77c\uce7c\ub77c \uc628\ucc9c\uc774 \uac00\uc7a5 \uc720\uba85\ud55c \ub0a8\ub140 \ud63c\ud0d5 \uc0ac\uc6b0\ub098\uc774\ub2e4. \ub3c5\uc77c\uc758 \uc0ac\uc6b0\ub098\ub294 \ud63c\ud0d5\uc774\uc9c0\ub9cc, \ud63c\ud0d5\uc774\ub77c\ub294 \uba85\uce6d\ubcf4\ub2e4 \u2018\uc0ac\uc6b0\ub098\u2019\ub77c\ub294 \uc6a9\uc5b4\ub97c \uc0ac\uc6a9\ud55c\ub2e4. \uc218\uac74 \ub300\uc5ec\ub8cc\ub294 \ub530\ub85c \uc9c0\ubd88\ud574\uc57c\ud558\uba70 \uc544\uc6b0\ud504\uad6c\uc218(Aufguss)\ub77c\uace0 \ud574\uc11c \ub9e4\uc2dc \uc815\uac01 \ub610\ub294 \ub9e4 30\ubd84, 15\ubd84\ub9c8\ub2e4 \uc0ac\uc6b0\ub098 \ub0b4\ubd80\uc758 \uc628\ub3c4\ub97c \ub192\uc778\ub2e4. \uc77c\ubc18\uc801\uc778 \uac00\uaca9\uc740 2\uc2dc\uac04\ub3d9\uc548 \uc57d 10~12\uc720\ub85c, 4\uc2dc\uac04\ub3d9\uc548 15~17\uc6b0\ub85c, \ud558\ub8e8 \uc774\uc6a9\uad8c\uc740 18~20\uc720\ub85c \uc815\ub3c4 \ucc45\uc815\ub41c\ub2e4. \uac00\uaca9\uc740 \uac1c\ubcc4 \uc0ac\uc6b0\ub098\uc5d0 \ub530\ub77c \ucc28\uc774\uac00 \uc788\uc9c0\ub9cc \ud070 \ucc28\uc774\ub294 \uc5c6\ub2e4. \ub300\ubd80\ubd84\uc758 \ud63c\uc695 \ub204\ub4dc \uc0ac\uc6b0\ub098\uc758 \uacbd\uc6b0\uc5d0\ub294 16\uc138\uc758 \ub098\uc774 \uc81c\ud55c\uc744 \ub454\ub2e4. \uc774\ub294 \uc0ac\uc6b0\ub098\ub97c \ud734\uc2dd \uadf8 \uc790\uccb4\ub85c \uc5ec\uae30\ub294 \uc778\uc2dd\uc774 \ud06c\uae30 \ub54c\ubb38\uc774\ub2e4. \uae30\ubcf8\uc801\uc778 \uc900\ube44\ubb3c\uc5d0\ub294 \uc138\uba74\ub3c4\uad6c\uc640 \ub450 \uc7a5\uc758 \uc218\uac74 \uc815\ub3c4\uc774\ub2e4. \uc774\ub54c \uc218\uac74\uc758 \uc6a9\ub3c4\ub294 \ud55c \uc7a5\uc740 \ubab8\uc744 \ub9d0\ub9ac\uae30 \uc704\ud55c \uac83\uc774\uace0 \ub2e4\ub978 \ud55c \uc7a5\uc740 \uc0ac\uc6b0\ub098 \ub0b4\ubd80\uc5d0\uc11c \ucc29\uc11d\ud560 \ub54c\ub9c8\ub2e4 \uc0ac\uc6a9\ud560 \uae54\uac1c\uc6a9\uc73c\ub85c \uc4f0\uc778\ub2e4.", "paragraph_answer": "\ub3c5\uc77c\uc758 \ud63c\uc695 \uc0ac\uc6b0\ub098 \ub294 \uc77c\ubcf8\uacfc\ub294 \ub2ec\ub9ac \uaddc\ubaa8\uac00 \ud06c\uba70 \ub0a8\ub140\ub178\uc18c\uac00 \ub9ce\uc774 \uc774\uc6a9\ud55c\ub2e4. \ud2b9\ud788 \ubc14\ub374\ubc14\ub374\uc758 \ud504\ub9ac\ub4dc\ub9ac\ud788 \uc628\ucc9c\uacfc \uce74\ub77c\uce7c\ub77c \uc628\ucc9c\uc774 \uac00\uc7a5 \uc720\uba85\ud55c \ub0a8\ub140 \ud63c\ud0d5 \uc0ac\uc6b0\ub098\uc774\ub2e4. \ub3c5\uc77c\uc758 \uc0ac\uc6b0\ub098\ub294 \ud63c\ud0d5\uc774\uc9c0\ub9cc, \ud63c\ud0d5\uc774\ub77c\ub294 \uba85\uce6d\ubcf4\ub2e4 \u2018\uc0ac\uc6b0\ub098\u2019\ub77c\ub294 \uc6a9\uc5b4\ub97c \uc0ac\uc6a9\ud55c\ub2e4. \uc218\uac74 \ub300\uc5ec\ub8cc\ub294 \ub530\ub85c \uc9c0\ubd88\ud574\uc57c\ud558\uba70 \uc544\uc6b0\ud504\uad6c\uc218(Aufguss)\ub77c\uace0 \ud574\uc11c \ub9e4\uc2dc \uc815\uac01 \ub610\ub294 \ub9e4 30\ubd84, 15\ubd84\ub9c8\ub2e4 \uc0ac\uc6b0\ub098 \ub0b4\ubd80\uc758 \uc628\ub3c4\ub97c \ub192\uc778\ub2e4. \uc77c\ubc18\uc801\uc778 \uac00\uaca9\uc740 2\uc2dc\uac04\ub3d9\uc548 \uc57d 10~12\uc720\ub85c, 4\uc2dc\uac04\ub3d9\uc548 15~17\uc6b0\ub85c, \ud558\ub8e8 \uc774\uc6a9\uad8c\uc740 18~20\uc720\ub85c \uc815\ub3c4 \ucc45\uc815\ub41c\ub2e4. \uac00\uaca9\uc740 \uac1c\ubcc4 \uc0ac\uc6b0\ub098\uc5d0 \ub530\ub77c \ucc28\uc774\uac00 \uc788\uc9c0\ub9cc \ud070 \ucc28\uc774\ub294 \uc5c6\ub2e4. \ub300\ubd80\ubd84\uc758 \ud63c\uc695 \ub204\ub4dc \uc0ac\uc6b0\ub098\uc758 \uacbd\uc6b0\uc5d0\ub294 16\uc138\uc758 \ub098\uc774 \uc81c\ud55c\uc744 \ub454\ub2e4. \uc774\ub294 \uc0ac\uc6b0\ub098\ub97c \ud734\uc2dd \uadf8 \uc790\uccb4\ub85c \uc5ec\uae30\ub294 \uc778\uc2dd\uc774 \ud06c\uae30 \ub54c\ubb38\uc774\ub2e4. \uae30\ubcf8\uc801\uc778 \uc900\ube44\ubb3c\uc5d0\ub294 \uc138\uba74\ub3c4\uad6c\uc640 \ub450 \uc7a5\uc758 \uc218\uac74 \uc815\ub3c4\uc774\ub2e4. \uc774\ub54c \uc218\uac74\uc758 \uc6a9\ub3c4\ub294 \ud55c \uc7a5\uc740 \ubab8\uc744 \ub9d0\ub9ac\uae30 \uc704\ud55c \uac83\uc774\uace0 \ub2e4\ub978 \ud55c \uc7a5\uc740 \uc0ac\uc6b0\ub098 \ub0b4\ubd80\uc5d0\uc11c \ucc29\uc11d\ud560 \ub54c\ub9c8\ub2e4 \uc0ac\uc6a9\ud560 \uae54\uac1c\uc6a9\uc73c\ub85c \uc4f0\uc778\ub2e4.", "sentence_answer": "\ub3c5\uc77c\uc758 \ud63c\uc695 \uc0ac\uc6b0\ub098 \ub294 \uc77c\ubcf8\uacfc\ub294 \ub2ec\ub9ac \uaddc\ubaa8\uac00 \ud06c\uba70 \ub0a8\ub140\ub178\uc18c\uac00 \ub9ce\uc774 \uc774\uc6a9\ud55c\ub2e4.", "paragraph_id": "6578532-3-0", "paragraph_question": "question: \ub3c5\uc77c\uc758 \uc0ac\uc6b0\ub098\ub294 \ud63c\ud0d5\uc774\uc9c0\ub9cc, \ud63c\ud0d5\uc774\ub77c\ub294 \uc6a9\uc5b4\uac00 \uc544\ub2cc \uc5b4\ub5a4 \uc6a9\uc5b4\ub97c \uc0ac\uc6a9\ud558\ub294\uac00?, context: \ub3c5\uc77c\uc758 \ud63c\uc695 \uc0ac\uc6b0\ub098\ub294 \uc77c\ubcf8\uacfc\ub294 \ub2ec\ub9ac \uaddc\ubaa8\uac00 \ud06c\uba70 \ub0a8\ub140\ub178\uc18c\uac00 \ub9ce\uc774 \uc774\uc6a9\ud55c\ub2e4. \ud2b9\ud788 \ubc14\ub374\ubc14\ub374\uc758 \ud504\ub9ac\ub4dc\ub9ac\ud788 \uc628\ucc9c\uacfc \uce74\ub77c\uce7c\ub77c \uc628\ucc9c\uc774 \uac00\uc7a5 \uc720\uba85\ud55c \ub0a8\ub140 \ud63c\ud0d5 \uc0ac\uc6b0\ub098\uc774\ub2e4. \ub3c5\uc77c\uc758 \uc0ac\uc6b0\ub098\ub294 \ud63c\ud0d5\uc774\uc9c0\ub9cc, \ud63c\ud0d5\uc774\ub77c\ub294 \uba85\uce6d\ubcf4\ub2e4 \u2018\uc0ac\uc6b0\ub098\u2019\ub77c\ub294 \uc6a9\uc5b4\ub97c \uc0ac\uc6a9\ud55c\ub2e4. \uc218\uac74 \ub300\uc5ec\ub8cc\ub294 \ub530\ub85c \uc9c0\ubd88\ud574\uc57c\ud558\uba70 \uc544\uc6b0\ud504\uad6c\uc218(Aufguss)\ub77c\uace0 \ud574\uc11c \ub9e4\uc2dc \uc815\uac01 \ub610\ub294 \ub9e4 30\ubd84, 15\ubd84\ub9c8\ub2e4 \uc0ac\uc6b0\ub098 \ub0b4\ubd80\uc758 \uc628\ub3c4\ub97c \ub192\uc778\ub2e4. \uc77c\ubc18\uc801\uc778 \uac00\uaca9\uc740 2\uc2dc\uac04\ub3d9\uc548 \uc57d 10~12\uc720\ub85c, 4\uc2dc\uac04\ub3d9\uc548 15~17\uc6b0\ub85c, \ud558\ub8e8 \uc774\uc6a9\uad8c\uc740 18~20\uc720\ub85c \uc815\ub3c4 \ucc45\uc815\ub41c\ub2e4. \uac00\uaca9\uc740 \uac1c\ubcc4 \uc0ac\uc6b0\ub098\uc5d0 \ub530\ub77c \ucc28\uc774\uac00 \uc788\uc9c0\ub9cc \ud070 \ucc28\uc774\ub294 \uc5c6\ub2e4. \ub300\ubd80\ubd84\uc758 \ud63c\uc695 \ub204\ub4dc \uc0ac\uc6b0\ub098\uc758 \uacbd\uc6b0\uc5d0\ub294 16\uc138\uc758 \ub098\uc774 \uc81c\ud55c\uc744 \ub454\ub2e4. \uc774\ub294 \uc0ac\uc6b0\ub098\ub97c \ud734\uc2dd \uadf8 \uc790\uccb4\ub85c \uc5ec\uae30\ub294 \uc778\uc2dd\uc774 \ud06c\uae30 \ub54c\ubb38\uc774\ub2e4. \uae30\ubcf8\uc801\uc778 \uc900\ube44\ubb3c\uc5d0\ub294 \uc138\uba74\ub3c4\uad6c\uc640 \ub450 \uc7a5\uc758 \uc218\uac74 \uc815\ub3c4\uc774\ub2e4. \uc774\ub54c \uc218\uac74\uc758 \uc6a9\ub3c4\ub294 \ud55c \uc7a5\uc740 \ubab8\uc744 \ub9d0\ub9ac\uae30 \uc704\ud55c \uac83\uc774\uace0 \ub2e4\ub978 \ud55c \uc7a5\uc740 \uc0ac\uc6b0\ub098 \ub0b4\ubd80\uc5d0\uc11c \ucc29\uc11d\ud560 \ub54c\ub9c8\ub2e4 \uc0ac\uc6a9\ud560 \uae54\uac1c\uc6a9\uc73c\ub85c \uc4f0\uc778\ub2e4."} {"question": "\uc5ec\uc131 \ubb38\uc81c\ub97c \uace0\ubbfc\ud558\uace0 \ub0a8\uc131\uc911\uc2ec\uc801 \ub300\ud559 \ubb38\ud654\ub97c \uc7ac\uad6c\uc131\ud558\uae30 \uc704\ud574 \ud65c\ub3d9\ud558\ub294 \uae30\uad00\uc740?", "paragraph": "\ucd1d\uc5ec\ud559\uc0dd\ud68c\ub294 \uc5ec\uc131 \ubb38\uc81c\ub97c \uace0\ubbfc\ud558\uace0 \ub0a8\uc131\uc911\uc2ec\uc801 \ub300\ud559 \ubb38\ud654\ub97c \uc5ec\uc131\uc8fc\uc758\uc801\uc73c\ub85c \uc7ac\uad6c\uc131\ud558\uae30 \uc704\ud574\uc11c \ud65c\ub3d9\ud558\uace0 \uc788\ub2e4. \ucd1d\uc5ec\ud559\uc0dd\ud68c\ub294 1988\ub144 3\uc6d4\uc5d0 \uae30\uc874 \ucd1d\ud559\uc0dd\ud68c \uc0b0\ud558 \uc5ec\ud559\uc0dd\ubd80\uc758 \ud55c\uacc4\uc640 \ud559\ub0b4\uc5d0\uc11c \uc18c\uc218\uc758 \uc704\uce58\uc5d0 \uc788\ub358 \uc5ec\ud559\uc0dd \ubb38\uc81c\ub97c \uadf9\ubcf5\ud558\uace0\uc790 \ubc1c\uc871\ud558\uc600\ub2e4. \ub9e4\ub144 \uc120\uac70\ub97c \ud1b5\ud574 \ucd1d\uc5ec\ud559\uc0dd\ud68c\uac00 \uc120\ucd9c\ub418\uace0 \uc788\uc73c\uba70 2016\ub144 \uae30\uc900\uc73c\ub85c \uc81c27\ub300 \ucd1d\uc5ec\ud559\uc0dd\ud68c \u2018\uc787\ub2e4\u2019\uac00 \ud65c\ub3d9\ud558\uace0 \uc788\ub2e4. \ucd1d\uc5ec\ud559\uc0dd\ud68c\ub294 \ud0c4\uc0dd \uc774\ub798 \uc704\uc0c1\uc774 \ubaa8\ud638\ud558\ub2e4\ub294 \ube44\ud310\uc744 \ubc1b\uc544\uc654\uc9c0\ub9cc \ub9e4\ub144 \ub0a8\uc131 \uc911\uc2ec\uc758 \ucea0\ud37c\uc2a4 \ubb38\ud654\ub97c \uadf9\ubcf5\ud558\uae30 \uc704\ud55c \ud589\uc0ac\ub97c \uc5f4\uc5b4\uc654\uace0 2000\ub144 2\uc6d4\ubd80\ud130\ub294 \u2018\uc0c8\ub0b4\uae30\ubc30\uc6c0\ud130 \ubc18\uc131\ud3ed\ub825 \ub0b4\uaddc\u2019\ub97c \uc81c\uc815\ud574 \uc2dc\ud589\ud558\uae30\ub3c4 \ud558\uc600\ub2e4. \uc774\uc640 \ub354\ubd88\uc5b4 \uc5ec\ud559\uc0dd\ucc98\ub294 1956\ub144\uc5d0 \uc124\ub9bd\ub41c \uc774\ub798 \uc131\ud3ed\ub825\uc0c1\ub2f4\uc2e4 \uac1c\uc124, \ubc18\uc131\ud3ed\ub825\ud559\uce59 \uc81c\uc815 \ub4f1 \ub0a8\ub140\ud3c9\ub4f1\uc801 \uad50\uc721 \ud658\uacbd\uc744 \ub9c8\ub828\ud558\uae30 \uc704\ud574 \ub178\ub825\ud558\uc600\ub2e4. 2002\ub144\uc5d0\ub294 \uc5ec\uc131\uc778\ub825\uac1c\ubc1c\uc5f0\uad6c\uc6d0\uc774 \ubc1c\uc871\ub418\uc5b4 \uc5ec\ud559\uc0dd\ub4e4\uc758 \uc0ac\ud68c \uc9c4\ucd9c \ub4f1\uc744 \uc704\ud55c \ud559\ub0b4\uc678\uc801\uc778 \ud65c\ub3d9\uc744 \uc218\ud589\ud558\uace0 \uc788\ub2e4.", "answer": "\ucd1d\uc5ec\ud559\uc0dd\ud68c", "sentence": "\ucd1d\uc5ec\ud559\uc0dd\ud68c \ub294 \uc5ec\uc131 \ubb38\uc81c\ub97c \uace0\ubbfc\ud558\uace0 \ub0a8\uc131\uc911\uc2ec\uc801 \ub300\ud559 \ubb38\ud654\ub97c \uc5ec\uc131\uc8fc\uc758\uc801\uc73c\ub85c \uc7ac\uad6c\uc131\ud558\uae30 \uc704\ud574\uc11c \ud65c\ub3d9\ud558\uace0 \uc788\ub2e4.", "paragraph_sentence": " \ucd1d\uc5ec\ud559\uc0dd\ud68c \ub294 \uc5ec\uc131 \ubb38\uc81c\ub97c \uace0\ubbfc\ud558\uace0 \ub0a8\uc131\uc911\uc2ec\uc801 \ub300\ud559 \ubb38\ud654\ub97c \uc5ec\uc131\uc8fc\uc758\uc801\uc73c\ub85c \uc7ac\uad6c\uc131\ud558\uae30 \uc704\ud574\uc11c \ud65c\ub3d9\ud558\uace0 \uc788\ub2e4. \ucd1d\uc5ec\ud559\uc0dd\ud68c\ub294 1988\ub144 3\uc6d4\uc5d0 \uae30\uc874 \ucd1d\ud559\uc0dd\ud68c \uc0b0\ud558 \uc5ec\ud559\uc0dd\ubd80\uc758 \ud55c\uacc4\uc640 \ud559\ub0b4\uc5d0\uc11c \uc18c\uc218\uc758 \uc704\uce58\uc5d0 \uc788\ub358 \uc5ec\ud559\uc0dd \ubb38\uc81c\ub97c \uadf9\ubcf5\ud558\uace0\uc790 \ubc1c\uc871\ud558\uc600\ub2e4. \ub9e4\ub144 \uc120\uac70\ub97c \ud1b5\ud574 \ucd1d\uc5ec\ud559\uc0dd\ud68c\uac00 \uc120\ucd9c\ub418\uace0 \uc788\uc73c\uba70 2016\ub144 \uae30\uc900\uc73c\ub85c \uc81c27\ub300 \ucd1d\uc5ec\ud559\uc0dd\ud68c \u2018\uc787\ub2e4\u2019\uac00 \ud65c\ub3d9\ud558\uace0 \uc788\ub2e4. \ucd1d\uc5ec\ud559\uc0dd\ud68c\ub294 \ud0c4\uc0dd \uc774\ub798 \uc704\uc0c1\uc774 \ubaa8\ud638\ud558\ub2e4\ub294 \ube44\ud310\uc744 \ubc1b\uc544\uc654\uc9c0\ub9cc \ub9e4\ub144 \ub0a8\uc131 \uc911\uc2ec\uc758 \ucea0\ud37c\uc2a4 \ubb38\ud654\ub97c \uadf9\ubcf5\ud558\uae30 \uc704\ud55c \ud589\uc0ac\ub97c \uc5f4\uc5b4\uc654\uace0 2000\ub144 2\uc6d4\ubd80\ud130\ub294 \u2018\uc0c8\ub0b4\uae30\ubc30\uc6c0\ud130 \ubc18\uc131\ud3ed\ub825 \ub0b4\uaddc\u2019\ub97c \uc81c\uc815\ud574 \uc2dc\ud589\ud558\uae30\ub3c4 \ud558\uc600\ub2e4. \uc774\uc640 \ub354\ubd88\uc5b4 \uc5ec\ud559\uc0dd\ucc98\ub294 1956\ub144\uc5d0 \uc124\ub9bd\ub41c \uc774\ub798 \uc131\ud3ed\ub825\uc0c1\ub2f4\uc2e4 \uac1c\uc124, \ubc18\uc131\ud3ed\ub825\ud559\uce59 \uc81c\uc815 \ub4f1 \ub0a8\ub140\ud3c9\ub4f1\uc801 \uad50\uc721 \ud658\uacbd\uc744 \ub9c8\ub828\ud558\uae30 \uc704\ud574 \ub178\ub825\ud558\uc600\ub2e4. 2002\ub144\uc5d0\ub294 \uc5ec\uc131\uc778\ub825\uac1c\ubc1c\uc5f0\uad6c\uc6d0\uc774 \ubc1c\uc871\ub418\uc5b4 \uc5ec\ud559\uc0dd\ub4e4\uc758 \uc0ac\ud68c \uc9c4\ucd9c \ub4f1\uc744 \uc704\ud55c \ud559\ub0b4\uc678\uc801\uc778 \ud65c\ub3d9\uc744 \uc218\ud589\ud558\uace0 \uc788\ub2e4.", "paragraph_answer": " \ucd1d\uc5ec\ud559\uc0dd\ud68c \ub294 \uc5ec\uc131 \ubb38\uc81c\ub97c \uace0\ubbfc\ud558\uace0 \ub0a8\uc131\uc911\uc2ec\uc801 \ub300\ud559 \ubb38\ud654\ub97c \uc5ec\uc131\uc8fc\uc758\uc801\uc73c\ub85c \uc7ac\uad6c\uc131\ud558\uae30 \uc704\ud574\uc11c \ud65c\ub3d9\ud558\uace0 \uc788\ub2e4. \ucd1d\uc5ec\ud559\uc0dd\ud68c\ub294 1988\ub144 3\uc6d4\uc5d0 \uae30\uc874 \ucd1d\ud559\uc0dd\ud68c \uc0b0\ud558 \uc5ec\ud559\uc0dd\ubd80\uc758 \ud55c\uacc4\uc640 \ud559\ub0b4\uc5d0\uc11c \uc18c\uc218\uc758 \uc704\uce58\uc5d0 \uc788\ub358 \uc5ec\ud559\uc0dd \ubb38\uc81c\ub97c \uadf9\ubcf5\ud558\uace0\uc790 \ubc1c\uc871\ud558\uc600\ub2e4. \ub9e4\ub144 \uc120\uac70\ub97c \ud1b5\ud574 \ucd1d\uc5ec\ud559\uc0dd\ud68c\uac00 \uc120\ucd9c\ub418\uace0 \uc788\uc73c\uba70 2016\ub144 \uae30\uc900\uc73c\ub85c \uc81c27\ub300 \ucd1d\uc5ec\ud559\uc0dd\ud68c \u2018\uc787\ub2e4\u2019\uac00 \ud65c\ub3d9\ud558\uace0 \uc788\ub2e4. \ucd1d\uc5ec\ud559\uc0dd\ud68c\ub294 \ud0c4\uc0dd \uc774\ub798 \uc704\uc0c1\uc774 \ubaa8\ud638\ud558\ub2e4\ub294 \ube44\ud310\uc744 \ubc1b\uc544\uc654\uc9c0\ub9cc \ub9e4\ub144 \ub0a8\uc131 \uc911\uc2ec\uc758 \ucea0\ud37c\uc2a4 \ubb38\ud654\ub97c \uadf9\ubcf5\ud558\uae30 \uc704\ud55c \ud589\uc0ac\ub97c \uc5f4\uc5b4\uc654\uace0 2000\ub144 2\uc6d4\ubd80\ud130\ub294 \u2018\uc0c8\ub0b4\uae30\ubc30\uc6c0\ud130 \ubc18\uc131\ud3ed\ub825 \ub0b4\uaddc\u2019\ub97c \uc81c\uc815\ud574 \uc2dc\ud589\ud558\uae30\ub3c4 \ud558\uc600\ub2e4. \uc774\uc640 \ub354\ubd88\uc5b4 \uc5ec\ud559\uc0dd\ucc98\ub294 1956\ub144\uc5d0 \uc124\ub9bd\ub41c \uc774\ub798 \uc131\ud3ed\ub825\uc0c1\ub2f4\uc2e4 \uac1c\uc124, \ubc18\uc131\ud3ed\ub825\ud559\uce59 \uc81c\uc815 \ub4f1 \ub0a8\ub140\ud3c9\ub4f1\uc801 \uad50\uc721 \ud658\uacbd\uc744 \ub9c8\ub828\ud558\uae30 \uc704\ud574 \ub178\ub825\ud558\uc600\ub2e4. 2002\ub144\uc5d0\ub294 \uc5ec\uc131\uc778\ub825\uac1c\ubc1c\uc5f0\uad6c\uc6d0\uc774 \ubc1c\uc871\ub418\uc5b4 \uc5ec\ud559\uc0dd\ub4e4\uc758 \uc0ac\ud68c \uc9c4\ucd9c \ub4f1\uc744 \uc704\ud55c \ud559\ub0b4\uc678\uc801\uc778 \ud65c\ub3d9\uc744 \uc218\ud589\ud558\uace0 \uc788\ub2e4.", "sentence_answer": " \ucd1d\uc5ec\ud559\uc0dd\ud68c \ub294 \uc5ec\uc131 \ubb38\uc81c\ub97c \uace0\ubbfc\ud558\uace0 \ub0a8\uc131\uc911\uc2ec\uc801 \ub300\ud559 \ubb38\ud654\ub97c \uc5ec\uc131\uc8fc\uc758\uc801\uc73c\ub85c \uc7ac\uad6c\uc131\ud558\uae30 \uc704\ud574\uc11c \ud65c\ub3d9\ud558\uace0 \uc788\ub2e4.", "paragraph_id": "6541057-12-0", "paragraph_question": "question: \uc5ec\uc131 \ubb38\uc81c\ub97c \uace0\ubbfc\ud558\uace0 \ub0a8\uc131\uc911\uc2ec\uc801 \ub300\ud559 \ubb38\ud654\ub97c \uc7ac\uad6c\uc131\ud558\uae30 \uc704\ud574 \ud65c\ub3d9\ud558\ub294 \uae30\uad00\uc740?, context: \ucd1d\uc5ec\ud559\uc0dd\ud68c\ub294 \uc5ec\uc131 \ubb38\uc81c\ub97c \uace0\ubbfc\ud558\uace0 \ub0a8\uc131\uc911\uc2ec\uc801 \ub300\ud559 \ubb38\ud654\ub97c \uc5ec\uc131\uc8fc\uc758\uc801\uc73c\ub85c \uc7ac\uad6c\uc131\ud558\uae30 \uc704\ud574\uc11c \ud65c\ub3d9\ud558\uace0 \uc788\ub2e4. \ucd1d\uc5ec\ud559\uc0dd\ud68c\ub294 1988\ub144 3\uc6d4\uc5d0 \uae30\uc874 \ucd1d\ud559\uc0dd\ud68c \uc0b0\ud558 \uc5ec\ud559\uc0dd\ubd80\uc758 \ud55c\uacc4\uc640 \ud559\ub0b4\uc5d0\uc11c \uc18c\uc218\uc758 \uc704\uce58\uc5d0 \uc788\ub358 \uc5ec\ud559\uc0dd \ubb38\uc81c\ub97c \uadf9\ubcf5\ud558\uace0\uc790 \ubc1c\uc871\ud558\uc600\ub2e4. \ub9e4\ub144 \uc120\uac70\ub97c \ud1b5\ud574 \ucd1d\uc5ec\ud559\uc0dd\ud68c\uac00 \uc120\ucd9c\ub418\uace0 \uc788\uc73c\uba70 2016\ub144 \uae30\uc900\uc73c\ub85c \uc81c27\ub300 \ucd1d\uc5ec\ud559\uc0dd\ud68c \u2018\uc787\ub2e4\u2019\uac00 \ud65c\ub3d9\ud558\uace0 \uc788\ub2e4. \ucd1d\uc5ec\ud559\uc0dd\ud68c\ub294 \ud0c4\uc0dd \uc774\ub798 \uc704\uc0c1\uc774 \ubaa8\ud638\ud558\ub2e4\ub294 \ube44\ud310\uc744 \ubc1b\uc544\uc654\uc9c0\ub9cc \ub9e4\ub144 \ub0a8\uc131 \uc911\uc2ec\uc758 \ucea0\ud37c\uc2a4 \ubb38\ud654\ub97c \uadf9\ubcf5\ud558\uae30 \uc704\ud55c \ud589\uc0ac\ub97c \uc5f4\uc5b4\uc654\uace0 2000\ub144 2\uc6d4\ubd80\ud130\ub294 \u2018\uc0c8\ub0b4\uae30\ubc30\uc6c0\ud130 \ubc18\uc131\ud3ed\ub825 \ub0b4\uaddc\u2019\ub97c \uc81c\uc815\ud574 \uc2dc\ud589\ud558\uae30\ub3c4 \ud558\uc600\ub2e4. \uc774\uc640 \ub354\ubd88\uc5b4 \uc5ec\ud559\uc0dd\ucc98\ub294 1956\ub144\uc5d0 \uc124\ub9bd\ub41c \uc774\ub798 \uc131\ud3ed\ub825\uc0c1\ub2f4\uc2e4 \uac1c\uc124, \ubc18\uc131\ud3ed\ub825\ud559\uce59 \uc81c\uc815 \ub4f1 \ub0a8\ub140\ud3c9\ub4f1\uc801 \uad50\uc721 \ud658\uacbd\uc744 \ub9c8\ub828\ud558\uae30 \uc704\ud574 \ub178\ub825\ud558\uc600\ub2e4. 2002\ub144\uc5d0\ub294 \uc5ec\uc131\uc778\ub825\uac1c\ubc1c\uc5f0\uad6c\uc6d0\uc774 \ubc1c\uc871\ub418\uc5b4 \uc5ec\ud559\uc0dd\ub4e4\uc758 \uc0ac\ud68c \uc9c4\ucd9c \ub4f1\uc744 \uc704\ud55c \ud559\ub0b4\uc678\uc801\uc778 \ud65c\ub3d9\uc744 \uc218\ud589\ud558\uace0 \uc788\ub2e4."} {"question": "\ubbf8\uad6d\uc774 \ud0dc\ud3c9\uc591 \uc804\uc7c1\uc5d0 \ucc38\uc5ec\ud558\uac8c \ub41c \uacc4\uae30\ub294?", "paragraph": "1941\ub144 12\uc6d4 7\uc77c\uc5d0 \uc77c\ubcf8\uad70\uc740 \ud558\uc640\uc774\uc5d0 \uc8fc\ub454\ud55c \ubbf8\ud574\uad70 \ud0dc\ud3c9\uc591 \ud568\ub300\uc5d0 \ub300\ud558\uc5ec \uc9c4\uc8fc\ub9cc \uacf5\uaca9\uc744 \uac10\ud589\ud558\uc600\ub2e4. \uc9c4\uc8fc\ub9cc \uacf5\uaca9\uc73c\ub85c \ub9ce\uc740 \ubbf8\uad6d \uc804\ud22c\ud568\ub4e4\uc774 \uc190\uc0c1\uc744 \uc785\uc5c8\uace0 \ubbf8\uad6d\uc774 \uacf5\uc2dd\uc801\uc73c\ub85c \uc804\uc7c1\uc5d0 \ucc38\uac00\ud558\uac8c \ub41c\ub2e4. \uc77c\ubcf8\uad70 \uc9c0\ud718\ubd80\ub294 \uc9c4\uc8fc\ub9cc \uacf5\uaca9\uc744 \ud1b5\ud558\uc5ec \ubbf8\ud574\uad70\uc744 \ubb34\ub825\ud654\uc2dc\ud0a4\uace0 \ucc9c\uc5f0\uc790\uc6d0\uc5d0 \ub300\ud55c \uc811\uadfc\uad8c\uc744 \ud655\ubcf4\ud558\uba70, \ud0dc\ud3c9\uc591\uacfc \uc544\uc2dc\uc544 \uc9c0\uc5ed\uc5d0\uc11c \ubbf8\uad70\uc758 \uacf5\uaca9\uc5d0 \ub300\ud56d\ud55c \ubc29\uc5b4 \uac70\uc810\uc744 \ud68d\ub4f1\ud558\uace0\uc790 \ud558\uc600\ub2e4. \uc774\uc640 \uac19\uc740 \ubaa9\uc801 \ud558\uc5d0 \uc77c\ubcf8\uad70\uc740 \ud544\ub9ac\ud540, \ud0c0\uc774, \ub9d0\ub808\uc774 \ubc18\ub3c4, \uc2f1\uac00\ud3ec\ub974, \ubc84\ub9c8, \ub124\ub35c\ub780\ub4dc\ub839 \ub3d9\uc778\ub3c4, \uc6e8\uc774\ud06c \uc12c, \uae38\ubc84\ud2b8\uc12c, \ub274\ube0c\ub9ac\ud2bc \uc12c, \uad0c \ub97c \uc21c\ucc28\uc801\uc73c\ub85c \uacf5\uaca9\ud558\uc5ec \uc810\ub839\ud558\uc600\uc73c\uba70, \ud574\ub2f9 \uc9c0\uc5ed\uc5d0 \ub300\ud55c \uc2dd\ubbfc \uc9c0\ubc30\uad8c\uc744 \ud589\uc0ac\ud558\uace0 \uc788\ub358 \uc601\uad6d, \ud638\uc8fc, \ub124\ub378\ub780\ub4dc\uc5d0 \uc774\uc5b4 \ubbf8\uad6d\uc774 \ud0dc\ud3c9\uc591 \uc804\uc7c1\uc5d0 \uacf5\uc2dd\uc801\uc73c\ub85c \ucc38\uc804\ud558\uac8c \ub41c\ub2e4.", "answer": "\uc9c4\uc8fc\ub9cc \uacf5\uaca9", "sentence": "1941\ub144 12\uc6d4 7\uc77c\uc5d0 \uc77c\ubcf8\uad70\uc740 \ud558\uc640\uc774\uc5d0 \uc8fc\ub454\ud55c \ubbf8\ud574\uad70 \ud0dc\ud3c9\uc591 \ud568\ub300\uc5d0 \ub300\ud558\uc5ec \uc9c4\uc8fc\ub9cc \uacf5\uaca9 \uc744", "paragraph_sentence": " 1941\ub144 12\uc6d4 7\uc77c\uc5d0 \uc77c\ubcf8\uad70\uc740 \ud558\uc640\uc774\uc5d0 \uc8fc\ub454\ud55c \ubbf8\ud574\uad70 \ud0dc\ud3c9\uc591 \ud568\ub300\uc5d0 \ub300\ud558\uc5ec \uc9c4\uc8fc\ub9cc \uacf5\uaca9 \uc744 \uac10\ud589\ud558\uc600\ub2e4. \uc9c4\uc8fc\ub9cc \uacf5\uaca9\uc73c\ub85c \ub9ce\uc740 \ubbf8\uad6d \uc804\ud22c\ud568\ub4e4\uc774 \uc190\uc0c1\uc744 \uc785\uc5c8\uace0 \ubbf8\uad6d\uc774 \uacf5\uc2dd\uc801\uc73c\ub85c \uc804\uc7c1\uc5d0 \ucc38\uac00\ud558\uac8c \ub41c\ub2e4. \uc77c\ubcf8\uad70 \uc9c0\ud718\ubd80\ub294 \uc9c4\uc8fc\ub9cc \uacf5\uaca9\uc744 \ud1b5\ud558\uc5ec \ubbf8\ud574\uad70\uc744 \ubb34\ub825\ud654\uc2dc\ud0a4\uace0 \ucc9c\uc5f0\uc790\uc6d0\uc5d0 \ub300\ud55c \uc811\uadfc\uad8c\uc744 \ud655\ubcf4\ud558\uba70, \ud0dc\ud3c9\uc591\uacfc \uc544\uc2dc\uc544 \uc9c0\uc5ed\uc5d0\uc11c \ubbf8\uad70\uc758 \uacf5\uaca9\uc5d0 \ub300\ud56d\ud55c \ubc29\uc5b4 \uac70\uc810\uc744 \ud68d\ub4f1\ud558\uace0\uc790 \ud558\uc600\ub2e4. \uc774\uc640 \uac19\uc740 \ubaa9\uc801 \ud558\uc5d0 \uc77c\ubcf8\uad70\uc740 \ud544\ub9ac\ud540, \ud0c0\uc774, \ub9d0\ub808\uc774 \ubc18\ub3c4, \uc2f1\uac00\ud3ec\ub974, \ubc84\ub9c8, \ub124\ub35c\ub780\ub4dc\ub839 \ub3d9\uc778\ub3c4, \uc6e8\uc774\ud06c \uc12c, \uae38\ubc84\ud2b8\uc12c, \ub274\ube0c\ub9ac\ud2bc \uc12c, \uad0c \ub97c \uc21c\ucc28\uc801\uc73c\ub85c \uacf5\uaca9\ud558\uc5ec \uc810\ub839\ud558\uc600\uc73c\uba70, \ud574\ub2f9 \uc9c0\uc5ed\uc5d0 \ub300\ud55c \uc2dd\ubbfc \uc9c0\ubc30\uad8c\uc744 \ud589\uc0ac\ud558\uace0 \uc788\ub358 \uc601\uad6d, \ud638\uc8fc, \ub124\ub378\ub780\ub4dc\uc5d0 \uc774\uc5b4 \ubbf8\uad6d\uc774 \ud0dc\ud3c9\uc591 \uc804\uc7c1\uc5d0 \uacf5\uc2dd\uc801\uc73c\ub85c \ucc38\uc804\ud558\uac8c \ub41c\ub2e4.", "paragraph_answer": "1941\ub144 12\uc6d4 7\uc77c\uc5d0 \uc77c\ubcf8\uad70\uc740 \ud558\uc640\uc774\uc5d0 \uc8fc\ub454\ud55c \ubbf8\ud574\uad70 \ud0dc\ud3c9\uc591 \ud568\ub300\uc5d0 \ub300\ud558\uc5ec \uc9c4\uc8fc\ub9cc \uacf5\uaca9 \uc744 \uac10\ud589\ud558\uc600\ub2e4. \uc9c4\uc8fc\ub9cc \uacf5\uaca9\uc73c\ub85c \ub9ce\uc740 \ubbf8\uad6d \uc804\ud22c\ud568\ub4e4\uc774 \uc190\uc0c1\uc744 \uc785\uc5c8\uace0 \ubbf8\uad6d\uc774 \uacf5\uc2dd\uc801\uc73c\ub85c \uc804\uc7c1\uc5d0 \ucc38\uac00\ud558\uac8c \ub41c\ub2e4. \uc77c\ubcf8\uad70 \uc9c0\ud718\ubd80\ub294 \uc9c4\uc8fc\ub9cc \uacf5\uaca9\uc744 \ud1b5\ud558\uc5ec \ubbf8\ud574\uad70\uc744 \ubb34\ub825\ud654\uc2dc\ud0a4\uace0 \ucc9c\uc5f0\uc790\uc6d0\uc5d0 \ub300\ud55c \uc811\uadfc\uad8c\uc744 \ud655\ubcf4\ud558\uba70, \ud0dc\ud3c9\uc591\uacfc \uc544\uc2dc\uc544 \uc9c0\uc5ed\uc5d0\uc11c \ubbf8\uad70\uc758 \uacf5\uaca9\uc5d0 \ub300\ud56d\ud55c \ubc29\uc5b4 \uac70\uc810\uc744 \ud68d\ub4f1\ud558\uace0\uc790 \ud558\uc600\ub2e4. \uc774\uc640 \uac19\uc740 \ubaa9\uc801 \ud558\uc5d0 \uc77c\ubcf8\uad70\uc740 \ud544\ub9ac\ud540, \ud0c0\uc774, \ub9d0\ub808\uc774 \ubc18\ub3c4, \uc2f1\uac00\ud3ec\ub974, \ubc84\ub9c8, \ub124\ub35c\ub780\ub4dc\ub839 \ub3d9\uc778\ub3c4, \uc6e8\uc774\ud06c \uc12c, \uae38\ubc84\ud2b8\uc12c, \ub274\ube0c\ub9ac\ud2bc \uc12c, \uad0c \ub97c \uc21c\ucc28\uc801\uc73c\ub85c \uacf5\uaca9\ud558\uc5ec \uc810\ub839\ud558\uc600\uc73c\uba70, \ud574\ub2f9 \uc9c0\uc5ed\uc5d0 \ub300\ud55c \uc2dd\ubbfc \uc9c0\ubc30\uad8c\uc744 \ud589\uc0ac\ud558\uace0 \uc788\ub358 \uc601\uad6d, \ud638\uc8fc, \ub124\ub378\ub780\ub4dc\uc5d0 \uc774\uc5b4 \ubbf8\uad6d\uc774 \ud0dc\ud3c9\uc591 \uc804\uc7c1\uc5d0 \uacf5\uc2dd\uc801\uc73c\ub85c \ucc38\uc804\ud558\uac8c \ub41c\ub2e4.", "sentence_answer": "1941\ub144 12\uc6d4 7\uc77c\uc5d0 \uc77c\ubcf8\uad70\uc740 \ud558\uc640\uc774\uc5d0 \uc8fc\ub454\ud55c \ubbf8\ud574\uad70 \ud0dc\ud3c9\uc591 \ud568\ub300\uc5d0 \ub300\ud558\uc5ec \uc9c4\uc8fc\ub9cc \uacf5\uaca9 \uc744", "paragraph_id": "6369962-0-0", "paragraph_question": "question: \ubbf8\uad6d\uc774 \ud0dc\ud3c9\uc591 \uc804\uc7c1\uc5d0 \ucc38\uc5ec\ud558\uac8c \ub41c \uacc4\uae30\ub294?, context: 1941\ub144 12\uc6d4 7\uc77c\uc5d0 \uc77c\ubcf8\uad70\uc740 \ud558\uc640\uc774\uc5d0 \uc8fc\ub454\ud55c \ubbf8\ud574\uad70 \ud0dc\ud3c9\uc591 \ud568\ub300\uc5d0 \ub300\ud558\uc5ec \uc9c4\uc8fc\ub9cc \uacf5\uaca9\uc744 \uac10\ud589\ud558\uc600\ub2e4. \uc9c4\uc8fc\ub9cc \uacf5\uaca9\uc73c\ub85c \ub9ce\uc740 \ubbf8\uad6d \uc804\ud22c\ud568\ub4e4\uc774 \uc190\uc0c1\uc744 \uc785\uc5c8\uace0 \ubbf8\uad6d\uc774 \uacf5\uc2dd\uc801\uc73c\ub85c \uc804\uc7c1\uc5d0 \ucc38\uac00\ud558\uac8c \ub41c\ub2e4. \uc77c\ubcf8\uad70 \uc9c0\ud718\ubd80\ub294 \uc9c4\uc8fc\ub9cc \uacf5\uaca9\uc744 \ud1b5\ud558\uc5ec \ubbf8\ud574\uad70\uc744 \ubb34\ub825\ud654\uc2dc\ud0a4\uace0 \ucc9c\uc5f0\uc790\uc6d0\uc5d0 \ub300\ud55c \uc811\uadfc\uad8c\uc744 \ud655\ubcf4\ud558\uba70, \ud0dc\ud3c9\uc591\uacfc \uc544\uc2dc\uc544 \uc9c0\uc5ed\uc5d0\uc11c \ubbf8\uad70\uc758 \uacf5\uaca9\uc5d0 \ub300\ud56d\ud55c \ubc29\uc5b4 \uac70\uc810\uc744 \ud68d\ub4f1\ud558\uace0\uc790 \ud558\uc600\ub2e4. \uc774\uc640 \uac19\uc740 \ubaa9\uc801 \ud558\uc5d0 \uc77c\ubcf8\uad70\uc740 \ud544\ub9ac\ud540, \ud0c0\uc774, \ub9d0\ub808\uc774 \ubc18\ub3c4, \uc2f1\uac00\ud3ec\ub974, \ubc84\ub9c8, \ub124\ub35c\ub780\ub4dc\ub839 \ub3d9\uc778\ub3c4, \uc6e8\uc774\ud06c \uc12c, \uae38\ubc84\ud2b8\uc12c, \ub274\ube0c\ub9ac\ud2bc \uc12c, \uad0c \ub97c \uc21c\ucc28\uc801\uc73c\ub85c \uacf5\uaca9\ud558\uc5ec \uc810\ub839\ud558\uc600\uc73c\uba70, \ud574\ub2f9 \uc9c0\uc5ed\uc5d0 \ub300\ud55c \uc2dd\ubbfc \uc9c0\ubc30\uad8c\uc744 \ud589\uc0ac\ud558\uace0 \uc788\ub358 \uc601\uad6d, \ud638\uc8fc, \ub124\ub378\ub780\ub4dc\uc5d0 \uc774\uc5b4 \ubbf8\uad6d\uc774 \ud0dc\ud3c9\uc591 \uc804\uc7c1\uc5d0 \uacf5\uc2dd\uc801\uc73c\ub85c \ucc38\uc804\ud558\uac8c \ub41c\ub2e4."} {"question": "\uc131\uacbd \ud788\ube0c\ub9ac\uc5b4 '\uc5d8\ub85c\ud798'\uc758 \ub73b\uc740?", "paragraph": "\uae30\uc6d0\uc804 2~3\uc138\uae30 \ub3d9\uc548 \ubc1c\uc804\ud55c \uc720\ub300 \uc804\ud1b5\uc5d0 \ub530\ub77c \ud14c\ud2b8\ub77c\uadf8\ub77c\ub9c8\ud1a4\uc740 \uae30\ub85d\uc740 \ub418\uc5c8\uc9c0\ub9cc \ubc1c\uc74c\ub418\uc9c0\ub294 \uc54a\ub294\ub2e4. \uc77d\uc744 \ub54c \ub300\uccb4\ub418\ub294 \ubc1c\uc74c\uc740 \ubcf8\ubb38\uc5d0 \u05d9\u05b0\u05d4\u05b9\u05d5\u05b8\u05d4\uac00 \ub098\ud0c0\ub098\ub294 \uc2e0\uc131\ud55c \uc774\ub984\uc744 \ub300\uccb4\ud55c\ub2e4. 19\uc138\uae30 \ud788\ube0c\ub9ac\uc5b4 \ud559\uc790 \uac8c\uc81c\ub2c8\uc6b0\uc2a4\ub294 \"\uc544\ub3c4\ub098\uc774\"(Adonai, \uc8fc)\uc640 \"\uc5d8\ub85c\ud798\"(\ud558\ub098\ub2d8)\uc774\ub77c\ub294 \uc774\ub984\uc744 \ub300\uccb4\ud558\ub294 \ubaa8\uc74c\uc790\ub294 \uc774\ub7ec\ud55c \ub300\uccb4 \ubc1c\uc74c\uc774 \uc0ac\uc6a9\ub418\uc5c8\ub2e4\ub294 \uac83\uc744 \ud45c\uc2dc\ud558\uae30 \uc704\ud558\uc5ec \ub9c8\uc18c\ub77c \ubcf8\ubb38\uc5d0 \uc758\ud574 \uc0bd\uc785\ub418\uc5c8\ub2e4\uace0 \uc120\uc5b8\ud558\uc600\uace0 \uc774\ub7ec\ud55c \ucd94\uce21\uc774 \ub110\ub9ac \ud1b5\uc6a9\ub418\uace0 \uc788\ub2e4. \u05d9\u05d4\u05d5\u05d4\uac00 \uc544\ub3c4\ub098\uc774 \ubcf4\ub2e4 \uc55e\uc11c\uac70\ub098 \ub4a4\ub97c \ub530\ub97c \ub54c \ub9c8\uc18c\ub77c \ubcf8\ubb38\uc740 \uc5d8\ub85c\ud798\uc758 \ubaa8\uc74c \ubd80\ubd84\uc744 \ud14c\ud2b8\ub77c\uadf8\ub77c\ub9c8\ud1a4\uc73c\ub85c \ud45c\uae30\ud558\uc5ec \ub193\uc558\uace0 \uc5ec\uae30\uc11c \"\uc5d8\ub85c\ud798\"\uc73c\ub85c \uc77d\ud614\ub358 \ud14c\ud2b8\ub77c\uadf8\ub77c\ub9c8\ud1a4 \u05d9\u05b1\u05d4\u05b9\u05d5\u05b4\u05d4\uc758 \ub2e4\ub978 \ubaa8\uc74c\ud654\ub97c \ub9cc\ub4e4\uc5b4\ub0c8\ub2e4. \uc774\ub7ec\ud55c \ucd94\ub860\uc5d0 \ub530\ub77c \uc5ec\ud638\uc640 \u05d9\u05b0\u05d4\u05b9\u05d5\u05b8\u05d4\ub77c\ub294 \ud615\ud0dc\ub294 \"\ud63c\uc131\uc801\uc778 \ud615\ud0dc\"\uc758 \ud2b9\uc9d5\uc744 \uac16\uac8c \ub418\uc5c8\uace0 \uc2ec\uc9c0\uc5b4\ub294 \"\ucca0\ud559\uc801 \ubd88\uac00\ub2a5\"\uc73c\ub85c \ubb18\uc0ac\ub418\uae30\ub3c4 \ud588\ub2e4.", "answer": "\ud558\ub098\ub2d8", "sentence": "19\uc138\uae30 \ud788\ube0c\ub9ac\uc5b4 \ud559\uc790 \uac8c\uc81c\ub2c8\uc6b0\uc2a4\ub294 \"\uc544\ub3c4\ub098\uc774\"(Adonai, \uc8fc)\uc640 \"\uc5d8\ub85c\ud798\"( \ud558\ub098\ub2d8 )", "paragraph_sentence": "\uae30\uc6d0\uc804 2~3\uc138\uae30 \ub3d9\uc548 \ubc1c\uc804\ud55c \uc720\ub300 \uc804\ud1b5\uc5d0 \ub530\ub77c \ud14c\ud2b8\ub77c\uadf8\ub77c\ub9c8\ud1a4\uc740 \uae30\ub85d\uc740 \ub418\uc5c8\uc9c0\ub9cc \ubc1c\uc74c\ub418\uc9c0\ub294 \uc54a\ub294\ub2e4. \uc77d\uc744 \ub54c \ub300\uccb4\ub418\ub294 \ubc1c\uc74c\uc740 \ubcf8\ubb38\uc5d0 \u05d9\u05b0\u05d4\u05b9\u05d5\u05b8\u05d4\uac00 \ub098\ud0c0\ub098\ub294 \uc2e0\uc131\ud55c \uc774\ub984\uc744 \ub300\uccb4\ud55c\ub2e4. 19\uc138\uae30 \ud788\ube0c\ub9ac\uc5b4 \ud559\uc790 \uac8c\uc81c\ub2c8\uc6b0\uc2a4\ub294 \"\uc544\ub3c4\ub098\uc774\"(Adonai, \uc8fc)\uc640 \"\uc5d8\ub85c\ud798\"( \ud558\ub098\ub2d8 ) \uc774\ub77c\ub294 \uc774\ub984\uc744 \ub300\uccb4\ud558\ub294 \ubaa8\uc74c\uc790\ub294 \uc774\ub7ec\ud55c \ub300\uccb4 \ubc1c\uc74c\uc774 \uc0ac\uc6a9\ub418\uc5c8\ub2e4\ub294 \uac83\uc744 \ud45c\uc2dc\ud558\uae30 \uc704\ud558\uc5ec \ub9c8\uc18c\ub77c \ubcf8\ubb38\uc5d0 \uc758\ud574 \uc0bd\uc785\ub418\uc5c8\ub2e4\uace0 \uc120\uc5b8\ud558\uc600\uace0 \uc774\ub7ec\ud55c \ucd94\uce21\uc774 \ub110\ub9ac \ud1b5\uc6a9\ub418\uace0 \uc788\ub2e4. \u05d9\u05d4\u05d5\u05d4\uac00 \uc544\ub3c4\ub098\uc774 \ubcf4\ub2e4 \uc55e\uc11c\uac70\ub098 \ub4a4\ub97c \ub530\ub97c \ub54c \ub9c8\uc18c\ub77c \ubcf8\ubb38\uc740 \uc5d8\ub85c\ud798\uc758 \ubaa8\uc74c \ubd80\ubd84\uc744 \ud14c\ud2b8\ub77c\uadf8\ub77c\ub9c8\ud1a4\uc73c\ub85c \ud45c\uae30\ud558\uc5ec \ub193\uc558\uace0 \uc5ec\uae30\uc11c \"\uc5d8\ub85c\ud798\"\uc73c\ub85c \uc77d\ud614\ub358 \ud14c\ud2b8\ub77c\uadf8\ub77c\ub9c8\ud1a4 \u05d9\u05b1\u05d4\u05b9\u05d5\u05b4\u05d4\uc758 \ub2e4\ub978 \ubaa8\uc74c\ud654\ub97c \ub9cc\ub4e4\uc5b4\ub0c8\ub2e4. \uc774\ub7ec\ud55c \ucd94\ub860\uc5d0 \ub530\ub77c \uc5ec\ud638\uc640 \u05d9\u05b0\u05d4\u05b9\u05d5\u05b8\u05d4\ub77c\ub294 \ud615\ud0dc\ub294 \"\ud63c\uc131\uc801\uc778 \ud615\ud0dc\"\uc758 \ud2b9\uc9d5\uc744 \uac16\uac8c \ub418\uc5c8\uace0 \uc2ec\uc9c0\uc5b4\ub294 \"\ucca0\ud559\uc801 \ubd88\uac00\ub2a5\"\uc73c\ub85c \ubb18\uc0ac\ub418\uae30\ub3c4 \ud588\ub2e4.", "paragraph_answer": "\uae30\uc6d0\uc804 2~3\uc138\uae30 \ub3d9\uc548 \ubc1c\uc804\ud55c \uc720\ub300 \uc804\ud1b5\uc5d0 \ub530\ub77c \ud14c\ud2b8\ub77c\uadf8\ub77c\ub9c8\ud1a4\uc740 \uae30\ub85d\uc740 \ub418\uc5c8\uc9c0\ub9cc \ubc1c\uc74c\ub418\uc9c0\ub294 \uc54a\ub294\ub2e4. \uc77d\uc744 \ub54c \ub300\uccb4\ub418\ub294 \ubc1c\uc74c\uc740 \ubcf8\ubb38\uc5d0 \u05d9\u05b0\u05d4\u05b9\u05d5\u05b8\u05d4\uac00 \ub098\ud0c0\ub098\ub294 \uc2e0\uc131\ud55c \uc774\ub984\uc744 \ub300\uccb4\ud55c\ub2e4. 19\uc138\uae30 \ud788\ube0c\ub9ac\uc5b4 \ud559\uc790 \uac8c\uc81c\ub2c8\uc6b0\uc2a4\ub294 \"\uc544\ub3c4\ub098\uc774\"(Adonai, \uc8fc)\uc640 \"\uc5d8\ub85c\ud798\"( \ud558\ub098\ub2d8 )\uc774\ub77c\ub294 \uc774\ub984\uc744 \ub300\uccb4\ud558\ub294 \ubaa8\uc74c\uc790\ub294 \uc774\ub7ec\ud55c \ub300\uccb4 \ubc1c\uc74c\uc774 \uc0ac\uc6a9\ub418\uc5c8\ub2e4\ub294 \uac83\uc744 \ud45c\uc2dc\ud558\uae30 \uc704\ud558\uc5ec \ub9c8\uc18c\ub77c \ubcf8\ubb38\uc5d0 \uc758\ud574 \uc0bd\uc785\ub418\uc5c8\ub2e4\uace0 \uc120\uc5b8\ud558\uc600\uace0 \uc774\ub7ec\ud55c \ucd94\uce21\uc774 \ub110\ub9ac \ud1b5\uc6a9\ub418\uace0 \uc788\ub2e4. \u05d9\u05d4\u05d5\u05d4\uac00 \uc544\ub3c4\ub098\uc774 \ubcf4\ub2e4 \uc55e\uc11c\uac70\ub098 \ub4a4\ub97c \ub530\ub97c \ub54c \ub9c8\uc18c\ub77c \ubcf8\ubb38\uc740 \uc5d8\ub85c\ud798\uc758 \ubaa8\uc74c \ubd80\ubd84\uc744 \ud14c\ud2b8\ub77c\uadf8\ub77c\ub9c8\ud1a4\uc73c\ub85c \ud45c\uae30\ud558\uc5ec \ub193\uc558\uace0 \uc5ec\uae30\uc11c \"\uc5d8\ub85c\ud798\"\uc73c\ub85c \uc77d\ud614\ub358 \ud14c\ud2b8\ub77c\uadf8\ub77c\ub9c8\ud1a4 \u05d9\u05b1\u05d4\u05b9\u05d5\u05b4\u05d4\uc758 \ub2e4\ub978 \ubaa8\uc74c\ud654\ub97c \ub9cc\ub4e4\uc5b4\ub0c8\ub2e4. \uc774\ub7ec\ud55c \ucd94\ub860\uc5d0 \ub530\ub77c \uc5ec\ud638\uc640 \u05d9\u05b0\u05d4\u05b9\u05d5\u05b8\u05d4\ub77c\ub294 \ud615\ud0dc\ub294 \"\ud63c\uc131\uc801\uc778 \ud615\ud0dc\"\uc758 \ud2b9\uc9d5\uc744 \uac16\uac8c \ub418\uc5c8\uace0 \uc2ec\uc9c0\uc5b4\ub294 \"\ucca0\ud559\uc801 \ubd88\uac00\ub2a5\"\uc73c\ub85c \ubb18\uc0ac\ub418\uae30\ub3c4 \ud588\ub2e4.", "sentence_answer": "19\uc138\uae30 \ud788\ube0c\ub9ac\uc5b4 \ud559\uc790 \uac8c\uc81c\ub2c8\uc6b0\uc2a4\ub294 \"\uc544\ub3c4\ub098\uc774\"(Adonai, \uc8fc)\uc640 \"\uc5d8\ub85c\ud798\"( \ud558\ub098\ub2d8 )", "paragraph_id": "6524006-1-1", "paragraph_question": "question: \uc131\uacbd \ud788\ube0c\ub9ac\uc5b4 '\uc5d8\ub85c\ud798'\uc758 \ub73b\uc740?, context: \uae30\uc6d0\uc804 2~3\uc138\uae30 \ub3d9\uc548 \ubc1c\uc804\ud55c \uc720\ub300 \uc804\ud1b5\uc5d0 \ub530\ub77c \ud14c\ud2b8\ub77c\uadf8\ub77c\ub9c8\ud1a4\uc740 \uae30\ub85d\uc740 \ub418\uc5c8\uc9c0\ub9cc \ubc1c\uc74c\ub418\uc9c0\ub294 \uc54a\ub294\ub2e4. \uc77d\uc744 \ub54c \ub300\uccb4\ub418\ub294 \ubc1c\uc74c\uc740 \ubcf8\ubb38\uc5d0 \u05d9\u05b0\u05d4\u05b9\u05d5\u05b8\u05d4\uac00 \ub098\ud0c0\ub098\ub294 \uc2e0\uc131\ud55c \uc774\ub984\uc744 \ub300\uccb4\ud55c\ub2e4. 19\uc138\uae30 \ud788\ube0c\ub9ac\uc5b4 \ud559\uc790 \uac8c\uc81c\ub2c8\uc6b0\uc2a4\ub294 \"\uc544\ub3c4\ub098\uc774\"(Adonai, \uc8fc)\uc640 \"\uc5d8\ub85c\ud798\"(\ud558\ub098\ub2d8)\uc774\ub77c\ub294 \uc774\ub984\uc744 \ub300\uccb4\ud558\ub294 \ubaa8\uc74c\uc790\ub294 \uc774\ub7ec\ud55c \ub300\uccb4 \ubc1c\uc74c\uc774 \uc0ac\uc6a9\ub418\uc5c8\ub2e4\ub294 \uac83\uc744 \ud45c\uc2dc\ud558\uae30 \uc704\ud558\uc5ec \ub9c8\uc18c\ub77c \ubcf8\ubb38\uc5d0 \uc758\ud574 \uc0bd\uc785\ub418\uc5c8\ub2e4\uace0 \uc120\uc5b8\ud558\uc600\uace0 \uc774\ub7ec\ud55c \ucd94\uce21\uc774 \ub110\ub9ac \ud1b5\uc6a9\ub418\uace0 \uc788\ub2e4. \u05d9\u05d4\u05d5\u05d4\uac00 \uc544\ub3c4\ub098\uc774 \ubcf4\ub2e4 \uc55e\uc11c\uac70\ub098 \ub4a4\ub97c \ub530\ub97c \ub54c \ub9c8\uc18c\ub77c \ubcf8\ubb38\uc740 \uc5d8\ub85c\ud798\uc758 \ubaa8\uc74c \ubd80\ubd84\uc744 \ud14c\ud2b8\ub77c\uadf8\ub77c\ub9c8\ud1a4\uc73c\ub85c \ud45c\uae30\ud558\uc5ec \ub193\uc558\uace0 \uc5ec\uae30\uc11c \"\uc5d8\ub85c\ud798\"\uc73c\ub85c \uc77d\ud614\ub358 \ud14c\ud2b8\ub77c\uadf8\ub77c\ub9c8\ud1a4 \u05d9\u05b1\u05d4\u05b9\u05d5\u05b4\u05d4\uc758 \ub2e4\ub978 \ubaa8\uc74c\ud654\ub97c \ub9cc\ub4e4\uc5b4\ub0c8\ub2e4. \uc774\ub7ec\ud55c \ucd94\ub860\uc5d0 \ub530\ub77c \uc5ec\ud638\uc640 \u05d9\u05b0\u05d4\u05b9\u05d5\u05b8\u05d4\ub77c\ub294 \ud615\ud0dc\ub294 \"\ud63c\uc131\uc801\uc778 \ud615\ud0dc\"\uc758 \ud2b9\uc9d5\uc744 \uac16\uac8c \ub418\uc5c8\uace0 \uc2ec\uc9c0\uc5b4\ub294 \"\ucca0\ud559\uc801 \ubd88\uac00\ub2a5\"\uc73c\ub85c \ubb18\uc0ac\ub418\uae30\ub3c4 \ud588\ub2e4."} {"question": "\uc77c\ubcf8\uc774 \uac00\uc838\uac04 \ud55c\uad6d \uc120\ubc15\uc758 \ubb34\uac8c\ub294 \uc5bc\ub9c8\uc778\uac00\uc694?", "paragraph": "1960\ub144 3\uc6d4 19\uc77c, \uc591\uc720\ucc2c\uc740 \uc77c\ubcf8 \uacf5\ub3d9(\u5171\u540c) \ud1b5\uc2e0\uae30\uc790\uc640\uc758 \ud68c\uacac\uc5d0\uc11c \ud55c\uc77c\uac04\uc758 \uc5ec\ub7ec \ubb38\uc81c\uc5d0 \uad00\ud558\uc5ec \ub2e4\uc74c\uacfc \uac19\uc774 \ub9d0\ud558\uc600\ub2e4. \u25b3 \ud55c\uc77c\ubb38\uc81c : \uc591\uc720\ucc2c\uc740 \ud55c\uad6d\uacfc \uc77c\ubcf8\uc774 1960\ub144 3\uc6d4\uc5d0 \uc5b5\ub958\uc790\ub97c \uc0c1\ud638\uc11d\ubc29\ud558\uae30\ub85c \ud55c \ud569\uc758\ub294 \uc591\uad6d\uac04\uc758 \uc8fc\uc694\ud55c \uc5ec\ub7ec\uac00\uc9c0 \ubb38\uc81c\ud574\uacb0\uc5d0 \ub354\ud55c\uce35\uc758 \uc9c4\uc804\uc744 \uc57d\uc18d\ud574\uc8fc\ub294 \uac83\uc774\ub77c\uace0 \ub9d0\ud558\uc600\ub2e4. \ud55c \uc778\ud130\ubdf0\uc5d0\uc11c \uc591\uc720\ucc2c\uc740 \uae30\ud0c0 \ubaa8\ub4e0 \uc870\uac74\uc5d0 \ud569\uc758\uac00 \uc774\ub8e8\uc5b4\uc9c4\ub2e4\uba74 \ub17c\uc7c1 \uc911\uc778 \ud55c\uc77c\uac04\uc758 \uc5b4\ub85c\ubb38\uc81c\uc758 \ub9cc\uc871\ud560\ub9cc\ud55c \uc870\uc815\uc774 \ud68c\ub2f4\uc11d\uc0c1\uc5d0\uc11c \uc774\ub8e8\uc5b4\uc9c8 \uac83\uc774\ub77c\uace0 \ub9d0\ud558\uc600\ub2e4. \uadf8\ub7ec\ub098 \uc591\uc720\ucc2c\uc740 \uc5b4\ub85c\uc120\ubb38\uc81c\uac00 \ud1a0\uc758\ub418\uae30 \uc804\uc5d0 \uc801\uc5b4\ub3c4 \ub2e4\uc74c\uacfc \uac19\uc740 \uc77c\uacf1\uac00\uc9c0 \ubb38\uc81c\uac00 \ud574\uacb0\ub418\uc5b4\uc57c \ud55c\ub2e4\uace0 \uc5b8\uba85\ud558\uc600\ub2e4. 1. \uc81c2\ucc28\uc138\uacc4\ub300\uc804 \uc804 \ub610\ub294 \ub300\uc804\uc911\uc5d0 \uc77c\ubcf8\uc73c\ub85c \ub04c\ub824\uac04 \ud55c\uad6d\uc778 \ub178\ub3d9\uc790\ub4e4\uc5d0 \ub300\ud55c \ubcf4\uc0c1\ubb38\uc81c 2. \ud55c\uad6d\uc73c\ub85c \uc1a1\ud658\ub418\ub294 \uc7ac\uc77c\ud55c\uad50(\u97d3\u50d1)\ub4e4\uc774 \uc7ac\uc0b0\uc744 \ubc18\ucd9c\ud560 \uc218 \uc788\ub3c4\ub85d \ud558\ub294 \ud611\uc815 3. \uc77c\ubcf8\uc5d0 \ub0a8\uc544\uc788\uae30\ub97c \ud76c\ub9dd\ud558\ub294 \ud55c\uad50\ub4e4\uc5d0\uac8c \uac15\uc81c\uc1a1\ud658\uc758 \uacf5\ud3ec\ub97c \uc5c6\uc560\uae30 \uc704\ud55c \uc815\ub2f9\ud55c \uac70\uc8fc\uad8c\uc758 \uc778\uc815 4. \ud55c\uad6d\uad6d\ubcf4\uc758 \ubc18\ud658 5. \uc77c\ubcf8\uc5d0 \uc788\ub294 \ud55c\uad6d\uc7ac\uc0b0\uad8c\uc758 \ud574\uacb0 6. \uc77c\ubcf8\uc5d0 \uac00\uc838\uac04 \ud55c\uad6d \uc120\ubc15 7\ub9cc5\ucc9c\ud1a4\uc758 \ubc18\ud658 7. \ud55c\uc77c\uac04\uc758 \ud574\uc800\uc804\uc2e0\uc5d0 \uad00\ud55c \ubb38\uc81c\uc758 \ud574\uacb0 \u25b3 \uc601\ud574\ubb38\uc81c : \uc774\uc0c1\uc758 \uc870\uac74 \uc218\ub77d\uc73c\ub85c \ud3c9\ud654\uc120 \ubb38\uc81c\ub294 \ud574\uacb0\ub418\ub098 \ud3c9\ud654\uc120\ubb38\uc81c\uc640 \uc601\ud574\ubb38\uc81c\ub294 \ubcc4\uac1c\uc758 \uac83\uc774\ub2e4. \u25b3 \ud574\uc591\ubc95 \ud68c\uc758 : \ud55c\uad6d\uc815\ubcf4\ub294 \ud574\uc591\ubc95\ud68c\uc758\uc5d0\uc11c \ubbf8\uad6d\uc548\uc744 \uc9c0\uc9c0\ud558\ub3c4\ub85d \ub300\ud45c\ub4e4\uc5d0\uac8c \ud6c8\ub839\ud558\uc600\ub2e4. \uadf8\ub7ec\ub098 \ud3c9\ud654\uc120\ubb38\uc81c\ub294 \uc601\ud574\ubb38\uc81c\uc640\ub294 \ubcc4\ub3c4\ub85c \ud55c\uc77c\uc591\uad6d \uc0ac\uc774\uc5d0\uc11c \ud574\uacb0\ub418\uc5b4\uc57c \ud55c\ub2e4. \uc591\uc720\ucc2c\uc740 \uc774\uc0c1 \uc870\uac74\uc744 \uc77c\ubcf8\uc774 \uc218\ub77d\ud55c\ub2e4\uba74 \ud55c\uc77c\uad00\uacc4\ub294 \uc815\uc0c1\ud654\ub418\uace0 \ud3c9\ud654\uc120\ubb38\uc81c\uc640 \uc5b4\ub85c\ubb38\uc81c\ub294 \ud574\uacb0\ub420 \uac83\uc774\ub77c\uace0 \ub9d0\ud558\uc600\ub2e4. \uadf8\ub294 \uc774\uc5b4 \ud5c8\ud130 \ubbf8\uad6d \uad6d\ubb34\uc7a5\uad00\uacfc\uc758 \ud68c\ub2f4\uc740 \uc2dc\uc885 \ubd80\ub4dc\ub7ec\uc6b4 \ubd84\uc704\uae30 \uc18d\uc5d0\uc11c \uc774\ub8e8\uc5b4\uc84c\ub2e4\uace0 \ub9d0\ud558\uc600\ub2e4.", "answer": "7\ub9cc5\ucc9c\ud1a4", "sentence": "\uc77c\ubcf8\uc5d0 \uac00\uc838\uac04 \ud55c\uad6d \uc120\ubc15 7\ub9cc5\ucc9c\ud1a4 \uc758 \ubc18\ud658 7.", "paragraph_sentence": "1960\ub144 3\uc6d4 19\uc77c, \uc591\uc720\ucc2c\uc740 \uc77c\ubcf8 \uacf5\ub3d9(\u5171\u540c) \ud1b5\uc2e0\uae30\uc790\uc640\uc758 \ud68c\uacac\uc5d0\uc11c \ud55c\uc77c\uac04\uc758 \uc5ec\ub7ec \ubb38\uc81c\uc5d0 \uad00\ud558\uc5ec \ub2e4\uc74c\uacfc \uac19\uc774 \ub9d0\ud558\uc600\ub2e4. \u25b3 \ud55c\uc77c\ubb38\uc81c : \uc591\uc720\ucc2c\uc740 \ud55c\uad6d\uacfc \uc77c\ubcf8\uc774 1960\ub144 3\uc6d4\uc5d0 \uc5b5\ub958\uc790\ub97c \uc0c1\ud638\uc11d\ubc29\ud558\uae30\ub85c \ud55c \ud569\uc758\ub294 \uc591\uad6d\uac04\uc758 \uc8fc\uc694\ud55c \uc5ec\ub7ec\uac00\uc9c0 \ubb38\uc81c\ud574\uacb0\uc5d0 \ub354\ud55c\uce35\uc758 \uc9c4\uc804\uc744 \uc57d\uc18d\ud574\uc8fc\ub294 \uac83\uc774\ub77c\uace0 \ub9d0\ud558\uc600\ub2e4. \ud55c \uc778\ud130\ubdf0\uc5d0\uc11c \uc591\uc720\ucc2c\uc740 \uae30\ud0c0 \ubaa8\ub4e0 \uc870\uac74\uc5d0 \ud569\uc758\uac00 \uc774\ub8e8\uc5b4\uc9c4\ub2e4\uba74 \ub17c\uc7c1 \uc911\uc778 \ud55c\uc77c\uac04\uc758 \uc5b4\ub85c\ubb38\uc81c\uc758 \ub9cc\uc871\ud560\ub9cc\ud55c \uc870\uc815\uc774 \ud68c\ub2f4\uc11d\uc0c1\uc5d0\uc11c \uc774\ub8e8\uc5b4\uc9c8 \uac83\uc774\ub77c\uace0 \ub9d0\ud558\uc600\ub2e4. \uadf8\ub7ec\ub098 \uc591\uc720\ucc2c\uc740 \uc5b4\ub85c\uc120\ubb38\uc81c\uac00 \ud1a0\uc758\ub418\uae30 \uc804\uc5d0 \uc801\uc5b4\ub3c4 \ub2e4\uc74c\uacfc \uac19\uc740 \uc77c\uacf1\uac00\uc9c0 \ubb38\uc81c\uac00 \ud574\uacb0\ub418\uc5b4\uc57c \ud55c\ub2e4\uace0 \uc5b8\uba85\ud558\uc600\ub2e4. 1. \uc81c2\ucc28\uc138\uacc4\ub300\uc804 \uc804 \ub610\ub294 \ub300\uc804\uc911\uc5d0 \uc77c\ubcf8\uc73c\ub85c \ub04c\ub824\uac04 \ud55c\uad6d\uc778 \ub178\ub3d9\uc790\ub4e4\uc5d0 \ub300\ud55c \ubcf4\uc0c1\ubb38\uc81c 2. \ud55c\uad6d\uc73c\ub85c \uc1a1\ud658\ub418\ub294 \uc7ac\uc77c\ud55c\uad50(\u97d3\u50d1)\ub4e4\uc774 \uc7ac\uc0b0\uc744 \ubc18\ucd9c\ud560 \uc218 \uc788\ub3c4\ub85d \ud558\ub294 \ud611\uc815 3. \uc77c\ubcf8\uc5d0 \ub0a8\uc544\uc788\uae30\ub97c \ud76c\ub9dd\ud558\ub294 \ud55c\uad50\ub4e4\uc5d0\uac8c \uac15\uc81c\uc1a1\ud658\uc758 \uacf5\ud3ec\ub97c \uc5c6\uc560\uae30 \uc704\ud55c \uc815\ub2f9\ud55c \uac70\uc8fc\uad8c\uc758 \uc778\uc815 4. \ud55c\uad6d\uad6d\ubcf4\uc758 \ubc18\ud658 5. \uc77c\ubcf8\uc5d0 \uc788\ub294 \ud55c\uad6d\uc7ac\uc0b0\uad8c\uc758 \ud574\uacb0 6. \uc77c\ubcf8\uc5d0 \uac00\uc838\uac04 \ud55c\uad6d \uc120\ubc15 7\ub9cc5\ucc9c\ud1a4 \uc758 \ubc18\ud658 7. \ud55c\uc77c\uac04\uc758 \ud574\uc800\uc804\uc2e0\uc5d0 \uad00\ud55c \ubb38\uc81c\uc758 \ud574\uacb0 \u25b3 \uc601\ud574\ubb38\uc81c : \uc774\uc0c1\uc758 \uc870\uac74 \uc218\ub77d\uc73c\ub85c \ud3c9\ud654\uc120 \ubb38\uc81c\ub294 \ud574\uacb0\ub418\ub098 \ud3c9\ud654\uc120\ubb38\uc81c\uc640 \uc601\ud574\ubb38\uc81c\ub294 \ubcc4\uac1c\uc758 \uac83\uc774\ub2e4. \u25b3 \ud574\uc591\ubc95 \ud68c\uc758 : \ud55c\uad6d\uc815\ubcf4\ub294 \ud574\uc591\ubc95\ud68c\uc758\uc5d0\uc11c \ubbf8\uad6d\uc548\uc744 \uc9c0\uc9c0\ud558\ub3c4\ub85d \ub300\ud45c\ub4e4\uc5d0\uac8c \ud6c8\ub839\ud558\uc600\ub2e4. \uadf8\ub7ec\ub098 \ud3c9\ud654\uc120\ubb38\uc81c\ub294 \uc601\ud574\ubb38\uc81c\uc640\ub294 \ubcc4\ub3c4\ub85c \ud55c\uc77c\uc591\uad6d \uc0ac\uc774\uc5d0\uc11c \ud574\uacb0\ub418\uc5b4\uc57c \ud55c\ub2e4. \uc591\uc720\ucc2c\uc740 \uc774\uc0c1 \uc870\uac74\uc744 \uc77c\ubcf8\uc774 \uc218\ub77d\ud55c\ub2e4\uba74 \ud55c\uc77c\uad00\uacc4\ub294 \uc815\uc0c1\ud654\ub418\uace0 \ud3c9\ud654\uc120\ubb38\uc81c\uc640 \uc5b4\ub85c\ubb38\uc81c\ub294 \ud574\uacb0\ub420 \uac83\uc774\ub77c\uace0 \ub9d0\ud558\uc600\ub2e4. \uadf8\ub294 \uc774\uc5b4 \ud5c8\ud130 \ubbf8\uad6d \uad6d\ubb34\uc7a5\uad00\uacfc\uc758 \ud68c\ub2f4\uc740 \uc2dc\uc885 \ubd80\ub4dc\ub7ec\uc6b4 \ubd84\uc704\uae30 \uc18d\uc5d0\uc11c \uc774\ub8e8\uc5b4\uc84c\ub2e4\uace0 \ub9d0\ud558\uc600\ub2e4.", "paragraph_answer": "1960\ub144 3\uc6d4 19\uc77c, \uc591\uc720\ucc2c\uc740 \uc77c\ubcf8 \uacf5\ub3d9(\u5171\u540c) \ud1b5\uc2e0\uae30\uc790\uc640\uc758 \ud68c\uacac\uc5d0\uc11c \ud55c\uc77c\uac04\uc758 \uc5ec\ub7ec \ubb38\uc81c\uc5d0 \uad00\ud558\uc5ec \ub2e4\uc74c\uacfc \uac19\uc774 \ub9d0\ud558\uc600\ub2e4. \u25b3 \ud55c\uc77c\ubb38\uc81c : \uc591\uc720\ucc2c\uc740 \ud55c\uad6d\uacfc \uc77c\ubcf8\uc774 1960\ub144 3\uc6d4\uc5d0 \uc5b5\ub958\uc790\ub97c \uc0c1\ud638\uc11d\ubc29\ud558\uae30\ub85c \ud55c \ud569\uc758\ub294 \uc591\uad6d\uac04\uc758 \uc8fc\uc694\ud55c \uc5ec\ub7ec\uac00\uc9c0 \ubb38\uc81c\ud574\uacb0\uc5d0 \ub354\ud55c\uce35\uc758 \uc9c4\uc804\uc744 \uc57d\uc18d\ud574\uc8fc\ub294 \uac83\uc774\ub77c\uace0 \ub9d0\ud558\uc600\ub2e4. \ud55c \uc778\ud130\ubdf0\uc5d0\uc11c \uc591\uc720\ucc2c\uc740 \uae30\ud0c0 \ubaa8\ub4e0 \uc870\uac74\uc5d0 \ud569\uc758\uac00 \uc774\ub8e8\uc5b4\uc9c4\ub2e4\uba74 \ub17c\uc7c1 \uc911\uc778 \ud55c\uc77c\uac04\uc758 \uc5b4\ub85c\ubb38\uc81c\uc758 \ub9cc\uc871\ud560\ub9cc\ud55c \uc870\uc815\uc774 \ud68c\ub2f4\uc11d\uc0c1\uc5d0\uc11c \uc774\ub8e8\uc5b4\uc9c8 \uac83\uc774\ub77c\uace0 \ub9d0\ud558\uc600\ub2e4. \uadf8\ub7ec\ub098 \uc591\uc720\ucc2c\uc740 \uc5b4\ub85c\uc120\ubb38\uc81c\uac00 \ud1a0\uc758\ub418\uae30 \uc804\uc5d0 \uc801\uc5b4\ub3c4 \ub2e4\uc74c\uacfc \uac19\uc740 \uc77c\uacf1\uac00\uc9c0 \ubb38\uc81c\uac00 \ud574\uacb0\ub418\uc5b4\uc57c \ud55c\ub2e4\uace0 \uc5b8\uba85\ud558\uc600\ub2e4. 1. \uc81c2\ucc28\uc138\uacc4\ub300\uc804 \uc804 \ub610\ub294 \ub300\uc804\uc911\uc5d0 \uc77c\ubcf8\uc73c\ub85c \ub04c\ub824\uac04 \ud55c\uad6d\uc778 \ub178\ub3d9\uc790\ub4e4\uc5d0 \ub300\ud55c \ubcf4\uc0c1\ubb38\uc81c 2. \ud55c\uad6d\uc73c\ub85c \uc1a1\ud658\ub418\ub294 \uc7ac\uc77c\ud55c\uad50(\u97d3\u50d1)\ub4e4\uc774 \uc7ac\uc0b0\uc744 \ubc18\ucd9c\ud560 \uc218 \uc788\ub3c4\ub85d \ud558\ub294 \ud611\uc815 3. \uc77c\ubcf8\uc5d0 \ub0a8\uc544\uc788\uae30\ub97c \ud76c\ub9dd\ud558\ub294 \ud55c\uad50\ub4e4\uc5d0\uac8c \uac15\uc81c\uc1a1\ud658\uc758 \uacf5\ud3ec\ub97c \uc5c6\uc560\uae30 \uc704\ud55c \uc815\ub2f9\ud55c \uac70\uc8fc\uad8c\uc758 \uc778\uc815 4. \ud55c\uad6d\uad6d\ubcf4\uc758 \ubc18\ud658 5. \uc77c\ubcf8\uc5d0 \uc788\ub294 \ud55c\uad6d\uc7ac\uc0b0\uad8c\uc758 \ud574\uacb0 6. \uc77c\ubcf8\uc5d0 \uac00\uc838\uac04 \ud55c\uad6d \uc120\ubc15 7\ub9cc5\ucc9c\ud1a4 \uc758 \ubc18\ud658 7. \ud55c\uc77c\uac04\uc758 \ud574\uc800\uc804\uc2e0\uc5d0 \uad00\ud55c \ubb38\uc81c\uc758 \ud574\uacb0 \u25b3 \uc601\ud574\ubb38\uc81c : \uc774\uc0c1\uc758 \uc870\uac74 \uc218\ub77d\uc73c\ub85c \ud3c9\ud654\uc120 \ubb38\uc81c\ub294 \ud574\uacb0\ub418\ub098 \ud3c9\ud654\uc120\ubb38\uc81c\uc640 \uc601\ud574\ubb38\uc81c\ub294 \ubcc4\uac1c\uc758 \uac83\uc774\ub2e4. \u25b3 \ud574\uc591\ubc95 \ud68c\uc758 : \ud55c\uad6d\uc815\ubcf4\ub294 \ud574\uc591\ubc95\ud68c\uc758\uc5d0\uc11c \ubbf8\uad6d\uc548\uc744 \uc9c0\uc9c0\ud558\ub3c4\ub85d \ub300\ud45c\ub4e4\uc5d0\uac8c \ud6c8\ub839\ud558\uc600\ub2e4. \uadf8\ub7ec\ub098 \ud3c9\ud654\uc120\ubb38\uc81c\ub294 \uc601\ud574\ubb38\uc81c\uc640\ub294 \ubcc4\ub3c4\ub85c \ud55c\uc77c\uc591\uad6d \uc0ac\uc774\uc5d0\uc11c \ud574\uacb0\ub418\uc5b4\uc57c \ud55c\ub2e4. \uc591\uc720\ucc2c\uc740 \uc774\uc0c1 \uc870\uac74\uc744 \uc77c\ubcf8\uc774 \uc218\ub77d\ud55c\ub2e4\uba74 \ud55c\uc77c\uad00\uacc4\ub294 \uc815\uc0c1\ud654\ub418\uace0 \ud3c9\ud654\uc120\ubb38\uc81c\uc640 \uc5b4\ub85c\ubb38\uc81c\ub294 \ud574\uacb0\ub420 \uac83\uc774\ub77c\uace0 \ub9d0\ud558\uc600\ub2e4. \uadf8\ub294 \uc774\uc5b4 \ud5c8\ud130 \ubbf8\uad6d \uad6d\ubb34\uc7a5\uad00\uacfc\uc758 \ud68c\ub2f4\uc740 \uc2dc\uc885 \ubd80\ub4dc\ub7ec\uc6b4 \ubd84\uc704\uae30 \uc18d\uc5d0\uc11c \uc774\ub8e8\uc5b4\uc84c\ub2e4\uace0 \ub9d0\ud558\uc600\ub2e4.", "sentence_answer": "\uc77c\ubcf8\uc5d0 \uac00\uc838\uac04 \ud55c\uad6d \uc120\ubc15 7\ub9cc5\ucc9c\ud1a4 \uc758 \ubc18\ud658 7.", "paragraph_id": "6522887-4-0", "paragraph_question": "question: \uc77c\ubcf8\uc774 \uac00\uc838\uac04 \ud55c\uad6d \uc120\ubc15\uc758 \ubb34\uac8c\ub294 \uc5bc\ub9c8\uc778\uac00\uc694?, context: 1960\ub144 3\uc6d4 19\uc77c, \uc591\uc720\ucc2c\uc740 \uc77c\ubcf8 \uacf5\ub3d9(\u5171\u540c) \ud1b5\uc2e0\uae30\uc790\uc640\uc758 \ud68c\uacac\uc5d0\uc11c \ud55c\uc77c\uac04\uc758 \uc5ec\ub7ec \ubb38\uc81c\uc5d0 \uad00\ud558\uc5ec \ub2e4\uc74c\uacfc \uac19\uc774 \ub9d0\ud558\uc600\ub2e4. \u25b3 \ud55c\uc77c\ubb38\uc81c : \uc591\uc720\ucc2c\uc740 \ud55c\uad6d\uacfc \uc77c\ubcf8\uc774 1960\ub144 3\uc6d4\uc5d0 \uc5b5\ub958\uc790\ub97c \uc0c1\ud638\uc11d\ubc29\ud558\uae30\ub85c \ud55c \ud569\uc758\ub294 \uc591\uad6d\uac04\uc758 \uc8fc\uc694\ud55c \uc5ec\ub7ec\uac00\uc9c0 \ubb38\uc81c\ud574\uacb0\uc5d0 \ub354\ud55c\uce35\uc758 \uc9c4\uc804\uc744 \uc57d\uc18d\ud574\uc8fc\ub294 \uac83\uc774\ub77c\uace0 \ub9d0\ud558\uc600\ub2e4. \ud55c \uc778\ud130\ubdf0\uc5d0\uc11c \uc591\uc720\ucc2c\uc740 \uae30\ud0c0 \ubaa8\ub4e0 \uc870\uac74\uc5d0 \ud569\uc758\uac00 \uc774\ub8e8\uc5b4\uc9c4\ub2e4\uba74 \ub17c\uc7c1 \uc911\uc778 \ud55c\uc77c\uac04\uc758 \uc5b4\ub85c\ubb38\uc81c\uc758 \ub9cc\uc871\ud560\ub9cc\ud55c \uc870\uc815\uc774 \ud68c\ub2f4\uc11d\uc0c1\uc5d0\uc11c \uc774\ub8e8\uc5b4\uc9c8 \uac83\uc774\ub77c\uace0 \ub9d0\ud558\uc600\ub2e4. \uadf8\ub7ec\ub098 \uc591\uc720\ucc2c\uc740 \uc5b4\ub85c\uc120\ubb38\uc81c\uac00 \ud1a0\uc758\ub418\uae30 \uc804\uc5d0 \uc801\uc5b4\ub3c4 \ub2e4\uc74c\uacfc \uac19\uc740 \uc77c\uacf1\uac00\uc9c0 \ubb38\uc81c\uac00 \ud574\uacb0\ub418\uc5b4\uc57c \ud55c\ub2e4\uace0 \uc5b8\uba85\ud558\uc600\ub2e4. 1. \uc81c2\ucc28\uc138\uacc4\ub300\uc804 \uc804 \ub610\ub294 \ub300\uc804\uc911\uc5d0 \uc77c\ubcf8\uc73c\ub85c \ub04c\ub824\uac04 \ud55c\uad6d\uc778 \ub178\ub3d9\uc790\ub4e4\uc5d0 \ub300\ud55c \ubcf4\uc0c1\ubb38\uc81c 2. \ud55c\uad6d\uc73c\ub85c \uc1a1\ud658\ub418\ub294 \uc7ac\uc77c\ud55c\uad50(\u97d3\u50d1)\ub4e4\uc774 \uc7ac\uc0b0\uc744 \ubc18\ucd9c\ud560 \uc218 \uc788\ub3c4\ub85d \ud558\ub294 \ud611\uc815 3. \uc77c\ubcf8\uc5d0 \ub0a8\uc544\uc788\uae30\ub97c \ud76c\ub9dd\ud558\ub294 \ud55c\uad50\ub4e4\uc5d0\uac8c \uac15\uc81c\uc1a1\ud658\uc758 \uacf5\ud3ec\ub97c \uc5c6\uc560\uae30 \uc704\ud55c \uc815\ub2f9\ud55c \uac70\uc8fc\uad8c\uc758 \uc778\uc815 4. \ud55c\uad6d\uad6d\ubcf4\uc758 \ubc18\ud658 5. \uc77c\ubcf8\uc5d0 \uc788\ub294 \ud55c\uad6d\uc7ac\uc0b0\uad8c\uc758 \ud574\uacb0 6. \uc77c\ubcf8\uc5d0 \uac00\uc838\uac04 \ud55c\uad6d \uc120\ubc15 7\ub9cc5\ucc9c\ud1a4\uc758 \ubc18\ud658 7. \ud55c\uc77c\uac04\uc758 \ud574\uc800\uc804\uc2e0\uc5d0 \uad00\ud55c \ubb38\uc81c\uc758 \ud574\uacb0 \u25b3 \uc601\ud574\ubb38\uc81c : \uc774\uc0c1\uc758 \uc870\uac74 \uc218\ub77d\uc73c\ub85c \ud3c9\ud654\uc120 \ubb38\uc81c\ub294 \ud574\uacb0\ub418\ub098 \ud3c9\ud654\uc120\ubb38\uc81c\uc640 \uc601\ud574\ubb38\uc81c\ub294 \ubcc4\uac1c\uc758 \uac83\uc774\ub2e4. \u25b3 \ud574\uc591\ubc95 \ud68c\uc758 : \ud55c\uad6d\uc815\ubcf4\ub294 \ud574\uc591\ubc95\ud68c\uc758\uc5d0\uc11c \ubbf8\uad6d\uc548\uc744 \uc9c0\uc9c0\ud558\ub3c4\ub85d \ub300\ud45c\ub4e4\uc5d0\uac8c \ud6c8\ub839\ud558\uc600\ub2e4. \uadf8\ub7ec\ub098 \ud3c9\ud654\uc120\ubb38\uc81c\ub294 \uc601\ud574\ubb38\uc81c\uc640\ub294 \ubcc4\ub3c4\ub85c \ud55c\uc77c\uc591\uad6d \uc0ac\uc774\uc5d0\uc11c \ud574\uacb0\ub418\uc5b4\uc57c \ud55c\ub2e4. \uc591\uc720\ucc2c\uc740 \uc774\uc0c1 \uc870\uac74\uc744 \uc77c\ubcf8\uc774 \uc218\ub77d\ud55c\ub2e4\uba74 \ud55c\uc77c\uad00\uacc4\ub294 \uc815\uc0c1\ud654\ub418\uace0 \ud3c9\ud654\uc120\ubb38\uc81c\uc640 \uc5b4\ub85c\ubb38\uc81c\ub294 \ud574\uacb0\ub420 \uac83\uc774\ub77c\uace0 \ub9d0\ud558\uc600\ub2e4. \uadf8\ub294 \uc774\uc5b4 \ud5c8\ud130 \ubbf8\uad6d \uad6d\ubb34\uc7a5\uad00\uacfc\uc758 \ud68c\ub2f4\uc740 \uc2dc\uc885 \ubd80\ub4dc\ub7ec\uc6b4 \ubd84\uc704\uae30 \uc18d\uc5d0\uc11c \uc774\ub8e8\uc5b4\uc84c\ub2e4\uace0 \ub9d0\ud558\uc600\ub2e4."} {"question": "1933\ub144 \ub2f9\uc2dc \uc544\ud504\ub9ac\uce74\uacc4 \ubbf8\uad6d\uc778\ub4e4\uc740 \uc0dd\uacc4\ub97c \uc704\ud574 \ud50c\ub79c\ud14c\uc774\uc158 \ub18d\uc7a5\uc5d0\uc11c \uc5b4\ub5a0\ud55c \uc2e0\ubd84\uc73c\ub85c \uc77c\ud558\uc600\ub098?", "paragraph": "\ub300 \uc774\uc8fc \uae30\uac04\uc774\ub358 1933\ub144 4\uc6d4, \uad6d\uc81c \ud68c\uc7a5\uc774\ub358 \uc544\uc774\ub2e4 \uc7ad\uc2a8\uc740 \ubbf8\uc2dc\uc2dc\ud53c \ub809\uc2f1\ud134\uc758 \uc62c \uc138\uc778\ud2b8 \uacf5\uc5c5\ud559\uad50\ub97c \ubc29\ubb38\ud558\uc600\ub2e4. \uadf8\ub140\ub294 \ubbf8\uc2dc\uc2dc\ud53c \ub378\ud0c0\uc758 \uc5b4\ub824\uc6b4 \ud658\uacbd\uc744 \ub3cc\uc544\ubcf4\uc558\ub294\ub370, \uc77c\ubd80 \uad50\uc0ac\ub4e4\uc740 \uacfc\uac70\uc5d0 7\ud559\ub144 \uad50\uc721\uc744 \ubc1b\uc9c0\ub3c4 \uc54a\uc558\ub2e4. \ub18d\uc0b0\ubb3c \uac00\uaca9\uc774 \uacc4\uc18d \ub5a8\uc5b4\uc84c\uae30 \ub54c\ubb38\uc5d0, \uc544\ud504\ub9ac\uce74\uacc4 \ubbf8\uad6d\uc778\ub4e4\uc740 \ud50c\ub79c\ud14c\uc774\uc158 \ub18d\uc7a5\uc5d0\uc11c \uc18c\uc791\ub18d\uc73c\ub85c\uc11c \uc0dd\uacc4\ub97c \uc774\uc5b4\ub098\uac00\uace0 \uc788\uc5c8\ub2e4. 1934\ub144 \uc5ec\ub984 \uc544\uc774\ub2e4 \uc7ad\uc2a8\uc740 \uc2dc\uace8 \uad50\uc0ac\ub4e4\uc744 \ud6c8\ub828\uc2dc\ud0a4\uae30 \uc704\ud55c \uc5ec\ub984 \ud559\uad50\ub97c \uad6c\uc0c1\ud558\uc600\ub2e4. \uadf8\ub140\ub294 \ucd1d 22\uba85\uc758 \uad50\uc0ac \ud559\uc0dd, 243\uba85\uc758 \uc5b4\ub9b0\uc774\ub4e4\uc744 \ub370\ub9ac\uace0 \ud559\uad50\ub97c \uc6b4\uc601\ud588\ub2e4. \uc5b4\ub978\ub4e4 48\uba85\uc744 \uc704\ud55c 9\uac1c \ud559\uae09\uc744 \uc5f4\uc5c8\ub2e4. \uadf8 \ud559\uad50\uc758 \ub3c4\uc11c\uad00\uc740 2600\uad8c\uc758 \ucc45\uc744 \ubcf4\uc720\ud558\uac8c \ub418\uc5b4, \ud648\uc988 \uce74\uc6b4\ud2f0 \uc804\uccb4\uc5d0\uc11c \uac00\uc7a5 \ud070 \ub3c4\uc11c\uad00\uc774 \ub418\uc5c8\ub2e4.", "answer": "\uc18c\uc791\ub18d", "sentence": "\ub18d\uc0b0\ubb3c \uac00\uaca9\uc774 \uacc4\uc18d \ub5a8\uc5b4\uc84c\uae30 \ub54c\ubb38\uc5d0, \uc544\ud504\ub9ac\uce74\uacc4 \ubbf8\uad6d\uc778\ub4e4\uc740 \ud50c\ub79c\ud14c\uc774\uc158 \ub18d\uc7a5\uc5d0\uc11c \uc18c\uc791\ub18d \uc73c\ub85c\uc11c \uc0dd\uacc4\ub97c \uc774\uc5b4\ub098\uac00\uace0 \uc788\uc5c8\ub2e4.", "paragraph_sentence": "\ub300 \uc774\uc8fc \uae30\uac04\uc774\ub358 1933\ub144 4\uc6d4, \uad6d\uc81c \ud68c\uc7a5\uc774\ub358 \uc544\uc774\ub2e4 \uc7ad\uc2a8\uc740 \ubbf8\uc2dc\uc2dc\ud53c \ub809\uc2f1\ud134\uc758 \uc62c \uc138\uc778\ud2b8 \uacf5\uc5c5\ud559\uad50\ub97c \ubc29\ubb38\ud558\uc600\ub2e4. \uadf8\ub140\ub294 \ubbf8\uc2dc\uc2dc\ud53c \ub378\ud0c0\uc758 \uc5b4\ub824\uc6b4 \ud658\uacbd\uc744 \ub3cc\uc544\ubcf4\uc558\ub294\ub370, \uc77c\ubd80 \uad50\uc0ac\ub4e4\uc740 \uacfc\uac70\uc5d0 7\ud559\ub144 \uad50\uc721\uc744 \ubc1b\uc9c0\ub3c4 \uc54a\uc558\ub2e4. \ub18d\uc0b0\ubb3c \uac00\uaca9\uc774 \uacc4\uc18d \ub5a8\uc5b4\uc84c\uae30 \ub54c\ubb38\uc5d0, \uc544\ud504\ub9ac\uce74\uacc4 \ubbf8\uad6d\uc778\ub4e4\uc740 \ud50c\ub79c\ud14c\uc774\uc158 \ub18d\uc7a5\uc5d0\uc11c \uc18c\uc791\ub18d \uc73c\ub85c\uc11c \uc0dd\uacc4\ub97c \uc774\uc5b4\ub098\uac00\uace0 \uc788\uc5c8\ub2e4. 1934\ub144 \uc5ec\ub984 \uc544\uc774\ub2e4 \uc7ad\uc2a8\uc740 \uc2dc\uace8 \uad50\uc0ac\ub4e4\uc744 \ud6c8\ub828\uc2dc\ud0a4\uae30 \uc704\ud55c \uc5ec\ub984 \ud559\uad50\ub97c \uad6c\uc0c1\ud558\uc600\ub2e4. \uadf8\ub140\ub294 \ucd1d 22\uba85\uc758 \uad50\uc0ac \ud559\uc0dd, 243\uba85\uc758 \uc5b4\ub9b0\uc774\ub4e4\uc744 \ub370\ub9ac\uace0 \ud559\uad50\ub97c \uc6b4\uc601\ud588\ub2e4. \uc5b4\ub978\ub4e4 48\uba85\uc744 \uc704\ud55c 9\uac1c \ud559\uae09\uc744 \uc5f4\uc5c8\ub2e4. \uadf8 \ud559\uad50\uc758 \ub3c4\uc11c\uad00\uc740 2600\uad8c\uc758 \ucc45\uc744 \ubcf4\uc720\ud558\uac8c \ub418\uc5b4, \ud648\uc988 \uce74\uc6b4\ud2f0 \uc804\uccb4\uc5d0\uc11c \uac00\uc7a5 \ud070 \ub3c4\uc11c\uad00\uc774 \ub418\uc5c8\ub2e4.", "paragraph_answer": "\ub300 \uc774\uc8fc \uae30\uac04\uc774\ub358 1933\ub144 4\uc6d4, \uad6d\uc81c \ud68c\uc7a5\uc774\ub358 \uc544\uc774\ub2e4 \uc7ad\uc2a8\uc740 \ubbf8\uc2dc\uc2dc\ud53c \ub809\uc2f1\ud134\uc758 \uc62c \uc138\uc778\ud2b8 \uacf5\uc5c5\ud559\uad50\ub97c \ubc29\ubb38\ud558\uc600\ub2e4. \uadf8\ub140\ub294 \ubbf8\uc2dc\uc2dc\ud53c \ub378\ud0c0\uc758 \uc5b4\ub824\uc6b4 \ud658\uacbd\uc744 \ub3cc\uc544\ubcf4\uc558\ub294\ub370, \uc77c\ubd80 \uad50\uc0ac\ub4e4\uc740 \uacfc\uac70\uc5d0 7\ud559\ub144 \uad50\uc721\uc744 \ubc1b\uc9c0\ub3c4 \uc54a\uc558\ub2e4. \ub18d\uc0b0\ubb3c \uac00\uaca9\uc774 \uacc4\uc18d \ub5a8\uc5b4\uc84c\uae30 \ub54c\ubb38\uc5d0, \uc544\ud504\ub9ac\uce74\uacc4 \ubbf8\uad6d\uc778\ub4e4\uc740 \ud50c\ub79c\ud14c\uc774\uc158 \ub18d\uc7a5\uc5d0\uc11c \uc18c\uc791\ub18d \uc73c\ub85c\uc11c \uc0dd\uacc4\ub97c \uc774\uc5b4\ub098\uac00\uace0 \uc788\uc5c8\ub2e4. 1934\ub144 \uc5ec\ub984 \uc544\uc774\ub2e4 \uc7ad\uc2a8\uc740 \uc2dc\uace8 \uad50\uc0ac\ub4e4\uc744 \ud6c8\ub828\uc2dc\ud0a4\uae30 \uc704\ud55c \uc5ec\ub984 \ud559\uad50\ub97c \uad6c\uc0c1\ud558\uc600\ub2e4. \uadf8\ub140\ub294 \ucd1d 22\uba85\uc758 \uad50\uc0ac \ud559\uc0dd, 243\uba85\uc758 \uc5b4\ub9b0\uc774\ub4e4\uc744 \ub370\ub9ac\uace0 \ud559\uad50\ub97c \uc6b4\uc601\ud588\ub2e4. \uc5b4\ub978\ub4e4 48\uba85\uc744 \uc704\ud55c 9\uac1c \ud559\uae09\uc744 \uc5f4\uc5c8\ub2e4. \uadf8 \ud559\uad50\uc758 \ub3c4\uc11c\uad00\uc740 2600\uad8c\uc758 \ucc45\uc744 \ubcf4\uc720\ud558\uac8c \ub418\uc5b4, \ud648\uc988 \uce74\uc6b4\ud2f0 \uc804\uccb4\uc5d0\uc11c \uac00\uc7a5 \ud070 \ub3c4\uc11c\uad00\uc774 \ub418\uc5c8\ub2e4.", "sentence_answer": "\ub18d\uc0b0\ubb3c \uac00\uaca9\uc774 \uacc4\uc18d \ub5a8\uc5b4\uc84c\uae30 \ub54c\ubb38\uc5d0, \uc544\ud504\ub9ac\uce74\uacc4 \ubbf8\uad6d\uc778\ub4e4\uc740 \ud50c\ub79c\ud14c\uc774\uc158 \ub18d\uc7a5\uc5d0\uc11c \uc18c\uc791\ub18d \uc73c\ub85c\uc11c \uc0dd\uacc4\ub97c \uc774\uc5b4\ub098\uac00\uace0 \uc788\uc5c8\ub2e4.", "paragraph_id": "6604746-2-1", "paragraph_question": "question: 1933\ub144 \ub2f9\uc2dc \uc544\ud504\ub9ac\uce74\uacc4 \ubbf8\uad6d\uc778\ub4e4\uc740 \uc0dd\uacc4\ub97c \uc704\ud574 \ud50c\ub79c\ud14c\uc774\uc158 \ub18d\uc7a5\uc5d0\uc11c \uc5b4\ub5a0\ud55c \uc2e0\ubd84\uc73c\ub85c \uc77c\ud558\uc600\ub098?, context: \ub300 \uc774\uc8fc \uae30\uac04\uc774\ub358 1933\ub144 4\uc6d4, \uad6d\uc81c \ud68c\uc7a5\uc774\ub358 \uc544\uc774\ub2e4 \uc7ad\uc2a8\uc740 \ubbf8\uc2dc\uc2dc\ud53c \ub809\uc2f1\ud134\uc758 \uc62c \uc138\uc778\ud2b8 \uacf5\uc5c5\ud559\uad50\ub97c \ubc29\ubb38\ud558\uc600\ub2e4. \uadf8\ub140\ub294 \ubbf8\uc2dc\uc2dc\ud53c \ub378\ud0c0\uc758 \uc5b4\ub824\uc6b4 \ud658\uacbd\uc744 \ub3cc\uc544\ubcf4\uc558\ub294\ub370, \uc77c\ubd80 \uad50\uc0ac\ub4e4\uc740 \uacfc\uac70\uc5d0 7\ud559\ub144 \uad50\uc721\uc744 \ubc1b\uc9c0\ub3c4 \uc54a\uc558\ub2e4. \ub18d\uc0b0\ubb3c \uac00\uaca9\uc774 \uacc4\uc18d \ub5a8\uc5b4\uc84c\uae30 \ub54c\ubb38\uc5d0, \uc544\ud504\ub9ac\uce74\uacc4 \ubbf8\uad6d\uc778\ub4e4\uc740 \ud50c\ub79c\ud14c\uc774\uc158 \ub18d\uc7a5\uc5d0\uc11c \uc18c\uc791\ub18d\uc73c\ub85c\uc11c \uc0dd\uacc4\ub97c \uc774\uc5b4\ub098\uac00\uace0 \uc788\uc5c8\ub2e4. 1934\ub144 \uc5ec\ub984 \uc544\uc774\ub2e4 \uc7ad\uc2a8\uc740 \uc2dc\uace8 \uad50\uc0ac\ub4e4\uc744 \ud6c8\ub828\uc2dc\ud0a4\uae30 \uc704\ud55c \uc5ec\ub984 \ud559\uad50\ub97c \uad6c\uc0c1\ud558\uc600\ub2e4. \uadf8\ub140\ub294 \ucd1d 22\uba85\uc758 \uad50\uc0ac \ud559\uc0dd, 243\uba85\uc758 \uc5b4\ub9b0\uc774\ub4e4\uc744 \ub370\ub9ac\uace0 \ud559\uad50\ub97c \uc6b4\uc601\ud588\ub2e4. \uc5b4\ub978\ub4e4 48\uba85\uc744 \uc704\ud55c 9\uac1c \ud559\uae09\uc744 \uc5f4\uc5c8\ub2e4. \uadf8 \ud559\uad50\uc758 \ub3c4\uc11c\uad00\uc740 2600\uad8c\uc758 \ucc45\uc744 \ubcf4\uc720\ud558\uac8c \ub418\uc5b4, \ud648\uc988 \uce74\uc6b4\ud2f0 \uc804\uccb4\uc5d0\uc11c \uac00\uc7a5 \ud070 \ub3c4\uc11c\uad00\uc774 \ub418\uc5c8\ub2e4."} {"question": "\ub4dc\ub77c\ub9c8 <\uad81S>\uc5d0\uc11c \ubc15\uc2e0\ud61c\uac00 \ub9e1\uc740 \uadf9\uc911 \uc778\ubb3c\uc758 \uc774\ub984\uc740?", "paragraph": "\ubc15\uc2e0\ud61c\ub294 \uc774\ud6c4 \ub2e8\ub9c9\uadf9 \u300a\ub4dc\ub77c\ub9c8\uc2dc\ud2f0 - \uccab\ucc28\ub97c \uae30\ub2e4\ub9ac\uba70\u300b(2004), \u300a\ubca0\uc2a4\ud2b8\uadf9\uc7a5 - \ubca0\ub9ac \uba54\ub9ac \ud06c\ub9ac\uc2a4\ub9c8\uc2a4\u300b(2004), \u300a\uc0c8 \uc544\ube60\ub294 \uc2a4\ubb3c \uc544\ud649\u300b(2005)\uc640 \u300a\ud55c\ubf18 \ub4dc\ub77c\ub9c8\u300b(2004), \u300a\ubca0\uc2a4\ud2b8\uadf9\uc7a5 - \uc5b4\ub290 \uba4b\uc9c4 \ub0a0\u300b(2005), \u300a\ub4dc\ub77c\ub9c8\uc2dc\ud2f0 - \ud0a4\ub2e4\ub9ac \uc544\uc800\uc528\u300b(2005)\uc640 \uc2dc\ud2b8\ucf64 \u300a\ud63c\uc790\uac00 \uc544\ub2c8\uc57c\u300b(2004-05), \u300a\uadc0\uc5fd\uac70\ub098 \ubbf8\uce58\uac70\ub098\u300b(2005)\uc5d0 \ucd9c\uc5f0\ud558\uba70 \ubc30\uc6b0\ub85c\uc11c \uc5f0\uae30\ub825\uc744 \ub2e4\uc838\ub098\uac14\ub2e4. 2004\ub144\ubd80\ud130 2005\ub144\uae4c\uc9c0 \ubc15\uc2e0\ud61c\ub294 \uc0ac\uc804 \uc81c\uc791 \ub41c \ud4e8\uc804 \uc0ac\uadf9 \ub4dc\ub77c\ub9c8 \u300a\ube44\ucc9c\ubb34\u300b\uc5d0 \uce90\uc2a4\ud305 \ub418\uc5b4 \uc911\uad6d \ub85c\ucf00 \ucd2c\uc601\ud558\uc600\ub2e4. \uadf8\ub9ac\uace0 2006\ub144 2\uc6d4, \uccab \ubc29\uc601\ud55c \ub4dc\ub77c\ub9c8 \u300a\ucc9c\uad6d\uc758 \ub098\ubb34\u300b\uc5d0\uc11c \uc5ec\uc8fc\uc778\uacf5 \ud558\ub098 \uc5ed\uc5d0 \uce90\uc2a4\ud305 \ub418\uc5c8\uace0, \u2018\ucc9c\uad6d\uc758 \uacc4\ub2e8\u2019\uc5d0\uc11c \uadf8\ub140\ub97c \ubc1c\ud0c1\ud588\ub358 \uc774\uc7a5\uc218 \uac10\ub3c5\uc740 \uc5ec\uc8fc\uc778\uacf5\uc73c\ub85c \uace0\ub4f1\ud559\uc0dd\uc774\uc5c8\ub358 \ubc15\uc2e0\ud61c\ub97c \ub610\ub2e4\uc2dc \ubc1c\ud0c1\ud588\ub2e4. \uac19\uc740 \ud574 4\uc6d4, \uc601\ud654 \u300a\ub3c4\ub9c8\ubc40\u300b\uc73c\ub85c \uccab \uc2a4\ud06c\ub9b0 \ub370\ubdd4\ub97c \ud558\uc600\ub2e4. \uc774\ub4ec\ud574, 2007\ub144 \ud669\uc778\ub8b0 \uac10\ub3c5\uc740 \ub4dc\ub77c\ub9c8 \u300a\ube44\ucc9c\ubb34\u300b\uc758 \uc81c\uc791 \ud604\uc7a5\uc5d0\uc11c \ub208\uc5ec\uaca8\ubcf8 \ubc15\uc2e0\ud61c\uc5d0\uac8c \ub4dc\ub77c\ub9c8 \u300a\uad81S\u300b\uc758 \uadc0\uc871 \uc18c\ub140 \uc2e0\uc138\ub839 \uc5ed\uc73c\ub85c \uce90\uc2a4\ud305\ud588\uc73c\ub098 \uc774 \ub4dc\ub77c\ub9c8\ub294 \uc804\uc791\uacfc \ub2ec\ub9ac \uc9c4\ubd80\ud55c \ub0b4\uc6a9\uacfc \uc124\uc815\uc73c\ub85c \ud765\ud589\uc5d0\ub294 \ud06c\uac8c \uc2e4\ud328\ud558\uc600\ub2e4. \uac19\uc740 \ud574 5\uc6d4 \uac1c\ubd09 \ub41c \ud638\ub7ec \uc601\ud654 \u300a\uc804\uc124\uc758 \uace0\ud5a5\u300b\uacfc \ub4dc\ub77c\ub9c8 \u300a\uc6b0\ub9ac\ub97c \ud589\ubcf5\ud558\uac8c \ud558\ub294 \uba87 \uac00\uc9c0 \uc9c8\ubb38\u300b\uc5d0 \ucd9c\uc5f0\ud588\ub2e4. 7\uc6d4\ubd80\ud130 \ubc29\uc601 \ub41c \ub4dc\ub77c\ub9c8 \u300a\uae4d\ub450\uae30\u300b\uc5d0\uc11c\ub294 \uc808\uc5d0\uc11c \uc790\ub780 \uc5ec\uc8fc\uc778\uacf5 \uc7a5\uc0ac\uc57c \uc5ed\uc744 \ub9e1\uc544 \uc9c0\uae08\uae4c\uc9c0 \uace0\uc218\ud574\uc654\ub358 \uae34 \uba38\ub9ac\uce74\ub77d\uc744 \uc9e7\uac8c \uc790\ub974\ub294 \ub4f1 \ud30c\uaca9 \ubcc0\uc2e0\uc744 \ud558\uc600\ub2e4. \ubc15\uc2e0\ud61c\ub294 2007\ub144\ubd80\ud130 2008\ub144\uae4c\uc9c0 \uc608\ub2a5 \ud504\ub85c\uadf8\ub7a8 \u300a\ud658\uc0c1\uc758 \uc9dd\uafcd\u300b\uc758 \uc9c4\ud589\uc744 \ub9e1\uc544 MBC \ubc29\uc1a1\uc5f0\uc608\ub300\uc0c1 \uc1fc\ubc84\ub77c\uc774\uc5b4\ud2f0\ubd80\ubb38 \uc5ec\uc790\uc2e0\uc778\uc0c1\uc744 \uc218\uc0c1\ud588\ub2e4.", "answer": "\uc2e0\uc138\ub839", "sentence": "\uc774\ub4ec\ud574, 2007\ub144 \ud669\uc778\ub8b0 \uac10\ub3c5\uc740 \ub4dc\ub77c\ub9c8 \u300a\ube44\ucc9c\ubb34\u300b\uc758 \uc81c\uc791 \ud604\uc7a5\uc5d0\uc11c \ub208\uc5ec\uaca8\ubcf8 \ubc15\uc2e0\ud61c\uc5d0\uac8c \ub4dc\ub77c\ub9c8 \u300a\uad81S\u300b\uc758 \uadc0\uc871 \uc18c\ub140 \uc2e0\uc138\ub839 \uc5ed\uc73c\ub85c \uce90\uc2a4\ud305\ud588\uc73c\ub098 \uc774 \ub4dc\ub77c\ub9c8\ub294 \uc804\uc791\uacfc \ub2ec\ub9ac \uc9c4\ubd80\ud55c \ub0b4\uc6a9\uacfc \uc124\uc815\uc73c\ub85c \ud765\ud589\uc5d0\ub294 \ud06c\uac8c \uc2e4\ud328\ud558\uc600\ub2e4.", "paragraph_sentence": "\ubc15\uc2e0\ud61c\ub294 \uc774\ud6c4 \ub2e8\ub9c9\uadf9 \u300a\ub4dc\ub77c\ub9c8\uc2dc\ud2f0 - \uccab\ucc28\ub97c \uae30\ub2e4\ub9ac\uba70\u300b(2004), \u300a\ubca0\uc2a4\ud2b8\uadf9\uc7a5 - \ubca0\ub9ac \uba54\ub9ac \ud06c\ub9ac\uc2a4\ub9c8\uc2a4\u300b(2004), \u300a\uc0c8 \uc544\ube60\ub294 \uc2a4\ubb3c \uc544\ud649\u300b(2005)\uc640 \u300a\ud55c\ubf18 \ub4dc\ub77c\ub9c8\u300b(2004), \u300a\ubca0\uc2a4\ud2b8\uadf9\uc7a5 - \uc5b4\ub290 \uba4b\uc9c4 \ub0a0\u300b(2005), \u300a\ub4dc\ub77c\ub9c8\uc2dc\ud2f0 - \ud0a4\ub2e4\ub9ac \uc544\uc800\uc528\u300b(2005)\uc640 \uc2dc\ud2b8\ucf64 \u300a\ud63c\uc790\uac00 \uc544\ub2c8\uc57c\u300b(2004-05), \u300a\uadc0\uc5fd\uac70\ub098 \ubbf8\uce58\uac70\ub098\u300b(2005)\uc5d0 \ucd9c\uc5f0\ud558\uba70 \ubc30\uc6b0\ub85c\uc11c \uc5f0\uae30\ub825\uc744 \ub2e4\uc838\ub098\uac14\ub2e4. 2004\ub144\ubd80\ud130 2005\ub144\uae4c\uc9c0 \ubc15\uc2e0\ud61c\ub294 \uc0ac\uc804 \uc81c\uc791 \ub41c \ud4e8\uc804 \uc0ac\uadf9 \ub4dc\ub77c\ub9c8 \u300a\ube44\ucc9c\ubb34\u300b\uc5d0 \uce90\uc2a4\ud305 \ub418\uc5b4 \uc911\uad6d \ub85c\ucf00 \ucd2c\uc601\ud558\uc600\ub2e4. \uadf8\ub9ac\uace0 2006\ub144 2\uc6d4, \uccab \ubc29\uc601\ud55c \ub4dc\ub77c\ub9c8 \u300a\ucc9c\uad6d\uc758 \ub098\ubb34\u300b\uc5d0\uc11c \uc5ec\uc8fc\uc778\uacf5 \ud558\ub098 \uc5ed\uc5d0 \uce90\uc2a4\ud305 \ub418\uc5c8\uace0, \u2018\ucc9c\uad6d\uc758 \uacc4\ub2e8\u2019\uc5d0\uc11c \uadf8\ub140\ub97c \ubc1c\ud0c1\ud588\ub358 \uc774\uc7a5\uc218 \uac10\ub3c5\uc740 \uc5ec\uc8fc\uc778\uacf5\uc73c\ub85c \uace0\ub4f1\ud559\uc0dd\uc774\uc5c8\ub358 \ubc15\uc2e0\ud61c\ub97c \ub610\ub2e4\uc2dc \ubc1c\ud0c1\ud588\ub2e4. \uac19\uc740 \ud574 4\uc6d4, \uc601\ud654 \u300a\ub3c4\ub9c8\ubc40\u300b\uc73c\ub85c \uccab \uc2a4\ud06c\ub9b0 \ub370\ubdd4\ub97c \ud558\uc600\ub2e4. \uc774\ub4ec\ud574, 2007\ub144 \ud669\uc778\ub8b0 \uac10\ub3c5\uc740 \ub4dc\ub77c\ub9c8 \u300a\ube44\ucc9c\ubb34\u300b\uc758 \uc81c\uc791 \ud604\uc7a5\uc5d0\uc11c \ub208\uc5ec\uaca8\ubcf8 \ubc15\uc2e0\ud61c\uc5d0\uac8c \ub4dc\ub77c\ub9c8 \u300a\uad81S\u300b\uc758 \uadc0\uc871 \uc18c\ub140 \uc2e0\uc138\ub839 \uc5ed\uc73c\ub85c \uce90\uc2a4\ud305\ud588\uc73c\ub098 \uc774 \ub4dc\ub77c\ub9c8\ub294 \uc804\uc791\uacfc \ub2ec\ub9ac \uc9c4\ubd80\ud55c \ub0b4\uc6a9\uacfc \uc124\uc815\uc73c\ub85c \ud765\ud589\uc5d0\ub294 \ud06c\uac8c \uc2e4\ud328\ud558\uc600\ub2e4. \uac19\uc740 \ud574 5\uc6d4 \uac1c\ubd09 \ub41c \ud638\ub7ec \uc601\ud654 \u300a\uc804\uc124\uc758 \uace0\ud5a5\u300b\uacfc \ub4dc\ub77c\ub9c8 \u300a\uc6b0\ub9ac\ub97c \ud589\ubcf5\ud558\uac8c \ud558\ub294 \uba87 \uac00\uc9c0 \uc9c8\ubb38\u300b\uc5d0 \ucd9c\uc5f0\ud588\ub2e4. 7\uc6d4\ubd80\ud130 \ubc29\uc601 \ub41c \ub4dc\ub77c\ub9c8 \u300a\uae4d\ub450\uae30\u300b\uc5d0\uc11c\ub294 \uc808\uc5d0\uc11c \uc790\ub780 \uc5ec\uc8fc\uc778\uacf5 \uc7a5\uc0ac\uc57c \uc5ed\uc744 \ub9e1\uc544 \uc9c0\uae08\uae4c\uc9c0 \uace0\uc218\ud574\uc654\ub358 \uae34 \uba38\ub9ac\uce74\ub77d\uc744 \uc9e7\uac8c \uc790\ub974\ub294 \ub4f1 \ud30c\uaca9 \ubcc0\uc2e0\uc744 \ud558\uc600\ub2e4. \ubc15\uc2e0\ud61c\ub294 2007\ub144\ubd80\ud130 2008\ub144\uae4c\uc9c0 \uc608\ub2a5 \ud504\ub85c\uadf8\ub7a8 \u300a\ud658\uc0c1\uc758 \uc9dd\uafcd\u300b\uc758 \uc9c4\ud589\uc744 \ub9e1\uc544 MBC \ubc29\uc1a1\uc5f0\uc608\ub300\uc0c1 \uc1fc\ubc84\ub77c\uc774\uc5b4\ud2f0\ubd80\ubb38 \uc5ec\uc790\uc2e0\uc778\uc0c1\uc744 \uc218\uc0c1\ud588\ub2e4.", "paragraph_answer": "\ubc15\uc2e0\ud61c\ub294 \uc774\ud6c4 \ub2e8\ub9c9\uadf9 \u300a\ub4dc\ub77c\ub9c8\uc2dc\ud2f0 - \uccab\ucc28\ub97c \uae30\ub2e4\ub9ac\uba70\u300b(2004), \u300a\ubca0\uc2a4\ud2b8\uadf9\uc7a5 - \ubca0\ub9ac \uba54\ub9ac \ud06c\ub9ac\uc2a4\ub9c8\uc2a4\u300b(2004), \u300a\uc0c8 \uc544\ube60\ub294 \uc2a4\ubb3c \uc544\ud649\u300b(2005)\uc640 \u300a\ud55c\ubf18 \ub4dc\ub77c\ub9c8\u300b(2004), \u300a\ubca0\uc2a4\ud2b8\uadf9\uc7a5 - \uc5b4\ub290 \uba4b\uc9c4 \ub0a0\u300b(2005), \u300a\ub4dc\ub77c\ub9c8\uc2dc\ud2f0 - \ud0a4\ub2e4\ub9ac \uc544\uc800\uc528\u300b(2005)\uc640 \uc2dc\ud2b8\ucf64 \u300a\ud63c\uc790\uac00 \uc544\ub2c8\uc57c\u300b(2004-05), \u300a\uadc0\uc5fd\uac70\ub098 \ubbf8\uce58\uac70\ub098\u300b(2005)\uc5d0 \ucd9c\uc5f0\ud558\uba70 \ubc30\uc6b0\ub85c\uc11c \uc5f0\uae30\ub825\uc744 \ub2e4\uc838\ub098\uac14\ub2e4. 2004\ub144\ubd80\ud130 2005\ub144\uae4c\uc9c0 \ubc15\uc2e0\ud61c\ub294 \uc0ac\uc804 \uc81c\uc791 \ub41c \ud4e8\uc804 \uc0ac\uadf9 \ub4dc\ub77c\ub9c8 \u300a\ube44\ucc9c\ubb34\u300b\uc5d0 \uce90\uc2a4\ud305 \ub418\uc5b4 \uc911\uad6d \ub85c\ucf00 \ucd2c\uc601\ud558\uc600\ub2e4. \uadf8\ub9ac\uace0 2006\ub144 2\uc6d4, \uccab \ubc29\uc601\ud55c \ub4dc\ub77c\ub9c8 \u300a\ucc9c\uad6d\uc758 \ub098\ubb34\u300b\uc5d0\uc11c \uc5ec\uc8fc\uc778\uacf5 \ud558\ub098 \uc5ed\uc5d0 \uce90\uc2a4\ud305 \ub418\uc5c8\uace0, \u2018\ucc9c\uad6d\uc758 \uacc4\ub2e8\u2019\uc5d0\uc11c \uadf8\ub140\ub97c \ubc1c\ud0c1\ud588\ub358 \uc774\uc7a5\uc218 \uac10\ub3c5\uc740 \uc5ec\uc8fc\uc778\uacf5\uc73c\ub85c \uace0\ub4f1\ud559\uc0dd\uc774\uc5c8\ub358 \ubc15\uc2e0\ud61c\ub97c \ub610\ub2e4\uc2dc \ubc1c\ud0c1\ud588\ub2e4. \uac19\uc740 \ud574 4\uc6d4, \uc601\ud654 \u300a\ub3c4\ub9c8\ubc40\u300b\uc73c\ub85c \uccab \uc2a4\ud06c\ub9b0 \ub370\ubdd4\ub97c \ud558\uc600\ub2e4. \uc774\ub4ec\ud574, 2007\ub144 \ud669\uc778\ub8b0 \uac10\ub3c5\uc740 \ub4dc\ub77c\ub9c8 \u300a\ube44\ucc9c\ubb34\u300b\uc758 \uc81c\uc791 \ud604\uc7a5\uc5d0\uc11c \ub208\uc5ec\uaca8\ubcf8 \ubc15\uc2e0\ud61c\uc5d0\uac8c \ub4dc\ub77c\ub9c8 \u300a\uad81S\u300b\uc758 \uadc0\uc871 \uc18c\ub140 \uc2e0\uc138\ub839 \uc5ed\uc73c\ub85c \uce90\uc2a4\ud305\ud588\uc73c\ub098 \uc774 \ub4dc\ub77c\ub9c8\ub294 \uc804\uc791\uacfc \ub2ec\ub9ac \uc9c4\ubd80\ud55c \ub0b4\uc6a9\uacfc \uc124\uc815\uc73c\ub85c \ud765\ud589\uc5d0\ub294 \ud06c\uac8c \uc2e4\ud328\ud558\uc600\ub2e4. \uac19\uc740 \ud574 5\uc6d4 \uac1c\ubd09 \ub41c \ud638\ub7ec \uc601\ud654 \u300a\uc804\uc124\uc758 \uace0\ud5a5\u300b\uacfc \ub4dc\ub77c\ub9c8 \u300a\uc6b0\ub9ac\ub97c \ud589\ubcf5\ud558\uac8c \ud558\ub294 \uba87 \uac00\uc9c0 \uc9c8\ubb38\u300b\uc5d0 \ucd9c\uc5f0\ud588\ub2e4. 7\uc6d4\ubd80\ud130 \ubc29\uc601 \ub41c \ub4dc\ub77c\ub9c8 \u300a\uae4d\ub450\uae30\u300b\uc5d0\uc11c\ub294 \uc808\uc5d0\uc11c \uc790\ub780 \uc5ec\uc8fc\uc778\uacf5 \uc7a5\uc0ac\uc57c \uc5ed\uc744 \ub9e1\uc544 \uc9c0\uae08\uae4c\uc9c0 \uace0\uc218\ud574\uc654\ub358 \uae34 \uba38\ub9ac\uce74\ub77d\uc744 \uc9e7\uac8c \uc790\ub974\ub294 \ub4f1 \ud30c\uaca9 \ubcc0\uc2e0\uc744 \ud558\uc600\ub2e4. \ubc15\uc2e0\ud61c\ub294 2007\ub144\ubd80\ud130 2008\ub144\uae4c\uc9c0 \uc608\ub2a5 \ud504\ub85c\uadf8\ub7a8 \u300a\ud658\uc0c1\uc758 \uc9dd\uafcd\u300b\uc758 \uc9c4\ud589\uc744 \ub9e1\uc544 MBC \ubc29\uc1a1\uc5f0\uc608\ub300\uc0c1 \uc1fc\ubc84\ub77c\uc774\uc5b4\ud2f0\ubd80\ubb38 \uc5ec\uc790\uc2e0\uc778\uc0c1\uc744 \uc218\uc0c1\ud588\ub2e4.", "sentence_answer": "\uc774\ub4ec\ud574, 2007\ub144 \ud669\uc778\ub8b0 \uac10\ub3c5\uc740 \ub4dc\ub77c\ub9c8 \u300a\ube44\ucc9c\ubb34\u300b\uc758 \uc81c\uc791 \ud604\uc7a5\uc5d0\uc11c \ub208\uc5ec\uaca8\ubcf8 \ubc15\uc2e0\ud61c\uc5d0\uac8c \ub4dc\ub77c\ub9c8 \u300a\uad81S\u300b\uc758 \uadc0\uc871 \uc18c\ub140 \uc2e0\uc138\ub839 \uc5ed\uc73c\ub85c \uce90\uc2a4\ud305\ud588\uc73c\ub098 \uc774 \ub4dc\ub77c\ub9c8\ub294 \uc804\uc791\uacfc \ub2ec\ub9ac \uc9c4\ubd80\ud55c \ub0b4\uc6a9\uacfc \uc124\uc815\uc73c\ub85c \ud765\ud589\uc5d0\ub294 \ud06c\uac8c \uc2e4\ud328\ud558\uc600\ub2e4.", "paragraph_id": "6507513-1-2", "paragraph_question": "question: \ub4dc\ub77c\ub9c8 <\uad81S>\uc5d0\uc11c \ubc15\uc2e0\ud61c\uac00 \ub9e1\uc740 \uadf9\uc911 \uc778\ubb3c\uc758 \uc774\ub984\uc740?, context: \ubc15\uc2e0\ud61c\ub294 \uc774\ud6c4 \ub2e8\ub9c9\uadf9 \u300a\ub4dc\ub77c\ub9c8\uc2dc\ud2f0 - \uccab\ucc28\ub97c \uae30\ub2e4\ub9ac\uba70\u300b(2004), \u300a\ubca0\uc2a4\ud2b8\uadf9\uc7a5 - \ubca0\ub9ac \uba54\ub9ac \ud06c\ub9ac\uc2a4\ub9c8\uc2a4\u300b(2004), \u300a\uc0c8 \uc544\ube60\ub294 \uc2a4\ubb3c \uc544\ud649\u300b(2005)\uc640 \u300a\ud55c\ubf18 \ub4dc\ub77c\ub9c8\u300b(2004), \u300a\ubca0\uc2a4\ud2b8\uadf9\uc7a5 - \uc5b4\ub290 \uba4b\uc9c4 \ub0a0\u300b(2005), \u300a\ub4dc\ub77c\ub9c8\uc2dc\ud2f0 - \ud0a4\ub2e4\ub9ac \uc544\uc800\uc528\u300b(2005)\uc640 \uc2dc\ud2b8\ucf64 \u300a\ud63c\uc790\uac00 \uc544\ub2c8\uc57c\u300b(2004-05), \u300a\uadc0\uc5fd\uac70\ub098 \ubbf8\uce58\uac70\ub098\u300b(2005)\uc5d0 \ucd9c\uc5f0\ud558\uba70 \ubc30\uc6b0\ub85c\uc11c \uc5f0\uae30\ub825\uc744 \ub2e4\uc838\ub098\uac14\ub2e4. 2004\ub144\ubd80\ud130 2005\ub144\uae4c\uc9c0 \ubc15\uc2e0\ud61c\ub294 \uc0ac\uc804 \uc81c\uc791 \ub41c \ud4e8\uc804 \uc0ac\uadf9 \ub4dc\ub77c\ub9c8 \u300a\ube44\ucc9c\ubb34\u300b\uc5d0 \uce90\uc2a4\ud305 \ub418\uc5b4 \uc911\uad6d \ub85c\ucf00 \ucd2c\uc601\ud558\uc600\ub2e4. \uadf8\ub9ac\uace0 2006\ub144 2\uc6d4, \uccab \ubc29\uc601\ud55c \ub4dc\ub77c\ub9c8 \u300a\ucc9c\uad6d\uc758 \ub098\ubb34\u300b\uc5d0\uc11c \uc5ec\uc8fc\uc778\uacf5 \ud558\ub098 \uc5ed\uc5d0 \uce90\uc2a4\ud305 \ub418\uc5c8\uace0, \u2018\ucc9c\uad6d\uc758 \uacc4\ub2e8\u2019\uc5d0\uc11c \uadf8\ub140\ub97c \ubc1c\ud0c1\ud588\ub358 \uc774\uc7a5\uc218 \uac10\ub3c5\uc740 \uc5ec\uc8fc\uc778\uacf5\uc73c\ub85c \uace0\ub4f1\ud559\uc0dd\uc774\uc5c8\ub358 \ubc15\uc2e0\ud61c\ub97c \ub610\ub2e4\uc2dc \ubc1c\ud0c1\ud588\ub2e4. \uac19\uc740 \ud574 4\uc6d4, \uc601\ud654 \u300a\ub3c4\ub9c8\ubc40\u300b\uc73c\ub85c \uccab \uc2a4\ud06c\ub9b0 \ub370\ubdd4\ub97c \ud558\uc600\ub2e4. \uc774\ub4ec\ud574, 2007\ub144 \ud669\uc778\ub8b0 \uac10\ub3c5\uc740 \ub4dc\ub77c\ub9c8 \u300a\ube44\ucc9c\ubb34\u300b\uc758 \uc81c\uc791 \ud604\uc7a5\uc5d0\uc11c \ub208\uc5ec\uaca8\ubcf8 \ubc15\uc2e0\ud61c\uc5d0\uac8c \ub4dc\ub77c\ub9c8 \u300a\uad81S\u300b\uc758 \uadc0\uc871 \uc18c\ub140 \uc2e0\uc138\ub839 \uc5ed\uc73c\ub85c \uce90\uc2a4\ud305\ud588\uc73c\ub098 \uc774 \ub4dc\ub77c\ub9c8\ub294 \uc804\uc791\uacfc \ub2ec\ub9ac \uc9c4\ubd80\ud55c \ub0b4\uc6a9\uacfc \uc124\uc815\uc73c\ub85c \ud765\ud589\uc5d0\ub294 \ud06c\uac8c \uc2e4\ud328\ud558\uc600\ub2e4. \uac19\uc740 \ud574 5\uc6d4 \uac1c\ubd09 \ub41c \ud638\ub7ec \uc601\ud654 \u300a\uc804\uc124\uc758 \uace0\ud5a5\u300b\uacfc \ub4dc\ub77c\ub9c8 \u300a\uc6b0\ub9ac\ub97c \ud589\ubcf5\ud558\uac8c \ud558\ub294 \uba87 \uac00\uc9c0 \uc9c8\ubb38\u300b\uc5d0 \ucd9c\uc5f0\ud588\ub2e4. 7\uc6d4\ubd80\ud130 \ubc29\uc601 \ub41c \ub4dc\ub77c\ub9c8 \u300a\uae4d\ub450\uae30\u300b\uc5d0\uc11c\ub294 \uc808\uc5d0\uc11c \uc790\ub780 \uc5ec\uc8fc\uc778\uacf5 \uc7a5\uc0ac\uc57c \uc5ed\uc744 \ub9e1\uc544 \uc9c0\uae08\uae4c\uc9c0 \uace0\uc218\ud574\uc654\ub358 \uae34 \uba38\ub9ac\uce74\ub77d\uc744 \uc9e7\uac8c \uc790\ub974\ub294 \ub4f1 \ud30c\uaca9 \ubcc0\uc2e0\uc744 \ud558\uc600\ub2e4. \ubc15\uc2e0\ud61c\ub294 2007\ub144\ubd80\ud130 2008\ub144\uae4c\uc9c0 \uc608\ub2a5 \ud504\ub85c\uadf8\ub7a8 \u300a\ud658\uc0c1\uc758 \uc9dd\uafcd\u300b\uc758 \uc9c4\ud589\uc744 \ub9e1\uc544 MBC \ubc29\uc1a1\uc5f0\uc608\ub300\uc0c1 \uc1fc\ubc84\ub77c\uc774\uc5b4\ud2f0\ubd80\ubb38 \uc5ec\uc790\uc2e0\uc778\uc0c1\uc744 \uc218\uc0c1\ud588\ub2e4."} {"question": "\uc774\uc885\uc6b1\uc774 \ud638\uc8fc\uc804\uc5d0\uc11c \ub300\uc2e0 \uc8fc\uc790\ub85c \ub098\uac04 \uc120\uc218\ub294?", "paragraph": "2007\ub144 3\uc6d4\uc5d0 \ub450\uc0b0 \ubca0\uc5b4\uc2a4\uc758 \uac10\ub3c5\uc744 \ub9e1\uace0 \uc788\ub358 \uae40\uacbd\ubb38 \uac10\ub3c5\uc774 \uc774\ubbf8 \ub300\ud45c\ud300 \uac10\ub3c5\uc73c\ub85c \uc120\uc784\ub418\uc5c8\uace0, 11\uc6d4 30\uc77c\uc5d0 \uc544\uc2dc\uc544 \uc57c\uad6c \uc120\uc218\uad8c \ub300\ud68c \uacb8 \uc62c\ub9bc\ud53d \uc608\uc120\uc758 \ucd5c\uc885 \uc5d4\ud2b8\ub9ac\uac00 \ubc1c\ud45c\ub418\uc5c8\ub2e4. \uadf8\ub294 \uc678\uc57c\uc218 \uc911\uc5d0\uc11c\ub294 \uac19\uc740 \ud300 \ubbfc\ubcd1\ud5cc\uacfc \ud568\uaed8 \ud3ec\ud568\ub418\uc5c8\ub2e4. \ub300\ub9cc\uacfc\uc758 \uccab \uacbd\uae30\uc5d0\uc11c 0-1\ub85c \ub4a4\uc9c0\ub294 \uc0c1\ud669\uc778 5\ud68c\ucd08 2\uc0ac 1, 3\ub8e8\uc5d0\uc11c \ub450 \ubc88\uc9f8 \ud0c0\uc11d\uc5d0 \ub4e4\uc5b4\uc11c \uc120\ubc1c \ud22c\uc218 \ub9b0\uc5b8\uc704\ub85c\ubd80\ud130 \uacb0\uc2b9 \uc5ed\uc804 3\uc810 \ud648\ub7f0\uc744 \ubf51\uc544\ub0c8\ub2e4. \uadf8\ub294 \uc77c\ubcf8\uacfc\uc758 \ub450 \ubc88\uc9f8 \uacbd\uae30\uc5d0\uc11c\ub294 1\ubc88 \ud0c0\uc790\ub85c \uc608\uc0c1\ub418\uc5c8\uc73c\ub098, 10\ubd84\uc804\uc5d0 \uc624\ub354\uac00 \ubc14\ub00c\uba74\uc11c 8\ubc88 \ud0c0\uc790\ub85c \ubc14\ub00c\uc5c8\uace0, 2-4\ub85c \ub4a4\uc9c4 8\ud68c \ub9d0\uc5d0 \ud76c\uc0dd \ud50c\ub77c\uc774\ub97c \uce58\uba74\uc11c 1\ud0c0\uc810\uc744 \uc62c\ub838\uc73c\ub098 \ud300\uc740 4-3\uc73c\ub85c \ud328\ud588\ub2e4. \ud544\ub9ac\ud540\uacfc\uc758 \uc138 \ubc88\uc9f8 \uacbd\uae30\uc5d0\uc11c \uc801\uc2dc\ud0c0\ub85c 1\ud0c0\uc810\uc744 \uc62c\ub838\uace0, \ud300\uc740 13-1\uc758 \uc810\uc218\ub85c \ucf5c\ub4dc \uac8c\uc784\uc73c\ub85c \uc2b9\ub9ac\ud588\ub2e4. \uadf8 \uacb0\uacfc \ub300\ud45c\ud300\uc740 2\uc2b9 1\ud328\ub85c 2\uc704\uc5d0 \uba38\ubb3c\uba70 3\uc2b9\uc744 \uae30\ub85d\ud55c \uc77c\ubcf8\uc5d0\uac8c \ubc00\ub824 \uc62c\ub9bc\ud53d \uc9c1\ud589\uc5d0 \uc2e4\ud328\ud588\uace0, 2\ucc28 \uc608\uc120\uc5d0 \ucc38\uac00\ud558\uac8c \ub418\uc5c8\ub2e4. \uc62c\ub9bc\ud53d \ucd5c\uc885 \uc608\uc120 \uc5d4\ud2b8\ub9ac\ub294 2008\ub144 3\uc6d4 5\uc77c \ubc1c\ud45c\ub418\uc5c8\uace0, \uac19\uc740 \ud300 \uc911\uc5d0\ub294 \ud22c\uc218 \uae40\uc120\uc6b0, \ub0b4\uc57c\uc218 \uae40\ub3d9\uc8fc, \uace0\uc601\ubbfc, \uadf8\ub9ac\uace0 \uc0c1\ubb34\uc5d0 \ubcf5\ubb34 \uc911\uc778 \uc190\uc2dc\ud5cc\uacfc \ud568\uaed8 \ud3ec\ud568\ub418\uc5c8\ub2e4. \uccab \uacbd\uae30 \ub0a8\uc544\uacf5\uc804\uc5d0\uc11c \uc120\ub450\ud0c0\uc790\ub85c \ub098\uc640 \uccab \ud0c0\uc11d \ubab8\uc5d0 \ub9de\ub294 \ubcfc\ub85c \ucd9c\ub8e8\ud558\uace0 \ub3c4\ub8e8\uc5d0 \uc131\uacf5\ud55c \ub4a4 \uc774\uc2b9\uc5fd\uc758 2\ub8e8\ud0c0 \ub54c \ub4dd\uc810\uc744 \ud588\ub2e4. \uadf8\ub7ec\ub098 \uc774\ud6c4 4\ud0c0\uc218 \ubb34\uc548\ud0c0\uc5d0 \uadf8\uce58\uba74\uc11c \ud638\uc8fc\uc804\uc5d0\uc11c\ub294 \uc774\uc6a9\uaddc\uc5d0\uac8c \uc120\ubc1c \ucd9c\uc804\uc744 \ub0b4\uc8fc\uc5c8\ub2e4. \ud638\uc8fc\uc804\uc5d0\uc11c \uc9c4\uac11\uc6a9 \ub300\uc2e0 \ub300\uc8fc\uc790\ub85c \ucd9c\uc7a5\ub418\uc5c8\ub2e4. \uadf8\ub294 9\uc77c \uba55\uc2dc\ucf54\uc804\uc5d0\uc11c\ub294 3-1\ub85c \uc774\uae30\uace0 \uc788\ub294 \uc0c1\ud669\uc5d0\uc11c \uc774\ubc88 \ub300\ud68c \ub300\ud45c\ud300 \uccab 3\ub8e8\ud0c0\ub97c \ud130\ub728\ub9ac\uba70 2\ud0c0\uc810\uc744 \uc62c\ub838\uc73c\uba70, 6\ud0c0\uc218 1\uc548\ud0c0\ub97c \uae30\ub85d\ud588\ub2e4. 10\uc77c \uc2a4\ud398\uc778\uc804\uc5d0\uc11c 2\ubc88 \ud0c0\uc790\ub85c \uc120\ubc1c \ucd9c\uc7a5\ud588\uc73c\uba70, 1\uc548\ud0c0 3\ubcfc\ub137\uc73c\ub85c \ub192\uc740 \ucd9c\ub8e8\uc728\uc744 \ubcf4\uc600\ub2e4.\uc774\ud2c0 \ub4a4 \ub3c5\uc77c\uc804\uc5d0\uc11c 2\ubc88 \ud0c0\uc790\ub85c \ucd9c\uc804, \ubcfc\ub137\uc744 \uc5bb\uc5b4\ub0c8\ub2e4. 13\uc77c \uce90\ub098\ub2e4\uc804\uc5d0\uc11c\ub294 2\ubc88 \ud0c0\uc790\ub85c \uc120\ubc1c \ucd9c\uc7a5\ud574 \uccab \ud0c0\uc11d\uc5d0\uc11c 3\ub8e8\ub97c \uaff0\ub6ab\ub294 \uc801\uc2dc 2\ub8e8\ud0c0\ub97c \ud130\ub728\ub824 1\ud0c0\uc810\uc744 \uc62c\ub838\uc73c\ub098 \ud300\uc740 4-3\uc73c\ub85c \ud328\ud588\ub2e4. \ub300\ud55c\ubbfc\uad6d \ub300\ud45c\ud300\uc740 \ub9c8\uc9c0\ub9c9 \uacbd\uae30 \ub300\ub9cc\uc804\uc5d0\uc11c 4-3\uc73c\ub85c \uc2b9\ub9ac\ud558\uba74\uc11c, 2\uc704(6\uc2b9 1\ud328)\ub85c \uc62c\ub9bc\ud53d \ubcf8\uc120\uc5d0 \uc9c4\ucd9c\ud588\ub2e4. \uc62c\ub9bc\ud53d \uc5d4\ud2b8\ub9ac\ub294 2008\ub144 7\uc6d4 14\uc77c \ubc1c\ud45c\ub418\uc5c8\uace0, \uc678\uc57c\uc218 \uc911\uc5d0\uc11c \uac19\uc740 \ud300 \uae40\ud604\uc218\uc640 \ud568\uaed8 \ud3ec\ud568\ub418\uc5c8\ub2e4. \uc62c\ub9bc\ud53d \uc57c\uad6c \ubcf8\uc120\uc740 8\uc6d4 13\uc77c\ubd80\ud130 \uc2dc\uc791\ub418\uc5c8\ub2e4. \uccab \ub0a0 \ubbf8\uad6d\uc804\uc5d0\uc11c 4\ud0c0\uc218 2\uc548\ud0c0 1\ud0c0\uc810 1\ub4dd\uc810\uc744 \uae30\ub85d\ud588\uace0, \ud2b9\ud788 7-7\uc758 \uc0c1\ud669\uc774\ub358 9\ud68c\ub9d0 1\uc0ac 3\ub8e8 \ub05d\ub0b4\uae30 \ud76c\uc0dd \ud50c\ub77c\uc774\ub97c \uccd0\ub0c8\ub2e4. 15\uc77c \uce90\ub098\ub2e4\uc804\uc5d0\uc11c\ub294 4\ud68c 1\uc0ac\uc5d0\uc11c \ube57\ub9de\uc740 \uc911\uc804\uc548\ud0c0\uc131 \ud0c0\uad6c\ub97c \uc804\ub825\uc9c8\uc8fc\ud574 \ub2e4\uc774\ube59 \uce90\uce58\ub97c \ud558\uba70 1-0\uc758 \ub958\ud604\uc9c4\uc758 \uc644\ubd09\uc2b9\uc5d0 \uc77c\uc870\ud588\ub2e4. \ub2e4\uc74c \ub0a0 \uc77c\ubcf8\uc804\uc5d0\uc11c\ub294 9\ud68c\ucd08 2\uc0ac \uc774\ud6c4 3\ub8e8\uc218 \uc55e\uc5d0 \ub5a8\uc5b4\uc9c0\ub294 \uc808\ubb18\ud55c \uae30\uc2b5\ubc88\ud2b8\ub85c 4-2, \uc774\ud6c4 \ub204 \uc0c1\uc5d0 \ub098\uac04 \uc774\ud6c4 \ub3c4\ub8e8\ub97c \uc2dc\ub3c4\ud588\uace0 \ud3ec\uc218 \uc544\ubca0\uc758 \uc545\uc1a1\uad6c\ub85c 3\ub8e8\uc5d0 \uc548\ucc29\ud588\uace0 3\ub8e8\uc8fc\uc790\uac00 \ud648\uc73c\ub85c \ub4e4\uc5b4\uc624\uba70 \ub610 \ub2e4\uc2dc \ud55c \uc810\uc744 \ucd94\uac00\ud574 5-2\ub97c \ub9cc\ub4e4\uc5b4\ub0c8\ub2e4. 17\uc77c\uc5d0\ub294 3\uc77c\uac04 \ubbf8\ub904\uc84c\ub358 \uc911\uad6d\uc804\uc5d0\uc11c\uc758 11\ud68c \uc2b9\ubd80\uce58\uae30\uc5d0\uc11c 2\ub8e8 \uc8fc\uc790\ub85c \ub098\uac00 \uc774\uc2b9\uc5fd\uc758 \ub05d\ub0b4\uae30 \uc548\ud0c0 \ub54c \ud648\uc5d0 \ub4e4\uc5b4\uc640 \ub4dd\uc810\ud574 1-0\uc73c\ub85c \uc2b9\ub9ac\ud588\ub2e4. 18\uc77c \ub300\ub9cc\uc804\uc5d0\uc11c\ub294 1\ud68c \ub0b4\uc57c \uc548\ud0c0\ub97c \uce58\uace0 \ub4dd\uc810\ud588\uc73c\ub098, 7\ud68c\ub9d0\uc5d0\ub294 \uc678\uc57c \ub72c\uacf5\uc744 \ud39c\uc2a4 \uc55e\uc5d0\uc11c \ub193\uce58\ub294 \uc2e4\uc218\ub97c \ubc94\ud588\ub2e4. 19\uc77c \ucfe0\ubc14\uc804\uc5d0\ub294 7\ud68c\uc5d0 \uad50\uccb4 \ud22c\uc785\ub410\uace0 \uc218\ube44\ub97c \ub9c8\uce58\uace0 \ub4e4\uc5b4\uc120 \ud0c0\uc11d\uc5d0\uc11c 1\ud0c0\uc810 \uc801\uc2dc\ud0c0\ub97c \uccd0 \ub0b4\uba70 \uc410\uae30 \ud0c0\uc810\uc744 \ub9cc\ub4e4\uc5b4\ub0c8\ub2e4. 20\uc77c \ub124\ub35c\ub780\ub4dc\uc804\uc5d0\ub294 5\ud0c0\uc218 3\uc548\ud0c0\ub85c \ud65c\uc57d\ud588\uace0, \uc608\uc120 \uc131\uc801\uc740 27\ud0c0\uc218 10\uc548\ud0c0, \ud0c0\uc728 0.370, 3\ud0c0\uc810, 4\ub4dd\uc810, 2\ubcfc\ub137, 2\ub3c4\ub8e8\ub97c \uae30\ub85d\ud588\ub2e4. \uc77c\ubcf8\uacfc\uc758 \uc900\uacb0\uc2b9\uc804\uc5d0\uc11c \ud070 \ud65c\uc57d\uc740 \ud558\uc9c0 \ubabb\ud588\ub2e4. \ucfe0\ubc14\uc640\uc758 \uacb0\uc2b9\uc804\uc5d0\uc11c\ub294 7\ud68c\ucd08 \ubcfc\ub137\uc73c\ub85c \ucd9c\ub8e8\ud588\uace0, \ub2e4\uc74c \ud0c0\uc790 \uc774\uc6a9\uaddc\uac00 2\ub8e8\ud0c0\ub97c \uccd0\ub0c8\uc73c\ub098 \uc544\uc6c3 \uce74\uc6b4\ud2b8\ub97c \ucc29\uac01\ud55c \uadf8\uac00 \ud648\uc73c\ub85c \ub4e4\uc5b4\uc624\uc9c0 \uc54a\ub294 \uc2e4\uc218\ub97c \ubc94\ud588\ub2e4. \ud558\uc9c0\ub9cc \ub300\ud45c\ud300\uc740 3-2\ub85c \uc2b9\ub9ac\ud558\uba74\uc11c 9\uc804 \uc804\uc2b9 \uae08\uba54\ub2ec\uc744 \uc774\ub8e8\uc5b4\ub0c8\ub2e4.", "answer": "\uc9c4\uac11\uc6a9", "sentence": "\ud638\uc8fc\uc804\uc5d0\uc11c \uc9c4\uac11\uc6a9 \ub300\uc2e0 \ub300\uc8fc\uc790\ub85c \ucd9c\uc7a5\ub418\uc5c8\ub2e4.", "paragraph_sentence": "2007\ub144 3\uc6d4\uc5d0 \ub450\uc0b0 \ubca0\uc5b4\uc2a4\uc758 \uac10\ub3c5\uc744 \ub9e1\uace0 \uc788\ub358 \uae40\uacbd\ubb38 \uac10\ub3c5\uc774 \uc774\ubbf8 \ub300\ud45c\ud300 \uac10\ub3c5\uc73c\ub85c \uc120\uc784\ub418\uc5c8\uace0, 11\uc6d4 30\uc77c\uc5d0 \uc544\uc2dc\uc544 \uc57c\uad6c \uc120\uc218\uad8c \ub300\ud68c \uacb8 \uc62c\ub9bc\ud53d \uc608\uc120\uc758 \ucd5c\uc885 \uc5d4\ud2b8\ub9ac\uac00 \ubc1c\ud45c\ub418\uc5c8\ub2e4. \uadf8\ub294 \uc678\uc57c\uc218 \uc911\uc5d0\uc11c\ub294 \uac19\uc740 \ud300 \ubbfc\ubcd1\ud5cc\uacfc \ud568\uaed8 \ud3ec\ud568\ub418\uc5c8\ub2e4. \ub300\ub9cc\uacfc\uc758 \uccab \uacbd\uae30\uc5d0\uc11c 0-1\ub85c \ub4a4\uc9c0\ub294 \uc0c1\ud669\uc778 5\ud68c\ucd08 2\uc0ac 1, 3\ub8e8\uc5d0\uc11c \ub450 \ubc88\uc9f8 \ud0c0\uc11d\uc5d0 \ub4e4\uc5b4\uc11c \uc120\ubc1c \ud22c\uc218 \ub9b0\uc5b8\uc704\ub85c\ubd80\ud130 \uacb0\uc2b9 \uc5ed\uc804 3\uc810 \ud648\ub7f0\uc744 \ubf51\uc544\ub0c8\ub2e4. \uadf8\ub294 \uc77c\ubcf8\uacfc\uc758 \ub450 \ubc88\uc9f8 \uacbd\uae30\uc5d0\uc11c\ub294 1\ubc88 \ud0c0\uc790\ub85c \uc608\uc0c1\ub418\uc5c8\uc73c\ub098, 10\ubd84\uc804\uc5d0 \uc624\ub354\uac00 \ubc14\ub00c\uba74\uc11c 8\ubc88 \ud0c0\uc790\ub85c \ubc14\ub00c\uc5c8\uace0, 2-4\ub85c \ub4a4\uc9c4 8\ud68c \ub9d0\uc5d0 \ud76c\uc0dd \ud50c\ub77c\uc774\ub97c \uce58\uba74\uc11c 1\ud0c0\uc810\uc744 \uc62c\ub838\uc73c\ub098 \ud300\uc740 4-3\uc73c\ub85c \ud328\ud588\ub2e4. \ud544\ub9ac\ud540\uacfc\uc758 \uc138 \ubc88\uc9f8 \uacbd\uae30\uc5d0\uc11c \uc801\uc2dc\ud0c0\ub85c 1\ud0c0\uc810\uc744 \uc62c\ub838\uace0, \ud300\uc740 13-1\uc758 \uc810\uc218\ub85c \ucf5c\ub4dc \uac8c\uc784\uc73c\ub85c \uc2b9\ub9ac\ud588\ub2e4. \uadf8 \uacb0\uacfc \ub300\ud45c\ud300\uc740 2\uc2b9 1\ud328\ub85c 2\uc704\uc5d0 \uba38\ubb3c\uba70 3\uc2b9\uc744 \uae30\ub85d\ud55c \uc77c\ubcf8\uc5d0\uac8c \ubc00\ub824 \uc62c\ub9bc\ud53d \uc9c1\ud589\uc5d0 \uc2e4\ud328\ud588\uace0, 2\ucc28 \uc608\uc120\uc5d0 \ucc38\uac00\ud558\uac8c \ub418\uc5c8\ub2e4. \uc62c\ub9bc\ud53d \ucd5c\uc885 \uc608\uc120 \uc5d4\ud2b8\ub9ac\ub294 2008\ub144 3\uc6d4 5\uc77c \ubc1c\ud45c\ub418\uc5c8\uace0, \uac19\uc740 \ud300 \uc911\uc5d0\ub294 \ud22c\uc218 \uae40\uc120\uc6b0, \ub0b4\uc57c\uc218 \uae40\ub3d9\uc8fc, \uace0\uc601\ubbfc, \uadf8\ub9ac\uace0 \uc0c1\ubb34\uc5d0 \ubcf5\ubb34 \uc911\uc778 \uc190\uc2dc\ud5cc\uacfc \ud568\uaed8 \ud3ec\ud568\ub418\uc5c8\ub2e4. \uccab \uacbd\uae30 \ub0a8\uc544\uacf5\uc804\uc5d0\uc11c \uc120\ub450\ud0c0\uc790\ub85c \ub098\uc640 \uccab \ud0c0\uc11d \ubab8\uc5d0 \ub9de\ub294 \ubcfc\ub85c \ucd9c\ub8e8\ud558\uace0 \ub3c4\ub8e8\uc5d0 \uc131\uacf5\ud55c \ub4a4 \uc774\uc2b9\uc5fd\uc758 2\ub8e8\ud0c0 \ub54c \ub4dd\uc810\uc744 \ud588\ub2e4. \uadf8\ub7ec\ub098 \uc774\ud6c4 4\ud0c0\uc218 \ubb34\uc548\ud0c0\uc5d0 \uadf8\uce58\uba74\uc11c \ud638\uc8fc\uc804\uc5d0\uc11c\ub294 \uc774\uc6a9\uaddc\uc5d0\uac8c \uc120\ubc1c \ucd9c\uc804\uc744 \ub0b4\uc8fc\uc5c8\ub2e4. \ud638\uc8fc\uc804\uc5d0\uc11c \uc9c4\uac11\uc6a9 \ub300\uc2e0 \ub300\uc8fc\uc790\ub85c \ucd9c\uc7a5\ub418\uc5c8\ub2e4. \uadf8\ub294 9\uc77c \uba55\uc2dc\ucf54\uc804\uc5d0\uc11c\ub294 3-1\ub85c \uc774\uae30\uace0 \uc788\ub294 \uc0c1\ud669\uc5d0\uc11c \uc774\ubc88 \ub300\ud68c \ub300\ud45c\ud300 \uccab 3\ub8e8\ud0c0\ub97c \ud130\ub728\ub9ac\uba70 2\ud0c0\uc810\uc744 \uc62c\ub838\uc73c\uba70, 6\ud0c0\uc218 1\uc548\ud0c0\ub97c \uae30\ub85d\ud588\ub2e4. 10\uc77c \uc2a4\ud398\uc778\uc804\uc5d0\uc11c 2\ubc88 \ud0c0\uc790\ub85c \uc120\ubc1c \ucd9c\uc7a5\ud588\uc73c\uba70, 1\uc548\ud0c0 3\ubcfc\ub137\uc73c\ub85c \ub192\uc740 \ucd9c\ub8e8\uc728\uc744 \ubcf4\uc600\ub2e4.\uc774\ud2c0 \ub4a4 \ub3c5\uc77c\uc804\uc5d0\uc11c 2\ubc88 \ud0c0\uc790\ub85c \ucd9c\uc804, \ubcfc\ub137\uc744 \uc5bb\uc5b4\ub0c8\ub2e4. 13\uc77c \uce90\ub098\ub2e4\uc804\uc5d0\uc11c\ub294 2\ubc88 \ud0c0\uc790\ub85c \uc120\ubc1c \ucd9c\uc7a5\ud574 \uccab \ud0c0\uc11d\uc5d0\uc11c 3\ub8e8\ub97c \uaff0\ub6ab\ub294 \uc801\uc2dc 2\ub8e8\ud0c0\ub97c \ud130\ub728\ub824 1\ud0c0\uc810\uc744 \uc62c\ub838\uc73c\ub098 \ud300\uc740 4-3\uc73c\ub85c \ud328\ud588\ub2e4. \ub300\ud55c\ubbfc\uad6d \ub300\ud45c\ud300\uc740 \ub9c8\uc9c0\ub9c9 \uacbd\uae30 \ub300\ub9cc\uc804\uc5d0\uc11c 4-3\uc73c\ub85c \uc2b9\ub9ac\ud558\uba74\uc11c, 2\uc704(6\uc2b9 1\ud328)\ub85c \uc62c\ub9bc\ud53d \ubcf8\uc120\uc5d0 \uc9c4\ucd9c\ud588\ub2e4. \uc62c\ub9bc\ud53d \uc5d4\ud2b8\ub9ac\ub294 2008\ub144 7\uc6d4 14\uc77c \ubc1c\ud45c\ub418\uc5c8\uace0, \uc678\uc57c\uc218 \uc911\uc5d0\uc11c \uac19\uc740 \ud300 \uae40\ud604\uc218\uc640 \ud568\uaed8 \ud3ec\ud568\ub418\uc5c8\ub2e4. \uc62c\ub9bc\ud53d \uc57c\uad6c \ubcf8\uc120\uc740 8\uc6d4 13\uc77c\ubd80\ud130 \uc2dc\uc791\ub418\uc5c8\ub2e4. \uccab \ub0a0 \ubbf8\uad6d\uc804\uc5d0\uc11c 4\ud0c0\uc218 2\uc548\ud0c0 1\ud0c0\uc810 1\ub4dd\uc810\uc744 \uae30\ub85d\ud588\uace0, \ud2b9\ud788 7-7\uc758 \uc0c1\ud669\uc774\ub358 9\ud68c\ub9d0 1\uc0ac 3\ub8e8 \ub05d\ub0b4\uae30 \ud76c\uc0dd \ud50c\ub77c\uc774\ub97c \uccd0\ub0c8\ub2e4. 15\uc77c \uce90\ub098\ub2e4\uc804\uc5d0\uc11c\ub294 4\ud68c 1\uc0ac\uc5d0\uc11c \ube57\ub9de\uc740 \uc911\uc804\uc548\ud0c0\uc131 \ud0c0\uad6c\ub97c \uc804\ub825\uc9c8\uc8fc\ud574 \ub2e4\uc774\ube59 \uce90\uce58\ub97c \ud558\uba70 1-0\uc758 \ub958\ud604\uc9c4\uc758 \uc644\ubd09\uc2b9\uc5d0 \uc77c\uc870\ud588\ub2e4. \ub2e4\uc74c \ub0a0 \uc77c\ubcf8\uc804\uc5d0\uc11c\ub294 9\ud68c\ucd08 2\uc0ac \uc774\ud6c4 3\ub8e8\uc218 \uc55e\uc5d0 \ub5a8\uc5b4\uc9c0\ub294 \uc808\ubb18\ud55c \uae30\uc2b5\ubc88\ud2b8\ub85c 4-2, \uc774\ud6c4 \ub204 \uc0c1\uc5d0 \ub098\uac04 \uc774\ud6c4 \ub3c4\ub8e8\ub97c \uc2dc\ub3c4\ud588\uace0 \ud3ec\uc218 \uc544\ubca0\uc758 \uc545\uc1a1\uad6c\ub85c 3\ub8e8\uc5d0 \uc548\ucc29\ud588\uace0 3\ub8e8\uc8fc\uc790\uac00 \ud648\uc73c\ub85c \ub4e4\uc5b4\uc624\uba70 \ub610 \ub2e4\uc2dc \ud55c \uc810\uc744 \ucd94\uac00\ud574 5-2\ub97c \ub9cc\ub4e4\uc5b4\ub0c8\ub2e4. 17\uc77c\uc5d0\ub294 3\uc77c\uac04 \ubbf8\ub904\uc84c\ub358 \uc911\uad6d\uc804\uc5d0\uc11c\uc758 11\ud68c \uc2b9\ubd80\uce58\uae30\uc5d0\uc11c 2\ub8e8 \uc8fc\uc790\ub85c \ub098\uac00 \uc774\uc2b9\uc5fd\uc758 \ub05d\ub0b4\uae30 \uc548\ud0c0 \ub54c \ud648\uc5d0 \ub4e4\uc5b4\uc640 \ub4dd\uc810\ud574 1-0\uc73c\ub85c \uc2b9\ub9ac\ud588\ub2e4. 18\uc77c \ub300\ub9cc\uc804\uc5d0\uc11c\ub294 1\ud68c \ub0b4\uc57c \uc548\ud0c0\ub97c \uce58\uace0 \ub4dd\uc810\ud588\uc73c\ub098, 7\ud68c\ub9d0\uc5d0\ub294 \uc678\uc57c \ub72c\uacf5\uc744 \ud39c\uc2a4 \uc55e\uc5d0\uc11c \ub193\uce58\ub294 \uc2e4\uc218\ub97c \ubc94\ud588\ub2e4. 19\uc77c \ucfe0\ubc14\uc804\uc5d0\ub294 7\ud68c\uc5d0 \uad50\uccb4 \ud22c\uc785\ub410\uace0 \uc218\ube44\ub97c \ub9c8\uce58\uace0 \ub4e4\uc5b4\uc120 \ud0c0\uc11d\uc5d0\uc11c 1\ud0c0\uc810 \uc801\uc2dc\ud0c0\ub97c \uccd0 \ub0b4\uba70 \uc410\uae30 \ud0c0\uc810\uc744 \ub9cc\ub4e4\uc5b4\ub0c8\ub2e4. 20\uc77c \ub124\ub35c\ub780\ub4dc\uc804\uc5d0\ub294 5\ud0c0\uc218 3\uc548\ud0c0\ub85c \ud65c\uc57d\ud588\uace0, \uc608\uc120 \uc131\uc801\uc740 27\ud0c0\uc218 10\uc548\ud0c0, \ud0c0\uc728 0.370, 3\ud0c0\uc810, 4\ub4dd\uc810, 2\ubcfc\ub137, 2\ub3c4\ub8e8\ub97c \uae30\ub85d\ud588\ub2e4. \uc77c\ubcf8\uacfc\uc758 \uc900\uacb0\uc2b9\uc804\uc5d0\uc11c \ud070 \ud65c\uc57d\uc740 \ud558\uc9c0 \ubabb\ud588\ub2e4. \ucfe0\ubc14\uc640\uc758 \uacb0\uc2b9\uc804\uc5d0\uc11c\ub294 7\ud68c\ucd08 \ubcfc\ub137\uc73c\ub85c \ucd9c\ub8e8\ud588\uace0, \ub2e4\uc74c \ud0c0\uc790 \uc774\uc6a9\uaddc\uac00 2\ub8e8\ud0c0\ub97c \uccd0\ub0c8\uc73c\ub098 \uc544\uc6c3 \uce74\uc6b4\ud2b8\ub97c \ucc29\uac01\ud55c \uadf8\uac00 \ud648\uc73c\ub85c \ub4e4\uc5b4\uc624\uc9c0 \uc54a\ub294 \uc2e4\uc218\ub97c \ubc94\ud588\ub2e4. \ud558\uc9c0\ub9cc \ub300\ud45c\ud300\uc740 3-2\ub85c \uc2b9\ub9ac\ud558\uba74\uc11c 9\uc804 \uc804\uc2b9 \uae08\uba54\ub2ec\uc744 \uc774\ub8e8\uc5b4\ub0c8\ub2e4.", "paragraph_answer": "2007\ub144 3\uc6d4\uc5d0 \ub450\uc0b0 \ubca0\uc5b4\uc2a4\uc758 \uac10\ub3c5\uc744 \ub9e1\uace0 \uc788\ub358 \uae40\uacbd\ubb38 \uac10\ub3c5\uc774 \uc774\ubbf8 \ub300\ud45c\ud300 \uac10\ub3c5\uc73c\ub85c \uc120\uc784\ub418\uc5c8\uace0, 11\uc6d4 30\uc77c\uc5d0 \uc544\uc2dc\uc544 \uc57c\uad6c \uc120\uc218\uad8c \ub300\ud68c \uacb8 \uc62c\ub9bc\ud53d \uc608\uc120\uc758 \ucd5c\uc885 \uc5d4\ud2b8\ub9ac\uac00 \ubc1c\ud45c\ub418\uc5c8\ub2e4. \uadf8\ub294 \uc678\uc57c\uc218 \uc911\uc5d0\uc11c\ub294 \uac19\uc740 \ud300 \ubbfc\ubcd1\ud5cc\uacfc \ud568\uaed8 \ud3ec\ud568\ub418\uc5c8\ub2e4. \ub300\ub9cc\uacfc\uc758 \uccab \uacbd\uae30\uc5d0\uc11c 0-1\ub85c \ub4a4\uc9c0\ub294 \uc0c1\ud669\uc778 5\ud68c\ucd08 2\uc0ac 1, 3\ub8e8\uc5d0\uc11c \ub450 \ubc88\uc9f8 \ud0c0\uc11d\uc5d0 \ub4e4\uc5b4\uc11c \uc120\ubc1c \ud22c\uc218 \ub9b0\uc5b8\uc704\ub85c\ubd80\ud130 \uacb0\uc2b9 \uc5ed\uc804 3\uc810 \ud648\ub7f0\uc744 \ubf51\uc544\ub0c8\ub2e4. \uadf8\ub294 \uc77c\ubcf8\uacfc\uc758 \ub450 \ubc88\uc9f8 \uacbd\uae30\uc5d0\uc11c\ub294 1\ubc88 \ud0c0\uc790\ub85c \uc608\uc0c1\ub418\uc5c8\uc73c\ub098, 10\ubd84\uc804\uc5d0 \uc624\ub354\uac00 \ubc14\ub00c\uba74\uc11c 8\ubc88 \ud0c0\uc790\ub85c \ubc14\ub00c\uc5c8\uace0, 2-4\ub85c \ub4a4\uc9c4 8\ud68c \ub9d0\uc5d0 \ud76c\uc0dd \ud50c\ub77c\uc774\ub97c \uce58\uba74\uc11c 1\ud0c0\uc810\uc744 \uc62c\ub838\uc73c\ub098 \ud300\uc740 4-3\uc73c\ub85c \ud328\ud588\ub2e4. \ud544\ub9ac\ud540\uacfc\uc758 \uc138 \ubc88\uc9f8 \uacbd\uae30\uc5d0\uc11c \uc801\uc2dc\ud0c0\ub85c 1\ud0c0\uc810\uc744 \uc62c\ub838\uace0, \ud300\uc740 13-1\uc758 \uc810\uc218\ub85c \ucf5c\ub4dc \uac8c\uc784\uc73c\ub85c \uc2b9\ub9ac\ud588\ub2e4. \uadf8 \uacb0\uacfc \ub300\ud45c\ud300\uc740 2\uc2b9 1\ud328\ub85c 2\uc704\uc5d0 \uba38\ubb3c\uba70 3\uc2b9\uc744 \uae30\ub85d\ud55c \uc77c\ubcf8\uc5d0\uac8c \ubc00\ub824 \uc62c\ub9bc\ud53d \uc9c1\ud589\uc5d0 \uc2e4\ud328\ud588\uace0, 2\ucc28 \uc608\uc120\uc5d0 \ucc38\uac00\ud558\uac8c \ub418\uc5c8\ub2e4. \uc62c\ub9bc\ud53d \ucd5c\uc885 \uc608\uc120 \uc5d4\ud2b8\ub9ac\ub294 2008\ub144 3\uc6d4 5\uc77c \ubc1c\ud45c\ub418\uc5c8\uace0, \uac19\uc740 \ud300 \uc911\uc5d0\ub294 \ud22c\uc218 \uae40\uc120\uc6b0, \ub0b4\uc57c\uc218 \uae40\ub3d9\uc8fc, \uace0\uc601\ubbfc, \uadf8\ub9ac\uace0 \uc0c1\ubb34\uc5d0 \ubcf5\ubb34 \uc911\uc778 \uc190\uc2dc\ud5cc\uacfc \ud568\uaed8 \ud3ec\ud568\ub418\uc5c8\ub2e4. \uccab \uacbd\uae30 \ub0a8\uc544\uacf5\uc804\uc5d0\uc11c \uc120\ub450\ud0c0\uc790\ub85c \ub098\uc640 \uccab \ud0c0\uc11d \ubab8\uc5d0 \ub9de\ub294 \ubcfc\ub85c \ucd9c\ub8e8\ud558\uace0 \ub3c4\ub8e8\uc5d0 \uc131\uacf5\ud55c \ub4a4 \uc774\uc2b9\uc5fd\uc758 2\ub8e8\ud0c0 \ub54c \ub4dd\uc810\uc744 \ud588\ub2e4. \uadf8\ub7ec\ub098 \uc774\ud6c4 4\ud0c0\uc218 \ubb34\uc548\ud0c0\uc5d0 \uadf8\uce58\uba74\uc11c \ud638\uc8fc\uc804\uc5d0\uc11c\ub294 \uc774\uc6a9\uaddc\uc5d0\uac8c \uc120\ubc1c \ucd9c\uc804\uc744 \ub0b4\uc8fc\uc5c8\ub2e4. \ud638\uc8fc\uc804\uc5d0\uc11c \uc9c4\uac11\uc6a9 \ub300\uc2e0 \ub300\uc8fc\uc790\ub85c \ucd9c\uc7a5\ub418\uc5c8\ub2e4. \uadf8\ub294 9\uc77c \uba55\uc2dc\ucf54\uc804\uc5d0\uc11c\ub294 3-1\ub85c \uc774\uae30\uace0 \uc788\ub294 \uc0c1\ud669\uc5d0\uc11c \uc774\ubc88 \ub300\ud68c \ub300\ud45c\ud300 \uccab 3\ub8e8\ud0c0\ub97c \ud130\ub728\ub9ac\uba70 2\ud0c0\uc810\uc744 \uc62c\ub838\uc73c\uba70, 6\ud0c0\uc218 1\uc548\ud0c0\ub97c \uae30\ub85d\ud588\ub2e4. 10\uc77c \uc2a4\ud398\uc778\uc804\uc5d0\uc11c 2\ubc88 \ud0c0\uc790\ub85c \uc120\ubc1c \ucd9c\uc7a5\ud588\uc73c\uba70, 1\uc548\ud0c0 3\ubcfc\ub137\uc73c\ub85c \ub192\uc740 \ucd9c\ub8e8\uc728\uc744 \ubcf4\uc600\ub2e4.\uc774\ud2c0 \ub4a4 \ub3c5\uc77c\uc804\uc5d0\uc11c 2\ubc88 \ud0c0\uc790\ub85c \ucd9c\uc804, \ubcfc\ub137\uc744 \uc5bb\uc5b4\ub0c8\ub2e4. 13\uc77c \uce90\ub098\ub2e4\uc804\uc5d0\uc11c\ub294 2\ubc88 \ud0c0\uc790\ub85c \uc120\ubc1c \ucd9c\uc7a5\ud574 \uccab \ud0c0\uc11d\uc5d0\uc11c 3\ub8e8\ub97c \uaff0\ub6ab\ub294 \uc801\uc2dc 2\ub8e8\ud0c0\ub97c \ud130\ub728\ub824 1\ud0c0\uc810\uc744 \uc62c\ub838\uc73c\ub098 \ud300\uc740 4-3\uc73c\ub85c \ud328\ud588\ub2e4. \ub300\ud55c\ubbfc\uad6d \ub300\ud45c\ud300\uc740 \ub9c8\uc9c0\ub9c9 \uacbd\uae30 \ub300\ub9cc\uc804\uc5d0\uc11c 4-3\uc73c\ub85c \uc2b9\ub9ac\ud558\uba74\uc11c, 2\uc704(6\uc2b9 1\ud328)\ub85c \uc62c\ub9bc\ud53d \ubcf8\uc120\uc5d0 \uc9c4\ucd9c\ud588\ub2e4. \uc62c\ub9bc\ud53d \uc5d4\ud2b8\ub9ac\ub294 2008\ub144 7\uc6d4 14\uc77c \ubc1c\ud45c\ub418\uc5c8\uace0, \uc678\uc57c\uc218 \uc911\uc5d0\uc11c \uac19\uc740 \ud300 \uae40\ud604\uc218\uc640 \ud568\uaed8 \ud3ec\ud568\ub418\uc5c8\ub2e4. \uc62c\ub9bc\ud53d \uc57c\uad6c \ubcf8\uc120\uc740 8\uc6d4 13\uc77c\ubd80\ud130 \uc2dc\uc791\ub418\uc5c8\ub2e4. \uccab \ub0a0 \ubbf8\uad6d\uc804\uc5d0\uc11c 4\ud0c0\uc218 2\uc548\ud0c0 1\ud0c0\uc810 1\ub4dd\uc810\uc744 \uae30\ub85d\ud588\uace0, \ud2b9\ud788 7-7\uc758 \uc0c1\ud669\uc774\ub358 9\ud68c\ub9d0 1\uc0ac 3\ub8e8 \ub05d\ub0b4\uae30 \ud76c\uc0dd \ud50c\ub77c\uc774\ub97c \uccd0\ub0c8\ub2e4. 15\uc77c \uce90\ub098\ub2e4\uc804\uc5d0\uc11c\ub294 4\ud68c 1\uc0ac\uc5d0\uc11c \ube57\ub9de\uc740 \uc911\uc804\uc548\ud0c0\uc131 \ud0c0\uad6c\ub97c \uc804\ub825\uc9c8\uc8fc\ud574 \ub2e4\uc774\ube59 \uce90\uce58\ub97c \ud558\uba70 1-0\uc758 \ub958\ud604\uc9c4\uc758 \uc644\ubd09\uc2b9\uc5d0 \uc77c\uc870\ud588\ub2e4. \ub2e4\uc74c \ub0a0 \uc77c\ubcf8\uc804\uc5d0\uc11c\ub294 9\ud68c\ucd08 2\uc0ac \uc774\ud6c4 3\ub8e8\uc218 \uc55e\uc5d0 \ub5a8\uc5b4\uc9c0\ub294 \uc808\ubb18\ud55c \uae30\uc2b5\ubc88\ud2b8\ub85c 4-2, \uc774\ud6c4 \ub204 \uc0c1\uc5d0 \ub098\uac04 \uc774\ud6c4 \ub3c4\ub8e8\ub97c \uc2dc\ub3c4\ud588\uace0 \ud3ec\uc218 \uc544\ubca0\uc758 \uc545\uc1a1\uad6c\ub85c 3\ub8e8\uc5d0 \uc548\ucc29\ud588\uace0 3\ub8e8\uc8fc\uc790\uac00 \ud648\uc73c\ub85c \ub4e4\uc5b4\uc624\uba70 \ub610 \ub2e4\uc2dc \ud55c \uc810\uc744 \ucd94\uac00\ud574 5-2\ub97c \ub9cc\ub4e4\uc5b4\ub0c8\ub2e4. 17\uc77c\uc5d0\ub294 3\uc77c\uac04 \ubbf8\ub904\uc84c\ub358 \uc911\uad6d\uc804\uc5d0\uc11c\uc758 11\ud68c \uc2b9\ubd80\uce58\uae30\uc5d0\uc11c 2\ub8e8 \uc8fc\uc790\ub85c \ub098\uac00 \uc774\uc2b9\uc5fd\uc758 \ub05d\ub0b4\uae30 \uc548\ud0c0 \ub54c \ud648\uc5d0 \ub4e4\uc5b4\uc640 \ub4dd\uc810\ud574 1-0\uc73c\ub85c \uc2b9\ub9ac\ud588\ub2e4. 18\uc77c \ub300\ub9cc\uc804\uc5d0\uc11c\ub294 1\ud68c \ub0b4\uc57c \uc548\ud0c0\ub97c \uce58\uace0 \ub4dd\uc810\ud588\uc73c\ub098, 7\ud68c\ub9d0\uc5d0\ub294 \uc678\uc57c \ub72c\uacf5\uc744 \ud39c\uc2a4 \uc55e\uc5d0\uc11c \ub193\uce58\ub294 \uc2e4\uc218\ub97c \ubc94\ud588\ub2e4. 19\uc77c \ucfe0\ubc14\uc804\uc5d0\ub294 7\ud68c\uc5d0 \uad50\uccb4 \ud22c\uc785\ub410\uace0 \uc218\ube44\ub97c \ub9c8\uce58\uace0 \ub4e4\uc5b4\uc120 \ud0c0\uc11d\uc5d0\uc11c 1\ud0c0\uc810 \uc801\uc2dc\ud0c0\ub97c \uccd0 \ub0b4\uba70 \uc410\uae30 \ud0c0\uc810\uc744 \ub9cc\ub4e4\uc5b4\ub0c8\ub2e4. 20\uc77c \ub124\ub35c\ub780\ub4dc\uc804\uc5d0\ub294 5\ud0c0\uc218 3\uc548\ud0c0\ub85c \ud65c\uc57d\ud588\uace0, \uc608\uc120 \uc131\uc801\uc740 27\ud0c0\uc218 10\uc548\ud0c0, \ud0c0\uc728 0.370, 3\ud0c0\uc810, 4\ub4dd\uc810, 2\ubcfc\ub137, 2\ub3c4\ub8e8\ub97c \uae30\ub85d\ud588\ub2e4. \uc77c\ubcf8\uacfc\uc758 \uc900\uacb0\uc2b9\uc804\uc5d0\uc11c \ud070 \ud65c\uc57d\uc740 \ud558\uc9c0 \ubabb\ud588\ub2e4. \ucfe0\ubc14\uc640\uc758 \uacb0\uc2b9\uc804\uc5d0\uc11c\ub294 7\ud68c\ucd08 \ubcfc\ub137\uc73c\ub85c \ucd9c\ub8e8\ud588\uace0, \ub2e4\uc74c \ud0c0\uc790 \uc774\uc6a9\uaddc\uac00 2\ub8e8\ud0c0\ub97c \uccd0\ub0c8\uc73c\ub098 \uc544\uc6c3 \uce74\uc6b4\ud2b8\ub97c \ucc29\uac01\ud55c \uadf8\uac00 \ud648\uc73c\ub85c \ub4e4\uc5b4\uc624\uc9c0 \uc54a\ub294 \uc2e4\uc218\ub97c \ubc94\ud588\ub2e4. \ud558\uc9c0\ub9cc \ub300\ud45c\ud300\uc740 3-2\ub85c \uc2b9\ub9ac\ud558\uba74\uc11c 9\uc804 \uc804\uc2b9 \uae08\uba54\ub2ec\uc744 \uc774\ub8e8\uc5b4\ub0c8\ub2e4.", "sentence_answer": "\ud638\uc8fc\uc804\uc5d0\uc11c \uc9c4\uac11\uc6a9 \ub300\uc2e0 \ub300\uc8fc\uc790\ub85c \ucd9c\uc7a5\ub418\uc5c8\ub2e4.", "paragraph_id": "6547838-12-0", "paragraph_question": "question: \uc774\uc885\uc6b1\uc774 \ud638\uc8fc\uc804\uc5d0\uc11c \ub300\uc2e0 \uc8fc\uc790\ub85c \ub098\uac04 \uc120\uc218\ub294?, context: 2007\ub144 3\uc6d4\uc5d0 \ub450\uc0b0 \ubca0\uc5b4\uc2a4\uc758 \uac10\ub3c5\uc744 \ub9e1\uace0 \uc788\ub358 \uae40\uacbd\ubb38 \uac10\ub3c5\uc774 \uc774\ubbf8 \ub300\ud45c\ud300 \uac10\ub3c5\uc73c\ub85c \uc120\uc784\ub418\uc5c8\uace0, 11\uc6d4 30\uc77c\uc5d0 \uc544\uc2dc\uc544 \uc57c\uad6c \uc120\uc218\uad8c \ub300\ud68c \uacb8 \uc62c\ub9bc\ud53d \uc608\uc120\uc758 \ucd5c\uc885 \uc5d4\ud2b8\ub9ac\uac00 \ubc1c\ud45c\ub418\uc5c8\ub2e4. \uadf8\ub294 \uc678\uc57c\uc218 \uc911\uc5d0\uc11c\ub294 \uac19\uc740 \ud300 \ubbfc\ubcd1\ud5cc\uacfc \ud568\uaed8 \ud3ec\ud568\ub418\uc5c8\ub2e4. \ub300\ub9cc\uacfc\uc758 \uccab \uacbd\uae30\uc5d0\uc11c 0-1\ub85c \ub4a4\uc9c0\ub294 \uc0c1\ud669\uc778 5\ud68c\ucd08 2\uc0ac 1, 3\ub8e8\uc5d0\uc11c \ub450 \ubc88\uc9f8 \ud0c0\uc11d\uc5d0 \ub4e4\uc5b4\uc11c \uc120\ubc1c \ud22c\uc218 \ub9b0\uc5b8\uc704\ub85c\ubd80\ud130 \uacb0\uc2b9 \uc5ed\uc804 3\uc810 \ud648\ub7f0\uc744 \ubf51\uc544\ub0c8\ub2e4. \uadf8\ub294 \uc77c\ubcf8\uacfc\uc758 \ub450 \ubc88\uc9f8 \uacbd\uae30\uc5d0\uc11c\ub294 1\ubc88 \ud0c0\uc790\ub85c \uc608\uc0c1\ub418\uc5c8\uc73c\ub098, 10\ubd84\uc804\uc5d0 \uc624\ub354\uac00 \ubc14\ub00c\uba74\uc11c 8\ubc88 \ud0c0\uc790\ub85c \ubc14\ub00c\uc5c8\uace0, 2-4\ub85c \ub4a4\uc9c4 8\ud68c \ub9d0\uc5d0 \ud76c\uc0dd \ud50c\ub77c\uc774\ub97c \uce58\uba74\uc11c 1\ud0c0\uc810\uc744 \uc62c\ub838\uc73c\ub098 \ud300\uc740 4-3\uc73c\ub85c \ud328\ud588\ub2e4. \ud544\ub9ac\ud540\uacfc\uc758 \uc138 \ubc88\uc9f8 \uacbd\uae30\uc5d0\uc11c \uc801\uc2dc\ud0c0\ub85c 1\ud0c0\uc810\uc744 \uc62c\ub838\uace0, \ud300\uc740 13-1\uc758 \uc810\uc218\ub85c \ucf5c\ub4dc \uac8c\uc784\uc73c\ub85c \uc2b9\ub9ac\ud588\ub2e4. \uadf8 \uacb0\uacfc \ub300\ud45c\ud300\uc740 2\uc2b9 1\ud328\ub85c 2\uc704\uc5d0 \uba38\ubb3c\uba70 3\uc2b9\uc744 \uae30\ub85d\ud55c \uc77c\ubcf8\uc5d0\uac8c \ubc00\ub824 \uc62c\ub9bc\ud53d \uc9c1\ud589\uc5d0 \uc2e4\ud328\ud588\uace0, 2\ucc28 \uc608\uc120\uc5d0 \ucc38\uac00\ud558\uac8c \ub418\uc5c8\ub2e4. \uc62c\ub9bc\ud53d \ucd5c\uc885 \uc608\uc120 \uc5d4\ud2b8\ub9ac\ub294 2008\ub144 3\uc6d4 5\uc77c \ubc1c\ud45c\ub418\uc5c8\uace0, \uac19\uc740 \ud300 \uc911\uc5d0\ub294 \ud22c\uc218 \uae40\uc120\uc6b0, \ub0b4\uc57c\uc218 \uae40\ub3d9\uc8fc, \uace0\uc601\ubbfc, \uadf8\ub9ac\uace0 \uc0c1\ubb34\uc5d0 \ubcf5\ubb34 \uc911\uc778 \uc190\uc2dc\ud5cc\uacfc \ud568\uaed8 \ud3ec\ud568\ub418\uc5c8\ub2e4. \uccab \uacbd\uae30 \ub0a8\uc544\uacf5\uc804\uc5d0\uc11c \uc120\ub450\ud0c0\uc790\ub85c \ub098\uc640 \uccab \ud0c0\uc11d \ubab8\uc5d0 \ub9de\ub294 \ubcfc\ub85c \ucd9c\ub8e8\ud558\uace0 \ub3c4\ub8e8\uc5d0 \uc131\uacf5\ud55c \ub4a4 \uc774\uc2b9\uc5fd\uc758 2\ub8e8\ud0c0 \ub54c \ub4dd\uc810\uc744 \ud588\ub2e4. \uadf8\ub7ec\ub098 \uc774\ud6c4 4\ud0c0\uc218 \ubb34\uc548\ud0c0\uc5d0 \uadf8\uce58\uba74\uc11c \ud638\uc8fc\uc804\uc5d0\uc11c\ub294 \uc774\uc6a9\uaddc\uc5d0\uac8c \uc120\ubc1c \ucd9c\uc804\uc744 \ub0b4\uc8fc\uc5c8\ub2e4. \ud638\uc8fc\uc804\uc5d0\uc11c \uc9c4\uac11\uc6a9 \ub300\uc2e0 \ub300\uc8fc\uc790\ub85c \ucd9c\uc7a5\ub418\uc5c8\ub2e4. \uadf8\ub294 9\uc77c \uba55\uc2dc\ucf54\uc804\uc5d0\uc11c\ub294 3-1\ub85c \uc774\uae30\uace0 \uc788\ub294 \uc0c1\ud669\uc5d0\uc11c \uc774\ubc88 \ub300\ud68c \ub300\ud45c\ud300 \uccab 3\ub8e8\ud0c0\ub97c \ud130\ub728\ub9ac\uba70 2\ud0c0\uc810\uc744 \uc62c\ub838\uc73c\uba70, 6\ud0c0\uc218 1\uc548\ud0c0\ub97c \uae30\ub85d\ud588\ub2e4. 10\uc77c \uc2a4\ud398\uc778\uc804\uc5d0\uc11c 2\ubc88 \ud0c0\uc790\ub85c \uc120\ubc1c \ucd9c\uc7a5\ud588\uc73c\uba70, 1\uc548\ud0c0 3\ubcfc\ub137\uc73c\ub85c \ub192\uc740 \ucd9c\ub8e8\uc728\uc744 \ubcf4\uc600\ub2e4.\uc774\ud2c0 \ub4a4 \ub3c5\uc77c\uc804\uc5d0\uc11c 2\ubc88 \ud0c0\uc790\ub85c \ucd9c\uc804, \ubcfc\ub137\uc744 \uc5bb\uc5b4\ub0c8\ub2e4. 13\uc77c \uce90\ub098\ub2e4\uc804\uc5d0\uc11c\ub294 2\ubc88 \ud0c0\uc790\ub85c \uc120\ubc1c \ucd9c\uc7a5\ud574 \uccab \ud0c0\uc11d\uc5d0\uc11c 3\ub8e8\ub97c \uaff0\ub6ab\ub294 \uc801\uc2dc 2\ub8e8\ud0c0\ub97c \ud130\ub728\ub824 1\ud0c0\uc810\uc744 \uc62c\ub838\uc73c\ub098 \ud300\uc740 4-3\uc73c\ub85c \ud328\ud588\ub2e4. \ub300\ud55c\ubbfc\uad6d \ub300\ud45c\ud300\uc740 \ub9c8\uc9c0\ub9c9 \uacbd\uae30 \ub300\ub9cc\uc804\uc5d0\uc11c 4-3\uc73c\ub85c \uc2b9\ub9ac\ud558\uba74\uc11c, 2\uc704(6\uc2b9 1\ud328)\ub85c \uc62c\ub9bc\ud53d \ubcf8\uc120\uc5d0 \uc9c4\ucd9c\ud588\ub2e4. \uc62c\ub9bc\ud53d \uc5d4\ud2b8\ub9ac\ub294 2008\ub144 7\uc6d4 14\uc77c \ubc1c\ud45c\ub418\uc5c8\uace0, \uc678\uc57c\uc218 \uc911\uc5d0\uc11c \uac19\uc740 \ud300 \uae40\ud604\uc218\uc640 \ud568\uaed8 \ud3ec\ud568\ub418\uc5c8\ub2e4. \uc62c\ub9bc\ud53d \uc57c\uad6c \ubcf8\uc120\uc740 8\uc6d4 13\uc77c\ubd80\ud130 \uc2dc\uc791\ub418\uc5c8\ub2e4. \uccab \ub0a0 \ubbf8\uad6d\uc804\uc5d0\uc11c 4\ud0c0\uc218 2\uc548\ud0c0 1\ud0c0\uc810 1\ub4dd\uc810\uc744 \uae30\ub85d\ud588\uace0, \ud2b9\ud788 7-7\uc758 \uc0c1\ud669\uc774\ub358 9\ud68c\ub9d0 1\uc0ac 3\ub8e8 \ub05d\ub0b4\uae30 \ud76c\uc0dd \ud50c\ub77c\uc774\ub97c \uccd0\ub0c8\ub2e4. 15\uc77c \uce90\ub098\ub2e4\uc804\uc5d0\uc11c\ub294 4\ud68c 1\uc0ac\uc5d0\uc11c \ube57\ub9de\uc740 \uc911\uc804\uc548\ud0c0\uc131 \ud0c0\uad6c\ub97c \uc804\ub825\uc9c8\uc8fc\ud574 \ub2e4\uc774\ube59 \uce90\uce58\ub97c \ud558\uba70 1-0\uc758 \ub958\ud604\uc9c4\uc758 \uc644\ubd09\uc2b9\uc5d0 \uc77c\uc870\ud588\ub2e4. \ub2e4\uc74c \ub0a0 \uc77c\ubcf8\uc804\uc5d0\uc11c\ub294 9\ud68c\ucd08 2\uc0ac \uc774\ud6c4 3\ub8e8\uc218 \uc55e\uc5d0 \ub5a8\uc5b4\uc9c0\ub294 \uc808\ubb18\ud55c \uae30\uc2b5\ubc88\ud2b8\ub85c 4-2, \uc774\ud6c4 \ub204 \uc0c1\uc5d0 \ub098\uac04 \uc774\ud6c4 \ub3c4\ub8e8\ub97c \uc2dc\ub3c4\ud588\uace0 \ud3ec\uc218 \uc544\ubca0\uc758 \uc545\uc1a1\uad6c\ub85c 3\ub8e8\uc5d0 \uc548\ucc29\ud588\uace0 3\ub8e8\uc8fc\uc790\uac00 \ud648\uc73c\ub85c \ub4e4\uc5b4\uc624\uba70 \ub610 \ub2e4\uc2dc \ud55c \uc810\uc744 \ucd94\uac00\ud574 5-2\ub97c \ub9cc\ub4e4\uc5b4\ub0c8\ub2e4. 17\uc77c\uc5d0\ub294 3\uc77c\uac04 \ubbf8\ub904\uc84c\ub358 \uc911\uad6d\uc804\uc5d0\uc11c\uc758 11\ud68c \uc2b9\ubd80\uce58\uae30\uc5d0\uc11c 2\ub8e8 \uc8fc\uc790\ub85c \ub098\uac00 \uc774\uc2b9\uc5fd\uc758 \ub05d\ub0b4\uae30 \uc548\ud0c0 \ub54c \ud648\uc5d0 \ub4e4\uc5b4\uc640 \ub4dd\uc810\ud574 1-0\uc73c\ub85c \uc2b9\ub9ac\ud588\ub2e4. 18\uc77c \ub300\ub9cc\uc804\uc5d0\uc11c\ub294 1\ud68c \ub0b4\uc57c \uc548\ud0c0\ub97c \uce58\uace0 \ub4dd\uc810\ud588\uc73c\ub098, 7\ud68c\ub9d0\uc5d0\ub294 \uc678\uc57c \ub72c\uacf5\uc744 \ud39c\uc2a4 \uc55e\uc5d0\uc11c \ub193\uce58\ub294 \uc2e4\uc218\ub97c \ubc94\ud588\ub2e4. 19\uc77c \ucfe0\ubc14\uc804\uc5d0\ub294 7\ud68c\uc5d0 \uad50\uccb4 \ud22c\uc785\ub410\uace0 \uc218\ube44\ub97c \ub9c8\uce58\uace0 \ub4e4\uc5b4\uc120 \ud0c0\uc11d\uc5d0\uc11c 1\ud0c0\uc810 \uc801\uc2dc\ud0c0\ub97c \uccd0 \ub0b4\uba70 \uc410\uae30 \ud0c0\uc810\uc744 \ub9cc\ub4e4\uc5b4\ub0c8\ub2e4. 20\uc77c \ub124\ub35c\ub780\ub4dc\uc804\uc5d0\ub294 5\ud0c0\uc218 3\uc548\ud0c0\ub85c \ud65c\uc57d\ud588\uace0, \uc608\uc120 \uc131\uc801\uc740 27\ud0c0\uc218 10\uc548\ud0c0, \ud0c0\uc728 0.370, 3\ud0c0\uc810, 4\ub4dd\uc810, 2\ubcfc\ub137, 2\ub3c4\ub8e8\ub97c \uae30\ub85d\ud588\ub2e4. \uc77c\ubcf8\uacfc\uc758 \uc900\uacb0\uc2b9\uc804\uc5d0\uc11c \ud070 \ud65c\uc57d\uc740 \ud558\uc9c0 \ubabb\ud588\ub2e4. \ucfe0\ubc14\uc640\uc758 \uacb0\uc2b9\uc804\uc5d0\uc11c\ub294 7\ud68c\ucd08 \ubcfc\ub137\uc73c\ub85c \ucd9c\ub8e8\ud588\uace0, \ub2e4\uc74c \ud0c0\uc790 \uc774\uc6a9\uaddc\uac00 2\ub8e8\ud0c0\ub97c \uccd0\ub0c8\uc73c\ub098 \uc544\uc6c3 \uce74\uc6b4\ud2b8\ub97c \ucc29\uac01\ud55c \uadf8\uac00 \ud648\uc73c\ub85c \ub4e4\uc5b4\uc624\uc9c0 \uc54a\ub294 \uc2e4\uc218\ub97c \ubc94\ud588\ub2e4. \ud558\uc9c0\ub9cc \ub300\ud45c\ud300\uc740 3-2\ub85c \uc2b9\ub9ac\ud558\uba74\uc11c 9\uc804 \uc804\uc2b9 \uae08\uba54\ub2ec\uc744 \uc774\ub8e8\uc5b4\ub0c8\ub2e4."} {"question": "8\uccb4\uc9c8\ub860\uc5d0\uc11c \uccb4\uc9c8 \uac10\ubcc4\uc774 \uc27d\uc9c0\uc54a\uc544 \uc5b4\ub5a4 \ubc29\ubc95\uc744 \uc5f0\uad6c\uc9c4\uc5d0 \uc758\ub8b0\ud588\ub098?", "paragraph": "'\uc77c\ucc0d\uc774 1965\ub144 \uad6d\uc81c\ud559\ud68c\uc5d0\uc11c \uc774\ub7ec\ud55c 8\uccb4\uc9c8\ub860\uc744 \ubc1c\ud45c\ud55c\uc9c0 50\ub144\uc774 \uac00\uae4c\uc6cc\uc624\ub294 \uc624\ub298\uc5d0 \uc774\ub974\uae30\uae4c\uc9c0, \ub0b4 \uc790\uc2e0\uc774 8\uccb4\uc9c8\ub860\uc5d0 \ub300\ud55c \uacf5\uac1c \uac15\uc5f0\uc774\ub098 \ubc1c\ud45c\ub97c \ubbf8\ub8e8\uc5b4 \uc628 \uac83\uc740 \uc544\uc9c1 \ub204\uad6c\ub098\uac00 \ud560 \uc218 \uc788\ub294 \uc27d\uace0 \uc815\ud655\ud55c \uccb4\uc9c8 \uac10\ubcc4\ubc95\uc744 \ubc1c\uacac\ud560 \uc218 \uc5c6\uc5c8\uae30 \ub54c\ubb38\uc774\ub2e4. \ud604\uc7ac \ub0b4\uac00 \uc0ac\uc6a9\ud558\ub294 \uac10\ubcc4\ubc95\uc740 \ub0b4 \uc790\uc2e0\uc758 \uc5f0\uad6c\ub85c \ubc1c\uacac\ud55c \uc778\uac04 \uc694\uace8\ub3d9\ub9e5(\u6493\u9aa8\u52d5\u8108)\uc5d0\uc11c \ud53c\uac80\uc790\uc758 \uccb4\uc9c8\uc758 \uc2f8\uc778(sign)\uc744 \ucc3e\ub294 \uac83\uc778\ub370, \uc774 \ub610\ud55c \uc27d\uc9c0 \uc54a\uc544 \uc624\uc9c4\ud558\uae30 \uc26c\uc6b0\ubbc0\ub85c \ub204\uad6c\ub098\uac00 \uc27d\uac8c \uc0ac\uc6a9\ud560 \uc218 \uc788\ub294 \uacfc\ud559\uc801\uc778 \ubc29\ubc95\uc744 \uad6d\ub0b4\uc678 \uc5f0\uad6c\uc9c4\uc5d0 \uc758\ub8b0\ud558\uc5ec \ubcf4\uc558\uc73c\ub098 \uc774\ub807\ub2e4 \ud560 \uc131\uacfc\uac00 \uc5c6\uc73c\ubbc0\ub85c \ucd5c\ud6c4\uc758 \ubc29\ubc95\uc73c\ub85c \ub098\uc758 \uac10\ubcc4\ubc95\uc5d0 \uc758\ud558\uc5ec \ud310\ub2e8\ub41c 8\uccb4\uc9c8 \uc911 \uac19\uc740 \uccb4\uc9c8\ub4e4\uc5d0\uc11c \ub3d9\uc77c\uc810\uc744, \uadf8\ub9ac\uace0 \uc11c\ub85c \ub2e4\ub978 \uccb4\uc9c8\ub4e4\uc5d0\uc11c \ucc28\uc774\uc810\uc744 \uacfc\ud559\uc801\uc73c\ub85c \ucc3e\uc544\ub0b4\ub294 \ubc29\ubc95\uc744 \uc54c\uc544\ubcf4\uace0 \uc788\ub294 \uc911\uc774\ub2e4.'", "answer": "\uacfc\ud559\uc801\uc778 \ubc29\ubc95", "sentence": "\ud604\uc7ac \ub0b4\uac00 \uc0ac\uc6a9\ud558\ub294 \uac10\ubcc4\ubc95\uc740 \ub0b4 \uc790\uc2e0\uc758 \uc5f0\uad6c\ub85c \ubc1c\uacac\ud55c \uc778\uac04 \uc694\uace8\ub3d9\ub9e5(\u6493\u9aa8\u52d5\u8108)\uc5d0\uc11c \ud53c\uac80\uc790\uc758 \uccb4\uc9c8\uc758 \uc2f8\uc778(sign)\uc744 \ucc3e\ub294 \uac83\uc778\ub370, \uc774 \ub610\ud55c \uc27d\uc9c0 \uc54a\uc544 \uc624\uc9c4\ud558\uae30 \uc26c\uc6b0\ubbc0\ub85c \ub204\uad6c\ub098\uac00 \uc27d\uac8c \uc0ac\uc6a9\ud560 \uc218 \uc788\ub294 \uacfc\ud559\uc801\uc778 \ubc29\ubc95 \uc744 \uad6d\ub0b4\uc678 \uc5f0\uad6c\uc9c4\uc5d0 \uc758\ub8b0\ud558\uc5ec \ubcf4\uc558\uc73c\ub098 \uc774\ub807\ub2e4 \ud560 \uc131\uacfc\uac00 \uc5c6\uc73c\ubbc0\ub85c \ucd5c\ud6c4\uc758 \ubc29\ubc95\uc73c\ub85c \ub098\uc758 \uac10\ubcc4\ubc95\uc5d0 \uc758\ud558\uc5ec \ud310\ub2e8\ub41c 8\uccb4\uc9c8 \uc911 \uac19\uc740 \uccb4\uc9c8\ub4e4\uc5d0\uc11c \ub3d9\uc77c\uc810\uc744, \uadf8\ub9ac\uace0 \uc11c\ub85c \ub2e4\ub978 \uccb4\uc9c8\ub4e4\uc5d0\uc11c \ucc28\uc774\uc810\uc744 \uacfc\ud559\uc801\uc73c\ub85c \ucc3e\uc544\ub0b4\ub294 \ubc29\ubc95\uc744 \uc54c\uc544\ubcf4\uace0 \uc788\ub294 \uc911\uc774\ub2e4.", "paragraph_sentence": "'\uc77c\ucc0d\uc774 1965\ub144 \uad6d\uc81c\ud559\ud68c\uc5d0\uc11c \uc774\ub7ec\ud55c 8\uccb4\uc9c8\ub860\uc744 \ubc1c\ud45c\ud55c\uc9c0 50\ub144\uc774 \uac00\uae4c\uc6cc\uc624\ub294 \uc624\ub298\uc5d0 \uc774\ub974\uae30\uae4c\uc9c0, \ub0b4 \uc790\uc2e0\uc774 8\uccb4\uc9c8\ub860\uc5d0 \ub300\ud55c \uacf5\uac1c \uac15\uc5f0\uc774\ub098 \ubc1c\ud45c\ub97c \ubbf8\ub8e8\uc5b4 \uc628 \uac83\uc740 \uc544\uc9c1 \ub204\uad6c\ub098\uac00 \ud560 \uc218 \uc788\ub294 \uc27d\uace0 \uc815\ud655\ud55c \uccb4\uc9c8 \uac10\ubcc4\ubc95\uc744 \ubc1c\uacac\ud560 \uc218 \uc5c6\uc5c8\uae30 \ub54c\ubb38\uc774\ub2e4. \ud604\uc7ac \ub0b4\uac00 \uc0ac\uc6a9\ud558\ub294 \uac10\ubcc4\ubc95\uc740 \ub0b4 \uc790\uc2e0\uc758 \uc5f0\uad6c\ub85c \ubc1c\uacac\ud55c \uc778\uac04 \uc694\uace8\ub3d9\ub9e5(\u6493\u9aa8\u52d5\u8108)\uc5d0\uc11c \ud53c\uac80\uc790\uc758 \uccb4\uc9c8\uc758 \uc2f8\uc778(sign)\uc744 \ucc3e\ub294 \uac83\uc778\ub370, \uc774 \ub610\ud55c \uc27d\uc9c0 \uc54a\uc544 \uc624\uc9c4\ud558\uae30 \uc26c\uc6b0\ubbc0\ub85c \ub204\uad6c\ub098\uac00 \uc27d\uac8c \uc0ac\uc6a9\ud560 \uc218 \uc788\ub294 \uacfc\ud559\uc801\uc778 \ubc29\ubc95 \uc744 \uad6d\ub0b4\uc678 \uc5f0\uad6c\uc9c4\uc5d0 \uc758\ub8b0\ud558\uc5ec \ubcf4\uc558\uc73c\ub098 \uc774\ub807\ub2e4 \ud560 \uc131\uacfc\uac00 \uc5c6\uc73c\ubbc0\ub85c \ucd5c\ud6c4\uc758 \ubc29\ubc95\uc73c\ub85c \ub098\uc758 \uac10\ubcc4\ubc95\uc5d0 \uc758\ud558\uc5ec \ud310\ub2e8\ub41c 8\uccb4\uc9c8 \uc911 \uac19\uc740 \uccb4\uc9c8\ub4e4\uc5d0\uc11c \ub3d9\uc77c\uc810\uc744, \uadf8\ub9ac\uace0 \uc11c\ub85c \ub2e4\ub978 \uccb4\uc9c8\ub4e4\uc5d0\uc11c \ucc28\uc774\uc810\uc744 \uacfc\ud559\uc801\uc73c\ub85c \ucc3e\uc544\ub0b4\ub294 \ubc29\ubc95\uc744 \uc54c\uc544\ubcf4\uace0 \uc788\ub294 \uc911\uc774\ub2e4. '", "paragraph_answer": "'\uc77c\ucc0d\uc774 1965\ub144 \uad6d\uc81c\ud559\ud68c\uc5d0\uc11c \uc774\ub7ec\ud55c 8\uccb4\uc9c8\ub860\uc744 \ubc1c\ud45c\ud55c\uc9c0 50\ub144\uc774 \uac00\uae4c\uc6cc\uc624\ub294 \uc624\ub298\uc5d0 \uc774\ub974\uae30\uae4c\uc9c0, \ub0b4 \uc790\uc2e0\uc774 8\uccb4\uc9c8\ub860\uc5d0 \ub300\ud55c \uacf5\uac1c \uac15\uc5f0\uc774\ub098 \ubc1c\ud45c\ub97c \ubbf8\ub8e8\uc5b4 \uc628 \uac83\uc740 \uc544\uc9c1 \ub204\uad6c\ub098\uac00 \ud560 \uc218 \uc788\ub294 \uc27d\uace0 \uc815\ud655\ud55c \uccb4\uc9c8 \uac10\ubcc4\ubc95\uc744 \ubc1c\uacac\ud560 \uc218 \uc5c6\uc5c8\uae30 \ub54c\ubb38\uc774\ub2e4. \ud604\uc7ac \ub0b4\uac00 \uc0ac\uc6a9\ud558\ub294 \uac10\ubcc4\ubc95\uc740 \ub0b4 \uc790\uc2e0\uc758 \uc5f0\uad6c\ub85c \ubc1c\uacac\ud55c \uc778\uac04 \uc694\uace8\ub3d9\ub9e5(\u6493\u9aa8\u52d5\u8108)\uc5d0\uc11c \ud53c\uac80\uc790\uc758 \uccb4\uc9c8\uc758 \uc2f8\uc778(sign)\uc744 \ucc3e\ub294 \uac83\uc778\ub370, \uc774 \ub610\ud55c \uc27d\uc9c0 \uc54a\uc544 \uc624\uc9c4\ud558\uae30 \uc26c\uc6b0\ubbc0\ub85c \ub204\uad6c\ub098\uac00 \uc27d\uac8c \uc0ac\uc6a9\ud560 \uc218 \uc788\ub294 \uacfc\ud559\uc801\uc778 \ubc29\ubc95 \uc744 \uad6d\ub0b4\uc678 \uc5f0\uad6c\uc9c4\uc5d0 \uc758\ub8b0\ud558\uc5ec \ubcf4\uc558\uc73c\ub098 \uc774\ub807\ub2e4 \ud560 \uc131\uacfc\uac00 \uc5c6\uc73c\ubbc0\ub85c \ucd5c\ud6c4\uc758 \ubc29\ubc95\uc73c\ub85c \ub098\uc758 \uac10\ubcc4\ubc95\uc5d0 \uc758\ud558\uc5ec \ud310\ub2e8\ub41c 8\uccb4\uc9c8 \uc911 \uac19\uc740 \uccb4\uc9c8\ub4e4\uc5d0\uc11c \ub3d9\uc77c\uc810\uc744, \uadf8\ub9ac\uace0 \uc11c\ub85c \ub2e4\ub978 \uccb4\uc9c8\ub4e4\uc5d0\uc11c \ucc28\uc774\uc810\uc744 \uacfc\ud559\uc801\uc73c\ub85c \ucc3e\uc544\ub0b4\ub294 \ubc29\ubc95\uc744 \uc54c\uc544\ubcf4\uace0 \uc788\ub294 \uc911\uc774\ub2e4.'", "sentence_answer": "\ud604\uc7ac \ub0b4\uac00 \uc0ac\uc6a9\ud558\ub294 \uac10\ubcc4\ubc95\uc740 \ub0b4 \uc790\uc2e0\uc758 \uc5f0\uad6c\ub85c \ubc1c\uacac\ud55c \uc778\uac04 \uc694\uace8\ub3d9\ub9e5(\u6493\u9aa8\u52d5\u8108)\uc5d0\uc11c \ud53c\uac80\uc790\uc758 \uccb4\uc9c8\uc758 \uc2f8\uc778(sign)\uc744 \ucc3e\ub294 \uac83\uc778\ub370, \uc774 \ub610\ud55c \uc27d\uc9c0 \uc54a\uc544 \uc624\uc9c4\ud558\uae30 \uc26c\uc6b0\ubbc0\ub85c \ub204\uad6c\ub098\uac00 \uc27d\uac8c \uc0ac\uc6a9\ud560 \uc218 \uc788\ub294 \uacfc\ud559\uc801\uc778 \ubc29\ubc95 \uc744 \uad6d\ub0b4\uc678 \uc5f0\uad6c\uc9c4\uc5d0 \uc758\ub8b0\ud558\uc5ec \ubcf4\uc558\uc73c\ub098 \uc774\ub807\ub2e4 \ud560 \uc131\uacfc\uac00 \uc5c6\uc73c\ubbc0\ub85c \ucd5c\ud6c4\uc758 \ubc29\ubc95\uc73c\ub85c \ub098\uc758 \uac10\ubcc4\ubc95\uc5d0 \uc758\ud558\uc5ec \ud310\ub2e8\ub41c 8\uccb4\uc9c8 \uc911 \uac19\uc740 \uccb4\uc9c8\ub4e4\uc5d0\uc11c \ub3d9\uc77c\uc810\uc744, \uadf8\ub9ac\uace0 \uc11c\ub85c \ub2e4\ub978 \uccb4\uc9c8\ub4e4\uc5d0\uc11c \ucc28\uc774\uc810\uc744 \uacfc\ud559\uc801\uc73c\ub85c \ucc3e\uc544\ub0b4\ub294 \ubc29\ubc95\uc744 \uc54c\uc544\ubcf4\uace0 \uc788\ub294 \uc911\uc774\ub2e4.", "paragraph_id": "6566734-8-2", "paragraph_question": "question: 8\uccb4\uc9c8\ub860\uc5d0\uc11c \uccb4\uc9c8 \uac10\ubcc4\uc774 \uc27d\uc9c0\uc54a\uc544 \uc5b4\ub5a4 \ubc29\ubc95\uc744 \uc5f0\uad6c\uc9c4\uc5d0 \uc758\ub8b0\ud588\ub098?, context: '\uc77c\ucc0d\uc774 1965\ub144 \uad6d\uc81c\ud559\ud68c\uc5d0\uc11c \uc774\ub7ec\ud55c 8\uccb4\uc9c8\ub860\uc744 \ubc1c\ud45c\ud55c\uc9c0 50\ub144\uc774 \uac00\uae4c\uc6cc\uc624\ub294 \uc624\ub298\uc5d0 \uc774\ub974\uae30\uae4c\uc9c0, \ub0b4 \uc790\uc2e0\uc774 8\uccb4\uc9c8\ub860\uc5d0 \ub300\ud55c \uacf5\uac1c \uac15\uc5f0\uc774\ub098 \ubc1c\ud45c\ub97c \ubbf8\ub8e8\uc5b4 \uc628 \uac83\uc740 \uc544\uc9c1 \ub204\uad6c\ub098\uac00 \ud560 \uc218 \uc788\ub294 \uc27d\uace0 \uc815\ud655\ud55c \uccb4\uc9c8 \uac10\ubcc4\ubc95\uc744 \ubc1c\uacac\ud560 \uc218 \uc5c6\uc5c8\uae30 \ub54c\ubb38\uc774\ub2e4. \ud604\uc7ac \ub0b4\uac00 \uc0ac\uc6a9\ud558\ub294 \uac10\ubcc4\ubc95\uc740 \ub0b4 \uc790\uc2e0\uc758 \uc5f0\uad6c\ub85c \ubc1c\uacac\ud55c \uc778\uac04 \uc694\uace8\ub3d9\ub9e5(\u6493\u9aa8\u52d5\u8108)\uc5d0\uc11c \ud53c\uac80\uc790\uc758 \uccb4\uc9c8\uc758 \uc2f8\uc778(sign)\uc744 \ucc3e\ub294 \uac83\uc778\ub370, \uc774 \ub610\ud55c \uc27d\uc9c0 \uc54a\uc544 \uc624\uc9c4\ud558\uae30 \uc26c\uc6b0\ubbc0\ub85c \ub204\uad6c\ub098\uac00 \uc27d\uac8c \uc0ac\uc6a9\ud560 \uc218 \uc788\ub294 \uacfc\ud559\uc801\uc778 \ubc29\ubc95\uc744 \uad6d\ub0b4\uc678 \uc5f0\uad6c\uc9c4\uc5d0 \uc758\ub8b0\ud558\uc5ec \ubcf4\uc558\uc73c\ub098 \uc774\ub807\ub2e4 \ud560 \uc131\uacfc\uac00 \uc5c6\uc73c\ubbc0\ub85c \ucd5c\ud6c4\uc758 \ubc29\ubc95\uc73c\ub85c \ub098\uc758 \uac10\ubcc4\ubc95\uc5d0 \uc758\ud558\uc5ec \ud310\ub2e8\ub41c 8\uccb4\uc9c8 \uc911 \uac19\uc740 \uccb4\uc9c8\ub4e4\uc5d0\uc11c \ub3d9\uc77c\uc810\uc744, \uadf8\ub9ac\uace0 \uc11c\ub85c \ub2e4\ub978 \uccb4\uc9c8\ub4e4\uc5d0\uc11c \ucc28\uc774\uc810\uc744 \uacfc\ud559\uc801\uc73c\ub85c \ucc3e\uc544\ub0b4\ub294 \ubc29\ubc95\uc744 \uc54c\uc544\ubcf4\uace0 \uc788\ub294 \uc911\uc774\ub2e4.'"} {"question": "\uace0\ub824\ub300\ud559\uad50\uc758 \ubaa8\ud0dc\uc778 \ubcf4\uc131\uc804\ubb38\ud559\uad50\uac00 \uc124\ub9bd\ub41c \ub0a0\uc9dc\ub294?", "paragraph": "1905\ub144 5\uc6d4 5\uc77c\uc5d0 \uc124\ub9bd\ub3fc \ud55c\uad6d \ucd5c\ucd08\uc758 \ub300\ud559\uad50\uc774\uc790 \uc870\uc120\uc778\uc774 \uc138\uc6b4 \uac00\uc7a5 \uc624\ub798\ub41c \uadfc\ub300\uc801 \uace0\ub4f1 \uad50\uc721 \uae30\uad00\uc73c\ub85c \uae30\ub85d \ub41c \ubcf4\uc131\uc804\ubb38\ud559\uad50(\u666e\u6210\u5c08\u9580\u5b78\u6821)\uac00 \uace0\ub824\ub300\ud559\uad50\uc758 \ubaa8\ud0dc\ub2e4. \u2018\ubcf4\uc131\u2019(\u666e\u6210)\uc774\ub77c\ub294 \uc774\ub984\uc740 \ub300\ud55c\uc81c\uad6d \uad11\ubb34 9\ub144\uc778 1905\ub144\uc5d0 \ub0b4\uc7a5\uc6d0\uacbd, \uc989 \ud669\uc2e4\uc758 \uc7ac\uc815\uc744 \ub2f4\ub2f9\ud558\ub294 \ub300\uc2e0\uc778 \uc774\uc6a9\uc775\uc774 \uace0\uc885\ud669\uc81c\uc758 \uc7ac\uc815\uc801 \ud6c4\uc6d0\uacfc \u2018\ubcf4\uc131\u2019(\u666e\u6210)\uc774\ub77c\ub294 \uad50\uba85(\u6821\u540d)\uacfc \ud669\uc2e4 \ubb38\uc7a5\uc778 \u2018\uc774\ud654\ubb38\u2019(\u674e\u82b1\u7d0b)\uc744 \uad50\ud45c(\u6821\u8868)\ub85c \ud558\uc0ac\ubc1b\uc740 \ub370 \uadf8 \uae30\uc6d0\uc744 \ub454\ub2e4. \ud558\uc9c0\ub9cc \uc2dc\uc791 \uacfc\uc815 \uc790\uccb4\uac00 \uc21c\ud0c4\ud588\ub358 \uac83\uc740 \uc544\ub2c8\ub2e4. \uc744\uc0ac\ub291\uc57d \uc774\ud6c4 \uc774\uc6a9\uc775\uc758 \ub9dd\uba85\uacfc \ubcc4\uc138\ub85c \uacbd\uc601\ub09c\uc5d0 \ube60\uc9c4 \ud559\uad50\ub97c 1910\ub144 12\uc6d4\uc5d0 \uc190\ubcd1\ud76c\uac00 \ub3c4\uc8fc\ub85c \uc788\ub294 \ucc9c\ub3c4\uad50\uc5d0\uc11c \uc778\uc218\ud558\uc600\uc73c\ub098 \uc190\ubcd1\ud76c\uac00 3\u00b71 \uc6b4\ub3d9 \uc9c1\ud6c4 \ud53c\uac80\ub418\ub294 \ub4f1 \ub09c\uad00\uc5d0 \ubd09\ucc29\ud558\ub294 \uc77c\uc774 \uc7a6\uc558\ub2e4. \uc774\ud6c4 1932\ub144 3\uc6d4 \uc7ac\uc815\ub09c\uc5d0 \ube60\uc9c4 \ud559\uad50\ub97c \uae40\uc131\uc218\uc758 \uc8fc\ub3c4\ub85c \uc7ac\ub2e8\ubc95\uc778 \uc911\uc559\ud559\uc6d0\uc774 \uc778\uc218\ud558\uac8c\ub410\ub2e4. \ud604\uc7ac \ud559\uad50\uac00 \uc704\uce58\ud55c \uc131\ubd81\uad6c \uc548\uc554\ub3d9\uc73c\ub85c\uc758 \uc774\uc804\uc740 1934\ub144 9\uc6d4\uc5d0 \uc774\ub8e8\uc5b4\uc84c\ub2e4. \uc778\ubb38\uc0ac\ud68c\uacc4 \uc9c0\uc5ed\uc758 \uc911\uc559\uc5d0 \uc704\uce58\ud55c \ubcf8\uad00\uc740 1933\ub144 9\uc6d4\uc5d0 \ucc29\uacf5\ud558\uc5ec \uc774\ub4ec\ud574 1934\ub144 9\uc6d4 \uc900\uacf5\ub410\uc73c\uba70, 1937\ub144\uc5d0\ub294 \ubcf4\uc131\uc804\ubb38\ud559\uad50 \ucc3d\ub9bd 30\uc8fc\ub144 \uae30\ub150\uc0ac\uc5c5\uc73c\ub85c \ub3c4\uc11c\uad00\uc744 \uc900\uacf5 \ubc0f \uac1c\uad00\ud558\uc600\ub2e4. \ud604\uc7ac \ubcf8\uad00\uacfc \uc911\uc559\ub3c4\uc11c\uad00 \uad6c\uad00\uc740 \uac01\uac01 \uc0ac\uc801 \uc81c285\ud638\uc640 \uc81c286\ud638\ub85c \uc9c0\uc815\ub3fc \uc788\ub2e4", "answer": "1905\ub144 5\uc6d4 5\uc77c", "sentence": "1905\ub144 5\uc6d4 5\uc77c \uc5d0 \uc124\ub9bd\ub3fc \ud55c\uad6d \ucd5c\ucd08\uc758 \ub300\ud559\uad50\uc774\uc790 \uc870\uc120\uc778\uc774 \uc138\uc6b4 \uac00\uc7a5 \uc624\ub798\ub41c \uadfc\ub300\uc801 \uace0\ub4f1 \uad50\uc721 \uae30\uad00\uc73c\ub85c \uae30\ub85d \ub41c \ubcf4\uc131\uc804\ubb38\ud559\uad50(\u666e\u6210\u5c08\u9580\u5b78\u6821)\uac00 \uace0\ub824\ub300\ud559\uad50\uc758 \ubaa8\ud0dc\ub2e4.", "paragraph_sentence": " 1905\ub144 5\uc6d4 5\uc77c \uc5d0 \uc124\ub9bd\ub3fc \ud55c\uad6d \ucd5c\ucd08\uc758 \ub300\ud559\uad50\uc774\uc790 \uc870\uc120\uc778\uc774 \uc138\uc6b4 \uac00\uc7a5 \uc624\ub798\ub41c \uadfc\ub300\uc801 \uace0\ub4f1 \uad50\uc721 \uae30\uad00\uc73c\ub85c \uae30\ub85d \ub41c \ubcf4\uc131\uc804\ubb38\ud559\uad50(\u666e\u6210\u5c08\u9580\u5b78\u6821)\uac00 \uace0\ub824\ub300\ud559\uad50\uc758 \ubaa8\ud0dc\ub2e4. \u2018\ubcf4\uc131\u2019(\u666e\u6210)\uc774\ub77c\ub294 \uc774\ub984\uc740 \ub300\ud55c\uc81c\uad6d \uad11\ubb34 9\ub144\uc778 1905\ub144\uc5d0 \ub0b4\uc7a5\uc6d0\uacbd, \uc989 \ud669\uc2e4\uc758 \uc7ac\uc815\uc744 \ub2f4\ub2f9\ud558\ub294 \ub300\uc2e0\uc778 \uc774\uc6a9\uc775\uc774 \uace0\uc885\ud669\uc81c\uc758 \uc7ac\uc815\uc801 \ud6c4\uc6d0\uacfc \u2018\ubcf4\uc131\u2019(\u666e\u6210)\uc774\ub77c\ub294 \uad50\uba85(\u6821\u540d)\uacfc \ud669\uc2e4 \ubb38\uc7a5\uc778 \u2018\uc774\ud654\ubb38\u2019(\u674e\u82b1\u7d0b)\uc744 \uad50\ud45c(\u6821\u8868)\ub85c \ud558\uc0ac\ubc1b\uc740 \ub370 \uadf8 \uae30\uc6d0\uc744 \ub454\ub2e4. \ud558\uc9c0\ub9cc \uc2dc\uc791 \uacfc\uc815 \uc790\uccb4\uac00 \uc21c\ud0c4\ud588\ub358 \uac83\uc740 \uc544\ub2c8\ub2e4. \uc744\uc0ac\ub291\uc57d \uc774\ud6c4 \uc774\uc6a9\uc775\uc758 \ub9dd\uba85\uacfc \ubcc4\uc138\ub85c \uacbd\uc601\ub09c\uc5d0 \ube60\uc9c4 \ud559\uad50\ub97c 1910\ub144 12\uc6d4\uc5d0 \uc190\ubcd1\ud76c\uac00 \ub3c4\uc8fc\ub85c \uc788\ub294 \ucc9c\ub3c4\uad50\uc5d0\uc11c \uc778\uc218\ud558\uc600\uc73c\ub098 \uc190\ubcd1\ud76c\uac00 3\u00b71 \uc6b4\ub3d9 \uc9c1\ud6c4 \ud53c\uac80\ub418\ub294 \ub4f1 \ub09c\uad00\uc5d0 \ubd09\ucc29\ud558\ub294 \uc77c\uc774 \uc7a6\uc558\ub2e4. \uc774\ud6c4 1932\ub144 3\uc6d4 \uc7ac\uc815\ub09c\uc5d0 \ube60\uc9c4 \ud559\uad50\ub97c \uae40\uc131\uc218\uc758 \uc8fc\ub3c4\ub85c \uc7ac\ub2e8\ubc95\uc778 \uc911\uc559\ud559\uc6d0\uc774 \uc778\uc218\ud558\uac8c\ub410\ub2e4. \ud604\uc7ac \ud559\uad50\uac00 \uc704\uce58\ud55c \uc131\ubd81\uad6c \uc548\uc554\ub3d9\uc73c\ub85c\uc758 \uc774\uc804\uc740 1934\ub144 9\uc6d4\uc5d0 \uc774\ub8e8\uc5b4\uc84c\ub2e4. \uc778\ubb38\uc0ac\ud68c\uacc4 \uc9c0\uc5ed\uc758 \uc911\uc559\uc5d0 \uc704\uce58\ud55c \ubcf8\uad00\uc740 1933\ub144 9\uc6d4\uc5d0 \ucc29\uacf5\ud558\uc5ec \uc774\ub4ec\ud574 1934\ub144 9\uc6d4 \uc900\uacf5\ub410\uc73c\uba70, 1937\ub144\uc5d0\ub294 \ubcf4\uc131\uc804\ubb38\ud559\uad50 \ucc3d\ub9bd 30\uc8fc\ub144 \uae30\ub150\uc0ac\uc5c5\uc73c\ub85c \ub3c4\uc11c\uad00\uc744 \uc900\uacf5 \ubc0f \uac1c\uad00\ud558\uc600\ub2e4. \ud604\uc7ac \ubcf8\uad00\uacfc \uc911\uc559\ub3c4\uc11c\uad00 \uad6c\uad00\uc740 \uac01\uac01 \uc0ac\uc801 \uc81c285\ud638\uc640 \uc81c286\ud638\ub85c \uc9c0\uc815\ub3fc \uc788\ub2e4", "paragraph_answer": " 1905\ub144 5\uc6d4 5\uc77c \uc5d0 \uc124\ub9bd\ub3fc \ud55c\uad6d \ucd5c\ucd08\uc758 \ub300\ud559\uad50\uc774\uc790 \uc870\uc120\uc778\uc774 \uc138\uc6b4 \uac00\uc7a5 \uc624\ub798\ub41c \uadfc\ub300\uc801 \uace0\ub4f1 \uad50\uc721 \uae30\uad00\uc73c\ub85c \uae30\ub85d \ub41c \ubcf4\uc131\uc804\ubb38\ud559\uad50(\u666e\u6210\u5c08\u9580\u5b78\u6821)\uac00 \uace0\ub824\ub300\ud559\uad50\uc758 \ubaa8\ud0dc\ub2e4. \u2018\ubcf4\uc131\u2019(\u666e\u6210)\uc774\ub77c\ub294 \uc774\ub984\uc740 \ub300\ud55c\uc81c\uad6d \uad11\ubb34 9\ub144\uc778 1905\ub144\uc5d0 \ub0b4\uc7a5\uc6d0\uacbd, \uc989 \ud669\uc2e4\uc758 \uc7ac\uc815\uc744 \ub2f4\ub2f9\ud558\ub294 \ub300\uc2e0\uc778 \uc774\uc6a9\uc775\uc774 \uace0\uc885\ud669\uc81c\uc758 \uc7ac\uc815\uc801 \ud6c4\uc6d0\uacfc \u2018\ubcf4\uc131\u2019(\u666e\u6210)\uc774\ub77c\ub294 \uad50\uba85(\u6821\u540d)\uacfc \ud669\uc2e4 \ubb38\uc7a5\uc778 \u2018\uc774\ud654\ubb38\u2019(\u674e\u82b1\u7d0b)\uc744 \uad50\ud45c(\u6821\u8868)\ub85c \ud558\uc0ac\ubc1b\uc740 \ub370 \uadf8 \uae30\uc6d0\uc744 \ub454\ub2e4. \ud558\uc9c0\ub9cc \uc2dc\uc791 \uacfc\uc815 \uc790\uccb4\uac00 \uc21c\ud0c4\ud588\ub358 \uac83\uc740 \uc544\ub2c8\ub2e4. \uc744\uc0ac\ub291\uc57d \uc774\ud6c4 \uc774\uc6a9\uc775\uc758 \ub9dd\uba85\uacfc \ubcc4\uc138\ub85c \uacbd\uc601\ub09c\uc5d0 \ube60\uc9c4 \ud559\uad50\ub97c 1910\ub144 12\uc6d4\uc5d0 \uc190\ubcd1\ud76c\uac00 \ub3c4\uc8fc\ub85c \uc788\ub294 \ucc9c\ub3c4\uad50\uc5d0\uc11c \uc778\uc218\ud558\uc600\uc73c\ub098 \uc190\ubcd1\ud76c\uac00 3\u00b71 \uc6b4\ub3d9 \uc9c1\ud6c4 \ud53c\uac80\ub418\ub294 \ub4f1 \ub09c\uad00\uc5d0 \ubd09\ucc29\ud558\ub294 \uc77c\uc774 \uc7a6\uc558\ub2e4. \uc774\ud6c4 1932\ub144 3\uc6d4 \uc7ac\uc815\ub09c\uc5d0 \ube60\uc9c4 \ud559\uad50\ub97c \uae40\uc131\uc218\uc758 \uc8fc\ub3c4\ub85c \uc7ac\ub2e8\ubc95\uc778 \uc911\uc559\ud559\uc6d0\uc774 \uc778\uc218\ud558\uac8c\ub410\ub2e4. \ud604\uc7ac \ud559\uad50\uac00 \uc704\uce58\ud55c \uc131\ubd81\uad6c \uc548\uc554\ub3d9\uc73c\ub85c\uc758 \uc774\uc804\uc740 1934\ub144 9\uc6d4\uc5d0 \uc774\ub8e8\uc5b4\uc84c\ub2e4. \uc778\ubb38\uc0ac\ud68c\uacc4 \uc9c0\uc5ed\uc758 \uc911\uc559\uc5d0 \uc704\uce58\ud55c \ubcf8\uad00\uc740 1933\ub144 9\uc6d4\uc5d0 \ucc29\uacf5\ud558\uc5ec \uc774\ub4ec\ud574 1934\ub144 9\uc6d4 \uc900\uacf5\ub410\uc73c\uba70, 1937\ub144\uc5d0\ub294 \ubcf4\uc131\uc804\ubb38\ud559\uad50 \ucc3d\ub9bd 30\uc8fc\ub144 \uae30\ub150\uc0ac\uc5c5\uc73c\ub85c \ub3c4\uc11c\uad00\uc744 \uc900\uacf5 \ubc0f \uac1c\uad00\ud558\uc600\ub2e4. \ud604\uc7ac \ubcf8\uad00\uacfc \uc911\uc559\ub3c4\uc11c\uad00 \uad6c\uad00\uc740 \uac01\uac01 \uc0ac\uc801 \uc81c285\ud638\uc640 \uc81c286\ud638\ub85c \uc9c0\uc815\ub3fc \uc788\ub2e4", "sentence_answer": " 1905\ub144 5\uc6d4 5\uc77c \uc5d0 \uc124\ub9bd\ub3fc \ud55c\uad6d \ucd5c\ucd08\uc758 \ub300\ud559\uad50\uc774\uc790 \uc870\uc120\uc778\uc774 \uc138\uc6b4 \uac00\uc7a5 \uc624\ub798\ub41c \uadfc\ub300\uc801 \uace0\ub4f1 \uad50\uc721 \uae30\uad00\uc73c\ub85c \uae30\ub85d \ub41c \ubcf4\uc131\uc804\ubb38\ud559\uad50(\u666e\u6210\u5c08\u9580\u5b78\u6821)\uac00 \uace0\ub824\ub300\ud559\uad50\uc758 \ubaa8\ud0dc\ub2e4.", "paragraph_id": "6474998-0-1", "paragraph_question": "question: \uace0\ub824\ub300\ud559\uad50\uc758 \ubaa8\ud0dc\uc778 \ubcf4\uc131\uc804\ubb38\ud559\uad50\uac00 \uc124\ub9bd\ub41c \ub0a0\uc9dc\ub294?, context: 1905\ub144 5\uc6d4 5\uc77c\uc5d0 \uc124\ub9bd\ub3fc \ud55c\uad6d \ucd5c\ucd08\uc758 \ub300\ud559\uad50\uc774\uc790 \uc870\uc120\uc778\uc774 \uc138\uc6b4 \uac00\uc7a5 \uc624\ub798\ub41c \uadfc\ub300\uc801 \uace0\ub4f1 \uad50\uc721 \uae30\uad00\uc73c\ub85c \uae30\ub85d \ub41c \ubcf4\uc131\uc804\ubb38\ud559\uad50(\u666e\u6210\u5c08\u9580\u5b78\u6821)\uac00 \uace0\ub824\ub300\ud559\uad50\uc758 \ubaa8\ud0dc\ub2e4. \u2018\ubcf4\uc131\u2019(\u666e\u6210)\uc774\ub77c\ub294 \uc774\ub984\uc740 \ub300\ud55c\uc81c\uad6d \uad11\ubb34 9\ub144\uc778 1905\ub144\uc5d0 \ub0b4\uc7a5\uc6d0\uacbd, \uc989 \ud669\uc2e4\uc758 \uc7ac\uc815\uc744 \ub2f4\ub2f9\ud558\ub294 \ub300\uc2e0\uc778 \uc774\uc6a9\uc775\uc774 \uace0\uc885\ud669\uc81c\uc758 \uc7ac\uc815\uc801 \ud6c4\uc6d0\uacfc \u2018\ubcf4\uc131\u2019(\u666e\u6210)\uc774\ub77c\ub294 \uad50\uba85(\u6821\u540d)\uacfc \ud669\uc2e4 \ubb38\uc7a5\uc778 \u2018\uc774\ud654\ubb38\u2019(\u674e\u82b1\u7d0b)\uc744 \uad50\ud45c(\u6821\u8868)\ub85c \ud558\uc0ac\ubc1b\uc740 \ub370 \uadf8 \uae30\uc6d0\uc744 \ub454\ub2e4. \ud558\uc9c0\ub9cc \uc2dc\uc791 \uacfc\uc815 \uc790\uccb4\uac00 \uc21c\ud0c4\ud588\ub358 \uac83\uc740 \uc544\ub2c8\ub2e4. \uc744\uc0ac\ub291\uc57d \uc774\ud6c4 \uc774\uc6a9\uc775\uc758 \ub9dd\uba85\uacfc \ubcc4\uc138\ub85c \uacbd\uc601\ub09c\uc5d0 \ube60\uc9c4 \ud559\uad50\ub97c 1910\ub144 12\uc6d4\uc5d0 \uc190\ubcd1\ud76c\uac00 \ub3c4\uc8fc\ub85c \uc788\ub294 \ucc9c\ub3c4\uad50\uc5d0\uc11c \uc778\uc218\ud558\uc600\uc73c\ub098 \uc190\ubcd1\ud76c\uac00 3\u00b71 \uc6b4\ub3d9 \uc9c1\ud6c4 \ud53c\uac80\ub418\ub294 \ub4f1 \ub09c\uad00\uc5d0 \ubd09\ucc29\ud558\ub294 \uc77c\uc774 \uc7a6\uc558\ub2e4. \uc774\ud6c4 1932\ub144 3\uc6d4 \uc7ac\uc815\ub09c\uc5d0 \ube60\uc9c4 \ud559\uad50\ub97c \uae40\uc131\uc218\uc758 \uc8fc\ub3c4\ub85c \uc7ac\ub2e8\ubc95\uc778 \uc911\uc559\ud559\uc6d0\uc774 \uc778\uc218\ud558\uac8c\ub410\ub2e4. \ud604\uc7ac \ud559\uad50\uac00 \uc704\uce58\ud55c \uc131\ubd81\uad6c \uc548\uc554\ub3d9\uc73c\ub85c\uc758 \uc774\uc804\uc740 1934\ub144 9\uc6d4\uc5d0 \uc774\ub8e8\uc5b4\uc84c\ub2e4. \uc778\ubb38\uc0ac\ud68c\uacc4 \uc9c0\uc5ed\uc758 \uc911\uc559\uc5d0 \uc704\uce58\ud55c \ubcf8\uad00\uc740 1933\ub144 9\uc6d4\uc5d0 \ucc29\uacf5\ud558\uc5ec \uc774\ub4ec\ud574 1934\ub144 9\uc6d4 \uc900\uacf5\ub410\uc73c\uba70, 1937\ub144\uc5d0\ub294 \ubcf4\uc131\uc804\ubb38\ud559\uad50 \ucc3d\ub9bd 30\uc8fc\ub144 \uae30\ub150\uc0ac\uc5c5\uc73c\ub85c \ub3c4\uc11c\uad00\uc744 \uc900\uacf5 \ubc0f \uac1c\uad00\ud558\uc600\ub2e4. \ud604\uc7ac \ubcf8\uad00\uacfc \uc911\uc559\ub3c4\uc11c\uad00 \uad6c\uad00\uc740 \uac01\uac01 \uc0ac\uc801 \uc81c285\ud638\uc640 \uc81c286\ud638\ub85c \uc9c0\uc815\ub3fc \uc788\ub2e4"} {"question": "\uce74\uc2dc\ub2c8-\ud558\uc704\ud5cc\uc2a4\ub294 \ubb34\uc5c7\uc778\uac00\uc694?", "paragraph": "\uce74\uc2dc\ub2c8-\ud558\uc704\ud5cc\uc2a4(Cassini-Huygens)\ub294 \ubbf8\uad6d\uacfc \uc720\ub7fd\uc758 \uacf5\ub3d9 \ud1a0\uc131 \ubb34\uc778 \ud0d0\uc0ac\uc120\uc774\ub2e4. \uce74\uc2dc\ub2c8-\ud558\uc704\ud5cc\uc2a4\ub294 NASA \uce74\uc2dc\ub2c8 \uada4\ub3c4\uc120\uacfc ESA \ud558\uc704\ud5cc\uc2a4 \ud0d0\uc0ac\uc120(\uc774\ud0c8\ub9ac\uc544 \ucd9c\uc2e0 \ud504\ub791\uc2a4 \ucc9c\ubb38\ud559\uc790 \uc870\ubc18\ub2c8 \ub3c4\uba54\ub2c8\ucf54 \uce74\uc2dc\ub2c8\uc640 \ub124\ub35c\ub780\ub4dc\uc758 \ucc9c\ubb38\ud559\uc790, \uc218\ud559\uc790, \ubb3c\ub9ac\ud559\uc790 \ud06c\ub9ac\uc2a4\ud2f0\uc548 \ud558\uc704\ud5cc\uc2a4\uc758 \uc774\ub984\uc744 \ub534 \uac83\uc774\ub2e4) \ub458\ub85c \ub098\ub25c\ub2e4. \uce74\uc2dc\ub2c8-\ud558\uc704\ud5cc\uc2a4\ub294 1997\ub144 10\uc6d4 15\uc77c \ubc1c\uc0ac\ub418\uc5c8\uc73c\uba70 2004\ub144 7\uc6d4 1\uc77c \ud1a0\uc131 \uada4\ub3c4\uc5d0 \uc9c4\uc785\ud558\uc600\ub2e4. \uc81c\ud2b8 \ucd94\uc9c4 \uc5f0\uad6c\uc18c(JPL)\uc5d0 \uc758\ud558\uba74 \ud558\uc704\ud5cc\uc2a4 \ud0d0\uc0ac\uc120\uc740 2004\ub144 12\uc6d4 25\uc77c UTC 2:00 \ubb34\ub835 \ubaa8\uc120\uc5d0\uc11c \ubd84\ub9ac\ub418\uc5b4 2005\ub144 1\uc6d4 14\uc77c \ud0c0\uc774\ud0c4\uc758 \ub300\uae30\uad8c\uc5d0 \uc9c4\uc785\ud558\uc600\ub2e4. \ud558\uc704\ud5cc\uc2a4\ub294 \ud0c0\uc774\ud0c4\uc758 \ud45c\uba74\uc5d0 \ucc29\ub959\ud558\uae30\uae4c\uc9c0\uc758 \ud0c0\uc774\ud0c4\uc758 \uc790\ub8cc\ub97c \uc9c0\uad6c\ub85c \ubcf4\ub0c8\ub2e4. \uce74\uc2dc\ub2c8-\ud558\uc704\ud5cc\uc2a4\ub294 \ud1a0\uc131\uc744 \uacf5\uc804\ud558\ub294 \ud0d0\uc0ac\uc120\uc73c\ub85c\ub294 \ucd5c\ucd08\uc774\uba70, \ud1a0\uc131\uc744 \ubc29\ubb38\ud55c \uae30\uccb4\ub85c\ub294 \ub124 \ubc88\uc9f8\uc774\ub2e4.", "answer": "\ud1a0\uc131 \ubb34\uc778 \ud0d0\uc0ac\uc120", "sentence": "\uce74\uc2dc\ub2c8-\ud558\uc704\ud5cc\uc2a4(Cassini-Huygens)\ub294 \ubbf8\uad6d\uacfc \uc720\ub7fd\uc758 \uacf5\ub3d9 \ud1a0\uc131 \ubb34\uc778 \ud0d0\uc0ac\uc120 \uc774\ub2e4.", "paragraph_sentence": " \uce74\uc2dc\ub2c8-\ud558\uc704\ud5cc\uc2a4(Cassini-Huygens)\ub294 \ubbf8\uad6d\uacfc \uc720\ub7fd\uc758 \uacf5\ub3d9 \ud1a0\uc131 \ubb34\uc778 \ud0d0\uc0ac\uc120 \uc774\ub2e4. \uce74\uc2dc\ub2c8-\ud558\uc704\ud5cc\uc2a4\ub294 NASA \uce74\uc2dc\ub2c8 \uada4\ub3c4\uc120\uacfc ESA \ud558\uc704\ud5cc\uc2a4 \ud0d0\uc0ac\uc120(\uc774\ud0c8\ub9ac\uc544 \ucd9c\uc2e0 \ud504\ub791\uc2a4 \ucc9c\ubb38\ud559\uc790 \uc870\ubc18\ub2c8 \ub3c4\uba54\ub2c8\ucf54 \uce74\uc2dc\ub2c8\uc640 \ub124\ub35c\ub780\ub4dc\uc758 \ucc9c\ubb38\ud559\uc790, \uc218\ud559\uc790, \ubb3c\ub9ac\ud559\uc790 \ud06c\ub9ac\uc2a4\ud2f0\uc548 \ud558\uc704\ud5cc\uc2a4\uc758 \uc774\ub984\uc744 \ub534 \uac83\uc774\ub2e4) \ub458\ub85c \ub098\ub25c\ub2e4. \uce74\uc2dc\ub2c8-\ud558\uc704\ud5cc\uc2a4\ub294 1997\ub144 10\uc6d4 15\uc77c \ubc1c\uc0ac\ub418\uc5c8\uc73c\uba70 2004\ub144 7\uc6d4 1\uc77c \ud1a0\uc131 \uada4\ub3c4\uc5d0 \uc9c4\uc785\ud558\uc600\ub2e4. \uc81c\ud2b8 \ucd94\uc9c4 \uc5f0\uad6c\uc18c(JPL)\uc5d0 \uc758\ud558\uba74 \ud558\uc704\ud5cc\uc2a4 \ud0d0\uc0ac\uc120\uc740 2004\ub144 12\uc6d4 25\uc77c UTC 2:00 \ubb34\ub835 \ubaa8\uc120\uc5d0\uc11c \ubd84\ub9ac\ub418\uc5b4 2005\ub144 1\uc6d4 14\uc77c \ud0c0\uc774\ud0c4\uc758 \ub300\uae30\uad8c\uc5d0 \uc9c4\uc785\ud558\uc600\ub2e4. \ud558\uc704\ud5cc\uc2a4\ub294 \ud0c0\uc774\ud0c4\uc758 \ud45c\uba74\uc5d0 \ucc29\ub959\ud558\uae30\uae4c\uc9c0\uc758 \ud0c0\uc774\ud0c4\uc758 \uc790\ub8cc\ub97c \uc9c0\uad6c\ub85c \ubcf4\ub0c8\ub2e4. \uce74\uc2dc\ub2c8-\ud558\uc704\ud5cc\uc2a4\ub294 \ud1a0\uc131\uc744 \uacf5\uc804\ud558\ub294 \ud0d0\uc0ac\uc120\uc73c\ub85c\ub294 \ucd5c\ucd08\uc774\uba70, \ud1a0\uc131\uc744 \ubc29\ubb38\ud55c \uae30\uccb4\ub85c\ub294 \ub124 \ubc88\uc9f8\uc774\ub2e4.", "paragraph_answer": "\uce74\uc2dc\ub2c8-\ud558\uc704\ud5cc\uc2a4(Cassini-Huygens)\ub294 \ubbf8\uad6d\uacfc \uc720\ub7fd\uc758 \uacf5\ub3d9 \ud1a0\uc131 \ubb34\uc778 \ud0d0\uc0ac\uc120 \uc774\ub2e4. \uce74\uc2dc\ub2c8-\ud558\uc704\ud5cc\uc2a4\ub294 NASA \uce74\uc2dc\ub2c8 \uada4\ub3c4\uc120\uacfc ESA \ud558\uc704\ud5cc\uc2a4 \ud0d0\uc0ac\uc120(\uc774\ud0c8\ub9ac\uc544 \ucd9c\uc2e0 \ud504\ub791\uc2a4 \ucc9c\ubb38\ud559\uc790 \uc870\ubc18\ub2c8 \ub3c4\uba54\ub2c8\ucf54 \uce74\uc2dc\ub2c8\uc640 \ub124\ub35c\ub780\ub4dc\uc758 \ucc9c\ubb38\ud559\uc790, \uc218\ud559\uc790, \ubb3c\ub9ac\ud559\uc790 \ud06c\ub9ac\uc2a4\ud2f0\uc548 \ud558\uc704\ud5cc\uc2a4\uc758 \uc774\ub984\uc744 \ub534 \uac83\uc774\ub2e4) \ub458\ub85c \ub098\ub25c\ub2e4. \uce74\uc2dc\ub2c8-\ud558\uc704\ud5cc\uc2a4\ub294 1997\ub144 10\uc6d4 15\uc77c \ubc1c\uc0ac\ub418\uc5c8\uc73c\uba70 2004\ub144 7\uc6d4 1\uc77c \ud1a0\uc131 \uada4\ub3c4\uc5d0 \uc9c4\uc785\ud558\uc600\ub2e4. \uc81c\ud2b8 \ucd94\uc9c4 \uc5f0\uad6c\uc18c(JPL)\uc5d0 \uc758\ud558\uba74 \ud558\uc704\ud5cc\uc2a4 \ud0d0\uc0ac\uc120\uc740 2004\ub144 12\uc6d4 25\uc77c UTC 2:00 \ubb34\ub835 \ubaa8\uc120\uc5d0\uc11c \ubd84\ub9ac\ub418\uc5b4 2005\ub144 1\uc6d4 14\uc77c \ud0c0\uc774\ud0c4\uc758 \ub300\uae30\uad8c\uc5d0 \uc9c4\uc785\ud558\uc600\ub2e4. \ud558\uc704\ud5cc\uc2a4\ub294 \ud0c0\uc774\ud0c4\uc758 \ud45c\uba74\uc5d0 \ucc29\ub959\ud558\uae30\uae4c\uc9c0\uc758 \ud0c0\uc774\ud0c4\uc758 \uc790\ub8cc\ub97c \uc9c0\uad6c\ub85c \ubcf4\ub0c8\ub2e4. \uce74\uc2dc\ub2c8-\ud558\uc704\ud5cc\uc2a4\ub294 \ud1a0\uc131\uc744 \uacf5\uc804\ud558\ub294 \ud0d0\uc0ac\uc120\uc73c\ub85c\ub294 \ucd5c\ucd08\uc774\uba70, \ud1a0\uc131\uc744 \ubc29\ubb38\ud55c \uae30\uccb4\ub85c\ub294 \ub124 \ubc88\uc9f8\uc774\ub2e4.", "sentence_answer": "\uce74\uc2dc\ub2c8-\ud558\uc704\ud5cc\uc2a4(Cassini-Huygens)\ub294 \ubbf8\uad6d\uacfc \uc720\ub7fd\uc758 \uacf5\ub3d9 \ud1a0\uc131 \ubb34\uc778 \ud0d0\uc0ac\uc120 \uc774\ub2e4.", "paragraph_id": "6561275-0-0", "paragraph_question": "question: \uce74\uc2dc\ub2c8-\ud558\uc704\ud5cc\uc2a4\ub294 \ubb34\uc5c7\uc778\uac00\uc694?, context: \uce74\uc2dc\ub2c8-\ud558\uc704\ud5cc\uc2a4(Cassini-Huygens)\ub294 \ubbf8\uad6d\uacfc \uc720\ub7fd\uc758 \uacf5\ub3d9 \ud1a0\uc131 \ubb34\uc778 \ud0d0\uc0ac\uc120\uc774\ub2e4. \uce74\uc2dc\ub2c8-\ud558\uc704\ud5cc\uc2a4\ub294 NASA \uce74\uc2dc\ub2c8 \uada4\ub3c4\uc120\uacfc ESA \ud558\uc704\ud5cc\uc2a4 \ud0d0\uc0ac\uc120(\uc774\ud0c8\ub9ac\uc544 \ucd9c\uc2e0 \ud504\ub791\uc2a4 \ucc9c\ubb38\ud559\uc790 \uc870\ubc18\ub2c8 \ub3c4\uba54\ub2c8\ucf54 \uce74\uc2dc\ub2c8\uc640 \ub124\ub35c\ub780\ub4dc\uc758 \ucc9c\ubb38\ud559\uc790, \uc218\ud559\uc790, \ubb3c\ub9ac\ud559\uc790 \ud06c\ub9ac\uc2a4\ud2f0\uc548 \ud558\uc704\ud5cc\uc2a4\uc758 \uc774\ub984\uc744 \ub534 \uac83\uc774\ub2e4) \ub458\ub85c \ub098\ub25c\ub2e4. \uce74\uc2dc\ub2c8-\ud558\uc704\ud5cc\uc2a4\ub294 1997\ub144 10\uc6d4 15\uc77c \ubc1c\uc0ac\ub418\uc5c8\uc73c\uba70 2004\ub144 7\uc6d4 1\uc77c \ud1a0\uc131 \uada4\ub3c4\uc5d0 \uc9c4\uc785\ud558\uc600\ub2e4. \uc81c\ud2b8 \ucd94\uc9c4 \uc5f0\uad6c\uc18c(JPL)\uc5d0 \uc758\ud558\uba74 \ud558\uc704\ud5cc\uc2a4 \ud0d0\uc0ac\uc120\uc740 2004\ub144 12\uc6d4 25\uc77c UTC 2:00 \ubb34\ub835 \ubaa8\uc120\uc5d0\uc11c \ubd84\ub9ac\ub418\uc5b4 2005\ub144 1\uc6d4 14\uc77c \ud0c0\uc774\ud0c4\uc758 \ub300\uae30\uad8c\uc5d0 \uc9c4\uc785\ud558\uc600\ub2e4. \ud558\uc704\ud5cc\uc2a4\ub294 \ud0c0\uc774\ud0c4\uc758 \ud45c\uba74\uc5d0 \ucc29\ub959\ud558\uae30\uae4c\uc9c0\uc758 \ud0c0\uc774\ud0c4\uc758 \uc790\ub8cc\ub97c \uc9c0\uad6c\ub85c \ubcf4\ub0c8\ub2e4. \uce74\uc2dc\ub2c8-\ud558\uc704\ud5cc\uc2a4\ub294 \ud1a0\uc131\uc744 \uacf5\uc804\ud558\ub294 \ud0d0\uc0ac\uc120\uc73c\ub85c\ub294 \ucd5c\ucd08\uc774\uba70, \ud1a0\uc131\uc744 \ubc29\ubb38\ud55c \uae30\uccb4\ub85c\ub294 \ub124 \ubc88\uc9f8\uc774\ub2e4."} {"question": "\ucd5c\uc9c4\uc2e4 \uc0ac\ub9dd\uc758 \uc9c1\uc811\uc801 \uc0ac\uc778\uc740 \ubb34\uc5c7\uc778\uac00?", "paragraph": "2008\ub144 10\uc6d4 2\uc77c \uc624\uc804 6\uc2dc 15\ubd84, \uc11c\uc6b8\ud2b9\ubcc4\uc2dc \uc11c\ucd08\uad6c \uc7a0\uc6d0\ub3d9\uc758 \uc790\ud0dd \uc695\uc2e4\uc5d0\uc11c \uc228\uc9c4 \ucc44\ub85c \ubc1c\uacac\ub418\uc5c8\ub2e4. \uacbd\ucc30\uc740 \uc790\uc0b4\ub85c \uc7a0\uc815\uc801\uc778 \uacb0\ub860\uc744 \ub0b4\ub838\uace0, \uc9c1\uc811\uc801\uc778 \uc0ac\uc778\uc740 \uacbd\ubd80\uc555\ubc15\uc9c8\uc2dd\uc0ac\uc600\ub2e4. \uacbd\ucc30 \uc870\uc0ac\uc5d0\uc11c \ucd5c\uc9c4\uc2e4\uc740 \uacbd\ud55c \uc218\uc900\uc758 \uc6b0\uc6b8\uc99d \uc99d\uc138\uac00 \uc788\uc5c8\uace0, \uc790\ub140\uc758 \uc591\uc721 \uacfc \uc5f0\uc608\uc778\uc758 \uc704\uc0c1 \ub4f1\uacfc \uad00\ub828\ud558\uc5ec \uace0\ubbfc\ud588\ub358 \uac83\uc73c\ub85c \ubc1d\ud600\uc84c\ub2e4. \ucc98\uc74c\uc5d0 \uc720\uc871\ub4e4\uc740 \uac80\ucc30\uc774 \ub0b4\ub9b0 \ubd80\uac80 \uacb0\uc815\uc5d0 \ubc18\ub300\ud588\uc73c\ub098 \uc720\uc871\ub4e4 \uc911 \ud55c \uba85\uc774 \ubd80\uac80 \ub3d9\uc758\uc11c\uc5d0 \uc11c\uba85\ud568\uc5d0 \ub530\ub77c \uac15\ub0a8\uc131\ubaa8\ubcd1\uc6d0\uc5d0\uc11c 2008\ub144 10\uc6d4 2\uc77c \uc624\ud6c4 9\uc2dc\ubd80\ud130 \uc624\ud6c4 10\uc2dc 20\ubd84\uae4c\uc9c0 1\uc2dc\uac04 20\ubd84\uc5d0 \uac78\uccd0 \ubd80\uac80\uc774 \uc9c4\ud589\ub418\uc5c8\ub2e4. \uad6d\ub9bd\uacfc\ud559\uc218\uc0ac\uc5f0\uad6c\uc18c\ub294 \ubd80\uac80 \uacb0\uacfc \ud0c0\uc0b4\uc744 \uc99d\uba85\ud560 \ub9cc\ud55c \ub2e8\uc11c\ub97c \ubc1c\uacac\ud560 \uc218 \uc5c6\ub2e4\ub294 \ubd80\uac80 \uc18c\uacac\uc744 \ubc1d\ud614\ub2e4. 2008\ub144 10\uc6d4 3\uc77c, \uacbd\ucc30\uc740 \uacf5\uc2dd \ube0c\ub9ac\ud551\uc5d0\uc11c \uc790\uc0b4\uc774\ub77c\uace0 \ubc1c\ud45c\ud558\uc600\ub2e4.", "answer": "\uacbd\ubd80\uc555\ubc15\uc9c8\uc2dd\uc0ac", "sentence": "\uacbd\ucc30\uc740 \uc790\uc0b4\ub85c \uc7a0\uc815\uc801\uc778 \uacb0\ub860\uc744 \ub0b4\ub838\uace0, \uc9c1\uc811\uc801\uc778 \uc0ac\uc778\uc740 \uacbd\ubd80\uc555\ubc15\uc9c8\uc2dd\uc0ac \uc600\ub2e4.", "paragraph_sentence": "2008\ub144 10\uc6d4 2\uc77c \uc624\uc804 6\uc2dc 15\ubd84, \uc11c\uc6b8\ud2b9\ubcc4\uc2dc \uc11c\ucd08\uad6c \uc7a0\uc6d0\ub3d9\uc758 \uc790\ud0dd \uc695\uc2e4\uc5d0\uc11c \uc228\uc9c4 \ucc44\ub85c \ubc1c\uacac\ub418\uc5c8\ub2e4. \uacbd\ucc30\uc740 \uc790\uc0b4\ub85c \uc7a0\uc815\uc801\uc778 \uacb0\ub860\uc744 \ub0b4\ub838\uace0, \uc9c1\uc811\uc801\uc778 \uc0ac\uc778\uc740 \uacbd\ubd80\uc555\ubc15\uc9c8\uc2dd\uc0ac \uc600\ub2e4. \uacbd\ucc30 \uc870\uc0ac\uc5d0\uc11c \ucd5c\uc9c4\uc2e4\uc740 \uacbd\ud55c \uc218\uc900\uc758 \uc6b0\uc6b8\uc99d \uc99d\uc138\uac00 \uc788\uc5c8\uace0, \uc790\ub140\uc758 \uc591\uc721 \uacfc \uc5f0\uc608\uc778\uc758 \uc704\uc0c1 \ub4f1\uacfc \uad00\ub828\ud558\uc5ec \uace0\ubbfc\ud588\ub358 \uac83\uc73c\ub85c \ubc1d\ud600\uc84c\ub2e4. \ucc98\uc74c\uc5d0 \uc720\uc871\ub4e4\uc740 \uac80\ucc30\uc774 \ub0b4\ub9b0 \ubd80\uac80 \uacb0\uc815\uc5d0 \ubc18\ub300\ud588\uc73c\ub098 \uc720\uc871\ub4e4 \uc911 \ud55c \uba85\uc774 \ubd80\uac80 \ub3d9\uc758\uc11c\uc5d0 \uc11c\uba85\ud568\uc5d0 \ub530\ub77c \uac15\ub0a8\uc131\ubaa8\ubcd1\uc6d0\uc5d0\uc11c 2008\ub144 10\uc6d4 2\uc77c \uc624\ud6c4 9\uc2dc\ubd80\ud130 \uc624\ud6c4 10\uc2dc 20\ubd84\uae4c\uc9c0 1\uc2dc\uac04 20\ubd84\uc5d0 \uac78\uccd0 \ubd80\uac80\uc774 \uc9c4\ud589\ub418\uc5c8\ub2e4. \uad6d\ub9bd\uacfc\ud559\uc218\uc0ac\uc5f0\uad6c\uc18c\ub294 \ubd80\uac80 \uacb0\uacfc \ud0c0\uc0b4\uc744 \uc99d\uba85\ud560 \ub9cc\ud55c \ub2e8\uc11c\ub97c \ubc1c\uacac\ud560 \uc218 \uc5c6\ub2e4\ub294 \ubd80\uac80 \uc18c\uacac\uc744 \ubc1d\ud614\ub2e4. 2008\ub144 10\uc6d4 3\uc77c, \uacbd\ucc30\uc740 \uacf5\uc2dd \ube0c\ub9ac\ud551\uc5d0\uc11c \uc790\uc0b4\uc774\ub77c\uace0 \ubc1c\ud45c\ud558\uc600\ub2e4.", "paragraph_answer": "2008\ub144 10\uc6d4 2\uc77c \uc624\uc804 6\uc2dc 15\ubd84, \uc11c\uc6b8\ud2b9\ubcc4\uc2dc \uc11c\ucd08\uad6c \uc7a0\uc6d0\ub3d9\uc758 \uc790\ud0dd \uc695\uc2e4\uc5d0\uc11c \uc228\uc9c4 \ucc44\ub85c \ubc1c\uacac\ub418\uc5c8\ub2e4. \uacbd\ucc30\uc740 \uc790\uc0b4\ub85c \uc7a0\uc815\uc801\uc778 \uacb0\ub860\uc744 \ub0b4\ub838\uace0, \uc9c1\uc811\uc801\uc778 \uc0ac\uc778\uc740 \uacbd\ubd80\uc555\ubc15\uc9c8\uc2dd\uc0ac \uc600\ub2e4. \uacbd\ucc30 \uc870\uc0ac\uc5d0\uc11c \ucd5c\uc9c4\uc2e4\uc740 \uacbd\ud55c \uc218\uc900\uc758 \uc6b0\uc6b8\uc99d \uc99d\uc138\uac00 \uc788\uc5c8\uace0, \uc790\ub140\uc758 \uc591\uc721 \uacfc \uc5f0\uc608\uc778\uc758 \uc704\uc0c1 \ub4f1\uacfc \uad00\ub828\ud558\uc5ec \uace0\ubbfc\ud588\ub358 \uac83\uc73c\ub85c \ubc1d\ud600\uc84c\ub2e4. \ucc98\uc74c\uc5d0 \uc720\uc871\ub4e4\uc740 \uac80\ucc30\uc774 \ub0b4\ub9b0 \ubd80\uac80 \uacb0\uc815\uc5d0 \ubc18\ub300\ud588\uc73c\ub098 \uc720\uc871\ub4e4 \uc911 \ud55c \uba85\uc774 \ubd80\uac80 \ub3d9\uc758\uc11c\uc5d0 \uc11c\uba85\ud568\uc5d0 \ub530\ub77c \uac15\ub0a8\uc131\ubaa8\ubcd1\uc6d0\uc5d0\uc11c 2008\ub144 10\uc6d4 2\uc77c \uc624\ud6c4 9\uc2dc\ubd80\ud130 \uc624\ud6c4 10\uc2dc 20\ubd84\uae4c\uc9c0 1\uc2dc\uac04 20\ubd84\uc5d0 \uac78\uccd0 \ubd80\uac80\uc774 \uc9c4\ud589\ub418\uc5c8\ub2e4. \uad6d\ub9bd\uacfc\ud559\uc218\uc0ac\uc5f0\uad6c\uc18c\ub294 \ubd80\uac80 \uacb0\uacfc \ud0c0\uc0b4\uc744 \uc99d\uba85\ud560 \ub9cc\ud55c \ub2e8\uc11c\ub97c \ubc1c\uacac\ud560 \uc218 \uc5c6\ub2e4\ub294 \ubd80\uac80 \uc18c\uacac\uc744 \ubc1d\ud614\ub2e4. 2008\ub144 10\uc6d4 3\uc77c, \uacbd\ucc30\uc740 \uacf5\uc2dd \ube0c\ub9ac\ud551\uc5d0\uc11c \uc790\uc0b4\uc774\ub77c\uace0 \ubc1c\ud45c\ud558\uc600\ub2e4.", "sentence_answer": "\uacbd\ucc30\uc740 \uc790\uc0b4\ub85c \uc7a0\uc815\uc801\uc778 \uacb0\ub860\uc744 \ub0b4\ub838\uace0, \uc9c1\uc811\uc801\uc778 \uc0ac\uc778\uc740 \uacbd\ubd80\uc555\ubc15\uc9c8\uc2dd\uc0ac \uc600\ub2e4.", "paragraph_id": "6560245-0-0", "paragraph_question": "question: \ucd5c\uc9c4\uc2e4 \uc0ac\ub9dd\uc758 \uc9c1\uc811\uc801 \uc0ac\uc778\uc740 \ubb34\uc5c7\uc778\uac00?, context: 2008\ub144 10\uc6d4 2\uc77c \uc624\uc804 6\uc2dc 15\ubd84, \uc11c\uc6b8\ud2b9\ubcc4\uc2dc \uc11c\ucd08\uad6c \uc7a0\uc6d0\ub3d9\uc758 \uc790\ud0dd \uc695\uc2e4\uc5d0\uc11c \uc228\uc9c4 \ucc44\ub85c \ubc1c\uacac\ub418\uc5c8\ub2e4. \uacbd\ucc30\uc740 \uc790\uc0b4\ub85c \uc7a0\uc815\uc801\uc778 \uacb0\ub860\uc744 \ub0b4\ub838\uace0, \uc9c1\uc811\uc801\uc778 \uc0ac\uc778\uc740 \uacbd\ubd80\uc555\ubc15\uc9c8\uc2dd\uc0ac\uc600\ub2e4. \uacbd\ucc30 \uc870\uc0ac\uc5d0\uc11c \ucd5c\uc9c4\uc2e4\uc740 \uacbd\ud55c \uc218\uc900\uc758 \uc6b0\uc6b8\uc99d \uc99d\uc138\uac00 \uc788\uc5c8\uace0, \uc790\ub140\uc758 \uc591\uc721 \uacfc \uc5f0\uc608\uc778\uc758 \uc704\uc0c1 \ub4f1\uacfc \uad00\ub828\ud558\uc5ec \uace0\ubbfc\ud588\ub358 \uac83\uc73c\ub85c \ubc1d\ud600\uc84c\ub2e4. \ucc98\uc74c\uc5d0 \uc720\uc871\ub4e4\uc740 \uac80\ucc30\uc774 \ub0b4\ub9b0 \ubd80\uac80 \uacb0\uc815\uc5d0 \ubc18\ub300\ud588\uc73c\ub098 \uc720\uc871\ub4e4 \uc911 \ud55c \uba85\uc774 \ubd80\uac80 \ub3d9\uc758\uc11c\uc5d0 \uc11c\uba85\ud568\uc5d0 \ub530\ub77c \uac15\ub0a8\uc131\ubaa8\ubcd1\uc6d0\uc5d0\uc11c 2008\ub144 10\uc6d4 2\uc77c \uc624\ud6c4 9\uc2dc\ubd80\ud130 \uc624\ud6c4 10\uc2dc 20\ubd84\uae4c\uc9c0 1\uc2dc\uac04 20\ubd84\uc5d0 \uac78\uccd0 \ubd80\uac80\uc774 \uc9c4\ud589\ub418\uc5c8\ub2e4. \uad6d\ub9bd\uacfc\ud559\uc218\uc0ac\uc5f0\uad6c\uc18c\ub294 \ubd80\uac80 \uacb0\uacfc \ud0c0\uc0b4\uc744 \uc99d\uba85\ud560 \ub9cc\ud55c \ub2e8\uc11c\ub97c \ubc1c\uacac\ud560 \uc218 \uc5c6\ub2e4\ub294 \ubd80\uac80 \uc18c\uacac\uc744 \ubc1d\ud614\ub2e4. 2008\ub144 10\uc6d4 3\uc77c, \uacbd\ucc30\uc740 \uacf5\uc2dd \ube0c\ub9ac\ud551\uc5d0\uc11c \uc790\uc0b4\uc774\ub77c\uace0 \ubc1c\ud45c\ud558\uc600\ub2e4."} {"question": "\uc774\uc11d\uae30 \uccb4\ud3ec\ub3d9\uc758\uc6d0\uc548\uc5d0 \ubc18\ub300\ud55c \uad6d\ud68c\uc758\uc6d0\uc740 \uba87 \uba85?", "paragraph": "2013\ub144 9\uc6d4 4\uc77c \uc815\ubd80\ub97c \ub300\ud45c\ud558\uc5ec \uad6d\ud68c\uc5d0\uc11c \uc774\uc11d\uae30 \uc758\uc6d0\uc5d0 \ub300\ud55c \uccb4\ud3ec\ub3d9\uc758 \uc694\uccad \uc774\uc720\ub97c \uc124\uba85\ud55c \ud669\uad50\uc548 \uc7a5\uad00\uc740 \uc774 \uc0ac\uac74 \ubc94\ud589\uc744 \uc790\uc720\ubbfc\uc8fc\uc801 \uae30\ubcf8\uc9c8\uc11c\uc5d0 \ub300\ud55c \uc815\uba74\ub3c4\uc804\uc774\uc790 \uc704\ud611\uc774\ub77c \ud558\uc600\ub2e4. \uc774\uc11d\uae30\uc5d0 \ub300\ud55c \uccb4\ud3ec\ub3d9\uc758\uc548\uc740 \uc7ac\uc11d\uc758\uc6d0 289\uba85 \uac00\uc6b4\ub370 \ucc2c\uc131 258\uba85, \ubc18\ub300 14\uba85, \uae30\uad8c 11\uba85, \ubb34\ud6a8 6\uba85 \ub4f1 \uc555\ub3c4\uc801\uc778 \ud45c\ucc28\ub85c \ud1b5\uacfc\ub418\uc5c8\uace0, \uc218\uc6d0\uc9c0\ubc29\ubc95\uc6d0\uc5d0\uc11c\ub294 \uac19\uc740 \ub2ec 5\uc77c \uc774\uc11d\uae30\uc5d0 \ub300\ud55c \uad6c\uc18d\uc601\uc7a5\uc744 \ubc1c\ubd80\ud558\uc600\uc73c\uba70 \uac80\ucc30\uc740 \uac19\uc740 \ub2ec 26\uc77c \uc774\uc11d\uae30 \ub4f1\uc744 \ub0b4\ub780\uc74c\ubaa8 \ub4f1 \ud610\uc758\ub85c \uae30\uc18c\ud55c \ud6c4 2014\ub144 2\uc6d4 3\uc77c \uacb0\uc2ec \uacf5\ud310\uc5d0\uc11c \uc9d5\uc5ed 20\ub144\uc744 \uad6c\ud615\ud558\uc600\ub2e4. 2014\ub144 2\uc6d4 17\uc77c \uc218\uc6d0\uc9c0\ubc95\uc740 \ub0b4\ub780\uc74c\ubaa8, \uc120\ub3d9 \ud610\uc758\ub97c \ubaa8\ub450 \uc720\uc8c4\ub85c \uc778\uc815\ud558\uc5ec \uc774\uc11d\uae30\uc5d0 \ub300\ud558\uc5ec \uc9d5\uc5ed 12\ub144\uacfc \uc790\uaca9\uc815\uc9c0 10\ub144\uc744 \uc120\uace0\ud558\uc600\ub2e4. \uc774\uc5d0 \uac80\ucc30\uacfc \ud53c\uace0\uc778\uce21\uc740 \ubaa8\ub450 \ud56d\uc18c\ub97c \ud558\uc600\uace0 2014\ub144 8\uc6d4 11\uc77c \uc11c\uc6b8\uace0\ubc95 \uc7ac\ud310\ubd80\ub294 \ub0b4\ub780\uc120\ub3d9\uc5d0 \ub300\ud558\uc5ec\ub294 \uc720\uc8c4\ub97c \uc778\uc815\ud558\uc600\uc73c\ub098 \ub0b4\ub780\uc74c\ubaa8\ub294 \ubb34\uc8c4\ub85c \ud310\ub2e8\ud558\uace0 \uc774\uc11d\uae30\uc5d0 \ub300\ud558\uc5ec \uc9d5\uc5ed 9\ub144\uc5d0 \uc790\uaca9\uc815\uc9c0 7\ub144\uc744 \uc120\uace0\ud558\uc600\ub2e4. \uc5ed\uc2dc \uc591\uce21\uc774 \ubaa8\ub450 \uc0c1\uace0\ud558\uc5ec \uc774 \uc0ac\uac74\uc740 \uc804\uc6d0\ud569\uc758\uccb4\uc5d0 \ud68c\ubd80\ub418\uc5c8\ub294\ub370, 2015\ub144 1\uc6d4 22\uc77c \ub300\ubc95\uc6d0\uc740 \ub0b4\ub780\uc120\ub3d9\uc740 \uc778\uc815\ub418\ub098 \ub0b4\ub780\uc74c\ubaa8\ub294 \uc778\uc815\ub418\uc9c0 \uc54a\ub294\ub2e4\ub294 \ucde8\uc9c0\uc758 \ub2e4\uc218 \uc758\uacac\uc5d0 \ub530\ub77c \uc0c1\uace0\ub97c \uae30\uac01\ud558\uc600\uace0, \uc774\uc11d\uae30\uc5d0 \ub300\ud574 \uc9d5\uc5ed 9\ub144\uc744 \uc120\uace0\ud55c \uc6d0\uc2ec\uc740 \ud655\uc815\ub418\uc5c8\ub2e4.", "answer": "14\uba85", "sentence": "\uc774\uc11d\uae30\uc5d0 \ub300\ud55c \uccb4\ud3ec\ub3d9\uc758\uc548\uc740 \uc7ac\uc11d\uc758\uc6d0 289\uba85 \uac00\uc6b4\ub370 \ucc2c\uc131 258\uba85, \ubc18\ub300 14\uba85 ,", "paragraph_sentence": "2013\ub144 9\uc6d4 4\uc77c \uc815\ubd80\ub97c \ub300\ud45c\ud558\uc5ec \uad6d\ud68c\uc5d0\uc11c \uc774\uc11d\uae30 \uc758\uc6d0\uc5d0 \ub300\ud55c \uccb4\ud3ec\ub3d9\uc758 \uc694\uccad \uc774\uc720\ub97c \uc124\uba85\ud55c \ud669\uad50\uc548 \uc7a5\uad00\uc740 \uc774 \uc0ac\uac74 \ubc94\ud589\uc744 \uc790\uc720\ubbfc\uc8fc\uc801 \uae30\ubcf8\uc9c8\uc11c\uc5d0 \ub300\ud55c \uc815\uba74\ub3c4\uc804\uc774\uc790 \uc704\ud611\uc774\ub77c \ud558\uc600\ub2e4. \uc774\uc11d\uae30\uc5d0 \ub300\ud55c \uccb4\ud3ec\ub3d9\uc758\uc548\uc740 \uc7ac\uc11d\uc758\uc6d0 289\uba85 \uac00\uc6b4\ub370 \ucc2c\uc131 258\uba85, \ubc18\ub300 14\uba85 , \uae30\uad8c 11\uba85, \ubb34\ud6a8 6\uba85 \ub4f1 \uc555\ub3c4\uc801\uc778 \ud45c\ucc28\ub85c \ud1b5\uacfc\ub418\uc5c8\uace0, \uc218\uc6d0\uc9c0\ubc29\ubc95\uc6d0\uc5d0\uc11c\ub294 \uac19\uc740 \ub2ec 5\uc77c \uc774\uc11d\uae30\uc5d0 \ub300\ud55c \uad6c\uc18d\uc601\uc7a5\uc744 \ubc1c\ubd80\ud558\uc600\uc73c\uba70 \uac80\ucc30\uc740 \uac19\uc740 \ub2ec 26\uc77c \uc774\uc11d\uae30 \ub4f1\uc744 \ub0b4\ub780\uc74c\ubaa8 \ub4f1 \ud610\uc758\ub85c \uae30\uc18c\ud55c \ud6c4 2014\ub144 2\uc6d4 3\uc77c \uacb0\uc2ec \uacf5\ud310\uc5d0\uc11c \uc9d5\uc5ed 20\ub144\uc744 \uad6c\ud615\ud558\uc600\ub2e4. 2014\ub144 2\uc6d4 17\uc77c \uc218\uc6d0\uc9c0\ubc95\uc740 \ub0b4\ub780\uc74c\ubaa8, \uc120\ub3d9 \ud610\uc758\ub97c \ubaa8\ub450 \uc720\uc8c4\ub85c \uc778\uc815\ud558\uc5ec \uc774\uc11d\uae30\uc5d0 \ub300\ud558\uc5ec \uc9d5\uc5ed 12\ub144\uacfc \uc790\uaca9\uc815\uc9c0 10\ub144\uc744 \uc120\uace0\ud558\uc600\ub2e4. \uc774\uc5d0 \uac80\ucc30\uacfc \ud53c\uace0\uc778\uce21\uc740 \ubaa8\ub450 \ud56d\uc18c\ub97c \ud558\uc600\uace0 2014\ub144 8\uc6d4 11\uc77c \uc11c\uc6b8\uace0\ubc95 \uc7ac\ud310\ubd80\ub294 \ub0b4\ub780\uc120\ub3d9\uc5d0 \ub300\ud558\uc5ec\ub294 \uc720\uc8c4\ub97c \uc778\uc815\ud558\uc600\uc73c\ub098 \ub0b4\ub780\uc74c\ubaa8\ub294 \ubb34\uc8c4\ub85c \ud310\ub2e8\ud558\uace0 \uc774\uc11d\uae30\uc5d0 \ub300\ud558\uc5ec \uc9d5\uc5ed 9\ub144\uc5d0 \uc790\uaca9\uc815\uc9c0 7\ub144\uc744 \uc120\uace0\ud558\uc600\ub2e4. \uc5ed\uc2dc \uc591\uce21\uc774 \ubaa8\ub450 \uc0c1\uace0\ud558\uc5ec \uc774 \uc0ac\uac74\uc740 \uc804\uc6d0\ud569\uc758\uccb4\uc5d0 \ud68c\ubd80\ub418\uc5c8\ub294\ub370, 2015\ub144 1\uc6d4 22\uc77c \ub300\ubc95\uc6d0\uc740 \ub0b4\ub780\uc120\ub3d9\uc740 \uc778\uc815\ub418\ub098 \ub0b4\ub780\uc74c\ubaa8\ub294 \uc778\uc815\ub418\uc9c0 \uc54a\ub294\ub2e4\ub294 \ucde8\uc9c0\uc758 \ub2e4\uc218 \uc758\uacac\uc5d0 \ub530\ub77c \uc0c1\uace0\ub97c \uae30\uac01\ud558\uc600\uace0, \uc774\uc11d\uae30\uc5d0 \ub300\ud574 \uc9d5\uc5ed 9\ub144\uc744 \uc120\uace0\ud55c \uc6d0\uc2ec\uc740 \ud655\uc815\ub418\uc5c8\ub2e4.", "paragraph_answer": "2013\ub144 9\uc6d4 4\uc77c \uc815\ubd80\ub97c \ub300\ud45c\ud558\uc5ec \uad6d\ud68c\uc5d0\uc11c \uc774\uc11d\uae30 \uc758\uc6d0\uc5d0 \ub300\ud55c \uccb4\ud3ec\ub3d9\uc758 \uc694\uccad \uc774\uc720\ub97c \uc124\uba85\ud55c \ud669\uad50\uc548 \uc7a5\uad00\uc740 \uc774 \uc0ac\uac74 \ubc94\ud589\uc744 \uc790\uc720\ubbfc\uc8fc\uc801 \uae30\ubcf8\uc9c8\uc11c\uc5d0 \ub300\ud55c \uc815\uba74\ub3c4\uc804\uc774\uc790 \uc704\ud611\uc774\ub77c \ud558\uc600\ub2e4. \uc774\uc11d\uae30\uc5d0 \ub300\ud55c \uccb4\ud3ec\ub3d9\uc758\uc548\uc740 \uc7ac\uc11d\uc758\uc6d0 289\uba85 \uac00\uc6b4\ub370 \ucc2c\uc131 258\uba85, \ubc18\ub300 14\uba85 , \uae30\uad8c 11\uba85, \ubb34\ud6a8 6\uba85 \ub4f1 \uc555\ub3c4\uc801\uc778 \ud45c\ucc28\ub85c \ud1b5\uacfc\ub418\uc5c8\uace0, \uc218\uc6d0\uc9c0\ubc29\ubc95\uc6d0\uc5d0\uc11c\ub294 \uac19\uc740 \ub2ec 5\uc77c \uc774\uc11d\uae30\uc5d0 \ub300\ud55c \uad6c\uc18d\uc601\uc7a5\uc744 \ubc1c\ubd80\ud558\uc600\uc73c\uba70 \uac80\ucc30\uc740 \uac19\uc740 \ub2ec 26\uc77c \uc774\uc11d\uae30 \ub4f1\uc744 \ub0b4\ub780\uc74c\ubaa8 \ub4f1 \ud610\uc758\ub85c \uae30\uc18c\ud55c \ud6c4 2014\ub144 2\uc6d4 3\uc77c \uacb0\uc2ec \uacf5\ud310\uc5d0\uc11c \uc9d5\uc5ed 20\ub144\uc744 \uad6c\ud615\ud558\uc600\ub2e4. 2014\ub144 2\uc6d4 17\uc77c \uc218\uc6d0\uc9c0\ubc95\uc740 \ub0b4\ub780\uc74c\ubaa8, \uc120\ub3d9 \ud610\uc758\ub97c \ubaa8\ub450 \uc720\uc8c4\ub85c \uc778\uc815\ud558\uc5ec \uc774\uc11d\uae30\uc5d0 \ub300\ud558\uc5ec \uc9d5\uc5ed 12\ub144\uacfc \uc790\uaca9\uc815\uc9c0 10\ub144\uc744 \uc120\uace0\ud558\uc600\ub2e4. \uc774\uc5d0 \uac80\ucc30\uacfc \ud53c\uace0\uc778\uce21\uc740 \ubaa8\ub450 \ud56d\uc18c\ub97c \ud558\uc600\uace0 2014\ub144 8\uc6d4 11\uc77c \uc11c\uc6b8\uace0\ubc95 \uc7ac\ud310\ubd80\ub294 \ub0b4\ub780\uc120\ub3d9\uc5d0 \ub300\ud558\uc5ec\ub294 \uc720\uc8c4\ub97c \uc778\uc815\ud558\uc600\uc73c\ub098 \ub0b4\ub780\uc74c\ubaa8\ub294 \ubb34\uc8c4\ub85c \ud310\ub2e8\ud558\uace0 \uc774\uc11d\uae30\uc5d0 \ub300\ud558\uc5ec \uc9d5\uc5ed 9\ub144\uc5d0 \uc790\uaca9\uc815\uc9c0 7\ub144\uc744 \uc120\uace0\ud558\uc600\ub2e4. \uc5ed\uc2dc \uc591\uce21\uc774 \ubaa8\ub450 \uc0c1\uace0\ud558\uc5ec \uc774 \uc0ac\uac74\uc740 \uc804\uc6d0\ud569\uc758\uccb4\uc5d0 \ud68c\ubd80\ub418\uc5c8\ub294\ub370, 2015\ub144 1\uc6d4 22\uc77c \ub300\ubc95\uc6d0\uc740 \ub0b4\ub780\uc120\ub3d9\uc740 \uc778\uc815\ub418\ub098 \ub0b4\ub780\uc74c\ubaa8\ub294 \uc778\uc815\ub418\uc9c0 \uc54a\ub294\ub2e4\ub294 \ucde8\uc9c0\uc758 \ub2e4\uc218 \uc758\uacac\uc5d0 \ub530\ub77c \uc0c1\uace0\ub97c \uae30\uac01\ud558\uc600\uace0, \uc774\uc11d\uae30\uc5d0 \ub300\ud574 \uc9d5\uc5ed 9\ub144\uc744 \uc120\uace0\ud55c \uc6d0\uc2ec\uc740 \ud655\uc815\ub418\uc5c8\ub2e4.", "sentence_answer": "\uc774\uc11d\uae30\uc5d0 \ub300\ud55c \uccb4\ud3ec\ub3d9\uc758\uc548\uc740 \uc7ac\uc11d\uc758\uc6d0 289\uba85 \uac00\uc6b4\ub370 \ucc2c\uc131 258\uba85, \ubc18\ub300 14\uba85 ,", "paragraph_id": "6366843-5-1", "paragraph_question": "question: \uc774\uc11d\uae30 \uccb4\ud3ec\ub3d9\uc758\uc6d0\uc548\uc5d0 \ubc18\ub300\ud55c \uad6d\ud68c\uc758\uc6d0\uc740 \uba87 \uba85?, context: 2013\ub144 9\uc6d4 4\uc77c \uc815\ubd80\ub97c \ub300\ud45c\ud558\uc5ec \uad6d\ud68c\uc5d0\uc11c \uc774\uc11d\uae30 \uc758\uc6d0\uc5d0 \ub300\ud55c \uccb4\ud3ec\ub3d9\uc758 \uc694\uccad \uc774\uc720\ub97c \uc124\uba85\ud55c \ud669\uad50\uc548 \uc7a5\uad00\uc740 \uc774 \uc0ac\uac74 \ubc94\ud589\uc744 \uc790\uc720\ubbfc\uc8fc\uc801 \uae30\ubcf8\uc9c8\uc11c\uc5d0 \ub300\ud55c \uc815\uba74\ub3c4\uc804\uc774\uc790 \uc704\ud611\uc774\ub77c \ud558\uc600\ub2e4. \uc774\uc11d\uae30\uc5d0 \ub300\ud55c \uccb4\ud3ec\ub3d9\uc758\uc548\uc740 \uc7ac\uc11d\uc758\uc6d0 289\uba85 \uac00\uc6b4\ub370 \ucc2c\uc131 258\uba85, \ubc18\ub300 14\uba85, \uae30\uad8c 11\uba85, \ubb34\ud6a8 6\uba85 \ub4f1 \uc555\ub3c4\uc801\uc778 \ud45c\ucc28\ub85c \ud1b5\uacfc\ub418\uc5c8\uace0, \uc218\uc6d0\uc9c0\ubc29\ubc95\uc6d0\uc5d0\uc11c\ub294 \uac19\uc740 \ub2ec 5\uc77c \uc774\uc11d\uae30\uc5d0 \ub300\ud55c \uad6c\uc18d\uc601\uc7a5\uc744 \ubc1c\ubd80\ud558\uc600\uc73c\uba70 \uac80\ucc30\uc740 \uac19\uc740 \ub2ec 26\uc77c \uc774\uc11d\uae30 \ub4f1\uc744 \ub0b4\ub780\uc74c\ubaa8 \ub4f1 \ud610\uc758\ub85c \uae30\uc18c\ud55c \ud6c4 2014\ub144 2\uc6d4 3\uc77c \uacb0\uc2ec \uacf5\ud310\uc5d0\uc11c \uc9d5\uc5ed 20\ub144\uc744 \uad6c\ud615\ud558\uc600\ub2e4. 2014\ub144 2\uc6d4 17\uc77c \uc218\uc6d0\uc9c0\ubc95\uc740 \ub0b4\ub780\uc74c\ubaa8, \uc120\ub3d9 \ud610\uc758\ub97c \ubaa8\ub450 \uc720\uc8c4\ub85c \uc778\uc815\ud558\uc5ec \uc774\uc11d\uae30\uc5d0 \ub300\ud558\uc5ec \uc9d5\uc5ed 12\ub144\uacfc \uc790\uaca9\uc815\uc9c0 10\ub144\uc744 \uc120\uace0\ud558\uc600\ub2e4. \uc774\uc5d0 \uac80\ucc30\uacfc \ud53c\uace0\uc778\uce21\uc740 \ubaa8\ub450 \ud56d\uc18c\ub97c \ud558\uc600\uace0 2014\ub144 8\uc6d4 11\uc77c \uc11c\uc6b8\uace0\ubc95 \uc7ac\ud310\ubd80\ub294 \ub0b4\ub780\uc120\ub3d9\uc5d0 \ub300\ud558\uc5ec\ub294 \uc720\uc8c4\ub97c \uc778\uc815\ud558\uc600\uc73c\ub098 \ub0b4\ub780\uc74c\ubaa8\ub294 \ubb34\uc8c4\ub85c \ud310\ub2e8\ud558\uace0 \uc774\uc11d\uae30\uc5d0 \ub300\ud558\uc5ec \uc9d5\uc5ed 9\ub144\uc5d0 \uc790\uaca9\uc815\uc9c0 7\ub144\uc744 \uc120\uace0\ud558\uc600\ub2e4. \uc5ed\uc2dc \uc591\uce21\uc774 \ubaa8\ub450 \uc0c1\uace0\ud558\uc5ec \uc774 \uc0ac\uac74\uc740 \uc804\uc6d0\ud569\uc758\uccb4\uc5d0 \ud68c\ubd80\ub418\uc5c8\ub294\ub370, 2015\ub144 1\uc6d4 22\uc77c \ub300\ubc95\uc6d0\uc740 \ub0b4\ub780\uc120\ub3d9\uc740 \uc778\uc815\ub418\ub098 \ub0b4\ub780\uc74c\ubaa8\ub294 \uc778\uc815\ub418\uc9c0 \uc54a\ub294\ub2e4\ub294 \ucde8\uc9c0\uc758 \ub2e4\uc218 \uc758\uacac\uc5d0 \ub530\ub77c \uc0c1\uace0\ub97c \uae30\uac01\ud558\uc600\uace0, \uc774\uc11d\uae30\uc5d0 \ub300\ud574 \uc9d5\uc5ed 9\ub144\uc744 \uc120\uace0\ud55c \uc6d0\uc2ec\uc740 \ud655\uc815\ub418\uc5c8\ub2e4."} {"question": "\uc21c\ucc9c\ub300\ud559\uad50\uac00 4\ub144\uc81c \ub300\ud559\uc73c\ub85c\uc758 \uac1c\ud3b8\uacfc \ud568\uaed8 \ub18d\ud654\uacfc\ud559\uacfc\ub97c \uac1c\uc124\ud55c \uac83\uc740 \uba87 \ub144\ub3c4\uc778\uac00?", "paragraph": "1935\ub144 \uc120\uac01\uc790 \uc6b0\uc11d \uae40\uc885\uc775\uc774 \ud2b9\uc9c0\ub97c \uae30\ubd80\ud558\uc5ec \uc21c\ucc9c\uacf5\ub9bd\ub18d\uc5c5\ud559\uad50\ub85c \uac1c\uad50\ud558\uc600\ub2e4. \uc21c\ucc9c\ub18d\ub9bc\uc911\ud559\uad50(1946)-\uc21c\ucc9c\ub18d\ub9bc\uace0\ub4f1\ud559\uad50(1951)-\uc21c\ucc9c\ub18d\ub9bc\uace0\ub4f1\uc804\ubb38\ud559\uad50(1965)-\uc21c\ucc9c\ub18d\ub9bc\uc804\ubb38\ud559\uad50(1973)\ub97c \uac70\uccd0 \ub300\ud559\uc758 \ud3b8\uc81c\uc778 \uc21c\ucc9c\ub18d\uc5c5 \uc804\ubb38\ub300\ud559\uc73c\ub85c \uc2b9\uaca9, \uace0\ub4f1\uad50\uc721\uae30\uad00\uc73c\ub85c \uc790\ub9ac\uc7a1\uac8c \ub418\uc5c8\ub2e4. 1982\ub144\uc5d0 4\ub144\uc81c \ub300\ud559\uc73c\ub85c\uc758 \uac1c\ud3b8\uacfc \ud568\uaed8 \ub18d\ud559\uacfc\ud559\uacfc\ub97c \uac1c\uc124\ud558\uc600\ub2e4. 1987\ub144\uc5d0 \ub4f1 10\uac1c \uc11d\uc0ac\uacfc\uc815\uc758 \ub300\ud559\uc6d0\uc774 \uc124\uce58\ub418\uc5c8\uc73c\uba70, 1991\ub144\uc5d0 5\uac1c \ub2e8\uacfc\ub300\ud559\uc744 \uac16\ucd98 \uc885\ud569\ub300\ud559\uc73c\ub85c \uc2b9\uaca9 \ub418\uc5c8\ub2e4. \uc774\ud6c4 \ubc1c\uc804\uc744 \uac70\ub4ed\ud558\uc5ec \uad50\uc721\ub300\ud559\uc6d0, \uc0b0\uc5c5\ub300\ud559\uc6d0, \uacbd\uc601\ud589\uc815\ub300\ud559\uc6d0, \uc815\ubcf4\uacfc\ud559\ub300\ud559\uc6d0\uc744 \uc124\uce58\ud558\uc600\uc73c\uba70, \ub300\ud559\uc6d0\uc5d0 \ubc15\uc0ac\uacfc\uc815\uc774 \uc124\uce58\ub418\uc5c8\ub2e4. 1994\ub144\uc5d0\ub294 \ud3ec\ud56d\uc81c\ucca0\ub85c\ubd80\ud130 \uc9c0\uc5ed \uc778\uc7ac\uc591\uc131\uc744 \uc704\ud574 100\uc5b5\uc6d0\uc744 \uae30\ubd80 \ubc1b\uc544 \uc7ac\ub2e8\ubc95\uc778 \uc21c\ucc9c\ub300\ud559\uad50\ud559\uc220\uc7a5\ud559\uc7ac\ub2e8\uc744 \uc124\ub9bd\ud558\uc600\uace0, 1998\ub144 \uc5d0\ub294 \ud3c9\uc0dd\uad50\uc721\uc6d0\uc744 \uc124\ub9bd\ud558\uc5ec \uc601\ub18d\uad50\uc721\uc6d0, \uc5b4\ud559\uc6d0\uacfc \ud568\uaed8 \uc9c0\uc5ed\ubbfc \uc7ac\uad50\uc721 \uae30\uad00\uc73c\ub85c\uc11c \uc9c0\uc5ed\ubc1c\uc804\uc5d0 \uae30\uc5ec\ud558\uace0 \uc788\uc73c\uba70, 2001\ub144\uc5d0\ub294 \ub3c4\uc11c\uad00\uacfc \uc815\ubcf4\uc804\uc0b0\uc6d0\uc744 \uac1c\uc6d0\ud558\uace0 \uc804\uc790\ub3c4\uc11c\uad00\uc744 \uad6c\ucd95\ud558\uc5ec \uc9c0\uc2dd\uc815\ubcf4\uc6d0\uc73c\ub85c\uc11c \uc5ed\ud560\uc744 \uc218\ud589\ud558\uace0 \uc788\ub2e4. \uad6d\ub9bd\ub300\ud559 \ub0b4\ubd80\ud601\uc2e0 \ud3c9\uac00\uc5d0\uc11c 44\uac1c \uad6d\ub9bd\ub300\ud559\uc911 10\uac1c \uc6b0\uc218\ub300\ud559\uc5d0 \uc120\uc815\ub418\uc5c8\ub2e4. \ub300\ud559 \uac1c\ud601\uacfc \ub0b4\ubd80\ud601\uc2e0\uc5d0 \ubc15\ucc28\ub97c \uac00\ud558\uc5ec \ub18d\ub9bc\uacc4 \ud2b9\uc131\ud654 \ub300\ud559 \uc9c0\uc815, BK 21\uc0ac\uc5c5\uc790 \uc9c0\uc815 \ub4f1 \ub9ce\uc740 \uad6d\ucc45\uc0ac\uc5c5 \uc758 \uc218\ud589\uae30\uad00\uc73c\ub85c \uc120\uc815\ub418\uc5c8\uace0,2000\ub144\uc5d0\ub294 \uad50\uc721\uc778\uc801\uc790\uc6d0\ubd80\uc758 \uc7ac\uc815\uc9c0\uc6d0\ud3c9\uac00\uc5d0\uc11c \uc804\uad6d 182\uac1c \ub300\ud559\uc911 \uc885 \ud569 17\uc704\ub97c \ucc28\uc9c0\ud558\uc600\uc73c\uba70, \uad6d\ub9bd\ub300\ud559 \ub0b4\ubd80\ud601\uc2e0 \ud3c9\uac00\uc5d0\uc11c 44\uac1c \uad6d\ub9bd\ub300\ud559\uc911 10\uac1c \uc6b0\uc218\ub300\ud559\uc5d0 \uc120\uc815\ub418\uc5c8\ub2e4.2002\ub144\uc5d0\ub294 \uad6d\ub9bd\ub300\ud559 \uc790\uccb4\ubc1c\uc804\uacc4\ud68d \ucd94\uc9c4 \uc2e4\uc801\ud3c9\uac00\uc5d0\uc11c \uad50\uc721 \ubc0f \uc5f0\uad6c\ubd80\ubb38\uc758 \uc131\uacfc \ubd84\uc57c\uc5d0\uc11c \uc6b0\uc218\ub300\ud559\uc73c\ub85c \uc120\uc815\ub418\uc5c8\ub2e4.", "answer": "1982\ub144", "sentence": "1982\ub144 \uc5d0 4\ub144\uc81c \ub300\ud559\uc73c\ub85c\uc758 \uac1c\ud3b8\uacfc \ud568\uaed8 \ub18d\ud559\uacfc\ud559\uacfc\ub97c \uac1c\uc124\ud558\uc600\ub2e4.", "paragraph_sentence": "1935\ub144 \uc120\uac01\uc790 \uc6b0\uc11d \uae40\uc885\uc775\uc774 \ud2b9\uc9c0\ub97c \uae30\ubd80\ud558\uc5ec \uc21c\ucc9c\uacf5\ub9bd\ub18d\uc5c5\ud559\uad50\ub85c \uac1c\uad50\ud558\uc600\ub2e4. \uc21c\ucc9c\ub18d\ub9bc\uc911\ud559\uad50(1946)-\uc21c\ucc9c\ub18d\ub9bc\uace0\ub4f1\ud559\uad50(1951)-\uc21c\ucc9c\ub18d\ub9bc\uace0\ub4f1\uc804\ubb38\ud559\uad50(1965)-\uc21c\ucc9c\ub18d\ub9bc\uc804\ubb38\ud559\uad50(1973)\ub97c \uac70\uccd0 \ub300\ud559\uc758 \ud3b8\uc81c\uc778 \uc21c\ucc9c\ub18d\uc5c5 \uc804\ubb38\ub300\ud559\uc73c\ub85c \uc2b9\uaca9, \uace0\ub4f1\uad50\uc721\uae30\uad00\uc73c\ub85c \uc790\ub9ac\uc7a1\uac8c \ub418\uc5c8\ub2e4. 1982\ub144 \uc5d0 4\ub144\uc81c \ub300\ud559\uc73c\ub85c\uc758 \uac1c\ud3b8\uacfc \ud568\uaed8 \ub18d\ud559\uacfc\ud559\uacfc\ub97c \uac1c\uc124\ud558\uc600\ub2e4. 1987\ub144\uc5d0 \ub4f1 10\uac1c \uc11d\uc0ac\uacfc\uc815\uc758 \ub300\ud559\uc6d0\uc774 \uc124\uce58\ub418\uc5c8\uc73c\uba70, 1991\ub144\uc5d0 5\uac1c \ub2e8\uacfc\ub300\ud559\uc744 \uac16\ucd98 \uc885\ud569\ub300\ud559\uc73c\ub85c \uc2b9\uaca9 \ub418\uc5c8\ub2e4. \uc774\ud6c4 \ubc1c\uc804\uc744 \uac70\ub4ed\ud558\uc5ec \uad50\uc721\ub300\ud559\uc6d0, \uc0b0\uc5c5\ub300\ud559\uc6d0, \uacbd\uc601\ud589\uc815\ub300\ud559\uc6d0, \uc815\ubcf4\uacfc\ud559\ub300\ud559\uc6d0\uc744 \uc124\uce58\ud558\uc600\uc73c\uba70, \ub300\ud559\uc6d0\uc5d0 \ubc15\uc0ac\uacfc\uc815\uc774 \uc124\uce58\ub418\uc5c8\ub2e4. 1994\ub144\uc5d0\ub294 \ud3ec\ud56d\uc81c\ucca0\ub85c\ubd80\ud130 \uc9c0\uc5ed \uc778\uc7ac\uc591\uc131\uc744 \uc704\ud574 100\uc5b5\uc6d0\uc744 \uae30\ubd80 \ubc1b\uc544 \uc7ac\ub2e8\ubc95\uc778 \uc21c\ucc9c\ub300\ud559\uad50\ud559\uc220\uc7a5\ud559\uc7ac\ub2e8\uc744 \uc124\ub9bd\ud558\uc600\uace0, 1998\ub144 \uc5d0\ub294 \ud3c9\uc0dd\uad50\uc721\uc6d0\uc744 \uc124\ub9bd\ud558\uc5ec \uc601\ub18d\uad50\uc721\uc6d0, \uc5b4\ud559\uc6d0\uacfc \ud568\uaed8 \uc9c0\uc5ed\ubbfc \uc7ac\uad50\uc721 \uae30\uad00\uc73c\ub85c\uc11c \uc9c0\uc5ed\ubc1c\uc804\uc5d0 \uae30\uc5ec\ud558\uace0 \uc788\uc73c\uba70, 2001\ub144\uc5d0\ub294 \ub3c4\uc11c\uad00\uacfc \uc815\ubcf4\uc804\uc0b0\uc6d0\uc744 \uac1c\uc6d0\ud558\uace0 \uc804\uc790\ub3c4\uc11c\uad00\uc744 \uad6c\ucd95\ud558\uc5ec \uc9c0\uc2dd\uc815\ubcf4\uc6d0\uc73c\ub85c\uc11c \uc5ed\ud560\uc744 \uc218\ud589\ud558\uace0 \uc788\ub2e4. \uad6d\ub9bd\ub300\ud559 \ub0b4\ubd80\ud601\uc2e0 \ud3c9\uac00\uc5d0\uc11c 44\uac1c \uad6d\ub9bd\ub300\ud559\uc911 10\uac1c \uc6b0\uc218\ub300\ud559\uc5d0 \uc120\uc815\ub418\uc5c8\ub2e4. \ub300\ud559 \uac1c\ud601\uacfc \ub0b4\ubd80\ud601\uc2e0\uc5d0 \ubc15\ucc28\ub97c \uac00\ud558\uc5ec \ub18d\ub9bc\uacc4 \ud2b9\uc131\ud654 \ub300\ud559 \uc9c0\uc815, BK 21\uc0ac\uc5c5\uc790 \uc9c0\uc815 \ub4f1 \ub9ce\uc740 \uad6d\ucc45\uc0ac\uc5c5 \uc758 \uc218\ud589\uae30\uad00\uc73c\ub85c \uc120\uc815\ub418\uc5c8\uace0,2000\ub144\uc5d0\ub294 \uad50\uc721\uc778\uc801\uc790\uc6d0\ubd80\uc758 \uc7ac\uc815\uc9c0\uc6d0\ud3c9\uac00\uc5d0\uc11c \uc804\uad6d 182\uac1c \ub300\ud559\uc911 \uc885 \ud569 17\uc704\ub97c \ucc28\uc9c0\ud558\uc600\uc73c\uba70, \uad6d\ub9bd\ub300\ud559 \ub0b4\ubd80\ud601\uc2e0 \ud3c9\uac00\uc5d0\uc11c 44\uac1c \uad6d\ub9bd\ub300\ud559\uc911 10\uac1c \uc6b0\uc218\ub300\ud559\uc5d0 \uc120\uc815\ub418\uc5c8\ub2e4.2002\ub144\uc5d0\ub294 \uad6d\ub9bd\ub300\ud559 \uc790\uccb4\ubc1c\uc804\uacc4\ud68d \ucd94\uc9c4 \uc2e4\uc801\ud3c9\uac00\uc5d0\uc11c \uad50\uc721 \ubc0f \uc5f0\uad6c\ubd80\ubb38\uc758 \uc131\uacfc \ubd84\uc57c\uc5d0\uc11c \uc6b0\uc218\ub300\ud559\uc73c\ub85c \uc120\uc815\ub418\uc5c8\ub2e4.", "paragraph_answer": "1935\ub144 \uc120\uac01\uc790 \uc6b0\uc11d \uae40\uc885\uc775\uc774 \ud2b9\uc9c0\ub97c \uae30\ubd80\ud558\uc5ec \uc21c\ucc9c\uacf5\ub9bd\ub18d\uc5c5\ud559\uad50\ub85c \uac1c\uad50\ud558\uc600\ub2e4. \uc21c\ucc9c\ub18d\ub9bc\uc911\ud559\uad50(1946)-\uc21c\ucc9c\ub18d\ub9bc\uace0\ub4f1\ud559\uad50(1951)-\uc21c\ucc9c\ub18d\ub9bc\uace0\ub4f1\uc804\ubb38\ud559\uad50(1965)-\uc21c\ucc9c\ub18d\ub9bc\uc804\ubb38\ud559\uad50(1973)\ub97c \uac70\uccd0 \ub300\ud559\uc758 \ud3b8\uc81c\uc778 \uc21c\ucc9c\ub18d\uc5c5 \uc804\ubb38\ub300\ud559\uc73c\ub85c \uc2b9\uaca9, \uace0\ub4f1\uad50\uc721\uae30\uad00\uc73c\ub85c \uc790\ub9ac\uc7a1\uac8c \ub418\uc5c8\ub2e4. 1982\ub144 \uc5d0 4\ub144\uc81c \ub300\ud559\uc73c\ub85c\uc758 \uac1c\ud3b8\uacfc \ud568\uaed8 \ub18d\ud559\uacfc\ud559\uacfc\ub97c \uac1c\uc124\ud558\uc600\ub2e4. 1987\ub144\uc5d0 \ub4f1 10\uac1c \uc11d\uc0ac\uacfc\uc815\uc758 \ub300\ud559\uc6d0\uc774 \uc124\uce58\ub418\uc5c8\uc73c\uba70, 1991\ub144\uc5d0 5\uac1c \ub2e8\uacfc\ub300\ud559\uc744 \uac16\ucd98 \uc885\ud569\ub300\ud559\uc73c\ub85c \uc2b9\uaca9 \ub418\uc5c8\ub2e4. \uc774\ud6c4 \ubc1c\uc804\uc744 \uac70\ub4ed\ud558\uc5ec \uad50\uc721\ub300\ud559\uc6d0, \uc0b0\uc5c5\ub300\ud559\uc6d0, \uacbd\uc601\ud589\uc815\ub300\ud559\uc6d0, \uc815\ubcf4\uacfc\ud559\ub300\ud559\uc6d0\uc744 \uc124\uce58\ud558\uc600\uc73c\uba70, \ub300\ud559\uc6d0\uc5d0 \ubc15\uc0ac\uacfc\uc815\uc774 \uc124\uce58\ub418\uc5c8\ub2e4. 1994\ub144\uc5d0\ub294 \ud3ec\ud56d\uc81c\ucca0\ub85c\ubd80\ud130 \uc9c0\uc5ed \uc778\uc7ac\uc591\uc131\uc744 \uc704\ud574 100\uc5b5\uc6d0\uc744 \uae30\ubd80 \ubc1b\uc544 \uc7ac\ub2e8\ubc95\uc778 \uc21c\ucc9c\ub300\ud559\uad50\ud559\uc220\uc7a5\ud559\uc7ac\ub2e8\uc744 \uc124\ub9bd\ud558\uc600\uace0, 1998\ub144 \uc5d0\ub294 \ud3c9\uc0dd\uad50\uc721\uc6d0\uc744 \uc124\ub9bd\ud558\uc5ec \uc601\ub18d\uad50\uc721\uc6d0, \uc5b4\ud559\uc6d0\uacfc \ud568\uaed8 \uc9c0\uc5ed\ubbfc \uc7ac\uad50\uc721 \uae30\uad00\uc73c\ub85c\uc11c \uc9c0\uc5ed\ubc1c\uc804\uc5d0 \uae30\uc5ec\ud558\uace0 \uc788\uc73c\uba70, 2001\ub144\uc5d0\ub294 \ub3c4\uc11c\uad00\uacfc \uc815\ubcf4\uc804\uc0b0\uc6d0\uc744 \uac1c\uc6d0\ud558\uace0 \uc804\uc790\ub3c4\uc11c\uad00\uc744 \uad6c\ucd95\ud558\uc5ec \uc9c0\uc2dd\uc815\ubcf4\uc6d0\uc73c\ub85c\uc11c \uc5ed\ud560\uc744 \uc218\ud589\ud558\uace0 \uc788\ub2e4. \uad6d\ub9bd\ub300\ud559 \ub0b4\ubd80\ud601\uc2e0 \ud3c9\uac00\uc5d0\uc11c 44\uac1c \uad6d\ub9bd\ub300\ud559\uc911 10\uac1c \uc6b0\uc218\ub300\ud559\uc5d0 \uc120\uc815\ub418\uc5c8\ub2e4. \ub300\ud559 \uac1c\ud601\uacfc \ub0b4\ubd80\ud601\uc2e0\uc5d0 \ubc15\ucc28\ub97c \uac00\ud558\uc5ec \ub18d\ub9bc\uacc4 \ud2b9\uc131\ud654 \ub300\ud559 \uc9c0\uc815, BK 21\uc0ac\uc5c5\uc790 \uc9c0\uc815 \ub4f1 \ub9ce\uc740 \uad6d\ucc45\uc0ac\uc5c5 \uc758 \uc218\ud589\uae30\uad00\uc73c\ub85c \uc120\uc815\ub418\uc5c8\uace0,2000\ub144\uc5d0\ub294 \uad50\uc721\uc778\uc801\uc790\uc6d0\ubd80\uc758 \uc7ac\uc815\uc9c0\uc6d0\ud3c9\uac00\uc5d0\uc11c \uc804\uad6d 182\uac1c \ub300\ud559\uc911 \uc885 \ud569 17\uc704\ub97c \ucc28\uc9c0\ud558\uc600\uc73c\uba70, \uad6d\ub9bd\ub300\ud559 \ub0b4\ubd80\ud601\uc2e0 \ud3c9\uac00\uc5d0\uc11c 44\uac1c \uad6d\ub9bd\ub300\ud559\uc911 10\uac1c \uc6b0\uc218\ub300\ud559\uc5d0 \uc120\uc815\ub418\uc5c8\ub2e4.2002\ub144\uc5d0\ub294 \uad6d\ub9bd\ub300\ud559 \uc790\uccb4\ubc1c\uc804\uacc4\ud68d \ucd94\uc9c4 \uc2e4\uc801\ud3c9\uac00\uc5d0\uc11c \uad50\uc721 \ubc0f \uc5f0\uad6c\ubd80\ubb38\uc758 \uc131\uacfc \ubd84\uc57c\uc5d0\uc11c \uc6b0\uc218\ub300\ud559\uc73c\ub85c \uc120\uc815\ub418\uc5c8\ub2e4.", "sentence_answer": " 1982\ub144 \uc5d0 4\ub144\uc81c \ub300\ud559\uc73c\ub85c\uc758 \uac1c\ud3b8\uacfc \ud568\uaed8 \ub18d\ud559\uacfc\ud559\uacfc\ub97c \uac1c\uc124\ud558\uc600\ub2e4.", "paragraph_id": "6519025-0-1", "paragraph_question": "question: \uc21c\ucc9c\ub300\ud559\uad50\uac00 4\ub144\uc81c \ub300\ud559\uc73c\ub85c\uc758 \uac1c\ud3b8\uacfc \ud568\uaed8 \ub18d\ud654\uacfc\ud559\uacfc\ub97c \uac1c\uc124\ud55c \uac83\uc740 \uba87 \ub144\ub3c4\uc778\uac00?, context: 1935\ub144 \uc120\uac01\uc790 \uc6b0\uc11d \uae40\uc885\uc775\uc774 \ud2b9\uc9c0\ub97c \uae30\ubd80\ud558\uc5ec \uc21c\ucc9c\uacf5\ub9bd\ub18d\uc5c5\ud559\uad50\ub85c \uac1c\uad50\ud558\uc600\ub2e4. \uc21c\ucc9c\ub18d\ub9bc\uc911\ud559\uad50(1946)-\uc21c\ucc9c\ub18d\ub9bc\uace0\ub4f1\ud559\uad50(1951)-\uc21c\ucc9c\ub18d\ub9bc\uace0\ub4f1\uc804\ubb38\ud559\uad50(1965)-\uc21c\ucc9c\ub18d\ub9bc\uc804\ubb38\ud559\uad50(1973)\ub97c \uac70\uccd0 \ub300\ud559\uc758 \ud3b8\uc81c\uc778 \uc21c\ucc9c\ub18d\uc5c5 \uc804\ubb38\ub300\ud559\uc73c\ub85c \uc2b9\uaca9, \uace0\ub4f1\uad50\uc721\uae30\uad00\uc73c\ub85c \uc790\ub9ac\uc7a1\uac8c \ub418\uc5c8\ub2e4. 1982\ub144\uc5d0 4\ub144\uc81c \ub300\ud559\uc73c\ub85c\uc758 \uac1c\ud3b8\uacfc \ud568\uaed8 \ub18d\ud559\uacfc\ud559\uacfc\ub97c \uac1c\uc124\ud558\uc600\ub2e4. 1987\ub144\uc5d0 \ub4f1 10\uac1c \uc11d\uc0ac\uacfc\uc815\uc758 \ub300\ud559\uc6d0\uc774 \uc124\uce58\ub418\uc5c8\uc73c\uba70, 1991\ub144\uc5d0 5\uac1c \ub2e8\uacfc\ub300\ud559\uc744 \uac16\ucd98 \uc885\ud569\ub300\ud559\uc73c\ub85c \uc2b9\uaca9 \ub418\uc5c8\ub2e4. \uc774\ud6c4 \ubc1c\uc804\uc744 \uac70\ub4ed\ud558\uc5ec \uad50\uc721\ub300\ud559\uc6d0, \uc0b0\uc5c5\ub300\ud559\uc6d0, \uacbd\uc601\ud589\uc815\ub300\ud559\uc6d0, \uc815\ubcf4\uacfc\ud559\ub300\ud559\uc6d0\uc744 \uc124\uce58\ud558\uc600\uc73c\uba70, \ub300\ud559\uc6d0\uc5d0 \ubc15\uc0ac\uacfc\uc815\uc774 \uc124\uce58\ub418\uc5c8\ub2e4. 1994\ub144\uc5d0\ub294 \ud3ec\ud56d\uc81c\ucca0\ub85c\ubd80\ud130 \uc9c0\uc5ed \uc778\uc7ac\uc591\uc131\uc744 \uc704\ud574 100\uc5b5\uc6d0\uc744 \uae30\ubd80 \ubc1b\uc544 \uc7ac\ub2e8\ubc95\uc778 \uc21c\ucc9c\ub300\ud559\uad50\ud559\uc220\uc7a5\ud559\uc7ac\ub2e8\uc744 \uc124\ub9bd\ud558\uc600\uace0, 1998\ub144 \uc5d0\ub294 \ud3c9\uc0dd\uad50\uc721\uc6d0\uc744 \uc124\ub9bd\ud558\uc5ec \uc601\ub18d\uad50\uc721\uc6d0, \uc5b4\ud559\uc6d0\uacfc \ud568\uaed8 \uc9c0\uc5ed\ubbfc \uc7ac\uad50\uc721 \uae30\uad00\uc73c\ub85c\uc11c \uc9c0\uc5ed\ubc1c\uc804\uc5d0 \uae30\uc5ec\ud558\uace0 \uc788\uc73c\uba70, 2001\ub144\uc5d0\ub294 \ub3c4\uc11c\uad00\uacfc \uc815\ubcf4\uc804\uc0b0\uc6d0\uc744 \uac1c\uc6d0\ud558\uace0 \uc804\uc790\ub3c4\uc11c\uad00\uc744 \uad6c\ucd95\ud558\uc5ec \uc9c0\uc2dd\uc815\ubcf4\uc6d0\uc73c\ub85c\uc11c \uc5ed\ud560\uc744 \uc218\ud589\ud558\uace0 \uc788\ub2e4. \uad6d\ub9bd\ub300\ud559 \ub0b4\ubd80\ud601\uc2e0 \ud3c9\uac00\uc5d0\uc11c 44\uac1c \uad6d\ub9bd\ub300\ud559\uc911 10\uac1c \uc6b0\uc218\ub300\ud559\uc5d0 \uc120\uc815\ub418\uc5c8\ub2e4. \ub300\ud559 \uac1c\ud601\uacfc \ub0b4\ubd80\ud601\uc2e0\uc5d0 \ubc15\ucc28\ub97c \uac00\ud558\uc5ec \ub18d\ub9bc\uacc4 \ud2b9\uc131\ud654 \ub300\ud559 \uc9c0\uc815, BK 21\uc0ac\uc5c5\uc790 \uc9c0\uc815 \ub4f1 \ub9ce\uc740 \uad6d\ucc45\uc0ac\uc5c5 \uc758 \uc218\ud589\uae30\uad00\uc73c\ub85c \uc120\uc815\ub418\uc5c8\uace0,2000\ub144\uc5d0\ub294 \uad50\uc721\uc778\uc801\uc790\uc6d0\ubd80\uc758 \uc7ac\uc815\uc9c0\uc6d0\ud3c9\uac00\uc5d0\uc11c \uc804\uad6d 182\uac1c \ub300\ud559\uc911 \uc885 \ud569 17\uc704\ub97c \ucc28\uc9c0\ud558\uc600\uc73c\uba70, \uad6d\ub9bd\ub300\ud559 \ub0b4\ubd80\ud601\uc2e0 \ud3c9\uac00\uc5d0\uc11c 44\uac1c \uad6d\ub9bd\ub300\ud559\uc911 10\uac1c \uc6b0\uc218\ub300\ud559\uc5d0 \uc120\uc815\ub418\uc5c8\ub2e4.2002\ub144\uc5d0\ub294 \uad6d\ub9bd\ub300\ud559 \uc790\uccb4\ubc1c\uc804\uacc4\ud68d \ucd94\uc9c4 \uc2e4\uc801\ud3c9\uac00\uc5d0\uc11c \uad50\uc721 \ubc0f \uc5f0\uad6c\ubd80\ubb38\uc758 \uc131\uacfc \ubd84\uc57c\uc5d0\uc11c \uc6b0\uc218\ub300\ud559\uc73c\ub85c \uc120\uc815\ub418\uc5c8\ub2e4."} {"question": "\ud788\uc5b4\ub85c\uc988\uac00 2010\ub144 7\uc6d4 \uc790\uc774\uc5b8\uce20\uc640 2:1 \uc120\uc218 \ud2b8\ub808\uc774\ud2b8\ub97c \uc9c4\ud589\ud560 \ub54c \ub0b4\uc900 \uc120\uc218\ub294?", "paragraph": "\ub125\uc13c \ud788\uc5b4\ub85c\uc988\ub294 2010 \uc2dc\uc98c \ucd08\ubc18\uc5d0 \ud2b8\ub808\uc774\ub4dc\ub85c \ub450\uc0b0 \ubca0\uc5b4\uc2a4\uc5d0\uc11c \uc774\uc801\ud574 \uc628 \uae08\ubbfc\ucca0\uc758 \ud65c\uc57d \ub4f1\uc73c\ub85c \uc2b9\uc2b9\uc7a5\uad6c\ud558\ub2e4\uac00 \uccab \uace0\ube44\ub97c \ub118\uc9c0 \ubabb\ud558\uace0 7\uc5f0\ud328\ub97c \ub2f9\ud558\ub294 \ub4f1 \ubd80\uc9c4\uc744 \uacaa\uace0 \uc788\uc5c8\ub2e4. \uc774\uc5d0\ub294 2009\ub144\uc5d0 \uc88b\uc740 \ud65c\uc57d\uc744 \ubcf4\uc5ec \uc900 \ub0b4\uc57c\uc218 \ud669\uc7ac\uade0\uc774 \uc6d0\ub798 \uac00\uc9c0\uace0 \uc788\ub358 \uc67c\ucabd \uc190\ubaa9\uc758 \ud1b5\uc99d\uc73c\ub85c \uc804\ub825\uc5d0\uc11c \ube60\uc9c4 \uac83\ub3c4 \ud55c \uac00\uc9c0 \uc774\uc720\uc600\ub2e4. \ub610\ud55c \ud669\uc7ac\uade0\uc740 \uae30\ub300 \uc774\ud558\uc758 \ud65c\uc57d\uc5d0 7\uc6d4 \ucd08\uc5d0 2\uad70\uc73c\ub85c \ub0b4\ub824\uac14\ub2e4. \ub125\uc13c\uc774 \ud558\uc704\uad8c\uc5d0 \uba38\ubb34\ub974\uba70 \uc804\ubc18\uae30\uac00 \uac70\uc758 \uc885\ub8cc\ub418\uace0 \uc788\ub358 2010\ub144 7\uc6d4 20\uc77c, \uc8fc\uc804 3\ub8e8\uc218 \uc774\ub300\ud638\uc758 \uc218\ube44 \ub2a5\ub825 \ubd80\uc871\uacfc \uc8fc\uc804 \uc720\uaca9\uc218 \ubc15\uae30\ud601\uc758 \ubd80\uc0c1\uc73c\ub85c \ub0b4\uc57c\uc9c4\uc758 \ubd88\uc548\uc744 \uacaa\uace0 \uc788\ub358 \ub86f\ub370 \uc790\uc774\uc5b8\uce20\ub294 \ub125\uc13c \ud788\uc5b4\ub85c\uc988\uc640 \ub0b4\uc57c \uac15\ud654\ub97c \uc704\ud55c \ud2b8\ub808\uc774\ub4dc\ub97c \ub2e8\ud589\ud588\ub2e4. \ud615\uc2dd\uc801\uc73c\ub85c\ub294 \uc774\uc801\ub8cc\uac00 \uc5c6\ub294 2:1 \uc120\uc218 \ud2b8\ub808\uc774\ub4dc\uc600\uc73c\uba70, \ub125\uc13c\uc740 \uc720\uaca9\uc218 \uacb8 3\ub8e8\uc218\uc778 \ud669\uc7ac\uade0\uc744 \ub0b4\uc8fc\uace0 \ub86f\ub370\ub294 \ubc31\uc5c5 \ub0b4\uc57c\uc218 \uae40\ubbfc\uc131\uacfc \uc720\ub9dd\uc8fc \ud22c\uc218 \uae40\uc218\ud654\ub97c \ub0b4\uc8fc\uba70 \ud2b8\ub808\uc774\ub4dc\uac00 \uc131\uc0ac\ub418\uc5c8\ub2e4. \uc774\uac83\uc740 \ub125\uc13c \ud788\uc5b4\ub85c\uc988\uac00 \uad6c\ub2e8 \ucc28\uc6d0\uc5d0\uc11c \ub86f\ub370 \uc790\uc774\uc5b8\uce20\uc640 \ud611\uc0c1\ud55c \uac83\uc774\uba70, \ub125\uc13c\uc758 \uac10\ub3c5\uc778 \uae40\uc2dc\uc9c4\uc740 \ud611\uc0c1\uc758 \uc9c4\ud589\uc0c1\ud669\uc744 \uc804\ud600 \ubaa8\ub974\uace0 \uc788\uc5c8\ub2e4.", "answer": "\ud669\uc7ac\uade0", "sentence": "\uc774\uc5d0\ub294 2009\ub144\uc5d0 \uc88b\uc740 \ud65c\uc57d\uc744 \ubcf4\uc5ec \uc900 \ub0b4\uc57c\uc218 \ud669\uc7ac\uade0 \uc774 \uc6d0\ub798 \uac00\uc9c0\uace0 \uc788\ub358 \uc67c\ucabd \uc190\ubaa9\uc758 \ud1b5\uc99d\uc73c\ub85c \uc804\ub825\uc5d0\uc11c \ube60\uc9c4 \uac83\ub3c4 \ud55c \uac00\uc9c0 \uc774\uc720\uc600\ub2e4.", "paragraph_sentence": "\ub125\uc13c \ud788\uc5b4\ub85c\uc988\ub294 2010 \uc2dc\uc98c \ucd08\ubc18\uc5d0 \ud2b8\ub808\uc774\ub4dc\ub85c \ub450\uc0b0 \ubca0\uc5b4\uc2a4\uc5d0\uc11c \uc774\uc801\ud574 \uc628 \uae08\ubbfc\ucca0\uc758 \ud65c\uc57d \ub4f1\uc73c\ub85c \uc2b9\uc2b9\uc7a5\uad6c\ud558\ub2e4\uac00 \uccab \uace0\ube44\ub97c \ub118\uc9c0 \ubabb\ud558\uace0 7\uc5f0\ud328\ub97c \ub2f9\ud558\ub294 \ub4f1 \ubd80\uc9c4\uc744 \uacaa\uace0 \uc788\uc5c8\ub2e4. \uc774\uc5d0\ub294 2009\ub144\uc5d0 \uc88b\uc740 \ud65c\uc57d\uc744 \ubcf4\uc5ec \uc900 \ub0b4\uc57c\uc218 \ud669\uc7ac\uade0 \uc774 \uc6d0\ub798 \uac00\uc9c0\uace0 \uc788\ub358 \uc67c\ucabd \uc190\ubaa9\uc758 \ud1b5\uc99d\uc73c\ub85c \uc804\ub825\uc5d0\uc11c \ube60\uc9c4 \uac83\ub3c4 \ud55c \uac00\uc9c0 \uc774\uc720\uc600\ub2e4. \ub610\ud55c \ud669\uc7ac\uade0\uc740 \uae30\ub300 \uc774\ud558\uc758 \ud65c\uc57d\uc5d0 7\uc6d4 \ucd08\uc5d0 2\uad70\uc73c\ub85c \ub0b4\ub824\uac14\ub2e4. \ub125\uc13c\uc774 \ud558\uc704\uad8c\uc5d0 \uba38\ubb34\ub974\uba70 \uc804\ubc18\uae30\uac00 \uac70\uc758 \uc885\ub8cc\ub418\uace0 \uc788\ub358 2010\ub144 7\uc6d4 20\uc77c, \uc8fc\uc804 3\ub8e8\uc218 \uc774\ub300\ud638\uc758 \uc218\ube44 \ub2a5\ub825 \ubd80\uc871\uacfc \uc8fc\uc804 \uc720\uaca9\uc218 \ubc15\uae30\ud601\uc758 \ubd80\uc0c1\uc73c\ub85c \ub0b4\uc57c\uc9c4\uc758 \ubd88\uc548\uc744 \uacaa\uace0 \uc788\ub358 \ub86f\ub370 \uc790\uc774\uc5b8\uce20\ub294 \ub125\uc13c \ud788\uc5b4\ub85c\uc988\uc640 \ub0b4\uc57c \uac15\ud654\ub97c \uc704\ud55c \ud2b8\ub808\uc774\ub4dc\ub97c \ub2e8\ud589\ud588\ub2e4. \ud615\uc2dd\uc801\uc73c\ub85c\ub294 \uc774\uc801\ub8cc\uac00 \uc5c6\ub294 2:1 \uc120\uc218 \ud2b8\ub808\uc774\ub4dc\uc600\uc73c\uba70, \ub125\uc13c\uc740 \uc720\uaca9\uc218 \uacb8 3\ub8e8\uc218\uc778 \ud669\uc7ac\uade0\uc744 \ub0b4\uc8fc\uace0 \ub86f\ub370\ub294 \ubc31\uc5c5 \ub0b4\uc57c\uc218 \uae40\ubbfc\uc131\uacfc \uc720\ub9dd\uc8fc \ud22c\uc218 \uae40\uc218\ud654\ub97c \ub0b4\uc8fc\uba70 \ud2b8\ub808\uc774\ub4dc\uac00 \uc131\uc0ac\ub418\uc5c8\ub2e4. \uc774\uac83\uc740 \ub125\uc13c \ud788\uc5b4\ub85c\uc988\uac00 \uad6c\ub2e8 \ucc28\uc6d0\uc5d0\uc11c \ub86f\ub370 \uc790\uc774\uc5b8\uce20\uc640 \ud611\uc0c1\ud55c \uac83\uc774\uba70, \ub125\uc13c\uc758 \uac10\ub3c5\uc778 \uae40\uc2dc\uc9c4\uc740 \ud611\uc0c1\uc758 \uc9c4\ud589\uc0c1\ud669\uc744 \uc804\ud600 \ubaa8\ub974\uace0 \uc788\uc5c8\ub2e4.", "paragraph_answer": "\ub125\uc13c \ud788\uc5b4\ub85c\uc988\ub294 2010 \uc2dc\uc98c \ucd08\ubc18\uc5d0 \ud2b8\ub808\uc774\ub4dc\ub85c \ub450\uc0b0 \ubca0\uc5b4\uc2a4\uc5d0\uc11c \uc774\uc801\ud574 \uc628 \uae08\ubbfc\ucca0\uc758 \ud65c\uc57d \ub4f1\uc73c\ub85c \uc2b9\uc2b9\uc7a5\uad6c\ud558\ub2e4\uac00 \uccab \uace0\ube44\ub97c \ub118\uc9c0 \ubabb\ud558\uace0 7\uc5f0\ud328\ub97c \ub2f9\ud558\ub294 \ub4f1 \ubd80\uc9c4\uc744 \uacaa\uace0 \uc788\uc5c8\ub2e4. \uc774\uc5d0\ub294 2009\ub144\uc5d0 \uc88b\uc740 \ud65c\uc57d\uc744 \ubcf4\uc5ec \uc900 \ub0b4\uc57c\uc218 \ud669\uc7ac\uade0 \uc774 \uc6d0\ub798 \uac00\uc9c0\uace0 \uc788\ub358 \uc67c\ucabd \uc190\ubaa9\uc758 \ud1b5\uc99d\uc73c\ub85c \uc804\ub825\uc5d0\uc11c \ube60\uc9c4 \uac83\ub3c4 \ud55c \uac00\uc9c0 \uc774\uc720\uc600\ub2e4. \ub610\ud55c \ud669\uc7ac\uade0\uc740 \uae30\ub300 \uc774\ud558\uc758 \ud65c\uc57d\uc5d0 7\uc6d4 \ucd08\uc5d0 2\uad70\uc73c\ub85c \ub0b4\ub824\uac14\ub2e4. \ub125\uc13c\uc774 \ud558\uc704\uad8c\uc5d0 \uba38\ubb34\ub974\uba70 \uc804\ubc18\uae30\uac00 \uac70\uc758 \uc885\ub8cc\ub418\uace0 \uc788\ub358 2010\ub144 7\uc6d4 20\uc77c, \uc8fc\uc804 3\ub8e8\uc218 \uc774\ub300\ud638\uc758 \uc218\ube44 \ub2a5\ub825 \ubd80\uc871\uacfc \uc8fc\uc804 \uc720\uaca9\uc218 \ubc15\uae30\ud601\uc758 \ubd80\uc0c1\uc73c\ub85c \ub0b4\uc57c\uc9c4\uc758 \ubd88\uc548\uc744 \uacaa\uace0 \uc788\ub358 \ub86f\ub370 \uc790\uc774\uc5b8\uce20\ub294 \ub125\uc13c \ud788\uc5b4\ub85c\uc988\uc640 \ub0b4\uc57c \uac15\ud654\ub97c \uc704\ud55c \ud2b8\ub808\uc774\ub4dc\ub97c \ub2e8\ud589\ud588\ub2e4. \ud615\uc2dd\uc801\uc73c\ub85c\ub294 \uc774\uc801\ub8cc\uac00 \uc5c6\ub294 2:1 \uc120\uc218 \ud2b8\ub808\uc774\ub4dc\uc600\uc73c\uba70, \ub125\uc13c\uc740 \uc720\uaca9\uc218 \uacb8 3\ub8e8\uc218\uc778 \ud669\uc7ac\uade0\uc744 \ub0b4\uc8fc\uace0 \ub86f\ub370\ub294 \ubc31\uc5c5 \ub0b4\uc57c\uc218 \uae40\ubbfc\uc131\uacfc \uc720\ub9dd\uc8fc \ud22c\uc218 \uae40\uc218\ud654\ub97c \ub0b4\uc8fc\uba70 \ud2b8\ub808\uc774\ub4dc\uac00 \uc131\uc0ac\ub418\uc5c8\ub2e4. \uc774\uac83\uc740 \ub125\uc13c \ud788\uc5b4\ub85c\uc988\uac00 \uad6c\ub2e8 \ucc28\uc6d0\uc5d0\uc11c \ub86f\ub370 \uc790\uc774\uc5b8\uce20\uc640 \ud611\uc0c1\ud55c \uac83\uc774\uba70, \ub125\uc13c\uc758 \uac10\ub3c5\uc778 \uae40\uc2dc\uc9c4\uc740 \ud611\uc0c1\uc758 \uc9c4\ud589\uc0c1\ud669\uc744 \uc804\ud600 \ubaa8\ub974\uace0 \uc788\uc5c8\ub2e4.", "sentence_answer": "\uc774\uc5d0\ub294 2009\ub144\uc5d0 \uc88b\uc740 \ud65c\uc57d\uc744 \ubcf4\uc5ec \uc900 \ub0b4\uc57c\uc218 \ud669\uc7ac\uade0 \uc774 \uc6d0\ub798 \uac00\uc9c0\uace0 \uc788\ub358 \uc67c\ucabd \uc190\ubaa9\uc758 \ud1b5\uc99d\uc73c\ub85c \uc804\ub825\uc5d0\uc11c \ube60\uc9c4 \uac83\ub3c4 \ud55c \uac00\uc9c0 \uc774\uc720\uc600\ub2e4.", "paragraph_id": "6488904-9-1", "paragraph_question": "question: \ud788\uc5b4\ub85c\uc988\uac00 2010\ub144 7\uc6d4 \uc790\uc774\uc5b8\uce20\uc640 2:1 \uc120\uc218 \ud2b8\ub808\uc774\ud2b8\ub97c \uc9c4\ud589\ud560 \ub54c \ub0b4\uc900 \uc120\uc218\ub294?, context: \ub125\uc13c \ud788\uc5b4\ub85c\uc988\ub294 2010 \uc2dc\uc98c \ucd08\ubc18\uc5d0 \ud2b8\ub808\uc774\ub4dc\ub85c \ub450\uc0b0 \ubca0\uc5b4\uc2a4\uc5d0\uc11c \uc774\uc801\ud574 \uc628 \uae08\ubbfc\ucca0\uc758 \ud65c\uc57d \ub4f1\uc73c\ub85c \uc2b9\uc2b9\uc7a5\uad6c\ud558\ub2e4\uac00 \uccab \uace0\ube44\ub97c \ub118\uc9c0 \ubabb\ud558\uace0 7\uc5f0\ud328\ub97c \ub2f9\ud558\ub294 \ub4f1 \ubd80\uc9c4\uc744 \uacaa\uace0 \uc788\uc5c8\ub2e4. \uc774\uc5d0\ub294 2009\ub144\uc5d0 \uc88b\uc740 \ud65c\uc57d\uc744 \ubcf4\uc5ec \uc900 \ub0b4\uc57c\uc218 \ud669\uc7ac\uade0\uc774 \uc6d0\ub798 \uac00\uc9c0\uace0 \uc788\ub358 \uc67c\ucabd \uc190\ubaa9\uc758 \ud1b5\uc99d\uc73c\ub85c \uc804\ub825\uc5d0\uc11c \ube60\uc9c4 \uac83\ub3c4 \ud55c \uac00\uc9c0 \uc774\uc720\uc600\ub2e4. \ub610\ud55c \ud669\uc7ac\uade0\uc740 \uae30\ub300 \uc774\ud558\uc758 \ud65c\uc57d\uc5d0 7\uc6d4 \ucd08\uc5d0 2\uad70\uc73c\ub85c \ub0b4\ub824\uac14\ub2e4. \ub125\uc13c\uc774 \ud558\uc704\uad8c\uc5d0 \uba38\ubb34\ub974\uba70 \uc804\ubc18\uae30\uac00 \uac70\uc758 \uc885\ub8cc\ub418\uace0 \uc788\ub358 2010\ub144 7\uc6d4 20\uc77c, \uc8fc\uc804 3\ub8e8\uc218 \uc774\ub300\ud638\uc758 \uc218\ube44 \ub2a5\ub825 \ubd80\uc871\uacfc \uc8fc\uc804 \uc720\uaca9\uc218 \ubc15\uae30\ud601\uc758 \ubd80\uc0c1\uc73c\ub85c \ub0b4\uc57c\uc9c4\uc758 \ubd88\uc548\uc744 \uacaa\uace0 \uc788\ub358 \ub86f\ub370 \uc790\uc774\uc5b8\uce20\ub294 \ub125\uc13c \ud788\uc5b4\ub85c\uc988\uc640 \ub0b4\uc57c \uac15\ud654\ub97c \uc704\ud55c \ud2b8\ub808\uc774\ub4dc\ub97c \ub2e8\ud589\ud588\ub2e4. \ud615\uc2dd\uc801\uc73c\ub85c\ub294 \uc774\uc801\ub8cc\uac00 \uc5c6\ub294 2:1 \uc120\uc218 \ud2b8\ub808\uc774\ub4dc\uc600\uc73c\uba70, \ub125\uc13c\uc740 \uc720\uaca9\uc218 \uacb8 3\ub8e8\uc218\uc778 \ud669\uc7ac\uade0\uc744 \ub0b4\uc8fc\uace0 \ub86f\ub370\ub294 \ubc31\uc5c5 \ub0b4\uc57c\uc218 \uae40\ubbfc\uc131\uacfc \uc720\ub9dd\uc8fc \ud22c\uc218 \uae40\uc218\ud654\ub97c \ub0b4\uc8fc\uba70 \ud2b8\ub808\uc774\ub4dc\uac00 \uc131\uc0ac\ub418\uc5c8\ub2e4. \uc774\uac83\uc740 \ub125\uc13c \ud788\uc5b4\ub85c\uc988\uac00 \uad6c\ub2e8 \ucc28\uc6d0\uc5d0\uc11c \ub86f\ub370 \uc790\uc774\uc5b8\uce20\uc640 \ud611\uc0c1\ud55c \uac83\uc774\uba70, \ub125\uc13c\uc758 \uac10\ub3c5\uc778 \uae40\uc2dc\uc9c4\uc740 \ud611\uc0c1\uc758 \uc9c4\ud589\uc0c1\ud669\uc744 \uc804\ud600 \ubaa8\ub974\uace0 \uc788\uc5c8\ub2e4."} {"question": "\ub9cc\uc288\ud0c0\uc778\uc740 \uc5b4\ub514\uc758 \ucd1d\ud1b5\ubcf8\ubd80\ub85c \uac00\uc11c \ud788\ud2c0\ub7ec\ub97c \uc124\ub4dd\ud588\ub294\uac00?", "paragraph": "1\uc6d4, \ub9cc\uc288\ud0c0\uc778\uc740 \uc18c\ub828\uad70\uc758 \uacf5\uc138\uc5d0 \ubc00\ub824 \ub354\uc6b1 \uc11c\ucabd\uc73c\ub85c \ud6c4\ud1f4\ud588\ub2e4. \ud788\ud2c0\ub7ec\uc758 \ud5c8\ub77d\ub3c4 \uc5c6\uc774 \ub9cc\uc288\ud0c0\uc778\uc740 \ub0a8\ubd80 \uc9d1\ub2e8\uad70 \uc81c11\uad70\ub2e8\uacfc \uc81c42\uad70\ub2e8(6\uac1c \uc0ac\ub2e8 56,000 \uc5ec\uba85)\uc5d0\uac8c 1944\ub144 2\uc6d4 16\uc77c\uc5d0\uc11c 17\uc77c\ub85c \ub0a8\uc5b4\uac00\ub294 \uc57c\uc74c\uc744 \ud2c8\ud0c0 \ucf54\ub974\uc28c \ub3cc\ucd9c\ubd80\ub97c \ud3ec\uae30\ud558\uace0 \ud0c8\ucd9c\ud558\ub77c\ub294 \uba85\ub839\uc744 \ub0b4\ub838\ub2e4. \uc18c\ub828\uad70\uc740 3\uc6d4 \ucd08\uae4c\uc9c0 \ub3c5\uc77c\uad70\uc744 \uac15 \ub108\uba38\ub85c \ubc00\uc5b4 \ubc84\ub838\ub2e4. 3\uc6d4 19\uc77c, \ud604 \uc2dc\uc810 \uc774\ud6c4 \uc804\uc6d0 \uc790\uae30 \uc790\ub9ac\ub97c \uc0ac\uc218\ud558\ub77c\ub294 \ud788\ud2c0\ub7ec\uc758 \uba85\ub839\uc774 \ub0b4\ub824\uc84c\uace0, \uc774 \ub54c\ubb38\uc5d0 \ub9cc\uc288\ud0c0\uc778\uc758 \uc81c1\uae30\uac11\uad70\uc740 \ud788\ud2c0\ub7ec\uc758 \ud0c8\ucd9c \uba85\ub839\uc774 \uc81c\ub54c \ub0b4\ub824\uc624\uc9c0 \uc54a\uc544 \ud3ec\uc704\ub2f9\ud558\uace0 \ub9d0\uc558\ub2e4(3\uc6d4 21\uc77c). \ub9cc\uc288\ud0c0\uc778\uc740 \ub9ac\ube44\uc6b0\uc758 \ucd1d\ud1b5\ubcf8\ubd80\ub85c \ub0a0\uc544\uac00\uc11c \ud788\ud2c0\ub7ec\ub97c \uc124\ub4dd\ud588\ub2e4. \ud788\ud2c0\ub7ec\ub294 \uaca8\uc6b0 \ub3d9\uc758\ud588\uc9c0\ub9cc, 3\uc6d4 30\uc77c \ub9cc\uc288\ud0c0\uc778\uc744 \uc790\ub974\uace0 \uadf8\ub97c \ubaa8\ub378\uacfc \uad50\ub300\uc2dc\ud0a8\ub2e4.", "answer": "\ub9ac\ube44\uc6b0", "sentence": "\ub9cc\uc288\ud0c0\uc778\uc740 \ub9ac\ube44\uc6b0 \uc758 \ucd1d\ud1b5\ubcf8\ubd80\ub85c \ub0a0\uc544\uac00\uc11c \ud788\ud2c0\ub7ec\ub97c \uc124\ub4dd\ud588\ub2e4.", "paragraph_sentence": "1\uc6d4, \ub9cc\uc288\ud0c0\uc778\uc740 \uc18c\ub828\uad70\uc758 \uacf5\uc138\uc5d0 \ubc00\ub824 \ub354\uc6b1 \uc11c\ucabd\uc73c\ub85c \ud6c4\ud1f4\ud588\ub2e4. \ud788\ud2c0\ub7ec\uc758 \ud5c8\ub77d\ub3c4 \uc5c6\uc774 \ub9cc\uc288\ud0c0\uc778\uc740 \ub0a8\ubd80 \uc9d1\ub2e8\uad70 \uc81c11\uad70\ub2e8\uacfc \uc81c42\uad70\ub2e8(6\uac1c \uc0ac\ub2e8 56,000 \uc5ec\uba85)\uc5d0\uac8c 1944\ub144 2\uc6d4 16\uc77c\uc5d0\uc11c 17\uc77c\ub85c \ub0a8\uc5b4\uac00\ub294 \uc57c\uc74c\uc744 \ud2c8\ud0c0 \ucf54\ub974\uc28c \ub3cc\ucd9c\ubd80\ub97c \ud3ec\uae30\ud558\uace0 \ud0c8\ucd9c\ud558\ub77c\ub294 \uba85\ub839\uc744 \ub0b4\ub838\ub2e4. \uc18c\ub828\uad70\uc740 3\uc6d4 \ucd08\uae4c\uc9c0 \ub3c5\uc77c\uad70\uc744 \uac15 \ub108\uba38\ub85c \ubc00\uc5b4 \ubc84\ub838\ub2e4. 3\uc6d4 19\uc77c, \ud604 \uc2dc\uc810 \uc774\ud6c4 \uc804\uc6d0 \uc790\uae30 \uc790\ub9ac\ub97c \uc0ac\uc218\ud558\ub77c\ub294 \ud788\ud2c0\ub7ec\uc758 \uba85\ub839\uc774 \ub0b4\ub824\uc84c\uace0, \uc774 \ub54c\ubb38\uc5d0 \ub9cc\uc288\ud0c0\uc778\uc758 \uc81c1\uae30\uac11\uad70\uc740 \ud788\ud2c0\ub7ec\uc758 \ud0c8\ucd9c \uba85\ub839\uc774 \uc81c\ub54c \ub0b4\ub824\uc624\uc9c0 \uc54a\uc544 \ud3ec\uc704\ub2f9\ud558\uace0 \ub9d0\uc558\ub2e4(3\uc6d4 21\uc77c). \ub9cc\uc288\ud0c0\uc778\uc740 \ub9ac\ube44\uc6b0 \uc758 \ucd1d\ud1b5\ubcf8\ubd80\ub85c \ub0a0\uc544\uac00\uc11c \ud788\ud2c0\ub7ec\ub97c \uc124\ub4dd\ud588\ub2e4. \ud788\ud2c0\ub7ec\ub294 \uaca8\uc6b0 \ub3d9\uc758\ud588\uc9c0\ub9cc, 3\uc6d4 30\uc77c \ub9cc\uc288\ud0c0\uc778\uc744 \uc790\ub974\uace0 \uadf8\ub97c \ubaa8\ub378\uacfc \uad50\ub300\uc2dc\ud0a8\ub2e4.", "paragraph_answer": "1\uc6d4, \ub9cc\uc288\ud0c0\uc778\uc740 \uc18c\ub828\uad70\uc758 \uacf5\uc138\uc5d0 \ubc00\ub824 \ub354\uc6b1 \uc11c\ucabd\uc73c\ub85c \ud6c4\ud1f4\ud588\ub2e4. \ud788\ud2c0\ub7ec\uc758 \ud5c8\ub77d\ub3c4 \uc5c6\uc774 \ub9cc\uc288\ud0c0\uc778\uc740 \ub0a8\ubd80 \uc9d1\ub2e8\uad70 \uc81c11\uad70\ub2e8\uacfc \uc81c42\uad70\ub2e8(6\uac1c \uc0ac\ub2e8 56,000 \uc5ec\uba85)\uc5d0\uac8c 1944\ub144 2\uc6d4 16\uc77c\uc5d0\uc11c 17\uc77c\ub85c \ub0a8\uc5b4\uac00\ub294 \uc57c\uc74c\uc744 \ud2c8\ud0c0 \ucf54\ub974\uc28c \ub3cc\ucd9c\ubd80\ub97c \ud3ec\uae30\ud558\uace0 \ud0c8\ucd9c\ud558\ub77c\ub294 \uba85\ub839\uc744 \ub0b4\ub838\ub2e4. \uc18c\ub828\uad70\uc740 3\uc6d4 \ucd08\uae4c\uc9c0 \ub3c5\uc77c\uad70\uc744 \uac15 \ub108\uba38\ub85c \ubc00\uc5b4 \ubc84\ub838\ub2e4. 3\uc6d4 19\uc77c, \ud604 \uc2dc\uc810 \uc774\ud6c4 \uc804\uc6d0 \uc790\uae30 \uc790\ub9ac\ub97c \uc0ac\uc218\ud558\ub77c\ub294 \ud788\ud2c0\ub7ec\uc758 \uba85\ub839\uc774 \ub0b4\ub824\uc84c\uace0, \uc774 \ub54c\ubb38\uc5d0 \ub9cc\uc288\ud0c0\uc778\uc758 \uc81c1\uae30\uac11\uad70\uc740 \ud788\ud2c0\ub7ec\uc758 \ud0c8\ucd9c \uba85\ub839\uc774 \uc81c\ub54c \ub0b4\ub824\uc624\uc9c0 \uc54a\uc544 \ud3ec\uc704\ub2f9\ud558\uace0 \ub9d0\uc558\ub2e4(3\uc6d4 21\uc77c). \ub9cc\uc288\ud0c0\uc778\uc740 \ub9ac\ube44\uc6b0 \uc758 \ucd1d\ud1b5\ubcf8\ubd80\ub85c \ub0a0\uc544\uac00\uc11c \ud788\ud2c0\ub7ec\ub97c \uc124\ub4dd\ud588\ub2e4. \ud788\ud2c0\ub7ec\ub294 \uaca8\uc6b0 \ub3d9\uc758\ud588\uc9c0\ub9cc, 3\uc6d4 30\uc77c \ub9cc\uc288\ud0c0\uc778\uc744 \uc790\ub974\uace0 \uadf8\ub97c \ubaa8\ub378\uacfc \uad50\ub300\uc2dc\ud0a8\ub2e4.", "sentence_answer": "\ub9cc\uc288\ud0c0\uc778\uc740 \ub9ac\ube44\uc6b0 \uc758 \ucd1d\ud1b5\ubcf8\ubd80\ub85c \ub0a0\uc544\uac00\uc11c \ud788\ud2c0\ub7ec\ub97c \uc124\ub4dd\ud588\ub2e4.", "paragraph_id": "6586570-29-1", "paragraph_question": "question: \ub9cc\uc288\ud0c0\uc778\uc740 \uc5b4\ub514\uc758 \ucd1d\ud1b5\ubcf8\ubd80\ub85c \uac00\uc11c \ud788\ud2c0\ub7ec\ub97c \uc124\ub4dd\ud588\ub294\uac00?, context: 1\uc6d4, \ub9cc\uc288\ud0c0\uc778\uc740 \uc18c\ub828\uad70\uc758 \uacf5\uc138\uc5d0 \ubc00\ub824 \ub354\uc6b1 \uc11c\ucabd\uc73c\ub85c \ud6c4\ud1f4\ud588\ub2e4. \ud788\ud2c0\ub7ec\uc758 \ud5c8\ub77d\ub3c4 \uc5c6\uc774 \ub9cc\uc288\ud0c0\uc778\uc740 \ub0a8\ubd80 \uc9d1\ub2e8\uad70 \uc81c11\uad70\ub2e8\uacfc \uc81c42\uad70\ub2e8(6\uac1c \uc0ac\ub2e8 56,000 \uc5ec\uba85)\uc5d0\uac8c 1944\ub144 2\uc6d4 16\uc77c\uc5d0\uc11c 17\uc77c\ub85c \ub0a8\uc5b4\uac00\ub294 \uc57c\uc74c\uc744 \ud2c8\ud0c0 \ucf54\ub974\uc28c \ub3cc\ucd9c\ubd80\ub97c \ud3ec\uae30\ud558\uace0 \ud0c8\ucd9c\ud558\ub77c\ub294 \uba85\ub839\uc744 \ub0b4\ub838\ub2e4. \uc18c\ub828\uad70\uc740 3\uc6d4 \ucd08\uae4c\uc9c0 \ub3c5\uc77c\uad70\uc744 \uac15 \ub108\uba38\ub85c \ubc00\uc5b4 \ubc84\ub838\ub2e4. 3\uc6d4 19\uc77c, \ud604 \uc2dc\uc810 \uc774\ud6c4 \uc804\uc6d0 \uc790\uae30 \uc790\ub9ac\ub97c \uc0ac\uc218\ud558\ub77c\ub294 \ud788\ud2c0\ub7ec\uc758 \uba85\ub839\uc774 \ub0b4\ub824\uc84c\uace0, \uc774 \ub54c\ubb38\uc5d0 \ub9cc\uc288\ud0c0\uc778\uc758 \uc81c1\uae30\uac11\uad70\uc740 \ud788\ud2c0\ub7ec\uc758 \ud0c8\ucd9c \uba85\ub839\uc774 \uc81c\ub54c \ub0b4\ub824\uc624\uc9c0 \uc54a\uc544 \ud3ec\uc704\ub2f9\ud558\uace0 \ub9d0\uc558\ub2e4(3\uc6d4 21\uc77c). \ub9cc\uc288\ud0c0\uc778\uc740 \ub9ac\ube44\uc6b0\uc758 \ucd1d\ud1b5\ubcf8\ubd80\ub85c \ub0a0\uc544\uac00\uc11c \ud788\ud2c0\ub7ec\ub97c \uc124\ub4dd\ud588\ub2e4. \ud788\ud2c0\ub7ec\ub294 \uaca8\uc6b0 \ub3d9\uc758\ud588\uc9c0\ub9cc, 3\uc6d4 30\uc77c \ub9cc\uc288\ud0c0\uc778\uc744 \uc790\ub974\uace0 \uadf8\ub97c \ubaa8\ub378\uacfc \uad50\ub300\uc2dc\ud0a8\ub2e4."} {"question": "\uc804\ubd81 \ud604\ub300 \ubaa8\ud130\uc2a4\uac00 2008\ub144 K\ub9ac\uadf8\uc5d0\uc11c \ucd08\ubc18 \ubd80\uc9c4\uc744 \ub51b\uace0 \uc2dc\uc98c\uc744 \ucd5c\uc885 \ub9c8\uac10\ud55c \uc21c\uc704\ub294?", "paragraph": "2005\ub144 7\uc6d4 3\uc77c, \uc870\uc724\ud658\uc758 \ud6c4\uc784\uc73c\ub85c \uc804\ubd81 \ud604\ub300 \ubaa8\ud130\uc2a4\uc758 \uac10\ub3c5\uc73c\ub85c \ubd80\uc784\ud558\uc600\ub2e4. \ub2f9\uc2dc \uc911\ud558\uc704\uad8c\uc744 \ub9f4\ub3cc\uace0 \uc788\uc5c8\ub358 \uc804\ubd81\uc758 \uc9c0\ud718\ubd09\uc744 \uc7a1\uc740 \ucd5c\uac15\ud76c \uac10\ub3c5\uc740 \ubd80\uc784 \uccab \ud574 \uc815\uaddc\ub9ac\uadf8\uc5d0\uc11c\ub294 \uc88b\uc740 \uc131\uc801\uc744 \ub0b4\uc9c0 \ubabb\ud558\uc600\uc73c\ub098 FA\ucef5\uc5d0\uc11c \uc6b0\uc2b9\uc744 \ucc28\uc9c0\ud574 2006\ub144 ACL\uc5d0 \ucd9c\uc804\ud558\uac8c \ub418\uc5c8\ub2e4. AFC \ucc54\ud53c\uc5b8\uc2a4\ub9ac\uadf8\uc5d0\uc11c \uc5fc\uae30\ud6c8, \uae40\ud615\ubc94, \uc81c \uce74\ub97c\ub8e8\uc2a4 \ub4f1\uc744 \uc55e\uc138\uc6cc \uc870\ubcc4 \uc608\uc120\uacfc \ubcf8\uc120 \ud1a0\ub108\uba3c\ud2b8\uc5d0\uc11c \uc5f0\uc774\uc740 \uc5ed\uc804\uc2b9\uc744 \uc77c\uad6c\uc5b4 \ub0b4\uba70 \uadf9\uc801\uc778 \uc6b0\uc2b9\uc744 \ucc28\uc9c0\ud558\uc600\ub2e4. 2008 K\ub9ac\uadf8\ub97c \uc55e\ub450\uace0 \ub300\ub300\uc801\uc778 \ub9ac\ube4c\ub529\uc744 \uc2e4\uc2dc\ud558\uc600\ub294\ub370, \uc2dc\uc98c \ucd08\ubc18 \ubd80\uc9c4\ud55c \ubaa8\uc2b5\uc744 \ubcf4\uc5ec \uc9c8\ud0c0\ub97c \ubc1b\uae30\ub3c4 \ud558\uc600\ub2e4. \ud558\uc9c0\ub9cc \ub9ac\uadf8 \ud6c4\ubc18\uae30\uc5d0 \ub4e4\uc5b4\uc11c\uba70 \uc774\ub0b4 \uacbd\uae30\ub825\uc744 \ub04c\uc5b4\uc62c\ub824 \uc815\uaddc \ub9ac\uadf8 \ucd5c\uc885 4\uc704\ub85c \uc2dc\uc98c\uc744 \ub9c8\uac10\ud558\uc600\ub2e4. \ud2b9\uc720\uc758 \uce5c\ud654\ub825\uc73c\ub85c \uc778\ud574 \uc804\ubd81 \ud604\ub300 \ubaa8\ud130\uc2a4\uc758 \ud074\ub7fd \ud558\uc6b0\uc2a4 \ubc0f \ud6c8\ub828\uc7a5\uc774 \uc18c\uc7ac\ud558\uace0 \uc788\ub294 \uc804\ub77c\ubd81\ub3c4 \uc644\uc8fc\uad70 \ubd09\ub3d9\uc74d\uc758 \uc9c0\uba85\uc744 \ubcf8\ub530\uc11c \ubd09\ub3d9 \uc774\uc7a5\uc774\ub77c\ub294 \ubcc4\uba85\uc744 \uc5bb\uc5c8\ub2e4.", "answer": "4\uc704", "sentence": "\ud558\uc9c0\ub9cc \ub9ac\uadf8 \ud6c4\ubc18\uae30\uc5d0 \ub4e4\uc5b4\uc11c\uba70 \uc774\ub0b4 \uacbd\uae30\ub825\uc744 \ub04c\uc5b4\uc62c\ub824 \uc815\uaddc \ub9ac\uadf8 \ucd5c\uc885 4\uc704 \ub85c \uc2dc\uc98c\uc744 \ub9c8\uac10\ud558\uc600\ub2e4.", "paragraph_sentence": "2005\ub144 7\uc6d4 3\uc77c, \uc870\uc724\ud658\uc758 \ud6c4\uc784\uc73c\ub85c \uc804\ubd81 \ud604\ub300 \ubaa8\ud130\uc2a4\uc758 \uac10\ub3c5\uc73c\ub85c \ubd80\uc784\ud558\uc600\ub2e4. \ub2f9\uc2dc \uc911\ud558\uc704\uad8c\uc744 \ub9f4\ub3cc\uace0 \uc788\uc5c8\ub358 \uc804\ubd81\uc758 \uc9c0\ud718\ubd09\uc744 \uc7a1\uc740 \ucd5c\uac15\ud76c \uac10\ub3c5\uc740 \ubd80\uc784 \uccab \ud574 \uc815\uaddc\ub9ac\uadf8\uc5d0\uc11c\ub294 \uc88b\uc740 \uc131\uc801\uc744 \ub0b4\uc9c0 \ubabb\ud558\uc600\uc73c\ub098 FA\ucef5\uc5d0\uc11c \uc6b0\uc2b9\uc744 \ucc28\uc9c0\ud574 2006\ub144 ACL\uc5d0 \ucd9c\uc804\ud558\uac8c \ub418\uc5c8\ub2e4. AFC \ucc54\ud53c\uc5b8\uc2a4\ub9ac\uadf8\uc5d0\uc11c \uc5fc\uae30\ud6c8, \uae40\ud615\ubc94, \uc81c \uce74\ub97c\ub8e8\uc2a4 \ub4f1\uc744 \uc55e\uc138\uc6cc \uc870\ubcc4 \uc608\uc120\uacfc \ubcf8\uc120 \ud1a0\ub108\uba3c\ud2b8\uc5d0\uc11c \uc5f0\uc774\uc740 \uc5ed\uc804\uc2b9\uc744 \uc77c\uad6c\uc5b4 \ub0b4\uba70 \uadf9\uc801\uc778 \uc6b0\uc2b9\uc744 \ucc28\uc9c0\ud558\uc600\ub2e4. 2008 K\ub9ac\uadf8\ub97c \uc55e\ub450\uace0 \ub300\ub300\uc801\uc778 \ub9ac\ube4c\ub529\uc744 \uc2e4\uc2dc\ud558\uc600\ub294\ub370, \uc2dc\uc98c \ucd08\ubc18 \ubd80\uc9c4\ud55c \ubaa8\uc2b5\uc744 \ubcf4\uc5ec \uc9c8\ud0c0\ub97c \ubc1b\uae30\ub3c4 \ud558\uc600\ub2e4. \ud558\uc9c0\ub9cc \ub9ac\uadf8 \ud6c4\ubc18\uae30\uc5d0 \ub4e4\uc5b4\uc11c\uba70 \uc774\ub0b4 \uacbd\uae30\ub825\uc744 \ub04c\uc5b4\uc62c\ub824 \uc815\uaddc \ub9ac\uadf8 \ucd5c\uc885 4\uc704 \ub85c \uc2dc\uc98c\uc744 \ub9c8\uac10\ud558\uc600\ub2e4. \ud2b9\uc720\uc758 \uce5c\ud654\ub825\uc73c\ub85c \uc778\ud574 \uc804\ubd81 \ud604\ub300 \ubaa8\ud130\uc2a4\uc758 \ud074\ub7fd \ud558\uc6b0\uc2a4 \ubc0f \ud6c8\ub828\uc7a5\uc774 \uc18c\uc7ac\ud558\uace0 \uc788\ub294 \uc804\ub77c\ubd81\ub3c4 \uc644\uc8fc\uad70 \ubd09\ub3d9\uc74d\uc758 \uc9c0\uba85\uc744 \ubcf8\ub530\uc11c \ubd09\ub3d9 \uc774\uc7a5\uc774\ub77c\ub294 \ubcc4\uba85\uc744 \uc5bb\uc5c8\ub2e4.", "paragraph_answer": "2005\ub144 7\uc6d4 3\uc77c, \uc870\uc724\ud658\uc758 \ud6c4\uc784\uc73c\ub85c \uc804\ubd81 \ud604\ub300 \ubaa8\ud130\uc2a4\uc758 \uac10\ub3c5\uc73c\ub85c \ubd80\uc784\ud558\uc600\ub2e4. \ub2f9\uc2dc \uc911\ud558\uc704\uad8c\uc744 \ub9f4\ub3cc\uace0 \uc788\uc5c8\ub358 \uc804\ubd81\uc758 \uc9c0\ud718\ubd09\uc744 \uc7a1\uc740 \ucd5c\uac15\ud76c \uac10\ub3c5\uc740 \ubd80\uc784 \uccab \ud574 \uc815\uaddc\ub9ac\uadf8\uc5d0\uc11c\ub294 \uc88b\uc740 \uc131\uc801\uc744 \ub0b4\uc9c0 \ubabb\ud558\uc600\uc73c\ub098 FA\ucef5\uc5d0\uc11c \uc6b0\uc2b9\uc744 \ucc28\uc9c0\ud574 2006\ub144 ACL\uc5d0 \ucd9c\uc804\ud558\uac8c \ub418\uc5c8\ub2e4. AFC \ucc54\ud53c\uc5b8\uc2a4\ub9ac\uadf8\uc5d0\uc11c \uc5fc\uae30\ud6c8, \uae40\ud615\ubc94, \uc81c \uce74\ub97c\ub8e8\uc2a4 \ub4f1\uc744 \uc55e\uc138\uc6cc \uc870\ubcc4 \uc608\uc120\uacfc \ubcf8\uc120 \ud1a0\ub108\uba3c\ud2b8\uc5d0\uc11c \uc5f0\uc774\uc740 \uc5ed\uc804\uc2b9\uc744 \uc77c\uad6c\uc5b4 \ub0b4\uba70 \uadf9\uc801\uc778 \uc6b0\uc2b9\uc744 \ucc28\uc9c0\ud558\uc600\ub2e4. 2008 K\ub9ac\uadf8\ub97c \uc55e\ub450\uace0 \ub300\ub300\uc801\uc778 \ub9ac\ube4c\ub529\uc744 \uc2e4\uc2dc\ud558\uc600\ub294\ub370, \uc2dc\uc98c \ucd08\ubc18 \ubd80\uc9c4\ud55c \ubaa8\uc2b5\uc744 \ubcf4\uc5ec \uc9c8\ud0c0\ub97c \ubc1b\uae30\ub3c4 \ud558\uc600\ub2e4. \ud558\uc9c0\ub9cc \ub9ac\uadf8 \ud6c4\ubc18\uae30\uc5d0 \ub4e4\uc5b4\uc11c\uba70 \uc774\ub0b4 \uacbd\uae30\ub825\uc744 \ub04c\uc5b4\uc62c\ub824 \uc815\uaddc \ub9ac\uadf8 \ucd5c\uc885 4\uc704 \ub85c \uc2dc\uc98c\uc744 \ub9c8\uac10\ud558\uc600\ub2e4. \ud2b9\uc720\uc758 \uce5c\ud654\ub825\uc73c\ub85c \uc778\ud574 \uc804\ubd81 \ud604\ub300 \ubaa8\ud130\uc2a4\uc758 \ud074\ub7fd \ud558\uc6b0\uc2a4 \ubc0f \ud6c8\ub828\uc7a5\uc774 \uc18c\uc7ac\ud558\uace0 \uc788\ub294 \uc804\ub77c\ubd81\ub3c4 \uc644\uc8fc\uad70 \ubd09\ub3d9\uc74d\uc758 \uc9c0\uba85\uc744 \ubcf8\ub530\uc11c \ubd09\ub3d9 \uc774\uc7a5\uc774\ub77c\ub294 \ubcc4\uba85\uc744 \uc5bb\uc5c8\ub2e4.", "sentence_answer": "\ud558\uc9c0\ub9cc \ub9ac\uadf8 \ud6c4\ubc18\uae30\uc5d0 \ub4e4\uc5b4\uc11c\uba70 \uc774\ub0b4 \uacbd\uae30\ub825\uc744 \ub04c\uc5b4\uc62c\ub824 \uc815\uaddc \ub9ac\uadf8 \ucd5c\uc885 4\uc704 \ub85c \uc2dc\uc98c\uc744 \ub9c8\uac10\ud558\uc600\ub2e4.", "paragraph_id": "5899667-0-1", "paragraph_question": "question: \uc804\ubd81 \ud604\ub300 \ubaa8\ud130\uc2a4\uac00 2008\ub144 K\ub9ac\uadf8\uc5d0\uc11c \ucd08\ubc18 \ubd80\uc9c4\uc744 \ub51b\uace0 \uc2dc\uc98c\uc744 \ucd5c\uc885 \ub9c8\uac10\ud55c \uc21c\uc704\ub294?, context: 2005\ub144 7\uc6d4 3\uc77c, \uc870\uc724\ud658\uc758 \ud6c4\uc784\uc73c\ub85c \uc804\ubd81 \ud604\ub300 \ubaa8\ud130\uc2a4\uc758 \uac10\ub3c5\uc73c\ub85c \ubd80\uc784\ud558\uc600\ub2e4. \ub2f9\uc2dc \uc911\ud558\uc704\uad8c\uc744 \ub9f4\ub3cc\uace0 \uc788\uc5c8\ub358 \uc804\ubd81\uc758 \uc9c0\ud718\ubd09\uc744 \uc7a1\uc740 \ucd5c\uac15\ud76c \uac10\ub3c5\uc740 \ubd80\uc784 \uccab \ud574 \uc815\uaddc\ub9ac\uadf8\uc5d0\uc11c\ub294 \uc88b\uc740 \uc131\uc801\uc744 \ub0b4\uc9c0 \ubabb\ud558\uc600\uc73c\ub098 FA\ucef5\uc5d0\uc11c \uc6b0\uc2b9\uc744 \ucc28\uc9c0\ud574 2006\ub144 ACL\uc5d0 \ucd9c\uc804\ud558\uac8c \ub418\uc5c8\ub2e4. AFC \ucc54\ud53c\uc5b8\uc2a4\ub9ac\uadf8\uc5d0\uc11c \uc5fc\uae30\ud6c8, \uae40\ud615\ubc94, \uc81c \uce74\ub97c\ub8e8\uc2a4 \ub4f1\uc744 \uc55e\uc138\uc6cc \uc870\ubcc4 \uc608\uc120\uacfc \ubcf8\uc120 \ud1a0\ub108\uba3c\ud2b8\uc5d0\uc11c \uc5f0\uc774\uc740 \uc5ed\uc804\uc2b9\uc744 \uc77c\uad6c\uc5b4 \ub0b4\uba70 \uadf9\uc801\uc778 \uc6b0\uc2b9\uc744 \ucc28\uc9c0\ud558\uc600\ub2e4. 2008 K\ub9ac\uadf8\ub97c \uc55e\ub450\uace0 \ub300\ub300\uc801\uc778 \ub9ac\ube4c\ub529\uc744 \uc2e4\uc2dc\ud558\uc600\ub294\ub370, \uc2dc\uc98c \ucd08\ubc18 \ubd80\uc9c4\ud55c \ubaa8\uc2b5\uc744 \ubcf4\uc5ec \uc9c8\ud0c0\ub97c \ubc1b\uae30\ub3c4 \ud558\uc600\ub2e4. \ud558\uc9c0\ub9cc \ub9ac\uadf8 \ud6c4\ubc18\uae30\uc5d0 \ub4e4\uc5b4\uc11c\uba70 \uc774\ub0b4 \uacbd\uae30\ub825\uc744 \ub04c\uc5b4\uc62c\ub824 \uc815\uaddc \ub9ac\uadf8 \ucd5c\uc885 4\uc704\ub85c \uc2dc\uc98c\uc744 \ub9c8\uac10\ud558\uc600\ub2e4. \ud2b9\uc720\uc758 \uce5c\ud654\ub825\uc73c\ub85c \uc778\ud574 \uc804\ubd81 \ud604\ub300 \ubaa8\ud130\uc2a4\uc758 \ud074\ub7fd \ud558\uc6b0\uc2a4 \ubc0f \ud6c8\ub828\uc7a5\uc774 \uc18c\uc7ac\ud558\uace0 \uc788\ub294 \uc804\ub77c\ubd81\ub3c4 \uc644\uc8fc\uad70 \ubd09\ub3d9\uc74d\uc758 \uc9c0\uba85\uc744 \ubcf8\ub530\uc11c \ubd09\ub3d9 \uc774\uc7a5\uc774\ub77c\ub294 \ubcc4\uba85\uc744 \uc5bb\uc5c8\ub2e4."} {"question": "2011\ub144 \uae30\uc900 \ub274\uc695\uc758 \uc778\uad6c\ub294?", "paragraph": "\ub274\uc695\uc758 \uc778\uad6c\ub294 2011\ub144 \uae30\uc900\uc73c\ub85c 8,244,910\ub9cc \uba85\uc73c\ub85c, \ubbf8\uad6d\uc5d0\uc11c \uac00\uc7a5 \uc778\uad6c\uac00 \ub9ce\uc740 \ub3c4\uc2dc\uc774\ub2e4. \uc9c0\ub09c 30\ub144\ubd80\ud130 \ud604\uc7ac\uae4c\uc9c0, \ub274\uc695\uc758 \uc778\uad6c\ub294 \uc99d\uac00 \ucd94\uc138\uc5d0 \uc788\uc73c\uba70, \uc778\uad6c\ud559\uc790\ub4e4\uc740 \uc778\uad6c\ub294 2030\ub144\uae4c\uc9c0 \uc778\uad6c\ub294 920\ub9cc \uba85\uc5d0\uc11c 950\ub9cc \uba85\uc5d0 \ub2ec\ud560 \uac83\uc774\ub77c\uace0 \ucd94\uacc4\ud558\uace0 \uc788\ub2e4. 2010\ub144 \ubbf8\uad6d \uc778\uad6c\uc870\uc0ac\uad6d\uc5d0 \ub530\ub974\uba74 2000\ub144 800\ub9cc\uc774\ub358 \uc778\uad6c\uac00 2.1% \uc99d\uac00\ud574 \ucd5c\uace0 \uae30\ub85d\uc778 8,175,133\ub9cc \uba85\uc744 \uae30\ub85d\ud588\ub2e4\uace0 \ubc1c\ud45c\ud558\uc600\uace0, \uc774\ub294 \ub85c\uc2a4\uc564\uc824\ub808\uc2a4\uc640 \uc2dc\uce74\uace0\ub97c \ud569\uce5c \uc778\uad6c\ubcf4\ub2e4 \ub354 \ub9ce\ub2e4. \uc0cc\ud504\ub780\uc2dc\uc2a4\ucf54 \ubca0\uc774 \uc5d0\uc5b4\ub9ac\uc5b4\uc758 \ucd1d \uc778\uad6c\ubcf4\ub2e4\ub3c4 \ub9ce\ub2e4. \ub274\uc695\uc758 \uc778\uad6c \ubc00\ub3c4\ub294 1km\ub2f9 1\ub9cc 194\uba85\uc774\uba70, \uc778\uad6c 10\ub9cc \uba85 \uc774\uc0c1 \uc9c0\uc790\uccb4 \uc911 \uac00\uc7a5 \ub192\ub2e4. \ub9e8\ud574\ud2bc\uc758 \uc778\uad6c \ubc00\ub3c4\ub294 1km\ub2f9 2\ub9cc 5846\uba85\uc774\uba70, \ubbf8\uad6d\uc758 \uad70 \uc911 1\uc704\uc774\ub2e4.", "answer": "8,244,910\ub9cc", "sentence": "\ub274\uc695\uc758 \uc778\uad6c\ub294 2011\ub144 \uae30\uc900\uc73c\ub85c 8,244,910\ub9cc \uba85\uc73c\ub85c, \ubbf8\uad6d\uc5d0\uc11c \uac00\uc7a5 \uc778\uad6c\uac00 \ub9ce\uc740 \ub3c4\uc2dc\uc774\ub2e4.", "paragraph_sentence": " \ub274\uc695\uc758 \uc778\uad6c\ub294 2011\ub144 \uae30\uc900\uc73c\ub85c 8,244,910\ub9cc \uba85\uc73c\ub85c, \ubbf8\uad6d\uc5d0\uc11c \uac00\uc7a5 \uc778\uad6c\uac00 \ub9ce\uc740 \ub3c4\uc2dc\uc774\ub2e4. \uc9c0\ub09c 30\ub144\ubd80\ud130 \ud604\uc7ac\uae4c\uc9c0, \ub274\uc695\uc758 \uc778\uad6c\ub294 \uc99d\uac00 \ucd94\uc138\uc5d0 \uc788\uc73c\uba70, \uc778\uad6c\ud559\uc790\ub4e4\uc740 \uc778\uad6c\ub294 2030\ub144\uae4c\uc9c0 \uc778\uad6c\ub294 920\ub9cc \uba85\uc5d0\uc11c 950\ub9cc \uba85\uc5d0 \ub2ec\ud560 \uac83\uc774\ub77c\uace0 \ucd94\uacc4\ud558\uace0 \uc788\ub2e4. 2010\ub144 \ubbf8\uad6d \uc778\uad6c\uc870\uc0ac\uad6d\uc5d0 \ub530\ub974\uba74 2000\ub144 800\ub9cc\uc774\ub358 \uc778\uad6c\uac00 2.1% \uc99d\uac00\ud574 \ucd5c\uace0 \uae30\ub85d\uc778 8,175,133\ub9cc \uba85\uc744 \uae30\ub85d\ud588\ub2e4\uace0 \ubc1c\ud45c\ud558\uc600\uace0, \uc774\ub294 \ub85c\uc2a4\uc564\uc824\ub808\uc2a4\uc640 \uc2dc\uce74\uace0\ub97c \ud569\uce5c \uc778\uad6c\ubcf4\ub2e4 \ub354 \ub9ce\ub2e4. \uc0cc\ud504\ub780\uc2dc\uc2a4\ucf54 \ubca0\uc774 \uc5d0\uc5b4\ub9ac\uc5b4\uc758 \ucd1d \uc778\uad6c\ubcf4\ub2e4\ub3c4 \ub9ce\ub2e4. \ub274\uc695\uc758 \uc778\uad6c \ubc00\ub3c4\ub294 1km\ub2f9 1\ub9cc 194\uba85\uc774\uba70, \uc778\uad6c 10\ub9cc \uba85 \uc774\uc0c1 \uc9c0\uc790\uccb4 \uc911 \uac00\uc7a5 \ub192\ub2e4. \ub9e8\ud574\ud2bc\uc758 \uc778\uad6c \ubc00\ub3c4\ub294 1km\ub2f9 2\ub9cc 5846\uba85\uc774\uba70, \ubbf8\uad6d\uc758 \uad70 \uc911 1\uc704\uc774\ub2e4.", "paragraph_answer": "\ub274\uc695\uc758 \uc778\uad6c\ub294 2011\ub144 \uae30\uc900\uc73c\ub85c 8,244,910\ub9cc \uba85\uc73c\ub85c, \ubbf8\uad6d\uc5d0\uc11c \uac00\uc7a5 \uc778\uad6c\uac00 \ub9ce\uc740 \ub3c4\uc2dc\uc774\ub2e4. \uc9c0\ub09c 30\ub144\ubd80\ud130 \ud604\uc7ac\uae4c\uc9c0, \ub274\uc695\uc758 \uc778\uad6c\ub294 \uc99d\uac00 \ucd94\uc138\uc5d0 \uc788\uc73c\uba70, \uc778\uad6c\ud559\uc790\ub4e4\uc740 \uc778\uad6c\ub294 2030\ub144\uae4c\uc9c0 \uc778\uad6c\ub294 920\ub9cc \uba85\uc5d0\uc11c 950\ub9cc \uba85\uc5d0 \ub2ec\ud560 \uac83\uc774\ub77c\uace0 \ucd94\uacc4\ud558\uace0 \uc788\ub2e4. 2010\ub144 \ubbf8\uad6d \uc778\uad6c\uc870\uc0ac\uad6d\uc5d0 \ub530\ub974\uba74 2000\ub144 800\ub9cc\uc774\ub358 \uc778\uad6c\uac00 2.1% \uc99d\uac00\ud574 \ucd5c\uace0 \uae30\ub85d\uc778 8,175,133\ub9cc \uba85\uc744 \uae30\ub85d\ud588\ub2e4\uace0 \ubc1c\ud45c\ud558\uc600\uace0, \uc774\ub294 \ub85c\uc2a4\uc564\uc824\ub808\uc2a4\uc640 \uc2dc\uce74\uace0\ub97c \ud569\uce5c \uc778\uad6c\ubcf4\ub2e4 \ub354 \ub9ce\ub2e4. \uc0cc\ud504\ub780\uc2dc\uc2a4\ucf54 \ubca0\uc774 \uc5d0\uc5b4\ub9ac\uc5b4\uc758 \ucd1d \uc778\uad6c\ubcf4\ub2e4\ub3c4 \ub9ce\ub2e4. \ub274\uc695\uc758 \uc778\uad6c \ubc00\ub3c4\ub294 1km\ub2f9 1\ub9cc 194\uba85\uc774\uba70, \uc778\uad6c 10\ub9cc \uba85 \uc774\uc0c1 \uc9c0\uc790\uccb4 \uc911 \uac00\uc7a5 \ub192\ub2e4. \ub9e8\ud574\ud2bc\uc758 \uc778\uad6c \ubc00\ub3c4\ub294 1km\ub2f9 2\ub9cc 5846\uba85\uc774\uba70, \ubbf8\uad6d\uc758 \uad70 \uc911 1\uc704\uc774\ub2e4.", "sentence_answer": "\ub274\uc695\uc758 \uc778\uad6c\ub294 2011\ub144 \uae30\uc900\uc73c\ub85c 8,244,910\ub9cc \uba85\uc73c\ub85c, \ubbf8\uad6d\uc5d0\uc11c \uac00\uc7a5 \uc778\uad6c\uac00 \ub9ce\uc740 \ub3c4\uc2dc\uc774\ub2e4.", "paragraph_id": "6383054-3-0", "paragraph_question": "question: 2011\ub144 \uae30\uc900 \ub274\uc695\uc758 \uc778\uad6c\ub294?, context: \ub274\uc695\uc758 \uc778\uad6c\ub294 2011\ub144 \uae30\uc900\uc73c\ub85c 8,244,910\ub9cc \uba85\uc73c\ub85c, \ubbf8\uad6d\uc5d0\uc11c \uac00\uc7a5 \uc778\uad6c\uac00 \ub9ce\uc740 \ub3c4\uc2dc\uc774\ub2e4. \uc9c0\ub09c 30\ub144\ubd80\ud130 \ud604\uc7ac\uae4c\uc9c0, \ub274\uc695\uc758 \uc778\uad6c\ub294 \uc99d\uac00 \ucd94\uc138\uc5d0 \uc788\uc73c\uba70, \uc778\uad6c\ud559\uc790\ub4e4\uc740 \uc778\uad6c\ub294 2030\ub144\uae4c\uc9c0 \uc778\uad6c\ub294 920\ub9cc \uba85\uc5d0\uc11c 950\ub9cc \uba85\uc5d0 \ub2ec\ud560 \uac83\uc774\ub77c\uace0 \ucd94\uacc4\ud558\uace0 \uc788\ub2e4. 2010\ub144 \ubbf8\uad6d \uc778\uad6c\uc870\uc0ac\uad6d\uc5d0 \ub530\ub974\uba74 2000\ub144 800\ub9cc\uc774\ub358 \uc778\uad6c\uac00 2.1% \uc99d\uac00\ud574 \ucd5c\uace0 \uae30\ub85d\uc778 8,175,133\ub9cc \uba85\uc744 \uae30\ub85d\ud588\ub2e4\uace0 \ubc1c\ud45c\ud558\uc600\uace0, \uc774\ub294 \ub85c\uc2a4\uc564\uc824\ub808\uc2a4\uc640 \uc2dc\uce74\uace0\ub97c \ud569\uce5c \uc778\uad6c\ubcf4\ub2e4 \ub354 \ub9ce\ub2e4. \uc0cc\ud504\ub780\uc2dc\uc2a4\ucf54 \ubca0\uc774 \uc5d0\uc5b4\ub9ac\uc5b4\uc758 \ucd1d \uc778\uad6c\ubcf4\ub2e4\ub3c4 \ub9ce\ub2e4. \ub274\uc695\uc758 \uc778\uad6c \ubc00\ub3c4\ub294 1km\ub2f9 1\ub9cc 194\uba85\uc774\uba70, \uc778\uad6c 10\ub9cc \uba85 \uc774\uc0c1 \uc9c0\uc790\uccb4 \uc911 \uac00\uc7a5 \ub192\ub2e4. \ub9e8\ud574\ud2bc\uc758 \uc778\uad6c \ubc00\ub3c4\ub294 1km\ub2f9 2\ub9cc 5846\uba85\uc774\uba70, \ubbf8\uad6d\uc758 \uad70 \uc911 1\uc704\uc774\ub2e4."} {"question": "\ud55c\uc871\ub3d9\ub9f9\ud68c\ub97c \uc870\uc9c1\ud55c \ud574\ub294 \uc5b8\uc81c\uc778\uac00?", "paragraph": "\uadf8\ub7ec\ub098 \uccad\ub144\uacc4\uce35\uacfc \uc720\ub9bc\uacc4\uce35\uacfc\uc758 \uc54c\ub825\uc73c\ub85c \ub3c5\ub9bd\uc6b4\ub3d9 \uc120\uc0c1\uc5d0 \ubd84\uaddc\uac00 \uc77c\uc5b4\ub098\uc790 \uadf8\ub294 \ubd81\uacbd(\u5317\u4eac)\uc73c\ub85c \uac00\ubc84\ub838\ub2e4. \uadf8\ub7ec\uc790 \uadf8\uc640 20\uc5ec\ub144 \uac04\uc744 \ub3d9\uace0\ub3d9\ub77d\ud558\ub358 \uc804\ub355\uc6d0(\u5168\u5fb7\u5143)\uc740 \ud1b5\uc758\ubd80\uc758 \ubd80\ud65c\uc744 \uc8fc\uc7a5\ud558\uc5ec \uadf8\ub97c \ub2e4\uc2dc \ub3cc\uc544\uc624\ub3c4\ub85d \ud558\uc600\uc73c\uba70, 1923\ub144 12\uc6d4\uc5d0 \uad70\uc0ac\ubd80\ubb38\uc744 \ud655\ub300\ud558\uc5ec \uc9c4\uc6a9\uc744 \uac15\ud654\ud558\uac8c \ub418\uc790 \uadf8\ub294 \ud1b5\uc758\ubd80\uc758 \uad70\uc0ac\ubd80\uc7a5\uc5d0 \uc784\uba85\ub418\uc5c8\ub2e4. \uc774\ud574\uc5d0 \uadf8\ub294 \ud658\uc778\ud604(\u6853\u4ec1\u7e23)\uc5d0\uc11c \uc190\uadf9\uc7a5(\u5b6b\u514b\u7ae0) \ub3c5\uace0\uc6b1(\u7368\u5b64\u65ed) \ub4f1\uacfc \ud568\uaed8 \ud55c\uad50\uacf5\ud68c(\u97d3\u50d1\u516c\u6703)\ub97c \uc870\uc9c1\ud558\uc5ec \uad6c\uad6d\ud65c\ub3d9\uc744 \uacc4\uc18d\ud558\ub2e4\uac00 \uc77c\uc81c\uc758 \ucd94\uaca9\uc744 \ud53c\ud558\uc5ec \ub2e4\uc2dc \ubd81\uacbd\uc73c\ub85c \uac74\ub108\uac14\ub2e4. 1925\ub144 5\uc6d4 31\uc77c\uc5d0 \uc784\uc2dc\uc815\ubd80\uc5d0 \ub0b4\ubd84\uc774 \uc77c\uc5b4\ub098\uc790 \uadf8\ub294 \ubd81\uacbd\uc5d0\uc11c \uc774\ub97c \uaddc\ud0c4\ud558\ub294 \uad50\uc815\uc11c(\u77ef\u6b63\u66f8)\ub97c \uc791\uc131\ud558\uc5ec \uc5f0\ubcd1\ud638(\u5ef6\u79c9\u660a), \ubc15\uc22d\ubcd1(\u6734\u5d07\u79c9) \ub4f1\uacfc \uac19\uc774 \uc11c\uba85 \ubc30\ud3ec\ud558\uc600\ub2e4. 1930\ub144 2\uc6d4\uc5d0\ub294 \ubd81\uacbd\uc5d0\uc11c \uc870\uc131\ud658(\u66fa\u6210\u7165) \uc190\uc77c\ubbfc(\u5b6b\u9038\u6c11) \ub4f1\uacfc \ud55c\uc871\ub3d9\ub9f9\ud68c(\u97d3\u65cf\u540c\u76df\u6703)\ub97c \uc870\uc9c1\ud558\uc600\uc73c\uba70, \uac19\uc740 \ud574 7\uc6d4\uc5d0\ub294 \uac15\uad6c\uc6b0(\u59dc\u4e5d\u79b9) \ub4f1\uacfc \uc870\uc120\ud601\uba85\ub2f9 \uc81c1\uc9c0\ubd80\ub97c \uc870\uc9c1\ud558\uc5ec \ud56d\uc77c\ud22c\uc7c1\uc744 \uacc4\uc18d\ud558\uc600\ub2e4.", "answer": "1930\ub144", "sentence": "1930\ub144 2\uc6d4\uc5d0\ub294 \ubd81\uacbd\uc5d0\uc11c \uc870\uc131\ud658(\u66fa\u6210\u7165) \uc190\uc77c\ubbfc(\u5b6b\u9038\u6c11) \ub4f1\uacfc \ud55c\uc871\ub3d9\ub9f9\ud68c(\u97d3\u65cf\u540c\u76df\u6703)\ub97c \uc870\uc9c1\ud558\uc600\uc73c\uba70, \uac19\uc740 \ud574 7\uc6d4\uc5d0\ub294 \uac15\uad6c\uc6b0(\u59dc\u4e5d\u79b9) \ub4f1\uacfc \uc870\uc120\ud601\uba85\ub2f9 \uc81c1\uc9c0\ubd80\ub97c \uc870\uc9c1\ud558\uc5ec \ud56d\uc77c\ud22c\uc7c1\uc744 \uacc4\uc18d\ud558\uc600\ub2e4.", "paragraph_sentence": "\uadf8\ub7ec\ub098 \uccad\ub144\uacc4\uce35\uacfc \uc720\ub9bc\uacc4\uce35\uacfc\uc758 \uc54c\ub825\uc73c\ub85c \ub3c5\ub9bd\uc6b4\ub3d9 \uc120\uc0c1\uc5d0 \ubd84\uaddc\uac00 \uc77c\uc5b4\ub098\uc790 \uadf8\ub294 \ubd81\uacbd(\u5317\u4eac)\uc73c\ub85c \uac00\ubc84\ub838\ub2e4. \uadf8\ub7ec\uc790 \uadf8\uc640 20\uc5ec\ub144 \uac04\uc744 \ub3d9\uace0\ub3d9\ub77d\ud558\ub358 \uc804\ub355\uc6d0(\u5168\u5fb7\u5143)\uc740 \ud1b5\uc758\ubd80\uc758 \ubd80\ud65c\uc744 \uc8fc\uc7a5\ud558\uc5ec \uadf8\ub97c \ub2e4\uc2dc \ub3cc\uc544\uc624\ub3c4\ub85d \ud558\uc600\uc73c\uba70, 1923\ub144 12\uc6d4\uc5d0 \uad70\uc0ac\ubd80\ubb38\uc744 \ud655\ub300\ud558\uc5ec \uc9c4\uc6a9\uc744 \uac15\ud654\ud558\uac8c \ub418\uc790 \uadf8\ub294 \ud1b5\uc758\ubd80\uc758 \uad70\uc0ac\ubd80\uc7a5\uc5d0 \uc784\uba85\ub418\uc5c8\ub2e4. \uc774\ud574\uc5d0 \uadf8\ub294 \ud658\uc778\ud604(\u6853\u4ec1\u7e23)\uc5d0\uc11c \uc190\uadf9\uc7a5(\u5b6b\u514b\u7ae0) \ub3c5\uace0\uc6b1(\u7368\u5b64\u65ed) \ub4f1\uacfc \ud568\uaed8 \ud55c\uad50\uacf5\ud68c(\u97d3\u50d1\u516c\u6703)\ub97c \uc870\uc9c1\ud558\uc5ec \uad6c\uad6d\ud65c\ub3d9\uc744 \uacc4\uc18d\ud558\ub2e4\uac00 \uc77c\uc81c\uc758 \ucd94\uaca9\uc744 \ud53c\ud558\uc5ec \ub2e4\uc2dc \ubd81\uacbd\uc73c\ub85c \uac74\ub108\uac14\ub2e4. 1925\ub144 5\uc6d4 31\uc77c\uc5d0 \uc784\uc2dc\uc815\ubd80\uc5d0 \ub0b4\ubd84\uc774 \uc77c\uc5b4\ub098\uc790 \uadf8\ub294 \ubd81\uacbd\uc5d0\uc11c \uc774\ub97c \uaddc\ud0c4\ud558\ub294 \uad50\uc815\uc11c(\u77ef\u6b63\u66f8)\ub97c \uc791\uc131\ud558\uc5ec \uc5f0\ubcd1\ud638(\u5ef6\u79c9\u660a), \ubc15\uc22d\ubcd1(\u6734\u5d07\u79c9) \ub4f1\uacfc \uac19\uc774 \uc11c\uba85 \ubc30\ud3ec\ud558\uc600\ub2e4. 1930\ub144 2\uc6d4\uc5d0\ub294 \ubd81\uacbd\uc5d0\uc11c \uc870\uc131\ud658(\u66fa\u6210\u7165) \uc190\uc77c\ubbfc(\u5b6b\u9038\u6c11) \ub4f1\uacfc \ud55c\uc871\ub3d9\ub9f9\ud68c(\u97d3\u65cf\u540c\u76df\u6703)\ub97c \uc870\uc9c1\ud558\uc600\uc73c\uba70, \uac19\uc740 \ud574 7\uc6d4\uc5d0\ub294 \uac15\uad6c\uc6b0(\u59dc\u4e5d\u79b9) \ub4f1\uacfc \uc870\uc120\ud601\uba85\ub2f9 \uc81c1\uc9c0\ubd80\ub97c \uc870\uc9c1\ud558\uc5ec \ud56d\uc77c\ud22c\uc7c1\uc744 \uacc4\uc18d\ud558\uc600\ub2e4. ", "paragraph_answer": "\uadf8\ub7ec\ub098 \uccad\ub144\uacc4\uce35\uacfc \uc720\ub9bc\uacc4\uce35\uacfc\uc758 \uc54c\ub825\uc73c\ub85c \ub3c5\ub9bd\uc6b4\ub3d9 \uc120\uc0c1\uc5d0 \ubd84\uaddc\uac00 \uc77c\uc5b4\ub098\uc790 \uadf8\ub294 \ubd81\uacbd(\u5317\u4eac)\uc73c\ub85c \uac00\ubc84\ub838\ub2e4. \uadf8\ub7ec\uc790 \uadf8\uc640 20\uc5ec\ub144 \uac04\uc744 \ub3d9\uace0\ub3d9\ub77d\ud558\ub358 \uc804\ub355\uc6d0(\u5168\u5fb7\u5143)\uc740 \ud1b5\uc758\ubd80\uc758 \ubd80\ud65c\uc744 \uc8fc\uc7a5\ud558\uc5ec \uadf8\ub97c \ub2e4\uc2dc \ub3cc\uc544\uc624\ub3c4\ub85d \ud558\uc600\uc73c\uba70, 1923\ub144 12\uc6d4\uc5d0 \uad70\uc0ac\ubd80\ubb38\uc744 \ud655\ub300\ud558\uc5ec \uc9c4\uc6a9\uc744 \uac15\ud654\ud558\uac8c \ub418\uc790 \uadf8\ub294 \ud1b5\uc758\ubd80\uc758 \uad70\uc0ac\ubd80\uc7a5\uc5d0 \uc784\uba85\ub418\uc5c8\ub2e4. \uc774\ud574\uc5d0 \uadf8\ub294 \ud658\uc778\ud604(\u6853\u4ec1\u7e23)\uc5d0\uc11c \uc190\uadf9\uc7a5(\u5b6b\u514b\u7ae0) \ub3c5\uace0\uc6b1(\u7368\u5b64\u65ed) \ub4f1\uacfc \ud568\uaed8 \ud55c\uad50\uacf5\ud68c(\u97d3\u50d1\u516c\u6703)\ub97c \uc870\uc9c1\ud558\uc5ec \uad6c\uad6d\ud65c\ub3d9\uc744 \uacc4\uc18d\ud558\ub2e4\uac00 \uc77c\uc81c\uc758 \ucd94\uaca9\uc744 \ud53c\ud558\uc5ec \ub2e4\uc2dc \ubd81\uacbd\uc73c\ub85c \uac74\ub108\uac14\ub2e4. 1925\ub144 5\uc6d4 31\uc77c\uc5d0 \uc784\uc2dc\uc815\ubd80\uc5d0 \ub0b4\ubd84\uc774 \uc77c\uc5b4\ub098\uc790 \uadf8\ub294 \ubd81\uacbd\uc5d0\uc11c \uc774\ub97c \uaddc\ud0c4\ud558\ub294 \uad50\uc815\uc11c(\u77ef\u6b63\u66f8)\ub97c \uc791\uc131\ud558\uc5ec \uc5f0\ubcd1\ud638(\u5ef6\u79c9\u660a), \ubc15\uc22d\ubcd1(\u6734\u5d07\u79c9) \ub4f1\uacfc \uac19\uc774 \uc11c\uba85 \ubc30\ud3ec\ud558\uc600\ub2e4. 1930\ub144 2\uc6d4\uc5d0\ub294 \ubd81\uacbd\uc5d0\uc11c \uc870\uc131\ud658(\u66fa\u6210\u7165) \uc190\uc77c\ubbfc(\u5b6b\u9038\u6c11) \ub4f1\uacfc \ud55c\uc871\ub3d9\ub9f9\ud68c(\u97d3\u65cf\u540c\u76df\u6703)\ub97c \uc870\uc9c1\ud558\uc600\uc73c\uba70, \uac19\uc740 \ud574 7\uc6d4\uc5d0\ub294 \uac15\uad6c\uc6b0(\u59dc\u4e5d\u79b9) \ub4f1\uacfc \uc870\uc120\ud601\uba85\ub2f9 \uc81c1\uc9c0\ubd80\ub97c \uc870\uc9c1\ud558\uc5ec \ud56d\uc77c\ud22c\uc7c1\uc744 \uacc4\uc18d\ud558\uc600\ub2e4.", "sentence_answer": " 1930\ub144 2\uc6d4\uc5d0\ub294 \ubd81\uacbd\uc5d0\uc11c \uc870\uc131\ud658(\u66fa\u6210\u7165) \uc190\uc77c\ubbfc(\u5b6b\u9038\u6c11) \ub4f1\uacfc \ud55c\uc871\ub3d9\ub9f9\ud68c(\u97d3\u65cf\u540c\u76df\u6703)\ub97c \uc870\uc9c1\ud558\uc600\uc73c\uba70, \uac19\uc740 \ud574 7\uc6d4\uc5d0\ub294 \uac15\uad6c\uc6b0(\u59dc\u4e5d\u79b9) \ub4f1\uacfc \uc870\uc120\ud601\uba85\ub2f9 \uc81c1\uc9c0\ubd80\ub97c \uc870\uc9c1\ud558\uc5ec \ud56d\uc77c\ud22c\uc7c1\uc744 \uacc4\uc18d\ud558\uc600\ub2e4.", "paragraph_id": "6161564-3-1", "paragraph_question": "question: \ud55c\uc871\ub3d9\ub9f9\ud68c\ub97c \uc870\uc9c1\ud55c \ud574\ub294 \uc5b8\uc81c\uc778\uac00?, context: \uadf8\ub7ec\ub098 \uccad\ub144\uacc4\uce35\uacfc \uc720\ub9bc\uacc4\uce35\uacfc\uc758 \uc54c\ub825\uc73c\ub85c \ub3c5\ub9bd\uc6b4\ub3d9 \uc120\uc0c1\uc5d0 \ubd84\uaddc\uac00 \uc77c\uc5b4\ub098\uc790 \uadf8\ub294 \ubd81\uacbd(\u5317\u4eac)\uc73c\ub85c \uac00\ubc84\ub838\ub2e4. \uadf8\ub7ec\uc790 \uadf8\uc640 20\uc5ec\ub144 \uac04\uc744 \ub3d9\uace0\ub3d9\ub77d\ud558\ub358 \uc804\ub355\uc6d0(\u5168\u5fb7\u5143)\uc740 \ud1b5\uc758\ubd80\uc758 \ubd80\ud65c\uc744 \uc8fc\uc7a5\ud558\uc5ec \uadf8\ub97c \ub2e4\uc2dc \ub3cc\uc544\uc624\ub3c4\ub85d \ud558\uc600\uc73c\uba70, 1923\ub144 12\uc6d4\uc5d0 \uad70\uc0ac\ubd80\ubb38\uc744 \ud655\ub300\ud558\uc5ec \uc9c4\uc6a9\uc744 \uac15\ud654\ud558\uac8c \ub418\uc790 \uadf8\ub294 \ud1b5\uc758\ubd80\uc758 \uad70\uc0ac\ubd80\uc7a5\uc5d0 \uc784\uba85\ub418\uc5c8\ub2e4. \uc774\ud574\uc5d0 \uadf8\ub294 \ud658\uc778\ud604(\u6853\u4ec1\u7e23)\uc5d0\uc11c \uc190\uadf9\uc7a5(\u5b6b\u514b\u7ae0) \ub3c5\uace0\uc6b1(\u7368\u5b64\u65ed) \ub4f1\uacfc \ud568\uaed8 \ud55c\uad50\uacf5\ud68c(\u97d3\u50d1\u516c\u6703)\ub97c \uc870\uc9c1\ud558\uc5ec \uad6c\uad6d\ud65c\ub3d9\uc744 \uacc4\uc18d\ud558\ub2e4\uac00 \uc77c\uc81c\uc758 \ucd94\uaca9\uc744 \ud53c\ud558\uc5ec \ub2e4\uc2dc \ubd81\uacbd\uc73c\ub85c \uac74\ub108\uac14\ub2e4. 1925\ub144 5\uc6d4 31\uc77c\uc5d0 \uc784\uc2dc\uc815\ubd80\uc5d0 \ub0b4\ubd84\uc774 \uc77c\uc5b4\ub098\uc790 \uadf8\ub294 \ubd81\uacbd\uc5d0\uc11c \uc774\ub97c \uaddc\ud0c4\ud558\ub294 \uad50\uc815\uc11c(\u77ef\u6b63\u66f8)\ub97c \uc791\uc131\ud558\uc5ec \uc5f0\ubcd1\ud638(\u5ef6\u79c9\u660a), \ubc15\uc22d\ubcd1(\u6734\u5d07\u79c9) \ub4f1\uacfc \uac19\uc774 \uc11c\uba85 \ubc30\ud3ec\ud558\uc600\ub2e4. 1930\ub144 2\uc6d4\uc5d0\ub294 \ubd81\uacbd\uc5d0\uc11c \uc870\uc131\ud658(\u66fa\u6210\u7165) \uc190\uc77c\ubbfc(\u5b6b\u9038\u6c11) \ub4f1\uacfc \ud55c\uc871\ub3d9\ub9f9\ud68c(\u97d3\u65cf\u540c\u76df\u6703)\ub97c \uc870\uc9c1\ud558\uc600\uc73c\uba70, \uac19\uc740 \ud574 7\uc6d4\uc5d0\ub294 \uac15\uad6c\uc6b0(\u59dc\u4e5d\u79b9) \ub4f1\uacfc \uc870\uc120\ud601\uba85\ub2f9 \uc81c1\uc9c0\ubd80\ub97c \uc870\uc9c1\ud558\uc5ec \ud56d\uc77c\ud22c\uc7c1\uc744 \uacc4\uc18d\ud558\uc600\ub2e4."} {"question": "1991\ub144 \ud1b0 \uae00\ub798\ube48\uc758 \ubc29\uc5b4\uc728\uc740?", "paragraph": "1991\ub144\uc5d0\ub294 5\uc6d4\uae4c\uc9c0 6\uc2b9 0\ud328\ub85c \uc774\ub2ec\uc758 \ud22c\uc218\ub97c \ubc1b\uace0, \uc790\uc2e0 \uc778\uc0dd\uc5d0\uc11c \ucc98\uc74c\uc73c\ub85c \uc62c\uc2a4\ud0c0\uc804 \ucd9c\uc804\uacfc \ud568\uaed8 \uc120\ubc1c\ud22c\uc218\ub77c\ub294 \uc18c\uc784\uc744 \ub9e1\uac8c \ub41c\ub2e4. \uadf8\ub9ac\uace0 \uc2dc\uc98c\uc744 \ub05d\ub0bc \ub2f9\uc2dc\uc758 \uc131\uc801\uc740 20\uc2b9 11\ud328, \ubc29\uc5b4\uc728 2.55\ub77c\ub294 \uc5c4\uccad\ub09c \uc131\uc801\uacfc \ud568\uaed8 \ub2e4\uc2b9\uc655\uc758 \ud0c0\uc774\ud2c0\uc744 \ub530\ub0b8\ub2e4. \uadf8\ub9ac\uace0 \uc791\ub144\uc5d0 \ucd5c\ud558\uc704\uc600\ub358 \ud300\uc740 9\ub144 \ub9cc\uc5d0 \ub9ac\uadf8\uc6b0\uc2b9\uc744 \ucc28\uc9c0\ud55c\ub2e4. \ud53c\uce20\ubc84\uadf8 \ud30c\uc774\ub9ac\uce20\uc640\uc758 \ub9ac\uadf8 \ucc54\ud53c\uc5b8\uc2ed \uc2dc\ub9ac\uc988\uc5d0\uc11c\ub294 2\ud328\ub97c \ud588\uc9c0\ub9cc, \ud300\uc740 33\ub144 \ub9cc\uc5d0 \uc560\ud2c0\ub79c\ud0c0\ub85c \uc774\uc804\ud55c \ud6c4 \ucc98\uc74c\uc73c\ub85c \ub9ac\uadf8\uc6b0\uc2b9\uc744 \ud55c\ub2e4. \ubbf8\ub124\uc18c\ud0c0 \ud2b8\uc708\uc2a4\uc640\uc758 \uc6d4\ub4dc\uc2dc\ub9ac\uc988\uc5d0\uc11c\ub294 1\uc2b9 1\ud328\ub97c \ud588\uc9c0\ub9cc, \ud300\uc740 3\uc2b9 4\ud328\ub85c \uc544\uc27d\uac8c \ud328\ud1f4\ud558\uace0 \ub9cc\ub2e4. \uc2dc\uc98c\uc774 \ub05d\ub09c \ud6c4\uc5d0\ub294 \ucc98\uc74c\uc73c\ub85c \uc0ac\uc774 \uc601 \uc0c1\uc744 \uc218\uc0c1\ud588\ub2e4. 1992\ub144\uc5d0\ub294 1954\ub144, 1955\ub144\uc5d0 2\ub144\uc5f0\uc18d\uc73c\ub85c \uc62c\uc2a4\ud0c0\uc804\uc5d0\uc11c \uc120\ubc1c\ub85c \ucd9c\uc804\ud55c \ub85c\ube48 \ub85c\ubc84\uce20 \uc774\ub798 \ucc98\uc74c\uc73c\ub85c 2\ub144\uc5f0\uc18d\uc73c\ub85c \uc62c\uc2a4\ud0c0\uc804 \uc120\ubc1c\ub85c \ubf51\ud614\uc9c0\ub9cc, 1\ud68c 1\uc544\uc6c3\ubd80\ud130 7\uc5f0\uc18d\uc73c\ub85c \uc548\ud0c0\ub97c \ub9de\ub294 \ub4f1 2\ud68c\ub3c4\uc911\uae4c\uc9c0 5\uc2e4\uc810\uc758 \ub09c\uc870\ub97c \ubcf4\uc600\ub2e4. \uc2dc\uc98c\uc5d0\uc11c\ub294 20\uc2b9 8\ud328, \ubc29\uc5b4\uc728 2.76\uacfc \ub9ac\uadf8 \ucd5c\ub2e4\uc778 5\ubc88\uc758 \uc644\ubd09\uc744 \uae30\ub85d\ud558\uba70, \ud300\uc758 2\ub144\uc5f0\uc18d \uc9c0\uad6c\uc6b0\uc2b9\uc5d0 \uacf5\ud5cc\ud55c\ub2e4. \uc804\ub144\uacfc \uac19\uc774 \ud53c\uce20\ubc84\uadf8 \ud30c\uc774\ub9ac\uce20\uc640 \ub300\uacb0\uc774 \ub418\uc5c8\ub358 \ub9ac\uadf8 \ucc54\ud53c\uc5b8\uc2ed \uc2dc\ub9ac\uc988\uc5d0\uc11c\ub294 1\ucc28\uc804\uc5d0 \uc120\ubc1c\ub4f1\ud310\ud558\uc5ec 4\uc548\ud0c0 \ubb34\uc0ac\uad6c 1\uc2e4\uc810\uc73c\ub85c \uc644\ud22c\uc2b9\uc744 \ud588\uc9c0\ub9cc, \ud300\uc740 3\uc2b9 4\ud328\ub85c \ud328\ud1f4\ud55c\ub2e4. \uc0ac\uc774 \uc601 \uc0c1\uc758 \ud22c\ud45c\uc5d0\uc11c\ub294 \uc2dc\uce74\uace0 \ucef5\uc2a4\uc758 \uadf8\ub808\uadf8 \ub9e4\ub355\uc2a4\uc5d0 \uc774\uc5b4 2\uc704\uac00 \ub410\ub2e4.", "answer": "2.55", "sentence": "\uadf8\ub9ac\uace0 \uc2dc\uc98c\uc744 \ub05d\ub0bc \ub2f9\uc2dc\uc758 \uc131\uc801\uc740 20\uc2b9 11\ud328, \ubc29\uc5b4\uc728 2.55 \ub77c\ub294 \uc5c4\uccad\ub09c \uc131\uc801\uacfc \ud568\uaed8 \ub2e4\uc2b9\uc655\uc758 \ud0c0\uc774\ud2c0\uc744 \ub530\ub0b8\ub2e4.", "paragraph_sentence": "1991\ub144\uc5d0\ub294 5\uc6d4\uae4c\uc9c0 6\uc2b9 0\ud328\ub85c \uc774\ub2ec\uc758 \ud22c\uc218\ub97c \ubc1b\uace0, \uc790\uc2e0 \uc778\uc0dd\uc5d0\uc11c \ucc98\uc74c\uc73c\ub85c \uc62c\uc2a4\ud0c0\uc804 \ucd9c\uc804\uacfc \ud568\uaed8 \uc120\ubc1c\ud22c\uc218\ub77c\ub294 \uc18c\uc784\uc744 \ub9e1\uac8c \ub41c\ub2e4. \uadf8\ub9ac\uace0 \uc2dc\uc98c\uc744 \ub05d\ub0bc \ub2f9\uc2dc\uc758 \uc131\uc801\uc740 20\uc2b9 11\ud328, \ubc29\uc5b4\uc728 2.55 \ub77c\ub294 \uc5c4\uccad\ub09c \uc131\uc801\uacfc \ud568\uaed8 \ub2e4\uc2b9\uc655\uc758 \ud0c0\uc774\ud2c0\uc744 \ub530\ub0b8\ub2e4. \uadf8\ub9ac\uace0 \uc791\ub144\uc5d0 \ucd5c\ud558\uc704\uc600\ub358 \ud300\uc740 9\ub144 \ub9cc\uc5d0 \ub9ac\uadf8\uc6b0\uc2b9\uc744 \ucc28\uc9c0\ud55c\ub2e4. \ud53c\uce20\ubc84\uadf8 \ud30c\uc774\ub9ac\uce20\uc640\uc758 \ub9ac\uadf8 \ucc54\ud53c\uc5b8\uc2ed \uc2dc\ub9ac\uc988\uc5d0\uc11c\ub294 2\ud328\ub97c \ud588\uc9c0\ub9cc, \ud300\uc740 33\ub144 \ub9cc\uc5d0 \uc560\ud2c0\ub79c\ud0c0\ub85c \uc774\uc804\ud55c \ud6c4 \ucc98\uc74c\uc73c\ub85c \ub9ac\uadf8\uc6b0\uc2b9\uc744 \ud55c\ub2e4. \ubbf8\ub124\uc18c\ud0c0 \ud2b8\uc708\uc2a4\uc640\uc758 \uc6d4\ub4dc\uc2dc\ub9ac\uc988\uc5d0\uc11c\ub294 1\uc2b9 1\ud328\ub97c \ud588\uc9c0\ub9cc, \ud300\uc740 3\uc2b9 4\ud328\ub85c \uc544\uc27d\uac8c \ud328\ud1f4\ud558\uace0 \ub9cc\ub2e4. \uc2dc\uc98c\uc774 \ub05d\ub09c \ud6c4\uc5d0\ub294 \ucc98\uc74c\uc73c\ub85c \uc0ac\uc774 \uc601 \uc0c1\uc744 \uc218\uc0c1\ud588\ub2e4. 1992\ub144\uc5d0\ub294 1954\ub144, 1955\ub144\uc5d0 2\ub144\uc5f0\uc18d\uc73c\ub85c \uc62c\uc2a4\ud0c0\uc804\uc5d0\uc11c \uc120\ubc1c\ub85c \ucd9c\uc804\ud55c \ub85c\ube48 \ub85c\ubc84\uce20 \uc774\ub798 \ucc98\uc74c\uc73c\ub85c 2\ub144\uc5f0\uc18d\uc73c\ub85c \uc62c\uc2a4\ud0c0\uc804 \uc120\ubc1c\ub85c \ubf51\ud614\uc9c0\ub9cc, 1\ud68c 1\uc544\uc6c3\ubd80\ud130 7\uc5f0\uc18d\uc73c\ub85c \uc548\ud0c0\ub97c \ub9de\ub294 \ub4f1 2\ud68c\ub3c4\uc911\uae4c\uc9c0 5\uc2e4\uc810\uc758 \ub09c\uc870\ub97c \ubcf4\uc600\ub2e4. \uc2dc\uc98c\uc5d0\uc11c\ub294 20\uc2b9 8\ud328, \ubc29\uc5b4\uc728 2.76\uacfc \ub9ac\uadf8 \ucd5c\ub2e4\uc778 5\ubc88\uc758 \uc644\ubd09\uc744 \uae30\ub85d\ud558\uba70, \ud300\uc758 2\ub144\uc5f0\uc18d \uc9c0\uad6c\uc6b0\uc2b9\uc5d0 \uacf5\ud5cc\ud55c\ub2e4. \uc804\ub144\uacfc \uac19\uc774 \ud53c\uce20\ubc84\uadf8 \ud30c\uc774\ub9ac\uce20\uc640 \ub300\uacb0\uc774 \ub418\uc5c8\ub358 \ub9ac\uadf8 \ucc54\ud53c\uc5b8\uc2ed \uc2dc\ub9ac\uc988\uc5d0\uc11c\ub294 1\ucc28\uc804\uc5d0 \uc120\ubc1c\ub4f1\ud310\ud558\uc5ec 4\uc548\ud0c0 \ubb34\uc0ac\uad6c 1\uc2e4\uc810\uc73c\ub85c \uc644\ud22c\uc2b9\uc744 \ud588\uc9c0\ub9cc, \ud300\uc740 3\uc2b9 4\ud328\ub85c \ud328\ud1f4\ud55c\ub2e4. \uc0ac\uc774 \uc601 \uc0c1\uc758 \ud22c\ud45c\uc5d0\uc11c\ub294 \uc2dc\uce74\uace0 \ucef5\uc2a4\uc758 \uadf8\ub808\uadf8 \ub9e4\ub355\uc2a4\uc5d0 \uc774\uc5b4 2\uc704\uac00 \ub410\ub2e4.", "paragraph_answer": "1991\ub144\uc5d0\ub294 5\uc6d4\uae4c\uc9c0 6\uc2b9 0\ud328\ub85c \uc774\ub2ec\uc758 \ud22c\uc218\ub97c \ubc1b\uace0, \uc790\uc2e0 \uc778\uc0dd\uc5d0\uc11c \ucc98\uc74c\uc73c\ub85c \uc62c\uc2a4\ud0c0\uc804 \ucd9c\uc804\uacfc \ud568\uaed8 \uc120\ubc1c\ud22c\uc218\ub77c\ub294 \uc18c\uc784\uc744 \ub9e1\uac8c \ub41c\ub2e4. \uadf8\ub9ac\uace0 \uc2dc\uc98c\uc744 \ub05d\ub0bc \ub2f9\uc2dc\uc758 \uc131\uc801\uc740 20\uc2b9 11\ud328, \ubc29\uc5b4\uc728 2.55 \ub77c\ub294 \uc5c4\uccad\ub09c \uc131\uc801\uacfc \ud568\uaed8 \ub2e4\uc2b9\uc655\uc758 \ud0c0\uc774\ud2c0\uc744 \ub530\ub0b8\ub2e4. \uadf8\ub9ac\uace0 \uc791\ub144\uc5d0 \ucd5c\ud558\uc704\uc600\ub358 \ud300\uc740 9\ub144 \ub9cc\uc5d0 \ub9ac\uadf8\uc6b0\uc2b9\uc744 \ucc28\uc9c0\ud55c\ub2e4. \ud53c\uce20\ubc84\uadf8 \ud30c\uc774\ub9ac\uce20\uc640\uc758 \ub9ac\uadf8 \ucc54\ud53c\uc5b8\uc2ed \uc2dc\ub9ac\uc988\uc5d0\uc11c\ub294 2\ud328\ub97c \ud588\uc9c0\ub9cc, \ud300\uc740 33\ub144 \ub9cc\uc5d0 \uc560\ud2c0\ub79c\ud0c0\ub85c \uc774\uc804\ud55c \ud6c4 \ucc98\uc74c\uc73c\ub85c \ub9ac\uadf8\uc6b0\uc2b9\uc744 \ud55c\ub2e4. \ubbf8\ub124\uc18c\ud0c0 \ud2b8\uc708\uc2a4\uc640\uc758 \uc6d4\ub4dc\uc2dc\ub9ac\uc988\uc5d0\uc11c\ub294 1\uc2b9 1\ud328\ub97c \ud588\uc9c0\ub9cc, \ud300\uc740 3\uc2b9 4\ud328\ub85c \uc544\uc27d\uac8c \ud328\ud1f4\ud558\uace0 \ub9cc\ub2e4. \uc2dc\uc98c\uc774 \ub05d\ub09c \ud6c4\uc5d0\ub294 \ucc98\uc74c\uc73c\ub85c \uc0ac\uc774 \uc601 \uc0c1\uc744 \uc218\uc0c1\ud588\ub2e4. 1992\ub144\uc5d0\ub294 1954\ub144, 1955\ub144\uc5d0 2\ub144\uc5f0\uc18d\uc73c\ub85c \uc62c\uc2a4\ud0c0\uc804\uc5d0\uc11c \uc120\ubc1c\ub85c \ucd9c\uc804\ud55c \ub85c\ube48 \ub85c\ubc84\uce20 \uc774\ub798 \ucc98\uc74c\uc73c\ub85c 2\ub144\uc5f0\uc18d\uc73c\ub85c \uc62c\uc2a4\ud0c0\uc804 \uc120\ubc1c\ub85c \ubf51\ud614\uc9c0\ub9cc, 1\ud68c 1\uc544\uc6c3\ubd80\ud130 7\uc5f0\uc18d\uc73c\ub85c \uc548\ud0c0\ub97c \ub9de\ub294 \ub4f1 2\ud68c\ub3c4\uc911\uae4c\uc9c0 5\uc2e4\uc810\uc758 \ub09c\uc870\ub97c \ubcf4\uc600\ub2e4. \uc2dc\uc98c\uc5d0\uc11c\ub294 20\uc2b9 8\ud328, \ubc29\uc5b4\uc728 2.76\uacfc \ub9ac\uadf8 \ucd5c\ub2e4\uc778 5\ubc88\uc758 \uc644\ubd09\uc744 \uae30\ub85d\ud558\uba70, \ud300\uc758 2\ub144\uc5f0\uc18d \uc9c0\uad6c\uc6b0\uc2b9\uc5d0 \uacf5\ud5cc\ud55c\ub2e4. \uc804\ub144\uacfc \uac19\uc774 \ud53c\uce20\ubc84\uadf8 \ud30c\uc774\ub9ac\uce20\uc640 \ub300\uacb0\uc774 \ub418\uc5c8\ub358 \ub9ac\uadf8 \ucc54\ud53c\uc5b8\uc2ed \uc2dc\ub9ac\uc988\uc5d0\uc11c\ub294 1\ucc28\uc804\uc5d0 \uc120\ubc1c\ub4f1\ud310\ud558\uc5ec 4\uc548\ud0c0 \ubb34\uc0ac\uad6c 1\uc2e4\uc810\uc73c\ub85c \uc644\ud22c\uc2b9\uc744 \ud588\uc9c0\ub9cc, \ud300\uc740 3\uc2b9 4\ud328\ub85c \ud328\ud1f4\ud55c\ub2e4. \uc0ac\uc774 \uc601 \uc0c1\uc758 \ud22c\ud45c\uc5d0\uc11c\ub294 \uc2dc\uce74\uace0 \ucef5\uc2a4\uc758 \uadf8\ub808\uadf8 \ub9e4\ub355\uc2a4\uc5d0 \uc774\uc5b4 2\uc704\uac00 \ub410\ub2e4.", "sentence_answer": "\uadf8\ub9ac\uace0 \uc2dc\uc98c\uc744 \ub05d\ub0bc \ub2f9\uc2dc\uc758 \uc131\uc801\uc740 20\uc2b9 11\ud328, \ubc29\uc5b4\uc728 2.55 \ub77c\ub294 \uc5c4\uccad\ub09c \uc131\uc801\uacfc \ud568\uaed8 \ub2e4\uc2b9\uc655\uc758 \ud0c0\uc774\ud2c0\uc744 \ub530\ub0b8\ub2e4.", "paragraph_id": "6599752-0-1", "paragraph_question": "question: 1991\ub144 \ud1b0 \uae00\ub798\ube48\uc758 \ubc29\uc5b4\uc728\uc740?, context: 1991\ub144\uc5d0\ub294 5\uc6d4\uae4c\uc9c0 6\uc2b9 0\ud328\ub85c \uc774\ub2ec\uc758 \ud22c\uc218\ub97c \ubc1b\uace0, \uc790\uc2e0 \uc778\uc0dd\uc5d0\uc11c \ucc98\uc74c\uc73c\ub85c \uc62c\uc2a4\ud0c0\uc804 \ucd9c\uc804\uacfc \ud568\uaed8 \uc120\ubc1c\ud22c\uc218\ub77c\ub294 \uc18c\uc784\uc744 \ub9e1\uac8c \ub41c\ub2e4. \uadf8\ub9ac\uace0 \uc2dc\uc98c\uc744 \ub05d\ub0bc \ub2f9\uc2dc\uc758 \uc131\uc801\uc740 20\uc2b9 11\ud328, \ubc29\uc5b4\uc728 2.55\ub77c\ub294 \uc5c4\uccad\ub09c \uc131\uc801\uacfc \ud568\uaed8 \ub2e4\uc2b9\uc655\uc758 \ud0c0\uc774\ud2c0\uc744 \ub530\ub0b8\ub2e4. \uadf8\ub9ac\uace0 \uc791\ub144\uc5d0 \ucd5c\ud558\uc704\uc600\ub358 \ud300\uc740 9\ub144 \ub9cc\uc5d0 \ub9ac\uadf8\uc6b0\uc2b9\uc744 \ucc28\uc9c0\ud55c\ub2e4. \ud53c\uce20\ubc84\uadf8 \ud30c\uc774\ub9ac\uce20\uc640\uc758 \ub9ac\uadf8 \ucc54\ud53c\uc5b8\uc2ed \uc2dc\ub9ac\uc988\uc5d0\uc11c\ub294 2\ud328\ub97c \ud588\uc9c0\ub9cc, \ud300\uc740 33\ub144 \ub9cc\uc5d0 \uc560\ud2c0\ub79c\ud0c0\ub85c \uc774\uc804\ud55c \ud6c4 \ucc98\uc74c\uc73c\ub85c \ub9ac\uadf8\uc6b0\uc2b9\uc744 \ud55c\ub2e4. \ubbf8\ub124\uc18c\ud0c0 \ud2b8\uc708\uc2a4\uc640\uc758 \uc6d4\ub4dc\uc2dc\ub9ac\uc988\uc5d0\uc11c\ub294 1\uc2b9 1\ud328\ub97c \ud588\uc9c0\ub9cc, \ud300\uc740 3\uc2b9 4\ud328\ub85c \uc544\uc27d\uac8c \ud328\ud1f4\ud558\uace0 \ub9cc\ub2e4. \uc2dc\uc98c\uc774 \ub05d\ub09c \ud6c4\uc5d0\ub294 \ucc98\uc74c\uc73c\ub85c \uc0ac\uc774 \uc601 \uc0c1\uc744 \uc218\uc0c1\ud588\ub2e4. 1992\ub144\uc5d0\ub294 1954\ub144, 1955\ub144\uc5d0 2\ub144\uc5f0\uc18d\uc73c\ub85c \uc62c\uc2a4\ud0c0\uc804\uc5d0\uc11c \uc120\ubc1c\ub85c \ucd9c\uc804\ud55c \ub85c\ube48 \ub85c\ubc84\uce20 \uc774\ub798 \ucc98\uc74c\uc73c\ub85c 2\ub144\uc5f0\uc18d\uc73c\ub85c \uc62c\uc2a4\ud0c0\uc804 \uc120\ubc1c\ub85c \ubf51\ud614\uc9c0\ub9cc, 1\ud68c 1\uc544\uc6c3\ubd80\ud130 7\uc5f0\uc18d\uc73c\ub85c \uc548\ud0c0\ub97c \ub9de\ub294 \ub4f1 2\ud68c\ub3c4\uc911\uae4c\uc9c0 5\uc2e4\uc810\uc758 \ub09c\uc870\ub97c \ubcf4\uc600\ub2e4. \uc2dc\uc98c\uc5d0\uc11c\ub294 20\uc2b9 8\ud328, \ubc29\uc5b4\uc728 2.76\uacfc \ub9ac\uadf8 \ucd5c\ub2e4\uc778 5\ubc88\uc758 \uc644\ubd09\uc744 \uae30\ub85d\ud558\uba70, \ud300\uc758 2\ub144\uc5f0\uc18d \uc9c0\uad6c\uc6b0\uc2b9\uc5d0 \uacf5\ud5cc\ud55c\ub2e4. \uc804\ub144\uacfc \uac19\uc774 \ud53c\uce20\ubc84\uadf8 \ud30c\uc774\ub9ac\uce20\uc640 \ub300\uacb0\uc774 \ub418\uc5c8\ub358 \ub9ac\uadf8 \ucc54\ud53c\uc5b8\uc2ed \uc2dc\ub9ac\uc988\uc5d0\uc11c\ub294 1\ucc28\uc804\uc5d0 \uc120\ubc1c\ub4f1\ud310\ud558\uc5ec 4\uc548\ud0c0 \ubb34\uc0ac\uad6c 1\uc2e4\uc810\uc73c\ub85c \uc644\ud22c\uc2b9\uc744 \ud588\uc9c0\ub9cc, \ud300\uc740 3\uc2b9 4\ud328\ub85c \ud328\ud1f4\ud55c\ub2e4. \uc0ac\uc774 \uc601 \uc0c1\uc758 \ud22c\ud45c\uc5d0\uc11c\ub294 \uc2dc\uce74\uace0 \ucef5\uc2a4\uc758 \uadf8\ub808\uadf8 \ub9e4\ub355\uc2a4\uc5d0 \uc774\uc5b4 2\uc704\uac00 \ub410\ub2e4."} {"question": "\uc720\uc9c0 \uc815\ube44\ub97c \uc704\ud574 \uc0ac\uc138\ud56d\uc5d0 \uc815\ubc15\ud588\ub358 \ub77c\uc2a8\uc740 \ub2e4\uc2dc \ub3d9\ud574\ub85c \ub3cc\uc544\uac00 \ub204\uad6c\uc640 \ud568\uaed8 \uc791\uc804\uc744 \uc218\ud589\ud588\ub098?", "paragraph": "1967\ub144 8\uc6d4, \uc11c\ud0dc\ud3c9\uc591\uc5d0 \ubc30\uce58\ub41c \ub77c\uc2a8\uc740 1968\ub144 1\uc6d4 6\uc77c\ubd80\ud130 10\uc77c\uae4c\uc9c0 \ub300\uc7a0\uc218\ud568 \ud6c8\ub828 \uc9c4\ud589\uc744 \ub2f4\ub2f9\ud588\ub2e4. 1\uc6d4 15\uc77c\ubd80\ud130 21\uc77c\uae4c\uc9c0 \ud64d\ucf69\uc5d0 \uc815\ubc15\ud588\uace0, 21\uc77c\uc5d0 \ud0c0\uc774\uc644\uc758 \uce74\uc624\uc29d\uc73c\ub85c \ud5a5\ud588\ub2e4. \uce74\uc624\uc29d\uc5d0 \ub3c4\ucc29\ud55c \uc9c0 5\uc77c \ud6c4\uc5d0 \ub77c\uc2a8\uc740 \uae09\ud788 \ud478\uc5d0\ube14\ub85c \ud638 \ub0a9\uce58 \uc0ac\uac74\uc774 \ubc8c\uc5b4\uc9c4 \ub3d9\ud574\ub85c \uae09\ud30c\ub418\uc5b4 \uba3c\uc800 \uc640 \uc788\ub358 \ub2e4\ub978 \ud568\ub300\uc5d0 \ud569\ub958\ud588\ub2e4. \ub77c\uc2a8 \ud638\ub294 71\uae30\ub3d9\ud568\ub300(TF 71)\uc758 23 \uad6c\ucd95\ud568\uc804\ub300\ub85c \uc9c0\uc815\ub418\uc5b4 1\uc6d4 31\uc77c\ubd80\ud130 3\uc6d4 2\uc77c\uae4c\uc9c0 \ub3d9\ud574\uc5d0\uc11c \uc791\uc804\ud588\ub2e4. 3\uc6d4 2\uc77c ~ 12\uc77c\uc5d0\ub294 \uc720\uc9c0 \uc815\ube44\ub97c \uc704\ud574 \uc0ac\uc138\ubcf4 \ud56d\uc5d0 \uc815\ubc15\ud588\uace0, 3\uc6d4 13\uc77c\uc5d0 \ub2e4\uc2dc \ub3d9\ud574\ub85c \ub3cc\uc544\uac00 21\uc77c\uae4c\uc9c0 \ubcfc\ud2f0\ubaa8\uc5b4\uae09 \uc21c\uc591\ud568 \uce94\ubc84\ub77c\uc640 \ud568\uaed8 \uc791\uc804\ud588\uc73c\uba70, 3\uc6d4 22\uc77c\uacfc 23\uc77c\uc5d0 \uac78\uccd0 \uc77c\ubcf8\uc758 \uc0ac\uc138\ubcf4 \ud56d\uc5d0\uc11c \ubcf5\uadc0\ud560 \uc900\ube44\ub97c \ud588\ub2e4.", "answer": "\uce94\ubc84\ub77c", "sentence": "3\uc6d4 2\uc77c ~ 12\uc77c\uc5d0\ub294 \uc720\uc9c0 \uc815\ube44\ub97c \uc704\ud574 \uc0ac\uc138\ubcf4 \ud56d\uc5d0 \uc815\ubc15\ud588\uace0, 3\uc6d4 13\uc77c\uc5d0 \ub2e4\uc2dc \ub3d9\ud574\ub85c \ub3cc\uc544\uac00 21\uc77c\uae4c\uc9c0 \ubcfc\ud2f0\ubaa8\uc5b4\uae09 \uc21c\uc591\ud568 \uce94\ubc84\ub77c \uc640 \ud568\uaed8 \uc791\uc804\ud588\uc73c\uba70, 3\uc6d4 22\uc77c\uacfc 23\uc77c\uc5d0 \uac78\uccd0 \uc77c\ubcf8\uc758 \uc0ac\uc138\ubcf4 \ud56d\uc5d0\uc11c \ubcf5\uadc0\ud560 \uc900\ube44\ub97c \ud588\ub2e4.", "paragraph_sentence": "1967\ub144 8\uc6d4, \uc11c\ud0dc\ud3c9\uc591\uc5d0 \ubc30\uce58\ub41c \ub77c\uc2a8\uc740 1968\ub144 1\uc6d4 6\uc77c\ubd80\ud130 10\uc77c\uae4c\uc9c0 \ub300\uc7a0\uc218\ud568 \ud6c8\ub828 \uc9c4\ud589\uc744 \ub2f4\ub2f9\ud588\ub2e4. 1\uc6d4 15\uc77c\ubd80\ud130 21\uc77c\uae4c\uc9c0 \ud64d\ucf69\uc5d0 \uc815\ubc15\ud588\uace0, 21\uc77c\uc5d0 \ud0c0\uc774\uc644\uc758 \uce74\uc624\uc29d\uc73c\ub85c \ud5a5\ud588\ub2e4. \uce74\uc624\uc29d\uc5d0 \ub3c4\ucc29\ud55c \uc9c0 5\uc77c \ud6c4\uc5d0 \ub77c\uc2a8\uc740 \uae09\ud788 \ud478\uc5d0\ube14\ub85c \ud638 \ub0a9\uce58 \uc0ac\uac74\uc774 \ubc8c\uc5b4\uc9c4 \ub3d9\ud574\ub85c \uae09\ud30c\ub418\uc5b4 \uba3c\uc800 \uc640 \uc788\ub358 \ub2e4\ub978 \ud568\ub300\uc5d0 \ud569\ub958\ud588\ub2e4. \ub77c\uc2a8 \ud638\ub294 71\uae30\ub3d9\ud568\ub300(TF 71)\uc758 23 \uad6c\ucd95\ud568\uc804\ub300\ub85c \uc9c0\uc815\ub418\uc5b4 1\uc6d4 31\uc77c\ubd80\ud130 3\uc6d4 2\uc77c\uae4c\uc9c0 \ub3d9\ud574\uc5d0\uc11c \uc791\uc804\ud588\ub2e4. 3\uc6d4 2\uc77c ~ 12\uc77c\uc5d0\ub294 \uc720\uc9c0 \uc815\ube44\ub97c \uc704\ud574 \uc0ac\uc138\ubcf4 \ud56d\uc5d0 \uc815\ubc15\ud588\uace0, 3\uc6d4 13\uc77c\uc5d0 \ub2e4\uc2dc \ub3d9\ud574\ub85c \ub3cc\uc544\uac00 21\uc77c\uae4c\uc9c0 \ubcfc\ud2f0\ubaa8\uc5b4\uae09 \uc21c\uc591\ud568 \uce94\ubc84\ub77c \uc640 \ud568\uaed8 \uc791\uc804\ud588\uc73c\uba70, 3\uc6d4 22\uc77c\uacfc 23\uc77c\uc5d0 \uac78\uccd0 \uc77c\ubcf8\uc758 \uc0ac\uc138\ubcf4 \ud56d\uc5d0\uc11c \ubcf5\uadc0\ud560 \uc900\ube44\ub97c \ud588\ub2e4. ", "paragraph_answer": "1967\ub144 8\uc6d4, \uc11c\ud0dc\ud3c9\uc591\uc5d0 \ubc30\uce58\ub41c \ub77c\uc2a8\uc740 1968\ub144 1\uc6d4 6\uc77c\ubd80\ud130 10\uc77c\uae4c\uc9c0 \ub300\uc7a0\uc218\ud568 \ud6c8\ub828 \uc9c4\ud589\uc744 \ub2f4\ub2f9\ud588\ub2e4. 1\uc6d4 15\uc77c\ubd80\ud130 21\uc77c\uae4c\uc9c0 \ud64d\ucf69\uc5d0 \uc815\ubc15\ud588\uace0, 21\uc77c\uc5d0 \ud0c0\uc774\uc644\uc758 \uce74\uc624\uc29d\uc73c\ub85c \ud5a5\ud588\ub2e4. \uce74\uc624\uc29d\uc5d0 \ub3c4\ucc29\ud55c \uc9c0 5\uc77c \ud6c4\uc5d0 \ub77c\uc2a8\uc740 \uae09\ud788 \ud478\uc5d0\ube14\ub85c \ud638 \ub0a9\uce58 \uc0ac\uac74\uc774 \ubc8c\uc5b4\uc9c4 \ub3d9\ud574\ub85c \uae09\ud30c\ub418\uc5b4 \uba3c\uc800 \uc640 \uc788\ub358 \ub2e4\ub978 \ud568\ub300\uc5d0 \ud569\ub958\ud588\ub2e4. \ub77c\uc2a8 \ud638\ub294 71\uae30\ub3d9\ud568\ub300(TF 71)\uc758 23 \uad6c\ucd95\ud568\uc804\ub300\ub85c \uc9c0\uc815\ub418\uc5b4 1\uc6d4 31\uc77c\ubd80\ud130 3\uc6d4 2\uc77c\uae4c\uc9c0 \ub3d9\ud574\uc5d0\uc11c \uc791\uc804\ud588\ub2e4. 3\uc6d4 2\uc77c ~ 12\uc77c\uc5d0\ub294 \uc720\uc9c0 \uc815\ube44\ub97c \uc704\ud574 \uc0ac\uc138\ubcf4 \ud56d\uc5d0 \uc815\ubc15\ud588\uace0, 3\uc6d4 13\uc77c\uc5d0 \ub2e4\uc2dc \ub3d9\ud574\ub85c \ub3cc\uc544\uac00 21\uc77c\uae4c\uc9c0 \ubcfc\ud2f0\ubaa8\uc5b4\uae09 \uc21c\uc591\ud568 \uce94\ubc84\ub77c \uc640 \ud568\uaed8 \uc791\uc804\ud588\uc73c\uba70, 3\uc6d4 22\uc77c\uacfc 23\uc77c\uc5d0 \uac78\uccd0 \uc77c\ubcf8\uc758 \uc0ac\uc138\ubcf4 \ud56d\uc5d0\uc11c \ubcf5\uadc0\ud560 \uc900\ube44\ub97c \ud588\ub2e4.", "sentence_answer": "3\uc6d4 2\uc77c ~ 12\uc77c\uc5d0\ub294 \uc720\uc9c0 \uc815\ube44\ub97c \uc704\ud574 \uc0ac\uc138\ubcf4 \ud56d\uc5d0 \uc815\ubc15\ud588\uace0, 3\uc6d4 13\uc77c\uc5d0 \ub2e4\uc2dc \ub3d9\ud574\ub85c \ub3cc\uc544\uac00 21\uc77c\uae4c\uc9c0 \ubcfc\ud2f0\ubaa8\uc5b4\uae09 \uc21c\uc591\ud568 \uce94\ubc84\ub77c \uc640 \ud568\uaed8 \uc791\uc804\ud588\uc73c\uba70, 3\uc6d4 22\uc77c\uacfc 23\uc77c\uc5d0 \uac78\uccd0 \uc77c\ubcf8\uc758 \uc0ac\uc138\ubcf4 \ud56d\uc5d0\uc11c \ubcf5\uadc0\ud560 \uc900\ube44\ub97c \ud588\ub2e4.", "paragraph_id": "6499823-2-1", "paragraph_question": "question: \uc720\uc9c0 \uc815\ube44\ub97c \uc704\ud574 \uc0ac\uc138\ud56d\uc5d0 \uc815\ubc15\ud588\ub358 \ub77c\uc2a8\uc740 \ub2e4\uc2dc \ub3d9\ud574\ub85c \ub3cc\uc544\uac00 \ub204\uad6c\uc640 \ud568\uaed8 \uc791\uc804\uc744 \uc218\ud589\ud588\ub098?, context: 1967\ub144 8\uc6d4, \uc11c\ud0dc\ud3c9\uc591\uc5d0 \ubc30\uce58\ub41c \ub77c\uc2a8\uc740 1968\ub144 1\uc6d4 6\uc77c\ubd80\ud130 10\uc77c\uae4c\uc9c0 \ub300\uc7a0\uc218\ud568 \ud6c8\ub828 \uc9c4\ud589\uc744 \ub2f4\ub2f9\ud588\ub2e4. 1\uc6d4 15\uc77c\ubd80\ud130 21\uc77c\uae4c\uc9c0 \ud64d\ucf69\uc5d0 \uc815\ubc15\ud588\uace0, 21\uc77c\uc5d0 \ud0c0\uc774\uc644\uc758 \uce74\uc624\uc29d\uc73c\ub85c \ud5a5\ud588\ub2e4. \uce74\uc624\uc29d\uc5d0 \ub3c4\ucc29\ud55c \uc9c0 5\uc77c \ud6c4\uc5d0 \ub77c\uc2a8\uc740 \uae09\ud788 \ud478\uc5d0\ube14\ub85c \ud638 \ub0a9\uce58 \uc0ac\uac74\uc774 \ubc8c\uc5b4\uc9c4 \ub3d9\ud574\ub85c \uae09\ud30c\ub418\uc5b4 \uba3c\uc800 \uc640 \uc788\ub358 \ub2e4\ub978 \ud568\ub300\uc5d0 \ud569\ub958\ud588\ub2e4. \ub77c\uc2a8 \ud638\ub294 71\uae30\ub3d9\ud568\ub300(TF 71)\uc758 23 \uad6c\ucd95\ud568\uc804\ub300\ub85c \uc9c0\uc815\ub418\uc5b4 1\uc6d4 31\uc77c\ubd80\ud130 3\uc6d4 2\uc77c\uae4c\uc9c0 \ub3d9\ud574\uc5d0\uc11c \uc791\uc804\ud588\ub2e4. 3\uc6d4 2\uc77c ~ 12\uc77c\uc5d0\ub294 \uc720\uc9c0 \uc815\ube44\ub97c \uc704\ud574 \uc0ac\uc138\ubcf4 \ud56d\uc5d0 \uc815\ubc15\ud588\uace0, 3\uc6d4 13\uc77c\uc5d0 \ub2e4\uc2dc \ub3d9\ud574\ub85c \ub3cc\uc544\uac00 21\uc77c\uae4c\uc9c0 \ubcfc\ud2f0\ubaa8\uc5b4\uae09 \uc21c\uc591\ud568 \uce94\ubc84\ub77c\uc640 \ud568\uaed8 \uc791\uc804\ud588\uc73c\uba70, 3\uc6d4 22\uc77c\uacfc 23\uc77c\uc5d0 \uac78\uccd0 \uc77c\ubcf8\uc758 \uc0ac\uc138\ubcf4 \ud56d\uc5d0\uc11c \ubcf5\uadc0\ud560 \uc900\ube44\ub97c \ud588\ub2e4."} {"question": "\ud574\ub2ec\uc774 \uadc0\ud574\uc9c0\uba74\uc11c \uac00\uaca9\uc774 \uc0c1\uc2b9\ud55c \uac83\uc740?", "paragraph": "\uacb0\uad6d \ud574\ub2ec \uac1c\uccb4\uc218\uac00 \ub108\ubb34 \uac10\uc18c\ud588\uae30 \ub54c\ubb38\uc5d0, \uc0c1\uc5c5\uc801 \uc218\ub835\uc774 \ubd88\uac00\ub2a5\ud55c \uc9c0\uacbd\uc5d0 \uc774\ub974\ub800\ub2e4. \uc54c\ub958\uc0e8 \uc5f4\ub3c4\uc5d0\uc11c\uc758 \ud574\ub2ec \uc0ac\ub0e5\uc740 \ub7ec\uc2dc\uc544-\uc544\uba54\ub9ac\uce74 \ud68c\uc0ac\uac00 \ubcf4\uc874 \uce21\uc815\uc744 \ud55c 1808\ub144 \uc774\ud6c4 \uc911\ub2e8\ub418\uc5c8\ub2e4. 1834\ub144\uc5d0\ub294 \ud68c\uc0ac \uce21\uc5d0\uc11c \ucd94\uac00\uc801 \uaddc\uc81c\ub97c \uc694\uccad\ud588\ub2e4. 1867\ub144\uc5d0 \ub7ec\uc2dc\uc544\uac00 \ubbf8\uad6d\uc5d0 \uc54c\ub798\uc2a4\uce74\ub97c \ub9e4\uac01\ud588\uc744 \ub2f9\uc2dc \uc54c\ub798\uc2a4\uce74\uc758 \ud574\ub2ec \uac1c\uccb4\uad70\uc740 100,000 \ub9c8\ub9ac \uc774\uc0c1\uc73c\ub85c \ub418\uc0b4\uc544\ub098 \uc788\uc5c8\ub2e4. \ud558\uc9c0\ub9cc \ubbf8\uad6d\uc778\ub4e4\uc774 \uc0ac\ub0e5\uc744 \uc7ac\uac1c\ud558\uc790 \ud574\ub2ec\uc740 \ub610\ub2e4\uc2dc \ube60\ub974\uac8c \uc528\uac00 \ub9d0\ub790\ub2e4. \ud574\ub2ec\uc774 \uadc0\ud574\uc9d0\uc5d0 \ub530\ub77c \ud574\ub2ec \ubaa8\ud53c \uac12\ub3c4 \uc0c1\uc2b9\ud588\ub2e4. 1880\ub144\ub300\uc5d0 \ub7f0\ub358 \uc2dc\uc7a5\uc5d0\uc11c \ud574\ub2ec \uac00\uc8fd\uc758 \uac00\uaca9\uc740 105~165 \ub2ec\ub7ec\uc600\uc73c\ub098, 1903\ub144\uc5d0\ub294 1,125 \ub2ec\ub7ec\uae4c\uc9c0 \uce58\uc19f\uc558\ub2e4. 1911\ub144, \ub7ec\uc2dc\uc544, \uc77c\ubcf8, \ub300\uc601\uc81c\uad6d(\uce90\ub098\ub2e4), \ubbf8\uad6d\uc740 \ud574\ub2ec \ubc0f \ubb3c\uac1c\ub958 \ubcf4\ud638 \uad6d\uc81c\uc870\uc57d\uc744 \uccb4\uacb0\ud558\uace0, \ud574\ub2ec \uc218\ub835 \uacb0\uacfc\ubb3c\uc5d0 \ub300\ud55c \uc9c0\ubd88 \uc911\uc9c0\ub97c \uacb0\uc758\ud588\ub2e4. \uc774\ub54c \uc57c\uc0dd \ud574\ub2ec\uc740 1,000~2,000\uc5ec \ub9c8\ub9ac\uac00 \ub0a8\uc544 \uc788\uc744 \ubfd0\uc73c\ub85c, \ub9ce\uc740 \uc0ac\ub78c\ub4e4\uc740 \uc774 \uc885\uc774 \uba78\uc885\ub420 \uac83\uc774\ub77c\uace0 \uc0dd\uac01\ud588\ub2e4.", "answer": "\ud574\ub2ec \ubaa8\ud53c", "sentence": "\ud574\ub2ec\uc774 \uadc0\ud574\uc9d0\uc5d0 \ub530\ub77c \ud574\ub2ec \ubaa8\ud53c \uac12\ub3c4 \uc0c1\uc2b9\ud588\ub2e4.", "paragraph_sentence": "\uacb0\uad6d \ud574\ub2ec \uac1c\uccb4\uc218\uac00 \ub108\ubb34 \uac10\uc18c\ud588\uae30 \ub54c\ubb38\uc5d0, \uc0c1\uc5c5\uc801 \uc218\ub835\uc774 \ubd88\uac00\ub2a5\ud55c \uc9c0\uacbd\uc5d0 \uc774\ub974\ub800\ub2e4. \uc54c\ub958\uc0e8 \uc5f4\ub3c4\uc5d0\uc11c\uc758 \ud574\ub2ec \uc0ac\ub0e5\uc740 \ub7ec\uc2dc\uc544-\uc544\uba54\ub9ac\uce74 \ud68c\uc0ac\uac00 \ubcf4\uc874 \uce21\uc815\uc744 \ud55c 1808\ub144 \uc774\ud6c4 \uc911\ub2e8\ub418\uc5c8\ub2e4. 1834\ub144\uc5d0\ub294 \ud68c\uc0ac \uce21\uc5d0\uc11c \ucd94\uac00\uc801 \uaddc\uc81c\ub97c \uc694\uccad\ud588\ub2e4. 1867\ub144\uc5d0 \ub7ec\uc2dc\uc544\uac00 \ubbf8\uad6d\uc5d0 \uc54c\ub798\uc2a4\uce74\ub97c \ub9e4\uac01\ud588\uc744 \ub2f9\uc2dc \uc54c\ub798\uc2a4\uce74\uc758 \ud574\ub2ec \uac1c\uccb4\uad70\uc740 100,000 \ub9c8\ub9ac \uc774\uc0c1\uc73c\ub85c \ub418\uc0b4\uc544\ub098 \uc788\uc5c8\ub2e4. \ud558\uc9c0\ub9cc \ubbf8\uad6d\uc778\ub4e4\uc774 \uc0ac\ub0e5\uc744 \uc7ac\uac1c\ud558\uc790 \ud574\ub2ec\uc740 \ub610\ub2e4\uc2dc \ube60\ub974\uac8c \uc528\uac00 \ub9d0\ub790\ub2e4. \ud574\ub2ec\uc774 \uadc0\ud574\uc9d0\uc5d0 \ub530\ub77c \ud574\ub2ec \ubaa8\ud53c \uac12\ub3c4 \uc0c1\uc2b9\ud588\ub2e4. 1880\ub144\ub300\uc5d0 \ub7f0\ub358 \uc2dc\uc7a5\uc5d0\uc11c \ud574\ub2ec \uac00\uc8fd\uc758 \uac00\uaca9\uc740 105~165 \ub2ec\ub7ec\uc600\uc73c\ub098, 1903\ub144\uc5d0\ub294 1,125 \ub2ec\ub7ec\uae4c\uc9c0 \uce58\uc19f\uc558\ub2e4. 1911\ub144, \ub7ec\uc2dc\uc544, \uc77c\ubcf8, \ub300\uc601\uc81c\uad6d(\uce90\ub098\ub2e4), \ubbf8\uad6d\uc740 \ud574\ub2ec \ubc0f \ubb3c\uac1c\ub958 \ubcf4\ud638 \uad6d\uc81c\uc870\uc57d\uc744 \uccb4\uacb0\ud558\uace0, \ud574\ub2ec \uc218\ub835 \uacb0\uacfc\ubb3c\uc5d0 \ub300\ud55c \uc9c0\ubd88 \uc911\uc9c0\ub97c \uacb0\uc758\ud588\ub2e4. \uc774\ub54c \uc57c\uc0dd \ud574\ub2ec\uc740 1,000~2,000\uc5ec \ub9c8\ub9ac\uac00 \ub0a8\uc544 \uc788\uc744 \ubfd0\uc73c\ub85c, \ub9ce\uc740 \uc0ac\ub78c\ub4e4\uc740 \uc774 \uc885\uc774 \uba78\uc885\ub420 \uac83\uc774\ub77c\uace0 \uc0dd\uac01\ud588\ub2e4.", "paragraph_answer": "\uacb0\uad6d \ud574\ub2ec \uac1c\uccb4\uc218\uac00 \ub108\ubb34 \uac10\uc18c\ud588\uae30 \ub54c\ubb38\uc5d0, \uc0c1\uc5c5\uc801 \uc218\ub835\uc774 \ubd88\uac00\ub2a5\ud55c \uc9c0\uacbd\uc5d0 \uc774\ub974\ub800\ub2e4. \uc54c\ub958\uc0e8 \uc5f4\ub3c4\uc5d0\uc11c\uc758 \ud574\ub2ec \uc0ac\ub0e5\uc740 \ub7ec\uc2dc\uc544-\uc544\uba54\ub9ac\uce74 \ud68c\uc0ac\uac00 \ubcf4\uc874 \uce21\uc815\uc744 \ud55c 1808\ub144 \uc774\ud6c4 \uc911\ub2e8\ub418\uc5c8\ub2e4. 1834\ub144\uc5d0\ub294 \ud68c\uc0ac \uce21\uc5d0\uc11c \ucd94\uac00\uc801 \uaddc\uc81c\ub97c \uc694\uccad\ud588\ub2e4. 1867\ub144\uc5d0 \ub7ec\uc2dc\uc544\uac00 \ubbf8\uad6d\uc5d0 \uc54c\ub798\uc2a4\uce74\ub97c \ub9e4\uac01\ud588\uc744 \ub2f9\uc2dc \uc54c\ub798\uc2a4\uce74\uc758 \ud574\ub2ec \uac1c\uccb4\uad70\uc740 100,000 \ub9c8\ub9ac \uc774\uc0c1\uc73c\ub85c \ub418\uc0b4\uc544\ub098 \uc788\uc5c8\ub2e4. \ud558\uc9c0\ub9cc \ubbf8\uad6d\uc778\ub4e4\uc774 \uc0ac\ub0e5\uc744 \uc7ac\uac1c\ud558\uc790 \ud574\ub2ec\uc740 \ub610\ub2e4\uc2dc \ube60\ub974\uac8c \uc528\uac00 \ub9d0\ub790\ub2e4. \ud574\ub2ec\uc774 \uadc0\ud574\uc9d0\uc5d0 \ub530\ub77c \ud574\ub2ec \ubaa8\ud53c \uac12\ub3c4 \uc0c1\uc2b9\ud588\ub2e4. 1880\ub144\ub300\uc5d0 \ub7f0\ub358 \uc2dc\uc7a5\uc5d0\uc11c \ud574\ub2ec \uac00\uc8fd\uc758 \uac00\uaca9\uc740 105~165 \ub2ec\ub7ec\uc600\uc73c\ub098, 1903\ub144\uc5d0\ub294 1,125 \ub2ec\ub7ec\uae4c\uc9c0 \uce58\uc19f\uc558\ub2e4. 1911\ub144, \ub7ec\uc2dc\uc544, \uc77c\ubcf8, \ub300\uc601\uc81c\uad6d(\uce90\ub098\ub2e4), \ubbf8\uad6d\uc740 \ud574\ub2ec \ubc0f \ubb3c\uac1c\ub958 \ubcf4\ud638 \uad6d\uc81c\uc870\uc57d\uc744 \uccb4\uacb0\ud558\uace0, \ud574\ub2ec \uc218\ub835 \uacb0\uacfc\ubb3c\uc5d0 \ub300\ud55c \uc9c0\ubd88 \uc911\uc9c0\ub97c \uacb0\uc758\ud588\ub2e4. \uc774\ub54c \uc57c\uc0dd \ud574\ub2ec\uc740 1,000~2,000\uc5ec \ub9c8\ub9ac\uac00 \ub0a8\uc544 \uc788\uc744 \ubfd0\uc73c\ub85c, \ub9ce\uc740 \uc0ac\ub78c\ub4e4\uc740 \uc774 \uc885\uc774 \uba78\uc885\ub420 \uac83\uc774\ub77c\uace0 \uc0dd\uac01\ud588\ub2e4.", "sentence_answer": "\ud574\ub2ec\uc774 \uadc0\ud574\uc9d0\uc5d0 \ub530\ub77c \ud574\ub2ec \ubaa8\ud53c \uac12\ub3c4 \uc0c1\uc2b9\ud588\ub2e4.", "paragraph_id": "6570452-17-2", "paragraph_question": "question: \ud574\ub2ec\uc774 \uadc0\ud574\uc9c0\uba74\uc11c \uac00\uaca9\uc774 \uc0c1\uc2b9\ud55c \uac83\uc740?, context: \uacb0\uad6d \ud574\ub2ec \uac1c\uccb4\uc218\uac00 \ub108\ubb34 \uac10\uc18c\ud588\uae30 \ub54c\ubb38\uc5d0, \uc0c1\uc5c5\uc801 \uc218\ub835\uc774 \ubd88\uac00\ub2a5\ud55c \uc9c0\uacbd\uc5d0 \uc774\ub974\ub800\ub2e4. \uc54c\ub958\uc0e8 \uc5f4\ub3c4\uc5d0\uc11c\uc758 \ud574\ub2ec \uc0ac\ub0e5\uc740 \ub7ec\uc2dc\uc544-\uc544\uba54\ub9ac\uce74 \ud68c\uc0ac\uac00 \ubcf4\uc874 \uce21\uc815\uc744 \ud55c 1808\ub144 \uc774\ud6c4 \uc911\ub2e8\ub418\uc5c8\ub2e4. 1834\ub144\uc5d0\ub294 \ud68c\uc0ac \uce21\uc5d0\uc11c \ucd94\uac00\uc801 \uaddc\uc81c\ub97c \uc694\uccad\ud588\ub2e4. 1867\ub144\uc5d0 \ub7ec\uc2dc\uc544\uac00 \ubbf8\uad6d\uc5d0 \uc54c\ub798\uc2a4\uce74\ub97c \ub9e4\uac01\ud588\uc744 \ub2f9\uc2dc \uc54c\ub798\uc2a4\uce74\uc758 \ud574\ub2ec \uac1c\uccb4\uad70\uc740 100,000 \ub9c8\ub9ac \uc774\uc0c1\uc73c\ub85c \ub418\uc0b4\uc544\ub098 \uc788\uc5c8\ub2e4. \ud558\uc9c0\ub9cc \ubbf8\uad6d\uc778\ub4e4\uc774 \uc0ac\ub0e5\uc744 \uc7ac\uac1c\ud558\uc790 \ud574\ub2ec\uc740 \ub610\ub2e4\uc2dc \ube60\ub974\uac8c \uc528\uac00 \ub9d0\ub790\ub2e4. \ud574\ub2ec\uc774 \uadc0\ud574\uc9d0\uc5d0 \ub530\ub77c \ud574\ub2ec \ubaa8\ud53c \uac12\ub3c4 \uc0c1\uc2b9\ud588\ub2e4. 1880\ub144\ub300\uc5d0 \ub7f0\ub358 \uc2dc\uc7a5\uc5d0\uc11c \ud574\ub2ec \uac00\uc8fd\uc758 \uac00\uaca9\uc740 105~165 \ub2ec\ub7ec\uc600\uc73c\ub098, 1903\ub144\uc5d0\ub294 1,125 \ub2ec\ub7ec\uae4c\uc9c0 \uce58\uc19f\uc558\ub2e4. 1911\ub144, \ub7ec\uc2dc\uc544, \uc77c\ubcf8, \ub300\uc601\uc81c\uad6d(\uce90\ub098\ub2e4), \ubbf8\uad6d\uc740 \ud574\ub2ec \ubc0f \ubb3c\uac1c\ub958 \ubcf4\ud638 \uad6d\uc81c\uc870\uc57d\uc744 \uccb4\uacb0\ud558\uace0, \ud574\ub2ec \uc218\ub835 \uacb0\uacfc\ubb3c\uc5d0 \ub300\ud55c \uc9c0\ubd88 \uc911\uc9c0\ub97c \uacb0\uc758\ud588\ub2e4. \uc774\ub54c \uc57c\uc0dd \ud574\ub2ec\uc740 1,000~2,000\uc5ec \ub9c8\ub9ac\uac00 \ub0a8\uc544 \uc788\uc744 \ubfd0\uc73c\ub85c, \ub9ce\uc740 \uc0ac\ub78c\ub4e4\uc740 \uc774 \uc885\uc774 \uba78\uc885\ub420 \uac83\uc774\ub77c\uace0 \uc0dd\uac01\ud588\ub2e4."} {"question": "\ud504\ub79c\uc2dc\uc2a4 \ud30c\ucee4 \uc694\ucee4\uac00 \uc8fc\uc7a5\ud55c \uacf5\uc0b0\uc8fc\uc758\uc640 \ub124\uc624 \ud30c\uc2dc\uc2a4\ud2b8 \uac04\uc758 \ub3d9\ub9f9\uc740 \ubb34\uc5c7\uc778\uac00?", "paragraph": "\uc81c3\uc758 \uc704\uce58\ub77c\ub294 \uc6a9\uc5b4\ub294 \uc720\ub7fd\uc5d0\uc11c \uc81c3\uc758 \uc704\uce58\uc758 \uc120\uad6c\uc790\ub85c \uac04\uc8fc\ub418\ub294 \ubbfc\uc871 \ubcfc\uc170\ube44\uc998(\uadf9\uc6b0 \uad6d\uc218\uc8fc\uc758\uc640 \uadf9\uc88c \ubcfc\uc170\ube44\ud0a4\uc801 \uc0ac\ud68c\uc8fc\uc758\uc758 \ud63c\ud569) \uadf8\ub9ac\uace0 \uc288\ud2b8\ub77c\uc11c\uc8fc\uc758(\ubbfc\uc871 \uc0ac\ud68c\uc8fc\uc758\uc758 \uc0ac\ud68c\uc8fc\uc758\uc801 \ud615\ud0dc\ub85c\uc11c \ub178\ub3d9\uc790\uc640 \uae09\uc9c4\uc801\uc778 \ub300\uc911\ud589\ub3d9 \ub4f1\uc5d0 \uae30\ubc18\ud588\uace0, \ub098\uce58\ub2f9 \ub0b4\uc5d0\uc11c \uc88c\uc775\uc774\uc600\uc9c0\ub9cc, 1934\ub144 \uc7a5\uac80\uc758 \ubc24 \uc0ac\uac74\uc5d0\uc11c \uc219\uccad\ub41c\ub2e4.) \uac19\uc740 \uc815\uce58 \uc6b4\ub3d9\uc5d0\uc11c \ube44\ub86f\ub418\uc5c8\ub2e4. \ub124\uc624 \ud30c\uc2dc\uc2a4\ud2b8, \ub124\uc624 \ub098\uce58 \uc791\uac00\uc778 \ud504\ub79c\uc2dc\uc2a4 \ud30c\ucee4 \uc694\ud0a4(Francis Parker Yockey)\ub294 \uacf5\uc0b0\uc8fc\uc758\uc790\uc640 \ud30c\uc2dc\uc2a4\ud2b8 \uc0ac\uc774\uc758 \ubcf8\uc9c8\uc0c1 \ubc18\ubbf8, \ubc18\uc720\ub300\uc8fc\uc758\uc801 \ud2b9\uc9d5\uc744 \uac00\uc9c0\ub294 \ub3d9\ub9f9\uc744 \uc81c\uc548\ud588\uace0 \uc774\ub97c \uc801\uac08\uc0c9 \ub3d9\ub9f9(Red-Brown Alliance, \uc801\uc0c9\uc740 \ubcfc\uc170\ube44\uc998\uc744 \uac08\uc0c9\uc740 \ub098\uce58\ub97c \uc0c1\uc9d5)\uc774\ub77c\uace0 \ubd88\ub800\ub2e4. \uc694\ud0a4\ub294 \uc81c3\uc138\uacc4\uc758 \ud574\ubc29 \uc6b4\ub3d9\uc744 \uc9c0\uc9c0\ud588\ub2e4. \ubbfc\uc871\ud574\ubc29\ud22c\uc7c1\uc774 \uc778\uc885\ubd84\ub9ac\uc640 \ubbfc\uc871\ub2e4\uc6d0\uc8fc\uc758\uc5d0 \ub3c4\uc6c0\uc774 \ub418\ub294\uac78\ub85c \uac04\uc8fc\ud588\uae30 \ub54c\ubb38\uc774\ub2e4.", "answer": "\uc801\uac08\uc0c9 \ub3d9\ub9f9", "sentence": "\uc774\ub97c \uc801\uac08\uc0c9 \ub3d9\ub9f9 (Red-Brown Alliance, \uc801\uc0c9\uc740 \ubcfc\uc170\ube44\uc998\uc744 \uac08\uc0c9\uc740 \ub098\uce58\ub97c \uc0c1\uc9d5)\uc774\ub77c\uace0 \ubd88\ub800\ub2e4.", "paragraph_sentence": "\uc81c3\uc758 \uc704\uce58\ub77c\ub294 \uc6a9\uc5b4\ub294 \uc720\ub7fd\uc5d0\uc11c \uc81c3\uc758 \uc704\uce58\uc758 \uc120\uad6c\uc790\ub85c \uac04\uc8fc\ub418\ub294 \ubbfc\uc871 \ubcfc\uc170\ube44\uc998(\uadf9\uc6b0 \uad6d\uc218\uc8fc\uc758\uc640 \uadf9\uc88c \ubcfc\uc170\ube44\ud0a4\uc801 \uc0ac\ud68c\uc8fc\uc758\uc758 \ud63c\ud569) \uadf8\ub9ac\uace0 \uc288\ud2b8\ub77c\uc11c\uc8fc\uc758(\ubbfc\uc871 \uc0ac\ud68c\uc8fc\uc758\uc758 \uc0ac\ud68c\uc8fc\uc758\uc801 \ud615\ud0dc\ub85c\uc11c \ub178\ub3d9\uc790\uc640 \uae09\uc9c4\uc801\uc778 \ub300\uc911\ud589\ub3d9 \ub4f1\uc5d0 \uae30\ubc18\ud588\uace0, \ub098\uce58\ub2f9 \ub0b4\uc5d0\uc11c \uc88c\uc775\uc774\uc600\uc9c0\ub9cc, 1934\ub144 \uc7a5\uac80\uc758 \ubc24 \uc0ac\uac74\uc5d0\uc11c \uc219\uccad\ub41c\ub2e4. ) \uac19\uc740 \uc815\uce58 \uc6b4\ub3d9\uc5d0\uc11c \ube44\ub86f\ub418\uc5c8\ub2e4. \ub124\uc624 \ud30c\uc2dc\uc2a4\ud2b8, \ub124\uc624 \ub098\uce58 \uc791\uac00\uc778 \ud504\ub79c\uc2dc\uc2a4 \ud30c\ucee4 \uc694\ud0a4(Francis Parker Yockey)\ub294 \uacf5\uc0b0\uc8fc\uc758\uc790\uc640 \ud30c\uc2dc\uc2a4\ud2b8 \uc0ac\uc774\uc758 \ubcf8\uc9c8\uc0c1 \ubc18\ubbf8, \ubc18\uc720\ub300\uc8fc\uc758\uc801 \ud2b9\uc9d5\uc744 \uac00\uc9c0\ub294 \ub3d9\ub9f9\uc744 \uc81c\uc548\ud588\uace0 \uc774\ub97c \uc801\uac08\uc0c9 \ub3d9\ub9f9 (Red-Brown Alliance, \uc801\uc0c9\uc740 \ubcfc\uc170\ube44\uc998\uc744 \uac08\uc0c9\uc740 \ub098\uce58\ub97c \uc0c1\uc9d5)\uc774\ub77c\uace0 \ubd88\ub800\ub2e4. \uc694\ud0a4\ub294 \uc81c3\uc138\uacc4\uc758 \ud574\ubc29 \uc6b4\ub3d9\uc744 \uc9c0\uc9c0\ud588\ub2e4. \ubbfc\uc871\ud574\ubc29\ud22c\uc7c1\uc774 \uc778\uc885\ubd84\ub9ac\uc640 \ubbfc\uc871\ub2e4\uc6d0\uc8fc\uc758\uc5d0 \ub3c4\uc6c0\uc774 \ub418\ub294\uac78\ub85c \uac04\uc8fc\ud588\uae30 \ub54c\ubb38\uc774\ub2e4.", "paragraph_answer": "\uc81c3\uc758 \uc704\uce58\ub77c\ub294 \uc6a9\uc5b4\ub294 \uc720\ub7fd\uc5d0\uc11c \uc81c3\uc758 \uc704\uce58\uc758 \uc120\uad6c\uc790\ub85c \uac04\uc8fc\ub418\ub294 \ubbfc\uc871 \ubcfc\uc170\ube44\uc998(\uadf9\uc6b0 \uad6d\uc218\uc8fc\uc758\uc640 \uadf9\uc88c \ubcfc\uc170\ube44\ud0a4\uc801 \uc0ac\ud68c\uc8fc\uc758\uc758 \ud63c\ud569) \uadf8\ub9ac\uace0 \uc288\ud2b8\ub77c\uc11c\uc8fc\uc758(\ubbfc\uc871 \uc0ac\ud68c\uc8fc\uc758\uc758 \uc0ac\ud68c\uc8fc\uc758\uc801 \ud615\ud0dc\ub85c\uc11c \ub178\ub3d9\uc790\uc640 \uae09\uc9c4\uc801\uc778 \ub300\uc911\ud589\ub3d9 \ub4f1\uc5d0 \uae30\ubc18\ud588\uace0, \ub098\uce58\ub2f9 \ub0b4\uc5d0\uc11c \uc88c\uc775\uc774\uc600\uc9c0\ub9cc, 1934\ub144 \uc7a5\uac80\uc758 \ubc24 \uc0ac\uac74\uc5d0\uc11c \uc219\uccad\ub41c\ub2e4.) \uac19\uc740 \uc815\uce58 \uc6b4\ub3d9\uc5d0\uc11c \ube44\ub86f\ub418\uc5c8\ub2e4. \ub124\uc624 \ud30c\uc2dc\uc2a4\ud2b8, \ub124\uc624 \ub098\uce58 \uc791\uac00\uc778 \ud504\ub79c\uc2dc\uc2a4 \ud30c\ucee4 \uc694\ud0a4(Francis Parker Yockey)\ub294 \uacf5\uc0b0\uc8fc\uc758\uc790\uc640 \ud30c\uc2dc\uc2a4\ud2b8 \uc0ac\uc774\uc758 \ubcf8\uc9c8\uc0c1 \ubc18\ubbf8, \ubc18\uc720\ub300\uc8fc\uc758\uc801 \ud2b9\uc9d5\uc744 \uac00\uc9c0\ub294 \ub3d9\ub9f9\uc744 \uc81c\uc548\ud588\uace0 \uc774\ub97c \uc801\uac08\uc0c9 \ub3d9\ub9f9 (Red-Brown Alliance, \uc801\uc0c9\uc740 \ubcfc\uc170\ube44\uc998\uc744 \uac08\uc0c9\uc740 \ub098\uce58\ub97c \uc0c1\uc9d5)\uc774\ub77c\uace0 \ubd88\ub800\ub2e4. \uc694\ud0a4\ub294 \uc81c3\uc138\uacc4\uc758 \ud574\ubc29 \uc6b4\ub3d9\uc744 \uc9c0\uc9c0\ud588\ub2e4. \ubbfc\uc871\ud574\ubc29\ud22c\uc7c1\uc774 \uc778\uc885\ubd84\ub9ac\uc640 \ubbfc\uc871\ub2e4\uc6d0\uc8fc\uc758\uc5d0 \ub3c4\uc6c0\uc774 \ub418\ub294\uac78\ub85c \uac04\uc8fc\ud588\uae30 \ub54c\ubb38\uc774\ub2e4.", "sentence_answer": "\uc774\ub97c \uc801\uac08\uc0c9 \ub3d9\ub9f9 (Red-Brown Alliance, \uc801\uc0c9\uc740 \ubcfc\uc170\ube44\uc998\uc744 \uac08\uc0c9\uc740 \ub098\uce58\ub97c \uc0c1\uc9d5)\uc774\ub77c\uace0 \ubd88\ub800\ub2e4.", "paragraph_id": "6550836-2-1", "paragraph_question": "question: \ud504\ub79c\uc2dc\uc2a4 \ud30c\ucee4 \uc694\ucee4\uac00 \uc8fc\uc7a5\ud55c \uacf5\uc0b0\uc8fc\uc758\uc640 \ub124\uc624 \ud30c\uc2dc\uc2a4\ud2b8 \uac04\uc758 \ub3d9\ub9f9\uc740 \ubb34\uc5c7\uc778\uac00?, context: \uc81c3\uc758 \uc704\uce58\ub77c\ub294 \uc6a9\uc5b4\ub294 \uc720\ub7fd\uc5d0\uc11c \uc81c3\uc758 \uc704\uce58\uc758 \uc120\uad6c\uc790\ub85c \uac04\uc8fc\ub418\ub294 \ubbfc\uc871 \ubcfc\uc170\ube44\uc998(\uadf9\uc6b0 \uad6d\uc218\uc8fc\uc758\uc640 \uadf9\uc88c \ubcfc\uc170\ube44\ud0a4\uc801 \uc0ac\ud68c\uc8fc\uc758\uc758 \ud63c\ud569) \uadf8\ub9ac\uace0 \uc288\ud2b8\ub77c\uc11c\uc8fc\uc758(\ubbfc\uc871 \uc0ac\ud68c\uc8fc\uc758\uc758 \uc0ac\ud68c\uc8fc\uc758\uc801 \ud615\ud0dc\ub85c\uc11c \ub178\ub3d9\uc790\uc640 \uae09\uc9c4\uc801\uc778 \ub300\uc911\ud589\ub3d9 \ub4f1\uc5d0 \uae30\ubc18\ud588\uace0, \ub098\uce58\ub2f9 \ub0b4\uc5d0\uc11c \uc88c\uc775\uc774\uc600\uc9c0\ub9cc, 1934\ub144 \uc7a5\uac80\uc758 \ubc24 \uc0ac\uac74\uc5d0\uc11c \uc219\uccad\ub41c\ub2e4.) \uac19\uc740 \uc815\uce58 \uc6b4\ub3d9\uc5d0\uc11c \ube44\ub86f\ub418\uc5c8\ub2e4. \ub124\uc624 \ud30c\uc2dc\uc2a4\ud2b8, \ub124\uc624 \ub098\uce58 \uc791\uac00\uc778 \ud504\ub79c\uc2dc\uc2a4 \ud30c\ucee4 \uc694\ud0a4(Francis Parker Yockey)\ub294 \uacf5\uc0b0\uc8fc\uc758\uc790\uc640 \ud30c\uc2dc\uc2a4\ud2b8 \uc0ac\uc774\uc758 \ubcf8\uc9c8\uc0c1 \ubc18\ubbf8, \ubc18\uc720\ub300\uc8fc\uc758\uc801 \ud2b9\uc9d5\uc744 \uac00\uc9c0\ub294 \ub3d9\ub9f9\uc744 \uc81c\uc548\ud588\uace0 \uc774\ub97c \uc801\uac08\uc0c9 \ub3d9\ub9f9(Red-Brown Alliance, \uc801\uc0c9\uc740 \ubcfc\uc170\ube44\uc998\uc744 \uac08\uc0c9\uc740 \ub098\uce58\ub97c \uc0c1\uc9d5)\uc774\ub77c\uace0 \ubd88\ub800\ub2e4. \uc694\ud0a4\ub294 \uc81c3\uc138\uacc4\uc758 \ud574\ubc29 \uc6b4\ub3d9\uc744 \uc9c0\uc9c0\ud588\ub2e4. \ubbfc\uc871\ud574\ubc29\ud22c\uc7c1\uc774 \uc778\uc885\ubd84\ub9ac\uc640 \ubbfc\uc871\ub2e4\uc6d0\uc8fc\uc758\uc5d0 \ub3c4\uc6c0\uc774 \ub418\ub294\uac78\ub85c \uac04\uc8fc\ud588\uae30 \ub54c\ubb38\uc774\ub2e4."} {"question": "\ub178\ubb34\ud604\uc774 1995\ub144, \ubd80\uc0b0\uad11\uc5ed\uc2dc\uc7a5 \uc120\uac70\uc5d0\uc11c \uc5bb\uc740 \ub4dd\ud45c\uc728\uc740?", "paragraph": "1991\ub144 10\uc6d4 14\ub300 \ucd1d\uc120\uc744 \ucf54\uc55e\uc5d0 \ub454 \uc2dc\uc810\uc5d0\uc11c '\uc8fc\uac04\uc870\uc120'\uc774 \uac8c\uc7ac\ud55c \u2018\ub178 \uc758\uc6d0\uc740 \uacfc\uc5f0 \uc0c1\ub2f9\ud55c \uc7ac\uc0b0\uac00\uc778\uac00\u2019\ub77c\ub294 \uc81c\ubaa9\uc758 \uae30\uc0ac\ub97c \ubcf4\ub3c4\ud588\ub2e4. \uc774 \uae30\uc0ac\ub294 \uc778\uad8c \ubcc0\ud638\uc0ac\ub85c \uc54c\ub824\uc9c4 \ub2f9\uc2dc \ub178\ubb34\ud604 \uc758\uc6d0\uc774 \ubd80\ub3d9\uc0b0 \ud22c\uae30\uc758 \uc804\ub825\uc774 \uc788\uace0 \ud638\ud654 \uc694\ud2b8\ub97c \uc18c\uc720\ud558\uace0 \uc788\ub2e4\ub294 \ub4f1 \uc7ac\uc0b0 \uaddc\ubaa8 \ubc0f \ud615\uc131 \uacfc\uc815\uc758 \uc758\ud639\uc744 \ubcf4\ub3c4\ud588\ub2e4. \ub178\ubb34\ud604 \uc758\uc6d0\uc740 \uba85\uc608\ud6fc\uc190 \uc18c\uc1a1\uc744 \uc81c\uae30\ud588\uace0 1\ub144\uc5ec \ub9cc\uc5d0 \uc2b9\uc18c \ud310\uacb0\uc744 \ubc1b\uc544\ub0c8\ub2e4. \uc774 \uae30\uc0ac\uac00 \uc120\uac70\uc5d0 \uc5b4\ub290 \uc815\ub3c4 \uc601\ud5a5\uc744 \ubbf8\ucce4\ub294\uc9c0\ub294 \uacc4\ub7c9\ud560 \uc218 \uc5c6\uc73c\ub098 \uacb0\uad6d \ub178\ubb34\ud604\uc740 1992\ub144 \ubbfc\uc8fc\ub2f9 (1991\ub144) \ud6c4\ubcf4\ub85c \ubd80\uc0b0 \ub3d9\uad6c\uc5d0 \ucd9c\ub9c8\ud558\uc600\ub2e4\uac00 \ub099\uc120\ud558\uba74\uc11c \uc7ac\uc120\uc5d0 \uc2e4\ud328\ud588\ub2e4. 1993\ub144 \ubbfc\uc8fc\ub2f9 \ucd5c\uc5f0\uc18c \ucd5c\uace0\uc704\uc6d0\uc774 \ub418\uc5c8\ub2e4. 1995\ub144\uc5d0\ub294 \ubbfc\uc8fc\ub2f9 \ud6c4\ubcf4\ub85c\uc11c \ubd80\uc0b0\uad11\uc5ed\uc2dc\uc7a5 \uc120\uac70\uc5d0 \ucd9c\ub9c8\ud558\uc5ec 36.7%\uc758 \ub4dd\ud45c\uc728\uc744 \uc5bb\uc5c8\uc73c\ub098 \uacb0\uad6d \ub099\uc120\ud588\ub2e4.", "answer": "36.7%", "sentence": "1995\ub144\uc5d0\ub294 \ubbfc\uc8fc\ub2f9 \ud6c4\ubcf4\ub85c\uc11c \ubd80\uc0b0\uad11\uc5ed\uc2dc\uc7a5 \uc120\uac70\uc5d0 \ucd9c\ub9c8\ud558\uc5ec 36.7% \uc758 \ub4dd\ud45c\uc728\uc744 \uc5bb\uc5c8\uc73c\ub098 \uacb0\uad6d \ub099\uc120\ud588\ub2e4.", "paragraph_sentence": "1991\ub144 10\uc6d4 14\ub300 \ucd1d\uc120\uc744 \ucf54\uc55e\uc5d0 \ub454 \uc2dc\uc810\uc5d0\uc11c '\uc8fc\uac04\uc870\uc120'\uc774 \uac8c\uc7ac\ud55c \u2018\ub178 \uc758\uc6d0\uc740 \uacfc\uc5f0 \uc0c1\ub2f9\ud55c \uc7ac\uc0b0\uac00\uc778\uac00\u2019\ub77c\ub294 \uc81c\ubaa9\uc758 \uae30\uc0ac\ub97c \ubcf4\ub3c4\ud588\ub2e4. \uc774 \uae30\uc0ac\ub294 \uc778\uad8c \ubcc0\ud638\uc0ac\ub85c \uc54c\ub824\uc9c4 \ub2f9\uc2dc \ub178\ubb34\ud604 \uc758\uc6d0\uc774 \ubd80\ub3d9\uc0b0 \ud22c\uae30\uc758 \uc804\ub825\uc774 \uc788\uace0 \ud638\ud654 \uc694\ud2b8\ub97c \uc18c\uc720\ud558\uace0 \uc788\ub2e4\ub294 \ub4f1 \uc7ac\uc0b0 \uaddc\ubaa8 \ubc0f \ud615\uc131 \uacfc\uc815\uc758 \uc758\ud639\uc744 \ubcf4\ub3c4\ud588\ub2e4. \ub178\ubb34\ud604 \uc758\uc6d0\uc740 \uba85\uc608\ud6fc\uc190 \uc18c\uc1a1\uc744 \uc81c\uae30\ud588\uace0 1\ub144\uc5ec \ub9cc\uc5d0 \uc2b9\uc18c \ud310\uacb0\uc744 \ubc1b\uc544\ub0c8\ub2e4. \uc774 \uae30\uc0ac\uac00 \uc120\uac70\uc5d0 \uc5b4\ub290 \uc815\ub3c4 \uc601\ud5a5\uc744 \ubbf8\ucce4\ub294\uc9c0\ub294 \uacc4\ub7c9\ud560 \uc218 \uc5c6\uc73c\ub098 \uacb0\uad6d \ub178\ubb34\ud604\uc740 1992\ub144 \ubbfc\uc8fc\ub2f9 (1991\ub144) \ud6c4\ubcf4\ub85c \ubd80\uc0b0 \ub3d9\uad6c\uc5d0 \ucd9c\ub9c8\ud558\uc600\ub2e4\uac00 \ub099\uc120\ud558\uba74\uc11c \uc7ac\uc120\uc5d0 \uc2e4\ud328\ud588\ub2e4. 1993\ub144 \ubbfc\uc8fc\ub2f9 \ucd5c\uc5f0\uc18c \ucd5c\uace0\uc704\uc6d0\uc774 \ub418\uc5c8\ub2e4. 1995\ub144\uc5d0\ub294 \ubbfc\uc8fc\ub2f9 \ud6c4\ubcf4\ub85c\uc11c \ubd80\uc0b0\uad11\uc5ed\uc2dc\uc7a5 \uc120\uac70\uc5d0 \ucd9c\ub9c8\ud558\uc5ec 36.7% \uc758 \ub4dd\ud45c\uc728\uc744 \uc5bb\uc5c8\uc73c\ub098 \uacb0\uad6d \ub099\uc120\ud588\ub2e4. ", "paragraph_answer": "1991\ub144 10\uc6d4 14\ub300 \ucd1d\uc120\uc744 \ucf54\uc55e\uc5d0 \ub454 \uc2dc\uc810\uc5d0\uc11c '\uc8fc\uac04\uc870\uc120'\uc774 \uac8c\uc7ac\ud55c \u2018\ub178 \uc758\uc6d0\uc740 \uacfc\uc5f0 \uc0c1\ub2f9\ud55c \uc7ac\uc0b0\uac00\uc778\uac00\u2019\ub77c\ub294 \uc81c\ubaa9\uc758 \uae30\uc0ac\ub97c \ubcf4\ub3c4\ud588\ub2e4. \uc774 \uae30\uc0ac\ub294 \uc778\uad8c \ubcc0\ud638\uc0ac\ub85c \uc54c\ub824\uc9c4 \ub2f9\uc2dc \ub178\ubb34\ud604 \uc758\uc6d0\uc774 \ubd80\ub3d9\uc0b0 \ud22c\uae30\uc758 \uc804\ub825\uc774 \uc788\uace0 \ud638\ud654 \uc694\ud2b8\ub97c \uc18c\uc720\ud558\uace0 \uc788\ub2e4\ub294 \ub4f1 \uc7ac\uc0b0 \uaddc\ubaa8 \ubc0f \ud615\uc131 \uacfc\uc815\uc758 \uc758\ud639\uc744 \ubcf4\ub3c4\ud588\ub2e4. \ub178\ubb34\ud604 \uc758\uc6d0\uc740 \uba85\uc608\ud6fc\uc190 \uc18c\uc1a1\uc744 \uc81c\uae30\ud588\uace0 1\ub144\uc5ec \ub9cc\uc5d0 \uc2b9\uc18c \ud310\uacb0\uc744 \ubc1b\uc544\ub0c8\ub2e4. \uc774 \uae30\uc0ac\uac00 \uc120\uac70\uc5d0 \uc5b4\ub290 \uc815\ub3c4 \uc601\ud5a5\uc744 \ubbf8\ucce4\ub294\uc9c0\ub294 \uacc4\ub7c9\ud560 \uc218 \uc5c6\uc73c\ub098 \uacb0\uad6d \ub178\ubb34\ud604\uc740 1992\ub144 \ubbfc\uc8fc\ub2f9 (1991\ub144) \ud6c4\ubcf4\ub85c \ubd80\uc0b0 \ub3d9\uad6c\uc5d0 \ucd9c\ub9c8\ud558\uc600\ub2e4\uac00 \ub099\uc120\ud558\uba74\uc11c \uc7ac\uc120\uc5d0 \uc2e4\ud328\ud588\ub2e4. 1993\ub144 \ubbfc\uc8fc\ub2f9 \ucd5c\uc5f0\uc18c \ucd5c\uace0\uc704\uc6d0\uc774 \ub418\uc5c8\ub2e4. 1995\ub144\uc5d0\ub294 \ubbfc\uc8fc\ub2f9 \ud6c4\ubcf4\ub85c\uc11c \ubd80\uc0b0\uad11\uc5ed\uc2dc\uc7a5 \uc120\uac70\uc5d0 \ucd9c\ub9c8\ud558\uc5ec 36.7% \uc758 \ub4dd\ud45c\uc728\uc744 \uc5bb\uc5c8\uc73c\ub098 \uacb0\uad6d \ub099\uc120\ud588\ub2e4.", "sentence_answer": "1995\ub144\uc5d0\ub294 \ubbfc\uc8fc\ub2f9 \ud6c4\ubcf4\ub85c\uc11c \ubd80\uc0b0\uad11\uc5ed\uc2dc\uc7a5 \uc120\uac70\uc5d0 \ucd9c\ub9c8\ud558\uc5ec 36.7% \uc758 \ub4dd\ud45c\uc728\uc744 \uc5bb\uc5c8\uc73c\ub098 \uacb0\uad6d \ub099\uc120\ud588\ub2e4.", "paragraph_id": "6464960-6-2", "paragraph_question": "question: \ub178\ubb34\ud604\uc774 1995\ub144, \ubd80\uc0b0\uad11\uc5ed\uc2dc\uc7a5 \uc120\uac70\uc5d0\uc11c \uc5bb\uc740 \ub4dd\ud45c\uc728\uc740?, context: 1991\ub144 10\uc6d4 14\ub300 \ucd1d\uc120\uc744 \ucf54\uc55e\uc5d0 \ub454 \uc2dc\uc810\uc5d0\uc11c '\uc8fc\uac04\uc870\uc120'\uc774 \uac8c\uc7ac\ud55c \u2018\ub178 \uc758\uc6d0\uc740 \uacfc\uc5f0 \uc0c1\ub2f9\ud55c \uc7ac\uc0b0\uac00\uc778\uac00\u2019\ub77c\ub294 \uc81c\ubaa9\uc758 \uae30\uc0ac\ub97c \ubcf4\ub3c4\ud588\ub2e4. \uc774 \uae30\uc0ac\ub294 \uc778\uad8c \ubcc0\ud638\uc0ac\ub85c \uc54c\ub824\uc9c4 \ub2f9\uc2dc \ub178\ubb34\ud604 \uc758\uc6d0\uc774 \ubd80\ub3d9\uc0b0 \ud22c\uae30\uc758 \uc804\ub825\uc774 \uc788\uace0 \ud638\ud654 \uc694\ud2b8\ub97c \uc18c\uc720\ud558\uace0 \uc788\ub2e4\ub294 \ub4f1 \uc7ac\uc0b0 \uaddc\ubaa8 \ubc0f \ud615\uc131 \uacfc\uc815\uc758 \uc758\ud639\uc744 \ubcf4\ub3c4\ud588\ub2e4. \ub178\ubb34\ud604 \uc758\uc6d0\uc740 \uba85\uc608\ud6fc\uc190 \uc18c\uc1a1\uc744 \uc81c\uae30\ud588\uace0 1\ub144\uc5ec \ub9cc\uc5d0 \uc2b9\uc18c \ud310\uacb0\uc744 \ubc1b\uc544\ub0c8\ub2e4. \uc774 \uae30\uc0ac\uac00 \uc120\uac70\uc5d0 \uc5b4\ub290 \uc815\ub3c4 \uc601\ud5a5\uc744 \ubbf8\ucce4\ub294\uc9c0\ub294 \uacc4\ub7c9\ud560 \uc218 \uc5c6\uc73c\ub098 \uacb0\uad6d \ub178\ubb34\ud604\uc740 1992\ub144 \ubbfc\uc8fc\ub2f9 (1991\ub144) \ud6c4\ubcf4\ub85c \ubd80\uc0b0 \ub3d9\uad6c\uc5d0 \ucd9c\ub9c8\ud558\uc600\ub2e4\uac00 \ub099\uc120\ud558\uba74\uc11c \uc7ac\uc120\uc5d0 \uc2e4\ud328\ud588\ub2e4. 1993\ub144 \ubbfc\uc8fc\ub2f9 \ucd5c\uc5f0\uc18c \ucd5c\uace0\uc704\uc6d0\uc774 \ub418\uc5c8\ub2e4. 1995\ub144\uc5d0\ub294 \ubbfc\uc8fc\ub2f9 \ud6c4\ubcf4\ub85c\uc11c \ubd80\uc0b0\uad11\uc5ed\uc2dc\uc7a5 \uc120\uac70\uc5d0 \ucd9c\ub9c8\ud558\uc5ec 36.7%\uc758 \ub4dd\ud45c\uc728\uc744 \uc5bb\uc5c8\uc73c\ub098 \uacb0\uad6d \ub099\uc120\ud588\ub2e4."} {"question": "\uc774\ub9c8\ub2c8\uc2dc\uc758 \uc124\uc740 \uc870\uc120\uc758 \uc0ac\ud559\uc790\uc774\uc790 \uc5b8\ub860\uc778\uc778 \ub204\uac00 \ub3d9\uc870\ud558\uc600\ub098?", "paragraph": "\ud55c\ud3b8 \uc4f0\ub2e4\uc758 \uc124\uc774 \ubc1c\ud45c\ub418\uace0 \uc57d 10\ub144 \ub4a4, \uc624\ud558\ub77c \ub3c4\uc2dc\ud0c0\ucf00(\u5927\u539f\u5229\u6b66)\uc640 \uc624\ud0c0 \uc1fc\uace0(\u5c0f\u7530\u7701\u543e)\uc5d0 \uc758\ud574 \uae30\ub85d\uc5d0 \ub4f1\uc7a5\ud558\ub294 \ubc31\uac15(\ubc31\ucd0c\uac15)\uc744 \ud1b5\uc124\uc5d0\uc11c\ucc98\ub7fc \ucda9\uccad\ub3c4\uc758 \uae08\uac15\uc774 \uc544\ub2c8\ub77c \uc9c0\uae08\uc758 \uc804\ub77c\ubd81\ub3c4 \uc9c0\uc5ed\uc5d0\uc11c \ucc3e\uace0\uc790 \ud558\ub294 \uc2dc\ub3c4\uac00 \uc788\uc5c8\uace0, \uc774\ub54c \uc8fc\ub958\uc131\ub3c4 \ucda9\uccad\ub3c4 \uc9c0\uc5ed\uc774 \uc544\ub2cc \uc804\ub77c\ubd81\ub3c4 \uc9c0\uc5ed\uc758 \uc5b4\ub518\uac00\ub85c \ucd94\uc815\ud558\ub294 \ud559\uc124\uc774 \uc81c\uae30\ub418\uc5c8\ub2e4. \uc774\ub9c8\ub2c8\uc2dc \ub958(\u4eca\u897f\u9f8d)\ub3c4 \uc8fc\ub958\uc131\uc744 \uc804\ub77c\ubd81\ub3c4 \uc9c0\uc5ed\uc5d0\uc11c \ucc3e\uace0\uc790 \ud588\ub294\ub370, \uc0ac\ub9dd \uc9c1\uc804\uc5d0 \uc900\ube44\uc911\uc774\ub358 \u300c\uc8fc\ub958\uc131\uace0\u300d\uc5d0\uc11c \uc804\ub77c\ubd81\ub3c4 \uace0\ubd80\uba74\uc758 \ub450\uc2b9\uc0b0(\u6597\u5347\u5c71)\uc73c\ub85c \ube44\uc815\ud588\ub358 \uc124\uc744 \ubc14\uafb8\uc5b4, \uc804\ub77c\ubd81\ub3c4 \uc9c0\ubc29\uc744 \uc5ec\ud589\ud558\uba74\uc11c \uc4f4 \uae30\ud589\ubb38 \u300c\uc5ec\ud589\uc7a1\uae30\u300d\uc5d0\uc11c \ubd80\uc548\uc758 \uc704\uae08\uc554\uc0b0\uc131\uc744 \uc8fc\ub958\uc131\uc73c\ub85c \uc9c0\ubaa9\ud588\ub2e4. \uc774\ub9c8\ub2c8\uc2dc\uc758 \uc124\uc740 \ub2f9\uc2dc \uc870\uc120\uc758 \uc0ac\ud559\uc790\uc774\uc790 \uc5b8\ub860\uc778\uc774\uc5c8\ub358 \uc548\uc7ac\ud64d\ub3c4 \ub3d9\uc870\ud558\uc600\ub2e4.", "answer": "\uc548\uc7ac\ud64d", "sentence": "\uc774\ub9c8\ub2c8\uc2dc\uc758 \uc124\uc740 \ub2f9\uc2dc \uc870\uc120\uc758 \uc0ac\ud559\uc790\uc774\uc790 \uc5b8\ub860\uc778\uc774\uc5c8\ub358 \uc548\uc7ac\ud64d \ub3c4 \ub3d9\uc870\ud558\uc600\ub2e4.", "paragraph_sentence": "\ud55c\ud3b8 \uc4f0\ub2e4\uc758 \uc124\uc774 \ubc1c\ud45c\ub418\uace0 \uc57d 10\ub144 \ub4a4, \uc624\ud558\ub77c \ub3c4\uc2dc\ud0c0\ucf00(\u5927\u539f\u5229\u6b66)\uc640 \uc624\ud0c0 \uc1fc\uace0(\u5c0f\u7530\u7701\u543e)\uc5d0 \uc758\ud574 \uae30\ub85d\uc5d0 \ub4f1\uc7a5\ud558\ub294 \ubc31\uac15(\ubc31\ucd0c\uac15)\uc744 \ud1b5\uc124\uc5d0\uc11c\ucc98\ub7fc \ucda9\uccad\ub3c4\uc758 \uae08\uac15\uc774 \uc544\ub2c8\ub77c \uc9c0\uae08\uc758 \uc804\ub77c\ubd81\ub3c4 \uc9c0\uc5ed\uc5d0\uc11c \ucc3e\uace0\uc790 \ud558\ub294 \uc2dc\ub3c4\uac00 \uc788\uc5c8\uace0, \uc774\ub54c \uc8fc\ub958\uc131\ub3c4 \ucda9\uccad\ub3c4 \uc9c0\uc5ed\uc774 \uc544\ub2cc \uc804\ub77c\ubd81\ub3c4 \uc9c0\uc5ed\uc758 \uc5b4\ub518\uac00\ub85c \ucd94\uc815\ud558\ub294 \ud559\uc124\uc774 \uc81c\uae30\ub418\uc5c8\ub2e4. \uc774\ub9c8\ub2c8\uc2dc \ub958(\u4eca\u897f\u9f8d)\ub3c4 \uc8fc\ub958\uc131\uc744 \uc804\ub77c\ubd81\ub3c4 \uc9c0\uc5ed\uc5d0\uc11c \ucc3e\uace0\uc790 \ud588\ub294\ub370, \uc0ac\ub9dd \uc9c1\uc804\uc5d0 \uc900\ube44\uc911\uc774\ub358 \u300c\uc8fc\ub958\uc131\uace0\u300d\uc5d0\uc11c \uc804\ub77c\ubd81\ub3c4 \uace0\ubd80\uba74\uc758 \ub450\uc2b9\uc0b0(\u6597\u5347\u5c71)\uc73c\ub85c \ube44\uc815\ud588\ub358 \uc124\uc744 \ubc14\uafb8\uc5b4, \uc804\ub77c\ubd81\ub3c4 \uc9c0\ubc29\uc744 \uc5ec\ud589\ud558\uba74\uc11c \uc4f4 \uae30\ud589\ubb38 \u300c\uc5ec\ud589\uc7a1\uae30\u300d\uc5d0\uc11c \ubd80\uc548\uc758 \uc704\uae08\uc554\uc0b0\uc131\uc744 \uc8fc\ub958\uc131\uc73c\ub85c \uc9c0\ubaa9\ud588\ub2e4. \uc774\ub9c8\ub2c8\uc2dc\uc758 \uc124\uc740 \ub2f9\uc2dc \uc870\uc120\uc758 \uc0ac\ud559\uc790\uc774\uc790 \uc5b8\ub860\uc778\uc774\uc5c8\ub358 \uc548\uc7ac\ud64d \ub3c4 \ub3d9\uc870\ud558\uc600\ub2e4. ", "paragraph_answer": "\ud55c\ud3b8 \uc4f0\ub2e4\uc758 \uc124\uc774 \ubc1c\ud45c\ub418\uace0 \uc57d 10\ub144 \ub4a4, \uc624\ud558\ub77c \ub3c4\uc2dc\ud0c0\ucf00(\u5927\u539f\u5229\u6b66)\uc640 \uc624\ud0c0 \uc1fc\uace0(\u5c0f\u7530\u7701\u543e)\uc5d0 \uc758\ud574 \uae30\ub85d\uc5d0 \ub4f1\uc7a5\ud558\ub294 \ubc31\uac15(\ubc31\ucd0c\uac15)\uc744 \ud1b5\uc124\uc5d0\uc11c\ucc98\ub7fc \ucda9\uccad\ub3c4\uc758 \uae08\uac15\uc774 \uc544\ub2c8\ub77c \uc9c0\uae08\uc758 \uc804\ub77c\ubd81\ub3c4 \uc9c0\uc5ed\uc5d0\uc11c \ucc3e\uace0\uc790 \ud558\ub294 \uc2dc\ub3c4\uac00 \uc788\uc5c8\uace0, \uc774\ub54c \uc8fc\ub958\uc131\ub3c4 \ucda9\uccad\ub3c4 \uc9c0\uc5ed\uc774 \uc544\ub2cc \uc804\ub77c\ubd81\ub3c4 \uc9c0\uc5ed\uc758 \uc5b4\ub518\uac00\ub85c \ucd94\uc815\ud558\ub294 \ud559\uc124\uc774 \uc81c\uae30\ub418\uc5c8\ub2e4. \uc774\ub9c8\ub2c8\uc2dc \ub958(\u4eca\u897f\u9f8d)\ub3c4 \uc8fc\ub958\uc131\uc744 \uc804\ub77c\ubd81\ub3c4 \uc9c0\uc5ed\uc5d0\uc11c \ucc3e\uace0\uc790 \ud588\ub294\ub370, \uc0ac\ub9dd \uc9c1\uc804\uc5d0 \uc900\ube44\uc911\uc774\ub358 \u300c\uc8fc\ub958\uc131\uace0\u300d\uc5d0\uc11c \uc804\ub77c\ubd81\ub3c4 \uace0\ubd80\uba74\uc758 \ub450\uc2b9\uc0b0(\u6597\u5347\u5c71)\uc73c\ub85c \ube44\uc815\ud588\ub358 \uc124\uc744 \ubc14\uafb8\uc5b4, \uc804\ub77c\ubd81\ub3c4 \uc9c0\ubc29\uc744 \uc5ec\ud589\ud558\uba74\uc11c \uc4f4 \uae30\ud589\ubb38 \u300c\uc5ec\ud589\uc7a1\uae30\u300d\uc5d0\uc11c \ubd80\uc548\uc758 \uc704\uae08\uc554\uc0b0\uc131\uc744 \uc8fc\ub958\uc131\uc73c\ub85c \uc9c0\ubaa9\ud588\ub2e4. \uc774\ub9c8\ub2c8\uc2dc\uc758 \uc124\uc740 \ub2f9\uc2dc \uc870\uc120\uc758 \uc0ac\ud559\uc790\uc774\uc790 \uc5b8\ub860\uc778\uc774\uc5c8\ub358 \uc548\uc7ac\ud64d \ub3c4 \ub3d9\uc870\ud558\uc600\ub2e4.", "sentence_answer": "\uc774\ub9c8\ub2c8\uc2dc\uc758 \uc124\uc740 \ub2f9\uc2dc \uc870\uc120\uc758 \uc0ac\ud559\uc790\uc774\uc790 \uc5b8\ub860\uc778\uc774\uc5c8\ub358 \uc548\uc7ac\ud64d \ub3c4 \ub3d9\uc870\ud558\uc600\ub2e4.", "paragraph_id": "6595924-3-1", "paragraph_question": "question: \uc774\ub9c8\ub2c8\uc2dc\uc758 \uc124\uc740 \uc870\uc120\uc758 \uc0ac\ud559\uc790\uc774\uc790 \uc5b8\ub860\uc778\uc778 \ub204\uac00 \ub3d9\uc870\ud558\uc600\ub098?, context: \ud55c\ud3b8 \uc4f0\ub2e4\uc758 \uc124\uc774 \ubc1c\ud45c\ub418\uace0 \uc57d 10\ub144 \ub4a4, \uc624\ud558\ub77c \ub3c4\uc2dc\ud0c0\ucf00(\u5927\u539f\u5229\u6b66)\uc640 \uc624\ud0c0 \uc1fc\uace0(\u5c0f\u7530\u7701\u543e)\uc5d0 \uc758\ud574 \uae30\ub85d\uc5d0 \ub4f1\uc7a5\ud558\ub294 \ubc31\uac15(\ubc31\ucd0c\uac15)\uc744 \ud1b5\uc124\uc5d0\uc11c\ucc98\ub7fc \ucda9\uccad\ub3c4\uc758 \uae08\uac15\uc774 \uc544\ub2c8\ub77c \uc9c0\uae08\uc758 \uc804\ub77c\ubd81\ub3c4 \uc9c0\uc5ed\uc5d0\uc11c \ucc3e\uace0\uc790 \ud558\ub294 \uc2dc\ub3c4\uac00 \uc788\uc5c8\uace0, \uc774\ub54c \uc8fc\ub958\uc131\ub3c4 \ucda9\uccad\ub3c4 \uc9c0\uc5ed\uc774 \uc544\ub2cc \uc804\ub77c\ubd81\ub3c4 \uc9c0\uc5ed\uc758 \uc5b4\ub518\uac00\ub85c \ucd94\uc815\ud558\ub294 \ud559\uc124\uc774 \uc81c\uae30\ub418\uc5c8\ub2e4. \uc774\ub9c8\ub2c8\uc2dc \ub958(\u4eca\u897f\u9f8d)\ub3c4 \uc8fc\ub958\uc131\uc744 \uc804\ub77c\ubd81\ub3c4 \uc9c0\uc5ed\uc5d0\uc11c \ucc3e\uace0\uc790 \ud588\ub294\ub370, \uc0ac\ub9dd \uc9c1\uc804\uc5d0 \uc900\ube44\uc911\uc774\ub358 \u300c\uc8fc\ub958\uc131\uace0\u300d\uc5d0\uc11c \uc804\ub77c\ubd81\ub3c4 \uace0\ubd80\uba74\uc758 \ub450\uc2b9\uc0b0(\u6597\u5347\u5c71)\uc73c\ub85c \ube44\uc815\ud588\ub358 \uc124\uc744 \ubc14\uafb8\uc5b4, \uc804\ub77c\ubd81\ub3c4 \uc9c0\ubc29\uc744 \uc5ec\ud589\ud558\uba74\uc11c \uc4f4 \uae30\ud589\ubb38 \u300c\uc5ec\ud589\uc7a1\uae30\u300d\uc5d0\uc11c \ubd80\uc548\uc758 \uc704\uae08\uc554\uc0b0\uc131\uc744 \uc8fc\ub958\uc131\uc73c\ub85c \uc9c0\ubaa9\ud588\ub2e4. \uc774\ub9c8\ub2c8\uc2dc\uc758 \uc124\uc740 \ub2f9\uc2dc \uc870\uc120\uc758 \uc0ac\ud559\uc790\uc774\uc790 \uc5b8\ub860\uc778\uc774\uc5c8\ub358 \uc548\uc7ac\ud64d\ub3c4 \ub3d9\uc870\ud558\uc600\ub2e4."} {"question": "\uad6d\uc655 \ub8e8\uc774 14\uc138 \uc190\uc790\uc774\uba70, \ub8e8\uc774\uc870\uc81c\ud504 \ub4dc \ubd80\ub974\ubd09\uc5d0 \ube44\ud574 \uacbd\ud5d8\uc774 \uc801\uc5c8\ub358 \uad70\uc778\uc740?", "paragraph": "\uc601\uad6d\uacfc \ub124\ub35c\ub780\ub4dc \uacf5\ud654\uad6d, \uadf8\ub9ac\uace0 \uc2e0\uc131\ub85c\ub9c8\uc81c\uad6d\uc740 \ud504\ub791\uc2a4 \uc655\uad6d\uc640 \uc2a4\ud398\uc778\uc774 \ud1b5\ud569\ub420 \uc218 \uc788\ub2e4\ub294 \uc0dd\uac01\uc5d0 \uc804\uc728\ud558\uc600\uace0, \uc774\ub294 \uadf8\ub4e4\ub85c \ud558\uc5ec\uae08 \ud504\ub791\uc2a4\uc5d0 \ub300\ud56d\ud558\ub294 \ub3d9\ub9f9\uc744 \uacb0\uc131\ud558\uac8c \ub9cc\ub4e4\uc5b4 \uc2a4\ud398\uc778 \uc655\uc704 \uacc4\uc2b9 \uc804\uc7c1\uc744 \uc77c\uc73c\ud0a4\ub3c4\ub85d \ud558\uc600\ub2e4. \ub3d9\ub9f9\uad70\uc758 \uc0ac\ub839\uad00 \ub9d0\ubc84\ub7ec \uacf5\uc791 1\uc138 \uc874 \ucc98\uce60(John Churchill, 1st Duke of Marlborough)\uacfc \uadf8\uc758 \ucd5c\uace0\uc758 \ub300\ub9ac\uc778\uc774\ub77c \ud560 \uc218 \uc788\ub294 \uc0ac\ubcf4\uc774\uc544\uc758 \uc678\uc820 \uacf5\uc791(Prince Eug\u00e8ne of Savoy)\uc740 \uce5c\ud55c \uce5c\uad6c\uc0ac\uc774\uc774\uae30\ub3c4 \ud588\ub2e4. \uadf8\ub7ec\ub098 \ubc18\ub300\ub85c \ud504\ub791\uc2a4\uad70\uc758 \uc7a5\uad70\ub4e4\uc740 \uc11c\ub85c \uc0ac\uc774\uac00 \uc88b\uc9c0 \uc54a\uc558\ub2e4. \ubc29\ub3d4 \uacf5\uc791 \ub8e8\uc774\uc870\uc81c\ud504 \ub4dc \ubd80\ub974\ubd09(Louis Joseph, duc de Vend\u00f4me)\uc740 \uacbd\ud5d8\uc774 \ub9ce\uc740 \ubca0\ud14c\ub791 \uad70\uc778\uc774\uc5c8\ub2e4. \ud558\uc9c0\ub9cc \ubd80\ub974\uace0\ub274 \uacf5\uc791(Duke of Burgundy)\uc740 \uc0c1\ub300\uc801\uc73c\ub85c \uacbd\ud5d8\uc774 \uc801\uace0 \uad6d\uc655 \ub8e8\uc774 14\uc138\uc758 \uc190\uc790\ub77c\ub294 \uc790\uc2e0\uc758 \uc9c0\uc704 \ub355\ubd84\uc5d0 \ub9ce\uc740 \uc774\uc775\uc744 \ubcf4\uace0 \uc788\ub358 \uc778\ubb3c\uc774\uc5c8\ub2e4.", "answer": "\ubd80\ub974\uace0\ub274 \uacf5\uc791", "sentence": "\ud558\uc9c0\ub9cc \ubd80\ub974\uace0\ub274 \uacf5\uc791 (Duke of Burgundy)\uc740 \uc0c1\ub300\uc801\uc73c\ub85c \uacbd\ud5d8\uc774 \uc801\uace0 \uad6d\uc655 \ub8e8\uc774 14\uc138\uc758 \uc190\uc790\ub77c\ub294 \uc790\uc2e0\uc758 \uc9c0\uc704 \ub355\ubd84\uc5d0 \ub9ce\uc740 \uc774\uc775\uc744 \ubcf4\uace0 \uc788\ub358 \uc778\ubb3c\uc774\uc5c8\ub2e4.", "paragraph_sentence": "\uc601\uad6d\uacfc \ub124\ub35c\ub780\ub4dc \uacf5\ud654\uad6d, \uadf8\ub9ac\uace0 \uc2e0\uc131\ub85c\ub9c8\uc81c\uad6d\uc740 \ud504\ub791\uc2a4 \uc655\uad6d\uc640 \uc2a4\ud398\uc778\uc774 \ud1b5\ud569\ub420 \uc218 \uc788\ub2e4\ub294 \uc0dd\uac01\uc5d0 \uc804\uc728\ud558\uc600\uace0, \uc774\ub294 \uadf8\ub4e4\ub85c \ud558\uc5ec\uae08 \ud504\ub791\uc2a4\uc5d0 \ub300\ud56d\ud558\ub294 \ub3d9\ub9f9\uc744 \uacb0\uc131\ud558\uac8c \ub9cc\ub4e4\uc5b4 \uc2a4\ud398\uc778 \uc655\uc704 \uacc4\uc2b9 \uc804\uc7c1\uc744 \uc77c\uc73c\ud0a4\ub3c4\ub85d \ud558\uc600\ub2e4. \ub3d9\ub9f9\uad70\uc758 \uc0ac\ub839\uad00 \ub9d0\ubc84\ub7ec \uacf5\uc791 1\uc138 \uc874 \ucc98\uce60(John Churchill, 1st Duke of Marlborough)\uacfc \uadf8\uc758 \ucd5c\uace0\uc758 \ub300\ub9ac\uc778\uc774\ub77c \ud560 \uc218 \uc788\ub294 \uc0ac\ubcf4\uc774\uc544\uc758 \uc678\uc820 \uacf5\uc791(Prince Eug\u00e8ne of Savoy)\uc740 \uce5c\ud55c \uce5c\uad6c\uc0ac\uc774\uc774\uae30\ub3c4 \ud588\ub2e4. \uadf8\ub7ec\ub098 \ubc18\ub300\ub85c \ud504\ub791\uc2a4\uad70\uc758 \uc7a5\uad70\ub4e4\uc740 \uc11c\ub85c \uc0ac\uc774\uac00 \uc88b\uc9c0 \uc54a\uc558\ub2e4. \ubc29\ub3d4 \uacf5\uc791 \ub8e8\uc774\uc870\uc81c\ud504 \ub4dc \ubd80\ub974\ubd09(Louis Joseph, duc de Vend\u00f4me)\uc740 \uacbd\ud5d8\uc774 \ub9ce\uc740 \ubca0\ud14c\ub791 \uad70\uc778\uc774\uc5c8\ub2e4. \ud558\uc9c0\ub9cc \ubd80\ub974\uace0\ub274 \uacf5\uc791 (Duke of Burgundy)\uc740 \uc0c1\ub300\uc801\uc73c\ub85c \uacbd\ud5d8\uc774 \uc801\uace0 \uad6d\uc655 \ub8e8\uc774 14\uc138\uc758 \uc190\uc790\ub77c\ub294 \uc790\uc2e0\uc758 \uc9c0\uc704 \ub355\ubd84\uc5d0 \ub9ce\uc740 \uc774\uc775\uc744 \ubcf4\uace0 \uc788\ub358 \uc778\ubb3c\uc774\uc5c8\ub2e4. ", "paragraph_answer": "\uc601\uad6d\uacfc \ub124\ub35c\ub780\ub4dc \uacf5\ud654\uad6d, \uadf8\ub9ac\uace0 \uc2e0\uc131\ub85c\ub9c8\uc81c\uad6d\uc740 \ud504\ub791\uc2a4 \uc655\uad6d\uc640 \uc2a4\ud398\uc778\uc774 \ud1b5\ud569\ub420 \uc218 \uc788\ub2e4\ub294 \uc0dd\uac01\uc5d0 \uc804\uc728\ud558\uc600\uace0, \uc774\ub294 \uadf8\ub4e4\ub85c \ud558\uc5ec\uae08 \ud504\ub791\uc2a4\uc5d0 \ub300\ud56d\ud558\ub294 \ub3d9\ub9f9\uc744 \uacb0\uc131\ud558\uac8c \ub9cc\ub4e4\uc5b4 \uc2a4\ud398\uc778 \uc655\uc704 \uacc4\uc2b9 \uc804\uc7c1\uc744 \uc77c\uc73c\ud0a4\ub3c4\ub85d \ud558\uc600\ub2e4. \ub3d9\ub9f9\uad70\uc758 \uc0ac\ub839\uad00 \ub9d0\ubc84\ub7ec \uacf5\uc791 1\uc138 \uc874 \ucc98\uce60(John Churchill, 1st Duke of Marlborough)\uacfc \uadf8\uc758 \ucd5c\uace0\uc758 \ub300\ub9ac\uc778\uc774\ub77c \ud560 \uc218 \uc788\ub294 \uc0ac\ubcf4\uc774\uc544\uc758 \uc678\uc820 \uacf5\uc791(Prince Eug\u00e8ne of Savoy)\uc740 \uce5c\ud55c \uce5c\uad6c\uc0ac\uc774\uc774\uae30\ub3c4 \ud588\ub2e4. \uadf8\ub7ec\ub098 \ubc18\ub300\ub85c \ud504\ub791\uc2a4\uad70\uc758 \uc7a5\uad70\ub4e4\uc740 \uc11c\ub85c \uc0ac\uc774\uac00 \uc88b\uc9c0 \uc54a\uc558\ub2e4. \ubc29\ub3d4 \uacf5\uc791 \ub8e8\uc774\uc870\uc81c\ud504 \ub4dc \ubd80\ub974\ubd09(Louis Joseph, duc de Vend\u00f4me)\uc740 \uacbd\ud5d8\uc774 \ub9ce\uc740 \ubca0\ud14c\ub791 \uad70\uc778\uc774\uc5c8\ub2e4. \ud558\uc9c0\ub9cc \ubd80\ub974\uace0\ub274 \uacf5\uc791 (Duke of Burgundy)\uc740 \uc0c1\ub300\uc801\uc73c\ub85c \uacbd\ud5d8\uc774 \uc801\uace0 \uad6d\uc655 \ub8e8\uc774 14\uc138\uc758 \uc190\uc790\ub77c\ub294 \uc790\uc2e0\uc758 \uc9c0\uc704 \ub355\ubd84\uc5d0 \ub9ce\uc740 \uc774\uc775\uc744 \ubcf4\uace0 \uc788\ub358 \uc778\ubb3c\uc774\uc5c8\ub2e4.", "sentence_answer": "\ud558\uc9c0\ub9cc \ubd80\ub974\uace0\ub274 \uacf5\uc791 (Duke of Burgundy)\uc740 \uc0c1\ub300\uc801\uc73c\ub85c \uacbd\ud5d8\uc774 \uc801\uace0 \uad6d\uc655 \ub8e8\uc774 14\uc138\uc758 \uc190\uc790\ub77c\ub294 \uc790\uc2e0\uc758 \uc9c0\uc704 \ub355\ubd84\uc5d0 \ub9ce\uc740 \uc774\uc775\uc744 \ubcf4\uace0 \uc788\ub358 \uc778\ubb3c\uc774\uc5c8\ub2e4.", "paragraph_id": "6587919-0-0", "paragraph_question": "question: \uad6d\uc655 \ub8e8\uc774 14\uc138 \uc190\uc790\uc774\uba70, \ub8e8\uc774\uc870\uc81c\ud504 \ub4dc \ubd80\ub974\ubd09\uc5d0 \ube44\ud574 \uacbd\ud5d8\uc774 \uc801\uc5c8\ub358 \uad70\uc778\uc740?, context: \uc601\uad6d\uacfc \ub124\ub35c\ub780\ub4dc \uacf5\ud654\uad6d, \uadf8\ub9ac\uace0 \uc2e0\uc131\ub85c\ub9c8\uc81c\uad6d\uc740 \ud504\ub791\uc2a4 \uc655\uad6d\uc640 \uc2a4\ud398\uc778\uc774 \ud1b5\ud569\ub420 \uc218 \uc788\ub2e4\ub294 \uc0dd\uac01\uc5d0 \uc804\uc728\ud558\uc600\uace0, \uc774\ub294 \uadf8\ub4e4\ub85c \ud558\uc5ec\uae08 \ud504\ub791\uc2a4\uc5d0 \ub300\ud56d\ud558\ub294 \ub3d9\ub9f9\uc744 \uacb0\uc131\ud558\uac8c \ub9cc\ub4e4\uc5b4 \uc2a4\ud398\uc778 \uc655\uc704 \uacc4\uc2b9 \uc804\uc7c1\uc744 \uc77c\uc73c\ud0a4\ub3c4\ub85d \ud558\uc600\ub2e4. \ub3d9\ub9f9\uad70\uc758 \uc0ac\ub839\uad00 \ub9d0\ubc84\ub7ec \uacf5\uc791 1\uc138 \uc874 \ucc98\uce60(John Churchill, 1st Duke of Marlborough)\uacfc \uadf8\uc758 \ucd5c\uace0\uc758 \ub300\ub9ac\uc778\uc774\ub77c \ud560 \uc218 \uc788\ub294 \uc0ac\ubcf4\uc774\uc544\uc758 \uc678\uc820 \uacf5\uc791(Prince Eug\u00e8ne of Savoy)\uc740 \uce5c\ud55c \uce5c\uad6c\uc0ac\uc774\uc774\uae30\ub3c4 \ud588\ub2e4. \uadf8\ub7ec\ub098 \ubc18\ub300\ub85c \ud504\ub791\uc2a4\uad70\uc758 \uc7a5\uad70\ub4e4\uc740 \uc11c\ub85c \uc0ac\uc774\uac00 \uc88b\uc9c0 \uc54a\uc558\ub2e4. \ubc29\ub3d4 \uacf5\uc791 \ub8e8\uc774\uc870\uc81c\ud504 \ub4dc \ubd80\ub974\ubd09(Louis Joseph, duc de Vend\u00f4me)\uc740 \uacbd\ud5d8\uc774 \ub9ce\uc740 \ubca0\ud14c\ub791 \uad70\uc778\uc774\uc5c8\ub2e4. \ud558\uc9c0\ub9cc \ubd80\ub974\uace0\ub274 \uacf5\uc791(Duke of Burgundy)\uc740 \uc0c1\ub300\uc801\uc73c\ub85c \uacbd\ud5d8\uc774 \uc801\uace0 \uad6d\uc655 \ub8e8\uc774 14\uc138\uc758 \uc190\uc790\ub77c\ub294 \uc790\uc2e0\uc758 \uc9c0\uc704 \ub355\ubd84\uc5d0 \ub9ce\uc740 \uc774\uc775\uc744 \ubcf4\uace0 \uc788\ub358 \uc778\ubb3c\uc774\uc5c8\ub2e4."} {"question": "\uace0\uc870\uc120\uc5d0\uc11c \uad00\uc9c1\uc744 \uac00\uc9c0\uace0 \uc788\uc73c\uba74\uc11c \ub3c5\uc790\uc801\uc778 \uc138\ub825\uae30\ubc18\uc744 \uac00\uc9c4 \uc0ac\ub78c\ub4e4\uc758 \uc608\ub85c \uc870\uc120\uc0c1\uacfc \ub204\uac00\uc788\ub294\uac00?", "paragraph": "\uadf8\ub7f0\ub370 \uc774\ub7ec\ud55c \uad00\uc9c1\uc744 \uac16\uace0 \uc788\uc73c\uba74\uc11c \ub3c5\uc790\uc801\uc778 \uc138\ub825 \uae30\ubc18\uc744 \uc9c0\ub2cc \uc790\ub4e4\uc774 \uc788\uc5c8\ub2e4. \ud55c\ub098\ub77c\uc5d0 \ub300\ud55c \uc678\uad50 \uc815\ucc45\uc5d0\uc11c \uc655\uacfc \uc758\uacac\uc774 \ub9de\uc9c0 \uc54a\uc790 \ud718\ud558\uc758 2\ucc9c \ud638\ub97c \uc774\ub04c\uace0 \ud55c\ubc18\ub3c4 \ub0a8\ubd80\uc9c0\uc5ed\uc73c\ub85c \uc774\ud0c8\ud574 \uac04 \uc870\uc120\uc0c1(\u671d\u9bae\u76f8) \uc5ed\uacc4\uacbd(\u6b77\u8c3f\u537f) \uac19\uc740 \uc774\uac00 \uadf8\ub7f0 \uc608\uc774\ub2e4. \ud55c\ub098\ub77c\uc640 \uc804\uc7c1 \uc911 \uc804\uc120\uc744 \uc774\ud0c8\ud558\uc5ec \uc218\ub3c4\uac00 \ud568\ub77d\ub418\ub294 \uacb0\uc815\uc801\uc778 \uc6d0\uc778\uc744 \uc81c\uacf5\ud55c \uc774\uacc4\uc0c1(\u5c3c\u8c3f\u76f8) \uc0bc(\u53c3) \ub4f1 \uc138 \uba85\uc758 \u2018\uc0c1(\u76f8)\u2019\ub3c4 \uadf8\ub7ec\ud55c \uc774\ub4e4\ub85c \ucd94\uce21\ub418\ub294\ub370, \uc0c1\uc740 \uc77c\uc815\ud55c \uc138\ub825 \uc9d1\ub2e8\uc758 \ub300\ud45c\ub85c\uc11c \uc911\uc559\uc815\ubd80\uc5d0 \ucc38\uc5ec\ud55c \uc774\ub4e4\uc774 \uc9c0\ub2cc \uad00\uc9c1\uc758 \uc774\ub984\uc73c\ub85c \uc5ec\uaca8\uc9c4\ub2e4. \ub2f9\uc2dc \uc704\ub9cc\uc870\uc120\uc5d0\ub294 \uc5ec\ub7ec \uba85\uc758 \uc0c1\uc774 \uc788\uc5c8\ub294\ub370, \uc774\ub984\uc73c\ub85c \ubcf4\uc544 \ub2e4\uc218\uac00 \uace0\uc870\uc120 \uc778\uc774\uc5c8\ub2e4. \uc0c1\uc758 \ud718\ud558\uc5d0 \uc788\ub358 \uc9d1\ub2e8\uc5d0 \ub300\ud574\uc120 \uc655\uc2e4\uc758 \ud1b5\uc81c\ub825\uc774 \uc5b4\ub290 \uc815\ub3c4 \ubbf8\ucce4\uaca0\uc9c0\ub9cc, \uc801\uc5b4\ub3c4 \uac01 \uc9d1\ub2e8 \ub0b4\ubd80\uc758 \uc77c\uc740 \uc790\uce58\uc801\uc73c\ub85c \uc774\ub8e8\uc5b4\uc84c\uc744 \uac83\uc774\ub2e4. \uc5ed\uacc4\uacbd \ub4f1\uc758 \uc9d1\ub2e8\uc801\uc778 \uc774\ud0c8\ud589\uc704\uac00 \uac00\ub2a5\ud588\ub2e4\ub294 \uc810\uc774 \ubc14\ub85c \uc774\ub97c \ub9d0\ud574\uc900\ub2e4.", "answer": "\uc5ed\uacc4\uacbd", "sentence": "\ud55c\ub098\ub77c\uc5d0 \ub300\ud55c \uc678\uad50 \uc815\ucc45\uc5d0\uc11c \uc655\uacfc \uc758\uacac\uc774 \ub9de\uc9c0 \uc54a\uc790 \ud718\ud558\uc758 2\ucc9c \ud638\ub97c \uc774\ub04c\uace0 \ud55c\ubc18\ub3c4 \ub0a8\ubd80\uc9c0\uc5ed\uc73c\ub85c \uc774\ud0c8\ud574 \uac04 \uc870\uc120\uc0c1(\u671d\u9bae\u76f8) \uc5ed\uacc4\uacbd (\u6b77\u8c3f\u537f) \uac19\uc740 \uc774\uac00 \uadf8\ub7f0 \uc608\uc774\ub2e4.", "paragraph_sentence": "\uadf8\ub7f0\ub370 \uc774\ub7ec\ud55c \uad00\uc9c1\uc744 \uac16\uace0 \uc788\uc73c\uba74\uc11c \ub3c5\uc790\uc801\uc778 \uc138\ub825 \uae30\ubc18\uc744 \uc9c0\ub2cc \uc790\ub4e4\uc774 \uc788\uc5c8\ub2e4. \ud55c\ub098\ub77c\uc5d0 \ub300\ud55c \uc678\uad50 \uc815\ucc45\uc5d0\uc11c \uc655\uacfc \uc758\uacac\uc774 \ub9de\uc9c0 \uc54a\uc790 \ud718\ud558\uc758 2\ucc9c \ud638\ub97c \uc774\ub04c\uace0 \ud55c\ubc18\ub3c4 \ub0a8\ubd80\uc9c0\uc5ed\uc73c\ub85c \uc774\ud0c8\ud574 \uac04 \uc870\uc120\uc0c1(\u671d\u9bae\u76f8) \uc5ed\uacc4\uacbd (\u6b77\u8c3f\u537f) \uac19\uc740 \uc774\uac00 \uadf8\ub7f0 \uc608\uc774\ub2e4. \ud55c\ub098\ub77c\uc640 \uc804\uc7c1 \uc911 \uc804\uc120\uc744 \uc774\ud0c8\ud558\uc5ec \uc218\ub3c4\uac00 \ud568\ub77d\ub418\ub294 \uacb0\uc815\uc801\uc778 \uc6d0\uc778\uc744 \uc81c\uacf5\ud55c \uc774\uacc4\uc0c1(\u5c3c\u8c3f\u76f8) \uc0bc(\u53c3) \ub4f1 \uc138 \uba85\uc758 \u2018\uc0c1(\u76f8)\u2019\ub3c4 \uadf8\ub7ec\ud55c \uc774\ub4e4\ub85c \ucd94\uce21\ub418\ub294\ub370, \uc0c1\uc740 \uc77c\uc815\ud55c \uc138\ub825 \uc9d1\ub2e8\uc758 \ub300\ud45c\ub85c\uc11c \uc911\uc559\uc815\ubd80\uc5d0 \ucc38\uc5ec\ud55c \uc774\ub4e4\uc774 \uc9c0\ub2cc \uad00\uc9c1\uc758 \uc774\ub984\uc73c\ub85c \uc5ec\uaca8\uc9c4\ub2e4. \ub2f9\uc2dc \uc704\ub9cc\uc870\uc120\uc5d0\ub294 \uc5ec\ub7ec \uba85\uc758 \uc0c1\uc774 \uc788\uc5c8\ub294\ub370, \uc774\ub984\uc73c\ub85c \ubcf4\uc544 \ub2e4\uc218\uac00 \uace0\uc870\uc120 \uc778\uc774\uc5c8\ub2e4. \uc0c1\uc758 \ud718\ud558\uc5d0 \uc788\ub358 \uc9d1\ub2e8\uc5d0 \ub300\ud574\uc120 \uc655\uc2e4\uc758 \ud1b5\uc81c\ub825\uc774 \uc5b4\ub290 \uc815\ub3c4 \ubbf8\ucce4\uaca0\uc9c0\ub9cc, \uc801\uc5b4\ub3c4 \uac01 \uc9d1\ub2e8 \ub0b4\ubd80\uc758 \uc77c\uc740 \uc790\uce58\uc801\uc73c\ub85c \uc774\ub8e8\uc5b4\uc84c\uc744 \uac83\uc774\ub2e4. \uc5ed\uacc4\uacbd \ub4f1\uc758 \uc9d1\ub2e8\uc801\uc778 \uc774\ud0c8\ud589\uc704\uac00 \uac00\ub2a5\ud588\ub2e4\ub294 \uc810\uc774 \ubc14\ub85c \uc774\ub97c \ub9d0\ud574\uc900\ub2e4.", "paragraph_answer": "\uadf8\ub7f0\ub370 \uc774\ub7ec\ud55c \uad00\uc9c1\uc744 \uac16\uace0 \uc788\uc73c\uba74\uc11c \ub3c5\uc790\uc801\uc778 \uc138\ub825 \uae30\ubc18\uc744 \uc9c0\ub2cc \uc790\ub4e4\uc774 \uc788\uc5c8\ub2e4. \ud55c\ub098\ub77c\uc5d0 \ub300\ud55c \uc678\uad50 \uc815\ucc45\uc5d0\uc11c \uc655\uacfc \uc758\uacac\uc774 \ub9de\uc9c0 \uc54a\uc790 \ud718\ud558\uc758 2\ucc9c \ud638\ub97c \uc774\ub04c\uace0 \ud55c\ubc18\ub3c4 \ub0a8\ubd80\uc9c0\uc5ed\uc73c\ub85c \uc774\ud0c8\ud574 \uac04 \uc870\uc120\uc0c1(\u671d\u9bae\u76f8) \uc5ed\uacc4\uacbd (\u6b77\u8c3f\u537f) \uac19\uc740 \uc774\uac00 \uadf8\ub7f0 \uc608\uc774\ub2e4. \ud55c\ub098\ub77c\uc640 \uc804\uc7c1 \uc911 \uc804\uc120\uc744 \uc774\ud0c8\ud558\uc5ec \uc218\ub3c4\uac00 \ud568\ub77d\ub418\ub294 \uacb0\uc815\uc801\uc778 \uc6d0\uc778\uc744 \uc81c\uacf5\ud55c \uc774\uacc4\uc0c1(\u5c3c\u8c3f\u76f8) \uc0bc(\u53c3) \ub4f1 \uc138 \uba85\uc758 \u2018\uc0c1(\u76f8)\u2019\ub3c4 \uadf8\ub7ec\ud55c \uc774\ub4e4\ub85c \ucd94\uce21\ub418\ub294\ub370, \uc0c1\uc740 \uc77c\uc815\ud55c \uc138\ub825 \uc9d1\ub2e8\uc758 \ub300\ud45c\ub85c\uc11c \uc911\uc559\uc815\ubd80\uc5d0 \ucc38\uc5ec\ud55c \uc774\ub4e4\uc774 \uc9c0\ub2cc \uad00\uc9c1\uc758 \uc774\ub984\uc73c\ub85c \uc5ec\uaca8\uc9c4\ub2e4. \ub2f9\uc2dc \uc704\ub9cc\uc870\uc120\uc5d0\ub294 \uc5ec\ub7ec \uba85\uc758 \uc0c1\uc774 \uc788\uc5c8\ub294\ub370, \uc774\ub984\uc73c\ub85c \ubcf4\uc544 \ub2e4\uc218\uac00 \uace0\uc870\uc120 \uc778\uc774\uc5c8\ub2e4. \uc0c1\uc758 \ud718\ud558\uc5d0 \uc788\ub358 \uc9d1\ub2e8\uc5d0 \ub300\ud574\uc120 \uc655\uc2e4\uc758 \ud1b5\uc81c\ub825\uc774 \uc5b4\ub290 \uc815\ub3c4 \ubbf8\ucce4\uaca0\uc9c0\ub9cc, \uc801\uc5b4\ub3c4 \uac01 \uc9d1\ub2e8 \ub0b4\ubd80\uc758 \uc77c\uc740 \uc790\uce58\uc801\uc73c\ub85c \uc774\ub8e8\uc5b4\uc84c\uc744 \uac83\uc774\ub2e4. \uc5ed\uacc4\uacbd \ub4f1\uc758 \uc9d1\ub2e8\uc801\uc778 \uc774\ud0c8\ud589\uc704\uac00 \uac00\ub2a5\ud588\ub2e4\ub294 \uc810\uc774 \ubc14\ub85c \uc774\ub97c \ub9d0\ud574\uc900\ub2e4.", "sentence_answer": "\ud55c\ub098\ub77c\uc5d0 \ub300\ud55c \uc678\uad50 \uc815\ucc45\uc5d0\uc11c \uc655\uacfc \uc758\uacac\uc774 \ub9de\uc9c0 \uc54a\uc790 \ud718\ud558\uc758 2\ucc9c \ud638\ub97c \uc774\ub04c\uace0 \ud55c\ubc18\ub3c4 \ub0a8\ubd80\uc9c0\uc5ed\uc73c\ub85c \uc774\ud0c8\ud574 \uac04 \uc870\uc120\uc0c1(\u671d\u9bae\u76f8) \uc5ed\uacc4\uacbd (\u6b77\u8c3f\u537f) \uac19\uc740 \uc774\uac00 \uadf8\ub7f0 \uc608\uc774\ub2e4.", "paragraph_id": "6471479-8-1", "paragraph_question": "question: \uace0\uc870\uc120\uc5d0\uc11c \uad00\uc9c1\uc744 \uac00\uc9c0\uace0 \uc788\uc73c\uba74\uc11c \ub3c5\uc790\uc801\uc778 \uc138\ub825\uae30\ubc18\uc744 \uac00\uc9c4 \uc0ac\ub78c\ub4e4\uc758 \uc608\ub85c \uc870\uc120\uc0c1\uacfc \ub204\uac00\uc788\ub294\uac00?, context: \uadf8\ub7f0\ub370 \uc774\ub7ec\ud55c \uad00\uc9c1\uc744 \uac16\uace0 \uc788\uc73c\uba74\uc11c \ub3c5\uc790\uc801\uc778 \uc138\ub825 \uae30\ubc18\uc744 \uc9c0\ub2cc \uc790\ub4e4\uc774 \uc788\uc5c8\ub2e4. \ud55c\ub098\ub77c\uc5d0 \ub300\ud55c \uc678\uad50 \uc815\ucc45\uc5d0\uc11c \uc655\uacfc \uc758\uacac\uc774 \ub9de\uc9c0 \uc54a\uc790 \ud718\ud558\uc758 2\ucc9c \ud638\ub97c \uc774\ub04c\uace0 \ud55c\ubc18\ub3c4 \ub0a8\ubd80\uc9c0\uc5ed\uc73c\ub85c \uc774\ud0c8\ud574 \uac04 \uc870\uc120\uc0c1(\u671d\u9bae\u76f8) \uc5ed\uacc4\uacbd(\u6b77\u8c3f\u537f) \uac19\uc740 \uc774\uac00 \uadf8\ub7f0 \uc608\uc774\ub2e4. \ud55c\ub098\ub77c\uc640 \uc804\uc7c1 \uc911 \uc804\uc120\uc744 \uc774\ud0c8\ud558\uc5ec \uc218\ub3c4\uac00 \ud568\ub77d\ub418\ub294 \uacb0\uc815\uc801\uc778 \uc6d0\uc778\uc744 \uc81c\uacf5\ud55c \uc774\uacc4\uc0c1(\u5c3c\u8c3f\u76f8) \uc0bc(\u53c3) \ub4f1 \uc138 \uba85\uc758 \u2018\uc0c1(\u76f8)\u2019\ub3c4 \uadf8\ub7ec\ud55c \uc774\ub4e4\ub85c \ucd94\uce21\ub418\ub294\ub370, \uc0c1\uc740 \uc77c\uc815\ud55c \uc138\ub825 \uc9d1\ub2e8\uc758 \ub300\ud45c\ub85c\uc11c \uc911\uc559\uc815\ubd80\uc5d0 \ucc38\uc5ec\ud55c \uc774\ub4e4\uc774 \uc9c0\ub2cc \uad00\uc9c1\uc758 \uc774\ub984\uc73c\ub85c \uc5ec\uaca8\uc9c4\ub2e4. \ub2f9\uc2dc \uc704\ub9cc\uc870\uc120\uc5d0\ub294 \uc5ec\ub7ec \uba85\uc758 \uc0c1\uc774 \uc788\uc5c8\ub294\ub370, \uc774\ub984\uc73c\ub85c \ubcf4\uc544 \ub2e4\uc218\uac00 \uace0\uc870\uc120 \uc778\uc774\uc5c8\ub2e4. \uc0c1\uc758 \ud718\ud558\uc5d0 \uc788\ub358 \uc9d1\ub2e8\uc5d0 \ub300\ud574\uc120 \uc655\uc2e4\uc758 \ud1b5\uc81c\ub825\uc774 \uc5b4\ub290 \uc815\ub3c4 \ubbf8\ucce4\uaca0\uc9c0\ub9cc, \uc801\uc5b4\ub3c4 \uac01 \uc9d1\ub2e8 \ub0b4\ubd80\uc758 \uc77c\uc740 \uc790\uce58\uc801\uc73c\ub85c \uc774\ub8e8\uc5b4\uc84c\uc744 \uac83\uc774\ub2e4. \uc5ed\uacc4\uacbd \ub4f1\uc758 \uc9d1\ub2e8\uc801\uc778 \uc774\ud0c8\ud589\uc704\uac00 \uac00\ub2a5\ud588\ub2e4\ub294 \uc810\uc774 \ubc14\ub85c \uc774\ub97c \ub9d0\ud574\uc900\ub2e4."} {"question": "\uc601\uc5b4\ub294 \uc5b4\ub5a4 \uc5b4\uad70\uc5d0 \uc18d\ud558\ub294\uac00?", "paragraph": "\uc601\uc5b4\ub294 \uc575\uae00\ub85c\ud504\ub9ac\uc9c0\uc544\uc5b4\uc640 \uc800\uc9c0 \uac8c\ub974\ub9cc\uc5b4 \ub97c \uae30\uc6d0\uc73c\ub85c \ud558\ub294 \uc11c\uac8c\ub974\ub9cc\uc5b4\uad70\uc758 \ud558\ub098\uc774\ub2e4. \uc774 \uc5b8\uc5b4\ub4e4\uc740 \ub85c\ub9c8 \uc18d\uc8fc\uc2dc\uae30 \uc774\ud6c4 5\uc138\uae30\uae4c\uc9c0 \uc624\ub298\ub0a0 \ub3c5\uc77c, \ub374\ub9c8\ud06c, \ub124\ub35c\ub780\ub4dc \ub4f1\uc758 \uc9c0\uc5ed\uc5d0 \ud574\ub2f9\ud558\ub294 \uc720\ub7fd \ubd81\uc11c\ubd80\uc5d0\uc11c \ub2e4\uc591\ud55c \uacbd\ub85c\ub97c \ud1b5\ud574 \ube0c\ub9ac\ud2bc \uc12c\uc73c\ub85c \uc720\uc785\ub418\uc5c8\ub2e4. \uc774\ub807\uac8c \ube0c\ub9ac\ud2bc \uc12c\uc5d0 \uc815\ucc29\ud55c \uac8c\ub974\ub9cc\uc871\uc758 \ud558\ub098\uac00 \uc575\uae00\uc871\uc73c\ub85c \uc774\ub4e4\uc758 \uc774\ub984 \uc575\uae00(\uc601\uc5b4: Angles)\uc740 \uc774\ub4e4\uc758 \uc6d0\ub798 \uac70\uc8fc\ud558\uc600\ub358 \uacf3\uc758 \uc9c0\uba85 \uc575\uae00\ub978\uc5d0\uc11c \uc720\ub798\ud55c \uac83\uc73c\ub85c \uc624\ub298\ub0a0\uc758 \uc290\ub808\uc2a4\ube44\ud788 \uc778\uadfc \uc9c0\uc5ed\uc774\ub2e4. \ube0c\ub9ac\ud2bc\uc774\ub77c\ub294 \uc774\ub984\uc740 \ubca0\ub2e4 \ubca0\ub124\ub77c\ube4c\ub9ac\uc2a4\uc758 \u300a\uc789\uae00\ub79c\ub4dc \uad50\ud68c\uc0ac\u300b\uc5d0 \ucd5c\ucd08\ub85c \ub4f1\uc7a5\ud55c\ub2e4. \ube0c\ub9ac\ud2bc \uc12c\uc5d0 \uc774\uc8fc\ud574 \uc628 \uc575\uae00\uc871\uc740 \uc0c8 \uc815\ucc29\uc9c0\ub97c \uc790\uc2e0\ub4e4\uc758 \ub545\uc774\ub77c\ub294 \uc758\ubbf8\uc758 \uc789\uae00\ub79c\ub4dc(\uc601\uc5b4: England, Engla + land)\ub85c \ubd88\ub800\ub2e4. \uc601\uc5b4(\uc601\uc5b4: English)\uc5ed\uc2dc \uc575\uae00\uc871\uc758 \ub9d0\uc774\ub780 \ub73b\uc774\ub2e4.", "answer": "\uc11c\uac8c\ub974\ub9cc\uc5b4\uad70", "sentence": "\uc601\uc5b4\ub294 \uc575\uae00\ub85c\ud504\ub9ac\uc9c0\uc544\uc5b4\uc640 \uc800\uc9c0 \uac8c\ub974\ub9cc\uc5b4 \ub97c \uae30\uc6d0\uc73c\ub85c \ud558\ub294 \uc11c\uac8c\ub974\ub9cc\uc5b4\uad70 \uc758 \ud558\ub098\uc774\ub2e4.", "paragraph_sentence": " \uc601\uc5b4\ub294 \uc575\uae00\ub85c\ud504\ub9ac\uc9c0\uc544\uc5b4\uc640 \uc800\uc9c0 \uac8c\ub974\ub9cc\uc5b4 \ub97c \uae30\uc6d0\uc73c\ub85c \ud558\ub294 \uc11c\uac8c\ub974\ub9cc\uc5b4\uad70 \uc758 \ud558\ub098\uc774\ub2e4. \uc774 \uc5b8\uc5b4\ub4e4\uc740 \ub85c\ub9c8 \uc18d\uc8fc\uc2dc\uae30 \uc774\ud6c4 5\uc138\uae30\uae4c\uc9c0 \uc624\ub298\ub0a0 \ub3c5\uc77c, \ub374\ub9c8\ud06c, \ub124\ub35c\ub780\ub4dc \ub4f1\uc758 \uc9c0\uc5ed\uc5d0 \ud574\ub2f9\ud558\ub294 \uc720\ub7fd \ubd81\uc11c\ubd80\uc5d0\uc11c \ub2e4\uc591\ud55c \uacbd\ub85c\ub97c \ud1b5\ud574 \ube0c\ub9ac\ud2bc \uc12c\uc73c\ub85c \uc720\uc785\ub418\uc5c8\ub2e4. \uc774\ub807\uac8c \ube0c\ub9ac\ud2bc \uc12c\uc5d0 \uc815\ucc29\ud55c \uac8c\ub974\ub9cc\uc871\uc758 \ud558\ub098\uac00 \uc575\uae00\uc871\uc73c\ub85c \uc774\ub4e4\uc758 \uc774\ub984 \uc575\uae00(\uc601\uc5b4: Angles)\uc740 \uc774\ub4e4\uc758 \uc6d0\ub798 \uac70\uc8fc\ud558\uc600\ub358 \uacf3\uc758 \uc9c0\uba85 \uc575\uae00\ub978\uc5d0\uc11c \uc720\ub798\ud55c \uac83\uc73c\ub85c \uc624\ub298\ub0a0\uc758 \uc290\ub808\uc2a4\ube44\ud788 \uc778\uadfc \uc9c0\uc5ed\uc774\ub2e4. \ube0c\ub9ac\ud2bc\uc774\ub77c\ub294 \uc774\ub984\uc740 \ubca0\ub2e4 \ubca0\ub124\ub77c\ube4c\ub9ac\uc2a4\uc758 \u300a\uc789\uae00\ub79c\ub4dc \uad50\ud68c\uc0ac\u300b\uc5d0 \ucd5c\ucd08\ub85c \ub4f1\uc7a5\ud55c\ub2e4. \ube0c\ub9ac\ud2bc \uc12c\uc5d0 \uc774\uc8fc\ud574 \uc628 \uc575\uae00\uc871\uc740 \uc0c8 \uc815\ucc29\uc9c0\ub97c \uc790\uc2e0\ub4e4\uc758 \ub545\uc774\ub77c\ub294 \uc758\ubbf8\uc758 \uc789\uae00\ub79c\ub4dc(\uc601\uc5b4: England, Engla + land)\ub85c \ubd88\ub800\ub2e4. \uc601\uc5b4(\uc601\uc5b4: English)\uc5ed\uc2dc \uc575\uae00\uc871\uc758 \ub9d0\uc774\ub780 \ub73b\uc774\ub2e4.", "paragraph_answer": "\uc601\uc5b4\ub294 \uc575\uae00\ub85c\ud504\ub9ac\uc9c0\uc544\uc5b4\uc640 \uc800\uc9c0 \uac8c\ub974\ub9cc\uc5b4 \ub97c \uae30\uc6d0\uc73c\ub85c \ud558\ub294 \uc11c\uac8c\ub974\ub9cc\uc5b4\uad70 \uc758 \ud558\ub098\uc774\ub2e4. \uc774 \uc5b8\uc5b4\ub4e4\uc740 \ub85c\ub9c8 \uc18d\uc8fc\uc2dc\uae30 \uc774\ud6c4 5\uc138\uae30\uae4c\uc9c0 \uc624\ub298\ub0a0 \ub3c5\uc77c, \ub374\ub9c8\ud06c, \ub124\ub35c\ub780\ub4dc \ub4f1\uc758 \uc9c0\uc5ed\uc5d0 \ud574\ub2f9\ud558\ub294 \uc720\ub7fd \ubd81\uc11c\ubd80\uc5d0\uc11c \ub2e4\uc591\ud55c \uacbd\ub85c\ub97c \ud1b5\ud574 \ube0c\ub9ac\ud2bc \uc12c\uc73c\ub85c \uc720\uc785\ub418\uc5c8\ub2e4. \uc774\ub807\uac8c \ube0c\ub9ac\ud2bc \uc12c\uc5d0 \uc815\ucc29\ud55c \uac8c\ub974\ub9cc\uc871\uc758 \ud558\ub098\uac00 \uc575\uae00\uc871\uc73c\ub85c \uc774\ub4e4\uc758 \uc774\ub984 \uc575\uae00(\uc601\uc5b4: Angles)\uc740 \uc774\ub4e4\uc758 \uc6d0\ub798 \uac70\uc8fc\ud558\uc600\ub358 \uacf3\uc758 \uc9c0\uba85 \uc575\uae00\ub978\uc5d0\uc11c \uc720\ub798\ud55c \uac83\uc73c\ub85c \uc624\ub298\ub0a0\uc758 \uc290\ub808\uc2a4\ube44\ud788 \uc778\uadfc \uc9c0\uc5ed\uc774\ub2e4. \ube0c\ub9ac\ud2bc\uc774\ub77c\ub294 \uc774\ub984\uc740 \ubca0\ub2e4 \ubca0\ub124\ub77c\ube4c\ub9ac\uc2a4\uc758 \u300a\uc789\uae00\ub79c\ub4dc \uad50\ud68c\uc0ac\u300b\uc5d0 \ucd5c\ucd08\ub85c \ub4f1\uc7a5\ud55c\ub2e4. \ube0c\ub9ac\ud2bc \uc12c\uc5d0 \uc774\uc8fc\ud574 \uc628 \uc575\uae00\uc871\uc740 \uc0c8 \uc815\ucc29\uc9c0\ub97c \uc790\uc2e0\ub4e4\uc758 \ub545\uc774\ub77c\ub294 \uc758\ubbf8\uc758 \uc789\uae00\ub79c\ub4dc(\uc601\uc5b4: England, Engla + land)\ub85c \ubd88\ub800\ub2e4. \uc601\uc5b4(\uc601\uc5b4: English)\uc5ed\uc2dc \uc575\uae00\uc871\uc758 \ub9d0\uc774\ub780 \ub73b\uc774\ub2e4.", "sentence_answer": "\uc601\uc5b4\ub294 \uc575\uae00\ub85c\ud504\ub9ac\uc9c0\uc544\uc5b4\uc640 \uc800\uc9c0 \uac8c\ub974\ub9cc\uc5b4 \ub97c \uae30\uc6d0\uc73c\ub85c \ud558\ub294 \uc11c\uac8c\ub974\ub9cc\uc5b4\uad70 \uc758 \ud558\ub098\uc774\ub2e4.", "paragraph_id": "6465644-0-0", "paragraph_question": "question: \uc601\uc5b4\ub294 \uc5b4\ub5a4 \uc5b4\uad70\uc5d0 \uc18d\ud558\ub294\uac00?, context: \uc601\uc5b4\ub294 \uc575\uae00\ub85c\ud504\ub9ac\uc9c0\uc544\uc5b4\uc640 \uc800\uc9c0 \uac8c\ub974\ub9cc\uc5b4 \ub97c \uae30\uc6d0\uc73c\ub85c \ud558\ub294 \uc11c\uac8c\ub974\ub9cc\uc5b4\uad70\uc758 \ud558\ub098\uc774\ub2e4. \uc774 \uc5b8\uc5b4\ub4e4\uc740 \ub85c\ub9c8 \uc18d\uc8fc\uc2dc\uae30 \uc774\ud6c4 5\uc138\uae30\uae4c\uc9c0 \uc624\ub298\ub0a0 \ub3c5\uc77c, \ub374\ub9c8\ud06c, \ub124\ub35c\ub780\ub4dc \ub4f1\uc758 \uc9c0\uc5ed\uc5d0 \ud574\ub2f9\ud558\ub294 \uc720\ub7fd \ubd81\uc11c\ubd80\uc5d0\uc11c \ub2e4\uc591\ud55c \uacbd\ub85c\ub97c \ud1b5\ud574 \ube0c\ub9ac\ud2bc \uc12c\uc73c\ub85c \uc720\uc785\ub418\uc5c8\ub2e4. \uc774\ub807\uac8c \ube0c\ub9ac\ud2bc \uc12c\uc5d0 \uc815\ucc29\ud55c \uac8c\ub974\ub9cc\uc871\uc758 \ud558\ub098\uac00 \uc575\uae00\uc871\uc73c\ub85c \uc774\ub4e4\uc758 \uc774\ub984 \uc575\uae00(\uc601\uc5b4: Angles)\uc740 \uc774\ub4e4\uc758 \uc6d0\ub798 \uac70\uc8fc\ud558\uc600\ub358 \uacf3\uc758 \uc9c0\uba85 \uc575\uae00\ub978\uc5d0\uc11c \uc720\ub798\ud55c \uac83\uc73c\ub85c \uc624\ub298\ub0a0\uc758 \uc290\ub808\uc2a4\ube44\ud788 \uc778\uadfc \uc9c0\uc5ed\uc774\ub2e4. \ube0c\ub9ac\ud2bc\uc774\ub77c\ub294 \uc774\ub984\uc740 \ubca0\ub2e4 \ubca0\ub124\ub77c\ube4c\ub9ac\uc2a4\uc758 \u300a\uc789\uae00\ub79c\ub4dc \uad50\ud68c\uc0ac\u300b\uc5d0 \ucd5c\ucd08\ub85c \ub4f1\uc7a5\ud55c\ub2e4. \ube0c\ub9ac\ud2bc \uc12c\uc5d0 \uc774\uc8fc\ud574 \uc628 \uc575\uae00\uc871\uc740 \uc0c8 \uc815\ucc29\uc9c0\ub97c \uc790\uc2e0\ub4e4\uc758 \ub545\uc774\ub77c\ub294 \uc758\ubbf8\uc758 \uc789\uae00\ub79c\ub4dc(\uc601\uc5b4: England, Engla + land)\ub85c \ubd88\ub800\ub2e4. \uc601\uc5b4(\uc601\uc5b4: English)\uc5ed\uc2dc \uc575\uae00\uc871\uc758 \ub9d0\uc774\ub780 \ub73b\uc774\ub2e4."} {"question": "\uc11c\uc6b8 1945\uc758 \ucd5c\uc6b4\uc6b0 \uc5ed\uc774 KBS \uc5f0\uae30\ub300\uc0c1\uc5d0\uc11c \uc218\uc0c1\ud55c \uc0c1\uc740?", "paragraph": "1998\ub144 KBS \u300a\ucea0\ud37c\uc2a4 \uc601\uc0c1\uac00\uc694 - \uba85\uc9c0\ub300\ud559\uad50\ud3b8\u300b\uc5d0 \ub2f9\uc2dc \ub300\ud559 \ub3d9\uae30\uc600\ub358 \uac1c\uadf8\ub9e8 \uc774\uc2b9\uc724 \ub4f1\uacfc \ud568\uaed8 '\uc804\ud1b5 \ubb34\uc608 \ub3d9\uc544\ub9ac'\ub85c\uc11c \ucc28\ub825\uc1fc\ub97c \uc120\ubcf4\uc5ec 1\ub4f1\uc744 \ud558\uc600\ub2e4. \uc774\ub97c \uacc4\uae30\ub85c \uc787\ub2ec\uc544 SBS \uc694\ub9ac \ud504\ub85c\uadf8\ub7a8 \u300a\ucd5c\uace0\uc758 \ubc25\uc0c1\u300b\uc5d0\uc11c \ub300\ud559\uc0dd\uc73c\ub85c \ucd9c\uc5f0\ud558\uc600\uace0 \u300a\uc9c4\uc2e4\uac8c\uc784 - 40kg\uc744 \ub2e4\uc774\uc5b4\ud2b8\ud55c \ub0a8\uc790\ub294 \ub204\uad6c\uc778\uac00\ud3b8\u300b\uc5d0\uc11c\ub294 '\uc0b4\ucc10 \uc815\uc6b0\uc131'\uc73c\ub85c \ucd9c\uc5f0\ud558\uae30\ub3c4 \ud558\uc600\ub2e4. 2000\ub144 MBC \uc2dc\ud2b8\ucf64\u300a\uae41\uc2a4\uac00\uc871\u300b\uc73c\ub85c \ub370\ubdd4\ud558\uc600\ub2e4. 2002\ub144 SBS \ub4dc\ub77c\ub9c8\u300a\uba85\ub791\uc18c\ub140 \uc131\uacf5\uae30\u300b\uc5d0\uc11c \uc545\uc5ed\uc5f0\uae30\ub97c \ud558\uba74\uc11c \uc774\ub984\uc744 \uc54c\ub838\ub2e4. 2006\ub144 KBS \ub4dc\ub77c\ub9c8\u300a\uc11c\uc6b8 1945\u300b\uc758 \ucd5c\uc6b4\ud601 \uc5ed\uc73c\ub85c KBS \uc5f0\uae30\ub300\uc0c1\uc5d0\uc11c \ucd5c\uc6b0\uc218\uc0c1\uc744 \uc218\uc0c1\ud558\uae30\ub3c4 \ud588\ub2e4. 2017\ub144 1\uc6d4 22\uc77c\uc5d0 \ubc30\uc6b0 \ubc15\ud558\uc120\uacfc \uacb0\ud63c\ud588\ub2e4.", "answer": "\ucd5c\uc6b0\uc218\uc0c1", "sentence": "2006\ub144 KBS \ub4dc\ub77c\ub9c8\u300a\uc11c\uc6b8 1945\u300b\uc758 \ucd5c\uc6b4\ud601 \uc5ed\uc73c\ub85c KBS \uc5f0\uae30\ub300\uc0c1\uc5d0\uc11c \ucd5c\uc6b0\uc218\uc0c1 \uc744 \uc218\uc0c1\ud558\uae30\ub3c4 \ud588\ub2e4.", "paragraph_sentence": "1998\ub144 KBS \u300a\ucea0\ud37c\uc2a4 \uc601\uc0c1\uac00\uc694 - \uba85\uc9c0\ub300\ud559\uad50\ud3b8\u300b\uc5d0 \ub2f9\uc2dc \ub300\ud559 \ub3d9\uae30\uc600\ub358 \uac1c\uadf8\ub9e8 \uc774\uc2b9\uc724 \ub4f1\uacfc \ud568\uaed8 '\uc804\ud1b5 \ubb34\uc608 \ub3d9\uc544\ub9ac'\ub85c\uc11c \ucc28\ub825\uc1fc\ub97c \uc120\ubcf4\uc5ec 1\ub4f1\uc744 \ud558\uc600\ub2e4. \uc774\ub97c \uacc4\uae30\ub85c \uc787\ub2ec\uc544 SBS \uc694\ub9ac \ud504\ub85c\uadf8\ub7a8 \u300a\ucd5c\uace0\uc758 \ubc25\uc0c1\u300b\uc5d0\uc11c \ub300\ud559\uc0dd\uc73c\ub85c \ucd9c\uc5f0\ud558\uc600\uace0 \u300a\uc9c4\uc2e4\uac8c\uc784 - 40kg\uc744 \ub2e4\uc774\uc5b4\ud2b8\ud55c \ub0a8\uc790\ub294 \ub204\uad6c\uc778\uac00\ud3b8\u300b\uc5d0\uc11c\ub294 '\uc0b4\ucc10 \uc815\uc6b0\uc131'\uc73c\ub85c \ucd9c\uc5f0\ud558\uae30\ub3c4 \ud558\uc600\ub2e4. 2000\ub144 MBC \uc2dc\ud2b8\ucf64\u300a\uae41\uc2a4\uac00\uc871\u300b\uc73c\ub85c \ub370\ubdd4\ud558\uc600\ub2e4. 2002\ub144 SBS \ub4dc\ub77c\ub9c8\u300a\uba85\ub791\uc18c\ub140 \uc131\uacf5\uae30\u300b\uc5d0\uc11c \uc545\uc5ed\uc5f0\uae30\ub97c \ud558\uba74\uc11c \uc774\ub984\uc744 \uc54c\ub838\ub2e4. 2006\ub144 KBS \ub4dc\ub77c\ub9c8\u300a\uc11c\uc6b8 1945\u300b\uc758 \ucd5c\uc6b4\ud601 \uc5ed\uc73c\ub85c KBS \uc5f0\uae30\ub300\uc0c1\uc5d0\uc11c \ucd5c\uc6b0\uc218\uc0c1 \uc744 \uc218\uc0c1\ud558\uae30\ub3c4 \ud588\ub2e4. 2017\ub144 1\uc6d4 22\uc77c\uc5d0 \ubc30\uc6b0 \ubc15\ud558\uc120\uacfc \uacb0\ud63c\ud588\ub2e4.", "paragraph_answer": "1998\ub144 KBS \u300a\ucea0\ud37c\uc2a4 \uc601\uc0c1\uac00\uc694 - \uba85\uc9c0\ub300\ud559\uad50\ud3b8\u300b\uc5d0 \ub2f9\uc2dc \ub300\ud559 \ub3d9\uae30\uc600\ub358 \uac1c\uadf8\ub9e8 \uc774\uc2b9\uc724 \ub4f1\uacfc \ud568\uaed8 '\uc804\ud1b5 \ubb34\uc608 \ub3d9\uc544\ub9ac'\ub85c\uc11c \ucc28\ub825\uc1fc\ub97c \uc120\ubcf4\uc5ec 1\ub4f1\uc744 \ud558\uc600\ub2e4. \uc774\ub97c \uacc4\uae30\ub85c \uc787\ub2ec\uc544 SBS \uc694\ub9ac \ud504\ub85c\uadf8\ub7a8 \u300a\ucd5c\uace0\uc758 \ubc25\uc0c1\u300b\uc5d0\uc11c \ub300\ud559\uc0dd\uc73c\ub85c \ucd9c\uc5f0\ud558\uc600\uace0 \u300a\uc9c4\uc2e4\uac8c\uc784 - 40kg\uc744 \ub2e4\uc774\uc5b4\ud2b8\ud55c \ub0a8\uc790\ub294 \ub204\uad6c\uc778\uac00\ud3b8\u300b\uc5d0\uc11c\ub294 '\uc0b4\ucc10 \uc815\uc6b0\uc131'\uc73c\ub85c \ucd9c\uc5f0\ud558\uae30\ub3c4 \ud558\uc600\ub2e4. 2000\ub144 MBC \uc2dc\ud2b8\ucf64\u300a\uae41\uc2a4\uac00\uc871\u300b\uc73c\ub85c \ub370\ubdd4\ud558\uc600\ub2e4. 2002\ub144 SBS \ub4dc\ub77c\ub9c8\u300a\uba85\ub791\uc18c\ub140 \uc131\uacf5\uae30\u300b\uc5d0\uc11c \uc545\uc5ed\uc5f0\uae30\ub97c \ud558\uba74\uc11c \uc774\ub984\uc744 \uc54c\ub838\ub2e4. 2006\ub144 KBS \ub4dc\ub77c\ub9c8\u300a\uc11c\uc6b8 1945\u300b\uc758 \ucd5c\uc6b4\ud601 \uc5ed\uc73c\ub85c KBS \uc5f0\uae30\ub300\uc0c1\uc5d0\uc11c \ucd5c\uc6b0\uc218\uc0c1 \uc744 \uc218\uc0c1\ud558\uae30\ub3c4 \ud588\ub2e4. 2017\ub144 1\uc6d4 22\uc77c\uc5d0 \ubc30\uc6b0 \ubc15\ud558\uc120\uacfc \uacb0\ud63c\ud588\ub2e4.", "sentence_answer": "2006\ub144 KBS \ub4dc\ub77c\ub9c8\u300a\uc11c\uc6b8 1945\u300b\uc758 \ucd5c\uc6b4\ud601 \uc5ed\uc73c\ub85c KBS \uc5f0\uae30\ub300\uc0c1\uc5d0\uc11c \ucd5c\uc6b0\uc218\uc0c1 \uc744 \uc218\uc0c1\ud558\uae30\ub3c4 \ud588\ub2e4.", "paragraph_id": "6311933-0-2", "paragraph_question": "question: \uc11c\uc6b8 1945\uc758 \ucd5c\uc6b4\uc6b0 \uc5ed\uc774 KBS \uc5f0\uae30\ub300\uc0c1\uc5d0\uc11c \uc218\uc0c1\ud55c \uc0c1\uc740?, context: 1998\ub144 KBS \u300a\ucea0\ud37c\uc2a4 \uc601\uc0c1\uac00\uc694 - \uba85\uc9c0\ub300\ud559\uad50\ud3b8\u300b\uc5d0 \ub2f9\uc2dc \ub300\ud559 \ub3d9\uae30\uc600\ub358 \uac1c\uadf8\ub9e8 \uc774\uc2b9\uc724 \ub4f1\uacfc \ud568\uaed8 '\uc804\ud1b5 \ubb34\uc608 \ub3d9\uc544\ub9ac'\ub85c\uc11c \ucc28\ub825\uc1fc\ub97c \uc120\ubcf4\uc5ec 1\ub4f1\uc744 \ud558\uc600\ub2e4. \uc774\ub97c \uacc4\uae30\ub85c \uc787\ub2ec\uc544 SBS \uc694\ub9ac \ud504\ub85c\uadf8\ub7a8 \u300a\ucd5c\uace0\uc758 \ubc25\uc0c1\u300b\uc5d0\uc11c \ub300\ud559\uc0dd\uc73c\ub85c \ucd9c\uc5f0\ud558\uc600\uace0 \u300a\uc9c4\uc2e4\uac8c\uc784 - 40kg\uc744 \ub2e4\uc774\uc5b4\ud2b8\ud55c \ub0a8\uc790\ub294 \ub204\uad6c\uc778\uac00\ud3b8\u300b\uc5d0\uc11c\ub294 '\uc0b4\ucc10 \uc815\uc6b0\uc131'\uc73c\ub85c \ucd9c\uc5f0\ud558\uae30\ub3c4 \ud558\uc600\ub2e4. 2000\ub144 MBC \uc2dc\ud2b8\ucf64\u300a\uae41\uc2a4\uac00\uc871\u300b\uc73c\ub85c \ub370\ubdd4\ud558\uc600\ub2e4. 2002\ub144 SBS \ub4dc\ub77c\ub9c8\u300a\uba85\ub791\uc18c\ub140 \uc131\uacf5\uae30\u300b\uc5d0\uc11c \uc545\uc5ed\uc5f0\uae30\ub97c \ud558\uba74\uc11c \uc774\ub984\uc744 \uc54c\ub838\ub2e4. 2006\ub144 KBS \ub4dc\ub77c\ub9c8\u300a\uc11c\uc6b8 1945\u300b\uc758 \ucd5c\uc6b4\ud601 \uc5ed\uc73c\ub85c KBS \uc5f0\uae30\ub300\uc0c1\uc5d0\uc11c \ucd5c\uc6b0\uc218\uc0c1\uc744 \uc218\uc0c1\ud558\uae30\ub3c4 \ud588\ub2e4. 2017\ub144 1\uc6d4 22\uc77c\uc5d0 \ubc30\uc6b0 \ubc15\ud558\uc120\uacfc \uacb0\ud63c\ud588\ub2e4."} {"question": "\uc791\uc13c \uacf5\uad6d\uc774 \ud504\ub791\uc2a4\uc758 \uc9c0\uc6d0\uc744 \ubc1b\uae30 \uc704\ud574 \uccb4\uacb0\ud55c \uc870\uc57d\uc740?", "paragraph": "\uac00\ud1a8\ub9ad \uad50\ud68c\uc758 \uc7ac\uc0b0\ub4e4\uc744 \uc555\uc218\ud560 \uc218 \uc788\uc5c8\uae30 \ub54c\ubb38\uc5d0 \ub8e8\ud130\uad50\ud68c\ub098 \uc7a5 \uce7c\ubc45\uc758 \uac1c\ud601\uad50\ud68c\ub85c \uac1c\uc885\ud558\ub294 \uc601\uc8fc\ub4e4\uc740 \ub298\uc5b4\ub9cc \uac14\uc73c\uba70, \uc774\uc5b4 \uae30\uc0ac\ub4e4\uacfc \ub3c4\uc2dc\ubbfc\ub4e4\ub3c4 \ub8e8\ud130\uad50\ud68c\ub098 \uac1c\ud601\uad50\ud68c\ub85c \uac1c\uc885\ud558\uae30 \uc2dc\uc791\ud588\ub2e4. \uacb0\uad6d 1526\ub144\uc5d0 \uc5f4\ub9b0 \uc288\ud30c\uc774\uc5b4 \uc81c\uad6d \uc758\ud68c\uc5d0\uc11c, \uc624\uc2a4\ub9cc \uc81c\uad6d \ub4f1\uacfc\uc758 \uc804\uc7c1\uc744 \uc704\ud574 \uc601\uc8fc\ub4e4\uc758 \ud798\uc744 \ud544\uc694\ub85c \ud588\ub358 \uc2e0\uc131 \ub85c\ub9c8 \uc81c\uad6d\uc740 \uc601\uc8fc\ub4e4\uc758 \ub8e8\ud130\uad50\ud68c\uc758 \uc2e0\uc559\uc744 \uc778\uc815\ud588\uc9c0\ub9cc, 1529\ub144 \ube48 \uacf5\ubc29\uc804\uc5d0\uc11c \uc2e0\uc131 \ub85c\ub9c8 \uc81c\uad6d\uc774 \uc2b9\ub9ac\ud55c \uc774\ud6c4 \ud669\uc81c \uce74\ub97c 5\uc138\ub294 \uae30\uc874\uc758 \u300a\ubcf4\ub984\uc2a4 \uce59\ub839\u300b\uc744 \ub2e4\uc2dc \ubc1c\ud45c\ud574 \ub8e8\ud130\uad50\ud68c\ub97c \ud0c4\uc555\ud558\ub824 \ud588\ub2e4. \uac1c\uc2e0\uad50\ub3c4 \uc601\uc8fc\ub4e4\uc740 1531\ub144 2\uc6d4 27\uc77c \ud280\ub9c1\uac90\uc5d0 \ubaa8\uc5ec \uc288\ub9d0\uce7c\ub374 \ub3d9\ub9f9\uc744 \uacb0\uc131\ud558\uace0 \uce74\ub97c 5\uc138\uc640 \ub85c\ub9c8 \uac00\ud1a8\ub9ad \uad50\ud68c\uc5d0 \uc800\ud56d\ud588\ub2e4. \uc288\ub9d0\uce7c\ub374 \ub3d9\ub9f9\uacfc \uc2e0\uc131 \ub85c\ub9c8 \uc81c\uad6d \uc0ac\uc774\uc758 \uc804\uc7c1\uc740 \uc624\uc2a4\ub9cc\ud280\ub974\ud06c \uc81c\uad6d\uc758 \uc7ac\uacf5\uaca9\uacfc \uc791\uc13c \uacf5\uad6d\uc758 \ub3d9\ub9f9 \uc774\ud0c8 \ub4f1\uc73c\ub85c \ud63c\uc120\uc744 \ube5a\ub2e4, \uc791\uc13c \uacf5\uad6d\uc774 \ub2e4\uc2dc \ubc18\ud569\uc2a4\ubd80\ub974\ud06c \ub3d9\ub9f9\uc744 \uacb0\uc131\ud558\uace0 \ud504\ub791\uc2a4 \uc655 \uc559\ub9ac 2\uc138\uc640 \u300a\uc0f9\ubcf4\ub974 \uc870\uc57d\u300b\uc744 \uccb4\uacb0\ud574 \ud504\ub791\uc2a4\uc758 \uc9c0\uc6d0\uc744 \ubc1b\uc558\ub2e4. \ub300\uc2e0 \ubc18\ud569\uc2a4\ubd80\ub974\ud06c \ub3d9\ub9f9\uc740 \ud504\ub791\uc2a4\uc5d0 \uce89\ube0c\ub808, \uba54\uce20, \ud234, \ubca0\ub974\ub42d \ub4f1\uc744 \ub118\uaca8\uc8fc\uae30\ub85c \ud588\ub2e4.", "answer": "\u300a\uc0f9\ubcf4\ub974 \uc870\uc57d\u300b", "sentence": "\uc288\ub9d0\uce7c\ub374 \ub3d9\ub9f9\uacfc \uc2e0\uc131 \ub85c\ub9c8 \uc81c\uad6d \uc0ac\uc774\uc758 \uc804\uc7c1\uc740 \uc624\uc2a4\ub9cc\ud280\ub974\ud06c \uc81c\uad6d\uc758 \uc7ac\uacf5\uaca9\uacfc \uc791\uc13c \uacf5\uad6d\uc758 \ub3d9\ub9f9 \uc774\ud0c8 \ub4f1\uc73c\ub85c \ud63c\uc120\uc744 \ube5a\ub2e4, \uc791\uc13c \uacf5\uad6d\uc774 \ub2e4\uc2dc \ubc18\ud569\uc2a4\ubd80\ub974\ud06c \ub3d9\ub9f9\uc744 \uacb0\uc131\ud558\uace0 \ud504\ub791\uc2a4 \uc655 \uc559\ub9ac 2\uc138\uc640 \u300a\uc0f9\ubcf4\ub974 \uc870\uc57d\u300b \uc744 \uccb4\uacb0\ud574 \ud504\ub791\uc2a4\uc758 \uc9c0\uc6d0\uc744 \ubc1b\uc558\ub2e4.", "paragraph_sentence": "\uac00\ud1a8\ub9ad \uad50\ud68c\uc758 \uc7ac\uc0b0\ub4e4\uc744 \uc555\uc218\ud560 \uc218 \uc788\uc5c8\uae30 \ub54c\ubb38\uc5d0 \ub8e8\ud130\uad50\ud68c\ub098 \uc7a5 \uce7c\ubc45\uc758 \uac1c\ud601\uad50\ud68c\ub85c \uac1c\uc885\ud558\ub294 \uc601\uc8fc\ub4e4\uc740 \ub298\uc5b4\ub9cc \uac14\uc73c\uba70, \uc774\uc5b4 \uae30\uc0ac\ub4e4\uacfc \ub3c4\uc2dc\ubbfc\ub4e4\ub3c4 \ub8e8\ud130\uad50\ud68c\ub098 \uac1c\ud601\uad50\ud68c\ub85c \uac1c\uc885\ud558\uae30 \uc2dc\uc791\ud588\ub2e4. \uacb0\uad6d 1526\ub144\uc5d0 \uc5f4\ub9b0 \uc288\ud30c\uc774\uc5b4 \uc81c\uad6d \uc758\ud68c\uc5d0\uc11c, \uc624\uc2a4\ub9cc \uc81c\uad6d \ub4f1\uacfc\uc758 \uc804\uc7c1\uc744 \uc704\ud574 \uc601\uc8fc\ub4e4\uc758 \ud798\uc744 \ud544\uc694\ub85c \ud588\ub358 \uc2e0\uc131 \ub85c\ub9c8 \uc81c\uad6d\uc740 \uc601\uc8fc\ub4e4\uc758 \ub8e8\ud130\uad50\ud68c\uc758 \uc2e0\uc559\uc744 \uc778\uc815\ud588\uc9c0\ub9cc, 1529\ub144 \ube48 \uacf5\ubc29\uc804\uc5d0\uc11c \uc2e0\uc131 \ub85c\ub9c8 \uc81c\uad6d\uc774 \uc2b9\ub9ac\ud55c \uc774\ud6c4 \ud669\uc81c \uce74\ub97c 5\uc138\ub294 \uae30\uc874\uc758 \u300a\ubcf4\ub984\uc2a4 \uce59\ub839\u300b\uc744 \ub2e4\uc2dc \ubc1c\ud45c\ud574 \ub8e8\ud130\uad50\ud68c\ub97c \ud0c4\uc555\ud558\ub824 \ud588\ub2e4. \uac1c\uc2e0\uad50\ub3c4 \uc601\uc8fc\ub4e4\uc740 1531\ub144 2\uc6d4 27\uc77c \ud280\ub9c1\uac90\uc5d0 \ubaa8\uc5ec \uc288\ub9d0\uce7c\ub374 \ub3d9\ub9f9\uc744 \uacb0\uc131\ud558\uace0 \uce74\ub97c 5\uc138\uc640 \ub85c\ub9c8 \uac00\ud1a8\ub9ad \uad50\ud68c\uc5d0 \uc800\ud56d\ud588\ub2e4. \uc288\ub9d0\uce7c\ub374 \ub3d9\ub9f9\uacfc \uc2e0\uc131 \ub85c\ub9c8 \uc81c\uad6d \uc0ac\uc774\uc758 \uc804\uc7c1\uc740 \uc624\uc2a4\ub9cc\ud280\ub974\ud06c \uc81c\uad6d\uc758 \uc7ac\uacf5\uaca9\uacfc \uc791\uc13c \uacf5\uad6d\uc758 \ub3d9\ub9f9 \uc774\ud0c8 \ub4f1\uc73c\ub85c \ud63c\uc120\uc744 \ube5a\ub2e4, \uc791\uc13c \uacf5\uad6d\uc774 \ub2e4\uc2dc \ubc18\ud569\uc2a4\ubd80\ub974\ud06c \ub3d9\ub9f9\uc744 \uacb0\uc131\ud558\uace0 \ud504\ub791\uc2a4 \uc655 \uc559\ub9ac 2\uc138\uc640 \u300a\uc0f9\ubcf4\ub974 \uc870\uc57d\u300b \uc744 \uccb4\uacb0\ud574 \ud504\ub791\uc2a4\uc758 \uc9c0\uc6d0\uc744 \ubc1b\uc558\ub2e4. \ub300\uc2e0 \ubc18\ud569\uc2a4\ubd80\ub974\ud06c \ub3d9\ub9f9\uc740 \ud504\ub791\uc2a4\uc5d0 \uce89\ube0c\ub808, \uba54\uce20, \ud234, \ubca0\ub974\ub42d \ub4f1\uc744 \ub118\uaca8\uc8fc\uae30\ub85c \ud588\ub2e4.", "paragraph_answer": "\uac00\ud1a8\ub9ad \uad50\ud68c\uc758 \uc7ac\uc0b0\ub4e4\uc744 \uc555\uc218\ud560 \uc218 \uc788\uc5c8\uae30 \ub54c\ubb38\uc5d0 \ub8e8\ud130\uad50\ud68c\ub098 \uc7a5 \uce7c\ubc45\uc758 \uac1c\ud601\uad50\ud68c\ub85c \uac1c\uc885\ud558\ub294 \uc601\uc8fc\ub4e4\uc740 \ub298\uc5b4\ub9cc \uac14\uc73c\uba70, \uc774\uc5b4 \uae30\uc0ac\ub4e4\uacfc \ub3c4\uc2dc\ubbfc\ub4e4\ub3c4 \ub8e8\ud130\uad50\ud68c\ub098 \uac1c\ud601\uad50\ud68c\ub85c \uac1c\uc885\ud558\uae30 \uc2dc\uc791\ud588\ub2e4. \uacb0\uad6d 1526\ub144\uc5d0 \uc5f4\ub9b0 \uc288\ud30c\uc774\uc5b4 \uc81c\uad6d \uc758\ud68c\uc5d0\uc11c, \uc624\uc2a4\ub9cc \uc81c\uad6d \ub4f1\uacfc\uc758 \uc804\uc7c1\uc744 \uc704\ud574 \uc601\uc8fc\ub4e4\uc758 \ud798\uc744 \ud544\uc694\ub85c \ud588\ub358 \uc2e0\uc131 \ub85c\ub9c8 \uc81c\uad6d\uc740 \uc601\uc8fc\ub4e4\uc758 \ub8e8\ud130\uad50\ud68c\uc758 \uc2e0\uc559\uc744 \uc778\uc815\ud588\uc9c0\ub9cc, 1529\ub144 \ube48 \uacf5\ubc29\uc804\uc5d0\uc11c \uc2e0\uc131 \ub85c\ub9c8 \uc81c\uad6d\uc774 \uc2b9\ub9ac\ud55c \uc774\ud6c4 \ud669\uc81c \uce74\ub97c 5\uc138\ub294 \uae30\uc874\uc758 \u300a\ubcf4\ub984\uc2a4 \uce59\ub839\u300b\uc744 \ub2e4\uc2dc \ubc1c\ud45c\ud574 \ub8e8\ud130\uad50\ud68c\ub97c \ud0c4\uc555\ud558\ub824 \ud588\ub2e4. \uac1c\uc2e0\uad50\ub3c4 \uc601\uc8fc\ub4e4\uc740 1531\ub144 2\uc6d4 27\uc77c \ud280\ub9c1\uac90\uc5d0 \ubaa8\uc5ec \uc288\ub9d0\uce7c\ub374 \ub3d9\ub9f9\uc744 \uacb0\uc131\ud558\uace0 \uce74\ub97c 5\uc138\uc640 \ub85c\ub9c8 \uac00\ud1a8\ub9ad \uad50\ud68c\uc5d0 \uc800\ud56d\ud588\ub2e4. \uc288\ub9d0\uce7c\ub374 \ub3d9\ub9f9\uacfc \uc2e0\uc131 \ub85c\ub9c8 \uc81c\uad6d \uc0ac\uc774\uc758 \uc804\uc7c1\uc740 \uc624\uc2a4\ub9cc\ud280\ub974\ud06c \uc81c\uad6d\uc758 \uc7ac\uacf5\uaca9\uacfc \uc791\uc13c \uacf5\uad6d\uc758 \ub3d9\ub9f9 \uc774\ud0c8 \ub4f1\uc73c\ub85c \ud63c\uc120\uc744 \ube5a\ub2e4, \uc791\uc13c \uacf5\uad6d\uc774 \ub2e4\uc2dc \ubc18\ud569\uc2a4\ubd80\ub974\ud06c \ub3d9\ub9f9\uc744 \uacb0\uc131\ud558\uace0 \ud504\ub791\uc2a4 \uc655 \uc559\ub9ac 2\uc138\uc640 \u300a\uc0f9\ubcf4\ub974 \uc870\uc57d\u300b \uc744 \uccb4\uacb0\ud574 \ud504\ub791\uc2a4\uc758 \uc9c0\uc6d0\uc744 \ubc1b\uc558\ub2e4. \ub300\uc2e0 \ubc18\ud569\uc2a4\ubd80\ub974\ud06c \ub3d9\ub9f9\uc740 \ud504\ub791\uc2a4\uc5d0 \uce89\ube0c\ub808, \uba54\uce20, \ud234, \ubca0\ub974\ub42d \ub4f1\uc744 \ub118\uaca8\uc8fc\uae30\ub85c \ud588\ub2e4.", "sentence_answer": "\uc288\ub9d0\uce7c\ub374 \ub3d9\ub9f9\uacfc \uc2e0\uc131 \ub85c\ub9c8 \uc81c\uad6d \uc0ac\uc774\uc758 \uc804\uc7c1\uc740 \uc624\uc2a4\ub9cc\ud280\ub974\ud06c \uc81c\uad6d\uc758 \uc7ac\uacf5\uaca9\uacfc \uc791\uc13c \uacf5\uad6d\uc758 \ub3d9\ub9f9 \uc774\ud0c8 \ub4f1\uc73c\ub85c \ud63c\uc120\uc744 \ube5a\ub2e4, \uc791\uc13c \uacf5\uad6d\uc774 \ub2e4\uc2dc \ubc18\ud569\uc2a4\ubd80\ub974\ud06c \ub3d9\ub9f9\uc744 \uacb0\uc131\ud558\uace0 \ud504\ub791\uc2a4 \uc655 \uc559\ub9ac 2\uc138\uc640 \u300a\uc0f9\ubcf4\ub974 \uc870\uc57d\u300b \uc744 \uccb4\uacb0\ud574 \ud504\ub791\uc2a4\uc758 \uc9c0\uc6d0\uc744 \ubc1b\uc558\ub2e4.", "paragraph_id": "5782587-8-2", "paragraph_question": "question: \uc791\uc13c \uacf5\uad6d\uc774 \ud504\ub791\uc2a4\uc758 \uc9c0\uc6d0\uc744 \ubc1b\uae30 \uc704\ud574 \uccb4\uacb0\ud55c \uc870\uc57d\uc740?, context: \uac00\ud1a8\ub9ad \uad50\ud68c\uc758 \uc7ac\uc0b0\ub4e4\uc744 \uc555\uc218\ud560 \uc218 \uc788\uc5c8\uae30 \ub54c\ubb38\uc5d0 \ub8e8\ud130\uad50\ud68c\ub098 \uc7a5 \uce7c\ubc45\uc758 \uac1c\ud601\uad50\ud68c\ub85c \uac1c\uc885\ud558\ub294 \uc601\uc8fc\ub4e4\uc740 \ub298\uc5b4\ub9cc \uac14\uc73c\uba70, \uc774\uc5b4 \uae30\uc0ac\ub4e4\uacfc \ub3c4\uc2dc\ubbfc\ub4e4\ub3c4 \ub8e8\ud130\uad50\ud68c\ub098 \uac1c\ud601\uad50\ud68c\ub85c \uac1c\uc885\ud558\uae30 \uc2dc\uc791\ud588\ub2e4. \uacb0\uad6d 1526\ub144\uc5d0 \uc5f4\ub9b0 \uc288\ud30c\uc774\uc5b4 \uc81c\uad6d \uc758\ud68c\uc5d0\uc11c, \uc624\uc2a4\ub9cc \uc81c\uad6d \ub4f1\uacfc\uc758 \uc804\uc7c1\uc744 \uc704\ud574 \uc601\uc8fc\ub4e4\uc758 \ud798\uc744 \ud544\uc694\ub85c \ud588\ub358 \uc2e0\uc131 \ub85c\ub9c8 \uc81c\uad6d\uc740 \uc601\uc8fc\ub4e4\uc758 \ub8e8\ud130\uad50\ud68c\uc758 \uc2e0\uc559\uc744 \uc778\uc815\ud588\uc9c0\ub9cc, 1529\ub144 \ube48 \uacf5\ubc29\uc804\uc5d0\uc11c \uc2e0\uc131 \ub85c\ub9c8 \uc81c\uad6d\uc774 \uc2b9\ub9ac\ud55c \uc774\ud6c4 \ud669\uc81c \uce74\ub97c 5\uc138\ub294 \uae30\uc874\uc758 \u300a\ubcf4\ub984\uc2a4 \uce59\ub839\u300b\uc744 \ub2e4\uc2dc \ubc1c\ud45c\ud574 \ub8e8\ud130\uad50\ud68c\ub97c \ud0c4\uc555\ud558\ub824 \ud588\ub2e4. \uac1c\uc2e0\uad50\ub3c4 \uc601\uc8fc\ub4e4\uc740 1531\ub144 2\uc6d4 27\uc77c \ud280\ub9c1\uac90\uc5d0 \ubaa8\uc5ec \uc288\ub9d0\uce7c\ub374 \ub3d9\ub9f9\uc744 \uacb0\uc131\ud558\uace0 \uce74\ub97c 5\uc138\uc640 \ub85c\ub9c8 \uac00\ud1a8\ub9ad \uad50\ud68c\uc5d0 \uc800\ud56d\ud588\ub2e4. \uc288\ub9d0\uce7c\ub374 \ub3d9\ub9f9\uacfc \uc2e0\uc131 \ub85c\ub9c8 \uc81c\uad6d \uc0ac\uc774\uc758 \uc804\uc7c1\uc740 \uc624\uc2a4\ub9cc\ud280\ub974\ud06c \uc81c\uad6d\uc758 \uc7ac\uacf5\uaca9\uacfc \uc791\uc13c \uacf5\uad6d\uc758 \ub3d9\ub9f9 \uc774\ud0c8 \ub4f1\uc73c\ub85c \ud63c\uc120\uc744 \ube5a\ub2e4, \uc791\uc13c \uacf5\uad6d\uc774 \ub2e4\uc2dc \ubc18\ud569\uc2a4\ubd80\ub974\ud06c \ub3d9\ub9f9\uc744 \uacb0\uc131\ud558\uace0 \ud504\ub791\uc2a4 \uc655 \uc559\ub9ac 2\uc138\uc640 \u300a\uc0f9\ubcf4\ub974 \uc870\uc57d\u300b\uc744 \uccb4\uacb0\ud574 \ud504\ub791\uc2a4\uc758 \uc9c0\uc6d0\uc744 \ubc1b\uc558\ub2e4. \ub300\uc2e0 \ubc18\ud569\uc2a4\ubd80\ub974\ud06c \ub3d9\ub9f9\uc740 \ud504\ub791\uc2a4\uc5d0 \uce89\ube0c\ub808, \uba54\uce20, \ud234, \ubca0\ub974\ub42d \ub4f1\uc744 \ub118\uaca8\uc8fc\uae30\ub85c \ud588\ub2e4."} {"question": "\uc5f0\ud76c\uc804\ubb38\ud559\uad50\uac00 \uacbd\uae30\ub3c4 \uace0\uc591\uad70 \uc5f0\ud76c\uba74\uc5d0\uc11c \ub9e4\uc785\ud55c \ud1a0\uc9c0\uc758 \ud3c9\uc218\ub294?", "paragraph": "\uc2e0\ucd0c\ucea0\ud37c\uc2a4(\u65b0\u6751-)\ub294 \uc11c\uc6b8\ud2b9\ubcc4\uc2dc \uc11c\ub300\ubb38\uad6c \uc5f0\uc138\ub85c 50 (\uc2e0\ucd0c\ub3d9)\uc5d0 \uc704\uce58\ud55c \uc5f0\uc138\ub300\ud559\uad50\uc758 \uc81c1\ucea0\ud37c\uc2a4\uc774\ub2e4. \uc5b8\ub354\uc6b0\ub4dc\uc758 \ud070\ud615\uc778 \uc874 \ud1a0\ub9c8\uc2a4 \uc5b8\ub354\uc6b0\ub4dc(John Tomas Underwood)\ub294 10\ub9cc \ub2ec\ub7ec\ub97c, \ub85c\uc2a4\uc564\uc824\ub808\uc2a4\uc758 \ucc30\uc2a4 \uc2a4\ud300\uc2a8(Charles S. M. Stimson)\uc740 \ub300\ud559 \uc124\ub9bd\uc6a9 \uc790\uae08\uc73c\ub85c 2\ub9cc 5\ucc9c \ub2ec\ub7ec\ub97c \uc5f0\ud76c\uc804\ubb38\ud559\uad50\uc5d0 \uae30\ubd80\ud558\uc600\ub2e4. \uc5b8\ub354\uc6b0\ub4dc\ub294 \ubbf8\uad6d\uc5d0 \ub3cc\uc544\uac00\uc11c \uad50\uc0ac \uac74\ub9bd\uc6a9 \uae30\ubd80\uae08 \ubaa8\uae08\uc744 \ud588\ub294\ub370 \uc2a4\ud300\uc2a8\uc5d0\uac8c\uc11c \uc704 \uae30\ubd80\uae08\uc744 \uc5bb\uc740 \ud6c4 \uc0ac\ub9dd\ud558\uc600\uace0, \ud6c4\uc784 \uad50\uc7a5\uc778 \uc5d0\ube44\uc2a8\uc774 \uae30\ubd80\uae08\uc73c\ub85c \uad50\uc0ac\ub97c \uac74\ub9bd\ud558\uc600\ub2e4. \uc5f0\ud76c\uc804\ubb38\ud559\uad50\ub294 \ub2f9\uc2dc \uacbd\uae30\ub3c4 \uace0\uc591\uad70 \uc5f0\ud76c\uba74 \uc2e0\ucd0c\ub9ac\uc758 \uc232\uc774 \uc6b8\ucc3d\ud55c \ud1a0\uc9c0 190,320\ud3c9\uc744 \ub9e4\uc785\ud558\uc600\ub2e4. \uc774\uacf3\uc740 \uc870\uc120 \uc815\uc885 \ub54c \uc138\uc6cc\uc9c4 \uc5f0\ud76c\uad81(\u5ef6\u79a7\u5bae) \ud130\uc640 \uc601\uc870(\u82f1\u7956)\uc758 \ud6c4\uad81 \uc601\ube48 \uc774\uc528(\u6620\u5b2a\u674e\u6c0f)\uc758 \uc218\uacbd\uc6d0(\u7dac\u6176\u5712)\uc774 \uc790\ub9ac \uc7a1\uace0 \uc788\uc5c8\uace0, \ud559\uad50\uba85 \u2018\uc5f0\ud76c\u2019\ub294 \uc5ec\uae30\uc11c \uc5f0\uc720\ud558\uc600\ub2e4. \ucea0\ud37c\uc2a4 \ub9c8\uc2a4\ud130\ud50c\ub79c\uacfc \uad50\uc0ac\ub3d9 \uc124\uacc4\ub294 \uc608\uc77c \ub300\ud559\uad50 \uac74\ucd95\uacfc\ub97c \uc878\uc5c5\ud55c \ubbf8\uad6d\uc778 \uac74\ucd95\uac00 \ud5e8\ub9ac \uba38\ud53c(Henry Killian Murphy)\uc5d0 \uc758\ud558\uc5ec \uc791\uc131\ub418\uc5c8\ub2e4. \ud604\uc7ac\uc758 \ubc31\uc591\ub85c \ub4f1 \uc2e0\ucd0c\ucea0\ud37c\uc2a4\uc758 \uace8\uaca9\uc740 \ucd08\uae30 \ub9c8\uc2a4\ud130\ud50c\ub79c\uc5d0 \uc758\ud574 \uc870\uc131\ub418\uc5c8\ub2e4. \ucd5c\ucd08\uc758 \uac74\ubb3c\uc740 \uce58\uc6d0\uad00(\u81f4\u9060\u9928)\uc774\uba70, \ubaa9\uc870 2\uce35\uc73c\ub85c 1918\ub144\uc5d0 \uc900\uacf5\ub418\uc5b4 1950\ub144\uae4c\uc9c0 \uc788\uc5c8\ub2e4. \uce58\uc6d0\uad00 \uc774\uc678\uc5d0\ub3c4 \uc2a4\ud300\uc2a8\uad00, \uc5b8\ub354\uc6b0\ub4dc\uad00, \uc544\ud39c\uc824\ub7ec\uad00, \ud55c\uacbd\uad00 \ub4f1\uc774 \uc900\uacf5\ub418\uc5c8\uc73c\uba70, \uae30\uc219\uc0ac\uc778 \ud540\uc2a8\uad00\uacfc \ub178\ucc9c\uadf9\uc7a5, \ub3cc\uce35\uacc4 \ub4f1\uc774 \ub9cc\ub4e4\uc5b4\uc84c\ub2e4.", "answer": "190,320\ud3c9", "sentence": "\uc5f0\ud76c\uc804\ubb38\ud559\uad50\ub294 \ub2f9\uc2dc \uacbd\uae30\ub3c4 \uace0\uc591\uad70 \uc5f0\ud76c\uba74 \uc2e0\ucd0c\ub9ac\uc758 \uc232\uc774 \uc6b8\ucc3d\ud55c \ud1a0\uc9c0 190,320\ud3c9 \uc744 \ub9e4\uc785\ud558\uc600\ub2e4.", "paragraph_sentence": "\uc2e0\ucd0c\ucea0\ud37c\uc2a4(\u65b0\u6751-)\ub294 \uc11c\uc6b8\ud2b9\ubcc4\uc2dc \uc11c\ub300\ubb38\uad6c \uc5f0\uc138\ub85c 50 (\uc2e0\ucd0c\ub3d9)\uc5d0 \uc704\uce58\ud55c \uc5f0\uc138\ub300\ud559\uad50\uc758 \uc81c1\ucea0\ud37c\uc2a4\uc774\ub2e4. \uc5b8\ub354\uc6b0\ub4dc\uc758 \ud070\ud615\uc778 \uc874 \ud1a0\ub9c8\uc2a4 \uc5b8\ub354\uc6b0\ub4dc(John Tomas Underwood)\ub294 10\ub9cc \ub2ec\ub7ec\ub97c, \ub85c\uc2a4\uc564\uc824\ub808\uc2a4\uc758 \ucc30\uc2a4 \uc2a4\ud300\uc2a8(Charles S. M. Stimson)\uc740 \ub300\ud559 \uc124\ub9bd\uc6a9 \uc790\uae08\uc73c\ub85c 2\ub9cc 5\ucc9c \ub2ec\ub7ec\ub97c \uc5f0\ud76c\uc804\ubb38\ud559\uad50\uc5d0 \uae30\ubd80\ud558\uc600\ub2e4. \uc5b8\ub354\uc6b0\ub4dc\ub294 \ubbf8\uad6d\uc5d0 \ub3cc\uc544\uac00\uc11c \uad50\uc0ac \uac74\ub9bd\uc6a9 \uae30\ubd80\uae08 \ubaa8\uae08\uc744 \ud588\ub294\ub370 \uc2a4\ud300\uc2a8\uc5d0\uac8c\uc11c \uc704 \uae30\ubd80\uae08\uc744 \uc5bb\uc740 \ud6c4 \uc0ac\ub9dd\ud558\uc600\uace0, \ud6c4\uc784 \uad50\uc7a5\uc778 \uc5d0\ube44\uc2a8\uc774 \uae30\ubd80\uae08\uc73c\ub85c \uad50\uc0ac\ub97c \uac74\ub9bd\ud558\uc600\ub2e4. \uc5f0\ud76c\uc804\ubb38\ud559\uad50\ub294 \ub2f9\uc2dc \uacbd\uae30\ub3c4 \uace0\uc591\uad70 \uc5f0\ud76c\uba74 \uc2e0\ucd0c\ub9ac\uc758 \uc232\uc774 \uc6b8\ucc3d\ud55c \ud1a0\uc9c0 190,320\ud3c9 \uc744 \ub9e4\uc785\ud558\uc600\ub2e4. \uc774\uacf3\uc740 \uc870\uc120 \uc815\uc885 \ub54c \uc138\uc6cc\uc9c4 \uc5f0\ud76c\uad81(\u5ef6\u79a7\u5bae) \ud130\uc640 \uc601\uc870(\u82f1\u7956)\uc758 \ud6c4\uad81 \uc601\ube48 \uc774\uc528(\u6620\u5b2a\u674e\u6c0f)\uc758 \uc218\uacbd\uc6d0(\u7dac\u6176\u5712)\uc774 \uc790\ub9ac \uc7a1\uace0 \uc788\uc5c8\uace0, \ud559\uad50\uba85 \u2018\uc5f0\ud76c\u2019\ub294 \uc5ec\uae30\uc11c \uc5f0\uc720\ud558\uc600\ub2e4. \ucea0\ud37c\uc2a4 \ub9c8\uc2a4\ud130\ud50c\ub79c\uacfc \uad50\uc0ac\ub3d9 \uc124\uacc4\ub294 \uc608\uc77c \ub300\ud559\uad50 \uac74\ucd95\uacfc\ub97c \uc878\uc5c5\ud55c \ubbf8\uad6d\uc778 \uac74\ucd95\uac00 \ud5e8\ub9ac \uba38\ud53c(Henry Killian Murphy)\uc5d0 \uc758\ud558\uc5ec \uc791\uc131\ub418\uc5c8\ub2e4. \ud604\uc7ac\uc758 \ubc31\uc591\ub85c \ub4f1 \uc2e0\ucd0c\ucea0\ud37c\uc2a4\uc758 \uace8\uaca9\uc740 \ucd08\uae30 \ub9c8\uc2a4\ud130\ud50c\ub79c\uc5d0 \uc758\ud574 \uc870\uc131\ub418\uc5c8\ub2e4. \ucd5c\ucd08\uc758 \uac74\ubb3c\uc740 \uce58\uc6d0\uad00(\u81f4\u9060\u9928)\uc774\uba70, \ubaa9\uc870 2\uce35\uc73c\ub85c 1918\ub144\uc5d0 \uc900\uacf5\ub418\uc5b4 1950\ub144\uae4c\uc9c0 \uc788\uc5c8\ub2e4. \uce58\uc6d0\uad00 \uc774\uc678\uc5d0\ub3c4 \uc2a4\ud300\uc2a8\uad00, \uc5b8\ub354\uc6b0\ub4dc\uad00, \uc544\ud39c\uc824\ub7ec\uad00, \ud55c\uacbd\uad00 \ub4f1\uc774 \uc900\uacf5\ub418\uc5c8\uc73c\uba70, \uae30\uc219\uc0ac\uc778 \ud540\uc2a8\uad00\uacfc \ub178\ucc9c\uadf9\uc7a5, \ub3cc\uce35\uacc4 \ub4f1\uc774 \ub9cc\ub4e4\uc5b4\uc84c\ub2e4.", "paragraph_answer": "\uc2e0\ucd0c\ucea0\ud37c\uc2a4(\u65b0\u6751-)\ub294 \uc11c\uc6b8\ud2b9\ubcc4\uc2dc \uc11c\ub300\ubb38\uad6c \uc5f0\uc138\ub85c 50 (\uc2e0\ucd0c\ub3d9)\uc5d0 \uc704\uce58\ud55c \uc5f0\uc138\ub300\ud559\uad50\uc758 \uc81c1\ucea0\ud37c\uc2a4\uc774\ub2e4. \uc5b8\ub354\uc6b0\ub4dc\uc758 \ud070\ud615\uc778 \uc874 \ud1a0\ub9c8\uc2a4 \uc5b8\ub354\uc6b0\ub4dc(John Tomas Underwood)\ub294 10\ub9cc \ub2ec\ub7ec\ub97c, \ub85c\uc2a4\uc564\uc824\ub808\uc2a4\uc758 \ucc30\uc2a4 \uc2a4\ud300\uc2a8(Charles S. M. Stimson)\uc740 \ub300\ud559 \uc124\ub9bd\uc6a9 \uc790\uae08\uc73c\ub85c 2\ub9cc 5\ucc9c \ub2ec\ub7ec\ub97c \uc5f0\ud76c\uc804\ubb38\ud559\uad50\uc5d0 \uae30\ubd80\ud558\uc600\ub2e4. \uc5b8\ub354\uc6b0\ub4dc\ub294 \ubbf8\uad6d\uc5d0 \ub3cc\uc544\uac00\uc11c \uad50\uc0ac \uac74\ub9bd\uc6a9 \uae30\ubd80\uae08 \ubaa8\uae08\uc744 \ud588\ub294\ub370 \uc2a4\ud300\uc2a8\uc5d0\uac8c\uc11c \uc704 \uae30\ubd80\uae08\uc744 \uc5bb\uc740 \ud6c4 \uc0ac\ub9dd\ud558\uc600\uace0, \ud6c4\uc784 \uad50\uc7a5\uc778 \uc5d0\ube44\uc2a8\uc774 \uae30\ubd80\uae08\uc73c\ub85c \uad50\uc0ac\ub97c \uac74\ub9bd\ud558\uc600\ub2e4. \uc5f0\ud76c\uc804\ubb38\ud559\uad50\ub294 \ub2f9\uc2dc \uacbd\uae30\ub3c4 \uace0\uc591\uad70 \uc5f0\ud76c\uba74 \uc2e0\ucd0c\ub9ac\uc758 \uc232\uc774 \uc6b8\ucc3d\ud55c \ud1a0\uc9c0 190,320\ud3c9 \uc744 \ub9e4\uc785\ud558\uc600\ub2e4. \uc774\uacf3\uc740 \uc870\uc120 \uc815\uc885 \ub54c \uc138\uc6cc\uc9c4 \uc5f0\ud76c\uad81(\u5ef6\u79a7\u5bae) \ud130\uc640 \uc601\uc870(\u82f1\u7956)\uc758 \ud6c4\uad81 \uc601\ube48 \uc774\uc528(\u6620\u5b2a\u674e\u6c0f)\uc758 \uc218\uacbd\uc6d0(\u7dac\u6176\u5712)\uc774 \uc790\ub9ac \uc7a1\uace0 \uc788\uc5c8\uace0, \ud559\uad50\uba85 \u2018\uc5f0\ud76c\u2019\ub294 \uc5ec\uae30\uc11c \uc5f0\uc720\ud558\uc600\ub2e4. \ucea0\ud37c\uc2a4 \ub9c8\uc2a4\ud130\ud50c\ub79c\uacfc \uad50\uc0ac\ub3d9 \uc124\uacc4\ub294 \uc608\uc77c \ub300\ud559\uad50 \uac74\ucd95\uacfc\ub97c \uc878\uc5c5\ud55c \ubbf8\uad6d\uc778 \uac74\ucd95\uac00 \ud5e8\ub9ac \uba38\ud53c(Henry Killian Murphy)\uc5d0 \uc758\ud558\uc5ec \uc791\uc131\ub418\uc5c8\ub2e4. \ud604\uc7ac\uc758 \ubc31\uc591\ub85c \ub4f1 \uc2e0\ucd0c\ucea0\ud37c\uc2a4\uc758 \uace8\uaca9\uc740 \ucd08\uae30 \ub9c8\uc2a4\ud130\ud50c\ub79c\uc5d0 \uc758\ud574 \uc870\uc131\ub418\uc5c8\ub2e4. \ucd5c\ucd08\uc758 \uac74\ubb3c\uc740 \uce58\uc6d0\uad00(\u81f4\u9060\u9928)\uc774\uba70, \ubaa9\uc870 2\uce35\uc73c\ub85c 1918\ub144\uc5d0 \uc900\uacf5\ub418\uc5b4 1950\ub144\uae4c\uc9c0 \uc788\uc5c8\ub2e4. \uce58\uc6d0\uad00 \uc774\uc678\uc5d0\ub3c4 \uc2a4\ud300\uc2a8\uad00, \uc5b8\ub354\uc6b0\ub4dc\uad00, \uc544\ud39c\uc824\ub7ec\uad00, \ud55c\uacbd\uad00 \ub4f1\uc774 \uc900\uacf5\ub418\uc5c8\uc73c\uba70, \uae30\uc219\uc0ac\uc778 \ud540\uc2a8\uad00\uacfc \ub178\ucc9c\uadf9\uc7a5, \ub3cc\uce35\uacc4 \ub4f1\uc774 \ub9cc\ub4e4\uc5b4\uc84c\ub2e4.", "sentence_answer": "\uc5f0\ud76c\uc804\ubb38\ud559\uad50\ub294 \ub2f9\uc2dc \uacbd\uae30\ub3c4 \uace0\uc591\uad70 \uc5f0\ud76c\uba74 \uc2e0\ucd0c\ub9ac\uc758 \uc232\uc774 \uc6b8\ucc3d\ud55c \ud1a0\uc9c0 190,320\ud3c9 \uc744 \ub9e4\uc785\ud558\uc600\ub2e4.", "paragraph_id": "6541057-5-2", "paragraph_question": "question: \uc5f0\ud76c\uc804\ubb38\ud559\uad50\uac00 \uacbd\uae30\ub3c4 \uace0\uc591\uad70 \uc5f0\ud76c\uba74\uc5d0\uc11c \ub9e4\uc785\ud55c \ud1a0\uc9c0\uc758 \ud3c9\uc218\ub294?, context: \uc2e0\ucd0c\ucea0\ud37c\uc2a4(\u65b0\u6751-)\ub294 \uc11c\uc6b8\ud2b9\ubcc4\uc2dc \uc11c\ub300\ubb38\uad6c \uc5f0\uc138\ub85c 50 (\uc2e0\ucd0c\ub3d9)\uc5d0 \uc704\uce58\ud55c \uc5f0\uc138\ub300\ud559\uad50\uc758 \uc81c1\ucea0\ud37c\uc2a4\uc774\ub2e4. \uc5b8\ub354\uc6b0\ub4dc\uc758 \ud070\ud615\uc778 \uc874 \ud1a0\ub9c8\uc2a4 \uc5b8\ub354\uc6b0\ub4dc(John Tomas Underwood)\ub294 10\ub9cc \ub2ec\ub7ec\ub97c, \ub85c\uc2a4\uc564\uc824\ub808\uc2a4\uc758 \ucc30\uc2a4 \uc2a4\ud300\uc2a8(Charles S. M. Stimson)\uc740 \ub300\ud559 \uc124\ub9bd\uc6a9 \uc790\uae08\uc73c\ub85c 2\ub9cc 5\ucc9c \ub2ec\ub7ec\ub97c \uc5f0\ud76c\uc804\ubb38\ud559\uad50\uc5d0 \uae30\ubd80\ud558\uc600\ub2e4. \uc5b8\ub354\uc6b0\ub4dc\ub294 \ubbf8\uad6d\uc5d0 \ub3cc\uc544\uac00\uc11c \uad50\uc0ac \uac74\ub9bd\uc6a9 \uae30\ubd80\uae08 \ubaa8\uae08\uc744 \ud588\ub294\ub370 \uc2a4\ud300\uc2a8\uc5d0\uac8c\uc11c \uc704 \uae30\ubd80\uae08\uc744 \uc5bb\uc740 \ud6c4 \uc0ac\ub9dd\ud558\uc600\uace0, \ud6c4\uc784 \uad50\uc7a5\uc778 \uc5d0\ube44\uc2a8\uc774 \uae30\ubd80\uae08\uc73c\ub85c \uad50\uc0ac\ub97c \uac74\ub9bd\ud558\uc600\ub2e4. \uc5f0\ud76c\uc804\ubb38\ud559\uad50\ub294 \ub2f9\uc2dc \uacbd\uae30\ub3c4 \uace0\uc591\uad70 \uc5f0\ud76c\uba74 \uc2e0\ucd0c\ub9ac\uc758 \uc232\uc774 \uc6b8\ucc3d\ud55c \ud1a0\uc9c0 190,320\ud3c9\uc744 \ub9e4\uc785\ud558\uc600\ub2e4. \uc774\uacf3\uc740 \uc870\uc120 \uc815\uc885 \ub54c \uc138\uc6cc\uc9c4 \uc5f0\ud76c\uad81(\u5ef6\u79a7\u5bae) \ud130\uc640 \uc601\uc870(\u82f1\u7956)\uc758 \ud6c4\uad81 \uc601\ube48 \uc774\uc528(\u6620\u5b2a\u674e\u6c0f)\uc758 \uc218\uacbd\uc6d0(\u7dac\u6176\u5712)\uc774 \uc790\ub9ac \uc7a1\uace0 \uc788\uc5c8\uace0, \ud559\uad50\uba85 \u2018\uc5f0\ud76c\u2019\ub294 \uc5ec\uae30\uc11c \uc5f0\uc720\ud558\uc600\ub2e4. \ucea0\ud37c\uc2a4 \ub9c8\uc2a4\ud130\ud50c\ub79c\uacfc \uad50\uc0ac\ub3d9 \uc124\uacc4\ub294 \uc608\uc77c \ub300\ud559\uad50 \uac74\ucd95\uacfc\ub97c \uc878\uc5c5\ud55c \ubbf8\uad6d\uc778 \uac74\ucd95\uac00 \ud5e8\ub9ac \uba38\ud53c(Henry Killian Murphy)\uc5d0 \uc758\ud558\uc5ec \uc791\uc131\ub418\uc5c8\ub2e4. \ud604\uc7ac\uc758 \ubc31\uc591\ub85c \ub4f1 \uc2e0\ucd0c\ucea0\ud37c\uc2a4\uc758 \uace8\uaca9\uc740 \ucd08\uae30 \ub9c8\uc2a4\ud130\ud50c\ub79c\uc5d0 \uc758\ud574 \uc870\uc131\ub418\uc5c8\ub2e4. \ucd5c\ucd08\uc758 \uac74\ubb3c\uc740 \uce58\uc6d0\uad00(\u81f4\u9060\u9928)\uc774\uba70, \ubaa9\uc870 2\uce35\uc73c\ub85c 1918\ub144\uc5d0 \uc900\uacf5\ub418\uc5b4 1950\ub144\uae4c\uc9c0 \uc788\uc5c8\ub2e4. \uce58\uc6d0\uad00 \uc774\uc678\uc5d0\ub3c4 \uc2a4\ud300\uc2a8\uad00, \uc5b8\ub354\uc6b0\ub4dc\uad00, \uc544\ud39c\uc824\ub7ec\uad00, \ud55c\uacbd\uad00 \ub4f1\uc774 \uc900\uacf5\ub418\uc5c8\uc73c\uba70, \uae30\uc219\uc0ac\uc778 \ud540\uc2a8\uad00\uacfc \ub178\ucc9c\uadf9\uc7a5, \ub3cc\uce35\uacc4 \ub4f1\uc774 \ub9cc\ub4e4\uc5b4\uc84c\ub2e4."} {"question": "\uc774\uc2ac\ub78c\uad50\ub294 \uae30\ub3c5\uad50\uc640 \uac19\uc740 \ubb34\uc5c7\uc774 \uc874\uc7ac\ud558\uc9c0 \uc54a\uc2b5\ub2c8\uae4c?", "paragraph": "\uc774\uc2ac\ub78c\uad50\uc640 \uae30\ub3c5\uad50\ub294 \ub3d9\uc77c\ud55c \uc808\ub300\uc790\ub97c \uc22d\ubc30\ud558\uba70, \ucc9c\uad6d\uacfc \uc9c0\uc625\uc73c\ub85c \ub098\ud0c0\ub098\ub294 \uc0ac\ud6c4\uc138\uacc4\ub97c \ubbff\ub294\ub2e4\ub294 \uc810, \uadf8\ub9ac\uc2a4\ub3c4\uc758 \uc5c5\uc801\uacfc \uc131\uacbd\uc744 \uc874\uc911\ud55c\ub2e4\ub294 \uc810 \ub4f1 \ub9ce\uc740 \uacf5\ud1b5\uc810\uc744 \uc9c0\ub2c8\uace0 \uc788\ub2e4. \uadf8\ub7ec\ub098 \uc774\uc2ac\ub78c\uad50\uc758 \uacbd\uc6b0 \uae30\ub3c5\uad50\uc640 \uac19\uc740 \uc6d0\uc8c4\uc758\uc2dd\uc774 \uc874\uc7ac\ud558\uc9c0 \uc54a\ub294\ub2e4. \uc778\uac04\uc740 \uc4f0\uc5ec\uc9c0\uc9c0 \uc54a\uc740 \ucc45\uacfc \uac19\uc544\uc11c, \uc0ac\ud68c \ud658\uacbd\uacfc \uad50\uc721\uc5d0 \uc758\ud574 \uc545\ud55c \uae38\ub85c \ube60\uc838\ub4e4 \ubfd0 \uadf8 \uc790\uccb4\ub85c\ub294 \uc120\ud558\uc9c0\ub3c4, \uc545\ud558\uc9c0\ub3c4 \uc54a\uc740 \uc874\uc7ac\ub85c \ubcf8\ub2e4. \ub610\ud55c \uae30\ub3c5\uad50\uac00 \uc8fc\uc7a5\ud558\ub294'\ub300\uc18d'\uc758 \uac1c\ub150\ub3c4 \ubd80\uc815\ud558\ub294\ub370 \uac1c\uc778\uc774 \uc800\uc9c0\ub978 \uc8c4\ub294 \uc790\uae30 \uc2a4\uc2a4\ub85c\uac00 \uc2e0\uc5d0\uac8c \ud68c\uac1c\ud568\uc73c\ub85c\uc368\ub9cc \uc6a9\uc11c\ubc1b\uc744 \uc218 \uc788\ub2e4\uace0 \uc0dd\uac01\ud55c\ub2e4. \uac19\uc740 \ub9e5\ub77d\uc5d0\uc11c, \uc544\ub2f4\uacfc \uc774\ube0c\ub294 \uc21c\uac04\uc801\uc778 \uc720\ud639\uc5d0 \ub118\uc5b4\uac00 \ud0c0\ub77d\ud588\uc9c0\ub9cc \uacb0\uad6d \uc6a9\uc11c\ub97c \uad6c\ud558\uace0 \uad6c\uc6d0\ubc1b\uc558\ub2e4\uace0 \uc774\uc57c\uae30\ud55c\ub2e4. \uc989, \uce74\uc778\uacfc \uc544\ubca8\uc758 \uc8c4, \uadf8\ub9ac\uace0 \uadf8 \ud6c4\uc190\uc73c\ub85c \ub04a\uc784\uc5c6\uc774 \uc774\uc5b4\uc9c0\ub294 \uc778\uac04\uc758 \uc8c4\ub294 \uc870\uc0c1\uc73c\ub85c\ubd80\ud130 \ubb3c\ub824\ubc1b\uc740 \uac83\uc774 \uc544\ub2cc \uadf8\ub4e4\uc758 \uc798\ubabb\uc774\ub77c\ub294 \uac83\uc774\ub2e4. \uc774\uc2ac\ub78c\uc740 \uae30\ub3c5\uad50\uc640 \ub9c8\ucc2c\uac00\uc9c0\ub85c \uadf8\ub9ac\uc2a4\ub3c4\ub97c \uc704\ub300\ud55c \uc608\uc5b8\uc790\ub85c \uc874\uc911\ud55c\ub2e4. \uadf8\ub7ec\ub098 \uadf8\ub294 \uc5b4\ub514\uae4c\uc9c0\ub098 \uc2e0\uc774 \uc790\uc2e0\uacfc \uc778\uac04\uc758 \uc911\uc7ac\uc790\ub85c\uc11c \uc120\ud0dd\ud55c \uc5ec\ub7ec \uc608\uc5b8\uc790 \uc911 \ud558\ub098\uc77c\ubfd0, \uacb0\uc815\uc801\uc778 \uc608\uc5b8\uc790\ub294 \ubb34\ud568\ub9c8\ub4dc \ud55c \uc0ac\ub78c\uc774\ub77c\uace0 \ubcf8\ub2e4. \uc131\uacbd \uc5ed\uc2dc \uc2e0\uc758 \ub9d0\uc500\uc774 \uc77c\ubd80 \ub4e4\uc5b4\uc788\ub2e4\ub294 \uac83\uc5d0\ub294 \ub3d9\uc758\ud558\ub098 \uadf8\uac83\uc740 \uc138\uc6d4\uc744 \uac70\uce58\uba74\uc11c \uc5ec\ub7ec \uc131\uc9c1\uc790\ub4e4\uc5d0 \uc758\ud574 \uc65c\uace1\ub418\uace0 \ubcc0\ud615\ub41c \uce21\uba74\uc774 \ub9ce\ub2e4\uace0 \ubcf8\ub2e4. \uacb0\uad6d \uc774\uc2ac\ub78c\uad50\ub3c4\ub4e4\uc774 \uac00\uce58\ud310\ub2e8\uc758 \uae30\uc900\uc73c\ub85c \uc5ec\uae30\ub294 \uac83\uc740 '\uafb8\ub780'\uc774\uba70, \uafb8\ub780\uc740 \uc55e\uc73c\ub85c\ub3c4 \ub367\ubd99\uc5ec\uc9c0\uc9c0 \uc54a\uc744 \uc644\uc804\ud55c \ud615\ud0dc\uc758 \uc131\uc11c\uc774\uae30 \ub54c\ubb38\uc5d0 \uc808\ub300\uc801\uc774\ub2e4.", "answer": "\uc6d0\uc8c4\uc758\uc2dd", "sentence": "\uadf8\ub7ec\ub098 \uc774\uc2ac\ub78c\uad50\uc758 \uacbd\uc6b0 \uae30\ub3c5\uad50\uc640 \uac19\uc740 \uc6d0\uc8c4\uc758\uc2dd \uc774 \uc874\uc7ac\ud558\uc9c0 \uc54a\ub294\ub2e4.", "paragraph_sentence": "\uc774\uc2ac\ub78c\uad50\uc640 \uae30\ub3c5\uad50\ub294 \ub3d9\uc77c\ud55c \uc808\ub300\uc790\ub97c \uc22d\ubc30\ud558\uba70, \ucc9c\uad6d\uacfc \uc9c0\uc625\uc73c\ub85c \ub098\ud0c0\ub098\ub294 \uc0ac\ud6c4\uc138\uacc4\ub97c \ubbff\ub294\ub2e4\ub294 \uc810, \uadf8\ub9ac\uc2a4\ub3c4\uc758 \uc5c5\uc801\uacfc \uc131\uacbd\uc744 \uc874\uc911\ud55c\ub2e4\ub294 \uc810 \ub4f1 \ub9ce\uc740 \uacf5\ud1b5\uc810\uc744 \uc9c0\ub2c8\uace0 \uc788\ub2e4. \uadf8\ub7ec\ub098 \uc774\uc2ac\ub78c\uad50\uc758 \uacbd\uc6b0 \uae30\ub3c5\uad50\uc640 \uac19\uc740 \uc6d0\uc8c4\uc758\uc2dd \uc774 \uc874\uc7ac\ud558\uc9c0 \uc54a\ub294\ub2e4. \uc778\uac04\uc740 \uc4f0\uc5ec\uc9c0\uc9c0 \uc54a\uc740 \ucc45\uacfc \uac19\uc544\uc11c, \uc0ac\ud68c \ud658\uacbd\uacfc \uad50\uc721\uc5d0 \uc758\ud574 \uc545\ud55c \uae38\ub85c \ube60\uc838\ub4e4 \ubfd0 \uadf8 \uc790\uccb4\ub85c\ub294 \uc120\ud558\uc9c0\ub3c4, \uc545\ud558\uc9c0\ub3c4 \uc54a\uc740 \uc874\uc7ac\ub85c \ubcf8\ub2e4. \ub610\ud55c \uae30\ub3c5\uad50\uac00 \uc8fc\uc7a5\ud558\ub294'\ub300\uc18d'\uc758 \uac1c\ub150\ub3c4 \ubd80\uc815\ud558\ub294\ub370 \uac1c\uc778\uc774 \uc800\uc9c0\ub978 \uc8c4\ub294 \uc790\uae30 \uc2a4\uc2a4\ub85c\uac00 \uc2e0\uc5d0\uac8c \ud68c\uac1c\ud568\uc73c\ub85c\uc368\ub9cc \uc6a9\uc11c\ubc1b\uc744 \uc218 \uc788\ub2e4\uace0 \uc0dd\uac01\ud55c\ub2e4. \uac19\uc740 \ub9e5\ub77d\uc5d0\uc11c, \uc544\ub2f4\uacfc \uc774\ube0c\ub294 \uc21c\uac04\uc801\uc778 \uc720\ud639\uc5d0 \ub118\uc5b4\uac00 \ud0c0\ub77d\ud588\uc9c0\ub9cc \uacb0\uad6d \uc6a9\uc11c\ub97c \uad6c\ud558\uace0 \uad6c\uc6d0\ubc1b\uc558\ub2e4\uace0 \uc774\uc57c\uae30\ud55c\ub2e4. \uc989, \uce74\uc778\uacfc \uc544\ubca8\uc758 \uc8c4, \uadf8\ub9ac\uace0 \uadf8 \ud6c4\uc190\uc73c\ub85c \ub04a\uc784\uc5c6\uc774 \uc774\uc5b4\uc9c0\ub294 \uc778\uac04\uc758 \uc8c4\ub294 \uc870\uc0c1\uc73c\ub85c\ubd80\ud130 \ubb3c\ub824\ubc1b\uc740 \uac83\uc774 \uc544\ub2cc \uadf8\ub4e4\uc758 \uc798\ubabb\uc774\ub77c\ub294 \uac83\uc774\ub2e4. \uc774\uc2ac\ub78c\uc740 \uae30\ub3c5\uad50\uc640 \ub9c8\ucc2c\uac00\uc9c0\ub85c \uadf8\ub9ac\uc2a4\ub3c4\ub97c \uc704\ub300\ud55c \uc608\uc5b8\uc790\ub85c \uc874\uc911\ud55c\ub2e4. \uadf8\ub7ec\ub098 \uadf8\ub294 \uc5b4\ub514\uae4c\uc9c0\ub098 \uc2e0\uc774 \uc790\uc2e0\uacfc \uc778\uac04\uc758 \uc911\uc7ac\uc790\ub85c\uc11c \uc120\ud0dd\ud55c \uc5ec\ub7ec \uc608\uc5b8\uc790 \uc911 \ud558\ub098\uc77c\ubfd0, \uacb0\uc815\uc801\uc778 \uc608\uc5b8\uc790\ub294 \ubb34\ud568\ub9c8\ub4dc \ud55c \uc0ac\ub78c\uc774\ub77c\uace0 \ubcf8\ub2e4. \uc131\uacbd \uc5ed\uc2dc \uc2e0\uc758 \ub9d0\uc500\uc774 \uc77c\ubd80 \ub4e4\uc5b4\uc788\ub2e4\ub294 \uac83\uc5d0\ub294 \ub3d9\uc758\ud558\ub098 \uadf8\uac83\uc740 \uc138\uc6d4\uc744 \uac70\uce58\uba74\uc11c \uc5ec\ub7ec \uc131\uc9c1\uc790\ub4e4\uc5d0 \uc758\ud574 \uc65c\uace1\ub418\uace0 \ubcc0\ud615\ub41c \uce21\uba74\uc774 \ub9ce\ub2e4\uace0 \ubcf8\ub2e4. \uacb0\uad6d \uc774\uc2ac\ub78c\uad50\ub3c4\ub4e4\uc774 \uac00\uce58\ud310\ub2e8\uc758 \uae30\uc900\uc73c\ub85c \uc5ec\uae30\ub294 \uac83\uc740 '\uafb8\ub780'\uc774\uba70, \uafb8\ub780\uc740 \uc55e\uc73c\ub85c\ub3c4 \ub367\ubd99\uc5ec\uc9c0\uc9c0 \uc54a\uc744 \uc644\uc804\ud55c \ud615\ud0dc\uc758 \uc131\uc11c\uc774\uae30 \ub54c\ubb38\uc5d0 \uc808\ub300\uc801\uc774\ub2e4.", "paragraph_answer": "\uc774\uc2ac\ub78c\uad50\uc640 \uae30\ub3c5\uad50\ub294 \ub3d9\uc77c\ud55c \uc808\ub300\uc790\ub97c \uc22d\ubc30\ud558\uba70, \ucc9c\uad6d\uacfc \uc9c0\uc625\uc73c\ub85c \ub098\ud0c0\ub098\ub294 \uc0ac\ud6c4\uc138\uacc4\ub97c \ubbff\ub294\ub2e4\ub294 \uc810, \uadf8\ub9ac\uc2a4\ub3c4\uc758 \uc5c5\uc801\uacfc \uc131\uacbd\uc744 \uc874\uc911\ud55c\ub2e4\ub294 \uc810 \ub4f1 \ub9ce\uc740 \uacf5\ud1b5\uc810\uc744 \uc9c0\ub2c8\uace0 \uc788\ub2e4. \uadf8\ub7ec\ub098 \uc774\uc2ac\ub78c\uad50\uc758 \uacbd\uc6b0 \uae30\ub3c5\uad50\uc640 \uac19\uc740 \uc6d0\uc8c4\uc758\uc2dd \uc774 \uc874\uc7ac\ud558\uc9c0 \uc54a\ub294\ub2e4. \uc778\uac04\uc740 \uc4f0\uc5ec\uc9c0\uc9c0 \uc54a\uc740 \ucc45\uacfc \uac19\uc544\uc11c, \uc0ac\ud68c \ud658\uacbd\uacfc \uad50\uc721\uc5d0 \uc758\ud574 \uc545\ud55c \uae38\ub85c \ube60\uc838\ub4e4 \ubfd0 \uadf8 \uc790\uccb4\ub85c\ub294 \uc120\ud558\uc9c0\ub3c4, \uc545\ud558\uc9c0\ub3c4 \uc54a\uc740 \uc874\uc7ac\ub85c \ubcf8\ub2e4. \ub610\ud55c \uae30\ub3c5\uad50\uac00 \uc8fc\uc7a5\ud558\ub294'\ub300\uc18d'\uc758 \uac1c\ub150\ub3c4 \ubd80\uc815\ud558\ub294\ub370 \uac1c\uc778\uc774 \uc800\uc9c0\ub978 \uc8c4\ub294 \uc790\uae30 \uc2a4\uc2a4\ub85c\uac00 \uc2e0\uc5d0\uac8c \ud68c\uac1c\ud568\uc73c\ub85c\uc368\ub9cc \uc6a9\uc11c\ubc1b\uc744 \uc218 \uc788\ub2e4\uace0 \uc0dd\uac01\ud55c\ub2e4. \uac19\uc740 \ub9e5\ub77d\uc5d0\uc11c, \uc544\ub2f4\uacfc \uc774\ube0c\ub294 \uc21c\uac04\uc801\uc778 \uc720\ud639\uc5d0 \ub118\uc5b4\uac00 \ud0c0\ub77d\ud588\uc9c0\ub9cc \uacb0\uad6d \uc6a9\uc11c\ub97c \uad6c\ud558\uace0 \uad6c\uc6d0\ubc1b\uc558\ub2e4\uace0 \uc774\uc57c\uae30\ud55c\ub2e4. \uc989, \uce74\uc778\uacfc \uc544\ubca8\uc758 \uc8c4, \uadf8\ub9ac\uace0 \uadf8 \ud6c4\uc190\uc73c\ub85c \ub04a\uc784\uc5c6\uc774 \uc774\uc5b4\uc9c0\ub294 \uc778\uac04\uc758 \uc8c4\ub294 \uc870\uc0c1\uc73c\ub85c\ubd80\ud130 \ubb3c\ub824\ubc1b\uc740 \uac83\uc774 \uc544\ub2cc \uadf8\ub4e4\uc758 \uc798\ubabb\uc774\ub77c\ub294 \uac83\uc774\ub2e4. \uc774\uc2ac\ub78c\uc740 \uae30\ub3c5\uad50\uc640 \ub9c8\ucc2c\uac00\uc9c0\ub85c \uadf8\ub9ac\uc2a4\ub3c4\ub97c \uc704\ub300\ud55c \uc608\uc5b8\uc790\ub85c \uc874\uc911\ud55c\ub2e4. \uadf8\ub7ec\ub098 \uadf8\ub294 \uc5b4\ub514\uae4c\uc9c0\ub098 \uc2e0\uc774 \uc790\uc2e0\uacfc \uc778\uac04\uc758 \uc911\uc7ac\uc790\ub85c\uc11c \uc120\ud0dd\ud55c \uc5ec\ub7ec \uc608\uc5b8\uc790 \uc911 \ud558\ub098\uc77c\ubfd0, \uacb0\uc815\uc801\uc778 \uc608\uc5b8\uc790\ub294 \ubb34\ud568\ub9c8\ub4dc \ud55c \uc0ac\ub78c\uc774\ub77c\uace0 \ubcf8\ub2e4. \uc131\uacbd \uc5ed\uc2dc \uc2e0\uc758 \ub9d0\uc500\uc774 \uc77c\ubd80 \ub4e4\uc5b4\uc788\ub2e4\ub294 \uac83\uc5d0\ub294 \ub3d9\uc758\ud558\ub098 \uadf8\uac83\uc740 \uc138\uc6d4\uc744 \uac70\uce58\uba74\uc11c \uc5ec\ub7ec \uc131\uc9c1\uc790\ub4e4\uc5d0 \uc758\ud574 \uc65c\uace1\ub418\uace0 \ubcc0\ud615\ub41c \uce21\uba74\uc774 \ub9ce\ub2e4\uace0 \ubcf8\ub2e4. \uacb0\uad6d \uc774\uc2ac\ub78c\uad50\ub3c4\ub4e4\uc774 \uac00\uce58\ud310\ub2e8\uc758 \uae30\uc900\uc73c\ub85c \uc5ec\uae30\ub294 \uac83\uc740 '\uafb8\ub780'\uc774\uba70, \uafb8\ub780\uc740 \uc55e\uc73c\ub85c\ub3c4 \ub367\ubd99\uc5ec\uc9c0\uc9c0 \uc54a\uc744 \uc644\uc804\ud55c \ud615\ud0dc\uc758 \uc131\uc11c\uc774\uae30 \ub54c\ubb38\uc5d0 \uc808\ub300\uc801\uc774\ub2e4.", "sentence_answer": "\uadf8\ub7ec\ub098 \uc774\uc2ac\ub78c\uad50\uc758 \uacbd\uc6b0 \uae30\ub3c5\uad50\uc640 \uac19\uc740 \uc6d0\uc8c4\uc758\uc2dd \uc774 \uc874\uc7ac\ud558\uc9c0 \uc54a\ub294\ub2e4.", "paragraph_id": "6545747-2-0", "paragraph_question": "question: \uc774\uc2ac\ub78c\uad50\ub294 \uae30\ub3c5\uad50\uc640 \uac19\uc740 \ubb34\uc5c7\uc774 \uc874\uc7ac\ud558\uc9c0 \uc54a\uc2b5\ub2c8\uae4c?, context: \uc774\uc2ac\ub78c\uad50\uc640 \uae30\ub3c5\uad50\ub294 \ub3d9\uc77c\ud55c \uc808\ub300\uc790\ub97c \uc22d\ubc30\ud558\uba70, \ucc9c\uad6d\uacfc \uc9c0\uc625\uc73c\ub85c \ub098\ud0c0\ub098\ub294 \uc0ac\ud6c4\uc138\uacc4\ub97c \ubbff\ub294\ub2e4\ub294 \uc810, \uadf8\ub9ac\uc2a4\ub3c4\uc758 \uc5c5\uc801\uacfc \uc131\uacbd\uc744 \uc874\uc911\ud55c\ub2e4\ub294 \uc810 \ub4f1 \ub9ce\uc740 \uacf5\ud1b5\uc810\uc744 \uc9c0\ub2c8\uace0 \uc788\ub2e4. \uadf8\ub7ec\ub098 \uc774\uc2ac\ub78c\uad50\uc758 \uacbd\uc6b0 \uae30\ub3c5\uad50\uc640 \uac19\uc740 \uc6d0\uc8c4\uc758\uc2dd\uc774 \uc874\uc7ac\ud558\uc9c0 \uc54a\ub294\ub2e4. \uc778\uac04\uc740 \uc4f0\uc5ec\uc9c0\uc9c0 \uc54a\uc740 \ucc45\uacfc \uac19\uc544\uc11c, \uc0ac\ud68c \ud658\uacbd\uacfc \uad50\uc721\uc5d0 \uc758\ud574 \uc545\ud55c \uae38\ub85c \ube60\uc838\ub4e4 \ubfd0 \uadf8 \uc790\uccb4\ub85c\ub294 \uc120\ud558\uc9c0\ub3c4, \uc545\ud558\uc9c0\ub3c4 \uc54a\uc740 \uc874\uc7ac\ub85c \ubcf8\ub2e4. \ub610\ud55c \uae30\ub3c5\uad50\uac00 \uc8fc\uc7a5\ud558\ub294'\ub300\uc18d'\uc758 \uac1c\ub150\ub3c4 \ubd80\uc815\ud558\ub294\ub370 \uac1c\uc778\uc774 \uc800\uc9c0\ub978 \uc8c4\ub294 \uc790\uae30 \uc2a4\uc2a4\ub85c\uac00 \uc2e0\uc5d0\uac8c \ud68c\uac1c\ud568\uc73c\ub85c\uc368\ub9cc \uc6a9\uc11c\ubc1b\uc744 \uc218 \uc788\ub2e4\uace0 \uc0dd\uac01\ud55c\ub2e4. \uac19\uc740 \ub9e5\ub77d\uc5d0\uc11c, \uc544\ub2f4\uacfc \uc774\ube0c\ub294 \uc21c\uac04\uc801\uc778 \uc720\ud639\uc5d0 \ub118\uc5b4\uac00 \ud0c0\ub77d\ud588\uc9c0\ub9cc \uacb0\uad6d \uc6a9\uc11c\ub97c \uad6c\ud558\uace0 \uad6c\uc6d0\ubc1b\uc558\ub2e4\uace0 \uc774\uc57c\uae30\ud55c\ub2e4. \uc989, \uce74\uc778\uacfc \uc544\ubca8\uc758 \uc8c4, \uadf8\ub9ac\uace0 \uadf8 \ud6c4\uc190\uc73c\ub85c \ub04a\uc784\uc5c6\uc774 \uc774\uc5b4\uc9c0\ub294 \uc778\uac04\uc758 \uc8c4\ub294 \uc870\uc0c1\uc73c\ub85c\ubd80\ud130 \ubb3c\ub824\ubc1b\uc740 \uac83\uc774 \uc544\ub2cc \uadf8\ub4e4\uc758 \uc798\ubabb\uc774\ub77c\ub294 \uac83\uc774\ub2e4. \uc774\uc2ac\ub78c\uc740 \uae30\ub3c5\uad50\uc640 \ub9c8\ucc2c\uac00\uc9c0\ub85c \uadf8\ub9ac\uc2a4\ub3c4\ub97c \uc704\ub300\ud55c \uc608\uc5b8\uc790\ub85c \uc874\uc911\ud55c\ub2e4. \uadf8\ub7ec\ub098 \uadf8\ub294 \uc5b4\ub514\uae4c\uc9c0\ub098 \uc2e0\uc774 \uc790\uc2e0\uacfc \uc778\uac04\uc758 \uc911\uc7ac\uc790\ub85c\uc11c \uc120\ud0dd\ud55c \uc5ec\ub7ec \uc608\uc5b8\uc790 \uc911 \ud558\ub098\uc77c\ubfd0, \uacb0\uc815\uc801\uc778 \uc608\uc5b8\uc790\ub294 \ubb34\ud568\ub9c8\ub4dc \ud55c \uc0ac\ub78c\uc774\ub77c\uace0 \ubcf8\ub2e4. \uc131\uacbd \uc5ed\uc2dc \uc2e0\uc758 \ub9d0\uc500\uc774 \uc77c\ubd80 \ub4e4\uc5b4\uc788\ub2e4\ub294 \uac83\uc5d0\ub294 \ub3d9\uc758\ud558\ub098 \uadf8\uac83\uc740 \uc138\uc6d4\uc744 \uac70\uce58\uba74\uc11c \uc5ec\ub7ec \uc131\uc9c1\uc790\ub4e4\uc5d0 \uc758\ud574 \uc65c\uace1\ub418\uace0 \ubcc0\ud615\ub41c \uce21\uba74\uc774 \ub9ce\ub2e4\uace0 \ubcf8\ub2e4. \uacb0\uad6d \uc774\uc2ac\ub78c\uad50\ub3c4\ub4e4\uc774 \uac00\uce58\ud310\ub2e8\uc758 \uae30\uc900\uc73c\ub85c \uc5ec\uae30\ub294 \uac83\uc740 '\uafb8\ub780'\uc774\uba70, \uafb8\ub780\uc740 \uc55e\uc73c\ub85c\ub3c4 \ub367\ubd99\uc5ec\uc9c0\uc9c0 \uc54a\uc744 \uc644\uc804\ud55c \ud615\ud0dc\uc758 \uc131\uc11c\uc774\uae30 \ub54c\ubb38\uc5d0 \uc808\ub300\uc801\uc774\ub2e4."} {"question": "\uce7c\ub9ac\uc2a4\ud1a0\uc758 \ud45c\uba74 \uc911 \uc624\ub798\ub41c \ucda9\ub3cc\uad6c\ub4e4\uc774 \uc9c0\uc6cc\uc9c4 \ud754\uc801\uc774 \uc788\ub294 \ubc1d\uc740 \ud3c9\uc6d0\uc740?", "paragraph": "\uce7c\ub9ac\uc2a4\ud1a0\uc758 \ud45c\uba74\uc740 \ucda9\ub3cc\uad6c\uac00 \ub9ce\uc740 \ud3c9\uc6d0, \ubc1d\uc740 \ud3c9\uc6d0, \ubc1d\uace0 \ubd80\ub4dc\ub7ec\uc6b4 \ud3c9\uc6d0, \ud2b9\uc815\ud55c \ucda9\ub3cc\uad6c\ub4e4\uacfc \uad00\ub828\ub41c \ub2e4\uc591\ud55c \uad6c\uc131 \ub2e8\uc704\ub4e4\ub85c \ub098\ub20c \uc218 \uc788\ub2e4. \ucda9\ub3cc\uad6c\uac00 \ub9ce\uc740 \ud3c9\uc6d0\ub4e4\uc740 \ud45c\uba74\uc758 \ub300\ubd80\ubd84\uc744 \uad6c\uc131\ud558\uace0, \uc5bc\uc74c\uacfc \uc554\uc11d\uc774 \uc11e\uc5ec \ub9cc\ub4e4\uc5b4\uc9c4 \uc624\ub798 \ub41c \uc554\uad8c\uc5d0 \ud574\ub2f9\ud55c\ub2e4. \ubc1d\uc740 \ud3c9\uc6d0\ub4e4\uc740 \uc544\uc2a4\uac00\ub974\ub4dc\ub098 \ub85c\ud508\uacfc \uac19\uc740 \ubc1d\uc740 \ucda9\ub3cc\uad6c\ub4e4\uacfc \ud3a0\ub984\uc2dc\uc2a4\ud2b8\ub77c\uace0 \ubd88\ub9ac\ub294 \uc624\ub798\ub41c \ucda9\ub3cc\uad6c\ub4e4\uc774 \uc9c0\uc6cc\uc9c4 \ud754\uc801\ub4e4, \ub2e4\ud658 \ucda9\ub3cc\uad6c\uc758 \uc911\uc2ec\ubd80, \ucda9\ub3cc\uad6c \ud3c9\uc6d0\uc5d0\uc11c \uc678\ub5a8\uc5b4\uc9c4 \ubd80\ubd84\ub4e4\uc744 \ud3ec\ud568\ud558\ub294 \ud3c9\uc6d0\uc774\ub2e4. \uc774\ub7ec\ud55c \ubc1d\uc740 \ud3c9\uc6d0\ub4e4\uc740 \uc6b4\uc11d \ucda9\ub3cc\ub85c \ubc1c\uc0dd\ud55c \ud1f4\uc801\ubb3c\uc774\ub77c\uace0 \uc5ec\uaca8\uc9c4\ub2e4. \ubc1d\uace0 \ubd80\ub4dc\ub7ec\uc6b4 \ud3c9\uc6d0\uc740 \uce7c\ub9ac\uc2a4\ud1a0 \ud45c\uba74\uc758 \uc77c\ubd80\ubd84\ub9cc\uc744 \uad6c\uc131\ud558\uace0 \uc788\uace0, \ubc1c\ud560\ub77c\ub098 \uc544\uc2a4\uac00\ub974\ub4dc \ucda9\ub3cc\uad6c \ud3c9\uc6d0\uc758 \uace0\ub9bd\ub41c \uc9c0\uc5ed, \uc989 \uc0b0\ub9c8\ub8e8\ub098 \uc8fc\uc0c1\ud574\ubd84\uc5d0\uc11c \ubc1c\uacac\ub41c\ub2e4. \uc774\ub4e4\uc740 \ub0b4\uc778\uc131 \ud65c\ub3d9\uc73c\ub85c \uc5f0\uacb0\ub418\uc5b4 \uc788\uc73c\ub9ac\ub77c\uace0 \uc0dd\uac01\ub418\uc5c8\uc5c8\uc9c0\ub9cc, \uac08\ub9b4\ub808\uc624 \ud0d0\uc0ac\uc120\uc758 \uace0\ud574\uc0c1\ub3c4 \uc0ac\uc9c4\uc5d0\uc11c\ub294 \ubc1d\uace0 \ubd80\ub4dc\ub7ec\uc6b4 \ud3c9\uc6d0\ub4e4\uc774 \ub9e4\uc6b0 \uade0\uc5f4\uc774 \ub9ce\uace0 \uc6b0\ub458\ud22c\ub458\ud55c \uc9c0\ud615\uacfc \ubc00\uc811\ud55c \uc5f0\uad00\uc774 \uc788\uace0, \ud45c\uba74\uc774 \uc0c8\ub85c\uc774 \ub9cc\ub4e4\uc5b4\uc9c4 \ud754\uc801\uc740 \uc804\ud600 \uad00\ucc30\ub418\uc9c0 \uc54a\uc558\ub2e4. \ub610\ud55c \uac08\ub9b4\ub808\uc624\uc758 \uc0ac\uc9c4\ub4e4\uc740 \ub9c8\uce58 \uc8fc\ubcc0 \uc9c0\ud615\uc5d0 \ud3ec\uc704\ub41c \uac83\ucc98\ub7fc \ubcf4\uc774\ub294, \ucd1d \ub113\uc774 10,000 km \uc774\ud558\uc758 \uc791\uace0 \uc5b4\ub450\uc6b0\uba70 \ubd80\ub4dc\ub7ec\uc6b4 \ubd80\ubd84\ub4e4\uc744 \ubcf4\uc5ec\uc8fc\uc5c8\uc73c\uba70, \uc774\ub4e4\uc740 \uc5bc\uc74c\ud654\uc0b0\uc73c\ub85c \uc778\ud574 \uc0dd\uae34 \uac83\uc77c \uc218\ub3c4 \uc788\ub2e4. \ubc1d\uc740 \ud3c9\uc6d0\uacfc \ub2e4\uc591\ud55c \ubd80\ub4dc\ub7ec\uc6b4 \ud3c9\uc6d0\ub4e4\uc740 \uc8fc\ubcc0 \ubc30\uacbd\uc758 \ucda9\ub3cc\uad6c \ud3c9\uc6d0\ub4e4\ubcf4\ub2e4 \uc57d\uac04 \uc80a\uace0 \ucda9\ub3cc\uad6c\ub4e4\uc774 \uc801\ub2e4.", "answer": "\ud3a0\ub984\uc2dc\uc2a4\ud2b8", "sentence": "\ubc1d\uc740 \ud3c9\uc6d0\ub4e4\uc740 \uc544\uc2a4\uac00\ub974\ub4dc\ub098 \ub85c\ud508\uacfc \uac19\uc740 \ubc1d\uc740 \ucda9\ub3cc\uad6c\ub4e4\uacfc \ud3a0\ub984\uc2dc\uc2a4\ud2b8 \ub77c\uace0 \ubd88\ub9ac\ub294 \uc624\ub798\ub41c \ucda9\ub3cc\uad6c\ub4e4\uc774 \uc9c0\uc6cc\uc9c4 \ud754\uc801\ub4e4, \ub2e4\ud658 \ucda9\ub3cc\uad6c\uc758 \uc911\uc2ec\ubd80, \ucda9\ub3cc\uad6c \ud3c9\uc6d0\uc5d0\uc11c \uc678\ub5a8\uc5b4\uc9c4 \ubd80\ubd84\ub4e4\uc744 \ud3ec\ud568\ud558\ub294 \ud3c9\uc6d0\uc774\ub2e4.", "paragraph_sentence": "\uce7c\ub9ac\uc2a4\ud1a0\uc758 \ud45c\uba74\uc740 \ucda9\ub3cc\uad6c\uac00 \ub9ce\uc740 \ud3c9\uc6d0, \ubc1d\uc740 \ud3c9\uc6d0, \ubc1d\uace0 \ubd80\ub4dc\ub7ec\uc6b4 \ud3c9\uc6d0, \ud2b9\uc815\ud55c \ucda9\ub3cc\uad6c\ub4e4\uacfc \uad00\ub828\ub41c \ub2e4\uc591\ud55c \uad6c\uc131 \ub2e8\uc704\ub4e4\ub85c \ub098\ub20c \uc218 \uc788\ub2e4. \ucda9\ub3cc\uad6c\uac00 \ub9ce\uc740 \ud3c9\uc6d0\ub4e4\uc740 \ud45c\uba74\uc758 \ub300\ubd80\ubd84\uc744 \uad6c\uc131\ud558\uace0, \uc5bc\uc74c\uacfc \uc554\uc11d\uc774 \uc11e\uc5ec \ub9cc\ub4e4\uc5b4\uc9c4 \uc624\ub798 \ub41c \uc554\uad8c\uc5d0 \ud574\ub2f9\ud55c\ub2e4. \ubc1d\uc740 \ud3c9\uc6d0\ub4e4\uc740 \uc544\uc2a4\uac00\ub974\ub4dc\ub098 \ub85c\ud508\uacfc \uac19\uc740 \ubc1d\uc740 \ucda9\ub3cc\uad6c\ub4e4\uacfc \ud3a0\ub984\uc2dc\uc2a4\ud2b8 \ub77c\uace0 \ubd88\ub9ac\ub294 \uc624\ub798\ub41c \ucda9\ub3cc\uad6c\ub4e4\uc774 \uc9c0\uc6cc\uc9c4 \ud754\uc801\ub4e4, \ub2e4\ud658 \ucda9\ub3cc\uad6c\uc758 \uc911\uc2ec\ubd80, \ucda9\ub3cc\uad6c \ud3c9\uc6d0\uc5d0\uc11c \uc678\ub5a8\uc5b4\uc9c4 \ubd80\ubd84\ub4e4\uc744 \ud3ec\ud568\ud558\ub294 \ud3c9\uc6d0\uc774\ub2e4. \uc774\ub7ec\ud55c \ubc1d\uc740 \ud3c9\uc6d0\ub4e4\uc740 \uc6b4\uc11d \ucda9\ub3cc\ub85c \ubc1c\uc0dd\ud55c \ud1f4\uc801\ubb3c\uc774\ub77c\uace0 \uc5ec\uaca8\uc9c4\ub2e4. \ubc1d\uace0 \ubd80\ub4dc\ub7ec\uc6b4 \ud3c9\uc6d0\uc740 \uce7c\ub9ac\uc2a4\ud1a0 \ud45c\uba74\uc758 \uc77c\ubd80\ubd84\ub9cc\uc744 \uad6c\uc131\ud558\uace0 \uc788\uace0, \ubc1c\ud560\ub77c\ub098 \uc544\uc2a4\uac00\ub974\ub4dc \ucda9\ub3cc\uad6c \ud3c9\uc6d0\uc758 \uace0\ub9bd\ub41c \uc9c0\uc5ed, \uc989 \uc0b0\ub9c8\ub8e8\ub098 \uc8fc\uc0c1\ud574\ubd84\uc5d0\uc11c \ubc1c\uacac\ub41c\ub2e4. \uc774\ub4e4\uc740 \ub0b4\uc778\uc131 \ud65c\ub3d9\uc73c\ub85c \uc5f0\uacb0\ub418\uc5b4 \uc788\uc73c\ub9ac\ub77c\uace0 \uc0dd\uac01\ub418\uc5c8\uc5c8\uc9c0\ub9cc, \uac08\ub9b4\ub808\uc624 \ud0d0\uc0ac\uc120\uc758 \uace0\ud574\uc0c1\ub3c4 \uc0ac\uc9c4\uc5d0\uc11c\ub294 \ubc1d\uace0 \ubd80\ub4dc\ub7ec\uc6b4 \ud3c9\uc6d0\ub4e4\uc774 \ub9e4\uc6b0 \uade0\uc5f4\uc774 \ub9ce\uace0 \uc6b0\ub458\ud22c\ub458\ud55c \uc9c0\ud615\uacfc \ubc00\uc811\ud55c \uc5f0\uad00\uc774 \uc788\uace0, \ud45c\uba74\uc774 \uc0c8\ub85c\uc774 \ub9cc\ub4e4\uc5b4\uc9c4 \ud754\uc801\uc740 \uc804\ud600 \uad00\ucc30\ub418\uc9c0 \uc54a\uc558\ub2e4. \ub610\ud55c \uac08\ub9b4\ub808\uc624\uc758 \uc0ac\uc9c4\ub4e4\uc740 \ub9c8\uce58 \uc8fc\ubcc0 \uc9c0\ud615\uc5d0 \ud3ec\uc704\ub41c \uac83\ucc98\ub7fc \ubcf4\uc774\ub294, \ucd1d \ub113\uc774 10,000 km \uc774\ud558\uc758 \uc791\uace0 \uc5b4\ub450\uc6b0\uba70 \ubd80\ub4dc\ub7ec\uc6b4 \ubd80\ubd84\ub4e4\uc744 \ubcf4\uc5ec\uc8fc\uc5c8\uc73c\uba70, \uc774\ub4e4\uc740 \uc5bc\uc74c\ud654\uc0b0\uc73c\ub85c \uc778\ud574 \uc0dd\uae34 \uac83\uc77c \uc218\ub3c4 \uc788\ub2e4. \ubc1d\uc740 \ud3c9\uc6d0\uacfc \ub2e4\uc591\ud55c \ubd80\ub4dc\ub7ec\uc6b4 \ud3c9\uc6d0\ub4e4\uc740 \uc8fc\ubcc0 \ubc30\uacbd\uc758 \ucda9\ub3cc\uad6c \ud3c9\uc6d0\ub4e4\ubcf4\ub2e4 \uc57d\uac04 \uc80a\uace0 \ucda9\ub3cc\uad6c\ub4e4\uc774 \uc801\ub2e4.", "paragraph_answer": "\uce7c\ub9ac\uc2a4\ud1a0\uc758 \ud45c\uba74\uc740 \ucda9\ub3cc\uad6c\uac00 \ub9ce\uc740 \ud3c9\uc6d0, \ubc1d\uc740 \ud3c9\uc6d0, \ubc1d\uace0 \ubd80\ub4dc\ub7ec\uc6b4 \ud3c9\uc6d0, \ud2b9\uc815\ud55c \ucda9\ub3cc\uad6c\ub4e4\uacfc \uad00\ub828\ub41c \ub2e4\uc591\ud55c \uad6c\uc131 \ub2e8\uc704\ub4e4\ub85c \ub098\ub20c \uc218 \uc788\ub2e4. \ucda9\ub3cc\uad6c\uac00 \ub9ce\uc740 \ud3c9\uc6d0\ub4e4\uc740 \ud45c\uba74\uc758 \ub300\ubd80\ubd84\uc744 \uad6c\uc131\ud558\uace0, \uc5bc\uc74c\uacfc \uc554\uc11d\uc774 \uc11e\uc5ec \ub9cc\ub4e4\uc5b4\uc9c4 \uc624\ub798 \ub41c \uc554\uad8c\uc5d0 \ud574\ub2f9\ud55c\ub2e4. \ubc1d\uc740 \ud3c9\uc6d0\ub4e4\uc740 \uc544\uc2a4\uac00\ub974\ub4dc\ub098 \ub85c\ud508\uacfc \uac19\uc740 \ubc1d\uc740 \ucda9\ub3cc\uad6c\ub4e4\uacfc \ud3a0\ub984\uc2dc\uc2a4\ud2b8 \ub77c\uace0 \ubd88\ub9ac\ub294 \uc624\ub798\ub41c \ucda9\ub3cc\uad6c\ub4e4\uc774 \uc9c0\uc6cc\uc9c4 \ud754\uc801\ub4e4, \ub2e4\ud658 \ucda9\ub3cc\uad6c\uc758 \uc911\uc2ec\ubd80, \ucda9\ub3cc\uad6c \ud3c9\uc6d0\uc5d0\uc11c \uc678\ub5a8\uc5b4\uc9c4 \ubd80\ubd84\ub4e4\uc744 \ud3ec\ud568\ud558\ub294 \ud3c9\uc6d0\uc774\ub2e4. \uc774\ub7ec\ud55c \ubc1d\uc740 \ud3c9\uc6d0\ub4e4\uc740 \uc6b4\uc11d \ucda9\ub3cc\ub85c \ubc1c\uc0dd\ud55c \ud1f4\uc801\ubb3c\uc774\ub77c\uace0 \uc5ec\uaca8\uc9c4\ub2e4. \ubc1d\uace0 \ubd80\ub4dc\ub7ec\uc6b4 \ud3c9\uc6d0\uc740 \uce7c\ub9ac\uc2a4\ud1a0 \ud45c\uba74\uc758 \uc77c\ubd80\ubd84\ub9cc\uc744 \uad6c\uc131\ud558\uace0 \uc788\uace0, \ubc1c\ud560\ub77c\ub098 \uc544\uc2a4\uac00\ub974\ub4dc \ucda9\ub3cc\uad6c \ud3c9\uc6d0\uc758 \uace0\ub9bd\ub41c \uc9c0\uc5ed, \uc989 \uc0b0\ub9c8\ub8e8\ub098 \uc8fc\uc0c1\ud574\ubd84\uc5d0\uc11c \ubc1c\uacac\ub41c\ub2e4. \uc774\ub4e4\uc740 \ub0b4\uc778\uc131 \ud65c\ub3d9\uc73c\ub85c \uc5f0\uacb0\ub418\uc5b4 \uc788\uc73c\ub9ac\ub77c\uace0 \uc0dd\uac01\ub418\uc5c8\uc5c8\uc9c0\ub9cc, \uac08\ub9b4\ub808\uc624 \ud0d0\uc0ac\uc120\uc758 \uace0\ud574\uc0c1\ub3c4 \uc0ac\uc9c4\uc5d0\uc11c\ub294 \ubc1d\uace0 \ubd80\ub4dc\ub7ec\uc6b4 \ud3c9\uc6d0\ub4e4\uc774 \ub9e4\uc6b0 \uade0\uc5f4\uc774 \ub9ce\uace0 \uc6b0\ub458\ud22c\ub458\ud55c \uc9c0\ud615\uacfc \ubc00\uc811\ud55c \uc5f0\uad00\uc774 \uc788\uace0, \ud45c\uba74\uc774 \uc0c8\ub85c\uc774 \ub9cc\ub4e4\uc5b4\uc9c4 \ud754\uc801\uc740 \uc804\ud600 \uad00\ucc30\ub418\uc9c0 \uc54a\uc558\ub2e4. \ub610\ud55c \uac08\ub9b4\ub808\uc624\uc758 \uc0ac\uc9c4\ub4e4\uc740 \ub9c8\uce58 \uc8fc\ubcc0 \uc9c0\ud615\uc5d0 \ud3ec\uc704\ub41c \uac83\ucc98\ub7fc \ubcf4\uc774\ub294, \ucd1d \ub113\uc774 10,000 km \uc774\ud558\uc758 \uc791\uace0 \uc5b4\ub450\uc6b0\uba70 \ubd80\ub4dc\ub7ec\uc6b4 \ubd80\ubd84\ub4e4\uc744 \ubcf4\uc5ec\uc8fc\uc5c8\uc73c\uba70, \uc774\ub4e4\uc740 \uc5bc\uc74c\ud654\uc0b0\uc73c\ub85c \uc778\ud574 \uc0dd\uae34 \uac83\uc77c \uc218\ub3c4 \uc788\ub2e4. \ubc1d\uc740 \ud3c9\uc6d0\uacfc \ub2e4\uc591\ud55c \ubd80\ub4dc\ub7ec\uc6b4 \ud3c9\uc6d0\ub4e4\uc740 \uc8fc\ubcc0 \ubc30\uacbd\uc758 \ucda9\ub3cc\uad6c \ud3c9\uc6d0\ub4e4\ubcf4\ub2e4 \uc57d\uac04 \uc80a\uace0 \ucda9\ub3cc\uad6c\ub4e4\uc774 \uc801\ub2e4.", "sentence_answer": "\ubc1d\uc740 \ud3c9\uc6d0\ub4e4\uc740 \uc544\uc2a4\uac00\ub974\ub4dc\ub098 \ub85c\ud508\uacfc \uac19\uc740 \ubc1d\uc740 \ucda9\ub3cc\uad6c\ub4e4\uacfc \ud3a0\ub984\uc2dc\uc2a4\ud2b8 \ub77c\uace0 \ubd88\ub9ac\ub294 \uc624\ub798\ub41c \ucda9\ub3cc\uad6c\ub4e4\uc774 \uc9c0\uc6cc\uc9c4 \ud754\uc801\ub4e4, \ub2e4\ud658 \ucda9\ub3cc\uad6c\uc758 \uc911\uc2ec\ubd80, \ucda9\ub3cc\uad6c \ud3c9\uc6d0\uc5d0\uc11c \uc678\ub5a8\uc5b4\uc9c4 \ubd80\ubd84\ub4e4\uc744 \ud3ec\ud568\ud558\ub294 \ud3c9\uc6d0\uc774\ub2e4.", "paragraph_id": "6561307-4-0", "paragraph_question": "question: \uce7c\ub9ac\uc2a4\ud1a0\uc758 \ud45c\uba74 \uc911 \uc624\ub798\ub41c \ucda9\ub3cc\uad6c\ub4e4\uc774 \uc9c0\uc6cc\uc9c4 \ud754\uc801\uc774 \uc788\ub294 \ubc1d\uc740 \ud3c9\uc6d0\uc740?, context: \uce7c\ub9ac\uc2a4\ud1a0\uc758 \ud45c\uba74\uc740 \ucda9\ub3cc\uad6c\uac00 \ub9ce\uc740 \ud3c9\uc6d0, \ubc1d\uc740 \ud3c9\uc6d0, \ubc1d\uace0 \ubd80\ub4dc\ub7ec\uc6b4 \ud3c9\uc6d0, \ud2b9\uc815\ud55c \ucda9\ub3cc\uad6c\ub4e4\uacfc \uad00\ub828\ub41c \ub2e4\uc591\ud55c \uad6c\uc131 \ub2e8\uc704\ub4e4\ub85c \ub098\ub20c \uc218 \uc788\ub2e4. \ucda9\ub3cc\uad6c\uac00 \ub9ce\uc740 \ud3c9\uc6d0\ub4e4\uc740 \ud45c\uba74\uc758 \ub300\ubd80\ubd84\uc744 \uad6c\uc131\ud558\uace0, \uc5bc\uc74c\uacfc \uc554\uc11d\uc774 \uc11e\uc5ec \ub9cc\ub4e4\uc5b4\uc9c4 \uc624\ub798 \ub41c \uc554\uad8c\uc5d0 \ud574\ub2f9\ud55c\ub2e4. \ubc1d\uc740 \ud3c9\uc6d0\ub4e4\uc740 \uc544\uc2a4\uac00\ub974\ub4dc\ub098 \ub85c\ud508\uacfc \uac19\uc740 \ubc1d\uc740 \ucda9\ub3cc\uad6c\ub4e4\uacfc \ud3a0\ub984\uc2dc\uc2a4\ud2b8\ub77c\uace0 \ubd88\ub9ac\ub294 \uc624\ub798\ub41c \ucda9\ub3cc\uad6c\ub4e4\uc774 \uc9c0\uc6cc\uc9c4 \ud754\uc801\ub4e4, \ub2e4\ud658 \ucda9\ub3cc\uad6c\uc758 \uc911\uc2ec\ubd80, \ucda9\ub3cc\uad6c \ud3c9\uc6d0\uc5d0\uc11c \uc678\ub5a8\uc5b4\uc9c4 \ubd80\ubd84\ub4e4\uc744 \ud3ec\ud568\ud558\ub294 \ud3c9\uc6d0\uc774\ub2e4. \uc774\ub7ec\ud55c \ubc1d\uc740 \ud3c9\uc6d0\ub4e4\uc740 \uc6b4\uc11d \ucda9\ub3cc\ub85c \ubc1c\uc0dd\ud55c \ud1f4\uc801\ubb3c\uc774\ub77c\uace0 \uc5ec\uaca8\uc9c4\ub2e4. \ubc1d\uace0 \ubd80\ub4dc\ub7ec\uc6b4 \ud3c9\uc6d0\uc740 \uce7c\ub9ac\uc2a4\ud1a0 \ud45c\uba74\uc758 \uc77c\ubd80\ubd84\ub9cc\uc744 \uad6c\uc131\ud558\uace0 \uc788\uace0, \ubc1c\ud560\ub77c\ub098 \uc544\uc2a4\uac00\ub974\ub4dc \ucda9\ub3cc\uad6c \ud3c9\uc6d0\uc758 \uace0\ub9bd\ub41c \uc9c0\uc5ed, \uc989 \uc0b0\ub9c8\ub8e8\ub098 \uc8fc\uc0c1\ud574\ubd84\uc5d0\uc11c \ubc1c\uacac\ub41c\ub2e4. \uc774\ub4e4\uc740 \ub0b4\uc778\uc131 \ud65c\ub3d9\uc73c\ub85c \uc5f0\uacb0\ub418\uc5b4 \uc788\uc73c\ub9ac\ub77c\uace0 \uc0dd\uac01\ub418\uc5c8\uc5c8\uc9c0\ub9cc, \uac08\ub9b4\ub808\uc624 \ud0d0\uc0ac\uc120\uc758 \uace0\ud574\uc0c1\ub3c4 \uc0ac\uc9c4\uc5d0\uc11c\ub294 \ubc1d\uace0 \ubd80\ub4dc\ub7ec\uc6b4 \ud3c9\uc6d0\ub4e4\uc774 \ub9e4\uc6b0 \uade0\uc5f4\uc774 \ub9ce\uace0 \uc6b0\ub458\ud22c\ub458\ud55c \uc9c0\ud615\uacfc \ubc00\uc811\ud55c \uc5f0\uad00\uc774 \uc788\uace0, \ud45c\uba74\uc774 \uc0c8\ub85c\uc774 \ub9cc\ub4e4\uc5b4\uc9c4 \ud754\uc801\uc740 \uc804\ud600 \uad00\ucc30\ub418\uc9c0 \uc54a\uc558\ub2e4. \ub610\ud55c \uac08\ub9b4\ub808\uc624\uc758 \uc0ac\uc9c4\ub4e4\uc740 \ub9c8\uce58 \uc8fc\ubcc0 \uc9c0\ud615\uc5d0 \ud3ec\uc704\ub41c \uac83\ucc98\ub7fc \ubcf4\uc774\ub294, \ucd1d \ub113\uc774 10,000 km \uc774\ud558\uc758 \uc791\uace0 \uc5b4\ub450\uc6b0\uba70 \ubd80\ub4dc\ub7ec\uc6b4 \ubd80\ubd84\ub4e4\uc744 \ubcf4\uc5ec\uc8fc\uc5c8\uc73c\uba70, \uc774\ub4e4\uc740 \uc5bc\uc74c\ud654\uc0b0\uc73c\ub85c \uc778\ud574 \uc0dd\uae34 \uac83\uc77c \uc218\ub3c4 \uc788\ub2e4. \ubc1d\uc740 \ud3c9\uc6d0\uacfc \ub2e4\uc591\ud55c \ubd80\ub4dc\ub7ec\uc6b4 \ud3c9\uc6d0\ub4e4\uc740 \uc8fc\ubcc0 \ubc30\uacbd\uc758 \ucda9\ub3cc\uad6c \ud3c9\uc6d0\ub4e4\ubcf4\ub2e4 \uc57d\uac04 \uc80a\uace0 \ucda9\ub3cc\uad6c\ub4e4\uc774 \uc801\ub2e4."} {"question": "\"\ubbfc\uc871\uad6d\uac00 \uc218\ub9bd\uc5d0 \uae30\uc5ec\ud560 \uc9c0\uc5ed\uc0ac\ud68c\uc758 \uc778\uc7ac\ub97c \uc591\uc131\"\uc774\ub77c\ub294 \ub9d0\uc740 \uc870\uc120\ub300\ud559\uad50\uc758 \ubb34\uc5c7\uc744 \ub098\ud0c0\ub0b4\uace0 \uc788\uc744\uae4c?", "paragraph": "1946\ub144 8\uc6d4 5\uc77c \u201c\uac1c\uc131\uad50\uc721\u00b7\uc0dd\uc0b0\uad50\uc721\u00b7\uc601\uc7ac\uad50\uc721\u201d\uc758 \uac74\ud559 \uc774\ub150\uacfc \u201c\ubbfc\uc871\uad6d\uac00 \uc218\ub9bd\uc5d0 \uae30\uc5ec\ud560 \uc9c0\uc5ed\uc0ac\ud68c\uc758 \uc778\uc7ac\ub97c \uc591\uc131\u201d\uc774\ub77c\ub294 \uc124\ub9bd \uc774\ub150\uc73c\ub85c \uad11\uc8fc\uc11c\uc911\ud559\uad50\uc5d0\uc11c \ubc1c\uc871\uc2dd\uc744 \ud588\uc73c\uba70 \ub3d9\ub144 9\uc6d4 9\uc77c \uad11\uc8fc\uc57c\uac04\ub300\ud559\uc6d0(\u5149\u5dde\u591c\u9593\u5927\u5b78\u5712)\uc774 \uc124\ub9bd \uc778\uac00\ub418\uc5c8\uace0 9\uc6d4 29\uc77c \uad11\uc8fc\uc57c\uac04\ub300\ud559\uc6d0[4\ud559\ubd80, 12\ud559\uacfc, 1,194\uba85]\uc744 \uac1c\uad50\ud558\uc600\ub2e4. \uadf8 \ud574 11\uc6d4 23\uc77c \uad11\uc8fc \uc9c0\uc5ed\uc5d0 \uad6d\ud55c\ub41c \ub300\ud559\uc774 \uc544\ub2cc \uc804\uad6d\uc758 \uc778\uc7ac\ub97c \ud0a4\uc6b0\ub294 \ub300\ud559\uc774 \ub9cc\ub4e4\uc5b4\uc9c0\uae30\ub97c \ubc14\ub77c\uba74\uc11c \uc815\ubd80\uc218\ub9bd \uc804\uc5d0 \uc778\uc7ac \uc591\uc131\uc774 \ubbfc\uc871\uacfc \uad6d\uac00\uc758 \ubbf8\ub798\ub97c \uc774\ub044\ub294 \ucd08\uc11d\uc774\ub77c\ub294 \uc0ac\uc2e4\uc744 \uba85\uc2ec\ud558\uace0 \uc11c\ubbfc\ud638 \uc804\ub77c\ub0a8\ub3c4\uc9c0\uc0ac\uc758 \ub73b\uc744 \uc887\uc544 \uad50\uba85\uc744 '\ud574\ub728\ub294 \uc544\uce68'\uc774\ub77c\ub294 \ubbfc\uc871 \uacf5\ub3d9\uccb4\ub97c \uc0c1\uc9d5\ud558\uba74\uc11c \uc9c4\ucde8\uc131\uc744 \ub764 \uae30\uc0c1\uc744 \ud568\uc758\ud55c \uc870\uc120\ub300\ud559(\u671d\u9bae\u5927\u5b78)\uc73c\ub85c \uac1c\uce6d\ud558\uc600\uace0 1947\ub144 \uc124\ub9bd\ub3d9\uc9c0\ud68c\ub294 \ud68c\uc6d0 7\ub9cc 2\ucc9c\uc5ec \uba85\uc744 \ud655\ubcf4\ud558\uace0 1948\ub144 5\uc6d4 26\uc77c \uc7ac\ub2e8\ubc95\uc778 \uc870\uc120\ub300\ud559\uc774 \ubbf8\uad70\uc815\uccad\uc5d0\uc11c \uc124\ub9bd\uc774 \uc778\uac00\ub418\uc5c8\ub294\ub370 \uc774\ub294 1988\ub144 \u300a\ud55c\uaca8\ub808\uc2e0\ubb38\u300b\uc774 \ucc3d\uac04\ub420 \ub2f9\uc2dc \uad6d\ubbfc 4\ucc9c\uc5ec\ub9cc \uba85 \uc911\uc5d0\uc11c \ubaa8\uc9d1\ud55c \uc8fc\uc8fc\uac00 6\ub9cc 1\ucc9c\uc5ec \uba85\uc774\uc5c8\ub2e4\ub294 \uc0ac\uc2e4\uc744 \uc0c1\uae30\ud560 \ub54c \uc804\uad6d \uc778\uad6c\uac00 1,600\ub9cc \uba85\uc774\ub358 1947\ub144 \ub2f9\uc2dc \ud638\ub0a8 \uc9c0\uc5ed\uc744 \uc911\uc2ec\uc73c\ub85c 7\ub9cc 2\ucc9c\uc5ec \uba85\uc774 \uc131\uae08\uc744 \ub0b4\uc5b4 \ub300\ud559\uc744 \uac74\uc124\ud558\uc600\ub2e4\ub294 \uc0ac\uc2e4\uc5d0 \ud070 \uc758\ubbf8\ub97c \ub458 \uc218 \uc788\ub2e4.", "answer": "\uc124\ub9bd \uc774\ub150", "sentence": "1946\ub144 8\uc6d4 5\uc77c \u201c\uac1c\uc131\uad50\uc721\u00b7\uc0dd\uc0b0\uad50\uc721\u00b7\uc601\uc7ac\uad50\uc721\u201d\uc758 \uac74\ud559 \uc774\ub150\uacfc \u201c\ubbfc\uc871\uad6d\uac00 \uc218\ub9bd\uc5d0 \uae30\uc5ec\ud560 \uc9c0\uc5ed\uc0ac\ud68c\uc758 \uc778\uc7ac\ub97c \uc591\uc131\u201d\uc774\ub77c\ub294 \uc124\ub9bd \uc774\ub150 \uc73c\ub85c \uad11\uc8fc\uc11c\uc911\ud559\uad50\uc5d0\uc11c \ubc1c\uc871\uc2dd\uc744 \ud588\uc73c\uba70 \ub3d9\ub144 9\uc6d4 9\uc77c \uad11\uc8fc\uc57c\uac04\ub300\ud559\uc6d0(\u5149\u5dde\u591c\u9593\u5927\u5b78\u5712)\uc774 \uc124\ub9bd \uc778\uac00\ub418\uc5c8\uace0 9\uc6d4 29\uc77c \uad11\uc8fc\uc57c\uac04\ub300\ud559\uc6d0[4\ud559\ubd80, 12\ud559\uacfc, 1,194\uba85]\uc744 \uac1c\uad50\ud558\uc600\ub2e4.", "paragraph_sentence": " 1946\ub144 8\uc6d4 5\uc77c \u201c\uac1c\uc131\uad50\uc721\u00b7\uc0dd\uc0b0\uad50\uc721\u00b7\uc601\uc7ac\uad50\uc721\u201d\uc758 \uac74\ud559 \uc774\ub150\uacfc \u201c\ubbfc\uc871\uad6d\uac00 \uc218\ub9bd\uc5d0 \uae30\uc5ec\ud560 \uc9c0\uc5ed\uc0ac\ud68c\uc758 \uc778\uc7ac\ub97c \uc591\uc131\u201d\uc774\ub77c\ub294 \uc124\ub9bd \uc774\ub150 \uc73c\ub85c \uad11\uc8fc\uc11c\uc911\ud559\uad50\uc5d0\uc11c \ubc1c\uc871\uc2dd\uc744 \ud588\uc73c\uba70 \ub3d9\ub144 9\uc6d4 9\uc77c \uad11\uc8fc\uc57c\uac04\ub300\ud559\uc6d0(\u5149\u5dde\u591c\u9593\u5927\u5b78\u5712)\uc774 \uc124\ub9bd \uc778\uac00\ub418\uc5c8\uace0 9\uc6d4 29\uc77c \uad11\uc8fc\uc57c\uac04\ub300\ud559\uc6d0[4\ud559\ubd80, 12\ud559\uacfc, 1,194\uba85]\uc744 \uac1c\uad50\ud558\uc600\ub2e4. \uadf8 \ud574 11\uc6d4 23\uc77c \uad11\uc8fc \uc9c0\uc5ed\uc5d0 \uad6d\ud55c\ub41c \ub300\ud559\uc774 \uc544\ub2cc \uc804\uad6d\uc758 \uc778\uc7ac\ub97c \ud0a4\uc6b0\ub294 \ub300\ud559\uc774 \ub9cc\ub4e4\uc5b4\uc9c0\uae30\ub97c \ubc14\ub77c\uba74\uc11c \uc815\ubd80\uc218\ub9bd \uc804\uc5d0 \uc778\uc7ac \uc591\uc131\uc774 \ubbfc\uc871\uacfc \uad6d\uac00\uc758 \ubbf8\ub798\ub97c \uc774\ub044\ub294 \ucd08\uc11d\uc774\ub77c\ub294 \uc0ac\uc2e4\uc744 \uba85\uc2ec\ud558\uace0 \uc11c\ubbfc\ud638 \uc804\ub77c\ub0a8\ub3c4\uc9c0\uc0ac\uc758 \ub73b\uc744 \uc887\uc544 \uad50\uba85\uc744 '\ud574\ub728\ub294 \uc544\uce68'\uc774\ub77c\ub294 \ubbfc\uc871 \uacf5\ub3d9\uccb4\ub97c \uc0c1\uc9d5\ud558\uba74\uc11c \uc9c4\ucde8\uc131\uc744 \ub764 \uae30\uc0c1\uc744 \ud568\uc758\ud55c \uc870\uc120\ub300\ud559(\u671d\u9bae\u5927\u5b78)\uc73c\ub85c \uac1c\uce6d\ud558\uc600\uace0 1947\ub144 \uc124\ub9bd\ub3d9\uc9c0\ud68c\ub294 \ud68c\uc6d0 7\ub9cc 2\ucc9c\uc5ec \uba85\uc744 \ud655\ubcf4\ud558\uace0 1948\ub144 5\uc6d4 26\uc77c \uc7ac\ub2e8\ubc95\uc778 \uc870\uc120\ub300\ud559\uc774 \ubbf8\uad70\uc815\uccad\uc5d0\uc11c \uc124\ub9bd\uc774 \uc778\uac00\ub418\uc5c8\ub294\ub370 \uc774\ub294 1988\ub144 \u300a\ud55c\uaca8\ub808\uc2e0\ubb38\u300b\uc774 \ucc3d\uac04\ub420 \ub2f9\uc2dc \uad6d\ubbfc 4\ucc9c\uc5ec\ub9cc \uba85 \uc911\uc5d0\uc11c \ubaa8\uc9d1\ud55c \uc8fc\uc8fc\uac00 6\ub9cc 1\ucc9c\uc5ec \uba85\uc774\uc5c8\ub2e4\ub294 \uc0ac\uc2e4\uc744 \uc0c1\uae30\ud560 \ub54c \uc804\uad6d \uc778\uad6c\uac00 1,600\ub9cc \uba85\uc774\ub358 1947\ub144 \ub2f9\uc2dc \ud638\ub0a8 \uc9c0\uc5ed\uc744 \uc911\uc2ec\uc73c\ub85c 7\ub9cc 2\ucc9c\uc5ec \uba85\uc774 \uc131\uae08\uc744 \ub0b4\uc5b4 \ub300\ud559\uc744 \uac74\uc124\ud558\uc600\ub2e4\ub294 \uc0ac\uc2e4\uc5d0 \ud070 \uc758\ubbf8\ub97c \ub458 \uc218 \uc788\ub2e4.", "paragraph_answer": "1946\ub144 8\uc6d4 5\uc77c \u201c\uac1c\uc131\uad50\uc721\u00b7\uc0dd\uc0b0\uad50\uc721\u00b7\uc601\uc7ac\uad50\uc721\u201d\uc758 \uac74\ud559 \uc774\ub150\uacfc \u201c\ubbfc\uc871\uad6d\uac00 \uc218\ub9bd\uc5d0 \uae30\uc5ec\ud560 \uc9c0\uc5ed\uc0ac\ud68c\uc758 \uc778\uc7ac\ub97c \uc591\uc131\u201d\uc774\ub77c\ub294 \uc124\ub9bd \uc774\ub150 \uc73c\ub85c \uad11\uc8fc\uc11c\uc911\ud559\uad50\uc5d0\uc11c \ubc1c\uc871\uc2dd\uc744 \ud588\uc73c\uba70 \ub3d9\ub144 9\uc6d4 9\uc77c \uad11\uc8fc\uc57c\uac04\ub300\ud559\uc6d0(\u5149\u5dde\u591c\u9593\u5927\u5b78\u5712)\uc774 \uc124\ub9bd \uc778\uac00\ub418\uc5c8\uace0 9\uc6d4 29\uc77c \uad11\uc8fc\uc57c\uac04\ub300\ud559\uc6d0[4\ud559\ubd80, 12\ud559\uacfc, 1,194\uba85]\uc744 \uac1c\uad50\ud558\uc600\ub2e4. \uadf8 \ud574 11\uc6d4 23\uc77c \uad11\uc8fc \uc9c0\uc5ed\uc5d0 \uad6d\ud55c\ub41c \ub300\ud559\uc774 \uc544\ub2cc \uc804\uad6d\uc758 \uc778\uc7ac\ub97c \ud0a4\uc6b0\ub294 \ub300\ud559\uc774 \ub9cc\ub4e4\uc5b4\uc9c0\uae30\ub97c \ubc14\ub77c\uba74\uc11c \uc815\ubd80\uc218\ub9bd \uc804\uc5d0 \uc778\uc7ac \uc591\uc131\uc774 \ubbfc\uc871\uacfc \uad6d\uac00\uc758 \ubbf8\ub798\ub97c \uc774\ub044\ub294 \ucd08\uc11d\uc774\ub77c\ub294 \uc0ac\uc2e4\uc744 \uba85\uc2ec\ud558\uace0 \uc11c\ubbfc\ud638 \uc804\ub77c\ub0a8\ub3c4\uc9c0\uc0ac\uc758 \ub73b\uc744 \uc887\uc544 \uad50\uba85\uc744 '\ud574\ub728\ub294 \uc544\uce68'\uc774\ub77c\ub294 \ubbfc\uc871 \uacf5\ub3d9\uccb4\ub97c \uc0c1\uc9d5\ud558\uba74\uc11c \uc9c4\ucde8\uc131\uc744 \ub764 \uae30\uc0c1\uc744 \ud568\uc758\ud55c \uc870\uc120\ub300\ud559(\u671d\u9bae\u5927\u5b78)\uc73c\ub85c \uac1c\uce6d\ud558\uc600\uace0 1947\ub144 \uc124\ub9bd\ub3d9\uc9c0\ud68c\ub294 \ud68c\uc6d0 7\ub9cc 2\ucc9c\uc5ec \uba85\uc744 \ud655\ubcf4\ud558\uace0 1948\ub144 5\uc6d4 26\uc77c \uc7ac\ub2e8\ubc95\uc778 \uc870\uc120\ub300\ud559\uc774 \ubbf8\uad70\uc815\uccad\uc5d0\uc11c \uc124\ub9bd\uc774 \uc778\uac00\ub418\uc5c8\ub294\ub370 \uc774\ub294 1988\ub144 \u300a\ud55c\uaca8\ub808\uc2e0\ubb38\u300b\uc774 \ucc3d\uac04\ub420 \ub2f9\uc2dc \uad6d\ubbfc 4\ucc9c\uc5ec\ub9cc \uba85 \uc911\uc5d0\uc11c \ubaa8\uc9d1\ud55c \uc8fc\uc8fc\uac00 6\ub9cc 1\ucc9c\uc5ec \uba85\uc774\uc5c8\ub2e4\ub294 \uc0ac\uc2e4\uc744 \uc0c1\uae30\ud560 \ub54c \uc804\uad6d \uc778\uad6c\uac00 1,600\ub9cc \uba85\uc774\ub358 1947\ub144 \ub2f9\uc2dc \ud638\ub0a8 \uc9c0\uc5ed\uc744 \uc911\uc2ec\uc73c\ub85c 7\ub9cc 2\ucc9c\uc5ec \uba85\uc774 \uc131\uae08\uc744 \ub0b4\uc5b4 \ub300\ud559\uc744 \uac74\uc124\ud558\uc600\ub2e4\ub294 \uc0ac\uc2e4\uc5d0 \ud070 \uc758\ubbf8\ub97c \ub458 \uc218 \uc788\ub2e4.", "sentence_answer": "1946\ub144 8\uc6d4 5\uc77c \u201c\uac1c\uc131\uad50\uc721\u00b7\uc0dd\uc0b0\uad50\uc721\u00b7\uc601\uc7ac\uad50\uc721\u201d\uc758 \uac74\ud559 \uc774\ub150\uacfc \u201c\ubbfc\uc871\uad6d\uac00 \uc218\ub9bd\uc5d0 \uae30\uc5ec\ud560 \uc9c0\uc5ed\uc0ac\ud68c\uc758 \uc778\uc7ac\ub97c \uc591\uc131\u201d\uc774\ub77c\ub294 \uc124\ub9bd \uc774\ub150 \uc73c\ub85c \uad11\uc8fc\uc11c\uc911\ud559\uad50\uc5d0\uc11c \ubc1c\uc871\uc2dd\uc744 \ud588\uc73c\uba70 \ub3d9\ub144 9\uc6d4 9\uc77c \uad11\uc8fc\uc57c\uac04\ub300\ud559\uc6d0(\u5149\u5dde\u591c\u9593\u5927\u5b78\u5712)\uc774 \uc124\ub9bd \uc778\uac00\ub418\uc5c8\uace0 9\uc6d4 29\uc77c \uad11\uc8fc\uc57c\uac04\ub300\ud559\uc6d0[4\ud559\ubd80, 12\ud559\uacfc, 1,194\uba85]\uc744 \uac1c\uad50\ud558\uc600\ub2e4.", "paragraph_id": "6392948-2-0", "paragraph_question": "question: \"\ubbfc\uc871\uad6d\uac00 \uc218\ub9bd\uc5d0 \uae30\uc5ec\ud560 \uc9c0\uc5ed\uc0ac\ud68c\uc758 \uc778\uc7ac\ub97c \uc591\uc131\"\uc774\ub77c\ub294 \ub9d0\uc740 \uc870\uc120\ub300\ud559\uad50\uc758 \ubb34\uc5c7\uc744 \ub098\ud0c0\ub0b4\uace0 \uc788\uc744\uae4c?, context: 1946\ub144 8\uc6d4 5\uc77c \u201c\uac1c\uc131\uad50\uc721\u00b7\uc0dd\uc0b0\uad50\uc721\u00b7\uc601\uc7ac\uad50\uc721\u201d\uc758 \uac74\ud559 \uc774\ub150\uacfc \u201c\ubbfc\uc871\uad6d\uac00 \uc218\ub9bd\uc5d0 \uae30\uc5ec\ud560 \uc9c0\uc5ed\uc0ac\ud68c\uc758 \uc778\uc7ac\ub97c \uc591\uc131\u201d\uc774\ub77c\ub294 \uc124\ub9bd \uc774\ub150\uc73c\ub85c \uad11\uc8fc\uc11c\uc911\ud559\uad50\uc5d0\uc11c \ubc1c\uc871\uc2dd\uc744 \ud588\uc73c\uba70 \ub3d9\ub144 9\uc6d4 9\uc77c \uad11\uc8fc\uc57c\uac04\ub300\ud559\uc6d0(\u5149\u5dde\u591c\u9593\u5927\u5b78\u5712)\uc774 \uc124\ub9bd \uc778\uac00\ub418\uc5c8\uace0 9\uc6d4 29\uc77c \uad11\uc8fc\uc57c\uac04\ub300\ud559\uc6d0[4\ud559\ubd80, 12\ud559\uacfc, 1,194\uba85]\uc744 \uac1c\uad50\ud558\uc600\ub2e4. \uadf8 \ud574 11\uc6d4 23\uc77c \uad11\uc8fc \uc9c0\uc5ed\uc5d0 \uad6d\ud55c\ub41c \ub300\ud559\uc774 \uc544\ub2cc \uc804\uad6d\uc758 \uc778\uc7ac\ub97c \ud0a4\uc6b0\ub294 \ub300\ud559\uc774 \ub9cc\ub4e4\uc5b4\uc9c0\uae30\ub97c \ubc14\ub77c\uba74\uc11c \uc815\ubd80\uc218\ub9bd \uc804\uc5d0 \uc778\uc7ac \uc591\uc131\uc774 \ubbfc\uc871\uacfc \uad6d\uac00\uc758 \ubbf8\ub798\ub97c \uc774\ub044\ub294 \ucd08\uc11d\uc774\ub77c\ub294 \uc0ac\uc2e4\uc744 \uba85\uc2ec\ud558\uace0 \uc11c\ubbfc\ud638 \uc804\ub77c\ub0a8\ub3c4\uc9c0\uc0ac\uc758 \ub73b\uc744 \uc887\uc544 \uad50\uba85\uc744 '\ud574\ub728\ub294 \uc544\uce68'\uc774\ub77c\ub294 \ubbfc\uc871 \uacf5\ub3d9\uccb4\ub97c \uc0c1\uc9d5\ud558\uba74\uc11c \uc9c4\ucde8\uc131\uc744 \ub764 \uae30\uc0c1\uc744 \ud568\uc758\ud55c \uc870\uc120\ub300\ud559(\u671d\u9bae\u5927\u5b78)\uc73c\ub85c \uac1c\uce6d\ud558\uc600\uace0 1947\ub144 \uc124\ub9bd\ub3d9\uc9c0\ud68c\ub294 \ud68c\uc6d0 7\ub9cc 2\ucc9c\uc5ec \uba85\uc744 \ud655\ubcf4\ud558\uace0 1948\ub144 5\uc6d4 26\uc77c \uc7ac\ub2e8\ubc95\uc778 \uc870\uc120\ub300\ud559\uc774 \ubbf8\uad70\uc815\uccad\uc5d0\uc11c \uc124\ub9bd\uc774 \uc778\uac00\ub418\uc5c8\ub294\ub370 \uc774\ub294 1988\ub144 \u300a\ud55c\uaca8\ub808\uc2e0\ubb38\u300b\uc774 \ucc3d\uac04\ub420 \ub2f9\uc2dc \uad6d\ubbfc 4\ucc9c\uc5ec\ub9cc \uba85 \uc911\uc5d0\uc11c \ubaa8\uc9d1\ud55c \uc8fc\uc8fc\uac00 6\ub9cc 1\ucc9c\uc5ec \uba85\uc774\uc5c8\ub2e4\ub294 \uc0ac\uc2e4\uc744 \uc0c1\uae30\ud560 \ub54c \uc804\uad6d \uc778\uad6c\uac00 1,600\ub9cc \uba85\uc774\ub358 1947\ub144 \ub2f9\uc2dc \ud638\ub0a8 \uc9c0\uc5ed\uc744 \uc911\uc2ec\uc73c\ub85c 7\ub9cc 2\ucc9c\uc5ec \uba85\uc774 \uc131\uae08\uc744 \ub0b4\uc5b4 \ub300\ud559\uc744 \uac74\uc124\ud558\uc600\ub2e4\ub294 \uc0ac\uc2e4\uc5d0 \ud070 \uc758\ubbf8\ub97c \ub458 \uc218 \uc788\ub2e4."} {"question": "\uc911\ub3d9 \ubd81\uc544\ud504\ub9ac\uce74\ub4f1 \uc544\ub78d\uad8c\uc5d0\uc11c \uc0ac\uc6a9\ud558\ub294 \uacf5\uc2dd\uc5b8\uc5b4\ub294?", "paragraph": "\ud604\ub300 \ud45c\uc900 \uc544\ub78d\uc5b4\ub294 \uace0\uc804 \uc544\ub78d\uc5b4\ub97c \ubc14\ud0d5\uc73c\ub85c 19\uc138\uae30 \ucd08\ubc18\uc5d0 \ud615\uc131\ub41c \uc544\ub78d\uc5b4\ub85c, \uc911\ub3d9, \ubd81\uc544\ud504\ub9ac\uce74, \uc18c\ub9d0\ub9ac\uc544 \ubc18\ub3c4 \uc9c0\uc5ed\uc758 \ud45c\uc900 \ud45c\uae30 \uc218\ub2e8\uc73c\ub85c, \uc601\uc5b4, \ud504\ub791\uc2a4\uc5b4, \uc2a4\ud398\uc778\uc5b4, \ub7ec\uc2dc\uc544\uc5b4, \uc911\uad6d\uc5b4\uc640 \ub354\ubd88\uc5b4 \uc720\uc5d4\uc758 6\uac1c \uacf5\uc6a9\uc5b4 \uc911 \ud558\ub098\uc774\ub2e4. \uc544\ub78d \uc9c0\uc5ed\uc5d0\uc11c \ubc1c\ud589\ub418\ub294 \ubb38\uac74 - \ub300\ub2e4\uc218\uc758 \ucc45, \uc2e0\ubb38, \uc7a1\uc9c0, \uacf5\ubb38\uc11c, \uc5b4\ub9b0\uc774\ub97c \uc704\ud55c \uae30\ucd08 \uad50\uc7ac \ubc0f \uc77d\uc744\uac70\ub9ac - \uc740 \uc774 \uc544\ub78d\uc5b4\ub85c \uc791\uc131\ub41c\ub2e4. '\uc544\ub78d\uc5b4'\ub97c \uc0ac\uc6a9\ud558\ub294 \uc9c0\uc5ed\uc758 \uc720\uc77c\ud55c \uacf5\uc2dd \uc5b8\uc5b4\uc778 \ud45c\uc900 \uc544\ub78d\uc5b4\ub294 \ud559\uad50 \uad50\uacfc \uacfc\uc815\uc5d0\uc11c \uac00\ub974\uccd0\uc9c0\ub294 \uc720\uc77c\ud55c \uc544\ub78d\uc5b4\uc774\uae30\ub3c4 \ud558\ub2e4. \ud604\ub300 \ud45c\uc900 \uc544\ub78d\uc5b4\ub294 \uace0\uc804 \uc544\ub78d\uc5b4(\ud2b9\ud788 \uc774\uc2ac\ub78c \uc774\uc804\uc758 \uc2dc\ub300\ubd80\ud130, \uafb8\ub780\uc744 \ube44\ub86f\ud55c \uc555\ubc14\uc2a4 \uc2dc\ub300\uc758 \uc5b8\uc5b4)\ub97c \uadfc\uac04\uc73c\ub85c \ud558\uace0 \uc788\ub2e4. \uafb8\ub780 \ubc0f \uace0\ub300 \uc6b0\ub9c8\ubbf8\uc57c \ubc0f \uc555\ubc14\uc2a4 \uc655\uc870 \uc2dc\ub300\uc5d0 \uc791\uc131\ub41c \ubb38\uac74\ub4e4\uc758 \ud604\ub300\uc5b4 \ud310\uc744 \uc791\uc131\ud558\ub294\ub370 \uc0ac\uc6a9\ub418\uae30\ub3c4 \ud55c\ub2e4.", "answer": "\ud45c\uc900 \uc544\ub78d\uc5b4", "sentence": "\ud604\ub300 \ud45c\uc900 \uc544\ub78d\uc5b4 \ub294 \uace0\uc804 \uc544\ub78d\uc5b4\ub97c \ubc14\ud0d5\uc73c\ub85c 19\uc138\uae30 \ucd08\ubc18\uc5d0 \ud615\uc131\ub41c \uc544\ub78d\uc5b4\ub85c, \uc911\ub3d9, \ubd81\uc544\ud504\ub9ac\uce74, \uc18c\ub9d0\ub9ac\uc544 \ubc18\ub3c4 \uc9c0\uc5ed\uc758 \ud45c\uc900 \ud45c\uae30 \uc218\ub2e8\uc73c\ub85c, \uc601\uc5b4, \ud504\ub791\uc2a4\uc5b4, \uc2a4\ud398\uc778\uc5b4, \ub7ec\uc2dc\uc544\uc5b4, \uc911\uad6d\uc5b4\uc640 \ub354\ubd88\uc5b4 \uc720\uc5d4\uc758 6\uac1c \uacf5\uc6a9\uc5b4 \uc911 \ud558\ub098\uc774\ub2e4.", "paragraph_sentence": " \ud604\ub300 \ud45c\uc900 \uc544\ub78d\uc5b4 \ub294 \uace0\uc804 \uc544\ub78d\uc5b4\ub97c \ubc14\ud0d5\uc73c\ub85c 19\uc138\uae30 \ucd08\ubc18\uc5d0 \ud615\uc131\ub41c \uc544\ub78d\uc5b4\ub85c, \uc911\ub3d9, \ubd81\uc544\ud504\ub9ac\uce74, \uc18c\ub9d0\ub9ac\uc544 \ubc18\ub3c4 \uc9c0\uc5ed\uc758 \ud45c\uc900 \ud45c\uae30 \uc218\ub2e8\uc73c\ub85c, \uc601\uc5b4, \ud504\ub791\uc2a4\uc5b4, \uc2a4\ud398\uc778\uc5b4, \ub7ec\uc2dc\uc544\uc5b4, \uc911\uad6d\uc5b4\uc640 \ub354\ubd88\uc5b4 \uc720\uc5d4\uc758 6\uac1c \uacf5\uc6a9\uc5b4 \uc911 \ud558\ub098\uc774\ub2e4. \uc544\ub78d \uc9c0\uc5ed\uc5d0\uc11c \ubc1c\ud589\ub418\ub294 \ubb38\uac74 - \ub300\ub2e4\uc218\uc758 \ucc45, \uc2e0\ubb38, \uc7a1\uc9c0, \uacf5\ubb38\uc11c, \uc5b4\ub9b0\uc774\ub97c \uc704\ud55c \uae30\ucd08 \uad50\uc7ac \ubc0f \uc77d\uc744\uac70\ub9ac - \uc740 \uc774 \uc544\ub78d\uc5b4\ub85c \uc791\uc131\ub41c\ub2e4. '\uc544\ub78d\uc5b4'\ub97c \uc0ac\uc6a9\ud558\ub294 \uc9c0\uc5ed\uc758 \uc720\uc77c\ud55c \uacf5\uc2dd \uc5b8\uc5b4\uc778 \ud45c\uc900 \uc544\ub78d\uc5b4\ub294 \ud559\uad50 \uad50\uacfc \uacfc\uc815\uc5d0\uc11c \uac00\ub974\uccd0\uc9c0\ub294 \uc720\uc77c\ud55c \uc544\ub78d\uc5b4\uc774\uae30\ub3c4 \ud558\ub2e4. \ud604\ub300 \ud45c\uc900 \uc544\ub78d\uc5b4\ub294 \uace0\uc804 \uc544\ub78d\uc5b4(\ud2b9\ud788 \uc774\uc2ac\ub78c \uc774\uc804\uc758 \uc2dc\ub300\ubd80\ud130, \uafb8\ub780\uc744 \ube44\ub86f\ud55c \uc555\ubc14\uc2a4 \uc2dc\ub300\uc758 \uc5b8\uc5b4)\ub97c \uadfc\uac04\uc73c\ub85c \ud558\uace0 \uc788\ub2e4. \uafb8\ub780 \ubc0f \uace0\ub300 \uc6b0\ub9c8\ubbf8\uc57c \ubc0f \uc555\ubc14\uc2a4 \uc655\uc870 \uc2dc\ub300\uc5d0 \uc791\uc131\ub41c \ubb38\uac74\ub4e4\uc758 \ud604\ub300\uc5b4 \ud310\uc744 \uc791\uc131\ud558\ub294\ub370 \uc0ac\uc6a9\ub418\uae30\ub3c4 \ud55c\ub2e4.", "paragraph_answer": "\ud604\ub300 \ud45c\uc900 \uc544\ub78d\uc5b4 \ub294 \uace0\uc804 \uc544\ub78d\uc5b4\ub97c \ubc14\ud0d5\uc73c\ub85c 19\uc138\uae30 \ucd08\ubc18\uc5d0 \ud615\uc131\ub41c \uc544\ub78d\uc5b4\ub85c, \uc911\ub3d9, \ubd81\uc544\ud504\ub9ac\uce74, \uc18c\ub9d0\ub9ac\uc544 \ubc18\ub3c4 \uc9c0\uc5ed\uc758 \ud45c\uc900 \ud45c\uae30 \uc218\ub2e8\uc73c\ub85c, \uc601\uc5b4, \ud504\ub791\uc2a4\uc5b4, \uc2a4\ud398\uc778\uc5b4, \ub7ec\uc2dc\uc544\uc5b4, \uc911\uad6d\uc5b4\uc640 \ub354\ubd88\uc5b4 \uc720\uc5d4\uc758 6\uac1c \uacf5\uc6a9\uc5b4 \uc911 \ud558\ub098\uc774\ub2e4. \uc544\ub78d \uc9c0\uc5ed\uc5d0\uc11c \ubc1c\ud589\ub418\ub294 \ubb38\uac74 - \ub300\ub2e4\uc218\uc758 \ucc45, \uc2e0\ubb38, \uc7a1\uc9c0, \uacf5\ubb38\uc11c, \uc5b4\ub9b0\uc774\ub97c \uc704\ud55c \uae30\ucd08 \uad50\uc7ac \ubc0f \uc77d\uc744\uac70\ub9ac - \uc740 \uc774 \uc544\ub78d\uc5b4\ub85c \uc791\uc131\ub41c\ub2e4. '\uc544\ub78d\uc5b4'\ub97c \uc0ac\uc6a9\ud558\ub294 \uc9c0\uc5ed\uc758 \uc720\uc77c\ud55c \uacf5\uc2dd \uc5b8\uc5b4\uc778 \ud45c\uc900 \uc544\ub78d\uc5b4\ub294 \ud559\uad50 \uad50\uacfc \uacfc\uc815\uc5d0\uc11c \uac00\ub974\uccd0\uc9c0\ub294 \uc720\uc77c\ud55c \uc544\ub78d\uc5b4\uc774\uae30\ub3c4 \ud558\ub2e4. \ud604\ub300 \ud45c\uc900 \uc544\ub78d\uc5b4\ub294 \uace0\uc804 \uc544\ub78d\uc5b4(\ud2b9\ud788 \uc774\uc2ac\ub78c \uc774\uc804\uc758 \uc2dc\ub300\ubd80\ud130, \uafb8\ub780\uc744 \ube44\ub86f\ud55c \uc555\ubc14\uc2a4 \uc2dc\ub300\uc758 \uc5b8\uc5b4)\ub97c \uadfc\uac04\uc73c\ub85c \ud558\uace0 \uc788\ub2e4. \uafb8\ub780 \ubc0f \uace0\ub300 \uc6b0\ub9c8\ubbf8\uc57c \ubc0f \uc555\ubc14\uc2a4 \uc655\uc870 \uc2dc\ub300\uc5d0 \uc791\uc131\ub41c \ubb38\uac74\ub4e4\uc758 \ud604\ub300\uc5b4 \ud310\uc744 \uc791\uc131\ud558\ub294\ub370 \uc0ac\uc6a9\ub418\uae30\ub3c4 \ud55c\ub2e4.", "sentence_answer": "\ud604\ub300 \ud45c\uc900 \uc544\ub78d\uc5b4 \ub294 \uace0\uc804 \uc544\ub78d\uc5b4\ub97c \ubc14\ud0d5\uc73c\ub85c 19\uc138\uae30 \ucd08\ubc18\uc5d0 \ud615\uc131\ub41c \uc544\ub78d\uc5b4\ub85c, \uc911\ub3d9, \ubd81\uc544\ud504\ub9ac\uce74, \uc18c\ub9d0\ub9ac\uc544 \ubc18\ub3c4 \uc9c0\uc5ed\uc758 \ud45c\uc900 \ud45c\uae30 \uc218\ub2e8\uc73c\ub85c, \uc601\uc5b4, \ud504\ub791\uc2a4\uc5b4, \uc2a4\ud398\uc778\uc5b4, \ub7ec\uc2dc\uc544\uc5b4, \uc911\uad6d\uc5b4\uc640 \ub354\ubd88\uc5b4 \uc720\uc5d4\uc758 6\uac1c \uacf5\uc6a9\uc5b4 \uc911 \ud558\ub098\uc774\ub2e4.", "paragraph_id": "6239076-0-1", "paragraph_question": "question: \uc911\ub3d9 \ubd81\uc544\ud504\ub9ac\uce74\ub4f1 \uc544\ub78d\uad8c\uc5d0\uc11c \uc0ac\uc6a9\ud558\ub294 \uacf5\uc2dd\uc5b8\uc5b4\ub294?, context: \ud604\ub300 \ud45c\uc900 \uc544\ub78d\uc5b4\ub294 \uace0\uc804 \uc544\ub78d\uc5b4\ub97c \ubc14\ud0d5\uc73c\ub85c 19\uc138\uae30 \ucd08\ubc18\uc5d0 \ud615\uc131\ub41c \uc544\ub78d\uc5b4\ub85c, \uc911\ub3d9, \ubd81\uc544\ud504\ub9ac\uce74, \uc18c\ub9d0\ub9ac\uc544 \ubc18\ub3c4 \uc9c0\uc5ed\uc758 \ud45c\uc900 \ud45c\uae30 \uc218\ub2e8\uc73c\ub85c, \uc601\uc5b4, \ud504\ub791\uc2a4\uc5b4, \uc2a4\ud398\uc778\uc5b4, \ub7ec\uc2dc\uc544\uc5b4, \uc911\uad6d\uc5b4\uc640 \ub354\ubd88\uc5b4 \uc720\uc5d4\uc758 6\uac1c \uacf5\uc6a9\uc5b4 \uc911 \ud558\ub098\uc774\ub2e4. \uc544\ub78d \uc9c0\uc5ed\uc5d0\uc11c \ubc1c\ud589\ub418\ub294 \ubb38\uac74 - \ub300\ub2e4\uc218\uc758 \ucc45, \uc2e0\ubb38, \uc7a1\uc9c0, \uacf5\ubb38\uc11c, \uc5b4\ub9b0\uc774\ub97c \uc704\ud55c \uae30\ucd08 \uad50\uc7ac \ubc0f \uc77d\uc744\uac70\ub9ac - \uc740 \uc774 \uc544\ub78d\uc5b4\ub85c \uc791\uc131\ub41c\ub2e4. '\uc544\ub78d\uc5b4'\ub97c \uc0ac\uc6a9\ud558\ub294 \uc9c0\uc5ed\uc758 \uc720\uc77c\ud55c \uacf5\uc2dd \uc5b8\uc5b4\uc778 \ud45c\uc900 \uc544\ub78d\uc5b4\ub294 \ud559\uad50 \uad50\uacfc \uacfc\uc815\uc5d0\uc11c \uac00\ub974\uccd0\uc9c0\ub294 \uc720\uc77c\ud55c \uc544\ub78d\uc5b4\uc774\uae30\ub3c4 \ud558\ub2e4. \ud604\ub300 \ud45c\uc900 \uc544\ub78d\uc5b4\ub294 \uace0\uc804 \uc544\ub78d\uc5b4(\ud2b9\ud788 \uc774\uc2ac\ub78c \uc774\uc804\uc758 \uc2dc\ub300\ubd80\ud130, \uafb8\ub780\uc744 \ube44\ub86f\ud55c \uc555\ubc14\uc2a4 \uc2dc\ub300\uc758 \uc5b8\uc5b4)\ub97c \uadfc\uac04\uc73c\ub85c \ud558\uace0 \uc788\ub2e4. \uafb8\ub780 \ubc0f \uace0\ub300 \uc6b0\ub9c8\ubbf8\uc57c \ubc0f \uc555\ubc14\uc2a4 \uc655\uc870 \uc2dc\ub300\uc5d0 \uc791\uc131\ub41c \ubb38\uac74\ub4e4\uc758 \ud604\ub300\uc5b4 \ud310\uc744 \uc791\uc131\ud558\ub294\ub370 \uc0ac\uc6a9\ub418\uae30\ub3c4 \ud55c\ub2e4."} {"question": "\ubcf4\ub85c\uc6b4\ub3d9\uc774 \uac00\uc7a5 \uaca9\ub82c\ud558\uac8c \uc77c\uc5b4\ub09c \uacf3\uc740 \uc5b4\ub514\uc778\uac00?", "paragraph": "\uccad\uc870\uc758 \uc774\ub7ec\ud55c \uc870\ucc98\ub294 \uacb0\uc815 \ub2f9\uc0ac\uc790\ub97c \uc81c\uc678\ud55c \ubaa8\ub4e0 \uc0ac\ub78c\uc744 \uaca9\ubd84\uc2dc\ucf30\ub2e4. \uc6b0\uc120 \ub0b4\uac01\uc774 \uc758\ud68c\uc5d0 \uc544\ubb34\ub7f0 \uc0c1\uc758 \uc5c6\uc774 \uc77c\ubc29\uc801\uc73c\ub85c \uacb0\uc815\ud558\uace0 \ud1b5\ubcf4\ud55c \uc0ac\ud56d\uc774\uc5c8\uace0 \uc815\ubd80\uac00 \uc57d\uc18d\ud55c \ubbfc\uc8fc\uc801\uc778 \uc808\ucc28\ub97c \uc644\uc804\ud788 \ubb34\uc2dc\ud588\uace0 \ub9cc\uc8fc\uc871\uc774 \ub3c5\uc810\ud55c \ub0b4\uac01\uc5d0\uc11c \uc678\uad6d \uc790\ubcf8\uc744 \ub04c\uc5b4\ub4e4\uc774\ub824 \ud588\ub2e4\ub294 \uac83\uc740 \ub2f9\uc2dc \uc911\uad6d \ubbfc\uc871\uc8fc\uc758\uc758 \uc815\uc11c\uc5d0 \ud06c\uac8c \uac70\uc2a4\ub974\ub294 \uacb0\uc815\uc774\uc5c8\uc73c\uba70 \uc774\ubbf8 \ud22c\uc790\ud55c \ubbfc\uac04\uc778\ub4e4\uc758 \uc774\uad8c\uc744 \ubc15\ud0c8\ud558\ub294 \uc870\ucc98\uc600\ub2e4. \uadf8\ub9ac\ud558\uc5ec \ud53c\ud574\ub97c \uc785\uc740 \uc9c0\uc5ed \uac00\uc6b4\ub370 \ud638\ub0a8 \uc7a5\uc0ac(\u9577\u6c99)\uc5d0\uc11c \ucc98\uc74c\uc73c\ub85c \uad6d\uc720\ud654\ub839\uc744 \ucde8\uc18c\ud558\ub3c4\ub85d \uc694\uad6c\ud558\uc600\ub2e4. \ub610, \uc870\uc138\uc5d0 \ud56d\uac70\ud558\uc790\ub294 \uc0c1\uc778\uc758 \ucca0\uc2dc\uc640 \ub3d9\ub9f9\ud734\ud559\ub3c4 \uc77c\uc5b4\ub0ac\ub2e4. \uc774\ud6c4 6\uc6d4 17\uc77c \u2018\ubcf4\ub85c \ub3d9\uc9c0\ud68c\u2019\uac00 \uc870\uc9c1\ub418\uc5b4 \ud638\ubd81\uacfc \uc591\uad11\uc5d0\uc11c\ub3c4 \ubcf4\ub85c\uc6b4\ub3d9\uc774 \uc77c\uc5b4\ub0ac\ub294\ub370, \uadf8 \uac00\uc6b4\ub370 \uc81c\uc77c \uaca9\ub82c\ud558\uac8c \uc77c\uc5b4\ub09c \uacf3\uc774 \uc4f0\ucd28 \uc131\uc774\uc5c8\ub2e4.", "answer": "\uc4f0\ucd28 \uc131", "sentence": "\uadf8 \uac00\uc6b4\ub370 \uc81c\uc77c \uaca9\ub82c\ud558\uac8c \uc77c\uc5b4\ub09c \uacf3\uc774 \uc4f0\ucd28 \uc131 \uc774\uc5c8\ub2e4.", "paragraph_sentence": "\uccad\uc870\uc758 \uc774\ub7ec\ud55c \uc870\ucc98\ub294 \uacb0\uc815 \ub2f9\uc0ac\uc790\ub97c \uc81c\uc678\ud55c \ubaa8\ub4e0 \uc0ac\ub78c\uc744 \uaca9\ubd84\uc2dc\ucf30\ub2e4. \uc6b0\uc120 \ub0b4\uac01\uc774 \uc758\ud68c\uc5d0 \uc544\ubb34\ub7f0 \uc0c1\uc758 \uc5c6\uc774 \uc77c\ubc29\uc801\uc73c\ub85c \uacb0\uc815\ud558\uace0 \ud1b5\ubcf4\ud55c \uc0ac\ud56d\uc774\uc5c8\uace0 \uc815\ubd80\uac00 \uc57d\uc18d\ud55c \ubbfc\uc8fc\uc801\uc778 \uc808\ucc28\ub97c \uc644\uc804\ud788 \ubb34\uc2dc\ud588\uace0 \ub9cc\uc8fc\uc871\uc774 \ub3c5\uc810\ud55c \ub0b4\uac01\uc5d0\uc11c \uc678\uad6d \uc790\ubcf8\uc744 \ub04c\uc5b4\ub4e4\uc774\ub824 \ud588\ub2e4\ub294 \uac83\uc740 \ub2f9\uc2dc \uc911\uad6d \ubbfc\uc871\uc8fc\uc758\uc758 \uc815\uc11c\uc5d0 \ud06c\uac8c \uac70\uc2a4\ub974\ub294 \uacb0\uc815\uc774\uc5c8\uc73c\uba70 \uc774\ubbf8 \ud22c\uc790\ud55c \ubbfc\uac04\uc778\ub4e4\uc758 \uc774\uad8c\uc744 \ubc15\ud0c8\ud558\ub294 \uc870\ucc98\uc600\ub2e4. \uadf8\ub9ac\ud558\uc5ec \ud53c\ud574\ub97c \uc785\uc740 \uc9c0\uc5ed \uac00\uc6b4\ub370 \ud638\ub0a8 \uc7a5\uc0ac(\u9577\u6c99)\uc5d0\uc11c \ucc98\uc74c\uc73c\ub85c \uad6d\uc720\ud654\ub839\uc744 \ucde8\uc18c\ud558\ub3c4\ub85d \uc694\uad6c\ud558\uc600\ub2e4. \ub610, \uc870\uc138\uc5d0 \ud56d\uac70\ud558\uc790\ub294 \uc0c1\uc778\uc758 \ucca0\uc2dc\uc640 \ub3d9\ub9f9\ud734\ud559\ub3c4 \uc77c\uc5b4\ub0ac\ub2e4. \uc774\ud6c4 6\uc6d4 17\uc77c \u2018\ubcf4\ub85c \ub3d9\uc9c0\ud68c\u2019\uac00 \uc870\uc9c1\ub418\uc5b4 \ud638\ubd81\uacfc \uc591\uad11\uc5d0\uc11c\ub3c4 \ubcf4\ub85c\uc6b4\ub3d9\uc774 \uc77c\uc5b4\ub0ac\ub294\ub370, \uadf8 \uac00\uc6b4\ub370 \uc81c\uc77c \uaca9\ub82c\ud558\uac8c \uc77c\uc5b4\ub09c \uacf3\uc774 \uc4f0\ucd28 \uc131 \uc774\uc5c8\ub2e4. ", "paragraph_answer": "\uccad\uc870\uc758 \uc774\ub7ec\ud55c \uc870\ucc98\ub294 \uacb0\uc815 \ub2f9\uc0ac\uc790\ub97c \uc81c\uc678\ud55c \ubaa8\ub4e0 \uc0ac\ub78c\uc744 \uaca9\ubd84\uc2dc\ucf30\ub2e4. \uc6b0\uc120 \ub0b4\uac01\uc774 \uc758\ud68c\uc5d0 \uc544\ubb34\ub7f0 \uc0c1\uc758 \uc5c6\uc774 \uc77c\ubc29\uc801\uc73c\ub85c \uacb0\uc815\ud558\uace0 \ud1b5\ubcf4\ud55c \uc0ac\ud56d\uc774\uc5c8\uace0 \uc815\ubd80\uac00 \uc57d\uc18d\ud55c \ubbfc\uc8fc\uc801\uc778 \uc808\ucc28\ub97c \uc644\uc804\ud788 \ubb34\uc2dc\ud588\uace0 \ub9cc\uc8fc\uc871\uc774 \ub3c5\uc810\ud55c \ub0b4\uac01\uc5d0\uc11c \uc678\uad6d \uc790\ubcf8\uc744 \ub04c\uc5b4\ub4e4\uc774\ub824 \ud588\ub2e4\ub294 \uac83\uc740 \ub2f9\uc2dc \uc911\uad6d \ubbfc\uc871\uc8fc\uc758\uc758 \uc815\uc11c\uc5d0 \ud06c\uac8c \uac70\uc2a4\ub974\ub294 \uacb0\uc815\uc774\uc5c8\uc73c\uba70 \uc774\ubbf8 \ud22c\uc790\ud55c \ubbfc\uac04\uc778\ub4e4\uc758 \uc774\uad8c\uc744 \ubc15\ud0c8\ud558\ub294 \uc870\ucc98\uc600\ub2e4. \uadf8\ub9ac\ud558\uc5ec \ud53c\ud574\ub97c \uc785\uc740 \uc9c0\uc5ed \uac00\uc6b4\ub370 \ud638\ub0a8 \uc7a5\uc0ac(\u9577\u6c99)\uc5d0\uc11c \ucc98\uc74c\uc73c\ub85c \uad6d\uc720\ud654\ub839\uc744 \ucde8\uc18c\ud558\ub3c4\ub85d \uc694\uad6c\ud558\uc600\ub2e4. \ub610, \uc870\uc138\uc5d0 \ud56d\uac70\ud558\uc790\ub294 \uc0c1\uc778\uc758 \ucca0\uc2dc\uc640 \ub3d9\ub9f9\ud734\ud559\ub3c4 \uc77c\uc5b4\ub0ac\ub2e4. \uc774\ud6c4 6\uc6d4 17\uc77c \u2018\ubcf4\ub85c \ub3d9\uc9c0\ud68c\u2019\uac00 \uc870\uc9c1\ub418\uc5b4 \ud638\ubd81\uacfc \uc591\uad11\uc5d0\uc11c\ub3c4 \ubcf4\ub85c\uc6b4\ub3d9\uc774 \uc77c\uc5b4\ub0ac\ub294\ub370, \uadf8 \uac00\uc6b4\ub370 \uc81c\uc77c \uaca9\ub82c\ud558\uac8c \uc77c\uc5b4\ub09c \uacf3\uc774 \uc4f0\ucd28 \uc131 \uc774\uc5c8\ub2e4.", "sentence_answer": "\uadf8 \uac00\uc6b4\ub370 \uc81c\uc77c \uaca9\ub82c\ud558\uac8c \uc77c\uc5b4\ub09c \uacf3\uc774 \uc4f0\ucd28 \uc131 \uc774\uc5c8\ub2e4.", "paragraph_id": "6457494-17-1", "paragraph_question": "question: \ubcf4\ub85c\uc6b4\ub3d9\uc774 \uac00\uc7a5 \uaca9\ub82c\ud558\uac8c \uc77c\uc5b4\ub09c \uacf3\uc740 \uc5b4\ub514\uc778\uac00?, context: \uccad\uc870\uc758 \uc774\ub7ec\ud55c \uc870\ucc98\ub294 \uacb0\uc815 \ub2f9\uc0ac\uc790\ub97c \uc81c\uc678\ud55c \ubaa8\ub4e0 \uc0ac\ub78c\uc744 \uaca9\ubd84\uc2dc\ucf30\ub2e4. \uc6b0\uc120 \ub0b4\uac01\uc774 \uc758\ud68c\uc5d0 \uc544\ubb34\ub7f0 \uc0c1\uc758 \uc5c6\uc774 \uc77c\ubc29\uc801\uc73c\ub85c \uacb0\uc815\ud558\uace0 \ud1b5\ubcf4\ud55c \uc0ac\ud56d\uc774\uc5c8\uace0 \uc815\ubd80\uac00 \uc57d\uc18d\ud55c \ubbfc\uc8fc\uc801\uc778 \uc808\ucc28\ub97c \uc644\uc804\ud788 \ubb34\uc2dc\ud588\uace0 \ub9cc\uc8fc\uc871\uc774 \ub3c5\uc810\ud55c \ub0b4\uac01\uc5d0\uc11c \uc678\uad6d \uc790\ubcf8\uc744 \ub04c\uc5b4\ub4e4\uc774\ub824 \ud588\ub2e4\ub294 \uac83\uc740 \ub2f9\uc2dc \uc911\uad6d \ubbfc\uc871\uc8fc\uc758\uc758 \uc815\uc11c\uc5d0 \ud06c\uac8c \uac70\uc2a4\ub974\ub294 \uacb0\uc815\uc774\uc5c8\uc73c\uba70 \uc774\ubbf8 \ud22c\uc790\ud55c \ubbfc\uac04\uc778\ub4e4\uc758 \uc774\uad8c\uc744 \ubc15\ud0c8\ud558\ub294 \uc870\ucc98\uc600\ub2e4. \uadf8\ub9ac\ud558\uc5ec \ud53c\ud574\ub97c \uc785\uc740 \uc9c0\uc5ed \uac00\uc6b4\ub370 \ud638\ub0a8 \uc7a5\uc0ac(\u9577\u6c99)\uc5d0\uc11c \ucc98\uc74c\uc73c\ub85c \uad6d\uc720\ud654\ub839\uc744 \ucde8\uc18c\ud558\ub3c4\ub85d \uc694\uad6c\ud558\uc600\ub2e4. \ub610, \uc870\uc138\uc5d0 \ud56d\uac70\ud558\uc790\ub294 \uc0c1\uc778\uc758 \ucca0\uc2dc\uc640 \ub3d9\ub9f9\ud734\ud559\ub3c4 \uc77c\uc5b4\ub0ac\ub2e4. \uc774\ud6c4 6\uc6d4 17\uc77c \u2018\ubcf4\ub85c \ub3d9\uc9c0\ud68c\u2019\uac00 \uc870\uc9c1\ub418\uc5b4 \ud638\ubd81\uacfc \uc591\uad11\uc5d0\uc11c\ub3c4 \ubcf4\ub85c\uc6b4\ub3d9\uc774 \uc77c\uc5b4\ub0ac\ub294\ub370, \uadf8 \uac00\uc6b4\ub370 \uc81c\uc77c \uaca9\ub82c\ud558\uac8c \uc77c\uc5b4\ub09c \uacf3\uc774 \uc4f0\ucd28 \uc131\uc774\uc5c8\ub2e4."} {"question": "\uace0\ub300 \uadf8\ub9ac\uc2a4\uc2dc\ub300\uc5d0 \ud0c4\uc0dd\ub41c \uc5ed\uc124\uc758 \uc774\ub984\uc740?", "paragraph": "\uba54\ud0c0\uc5b8\uc5b4\ub294 \ub2e4\uc591\ud55c \uac70\uc9d3\ub9d0\uc7c1\uc774\uc758 \uc5ed\uc124\uc744 \ud574\uacb0\ud558\ub294 \uacfc\uc815\uc5d0\uc11c \ub3c4\uc785\ub41c \uac1c\ub150\uc774\ub2e4. \uace0\ub300 \uadf8\ub9ac\uc2a4\uc2dc\ub300\uc5d0 \ud0c4\uc0dd\ub41c \uc5ed\uc124\uc778 \uc5d0\uc6b0\ubd88\ub9ac\ub370\uc2a4(\u0395\u1f51\u03b2\u03bf\u03c5\u03bb\u03af\u03b4\u03b7\u03c2)\uc758 \uc5ed\uc124\ubd80\ud130 \ub300\ud45c\uc801\uc778 \uc9d1\ud569\ub860\uc801 \uc5ed\uc124\uc758 \ud558\ub098\uc778 \ub7ec\uc140\uc758 \uc5ed\uc124\uacfc \uc758\ubbf8\ub860\uc801 \uc5ed\uc124\uc758 \ud558\ub098\uc778 \uc774\ubc1c\uc0ac\uc758 \uc5ed\uc124 \ub4f1\uae4c\uc9c0 \uc624\ub7ab\ub3d9\uc548 \uc815\ud655\ud788 \ucc38 \ub610\ub294 \uac70\uc9d3\uc73c\ub85c \uc99d\uba85\ud560 \uc218 \uc5c6\uc5b4 \ud574\uacb0 \ubd88\uac00\ub2a5\ud55c \ub09c\uc81c\ub85c \uc5ec\uaca8\uc838 \uc654\ub358 \ub2e4\uc591\ud55c \uac70\uc9d3\ub9d0\uc7c1\uc774\uc758 \uc5ed\uc124\ub4e4\uc740 20\uc138\uae30 \ud3f4\ub780\ub4dc\uc758 \ub17c\ub9ac\ud559\uc790\uc774\uc790 \uc218\ud559\uc790\uc778 \uc54c\ud504\ub808\ud2b8 \ud0c0\ub974\uc2a4\ud0a4 \uc5d0 \uc758\ud574 \ud574\uacb0\ub418\uc5c8\ub2e4. \uadf8\ub294 \uc5b4\ub5a4 \uc0ac\uc2e4\uc5d0 \ub300\ud574 \ub9d0\ud558\ub294 \ub9d0\uacfc \uadf8\uac83\uc5d0 \ub300\ud574 \ub2e4\uc2dc \ub9d0\ud558\ub294 \ub9d0\uc774 \uc11c\ub85c \uacc4\uce35\uc744 \ub2ec\ub9ac\ud55c\ub2e4\ub294 \uc0ac\uc2e4\uc744 \uae68\ub2eb\uc9c0 \ubabb\ud574 \ub450 \uc5b8\uc5b4\ub97c \ubd84\ub9ac\ud558\uc5ec \uc778\uc2dd\ud558\uc9c0 \ubabb\ud558\ub294 \uac83\uc73c\ub85c\ubd80\ud130 \uac70\uc9d3\ub9d0\uc7c1\uc774\uc758 \uc5ed\uc124\uacfc \uac19\uc740 \ub17c\ub9ac\uc801 \ubaa8\uc21c\uc774 \ubc1c\uc0dd\ud55c\ub2e4\uace0 \uc124\uba85\ud558\uc600\ub2e4. \uc774 \uacfc\uc815\uc5d0\uc11c \ud0c0\ub974\uc2a4\ud0a4\ub294 \uc804\uc790\uc758 \ub9d0\uc744 \ud6c4\uc790\uc758 \ub9d0\uc758 \ubaa9\ud45c\uac00 \ub418\ub294 \ub300\uc0c1\uc5b8\uc5b4\ub85c, \ud6c4\uc790\uc758 \ub9d0\uc740 \uc804\uc790\uc758 \ub9d0\uc744 \ub2e4\uc2dc \ud55c \ubc88 \uc11c\uc220\ud558\ub294 \uba54\ud0c0\uc5b8\uc5b4\ub85c \uaddc\uc815\ud558\uc600\ub2e4.", "answer": "\uc5d0\uc6b0\ubd88\ub9ac\ub370\uc2a4", "sentence": "\uace0\ub300 \uadf8\ub9ac\uc2a4\uc2dc\ub300\uc5d0 \ud0c4\uc0dd\ub41c \uc5ed\uc124\uc778 \uc5d0\uc6b0\ubd88\ub9ac\ub370\uc2a4 (\u0395\u1f51\u03b2\u03bf\u03c5\u03bb\u03af\u03b4\u03b7\u03c2)", "paragraph_sentence": "\uba54\ud0c0\uc5b8\uc5b4\ub294 \ub2e4\uc591\ud55c \uac70\uc9d3\ub9d0\uc7c1\uc774\uc758 \uc5ed\uc124\uc744 \ud574\uacb0\ud558\ub294 \uacfc\uc815\uc5d0\uc11c \ub3c4\uc785\ub41c \uac1c\ub150\uc774\ub2e4. \uace0\ub300 \uadf8\ub9ac\uc2a4\uc2dc\ub300\uc5d0 \ud0c4\uc0dd\ub41c \uc5ed\uc124\uc778 \uc5d0\uc6b0\ubd88\ub9ac\ub370\uc2a4 (\u0395\u1f51\u03b2\u03bf\u03c5\u03bb\u03af\u03b4\u03b7\u03c2) \uc758 \uc5ed\uc124\ubd80\ud130 \ub300\ud45c\uc801\uc778 \uc9d1\ud569\ub860\uc801 \uc5ed\uc124\uc758 \ud558\ub098\uc778 \ub7ec\uc140\uc758 \uc5ed\uc124\uacfc \uc758\ubbf8\ub860\uc801 \uc5ed\uc124\uc758 \ud558\ub098\uc778 \uc774\ubc1c\uc0ac\uc758 \uc5ed\uc124 \ub4f1\uae4c\uc9c0 \uc624\ub7ab\ub3d9\uc548 \uc815\ud655\ud788 \ucc38 \ub610\ub294 \uac70\uc9d3\uc73c\ub85c \uc99d\uba85\ud560 \uc218 \uc5c6\uc5b4 \ud574\uacb0 \ubd88\uac00\ub2a5\ud55c \ub09c\uc81c\ub85c \uc5ec\uaca8\uc838 \uc654\ub358 \ub2e4\uc591\ud55c \uac70\uc9d3\ub9d0\uc7c1\uc774\uc758 \uc5ed\uc124\ub4e4\uc740 20\uc138\uae30 \ud3f4\ub780\ub4dc\uc758 \ub17c\ub9ac\ud559\uc790\uc774\uc790 \uc218\ud559\uc790\uc778 \uc54c\ud504\ub808\ud2b8 \ud0c0\ub974\uc2a4\ud0a4 \uc5d0 \uc758\ud574 \ud574\uacb0\ub418\uc5c8\ub2e4. \uadf8\ub294 \uc5b4\ub5a4 \uc0ac\uc2e4\uc5d0 \ub300\ud574 \ub9d0\ud558\ub294 \ub9d0\uacfc \uadf8\uac83\uc5d0 \ub300\ud574 \ub2e4\uc2dc \ub9d0\ud558\ub294 \ub9d0\uc774 \uc11c\ub85c \uacc4\uce35\uc744 \ub2ec\ub9ac\ud55c\ub2e4\ub294 \uc0ac\uc2e4\uc744 \uae68\ub2eb\uc9c0 \ubabb\ud574 \ub450 \uc5b8\uc5b4\ub97c \ubd84\ub9ac\ud558\uc5ec \uc778\uc2dd\ud558\uc9c0 \ubabb\ud558\ub294 \uac83\uc73c\ub85c\ubd80\ud130 \uac70\uc9d3\ub9d0\uc7c1\uc774\uc758 \uc5ed\uc124\uacfc \uac19\uc740 \ub17c\ub9ac\uc801 \ubaa8\uc21c\uc774 \ubc1c\uc0dd\ud55c\ub2e4\uace0 \uc124\uba85\ud558\uc600\ub2e4. \uc774 \uacfc\uc815\uc5d0\uc11c \ud0c0\ub974\uc2a4\ud0a4\ub294 \uc804\uc790\uc758 \ub9d0\uc744 \ud6c4\uc790\uc758 \ub9d0\uc758 \ubaa9\ud45c\uac00 \ub418\ub294 \ub300\uc0c1\uc5b8\uc5b4\ub85c, \ud6c4\uc790\uc758 \ub9d0\uc740 \uc804\uc790\uc758 \ub9d0\uc744 \ub2e4\uc2dc \ud55c \ubc88 \uc11c\uc220\ud558\ub294 \uba54\ud0c0\uc5b8\uc5b4\ub85c \uaddc\uc815\ud558\uc600\ub2e4.", "paragraph_answer": "\uba54\ud0c0\uc5b8\uc5b4\ub294 \ub2e4\uc591\ud55c \uac70\uc9d3\ub9d0\uc7c1\uc774\uc758 \uc5ed\uc124\uc744 \ud574\uacb0\ud558\ub294 \uacfc\uc815\uc5d0\uc11c \ub3c4\uc785\ub41c \uac1c\ub150\uc774\ub2e4. \uace0\ub300 \uadf8\ub9ac\uc2a4\uc2dc\ub300\uc5d0 \ud0c4\uc0dd\ub41c \uc5ed\uc124\uc778 \uc5d0\uc6b0\ubd88\ub9ac\ub370\uc2a4 (\u0395\u1f51\u03b2\u03bf\u03c5\u03bb\u03af\u03b4\u03b7\u03c2)\uc758 \uc5ed\uc124\ubd80\ud130 \ub300\ud45c\uc801\uc778 \uc9d1\ud569\ub860\uc801 \uc5ed\uc124\uc758 \ud558\ub098\uc778 \ub7ec\uc140\uc758 \uc5ed\uc124\uacfc \uc758\ubbf8\ub860\uc801 \uc5ed\uc124\uc758 \ud558\ub098\uc778 \uc774\ubc1c\uc0ac\uc758 \uc5ed\uc124 \ub4f1\uae4c\uc9c0 \uc624\ub7ab\ub3d9\uc548 \uc815\ud655\ud788 \ucc38 \ub610\ub294 \uac70\uc9d3\uc73c\ub85c \uc99d\uba85\ud560 \uc218 \uc5c6\uc5b4 \ud574\uacb0 \ubd88\uac00\ub2a5\ud55c \ub09c\uc81c\ub85c \uc5ec\uaca8\uc838 \uc654\ub358 \ub2e4\uc591\ud55c \uac70\uc9d3\ub9d0\uc7c1\uc774\uc758 \uc5ed\uc124\ub4e4\uc740 20\uc138\uae30 \ud3f4\ub780\ub4dc\uc758 \ub17c\ub9ac\ud559\uc790\uc774\uc790 \uc218\ud559\uc790\uc778 \uc54c\ud504\ub808\ud2b8 \ud0c0\ub974\uc2a4\ud0a4 \uc5d0 \uc758\ud574 \ud574\uacb0\ub418\uc5c8\ub2e4. \uadf8\ub294 \uc5b4\ub5a4 \uc0ac\uc2e4\uc5d0 \ub300\ud574 \ub9d0\ud558\ub294 \ub9d0\uacfc \uadf8\uac83\uc5d0 \ub300\ud574 \ub2e4\uc2dc \ub9d0\ud558\ub294 \ub9d0\uc774 \uc11c\ub85c \uacc4\uce35\uc744 \ub2ec\ub9ac\ud55c\ub2e4\ub294 \uc0ac\uc2e4\uc744 \uae68\ub2eb\uc9c0 \ubabb\ud574 \ub450 \uc5b8\uc5b4\ub97c \ubd84\ub9ac\ud558\uc5ec \uc778\uc2dd\ud558\uc9c0 \ubabb\ud558\ub294 \uac83\uc73c\ub85c\ubd80\ud130 \uac70\uc9d3\ub9d0\uc7c1\uc774\uc758 \uc5ed\uc124\uacfc \uac19\uc740 \ub17c\ub9ac\uc801 \ubaa8\uc21c\uc774 \ubc1c\uc0dd\ud55c\ub2e4\uace0 \uc124\uba85\ud558\uc600\ub2e4. \uc774 \uacfc\uc815\uc5d0\uc11c \ud0c0\ub974\uc2a4\ud0a4\ub294 \uc804\uc790\uc758 \ub9d0\uc744 \ud6c4\uc790\uc758 \ub9d0\uc758 \ubaa9\ud45c\uac00 \ub418\ub294 \ub300\uc0c1\uc5b8\uc5b4\ub85c, \ud6c4\uc790\uc758 \ub9d0\uc740 \uc804\uc790\uc758 \ub9d0\uc744 \ub2e4\uc2dc \ud55c \ubc88 \uc11c\uc220\ud558\ub294 \uba54\ud0c0\uc5b8\uc5b4\ub85c \uaddc\uc815\ud558\uc600\ub2e4.", "sentence_answer": "\uace0\ub300 \uadf8\ub9ac\uc2a4\uc2dc\ub300\uc5d0 \ud0c4\uc0dd\ub41c \uc5ed\uc124\uc778 \uc5d0\uc6b0\ubd88\ub9ac\ub370\uc2a4 (\u0395\u1f51\u03b2\u03bf\u03c5\u03bb\u03af\u03b4\u03b7\u03c2)", "paragraph_id": "6502912-1-0", "paragraph_question": "question: \uace0\ub300 \uadf8\ub9ac\uc2a4\uc2dc\ub300\uc5d0 \ud0c4\uc0dd\ub41c \uc5ed\uc124\uc758 \uc774\ub984\uc740?, context: \uba54\ud0c0\uc5b8\uc5b4\ub294 \ub2e4\uc591\ud55c \uac70\uc9d3\ub9d0\uc7c1\uc774\uc758 \uc5ed\uc124\uc744 \ud574\uacb0\ud558\ub294 \uacfc\uc815\uc5d0\uc11c \ub3c4\uc785\ub41c \uac1c\ub150\uc774\ub2e4. \uace0\ub300 \uadf8\ub9ac\uc2a4\uc2dc\ub300\uc5d0 \ud0c4\uc0dd\ub41c \uc5ed\uc124\uc778 \uc5d0\uc6b0\ubd88\ub9ac\ub370\uc2a4(\u0395\u1f51\u03b2\u03bf\u03c5\u03bb\u03af\u03b4\u03b7\u03c2)\uc758 \uc5ed\uc124\ubd80\ud130 \ub300\ud45c\uc801\uc778 \uc9d1\ud569\ub860\uc801 \uc5ed\uc124\uc758 \ud558\ub098\uc778 \ub7ec\uc140\uc758 \uc5ed\uc124\uacfc \uc758\ubbf8\ub860\uc801 \uc5ed\uc124\uc758 \ud558\ub098\uc778 \uc774\ubc1c\uc0ac\uc758 \uc5ed\uc124 \ub4f1\uae4c\uc9c0 \uc624\ub7ab\ub3d9\uc548 \uc815\ud655\ud788 \ucc38 \ub610\ub294 \uac70\uc9d3\uc73c\ub85c \uc99d\uba85\ud560 \uc218 \uc5c6\uc5b4 \ud574\uacb0 \ubd88\uac00\ub2a5\ud55c \ub09c\uc81c\ub85c \uc5ec\uaca8\uc838 \uc654\ub358 \ub2e4\uc591\ud55c \uac70\uc9d3\ub9d0\uc7c1\uc774\uc758 \uc5ed\uc124\ub4e4\uc740 20\uc138\uae30 \ud3f4\ub780\ub4dc\uc758 \ub17c\ub9ac\ud559\uc790\uc774\uc790 \uc218\ud559\uc790\uc778 \uc54c\ud504\ub808\ud2b8 \ud0c0\ub974\uc2a4\ud0a4 \uc5d0 \uc758\ud574 \ud574\uacb0\ub418\uc5c8\ub2e4. \uadf8\ub294 \uc5b4\ub5a4 \uc0ac\uc2e4\uc5d0 \ub300\ud574 \ub9d0\ud558\ub294 \ub9d0\uacfc \uadf8\uac83\uc5d0 \ub300\ud574 \ub2e4\uc2dc \ub9d0\ud558\ub294 \ub9d0\uc774 \uc11c\ub85c \uacc4\uce35\uc744 \ub2ec\ub9ac\ud55c\ub2e4\ub294 \uc0ac\uc2e4\uc744 \uae68\ub2eb\uc9c0 \ubabb\ud574 \ub450 \uc5b8\uc5b4\ub97c \ubd84\ub9ac\ud558\uc5ec \uc778\uc2dd\ud558\uc9c0 \ubabb\ud558\ub294 \uac83\uc73c\ub85c\ubd80\ud130 \uac70\uc9d3\ub9d0\uc7c1\uc774\uc758 \uc5ed\uc124\uacfc \uac19\uc740 \ub17c\ub9ac\uc801 \ubaa8\uc21c\uc774 \ubc1c\uc0dd\ud55c\ub2e4\uace0 \uc124\uba85\ud558\uc600\ub2e4. \uc774 \uacfc\uc815\uc5d0\uc11c \ud0c0\ub974\uc2a4\ud0a4\ub294 \uc804\uc790\uc758 \ub9d0\uc744 \ud6c4\uc790\uc758 \ub9d0\uc758 \ubaa9\ud45c\uac00 \ub418\ub294 \ub300\uc0c1\uc5b8\uc5b4\ub85c, \ud6c4\uc790\uc758 \ub9d0\uc740 \uc804\uc790\uc758 \ub9d0\uc744 \ub2e4\uc2dc \ud55c \ubc88 \uc11c\uc220\ud558\ub294 \uba54\ud0c0\uc5b8\uc5b4\ub85c \uaddc\uc815\ud558\uc600\ub2e4."} {"question": "Chelmno \uc218\uc6a9\uc18c\ub294 \uc5b8\uc81c \ud574\ubc29\ub418\uc5c8\ub294\uac00?", "paragraph": "\uccab \ubc88\uc9f8 \uc8fc\uc694 \uc218\uc6a9\uc18c\uc600\ub358 Majdanek \uc218\uc6a9\uc18c\ub294 \uc9c4\uaca9\ud558\ub294 \uc18c\ub828\uad70\uc5d0 \uc758\ud574 1944\ub144 7\uc6d4 23\uc77c\uc5d0 \ubc1c\uacac\ub418\uc5c8\ub2e4. Chelmno \uc218\uc6a9\uc18c\ub294 \uc18c\ub828\uad70\uc5d0 \uc758\ud574 1945\ub144 1\uc6d4 20\uc77c\uc5d0 \ud574\ubc29\ub418\uc5c8\uace0, \uc544\uc6b0\uc288\ube44\uce20 \uc218\uc6a9\uc18c \ub610\ud55c \uc18c\ub828\uad70\uc5d0 \uc758\ud574 1945\ub144 1\uc6d4 27\uc77c\uc5d0 \ud574\ubc29\ub418\uc5c8\ub2e4. Buchenwald \uc218\uc6a9\uc18c\ub294 \ubbf8\uad70\uc5d0 \uc758\ud574 4\uc6d4 11\uc77c, Bergen-Belsen \uc218\uc6a9\uc18c\ub294 \uc601\uad6d\uad70\uc5d0 \uc758\ud574 4\uc6d4 15\uc77c, Dachau \uc218\uc6a9\uc18c\ub294 \ubbf8\uad70\uc5d0 \uc758\ud574 4\uc6d4 29\uc77c, Ravensbr\u00fcck \uc218\uc6a9\uc18c\ub294 \uac19\uc740 \ub0a0 \uc18c\ub828\uad70\uc5d0 \uc758\ud574, Mauthausen \uc218\uc6a9\uc18c\ub294 \ubbf8\uad70\uc5d0 \uc758\ud574 5\uc6d4 5\uc77c, Theresienstadt \uc218\uc6a9\uc18c\ub294 \uc18c\ub828\uad70\uc5d0 \uc758\ud574 5\uc6d4 8\uc77c\uc5d0 \ud574\ubc29\ub418\uc5c8\ub2e4. Treblinka, Sobibor, Belzec \uc218\uc6a9\uc18c\ub294 \ud574\ubc29\ub418\uc9c0 \ubabb\ud558\uace0 1943\ub144\uc5d0 \ub098\uce58\uc5d0 \uc758\ud574 \ud30c\uad34\ub418\uc5c8\ub2e4. \ubbf87\uad70 William W. Quinn \ub300\ub839\uc740 Dachau \uc218\uc6a9\uc18c\uc5d0 \ub300\ud574 \u201c\uc6b0\ub9ac \ubd80\ub300\ub294 \uadf8 \uacf3\uc5d0\uc11c \ubbff\uae30 \ud798\ub4e4 \uc815\ub3c4\ub85c \ub054\ucc0d\ud55c \uad11\uacbd, \uc18c\ub9ac, \uc545\ucde8\ub97c \ubaa9\uaca9\ud558\uc600\ub2e4. \ub3c4\uc800\ud788 \uc77c\ubc18\uc801\uc778 \uc0c1\uc2dd\uc73c\ub85c\ub294 \uc774\ud574\ud560 \uc218 \uc5c6\ub294 \uc5c4\uccad\ub09c \uc794\uc778\ud568\uc774\uc5c8\ub2e4\u201d\ub77c\uace0 \ud68c\uace0\ud55c\ub2e4.", "answer": "1945\ub144 1\uc6d4 20\uc77c", "sentence": "Chelmno \uc218\uc6a9\uc18c\ub294 \uc18c\ub828\uad70\uc5d0 \uc758\ud574 1945\ub144 1\uc6d4 20\uc77c \uc5d0", "paragraph_sentence": "\uccab \ubc88\uc9f8 \uc8fc\uc694 \uc218\uc6a9\uc18c\uc600\ub358 Majdanek \uc218\uc6a9\uc18c\ub294 \uc9c4\uaca9\ud558\ub294 \uc18c\ub828\uad70\uc5d0 \uc758\ud574 1944\ub144 7\uc6d4 23\uc77c\uc5d0 \ubc1c\uacac\ub418\uc5c8\ub2e4. Chelmno \uc218\uc6a9\uc18c\ub294 \uc18c\ub828\uad70\uc5d0 \uc758\ud574 1945\ub144 1\uc6d4 20\uc77c \uc5d0 \ud574\ubc29\ub418\uc5c8\uace0, \uc544\uc6b0\uc288\ube44\uce20 \uc218\uc6a9\uc18c \ub610\ud55c \uc18c\ub828\uad70\uc5d0 \uc758\ud574 1945\ub144 1\uc6d4 27\uc77c\uc5d0 \ud574\ubc29\ub418\uc5c8\ub2e4. Buchenwald \uc218\uc6a9\uc18c\ub294 \ubbf8\uad70\uc5d0 \uc758\ud574 4\uc6d4 11\uc77c, Bergen-Belsen \uc218\uc6a9\uc18c\ub294 \uc601\uad6d\uad70\uc5d0 \uc758\ud574 4\uc6d4 15\uc77c, Dachau \uc218\uc6a9\uc18c\ub294 \ubbf8\uad70\uc5d0 \uc758\ud574 4\uc6d4 29\uc77c, Ravensbr\u00fcck \uc218\uc6a9\uc18c\ub294 \uac19\uc740 \ub0a0 \uc18c\ub828\uad70\uc5d0 \uc758\ud574, Mauthausen \uc218\uc6a9\uc18c\ub294 \ubbf8\uad70\uc5d0 \uc758\ud574 5\uc6d4 5\uc77c, Theresienstadt \uc218\uc6a9\uc18c\ub294 \uc18c\ub828\uad70\uc5d0 \uc758\ud574 5\uc6d4 8\uc77c\uc5d0 \ud574\ubc29\ub418\uc5c8\ub2e4. Treblinka, Sobibor, Belzec \uc218\uc6a9\uc18c\ub294 \ud574\ubc29\ub418\uc9c0 \ubabb\ud558\uace0 1943\ub144\uc5d0 \ub098\uce58\uc5d0 \uc758\ud574 \ud30c\uad34\ub418\uc5c8\ub2e4. \ubbf87\uad70 William W. Quinn \ub300\ub839\uc740 Dachau \uc218\uc6a9\uc18c\uc5d0 \ub300\ud574 \u201c\uc6b0\ub9ac \ubd80\ub300\ub294 \uadf8 \uacf3\uc5d0\uc11c \ubbff\uae30 \ud798\ub4e4 \uc815\ub3c4\ub85c \ub054\ucc0d\ud55c \uad11\uacbd, \uc18c\ub9ac, \uc545\ucde8\ub97c \ubaa9\uaca9\ud558\uc600\ub2e4. \ub3c4\uc800\ud788 \uc77c\ubc18\uc801\uc778 \uc0c1\uc2dd\uc73c\ub85c\ub294 \uc774\ud574\ud560 \uc218 \uc5c6\ub294 \uc5c4\uccad\ub09c \uc794\uc778\ud568\uc774\uc5c8\ub2e4\u201d\ub77c\uace0 \ud68c\uace0\ud55c\ub2e4.", "paragraph_answer": "\uccab \ubc88\uc9f8 \uc8fc\uc694 \uc218\uc6a9\uc18c\uc600\ub358 Majdanek \uc218\uc6a9\uc18c\ub294 \uc9c4\uaca9\ud558\ub294 \uc18c\ub828\uad70\uc5d0 \uc758\ud574 1944\ub144 7\uc6d4 23\uc77c\uc5d0 \ubc1c\uacac\ub418\uc5c8\ub2e4. Chelmno \uc218\uc6a9\uc18c\ub294 \uc18c\ub828\uad70\uc5d0 \uc758\ud574 1945\ub144 1\uc6d4 20\uc77c \uc5d0 \ud574\ubc29\ub418\uc5c8\uace0, \uc544\uc6b0\uc288\ube44\uce20 \uc218\uc6a9\uc18c \ub610\ud55c \uc18c\ub828\uad70\uc5d0 \uc758\ud574 1945\ub144 1\uc6d4 27\uc77c\uc5d0 \ud574\ubc29\ub418\uc5c8\ub2e4. Buchenwald \uc218\uc6a9\uc18c\ub294 \ubbf8\uad70\uc5d0 \uc758\ud574 4\uc6d4 11\uc77c, Bergen-Belsen \uc218\uc6a9\uc18c\ub294 \uc601\uad6d\uad70\uc5d0 \uc758\ud574 4\uc6d4 15\uc77c, Dachau \uc218\uc6a9\uc18c\ub294 \ubbf8\uad70\uc5d0 \uc758\ud574 4\uc6d4 29\uc77c, Ravensbr\u00fcck \uc218\uc6a9\uc18c\ub294 \uac19\uc740 \ub0a0 \uc18c\ub828\uad70\uc5d0 \uc758\ud574, Mauthausen \uc218\uc6a9\uc18c\ub294 \ubbf8\uad70\uc5d0 \uc758\ud574 5\uc6d4 5\uc77c, Theresienstadt \uc218\uc6a9\uc18c\ub294 \uc18c\ub828\uad70\uc5d0 \uc758\ud574 5\uc6d4 8\uc77c\uc5d0 \ud574\ubc29\ub418\uc5c8\ub2e4. Treblinka, Sobibor, Belzec \uc218\uc6a9\uc18c\ub294 \ud574\ubc29\ub418\uc9c0 \ubabb\ud558\uace0 1943\ub144\uc5d0 \ub098\uce58\uc5d0 \uc758\ud574 \ud30c\uad34\ub418\uc5c8\ub2e4. \ubbf87\uad70 William W. Quinn \ub300\ub839\uc740 Dachau \uc218\uc6a9\uc18c\uc5d0 \ub300\ud574 \u201c\uc6b0\ub9ac \ubd80\ub300\ub294 \uadf8 \uacf3\uc5d0\uc11c \ubbff\uae30 \ud798\ub4e4 \uc815\ub3c4\ub85c \ub054\ucc0d\ud55c \uad11\uacbd, \uc18c\ub9ac, \uc545\ucde8\ub97c \ubaa9\uaca9\ud558\uc600\ub2e4. \ub3c4\uc800\ud788 \uc77c\ubc18\uc801\uc778 \uc0c1\uc2dd\uc73c\ub85c\ub294 \uc774\ud574\ud560 \uc218 \uc5c6\ub294 \uc5c4\uccad\ub09c \uc794\uc778\ud568\uc774\uc5c8\ub2e4\u201d\ub77c\uace0 \ud68c\uace0\ud55c\ub2e4.", "sentence_answer": "Chelmno \uc218\uc6a9\uc18c\ub294 \uc18c\ub828\uad70\uc5d0 \uc758\ud574 1945\ub144 1\uc6d4 20\uc77c \uc5d0", "paragraph_id": "6571961-30-1", "paragraph_question": "question: Chelmno \uc218\uc6a9\uc18c\ub294 \uc5b8\uc81c \ud574\ubc29\ub418\uc5c8\ub294\uac00?, context: \uccab \ubc88\uc9f8 \uc8fc\uc694 \uc218\uc6a9\uc18c\uc600\ub358 Majdanek \uc218\uc6a9\uc18c\ub294 \uc9c4\uaca9\ud558\ub294 \uc18c\ub828\uad70\uc5d0 \uc758\ud574 1944\ub144 7\uc6d4 23\uc77c\uc5d0 \ubc1c\uacac\ub418\uc5c8\ub2e4. Chelmno \uc218\uc6a9\uc18c\ub294 \uc18c\ub828\uad70\uc5d0 \uc758\ud574 1945\ub144 1\uc6d4 20\uc77c\uc5d0 \ud574\ubc29\ub418\uc5c8\uace0, \uc544\uc6b0\uc288\ube44\uce20 \uc218\uc6a9\uc18c \ub610\ud55c \uc18c\ub828\uad70\uc5d0 \uc758\ud574 1945\ub144 1\uc6d4 27\uc77c\uc5d0 \ud574\ubc29\ub418\uc5c8\ub2e4. Buchenwald \uc218\uc6a9\uc18c\ub294 \ubbf8\uad70\uc5d0 \uc758\ud574 4\uc6d4 11\uc77c, Bergen-Belsen \uc218\uc6a9\uc18c\ub294 \uc601\uad6d\uad70\uc5d0 \uc758\ud574 4\uc6d4 15\uc77c, Dachau \uc218\uc6a9\uc18c\ub294 \ubbf8\uad70\uc5d0 \uc758\ud574 4\uc6d4 29\uc77c, Ravensbr\u00fcck \uc218\uc6a9\uc18c\ub294 \uac19\uc740 \ub0a0 \uc18c\ub828\uad70\uc5d0 \uc758\ud574, Mauthausen \uc218\uc6a9\uc18c\ub294 \ubbf8\uad70\uc5d0 \uc758\ud574 5\uc6d4 5\uc77c, Theresienstadt \uc218\uc6a9\uc18c\ub294 \uc18c\ub828\uad70\uc5d0 \uc758\ud574 5\uc6d4 8\uc77c\uc5d0 \ud574\ubc29\ub418\uc5c8\ub2e4. Treblinka, Sobibor, Belzec \uc218\uc6a9\uc18c\ub294 \ud574\ubc29\ub418\uc9c0 \ubabb\ud558\uace0 1943\ub144\uc5d0 \ub098\uce58\uc5d0 \uc758\ud574 \ud30c\uad34\ub418\uc5c8\ub2e4. \ubbf87\uad70 William W. Quinn \ub300\ub839\uc740 Dachau \uc218\uc6a9\uc18c\uc5d0 \ub300\ud574 \u201c\uc6b0\ub9ac \ubd80\ub300\ub294 \uadf8 \uacf3\uc5d0\uc11c \ubbff\uae30 \ud798\ub4e4 \uc815\ub3c4\ub85c \ub054\ucc0d\ud55c \uad11\uacbd, \uc18c\ub9ac, \uc545\ucde8\ub97c \ubaa9\uaca9\ud558\uc600\ub2e4. \ub3c4\uc800\ud788 \uc77c\ubc18\uc801\uc778 \uc0c1\uc2dd\uc73c\ub85c\ub294 \uc774\ud574\ud560 \uc218 \uc5c6\ub294 \uc5c4\uccad\ub09c \uc794\uc778\ud568\uc774\uc5c8\ub2e4\u201d\ub77c\uace0 \ud68c\uace0\ud55c\ub2e4."} {"question": "\ub7f0\ub358 \uc804\uc6d0 \uc9c0\uc5ed\uc5d0\uc11c\uc758 \uc601\uc8fc\uc5d0 \ub300\ud55c \ubd88\ub9cc\uc774 \uace0\uc870\ub418\uc790 \uc9c0\uc5ed \uc9c0\uc2dd\uc778\ub4e4\uc774 \ub9e1\uae30\ub97c \uac70\ubd80\ud55c \uc5ed\ud560\uc740?", "paragraph": "\uc804\uc6d0 \uacf5\ub3d9\uccb4, \ud2b9\ud788 \uadf8 \uc911\uc5d0\uc11c\ub3c4 \uc789\uae00\ub79c\ub4dc \ub3d9\ub0a8\ubd80\ub294 \ub18d\ub178\uc81c\uc758 \uc791\ub3d9\uacfc \uc9c0\uc5ed \uc7a5\uc6d0 \uc7ac\ud310\uc18c\uc758 \uc138\uae08 \uc9d5\uc218\ub97c \ubd88\ub9cc\uc2a4\ub7ec\uc6cc\ud588\ub2e4. \ud2b9\ud788 \uc7ac\ud310\uc18c\ub97c \uc6b4\uc601\ud558\ub294 \uc601\uc8fc\ub4e4\uc774 \uace7 \uc6d0\uc131\uc744 \uc0ac\ub294 \ub178\ub3d9 \uc870\ub840\ub098 \uc655\uc2e4\uc5d0\uc11c \uc81c\uc815\ud55c \ubc95\uc758 \ud604\uc7a5 \uc9d1\ud589\uc790\ub85c \uae30\ub2a5\ud558\ub294 \uacbd\uc6b0\uac00 \uc7a6\uc558\uae30 \ub54c\ubb38\uc5d0 \ubd88\ub9cc\uc740 \uac00\uc911\ub418\uc5c8\ub2e4. \ucd0c\ub77d\uc758 \uc5d8\ub9ac\ud2b8 \uacc4\uce35 \ub2e4\uc218\ub294 \uc9c0\ubc29\uc815\ubd80\uc758 \uc5ed\ud560\uc744 \ub9e1\ub294 \uac83\uc744 \uac70\ubd80\ud588\uace0, \uc7ac\ud310\uc18c\uc758 \ud65c\ub3d9\uc744 \ubc29\ud574\ud558\uae30 \uc2dc\uc791\ud588\ub2e4. \uc7ac\ud310\uc18c\uc5d0 \uc758\ud574 \ubab0\uc218\ub41c \uac00\ucd95\ub4e4\uc740 \uadf8 \uc8fc\uc778\ub4e4\uc5d0 \uc758\ud574 \u2018\ud574\ubc29\u2019\ub418\uc5c8\uace0, \ubc95\uad00\ub4e4\uc740 \ud3ed\ud589\uc744 \ub2f9\ud588\ub2e4. \uc77c\ubd80\ub294 \uc804\ud1b5\uc801\uc778 \ubc95\uc744 \uc874\uc911\ud558\ub098 \ub7f0\ub358\uc5d0\uc11c \ub0b4\ub824\uc624\ub294 \uc99d\uc624\ubc1b\ub294 \uc911\uc559 \ubc95\ub960\uc5d0\uc11c\ub294 \ubd84\ub9ac\ub41c, \ub3c5\ub9bd\uc801\uc778 \ucd0c\ub77d \uacf5\ub3d9\uccb4\uc758 \ud0c4\uc0dd\uc744 \uc9c0\uc9c0\ud558\uae30 \uc2dc\uc791\ud588\ub2e4. \uc0ac\ud559\uc790 \ubbf8\ub9ac \ub8e8\ube48\uc740 \uc774 \uc0c1\ud669\uc744, \u201c\ubb38\uc81c\ub294 \ub098\ub78f\ubc95 \uadf8 \uc790\uccb4\ub77c\uae30\ubcf4\ub2e4, \uadf8 \ubc95\uc744 \uc801\uc6a9\ud558\uace0 \uc639\ud638\ud558\ub294 \ub370 \uc788\uc5c8\ub2e4\u201d\uace0 \ubb18\uc0ac\ud55c\ub2e4.", "answer": "\uc9c0\ubc29\uc815\ubd80\uc758 \uc5ed\ud560", "sentence": "\ucd0c\ub77d\uc758 \uc5d8\ub9ac\ud2b8 \uacc4\uce35 \ub2e4\uc218\ub294 \uc9c0\ubc29\uc815\ubd80\uc758 \uc5ed\ud560 \uc744 \ub9e1\ub294 \uac83\uc744 \uac70\ubd80\ud588\uace0, \uc7ac\ud310\uc18c\uc758 \ud65c\ub3d9\uc744 \ubc29\ud574\ud558\uae30 \uc2dc\uc791\ud588\ub2e4.", "paragraph_sentence": "\uc804\uc6d0 \uacf5\ub3d9\uccb4, \ud2b9\ud788 \uadf8 \uc911\uc5d0\uc11c\ub3c4 \uc789\uae00\ub79c\ub4dc \ub3d9\ub0a8\ubd80\ub294 \ub18d\ub178\uc81c\uc758 \uc791\ub3d9\uacfc \uc9c0\uc5ed \uc7a5\uc6d0 \uc7ac\ud310\uc18c\uc758 \uc138\uae08 \uc9d5\uc218\ub97c \ubd88\ub9cc\uc2a4\ub7ec\uc6cc\ud588\ub2e4. \ud2b9\ud788 \uc7ac\ud310\uc18c\ub97c \uc6b4\uc601\ud558\ub294 \uc601\uc8fc\ub4e4\uc774 \uace7 \uc6d0\uc131\uc744 \uc0ac\ub294 \ub178\ub3d9 \uc870\ub840\ub098 \uc655\uc2e4\uc5d0\uc11c \uc81c\uc815\ud55c \ubc95\uc758 \ud604\uc7a5 \uc9d1\ud589\uc790\ub85c \uae30\ub2a5\ud558\ub294 \uacbd\uc6b0\uac00 \uc7a6\uc558\uae30 \ub54c\ubb38\uc5d0 \ubd88\ub9cc\uc740 \uac00\uc911\ub418\uc5c8\ub2e4. \ucd0c\ub77d\uc758 \uc5d8\ub9ac\ud2b8 \uacc4\uce35 \ub2e4\uc218\ub294 \uc9c0\ubc29\uc815\ubd80\uc758 \uc5ed\ud560 \uc744 \ub9e1\ub294 \uac83\uc744 \uac70\ubd80\ud588\uace0, \uc7ac\ud310\uc18c\uc758 \ud65c\ub3d9\uc744 \ubc29\ud574\ud558\uae30 \uc2dc\uc791\ud588\ub2e4. \uc7ac\ud310\uc18c\uc5d0 \uc758\ud574 \ubab0\uc218\ub41c \uac00\ucd95\ub4e4\uc740 \uadf8 \uc8fc\uc778\ub4e4\uc5d0 \uc758\ud574 \u2018\ud574\ubc29\u2019\ub418\uc5c8\uace0, \ubc95\uad00\ub4e4\uc740 \ud3ed\ud589\uc744 \ub2f9\ud588\ub2e4. \uc77c\ubd80\ub294 \uc804\ud1b5\uc801\uc778 \ubc95\uc744 \uc874\uc911\ud558\ub098 \ub7f0\ub358\uc5d0\uc11c \ub0b4\ub824\uc624\ub294 \uc99d\uc624\ubc1b\ub294 \uc911\uc559 \ubc95\ub960\uc5d0\uc11c\ub294 \ubd84\ub9ac\ub41c, \ub3c5\ub9bd\uc801\uc778 \ucd0c\ub77d \uacf5\ub3d9\uccb4\uc758 \ud0c4\uc0dd\uc744 \uc9c0\uc9c0\ud558\uae30 \uc2dc\uc791\ud588\ub2e4. \uc0ac\ud559\uc790 \ubbf8\ub9ac \ub8e8\ube48\uc740 \uc774 \uc0c1\ud669\uc744, \u201c\ubb38\uc81c\ub294 \ub098\ub78f\ubc95 \uadf8 \uc790\uccb4\ub77c\uae30\ubcf4\ub2e4, \uadf8 \ubc95\uc744 \uc801\uc6a9\ud558\uace0 \uc639\ud638\ud558\ub294 \ub370 \uc788\uc5c8\ub2e4\u201d\uace0 \ubb18\uc0ac\ud55c\ub2e4.", "paragraph_answer": "\uc804\uc6d0 \uacf5\ub3d9\uccb4, \ud2b9\ud788 \uadf8 \uc911\uc5d0\uc11c\ub3c4 \uc789\uae00\ub79c\ub4dc \ub3d9\ub0a8\ubd80\ub294 \ub18d\ub178\uc81c\uc758 \uc791\ub3d9\uacfc \uc9c0\uc5ed \uc7a5\uc6d0 \uc7ac\ud310\uc18c\uc758 \uc138\uae08 \uc9d5\uc218\ub97c \ubd88\ub9cc\uc2a4\ub7ec\uc6cc\ud588\ub2e4. \ud2b9\ud788 \uc7ac\ud310\uc18c\ub97c \uc6b4\uc601\ud558\ub294 \uc601\uc8fc\ub4e4\uc774 \uace7 \uc6d0\uc131\uc744 \uc0ac\ub294 \ub178\ub3d9 \uc870\ub840\ub098 \uc655\uc2e4\uc5d0\uc11c \uc81c\uc815\ud55c \ubc95\uc758 \ud604\uc7a5 \uc9d1\ud589\uc790\ub85c \uae30\ub2a5\ud558\ub294 \uacbd\uc6b0\uac00 \uc7a6\uc558\uae30 \ub54c\ubb38\uc5d0 \ubd88\ub9cc\uc740 \uac00\uc911\ub418\uc5c8\ub2e4. \ucd0c\ub77d\uc758 \uc5d8\ub9ac\ud2b8 \uacc4\uce35 \ub2e4\uc218\ub294 \uc9c0\ubc29\uc815\ubd80\uc758 \uc5ed\ud560 \uc744 \ub9e1\ub294 \uac83\uc744 \uac70\ubd80\ud588\uace0, \uc7ac\ud310\uc18c\uc758 \ud65c\ub3d9\uc744 \ubc29\ud574\ud558\uae30 \uc2dc\uc791\ud588\ub2e4. \uc7ac\ud310\uc18c\uc5d0 \uc758\ud574 \ubab0\uc218\ub41c \uac00\ucd95\ub4e4\uc740 \uadf8 \uc8fc\uc778\ub4e4\uc5d0 \uc758\ud574 \u2018\ud574\ubc29\u2019\ub418\uc5c8\uace0, \ubc95\uad00\ub4e4\uc740 \ud3ed\ud589\uc744 \ub2f9\ud588\ub2e4. \uc77c\ubd80\ub294 \uc804\ud1b5\uc801\uc778 \ubc95\uc744 \uc874\uc911\ud558\ub098 \ub7f0\ub358\uc5d0\uc11c \ub0b4\ub824\uc624\ub294 \uc99d\uc624\ubc1b\ub294 \uc911\uc559 \ubc95\ub960\uc5d0\uc11c\ub294 \ubd84\ub9ac\ub41c, \ub3c5\ub9bd\uc801\uc778 \ucd0c\ub77d \uacf5\ub3d9\uccb4\uc758 \ud0c4\uc0dd\uc744 \uc9c0\uc9c0\ud558\uae30 \uc2dc\uc791\ud588\ub2e4. \uc0ac\ud559\uc790 \ubbf8\ub9ac \ub8e8\ube48\uc740 \uc774 \uc0c1\ud669\uc744, \u201c\ubb38\uc81c\ub294 \ub098\ub78f\ubc95 \uadf8 \uc790\uccb4\ub77c\uae30\ubcf4\ub2e4, \uadf8 \ubc95\uc744 \uc801\uc6a9\ud558\uace0 \uc639\ud638\ud558\ub294 \ub370 \uc788\uc5c8\ub2e4\u201d\uace0 \ubb18\uc0ac\ud55c\ub2e4.", "sentence_answer": "\ucd0c\ub77d\uc758 \uc5d8\ub9ac\ud2b8 \uacc4\uce35 \ub2e4\uc218\ub294 \uc9c0\ubc29\uc815\ubd80\uc758 \uc5ed\ud560 \uc744 \ub9e1\ub294 \uac83\uc744 \uac70\ubd80\ud588\uace0, \uc7ac\ud310\uc18c\uc758 \ud65c\ub3d9\uc744 \ubc29\ud574\ud558\uae30 \uc2dc\uc791\ud588\ub2e4.", "paragraph_id": "6541489-12-1", "paragraph_question": "question: \ub7f0\ub358 \uc804\uc6d0 \uc9c0\uc5ed\uc5d0\uc11c\uc758 \uc601\uc8fc\uc5d0 \ub300\ud55c \ubd88\ub9cc\uc774 \uace0\uc870\ub418\uc790 \uc9c0\uc5ed \uc9c0\uc2dd\uc778\ub4e4\uc774 \ub9e1\uae30\ub97c \uac70\ubd80\ud55c \uc5ed\ud560\uc740?, context: \uc804\uc6d0 \uacf5\ub3d9\uccb4, \ud2b9\ud788 \uadf8 \uc911\uc5d0\uc11c\ub3c4 \uc789\uae00\ub79c\ub4dc \ub3d9\ub0a8\ubd80\ub294 \ub18d\ub178\uc81c\uc758 \uc791\ub3d9\uacfc \uc9c0\uc5ed \uc7a5\uc6d0 \uc7ac\ud310\uc18c\uc758 \uc138\uae08 \uc9d5\uc218\ub97c \ubd88\ub9cc\uc2a4\ub7ec\uc6cc\ud588\ub2e4. \ud2b9\ud788 \uc7ac\ud310\uc18c\ub97c \uc6b4\uc601\ud558\ub294 \uc601\uc8fc\ub4e4\uc774 \uace7 \uc6d0\uc131\uc744 \uc0ac\ub294 \ub178\ub3d9 \uc870\ub840\ub098 \uc655\uc2e4\uc5d0\uc11c \uc81c\uc815\ud55c \ubc95\uc758 \ud604\uc7a5 \uc9d1\ud589\uc790\ub85c \uae30\ub2a5\ud558\ub294 \uacbd\uc6b0\uac00 \uc7a6\uc558\uae30 \ub54c\ubb38\uc5d0 \ubd88\ub9cc\uc740 \uac00\uc911\ub418\uc5c8\ub2e4. \ucd0c\ub77d\uc758 \uc5d8\ub9ac\ud2b8 \uacc4\uce35 \ub2e4\uc218\ub294 \uc9c0\ubc29\uc815\ubd80\uc758 \uc5ed\ud560\uc744 \ub9e1\ub294 \uac83\uc744 \uac70\ubd80\ud588\uace0, \uc7ac\ud310\uc18c\uc758 \ud65c\ub3d9\uc744 \ubc29\ud574\ud558\uae30 \uc2dc\uc791\ud588\ub2e4. \uc7ac\ud310\uc18c\uc5d0 \uc758\ud574 \ubab0\uc218\ub41c \uac00\ucd95\ub4e4\uc740 \uadf8 \uc8fc\uc778\ub4e4\uc5d0 \uc758\ud574 \u2018\ud574\ubc29\u2019\ub418\uc5c8\uace0, \ubc95\uad00\ub4e4\uc740 \ud3ed\ud589\uc744 \ub2f9\ud588\ub2e4. \uc77c\ubd80\ub294 \uc804\ud1b5\uc801\uc778 \ubc95\uc744 \uc874\uc911\ud558\ub098 \ub7f0\ub358\uc5d0\uc11c \ub0b4\ub824\uc624\ub294 \uc99d\uc624\ubc1b\ub294 \uc911\uc559 \ubc95\ub960\uc5d0\uc11c\ub294 \ubd84\ub9ac\ub41c, \ub3c5\ub9bd\uc801\uc778 \ucd0c\ub77d \uacf5\ub3d9\uccb4\uc758 \ud0c4\uc0dd\uc744 \uc9c0\uc9c0\ud558\uae30 \uc2dc\uc791\ud588\ub2e4. \uc0ac\ud559\uc790 \ubbf8\ub9ac \ub8e8\ube48\uc740 \uc774 \uc0c1\ud669\uc744, \u201c\ubb38\uc81c\ub294 \ub098\ub78f\ubc95 \uadf8 \uc790\uccb4\ub77c\uae30\ubcf4\ub2e4, \uadf8 \ubc95\uc744 \uc801\uc6a9\ud558\uace0 \uc639\ud638\ud558\ub294 \ub370 \uc788\uc5c8\ub2e4\u201d\uace0 \ubb18\uc0ac\ud55c\ub2e4."} {"question": "\ub85c\ud06c\uc758 \ubaa9\ud45c\uc124\uc815\uc774\ub860\uc758 \ucd5c\uadfc\uc5f0\uad6c\uc758 \ucd94\uc138\ub294 \ubb34\uc5c7\uc744 \uc811\ubaa9\ud558\uc5ec \uc774\ub8e8\uc5b4\uc9c0\uace0 \uc788\ub294\uac00?", "paragraph": "\ub85c\ud06c\uc758 \ubaa9\ud45c\uc124\uc815\uc774\ub860\uc5d0 \ub300\ud574 \ucd5c\uadfc\uae4c\uc9c0\ub3c4 \uc5f0\uad6c\uac00 \uc9c0\uc18d\ub418\uace0 \uc788\ub2e4. \ucd5c\uadfc\uc5f0\uad6c\uc758 \ucd94\uc138\ub294 \uc774\ub860\uc801\uc778 \uc5f0\uad6c\ubcf4\ub2e4\ub3c4 \uc2e4\uc81c\uc0ac\ub840\uc5d0 \uc811\ubaa9\ud558\uc5ec \uc774\ub8e8\uc5b4\uc9c0\uace0 \uc788\ub2e4. \uc8fc\ub85c \ubaa9\ud45c \ub09c\uc774\ub3c4\uc640 \ubaa9\ud45c\ubab0\uc785, \ubaa9\ud45c\uc124\uc815\ubc29\ubc95\uc5d0 \ub300\ud574 \uc5f0\uad6c\uac00 \uc9c4\ud589\ub418\uace0 \uc788\uc73c\uba70, \ubaa9\ud45c\uc124\uc815\uc5d0 \ub530\ub978 \ud6a8\uacfc\ub97c \uc0ac\ub840\uac80\uc99d\uc744 \ud1b5\ud558\uc5ec \uc5f0\uad6c\ud558\uace0 \uc788\ub2e4. \ubaa9\ud45c\uc124\uc815\uc774 \uc9c1\ubb34\ud0dc\ub3c4\uc5d0 \ubbf8\uce58\ub294 \uc601\ud5a5\uc5d0 \ub300\ud55c \uc5f0\uad6c\uacb0\uacfc\ub97c \uc0b4\ud3b4\ubcf4\uba74, \uc9c1\ubb34\ub9cc\uc871\uc5d0 \uc8fc\uc548\uc810\uc744 \ub450\uc5b4 \ubaa9\ud45c\uc124\uc815\uacfc \uc9c1\ubb34\ud0dc\ub3c4\uc758 \uc0c1\uad00\uc131\uc744 \ubd84\uc11d\ud558\uace0 \uc788\ub2e4. \ubaa9\ud45c\uc124\uc815\uc744 \ud1b5\ud55c \uc9c1\ubb34\ub9cc\uc871\uc740 \uc870\uc9c1\ub3d9\uc77c\uc2dc\uc640 \uc870\uc9c1\ubab0\uc785\uc73c\ub85c \uc5f0\uacb0\ub418\uace0, \uc774\uac83\uc774 \uc870\uc9c1\uc758 \uacbd\uc7c1\ub825\uc744 \ub192\uc774\ub294 \uc5f0\uacb0 \uace0\ub9ac\uac00 \ub418\uc5b4 \uad81\uadf9\uc801\uc73c\ub85c\ub294 \uc870\uc9c1\uc758 \uc720\ud6a8\uc131\uc744 \ud5a5\uc0c1\uc73c\ub85c \uc774\uc5b4\uc9c4\ub2e4\ub294 \uacb0\ub860\uc744 \ub0b4\uace0 \uc788\ub2e4. \uae30\uc5c5\uc5d0\uc11c\ub294 \uc5b4\ub5bb\uac8c \ud574\uc57c \uc870\uc9c1\uacfc \uc0ac\ub78c\uc758 \uc131\uacfc\ub97c \ud5a5\uc0c1\uc2dc\ud0ac \uac83\uc778\uac00\uc5d0 \ub300\ud574\uc11c \uace0\ubbfc\uc744 \ud558\uac8c \ub418\uba70, \uc870\uc9c1 \ub0b4\uc5d0\uc11c \ub9c1\uac94\ub9cc \ud6a8\uacfc\ub97c \uc904\uc77c \uc218 \uc788\ub294 \ubc29\ubc95\uc5d0 \ub300\ud574 \ubaa9\ud45c\ubab0\uc785\uc744 \uc911\uc810\uc73c\ub85c \uc81c\uc57d\ud68c\uc0ac\ub97c \ud45c\ubcf8 \uc0bc\uc544 \uc2e4\uc99d\ubd84\uc11d\uc744 \ud588\ub2e4. \ucc38\uc5ec\uc801 \ubaa9\ud45c\uc124\uc815\uc774 \uc9c1\ubb34\ud0dc\ub3c4\uc5d0 \ubbf8\uce58\ub294 \uc601\ud5a5\uacfc \ucc38\uc5ec\uc801 \ubaa9\ud45c\uc124\uc815\uc774 \uc870\uc9c1\ubab0\uc785\uc5d0 \ubbf8\uce58\ub294 \uc601\ud5a5\uc5d0 \ub300\ud574 \uc5f0\uad6c\ud55c \uacb0\uacfc, \uc81c\uc57d\ud68c\uc0ac \uc601\uc5c5 \ubc0f \uc601\uc5c5\uad00\ub828 \uc9c1\uc6d0\ub4e4\uc5d0\uac8c \ubaa9\ud45c\uc124\uc815\uacfc \uc9c1\ubb34\ud0dc\ub3c4\uc5d0 \ubbf8\uce58\ub294 \uc601\ud5a5\uc740 \ubaa9\ud45c\uc758 \uba85\ud655\uc131\uacfc \ub09c\uc774\ub3c4, \ucc38\uc5ec\uc801 \ubaa9\ud45c\uc124\uc815\uacfc \ubaa9\ud45c\uc758 \ud53c\ub4dc\ubc31\uc774 \uc81c\uc57d\ud68c\uc0ac \uc9c1\uc6d0\ub4e4\uc758 \uc9c1\ubb34\ud0dc\ub3c4\ub97c \ub192\uc774\ub294 \ub370 \uc601\ud5a5\uc744 \ubbf8\uce5c\ub2e4\ub294 \uacb0\uacfc\ub97c \ubcf4\uc5ec\uc8fc\uace0 \uc788\ub2e4. \ubaa9\ud45c\uc758 \uad6c\uccb4\uc131\uacfc \uc778\uc13c\ud2f0\ube0c\uac00 \uacfc\uc5c5\uc131\uacfc \ubc0f \ub178\ub825\uc815\ub3c4\uc5d0 \ubbf8\uce58\ub294 \uc601\ud5a5\uc5d0 \ub300\ud55c \uc5f0\uad6c \uc5d0\uc120 \ub85c\ud06c\uc758 \ubaa9\ud45c\uc124\uc815\uc774\ub860\uc5d0 \ub300\ud55c \uc120\ud589\uc5f0\uad6c\ub97c \uadc0\ub0a9\uc801\uc73c\ub85c \ubd84\uc11d\ud574 \ubaa9\ud45c\uc758 \uad6c\uccb4\uc131\uacfc \ubaa9\ud45c\uc758 \ub09c\uc774\ub3c4\uac00 \uc131\uacfc\ub97c \uc88c\uc6b0\uc9c0\ud558\ub294\ub370 \ud070 \uc601\ud5a5\uc744 \ubbf8\uce58\uace0 \uc788\ub2e4\uace0 \ubcf8\ub2e4. \uadf8\uc5d0 \ubaa9\ud45c\uac00 \uad6c\uccb4\uc801\uc77c\uc218\ub85d \uacfc\uc5c5\uc131\uacfc \ubc0f \ub178\ub825\uc815\ub3c4\uc5d0 \uc815(+)\uc758 \uc601\ud5a5\uc744 \ubbf8\uce5c\ub2e4\ub294 \uac00\uc815\uc744 \uc81c\uc2dc\ud558\uace0 1\ucc28, 2\ucc28\uc2e4\ud5d8\uacfc \uc124\ubb38\uc9c0\uc5d0 \uc788\ub294 \uc870\uc791\uc810\uac80\ud56d\ubaa9\uc758 \uccb4\ud06c\uc5ec\ubd80\uc5d0 \ub530\ub77c \ubaa9\ud45c\uc758 \uad6c\uccb4\uc131\uc744 \uce21\uc815\ud558\uc600\ub2e4. \uc2e4\uc99d\uacb0\uacfc \uad6c\uccb4\uc801\uc774\uace0 \ub3c4\uc804\uc801\uc778 \ubaa9\ud45c\uac00 \uc8fc\uc5b4\uc9c8 \uacbd\uc6b0 \ubaa9\ud45c\uac00 \uc8fc\uc5b4\uc9c0\uc9c0 \uc54a\uac70\ub098 \ubaa8\ud638\ud55c \ubaa9\ud45c\uac00 \uc8fc\uc5b4\uc9c8 \ub54c \ubcf4\ub2e4 \uacfc\uc5c5\uc744 \uc218\ud589\ud568\uc5d0 \uc788\uc5b4\uc11c, \ub354 \ub9ce\uc740 \ub178\ub825\uc744 \uae30\uc6b8\uc774\uace0 \ub354 \ub192\uc740 \uacfc\uc5c5\uc131\uacfc\ub97c \ub0b4\uc9c0 \uc54a\ub294\ub2e4\ub294 \uacb0\ub860\uc774 \ub3c4\ucd9c\ub418\uc5c8\ub2e4. \uc774\ub7ec\ud55c \uacb0\ub860\uc740 \uc5f0\uad6c\uc758 \ud45c\ubcf8\uc758 \ud06c\ud0a4\uac00 \uc0c1\ub300\uc801\uc73c\ub85c \uc791\uc544 \ud55c\uacc4\uc810\uc744 \uac16\uae30\ub3c4 \ud558\uc9c0\ub9cc, \ubc18\ub4dc\uc2dc \uad6c\uccb4\uc801\uc778 \ubaa9\ud45c\uac00 \ubaa8\ud638\ud55c \ubaa9\ud45c\ubcf4\ub2e4 \ud56d\uc0c1 \uc815(+)\uc758 \uad00\uacc4\ub97c \uac16\uc9c0 \uc54a\ub294\ub2e4\ub294 \uc0ac\uc2e4\uc744 \ubcf4\uc5ec\uc8fc\uace0 \uc788\ub2e4.", "answer": "\uc2e4\uc81c\uc0ac\ub840", "sentence": "\ucd5c\uadfc\uc5f0\uad6c\uc758 \ucd94\uc138\ub294 \uc774\ub860\uc801\uc778 \uc5f0\uad6c\ubcf4\ub2e4\ub3c4 \uc2e4\uc81c\uc0ac\ub840 \uc5d0 \uc811\ubaa9\ud558\uc5ec \uc774\ub8e8\uc5b4\uc9c0\uace0 \uc788\ub2e4.", "paragraph_sentence": "\ub85c\ud06c\uc758 \ubaa9\ud45c\uc124\uc815\uc774\ub860\uc5d0 \ub300\ud574 \ucd5c\uadfc\uae4c\uc9c0\ub3c4 \uc5f0\uad6c\uac00 \uc9c0\uc18d\ub418\uace0 \uc788\ub2e4. \ucd5c\uadfc\uc5f0\uad6c\uc758 \ucd94\uc138\ub294 \uc774\ub860\uc801\uc778 \uc5f0\uad6c\ubcf4\ub2e4\ub3c4 \uc2e4\uc81c\uc0ac\ub840 \uc5d0 \uc811\ubaa9\ud558\uc5ec \uc774\ub8e8\uc5b4\uc9c0\uace0 \uc788\ub2e4. \uc8fc\ub85c \ubaa9\ud45c \ub09c\uc774\ub3c4\uc640 \ubaa9\ud45c\ubab0\uc785, \ubaa9\ud45c\uc124\uc815\ubc29\ubc95\uc5d0 \ub300\ud574 \uc5f0\uad6c\uac00 \uc9c4\ud589\ub418\uace0 \uc788\uc73c\uba70, \ubaa9\ud45c\uc124\uc815\uc5d0 \ub530\ub978 \ud6a8\uacfc\ub97c \uc0ac\ub840\uac80\uc99d\uc744 \ud1b5\ud558\uc5ec \uc5f0\uad6c\ud558\uace0 \uc788\ub2e4. \ubaa9\ud45c\uc124\uc815\uc774 \uc9c1\ubb34\ud0dc\ub3c4\uc5d0 \ubbf8\uce58\ub294 \uc601\ud5a5\uc5d0 \ub300\ud55c \uc5f0\uad6c\uacb0\uacfc\ub97c \uc0b4\ud3b4\ubcf4\uba74, \uc9c1\ubb34\ub9cc\uc871\uc5d0 \uc8fc\uc548\uc810\uc744 \ub450\uc5b4 \ubaa9\ud45c\uc124\uc815\uacfc \uc9c1\ubb34\ud0dc\ub3c4\uc758 \uc0c1\uad00\uc131\uc744 \ubd84\uc11d\ud558\uace0 \uc788\ub2e4. \ubaa9\ud45c\uc124\uc815\uc744 \ud1b5\ud55c \uc9c1\ubb34\ub9cc\uc871\uc740 \uc870\uc9c1\ub3d9\uc77c\uc2dc\uc640 \uc870\uc9c1\ubab0\uc785\uc73c\ub85c \uc5f0\uacb0\ub418\uace0, \uc774\uac83\uc774 \uc870\uc9c1\uc758 \uacbd\uc7c1\ub825\uc744 \ub192\uc774\ub294 \uc5f0\uacb0 \uace0\ub9ac\uac00 \ub418\uc5b4 \uad81\uadf9\uc801\uc73c\ub85c\ub294 \uc870\uc9c1\uc758 \uc720\ud6a8\uc131\uc744 \ud5a5\uc0c1\uc73c\ub85c \uc774\uc5b4\uc9c4\ub2e4\ub294 \uacb0\ub860\uc744 \ub0b4\uace0 \uc788\ub2e4. \uae30\uc5c5\uc5d0\uc11c\ub294 \uc5b4\ub5bb\uac8c \ud574\uc57c \uc870\uc9c1\uacfc \uc0ac\ub78c\uc758 \uc131\uacfc\ub97c \ud5a5\uc0c1\uc2dc\ud0ac \uac83\uc778\uac00\uc5d0 \ub300\ud574\uc11c \uace0\ubbfc\uc744 \ud558\uac8c \ub418\uba70, \uc870\uc9c1 \ub0b4\uc5d0\uc11c \ub9c1\uac94\ub9cc \ud6a8\uacfc\ub97c \uc904\uc77c \uc218 \uc788\ub294 \ubc29\ubc95\uc5d0 \ub300\ud574 \ubaa9\ud45c\ubab0\uc785\uc744 \uc911\uc810\uc73c\ub85c \uc81c\uc57d\ud68c\uc0ac\ub97c \ud45c\ubcf8 \uc0bc\uc544 \uc2e4\uc99d\ubd84\uc11d\uc744 \ud588\ub2e4. \ucc38\uc5ec\uc801 \ubaa9\ud45c\uc124\uc815\uc774 \uc9c1\ubb34\ud0dc\ub3c4\uc5d0 \ubbf8\uce58\ub294 \uc601\ud5a5\uacfc \ucc38\uc5ec\uc801 \ubaa9\ud45c\uc124\uc815\uc774 \uc870\uc9c1\ubab0\uc785\uc5d0 \ubbf8\uce58\ub294 \uc601\ud5a5\uc5d0 \ub300\ud574 \uc5f0\uad6c\ud55c \uacb0\uacfc, \uc81c\uc57d\ud68c\uc0ac \uc601\uc5c5 \ubc0f \uc601\uc5c5\uad00\ub828 \uc9c1\uc6d0\ub4e4\uc5d0\uac8c \ubaa9\ud45c\uc124\uc815\uacfc \uc9c1\ubb34\ud0dc\ub3c4\uc5d0 \ubbf8\uce58\ub294 \uc601\ud5a5\uc740 \ubaa9\ud45c\uc758 \uba85\ud655\uc131\uacfc \ub09c\uc774\ub3c4, \ucc38\uc5ec\uc801 \ubaa9\ud45c\uc124\uc815\uacfc \ubaa9\ud45c\uc758 \ud53c\ub4dc\ubc31\uc774 \uc81c\uc57d\ud68c\uc0ac \uc9c1\uc6d0\ub4e4\uc758 \uc9c1\ubb34\ud0dc\ub3c4\ub97c \ub192\uc774\ub294 \ub370 \uc601\ud5a5\uc744 \ubbf8\uce5c\ub2e4\ub294 \uacb0\uacfc\ub97c \ubcf4\uc5ec\uc8fc\uace0 \uc788\ub2e4. \ubaa9\ud45c\uc758 \uad6c\uccb4\uc131\uacfc \uc778\uc13c\ud2f0\ube0c\uac00 \uacfc\uc5c5\uc131\uacfc \ubc0f \ub178\ub825\uc815\ub3c4\uc5d0 \ubbf8\uce58\ub294 \uc601\ud5a5\uc5d0 \ub300\ud55c \uc5f0\uad6c \uc5d0\uc120 \ub85c\ud06c\uc758 \ubaa9\ud45c\uc124\uc815\uc774\ub860\uc5d0 \ub300\ud55c \uc120\ud589\uc5f0\uad6c\ub97c \uadc0\ub0a9\uc801\uc73c\ub85c \ubd84\uc11d\ud574 \ubaa9\ud45c\uc758 \uad6c\uccb4\uc131\uacfc \ubaa9\ud45c\uc758 \ub09c\uc774\ub3c4\uac00 \uc131\uacfc\ub97c \uc88c\uc6b0\uc9c0\ud558\ub294\ub370 \ud070 \uc601\ud5a5\uc744 \ubbf8\uce58\uace0 \uc788\ub2e4\uace0 \ubcf8\ub2e4. \uadf8\uc5d0 \ubaa9\ud45c\uac00 \uad6c\uccb4\uc801\uc77c\uc218\ub85d \uacfc\uc5c5\uc131\uacfc \ubc0f \ub178\ub825\uc815\ub3c4\uc5d0 \uc815(+)\uc758 \uc601\ud5a5\uc744 \ubbf8\uce5c\ub2e4\ub294 \uac00\uc815\uc744 \uc81c\uc2dc\ud558\uace0 1\ucc28, 2\ucc28\uc2e4\ud5d8\uacfc \uc124\ubb38\uc9c0\uc5d0 \uc788\ub294 \uc870\uc791\uc810\uac80\ud56d\ubaa9\uc758 \uccb4\ud06c\uc5ec\ubd80\uc5d0 \ub530\ub77c \ubaa9\ud45c\uc758 \uad6c\uccb4\uc131\uc744 \uce21\uc815\ud558\uc600\ub2e4. \uc2e4\uc99d\uacb0\uacfc \uad6c\uccb4\uc801\uc774\uace0 \ub3c4\uc804\uc801\uc778 \ubaa9\ud45c\uac00 \uc8fc\uc5b4\uc9c8 \uacbd\uc6b0 \ubaa9\ud45c\uac00 \uc8fc\uc5b4\uc9c0\uc9c0 \uc54a\uac70\ub098 \ubaa8\ud638\ud55c \ubaa9\ud45c\uac00 \uc8fc\uc5b4\uc9c8 \ub54c \ubcf4\ub2e4 \uacfc\uc5c5\uc744 \uc218\ud589\ud568\uc5d0 \uc788\uc5b4\uc11c, \ub354 \ub9ce\uc740 \ub178\ub825\uc744 \uae30\uc6b8\uc774\uace0 \ub354 \ub192\uc740 \uacfc\uc5c5\uc131\uacfc\ub97c \ub0b4\uc9c0 \uc54a\ub294\ub2e4\ub294 \uacb0\ub860\uc774 \ub3c4\ucd9c\ub418\uc5c8\ub2e4. \uc774\ub7ec\ud55c \uacb0\ub860\uc740 \uc5f0\uad6c\uc758 \ud45c\ubcf8\uc758 \ud06c\ud0a4\uac00 \uc0c1\ub300\uc801\uc73c\ub85c \uc791\uc544 \ud55c\uacc4\uc810\uc744 \uac16\uae30\ub3c4 \ud558\uc9c0\ub9cc, \ubc18\ub4dc\uc2dc \uad6c\uccb4\uc801\uc778 \ubaa9\ud45c\uac00 \ubaa8\ud638\ud55c \ubaa9\ud45c\ubcf4\ub2e4 \ud56d\uc0c1 \uc815(+)\uc758 \uad00\uacc4\ub97c \uac16\uc9c0 \uc54a\ub294\ub2e4\ub294 \uc0ac\uc2e4\uc744 \ubcf4\uc5ec\uc8fc\uace0 \uc788\ub2e4.", "paragraph_answer": "\ub85c\ud06c\uc758 \ubaa9\ud45c\uc124\uc815\uc774\ub860\uc5d0 \ub300\ud574 \ucd5c\uadfc\uae4c\uc9c0\ub3c4 \uc5f0\uad6c\uac00 \uc9c0\uc18d\ub418\uace0 \uc788\ub2e4. \ucd5c\uadfc\uc5f0\uad6c\uc758 \ucd94\uc138\ub294 \uc774\ub860\uc801\uc778 \uc5f0\uad6c\ubcf4\ub2e4\ub3c4 \uc2e4\uc81c\uc0ac\ub840 \uc5d0 \uc811\ubaa9\ud558\uc5ec \uc774\ub8e8\uc5b4\uc9c0\uace0 \uc788\ub2e4. \uc8fc\ub85c \ubaa9\ud45c \ub09c\uc774\ub3c4\uc640 \ubaa9\ud45c\ubab0\uc785, \ubaa9\ud45c\uc124\uc815\ubc29\ubc95\uc5d0 \ub300\ud574 \uc5f0\uad6c\uac00 \uc9c4\ud589\ub418\uace0 \uc788\uc73c\uba70, \ubaa9\ud45c\uc124\uc815\uc5d0 \ub530\ub978 \ud6a8\uacfc\ub97c \uc0ac\ub840\uac80\uc99d\uc744 \ud1b5\ud558\uc5ec \uc5f0\uad6c\ud558\uace0 \uc788\ub2e4. \ubaa9\ud45c\uc124\uc815\uc774 \uc9c1\ubb34\ud0dc\ub3c4\uc5d0 \ubbf8\uce58\ub294 \uc601\ud5a5\uc5d0 \ub300\ud55c \uc5f0\uad6c\uacb0\uacfc\ub97c \uc0b4\ud3b4\ubcf4\uba74, \uc9c1\ubb34\ub9cc\uc871\uc5d0 \uc8fc\uc548\uc810\uc744 \ub450\uc5b4 \ubaa9\ud45c\uc124\uc815\uacfc \uc9c1\ubb34\ud0dc\ub3c4\uc758 \uc0c1\uad00\uc131\uc744 \ubd84\uc11d\ud558\uace0 \uc788\ub2e4. \ubaa9\ud45c\uc124\uc815\uc744 \ud1b5\ud55c \uc9c1\ubb34\ub9cc\uc871\uc740 \uc870\uc9c1\ub3d9\uc77c\uc2dc\uc640 \uc870\uc9c1\ubab0\uc785\uc73c\ub85c \uc5f0\uacb0\ub418\uace0, \uc774\uac83\uc774 \uc870\uc9c1\uc758 \uacbd\uc7c1\ub825\uc744 \ub192\uc774\ub294 \uc5f0\uacb0 \uace0\ub9ac\uac00 \ub418\uc5b4 \uad81\uadf9\uc801\uc73c\ub85c\ub294 \uc870\uc9c1\uc758 \uc720\ud6a8\uc131\uc744 \ud5a5\uc0c1\uc73c\ub85c \uc774\uc5b4\uc9c4\ub2e4\ub294 \uacb0\ub860\uc744 \ub0b4\uace0 \uc788\ub2e4. \uae30\uc5c5\uc5d0\uc11c\ub294 \uc5b4\ub5bb\uac8c \ud574\uc57c \uc870\uc9c1\uacfc \uc0ac\ub78c\uc758 \uc131\uacfc\ub97c \ud5a5\uc0c1\uc2dc\ud0ac \uac83\uc778\uac00\uc5d0 \ub300\ud574\uc11c \uace0\ubbfc\uc744 \ud558\uac8c \ub418\uba70, \uc870\uc9c1 \ub0b4\uc5d0\uc11c \ub9c1\uac94\ub9cc \ud6a8\uacfc\ub97c \uc904\uc77c \uc218 \uc788\ub294 \ubc29\ubc95\uc5d0 \ub300\ud574 \ubaa9\ud45c\ubab0\uc785\uc744 \uc911\uc810\uc73c\ub85c \uc81c\uc57d\ud68c\uc0ac\ub97c \ud45c\ubcf8 \uc0bc\uc544 \uc2e4\uc99d\ubd84\uc11d\uc744 \ud588\ub2e4. \ucc38\uc5ec\uc801 \ubaa9\ud45c\uc124\uc815\uc774 \uc9c1\ubb34\ud0dc\ub3c4\uc5d0 \ubbf8\uce58\ub294 \uc601\ud5a5\uacfc \ucc38\uc5ec\uc801 \ubaa9\ud45c\uc124\uc815\uc774 \uc870\uc9c1\ubab0\uc785\uc5d0 \ubbf8\uce58\ub294 \uc601\ud5a5\uc5d0 \ub300\ud574 \uc5f0\uad6c\ud55c \uacb0\uacfc, \uc81c\uc57d\ud68c\uc0ac \uc601\uc5c5 \ubc0f \uc601\uc5c5\uad00\ub828 \uc9c1\uc6d0\ub4e4\uc5d0\uac8c \ubaa9\ud45c\uc124\uc815\uacfc \uc9c1\ubb34\ud0dc\ub3c4\uc5d0 \ubbf8\uce58\ub294 \uc601\ud5a5\uc740 \ubaa9\ud45c\uc758 \uba85\ud655\uc131\uacfc \ub09c\uc774\ub3c4, \ucc38\uc5ec\uc801 \ubaa9\ud45c\uc124\uc815\uacfc \ubaa9\ud45c\uc758 \ud53c\ub4dc\ubc31\uc774 \uc81c\uc57d\ud68c\uc0ac \uc9c1\uc6d0\ub4e4\uc758 \uc9c1\ubb34\ud0dc\ub3c4\ub97c \ub192\uc774\ub294 \ub370 \uc601\ud5a5\uc744 \ubbf8\uce5c\ub2e4\ub294 \uacb0\uacfc\ub97c \ubcf4\uc5ec\uc8fc\uace0 \uc788\ub2e4. \ubaa9\ud45c\uc758 \uad6c\uccb4\uc131\uacfc \uc778\uc13c\ud2f0\ube0c\uac00 \uacfc\uc5c5\uc131\uacfc \ubc0f \ub178\ub825\uc815\ub3c4\uc5d0 \ubbf8\uce58\ub294 \uc601\ud5a5\uc5d0 \ub300\ud55c \uc5f0\uad6c \uc5d0\uc120 \ub85c\ud06c\uc758 \ubaa9\ud45c\uc124\uc815\uc774\ub860\uc5d0 \ub300\ud55c \uc120\ud589\uc5f0\uad6c\ub97c \uadc0\ub0a9\uc801\uc73c\ub85c \ubd84\uc11d\ud574 \ubaa9\ud45c\uc758 \uad6c\uccb4\uc131\uacfc \ubaa9\ud45c\uc758 \ub09c\uc774\ub3c4\uac00 \uc131\uacfc\ub97c \uc88c\uc6b0\uc9c0\ud558\ub294\ub370 \ud070 \uc601\ud5a5\uc744 \ubbf8\uce58\uace0 \uc788\ub2e4\uace0 \ubcf8\ub2e4. \uadf8\uc5d0 \ubaa9\ud45c\uac00 \uad6c\uccb4\uc801\uc77c\uc218\ub85d \uacfc\uc5c5\uc131\uacfc \ubc0f \ub178\ub825\uc815\ub3c4\uc5d0 \uc815(+)\uc758 \uc601\ud5a5\uc744 \ubbf8\uce5c\ub2e4\ub294 \uac00\uc815\uc744 \uc81c\uc2dc\ud558\uace0 1\ucc28, 2\ucc28\uc2e4\ud5d8\uacfc \uc124\ubb38\uc9c0\uc5d0 \uc788\ub294 \uc870\uc791\uc810\uac80\ud56d\ubaa9\uc758 \uccb4\ud06c\uc5ec\ubd80\uc5d0 \ub530\ub77c \ubaa9\ud45c\uc758 \uad6c\uccb4\uc131\uc744 \uce21\uc815\ud558\uc600\ub2e4. \uc2e4\uc99d\uacb0\uacfc \uad6c\uccb4\uc801\uc774\uace0 \ub3c4\uc804\uc801\uc778 \ubaa9\ud45c\uac00 \uc8fc\uc5b4\uc9c8 \uacbd\uc6b0 \ubaa9\ud45c\uac00 \uc8fc\uc5b4\uc9c0\uc9c0 \uc54a\uac70\ub098 \ubaa8\ud638\ud55c \ubaa9\ud45c\uac00 \uc8fc\uc5b4\uc9c8 \ub54c \ubcf4\ub2e4 \uacfc\uc5c5\uc744 \uc218\ud589\ud568\uc5d0 \uc788\uc5b4\uc11c, \ub354 \ub9ce\uc740 \ub178\ub825\uc744 \uae30\uc6b8\uc774\uace0 \ub354 \ub192\uc740 \uacfc\uc5c5\uc131\uacfc\ub97c \ub0b4\uc9c0 \uc54a\ub294\ub2e4\ub294 \uacb0\ub860\uc774 \ub3c4\ucd9c\ub418\uc5c8\ub2e4. \uc774\ub7ec\ud55c \uacb0\ub860\uc740 \uc5f0\uad6c\uc758 \ud45c\ubcf8\uc758 \ud06c\ud0a4\uac00 \uc0c1\ub300\uc801\uc73c\ub85c \uc791\uc544 \ud55c\uacc4\uc810\uc744 \uac16\uae30\ub3c4 \ud558\uc9c0\ub9cc, \ubc18\ub4dc\uc2dc \uad6c\uccb4\uc801\uc778 \ubaa9\ud45c\uac00 \ubaa8\ud638\ud55c \ubaa9\ud45c\ubcf4\ub2e4 \ud56d\uc0c1 \uc815(+)\uc758 \uad00\uacc4\ub97c \uac16\uc9c0 \uc54a\ub294\ub2e4\ub294 \uc0ac\uc2e4\uc744 \ubcf4\uc5ec\uc8fc\uace0 \uc788\ub2e4.", "sentence_answer": "\ucd5c\uadfc\uc5f0\uad6c\uc758 \ucd94\uc138\ub294 \uc774\ub860\uc801\uc778 \uc5f0\uad6c\ubcf4\ub2e4\ub3c4 \uc2e4\uc81c\uc0ac\ub840 \uc5d0 \uc811\ubaa9\ud558\uc5ec \uc774\ub8e8\uc5b4\uc9c0\uace0 \uc788\ub2e4.", "paragraph_id": "6504583-2-0", "paragraph_question": "question: \ub85c\ud06c\uc758 \ubaa9\ud45c\uc124\uc815\uc774\ub860\uc758 \ucd5c\uadfc\uc5f0\uad6c\uc758 \ucd94\uc138\ub294 \ubb34\uc5c7\uc744 \uc811\ubaa9\ud558\uc5ec \uc774\ub8e8\uc5b4\uc9c0\uace0 \uc788\ub294\uac00?, context: \ub85c\ud06c\uc758 \ubaa9\ud45c\uc124\uc815\uc774\ub860\uc5d0 \ub300\ud574 \ucd5c\uadfc\uae4c\uc9c0\ub3c4 \uc5f0\uad6c\uac00 \uc9c0\uc18d\ub418\uace0 \uc788\ub2e4. \ucd5c\uadfc\uc5f0\uad6c\uc758 \ucd94\uc138\ub294 \uc774\ub860\uc801\uc778 \uc5f0\uad6c\ubcf4\ub2e4\ub3c4 \uc2e4\uc81c\uc0ac\ub840\uc5d0 \uc811\ubaa9\ud558\uc5ec \uc774\ub8e8\uc5b4\uc9c0\uace0 \uc788\ub2e4. \uc8fc\ub85c \ubaa9\ud45c \ub09c\uc774\ub3c4\uc640 \ubaa9\ud45c\ubab0\uc785, \ubaa9\ud45c\uc124\uc815\ubc29\ubc95\uc5d0 \ub300\ud574 \uc5f0\uad6c\uac00 \uc9c4\ud589\ub418\uace0 \uc788\uc73c\uba70, \ubaa9\ud45c\uc124\uc815\uc5d0 \ub530\ub978 \ud6a8\uacfc\ub97c \uc0ac\ub840\uac80\uc99d\uc744 \ud1b5\ud558\uc5ec \uc5f0\uad6c\ud558\uace0 \uc788\ub2e4. \ubaa9\ud45c\uc124\uc815\uc774 \uc9c1\ubb34\ud0dc\ub3c4\uc5d0 \ubbf8\uce58\ub294 \uc601\ud5a5\uc5d0 \ub300\ud55c \uc5f0\uad6c\uacb0\uacfc\ub97c \uc0b4\ud3b4\ubcf4\uba74, \uc9c1\ubb34\ub9cc\uc871\uc5d0 \uc8fc\uc548\uc810\uc744 \ub450\uc5b4 \ubaa9\ud45c\uc124\uc815\uacfc \uc9c1\ubb34\ud0dc\ub3c4\uc758 \uc0c1\uad00\uc131\uc744 \ubd84\uc11d\ud558\uace0 \uc788\ub2e4. \ubaa9\ud45c\uc124\uc815\uc744 \ud1b5\ud55c \uc9c1\ubb34\ub9cc\uc871\uc740 \uc870\uc9c1\ub3d9\uc77c\uc2dc\uc640 \uc870\uc9c1\ubab0\uc785\uc73c\ub85c \uc5f0\uacb0\ub418\uace0, \uc774\uac83\uc774 \uc870\uc9c1\uc758 \uacbd\uc7c1\ub825\uc744 \ub192\uc774\ub294 \uc5f0\uacb0 \uace0\ub9ac\uac00 \ub418\uc5b4 \uad81\uadf9\uc801\uc73c\ub85c\ub294 \uc870\uc9c1\uc758 \uc720\ud6a8\uc131\uc744 \ud5a5\uc0c1\uc73c\ub85c \uc774\uc5b4\uc9c4\ub2e4\ub294 \uacb0\ub860\uc744 \ub0b4\uace0 \uc788\ub2e4. \uae30\uc5c5\uc5d0\uc11c\ub294 \uc5b4\ub5bb\uac8c \ud574\uc57c \uc870\uc9c1\uacfc \uc0ac\ub78c\uc758 \uc131\uacfc\ub97c \ud5a5\uc0c1\uc2dc\ud0ac \uac83\uc778\uac00\uc5d0 \ub300\ud574\uc11c \uace0\ubbfc\uc744 \ud558\uac8c \ub418\uba70, \uc870\uc9c1 \ub0b4\uc5d0\uc11c \ub9c1\uac94\ub9cc \ud6a8\uacfc\ub97c \uc904\uc77c \uc218 \uc788\ub294 \ubc29\ubc95\uc5d0 \ub300\ud574 \ubaa9\ud45c\ubab0\uc785\uc744 \uc911\uc810\uc73c\ub85c \uc81c\uc57d\ud68c\uc0ac\ub97c \ud45c\ubcf8 \uc0bc\uc544 \uc2e4\uc99d\ubd84\uc11d\uc744 \ud588\ub2e4. \ucc38\uc5ec\uc801 \ubaa9\ud45c\uc124\uc815\uc774 \uc9c1\ubb34\ud0dc\ub3c4\uc5d0 \ubbf8\uce58\ub294 \uc601\ud5a5\uacfc \ucc38\uc5ec\uc801 \ubaa9\ud45c\uc124\uc815\uc774 \uc870\uc9c1\ubab0\uc785\uc5d0 \ubbf8\uce58\ub294 \uc601\ud5a5\uc5d0 \ub300\ud574 \uc5f0\uad6c\ud55c \uacb0\uacfc, \uc81c\uc57d\ud68c\uc0ac \uc601\uc5c5 \ubc0f \uc601\uc5c5\uad00\ub828 \uc9c1\uc6d0\ub4e4\uc5d0\uac8c \ubaa9\ud45c\uc124\uc815\uacfc \uc9c1\ubb34\ud0dc\ub3c4\uc5d0 \ubbf8\uce58\ub294 \uc601\ud5a5\uc740 \ubaa9\ud45c\uc758 \uba85\ud655\uc131\uacfc \ub09c\uc774\ub3c4, \ucc38\uc5ec\uc801 \ubaa9\ud45c\uc124\uc815\uacfc \ubaa9\ud45c\uc758 \ud53c\ub4dc\ubc31\uc774 \uc81c\uc57d\ud68c\uc0ac \uc9c1\uc6d0\ub4e4\uc758 \uc9c1\ubb34\ud0dc\ub3c4\ub97c \ub192\uc774\ub294 \ub370 \uc601\ud5a5\uc744 \ubbf8\uce5c\ub2e4\ub294 \uacb0\uacfc\ub97c \ubcf4\uc5ec\uc8fc\uace0 \uc788\ub2e4. \ubaa9\ud45c\uc758 \uad6c\uccb4\uc131\uacfc \uc778\uc13c\ud2f0\ube0c\uac00 \uacfc\uc5c5\uc131\uacfc \ubc0f \ub178\ub825\uc815\ub3c4\uc5d0 \ubbf8\uce58\ub294 \uc601\ud5a5\uc5d0 \ub300\ud55c \uc5f0\uad6c \uc5d0\uc120 \ub85c\ud06c\uc758 \ubaa9\ud45c\uc124\uc815\uc774\ub860\uc5d0 \ub300\ud55c \uc120\ud589\uc5f0\uad6c\ub97c \uadc0\ub0a9\uc801\uc73c\ub85c \ubd84\uc11d\ud574 \ubaa9\ud45c\uc758 \uad6c\uccb4\uc131\uacfc \ubaa9\ud45c\uc758 \ub09c\uc774\ub3c4\uac00 \uc131\uacfc\ub97c \uc88c\uc6b0\uc9c0\ud558\ub294\ub370 \ud070 \uc601\ud5a5\uc744 \ubbf8\uce58\uace0 \uc788\ub2e4\uace0 \ubcf8\ub2e4. \uadf8\uc5d0 \ubaa9\ud45c\uac00 \uad6c\uccb4\uc801\uc77c\uc218\ub85d \uacfc\uc5c5\uc131\uacfc \ubc0f \ub178\ub825\uc815\ub3c4\uc5d0 \uc815(+)\uc758 \uc601\ud5a5\uc744 \ubbf8\uce5c\ub2e4\ub294 \uac00\uc815\uc744 \uc81c\uc2dc\ud558\uace0 1\ucc28, 2\ucc28\uc2e4\ud5d8\uacfc \uc124\ubb38\uc9c0\uc5d0 \uc788\ub294 \uc870\uc791\uc810\uac80\ud56d\ubaa9\uc758 \uccb4\ud06c\uc5ec\ubd80\uc5d0 \ub530\ub77c \ubaa9\ud45c\uc758 \uad6c\uccb4\uc131\uc744 \uce21\uc815\ud558\uc600\ub2e4. \uc2e4\uc99d\uacb0\uacfc \uad6c\uccb4\uc801\uc774\uace0 \ub3c4\uc804\uc801\uc778 \ubaa9\ud45c\uac00 \uc8fc\uc5b4\uc9c8 \uacbd\uc6b0 \ubaa9\ud45c\uac00 \uc8fc\uc5b4\uc9c0\uc9c0 \uc54a\uac70\ub098 \ubaa8\ud638\ud55c \ubaa9\ud45c\uac00 \uc8fc\uc5b4\uc9c8 \ub54c \ubcf4\ub2e4 \uacfc\uc5c5\uc744 \uc218\ud589\ud568\uc5d0 \uc788\uc5b4\uc11c, \ub354 \ub9ce\uc740 \ub178\ub825\uc744 \uae30\uc6b8\uc774\uace0 \ub354 \ub192\uc740 \uacfc\uc5c5\uc131\uacfc\ub97c \ub0b4\uc9c0 \uc54a\ub294\ub2e4\ub294 \uacb0\ub860\uc774 \ub3c4\ucd9c\ub418\uc5c8\ub2e4. \uc774\ub7ec\ud55c \uacb0\ub860\uc740 \uc5f0\uad6c\uc758 \ud45c\ubcf8\uc758 \ud06c\ud0a4\uac00 \uc0c1\ub300\uc801\uc73c\ub85c \uc791\uc544 \ud55c\uacc4\uc810\uc744 \uac16\uae30\ub3c4 \ud558\uc9c0\ub9cc, \ubc18\ub4dc\uc2dc \uad6c\uccb4\uc801\uc778 \ubaa9\ud45c\uac00 \ubaa8\ud638\ud55c \ubaa9\ud45c\ubcf4\ub2e4 \ud56d\uc0c1 \uc815(+)\uc758 \uad00\uacc4\ub97c \uac16\uc9c0 \uc54a\ub294\ub2e4\ub294 \uc0ac\uc2e4\uc744 \ubcf4\uc5ec\uc8fc\uace0 \uc788\ub2e4."} {"question": "\uc704\uae30\uc5d0 \ube60\uc9c4 \ub098\ub77c\ub97c \uad6c\ud574\ub0b8 \uc655\uc5d0\uac8c \uc8fc\uc5b4\uc9c0\ub294 \ubb18\ud638\ub294?", "paragraph": "\ud754\ud788 \uc4f0\uace0 \ub110\ub9ac \uc54c\ub824\uc9c4 \ubb18\ud638\ub85c\ub294 \ub098\ub77c\ub97c \uc138\uc6b0\uac70\ub098 \ub098\ub77c\ub97c \uc138\uc6b0\uae30 \uc774\uc804\uc5d0 \uae30\ud2c0\uc744 \ub2e4\uc9c4 \uad70\uc8fc\uc5d0\uac8c \ubd99\uc774\ub294 \ud0dc\uc870(\u592a\u7956)\ub098 \uace0\uc870(\u9ad8\u7956), \ub098\ub77c\uc758 \uae30\ud2c0\uc744 \ub2e4\uc9c4 \ud0dc\uc885(\u592a\u5b97)\uc774\ub098 \uc138\uc870(\u4e16\u7956), \ub098\ub77c\ub97c \ubc1c\uc804\uc2dc\ud0a4\uace0 \uc911\ud765\ud55c \uc138\uc885(\u4e16\u5b97) \ub610\ub294 \uace0\uc885(\u9ad8\u5b97), \uadf8\ub9ac\uace0 \ub098\ub77c\ub97c \uc704\uae30\uc5d0\uc11c \uad6c\ud574\ub0b4\uac70\ub098 \ub9e5\uc774 \ub04a\uacbc\ub358 \uc655\ud1b5\uc744 \ub2e4\uc2dc \ubc14\ub85c\uc7a1\uc740 \uc911\uc885(\u4e2d\u5b97) \ub4f1\uc774 \uc788\ub2e4. \ubcf8\ub798 \ubb18\ud638\uac00 \ucc98\uc74c \ub9cc\ub4e4\uc5b4\uc84c\uc744 \ub54c\uc5d0\ub294 \ub2e8\uc9c0 \uc7ac\uc704\ud558\uc5ec \ub098\ub77c\ub97c \ub2e4\uc2a4\ub838\ub2e4\uace0 \ubc1b\ub294 \uac83\uc774 \uc544\ub2c8\ub77c \uad6d\uac00\uc5d0 \ud2b9\ubcc4\ud55c \uacf5\ub355\uc744 \ub0a8\uaca8 \uc5b5\uc870\ucc3d\uc0dd\uc774 \ubaa8\ub450 \uadf8 \uacf5\uc744 \uce58\ud558\ud558\uace0 \uc6b0\ub7ec\ub7ec\ubcfc \ub54c \ube44\ub85c\uc18c \ud0dc\ubb18\uc5d0 \ubd09\uc548\ud558\uace0 \uc62c\ub9b0 \ud638\uce6d\uc774\uc5c8\uc73c\ubbc0\ub85c, \ub098\ub77c\ub97c \ub2e4\uc2a4\ub9b0 \uad70\uc8fc \uac00\uc6b4\ub370\uc11c\ub3c4 \uc18c\uc218\ub9cc\uc774 \ubc1b\uc744 \uc218 \uc788\ub358 \uc601\uc608\ub85c\uc6b4 \uce6d\ud638\uc600\ub2e4. \uadf8\ub7ec\ub098 \ud6c4\ub300\uc5d0 \uac08\uc218\ub85d \uc7ac\uc704\ud55c \ubaa8\ub4e0 \ud669\uc81c\ub4e4\uc744 \ud0dc\ubb18\uc5d0 \ubd09\uc548\ud558\uba74\uc11c \ubb18\ud638\ub3c4 \uac19\uc774 \uc62c\ub9ac\uae30 \uc2dc\uc791\ud558\uc600\ub2e4.", "answer": "\uc911\uc885", "sentence": "\ud754\ud788 \uc4f0\uace0 \ub110\ub9ac \uc54c\ub824\uc9c4 \ubb18\ud638\ub85c\ub294 \ub098\ub77c\ub97c \uc138\uc6b0\uac70\ub098 \ub098\ub77c\ub97c \uc138\uc6b0\uae30 \uc774\uc804\uc5d0 \uae30\ud2c0\uc744 \ub2e4\uc9c4 \uad70\uc8fc\uc5d0\uac8c \ubd99\uc774\ub294 \ud0dc\uc870(\u592a\u7956)\ub098 \uace0\uc870(\u9ad8\u7956), \ub098\ub77c\uc758 \uae30\ud2c0\uc744 \ub2e4\uc9c4 \ud0dc\uc885(\u592a\u5b97)\uc774\ub098 \uc138\uc870(\u4e16\u7956), \ub098\ub77c\ub97c \ubc1c\uc804\uc2dc\ud0a4\uace0 \uc911\ud765\ud55c \uc138\uc885(\u4e16\u5b97) \ub610\ub294 \uace0\uc885(\u9ad8\u5b97), \uadf8\ub9ac\uace0 \ub098\ub77c\ub97c \uc704\uae30\uc5d0\uc11c \uad6c\ud574\ub0b4\uac70\ub098 \ub9e5\uc774 \ub04a\uacbc\ub358 \uc655\ud1b5\uc744 \ub2e4\uc2dc \ubc14\ub85c\uc7a1\uc740 \uc911\uc885 (\u4e2d\u5b97)", "paragraph_sentence": " \ud754\ud788 \uc4f0\uace0 \ub110\ub9ac \uc54c\ub824\uc9c4 \ubb18\ud638\ub85c\ub294 \ub098\ub77c\ub97c \uc138\uc6b0\uac70\ub098 \ub098\ub77c\ub97c \uc138\uc6b0\uae30 \uc774\uc804\uc5d0 \uae30\ud2c0\uc744 \ub2e4\uc9c4 \uad70\uc8fc\uc5d0\uac8c \ubd99\uc774\ub294 \ud0dc\uc870(\u592a\u7956)\ub098 \uace0\uc870(\u9ad8\u7956), \ub098\ub77c\uc758 \uae30\ud2c0\uc744 \ub2e4\uc9c4 \ud0dc\uc885(\u592a\u5b97)\uc774\ub098 \uc138\uc870(\u4e16\u7956), \ub098\ub77c\ub97c \ubc1c\uc804\uc2dc\ud0a4\uace0 \uc911\ud765\ud55c \uc138\uc885(\u4e16\u5b97) \ub610\ub294 \uace0\uc885(\u9ad8\u5b97), \uadf8\ub9ac\uace0 \ub098\ub77c\ub97c \uc704\uae30\uc5d0\uc11c \uad6c\ud574\ub0b4\uac70\ub098 \ub9e5\uc774 \ub04a\uacbc\ub358 \uc655\ud1b5\uc744 \ub2e4\uc2dc \ubc14\ub85c\uc7a1\uc740 \uc911\uc885 (\u4e2d\u5b97) \ub4f1\uc774 \uc788\ub2e4. \ubcf8\ub798 \ubb18\ud638\uac00 \ucc98\uc74c \ub9cc\ub4e4\uc5b4\uc84c\uc744 \ub54c\uc5d0\ub294 \ub2e8\uc9c0 \uc7ac\uc704\ud558\uc5ec \ub098\ub77c\ub97c \ub2e4\uc2a4\ub838\ub2e4\uace0 \ubc1b\ub294 \uac83\uc774 \uc544\ub2c8\ub77c \uad6d\uac00\uc5d0 \ud2b9\ubcc4\ud55c \uacf5\ub355\uc744 \ub0a8\uaca8 \uc5b5\uc870\ucc3d\uc0dd\uc774 \ubaa8\ub450 \uadf8 \uacf5\uc744 \uce58\ud558\ud558\uace0 \uc6b0\ub7ec\ub7ec\ubcfc \ub54c \ube44\ub85c\uc18c \ud0dc\ubb18\uc5d0 \ubd09\uc548\ud558\uace0 \uc62c\ub9b0 \ud638\uce6d\uc774\uc5c8\uc73c\ubbc0\ub85c, \ub098\ub77c\ub97c \ub2e4\uc2a4\ub9b0 \uad70\uc8fc \uac00\uc6b4\ub370\uc11c\ub3c4 \uc18c\uc218\ub9cc\uc774 \ubc1b\uc744 \uc218 \uc788\ub358 \uc601\uc608\ub85c\uc6b4 \uce6d\ud638\uc600\ub2e4. \uadf8\ub7ec\ub098 \ud6c4\ub300\uc5d0 \uac08\uc218\ub85d \uc7ac\uc704\ud55c \ubaa8\ub4e0 \ud669\uc81c\ub4e4\uc744 \ud0dc\ubb18\uc5d0 \ubd09\uc548\ud558\uba74\uc11c \ubb18\ud638\ub3c4 \uac19\uc774 \uc62c\ub9ac\uae30 \uc2dc\uc791\ud558\uc600\ub2e4.", "paragraph_answer": "\ud754\ud788 \uc4f0\uace0 \ub110\ub9ac \uc54c\ub824\uc9c4 \ubb18\ud638\ub85c\ub294 \ub098\ub77c\ub97c \uc138\uc6b0\uac70\ub098 \ub098\ub77c\ub97c \uc138\uc6b0\uae30 \uc774\uc804\uc5d0 \uae30\ud2c0\uc744 \ub2e4\uc9c4 \uad70\uc8fc\uc5d0\uac8c \ubd99\uc774\ub294 \ud0dc\uc870(\u592a\u7956)\ub098 \uace0\uc870(\u9ad8\u7956), \ub098\ub77c\uc758 \uae30\ud2c0\uc744 \ub2e4\uc9c4 \ud0dc\uc885(\u592a\u5b97)\uc774\ub098 \uc138\uc870(\u4e16\u7956), \ub098\ub77c\ub97c \ubc1c\uc804\uc2dc\ud0a4\uace0 \uc911\ud765\ud55c \uc138\uc885(\u4e16\u5b97) \ub610\ub294 \uace0\uc885(\u9ad8\u5b97), \uadf8\ub9ac\uace0 \ub098\ub77c\ub97c \uc704\uae30\uc5d0\uc11c \uad6c\ud574\ub0b4\uac70\ub098 \ub9e5\uc774 \ub04a\uacbc\ub358 \uc655\ud1b5\uc744 \ub2e4\uc2dc \ubc14\ub85c\uc7a1\uc740 \uc911\uc885 (\u4e2d\u5b97) \ub4f1\uc774 \uc788\ub2e4. \ubcf8\ub798 \ubb18\ud638\uac00 \ucc98\uc74c \ub9cc\ub4e4\uc5b4\uc84c\uc744 \ub54c\uc5d0\ub294 \ub2e8\uc9c0 \uc7ac\uc704\ud558\uc5ec \ub098\ub77c\ub97c \ub2e4\uc2a4\ub838\ub2e4\uace0 \ubc1b\ub294 \uac83\uc774 \uc544\ub2c8\ub77c \uad6d\uac00\uc5d0 \ud2b9\ubcc4\ud55c \uacf5\ub355\uc744 \ub0a8\uaca8 \uc5b5\uc870\ucc3d\uc0dd\uc774 \ubaa8\ub450 \uadf8 \uacf5\uc744 \uce58\ud558\ud558\uace0 \uc6b0\ub7ec\ub7ec\ubcfc \ub54c \ube44\ub85c\uc18c \ud0dc\ubb18\uc5d0 \ubd09\uc548\ud558\uace0 \uc62c\ub9b0 \ud638\uce6d\uc774\uc5c8\uc73c\ubbc0\ub85c, \ub098\ub77c\ub97c \ub2e4\uc2a4\ub9b0 \uad70\uc8fc \uac00\uc6b4\ub370\uc11c\ub3c4 \uc18c\uc218\ub9cc\uc774 \ubc1b\uc744 \uc218 \uc788\ub358 \uc601\uc608\ub85c\uc6b4 \uce6d\ud638\uc600\ub2e4. \uadf8\ub7ec\ub098 \ud6c4\ub300\uc5d0 \uac08\uc218\ub85d \uc7ac\uc704\ud55c \ubaa8\ub4e0 \ud669\uc81c\ub4e4\uc744 \ud0dc\ubb18\uc5d0 \ubd09\uc548\ud558\uba74\uc11c \ubb18\ud638\ub3c4 \uac19\uc774 \uc62c\ub9ac\uae30 \uc2dc\uc791\ud558\uc600\ub2e4.", "sentence_answer": "\ud754\ud788 \uc4f0\uace0 \ub110\ub9ac \uc54c\ub824\uc9c4 \ubb18\ud638\ub85c\ub294 \ub098\ub77c\ub97c \uc138\uc6b0\uac70\ub098 \ub098\ub77c\ub97c \uc138\uc6b0\uae30 \uc774\uc804\uc5d0 \uae30\ud2c0\uc744 \ub2e4\uc9c4 \uad70\uc8fc\uc5d0\uac8c \ubd99\uc774\ub294 \ud0dc\uc870(\u592a\u7956)\ub098 \uace0\uc870(\u9ad8\u7956), \ub098\ub77c\uc758 \uae30\ud2c0\uc744 \ub2e4\uc9c4 \ud0dc\uc885(\u592a\u5b97)\uc774\ub098 \uc138\uc870(\u4e16\u7956), \ub098\ub77c\ub97c \ubc1c\uc804\uc2dc\ud0a4\uace0 \uc911\ud765\ud55c \uc138\uc885(\u4e16\u5b97) \ub610\ub294 \uace0\uc885(\u9ad8\u5b97), \uadf8\ub9ac\uace0 \ub098\ub77c\ub97c \uc704\uae30\uc5d0\uc11c \uad6c\ud574\ub0b4\uac70\ub098 \ub9e5\uc774 \ub04a\uacbc\ub358 \uc655\ud1b5\uc744 \ub2e4\uc2dc \ubc14\ub85c\uc7a1\uc740 \uc911\uc885 (\u4e2d\u5b97)", "paragraph_id": "6504691-3-0", "paragraph_question": "question: \uc704\uae30\uc5d0 \ube60\uc9c4 \ub098\ub77c\ub97c \uad6c\ud574\ub0b8 \uc655\uc5d0\uac8c \uc8fc\uc5b4\uc9c0\ub294 \ubb18\ud638\ub294?, context: \ud754\ud788 \uc4f0\uace0 \ub110\ub9ac \uc54c\ub824\uc9c4 \ubb18\ud638\ub85c\ub294 \ub098\ub77c\ub97c \uc138\uc6b0\uac70\ub098 \ub098\ub77c\ub97c \uc138\uc6b0\uae30 \uc774\uc804\uc5d0 \uae30\ud2c0\uc744 \ub2e4\uc9c4 \uad70\uc8fc\uc5d0\uac8c \ubd99\uc774\ub294 \ud0dc\uc870(\u592a\u7956)\ub098 \uace0\uc870(\u9ad8\u7956), \ub098\ub77c\uc758 \uae30\ud2c0\uc744 \ub2e4\uc9c4 \ud0dc\uc885(\u592a\u5b97)\uc774\ub098 \uc138\uc870(\u4e16\u7956), \ub098\ub77c\ub97c \ubc1c\uc804\uc2dc\ud0a4\uace0 \uc911\ud765\ud55c \uc138\uc885(\u4e16\u5b97) \ub610\ub294 \uace0\uc885(\u9ad8\u5b97), \uadf8\ub9ac\uace0 \ub098\ub77c\ub97c \uc704\uae30\uc5d0\uc11c \uad6c\ud574\ub0b4\uac70\ub098 \ub9e5\uc774 \ub04a\uacbc\ub358 \uc655\ud1b5\uc744 \ub2e4\uc2dc \ubc14\ub85c\uc7a1\uc740 \uc911\uc885(\u4e2d\u5b97) \ub4f1\uc774 \uc788\ub2e4. \ubcf8\ub798 \ubb18\ud638\uac00 \ucc98\uc74c \ub9cc\ub4e4\uc5b4\uc84c\uc744 \ub54c\uc5d0\ub294 \ub2e8\uc9c0 \uc7ac\uc704\ud558\uc5ec \ub098\ub77c\ub97c \ub2e4\uc2a4\ub838\ub2e4\uace0 \ubc1b\ub294 \uac83\uc774 \uc544\ub2c8\ub77c \uad6d\uac00\uc5d0 \ud2b9\ubcc4\ud55c \uacf5\ub355\uc744 \ub0a8\uaca8 \uc5b5\uc870\ucc3d\uc0dd\uc774 \ubaa8\ub450 \uadf8 \uacf5\uc744 \uce58\ud558\ud558\uace0 \uc6b0\ub7ec\ub7ec\ubcfc \ub54c \ube44\ub85c\uc18c \ud0dc\ubb18\uc5d0 \ubd09\uc548\ud558\uace0 \uc62c\ub9b0 \ud638\uce6d\uc774\uc5c8\uc73c\ubbc0\ub85c, \ub098\ub77c\ub97c \ub2e4\uc2a4\ub9b0 \uad70\uc8fc \uac00\uc6b4\ub370\uc11c\ub3c4 \uc18c\uc218\ub9cc\uc774 \ubc1b\uc744 \uc218 \uc788\ub358 \uc601\uc608\ub85c\uc6b4 \uce6d\ud638\uc600\ub2e4. \uadf8\ub7ec\ub098 \ud6c4\ub300\uc5d0 \uac08\uc218\ub85d \uc7ac\uc704\ud55c \ubaa8\ub4e0 \ud669\uc81c\ub4e4\uc744 \ud0dc\ubb18\uc5d0 \ubd09\uc548\ud558\uba74\uc11c \ubb18\ud638\ub3c4 \uac19\uc774 \uc62c\ub9ac\uae30 \uc2dc\uc791\ud558\uc600\ub2e4."} {"question": "\uad00\ub3d9\ub300\uc9c0\uc9c4\uc740 \uc5b8\uc81c \ubc1c\uc0dd\ud588\ub098\uc694?", "paragraph": "\uc81c1\ucc28 \uc138\uacc4 \ub300\uc804\uc774 \ub05d\ub098\uace0 \ub098\uc11c \ubd88\ud669\uc774 \uc654\uc744 \ub54c\ub3c4 \ub610 \ub2e4\uc2dc \uacbd\uc601 \uc704\uae30\uc5d0 \ube60\uc84c\ub2e4. \uc774\ub54c\ub294 1919\ub144\uc758 \ub178\ub3d9\uc7c1\uc758, 1922\ub144\uc5d0 \uc6cc\uc2f1\ud134 \uad70\ucd95 \uc870\uc57d, 1923\ub144\uc758 \uad00\ub3d9\ub300\uc9c0\uc9c4\uc774 \uacb9\ucce4\ub2e4. \ub610\ud55c 1927\ub144 \uae08\uc735 \uacf5\ud669\uc5d0 \uc758\ud574 \uc8fc\uac70\ub798 \uc740\ud589 15\uac1c \uc740\ud589\uc774 \uc784\uc2dc \ud734\uc5c5\ud558\uba74\uc11c \uc790\uae08 \uc0ac\uc815\uc774 \uae09\uaca9\ud788 \uc545\ud654\ud588\ub2e4. \uc815\ubd80\uc640 \uace0\ubca0 \uc2dc\uc758 \uc6d0\uc870\ub85c \uc9c1\uc6d0 \uc57d 3,500\uba85\uc758 \ud574\uace0 \ud734\uc5c5, \uc784\uc6d0 \ubcf4\uc218 \uc0ad\uac10, \ub9c8\uce20\uce74\ud0c0 \uac00\ubb38\uacfc \uac00\uc640\uc0ac\ud0a4 \uac00\ubb38\uc5d0\uc11c \uc0ac\uc7ac\ub97c \ucd9c\uc5f0\ud558\ub294 \ub4f1\uc744 \ud1b5\ud574 \uacbd\uc601 \uc7ac\uac74\uc744 \uc2dc\ub3c4\ud588\ub2e4. \ub610\ud55c \ud6a8\uace0 \uacf5\uc7a5\uc744 \ubd84\ub9ac\ud558\uc5ec \u3008\uac00\uc640\uc0ac\ud0a4 \ucc28\ub7c9\u3009\uc73c\ub85c \ub3c5\ub9bd\uc801 \uc800\ub2f9\uad8c\uc744 \uc124\uc815\ud558\uace0, \ubcf8\uc0ac \uacf5\uc7a5\uacfc \u847a\u5408\uacf5\uc7a5\uc744 \ub2f4\ubcf4\ub85c \ub300\ucd9c\uc744 \ubc1b\uc558\ub2e4. \uc77c\ubcf8 \uc81c\uad6d \ud574\uad70\ub3c4 \uce78\uc138\uc774 \ubcf8\ubd80\uc5d0 \uc784\uc2dc\ud568\uc120 \uac74\uc870\ubd80\ub97c \uc124\uce58\ud558\uace0 \ud568\uc815 \uac74\uc870\ub97c \uacc4\uc18d\ud560 \uc218 \uc788\ub3c4\ub85d \ub3c4\uc654\ub2e4.", "answer": "1923\ub144", "sentence": "\uc774\ub54c\ub294 1919\ub144\uc758 \ub178\ub3d9\uc7c1\uc758, 1922\ub144\uc5d0 \uc6cc\uc2f1\ud134 \uad70\ucd95 \uc870\uc57d, 1923\ub144 \uc758 \uad00\ub3d9\ub300\uc9c0\uc9c4\uc774 \uacb9\ucce4\ub2e4.", "paragraph_sentence": "\uc81c1\ucc28 \uc138\uacc4 \ub300\uc804\uc774 \ub05d\ub098\uace0 \ub098\uc11c \ubd88\ud669\uc774 \uc654\uc744 \ub54c\ub3c4 \ub610 \ub2e4\uc2dc \uacbd\uc601 \uc704\uae30\uc5d0 \ube60\uc84c\ub2e4. \uc774\ub54c\ub294 1919\ub144\uc758 \ub178\ub3d9\uc7c1\uc758, 1922\ub144\uc5d0 \uc6cc\uc2f1\ud134 \uad70\ucd95 \uc870\uc57d, 1923\ub144 \uc758 \uad00\ub3d9\ub300\uc9c0\uc9c4\uc774 \uacb9\ucce4\ub2e4. \ub610\ud55c 1927\ub144 \uae08\uc735 \uacf5\ud669\uc5d0 \uc758\ud574 \uc8fc\uac70\ub798 \uc740\ud589 15\uac1c \uc740\ud589\uc774 \uc784\uc2dc \ud734\uc5c5\ud558\uba74\uc11c \uc790\uae08 \uc0ac\uc815\uc774 \uae09\uaca9\ud788 \uc545\ud654\ud588\ub2e4. \uc815\ubd80\uc640 \uace0\ubca0 \uc2dc\uc758 \uc6d0\uc870\ub85c \uc9c1\uc6d0 \uc57d 3,500\uba85\uc758 \ud574\uace0 \ud734\uc5c5, \uc784\uc6d0 \ubcf4\uc218 \uc0ad\uac10, \ub9c8\uce20\uce74\ud0c0 \uac00\ubb38\uacfc \uac00\uc640\uc0ac\ud0a4 \uac00\ubb38\uc5d0\uc11c \uc0ac\uc7ac\ub97c \ucd9c\uc5f0\ud558\ub294 \ub4f1\uc744 \ud1b5\ud574 \uacbd\uc601 \uc7ac\uac74\uc744 \uc2dc\ub3c4\ud588\ub2e4. \ub610\ud55c \ud6a8\uace0 \uacf5\uc7a5\uc744 \ubd84\ub9ac\ud558\uc5ec \u3008\uac00\uc640\uc0ac\ud0a4 \ucc28\ub7c9\u3009\uc73c\ub85c \ub3c5\ub9bd\uc801 \uc800\ub2f9\uad8c\uc744 \uc124\uc815\ud558\uace0, \ubcf8\uc0ac \uacf5\uc7a5\uacfc \u847a\u5408\uacf5\uc7a5\uc744 \ub2f4\ubcf4\ub85c \ub300\ucd9c\uc744 \ubc1b\uc558\ub2e4. \uc77c\ubcf8 \uc81c\uad6d \ud574\uad70\ub3c4 \uce78\uc138\uc774 \ubcf8\ubd80\uc5d0 \uc784\uc2dc\ud568\uc120 \uac74\uc870\ubd80\ub97c \uc124\uce58\ud558\uace0 \ud568\uc815 \uac74\uc870\ub97c \uacc4\uc18d\ud560 \uc218 \uc788\ub3c4\ub85d \ub3c4\uc654\ub2e4.", "paragraph_answer": "\uc81c1\ucc28 \uc138\uacc4 \ub300\uc804\uc774 \ub05d\ub098\uace0 \ub098\uc11c \ubd88\ud669\uc774 \uc654\uc744 \ub54c\ub3c4 \ub610 \ub2e4\uc2dc \uacbd\uc601 \uc704\uae30\uc5d0 \ube60\uc84c\ub2e4. \uc774\ub54c\ub294 1919\ub144\uc758 \ub178\ub3d9\uc7c1\uc758, 1922\ub144\uc5d0 \uc6cc\uc2f1\ud134 \uad70\ucd95 \uc870\uc57d, 1923\ub144 \uc758 \uad00\ub3d9\ub300\uc9c0\uc9c4\uc774 \uacb9\ucce4\ub2e4. \ub610\ud55c 1927\ub144 \uae08\uc735 \uacf5\ud669\uc5d0 \uc758\ud574 \uc8fc\uac70\ub798 \uc740\ud589 15\uac1c \uc740\ud589\uc774 \uc784\uc2dc \ud734\uc5c5\ud558\uba74\uc11c \uc790\uae08 \uc0ac\uc815\uc774 \uae09\uaca9\ud788 \uc545\ud654\ud588\ub2e4. \uc815\ubd80\uc640 \uace0\ubca0 \uc2dc\uc758 \uc6d0\uc870\ub85c \uc9c1\uc6d0 \uc57d 3,500\uba85\uc758 \ud574\uace0 \ud734\uc5c5, \uc784\uc6d0 \ubcf4\uc218 \uc0ad\uac10, \ub9c8\uce20\uce74\ud0c0 \uac00\ubb38\uacfc \uac00\uc640\uc0ac\ud0a4 \uac00\ubb38\uc5d0\uc11c \uc0ac\uc7ac\ub97c \ucd9c\uc5f0\ud558\ub294 \ub4f1\uc744 \ud1b5\ud574 \uacbd\uc601 \uc7ac\uac74\uc744 \uc2dc\ub3c4\ud588\ub2e4. \ub610\ud55c \ud6a8\uace0 \uacf5\uc7a5\uc744 \ubd84\ub9ac\ud558\uc5ec \u3008\uac00\uc640\uc0ac\ud0a4 \ucc28\ub7c9\u3009\uc73c\ub85c \ub3c5\ub9bd\uc801 \uc800\ub2f9\uad8c\uc744 \uc124\uc815\ud558\uace0, \ubcf8\uc0ac \uacf5\uc7a5\uacfc \u847a\u5408\uacf5\uc7a5\uc744 \ub2f4\ubcf4\ub85c \ub300\ucd9c\uc744 \ubc1b\uc558\ub2e4. \uc77c\ubcf8 \uc81c\uad6d \ud574\uad70\ub3c4 \uce78\uc138\uc774 \ubcf8\ubd80\uc5d0 \uc784\uc2dc\ud568\uc120 \uac74\uc870\ubd80\ub97c \uc124\uce58\ud558\uace0 \ud568\uc815 \uac74\uc870\ub97c \uacc4\uc18d\ud560 \uc218 \uc788\ub3c4\ub85d \ub3c4\uc654\ub2e4.", "sentence_answer": "\uc774\ub54c\ub294 1919\ub144\uc758 \ub178\ub3d9\uc7c1\uc758, 1922\ub144\uc5d0 \uc6cc\uc2f1\ud134 \uad70\ucd95 \uc870\uc57d, 1923\ub144 \uc758 \uad00\ub3d9\ub300\uc9c0\uc9c4\uc774 \uacb9\ucce4\ub2e4.", "paragraph_id": "6472155-2-2", "paragraph_question": "question: \uad00\ub3d9\ub300\uc9c0\uc9c4\uc740 \uc5b8\uc81c \ubc1c\uc0dd\ud588\ub098\uc694?, context: \uc81c1\ucc28 \uc138\uacc4 \ub300\uc804\uc774 \ub05d\ub098\uace0 \ub098\uc11c \ubd88\ud669\uc774 \uc654\uc744 \ub54c\ub3c4 \ub610 \ub2e4\uc2dc \uacbd\uc601 \uc704\uae30\uc5d0 \ube60\uc84c\ub2e4. \uc774\ub54c\ub294 1919\ub144\uc758 \ub178\ub3d9\uc7c1\uc758, 1922\ub144\uc5d0 \uc6cc\uc2f1\ud134 \uad70\ucd95 \uc870\uc57d, 1923\ub144\uc758 \uad00\ub3d9\ub300\uc9c0\uc9c4\uc774 \uacb9\ucce4\ub2e4. \ub610\ud55c 1927\ub144 \uae08\uc735 \uacf5\ud669\uc5d0 \uc758\ud574 \uc8fc\uac70\ub798 \uc740\ud589 15\uac1c \uc740\ud589\uc774 \uc784\uc2dc \ud734\uc5c5\ud558\uba74\uc11c \uc790\uae08 \uc0ac\uc815\uc774 \uae09\uaca9\ud788 \uc545\ud654\ud588\ub2e4. \uc815\ubd80\uc640 \uace0\ubca0 \uc2dc\uc758 \uc6d0\uc870\ub85c \uc9c1\uc6d0 \uc57d 3,500\uba85\uc758 \ud574\uace0 \ud734\uc5c5, \uc784\uc6d0 \ubcf4\uc218 \uc0ad\uac10, \ub9c8\uce20\uce74\ud0c0 \uac00\ubb38\uacfc \uac00\uc640\uc0ac\ud0a4 \uac00\ubb38\uc5d0\uc11c \uc0ac\uc7ac\ub97c \ucd9c\uc5f0\ud558\ub294 \ub4f1\uc744 \ud1b5\ud574 \uacbd\uc601 \uc7ac\uac74\uc744 \uc2dc\ub3c4\ud588\ub2e4. \ub610\ud55c \ud6a8\uace0 \uacf5\uc7a5\uc744 \ubd84\ub9ac\ud558\uc5ec \u3008\uac00\uc640\uc0ac\ud0a4 \ucc28\ub7c9\u3009\uc73c\ub85c \ub3c5\ub9bd\uc801 \uc800\ub2f9\uad8c\uc744 \uc124\uc815\ud558\uace0, \ubcf8\uc0ac \uacf5\uc7a5\uacfc \u847a\u5408\uacf5\uc7a5\uc744 \ub2f4\ubcf4\ub85c \ub300\ucd9c\uc744 \ubc1b\uc558\ub2e4. \uc77c\ubcf8 \uc81c\uad6d \ud574\uad70\ub3c4 \uce78\uc138\uc774 \ubcf8\ubd80\uc5d0 \uc784\uc2dc\ud568\uc120 \uac74\uc870\ubd80\ub97c \uc124\uce58\ud558\uace0 \ud568\uc815 \uac74\uc870\ub97c \uacc4\uc18d\ud560 \uc218 \uc788\ub3c4\ub85d \ub3c4\uc654\ub2e4."} {"question": "\uc804\ud6c4 \ubd81\ubbf8\uc5d0 \ub0a8\uaca8\uc9c4 \uc601\uad6d\uad70\uc758 \uc218\ub294?", "paragraph": "\uc601\uad6d \ubd81\ubbf8 \ucd1d\uc0ac\ub839\uad00 \uc560\uba38\uc2a4\ud2b8 \uc7a5\uad70\uc740 \ud1a0\ucc29 \uc778\ub514\uc5b8\uc5d0 \uad00\ub9ac \uc815\ucc45\uc5d0 \ub300\ud55c \uc804\ubc18\uc801\uc778 \ucc45\uc784\uc744 \uc9c0\uace0 \uc788\uc5c8\uc73c\uba70, \uadf8\uac83\uc5d0\ub294 \uad70\uc0ac\uc801 \uc5c5\ubb34\uc640 \ubaa8\ud53c \ubb34\uc5ed \uaddc\uc81c\uc640\ub3c4 \uad00\ub828\uc774 \uc788\uc5c8\ub2e4. \uc560\uba38\uc2a4\ud2b8\ub294 \ud504\ub791\uc2a4\uac00 \ubb34\ub300\uc5d0\uc11c \uc0ac\ub77c\uc9c0\uba74 \uc778\ub514\uc5b8\uc740 \uc601\uad6d\uc758 \uc9c0\ubc30\ub97c \ubc1b\uc544\ub4e4\uc77c \uc218\ubc16\uc5d0 \uc5c6\uc744 \uac83\uc774\ub77c\uace0 \ubbff\uc5c8\ub2e4. \uc560\uba38\uc2a4\ud2b8\ub294 \ub610\ud55c \uc778\ub514\uc5b8\uc774 \uc601\uad6d\uad70\uc5d0 \ub300\ud574\uc11c \uc5b4\ub5a0\ud55c \uc800\ud56d\ub3c4 \ud560 \uc218 \uc5c6\uc744 \uac70\ub77c\uace0 \ubbff\uc5c8\ub2e4. \uadf8\ub798\uc11c \ubd81\ubbf8\uc5d0\uc11c \uadf8\uc758 \ud718\ud558\uc5d0 8,000\uba85 \uc911 \ubd88\uacfc 500\uba85 \uc815\ub3c4\ub97c \uc804\uc7c1\uc774 \ub05d\ub09c \uc774 \uc9c0\uc5ed\uc5d0 \uc8fc\ub454\uc2dc\ucf30\ub2e4. \uc560\uba38\uc2a4\ud2b8\uc640 \ub514\ud2b8\ub85c\uc774\ud2b8 \uc694\uc0c8\uc758 \uc9c0\ud718\uad00 \ud5e8\ub9ac \uae00\ub798\ub4dc\uc708 \uc18c\ub839 \uac19\uc740 \uc7a5\uad50\ub4e4\uc740 \uc778\ub514\uc5b8\uc5d0 \ub300\ud55c \uacbd\uba78\uc744 \uc228\uae30\ub824 \ud558\uc9c0 \uc54a\uc558\ub2e4. \ubd09\uae30\uc5d0 \ucc38\uac00\ud55c \uc778\ub514\uc5b8 \ubd80\uc871\uc740 \u201c\uae00\ub798\ub4dc\uc708\uc774 \uc6b0\ub9ac\ub97c \ub178\uc608\ub098 \uac1c\ucc98\ub7fc \ub2e4\ub8ec\ub2e4\u201d\uace0 \uc790\uc8fc \ubd88\ud3c9\uc744 \ud588\ub2e4.", "answer": "500\uba85 \uc815\ub3c4", "sentence": "\uadf8\ub798\uc11c \ubd81\ubbf8\uc5d0\uc11c \uadf8\uc758 \ud718\ud558\uc5d0 8,000\uba85 \uc911 \ubd88\uacfc 500\uba85 \uc815\ub3c4 \ub97c \uc804\uc7c1\uc774 \ub05d\ub09c \uc774 \uc9c0\uc5ed\uc5d0 \uc8fc\ub454\uc2dc\ucf30\ub2e4.", "paragraph_sentence": "\uc601\uad6d \ubd81\ubbf8 \ucd1d\uc0ac\ub839\uad00 \uc560\uba38\uc2a4\ud2b8 \uc7a5\uad70\uc740 \ud1a0\ucc29 \uc778\ub514\uc5b8\uc5d0 \uad00\ub9ac \uc815\ucc45\uc5d0 \ub300\ud55c \uc804\ubc18\uc801\uc778 \ucc45\uc784\uc744 \uc9c0\uace0 \uc788\uc5c8\uc73c\uba70, \uadf8\uac83\uc5d0\ub294 \uad70\uc0ac\uc801 \uc5c5\ubb34\uc640 \ubaa8\ud53c \ubb34\uc5ed \uaddc\uc81c\uc640\ub3c4 \uad00\ub828\uc774 \uc788\uc5c8\ub2e4. \uc560\uba38\uc2a4\ud2b8\ub294 \ud504\ub791\uc2a4\uac00 \ubb34\ub300\uc5d0\uc11c \uc0ac\ub77c\uc9c0\uba74 \uc778\ub514\uc5b8\uc740 \uc601\uad6d\uc758 \uc9c0\ubc30\ub97c \ubc1b\uc544\ub4e4\uc77c \uc218\ubc16\uc5d0 \uc5c6\uc744 \uac83\uc774\ub77c\uace0 \ubbff\uc5c8\ub2e4. \uc560\uba38\uc2a4\ud2b8\ub294 \ub610\ud55c \uc778\ub514\uc5b8\uc774 \uc601\uad6d\uad70\uc5d0 \ub300\ud574\uc11c \uc5b4\ub5a0\ud55c \uc800\ud56d\ub3c4 \ud560 \uc218 \uc5c6\uc744 \uac70\ub77c\uace0 \ubbff\uc5c8\ub2e4. \uadf8\ub798\uc11c \ubd81\ubbf8\uc5d0\uc11c \uadf8\uc758 \ud718\ud558\uc5d0 8,000\uba85 \uc911 \ubd88\uacfc 500\uba85 \uc815\ub3c4 \ub97c \uc804\uc7c1\uc774 \ub05d\ub09c \uc774 \uc9c0\uc5ed\uc5d0 \uc8fc\ub454\uc2dc\ucf30\ub2e4. \uc560\uba38\uc2a4\ud2b8\uc640 \ub514\ud2b8\ub85c\uc774\ud2b8 \uc694\uc0c8\uc758 \uc9c0\ud718\uad00 \ud5e8\ub9ac \uae00\ub798\ub4dc\uc708 \uc18c\ub839 \uac19\uc740 \uc7a5\uad50\ub4e4\uc740 \uc778\ub514\uc5b8\uc5d0 \ub300\ud55c \uacbd\uba78\uc744 \uc228\uae30\ub824 \ud558\uc9c0 \uc54a\uc558\ub2e4. \ubd09\uae30\uc5d0 \ucc38\uac00\ud55c \uc778\ub514\uc5b8 \ubd80\uc871\uc740 \u201c\uae00\ub798\ub4dc\uc708\uc774 \uc6b0\ub9ac\ub97c \ub178\uc608\ub098 \uac1c\ucc98\ub7fc \ub2e4\ub8ec\ub2e4\u201d\uace0 \uc790\uc8fc \ubd88\ud3c9\uc744 \ud588\ub2e4.", "paragraph_answer": "\uc601\uad6d \ubd81\ubbf8 \ucd1d\uc0ac\ub839\uad00 \uc560\uba38\uc2a4\ud2b8 \uc7a5\uad70\uc740 \ud1a0\ucc29 \uc778\ub514\uc5b8\uc5d0 \uad00\ub9ac \uc815\ucc45\uc5d0 \ub300\ud55c \uc804\ubc18\uc801\uc778 \ucc45\uc784\uc744 \uc9c0\uace0 \uc788\uc5c8\uc73c\uba70, \uadf8\uac83\uc5d0\ub294 \uad70\uc0ac\uc801 \uc5c5\ubb34\uc640 \ubaa8\ud53c \ubb34\uc5ed \uaddc\uc81c\uc640\ub3c4 \uad00\ub828\uc774 \uc788\uc5c8\ub2e4. \uc560\uba38\uc2a4\ud2b8\ub294 \ud504\ub791\uc2a4\uac00 \ubb34\ub300\uc5d0\uc11c \uc0ac\ub77c\uc9c0\uba74 \uc778\ub514\uc5b8\uc740 \uc601\uad6d\uc758 \uc9c0\ubc30\ub97c \ubc1b\uc544\ub4e4\uc77c \uc218\ubc16\uc5d0 \uc5c6\uc744 \uac83\uc774\ub77c\uace0 \ubbff\uc5c8\ub2e4. \uc560\uba38\uc2a4\ud2b8\ub294 \ub610\ud55c \uc778\ub514\uc5b8\uc774 \uc601\uad6d\uad70\uc5d0 \ub300\ud574\uc11c \uc5b4\ub5a0\ud55c \uc800\ud56d\ub3c4 \ud560 \uc218 \uc5c6\uc744 \uac70\ub77c\uace0 \ubbff\uc5c8\ub2e4. \uadf8\ub798\uc11c \ubd81\ubbf8\uc5d0\uc11c \uadf8\uc758 \ud718\ud558\uc5d0 8,000\uba85 \uc911 \ubd88\uacfc 500\uba85 \uc815\ub3c4 \ub97c \uc804\uc7c1\uc774 \ub05d\ub09c \uc774 \uc9c0\uc5ed\uc5d0 \uc8fc\ub454\uc2dc\ucf30\ub2e4. \uc560\uba38\uc2a4\ud2b8\uc640 \ub514\ud2b8\ub85c\uc774\ud2b8 \uc694\uc0c8\uc758 \uc9c0\ud718\uad00 \ud5e8\ub9ac \uae00\ub798\ub4dc\uc708 \uc18c\ub839 \uac19\uc740 \uc7a5\uad50\ub4e4\uc740 \uc778\ub514\uc5b8\uc5d0 \ub300\ud55c \uacbd\uba78\uc744 \uc228\uae30\ub824 \ud558\uc9c0 \uc54a\uc558\ub2e4. \ubd09\uae30\uc5d0 \ucc38\uac00\ud55c \uc778\ub514\uc5b8 \ubd80\uc871\uc740 \u201c\uae00\ub798\ub4dc\uc708\uc774 \uc6b0\ub9ac\ub97c \ub178\uc608\ub098 \uac1c\ucc98\ub7fc \ub2e4\ub8ec\ub2e4\u201d\uace0 \uc790\uc8fc \ubd88\ud3c9\uc744 \ud588\ub2e4.", "sentence_answer": "\uadf8\ub798\uc11c \ubd81\ubbf8\uc5d0\uc11c \uadf8\uc758 \ud718\ud558\uc5d0 8,000\uba85 \uc911 \ubd88\uacfc 500\uba85 \uc815\ub3c4 \ub97c \uc804\uc7c1\uc774 \ub05d\ub09c \uc774 \uc9c0\uc5ed\uc5d0 \uc8fc\ub454\uc2dc\ucf30\ub2e4.", "paragraph_id": "6567978-3-1", "paragraph_question": "question: \uc804\ud6c4 \ubd81\ubbf8\uc5d0 \ub0a8\uaca8\uc9c4 \uc601\uad6d\uad70\uc758 \uc218\ub294?, context: \uc601\uad6d \ubd81\ubbf8 \ucd1d\uc0ac\ub839\uad00 \uc560\uba38\uc2a4\ud2b8 \uc7a5\uad70\uc740 \ud1a0\ucc29 \uc778\ub514\uc5b8\uc5d0 \uad00\ub9ac \uc815\ucc45\uc5d0 \ub300\ud55c \uc804\ubc18\uc801\uc778 \ucc45\uc784\uc744 \uc9c0\uace0 \uc788\uc5c8\uc73c\uba70, \uadf8\uac83\uc5d0\ub294 \uad70\uc0ac\uc801 \uc5c5\ubb34\uc640 \ubaa8\ud53c \ubb34\uc5ed \uaddc\uc81c\uc640\ub3c4 \uad00\ub828\uc774 \uc788\uc5c8\ub2e4. \uc560\uba38\uc2a4\ud2b8\ub294 \ud504\ub791\uc2a4\uac00 \ubb34\ub300\uc5d0\uc11c \uc0ac\ub77c\uc9c0\uba74 \uc778\ub514\uc5b8\uc740 \uc601\uad6d\uc758 \uc9c0\ubc30\ub97c \ubc1b\uc544\ub4e4\uc77c \uc218\ubc16\uc5d0 \uc5c6\uc744 \uac83\uc774\ub77c\uace0 \ubbff\uc5c8\ub2e4. \uc560\uba38\uc2a4\ud2b8\ub294 \ub610\ud55c \uc778\ub514\uc5b8\uc774 \uc601\uad6d\uad70\uc5d0 \ub300\ud574\uc11c \uc5b4\ub5a0\ud55c \uc800\ud56d\ub3c4 \ud560 \uc218 \uc5c6\uc744 \uac70\ub77c\uace0 \ubbff\uc5c8\ub2e4. \uadf8\ub798\uc11c \ubd81\ubbf8\uc5d0\uc11c \uadf8\uc758 \ud718\ud558\uc5d0 8,000\uba85 \uc911 \ubd88\uacfc 500\uba85 \uc815\ub3c4\ub97c \uc804\uc7c1\uc774 \ub05d\ub09c \uc774 \uc9c0\uc5ed\uc5d0 \uc8fc\ub454\uc2dc\ucf30\ub2e4. \uc560\uba38\uc2a4\ud2b8\uc640 \ub514\ud2b8\ub85c\uc774\ud2b8 \uc694\uc0c8\uc758 \uc9c0\ud718\uad00 \ud5e8\ub9ac \uae00\ub798\ub4dc\uc708 \uc18c\ub839 \uac19\uc740 \uc7a5\uad50\ub4e4\uc740 \uc778\ub514\uc5b8\uc5d0 \ub300\ud55c \uacbd\uba78\uc744 \uc228\uae30\ub824 \ud558\uc9c0 \uc54a\uc558\ub2e4. \ubd09\uae30\uc5d0 \ucc38\uac00\ud55c \uc778\ub514\uc5b8 \ubd80\uc871\uc740 \u201c\uae00\ub798\ub4dc\uc708\uc774 \uc6b0\ub9ac\ub97c \ub178\uc608\ub098 \uac1c\ucc98\ub7fc \ub2e4\ub8ec\ub2e4\u201d\uace0 \uc790\uc8fc \ubd88\ud3c9\uc744 \ud588\ub2e4."} {"question": "\uc21c\ucc9c\ub300\ud559\uad50 \ub3d9\uc544\ub9ac\uac00 \ub3d9\uc544\ub9ac\ubc29\uc744 \uc5bb\uae30 \uc704\ud574 \uc5b4\ub5a4 \ub2e8\uccb4\uc5d0 \uc120\ubc1c\ub418\uc5b4\uc57c \ud558\ub294\uac00?", "paragraph": "\uc21c\ucc9c\ub300\ud559\uad50 \ub3d9\uc544\ub9ac\ub294 \ud06c\uac8c \ud559\uc220\ubd84\uacfc, \ubb38\uc608\ubd84\uacfc, \ucde8\uccb4\ubd84\uacfc, \uc885\uad50 \ubd84\uacfc\ub85c \ub098\ub25c\ub2e4. \ud559\uc220\ubd84\uacfc\uc5d0\ub294 \ub2e4\ubb3c, \uc0ac\ub791\ubc29, SOS, \uc778\ud130\ub7ec\ubc45, RCY, \uc791\uc740\ub098\ubb34, YMCA, GES, Youth-JC, \ub108\uc640\ub2e4\ub824, \ub85c\ud0c0\ub809\uc2a4, \ubd09\ub3d9\uc774, \uc190\uc9d3\ubc1c\uc9d3, \uc608\uadf8\ub9ac\ub098, \uc4f0\ub2f4\uc4f0\ub2f4, IYF, \ud558\ubaa8\ub2c8\uac00 \uc788\uc73c\uba70 \ubb38\uc608\ubd84\uacfc\uc5d0\ub294 \ud55c\uc6b8, \ud5a5\ub9bc\ub2e4\ud68c, \uc0ac\uc9c4\uc608\uc220\ud68c, \uc544\ub791\ud6fc\uc988, \ud06c\ub808\ud30c\uc2a4, \uba54\uc544\ub9ac, \ubba4\uc988\uadf8\ub9b0\ube44, \uc11d\uacf5, \ubaa8\uaf2c\uc9c0, P&B, \ubcf4\uc774\uc2a4\ud399\ud130, \ub9ac\uc2a4\ud399, \ubb38\uc678\ud55c\uc774 \uc788\uace0 \ucde8\uccb4\ubd84\uacfc\uc5d0\ub294 UCDC, \ud398\ub2ec, \uc0b0\uc545\ud68c, \ub2e4\uc774\uc544\ubaac\uc988, \uc778\ud130\uc149\ud2b8, \uad74\ub9ac\uc138, \ud0dd\uacac, \ud5a5\ub9bc\ud14c\ub2c8\uc2a4\uac00 \uc788\uc73c\uba70 \uc885\uad50\ubd84\uacfc\uc5d0\ub294 CCC, DSM, SFC, \ub124\ube44\uac8c\uc774\ud1a0, \ube44\ud30c\uc640\uc218\uae08, \uc778\ud130\ucf65, ESF, \uc5f0 \ub4f1\uc774 \uc788\ub2e4. \uc774 \ubc16\uc5d0\ub3c4 \ub354 \ub9ce\uc740 \ub3d9\uc544\ub9ac\uac00 \uc874\uc7ac\ud558\uba70, \ub3d9\uc544\ub9ac \ubc29\uc744 \uc5bb\uae30 \uc704\ud574\uc11c\ub294 \ub3d9\uc544\ub9ac\ud68c\uc5d0 \uc758\ud574 \uc120\ubc1c\uc774 \ub418\uc5b4\uc57c \ud55c\ub2e4.", "answer": "\ub3d9\uc544\ub9ac\ud68c", "sentence": "\uc774 \ubc16\uc5d0\ub3c4 \ub354 \ub9ce\uc740 \ub3d9\uc544\ub9ac\uac00 \uc874\uc7ac\ud558\uba70, \ub3d9\uc544\ub9ac \ubc29\uc744 \uc5bb\uae30 \uc704\ud574\uc11c\ub294 \ub3d9\uc544\ub9ac\ud68c \uc5d0 \uc758\ud574 \uc120\ubc1c\uc774 \ub418\uc5b4\uc57c \ud55c\ub2e4.", "paragraph_sentence": "\uc21c\ucc9c\ub300\ud559\uad50 \ub3d9\uc544\ub9ac\ub294 \ud06c\uac8c \ud559\uc220\ubd84\uacfc, \ubb38\uc608\ubd84\uacfc, \ucde8\uccb4\ubd84\uacfc, \uc885\uad50 \ubd84\uacfc\ub85c \ub098\ub25c\ub2e4. \ud559\uc220\ubd84\uacfc\uc5d0\ub294 \ub2e4\ubb3c, \uc0ac\ub791\ubc29, SOS, \uc778\ud130\ub7ec\ubc45, RCY, \uc791\uc740\ub098\ubb34, YMCA, GES, Youth-JC, \ub108\uc640\ub2e4\ub824, \ub85c\ud0c0\ub809\uc2a4, \ubd09\ub3d9\uc774, \uc190\uc9d3\ubc1c\uc9d3, \uc608\uadf8\ub9ac\ub098, \uc4f0\ub2f4\uc4f0\ub2f4, IYF, \ud558\ubaa8\ub2c8\uac00 \uc788\uc73c\uba70 \ubb38\uc608\ubd84\uacfc\uc5d0\ub294 \ud55c\uc6b8, \ud5a5\ub9bc\ub2e4\ud68c, \uc0ac\uc9c4\uc608\uc220\ud68c, \uc544\ub791\ud6fc\uc988, \ud06c\ub808\ud30c\uc2a4, \uba54\uc544\ub9ac, \ubba4\uc988\uadf8\ub9b0\ube44, \uc11d\uacf5, \ubaa8\uaf2c\uc9c0, P&B, \ubcf4\uc774\uc2a4\ud399\ud130, \ub9ac\uc2a4\ud399, \ubb38\uc678\ud55c\uc774 \uc788\uace0 \ucde8\uccb4\ubd84\uacfc\uc5d0\ub294 UCDC, \ud398\ub2ec, \uc0b0\uc545\ud68c, \ub2e4\uc774\uc544\ubaac\uc988, \uc778\ud130\uc149\ud2b8, \uad74\ub9ac\uc138, \ud0dd\uacac, \ud5a5\ub9bc\ud14c\ub2c8\uc2a4\uac00 \uc788\uc73c\uba70 \uc885\uad50\ubd84\uacfc\uc5d0\ub294 CCC, DSM, SFC, \ub124\ube44\uac8c\uc774\ud1a0, \ube44\ud30c\uc640\uc218\uae08, \uc778\ud130\ucf65, ESF, \uc5f0 \ub4f1\uc774 \uc788\ub2e4. \uc774 \ubc16\uc5d0\ub3c4 \ub354 \ub9ce\uc740 \ub3d9\uc544\ub9ac\uac00 \uc874\uc7ac\ud558\uba70, \ub3d9\uc544\ub9ac \ubc29\uc744 \uc5bb\uae30 \uc704\ud574\uc11c\ub294 \ub3d9\uc544\ub9ac\ud68c \uc5d0 \uc758\ud574 \uc120\ubc1c\uc774 \ub418\uc5b4\uc57c \ud55c\ub2e4. ", "paragraph_answer": "\uc21c\ucc9c\ub300\ud559\uad50 \ub3d9\uc544\ub9ac\ub294 \ud06c\uac8c \ud559\uc220\ubd84\uacfc, \ubb38\uc608\ubd84\uacfc, \ucde8\uccb4\ubd84\uacfc, \uc885\uad50 \ubd84\uacfc\ub85c \ub098\ub25c\ub2e4. \ud559\uc220\ubd84\uacfc\uc5d0\ub294 \ub2e4\ubb3c, \uc0ac\ub791\ubc29, SOS, \uc778\ud130\ub7ec\ubc45, RCY, \uc791\uc740\ub098\ubb34, YMCA, GES, Youth-JC, \ub108\uc640\ub2e4\ub824, \ub85c\ud0c0\ub809\uc2a4, \ubd09\ub3d9\uc774, \uc190\uc9d3\ubc1c\uc9d3, \uc608\uadf8\ub9ac\ub098, \uc4f0\ub2f4\uc4f0\ub2f4, IYF, \ud558\ubaa8\ub2c8\uac00 \uc788\uc73c\uba70 \ubb38\uc608\ubd84\uacfc\uc5d0\ub294 \ud55c\uc6b8, \ud5a5\ub9bc\ub2e4\ud68c, \uc0ac\uc9c4\uc608\uc220\ud68c, \uc544\ub791\ud6fc\uc988, \ud06c\ub808\ud30c\uc2a4, \uba54\uc544\ub9ac, \ubba4\uc988\uadf8\ub9b0\ube44, \uc11d\uacf5, \ubaa8\uaf2c\uc9c0, P&B, \ubcf4\uc774\uc2a4\ud399\ud130, \ub9ac\uc2a4\ud399, \ubb38\uc678\ud55c\uc774 \uc788\uace0 \ucde8\uccb4\ubd84\uacfc\uc5d0\ub294 UCDC, \ud398\ub2ec, \uc0b0\uc545\ud68c, \ub2e4\uc774\uc544\ubaac\uc988, \uc778\ud130\uc149\ud2b8, \uad74\ub9ac\uc138, \ud0dd\uacac, \ud5a5\ub9bc\ud14c\ub2c8\uc2a4\uac00 \uc788\uc73c\uba70 \uc885\uad50\ubd84\uacfc\uc5d0\ub294 CCC, DSM, SFC, \ub124\ube44\uac8c\uc774\ud1a0, \ube44\ud30c\uc640\uc218\uae08, \uc778\ud130\ucf65, ESF, \uc5f0 \ub4f1\uc774 \uc788\ub2e4. \uc774 \ubc16\uc5d0\ub3c4 \ub354 \ub9ce\uc740 \ub3d9\uc544\ub9ac\uac00 \uc874\uc7ac\ud558\uba70, \ub3d9\uc544\ub9ac \ubc29\uc744 \uc5bb\uae30 \uc704\ud574\uc11c\ub294 \ub3d9\uc544\ub9ac\ud68c \uc5d0 \uc758\ud574 \uc120\ubc1c\uc774 \ub418\uc5b4\uc57c \ud55c\ub2e4.", "sentence_answer": "\uc774 \ubc16\uc5d0\ub3c4 \ub354 \ub9ce\uc740 \ub3d9\uc544\ub9ac\uac00 \uc874\uc7ac\ud558\uba70, \ub3d9\uc544\ub9ac \ubc29\uc744 \uc5bb\uae30 \uc704\ud574\uc11c\ub294 \ub3d9\uc544\ub9ac\ud68c \uc5d0 \uc758\ud574 \uc120\ubc1c\uc774 \ub418\uc5b4\uc57c \ud55c\ub2e4.", "paragraph_id": "6519025-11-2", "paragraph_question": "question: \uc21c\ucc9c\ub300\ud559\uad50 \ub3d9\uc544\ub9ac\uac00 \ub3d9\uc544\ub9ac\ubc29\uc744 \uc5bb\uae30 \uc704\ud574 \uc5b4\ub5a4 \ub2e8\uccb4\uc5d0 \uc120\ubc1c\ub418\uc5b4\uc57c \ud558\ub294\uac00?, context: \uc21c\ucc9c\ub300\ud559\uad50 \ub3d9\uc544\ub9ac\ub294 \ud06c\uac8c \ud559\uc220\ubd84\uacfc, \ubb38\uc608\ubd84\uacfc, \ucde8\uccb4\ubd84\uacfc, \uc885\uad50 \ubd84\uacfc\ub85c \ub098\ub25c\ub2e4. \ud559\uc220\ubd84\uacfc\uc5d0\ub294 \ub2e4\ubb3c, \uc0ac\ub791\ubc29, SOS, \uc778\ud130\ub7ec\ubc45, RCY, \uc791\uc740\ub098\ubb34, YMCA, GES, Youth-JC, \ub108\uc640\ub2e4\ub824, \ub85c\ud0c0\ub809\uc2a4, \ubd09\ub3d9\uc774, \uc190\uc9d3\ubc1c\uc9d3, \uc608\uadf8\ub9ac\ub098, \uc4f0\ub2f4\uc4f0\ub2f4, IYF, \ud558\ubaa8\ub2c8\uac00 \uc788\uc73c\uba70 \ubb38\uc608\ubd84\uacfc\uc5d0\ub294 \ud55c\uc6b8, \ud5a5\ub9bc\ub2e4\ud68c, \uc0ac\uc9c4\uc608\uc220\ud68c, \uc544\ub791\ud6fc\uc988, \ud06c\ub808\ud30c\uc2a4, \uba54\uc544\ub9ac, \ubba4\uc988\uadf8\ub9b0\ube44, \uc11d\uacf5, \ubaa8\uaf2c\uc9c0, P&B, \ubcf4\uc774\uc2a4\ud399\ud130, \ub9ac\uc2a4\ud399, \ubb38\uc678\ud55c\uc774 \uc788\uace0 \ucde8\uccb4\ubd84\uacfc\uc5d0\ub294 UCDC, \ud398\ub2ec, \uc0b0\uc545\ud68c, \ub2e4\uc774\uc544\ubaac\uc988, \uc778\ud130\uc149\ud2b8, \uad74\ub9ac\uc138, \ud0dd\uacac, \ud5a5\ub9bc\ud14c\ub2c8\uc2a4\uac00 \uc788\uc73c\uba70 \uc885\uad50\ubd84\uacfc\uc5d0\ub294 CCC, DSM, SFC, \ub124\ube44\uac8c\uc774\ud1a0, \ube44\ud30c\uc640\uc218\uae08, \uc778\ud130\ucf65, ESF, \uc5f0 \ub4f1\uc774 \uc788\ub2e4. \uc774 \ubc16\uc5d0\ub3c4 \ub354 \ub9ce\uc740 \ub3d9\uc544\ub9ac\uac00 \uc874\uc7ac\ud558\uba70, \ub3d9\uc544\ub9ac \ubc29\uc744 \uc5bb\uae30 \uc704\ud574\uc11c\ub294 \ub3d9\uc544\ub9ac\ud68c\uc5d0 \uc758\ud574 \uc120\ubc1c\uc774 \ub418\uc5b4\uc57c \ud55c\ub2e4."} {"question": "\ub958\uadfc\ucca0\uc774 \ubc1c\uba85\uc790 \ub610\ub294 \ud2b9\ud5c8\uad8c\uc790\ub85c \ub4f1\ub85d\ub41c \ud2b9\ud5c8\uc758 \uc218\ub294?", "paragraph": "\ucd94\uac04\ud310\uad00\uc808\uc720\uaca9\uc870\uc808\uc6b4\ub3d9\uae30(\ub4f1\ub85d\ubc88\ud638 2000455520000(19900214)), \ucd94\uac04\ud310\uad00\uc808\uc720\uaca9\uc870\uc808\uc6b4\ub3d9\uae30\uc758\ud3b8\uce21\ub192\uc774\uc870\uc808\uc7a5\uce58(\ub4f1\ub85d\ubc88\ud638 2000455510000(19900214)), \uc2e0\uccb4\uad50\uc815\uc6a9 \uc7a5\uce58, \uc2e0\uccb4\uad50\uc815\uc6a9 \uc7a5\uce58\uc758 \uc81c\uc5b4\uae30 \ubc0f \uc81c\uc5b4 \ubc29\ubc95(\ub4f1\ub85d\ubc88\ud638 1011633420000 (20120629)), \uc2e0\uccb4\uad50\uc815\uc6a9 \uc7a5\uce58 \ubc0f \uadf8\uc758 \ud655\uc7a5\ud504\ub808\uc784(\ub4f1\ub85d\ubc88\ud638 1011132420000 (20120131))\uc640 \uac19\uc740 \uc0c1\uae30 4\uac1c\uc758 \ud2b9\ud5c8\uc758 \ubc1c\uba85\uc790\ub85c \ud639\uc740 \ud2b9\ud5c8\uad8c\uc790\ub85c \ub4f1\ub85d\ub418\uc5b4 \uc788\ub2e4. \uadfc\uc721\uc774\uc644\uae30(\ucd9c\uc6d0\ubc88\ud638 10-2010-0045311 (20100514)), \uce58\ub8cc\uc6a9 \uce68\ub300 \ubc0f \uc758\uc790 \uacb8\uc6a9 \uc7a5\uce58(\ucd9c\uc6d0\ubc88\ud638 10-2009-0119509 (20091204))\uc758 2\uac1c\uc758 \uc2ec\uc0ac \uacc4\ub958 \uc911\uc778 \ucd9c\uc6d0\uc5d0 \ub300\ud558\uc5ec \ubc1c\uba85\uc790\ub85c \ub4f1\ub85d\ub418\uc5b4 \uc788\ub2e4.", "answer": "4\uac1c", "sentence": "\ucd94\uac04\ud310\uad00\uc808\uc720\uaca9\uc870\uc808\uc6b4\ub3d9\uae30(\ub4f1\ub85d\ubc88\ud638 2000455520000(19900214)), \ucd94\uac04\ud310\uad00\uc808\uc720\uaca9\uc870\uc808\uc6b4\ub3d9\uae30\uc758\ud3b8\uce21\ub192\uc774\uc870\uc808\uc7a5\uce58(\ub4f1\ub85d\ubc88\ud638 2000455510000(19900214)), \uc2e0\uccb4\uad50\uc815\uc6a9 \uc7a5\uce58, \uc2e0\uccb4\uad50\uc815\uc6a9 \uc7a5\uce58\uc758 \uc81c\uc5b4\uae30 \ubc0f \uc81c\uc5b4 \ubc29\ubc95(\ub4f1\ub85d\ubc88\ud638 1011633420000 (20120629)), \uc2e0\uccb4\uad50\uc815\uc6a9 \uc7a5\uce58 \ubc0f \uadf8\uc758 \ud655\uc7a5\ud504\ub808\uc784(\ub4f1\ub85d\ubc88\ud638 1011132420000 (20120131))\uc640 \uac19\uc740 \uc0c1\uae30 4\uac1c \uc758 \ud2b9\ud5c8\uc758 \ubc1c\uba85\uc790\ub85c \ud639\uc740 \ud2b9\ud5c8\uad8c\uc790\ub85c \ub4f1\ub85d\ub418\uc5b4 \uc788\ub2e4.", "paragraph_sentence": " \ucd94\uac04\ud310\uad00\uc808\uc720\uaca9\uc870\uc808\uc6b4\ub3d9\uae30(\ub4f1\ub85d\ubc88\ud638 2000455520000(19900214)), \ucd94\uac04\ud310\uad00\uc808\uc720\uaca9\uc870\uc808\uc6b4\ub3d9\uae30\uc758\ud3b8\uce21\ub192\uc774\uc870\uc808\uc7a5\uce58(\ub4f1\ub85d\ubc88\ud638 2000455510000(19900214)), \uc2e0\uccb4\uad50\uc815\uc6a9 \uc7a5\uce58, \uc2e0\uccb4\uad50\uc815\uc6a9 \uc7a5\uce58\uc758 \uc81c\uc5b4\uae30 \ubc0f \uc81c\uc5b4 \ubc29\ubc95(\ub4f1\ub85d\ubc88\ud638 1011633420000 (20120629)), \uc2e0\uccb4\uad50\uc815\uc6a9 \uc7a5\uce58 \ubc0f \uadf8\uc758 \ud655\uc7a5\ud504\ub808\uc784(\ub4f1\ub85d\ubc88\ud638 1011132420000 (20120131))\uc640 \uac19\uc740 \uc0c1\uae30 4\uac1c \uc758 \ud2b9\ud5c8\uc758 \ubc1c\uba85\uc790\ub85c \ud639\uc740 \ud2b9\ud5c8\uad8c\uc790\ub85c \ub4f1\ub85d\ub418\uc5b4 \uc788\ub2e4. \uadfc\uc721\uc774\uc644\uae30(\ucd9c\uc6d0\ubc88\ud638 10-2010-0045311 (20100514)), \uce58\ub8cc\uc6a9 \uce68\ub300 \ubc0f \uc758\uc790 \uacb8\uc6a9 \uc7a5\uce58(\ucd9c\uc6d0\ubc88\ud638 10-2009-0119509 (20091204))\uc758 2\uac1c\uc758 \uc2ec\uc0ac \uacc4\ub958 \uc911\uc778 \ucd9c\uc6d0\uc5d0 \ub300\ud558\uc5ec \ubc1c\uba85\uc790\ub85c \ub4f1\ub85d\ub418\uc5b4 \uc788\ub2e4.", "paragraph_answer": "\ucd94\uac04\ud310\uad00\uc808\uc720\uaca9\uc870\uc808\uc6b4\ub3d9\uae30(\ub4f1\ub85d\ubc88\ud638 2000455520000(19900214)), \ucd94\uac04\ud310\uad00\uc808\uc720\uaca9\uc870\uc808\uc6b4\ub3d9\uae30\uc758\ud3b8\uce21\ub192\uc774\uc870\uc808\uc7a5\uce58(\ub4f1\ub85d\ubc88\ud638 2000455510000(19900214)), \uc2e0\uccb4\uad50\uc815\uc6a9 \uc7a5\uce58, \uc2e0\uccb4\uad50\uc815\uc6a9 \uc7a5\uce58\uc758 \uc81c\uc5b4\uae30 \ubc0f \uc81c\uc5b4 \ubc29\ubc95(\ub4f1\ub85d\ubc88\ud638 1011633420000 (20120629)), \uc2e0\uccb4\uad50\uc815\uc6a9 \uc7a5\uce58 \ubc0f \uadf8\uc758 \ud655\uc7a5\ud504\ub808\uc784(\ub4f1\ub85d\ubc88\ud638 1011132420000 (20120131))\uc640 \uac19\uc740 \uc0c1\uae30 4\uac1c \uc758 \ud2b9\ud5c8\uc758 \ubc1c\uba85\uc790\ub85c \ud639\uc740 \ud2b9\ud5c8\uad8c\uc790\ub85c \ub4f1\ub85d\ub418\uc5b4 \uc788\ub2e4. \uadfc\uc721\uc774\uc644\uae30(\ucd9c\uc6d0\ubc88\ud638 10-2010-0045311 (20100514)), \uce58\ub8cc\uc6a9 \uce68\ub300 \ubc0f \uc758\uc790 \uacb8\uc6a9 \uc7a5\uce58(\ucd9c\uc6d0\ubc88\ud638 10-2009-0119509 (20091204))\uc758 2\uac1c\uc758 \uc2ec\uc0ac \uacc4\ub958 \uc911\uc778 \ucd9c\uc6d0\uc5d0 \ub300\ud558\uc5ec \ubc1c\uba85\uc790\ub85c \ub4f1\ub85d\ub418\uc5b4 \uc788\ub2e4.", "sentence_answer": "\ucd94\uac04\ud310\uad00\uc808\uc720\uaca9\uc870\uc808\uc6b4\ub3d9\uae30(\ub4f1\ub85d\ubc88\ud638 2000455520000(19900214)), \ucd94\uac04\ud310\uad00\uc808\uc720\uaca9\uc870\uc808\uc6b4\ub3d9\uae30\uc758\ud3b8\uce21\ub192\uc774\uc870\uc808\uc7a5\uce58(\ub4f1\ub85d\ubc88\ud638 2000455510000(19900214)), \uc2e0\uccb4\uad50\uc815\uc6a9 \uc7a5\uce58, \uc2e0\uccb4\uad50\uc815\uc6a9 \uc7a5\uce58\uc758 \uc81c\uc5b4\uae30 \ubc0f \uc81c\uc5b4 \ubc29\ubc95(\ub4f1\ub85d\ubc88\ud638 1011633420000 (20120629)), \uc2e0\uccb4\uad50\uc815\uc6a9 \uc7a5\uce58 \ubc0f \uadf8\uc758 \ud655\uc7a5\ud504\ub808\uc784(\ub4f1\ub85d\ubc88\ud638 1011132420000 (20120131))\uc640 \uac19\uc740 \uc0c1\uae30 4\uac1c \uc758 \ud2b9\ud5c8\uc758 \ubc1c\uba85\uc790\ub85c \ud639\uc740 \ud2b9\ud5c8\uad8c\uc790\ub85c \ub4f1\ub85d\ub418\uc5b4 \uc788\ub2e4.", "paragraph_id": "6513846-2-0", "paragraph_question": "question: \ub958\uadfc\ucca0\uc774 \ubc1c\uba85\uc790 \ub610\ub294 \ud2b9\ud5c8\uad8c\uc790\ub85c \ub4f1\ub85d\ub41c \ud2b9\ud5c8\uc758 \uc218\ub294?, context: \ucd94\uac04\ud310\uad00\uc808\uc720\uaca9\uc870\uc808\uc6b4\ub3d9\uae30(\ub4f1\ub85d\ubc88\ud638 2000455520000(19900214)), \ucd94\uac04\ud310\uad00\uc808\uc720\uaca9\uc870\uc808\uc6b4\ub3d9\uae30\uc758\ud3b8\uce21\ub192\uc774\uc870\uc808\uc7a5\uce58(\ub4f1\ub85d\ubc88\ud638 2000455510000(19900214)), \uc2e0\uccb4\uad50\uc815\uc6a9 \uc7a5\uce58, \uc2e0\uccb4\uad50\uc815\uc6a9 \uc7a5\uce58\uc758 \uc81c\uc5b4\uae30 \ubc0f \uc81c\uc5b4 \ubc29\ubc95(\ub4f1\ub85d\ubc88\ud638 1011633420000 (20120629)), \uc2e0\uccb4\uad50\uc815\uc6a9 \uc7a5\uce58 \ubc0f \uadf8\uc758 \ud655\uc7a5\ud504\ub808\uc784(\ub4f1\ub85d\ubc88\ud638 1011132420000 (20120131))\uc640 \uac19\uc740 \uc0c1\uae30 4\uac1c\uc758 \ud2b9\ud5c8\uc758 \ubc1c\uba85\uc790\ub85c \ud639\uc740 \ud2b9\ud5c8\uad8c\uc790\ub85c \ub4f1\ub85d\ub418\uc5b4 \uc788\ub2e4. \uadfc\uc721\uc774\uc644\uae30(\ucd9c\uc6d0\ubc88\ud638 10-2010-0045311 (20100514)), \uce58\ub8cc\uc6a9 \uce68\ub300 \ubc0f \uc758\uc790 \uacb8\uc6a9 \uc7a5\uce58(\ucd9c\uc6d0\ubc88\ud638 10-2009-0119509 (20091204))\uc758 2\uac1c\uc758 \uc2ec\uc0ac \uacc4\ub958 \uc911\uc778 \ucd9c\uc6d0\uc5d0 \ub300\ud558\uc5ec \ubc1c\uba85\uc790\ub85c \ub4f1\ub85d\ub418\uc5b4 \uc788\ub2e4."} {"question": "\uac15\ud76c\uc81c\uac00 \uc22d\uc0c1\ud55c \ud559\ubb38\uc740?", "paragraph": "\ud6cc\ub96d\ud55c \uc815\uce58\ub97c \ud3bc\uce5c \uac15\ud76c\uc81c\uc600\uc9c0\ub9cc, \uadf8\uc758 \uc790\uc2dd\ub4e4\uc740 \uadf8\ub2e4\uc9c0 \ud070 \uadf8\ub987\uc774 \ub418\uc9c0 \ubabb\ud588\ub2e4. \uac15\ud76c\uc81c\ub294 \ud669\uc790\ub4e4 \ubaa8\ub450\ub97c \uc790\uc2e0\uc758 \uacbd\uc5f0 \ud1a0\ub860\uc7a5\uc778 \ub0a8\uc11c\ubc29\uc5d0\uc11c \uad50\uc721\uc2dc\ucf1c \uc720\ud559\uc758 \uc0ac\uc0c1\uc774 \ubab8\uc5d0 \ubc34 \ud669\uc790\ub85c \ub9cc\ub4e4\ub824 \ud558\uc600\uc73c\ub098 \ub300\ubd80\ubd84 \uadf8\ub9ac \ub418\uc9c0 \ubabb\ud558\uc600\ub2e4. \uc81c2\ud669\uc790\uc774\uba70 \uac15\ud76c\uc81c\uc758 \uc720\uc77c\ud55c \uc801\uc790\uc774\uc790 \uc801\uc7a5\ub0a8\uc778 \uc724\uc789(\u80e4\u793d)\uc740 \uccab \ubc88\uc9f8 \ud669\ud6c4\uc778 \ud6a8\uc131\uc778\ud669\ud6c4\uc758 \uc18c\uc0dd\uc73c\ub85c, \ud6a8\uc131\uc778\ud669\ud6c4\uac00 1674\ub144(\uac15\ud76c 13\ub144)\uc5d0 \uc724\uc789\uc744 \ub0b3\uc740 \uc9c1\ud6c4 \ub09c\uc0b0\uc73c\ub85c \ubd95\uc5b4\ud558\uc790 \uc774\ub97c \uc2ac\ud37c\ud55c \uac15\ud76c\uc81c\uac00 \uc774\ub4ec\ud574\uc778 1675\ub144(\uac15\ud76c 14\ub144)\uc5d0 \ubc14\ub85c \ud669\ud0dc\uc790\ub85c \ucc45\ubd09\ud558\uc600\ub2e4. \uc774\uac83\uc740 \ubcf8\ub798 \uac00\uc7a5 \uc720\ub2a5\ud55c \uc544\ub4e4\uc744 \ud6c4\uacc4\uc790\ub85c \uc0bc\ub294 \ub9cc\uc8fc\uc871\uc758 \uc804\ud1b5\uc744 \uae68\uace0 \ud55c\uc871\uc758 \uc804\ud1b5\uc744 \ub3c4\uc785\ud558\ub294 \uac83\uc774\uc5c8\uae30\uc5d0 \ub9cc\uc8fc\uc871 \ub300\uc2e0\ub4e4 \uc0ac\uc774\uc5d0\uc11c \ub17c\uc7c1\uc774 \uc788\uc5c8\uc73c\ub098, \uc720\ud559\uc744 \uc22d\uc0c1\ud55c \uac15\ud76c\uc81c\ub294 \uc774\ub7f0 \ub17c\uc758\ub97c \ub2e8\ud638\ud788 \uc77c\ucd95\ud558\uc600\ub2e4. \uc774\uc5d0 \uc11c\uc7a5\uc790\uc774\uace0 \uc81c1\ud669\uc790\uc758 \uc791\uc704\ub97c \ubc1b\uc558\uc73c\uba70 \uc724\uc789\ubcf4\ub2e4 \ub450 \uc0b4\uc774 \ub9ce\uc740 \uc724\uc2dc(\u80e4\u7994)\uac00 \ub300\uc2e0\ub4e4 \uc0ac\uc774\uc5d0\uc11c \uc724\uc789\uc758 \uacbd\uc7c1 \uc0c1\ub300\ub85c \ub5a0\uc62c\ub790\uc73c\ub098 \uac15\ud76c\uc81c\ub294 \uc5ec\ub7ec \ubc88 \uacf5\uc2dd\uc11d\uc0c1\uc5d0\uc11c \uc724\uc789\uc744 \ud6c4\uacc4\uc790\ub85c \ucc9c\uba85\ud558\uc5ec, \uc81c\uc704\ub294 \uc724\uc789\uc774 \uc2b9\uacc4\ubc1b\ub294 \uac83\uc73c\ub85c \ud655\uc815\ub418\uc5b4 \uac14\ub2e4.", "answer": "\uc720\ud559", "sentence": "\uac15\ud76c\uc81c\ub294 \ud669\uc790\ub4e4 \ubaa8\ub450\ub97c \uc790\uc2e0\uc758 \uacbd\uc5f0 \ud1a0\ub860\uc7a5\uc778 \ub0a8\uc11c\ubc29\uc5d0\uc11c \uad50\uc721\uc2dc\ucf1c \uc720\ud559 \uc758 \uc0ac\uc0c1\uc774 \ubab8\uc5d0 \ubc34 \ud669\uc790\ub85c \ub9cc\ub4e4\ub824 \ud558\uc600\uc73c\ub098 \ub300\ubd80\ubd84 \uadf8\ub9ac \ub418\uc9c0 \ubabb\ud558\uc600\ub2e4.", "paragraph_sentence": "\ud6cc\ub96d\ud55c \uc815\uce58\ub97c \ud3bc\uce5c \uac15\ud76c\uc81c\uc600\uc9c0\ub9cc, \uadf8\uc758 \uc790\uc2dd\ub4e4\uc740 \uadf8\ub2e4\uc9c0 \ud070 \uadf8\ub987\uc774 \ub418\uc9c0 \ubabb\ud588\ub2e4. \uac15\ud76c\uc81c\ub294 \ud669\uc790\ub4e4 \ubaa8\ub450\ub97c \uc790\uc2e0\uc758 \uacbd\uc5f0 \ud1a0\ub860\uc7a5\uc778 \ub0a8\uc11c\ubc29\uc5d0\uc11c \uad50\uc721\uc2dc\ucf1c \uc720\ud559 \uc758 \uc0ac\uc0c1\uc774 \ubab8\uc5d0 \ubc34 \ud669\uc790\ub85c \ub9cc\ub4e4\ub824 \ud558\uc600\uc73c\ub098 \ub300\ubd80\ubd84 \uadf8\ub9ac \ub418\uc9c0 \ubabb\ud558\uc600\ub2e4. \uc81c2\ud669\uc790\uc774\uba70 \uac15\ud76c\uc81c\uc758 \uc720\uc77c\ud55c \uc801\uc790\uc774\uc790 \uc801\uc7a5\ub0a8\uc778 \uc724\uc789(\u80e4\u793d)\uc740 \uccab \ubc88\uc9f8 \ud669\ud6c4\uc778 \ud6a8\uc131\uc778\ud669\ud6c4\uc758 \uc18c\uc0dd\uc73c\ub85c, \ud6a8\uc131\uc778\ud669\ud6c4\uac00 1674\ub144(\uac15\ud76c 13\ub144)\uc5d0 \uc724\uc789\uc744 \ub0b3\uc740 \uc9c1\ud6c4 \ub09c\uc0b0\uc73c\ub85c \ubd95\uc5b4\ud558\uc790 \uc774\ub97c \uc2ac\ud37c\ud55c \uac15\ud76c\uc81c\uac00 \uc774\ub4ec\ud574\uc778 1675\ub144(\uac15\ud76c 14\ub144)\uc5d0 \ubc14\ub85c \ud669\ud0dc\uc790\ub85c \ucc45\ubd09\ud558\uc600\ub2e4. \uc774\uac83\uc740 \ubcf8\ub798 \uac00\uc7a5 \uc720\ub2a5\ud55c \uc544\ub4e4\uc744 \ud6c4\uacc4\uc790\ub85c \uc0bc\ub294 \ub9cc\uc8fc\uc871\uc758 \uc804\ud1b5\uc744 \uae68\uace0 \ud55c\uc871\uc758 \uc804\ud1b5\uc744 \ub3c4\uc785\ud558\ub294 \uac83\uc774\uc5c8\uae30\uc5d0 \ub9cc\uc8fc\uc871 \ub300\uc2e0\ub4e4 \uc0ac\uc774\uc5d0\uc11c \ub17c\uc7c1\uc774 \uc788\uc5c8\uc73c\ub098, \uc720\ud559\uc744 \uc22d\uc0c1\ud55c \uac15\ud76c\uc81c\ub294 \uc774\ub7f0 \ub17c\uc758\ub97c \ub2e8\ud638\ud788 \uc77c\ucd95\ud558\uc600\ub2e4. \uc774\uc5d0 \uc11c\uc7a5\uc790\uc774\uace0 \uc81c1\ud669\uc790\uc758 \uc791\uc704\ub97c \ubc1b\uc558\uc73c\uba70 \uc724\uc789\ubcf4\ub2e4 \ub450 \uc0b4\uc774 \ub9ce\uc740 \uc724\uc2dc(\u80e4\u7994)\uac00 \ub300\uc2e0\ub4e4 \uc0ac\uc774\uc5d0\uc11c \uc724\uc789\uc758 \uacbd\uc7c1 \uc0c1\ub300\ub85c \ub5a0\uc62c\ub790\uc73c\ub098 \uac15\ud76c\uc81c\ub294 \uc5ec\ub7ec \ubc88 \uacf5\uc2dd\uc11d\uc0c1\uc5d0\uc11c \uc724\uc789\uc744 \ud6c4\uacc4\uc790\ub85c \ucc9c\uba85\ud558\uc5ec, \uc81c\uc704\ub294 \uc724\uc789\uc774 \uc2b9\uacc4\ubc1b\ub294 \uac83\uc73c\ub85c \ud655\uc815\ub418\uc5b4 \uac14\ub2e4.", "paragraph_answer": "\ud6cc\ub96d\ud55c \uc815\uce58\ub97c \ud3bc\uce5c \uac15\ud76c\uc81c\uc600\uc9c0\ub9cc, \uadf8\uc758 \uc790\uc2dd\ub4e4\uc740 \uadf8\ub2e4\uc9c0 \ud070 \uadf8\ub987\uc774 \ub418\uc9c0 \ubabb\ud588\ub2e4. \uac15\ud76c\uc81c\ub294 \ud669\uc790\ub4e4 \ubaa8\ub450\ub97c \uc790\uc2e0\uc758 \uacbd\uc5f0 \ud1a0\ub860\uc7a5\uc778 \ub0a8\uc11c\ubc29\uc5d0\uc11c \uad50\uc721\uc2dc\ucf1c \uc720\ud559 \uc758 \uc0ac\uc0c1\uc774 \ubab8\uc5d0 \ubc34 \ud669\uc790\ub85c \ub9cc\ub4e4\ub824 \ud558\uc600\uc73c\ub098 \ub300\ubd80\ubd84 \uadf8\ub9ac \ub418\uc9c0 \ubabb\ud558\uc600\ub2e4. \uc81c2\ud669\uc790\uc774\uba70 \uac15\ud76c\uc81c\uc758 \uc720\uc77c\ud55c \uc801\uc790\uc774\uc790 \uc801\uc7a5\ub0a8\uc778 \uc724\uc789(\u80e4\u793d)\uc740 \uccab \ubc88\uc9f8 \ud669\ud6c4\uc778 \ud6a8\uc131\uc778\ud669\ud6c4\uc758 \uc18c\uc0dd\uc73c\ub85c, \ud6a8\uc131\uc778\ud669\ud6c4\uac00 1674\ub144(\uac15\ud76c 13\ub144)\uc5d0 \uc724\uc789\uc744 \ub0b3\uc740 \uc9c1\ud6c4 \ub09c\uc0b0\uc73c\ub85c \ubd95\uc5b4\ud558\uc790 \uc774\ub97c \uc2ac\ud37c\ud55c \uac15\ud76c\uc81c\uac00 \uc774\ub4ec\ud574\uc778 1675\ub144(\uac15\ud76c 14\ub144)\uc5d0 \ubc14\ub85c \ud669\ud0dc\uc790\ub85c \ucc45\ubd09\ud558\uc600\ub2e4. \uc774\uac83\uc740 \ubcf8\ub798 \uac00\uc7a5 \uc720\ub2a5\ud55c \uc544\ub4e4\uc744 \ud6c4\uacc4\uc790\ub85c \uc0bc\ub294 \ub9cc\uc8fc\uc871\uc758 \uc804\ud1b5\uc744 \uae68\uace0 \ud55c\uc871\uc758 \uc804\ud1b5\uc744 \ub3c4\uc785\ud558\ub294 \uac83\uc774\uc5c8\uae30\uc5d0 \ub9cc\uc8fc\uc871 \ub300\uc2e0\ub4e4 \uc0ac\uc774\uc5d0\uc11c \ub17c\uc7c1\uc774 \uc788\uc5c8\uc73c\ub098, \uc720\ud559\uc744 \uc22d\uc0c1\ud55c \uac15\ud76c\uc81c\ub294 \uc774\ub7f0 \ub17c\uc758\ub97c \ub2e8\ud638\ud788 \uc77c\ucd95\ud558\uc600\ub2e4. \uc774\uc5d0 \uc11c\uc7a5\uc790\uc774\uace0 \uc81c1\ud669\uc790\uc758 \uc791\uc704\ub97c \ubc1b\uc558\uc73c\uba70 \uc724\uc789\ubcf4\ub2e4 \ub450 \uc0b4\uc774 \ub9ce\uc740 \uc724\uc2dc(\u80e4\u7994)\uac00 \ub300\uc2e0\ub4e4 \uc0ac\uc774\uc5d0\uc11c \uc724\uc789\uc758 \uacbd\uc7c1 \uc0c1\ub300\ub85c \ub5a0\uc62c\ub790\uc73c\ub098 \uac15\ud76c\uc81c\ub294 \uc5ec\ub7ec \ubc88 \uacf5\uc2dd\uc11d\uc0c1\uc5d0\uc11c \uc724\uc789\uc744 \ud6c4\uacc4\uc790\ub85c \ucc9c\uba85\ud558\uc5ec, \uc81c\uc704\ub294 \uc724\uc789\uc774 \uc2b9\uacc4\ubc1b\ub294 \uac83\uc73c\ub85c \ud655\uc815\ub418\uc5b4 \uac14\ub2e4.", "sentence_answer": "\uac15\ud76c\uc81c\ub294 \ud669\uc790\ub4e4 \ubaa8\ub450\ub97c \uc790\uc2e0\uc758 \uacbd\uc5f0 \ud1a0\ub860\uc7a5\uc778 \ub0a8\uc11c\ubc29\uc5d0\uc11c \uad50\uc721\uc2dc\ucf1c \uc720\ud559 \uc758 \uc0ac\uc0c1\uc774 \ubab8\uc5d0 \ubc34 \ud669\uc790\ub85c \ub9cc\ub4e4\ub824 \ud558\uc600\uc73c\ub098 \ub300\ubd80\ubd84 \uadf8\ub9ac \ub418\uc9c0 \ubabb\ud558\uc600\ub2e4.", "paragraph_id": "6478740-29-1", "paragraph_question": "question: \uac15\ud76c\uc81c\uac00 \uc22d\uc0c1\ud55c \ud559\ubb38\uc740?, context: \ud6cc\ub96d\ud55c \uc815\uce58\ub97c \ud3bc\uce5c \uac15\ud76c\uc81c\uc600\uc9c0\ub9cc, \uadf8\uc758 \uc790\uc2dd\ub4e4\uc740 \uadf8\ub2e4\uc9c0 \ud070 \uadf8\ub987\uc774 \ub418\uc9c0 \ubabb\ud588\ub2e4. \uac15\ud76c\uc81c\ub294 \ud669\uc790\ub4e4 \ubaa8\ub450\ub97c \uc790\uc2e0\uc758 \uacbd\uc5f0 \ud1a0\ub860\uc7a5\uc778 \ub0a8\uc11c\ubc29\uc5d0\uc11c \uad50\uc721\uc2dc\ucf1c \uc720\ud559\uc758 \uc0ac\uc0c1\uc774 \ubab8\uc5d0 \ubc34 \ud669\uc790\ub85c \ub9cc\ub4e4\ub824 \ud558\uc600\uc73c\ub098 \ub300\ubd80\ubd84 \uadf8\ub9ac \ub418\uc9c0 \ubabb\ud558\uc600\ub2e4. \uc81c2\ud669\uc790\uc774\uba70 \uac15\ud76c\uc81c\uc758 \uc720\uc77c\ud55c \uc801\uc790\uc774\uc790 \uc801\uc7a5\ub0a8\uc778 \uc724\uc789(\u80e4\u793d)\uc740 \uccab \ubc88\uc9f8 \ud669\ud6c4\uc778 \ud6a8\uc131\uc778\ud669\ud6c4\uc758 \uc18c\uc0dd\uc73c\ub85c, \ud6a8\uc131\uc778\ud669\ud6c4\uac00 1674\ub144(\uac15\ud76c 13\ub144)\uc5d0 \uc724\uc789\uc744 \ub0b3\uc740 \uc9c1\ud6c4 \ub09c\uc0b0\uc73c\ub85c \ubd95\uc5b4\ud558\uc790 \uc774\ub97c \uc2ac\ud37c\ud55c \uac15\ud76c\uc81c\uac00 \uc774\ub4ec\ud574\uc778 1675\ub144(\uac15\ud76c 14\ub144)\uc5d0 \ubc14\ub85c \ud669\ud0dc\uc790\ub85c \ucc45\ubd09\ud558\uc600\ub2e4. \uc774\uac83\uc740 \ubcf8\ub798 \uac00\uc7a5 \uc720\ub2a5\ud55c \uc544\ub4e4\uc744 \ud6c4\uacc4\uc790\ub85c \uc0bc\ub294 \ub9cc\uc8fc\uc871\uc758 \uc804\ud1b5\uc744 \uae68\uace0 \ud55c\uc871\uc758 \uc804\ud1b5\uc744 \ub3c4\uc785\ud558\ub294 \uac83\uc774\uc5c8\uae30\uc5d0 \ub9cc\uc8fc\uc871 \ub300\uc2e0\ub4e4 \uc0ac\uc774\uc5d0\uc11c \ub17c\uc7c1\uc774 \uc788\uc5c8\uc73c\ub098, \uc720\ud559\uc744 \uc22d\uc0c1\ud55c \uac15\ud76c\uc81c\ub294 \uc774\ub7f0 \ub17c\uc758\ub97c \ub2e8\ud638\ud788 \uc77c\ucd95\ud558\uc600\ub2e4. \uc774\uc5d0 \uc11c\uc7a5\uc790\uc774\uace0 \uc81c1\ud669\uc790\uc758 \uc791\uc704\ub97c \ubc1b\uc558\uc73c\uba70 \uc724\uc789\ubcf4\ub2e4 \ub450 \uc0b4\uc774 \ub9ce\uc740 \uc724\uc2dc(\u80e4\u7994)\uac00 \ub300\uc2e0\ub4e4 \uc0ac\uc774\uc5d0\uc11c \uc724\uc789\uc758 \uacbd\uc7c1 \uc0c1\ub300\ub85c \ub5a0\uc62c\ub790\uc73c\ub098 \uac15\ud76c\uc81c\ub294 \uc5ec\ub7ec \ubc88 \uacf5\uc2dd\uc11d\uc0c1\uc5d0\uc11c \uc724\uc789\uc744 \ud6c4\uacc4\uc790\ub85c \ucc9c\uba85\ud558\uc5ec, \uc81c\uc704\ub294 \uc724\uc789\uc774 \uc2b9\uacc4\ubc1b\ub294 \uac83\uc73c\ub85c \ud655\uc815\ub418\uc5b4 \uac14\ub2e4."} {"question": "\ubc00\uc591 \uc5ec\uc911\uc0dd \uc9d1\ub2e8 \uc131\ud3ed\ud589 \uc0ac\uac74\uc758 \uc218\uc0ac\uacfc\uc815\uc5d0\uc11c \uba3c\uc800 \uad6c\uc18d\ud55c 3\uba85\uc678 \uad6c\uc18d\uc601\uc7a5\uc774 \uc2e0\uccad\ub41c \ubc94\uc8c4\ud610\uc758\uac00 \uba85\ubc31\ud55c \uac00\ud574\uc790\ub294 \uba87 \uba85\uc778\uac00?", "paragraph": "\uc218\uc0ac \uacfc\uc815\uc774 \ubbf8\ud761\ud558\uace0 \uac00\ud574\uc790\ub4e4\uc5d0 \ub300\ud55c \ucc98\ubc8c\uc774 \ubbf8\uc57d\ud558\uc790 \ub124\ud2f0\uc98c\uacfc \uc2dc\ubbfc\ub2e8\uccb4 \ub4f1\uc758 \ubd84\ub178\uc640 \ud56d\uc758\uac00 \uacc4\uc18d\ub418\uc5c8\ub2e4. \uacbd\ucc30\uc740 \uacf5\uc9c0\ubb38\uc5d0\uc11c \"\uac00\ud574 \ud559\uc0dd \ub300\ubd80\ubd84\uc744 \ud6c8\ubc29\ud588\ub2e4\ub294 \uac83\uc740 \uc0ac\uc2e4\uacfc \ub2e4\ub974\uba70 \uc6b0\uc120 \ud610\uc758\uac00 \ubb34\uac70\uc6b4 3\uba85\uc744 \uad6c\uc18d\ud558\uace0 \ub098\uba38\uc9c0\ub294 \uacc4\uc18d \uc218\uc0ac\ud558\uace0 \uc788\ub2e4\", \"\uc218\uc0ac\uacfc\uc815\uc5d0\uc11c \ud53c\ud574\uc790 \ubcf4\ud638\uc5d0 \ucd5c\uc120\uc744 \ub2e4\ud588\ub2e4\"\uace0 \ud574\uba85\ud588\ub2e4. \uc774\uc5b4 \uacbd\ucc30\uc740 \"\uc5b8\ub860\uacf5\uac1c \uacfc\uc815\uc5d0\uc11c\ub3c4 \uc778\uc801\uc0ac\ud56d\uc744 \uac00\uba85\uc73c\ub85c \ud558\ub294 \ub4f1 \ud53c\ud574\uc790 \ubcf4\ud638\ub97c \uc704\ud574 \ub178\ub825\ud588\uc73c\ub098 \uc5ec\uacbd\uc870\uc0ac\uad00\uc744 \ubc30\uce58\ud558\uc9c0 \uc54a\uc740 \uc810\uc740 \ubbf8\ud761\ud588\ub2e4\"\uace0 \uc2dc\uc778\ud558\uba74\uc11c \"\uc758\ud639\uc5c6\uc774 \ucca0\uc800\ud788 \uc218\uc0ac\ud558\uaca0\ub2e4\"\uace0 \ubc1d\ud614\ub2e4. \ub610\ud55c \ucc98\uc74c \ubc1c\ud45c\ub41c 41\uba85 \uc911\uc5d0\uc11c\ub3c4 \ucc98\ubc8c\uc774 \uadf9\uc18c\uc218\ub85c \uadf8\uce5c \uc810\uc774 \uc9c0\uc801\ub418\uc5c8\ub2e4. \uacbd\ucc30\uc740 \uc5ec\uae30\uc5d0 \uadf8\uce58\uc9c0 \uc54a\uace0 \ucd94\uac00\uacf5\uc9c0\ub97c \ud1b5\ud574 \"\uba3c\uc800 \uad6c\uc18d\ud55c 3\uba85\uc678 \ubc94\uc8c4\ud610\uc758\uac00 \uba85\ubc31\ud55c 14\uba85\uc5d0 \ub300\ud574 \uad6c\uc18d\uc601\uc7a5\uc744 \uc2e0\uccad\ud588\ub2e4\"\uba70 \"\uac00\ud574\uc790\uce21\uc774 \ud53c\ud574\uc790\ub97c \ud611\ubc15\ud55c \uc0ac\uc2e4\uc5d0 \ub300\ud574\uc11c\ub3c4 \uc218\uc0ac\ud574 \uc5c4\uc815 \ucc98\ubc8c\ud558\uaca0\ub2e4\"\uace0 \uc54c\ub838\ub2e4. \uadf8\ub7ec\ub098 \uc218\uc0ac \uacfc\uc815\uc5d0\uc11c\uc758 \uc0c8\ub85c\uc6b4 \ubb38\uc81c\uc810\uc774 \ub098\ud0c0\ub098\uac8c \ub41c\ub2e4.", "answer": "14\uba85", "sentence": "\uacbd\ucc30\uc740 \uc5ec\uae30\uc5d0 \uadf8\uce58\uc9c0 \uc54a\uace0 \ucd94\uac00\uacf5\uc9c0\ub97c \ud1b5\ud574 \"\uba3c\uc800 \uad6c\uc18d\ud55c 3\uba85\uc678 \ubc94\uc8c4\ud610\uc758\uac00 \uba85\ubc31\ud55c 14\uba85 \uc5d0 \ub300\ud574 \uad6c\uc18d\uc601\uc7a5\uc744 \uc2e0\uccad\ud588\ub2e4\"\uba70 \"\uac00\ud574\uc790\uce21\uc774 \ud53c\ud574\uc790\ub97c \ud611\ubc15\ud55c \uc0ac\uc2e4\uc5d0 \ub300\ud574\uc11c\ub3c4 \uc218\uc0ac\ud574 \uc5c4\uc815 \ucc98\ubc8c\ud558\uaca0\ub2e4\"\uace0 \uc54c\ub838\ub2e4.", "paragraph_sentence": "\uc218\uc0ac \uacfc\uc815\uc774 \ubbf8\ud761\ud558\uace0 \uac00\ud574\uc790\ub4e4\uc5d0 \ub300\ud55c \ucc98\ubc8c\uc774 \ubbf8\uc57d\ud558\uc790 \ub124\ud2f0\uc98c\uacfc \uc2dc\ubbfc\ub2e8\uccb4 \ub4f1\uc758 \ubd84\ub178\uc640 \ud56d\uc758\uac00 \uacc4\uc18d\ub418\uc5c8\ub2e4. \uacbd\ucc30\uc740 \uacf5\uc9c0\ubb38\uc5d0\uc11c \"\uac00\ud574 \ud559\uc0dd \ub300\ubd80\ubd84\uc744 \ud6c8\ubc29\ud588\ub2e4\ub294 \uac83\uc740 \uc0ac\uc2e4\uacfc \ub2e4\ub974\uba70 \uc6b0\uc120 \ud610\uc758\uac00 \ubb34\uac70\uc6b4 3\uba85\uc744 \uad6c\uc18d\ud558\uace0 \ub098\uba38\uc9c0\ub294 \uacc4\uc18d \uc218\uc0ac\ud558\uace0 \uc788\ub2e4\", \"\uc218\uc0ac\uacfc\uc815\uc5d0\uc11c \ud53c\ud574\uc790 \ubcf4\ud638\uc5d0 \ucd5c\uc120\uc744 \ub2e4\ud588\ub2e4\"\uace0 \ud574\uba85\ud588\ub2e4. \uc774\uc5b4 \uacbd\ucc30\uc740 \"\uc5b8\ub860\uacf5\uac1c \uacfc\uc815\uc5d0\uc11c\ub3c4 \uc778\uc801\uc0ac\ud56d\uc744 \uac00\uba85\uc73c\ub85c \ud558\ub294 \ub4f1 \ud53c\ud574\uc790 \ubcf4\ud638\ub97c \uc704\ud574 \ub178\ub825\ud588\uc73c\ub098 \uc5ec\uacbd\uc870\uc0ac\uad00\uc744 \ubc30\uce58\ud558\uc9c0 \uc54a\uc740 \uc810\uc740 \ubbf8\ud761\ud588\ub2e4\"\uace0 \uc2dc\uc778\ud558\uba74\uc11c \"\uc758\ud639\uc5c6\uc774 \ucca0\uc800\ud788 \uc218\uc0ac\ud558\uaca0\ub2e4\"\uace0 \ubc1d\ud614\ub2e4. \ub610\ud55c \ucc98\uc74c \ubc1c\ud45c\ub41c 41\uba85 \uc911\uc5d0\uc11c\ub3c4 \ucc98\ubc8c\uc774 \uadf9\uc18c\uc218\ub85c \uadf8\uce5c \uc810\uc774 \uc9c0\uc801\ub418\uc5c8\ub2e4. \uacbd\ucc30\uc740 \uc5ec\uae30\uc5d0 \uadf8\uce58\uc9c0 \uc54a\uace0 \ucd94\uac00\uacf5\uc9c0\ub97c \ud1b5\ud574 \"\uba3c\uc800 \uad6c\uc18d\ud55c 3\uba85\uc678 \ubc94\uc8c4\ud610\uc758\uac00 \uba85\ubc31\ud55c 14\uba85 \uc5d0 \ub300\ud574 \uad6c\uc18d\uc601\uc7a5\uc744 \uc2e0\uccad\ud588\ub2e4\"\uba70 \"\uac00\ud574\uc790\uce21\uc774 \ud53c\ud574\uc790\ub97c \ud611\ubc15\ud55c \uc0ac\uc2e4\uc5d0 \ub300\ud574\uc11c\ub3c4 \uc218\uc0ac\ud574 \uc5c4\uc815 \ucc98\ubc8c\ud558\uaca0\ub2e4\"\uace0 \uc54c\ub838\ub2e4. \uadf8\ub7ec\ub098 \uc218\uc0ac \uacfc\uc815\uc5d0\uc11c\uc758 \uc0c8\ub85c\uc6b4 \ubb38\uc81c\uc810\uc774 \ub098\ud0c0\ub098\uac8c \ub41c\ub2e4.", "paragraph_answer": "\uc218\uc0ac \uacfc\uc815\uc774 \ubbf8\ud761\ud558\uace0 \uac00\ud574\uc790\ub4e4\uc5d0 \ub300\ud55c \ucc98\ubc8c\uc774 \ubbf8\uc57d\ud558\uc790 \ub124\ud2f0\uc98c\uacfc \uc2dc\ubbfc\ub2e8\uccb4 \ub4f1\uc758 \ubd84\ub178\uc640 \ud56d\uc758\uac00 \uacc4\uc18d\ub418\uc5c8\ub2e4. \uacbd\ucc30\uc740 \uacf5\uc9c0\ubb38\uc5d0\uc11c \"\uac00\ud574 \ud559\uc0dd \ub300\ubd80\ubd84\uc744 \ud6c8\ubc29\ud588\ub2e4\ub294 \uac83\uc740 \uc0ac\uc2e4\uacfc \ub2e4\ub974\uba70 \uc6b0\uc120 \ud610\uc758\uac00 \ubb34\uac70\uc6b4 3\uba85\uc744 \uad6c\uc18d\ud558\uace0 \ub098\uba38\uc9c0\ub294 \uacc4\uc18d \uc218\uc0ac\ud558\uace0 \uc788\ub2e4\", \"\uc218\uc0ac\uacfc\uc815\uc5d0\uc11c \ud53c\ud574\uc790 \ubcf4\ud638\uc5d0 \ucd5c\uc120\uc744 \ub2e4\ud588\ub2e4\"\uace0 \ud574\uba85\ud588\ub2e4. \uc774\uc5b4 \uacbd\ucc30\uc740 \"\uc5b8\ub860\uacf5\uac1c \uacfc\uc815\uc5d0\uc11c\ub3c4 \uc778\uc801\uc0ac\ud56d\uc744 \uac00\uba85\uc73c\ub85c \ud558\ub294 \ub4f1 \ud53c\ud574\uc790 \ubcf4\ud638\ub97c \uc704\ud574 \ub178\ub825\ud588\uc73c\ub098 \uc5ec\uacbd\uc870\uc0ac\uad00\uc744 \ubc30\uce58\ud558\uc9c0 \uc54a\uc740 \uc810\uc740 \ubbf8\ud761\ud588\ub2e4\"\uace0 \uc2dc\uc778\ud558\uba74\uc11c \"\uc758\ud639\uc5c6\uc774 \ucca0\uc800\ud788 \uc218\uc0ac\ud558\uaca0\ub2e4\"\uace0 \ubc1d\ud614\ub2e4. \ub610\ud55c \ucc98\uc74c \ubc1c\ud45c\ub41c 41\uba85 \uc911\uc5d0\uc11c\ub3c4 \ucc98\ubc8c\uc774 \uadf9\uc18c\uc218\ub85c \uadf8\uce5c \uc810\uc774 \uc9c0\uc801\ub418\uc5c8\ub2e4. \uacbd\ucc30\uc740 \uc5ec\uae30\uc5d0 \uadf8\uce58\uc9c0 \uc54a\uace0 \ucd94\uac00\uacf5\uc9c0\ub97c \ud1b5\ud574 \"\uba3c\uc800 \uad6c\uc18d\ud55c 3\uba85\uc678 \ubc94\uc8c4\ud610\uc758\uac00 \uba85\ubc31\ud55c 14\uba85 \uc5d0 \ub300\ud574 \uad6c\uc18d\uc601\uc7a5\uc744 \uc2e0\uccad\ud588\ub2e4\"\uba70 \"\uac00\ud574\uc790\uce21\uc774 \ud53c\ud574\uc790\ub97c \ud611\ubc15\ud55c \uc0ac\uc2e4\uc5d0 \ub300\ud574\uc11c\ub3c4 \uc218\uc0ac\ud574 \uc5c4\uc815 \ucc98\ubc8c\ud558\uaca0\ub2e4\"\uace0 \uc54c\ub838\ub2e4. \uadf8\ub7ec\ub098 \uc218\uc0ac \uacfc\uc815\uc5d0\uc11c\uc758 \uc0c8\ub85c\uc6b4 \ubb38\uc81c\uc810\uc774 \ub098\ud0c0\ub098\uac8c \ub41c\ub2e4.", "sentence_answer": "\uacbd\ucc30\uc740 \uc5ec\uae30\uc5d0 \uadf8\uce58\uc9c0 \uc54a\uace0 \ucd94\uac00\uacf5\uc9c0\ub97c \ud1b5\ud574 \"\uba3c\uc800 \uad6c\uc18d\ud55c 3\uba85\uc678 \ubc94\uc8c4\ud610\uc758\uac00 \uba85\ubc31\ud55c 14\uba85 \uc5d0 \ub300\ud574 \uad6c\uc18d\uc601\uc7a5\uc744 \uc2e0\uccad\ud588\ub2e4\"\uba70 \"\uac00\ud574\uc790\uce21\uc774 \ud53c\ud574\uc790\ub97c \ud611\ubc15\ud55c \uc0ac\uc2e4\uc5d0 \ub300\ud574\uc11c\ub3c4 \uc218\uc0ac\ud574 \uc5c4\uc815 \ucc98\ubc8c\ud558\uaca0\ub2e4\"\uace0 \uc54c\ub838\ub2e4.", "paragraph_id": "6515514-12-1", "paragraph_question": "question: \ubc00\uc591 \uc5ec\uc911\uc0dd \uc9d1\ub2e8 \uc131\ud3ed\ud589 \uc0ac\uac74\uc758 \uc218\uc0ac\uacfc\uc815\uc5d0\uc11c \uba3c\uc800 \uad6c\uc18d\ud55c 3\uba85\uc678 \uad6c\uc18d\uc601\uc7a5\uc774 \uc2e0\uccad\ub41c \ubc94\uc8c4\ud610\uc758\uac00 \uba85\ubc31\ud55c \uac00\ud574\uc790\ub294 \uba87 \uba85\uc778\uac00?, context: \uc218\uc0ac \uacfc\uc815\uc774 \ubbf8\ud761\ud558\uace0 \uac00\ud574\uc790\ub4e4\uc5d0 \ub300\ud55c \ucc98\ubc8c\uc774 \ubbf8\uc57d\ud558\uc790 \ub124\ud2f0\uc98c\uacfc \uc2dc\ubbfc\ub2e8\uccb4 \ub4f1\uc758 \ubd84\ub178\uc640 \ud56d\uc758\uac00 \uacc4\uc18d\ub418\uc5c8\ub2e4. \uacbd\ucc30\uc740 \uacf5\uc9c0\ubb38\uc5d0\uc11c \"\uac00\ud574 \ud559\uc0dd \ub300\ubd80\ubd84\uc744 \ud6c8\ubc29\ud588\ub2e4\ub294 \uac83\uc740 \uc0ac\uc2e4\uacfc \ub2e4\ub974\uba70 \uc6b0\uc120 \ud610\uc758\uac00 \ubb34\uac70\uc6b4 3\uba85\uc744 \uad6c\uc18d\ud558\uace0 \ub098\uba38\uc9c0\ub294 \uacc4\uc18d \uc218\uc0ac\ud558\uace0 \uc788\ub2e4\", \"\uc218\uc0ac\uacfc\uc815\uc5d0\uc11c \ud53c\ud574\uc790 \ubcf4\ud638\uc5d0 \ucd5c\uc120\uc744 \ub2e4\ud588\ub2e4\"\uace0 \ud574\uba85\ud588\ub2e4. \uc774\uc5b4 \uacbd\ucc30\uc740 \"\uc5b8\ub860\uacf5\uac1c \uacfc\uc815\uc5d0\uc11c\ub3c4 \uc778\uc801\uc0ac\ud56d\uc744 \uac00\uba85\uc73c\ub85c \ud558\ub294 \ub4f1 \ud53c\ud574\uc790 \ubcf4\ud638\ub97c \uc704\ud574 \ub178\ub825\ud588\uc73c\ub098 \uc5ec\uacbd\uc870\uc0ac\uad00\uc744 \ubc30\uce58\ud558\uc9c0 \uc54a\uc740 \uc810\uc740 \ubbf8\ud761\ud588\ub2e4\"\uace0 \uc2dc\uc778\ud558\uba74\uc11c \"\uc758\ud639\uc5c6\uc774 \ucca0\uc800\ud788 \uc218\uc0ac\ud558\uaca0\ub2e4\"\uace0 \ubc1d\ud614\ub2e4. \ub610\ud55c \ucc98\uc74c \ubc1c\ud45c\ub41c 41\uba85 \uc911\uc5d0\uc11c\ub3c4 \ucc98\ubc8c\uc774 \uadf9\uc18c\uc218\ub85c \uadf8\uce5c \uc810\uc774 \uc9c0\uc801\ub418\uc5c8\ub2e4. \uacbd\ucc30\uc740 \uc5ec\uae30\uc5d0 \uadf8\uce58\uc9c0 \uc54a\uace0 \ucd94\uac00\uacf5\uc9c0\ub97c \ud1b5\ud574 \"\uba3c\uc800 \uad6c\uc18d\ud55c 3\uba85\uc678 \ubc94\uc8c4\ud610\uc758\uac00 \uba85\ubc31\ud55c 14\uba85\uc5d0 \ub300\ud574 \uad6c\uc18d\uc601\uc7a5\uc744 \uc2e0\uccad\ud588\ub2e4\"\uba70 \"\uac00\ud574\uc790\uce21\uc774 \ud53c\ud574\uc790\ub97c \ud611\ubc15\ud55c \uc0ac\uc2e4\uc5d0 \ub300\ud574\uc11c\ub3c4 \uc218\uc0ac\ud574 \uc5c4\uc815 \ucc98\ubc8c\ud558\uaca0\ub2e4\"\uace0 \uc54c\ub838\ub2e4. \uadf8\ub7ec\ub098 \uc218\uc0ac \uacfc\uc815\uc5d0\uc11c\uc758 \uc0c8\ub85c\uc6b4 \ubb38\uc81c\uc810\uc774 \ub098\ud0c0\ub098\uac8c \ub41c\ub2e4."} {"question": "2007\ub144 6\uc6d4\uc5d0 \"ZARD\uc758 \uc791\ud488 \uc911 \uc88b\uc544\ud558\ub294 \uc2f1\uae00 \uc568\ubc94\"\uc744 \uc8fc\uc81c\ub85c \ud55c \uc124\ubb38\uc5d0\uc11c <\uc601\uc6d0>\uc740 \uba87 \ubc88\uc9f8\ub85c \ub192\uc740 \uc9c0\uc9c0\ub97c \uc5bb\uc5c8\ub294\uac00?", "paragraph": "\u3008\uc601\uc6d0\u3009\uc740 ZARD\uc758 \ud32c\ub4e4\uc5d0\uac8c \ub9ce\uc740 \uc0ac\ub791\uc744 \ubc1b\uc558\ub358 \uace1\uc774\uae30\ub3c4 \ud558\ub2e4. \uc0ac\uce74\uc774\uac00 \uc0ac\ub9dd\ud55c \ubc14\ub85c \ub2e4\uc74c \ub2ec\uc778 2007\ub144 6\uc6d4, \uc624\ub9ac\ucf58\uc5d0\uc11c\ub294 \ud32c\ub4e4\uc744 \ub300\uc0c1\uc73c\ub85c \"ZARD\uc758 \uc791\ud488 \uc911\uc5d0\uc11c \uc88b\uc544\ud558\ub294 \uc2f1\uae00 \uc568\ubc94\"\uc744 \uc8fc\uc81c\ub85c \uc124\ubb38\uc744 \uc9c4\ud589\ud588\ub294\ub370, 39.0%(\ubcf5\uc218\uc751\ub2f5)\uc758 \ub4dd\ud45c\ub97c \uae30\ub85d\ud558\uba70 \u3008\uc9c0\uc9c0\ub9c8\uc138\uc694\u3009, \u3008\ud754\ub4e4\ub9ac\ub294 \ub9c8\uc74c\u3009, \u3008\ub9c8\uc774 \ud504\ub80c\ub4dc\u3009, \u3008\uc6b4\uba85\uc758 \ub8f0\ub81b\uc744 \ub3cc\ub9ac\uba70\u3009\uc5d0 \uc774\uc5b4 \ub2e4\uc12f \ubc88\uc9f8\ub85c \ub192\uc740 \uc9c0\uc9c0\ub97c \uc5bb\uc5c8\ub2e4. \uc774 \ud22c\ud45c \uacb0\uacfc\ub294 \uc624\ub9ac\ucf58\uc774 \uc81c\uacf5\ud558\ub294 ZARD \ucf58\ud150\uce20 \ub0b4 \"ZARD\uc758 \uc88b\uc544\ud558\ub294 \uc2f1\uae00\uc744 \uc77c\uac70\uc18c\uac1c\"\ub77c\ub294 \uc81c\ubaa9\uc758 \ud2b9\uc9d1 \ud398\uc774\uc9c0\uc5d0 \ubc18\uc601\ub418\uc5b4 \uc788\ub2e4. \uac19\uc740 \ub2ec 27\uc77c, \uadf8\ub140\uc758 \ucd94\ubaa8\ud68c\uc5d0\uc11c \uc0c8 \ubca0\uc2a4\ud2b8 \uc568\ubc94 \uc81c\uc791\uc774 \uc5b8\uae09\ub418\uc5c8\ub294\ub370, \uc774 \uc568\ubc94\uc740 \ud32c\ub4e4\uc774 \uc778\ud130\ub137\uacfc \uc5fd\uc11c\ub97c \ud1b5\ud574 \uc218\ub85d\uace1\uc744 \uacb0\uc815\ud560 \uc218 \uc788\uc5c8\ub2e4. \ucd1d 50\ub9cc \ud45c \uc774\uc0c1\uc774 \uc9d1\uacc4\ub418\uc5c8\uc73c\uba70, \u3008\uadf8 \ubbf8\uc18c\ub97c \uc78a\uc9c0 \ub9d0\uc544\uc694\u3009, \u3008\uc18c\ub140 \uc2dc\uc808\ub85c \ub418\ub3cc\uc544\uac04 \uac83\ucc98\ub7fc\u3009, \u3008\ub354\uc5c6\uc774 \uc18c\uc911\ud55c \uac83\u3009\uc5d0 \uc774\uc5b4 4\uc704\ub97c \ucc28\uc9c0\ud588\ub2e4. \ud22c\ud45c\ub97c \ud1b5\ud574 \ub9cc\ub4e4\uc5b4\uc9c4 \uc568\ubc94\uc740 2008\ub144 1\uc6d4 \u300aZARD Request Best ~beautiful memory~\u300b\ub77c\ub294 \uc774\ub984\uc73c\ub85c\uc11c \ubc1c\ub9e4\ub418\uc5c8\uace0, \uace1\uc740 2007\ub144 \ucd94\ubaa8 \ub77c\uc774\ube0c\uc5d0\uc11c \ubd88\ub838\ub358 \ubc84\uc804\uc774 \uc218\ub85d\ub418\uc5c8\ub2e4.", "answer": "\ub2e4\uc12f \ubc88\uc9f8", "sentence": "\uc0ac\uce74\uc774\uac00 \uc0ac\ub9dd\ud55c \ubc14\ub85c \ub2e4\uc74c \ub2ec\uc778 2007\ub144 6\uc6d4, \uc624\ub9ac\ucf58\uc5d0\uc11c\ub294 \ud32c\ub4e4\uc744 \ub300\uc0c1\uc73c\ub85c \"ZARD\uc758 \uc791\ud488 \uc911\uc5d0\uc11c \uc88b\uc544\ud558\ub294 \uc2f1\uae00 \uc568\ubc94\"\uc744 \uc8fc\uc81c\ub85c \uc124\ubb38\uc744 \uc9c4\ud589\ud588\ub294\ub370, 39.0%(\ubcf5\uc218\uc751\ub2f5)\uc758 \ub4dd\ud45c\ub97c \uae30\ub85d\ud558\uba70 \u3008\uc9c0\uc9c0\ub9c8\uc138\uc694\u3009, \u3008\ud754\ub4e4\ub9ac\ub294 \ub9c8\uc74c\u3009, \u3008\ub9c8\uc774 \ud504\ub80c\ub4dc\u3009, \u3008\uc6b4\uba85\uc758 \ub8f0\ub81b\uc744 \ub3cc\ub9ac\uba70\u3009\uc5d0 \uc774\uc5b4 \ub2e4\uc12f \ubc88\uc9f8 \ub85c \ub192\uc740 \uc9c0\uc9c0\ub97c \uc5bb\uc5c8\ub2e4.", "paragraph_sentence": "\u3008\uc601\uc6d0\u3009\uc740 ZARD\uc758 \ud32c\ub4e4\uc5d0\uac8c \ub9ce\uc740 \uc0ac\ub791\uc744 \ubc1b\uc558\ub358 \uace1\uc774\uae30\ub3c4 \ud558\ub2e4. \uc0ac\uce74\uc774\uac00 \uc0ac\ub9dd\ud55c \ubc14\ub85c \ub2e4\uc74c \ub2ec\uc778 2007\ub144 6\uc6d4, \uc624\ub9ac\ucf58\uc5d0\uc11c\ub294 \ud32c\ub4e4\uc744 \ub300\uc0c1\uc73c\ub85c \"ZARD\uc758 \uc791\ud488 \uc911\uc5d0\uc11c \uc88b\uc544\ud558\ub294 \uc2f1\uae00 \uc568\ubc94\"\uc744 \uc8fc\uc81c\ub85c \uc124\ubb38\uc744 \uc9c4\ud589\ud588\ub294\ub370, 39.0%(\ubcf5\uc218\uc751\ub2f5)\uc758 \ub4dd\ud45c\ub97c \uae30\ub85d\ud558\uba70 \u3008\uc9c0\uc9c0\ub9c8\uc138\uc694\u3009, \u3008\ud754\ub4e4\ub9ac\ub294 \ub9c8\uc74c\u3009, \u3008\ub9c8\uc774 \ud504\ub80c\ub4dc\u3009, \u3008\uc6b4\uba85\uc758 \ub8f0\ub81b\uc744 \ub3cc\ub9ac\uba70\u3009\uc5d0 \uc774\uc5b4 \ub2e4\uc12f \ubc88\uc9f8 \ub85c \ub192\uc740 \uc9c0\uc9c0\ub97c \uc5bb\uc5c8\ub2e4. \uc774 \ud22c\ud45c \uacb0\uacfc\ub294 \uc624\ub9ac\ucf58\uc774 \uc81c\uacf5\ud558\ub294 ZARD \ucf58\ud150\uce20 \ub0b4 \"ZARD\uc758 \uc88b\uc544\ud558\ub294 \uc2f1\uae00\uc744 \uc77c\uac70\uc18c\uac1c\"\ub77c\ub294 \uc81c\ubaa9\uc758 \ud2b9\uc9d1 \ud398\uc774\uc9c0\uc5d0 \ubc18\uc601\ub418\uc5b4 \uc788\ub2e4. \uac19\uc740 \ub2ec 27\uc77c, \uadf8\ub140\uc758 \ucd94\ubaa8\ud68c\uc5d0\uc11c \uc0c8 \ubca0\uc2a4\ud2b8 \uc568\ubc94 \uc81c\uc791\uc774 \uc5b8\uae09\ub418\uc5c8\ub294\ub370, \uc774 \uc568\ubc94\uc740 \ud32c\ub4e4\uc774 \uc778\ud130\ub137\uacfc \uc5fd\uc11c\ub97c \ud1b5\ud574 \uc218\ub85d\uace1\uc744 \uacb0\uc815\ud560 \uc218 \uc788\uc5c8\ub2e4. \ucd1d 50\ub9cc \ud45c \uc774\uc0c1\uc774 \uc9d1\uacc4\ub418\uc5c8\uc73c\uba70, \u3008\uadf8 \ubbf8\uc18c\ub97c \uc78a\uc9c0 \ub9d0\uc544\uc694\u3009, \u3008\uc18c\ub140 \uc2dc\uc808\ub85c \ub418\ub3cc\uc544\uac04 \uac83\ucc98\ub7fc\u3009, \u3008\ub354\uc5c6\uc774 \uc18c\uc911\ud55c \uac83\u3009\uc5d0 \uc774\uc5b4 4\uc704\ub97c \ucc28\uc9c0\ud588\ub2e4. \ud22c\ud45c\ub97c \ud1b5\ud574 \ub9cc\ub4e4\uc5b4\uc9c4 \uc568\ubc94\uc740 2008\ub144 1\uc6d4 \u300aZARD Request Best ~beautiful memory~\u300b\ub77c\ub294 \uc774\ub984\uc73c\ub85c\uc11c \ubc1c\ub9e4\ub418\uc5c8\uace0, \uace1\uc740 2007\ub144 \ucd94\ubaa8 \ub77c\uc774\ube0c\uc5d0\uc11c \ubd88\ub838\ub358 \ubc84\uc804\uc774 \uc218\ub85d\ub418\uc5c8\ub2e4.", "paragraph_answer": "\u3008\uc601\uc6d0\u3009\uc740 ZARD\uc758 \ud32c\ub4e4\uc5d0\uac8c \ub9ce\uc740 \uc0ac\ub791\uc744 \ubc1b\uc558\ub358 \uace1\uc774\uae30\ub3c4 \ud558\ub2e4. \uc0ac\uce74\uc774\uac00 \uc0ac\ub9dd\ud55c \ubc14\ub85c \ub2e4\uc74c \ub2ec\uc778 2007\ub144 6\uc6d4, \uc624\ub9ac\ucf58\uc5d0\uc11c\ub294 \ud32c\ub4e4\uc744 \ub300\uc0c1\uc73c\ub85c \"ZARD\uc758 \uc791\ud488 \uc911\uc5d0\uc11c \uc88b\uc544\ud558\ub294 \uc2f1\uae00 \uc568\ubc94\"\uc744 \uc8fc\uc81c\ub85c \uc124\ubb38\uc744 \uc9c4\ud589\ud588\ub294\ub370, 39.0%(\ubcf5\uc218\uc751\ub2f5)\uc758 \ub4dd\ud45c\ub97c \uae30\ub85d\ud558\uba70 \u3008\uc9c0\uc9c0\ub9c8\uc138\uc694\u3009, \u3008\ud754\ub4e4\ub9ac\ub294 \ub9c8\uc74c\u3009, \u3008\ub9c8\uc774 \ud504\ub80c\ub4dc\u3009, \u3008\uc6b4\uba85\uc758 \ub8f0\ub81b\uc744 \ub3cc\ub9ac\uba70\u3009\uc5d0 \uc774\uc5b4 \ub2e4\uc12f \ubc88\uc9f8 \ub85c \ub192\uc740 \uc9c0\uc9c0\ub97c \uc5bb\uc5c8\ub2e4. \uc774 \ud22c\ud45c \uacb0\uacfc\ub294 \uc624\ub9ac\ucf58\uc774 \uc81c\uacf5\ud558\ub294 ZARD \ucf58\ud150\uce20 \ub0b4 \"ZARD\uc758 \uc88b\uc544\ud558\ub294 \uc2f1\uae00\uc744 \uc77c\uac70\uc18c\uac1c\"\ub77c\ub294 \uc81c\ubaa9\uc758 \ud2b9\uc9d1 \ud398\uc774\uc9c0\uc5d0 \ubc18\uc601\ub418\uc5b4 \uc788\ub2e4. \uac19\uc740 \ub2ec 27\uc77c, \uadf8\ub140\uc758 \ucd94\ubaa8\ud68c\uc5d0\uc11c \uc0c8 \ubca0\uc2a4\ud2b8 \uc568\ubc94 \uc81c\uc791\uc774 \uc5b8\uae09\ub418\uc5c8\ub294\ub370, \uc774 \uc568\ubc94\uc740 \ud32c\ub4e4\uc774 \uc778\ud130\ub137\uacfc \uc5fd\uc11c\ub97c \ud1b5\ud574 \uc218\ub85d\uace1\uc744 \uacb0\uc815\ud560 \uc218 \uc788\uc5c8\ub2e4. \ucd1d 50\ub9cc \ud45c \uc774\uc0c1\uc774 \uc9d1\uacc4\ub418\uc5c8\uc73c\uba70, \u3008\uadf8 \ubbf8\uc18c\ub97c \uc78a\uc9c0 \ub9d0\uc544\uc694\u3009, \u3008\uc18c\ub140 \uc2dc\uc808\ub85c \ub418\ub3cc\uc544\uac04 \uac83\ucc98\ub7fc\u3009, \u3008\ub354\uc5c6\uc774 \uc18c\uc911\ud55c \uac83\u3009\uc5d0 \uc774\uc5b4 4\uc704\ub97c \ucc28\uc9c0\ud588\ub2e4. \ud22c\ud45c\ub97c \ud1b5\ud574 \ub9cc\ub4e4\uc5b4\uc9c4 \uc568\ubc94\uc740 2008\ub144 1\uc6d4 \u300aZARD Request Best ~beautiful memory~\u300b\ub77c\ub294 \uc774\ub984\uc73c\ub85c\uc11c \ubc1c\ub9e4\ub418\uc5c8\uace0, \uace1\uc740 2007\ub144 \ucd94\ubaa8 \ub77c\uc774\ube0c\uc5d0\uc11c \ubd88\ub838\ub358 \ubc84\uc804\uc774 \uc218\ub85d\ub418\uc5c8\ub2e4.", "sentence_answer": "\uc0ac\uce74\uc774\uac00 \uc0ac\ub9dd\ud55c \ubc14\ub85c \ub2e4\uc74c \ub2ec\uc778 2007\ub144 6\uc6d4, \uc624\ub9ac\ucf58\uc5d0\uc11c\ub294 \ud32c\ub4e4\uc744 \ub300\uc0c1\uc73c\ub85c \"ZARD\uc758 \uc791\ud488 \uc911\uc5d0\uc11c \uc88b\uc544\ud558\ub294 \uc2f1\uae00 \uc568\ubc94\"\uc744 \uc8fc\uc81c\ub85c \uc124\ubb38\uc744 \uc9c4\ud589\ud588\ub294\ub370, 39.0%(\ubcf5\uc218\uc751\ub2f5)\uc758 \ub4dd\ud45c\ub97c \uae30\ub85d\ud558\uba70 \u3008\uc9c0\uc9c0\ub9c8\uc138\uc694\u3009, \u3008\ud754\ub4e4\ub9ac\ub294 \ub9c8\uc74c\u3009, \u3008\ub9c8\uc774 \ud504\ub80c\ub4dc\u3009, \u3008\uc6b4\uba85\uc758 \ub8f0\ub81b\uc744 \ub3cc\ub9ac\uba70\u3009\uc5d0 \uc774\uc5b4 \ub2e4\uc12f \ubc88\uc9f8 \ub85c \ub192\uc740 \uc9c0\uc9c0\ub97c \uc5bb\uc5c8\ub2e4.", "paragraph_id": "6458830-6-0", "paragraph_question": "question: 2007\ub144 6\uc6d4\uc5d0 \"ZARD\uc758 \uc791\ud488 \uc911 \uc88b\uc544\ud558\ub294 \uc2f1\uae00 \uc568\ubc94\"\uc744 \uc8fc\uc81c\ub85c \ud55c \uc124\ubb38\uc5d0\uc11c <\uc601\uc6d0>\uc740 \uba87 \ubc88\uc9f8\ub85c \ub192\uc740 \uc9c0\uc9c0\ub97c \uc5bb\uc5c8\ub294\uac00?, context: \u3008\uc601\uc6d0\u3009\uc740 ZARD\uc758 \ud32c\ub4e4\uc5d0\uac8c \ub9ce\uc740 \uc0ac\ub791\uc744 \ubc1b\uc558\ub358 \uace1\uc774\uae30\ub3c4 \ud558\ub2e4. \uc0ac\uce74\uc774\uac00 \uc0ac\ub9dd\ud55c \ubc14\ub85c \ub2e4\uc74c \ub2ec\uc778 2007\ub144 6\uc6d4, \uc624\ub9ac\ucf58\uc5d0\uc11c\ub294 \ud32c\ub4e4\uc744 \ub300\uc0c1\uc73c\ub85c \"ZARD\uc758 \uc791\ud488 \uc911\uc5d0\uc11c \uc88b\uc544\ud558\ub294 \uc2f1\uae00 \uc568\ubc94\"\uc744 \uc8fc\uc81c\ub85c \uc124\ubb38\uc744 \uc9c4\ud589\ud588\ub294\ub370, 39.0%(\ubcf5\uc218\uc751\ub2f5)\uc758 \ub4dd\ud45c\ub97c \uae30\ub85d\ud558\uba70 \u3008\uc9c0\uc9c0\ub9c8\uc138\uc694\u3009, \u3008\ud754\ub4e4\ub9ac\ub294 \ub9c8\uc74c\u3009, \u3008\ub9c8\uc774 \ud504\ub80c\ub4dc\u3009, \u3008\uc6b4\uba85\uc758 \ub8f0\ub81b\uc744 \ub3cc\ub9ac\uba70\u3009\uc5d0 \uc774\uc5b4 \ub2e4\uc12f \ubc88\uc9f8\ub85c \ub192\uc740 \uc9c0\uc9c0\ub97c \uc5bb\uc5c8\ub2e4. \uc774 \ud22c\ud45c \uacb0\uacfc\ub294 \uc624\ub9ac\ucf58\uc774 \uc81c\uacf5\ud558\ub294 ZARD \ucf58\ud150\uce20 \ub0b4 \"ZARD\uc758 \uc88b\uc544\ud558\ub294 \uc2f1\uae00\uc744 \uc77c\uac70\uc18c\uac1c\"\ub77c\ub294 \uc81c\ubaa9\uc758 \ud2b9\uc9d1 \ud398\uc774\uc9c0\uc5d0 \ubc18\uc601\ub418\uc5b4 \uc788\ub2e4. \uac19\uc740 \ub2ec 27\uc77c, \uadf8\ub140\uc758 \ucd94\ubaa8\ud68c\uc5d0\uc11c \uc0c8 \ubca0\uc2a4\ud2b8 \uc568\ubc94 \uc81c\uc791\uc774 \uc5b8\uae09\ub418\uc5c8\ub294\ub370, \uc774 \uc568\ubc94\uc740 \ud32c\ub4e4\uc774 \uc778\ud130\ub137\uacfc \uc5fd\uc11c\ub97c \ud1b5\ud574 \uc218\ub85d\uace1\uc744 \uacb0\uc815\ud560 \uc218 \uc788\uc5c8\ub2e4. \ucd1d 50\ub9cc \ud45c \uc774\uc0c1\uc774 \uc9d1\uacc4\ub418\uc5c8\uc73c\uba70, \u3008\uadf8 \ubbf8\uc18c\ub97c \uc78a\uc9c0 \ub9d0\uc544\uc694\u3009, \u3008\uc18c\ub140 \uc2dc\uc808\ub85c \ub418\ub3cc\uc544\uac04 \uac83\ucc98\ub7fc\u3009, \u3008\ub354\uc5c6\uc774 \uc18c\uc911\ud55c \uac83\u3009\uc5d0 \uc774\uc5b4 4\uc704\ub97c \ucc28\uc9c0\ud588\ub2e4. \ud22c\ud45c\ub97c \ud1b5\ud574 \ub9cc\ub4e4\uc5b4\uc9c4 \uc568\ubc94\uc740 2008\ub144 1\uc6d4 \u300aZARD Request Best ~beautiful memory~\u300b\ub77c\ub294 \uc774\ub984\uc73c\ub85c\uc11c \ubc1c\ub9e4\ub418\uc5c8\uace0, \uace1\uc740 2007\ub144 \ucd94\ubaa8 \ub77c\uc774\ube0c\uc5d0\uc11c \ubd88\ub838\ub358 \ubc84\uc804\uc774 \uc218\ub85d\ub418\uc5c8\ub2e4."} {"question": "\ud30c\ub098\ud2f0\ub124\ucf54 \uacbd\uae30\uc7a5\uc740 1896\ub144 \uc5f4\ub9b0 \uc544\ud14c\ub124 \uc62c\ub9bc\ud53d \uc900\ube44\ub97c \uc704\ud574 \uba87 \ubc88\uc774\ub098 \uc815\ube44\ud588\ub098?", "paragraph": "1859\ub144 \uc790\ud30c\uc2a4 \uc62c\ub9bc\ud53d\uc5d0 \ucc38\uac00\ud55c \uc120\uc218\uc758 \uc218\ub294 250 \uba85\uc744 \ub118\uc9c0 \ubabb\ud588\ub2e4. \uc5d0\ubc29\uac94\ub9ac\uc2a4 \uc790\ud30c\uc2a4\ub294 \"\uc9c0\ub09c \uc790\ud30c\uc2a4 \uc62c\ub9bc\ud53d\uc744 \ud3ec\ud568, 1896\ub144\uc5d0 \uac1c\ucd5c\ub420 2\ubc88\uc9f8 \uc62c\ub9bc\ud53d\uc744 \uc704\ud574 \ud30c\ub098\ud2f0\ub124\ucf54 \uacbd\uae30\uc7a5\uc744 \ubcf4\uc218 \ud574\uc57c\ud55c\ub2e4.\" \ub77c\ub294 \ucda9\uace0\ub97c \ud558\uc9c0\ub9cc, \uadf8\ub9ac\uc2a4 \uc815\ubd80\ub294 \uadf8\uc758 \ub9d0\uc744 \ub4e3\uc9c0 \uc54a\uc558\uace0 \uacb0\uad6d 1896\ub144 \uc544\ud14c\ub124 \uc62c\ub9bc\ud53d \uc900\ube44\ub97c \uc704\ud574 \ud30c\ub098\ud2f0\ub124\ucf54 \uacbd\uae30\uc7a5\uc740 \ub450 \ubc88\uc774\ub098 \uc815\ube44\ud574\uc57c \ud588\ub2e4. 1\ud68c \ub300\ud68c \uc815\uc2dd\uc885\ubaa9\uc73c\ub85c\ub294 9\uc885\ubaa9\uc774 \uc788\uc5c8\ub294\ub370 \uc721\uc0c1, \uc0ac\uc774\ud074, \ud39c\uc2f1, \uccb4\uc870, \uc0ac\uaca9, \uc218\uc601, \ud14c\ub2c8\uc2a4, \uc5ed\ub3c4, \ub808\uc2ac\ub9c1\uc774 \uc788\uc5c8\uc73c\uba70, \uc870\uc815\ub3c4 \uc815\uc2dd\uc885\ubaa9\uc774\uc5c8\uc73c\ub098 \ub9e4\uc6b0 \ub098\uc05c \ub0a0\uc528\ub85c \uc778\ud574 \uc870\uc815 \uacbd\uae30\ub294 \ucde8\uc18c\ub418\uc5c8\ub2e4. \ud39c\uc2f1 \uacbd\uae30\ub294 \uc5ed\uc0ac\uc801 \uac74\ubb3c\uc778 \uc790\ud53c\uc628(\uc5d0\ubc18\uc824\ub9ac\uc2a4 \uc790\ud30c\uc2a4\uc758 \uc774\ub984\uc744 \ub534 \uac83\uc774\ub2e4)\uc5d0\uc11c \uc5f4\ub838\ub2e4. \uadf8\ub9ac\uc2a4\uc758 \uad00\ub9ac\ub4e4\uacfc \uad6d\ubbfc\ub4e4\uc740 \uc62c\ub9bc\ud53d \uacbd\uae30 \uac1c\ucd5c\uc5d0 \uc5f4\uad11\uc801\uc774\uc5c8\ub2e4. \ub9ce\uc740 \uc120\uc218\ub4e4\uc774 \uc774\uc5d0 \ub3d9\uac10\ud558\uba74\uc11c \uc55e\uc73c\ub85c\ub3c4 \uc62c\ub9bc\ud53d \ub300\ud68c\ub97c \uc544\ud14c\ub124\uc5d0\uc11c \uc601\uad6c\ud788 \uac1c\ucd5c\ud574\uc57c \ud55c\ub2e4\uace0 \uc694\uad6c\ud558\uae30\uae4c\uc9c0 \ud558\uc600\ub2e4. \uadf8\ub7ec\ub098 \uad6d\uc81c\uc62c\ub9bc\ud53d\uc704\uc6d0\ud68c(IOC)\ub294 \uadfc\ub300 \uc62c\ub9bc\ud53d\uc740 \uc21c\ud658 \uac1c\ucd5c\ub85c \uc5f4\ub9ac\ub294 \uc138\uacc4\uc801\uc778 \ud589\uc0ac\uac00 \ub418\uc5b4\uc57c \ud55c\ub2e4\uace0 \uc0dd\uac01\ud588\ub2e4. \uacb0\uad6d 2\ud68c \uc62c\ub9bc\ud53d\uc740 \ud504\ub791\uc2a4 \ud30c\ub9ac\uc5d0\uc11c \uc5f4\uae30\ub85c \uacb0\uc815\ub418\uc5c8\ub2e4.", "answer": "\ub450 \ubc88", "sentence": "\uc5d0\ubc29\uac94\ub9ac\uc2a4 \uc790\ud30c\uc2a4\ub294 \"\uc9c0\ub09c \uc790\ud30c\uc2a4 \uc62c\ub9bc\ud53d\uc744 \ud3ec\ud568, 1896\ub144\uc5d0 \uac1c\ucd5c\ub420 2\ubc88\uc9f8 \uc62c\ub9bc\ud53d\uc744 \uc704\ud574 \ud30c\ub098\ud2f0\ub124\ucf54 \uacbd\uae30\uc7a5\uc744 \ubcf4\uc218 \ud574\uc57c\ud55c\ub2e4.\" \ub77c\ub294 \ucda9\uace0\ub97c \ud558\uc9c0\ub9cc, \uadf8\ub9ac\uc2a4 \uc815\ubd80\ub294 \uadf8\uc758 \ub9d0\uc744 \ub4e3\uc9c0 \uc54a\uc558\uace0 \uacb0\uad6d 1896\ub144 \uc544\ud14c\ub124 \uc62c\ub9bc\ud53d \uc900\ube44\ub97c \uc704\ud574 \ud30c\ub098\ud2f0\ub124\ucf54 \uacbd\uae30\uc7a5\uc740 \ub450 \ubc88 \uc774\ub098 \uc815\ube44\ud574\uc57c \ud588\ub2e4.", "paragraph_sentence": "1859\ub144 \uc790\ud30c\uc2a4 \uc62c\ub9bc\ud53d\uc5d0 \ucc38\uac00\ud55c \uc120\uc218\uc758 \uc218\ub294 250 \uba85\uc744 \ub118\uc9c0 \ubabb\ud588\ub2e4. \uc5d0\ubc29\uac94\ub9ac\uc2a4 \uc790\ud30c\uc2a4\ub294 \"\uc9c0\ub09c \uc790\ud30c\uc2a4 \uc62c\ub9bc\ud53d\uc744 \ud3ec\ud568, 1896\ub144\uc5d0 \uac1c\ucd5c\ub420 2\ubc88\uc9f8 \uc62c\ub9bc\ud53d\uc744 \uc704\ud574 \ud30c\ub098\ud2f0\ub124\ucf54 \uacbd\uae30\uc7a5\uc744 \ubcf4\uc218 \ud574\uc57c\ud55c\ub2e4.\" \ub77c\ub294 \ucda9\uace0\ub97c \ud558\uc9c0\ub9cc, \uadf8\ub9ac\uc2a4 \uc815\ubd80\ub294 \uadf8\uc758 \ub9d0\uc744 \ub4e3\uc9c0 \uc54a\uc558\uace0 \uacb0\uad6d 1896\ub144 \uc544\ud14c\ub124 \uc62c\ub9bc\ud53d \uc900\ube44\ub97c \uc704\ud574 \ud30c\ub098\ud2f0\ub124\ucf54 \uacbd\uae30\uc7a5\uc740 \ub450 \ubc88 \uc774\ub098 \uc815\ube44\ud574\uc57c \ud588\ub2e4. 1\ud68c \ub300\ud68c \uc815\uc2dd\uc885\ubaa9\uc73c\ub85c\ub294 9\uc885\ubaa9\uc774 \uc788\uc5c8\ub294\ub370 \uc721\uc0c1, \uc0ac\uc774\ud074, \ud39c\uc2f1, \uccb4\uc870, \uc0ac\uaca9, \uc218\uc601, \ud14c\ub2c8\uc2a4, \uc5ed\ub3c4, \ub808\uc2ac\ub9c1\uc774 \uc788\uc5c8\uc73c\uba70, \uc870\uc815\ub3c4 \uc815\uc2dd\uc885\ubaa9\uc774\uc5c8\uc73c\ub098 \ub9e4\uc6b0 \ub098\uc05c \ub0a0\uc528\ub85c \uc778\ud574 \uc870\uc815 \uacbd\uae30\ub294 \ucde8\uc18c\ub418\uc5c8\ub2e4. \ud39c\uc2f1 \uacbd\uae30\ub294 \uc5ed\uc0ac\uc801 \uac74\ubb3c\uc778 \uc790\ud53c\uc628(\uc5d0\ubc18\uc824\ub9ac\uc2a4 \uc790\ud30c\uc2a4\uc758 \uc774\ub984\uc744 \ub534 \uac83\uc774\ub2e4)\uc5d0\uc11c \uc5f4\ub838\ub2e4. \uadf8\ub9ac\uc2a4\uc758 \uad00\ub9ac\ub4e4\uacfc \uad6d\ubbfc\ub4e4\uc740 \uc62c\ub9bc\ud53d \uacbd\uae30 \uac1c\ucd5c\uc5d0 \uc5f4\uad11\uc801\uc774\uc5c8\ub2e4. \ub9ce\uc740 \uc120\uc218\ub4e4\uc774 \uc774\uc5d0 \ub3d9\uac10\ud558\uba74\uc11c \uc55e\uc73c\ub85c\ub3c4 \uc62c\ub9bc\ud53d \ub300\ud68c\ub97c \uc544\ud14c\ub124\uc5d0\uc11c \uc601\uad6c\ud788 \uac1c\ucd5c\ud574\uc57c \ud55c\ub2e4\uace0 \uc694\uad6c\ud558\uae30\uae4c\uc9c0 \ud558\uc600\ub2e4. \uadf8\ub7ec\ub098 \uad6d\uc81c\uc62c\ub9bc\ud53d\uc704\uc6d0\ud68c(IOC)\ub294 \uadfc\ub300 \uc62c\ub9bc\ud53d\uc740 \uc21c\ud658 \uac1c\ucd5c\ub85c \uc5f4\ub9ac\ub294 \uc138\uacc4\uc801\uc778 \ud589\uc0ac\uac00 \ub418\uc5b4\uc57c \ud55c\ub2e4\uace0 \uc0dd\uac01\ud588\ub2e4. \uacb0\uad6d 2\ud68c \uc62c\ub9bc\ud53d\uc740 \ud504\ub791\uc2a4 \ud30c\ub9ac\uc5d0\uc11c \uc5f4\uae30\ub85c \uacb0\uc815\ub418\uc5c8\ub2e4.", "paragraph_answer": "1859\ub144 \uc790\ud30c\uc2a4 \uc62c\ub9bc\ud53d\uc5d0 \ucc38\uac00\ud55c \uc120\uc218\uc758 \uc218\ub294 250 \uba85\uc744 \ub118\uc9c0 \ubabb\ud588\ub2e4. \uc5d0\ubc29\uac94\ub9ac\uc2a4 \uc790\ud30c\uc2a4\ub294 \"\uc9c0\ub09c \uc790\ud30c\uc2a4 \uc62c\ub9bc\ud53d\uc744 \ud3ec\ud568, 1896\ub144\uc5d0 \uac1c\ucd5c\ub420 2\ubc88\uc9f8 \uc62c\ub9bc\ud53d\uc744 \uc704\ud574 \ud30c\ub098\ud2f0\ub124\ucf54 \uacbd\uae30\uc7a5\uc744 \ubcf4\uc218 \ud574\uc57c\ud55c\ub2e4.\" \ub77c\ub294 \ucda9\uace0\ub97c \ud558\uc9c0\ub9cc, \uadf8\ub9ac\uc2a4 \uc815\ubd80\ub294 \uadf8\uc758 \ub9d0\uc744 \ub4e3\uc9c0 \uc54a\uc558\uace0 \uacb0\uad6d 1896\ub144 \uc544\ud14c\ub124 \uc62c\ub9bc\ud53d \uc900\ube44\ub97c \uc704\ud574 \ud30c\ub098\ud2f0\ub124\ucf54 \uacbd\uae30\uc7a5\uc740 \ub450 \ubc88 \uc774\ub098 \uc815\ube44\ud574\uc57c \ud588\ub2e4. 1\ud68c \ub300\ud68c \uc815\uc2dd\uc885\ubaa9\uc73c\ub85c\ub294 9\uc885\ubaa9\uc774 \uc788\uc5c8\ub294\ub370 \uc721\uc0c1, \uc0ac\uc774\ud074, \ud39c\uc2f1, \uccb4\uc870, \uc0ac\uaca9, \uc218\uc601, \ud14c\ub2c8\uc2a4, \uc5ed\ub3c4, \ub808\uc2ac\ub9c1\uc774 \uc788\uc5c8\uc73c\uba70, \uc870\uc815\ub3c4 \uc815\uc2dd\uc885\ubaa9\uc774\uc5c8\uc73c\ub098 \ub9e4\uc6b0 \ub098\uc05c \ub0a0\uc528\ub85c \uc778\ud574 \uc870\uc815 \uacbd\uae30\ub294 \ucde8\uc18c\ub418\uc5c8\ub2e4. \ud39c\uc2f1 \uacbd\uae30\ub294 \uc5ed\uc0ac\uc801 \uac74\ubb3c\uc778 \uc790\ud53c\uc628(\uc5d0\ubc18\uc824\ub9ac\uc2a4 \uc790\ud30c\uc2a4\uc758 \uc774\ub984\uc744 \ub534 \uac83\uc774\ub2e4)\uc5d0\uc11c \uc5f4\ub838\ub2e4. \uadf8\ub9ac\uc2a4\uc758 \uad00\ub9ac\ub4e4\uacfc \uad6d\ubbfc\ub4e4\uc740 \uc62c\ub9bc\ud53d \uacbd\uae30 \uac1c\ucd5c\uc5d0 \uc5f4\uad11\uc801\uc774\uc5c8\ub2e4. \ub9ce\uc740 \uc120\uc218\ub4e4\uc774 \uc774\uc5d0 \ub3d9\uac10\ud558\uba74\uc11c \uc55e\uc73c\ub85c\ub3c4 \uc62c\ub9bc\ud53d \ub300\ud68c\ub97c \uc544\ud14c\ub124\uc5d0\uc11c \uc601\uad6c\ud788 \uac1c\ucd5c\ud574\uc57c \ud55c\ub2e4\uace0 \uc694\uad6c\ud558\uae30\uae4c\uc9c0 \ud558\uc600\ub2e4. \uadf8\ub7ec\ub098 \uad6d\uc81c\uc62c\ub9bc\ud53d\uc704\uc6d0\ud68c(IOC)\ub294 \uadfc\ub300 \uc62c\ub9bc\ud53d\uc740 \uc21c\ud658 \uac1c\ucd5c\ub85c \uc5f4\ub9ac\ub294 \uc138\uacc4\uc801\uc778 \ud589\uc0ac\uac00 \ub418\uc5b4\uc57c \ud55c\ub2e4\uace0 \uc0dd\uac01\ud588\ub2e4. \uacb0\uad6d 2\ud68c \uc62c\ub9bc\ud53d\uc740 \ud504\ub791\uc2a4 \ud30c\ub9ac\uc5d0\uc11c \uc5f4\uae30\ub85c \uacb0\uc815\ub418\uc5c8\ub2e4.", "sentence_answer": "\uc5d0\ubc29\uac94\ub9ac\uc2a4 \uc790\ud30c\uc2a4\ub294 \"\uc9c0\ub09c \uc790\ud30c\uc2a4 \uc62c\ub9bc\ud53d\uc744 \ud3ec\ud568, 1896\ub144\uc5d0 \uac1c\ucd5c\ub420 2\ubc88\uc9f8 \uc62c\ub9bc\ud53d\uc744 \uc704\ud574 \ud30c\ub098\ud2f0\ub124\ucf54 \uacbd\uae30\uc7a5\uc744 \ubcf4\uc218 \ud574\uc57c\ud55c\ub2e4.\" \ub77c\ub294 \ucda9\uace0\ub97c \ud558\uc9c0\ub9cc, \uadf8\ub9ac\uc2a4 \uc815\ubd80\ub294 \uadf8\uc758 \ub9d0\uc744 \ub4e3\uc9c0 \uc54a\uc558\uace0 \uacb0\uad6d 1896\ub144 \uc544\ud14c\ub124 \uc62c\ub9bc\ud53d \uc900\ube44\ub97c \uc704\ud574 \ud30c\ub098\ud2f0\ub124\ucf54 \uacbd\uae30\uc7a5\uc740 \ub450 \ubc88 \uc774\ub098 \uc815\ube44\ud574\uc57c \ud588\ub2e4.", "paragraph_id": "6525547-6-0", "paragraph_question": "question: \ud30c\ub098\ud2f0\ub124\ucf54 \uacbd\uae30\uc7a5\uc740 1896\ub144 \uc5f4\ub9b0 \uc544\ud14c\ub124 \uc62c\ub9bc\ud53d \uc900\ube44\ub97c \uc704\ud574 \uba87 \ubc88\uc774\ub098 \uc815\ube44\ud588\ub098?, context: 1859\ub144 \uc790\ud30c\uc2a4 \uc62c\ub9bc\ud53d\uc5d0 \ucc38\uac00\ud55c \uc120\uc218\uc758 \uc218\ub294 250 \uba85\uc744 \ub118\uc9c0 \ubabb\ud588\ub2e4. \uc5d0\ubc29\uac94\ub9ac\uc2a4 \uc790\ud30c\uc2a4\ub294 \"\uc9c0\ub09c \uc790\ud30c\uc2a4 \uc62c\ub9bc\ud53d\uc744 \ud3ec\ud568, 1896\ub144\uc5d0 \uac1c\ucd5c\ub420 2\ubc88\uc9f8 \uc62c\ub9bc\ud53d\uc744 \uc704\ud574 \ud30c\ub098\ud2f0\ub124\ucf54 \uacbd\uae30\uc7a5\uc744 \ubcf4\uc218 \ud574\uc57c\ud55c\ub2e4.\" \ub77c\ub294 \ucda9\uace0\ub97c \ud558\uc9c0\ub9cc, \uadf8\ub9ac\uc2a4 \uc815\ubd80\ub294 \uadf8\uc758 \ub9d0\uc744 \ub4e3\uc9c0 \uc54a\uc558\uace0 \uacb0\uad6d 1896\ub144 \uc544\ud14c\ub124 \uc62c\ub9bc\ud53d \uc900\ube44\ub97c \uc704\ud574 \ud30c\ub098\ud2f0\ub124\ucf54 \uacbd\uae30\uc7a5\uc740 \ub450 \ubc88\uc774\ub098 \uc815\ube44\ud574\uc57c \ud588\ub2e4. 1\ud68c \ub300\ud68c \uc815\uc2dd\uc885\ubaa9\uc73c\ub85c\ub294 9\uc885\ubaa9\uc774 \uc788\uc5c8\ub294\ub370 \uc721\uc0c1, \uc0ac\uc774\ud074, \ud39c\uc2f1, \uccb4\uc870, \uc0ac\uaca9, \uc218\uc601, \ud14c\ub2c8\uc2a4, \uc5ed\ub3c4, \ub808\uc2ac\ub9c1\uc774 \uc788\uc5c8\uc73c\uba70, \uc870\uc815\ub3c4 \uc815\uc2dd\uc885\ubaa9\uc774\uc5c8\uc73c\ub098 \ub9e4\uc6b0 \ub098\uc05c \ub0a0\uc528\ub85c \uc778\ud574 \uc870\uc815 \uacbd\uae30\ub294 \ucde8\uc18c\ub418\uc5c8\ub2e4. \ud39c\uc2f1 \uacbd\uae30\ub294 \uc5ed\uc0ac\uc801 \uac74\ubb3c\uc778 \uc790\ud53c\uc628(\uc5d0\ubc18\uc824\ub9ac\uc2a4 \uc790\ud30c\uc2a4\uc758 \uc774\ub984\uc744 \ub534 \uac83\uc774\ub2e4)\uc5d0\uc11c \uc5f4\ub838\ub2e4. \uadf8\ub9ac\uc2a4\uc758 \uad00\ub9ac\ub4e4\uacfc \uad6d\ubbfc\ub4e4\uc740 \uc62c\ub9bc\ud53d \uacbd\uae30 \uac1c\ucd5c\uc5d0 \uc5f4\uad11\uc801\uc774\uc5c8\ub2e4. \ub9ce\uc740 \uc120\uc218\ub4e4\uc774 \uc774\uc5d0 \ub3d9\uac10\ud558\uba74\uc11c \uc55e\uc73c\ub85c\ub3c4 \uc62c\ub9bc\ud53d \ub300\ud68c\ub97c \uc544\ud14c\ub124\uc5d0\uc11c \uc601\uad6c\ud788 \uac1c\ucd5c\ud574\uc57c \ud55c\ub2e4\uace0 \uc694\uad6c\ud558\uae30\uae4c\uc9c0 \ud558\uc600\ub2e4. \uadf8\ub7ec\ub098 \uad6d\uc81c\uc62c\ub9bc\ud53d\uc704\uc6d0\ud68c(IOC)\ub294 \uadfc\ub300 \uc62c\ub9bc\ud53d\uc740 \uc21c\ud658 \uac1c\ucd5c\ub85c \uc5f4\ub9ac\ub294 \uc138\uacc4\uc801\uc778 \ud589\uc0ac\uac00 \ub418\uc5b4\uc57c \ud55c\ub2e4\uace0 \uc0dd\uac01\ud588\ub2e4. \uacb0\uad6d 2\ud68c \uc62c\ub9bc\ud53d\uc740 \ud504\ub791\uc2a4 \ud30c\ub9ac\uc5d0\uc11c \uc5f4\uae30\ub85c \uacb0\uc815\ub418\uc5c8\ub2e4."} {"question": "\uc2a4\ud398\uc778\uc774 \ud30c\uacac\ud55c \ub098\ud3f4\ub9ac \ubd80\uc655 \uc911 \ud398\ub4dc\ub85c \uc54c\ubc14\ub808\uc2a4 \ub370\ud1a8\ub808\ub3c4\uac00 \uc9c0\uc9c0\ud588\ub358 \uac83\uc740?", "paragraph": "\uc2dc\uce60\ub9ac\uc544\uc640 \ub098\ud3f4\ub9ac\ub294 1458\ub144\uc5d0 \ub098\ub258\uc5c8\uc9c0\ub9cc \ud398\ub780\ud14c 1\uc138 \uc9c0\ubc30\ud558\uc758 \uc18d\uad6d\uc73c\ub85c \ub0a8\uc544\uc788\uc5c8\ub2e4. \uc0c8 \uc655\uc870\ub294 \ub098\ud3f4\ub9ac\uc758 \ubb34\uc5ed\uc744 \uc774\ubca0\ub9ac\uc544 \ubc18\ub3c4\uc640 \uad00\uacc4\ub97c \uc99d\ub300\uc2dc\ucf30\ub2e4. \ub098\ud3f4\ub9ac\ub294 \ub77c\uc6b0\ub77c\ub098, \ub2e4 \uba54\uc2dc\ub098, \uc0b0\ub098\ucc28\ub85c, \ud3f4\ub9ac\uce58\uc544\ub178 \uac19\uc740 \uc608\uc220\uac00\ub4e4\uc774 \ubaa8\uc5ec\ub4e4\uc5b4 \ub974\ub124\uc0c1\uc2a4\uc758 \uc911\uc2ec\uc9c0\uac00 \ub410\ub2e4. 1501\ub144\uc5d0 \ub098\ud3f4\ub9ac\ub294 \ub098\ud3f4\ub9ac\uc758 \uad6d\uc655 \ud398\ub370\ub9ac\ucf54\uac00 \ud504\ub791\uc2a4\ub85c \uc7a1\ud600\uac00\ub358 \ub8e8\uc774 12\uc138 \uc2dc\uae30\uc5d0 \ud504\ub791\uc2a4\ub85c\ubd80\ud130 \uc9c1\uc811 \ud1b5\uce58\ub97c 4\ub144\uac04 \ubc1b\uc558\ub2e4. \uc2a4\ud398\uc778\uc774 \uac00\ub9b4\ub9ac\uc544\ub178 \uc804\ud22c \uc804\ud22c\uc5d0\uc11c \uc2b9\ub9ac\ud558\uc5ec, \ub098\ud3f4\ub9ac\ub294 \ud569\uc2a4\ubd80\ub974\ud06c \uc2a4\ud398\uc778 \uc2dc\uae30 \ub0b4\ub0b4 \uc2a4\ud398\uc778 \uc81c\uad6d\uc758 \uc77c\ubd80\uac00 \ub418\uc5c8\ub2e4. \uc2a4\ud398\uc778 \uad6d\uc655\uc740 \uc9c0\uc5ed \ubb38\uc81c\ub97c \uc989\uc2dc \ub2e4\ub8e8\uae30\uc704\ud574 \ub098\ud3f4\ub9ac\uc5d0 \ubd80\uc655\uc744 \ud30c\uacac\ud588\ub2e4. \uc774\uc911 \uac00\uc7a5 \uc911\uc694\ud55c \uc778\ubb3c\uc740 \ub3c4\uc2dc\uc18d \uc0ac\ud68c, \uacbd\uc81c, \ub3c4\uc2dc \ubc1c\uc804\ub4f1\uc5d0 \uc0c1\ub2f9\ud55c \ucc45\uc784\uc744 \uac00\uc9c0\uace0 \uc788\uc5c8\uace0 \uc885\uad50 \uc7ac\ud310\uc744 \uc9c0\uc9c0\ud588\ub358 \ud398\ub4dc\ub85c \uc54c\ubc14\ub808\uc2a4 \ub370 \ud1a8\ub808\ub3c4\ub2e4.", "answer": "\uc885\uad50 \uc7ac\ud310", "sentence": "\uc774\uc911 \uac00\uc7a5 \uc911\uc694\ud55c \uc778\ubb3c\uc740 \ub3c4\uc2dc\uc18d \uc0ac\ud68c, \uacbd\uc81c, \ub3c4\uc2dc \ubc1c\uc804\ub4f1\uc5d0 \uc0c1\ub2f9\ud55c \ucc45\uc784\uc744 \uac00\uc9c0\uace0 \uc788\uc5c8\uace0 \uc885\uad50 \uc7ac\ud310 \uc744 \uc9c0\uc9c0\ud588\ub358 \ud398\ub4dc\ub85c \uc54c\ubc14\ub808\uc2a4 \ub370 \ud1a8\ub808\ub3c4\ub2e4.", "paragraph_sentence": "\uc2dc\uce60\ub9ac\uc544\uc640 \ub098\ud3f4\ub9ac\ub294 1458\ub144\uc5d0 \ub098\ub258\uc5c8\uc9c0\ub9cc \ud398\ub780\ud14c 1\uc138 \uc9c0\ubc30\ud558\uc758 \uc18d\uad6d\uc73c\ub85c \ub0a8\uc544\uc788\uc5c8\ub2e4. \uc0c8 \uc655\uc870\ub294 \ub098\ud3f4\ub9ac\uc758 \ubb34\uc5ed\uc744 \uc774\ubca0\ub9ac\uc544 \ubc18\ub3c4\uc640 \uad00\uacc4\ub97c \uc99d\ub300\uc2dc\ucf30\ub2e4. \ub098\ud3f4\ub9ac\ub294 \ub77c\uc6b0\ub77c\ub098, \ub2e4 \uba54\uc2dc\ub098, \uc0b0\ub098\ucc28\ub85c, \ud3f4\ub9ac\uce58\uc544\ub178 \uac19\uc740 \uc608\uc220\uac00\ub4e4\uc774 \ubaa8\uc5ec\ub4e4\uc5b4 \ub974\ub124\uc0c1\uc2a4\uc758 \uc911\uc2ec\uc9c0\uac00 \ub410\ub2e4. 1501\ub144\uc5d0 \ub098\ud3f4\ub9ac\ub294 \ub098\ud3f4\ub9ac\uc758 \uad6d\uc655 \ud398\ub370\ub9ac\ucf54\uac00 \ud504\ub791\uc2a4\ub85c \uc7a1\ud600\uac00\ub358 \ub8e8\uc774 12\uc138 \uc2dc\uae30\uc5d0 \ud504\ub791\uc2a4\ub85c\ubd80\ud130 \uc9c1\uc811 \ud1b5\uce58\ub97c 4\ub144\uac04 \ubc1b\uc558\ub2e4. \uc2a4\ud398\uc778\uc774 \uac00\ub9b4\ub9ac\uc544\ub178 \uc804\ud22c \uc804\ud22c\uc5d0\uc11c \uc2b9\ub9ac\ud558\uc5ec, \ub098\ud3f4\ub9ac\ub294 \ud569\uc2a4\ubd80\ub974\ud06c \uc2a4\ud398\uc778 \uc2dc\uae30 \ub0b4\ub0b4 \uc2a4\ud398\uc778 \uc81c\uad6d\uc758 \uc77c\ubd80\uac00 \ub418\uc5c8\ub2e4. \uc2a4\ud398\uc778 \uad6d\uc655\uc740 \uc9c0\uc5ed \ubb38\uc81c\ub97c \uc989\uc2dc \ub2e4\ub8e8\uae30\uc704\ud574 \ub098\ud3f4\ub9ac\uc5d0 \ubd80\uc655\uc744 \ud30c\uacac\ud588\ub2e4. \uc774\uc911 \uac00\uc7a5 \uc911\uc694\ud55c \uc778\ubb3c\uc740 \ub3c4\uc2dc\uc18d \uc0ac\ud68c, \uacbd\uc81c, \ub3c4\uc2dc \ubc1c\uc804\ub4f1\uc5d0 \uc0c1\ub2f9\ud55c \ucc45\uc784\uc744 \uac00\uc9c0\uace0 \uc788\uc5c8\uace0 \uc885\uad50 \uc7ac\ud310 \uc744 \uc9c0\uc9c0\ud588\ub358 \ud398\ub4dc\ub85c \uc54c\ubc14\ub808\uc2a4 \ub370 \ud1a8\ub808\ub3c4\ub2e4. ", "paragraph_answer": "\uc2dc\uce60\ub9ac\uc544\uc640 \ub098\ud3f4\ub9ac\ub294 1458\ub144\uc5d0 \ub098\ub258\uc5c8\uc9c0\ub9cc \ud398\ub780\ud14c 1\uc138 \uc9c0\ubc30\ud558\uc758 \uc18d\uad6d\uc73c\ub85c \ub0a8\uc544\uc788\uc5c8\ub2e4. \uc0c8 \uc655\uc870\ub294 \ub098\ud3f4\ub9ac\uc758 \ubb34\uc5ed\uc744 \uc774\ubca0\ub9ac\uc544 \ubc18\ub3c4\uc640 \uad00\uacc4\ub97c \uc99d\ub300\uc2dc\ucf30\ub2e4. \ub098\ud3f4\ub9ac\ub294 \ub77c\uc6b0\ub77c\ub098, \ub2e4 \uba54\uc2dc\ub098, \uc0b0\ub098\ucc28\ub85c, \ud3f4\ub9ac\uce58\uc544\ub178 \uac19\uc740 \uc608\uc220\uac00\ub4e4\uc774 \ubaa8\uc5ec\ub4e4\uc5b4 \ub974\ub124\uc0c1\uc2a4\uc758 \uc911\uc2ec\uc9c0\uac00 \ub410\ub2e4. 1501\ub144\uc5d0 \ub098\ud3f4\ub9ac\ub294 \ub098\ud3f4\ub9ac\uc758 \uad6d\uc655 \ud398\ub370\ub9ac\ucf54\uac00 \ud504\ub791\uc2a4\ub85c \uc7a1\ud600\uac00\ub358 \ub8e8\uc774 12\uc138 \uc2dc\uae30\uc5d0 \ud504\ub791\uc2a4\ub85c\ubd80\ud130 \uc9c1\uc811 \ud1b5\uce58\ub97c 4\ub144\uac04 \ubc1b\uc558\ub2e4. \uc2a4\ud398\uc778\uc774 \uac00\ub9b4\ub9ac\uc544\ub178 \uc804\ud22c \uc804\ud22c\uc5d0\uc11c \uc2b9\ub9ac\ud558\uc5ec, \ub098\ud3f4\ub9ac\ub294 \ud569\uc2a4\ubd80\ub974\ud06c \uc2a4\ud398\uc778 \uc2dc\uae30 \ub0b4\ub0b4 \uc2a4\ud398\uc778 \uc81c\uad6d\uc758 \uc77c\ubd80\uac00 \ub418\uc5c8\ub2e4. \uc2a4\ud398\uc778 \uad6d\uc655\uc740 \uc9c0\uc5ed \ubb38\uc81c\ub97c \uc989\uc2dc \ub2e4\ub8e8\uae30\uc704\ud574 \ub098\ud3f4\ub9ac\uc5d0 \ubd80\uc655\uc744 \ud30c\uacac\ud588\ub2e4. \uc774\uc911 \uac00\uc7a5 \uc911\uc694\ud55c \uc778\ubb3c\uc740 \ub3c4\uc2dc\uc18d \uc0ac\ud68c, \uacbd\uc81c, \ub3c4\uc2dc \ubc1c\uc804\ub4f1\uc5d0 \uc0c1\ub2f9\ud55c \ucc45\uc784\uc744 \uac00\uc9c0\uace0 \uc788\uc5c8\uace0 \uc885\uad50 \uc7ac\ud310 \uc744 \uc9c0\uc9c0\ud588\ub358 \ud398\ub4dc\ub85c \uc54c\ubc14\ub808\uc2a4 \ub370 \ud1a8\ub808\ub3c4\ub2e4.", "sentence_answer": "\uc774\uc911 \uac00\uc7a5 \uc911\uc694\ud55c \uc778\ubb3c\uc740 \ub3c4\uc2dc\uc18d \uc0ac\ud68c, \uacbd\uc81c, \ub3c4\uc2dc \ubc1c\uc804\ub4f1\uc5d0 \uc0c1\ub2f9\ud55c \ucc45\uc784\uc744 \uac00\uc9c0\uace0 \uc788\uc5c8\uace0 \uc885\uad50 \uc7ac\ud310 \uc744 \uc9c0\uc9c0\ud588\ub358 \ud398\ub4dc\ub85c \uc54c\ubc14\ub808\uc2a4 \ub370 \ud1a8\ub808\ub3c4\ub2e4.", "paragraph_id": "6464201-4-2", "paragraph_question": "question: \uc2a4\ud398\uc778\uc774 \ud30c\uacac\ud55c \ub098\ud3f4\ub9ac \ubd80\uc655 \uc911 \ud398\ub4dc\ub85c \uc54c\ubc14\ub808\uc2a4 \ub370\ud1a8\ub808\ub3c4\uac00 \uc9c0\uc9c0\ud588\ub358 \uac83\uc740?, context: \uc2dc\uce60\ub9ac\uc544\uc640 \ub098\ud3f4\ub9ac\ub294 1458\ub144\uc5d0 \ub098\ub258\uc5c8\uc9c0\ub9cc \ud398\ub780\ud14c 1\uc138 \uc9c0\ubc30\ud558\uc758 \uc18d\uad6d\uc73c\ub85c \ub0a8\uc544\uc788\uc5c8\ub2e4. \uc0c8 \uc655\uc870\ub294 \ub098\ud3f4\ub9ac\uc758 \ubb34\uc5ed\uc744 \uc774\ubca0\ub9ac\uc544 \ubc18\ub3c4\uc640 \uad00\uacc4\ub97c \uc99d\ub300\uc2dc\ucf30\ub2e4. \ub098\ud3f4\ub9ac\ub294 \ub77c\uc6b0\ub77c\ub098, \ub2e4 \uba54\uc2dc\ub098, \uc0b0\ub098\ucc28\ub85c, \ud3f4\ub9ac\uce58\uc544\ub178 \uac19\uc740 \uc608\uc220\uac00\ub4e4\uc774 \ubaa8\uc5ec\ub4e4\uc5b4 \ub974\ub124\uc0c1\uc2a4\uc758 \uc911\uc2ec\uc9c0\uac00 \ub410\ub2e4. 1501\ub144\uc5d0 \ub098\ud3f4\ub9ac\ub294 \ub098\ud3f4\ub9ac\uc758 \uad6d\uc655 \ud398\ub370\ub9ac\ucf54\uac00 \ud504\ub791\uc2a4\ub85c \uc7a1\ud600\uac00\ub358 \ub8e8\uc774 12\uc138 \uc2dc\uae30\uc5d0 \ud504\ub791\uc2a4\ub85c\ubd80\ud130 \uc9c1\uc811 \ud1b5\uce58\ub97c 4\ub144\uac04 \ubc1b\uc558\ub2e4. \uc2a4\ud398\uc778\uc774 \uac00\ub9b4\ub9ac\uc544\ub178 \uc804\ud22c \uc804\ud22c\uc5d0\uc11c \uc2b9\ub9ac\ud558\uc5ec, \ub098\ud3f4\ub9ac\ub294 \ud569\uc2a4\ubd80\ub974\ud06c \uc2a4\ud398\uc778 \uc2dc\uae30 \ub0b4\ub0b4 \uc2a4\ud398\uc778 \uc81c\uad6d\uc758 \uc77c\ubd80\uac00 \ub418\uc5c8\ub2e4. \uc2a4\ud398\uc778 \uad6d\uc655\uc740 \uc9c0\uc5ed \ubb38\uc81c\ub97c \uc989\uc2dc \ub2e4\ub8e8\uae30\uc704\ud574 \ub098\ud3f4\ub9ac\uc5d0 \ubd80\uc655\uc744 \ud30c\uacac\ud588\ub2e4. \uc774\uc911 \uac00\uc7a5 \uc911\uc694\ud55c \uc778\ubb3c\uc740 \ub3c4\uc2dc\uc18d \uc0ac\ud68c, \uacbd\uc81c, \ub3c4\uc2dc \ubc1c\uc804\ub4f1\uc5d0 \uc0c1\ub2f9\ud55c \ucc45\uc784\uc744 \uac00\uc9c0\uace0 \uc788\uc5c8\uace0 \uc885\uad50 \uc7ac\ud310\uc744 \uc9c0\uc9c0\ud588\ub358 \ud398\ub4dc\ub85c \uc54c\ubc14\ub808\uc2a4 \ub370 \ud1a8\ub808\ub3c4\ub2e4."} {"question": "\ubb38\ud76c\uc625\uc774 \ub2e4\ub154\ub358 \uc11c\uc6b8\uc608\uc220\ub300\ud559\uc758 \ud559\uacfc\ub294?", "paragraph": "1988\ub144 \ubb38\ud76c\uc625\uc740 \uc5b4\ub824\uc6b4 \uac00\uc815\ud615\ud3b8 \uc18d\uc5d0\uc11c\ub3c4 \uc5f4\uc2ec\ud788 \uacf5\ubd80\ud558\uc5ec \uc11c\uc6b8\uc608\uc220\ub300\ud559 \uc2e4\uc6a9\uc74c\uc545\ud559\uacfc\uc5d0 \uc785\ud559\ud558\uc600\uace0 \ub300\ud559\uc0dd \uac00\uc218\uac00 \ub418\uc5c8\uc73c\uba70 \uadf8 \ud574 \uc2ac\ub85c\uc6b0 \ud15c\ud3ec\uc758 \uc815\ud1b5 \ud2b8\ub85c\ud2b8\u300a\uac00\ub294 \ub2d8 \uac00\ub294 \uc815\u300b(\uc815\uc740\uc774 \uc791\uc0ac/ \ub0a8\uad6d\uc778 \uc791\uace1)\uc744 \ubc1c\ud45c\ud558\uc600\ub2e4. \ub370\ubdd4\uc640 \ub3d9\uc2dc\uc5d0 \ud070 \uc131\uacfc\ub97c \uac70\ub450\uc5b4 \ub300\uc911\ub4e4\uc758 \uc0ac\ub791\uc744 \ubc1b\uc558\ub358 \ubb38\ud76c\uc625\uc740 1989\ub144\uc5d0 \uc815\ud1b5 \ud2b8\ub86f \ub9ac\ub4ec\uc744 \uacbd\ucf8c\ud558\uac8c \ud604\ub300\ud654\ud55c \ud3ed\uc2a4 \ud2b8\ub85c\ud2b8\u300a\uc0ac\ub791\uc758 \uac70\ub9ac\u300b(\uc815\uc740\uc774 \uc791\uc0ac/ \ub0a8\uad6d\uc778 \uc791\uace1/ \uae40\uc6a9\ub144 \ud3b8\uace1)\ub85c \ubc29\uc1a1 \uc74c\uc545\ub2e4\ubc29 \ub4f1\uc9c0\uc758 \uc778\uae30\ucc28\ud2b8\uc5d0\uc11c \uc0c1\uc704\uad8c\uc5d0 \uc62c\ub77c \uac00\uc218\ub85c\uc11c\uc758 \uc804\uc131\uae30\ub97c \ub9de\uc774\ud558\uba74\uc11c \ud2b8\ub85c\ud2b8 \uac00\uc218\uc758 \ub9c9\ub0b4\ub85c\uc11c \uae30\ub300\ub97c \ubaa8\uc558\ub2e4.\u300a\uc0ac\ub791\uc758 \uac70\ub9ac\u300b\ub294 \uc815\uc7ac\uc740\uc774 1984\ub144\uc5d0 \uba3c\uc800 \ucde8\uc785\ud558\uc5ec \ubd88\ub800\ub294\ub370 2\uc808 \uae4c\uc9c0 \ubc16\uc5d0 \uc5c6\uc5c8\ub294\ub370 \ubb38\ud76c\uc625\uc774 2\uc9d1 \ud0c0\uc774\ud2c0\uace1\uc73c\ub85c \ub9ac\uba54\uc774\ud06c \ud558\uba74\uc11c 2\uc808\uc774 \ub05d\ub098\uace0 1\uc808\uc744 \ud55c\ubc88 \ub354 \ubc18\ubcf5\ud558\uba74\uc11c 3\uc808\ub85c \ud558\uc600\ub2e4. \ub610\ud55c \uc77c\ubcf8 \uc624\uc0ac\uce74\uc5d0\uc11c \uad50\ud3ec \uc704\ubb38 \uacf5\uc5f0\uc744 \uc2dc\uc791\uc73c\ub85c \uc77c\ubcf8\uc5d0\ub3c4 \uc9c4\ucd9c\ud558\uc600\ub2e4. \ubb38\ud76c\uc625\uc740 1990\ub144\uc5d0\u300a\uac15\ub0a8 \uba4b\uc7c1\uc774\u300b(\uc815\uc740\uc774 \uc791\uc0ac/ \ub0a8\uad6d\uc778 \uc791\uace1)\uac00 \ud788\ud2b8\ud558\uc5ec \uc778\uae30\uac00\ub3c4\ub97c \ub2ec\ub9ac\uace0 \uc788\uc744 \ub54c \ub2f9\uc2dc KBS \uac00\uc694\ub300\uc0c1\uc744 \uc218\uc0c1\ud55c \uc120\ubc30\uac00\uc218 \ud604\ucca0\uacfc \ud63c\uc131\ub4c0\uc5e3\uc73c\ub85c\u300a\uc798\ud588\uad70 \uc798\ud588\uc5b4 \uba54\ub4e4\ub9ac\u300b\ub97c \ubc1c\ud45c\ud558\uc600\ub294\ub370 \uae30\uc874\uc5d0 \ubb34\uba85\uac00\uc218\uac00 \ud63c\uc131 \ub4c0\uc5e3\ud55c \uc74c\ubc18\uc774 \uc544\ub2cc \uc778\uae30\uac00\uc218\ub85c\uc11c \ud63c\uc131 \ub4c0\uc5e3\ud55c \uc74c\ubc18\uc73c\ub85c\uc11c \ucc98\uc74c\uc774\uc5c8\ub2e4. \uadf8\ub9ac\uace0\u300a\ud56d\uad6c \uba54\ub4e4\ub9ac\u300b,\u300a\ud574\uae08\uac00\uc694 \uba54\ub4e4\ub9ac\u300b \ub4f1\uc744 \ubc1c\ud45c\ud558\uba74\uc11c \uae30\uc874\uc758 \uba54\ub4e4\ub9ac \uc74c\ubc18\uc744 \ubc1c\ud45c\ud558\uace0 \uc815\ud1b5 \ud2b8\ub85c\ud2b8\ub97c \uace0\uc218\ud558\ub358 \uc774\ubbf8\uc790, \uae40\uc5f0\uc790, \uc8fc\ud604\ubbf8\uc758 \ub4a4\ub97c \uc774\uc5b4 \ub300\ud55c\ubbfc\uad6d\uc5d0\uc11c \uba87 \uc548\ub418\ub294 \uc815\ud1b5 \ud2b8\ub85c\ud2b8 \uac00\uc218\uc774\uc790 \uba54\ub4e4\ub9ac \uac00\uc218\ub85c \uc790\ub9ac\ub97c \uad73\ud788\uac8c \ub418\uc5c8\ub2e4. 1991\ub144\uc5d0 \ubc1c\ud45c\ud55c \uc5d4\uce74\ud48d\uc758 \uc815\ud1b5 \ud2b8\ub85c\ud2b8\u300a\uc131\uc740 \uae40\uc774\uc694\u300b(\uc870\ub3d9\uc0b0 \uc791\uc0ac/ \uc6d0\ud76c\uba85 \uc791\uace1)\ub294 \ubb38\ud76c\uc625\ud558\uba74 \ub5a0\uc624\ub974\ub294 \ub178\ub798 \uc911 \ud558\ub098\ub85c \uc190 \uaf3d\ud788\uba70 \ub2f9\uc2dc \uc81c14\ub300 \uad6d\ud68c\uc758\uc6d0 \uc120\uac70 \ub54c \uae40\uc528 \ud6c4\ubcf4\uc790\ub4e4\uc5d0\uac8c \uc804\ud3ed\uc801\uc778 \uc9c0\uc9c0\ub97c \ubc1b\uc73c\uba74\uc11c \uc0c1\uc704\uad8c\uc5d0 \uc7ac\ub3c4\uc785\uc5d0 \uc131\uacf5\ud588\ub2e4. \uadf8\ub9ac\uace0 \uc120\uac70\ucca0\uc744 \ub9de\uc544 \uac01 \uc720\uc138\uc7a5\uc5d0\uc11c \uac00\uc0ac\ubc14\uafd4 \ubd80\ub974\uae30 \ub178\ub798\ub85c \uac01\uad11\uc744 \ubc1b\uc544 \ub4dc\ub514\uc5b4 \ud2b8\ub85c\ud2b8\uacc4 \uc815\uc0c1\uc758 \uc790\ub9ac\uc5d0 \uc548\ucc29\ud588\ub2e4.", "answer": "\uc2e4\uc6a9\uc74c\uc545\ud559\uacfc", "sentence": "1988\ub144 \ubb38\ud76c\uc625\uc740 \uc5b4\ub824\uc6b4 \uac00\uc815\ud615\ud3b8 \uc18d\uc5d0\uc11c\ub3c4 \uc5f4\uc2ec\ud788 \uacf5\ubd80\ud558\uc5ec \uc11c\uc6b8\uc608\uc220\ub300\ud559 \uc2e4\uc6a9\uc74c\uc545\ud559\uacfc \uc5d0", "paragraph_sentence": " 1988\ub144 \ubb38\ud76c\uc625\uc740 \uc5b4\ub824\uc6b4 \uac00\uc815\ud615\ud3b8 \uc18d\uc5d0\uc11c\ub3c4 \uc5f4\uc2ec\ud788 \uacf5\ubd80\ud558\uc5ec \uc11c\uc6b8\uc608\uc220\ub300\ud559 \uc2e4\uc6a9\uc74c\uc545\ud559\uacfc \uc5d0 \uc785\ud559\ud558\uc600\uace0 \ub300\ud559\uc0dd \uac00\uc218\uac00 \ub418\uc5c8\uc73c\uba70 \uadf8 \ud574 \uc2ac\ub85c\uc6b0 \ud15c\ud3ec\uc758 \uc815\ud1b5 \ud2b8\ub85c\ud2b8\u300a\uac00\ub294 \ub2d8 \uac00\ub294 \uc815\u300b(\uc815\uc740\uc774 \uc791\uc0ac/ \ub0a8\uad6d\uc778 \uc791\uace1)\uc744 \ubc1c\ud45c\ud558\uc600\ub2e4. \ub370\ubdd4\uc640 \ub3d9\uc2dc\uc5d0 \ud070 \uc131\uacfc\ub97c \uac70\ub450\uc5b4 \ub300\uc911\ub4e4\uc758 \uc0ac\ub791\uc744 \ubc1b\uc558\ub358 \ubb38\ud76c\uc625\uc740 1989\ub144\uc5d0 \uc815\ud1b5 \ud2b8\ub86f \ub9ac\ub4ec\uc744 \uacbd\ucf8c\ud558\uac8c \ud604\ub300\ud654\ud55c \ud3ed\uc2a4 \ud2b8\ub85c\ud2b8\u300a\uc0ac\ub791\uc758 \uac70\ub9ac\u300b(\uc815\uc740\uc774 \uc791\uc0ac/ \ub0a8\uad6d\uc778 \uc791\uace1/ \uae40\uc6a9\ub144 \ud3b8\uace1)\ub85c \ubc29\uc1a1 \uc74c\uc545\ub2e4\ubc29 \ub4f1\uc9c0\uc758 \uc778\uae30\ucc28\ud2b8\uc5d0\uc11c \uc0c1\uc704\uad8c\uc5d0 \uc62c\ub77c \uac00\uc218\ub85c\uc11c\uc758 \uc804\uc131\uae30\ub97c \ub9de\uc774\ud558\uba74\uc11c \ud2b8\ub85c\ud2b8 \uac00\uc218\uc758 \ub9c9\ub0b4\ub85c\uc11c \uae30\ub300\ub97c \ubaa8\uc558\ub2e4.\u300a\uc0ac\ub791\uc758 \uac70\ub9ac\u300b\ub294 \uc815\uc7ac\uc740\uc774 1984\ub144\uc5d0 \uba3c\uc800 \ucde8\uc785\ud558\uc5ec \ubd88\ub800\ub294\ub370 2\uc808 \uae4c\uc9c0 \ubc16\uc5d0 \uc5c6\uc5c8\ub294\ub370 \ubb38\ud76c\uc625\uc774 2\uc9d1 \ud0c0\uc774\ud2c0\uace1\uc73c\ub85c \ub9ac\uba54\uc774\ud06c \ud558\uba74\uc11c 2\uc808\uc774 \ub05d\ub098\uace0 1\uc808\uc744 \ud55c\ubc88 \ub354 \ubc18\ubcf5\ud558\uba74\uc11c 3\uc808\ub85c \ud558\uc600\ub2e4. \ub610\ud55c \uc77c\ubcf8 \uc624\uc0ac\uce74\uc5d0\uc11c \uad50\ud3ec \uc704\ubb38 \uacf5\uc5f0\uc744 \uc2dc\uc791\uc73c\ub85c \uc77c\ubcf8\uc5d0\ub3c4 \uc9c4\ucd9c\ud558\uc600\ub2e4. \ubb38\ud76c\uc625\uc740 1990\ub144\uc5d0\u300a\uac15\ub0a8 \uba4b\uc7c1\uc774\u300b(\uc815\uc740\uc774 \uc791\uc0ac/ \ub0a8\uad6d\uc778 \uc791\uace1)\uac00 \ud788\ud2b8\ud558\uc5ec \uc778\uae30\uac00\ub3c4\ub97c \ub2ec\ub9ac\uace0 \uc788\uc744 \ub54c \ub2f9\uc2dc KBS \uac00\uc694\ub300\uc0c1\uc744 \uc218\uc0c1\ud55c \uc120\ubc30\uac00\uc218 \ud604\ucca0\uacfc \ud63c\uc131\ub4c0\uc5e3\uc73c\ub85c\u300a\uc798\ud588\uad70 \uc798\ud588\uc5b4 \uba54\ub4e4\ub9ac\u300b\ub97c \ubc1c\ud45c\ud558\uc600\ub294\ub370 \uae30\uc874\uc5d0 \ubb34\uba85\uac00\uc218\uac00 \ud63c\uc131 \ub4c0\uc5e3\ud55c \uc74c\ubc18\uc774 \uc544\ub2cc \uc778\uae30\uac00\uc218\ub85c\uc11c \ud63c\uc131 \ub4c0\uc5e3\ud55c \uc74c\ubc18\uc73c\ub85c\uc11c \ucc98\uc74c\uc774\uc5c8\ub2e4. \uadf8\ub9ac\uace0\u300a\ud56d\uad6c \uba54\ub4e4\ub9ac\u300b,\u300a\ud574\uae08\uac00\uc694 \uba54\ub4e4\ub9ac\u300b \ub4f1\uc744 \ubc1c\ud45c\ud558\uba74\uc11c \uae30\uc874\uc758 \uba54\ub4e4\ub9ac \uc74c\ubc18\uc744 \ubc1c\ud45c\ud558\uace0 \uc815\ud1b5 \ud2b8\ub85c\ud2b8\ub97c \uace0\uc218\ud558\ub358 \uc774\ubbf8\uc790, \uae40\uc5f0\uc790, \uc8fc\ud604\ubbf8\uc758 \ub4a4\ub97c \uc774\uc5b4 \ub300\ud55c\ubbfc\uad6d\uc5d0\uc11c \uba87 \uc548\ub418\ub294 \uc815\ud1b5 \ud2b8\ub85c\ud2b8 \uac00\uc218\uc774\uc790 \uba54\ub4e4\ub9ac \uac00\uc218\ub85c \uc790\ub9ac\ub97c \uad73\ud788\uac8c \ub418\uc5c8\ub2e4. 1991\ub144\uc5d0 \ubc1c\ud45c\ud55c \uc5d4\uce74\ud48d\uc758 \uc815\ud1b5 \ud2b8\ub85c\ud2b8\u300a\uc131\uc740 \uae40\uc774\uc694\u300b(\uc870\ub3d9\uc0b0 \uc791\uc0ac/ \uc6d0\ud76c\uba85 \uc791\uace1)\ub294 \ubb38\ud76c\uc625\ud558\uba74 \ub5a0\uc624\ub974\ub294 \ub178\ub798 \uc911 \ud558\ub098\ub85c \uc190 \uaf3d\ud788\uba70 \ub2f9\uc2dc \uc81c14\ub300 \uad6d\ud68c\uc758\uc6d0 \uc120\uac70 \ub54c \uae40\uc528 \ud6c4\ubcf4\uc790\ub4e4\uc5d0\uac8c \uc804\ud3ed\uc801\uc778 \uc9c0\uc9c0\ub97c \ubc1b\uc73c\uba74\uc11c \uc0c1\uc704\uad8c\uc5d0 \uc7ac\ub3c4\uc785\uc5d0 \uc131\uacf5\ud588\ub2e4. \uadf8\ub9ac\uace0 \uc120\uac70\ucca0\uc744 \ub9de\uc544 \uac01 \uc720\uc138\uc7a5\uc5d0\uc11c \uac00\uc0ac\ubc14\uafd4 \ubd80\ub974\uae30 \ub178\ub798\ub85c \uac01\uad11\uc744 \ubc1b\uc544 \ub4dc\ub514\uc5b4 \ud2b8\ub85c\ud2b8\uacc4 \uc815\uc0c1\uc758 \uc790\ub9ac\uc5d0 \uc548\ucc29\ud588\ub2e4.", "paragraph_answer": "1988\ub144 \ubb38\ud76c\uc625\uc740 \uc5b4\ub824\uc6b4 \uac00\uc815\ud615\ud3b8 \uc18d\uc5d0\uc11c\ub3c4 \uc5f4\uc2ec\ud788 \uacf5\ubd80\ud558\uc5ec \uc11c\uc6b8\uc608\uc220\ub300\ud559 \uc2e4\uc6a9\uc74c\uc545\ud559\uacfc \uc5d0 \uc785\ud559\ud558\uc600\uace0 \ub300\ud559\uc0dd \uac00\uc218\uac00 \ub418\uc5c8\uc73c\uba70 \uadf8 \ud574 \uc2ac\ub85c\uc6b0 \ud15c\ud3ec\uc758 \uc815\ud1b5 \ud2b8\ub85c\ud2b8\u300a\uac00\ub294 \ub2d8 \uac00\ub294 \uc815\u300b(\uc815\uc740\uc774 \uc791\uc0ac/ \ub0a8\uad6d\uc778 \uc791\uace1)\uc744 \ubc1c\ud45c\ud558\uc600\ub2e4. \ub370\ubdd4\uc640 \ub3d9\uc2dc\uc5d0 \ud070 \uc131\uacfc\ub97c \uac70\ub450\uc5b4 \ub300\uc911\ub4e4\uc758 \uc0ac\ub791\uc744 \ubc1b\uc558\ub358 \ubb38\ud76c\uc625\uc740 1989\ub144\uc5d0 \uc815\ud1b5 \ud2b8\ub86f \ub9ac\ub4ec\uc744 \uacbd\ucf8c\ud558\uac8c \ud604\ub300\ud654\ud55c \ud3ed\uc2a4 \ud2b8\ub85c\ud2b8\u300a\uc0ac\ub791\uc758 \uac70\ub9ac\u300b(\uc815\uc740\uc774 \uc791\uc0ac/ \ub0a8\uad6d\uc778 \uc791\uace1/ \uae40\uc6a9\ub144 \ud3b8\uace1)\ub85c \ubc29\uc1a1 \uc74c\uc545\ub2e4\ubc29 \ub4f1\uc9c0\uc758 \uc778\uae30\ucc28\ud2b8\uc5d0\uc11c \uc0c1\uc704\uad8c\uc5d0 \uc62c\ub77c \uac00\uc218\ub85c\uc11c\uc758 \uc804\uc131\uae30\ub97c \ub9de\uc774\ud558\uba74\uc11c \ud2b8\ub85c\ud2b8 \uac00\uc218\uc758 \ub9c9\ub0b4\ub85c\uc11c \uae30\ub300\ub97c \ubaa8\uc558\ub2e4.\u300a\uc0ac\ub791\uc758 \uac70\ub9ac\u300b\ub294 \uc815\uc7ac\uc740\uc774 1984\ub144\uc5d0 \uba3c\uc800 \ucde8\uc785\ud558\uc5ec \ubd88\ub800\ub294\ub370 2\uc808 \uae4c\uc9c0 \ubc16\uc5d0 \uc5c6\uc5c8\ub294\ub370 \ubb38\ud76c\uc625\uc774 2\uc9d1 \ud0c0\uc774\ud2c0\uace1\uc73c\ub85c \ub9ac\uba54\uc774\ud06c \ud558\uba74\uc11c 2\uc808\uc774 \ub05d\ub098\uace0 1\uc808\uc744 \ud55c\ubc88 \ub354 \ubc18\ubcf5\ud558\uba74\uc11c 3\uc808\ub85c \ud558\uc600\ub2e4. \ub610\ud55c \uc77c\ubcf8 \uc624\uc0ac\uce74\uc5d0\uc11c \uad50\ud3ec \uc704\ubb38 \uacf5\uc5f0\uc744 \uc2dc\uc791\uc73c\ub85c \uc77c\ubcf8\uc5d0\ub3c4 \uc9c4\ucd9c\ud558\uc600\ub2e4. \ubb38\ud76c\uc625\uc740 1990\ub144\uc5d0\u300a\uac15\ub0a8 \uba4b\uc7c1\uc774\u300b(\uc815\uc740\uc774 \uc791\uc0ac/ \ub0a8\uad6d\uc778 \uc791\uace1)\uac00 \ud788\ud2b8\ud558\uc5ec \uc778\uae30\uac00\ub3c4\ub97c \ub2ec\ub9ac\uace0 \uc788\uc744 \ub54c \ub2f9\uc2dc KBS \uac00\uc694\ub300\uc0c1\uc744 \uc218\uc0c1\ud55c \uc120\ubc30\uac00\uc218 \ud604\ucca0\uacfc \ud63c\uc131\ub4c0\uc5e3\uc73c\ub85c\u300a\uc798\ud588\uad70 \uc798\ud588\uc5b4 \uba54\ub4e4\ub9ac\u300b\ub97c \ubc1c\ud45c\ud558\uc600\ub294\ub370 \uae30\uc874\uc5d0 \ubb34\uba85\uac00\uc218\uac00 \ud63c\uc131 \ub4c0\uc5e3\ud55c \uc74c\ubc18\uc774 \uc544\ub2cc \uc778\uae30\uac00\uc218\ub85c\uc11c \ud63c\uc131 \ub4c0\uc5e3\ud55c \uc74c\ubc18\uc73c\ub85c\uc11c \ucc98\uc74c\uc774\uc5c8\ub2e4. \uadf8\ub9ac\uace0\u300a\ud56d\uad6c \uba54\ub4e4\ub9ac\u300b,\u300a\ud574\uae08\uac00\uc694 \uba54\ub4e4\ub9ac\u300b \ub4f1\uc744 \ubc1c\ud45c\ud558\uba74\uc11c \uae30\uc874\uc758 \uba54\ub4e4\ub9ac \uc74c\ubc18\uc744 \ubc1c\ud45c\ud558\uace0 \uc815\ud1b5 \ud2b8\ub85c\ud2b8\ub97c \uace0\uc218\ud558\ub358 \uc774\ubbf8\uc790, \uae40\uc5f0\uc790, \uc8fc\ud604\ubbf8\uc758 \ub4a4\ub97c \uc774\uc5b4 \ub300\ud55c\ubbfc\uad6d\uc5d0\uc11c \uba87 \uc548\ub418\ub294 \uc815\ud1b5 \ud2b8\ub85c\ud2b8 \uac00\uc218\uc774\uc790 \uba54\ub4e4\ub9ac \uac00\uc218\ub85c \uc790\ub9ac\ub97c \uad73\ud788\uac8c \ub418\uc5c8\ub2e4. 1991\ub144\uc5d0 \ubc1c\ud45c\ud55c \uc5d4\uce74\ud48d\uc758 \uc815\ud1b5 \ud2b8\ub85c\ud2b8\u300a\uc131\uc740 \uae40\uc774\uc694\u300b(\uc870\ub3d9\uc0b0 \uc791\uc0ac/ \uc6d0\ud76c\uba85 \uc791\uace1)\ub294 \ubb38\ud76c\uc625\ud558\uba74 \ub5a0\uc624\ub974\ub294 \ub178\ub798 \uc911 \ud558\ub098\ub85c \uc190 \uaf3d\ud788\uba70 \ub2f9\uc2dc \uc81c14\ub300 \uad6d\ud68c\uc758\uc6d0 \uc120\uac70 \ub54c \uae40\uc528 \ud6c4\ubcf4\uc790\ub4e4\uc5d0\uac8c \uc804\ud3ed\uc801\uc778 \uc9c0\uc9c0\ub97c \ubc1b\uc73c\uba74\uc11c \uc0c1\uc704\uad8c\uc5d0 \uc7ac\ub3c4\uc785\uc5d0 \uc131\uacf5\ud588\ub2e4. \uadf8\ub9ac\uace0 \uc120\uac70\ucca0\uc744 \ub9de\uc544 \uac01 \uc720\uc138\uc7a5\uc5d0\uc11c \uac00\uc0ac\ubc14\uafd4 \ubd80\ub974\uae30 \ub178\ub798\ub85c \uac01\uad11\uc744 \ubc1b\uc544 \ub4dc\ub514\uc5b4 \ud2b8\ub85c\ud2b8\uacc4 \uc815\uc0c1\uc758 \uc790\ub9ac\uc5d0 \uc548\ucc29\ud588\ub2e4.", "sentence_answer": "1988\ub144 \ubb38\ud76c\uc625\uc740 \uc5b4\ub824\uc6b4 \uac00\uc815\ud615\ud3b8 \uc18d\uc5d0\uc11c\ub3c4 \uc5f4\uc2ec\ud788 \uacf5\ubd80\ud558\uc5ec \uc11c\uc6b8\uc608\uc220\ub300\ud559 \uc2e4\uc6a9\uc74c\uc545\ud559\uacfc \uc5d0", "paragraph_id": "6091146-1-0", "paragraph_question": "question: \ubb38\ud76c\uc625\uc774 \ub2e4\ub154\ub358 \uc11c\uc6b8\uc608\uc220\ub300\ud559\uc758 \ud559\uacfc\ub294?, context: 1988\ub144 \ubb38\ud76c\uc625\uc740 \uc5b4\ub824\uc6b4 \uac00\uc815\ud615\ud3b8 \uc18d\uc5d0\uc11c\ub3c4 \uc5f4\uc2ec\ud788 \uacf5\ubd80\ud558\uc5ec \uc11c\uc6b8\uc608\uc220\ub300\ud559 \uc2e4\uc6a9\uc74c\uc545\ud559\uacfc\uc5d0 \uc785\ud559\ud558\uc600\uace0 \ub300\ud559\uc0dd \uac00\uc218\uac00 \ub418\uc5c8\uc73c\uba70 \uadf8 \ud574 \uc2ac\ub85c\uc6b0 \ud15c\ud3ec\uc758 \uc815\ud1b5 \ud2b8\ub85c\ud2b8\u300a\uac00\ub294 \ub2d8 \uac00\ub294 \uc815\u300b(\uc815\uc740\uc774 \uc791\uc0ac/ \ub0a8\uad6d\uc778 \uc791\uace1)\uc744 \ubc1c\ud45c\ud558\uc600\ub2e4. \ub370\ubdd4\uc640 \ub3d9\uc2dc\uc5d0 \ud070 \uc131\uacfc\ub97c \uac70\ub450\uc5b4 \ub300\uc911\ub4e4\uc758 \uc0ac\ub791\uc744 \ubc1b\uc558\ub358 \ubb38\ud76c\uc625\uc740 1989\ub144\uc5d0 \uc815\ud1b5 \ud2b8\ub86f \ub9ac\ub4ec\uc744 \uacbd\ucf8c\ud558\uac8c \ud604\ub300\ud654\ud55c \ud3ed\uc2a4 \ud2b8\ub85c\ud2b8\u300a\uc0ac\ub791\uc758 \uac70\ub9ac\u300b(\uc815\uc740\uc774 \uc791\uc0ac/ \ub0a8\uad6d\uc778 \uc791\uace1/ \uae40\uc6a9\ub144 \ud3b8\uace1)\ub85c \ubc29\uc1a1 \uc74c\uc545\ub2e4\ubc29 \ub4f1\uc9c0\uc758 \uc778\uae30\ucc28\ud2b8\uc5d0\uc11c \uc0c1\uc704\uad8c\uc5d0 \uc62c\ub77c \uac00\uc218\ub85c\uc11c\uc758 \uc804\uc131\uae30\ub97c \ub9de\uc774\ud558\uba74\uc11c \ud2b8\ub85c\ud2b8 \uac00\uc218\uc758 \ub9c9\ub0b4\ub85c\uc11c \uae30\ub300\ub97c \ubaa8\uc558\ub2e4.\u300a\uc0ac\ub791\uc758 \uac70\ub9ac\u300b\ub294 \uc815\uc7ac\uc740\uc774 1984\ub144\uc5d0 \uba3c\uc800 \ucde8\uc785\ud558\uc5ec \ubd88\ub800\ub294\ub370 2\uc808 \uae4c\uc9c0 \ubc16\uc5d0 \uc5c6\uc5c8\ub294\ub370 \ubb38\ud76c\uc625\uc774 2\uc9d1 \ud0c0\uc774\ud2c0\uace1\uc73c\ub85c \ub9ac\uba54\uc774\ud06c \ud558\uba74\uc11c 2\uc808\uc774 \ub05d\ub098\uace0 1\uc808\uc744 \ud55c\ubc88 \ub354 \ubc18\ubcf5\ud558\uba74\uc11c 3\uc808\ub85c \ud558\uc600\ub2e4. \ub610\ud55c \uc77c\ubcf8 \uc624\uc0ac\uce74\uc5d0\uc11c \uad50\ud3ec \uc704\ubb38 \uacf5\uc5f0\uc744 \uc2dc\uc791\uc73c\ub85c \uc77c\ubcf8\uc5d0\ub3c4 \uc9c4\ucd9c\ud558\uc600\ub2e4. \ubb38\ud76c\uc625\uc740 1990\ub144\uc5d0\u300a\uac15\ub0a8 \uba4b\uc7c1\uc774\u300b(\uc815\uc740\uc774 \uc791\uc0ac/ \ub0a8\uad6d\uc778 \uc791\uace1)\uac00 \ud788\ud2b8\ud558\uc5ec \uc778\uae30\uac00\ub3c4\ub97c \ub2ec\ub9ac\uace0 \uc788\uc744 \ub54c \ub2f9\uc2dc KBS \uac00\uc694\ub300\uc0c1\uc744 \uc218\uc0c1\ud55c \uc120\ubc30\uac00\uc218 \ud604\ucca0\uacfc \ud63c\uc131\ub4c0\uc5e3\uc73c\ub85c\u300a\uc798\ud588\uad70 \uc798\ud588\uc5b4 \uba54\ub4e4\ub9ac\u300b\ub97c \ubc1c\ud45c\ud558\uc600\ub294\ub370 \uae30\uc874\uc5d0 \ubb34\uba85\uac00\uc218\uac00 \ud63c\uc131 \ub4c0\uc5e3\ud55c \uc74c\ubc18\uc774 \uc544\ub2cc \uc778\uae30\uac00\uc218\ub85c\uc11c \ud63c\uc131 \ub4c0\uc5e3\ud55c \uc74c\ubc18\uc73c\ub85c\uc11c \ucc98\uc74c\uc774\uc5c8\ub2e4. \uadf8\ub9ac\uace0\u300a\ud56d\uad6c \uba54\ub4e4\ub9ac\u300b,\u300a\ud574\uae08\uac00\uc694 \uba54\ub4e4\ub9ac\u300b \ub4f1\uc744 \ubc1c\ud45c\ud558\uba74\uc11c \uae30\uc874\uc758 \uba54\ub4e4\ub9ac \uc74c\ubc18\uc744 \ubc1c\ud45c\ud558\uace0 \uc815\ud1b5 \ud2b8\ub85c\ud2b8\ub97c \uace0\uc218\ud558\ub358 \uc774\ubbf8\uc790, \uae40\uc5f0\uc790, \uc8fc\ud604\ubbf8\uc758 \ub4a4\ub97c \uc774\uc5b4 \ub300\ud55c\ubbfc\uad6d\uc5d0\uc11c \uba87 \uc548\ub418\ub294 \uc815\ud1b5 \ud2b8\ub85c\ud2b8 \uac00\uc218\uc774\uc790 \uba54\ub4e4\ub9ac \uac00\uc218\ub85c \uc790\ub9ac\ub97c \uad73\ud788\uac8c \ub418\uc5c8\ub2e4. 1991\ub144\uc5d0 \ubc1c\ud45c\ud55c \uc5d4\uce74\ud48d\uc758 \uc815\ud1b5 \ud2b8\ub85c\ud2b8\u300a\uc131\uc740 \uae40\uc774\uc694\u300b(\uc870\ub3d9\uc0b0 \uc791\uc0ac/ \uc6d0\ud76c\uba85 \uc791\uace1)\ub294 \ubb38\ud76c\uc625\ud558\uba74 \ub5a0\uc624\ub974\ub294 \ub178\ub798 \uc911 \ud558\ub098\ub85c \uc190 \uaf3d\ud788\uba70 \ub2f9\uc2dc \uc81c14\ub300 \uad6d\ud68c\uc758\uc6d0 \uc120\uac70 \ub54c \uae40\uc528 \ud6c4\ubcf4\uc790\ub4e4\uc5d0\uac8c \uc804\ud3ed\uc801\uc778 \uc9c0\uc9c0\ub97c \ubc1b\uc73c\uba74\uc11c \uc0c1\uc704\uad8c\uc5d0 \uc7ac\ub3c4\uc785\uc5d0 \uc131\uacf5\ud588\ub2e4. \uadf8\ub9ac\uace0 \uc120\uac70\ucca0\uc744 \ub9de\uc544 \uac01 \uc720\uc138\uc7a5\uc5d0\uc11c \uac00\uc0ac\ubc14\uafd4 \ubd80\ub974\uae30 \ub178\ub798\ub85c \uac01\uad11\uc744 \ubc1b\uc544 \ub4dc\ub514\uc5b4 \ud2b8\ub85c\ud2b8\uacc4 \uc815\uc0c1\uc758 \uc790\ub9ac\uc5d0 \uc548\ucc29\ud588\ub2e4."} {"question": "\uc911\uad6d\uc744 \uad6d\ube48\ubc29\ubb38\ud558\uba74\uc11c \ud640\ub300 \ub17c\ub780\uae4c\uc9c0 \uc77c\uc5c8\ub358 \ub300\ud1b5\ub839 \uc601\uc811 \uc778\uc0ac\uc758 \uc9c1\uae09\uc740?", "paragraph": "\ud558\uc9c0\ub9cc \uc911\uad6d\uc744 \uad6d\ube48\ubc29\ubb38\ud55c \uac83\uce58\uace0\ub294 \uaca9\uc774 \ubd80\uc871\ud558\ub2e4\ub294 \ube44\ud310\ub3c4 \ub098\uc654\ub2e4. \ubc29\uc911 \uc804\uc5d0 \uc911\uad6d\uc911\uc559\ud154\ub808\ube44\uc804\uc774 \ubb38\uc7ac\uc778\uacfc\uc758 \uc0ac\uc804 \uc778\ud130\ubdf0\uc5d0\uc11c 3\ubd88 \uc815\ucc45\uc5d0 \ub300\ud55c \uc555\ubc15\uc131 \uc9c8\ubb38\uc744 \uc3df\uc544\ub0c8\uc73c\uba70, \ubc29\ubb38\ud588\uc744 \ub54c\ub294 \ucc28\uad00\ubcf4\uae09 \uc778\uc0ac\uac00 \ub300\ud1b5\ub839\uc744 \uc601\uc811\ud558\uba74\uc11c \ud640\ub300 \ub17c\ub780\uae4c\uc9c0 \uc77c\uc5c8\ub2e4. \uacbd\uc81c \uc0ac\uc808\ub2e8\ub3c4 \uc7ac\ubc8c \ucd1d\uc218\ub4e4\uc774 \ud3ec\ud568\ub41c \uc5ed\ub300 \ucd5c\ub300 \uaddc\ubaa8\ub85c \uad6c\uc131\ub418\uc5c8\uc9c0\ub9cc \uc911\uad6d \uce21\uc5d0\uc11c\ub294 \uae30\uc5c5\uc758 2, 3\uc778\uc790\ub4e4\uc774 \ucc38\uc11d\ud574 \uae09\uc774 \ub9de\uc9c0 \uc54a\uc544 \uc2e4\ubb34\uc801\uc778 \ud68c\ub2f4\uc774 \uc9c4\ud589\ub418\uc9c0 \ubabb\ud588\ub2e4\ub294 \uc598\uae30\ub3c4 \ub098\uc654\ub2e4. \uc2ec\uc9c0\uc5b4 '\ud55c\uc911 \uacbd\uc81c\u00b7\ubb34\uc5ed \ud30c\ud2b8\ub108\uc2ed' \ud589\uc0ac\uc5d0\uc11c\ub294 \ubb38\uc7ac\uc778\uc744 \ucde8\uc7ac\ud558\ub294 \uccad\uc640\ub300 \ucd9c\uc785 \uc0ac\uc9c4\uae30\uc0ac 2\uba85\uc744 \uc911\uad6d \uce21 \uacbd\ud638\uc6d0\ub4e4\uc774 \uacfc\uc789 \ub300\uc751\ud558\uba74\uc11c \ud3ed\ud589\ud558\ub294 \uc0ac\uac74\uc774 \uc77c\uc5b4\ub098\uba74\uc11c \uc678\uad50\uc801 \uacb0\ub840\ub77c\ub294 \ube44\ud310\uc774 \ub098\uc654\ub2e4. \uc0ac\ub4dc \uc5b8\uae09\ub3c4 \uc218\uc704\uac00 \ub0ae\uc544\uc9c0\uae34 \ud588\uc9c0\ub9cc \uc5ec\uc804\ud788 \uc785\uc7a5\ucc28\ub97c \uc881\ud788\uc9c0 \ubabb\ud574 \uacf5\ub3d9\uc131\uba85\uc774\ub098 \uacf5\ub3d9 \uc5b8\ub860 \ubc1c\ud45c\ubb38\ub3c4 \ucc44\ud0dd\ud558\uc9c0 \ubabb\ud558\uc5ec \uad00\uacc4 \ubcf5\uc6d0\uc774 \uc27d\uc9c0 \uc54a\uc74c\uc744 \ubcf4\uc774\uae30\ub3c4 \ud588\ub2e4. \uc774\ub294 \uc911\uad6d\uc774 \uc544\uc9c1 \uc0ac\ub4dc\uc5d0 \ub300\ud55c \uc559\uae08\uc774 \ub0a8\uc544\uc788\ub294 \uc0c1\ud669\uc5d0\uc11c \ud55c\uad6d\uc774 \uc5f0\ub0b4 \ubc29\uc911\uc744 \uc11c\ub450\ub974\uba74\uc11c \uc77c\uc5b4\ub0ac\ub2e4\ub294 \ubc18\uc751\uc774\ub2e4.", "answer": "\ucc28\uad00\ubcf4\uae09", "sentence": "\ubc29\uc911 \uc804\uc5d0 \uc911\uad6d\uc911\uc559\ud154\ub808\ube44\uc804\uc774 \ubb38\uc7ac\uc778\uacfc\uc758 \uc0ac\uc804 \uc778\ud130\ubdf0\uc5d0\uc11c 3\ubd88 \uc815\ucc45\uc5d0 \ub300\ud55c \uc555\ubc15\uc131 \uc9c8\ubb38\uc744 \uc3df\uc544\ub0c8\uc73c\uba70, \ubc29\ubb38\ud588\uc744 \ub54c\ub294 \ucc28\uad00\ubcf4\uae09 \uc778\uc0ac\uac00 \ub300\ud1b5\ub839\uc744 \uc601\uc811\ud558\uba74\uc11c \ud640\ub300 \ub17c\ub780\uae4c\uc9c0 \uc77c\uc5c8\ub2e4.", "paragraph_sentence": "\ud558\uc9c0\ub9cc \uc911\uad6d\uc744 \uad6d\ube48\ubc29\ubb38\ud55c \uac83\uce58\uace0\ub294 \uaca9\uc774 \ubd80\uc871\ud558\ub2e4\ub294 \ube44\ud310\ub3c4 \ub098\uc654\ub2e4. \ubc29\uc911 \uc804\uc5d0 \uc911\uad6d\uc911\uc559\ud154\ub808\ube44\uc804\uc774 \ubb38\uc7ac\uc778\uacfc\uc758 \uc0ac\uc804 \uc778\ud130\ubdf0\uc5d0\uc11c 3\ubd88 \uc815\ucc45\uc5d0 \ub300\ud55c \uc555\ubc15\uc131 \uc9c8\ubb38\uc744 \uc3df\uc544\ub0c8\uc73c\uba70, \ubc29\ubb38\ud588\uc744 \ub54c\ub294 \ucc28\uad00\ubcf4\uae09 \uc778\uc0ac\uac00 \ub300\ud1b5\ub839\uc744 \uc601\uc811\ud558\uba74\uc11c \ud640\ub300 \ub17c\ub780\uae4c\uc9c0 \uc77c\uc5c8\ub2e4. \uacbd\uc81c \uc0ac\uc808\ub2e8\ub3c4 \uc7ac\ubc8c \ucd1d\uc218\ub4e4\uc774 \ud3ec\ud568\ub41c \uc5ed\ub300 \ucd5c\ub300 \uaddc\ubaa8\ub85c \uad6c\uc131\ub418\uc5c8\uc9c0\ub9cc \uc911\uad6d \uce21\uc5d0\uc11c\ub294 \uae30\uc5c5\uc758 2, 3\uc778\uc790\ub4e4\uc774 \ucc38\uc11d\ud574 \uae09\uc774 \ub9de\uc9c0 \uc54a\uc544 \uc2e4\ubb34\uc801\uc778 \ud68c\ub2f4\uc774 \uc9c4\ud589\ub418\uc9c0 \ubabb\ud588\ub2e4\ub294 \uc598\uae30\ub3c4 \ub098\uc654\ub2e4. \uc2ec\uc9c0\uc5b4 '\ud55c\uc911 \uacbd\uc81c\u00b7\ubb34\uc5ed \ud30c\ud2b8\ub108\uc2ed' \ud589\uc0ac\uc5d0\uc11c\ub294 \ubb38\uc7ac\uc778\uc744 \ucde8\uc7ac\ud558\ub294 \uccad\uc640\ub300 \ucd9c\uc785 \uc0ac\uc9c4\uae30\uc0ac 2\uba85\uc744 \uc911\uad6d \uce21 \uacbd\ud638\uc6d0\ub4e4\uc774 \uacfc\uc789 \ub300\uc751\ud558\uba74\uc11c \ud3ed\ud589\ud558\ub294 \uc0ac\uac74\uc774 \uc77c\uc5b4\ub098\uba74\uc11c \uc678\uad50\uc801 \uacb0\ub840\ub77c\ub294 \ube44\ud310\uc774 \ub098\uc654\ub2e4. \uc0ac\ub4dc \uc5b8\uae09\ub3c4 \uc218\uc704\uac00 \ub0ae\uc544\uc9c0\uae34 \ud588\uc9c0\ub9cc \uc5ec\uc804\ud788 \uc785\uc7a5\ucc28\ub97c \uc881\ud788\uc9c0 \ubabb\ud574 \uacf5\ub3d9\uc131\uba85\uc774\ub098 \uacf5\ub3d9 \uc5b8\ub860 \ubc1c\ud45c\ubb38\ub3c4 \ucc44\ud0dd\ud558\uc9c0 \ubabb\ud558\uc5ec \uad00\uacc4 \ubcf5\uc6d0\uc774 \uc27d\uc9c0 \uc54a\uc74c\uc744 \ubcf4\uc774\uae30\ub3c4 \ud588\ub2e4. \uc774\ub294 \uc911\uad6d\uc774 \uc544\uc9c1 \uc0ac\ub4dc\uc5d0 \ub300\ud55c \uc559\uae08\uc774 \ub0a8\uc544\uc788\ub294 \uc0c1\ud669\uc5d0\uc11c \ud55c\uad6d\uc774 \uc5f0\ub0b4 \ubc29\uc911\uc744 \uc11c\ub450\ub974\uba74\uc11c \uc77c\uc5b4\ub0ac\ub2e4\ub294 \ubc18\uc751\uc774\ub2e4.", "paragraph_answer": "\ud558\uc9c0\ub9cc \uc911\uad6d\uc744 \uad6d\ube48\ubc29\ubb38\ud55c \uac83\uce58\uace0\ub294 \uaca9\uc774 \ubd80\uc871\ud558\ub2e4\ub294 \ube44\ud310\ub3c4 \ub098\uc654\ub2e4. \ubc29\uc911 \uc804\uc5d0 \uc911\uad6d\uc911\uc559\ud154\ub808\ube44\uc804\uc774 \ubb38\uc7ac\uc778\uacfc\uc758 \uc0ac\uc804 \uc778\ud130\ubdf0\uc5d0\uc11c 3\ubd88 \uc815\ucc45\uc5d0 \ub300\ud55c \uc555\ubc15\uc131 \uc9c8\ubb38\uc744 \uc3df\uc544\ub0c8\uc73c\uba70, \ubc29\ubb38\ud588\uc744 \ub54c\ub294 \ucc28\uad00\ubcf4\uae09 \uc778\uc0ac\uac00 \ub300\ud1b5\ub839\uc744 \uc601\uc811\ud558\uba74\uc11c \ud640\ub300 \ub17c\ub780\uae4c\uc9c0 \uc77c\uc5c8\ub2e4. \uacbd\uc81c \uc0ac\uc808\ub2e8\ub3c4 \uc7ac\ubc8c \ucd1d\uc218\ub4e4\uc774 \ud3ec\ud568\ub41c \uc5ed\ub300 \ucd5c\ub300 \uaddc\ubaa8\ub85c \uad6c\uc131\ub418\uc5c8\uc9c0\ub9cc \uc911\uad6d \uce21\uc5d0\uc11c\ub294 \uae30\uc5c5\uc758 2, 3\uc778\uc790\ub4e4\uc774 \ucc38\uc11d\ud574 \uae09\uc774 \ub9de\uc9c0 \uc54a\uc544 \uc2e4\ubb34\uc801\uc778 \ud68c\ub2f4\uc774 \uc9c4\ud589\ub418\uc9c0 \ubabb\ud588\ub2e4\ub294 \uc598\uae30\ub3c4 \ub098\uc654\ub2e4. \uc2ec\uc9c0\uc5b4 '\ud55c\uc911 \uacbd\uc81c\u00b7\ubb34\uc5ed \ud30c\ud2b8\ub108\uc2ed' \ud589\uc0ac\uc5d0\uc11c\ub294 \ubb38\uc7ac\uc778\uc744 \ucde8\uc7ac\ud558\ub294 \uccad\uc640\ub300 \ucd9c\uc785 \uc0ac\uc9c4\uae30\uc0ac 2\uba85\uc744 \uc911\uad6d \uce21 \uacbd\ud638\uc6d0\ub4e4\uc774 \uacfc\uc789 \ub300\uc751\ud558\uba74\uc11c \ud3ed\ud589\ud558\ub294 \uc0ac\uac74\uc774 \uc77c\uc5b4\ub098\uba74\uc11c \uc678\uad50\uc801 \uacb0\ub840\ub77c\ub294 \ube44\ud310\uc774 \ub098\uc654\ub2e4. \uc0ac\ub4dc \uc5b8\uae09\ub3c4 \uc218\uc704\uac00 \ub0ae\uc544\uc9c0\uae34 \ud588\uc9c0\ub9cc \uc5ec\uc804\ud788 \uc785\uc7a5\ucc28\ub97c \uc881\ud788\uc9c0 \ubabb\ud574 \uacf5\ub3d9\uc131\uba85\uc774\ub098 \uacf5\ub3d9 \uc5b8\ub860 \ubc1c\ud45c\ubb38\ub3c4 \ucc44\ud0dd\ud558\uc9c0 \ubabb\ud558\uc5ec \uad00\uacc4 \ubcf5\uc6d0\uc774 \uc27d\uc9c0 \uc54a\uc74c\uc744 \ubcf4\uc774\uae30\ub3c4 \ud588\ub2e4. \uc774\ub294 \uc911\uad6d\uc774 \uc544\uc9c1 \uc0ac\ub4dc\uc5d0 \ub300\ud55c \uc559\uae08\uc774 \ub0a8\uc544\uc788\ub294 \uc0c1\ud669\uc5d0\uc11c \ud55c\uad6d\uc774 \uc5f0\ub0b4 \ubc29\uc911\uc744 \uc11c\ub450\ub974\uba74\uc11c \uc77c\uc5b4\ub0ac\ub2e4\ub294 \ubc18\uc751\uc774\ub2e4.", "sentence_answer": "\ubc29\uc911 \uc804\uc5d0 \uc911\uad6d\uc911\uc559\ud154\ub808\ube44\uc804\uc774 \ubb38\uc7ac\uc778\uacfc\uc758 \uc0ac\uc804 \uc778\ud130\ubdf0\uc5d0\uc11c 3\ubd88 \uc815\ucc45\uc5d0 \ub300\ud55c \uc555\ubc15\uc131 \uc9c8\ubb38\uc744 \uc3df\uc544\ub0c8\uc73c\uba70, \ubc29\ubb38\ud588\uc744 \ub54c\ub294 \ucc28\uad00\ubcf4\uae09 \uc778\uc0ac\uac00 \ub300\ud1b5\ub839\uc744 \uc601\uc811\ud558\uba74\uc11c \ud640\ub300 \ub17c\ub780\uae4c\uc9c0 \uc77c\uc5c8\ub2e4.", "paragraph_id": "6580792-11-1", "paragraph_question": "question: \uc911\uad6d\uc744 \uad6d\ube48\ubc29\ubb38\ud558\uba74\uc11c \ud640\ub300 \ub17c\ub780\uae4c\uc9c0 \uc77c\uc5c8\ub358 \ub300\ud1b5\ub839 \uc601\uc811 \uc778\uc0ac\uc758 \uc9c1\uae09\uc740?, context: \ud558\uc9c0\ub9cc \uc911\uad6d\uc744 \uad6d\ube48\ubc29\ubb38\ud55c \uac83\uce58\uace0\ub294 \uaca9\uc774 \ubd80\uc871\ud558\ub2e4\ub294 \ube44\ud310\ub3c4 \ub098\uc654\ub2e4. \ubc29\uc911 \uc804\uc5d0 \uc911\uad6d\uc911\uc559\ud154\ub808\ube44\uc804\uc774 \ubb38\uc7ac\uc778\uacfc\uc758 \uc0ac\uc804 \uc778\ud130\ubdf0\uc5d0\uc11c 3\ubd88 \uc815\ucc45\uc5d0 \ub300\ud55c \uc555\ubc15\uc131 \uc9c8\ubb38\uc744 \uc3df\uc544\ub0c8\uc73c\uba70, \ubc29\ubb38\ud588\uc744 \ub54c\ub294 \ucc28\uad00\ubcf4\uae09 \uc778\uc0ac\uac00 \ub300\ud1b5\ub839\uc744 \uc601\uc811\ud558\uba74\uc11c \ud640\ub300 \ub17c\ub780\uae4c\uc9c0 \uc77c\uc5c8\ub2e4. \uacbd\uc81c \uc0ac\uc808\ub2e8\ub3c4 \uc7ac\ubc8c \ucd1d\uc218\ub4e4\uc774 \ud3ec\ud568\ub41c \uc5ed\ub300 \ucd5c\ub300 \uaddc\ubaa8\ub85c \uad6c\uc131\ub418\uc5c8\uc9c0\ub9cc \uc911\uad6d \uce21\uc5d0\uc11c\ub294 \uae30\uc5c5\uc758 2, 3\uc778\uc790\ub4e4\uc774 \ucc38\uc11d\ud574 \uae09\uc774 \ub9de\uc9c0 \uc54a\uc544 \uc2e4\ubb34\uc801\uc778 \ud68c\ub2f4\uc774 \uc9c4\ud589\ub418\uc9c0 \ubabb\ud588\ub2e4\ub294 \uc598\uae30\ub3c4 \ub098\uc654\ub2e4. \uc2ec\uc9c0\uc5b4 '\ud55c\uc911 \uacbd\uc81c\u00b7\ubb34\uc5ed \ud30c\ud2b8\ub108\uc2ed' \ud589\uc0ac\uc5d0\uc11c\ub294 \ubb38\uc7ac\uc778\uc744 \ucde8\uc7ac\ud558\ub294 \uccad\uc640\ub300 \ucd9c\uc785 \uc0ac\uc9c4\uae30\uc0ac 2\uba85\uc744 \uc911\uad6d \uce21 \uacbd\ud638\uc6d0\ub4e4\uc774 \uacfc\uc789 \ub300\uc751\ud558\uba74\uc11c \ud3ed\ud589\ud558\ub294 \uc0ac\uac74\uc774 \uc77c\uc5b4\ub098\uba74\uc11c \uc678\uad50\uc801 \uacb0\ub840\ub77c\ub294 \ube44\ud310\uc774 \ub098\uc654\ub2e4. \uc0ac\ub4dc \uc5b8\uae09\ub3c4 \uc218\uc704\uac00 \ub0ae\uc544\uc9c0\uae34 \ud588\uc9c0\ub9cc \uc5ec\uc804\ud788 \uc785\uc7a5\ucc28\ub97c \uc881\ud788\uc9c0 \ubabb\ud574 \uacf5\ub3d9\uc131\uba85\uc774\ub098 \uacf5\ub3d9 \uc5b8\ub860 \ubc1c\ud45c\ubb38\ub3c4 \ucc44\ud0dd\ud558\uc9c0 \ubabb\ud558\uc5ec \uad00\uacc4 \ubcf5\uc6d0\uc774 \uc27d\uc9c0 \uc54a\uc74c\uc744 \ubcf4\uc774\uae30\ub3c4 \ud588\ub2e4. \uc774\ub294 \uc911\uad6d\uc774 \uc544\uc9c1 \uc0ac\ub4dc\uc5d0 \ub300\ud55c \uc559\uae08\uc774 \ub0a8\uc544\uc788\ub294 \uc0c1\ud669\uc5d0\uc11c \ud55c\uad6d\uc774 \uc5f0\ub0b4 \ubc29\uc911\uc744 \uc11c\ub450\ub974\uba74\uc11c \uc77c\uc5b4\ub0ac\ub2e4\ub294 \ubc18\uc751\uc774\ub2e4."} {"question": "\ud55c\ud6a8\uc8fc\ub294 \uc5b8\uc81c \ud604\uc7ac \uc774\ub984\uc73c\ub85c \uac1c\uba85?", "paragraph": "\ud55c\ud6a8\uc8fc\ub294 1987\ub144 2\uc6d4 22\uc77c, \ucda9\uccad\ubd81\ub3c4 \uccad\uc8fc\uc2dc \uc728\ub7c9\ub3d9\uc5d0\uc11c \ub300\ud55c\ubbfc\uad6d \uacf5\uad70 \uc7a5\uad50 \uc544\ubc84\uc9c0\uc640 \uc720\uce58\uc6d0 \uad50\uc0ac \ucd9c\uc2e0 \uc5b4\uba38\ub2c8 \uc0ac\uc774\uc5d0\uc11c \ud0dc\uc5b4\ub0ac\ub2e4. \ud615\uc81c\ub85c\ub294 \uacf5\uad70 \uc7a5\uad50\uc778 \ub3d9\uc0dd\uc774 \uc788\ub2e4. \ucd9c\uc0dd \ub2f9\uc2dc \uc774\ub984\uc740 \ud55c\uc9c0\uc601\uc774\uc5c8\uc73c\ub098, \ucd08\ub4f1\ud559\uad50 3\ud559\ub144\ub54c \ud604\uc7ac \uc774\ub984\uc73c\ub85c \uac1c\uba85\ud558\uc600\ub2e4. \ud55c\ud6a8\uc8fc\ub294 \ube60\ub978 \ub144\uc0dd\uc73c\ub85c 1\ub144 \uc77c\ucc0d 7\uc0b4 \ub54c \ud559\uad50\ub97c \uc785\ud559\ud588\ub2e4. \ub355\uc131\ucd08\ub4f1\ud559\uad50\ub97c \ub2e4\ub2d0 \ub2f9\uc2dc \uafc8\uc774 \ub2e4\uc591\ud588\ub294\ub370 \uc2dc\uc778, \ud654\uac00, \uc120\uc0dd\ub2d8, \uc870\ub828\uc0ac, \uc218\uc758\uc0ac\uac00 \ub418\uace0 \uc2f6\uc5c8\ub2e4\uace0 \ud55c\ub2e4. \uc774\ud6c4 \uccad\uc8fc \uc728\ub7c9\uc911\ud559\uad50\ub97c \uc878\uc5c5\ud558\uace0 \uccad\uc8fc\uc5ec\uc790\uace0\ub4f1\ud559\uad50\uc5d0 \uc785\ud559\ud588\ub2e4\uac00 \uc131\ub0a8\uc2dc \ubd84\ub2f9\uad6c \uc18c\uc7ac\uc758 \ubd88\uace1\uace0\ub4f1\ud559\uad50\ub85c \uc804\ud559\ud588\ub2e4. \uace0\ub4f1\ud559\uad50 \uc2dc\uc808\uc5d0\ub294 \uc778\ubb38\uacc4\ub85c \uc9c4\ud559\ud588\uc5c8\ub294\ub370, \uc774 \ub54c\ubb38\uc5d0 \uc7a0\uc2dc \uc120\uc0dd\ub2d8\uc774 \ubaa9\ud45c\uc600\uc5c8\uace0 \uc601\ud654 \uac10\ub3c5\uc5d0 \ub300\ud55c \uafc8\ub3c4 \uc788\uc5c8\ub2e4. \uadf8\ub7ec\ub358 \uc911 2003\ub144 2\uc6d4 22\uc77c, \uace0\ub4f1\ud559\uad50 1\ud559\ub144 \uaca8\uc6b8\ubc29\ud559\ub54c \ubbf8\uc2a4 \ube59\uadf8\ub808 \uc120\ubc1c\ub300\ud68c\uc5d0\uc11c \ub300\uc0c1\uc744 \uc218\uc0c1\ud558\uba70 \uc5f0\uc608\uacc4 \ub370\ubdd4\ub97c \uc900\ube44\ub97c \ud588\ub2e4.", "answer": "\ucd08\ub4f1\ud559\uad50 3\ud559\ub144\ub54c", "sentence": "\ucd9c\uc0dd \ub2f9\uc2dc \uc774\ub984\uc740 \ud55c\uc9c0\uc601\uc774\uc5c8\uc73c\ub098, \ucd08\ub4f1\ud559\uad50 3\ud559\ub144\ub54c \ud604\uc7ac \uc774\ub984\uc73c\ub85c \uac1c\uba85\ud558\uc600\ub2e4.", "paragraph_sentence": "\ud55c\ud6a8\uc8fc\ub294 1987\ub144 2\uc6d4 22\uc77c, \ucda9\uccad\ubd81\ub3c4 \uccad\uc8fc\uc2dc \uc728\ub7c9\ub3d9\uc5d0\uc11c \ub300\ud55c\ubbfc\uad6d \uacf5\uad70 \uc7a5\uad50 \uc544\ubc84\uc9c0\uc640 \uc720\uce58\uc6d0 \uad50\uc0ac \ucd9c\uc2e0 \uc5b4\uba38\ub2c8 \uc0ac\uc774\uc5d0\uc11c \ud0dc\uc5b4\ub0ac\ub2e4. \ud615\uc81c\ub85c\ub294 \uacf5\uad70 \uc7a5\uad50\uc778 \ub3d9\uc0dd\uc774 \uc788\ub2e4. \ucd9c\uc0dd \ub2f9\uc2dc \uc774\ub984\uc740 \ud55c\uc9c0\uc601\uc774\uc5c8\uc73c\ub098, \ucd08\ub4f1\ud559\uad50 3\ud559\ub144\ub54c \ud604\uc7ac \uc774\ub984\uc73c\ub85c \uac1c\uba85\ud558\uc600\ub2e4. \ud55c\ud6a8\uc8fc\ub294 \ube60\ub978 \ub144\uc0dd\uc73c\ub85c 1\ub144 \uc77c\ucc0d 7\uc0b4 \ub54c \ud559\uad50\ub97c \uc785\ud559\ud588\ub2e4. \ub355\uc131\ucd08\ub4f1\ud559\uad50\ub97c \ub2e4\ub2d0 \ub2f9\uc2dc \uafc8\uc774 \ub2e4\uc591\ud588\ub294\ub370 \uc2dc\uc778, \ud654\uac00, \uc120\uc0dd\ub2d8, \uc870\ub828\uc0ac, \uc218\uc758\uc0ac\uac00 \ub418\uace0 \uc2f6\uc5c8\ub2e4\uace0 \ud55c\ub2e4. \uc774\ud6c4 \uccad\uc8fc \uc728\ub7c9\uc911\ud559\uad50\ub97c \uc878\uc5c5\ud558\uace0 \uccad\uc8fc\uc5ec\uc790\uace0\ub4f1\ud559\uad50\uc5d0 \uc785\ud559\ud588\ub2e4\uac00 \uc131\ub0a8\uc2dc \ubd84\ub2f9\uad6c \uc18c\uc7ac\uc758 \ubd88\uace1\uace0\ub4f1\ud559\uad50\ub85c \uc804\ud559\ud588\ub2e4. \uace0\ub4f1\ud559\uad50 \uc2dc\uc808\uc5d0\ub294 \uc778\ubb38\uacc4\ub85c \uc9c4\ud559\ud588\uc5c8\ub294\ub370, \uc774 \ub54c\ubb38\uc5d0 \uc7a0\uc2dc \uc120\uc0dd\ub2d8\uc774 \ubaa9\ud45c\uc600\uc5c8\uace0 \uc601\ud654 \uac10\ub3c5\uc5d0 \ub300\ud55c \uafc8\ub3c4 \uc788\uc5c8\ub2e4. \uadf8\ub7ec\ub358 \uc911 2003\ub144 2\uc6d4 22\uc77c, \uace0\ub4f1\ud559\uad50 1\ud559\ub144 \uaca8\uc6b8\ubc29\ud559\ub54c \ubbf8\uc2a4 \ube59\uadf8\ub808 \uc120\ubc1c\ub300\ud68c\uc5d0\uc11c \ub300\uc0c1\uc744 \uc218\uc0c1\ud558\uba70 \uc5f0\uc608\uacc4 \ub370\ubdd4\ub97c \uc900\ube44\ub97c \ud588\ub2e4.", "paragraph_answer": "\ud55c\ud6a8\uc8fc\ub294 1987\ub144 2\uc6d4 22\uc77c, \ucda9\uccad\ubd81\ub3c4 \uccad\uc8fc\uc2dc \uc728\ub7c9\ub3d9\uc5d0\uc11c \ub300\ud55c\ubbfc\uad6d \uacf5\uad70 \uc7a5\uad50 \uc544\ubc84\uc9c0\uc640 \uc720\uce58\uc6d0 \uad50\uc0ac \ucd9c\uc2e0 \uc5b4\uba38\ub2c8 \uc0ac\uc774\uc5d0\uc11c \ud0dc\uc5b4\ub0ac\ub2e4. \ud615\uc81c\ub85c\ub294 \uacf5\uad70 \uc7a5\uad50\uc778 \ub3d9\uc0dd\uc774 \uc788\ub2e4. \ucd9c\uc0dd \ub2f9\uc2dc \uc774\ub984\uc740 \ud55c\uc9c0\uc601\uc774\uc5c8\uc73c\ub098, \ucd08\ub4f1\ud559\uad50 3\ud559\ub144\ub54c \ud604\uc7ac \uc774\ub984\uc73c\ub85c \uac1c\uba85\ud558\uc600\ub2e4. \ud55c\ud6a8\uc8fc\ub294 \ube60\ub978 \ub144\uc0dd\uc73c\ub85c 1\ub144 \uc77c\ucc0d 7\uc0b4 \ub54c \ud559\uad50\ub97c \uc785\ud559\ud588\ub2e4. \ub355\uc131\ucd08\ub4f1\ud559\uad50\ub97c \ub2e4\ub2d0 \ub2f9\uc2dc \uafc8\uc774 \ub2e4\uc591\ud588\ub294\ub370 \uc2dc\uc778, \ud654\uac00, \uc120\uc0dd\ub2d8, \uc870\ub828\uc0ac, \uc218\uc758\uc0ac\uac00 \ub418\uace0 \uc2f6\uc5c8\ub2e4\uace0 \ud55c\ub2e4. \uc774\ud6c4 \uccad\uc8fc \uc728\ub7c9\uc911\ud559\uad50\ub97c \uc878\uc5c5\ud558\uace0 \uccad\uc8fc\uc5ec\uc790\uace0\ub4f1\ud559\uad50\uc5d0 \uc785\ud559\ud588\ub2e4\uac00 \uc131\ub0a8\uc2dc \ubd84\ub2f9\uad6c \uc18c\uc7ac\uc758 \ubd88\uace1\uace0\ub4f1\ud559\uad50\ub85c \uc804\ud559\ud588\ub2e4. \uace0\ub4f1\ud559\uad50 \uc2dc\uc808\uc5d0\ub294 \uc778\ubb38\uacc4\ub85c \uc9c4\ud559\ud588\uc5c8\ub294\ub370, \uc774 \ub54c\ubb38\uc5d0 \uc7a0\uc2dc \uc120\uc0dd\ub2d8\uc774 \ubaa9\ud45c\uc600\uc5c8\uace0 \uc601\ud654 \uac10\ub3c5\uc5d0 \ub300\ud55c \uafc8\ub3c4 \uc788\uc5c8\ub2e4. \uadf8\ub7ec\ub358 \uc911 2003\ub144 2\uc6d4 22\uc77c, \uace0\ub4f1\ud559\uad50 1\ud559\ub144 \uaca8\uc6b8\ubc29\ud559\ub54c \ubbf8\uc2a4 \ube59\uadf8\ub808 \uc120\ubc1c\ub300\ud68c\uc5d0\uc11c \ub300\uc0c1\uc744 \uc218\uc0c1\ud558\uba70 \uc5f0\uc608\uacc4 \ub370\ubdd4\ub97c \uc900\ube44\ub97c \ud588\ub2e4.", "sentence_answer": "\ucd9c\uc0dd \ub2f9\uc2dc \uc774\ub984\uc740 \ud55c\uc9c0\uc601\uc774\uc5c8\uc73c\ub098, \ucd08\ub4f1\ud559\uad50 3\ud559\ub144\ub54c \ud604\uc7ac \uc774\ub984\uc73c\ub85c \uac1c\uba85\ud558\uc600\ub2e4.", "paragraph_id": "5940051-1-1", "paragraph_question": "question: \ud55c\ud6a8\uc8fc\ub294 \uc5b8\uc81c \ud604\uc7ac \uc774\ub984\uc73c\ub85c \uac1c\uba85?, context: \ud55c\ud6a8\uc8fc\ub294 1987\ub144 2\uc6d4 22\uc77c, \ucda9\uccad\ubd81\ub3c4 \uccad\uc8fc\uc2dc \uc728\ub7c9\ub3d9\uc5d0\uc11c \ub300\ud55c\ubbfc\uad6d \uacf5\uad70 \uc7a5\uad50 \uc544\ubc84\uc9c0\uc640 \uc720\uce58\uc6d0 \uad50\uc0ac \ucd9c\uc2e0 \uc5b4\uba38\ub2c8 \uc0ac\uc774\uc5d0\uc11c \ud0dc\uc5b4\ub0ac\ub2e4. \ud615\uc81c\ub85c\ub294 \uacf5\uad70 \uc7a5\uad50\uc778 \ub3d9\uc0dd\uc774 \uc788\ub2e4. \ucd9c\uc0dd \ub2f9\uc2dc \uc774\ub984\uc740 \ud55c\uc9c0\uc601\uc774\uc5c8\uc73c\ub098, \ucd08\ub4f1\ud559\uad50 3\ud559\ub144\ub54c \ud604\uc7ac \uc774\ub984\uc73c\ub85c \uac1c\uba85\ud558\uc600\ub2e4. \ud55c\ud6a8\uc8fc\ub294 \ube60\ub978 \ub144\uc0dd\uc73c\ub85c 1\ub144 \uc77c\ucc0d 7\uc0b4 \ub54c \ud559\uad50\ub97c \uc785\ud559\ud588\ub2e4. \ub355\uc131\ucd08\ub4f1\ud559\uad50\ub97c \ub2e4\ub2d0 \ub2f9\uc2dc \uafc8\uc774 \ub2e4\uc591\ud588\ub294\ub370 \uc2dc\uc778, \ud654\uac00, \uc120\uc0dd\ub2d8, \uc870\ub828\uc0ac, \uc218\uc758\uc0ac\uac00 \ub418\uace0 \uc2f6\uc5c8\ub2e4\uace0 \ud55c\ub2e4. \uc774\ud6c4 \uccad\uc8fc \uc728\ub7c9\uc911\ud559\uad50\ub97c \uc878\uc5c5\ud558\uace0 \uccad\uc8fc\uc5ec\uc790\uace0\ub4f1\ud559\uad50\uc5d0 \uc785\ud559\ud588\ub2e4\uac00 \uc131\ub0a8\uc2dc \ubd84\ub2f9\uad6c \uc18c\uc7ac\uc758 \ubd88\uace1\uace0\ub4f1\ud559\uad50\ub85c \uc804\ud559\ud588\ub2e4. \uace0\ub4f1\ud559\uad50 \uc2dc\uc808\uc5d0\ub294 \uc778\ubb38\uacc4\ub85c \uc9c4\ud559\ud588\uc5c8\ub294\ub370, \uc774 \ub54c\ubb38\uc5d0 \uc7a0\uc2dc \uc120\uc0dd\ub2d8\uc774 \ubaa9\ud45c\uc600\uc5c8\uace0 \uc601\ud654 \uac10\ub3c5\uc5d0 \ub300\ud55c \uafc8\ub3c4 \uc788\uc5c8\ub2e4. \uadf8\ub7ec\ub358 \uc911 2003\ub144 2\uc6d4 22\uc77c, \uace0\ub4f1\ud559\uad50 1\ud559\ub144 \uaca8\uc6b8\ubc29\ud559\ub54c \ubbf8\uc2a4 \ube59\uadf8\ub808 \uc120\ubc1c\ub300\ud68c\uc5d0\uc11c \ub300\uc0c1\uc744 \uc218\uc0c1\ud558\uba70 \uc5f0\uc608\uacc4 \ub370\ubdd4\ub97c \uc900\ube44\ub97c \ud588\ub2e4."} {"question": "\uc81c\uc778 \ud06c\ub798\uc774\ub4dc \uc870\uc9c0\uac00 1972\ub144\uc5d0 \uc4f4 \uc18c\uc124\uc758 \uc81c\ubaa9\uc740 \ubb34\uc5c7\uc778\uac00?", "paragraph": "\ud314\ub9ac \ubaa8\uc653\uc774 1963\ub144 \uc791\uc131\ud55c \ud53d\uc158 \ud68c\uace0\ub85d \u300a\uc6b8\uc9c0\uc54a\ub294 \ub291\ub300\u300b(Never Cry Wolf)\ub294 \ub300\uc911 \ubb38\ud559\uc5d0\uc11c \ucc98\uc74c\uc73c\ub85c \ub291\ub300\ub97c \uae0d\uc815\uc801\uc73c\ub85c \ud45c\ud604\ud55c \uc791\ud488\uc774\uba70, \ub610\ud55c \ub291\ub300\uc5d0 \uad00\ud55c \uc791\ud488 \uc911 \uac00\uc7a5 \uc778\uae30\uc788\ub294 \uc791\ud488\uc73c\ub85c \uc5ec\uaca8\uc84c\uc73c\uba70 \uc601\ud654\ud654\ub418\uace0 \uc218\uc2ed\ub144 \uac04 \ud559\uad50\uc5d0\uc11c \uac00\ub974\uce58\uae30\ub3c4 \ud558\uc600\ub2e4. \ube44\ub85d \ub291\ub300\ub4e4\uc744 \ub300\uc911\uc758 \uc778\uc2dd\uc5d0\uc11c \uc0ac\ub791\uc2a4\ub7fd\uac8c \ubc14\uafd4\ub193\uc558\uc73c\ub098, \uc0ac\uc2e4 \ubd80\uc815\ud655\ud558\uac8c \ud611\ub825\uc801\uc774\uba70 \uadc0\uc871\uc801\uc774\uace0 \ubd80\uc801\uc808\ud55c \uc9c0\uc2dc\uc5d0 \uac70\ubd80\ud558\ub294 \ub4f1\uc758 \ub291\ub300\uc5d0 \ub300\ud55c \uc798\ubabb\ub41c \uc778\uc2dd\uc744 \uac00\uc838\uc654\ub2e4\uace0 \ube44\ud310\ubc1b\uae30\ub3c4 \ud558\uc600\ub2e4. \uc774 \ucc45\uc740 \ub098\uc911\uc5d0 \uc18c\ube44\uc5d0\ud2b8 \uc5f0\ubc29 \uacf5\uc0b0\ub2f9\uc774 \ub291\ub300\uc758 \uc778\uac04 \uacf5\uaca9\uc5d0 \uad00\ud55c \uc62c\ubc14\ub978 \uc815\ubcf4 \ucea0\ud328\uc778\uc73c\ub85c \uc774\uc6a9\ub418\uae30\ub3c4 \ud558\uc600\ub2e4. \ubaa8\uc653\uc5d0 \uad00\ud55c \uc774\ub7ec\ud55c \ube44\uc2b7\ud55c \ube44\ud310\uc73c\ub85c \ubbf8\uc0ac \ub370\ud3ec\uc138\uce74\uc758 \u300a\ubbf8\uc0ac: \uc5b4 \uba54\ubaa8\ub9ac\uc81c \uc624\ube0c \ub354 \ud640\ub85c\ucf54\uc2a4\ud2b8\u300b(Misha: A M\u00e9moire of the Holocaust Years)\uc5d0\uc11c\ub294 \ud3f4\ub780\ub4dc \uc810\ub839\uae30 \ub3d9\uc548 \ub291\ub300\uac00 \uae30\ub978 \uc57c\uc0dd\uc544\uc5d0 \ub300\ud55c \ud55c \uc800\uc790\uc758 \uac70\uc9d3 \ub17c\ub780\uc774 \ub098\uc628\ub2e4. \uc81c\uc778 \ud06c\ub798\uc774\ub4dc \uc870\uc9c0\uc758 1972\ub144 \uc18c\uc124 \u300a\uc904\ub9ac\uc5d0\uc758 \ub291\ub300\u300b(Julie of the Wolves)\uc758 1\ubd80\uc5d0\uc11c\ub294 \uc18c\ub140\uc640 \ub291\ub300 \ubb34\ub9ac \uc0ac\uc774\uc758 \uad00\uacc4\ub97c \ub2e4\ub8e8\uace0 \uc788\ub2e4. \uc774 \ubd80\uc758 \ub9c8\uc9c0\ub9c9 \ud56d\ubaa9\uc5d0\uc11c\ub294 \ub291\ub300\uc758 \uad00\uc810\uc5d0\uc11c \ubc14\ub77c\ubcf4\uba70 \uc758\uc778\ud654\ub418\uc5c8\uc73c\ub098 \uc0dd\ubb3c\ud559, \ub3d9\ubb3c\ud559\uc801\uc73c\ub85c \ub3d9\ubb3c\uc744 \ubc14\ub77c\ubcf8\ub2e4. \uc5ec\ub7ec \ud604\ub300 \uc544\ub3d9 \ubb38\ud559 \uc791\uac00\ub4e4\uc740 \ub291\ub300\ub97c \ub354 \uae0d\uc815\uc801\uc73c\ub85c \ubc14\ub77c\ubcf4\uae30 \uc704\ud574 \uace0\uc804 \ub3d9\ud654\uc758 \ub291\ub300 \uc778\uc2dd\uc744 \uace0\uce58\uace0 \uc788\ub2e4. \uc774 \uc608\ub85c \ubbf8\uad6d\uc758 \u3008\ud658\uacbd\uc801 \ub291\ub300\uc640 \uc544\uae30\ub3fc\uc9c0 \uc0bc\ud615\uc81c\u3009(Eco-wolf and the Three Little Pigs)\uc640 \u3008\uc591\uce58\uae30 \uc18c\ub144\u3009(The Wolf who cried Boy) \ub3d9\ud654\uac00 \uc788\ub2e4.", "answer": "\uc904\ub9ac\uc5d0\uc758 \ub291\ub300", "sentence": "\uc81c\uc778 \ud06c\ub798\uc774\ub4dc \uc870\uc9c0\uc758 1972\ub144 \uc18c\uc124 \u300a \uc904\ub9ac\uc5d0\uc758 \ub291\ub300 \u300b(Julie of the Wolves)\uc758 1\ubd80\uc5d0\uc11c\ub294 \uc18c\ub140\uc640 \ub291\ub300 \ubb34\ub9ac \uc0ac\uc774\uc758 \uad00\uacc4\ub97c \ub2e4\ub8e8\uace0 \uc788\ub2e4.", "paragraph_sentence": "\ud314\ub9ac \ubaa8\uc653\uc774 1963\ub144 \uc791\uc131\ud55c \ud53d\uc158 \ud68c\uace0\ub85d \u300a\uc6b8\uc9c0\uc54a\ub294 \ub291\ub300\u300b(Never Cry Wolf)\ub294 \ub300\uc911 \ubb38\ud559\uc5d0\uc11c \ucc98\uc74c\uc73c\ub85c \ub291\ub300\ub97c \uae0d\uc815\uc801\uc73c\ub85c \ud45c\ud604\ud55c \uc791\ud488\uc774\uba70, \ub610\ud55c \ub291\ub300\uc5d0 \uad00\ud55c \uc791\ud488 \uc911 \uac00\uc7a5 \uc778\uae30\uc788\ub294 \uc791\ud488\uc73c\ub85c \uc5ec\uaca8\uc84c\uc73c\uba70 \uc601\ud654\ud654\ub418\uace0 \uc218\uc2ed\ub144 \uac04 \ud559\uad50\uc5d0\uc11c \uac00\ub974\uce58\uae30\ub3c4 \ud558\uc600\ub2e4. \ube44\ub85d \ub291\ub300\ub4e4\uc744 \ub300\uc911\uc758 \uc778\uc2dd\uc5d0\uc11c \uc0ac\ub791\uc2a4\ub7fd\uac8c \ubc14\uafd4\ub193\uc558\uc73c\ub098, \uc0ac\uc2e4 \ubd80\uc815\ud655\ud558\uac8c \ud611\ub825\uc801\uc774\uba70 \uadc0\uc871\uc801\uc774\uace0 \ubd80\uc801\uc808\ud55c \uc9c0\uc2dc\uc5d0 \uac70\ubd80\ud558\ub294 \ub4f1\uc758 \ub291\ub300\uc5d0 \ub300\ud55c \uc798\ubabb\ub41c \uc778\uc2dd\uc744 \uac00\uc838\uc654\ub2e4\uace0 \ube44\ud310\ubc1b\uae30\ub3c4 \ud558\uc600\ub2e4. \uc774 \ucc45\uc740 \ub098\uc911\uc5d0 \uc18c\ube44\uc5d0\ud2b8 \uc5f0\ubc29 \uacf5\uc0b0\ub2f9\uc774 \ub291\ub300\uc758 \uc778\uac04 \uacf5\uaca9\uc5d0 \uad00\ud55c \uc62c\ubc14\ub978 \uc815\ubcf4 \ucea0\ud328\uc778\uc73c\ub85c \uc774\uc6a9\ub418\uae30\ub3c4 \ud558\uc600\ub2e4. \ubaa8\uc653\uc5d0 \uad00\ud55c \uc774\ub7ec\ud55c \ube44\uc2b7\ud55c \ube44\ud310\uc73c\ub85c \ubbf8\uc0ac \ub370\ud3ec\uc138\uce74\uc758 \u300a\ubbf8\uc0ac: \uc5b4 \uba54\ubaa8\ub9ac\uc81c \uc624\ube0c \ub354 \ud640\ub85c\ucf54\uc2a4\ud2b8\u300b(Misha: A M\u00e9moire of the Holocaust Years)\uc5d0\uc11c\ub294 \ud3f4\ub780\ub4dc \uc810\ub839\uae30 \ub3d9\uc548 \ub291\ub300\uac00 \uae30\ub978 \uc57c\uc0dd\uc544\uc5d0 \ub300\ud55c \ud55c \uc800\uc790\uc758 \uac70\uc9d3 \ub17c\ub780\uc774 \ub098\uc628\ub2e4. \uc81c\uc778 \ud06c\ub798\uc774\ub4dc \uc870\uc9c0\uc758 1972\ub144 \uc18c\uc124 \u300a \uc904\ub9ac\uc5d0\uc758 \ub291\ub300 \u300b(Julie of the Wolves)\uc758 1\ubd80\uc5d0\uc11c\ub294 \uc18c\ub140\uc640 \ub291\ub300 \ubb34\ub9ac \uc0ac\uc774\uc758 \uad00\uacc4\ub97c \ub2e4\ub8e8\uace0 \uc788\ub2e4. \uc774 \ubd80\uc758 \ub9c8\uc9c0\ub9c9 \ud56d\ubaa9\uc5d0\uc11c\ub294 \ub291\ub300\uc758 \uad00\uc810\uc5d0\uc11c \ubc14\ub77c\ubcf4\uba70 \uc758\uc778\ud654\ub418\uc5c8\uc73c\ub098 \uc0dd\ubb3c\ud559, \ub3d9\ubb3c\ud559\uc801\uc73c\ub85c \ub3d9\ubb3c\uc744 \ubc14\ub77c\ubcf8\ub2e4. \uc5ec\ub7ec \ud604\ub300 \uc544\ub3d9 \ubb38\ud559 \uc791\uac00\ub4e4\uc740 \ub291\ub300\ub97c \ub354 \uae0d\uc815\uc801\uc73c\ub85c \ubc14\ub77c\ubcf4\uae30 \uc704\ud574 \uace0\uc804 \ub3d9\ud654\uc758 \ub291\ub300 \uc778\uc2dd\uc744 \uace0\uce58\uace0 \uc788\ub2e4. \uc774 \uc608\ub85c \ubbf8\uad6d\uc758 \u3008\ud658\uacbd\uc801 \ub291\ub300\uc640 \uc544\uae30\ub3fc\uc9c0 \uc0bc\ud615\uc81c\u3009(Eco-wolf and the Three Little Pigs)\uc640 \u3008\uc591\uce58\uae30 \uc18c\ub144\u3009(The Wolf who cried Boy) \ub3d9\ud654\uac00 \uc788\ub2e4.", "paragraph_answer": "\ud314\ub9ac \ubaa8\uc653\uc774 1963\ub144 \uc791\uc131\ud55c \ud53d\uc158 \ud68c\uace0\ub85d \u300a\uc6b8\uc9c0\uc54a\ub294 \ub291\ub300\u300b(Never Cry Wolf)\ub294 \ub300\uc911 \ubb38\ud559\uc5d0\uc11c \ucc98\uc74c\uc73c\ub85c \ub291\ub300\ub97c \uae0d\uc815\uc801\uc73c\ub85c \ud45c\ud604\ud55c \uc791\ud488\uc774\uba70, \ub610\ud55c \ub291\ub300\uc5d0 \uad00\ud55c \uc791\ud488 \uc911 \uac00\uc7a5 \uc778\uae30\uc788\ub294 \uc791\ud488\uc73c\ub85c \uc5ec\uaca8\uc84c\uc73c\uba70 \uc601\ud654\ud654\ub418\uace0 \uc218\uc2ed\ub144 \uac04 \ud559\uad50\uc5d0\uc11c \uac00\ub974\uce58\uae30\ub3c4 \ud558\uc600\ub2e4. \ube44\ub85d \ub291\ub300\ub4e4\uc744 \ub300\uc911\uc758 \uc778\uc2dd\uc5d0\uc11c \uc0ac\ub791\uc2a4\ub7fd\uac8c \ubc14\uafd4\ub193\uc558\uc73c\ub098, \uc0ac\uc2e4 \ubd80\uc815\ud655\ud558\uac8c \ud611\ub825\uc801\uc774\uba70 \uadc0\uc871\uc801\uc774\uace0 \ubd80\uc801\uc808\ud55c \uc9c0\uc2dc\uc5d0 \uac70\ubd80\ud558\ub294 \ub4f1\uc758 \ub291\ub300\uc5d0 \ub300\ud55c \uc798\ubabb\ub41c \uc778\uc2dd\uc744 \uac00\uc838\uc654\ub2e4\uace0 \ube44\ud310\ubc1b\uae30\ub3c4 \ud558\uc600\ub2e4. \uc774 \ucc45\uc740 \ub098\uc911\uc5d0 \uc18c\ube44\uc5d0\ud2b8 \uc5f0\ubc29 \uacf5\uc0b0\ub2f9\uc774 \ub291\ub300\uc758 \uc778\uac04 \uacf5\uaca9\uc5d0 \uad00\ud55c \uc62c\ubc14\ub978 \uc815\ubcf4 \ucea0\ud328\uc778\uc73c\ub85c \uc774\uc6a9\ub418\uae30\ub3c4 \ud558\uc600\ub2e4. \ubaa8\uc653\uc5d0 \uad00\ud55c \uc774\ub7ec\ud55c \ube44\uc2b7\ud55c \ube44\ud310\uc73c\ub85c \ubbf8\uc0ac \ub370\ud3ec\uc138\uce74\uc758 \u300a\ubbf8\uc0ac: \uc5b4 \uba54\ubaa8\ub9ac\uc81c \uc624\ube0c \ub354 \ud640\ub85c\ucf54\uc2a4\ud2b8\u300b(Misha: A M\u00e9moire of the Holocaust Years)\uc5d0\uc11c\ub294 \ud3f4\ub780\ub4dc \uc810\ub839\uae30 \ub3d9\uc548 \ub291\ub300\uac00 \uae30\ub978 \uc57c\uc0dd\uc544\uc5d0 \ub300\ud55c \ud55c \uc800\uc790\uc758 \uac70\uc9d3 \ub17c\ub780\uc774 \ub098\uc628\ub2e4. \uc81c\uc778 \ud06c\ub798\uc774\ub4dc \uc870\uc9c0\uc758 1972\ub144 \uc18c\uc124 \u300a \uc904\ub9ac\uc5d0\uc758 \ub291\ub300 \u300b(Julie of the Wolves)\uc758 1\ubd80\uc5d0\uc11c\ub294 \uc18c\ub140\uc640 \ub291\ub300 \ubb34\ub9ac \uc0ac\uc774\uc758 \uad00\uacc4\ub97c \ub2e4\ub8e8\uace0 \uc788\ub2e4. \uc774 \ubd80\uc758 \ub9c8\uc9c0\ub9c9 \ud56d\ubaa9\uc5d0\uc11c\ub294 \ub291\ub300\uc758 \uad00\uc810\uc5d0\uc11c \ubc14\ub77c\ubcf4\uba70 \uc758\uc778\ud654\ub418\uc5c8\uc73c\ub098 \uc0dd\ubb3c\ud559, \ub3d9\ubb3c\ud559\uc801\uc73c\ub85c \ub3d9\ubb3c\uc744 \ubc14\ub77c\ubcf8\ub2e4. \uc5ec\ub7ec \ud604\ub300 \uc544\ub3d9 \ubb38\ud559 \uc791\uac00\ub4e4\uc740 \ub291\ub300\ub97c \ub354 \uae0d\uc815\uc801\uc73c\ub85c \ubc14\ub77c\ubcf4\uae30 \uc704\ud574 \uace0\uc804 \ub3d9\ud654\uc758 \ub291\ub300 \uc778\uc2dd\uc744 \uace0\uce58\uace0 \uc788\ub2e4. \uc774 \uc608\ub85c \ubbf8\uad6d\uc758 \u3008\ud658\uacbd\uc801 \ub291\ub300\uc640 \uc544\uae30\ub3fc\uc9c0 \uc0bc\ud615\uc81c\u3009(Eco-wolf and the Three Little Pigs)\uc640 \u3008\uc591\uce58\uae30 \uc18c\ub144\u3009(The Wolf who cried Boy) \ub3d9\ud654\uac00 \uc788\ub2e4.", "sentence_answer": "\uc81c\uc778 \ud06c\ub798\uc774\ub4dc \uc870\uc9c0\uc758 1972\ub144 \uc18c\uc124 \u300a \uc904\ub9ac\uc5d0\uc758 \ub291\ub300 \u300b(Julie of the Wolves)\uc758 1\ubd80\uc5d0\uc11c\ub294 \uc18c\ub140\uc640 \ub291\ub300 \ubb34\ub9ac \uc0ac\uc774\uc758 \uad00\uacc4\ub97c \ub2e4\ub8e8\uace0 \uc788\ub2e4.", "paragraph_id": "6572454-46-1", "paragraph_question": "question: \uc81c\uc778 \ud06c\ub798\uc774\ub4dc \uc870\uc9c0\uac00 1972\ub144\uc5d0 \uc4f4 \uc18c\uc124\uc758 \uc81c\ubaa9\uc740 \ubb34\uc5c7\uc778\uac00?, context: \ud314\ub9ac \ubaa8\uc653\uc774 1963\ub144 \uc791\uc131\ud55c \ud53d\uc158 \ud68c\uace0\ub85d \u300a\uc6b8\uc9c0\uc54a\ub294 \ub291\ub300\u300b(Never Cry Wolf)\ub294 \ub300\uc911 \ubb38\ud559\uc5d0\uc11c \ucc98\uc74c\uc73c\ub85c \ub291\ub300\ub97c \uae0d\uc815\uc801\uc73c\ub85c \ud45c\ud604\ud55c \uc791\ud488\uc774\uba70, \ub610\ud55c \ub291\ub300\uc5d0 \uad00\ud55c \uc791\ud488 \uc911 \uac00\uc7a5 \uc778\uae30\uc788\ub294 \uc791\ud488\uc73c\ub85c \uc5ec\uaca8\uc84c\uc73c\uba70 \uc601\ud654\ud654\ub418\uace0 \uc218\uc2ed\ub144 \uac04 \ud559\uad50\uc5d0\uc11c \uac00\ub974\uce58\uae30\ub3c4 \ud558\uc600\ub2e4. \ube44\ub85d \ub291\ub300\ub4e4\uc744 \ub300\uc911\uc758 \uc778\uc2dd\uc5d0\uc11c \uc0ac\ub791\uc2a4\ub7fd\uac8c \ubc14\uafd4\ub193\uc558\uc73c\ub098, \uc0ac\uc2e4 \ubd80\uc815\ud655\ud558\uac8c \ud611\ub825\uc801\uc774\uba70 \uadc0\uc871\uc801\uc774\uace0 \ubd80\uc801\uc808\ud55c \uc9c0\uc2dc\uc5d0 \uac70\ubd80\ud558\ub294 \ub4f1\uc758 \ub291\ub300\uc5d0 \ub300\ud55c \uc798\ubabb\ub41c \uc778\uc2dd\uc744 \uac00\uc838\uc654\ub2e4\uace0 \ube44\ud310\ubc1b\uae30\ub3c4 \ud558\uc600\ub2e4. \uc774 \ucc45\uc740 \ub098\uc911\uc5d0 \uc18c\ube44\uc5d0\ud2b8 \uc5f0\ubc29 \uacf5\uc0b0\ub2f9\uc774 \ub291\ub300\uc758 \uc778\uac04 \uacf5\uaca9\uc5d0 \uad00\ud55c \uc62c\ubc14\ub978 \uc815\ubcf4 \ucea0\ud328\uc778\uc73c\ub85c \uc774\uc6a9\ub418\uae30\ub3c4 \ud558\uc600\ub2e4. \ubaa8\uc653\uc5d0 \uad00\ud55c \uc774\ub7ec\ud55c \ube44\uc2b7\ud55c \ube44\ud310\uc73c\ub85c \ubbf8\uc0ac \ub370\ud3ec\uc138\uce74\uc758 \u300a\ubbf8\uc0ac: \uc5b4 \uba54\ubaa8\ub9ac\uc81c \uc624\ube0c \ub354 \ud640\ub85c\ucf54\uc2a4\ud2b8\u300b(Misha: A M\u00e9moire of the Holocaust Years)\uc5d0\uc11c\ub294 \ud3f4\ub780\ub4dc \uc810\ub839\uae30 \ub3d9\uc548 \ub291\ub300\uac00 \uae30\ub978 \uc57c\uc0dd\uc544\uc5d0 \ub300\ud55c \ud55c \uc800\uc790\uc758 \uac70\uc9d3 \ub17c\ub780\uc774 \ub098\uc628\ub2e4. \uc81c\uc778 \ud06c\ub798\uc774\ub4dc \uc870\uc9c0\uc758 1972\ub144 \uc18c\uc124 \u300a\uc904\ub9ac\uc5d0\uc758 \ub291\ub300\u300b(Julie of the Wolves)\uc758 1\ubd80\uc5d0\uc11c\ub294 \uc18c\ub140\uc640 \ub291\ub300 \ubb34\ub9ac \uc0ac\uc774\uc758 \uad00\uacc4\ub97c \ub2e4\ub8e8\uace0 \uc788\ub2e4. \uc774 \ubd80\uc758 \ub9c8\uc9c0\ub9c9 \ud56d\ubaa9\uc5d0\uc11c\ub294 \ub291\ub300\uc758 \uad00\uc810\uc5d0\uc11c \ubc14\ub77c\ubcf4\uba70 \uc758\uc778\ud654\ub418\uc5c8\uc73c\ub098 \uc0dd\ubb3c\ud559, \ub3d9\ubb3c\ud559\uc801\uc73c\ub85c \ub3d9\ubb3c\uc744 \ubc14\ub77c\ubcf8\ub2e4. \uc5ec\ub7ec \ud604\ub300 \uc544\ub3d9 \ubb38\ud559 \uc791\uac00\ub4e4\uc740 \ub291\ub300\ub97c \ub354 \uae0d\uc815\uc801\uc73c\ub85c \ubc14\ub77c\ubcf4\uae30 \uc704\ud574 \uace0\uc804 \ub3d9\ud654\uc758 \ub291\ub300 \uc778\uc2dd\uc744 \uace0\uce58\uace0 \uc788\ub2e4. \uc774 \uc608\ub85c \ubbf8\uad6d\uc758 \u3008\ud658\uacbd\uc801 \ub291\ub300\uc640 \uc544\uae30\ub3fc\uc9c0 \uc0bc\ud615\uc81c\u3009(Eco-wolf and the Three Little Pigs)\uc640 \u3008\uc591\uce58\uae30 \uc18c\ub144\u3009(The Wolf who cried Boy) \ub3d9\ud654\uac00 \uc788\ub2e4."} {"question": "\ub525 \uc784\ud329\ud2b8\ub294 \uc790\uc2e0\uc758 \uc704\uce58\uc640 \uada4\ub3c4 \uacb0\uc815\uc744 \uc704\ud574 \ubb34\uc5c7\uc744 \uad00\uce21\ud558\uc600\ub294\uac00?", "paragraph": "6\uc6d4 14\uc77c\uacfc 6\uc6d4 22\uc77c, \ub525 \uc784\ud329\ud2b8\ub294 \ud61c\uc131 \ud45c\uba74\uc5d0\uc11c\uc758 \ud3ed\ubc1c\uc744 \uad00\uce21\ud588\uace0, \uc774\ud6c4(22\uc77c)\uc758 \ud3ed\ubc1c\uc740 \uc57d 6\ubc30 \uc815\ub3c4 \ucef8\ub2e4. \ud0d0\uc0ac\uc120\uc740 \uc790\uc2e0\uc758 \uc704\uce58\uc640 \uada4\ub3c4\ub97c \uacb0\uc815\ud558\uae30 \uc704\ud574 \uba3c \ubcc4\uc744 \uad00\uce21\ud558\uc600\ub2e4. \uc81c\ud2b8 \ucd94\uc9c4 \uc5f0\uad6c\uc18c\uc758 \ub3c8 \uc694\uba3c\uc2a4\ub294 \uc784\ubb34\uc5d0 \ub300\ud55c \uc870\uc0ac \uacb0\uacfc\ub97c \"\uc9c0\uad6c\uc5d0 \uc2e0\ud638\uac00 \ub2ff\ub294 \ub370\ub294 7\u00bd\ubd84 \uc815\ub3c4 \uc18c\uc694\ub429\ub2c8\ub2e4. \uadf8\ub798\uc11c \uc774 \uc77c\uc744 \uc870\uc774\uc2a4\ud2f1\uc744 \uac00\uc9c0\uace0\ub294 \ud560 \uc218 \uc5c6\uc2b5\ub2c8\ub2e4. \uadfc\uc811 \ube44\ud589\uc120\ucc98\ub7fc \ucda9\ub3cc\uae30\ub3c4 \ub611\ub611\ud558\ub2e4\ub294 \uc0ac\uc2e4\uc5d0 \uc758\uc874\ud574\uc57c \ud569\ub2c8\ub2e4. \uadf8\ub798\uc11c \uc0ac\uc804\uc5d0 \uc815\ubcf4\ub97c \ubaa8\uc544\uc11c \uc815\ubcf4 \ubaa8\uc74c\uc9d1\uc744 \uad6c\ucd95\ud55c \ub2e4\uc74c \uadf8 \uc77c\uc744 \uc2dc\uc791\ud574\uc57c \ud569\ub2c8\ub2e4.\"\ub77c\uace0 \uc774\uc57c\uae30\ud588\ub2e4. 6\uc6d4 23\uc77c, \ub450 \ubc88\uc758 \ucd5c\uc885 \uada4\ub3c4 \uc218\uc815 \uc808\ucc28 \uc911 \uccab \ubc88\uc9f8 \uc808\ucc28\uac00 \uc131\uacf5\uc801\uc73c\ub85c \ud589\ud574\uc84c\ub2e4. 6 m/s\uc758 \uc18d\ub3c4 \ubcc0\ud654\uac00 \uc774\ub8e8\uc5b4\uc84c\uace0, \uadfc\uc811 \ube44\ud589\uc120\uc774 \ud61c\uc131\uc758 \ud45c\uba74\uc73c\ub85c\ubd80\ud130 100km \uc704\ub97c \ud1b5\uacfc\ud558\ub3c4\ub85d \uada4\ub3c4\uac00 \ubc14\ub00c\uc5c8\ub2e4.", "answer": "\uba3c \ubcc4", "sentence": "\ud0d0\uc0ac\uc120\uc740 \uc790\uc2e0\uc758 \uc704\uce58\uc640 \uada4\ub3c4\ub97c \uacb0\uc815\ud558\uae30 \uc704\ud574 \uba3c \ubcc4 \uc744 \uad00\uce21\ud558\uc600\ub2e4.", "paragraph_sentence": "6\uc6d4 14\uc77c\uacfc 6\uc6d4 22\uc77c, \ub525 \uc784\ud329\ud2b8\ub294 \ud61c\uc131 \ud45c\uba74\uc5d0\uc11c\uc758 \ud3ed\ubc1c\uc744 \uad00\uce21\ud588\uace0, \uc774\ud6c4(22\uc77c)\uc758 \ud3ed\ubc1c\uc740 \uc57d 6\ubc30 \uc815\ub3c4 \ucef8\ub2e4. \ud0d0\uc0ac\uc120\uc740 \uc790\uc2e0\uc758 \uc704\uce58\uc640 \uada4\ub3c4\ub97c \uacb0\uc815\ud558\uae30 \uc704\ud574 \uba3c \ubcc4 \uc744 \uad00\uce21\ud558\uc600\ub2e4. \uc81c\ud2b8 \ucd94\uc9c4 \uc5f0\uad6c\uc18c\uc758 \ub3c8 \uc694\uba3c\uc2a4\ub294 \uc784\ubb34\uc5d0 \ub300\ud55c \uc870\uc0ac \uacb0\uacfc\ub97c \"\uc9c0\uad6c\uc5d0 \uc2e0\ud638\uac00 \ub2ff\ub294 \ub370\ub294 7\u00bd\ubd84 \uc815\ub3c4 \uc18c\uc694\ub429\ub2c8\ub2e4. \uadf8\ub798\uc11c \uc774 \uc77c\uc744 \uc870\uc774\uc2a4\ud2f1\uc744 \uac00\uc9c0\uace0\ub294 \ud560 \uc218 \uc5c6\uc2b5\ub2c8\ub2e4. \uadfc\uc811 \ube44\ud589\uc120\ucc98\ub7fc \ucda9\ub3cc\uae30\ub3c4 \ub611\ub611\ud558\ub2e4\ub294 \uc0ac\uc2e4\uc5d0 \uc758\uc874\ud574\uc57c \ud569\ub2c8\ub2e4. \uadf8\ub798\uc11c \uc0ac\uc804\uc5d0 \uc815\ubcf4\ub97c \ubaa8\uc544\uc11c \uc815\ubcf4 \ubaa8\uc74c\uc9d1\uc744 \uad6c\ucd95\ud55c \ub2e4\uc74c \uadf8 \uc77c\uc744 \uc2dc\uc791\ud574\uc57c \ud569\ub2c8\ub2e4. \"\ub77c\uace0 \uc774\uc57c\uae30\ud588\ub2e4. 6\uc6d4 23\uc77c, \ub450 \ubc88\uc758 \ucd5c\uc885 \uada4\ub3c4 \uc218\uc815 \uc808\ucc28 \uc911 \uccab \ubc88\uc9f8 \uc808\ucc28\uac00 \uc131\uacf5\uc801\uc73c\ub85c \ud589\ud574\uc84c\ub2e4. 6 m/s\uc758 \uc18d\ub3c4 \ubcc0\ud654\uac00 \uc774\ub8e8\uc5b4\uc84c\uace0, \uadfc\uc811 \ube44\ud589\uc120\uc774 \ud61c\uc131\uc758 \ud45c\uba74\uc73c\ub85c\ubd80\ud130 100km \uc704\ub97c \ud1b5\uacfc\ud558\ub3c4\ub85d \uada4\ub3c4\uac00 \ubc14\ub00c\uc5c8\ub2e4.", "paragraph_answer": "6\uc6d4 14\uc77c\uacfc 6\uc6d4 22\uc77c, \ub525 \uc784\ud329\ud2b8\ub294 \ud61c\uc131 \ud45c\uba74\uc5d0\uc11c\uc758 \ud3ed\ubc1c\uc744 \uad00\uce21\ud588\uace0, \uc774\ud6c4(22\uc77c)\uc758 \ud3ed\ubc1c\uc740 \uc57d 6\ubc30 \uc815\ub3c4 \ucef8\ub2e4. \ud0d0\uc0ac\uc120\uc740 \uc790\uc2e0\uc758 \uc704\uce58\uc640 \uada4\ub3c4\ub97c \uacb0\uc815\ud558\uae30 \uc704\ud574 \uba3c \ubcc4 \uc744 \uad00\uce21\ud558\uc600\ub2e4. \uc81c\ud2b8 \ucd94\uc9c4 \uc5f0\uad6c\uc18c\uc758 \ub3c8 \uc694\uba3c\uc2a4\ub294 \uc784\ubb34\uc5d0 \ub300\ud55c \uc870\uc0ac \uacb0\uacfc\ub97c \"\uc9c0\uad6c\uc5d0 \uc2e0\ud638\uac00 \ub2ff\ub294 \ub370\ub294 7\u00bd\ubd84 \uc815\ub3c4 \uc18c\uc694\ub429\ub2c8\ub2e4. \uadf8\ub798\uc11c \uc774 \uc77c\uc744 \uc870\uc774\uc2a4\ud2f1\uc744 \uac00\uc9c0\uace0\ub294 \ud560 \uc218 \uc5c6\uc2b5\ub2c8\ub2e4. \uadfc\uc811 \ube44\ud589\uc120\ucc98\ub7fc \ucda9\ub3cc\uae30\ub3c4 \ub611\ub611\ud558\ub2e4\ub294 \uc0ac\uc2e4\uc5d0 \uc758\uc874\ud574\uc57c \ud569\ub2c8\ub2e4. \uadf8\ub798\uc11c \uc0ac\uc804\uc5d0 \uc815\ubcf4\ub97c \ubaa8\uc544\uc11c \uc815\ubcf4 \ubaa8\uc74c\uc9d1\uc744 \uad6c\ucd95\ud55c \ub2e4\uc74c \uadf8 \uc77c\uc744 \uc2dc\uc791\ud574\uc57c \ud569\ub2c8\ub2e4.\"\ub77c\uace0 \uc774\uc57c\uae30\ud588\ub2e4. 6\uc6d4 23\uc77c, \ub450 \ubc88\uc758 \ucd5c\uc885 \uada4\ub3c4 \uc218\uc815 \uc808\ucc28 \uc911 \uccab \ubc88\uc9f8 \uc808\ucc28\uac00 \uc131\uacf5\uc801\uc73c\ub85c \ud589\ud574\uc84c\ub2e4. 6 m/s\uc758 \uc18d\ub3c4 \ubcc0\ud654\uac00 \uc774\ub8e8\uc5b4\uc84c\uace0, \uadfc\uc811 \ube44\ud589\uc120\uc774 \ud61c\uc131\uc758 \ud45c\uba74\uc73c\ub85c\ubd80\ud130 100km \uc704\ub97c \ud1b5\uacfc\ud558\ub3c4\ub85d \uada4\ub3c4\uac00 \ubc14\ub00c\uc5c8\ub2e4.", "sentence_answer": "\ud0d0\uc0ac\uc120\uc740 \uc790\uc2e0\uc758 \uc704\uce58\uc640 \uada4\ub3c4\ub97c \uacb0\uc815\ud558\uae30 \uc704\ud574 \uba3c \ubcc4 \uc744 \uad00\uce21\ud558\uc600\ub2e4.", "paragraph_id": "6534065-3-1", "paragraph_question": "question: \ub525 \uc784\ud329\ud2b8\ub294 \uc790\uc2e0\uc758 \uc704\uce58\uc640 \uada4\ub3c4 \uacb0\uc815\uc744 \uc704\ud574 \ubb34\uc5c7\uc744 \uad00\uce21\ud558\uc600\ub294\uac00?, context: 6\uc6d4 14\uc77c\uacfc 6\uc6d4 22\uc77c, \ub525 \uc784\ud329\ud2b8\ub294 \ud61c\uc131 \ud45c\uba74\uc5d0\uc11c\uc758 \ud3ed\ubc1c\uc744 \uad00\uce21\ud588\uace0, \uc774\ud6c4(22\uc77c)\uc758 \ud3ed\ubc1c\uc740 \uc57d 6\ubc30 \uc815\ub3c4 \ucef8\ub2e4. \ud0d0\uc0ac\uc120\uc740 \uc790\uc2e0\uc758 \uc704\uce58\uc640 \uada4\ub3c4\ub97c \uacb0\uc815\ud558\uae30 \uc704\ud574 \uba3c \ubcc4\uc744 \uad00\uce21\ud558\uc600\ub2e4. \uc81c\ud2b8 \ucd94\uc9c4 \uc5f0\uad6c\uc18c\uc758 \ub3c8 \uc694\uba3c\uc2a4\ub294 \uc784\ubb34\uc5d0 \ub300\ud55c \uc870\uc0ac \uacb0\uacfc\ub97c \"\uc9c0\uad6c\uc5d0 \uc2e0\ud638\uac00 \ub2ff\ub294 \ub370\ub294 7\u00bd\ubd84 \uc815\ub3c4 \uc18c\uc694\ub429\ub2c8\ub2e4. \uadf8\ub798\uc11c \uc774 \uc77c\uc744 \uc870\uc774\uc2a4\ud2f1\uc744 \uac00\uc9c0\uace0\ub294 \ud560 \uc218 \uc5c6\uc2b5\ub2c8\ub2e4. \uadfc\uc811 \ube44\ud589\uc120\ucc98\ub7fc \ucda9\ub3cc\uae30\ub3c4 \ub611\ub611\ud558\ub2e4\ub294 \uc0ac\uc2e4\uc5d0 \uc758\uc874\ud574\uc57c \ud569\ub2c8\ub2e4. \uadf8\ub798\uc11c \uc0ac\uc804\uc5d0 \uc815\ubcf4\ub97c \ubaa8\uc544\uc11c \uc815\ubcf4 \ubaa8\uc74c\uc9d1\uc744 \uad6c\ucd95\ud55c \ub2e4\uc74c \uadf8 \uc77c\uc744 \uc2dc\uc791\ud574\uc57c \ud569\ub2c8\ub2e4.\"\ub77c\uace0 \uc774\uc57c\uae30\ud588\ub2e4. 6\uc6d4 23\uc77c, \ub450 \ubc88\uc758 \ucd5c\uc885 \uada4\ub3c4 \uc218\uc815 \uc808\ucc28 \uc911 \uccab \ubc88\uc9f8 \uc808\ucc28\uac00 \uc131\uacf5\uc801\uc73c\ub85c \ud589\ud574\uc84c\ub2e4. 6 m/s\uc758 \uc18d\ub3c4 \ubcc0\ud654\uac00 \uc774\ub8e8\uc5b4\uc84c\uace0, \uadfc\uc811 \ube44\ud589\uc120\uc774 \ud61c\uc131\uc758 \ud45c\uba74\uc73c\ub85c\ubd80\ud130 100km \uc704\ub97c \ud1b5\uacfc\ud558\ub3c4\ub85d \uada4\ub3c4\uac00 \ubc14\ub00c\uc5c8\ub2e4."} {"question": "\uc774\ub514\uc57c\ucee4\ud53c\uac00 \ucc3d\ub9bd 15\uc8fc\ub144 \uae30\ub150\ud589\uc0ac\uc640 \uc774\ub514\uc57c \ubba4\uc9c1\ud398\uc2a4\ud0c0 \ucd08\uccad \uc778\uc6d0 \ud655\ub300 \ub4f1\uc73c\ub85c \ucd94\uac00 \uc9c0\ucd9c\ud55c \uae08\uc561\uc740?", "paragraph": "\uc774\ub514\uc57c\uc758 \uc9c0\ub09c\ud574(2016\ub144) \ub9e4\ucd9c\uc561\uc740 \ubcc4\ub3c4\uc7ac\ubb34\uc81c\ud45c \uae30\uc900 1535\uc5b5\uc6d0\uc73c\ub85c \uc804\ub144\ub3c4\uc758 1355\uc5b5\uc6d0 \ub300\ube44 13.3% \uc99d\uac00\ud558\uba70 \ucc3d\uc0ac \uc774\ub798 \ucd5c\ub300 \uc2e4\uc801\uc744 \uae30\ub85d\ud588\ub2e4. \uc601\uc774158\uc5b5\uc6d0\uc73c\ub85c \uc804\ub144\ub3c4\uc758 163\uc5b5\uc6d0\ubcf4\ub2e4 5\uc5b5\uc6d0 \uac00\ub7c9\uc774 \uc904\uc5c8\ub2e4. \uc774\ub514\uc57c\ub294 \uc2e0\uc81c\ud488 \uac1c\ubc1c\uacfc \ub9e4\ucd9c \uc99d\uac00\ub85c \ub530\ub978 \ubb3c\ub958\ube44, \uad11\uace0\uc120\uc804\ube44 \uc99d\uac00, \ud2b9\ud788 \uc9c0\ub09c\ud574 \ucc3d\ub9bd 15\uc8fc\ub144\uc744 \ub9de\uc544 \uac01\uc885 \uae30\ub150\ud589\uc0ac\uc640 \ub9e4\ub144 \uc2e4\uc2dc\uc911\uc778 \uc774\ub514\uc57c \ubba4\uc9c1 \ud398\uc2a4\ud0c0 \ucd08\uccad \uc778\uc6d0\uc744 \ud655\ub300 \ud558\uace0 \uc720\uba85 \uac00\uc218 \ub4f1\uc744 \uc12d\uc678\ud558\ub290\ub77c \uc57d 15\uc5b5\uc6d0 \uac00\ub7c9\uc744 \ucd94\uac00 \uc9c0\ucd9c\ub4f1\uc758 \ud56d\ubaa9\uc73c\ub85c \ub354 \ub098\uc740 \uc601\uc5c5\uc5ec\uac74 \uad6c\ucd95\uc5d0 \ud22c\uc785\ub41c \ube44\uc6a9\uc774\uae30\ub54c\ubb38\uc5d0 \ubd80\uc815\uc801 \uc2e4\uc801\uc73c\ub85c \ubcfc \uac83\uc740 \uc544\ub2c8\ub77c\ub294 \uac83\uc774 \uc7ac\ubb34\uc804\ubb38\uac00\ub4e4\uc740 \ub9d0\ud55c\ub2e4. \ub354 \ub098\uc740 \uba54\ub274 \uac1c\ubc1c\uc744 \uc704\ud55c \uc9c0\ucd9c \ub4f1\uc774 \uc788\uc5c8\uc74c\uc744 \uac10\uc548\ud55c\ub2e4\uba74 \ub098\uc05c \uc131\uc801\uc740 \uc544\ub2c8\ub77c\ub294 \uc124\uba85\uc778 \uac83\uc774\ub2e4.", "answer": "15\uc5b5\uc6d0", "sentence": "\uc57d 15\uc5b5\uc6d0 \uac00\ub7c9\uc744 \ucd94\uac00 \uc9c0\ucd9c\ub4f1\uc758 \ud56d\ubaa9\uc73c\ub85c \ub354 \ub098\uc740 \uc601\uc5c5\uc5ec\uac74 \uad6c\ucd95\uc5d0 \ud22c\uc785\ub41c \ube44\uc6a9\uc774\uae30\ub54c\ubb38\uc5d0 \ubd80\uc815\uc801 \uc2e4\uc801\uc73c\ub85c \ubcfc \uac83\uc740 \uc544\ub2c8\ub77c\ub294 \uac83\uc774 \uc7ac\ubb34\uc804\ubb38\uac00\ub4e4\uc740 \ub9d0\ud55c\ub2e4.", "paragraph_sentence": "\uc774\ub514\uc57c\uc758 \uc9c0\ub09c\ud574(2016\ub144) \ub9e4\ucd9c\uc561\uc740 \ubcc4\ub3c4\uc7ac\ubb34\uc81c\ud45c \uae30\uc900 1535\uc5b5\uc6d0\uc73c\ub85c \uc804\ub144\ub3c4\uc758 1355\uc5b5\uc6d0 \ub300\ube44 13.3% \uc99d\uac00\ud558\uba70 \ucc3d\uc0ac \uc774\ub798 \ucd5c\ub300 \uc2e4\uc801\uc744 \uae30\ub85d\ud588\ub2e4. \uc601\uc774158\uc5b5\uc6d0\uc73c\ub85c \uc804\ub144\ub3c4\uc758 163\uc5b5\uc6d0\ubcf4\ub2e4 5\uc5b5\uc6d0 \uac00\ub7c9\uc774 \uc904\uc5c8\ub2e4. \uc774\ub514\uc57c\ub294 \uc2e0\uc81c\ud488 \uac1c\ubc1c\uacfc \ub9e4\ucd9c \uc99d\uac00\ub85c \ub530\ub978 \ubb3c\ub958\ube44, \uad11\uace0\uc120\uc804\ube44 \uc99d\uac00, \ud2b9\ud788 \uc9c0\ub09c\ud574 \ucc3d\ub9bd 15\uc8fc\ub144\uc744 \ub9de\uc544 \uac01\uc885 \uae30\ub150\ud589\uc0ac\uc640 \ub9e4\ub144 \uc2e4\uc2dc\uc911\uc778 \uc774\ub514\uc57c \ubba4\uc9c1 \ud398\uc2a4\ud0c0 \ucd08\uccad \uc778\uc6d0\uc744 \ud655\ub300 \ud558\uace0 \uc720\uba85 \uac00\uc218 \ub4f1\uc744 \uc12d\uc678\ud558\ub290\ub77c \uc57d 15\uc5b5\uc6d0 \uac00\ub7c9\uc744 \ucd94\uac00 \uc9c0\ucd9c\ub4f1\uc758 \ud56d\ubaa9\uc73c\ub85c \ub354 \ub098\uc740 \uc601\uc5c5\uc5ec\uac74 \uad6c\ucd95\uc5d0 \ud22c\uc785\ub41c \ube44\uc6a9\uc774\uae30\ub54c\ubb38\uc5d0 \ubd80\uc815\uc801 \uc2e4\uc801\uc73c\ub85c \ubcfc \uac83\uc740 \uc544\ub2c8\ub77c\ub294 \uac83\uc774 \uc7ac\ubb34\uc804\ubb38\uac00\ub4e4\uc740 \ub9d0\ud55c\ub2e4. \ub354 \ub098\uc740 \uba54\ub274 \uac1c\ubc1c\uc744 \uc704\ud55c \uc9c0\ucd9c \ub4f1\uc774 \uc788\uc5c8\uc74c\uc744 \uac10\uc548\ud55c\ub2e4\uba74 \ub098\uc05c \uc131\uc801\uc740 \uc544\ub2c8\ub77c\ub294 \uc124\uba85\uc778 \uac83\uc774\ub2e4.", "paragraph_answer": "\uc774\ub514\uc57c\uc758 \uc9c0\ub09c\ud574(2016\ub144) \ub9e4\ucd9c\uc561\uc740 \ubcc4\ub3c4\uc7ac\ubb34\uc81c\ud45c \uae30\uc900 1535\uc5b5\uc6d0\uc73c\ub85c \uc804\ub144\ub3c4\uc758 1355\uc5b5\uc6d0 \ub300\ube44 13.3% \uc99d\uac00\ud558\uba70 \ucc3d\uc0ac \uc774\ub798 \ucd5c\ub300 \uc2e4\uc801\uc744 \uae30\ub85d\ud588\ub2e4. \uc601\uc774158\uc5b5\uc6d0\uc73c\ub85c \uc804\ub144\ub3c4\uc758 163\uc5b5\uc6d0\ubcf4\ub2e4 5\uc5b5\uc6d0 \uac00\ub7c9\uc774 \uc904\uc5c8\ub2e4. \uc774\ub514\uc57c\ub294 \uc2e0\uc81c\ud488 \uac1c\ubc1c\uacfc \ub9e4\ucd9c \uc99d\uac00\ub85c \ub530\ub978 \ubb3c\ub958\ube44, \uad11\uace0\uc120\uc804\ube44 \uc99d\uac00, \ud2b9\ud788 \uc9c0\ub09c\ud574 \ucc3d\ub9bd 15\uc8fc\ub144\uc744 \ub9de\uc544 \uac01\uc885 \uae30\ub150\ud589\uc0ac\uc640 \ub9e4\ub144 \uc2e4\uc2dc\uc911\uc778 \uc774\ub514\uc57c \ubba4\uc9c1 \ud398\uc2a4\ud0c0 \ucd08\uccad \uc778\uc6d0\uc744 \ud655\ub300 \ud558\uace0 \uc720\uba85 \uac00\uc218 \ub4f1\uc744 \uc12d\uc678\ud558\ub290\ub77c \uc57d 15\uc5b5\uc6d0 \uac00\ub7c9\uc744 \ucd94\uac00 \uc9c0\ucd9c\ub4f1\uc758 \ud56d\ubaa9\uc73c\ub85c \ub354 \ub098\uc740 \uc601\uc5c5\uc5ec\uac74 \uad6c\ucd95\uc5d0 \ud22c\uc785\ub41c \ube44\uc6a9\uc774\uae30\ub54c\ubb38\uc5d0 \ubd80\uc815\uc801 \uc2e4\uc801\uc73c\ub85c \ubcfc \uac83\uc740 \uc544\ub2c8\ub77c\ub294 \uac83\uc774 \uc7ac\ubb34\uc804\ubb38\uac00\ub4e4\uc740 \ub9d0\ud55c\ub2e4. \ub354 \ub098\uc740 \uba54\ub274 \uac1c\ubc1c\uc744 \uc704\ud55c \uc9c0\ucd9c \ub4f1\uc774 \uc788\uc5c8\uc74c\uc744 \uac10\uc548\ud55c\ub2e4\uba74 \ub098\uc05c \uc131\uc801\uc740 \uc544\ub2c8\ub77c\ub294 \uc124\uba85\uc778 \uac83\uc774\ub2e4.", "sentence_answer": "\uc57d 15\uc5b5\uc6d0 \uac00\ub7c9\uc744 \ucd94\uac00 \uc9c0\ucd9c\ub4f1\uc758 \ud56d\ubaa9\uc73c\ub85c \ub354 \ub098\uc740 \uc601\uc5c5\uc5ec\uac74 \uad6c\ucd95\uc5d0 \ud22c\uc785\ub41c \ube44\uc6a9\uc774\uae30\ub54c\ubb38\uc5d0 \ubd80\uc815\uc801 \uc2e4\uc801\uc73c\ub85c \ubcfc \uac83\uc740 \uc544\ub2c8\ub77c\ub294 \uac83\uc774 \uc7ac\ubb34\uc804\ubb38\uac00\ub4e4\uc740 \ub9d0\ud55c\ub2e4.", "paragraph_id": "6544964-4-2", "paragraph_question": "question: \uc774\ub514\uc57c\ucee4\ud53c\uac00 \ucc3d\ub9bd 15\uc8fc\ub144 \uae30\ub150\ud589\uc0ac\uc640 \uc774\ub514\uc57c \ubba4\uc9c1\ud398\uc2a4\ud0c0 \ucd08\uccad \uc778\uc6d0 \ud655\ub300 \ub4f1\uc73c\ub85c \ucd94\uac00 \uc9c0\ucd9c\ud55c \uae08\uc561\uc740?, context: \uc774\ub514\uc57c\uc758 \uc9c0\ub09c\ud574(2016\ub144) \ub9e4\ucd9c\uc561\uc740 \ubcc4\ub3c4\uc7ac\ubb34\uc81c\ud45c \uae30\uc900 1535\uc5b5\uc6d0\uc73c\ub85c \uc804\ub144\ub3c4\uc758 1355\uc5b5\uc6d0 \ub300\ube44 13.3% \uc99d\uac00\ud558\uba70 \ucc3d\uc0ac \uc774\ub798 \ucd5c\ub300 \uc2e4\uc801\uc744 \uae30\ub85d\ud588\ub2e4. \uc601\uc774158\uc5b5\uc6d0\uc73c\ub85c \uc804\ub144\ub3c4\uc758 163\uc5b5\uc6d0\ubcf4\ub2e4 5\uc5b5\uc6d0 \uac00\ub7c9\uc774 \uc904\uc5c8\ub2e4. \uc774\ub514\uc57c\ub294 \uc2e0\uc81c\ud488 \uac1c\ubc1c\uacfc \ub9e4\ucd9c \uc99d\uac00\ub85c \ub530\ub978 \ubb3c\ub958\ube44, \uad11\uace0\uc120\uc804\ube44 \uc99d\uac00, \ud2b9\ud788 \uc9c0\ub09c\ud574 \ucc3d\ub9bd 15\uc8fc\ub144\uc744 \ub9de\uc544 \uac01\uc885 \uae30\ub150\ud589\uc0ac\uc640 \ub9e4\ub144 \uc2e4\uc2dc\uc911\uc778 \uc774\ub514\uc57c \ubba4\uc9c1 \ud398\uc2a4\ud0c0 \ucd08\uccad \uc778\uc6d0\uc744 \ud655\ub300 \ud558\uace0 \uc720\uba85 \uac00\uc218 \ub4f1\uc744 \uc12d\uc678\ud558\ub290\ub77c \uc57d 15\uc5b5\uc6d0 \uac00\ub7c9\uc744 \ucd94\uac00 \uc9c0\ucd9c\ub4f1\uc758 \ud56d\ubaa9\uc73c\ub85c \ub354 \ub098\uc740 \uc601\uc5c5\uc5ec\uac74 \uad6c\ucd95\uc5d0 \ud22c\uc785\ub41c \ube44\uc6a9\uc774\uae30\ub54c\ubb38\uc5d0 \ubd80\uc815\uc801 \uc2e4\uc801\uc73c\ub85c \ubcfc \uac83\uc740 \uc544\ub2c8\ub77c\ub294 \uac83\uc774 \uc7ac\ubb34\uc804\ubb38\uac00\ub4e4\uc740 \ub9d0\ud55c\ub2e4. \ub354 \ub098\uc740 \uba54\ub274 \uac1c\ubc1c\uc744 \uc704\ud55c \uc9c0\ucd9c \ub4f1\uc774 \uc788\uc5c8\uc74c\uc744 \uac10\uc548\ud55c\ub2e4\uba74 \ub098\uc05c \uc131\uc801\uc740 \uc544\ub2c8\ub77c\ub294 \uc124\uba85\uc778 \uac83\uc774\ub2e4."} {"question": "2001\ub144 \uc2dc\uc98c 116\uc2b9\uc744 \uac70\ub454 \uc624\ud074\ub79c\ub4dc\uc640 \uac19\uc740 \uc9c0\uad6c\uc758 \ud300\uc740?", "paragraph": "2001\ub144 \uc2dc\uc98c \uc624\ud074\ub79c\ub4dc \uc560\uc2ac\ub808\ud2f1\uc2a4\ub294 102\uc2b9 60\ud328\ub97c \uac70\ub450\uc5c8\uc73c\ub098, \uac19\uc740 \uc9c0\uad6c\uc758 \uc2dc\uc560\ud2c0 \ub9e4\ub9ac\ub108\uc2a4\uac00 116\uc2b9\uc744 \uac70\ub450\uba74\uc11c \uc640\uc77c\ub4dc\uce74\ub4dc\ub85c \ud3ec\uc2a4\ud2b8\uc2dc\uc98c\uc5d0 \uc9c4\ucd9c\ud588\ub2e4. \uc624\ud074\ub79c\ub4dc\uac00 \uc18d\ud55c \uc9c0\uad6c \uc774\uc678\uc758 \ub2e4\ub978 \ub2e4\uc12f \uc9c0\uad6c \uc6b0\uc2b9\ud300\ub4e4 \uc911 \uc5d0\uc774\uc2a4\ubcf4\ub2e4 \ub9ce\uc740 \uc2b9\uc218\ub97c \uae30\ub85d\ud55c \ud300\uc740 \uc5c6\uc5c8\ub2e4. \uc560\uc2ac\ub808\ud2f1\uc2a4\ub294 \ub514\ube44\uc804 \uc2dc\ub9ac\uc988\uc5d0\uc11c \uc591\ud0a4\uc2a4\uc640 \uc7ac\ub300\uacb0\uc744 \ud588\ub2e4. \ub871\uc740 1\ucc28\uc804\uc5d0\uc11c \ub450 \uac1c\uc758 \ud648\ub7f0\uc744 \ud130\ub728\ub9ac\uba70 \ud300\uc758 5-3 \uc2b9\ub9ac\ub97c \uc774\ub04c\uc5c8\ub2e4. \uc774\uc740 3\ucc28\uc804\uc5d0\uc11c\ub294 \ud300\uc774 1-0\uc73c\ub85c \uc9c0\uace0 \uc788\ub294 \uc0c1\ud669\uc5d0\uc11c, 2\uc544\uc6c3 1\ub8e8 \uc8fc\uc790 \uc81c\ub808\ubbf8 \uc9c0\uc554\ube44\uac00 \ucd9c\ub8e8\ud55c \uac00\uc6b4\ub370 \ud0c0\uc11d\uc5d0 \ub4e4\uc5b4\uc11c \uc678\uc57c \uc6b0\uce21\uc73c\ub85c \ud0c0\uad6c\ub97c \ub0a0\ub824\ubcf4\ub0c8\ub2e4. \ud558\uc9c0\ub9cc \uc774 \ub54c 3\ub8e8\ub97c \ub3cc\uace0 \ud648\uc73c\ub85c \ub4e4\uc5b4\uc624\ub358 \uc9c0\uc554\ube44\uac00 \ub370\ub9ad \uc9c0\ud130\uc758 \uc720\uba85\ud55c '\ub354 \ud50c\ub9bd'(The Flip) \ud50c\ub808\uc774\ub85c \uc544\uc6c3\uc744 \ub2f9\ud588\uace0, \uc5d0\uc774\uc2a4\ub294 \uadf8 \uc810\uc218 \uadf8\ub300\ub85c \ud328\ubc30\ud588\ub2e4. 5\ucc28\uc804 \uc2dc\ub9ac\uc988\uc5d0\uc11c 2\uc5f0\uc2b9 \ub4a4 3\uc5f0\ud328\ub97c \ub2f9\ud558\uba70 \uc5d0\uc774\uc2a4\ub294 2\ub144 \uc5f0\uc18d \ucc54\ud53c\uc5b8\uc2ed \uc2dc\ub9ac\uc988 \uc9c4\ucd9c\uc5d0 \uc2e4\ud328\ud588\ub2e4. \ub871\uc740 \uc2dc\ub9ac\uc988 5\uacbd\uae30, 7\uc548\ud0c0, 2\ud648\ub7f0, 3\ud0c0\uc810, \ud0c0\uc728 .389\ub97c \uae30\ub85d\ud588\ub2e4.", "answer": "\uc2dc\uc560\ud2c0 \ub9e4\ub9ac\ub108\uc2a4", "sentence": "\uac19\uc740 \uc9c0\uad6c\uc758 \uc2dc\uc560\ud2c0 \ub9e4\ub9ac\ub108\uc2a4 \uac00 116\uc2b9\uc744 \uac70\ub450\uba74\uc11c \uc640\uc77c\ub4dc\uce74\ub4dc\ub85c \ud3ec\uc2a4\ud2b8\uc2dc\uc98c\uc5d0 \uc9c4\ucd9c\ud588\ub2e4.", "paragraph_sentence": "2001\ub144 \uc2dc\uc98c \uc624\ud074\ub79c\ub4dc \uc560\uc2ac\ub808\ud2f1\uc2a4\ub294 102\uc2b9 60\ud328\ub97c \uac70\ub450\uc5c8\uc73c\ub098, \uac19\uc740 \uc9c0\uad6c\uc758 \uc2dc\uc560\ud2c0 \ub9e4\ub9ac\ub108\uc2a4 \uac00 116\uc2b9\uc744 \uac70\ub450\uba74\uc11c \uc640\uc77c\ub4dc\uce74\ub4dc\ub85c \ud3ec\uc2a4\ud2b8\uc2dc\uc98c\uc5d0 \uc9c4\ucd9c\ud588\ub2e4. \uc624\ud074\ub79c\ub4dc\uac00 \uc18d\ud55c \uc9c0\uad6c \uc774\uc678\uc758 \ub2e4\ub978 \ub2e4\uc12f \uc9c0\uad6c \uc6b0\uc2b9\ud300\ub4e4 \uc911 \uc5d0\uc774\uc2a4\ubcf4\ub2e4 \ub9ce\uc740 \uc2b9\uc218\ub97c \uae30\ub85d\ud55c \ud300\uc740 \uc5c6\uc5c8\ub2e4. \uc560\uc2ac\ub808\ud2f1\uc2a4\ub294 \ub514\ube44\uc804 \uc2dc\ub9ac\uc988\uc5d0\uc11c \uc591\ud0a4\uc2a4\uc640 \uc7ac\ub300\uacb0\uc744 \ud588\ub2e4. \ub871\uc740 1\ucc28\uc804\uc5d0\uc11c \ub450 \uac1c\uc758 \ud648\ub7f0\uc744 \ud130\ub728\ub9ac\uba70 \ud300\uc758 5-3 \uc2b9\ub9ac\ub97c \uc774\ub04c\uc5c8\ub2e4. \uc774\uc740 3\ucc28\uc804\uc5d0\uc11c\ub294 \ud300\uc774 1-0\uc73c\ub85c \uc9c0\uace0 \uc788\ub294 \uc0c1\ud669\uc5d0\uc11c, 2\uc544\uc6c3 1\ub8e8 \uc8fc\uc790 \uc81c\ub808\ubbf8 \uc9c0\uc554\ube44\uac00 \ucd9c\ub8e8\ud55c \uac00\uc6b4\ub370 \ud0c0\uc11d\uc5d0 \ub4e4\uc5b4\uc11c \uc678\uc57c \uc6b0\uce21\uc73c\ub85c \ud0c0\uad6c\ub97c \ub0a0\ub824\ubcf4\ub0c8\ub2e4. \ud558\uc9c0\ub9cc \uc774 \ub54c 3\ub8e8\ub97c \ub3cc\uace0 \ud648\uc73c\ub85c \ub4e4\uc5b4\uc624\ub358 \uc9c0\uc554\ube44\uac00 \ub370\ub9ad \uc9c0\ud130\uc758 \uc720\uba85\ud55c '\ub354 \ud50c\ub9bd'(The Flip) \ud50c\ub808\uc774\ub85c \uc544\uc6c3\uc744 \ub2f9\ud588\uace0, \uc5d0\uc774\uc2a4\ub294 \uadf8 \uc810\uc218 \uadf8\ub300\ub85c \ud328\ubc30\ud588\ub2e4. 5\ucc28\uc804 \uc2dc\ub9ac\uc988\uc5d0\uc11c 2\uc5f0\uc2b9 \ub4a4 3\uc5f0\ud328\ub97c \ub2f9\ud558\uba70 \uc5d0\uc774\uc2a4\ub294 2\ub144 \uc5f0\uc18d \ucc54\ud53c\uc5b8\uc2ed \uc2dc\ub9ac\uc988 \uc9c4\ucd9c\uc5d0 \uc2e4\ud328\ud588\ub2e4. \ub871\uc740 \uc2dc\ub9ac\uc988 5\uacbd\uae30, 7\uc548\ud0c0, 2\ud648\ub7f0, 3\ud0c0\uc810, \ud0c0\uc728 .389\ub97c \uae30\ub85d\ud588\ub2e4.", "paragraph_answer": "2001\ub144 \uc2dc\uc98c \uc624\ud074\ub79c\ub4dc \uc560\uc2ac\ub808\ud2f1\uc2a4\ub294 102\uc2b9 60\ud328\ub97c \uac70\ub450\uc5c8\uc73c\ub098, \uac19\uc740 \uc9c0\uad6c\uc758 \uc2dc\uc560\ud2c0 \ub9e4\ub9ac\ub108\uc2a4 \uac00 116\uc2b9\uc744 \uac70\ub450\uba74\uc11c \uc640\uc77c\ub4dc\uce74\ub4dc\ub85c \ud3ec\uc2a4\ud2b8\uc2dc\uc98c\uc5d0 \uc9c4\ucd9c\ud588\ub2e4. \uc624\ud074\ub79c\ub4dc\uac00 \uc18d\ud55c \uc9c0\uad6c \uc774\uc678\uc758 \ub2e4\ub978 \ub2e4\uc12f \uc9c0\uad6c \uc6b0\uc2b9\ud300\ub4e4 \uc911 \uc5d0\uc774\uc2a4\ubcf4\ub2e4 \ub9ce\uc740 \uc2b9\uc218\ub97c \uae30\ub85d\ud55c \ud300\uc740 \uc5c6\uc5c8\ub2e4. \uc560\uc2ac\ub808\ud2f1\uc2a4\ub294 \ub514\ube44\uc804 \uc2dc\ub9ac\uc988\uc5d0\uc11c \uc591\ud0a4\uc2a4\uc640 \uc7ac\ub300\uacb0\uc744 \ud588\ub2e4. \ub871\uc740 1\ucc28\uc804\uc5d0\uc11c \ub450 \uac1c\uc758 \ud648\ub7f0\uc744 \ud130\ub728\ub9ac\uba70 \ud300\uc758 5-3 \uc2b9\ub9ac\ub97c \uc774\ub04c\uc5c8\ub2e4. \uc774\uc740 3\ucc28\uc804\uc5d0\uc11c\ub294 \ud300\uc774 1-0\uc73c\ub85c \uc9c0\uace0 \uc788\ub294 \uc0c1\ud669\uc5d0\uc11c, 2\uc544\uc6c3 1\ub8e8 \uc8fc\uc790 \uc81c\ub808\ubbf8 \uc9c0\uc554\ube44\uac00 \ucd9c\ub8e8\ud55c \uac00\uc6b4\ub370 \ud0c0\uc11d\uc5d0 \ub4e4\uc5b4\uc11c \uc678\uc57c \uc6b0\uce21\uc73c\ub85c \ud0c0\uad6c\ub97c \ub0a0\ub824\ubcf4\ub0c8\ub2e4. \ud558\uc9c0\ub9cc \uc774 \ub54c 3\ub8e8\ub97c \ub3cc\uace0 \ud648\uc73c\ub85c \ub4e4\uc5b4\uc624\ub358 \uc9c0\uc554\ube44\uac00 \ub370\ub9ad \uc9c0\ud130\uc758 \uc720\uba85\ud55c '\ub354 \ud50c\ub9bd'(The Flip) \ud50c\ub808\uc774\ub85c \uc544\uc6c3\uc744 \ub2f9\ud588\uace0, \uc5d0\uc774\uc2a4\ub294 \uadf8 \uc810\uc218 \uadf8\ub300\ub85c \ud328\ubc30\ud588\ub2e4. 5\ucc28\uc804 \uc2dc\ub9ac\uc988\uc5d0\uc11c 2\uc5f0\uc2b9 \ub4a4 3\uc5f0\ud328\ub97c \ub2f9\ud558\uba70 \uc5d0\uc774\uc2a4\ub294 2\ub144 \uc5f0\uc18d \ucc54\ud53c\uc5b8\uc2ed \uc2dc\ub9ac\uc988 \uc9c4\ucd9c\uc5d0 \uc2e4\ud328\ud588\ub2e4. \ub871\uc740 \uc2dc\ub9ac\uc988 5\uacbd\uae30, 7\uc548\ud0c0, 2\ud648\ub7f0, 3\ud0c0\uc810, \ud0c0\uc728 .389\ub97c \uae30\ub85d\ud588\ub2e4.", "sentence_answer": "\uac19\uc740 \uc9c0\uad6c\uc758 \uc2dc\uc560\ud2c0 \ub9e4\ub9ac\ub108\uc2a4 \uac00 116\uc2b9\uc744 \uac70\ub450\uba74\uc11c \uc640\uc77c\ub4dc\uce74\ub4dc\ub85c \ud3ec\uc2a4\ud2b8\uc2dc\uc98c\uc5d0 \uc9c4\ucd9c\ud588\ub2e4.", "paragraph_id": "6466818-8-1", "paragraph_question": "question: 2001\ub144 \uc2dc\uc98c 116\uc2b9\uc744 \uac70\ub454 \uc624\ud074\ub79c\ub4dc\uc640 \uac19\uc740 \uc9c0\uad6c\uc758 \ud300\uc740?, context: 2001\ub144 \uc2dc\uc98c \uc624\ud074\ub79c\ub4dc \uc560\uc2ac\ub808\ud2f1\uc2a4\ub294 102\uc2b9 60\ud328\ub97c \uac70\ub450\uc5c8\uc73c\ub098, \uac19\uc740 \uc9c0\uad6c\uc758 \uc2dc\uc560\ud2c0 \ub9e4\ub9ac\ub108\uc2a4\uac00 116\uc2b9\uc744 \uac70\ub450\uba74\uc11c \uc640\uc77c\ub4dc\uce74\ub4dc\ub85c \ud3ec\uc2a4\ud2b8\uc2dc\uc98c\uc5d0 \uc9c4\ucd9c\ud588\ub2e4. \uc624\ud074\ub79c\ub4dc\uac00 \uc18d\ud55c \uc9c0\uad6c \uc774\uc678\uc758 \ub2e4\ub978 \ub2e4\uc12f \uc9c0\uad6c \uc6b0\uc2b9\ud300\ub4e4 \uc911 \uc5d0\uc774\uc2a4\ubcf4\ub2e4 \ub9ce\uc740 \uc2b9\uc218\ub97c \uae30\ub85d\ud55c \ud300\uc740 \uc5c6\uc5c8\ub2e4. \uc560\uc2ac\ub808\ud2f1\uc2a4\ub294 \ub514\ube44\uc804 \uc2dc\ub9ac\uc988\uc5d0\uc11c \uc591\ud0a4\uc2a4\uc640 \uc7ac\ub300\uacb0\uc744 \ud588\ub2e4. \ub871\uc740 1\ucc28\uc804\uc5d0\uc11c \ub450 \uac1c\uc758 \ud648\ub7f0\uc744 \ud130\ub728\ub9ac\uba70 \ud300\uc758 5-3 \uc2b9\ub9ac\ub97c \uc774\ub04c\uc5c8\ub2e4. \uc774\uc740 3\ucc28\uc804\uc5d0\uc11c\ub294 \ud300\uc774 1-0\uc73c\ub85c \uc9c0\uace0 \uc788\ub294 \uc0c1\ud669\uc5d0\uc11c, 2\uc544\uc6c3 1\ub8e8 \uc8fc\uc790 \uc81c\ub808\ubbf8 \uc9c0\uc554\ube44\uac00 \ucd9c\ub8e8\ud55c \uac00\uc6b4\ub370 \ud0c0\uc11d\uc5d0 \ub4e4\uc5b4\uc11c \uc678\uc57c \uc6b0\uce21\uc73c\ub85c \ud0c0\uad6c\ub97c \ub0a0\ub824\ubcf4\ub0c8\ub2e4. \ud558\uc9c0\ub9cc \uc774 \ub54c 3\ub8e8\ub97c \ub3cc\uace0 \ud648\uc73c\ub85c \ub4e4\uc5b4\uc624\ub358 \uc9c0\uc554\ube44\uac00 \ub370\ub9ad \uc9c0\ud130\uc758 \uc720\uba85\ud55c '\ub354 \ud50c\ub9bd'(The Flip) \ud50c\ub808\uc774\ub85c \uc544\uc6c3\uc744 \ub2f9\ud588\uace0, \uc5d0\uc774\uc2a4\ub294 \uadf8 \uc810\uc218 \uadf8\ub300\ub85c \ud328\ubc30\ud588\ub2e4. 5\ucc28\uc804 \uc2dc\ub9ac\uc988\uc5d0\uc11c 2\uc5f0\uc2b9 \ub4a4 3\uc5f0\ud328\ub97c \ub2f9\ud558\uba70 \uc5d0\uc774\uc2a4\ub294 2\ub144 \uc5f0\uc18d \ucc54\ud53c\uc5b8\uc2ed \uc2dc\ub9ac\uc988 \uc9c4\ucd9c\uc5d0 \uc2e4\ud328\ud588\ub2e4. \ub871\uc740 \uc2dc\ub9ac\uc988 5\uacbd\uae30, 7\uc548\ud0c0, 2\ud648\ub7f0, 3\ud0c0\uc810, \ud0c0\uc728 .389\ub97c \uae30\ub85d\ud588\ub2e4."} {"question": "\ubaa8\ub4e0 DVD \uc2dc\ub9ac\uc988\ub97c \ub2e4\uc6b4\ub85c\ub4dc\ud560 \uc218 \uc788\ub294 \uc0ac\uc774\ud2b8\ub294 \uc5b4\ub514\uc778\uac00?", "paragraph": "\uc6cc\ub108 \ud648 \ube44\ub514\uc624\uac00 \uc9c0\uc5ed \ucf54\ub4dc 1, \uc9c0\uc5ed \ucf54\ub4dc 2 \ud615\uc2dd\uc758 \uc5ec\ub7ec DVD \uc2dc\ub9ac\uc988\ub97c \ubc1c\ub9e4\ud558\uc600\ub2e4. \u300aMy Two Favorite People\u300b, \u300aIt Came from the Nightosphere\u300b, \u300aJake vs. Me-Mow\u300b, \u300aFionna and Cake\u300b, \u300aThe Suitor\u300b, \u300aPrincess Day\u300b, \u300aThe Enchiridion\u300b\uc774 \ub450 \ubc88\uc9f8 \uc2dc\uc98c\uc758 \uc77c\ubd80 \uc5d0\ud53c\uc18c\ub4dc\ub97c \ud3ec\ud568\ud558\uc5ec \ubc1c\ub9e4\ub418\uc5c8\ub2e4. 2013\ub144 6\uc6d4 4\uc77c\uc5d0\ub294 \ub450 \ubc88\uc9f8 \uc2dc\uc98c\uc758 \ubaa8\ub4e0 \uc5d0\ud53c\uc18c\ub4dc\ub97c \ub2f4\uc740 DVD\uc640 \ube14\ub8e8\ub808\uc774 \ub514\uc2a4\ud06c\uac00 \ubc1c\ub9e4\ub418\uc5c8\ub2e4. \ubaa8\ub4e0 DVD \uc2dc\ub9ac\uc988\ub294 \uce74\ud230 \ub124\ud2b8\uc6cc\ud06c \uc20d\uc5d0\uc11c \uad6c\uc785\uc774 \uac00\ub2a5\ud558\uace0, \uac01 \uc5d0\ud53c\uc18c\ub4dc\ub4e4\uc740 \uc544\uc774\ud2a0\uc2a4 \uc2a4\ud1a0\uc5b4\uc640 \uc544\ub9c8\uc874\ub2f7\ucef4\uc5d0\uc11c \ub2e4\uc6b4\ub85c\ub4dc\uac00 \uac00\ub2a5\ud558\ub2e4. 2014\ub144 3\uc6d4 30\uc5d0\ub294 \ub137\ud50c\ub9ad\uc2a4\uc5d0\uc11c \ub450 \ubc88\uc9f8 \uc2dc\uc98c\uc758 \uc628\ub77c\uc778 \uc2a4\ud2b8\ub9ac\ubc0d \uc11c\ube44\uc2a4\ub97c \uc2dc\uc791\ud558\uc600\uc73c\ub098 2015\ub144 3\uc6d4 30\uc77c\uc790\ub85c \uc11c\ube44\uc2a4\ub97c \uc885\ub8cc\ud558\uc600\ub2e4. \ub300\uc2e0 \ub3d9\ub144 5\uc6d4 1\uc77c\ubd80\ud130 \uc2dc\uc98c 1~6 \uc804 \uc5d0\ud53c\uc18c\ub4dc\uac00 \ud6cc\ub8e8\uc5d0\uc11c \uc81c\uacf5\ub418\uace0 \uc788\ub2e4.", "answer": "\uc544\ub9c8\uc874\ub2f7\ucef4", "sentence": "\ubaa8\ub4e0 DVD \uc2dc\ub9ac\uc988\ub294 \uce74\ud230 \ub124\ud2b8\uc6cc\ud06c \uc20d\uc5d0\uc11c \uad6c\uc785\uc774 \uac00\ub2a5\ud558\uace0, \uac01 \uc5d0\ud53c\uc18c\ub4dc\ub4e4\uc740 \uc544\uc774\ud2a0\uc2a4 \uc2a4\ud1a0\uc5b4\uc640 \uc544\ub9c8\uc874\ub2f7\ucef4 \uc5d0\uc11c \ub2e4\uc6b4\ub85c\ub4dc\uac00 \uac00\ub2a5\ud558\ub2e4.", "paragraph_sentence": "\uc6cc\ub108 \ud648 \ube44\ub514\uc624\uac00 \uc9c0\uc5ed \ucf54\ub4dc 1, \uc9c0\uc5ed \ucf54\ub4dc 2 \ud615\uc2dd\uc758 \uc5ec\ub7ec DVD \uc2dc\ub9ac\uc988\ub97c \ubc1c\ub9e4\ud558\uc600\ub2e4. \u300aMy Two Favorite People\u300b, \u300aIt Came from the Nightosphere\u300b, \u300aJake vs. Me-Mow\u300b, \u300aFionna and Cake\u300b, \u300aThe Suitor\u300b, \u300aPrincess Day\u300b, \u300aThe Enchiridion\u300b\uc774 \ub450 \ubc88\uc9f8 \uc2dc\uc98c\uc758 \uc77c\ubd80 \uc5d0\ud53c\uc18c\ub4dc\ub97c \ud3ec\ud568\ud558\uc5ec \ubc1c\ub9e4\ub418\uc5c8\ub2e4. 2013\ub144 6\uc6d4 4\uc77c\uc5d0\ub294 \ub450 \ubc88\uc9f8 \uc2dc\uc98c\uc758 \ubaa8\ub4e0 \uc5d0\ud53c\uc18c\ub4dc\ub97c \ub2f4\uc740 DVD\uc640 \ube14\ub8e8\ub808\uc774 \ub514\uc2a4\ud06c\uac00 \ubc1c\ub9e4\ub418\uc5c8\ub2e4. \ubaa8\ub4e0 DVD \uc2dc\ub9ac\uc988\ub294 \uce74\ud230 \ub124\ud2b8\uc6cc\ud06c \uc20d\uc5d0\uc11c \uad6c\uc785\uc774 \uac00\ub2a5\ud558\uace0, \uac01 \uc5d0\ud53c\uc18c\ub4dc\ub4e4\uc740 \uc544\uc774\ud2a0\uc2a4 \uc2a4\ud1a0\uc5b4\uc640 \uc544\ub9c8\uc874\ub2f7\ucef4 \uc5d0\uc11c \ub2e4\uc6b4\ub85c\ub4dc\uac00 \uac00\ub2a5\ud558\ub2e4. 2014\ub144 3\uc6d4 30\uc5d0\ub294 \ub137\ud50c\ub9ad\uc2a4\uc5d0\uc11c \ub450 \ubc88\uc9f8 \uc2dc\uc98c\uc758 \uc628\ub77c\uc778 \uc2a4\ud2b8\ub9ac\ubc0d \uc11c\ube44\uc2a4\ub97c \uc2dc\uc791\ud558\uc600\uc73c\ub098 2015\ub144 3\uc6d4 30\uc77c\uc790\ub85c \uc11c\ube44\uc2a4\ub97c \uc885\ub8cc\ud558\uc600\ub2e4. \ub300\uc2e0 \ub3d9\ub144 5\uc6d4 1\uc77c\ubd80\ud130 \uc2dc\uc98c 1~6 \uc804 \uc5d0\ud53c\uc18c\ub4dc\uac00 \ud6cc\ub8e8\uc5d0\uc11c \uc81c\uacf5\ub418\uace0 \uc788\ub2e4.", "paragraph_answer": "\uc6cc\ub108 \ud648 \ube44\ub514\uc624\uac00 \uc9c0\uc5ed \ucf54\ub4dc 1, \uc9c0\uc5ed \ucf54\ub4dc 2 \ud615\uc2dd\uc758 \uc5ec\ub7ec DVD \uc2dc\ub9ac\uc988\ub97c \ubc1c\ub9e4\ud558\uc600\ub2e4. \u300aMy Two Favorite People\u300b, \u300aIt Came from the Nightosphere\u300b, \u300aJake vs. Me-Mow\u300b, \u300aFionna and Cake\u300b, \u300aThe Suitor\u300b, \u300aPrincess Day\u300b, \u300aThe Enchiridion\u300b\uc774 \ub450 \ubc88\uc9f8 \uc2dc\uc98c\uc758 \uc77c\ubd80 \uc5d0\ud53c\uc18c\ub4dc\ub97c \ud3ec\ud568\ud558\uc5ec \ubc1c\ub9e4\ub418\uc5c8\ub2e4. 2013\ub144 6\uc6d4 4\uc77c\uc5d0\ub294 \ub450 \ubc88\uc9f8 \uc2dc\uc98c\uc758 \ubaa8\ub4e0 \uc5d0\ud53c\uc18c\ub4dc\ub97c \ub2f4\uc740 DVD\uc640 \ube14\ub8e8\ub808\uc774 \ub514\uc2a4\ud06c\uac00 \ubc1c\ub9e4\ub418\uc5c8\ub2e4. \ubaa8\ub4e0 DVD \uc2dc\ub9ac\uc988\ub294 \uce74\ud230 \ub124\ud2b8\uc6cc\ud06c \uc20d\uc5d0\uc11c \uad6c\uc785\uc774 \uac00\ub2a5\ud558\uace0, \uac01 \uc5d0\ud53c\uc18c\ub4dc\ub4e4\uc740 \uc544\uc774\ud2a0\uc2a4 \uc2a4\ud1a0\uc5b4\uc640 \uc544\ub9c8\uc874\ub2f7\ucef4 \uc5d0\uc11c \ub2e4\uc6b4\ub85c\ub4dc\uac00 \uac00\ub2a5\ud558\ub2e4. 2014\ub144 3\uc6d4 30\uc5d0\ub294 \ub137\ud50c\ub9ad\uc2a4\uc5d0\uc11c \ub450 \ubc88\uc9f8 \uc2dc\uc98c\uc758 \uc628\ub77c\uc778 \uc2a4\ud2b8\ub9ac\ubc0d \uc11c\ube44\uc2a4\ub97c \uc2dc\uc791\ud558\uc600\uc73c\ub098 2015\ub144 3\uc6d4 30\uc77c\uc790\ub85c \uc11c\ube44\uc2a4\ub97c \uc885\ub8cc\ud558\uc600\ub2e4. \ub300\uc2e0 \ub3d9\ub144 5\uc6d4 1\uc77c\ubd80\ud130 \uc2dc\uc98c 1~6 \uc804 \uc5d0\ud53c\uc18c\ub4dc\uac00 \ud6cc\ub8e8\uc5d0\uc11c \uc81c\uacf5\ub418\uace0 \uc788\ub2e4.", "sentence_answer": "\ubaa8\ub4e0 DVD \uc2dc\ub9ac\uc988\ub294 \uce74\ud230 \ub124\ud2b8\uc6cc\ud06c \uc20d\uc5d0\uc11c \uad6c\uc785\uc774 \uac00\ub2a5\ud558\uace0, \uac01 \uc5d0\ud53c\uc18c\ub4dc\ub4e4\uc740 \uc544\uc774\ud2a0\uc2a4 \uc2a4\ud1a0\uc5b4\uc640 \uc544\ub9c8\uc874\ub2f7\ucef4 \uc5d0\uc11c \ub2e4\uc6b4\ub85c\ub4dc\uac00 \uac00\ub2a5\ud558\ub2e4.", "paragraph_id": "6358308-9-1", "paragraph_question": "question: \ubaa8\ub4e0 DVD \uc2dc\ub9ac\uc988\ub97c \ub2e4\uc6b4\ub85c\ub4dc\ud560 \uc218 \uc788\ub294 \uc0ac\uc774\ud2b8\ub294 \uc5b4\ub514\uc778\uac00?, context: \uc6cc\ub108 \ud648 \ube44\ub514\uc624\uac00 \uc9c0\uc5ed \ucf54\ub4dc 1, \uc9c0\uc5ed \ucf54\ub4dc 2 \ud615\uc2dd\uc758 \uc5ec\ub7ec DVD \uc2dc\ub9ac\uc988\ub97c \ubc1c\ub9e4\ud558\uc600\ub2e4. \u300aMy Two Favorite People\u300b, \u300aIt Came from the Nightosphere\u300b, \u300aJake vs. Me-Mow\u300b, \u300aFionna and Cake\u300b, \u300aThe Suitor\u300b, \u300aPrincess Day\u300b, \u300aThe Enchiridion\u300b\uc774 \ub450 \ubc88\uc9f8 \uc2dc\uc98c\uc758 \uc77c\ubd80 \uc5d0\ud53c\uc18c\ub4dc\ub97c \ud3ec\ud568\ud558\uc5ec \ubc1c\ub9e4\ub418\uc5c8\ub2e4. 2013\ub144 6\uc6d4 4\uc77c\uc5d0\ub294 \ub450 \ubc88\uc9f8 \uc2dc\uc98c\uc758 \ubaa8\ub4e0 \uc5d0\ud53c\uc18c\ub4dc\ub97c \ub2f4\uc740 DVD\uc640 \ube14\ub8e8\ub808\uc774 \ub514\uc2a4\ud06c\uac00 \ubc1c\ub9e4\ub418\uc5c8\ub2e4. \ubaa8\ub4e0 DVD \uc2dc\ub9ac\uc988\ub294 \uce74\ud230 \ub124\ud2b8\uc6cc\ud06c \uc20d\uc5d0\uc11c \uad6c\uc785\uc774 \uac00\ub2a5\ud558\uace0, \uac01 \uc5d0\ud53c\uc18c\ub4dc\ub4e4\uc740 \uc544\uc774\ud2a0\uc2a4 \uc2a4\ud1a0\uc5b4\uc640 \uc544\ub9c8\uc874\ub2f7\ucef4\uc5d0\uc11c \ub2e4\uc6b4\ub85c\ub4dc\uac00 \uac00\ub2a5\ud558\ub2e4. 2014\ub144 3\uc6d4 30\uc5d0\ub294 \ub137\ud50c\ub9ad\uc2a4\uc5d0\uc11c \ub450 \ubc88\uc9f8 \uc2dc\uc98c\uc758 \uc628\ub77c\uc778 \uc2a4\ud2b8\ub9ac\ubc0d \uc11c\ube44\uc2a4\ub97c \uc2dc\uc791\ud558\uc600\uc73c\ub098 2015\ub144 3\uc6d4 30\uc77c\uc790\ub85c \uc11c\ube44\uc2a4\ub97c \uc885\ub8cc\ud558\uc600\ub2e4. \ub300\uc2e0 \ub3d9\ub144 5\uc6d4 1\uc77c\ubd80\ud130 \uc2dc\uc98c 1~6 \uc804 \uc5d0\ud53c\uc18c\ub4dc\uac00 \ud6cc\ub8e8\uc5d0\uc11c \uc81c\uacf5\ub418\uace0 \uc788\ub2e4."} {"question": "1962\ub144 6\uc6d4\uc5d0 \uad6d\uc81c \uc6d0\uc790\ub825\uae30\uad6c\uac00 \uc774\ub860\ubb3c\ub9ac\ud559 \uc138\ubbf8\ub098\ub97c \uac1c\ucd5c\ud55c \ub098\ub77c\ub294?", "paragraph": "1962\ub144 6\uc6d4 \ucd08, \uc774\ud718\uc18c\ub294 \uad6d\uc81c \uc6d0\uc790\ub825 \uae30\uad6c\uac00 \uc8fc\ucd5c\ud558\ub294 \uc774\ud0c8\ub9ac\uc544 \ud2b8\ub9ac\uc5d0\uc2a4\ud14c \uc774\ub860\ubb3c\ub9ac\ud559 \uc138\ubbf8\ub098(Seminar on Theoretical Physics)\uc5d0 \ucc38\uc11d\ud560 10\uc778\uc758 \ubbf8\uad6d \ub300\ud45c\ub2e8 \uc77c\uc6d0\uc73c\ub85c \uc120\ucd9c\ub410\ub2e4. \ud2b8\ub9ac\uc5d0\uc2a4\ud14c \uc774\ub860\ubb3c\ub9ac\ud559 \uc138\ubbf8\ub098\ub294 1962\ub144 7\uc6d4 16\uc77c\ubd80\ud130 8\uc6d4 25\uc77c\uae4c\uc9c0 \uac1c\ucd5c\ub410\ub2e4. \uc774 \ubb34\ub835\uc758 \uc774\ud718\uc18c\ub294 \uc80a\uc740 \uc5f0\uad6c\uc790\ub85c\uc11c \ubbf8\uad6d \ub0b4\uc5d0\uc11c \ubb34\uc2dc\ubabb\ud560 \uba85\uc131\uacfc \ub6f0\uc5b4\ub09c \uc7ac\ub2a5\uc744 \uac00\uc9c0\uace0 \uc788\ub358 \uac83\uc740 \uc0ac\uc2e4\uc774\uc9c0\ub9cc, \uadf8 \ub2e8\uacc4\uc5d0\uc11c\ub294 \uc544\uc9c1\uc740 \uc5b4\ub5a4 \ud070 \uc131\uacfc\ub97c \ub0b4\uc9c0\ub294 \uc54a\uc558\ub2e4. \uadf8\uc758 \uc911\uc694\ud55c \ud559\ubb38\uc801 \uc131\uacfc\ub294 \ubaa8\ub450 1970\ub144\ub300\uc5d0 \uc774\ub974\ub7ec\uc11c\uc57c \ub098\ud0c0\ub098\uae30 \uc2dc\uc791\ud55c\ub2e4. \uadf8\ub7ec\ubbc0\ub85c \u300860\ub144\uc5d0 \uc911\ubc18\uc5d0 \uc774\ubbf8 \ub178\ubca8\uc0c1\uc744 \uc8fc\uc5b4\uc57c \ud588\ub2e4\u3009\uac70\ub098 \u3008\ub0b4 \ubc11\uc5d0 \uc544\uc778\uc288\ud0c0\uc778\ub3c4 \uc788\uc5c8\uc9c0\ub9cc \uc774\ud718\uc18c\uac00 \ub354 \ub6f0\uc5b4\ub0ac\ub2e4\u3009\ub294 \ub4f1\uc758 \uba87\uba87 \uc18c\uc124\ub4e4\uc5d0\uc11c\uc758 \ubb18\uc0ac\ub294 \uc9c0\ub098\uce58\uac8c \uacfc\uc7a5\ub41c \uac83\uc73c\ub85c \uc0ac\uc2e4\uacfc \ub2e4\ub974\ub2e4.", "answer": "\uc774\ud0c8\ub9ac\uc544", "sentence": "1962\ub144 6\uc6d4 \ucd08, \uc774\ud718\uc18c\ub294 \uad6d\uc81c \uc6d0\uc790\ub825 \uae30\uad6c\uac00 \uc8fc\ucd5c\ud558\ub294 \uc774\ud0c8\ub9ac\uc544 \ud2b8\ub9ac\uc5d0\uc2a4\ud14c \uc774\ub860\ubb3c\ub9ac\ud559 \uc138\ubbf8\ub098(Seminar on Theoretical Physics)\uc5d0 \ucc38\uc11d\ud560 10\uc778\uc758 \ubbf8\uad6d \ub300\ud45c\ub2e8 \uc77c\uc6d0\uc73c\ub85c \uc120\ucd9c\ub410\ub2e4.", "paragraph_sentence": " 1962\ub144 6\uc6d4 \ucd08, \uc774\ud718\uc18c\ub294 \uad6d\uc81c \uc6d0\uc790\ub825 \uae30\uad6c\uac00 \uc8fc\ucd5c\ud558\ub294 \uc774\ud0c8\ub9ac\uc544 \ud2b8\ub9ac\uc5d0\uc2a4\ud14c \uc774\ub860\ubb3c\ub9ac\ud559 \uc138\ubbf8\ub098(Seminar on Theoretical Physics)\uc5d0 \ucc38\uc11d\ud560 10\uc778\uc758 \ubbf8\uad6d \ub300\ud45c\ub2e8 \uc77c\uc6d0\uc73c\ub85c \uc120\ucd9c\ub410\ub2e4. \ud2b8\ub9ac\uc5d0\uc2a4\ud14c \uc774\ub860\ubb3c\ub9ac\ud559 \uc138\ubbf8\ub098\ub294 1962\ub144 7\uc6d4 16\uc77c\ubd80\ud130 8\uc6d4 25\uc77c\uae4c\uc9c0 \uac1c\ucd5c\ub410\ub2e4. \uc774 \ubb34\ub835\uc758 \uc774\ud718\uc18c\ub294 \uc80a\uc740 \uc5f0\uad6c\uc790\ub85c\uc11c \ubbf8\uad6d \ub0b4\uc5d0\uc11c \ubb34\uc2dc\ubabb\ud560 \uba85\uc131\uacfc \ub6f0\uc5b4\ub09c \uc7ac\ub2a5\uc744 \uac00\uc9c0\uace0 \uc788\ub358 \uac83\uc740 \uc0ac\uc2e4\uc774\uc9c0\ub9cc, \uadf8 \ub2e8\uacc4\uc5d0\uc11c\ub294 \uc544\uc9c1\uc740 \uc5b4\ub5a4 \ud070 \uc131\uacfc\ub97c \ub0b4\uc9c0\ub294 \uc54a\uc558\ub2e4. \uadf8\uc758 \uc911\uc694\ud55c \ud559\ubb38\uc801 \uc131\uacfc\ub294 \ubaa8\ub450 1970\ub144\ub300\uc5d0 \uc774\ub974\ub7ec\uc11c\uc57c \ub098\ud0c0\ub098\uae30 \uc2dc\uc791\ud55c\ub2e4. \uadf8\ub7ec\ubbc0\ub85c \u300860\ub144\uc5d0 \uc911\ubc18\uc5d0 \uc774\ubbf8 \ub178\ubca8\uc0c1\uc744 \uc8fc\uc5b4\uc57c \ud588\ub2e4\u3009\uac70\ub098 \u3008\ub0b4 \ubc11\uc5d0 \uc544\uc778\uc288\ud0c0\uc778\ub3c4 \uc788\uc5c8\uc9c0\ub9cc \uc774\ud718\uc18c\uac00 \ub354 \ub6f0\uc5b4\ub0ac\ub2e4\u3009\ub294 \ub4f1\uc758 \uba87\uba87 \uc18c\uc124\ub4e4\uc5d0\uc11c\uc758 \ubb18\uc0ac\ub294 \uc9c0\ub098\uce58\uac8c \uacfc\uc7a5\ub41c \uac83\uc73c\ub85c \uc0ac\uc2e4\uacfc \ub2e4\ub974\ub2e4.", "paragraph_answer": "1962\ub144 6\uc6d4 \ucd08, \uc774\ud718\uc18c\ub294 \uad6d\uc81c \uc6d0\uc790\ub825 \uae30\uad6c\uac00 \uc8fc\ucd5c\ud558\ub294 \uc774\ud0c8\ub9ac\uc544 \ud2b8\ub9ac\uc5d0\uc2a4\ud14c \uc774\ub860\ubb3c\ub9ac\ud559 \uc138\ubbf8\ub098(Seminar on Theoretical Physics)\uc5d0 \ucc38\uc11d\ud560 10\uc778\uc758 \ubbf8\uad6d \ub300\ud45c\ub2e8 \uc77c\uc6d0\uc73c\ub85c \uc120\ucd9c\ub410\ub2e4. \ud2b8\ub9ac\uc5d0\uc2a4\ud14c \uc774\ub860\ubb3c\ub9ac\ud559 \uc138\ubbf8\ub098\ub294 1962\ub144 7\uc6d4 16\uc77c\ubd80\ud130 8\uc6d4 25\uc77c\uae4c\uc9c0 \uac1c\ucd5c\ub410\ub2e4. \uc774 \ubb34\ub835\uc758 \uc774\ud718\uc18c\ub294 \uc80a\uc740 \uc5f0\uad6c\uc790\ub85c\uc11c \ubbf8\uad6d \ub0b4\uc5d0\uc11c \ubb34\uc2dc\ubabb\ud560 \uba85\uc131\uacfc \ub6f0\uc5b4\ub09c \uc7ac\ub2a5\uc744 \uac00\uc9c0\uace0 \uc788\ub358 \uac83\uc740 \uc0ac\uc2e4\uc774\uc9c0\ub9cc, \uadf8 \ub2e8\uacc4\uc5d0\uc11c\ub294 \uc544\uc9c1\uc740 \uc5b4\ub5a4 \ud070 \uc131\uacfc\ub97c \ub0b4\uc9c0\ub294 \uc54a\uc558\ub2e4. \uadf8\uc758 \uc911\uc694\ud55c \ud559\ubb38\uc801 \uc131\uacfc\ub294 \ubaa8\ub450 1970\ub144\ub300\uc5d0 \uc774\ub974\ub7ec\uc11c\uc57c \ub098\ud0c0\ub098\uae30 \uc2dc\uc791\ud55c\ub2e4. \uadf8\ub7ec\ubbc0\ub85c \u300860\ub144\uc5d0 \uc911\ubc18\uc5d0 \uc774\ubbf8 \ub178\ubca8\uc0c1\uc744 \uc8fc\uc5b4\uc57c \ud588\ub2e4\u3009\uac70\ub098 \u3008\ub0b4 \ubc11\uc5d0 \uc544\uc778\uc288\ud0c0\uc778\ub3c4 \uc788\uc5c8\uc9c0\ub9cc \uc774\ud718\uc18c\uac00 \ub354 \ub6f0\uc5b4\ub0ac\ub2e4\u3009\ub294 \ub4f1\uc758 \uba87\uba87 \uc18c\uc124\ub4e4\uc5d0\uc11c\uc758 \ubb18\uc0ac\ub294 \uc9c0\ub098\uce58\uac8c \uacfc\uc7a5\ub41c \uac83\uc73c\ub85c \uc0ac\uc2e4\uacfc \ub2e4\ub974\ub2e4.", "sentence_answer": "1962\ub144 6\uc6d4 \ucd08, \uc774\ud718\uc18c\ub294 \uad6d\uc81c \uc6d0\uc790\ub825 \uae30\uad6c\uac00 \uc8fc\ucd5c\ud558\ub294 \uc774\ud0c8\ub9ac\uc544 \ud2b8\ub9ac\uc5d0\uc2a4\ud14c \uc774\ub860\ubb3c\ub9ac\ud559 \uc138\ubbf8\ub098(Seminar on Theoretical Physics)\uc5d0 \ucc38\uc11d\ud560 10\uc778\uc758 \ubbf8\uad6d \ub300\ud45c\ub2e8 \uc77c\uc6d0\uc73c\ub85c \uc120\ucd9c\ub410\ub2e4.", "paragraph_id": "6329268-7-0", "paragraph_question": "question: 1962\ub144 6\uc6d4\uc5d0 \uad6d\uc81c \uc6d0\uc790\ub825\uae30\uad6c\uac00 \uc774\ub860\ubb3c\ub9ac\ud559 \uc138\ubbf8\ub098\ub97c \uac1c\ucd5c\ud55c \ub098\ub77c\ub294?, context: 1962\ub144 6\uc6d4 \ucd08, \uc774\ud718\uc18c\ub294 \uad6d\uc81c \uc6d0\uc790\ub825 \uae30\uad6c\uac00 \uc8fc\ucd5c\ud558\ub294 \uc774\ud0c8\ub9ac\uc544 \ud2b8\ub9ac\uc5d0\uc2a4\ud14c \uc774\ub860\ubb3c\ub9ac\ud559 \uc138\ubbf8\ub098(Seminar on Theoretical Physics)\uc5d0 \ucc38\uc11d\ud560 10\uc778\uc758 \ubbf8\uad6d \ub300\ud45c\ub2e8 \uc77c\uc6d0\uc73c\ub85c \uc120\ucd9c\ub410\ub2e4. \ud2b8\ub9ac\uc5d0\uc2a4\ud14c \uc774\ub860\ubb3c\ub9ac\ud559 \uc138\ubbf8\ub098\ub294 1962\ub144 7\uc6d4 16\uc77c\ubd80\ud130 8\uc6d4 25\uc77c\uae4c\uc9c0 \uac1c\ucd5c\ub410\ub2e4. \uc774 \ubb34\ub835\uc758 \uc774\ud718\uc18c\ub294 \uc80a\uc740 \uc5f0\uad6c\uc790\ub85c\uc11c \ubbf8\uad6d \ub0b4\uc5d0\uc11c \ubb34\uc2dc\ubabb\ud560 \uba85\uc131\uacfc \ub6f0\uc5b4\ub09c \uc7ac\ub2a5\uc744 \uac00\uc9c0\uace0 \uc788\ub358 \uac83\uc740 \uc0ac\uc2e4\uc774\uc9c0\ub9cc, \uadf8 \ub2e8\uacc4\uc5d0\uc11c\ub294 \uc544\uc9c1\uc740 \uc5b4\ub5a4 \ud070 \uc131\uacfc\ub97c \ub0b4\uc9c0\ub294 \uc54a\uc558\ub2e4. \uadf8\uc758 \uc911\uc694\ud55c \ud559\ubb38\uc801 \uc131\uacfc\ub294 \ubaa8\ub450 1970\ub144\ub300\uc5d0 \uc774\ub974\ub7ec\uc11c\uc57c \ub098\ud0c0\ub098\uae30 \uc2dc\uc791\ud55c\ub2e4. \uadf8\ub7ec\ubbc0\ub85c \u300860\ub144\uc5d0 \uc911\ubc18\uc5d0 \uc774\ubbf8 \ub178\ubca8\uc0c1\uc744 \uc8fc\uc5b4\uc57c \ud588\ub2e4\u3009\uac70\ub098 \u3008\ub0b4 \ubc11\uc5d0 \uc544\uc778\uc288\ud0c0\uc778\ub3c4 \uc788\uc5c8\uc9c0\ub9cc \uc774\ud718\uc18c\uac00 \ub354 \ub6f0\uc5b4\ub0ac\ub2e4\u3009\ub294 \ub4f1\uc758 \uba87\uba87 \uc18c\uc124\ub4e4\uc5d0\uc11c\uc758 \ubb18\uc0ac\ub294 \uc9c0\ub098\uce58\uac8c \uacfc\uc7a5\ub41c \uac83\uc73c\ub85c \uc0ac\uc2e4\uacfc \ub2e4\ub974\ub2e4."} {"question": "\ube44\ud2b8\ucf54\uc778 \uc8fc\uc18c\ub85c \uc1a1\uc2e0\ub418\uc5b4\uc11c\ub294 \uc548 \ub418\ub294 \ube44\ud2b8\ucf54\uc778 \uce90\uc2dc\uc758 \uc601\uc22b\uc790 \uc8fc\uc18c\ub294 \ubb34\uc5c7\uacfc \ub3d9\uc77c\ud55c\uac00?", "paragraph": "\ube44\ud2b8\ucf54\uc778 \uce90\uc2dc(BCH/BCC) \uc8fc\uc18c\ub294 \ub354 \uc0c8\ub85c\uc6b4 P2SH \uc2a4\ud0c0\uc77c(\ucd5c\ub300 35\uc790) \ud0c0\uc785\uc758 \uc8fc\uc18c\ub97c \uc0ac\uc6a9\ud558\uba70 \uc774\ub294 \uc22b\uc790 3\uc73c\ub85c \uc2dc\uc791\ud55c\ub2e4. \uc774\ub97c\ud14c\uba74 \ube44\ud2b8\ucf54\uc778 \uc8fc\uc18c(BTC)\uc758 \uacbd\uc6b0 \"3NL8ZNQyt1Qr6N8ANR1tLrLava3QbYQBKe\"\uc640 \uac19\uace0, \ube44\ud2b8\ucf54\uc778 \uce90\uc2dc \uc8fc\uc18c(BCH/BCC)\ub294 \"3QM6DNHFpn5qEqWXP1MzwUfbTBzTisfpaw\"\uc640 \uac19\ub2e4. \uc601\uc22b\uc790 \uc8fc\uc18c \uc2a4\ud0c0\uc77c\uc774 \uc624\ub9ac\uc9c0\ub110 \ube44\ud2b8\ucf54\uc778(BTC)\uacfc \ub3d9\uc77c\ud558\uc9c0\ub9cc \ube44\ud2b8\ucf54\uc778 \uce90\uc2dc(BCH/BCC)\ub294 \ube44\ud2b8\ucf54\uc778(BTC) \uc8fc\uc18c\ub85c \uc1a1\uc2e0\ub418\uc5b4\uc11c\ub294 \uc548 \ub41c\ub2e4\ub294 \uac83\uc774 \uc911\uc694\ud558\ub2e4. \uc798\ubabb\ub41c \uc9c0\uac11 \ud0c0\uc785\uc73c\ub85c \uc1a1\uc2e0\ud560 \uacbd\uc6b0 \ucf54\uc778\ub4e4\uc740 \uc190\uc2e4\ub418\uc5b4 \ub3cc\uc774\ud0ac \uc218 \uc5c6\uac8c \ub41c\ub2e4. \uc624\ub9ac\uc9c0\ub110 \ube44\ud2b8\ucf54\uc778\ucc98\ub7fc \ube44\ud2b8\ucf54\uc778 \uce90\uc2dc \uc8fc\uc18c\ub294 \ud55c \ubc88 \uc774\uc0c1 \uc0ac\uc6a9\ud560 \uc218 \uc788\uc73c\ub098 \ud504\ub77c\uc774\ubc84\uc2dc\uac00 \uac71\uc815\ub420 \uacbd\uc6b0 \uc7ac\uc0ac\uc6a9\ud558\uc9c0 \uc54a\ub294 \uac83\uc774 \uc88b\ub2e4. \uadf8\ub7ec\ub098 \uc8fc\uc18c \ud615\uc2dd\uc744 \ubcc0\uacbd\ud560 \uacc4\ud68d\uc774 \uc788\ub2e4.", "answer": "\uc624\ub9ac\uc9c0\ub110 \ube44\ud2b8\ucf54\uc778", "sentence": "\uc601\uc22b\uc790 \uc8fc\uc18c \uc2a4\ud0c0\uc77c\uc774 \uc624\ub9ac\uc9c0\ub110 \ube44\ud2b8\ucf54\uc778 (BTC)", "paragraph_sentence": "\ube44\ud2b8\ucf54\uc778 \uce90\uc2dc(BCH/BCC) \uc8fc\uc18c\ub294 \ub354 \uc0c8\ub85c\uc6b4 P2SH \uc2a4\ud0c0\uc77c(\ucd5c\ub300 35\uc790) \ud0c0\uc785\uc758 \uc8fc\uc18c\ub97c \uc0ac\uc6a9\ud558\uba70 \uc774\ub294 \uc22b\uc790 3\uc73c\ub85c \uc2dc\uc791\ud55c\ub2e4. \uc774\ub97c\ud14c\uba74 \ube44\ud2b8\ucf54\uc778 \uc8fc\uc18c(BTC)\uc758 \uacbd\uc6b0 \"3NL8ZNQyt1Qr6N8ANR1tLrLava3QbYQBKe\"\uc640 \uac19\uace0, \ube44\ud2b8\ucf54\uc778 \uce90\uc2dc \uc8fc\uc18c(BCH/BCC)\ub294 \"3QM6DNHFpn5qEqWXP1MzwUfbTBzTisfpaw\"\uc640 \uac19\ub2e4. \uc601\uc22b\uc790 \uc8fc\uc18c \uc2a4\ud0c0\uc77c\uc774 \uc624\ub9ac\uc9c0\ub110 \ube44\ud2b8\ucf54\uc778 (BTC) \uacfc \ub3d9\uc77c\ud558\uc9c0\ub9cc \ube44\ud2b8\ucf54\uc778 \uce90\uc2dc(BCH/BCC)\ub294 \ube44\ud2b8\ucf54\uc778(BTC) \uc8fc\uc18c\ub85c \uc1a1\uc2e0\ub418\uc5b4\uc11c\ub294 \uc548 \ub41c\ub2e4\ub294 \uac83\uc774 \uc911\uc694\ud558\ub2e4. \uc798\ubabb\ub41c \uc9c0\uac11 \ud0c0\uc785\uc73c\ub85c \uc1a1\uc2e0\ud560 \uacbd\uc6b0 \ucf54\uc778\ub4e4\uc740 \uc190\uc2e4\ub418\uc5b4 \ub3cc\uc774\ud0ac \uc218 \uc5c6\uac8c \ub41c\ub2e4. \uc624\ub9ac\uc9c0\ub110 \ube44\ud2b8\ucf54\uc778\ucc98\ub7fc \ube44\ud2b8\ucf54\uc778 \uce90\uc2dc \uc8fc\uc18c\ub294 \ud55c \ubc88 \uc774\uc0c1 \uc0ac\uc6a9\ud560 \uc218 \uc788\uc73c\ub098 \ud504\ub77c\uc774\ubc84\uc2dc\uac00 \uac71\uc815\ub420 \uacbd\uc6b0 \uc7ac\uc0ac\uc6a9\ud558\uc9c0 \uc54a\ub294 \uac83\uc774 \uc88b\ub2e4. \uadf8\ub7ec\ub098 \uc8fc\uc18c \ud615\uc2dd\uc744 \ubcc0\uacbd\ud560 \uacc4\ud68d\uc774 \uc788\ub2e4.", "paragraph_answer": "\ube44\ud2b8\ucf54\uc778 \uce90\uc2dc(BCH/BCC) \uc8fc\uc18c\ub294 \ub354 \uc0c8\ub85c\uc6b4 P2SH \uc2a4\ud0c0\uc77c(\ucd5c\ub300 35\uc790) \ud0c0\uc785\uc758 \uc8fc\uc18c\ub97c \uc0ac\uc6a9\ud558\uba70 \uc774\ub294 \uc22b\uc790 3\uc73c\ub85c \uc2dc\uc791\ud55c\ub2e4. \uc774\ub97c\ud14c\uba74 \ube44\ud2b8\ucf54\uc778 \uc8fc\uc18c(BTC)\uc758 \uacbd\uc6b0 \"3NL8ZNQyt1Qr6N8ANR1tLrLava3QbYQBKe\"\uc640 \uac19\uace0, \ube44\ud2b8\ucf54\uc778 \uce90\uc2dc \uc8fc\uc18c(BCH/BCC)\ub294 \"3QM6DNHFpn5qEqWXP1MzwUfbTBzTisfpaw\"\uc640 \uac19\ub2e4. \uc601\uc22b\uc790 \uc8fc\uc18c \uc2a4\ud0c0\uc77c\uc774 \uc624\ub9ac\uc9c0\ub110 \ube44\ud2b8\ucf54\uc778 (BTC)\uacfc \ub3d9\uc77c\ud558\uc9c0\ub9cc \ube44\ud2b8\ucf54\uc778 \uce90\uc2dc(BCH/BCC)\ub294 \ube44\ud2b8\ucf54\uc778(BTC) \uc8fc\uc18c\ub85c \uc1a1\uc2e0\ub418\uc5b4\uc11c\ub294 \uc548 \ub41c\ub2e4\ub294 \uac83\uc774 \uc911\uc694\ud558\ub2e4. \uc798\ubabb\ub41c \uc9c0\uac11 \ud0c0\uc785\uc73c\ub85c \uc1a1\uc2e0\ud560 \uacbd\uc6b0 \ucf54\uc778\ub4e4\uc740 \uc190\uc2e4\ub418\uc5b4 \ub3cc\uc774\ud0ac \uc218 \uc5c6\uac8c \ub41c\ub2e4. \uc624\ub9ac\uc9c0\ub110 \ube44\ud2b8\ucf54\uc778\ucc98\ub7fc \ube44\ud2b8\ucf54\uc778 \uce90\uc2dc \uc8fc\uc18c\ub294 \ud55c \ubc88 \uc774\uc0c1 \uc0ac\uc6a9\ud560 \uc218 \uc788\uc73c\ub098 \ud504\ub77c\uc774\ubc84\uc2dc\uac00 \uac71\uc815\ub420 \uacbd\uc6b0 \uc7ac\uc0ac\uc6a9\ud558\uc9c0 \uc54a\ub294 \uac83\uc774 \uc88b\ub2e4. \uadf8\ub7ec\ub098 \uc8fc\uc18c \ud615\uc2dd\uc744 \ubcc0\uacbd\ud560 \uacc4\ud68d\uc774 \uc788\ub2e4.", "sentence_answer": "\uc601\uc22b\uc790 \uc8fc\uc18c \uc2a4\ud0c0\uc77c\uc774 \uc624\ub9ac\uc9c0\ub110 \ube44\ud2b8\ucf54\uc778 (BTC)", "paragraph_id": "6531189-2-1", "paragraph_question": "question: \ube44\ud2b8\ucf54\uc778 \uc8fc\uc18c\ub85c \uc1a1\uc2e0\ub418\uc5b4\uc11c\ub294 \uc548 \ub418\ub294 \ube44\ud2b8\ucf54\uc778 \uce90\uc2dc\uc758 \uc601\uc22b\uc790 \uc8fc\uc18c\ub294 \ubb34\uc5c7\uacfc \ub3d9\uc77c\ud55c\uac00?, context: \ube44\ud2b8\ucf54\uc778 \uce90\uc2dc(BCH/BCC) \uc8fc\uc18c\ub294 \ub354 \uc0c8\ub85c\uc6b4 P2SH \uc2a4\ud0c0\uc77c(\ucd5c\ub300 35\uc790) \ud0c0\uc785\uc758 \uc8fc\uc18c\ub97c \uc0ac\uc6a9\ud558\uba70 \uc774\ub294 \uc22b\uc790 3\uc73c\ub85c \uc2dc\uc791\ud55c\ub2e4. \uc774\ub97c\ud14c\uba74 \ube44\ud2b8\ucf54\uc778 \uc8fc\uc18c(BTC)\uc758 \uacbd\uc6b0 \"3NL8ZNQyt1Qr6N8ANR1tLrLava3QbYQBKe\"\uc640 \uac19\uace0, \ube44\ud2b8\ucf54\uc778 \uce90\uc2dc \uc8fc\uc18c(BCH/BCC)\ub294 \"3QM6DNHFpn5qEqWXP1MzwUfbTBzTisfpaw\"\uc640 \uac19\ub2e4. \uc601\uc22b\uc790 \uc8fc\uc18c \uc2a4\ud0c0\uc77c\uc774 \uc624\ub9ac\uc9c0\ub110 \ube44\ud2b8\ucf54\uc778(BTC)\uacfc \ub3d9\uc77c\ud558\uc9c0\ub9cc \ube44\ud2b8\ucf54\uc778 \uce90\uc2dc(BCH/BCC)\ub294 \ube44\ud2b8\ucf54\uc778(BTC) \uc8fc\uc18c\ub85c \uc1a1\uc2e0\ub418\uc5b4\uc11c\ub294 \uc548 \ub41c\ub2e4\ub294 \uac83\uc774 \uc911\uc694\ud558\ub2e4. \uc798\ubabb\ub41c \uc9c0\uac11 \ud0c0\uc785\uc73c\ub85c \uc1a1\uc2e0\ud560 \uacbd\uc6b0 \ucf54\uc778\ub4e4\uc740 \uc190\uc2e4\ub418\uc5b4 \ub3cc\uc774\ud0ac \uc218 \uc5c6\uac8c \ub41c\ub2e4. \uc624\ub9ac\uc9c0\ub110 \ube44\ud2b8\ucf54\uc778\ucc98\ub7fc \ube44\ud2b8\ucf54\uc778 \uce90\uc2dc \uc8fc\uc18c\ub294 \ud55c \ubc88 \uc774\uc0c1 \uc0ac\uc6a9\ud560 \uc218 \uc788\uc73c\ub098 \ud504\ub77c\uc774\ubc84\uc2dc\uac00 \uac71\uc815\ub420 \uacbd\uc6b0 \uc7ac\uc0ac\uc6a9\ud558\uc9c0 \uc54a\ub294 \uac83\uc774 \uc88b\ub2e4. \uadf8\ub7ec\ub098 \uc8fc\uc18c \ud615\uc2dd\uc744 \ubcc0\uacbd\ud560 \uacc4\ud68d\uc774 \uc788\ub2e4."} {"question": "\uad11\ud574\uad70 \uc2dc\uae30 \uc218\uc785\uc774 \ub9ce\uc740 \uc0ac\ub78c\ub4e4\uc774 \ub354 \ub9ce\uc740 \uc138\uae08\uc744 \ub0b4\uace0 \uc218\uc785\uc774 \uc801\uc740 \uc0ac\ub78c\ub4e4\uc774 \ub354 \uc801\uc740 \uc138\uae08\uc744 \ub0bc \uc218 \uc788\ub3c4\ub85d \ud55c \uc138\uae08\uc81c\ub3c4\uc758 \uc774\ub984\uc740?", "paragraph": "\uc784\uae08\uc774 \ub41c \uad11\ud574\uad70\uc740 \uc989\uc704 \ucd08\ubd80\ud130 \uc548\uc73c\ub85c\ub294 \uc655\uad8c\uc744 \uac15\ud654\ud558\uae30 \uc704\ud574 \uad81\uad90\uc744 \uc9c0\uc5b4 \uacbd\uc81c\uac00 \ud30c\ud0c4 \ub0ac\uace0 \uc774\uc810\uc740 \uad11\ud574\uad70\uc744 \uae0d\uc815\uc801\uc73c\ub85c \ud3c9\uac00\ud558\ub294 \ud55c\uba85\uae30 \uad50\uc218 \uc870\ucc28\ub3c4 \"\uad11\ud574\uad70\uc740 \ud0c1\uc6d4\ud55c \uc678\uad50 \uc815\ucc45\uc744 \uad81\uad90 \uacf5\uc0ac\ub85c \ubaa8\ub450 \ub9d0\uc544 \uba39\uc5c8\ub2e4\" \ub77c\uba70 \ube44\ud310\ud55c\ub2e4. 1608\ub144 \uc120\ud61c\uccad\uc744 \ub450\uc5b4 \uacbd\uae30\ub3c4\uc5d0\uc11c \uc300\ub85c \uc870\uc138\ub97c \ub0b4\ub3c4\ub85d \ud568\uc73c\ub85c\uc368 \uc18c\ub4dd\uc5d0 \ub530\ub77c \uc138\uae08\uc744 \ub0b4\ub294 \uc870\uc138\uac1c\ud601\uc778 \ub300\ub3d9\ubc95\uc744 \uc5b5\uc9c0\ub85c \uc2dc\ud589\ud588\uc73c\ub098,\ub450 \ubc88\uc774\ub098 \ud3d0\uc9c0\ud558\ub824 \ud588\ub2e4. 1611\ub144 \uc591\uc804 \uc0ac\uc5c5\uc744 \ubc8c\uc600\ub2e4\uace0 \ud558\ub098 \uc120\uc870 \ub300\uc758 \uacc4\ubb18\uc591\uc804, \uc778\uc870 \ub300\uc758 \uac11\uc220\uc591\uc804 \uc774\uc678\uc5d0\ub294 \uc591\uc804\uc744 \uc2dc\ud589\ud55c \uae30\ub85d\uc774 \uc5c6\ub2e4. \uc774\uc5b4 \uc784\uc9c4\uc65c\ub780 \ub54c \ud654\uc7ac\ub85c \uc18c\uc2e4\ub41c \ucc3d\ub355\uad81, \uacbd\ud76c\uad81, \ucc3d\uacbd\uad81\uc744 \uc7ac\uac74\ud558\uace0 \uc778\uacbd\uad81\uc744 \uac74\uc124\ud588\uc73c\uba70, \uc784\uc9c4\uc65c\ub780 \ub54c \uc18c\uc2e4\ub41c \uc11c\uc801 \uac04\ud589\uc5d0\ub3c4 \ud798\uc368 \u300a\uc2e0\uc99d\ub3d9\uad6d\uc5ec\uc9c0\uc2b9\ub78c\u300b, \u300a\uc6a9\ube44\uc5b4\ucc9c\uac00\u300b ,\u300a\ub3d9\uad6d\uc2e0\uc18d\uc0bc\uac15\ud589\uc2e4\u300b \ub4f1\uc744 \ub2e4\uc2dc \uac04\ud589\ud588\ub2e4. \ud5c8\uade0\uc758 \u300a\ud64d\uae38\ub3d9\uc804\u300b, \ud5c8\uc900\uc758 \ud55c\uc758\ud559\ucc45\uc778\u300a\ub3d9\uc758\ubcf4\uac10\u300b \ub4f1\ub3c4 \uc774 \uc2dc\uae30\uc5d0 \uc644\uc131\ub418\uc5c8\ub2e4.", "answer": "\ub300\ub3d9\ubc95", "sentence": "1608\ub144 \uc120\ud61c\uccad\uc744 \ub450\uc5b4 \uacbd\uae30\ub3c4\uc5d0\uc11c \uc300\ub85c \uc870\uc138\ub97c \ub0b4\ub3c4\ub85d \ud568\uc73c\ub85c\uc368 \uc18c\ub4dd\uc5d0 \ub530\ub77c \uc138\uae08\uc744 \ub0b4\ub294 \uc870\uc138\uac1c\ud601\uc778 \ub300\ub3d9\ubc95 \uc744 \uc5b5\uc9c0\ub85c \uc2dc\ud589\ud588\uc73c\ub098,\ub450 \ubc88\uc774\ub098 \ud3d0\uc9c0\ud558\ub824 \ud588\ub2e4.", "paragraph_sentence": "\uc784\uae08\uc774 \ub41c \uad11\ud574\uad70\uc740 \uc989\uc704 \ucd08\ubd80\ud130 \uc548\uc73c\ub85c\ub294 \uc655\uad8c\uc744 \uac15\ud654\ud558\uae30 \uc704\ud574 \uad81\uad90\uc744 \uc9c0\uc5b4 \uacbd\uc81c\uac00 \ud30c\ud0c4 \ub0ac\uace0 \uc774\uc810\uc740 \uad11\ud574\uad70\uc744 \uae0d\uc815\uc801\uc73c\ub85c \ud3c9\uac00\ud558\ub294 \ud55c\uba85\uae30 \uad50\uc218 \uc870\ucc28\ub3c4 \"\uad11\ud574\uad70\uc740 \ud0c1\uc6d4\ud55c \uc678\uad50 \uc815\ucc45\uc744 \uad81\uad90 \uacf5\uc0ac\ub85c \ubaa8\ub450 \ub9d0\uc544 \uba39\uc5c8\ub2e4\" \ub77c\uba70 \ube44\ud310\ud55c\ub2e4. 1608\ub144 \uc120\ud61c\uccad\uc744 \ub450\uc5b4 \uacbd\uae30\ub3c4\uc5d0\uc11c \uc300\ub85c \uc870\uc138\ub97c \ub0b4\ub3c4\ub85d \ud568\uc73c\ub85c\uc368 \uc18c\ub4dd\uc5d0 \ub530\ub77c \uc138\uae08\uc744 \ub0b4\ub294 \uc870\uc138\uac1c\ud601\uc778 \ub300\ub3d9\ubc95 \uc744 \uc5b5\uc9c0\ub85c \uc2dc\ud589\ud588\uc73c\ub098,\ub450 \ubc88\uc774\ub098 \ud3d0\uc9c0\ud558\ub824 \ud588\ub2e4. 1611\ub144 \uc591\uc804 \uc0ac\uc5c5\uc744 \ubc8c\uc600\ub2e4\uace0 \ud558\ub098 \uc120\uc870 \ub300\uc758 \uacc4\ubb18\uc591\uc804, \uc778\uc870 \ub300\uc758 \uac11\uc220\uc591\uc804 \uc774\uc678\uc5d0\ub294 \uc591\uc804\uc744 \uc2dc\ud589\ud55c \uae30\ub85d\uc774 \uc5c6\ub2e4. \uc774\uc5b4 \uc784\uc9c4\uc65c\ub780 \ub54c \ud654\uc7ac\ub85c \uc18c\uc2e4\ub41c \ucc3d\ub355\uad81, \uacbd\ud76c\uad81, \ucc3d\uacbd\uad81\uc744 \uc7ac\uac74\ud558\uace0 \uc778\uacbd\uad81\uc744 \uac74\uc124\ud588\uc73c\uba70, \uc784\uc9c4\uc65c\ub780 \ub54c \uc18c\uc2e4\ub41c \uc11c\uc801 \uac04\ud589\uc5d0\ub3c4 \ud798\uc368 \u300a\uc2e0\uc99d\ub3d9\uad6d\uc5ec\uc9c0\uc2b9\ub78c\u300b, \u300a\uc6a9\ube44\uc5b4\ucc9c\uac00\u300b ,\u300a\ub3d9\uad6d\uc2e0\uc18d\uc0bc\uac15\ud589\uc2e4\u300b \ub4f1\uc744 \ub2e4\uc2dc \uac04\ud589\ud588\ub2e4. \ud5c8\uade0\uc758 \u300a\ud64d\uae38\ub3d9\uc804\u300b, \ud5c8\uc900\uc758 \ud55c\uc758\ud559\ucc45\uc778\u300a\ub3d9\uc758\ubcf4\uac10\u300b \ub4f1\ub3c4 \uc774 \uc2dc\uae30\uc5d0 \uc644\uc131\ub418\uc5c8\ub2e4.", "paragraph_answer": "\uc784\uae08\uc774 \ub41c \uad11\ud574\uad70\uc740 \uc989\uc704 \ucd08\ubd80\ud130 \uc548\uc73c\ub85c\ub294 \uc655\uad8c\uc744 \uac15\ud654\ud558\uae30 \uc704\ud574 \uad81\uad90\uc744 \uc9c0\uc5b4 \uacbd\uc81c\uac00 \ud30c\ud0c4 \ub0ac\uace0 \uc774\uc810\uc740 \uad11\ud574\uad70\uc744 \uae0d\uc815\uc801\uc73c\ub85c \ud3c9\uac00\ud558\ub294 \ud55c\uba85\uae30 \uad50\uc218 \uc870\ucc28\ub3c4 \"\uad11\ud574\uad70\uc740 \ud0c1\uc6d4\ud55c \uc678\uad50 \uc815\ucc45\uc744 \uad81\uad90 \uacf5\uc0ac\ub85c \ubaa8\ub450 \ub9d0\uc544 \uba39\uc5c8\ub2e4\" \ub77c\uba70 \ube44\ud310\ud55c\ub2e4. 1608\ub144 \uc120\ud61c\uccad\uc744 \ub450\uc5b4 \uacbd\uae30\ub3c4\uc5d0\uc11c \uc300\ub85c \uc870\uc138\ub97c \ub0b4\ub3c4\ub85d \ud568\uc73c\ub85c\uc368 \uc18c\ub4dd\uc5d0 \ub530\ub77c \uc138\uae08\uc744 \ub0b4\ub294 \uc870\uc138\uac1c\ud601\uc778 \ub300\ub3d9\ubc95 \uc744 \uc5b5\uc9c0\ub85c \uc2dc\ud589\ud588\uc73c\ub098,\ub450 \ubc88\uc774\ub098 \ud3d0\uc9c0\ud558\ub824 \ud588\ub2e4. 1611\ub144 \uc591\uc804 \uc0ac\uc5c5\uc744 \ubc8c\uc600\ub2e4\uace0 \ud558\ub098 \uc120\uc870 \ub300\uc758 \uacc4\ubb18\uc591\uc804, \uc778\uc870 \ub300\uc758 \uac11\uc220\uc591\uc804 \uc774\uc678\uc5d0\ub294 \uc591\uc804\uc744 \uc2dc\ud589\ud55c \uae30\ub85d\uc774 \uc5c6\ub2e4. \uc774\uc5b4 \uc784\uc9c4\uc65c\ub780 \ub54c \ud654\uc7ac\ub85c \uc18c\uc2e4\ub41c \ucc3d\ub355\uad81, \uacbd\ud76c\uad81, \ucc3d\uacbd\uad81\uc744 \uc7ac\uac74\ud558\uace0 \uc778\uacbd\uad81\uc744 \uac74\uc124\ud588\uc73c\uba70, \uc784\uc9c4\uc65c\ub780 \ub54c \uc18c\uc2e4\ub41c \uc11c\uc801 \uac04\ud589\uc5d0\ub3c4 \ud798\uc368 \u300a\uc2e0\uc99d\ub3d9\uad6d\uc5ec\uc9c0\uc2b9\ub78c\u300b, \u300a\uc6a9\ube44\uc5b4\ucc9c\uac00\u300b ,\u300a\ub3d9\uad6d\uc2e0\uc18d\uc0bc\uac15\ud589\uc2e4\u300b \ub4f1\uc744 \ub2e4\uc2dc \uac04\ud589\ud588\ub2e4. \ud5c8\uade0\uc758 \u300a\ud64d\uae38\ub3d9\uc804\u300b, \ud5c8\uc900\uc758 \ud55c\uc758\ud559\ucc45\uc778\u300a\ub3d9\uc758\ubcf4\uac10\u300b \ub4f1\ub3c4 \uc774 \uc2dc\uae30\uc5d0 \uc644\uc131\ub418\uc5c8\ub2e4.", "sentence_answer": "1608\ub144 \uc120\ud61c\uccad\uc744 \ub450\uc5b4 \uacbd\uae30\ub3c4\uc5d0\uc11c \uc300\ub85c \uc870\uc138\ub97c \ub0b4\ub3c4\ub85d \ud568\uc73c\ub85c\uc368 \uc18c\ub4dd\uc5d0 \ub530\ub77c \uc138\uae08\uc744 \ub0b4\ub294 \uc870\uc138\uac1c\ud601\uc778 \ub300\ub3d9\ubc95 \uc744 \uc5b5\uc9c0\ub85c \uc2dc\ud589\ud588\uc73c\ub098,\ub450 \ubc88\uc774\ub098 \ud3d0\uc9c0\ud558\ub824 \ud588\ub2e4.", "paragraph_id": "6339795-2-0", "paragraph_question": "question: \uad11\ud574\uad70 \uc2dc\uae30 \uc218\uc785\uc774 \ub9ce\uc740 \uc0ac\ub78c\ub4e4\uc774 \ub354 \ub9ce\uc740 \uc138\uae08\uc744 \ub0b4\uace0 \uc218\uc785\uc774 \uc801\uc740 \uc0ac\ub78c\ub4e4\uc774 \ub354 \uc801\uc740 \uc138\uae08\uc744 \ub0bc \uc218 \uc788\ub3c4\ub85d \ud55c \uc138\uae08\uc81c\ub3c4\uc758 \uc774\ub984\uc740?, context: \uc784\uae08\uc774 \ub41c \uad11\ud574\uad70\uc740 \uc989\uc704 \ucd08\ubd80\ud130 \uc548\uc73c\ub85c\ub294 \uc655\uad8c\uc744 \uac15\ud654\ud558\uae30 \uc704\ud574 \uad81\uad90\uc744 \uc9c0\uc5b4 \uacbd\uc81c\uac00 \ud30c\ud0c4 \ub0ac\uace0 \uc774\uc810\uc740 \uad11\ud574\uad70\uc744 \uae0d\uc815\uc801\uc73c\ub85c \ud3c9\uac00\ud558\ub294 \ud55c\uba85\uae30 \uad50\uc218 \uc870\ucc28\ub3c4 \"\uad11\ud574\uad70\uc740 \ud0c1\uc6d4\ud55c \uc678\uad50 \uc815\ucc45\uc744 \uad81\uad90 \uacf5\uc0ac\ub85c \ubaa8\ub450 \ub9d0\uc544 \uba39\uc5c8\ub2e4\" \ub77c\uba70 \ube44\ud310\ud55c\ub2e4. 1608\ub144 \uc120\ud61c\uccad\uc744 \ub450\uc5b4 \uacbd\uae30\ub3c4\uc5d0\uc11c \uc300\ub85c \uc870\uc138\ub97c \ub0b4\ub3c4\ub85d \ud568\uc73c\ub85c\uc368 \uc18c\ub4dd\uc5d0 \ub530\ub77c \uc138\uae08\uc744 \ub0b4\ub294 \uc870\uc138\uac1c\ud601\uc778 \ub300\ub3d9\ubc95\uc744 \uc5b5\uc9c0\ub85c \uc2dc\ud589\ud588\uc73c\ub098,\ub450 \ubc88\uc774\ub098 \ud3d0\uc9c0\ud558\ub824 \ud588\ub2e4. 1611\ub144 \uc591\uc804 \uc0ac\uc5c5\uc744 \ubc8c\uc600\ub2e4\uace0 \ud558\ub098 \uc120\uc870 \ub300\uc758 \uacc4\ubb18\uc591\uc804, \uc778\uc870 \ub300\uc758 \uac11\uc220\uc591\uc804 \uc774\uc678\uc5d0\ub294 \uc591\uc804\uc744 \uc2dc\ud589\ud55c \uae30\ub85d\uc774 \uc5c6\ub2e4. \uc774\uc5b4 \uc784\uc9c4\uc65c\ub780 \ub54c \ud654\uc7ac\ub85c \uc18c\uc2e4\ub41c \ucc3d\ub355\uad81, \uacbd\ud76c\uad81, \ucc3d\uacbd\uad81\uc744 \uc7ac\uac74\ud558\uace0 \uc778\uacbd\uad81\uc744 \uac74\uc124\ud588\uc73c\uba70, \uc784\uc9c4\uc65c\ub780 \ub54c \uc18c\uc2e4\ub41c \uc11c\uc801 \uac04\ud589\uc5d0\ub3c4 \ud798\uc368 \u300a\uc2e0\uc99d\ub3d9\uad6d\uc5ec\uc9c0\uc2b9\ub78c\u300b, \u300a\uc6a9\ube44\uc5b4\ucc9c\uac00\u300b ,\u300a\ub3d9\uad6d\uc2e0\uc18d\uc0bc\uac15\ud589\uc2e4\u300b \ub4f1\uc744 \ub2e4\uc2dc \uac04\ud589\ud588\ub2e4. \ud5c8\uade0\uc758 \u300a\ud64d\uae38\ub3d9\uc804\u300b, \ud5c8\uc900\uc758 \ud55c\uc758\ud559\ucc45\uc778\u300a\ub3d9\uc758\ubcf4\uac10\u300b \ub4f1\ub3c4 \uc774 \uc2dc\uae30\uc5d0 \uc644\uc131\ub418\uc5c8\ub2e4."} {"question": "\uc5f0\ud3c9\ub3c4 \ud3ec\uaca9\uc758 \uc774\uc720\uc5d0 \ub300\ud55c \uac00\uc7a5 \uc124\ub4dd\ub825\uc788\ub294 \uc8fc\uc7a5\uc740?", "paragraph": "2010\ub144 11\uc6d4 23\uc77c\uc758 \uc5f0\ud3c9\ub3c4 \ud3ec\uaca9 \uc0ac\uac74\uc740 \uc870\uc120\ubbfc\uc8fc\uc8fc\uc758\uc778\ubbfc\uacf5\ud654\uad6d\uc758 \ucca0\uc800\ud55c \uacc4\ud68d \uc544\ub798 \uc2e4\uc2dc\ub418\uc5c8\ub2e4\ub294 \ubd84\uc11d\uc774 \uc9c0\ubc30\uc801\uc774\ub2e4. \uadf8 \uc774\uc720\ub294 \ub300\ud55c\ubbfc\uad6d \uad6d\uad70\uc774 \uc870\uc120\ubbfc\uc8fc\uc8fc\uc758\uc778\ubbfc\uacf5\ud654\uad6d\uc5d0 \uc11c\ud574\uc0c1\uc758 \uad70\uc0ac\ud6c8\ub828\uc744 \ud1b5\ubcf4\ud55c \ubc14 \uc788\uae30 \ub54c\ubb38\uc5d0 \uae40\uc815\uc77c\uc758 \ub3d9\uc758 \ud558\uc5d0 \uc774\ub8e8\uc5b4\uc84c\uc744 \uac00\ub2a5\uc131\uc774 \ub192\ub2e4\ub294 \uac83\uc774\ub2e4. \ub610\ud55c \uc870\uc120\ubbfc\uc8fc\uc8fc\uc758\uc778\ubbfc\uacf5\ud654\uad6d \uae40\uc815\uc77c \uad6d\ubc29\uc704\uc6d0\uc7a5\uc758 \ud6c4\uacc4\uc790 \uae40\uc815\uc740\uc774 \uc5f0\ud3c9\ub3c4 \ub3c4\ubc1c \uc900\ube44\ub97c \uc9c0\ub09c\ub2ec\uc5d0 \uc774\ubbf8 \uc9c0\uc2dc\ud55c \uc815\ud669\ub3c4 \uc788\uc5c8\ub294\ub370, \uc77c\ubcf8 \uc544\uc0ac\ud788 \uc2e0\ubb38\uc740 12\uc6d4 1\uc77c \ubd81\ud55c \uc18c\uc2dd\ud1b5\uc744 \uc778\uc6a9\ud574 \u201c\uae40\uc815\uc740\uc758 \uc774\ub984\uc73c\ub85c \uc9c0\ub09c\ub2ec \ucd08 \u2018\uc801\uc758 \ub3c4\ubc1c \ud589\uc704\uc5d0 \uc5b8\uc81c\ub77c\ub3c4 \ubc18\uaca9\ud560 \uc218 \uc788\ub294 \ud0dc\uc138\ub97c \uac16\ucd94\ub77c\u2019\ub294 \uc9c0\ub839\uc774 \uc870\uc120\uc778\ubbfc\uad70 \uac04\ubd80\ub4e4\uc5d0\uac8c \ud558\ub2ec\ub410\ub2e4\u201d\uace0 \ubcf4\ub3c4\ud588\ub2e4. \uc774\uac83\uc774 \uc0ac\uc2e4\uc774\ub77c\uba74 \uc5f0\ud3c9\ub3c4 \ud3ec\uaca9\uc740 \uc608\uc815\ub410\ub358 \ud589\ub3d9\uc774\uba70 '\ucda9\ubd84\ud55c \uae30\uac04\uc744 \uac00\uc9c0\uace0 \uc900\ube44'\ud55c \uac83\uc73c\ub85c, \uc870\uc120\uc778\ubbfc\uad70\uc774 \uae40\uc815\uc740\uc758 \uc9c0\uc2dc\ub97c \ubc1b\uace0 \uacf5\uaca9 \uae30\ud68c\ub97c \uc5ff\ubcf4\uace0 \uc788\ub2e4\uac00 \ub300\ud55c\ubbfc\uad6d \uad6d\uad70\uc758 \uc9c0\ub09c 23\uc77c \ud6c8\ub828\uc744 \uad6c\uc2e4\ub85c \ud3ec\uaca9\uc744 \uac10\ud589\ud588\uc744 \uac00\ub2a5\uc131\uc774 \ub192\uc73c\uba70, \uc5f0\ud3c9\ub3c4 \ubb34\ub825\uacf5\uaca9 \uc9c1\uc804 \uc870\uc120\ubbfc\uc8fc\uc8fc\uc758\uc778\ubbfc\uacf5\ud654\uad6d\uc758 \ubbf8\uadf8-23\uae30 5\ub300\uac00 \uc11c\ud574 5\ub3c4 \uc778\uadfc\uc5d0\uc11c \ucd08\uacc4\ube44\ud589\uc744 \ubc8c\uc600\ub358 \uc0ac\uc2e4\ub3c4 \ub4dc\ub7ec\ub0ac\ub2e4. \ub610\ud55c \uae40\uc815\uc740 \ud6c4\uacc4 \uad6c\ub3c4\ub97c \ub354\uc6b1 \ud655\ub9bd\ud558\uae30 \uc704\ud55c \uac83\uc774\ub77c\ub294 \ubd84\uc11d\uc774 \uc81c\uae30\ub41c\ub2e4. \ud2b9\ud788 \uae40\uc815\uc740\uc740 \ubd81\uce21\uc5d0\uc11c \ud3ec\ubcd1 \uc804\ubb38\uac00\ub85c \ubbf8\ud654\ub418\uace0 \uc788\ub2e4\ub294 \uc810\uc5d0\uc11c \uadf8\uc758 \uc5c5\uc801\uc744 \uadf9\ub300\ud654 \ud558\uae30 \uc704\ud55c \uac83\uc774\ub77c\ub294 \uc8fc\uc7a5\uc774 \uc124\ub4dd\ub825\uc744 \uc5bb\uace0 \uc788\ub2e4.", "answer": "\ud6c4\uacc4 \uad6c\ub3c4\ub97c \ub354\uc6b1 \ud655\ub9bd", "sentence": "\ub610\ud55c \uae40\uc815\uc740 \ud6c4\uacc4 \uad6c\ub3c4\ub97c \ub354\uc6b1 \ud655\ub9bd \ud558\uae30 \uc704\ud55c \uac83\uc774\ub77c\ub294 \ubd84\uc11d\uc774 \uc81c\uae30\ub41c\ub2e4.", "paragraph_sentence": "2010\ub144 11\uc6d4 23\uc77c\uc758 \uc5f0\ud3c9\ub3c4 \ud3ec\uaca9 \uc0ac\uac74\uc740 \uc870\uc120\ubbfc\uc8fc\uc8fc\uc758\uc778\ubbfc\uacf5\ud654\uad6d\uc758 \ucca0\uc800\ud55c \uacc4\ud68d \uc544\ub798 \uc2e4\uc2dc\ub418\uc5c8\ub2e4\ub294 \ubd84\uc11d\uc774 \uc9c0\ubc30\uc801\uc774\ub2e4. \uadf8 \uc774\uc720\ub294 \ub300\ud55c\ubbfc\uad6d \uad6d\uad70\uc774 \uc870\uc120\ubbfc\uc8fc\uc8fc\uc758\uc778\ubbfc\uacf5\ud654\uad6d\uc5d0 \uc11c\ud574\uc0c1\uc758 \uad70\uc0ac\ud6c8\ub828\uc744 \ud1b5\ubcf4\ud55c \ubc14 \uc788\uae30 \ub54c\ubb38\uc5d0 \uae40\uc815\uc77c\uc758 \ub3d9\uc758 \ud558\uc5d0 \uc774\ub8e8\uc5b4\uc84c\uc744 \uac00\ub2a5\uc131\uc774 \ub192\ub2e4\ub294 \uac83\uc774\ub2e4. \ub610\ud55c \uc870\uc120\ubbfc\uc8fc\uc8fc\uc758\uc778\ubbfc\uacf5\ud654\uad6d \uae40\uc815\uc77c \uad6d\ubc29\uc704\uc6d0\uc7a5\uc758 \ud6c4\uacc4\uc790 \uae40\uc815\uc740\uc774 \uc5f0\ud3c9\ub3c4 \ub3c4\ubc1c \uc900\ube44\ub97c \uc9c0\ub09c\ub2ec\uc5d0 \uc774\ubbf8 \uc9c0\uc2dc\ud55c \uc815\ud669\ub3c4 \uc788\uc5c8\ub294\ub370, \uc77c\ubcf8 \uc544\uc0ac\ud788 \uc2e0\ubb38\uc740 12\uc6d4 1\uc77c \ubd81\ud55c \uc18c\uc2dd\ud1b5\uc744 \uc778\uc6a9\ud574 \u201c\uae40\uc815\uc740\uc758 \uc774\ub984\uc73c\ub85c \uc9c0\ub09c\ub2ec \ucd08 \u2018\uc801\uc758 \ub3c4\ubc1c \ud589\uc704\uc5d0 \uc5b8\uc81c\ub77c\ub3c4 \ubc18\uaca9\ud560 \uc218 \uc788\ub294 \ud0dc\uc138\ub97c \uac16\ucd94\ub77c\u2019\ub294 \uc9c0\ub839\uc774 \uc870\uc120\uc778\ubbfc\uad70 \uac04\ubd80\ub4e4\uc5d0\uac8c \ud558\ub2ec\ub410\ub2e4\u201d\uace0 \ubcf4\ub3c4\ud588\ub2e4. \uc774\uac83\uc774 \uc0ac\uc2e4\uc774\ub77c\uba74 \uc5f0\ud3c9\ub3c4 \ud3ec\uaca9\uc740 \uc608\uc815\ub410\ub358 \ud589\ub3d9\uc774\uba70 '\ucda9\ubd84\ud55c \uae30\uac04\uc744 \uac00\uc9c0\uace0 \uc900\ube44'\ud55c \uac83\uc73c\ub85c, \uc870\uc120\uc778\ubbfc\uad70\uc774 \uae40\uc815\uc740\uc758 \uc9c0\uc2dc\ub97c \ubc1b\uace0 \uacf5\uaca9 \uae30\ud68c\ub97c \uc5ff\ubcf4\uace0 \uc788\ub2e4\uac00 \ub300\ud55c\ubbfc\uad6d \uad6d\uad70\uc758 \uc9c0\ub09c 23\uc77c \ud6c8\ub828\uc744 \uad6c\uc2e4\ub85c \ud3ec\uaca9\uc744 \uac10\ud589\ud588\uc744 \uac00\ub2a5\uc131\uc774 \ub192\uc73c\uba70, \uc5f0\ud3c9\ub3c4 \ubb34\ub825\uacf5\uaca9 \uc9c1\uc804 \uc870\uc120\ubbfc\uc8fc\uc8fc\uc758\uc778\ubbfc\uacf5\ud654\uad6d\uc758 \ubbf8\uadf8-23\uae30 5\ub300\uac00 \uc11c\ud574 5\ub3c4 \uc778\uadfc\uc5d0\uc11c \ucd08\uacc4\ube44\ud589\uc744 \ubc8c\uc600\ub358 \uc0ac\uc2e4\ub3c4 \ub4dc\ub7ec\ub0ac\ub2e4. \ub610\ud55c \uae40\uc815\uc740 \ud6c4\uacc4 \uad6c\ub3c4\ub97c \ub354\uc6b1 \ud655\ub9bd \ud558\uae30 \uc704\ud55c \uac83\uc774\ub77c\ub294 \ubd84\uc11d\uc774 \uc81c\uae30\ub41c\ub2e4. \ud2b9\ud788 \uae40\uc815\uc740\uc740 \ubd81\uce21\uc5d0\uc11c \ud3ec\ubcd1 \uc804\ubb38\uac00\ub85c \ubbf8\ud654\ub418\uace0 \uc788\ub2e4\ub294 \uc810\uc5d0\uc11c \uadf8\uc758 \uc5c5\uc801\uc744 \uadf9\ub300\ud654 \ud558\uae30 \uc704\ud55c \uac83\uc774\ub77c\ub294 \uc8fc\uc7a5\uc774 \uc124\ub4dd\ub825\uc744 \uc5bb\uace0 \uc788\ub2e4.", "paragraph_answer": "2010\ub144 11\uc6d4 23\uc77c\uc758 \uc5f0\ud3c9\ub3c4 \ud3ec\uaca9 \uc0ac\uac74\uc740 \uc870\uc120\ubbfc\uc8fc\uc8fc\uc758\uc778\ubbfc\uacf5\ud654\uad6d\uc758 \ucca0\uc800\ud55c \uacc4\ud68d \uc544\ub798 \uc2e4\uc2dc\ub418\uc5c8\ub2e4\ub294 \ubd84\uc11d\uc774 \uc9c0\ubc30\uc801\uc774\ub2e4. \uadf8 \uc774\uc720\ub294 \ub300\ud55c\ubbfc\uad6d \uad6d\uad70\uc774 \uc870\uc120\ubbfc\uc8fc\uc8fc\uc758\uc778\ubbfc\uacf5\ud654\uad6d\uc5d0 \uc11c\ud574\uc0c1\uc758 \uad70\uc0ac\ud6c8\ub828\uc744 \ud1b5\ubcf4\ud55c \ubc14 \uc788\uae30 \ub54c\ubb38\uc5d0 \uae40\uc815\uc77c\uc758 \ub3d9\uc758 \ud558\uc5d0 \uc774\ub8e8\uc5b4\uc84c\uc744 \uac00\ub2a5\uc131\uc774 \ub192\ub2e4\ub294 \uac83\uc774\ub2e4. \ub610\ud55c \uc870\uc120\ubbfc\uc8fc\uc8fc\uc758\uc778\ubbfc\uacf5\ud654\uad6d \uae40\uc815\uc77c \uad6d\ubc29\uc704\uc6d0\uc7a5\uc758 \ud6c4\uacc4\uc790 \uae40\uc815\uc740\uc774 \uc5f0\ud3c9\ub3c4 \ub3c4\ubc1c \uc900\ube44\ub97c \uc9c0\ub09c\ub2ec\uc5d0 \uc774\ubbf8 \uc9c0\uc2dc\ud55c \uc815\ud669\ub3c4 \uc788\uc5c8\ub294\ub370, \uc77c\ubcf8 \uc544\uc0ac\ud788 \uc2e0\ubb38\uc740 12\uc6d4 1\uc77c \ubd81\ud55c \uc18c\uc2dd\ud1b5\uc744 \uc778\uc6a9\ud574 \u201c\uae40\uc815\uc740\uc758 \uc774\ub984\uc73c\ub85c \uc9c0\ub09c\ub2ec \ucd08 \u2018\uc801\uc758 \ub3c4\ubc1c \ud589\uc704\uc5d0 \uc5b8\uc81c\ub77c\ub3c4 \ubc18\uaca9\ud560 \uc218 \uc788\ub294 \ud0dc\uc138\ub97c \uac16\ucd94\ub77c\u2019\ub294 \uc9c0\ub839\uc774 \uc870\uc120\uc778\ubbfc\uad70 \uac04\ubd80\ub4e4\uc5d0\uac8c \ud558\ub2ec\ub410\ub2e4\u201d\uace0 \ubcf4\ub3c4\ud588\ub2e4. \uc774\uac83\uc774 \uc0ac\uc2e4\uc774\ub77c\uba74 \uc5f0\ud3c9\ub3c4 \ud3ec\uaca9\uc740 \uc608\uc815\ub410\ub358 \ud589\ub3d9\uc774\uba70 '\ucda9\ubd84\ud55c \uae30\uac04\uc744 \uac00\uc9c0\uace0 \uc900\ube44'\ud55c \uac83\uc73c\ub85c, \uc870\uc120\uc778\ubbfc\uad70\uc774 \uae40\uc815\uc740\uc758 \uc9c0\uc2dc\ub97c \ubc1b\uace0 \uacf5\uaca9 \uae30\ud68c\ub97c \uc5ff\ubcf4\uace0 \uc788\ub2e4\uac00 \ub300\ud55c\ubbfc\uad6d \uad6d\uad70\uc758 \uc9c0\ub09c 23\uc77c \ud6c8\ub828\uc744 \uad6c\uc2e4\ub85c \ud3ec\uaca9\uc744 \uac10\ud589\ud588\uc744 \uac00\ub2a5\uc131\uc774 \ub192\uc73c\uba70, \uc5f0\ud3c9\ub3c4 \ubb34\ub825\uacf5\uaca9 \uc9c1\uc804 \uc870\uc120\ubbfc\uc8fc\uc8fc\uc758\uc778\ubbfc\uacf5\ud654\uad6d\uc758 \ubbf8\uadf8-23\uae30 5\ub300\uac00 \uc11c\ud574 5\ub3c4 \uc778\uadfc\uc5d0\uc11c \ucd08\uacc4\ube44\ud589\uc744 \ubc8c\uc600\ub358 \uc0ac\uc2e4\ub3c4 \ub4dc\ub7ec\ub0ac\ub2e4. \ub610\ud55c \uae40\uc815\uc740 \ud6c4\uacc4 \uad6c\ub3c4\ub97c \ub354\uc6b1 \ud655\ub9bd \ud558\uae30 \uc704\ud55c \uac83\uc774\ub77c\ub294 \ubd84\uc11d\uc774 \uc81c\uae30\ub41c\ub2e4. \ud2b9\ud788 \uae40\uc815\uc740\uc740 \ubd81\uce21\uc5d0\uc11c \ud3ec\ubcd1 \uc804\ubb38\uac00\ub85c \ubbf8\ud654\ub418\uace0 \uc788\ub2e4\ub294 \uc810\uc5d0\uc11c \uadf8\uc758 \uc5c5\uc801\uc744 \uadf9\ub300\ud654 \ud558\uae30 \uc704\ud55c \uac83\uc774\ub77c\ub294 \uc8fc\uc7a5\uc774 \uc124\ub4dd\ub825\uc744 \uc5bb\uace0 \uc788\ub2e4.", "sentence_answer": "\ub610\ud55c \uae40\uc815\uc740 \ud6c4\uacc4 \uad6c\ub3c4\ub97c \ub354\uc6b1 \ud655\ub9bd \ud558\uae30 \uc704\ud55c \uac83\uc774\ub77c\ub294 \ubd84\uc11d\uc774 \uc81c\uae30\ub41c\ub2e4.", "paragraph_id": "6465630-2-1", "paragraph_question": "question: \uc5f0\ud3c9\ub3c4 \ud3ec\uaca9\uc758 \uc774\uc720\uc5d0 \ub300\ud55c \uac00\uc7a5 \uc124\ub4dd\ub825\uc788\ub294 \uc8fc\uc7a5\uc740?, context: 2010\ub144 11\uc6d4 23\uc77c\uc758 \uc5f0\ud3c9\ub3c4 \ud3ec\uaca9 \uc0ac\uac74\uc740 \uc870\uc120\ubbfc\uc8fc\uc8fc\uc758\uc778\ubbfc\uacf5\ud654\uad6d\uc758 \ucca0\uc800\ud55c \uacc4\ud68d \uc544\ub798 \uc2e4\uc2dc\ub418\uc5c8\ub2e4\ub294 \ubd84\uc11d\uc774 \uc9c0\ubc30\uc801\uc774\ub2e4. \uadf8 \uc774\uc720\ub294 \ub300\ud55c\ubbfc\uad6d \uad6d\uad70\uc774 \uc870\uc120\ubbfc\uc8fc\uc8fc\uc758\uc778\ubbfc\uacf5\ud654\uad6d\uc5d0 \uc11c\ud574\uc0c1\uc758 \uad70\uc0ac\ud6c8\ub828\uc744 \ud1b5\ubcf4\ud55c \ubc14 \uc788\uae30 \ub54c\ubb38\uc5d0 \uae40\uc815\uc77c\uc758 \ub3d9\uc758 \ud558\uc5d0 \uc774\ub8e8\uc5b4\uc84c\uc744 \uac00\ub2a5\uc131\uc774 \ub192\ub2e4\ub294 \uac83\uc774\ub2e4. \ub610\ud55c \uc870\uc120\ubbfc\uc8fc\uc8fc\uc758\uc778\ubbfc\uacf5\ud654\uad6d \uae40\uc815\uc77c \uad6d\ubc29\uc704\uc6d0\uc7a5\uc758 \ud6c4\uacc4\uc790 \uae40\uc815\uc740\uc774 \uc5f0\ud3c9\ub3c4 \ub3c4\ubc1c \uc900\ube44\ub97c \uc9c0\ub09c\ub2ec\uc5d0 \uc774\ubbf8 \uc9c0\uc2dc\ud55c \uc815\ud669\ub3c4 \uc788\uc5c8\ub294\ub370, \uc77c\ubcf8 \uc544\uc0ac\ud788 \uc2e0\ubb38\uc740 12\uc6d4 1\uc77c \ubd81\ud55c \uc18c\uc2dd\ud1b5\uc744 \uc778\uc6a9\ud574 \u201c\uae40\uc815\uc740\uc758 \uc774\ub984\uc73c\ub85c \uc9c0\ub09c\ub2ec \ucd08 \u2018\uc801\uc758 \ub3c4\ubc1c \ud589\uc704\uc5d0 \uc5b8\uc81c\ub77c\ub3c4 \ubc18\uaca9\ud560 \uc218 \uc788\ub294 \ud0dc\uc138\ub97c \uac16\ucd94\ub77c\u2019\ub294 \uc9c0\ub839\uc774 \uc870\uc120\uc778\ubbfc\uad70 \uac04\ubd80\ub4e4\uc5d0\uac8c \ud558\ub2ec\ub410\ub2e4\u201d\uace0 \ubcf4\ub3c4\ud588\ub2e4. \uc774\uac83\uc774 \uc0ac\uc2e4\uc774\ub77c\uba74 \uc5f0\ud3c9\ub3c4 \ud3ec\uaca9\uc740 \uc608\uc815\ub410\ub358 \ud589\ub3d9\uc774\uba70 '\ucda9\ubd84\ud55c \uae30\uac04\uc744 \uac00\uc9c0\uace0 \uc900\ube44'\ud55c \uac83\uc73c\ub85c, \uc870\uc120\uc778\ubbfc\uad70\uc774 \uae40\uc815\uc740\uc758 \uc9c0\uc2dc\ub97c \ubc1b\uace0 \uacf5\uaca9 \uae30\ud68c\ub97c \uc5ff\ubcf4\uace0 \uc788\ub2e4\uac00 \ub300\ud55c\ubbfc\uad6d \uad6d\uad70\uc758 \uc9c0\ub09c 23\uc77c \ud6c8\ub828\uc744 \uad6c\uc2e4\ub85c \ud3ec\uaca9\uc744 \uac10\ud589\ud588\uc744 \uac00\ub2a5\uc131\uc774 \ub192\uc73c\uba70, \uc5f0\ud3c9\ub3c4 \ubb34\ub825\uacf5\uaca9 \uc9c1\uc804 \uc870\uc120\ubbfc\uc8fc\uc8fc\uc758\uc778\ubbfc\uacf5\ud654\uad6d\uc758 \ubbf8\uadf8-23\uae30 5\ub300\uac00 \uc11c\ud574 5\ub3c4 \uc778\uadfc\uc5d0\uc11c \ucd08\uacc4\ube44\ud589\uc744 \ubc8c\uc600\ub358 \uc0ac\uc2e4\ub3c4 \ub4dc\ub7ec\ub0ac\ub2e4. \ub610\ud55c \uae40\uc815\uc740 \ud6c4\uacc4 \uad6c\ub3c4\ub97c \ub354\uc6b1 \ud655\ub9bd\ud558\uae30 \uc704\ud55c \uac83\uc774\ub77c\ub294 \ubd84\uc11d\uc774 \uc81c\uae30\ub41c\ub2e4. \ud2b9\ud788 \uae40\uc815\uc740\uc740 \ubd81\uce21\uc5d0\uc11c \ud3ec\ubcd1 \uc804\ubb38\uac00\ub85c \ubbf8\ud654\ub418\uace0 \uc788\ub2e4\ub294 \uc810\uc5d0\uc11c \uadf8\uc758 \uc5c5\uc801\uc744 \uadf9\ub300\ud654 \ud558\uae30 \uc704\ud55c \uac83\uc774\ub77c\ub294 \uc8fc\uc7a5\uc774 \uc124\ub4dd\ub825\uc744 \uc5bb\uace0 \uc788\ub2e4."} {"question": "\uc774\uc81c \uc0dd\uacc4\uc218\uc785\uc774 \uc788\uace0 \uc758\ub8cc\ubcf4\ud5d8\uc5d0 \uac00\uc785\ub41c \ud53c\ub09c\ubbfc\uc740 \uba87 \ub144\uac04\uc758 \uccb4\ub958\ud5c8\uac00\ub97c \ubc1b\uc744 \uc218 \uc788\ub294\uac00?", "paragraph": "\uc678\uad6d\uc778\ubc95\uacfc \ud53c\ub09c\ubbfc\ubc95\uc5d0\uc11c \uc218\ub9ce\uc740 \uac1c\ud601, \uc218\uc815\ud560 \ubd80\ubb38 \uc911 \uc815\ubd80\uc5f0\uc815 \uacc4\uc57d\uc5d0\ub294 \ud2b9\ud788 \ub2e4\uc74c\uc758 \ub450 \uac00\uc9c0 \uc548\uc774 \uc911\uc694\ud558\uac8c \ub2e4\ub8e8\uc5b4\uc9c0\uace0 \uc788\ub2e4. \uc678\uad6d\uc778\ub4e4\uc774 \ucde8\uc5c5\ud560 \uacbd\uc6b0 \ub178\ub3d9\ud5c8\uac00\ub97c \uc5bb\uae30 \uc704\ud574 \uc544\uc8fc \ubcf5\uc7a1\ud55c \ubc95\uc801\uc808\ucc28\ub97c \ubc1f\uc544\uc57c \ud558\ub294\ub370, \uc77c\uc790\ub9ac\uac00 \uc788\uc5b4\ub3c4(\ub300\ubd80\ubd84 \ub3c5\uc77c\uc778\uc774 \uc6d0\ud558\uc9c0 \uc54a\ub294) \ub178\ub3d9\ud5c8\uac00\uac00 \uc5c6\uc5b4\uc11c \ucde8\uc5c5\uc744 \ubabb\ud558\ub294 \uacbd\uc6b0\uac00 \ub9ce\ub2e4. \ud2b9\ud788, \uc774\ubbfc\uc790\ub4e4\uc744 \ub530\ub77c\uc624\ub294 \uac00\uc871\uc774\ub098 \ud53c\ub09c\ubbfc\ub4e4\uc774 \uc774 \uacbd\uc6b0\uc5d0 \ud574\ub2f9\ud55c\ub2e4. \uad6d\uac00\ub294 \uc815\ucc45\uc0c1 \uc774 \uc2e4\uc5c5\uc790\ub4e4\uc5d0\uac8c\ub3c4 \uc0ac\ud68c\ubcf5\uc9c0 \ubcf4\uc870\uae08\uc744 \uc9c0\uc6d0\ud574\uc57c \ud558\ubbc0\ub85c \uad6d\uac00\ub098 \ub2f9\uc0ac\uc790\ub4e4\uc5d0\uac8c \ubc14\ub78c\uc9c1\ud55c \uc0c1\ud669\uc774 \uc544\ub2c8\uae30 \ub54c\ubb38\uc5d0 \ubc95 \uc218\uc815\uc744 \ud558\uace0 \uc788\uc73c\ub098, \ub178\ub3d9\uc2dc\uc7a5\uacfc \uad00\ub828\ub41c \uc608\ubbfc\ud55c \ubb38\uc81c\uc774\ubbc0\ub85c \uc815\ubd80\ub3c4 \uc870\uc2ec\uc2a4\ub7fd\uac8c \uac1c\ud601\uc744 \ucd94\uc9c4\ud558\uace0 \uc788\ub2e4. \ub3c5\uc77c\uc5d0 \uac70\uc8fc\ud558\ub294 \ud53c\ub09c\ubbfc \uc911 \ub9ce\uc740 \uc0ac\ub78c\ub4e4\uc774 \ub3c5\uc77c\uc5d0 \uc774\ubbf8 10\ub144 \uc774\uc0c1\uc744 \uac70\uc8fc\ud558\uba70 \uac00\uc871\ub3c4 \uc788\uace0, \uc6d0\ub798 \uace0\ud5a5\uc73c\ub85c \ub3cc\uc544\uac00\ub294 \uac83\uc774 \ud798\ub4e4 \uacbd\uc6b0\uac00 \ub9ce\ub2e4(\ubca0\ud2b8\ub0a8 \ub09c\ubbfc\ub4e4). \uc774\ub4e4\uc740 3~6\uac1c\uc6d4\ub9c8\ub2e4 \uccb4\ub958 \uc5f0\uc7a5\uc744 \uc704\ud574 \uc2ec\ud55c \uace0\ud1b5\uc744 \ubc1b\uace0 \uc788\uc5c8\ub2e4. \uadf8\ub7ec\ub098 \uc774\uc81c\ub294 \uc0dd\uacc4\uc218\uc785\uc774 \uc788\uace0 \uc758\ub8cc\ubcf4\ud5d8\uc5d0 \uac00\uc785\ub418\uc5b4\uc788\uc73c\uba74, 1\ub144\uac04\uc758 \uccb4\ub958\ud5c8\uac00\ub97c \ubc1b\uc744 \uc218 \uc788\uac8c \ub418\uc5c8\ub2e4.", "answer": "1\ub144", "sentence": "\uadf8\ub7ec\ub098 \uc774\uc81c\ub294 \uc0dd\uacc4\uc218\uc785\uc774 \uc788\uace0 \uc758\ub8cc\ubcf4\ud5d8\uc5d0 \uac00\uc785\ub418\uc5b4\uc788\uc73c\uba74, 1\ub144 \uac04\uc758 \uccb4\ub958\ud5c8\uac00\ub97c \ubc1b\uc744 \uc218 \uc788\uac8c \ub418\uc5c8\ub2e4.", "paragraph_sentence": "\uc678\uad6d\uc778\ubc95\uacfc \ud53c\ub09c\ubbfc\ubc95\uc5d0\uc11c \uc218\ub9ce\uc740 \uac1c\ud601, \uc218\uc815\ud560 \ubd80\ubb38 \uc911 \uc815\ubd80\uc5f0\uc815 \uacc4\uc57d\uc5d0\ub294 \ud2b9\ud788 \ub2e4\uc74c\uc758 \ub450 \uac00\uc9c0 \uc548\uc774 \uc911\uc694\ud558\uac8c \ub2e4\ub8e8\uc5b4\uc9c0\uace0 \uc788\ub2e4. \uc678\uad6d\uc778\ub4e4\uc774 \ucde8\uc5c5\ud560 \uacbd\uc6b0 \ub178\ub3d9\ud5c8\uac00\ub97c \uc5bb\uae30 \uc704\ud574 \uc544\uc8fc \ubcf5\uc7a1\ud55c \ubc95\uc801\uc808\ucc28\ub97c \ubc1f\uc544\uc57c \ud558\ub294\ub370, \uc77c\uc790\ub9ac\uac00 \uc788\uc5b4\ub3c4(\ub300\ubd80\ubd84 \ub3c5\uc77c\uc778\uc774 \uc6d0\ud558\uc9c0 \uc54a\ub294) \ub178\ub3d9\ud5c8\uac00\uac00 \uc5c6\uc5b4\uc11c \ucde8\uc5c5\uc744 \ubabb\ud558\ub294 \uacbd\uc6b0\uac00 \ub9ce\ub2e4. \ud2b9\ud788, \uc774\ubbfc\uc790\ub4e4\uc744 \ub530\ub77c\uc624\ub294 \uac00\uc871\uc774\ub098 \ud53c\ub09c\ubbfc\ub4e4\uc774 \uc774 \uacbd\uc6b0\uc5d0 \ud574\ub2f9\ud55c\ub2e4. \uad6d\uac00\ub294 \uc815\ucc45\uc0c1 \uc774 \uc2e4\uc5c5\uc790\ub4e4\uc5d0\uac8c\ub3c4 \uc0ac\ud68c\ubcf5\uc9c0 \ubcf4\uc870\uae08\uc744 \uc9c0\uc6d0\ud574\uc57c \ud558\ubbc0\ub85c \uad6d\uac00\ub098 \ub2f9\uc0ac\uc790\ub4e4\uc5d0\uac8c \ubc14\ub78c\uc9c1\ud55c \uc0c1\ud669\uc774 \uc544\ub2c8\uae30 \ub54c\ubb38\uc5d0 \ubc95 \uc218\uc815\uc744 \ud558\uace0 \uc788\uc73c\ub098, \ub178\ub3d9\uc2dc\uc7a5\uacfc \uad00\ub828\ub41c \uc608\ubbfc\ud55c \ubb38\uc81c\uc774\ubbc0\ub85c \uc815\ubd80\ub3c4 \uc870\uc2ec\uc2a4\ub7fd\uac8c \uac1c\ud601\uc744 \ucd94\uc9c4\ud558\uace0 \uc788\ub2e4. \ub3c5\uc77c\uc5d0 \uac70\uc8fc\ud558\ub294 \ud53c\ub09c\ubbfc \uc911 \ub9ce\uc740 \uc0ac\ub78c\ub4e4\uc774 \ub3c5\uc77c\uc5d0 \uc774\ubbf8 10\ub144 \uc774\uc0c1\uc744 \uac70\uc8fc\ud558\uba70 \uac00\uc871\ub3c4 \uc788\uace0, \uc6d0\ub798 \uace0\ud5a5\uc73c\ub85c \ub3cc\uc544\uac00\ub294 \uac83\uc774 \ud798\ub4e4 \uacbd\uc6b0\uac00 \ub9ce\ub2e4(\ubca0\ud2b8\ub0a8 \ub09c\ubbfc\ub4e4). \uc774\ub4e4\uc740 3~6\uac1c\uc6d4\ub9c8\ub2e4 \uccb4\ub958 \uc5f0\uc7a5\uc744 \uc704\ud574 \uc2ec\ud55c \uace0\ud1b5\uc744 \ubc1b\uace0 \uc788\uc5c8\ub2e4. \uadf8\ub7ec\ub098 \uc774\uc81c\ub294 \uc0dd\uacc4\uc218\uc785\uc774 \uc788\uace0 \uc758\ub8cc\ubcf4\ud5d8\uc5d0 \uac00\uc785\ub418\uc5b4\uc788\uc73c\uba74, 1\ub144 \uac04\uc758 \uccb4\ub958\ud5c8\uac00\ub97c \ubc1b\uc744 \uc218 \uc788\uac8c \ub418\uc5c8\ub2e4. ", "paragraph_answer": "\uc678\uad6d\uc778\ubc95\uacfc \ud53c\ub09c\ubbfc\ubc95\uc5d0\uc11c \uc218\ub9ce\uc740 \uac1c\ud601, \uc218\uc815\ud560 \ubd80\ubb38 \uc911 \uc815\ubd80\uc5f0\uc815 \uacc4\uc57d\uc5d0\ub294 \ud2b9\ud788 \ub2e4\uc74c\uc758 \ub450 \uac00\uc9c0 \uc548\uc774 \uc911\uc694\ud558\uac8c \ub2e4\ub8e8\uc5b4\uc9c0\uace0 \uc788\ub2e4. \uc678\uad6d\uc778\ub4e4\uc774 \ucde8\uc5c5\ud560 \uacbd\uc6b0 \ub178\ub3d9\ud5c8\uac00\ub97c \uc5bb\uae30 \uc704\ud574 \uc544\uc8fc \ubcf5\uc7a1\ud55c \ubc95\uc801\uc808\ucc28\ub97c \ubc1f\uc544\uc57c \ud558\ub294\ub370, \uc77c\uc790\ub9ac\uac00 \uc788\uc5b4\ub3c4(\ub300\ubd80\ubd84 \ub3c5\uc77c\uc778\uc774 \uc6d0\ud558\uc9c0 \uc54a\ub294) \ub178\ub3d9\ud5c8\uac00\uac00 \uc5c6\uc5b4\uc11c \ucde8\uc5c5\uc744 \ubabb\ud558\ub294 \uacbd\uc6b0\uac00 \ub9ce\ub2e4. \ud2b9\ud788, \uc774\ubbfc\uc790\ub4e4\uc744 \ub530\ub77c\uc624\ub294 \uac00\uc871\uc774\ub098 \ud53c\ub09c\ubbfc\ub4e4\uc774 \uc774 \uacbd\uc6b0\uc5d0 \ud574\ub2f9\ud55c\ub2e4. \uad6d\uac00\ub294 \uc815\ucc45\uc0c1 \uc774 \uc2e4\uc5c5\uc790\ub4e4\uc5d0\uac8c\ub3c4 \uc0ac\ud68c\ubcf5\uc9c0 \ubcf4\uc870\uae08\uc744 \uc9c0\uc6d0\ud574\uc57c \ud558\ubbc0\ub85c \uad6d\uac00\ub098 \ub2f9\uc0ac\uc790\ub4e4\uc5d0\uac8c \ubc14\ub78c\uc9c1\ud55c \uc0c1\ud669\uc774 \uc544\ub2c8\uae30 \ub54c\ubb38\uc5d0 \ubc95 \uc218\uc815\uc744 \ud558\uace0 \uc788\uc73c\ub098, \ub178\ub3d9\uc2dc\uc7a5\uacfc \uad00\ub828\ub41c \uc608\ubbfc\ud55c \ubb38\uc81c\uc774\ubbc0\ub85c \uc815\ubd80\ub3c4 \uc870\uc2ec\uc2a4\ub7fd\uac8c \uac1c\ud601\uc744 \ucd94\uc9c4\ud558\uace0 \uc788\ub2e4. \ub3c5\uc77c\uc5d0 \uac70\uc8fc\ud558\ub294 \ud53c\ub09c\ubbfc \uc911 \ub9ce\uc740 \uc0ac\ub78c\ub4e4\uc774 \ub3c5\uc77c\uc5d0 \uc774\ubbf8 10\ub144 \uc774\uc0c1\uc744 \uac70\uc8fc\ud558\uba70 \uac00\uc871\ub3c4 \uc788\uace0, \uc6d0\ub798 \uace0\ud5a5\uc73c\ub85c \ub3cc\uc544\uac00\ub294 \uac83\uc774 \ud798\ub4e4 \uacbd\uc6b0\uac00 \ub9ce\ub2e4(\ubca0\ud2b8\ub0a8 \ub09c\ubbfc\ub4e4). \uc774\ub4e4\uc740 3~6\uac1c\uc6d4\ub9c8\ub2e4 \uccb4\ub958 \uc5f0\uc7a5\uc744 \uc704\ud574 \uc2ec\ud55c \uace0\ud1b5\uc744 \ubc1b\uace0 \uc788\uc5c8\ub2e4. \uadf8\ub7ec\ub098 \uc774\uc81c\ub294 \uc0dd\uacc4\uc218\uc785\uc774 \uc788\uace0 \uc758\ub8cc\ubcf4\ud5d8\uc5d0 \uac00\uc785\ub418\uc5b4\uc788\uc73c\uba74, 1\ub144 \uac04\uc758 \uccb4\ub958\ud5c8\uac00\ub97c \ubc1b\uc744 \uc218 \uc788\uac8c \ub418\uc5c8\ub2e4.", "sentence_answer": "\uadf8\ub7ec\ub098 \uc774\uc81c\ub294 \uc0dd\uacc4\uc218\uc785\uc774 \uc788\uace0 \uc758\ub8cc\ubcf4\ud5d8\uc5d0 \uac00\uc785\ub418\uc5b4\uc788\uc73c\uba74, 1\ub144 \uac04\uc758 \uccb4\ub958\ud5c8\uac00\ub97c \ubc1b\uc744 \uc218 \uc788\uac8c \ub418\uc5c8\ub2e4.", "paragraph_id": "6528734-7-1", "paragraph_question": "question: \uc774\uc81c \uc0dd\uacc4\uc218\uc785\uc774 \uc788\uace0 \uc758\ub8cc\ubcf4\ud5d8\uc5d0 \uac00\uc785\ub41c \ud53c\ub09c\ubbfc\uc740 \uba87 \ub144\uac04\uc758 \uccb4\ub958\ud5c8\uac00\ub97c \ubc1b\uc744 \uc218 \uc788\ub294\uac00?, context: \uc678\uad6d\uc778\ubc95\uacfc \ud53c\ub09c\ubbfc\ubc95\uc5d0\uc11c \uc218\ub9ce\uc740 \uac1c\ud601, \uc218\uc815\ud560 \ubd80\ubb38 \uc911 \uc815\ubd80\uc5f0\uc815 \uacc4\uc57d\uc5d0\ub294 \ud2b9\ud788 \ub2e4\uc74c\uc758 \ub450 \uac00\uc9c0 \uc548\uc774 \uc911\uc694\ud558\uac8c \ub2e4\ub8e8\uc5b4\uc9c0\uace0 \uc788\ub2e4. \uc678\uad6d\uc778\ub4e4\uc774 \ucde8\uc5c5\ud560 \uacbd\uc6b0 \ub178\ub3d9\ud5c8\uac00\ub97c \uc5bb\uae30 \uc704\ud574 \uc544\uc8fc \ubcf5\uc7a1\ud55c \ubc95\uc801\uc808\ucc28\ub97c \ubc1f\uc544\uc57c \ud558\ub294\ub370, \uc77c\uc790\ub9ac\uac00 \uc788\uc5b4\ub3c4(\ub300\ubd80\ubd84 \ub3c5\uc77c\uc778\uc774 \uc6d0\ud558\uc9c0 \uc54a\ub294) \ub178\ub3d9\ud5c8\uac00\uac00 \uc5c6\uc5b4\uc11c \ucde8\uc5c5\uc744 \ubabb\ud558\ub294 \uacbd\uc6b0\uac00 \ub9ce\ub2e4. \ud2b9\ud788, \uc774\ubbfc\uc790\ub4e4\uc744 \ub530\ub77c\uc624\ub294 \uac00\uc871\uc774\ub098 \ud53c\ub09c\ubbfc\ub4e4\uc774 \uc774 \uacbd\uc6b0\uc5d0 \ud574\ub2f9\ud55c\ub2e4. \uad6d\uac00\ub294 \uc815\ucc45\uc0c1 \uc774 \uc2e4\uc5c5\uc790\ub4e4\uc5d0\uac8c\ub3c4 \uc0ac\ud68c\ubcf5\uc9c0 \ubcf4\uc870\uae08\uc744 \uc9c0\uc6d0\ud574\uc57c \ud558\ubbc0\ub85c \uad6d\uac00\ub098 \ub2f9\uc0ac\uc790\ub4e4\uc5d0\uac8c \ubc14\ub78c\uc9c1\ud55c \uc0c1\ud669\uc774 \uc544\ub2c8\uae30 \ub54c\ubb38\uc5d0 \ubc95 \uc218\uc815\uc744 \ud558\uace0 \uc788\uc73c\ub098, \ub178\ub3d9\uc2dc\uc7a5\uacfc \uad00\ub828\ub41c \uc608\ubbfc\ud55c \ubb38\uc81c\uc774\ubbc0\ub85c \uc815\ubd80\ub3c4 \uc870\uc2ec\uc2a4\ub7fd\uac8c \uac1c\ud601\uc744 \ucd94\uc9c4\ud558\uace0 \uc788\ub2e4. \ub3c5\uc77c\uc5d0 \uac70\uc8fc\ud558\ub294 \ud53c\ub09c\ubbfc \uc911 \ub9ce\uc740 \uc0ac\ub78c\ub4e4\uc774 \ub3c5\uc77c\uc5d0 \uc774\ubbf8 10\ub144 \uc774\uc0c1\uc744 \uac70\uc8fc\ud558\uba70 \uac00\uc871\ub3c4 \uc788\uace0, \uc6d0\ub798 \uace0\ud5a5\uc73c\ub85c \ub3cc\uc544\uac00\ub294 \uac83\uc774 \ud798\ub4e4 \uacbd\uc6b0\uac00 \ub9ce\ub2e4(\ubca0\ud2b8\ub0a8 \ub09c\ubbfc\ub4e4). \uc774\ub4e4\uc740 3~6\uac1c\uc6d4\ub9c8\ub2e4 \uccb4\ub958 \uc5f0\uc7a5\uc744 \uc704\ud574 \uc2ec\ud55c \uace0\ud1b5\uc744 \ubc1b\uace0 \uc788\uc5c8\ub2e4. \uadf8\ub7ec\ub098 \uc774\uc81c\ub294 \uc0dd\uacc4\uc218\uc785\uc774 \uc788\uace0 \uc758\ub8cc\ubcf4\ud5d8\uc5d0 \uac00\uc785\ub418\uc5b4\uc788\uc73c\uba74, 1\ub144\uac04\uc758 \uccb4\ub958\ud5c8\uac00\ub97c \ubc1b\uc744 \uc218 \uc788\uac8c \ub418\uc5c8\ub2e4."} {"question": "\uc774\uc774\uac00 \ub2e4\uc2dc \uc7ac\uac00\uc0dd\ud65c\ub85c \ub3cc\uc544\uc654\uc744 \ub54c \ubc1b\uc544\uc900 \uc778\ubb3c\ub85c \ubc31\uc778\uac78\uacfc \ub2e4\ub978 \ud55c\uba85\uc740 \ub204\uad6c\uc778\uac00?", "paragraph": "\uc5b4\uba38\ub2c8 \uc2e0\uc0ac\uc784\ub2f9\uc758 \uc624\ub79c \ubcd1\ud658\uacfc \uc8fd\uc74c\uc740 \uadf8\uc5d0\uac8c \uc2ec\uc801, \uc815\uc2e0\uc801 \ucda9\uaca9\uc744 \uc8fc\uc5c8\ub2e4. \uadf8\ub294 \uc0ac\ub78c\uc774 \uc65c \ud0dc\uc5b4\ub098\uace0 \uc8fd\ub294\uac00\uc5d0 \ub300\ud574 \uacc4\uc18d \uace0\ubbfc\ud558\uba74\uc11c \ud55c\ub3d9\uc548 \ubc29\ud669\ud558\uac8c \ub41c\ub2e4. \uacb0\uad6d \uc2dc\ubb18\uc0b4\uc774\ub97c \ub9c8\uce5c \ub4a4 \uae08\uac15\uc0b0\uc73c\ub85c \ub4e4\uc5b4\uac00 \uc2b9\ub824\uac00 \ub418\uc5c8\uace0, \uadf8\uac00 \ub4a4\uc5d0 \ubd88\uad50\uc5d0 \uc785\ubb38\ud588\ub2e4\uac00 \ud658\uc18d\ud55c \ub4a4\uc5d0\ub3c4 \ubb38\uc81c \uc0bc\uc9c0 \uc54a\uace0 \ubc1b\uc544\uc900 \uac83\uc740 \uc2a4\uc2b9 \ubc31\uc778\uac78\uacfc \uc624\ub79c \uce5c\uad6c \uc131\ud63c\uc774\uc5c8\ub2e4. \uadf8\ub7ec\ub098 \uc774\ub54c\uc758 \uc785\uc0b0 \uacbd\ub825\uc740 \uadf8\uc758 \uc0dd\uc804\uc5d0\ub3c4 \uc1a1\uc751\uac1c \ub4f1\uc758 \ub3d9\uc778(\u6771\u4eba)\ub4e4\uacfc \ud5c8\ubaa9, \uc724\ud734, \uc724\uc120\ub3c4 \ub4f1\uc758 \ub0a8\uc778(\u5357\u4eba) \ub2f9\uc6d0\ub4e4\uc5d0\uac8c \uc774\ub2e8 \ud559\ubb38\uc5d0 \ube60\uc84c\ub2e4\ub294 \uc774\uc720\ub85c \uc0ac\uc0c1\uacf5\uc138\ub97c \ub2f9\ud558\ub294 \uc6d0\uc778\uc774 \ub41c\ub2e4. \uc774\uc774\uac00 \uc2b9\ub824\uc774\uba70 \ubd88\uad50\ub3c4\ub77c\ub294 \ub3d9\uc778, \ub0a8\uc778, \ubd81\uc778\uacc4\uc5f4 \uc720\ud559\uc790(\u5112\u5b78\u8005)\ub4e4\uc758 \uc0ac\uc0c1\uacf5\uc138\ub294 1910\ub144(\uc735\ud76c 4\ub144) \uc870\uc120\uc774 \uba78\ub9dd\ud560 \ub54c\uae4c\uc9c0\ub3c4 \uc9c0\uc18d\ub41c\ub2e4.", "answer": "\uc131\ud63c", "sentence": "\uacb0\uad6d \uc2dc\ubb18\uc0b4\uc774\ub97c \ub9c8\uce5c \ub4a4 \uae08\uac15\uc0b0\uc73c\ub85c \ub4e4\uc5b4\uac00 \uc2b9\ub824\uac00 \ub418\uc5c8\uace0, \uadf8\uac00 \ub4a4\uc5d0 \ubd88\uad50\uc5d0 \uc785\ubb38\ud588\ub2e4\uac00 \ud658\uc18d\ud55c \ub4a4\uc5d0\ub3c4 \ubb38\uc81c \uc0bc\uc9c0 \uc54a\uace0 \ubc1b\uc544\uc900 \uac83\uc740 \uc2a4\uc2b9 \ubc31\uc778\uac78\uacfc \uc624\ub79c \uce5c\uad6c \uc131\ud63c \uc774\uc5c8\ub2e4.", "paragraph_sentence": "\uc5b4\uba38\ub2c8 \uc2e0\uc0ac\uc784\ub2f9\uc758 \uc624\ub79c \ubcd1\ud658\uacfc \uc8fd\uc74c\uc740 \uadf8\uc5d0\uac8c \uc2ec\uc801, \uc815\uc2e0\uc801 \ucda9\uaca9\uc744 \uc8fc\uc5c8\ub2e4. \uadf8\ub294 \uc0ac\ub78c\uc774 \uc65c \ud0dc\uc5b4\ub098\uace0 \uc8fd\ub294\uac00\uc5d0 \ub300\ud574 \uacc4\uc18d \uace0\ubbfc\ud558\uba74\uc11c \ud55c\ub3d9\uc548 \ubc29\ud669\ud558\uac8c \ub41c\ub2e4. \uacb0\uad6d \uc2dc\ubb18\uc0b4\uc774\ub97c \ub9c8\uce5c \ub4a4 \uae08\uac15\uc0b0\uc73c\ub85c \ub4e4\uc5b4\uac00 \uc2b9\ub824\uac00 \ub418\uc5c8\uace0, \uadf8\uac00 \ub4a4\uc5d0 \ubd88\uad50\uc5d0 \uc785\ubb38\ud588\ub2e4\uac00 \ud658\uc18d\ud55c \ub4a4\uc5d0\ub3c4 \ubb38\uc81c \uc0bc\uc9c0 \uc54a\uace0 \ubc1b\uc544\uc900 \uac83\uc740 \uc2a4\uc2b9 \ubc31\uc778\uac78\uacfc \uc624\ub79c \uce5c\uad6c \uc131\ud63c \uc774\uc5c8\ub2e4. \uadf8\ub7ec\ub098 \uc774\ub54c\uc758 \uc785\uc0b0 \uacbd\ub825\uc740 \uadf8\uc758 \uc0dd\uc804\uc5d0\ub3c4 \uc1a1\uc751\uac1c \ub4f1\uc758 \ub3d9\uc778(\u6771\u4eba)\ub4e4\uacfc \ud5c8\ubaa9, \uc724\ud734, \uc724\uc120\ub3c4 \ub4f1\uc758 \ub0a8\uc778(\u5357\u4eba) \ub2f9\uc6d0\ub4e4\uc5d0\uac8c \uc774\ub2e8 \ud559\ubb38\uc5d0 \ube60\uc84c\ub2e4\ub294 \uc774\uc720\ub85c \uc0ac\uc0c1\uacf5\uc138\ub97c \ub2f9\ud558\ub294 \uc6d0\uc778\uc774 \ub41c\ub2e4. \uc774\uc774\uac00 \uc2b9\ub824\uc774\uba70 \ubd88\uad50\ub3c4\ub77c\ub294 \ub3d9\uc778, \ub0a8\uc778, \ubd81\uc778\uacc4\uc5f4 \uc720\ud559\uc790(\u5112\u5b78\u8005)\ub4e4\uc758 \uc0ac\uc0c1\uacf5\uc138\ub294 1910\ub144(\uc735\ud76c 4\ub144) \uc870\uc120\uc774 \uba78\ub9dd\ud560 \ub54c\uae4c\uc9c0\ub3c4 \uc9c0\uc18d\ub41c\ub2e4.", "paragraph_answer": "\uc5b4\uba38\ub2c8 \uc2e0\uc0ac\uc784\ub2f9\uc758 \uc624\ub79c \ubcd1\ud658\uacfc \uc8fd\uc74c\uc740 \uadf8\uc5d0\uac8c \uc2ec\uc801, \uc815\uc2e0\uc801 \ucda9\uaca9\uc744 \uc8fc\uc5c8\ub2e4. \uadf8\ub294 \uc0ac\ub78c\uc774 \uc65c \ud0dc\uc5b4\ub098\uace0 \uc8fd\ub294\uac00\uc5d0 \ub300\ud574 \uacc4\uc18d \uace0\ubbfc\ud558\uba74\uc11c \ud55c\ub3d9\uc548 \ubc29\ud669\ud558\uac8c \ub41c\ub2e4. \uacb0\uad6d \uc2dc\ubb18\uc0b4\uc774\ub97c \ub9c8\uce5c \ub4a4 \uae08\uac15\uc0b0\uc73c\ub85c \ub4e4\uc5b4\uac00 \uc2b9\ub824\uac00 \ub418\uc5c8\uace0, \uadf8\uac00 \ub4a4\uc5d0 \ubd88\uad50\uc5d0 \uc785\ubb38\ud588\ub2e4\uac00 \ud658\uc18d\ud55c \ub4a4\uc5d0\ub3c4 \ubb38\uc81c \uc0bc\uc9c0 \uc54a\uace0 \ubc1b\uc544\uc900 \uac83\uc740 \uc2a4\uc2b9 \ubc31\uc778\uac78\uacfc \uc624\ub79c \uce5c\uad6c \uc131\ud63c \uc774\uc5c8\ub2e4. \uadf8\ub7ec\ub098 \uc774\ub54c\uc758 \uc785\uc0b0 \uacbd\ub825\uc740 \uadf8\uc758 \uc0dd\uc804\uc5d0\ub3c4 \uc1a1\uc751\uac1c \ub4f1\uc758 \ub3d9\uc778(\u6771\u4eba)\ub4e4\uacfc \ud5c8\ubaa9, \uc724\ud734, \uc724\uc120\ub3c4 \ub4f1\uc758 \ub0a8\uc778(\u5357\u4eba) \ub2f9\uc6d0\ub4e4\uc5d0\uac8c \uc774\ub2e8 \ud559\ubb38\uc5d0 \ube60\uc84c\ub2e4\ub294 \uc774\uc720\ub85c \uc0ac\uc0c1\uacf5\uc138\ub97c \ub2f9\ud558\ub294 \uc6d0\uc778\uc774 \ub41c\ub2e4. \uc774\uc774\uac00 \uc2b9\ub824\uc774\uba70 \ubd88\uad50\ub3c4\ub77c\ub294 \ub3d9\uc778, \ub0a8\uc778, \ubd81\uc778\uacc4\uc5f4 \uc720\ud559\uc790(\u5112\u5b78\u8005)\ub4e4\uc758 \uc0ac\uc0c1\uacf5\uc138\ub294 1910\ub144(\uc735\ud76c 4\ub144) \uc870\uc120\uc774 \uba78\ub9dd\ud560 \ub54c\uae4c\uc9c0\ub3c4 \uc9c0\uc18d\ub41c\ub2e4.", "sentence_answer": "\uacb0\uad6d \uc2dc\ubb18\uc0b4\uc774\ub97c \ub9c8\uce5c \ub4a4 \uae08\uac15\uc0b0\uc73c\ub85c \ub4e4\uc5b4\uac00 \uc2b9\ub824\uac00 \ub418\uc5c8\uace0, \uadf8\uac00 \ub4a4\uc5d0 \ubd88\uad50\uc5d0 \uc785\ubb38\ud588\ub2e4\uac00 \ud658\uc18d\ud55c \ub4a4\uc5d0\ub3c4 \ubb38\uc81c \uc0bc\uc9c0 \uc54a\uace0 \ubc1b\uc544\uc900 \uac83\uc740 \uc2a4\uc2b9 \ubc31\uc778\uac78\uacfc \uc624\ub79c \uce5c\uad6c \uc131\ud63c \uc774\uc5c8\ub2e4.", "paragraph_id": "6545887-1-1", "paragraph_question": "question: \uc774\uc774\uac00 \ub2e4\uc2dc \uc7ac\uac00\uc0dd\ud65c\ub85c \ub3cc\uc544\uc654\uc744 \ub54c \ubc1b\uc544\uc900 \uc778\ubb3c\ub85c \ubc31\uc778\uac78\uacfc \ub2e4\ub978 \ud55c\uba85\uc740 \ub204\uad6c\uc778\uac00?, context: \uc5b4\uba38\ub2c8 \uc2e0\uc0ac\uc784\ub2f9\uc758 \uc624\ub79c \ubcd1\ud658\uacfc \uc8fd\uc74c\uc740 \uadf8\uc5d0\uac8c \uc2ec\uc801, \uc815\uc2e0\uc801 \ucda9\uaca9\uc744 \uc8fc\uc5c8\ub2e4. \uadf8\ub294 \uc0ac\ub78c\uc774 \uc65c \ud0dc\uc5b4\ub098\uace0 \uc8fd\ub294\uac00\uc5d0 \ub300\ud574 \uacc4\uc18d \uace0\ubbfc\ud558\uba74\uc11c \ud55c\ub3d9\uc548 \ubc29\ud669\ud558\uac8c \ub41c\ub2e4. \uacb0\uad6d \uc2dc\ubb18\uc0b4\uc774\ub97c \ub9c8\uce5c \ub4a4 \uae08\uac15\uc0b0\uc73c\ub85c \ub4e4\uc5b4\uac00 \uc2b9\ub824\uac00 \ub418\uc5c8\uace0, \uadf8\uac00 \ub4a4\uc5d0 \ubd88\uad50\uc5d0 \uc785\ubb38\ud588\ub2e4\uac00 \ud658\uc18d\ud55c \ub4a4\uc5d0\ub3c4 \ubb38\uc81c \uc0bc\uc9c0 \uc54a\uace0 \ubc1b\uc544\uc900 \uac83\uc740 \uc2a4\uc2b9 \ubc31\uc778\uac78\uacfc \uc624\ub79c \uce5c\uad6c \uc131\ud63c\uc774\uc5c8\ub2e4. \uadf8\ub7ec\ub098 \uc774\ub54c\uc758 \uc785\uc0b0 \uacbd\ub825\uc740 \uadf8\uc758 \uc0dd\uc804\uc5d0\ub3c4 \uc1a1\uc751\uac1c \ub4f1\uc758 \ub3d9\uc778(\u6771\u4eba)\ub4e4\uacfc \ud5c8\ubaa9, \uc724\ud734, \uc724\uc120\ub3c4 \ub4f1\uc758 \ub0a8\uc778(\u5357\u4eba) \ub2f9\uc6d0\ub4e4\uc5d0\uac8c \uc774\ub2e8 \ud559\ubb38\uc5d0 \ube60\uc84c\ub2e4\ub294 \uc774\uc720\ub85c \uc0ac\uc0c1\uacf5\uc138\ub97c \ub2f9\ud558\ub294 \uc6d0\uc778\uc774 \ub41c\ub2e4. \uc774\uc774\uac00 \uc2b9\ub824\uc774\uba70 \ubd88\uad50\ub3c4\ub77c\ub294 \ub3d9\uc778, \ub0a8\uc778, \ubd81\uc778\uacc4\uc5f4 \uc720\ud559\uc790(\u5112\u5b78\u8005)\ub4e4\uc758 \uc0ac\uc0c1\uacf5\uc138\ub294 1910\ub144(\uc735\ud76c 4\ub144) \uc870\uc120\uc774 \uba78\ub9dd\ud560 \ub54c\uae4c\uc9c0\ub3c4 \uc9c0\uc18d\ub41c\ub2e4."} {"question": "\ud604\uc874\uc9c8\uc11c\ub97c \ubc29\uc704\ud558\uac70\ub098 \uc639\ud638\ud558\ub824\ub294 \uc785\uc7a5\uc744 \uac00\uc9c4 \uc8fc\uc758\ub294?", "paragraph": "\uadf8\ub7ec\ub098 \ubcf4\uc218\uc640 \ud601\uc2e0\uc744 \ucd08\uc5ed\uc0ac\uc801\uc778 \uc778\uac04\ubcf8\ub798\uc758 \uae30\uc9c8\uc5d0 \ub454\ub2e4\ub294 \uac83\uc740 \ud0c0\ub2f9\uce58 \uc54a\ub2e4. \uadf8\uac83\uc740 \uc5b4\ub514\uae4c\uc9c0\ub098 \ud604\uc874\uc758 \uc0ac\ud68c\uc801\u00b7\uacbd\uc81c\uc801\u00b7\uc815\uce58\uc801 \uc9c8\uc11c\uc5d0 \ub300\ud55c \uc5b4\ub5a4 \uae30\ubcf8\uc790\uc138\uc758 \ubb38\uc81c\ub85c\uc11c \ud3ec\ucc29\ub418\uc5b4\uc57c \ud560 \uac83\uc774\ub2e4. \uc989 \ud604\uc874\uc758 \uc9c8\uc11c\ub97c \ubc29\uc704 \ub0b4\uc9c0 \uc639\ud638\ud558\ub824\ub294 \uc785\uc7a5\uc774 \ubcf4\uc218\uc8fc\uc758\uc774\uba70, \uc9c4\ubcf4\uc758 \uad00\ub150\uc5d0 \uc785\uac01\ud574\uc11c \ud604\uc874\uc9c8\uc11c\uc5d0 \ube44\ud310\uc801 \ub0b4\uc9c0 \ubd80\uc815\uc801\uc778 \uc785\uc7a5\uc744 \ucde8\ud558\ub294 \uac83\uc774 \ud601\uc2e0\uc8fc\uc758\uc774\ub2e4. \uadf8\ub7ec\ubbc0\ub85c \ud604\uc874 \uc9c8\uc11c\ud558\uc5d0\uc11c \uc9c0\ubc30\uc801 \uacc4\uce35\uc5d0 \uc788\ub294 \uc0ac\ub78c\uc740 \ubcf4\uc218\uc8fc\uc758\ub97c \uc9c0\uc9c0\ud558\uace0, \ud53c\uc9c0\ubc30\uacc4\uce35\uc758 \uc0ac\ub78c\ub4e4\uc740 \ud601\uc2e0\uc8fc\uc758\ub97c \uc9c0\uc9c0\ud558\uae30 \uc26c\uc6b4 \uac83\uc774\ub2e4. \ubcf4\uc218\uc640 \ud601\uc2e0\uc758 \uc758\ubbf8\ub97c \ud604\uc874\uc9c8\uc11c\ub97c \uae30\uc900\uc73c\ub85c \ud574\uc11c \uc0dd\uac01\ud560 \ub54c\ub294 \uadf8 \uc911\uac04\uc801 \uc785\uc7a5\uc740 \uc774\ub860\uc801\uc73c\ub85c \uc131\ub9bd\ub420 \uc218 \uc5c6\ub2e4. \uc18c\uc704 \uc911\uac04\uc815\ub2f9\uc740 \uc2e4\uc740 \ubcf4\uc218\uc815\ub2f9\uc758 \uc628\uac74\ud30c\uc640 \ud601\uc2e0\uc815\ub2f9\uc758 \uc628\uac74\ud30c\uc758 \uc9d1\ud569\uccb4\uc5d0 \ubd88\uacfc\ud558\ub2e4. \ud604\uc874 \uc9c8\uc11c\uc758 \uae30\ucd08\uc5d0 \uc911\ub300\ud55c \uc704\ud611\uc774 \uac00\ud574\uc9c8 \ub54c \uc911\uac04\uc815\ub2f9\uc774 \ud56d\uc0c1 \ubd84\uc5f4\ud560 \uc6b4\uba85\uc744 \uc9c0\ub2c8\uace0 \uc788\ub294 \uac83\uc740 \uadf8 \ub54c\ubb38\uc774\ub2e4.", "answer": "\ubcf4\uc218\uc8fc\uc758", "sentence": "\uc989 \ud604\uc874\uc758 \uc9c8\uc11c\ub97c \ubc29\uc704 \ub0b4\uc9c0 \uc639\ud638\ud558\ub824\ub294 \uc785\uc7a5\uc774 \ubcf4\uc218\uc8fc\uc758 \uc774\uba70, \uc9c4\ubcf4\uc758 \uad00\ub150\uc5d0 \uc785\uac01\ud574\uc11c \ud604\uc874\uc9c8\uc11c\uc5d0 \ube44\ud310\uc801 \ub0b4\uc9c0 \ubd80\uc815\uc801\uc778 \uc785\uc7a5\uc744 \ucde8\ud558\ub294 \uac83\uc774 \ud601\uc2e0\uc8fc\uc758\uc774\ub2e4.", "paragraph_sentence": "\uadf8\ub7ec\ub098 \ubcf4\uc218\uc640 \ud601\uc2e0\uc744 \ucd08\uc5ed\uc0ac\uc801\uc778 \uc778\uac04\ubcf8\ub798\uc758 \uae30\uc9c8\uc5d0 \ub454\ub2e4\ub294 \uac83\uc740 \ud0c0\ub2f9\uce58 \uc54a\ub2e4. \uadf8\uac83\uc740 \uc5b4\ub514\uae4c\uc9c0\ub098 \ud604\uc874\uc758 \uc0ac\ud68c\uc801\u00b7\uacbd\uc81c\uc801\u00b7\uc815\uce58\uc801 \uc9c8\uc11c\uc5d0 \ub300\ud55c \uc5b4\ub5a4 \uae30\ubcf8\uc790\uc138\uc758 \ubb38\uc81c\ub85c\uc11c \ud3ec\ucc29\ub418\uc5b4\uc57c \ud560 \uac83\uc774\ub2e4. \uc989 \ud604\uc874\uc758 \uc9c8\uc11c\ub97c \ubc29\uc704 \ub0b4\uc9c0 \uc639\ud638\ud558\ub824\ub294 \uc785\uc7a5\uc774 \ubcf4\uc218\uc8fc\uc758 \uc774\uba70, \uc9c4\ubcf4\uc758 \uad00\ub150\uc5d0 \uc785\uac01\ud574\uc11c \ud604\uc874\uc9c8\uc11c\uc5d0 \ube44\ud310\uc801 \ub0b4\uc9c0 \ubd80\uc815\uc801\uc778 \uc785\uc7a5\uc744 \ucde8\ud558\ub294 \uac83\uc774 \ud601\uc2e0\uc8fc\uc758\uc774\ub2e4. \uadf8\ub7ec\ubbc0\ub85c \ud604\uc874 \uc9c8\uc11c\ud558\uc5d0\uc11c \uc9c0\ubc30\uc801 \uacc4\uce35\uc5d0 \uc788\ub294 \uc0ac\ub78c\uc740 \ubcf4\uc218\uc8fc\uc758\ub97c \uc9c0\uc9c0\ud558\uace0, \ud53c\uc9c0\ubc30\uacc4\uce35\uc758 \uc0ac\ub78c\ub4e4\uc740 \ud601\uc2e0\uc8fc\uc758\ub97c \uc9c0\uc9c0\ud558\uae30 \uc26c\uc6b4 \uac83\uc774\ub2e4. \ubcf4\uc218\uc640 \ud601\uc2e0\uc758 \uc758\ubbf8\ub97c \ud604\uc874\uc9c8\uc11c\ub97c \uae30\uc900\uc73c\ub85c \ud574\uc11c \uc0dd\uac01\ud560 \ub54c\ub294 \uadf8 \uc911\uac04\uc801 \uc785\uc7a5\uc740 \uc774\ub860\uc801\uc73c\ub85c \uc131\ub9bd\ub420 \uc218 \uc5c6\ub2e4. \uc18c\uc704 \uc911\uac04\uc815\ub2f9\uc740 \uc2e4\uc740 \ubcf4\uc218\uc815\ub2f9\uc758 \uc628\uac74\ud30c\uc640 \ud601\uc2e0\uc815\ub2f9\uc758 \uc628\uac74\ud30c\uc758 \uc9d1\ud569\uccb4\uc5d0 \ubd88\uacfc\ud558\ub2e4. \ud604\uc874 \uc9c8\uc11c\uc758 \uae30\ucd08\uc5d0 \uc911\ub300\ud55c \uc704\ud611\uc774 \uac00\ud574\uc9c8 \ub54c \uc911\uac04\uc815\ub2f9\uc774 \ud56d\uc0c1 \ubd84\uc5f4\ud560 \uc6b4\uba85\uc744 \uc9c0\ub2c8\uace0 \uc788\ub294 \uac83\uc740 \uadf8 \ub54c\ubb38\uc774\ub2e4.", "paragraph_answer": "\uadf8\ub7ec\ub098 \ubcf4\uc218\uc640 \ud601\uc2e0\uc744 \ucd08\uc5ed\uc0ac\uc801\uc778 \uc778\uac04\ubcf8\ub798\uc758 \uae30\uc9c8\uc5d0 \ub454\ub2e4\ub294 \uac83\uc740 \ud0c0\ub2f9\uce58 \uc54a\ub2e4. \uadf8\uac83\uc740 \uc5b4\ub514\uae4c\uc9c0\ub098 \ud604\uc874\uc758 \uc0ac\ud68c\uc801\u00b7\uacbd\uc81c\uc801\u00b7\uc815\uce58\uc801 \uc9c8\uc11c\uc5d0 \ub300\ud55c \uc5b4\ub5a4 \uae30\ubcf8\uc790\uc138\uc758 \ubb38\uc81c\ub85c\uc11c \ud3ec\ucc29\ub418\uc5b4\uc57c \ud560 \uac83\uc774\ub2e4. \uc989 \ud604\uc874\uc758 \uc9c8\uc11c\ub97c \ubc29\uc704 \ub0b4\uc9c0 \uc639\ud638\ud558\ub824\ub294 \uc785\uc7a5\uc774 \ubcf4\uc218\uc8fc\uc758 \uc774\uba70, \uc9c4\ubcf4\uc758 \uad00\ub150\uc5d0 \uc785\uac01\ud574\uc11c \ud604\uc874\uc9c8\uc11c\uc5d0 \ube44\ud310\uc801 \ub0b4\uc9c0 \ubd80\uc815\uc801\uc778 \uc785\uc7a5\uc744 \ucde8\ud558\ub294 \uac83\uc774 \ud601\uc2e0\uc8fc\uc758\uc774\ub2e4. \uadf8\ub7ec\ubbc0\ub85c \ud604\uc874 \uc9c8\uc11c\ud558\uc5d0\uc11c \uc9c0\ubc30\uc801 \uacc4\uce35\uc5d0 \uc788\ub294 \uc0ac\ub78c\uc740 \ubcf4\uc218\uc8fc\uc758\ub97c \uc9c0\uc9c0\ud558\uace0, \ud53c\uc9c0\ubc30\uacc4\uce35\uc758 \uc0ac\ub78c\ub4e4\uc740 \ud601\uc2e0\uc8fc\uc758\ub97c \uc9c0\uc9c0\ud558\uae30 \uc26c\uc6b4 \uac83\uc774\ub2e4. \ubcf4\uc218\uc640 \ud601\uc2e0\uc758 \uc758\ubbf8\ub97c \ud604\uc874\uc9c8\uc11c\ub97c \uae30\uc900\uc73c\ub85c \ud574\uc11c \uc0dd\uac01\ud560 \ub54c\ub294 \uadf8 \uc911\uac04\uc801 \uc785\uc7a5\uc740 \uc774\ub860\uc801\uc73c\ub85c \uc131\ub9bd\ub420 \uc218 \uc5c6\ub2e4. \uc18c\uc704 \uc911\uac04\uc815\ub2f9\uc740 \uc2e4\uc740 \ubcf4\uc218\uc815\ub2f9\uc758 \uc628\uac74\ud30c\uc640 \ud601\uc2e0\uc815\ub2f9\uc758 \uc628\uac74\ud30c\uc758 \uc9d1\ud569\uccb4\uc5d0 \ubd88\uacfc\ud558\ub2e4. \ud604\uc874 \uc9c8\uc11c\uc758 \uae30\ucd08\uc5d0 \uc911\ub300\ud55c \uc704\ud611\uc774 \uac00\ud574\uc9c8 \ub54c \uc911\uac04\uc815\ub2f9\uc774 \ud56d\uc0c1 \ubd84\uc5f4\ud560 \uc6b4\uba85\uc744 \uc9c0\ub2c8\uace0 \uc788\ub294 \uac83\uc740 \uadf8 \ub54c\ubb38\uc774\ub2e4.", "sentence_answer": "\uc989 \ud604\uc874\uc758 \uc9c8\uc11c\ub97c \ubc29\uc704 \ub0b4\uc9c0 \uc639\ud638\ud558\ub824\ub294 \uc785\uc7a5\uc774 \ubcf4\uc218\uc8fc\uc758 \uc774\uba70, \uc9c4\ubcf4\uc758 \uad00\ub150\uc5d0 \uc785\uac01\ud574\uc11c \ud604\uc874\uc9c8\uc11c\uc5d0 \ube44\ud310\uc801 \ub0b4\uc9c0 \ubd80\uc815\uc801\uc778 \uc785\uc7a5\uc744 \ucde8\ud558\ub294 \uac83\uc774 \ud601\uc2e0\uc8fc\uc758\uc774\ub2e4.", "paragraph_id": "6657669-13-0", "paragraph_question": "question: \ud604\uc874\uc9c8\uc11c\ub97c \ubc29\uc704\ud558\uac70\ub098 \uc639\ud638\ud558\ub824\ub294 \uc785\uc7a5\uc744 \uac00\uc9c4 \uc8fc\uc758\ub294?, context: \uadf8\ub7ec\ub098 \ubcf4\uc218\uc640 \ud601\uc2e0\uc744 \ucd08\uc5ed\uc0ac\uc801\uc778 \uc778\uac04\ubcf8\ub798\uc758 \uae30\uc9c8\uc5d0 \ub454\ub2e4\ub294 \uac83\uc740 \ud0c0\ub2f9\uce58 \uc54a\ub2e4. \uadf8\uac83\uc740 \uc5b4\ub514\uae4c\uc9c0\ub098 \ud604\uc874\uc758 \uc0ac\ud68c\uc801\u00b7\uacbd\uc81c\uc801\u00b7\uc815\uce58\uc801 \uc9c8\uc11c\uc5d0 \ub300\ud55c \uc5b4\ub5a4 \uae30\ubcf8\uc790\uc138\uc758 \ubb38\uc81c\ub85c\uc11c \ud3ec\ucc29\ub418\uc5b4\uc57c \ud560 \uac83\uc774\ub2e4. \uc989 \ud604\uc874\uc758 \uc9c8\uc11c\ub97c \ubc29\uc704 \ub0b4\uc9c0 \uc639\ud638\ud558\ub824\ub294 \uc785\uc7a5\uc774 \ubcf4\uc218\uc8fc\uc758\uc774\uba70, \uc9c4\ubcf4\uc758 \uad00\ub150\uc5d0 \uc785\uac01\ud574\uc11c \ud604\uc874\uc9c8\uc11c\uc5d0 \ube44\ud310\uc801 \ub0b4\uc9c0 \ubd80\uc815\uc801\uc778 \uc785\uc7a5\uc744 \ucde8\ud558\ub294 \uac83\uc774 \ud601\uc2e0\uc8fc\uc758\uc774\ub2e4. \uadf8\ub7ec\ubbc0\ub85c \ud604\uc874 \uc9c8\uc11c\ud558\uc5d0\uc11c \uc9c0\ubc30\uc801 \uacc4\uce35\uc5d0 \uc788\ub294 \uc0ac\ub78c\uc740 \ubcf4\uc218\uc8fc\uc758\ub97c \uc9c0\uc9c0\ud558\uace0, \ud53c\uc9c0\ubc30\uacc4\uce35\uc758 \uc0ac\ub78c\ub4e4\uc740 \ud601\uc2e0\uc8fc\uc758\ub97c \uc9c0\uc9c0\ud558\uae30 \uc26c\uc6b4 \uac83\uc774\ub2e4. \ubcf4\uc218\uc640 \ud601\uc2e0\uc758 \uc758\ubbf8\ub97c \ud604\uc874\uc9c8\uc11c\ub97c \uae30\uc900\uc73c\ub85c \ud574\uc11c \uc0dd\uac01\ud560 \ub54c\ub294 \uadf8 \uc911\uac04\uc801 \uc785\uc7a5\uc740 \uc774\ub860\uc801\uc73c\ub85c \uc131\ub9bd\ub420 \uc218 \uc5c6\ub2e4. \uc18c\uc704 \uc911\uac04\uc815\ub2f9\uc740 \uc2e4\uc740 \ubcf4\uc218\uc815\ub2f9\uc758 \uc628\uac74\ud30c\uc640 \ud601\uc2e0\uc815\ub2f9\uc758 \uc628\uac74\ud30c\uc758 \uc9d1\ud569\uccb4\uc5d0 \ubd88\uacfc\ud558\ub2e4. \ud604\uc874 \uc9c8\uc11c\uc758 \uae30\ucd08\uc5d0 \uc911\ub300\ud55c \uc704\ud611\uc774 \uac00\ud574\uc9c8 \ub54c \uc911\uac04\uc815\ub2f9\uc774 \ud56d\uc0c1 \ubd84\uc5f4\ud560 \uc6b4\uba85\uc744 \uc9c0\ub2c8\uace0 \uc788\ub294 \uac83\uc740 \uadf8 \ub54c\ubb38\uc774\ub2e4."} {"question": "\ub2f9\uc2dc \ub125\uc13c\uc758 \ub2e8\uc7a5\uc758 \uc774\ub984\uc740?", "paragraph": "2010 \uc2dc\uc98c\uc774 \ud55c\uad6d\uc2dc\ub9ac\uc988\ub97c \ub05d\uc73c\ub85c \uc885\ub8cc\ub418\uc790 \uc0c8\ub85c\uc6b4 \uc2dc\uc98c\uc744 \uc704\ud55c \uc2a4\ud1a0\ube0c\ub9ac\uadf8\uac00 \uc2dc\uc791\ub418\uc5c8\ub2e4. \uc720\uc218\uc758 \ud2b8\ub808\uc774\ub4dc \uacbd\ud5d8\uc774 \uc788\ub294 \ub125\uc13c\uc5d0\uac8c \ud2b8\ub808\uc774\ub4dc \uc81c\uc758\uac00 \uc3df\uc544\uc9c4 \uac83\uc73c\ub85c \ubc1d\ud600\uc84c\ub2e4. \ubb38\uc758 \ubc1b\uc740 \uc120\uc218\ub4e4\ub85c\ub294 2010\ub144 \uad6c\uc6d0\uc655\uc5d0 \uc624\ub978 \uc190\uc2b9\ub77d, \uad6d\uac00\ub300\ud45c\ub85c \ucd9c\uc7a5\ud55c \ubc14 \uc788\ub294 \uc720\uaca9\uc218 \uac15\uc815\ud638, \uc2e0\uc778 \ud22c\uc218 \uace0\uc6d0\uc900, \uae40\uc131\ud604 \ub4f1\uc778\ub370, \uad6c\ub2e8\uc8fc \uc774\uc7a5\uc11d\uc740 \uac15\uc815\ud638\ub97c \ub530\ub85c \ubd88\ub7ec \ud2b8\ub808\uc774\ub4dc\ub294 \uc5c6\uc744 \uac83\uc774\ub77c\uace0 \uc57d\uc18d\ud55c \ub300\uc2e0 \uc190\uc2b9\ub77d \ub4f1\uc740 \ud2b8\ub808\uc774\ub4dc \uce74\ub4dc\uc5d0 \ub530\ub77c \uc774\uc801\ud560 \uc218\ub3c4 \uc788\ub2e4\uace0 \ud588\ub2e4. \ub125\uc13c\uc758 \uc870\ud0dc\ub8e1 \ub2e8\uc7a5\ub3c4 \"\uc190\uc2b9\ub77d\uc744 \ub370\ub824\uac00\ub294 \ud300\uc740 \ucd5c\uc18c\ud55c \uc911\uc2ec \ud0c0\uc790 \uc911 1\uba85\uc744 \ub0b4\uc918\uc57c \uaca9\uc774 \ub9de\ub294\ub2e4\" \ub77c\uace0 \ubc1c\uc5b8\ud558\uba70 \ub3c8\uc5d0 \uc88c\uc9c0\uc6b0\uc9c0\ub418\ub294 \ud2b8\ub808\uc774\ub4dc\ub294 \uc5c6\ub2e4\uace0 \ubc1d\ud614\uace0, \uc190\uc2b9\ub77d\uacfc \uac15\uc815\ud638\ub97c \ud300\uc758 \uac04\ud310\uc774\ub77c\uace0 \uce6d\ud558\uba70 \ub300\uc2e0 \uae09\uc774 \ub9de\ub294 \uc120\uc218\uc640 \ub9de\ud2b8\ub808\uc774\ub4dc\ub294 \ucd94\uc9c4\ud558\uaca0\ub2e4\ub294 \uc758\uc0ac\ub97c \ubc1d\ud614\ub2e4.", "answer": "\uc870\ud0dc\ub8e1", "sentence": "\ub125\uc13c\uc758 \uc870\ud0dc\ub8e1 \ub2e8\uc7a5\ub3c4 \"\uc190\uc2b9\ub77d\uc744 \ub370\ub824\uac00\ub294 \ud300\uc740 \ucd5c\uc18c\ud55c \uc911\uc2ec \ud0c0\uc790 \uc911 1\uba85\uc744 \ub0b4\uc918\uc57c \uaca9\uc774 \ub9de\ub294\ub2e4\" \ub77c\uace0 \ubc1c\uc5b8\ud558\uba70 \ub3c8\uc5d0 \uc88c\uc9c0\uc6b0\uc9c0\ub418\ub294 \ud2b8\ub808\uc774\ub4dc\ub294 \uc5c6\ub2e4\uace0 \ubc1d\ud614\uace0, \uc190\uc2b9\ub77d\uacfc \uac15\uc815\ud638\ub97c \ud300\uc758 \uac04\ud310\uc774\ub77c\uace0 \uce6d\ud558\uba70 \ub300\uc2e0 \uae09\uc774 \ub9de\ub294 \uc120\uc218\uc640 \ub9de\ud2b8\ub808\uc774\ub4dc\ub294 \ucd94\uc9c4\ud558\uaca0\ub2e4\ub294 \uc758\uc0ac\ub97c \ubc1d\ud614\ub2e4.", "paragraph_sentence": "2010 \uc2dc\uc98c\uc774 \ud55c\uad6d\uc2dc\ub9ac\uc988\ub97c \ub05d\uc73c\ub85c \uc885\ub8cc\ub418\uc790 \uc0c8\ub85c\uc6b4 \uc2dc\uc98c\uc744 \uc704\ud55c \uc2a4\ud1a0\ube0c\ub9ac\uadf8\uac00 \uc2dc\uc791\ub418\uc5c8\ub2e4. \uc720\uc218\uc758 \ud2b8\ub808\uc774\ub4dc \uacbd\ud5d8\uc774 \uc788\ub294 \ub125\uc13c\uc5d0\uac8c \ud2b8\ub808\uc774\ub4dc \uc81c\uc758\uac00 \uc3df\uc544\uc9c4 \uac83\uc73c\ub85c \ubc1d\ud600\uc84c\ub2e4. \ubb38\uc758 \ubc1b\uc740 \uc120\uc218\ub4e4\ub85c\ub294 2010\ub144 \uad6c\uc6d0\uc655\uc5d0 \uc624\ub978 \uc190\uc2b9\ub77d, \uad6d\uac00\ub300\ud45c\ub85c \ucd9c\uc7a5\ud55c \ubc14 \uc788\ub294 \uc720\uaca9\uc218 \uac15\uc815\ud638, \uc2e0\uc778 \ud22c\uc218 \uace0\uc6d0\uc900, \uae40\uc131\ud604 \ub4f1\uc778\ub370, \uad6c\ub2e8\uc8fc \uc774\uc7a5\uc11d\uc740 \uac15\uc815\ud638\ub97c \ub530\ub85c \ubd88\ub7ec \ud2b8\ub808\uc774\ub4dc\ub294 \uc5c6\uc744 \uac83\uc774\ub77c\uace0 \uc57d\uc18d\ud55c \ub300\uc2e0 \uc190\uc2b9\ub77d \ub4f1\uc740 \ud2b8\ub808\uc774\ub4dc \uce74\ub4dc\uc5d0 \ub530\ub77c \uc774\uc801\ud560 \uc218\ub3c4 \uc788\ub2e4\uace0 \ud588\ub2e4. \ub125\uc13c\uc758 \uc870\ud0dc\ub8e1 \ub2e8\uc7a5\ub3c4 \"\uc190\uc2b9\ub77d\uc744 \ub370\ub824\uac00\ub294 \ud300\uc740 \ucd5c\uc18c\ud55c \uc911\uc2ec \ud0c0\uc790 \uc911 1\uba85\uc744 \ub0b4\uc918\uc57c \uaca9\uc774 \ub9de\ub294\ub2e4\" \ub77c\uace0 \ubc1c\uc5b8\ud558\uba70 \ub3c8\uc5d0 \uc88c\uc9c0\uc6b0\uc9c0\ub418\ub294 \ud2b8\ub808\uc774\ub4dc\ub294 \uc5c6\ub2e4\uace0 \ubc1d\ud614\uace0, \uc190\uc2b9\ub77d\uacfc \uac15\uc815\ud638\ub97c \ud300\uc758 \uac04\ud310\uc774\ub77c\uace0 \uce6d\ud558\uba70 \ub300\uc2e0 \uae09\uc774 \ub9de\ub294 \uc120\uc218\uc640 \ub9de\ud2b8\ub808\uc774\ub4dc\ub294 \ucd94\uc9c4\ud558\uaca0\ub2e4\ub294 \uc758\uc0ac\ub97c \ubc1d\ud614\ub2e4. ", "paragraph_answer": "2010 \uc2dc\uc98c\uc774 \ud55c\uad6d\uc2dc\ub9ac\uc988\ub97c \ub05d\uc73c\ub85c \uc885\ub8cc\ub418\uc790 \uc0c8\ub85c\uc6b4 \uc2dc\uc98c\uc744 \uc704\ud55c \uc2a4\ud1a0\ube0c\ub9ac\uadf8\uac00 \uc2dc\uc791\ub418\uc5c8\ub2e4. \uc720\uc218\uc758 \ud2b8\ub808\uc774\ub4dc \uacbd\ud5d8\uc774 \uc788\ub294 \ub125\uc13c\uc5d0\uac8c \ud2b8\ub808\uc774\ub4dc \uc81c\uc758\uac00 \uc3df\uc544\uc9c4 \uac83\uc73c\ub85c \ubc1d\ud600\uc84c\ub2e4. \ubb38\uc758 \ubc1b\uc740 \uc120\uc218\ub4e4\ub85c\ub294 2010\ub144 \uad6c\uc6d0\uc655\uc5d0 \uc624\ub978 \uc190\uc2b9\ub77d, \uad6d\uac00\ub300\ud45c\ub85c \ucd9c\uc7a5\ud55c \ubc14 \uc788\ub294 \uc720\uaca9\uc218 \uac15\uc815\ud638, \uc2e0\uc778 \ud22c\uc218 \uace0\uc6d0\uc900, \uae40\uc131\ud604 \ub4f1\uc778\ub370, \uad6c\ub2e8\uc8fc \uc774\uc7a5\uc11d\uc740 \uac15\uc815\ud638\ub97c \ub530\ub85c \ubd88\ub7ec \ud2b8\ub808\uc774\ub4dc\ub294 \uc5c6\uc744 \uac83\uc774\ub77c\uace0 \uc57d\uc18d\ud55c \ub300\uc2e0 \uc190\uc2b9\ub77d \ub4f1\uc740 \ud2b8\ub808\uc774\ub4dc \uce74\ub4dc\uc5d0 \ub530\ub77c \uc774\uc801\ud560 \uc218\ub3c4 \uc788\ub2e4\uace0 \ud588\ub2e4. \ub125\uc13c\uc758 \uc870\ud0dc\ub8e1 \ub2e8\uc7a5\ub3c4 \"\uc190\uc2b9\ub77d\uc744 \ub370\ub824\uac00\ub294 \ud300\uc740 \ucd5c\uc18c\ud55c \uc911\uc2ec \ud0c0\uc790 \uc911 1\uba85\uc744 \ub0b4\uc918\uc57c \uaca9\uc774 \ub9de\ub294\ub2e4\" \ub77c\uace0 \ubc1c\uc5b8\ud558\uba70 \ub3c8\uc5d0 \uc88c\uc9c0\uc6b0\uc9c0\ub418\ub294 \ud2b8\ub808\uc774\ub4dc\ub294 \uc5c6\ub2e4\uace0 \ubc1d\ud614\uace0, \uc190\uc2b9\ub77d\uacfc \uac15\uc815\ud638\ub97c \ud300\uc758 \uac04\ud310\uc774\ub77c\uace0 \uce6d\ud558\uba70 \ub300\uc2e0 \uae09\uc774 \ub9de\ub294 \uc120\uc218\uc640 \ub9de\ud2b8\ub808\uc774\ub4dc\ub294 \ucd94\uc9c4\ud558\uaca0\ub2e4\ub294 \uc758\uc0ac\ub97c \ubc1d\ud614\ub2e4.", "sentence_answer": "\ub125\uc13c\uc758 \uc870\ud0dc\ub8e1 \ub2e8\uc7a5\ub3c4 \"\uc190\uc2b9\ub77d\uc744 \ub370\ub824\uac00\ub294 \ud300\uc740 \ucd5c\uc18c\ud55c \uc911\uc2ec \ud0c0\uc790 \uc911 1\uba85\uc744 \ub0b4\uc918\uc57c \uaca9\uc774 \ub9de\ub294\ub2e4\" \ub77c\uace0 \ubc1c\uc5b8\ud558\uba70 \ub3c8\uc5d0 \uc88c\uc9c0\uc6b0\uc9c0\ub418\ub294 \ud2b8\ub808\uc774\ub4dc\ub294 \uc5c6\ub2e4\uace0 \ubc1d\ud614\uace0, \uc190\uc2b9\ub77d\uacfc \uac15\uc815\ud638\ub97c \ud300\uc758 \uac04\ud310\uc774\ub77c\uace0 \uce6d\ud558\uba70 \ub300\uc2e0 \uae09\uc774 \ub9de\ub294 \uc120\uc218\uc640 \ub9de\ud2b8\ub808\uc774\ub4dc\ub294 \ucd94\uc9c4\ud558\uaca0\ub2e4\ub294 \uc758\uc0ac\ub97c \ubc1d\ud614\ub2e4.", "paragraph_id": "6533929-13-0", "paragraph_question": "question: \ub2f9\uc2dc \ub125\uc13c\uc758 \ub2e8\uc7a5\uc758 \uc774\ub984\uc740?, context: 2010 \uc2dc\uc98c\uc774 \ud55c\uad6d\uc2dc\ub9ac\uc988\ub97c \ub05d\uc73c\ub85c \uc885\ub8cc\ub418\uc790 \uc0c8\ub85c\uc6b4 \uc2dc\uc98c\uc744 \uc704\ud55c \uc2a4\ud1a0\ube0c\ub9ac\uadf8\uac00 \uc2dc\uc791\ub418\uc5c8\ub2e4. \uc720\uc218\uc758 \ud2b8\ub808\uc774\ub4dc \uacbd\ud5d8\uc774 \uc788\ub294 \ub125\uc13c\uc5d0\uac8c \ud2b8\ub808\uc774\ub4dc \uc81c\uc758\uac00 \uc3df\uc544\uc9c4 \uac83\uc73c\ub85c \ubc1d\ud600\uc84c\ub2e4. \ubb38\uc758 \ubc1b\uc740 \uc120\uc218\ub4e4\ub85c\ub294 2010\ub144 \uad6c\uc6d0\uc655\uc5d0 \uc624\ub978 \uc190\uc2b9\ub77d, \uad6d\uac00\ub300\ud45c\ub85c \ucd9c\uc7a5\ud55c \ubc14 \uc788\ub294 \uc720\uaca9\uc218 \uac15\uc815\ud638, \uc2e0\uc778 \ud22c\uc218 \uace0\uc6d0\uc900, \uae40\uc131\ud604 \ub4f1\uc778\ub370, \uad6c\ub2e8\uc8fc \uc774\uc7a5\uc11d\uc740 \uac15\uc815\ud638\ub97c \ub530\ub85c \ubd88\ub7ec \ud2b8\ub808\uc774\ub4dc\ub294 \uc5c6\uc744 \uac83\uc774\ub77c\uace0 \uc57d\uc18d\ud55c \ub300\uc2e0 \uc190\uc2b9\ub77d \ub4f1\uc740 \ud2b8\ub808\uc774\ub4dc \uce74\ub4dc\uc5d0 \ub530\ub77c \uc774\uc801\ud560 \uc218\ub3c4 \uc788\ub2e4\uace0 \ud588\ub2e4. \ub125\uc13c\uc758 \uc870\ud0dc\ub8e1 \ub2e8\uc7a5\ub3c4 \"\uc190\uc2b9\ub77d\uc744 \ub370\ub824\uac00\ub294 \ud300\uc740 \ucd5c\uc18c\ud55c \uc911\uc2ec \ud0c0\uc790 \uc911 1\uba85\uc744 \ub0b4\uc918\uc57c \uaca9\uc774 \ub9de\ub294\ub2e4\" \ub77c\uace0 \ubc1c\uc5b8\ud558\uba70 \ub3c8\uc5d0 \uc88c\uc9c0\uc6b0\uc9c0\ub418\ub294 \ud2b8\ub808\uc774\ub4dc\ub294 \uc5c6\ub2e4\uace0 \ubc1d\ud614\uace0, \uc190\uc2b9\ub77d\uacfc \uac15\uc815\ud638\ub97c \ud300\uc758 \uac04\ud310\uc774\ub77c\uace0 \uce6d\ud558\uba70 \ub300\uc2e0 \uae09\uc774 \ub9de\ub294 \uc120\uc218\uc640 \ub9de\ud2b8\ub808\uc774\ub4dc\ub294 \ucd94\uc9c4\ud558\uaca0\ub2e4\ub294 \uc758\uc0ac\ub97c \ubc1d\ud614\ub2e4."} {"question": "\uc77c\ubcf8 \uc815\ubd80\uc640\uc758 \ud611\uc0c1 \uacfc\uc815 \uc911 \uc77c\ubcf8\uc774 \ud611\uc0c1\uc548\uac74\uc73c\ub85c \ub123\uc73c\ub824 \ud588\ub358 \uac83\uc740?", "paragraph": "2005\ub144 \ud55c\uc77c\ud611\uc815\ubb38\uc11c\uac00 \uacf5\uac1c\ub410\ub2e4. \uc774\uc5d0 \ub300\ud574 \ubc15\uc815\ud76c \uc815\uad8c\uc774 \ub300\uc77c\uccad\uad6c\uad8c \ud3ec\uae30\ub9d0\uace0\ub3c4, \ud611\uc0c1\uacfc\uc815\uc5d0\uc11c \uc77c\ubcf8 \uc815\ubd80\ub294 \uc544\uc608 '\ub3c5\ub3c4\ub97c \ud3ed\ud30c\ud558\uc790'\uace0 \ud611\ubc15\uae4c\uc9c0 \ud558\uba70 '\ub3c5\ub3c4'\ub97c \ud611\uc0c1\uc548\uac74\uc73c\ub85c \ub123\uc73c\ub824 \ud588\ub2e4. \ub610\ud55c \ud55c\uc77c\uc5b4\uc5c5\ud611\uc0c1\uc744 \ub300\uc120\uc5d0 \ud65c\uc6a9\ud558\uace0 \ub300\uc77c\ubcf8 \ubc30\uc0c1\uad00\ub828 \uac1c\uc778\uccad\uad6c\uad8c\uc744 \ubb34\uc2dc\ud55c \uc810\ub3c4 \ub4dc\ub7ec\ub0ac\ub2e4. \ub2f9\uc2dc \uc815\ubd80\ub294 \uae30\uc874\uc758 40\ub9c8\uc77c \uc804\uad00\uc218\uc5ed\uc785\uc7a5\uc5d0\uc11c \ud6c4\ud1f4, \uc77c\ubcf8 \uc815\ubd80\uac00 \uc8fc\uc7a5\ud55c 12\ub9c8\uc77c \uc804\uad00\uc218\uc5ed \ubc29\uc548\uc744 \uc11c\ub458\ub7ec \uc218\uc6a9\ud588\uc73c\ub098 \uc5ec\ub860\uc545\ud654\ub97c \uc6b0\ub824\ud574 \uacf5\uac1c\uc2dc\uae30\ub97c \ub2a6\ucd98 \uac83\uc73c\ub85c \ub098\ud0c0\ub0ac\ub2e4. \uc784\ud5cc\uc601 \ubbfc\uc871\ubb38\uc81c\uc5f0\uad6c\uc18c \uc18c\uc7a5\uc740 \ubc15 \uc815\uad8c\uc774 61\ub144\ubd80\ud130 \ud55c\uc77c\ud611\uc815\uc744 \uccb4\uacb0\ud55c 65\ub144 \uc0ac\uc774 5\ub144\uac04\uc5d0 \uac78\uccd0 6\uac1c\uc758 \uc77c\ubcf8\uae30\uc5c5\ub4e4\ub85c\ubd80\ud130 \ubbfc\uc8fc\uacf5\ud654\ub2f9 \ucd1d\uc608\uc0b0 2/3\uc5d0 \ud574\ub2f9\ud558\ub294 6,600\ub9cc \ub2ec\ub7ec\ub97c \uc81c\uacf5 \ubc1b\uc558\ub2e4\uace0 \uc8fc\uc7a5\ud558\uc600\ub2e4. \uc77c\ubcf8\uce21 \uc678\uad50\ub77c\uc778\uc740 \ub9cc\uc8fc\uc778\ub9e5\uc774\uc5c8\ub2e4. \ud55c\ud638\uc11d \ud1b5\uc77c\ud559\uc5f0\uad6c\uc18c \uc18c\uc7a5\uc740 \"\ub3c5\ub3c4\uc601\uc720\uad8c\ubb38\uc81c\ub97c \ubd88\ubc95\uc801\uc73c\ub85c \ucc98\ub9ac\ud574\ubc84\ub9b0 \uc790\uae30\ub4e4\uc758 \uc8c4\uc0c1\uc774 \uc138\uc0c1\uc5d0 \uc54c\ub824\uc9c0\ub294 \uac83\uc744 \uaebc\ub9b0 \ud55c\uc77c \uc591\uad6d\uc815\ubd80\uc758 \uace0\uc704\uad00\ub9ac\ub4e4\uc740 \ubc00\uc57d\ubb38\uc11c\ub97c \uc601\uc6d0\ud55c \ube44\ubc00\ub85c \ubb3b\uc5b4\ub450\uae30\ub85c \uc57d\uc18d\ud558\uc600\uace0, \ubc00\uc57d\uc774 \uc788\uc5c8\ub2e4\ub294 \uc0ac\uc2e4\uc870\ucc28 \ub9d0\ud558\uc9c0 \uc54a\uc558\ub2e4. \u2018\ud55c\uc77c\ud611\ub825\u2019\uc744 \uc678\uccd0\uc628 \uc5ed\ub300\uc815\uad8c\ub4e4\uc758 \uc740\ud3d0\uc220\uc5d0 \uc138\uc0c1\uc774 \uac10\ucabd\uac19\uc774 \uc18d\uc558\ub358 \uac83\uc774\ub2e4. \uc774 \ub54c\uc758 \ubc00\uc57d \ud30c\uae30\ub9cc\uc774 \uc720\uc77c\ud55c \ud574\uacb0\ucc45\uc774\ub2e4\"\uace0 \ud3c9\ud588\ub2e4.", "answer": "\ub3c5\ub3c4", "sentence": "\uc774\uc5d0 \ub300\ud574 \ubc15\uc815\ud76c \uc815\uad8c\uc774 \ub300\uc77c\uccad\uad6c\uad8c \ud3ec\uae30\ub9d0\uace0\ub3c4, \ud611\uc0c1\uacfc\uc815\uc5d0\uc11c \uc77c\ubcf8 \uc815\ubd80\ub294 \uc544\uc608 ' \ub3c5\ub3c4 \ub97c \ud3ed\ud30c\ud558\uc790'\uace0 \ud611\ubc15\uae4c\uc9c0 \ud558\uba70 '\ub3c5\ub3c4'\ub97c \ud611\uc0c1\uc548\uac74\uc73c\ub85c \ub123\uc73c\ub824 \ud588\ub2e4.", "paragraph_sentence": "2005\ub144 \ud55c\uc77c\ud611\uc815\ubb38\uc11c\uac00 \uacf5\uac1c\ub410\ub2e4. \uc774\uc5d0 \ub300\ud574 \ubc15\uc815\ud76c \uc815\uad8c\uc774 \ub300\uc77c\uccad\uad6c\uad8c \ud3ec\uae30\ub9d0\uace0\ub3c4, \ud611\uc0c1\uacfc\uc815\uc5d0\uc11c \uc77c\ubcf8 \uc815\ubd80\ub294 \uc544\uc608 ' \ub3c5\ub3c4 \ub97c \ud3ed\ud30c\ud558\uc790'\uace0 \ud611\ubc15\uae4c\uc9c0 \ud558\uba70 '\ub3c5\ub3c4'\ub97c \ud611\uc0c1\uc548\uac74\uc73c\ub85c \ub123\uc73c\ub824 \ud588\ub2e4. \ub610\ud55c \ud55c\uc77c\uc5b4\uc5c5\ud611\uc0c1\uc744 \ub300\uc120\uc5d0 \ud65c\uc6a9\ud558\uace0 \ub300\uc77c\ubcf8 \ubc30\uc0c1\uad00\ub828 \uac1c\uc778\uccad\uad6c\uad8c\uc744 \ubb34\uc2dc\ud55c \uc810\ub3c4 \ub4dc\ub7ec\ub0ac\ub2e4. \ub2f9\uc2dc \uc815\ubd80\ub294 \uae30\uc874\uc758 40\ub9c8\uc77c \uc804\uad00\uc218\uc5ed\uc785\uc7a5\uc5d0\uc11c \ud6c4\ud1f4, \uc77c\ubcf8 \uc815\ubd80\uac00 \uc8fc\uc7a5\ud55c 12\ub9c8\uc77c \uc804\uad00\uc218\uc5ed \ubc29\uc548\uc744 \uc11c\ub458\ub7ec \uc218\uc6a9\ud588\uc73c\ub098 \uc5ec\ub860\uc545\ud654\ub97c \uc6b0\ub824\ud574 \uacf5\uac1c\uc2dc\uae30\ub97c \ub2a6\ucd98 \uac83\uc73c\ub85c \ub098\ud0c0\ub0ac\ub2e4. \uc784\ud5cc\uc601 \ubbfc\uc871\ubb38\uc81c\uc5f0\uad6c\uc18c \uc18c\uc7a5\uc740 \ubc15 \uc815\uad8c\uc774 61\ub144\ubd80\ud130 \ud55c\uc77c\ud611\uc815\uc744 \uccb4\uacb0\ud55c 65\ub144 \uc0ac\uc774 5\ub144\uac04\uc5d0 \uac78\uccd0 6\uac1c\uc758 \uc77c\ubcf8\uae30\uc5c5\ub4e4\ub85c\ubd80\ud130 \ubbfc\uc8fc\uacf5\ud654\ub2f9 \ucd1d\uc608\uc0b0 2/3\uc5d0 \ud574\ub2f9\ud558\ub294 6,600\ub9cc \ub2ec\ub7ec\ub97c \uc81c\uacf5 \ubc1b\uc558\ub2e4\uace0 \uc8fc\uc7a5\ud558\uc600\ub2e4. \uc77c\ubcf8\uce21 \uc678\uad50\ub77c\uc778\uc740 \ub9cc\uc8fc\uc778\ub9e5\uc774\uc5c8\ub2e4. \ud55c\ud638\uc11d \ud1b5\uc77c\ud559\uc5f0\uad6c\uc18c \uc18c\uc7a5\uc740 \"\ub3c5\ub3c4\uc601\uc720\uad8c\ubb38\uc81c\ub97c \ubd88\ubc95\uc801\uc73c\ub85c \ucc98\ub9ac\ud574\ubc84\ub9b0 \uc790\uae30\ub4e4\uc758 \uc8c4\uc0c1\uc774 \uc138\uc0c1\uc5d0 \uc54c\ub824\uc9c0\ub294 \uac83\uc744 \uaebc\ub9b0 \ud55c\uc77c \uc591\uad6d\uc815\ubd80\uc758 \uace0\uc704\uad00\ub9ac\ub4e4\uc740 \ubc00\uc57d\ubb38\uc11c\ub97c \uc601\uc6d0\ud55c \ube44\ubc00\ub85c \ubb3b\uc5b4\ub450\uae30\ub85c \uc57d\uc18d\ud558\uc600\uace0, \ubc00\uc57d\uc774 \uc788\uc5c8\ub2e4\ub294 \uc0ac\uc2e4\uc870\ucc28 \ub9d0\ud558\uc9c0 \uc54a\uc558\ub2e4. \u2018\ud55c\uc77c\ud611\ub825\u2019\uc744 \uc678\uccd0\uc628 \uc5ed\ub300\uc815\uad8c\ub4e4\uc758 \uc740\ud3d0\uc220\uc5d0 \uc138\uc0c1\uc774 \uac10\ucabd\uac19\uc774 \uc18d\uc558\ub358 \uac83\uc774\ub2e4. \uc774 \ub54c\uc758 \ubc00\uc57d \ud30c\uae30\ub9cc\uc774 \uc720\uc77c\ud55c \ud574\uacb0\ucc45\uc774\ub2e4\"\uace0 \ud3c9\ud588\ub2e4.", "paragraph_answer": "2005\ub144 \ud55c\uc77c\ud611\uc815\ubb38\uc11c\uac00 \uacf5\uac1c\ub410\ub2e4. \uc774\uc5d0 \ub300\ud574 \ubc15\uc815\ud76c \uc815\uad8c\uc774 \ub300\uc77c\uccad\uad6c\uad8c \ud3ec\uae30\ub9d0\uace0\ub3c4, \ud611\uc0c1\uacfc\uc815\uc5d0\uc11c \uc77c\ubcf8 \uc815\ubd80\ub294 \uc544\uc608 ' \ub3c5\ub3c4 \ub97c \ud3ed\ud30c\ud558\uc790'\uace0 \ud611\ubc15\uae4c\uc9c0 \ud558\uba70 '\ub3c5\ub3c4'\ub97c \ud611\uc0c1\uc548\uac74\uc73c\ub85c \ub123\uc73c\ub824 \ud588\ub2e4. \ub610\ud55c \ud55c\uc77c\uc5b4\uc5c5\ud611\uc0c1\uc744 \ub300\uc120\uc5d0 \ud65c\uc6a9\ud558\uace0 \ub300\uc77c\ubcf8 \ubc30\uc0c1\uad00\ub828 \uac1c\uc778\uccad\uad6c\uad8c\uc744 \ubb34\uc2dc\ud55c \uc810\ub3c4 \ub4dc\ub7ec\ub0ac\ub2e4. \ub2f9\uc2dc \uc815\ubd80\ub294 \uae30\uc874\uc758 40\ub9c8\uc77c \uc804\uad00\uc218\uc5ed\uc785\uc7a5\uc5d0\uc11c \ud6c4\ud1f4, \uc77c\ubcf8 \uc815\ubd80\uac00 \uc8fc\uc7a5\ud55c 12\ub9c8\uc77c \uc804\uad00\uc218\uc5ed \ubc29\uc548\uc744 \uc11c\ub458\ub7ec \uc218\uc6a9\ud588\uc73c\ub098 \uc5ec\ub860\uc545\ud654\ub97c \uc6b0\ub824\ud574 \uacf5\uac1c\uc2dc\uae30\ub97c \ub2a6\ucd98 \uac83\uc73c\ub85c \ub098\ud0c0\ub0ac\ub2e4. \uc784\ud5cc\uc601 \ubbfc\uc871\ubb38\uc81c\uc5f0\uad6c\uc18c \uc18c\uc7a5\uc740 \ubc15 \uc815\uad8c\uc774 61\ub144\ubd80\ud130 \ud55c\uc77c\ud611\uc815\uc744 \uccb4\uacb0\ud55c 65\ub144 \uc0ac\uc774 5\ub144\uac04\uc5d0 \uac78\uccd0 6\uac1c\uc758 \uc77c\ubcf8\uae30\uc5c5\ub4e4\ub85c\ubd80\ud130 \ubbfc\uc8fc\uacf5\ud654\ub2f9 \ucd1d\uc608\uc0b0 2/3\uc5d0 \ud574\ub2f9\ud558\ub294 6,600\ub9cc \ub2ec\ub7ec\ub97c \uc81c\uacf5 \ubc1b\uc558\ub2e4\uace0 \uc8fc\uc7a5\ud558\uc600\ub2e4. \uc77c\ubcf8\uce21 \uc678\uad50\ub77c\uc778\uc740 \ub9cc\uc8fc\uc778\ub9e5\uc774\uc5c8\ub2e4. \ud55c\ud638\uc11d \ud1b5\uc77c\ud559\uc5f0\uad6c\uc18c \uc18c\uc7a5\uc740 \"\ub3c5\ub3c4\uc601\uc720\uad8c\ubb38\uc81c\ub97c \ubd88\ubc95\uc801\uc73c\ub85c \ucc98\ub9ac\ud574\ubc84\ub9b0 \uc790\uae30\ub4e4\uc758 \uc8c4\uc0c1\uc774 \uc138\uc0c1\uc5d0 \uc54c\ub824\uc9c0\ub294 \uac83\uc744 \uaebc\ub9b0 \ud55c\uc77c \uc591\uad6d\uc815\ubd80\uc758 \uace0\uc704\uad00\ub9ac\ub4e4\uc740 \ubc00\uc57d\ubb38\uc11c\ub97c \uc601\uc6d0\ud55c \ube44\ubc00\ub85c \ubb3b\uc5b4\ub450\uae30\ub85c \uc57d\uc18d\ud558\uc600\uace0, \ubc00\uc57d\uc774 \uc788\uc5c8\ub2e4\ub294 \uc0ac\uc2e4\uc870\ucc28 \ub9d0\ud558\uc9c0 \uc54a\uc558\ub2e4. \u2018\ud55c\uc77c\ud611\ub825\u2019\uc744 \uc678\uccd0\uc628 \uc5ed\ub300\uc815\uad8c\ub4e4\uc758 \uc740\ud3d0\uc220\uc5d0 \uc138\uc0c1\uc774 \uac10\ucabd\uac19\uc774 \uc18d\uc558\ub358 \uac83\uc774\ub2e4. \uc774 \ub54c\uc758 \ubc00\uc57d \ud30c\uae30\ub9cc\uc774 \uc720\uc77c\ud55c \ud574\uacb0\ucc45\uc774\ub2e4\"\uace0 \ud3c9\ud588\ub2e4.", "sentence_answer": "\uc774\uc5d0 \ub300\ud574 \ubc15\uc815\ud76c \uc815\uad8c\uc774 \ub300\uc77c\uccad\uad6c\uad8c \ud3ec\uae30\ub9d0\uace0\ub3c4, \ud611\uc0c1\uacfc\uc815\uc5d0\uc11c \uc77c\ubcf8 \uc815\ubd80\ub294 \uc544\uc608 ' \ub3c5\ub3c4 \ub97c \ud3ed\ud30c\ud558\uc790'\uace0 \ud611\ubc15\uae4c\uc9c0 \ud558\uba70 '\ub3c5\ub3c4'\ub97c \ud611\uc0c1\uc548\uac74\uc73c\ub85c \ub123\uc73c\ub824 \ud588\ub2e4.", "paragraph_id": "6530241-31-1", "paragraph_question": "question: \uc77c\ubcf8 \uc815\ubd80\uc640\uc758 \ud611\uc0c1 \uacfc\uc815 \uc911 \uc77c\ubcf8\uc774 \ud611\uc0c1\uc548\uac74\uc73c\ub85c \ub123\uc73c\ub824 \ud588\ub358 \uac83\uc740?, context: 2005\ub144 \ud55c\uc77c\ud611\uc815\ubb38\uc11c\uac00 \uacf5\uac1c\ub410\ub2e4. \uc774\uc5d0 \ub300\ud574 \ubc15\uc815\ud76c \uc815\uad8c\uc774 \ub300\uc77c\uccad\uad6c\uad8c \ud3ec\uae30\ub9d0\uace0\ub3c4, \ud611\uc0c1\uacfc\uc815\uc5d0\uc11c \uc77c\ubcf8 \uc815\ubd80\ub294 \uc544\uc608 '\ub3c5\ub3c4\ub97c \ud3ed\ud30c\ud558\uc790'\uace0 \ud611\ubc15\uae4c\uc9c0 \ud558\uba70 '\ub3c5\ub3c4'\ub97c \ud611\uc0c1\uc548\uac74\uc73c\ub85c \ub123\uc73c\ub824 \ud588\ub2e4. \ub610\ud55c \ud55c\uc77c\uc5b4\uc5c5\ud611\uc0c1\uc744 \ub300\uc120\uc5d0 \ud65c\uc6a9\ud558\uace0 \ub300\uc77c\ubcf8 \ubc30\uc0c1\uad00\ub828 \uac1c\uc778\uccad\uad6c\uad8c\uc744 \ubb34\uc2dc\ud55c \uc810\ub3c4 \ub4dc\ub7ec\ub0ac\ub2e4. \ub2f9\uc2dc \uc815\ubd80\ub294 \uae30\uc874\uc758 40\ub9c8\uc77c \uc804\uad00\uc218\uc5ed\uc785\uc7a5\uc5d0\uc11c \ud6c4\ud1f4, \uc77c\ubcf8 \uc815\ubd80\uac00 \uc8fc\uc7a5\ud55c 12\ub9c8\uc77c \uc804\uad00\uc218\uc5ed \ubc29\uc548\uc744 \uc11c\ub458\ub7ec \uc218\uc6a9\ud588\uc73c\ub098 \uc5ec\ub860\uc545\ud654\ub97c \uc6b0\ub824\ud574 \uacf5\uac1c\uc2dc\uae30\ub97c \ub2a6\ucd98 \uac83\uc73c\ub85c \ub098\ud0c0\ub0ac\ub2e4. \uc784\ud5cc\uc601 \ubbfc\uc871\ubb38\uc81c\uc5f0\uad6c\uc18c \uc18c\uc7a5\uc740 \ubc15 \uc815\uad8c\uc774 61\ub144\ubd80\ud130 \ud55c\uc77c\ud611\uc815\uc744 \uccb4\uacb0\ud55c 65\ub144 \uc0ac\uc774 5\ub144\uac04\uc5d0 \uac78\uccd0 6\uac1c\uc758 \uc77c\ubcf8\uae30\uc5c5\ub4e4\ub85c\ubd80\ud130 \ubbfc\uc8fc\uacf5\ud654\ub2f9 \ucd1d\uc608\uc0b0 2/3\uc5d0 \ud574\ub2f9\ud558\ub294 6,600\ub9cc \ub2ec\ub7ec\ub97c \uc81c\uacf5 \ubc1b\uc558\ub2e4\uace0 \uc8fc\uc7a5\ud558\uc600\ub2e4. \uc77c\ubcf8\uce21 \uc678\uad50\ub77c\uc778\uc740 \ub9cc\uc8fc\uc778\ub9e5\uc774\uc5c8\ub2e4. \ud55c\ud638\uc11d \ud1b5\uc77c\ud559\uc5f0\uad6c\uc18c \uc18c\uc7a5\uc740 \"\ub3c5\ub3c4\uc601\uc720\uad8c\ubb38\uc81c\ub97c \ubd88\ubc95\uc801\uc73c\ub85c \ucc98\ub9ac\ud574\ubc84\ub9b0 \uc790\uae30\ub4e4\uc758 \uc8c4\uc0c1\uc774 \uc138\uc0c1\uc5d0 \uc54c\ub824\uc9c0\ub294 \uac83\uc744 \uaebc\ub9b0 \ud55c\uc77c \uc591\uad6d\uc815\ubd80\uc758 \uace0\uc704\uad00\ub9ac\ub4e4\uc740 \ubc00\uc57d\ubb38\uc11c\ub97c \uc601\uc6d0\ud55c \ube44\ubc00\ub85c \ubb3b\uc5b4\ub450\uae30\ub85c \uc57d\uc18d\ud558\uc600\uace0, \ubc00\uc57d\uc774 \uc788\uc5c8\ub2e4\ub294 \uc0ac\uc2e4\uc870\ucc28 \ub9d0\ud558\uc9c0 \uc54a\uc558\ub2e4. \u2018\ud55c\uc77c\ud611\ub825\u2019\uc744 \uc678\uccd0\uc628 \uc5ed\ub300\uc815\uad8c\ub4e4\uc758 \uc740\ud3d0\uc220\uc5d0 \uc138\uc0c1\uc774 \uac10\ucabd\uac19\uc774 \uc18d\uc558\ub358 \uac83\uc774\ub2e4. \uc774 \ub54c\uc758 \ubc00\uc57d \ud30c\uae30\ub9cc\uc774 \uc720\uc77c\ud55c \ud574\uacb0\ucc45\uc774\ub2e4\"\uace0 \ud3c9\ud588\ub2e4."} {"question": "\uc601\uad6d \uc815\ubd80\uac00 \uc218\uac10\uc790\uac00 \uc625\uc911\uc5d0\uc11c \ucd9c\ub9c8\ud558\ub294 \uac83\uc744 \ub9c9\uae30 \uc704\ud574 \ud1b5\uacfc \uc2dc\ud0a8 \uac83\uc740?", "paragraph": "\ub2e8\uc2dd\ud22c\uc7c1\uc5d0 \ucc38\uc5ec\ud558\ub358 \uc218\uac10\uc790 \uc544\ud649 \uba85\uc774 \uadf8\ud574 6\uc6d4 \uc544\uc77c\ub79c\ub4dc \uacf5\ud654\uad6d \ucd1d\uc120\uc5d0 \ucd9c\ub9c8\ud588\ub2e4. \ud0a4\uc5b4\ub7f0 \ub3c4\ud5c8\ud2f0(\ub2e8\uc2dd\ud22c\uc7c1 \ucc38\uc5ec\uc790)\uc640 \ud328\ub514 \uc544\uadf8\ub274(\ucc38\uc5ec\uc790 \uc544\ub2d8)\uac00 \uac01\uac01 \uce90\ubc88- \ubaa8\ub098\ud55c \uc120\uac70\uad6c\uc640 \ub77c\uc6b0\uc2a4 \uc120\uac70\uad6c\uc5d0\uc11c \ub2f9\uc120\ub418\uc5c8\uace0, \uc870 \ub9e5\ub3c4\ub12c\uc740 \uc2ac\ub77c\uc774\uace0-\ub9ac\ud2b8\ub9bc \uc120\uac70\uad6c\uc5d0\uc11c \uac04\ubc1c\uc758 \ucc28\uc774\ub85c \ubbf8\ub044\ub7ec\uc84c\ub2e4. \uc774\uc988\uc74c\uc5d0 \ubd81\uc544\uc77c\ub79c\ub4dc\uc5d0\uc11c\ub3c4 \uc9c0\ubc29\uc120\uac70\uac00 \uc788\uc5c8\uace0, IRA\uc640 \uc81c\ud734\ud558\ub294 \uc2e0\ud398\uc778\ub2f9\uc740 \ud6c4\ubcf4\ub97c \ub0b4\uc9c0 \uc54a\uc558\uc9c0\ub9cc \ub2e8\uc2dd\ud22c\uc7c1\uc790\ub4e4\uc744 \uc9c0\uc9c0\ud558\ub294 \uad70\uc18c\uc815\ub2f9 \ud6c4\ubcf4\ub4e4\uc774 \ucd9c\ub9c8\ud558\uc5ec \ub2f9\uc120\ub418\uc5c8\ub2e4. \uc544\uc77c\ub79c\ub4dc \ub3c5\ub9bd\ub2f9\uc774 21\uc11d\uc744 \uc5bb\uc5c8\uace0, \uc544\uc77c\ub79c\ub4dc \uacf5\ud654\uc0ac\ud68c\uc8fc\uc758\uc790\ub2f9(INLA\uc640 \uc81c\ud734\ud558\ub294 \uc6b0\ub2f9)\uacfc \uc778\ubbfc\ubbfc\uc8fc\ub2f9(\ud2b8\ub85c\uce20\ud0a4\uc8fc\uc758 \uc815\ub2f9)\uc774 \uac01\uac01 2\uc11d\uc744 \uc5bb\uc5c8\ub2e4. \uadf8 \uc678\uc5d0\ub3c4 \ub2e8\uc2dd\ud22c\uc7c1\uc744 \uc9c0\uc9c0\ud558\ub294 \uc5ec\ub7ec \ubb34\uc18c\uc18d \ud6c4\ubcf4\ub4e4\uc774 \ub2f9\uc120\ub418\uc5c8\ub2e4. \uc0cc\uc988\uac00 \uc8fd\uc5b4\uc11c \ud37c\ub9e4\ub108 \uc0ac\uc6b0\uc2a4\ud2f0\ub860 \uc120\uac70\uad6c\uc5d0\uc11c 2\ucc28 \ubcf4\uad90\uc120\uac70\uac00 \uc5f4\ub9ac\uac8c \ub418\uc5c8\ub294\ub370, \uc601\uad6d \uc815\ubd80\ub294 \ub610 \ub2e4\ub978 \uc218\uac10\uc790\uac00 \uc625\uc911\uc5d0\uc11c \ucd9c\ub9c8\ud558\ub294 \uac83\uc744 \ub9c9\uae30 \uc704\ud574 \uc7ac\ube68\ub9ac 1981\ub144 \uad6d\ubbfc\ub300\ud45c\uc870\ub840\ub97c \ud1b5\uacfc\uc2dc\ucf30\ub2e4.", "answer": "\uad6d\ubbfc\ub300\ud45c\uc870\ub840", "sentence": "\uc7ac\ube68\ub9ac 1981\ub144 \uad6d\ubbfc\ub300\ud45c\uc870\ub840 \ub97c \ud1b5\uacfc\uc2dc\ucf30\ub2e4.", "paragraph_sentence": "\ub2e8\uc2dd\ud22c\uc7c1\uc5d0 \ucc38\uc5ec\ud558\ub358 \uc218\uac10\uc790 \uc544\ud649 \uba85\uc774 \uadf8\ud574 6\uc6d4 \uc544\uc77c\ub79c\ub4dc \uacf5\ud654\uad6d \ucd1d\uc120\uc5d0 \ucd9c\ub9c8\ud588\ub2e4. \ud0a4\uc5b4\ub7f0 \ub3c4\ud5c8\ud2f0(\ub2e8\uc2dd\ud22c\uc7c1 \ucc38\uc5ec\uc790)\uc640 \ud328\ub514 \uc544\uadf8\ub274(\ucc38\uc5ec\uc790 \uc544\ub2d8)\uac00 \uac01\uac01 \uce90\ubc88- \ubaa8\ub098\ud55c \uc120\uac70\uad6c\uc640 \ub77c\uc6b0\uc2a4 \uc120\uac70\uad6c\uc5d0\uc11c \ub2f9\uc120\ub418\uc5c8\uace0, \uc870 \ub9e5\ub3c4\ub12c\uc740 \uc2ac\ub77c\uc774\uace0-\ub9ac\ud2b8\ub9bc \uc120\uac70\uad6c\uc5d0\uc11c \uac04\ubc1c\uc758 \ucc28\uc774\ub85c \ubbf8\ub044\ub7ec\uc84c\ub2e4. \uc774\uc988\uc74c\uc5d0 \ubd81\uc544\uc77c\ub79c\ub4dc\uc5d0\uc11c\ub3c4 \uc9c0\ubc29\uc120\uac70\uac00 \uc788\uc5c8\uace0, IRA\uc640 \uc81c\ud734\ud558\ub294 \uc2e0\ud398\uc778\ub2f9\uc740 \ud6c4\ubcf4\ub97c \ub0b4\uc9c0 \uc54a\uc558\uc9c0\ub9cc \ub2e8\uc2dd\ud22c\uc7c1\uc790\ub4e4\uc744 \uc9c0\uc9c0\ud558\ub294 \uad70\uc18c\uc815\ub2f9 \ud6c4\ubcf4\ub4e4\uc774 \ucd9c\ub9c8\ud558\uc5ec \ub2f9\uc120\ub418\uc5c8\ub2e4. \uc544\uc77c\ub79c\ub4dc \ub3c5\ub9bd\ub2f9\uc774 21\uc11d\uc744 \uc5bb\uc5c8\uace0, \uc544\uc77c\ub79c\ub4dc \uacf5\ud654\uc0ac\ud68c\uc8fc\uc758\uc790\ub2f9(INLA\uc640 \uc81c\ud734\ud558\ub294 \uc6b0\ub2f9)\uacfc \uc778\ubbfc\ubbfc\uc8fc\ub2f9(\ud2b8\ub85c\uce20\ud0a4\uc8fc\uc758 \uc815\ub2f9)\uc774 \uac01\uac01 2\uc11d\uc744 \uc5bb\uc5c8\ub2e4. \uadf8 \uc678\uc5d0\ub3c4 \ub2e8\uc2dd\ud22c\uc7c1\uc744 \uc9c0\uc9c0\ud558\ub294 \uc5ec\ub7ec \ubb34\uc18c\uc18d \ud6c4\ubcf4\ub4e4\uc774 \ub2f9\uc120\ub418\uc5c8\ub2e4. \uc0cc\uc988\uac00 \uc8fd\uc5b4\uc11c \ud37c\ub9e4\ub108 \uc0ac\uc6b0\uc2a4\ud2f0\ub860 \uc120\uac70\uad6c\uc5d0\uc11c 2\ucc28 \ubcf4\uad90\uc120\uac70\uac00 \uc5f4\ub9ac\uac8c \ub418\uc5c8\ub294\ub370, \uc601\uad6d \uc815\ubd80\ub294 \ub610 \ub2e4\ub978 \uc218\uac10\uc790\uac00 \uc625\uc911\uc5d0\uc11c \ucd9c\ub9c8\ud558\ub294 \uac83\uc744 \ub9c9\uae30 \uc704\ud574 \uc7ac\ube68\ub9ac 1981\ub144 \uad6d\ubbfc\ub300\ud45c\uc870\ub840 \ub97c \ud1b5\uacfc\uc2dc\ucf30\ub2e4. ", "paragraph_answer": "\ub2e8\uc2dd\ud22c\uc7c1\uc5d0 \ucc38\uc5ec\ud558\ub358 \uc218\uac10\uc790 \uc544\ud649 \uba85\uc774 \uadf8\ud574 6\uc6d4 \uc544\uc77c\ub79c\ub4dc \uacf5\ud654\uad6d \ucd1d\uc120\uc5d0 \ucd9c\ub9c8\ud588\ub2e4. \ud0a4\uc5b4\ub7f0 \ub3c4\ud5c8\ud2f0(\ub2e8\uc2dd\ud22c\uc7c1 \ucc38\uc5ec\uc790)\uc640 \ud328\ub514 \uc544\uadf8\ub274(\ucc38\uc5ec\uc790 \uc544\ub2d8)\uac00 \uac01\uac01 \uce90\ubc88- \ubaa8\ub098\ud55c \uc120\uac70\uad6c\uc640 \ub77c\uc6b0\uc2a4 \uc120\uac70\uad6c\uc5d0\uc11c \ub2f9\uc120\ub418\uc5c8\uace0, \uc870 \ub9e5\ub3c4\ub12c\uc740 \uc2ac\ub77c\uc774\uace0-\ub9ac\ud2b8\ub9bc \uc120\uac70\uad6c\uc5d0\uc11c \uac04\ubc1c\uc758 \ucc28\uc774\ub85c \ubbf8\ub044\ub7ec\uc84c\ub2e4. \uc774\uc988\uc74c\uc5d0 \ubd81\uc544\uc77c\ub79c\ub4dc\uc5d0\uc11c\ub3c4 \uc9c0\ubc29\uc120\uac70\uac00 \uc788\uc5c8\uace0, IRA\uc640 \uc81c\ud734\ud558\ub294 \uc2e0\ud398\uc778\ub2f9\uc740 \ud6c4\ubcf4\ub97c \ub0b4\uc9c0 \uc54a\uc558\uc9c0\ub9cc \ub2e8\uc2dd\ud22c\uc7c1\uc790\ub4e4\uc744 \uc9c0\uc9c0\ud558\ub294 \uad70\uc18c\uc815\ub2f9 \ud6c4\ubcf4\ub4e4\uc774 \ucd9c\ub9c8\ud558\uc5ec \ub2f9\uc120\ub418\uc5c8\ub2e4. \uc544\uc77c\ub79c\ub4dc \ub3c5\ub9bd\ub2f9\uc774 21\uc11d\uc744 \uc5bb\uc5c8\uace0, \uc544\uc77c\ub79c\ub4dc \uacf5\ud654\uc0ac\ud68c\uc8fc\uc758\uc790\ub2f9(INLA\uc640 \uc81c\ud734\ud558\ub294 \uc6b0\ub2f9)\uacfc \uc778\ubbfc\ubbfc\uc8fc\ub2f9(\ud2b8\ub85c\uce20\ud0a4\uc8fc\uc758 \uc815\ub2f9)\uc774 \uac01\uac01 2\uc11d\uc744 \uc5bb\uc5c8\ub2e4. \uadf8 \uc678\uc5d0\ub3c4 \ub2e8\uc2dd\ud22c\uc7c1\uc744 \uc9c0\uc9c0\ud558\ub294 \uc5ec\ub7ec \ubb34\uc18c\uc18d \ud6c4\ubcf4\ub4e4\uc774 \ub2f9\uc120\ub418\uc5c8\ub2e4. \uc0cc\uc988\uac00 \uc8fd\uc5b4\uc11c \ud37c\ub9e4\ub108 \uc0ac\uc6b0\uc2a4\ud2f0\ub860 \uc120\uac70\uad6c\uc5d0\uc11c 2\ucc28 \ubcf4\uad90\uc120\uac70\uac00 \uc5f4\ub9ac\uac8c \ub418\uc5c8\ub294\ub370, \uc601\uad6d \uc815\ubd80\ub294 \ub610 \ub2e4\ub978 \uc218\uac10\uc790\uac00 \uc625\uc911\uc5d0\uc11c \ucd9c\ub9c8\ud558\ub294 \uac83\uc744 \ub9c9\uae30 \uc704\ud574 \uc7ac\ube68\ub9ac 1981\ub144 \uad6d\ubbfc\ub300\ud45c\uc870\ub840 \ub97c \ud1b5\uacfc\uc2dc\ucf30\ub2e4.", "sentence_answer": "\uc7ac\ube68\ub9ac 1981\ub144 \uad6d\ubbfc\ub300\ud45c\uc870\ub840 \ub97c \ud1b5\uacfc\uc2dc\ucf30\ub2e4.", "paragraph_id": "6585474-8-2", "paragraph_question": "question: \uc601\uad6d \uc815\ubd80\uac00 \uc218\uac10\uc790\uac00 \uc625\uc911\uc5d0\uc11c \ucd9c\ub9c8\ud558\ub294 \uac83\uc744 \ub9c9\uae30 \uc704\ud574 \ud1b5\uacfc \uc2dc\ud0a8 \uac83\uc740?, context: \ub2e8\uc2dd\ud22c\uc7c1\uc5d0 \ucc38\uc5ec\ud558\ub358 \uc218\uac10\uc790 \uc544\ud649 \uba85\uc774 \uadf8\ud574 6\uc6d4 \uc544\uc77c\ub79c\ub4dc \uacf5\ud654\uad6d \ucd1d\uc120\uc5d0 \ucd9c\ub9c8\ud588\ub2e4. \ud0a4\uc5b4\ub7f0 \ub3c4\ud5c8\ud2f0(\ub2e8\uc2dd\ud22c\uc7c1 \ucc38\uc5ec\uc790)\uc640 \ud328\ub514 \uc544\uadf8\ub274(\ucc38\uc5ec\uc790 \uc544\ub2d8)\uac00 \uac01\uac01 \uce90\ubc88- \ubaa8\ub098\ud55c \uc120\uac70\uad6c\uc640 \ub77c\uc6b0\uc2a4 \uc120\uac70\uad6c\uc5d0\uc11c \ub2f9\uc120\ub418\uc5c8\uace0, \uc870 \ub9e5\ub3c4\ub12c\uc740 \uc2ac\ub77c\uc774\uace0-\ub9ac\ud2b8\ub9bc \uc120\uac70\uad6c\uc5d0\uc11c \uac04\ubc1c\uc758 \ucc28\uc774\ub85c \ubbf8\ub044\ub7ec\uc84c\ub2e4. \uc774\uc988\uc74c\uc5d0 \ubd81\uc544\uc77c\ub79c\ub4dc\uc5d0\uc11c\ub3c4 \uc9c0\ubc29\uc120\uac70\uac00 \uc788\uc5c8\uace0, IRA\uc640 \uc81c\ud734\ud558\ub294 \uc2e0\ud398\uc778\ub2f9\uc740 \ud6c4\ubcf4\ub97c \ub0b4\uc9c0 \uc54a\uc558\uc9c0\ub9cc \ub2e8\uc2dd\ud22c\uc7c1\uc790\ub4e4\uc744 \uc9c0\uc9c0\ud558\ub294 \uad70\uc18c\uc815\ub2f9 \ud6c4\ubcf4\ub4e4\uc774 \ucd9c\ub9c8\ud558\uc5ec \ub2f9\uc120\ub418\uc5c8\ub2e4. \uc544\uc77c\ub79c\ub4dc \ub3c5\ub9bd\ub2f9\uc774 21\uc11d\uc744 \uc5bb\uc5c8\uace0, \uc544\uc77c\ub79c\ub4dc \uacf5\ud654\uc0ac\ud68c\uc8fc\uc758\uc790\ub2f9(INLA\uc640 \uc81c\ud734\ud558\ub294 \uc6b0\ub2f9)\uacfc \uc778\ubbfc\ubbfc\uc8fc\ub2f9(\ud2b8\ub85c\uce20\ud0a4\uc8fc\uc758 \uc815\ub2f9)\uc774 \uac01\uac01 2\uc11d\uc744 \uc5bb\uc5c8\ub2e4. \uadf8 \uc678\uc5d0\ub3c4 \ub2e8\uc2dd\ud22c\uc7c1\uc744 \uc9c0\uc9c0\ud558\ub294 \uc5ec\ub7ec \ubb34\uc18c\uc18d \ud6c4\ubcf4\ub4e4\uc774 \ub2f9\uc120\ub418\uc5c8\ub2e4. \uc0cc\uc988\uac00 \uc8fd\uc5b4\uc11c \ud37c\ub9e4\ub108 \uc0ac\uc6b0\uc2a4\ud2f0\ub860 \uc120\uac70\uad6c\uc5d0\uc11c 2\ucc28 \ubcf4\uad90\uc120\uac70\uac00 \uc5f4\ub9ac\uac8c \ub418\uc5c8\ub294\ub370, \uc601\uad6d \uc815\ubd80\ub294 \ub610 \ub2e4\ub978 \uc218\uac10\uc790\uac00 \uc625\uc911\uc5d0\uc11c \ucd9c\ub9c8\ud558\ub294 \uac83\uc744 \ub9c9\uae30 \uc704\ud574 \uc7ac\ube68\ub9ac 1981\ub144 \uad6d\ubbfc\ub300\ud45c\uc870\ub840\ub97c \ud1b5\uacfc\uc2dc\ucf30\ub2e4."} {"question": "2017\ub144 8\uc6d4 7\uc77c \uc6cc\ub108\uc6d0\uc758 \ub370\ubdd4 \uc568\ubc94\uc758 \uc774\ub984\uc740?", "paragraph": "2017\ub144 8\uc6d4 7\uc77c, \ub370\ubdd4 \uc568\ubc94\uc774\uc790 \uccab \ubc88\uc9f8 \ubbf8\ub2c8 \uc568\ubc94 \u300a1X1=1 (TO BE ONE)\u300b\uc744 \ubc1c\ub9e4\ud558\uace0 \uace0\ucc99\uc2a4\uce74\uc774\ub3d4\uc5d0\uc11c \ub370\ubdd4 \uc1fc\ucf00\uc774\uc2a4 \ucf58\uc11c\ud2b8 \u3008\uc6cc\ub108\uc6d0 \ud504\ub9ac\ubbf8\uc5b4 \uc1fc\ucf58\u3009\uc744 \uac1c\ucd5c\ud558\uba70 \uacf5\uc2dd\uc801\uc73c\ub85c \ub370\ubdd4\ud558\uc600\ub2e4. \ubc1c\ub9e4 \uc9c1\ud6c4, \ud0c0\uc774\ud2c0\uace1 '\uc5d0\ub108\uc81c\ud2f1 (Energetic)'\uc740 \ubaa8\ub4e0 \uc74c\uc6d0 \ucc28\ud2b8\uc5d0\uc11c 1\uc704\ub97c \uae30\ub85d\ud558\uace0, '\ud65c\ud65c (Burn It Up)'\uc744 \ube44\ub86f\ud55c \uc218\ub85d\uace1\ub4e4\ub3c4 10\uc704\uad8c \ub0b4\uc5d0 \uc9c4\uc785\ud558\uc600\ub2e4. \uc74c\ubc18\ub3c4 \ubaa8\ub4e0 \uc74c\ubc18 \ucc28\ud2b8\uc5d0\uc11c 1\uc704\ub97c \uae30\ub85d\ud558\uace0, \ubc1c\ub9e4 \uccab \uc8fc 41\ub9cc \uc7a5\uc744 \ud310\ub9e4\ud574 \uc5ed\ub300 \ub300\ud55c\ubbfc\uad6d \uc544\uc774\ub3cc \uadf8\ub8f9 \uc74c\ubc18 \ucd08\ub3d9 5\uc704 (\ubc1c\ub9e4 \ub2f9\uc2dc \ucd08\ub3d9 3\uc704)\ub97c \uae30\ub85d\ud588\uc73c\uba70, \ud604\uc7ac\uae4c\uc9c0 76\ub9cc \uc7a5\uc744 \ud310\ub9e4\ud558\uc600\ub2e4. \ub610\ud55c, \uc74c\uc545 \ubc29\uc1a1\uc5d0\uc11c 1\uc704\ub97c \ud729\uc4f8\uc5b4 15\uad00\uc655\uc744 \ub2ec\uc131\ud588\ub2e4. \uac19\uc740 \ud574 11\uc6d4 13\uc77c, \uccab \ubc88\uc9f8 \ub9ac\ud328\ud0a4\uc9c0 \uc568\ubc94 \u300a1-1=0 (NOTHING WITHOUT YOU)\u300b\ub97c \ubc1c\ub9e4\ud558\uc600\ub2e4. \ubc1c\ub9e4 \uc9c1\ud6c4, \ud0c0\uc774\ud2c0\uace1 'Beautiful'\uc740 \ubaa8\ub4e0 \uc74c\uc6d0 \ucc28\ud2b8\uc5d0\uc11c 1\uc704\ub97c \uae30\ub85d\ud558\uace0, '\uac16\uace0 \uc2f6\uc5b4'\ub97c \ube44\ub86f\ud55c \uc218\ub85d\uace1\ub4e4\ub3c4 10\uc704\uad8c \ub0b4\uc5d0 \uc9c4\uc785\ud558\uc600\ub2e4. \uc74c\ubc18\ub3c4 \ubaa8\ub4e0 \uc74c\ubc18 \ucc28\ud2b8\uc5d0\uc11c 1\uc704\ub97c \uae30\ub85d\ud558\uace0, \ubc1c\ub9e4 \uccab \uc8fc 41\ub9cc \uc7a5\uc744 \ud310\ub9e4\ud574 \uc5ed\ub300 \ub300\ud55c\ubbfc\uad6d \uc544\uc774\ub3cc \uadf8\ub8f9 \uc74c\ubc18 \ucd08\ub3d9 4\uc704\ub97c \uae30\ub85d\ud588\uc73c\uba70, \ud604\uc7ac\uae4c\uc9c0 64\ub9cc \uc7a5\uc744 \ud310\ub9e4\ud558\uc600\ub2e4. \ub610\ud55c, \uc74c\uc545 \ubc29\uc1a1\uc5d0\uc11c 1\uc704\ub97c \ud729\uc4f8\uc5b4 8\uad00\uc655\uc744 \ub2ec\uc131\ud588\ub2e4. \uac19\uc740 \ud574 12\uc6d4 9\uc77c, \uacf5\uc2dd \uc751\uc6d0\ubd09 \ub108\ube14\ubd09\uc744 \ube44\ub86f\ud55c \uacf5\uc2dd \uad7f\uc988\uac00 \uacf5\uac1c\ub418\uc5c8\uc73c\uba70, 12\uc6d4 15\uc77c\ubd80\ud130 17\uc77c, 23\uc77c\ubd80\ud130 24\uc77c\uae4c\uc9c0 \ud32c\ubbf8\ud305 \ucf58\uc11c\ud2b8 \u3008\uc6cc\ub108\uc6d0 \ud504\ub9ac\ubbf8\uc5b4 \ud32c\ucf58\u3009\uc744 \uac1c\ucd5c\ud558\uc600\ub2e4. 2017\ub144 \ud55c \ud574 \ub3d9\uc548 \uc6cc\ub108\uc6d0\uc740 \ubd80\uc0b0 \uc6d0\uc544\uc2dc\uc544\ud398\uc2a4\ud2f0\ubc8c \uc5b4\uc6cc\uc988\uc5d0\uc11c \ub77c\uc774\uc9d5 \uc2a4\ud0c0\uc0c1\uc744 \uc218\uc0c1\ud558\uace0, \uc8fc\uac04\uc544\uc774\ub3cc \uc5b4\uc6cc\uc988\uc5d0\uc11c \uc544\uc774\ub3cc \ucc54\ud504\uc0c1\uc744 \uc218\uc0c1\ud588\uc73c\uba70, \ub300\ud55c\ubbfc\uad6d \ud1b1\uc2a4\ud0c0\uc0c1 \uc2dc\uc0c1\uc2dd\uc5d0\uc11c \ud1b1\uac00\uc218\uc0c1\uc744 \uc218\uc0c1\ud588\ub2e4. \ub610\ud55c, \uc18c\ub9ac\ubc14\ub2e4 \ubca0\uc2a4\ud2b8 \ucf00\uc774\ubba4\uc9c1 \uc5b4\uc6cc\uc988\uc5d0\uc11c \uc2e0\ud55c\ub958 \uc2e0\uc778\uc0c1\uacfc \uc2e0\ud55c\ub958 \ub77c\uc774\uc9d5 \ud56b \uc2a4\ud0c0\uc0c1\uc744 \uc218\uc0c1\ud558\uace0, \uc544\uc2dc\uc544 \uc544\ud2f0\uc2a4\ud2b8 \uc5b4\uc6cc\uc988\uc5d0\uc11c\ub3c4 \uc2e0\uc778\uc0c1\uacfc \uc288\ud37c\ub8e8\ud0a4 \uc0bc\uc131\ud398\uc774\uc0c1\uc744 \uc218\uc0c1\ud574 \uac01\uac01 2\uad00\uc655\uc744 \ub2ec\uc131\ud588\ub2e4. \uc774\uc5b4 \uc5e0\ub137 \uc544\uc2dc\uc548 \ubba4\uc9c1 \uc5b4\uc6cc\ub4dc\uc5d0\uc11c \ub0a8\uc790 \uadf8\ub8f9\uc0c1, \ub0a8\uc790 \uc2e0\uc778\uc0c1, \ubca0\uc2a4\ud2b8 \uc624\ube0c \ub125\uc2a4\ud2b8\uc0c1\uc744 \uc218\uc0c1\ud558\uace0, \uba5c\ub860 \ubba4\uc9c1 \uc5b4\uc6cc\ub4dc\uc5d0\uc11c\ub3c4 Top 10, \uc2e0\uc778\uc0c1, \uce74\uce74\uc624 \ud56b \uc2a4\ud0c0\uc0c1\uc744 \uc218\uc0c1\ud574 \uac01\uac01 3\uad00\uc655\uc744 \ub2ec\uc131\ud588\ub2e4. \uc6cc\ub108\uc6d0\uc740 \ub370\ubdd4 \uc568\ubc94\uacfc \ub9ac\ud328\ud0a4\uc9c0 \uc568\ubc94 \ud310\ub9e4\ub7c9\uc774 100\ub9cc\uc7a5\uc744 \ub118\uc5b4 2000\ub144 \uc774\ud6c4 \ucd5c\ucd08\ub85c \ub300\ud55c\ubbfc\uad6d \uc544\uc774\ub3cc \uadf8\ub8f9 \ub370\ubdd4 \uc568\ubc94 \ubc00\ub9ac\uc5b8\uc140\ub7ec\uac00 \ub418\uc5c8\ub2e4.", "answer": "1X1=1 (TO BE ONE)", "sentence": "2017\ub144 8\uc6d4 7\uc77c, \ub370\ubdd4 \uc568\ubc94\uc774\uc790 \uccab \ubc88\uc9f8 \ubbf8\ub2c8 \uc568\ubc94 \u300a 1X1=1 (TO BE ONE) \u300b\uc744 \ubc1c\ub9e4\ud558\uace0 \uace0\ucc99\uc2a4\uce74\uc774\ub3d4\uc5d0\uc11c \ub370\ubdd4 \uc1fc\ucf00\uc774\uc2a4 \ucf58\uc11c\ud2b8 \u3008\uc6cc\ub108\uc6d0 \ud504\ub9ac\ubbf8\uc5b4 \uc1fc\ucf58\u3009\uc744 \uac1c\ucd5c\ud558\uba70 \uacf5\uc2dd\uc801\uc73c\ub85c \ub370\ubdd4\ud558\uc600\ub2e4.", "paragraph_sentence": " 2017\ub144 8\uc6d4 7\uc77c, \ub370\ubdd4 \uc568\ubc94\uc774\uc790 \uccab \ubc88\uc9f8 \ubbf8\ub2c8 \uc568\ubc94 \u300a 1X1=1 (TO BE ONE) \u300b\uc744 \ubc1c\ub9e4\ud558\uace0 \uace0\ucc99\uc2a4\uce74\uc774\ub3d4\uc5d0\uc11c \ub370\ubdd4 \uc1fc\ucf00\uc774\uc2a4 \ucf58\uc11c\ud2b8 \u3008\uc6cc\ub108\uc6d0 \ud504\ub9ac\ubbf8\uc5b4 \uc1fc\ucf58\u3009\uc744 \uac1c\ucd5c\ud558\uba70 \uacf5\uc2dd\uc801\uc73c\ub85c \ub370\ubdd4\ud558\uc600\ub2e4. \ubc1c\ub9e4 \uc9c1\ud6c4, \ud0c0\uc774\ud2c0\uace1 '\uc5d0\ub108\uc81c\ud2f1 (Energetic)'\uc740 \ubaa8\ub4e0 \uc74c\uc6d0 \ucc28\ud2b8\uc5d0\uc11c 1\uc704\ub97c \uae30\ub85d\ud558\uace0, '\ud65c\ud65c (Burn It Up)'\uc744 \ube44\ub86f\ud55c \uc218\ub85d\uace1\ub4e4\ub3c4 10\uc704\uad8c \ub0b4\uc5d0 \uc9c4\uc785\ud558\uc600\ub2e4. \uc74c\ubc18\ub3c4 \ubaa8\ub4e0 \uc74c\ubc18 \ucc28\ud2b8\uc5d0\uc11c 1\uc704\ub97c \uae30\ub85d\ud558\uace0, \ubc1c\ub9e4 \uccab \uc8fc 41\ub9cc \uc7a5\uc744 \ud310\ub9e4\ud574 \uc5ed\ub300 \ub300\ud55c\ubbfc\uad6d \uc544\uc774\ub3cc \uadf8\ub8f9 \uc74c\ubc18 \ucd08\ub3d9 5\uc704 (\ubc1c\ub9e4 \ub2f9\uc2dc \ucd08\ub3d9 3\uc704)\ub97c \uae30\ub85d\ud588\uc73c\uba70, \ud604\uc7ac\uae4c\uc9c0 76\ub9cc \uc7a5\uc744 \ud310\ub9e4\ud558\uc600\ub2e4. \ub610\ud55c, \uc74c\uc545 \ubc29\uc1a1\uc5d0\uc11c 1\uc704\ub97c \ud729\uc4f8\uc5b4 15\uad00\uc655\uc744 \ub2ec\uc131\ud588\ub2e4. \uac19\uc740 \ud574 11\uc6d4 13\uc77c, \uccab \ubc88\uc9f8 \ub9ac\ud328\ud0a4\uc9c0 \uc568\ubc94 \u300a1-1=0 (NOTHING WITHOUT YOU)\u300b\ub97c \ubc1c\ub9e4\ud558\uc600\ub2e4. \ubc1c\ub9e4 \uc9c1\ud6c4, \ud0c0\uc774\ud2c0\uace1 'Beautiful'\uc740 \ubaa8\ub4e0 \uc74c\uc6d0 \ucc28\ud2b8\uc5d0\uc11c 1\uc704\ub97c \uae30\ub85d\ud558\uace0, '\uac16\uace0 \uc2f6\uc5b4'\ub97c \ube44\ub86f\ud55c \uc218\ub85d\uace1\ub4e4\ub3c4 10\uc704\uad8c \ub0b4\uc5d0 \uc9c4\uc785\ud558\uc600\ub2e4. \uc74c\ubc18\ub3c4 \ubaa8\ub4e0 \uc74c\ubc18 \ucc28\ud2b8\uc5d0\uc11c 1\uc704\ub97c \uae30\ub85d\ud558\uace0, \ubc1c\ub9e4 \uccab \uc8fc 41\ub9cc \uc7a5\uc744 \ud310\ub9e4\ud574 \uc5ed\ub300 \ub300\ud55c\ubbfc\uad6d \uc544\uc774\ub3cc \uadf8\ub8f9 \uc74c\ubc18 \ucd08\ub3d9 4\uc704\ub97c \uae30\ub85d\ud588\uc73c\uba70, \ud604\uc7ac\uae4c\uc9c0 64\ub9cc \uc7a5\uc744 \ud310\ub9e4\ud558\uc600\ub2e4. \ub610\ud55c, \uc74c\uc545 \ubc29\uc1a1\uc5d0\uc11c 1\uc704\ub97c \ud729\uc4f8\uc5b4 8\uad00\uc655\uc744 \ub2ec\uc131\ud588\ub2e4. \uac19\uc740 \ud574 12\uc6d4 9\uc77c, \uacf5\uc2dd \uc751\uc6d0\ubd09 \ub108\ube14\ubd09\uc744 \ube44\ub86f\ud55c \uacf5\uc2dd \uad7f\uc988\uac00 \uacf5\uac1c\ub418\uc5c8\uc73c\uba70, 12\uc6d4 15\uc77c\ubd80\ud130 17\uc77c, 23\uc77c\ubd80\ud130 24\uc77c\uae4c\uc9c0 \ud32c\ubbf8\ud305 \ucf58\uc11c\ud2b8 \u3008\uc6cc\ub108\uc6d0 \ud504\ub9ac\ubbf8\uc5b4 \ud32c\ucf58\u3009\uc744 \uac1c\ucd5c\ud558\uc600\ub2e4. 2017\ub144 \ud55c \ud574 \ub3d9\uc548 \uc6cc\ub108\uc6d0\uc740 \ubd80\uc0b0 \uc6d0\uc544\uc2dc\uc544\ud398\uc2a4\ud2f0\ubc8c \uc5b4\uc6cc\uc988\uc5d0\uc11c \ub77c\uc774\uc9d5 \uc2a4\ud0c0\uc0c1\uc744 \uc218\uc0c1\ud558\uace0, \uc8fc\uac04\uc544\uc774\ub3cc \uc5b4\uc6cc\uc988\uc5d0\uc11c \uc544\uc774\ub3cc \ucc54\ud504\uc0c1\uc744 \uc218\uc0c1\ud588\uc73c\uba70, \ub300\ud55c\ubbfc\uad6d \ud1b1\uc2a4\ud0c0\uc0c1 \uc2dc\uc0c1\uc2dd\uc5d0\uc11c \ud1b1\uac00\uc218\uc0c1\uc744 \uc218\uc0c1\ud588\ub2e4. \ub610\ud55c, \uc18c\ub9ac\ubc14\ub2e4 \ubca0\uc2a4\ud2b8 \ucf00\uc774\ubba4\uc9c1 \uc5b4\uc6cc\uc988\uc5d0\uc11c \uc2e0\ud55c\ub958 \uc2e0\uc778\uc0c1\uacfc \uc2e0\ud55c\ub958 \ub77c\uc774\uc9d5 \ud56b \uc2a4\ud0c0\uc0c1\uc744 \uc218\uc0c1\ud558\uace0, \uc544\uc2dc\uc544 \uc544\ud2f0\uc2a4\ud2b8 \uc5b4\uc6cc\uc988\uc5d0\uc11c\ub3c4 \uc2e0\uc778\uc0c1\uacfc \uc288\ud37c\ub8e8\ud0a4 \uc0bc\uc131\ud398\uc774\uc0c1\uc744 \uc218\uc0c1\ud574 \uac01\uac01 2\uad00\uc655\uc744 \ub2ec\uc131\ud588\ub2e4. \uc774\uc5b4 \uc5e0\ub137 \uc544\uc2dc\uc548 \ubba4\uc9c1 \uc5b4\uc6cc\ub4dc\uc5d0\uc11c \ub0a8\uc790 \uadf8\ub8f9\uc0c1, \ub0a8\uc790 \uc2e0\uc778\uc0c1, \ubca0\uc2a4\ud2b8 \uc624\ube0c \ub125\uc2a4\ud2b8\uc0c1\uc744 \uc218\uc0c1\ud558\uace0, \uba5c\ub860 \ubba4\uc9c1 \uc5b4\uc6cc\ub4dc\uc5d0\uc11c\ub3c4 Top 10, \uc2e0\uc778\uc0c1, \uce74\uce74\uc624 \ud56b \uc2a4\ud0c0\uc0c1\uc744 \uc218\uc0c1\ud574 \uac01\uac01 3\uad00\uc655\uc744 \ub2ec\uc131\ud588\ub2e4. \uc6cc\ub108\uc6d0\uc740 \ub370\ubdd4 \uc568\ubc94\uacfc \ub9ac\ud328\ud0a4\uc9c0 \uc568\ubc94 \ud310\ub9e4\ub7c9\uc774 100\ub9cc\uc7a5\uc744 \ub118\uc5b4 2000\ub144 \uc774\ud6c4 \ucd5c\ucd08\ub85c \ub300\ud55c\ubbfc\uad6d \uc544\uc774\ub3cc \uadf8\ub8f9 \ub370\ubdd4 \uc568\ubc94 \ubc00\ub9ac\uc5b8\uc140\ub7ec\uac00 \ub418\uc5c8\ub2e4.", "paragraph_answer": "2017\ub144 8\uc6d4 7\uc77c, \ub370\ubdd4 \uc568\ubc94\uc774\uc790 \uccab \ubc88\uc9f8 \ubbf8\ub2c8 \uc568\ubc94 \u300a 1X1=1 (TO BE ONE) \u300b\uc744 \ubc1c\ub9e4\ud558\uace0 \uace0\ucc99\uc2a4\uce74\uc774\ub3d4\uc5d0\uc11c \ub370\ubdd4 \uc1fc\ucf00\uc774\uc2a4 \ucf58\uc11c\ud2b8 \u3008\uc6cc\ub108\uc6d0 \ud504\ub9ac\ubbf8\uc5b4 \uc1fc\ucf58\u3009\uc744 \uac1c\ucd5c\ud558\uba70 \uacf5\uc2dd\uc801\uc73c\ub85c \ub370\ubdd4\ud558\uc600\ub2e4. \ubc1c\ub9e4 \uc9c1\ud6c4, \ud0c0\uc774\ud2c0\uace1 '\uc5d0\ub108\uc81c\ud2f1 (Energetic)'\uc740 \ubaa8\ub4e0 \uc74c\uc6d0 \ucc28\ud2b8\uc5d0\uc11c 1\uc704\ub97c \uae30\ub85d\ud558\uace0, '\ud65c\ud65c (Burn It Up)'\uc744 \ube44\ub86f\ud55c \uc218\ub85d\uace1\ub4e4\ub3c4 10\uc704\uad8c \ub0b4\uc5d0 \uc9c4\uc785\ud558\uc600\ub2e4. \uc74c\ubc18\ub3c4 \ubaa8\ub4e0 \uc74c\ubc18 \ucc28\ud2b8\uc5d0\uc11c 1\uc704\ub97c \uae30\ub85d\ud558\uace0, \ubc1c\ub9e4 \uccab \uc8fc 41\ub9cc \uc7a5\uc744 \ud310\ub9e4\ud574 \uc5ed\ub300 \ub300\ud55c\ubbfc\uad6d \uc544\uc774\ub3cc \uadf8\ub8f9 \uc74c\ubc18 \ucd08\ub3d9 5\uc704 (\ubc1c\ub9e4 \ub2f9\uc2dc \ucd08\ub3d9 3\uc704)\ub97c \uae30\ub85d\ud588\uc73c\uba70, \ud604\uc7ac\uae4c\uc9c0 76\ub9cc \uc7a5\uc744 \ud310\ub9e4\ud558\uc600\ub2e4. \ub610\ud55c, \uc74c\uc545 \ubc29\uc1a1\uc5d0\uc11c 1\uc704\ub97c \ud729\uc4f8\uc5b4 15\uad00\uc655\uc744 \ub2ec\uc131\ud588\ub2e4. \uac19\uc740 \ud574 11\uc6d4 13\uc77c, \uccab \ubc88\uc9f8 \ub9ac\ud328\ud0a4\uc9c0 \uc568\ubc94 \u300a1-1=0 (NOTHING WITHOUT YOU)\u300b\ub97c \ubc1c\ub9e4\ud558\uc600\ub2e4. \ubc1c\ub9e4 \uc9c1\ud6c4, \ud0c0\uc774\ud2c0\uace1 'Beautiful'\uc740 \ubaa8\ub4e0 \uc74c\uc6d0 \ucc28\ud2b8\uc5d0\uc11c 1\uc704\ub97c \uae30\ub85d\ud558\uace0, '\uac16\uace0 \uc2f6\uc5b4'\ub97c \ube44\ub86f\ud55c \uc218\ub85d\uace1\ub4e4\ub3c4 10\uc704\uad8c \ub0b4\uc5d0 \uc9c4\uc785\ud558\uc600\ub2e4. \uc74c\ubc18\ub3c4 \ubaa8\ub4e0 \uc74c\ubc18 \ucc28\ud2b8\uc5d0\uc11c 1\uc704\ub97c \uae30\ub85d\ud558\uace0, \ubc1c\ub9e4 \uccab \uc8fc 41\ub9cc \uc7a5\uc744 \ud310\ub9e4\ud574 \uc5ed\ub300 \ub300\ud55c\ubbfc\uad6d \uc544\uc774\ub3cc \uadf8\ub8f9 \uc74c\ubc18 \ucd08\ub3d9 4\uc704\ub97c \uae30\ub85d\ud588\uc73c\uba70, \ud604\uc7ac\uae4c\uc9c0 64\ub9cc \uc7a5\uc744 \ud310\ub9e4\ud558\uc600\ub2e4. \ub610\ud55c, \uc74c\uc545 \ubc29\uc1a1\uc5d0\uc11c 1\uc704\ub97c \ud729\uc4f8\uc5b4 8\uad00\uc655\uc744 \ub2ec\uc131\ud588\ub2e4. \uac19\uc740 \ud574 12\uc6d4 9\uc77c, \uacf5\uc2dd \uc751\uc6d0\ubd09 \ub108\ube14\ubd09\uc744 \ube44\ub86f\ud55c \uacf5\uc2dd \uad7f\uc988\uac00 \uacf5\uac1c\ub418\uc5c8\uc73c\uba70, 12\uc6d4 15\uc77c\ubd80\ud130 17\uc77c, 23\uc77c\ubd80\ud130 24\uc77c\uae4c\uc9c0 \ud32c\ubbf8\ud305 \ucf58\uc11c\ud2b8 \u3008\uc6cc\ub108\uc6d0 \ud504\ub9ac\ubbf8\uc5b4 \ud32c\ucf58\u3009\uc744 \uac1c\ucd5c\ud558\uc600\ub2e4. 2017\ub144 \ud55c \ud574 \ub3d9\uc548 \uc6cc\ub108\uc6d0\uc740 \ubd80\uc0b0 \uc6d0\uc544\uc2dc\uc544\ud398\uc2a4\ud2f0\ubc8c \uc5b4\uc6cc\uc988\uc5d0\uc11c \ub77c\uc774\uc9d5 \uc2a4\ud0c0\uc0c1\uc744 \uc218\uc0c1\ud558\uace0, \uc8fc\uac04\uc544\uc774\ub3cc \uc5b4\uc6cc\uc988\uc5d0\uc11c \uc544\uc774\ub3cc \ucc54\ud504\uc0c1\uc744 \uc218\uc0c1\ud588\uc73c\uba70, \ub300\ud55c\ubbfc\uad6d \ud1b1\uc2a4\ud0c0\uc0c1 \uc2dc\uc0c1\uc2dd\uc5d0\uc11c \ud1b1\uac00\uc218\uc0c1\uc744 \uc218\uc0c1\ud588\ub2e4. \ub610\ud55c, \uc18c\ub9ac\ubc14\ub2e4 \ubca0\uc2a4\ud2b8 \ucf00\uc774\ubba4\uc9c1 \uc5b4\uc6cc\uc988\uc5d0\uc11c \uc2e0\ud55c\ub958 \uc2e0\uc778\uc0c1\uacfc \uc2e0\ud55c\ub958 \ub77c\uc774\uc9d5 \ud56b \uc2a4\ud0c0\uc0c1\uc744 \uc218\uc0c1\ud558\uace0, \uc544\uc2dc\uc544 \uc544\ud2f0\uc2a4\ud2b8 \uc5b4\uc6cc\uc988\uc5d0\uc11c\ub3c4 \uc2e0\uc778\uc0c1\uacfc \uc288\ud37c\ub8e8\ud0a4 \uc0bc\uc131\ud398\uc774\uc0c1\uc744 \uc218\uc0c1\ud574 \uac01\uac01 2\uad00\uc655\uc744 \ub2ec\uc131\ud588\ub2e4. \uc774\uc5b4 \uc5e0\ub137 \uc544\uc2dc\uc548 \ubba4\uc9c1 \uc5b4\uc6cc\ub4dc\uc5d0\uc11c \ub0a8\uc790 \uadf8\ub8f9\uc0c1, \ub0a8\uc790 \uc2e0\uc778\uc0c1, \ubca0\uc2a4\ud2b8 \uc624\ube0c \ub125\uc2a4\ud2b8\uc0c1\uc744 \uc218\uc0c1\ud558\uace0, \uba5c\ub860 \ubba4\uc9c1 \uc5b4\uc6cc\ub4dc\uc5d0\uc11c\ub3c4 Top 10, \uc2e0\uc778\uc0c1, \uce74\uce74\uc624 \ud56b \uc2a4\ud0c0\uc0c1\uc744 \uc218\uc0c1\ud574 \uac01\uac01 3\uad00\uc655\uc744 \ub2ec\uc131\ud588\ub2e4. \uc6cc\ub108\uc6d0\uc740 \ub370\ubdd4 \uc568\ubc94\uacfc \ub9ac\ud328\ud0a4\uc9c0 \uc568\ubc94 \ud310\ub9e4\ub7c9\uc774 100\ub9cc\uc7a5\uc744 \ub118\uc5b4 2000\ub144 \uc774\ud6c4 \ucd5c\ucd08\ub85c \ub300\ud55c\ubbfc\uad6d \uc544\uc774\ub3cc \uadf8\ub8f9 \ub370\ubdd4 \uc568\ubc94 \ubc00\ub9ac\uc5b8\uc140\ub7ec\uac00 \ub418\uc5c8\ub2e4.", "sentence_answer": "2017\ub144 8\uc6d4 7\uc77c, \ub370\ubdd4 \uc568\ubc94\uc774\uc790 \uccab \ubc88\uc9f8 \ubbf8\ub2c8 \uc568\ubc94 \u300a 1X1=1 (TO BE ONE) \u300b\uc744 \ubc1c\ub9e4\ud558\uace0 \uace0\ucc99\uc2a4\uce74\uc774\ub3d4\uc5d0\uc11c \ub370\ubdd4 \uc1fc\ucf00\uc774\uc2a4 \ucf58\uc11c\ud2b8 \u3008\uc6cc\ub108\uc6d0 \ud504\ub9ac\ubbf8\uc5b4 \uc1fc\ucf58\u3009\uc744 \uac1c\ucd5c\ud558\uba70 \uacf5\uc2dd\uc801\uc73c\ub85c \ub370\ubdd4\ud558\uc600\ub2e4.", "paragraph_id": "6526496-1-0", "paragraph_question": "question: 2017\ub144 8\uc6d4 7\uc77c \uc6cc\ub108\uc6d0\uc758 \ub370\ubdd4 \uc568\ubc94\uc758 \uc774\ub984\uc740?, context: 2017\ub144 8\uc6d4 7\uc77c, \ub370\ubdd4 \uc568\ubc94\uc774\uc790 \uccab \ubc88\uc9f8 \ubbf8\ub2c8 \uc568\ubc94 \u300a1X1=1 (TO BE ONE)\u300b\uc744 \ubc1c\ub9e4\ud558\uace0 \uace0\ucc99\uc2a4\uce74\uc774\ub3d4\uc5d0\uc11c \ub370\ubdd4 \uc1fc\ucf00\uc774\uc2a4 \ucf58\uc11c\ud2b8 \u3008\uc6cc\ub108\uc6d0 \ud504\ub9ac\ubbf8\uc5b4 \uc1fc\ucf58\u3009\uc744 \uac1c\ucd5c\ud558\uba70 \uacf5\uc2dd\uc801\uc73c\ub85c \ub370\ubdd4\ud558\uc600\ub2e4. \ubc1c\ub9e4 \uc9c1\ud6c4, \ud0c0\uc774\ud2c0\uace1 '\uc5d0\ub108\uc81c\ud2f1 (Energetic)'\uc740 \ubaa8\ub4e0 \uc74c\uc6d0 \ucc28\ud2b8\uc5d0\uc11c 1\uc704\ub97c \uae30\ub85d\ud558\uace0, '\ud65c\ud65c (Burn It Up)'\uc744 \ube44\ub86f\ud55c \uc218\ub85d\uace1\ub4e4\ub3c4 10\uc704\uad8c \ub0b4\uc5d0 \uc9c4\uc785\ud558\uc600\ub2e4. \uc74c\ubc18\ub3c4 \ubaa8\ub4e0 \uc74c\ubc18 \ucc28\ud2b8\uc5d0\uc11c 1\uc704\ub97c \uae30\ub85d\ud558\uace0, \ubc1c\ub9e4 \uccab \uc8fc 41\ub9cc \uc7a5\uc744 \ud310\ub9e4\ud574 \uc5ed\ub300 \ub300\ud55c\ubbfc\uad6d \uc544\uc774\ub3cc \uadf8\ub8f9 \uc74c\ubc18 \ucd08\ub3d9 5\uc704 (\ubc1c\ub9e4 \ub2f9\uc2dc \ucd08\ub3d9 3\uc704)\ub97c \uae30\ub85d\ud588\uc73c\uba70, \ud604\uc7ac\uae4c\uc9c0 76\ub9cc \uc7a5\uc744 \ud310\ub9e4\ud558\uc600\ub2e4. \ub610\ud55c, \uc74c\uc545 \ubc29\uc1a1\uc5d0\uc11c 1\uc704\ub97c \ud729\uc4f8\uc5b4 15\uad00\uc655\uc744 \ub2ec\uc131\ud588\ub2e4. \uac19\uc740 \ud574 11\uc6d4 13\uc77c, \uccab \ubc88\uc9f8 \ub9ac\ud328\ud0a4\uc9c0 \uc568\ubc94 \u300a1-1=0 (NOTHING WITHOUT YOU)\u300b\ub97c \ubc1c\ub9e4\ud558\uc600\ub2e4. \ubc1c\ub9e4 \uc9c1\ud6c4, \ud0c0\uc774\ud2c0\uace1 'Beautiful'\uc740 \ubaa8\ub4e0 \uc74c\uc6d0 \ucc28\ud2b8\uc5d0\uc11c 1\uc704\ub97c \uae30\ub85d\ud558\uace0, '\uac16\uace0 \uc2f6\uc5b4'\ub97c \ube44\ub86f\ud55c \uc218\ub85d\uace1\ub4e4\ub3c4 10\uc704\uad8c \ub0b4\uc5d0 \uc9c4\uc785\ud558\uc600\ub2e4. \uc74c\ubc18\ub3c4 \ubaa8\ub4e0 \uc74c\ubc18 \ucc28\ud2b8\uc5d0\uc11c 1\uc704\ub97c \uae30\ub85d\ud558\uace0, \ubc1c\ub9e4 \uccab \uc8fc 41\ub9cc \uc7a5\uc744 \ud310\ub9e4\ud574 \uc5ed\ub300 \ub300\ud55c\ubbfc\uad6d \uc544\uc774\ub3cc \uadf8\ub8f9 \uc74c\ubc18 \ucd08\ub3d9 4\uc704\ub97c \uae30\ub85d\ud588\uc73c\uba70, \ud604\uc7ac\uae4c\uc9c0 64\ub9cc \uc7a5\uc744 \ud310\ub9e4\ud558\uc600\ub2e4. \ub610\ud55c, \uc74c\uc545 \ubc29\uc1a1\uc5d0\uc11c 1\uc704\ub97c \ud729\uc4f8\uc5b4 8\uad00\uc655\uc744 \ub2ec\uc131\ud588\ub2e4. \uac19\uc740 \ud574 12\uc6d4 9\uc77c, \uacf5\uc2dd \uc751\uc6d0\ubd09 \ub108\ube14\ubd09\uc744 \ube44\ub86f\ud55c \uacf5\uc2dd \uad7f\uc988\uac00 \uacf5\uac1c\ub418\uc5c8\uc73c\uba70, 12\uc6d4 15\uc77c\ubd80\ud130 17\uc77c, 23\uc77c\ubd80\ud130 24\uc77c\uae4c\uc9c0 \ud32c\ubbf8\ud305 \ucf58\uc11c\ud2b8 \u3008\uc6cc\ub108\uc6d0 \ud504\ub9ac\ubbf8\uc5b4 \ud32c\ucf58\u3009\uc744 \uac1c\ucd5c\ud558\uc600\ub2e4. 2017\ub144 \ud55c \ud574 \ub3d9\uc548 \uc6cc\ub108\uc6d0\uc740 \ubd80\uc0b0 \uc6d0\uc544\uc2dc\uc544\ud398\uc2a4\ud2f0\ubc8c \uc5b4\uc6cc\uc988\uc5d0\uc11c \ub77c\uc774\uc9d5 \uc2a4\ud0c0\uc0c1\uc744 \uc218\uc0c1\ud558\uace0, \uc8fc\uac04\uc544\uc774\ub3cc \uc5b4\uc6cc\uc988\uc5d0\uc11c \uc544\uc774\ub3cc \ucc54\ud504\uc0c1\uc744 \uc218\uc0c1\ud588\uc73c\uba70, \ub300\ud55c\ubbfc\uad6d \ud1b1\uc2a4\ud0c0\uc0c1 \uc2dc\uc0c1\uc2dd\uc5d0\uc11c \ud1b1\uac00\uc218\uc0c1\uc744 \uc218\uc0c1\ud588\ub2e4. \ub610\ud55c, \uc18c\ub9ac\ubc14\ub2e4 \ubca0\uc2a4\ud2b8 \ucf00\uc774\ubba4\uc9c1 \uc5b4\uc6cc\uc988\uc5d0\uc11c \uc2e0\ud55c\ub958 \uc2e0\uc778\uc0c1\uacfc \uc2e0\ud55c\ub958 \ub77c\uc774\uc9d5 \ud56b \uc2a4\ud0c0\uc0c1\uc744 \uc218\uc0c1\ud558\uace0, \uc544\uc2dc\uc544 \uc544\ud2f0\uc2a4\ud2b8 \uc5b4\uc6cc\uc988\uc5d0\uc11c\ub3c4 \uc2e0\uc778\uc0c1\uacfc \uc288\ud37c\ub8e8\ud0a4 \uc0bc\uc131\ud398\uc774\uc0c1\uc744 \uc218\uc0c1\ud574 \uac01\uac01 2\uad00\uc655\uc744 \ub2ec\uc131\ud588\ub2e4. \uc774\uc5b4 \uc5e0\ub137 \uc544\uc2dc\uc548 \ubba4\uc9c1 \uc5b4\uc6cc\ub4dc\uc5d0\uc11c \ub0a8\uc790 \uadf8\ub8f9\uc0c1, \ub0a8\uc790 \uc2e0\uc778\uc0c1, \ubca0\uc2a4\ud2b8 \uc624\ube0c \ub125\uc2a4\ud2b8\uc0c1\uc744 \uc218\uc0c1\ud558\uace0, \uba5c\ub860 \ubba4\uc9c1 \uc5b4\uc6cc\ub4dc\uc5d0\uc11c\ub3c4 Top 10, \uc2e0\uc778\uc0c1, \uce74\uce74\uc624 \ud56b \uc2a4\ud0c0\uc0c1\uc744 \uc218\uc0c1\ud574 \uac01\uac01 3\uad00\uc655\uc744 \ub2ec\uc131\ud588\ub2e4. \uc6cc\ub108\uc6d0\uc740 \ub370\ubdd4 \uc568\ubc94\uacfc \ub9ac\ud328\ud0a4\uc9c0 \uc568\ubc94 \ud310\ub9e4\ub7c9\uc774 100\ub9cc\uc7a5\uc744 \ub118\uc5b4 2000\ub144 \uc774\ud6c4 \ucd5c\ucd08\ub85c \ub300\ud55c\ubbfc\uad6d \uc544\uc774\ub3cc \uadf8\ub8f9 \ub370\ubdd4 \uc568\ubc94 \ubc00\ub9ac\uc5b8\uc140\ub7ec\uac00 \ub418\uc5c8\ub2e4."} {"question": "\ubb34\ud55c\ub3c4\uc804\uc774 \uc804 \uc138\uacc4 \ub124\ud2f0\uc98c\ub4e4\uc758 \uc8fc\ubaa9\uc744 \ubc1b\ub294 \uacf3\uc740?", "paragraph": "\uc774 \ud504\ub85c\uadf8\ub7a8\uc740 \uc5ec\ub7ec \uba74\uc5d0\uc11c \ub300\ud55c\ubbfc\uad6d\uc758 \ud154\ub808\ube44\uc804 \ud504\ub85c\uadf8\ub7a8 \ubb34\ud55c\ub3c4\uc804(MBC)\uacfc \ube44\uad50\ub418\ub294\ub370, \ub9e4\uc8fc \ud30c\uaca9\uc801\uc778 \ub3c4\uc804\uacfc \uc790\uc720\ubd84\ubc29\ud55c \ud504\ub85c\uadf8\ub7a8\uc758 \ud615\uc2dd\uacfc \ub0b4\uc6a9, \uadf8\ub9ac\uace0 \ud604\uc7ac \uadf8\ub4e4\uc758 \uad6d\uac00\uc5d0\uc11c \ucd5c\uace0\uc758 \uc778\uae30\ub97c \ub204\ub9ac\uba70 \uc720\ud29c\ube0c(YouTube)\ub97c \ud1b5\ud574 \uc804 \uc138\uacc4 \ub124\ud2f0\uc98c\ub4e4\uc758 \uc8fc\ubaa9\uc744 \ubc1b\ub294 \ub4f1\uc740 \ub450 \ud504\ub85c\uadf8\ub7a8 \ubaa8\ub450 \uc720\uc0ac\ud558\ub2e4. \ud558\uc9c0\ub9cc, \ubb34\ud55c\ub3c4\uc804\uc740 \uc18c\uc18d \ud76c\uadf9\uc778(6\uba85\uc758 '\ubb34\ud55c\ub3c4\uc804 \uba64\ubc84\ub4e4')\ub4e4\uc774 \uc77c\uc0c1 \uc0dd\ud65c\uc5d0\uc11c \uc77c\uc5b4\ub0a0\uc218\uc788\ub294 \uc0c1\ud669 \uc18d\uc5d0\uc11c \uc9c0\ub098\uce5c \uc124\uc815\uc774\ub098 \uafb8\ubc08\uc774 \uc5c6\ub294 '\ub9ac\uc5bc \ubc84\ub77c\uc774\uc5b4\ud2f0'\ub97c \uc9c0\ud5a5\ud568\uc5d0 \ube44\ud574, \uc774 \ud504\ub85c\uadf8\ub7a8\uc740 \ub2e4\uc591\ud55c \ubd84\uc57c(\ud2b9\ud788 \uc815\uce58\ub098 \uc131\uc801 \ubb38\uc81c \ub4f1)\uc758 \uc18c\uc7ac\ub97c \uc790\uc720\uc790\uc7ac\ub85c \ub118\ub098\ub4e4\uace0, \uac04\ud639 \uc0c1\ud669\uc124\uc815\uc744 \uc2ec\ud558\uac8c \ub3c4\uc785\ud558\ub294 \ub4f1, \uc624\uc2a4\ud2b8\ub808\uc77c\ub9ac\uc544 \uc0ac\ud68c\uc5d0\uc11c \ud5c8\uc6a9\ud558\ub294 '\ud45c\ud604\uc758 \uc790\uc720'\ub97c \ucd5c\ub300\ub85c \ubc1c\ud718\ud558\uc5ec \uc0ac\ud68c\ud604\uc0c1\uc774\ub098 \uc0c1\ud669\uc744 \ud48d\uc790\ud55c\ub2e4\ub294 \uce21\uba74\uc5d0\uc11c \ucc28\uc774\uac00 \uc788\ub2e4.", "answer": "\uc720\ud29c\ube0c", "sentence": "\uc774 \ud504\ub85c\uadf8\ub7a8\uc740 \uc5ec\ub7ec \uba74\uc5d0\uc11c \ub300\ud55c\ubbfc\uad6d\uc758 \ud154\ub808\ube44\uc804 \ud504\ub85c\uadf8\ub7a8 \ubb34\ud55c\ub3c4\uc804(MBC)\uacfc \ube44\uad50\ub418\ub294\ub370, \ub9e4\uc8fc \ud30c\uaca9\uc801\uc778 \ub3c4\uc804\uacfc \uc790\uc720\ubd84\ubc29\ud55c \ud504\ub85c\uadf8\ub7a8\uc758 \ud615\uc2dd\uacfc \ub0b4\uc6a9, \uadf8\ub9ac\uace0 \ud604\uc7ac \uadf8\ub4e4\uc758 \uad6d\uac00\uc5d0\uc11c \ucd5c\uace0\uc758 \uc778\uae30\ub97c \ub204\ub9ac\uba70 \uc720\ud29c\ube0c (YouTube)", "paragraph_sentence": " \uc774 \ud504\ub85c\uadf8\ub7a8\uc740 \uc5ec\ub7ec \uba74\uc5d0\uc11c \ub300\ud55c\ubbfc\uad6d\uc758 \ud154\ub808\ube44\uc804 \ud504\ub85c\uadf8\ub7a8 \ubb34\ud55c\ub3c4\uc804(MBC)\uacfc \ube44\uad50\ub418\ub294\ub370, \ub9e4\uc8fc \ud30c\uaca9\uc801\uc778 \ub3c4\uc804\uacfc \uc790\uc720\ubd84\ubc29\ud55c \ud504\ub85c\uadf8\ub7a8\uc758 \ud615\uc2dd\uacfc \ub0b4\uc6a9, \uadf8\ub9ac\uace0 \ud604\uc7ac \uadf8\ub4e4\uc758 \uad6d\uac00\uc5d0\uc11c \ucd5c\uace0\uc758 \uc778\uae30\ub97c \ub204\ub9ac\uba70 \uc720\ud29c\ube0c (YouTube) \ub97c \ud1b5\ud574 \uc804 \uc138\uacc4 \ub124\ud2f0\uc98c\ub4e4\uc758 \uc8fc\ubaa9\uc744 \ubc1b\ub294 \ub4f1\uc740 \ub450 \ud504\ub85c\uadf8\ub7a8 \ubaa8\ub450 \uc720\uc0ac\ud558\ub2e4. \ud558\uc9c0\ub9cc, \ubb34\ud55c\ub3c4\uc804\uc740 \uc18c\uc18d \ud76c\uadf9\uc778(6\uba85\uc758 '\ubb34\ud55c\ub3c4\uc804 \uba64\ubc84\ub4e4')\ub4e4\uc774 \uc77c\uc0c1 \uc0dd\ud65c\uc5d0\uc11c \uc77c\uc5b4\ub0a0\uc218\uc788\ub294 \uc0c1\ud669 \uc18d\uc5d0\uc11c \uc9c0\ub098\uce5c \uc124\uc815\uc774\ub098 \uafb8\ubc08\uc774 \uc5c6\ub294 '\ub9ac\uc5bc \ubc84\ub77c\uc774\uc5b4\ud2f0'\ub97c \uc9c0\ud5a5\ud568\uc5d0 \ube44\ud574, \uc774 \ud504\ub85c\uadf8\ub7a8\uc740 \ub2e4\uc591\ud55c \ubd84\uc57c(\ud2b9\ud788 \uc815\uce58\ub098 \uc131\uc801 \ubb38\uc81c \ub4f1)\uc758 \uc18c\uc7ac\ub97c \uc790\uc720\uc790\uc7ac\ub85c \ub118\ub098\ub4e4\uace0, \uac04\ud639 \uc0c1\ud669\uc124\uc815\uc744 \uc2ec\ud558\uac8c \ub3c4\uc785\ud558\ub294 \ub4f1, \uc624\uc2a4\ud2b8\ub808\uc77c\ub9ac\uc544 \uc0ac\ud68c\uc5d0\uc11c \ud5c8\uc6a9\ud558\ub294 '\ud45c\ud604\uc758 \uc790\uc720'\ub97c \ucd5c\ub300\ub85c \ubc1c\ud718\ud558\uc5ec \uc0ac\ud68c\ud604\uc0c1\uc774\ub098 \uc0c1\ud669\uc744 \ud48d\uc790\ud55c\ub2e4\ub294 \uce21\uba74\uc5d0\uc11c \ucc28\uc774\uac00 \uc788\ub2e4.", "paragraph_answer": "\uc774 \ud504\ub85c\uadf8\ub7a8\uc740 \uc5ec\ub7ec \uba74\uc5d0\uc11c \ub300\ud55c\ubbfc\uad6d\uc758 \ud154\ub808\ube44\uc804 \ud504\ub85c\uadf8\ub7a8 \ubb34\ud55c\ub3c4\uc804(MBC)\uacfc \ube44\uad50\ub418\ub294\ub370, \ub9e4\uc8fc \ud30c\uaca9\uc801\uc778 \ub3c4\uc804\uacfc \uc790\uc720\ubd84\ubc29\ud55c \ud504\ub85c\uadf8\ub7a8\uc758 \ud615\uc2dd\uacfc \ub0b4\uc6a9, \uadf8\ub9ac\uace0 \ud604\uc7ac \uadf8\ub4e4\uc758 \uad6d\uac00\uc5d0\uc11c \ucd5c\uace0\uc758 \uc778\uae30\ub97c \ub204\ub9ac\uba70 \uc720\ud29c\ube0c (YouTube)\ub97c \ud1b5\ud574 \uc804 \uc138\uacc4 \ub124\ud2f0\uc98c\ub4e4\uc758 \uc8fc\ubaa9\uc744 \ubc1b\ub294 \ub4f1\uc740 \ub450 \ud504\ub85c\uadf8\ub7a8 \ubaa8\ub450 \uc720\uc0ac\ud558\ub2e4. \ud558\uc9c0\ub9cc, \ubb34\ud55c\ub3c4\uc804\uc740 \uc18c\uc18d \ud76c\uadf9\uc778(6\uba85\uc758 '\ubb34\ud55c\ub3c4\uc804 \uba64\ubc84\ub4e4')\ub4e4\uc774 \uc77c\uc0c1 \uc0dd\ud65c\uc5d0\uc11c \uc77c\uc5b4\ub0a0\uc218\uc788\ub294 \uc0c1\ud669 \uc18d\uc5d0\uc11c \uc9c0\ub098\uce5c \uc124\uc815\uc774\ub098 \uafb8\ubc08\uc774 \uc5c6\ub294 '\ub9ac\uc5bc \ubc84\ub77c\uc774\uc5b4\ud2f0'\ub97c \uc9c0\ud5a5\ud568\uc5d0 \ube44\ud574, \uc774 \ud504\ub85c\uadf8\ub7a8\uc740 \ub2e4\uc591\ud55c \ubd84\uc57c(\ud2b9\ud788 \uc815\uce58\ub098 \uc131\uc801 \ubb38\uc81c \ub4f1)\uc758 \uc18c\uc7ac\ub97c \uc790\uc720\uc790\uc7ac\ub85c \ub118\ub098\ub4e4\uace0, \uac04\ud639 \uc0c1\ud669\uc124\uc815\uc744 \uc2ec\ud558\uac8c \ub3c4\uc785\ud558\ub294 \ub4f1, \uc624\uc2a4\ud2b8\ub808\uc77c\ub9ac\uc544 \uc0ac\ud68c\uc5d0\uc11c \ud5c8\uc6a9\ud558\ub294 '\ud45c\ud604\uc758 \uc790\uc720'\ub97c \ucd5c\ub300\ub85c \ubc1c\ud718\ud558\uc5ec \uc0ac\ud68c\ud604\uc0c1\uc774\ub098 \uc0c1\ud669\uc744 \ud48d\uc790\ud55c\ub2e4\ub294 \uce21\uba74\uc5d0\uc11c \ucc28\uc774\uac00 \uc788\ub2e4.", "sentence_answer": "\uc774 \ud504\ub85c\uadf8\ub7a8\uc740 \uc5ec\ub7ec \uba74\uc5d0\uc11c \ub300\ud55c\ubbfc\uad6d\uc758 \ud154\ub808\ube44\uc804 \ud504\ub85c\uadf8\ub7a8 \ubb34\ud55c\ub3c4\uc804(MBC)\uacfc \ube44\uad50\ub418\ub294\ub370, \ub9e4\uc8fc \ud30c\uaca9\uc801\uc778 \ub3c4\uc804\uacfc \uc790\uc720\ubd84\ubc29\ud55c \ud504\ub85c\uadf8\ub7a8\uc758 \ud615\uc2dd\uacfc \ub0b4\uc6a9, \uadf8\ub9ac\uace0 \ud604\uc7ac \uadf8\ub4e4\uc758 \uad6d\uac00\uc5d0\uc11c \ucd5c\uace0\uc758 \uc778\uae30\ub97c \ub204\ub9ac\uba70 \uc720\ud29c\ube0c (YouTube)", "paragraph_id": "6559656-2-1", "paragraph_question": "question: \ubb34\ud55c\ub3c4\uc804\uc774 \uc804 \uc138\uacc4 \ub124\ud2f0\uc98c\ub4e4\uc758 \uc8fc\ubaa9\uc744 \ubc1b\ub294 \uacf3\uc740?, context: \uc774 \ud504\ub85c\uadf8\ub7a8\uc740 \uc5ec\ub7ec \uba74\uc5d0\uc11c \ub300\ud55c\ubbfc\uad6d\uc758 \ud154\ub808\ube44\uc804 \ud504\ub85c\uadf8\ub7a8 \ubb34\ud55c\ub3c4\uc804(MBC)\uacfc \ube44\uad50\ub418\ub294\ub370, \ub9e4\uc8fc \ud30c\uaca9\uc801\uc778 \ub3c4\uc804\uacfc \uc790\uc720\ubd84\ubc29\ud55c \ud504\ub85c\uadf8\ub7a8\uc758 \ud615\uc2dd\uacfc \ub0b4\uc6a9, \uadf8\ub9ac\uace0 \ud604\uc7ac \uadf8\ub4e4\uc758 \uad6d\uac00\uc5d0\uc11c \ucd5c\uace0\uc758 \uc778\uae30\ub97c \ub204\ub9ac\uba70 \uc720\ud29c\ube0c(YouTube)\ub97c \ud1b5\ud574 \uc804 \uc138\uacc4 \ub124\ud2f0\uc98c\ub4e4\uc758 \uc8fc\ubaa9\uc744 \ubc1b\ub294 \ub4f1\uc740 \ub450 \ud504\ub85c\uadf8\ub7a8 \ubaa8\ub450 \uc720\uc0ac\ud558\ub2e4. \ud558\uc9c0\ub9cc, \ubb34\ud55c\ub3c4\uc804\uc740 \uc18c\uc18d \ud76c\uadf9\uc778(6\uba85\uc758 '\ubb34\ud55c\ub3c4\uc804 \uba64\ubc84\ub4e4')\ub4e4\uc774 \uc77c\uc0c1 \uc0dd\ud65c\uc5d0\uc11c \uc77c\uc5b4\ub0a0\uc218\uc788\ub294 \uc0c1\ud669 \uc18d\uc5d0\uc11c \uc9c0\ub098\uce5c \uc124\uc815\uc774\ub098 \uafb8\ubc08\uc774 \uc5c6\ub294 '\ub9ac\uc5bc \ubc84\ub77c\uc774\uc5b4\ud2f0'\ub97c \uc9c0\ud5a5\ud568\uc5d0 \ube44\ud574, \uc774 \ud504\ub85c\uadf8\ub7a8\uc740 \ub2e4\uc591\ud55c \ubd84\uc57c(\ud2b9\ud788 \uc815\uce58\ub098 \uc131\uc801 \ubb38\uc81c \ub4f1)\uc758 \uc18c\uc7ac\ub97c \uc790\uc720\uc790\uc7ac\ub85c \ub118\ub098\ub4e4\uace0, \uac04\ud639 \uc0c1\ud669\uc124\uc815\uc744 \uc2ec\ud558\uac8c \ub3c4\uc785\ud558\ub294 \ub4f1, \uc624\uc2a4\ud2b8\ub808\uc77c\ub9ac\uc544 \uc0ac\ud68c\uc5d0\uc11c \ud5c8\uc6a9\ud558\ub294 '\ud45c\ud604\uc758 \uc790\uc720'\ub97c \ucd5c\ub300\ub85c \ubc1c\ud718\ud558\uc5ec \uc0ac\ud68c\ud604\uc0c1\uc774\ub098 \uc0c1\ud669\uc744 \ud48d\uc790\ud55c\ub2e4\ub294 \uce21\uba74\uc5d0\uc11c \ucc28\uc774\uac00 \uc788\ub2e4."} {"question": "\uc778\ud130\ub137 \uc775\uc2a4\ud50c\ub85c\ub7ec 10\uc774 \ub9cc\ub4e4\uc5b4\uc9c8 \ub54c \uae4c\uc9c0 \uac78\ub9b0 \uc2dc\uac04\uc740?", "paragraph": "\ub9c8\uc774\ud06c\ub85c\uc18c\ud504\ud2b8\ub294 \ub77c\uc2a4\ubca0\uc774\uac70\uc2a4\uc758 MIX 11\uc5d0\uc11c \uc778\ud130\ub137 \uc775\uc2a4\ud50c\ub85c\ub7ec 10\uc744 \ubc1c\ud45c\ud558\uba74\uc11c \ucd5c\ucd08\uc758 \ud50c\ub7ab\ud3fc \ud504\ub9ac\ubdf0\ub97c \uacf5\uac1c\ud558\uc600\ub2e4. \uc774 \uc790\ub9ac\uc5d0\uc11c \uc778\ud130\ub137 \uc775\uc2a4\ud50c\ub85c\ub7ec 10\uc774 \uac1c\ubc1c\uc5d0 3\uc8fc \uc815\ub3c4 \uac78\ub9b0 \uac83\uc774\ub77c\uace0 \uc5b8\uae09\ud558\uc600\ub2e4. \uc774\ubc88 \ucd9c\uc2dc\ub294 CSS3 \uadf8\ub77c\ub514\uc5b8\ud2b8\ub97c \ud3ec\ud568\ud55c \ud45c\uc900 \uc9c0\uc6d0\uc744 \ub354 \uac1c\uc120\ud55c\ub2e4. \ub610\ud55c \ub9c8\uc774\ud06c\ub85c\uc18c\ud504\ud2b8\ub294 \ub370\uc2a4\ud06c\ud1b1 \ubaa8\ub4dc\uc5d0\uc11c\ub294 \uc561\ud2f0\ube0cX\ub97c \uc9c0\uc6d0\ud558\uc9c0\ub9cc \ubaa8\ub358 UI\uc5d0\uc11c\ub294 \uc9c0\uc6d0\ud558\uc9c0 \uc54a\ub294\ub2e4\uace0 \ubc1d\ud614\ub2e4. \uc778\ud130\ub137 \uc775\uc2a4\ud50c\ub85c\ub7ec 10\uc740 9\uc640 \ub2ec\ub9ac \ud55c\uad6d\uc758 \ub300\ubd80\ubd84\uc758 \uc6f9\uc5d0\uc11c \ud638\ud658\uc131 \ubb38\uc81c\ub97c \uc77c\uc73c\ud0a4\uba70 \uae09\uae30\uc57c\ub294 \uc778\ud130\ub137 \uc775\uc2a4\ud50c\ub85c\ub7ec 10\uc73c\ub85c\uc758 \uc5c5\ub370\uc774\ud2b8\ub97c \ub9c9\ub294 \ud0b7\uc744 \ub9c8\uc774\ud06c\ub85c\uc18c\ud504\ud2b8\uc5d0\uc11c \uc81c\uacf5\ud558\uae30\ub3c4 \ud588\ub2e4. \uc708\ub3c4\uc6b0 7, \uc708\ub3c4\uc6b0 \uc11c\ubc84 2008 R2, \uc708\ub3c4\uc6b0 8\uc6a9\uc740 \uc708\ub3c4\uc6b0 XP \uc784\ubca0\ub514\ub4dc, \uc708\ub3c4\uc6b0 8\uacfc \uc708\ub3c4\uc6b0 \ube44\uc2a4\ud0c0\uc6a9 \uc778\ud130\ub137 \uc775\uc2a4\ud50c\ub85c\ub7ec 7\uacfc \uc708\ub3c4\uc6b0 \ube44\uc2a4\ud0c0, \uc708\ub3c4\uc6b0 \uc11c\ubc84 2008, \uc708\ub3c4\uc6b0 7, \uc708\ub3c4\uc6b0 \uc11c\ubc84 2008 R2\uc6a9 \uc778\ud130\ub137 \uc775\uc2a4\ud50c\ub85c\ub7ec 8, \uadf8\ub9ac\uace0 \uc708\ub3c4\uc6b0 7, \uc708\ub3c4\uc6b0 \uc11c\ubc84 2008 R2\uc6a9 \uc778\ud130\ub137 \uc775\uc2a4\ud50c\ub85c\ub7ec 9\uc640 \uac19\uc774 \uc9c0\uc6d0\uc774 2016\ub144 1\uc6d4 12\uc77c \uc885\ub8cc\ub418\uc5c8\uc73c\uba70, \uc708\ub3c4\uc6b0 \uc11c\ubc84 2012\uc6a9\uc740 \uc708\ub3c4\uc6b0 8.1, \uc708\ub3c4\uc6b0 \uc11c\ubc84 2012, \uc708\ub3c4\uc6b0 \uc11c\ubc84 2012 R2 \uadf8\ub9ac\uace0 \uc708\ub3c4\uc6b0 8.1, \uc708\ub3c4\uc6b0 \uc11c\ubc84 2012 R2\uc6a9 \uc778\ud130\ub137 \uc775\uc2a4\ud50c\ub85c\ub7ec 11\uacfc \uac19\uc774 \uc9c0\uc6d0\uc774 2023\ub144 1\uc6d4 10\uc77c \uc885\ub8cc\ub420 \uc608\uc815\uc774\ub2e4.", "answer": "3\uc8fc \uc815\ub3c4", "sentence": "\uc774 \uc790\ub9ac\uc5d0\uc11c \uc778\ud130\ub137 \uc775\uc2a4\ud50c\ub85c\ub7ec 10\uc774 \uac1c\ubc1c\uc5d0 3\uc8fc \uc815\ub3c4 \uac78\ub9b0 \uac83\uc774\ub77c\uace0 \uc5b8\uae09\ud558\uc600\ub2e4.", "paragraph_sentence": "\ub9c8\uc774\ud06c\ub85c\uc18c\ud504\ud2b8\ub294 \ub77c\uc2a4\ubca0\uc774\uac70\uc2a4\uc758 MIX 11\uc5d0\uc11c \uc778\ud130\ub137 \uc775\uc2a4\ud50c\ub85c\ub7ec 10\uc744 \ubc1c\ud45c\ud558\uba74\uc11c \ucd5c\ucd08\uc758 \ud50c\ub7ab\ud3fc \ud504\ub9ac\ubdf0\ub97c \uacf5\uac1c\ud558\uc600\ub2e4. \uc774 \uc790\ub9ac\uc5d0\uc11c \uc778\ud130\ub137 \uc775\uc2a4\ud50c\ub85c\ub7ec 10\uc774 \uac1c\ubc1c\uc5d0 3\uc8fc \uc815\ub3c4 \uac78\ub9b0 \uac83\uc774\ub77c\uace0 \uc5b8\uae09\ud558\uc600\ub2e4. \uc774\ubc88 \ucd9c\uc2dc\ub294 CSS3 \uadf8\ub77c\ub514\uc5b8\ud2b8\ub97c \ud3ec\ud568\ud55c \ud45c\uc900 \uc9c0\uc6d0\uc744 \ub354 \uac1c\uc120\ud55c\ub2e4. \ub610\ud55c \ub9c8\uc774\ud06c\ub85c\uc18c\ud504\ud2b8\ub294 \ub370\uc2a4\ud06c\ud1b1 \ubaa8\ub4dc\uc5d0\uc11c\ub294 \uc561\ud2f0\ube0cX\ub97c \uc9c0\uc6d0\ud558\uc9c0\ub9cc \ubaa8\ub358 UI\uc5d0\uc11c\ub294 \uc9c0\uc6d0\ud558\uc9c0 \uc54a\ub294\ub2e4\uace0 \ubc1d\ud614\ub2e4. \uc778\ud130\ub137 \uc775\uc2a4\ud50c\ub85c\ub7ec 10\uc740 9\uc640 \ub2ec\ub9ac \ud55c\uad6d\uc758 \ub300\ubd80\ubd84\uc758 \uc6f9\uc5d0\uc11c \ud638\ud658\uc131 \ubb38\uc81c\ub97c \uc77c\uc73c\ud0a4\uba70 \uae09\uae30\uc57c\ub294 \uc778\ud130\ub137 \uc775\uc2a4\ud50c\ub85c\ub7ec 10\uc73c\ub85c\uc758 \uc5c5\ub370\uc774\ud2b8\ub97c \ub9c9\ub294 \ud0b7\uc744 \ub9c8\uc774\ud06c\ub85c\uc18c\ud504\ud2b8\uc5d0\uc11c \uc81c\uacf5\ud558\uae30\ub3c4 \ud588\ub2e4. \uc708\ub3c4\uc6b0 7, \uc708\ub3c4\uc6b0 \uc11c\ubc84 2008 R2, \uc708\ub3c4\uc6b0 8\uc6a9\uc740 \uc708\ub3c4\uc6b0 XP \uc784\ubca0\ub514\ub4dc, \uc708\ub3c4\uc6b0 8\uacfc \uc708\ub3c4\uc6b0 \ube44\uc2a4\ud0c0\uc6a9 \uc778\ud130\ub137 \uc775\uc2a4\ud50c\ub85c\ub7ec 7\uacfc \uc708\ub3c4\uc6b0 \ube44\uc2a4\ud0c0, \uc708\ub3c4\uc6b0 \uc11c\ubc84 2008, \uc708\ub3c4\uc6b0 7, \uc708\ub3c4\uc6b0 \uc11c\ubc84 2008 R2\uc6a9 \uc778\ud130\ub137 \uc775\uc2a4\ud50c\ub85c\ub7ec 8, \uadf8\ub9ac\uace0 \uc708\ub3c4\uc6b0 7, \uc708\ub3c4\uc6b0 \uc11c\ubc84 2008 R2\uc6a9 \uc778\ud130\ub137 \uc775\uc2a4\ud50c\ub85c\ub7ec 9\uc640 \uac19\uc774 \uc9c0\uc6d0\uc774 2016\ub144 1\uc6d4 12\uc77c \uc885\ub8cc\ub418\uc5c8\uc73c\uba70, \uc708\ub3c4\uc6b0 \uc11c\ubc84 2012\uc6a9\uc740 \uc708\ub3c4\uc6b0 8.1, \uc708\ub3c4\uc6b0 \uc11c\ubc84 2012, \uc708\ub3c4\uc6b0 \uc11c\ubc84 2012 R2 \uadf8\ub9ac\uace0 \uc708\ub3c4\uc6b0 8.1, \uc708\ub3c4\uc6b0 \uc11c\ubc84 2012 R2\uc6a9 \uc778\ud130\ub137 \uc775\uc2a4\ud50c\ub85c\ub7ec 11\uacfc \uac19\uc774 \uc9c0\uc6d0\uc774 2023\ub144 1\uc6d4 10\uc77c \uc885\ub8cc\ub420 \uc608\uc815\uc774\ub2e4.", "paragraph_answer": "\ub9c8\uc774\ud06c\ub85c\uc18c\ud504\ud2b8\ub294 \ub77c\uc2a4\ubca0\uc774\uac70\uc2a4\uc758 MIX 11\uc5d0\uc11c \uc778\ud130\ub137 \uc775\uc2a4\ud50c\ub85c\ub7ec 10\uc744 \ubc1c\ud45c\ud558\uba74\uc11c \ucd5c\ucd08\uc758 \ud50c\ub7ab\ud3fc \ud504\ub9ac\ubdf0\ub97c \uacf5\uac1c\ud558\uc600\ub2e4. \uc774 \uc790\ub9ac\uc5d0\uc11c \uc778\ud130\ub137 \uc775\uc2a4\ud50c\ub85c\ub7ec 10\uc774 \uac1c\ubc1c\uc5d0 3\uc8fc \uc815\ub3c4 \uac78\ub9b0 \uac83\uc774\ub77c\uace0 \uc5b8\uae09\ud558\uc600\ub2e4. \uc774\ubc88 \ucd9c\uc2dc\ub294 CSS3 \uadf8\ub77c\ub514\uc5b8\ud2b8\ub97c \ud3ec\ud568\ud55c \ud45c\uc900 \uc9c0\uc6d0\uc744 \ub354 \uac1c\uc120\ud55c\ub2e4. \ub610\ud55c \ub9c8\uc774\ud06c\ub85c\uc18c\ud504\ud2b8\ub294 \ub370\uc2a4\ud06c\ud1b1 \ubaa8\ub4dc\uc5d0\uc11c\ub294 \uc561\ud2f0\ube0cX\ub97c \uc9c0\uc6d0\ud558\uc9c0\ub9cc \ubaa8\ub358 UI\uc5d0\uc11c\ub294 \uc9c0\uc6d0\ud558\uc9c0 \uc54a\ub294\ub2e4\uace0 \ubc1d\ud614\ub2e4. \uc778\ud130\ub137 \uc775\uc2a4\ud50c\ub85c\ub7ec 10\uc740 9\uc640 \ub2ec\ub9ac \ud55c\uad6d\uc758 \ub300\ubd80\ubd84\uc758 \uc6f9\uc5d0\uc11c \ud638\ud658\uc131 \ubb38\uc81c\ub97c \uc77c\uc73c\ud0a4\uba70 \uae09\uae30\uc57c\ub294 \uc778\ud130\ub137 \uc775\uc2a4\ud50c\ub85c\ub7ec 10\uc73c\ub85c\uc758 \uc5c5\ub370\uc774\ud2b8\ub97c \ub9c9\ub294 \ud0b7\uc744 \ub9c8\uc774\ud06c\ub85c\uc18c\ud504\ud2b8\uc5d0\uc11c \uc81c\uacf5\ud558\uae30\ub3c4 \ud588\ub2e4. \uc708\ub3c4\uc6b0 7, \uc708\ub3c4\uc6b0 \uc11c\ubc84 2008 R2, \uc708\ub3c4\uc6b0 8\uc6a9\uc740 \uc708\ub3c4\uc6b0 XP \uc784\ubca0\ub514\ub4dc, \uc708\ub3c4\uc6b0 8\uacfc \uc708\ub3c4\uc6b0 \ube44\uc2a4\ud0c0\uc6a9 \uc778\ud130\ub137 \uc775\uc2a4\ud50c\ub85c\ub7ec 7\uacfc \uc708\ub3c4\uc6b0 \ube44\uc2a4\ud0c0, \uc708\ub3c4\uc6b0 \uc11c\ubc84 2008, \uc708\ub3c4\uc6b0 7, \uc708\ub3c4\uc6b0 \uc11c\ubc84 2008 R2\uc6a9 \uc778\ud130\ub137 \uc775\uc2a4\ud50c\ub85c\ub7ec 8, \uadf8\ub9ac\uace0 \uc708\ub3c4\uc6b0 7, \uc708\ub3c4\uc6b0 \uc11c\ubc84 2008 R2\uc6a9 \uc778\ud130\ub137 \uc775\uc2a4\ud50c\ub85c\ub7ec 9\uc640 \uac19\uc774 \uc9c0\uc6d0\uc774 2016\ub144 1\uc6d4 12\uc77c \uc885\ub8cc\ub418\uc5c8\uc73c\uba70, \uc708\ub3c4\uc6b0 \uc11c\ubc84 2012\uc6a9\uc740 \uc708\ub3c4\uc6b0 8.1, \uc708\ub3c4\uc6b0 \uc11c\ubc84 2012, \uc708\ub3c4\uc6b0 \uc11c\ubc84 2012 R2 \uadf8\ub9ac\uace0 \uc708\ub3c4\uc6b0 8.1, \uc708\ub3c4\uc6b0 \uc11c\ubc84 2012 R2\uc6a9 \uc778\ud130\ub137 \uc775\uc2a4\ud50c\ub85c\ub7ec 11\uacfc \uac19\uc774 \uc9c0\uc6d0\uc774 2023\ub144 1\uc6d4 10\uc77c \uc885\ub8cc\ub420 \uc608\uc815\uc774\ub2e4.", "sentence_answer": "\uc774 \uc790\ub9ac\uc5d0\uc11c \uc778\ud130\ub137 \uc775\uc2a4\ud50c\ub85c\ub7ec 10\uc774 \uac1c\ubc1c\uc5d0 3\uc8fc \uc815\ub3c4 \uac78\ub9b0 \uac83\uc774\ub77c\uace0 \uc5b8\uae09\ud558\uc600\ub2e4.", "paragraph_id": "6548255-5-0", "paragraph_question": "question: \uc778\ud130\ub137 \uc775\uc2a4\ud50c\ub85c\ub7ec 10\uc774 \ub9cc\ub4e4\uc5b4\uc9c8 \ub54c \uae4c\uc9c0 \uac78\ub9b0 \uc2dc\uac04\uc740?, context: \ub9c8\uc774\ud06c\ub85c\uc18c\ud504\ud2b8\ub294 \ub77c\uc2a4\ubca0\uc774\uac70\uc2a4\uc758 MIX 11\uc5d0\uc11c \uc778\ud130\ub137 \uc775\uc2a4\ud50c\ub85c\ub7ec 10\uc744 \ubc1c\ud45c\ud558\uba74\uc11c \ucd5c\ucd08\uc758 \ud50c\ub7ab\ud3fc \ud504\ub9ac\ubdf0\ub97c \uacf5\uac1c\ud558\uc600\ub2e4. \uc774 \uc790\ub9ac\uc5d0\uc11c \uc778\ud130\ub137 \uc775\uc2a4\ud50c\ub85c\ub7ec 10\uc774 \uac1c\ubc1c\uc5d0 3\uc8fc \uc815\ub3c4 \uac78\ub9b0 \uac83\uc774\ub77c\uace0 \uc5b8\uae09\ud558\uc600\ub2e4. \uc774\ubc88 \ucd9c\uc2dc\ub294 CSS3 \uadf8\ub77c\ub514\uc5b8\ud2b8\ub97c \ud3ec\ud568\ud55c \ud45c\uc900 \uc9c0\uc6d0\uc744 \ub354 \uac1c\uc120\ud55c\ub2e4. \ub610\ud55c \ub9c8\uc774\ud06c\ub85c\uc18c\ud504\ud2b8\ub294 \ub370\uc2a4\ud06c\ud1b1 \ubaa8\ub4dc\uc5d0\uc11c\ub294 \uc561\ud2f0\ube0cX\ub97c \uc9c0\uc6d0\ud558\uc9c0\ub9cc \ubaa8\ub358 UI\uc5d0\uc11c\ub294 \uc9c0\uc6d0\ud558\uc9c0 \uc54a\ub294\ub2e4\uace0 \ubc1d\ud614\ub2e4. \uc778\ud130\ub137 \uc775\uc2a4\ud50c\ub85c\ub7ec 10\uc740 9\uc640 \ub2ec\ub9ac \ud55c\uad6d\uc758 \ub300\ubd80\ubd84\uc758 \uc6f9\uc5d0\uc11c \ud638\ud658\uc131 \ubb38\uc81c\ub97c \uc77c\uc73c\ud0a4\uba70 \uae09\uae30\uc57c\ub294 \uc778\ud130\ub137 \uc775\uc2a4\ud50c\ub85c\ub7ec 10\uc73c\ub85c\uc758 \uc5c5\ub370\uc774\ud2b8\ub97c \ub9c9\ub294 \ud0b7\uc744 \ub9c8\uc774\ud06c\ub85c\uc18c\ud504\ud2b8\uc5d0\uc11c \uc81c\uacf5\ud558\uae30\ub3c4 \ud588\ub2e4. \uc708\ub3c4\uc6b0 7, \uc708\ub3c4\uc6b0 \uc11c\ubc84 2008 R2, \uc708\ub3c4\uc6b0 8\uc6a9\uc740 \uc708\ub3c4\uc6b0 XP \uc784\ubca0\ub514\ub4dc, \uc708\ub3c4\uc6b0 8\uacfc \uc708\ub3c4\uc6b0 \ube44\uc2a4\ud0c0\uc6a9 \uc778\ud130\ub137 \uc775\uc2a4\ud50c\ub85c\ub7ec 7\uacfc \uc708\ub3c4\uc6b0 \ube44\uc2a4\ud0c0, \uc708\ub3c4\uc6b0 \uc11c\ubc84 2008, \uc708\ub3c4\uc6b0 7, \uc708\ub3c4\uc6b0 \uc11c\ubc84 2008 R2\uc6a9 \uc778\ud130\ub137 \uc775\uc2a4\ud50c\ub85c\ub7ec 8, \uadf8\ub9ac\uace0 \uc708\ub3c4\uc6b0 7, \uc708\ub3c4\uc6b0 \uc11c\ubc84 2008 R2\uc6a9 \uc778\ud130\ub137 \uc775\uc2a4\ud50c\ub85c\ub7ec 9\uc640 \uac19\uc774 \uc9c0\uc6d0\uc774 2016\ub144 1\uc6d4 12\uc77c \uc885\ub8cc\ub418\uc5c8\uc73c\uba70, \uc708\ub3c4\uc6b0 \uc11c\ubc84 2012\uc6a9\uc740 \uc708\ub3c4\uc6b0 8.1, \uc708\ub3c4\uc6b0 \uc11c\ubc84 2012, \uc708\ub3c4\uc6b0 \uc11c\ubc84 2012 R2 \uadf8\ub9ac\uace0 \uc708\ub3c4\uc6b0 8.1, \uc708\ub3c4\uc6b0 \uc11c\ubc84 2012 R2\uc6a9 \uc778\ud130\ub137 \uc775\uc2a4\ud50c\ub85c\ub7ec 11\uacfc \uac19\uc774 \uc9c0\uc6d0\uc774 2023\ub144 1\uc6d4 10\uc77c \uc885\ub8cc\ub420 \uc608\uc815\uc774\ub2e4."} {"question": "\ubaa8\ud638\ud568\uc744 \uc904\uc774\uae30 \uc704\ud574 \ud55c \ub2ec\uc758 \uae30\uac04\uc744 \ub098\ud0c0\ub0b4\ub294 \uac83\uc740 \ubb34\uc5c7\uc778\uac00?", "paragraph": "\ubaa8\ud638\ud568\uc744 \uc904\uc774\uae30 \uc704\ud574, \"P1M\"\uc740 \ud55c \ub2ec\uc758 \uae30\uac04\uc774\uba70 \"PT1M\"\uc740 1\ubd84\uc758 \uae30\uac04\uc744 \ub9d0\ud55c\ub2e4(T\ub294 \uc2dc\uac04 \uac12 \uc55e\uc5d0 \uc624\ub294 \uc2dc\uac04 \uc9c0\uc815\uc790\uc784\uc744 \uc54c\uc544\ub450\uc790). \uc18c\uc218\uc810\uc774 \uac00\uc7a5 \uc791\uc740 \uac12\uc73c\ub85c \uc0ac\uc6a9\ub420 \uc218 \uc788\uc5b4, \"P0.5Y\"\ub294 \ubc18\ub144\uc744 \ub098\ud0c0\ub0b8\ub2e4. \uc18c\uc218\uc810\uc740 \"P0,5Y\" \ud639\uc740 \"P0.5Y\"\uc640 \uac19\uc774, \uc27c\ud45c \ud639\uc740 \ub9c8\uce68\ud45c\ub85c \uc9c0\uc815\ub420 \uc218 \uc788\ub2e4. \ud45c\uc900\uc740 \uc544\ub798\uc5d0\uc11c \uc8fc\uc9c0\ud558\uace0 \uc788\ub294 \uac83 \uc678\uc5d0 \"carry over points\"\uc744 \ucd08\uacfc\ud558\uace0 \uc788\ub294 \uae30\uac04 \ud45c\ud604\uc5d0\uc11c\uc758 \ub0a0\uc9dc\uc640 \uc2dc\uac04 \uac12\uc744 \uae08\uc9c0\ud558\uc9c0 \uc54a\ub294\ub2e4. \uadf8\ub798\uc11c \"PT36H\"\ub294 \"P1DT12H\"\uc640 \uac19\uc740 \uae30\uac04\uc744 \ub098\ud0c0\ub0b4\ub294\ub370 \uc0ac\uc6a9\ub420 \uc218\ub3c4 \uc788\ub2e4. \uadf8\ub7ec\ub098 \"PT36H\"\ub294 \uc77c\uad11 \uc808\uc57d \uc2dc\uac04(\uc11c\uba38 \ud0c0\uc784, daylight saving time)\uc5d0\uc11c \ud639\uc740 \uc73c\ub85c \uc804\ud658\ud558\ub294 \uacbd\uc6b0 \"P1DT12H\"\uc640 \uac19\uc9c0 \uc54a\ub2e4\ub294 \uac83\uc744 \uc54c\uc544\ub46c\uc57c \ud55c\ub2e4.", "answer": "P1M", "sentence": "\ubaa8\ud638\ud568\uc744 \uc904\uc774\uae30 \uc704\ud574, \" P1M \"\uc740 \ud55c \ub2ec\uc758 \uae30\uac04\uc774\uba70 \"PT1M\"\uc740 1\ubd84\uc758 \uae30\uac04\uc744 \ub9d0\ud55c\ub2e4(T\ub294 \uc2dc\uac04 \uac12 \uc55e\uc5d0 \uc624\ub294 \uc2dc\uac04 \uc9c0\uc815\uc790\uc784\uc744 \uc54c\uc544\ub450\uc790).", "paragraph_sentence": " \ubaa8\ud638\ud568\uc744 \uc904\uc774\uae30 \uc704\ud574, \" P1M \"\uc740 \ud55c \ub2ec\uc758 \uae30\uac04\uc774\uba70 \"PT1M\"\uc740 1\ubd84\uc758 \uae30\uac04\uc744 \ub9d0\ud55c\ub2e4(T\ub294 \uc2dc\uac04 \uac12 \uc55e\uc5d0 \uc624\ub294 \uc2dc\uac04 \uc9c0\uc815\uc790\uc784\uc744 \uc54c\uc544\ub450\uc790). \uc18c\uc218\uc810\uc774 \uac00\uc7a5 \uc791\uc740 \uac12\uc73c\ub85c \uc0ac\uc6a9\ub420 \uc218 \uc788\uc5b4, \"P0.5Y\"\ub294 \ubc18\ub144\uc744 \ub098\ud0c0\ub0b8\ub2e4. \uc18c\uc218\uc810\uc740 \"P0,5Y\" \ud639\uc740 \"P0.5Y\"\uc640 \uac19\uc774, \uc27c\ud45c \ud639\uc740 \ub9c8\uce68\ud45c\ub85c \uc9c0\uc815\ub420 \uc218 \uc788\ub2e4. \ud45c\uc900\uc740 \uc544\ub798\uc5d0\uc11c \uc8fc\uc9c0\ud558\uace0 \uc788\ub294 \uac83 \uc678\uc5d0 \"carry over points\"\uc744 \ucd08\uacfc\ud558\uace0 \uc788\ub294 \uae30\uac04 \ud45c\ud604\uc5d0\uc11c\uc758 \ub0a0\uc9dc\uc640 \uc2dc\uac04 \uac12\uc744 \uae08\uc9c0\ud558\uc9c0 \uc54a\ub294\ub2e4. \uadf8\ub798\uc11c \"PT36H\"\ub294 \"P1DT12H\"\uc640 \uac19\uc740 \uae30\uac04\uc744 \ub098\ud0c0\ub0b4\ub294\ub370 \uc0ac\uc6a9\ub420 \uc218\ub3c4 \uc788\ub2e4. \uadf8\ub7ec\ub098 \"PT36H\"\ub294 \uc77c\uad11 \uc808\uc57d \uc2dc\uac04(\uc11c\uba38 \ud0c0\uc784, daylight saving time)\uc5d0\uc11c \ud639\uc740 \uc73c\ub85c \uc804\ud658\ud558\ub294 \uacbd\uc6b0 \"P1DT12H\"\uc640 \uac19\uc9c0 \uc54a\ub2e4\ub294 \uac83\uc744 \uc54c\uc544\ub46c\uc57c \ud55c\ub2e4.", "paragraph_answer": "\ubaa8\ud638\ud568\uc744 \uc904\uc774\uae30 \uc704\ud574, \" P1M \"\uc740 \ud55c \ub2ec\uc758 \uae30\uac04\uc774\uba70 \"PT1M\"\uc740 1\ubd84\uc758 \uae30\uac04\uc744 \ub9d0\ud55c\ub2e4(T\ub294 \uc2dc\uac04 \uac12 \uc55e\uc5d0 \uc624\ub294 \uc2dc\uac04 \uc9c0\uc815\uc790\uc784\uc744 \uc54c\uc544\ub450\uc790). \uc18c\uc218\uc810\uc774 \uac00\uc7a5 \uc791\uc740 \uac12\uc73c\ub85c \uc0ac\uc6a9\ub420 \uc218 \uc788\uc5b4, \"P0.5Y\"\ub294 \ubc18\ub144\uc744 \ub098\ud0c0\ub0b8\ub2e4. \uc18c\uc218\uc810\uc740 \"P0,5Y\" \ud639\uc740 \"P0.5Y\"\uc640 \uac19\uc774, \uc27c\ud45c \ud639\uc740 \ub9c8\uce68\ud45c\ub85c \uc9c0\uc815\ub420 \uc218 \uc788\ub2e4. \ud45c\uc900\uc740 \uc544\ub798\uc5d0\uc11c \uc8fc\uc9c0\ud558\uace0 \uc788\ub294 \uac83 \uc678\uc5d0 \"carry over points\"\uc744 \ucd08\uacfc\ud558\uace0 \uc788\ub294 \uae30\uac04 \ud45c\ud604\uc5d0\uc11c\uc758 \ub0a0\uc9dc\uc640 \uc2dc\uac04 \uac12\uc744 \uae08\uc9c0\ud558\uc9c0 \uc54a\ub294\ub2e4. \uadf8\ub798\uc11c \"PT36H\"\ub294 \"P1DT12H\"\uc640 \uac19\uc740 \uae30\uac04\uc744 \ub098\ud0c0\ub0b4\ub294\ub370 \uc0ac\uc6a9\ub420 \uc218\ub3c4 \uc788\ub2e4. \uadf8\ub7ec\ub098 \"PT36H\"\ub294 \uc77c\uad11 \uc808\uc57d \uc2dc\uac04(\uc11c\uba38 \ud0c0\uc784, daylight saving time)\uc5d0\uc11c \ud639\uc740 \uc73c\ub85c \uc804\ud658\ud558\ub294 \uacbd\uc6b0 \"P1DT12H\"\uc640 \uac19\uc9c0 \uc54a\ub2e4\ub294 \uac83\uc744 \uc54c\uc544\ub46c\uc57c \ud55c\ub2e4.", "sentence_answer": "\ubaa8\ud638\ud568\uc744 \uc904\uc774\uae30 \uc704\ud574, \" P1M \"\uc740 \ud55c \ub2ec\uc758 \uae30\uac04\uc774\uba70 \"PT1M\"\uc740 1\ubd84\uc758 \uae30\uac04\uc744 \ub9d0\ud55c\ub2e4(T\ub294 \uc2dc\uac04 \uac12 \uc55e\uc5d0 \uc624\ub294 \uc2dc\uac04 \uc9c0\uc815\uc790\uc784\uc744 \uc54c\uc544\ub450\uc790).", "paragraph_id": "6585410-3-0", "paragraph_question": "question: \ubaa8\ud638\ud568\uc744 \uc904\uc774\uae30 \uc704\ud574 \ud55c \ub2ec\uc758 \uae30\uac04\uc744 \ub098\ud0c0\ub0b4\ub294 \uac83\uc740 \ubb34\uc5c7\uc778\uac00?, context: \ubaa8\ud638\ud568\uc744 \uc904\uc774\uae30 \uc704\ud574, \"P1M\"\uc740 \ud55c \ub2ec\uc758 \uae30\uac04\uc774\uba70 \"PT1M\"\uc740 1\ubd84\uc758 \uae30\uac04\uc744 \ub9d0\ud55c\ub2e4(T\ub294 \uc2dc\uac04 \uac12 \uc55e\uc5d0 \uc624\ub294 \uc2dc\uac04 \uc9c0\uc815\uc790\uc784\uc744 \uc54c\uc544\ub450\uc790). \uc18c\uc218\uc810\uc774 \uac00\uc7a5 \uc791\uc740 \uac12\uc73c\ub85c \uc0ac\uc6a9\ub420 \uc218 \uc788\uc5b4, \"P0.5Y\"\ub294 \ubc18\ub144\uc744 \ub098\ud0c0\ub0b8\ub2e4. \uc18c\uc218\uc810\uc740 \"P0,5Y\" \ud639\uc740 \"P0.5Y\"\uc640 \uac19\uc774, \uc27c\ud45c \ud639\uc740 \ub9c8\uce68\ud45c\ub85c \uc9c0\uc815\ub420 \uc218 \uc788\ub2e4. \ud45c\uc900\uc740 \uc544\ub798\uc5d0\uc11c \uc8fc\uc9c0\ud558\uace0 \uc788\ub294 \uac83 \uc678\uc5d0 \"carry over points\"\uc744 \ucd08\uacfc\ud558\uace0 \uc788\ub294 \uae30\uac04 \ud45c\ud604\uc5d0\uc11c\uc758 \ub0a0\uc9dc\uc640 \uc2dc\uac04 \uac12\uc744 \uae08\uc9c0\ud558\uc9c0 \uc54a\ub294\ub2e4. \uadf8\ub798\uc11c \"PT36H\"\ub294 \"P1DT12H\"\uc640 \uac19\uc740 \uae30\uac04\uc744 \ub098\ud0c0\ub0b4\ub294\ub370 \uc0ac\uc6a9\ub420 \uc218\ub3c4 \uc788\ub2e4. \uadf8\ub7ec\ub098 \"PT36H\"\ub294 \uc77c\uad11 \uc808\uc57d \uc2dc\uac04(\uc11c\uba38 \ud0c0\uc784, daylight saving time)\uc5d0\uc11c \ud639\uc740 \uc73c\ub85c \uc804\ud658\ud558\ub294 \uacbd\uc6b0 \"P1DT12H\"\uc640 \uac19\uc9c0 \uc54a\ub2e4\ub294 \uac83\uc744 \uc54c\uc544\ub46c\uc57c \ud55c\ub2e4."} {"question": "\ubab0\ub798\uc2a8\uc740 \uc218\uc220 \ud6c4 \uba87 \ub144\uc774 \uc9c0\ub098\uc11c \uc774\uc0ac\ub97c \uac14\ub294\uac00?", "paragraph": "Corkin\uc5d0 \ub530\ub974\uba74, \ubab0\ub798\uc2a8\uc758 \uae30\uc5b5 \ub2a5\ub825 \uc2e4\ud5d8\uc740 \ub610\ud55c \uacf5\uac04\uae30\uc5b5\uacfc \uacf5\uac04 \uc815\ubcf4\uc758 \ud504\ub85c\uc138\uc2a4\ub97c \ucc98\ub9ac\ud558\ub294 \uc2e0\uacbd \uad6c\uc870\uc5d0 \ub300\ud55c \uc778\uc0ac\uc774\ud2b8\ub97c \uc81c\uacf5\ud55c\ub2e4. \uadf8\uc758 \uc0c8\ub85c\uc6b4 \uc0bd\ud654\uc640 \uc0ac\uc2e4\uc801\uc778 \uc7a5\uae30 \uae30\uc5b5 \ud615\uc131 \uc77c\ubc18\uc801\uc778 \uc7a5\uc560\uc640 \ud2b9\uc815 \uacf5\uac04 \uae30\uc5b5 \uc2dc\ud5d8\uc758 \uc2ec\ud55c \uc7a5\uc560\uc5d0\ub3c4 \ubd88\uad6c\ud558\uace0 \ubab0\ub798\uc2a8\uc740 \uadf8\uac00 \uac70\uc8fc \uc9c0\ud615\uc758 \ub808\uc774\uc544\uc6c3\uc758 \ub9e4\uc6b0 \uc0c1\uc138\ud55c \uc9c0\ub3c4\ub97c \uadf8\ub9b4 \uc218 \uac00 \uc788\uc5c8\ub2e4. \uc774\uac83\uc740 \ubab0\ub798\uc2a8\uc774 \uc218\uc220\ud55c \ud6c4 5\ub144 \ud6c4\uc5d0 \uc774\uc0ac\ub97c \uac14\uae30 \ub54c\ubb38\uc5d0 \ub180\ub77c\uc6b4 \ubc1c\uacac\uc774\ub2e4. \ub530\ub77c\uc11c \uadf8\uc758 \uc2ec\ud55c \uc21c\ud589\uc131 \uae30\uc5b5 \uc0c1\uc2e4\uc99d\uacfc \uc778\uc0ac\uc774\ud2b8\uc640 \uac19\uc740 \ub2e4\ub978 \ucf00\uc774\uc2a4\ub294 \uc77c\ubc18\uc801\uc778 \uae30\ub300\ub294 \uc815\ubc00\ud55c \uae30\uc5b5\uc758 \ud68d\ub4dd\uc5d0 \uc7a5\uc560\uac00 \uc788\ub294 \uac83\uc774\ub2e4. Corkin\uc740 \ubab0\ub798\uc2a8\uc774 \ubc29\uc5d0\uc11c \ubc29\uc73c\ub85c \ub9e4\uc77c \uc6c0\uc9c1\uc778 \uacb0\uacfc\ub85c \uc790\uc2e0\uc758 \uc9d1\uc758 \uacf5\uac04 \ub808\uc774\uc544\uc6c3\uc758 \uc778\uc9c0 \uc9c0\ub3c4\ub97c \uad6c\ucd95\ud560 \uc218 \uc788\uc5c8\ub2e4\uace0 \uac00\uc815\ud588\ub2e4.", "answer": "5\ub144", "sentence": "\uc774\uac83\uc740 \ubab0\ub798\uc2a8\uc774 \uc218\uc220\ud55c \ud6c4 5\ub144 \ud6c4\uc5d0 \uc774\uc0ac\ub97c \uac14\uae30 \ub54c\ubb38\uc5d0 \ub180\ub77c\uc6b4 \ubc1c\uacac\uc774\ub2e4.", "paragraph_sentence": "Corkin\uc5d0 \ub530\ub974\uba74, \ubab0\ub798\uc2a8\uc758 \uae30\uc5b5 \ub2a5\ub825 \uc2e4\ud5d8\uc740 \ub610\ud55c \uacf5\uac04\uae30\uc5b5\uacfc \uacf5\uac04 \uc815\ubcf4\uc758 \ud504\ub85c\uc138\uc2a4\ub97c \ucc98\ub9ac\ud558\ub294 \uc2e0\uacbd \uad6c\uc870\uc5d0 \ub300\ud55c \uc778\uc0ac\uc774\ud2b8\ub97c \uc81c\uacf5\ud55c\ub2e4. \uadf8\uc758 \uc0c8\ub85c\uc6b4 \uc0bd\ud654\uc640 \uc0ac\uc2e4\uc801\uc778 \uc7a5\uae30 \uae30\uc5b5 \ud615\uc131 \uc77c\ubc18\uc801\uc778 \uc7a5\uc560\uc640 \ud2b9\uc815 \uacf5\uac04 \uae30\uc5b5 \uc2dc\ud5d8\uc758 \uc2ec\ud55c \uc7a5\uc560\uc5d0\ub3c4 \ubd88\uad6c\ud558\uace0 \ubab0\ub798\uc2a8\uc740 \uadf8\uac00 \uac70\uc8fc \uc9c0\ud615\uc758 \ub808\uc774\uc544\uc6c3\uc758 \ub9e4\uc6b0 \uc0c1\uc138\ud55c \uc9c0\ub3c4\ub97c \uadf8\ub9b4 \uc218 \uac00 \uc788\uc5c8\ub2e4. \uc774\uac83\uc740 \ubab0\ub798\uc2a8\uc774 \uc218\uc220\ud55c \ud6c4 5\ub144 \ud6c4\uc5d0 \uc774\uc0ac\ub97c \uac14\uae30 \ub54c\ubb38\uc5d0 \ub180\ub77c\uc6b4 \ubc1c\uacac\uc774\ub2e4. \ub530\ub77c\uc11c \uadf8\uc758 \uc2ec\ud55c \uc21c\ud589\uc131 \uae30\uc5b5 \uc0c1\uc2e4\uc99d\uacfc \uc778\uc0ac\uc774\ud2b8\uc640 \uac19\uc740 \ub2e4\ub978 \ucf00\uc774\uc2a4\ub294 \uc77c\ubc18\uc801\uc778 \uae30\ub300\ub294 \uc815\ubc00\ud55c \uae30\uc5b5\uc758 \ud68d\ub4dd\uc5d0 \uc7a5\uc560\uac00 \uc788\ub294 \uac83\uc774\ub2e4. Corkin\uc740 \ubab0\ub798\uc2a8\uc774 \ubc29\uc5d0\uc11c \ubc29\uc73c\ub85c \ub9e4\uc77c \uc6c0\uc9c1\uc778 \uacb0\uacfc\ub85c \uc790\uc2e0\uc758 \uc9d1\uc758 \uacf5\uac04 \ub808\uc774\uc544\uc6c3\uc758 \uc778\uc9c0 \uc9c0\ub3c4\ub97c \uad6c\ucd95\ud560 \uc218 \uc788\uc5c8\ub2e4\uace0 \uac00\uc815\ud588\ub2e4.", "paragraph_answer": "Corkin\uc5d0 \ub530\ub974\uba74, \ubab0\ub798\uc2a8\uc758 \uae30\uc5b5 \ub2a5\ub825 \uc2e4\ud5d8\uc740 \ub610\ud55c \uacf5\uac04\uae30\uc5b5\uacfc \uacf5\uac04 \uc815\ubcf4\uc758 \ud504\ub85c\uc138\uc2a4\ub97c \ucc98\ub9ac\ud558\ub294 \uc2e0\uacbd \uad6c\uc870\uc5d0 \ub300\ud55c \uc778\uc0ac\uc774\ud2b8\ub97c \uc81c\uacf5\ud55c\ub2e4. \uadf8\uc758 \uc0c8\ub85c\uc6b4 \uc0bd\ud654\uc640 \uc0ac\uc2e4\uc801\uc778 \uc7a5\uae30 \uae30\uc5b5 \ud615\uc131 \uc77c\ubc18\uc801\uc778 \uc7a5\uc560\uc640 \ud2b9\uc815 \uacf5\uac04 \uae30\uc5b5 \uc2dc\ud5d8\uc758 \uc2ec\ud55c \uc7a5\uc560\uc5d0\ub3c4 \ubd88\uad6c\ud558\uace0 \ubab0\ub798\uc2a8\uc740 \uadf8\uac00 \uac70\uc8fc \uc9c0\ud615\uc758 \ub808\uc774\uc544\uc6c3\uc758 \ub9e4\uc6b0 \uc0c1\uc138\ud55c \uc9c0\ub3c4\ub97c \uadf8\ub9b4 \uc218 \uac00 \uc788\uc5c8\ub2e4. \uc774\uac83\uc740 \ubab0\ub798\uc2a8\uc774 \uc218\uc220\ud55c \ud6c4 5\ub144 \ud6c4\uc5d0 \uc774\uc0ac\ub97c \uac14\uae30 \ub54c\ubb38\uc5d0 \ub180\ub77c\uc6b4 \ubc1c\uacac\uc774\ub2e4. \ub530\ub77c\uc11c \uadf8\uc758 \uc2ec\ud55c \uc21c\ud589\uc131 \uae30\uc5b5 \uc0c1\uc2e4\uc99d\uacfc \uc778\uc0ac\uc774\ud2b8\uc640 \uac19\uc740 \ub2e4\ub978 \ucf00\uc774\uc2a4\ub294 \uc77c\ubc18\uc801\uc778 \uae30\ub300\ub294 \uc815\ubc00\ud55c \uae30\uc5b5\uc758 \ud68d\ub4dd\uc5d0 \uc7a5\uc560\uac00 \uc788\ub294 \uac83\uc774\ub2e4. Corkin\uc740 \ubab0\ub798\uc2a8\uc774 \ubc29\uc5d0\uc11c \ubc29\uc73c\ub85c \ub9e4\uc77c \uc6c0\uc9c1\uc778 \uacb0\uacfc\ub85c \uc790\uc2e0\uc758 \uc9d1\uc758 \uacf5\uac04 \ub808\uc774\uc544\uc6c3\uc758 \uc778\uc9c0 \uc9c0\ub3c4\ub97c \uad6c\ucd95\ud560 \uc218 \uc788\uc5c8\ub2e4\uace0 \uac00\uc815\ud588\ub2e4.", "sentence_answer": "\uc774\uac83\uc740 \ubab0\ub798\uc2a8\uc774 \uc218\uc220\ud55c \ud6c4 5\ub144 \ud6c4\uc5d0 \uc774\uc0ac\ub97c \uac14\uae30 \ub54c\ubb38\uc5d0 \ub180\ub77c\uc6b4 \ubc1c\uacac\uc774\ub2e4.", "paragraph_id": "6237013-3-0", "paragraph_question": "question: \ubab0\ub798\uc2a8\uc740 \uc218\uc220 \ud6c4 \uba87 \ub144\uc774 \uc9c0\ub098\uc11c \uc774\uc0ac\ub97c \uac14\ub294\uac00?, context: Corkin\uc5d0 \ub530\ub974\uba74, \ubab0\ub798\uc2a8\uc758 \uae30\uc5b5 \ub2a5\ub825 \uc2e4\ud5d8\uc740 \ub610\ud55c \uacf5\uac04\uae30\uc5b5\uacfc \uacf5\uac04 \uc815\ubcf4\uc758 \ud504\ub85c\uc138\uc2a4\ub97c \ucc98\ub9ac\ud558\ub294 \uc2e0\uacbd \uad6c\uc870\uc5d0 \ub300\ud55c \uc778\uc0ac\uc774\ud2b8\ub97c \uc81c\uacf5\ud55c\ub2e4. \uadf8\uc758 \uc0c8\ub85c\uc6b4 \uc0bd\ud654\uc640 \uc0ac\uc2e4\uc801\uc778 \uc7a5\uae30 \uae30\uc5b5 \ud615\uc131 \uc77c\ubc18\uc801\uc778 \uc7a5\uc560\uc640 \ud2b9\uc815 \uacf5\uac04 \uae30\uc5b5 \uc2dc\ud5d8\uc758 \uc2ec\ud55c \uc7a5\uc560\uc5d0\ub3c4 \ubd88\uad6c\ud558\uace0 \ubab0\ub798\uc2a8\uc740 \uadf8\uac00 \uac70\uc8fc \uc9c0\ud615\uc758 \ub808\uc774\uc544\uc6c3\uc758 \ub9e4\uc6b0 \uc0c1\uc138\ud55c \uc9c0\ub3c4\ub97c \uadf8\ub9b4 \uc218 \uac00 \uc788\uc5c8\ub2e4. \uc774\uac83\uc740 \ubab0\ub798\uc2a8\uc774 \uc218\uc220\ud55c \ud6c4 5\ub144 \ud6c4\uc5d0 \uc774\uc0ac\ub97c \uac14\uae30 \ub54c\ubb38\uc5d0 \ub180\ub77c\uc6b4 \ubc1c\uacac\uc774\ub2e4. \ub530\ub77c\uc11c \uadf8\uc758 \uc2ec\ud55c \uc21c\ud589\uc131 \uae30\uc5b5 \uc0c1\uc2e4\uc99d\uacfc \uc778\uc0ac\uc774\ud2b8\uc640 \uac19\uc740 \ub2e4\ub978 \ucf00\uc774\uc2a4\ub294 \uc77c\ubc18\uc801\uc778 \uae30\ub300\ub294 \uc815\ubc00\ud55c \uae30\uc5b5\uc758 \ud68d\ub4dd\uc5d0 \uc7a5\uc560\uac00 \uc788\ub294 \uac83\uc774\ub2e4. Corkin\uc740 \ubab0\ub798\uc2a8\uc774 \ubc29\uc5d0\uc11c \ubc29\uc73c\ub85c \ub9e4\uc77c \uc6c0\uc9c1\uc778 \uacb0\uacfc\ub85c \uc790\uc2e0\uc758 \uc9d1\uc758 \uacf5\uac04 \ub808\uc774\uc544\uc6c3\uc758 \uc778\uc9c0 \uc9c0\ub3c4\ub97c \uad6c\ucd95\ud560 \uc218 \uc788\uc5c8\ub2e4\uace0 \uac00\uc815\ud588\ub2e4."} {"question": "\ub300\uad6c FC\uc758 \uccab \ubc88\uc9f8 \uac10\ub3c5\uc774 \ub41c \uc0ac\ub78c\uc740 \ub204\uad6c\uc778\uac00?", "paragraph": "\ub300\uad6c FC\ub294 2002\ub144 FIFA \uc6d4\ub4dc\ucef5\uc758 \ucd95\uad6c \ubd90\uc5d0 \ud798\uc785\uc5b4 \ub300\ud55c\ubbfc\uad6d \ucd5c\ucd08\ub85c \uc2dc\ubbfc \uad6c\ub2e8\uc758 \uac1c\ub150\uc744 \ub3c4\uc785\ud558\uba74\uc11c \ucc3d\ub2e8\ub418\uc5c8\ub2e4. \uc6d4\ub4dc\ucef5\uc774 \ub05d\ub09c \ud6c4 2002\ub144 8\uc6d4 6\uc77c \ucc3d\ub9bd \ud68c\uc758\ub97c \uc2dc\uc791\uc73c\ub85c 2002\ub144 10\uc6d4 9\uc77c (\uc8fc)\ub300\uad6c\uc2dc\ubbfc\ud504\ub85c\ucd95\uad6c\ub2e8\uc774\ub77c\ub294 \uacf5\uc2dd \uba85\uce6d\uc73c\ub85c \uccab \ucc3d\ub9bd \ucd1d\ud68c\ub97c \uc5f4\uba74\uc11c \ucc3d\ub2e8\ud558\uac8c \ub41c\ub2e4. \uc774\ud6c4 2002\ub144 11\uc6d4 15\uc77c\ubd80\ud130 12\uc6d4 24\uc77c\uac04 1\ucc28 \uc2dc\ubbfc\uc8fc \uacf5\ubaa8\ub97c \uc2e4\uc2dc\ud574 127\uc5b5\uc6d0\uc758 \uc790\uae08\uc744 \ud655\ubcf4\ud558\uc600\uc73c\uba70, 2002\ub144 12\uc6d4 26\uc77c \ud55c\uad6d\ud504\ub85c\ucd95\uad6c\uc5f0\ub9f9\uc5d0 \uc758\ud574 \ucc3d\ub2e8\uc774 \uc2b9\uc778\ub418\uc5c8\ub2e4. 2003\ub144 3\uc6d4 19\uc77c \uacf5\uc2dd \ucc3d\ub2e8\uc2dd\uc744 \uac70\ud589\ud558\uc600\uc73c\uba70 \ucd08\ub300 \uac10\ub3c5\uc73c\ub85c 1980\ub144\ub300\uc640 1990\ub144\ub300 \ub300\ud55c\ubbfc\uad6d \uad6d\uac00\ub300\ud45c\ud300\uc744 \uc774\ub04c\uc5c8\ub358 \ubc15\uc885\ud658\uc744 \uc120\uc784\ud558\uace0 K\ub9ac\uadf8 2003 \uc2dc\uc98c\uc5d0 \ucc38\uac00\ud558\uc600\ub2e4. \uac19\uc740 \ud574 \uad11\uc8fc \uc0c1\ubb34 \ubd88\uc0ac\uc870\uc758 \ucc38\uac00\ub85c \ucd1d 12\uac1c\uc758 \ud300\uc774 \ub9ac\uadf8\ub97c \uad6c\uc131\ud558\uc600\ub2e4. \uccab \ubc88\uc9f8 \uc2dc\uc98c\uc5d0 \uc624\uc8fc\ud3ec\uac00 \uc8fc\uc7a5\uc73c\ub85c \uc120\uc784\ub418\uc5c8\uace0, \uae40\ud559\ucca0\uc774 \ubd80\uc8fc\uc7a5\uc73c\ub85c \uc784\uba85\ub418\uc5c8\ub2e4. \ub450 \uba85\uc758 \uccb4\ucf54 \uc120\uc218\uc778 \ub85c\ub9cc\uacfc \uc580\uc744 \uc601\uc785\ud558\uc600\uace0, \ud130\ud0a4 \uad6d\uac00\ub300\ud45c \ucd9c\uc2e0 \uc218\ube44\uc218 \ub77c\ud798 \uc790\ud30c\ub974 \ub610\ud55c \uc120\uc218 \uc0dd\ud65c\uc758 \ud669\ud63c\uae30\uc5d0 \ub300\uad6c\ub85c \uc654\ub2e4.", "answer": "\ubc15\uc885\ud658", "sentence": "1980\ub144\ub300\uc640 1990\ub144\ub300 \ub300\ud55c\ubbfc\uad6d \uad6d\uac00\ub300\ud45c\ud300\uc744 \uc774\ub04c\uc5c8\ub358 \ubc15\uc885\ud658 \uc744 \uc120\uc784\ud558\uace0 K\ub9ac\uadf8 2003 \uc2dc\uc98c\uc5d0 \ucc38\uac00\ud558\uc600\ub2e4.", "paragraph_sentence": "\ub300\uad6c FC\ub294 2002\ub144 FIFA \uc6d4\ub4dc\ucef5\uc758 \ucd95\uad6c \ubd90\uc5d0 \ud798\uc785\uc5b4 \ub300\ud55c\ubbfc\uad6d \ucd5c\ucd08\ub85c \uc2dc\ubbfc \uad6c\ub2e8\uc758 \uac1c\ub150\uc744 \ub3c4\uc785\ud558\uba74\uc11c \ucc3d\ub2e8\ub418\uc5c8\ub2e4. \uc6d4\ub4dc\ucef5\uc774 \ub05d\ub09c \ud6c4 2002\ub144 8\uc6d4 6\uc77c \ucc3d\ub9bd \ud68c\uc758\ub97c \uc2dc\uc791\uc73c\ub85c 2002\ub144 10\uc6d4 9\uc77c (\uc8fc)\ub300\uad6c\uc2dc\ubbfc\ud504\ub85c\ucd95\uad6c\ub2e8\uc774\ub77c\ub294 \uacf5\uc2dd \uba85\uce6d\uc73c\ub85c \uccab \ucc3d\ub9bd \ucd1d\ud68c\ub97c \uc5f4\uba74\uc11c \ucc3d\ub2e8\ud558\uac8c \ub41c\ub2e4. \uc774\ud6c4 2002\ub144 11\uc6d4 15\uc77c\ubd80\ud130 12\uc6d4 24\uc77c\uac04 1\ucc28 \uc2dc\ubbfc\uc8fc \uacf5\ubaa8\ub97c \uc2e4\uc2dc\ud574 127\uc5b5\uc6d0\uc758 \uc790\uae08\uc744 \ud655\ubcf4\ud558\uc600\uc73c\uba70, 2002\ub144 12\uc6d4 26\uc77c \ud55c\uad6d\ud504\ub85c\ucd95\uad6c\uc5f0\ub9f9\uc5d0 \uc758\ud574 \ucc3d\ub2e8\uc774 \uc2b9\uc778\ub418\uc5c8\ub2e4. 2003\ub144 3\uc6d4 19\uc77c \uacf5\uc2dd \ucc3d\ub2e8\uc2dd\uc744 \uac70\ud589\ud558\uc600\uc73c\uba70 \ucd08\ub300 \uac10\ub3c5\uc73c\ub85c 1980\ub144\ub300\uc640 1990\ub144\ub300 \ub300\ud55c\ubbfc\uad6d \uad6d\uac00\ub300\ud45c\ud300\uc744 \uc774\ub04c\uc5c8\ub358 \ubc15\uc885\ud658 \uc744 \uc120\uc784\ud558\uace0 K\ub9ac\uadf8 2003 \uc2dc\uc98c\uc5d0 \ucc38\uac00\ud558\uc600\ub2e4. \uac19\uc740 \ud574 \uad11\uc8fc \uc0c1\ubb34 \ubd88\uc0ac\uc870\uc758 \ucc38\uac00\ub85c \ucd1d 12\uac1c\uc758 \ud300\uc774 \ub9ac\uadf8\ub97c \uad6c\uc131\ud558\uc600\ub2e4. \uccab \ubc88\uc9f8 \uc2dc\uc98c\uc5d0 \uc624\uc8fc\ud3ec\uac00 \uc8fc\uc7a5\uc73c\ub85c \uc120\uc784\ub418\uc5c8\uace0, \uae40\ud559\ucca0\uc774 \ubd80\uc8fc\uc7a5\uc73c\ub85c \uc784\uba85\ub418\uc5c8\ub2e4. \ub450 \uba85\uc758 \uccb4\ucf54 \uc120\uc218\uc778 \ub85c\ub9cc\uacfc \uc580\uc744 \uc601\uc785\ud558\uc600\uace0, \ud130\ud0a4 \uad6d\uac00\ub300\ud45c \ucd9c\uc2e0 \uc218\ube44\uc218 \ub77c\ud798 \uc790\ud30c\ub974 \ub610\ud55c \uc120\uc218 \uc0dd\ud65c\uc758 \ud669\ud63c\uae30\uc5d0 \ub300\uad6c\ub85c \uc654\ub2e4.", "paragraph_answer": "\ub300\uad6c FC\ub294 2002\ub144 FIFA \uc6d4\ub4dc\ucef5\uc758 \ucd95\uad6c \ubd90\uc5d0 \ud798\uc785\uc5b4 \ub300\ud55c\ubbfc\uad6d \ucd5c\ucd08\ub85c \uc2dc\ubbfc \uad6c\ub2e8\uc758 \uac1c\ub150\uc744 \ub3c4\uc785\ud558\uba74\uc11c \ucc3d\ub2e8\ub418\uc5c8\ub2e4. \uc6d4\ub4dc\ucef5\uc774 \ub05d\ub09c \ud6c4 2002\ub144 8\uc6d4 6\uc77c \ucc3d\ub9bd \ud68c\uc758\ub97c \uc2dc\uc791\uc73c\ub85c 2002\ub144 10\uc6d4 9\uc77c (\uc8fc)\ub300\uad6c\uc2dc\ubbfc\ud504\ub85c\ucd95\uad6c\ub2e8\uc774\ub77c\ub294 \uacf5\uc2dd \uba85\uce6d\uc73c\ub85c \uccab \ucc3d\ub9bd \ucd1d\ud68c\ub97c \uc5f4\uba74\uc11c \ucc3d\ub2e8\ud558\uac8c \ub41c\ub2e4. \uc774\ud6c4 2002\ub144 11\uc6d4 15\uc77c\ubd80\ud130 12\uc6d4 24\uc77c\uac04 1\ucc28 \uc2dc\ubbfc\uc8fc \uacf5\ubaa8\ub97c \uc2e4\uc2dc\ud574 127\uc5b5\uc6d0\uc758 \uc790\uae08\uc744 \ud655\ubcf4\ud558\uc600\uc73c\uba70, 2002\ub144 12\uc6d4 26\uc77c \ud55c\uad6d\ud504\ub85c\ucd95\uad6c\uc5f0\ub9f9\uc5d0 \uc758\ud574 \ucc3d\ub2e8\uc774 \uc2b9\uc778\ub418\uc5c8\ub2e4. 2003\ub144 3\uc6d4 19\uc77c \uacf5\uc2dd \ucc3d\ub2e8\uc2dd\uc744 \uac70\ud589\ud558\uc600\uc73c\uba70 \ucd08\ub300 \uac10\ub3c5\uc73c\ub85c 1980\ub144\ub300\uc640 1990\ub144\ub300 \ub300\ud55c\ubbfc\uad6d \uad6d\uac00\ub300\ud45c\ud300\uc744 \uc774\ub04c\uc5c8\ub358 \ubc15\uc885\ud658 \uc744 \uc120\uc784\ud558\uace0 K\ub9ac\uadf8 2003 \uc2dc\uc98c\uc5d0 \ucc38\uac00\ud558\uc600\ub2e4. \uac19\uc740 \ud574 \uad11\uc8fc \uc0c1\ubb34 \ubd88\uc0ac\uc870\uc758 \ucc38\uac00\ub85c \ucd1d 12\uac1c\uc758 \ud300\uc774 \ub9ac\uadf8\ub97c \uad6c\uc131\ud558\uc600\ub2e4. \uccab \ubc88\uc9f8 \uc2dc\uc98c\uc5d0 \uc624\uc8fc\ud3ec\uac00 \uc8fc\uc7a5\uc73c\ub85c \uc120\uc784\ub418\uc5c8\uace0, \uae40\ud559\ucca0\uc774 \ubd80\uc8fc\uc7a5\uc73c\ub85c \uc784\uba85\ub418\uc5c8\ub2e4. \ub450 \uba85\uc758 \uccb4\ucf54 \uc120\uc218\uc778 \ub85c\ub9cc\uacfc \uc580\uc744 \uc601\uc785\ud558\uc600\uace0, \ud130\ud0a4 \uad6d\uac00\ub300\ud45c \ucd9c\uc2e0 \uc218\ube44\uc218 \ub77c\ud798 \uc790\ud30c\ub974 \ub610\ud55c \uc120\uc218 \uc0dd\ud65c\uc758 \ud669\ud63c\uae30\uc5d0 \ub300\uad6c\ub85c \uc654\ub2e4.", "sentence_answer": "1980\ub144\ub300\uc640 1990\ub144\ub300 \ub300\ud55c\ubbfc\uad6d \uad6d\uac00\ub300\ud45c\ud300\uc744 \uc774\ub04c\uc5c8\ub358 \ubc15\uc885\ud658 \uc744 \uc120\uc784\ud558\uace0 K\ub9ac\uadf8 2003 \uc2dc\uc98c\uc5d0 \ucc38\uac00\ud558\uc600\ub2e4.", "paragraph_id": "6485302-0-1", "paragraph_question": "question: \ub300\uad6c FC\uc758 \uccab \ubc88\uc9f8 \uac10\ub3c5\uc774 \ub41c \uc0ac\ub78c\uc740 \ub204\uad6c\uc778\uac00?, context: \ub300\uad6c FC\ub294 2002\ub144 FIFA \uc6d4\ub4dc\ucef5\uc758 \ucd95\uad6c \ubd90\uc5d0 \ud798\uc785\uc5b4 \ub300\ud55c\ubbfc\uad6d \ucd5c\ucd08\ub85c \uc2dc\ubbfc \uad6c\ub2e8\uc758 \uac1c\ub150\uc744 \ub3c4\uc785\ud558\uba74\uc11c \ucc3d\ub2e8\ub418\uc5c8\ub2e4. \uc6d4\ub4dc\ucef5\uc774 \ub05d\ub09c \ud6c4 2002\ub144 8\uc6d4 6\uc77c \ucc3d\ub9bd \ud68c\uc758\ub97c \uc2dc\uc791\uc73c\ub85c 2002\ub144 10\uc6d4 9\uc77c (\uc8fc)\ub300\uad6c\uc2dc\ubbfc\ud504\ub85c\ucd95\uad6c\ub2e8\uc774\ub77c\ub294 \uacf5\uc2dd \uba85\uce6d\uc73c\ub85c \uccab \ucc3d\ub9bd \ucd1d\ud68c\ub97c \uc5f4\uba74\uc11c \ucc3d\ub2e8\ud558\uac8c \ub41c\ub2e4. \uc774\ud6c4 2002\ub144 11\uc6d4 15\uc77c\ubd80\ud130 12\uc6d4 24\uc77c\uac04 1\ucc28 \uc2dc\ubbfc\uc8fc \uacf5\ubaa8\ub97c \uc2e4\uc2dc\ud574 127\uc5b5\uc6d0\uc758 \uc790\uae08\uc744 \ud655\ubcf4\ud558\uc600\uc73c\uba70, 2002\ub144 12\uc6d4 26\uc77c \ud55c\uad6d\ud504\ub85c\ucd95\uad6c\uc5f0\ub9f9\uc5d0 \uc758\ud574 \ucc3d\ub2e8\uc774 \uc2b9\uc778\ub418\uc5c8\ub2e4. 2003\ub144 3\uc6d4 19\uc77c \uacf5\uc2dd \ucc3d\ub2e8\uc2dd\uc744 \uac70\ud589\ud558\uc600\uc73c\uba70 \ucd08\ub300 \uac10\ub3c5\uc73c\ub85c 1980\ub144\ub300\uc640 1990\ub144\ub300 \ub300\ud55c\ubbfc\uad6d \uad6d\uac00\ub300\ud45c\ud300\uc744 \uc774\ub04c\uc5c8\ub358 \ubc15\uc885\ud658\uc744 \uc120\uc784\ud558\uace0 K\ub9ac\uadf8 2003 \uc2dc\uc98c\uc5d0 \ucc38\uac00\ud558\uc600\ub2e4. \uac19\uc740 \ud574 \uad11\uc8fc \uc0c1\ubb34 \ubd88\uc0ac\uc870\uc758 \ucc38\uac00\ub85c \ucd1d 12\uac1c\uc758 \ud300\uc774 \ub9ac\uadf8\ub97c \uad6c\uc131\ud558\uc600\ub2e4. \uccab \ubc88\uc9f8 \uc2dc\uc98c\uc5d0 \uc624\uc8fc\ud3ec\uac00 \uc8fc\uc7a5\uc73c\ub85c \uc120\uc784\ub418\uc5c8\uace0, \uae40\ud559\ucca0\uc774 \ubd80\uc8fc\uc7a5\uc73c\ub85c \uc784\uba85\ub418\uc5c8\ub2e4. \ub450 \uba85\uc758 \uccb4\ucf54 \uc120\uc218\uc778 \ub85c\ub9cc\uacfc \uc580\uc744 \uc601\uc785\ud558\uc600\uace0, \ud130\ud0a4 \uad6d\uac00\ub300\ud45c \ucd9c\uc2e0 \uc218\ube44\uc218 \ub77c\ud798 \uc790\ud30c\ub974 \ub610\ud55c \uc120\uc218 \uc0dd\ud65c\uc758 \ud669\ud63c\uae30\uc5d0 \ub300\uad6c\ub85c \uc654\ub2e4."} {"question": "1848\ub144 9\uc6d4 16\uc77c\uc740 \uba87 \ubc88\uc9f8 \uad6d\ubbfc\uc758\ud68c \ud22c\ud45c\uc778\uac00?", "paragraph": "1848\ub144 9\uc6d4 16\uc77c\uc758 \ub450 \ubc88\uc9f8 \ud22c\ud45c\uc5d0\uc11c \uad6d\ubbfc\uc758\ud68c\ub294 \uc0ac\uc2e4\uc0c1\uc758 \uc0c1\ud0dc\uc5d0 \uc21c\uc751\ud558\uc5ec \uacfc\ubc18\uc218\ub97c \uac04\uc2e0\ud788 \ub118\uaca8 \ud3c9\ud654\uc870\uc57d\uc744 \ube44\uc900\ud588\ub2e4. \ud3c9\ud654\uc870\uc57d\uc740 \ub3c5\uc77c \ubbfc\uc871\uc8fc\uc758\uc5d0 \ubc18\ud558\ub294 \uac83\uc774\uc5c8\uace0, \uc774\uac83\uc758 \uc2b9\uc778\uc740 \uad6d\ubbfc\uc758\ud68c\uc758 \ubb34\ub2a5\ud568\uc744 \ub4dc\ub7ec\ub0c8\ub2e4. \uc774\ub85c \uc778\ud574 \ud504\ub791\ud06c\ud478\ub974\ud2b8\uc5d0\uc11c \ubbfc\uc911\ubd09\uae30\uc778 9\uc6d4 \ud601\uba85(Septemberrevolution)\uc774 \uc77c\uc5b4\ub098, \uc5ec\uae30\uc11c \uce74\uc9c0\ub178 \ub2f9\ud30c\uc5d0 \ud3ec\ud568\ub418\ub294 \uc758\uc6d0\uc778 \ud3a0\ub9ad\uc2a4 \ud4cc\ub974\uc2a4\ud2b8 \ub9ac\ud750\ub178\ube0c\uc2a4\ud0a4(Felix F\u00fcrst Lichnowsky)\uc640 \ud55c\uc2a4 \uc544\ub3cc\ud504 \uc5d0\ub974\ud2b8\ub9cc \ud3f0 \uc544\uc6b0\uc5b4\uc2a4\ubc1c\ud2b8(Hans Adolf Erdmann von Auerswald)\uac00 \uc0b4\ud574\ub2f9\ud588\uace0, \uad6d\ubbfc\uc758\ud68c\ub294 \uc774\uc5d0 \ub300\ud56d\ud574 \uacb0\uad6d \ud504\ub85c\uc774\uc13c\uacfc \uc624\uc2a4\ud2b8\ub9ac\uc544\uc758 \uc5f0\ubc29 \uad70\ub300\ub97c \ub9c8\uc778\uce20\uc5d0 \uc788\ub294 \uc5f0\ubc29 \uc694\uc0c8\ub85c \ubd88\ub7ec\uc62c \uc218\ubc16\uc5d0 \uc5c6\uc5c8\ub2e4.", "answer": "\ub450 \ubc88\uc9f8", "sentence": "1848\ub144 9\uc6d4 16\uc77c\uc758 \ub450 \ubc88\uc9f8 \ud22c\ud45c\uc5d0\uc11c \uad6d\ubbfc\uc758\ud68c\ub294 \uc0ac\uc2e4\uc0c1\uc758 \uc0c1\ud0dc\uc5d0 \uc21c\uc751\ud558\uc5ec \uacfc\ubc18\uc218\ub97c \uac04\uc2e0\ud788 \ub118\uaca8 \ud3c9\ud654\uc870\uc57d\uc744 \ube44\uc900\ud588\ub2e4.", "paragraph_sentence": " 1848\ub144 9\uc6d4 16\uc77c\uc758 \ub450 \ubc88\uc9f8 \ud22c\ud45c\uc5d0\uc11c \uad6d\ubbfc\uc758\ud68c\ub294 \uc0ac\uc2e4\uc0c1\uc758 \uc0c1\ud0dc\uc5d0 \uc21c\uc751\ud558\uc5ec \uacfc\ubc18\uc218\ub97c \uac04\uc2e0\ud788 \ub118\uaca8 \ud3c9\ud654\uc870\uc57d\uc744 \ube44\uc900\ud588\ub2e4. \ud3c9\ud654\uc870\uc57d\uc740 \ub3c5\uc77c \ubbfc\uc871\uc8fc\uc758\uc5d0 \ubc18\ud558\ub294 \uac83\uc774\uc5c8\uace0, \uc774\uac83\uc758 \uc2b9\uc778\uc740 \uad6d\ubbfc\uc758\ud68c\uc758 \ubb34\ub2a5\ud568\uc744 \ub4dc\ub7ec\ub0c8\ub2e4. \uc774\ub85c \uc778\ud574 \ud504\ub791\ud06c\ud478\ub974\ud2b8\uc5d0\uc11c \ubbfc\uc911\ubd09\uae30\uc778 9\uc6d4 \ud601\uba85(Septemberrevolution)\uc774 \uc77c\uc5b4\ub098, \uc5ec\uae30\uc11c \uce74\uc9c0\ub178 \ub2f9\ud30c\uc5d0 \ud3ec\ud568\ub418\ub294 \uc758\uc6d0\uc778 \ud3a0\ub9ad\uc2a4 \ud4cc\ub974\uc2a4\ud2b8 \ub9ac\ud750\ub178\ube0c\uc2a4\ud0a4(Felix F\u00fcrst Lichnowsky)\uc640 \ud55c\uc2a4 \uc544\ub3cc\ud504 \uc5d0\ub974\ud2b8\ub9cc \ud3f0 \uc544\uc6b0\uc5b4\uc2a4\ubc1c\ud2b8(Hans Adolf Erdmann von Auerswald)\uac00 \uc0b4\ud574\ub2f9\ud588\uace0, \uad6d\ubbfc\uc758\ud68c\ub294 \uc774\uc5d0 \ub300\ud56d\ud574 \uacb0\uad6d \ud504\ub85c\uc774\uc13c\uacfc \uc624\uc2a4\ud2b8\ub9ac\uc544\uc758 \uc5f0\ubc29 \uad70\ub300\ub97c \ub9c8\uc778\uce20\uc5d0 \uc788\ub294 \uc5f0\ubc29 \uc694\uc0c8\ub85c \ubd88\ub7ec\uc62c \uc218\ubc16\uc5d0 \uc5c6\uc5c8\ub2e4.", "paragraph_answer": "1848\ub144 9\uc6d4 16\uc77c\uc758 \ub450 \ubc88\uc9f8 \ud22c\ud45c\uc5d0\uc11c \uad6d\ubbfc\uc758\ud68c\ub294 \uc0ac\uc2e4\uc0c1\uc758 \uc0c1\ud0dc\uc5d0 \uc21c\uc751\ud558\uc5ec \uacfc\ubc18\uc218\ub97c \uac04\uc2e0\ud788 \ub118\uaca8 \ud3c9\ud654\uc870\uc57d\uc744 \ube44\uc900\ud588\ub2e4. \ud3c9\ud654\uc870\uc57d\uc740 \ub3c5\uc77c \ubbfc\uc871\uc8fc\uc758\uc5d0 \ubc18\ud558\ub294 \uac83\uc774\uc5c8\uace0, \uc774\uac83\uc758 \uc2b9\uc778\uc740 \uad6d\ubbfc\uc758\ud68c\uc758 \ubb34\ub2a5\ud568\uc744 \ub4dc\ub7ec\ub0c8\ub2e4. \uc774\ub85c \uc778\ud574 \ud504\ub791\ud06c\ud478\ub974\ud2b8\uc5d0\uc11c \ubbfc\uc911\ubd09\uae30\uc778 9\uc6d4 \ud601\uba85(Septemberrevolution)\uc774 \uc77c\uc5b4\ub098, \uc5ec\uae30\uc11c \uce74\uc9c0\ub178 \ub2f9\ud30c\uc5d0 \ud3ec\ud568\ub418\ub294 \uc758\uc6d0\uc778 \ud3a0\ub9ad\uc2a4 \ud4cc\ub974\uc2a4\ud2b8 \ub9ac\ud750\ub178\ube0c\uc2a4\ud0a4(Felix F\u00fcrst Lichnowsky)\uc640 \ud55c\uc2a4 \uc544\ub3cc\ud504 \uc5d0\ub974\ud2b8\ub9cc \ud3f0 \uc544\uc6b0\uc5b4\uc2a4\ubc1c\ud2b8(Hans Adolf Erdmann von Auerswald)\uac00 \uc0b4\ud574\ub2f9\ud588\uace0, \uad6d\ubbfc\uc758\ud68c\ub294 \uc774\uc5d0 \ub300\ud56d\ud574 \uacb0\uad6d \ud504\ub85c\uc774\uc13c\uacfc \uc624\uc2a4\ud2b8\ub9ac\uc544\uc758 \uc5f0\ubc29 \uad70\ub300\ub97c \ub9c8\uc778\uce20\uc5d0 \uc788\ub294 \uc5f0\ubc29 \uc694\uc0c8\ub85c \ubd88\ub7ec\uc62c \uc218\ubc16\uc5d0 \uc5c6\uc5c8\ub2e4.", "sentence_answer": "1848\ub144 9\uc6d4 16\uc77c\uc758 \ub450 \ubc88\uc9f8 \ud22c\ud45c\uc5d0\uc11c \uad6d\ubbfc\uc758\ud68c\ub294 \uc0ac\uc2e4\uc0c1\uc758 \uc0c1\ud0dc\uc5d0 \uc21c\uc751\ud558\uc5ec \uacfc\ubc18\uc218\ub97c \uac04\uc2e0\ud788 \ub118\uaca8 \ud3c9\ud654\uc870\uc57d\uc744 \ube44\uc900\ud588\ub2e4.", "paragraph_id": "6568666-19-0", "paragraph_question": "question: 1848\ub144 9\uc6d4 16\uc77c\uc740 \uba87 \ubc88\uc9f8 \uad6d\ubbfc\uc758\ud68c \ud22c\ud45c\uc778\uac00?, context: 1848\ub144 9\uc6d4 16\uc77c\uc758 \ub450 \ubc88\uc9f8 \ud22c\ud45c\uc5d0\uc11c \uad6d\ubbfc\uc758\ud68c\ub294 \uc0ac\uc2e4\uc0c1\uc758 \uc0c1\ud0dc\uc5d0 \uc21c\uc751\ud558\uc5ec \uacfc\ubc18\uc218\ub97c \uac04\uc2e0\ud788 \ub118\uaca8 \ud3c9\ud654\uc870\uc57d\uc744 \ube44\uc900\ud588\ub2e4. \ud3c9\ud654\uc870\uc57d\uc740 \ub3c5\uc77c \ubbfc\uc871\uc8fc\uc758\uc5d0 \ubc18\ud558\ub294 \uac83\uc774\uc5c8\uace0, \uc774\uac83\uc758 \uc2b9\uc778\uc740 \uad6d\ubbfc\uc758\ud68c\uc758 \ubb34\ub2a5\ud568\uc744 \ub4dc\ub7ec\ub0c8\ub2e4. \uc774\ub85c \uc778\ud574 \ud504\ub791\ud06c\ud478\ub974\ud2b8\uc5d0\uc11c \ubbfc\uc911\ubd09\uae30\uc778 9\uc6d4 \ud601\uba85(Septemberrevolution)\uc774 \uc77c\uc5b4\ub098, \uc5ec\uae30\uc11c \uce74\uc9c0\ub178 \ub2f9\ud30c\uc5d0 \ud3ec\ud568\ub418\ub294 \uc758\uc6d0\uc778 \ud3a0\ub9ad\uc2a4 \ud4cc\ub974\uc2a4\ud2b8 \ub9ac\ud750\ub178\ube0c\uc2a4\ud0a4(Felix F\u00fcrst Lichnowsky)\uc640 \ud55c\uc2a4 \uc544\ub3cc\ud504 \uc5d0\ub974\ud2b8\ub9cc \ud3f0 \uc544\uc6b0\uc5b4\uc2a4\ubc1c\ud2b8(Hans Adolf Erdmann von Auerswald)\uac00 \uc0b4\ud574\ub2f9\ud588\uace0, \uad6d\ubbfc\uc758\ud68c\ub294 \uc774\uc5d0 \ub300\ud56d\ud574 \uacb0\uad6d \ud504\ub85c\uc774\uc13c\uacfc \uc624\uc2a4\ud2b8\ub9ac\uc544\uc758 \uc5f0\ubc29 \uad70\ub300\ub97c \ub9c8\uc778\uce20\uc5d0 \uc788\ub294 \uc5f0\ubc29 \uc694\uc0c8\ub85c \ubd88\ub7ec\uc62c \uc218\ubc16\uc5d0 \uc5c6\uc5c8\ub2e4."} {"question": "\ud074\ub808\uba58\uc2a4 7\uc138\uac00 \uad50\uad6c \uc138\uc18d \uc7ac\uc0b0\uc744 \uc5b4\ub290 \uc655\uac00\uc5d0 \ub118\uacbc\ub294\uac00?", "paragraph": "\uc2e0\uc131 \ub85c\ub9c8 \uc81c\uad6d\uc758 \ud669\uc81c \uce74\ub97c 5\uc138\ub294 \ub85c\ub9c8 \uc57d\ud0c8 \uc18c\uc2dd\uc744 \uc811\ud558\uace0 \ubb34\ucc99 \ub2f9\ud669\ud558\uace0 \ub2f9\uc7a5 \uc911\uc9c0\ud560 \uac83\uc744 \uc9c0\uc2dc\ud558\uc600\uc9c0\ub9cc, \uc790\uc2e0\uc5d0\uac8c \ub300\ud56d\ud588\ub358 \uad50\ud669 \ud074\ub808\uba58\uc2a4 7\uc138\uac00 \uad70\ub300\uc640 \ub3c4\uc2dc\ub97c \uc783\uace0 \uc0b0\ud0c4\uc824\ub85c \uc131\uc5d0 \uc0ac\uc2e4\uc0c1 \uc720\ud3d0\ub098 \ub2e4\ub984\uc5c6\ub294 \uc2e0\uc138\ub85c \uc804\ub77d\ud55c \uc0ac\uc2e4 \uc790\uccb4\uc5d0 \ub300\ud574\uc11c\ub294 \uadf8\ub2e4\uc9c0 \uc2eb\uc5b4\ud558\ub294 \ub0b4\uc0c9\uc744 \ubcf4\uc774\uc9c0 \uc54a\uc558\ub2e4. \uc2e4\uc81c\ub85c \uce74\ub97c 5\uc138\ub294 \ub85c\ub9c8 \uc57d\ud0c8\uc5d0 \ubd80\ubd84\uc801\uc73c\ub85c\ub294 \ucc45\uc784\uc774 \uc788\ub294\ub370, \uc774\ub294 \uadf8\uac00 \ud074\ub808\uba58\uc2a4 7\uc138\ub97c \uac1c\uc778\uc801\uc73c\ub85c \ub9cc\ub098 \uc790\uc2e0\uc758 \uc694\uad6c\uc0ac\ud56d\uc744 \uc804\ud558\uae30 \uc704\ud574\uc11c \uc790\uc2e0\uc758 \uad70\ub300\uc5d0\uac8c \uc790\uc728\uc131\uc744 \ubd80\uacfc\ud588\ub2e4\ub294 \uc0ac\uc2e4\uc774 \uc54c\ub824\uc84c\uae30 \ub54c\ubb38\uc774\uc5c8\ub2e4. \ud074\ub808\uba58\uc2a4 7\uc138\ub294 \uce74\ub97c 5\uc138\ub97c \uc5b8\uc9e2\uac8c \ud560 \ub9cc\ud55c \ud589\ub3d9\uc744 \ud558\uc9c0 \uc54a\uc74c\uc73c\ub85c\uc368 \uadf8\uc640\uc758 \uc5b4\ub5a0\ud55c \ucda9\ub3cc\ub3c4 \ud53c\ud558\ub824\uace0 \ub178\ub825\ud558\uc600\ub2e4. \uc5b4\ub5a0\ud55c \uc8fc\uc800\ud568\uacfc \uc870\uac74 \uc5c6\uc774 \ud074\ub808\uba58\uc2a4 7\uc138\ub294 \uc6b8\ud2b8\ub808\ud750\ud2b8 \uad50\uad6c\uc758 \uc138\uc18d \uc7ac\uc0b0\uc744 \ud569\uc2a4\ubd80\ub974\ud06c \uc655\uac00\uc5d0 \uc591\ub3c4\ud55c\ub2e4\ub294 \uac83\uc5d0 \ub3d9\uc758\ud558\uc600\ub2e4.", "answer": "\ud569\uc2a4\ubd80\ub974\ud06c", "sentence": "\uc5b4\ub5a0\ud55c \uc8fc\uc800\ud568\uacfc \uc870\uac74 \uc5c6\uc774 \ud074\ub808\uba58\uc2a4 7\uc138\ub294 \uc6b8\ud2b8\ub808\ud750\ud2b8 \uad50\uad6c\uc758 \uc138\uc18d \uc7ac\uc0b0\uc744 \ud569\uc2a4\ubd80\ub974\ud06c \uc655\uac00\uc5d0 \uc591\ub3c4\ud55c\ub2e4\ub294 \uac83\uc5d0 \ub3d9\uc758\ud558\uc600\ub2e4.", "paragraph_sentence": "\uc2e0\uc131 \ub85c\ub9c8 \uc81c\uad6d\uc758 \ud669\uc81c \uce74\ub97c 5\uc138\ub294 \ub85c\ub9c8 \uc57d\ud0c8 \uc18c\uc2dd\uc744 \uc811\ud558\uace0 \ubb34\ucc99 \ub2f9\ud669\ud558\uace0 \ub2f9\uc7a5 \uc911\uc9c0\ud560 \uac83\uc744 \uc9c0\uc2dc\ud558\uc600\uc9c0\ub9cc, \uc790\uc2e0\uc5d0\uac8c \ub300\ud56d\ud588\ub358 \uad50\ud669 \ud074\ub808\uba58\uc2a4 7\uc138\uac00 \uad70\ub300\uc640 \ub3c4\uc2dc\ub97c \uc783\uace0 \uc0b0\ud0c4\uc824\ub85c \uc131\uc5d0 \uc0ac\uc2e4\uc0c1 \uc720\ud3d0\ub098 \ub2e4\ub984\uc5c6\ub294 \uc2e0\uc138\ub85c \uc804\ub77d\ud55c \uc0ac\uc2e4 \uc790\uccb4\uc5d0 \ub300\ud574\uc11c\ub294 \uadf8\ub2e4\uc9c0 \uc2eb\uc5b4\ud558\ub294 \ub0b4\uc0c9\uc744 \ubcf4\uc774\uc9c0 \uc54a\uc558\ub2e4. \uc2e4\uc81c\ub85c \uce74\ub97c 5\uc138\ub294 \ub85c\ub9c8 \uc57d\ud0c8\uc5d0 \ubd80\ubd84\uc801\uc73c\ub85c\ub294 \ucc45\uc784\uc774 \uc788\ub294\ub370, \uc774\ub294 \uadf8\uac00 \ud074\ub808\uba58\uc2a4 7\uc138\ub97c \uac1c\uc778\uc801\uc73c\ub85c \ub9cc\ub098 \uc790\uc2e0\uc758 \uc694\uad6c\uc0ac\ud56d\uc744 \uc804\ud558\uae30 \uc704\ud574\uc11c \uc790\uc2e0\uc758 \uad70\ub300\uc5d0\uac8c \uc790\uc728\uc131\uc744 \ubd80\uacfc\ud588\ub2e4\ub294 \uc0ac\uc2e4\uc774 \uc54c\ub824\uc84c\uae30 \ub54c\ubb38\uc774\uc5c8\ub2e4. \ud074\ub808\uba58\uc2a4 7\uc138\ub294 \uce74\ub97c 5\uc138\ub97c \uc5b8\uc9e2\uac8c \ud560 \ub9cc\ud55c \ud589\ub3d9\uc744 \ud558\uc9c0 \uc54a\uc74c\uc73c\ub85c\uc368 \uadf8\uc640\uc758 \uc5b4\ub5a0\ud55c \ucda9\ub3cc\ub3c4 \ud53c\ud558\ub824\uace0 \ub178\ub825\ud558\uc600\ub2e4. \uc5b4\ub5a0\ud55c \uc8fc\uc800\ud568\uacfc \uc870\uac74 \uc5c6\uc774 \ud074\ub808\uba58\uc2a4 7\uc138\ub294 \uc6b8\ud2b8\ub808\ud750\ud2b8 \uad50\uad6c\uc758 \uc138\uc18d \uc7ac\uc0b0\uc744 \ud569\uc2a4\ubd80\ub974\ud06c \uc655\uac00\uc5d0 \uc591\ub3c4\ud55c\ub2e4\ub294 \uac83\uc5d0 \ub3d9\uc758\ud558\uc600\ub2e4. ", "paragraph_answer": "\uc2e0\uc131 \ub85c\ub9c8 \uc81c\uad6d\uc758 \ud669\uc81c \uce74\ub97c 5\uc138\ub294 \ub85c\ub9c8 \uc57d\ud0c8 \uc18c\uc2dd\uc744 \uc811\ud558\uace0 \ubb34\ucc99 \ub2f9\ud669\ud558\uace0 \ub2f9\uc7a5 \uc911\uc9c0\ud560 \uac83\uc744 \uc9c0\uc2dc\ud558\uc600\uc9c0\ub9cc, \uc790\uc2e0\uc5d0\uac8c \ub300\ud56d\ud588\ub358 \uad50\ud669 \ud074\ub808\uba58\uc2a4 7\uc138\uac00 \uad70\ub300\uc640 \ub3c4\uc2dc\ub97c \uc783\uace0 \uc0b0\ud0c4\uc824\ub85c \uc131\uc5d0 \uc0ac\uc2e4\uc0c1 \uc720\ud3d0\ub098 \ub2e4\ub984\uc5c6\ub294 \uc2e0\uc138\ub85c \uc804\ub77d\ud55c \uc0ac\uc2e4 \uc790\uccb4\uc5d0 \ub300\ud574\uc11c\ub294 \uadf8\ub2e4\uc9c0 \uc2eb\uc5b4\ud558\ub294 \ub0b4\uc0c9\uc744 \ubcf4\uc774\uc9c0 \uc54a\uc558\ub2e4. \uc2e4\uc81c\ub85c \uce74\ub97c 5\uc138\ub294 \ub85c\ub9c8 \uc57d\ud0c8\uc5d0 \ubd80\ubd84\uc801\uc73c\ub85c\ub294 \ucc45\uc784\uc774 \uc788\ub294\ub370, \uc774\ub294 \uadf8\uac00 \ud074\ub808\uba58\uc2a4 7\uc138\ub97c \uac1c\uc778\uc801\uc73c\ub85c \ub9cc\ub098 \uc790\uc2e0\uc758 \uc694\uad6c\uc0ac\ud56d\uc744 \uc804\ud558\uae30 \uc704\ud574\uc11c \uc790\uc2e0\uc758 \uad70\ub300\uc5d0\uac8c \uc790\uc728\uc131\uc744 \ubd80\uacfc\ud588\ub2e4\ub294 \uc0ac\uc2e4\uc774 \uc54c\ub824\uc84c\uae30 \ub54c\ubb38\uc774\uc5c8\ub2e4. \ud074\ub808\uba58\uc2a4 7\uc138\ub294 \uce74\ub97c 5\uc138\ub97c \uc5b8\uc9e2\uac8c \ud560 \ub9cc\ud55c \ud589\ub3d9\uc744 \ud558\uc9c0 \uc54a\uc74c\uc73c\ub85c\uc368 \uadf8\uc640\uc758 \uc5b4\ub5a0\ud55c \ucda9\ub3cc\ub3c4 \ud53c\ud558\ub824\uace0 \ub178\ub825\ud558\uc600\ub2e4. \uc5b4\ub5a0\ud55c \uc8fc\uc800\ud568\uacfc \uc870\uac74 \uc5c6\uc774 \ud074\ub808\uba58\uc2a4 7\uc138\ub294 \uc6b8\ud2b8\ub808\ud750\ud2b8 \uad50\uad6c\uc758 \uc138\uc18d \uc7ac\uc0b0\uc744 \ud569\uc2a4\ubd80\ub974\ud06c \uc655\uac00\uc5d0 \uc591\ub3c4\ud55c\ub2e4\ub294 \uac83\uc5d0 \ub3d9\uc758\ud558\uc600\ub2e4.", "sentence_answer": "\uc5b4\ub5a0\ud55c \uc8fc\uc800\ud568\uacfc \uc870\uac74 \uc5c6\uc774 \ud074\ub808\uba58\uc2a4 7\uc138\ub294 \uc6b8\ud2b8\ub808\ud750\ud2b8 \uad50\uad6c\uc758 \uc138\uc18d \uc7ac\uc0b0\uc744 \ud569\uc2a4\ubd80\ub974\ud06c \uc655\uac00\uc5d0 \uc591\ub3c4\ud55c\ub2e4\ub294 \uac83\uc5d0 \ub3d9\uc758\ud558\uc600\ub2e4.", "paragraph_id": "6462506-4-1", "paragraph_question": "question: \ud074\ub808\uba58\uc2a4 7\uc138\uac00 \uad50\uad6c \uc138\uc18d \uc7ac\uc0b0\uc744 \uc5b4\ub290 \uc655\uac00\uc5d0 \ub118\uacbc\ub294\uac00?, context: \uc2e0\uc131 \ub85c\ub9c8 \uc81c\uad6d\uc758 \ud669\uc81c \uce74\ub97c 5\uc138\ub294 \ub85c\ub9c8 \uc57d\ud0c8 \uc18c\uc2dd\uc744 \uc811\ud558\uace0 \ubb34\ucc99 \ub2f9\ud669\ud558\uace0 \ub2f9\uc7a5 \uc911\uc9c0\ud560 \uac83\uc744 \uc9c0\uc2dc\ud558\uc600\uc9c0\ub9cc, \uc790\uc2e0\uc5d0\uac8c \ub300\ud56d\ud588\ub358 \uad50\ud669 \ud074\ub808\uba58\uc2a4 7\uc138\uac00 \uad70\ub300\uc640 \ub3c4\uc2dc\ub97c \uc783\uace0 \uc0b0\ud0c4\uc824\ub85c \uc131\uc5d0 \uc0ac\uc2e4\uc0c1 \uc720\ud3d0\ub098 \ub2e4\ub984\uc5c6\ub294 \uc2e0\uc138\ub85c \uc804\ub77d\ud55c \uc0ac\uc2e4 \uc790\uccb4\uc5d0 \ub300\ud574\uc11c\ub294 \uadf8\ub2e4\uc9c0 \uc2eb\uc5b4\ud558\ub294 \ub0b4\uc0c9\uc744 \ubcf4\uc774\uc9c0 \uc54a\uc558\ub2e4. \uc2e4\uc81c\ub85c \uce74\ub97c 5\uc138\ub294 \ub85c\ub9c8 \uc57d\ud0c8\uc5d0 \ubd80\ubd84\uc801\uc73c\ub85c\ub294 \ucc45\uc784\uc774 \uc788\ub294\ub370, \uc774\ub294 \uadf8\uac00 \ud074\ub808\uba58\uc2a4 7\uc138\ub97c \uac1c\uc778\uc801\uc73c\ub85c \ub9cc\ub098 \uc790\uc2e0\uc758 \uc694\uad6c\uc0ac\ud56d\uc744 \uc804\ud558\uae30 \uc704\ud574\uc11c \uc790\uc2e0\uc758 \uad70\ub300\uc5d0\uac8c \uc790\uc728\uc131\uc744 \ubd80\uacfc\ud588\ub2e4\ub294 \uc0ac\uc2e4\uc774 \uc54c\ub824\uc84c\uae30 \ub54c\ubb38\uc774\uc5c8\ub2e4. \ud074\ub808\uba58\uc2a4 7\uc138\ub294 \uce74\ub97c 5\uc138\ub97c \uc5b8\uc9e2\uac8c \ud560 \ub9cc\ud55c \ud589\ub3d9\uc744 \ud558\uc9c0 \uc54a\uc74c\uc73c\ub85c\uc368 \uadf8\uc640\uc758 \uc5b4\ub5a0\ud55c \ucda9\ub3cc\ub3c4 \ud53c\ud558\ub824\uace0 \ub178\ub825\ud558\uc600\ub2e4. \uc5b4\ub5a0\ud55c \uc8fc\uc800\ud568\uacfc \uc870\uac74 \uc5c6\uc774 \ud074\ub808\uba58\uc2a4 7\uc138\ub294 \uc6b8\ud2b8\ub808\ud750\ud2b8 \uad50\uad6c\uc758 \uc138\uc18d \uc7ac\uc0b0\uc744 \ud569\uc2a4\ubd80\ub974\ud06c \uc655\uac00\uc5d0 \uc591\ub3c4\ud55c\ub2e4\ub294 \uac83\uc5d0 \ub3d9\uc758\ud558\uc600\ub2e4."} {"question": "\ud638\uc5d4\ucd10\ub808\ub978 \ud718\ud558\uc758 \ubcf4\ubcd1 \ubc29\uc9c4\uc5d0 \uacf5\uaca9\uc744 \uac00\ud558\uace0 \ub204\uad6c\uc758 \uae30\ubcd1 \ubd80\ub300\uc758 \uacf5\uaca9\uc5d0 \uc800\ud56d\ud558\uc600\ub098?", "paragraph": "\uc138 \uac1c\uc758 \uc624\uc2a4\ud2b8\ub9ac\uc544 \ubd80\ub300\ub294 \ub9c8\uc744\uc758 \ubc18\ub3c4 \uc810\uac70\ud558\uc9c0 \ubabb\ud588\ub2e4. \uc544\uc2a4\ud398\ub978\uc758 \uc8fc\uc694 \uac70\uc810 \uc694\uc0c8\ub294 \ub9c8\uc138\ub098\uac00 \ub048\uc9c8\uae30\uac8c \ubc29\uc5b4\ud558\uace0 \uc788\uc5c8\uace0, \ubc24\uc774 \ub418\uc5b4\uc11c\uc57c \ud568\ub77d\ub418\uc5c8\ub2e4. \uadf8\ub7ec\ub294 \ub3d9\uc548 \ub450 \ub9c8\uc744 \uc0ac\uc774\uc640 \ub2e4\ub9ac\uc758 \uc55e\ubd80\ubd84\uc5d0 \uc788\ub358 \ud504\ub791\uc2a4\uc758 \uc804\uad70\uc774 \uc804\uc7a5\uc5d0 \ud22c\uc785\ub418\uc5b4 \uc624\uc2a4\ud2b8\ub9ac\uc544\uad70\uc758 \uce21\uba74\uc744 \uacf5\uaca9\ud558\uc600\ub2e4. \ub098\ud3f4\ub808\uc639\uc740 \ubd80\ub300\ub97c \ub098\ub204\uc5b4 \uc77c\ubd80 \ubcf4\ubcd1\ub300\ub97c \uc911\uc559\uc758 \uc804\uba74\uc73c\ub85c \uc774\ub3d9\uc2dc\ucf30\uace0, \uae30\ubcd1\ub9cc\uc73c\ub85c \uc774\ub8e8\uc5b4\uc9c4 \ubd80\ub300\uc5d0\uac8c \uba85\ub839\uc744 \ub0b4\ub824 \uae34 \uc5f4\uc744 \uc774\ub8e8\uc5b4 \uc544\uc2a4\ud398\ub978\uc744 \uacf5\uaca9\ud558\ub358 \uc624\uc2a4\ud2b8\ub9ac\uc544 \ud3ec\ubcd1\ub300\uc5d0\uac8c \uacf5\uaca9\ud560 \uac83\uc744 \uba85\ud588\ub2e4. \ud504\ub791\uc2a4\uad70\uc758 1\ucc28 \ub3cc\uaca9\uc740 \uc624\uc2a4\ud2b8\ub9ac\uc544\uad70\uc5d0\uac8c \uaca9\ud1f4 \ub2f9\ud588\uc73c\ub098, \ub450 \ubc88\uc9f8 \ud749\uac11\uae30\ubcd1(cuirassires)\uc758 \uacf5\uaca9\uc740 \uc131\uacf5\ud558\uc600\ub2e4. \ud504\ub791\uc2a4 \uae30\ubcd1\ub300\ub294 \uc624\uc2a4\ud2b8\ub9ac\uc544\uad70\uc758 \ud3ec\ub97c \ud0c8\ucde8\ud558\uace0 \uc5ec\uc138\ub97c \ubab0\uc544 \ud638\uc5d4\ucd10\ub808\ub978 \ud718\ud558\uc758 \ubcf4\ubcd1 \ubc29\uc9c4\uc5d0 \uacf5\uaca9\uc744 \uac00\ud558\uace0 \ub9ac\ud788\ud150\uc288\ud0c0\uc778(Lichtenstein)\uc758 \uae30\ubcd1 \ubd80\ub300\uc758 \uacf5\uaca9\uc5d0 \uc800\ud56d\ud558\uc600\ub2e4. \uadf8\ub7ec\ub098 \uc774\ub4e4\uc740 \uc774 \uc774\uc0c1\uc758 \ud589\ub3d9\uc744 \ud560 \uc218\ub294 \uc5c6\uc5c8\uae30\uc5d0 \uc885\uad6d\uc5d0\ub294 \ud1f4\uac01\ud558\uc5ec \uc6d0\ub798 \uc704\uce58\ub85c \ub3cc\uc544\uac14\ub2e4.", "answer": "\ub9ac\ud788\ud150\uc288\ud0c0\uc778", "sentence": "\ud504\ub791\uc2a4 \uae30\ubcd1\ub300\ub294 \uc624\uc2a4\ud2b8\ub9ac\uc544\uad70\uc758 \ud3ec\ub97c \ud0c8\ucde8\ud558\uace0 \uc5ec\uc138\ub97c \ubab0\uc544 \ud638\uc5d4\ucd10\ub808\ub978 \ud718\ud558\uc758 \ubcf4\ubcd1 \ubc29\uc9c4\uc5d0 \uacf5\uaca9\uc744 \uac00\ud558\uace0 \ub9ac\ud788\ud150\uc288\ud0c0\uc778 (Lichtenstein)", "paragraph_sentence": "\uc138 \uac1c\uc758 \uc624\uc2a4\ud2b8\ub9ac\uc544 \ubd80\ub300\ub294 \ub9c8\uc744\uc758 \ubc18\ub3c4 \uc810\uac70\ud558\uc9c0 \ubabb\ud588\ub2e4. \uc544\uc2a4\ud398\ub978\uc758 \uc8fc\uc694 \uac70\uc810 \uc694\uc0c8\ub294 \ub9c8\uc138\ub098\uac00 \ub048\uc9c8\uae30\uac8c \ubc29\uc5b4\ud558\uace0 \uc788\uc5c8\uace0, \ubc24\uc774 \ub418\uc5b4\uc11c\uc57c \ud568\ub77d\ub418\uc5c8\ub2e4. \uadf8\ub7ec\ub294 \ub3d9\uc548 \ub450 \ub9c8\uc744 \uc0ac\uc774\uc640 \ub2e4\ub9ac\uc758 \uc55e\ubd80\ubd84\uc5d0 \uc788\ub358 \ud504\ub791\uc2a4\uc758 \uc804\uad70\uc774 \uc804\uc7a5\uc5d0 \ud22c\uc785\ub418\uc5b4 \uc624\uc2a4\ud2b8\ub9ac\uc544\uad70\uc758 \uce21\uba74\uc744 \uacf5\uaca9\ud558\uc600\ub2e4. \ub098\ud3f4\ub808\uc639\uc740 \ubd80\ub300\ub97c \ub098\ub204\uc5b4 \uc77c\ubd80 \ubcf4\ubcd1\ub300\ub97c \uc911\uc559\uc758 \uc804\uba74\uc73c\ub85c \uc774\ub3d9\uc2dc\ucf30\uace0, \uae30\ubcd1\ub9cc\uc73c\ub85c \uc774\ub8e8\uc5b4\uc9c4 \ubd80\ub300\uc5d0\uac8c \uba85\ub839\uc744 \ub0b4\ub824 \uae34 \uc5f4\uc744 \uc774\ub8e8\uc5b4 \uc544\uc2a4\ud398\ub978\uc744 \uacf5\uaca9\ud558\ub358 \uc624\uc2a4\ud2b8\ub9ac\uc544 \ud3ec\ubcd1\ub300\uc5d0\uac8c \uacf5\uaca9\ud560 \uac83\uc744 \uba85\ud588\ub2e4. \ud504\ub791\uc2a4\uad70\uc758 1\ucc28 \ub3cc\uaca9\uc740 \uc624\uc2a4\ud2b8\ub9ac\uc544\uad70\uc5d0\uac8c \uaca9\ud1f4 \ub2f9\ud588\uc73c\ub098, \ub450 \ubc88\uc9f8 \ud749\uac11\uae30\ubcd1(cuirassires)\uc758 \uacf5\uaca9\uc740 \uc131\uacf5\ud558\uc600\ub2e4. \ud504\ub791\uc2a4 \uae30\ubcd1\ub300\ub294 \uc624\uc2a4\ud2b8\ub9ac\uc544\uad70\uc758 \ud3ec\ub97c \ud0c8\ucde8\ud558\uace0 \uc5ec\uc138\ub97c \ubab0\uc544 \ud638\uc5d4\ucd10\ub808\ub978 \ud718\ud558\uc758 \ubcf4\ubcd1 \ubc29\uc9c4\uc5d0 \uacf5\uaca9\uc744 \uac00\ud558\uace0 \ub9ac\ud788\ud150\uc288\ud0c0\uc778 (Lichtenstein) \uc758 \uae30\ubcd1 \ubd80\ub300\uc758 \uacf5\uaca9\uc5d0 \uc800\ud56d\ud558\uc600\ub2e4. \uadf8\ub7ec\ub098 \uc774\ub4e4\uc740 \uc774 \uc774\uc0c1\uc758 \ud589\ub3d9\uc744 \ud560 \uc218\ub294 \uc5c6\uc5c8\uae30\uc5d0 \uc885\uad6d\uc5d0\ub294 \ud1f4\uac01\ud558\uc5ec \uc6d0\ub798 \uc704\uce58\ub85c \ub3cc\uc544\uac14\ub2e4.", "paragraph_answer": "\uc138 \uac1c\uc758 \uc624\uc2a4\ud2b8\ub9ac\uc544 \ubd80\ub300\ub294 \ub9c8\uc744\uc758 \ubc18\ub3c4 \uc810\uac70\ud558\uc9c0 \ubabb\ud588\ub2e4. \uc544\uc2a4\ud398\ub978\uc758 \uc8fc\uc694 \uac70\uc810 \uc694\uc0c8\ub294 \ub9c8\uc138\ub098\uac00 \ub048\uc9c8\uae30\uac8c \ubc29\uc5b4\ud558\uace0 \uc788\uc5c8\uace0, \ubc24\uc774 \ub418\uc5b4\uc11c\uc57c \ud568\ub77d\ub418\uc5c8\ub2e4. \uadf8\ub7ec\ub294 \ub3d9\uc548 \ub450 \ub9c8\uc744 \uc0ac\uc774\uc640 \ub2e4\ub9ac\uc758 \uc55e\ubd80\ubd84\uc5d0 \uc788\ub358 \ud504\ub791\uc2a4\uc758 \uc804\uad70\uc774 \uc804\uc7a5\uc5d0 \ud22c\uc785\ub418\uc5b4 \uc624\uc2a4\ud2b8\ub9ac\uc544\uad70\uc758 \uce21\uba74\uc744 \uacf5\uaca9\ud558\uc600\ub2e4. \ub098\ud3f4\ub808\uc639\uc740 \ubd80\ub300\ub97c \ub098\ub204\uc5b4 \uc77c\ubd80 \ubcf4\ubcd1\ub300\ub97c \uc911\uc559\uc758 \uc804\uba74\uc73c\ub85c \uc774\ub3d9\uc2dc\ucf30\uace0, \uae30\ubcd1\ub9cc\uc73c\ub85c \uc774\ub8e8\uc5b4\uc9c4 \ubd80\ub300\uc5d0\uac8c \uba85\ub839\uc744 \ub0b4\ub824 \uae34 \uc5f4\uc744 \uc774\ub8e8\uc5b4 \uc544\uc2a4\ud398\ub978\uc744 \uacf5\uaca9\ud558\ub358 \uc624\uc2a4\ud2b8\ub9ac\uc544 \ud3ec\ubcd1\ub300\uc5d0\uac8c \uacf5\uaca9\ud560 \uac83\uc744 \uba85\ud588\ub2e4. \ud504\ub791\uc2a4\uad70\uc758 1\ucc28 \ub3cc\uaca9\uc740 \uc624\uc2a4\ud2b8\ub9ac\uc544\uad70\uc5d0\uac8c \uaca9\ud1f4 \ub2f9\ud588\uc73c\ub098, \ub450 \ubc88\uc9f8 \ud749\uac11\uae30\ubcd1(cuirassires)\uc758 \uacf5\uaca9\uc740 \uc131\uacf5\ud558\uc600\ub2e4. \ud504\ub791\uc2a4 \uae30\ubcd1\ub300\ub294 \uc624\uc2a4\ud2b8\ub9ac\uc544\uad70\uc758 \ud3ec\ub97c \ud0c8\ucde8\ud558\uace0 \uc5ec\uc138\ub97c \ubab0\uc544 \ud638\uc5d4\ucd10\ub808\ub978 \ud718\ud558\uc758 \ubcf4\ubcd1 \ubc29\uc9c4\uc5d0 \uacf5\uaca9\uc744 \uac00\ud558\uace0 \ub9ac\ud788\ud150\uc288\ud0c0\uc778 (Lichtenstein)\uc758 \uae30\ubcd1 \ubd80\ub300\uc758 \uacf5\uaca9\uc5d0 \uc800\ud56d\ud558\uc600\ub2e4. \uadf8\ub7ec\ub098 \uc774\ub4e4\uc740 \uc774 \uc774\uc0c1\uc758 \ud589\ub3d9\uc744 \ud560 \uc218\ub294 \uc5c6\uc5c8\uae30\uc5d0 \uc885\uad6d\uc5d0\ub294 \ud1f4\uac01\ud558\uc5ec \uc6d0\ub798 \uc704\uce58\ub85c \ub3cc\uc544\uac14\ub2e4.", "sentence_answer": "\ud504\ub791\uc2a4 \uae30\ubcd1\ub300\ub294 \uc624\uc2a4\ud2b8\ub9ac\uc544\uad70\uc758 \ud3ec\ub97c \ud0c8\ucde8\ud558\uace0 \uc5ec\uc138\ub97c \ubab0\uc544 \ud638\uc5d4\ucd10\ub808\ub978 \ud718\ud558\uc758 \ubcf4\ubcd1 \ubc29\uc9c4\uc5d0 \uacf5\uaca9\uc744 \uac00\ud558\uace0 \ub9ac\ud788\ud150\uc288\ud0c0\uc778 (Lichtenstein)", "paragraph_id": "6585190-3-1", "paragraph_question": "question: \ud638\uc5d4\ucd10\ub808\ub978 \ud718\ud558\uc758 \ubcf4\ubcd1 \ubc29\uc9c4\uc5d0 \uacf5\uaca9\uc744 \uac00\ud558\uace0 \ub204\uad6c\uc758 \uae30\ubcd1 \ubd80\ub300\uc758 \uacf5\uaca9\uc5d0 \uc800\ud56d\ud558\uc600\ub098?, context: \uc138 \uac1c\uc758 \uc624\uc2a4\ud2b8\ub9ac\uc544 \ubd80\ub300\ub294 \ub9c8\uc744\uc758 \ubc18\ub3c4 \uc810\uac70\ud558\uc9c0 \ubabb\ud588\ub2e4. \uc544\uc2a4\ud398\ub978\uc758 \uc8fc\uc694 \uac70\uc810 \uc694\uc0c8\ub294 \ub9c8\uc138\ub098\uac00 \ub048\uc9c8\uae30\uac8c \ubc29\uc5b4\ud558\uace0 \uc788\uc5c8\uace0, \ubc24\uc774 \ub418\uc5b4\uc11c\uc57c \ud568\ub77d\ub418\uc5c8\ub2e4. \uadf8\ub7ec\ub294 \ub3d9\uc548 \ub450 \ub9c8\uc744 \uc0ac\uc774\uc640 \ub2e4\ub9ac\uc758 \uc55e\ubd80\ubd84\uc5d0 \uc788\ub358 \ud504\ub791\uc2a4\uc758 \uc804\uad70\uc774 \uc804\uc7a5\uc5d0 \ud22c\uc785\ub418\uc5b4 \uc624\uc2a4\ud2b8\ub9ac\uc544\uad70\uc758 \uce21\uba74\uc744 \uacf5\uaca9\ud558\uc600\ub2e4. \ub098\ud3f4\ub808\uc639\uc740 \ubd80\ub300\ub97c \ub098\ub204\uc5b4 \uc77c\ubd80 \ubcf4\ubcd1\ub300\ub97c \uc911\uc559\uc758 \uc804\uba74\uc73c\ub85c \uc774\ub3d9\uc2dc\ucf30\uace0, \uae30\ubcd1\ub9cc\uc73c\ub85c \uc774\ub8e8\uc5b4\uc9c4 \ubd80\ub300\uc5d0\uac8c \uba85\ub839\uc744 \ub0b4\ub824 \uae34 \uc5f4\uc744 \uc774\ub8e8\uc5b4 \uc544\uc2a4\ud398\ub978\uc744 \uacf5\uaca9\ud558\ub358 \uc624\uc2a4\ud2b8\ub9ac\uc544 \ud3ec\ubcd1\ub300\uc5d0\uac8c \uacf5\uaca9\ud560 \uac83\uc744 \uba85\ud588\ub2e4. \ud504\ub791\uc2a4\uad70\uc758 1\ucc28 \ub3cc\uaca9\uc740 \uc624\uc2a4\ud2b8\ub9ac\uc544\uad70\uc5d0\uac8c \uaca9\ud1f4 \ub2f9\ud588\uc73c\ub098, \ub450 \ubc88\uc9f8 \ud749\uac11\uae30\ubcd1(cuirassires)\uc758 \uacf5\uaca9\uc740 \uc131\uacf5\ud558\uc600\ub2e4. \ud504\ub791\uc2a4 \uae30\ubcd1\ub300\ub294 \uc624\uc2a4\ud2b8\ub9ac\uc544\uad70\uc758 \ud3ec\ub97c \ud0c8\ucde8\ud558\uace0 \uc5ec\uc138\ub97c \ubab0\uc544 \ud638\uc5d4\ucd10\ub808\ub978 \ud718\ud558\uc758 \ubcf4\ubcd1 \ubc29\uc9c4\uc5d0 \uacf5\uaca9\uc744 \uac00\ud558\uace0 \ub9ac\ud788\ud150\uc288\ud0c0\uc778(Lichtenstein)\uc758 \uae30\ubcd1 \ubd80\ub300\uc758 \uacf5\uaca9\uc5d0 \uc800\ud56d\ud558\uc600\ub2e4. \uadf8\ub7ec\ub098 \uc774\ub4e4\uc740 \uc774 \uc774\uc0c1\uc758 \ud589\ub3d9\uc744 \ud560 \uc218\ub294 \uc5c6\uc5c8\uae30\uc5d0 \uc885\uad6d\uc5d0\ub294 \ud1f4\uac01\ud558\uc5ec \uc6d0\ub798 \uc704\uce58\ub85c \ub3cc\uc544\uac14\ub2e4."} {"question": "\ud45c\ud2b8\ub85c \ub808\uc624\ub2c8\ub3c4\ube44\uce58 \uce74\ud53c\ucc28\uc5d0\uac8c \uc2e0\ubcc0 \ubcf4\uc7a5 \uac01\uc11c\ub97c \uc368\uc8fc\uaca0\ub2e4\uace0 \ud588\uc9c0\ub9cc \uc57d\uc18d\uc744 \uc9c0\ud0a4\uc9c0 \uc54a\uc740 \uc778\ubb3c\uc740?", "paragraph": "\uc18c\ube44\uc5d0\ud2b8 \uc5f0\ubc29\uc758 \ubb3c\ub9ac\ud559\uc790\ub85c \ud45c\ud2b8\ub974 \ub808\uc624\ub2c8\ub3c4\ube44\uce58 \uce74\ud53c\ucc28\uac00 \uc788\ub2e4. \uadf8\ub294 1918\ub144\uc5d0 \uc0c1\ud2b8\ud398\ud14c\ub974\ubd80\ub974\ud06c \uacf5\uacfc \ub300\ud559\uad50\ub97c \uc878\uc5c5\ud558\uace0 1930\ub144\ub300\uc5d0 \ub300\uc601\uc81c\uad6d \ucf00\uc784\ube0c\ub9ac\uc9c0 \ub300\ud559\uad50 \ub4f1\uc5d0\uc11c \ud65c\uc57d\ud558\uba70 \uba85\uc131\uc744 \ub5a8\ucce4\ub2e4. \uc790\uc5f0\uc2a4\ub7fd\uac8c \ub2f9\uc2dc \uc18c\ub828 \uacf5\uc0b0\ub2f9 \uc11c\uae30\uc7a5\uc774\uc5c8\ub358 \uc774\uc624\uc2dc\ud504 \uc2a4\ud0c8\ub9b0\uc740 \uc6b0\uc218\ud55c \uc778\uc7ac\uc778 \uadf8\uac00 \ubaa8\uad6d\uc5d0\uc11c \uc5f0\uad6c\ud558\uae30\ub97c \ubc14\ub790\ub2e4. \ud558\uc9c0\ub9cc \uc18c\ub828\uc5d0 \uc5b5\ub958\ub420 \uac83\uc744 \uc5fc\ub824\ud55c \uce74\ud53c\ucc28\ub294 \uc2e0\ubcc0 \ubcf4\uc7a5\uc774 \ub418\uc9c0 \uc54a\uc73c\uba74 \ub3cc\uc544\uac00\uc9c0 \uc54a\uaca0\ub2e4\uba70 \uac70\uc808\ud588\ub294\ub370 \uc2a4\ud0c8\ub9b0\uc740 \uc774\uc5d0 \uc9c1\uc811 \uc2e0\ubcc0 \ubcf4\uc7a5 \uac01\uc11c\ub97c \uc368\uc8fc\uba74\uc11c\uae4c\uc9c0 \ubaa8\uad6d \ubc29\ubb38\uc744 \uc885\uc6a9\ud588\ub2e4. \uadf8\ub807\uac8c \ud558\uc5ec \uce74\ud53c\ucc28\ub294 \ub9e4\ub144 \uc5ec\ub984 \uc2a4\ud0c8\ub9b0\uc758 \uac01\uc11c\ub97c \ubc1b\uace0\uc11c \uc18c\ub828\uc744 \ubc29\ubb38\ud558\uac8c \ub410\ub294\ub370 \uc5b4\ub290 \ud574\uc5d0\ub294 \uce74\ud53c\ucc28\uc758 \uc5ec\ub984 \ubc29\ubb38 \uc2dc\uae30\uac00 \ub2e4\uac00\uc624\ub294\ub370\ub3c4 \ubd88\uad6c\ud558\uace0 \uc2a4\ud0c8\ub9b0\uc758 \uc2e0\ubcc0 \ubcf4\uc7a5 \uac01\uc11c\uac00 \ub3c4\ucc29\ud558\uc9c0 \uc54a\uc558\ub2e4. \ud558\uc9c0\ub9cc \ub300\uc0ac\uad00 \uad00\ub9ac\ub294 \ud589\uc815 \ucc98\ub9ac\uac00 \ub2a6\uc5b4\uc9c0\uace0 \uc788\ub294 \uac83\uc77c \ubfd0\uc774\ub2c8 \uac71\uc815 \ub9d0\uace0 \uc6b0\uc120 \uc18c\ub828\uc73c\ub85c \ucd9c\ubc1c\ud558\ub77c\uace0 \uc7ac\ucd09\ud558\uc600\ub2e4. \uc774\ubbf8 \uc5ec\ub7ec \ubc88 \uc18c\ub828\uc744 \uc655\ub798\ud55c \ubc14 \uc788\ub294 \uce74\ud53c\ucc28\ub294 \ubcc4 \uc758\uc2ec \uc5c6\uc774 \uadc0\uad6d\uae38\uc5d0 \uc62c\ub790\uc9c0\ub9cc \uc544\ub2c8\ub098 \ub2e4\ub97c\uae4c \uadf8 \uae38\ub85c \uacb0\uad6d \uc18c\ub828\uc5d0 \uc5b5\ub958\ub418\uace0 \ub9d0\uc558\uace0 \uaf3c\uc9dd\uc5c6\uc774 \ud3c9\uc0dd\uc744 \uadf8\uacf3\uc5d0\uc11c \uc9c0\ub0b4\uba74\uc11c \uc5f0\uad6c\ud560 \uc218\ubc16\uc5d0 \uc5c6\uac8c \ub410\ub2e4.", "answer": "\uc2a4\ud0c8\ub9b0", "sentence": "\uc790\uc5f0\uc2a4\ub7fd\uac8c \ub2f9\uc2dc \uc18c\ub828 \uacf5\uc0b0\ub2f9 \uc11c\uae30\uc7a5\uc774\uc5c8\ub358 \uc774\uc624\uc2dc\ud504 \uc2a4\ud0c8\ub9b0 \uc740 \uc6b0\uc218\ud55c \uc778\uc7ac\uc778 \uadf8\uac00 \ubaa8\uad6d\uc5d0\uc11c \uc5f0\uad6c\ud558\uae30\ub97c \ubc14\ub790\ub2e4.", "paragraph_sentence": "\uc18c\ube44\uc5d0\ud2b8 \uc5f0\ubc29\uc758 \ubb3c\ub9ac\ud559\uc790\ub85c \ud45c\ud2b8\ub974 \ub808\uc624\ub2c8\ub3c4\ube44\uce58 \uce74\ud53c\ucc28\uac00 \uc788\ub2e4. \uadf8\ub294 1918\ub144\uc5d0 \uc0c1\ud2b8\ud398\ud14c\ub974\ubd80\ub974\ud06c \uacf5\uacfc \ub300\ud559\uad50\ub97c \uc878\uc5c5\ud558\uace0 1930\ub144\ub300\uc5d0 \ub300\uc601\uc81c\uad6d \ucf00\uc784\ube0c\ub9ac\uc9c0 \ub300\ud559\uad50 \ub4f1\uc5d0\uc11c \ud65c\uc57d\ud558\uba70 \uba85\uc131\uc744 \ub5a8\ucce4\ub2e4. \uc790\uc5f0\uc2a4\ub7fd\uac8c \ub2f9\uc2dc \uc18c\ub828 \uacf5\uc0b0\ub2f9 \uc11c\uae30\uc7a5\uc774\uc5c8\ub358 \uc774\uc624\uc2dc\ud504 \uc2a4\ud0c8\ub9b0 \uc740 \uc6b0\uc218\ud55c \uc778\uc7ac\uc778 \uadf8\uac00 \ubaa8\uad6d\uc5d0\uc11c \uc5f0\uad6c\ud558\uae30\ub97c \ubc14\ub790\ub2e4. \ud558\uc9c0\ub9cc \uc18c\ub828\uc5d0 \uc5b5\ub958\ub420 \uac83\uc744 \uc5fc\ub824\ud55c \uce74\ud53c\ucc28\ub294 \uc2e0\ubcc0 \ubcf4\uc7a5\uc774 \ub418\uc9c0 \uc54a\uc73c\uba74 \ub3cc\uc544\uac00\uc9c0 \uc54a\uaca0\ub2e4\uba70 \uac70\uc808\ud588\ub294\ub370 \uc2a4\ud0c8\ub9b0\uc740 \uc774\uc5d0 \uc9c1\uc811 \uc2e0\ubcc0 \ubcf4\uc7a5 \uac01\uc11c\ub97c \uc368\uc8fc\uba74\uc11c\uae4c\uc9c0 \ubaa8\uad6d \ubc29\ubb38\uc744 \uc885\uc6a9\ud588\ub2e4. \uadf8\ub807\uac8c \ud558\uc5ec \uce74\ud53c\ucc28\ub294 \ub9e4\ub144 \uc5ec\ub984 \uc2a4\ud0c8\ub9b0\uc758 \uac01\uc11c\ub97c \ubc1b\uace0\uc11c \uc18c\ub828\uc744 \ubc29\ubb38\ud558\uac8c \ub410\ub294\ub370 \uc5b4\ub290 \ud574\uc5d0\ub294 \uce74\ud53c\ucc28\uc758 \uc5ec\ub984 \ubc29\ubb38 \uc2dc\uae30\uac00 \ub2e4\uac00\uc624\ub294\ub370\ub3c4 \ubd88\uad6c\ud558\uace0 \uc2a4\ud0c8\ub9b0\uc758 \uc2e0\ubcc0 \ubcf4\uc7a5 \uac01\uc11c\uac00 \ub3c4\ucc29\ud558\uc9c0 \uc54a\uc558\ub2e4. \ud558\uc9c0\ub9cc \ub300\uc0ac\uad00 \uad00\ub9ac\ub294 \ud589\uc815 \ucc98\ub9ac\uac00 \ub2a6\uc5b4\uc9c0\uace0 \uc788\ub294 \uac83\uc77c \ubfd0\uc774\ub2c8 \uac71\uc815 \ub9d0\uace0 \uc6b0\uc120 \uc18c\ub828\uc73c\ub85c \ucd9c\ubc1c\ud558\ub77c\uace0 \uc7ac\ucd09\ud558\uc600\ub2e4. \uc774\ubbf8 \uc5ec\ub7ec \ubc88 \uc18c\ub828\uc744 \uc655\ub798\ud55c \ubc14 \uc788\ub294 \uce74\ud53c\ucc28\ub294 \ubcc4 \uc758\uc2ec \uc5c6\uc774 \uadc0\uad6d\uae38\uc5d0 \uc62c\ub790\uc9c0\ub9cc \uc544\ub2c8\ub098 \ub2e4\ub97c\uae4c \uadf8 \uae38\ub85c \uacb0\uad6d \uc18c\ub828\uc5d0 \uc5b5\ub958\ub418\uace0 \ub9d0\uc558\uace0 \uaf3c\uc9dd\uc5c6\uc774 \ud3c9\uc0dd\uc744 \uadf8\uacf3\uc5d0\uc11c \uc9c0\ub0b4\uba74\uc11c \uc5f0\uad6c\ud560 \uc218\ubc16\uc5d0 \uc5c6\uac8c \ub410\ub2e4.", "paragraph_answer": "\uc18c\ube44\uc5d0\ud2b8 \uc5f0\ubc29\uc758 \ubb3c\ub9ac\ud559\uc790\ub85c \ud45c\ud2b8\ub974 \ub808\uc624\ub2c8\ub3c4\ube44\uce58 \uce74\ud53c\ucc28\uac00 \uc788\ub2e4. \uadf8\ub294 1918\ub144\uc5d0 \uc0c1\ud2b8\ud398\ud14c\ub974\ubd80\ub974\ud06c \uacf5\uacfc \ub300\ud559\uad50\ub97c \uc878\uc5c5\ud558\uace0 1930\ub144\ub300\uc5d0 \ub300\uc601\uc81c\uad6d \ucf00\uc784\ube0c\ub9ac\uc9c0 \ub300\ud559\uad50 \ub4f1\uc5d0\uc11c \ud65c\uc57d\ud558\uba70 \uba85\uc131\uc744 \ub5a8\ucce4\ub2e4. \uc790\uc5f0\uc2a4\ub7fd\uac8c \ub2f9\uc2dc \uc18c\ub828 \uacf5\uc0b0\ub2f9 \uc11c\uae30\uc7a5\uc774\uc5c8\ub358 \uc774\uc624\uc2dc\ud504 \uc2a4\ud0c8\ub9b0 \uc740 \uc6b0\uc218\ud55c \uc778\uc7ac\uc778 \uadf8\uac00 \ubaa8\uad6d\uc5d0\uc11c \uc5f0\uad6c\ud558\uae30\ub97c \ubc14\ub790\ub2e4. \ud558\uc9c0\ub9cc \uc18c\ub828\uc5d0 \uc5b5\ub958\ub420 \uac83\uc744 \uc5fc\ub824\ud55c \uce74\ud53c\ucc28\ub294 \uc2e0\ubcc0 \ubcf4\uc7a5\uc774 \ub418\uc9c0 \uc54a\uc73c\uba74 \ub3cc\uc544\uac00\uc9c0 \uc54a\uaca0\ub2e4\uba70 \uac70\uc808\ud588\ub294\ub370 \uc2a4\ud0c8\ub9b0\uc740 \uc774\uc5d0 \uc9c1\uc811 \uc2e0\ubcc0 \ubcf4\uc7a5 \uac01\uc11c\ub97c \uc368\uc8fc\uba74\uc11c\uae4c\uc9c0 \ubaa8\uad6d \ubc29\ubb38\uc744 \uc885\uc6a9\ud588\ub2e4. \uadf8\ub807\uac8c \ud558\uc5ec \uce74\ud53c\ucc28\ub294 \ub9e4\ub144 \uc5ec\ub984 \uc2a4\ud0c8\ub9b0\uc758 \uac01\uc11c\ub97c \ubc1b\uace0\uc11c \uc18c\ub828\uc744 \ubc29\ubb38\ud558\uac8c \ub410\ub294\ub370 \uc5b4\ub290 \ud574\uc5d0\ub294 \uce74\ud53c\ucc28\uc758 \uc5ec\ub984 \ubc29\ubb38 \uc2dc\uae30\uac00 \ub2e4\uac00\uc624\ub294\ub370\ub3c4 \ubd88\uad6c\ud558\uace0 \uc2a4\ud0c8\ub9b0\uc758 \uc2e0\ubcc0 \ubcf4\uc7a5 \uac01\uc11c\uac00 \ub3c4\ucc29\ud558\uc9c0 \uc54a\uc558\ub2e4. \ud558\uc9c0\ub9cc \ub300\uc0ac\uad00 \uad00\ub9ac\ub294 \ud589\uc815 \ucc98\ub9ac\uac00 \ub2a6\uc5b4\uc9c0\uace0 \uc788\ub294 \uac83\uc77c \ubfd0\uc774\ub2c8 \uac71\uc815 \ub9d0\uace0 \uc6b0\uc120 \uc18c\ub828\uc73c\ub85c \ucd9c\ubc1c\ud558\ub77c\uace0 \uc7ac\ucd09\ud558\uc600\ub2e4. \uc774\ubbf8 \uc5ec\ub7ec \ubc88 \uc18c\ub828\uc744 \uc655\ub798\ud55c \ubc14 \uc788\ub294 \uce74\ud53c\ucc28\ub294 \ubcc4 \uc758\uc2ec \uc5c6\uc774 \uadc0\uad6d\uae38\uc5d0 \uc62c\ub790\uc9c0\ub9cc \uc544\ub2c8\ub098 \ub2e4\ub97c\uae4c \uadf8 \uae38\ub85c \uacb0\uad6d \uc18c\ub828\uc5d0 \uc5b5\ub958\ub418\uace0 \ub9d0\uc558\uace0 \uaf3c\uc9dd\uc5c6\uc774 \ud3c9\uc0dd\uc744 \uadf8\uacf3\uc5d0\uc11c \uc9c0\ub0b4\uba74\uc11c \uc5f0\uad6c\ud560 \uc218\ubc16\uc5d0 \uc5c6\uac8c \ub410\ub2e4.", "sentence_answer": "\uc790\uc5f0\uc2a4\ub7fd\uac8c \ub2f9\uc2dc \uc18c\ub828 \uacf5\uc0b0\ub2f9 \uc11c\uae30\uc7a5\uc774\uc5c8\ub358 \uc774\uc624\uc2dc\ud504 \uc2a4\ud0c8\ub9b0 \uc740 \uc6b0\uc218\ud55c \uc778\uc7ac\uc778 \uadf8\uac00 \ubaa8\uad6d\uc5d0\uc11c \uc5f0\uad6c\ud558\uae30\ub97c \ubc14\ub790\ub2e4.", "paragraph_id": "6329268-12-2", "paragraph_question": "question: \ud45c\ud2b8\ub85c \ub808\uc624\ub2c8\ub3c4\ube44\uce58 \uce74\ud53c\ucc28\uc5d0\uac8c \uc2e0\ubcc0 \ubcf4\uc7a5 \uac01\uc11c\ub97c \uc368\uc8fc\uaca0\ub2e4\uace0 \ud588\uc9c0\ub9cc \uc57d\uc18d\uc744 \uc9c0\ud0a4\uc9c0 \uc54a\uc740 \uc778\ubb3c\uc740?, context: \uc18c\ube44\uc5d0\ud2b8 \uc5f0\ubc29\uc758 \ubb3c\ub9ac\ud559\uc790\ub85c \ud45c\ud2b8\ub974 \ub808\uc624\ub2c8\ub3c4\ube44\uce58 \uce74\ud53c\ucc28\uac00 \uc788\ub2e4. \uadf8\ub294 1918\ub144\uc5d0 \uc0c1\ud2b8\ud398\ud14c\ub974\ubd80\ub974\ud06c \uacf5\uacfc \ub300\ud559\uad50\ub97c \uc878\uc5c5\ud558\uace0 1930\ub144\ub300\uc5d0 \ub300\uc601\uc81c\uad6d \ucf00\uc784\ube0c\ub9ac\uc9c0 \ub300\ud559\uad50 \ub4f1\uc5d0\uc11c \ud65c\uc57d\ud558\uba70 \uba85\uc131\uc744 \ub5a8\ucce4\ub2e4. \uc790\uc5f0\uc2a4\ub7fd\uac8c \ub2f9\uc2dc \uc18c\ub828 \uacf5\uc0b0\ub2f9 \uc11c\uae30\uc7a5\uc774\uc5c8\ub358 \uc774\uc624\uc2dc\ud504 \uc2a4\ud0c8\ub9b0\uc740 \uc6b0\uc218\ud55c \uc778\uc7ac\uc778 \uadf8\uac00 \ubaa8\uad6d\uc5d0\uc11c \uc5f0\uad6c\ud558\uae30\ub97c \ubc14\ub790\ub2e4. \ud558\uc9c0\ub9cc \uc18c\ub828\uc5d0 \uc5b5\ub958\ub420 \uac83\uc744 \uc5fc\ub824\ud55c \uce74\ud53c\ucc28\ub294 \uc2e0\ubcc0 \ubcf4\uc7a5\uc774 \ub418\uc9c0 \uc54a\uc73c\uba74 \ub3cc\uc544\uac00\uc9c0 \uc54a\uaca0\ub2e4\uba70 \uac70\uc808\ud588\ub294\ub370 \uc2a4\ud0c8\ub9b0\uc740 \uc774\uc5d0 \uc9c1\uc811 \uc2e0\ubcc0 \ubcf4\uc7a5 \uac01\uc11c\ub97c \uc368\uc8fc\uba74\uc11c\uae4c\uc9c0 \ubaa8\uad6d \ubc29\ubb38\uc744 \uc885\uc6a9\ud588\ub2e4. \uadf8\ub807\uac8c \ud558\uc5ec \uce74\ud53c\ucc28\ub294 \ub9e4\ub144 \uc5ec\ub984 \uc2a4\ud0c8\ub9b0\uc758 \uac01\uc11c\ub97c \ubc1b\uace0\uc11c \uc18c\ub828\uc744 \ubc29\ubb38\ud558\uac8c \ub410\ub294\ub370 \uc5b4\ub290 \ud574\uc5d0\ub294 \uce74\ud53c\ucc28\uc758 \uc5ec\ub984 \ubc29\ubb38 \uc2dc\uae30\uac00 \ub2e4\uac00\uc624\ub294\ub370\ub3c4 \ubd88\uad6c\ud558\uace0 \uc2a4\ud0c8\ub9b0\uc758 \uc2e0\ubcc0 \ubcf4\uc7a5 \uac01\uc11c\uac00 \ub3c4\ucc29\ud558\uc9c0 \uc54a\uc558\ub2e4. \ud558\uc9c0\ub9cc \ub300\uc0ac\uad00 \uad00\ub9ac\ub294 \ud589\uc815 \ucc98\ub9ac\uac00 \ub2a6\uc5b4\uc9c0\uace0 \uc788\ub294 \uac83\uc77c \ubfd0\uc774\ub2c8 \uac71\uc815 \ub9d0\uace0 \uc6b0\uc120 \uc18c\ub828\uc73c\ub85c \ucd9c\ubc1c\ud558\ub77c\uace0 \uc7ac\ucd09\ud558\uc600\ub2e4. \uc774\ubbf8 \uc5ec\ub7ec \ubc88 \uc18c\ub828\uc744 \uc655\ub798\ud55c \ubc14 \uc788\ub294 \uce74\ud53c\ucc28\ub294 \ubcc4 \uc758\uc2ec \uc5c6\uc774 \uadc0\uad6d\uae38\uc5d0 \uc62c\ub790\uc9c0\ub9cc \uc544\ub2c8\ub098 \ub2e4\ub97c\uae4c \uadf8 \uae38\ub85c \uacb0\uad6d \uc18c\ub828\uc5d0 \uc5b5\ub958\ub418\uace0 \ub9d0\uc558\uace0 \uaf3c\uc9dd\uc5c6\uc774 \ud3c9\uc0dd\uc744 \uadf8\uacf3\uc5d0\uc11c \uc9c0\ub0b4\uba74\uc11c \uc5f0\uad6c\ud560 \uc218\ubc16\uc5d0 \uc5c6\uac8c \ub410\ub2e4."} {"question": "\uc0bc\uc790\uc6d0\uce59\uc744 \uc55e\uc138\uc6cc \uc124\ub9bd\ub41c \uac83\uc740?", "paragraph": "1950\ub144\ub300\uc5d0 \uc800\uc6b0\uc5b8\ub77c\uc774\ub294 \uc6b0\uc57c\uc624\ucb5d\uacfc \ud568\uaed8 \uae30\ub3c5\uad50 \uc120\uc5b8\ubb38\uc744 \uc791\uc131\ud558\uc5ec \uc911\uad6d \ub0b4\uc758 \uac1c\uc2e0\uad50\ub294 \uc0c8\ub85c\uc6b4 \uc815\ubd80\ub97c \uc9c0\uc9c0\ud558\uba70 \uc678\uad6d \uc81c\uad6d\uc8fc\uc758\ub97c \ubc30\uaca9\ud55c\ub2e4\uace0 \uc120\ud3ec\ud588\ub2e4. \uadf8\uac83\uc740 \uc790\uce58(\u81ea\u6cbb), \uc790\uc591(\u81ea\u990a), \uc790\uc804(\u81ea\u50b3)\uc774\ub77c\ub294 \u201c\uc0bc\uc790\uc6d0\uce59\u201d\uc744 \uc55e\uc138\uc6b4 \uc0bc\uc790\uc560\uad6d\uc6b4\ub3d9(TSPM)\uc744 \uc124\ub9bd\ud558\ub294 \uacc4\uae30\uac00 \ub418\uc5c8\ub2e4. \uc5bc\ub9c8 \uc9c0\ub098\uc9c0 \uc54a\uc544\uc11c \ub300\ub300\uc801\uc778 \uace0\uc18c\uc640 \uace0\ubc1c \ucea0\ud398\uc778\uc774 \uc2dc\uc791\ub418\uc5c8\uace0, \uadf8\uac83\uc740 \ud574\uc678 \uc120\uad50\uc0ac\ub4e4\uc744 \ucd94\ubc29\ud558\uace0 \ub9ce\uc740 \uc911\uad6d\uc778 \ud1a0\ucc29\ubbfc \uae30\ub3c5\uad50\uc778\ub4e4\uc744 \uccb4\ud3ec\ud558\uace0 \ud22c\uc625\ud558\ub294 \uac83\uc73c\ub85c \uc774\uc5b4\uc84c\ub294\ub370, \ud22c\uc625\ub41c \uc0ac\ub78c\ub4e4 \uc911\uc5d0\ub294 \uc6cc\uce58\ub9cc \ub2c8\uc640 \uc655\ubc0d\ub2e4\uc624(Wang Mingdao)\uc640 \uac19\uc740 \ub6f0\uc5b4\ub09c \uae30\ub3c5\uad50 \uad50\uc0ac\ub4e4\ub3c4 \ud3ec\ud568\ub418\uc5c8\ub2e4. \uc911\uad6d \uc815\ubd80\ub294 \uad6d\ub0b4\uc5d0\uc11c \uadfc\ub798\uc5d0 \ubc1c\uc0dd\ud588\ub358 \ud0dc\ud3c9\ucc9c\uad6d\uc758 \ub09c\uc774 \ubd80\ubd84\uc801\uc73c\ub85c\ub294 \uc885\uad50\uc801\uc778 \ub3d9\uae30 \ub54c\ubb38\uc774\uc5c8\ub2e4\uace0 \ubcf4\uace0, \uc885\uad50 \uc9c0\ub3c4\uc790\ub97c \ud3ec\ud568\ud558\uc5ec \ub204\uad6c\ub4e0\uc9c0 \uc0c1\ub2f9\ud55c \ucd94\uc885\uc790\ub4e4\uc744 \uac00\uc9c4 \uc778\ub3c4\uc790\ub4e4\uacfc \uadf8\ub4e4\uc758 \uc6c0\uc9c1\uc784\uc5d0 \ub300\ud574 \uae4a\uc740 \uc758\uad6c\uc2ec\uc744 \uac00\uc84c\ub2e4. \uc911\uad6d \uacf5\uc0b0 \uc815\ubd80 \uc790\uc2e0\ub4e4\ub3c4 \ucc98\uc74c\uc5d0\ub294 \ub18d\ucd0c \uc9c0\uc5ed\uc758 \uc18c\uaddc\ubaa8 \uc18c\uc791\ub18d\ub4e4\uc744 \uc6c0\uc9c1\uc774\uba74\uc11c \uc810\ucc28\uc801\uc73c\ub85c \uc138\ub825\uc744 \ud0a4\uc6e0\uace0 \uacb0\uad6d \uad6d\ubbfc\ub2f9 \uc815\uad8c\uc744 \ubb34\ub108\ub728\ub9b4 \uc218 \uc788\uc5c8\uae30 \ub54c\ubb38\uc774\uc5c8\ub2e4.", "answer": "\uc0bc\uc790\uc560\uad6d\uc6b4\ub3d9", "sentence": "\uadf8\uac83\uc740 \uc790\uce58(\u81ea\u6cbb), \uc790\uc591(\u81ea\u990a), \uc790\uc804(\u81ea\u50b3)\uc774\ub77c\ub294 \u201c\uc0bc\uc790\uc6d0\uce59\u201d\uc744 \uc55e\uc138\uc6b4 \uc0bc\uc790\uc560\uad6d\uc6b4\ub3d9 (TSPM)", "paragraph_sentence": "1950\ub144\ub300\uc5d0 \uc800\uc6b0\uc5b8\ub77c\uc774\ub294 \uc6b0\uc57c\uc624\ucb5d\uacfc \ud568\uaed8 \uae30\ub3c5\uad50 \uc120\uc5b8\ubb38\uc744 \uc791\uc131\ud558\uc5ec \uc911\uad6d \ub0b4\uc758 \uac1c\uc2e0\uad50\ub294 \uc0c8\ub85c\uc6b4 \uc815\ubd80\ub97c \uc9c0\uc9c0\ud558\uba70 \uc678\uad6d \uc81c\uad6d\uc8fc\uc758\ub97c \ubc30\uaca9\ud55c\ub2e4\uace0 \uc120\ud3ec\ud588\ub2e4. \uadf8\uac83\uc740 \uc790\uce58(\u81ea\u6cbb), \uc790\uc591(\u81ea\u990a), \uc790\uc804(\u81ea\u50b3)\uc774\ub77c\ub294 \u201c\uc0bc\uc790\uc6d0\uce59\u201d\uc744 \uc55e\uc138\uc6b4 \uc0bc\uc790\uc560\uad6d\uc6b4\ub3d9 (TSPM) \uc744 \uc124\ub9bd\ud558\ub294 \uacc4\uae30\uac00 \ub418\uc5c8\ub2e4. \uc5bc\ub9c8 \uc9c0\ub098\uc9c0 \uc54a\uc544\uc11c \ub300\ub300\uc801\uc778 \uace0\uc18c\uc640 \uace0\ubc1c \ucea0\ud398\uc778\uc774 \uc2dc\uc791\ub418\uc5c8\uace0, \uadf8\uac83\uc740 \ud574\uc678 \uc120\uad50\uc0ac\ub4e4\uc744 \ucd94\ubc29\ud558\uace0 \ub9ce\uc740 \uc911\uad6d\uc778 \ud1a0\ucc29\ubbfc \uae30\ub3c5\uad50\uc778\ub4e4\uc744 \uccb4\ud3ec\ud558\uace0 \ud22c\uc625\ud558\ub294 \uac83\uc73c\ub85c \uc774\uc5b4\uc84c\ub294\ub370, \ud22c\uc625\ub41c \uc0ac\ub78c\ub4e4 \uc911\uc5d0\ub294 \uc6cc\uce58\ub9cc \ub2c8\uc640 \uc655\ubc0d\ub2e4\uc624(Wang Mingdao)\uc640 \uac19\uc740 \ub6f0\uc5b4\ub09c \uae30\ub3c5\uad50 \uad50\uc0ac\ub4e4\ub3c4 \ud3ec\ud568\ub418\uc5c8\ub2e4. \uc911\uad6d \uc815\ubd80\ub294 \uad6d\ub0b4\uc5d0\uc11c \uadfc\ub798\uc5d0 \ubc1c\uc0dd\ud588\ub358 \ud0dc\ud3c9\ucc9c\uad6d\uc758 \ub09c\uc774 \ubd80\ubd84\uc801\uc73c\ub85c\ub294 \uc885\uad50\uc801\uc778 \ub3d9\uae30 \ub54c\ubb38\uc774\uc5c8\ub2e4\uace0 \ubcf4\uace0, \uc885\uad50 \uc9c0\ub3c4\uc790\ub97c \ud3ec\ud568\ud558\uc5ec \ub204\uad6c\ub4e0\uc9c0 \uc0c1\ub2f9\ud55c \ucd94\uc885\uc790\ub4e4\uc744 \uac00\uc9c4 \uc778\ub3c4\uc790\ub4e4\uacfc \uadf8\ub4e4\uc758 \uc6c0\uc9c1\uc784\uc5d0 \ub300\ud574 \uae4a\uc740 \uc758\uad6c\uc2ec\uc744 \uac00\uc84c\ub2e4. \uc911\uad6d \uacf5\uc0b0 \uc815\ubd80 \uc790\uc2e0\ub4e4\ub3c4 \ucc98\uc74c\uc5d0\ub294 \ub18d\ucd0c \uc9c0\uc5ed\uc758 \uc18c\uaddc\ubaa8 \uc18c\uc791\ub18d\ub4e4\uc744 \uc6c0\uc9c1\uc774\uba74\uc11c \uc810\ucc28\uc801\uc73c\ub85c \uc138\ub825\uc744 \ud0a4\uc6e0\uace0 \uacb0\uad6d \uad6d\ubbfc\ub2f9 \uc815\uad8c\uc744 \ubb34\ub108\ub728\ub9b4 \uc218 \uc788\uc5c8\uae30 \ub54c\ubb38\uc774\uc5c8\ub2e4.", "paragraph_answer": "1950\ub144\ub300\uc5d0 \uc800\uc6b0\uc5b8\ub77c\uc774\ub294 \uc6b0\uc57c\uc624\ucb5d\uacfc \ud568\uaed8 \uae30\ub3c5\uad50 \uc120\uc5b8\ubb38\uc744 \uc791\uc131\ud558\uc5ec \uc911\uad6d \ub0b4\uc758 \uac1c\uc2e0\uad50\ub294 \uc0c8\ub85c\uc6b4 \uc815\ubd80\ub97c \uc9c0\uc9c0\ud558\uba70 \uc678\uad6d \uc81c\uad6d\uc8fc\uc758\ub97c \ubc30\uaca9\ud55c\ub2e4\uace0 \uc120\ud3ec\ud588\ub2e4. \uadf8\uac83\uc740 \uc790\uce58(\u81ea\u6cbb), \uc790\uc591(\u81ea\u990a), \uc790\uc804(\u81ea\u50b3)\uc774\ub77c\ub294 \u201c\uc0bc\uc790\uc6d0\uce59\u201d\uc744 \uc55e\uc138\uc6b4 \uc0bc\uc790\uc560\uad6d\uc6b4\ub3d9 (TSPM)\uc744 \uc124\ub9bd\ud558\ub294 \uacc4\uae30\uac00 \ub418\uc5c8\ub2e4. \uc5bc\ub9c8 \uc9c0\ub098\uc9c0 \uc54a\uc544\uc11c \ub300\ub300\uc801\uc778 \uace0\uc18c\uc640 \uace0\ubc1c \ucea0\ud398\uc778\uc774 \uc2dc\uc791\ub418\uc5c8\uace0, \uadf8\uac83\uc740 \ud574\uc678 \uc120\uad50\uc0ac\ub4e4\uc744 \ucd94\ubc29\ud558\uace0 \ub9ce\uc740 \uc911\uad6d\uc778 \ud1a0\ucc29\ubbfc \uae30\ub3c5\uad50\uc778\ub4e4\uc744 \uccb4\ud3ec\ud558\uace0 \ud22c\uc625\ud558\ub294 \uac83\uc73c\ub85c \uc774\uc5b4\uc84c\ub294\ub370, \ud22c\uc625\ub41c \uc0ac\ub78c\ub4e4 \uc911\uc5d0\ub294 \uc6cc\uce58\ub9cc \ub2c8\uc640 \uc655\ubc0d\ub2e4\uc624(Wang Mingdao)\uc640 \uac19\uc740 \ub6f0\uc5b4\ub09c \uae30\ub3c5\uad50 \uad50\uc0ac\ub4e4\ub3c4 \ud3ec\ud568\ub418\uc5c8\ub2e4. \uc911\uad6d \uc815\ubd80\ub294 \uad6d\ub0b4\uc5d0\uc11c \uadfc\ub798\uc5d0 \ubc1c\uc0dd\ud588\ub358 \ud0dc\ud3c9\ucc9c\uad6d\uc758 \ub09c\uc774 \ubd80\ubd84\uc801\uc73c\ub85c\ub294 \uc885\uad50\uc801\uc778 \ub3d9\uae30 \ub54c\ubb38\uc774\uc5c8\ub2e4\uace0 \ubcf4\uace0, \uc885\uad50 \uc9c0\ub3c4\uc790\ub97c \ud3ec\ud568\ud558\uc5ec \ub204\uad6c\ub4e0\uc9c0 \uc0c1\ub2f9\ud55c \ucd94\uc885\uc790\ub4e4\uc744 \uac00\uc9c4 \uc778\ub3c4\uc790\ub4e4\uacfc \uadf8\ub4e4\uc758 \uc6c0\uc9c1\uc784\uc5d0 \ub300\ud574 \uae4a\uc740 \uc758\uad6c\uc2ec\uc744 \uac00\uc84c\ub2e4. \uc911\uad6d \uacf5\uc0b0 \uc815\ubd80 \uc790\uc2e0\ub4e4\ub3c4 \ucc98\uc74c\uc5d0\ub294 \ub18d\ucd0c \uc9c0\uc5ed\uc758 \uc18c\uaddc\ubaa8 \uc18c\uc791\ub18d\ub4e4\uc744 \uc6c0\uc9c1\uc774\uba74\uc11c \uc810\ucc28\uc801\uc73c\ub85c \uc138\ub825\uc744 \ud0a4\uc6e0\uace0 \uacb0\uad6d \uad6d\ubbfc\ub2f9 \uc815\uad8c\uc744 \ubb34\ub108\ub728\ub9b4 \uc218 \uc788\uc5c8\uae30 \ub54c\ubb38\uc774\uc5c8\ub2e4.", "sentence_answer": "\uadf8\uac83\uc740 \uc790\uce58(\u81ea\u6cbb), \uc790\uc591(\u81ea\u990a), \uc790\uc804(\u81ea\u50b3)\uc774\ub77c\ub294 \u201c\uc0bc\uc790\uc6d0\uce59\u201d\uc744 \uc55e\uc138\uc6b4 \uc0bc\uc790\uc560\uad6d\uc6b4\ub3d9 (TSPM)", "paragraph_id": "6571934-3-1", "paragraph_question": "question: \uc0bc\uc790\uc6d0\uce59\uc744 \uc55e\uc138\uc6cc \uc124\ub9bd\ub41c \uac83\uc740?, context: 1950\ub144\ub300\uc5d0 \uc800\uc6b0\uc5b8\ub77c\uc774\ub294 \uc6b0\uc57c\uc624\ucb5d\uacfc \ud568\uaed8 \uae30\ub3c5\uad50 \uc120\uc5b8\ubb38\uc744 \uc791\uc131\ud558\uc5ec \uc911\uad6d \ub0b4\uc758 \uac1c\uc2e0\uad50\ub294 \uc0c8\ub85c\uc6b4 \uc815\ubd80\ub97c \uc9c0\uc9c0\ud558\uba70 \uc678\uad6d \uc81c\uad6d\uc8fc\uc758\ub97c \ubc30\uaca9\ud55c\ub2e4\uace0 \uc120\ud3ec\ud588\ub2e4. \uadf8\uac83\uc740 \uc790\uce58(\u81ea\u6cbb), \uc790\uc591(\u81ea\u990a), \uc790\uc804(\u81ea\u50b3)\uc774\ub77c\ub294 \u201c\uc0bc\uc790\uc6d0\uce59\u201d\uc744 \uc55e\uc138\uc6b4 \uc0bc\uc790\uc560\uad6d\uc6b4\ub3d9(TSPM)\uc744 \uc124\ub9bd\ud558\ub294 \uacc4\uae30\uac00 \ub418\uc5c8\ub2e4. \uc5bc\ub9c8 \uc9c0\ub098\uc9c0 \uc54a\uc544\uc11c \ub300\ub300\uc801\uc778 \uace0\uc18c\uc640 \uace0\ubc1c \ucea0\ud398\uc778\uc774 \uc2dc\uc791\ub418\uc5c8\uace0, \uadf8\uac83\uc740 \ud574\uc678 \uc120\uad50\uc0ac\ub4e4\uc744 \ucd94\ubc29\ud558\uace0 \ub9ce\uc740 \uc911\uad6d\uc778 \ud1a0\ucc29\ubbfc \uae30\ub3c5\uad50\uc778\ub4e4\uc744 \uccb4\ud3ec\ud558\uace0 \ud22c\uc625\ud558\ub294 \uac83\uc73c\ub85c \uc774\uc5b4\uc84c\ub294\ub370, \ud22c\uc625\ub41c \uc0ac\ub78c\ub4e4 \uc911\uc5d0\ub294 \uc6cc\uce58\ub9cc \ub2c8\uc640 \uc655\ubc0d\ub2e4\uc624(Wang Mingdao)\uc640 \uac19\uc740 \ub6f0\uc5b4\ub09c \uae30\ub3c5\uad50 \uad50\uc0ac\ub4e4\ub3c4 \ud3ec\ud568\ub418\uc5c8\ub2e4. \uc911\uad6d \uc815\ubd80\ub294 \uad6d\ub0b4\uc5d0\uc11c \uadfc\ub798\uc5d0 \ubc1c\uc0dd\ud588\ub358 \ud0dc\ud3c9\ucc9c\uad6d\uc758 \ub09c\uc774 \ubd80\ubd84\uc801\uc73c\ub85c\ub294 \uc885\uad50\uc801\uc778 \ub3d9\uae30 \ub54c\ubb38\uc774\uc5c8\ub2e4\uace0 \ubcf4\uace0, \uc885\uad50 \uc9c0\ub3c4\uc790\ub97c \ud3ec\ud568\ud558\uc5ec \ub204\uad6c\ub4e0\uc9c0 \uc0c1\ub2f9\ud55c \ucd94\uc885\uc790\ub4e4\uc744 \uac00\uc9c4 \uc778\ub3c4\uc790\ub4e4\uacfc \uadf8\ub4e4\uc758 \uc6c0\uc9c1\uc784\uc5d0 \ub300\ud574 \uae4a\uc740 \uc758\uad6c\uc2ec\uc744 \uac00\uc84c\ub2e4. \uc911\uad6d \uacf5\uc0b0 \uc815\ubd80 \uc790\uc2e0\ub4e4\ub3c4 \ucc98\uc74c\uc5d0\ub294 \ub18d\ucd0c \uc9c0\uc5ed\uc758 \uc18c\uaddc\ubaa8 \uc18c\uc791\ub18d\ub4e4\uc744 \uc6c0\uc9c1\uc774\uba74\uc11c \uc810\ucc28\uc801\uc73c\ub85c \uc138\ub825\uc744 \ud0a4\uc6e0\uace0 \uacb0\uad6d \uad6d\ubbfc\ub2f9 \uc815\uad8c\uc744 \ubb34\ub108\ub728\ub9b4 \uc218 \uc788\uc5c8\uae30 \ub54c\ubb38\uc774\uc5c8\ub2e4."} {"question": "2007\ub144 \ub300\uc120 \ud6c4\ubcf4 \uacbd\uc120 \ub2f9\uc2dc \uc774\uba85\ubc15 \ub300\ud1b5\ub839\uc774 \ub0b4\uc138\uc6e0\ub358 \uc815\ucc45\uc740 \ubb34\uc5c7\uc778\uac00?", "paragraph": "\uc774\uba85\ubc15 \uc815\ubd80 \uc2dc\uc808 \ubc15\uadfc\ud61c\ub294 \uc774\uba85\ubc15\uacfc \uac70\ub9ac\ub97c \ub450\uc5c8\uc73c\ub098 \uc644\uc804\ud788 \ub3cc\uc544\uc11c\uc9c0\ub294 \uc54a\uc558\ub2e4. \ub300\ud1b5\ub839\uacfc \uba74\ub2f4\uc744 \ud558\uba74\uc11c \ub300\ud1b5\ud569\uc758 \uc758\uc9c0\ub97c \ub2f4\uace0 \ud65c\ub3d9\ud558\uc600\ub2e4. \uacfc\uac70 \uc815\ucc45\uc0c1 \ubcf8\uc9c8\uc801\uc73c\ub85c\ub294 \uc774\uba85\ubc15\uacfc \ud06c\uac8c \ub2e4\ub97c\ubc14 \uc5c6\ub2e4\ub294 \uc758\uacac\ub3c4 \uc788\uc5c8\ub2e4. \uadfc\uac70\ub85c\ub294 2007\ub144 \ud55c\ub098\ub77c\ub2f9 \ub300\uc120 \ud6c4\ubcf4 \uacbd\uc120 \ub54c \ubc15\uadfc\ud61c\uac00 \ub0b4\uc138\uc6e0\ub358 '\uc904\ud478\uc138'\ub098 \ub2f9\uc2dc \uacbd\uc7c1 \ud6c4\ubcf4\uc600\ub358 \uc774\uba85\ubc15\uc774 \ub0b4\uc138\uc6e0\ub358 '747' \ub458\ub2e4 \uacf5\ud1b5\uc801\uc73c\ub85c \uac10\uc138 \uae30\uc870\ub97c \ub0b4\uc138\uc6b4 \uc815\ucc45\uc774\ub77c\ub294 \uc810\uc774\ub2e4. \ub610\ud55c \ubc15\uadfc\ud61c\uac00 2008\ub144 12\uc6d4 \uad6d\ud68c\uc758 \uac10\uc138 \ubc95\uc548, 2009\ub144 3\uc6d4 \uae08\uc0b0\ubd84\ub9ac\uc644\ud654\ubc95\uc548, 2009\ub144 12\uc6d4 \uc608\uc0b0\uc548 \u2500 4\ub300\uac15 \uc608\uc0b0\uc774 \ud3ec\ud568\ub418\uc5c8\uace0 \uc774\ub978\ubc14 \ub0a0\uce58\uae30 \ud1b5\uacfc\ub85c \ub17c\ub780\uc774 \ub418\uc5c8\uc74c \u2500 \uc5d0 \ucc2c\uc131\ud574 \uc774\uba85\ubc15 \uc815\ubd80 \uc815\ucc45\uc5d0 \ud611\uc870\ud55c \uac83\ub3c4 \uc788\ub2e4. \uc774\ub97c \ube57\ub300\uc5b4 \uc815\uce58\uad8c\uacfc \uc778\ud130\ub137 \uc0c1\uc5d0\uc11c '\uc774\uba85\ubc15\uadfc\ud61c'\ub85c \ud45c\ud604\ud558\uae30\ub3c4 \ud55c\ub2e4. \uadf8\ub7ec\ub098 \uc774\uba85\ubc15\uacfc \ub2e4\ub974\ub2e4\uace0 \ubcfc \uc218 \uc788\ub294 \uac83\ub4e4\ub3c4 \uc788\ub2e4. \ub300\ud45c\uc801\uc73c\ub85c \uc774\uba85\ubc15 \ub300\ud1b5\ub839\uc774 \uc8fc\ub3c4\ud588\ub358 '\uc138\uc885\uc2dc \uc218\uc815\uc548'\uc744 \ubd80\uacb0\ub418\uac8c \ub9cc\ub4e0 \uac83\uc774\ub2e4. \ub610\ud55c \uc11c\uc6b8\uc2dc\uc758 \ubb34\uc0c1 \uae09\uc2dd \uc815\ucc45 \ub17c\ub780 \ub54c \ubc14\uc05c \uc77c\uc815 \uc18d\uc5d0 \ubd80\uc7ac\uc790 \ud22c\ud45c\uae4c\uc9c0 \ucc38\uc5ec\ud574 \uc624\uc138\ud6c8 \uc804 \uc11c\uc6b8\uc2dc\uc7a5\uc744 \uc9c0\uc6d0\ud588\ub358 \uc774\uba85\ubc15\uacfc\ub294 \ub2ec\ub9ac \uad00\ub828 \uc0ac\uc548\uc5d0 \uc785\uc7a5\uc744 \ubc1d\ud788\uc9c0 \uc54a\uc558\ub2e4. 2011\ub144 \ub9d0 \ube44\uc0c1\ub300\ucc45\uc704\uc6d0\uc7a5\uc744 \ub9e1\uc740 \ud6c4 \ub2f9\uba85, \ub2f9 \uc0c1\uc9d5\uc0c9, \uac15\ub839 \ub4f1\uc744 \ubcc0\uacbd\ud558\uba74\uc11c MB\uc815\uad8c\uacfc \uae30\uc874 \ud55c\ub098\ub77c\ub2f9\uacfc\uc758 \ucc28\ubcc4\ud654\ub97c \uc2dc\ub3c4\ud588\ub2e4. 2012\ub144 4\uc6d4 \ucd1d\uc120\uc744 \uc55e\ub450\uace0 \ubd88\uac70\uc9c4 \uad6d\ubb34\ucd1d\ub9ac\uc2e4\uc758 \ubbfc\uac04\uc778 \uc0ac\ucc30 \uc0ac\uac74 \uc545\uc7ac \uc18d\uc5d0\uc11c\ub3c4 \"\uc5b4\ub290 \uc815\uad8c\uc5d0\uc11c\ub098 \ubbfc\uac04\uc778 \uc0ac\ucc30\uc740 \uc77c\uc5b4\ub098\uba70 \ub098\ub3c4 \ud53c\ud574\uc790\"\ub77c\uba70 \ub17c\ub780\uc744 \ud53c\ud558\uae30\ub3c4 \ud588\ub2e4.", "answer": "747", "sentence": "\uadfc\uac70\ub85c\ub294 2007\ub144 \ud55c\ub098\ub77c\ub2f9 \ub300\uc120 \ud6c4\ubcf4 \uacbd\uc120 \ub54c \ubc15\uadfc\ud61c\uac00 \ub0b4\uc138\uc6e0\ub358 '\uc904\ud478\uc138'\ub098 \ub2f9\uc2dc \uacbd\uc7c1 \ud6c4\ubcf4\uc600\ub358 \uc774\uba85\ubc15\uc774 \ub0b4\uc138\uc6e0\ub358 ' 747 '", "paragraph_sentence": "\uc774\uba85\ubc15 \uc815\ubd80 \uc2dc\uc808 \ubc15\uadfc\ud61c\ub294 \uc774\uba85\ubc15\uacfc \uac70\ub9ac\ub97c \ub450\uc5c8\uc73c\ub098 \uc644\uc804\ud788 \ub3cc\uc544\uc11c\uc9c0\ub294 \uc54a\uc558\ub2e4. \ub300\ud1b5\ub839\uacfc \uba74\ub2f4\uc744 \ud558\uba74\uc11c \ub300\ud1b5\ud569\uc758 \uc758\uc9c0\ub97c \ub2f4\uace0 \ud65c\ub3d9\ud558\uc600\ub2e4. \uacfc\uac70 \uc815\ucc45\uc0c1 \ubcf8\uc9c8\uc801\uc73c\ub85c\ub294 \uc774\uba85\ubc15\uacfc \ud06c\uac8c \ub2e4\ub97c\ubc14 \uc5c6\ub2e4\ub294 \uc758\uacac\ub3c4 \uc788\uc5c8\ub2e4. \uadfc\uac70\ub85c\ub294 2007\ub144 \ud55c\ub098\ub77c\ub2f9 \ub300\uc120 \ud6c4\ubcf4 \uacbd\uc120 \ub54c \ubc15\uadfc\ud61c\uac00 \ub0b4\uc138\uc6e0\ub358 '\uc904\ud478\uc138'\ub098 \ub2f9\uc2dc \uacbd\uc7c1 \ud6c4\ubcf4\uc600\ub358 \uc774\uba85\ubc15\uc774 \ub0b4\uc138\uc6e0\ub358 ' 747 ' \ub458\ub2e4 \uacf5\ud1b5\uc801\uc73c\ub85c \uac10\uc138 \uae30\uc870\ub97c \ub0b4\uc138\uc6b4 \uc815\ucc45\uc774\ub77c\ub294 \uc810\uc774\ub2e4. \ub610\ud55c \ubc15\uadfc\ud61c\uac00 2008\ub144 12\uc6d4 \uad6d\ud68c\uc758 \uac10\uc138 \ubc95\uc548, 2009\ub144 3\uc6d4 \uae08\uc0b0\ubd84\ub9ac\uc644\ud654\ubc95\uc548, 2009\ub144 12\uc6d4 \uc608\uc0b0\uc548 \u2500 4\ub300\uac15 \uc608\uc0b0\uc774 \ud3ec\ud568\ub418\uc5c8\uace0 \uc774\ub978\ubc14 \ub0a0\uce58\uae30 \ud1b5\uacfc\ub85c \ub17c\ub780\uc774 \ub418\uc5c8\uc74c \u2500 \uc5d0 \ucc2c\uc131\ud574 \uc774\uba85\ubc15 \uc815\ubd80 \uc815\ucc45\uc5d0 \ud611\uc870\ud55c \uac83\ub3c4 \uc788\ub2e4. \uc774\ub97c \ube57\ub300\uc5b4 \uc815\uce58\uad8c\uacfc \uc778\ud130\ub137 \uc0c1\uc5d0\uc11c '\uc774\uba85\ubc15\uadfc\ud61c'\ub85c \ud45c\ud604\ud558\uae30\ub3c4 \ud55c\ub2e4. \uadf8\ub7ec\ub098 \uc774\uba85\ubc15\uacfc \ub2e4\ub974\ub2e4\uace0 \ubcfc \uc218 \uc788\ub294 \uac83\ub4e4\ub3c4 \uc788\ub2e4. \ub300\ud45c\uc801\uc73c\ub85c \uc774\uba85\ubc15 \ub300\ud1b5\ub839\uc774 \uc8fc\ub3c4\ud588\ub358 '\uc138\uc885\uc2dc \uc218\uc815\uc548'\uc744 \ubd80\uacb0\ub418\uac8c \ub9cc\ub4e0 \uac83\uc774\ub2e4. \ub610\ud55c \uc11c\uc6b8\uc2dc\uc758 \ubb34\uc0c1 \uae09\uc2dd \uc815\ucc45 \ub17c\ub780 \ub54c \ubc14\uc05c \uc77c\uc815 \uc18d\uc5d0 \ubd80\uc7ac\uc790 \ud22c\ud45c\uae4c\uc9c0 \ucc38\uc5ec\ud574 \uc624\uc138\ud6c8 \uc804 \uc11c\uc6b8\uc2dc\uc7a5\uc744 \uc9c0\uc6d0\ud588\ub358 \uc774\uba85\ubc15\uacfc\ub294 \ub2ec\ub9ac \uad00\ub828 \uc0ac\uc548\uc5d0 \uc785\uc7a5\uc744 \ubc1d\ud788\uc9c0 \uc54a\uc558\ub2e4. 2011\ub144 \ub9d0 \ube44\uc0c1\ub300\ucc45\uc704\uc6d0\uc7a5\uc744 \ub9e1\uc740 \ud6c4 \ub2f9\uba85, \ub2f9 \uc0c1\uc9d5\uc0c9, \uac15\ub839 \ub4f1\uc744 \ubcc0\uacbd\ud558\uba74\uc11c MB\uc815\uad8c\uacfc \uae30\uc874 \ud55c\ub098\ub77c\ub2f9\uacfc\uc758 \ucc28\ubcc4\ud654\ub97c \uc2dc\ub3c4\ud588\ub2e4. 2012\ub144 4\uc6d4 \ucd1d\uc120\uc744 \uc55e\ub450\uace0 \ubd88\uac70\uc9c4 \uad6d\ubb34\ucd1d\ub9ac\uc2e4\uc758 \ubbfc\uac04\uc778 \uc0ac\ucc30 \uc0ac\uac74 \uc545\uc7ac \uc18d\uc5d0\uc11c\ub3c4 \"\uc5b4\ub290 \uc815\uad8c\uc5d0\uc11c\ub098 \ubbfc\uac04\uc778 \uc0ac\ucc30\uc740 \uc77c\uc5b4\ub098\uba70 \ub098\ub3c4 \ud53c\ud574\uc790\"\ub77c\uba70 \ub17c\ub780\uc744 \ud53c\ud558\uae30\ub3c4 \ud588\ub2e4.", "paragraph_answer": "\uc774\uba85\ubc15 \uc815\ubd80 \uc2dc\uc808 \ubc15\uadfc\ud61c\ub294 \uc774\uba85\ubc15\uacfc \uac70\ub9ac\ub97c \ub450\uc5c8\uc73c\ub098 \uc644\uc804\ud788 \ub3cc\uc544\uc11c\uc9c0\ub294 \uc54a\uc558\ub2e4. \ub300\ud1b5\ub839\uacfc \uba74\ub2f4\uc744 \ud558\uba74\uc11c \ub300\ud1b5\ud569\uc758 \uc758\uc9c0\ub97c \ub2f4\uace0 \ud65c\ub3d9\ud558\uc600\ub2e4. \uacfc\uac70 \uc815\ucc45\uc0c1 \ubcf8\uc9c8\uc801\uc73c\ub85c\ub294 \uc774\uba85\ubc15\uacfc \ud06c\uac8c \ub2e4\ub97c\ubc14 \uc5c6\ub2e4\ub294 \uc758\uacac\ub3c4 \uc788\uc5c8\ub2e4. \uadfc\uac70\ub85c\ub294 2007\ub144 \ud55c\ub098\ub77c\ub2f9 \ub300\uc120 \ud6c4\ubcf4 \uacbd\uc120 \ub54c \ubc15\uadfc\ud61c\uac00 \ub0b4\uc138\uc6e0\ub358 '\uc904\ud478\uc138'\ub098 \ub2f9\uc2dc \uacbd\uc7c1 \ud6c4\ubcf4\uc600\ub358 \uc774\uba85\ubc15\uc774 \ub0b4\uc138\uc6e0\ub358 ' 747 ' \ub458\ub2e4 \uacf5\ud1b5\uc801\uc73c\ub85c \uac10\uc138 \uae30\uc870\ub97c \ub0b4\uc138\uc6b4 \uc815\ucc45\uc774\ub77c\ub294 \uc810\uc774\ub2e4. \ub610\ud55c \ubc15\uadfc\ud61c\uac00 2008\ub144 12\uc6d4 \uad6d\ud68c\uc758 \uac10\uc138 \ubc95\uc548, 2009\ub144 3\uc6d4 \uae08\uc0b0\ubd84\ub9ac\uc644\ud654\ubc95\uc548, 2009\ub144 12\uc6d4 \uc608\uc0b0\uc548 \u2500 4\ub300\uac15 \uc608\uc0b0\uc774 \ud3ec\ud568\ub418\uc5c8\uace0 \uc774\ub978\ubc14 \ub0a0\uce58\uae30 \ud1b5\uacfc\ub85c \ub17c\ub780\uc774 \ub418\uc5c8\uc74c \u2500 \uc5d0 \ucc2c\uc131\ud574 \uc774\uba85\ubc15 \uc815\ubd80 \uc815\ucc45\uc5d0 \ud611\uc870\ud55c \uac83\ub3c4 \uc788\ub2e4. \uc774\ub97c \ube57\ub300\uc5b4 \uc815\uce58\uad8c\uacfc \uc778\ud130\ub137 \uc0c1\uc5d0\uc11c '\uc774\uba85\ubc15\uadfc\ud61c'\ub85c \ud45c\ud604\ud558\uae30\ub3c4 \ud55c\ub2e4. \uadf8\ub7ec\ub098 \uc774\uba85\ubc15\uacfc \ub2e4\ub974\ub2e4\uace0 \ubcfc \uc218 \uc788\ub294 \uac83\ub4e4\ub3c4 \uc788\ub2e4. \ub300\ud45c\uc801\uc73c\ub85c \uc774\uba85\ubc15 \ub300\ud1b5\ub839\uc774 \uc8fc\ub3c4\ud588\ub358 '\uc138\uc885\uc2dc \uc218\uc815\uc548'\uc744 \ubd80\uacb0\ub418\uac8c \ub9cc\ub4e0 \uac83\uc774\ub2e4. \ub610\ud55c \uc11c\uc6b8\uc2dc\uc758 \ubb34\uc0c1 \uae09\uc2dd \uc815\ucc45 \ub17c\ub780 \ub54c \ubc14\uc05c \uc77c\uc815 \uc18d\uc5d0 \ubd80\uc7ac\uc790 \ud22c\ud45c\uae4c\uc9c0 \ucc38\uc5ec\ud574 \uc624\uc138\ud6c8 \uc804 \uc11c\uc6b8\uc2dc\uc7a5\uc744 \uc9c0\uc6d0\ud588\ub358 \uc774\uba85\ubc15\uacfc\ub294 \ub2ec\ub9ac \uad00\ub828 \uc0ac\uc548\uc5d0 \uc785\uc7a5\uc744 \ubc1d\ud788\uc9c0 \uc54a\uc558\ub2e4. 2011\ub144 \ub9d0 \ube44\uc0c1\ub300\ucc45\uc704\uc6d0\uc7a5\uc744 \ub9e1\uc740 \ud6c4 \ub2f9\uba85, \ub2f9 \uc0c1\uc9d5\uc0c9, \uac15\ub839 \ub4f1\uc744 \ubcc0\uacbd\ud558\uba74\uc11c MB\uc815\uad8c\uacfc \uae30\uc874 \ud55c\ub098\ub77c\ub2f9\uacfc\uc758 \ucc28\ubcc4\ud654\ub97c \uc2dc\ub3c4\ud588\ub2e4. 2012\ub144 4\uc6d4 \ucd1d\uc120\uc744 \uc55e\ub450\uace0 \ubd88\uac70\uc9c4 \uad6d\ubb34\ucd1d\ub9ac\uc2e4\uc758 \ubbfc\uac04\uc778 \uc0ac\ucc30 \uc0ac\uac74 \uc545\uc7ac \uc18d\uc5d0\uc11c\ub3c4 \"\uc5b4\ub290 \uc815\uad8c\uc5d0\uc11c\ub098 \ubbfc\uac04\uc778 \uc0ac\ucc30\uc740 \uc77c\uc5b4\ub098\uba70 \ub098\ub3c4 \ud53c\ud574\uc790\"\ub77c\uba70 \ub17c\ub780\uc744 \ud53c\ud558\uae30\ub3c4 \ud588\ub2e4.", "sentence_answer": "\uadfc\uac70\ub85c\ub294 2007\ub144 \ud55c\ub098\ub77c\ub2f9 \ub300\uc120 \ud6c4\ubcf4 \uacbd\uc120 \ub54c \ubc15\uadfc\ud61c\uac00 \ub0b4\uc138\uc6e0\ub358 '\uc904\ud478\uc138'\ub098 \ub2f9\uc2dc \uacbd\uc7c1 \ud6c4\ubcf4\uc600\ub358 \uc774\uba85\ubc15\uc774 \ub0b4\uc138\uc6e0\ub358 ' 747 '", "paragraph_id": "6506540-32-0", "paragraph_question": "question: 2007\ub144 \ub300\uc120 \ud6c4\ubcf4 \uacbd\uc120 \ub2f9\uc2dc \uc774\uba85\ubc15 \ub300\ud1b5\ub839\uc774 \ub0b4\uc138\uc6e0\ub358 \uc815\ucc45\uc740 \ubb34\uc5c7\uc778\uac00?, context: \uc774\uba85\ubc15 \uc815\ubd80 \uc2dc\uc808 \ubc15\uadfc\ud61c\ub294 \uc774\uba85\ubc15\uacfc \uac70\ub9ac\ub97c \ub450\uc5c8\uc73c\ub098 \uc644\uc804\ud788 \ub3cc\uc544\uc11c\uc9c0\ub294 \uc54a\uc558\ub2e4. \ub300\ud1b5\ub839\uacfc \uba74\ub2f4\uc744 \ud558\uba74\uc11c \ub300\ud1b5\ud569\uc758 \uc758\uc9c0\ub97c \ub2f4\uace0 \ud65c\ub3d9\ud558\uc600\ub2e4. \uacfc\uac70 \uc815\ucc45\uc0c1 \ubcf8\uc9c8\uc801\uc73c\ub85c\ub294 \uc774\uba85\ubc15\uacfc \ud06c\uac8c \ub2e4\ub97c\ubc14 \uc5c6\ub2e4\ub294 \uc758\uacac\ub3c4 \uc788\uc5c8\ub2e4. \uadfc\uac70\ub85c\ub294 2007\ub144 \ud55c\ub098\ub77c\ub2f9 \ub300\uc120 \ud6c4\ubcf4 \uacbd\uc120 \ub54c \ubc15\uadfc\ud61c\uac00 \ub0b4\uc138\uc6e0\ub358 '\uc904\ud478\uc138'\ub098 \ub2f9\uc2dc \uacbd\uc7c1 \ud6c4\ubcf4\uc600\ub358 \uc774\uba85\ubc15\uc774 \ub0b4\uc138\uc6e0\ub358 '747' \ub458\ub2e4 \uacf5\ud1b5\uc801\uc73c\ub85c \uac10\uc138 \uae30\uc870\ub97c \ub0b4\uc138\uc6b4 \uc815\ucc45\uc774\ub77c\ub294 \uc810\uc774\ub2e4. \ub610\ud55c \ubc15\uadfc\ud61c\uac00 2008\ub144 12\uc6d4 \uad6d\ud68c\uc758 \uac10\uc138 \ubc95\uc548, 2009\ub144 3\uc6d4 \uae08\uc0b0\ubd84\ub9ac\uc644\ud654\ubc95\uc548, 2009\ub144 12\uc6d4 \uc608\uc0b0\uc548 \u2500 4\ub300\uac15 \uc608\uc0b0\uc774 \ud3ec\ud568\ub418\uc5c8\uace0 \uc774\ub978\ubc14 \ub0a0\uce58\uae30 \ud1b5\uacfc\ub85c \ub17c\ub780\uc774 \ub418\uc5c8\uc74c \u2500 \uc5d0 \ucc2c\uc131\ud574 \uc774\uba85\ubc15 \uc815\ubd80 \uc815\ucc45\uc5d0 \ud611\uc870\ud55c \uac83\ub3c4 \uc788\ub2e4. \uc774\ub97c \ube57\ub300\uc5b4 \uc815\uce58\uad8c\uacfc \uc778\ud130\ub137 \uc0c1\uc5d0\uc11c '\uc774\uba85\ubc15\uadfc\ud61c'\ub85c \ud45c\ud604\ud558\uae30\ub3c4 \ud55c\ub2e4. \uadf8\ub7ec\ub098 \uc774\uba85\ubc15\uacfc \ub2e4\ub974\ub2e4\uace0 \ubcfc \uc218 \uc788\ub294 \uac83\ub4e4\ub3c4 \uc788\ub2e4. \ub300\ud45c\uc801\uc73c\ub85c \uc774\uba85\ubc15 \ub300\ud1b5\ub839\uc774 \uc8fc\ub3c4\ud588\ub358 '\uc138\uc885\uc2dc \uc218\uc815\uc548'\uc744 \ubd80\uacb0\ub418\uac8c \ub9cc\ub4e0 \uac83\uc774\ub2e4. \ub610\ud55c \uc11c\uc6b8\uc2dc\uc758 \ubb34\uc0c1 \uae09\uc2dd \uc815\ucc45 \ub17c\ub780 \ub54c \ubc14\uc05c \uc77c\uc815 \uc18d\uc5d0 \ubd80\uc7ac\uc790 \ud22c\ud45c\uae4c\uc9c0 \ucc38\uc5ec\ud574 \uc624\uc138\ud6c8 \uc804 \uc11c\uc6b8\uc2dc\uc7a5\uc744 \uc9c0\uc6d0\ud588\ub358 \uc774\uba85\ubc15\uacfc\ub294 \ub2ec\ub9ac \uad00\ub828 \uc0ac\uc548\uc5d0 \uc785\uc7a5\uc744 \ubc1d\ud788\uc9c0 \uc54a\uc558\ub2e4. 2011\ub144 \ub9d0 \ube44\uc0c1\ub300\ucc45\uc704\uc6d0\uc7a5\uc744 \ub9e1\uc740 \ud6c4 \ub2f9\uba85, \ub2f9 \uc0c1\uc9d5\uc0c9, \uac15\ub839 \ub4f1\uc744 \ubcc0\uacbd\ud558\uba74\uc11c MB\uc815\uad8c\uacfc \uae30\uc874 \ud55c\ub098\ub77c\ub2f9\uacfc\uc758 \ucc28\ubcc4\ud654\ub97c \uc2dc\ub3c4\ud588\ub2e4. 2012\ub144 4\uc6d4 \ucd1d\uc120\uc744 \uc55e\ub450\uace0 \ubd88\uac70\uc9c4 \uad6d\ubb34\ucd1d\ub9ac\uc2e4\uc758 \ubbfc\uac04\uc778 \uc0ac\ucc30 \uc0ac\uac74 \uc545\uc7ac \uc18d\uc5d0\uc11c\ub3c4 \"\uc5b4\ub290 \uc815\uad8c\uc5d0\uc11c\ub098 \ubbfc\uac04\uc778 \uc0ac\ucc30\uc740 \uc77c\uc5b4\ub098\uba70 \ub098\ub3c4 \ud53c\ud574\uc790\"\ub77c\uba70 \ub17c\ub780\uc744 \ud53c\ud558\uae30\ub3c4 \ud588\ub2e4."} {"question": "\ud6c8\ubbfc\uc815\uc74c \ud574\ub840\ubcf8\uc740 1997\ub144 10\uc6d4 \ubb34\uc5c7\uc73c\ub85c \ub4f1\uc7ac\ub418\uc5c8\ub294\uac00?", "paragraph": "\uadf8\ub7f0\ub370 \ub2f9\uc2dc \uc774 \uc9d1\uc548\uc758 \uc0ac\uc704\uc600\ub358 \uc774\uc6a9\uc900\uc774 \ub9e4\uc6d4\ub2f9\uc9d1 \ub4f1\uc744 \ube44\ub86f\ud558\uc5ec \ud6c8\ubbfc\uc815\uc74c \ud574\ub840\ubcf8\uc744 \ubab0\ub798 \ube7c\ub3cc\ub824 \uc548\ub3d9\uc758 \uc790\ud0dd\uc5d0\uc11c \ubcf4\uad00\ud558\uace0 \uc788\ub2e4\uac00 \uae40\ud0dc\uc900\uc744 \ud1b5\ud574 \uc804\ud615\ud544 \uc120\uc0dd\uc5d0\uac8c \ud6c8\ubbfc\uc815\uc74c \ud574\ub840\ubcf8\ub9cc \ub9cc\uc6d0\uc5d0 \ud310\ub9e4\ud558\uc600\ub2e4. \ucd5c\ucd08 \ud310\ub9e4\uac00\ub85c \ucc9c\uc6d0\uc744 \uc81c\uc2dc\ud558\uc600\uc73c\ub098 \uc6d0\ub798 \ubb38\ud654\uc7ac\uc758 \uac00\uce58\ub97c \uc815\ud655\ud788 \uce58\ub974\ub294 \uac83\uc73c\ub85c \uc720\uba85\ud588\ub358 \uc804\ud615\ud544\uc740 \uae08\uc561\uc774 \ub108\ubb34 \uc801\ub2e4\uace0 \uc0dd\uac01\ud558\uc5ec \uac70\uac04 \ub178\ub987\uc744 \ud55c \uae40\ud0dc\uc900\uc5d0\uac8c \ucc9c\uc6d0\uc744, \uadf8\ub9ac\uace0 \ud6c8\ubbfc\uc815\uc74c \ud574\ub840\ubcf8\uc758 \uac00\uaca9\uc73c\ub85c \ub9cc\uc6d0\uc744 \uce58\ub800\ub2e4. \ub2f9\uc2dc \ucc9c\uc6d0\uc774\uba74 \uc88b\uc740 \uae30\uc640\uc9d1 \ud55c\ucc44\ub97c \uc0b4 \uc218 \uc788\uc5c8\ub2e4\uace0 \ud55c\ub2e4. \uc774\ud6c4 \ud559\uacc4\uc5d0 \ud6c8\ubbfc\uc815\uc74c \ud574\ub840\ubcf8\uc774 \ubc1c\uacac\ub418\uba74\uc11c \ud559\uc790\ub4e4 \uc0ac\uc774\uc5d0 \ub17c\ub780\uc774 \uc77c\uc5c8\ub358 \ud55c\uae00 \ucc3d\uc81c \uc6d0\ub9ac\uac00 \uc774 \ud6c8\ubbfc\uc815\uc74c \ud574\ub840\ubcf8\uc5d0 \uc758\ud574 \ubc1d\ud600\uc84c\ub2e4. \ud6c4\uc5d0 \uc774 \ud6c8\ubbfc\uc815\uc74c \ud574\ub840\ubcf8\uc740 1962\ub144 12\uc6d4 20\uc77c \ub300\ud55c\ubbfc\uad6d\uc758 \uad6d\ubcf4 \uc81c 70\ud638\ub85c \uc9c0\uc815\ub418\uc5c8\ub2e4. \uc774\ud6c4 1997\ub144 10\uc6d4\uc5d0 \ub300\ud55c\ubbfc\uad6d\uc758 \uc720\ub124\uc2a4\ucf54 \uc138\uacc4\uae30\ub85d\uc720\uc0b0\uc73c\ub85c \ub4f1\uc7ac\ub418\uc5c8\uae30\ub3c4 \ud558\uc600\ub2e4. \uac70\uac04 \ub178\ub987\uc740 \ud55c \uae40\ud0dc\uc900\uacfc \uc774\uc6a9\uc900\uc740 \uc0ac\ud68c\uc8fc\uc758\uc790\ub85c, \ud310\ub9e4\ud55c \ub3c8\uc744 \uc0ac\ud68c\uc8fc\uc758 \uc6b4\ub3d9\uc5d0 \uc0ac\uc6a9\ud588\ub2e4\uace0 \uc804\ud574\uc9c0\uba70 \uad11\ubcf5 \ud6c4 \uae40\ud0dc\uc900\uc740 \uc77c\ub828\uc758 \uc0ac\uac74\uc73c\ub85c \uccb4\ud3ec\ub418\uc5b4 \uc0ac\ud615\ub418\uc5c8\uace0, \uc774\uc6a9\uc900\uc740 \uc6d4\ubd81\ud558\uc600\ub2e4\uace0 \ud55c\ub2e4.", "answer": "\uc720\ub124\uc2a4\ucf54 \uc138\uacc4\uae30\ub85d\uc720\uc0b0", "sentence": "\uc774\ud6c4 1997\ub144 10\uc6d4\uc5d0 \ub300\ud55c\ubbfc\uad6d\uc758 \uc720\ub124\uc2a4\ucf54 \uc138\uacc4\uae30\ub85d\uc720\uc0b0 \uc73c\ub85c \ub4f1\uc7ac\ub418\uc5c8\uae30\ub3c4 \ud558\uc600\ub2e4.", "paragraph_sentence": "\uadf8\ub7f0\ub370 \ub2f9\uc2dc \uc774 \uc9d1\uc548\uc758 \uc0ac\uc704\uc600\ub358 \uc774\uc6a9\uc900\uc774 \ub9e4\uc6d4\ub2f9\uc9d1 \ub4f1\uc744 \ube44\ub86f\ud558\uc5ec \ud6c8\ubbfc\uc815\uc74c \ud574\ub840\ubcf8\uc744 \ubab0\ub798 \ube7c\ub3cc\ub824 \uc548\ub3d9\uc758 \uc790\ud0dd\uc5d0\uc11c \ubcf4\uad00\ud558\uace0 \uc788\ub2e4\uac00 \uae40\ud0dc\uc900\uc744 \ud1b5\ud574 \uc804\ud615\ud544 \uc120\uc0dd\uc5d0\uac8c \ud6c8\ubbfc\uc815\uc74c \ud574\ub840\ubcf8\ub9cc \ub9cc\uc6d0\uc5d0 \ud310\ub9e4\ud558\uc600\ub2e4. \ucd5c\ucd08 \ud310\ub9e4\uac00\ub85c \ucc9c\uc6d0\uc744 \uc81c\uc2dc\ud558\uc600\uc73c\ub098 \uc6d0\ub798 \ubb38\ud654\uc7ac\uc758 \uac00\uce58\ub97c \uc815\ud655\ud788 \uce58\ub974\ub294 \uac83\uc73c\ub85c \uc720\uba85\ud588\ub358 \uc804\ud615\ud544\uc740 \uae08\uc561\uc774 \ub108\ubb34 \uc801\ub2e4\uace0 \uc0dd\uac01\ud558\uc5ec \uac70\uac04 \ub178\ub987\uc744 \ud55c \uae40\ud0dc\uc900\uc5d0\uac8c \ucc9c\uc6d0\uc744, \uadf8\ub9ac\uace0 \ud6c8\ubbfc\uc815\uc74c \ud574\ub840\ubcf8\uc758 \uac00\uaca9\uc73c\ub85c \ub9cc\uc6d0\uc744 \uce58\ub800\ub2e4. \ub2f9\uc2dc \ucc9c\uc6d0\uc774\uba74 \uc88b\uc740 \uae30\uc640\uc9d1 \ud55c\ucc44\ub97c \uc0b4 \uc218 \uc788\uc5c8\ub2e4\uace0 \ud55c\ub2e4. \uc774\ud6c4 \ud559\uacc4\uc5d0 \ud6c8\ubbfc\uc815\uc74c \ud574\ub840\ubcf8\uc774 \ubc1c\uacac\ub418\uba74\uc11c \ud559\uc790\ub4e4 \uc0ac\uc774\uc5d0 \ub17c\ub780\uc774 \uc77c\uc5c8\ub358 \ud55c\uae00 \ucc3d\uc81c \uc6d0\ub9ac\uac00 \uc774 \ud6c8\ubbfc\uc815\uc74c \ud574\ub840\ubcf8\uc5d0 \uc758\ud574 \ubc1d\ud600\uc84c\ub2e4. \ud6c4\uc5d0 \uc774 \ud6c8\ubbfc\uc815\uc74c \ud574\ub840\ubcf8\uc740 1962\ub144 12\uc6d4 20\uc77c \ub300\ud55c\ubbfc\uad6d\uc758 \uad6d\ubcf4 \uc81c 70\ud638\ub85c \uc9c0\uc815\ub418\uc5c8\ub2e4. \uc774\ud6c4 1997\ub144 10\uc6d4\uc5d0 \ub300\ud55c\ubbfc\uad6d\uc758 \uc720\ub124\uc2a4\ucf54 \uc138\uacc4\uae30\ub85d\uc720\uc0b0 \uc73c\ub85c \ub4f1\uc7ac\ub418\uc5c8\uae30\ub3c4 \ud558\uc600\ub2e4. \uac70\uac04 \ub178\ub987\uc740 \ud55c \uae40\ud0dc\uc900\uacfc \uc774\uc6a9\uc900\uc740 \uc0ac\ud68c\uc8fc\uc758\uc790\ub85c, \ud310\ub9e4\ud55c \ub3c8\uc744 \uc0ac\ud68c\uc8fc\uc758 \uc6b4\ub3d9\uc5d0 \uc0ac\uc6a9\ud588\ub2e4\uace0 \uc804\ud574\uc9c0\uba70 \uad11\ubcf5 \ud6c4 \uae40\ud0dc\uc900\uc740 \uc77c\ub828\uc758 \uc0ac\uac74\uc73c\ub85c \uccb4\ud3ec\ub418\uc5b4 \uc0ac\ud615\ub418\uc5c8\uace0, \uc774\uc6a9\uc900\uc740 \uc6d4\ubd81\ud558\uc600\ub2e4\uace0 \ud55c\ub2e4.", "paragraph_answer": "\uadf8\ub7f0\ub370 \ub2f9\uc2dc \uc774 \uc9d1\uc548\uc758 \uc0ac\uc704\uc600\ub358 \uc774\uc6a9\uc900\uc774 \ub9e4\uc6d4\ub2f9\uc9d1 \ub4f1\uc744 \ube44\ub86f\ud558\uc5ec \ud6c8\ubbfc\uc815\uc74c \ud574\ub840\ubcf8\uc744 \ubab0\ub798 \ube7c\ub3cc\ub824 \uc548\ub3d9\uc758 \uc790\ud0dd\uc5d0\uc11c \ubcf4\uad00\ud558\uace0 \uc788\ub2e4\uac00 \uae40\ud0dc\uc900\uc744 \ud1b5\ud574 \uc804\ud615\ud544 \uc120\uc0dd\uc5d0\uac8c \ud6c8\ubbfc\uc815\uc74c \ud574\ub840\ubcf8\ub9cc \ub9cc\uc6d0\uc5d0 \ud310\ub9e4\ud558\uc600\ub2e4. \ucd5c\ucd08 \ud310\ub9e4\uac00\ub85c \ucc9c\uc6d0\uc744 \uc81c\uc2dc\ud558\uc600\uc73c\ub098 \uc6d0\ub798 \ubb38\ud654\uc7ac\uc758 \uac00\uce58\ub97c \uc815\ud655\ud788 \uce58\ub974\ub294 \uac83\uc73c\ub85c \uc720\uba85\ud588\ub358 \uc804\ud615\ud544\uc740 \uae08\uc561\uc774 \ub108\ubb34 \uc801\ub2e4\uace0 \uc0dd\uac01\ud558\uc5ec \uac70\uac04 \ub178\ub987\uc744 \ud55c \uae40\ud0dc\uc900\uc5d0\uac8c \ucc9c\uc6d0\uc744, \uadf8\ub9ac\uace0 \ud6c8\ubbfc\uc815\uc74c \ud574\ub840\ubcf8\uc758 \uac00\uaca9\uc73c\ub85c \ub9cc\uc6d0\uc744 \uce58\ub800\ub2e4. \ub2f9\uc2dc \ucc9c\uc6d0\uc774\uba74 \uc88b\uc740 \uae30\uc640\uc9d1 \ud55c\ucc44\ub97c \uc0b4 \uc218 \uc788\uc5c8\ub2e4\uace0 \ud55c\ub2e4. \uc774\ud6c4 \ud559\uacc4\uc5d0 \ud6c8\ubbfc\uc815\uc74c \ud574\ub840\ubcf8\uc774 \ubc1c\uacac\ub418\uba74\uc11c \ud559\uc790\ub4e4 \uc0ac\uc774\uc5d0 \ub17c\ub780\uc774 \uc77c\uc5c8\ub358 \ud55c\uae00 \ucc3d\uc81c \uc6d0\ub9ac\uac00 \uc774 \ud6c8\ubbfc\uc815\uc74c \ud574\ub840\ubcf8\uc5d0 \uc758\ud574 \ubc1d\ud600\uc84c\ub2e4. \ud6c4\uc5d0 \uc774 \ud6c8\ubbfc\uc815\uc74c \ud574\ub840\ubcf8\uc740 1962\ub144 12\uc6d4 20\uc77c \ub300\ud55c\ubbfc\uad6d\uc758 \uad6d\ubcf4 \uc81c 70\ud638\ub85c \uc9c0\uc815\ub418\uc5c8\ub2e4. \uc774\ud6c4 1997\ub144 10\uc6d4\uc5d0 \ub300\ud55c\ubbfc\uad6d\uc758 \uc720\ub124\uc2a4\ucf54 \uc138\uacc4\uae30\ub85d\uc720\uc0b0 \uc73c\ub85c \ub4f1\uc7ac\ub418\uc5c8\uae30\ub3c4 \ud558\uc600\ub2e4. \uac70\uac04 \ub178\ub987\uc740 \ud55c \uae40\ud0dc\uc900\uacfc \uc774\uc6a9\uc900\uc740 \uc0ac\ud68c\uc8fc\uc758\uc790\ub85c, \ud310\ub9e4\ud55c \ub3c8\uc744 \uc0ac\ud68c\uc8fc\uc758 \uc6b4\ub3d9\uc5d0 \uc0ac\uc6a9\ud588\ub2e4\uace0 \uc804\ud574\uc9c0\uba70 \uad11\ubcf5 \ud6c4 \uae40\ud0dc\uc900\uc740 \uc77c\ub828\uc758 \uc0ac\uac74\uc73c\ub85c \uccb4\ud3ec\ub418\uc5b4 \uc0ac\ud615\ub418\uc5c8\uace0, \uc774\uc6a9\uc900\uc740 \uc6d4\ubd81\ud558\uc600\ub2e4\uace0 \ud55c\ub2e4.", "sentence_answer": "\uc774\ud6c4 1997\ub144 10\uc6d4\uc5d0 \ub300\ud55c\ubbfc\uad6d\uc758 \uc720\ub124\uc2a4\ucf54 \uc138\uacc4\uae30\ub85d\uc720\uc0b0 \uc73c\ub85c \ub4f1\uc7ac\ub418\uc5c8\uae30\ub3c4 \ud558\uc600\ub2e4.", "paragraph_id": "6549956-1-1", "paragraph_question": "question: \ud6c8\ubbfc\uc815\uc74c \ud574\ub840\ubcf8\uc740 1997\ub144 10\uc6d4 \ubb34\uc5c7\uc73c\ub85c \ub4f1\uc7ac\ub418\uc5c8\ub294\uac00?, context: \uadf8\ub7f0\ub370 \ub2f9\uc2dc \uc774 \uc9d1\uc548\uc758 \uc0ac\uc704\uc600\ub358 \uc774\uc6a9\uc900\uc774 \ub9e4\uc6d4\ub2f9\uc9d1 \ub4f1\uc744 \ube44\ub86f\ud558\uc5ec \ud6c8\ubbfc\uc815\uc74c \ud574\ub840\ubcf8\uc744 \ubab0\ub798 \ube7c\ub3cc\ub824 \uc548\ub3d9\uc758 \uc790\ud0dd\uc5d0\uc11c \ubcf4\uad00\ud558\uace0 \uc788\ub2e4\uac00 \uae40\ud0dc\uc900\uc744 \ud1b5\ud574 \uc804\ud615\ud544 \uc120\uc0dd\uc5d0\uac8c \ud6c8\ubbfc\uc815\uc74c \ud574\ub840\ubcf8\ub9cc \ub9cc\uc6d0\uc5d0 \ud310\ub9e4\ud558\uc600\ub2e4. \ucd5c\ucd08 \ud310\ub9e4\uac00\ub85c \ucc9c\uc6d0\uc744 \uc81c\uc2dc\ud558\uc600\uc73c\ub098 \uc6d0\ub798 \ubb38\ud654\uc7ac\uc758 \uac00\uce58\ub97c \uc815\ud655\ud788 \uce58\ub974\ub294 \uac83\uc73c\ub85c \uc720\uba85\ud588\ub358 \uc804\ud615\ud544\uc740 \uae08\uc561\uc774 \ub108\ubb34 \uc801\ub2e4\uace0 \uc0dd\uac01\ud558\uc5ec \uac70\uac04 \ub178\ub987\uc744 \ud55c \uae40\ud0dc\uc900\uc5d0\uac8c \ucc9c\uc6d0\uc744, \uadf8\ub9ac\uace0 \ud6c8\ubbfc\uc815\uc74c \ud574\ub840\ubcf8\uc758 \uac00\uaca9\uc73c\ub85c \ub9cc\uc6d0\uc744 \uce58\ub800\ub2e4. \ub2f9\uc2dc \ucc9c\uc6d0\uc774\uba74 \uc88b\uc740 \uae30\uc640\uc9d1 \ud55c\ucc44\ub97c \uc0b4 \uc218 \uc788\uc5c8\ub2e4\uace0 \ud55c\ub2e4. \uc774\ud6c4 \ud559\uacc4\uc5d0 \ud6c8\ubbfc\uc815\uc74c \ud574\ub840\ubcf8\uc774 \ubc1c\uacac\ub418\uba74\uc11c \ud559\uc790\ub4e4 \uc0ac\uc774\uc5d0 \ub17c\ub780\uc774 \uc77c\uc5c8\ub358 \ud55c\uae00 \ucc3d\uc81c \uc6d0\ub9ac\uac00 \uc774 \ud6c8\ubbfc\uc815\uc74c \ud574\ub840\ubcf8\uc5d0 \uc758\ud574 \ubc1d\ud600\uc84c\ub2e4. \ud6c4\uc5d0 \uc774 \ud6c8\ubbfc\uc815\uc74c \ud574\ub840\ubcf8\uc740 1962\ub144 12\uc6d4 20\uc77c \ub300\ud55c\ubbfc\uad6d\uc758 \uad6d\ubcf4 \uc81c 70\ud638\ub85c \uc9c0\uc815\ub418\uc5c8\ub2e4. \uc774\ud6c4 1997\ub144 10\uc6d4\uc5d0 \ub300\ud55c\ubbfc\uad6d\uc758 \uc720\ub124\uc2a4\ucf54 \uc138\uacc4\uae30\ub85d\uc720\uc0b0\uc73c\ub85c \ub4f1\uc7ac\ub418\uc5c8\uae30\ub3c4 \ud558\uc600\ub2e4. \uac70\uac04 \ub178\ub987\uc740 \ud55c \uae40\ud0dc\uc900\uacfc \uc774\uc6a9\uc900\uc740 \uc0ac\ud68c\uc8fc\uc758\uc790\ub85c, \ud310\ub9e4\ud55c \ub3c8\uc744 \uc0ac\ud68c\uc8fc\uc758 \uc6b4\ub3d9\uc5d0 \uc0ac\uc6a9\ud588\ub2e4\uace0 \uc804\ud574\uc9c0\uba70 \uad11\ubcf5 \ud6c4 \uae40\ud0dc\uc900\uc740 \uc77c\ub828\uc758 \uc0ac\uac74\uc73c\ub85c \uccb4\ud3ec\ub418\uc5b4 \uc0ac\ud615\ub418\uc5c8\uace0, \uc774\uc6a9\uc900\uc740 \uc6d4\ubd81\ud558\uc600\ub2e4\uace0 \ud55c\ub2e4."} {"question": "\uae40\uae30\uc2dd\uc740 \uc790\uc2e0\uc758 \uc8fc\uc7a5 \uc774\ud6c4 \uba87 \uac1c\uc6d4\ub9cc\uc5d0 4\uac1c\uad6d \uc720\uba85\ud55c \uad00\uad11\uc9c0\ub85c \ub5a0\ub0ac\ub294\uac00?", "paragraph": "2014\ub144 10\uc6d4 8\uc77c \uad6d\uac10\uc5d0\uc11c \uae40\uae30\uc2dd \uc758\uc6d0\uc740 \uc548\uc138\uc601 \uacbd\uc81c\u318d\uc778\ubb38\uc0ac\ud68c\uc5f0\uad6c\ud68c \uc774\uc0ac\uc7a5\uc5d0\uac8c \"\uc5f4\ud758\uc5d0 4\uac1c\uad6d\uc744 \uac14\ub294\ub370 \uacf5\uc801\uc73c\ub85c \uc5f0\uc218\ub97c \uc704\ud574\uc11c \uae30\uad00\uc5d0\uc11c \uc18c\uc694\ud55c \uc2dc\uac04\uc774 \ub531 9\uc2dc\uac04\uc785\ub2c8\ub2e4.\u2026\ub9e4\ub144 1\uc5b5\uc529 \ub4e4\uc5ec \uac00\uc9c0\uace0 \ud574\uc678\uc5f0\uc218\ub77c\uace0 \ud558\uba74\uc11c \uc0ac\uc2e4\uc0c1 \uadf8\ub0e5 \ud574\uc678 \uad00\uad11 \uc5ec\ud589\uc744 40\uba85\uc529 \ubcf4\ub0b4\uace0 \uc788\uc5b4\uc694.\u2026\uc65c \uc774\ub807\uac8c \uc5f0\uad6c\uae30\uad00 \ubd84\ub4e4\uc740 \uc2a4\uc704\uc2a4\uc640 \ud504\ub791\uc2a4\uc640 \uc774\ud0c8\ub9ac\uc544\ub97c \uc88b\uc544\ud558\ub294\uc9c0 \ubaa8\ub974\uc9c0\ub9cc \ucf54\uc2a4\uac00 \uac70\uc758 \uc2a4\uc704\uc2a4, \ud504\ub791\uc2a4, \uc774\ud0c8\ub9ac\uc544\uc758 \uac00\uc7a5 \uc720\uba85\ud55c \uad00\uad11\uc9c0\ub9cc \uac00\uace0 \uc788\uc2b5\ub2c8\ub2e4.\"\ub77c\uace0 \ube44\ub09c\ud588\ub2e4. \ud558\uc9c0\ub9cc 7\uac1c\uc6d4 \ud6c4\uc778 2015\ub144 5\uc6d4 \uae40\uae30\uc2dd\uc740 \uc5f4\ud758\uc5d0 4\uac1c\uad6d \uc720\uba85\ud55c \uad00\uad11\uc9c0\ub85c \ub5a0\ub0ac\ub2e4. \uadf8\uc758 \ud45c\ud604\ub300\ub85c\uba74 '\uadf8\ub0e5 \ud574\uc678 \uad00\uad11 \uc5ec\ud589'\uc774\ub2e4. KIEP\uac00 \uc9c0\uc6d0\ud55c 9\ubc15 10\uc77c\uac04\uc758 \ubbf8\uad6d\u318d\uc720\ub7fd \ucd9c\uc7a5\uc774\uc5c8\ub2e4.", "answer": "7\uac1c\uc6d4", "sentence": "\ud558\uc9c0\ub9cc 7\uac1c\uc6d4 \ud6c4\uc778 2015\ub144 5\uc6d4 \uae40\uae30\uc2dd\uc740 \uc5f4\ud758\uc5d0 4\uac1c\uad6d \uc720\uba85\ud55c \uad00\uad11\uc9c0\ub85c \ub5a0\ub0ac\ub2e4.", "paragraph_sentence": "2014\ub144 10\uc6d4 8\uc77c \uad6d\uac10\uc5d0\uc11c \uae40\uae30\uc2dd \uc758\uc6d0\uc740 \uc548\uc138\uc601 \uacbd\uc81c\u318d\uc778\ubb38\uc0ac\ud68c\uc5f0\uad6c\ud68c \uc774\uc0ac\uc7a5\uc5d0\uac8c \"\uc5f4\ud758\uc5d0 4\uac1c\uad6d\uc744 \uac14\ub294\ub370 \uacf5\uc801\uc73c\ub85c \uc5f0\uc218\ub97c \uc704\ud574\uc11c \uae30\uad00\uc5d0\uc11c \uc18c\uc694\ud55c \uc2dc\uac04\uc774 \ub531 9\uc2dc\uac04\uc785\ub2c8\ub2e4.\u2026\ub9e4\ub144 1\uc5b5\uc529 \ub4e4\uc5ec \uac00\uc9c0\uace0 \ud574\uc678\uc5f0\uc218\ub77c\uace0 \ud558\uba74\uc11c \uc0ac\uc2e4\uc0c1 \uadf8\ub0e5 \ud574\uc678 \uad00\uad11 \uc5ec\ud589\uc744 40\uba85\uc529 \ubcf4\ub0b4\uace0 \uc788\uc5b4\uc694.\u2026\uc65c \uc774\ub807\uac8c \uc5f0\uad6c\uae30\uad00 \ubd84\ub4e4\uc740 \uc2a4\uc704\uc2a4\uc640 \ud504\ub791\uc2a4\uc640 \uc774\ud0c8\ub9ac\uc544\ub97c \uc88b\uc544\ud558\ub294\uc9c0 \ubaa8\ub974\uc9c0\ub9cc \ucf54\uc2a4\uac00 \uac70\uc758 \uc2a4\uc704\uc2a4, \ud504\ub791\uc2a4, \uc774\ud0c8\ub9ac\uc544\uc758 \uac00\uc7a5 \uc720\uba85\ud55c \uad00\uad11\uc9c0\ub9cc \uac00\uace0 \uc788\uc2b5\ub2c8\ub2e4. \"\ub77c\uace0 \ube44\ub09c\ud588\ub2e4. \ud558\uc9c0\ub9cc 7\uac1c\uc6d4 \ud6c4\uc778 2015\ub144 5\uc6d4 \uae40\uae30\uc2dd\uc740 \uc5f4\ud758\uc5d0 4\uac1c\uad6d \uc720\uba85\ud55c \uad00\uad11\uc9c0\ub85c \ub5a0\ub0ac\ub2e4. \uadf8\uc758 \ud45c\ud604\ub300\ub85c\uba74 '\uadf8\ub0e5 \ud574\uc678 \uad00\uad11 \uc5ec\ud589'\uc774\ub2e4. KIEP\uac00 \uc9c0\uc6d0\ud55c 9\ubc15 10\uc77c\uac04\uc758 \ubbf8\uad6d\u318d\uc720\ub7fd \ucd9c\uc7a5\uc774\uc5c8\ub2e4.", "paragraph_answer": "2014\ub144 10\uc6d4 8\uc77c \uad6d\uac10\uc5d0\uc11c \uae40\uae30\uc2dd \uc758\uc6d0\uc740 \uc548\uc138\uc601 \uacbd\uc81c\u318d\uc778\ubb38\uc0ac\ud68c\uc5f0\uad6c\ud68c \uc774\uc0ac\uc7a5\uc5d0\uac8c \"\uc5f4\ud758\uc5d0 4\uac1c\uad6d\uc744 \uac14\ub294\ub370 \uacf5\uc801\uc73c\ub85c \uc5f0\uc218\ub97c \uc704\ud574\uc11c \uae30\uad00\uc5d0\uc11c \uc18c\uc694\ud55c \uc2dc\uac04\uc774 \ub531 9\uc2dc\uac04\uc785\ub2c8\ub2e4.\u2026\ub9e4\ub144 1\uc5b5\uc529 \ub4e4\uc5ec \uac00\uc9c0\uace0 \ud574\uc678\uc5f0\uc218\ub77c\uace0 \ud558\uba74\uc11c \uc0ac\uc2e4\uc0c1 \uadf8\ub0e5 \ud574\uc678 \uad00\uad11 \uc5ec\ud589\uc744 40\uba85\uc529 \ubcf4\ub0b4\uace0 \uc788\uc5b4\uc694.\u2026\uc65c \uc774\ub807\uac8c \uc5f0\uad6c\uae30\uad00 \ubd84\ub4e4\uc740 \uc2a4\uc704\uc2a4\uc640 \ud504\ub791\uc2a4\uc640 \uc774\ud0c8\ub9ac\uc544\ub97c \uc88b\uc544\ud558\ub294\uc9c0 \ubaa8\ub974\uc9c0\ub9cc \ucf54\uc2a4\uac00 \uac70\uc758 \uc2a4\uc704\uc2a4, \ud504\ub791\uc2a4, \uc774\ud0c8\ub9ac\uc544\uc758 \uac00\uc7a5 \uc720\uba85\ud55c \uad00\uad11\uc9c0\ub9cc \uac00\uace0 \uc788\uc2b5\ub2c8\ub2e4.\"\ub77c\uace0 \ube44\ub09c\ud588\ub2e4. \ud558\uc9c0\ub9cc 7\uac1c\uc6d4 \ud6c4\uc778 2015\ub144 5\uc6d4 \uae40\uae30\uc2dd\uc740 \uc5f4\ud758\uc5d0 4\uac1c\uad6d \uc720\uba85\ud55c \uad00\uad11\uc9c0\ub85c \ub5a0\ub0ac\ub2e4. \uadf8\uc758 \ud45c\ud604\ub300\ub85c\uba74 '\uadf8\ub0e5 \ud574\uc678 \uad00\uad11 \uc5ec\ud589'\uc774\ub2e4. KIEP\uac00 \uc9c0\uc6d0\ud55c 9\ubc15 10\uc77c\uac04\uc758 \ubbf8\uad6d\u318d\uc720\ub7fd \ucd9c\uc7a5\uc774\uc5c8\ub2e4.", "sentence_answer": "\ud558\uc9c0\ub9cc 7\uac1c\uc6d4 \ud6c4\uc778 2015\ub144 5\uc6d4 \uae40\uae30\uc2dd\uc740 \uc5f4\ud758\uc5d0 4\uac1c\uad6d \uc720\uba85\ud55c \uad00\uad11\uc9c0\ub85c \ub5a0\ub0ac\ub2e4.", "paragraph_id": "6481286-1-1", "paragraph_question": "question: \uae40\uae30\uc2dd\uc740 \uc790\uc2e0\uc758 \uc8fc\uc7a5 \uc774\ud6c4 \uba87 \uac1c\uc6d4\ub9cc\uc5d0 4\uac1c\uad6d \uc720\uba85\ud55c \uad00\uad11\uc9c0\ub85c \ub5a0\ub0ac\ub294\uac00?, context: 2014\ub144 10\uc6d4 8\uc77c \uad6d\uac10\uc5d0\uc11c \uae40\uae30\uc2dd \uc758\uc6d0\uc740 \uc548\uc138\uc601 \uacbd\uc81c\u318d\uc778\ubb38\uc0ac\ud68c\uc5f0\uad6c\ud68c \uc774\uc0ac\uc7a5\uc5d0\uac8c \"\uc5f4\ud758\uc5d0 4\uac1c\uad6d\uc744 \uac14\ub294\ub370 \uacf5\uc801\uc73c\ub85c \uc5f0\uc218\ub97c \uc704\ud574\uc11c \uae30\uad00\uc5d0\uc11c \uc18c\uc694\ud55c \uc2dc\uac04\uc774 \ub531 9\uc2dc\uac04\uc785\ub2c8\ub2e4.\u2026\ub9e4\ub144 1\uc5b5\uc529 \ub4e4\uc5ec \uac00\uc9c0\uace0 \ud574\uc678\uc5f0\uc218\ub77c\uace0 \ud558\uba74\uc11c \uc0ac\uc2e4\uc0c1 \uadf8\ub0e5 \ud574\uc678 \uad00\uad11 \uc5ec\ud589\uc744 40\uba85\uc529 \ubcf4\ub0b4\uace0 \uc788\uc5b4\uc694.\u2026\uc65c \uc774\ub807\uac8c \uc5f0\uad6c\uae30\uad00 \ubd84\ub4e4\uc740 \uc2a4\uc704\uc2a4\uc640 \ud504\ub791\uc2a4\uc640 \uc774\ud0c8\ub9ac\uc544\ub97c \uc88b\uc544\ud558\ub294\uc9c0 \ubaa8\ub974\uc9c0\ub9cc \ucf54\uc2a4\uac00 \uac70\uc758 \uc2a4\uc704\uc2a4, \ud504\ub791\uc2a4, \uc774\ud0c8\ub9ac\uc544\uc758 \uac00\uc7a5 \uc720\uba85\ud55c \uad00\uad11\uc9c0\ub9cc \uac00\uace0 \uc788\uc2b5\ub2c8\ub2e4.\"\ub77c\uace0 \ube44\ub09c\ud588\ub2e4. \ud558\uc9c0\ub9cc 7\uac1c\uc6d4 \ud6c4\uc778 2015\ub144 5\uc6d4 \uae40\uae30\uc2dd\uc740 \uc5f4\ud758\uc5d0 4\uac1c\uad6d \uc720\uba85\ud55c \uad00\uad11\uc9c0\ub85c \ub5a0\ub0ac\ub2e4. \uadf8\uc758 \ud45c\ud604\ub300\ub85c\uba74 '\uadf8\ub0e5 \ud574\uc678 \uad00\uad11 \uc5ec\ud589'\uc774\ub2e4. KIEP\uac00 \uc9c0\uc6d0\ud55c 9\ubc15 10\uc77c\uac04\uc758 \ubbf8\uad6d\u318d\uc720\ub7fd \ucd9c\uc7a5\uc774\uc5c8\ub2e4."} {"question": "\uc18c\ub828\uad70\uc5d0 \uc758\ud574 1944\ub144 \ubc1c\uacac\ub41c \uccab \ubc88\uc9f8 \uc218\uc6a9\uc18c\ub294 \ubb34\uc5c7\uc778\uac00?", "paragraph": "\uccab \ubc88\uc9f8 \uc8fc\uc694 \uc218\uc6a9\uc18c\uc600\ub358 Majdanek \uc218\uc6a9\uc18c\ub294 \uc9c4\uaca9\ud558\ub294 \uc18c\ub828\uad70\uc5d0 \uc758\ud574 1944\ub144 7\uc6d4 23\uc77c\uc5d0 \ubc1c\uacac\ub418\uc5c8\ub2e4. Chelmno \uc218\uc6a9\uc18c\ub294 \uc18c\ub828\uad70\uc5d0 \uc758\ud574 1945\ub144 1\uc6d4 20\uc77c\uc5d0 \ud574\ubc29\ub418\uc5c8\uace0, \uc544\uc6b0\uc288\ube44\uce20 \uc218\uc6a9\uc18c \ub610\ud55c \uc18c\ub828\uad70\uc5d0 \uc758\ud574 1945\ub144 1\uc6d4 27\uc77c\uc5d0 \ud574\ubc29\ub418\uc5c8\ub2e4. Buchenwald \uc218\uc6a9\uc18c\ub294 \ubbf8\uad70\uc5d0 \uc758\ud574 4\uc6d4 11\uc77c, Bergen-Belsen \uc218\uc6a9\uc18c\ub294 \uc601\uad6d\uad70\uc5d0 \uc758\ud574 4\uc6d4 15\uc77c, Dachau \uc218\uc6a9\uc18c\ub294 \ubbf8\uad70\uc5d0 \uc758\ud574 4\uc6d4 29\uc77c, Ravensbr\u00fcck \uc218\uc6a9\uc18c\ub294 \uac19\uc740 \ub0a0 \uc18c\ub828\uad70\uc5d0 \uc758\ud574, Mauthausen \uc218\uc6a9\uc18c\ub294 \ubbf8\uad70\uc5d0 \uc758\ud574 5\uc6d4 5\uc77c, Theresienstadt \uc218\uc6a9\uc18c\ub294 \uc18c\ub828\uad70\uc5d0 \uc758\ud574 5\uc6d4 8\uc77c\uc5d0 \ud574\ubc29\ub418\uc5c8\ub2e4. Treblinka, Sobibor, Belzec \uc218\uc6a9\uc18c\ub294 \ud574\ubc29\ub418\uc9c0 \ubabb\ud558\uace0 1943\ub144\uc5d0 \ub098\uce58\uc5d0 \uc758\ud574 \ud30c\uad34\ub418\uc5c8\ub2e4. \ubbf87\uad70 William W. Quinn \ub300\ub839\uc740 Dachau \uc218\uc6a9\uc18c\uc5d0 \ub300\ud574 \u201c\uc6b0\ub9ac \ubd80\ub300\ub294 \uadf8 \uacf3\uc5d0\uc11c \ubbff\uae30 \ud798\ub4e4 \uc815\ub3c4\ub85c \ub054\ucc0d\ud55c \uad11\uacbd, \uc18c\ub9ac, \uc545\ucde8\ub97c \ubaa9\uaca9\ud558\uc600\ub2e4. \ub3c4\uc800\ud788 \uc77c\ubc18\uc801\uc778 \uc0c1\uc2dd\uc73c\ub85c\ub294 \uc774\ud574\ud560 \uc218 \uc5c6\ub294 \uc5c4\uccad\ub09c \uc794\uc778\ud568\uc774\uc5c8\ub2e4\u201d\ub77c\uace0 \ud68c\uace0\ud55c\ub2e4.", "answer": "Majdanek \uc218\uc6a9\uc18c", "sentence": "\uccab \ubc88\uc9f8 \uc8fc\uc694 \uc218\uc6a9\uc18c\uc600\ub358 Majdanek \uc218\uc6a9\uc18c \ub294 \uc9c4\uaca9\ud558\ub294 \uc18c\ub828\uad70\uc5d0 \uc758\ud574 1944\ub144 7\uc6d4 23\uc77c\uc5d0 \ubc1c\uacac\ub418\uc5c8\ub2e4.", "paragraph_sentence": " \uccab \ubc88\uc9f8 \uc8fc\uc694 \uc218\uc6a9\uc18c\uc600\ub358 Majdanek \uc218\uc6a9\uc18c \ub294 \uc9c4\uaca9\ud558\ub294 \uc18c\ub828\uad70\uc5d0 \uc758\ud574 1944\ub144 7\uc6d4 23\uc77c\uc5d0 \ubc1c\uacac\ub418\uc5c8\ub2e4. Chelmno \uc218\uc6a9\uc18c\ub294 \uc18c\ub828\uad70\uc5d0 \uc758\ud574 1945\ub144 1\uc6d4 20\uc77c\uc5d0 \ud574\ubc29\ub418\uc5c8\uace0, \uc544\uc6b0\uc288\ube44\uce20 \uc218\uc6a9\uc18c \ub610\ud55c \uc18c\ub828\uad70\uc5d0 \uc758\ud574 1945\ub144 1\uc6d4 27\uc77c\uc5d0 \ud574\ubc29\ub418\uc5c8\ub2e4. Buchenwald \uc218\uc6a9\uc18c\ub294 \ubbf8\uad70\uc5d0 \uc758\ud574 4\uc6d4 11\uc77c, Bergen-Belsen \uc218\uc6a9\uc18c\ub294 \uc601\uad6d\uad70\uc5d0 \uc758\ud574 4\uc6d4 15\uc77c, Dachau \uc218\uc6a9\uc18c\ub294 \ubbf8\uad70\uc5d0 \uc758\ud574 4\uc6d4 29\uc77c, Ravensbr\u00fcck \uc218\uc6a9\uc18c\ub294 \uac19\uc740 \ub0a0 \uc18c\ub828\uad70\uc5d0 \uc758\ud574, Mauthausen \uc218\uc6a9\uc18c\ub294 \ubbf8\uad70\uc5d0 \uc758\ud574 5\uc6d4 5\uc77c, Theresienstadt \uc218\uc6a9\uc18c\ub294 \uc18c\ub828\uad70\uc5d0 \uc758\ud574 5\uc6d4 8\uc77c\uc5d0 \ud574\ubc29\ub418\uc5c8\ub2e4. Treblinka, Sobibor, Belzec \uc218\uc6a9\uc18c\ub294 \ud574\ubc29\ub418\uc9c0 \ubabb\ud558\uace0 1943\ub144\uc5d0 \ub098\uce58\uc5d0 \uc758\ud574 \ud30c\uad34\ub418\uc5c8\ub2e4. \ubbf87\uad70 William W. Quinn \ub300\ub839\uc740 Dachau \uc218\uc6a9\uc18c\uc5d0 \ub300\ud574 \u201c\uc6b0\ub9ac \ubd80\ub300\ub294 \uadf8 \uacf3\uc5d0\uc11c \ubbff\uae30 \ud798\ub4e4 \uc815\ub3c4\ub85c \ub054\ucc0d\ud55c \uad11\uacbd, \uc18c\ub9ac, \uc545\ucde8\ub97c \ubaa9\uaca9\ud558\uc600\ub2e4. \ub3c4\uc800\ud788 \uc77c\ubc18\uc801\uc778 \uc0c1\uc2dd\uc73c\ub85c\ub294 \uc774\ud574\ud560 \uc218 \uc5c6\ub294 \uc5c4\uccad\ub09c \uc794\uc778\ud568\uc774\uc5c8\ub2e4\u201d\ub77c\uace0 \ud68c\uace0\ud55c\ub2e4.", "paragraph_answer": "\uccab \ubc88\uc9f8 \uc8fc\uc694 \uc218\uc6a9\uc18c\uc600\ub358 Majdanek \uc218\uc6a9\uc18c \ub294 \uc9c4\uaca9\ud558\ub294 \uc18c\ub828\uad70\uc5d0 \uc758\ud574 1944\ub144 7\uc6d4 23\uc77c\uc5d0 \ubc1c\uacac\ub418\uc5c8\ub2e4. Chelmno \uc218\uc6a9\uc18c\ub294 \uc18c\ub828\uad70\uc5d0 \uc758\ud574 1945\ub144 1\uc6d4 20\uc77c\uc5d0 \ud574\ubc29\ub418\uc5c8\uace0, \uc544\uc6b0\uc288\ube44\uce20 \uc218\uc6a9\uc18c \ub610\ud55c \uc18c\ub828\uad70\uc5d0 \uc758\ud574 1945\ub144 1\uc6d4 27\uc77c\uc5d0 \ud574\ubc29\ub418\uc5c8\ub2e4. Buchenwald \uc218\uc6a9\uc18c\ub294 \ubbf8\uad70\uc5d0 \uc758\ud574 4\uc6d4 11\uc77c, Bergen-Belsen \uc218\uc6a9\uc18c\ub294 \uc601\uad6d\uad70\uc5d0 \uc758\ud574 4\uc6d4 15\uc77c, Dachau \uc218\uc6a9\uc18c\ub294 \ubbf8\uad70\uc5d0 \uc758\ud574 4\uc6d4 29\uc77c, Ravensbr\u00fcck \uc218\uc6a9\uc18c\ub294 \uac19\uc740 \ub0a0 \uc18c\ub828\uad70\uc5d0 \uc758\ud574, Mauthausen \uc218\uc6a9\uc18c\ub294 \ubbf8\uad70\uc5d0 \uc758\ud574 5\uc6d4 5\uc77c, Theresienstadt \uc218\uc6a9\uc18c\ub294 \uc18c\ub828\uad70\uc5d0 \uc758\ud574 5\uc6d4 8\uc77c\uc5d0 \ud574\ubc29\ub418\uc5c8\ub2e4. Treblinka, Sobibor, Belzec \uc218\uc6a9\uc18c\ub294 \ud574\ubc29\ub418\uc9c0 \ubabb\ud558\uace0 1943\ub144\uc5d0 \ub098\uce58\uc5d0 \uc758\ud574 \ud30c\uad34\ub418\uc5c8\ub2e4. \ubbf87\uad70 William W. Quinn \ub300\ub839\uc740 Dachau \uc218\uc6a9\uc18c\uc5d0 \ub300\ud574 \u201c\uc6b0\ub9ac \ubd80\ub300\ub294 \uadf8 \uacf3\uc5d0\uc11c \ubbff\uae30 \ud798\ub4e4 \uc815\ub3c4\ub85c \ub054\ucc0d\ud55c \uad11\uacbd, \uc18c\ub9ac, \uc545\ucde8\ub97c \ubaa9\uaca9\ud558\uc600\ub2e4. \ub3c4\uc800\ud788 \uc77c\ubc18\uc801\uc778 \uc0c1\uc2dd\uc73c\ub85c\ub294 \uc774\ud574\ud560 \uc218 \uc5c6\ub294 \uc5c4\uccad\ub09c \uc794\uc778\ud568\uc774\uc5c8\ub2e4\u201d\ub77c\uace0 \ud68c\uace0\ud55c\ub2e4.", "sentence_answer": "\uccab \ubc88\uc9f8 \uc8fc\uc694 \uc218\uc6a9\uc18c\uc600\ub358 Majdanek \uc218\uc6a9\uc18c \ub294 \uc9c4\uaca9\ud558\ub294 \uc18c\ub828\uad70\uc5d0 \uc758\ud574 1944\ub144 7\uc6d4 23\uc77c\uc5d0 \ubc1c\uacac\ub418\uc5c8\ub2e4.", "paragraph_id": "6571960-30-0", "paragraph_question": "question: \uc18c\ub828\uad70\uc5d0 \uc758\ud574 1944\ub144 \ubc1c\uacac\ub41c \uccab \ubc88\uc9f8 \uc218\uc6a9\uc18c\ub294 \ubb34\uc5c7\uc778\uac00?, context: \uccab \ubc88\uc9f8 \uc8fc\uc694 \uc218\uc6a9\uc18c\uc600\ub358 Majdanek \uc218\uc6a9\uc18c\ub294 \uc9c4\uaca9\ud558\ub294 \uc18c\ub828\uad70\uc5d0 \uc758\ud574 1944\ub144 7\uc6d4 23\uc77c\uc5d0 \ubc1c\uacac\ub418\uc5c8\ub2e4. Chelmno \uc218\uc6a9\uc18c\ub294 \uc18c\ub828\uad70\uc5d0 \uc758\ud574 1945\ub144 1\uc6d4 20\uc77c\uc5d0 \ud574\ubc29\ub418\uc5c8\uace0, \uc544\uc6b0\uc288\ube44\uce20 \uc218\uc6a9\uc18c \ub610\ud55c \uc18c\ub828\uad70\uc5d0 \uc758\ud574 1945\ub144 1\uc6d4 27\uc77c\uc5d0 \ud574\ubc29\ub418\uc5c8\ub2e4. Buchenwald \uc218\uc6a9\uc18c\ub294 \ubbf8\uad70\uc5d0 \uc758\ud574 4\uc6d4 11\uc77c, Bergen-Belsen \uc218\uc6a9\uc18c\ub294 \uc601\uad6d\uad70\uc5d0 \uc758\ud574 4\uc6d4 15\uc77c, Dachau \uc218\uc6a9\uc18c\ub294 \ubbf8\uad70\uc5d0 \uc758\ud574 4\uc6d4 29\uc77c, Ravensbr\u00fcck \uc218\uc6a9\uc18c\ub294 \uac19\uc740 \ub0a0 \uc18c\ub828\uad70\uc5d0 \uc758\ud574, Mauthausen \uc218\uc6a9\uc18c\ub294 \ubbf8\uad70\uc5d0 \uc758\ud574 5\uc6d4 5\uc77c, Theresienstadt \uc218\uc6a9\uc18c\ub294 \uc18c\ub828\uad70\uc5d0 \uc758\ud574 5\uc6d4 8\uc77c\uc5d0 \ud574\ubc29\ub418\uc5c8\ub2e4. Treblinka, Sobibor, Belzec \uc218\uc6a9\uc18c\ub294 \ud574\ubc29\ub418\uc9c0 \ubabb\ud558\uace0 1943\ub144\uc5d0 \ub098\uce58\uc5d0 \uc758\ud574 \ud30c\uad34\ub418\uc5c8\ub2e4. \ubbf87\uad70 William W. Quinn \ub300\ub839\uc740 Dachau \uc218\uc6a9\uc18c\uc5d0 \ub300\ud574 \u201c\uc6b0\ub9ac \ubd80\ub300\ub294 \uadf8 \uacf3\uc5d0\uc11c \ubbff\uae30 \ud798\ub4e4 \uc815\ub3c4\ub85c \ub054\ucc0d\ud55c \uad11\uacbd, \uc18c\ub9ac, \uc545\ucde8\ub97c \ubaa9\uaca9\ud558\uc600\ub2e4. \ub3c4\uc800\ud788 \uc77c\ubc18\uc801\uc778 \uc0c1\uc2dd\uc73c\ub85c\ub294 \uc774\ud574\ud560 \uc218 \uc5c6\ub294 \uc5c4\uccad\ub09c \uc794\uc778\ud568\uc774\uc5c8\ub2e4\u201d\ub77c\uace0 \ud68c\uace0\ud55c\ub2e4."} {"question": "\ub458\uc9f8 \ubd80\ubd84\uc5d0\uc11c \uc815\uc131\uc2a4\ub808 \uc74c\uc2dd\uc744 \uc62c\ub9ac\ub294 \uac83\uc744 \uc9c4\ucc2c\uc774\ub77c\uace0 \ud558\uba74, \uc220\uc744 \ubc14\uce58\ub294 \uac83\uc740 \ubb34\uc5c7\uc774\ub77c \ud558\ub294\uac00?", "paragraph": "\uc720\uad50 \uc870\uc0c1\uc81c\uc0ac\uc5d0\ub294 \uc0ac\ub2f9\uc81c(\u7960\u5802\u796d), \uc774\uc81c(\u723e\u796d), \uae30\uc81c(\u5fcc\u796d) \ub4f1\uc774 \uc788\ub294\ub370 \ud615\uc2dd\uc0c1 \ub2e4\uc18c \ucc28\uc774\ub294 \uc788\uc73c\ub098 \ub300\uccb4\ub85c 4\ubd80\ubd84\uc73c\ub85c \uad6c\uc131\ub3fc \uc788\ub2e4. \uccab\uc9f8 \ubd80\ubd84\uc740 \ub9c8\uc74c\uc744 \uc9d1\uc911\uc2dc\ud0a4\uace0 \uc2e0\ub839\uc758 \uc784\uc7ac(\u81e8\u9f4b)\ub97c \uc900\ube44\ud558\ub294 \ub2e8\uacc4\ub85c, \uc81c\uc0ac \uc804 \ub9c8\uc74c\uc744 \ubaa8\uc73c\ub294 \uc81c\uacc4(\u796d\u6212), \uc74c\uc2dd\uc744 \ucc28\ub824\ub193\ub294 \uc9c4\uc124(\u9673\u8a2d), \uc2e0\ub839\uc774 \uc784\uc7ac\ud558\uac8c \ud558\ub294 \uac15\uc2e0(\u964d\u795e) \ub4f1\uc774 \uc788\ub2e4. \ub458\uc9f8 \ubd80\ubd84\uc5d0\uc120 \ud6a8\uc131\uc758 \uc0c1\uc9d5\uc801 \ud45c\ud604\uc778 \uc81c\ubb3c\uc744 \ub4dc\ub9ac\uba74\uc11c \ud760\ud5a5\uc744 \uac04\uccad\ud55c\ub2e4. \uc5ec\uae30\uc5d0\ub294 \uc0dd\uc2dc\uc640 \uac19\uc774 \uc815\uc131\uc2a4\ub7fd\uac8c \uc74c\uc2dd\uc744 \uc62c\ub9ac\ub294 \uc9c4\ucc2c(\u9032\u994c)\uacfc \uc220\uc744 \ubc14\uce58\ub294 \ud5cc\uc791(\u737b\u7235) \ub4f1\uc774 \uc788\ub2e4. \uc14b\uc9f8 \ubd80\ubd84\uc740 \uc2e0\ub839\uc774 \uc81c\uc0ac\ub97c \ud760\ud5a5\ud558\uace0 \uac15\ubcf5\ud558\ub294 \uc758\uc2dd\uc774\ub2e4. \uc2e0\ub839\uc774 \ud760\ud5a5\ud558\ub3c4\ub85d \ubb38\uc744 \ub2eb\ub294 \ud569\ubb38(\u95d4\u9580)\uacfc \ub2e4\uc2dc \ub4e4\uc5b4\uac00\uc11c \ucc28\ub098 \uc22d\ub289\uc744 \ub4dc\ub9ac\ub294 \ud5cc\ub2e4(\u737b\u8336)\uc640 \uc81c\ubb3c\uc758 \uc77c\ubd80\ub97c \uc81c\uc8fc(\u796d\u4e3b)\uc5d0\uac8c \uba39\ub3c4\ub85d \ud558\ub294 \uc218\uc791(\u53d7\u6628), \uc2e0\ub839\uc758 \ud760\ud5a5\uc774 \ub05d\ub0ac\uc74c\uc744 \uc54c\ub9ac\ub294 \uc774\uc131(\u5229\u6210) \ub4f1\uc774 \uc788\ub2e4. \ub9c8\uc9c0\ub9c9 \ub137\uc9f8 \ubd80\ubd84\uc740 \uc2e0\ub839\uc5d0 \ub4dc\ub9ac\ub294 \uc758\uc2dd\uc744 \ub05d\ub0b4\ub294 \ub9c8\ubb34\ub9ac \uc758\uc2dd\uc73c\ub85c \uc791\ubcc4\uc778\uc0ac\ub97c \uc62c\ub9ac\ub294 \uc0ac\uc2e0(\u8fad\u795e)\uacfc \uc11c\ub85c \ucd95\ubcf5\ud558\uba74\uc11c \uc81c\ubb3c\uc744 \ub098\ub204\uc5b4\uba39\ub294 \uc74c\ubcf5(\u98ee\u798f) \ub4f1\uc774 \uc788\ub2e4.", "answer": "\ud5cc\uc791", "sentence": "\uc5ec\uae30\uc5d0\ub294 \uc0dd\uc2dc\uc640 \uac19\uc774 \uc815\uc131\uc2a4\ub7fd\uac8c \uc74c\uc2dd\uc744 \uc62c\ub9ac\ub294 \uc9c4\ucc2c(\u9032\u994c)\uacfc \uc220\uc744 \ubc14\uce58\ub294 \ud5cc\uc791 (\u737b\u7235)", "paragraph_sentence": "\uc720\uad50 \uc870\uc0c1\uc81c\uc0ac\uc5d0\ub294 \uc0ac\ub2f9\uc81c(\u7960\u5802\u796d), \uc774\uc81c(\u723e\u796d), \uae30\uc81c(\u5fcc\u796d) \ub4f1\uc774 \uc788\ub294\ub370 \ud615\uc2dd\uc0c1 \ub2e4\uc18c \ucc28\uc774\ub294 \uc788\uc73c\ub098 \ub300\uccb4\ub85c 4\ubd80\ubd84\uc73c\ub85c \uad6c\uc131\ub3fc \uc788\ub2e4. \uccab\uc9f8 \ubd80\ubd84\uc740 \ub9c8\uc74c\uc744 \uc9d1\uc911\uc2dc\ud0a4\uace0 \uc2e0\ub839\uc758 \uc784\uc7ac(\u81e8\u9f4b)\ub97c \uc900\ube44\ud558\ub294 \ub2e8\uacc4\ub85c, \uc81c\uc0ac \uc804 \ub9c8\uc74c\uc744 \ubaa8\uc73c\ub294 \uc81c\uacc4(\u796d\u6212), \uc74c\uc2dd\uc744 \ucc28\ub824\ub193\ub294 \uc9c4\uc124(\u9673\u8a2d), \uc2e0\ub839\uc774 \uc784\uc7ac\ud558\uac8c \ud558\ub294 \uac15\uc2e0(\u964d\u795e) \ub4f1\uc774 \uc788\ub2e4. \ub458\uc9f8 \ubd80\ubd84\uc5d0\uc120 \ud6a8\uc131\uc758 \uc0c1\uc9d5\uc801 \ud45c\ud604\uc778 \uc81c\ubb3c\uc744 \ub4dc\ub9ac\uba74\uc11c \ud760\ud5a5\uc744 \uac04\uccad\ud55c\ub2e4. \uc5ec\uae30\uc5d0\ub294 \uc0dd\uc2dc\uc640 \uac19\uc774 \uc815\uc131\uc2a4\ub7fd\uac8c \uc74c\uc2dd\uc744 \uc62c\ub9ac\ub294 \uc9c4\ucc2c(\u9032\u994c)\uacfc \uc220\uc744 \ubc14\uce58\ub294 \ud5cc\uc791 (\u737b\u7235) \ub4f1\uc774 \uc788\ub2e4. \uc14b\uc9f8 \ubd80\ubd84\uc740 \uc2e0\ub839\uc774 \uc81c\uc0ac\ub97c \ud760\ud5a5\ud558\uace0 \uac15\ubcf5\ud558\ub294 \uc758\uc2dd\uc774\ub2e4. \uc2e0\ub839\uc774 \ud760\ud5a5\ud558\ub3c4\ub85d \ubb38\uc744 \ub2eb\ub294 \ud569\ubb38(\u95d4\u9580)\uacfc \ub2e4\uc2dc \ub4e4\uc5b4\uac00\uc11c \ucc28\ub098 \uc22d\ub289\uc744 \ub4dc\ub9ac\ub294 \ud5cc\ub2e4(\u737b\u8336)\uc640 \uc81c\ubb3c\uc758 \uc77c\ubd80\ub97c \uc81c\uc8fc(\u796d\u4e3b)\uc5d0\uac8c \uba39\ub3c4\ub85d \ud558\ub294 \uc218\uc791(\u53d7\u6628), \uc2e0\ub839\uc758 \ud760\ud5a5\uc774 \ub05d\ub0ac\uc74c\uc744 \uc54c\ub9ac\ub294 \uc774\uc131(\u5229\u6210) \ub4f1\uc774 \uc788\ub2e4. \ub9c8\uc9c0\ub9c9 \ub137\uc9f8 \ubd80\ubd84\uc740 \uc2e0\ub839\uc5d0 \ub4dc\ub9ac\ub294 \uc758\uc2dd\uc744 \ub05d\ub0b4\ub294 \ub9c8\ubb34\ub9ac \uc758\uc2dd\uc73c\ub85c \uc791\ubcc4\uc778\uc0ac\ub97c \uc62c\ub9ac\ub294 \uc0ac\uc2e0(\u8fad\u795e)\uacfc \uc11c\ub85c \ucd95\ubcf5\ud558\uba74\uc11c \uc81c\ubb3c\uc744 \ub098\ub204\uc5b4\uba39\ub294 \uc74c\ubcf5(\u98ee\u798f) \ub4f1\uc774 \uc788\ub2e4.", "paragraph_answer": "\uc720\uad50 \uc870\uc0c1\uc81c\uc0ac\uc5d0\ub294 \uc0ac\ub2f9\uc81c(\u7960\u5802\u796d), \uc774\uc81c(\u723e\u796d), \uae30\uc81c(\u5fcc\u796d) \ub4f1\uc774 \uc788\ub294\ub370 \ud615\uc2dd\uc0c1 \ub2e4\uc18c \ucc28\uc774\ub294 \uc788\uc73c\ub098 \ub300\uccb4\ub85c 4\ubd80\ubd84\uc73c\ub85c \uad6c\uc131\ub3fc \uc788\ub2e4. \uccab\uc9f8 \ubd80\ubd84\uc740 \ub9c8\uc74c\uc744 \uc9d1\uc911\uc2dc\ud0a4\uace0 \uc2e0\ub839\uc758 \uc784\uc7ac(\u81e8\u9f4b)\ub97c \uc900\ube44\ud558\ub294 \ub2e8\uacc4\ub85c, \uc81c\uc0ac \uc804 \ub9c8\uc74c\uc744 \ubaa8\uc73c\ub294 \uc81c\uacc4(\u796d\u6212), \uc74c\uc2dd\uc744 \ucc28\ub824\ub193\ub294 \uc9c4\uc124(\u9673\u8a2d), \uc2e0\ub839\uc774 \uc784\uc7ac\ud558\uac8c \ud558\ub294 \uac15\uc2e0(\u964d\u795e) \ub4f1\uc774 \uc788\ub2e4. \ub458\uc9f8 \ubd80\ubd84\uc5d0\uc120 \ud6a8\uc131\uc758 \uc0c1\uc9d5\uc801 \ud45c\ud604\uc778 \uc81c\ubb3c\uc744 \ub4dc\ub9ac\uba74\uc11c \ud760\ud5a5\uc744 \uac04\uccad\ud55c\ub2e4. \uc5ec\uae30\uc5d0\ub294 \uc0dd\uc2dc\uc640 \uac19\uc774 \uc815\uc131\uc2a4\ub7fd\uac8c \uc74c\uc2dd\uc744 \uc62c\ub9ac\ub294 \uc9c4\ucc2c(\u9032\u994c)\uacfc \uc220\uc744 \ubc14\uce58\ub294 \ud5cc\uc791 (\u737b\u7235) \ub4f1\uc774 \uc788\ub2e4. \uc14b\uc9f8 \ubd80\ubd84\uc740 \uc2e0\ub839\uc774 \uc81c\uc0ac\ub97c \ud760\ud5a5\ud558\uace0 \uac15\ubcf5\ud558\ub294 \uc758\uc2dd\uc774\ub2e4. \uc2e0\ub839\uc774 \ud760\ud5a5\ud558\ub3c4\ub85d \ubb38\uc744 \ub2eb\ub294 \ud569\ubb38(\u95d4\u9580)\uacfc \ub2e4\uc2dc \ub4e4\uc5b4\uac00\uc11c \ucc28\ub098 \uc22d\ub289\uc744 \ub4dc\ub9ac\ub294 \ud5cc\ub2e4(\u737b\u8336)\uc640 \uc81c\ubb3c\uc758 \uc77c\ubd80\ub97c \uc81c\uc8fc(\u796d\u4e3b)\uc5d0\uac8c \uba39\ub3c4\ub85d \ud558\ub294 \uc218\uc791(\u53d7\u6628), \uc2e0\ub839\uc758 \ud760\ud5a5\uc774 \ub05d\ub0ac\uc74c\uc744 \uc54c\ub9ac\ub294 \uc774\uc131(\u5229\u6210) \ub4f1\uc774 \uc788\ub2e4. \ub9c8\uc9c0\ub9c9 \ub137\uc9f8 \ubd80\ubd84\uc740 \uc2e0\ub839\uc5d0 \ub4dc\ub9ac\ub294 \uc758\uc2dd\uc744 \ub05d\ub0b4\ub294 \ub9c8\ubb34\ub9ac \uc758\uc2dd\uc73c\ub85c \uc791\ubcc4\uc778\uc0ac\ub97c \uc62c\ub9ac\ub294 \uc0ac\uc2e0(\u8fad\u795e)\uacfc \uc11c\ub85c \ucd95\ubcf5\ud558\uba74\uc11c \uc81c\ubb3c\uc744 \ub098\ub204\uc5b4\uba39\ub294 \uc74c\ubcf5(\u98ee\u798f) \ub4f1\uc774 \uc788\ub2e4.", "sentence_answer": "\uc5ec\uae30\uc5d0\ub294 \uc0dd\uc2dc\uc640 \uac19\uc774 \uc815\uc131\uc2a4\ub7fd\uac8c \uc74c\uc2dd\uc744 \uc62c\ub9ac\ub294 \uc9c4\ucc2c(\u9032\u994c)\uacfc \uc220\uc744 \ubc14\uce58\ub294 \ud5cc\uc791 (\u737b\u7235)", "paragraph_id": "6560473-1-1", "paragraph_question": "question: \ub458\uc9f8 \ubd80\ubd84\uc5d0\uc11c \uc815\uc131\uc2a4\ub808 \uc74c\uc2dd\uc744 \uc62c\ub9ac\ub294 \uac83\uc744 \uc9c4\ucc2c\uc774\ub77c\uace0 \ud558\uba74, \uc220\uc744 \ubc14\uce58\ub294 \uac83\uc740 \ubb34\uc5c7\uc774\ub77c \ud558\ub294\uac00?, context: \uc720\uad50 \uc870\uc0c1\uc81c\uc0ac\uc5d0\ub294 \uc0ac\ub2f9\uc81c(\u7960\u5802\u796d), \uc774\uc81c(\u723e\u796d), \uae30\uc81c(\u5fcc\u796d) \ub4f1\uc774 \uc788\ub294\ub370 \ud615\uc2dd\uc0c1 \ub2e4\uc18c \ucc28\uc774\ub294 \uc788\uc73c\ub098 \ub300\uccb4\ub85c 4\ubd80\ubd84\uc73c\ub85c \uad6c\uc131\ub3fc \uc788\ub2e4. \uccab\uc9f8 \ubd80\ubd84\uc740 \ub9c8\uc74c\uc744 \uc9d1\uc911\uc2dc\ud0a4\uace0 \uc2e0\ub839\uc758 \uc784\uc7ac(\u81e8\u9f4b)\ub97c \uc900\ube44\ud558\ub294 \ub2e8\uacc4\ub85c, \uc81c\uc0ac \uc804 \ub9c8\uc74c\uc744 \ubaa8\uc73c\ub294 \uc81c\uacc4(\u796d\u6212), \uc74c\uc2dd\uc744 \ucc28\ub824\ub193\ub294 \uc9c4\uc124(\u9673\u8a2d), \uc2e0\ub839\uc774 \uc784\uc7ac\ud558\uac8c \ud558\ub294 \uac15\uc2e0(\u964d\u795e) \ub4f1\uc774 \uc788\ub2e4. \ub458\uc9f8 \ubd80\ubd84\uc5d0\uc120 \ud6a8\uc131\uc758 \uc0c1\uc9d5\uc801 \ud45c\ud604\uc778 \uc81c\ubb3c\uc744 \ub4dc\ub9ac\uba74\uc11c \ud760\ud5a5\uc744 \uac04\uccad\ud55c\ub2e4. \uc5ec\uae30\uc5d0\ub294 \uc0dd\uc2dc\uc640 \uac19\uc774 \uc815\uc131\uc2a4\ub7fd\uac8c \uc74c\uc2dd\uc744 \uc62c\ub9ac\ub294 \uc9c4\ucc2c(\u9032\u994c)\uacfc \uc220\uc744 \ubc14\uce58\ub294 \ud5cc\uc791(\u737b\u7235) \ub4f1\uc774 \uc788\ub2e4. \uc14b\uc9f8 \ubd80\ubd84\uc740 \uc2e0\ub839\uc774 \uc81c\uc0ac\ub97c \ud760\ud5a5\ud558\uace0 \uac15\ubcf5\ud558\ub294 \uc758\uc2dd\uc774\ub2e4. \uc2e0\ub839\uc774 \ud760\ud5a5\ud558\ub3c4\ub85d \ubb38\uc744 \ub2eb\ub294 \ud569\ubb38(\u95d4\u9580)\uacfc \ub2e4\uc2dc \ub4e4\uc5b4\uac00\uc11c \ucc28\ub098 \uc22d\ub289\uc744 \ub4dc\ub9ac\ub294 \ud5cc\ub2e4(\u737b\u8336)\uc640 \uc81c\ubb3c\uc758 \uc77c\ubd80\ub97c \uc81c\uc8fc(\u796d\u4e3b)\uc5d0\uac8c \uba39\ub3c4\ub85d \ud558\ub294 \uc218\uc791(\u53d7\u6628), \uc2e0\ub839\uc758 \ud760\ud5a5\uc774 \ub05d\ub0ac\uc74c\uc744 \uc54c\ub9ac\ub294 \uc774\uc131(\u5229\u6210) \ub4f1\uc774 \uc788\ub2e4. \ub9c8\uc9c0\ub9c9 \ub137\uc9f8 \ubd80\ubd84\uc740 \uc2e0\ub839\uc5d0 \ub4dc\ub9ac\ub294 \uc758\uc2dd\uc744 \ub05d\ub0b4\ub294 \ub9c8\ubb34\ub9ac \uc758\uc2dd\uc73c\ub85c \uc791\ubcc4\uc778\uc0ac\ub97c \uc62c\ub9ac\ub294 \uc0ac\uc2e0(\u8fad\u795e)\uacfc \uc11c\ub85c \ucd95\ubcf5\ud558\uba74\uc11c \uc81c\ubb3c\uc744 \ub098\ub204\uc5b4\uba39\ub294 \uc74c\ubcf5(\u98ee\u798f) \ub4f1\uc774 \uc788\ub2e4."} {"question": "2011\ub144 3\uc6d4 \ub77c\uc988\ubca0\ub9ac \ud30c\uc774 \ud504\ub85c\uc81d\ud2b8\ub97c \uc7a0\uc7ac\uc801\uc778 BBC \ub9c8\uc774\ud06c\ub85c 2\ub77c\uace0 \ud55c \uae30\uc220\uad00\ub828 \uc800\uc790\ub294 \ub204\uad6c\uc778\uac00?", "paragraph": "\uae30\uc220\uad00\ub828 \uc800\uc790\uc778 \uae00\ub79c \ubb34\ub514\ub294 2011\ub144 3\uc6d4 \uc774 \ud504\ub85c\uc81d\ud2b8\ub97c \"\uc7a0\uc7ac\uc801\uc778 BBC \ub9c8\uc774\ud06c\ub85c 2\"\uc774\ub77c\uace0 \ud558\uc600\uc73c\uba70, PC\ud638\ud658 \uae30\uacc4\ub97c \ub300\uccb4\ud560 \uc218\ub294 \uc5c6\uaca0\uc9c0\ub9cc, \ubcf4\ucda9\uc740 \ud560 \uc218 \uc788\uaca0\ub2e4\uace0 \uc124\uba85\ud558\uc600\ub2e4. 2012\ub144 3\uc6d4 \uc2a4\ud2f0\ube10 \ud53c\uce74\ub4dc\ub294 ITPRO\uc9c0\uc5d0\uc11c BBC \ub9c8\uc774\ud06c\ub85c\uc758 \ud6c4\uacc4\uc790\ub77c\uace0 \uc18c\uac10\uc744 \ub9d0\ud558\uc600\ub2e4. \ub2e4\uc74c \uc138\ub300 \ub9ac\ud3ec\ud2b8\uc758 \uacf5\ub3d9 \uc800\uc790\uc778 \uc54c\ub809\uc2a4 \ud638\ud504\ub294 \ucef4\ud4e8\ud130\uac00 \uc5b4\ub9b0\uc774\ub4e4\uc5d0\uac8c \ud504\ub85c\uadf8\ub798\ubc0d\uc758 \uc990\uac70\uc6c0\uc744 \uac00\ub974\uce60\uc218 \uc788\ub294 \ucef4\ud4e8\ud130\ub85c \uc778\ud574 \uc720\ub9dd\ud558\ub2e4\ub294 \ub9d0\uc744 \ud558\uc600\ub2e4. \uc54c\ub809\uc2a4 \ud638\ud504\uc640 \uacf5\uc800\ub97c \ud55c \uc774\uc548 \ub9ac\ube59\uc2a4\ud134\uc740 BBC\uc5d0\uc11c \ube4c\ub529 \uad00\ub9ac \uc2dc\uc2a4\ud15c\uc744 \uc9d1\uc5b4\ub123\uc740\ud6c4 BBC \ub098\ub178\ub77c\uace0 \uc774\ub984\uc744 \ubd99\uc77c\uc218 \uc788\uaca0\ub2e4\ub77c\uace0 \ub9d0\ud588\ub2e4. \ud06c\ub9ac\uc2a4 \uc70c\ub9ac\uc5c4\uc2a4\ub294 The Register\uc5d0\uc11c \ube44\ub85d \uadf8\uac8c \uadf8\ub4e4\uc774 \uc0ac\uc6a9\ud558\ub294\ub370 \uc720\uc6a9\ud558\uac8c \ub0a8\uc544 \uc788\uc744\ub54c \ud0a4\ub4dc \ub8e8\ube44, \uc2a4\ud06c\ub798\uce58, \ubca0\uc774\uc9c1\ub4f1\uc774 \ubbf8\ub798\uc758 \ub2a5\ub825\uc744 \uc313\ub294\ub370 \uc88b\uc740 \uc2dc\uc791\uc774\ub77c\uace0 \ubcf8\ub2e4\uace0 \ud558\uace0 \uc788\ub2e4. \ucef4\ud4e8\ud130 \uc5ed\uc0ac \uc13c\ud130\uc5d0\uc120 \uac15\ub825\ud558\uac8c \ub77c\uc988\ubca0\ub9ac \ud30c\uc774\ub97c \ubc00\uace0 \uc788\ub294\ub370, \ub77c\uc988\ubca0\ub9ac \ud30c\uc774\uc5d0 \ub300\ud574\uc11c \"\uc0c8 \uc2dc\ub300\uc758 \ubb38\uc9c0\uae30\"\uac19\uc740 \ub290\ub08c\uc774 \ub4e0\ub2e4\uace0 \ub9d0\ud588\ub2e4. \ub77c\uc988\ubca0\ub9ac \ud30c\uc774\uac00 \ub098\uc624\uae30\uc804, \ub77c\uc988\ubca0\ub9ac \ud30c\uc774 \ubcf4\ub4dc\uac00 \uad6c\uae00\uc758 \uc0dd\uac01\uc758 \uc724\uacfd\uc774 \uc601\uad6d \uacfc\ud559\uae30\uc220\uc758 \ubc1c\uc804\uc744 \uc5b4\ub5bb\uac8c \uac00\uc838\uc62c\uc9c0\uc5d0 \ub300\ud55c \ucea0\ube0c\ub9ac\uc9c0\uc5d0\uc11c\uc758 \uc774\ubca4\ud2b8\ub54c ARM\uc758 CEO\uc778 \uc6cc\ub80c \uc774\uc2a4\ud2b8\uc5d0 \uc758\ud574 \ucd9c\ud488\ub41c\uc801\uc774 \uc788\uc5c8\ub2e4.", "answer": "\uae00\ub79c \ubb34\ub514", "sentence": "\uae30\uc220\uad00\ub828 \uc800\uc790\uc778 \uae00\ub79c \ubb34\ub514 \ub294 2011\ub144 3\uc6d4 \uc774 \ud504\ub85c\uc81d\ud2b8\ub97c \"\uc7a0\uc7ac\uc801\uc778 BBC \ub9c8\uc774\ud06c\ub85c 2\"\uc774\ub77c\uace0 \ud558\uc600\uc73c\uba70, PC\ud638\ud658 \uae30\uacc4\ub97c \ub300\uccb4\ud560 \uc218\ub294 \uc5c6\uaca0\uc9c0\ub9cc, \ubcf4\ucda9\uc740 \ud560 \uc218 \uc788\uaca0\ub2e4\uace0 \uc124\uba85\ud558\uc600\ub2e4.", "paragraph_sentence": " \uae30\uc220\uad00\ub828 \uc800\uc790\uc778 \uae00\ub79c \ubb34\ub514 \ub294 2011\ub144 3\uc6d4 \uc774 \ud504\ub85c\uc81d\ud2b8\ub97c \"\uc7a0\uc7ac\uc801\uc778 BBC \ub9c8\uc774\ud06c\ub85c 2\"\uc774\ub77c\uace0 \ud558\uc600\uc73c\uba70, PC\ud638\ud658 \uae30\uacc4\ub97c \ub300\uccb4\ud560 \uc218\ub294 \uc5c6\uaca0\uc9c0\ub9cc, \ubcf4\ucda9\uc740 \ud560 \uc218 \uc788\uaca0\ub2e4\uace0 \uc124\uba85\ud558\uc600\ub2e4. 2012\ub144 3\uc6d4 \uc2a4\ud2f0\ube10 \ud53c\uce74\ub4dc\ub294 ITPRO\uc9c0\uc5d0\uc11c BBC \ub9c8\uc774\ud06c\ub85c\uc758 \ud6c4\uacc4\uc790\ub77c\uace0 \uc18c\uac10\uc744 \ub9d0\ud558\uc600\ub2e4. \ub2e4\uc74c \uc138\ub300 \ub9ac\ud3ec\ud2b8\uc758 \uacf5\ub3d9 \uc800\uc790\uc778 \uc54c\ub809\uc2a4 \ud638\ud504\ub294 \ucef4\ud4e8\ud130\uac00 \uc5b4\ub9b0\uc774\ub4e4\uc5d0\uac8c \ud504\ub85c\uadf8\ub798\ubc0d\uc758 \uc990\uac70\uc6c0\uc744 \uac00\ub974\uce60\uc218 \uc788\ub294 \ucef4\ud4e8\ud130\ub85c \uc778\ud574 \uc720\ub9dd\ud558\ub2e4\ub294 \ub9d0\uc744 \ud558\uc600\ub2e4. \uc54c\ub809\uc2a4 \ud638\ud504\uc640 \uacf5\uc800\ub97c \ud55c \uc774\uc548 \ub9ac\ube59\uc2a4\ud134\uc740 BBC\uc5d0\uc11c \ube4c\ub529 \uad00\ub9ac \uc2dc\uc2a4\ud15c\uc744 \uc9d1\uc5b4\ub123\uc740\ud6c4 BBC \ub098\ub178\ub77c\uace0 \uc774\ub984\uc744 \ubd99\uc77c\uc218 \uc788\uaca0\ub2e4\ub77c\uace0 \ub9d0\ud588\ub2e4. \ud06c\ub9ac\uc2a4 \uc70c\ub9ac\uc5c4\uc2a4\ub294 The Register\uc5d0\uc11c \ube44\ub85d \uadf8\uac8c \uadf8\ub4e4\uc774 \uc0ac\uc6a9\ud558\ub294\ub370 \uc720\uc6a9\ud558\uac8c \ub0a8\uc544 \uc788\uc744\ub54c \ud0a4\ub4dc \ub8e8\ube44, \uc2a4\ud06c\ub798\uce58, \ubca0\uc774\uc9c1\ub4f1\uc774 \ubbf8\ub798\uc758 \ub2a5\ub825\uc744 \uc313\ub294\ub370 \uc88b\uc740 \uc2dc\uc791\uc774\ub77c\uace0 \ubcf8\ub2e4\uace0 \ud558\uace0 \uc788\ub2e4. \ucef4\ud4e8\ud130 \uc5ed\uc0ac \uc13c\ud130\uc5d0\uc120 \uac15\ub825\ud558\uac8c \ub77c\uc988\ubca0\ub9ac \ud30c\uc774\ub97c \ubc00\uace0 \uc788\ub294\ub370, \ub77c\uc988\ubca0\ub9ac \ud30c\uc774\uc5d0 \ub300\ud574\uc11c \"\uc0c8 \uc2dc\ub300\uc758 \ubb38\uc9c0\uae30\"\uac19\uc740 \ub290\ub08c\uc774 \ub4e0\ub2e4\uace0 \ub9d0\ud588\ub2e4. \ub77c\uc988\ubca0\ub9ac \ud30c\uc774\uac00 \ub098\uc624\uae30\uc804, \ub77c\uc988\ubca0\ub9ac \ud30c\uc774 \ubcf4\ub4dc\uac00 \uad6c\uae00\uc758 \uc0dd\uac01\uc758 \uc724\uacfd\uc774 \uc601\uad6d \uacfc\ud559\uae30\uc220\uc758 \ubc1c\uc804\uc744 \uc5b4\ub5bb\uac8c \uac00\uc838\uc62c\uc9c0\uc5d0 \ub300\ud55c \ucea0\ube0c\ub9ac\uc9c0\uc5d0\uc11c\uc758 \uc774\ubca4\ud2b8\ub54c ARM\uc758 CEO\uc778 \uc6cc\ub80c \uc774\uc2a4\ud2b8\uc5d0 \uc758\ud574 \ucd9c\ud488\ub41c\uc801\uc774 \uc788\uc5c8\ub2e4.", "paragraph_answer": "\uae30\uc220\uad00\ub828 \uc800\uc790\uc778 \uae00\ub79c \ubb34\ub514 \ub294 2011\ub144 3\uc6d4 \uc774 \ud504\ub85c\uc81d\ud2b8\ub97c \"\uc7a0\uc7ac\uc801\uc778 BBC \ub9c8\uc774\ud06c\ub85c 2\"\uc774\ub77c\uace0 \ud558\uc600\uc73c\uba70, PC\ud638\ud658 \uae30\uacc4\ub97c \ub300\uccb4\ud560 \uc218\ub294 \uc5c6\uaca0\uc9c0\ub9cc, \ubcf4\ucda9\uc740 \ud560 \uc218 \uc788\uaca0\ub2e4\uace0 \uc124\uba85\ud558\uc600\ub2e4. 2012\ub144 3\uc6d4 \uc2a4\ud2f0\ube10 \ud53c\uce74\ub4dc\ub294 ITPRO\uc9c0\uc5d0\uc11c BBC \ub9c8\uc774\ud06c\ub85c\uc758 \ud6c4\uacc4\uc790\ub77c\uace0 \uc18c\uac10\uc744 \ub9d0\ud558\uc600\ub2e4. \ub2e4\uc74c \uc138\ub300 \ub9ac\ud3ec\ud2b8\uc758 \uacf5\ub3d9 \uc800\uc790\uc778 \uc54c\ub809\uc2a4 \ud638\ud504\ub294 \ucef4\ud4e8\ud130\uac00 \uc5b4\ub9b0\uc774\ub4e4\uc5d0\uac8c \ud504\ub85c\uadf8\ub798\ubc0d\uc758 \uc990\uac70\uc6c0\uc744 \uac00\ub974\uce60\uc218 \uc788\ub294 \ucef4\ud4e8\ud130\ub85c \uc778\ud574 \uc720\ub9dd\ud558\ub2e4\ub294 \ub9d0\uc744 \ud558\uc600\ub2e4. \uc54c\ub809\uc2a4 \ud638\ud504\uc640 \uacf5\uc800\ub97c \ud55c \uc774\uc548 \ub9ac\ube59\uc2a4\ud134\uc740 BBC\uc5d0\uc11c \ube4c\ub529 \uad00\ub9ac \uc2dc\uc2a4\ud15c\uc744 \uc9d1\uc5b4\ub123\uc740\ud6c4 BBC \ub098\ub178\ub77c\uace0 \uc774\ub984\uc744 \ubd99\uc77c\uc218 \uc788\uaca0\ub2e4\ub77c\uace0 \ub9d0\ud588\ub2e4. \ud06c\ub9ac\uc2a4 \uc70c\ub9ac\uc5c4\uc2a4\ub294 The Register\uc5d0\uc11c \ube44\ub85d \uadf8\uac8c \uadf8\ub4e4\uc774 \uc0ac\uc6a9\ud558\ub294\ub370 \uc720\uc6a9\ud558\uac8c \ub0a8\uc544 \uc788\uc744\ub54c \ud0a4\ub4dc \ub8e8\ube44, \uc2a4\ud06c\ub798\uce58, \ubca0\uc774\uc9c1\ub4f1\uc774 \ubbf8\ub798\uc758 \ub2a5\ub825\uc744 \uc313\ub294\ub370 \uc88b\uc740 \uc2dc\uc791\uc774\ub77c\uace0 \ubcf8\ub2e4\uace0 \ud558\uace0 \uc788\ub2e4. \ucef4\ud4e8\ud130 \uc5ed\uc0ac \uc13c\ud130\uc5d0\uc120 \uac15\ub825\ud558\uac8c \ub77c\uc988\ubca0\ub9ac \ud30c\uc774\ub97c \ubc00\uace0 \uc788\ub294\ub370, \ub77c\uc988\ubca0\ub9ac \ud30c\uc774\uc5d0 \ub300\ud574\uc11c \"\uc0c8 \uc2dc\ub300\uc758 \ubb38\uc9c0\uae30\"\uac19\uc740 \ub290\ub08c\uc774 \ub4e0\ub2e4\uace0 \ub9d0\ud588\ub2e4. \ub77c\uc988\ubca0\ub9ac \ud30c\uc774\uac00 \ub098\uc624\uae30\uc804, \ub77c\uc988\ubca0\ub9ac \ud30c\uc774 \ubcf4\ub4dc\uac00 \uad6c\uae00\uc758 \uc0dd\uac01\uc758 \uc724\uacfd\uc774 \uc601\uad6d \uacfc\ud559\uae30\uc220\uc758 \ubc1c\uc804\uc744 \uc5b4\ub5bb\uac8c \uac00\uc838\uc62c\uc9c0\uc5d0 \ub300\ud55c \ucea0\ube0c\ub9ac\uc9c0\uc5d0\uc11c\uc758 \uc774\ubca4\ud2b8\ub54c ARM\uc758 CEO\uc778 \uc6cc\ub80c \uc774\uc2a4\ud2b8\uc5d0 \uc758\ud574 \ucd9c\ud488\ub41c\uc801\uc774 \uc788\uc5c8\ub2e4.", "sentence_answer": "\uae30\uc220\uad00\ub828 \uc800\uc790\uc778 \uae00\ub79c \ubb34\ub514 \ub294 2011\ub144 3\uc6d4 \uc774 \ud504\ub85c\uc81d\ud2b8\ub97c \"\uc7a0\uc7ac\uc801\uc778 BBC \ub9c8\uc774\ud06c\ub85c 2\"\uc774\ub77c\uace0 \ud558\uc600\uc73c\uba70, PC\ud638\ud658 \uae30\uacc4\ub97c \ub300\uccb4\ud560 \uc218\ub294 \uc5c6\uaca0\uc9c0\ub9cc, \ubcf4\ucda9\uc740 \ud560 \uc218 \uc788\uaca0\ub2e4\uace0 \uc124\uba85\ud558\uc600\ub2e4.", "paragraph_id": "6513404-4-0", "paragraph_question": "question: 2011\ub144 3\uc6d4 \ub77c\uc988\ubca0\ub9ac \ud30c\uc774 \ud504\ub85c\uc81d\ud2b8\ub97c \uc7a0\uc7ac\uc801\uc778 BBC \ub9c8\uc774\ud06c\ub85c 2\ub77c\uace0 \ud55c \uae30\uc220\uad00\ub828 \uc800\uc790\ub294 \ub204\uad6c\uc778\uac00?, context: \uae30\uc220\uad00\ub828 \uc800\uc790\uc778 \uae00\ub79c \ubb34\ub514\ub294 2011\ub144 3\uc6d4 \uc774 \ud504\ub85c\uc81d\ud2b8\ub97c \"\uc7a0\uc7ac\uc801\uc778 BBC \ub9c8\uc774\ud06c\ub85c 2\"\uc774\ub77c\uace0 \ud558\uc600\uc73c\uba70, PC\ud638\ud658 \uae30\uacc4\ub97c \ub300\uccb4\ud560 \uc218\ub294 \uc5c6\uaca0\uc9c0\ub9cc, \ubcf4\ucda9\uc740 \ud560 \uc218 \uc788\uaca0\ub2e4\uace0 \uc124\uba85\ud558\uc600\ub2e4. 2012\ub144 3\uc6d4 \uc2a4\ud2f0\ube10 \ud53c\uce74\ub4dc\ub294 ITPRO\uc9c0\uc5d0\uc11c BBC \ub9c8\uc774\ud06c\ub85c\uc758 \ud6c4\uacc4\uc790\ub77c\uace0 \uc18c\uac10\uc744 \ub9d0\ud558\uc600\ub2e4. \ub2e4\uc74c \uc138\ub300 \ub9ac\ud3ec\ud2b8\uc758 \uacf5\ub3d9 \uc800\uc790\uc778 \uc54c\ub809\uc2a4 \ud638\ud504\ub294 \ucef4\ud4e8\ud130\uac00 \uc5b4\ub9b0\uc774\ub4e4\uc5d0\uac8c \ud504\ub85c\uadf8\ub798\ubc0d\uc758 \uc990\uac70\uc6c0\uc744 \uac00\ub974\uce60\uc218 \uc788\ub294 \ucef4\ud4e8\ud130\ub85c \uc778\ud574 \uc720\ub9dd\ud558\ub2e4\ub294 \ub9d0\uc744 \ud558\uc600\ub2e4. \uc54c\ub809\uc2a4 \ud638\ud504\uc640 \uacf5\uc800\ub97c \ud55c \uc774\uc548 \ub9ac\ube59\uc2a4\ud134\uc740 BBC\uc5d0\uc11c \ube4c\ub529 \uad00\ub9ac \uc2dc\uc2a4\ud15c\uc744 \uc9d1\uc5b4\ub123\uc740\ud6c4 BBC \ub098\ub178\ub77c\uace0 \uc774\ub984\uc744 \ubd99\uc77c\uc218 \uc788\uaca0\ub2e4\ub77c\uace0 \ub9d0\ud588\ub2e4. \ud06c\ub9ac\uc2a4 \uc70c\ub9ac\uc5c4\uc2a4\ub294 The Register\uc5d0\uc11c \ube44\ub85d \uadf8\uac8c \uadf8\ub4e4\uc774 \uc0ac\uc6a9\ud558\ub294\ub370 \uc720\uc6a9\ud558\uac8c \ub0a8\uc544 \uc788\uc744\ub54c \ud0a4\ub4dc \ub8e8\ube44, \uc2a4\ud06c\ub798\uce58, \ubca0\uc774\uc9c1\ub4f1\uc774 \ubbf8\ub798\uc758 \ub2a5\ub825\uc744 \uc313\ub294\ub370 \uc88b\uc740 \uc2dc\uc791\uc774\ub77c\uace0 \ubcf8\ub2e4\uace0 \ud558\uace0 \uc788\ub2e4. \ucef4\ud4e8\ud130 \uc5ed\uc0ac \uc13c\ud130\uc5d0\uc120 \uac15\ub825\ud558\uac8c \ub77c\uc988\ubca0\ub9ac \ud30c\uc774\ub97c \ubc00\uace0 \uc788\ub294\ub370, \ub77c\uc988\ubca0\ub9ac \ud30c\uc774\uc5d0 \ub300\ud574\uc11c \"\uc0c8 \uc2dc\ub300\uc758 \ubb38\uc9c0\uae30\"\uac19\uc740 \ub290\ub08c\uc774 \ub4e0\ub2e4\uace0 \ub9d0\ud588\ub2e4. \ub77c\uc988\ubca0\ub9ac \ud30c\uc774\uac00 \ub098\uc624\uae30\uc804, \ub77c\uc988\ubca0\ub9ac \ud30c\uc774 \ubcf4\ub4dc\uac00 \uad6c\uae00\uc758 \uc0dd\uac01\uc758 \uc724\uacfd\uc774 \uc601\uad6d \uacfc\ud559\uae30\uc220\uc758 \ubc1c\uc804\uc744 \uc5b4\ub5bb\uac8c \uac00\uc838\uc62c\uc9c0\uc5d0 \ub300\ud55c \ucea0\ube0c\ub9ac\uc9c0\uc5d0\uc11c\uc758 \uc774\ubca4\ud2b8\ub54c ARM\uc758 CEO\uc778 \uc6cc\ub80c \uc774\uc2a4\ud2b8\uc5d0 \uc758\ud574 \ucd9c\ud488\ub41c\uc801\uc774 \uc788\uc5c8\ub2e4."} {"question": "\uc624\uc138\ud6c8\uc740 \uc5b4\ub290\ub3c4\uc2dc \uc2dc\uc7a5\uc774\uc5c8\ub098?", "paragraph": "\uc774\uba85\ubc15 \uc815\ubd80\uc5d0\uc11c \ubc15\uadfc\ud61c\ub294 \uc774\uba85\ubc15\uacfc \uac70\ub9ac\ub97c \ub450\uace0 \uc788\uc73c\ub098 \uc644\uc804\ud788 \ub3cc\uc544\uc11c\uc9c0\ub294 \uc54a\uc558\ub2e4. \ub300\ud1b5\ub839\uacfc \uba74\ub2f4\uc744 \ud558\uba74\uc11c \ub300\ud1b5\ud569\uc758 \uc758\uc9c0\ub97c \ub2f4\uace0 \ud65c\ub3d9\ud558\uc600\ub2e4. \uacfc\uac70 \uc815\ucc45\uc0c1 \ubcf8\uc9c8\uc801\uc73c\ub85c\ub294 \uc774\uba85\ubc15\uacfc \ud06c\uac8c \ub2e4\ub97c\ubc14 \uc5c6\ub2e4\ub294 \uc758\uacac\ub3c4 \uc788\ub2e4. \uadfc\uac70\ub85c\ub294 2007\ub144 \ud55c\ub098\ub77c\ub2f9 \ub300\uc120\ud6c4\ubcf4 \uacbd\uc120 \ub54c \ubc15\uadfc\ud61c\uac00 \ub0b4\uc138\uc6e0\ub358 '\uc904\ud478\uc138'\ub098 \ub2f9\uc2dc \uacbd\uc7c1 \ud6c4\ubcf4\uc600\ub358 \uc774\uba85\ubc15\uc774 \ub0b4\uc138\uc6e0\ub358 '747' \ub458\ub2e4 \uacf5\ud1b5\uc801\uc73c\ub85c \uac10\uc138 \uae30\uc870\ub97c \ub0b4\uc138\uc6b4 \uc815\ucc45\uc774\ub77c\ub294 \uc810\uc774\ub2e4. \ub610\ud55c \ubc15\uadfc\ud61c\uac00 2008\ub144 12\uc6d4 \uad6d\ud68c\uc758 \uac10\uc138 \ubc95\uc548, 2009\ub144 3\uc6d4 \uae08\uc0b0\ubd84\ub9ac\uc644\ud654\ubc95\uc548, 2009\ub144 12\uc6d4 \uc608\uc0b0\uc548 \u2500 4\ub300\uac15 \uc608\uc0b0\uc774 \ud3ec\ud568\ub418\uc5c8\uace0 \uc774\ub978\ubc14 \ub0a0\uce58\uae30 \ud1b5\uacfc\ub85c \ub17c\ub780\uc774 \ub418\uc5c8\uc74c \u2500 \uc5d0 \ucc2c\uc131\ud574 \uc774\uba85\ubc15 \uc815\ubd80 \uc815\ucc45\uc5d0 \ud611\uc870\ud55c \uac83\ub3c4 \uc788\ub2e4. \uc774\ub97c \ube57\ub300\uc5b4 \uc815\uce58\uad8c\uacfc \uc778\ud130\ub137 \uc0c1\uc5d0\uc11c '\uc774\uba85\ubc15\uadfc\ud61c'\ub85c \ud45c\ud604\ud558\uae30\ub3c4 \ud55c\ub2e4. \uadf8\ub7ec\ub098 \uc774\uba85\ubc15\uacfc \ub2e4\ub974\ub2e4\uace0 \ubcfc \uc218 \uc788\ub294 \uac83\ub4e4\ub3c4 \uc788\ub2e4. \ub300\ud45c\uc801\uc73c\ub85c \uc774\uba85\ubc15 \ub300\ud1b5\ub839\uc774 \uc8fc\ub3c4\ud588\ub358 '\uc138\uc885\uc2dc \uc218\uc815\uc548'\uc744 \ubd80\uacb0\ub418\uac8c \ub9cc\ub4e0 \uac83\uc774\ub2e4. \ub610\ud55c \uc11c\uc6b8\uc2dc\uc758 \ubb34\uc0c1 \uae09\uc2dd \uc815\ucc45 \ub17c\ub780 \ub54c \ubc14\uc05c \uc77c\uc815 \uc18d\uc5d0 \ubd80\uc7ac\uc790 \ud22c\ud45c\uae4c\uc9c0 \ucc38\uc5ec\ud574 \uc624\uc138\ud6c8 \uc804 \uc11c\uc6b8\uc2dc\uc7a5\uc744 \uc9c0\uc6d0\ud588\ub358 \uc774\uba85\ubc15\uacfc\ub294 \ub2ec\ub9ac \uad00\ub828 \uc0ac\uc548\uc5d0 \uc785\uc7a5\uc744 \ubc1d\ud788\uc9c0 \uc54a\uc558\ub2e4. 2011\ub144 \ub9d0 \ube44\uc0c1\ub300\ucc45\uc704\uc6d0\uc7a5\uc744 \ub9e1\uc740 \ud6c4 \ub2f9\uba85, \ub2f9 \uc0c1\uc9d5\uc0c9, \uac15\ub839 \ub4f1\uc744 \ubcc0\uacbd\ud558\uba74\uc11c MB\uc815\uad8c\uacfc \uae30\uc874 \ud55c\ub098\ub77c\ub2f9\uacfc\uc758 \ucc28\ubcc4\ud654\ub97c \uc2dc\ub3c4\ud588\ub2e4. 2012\ub144 4\uc6d4 \ucd1d\uc120\uc744 \uc55e\ub450\uace0 \ubd88\uac70\uc9c4 \uad6d\ubb34\ucd1d\ub9ac\uc2e4\uc758 \ubbfc\uac04\uc778 \uc0ac\ucc30 \uc0ac\uac74 \uc545\uc7ac \uc18d\uc5d0\uc11c\ub3c4 \"\uc5b4\ub290 \uc815\uad8c\uc5d0\uc11c\ub098 \ubbfc\uac04\uc778 \uc0ac\ucc30\uc740 \uc77c\uc5b4\ub098\uba70 \ub098\ub3c4 \ud53c\ud574\uc790\"\ub77c\uba70 \ub17c\ub780\uc744 \ud53c\ud558\uae30\ub3c4 \ud588\ub2e4.", "answer": "\uc11c\uc6b8\uc2dc", "sentence": "\ub610\ud55c \uc11c\uc6b8\uc2dc \uc758 \ubb34\uc0c1 \uae09\uc2dd \uc815\ucc45 \ub17c\ub780 \ub54c \ubc14\uc05c \uc77c\uc815 \uc18d\uc5d0 \ubd80\uc7ac\uc790 \ud22c\ud45c\uae4c\uc9c0 \ucc38\uc5ec\ud574 \uc624\uc138\ud6c8 \uc804 \uc11c\uc6b8\uc2dc\uc7a5\uc744 \uc9c0\uc6d0\ud588\ub358 \uc774\uba85\ubc15\uacfc\ub294 \ub2ec\ub9ac \uad00\ub828 \uc0ac\uc548\uc5d0 \uc785\uc7a5\uc744 \ubc1d\ud788\uc9c0 \uc54a\uc558\ub2e4.", "paragraph_sentence": "\uc774\uba85\ubc15 \uc815\ubd80\uc5d0\uc11c \ubc15\uadfc\ud61c\ub294 \uc774\uba85\ubc15\uacfc \uac70\ub9ac\ub97c \ub450\uace0 \uc788\uc73c\ub098 \uc644\uc804\ud788 \ub3cc\uc544\uc11c\uc9c0\ub294 \uc54a\uc558\ub2e4. \ub300\ud1b5\ub839\uacfc \uba74\ub2f4\uc744 \ud558\uba74\uc11c \ub300\ud1b5\ud569\uc758 \uc758\uc9c0\ub97c \ub2f4\uace0 \ud65c\ub3d9\ud558\uc600\ub2e4. \uacfc\uac70 \uc815\ucc45\uc0c1 \ubcf8\uc9c8\uc801\uc73c\ub85c\ub294 \uc774\uba85\ubc15\uacfc \ud06c\uac8c \ub2e4\ub97c\ubc14 \uc5c6\ub2e4\ub294 \uc758\uacac\ub3c4 \uc788\ub2e4. \uadfc\uac70\ub85c\ub294 2007\ub144 \ud55c\ub098\ub77c\ub2f9 \ub300\uc120\ud6c4\ubcf4 \uacbd\uc120 \ub54c \ubc15\uadfc\ud61c\uac00 \ub0b4\uc138\uc6e0\ub358 '\uc904\ud478\uc138'\ub098 \ub2f9\uc2dc \uacbd\uc7c1 \ud6c4\ubcf4\uc600\ub358 \uc774\uba85\ubc15\uc774 \ub0b4\uc138\uc6e0\ub358 '747' \ub458\ub2e4 \uacf5\ud1b5\uc801\uc73c\ub85c \uac10\uc138 \uae30\uc870\ub97c \ub0b4\uc138\uc6b4 \uc815\ucc45\uc774\ub77c\ub294 \uc810\uc774\ub2e4. \ub610\ud55c \ubc15\uadfc\ud61c\uac00 2008\ub144 12\uc6d4 \uad6d\ud68c\uc758 \uac10\uc138 \ubc95\uc548, 2009\ub144 3\uc6d4 \uae08\uc0b0\ubd84\ub9ac\uc644\ud654\ubc95\uc548, 2009\ub144 12\uc6d4 \uc608\uc0b0\uc548 \u2500 4\ub300\uac15 \uc608\uc0b0\uc774 \ud3ec\ud568\ub418\uc5c8\uace0 \uc774\ub978\ubc14 \ub0a0\uce58\uae30 \ud1b5\uacfc\ub85c \ub17c\ub780\uc774 \ub418\uc5c8\uc74c \u2500 \uc5d0 \ucc2c\uc131\ud574 \uc774\uba85\ubc15 \uc815\ubd80 \uc815\ucc45\uc5d0 \ud611\uc870\ud55c \uac83\ub3c4 \uc788\ub2e4. \uc774\ub97c \ube57\ub300\uc5b4 \uc815\uce58\uad8c\uacfc \uc778\ud130\ub137 \uc0c1\uc5d0\uc11c '\uc774\uba85\ubc15\uadfc\ud61c'\ub85c \ud45c\ud604\ud558\uae30\ub3c4 \ud55c\ub2e4. \uadf8\ub7ec\ub098 \uc774\uba85\ubc15\uacfc \ub2e4\ub974\ub2e4\uace0 \ubcfc \uc218 \uc788\ub294 \uac83\ub4e4\ub3c4 \uc788\ub2e4. \ub300\ud45c\uc801\uc73c\ub85c \uc774\uba85\ubc15 \ub300\ud1b5\ub839\uc774 \uc8fc\ub3c4\ud588\ub358 '\uc138\uc885\uc2dc \uc218\uc815\uc548'\uc744 \ubd80\uacb0\ub418\uac8c \ub9cc\ub4e0 \uac83\uc774\ub2e4. \ub610\ud55c \uc11c\uc6b8\uc2dc \uc758 \ubb34\uc0c1 \uae09\uc2dd \uc815\ucc45 \ub17c\ub780 \ub54c \ubc14\uc05c \uc77c\uc815 \uc18d\uc5d0 \ubd80\uc7ac\uc790 \ud22c\ud45c\uae4c\uc9c0 \ucc38\uc5ec\ud574 \uc624\uc138\ud6c8 \uc804 \uc11c\uc6b8\uc2dc\uc7a5\uc744 \uc9c0\uc6d0\ud588\ub358 \uc774\uba85\ubc15\uacfc\ub294 \ub2ec\ub9ac \uad00\ub828 \uc0ac\uc548\uc5d0 \uc785\uc7a5\uc744 \ubc1d\ud788\uc9c0 \uc54a\uc558\ub2e4. 2011\ub144 \ub9d0 \ube44\uc0c1\ub300\ucc45\uc704\uc6d0\uc7a5\uc744 \ub9e1\uc740 \ud6c4 \ub2f9\uba85, \ub2f9 \uc0c1\uc9d5\uc0c9, \uac15\ub839 \ub4f1\uc744 \ubcc0\uacbd\ud558\uba74\uc11c MB\uc815\uad8c\uacfc \uae30\uc874 \ud55c\ub098\ub77c\ub2f9\uacfc\uc758 \ucc28\ubcc4\ud654\ub97c \uc2dc\ub3c4\ud588\ub2e4. 2012\ub144 4\uc6d4 \ucd1d\uc120\uc744 \uc55e\ub450\uace0 \ubd88\uac70\uc9c4 \uad6d\ubb34\ucd1d\ub9ac\uc2e4\uc758 \ubbfc\uac04\uc778 \uc0ac\ucc30 \uc0ac\uac74 \uc545\uc7ac \uc18d\uc5d0\uc11c\ub3c4 \"\uc5b4\ub290 \uc815\uad8c\uc5d0\uc11c\ub098 \ubbfc\uac04\uc778 \uc0ac\ucc30\uc740 \uc77c\uc5b4\ub098\uba70 \ub098\ub3c4 \ud53c\ud574\uc790\"\ub77c\uba70 \ub17c\ub780\uc744 \ud53c\ud558\uae30\ub3c4 \ud588\ub2e4.", "paragraph_answer": "\uc774\uba85\ubc15 \uc815\ubd80\uc5d0\uc11c \ubc15\uadfc\ud61c\ub294 \uc774\uba85\ubc15\uacfc \uac70\ub9ac\ub97c \ub450\uace0 \uc788\uc73c\ub098 \uc644\uc804\ud788 \ub3cc\uc544\uc11c\uc9c0\ub294 \uc54a\uc558\ub2e4. \ub300\ud1b5\ub839\uacfc \uba74\ub2f4\uc744 \ud558\uba74\uc11c \ub300\ud1b5\ud569\uc758 \uc758\uc9c0\ub97c \ub2f4\uace0 \ud65c\ub3d9\ud558\uc600\ub2e4. \uacfc\uac70 \uc815\ucc45\uc0c1 \ubcf8\uc9c8\uc801\uc73c\ub85c\ub294 \uc774\uba85\ubc15\uacfc \ud06c\uac8c \ub2e4\ub97c\ubc14 \uc5c6\ub2e4\ub294 \uc758\uacac\ub3c4 \uc788\ub2e4. \uadfc\uac70\ub85c\ub294 2007\ub144 \ud55c\ub098\ub77c\ub2f9 \ub300\uc120\ud6c4\ubcf4 \uacbd\uc120 \ub54c \ubc15\uadfc\ud61c\uac00 \ub0b4\uc138\uc6e0\ub358 '\uc904\ud478\uc138'\ub098 \ub2f9\uc2dc \uacbd\uc7c1 \ud6c4\ubcf4\uc600\ub358 \uc774\uba85\ubc15\uc774 \ub0b4\uc138\uc6e0\ub358 '747' \ub458\ub2e4 \uacf5\ud1b5\uc801\uc73c\ub85c \uac10\uc138 \uae30\uc870\ub97c \ub0b4\uc138\uc6b4 \uc815\ucc45\uc774\ub77c\ub294 \uc810\uc774\ub2e4. \ub610\ud55c \ubc15\uadfc\ud61c\uac00 2008\ub144 12\uc6d4 \uad6d\ud68c\uc758 \uac10\uc138 \ubc95\uc548, 2009\ub144 3\uc6d4 \uae08\uc0b0\ubd84\ub9ac\uc644\ud654\ubc95\uc548, 2009\ub144 12\uc6d4 \uc608\uc0b0\uc548 \u2500 4\ub300\uac15 \uc608\uc0b0\uc774 \ud3ec\ud568\ub418\uc5c8\uace0 \uc774\ub978\ubc14 \ub0a0\uce58\uae30 \ud1b5\uacfc\ub85c \ub17c\ub780\uc774 \ub418\uc5c8\uc74c \u2500 \uc5d0 \ucc2c\uc131\ud574 \uc774\uba85\ubc15 \uc815\ubd80 \uc815\ucc45\uc5d0 \ud611\uc870\ud55c \uac83\ub3c4 \uc788\ub2e4. \uc774\ub97c \ube57\ub300\uc5b4 \uc815\uce58\uad8c\uacfc \uc778\ud130\ub137 \uc0c1\uc5d0\uc11c '\uc774\uba85\ubc15\uadfc\ud61c'\ub85c \ud45c\ud604\ud558\uae30\ub3c4 \ud55c\ub2e4. \uadf8\ub7ec\ub098 \uc774\uba85\ubc15\uacfc \ub2e4\ub974\ub2e4\uace0 \ubcfc \uc218 \uc788\ub294 \uac83\ub4e4\ub3c4 \uc788\ub2e4. \ub300\ud45c\uc801\uc73c\ub85c \uc774\uba85\ubc15 \ub300\ud1b5\ub839\uc774 \uc8fc\ub3c4\ud588\ub358 '\uc138\uc885\uc2dc \uc218\uc815\uc548'\uc744 \ubd80\uacb0\ub418\uac8c \ub9cc\ub4e0 \uac83\uc774\ub2e4. \ub610\ud55c \uc11c\uc6b8\uc2dc \uc758 \ubb34\uc0c1 \uae09\uc2dd \uc815\ucc45 \ub17c\ub780 \ub54c \ubc14\uc05c \uc77c\uc815 \uc18d\uc5d0 \ubd80\uc7ac\uc790 \ud22c\ud45c\uae4c\uc9c0 \ucc38\uc5ec\ud574 \uc624\uc138\ud6c8 \uc804 \uc11c\uc6b8\uc2dc\uc7a5\uc744 \uc9c0\uc6d0\ud588\ub358 \uc774\uba85\ubc15\uacfc\ub294 \ub2ec\ub9ac \uad00\ub828 \uc0ac\uc548\uc5d0 \uc785\uc7a5\uc744 \ubc1d\ud788\uc9c0 \uc54a\uc558\ub2e4. 2011\ub144 \ub9d0 \ube44\uc0c1\ub300\ucc45\uc704\uc6d0\uc7a5\uc744 \ub9e1\uc740 \ud6c4 \ub2f9\uba85, \ub2f9 \uc0c1\uc9d5\uc0c9, \uac15\ub839 \ub4f1\uc744 \ubcc0\uacbd\ud558\uba74\uc11c MB\uc815\uad8c\uacfc \uae30\uc874 \ud55c\ub098\ub77c\ub2f9\uacfc\uc758 \ucc28\ubcc4\ud654\ub97c \uc2dc\ub3c4\ud588\ub2e4. 2012\ub144 4\uc6d4 \ucd1d\uc120\uc744 \uc55e\ub450\uace0 \ubd88\uac70\uc9c4 \uad6d\ubb34\ucd1d\ub9ac\uc2e4\uc758 \ubbfc\uac04\uc778 \uc0ac\ucc30 \uc0ac\uac74 \uc545\uc7ac \uc18d\uc5d0\uc11c\ub3c4 \"\uc5b4\ub290 \uc815\uad8c\uc5d0\uc11c\ub098 \ubbfc\uac04\uc778 \uc0ac\ucc30\uc740 \uc77c\uc5b4\ub098\uba70 \ub098\ub3c4 \ud53c\ud574\uc790\"\ub77c\uba70 \ub17c\ub780\uc744 \ud53c\ud558\uae30\ub3c4 \ud588\ub2e4.", "sentence_answer": "\ub610\ud55c \uc11c\uc6b8\uc2dc \uc758 \ubb34\uc0c1 \uae09\uc2dd \uc815\ucc45 \ub17c\ub780 \ub54c \ubc14\uc05c \uc77c\uc815 \uc18d\uc5d0 \ubd80\uc7ac\uc790 \ud22c\ud45c\uae4c\uc9c0 \ucc38\uc5ec\ud574 \uc624\uc138\ud6c8 \uc804 \uc11c\uc6b8\uc2dc\uc7a5\uc744 \uc9c0\uc6d0\ud588\ub358 \uc774\uba85\ubc15\uacfc\ub294 \ub2ec\ub9ac \uad00\ub828 \uc0ac\uc548\uc5d0 \uc785\uc7a5\uc744 \ubc1d\ud788\uc9c0 \uc54a\uc558\ub2e4.", "paragraph_id": "6534705-12-1", "paragraph_question": "question: \uc624\uc138\ud6c8\uc740 \uc5b4\ub290\ub3c4\uc2dc \uc2dc\uc7a5\uc774\uc5c8\ub098?, context: \uc774\uba85\ubc15 \uc815\ubd80\uc5d0\uc11c \ubc15\uadfc\ud61c\ub294 \uc774\uba85\ubc15\uacfc \uac70\ub9ac\ub97c \ub450\uace0 \uc788\uc73c\ub098 \uc644\uc804\ud788 \ub3cc\uc544\uc11c\uc9c0\ub294 \uc54a\uc558\ub2e4. \ub300\ud1b5\ub839\uacfc \uba74\ub2f4\uc744 \ud558\uba74\uc11c \ub300\ud1b5\ud569\uc758 \uc758\uc9c0\ub97c \ub2f4\uace0 \ud65c\ub3d9\ud558\uc600\ub2e4. \uacfc\uac70 \uc815\ucc45\uc0c1 \ubcf8\uc9c8\uc801\uc73c\ub85c\ub294 \uc774\uba85\ubc15\uacfc \ud06c\uac8c \ub2e4\ub97c\ubc14 \uc5c6\ub2e4\ub294 \uc758\uacac\ub3c4 \uc788\ub2e4. \uadfc\uac70\ub85c\ub294 2007\ub144 \ud55c\ub098\ub77c\ub2f9 \ub300\uc120\ud6c4\ubcf4 \uacbd\uc120 \ub54c \ubc15\uadfc\ud61c\uac00 \ub0b4\uc138\uc6e0\ub358 '\uc904\ud478\uc138'\ub098 \ub2f9\uc2dc \uacbd\uc7c1 \ud6c4\ubcf4\uc600\ub358 \uc774\uba85\ubc15\uc774 \ub0b4\uc138\uc6e0\ub358 '747' \ub458\ub2e4 \uacf5\ud1b5\uc801\uc73c\ub85c \uac10\uc138 \uae30\uc870\ub97c \ub0b4\uc138\uc6b4 \uc815\ucc45\uc774\ub77c\ub294 \uc810\uc774\ub2e4. \ub610\ud55c \ubc15\uadfc\ud61c\uac00 2008\ub144 12\uc6d4 \uad6d\ud68c\uc758 \uac10\uc138 \ubc95\uc548, 2009\ub144 3\uc6d4 \uae08\uc0b0\ubd84\ub9ac\uc644\ud654\ubc95\uc548, 2009\ub144 12\uc6d4 \uc608\uc0b0\uc548 \u2500 4\ub300\uac15 \uc608\uc0b0\uc774 \ud3ec\ud568\ub418\uc5c8\uace0 \uc774\ub978\ubc14 \ub0a0\uce58\uae30 \ud1b5\uacfc\ub85c \ub17c\ub780\uc774 \ub418\uc5c8\uc74c \u2500 \uc5d0 \ucc2c\uc131\ud574 \uc774\uba85\ubc15 \uc815\ubd80 \uc815\ucc45\uc5d0 \ud611\uc870\ud55c \uac83\ub3c4 \uc788\ub2e4. \uc774\ub97c \ube57\ub300\uc5b4 \uc815\uce58\uad8c\uacfc \uc778\ud130\ub137 \uc0c1\uc5d0\uc11c '\uc774\uba85\ubc15\uadfc\ud61c'\ub85c \ud45c\ud604\ud558\uae30\ub3c4 \ud55c\ub2e4. \uadf8\ub7ec\ub098 \uc774\uba85\ubc15\uacfc \ub2e4\ub974\ub2e4\uace0 \ubcfc \uc218 \uc788\ub294 \uac83\ub4e4\ub3c4 \uc788\ub2e4. \ub300\ud45c\uc801\uc73c\ub85c \uc774\uba85\ubc15 \ub300\ud1b5\ub839\uc774 \uc8fc\ub3c4\ud588\ub358 '\uc138\uc885\uc2dc \uc218\uc815\uc548'\uc744 \ubd80\uacb0\ub418\uac8c \ub9cc\ub4e0 \uac83\uc774\ub2e4. \ub610\ud55c \uc11c\uc6b8\uc2dc\uc758 \ubb34\uc0c1 \uae09\uc2dd \uc815\ucc45 \ub17c\ub780 \ub54c \ubc14\uc05c \uc77c\uc815 \uc18d\uc5d0 \ubd80\uc7ac\uc790 \ud22c\ud45c\uae4c\uc9c0 \ucc38\uc5ec\ud574 \uc624\uc138\ud6c8 \uc804 \uc11c\uc6b8\uc2dc\uc7a5\uc744 \uc9c0\uc6d0\ud588\ub358 \uc774\uba85\ubc15\uacfc\ub294 \ub2ec\ub9ac \uad00\ub828 \uc0ac\uc548\uc5d0 \uc785\uc7a5\uc744 \ubc1d\ud788\uc9c0 \uc54a\uc558\ub2e4. 2011\ub144 \ub9d0 \ube44\uc0c1\ub300\ucc45\uc704\uc6d0\uc7a5\uc744 \ub9e1\uc740 \ud6c4 \ub2f9\uba85, \ub2f9 \uc0c1\uc9d5\uc0c9, \uac15\ub839 \ub4f1\uc744 \ubcc0\uacbd\ud558\uba74\uc11c MB\uc815\uad8c\uacfc \uae30\uc874 \ud55c\ub098\ub77c\ub2f9\uacfc\uc758 \ucc28\ubcc4\ud654\ub97c \uc2dc\ub3c4\ud588\ub2e4. 2012\ub144 4\uc6d4 \ucd1d\uc120\uc744 \uc55e\ub450\uace0 \ubd88\uac70\uc9c4 \uad6d\ubb34\ucd1d\ub9ac\uc2e4\uc758 \ubbfc\uac04\uc778 \uc0ac\ucc30 \uc0ac\uac74 \uc545\uc7ac \uc18d\uc5d0\uc11c\ub3c4 \"\uc5b4\ub290 \uc815\uad8c\uc5d0\uc11c\ub098 \ubbfc\uac04\uc778 \uc0ac\ucc30\uc740 \uc77c\uc5b4\ub098\uba70 \ub098\ub3c4 \ud53c\ud574\uc790\"\ub77c\uba70 \ub17c\ub780\uc744 \ud53c\ud558\uae30\ub3c4 \ud588\ub2e4."} {"question": "\uc62c\ub9bc\ud53d \ucd5c\uc885 \uc608\uc120 \uc5d4\ud2b8\ub9ac\uac00 \ubc1c\ud45c\ub41c \ub0a0\uc9dc\ub294?", "paragraph": "2007\ub144 3\uc6d4\uc5d0 \ub450\uc0b0 \ubca0\uc5b4\uc2a4\uc758 \uac10\ub3c5\uc744 \ub9e1\uace0 \uc788\ub358 \uae40\uacbd\ubb38 \uac10\ub3c5\uc774 \uc774\ubbf8 \ub300\ud45c\ud300 \uac10\ub3c5\uc73c\ub85c \uc120\uc784\ub418\uc5c8\uace0, 11\uc6d4 30\uc77c\uc5d0 \uc544\uc2dc\uc544 \uc57c\uad6c \uc120\uc218\uad8c \ub300\ud68c \uacb8 \uc62c\ub9bc\ud53d \uc608\uc120\uc758 \ucd5c\uc885 \uc5d4\ud2b8\ub9ac\uac00 \ubc1c\ud45c\ub418\uc5c8\ub2e4. \uadf8\ub294 \uc678\uc57c\uc218 \uc911\uc5d0\uc11c\ub294 \uac19\uc740 \ud300 \ubbfc\ubcd1\ud5cc\uacfc \ud568\uaed8 \ud3ec\ud568\ub418\uc5c8\ub2e4. \ub300\ub9cc\uacfc\uc758 \uccab \uacbd\uae30\uc5d0\uc11c 0-1\ub85c \ub4a4\uc9c0\ub294 \uc0c1\ud669\uc778 5\ud68c\ucd08 2\uc0ac 1, 3\ub8e8\uc5d0\uc11c \ub450 \ubc88\uc9f8 \ud0c0\uc11d\uc5d0 \ub4e4\uc5b4\uc11c \uc120\ubc1c \ud22c\uc218 \ub9b0\uc5b8\uc704\ub85c\ubd80\ud130 \uacb0\uc2b9 \uc5ed\uc804 3\uc810 \ud648\ub7f0\uc744 \ubf51\uc544\ub0c8\ub2e4. \uadf8\ub294 \uc77c\ubcf8\uacfc\uc758 \ub450 \ubc88\uc9f8 \uacbd\uae30\uc5d0\uc11c\ub294 1\ubc88 \ud0c0\uc790\ub85c \uc608\uc0c1\ub418\uc5c8\uc73c\ub098, 10\ubd84\uc804\uc5d0 \uc624\ub354\uac00 \ubc14\ub00c\uba74\uc11c 8\ubc88 \ud0c0\uc790\ub85c \ubc14\ub00c\uc5c8\uace0, 2-4\ub85c \ub4a4\uc9c4 8\ud68c \ub9d0\uc5d0 \ud76c\uc0dd \ud50c\ub77c\uc774\ub97c \uce58\uba74\uc11c 1\ud0c0\uc810\uc744 \uc62c\ub838\uc73c\ub098 \ud300\uc740 4-3\uc73c\ub85c \ud328\ud588\ub2e4. \ud544\ub9ac\ud540\uacfc\uc758 \uc138 \ubc88\uc9f8 \uacbd\uae30\uc5d0\uc11c \uc801\uc2dc\ud0c0\ub85c 1\ud0c0\uc810\uc744 \uc62c\ub838\uace0, \ud300\uc740 13-1\uc758 \uc810\uc218\ub85c \ucf5c\ub4dc \uac8c\uc784\uc73c\ub85c \uc2b9\ub9ac\ud588\ub2e4. \uadf8 \uacb0\uacfc \ub300\ud45c\ud300\uc740 2\uc2b9 1\ud328\ub85c 2\uc704\uc5d0 \uba38\ubb3c\uba70 3\uc2b9\uc744 \uae30\ub85d\ud55c \uc77c\ubcf8\uc5d0\uac8c \ubc00\ub824 \uc62c\ub9bc\ud53d \uc9c1\ud589\uc5d0 \uc2e4\ud328\ud588\uace0, 2\ucc28 \uc608\uc120\uc5d0 \ucc38\uac00\ud558\uac8c \ub418\uc5c8\ub2e4. \uc62c\ub9bc\ud53d \ucd5c\uc885 \uc608\uc120 \uc5d4\ud2b8\ub9ac\ub294 2008\ub144 3\uc6d4 5\uc77c \ubc1c\ud45c\ub418\uc5c8\uace0, \uac19\uc740 \ud300 \uc911\uc5d0\ub294 \ud22c\uc218 \uae40\uc120\uc6b0, \ub0b4\uc57c\uc218 \uae40\ub3d9\uc8fc, \uace0\uc601\ubbfc, \uadf8\ub9ac\uace0 \uc0c1\ubb34\uc5d0 \ubcf5\ubb34 \uc911\uc778 \uc190\uc2dc\ud5cc\uacfc \ud568\uaed8 \ud3ec\ud568\ub418\uc5c8\ub2e4. \uccab \uacbd\uae30 \ub0a8\uc544\uacf5\uc804\uc5d0\uc11c \uc120\ub450\ud0c0\uc790\ub85c \ub098\uc640 \uccab \ud0c0\uc11d \ubab8\uc5d0 \ub9de\ub294 \ubcfc\ub85c \ucd9c\ub8e8\ud558\uace0 \ub3c4\ub8e8\uc5d0 \uc131\uacf5\ud55c \ub4a4 \uc774\uc2b9\uc5fd\uc758 2\ub8e8\ud0c0 \ub54c \ub4dd\uc810\uc744 \ud588\ub2e4. \uadf8\ub7ec\ub098 \uc774\ud6c4 4\ud0c0\uc218 \ubb34\uc548\ud0c0\uc5d0 \uadf8\uce58\uba74\uc11c \ud638\uc8fc\uc804\uc5d0\uc11c\ub294 \uc774\uc6a9\uaddc\uc5d0\uac8c \uc120\ubc1c \ucd9c\uc804\uc744 \ub0b4\uc8fc\uc5c8\ub2e4. \ud638\uc8fc\uc804\uc5d0\uc11c \uc9c4\uac11\uc6a9 \ub300\uc2e0 \ub300\uc8fc\uc790\ub85c \ucd9c\uc7a5\ub418\uc5c8\ub2e4. \uadf8\ub294 9\uc77c \uba55\uc2dc\ucf54\uc804\uc5d0\uc11c\ub294 3-1\ub85c \uc774\uae30\uace0 \uc788\ub294 \uc0c1\ud669\uc5d0\uc11c \uc774\ubc88 \ub300\ud68c \ub300\ud45c\ud300 \uccab 3\ub8e8\ud0c0\ub97c \ud130\ub728\ub9ac\uba70 2\ud0c0\uc810\uc744 \uc62c\ub838\uc73c\uba70, 6\ud0c0\uc218 1\uc548\ud0c0\ub97c \uae30\ub85d\ud588\ub2e4. 10\uc77c \uc2a4\ud398\uc778\uc804\uc5d0\uc11c 2\ubc88 \ud0c0\uc790\ub85c \uc120\ubc1c \ucd9c\uc7a5\ud588\uc73c\uba70, 1\uc548\ud0c0 3\ubcfc\ub137\uc73c\ub85c \ub192\uc740 \ucd9c\ub8e8\uc728\uc744 \ubcf4\uc600\ub2e4.\uc774\ud2c0 \ub4a4 \ub3c5\uc77c\uc804\uc5d0\uc11c 2\ubc88 \ud0c0\uc790\ub85c \ucd9c\uc804, \ubcfc\ub137\uc744 \uc5bb\uc5b4\ub0c8\ub2e4. 13\uc77c \uce90\ub098\ub2e4\uc804\uc5d0\uc11c\ub294 2\ubc88 \ud0c0\uc790\ub85c \uc120\ubc1c \ucd9c\uc7a5\ud574 \uccab \ud0c0\uc11d\uc5d0\uc11c 3\ub8e8\ub97c \uaff0\ub6ab\ub294 \uc801\uc2dc 2\ub8e8\ud0c0\ub97c \ud130\ub728\ub824 1\ud0c0\uc810\uc744 \uc62c\ub838\uc73c\ub098 \ud300\uc740 4-3\uc73c\ub85c \ud328\ud588\ub2e4. \ub300\ud55c\ubbfc\uad6d \ub300\ud45c\ud300\uc740 \ub9c8\uc9c0\ub9c9 \uacbd\uae30 \ub300\ub9cc\uc804\uc5d0\uc11c 4-3\uc73c\ub85c \uc2b9\ub9ac\ud558\uba74\uc11c, 2\uc704(6\uc2b9 1\ud328)\ub85c \uc62c\ub9bc\ud53d \ubcf8\uc120\uc5d0 \uc9c4\ucd9c\ud588\ub2e4. \uc62c\ub9bc\ud53d \uc5d4\ud2b8\ub9ac\ub294 2008\ub144 7\uc6d4 14\uc77c \ubc1c\ud45c\ub418\uc5c8\uace0, \uc678\uc57c\uc218 \uc911\uc5d0\uc11c \uac19\uc740 \ud300 \uae40\ud604\uc218\uc640 \ud568\uaed8 \ud3ec\ud568\ub418\uc5c8\ub2e4. \uc62c\ub9bc\ud53d \uc57c\uad6c \ubcf8\uc120\uc740 8\uc6d4 13\uc77c\ubd80\ud130 \uc2dc\uc791\ub418\uc5c8\ub2e4. \uccab \ub0a0 \ubbf8\uad6d\uc804\uc5d0\uc11c 4\ud0c0\uc218 2\uc548\ud0c0 1\ud0c0\uc810 1\ub4dd\uc810\uc744 \uae30\ub85d\ud588\uace0, \ud2b9\ud788 7-7\uc758 \uc0c1\ud669\uc774\ub358 9\ud68c\ub9d0 1\uc0ac 3\ub8e8 \ub05d\ub0b4\uae30 \ud76c\uc0dd \ud50c\ub77c\uc774\ub97c \uccd0\ub0c8\ub2e4. 15\uc77c \uce90\ub098\ub2e4\uc804\uc5d0\uc11c\ub294 4\ud68c 1\uc0ac\uc5d0\uc11c \ube57\ub9de\uc740 \uc911\uc804\uc548\ud0c0\uc131 \ud0c0\uad6c\ub97c \uc804\ub825\uc9c8\uc8fc\ud574 \ub2e4\uc774\ube59 \uce90\uce58\ub97c \ud558\uba70 1-0\uc758 \ub958\ud604\uc9c4\uc758 \uc644\ubd09\uc2b9\uc5d0 \uc77c\uc870\ud588\ub2e4. \ub2e4\uc74c \ub0a0 \uc77c\ubcf8\uc804\uc5d0\uc11c\ub294 9\ud68c\ucd08 2\uc0ac \uc774\ud6c4 3\ub8e8\uc218 \uc55e\uc5d0 \ub5a8\uc5b4\uc9c0\ub294 \uc808\ubb18\ud55c \uae30\uc2b5\ubc88\ud2b8\ub85c 4-2, \uc774\ud6c4 \ub204 \uc0c1\uc5d0 \ub098\uac04 \uc774\ud6c4 \ub3c4\ub8e8\ub97c \uc2dc\ub3c4\ud588\uace0 \ud3ec\uc218 \uc544\ubca0\uc758 \uc545\uc1a1\uad6c\ub85c 3\ub8e8\uc5d0 \uc548\ucc29\ud588\uace0 3\ub8e8\uc8fc\uc790\uac00 \ud648\uc73c\ub85c \ub4e4\uc5b4\uc624\uba70 \ub610 \ub2e4\uc2dc \ud55c \uc810\uc744 \ucd94\uac00\ud574 5-2\ub97c \ub9cc\ub4e4\uc5b4\ub0c8\ub2e4. 17\uc77c\uc5d0\ub294 3\uc77c\uac04 \ubbf8\ub904\uc84c\ub358 \uc911\uad6d\uc804\uc5d0\uc11c\uc758 11\ud68c \uc2b9\ubd80\uce58\uae30\uc5d0\uc11c 2\ub8e8 \uc8fc\uc790\ub85c \ub098\uac00 \uc774\uc2b9\uc5fd\uc758 \ub05d\ub0b4\uae30 \uc548\ud0c0 \ub54c \ud648\uc5d0 \ub4e4\uc5b4\uc640 \ub4dd\uc810\ud574 1-0\uc73c\ub85c \uc2b9\ub9ac\ud588\ub2e4. 18\uc77c \ub300\ub9cc\uc804\uc5d0\uc11c\ub294 1\ud68c \ub0b4\uc57c \uc548\ud0c0\ub97c \uce58\uace0 \ub4dd\uc810\ud588\uc73c\ub098, 7\ud68c\ub9d0\uc5d0\ub294 \uc678\uc57c \ub72c\uacf5\uc744 \ud39c\uc2a4 \uc55e\uc5d0\uc11c \ub193\uce58\ub294 \uc2e4\uc218\ub97c \ubc94\ud588\ub2e4. 19\uc77c \ucfe0\ubc14\uc804\uc5d0\ub294 7\ud68c\uc5d0 \uad50\uccb4 \ud22c\uc785\ub410\uace0 \uc218\ube44\ub97c \ub9c8\uce58\uace0 \ub4e4\uc5b4\uc120 \ud0c0\uc11d\uc5d0\uc11c 1\ud0c0\uc810 \uc801\uc2dc\ud0c0\ub97c \uccd0 \ub0b4\uba70 \uc410\uae30 \ud0c0\uc810\uc744 \ub9cc\ub4e4\uc5b4\ub0c8\ub2e4. 20\uc77c \ub124\ub35c\ub780\ub4dc\uc804\uc5d0\ub294 5\ud0c0\uc218 3\uc548\ud0c0\ub85c \ud65c\uc57d\ud588\uace0, \uc608\uc120 \uc131\uc801\uc740 27\ud0c0\uc218 10\uc548\ud0c0, \ud0c0\uc728 0.370, 3\ud0c0\uc810, 4\ub4dd\uc810, 2\ubcfc\ub137, 2\ub3c4\ub8e8\ub97c \uae30\ub85d\ud588\ub2e4. \uc77c\ubcf8\uacfc\uc758 \uc900\uacb0\uc2b9\uc804\uc5d0\uc11c \ud070 \ud65c\uc57d\uc740 \ud558\uc9c0 \ubabb\ud588\ub2e4. \ucfe0\ubc14\uc640\uc758 \uacb0\uc2b9\uc804\uc5d0\uc11c\ub294 7\ud68c\ucd08 \ubcfc\ub137\uc73c\ub85c \ucd9c\ub8e8\ud588\uace0, \ub2e4\uc74c \ud0c0\uc790 \uc774\uc6a9\uaddc\uac00 2\ub8e8\ud0c0\ub97c \uccd0\ub0c8\uc73c\ub098 \uc544\uc6c3 \uce74\uc6b4\ud2b8\ub97c \ucc29\uac01\ud55c \uadf8\uac00 \ud648\uc73c\ub85c \ub4e4\uc5b4\uc624\uc9c0 \uc54a\ub294 \uc2e4\uc218\ub97c \ubc94\ud588\ub2e4. \ud558\uc9c0\ub9cc \ub300\ud45c\ud300\uc740 3-2\ub85c \uc2b9\ub9ac\ud558\uba74\uc11c 9\uc804 \uc804\uc2b9 \uae08\uba54\ub2ec\uc744 \uc774\ub8e8\uc5b4\ub0c8\ub2e4.", "answer": "2008\ub144 3\uc6d4 5\uc77c", "sentence": "\uc62c\ub9bc\ud53d \ucd5c\uc885 \uc608\uc120 \uc5d4\ud2b8\ub9ac\ub294 2008\ub144 3\uc6d4 5\uc77c \ubc1c\ud45c\ub418\uc5c8\uace0, \uac19\uc740 \ud300 \uc911\uc5d0\ub294 \ud22c\uc218 \uae40\uc120\uc6b0, \ub0b4\uc57c\uc218 \uae40\ub3d9\uc8fc, \uace0\uc601\ubbfc, \uadf8\ub9ac\uace0 \uc0c1\ubb34\uc5d0 \ubcf5\ubb34 \uc911\uc778 \uc190\uc2dc\ud5cc\uacfc \ud568\uaed8 \ud3ec\ud568\ub418\uc5c8\ub2e4.", "paragraph_sentence": "2007\ub144 3\uc6d4\uc5d0 \ub450\uc0b0 \ubca0\uc5b4\uc2a4\uc758 \uac10\ub3c5\uc744 \ub9e1\uace0 \uc788\ub358 \uae40\uacbd\ubb38 \uac10\ub3c5\uc774 \uc774\ubbf8 \ub300\ud45c\ud300 \uac10\ub3c5\uc73c\ub85c \uc120\uc784\ub418\uc5c8\uace0, 11\uc6d4 30\uc77c\uc5d0 \uc544\uc2dc\uc544 \uc57c\uad6c \uc120\uc218\uad8c \ub300\ud68c \uacb8 \uc62c\ub9bc\ud53d \uc608\uc120\uc758 \ucd5c\uc885 \uc5d4\ud2b8\ub9ac\uac00 \ubc1c\ud45c\ub418\uc5c8\ub2e4. \uadf8\ub294 \uc678\uc57c\uc218 \uc911\uc5d0\uc11c\ub294 \uac19\uc740 \ud300 \ubbfc\ubcd1\ud5cc\uacfc \ud568\uaed8 \ud3ec\ud568\ub418\uc5c8\ub2e4. \ub300\ub9cc\uacfc\uc758 \uccab \uacbd\uae30\uc5d0\uc11c 0-1\ub85c \ub4a4\uc9c0\ub294 \uc0c1\ud669\uc778 5\ud68c\ucd08 2\uc0ac 1, 3\ub8e8\uc5d0\uc11c \ub450 \ubc88\uc9f8 \ud0c0\uc11d\uc5d0 \ub4e4\uc5b4\uc11c \uc120\ubc1c \ud22c\uc218 \ub9b0\uc5b8\uc704\ub85c\ubd80\ud130 \uacb0\uc2b9 \uc5ed\uc804 3\uc810 \ud648\ub7f0\uc744 \ubf51\uc544\ub0c8\ub2e4. \uadf8\ub294 \uc77c\ubcf8\uacfc\uc758 \ub450 \ubc88\uc9f8 \uacbd\uae30\uc5d0\uc11c\ub294 1\ubc88 \ud0c0\uc790\ub85c \uc608\uc0c1\ub418\uc5c8\uc73c\ub098, 10\ubd84\uc804\uc5d0 \uc624\ub354\uac00 \ubc14\ub00c\uba74\uc11c 8\ubc88 \ud0c0\uc790\ub85c \ubc14\ub00c\uc5c8\uace0, 2-4\ub85c \ub4a4\uc9c4 8\ud68c \ub9d0\uc5d0 \ud76c\uc0dd \ud50c\ub77c\uc774\ub97c \uce58\uba74\uc11c 1\ud0c0\uc810\uc744 \uc62c\ub838\uc73c\ub098 \ud300\uc740 4-3\uc73c\ub85c \ud328\ud588\ub2e4. \ud544\ub9ac\ud540\uacfc\uc758 \uc138 \ubc88\uc9f8 \uacbd\uae30\uc5d0\uc11c \uc801\uc2dc\ud0c0\ub85c 1\ud0c0\uc810\uc744 \uc62c\ub838\uace0, \ud300\uc740 13-1\uc758 \uc810\uc218\ub85c \ucf5c\ub4dc \uac8c\uc784\uc73c\ub85c \uc2b9\ub9ac\ud588\ub2e4. \uadf8 \uacb0\uacfc \ub300\ud45c\ud300\uc740 2\uc2b9 1\ud328\ub85c 2\uc704\uc5d0 \uba38\ubb3c\uba70 3\uc2b9\uc744 \uae30\ub85d\ud55c \uc77c\ubcf8\uc5d0\uac8c \ubc00\ub824 \uc62c\ub9bc\ud53d \uc9c1\ud589\uc5d0 \uc2e4\ud328\ud588\uace0, 2\ucc28 \uc608\uc120\uc5d0 \ucc38\uac00\ud558\uac8c \ub418\uc5c8\ub2e4. \uc62c\ub9bc\ud53d \ucd5c\uc885 \uc608\uc120 \uc5d4\ud2b8\ub9ac\ub294 2008\ub144 3\uc6d4 5\uc77c \ubc1c\ud45c\ub418\uc5c8\uace0, \uac19\uc740 \ud300 \uc911\uc5d0\ub294 \ud22c\uc218 \uae40\uc120\uc6b0, \ub0b4\uc57c\uc218 \uae40\ub3d9\uc8fc, \uace0\uc601\ubbfc, \uadf8\ub9ac\uace0 \uc0c1\ubb34\uc5d0 \ubcf5\ubb34 \uc911\uc778 \uc190\uc2dc\ud5cc\uacfc \ud568\uaed8 \ud3ec\ud568\ub418\uc5c8\ub2e4. \uccab \uacbd\uae30 \ub0a8\uc544\uacf5\uc804\uc5d0\uc11c \uc120\ub450\ud0c0\uc790\ub85c \ub098\uc640 \uccab \ud0c0\uc11d \ubab8\uc5d0 \ub9de\ub294 \ubcfc\ub85c \ucd9c\ub8e8\ud558\uace0 \ub3c4\ub8e8\uc5d0 \uc131\uacf5\ud55c \ub4a4 \uc774\uc2b9\uc5fd\uc758 2\ub8e8\ud0c0 \ub54c \ub4dd\uc810\uc744 \ud588\ub2e4. \uadf8\ub7ec\ub098 \uc774\ud6c4 4\ud0c0\uc218 \ubb34\uc548\ud0c0\uc5d0 \uadf8\uce58\uba74\uc11c \ud638\uc8fc\uc804\uc5d0\uc11c\ub294 \uc774\uc6a9\uaddc\uc5d0\uac8c \uc120\ubc1c \ucd9c\uc804\uc744 \ub0b4\uc8fc\uc5c8\ub2e4. \ud638\uc8fc\uc804\uc5d0\uc11c \uc9c4\uac11\uc6a9 \ub300\uc2e0 \ub300\uc8fc\uc790\ub85c \ucd9c\uc7a5\ub418\uc5c8\ub2e4. \uadf8\ub294 9\uc77c \uba55\uc2dc\ucf54\uc804\uc5d0\uc11c\ub294 3-1\ub85c \uc774\uae30\uace0 \uc788\ub294 \uc0c1\ud669\uc5d0\uc11c \uc774\ubc88 \ub300\ud68c \ub300\ud45c\ud300 \uccab 3\ub8e8\ud0c0\ub97c \ud130\ub728\ub9ac\uba70 2\ud0c0\uc810\uc744 \uc62c\ub838\uc73c\uba70, 6\ud0c0\uc218 1\uc548\ud0c0\ub97c \uae30\ub85d\ud588\ub2e4. 10\uc77c \uc2a4\ud398\uc778\uc804\uc5d0\uc11c 2\ubc88 \ud0c0\uc790\ub85c \uc120\ubc1c \ucd9c\uc7a5\ud588\uc73c\uba70, 1\uc548\ud0c0 3\ubcfc\ub137\uc73c\ub85c \ub192\uc740 \ucd9c\ub8e8\uc728\uc744 \ubcf4\uc600\ub2e4.\uc774\ud2c0 \ub4a4 \ub3c5\uc77c\uc804\uc5d0\uc11c 2\ubc88 \ud0c0\uc790\ub85c \ucd9c\uc804, \ubcfc\ub137\uc744 \uc5bb\uc5b4\ub0c8\ub2e4. 13\uc77c \uce90\ub098\ub2e4\uc804\uc5d0\uc11c\ub294 2\ubc88 \ud0c0\uc790\ub85c \uc120\ubc1c \ucd9c\uc7a5\ud574 \uccab \ud0c0\uc11d\uc5d0\uc11c 3\ub8e8\ub97c \uaff0\ub6ab\ub294 \uc801\uc2dc 2\ub8e8\ud0c0\ub97c \ud130\ub728\ub824 1\ud0c0\uc810\uc744 \uc62c\ub838\uc73c\ub098 \ud300\uc740 4-3\uc73c\ub85c \ud328\ud588\ub2e4. \ub300\ud55c\ubbfc\uad6d \ub300\ud45c\ud300\uc740 \ub9c8\uc9c0\ub9c9 \uacbd\uae30 \ub300\ub9cc\uc804\uc5d0\uc11c 4-3\uc73c\ub85c \uc2b9\ub9ac\ud558\uba74\uc11c, 2\uc704(6\uc2b9 1\ud328)\ub85c \uc62c\ub9bc\ud53d \ubcf8\uc120\uc5d0 \uc9c4\ucd9c\ud588\ub2e4. \uc62c\ub9bc\ud53d \uc5d4\ud2b8\ub9ac\ub294 2008\ub144 7\uc6d4 14\uc77c \ubc1c\ud45c\ub418\uc5c8\uace0, \uc678\uc57c\uc218 \uc911\uc5d0\uc11c \uac19\uc740 \ud300 \uae40\ud604\uc218\uc640 \ud568\uaed8 \ud3ec\ud568\ub418\uc5c8\ub2e4. \uc62c\ub9bc\ud53d \uc57c\uad6c \ubcf8\uc120\uc740 8\uc6d4 13\uc77c\ubd80\ud130 \uc2dc\uc791\ub418\uc5c8\ub2e4. \uccab \ub0a0 \ubbf8\uad6d\uc804\uc5d0\uc11c 4\ud0c0\uc218 2\uc548\ud0c0 1\ud0c0\uc810 1\ub4dd\uc810\uc744 \uae30\ub85d\ud588\uace0, \ud2b9\ud788 7-7\uc758 \uc0c1\ud669\uc774\ub358 9\ud68c\ub9d0 1\uc0ac 3\ub8e8 \ub05d\ub0b4\uae30 \ud76c\uc0dd \ud50c\ub77c\uc774\ub97c \uccd0\ub0c8\ub2e4. 15\uc77c \uce90\ub098\ub2e4\uc804\uc5d0\uc11c\ub294 4\ud68c 1\uc0ac\uc5d0\uc11c \ube57\ub9de\uc740 \uc911\uc804\uc548\ud0c0\uc131 \ud0c0\uad6c\ub97c \uc804\ub825\uc9c8\uc8fc\ud574 \ub2e4\uc774\ube59 \uce90\uce58\ub97c \ud558\uba70 1-0\uc758 \ub958\ud604\uc9c4\uc758 \uc644\ubd09\uc2b9\uc5d0 \uc77c\uc870\ud588\ub2e4. \ub2e4\uc74c \ub0a0 \uc77c\ubcf8\uc804\uc5d0\uc11c\ub294 9\ud68c\ucd08 2\uc0ac \uc774\ud6c4 3\ub8e8\uc218 \uc55e\uc5d0 \ub5a8\uc5b4\uc9c0\ub294 \uc808\ubb18\ud55c \uae30\uc2b5\ubc88\ud2b8\ub85c 4-2, \uc774\ud6c4 \ub204 \uc0c1\uc5d0 \ub098\uac04 \uc774\ud6c4 \ub3c4\ub8e8\ub97c \uc2dc\ub3c4\ud588\uace0 \ud3ec\uc218 \uc544\ubca0\uc758 \uc545\uc1a1\uad6c\ub85c 3\ub8e8\uc5d0 \uc548\ucc29\ud588\uace0 3\ub8e8\uc8fc\uc790\uac00 \ud648\uc73c\ub85c \ub4e4\uc5b4\uc624\uba70 \ub610 \ub2e4\uc2dc \ud55c \uc810\uc744 \ucd94\uac00\ud574 5-2\ub97c \ub9cc\ub4e4\uc5b4\ub0c8\ub2e4. 17\uc77c\uc5d0\ub294 3\uc77c\uac04 \ubbf8\ub904\uc84c\ub358 \uc911\uad6d\uc804\uc5d0\uc11c\uc758 11\ud68c \uc2b9\ubd80\uce58\uae30\uc5d0\uc11c 2\ub8e8 \uc8fc\uc790\ub85c \ub098\uac00 \uc774\uc2b9\uc5fd\uc758 \ub05d\ub0b4\uae30 \uc548\ud0c0 \ub54c \ud648\uc5d0 \ub4e4\uc5b4\uc640 \ub4dd\uc810\ud574 1-0\uc73c\ub85c \uc2b9\ub9ac\ud588\ub2e4. 18\uc77c \ub300\ub9cc\uc804\uc5d0\uc11c\ub294 1\ud68c \ub0b4\uc57c \uc548\ud0c0\ub97c \uce58\uace0 \ub4dd\uc810\ud588\uc73c\ub098, 7\ud68c\ub9d0\uc5d0\ub294 \uc678\uc57c \ub72c\uacf5\uc744 \ud39c\uc2a4 \uc55e\uc5d0\uc11c \ub193\uce58\ub294 \uc2e4\uc218\ub97c \ubc94\ud588\ub2e4. 19\uc77c \ucfe0\ubc14\uc804\uc5d0\ub294 7\ud68c\uc5d0 \uad50\uccb4 \ud22c\uc785\ub410\uace0 \uc218\ube44\ub97c \ub9c8\uce58\uace0 \ub4e4\uc5b4\uc120 \ud0c0\uc11d\uc5d0\uc11c 1\ud0c0\uc810 \uc801\uc2dc\ud0c0\ub97c \uccd0 \ub0b4\uba70 \uc410\uae30 \ud0c0\uc810\uc744 \ub9cc\ub4e4\uc5b4\ub0c8\ub2e4. 20\uc77c \ub124\ub35c\ub780\ub4dc\uc804\uc5d0\ub294 5\ud0c0\uc218 3\uc548\ud0c0\ub85c \ud65c\uc57d\ud588\uace0, \uc608\uc120 \uc131\uc801\uc740 27\ud0c0\uc218 10\uc548\ud0c0, \ud0c0\uc728 0.370, 3\ud0c0\uc810, 4\ub4dd\uc810, 2\ubcfc\ub137, 2\ub3c4\ub8e8\ub97c \uae30\ub85d\ud588\ub2e4. \uc77c\ubcf8\uacfc\uc758 \uc900\uacb0\uc2b9\uc804\uc5d0\uc11c \ud070 \ud65c\uc57d\uc740 \ud558\uc9c0 \ubabb\ud588\ub2e4. \ucfe0\ubc14\uc640\uc758 \uacb0\uc2b9\uc804\uc5d0\uc11c\ub294 7\ud68c\ucd08 \ubcfc\ub137\uc73c\ub85c \ucd9c\ub8e8\ud588\uace0, \ub2e4\uc74c \ud0c0\uc790 \uc774\uc6a9\uaddc\uac00 2\ub8e8\ud0c0\ub97c \uccd0\ub0c8\uc73c\ub098 \uc544\uc6c3 \uce74\uc6b4\ud2b8\ub97c \ucc29\uac01\ud55c \uadf8\uac00 \ud648\uc73c\ub85c \ub4e4\uc5b4\uc624\uc9c0 \uc54a\ub294 \uc2e4\uc218\ub97c \ubc94\ud588\ub2e4. \ud558\uc9c0\ub9cc \ub300\ud45c\ud300\uc740 3-2\ub85c \uc2b9\ub9ac\ud558\uba74\uc11c 9\uc804 \uc804\uc2b9 \uae08\uba54\ub2ec\uc744 \uc774\ub8e8\uc5b4\ub0c8\ub2e4.", "paragraph_answer": "2007\ub144 3\uc6d4\uc5d0 \ub450\uc0b0 \ubca0\uc5b4\uc2a4\uc758 \uac10\ub3c5\uc744 \ub9e1\uace0 \uc788\ub358 \uae40\uacbd\ubb38 \uac10\ub3c5\uc774 \uc774\ubbf8 \ub300\ud45c\ud300 \uac10\ub3c5\uc73c\ub85c \uc120\uc784\ub418\uc5c8\uace0, 11\uc6d4 30\uc77c\uc5d0 \uc544\uc2dc\uc544 \uc57c\uad6c \uc120\uc218\uad8c \ub300\ud68c \uacb8 \uc62c\ub9bc\ud53d \uc608\uc120\uc758 \ucd5c\uc885 \uc5d4\ud2b8\ub9ac\uac00 \ubc1c\ud45c\ub418\uc5c8\ub2e4. \uadf8\ub294 \uc678\uc57c\uc218 \uc911\uc5d0\uc11c\ub294 \uac19\uc740 \ud300 \ubbfc\ubcd1\ud5cc\uacfc \ud568\uaed8 \ud3ec\ud568\ub418\uc5c8\ub2e4. \ub300\ub9cc\uacfc\uc758 \uccab \uacbd\uae30\uc5d0\uc11c 0-1\ub85c \ub4a4\uc9c0\ub294 \uc0c1\ud669\uc778 5\ud68c\ucd08 2\uc0ac 1, 3\ub8e8\uc5d0\uc11c \ub450 \ubc88\uc9f8 \ud0c0\uc11d\uc5d0 \ub4e4\uc5b4\uc11c \uc120\ubc1c \ud22c\uc218 \ub9b0\uc5b8\uc704\ub85c\ubd80\ud130 \uacb0\uc2b9 \uc5ed\uc804 3\uc810 \ud648\ub7f0\uc744 \ubf51\uc544\ub0c8\ub2e4. \uadf8\ub294 \uc77c\ubcf8\uacfc\uc758 \ub450 \ubc88\uc9f8 \uacbd\uae30\uc5d0\uc11c\ub294 1\ubc88 \ud0c0\uc790\ub85c \uc608\uc0c1\ub418\uc5c8\uc73c\ub098, 10\ubd84\uc804\uc5d0 \uc624\ub354\uac00 \ubc14\ub00c\uba74\uc11c 8\ubc88 \ud0c0\uc790\ub85c \ubc14\ub00c\uc5c8\uace0, 2-4\ub85c \ub4a4\uc9c4 8\ud68c \ub9d0\uc5d0 \ud76c\uc0dd \ud50c\ub77c\uc774\ub97c \uce58\uba74\uc11c 1\ud0c0\uc810\uc744 \uc62c\ub838\uc73c\ub098 \ud300\uc740 4-3\uc73c\ub85c \ud328\ud588\ub2e4. \ud544\ub9ac\ud540\uacfc\uc758 \uc138 \ubc88\uc9f8 \uacbd\uae30\uc5d0\uc11c \uc801\uc2dc\ud0c0\ub85c 1\ud0c0\uc810\uc744 \uc62c\ub838\uace0, \ud300\uc740 13-1\uc758 \uc810\uc218\ub85c \ucf5c\ub4dc \uac8c\uc784\uc73c\ub85c \uc2b9\ub9ac\ud588\ub2e4. \uadf8 \uacb0\uacfc \ub300\ud45c\ud300\uc740 2\uc2b9 1\ud328\ub85c 2\uc704\uc5d0 \uba38\ubb3c\uba70 3\uc2b9\uc744 \uae30\ub85d\ud55c \uc77c\ubcf8\uc5d0\uac8c \ubc00\ub824 \uc62c\ub9bc\ud53d \uc9c1\ud589\uc5d0 \uc2e4\ud328\ud588\uace0, 2\ucc28 \uc608\uc120\uc5d0 \ucc38\uac00\ud558\uac8c \ub418\uc5c8\ub2e4. \uc62c\ub9bc\ud53d \ucd5c\uc885 \uc608\uc120 \uc5d4\ud2b8\ub9ac\ub294 2008\ub144 3\uc6d4 5\uc77c \ubc1c\ud45c\ub418\uc5c8\uace0, \uac19\uc740 \ud300 \uc911\uc5d0\ub294 \ud22c\uc218 \uae40\uc120\uc6b0, \ub0b4\uc57c\uc218 \uae40\ub3d9\uc8fc, \uace0\uc601\ubbfc, \uadf8\ub9ac\uace0 \uc0c1\ubb34\uc5d0 \ubcf5\ubb34 \uc911\uc778 \uc190\uc2dc\ud5cc\uacfc \ud568\uaed8 \ud3ec\ud568\ub418\uc5c8\ub2e4. \uccab \uacbd\uae30 \ub0a8\uc544\uacf5\uc804\uc5d0\uc11c \uc120\ub450\ud0c0\uc790\ub85c \ub098\uc640 \uccab \ud0c0\uc11d \ubab8\uc5d0 \ub9de\ub294 \ubcfc\ub85c \ucd9c\ub8e8\ud558\uace0 \ub3c4\ub8e8\uc5d0 \uc131\uacf5\ud55c \ub4a4 \uc774\uc2b9\uc5fd\uc758 2\ub8e8\ud0c0 \ub54c \ub4dd\uc810\uc744 \ud588\ub2e4. \uadf8\ub7ec\ub098 \uc774\ud6c4 4\ud0c0\uc218 \ubb34\uc548\ud0c0\uc5d0 \uadf8\uce58\uba74\uc11c \ud638\uc8fc\uc804\uc5d0\uc11c\ub294 \uc774\uc6a9\uaddc\uc5d0\uac8c \uc120\ubc1c \ucd9c\uc804\uc744 \ub0b4\uc8fc\uc5c8\ub2e4. \ud638\uc8fc\uc804\uc5d0\uc11c \uc9c4\uac11\uc6a9 \ub300\uc2e0 \ub300\uc8fc\uc790\ub85c \ucd9c\uc7a5\ub418\uc5c8\ub2e4. \uadf8\ub294 9\uc77c \uba55\uc2dc\ucf54\uc804\uc5d0\uc11c\ub294 3-1\ub85c \uc774\uae30\uace0 \uc788\ub294 \uc0c1\ud669\uc5d0\uc11c \uc774\ubc88 \ub300\ud68c \ub300\ud45c\ud300 \uccab 3\ub8e8\ud0c0\ub97c \ud130\ub728\ub9ac\uba70 2\ud0c0\uc810\uc744 \uc62c\ub838\uc73c\uba70, 6\ud0c0\uc218 1\uc548\ud0c0\ub97c \uae30\ub85d\ud588\ub2e4. 10\uc77c \uc2a4\ud398\uc778\uc804\uc5d0\uc11c 2\ubc88 \ud0c0\uc790\ub85c \uc120\ubc1c \ucd9c\uc7a5\ud588\uc73c\uba70, 1\uc548\ud0c0 3\ubcfc\ub137\uc73c\ub85c \ub192\uc740 \ucd9c\ub8e8\uc728\uc744 \ubcf4\uc600\ub2e4.\uc774\ud2c0 \ub4a4 \ub3c5\uc77c\uc804\uc5d0\uc11c 2\ubc88 \ud0c0\uc790\ub85c \ucd9c\uc804, \ubcfc\ub137\uc744 \uc5bb\uc5b4\ub0c8\ub2e4. 13\uc77c \uce90\ub098\ub2e4\uc804\uc5d0\uc11c\ub294 2\ubc88 \ud0c0\uc790\ub85c \uc120\ubc1c \ucd9c\uc7a5\ud574 \uccab \ud0c0\uc11d\uc5d0\uc11c 3\ub8e8\ub97c \uaff0\ub6ab\ub294 \uc801\uc2dc 2\ub8e8\ud0c0\ub97c \ud130\ub728\ub824 1\ud0c0\uc810\uc744 \uc62c\ub838\uc73c\ub098 \ud300\uc740 4-3\uc73c\ub85c \ud328\ud588\ub2e4. \ub300\ud55c\ubbfc\uad6d \ub300\ud45c\ud300\uc740 \ub9c8\uc9c0\ub9c9 \uacbd\uae30 \ub300\ub9cc\uc804\uc5d0\uc11c 4-3\uc73c\ub85c \uc2b9\ub9ac\ud558\uba74\uc11c, 2\uc704(6\uc2b9 1\ud328)\ub85c \uc62c\ub9bc\ud53d \ubcf8\uc120\uc5d0 \uc9c4\ucd9c\ud588\ub2e4. \uc62c\ub9bc\ud53d \uc5d4\ud2b8\ub9ac\ub294 2008\ub144 7\uc6d4 14\uc77c \ubc1c\ud45c\ub418\uc5c8\uace0, \uc678\uc57c\uc218 \uc911\uc5d0\uc11c \uac19\uc740 \ud300 \uae40\ud604\uc218\uc640 \ud568\uaed8 \ud3ec\ud568\ub418\uc5c8\ub2e4. \uc62c\ub9bc\ud53d \uc57c\uad6c \ubcf8\uc120\uc740 8\uc6d4 13\uc77c\ubd80\ud130 \uc2dc\uc791\ub418\uc5c8\ub2e4. \uccab \ub0a0 \ubbf8\uad6d\uc804\uc5d0\uc11c 4\ud0c0\uc218 2\uc548\ud0c0 1\ud0c0\uc810 1\ub4dd\uc810\uc744 \uae30\ub85d\ud588\uace0, \ud2b9\ud788 7-7\uc758 \uc0c1\ud669\uc774\ub358 9\ud68c\ub9d0 1\uc0ac 3\ub8e8 \ub05d\ub0b4\uae30 \ud76c\uc0dd \ud50c\ub77c\uc774\ub97c \uccd0\ub0c8\ub2e4. 15\uc77c \uce90\ub098\ub2e4\uc804\uc5d0\uc11c\ub294 4\ud68c 1\uc0ac\uc5d0\uc11c \ube57\ub9de\uc740 \uc911\uc804\uc548\ud0c0\uc131 \ud0c0\uad6c\ub97c \uc804\ub825\uc9c8\uc8fc\ud574 \ub2e4\uc774\ube59 \uce90\uce58\ub97c \ud558\uba70 1-0\uc758 \ub958\ud604\uc9c4\uc758 \uc644\ubd09\uc2b9\uc5d0 \uc77c\uc870\ud588\ub2e4. \ub2e4\uc74c \ub0a0 \uc77c\ubcf8\uc804\uc5d0\uc11c\ub294 9\ud68c\ucd08 2\uc0ac \uc774\ud6c4 3\ub8e8\uc218 \uc55e\uc5d0 \ub5a8\uc5b4\uc9c0\ub294 \uc808\ubb18\ud55c \uae30\uc2b5\ubc88\ud2b8\ub85c 4-2, \uc774\ud6c4 \ub204 \uc0c1\uc5d0 \ub098\uac04 \uc774\ud6c4 \ub3c4\ub8e8\ub97c \uc2dc\ub3c4\ud588\uace0 \ud3ec\uc218 \uc544\ubca0\uc758 \uc545\uc1a1\uad6c\ub85c 3\ub8e8\uc5d0 \uc548\ucc29\ud588\uace0 3\ub8e8\uc8fc\uc790\uac00 \ud648\uc73c\ub85c \ub4e4\uc5b4\uc624\uba70 \ub610 \ub2e4\uc2dc \ud55c \uc810\uc744 \ucd94\uac00\ud574 5-2\ub97c \ub9cc\ub4e4\uc5b4\ub0c8\ub2e4. 17\uc77c\uc5d0\ub294 3\uc77c\uac04 \ubbf8\ub904\uc84c\ub358 \uc911\uad6d\uc804\uc5d0\uc11c\uc758 11\ud68c \uc2b9\ubd80\uce58\uae30\uc5d0\uc11c 2\ub8e8 \uc8fc\uc790\ub85c \ub098\uac00 \uc774\uc2b9\uc5fd\uc758 \ub05d\ub0b4\uae30 \uc548\ud0c0 \ub54c \ud648\uc5d0 \ub4e4\uc5b4\uc640 \ub4dd\uc810\ud574 1-0\uc73c\ub85c \uc2b9\ub9ac\ud588\ub2e4. 18\uc77c \ub300\ub9cc\uc804\uc5d0\uc11c\ub294 1\ud68c \ub0b4\uc57c \uc548\ud0c0\ub97c \uce58\uace0 \ub4dd\uc810\ud588\uc73c\ub098, 7\ud68c\ub9d0\uc5d0\ub294 \uc678\uc57c \ub72c\uacf5\uc744 \ud39c\uc2a4 \uc55e\uc5d0\uc11c \ub193\uce58\ub294 \uc2e4\uc218\ub97c \ubc94\ud588\ub2e4. 19\uc77c \ucfe0\ubc14\uc804\uc5d0\ub294 7\ud68c\uc5d0 \uad50\uccb4 \ud22c\uc785\ub410\uace0 \uc218\ube44\ub97c \ub9c8\uce58\uace0 \ub4e4\uc5b4\uc120 \ud0c0\uc11d\uc5d0\uc11c 1\ud0c0\uc810 \uc801\uc2dc\ud0c0\ub97c \uccd0 \ub0b4\uba70 \uc410\uae30 \ud0c0\uc810\uc744 \ub9cc\ub4e4\uc5b4\ub0c8\ub2e4. 20\uc77c \ub124\ub35c\ub780\ub4dc\uc804\uc5d0\ub294 5\ud0c0\uc218 3\uc548\ud0c0\ub85c \ud65c\uc57d\ud588\uace0, \uc608\uc120 \uc131\uc801\uc740 27\ud0c0\uc218 10\uc548\ud0c0, \ud0c0\uc728 0.370, 3\ud0c0\uc810, 4\ub4dd\uc810, 2\ubcfc\ub137, 2\ub3c4\ub8e8\ub97c \uae30\ub85d\ud588\ub2e4. \uc77c\ubcf8\uacfc\uc758 \uc900\uacb0\uc2b9\uc804\uc5d0\uc11c \ud070 \ud65c\uc57d\uc740 \ud558\uc9c0 \ubabb\ud588\ub2e4. \ucfe0\ubc14\uc640\uc758 \uacb0\uc2b9\uc804\uc5d0\uc11c\ub294 7\ud68c\ucd08 \ubcfc\ub137\uc73c\ub85c \ucd9c\ub8e8\ud588\uace0, \ub2e4\uc74c \ud0c0\uc790 \uc774\uc6a9\uaddc\uac00 2\ub8e8\ud0c0\ub97c \uccd0\ub0c8\uc73c\ub098 \uc544\uc6c3 \uce74\uc6b4\ud2b8\ub97c \ucc29\uac01\ud55c \uadf8\uac00 \ud648\uc73c\ub85c \ub4e4\uc5b4\uc624\uc9c0 \uc54a\ub294 \uc2e4\uc218\ub97c \ubc94\ud588\ub2e4. \ud558\uc9c0\ub9cc \ub300\ud45c\ud300\uc740 3-2\ub85c \uc2b9\ub9ac\ud558\uba74\uc11c 9\uc804 \uc804\uc2b9 \uae08\uba54\ub2ec\uc744 \uc774\ub8e8\uc5b4\ub0c8\ub2e4.", "sentence_answer": "\uc62c\ub9bc\ud53d \ucd5c\uc885 \uc608\uc120 \uc5d4\ud2b8\ub9ac\ub294 2008\ub144 3\uc6d4 5\uc77c \ubc1c\ud45c\ub418\uc5c8\uace0, \uac19\uc740 \ud300 \uc911\uc5d0\ub294 \ud22c\uc218 \uae40\uc120\uc6b0, \ub0b4\uc57c\uc218 \uae40\ub3d9\uc8fc, \uace0\uc601\ubbfc, \uadf8\ub9ac\uace0 \uc0c1\ubb34\uc5d0 \ubcf5\ubb34 \uc911\uc778 \uc190\uc2dc\ud5cc\uacfc \ud568\uaed8 \ud3ec\ud568\ub418\uc5c8\ub2e4.", "paragraph_id": "6547804-12-1", "paragraph_question": "question: \uc62c\ub9bc\ud53d \ucd5c\uc885 \uc608\uc120 \uc5d4\ud2b8\ub9ac\uac00 \ubc1c\ud45c\ub41c \ub0a0\uc9dc\ub294?, context: 2007\ub144 3\uc6d4\uc5d0 \ub450\uc0b0 \ubca0\uc5b4\uc2a4\uc758 \uac10\ub3c5\uc744 \ub9e1\uace0 \uc788\ub358 \uae40\uacbd\ubb38 \uac10\ub3c5\uc774 \uc774\ubbf8 \ub300\ud45c\ud300 \uac10\ub3c5\uc73c\ub85c \uc120\uc784\ub418\uc5c8\uace0, 11\uc6d4 30\uc77c\uc5d0 \uc544\uc2dc\uc544 \uc57c\uad6c \uc120\uc218\uad8c \ub300\ud68c \uacb8 \uc62c\ub9bc\ud53d \uc608\uc120\uc758 \ucd5c\uc885 \uc5d4\ud2b8\ub9ac\uac00 \ubc1c\ud45c\ub418\uc5c8\ub2e4. \uadf8\ub294 \uc678\uc57c\uc218 \uc911\uc5d0\uc11c\ub294 \uac19\uc740 \ud300 \ubbfc\ubcd1\ud5cc\uacfc \ud568\uaed8 \ud3ec\ud568\ub418\uc5c8\ub2e4. \ub300\ub9cc\uacfc\uc758 \uccab \uacbd\uae30\uc5d0\uc11c 0-1\ub85c \ub4a4\uc9c0\ub294 \uc0c1\ud669\uc778 5\ud68c\ucd08 2\uc0ac 1, 3\ub8e8\uc5d0\uc11c \ub450 \ubc88\uc9f8 \ud0c0\uc11d\uc5d0 \ub4e4\uc5b4\uc11c \uc120\ubc1c \ud22c\uc218 \ub9b0\uc5b8\uc704\ub85c\ubd80\ud130 \uacb0\uc2b9 \uc5ed\uc804 3\uc810 \ud648\ub7f0\uc744 \ubf51\uc544\ub0c8\ub2e4. \uadf8\ub294 \uc77c\ubcf8\uacfc\uc758 \ub450 \ubc88\uc9f8 \uacbd\uae30\uc5d0\uc11c\ub294 1\ubc88 \ud0c0\uc790\ub85c \uc608\uc0c1\ub418\uc5c8\uc73c\ub098, 10\ubd84\uc804\uc5d0 \uc624\ub354\uac00 \ubc14\ub00c\uba74\uc11c 8\ubc88 \ud0c0\uc790\ub85c \ubc14\ub00c\uc5c8\uace0, 2-4\ub85c \ub4a4\uc9c4 8\ud68c \ub9d0\uc5d0 \ud76c\uc0dd \ud50c\ub77c\uc774\ub97c \uce58\uba74\uc11c 1\ud0c0\uc810\uc744 \uc62c\ub838\uc73c\ub098 \ud300\uc740 4-3\uc73c\ub85c \ud328\ud588\ub2e4. \ud544\ub9ac\ud540\uacfc\uc758 \uc138 \ubc88\uc9f8 \uacbd\uae30\uc5d0\uc11c \uc801\uc2dc\ud0c0\ub85c 1\ud0c0\uc810\uc744 \uc62c\ub838\uace0, \ud300\uc740 13-1\uc758 \uc810\uc218\ub85c \ucf5c\ub4dc \uac8c\uc784\uc73c\ub85c \uc2b9\ub9ac\ud588\ub2e4. \uadf8 \uacb0\uacfc \ub300\ud45c\ud300\uc740 2\uc2b9 1\ud328\ub85c 2\uc704\uc5d0 \uba38\ubb3c\uba70 3\uc2b9\uc744 \uae30\ub85d\ud55c \uc77c\ubcf8\uc5d0\uac8c \ubc00\ub824 \uc62c\ub9bc\ud53d \uc9c1\ud589\uc5d0 \uc2e4\ud328\ud588\uace0, 2\ucc28 \uc608\uc120\uc5d0 \ucc38\uac00\ud558\uac8c \ub418\uc5c8\ub2e4. \uc62c\ub9bc\ud53d \ucd5c\uc885 \uc608\uc120 \uc5d4\ud2b8\ub9ac\ub294 2008\ub144 3\uc6d4 5\uc77c \ubc1c\ud45c\ub418\uc5c8\uace0, \uac19\uc740 \ud300 \uc911\uc5d0\ub294 \ud22c\uc218 \uae40\uc120\uc6b0, \ub0b4\uc57c\uc218 \uae40\ub3d9\uc8fc, \uace0\uc601\ubbfc, \uadf8\ub9ac\uace0 \uc0c1\ubb34\uc5d0 \ubcf5\ubb34 \uc911\uc778 \uc190\uc2dc\ud5cc\uacfc \ud568\uaed8 \ud3ec\ud568\ub418\uc5c8\ub2e4. \uccab \uacbd\uae30 \ub0a8\uc544\uacf5\uc804\uc5d0\uc11c \uc120\ub450\ud0c0\uc790\ub85c \ub098\uc640 \uccab \ud0c0\uc11d \ubab8\uc5d0 \ub9de\ub294 \ubcfc\ub85c \ucd9c\ub8e8\ud558\uace0 \ub3c4\ub8e8\uc5d0 \uc131\uacf5\ud55c \ub4a4 \uc774\uc2b9\uc5fd\uc758 2\ub8e8\ud0c0 \ub54c \ub4dd\uc810\uc744 \ud588\ub2e4. \uadf8\ub7ec\ub098 \uc774\ud6c4 4\ud0c0\uc218 \ubb34\uc548\ud0c0\uc5d0 \uadf8\uce58\uba74\uc11c \ud638\uc8fc\uc804\uc5d0\uc11c\ub294 \uc774\uc6a9\uaddc\uc5d0\uac8c \uc120\ubc1c \ucd9c\uc804\uc744 \ub0b4\uc8fc\uc5c8\ub2e4. \ud638\uc8fc\uc804\uc5d0\uc11c \uc9c4\uac11\uc6a9 \ub300\uc2e0 \ub300\uc8fc\uc790\ub85c \ucd9c\uc7a5\ub418\uc5c8\ub2e4. \uadf8\ub294 9\uc77c \uba55\uc2dc\ucf54\uc804\uc5d0\uc11c\ub294 3-1\ub85c \uc774\uae30\uace0 \uc788\ub294 \uc0c1\ud669\uc5d0\uc11c \uc774\ubc88 \ub300\ud68c \ub300\ud45c\ud300 \uccab 3\ub8e8\ud0c0\ub97c \ud130\ub728\ub9ac\uba70 2\ud0c0\uc810\uc744 \uc62c\ub838\uc73c\uba70, 6\ud0c0\uc218 1\uc548\ud0c0\ub97c \uae30\ub85d\ud588\ub2e4. 10\uc77c \uc2a4\ud398\uc778\uc804\uc5d0\uc11c 2\ubc88 \ud0c0\uc790\ub85c \uc120\ubc1c \ucd9c\uc7a5\ud588\uc73c\uba70, 1\uc548\ud0c0 3\ubcfc\ub137\uc73c\ub85c \ub192\uc740 \ucd9c\ub8e8\uc728\uc744 \ubcf4\uc600\ub2e4.\uc774\ud2c0 \ub4a4 \ub3c5\uc77c\uc804\uc5d0\uc11c 2\ubc88 \ud0c0\uc790\ub85c \ucd9c\uc804, \ubcfc\ub137\uc744 \uc5bb\uc5b4\ub0c8\ub2e4. 13\uc77c \uce90\ub098\ub2e4\uc804\uc5d0\uc11c\ub294 2\ubc88 \ud0c0\uc790\ub85c \uc120\ubc1c \ucd9c\uc7a5\ud574 \uccab \ud0c0\uc11d\uc5d0\uc11c 3\ub8e8\ub97c \uaff0\ub6ab\ub294 \uc801\uc2dc 2\ub8e8\ud0c0\ub97c \ud130\ub728\ub824 1\ud0c0\uc810\uc744 \uc62c\ub838\uc73c\ub098 \ud300\uc740 4-3\uc73c\ub85c \ud328\ud588\ub2e4. \ub300\ud55c\ubbfc\uad6d \ub300\ud45c\ud300\uc740 \ub9c8\uc9c0\ub9c9 \uacbd\uae30 \ub300\ub9cc\uc804\uc5d0\uc11c 4-3\uc73c\ub85c \uc2b9\ub9ac\ud558\uba74\uc11c, 2\uc704(6\uc2b9 1\ud328)\ub85c \uc62c\ub9bc\ud53d \ubcf8\uc120\uc5d0 \uc9c4\ucd9c\ud588\ub2e4. \uc62c\ub9bc\ud53d \uc5d4\ud2b8\ub9ac\ub294 2008\ub144 7\uc6d4 14\uc77c \ubc1c\ud45c\ub418\uc5c8\uace0, \uc678\uc57c\uc218 \uc911\uc5d0\uc11c \uac19\uc740 \ud300 \uae40\ud604\uc218\uc640 \ud568\uaed8 \ud3ec\ud568\ub418\uc5c8\ub2e4. \uc62c\ub9bc\ud53d \uc57c\uad6c \ubcf8\uc120\uc740 8\uc6d4 13\uc77c\ubd80\ud130 \uc2dc\uc791\ub418\uc5c8\ub2e4. \uccab \ub0a0 \ubbf8\uad6d\uc804\uc5d0\uc11c 4\ud0c0\uc218 2\uc548\ud0c0 1\ud0c0\uc810 1\ub4dd\uc810\uc744 \uae30\ub85d\ud588\uace0, \ud2b9\ud788 7-7\uc758 \uc0c1\ud669\uc774\ub358 9\ud68c\ub9d0 1\uc0ac 3\ub8e8 \ub05d\ub0b4\uae30 \ud76c\uc0dd \ud50c\ub77c\uc774\ub97c \uccd0\ub0c8\ub2e4. 15\uc77c \uce90\ub098\ub2e4\uc804\uc5d0\uc11c\ub294 4\ud68c 1\uc0ac\uc5d0\uc11c \ube57\ub9de\uc740 \uc911\uc804\uc548\ud0c0\uc131 \ud0c0\uad6c\ub97c \uc804\ub825\uc9c8\uc8fc\ud574 \ub2e4\uc774\ube59 \uce90\uce58\ub97c \ud558\uba70 1-0\uc758 \ub958\ud604\uc9c4\uc758 \uc644\ubd09\uc2b9\uc5d0 \uc77c\uc870\ud588\ub2e4. \ub2e4\uc74c \ub0a0 \uc77c\ubcf8\uc804\uc5d0\uc11c\ub294 9\ud68c\ucd08 2\uc0ac \uc774\ud6c4 3\ub8e8\uc218 \uc55e\uc5d0 \ub5a8\uc5b4\uc9c0\ub294 \uc808\ubb18\ud55c \uae30\uc2b5\ubc88\ud2b8\ub85c 4-2, \uc774\ud6c4 \ub204 \uc0c1\uc5d0 \ub098\uac04 \uc774\ud6c4 \ub3c4\ub8e8\ub97c \uc2dc\ub3c4\ud588\uace0 \ud3ec\uc218 \uc544\ubca0\uc758 \uc545\uc1a1\uad6c\ub85c 3\ub8e8\uc5d0 \uc548\ucc29\ud588\uace0 3\ub8e8\uc8fc\uc790\uac00 \ud648\uc73c\ub85c \ub4e4\uc5b4\uc624\uba70 \ub610 \ub2e4\uc2dc \ud55c \uc810\uc744 \ucd94\uac00\ud574 5-2\ub97c \ub9cc\ub4e4\uc5b4\ub0c8\ub2e4. 17\uc77c\uc5d0\ub294 3\uc77c\uac04 \ubbf8\ub904\uc84c\ub358 \uc911\uad6d\uc804\uc5d0\uc11c\uc758 11\ud68c \uc2b9\ubd80\uce58\uae30\uc5d0\uc11c 2\ub8e8 \uc8fc\uc790\ub85c \ub098\uac00 \uc774\uc2b9\uc5fd\uc758 \ub05d\ub0b4\uae30 \uc548\ud0c0 \ub54c \ud648\uc5d0 \ub4e4\uc5b4\uc640 \ub4dd\uc810\ud574 1-0\uc73c\ub85c \uc2b9\ub9ac\ud588\ub2e4. 18\uc77c \ub300\ub9cc\uc804\uc5d0\uc11c\ub294 1\ud68c \ub0b4\uc57c \uc548\ud0c0\ub97c \uce58\uace0 \ub4dd\uc810\ud588\uc73c\ub098, 7\ud68c\ub9d0\uc5d0\ub294 \uc678\uc57c \ub72c\uacf5\uc744 \ud39c\uc2a4 \uc55e\uc5d0\uc11c \ub193\uce58\ub294 \uc2e4\uc218\ub97c \ubc94\ud588\ub2e4. 19\uc77c \ucfe0\ubc14\uc804\uc5d0\ub294 7\ud68c\uc5d0 \uad50\uccb4 \ud22c\uc785\ub410\uace0 \uc218\ube44\ub97c \ub9c8\uce58\uace0 \ub4e4\uc5b4\uc120 \ud0c0\uc11d\uc5d0\uc11c 1\ud0c0\uc810 \uc801\uc2dc\ud0c0\ub97c \uccd0 \ub0b4\uba70 \uc410\uae30 \ud0c0\uc810\uc744 \ub9cc\ub4e4\uc5b4\ub0c8\ub2e4. 20\uc77c \ub124\ub35c\ub780\ub4dc\uc804\uc5d0\ub294 5\ud0c0\uc218 3\uc548\ud0c0\ub85c \ud65c\uc57d\ud588\uace0, \uc608\uc120 \uc131\uc801\uc740 27\ud0c0\uc218 10\uc548\ud0c0, \ud0c0\uc728 0.370, 3\ud0c0\uc810, 4\ub4dd\uc810, 2\ubcfc\ub137, 2\ub3c4\ub8e8\ub97c \uae30\ub85d\ud588\ub2e4. \uc77c\ubcf8\uacfc\uc758 \uc900\uacb0\uc2b9\uc804\uc5d0\uc11c \ud070 \ud65c\uc57d\uc740 \ud558\uc9c0 \ubabb\ud588\ub2e4. \ucfe0\ubc14\uc640\uc758 \uacb0\uc2b9\uc804\uc5d0\uc11c\ub294 7\ud68c\ucd08 \ubcfc\ub137\uc73c\ub85c \ucd9c\ub8e8\ud588\uace0, \ub2e4\uc74c \ud0c0\uc790 \uc774\uc6a9\uaddc\uac00 2\ub8e8\ud0c0\ub97c \uccd0\ub0c8\uc73c\ub098 \uc544\uc6c3 \uce74\uc6b4\ud2b8\ub97c \ucc29\uac01\ud55c \uadf8\uac00 \ud648\uc73c\ub85c \ub4e4\uc5b4\uc624\uc9c0 \uc54a\ub294 \uc2e4\uc218\ub97c \ubc94\ud588\ub2e4. \ud558\uc9c0\ub9cc \ub300\ud45c\ud300\uc740 3-2\ub85c \uc2b9\ub9ac\ud558\uba74\uc11c 9\uc804 \uc804\uc2b9 \uae08\uba54\ub2ec\uc744 \uc774\ub8e8\uc5b4\ub0c8\ub2e4."} {"question": "\ub178\ubb34\ud604 \uc804 \ub300\ud1b5\ub839\uc744 \uc560\ub3c4\ud558\uba70 \ub300\uc911\uad6d \uad00\uacc4\uc758 \uc911\uc2dc,\ud55c\uc911 \uad00\uacc4\uc758 \uc804\uba74\uc801 \ubc1c\uc804 \ucd94\uc9c4\uc744 \uc704\ud574 \uae30\uc6b8\uc778 \uacf5\ud5cc\uc744 \uae30\uc5b5\ud558\uac8c\ub2e4\uace0 \ub9d0\ud55c \uc778\ubb3c\uc740 \ub204\uad6c\uc778\uac00?", "paragraph": "\ud6c4\uc9c4\ud0c0\uc624 \uc911\ud654\uc778\ubbfc\uacf5\ud654\uad6d \uc8fc\uc11d\uc740 \uc870\ubb38\uc5d0\uc11c \"\ub178 \uc804 \ub300\ud1b5\ub839\uc740 \ub098\uc758 \uc624\ub798\ub41c \uce5c\uad6c\"\ub77c\uba70 \"\uc7ac\uc784 \uae30\uac04\uc5d0 \uc911\uad6d\uacfc \ud55c\uad6d\uc758 \uc804\uba74\uc801 \ud611\ub825 \ub3d9\ubc18\uc790 \uad00\uacc4 \uc218\ub9bd \ubc0f \ubc1c\uc804\uc744 \uc704\ud574 \uc911\uc694\ud55c \uae30\uc5ec\ub97c \ud588\ub2e4\"\ub77c\uace0 \ubc1d\ud614\ub2e4. \uc6d0\uc790\ubc14\uc624 \ucd1d\ub9ac\ub294 \uc560\ub3c4\ud558\uba74\uc11c \ub178\ubb34\ud604\uc758 \ub300(\u5c0d)\uc911\uad6d \uad00\uacc4\uc758 \uc911\uc2dc, \ub178\ubb34\ud604\uc758 \uc194\uc9c1\ud568\uacfc \uc131\uc2e4\ud568\uc774 \uae4a\uc740 \uc778\uc0c1\uc744 \ubc1b\uc558\ub2e4\uba70 \ud55c\uc911 \uad00\uacc4\uc758 \uc804\uba74\uc801 \ubc1c\uc804 \ucd94\uc9c4\uc744 \uc704\ud574 \uae30\uc6b8\uc778 \uacf5\ud5cc\uc744 \uae30\uc5b5\ud558\uaca0\ub2e4\ub294 \uc18c\ud68c\ub97c \ub367\ubd99\uc600\ub2e4. \ubbf8\uad6d\uc758 \ubc84\ub77d \uc624\ubc14\ub9c8 \ub300\ud1b5\ub839\uc740 \uae34\uae09 \uc560\ub3c4 \uc131\uba85\uc744 \ubc1c\ud45c\ud558\uc600\ub2e4. \uc774 \uc131\uba85\uc5d0\uc11c \ub178\ubb34\ud604 \uc7ac\uc784 \uae30\uac04\uc5d0 \ud55c\uad6d\uacfc \ubbf8\uad6d \uac04\uc758 '\uac15\ub825\ud558\uace0 \ud65c\uae30\ucc2c'(strong and vital) \uad00\uacc4\ub97c \ud615\uc131\ud558\ub294 \ub370 \uae30\uc5ec\ud588\ub2e4\uba70 \uc560\ub3c4\uc758 \ub73b\uc744 \uc804\ud588\ub2e4. \uc544\uc18c \ub2e4\ub85c \uc77c\ubcf8 \ucd1d\ub9ac\ub294 \uc678\uc0c1 \uc2dc\uc808 \ub178 \uc804 \ub300\ud1b5\ub839\uc744 \ub9cc\ub0ac\ub358 \uae30\uc5b5\uc744 \ub5a0\uc62c\ub9ac\uba70 \uc560\ub3c4\ud588\ub2e4. \uc601\uad6d\uc758 \uc5d8\ub9ac\uc790\ubca0\uc2a4 2\uc138 \uc5ec\uc655\ub3c4 \uccad\uc640\ub300\uc5d0 \uc560\ub3c4 \uc870\ubb38\uc744 \ubcf4\ub0b4\uc5b4, \"\uc9c0\ub09c 2004\ub144 \ub178 \uc804 \ub300\ud1b5\ub839\uc758 \uc601\uad6d \uacf5\uc2dd \ubc29\ubb38\uc740 \ud55c\u00b7\uc601 \uc591\uad6d \uad00\uacc4 \uc99d\uc9c4\uc5d0 \uc911\uc694\ud55c \uc774\uc815\ud45c\uc600\ub2e4\"\ub77c\uace0 \uc804\ud588\ub2e4. \ubc18\uae30\ubb38 \uad6d\uc81c \uc5f0\ud569 \uc0ac\ubb34\ucd1d\uc7a5\uc740 \uc0ac\ub9dd \uad00\ub828 \uc131\uba85\uc744 \ubc1c\ud45c\ud558\uace0 \uc560\ub3c4\uc758 \ub73b\uc744 \ud45c\uba85\ud558\uba74\uc11c, \"\ub178 \uc804 \ub300\ud1b5\ub839\uc740 \ubbfc\uc8fc\uc8fc\uc758 \ucd09\uc9c4\uc744 \uc704\ud574 \uc5ec\ub7ec \uac00\uc9c0 \ub178\ub825\uc744 \uae30\uc6b8\uc600\ub2e4\"\ub77c\uace0 \ub178 \uc804 \ub300\ud1b5\ub839\uc744 \uce6d\uc1a1\ud588\ub2e4. \uace0\ub4e0 \ube0c\ub77c\uc6b4 \uc601\uad6d \ucd1d\ub9ac \uc5ed\uc2dc \uc560\ub3c4\uc758 \ub73b\uc744 \uc804\ud588\ub2e4.", "answer": "\uc6d0\uc790\ubc14\uc624 \ucd1d\ub9ac", "sentence": "\uc6d0\uc790\ubc14\uc624 \ucd1d\ub9ac \ub294 \uc560\ub3c4\ud558\uba74\uc11c \ub178\ubb34\ud604\uc758 \ub300(\u5c0d)\uc911\uad6d \uad00\uacc4\uc758 \uc911\uc2dc, \ub178\ubb34\ud604\uc758 \uc194\uc9c1\ud568\uacfc \uc131\uc2e4\ud568\uc774 \uae4a\uc740 \uc778\uc0c1\uc744 \ubc1b\uc558\ub2e4\uba70 \ud55c\uc911 \uad00\uacc4\uc758 \uc804\uba74\uc801 \ubc1c\uc804 \ucd94\uc9c4\uc744 \uc704\ud574 \uae30\uc6b8\uc778 \uacf5\ud5cc\uc744 \uae30\uc5b5\ud558\uaca0\ub2e4\ub294 \uc18c\ud68c\ub97c \ub367\ubd99\uc600\ub2e4.", "paragraph_sentence": "\ud6c4\uc9c4\ud0c0\uc624 \uc911\ud654\uc778\ubbfc\uacf5\ud654\uad6d \uc8fc\uc11d\uc740 \uc870\ubb38\uc5d0\uc11c \"\ub178 \uc804 \ub300\ud1b5\ub839\uc740 \ub098\uc758 \uc624\ub798\ub41c \uce5c\uad6c\"\ub77c\uba70 \"\uc7ac\uc784 \uae30\uac04\uc5d0 \uc911\uad6d\uacfc \ud55c\uad6d\uc758 \uc804\uba74\uc801 \ud611\ub825 \ub3d9\ubc18\uc790 \uad00\uacc4 \uc218\ub9bd \ubc0f \ubc1c\uc804\uc744 \uc704\ud574 \uc911\uc694\ud55c \uae30\uc5ec\ub97c \ud588\ub2e4\"\ub77c\uace0 \ubc1d\ud614\ub2e4. \uc6d0\uc790\ubc14\uc624 \ucd1d\ub9ac \ub294 \uc560\ub3c4\ud558\uba74\uc11c \ub178\ubb34\ud604\uc758 \ub300(\u5c0d)\uc911\uad6d \uad00\uacc4\uc758 \uc911\uc2dc, \ub178\ubb34\ud604\uc758 \uc194\uc9c1\ud568\uacfc \uc131\uc2e4\ud568\uc774 \uae4a\uc740 \uc778\uc0c1\uc744 \ubc1b\uc558\ub2e4\uba70 \ud55c\uc911 \uad00\uacc4\uc758 \uc804\uba74\uc801 \ubc1c\uc804 \ucd94\uc9c4\uc744 \uc704\ud574 \uae30\uc6b8\uc778 \uacf5\ud5cc\uc744 \uae30\uc5b5\ud558\uaca0\ub2e4\ub294 \uc18c\ud68c\ub97c \ub367\ubd99\uc600\ub2e4. \ubbf8\uad6d\uc758 \ubc84\ub77d \uc624\ubc14\ub9c8 \ub300\ud1b5\ub839\uc740 \uae34\uae09 \uc560\ub3c4 \uc131\uba85\uc744 \ubc1c\ud45c\ud558\uc600\ub2e4. \uc774 \uc131\uba85\uc5d0\uc11c \ub178\ubb34\ud604 \uc7ac\uc784 \uae30\uac04\uc5d0 \ud55c\uad6d\uacfc \ubbf8\uad6d \uac04\uc758 '\uac15\ub825\ud558\uace0 \ud65c\uae30\ucc2c'(strong and vital) \uad00\uacc4\ub97c \ud615\uc131\ud558\ub294 \ub370 \uae30\uc5ec\ud588\ub2e4\uba70 \uc560\ub3c4\uc758 \ub73b\uc744 \uc804\ud588\ub2e4. \uc544\uc18c \ub2e4\ub85c \uc77c\ubcf8 \ucd1d\ub9ac\ub294 \uc678\uc0c1 \uc2dc\uc808 \ub178 \uc804 \ub300\ud1b5\ub839\uc744 \ub9cc\ub0ac\ub358 \uae30\uc5b5\uc744 \ub5a0\uc62c\ub9ac\uba70 \uc560\ub3c4\ud588\ub2e4. \uc601\uad6d\uc758 \uc5d8\ub9ac\uc790\ubca0\uc2a4 2\uc138 \uc5ec\uc655\ub3c4 \uccad\uc640\ub300\uc5d0 \uc560\ub3c4 \uc870\ubb38\uc744 \ubcf4\ub0b4\uc5b4, \"\uc9c0\ub09c 2004\ub144 \ub178 \uc804 \ub300\ud1b5\ub839\uc758 \uc601\uad6d \uacf5\uc2dd \ubc29\ubb38\uc740 \ud55c\u00b7\uc601 \uc591\uad6d \uad00\uacc4 \uc99d\uc9c4\uc5d0 \uc911\uc694\ud55c \uc774\uc815\ud45c\uc600\ub2e4\"\ub77c\uace0 \uc804\ud588\ub2e4. \ubc18\uae30\ubb38 \uad6d\uc81c \uc5f0\ud569 \uc0ac\ubb34\ucd1d\uc7a5\uc740 \uc0ac\ub9dd \uad00\ub828 \uc131\uba85\uc744 \ubc1c\ud45c\ud558\uace0 \uc560\ub3c4\uc758 \ub73b\uc744 \ud45c\uba85\ud558\uba74\uc11c, \"\ub178 \uc804 \ub300\ud1b5\ub839\uc740 \ubbfc\uc8fc\uc8fc\uc758 \ucd09\uc9c4\uc744 \uc704\ud574 \uc5ec\ub7ec \uac00\uc9c0 \ub178\ub825\uc744 \uae30\uc6b8\uc600\ub2e4\"\ub77c\uace0 \ub178 \uc804 \ub300\ud1b5\ub839\uc744 \uce6d\uc1a1\ud588\ub2e4. \uace0\ub4e0 \ube0c\ub77c\uc6b4 \uc601\uad6d \ucd1d\ub9ac \uc5ed\uc2dc \uc560\ub3c4\uc758 \ub73b\uc744 \uc804\ud588\ub2e4.", "paragraph_answer": "\ud6c4\uc9c4\ud0c0\uc624 \uc911\ud654\uc778\ubbfc\uacf5\ud654\uad6d \uc8fc\uc11d\uc740 \uc870\ubb38\uc5d0\uc11c \"\ub178 \uc804 \ub300\ud1b5\ub839\uc740 \ub098\uc758 \uc624\ub798\ub41c \uce5c\uad6c\"\ub77c\uba70 \"\uc7ac\uc784 \uae30\uac04\uc5d0 \uc911\uad6d\uacfc \ud55c\uad6d\uc758 \uc804\uba74\uc801 \ud611\ub825 \ub3d9\ubc18\uc790 \uad00\uacc4 \uc218\ub9bd \ubc0f \ubc1c\uc804\uc744 \uc704\ud574 \uc911\uc694\ud55c \uae30\uc5ec\ub97c \ud588\ub2e4\"\ub77c\uace0 \ubc1d\ud614\ub2e4. \uc6d0\uc790\ubc14\uc624 \ucd1d\ub9ac \ub294 \uc560\ub3c4\ud558\uba74\uc11c \ub178\ubb34\ud604\uc758 \ub300(\u5c0d)\uc911\uad6d \uad00\uacc4\uc758 \uc911\uc2dc, \ub178\ubb34\ud604\uc758 \uc194\uc9c1\ud568\uacfc \uc131\uc2e4\ud568\uc774 \uae4a\uc740 \uc778\uc0c1\uc744 \ubc1b\uc558\ub2e4\uba70 \ud55c\uc911 \uad00\uacc4\uc758 \uc804\uba74\uc801 \ubc1c\uc804 \ucd94\uc9c4\uc744 \uc704\ud574 \uae30\uc6b8\uc778 \uacf5\ud5cc\uc744 \uae30\uc5b5\ud558\uaca0\ub2e4\ub294 \uc18c\ud68c\ub97c \ub367\ubd99\uc600\ub2e4. \ubbf8\uad6d\uc758 \ubc84\ub77d \uc624\ubc14\ub9c8 \ub300\ud1b5\ub839\uc740 \uae34\uae09 \uc560\ub3c4 \uc131\uba85\uc744 \ubc1c\ud45c\ud558\uc600\ub2e4. \uc774 \uc131\uba85\uc5d0\uc11c \ub178\ubb34\ud604 \uc7ac\uc784 \uae30\uac04\uc5d0 \ud55c\uad6d\uacfc \ubbf8\uad6d \uac04\uc758 '\uac15\ub825\ud558\uace0 \ud65c\uae30\ucc2c'(strong and vital) \uad00\uacc4\ub97c \ud615\uc131\ud558\ub294 \ub370 \uae30\uc5ec\ud588\ub2e4\uba70 \uc560\ub3c4\uc758 \ub73b\uc744 \uc804\ud588\ub2e4. \uc544\uc18c \ub2e4\ub85c \uc77c\ubcf8 \ucd1d\ub9ac\ub294 \uc678\uc0c1 \uc2dc\uc808 \ub178 \uc804 \ub300\ud1b5\ub839\uc744 \ub9cc\ub0ac\ub358 \uae30\uc5b5\uc744 \ub5a0\uc62c\ub9ac\uba70 \uc560\ub3c4\ud588\ub2e4. \uc601\uad6d\uc758 \uc5d8\ub9ac\uc790\ubca0\uc2a4 2\uc138 \uc5ec\uc655\ub3c4 \uccad\uc640\ub300\uc5d0 \uc560\ub3c4 \uc870\ubb38\uc744 \ubcf4\ub0b4\uc5b4, \"\uc9c0\ub09c 2004\ub144 \ub178 \uc804 \ub300\ud1b5\ub839\uc758 \uc601\uad6d \uacf5\uc2dd \ubc29\ubb38\uc740 \ud55c\u00b7\uc601 \uc591\uad6d \uad00\uacc4 \uc99d\uc9c4\uc5d0 \uc911\uc694\ud55c \uc774\uc815\ud45c\uc600\ub2e4\"\ub77c\uace0 \uc804\ud588\ub2e4. \ubc18\uae30\ubb38 \uad6d\uc81c \uc5f0\ud569 \uc0ac\ubb34\ucd1d\uc7a5\uc740 \uc0ac\ub9dd \uad00\ub828 \uc131\uba85\uc744 \ubc1c\ud45c\ud558\uace0 \uc560\ub3c4\uc758 \ub73b\uc744 \ud45c\uba85\ud558\uba74\uc11c, \"\ub178 \uc804 \ub300\ud1b5\ub839\uc740 \ubbfc\uc8fc\uc8fc\uc758 \ucd09\uc9c4\uc744 \uc704\ud574 \uc5ec\ub7ec \uac00\uc9c0 \ub178\ub825\uc744 \uae30\uc6b8\uc600\ub2e4\"\ub77c\uace0 \ub178 \uc804 \ub300\ud1b5\ub839\uc744 \uce6d\uc1a1\ud588\ub2e4. \uace0\ub4e0 \ube0c\ub77c\uc6b4 \uc601\uad6d \ucd1d\ub9ac \uc5ed\uc2dc \uc560\ub3c4\uc758 \ub73b\uc744 \uc804\ud588\ub2e4.", "sentence_answer": " \uc6d0\uc790\ubc14\uc624 \ucd1d\ub9ac \ub294 \uc560\ub3c4\ud558\uba74\uc11c \ub178\ubb34\ud604\uc758 \ub300(\u5c0d)\uc911\uad6d \uad00\uacc4\uc758 \uc911\uc2dc, \ub178\ubb34\ud604\uc758 \uc194\uc9c1\ud568\uacfc \uc131\uc2e4\ud568\uc774 \uae4a\uc740 \uc778\uc0c1\uc744 \ubc1b\uc558\ub2e4\uba70 \ud55c\uc911 \uad00\uacc4\uc758 \uc804\uba74\uc801 \ubc1c\uc804 \ucd94\uc9c4\uc744 \uc704\ud574 \uae30\uc6b8\uc778 \uacf5\ud5cc\uc744 \uae30\uc5b5\ud558\uaca0\ub2e4\ub294 \uc18c\ud68c\ub97c \ub367\ubd99\uc600\ub2e4.", "paragraph_id": "6483948-26-0", "paragraph_question": "question: \ub178\ubb34\ud604 \uc804 \ub300\ud1b5\ub839\uc744 \uc560\ub3c4\ud558\uba70 \ub300\uc911\uad6d \uad00\uacc4\uc758 \uc911\uc2dc,\ud55c\uc911 \uad00\uacc4\uc758 \uc804\uba74\uc801 \ubc1c\uc804 \ucd94\uc9c4\uc744 \uc704\ud574 \uae30\uc6b8\uc778 \uacf5\ud5cc\uc744 \uae30\uc5b5\ud558\uac8c\ub2e4\uace0 \ub9d0\ud55c \uc778\ubb3c\uc740 \ub204\uad6c\uc778\uac00?, context: \ud6c4\uc9c4\ud0c0\uc624 \uc911\ud654\uc778\ubbfc\uacf5\ud654\uad6d \uc8fc\uc11d\uc740 \uc870\ubb38\uc5d0\uc11c \"\ub178 \uc804 \ub300\ud1b5\ub839\uc740 \ub098\uc758 \uc624\ub798\ub41c \uce5c\uad6c\"\ub77c\uba70 \"\uc7ac\uc784 \uae30\uac04\uc5d0 \uc911\uad6d\uacfc \ud55c\uad6d\uc758 \uc804\uba74\uc801 \ud611\ub825 \ub3d9\ubc18\uc790 \uad00\uacc4 \uc218\ub9bd \ubc0f \ubc1c\uc804\uc744 \uc704\ud574 \uc911\uc694\ud55c \uae30\uc5ec\ub97c \ud588\ub2e4\"\ub77c\uace0 \ubc1d\ud614\ub2e4. \uc6d0\uc790\ubc14\uc624 \ucd1d\ub9ac\ub294 \uc560\ub3c4\ud558\uba74\uc11c \ub178\ubb34\ud604\uc758 \ub300(\u5c0d)\uc911\uad6d \uad00\uacc4\uc758 \uc911\uc2dc, \ub178\ubb34\ud604\uc758 \uc194\uc9c1\ud568\uacfc \uc131\uc2e4\ud568\uc774 \uae4a\uc740 \uc778\uc0c1\uc744 \ubc1b\uc558\ub2e4\uba70 \ud55c\uc911 \uad00\uacc4\uc758 \uc804\uba74\uc801 \ubc1c\uc804 \ucd94\uc9c4\uc744 \uc704\ud574 \uae30\uc6b8\uc778 \uacf5\ud5cc\uc744 \uae30\uc5b5\ud558\uaca0\ub2e4\ub294 \uc18c\ud68c\ub97c \ub367\ubd99\uc600\ub2e4. \ubbf8\uad6d\uc758 \ubc84\ub77d \uc624\ubc14\ub9c8 \ub300\ud1b5\ub839\uc740 \uae34\uae09 \uc560\ub3c4 \uc131\uba85\uc744 \ubc1c\ud45c\ud558\uc600\ub2e4. \uc774 \uc131\uba85\uc5d0\uc11c \ub178\ubb34\ud604 \uc7ac\uc784 \uae30\uac04\uc5d0 \ud55c\uad6d\uacfc \ubbf8\uad6d \uac04\uc758 '\uac15\ub825\ud558\uace0 \ud65c\uae30\ucc2c'(strong and vital) \uad00\uacc4\ub97c \ud615\uc131\ud558\ub294 \ub370 \uae30\uc5ec\ud588\ub2e4\uba70 \uc560\ub3c4\uc758 \ub73b\uc744 \uc804\ud588\ub2e4. \uc544\uc18c \ub2e4\ub85c \uc77c\ubcf8 \ucd1d\ub9ac\ub294 \uc678\uc0c1 \uc2dc\uc808 \ub178 \uc804 \ub300\ud1b5\ub839\uc744 \ub9cc\ub0ac\ub358 \uae30\uc5b5\uc744 \ub5a0\uc62c\ub9ac\uba70 \uc560\ub3c4\ud588\ub2e4. \uc601\uad6d\uc758 \uc5d8\ub9ac\uc790\ubca0\uc2a4 2\uc138 \uc5ec\uc655\ub3c4 \uccad\uc640\ub300\uc5d0 \uc560\ub3c4 \uc870\ubb38\uc744 \ubcf4\ub0b4\uc5b4, \"\uc9c0\ub09c 2004\ub144 \ub178 \uc804 \ub300\ud1b5\ub839\uc758 \uc601\uad6d \uacf5\uc2dd \ubc29\ubb38\uc740 \ud55c\u00b7\uc601 \uc591\uad6d \uad00\uacc4 \uc99d\uc9c4\uc5d0 \uc911\uc694\ud55c \uc774\uc815\ud45c\uc600\ub2e4\"\ub77c\uace0 \uc804\ud588\ub2e4. \ubc18\uae30\ubb38 \uad6d\uc81c \uc5f0\ud569 \uc0ac\ubb34\ucd1d\uc7a5\uc740 \uc0ac\ub9dd \uad00\ub828 \uc131\uba85\uc744 \ubc1c\ud45c\ud558\uace0 \uc560\ub3c4\uc758 \ub73b\uc744 \ud45c\uba85\ud558\uba74\uc11c, \"\ub178 \uc804 \ub300\ud1b5\ub839\uc740 \ubbfc\uc8fc\uc8fc\uc758 \ucd09\uc9c4\uc744 \uc704\ud574 \uc5ec\ub7ec \uac00\uc9c0 \ub178\ub825\uc744 \uae30\uc6b8\uc600\ub2e4\"\ub77c\uace0 \ub178 \uc804 \ub300\ud1b5\ub839\uc744 \uce6d\uc1a1\ud588\ub2e4. \uace0\ub4e0 \ube0c\ub77c\uc6b4 \uc601\uad6d \ucd1d\ub9ac \uc5ed\uc2dc \uc560\ub3c4\uc758 \ub73b\uc744 \uc804\ud588\ub2e4."} {"question": "2000\ub144\ub300 \uc774\ud6c4\ub97c \uae30\uc900\uc73c\ub85c \uc2e0\uc790\uc720\uc8fc\uc758 \uadf9\uc6b0 \uc6b4\ub3d9\uc740 \ub204\uad6c \uc911\uc2ec\uc73c\ub85c \uc774\ub8e8\uc5b4\uc84c\ub098?", "paragraph": "2000\ub144\ub300 \uc774\ud6c4\ub97c \uae30\uc900\uc73c\ub85c \uc804\ud5a5 \uc8fc\uc0ac\ud30c \ucd9c\uc2e0\uc774 \uc8fc\ucd95\uc774 \ub41c \uc2e0\uc790\uc720\uc8fc\uc758 \uadf9\uc6b0 \uc6b4\ub3d9\uc774\ub77c\uace0 \uc77c\ubd80 \ubd84\uc11d\uac00\ub4e4\uc5d0 \uc758\ud574 \ud3c9\uac00\ubc1b\ub294 \ub274\ub77c\uc774\ud2b8 \uc6b4\ub3d9\uc774 \uc0dd\uaca8\ub0ac\ub2e4. \ub300\ud55c\ubbfc\uad6d\uc758 \uc0ac\ud68c\ud559\uc790 \uc6b0\uc11d\ud6c8\uc740 \ud55c\uad6d\uc758 \uadf9\uc6b0\ub294 \ubc18\uc81c\uad6d\uc8fc\uc758\uc801 \ubbfc\uc871\uc8fc\uc758\uc640\ub294 \uad00\uacc4\uac00 \uc5c6\uc73c\uba70, \uae09\uc9c4\uc801\uc778 \uacbd\uc81c\uc801 \uc790\uc720\ubc29\uc784\uc8fc\uc758 \uc6b0\uc775\uc778 \ub3d9\uc2dc\uc5d0 \ubc18\ubbfc\uc8fc\uc8fc\uc758, \ubc18\uacf5\uc8fc\uc758, \uad70\ube44\uc99d\ucd95, \uc9c0\uc5ed\ucc28\ubcc4\uc8fc\uc758, \uc778\uc885\uc8fc\uc758 \uc131\ud5a5\uc744 \ub760\ub294 \ub124\uc624\ud30c\uc2dc\uc998\uacfc \uc720\uc0ac\ud55c \ubaa8\uc2b5\uc744 \ubcf4\uc774\uace0 \uc774\ub4e4\uc740 \uc2a4\uc2a4\ub85c\ub97c '\uc911\ub3c4\uc8fc\uc758\uc790' \ub610\ub294 '\uc790\uc720\uc8fc\uc758\uc790'\ub77c\uace0 \ubaa8\ud638\ud558\uac8c \uce6d\ud558\ub294 \uacbd\ud5a5\uc774 \uc788\uc73c\ub098, \uacbd\uc81c\uc801 \uc790\uc720\uc8fc\uc758\ub97c \uc8fc\uc7a5\ud55c\ub2e4\ub294 \uac83 \uc774\uc678\uc5d4 \uae30\uc874 \uc790\uc720\uc8fc\uc758\uc640 \ub9e4\uc6b0 \uc0c1\ubc18\ub41c \ubaa8\uc2b5\uc744 \ubcf4\uc774\uace0 \ub3d9\uc2dc\uc5d0 \uc790\uc720\uc8fc\uc758\uc5d0 \ub300\ud55c \uc774\ud574\ub3c4\uac00 \ub9e4\uc6b0 \ubd80\uc871\ud558\ub2e4\uace0 \uc8fc\uc7a5\ud588\ub2e4. \uc815\uce58\ud559\uc790 \ucc28\uc0c1\ucca0\uc740 \ud604\uc7ac \ud55c\uad6d \uadf9\uc6b0\uc758 \uc5ec\ub7ec\uac00\uc9c0 \ud2b9\uc9d5\uc774 \ubd84\ub2e8 \uccb4\uc81c\uc758 \ud2b9\uc218\uc131\uc5d0 \uae30\uc778\ud55c\ub2e4\uace0 \ubd84\uc11d\ud588\ub2e4. \uc77c\ubd80 \uc5b8\ub860\uc5d0\uc120 \uc774\ub4e4 \ub300\ub2e4\uc218\uac00 \ub137\uc6b0\uc775\uc5d0 \uc18d\ud558\uba70, \ud2b9\uc815 \uc778\ud130\ub137 \ucee4\ubba4\ub2c8\ud2f0\uc5d0\uc11c \uc790\uc2e0\ub4e4\uc758 \uc785\uc7a5\uc744 \ud53c\ub825\ud558\ub294 \ub3d9\uc2dc\uc5d0 \uc678\ubd80 \uc138\uacc4\uc5d0\uc11c \uc790\uc2e0\uc758 \uc815\uce58 \uc0ac\uc0c1\uc774 \ub4dc\ub7ec\ub098\ub294 \uac83\uc5d0 \ub300\ud574\uc11c \ub450\ub824\uc6c0\uc744 \uac00\uc9c0\uace0 \uc788\ub2e4\uace0 \ubd84\uc11d\ud558\uba70, \uba87\uba87 \ubd84\uc11d\uac00\ub4e4\uc740 \uc774\ub4e4\uc758 \ud589\ub3d9\uc744 \uc815\uce58\uc801 \uc131\ud5a5\uc5d0 \uc5f0\uad00\uc774 \uc788\ub2e4\uace0 \ubd84\uc11d\ud558\uc9c0 \uc54a\uace0 \ud558\ub098\uc758 \uc815\uc2e0\uc9c8\ud658\uc774\ub77c \uc8fc\uc7a5\ud55c\ub2e4.", "answer": "\uc804\ud5a5 \uc8fc\uc0ac\ud30c \ucd9c\uc2e0", "sentence": "2000\ub144\ub300 \uc774\ud6c4\ub97c \uae30\uc900\uc73c\ub85c \uc804\ud5a5 \uc8fc\uc0ac\ud30c \ucd9c\uc2e0 \uc774 \uc8fc\ucd95\uc774 \ub41c \uc2e0\uc790\uc720\uc8fc\uc758 \uadf9\uc6b0 \uc6b4\ub3d9\uc774\ub77c\uace0 \uc77c\ubd80 \ubd84\uc11d\uac00\ub4e4\uc5d0 \uc758\ud574 \ud3c9\uac00\ubc1b\ub294 \ub274\ub77c\uc774\ud2b8 \uc6b4\ub3d9\uc774 \uc0dd\uaca8\ub0ac\ub2e4.", "paragraph_sentence": " 2000\ub144\ub300 \uc774\ud6c4\ub97c \uae30\uc900\uc73c\ub85c \uc804\ud5a5 \uc8fc\uc0ac\ud30c \ucd9c\uc2e0 \uc774 \uc8fc\ucd95\uc774 \ub41c \uc2e0\uc790\uc720\uc8fc\uc758 \uadf9\uc6b0 \uc6b4\ub3d9\uc774\ub77c\uace0 \uc77c\ubd80 \ubd84\uc11d\uac00\ub4e4\uc5d0 \uc758\ud574 \ud3c9\uac00\ubc1b\ub294 \ub274\ub77c\uc774\ud2b8 \uc6b4\ub3d9\uc774 \uc0dd\uaca8\ub0ac\ub2e4. \ub300\ud55c\ubbfc\uad6d\uc758 \uc0ac\ud68c\ud559\uc790 \uc6b0\uc11d\ud6c8\uc740 \ud55c\uad6d\uc758 \uadf9\uc6b0\ub294 \ubc18\uc81c\uad6d\uc8fc\uc758\uc801 \ubbfc\uc871\uc8fc\uc758\uc640\ub294 \uad00\uacc4\uac00 \uc5c6\uc73c\uba70, \uae09\uc9c4\uc801\uc778 \uacbd\uc81c\uc801 \uc790\uc720\ubc29\uc784\uc8fc\uc758 \uc6b0\uc775\uc778 \ub3d9\uc2dc\uc5d0 \ubc18\ubbfc\uc8fc\uc8fc\uc758, \ubc18\uacf5\uc8fc\uc758, \uad70\ube44\uc99d\ucd95, \uc9c0\uc5ed\ucc28\ubcc4\uc8fc\uc758, \uc778\uc885\uc8fc\uc758 \uc131\ud5a5\uc744 \ub760\ub294 \ub124\uc624\ud30c\uc2dc\uc998\uacfc \uc720\uc0ac\ud55c \ubaa8\uc2b5\uc744 \ubcf4\uc774\uace0 \uc774\ub4e4\uc740 \uc2a4\uc2a4\ub85c\ub97c '\uc911\ub3c4\uc8fc\uc758\uc790' \ub610\ub294 '\uc790\uc720\uc8fc\uc758\uc790'\ub77c\uace0 \ubaa8\ud638\ud558\uac8c \uce6d\ud558\ub294 \uacbd\ud5a5\uc774 \uc788\uc73c\ub098, \uacbd\uc81c\uc801 \uc790\uc720\uc8fc\uc758\ub97c \uc8fc\uc7a5\ud55c\ub2e4\ub294 \uac83 \uc774\uc678\uc5d4 \uae30\uc874 \uc790\uc720\uc8fc\uc758\uc640 \ub9e4\uc6b0 \uc0c1\ubc18\ub41c \ubaa8\uc2b5\uc744 \ubcf4\uc774\uace0 \ub3d9\uc2dc\uc5d0 \uc790\uc720\uc8fc\uc758\uc5d0 \ub300\ud55c \uc774\ud574\ub3c4\uac00 \ub9e4\uc6b0 \ubd80\uc871\ud558\ub2e4\uace0 \uc8fc\uc7a5\ud588\ub2e4. \uc815\uce58\ud559\uc790 \ucc28\uc0c1\ucca0\uc740 \ud604\uc7ac \ud55c\uad6d \uadf9\uc6b0\uc758 \uc5ec\ub7ec\uac00\uc9c0 \ud2b9\uc9d5\uc774 \ubd84\ub2e8 \uccb4\uc81c\uc758 \ud2b9\uc218\uc131\uc5d0 \uae30\uc778\ud55c\ub2e4\uace0 \ubd84\uc11d\ud588\ub2e4. \uc77c\ubd80 \uc5b8\ub860\uc5d0\uc120 \uc774\ub4e4 \ub300\ub2e4\uc218\uac00 \ub137\uc6b0\uc775\uc5d0 \uc18d\ud558\uba70, \ud2b9\uc815 \uc778\ud130\ub137 \ucee4\ubba4\ub2c8\ud2f0\uc5d0\uc11c \uc790\uc2e0\ub4e4\uc758 \uc785\uc7a5\uc744 \ud53c\ub825\ud558\ub294 \ub3d9\uc2dc\uc5d0 \uc678\ubd80 \uc138\uacc4\uc5d0\uc11c \uc790\uc2e0\uc758 \uc815\uce58 \uc0ac\uc0c1\uc774 \ub4dc\ub7ec\ub098\ub294 \uac83\uc5d0 \ub300\ud574\uc11c \ub450\ub824\uc6c0\uc744 \uac00\uc9c0\uace0 \uc788\ub2e4\uace0 \ubd84\uc11d\ud558\uba70, \uba87\uba87 \ubd84\uc11d\uac00\ub4e4\uc740 \uc774\ub4e4\uc758 \ud589\ub3d9\uc744 \uc815\uce58\uc801 \uc131\ud5a5\uc5d0 \uc5f0\uad00\uc774 \uc788\ub2e4\uace0 \ubd84\uc11d\ud558\uc9c0 \uc54a\uace0 \ud558\ub098\uc758 \uc815\uc2e0\uc9c8\ud658\uc774\ub77c \uc8fc\uc7a5\ud55c\ub2e4.", "paragraph_answer": "2000\ub144\ub300 \uc774\ud6c4\ub97c \uae30\uc900\uc73c\ub85c \uc804\ud5a5 \uc8fc\uc0ac\ud30c \ucd9c\uc2e0 \uc774 \uc8fc\ucd95\uc774 \ub41c \uc2e0\uc790\uc720\uc8fc\uc758 \uadf9\uc6b0 \uc6b4\ub3d9\uc774\ub77c\uace0 \uc77c\ubd80 \ubd84\uc11d\uac00\ub4e4\uc5d0 \uc758\ud574 \ud3c9\uac00\ubc1b\ub294 \ub274\ub77c\uc774\ud2b8 \uc6b4\ub3d9\uc774 \uc0dd\uaca8\ub0ac\ub2e4. \ub300\ud55c\ubbfc\uad6d\uc758 \uc0ac\ud68c\ud559\uc790 \uc6b0\uc11d\ud6c8\uc740 \ud55c\uad6d\uc758 \uadf9\uc6b0\ub294 \ubc18\uc81c\uad6d\uc8fc\uc758\uc801 \ubbfc\uc871\uc8fc\uc758\uc640\ub294 \uad00\uacc4\uac00 \uc5c6\uc73c\uba70, \uae09\uc9c4\uc801\uc778 \uacbd\uc81c\uc801 \uc790\uc720\ubc29\uc784\uc8fc\uc758 \uc6b0\uc775\uc778 \ub3d9\uc2dc\uc5d0 \ubc18\ubbfc\uc8fc\uc8fc\uc758, \ubc18\uacf5\uc8fc\uc758, \uad70\ube44\uc99d\ucd95, \uc9c0\uc5ed\ucc28\ubcc4\uc8fc\uc758, \uc778\uc885\uc8fc\uc758 \uc131\ud5a5\uc744 \ub760\ub294 \ub124\uc624\ud30c\uc2dc\uc998\uacfc \uc720\uc0ac\ud55c \ubaa8\uc2b5\uc744 \ubcf4\uc774\uace0 \uc774\ub4e4\uc740 \uc2a4\uc2a4\ub85c\ub97c '\uc911\ub3c4\uc8fc\uc758\uc790' \ub610\ub294 '\uc790\uc720\uc8fc\uc758\uc790'\ub77c\uace0 \ubaa8\ud638\ud558\uac8c \uce6d\ud558\ub294 \uacbd\ud5a5\uc774 \uc788\uc73c\ub098, \uacbd\uc81c\uc801 \uc790\uc720\uc8fc\uc758\ub97c \uc8fc\uc7a5\ud55c\ub2e4\ub294 \uac83 \uc774\uc678\uc5d4 \uae30\uc874 \uc790\uc720\uc8fc\uc758\uc640 \ub9e4\uc6b0 \uc0c1\ubc18\ub41c \ubaa8\uc2b5\uc744 \ubcf4\uc774\uace0 \ub3d9\uc2dc\uc5d0 \uc790\uc720\uc8fc\uc758\uc5d0 \ub300\ud55c \uc774\ud574\ub3c4\uac00 \ub9e4\uc6b0 \ubd80\uc871\ud558\ub2e4\uace0 \uc8fc\uc7a5\ud588\ub2e4. \uc815\uce58\ud559\uc790 \ucc28\uc0c1\ucca0\uc740 \ud604\uc7ac \ud55c\uad6d \uadf9\uc6b0\uc758 \uc5ec\ub7ec\uac00\uc9c0 \ud2b9\uc9d5\uc774 \ubd84\ub2e8 \uccb4\uc81c\uc758 \ud2b9\uc218\uc131\uc5d0 \uae30\uc778\ud55c\ub2e4\uace0 \ubd84\uc11d\ud588\ub2e4. \uc77c\ubd80 \uc5b8\ub860\uc5d0\uc120 \uc774\ub4e4 \ub300\ub2e4\uc218\uac00 \ub137\uc6b0\uc775\uc5d0 \uc18d\ud558\uba70, \ud2b9\uc815 \uc778\ud130\ub137 \ucee4\ubba4\ub2c8\ud2f0\uc5d0\uc11c \uc790\uc2e0\ub4e4\uc758 \uc785\uc7a5\uc744 \ud53c\ub825\ud558\ub294 \ub3d9\uc2dc\uc5d0 \uc678\ubd80 \uc138\uacc4\uc5d0\uc11c \uc790\uc2e0\uc758 \uc815\uce58 \uc0ac\uc0c1\uc774 \ub4dc\ub7ec\ub098\ub294 \uac83\uc5d0 \ub300\ud574\uc11c \ub450\ub824\uc6c0\uc744 \uac00\uc9c0\uace0 \uc788\ub2e4\uace0 \ubd84\uc11d\ud558\uba70, \uba87\uba87 \ubd84\uc11d\uac00\ub4e4\uc740 \uc774\ub4e4\uc758 \ud589\ub3d9\uc744 \uc815\uce58\uc801 \uc131\ud5a5\uc5d0 \uc5f0\uad00\uc774 \uc788\ub2e4\uace0 \ubd84\uc11d\ud558\uc9c0 \uc54a\uace0 \ud558\ub098\uc758 \uc815\uc2e0\uc9c8\ud658\uc774\ub77c \uc8fc\uc7a5\ud55c\ub2e4.", "sentence_answer": "2000\ub144\ub300 \uc774\ud6c4\ub97c \uae30\uc900\uc73c\ub85c \uc804\ud5a5 \uc8fc\uc0ac\ud30c \ucd9c\uc2e0 \uc774 \uc8fc\ucd95\uc774 \ub41c \uc2e0\uc790\uc720\uc8fc\uc758 \uadf9\uc6b0 \uc6b4\ub3d9\uc774\ub77c\uace0 \uc77c\ubd80 \ubd84\uc11d\uac00\ub4e4\uc5d0 \uc758\ud574 \ud3c9\uac00\ubc1b\ub294 \ub274\ub77c\uc774\ud2b8 \uc6b4\ub3d9\uc774 \uc0dd\uaca8\ub0ac\ub2e4.", "paragraph_id": "5975881-1-0", "paragraph_question": "question: 2000\ub144\ub300 \uc774\ud6c4\ub97c \uae30\uc900\uc73c\ub85c \uc2e0\uc790\uc720\uc8fc\uc758 \uadf9\uc6b0 \uc6b4\ub3d9\uc740 \ub204\uad6c \uc911\uc2ec\uc73c\ub85c \uc774\ub8e8\uc5b4\uc84c\ub098?, context: 2000\ub144\ub300 \uc774\ud6c4\ub97c \uae30\uc900\uc73c\ub85c \uc804\ud5a5 \uc8fc\uc0ac\ud30c \ucd9c\uc2e0\uc774 \uc8fc\ucd95\uc774 \ub41c \uc2e0\uc790\uc720\uc8fc\uc758 \uadf9\uc6b0 \uc6b4\ub3d9\uc774\ub77c\uace0 \uc77c\ubd80 \ubd84\uc11d\uac00\ub4e4\uc5d0 \uc758\ud574 \ud3c9\uac00\ubc1b\ub294 \ub274\ub77c\uc774\ud2b8 \uc6b4\ub3d9\uc774 \uc0dd\uaca8\ub0ac\ub2e4. \ub300\ud55c\ubbfc\uad6d\uc758 \uc0ac\ud68c\ud559\uc790 \uc6b0\uc11d\ud6c8\uc740 \ud55c\uad6d\uc758 \uadf9\uc6b0\ub294 \ubc18\uc81c\uad6d\uc8fc\uc758\uc801 \ubbfc\uc871\uc8fc\uc758\uc640\ub294 \uad00\uacc4\uac00 \uc5c6\uc73c\uba70, \uae09\uc9c4\uc801\uc778 \uacbd\uc81c\uc801 \uc790\uc720\ubc29\uc784\uc8fc\uc758 \uc6b0\uc775\uc778 \ub3d9\uc2dc\uc5d0 \ubc18\ubbfc\uc8fc\uc8fc\uc758, \ubc18\uacf5\uc8fc\uc758, \uad70\ube44\uc99d\ucd95, \uc9c0\uc5ed\ucc28\ubcc4\uc8fc\uc758, \uc778\uc885\uc8fc\uc758 \uc131\ud5a5\uc744 \ub760\ub294 \ub124\uc624\ud30c\uc2dc\uc998\uacfc \uc720\uc0ac\ud55c \ubaa8\uc2b5\uc744 \ubcf4\uc774\uace0 \uc774\ub4e4\uc740 \uc2a4\uc2a4\ub85c\ub97c '\uc911\ub3c4\uc8fc\uc758\uc790' \ub610\ub294 '\uc790\uc720\uc8fc\uc758\uc790'\ub77c\uace0 \ubaa8\ud638\ud558\uac8c \uce6d\ud558\ub294 \uacbd\ud5a5\uc774 \uc788\uc73c\ub098, \uacbd\uc81c\uc801 \uc790\uc720\uc8fc\uc758\ub97c \uc8fc\uc7a5\ud55c\ub2e4\ub294 \uac83 \uc774\uc678\uc5d4 \uae30\uc874 \uc790\uc720\uc8fc\uc758\uc640 \ub9e4\uc6b0 \uc0c1\ubc18\ub41c \ubaa8\uc2b5\uc744 \ubcf4\uc774\uace0 \ub3d9\uc2dc\uc5d0 \uc790\uc720\uc8fc\uc758\uc5d0 \ub300\ud55c \uc774\ud574\ub3c4\uac00 \ub9e4\uc6b0 \ubd80\uc871\ud558\ub2e4\uace0 \uc8fc\uc7a5\ud588\ub2e4. \uc815\uce58\ud559\uc790 \ucc28\uc0c1\ucca0\uc740 \ud604\uc7ac \ud55c\uad6d \uadf9\uc6b0\uc758 \uc5ec\ub7ec\uac00\uc9c0 \ud2b9\uc9d5\uc774 \ubd84\ub2e8 \uccb4\uc81c\uc758 \ud2b9\uc218\uc131\uc5d0 \uae30\uc778\ud55c\ub2e4\uace0 \ubd84\uc11d\ud588\ub2e4. \uc77c\ubd80 \uc5b8\ub860\uc5d0\uc120 \uc774\ub4e4 \ub300\ub2e4\uc218\uac00 \ub137\uc6b0\uc775\uc5d0 \uc18d\ud558\uba70, \ud2b9\uc815 \uc778\ud130\ub137 \ucee4\ubba4\ub2c8\ud2f0\uc5d0\uc11c \uc790\uc2e0\ub4e4\uc758 \uc785\uc7a5\uc744 \ud53c\ub825\ud558\ub294 \ub3d9\uc2dc\uc5d0 \uc678\ubd80 \uc138\uacc4\uc5d0\uc11c \uc790\uc2e0\uc758 \uc815\uce58 \uc0ac\uc0c1\uc774 \ub4dc\ub7ec\ub098\ub294 \uac83\uc5d0 \ub300\ud574\uc11c \ub450\ub824\uc6c0\uc744 \uac00\uc9c0\uace0 \uc788\ub2e4\uace0 \ubd84\uc11d\ud558\uba70, \uba87\uba87 \ubd84\uc11d\uac00\ub4e4\uc740 \uc774\ub4e4\uc758 \ud589\ub3d9\uc744 \uc815\uce58\uc801 \uc131\ud5a5\uc5d0 \uc5f0\uad00\uc774 \uc788\ub2e4\uace0 \ubd84\uc11d\ud558\uc9c0 \uc54a\uace0 \ud558\ub098\uc758 \uc815\uc2e0\uc9c8\ud658\uc774\ub77c \uc8fc\uc7a5\ud55c\ub2e4."} {"question": "\uac1c\ubc1c \ub2e8\uacc4\uc5d0\uc11c \uc911\uc694\ud588\ub358 \ubaa9\ud45c\ub294?", "paragraph": "\u300a\uc624\ubc84\uc6cc\uce58\u300b\ub294 \uc0c9\ub9f9\uc744 \uc704\ud55c \uc811\uadfc\uc131 \uc124\uc815\uc744 \ud3ec\ud568, \ud3ed\ub113\uc740 \uc0ac\uc6a9\uc790\ub97c \ud655\ubcf4\ud560 \uc218 \uc788\ub294 \uc5ec\uc12f \uac00\uc9c0 \ud2b9\uc9d5\uc744 \ud568\uaed8 \uac1c\ubc1c\ud558\uc600\ub2e4. \uac1c\ubc1c \ub2e8\uacc4\uc5d0\uc11c \uc911\uc694\ud588\ub358 \ubaa9\ud45c \ud558\ub098\ub294 \ud50c\ub808\uc774\uc5b4\ub97c \uc704\ud55c '\uc804\ud22c\uc758 \uba85\ub8cc\uc131'\uc73c\ub85c, \ud50c\ub808\uc774\uc5b4\uac00 \uc0c8\ub85c\uc6b4 \uc9c0\uc5ed\uc73c\ub85c \uc774\ub3d9\ud558\uba74 \uc801 \uce90\ub9ad\ud130\uac00 \ub69c\ub837\ud558\uac8c \ubcf4\uc774\ub3c4\ub85d \ud588\ub2e4. \uc774\uac83\uc740 \ud50c\ub808\uc774\uc5b4\uac00 \uc804\uc7a5\uc5d0\uc11c \uc0ac\uc6a9\ud558\ub294 \uc0c9\uc870\uc640 \ucc44\ub3c4 \uc218\uc900\uc758 \ub300\uc870, \uba85\ubc31\ud558\uac8c \ub2e4\ub978 \uc724\uacfd\uc744 \uac16\uac8c \ub9cc\ub4e0 \uce90\ub9ad\ud130\ub4e4\ub85c \ud50c\ub808\uc774\uc5b4\uac00 \uba3c \uac70\ub9ac\uc5d0\uc11c \uc601\uc6c5\uc774 \uc544\uad70\uc778\uc9c0 \uc801\uad70\uc778\uc9c0 \uad6c\ubcc4\ud560 \uc218 \uc788\ub3c4\ub85d \ud568\uc73c\ub85c\uc368 \uac00\ub2a5\ud558\uc600\ub2e4. \ud300\uc740 \uac1c\ubc1c\ub3d9\uc548 \uacbd\uae30 \uc911\uac04\uc5d0 \ud50c\ub808\uc774\uc5b4\uac00 \uc601\uc6c5\uc744 \ubc14\uafc0 \uc218 \uc788\ub3c4\ub85d \ud558\ub294 \uac83\uc774 \uac8c\uc784\ud50c\ub808\uc774\uc5d0 \uc911\uc694\ud55c \uc5ed\ud560\uc744 \ud568\uc744 \uc54c\uc544\ub0c8\ub2e4. \u300a\uc624\ubc84\uc6cc\uce58\u300b\ub294 \ucc98\uc74c\uc5d0 \uc18c\uc561 \uacb0\uc81c\ub098 \ub2e4\uc6b4\ub85c\ub4dc \ucf58\ud150\uce20 \uad6c\uc785\uc774 \ud3ec\ud568\ub41c \ubd80\ubd84 \uc720\ub8cc\ud654 \uac8c\uc784 \ud615\ud0dc\ub85c \ucd9c\uc2dc\ub97c \uacc4\ud68d\ud588\uc73c\ub098, \uc774\ub7ec\ud55c \ubc1c\uacac\uc73c\ub85c \uc778\ud574 \ub2e8\ub3c5 \uad6c\ub9e4\ud615 \uac8c\uc784\uc73c\ub85c \ub178\uc120\uc744 \ubcc0\uacbd\ud558\uc600\ub2e4. \ucf08\ub7ec\ub294 \ud300\uc774 \ud50c\ub808\uc774\uc5b4\uac00 \uc0c1\ud669\uc5d0 \ub530\ub77c \uc5b4\ub290 \uc601\uc6c5\uc73c\ub85c\ub4e0 \uad50\uccb4\ud560 \uc218 \uc788\uae30\ub97c \uc6d0\ud588\uace0, \ud50c\ub808\uc774\uc5b4\ub4e4\uc774 \ud300\uc5d0\uc11c \uc544\ubb34\ub3c4 \uc601\uc6c5\uc744 \uad6c\uc785\ud558\uc9c0 \uc54a\ub294\ub2e4\uba74 \uc18c\uc561 \uacb0\uc81c\ub098 \ub2e4\uc6b4\ub85c\ub4dc \ucf58\ud150\uce20\uac00 \ub2a5\ub825\uc744 \uc81c\ud55c\ud560 \uc218 \uc788\ub294 \uac83\uc5d0 \uac00\uae4c\uc6cc\uc9c4\ub2e4\uace0 \ub9d0\ud558\uc600\ub2e4. \ucf08\ub7ec\ub294 \ub610\ud55c \uc18c\uc561 \uacb0\uc81c \ubc0f \ub2e4\uc6b4\ub85c\ub4dc \ucf58\ud150\uce20\ub85c \uc778\ud574 \uac8c\uc774\uba38\uac00 \ubaa8\ub4e0 \ud50c\ub808\uc774\uc5b4 \ub300\uc2e0\uc5d0 \ub611\uac19\uc740 \ucee8\ud150\uce20\ub97c \uac00\uc9c0\uace0 \uc788\ub294 \uce5c\uad6c\uc640\ub9cc \ud50c\ub808\uc774\ud558\uac8c \ub418\uc5b4 \ud50c\ub808\uc774\uc5b4 \uacf5\ub3d9\uccb4\ub97c \ud574\uccb4\uc2dc\ud0ac \uc218 \uc788\ub2e4\uace0 \uc5b8\uae09\ud558\uc600\ub2e4. \uac1c\ubc1c\uc758 \ub354 \ub192\uc740 \ubaa9\ud45c\ub294 \ub2e4\ub978 \uacbd\uc7c1 \uac8c\uc784 \ud658\uacbd\uc5d0\uc11c \uc885\uc885 \uc77c\uc5b4\ub098\ub294 \ubd80\uc815\uc801 \uc131\ud5a5\uc744 \ud53c\ud558\uace0, \uc774\uc57c\uae30\uac00 \uae0d\uc815\uc801\uc778 \uba54\uc2dc\uc9c0\ub97c \uc8fc\ub3c4\ub85d \ud558\ub294 \uc9c4\uc804\uc5d0 \ub367\ubd99\uc5ec \ubd80\uc815\uc801\uc778 \uc601\ud5a5\uc744 \uc900\ub2e4\uace0 \ub290\ub07c\ub294 \uc694\uc18c\ub97c \uc5c6\uc560\uae30 \uc704\ud574 \uac8c\uc784\ud50c\ub808\uc774 \ub514\uc790\uc778\uc5d0\uc11c \uba85\ud655\ud55c \uc120\ud0dd\uc744 \ud558\ub294 \uac83\uc774\uc5c8\ub2e4. \uc774\ub7ec\ud55c \uc120\ud0dd \uc911 \ud558\ub098\ub294 \ub2e4\uc591\ud55c \ud1b5\uacc4 \uc694\uc57d\uc5d0\uc11c \uc8fd\uc784/\uc8fd\uc74c(kill/death) \ud45c\uc2dc\ub97c \uc5c6\uc560\ub294 \uac83\uc774\uc5c8\uc73c\uba70, \uce90\ud50c\ub7f0\uc740 \"\uc5b4\ub5a4 \uc601\uc6c5\ub4e4\uc740 \ud65c\uc57d\uc744 \uc704\ud574\uc11c \uc8fd\uc784\uc744 \ud544\uc694\ub85c \ud558\uc9c0 \uc54a\ub294\ub2e4\"\ub77c\uace0 \uc5b8\uae09\ud558\uc600\ub2e4. \uce5c\uc120\uc801\uc778 \ud50c\ub808\uc774 \ud658\uacbd\uc744 \uc99d\uc9c4\uc2dc\ud0a4\uae30 \uc704\ud574, \ube14\ub9ac\uc790\ub4dc\ub294 '\ud0c8\uc8fc'(\uacbd\uae30\uac00 \ub05d\ub098\uae30 \uc804\uc5d0 \uc77c\ubd80\ub7ec \ub098\uac00\ub294 \ud589\uc704)\ub97c \ud558\ub294 \ud50c\ub808\uc774\uc5b4\uc5d0\uac8c \uacbd\uae30 \ud6c4 \uacbd\ud5d8\uce58\ub97c \uac10\uc18c\uc2dc\ud0a4\uace0, \ub354 \uc2ec\ud560 \uacbd\uc6b0 \ud50c\ub808\uc774\ub97c \uc601\uad6c\ud788 \uae08\uc9c0\ud558\ub294 \ubd88\uc774\uc775\uc744 \ubd80\uc5ec\ud55c\ub2e4. \ube14\ub9ac\uc790\ub4dc\ub294 2016\ub144 7\uc6d4 \uc870\uc900\uc120 \ubcf4\uc815 \ud504\ub85c\uadf8\ub7a8(\uc5d0\uc784\ud575)\uc744 \ube44\ub86f\ud55c \uc81c3\uc790 \uc678\ubd80 \ubcf4\uc815 \ud504\ub85c\uadf8\ub7a8\uc744 \uc0ac\uc6a9\ud558\ub294 \ud50c\ub808\uc774\uc5b4\ub4e4\uc5d0\uac8c \ub300\uaddc\ubaa8 \uc81c\uc7ac\ub97c \uac00\ud558\uc600\ub2e4. \u300a\uc624\ubc84\uc6cc\uce58\u300b \ucd9c\uc2dc \uc774\ud6c4, \ube14\ub9ac\uc790\ub4dc\ub294 \ud50c\ub808\uc774\uc5b4\uc5d0\uac8c \uac8c\uc784\uc5d0\uc11c \ud5c8\uc6a9\ub418\uc9c0 \uc54a\ub294 \uae30\ub2a5\uc744 \uc81c\uacf5\ud558\ub294 \uc6cc\uce58\uc624\ubc84 \ud0c0\uc77c\ub7f0\ud2b8\ub97c \uac1c\ubc1c\ud55c \ubcf4\uc2a4\ub79c\ub4dc \ud575\uc2a4\uc5d0 \ubc95\uc801 \uace0\uc18c\ub97c \ucde8\ud558\uc600\ub2e4.", "answer": "\uc804\ud22c\uc758 \uba85\ub8cc\uc131", "sentence": "\uac1c\ubc1c \ub2e8\uacc4\uc5d0\uc11c \uc911\uc694\ud588\ub358 \ubaa9\ud45c \ud558\ub098\ub294 \ud50c\ub808\uc774\uc5b4\ub97c \uc704\ud55c ' \uc804\ud22c\uc758 \uba85\ub8cc\uc131 '\uc73c\ub85c, \ud50c\ub808\uc774\uc5b4\uac00 \uc0c8\ub85c\uc6b4 \uc9c0\uc5ed\uc73c\ub85c \uc774\ub3d9\ud558\uba74 \uc801 \uce90\ub9ad\ud130\uac00 \ub69c\ub837\ud558\uac8c \ubcf4\uc774\ub3c4\ub85d \ud588\ub2e4.", "paragraph_sentence": "\u300a\uc624\ubc84\uc6cc\uce58\u300b\ub294 \uc0c9\ub9f9\uc744 \uc704\ud55c \uc811\uadfc\uc131 \uc124\uc815\uc744 \ud3ec\ud568, \ud3ed\ub113\uc740 \uc0ac\uc6a9\uc790\ub97c \ud655\ubcf4\ud560 \uc218 \uc788\ub294 \uc5ec\uc12f \uac00\uc9c0 \ud2b9\uc9d5\uc744 \ud568\uaed8 \uac1c\ubc1c\ud558\uc600\ub2e4. \uac1c\ubc1c \ub2e8\uacc4\uc5d0\uc11c \uc911\uc694\ud588\ub358 \ubaa9\ud45c \ud558\ub098\ub294 \ud50c\ub808\uc774\uc5b4\ub97c \uc704\ud55c ' \uc804\ud22c\uc758 \uba85\ub8cc\uc131 '\uc73c\ub85c, \ud50c\ub808\uc774\uc5b4\uac00 \uc0c8\ub85c\uc6b4 \uc9c0\uc5ed\uc73c\ub85c \uc774\ub3d9\ud558\uba74 \uc801 \uce90\ub9ad\ud130\uac00 \ub69c\ub837\ud558\uac8c \ubcf4\uc774\ub3c4\ub85d \ud588\ub2e4. \uc774\uac83\uc740 \ud50c\ub808\uc774\uc5b4\uac00 \uc804\uc7a5\uc5d0\uc11c \uc0ac\uc6a9\ud558\ub294 \uc0c9\uc870\uc640 \ucc44\ub3c4 \uc218\uc900\uc758 \ub300\uc870, \uba85\ubc31\ud558\uac8c \ub2e4\ub978 \uc724\uacfd\uc744 \uac16\uac8c \ub9cc\ub4e0 \uce90\ub9ad\ud130\ub4e4\ub85c \ud50c\ub808\uc774\uc5b4\uac00 \uba3c \uac70\ub9ac\uc5d0\uc11c \uc601\uc6c5\uc774 \uc544\uad70\uc778\uc9c0 \uc801\uad70\uc778\uc9c0 \uad6c\ubcc4\ud560 \uc218 \uc788\ub3c4\ub85d \ud568\uc73c\ub85c\uc368 \uac00\ub2a5\ud558\uc600\ub2e4. \ud300\uc740 \uac1c\ubc1c\ub3d9\uc548 \uacbd\uae30 \uc911\uac04\uc5d0 \ud50c\ub808\uc774\uc5b4\uac00 \uc601\uc6c5\uc744 \ubc14\uafc0 \uc218 \uc788\ub3c4\ub85d \ud558\ub294 \uac83\uc774 \uac8c\uc784\ud50c\ub808\uc774\uc5d0 \uc911\uc694\ud55c \uc5ed\ud560\uc744 \ud568\uc744 \uc54c\uc544\ub0c8\ub2e4. \u300a\uc624\ubc84\uc6cc\uce58\u300b\ub294 \ucc98\uc74c\uc5d0 \uc18c\uc561 \uacb0\uc81c\ub098 \ub2e4\uc6b4\ub85c\ub4dc \ucf58\ud150\uce20 \uad6c\uc785\uc774 \ud3ec\ud568\ub41c \ubd80\ubd84 \uc720\ub8cc\ud654 \uac8c\uc784 \ud615\ud0dc\ub85c \ucd9c\uc2dc\ub97c \uacc4\ud68d\ud588\uc73c\ub098, \uc774\ub7ec\ud55c \ubc1c\uacac\uc73c\ub85c \uc778\ud574 \ub2e8\ub3c5 \uad6c\ub9e4\ud615 \uac8c\uc784\uc73c\ub85c \ub178\uc120\uc744 \ubcc0\uacbd\ud558\uc600\ub2e4. \ucf08\ub7ec\ub294 \ud300\uc774 \ud50c\ub808\uc774\uc5b4\uac00 \uc0c1\ud669\uc5d0 \ub530\ub77c \uc5b4\ub290 \uc601\uc6c5\uc73c\ub85c\ub4e0 \uad50\uccb4\ud560 \uc218 \uc788\uae30\ub97c \uc6d0\ud588\uace0, \ud50c\ub808\uc774\uc5b4\ub4e4\uc774 \ud300\uc5d0\uc11c \uc544\ubb34\ub3c4 \uc601\uc6c5\uc744 \uad6c\uc785\ud558\uc9c0 \uc54a\ub294\ub2e4\uba74 \uc18c\uc561 \uacb0\uc81c\ub098 \ub2e4\uc6b4\ub85c\ub4dc \ucf58\ud150\uce20\uac00 \ub2a5\ub825\uc744 \uc81c\ud55c\ud560 \uc218 \uc788\ub294 \uac83\uc5d0 \uac00\uae4c\uc6cc\uc9c4\ub2e4\uace0 \ub9d0\ud558\uc600\ub2e4. \ucf08\ub7ec\ub294 \ub610\ud55c \uc18c\uc561 \uacb0\uc81c \ubc0f \ub2e4\uc6b4\ub85c\ub4dc \ucf58\ud150\uce20\ub85c \uc778\ud574 \uac8c\uc774\uba38\uac00 \ubaa8\ub4e0 \ud50c\ub808\uc774\uc5b4 \ub300\uc2e0\uc5d0 \ub611\uac19\uc740 \ucee8\ud150\uce20\ub97c \uac00\uc9c0\uace0 \uc788\ub294 \uce5c\uad6c\uc640\ub9cc \ud50c\ub808\uc774\ud558\uac8c \ub418\uc5b4 \ud50c\ub808\uc774\uc5b4 \uacf5\ub3d9\uccb4\ub97c \ud574\uccb4\uc2dc\ud0ac \uc218 \uc788\ub2e4\uace0 \uc5b8\uae09\ud558\uc600\ub2e4. \uac1c\ubc1c\uc758 \ub354 \ub192\uc740 \ubaa9\ud45c\ub294 \ub2e4\ub978 \uacbd\uc7c1 \uac8c\uc784 \ud658\uacbd\uc5d0\uc11c \uc885\uc885 \uc77c\uc5b4\ub098\ub294 \ubd80\uc815\uc801 \uc131\ud5a5\uc744 \ud53c\ud558\uace0, \uc774\uc57c\uae30\uac00 \uae0d\uc815\uc801\uc778 \uba54\uc2dc\uc9c0\ub97c \uc8fc\ub3c4\ub85d \ud558\ub294 \uc9c4\uc804\uc5d0 \ub367\ubd99\uc5ec \ubd80\uc815\uc801\uc778 \uc601\ud5a5\uc744 \uc900\ub2e4\uace0 \ub290\ub07c\ub294 \uc694\uc18c\ub97c \uc5c6\uc560\uae30 \uc704\ud574 \uac8c\uc784\ud50c\ub808\uc774 \ub514\uc790\uc778\uc5d0\uc11c \uba85\ud655\ud55c \uc120\ud0dd\uc744 \ud558\ub294 \uac83\uc774\uc5c8\ub2e4. \uc774\ub7ec\ud55c \uc120\ud0dd \uc911 \ud558\ub098\ub294 \ub2e4\uc591\ud55c \ud1b5\uacc4 \uc694\uc57d\uc5d0\uc11c \uc8fd\uc784/\uc8fd\uc74c(kill/death) \ud45c\uc2dc\ub97c \uc5c6\uc560\ub294 \uac83\uc774\uc5c8\uc73c\uba70, \uce90\ud50c\ub7f0\uc740 \"\uc5b4\ub5a4 \uc601\uc6c5\ub4e4\uc740 \ud65c\uc57d\uc744 \uc704\ud574\uc11c \uc8fd\uc784\uc744 \ud544\uc694\ub85c \ud558\uc9c0 \uc54a\ub294\ub2e4\"\ub77c\uace0 \uc5b8\uae09\ud558\uc600\ub2e4. \uce5c\uc120\uc801\uc778 \ud50c\ub808\uc774 \ud658\uacbd\uc744 \uc99d\uc9c4\uc2dc\ud0a4\uae30 \uc704\ud574, \ube14\ub9ac\uc790\ub4dc\ub294 '\ud0c8\uc8fc'(\uacbd\uae30\uac00 \ub05d\ub098\uae30 \uc804\uc5d0 \uc77c\ubd80\ub7ec \ub098\uac00\ub294 \ud589\uc704)\ub97c \ud558\ub294 \ud50c\ub808\uc774\uc5b4\uc5d0\uac8c \uacbd\uae30 \ud6c4 \uacbd\ud5d8\uce58\ub97c \uac10\uc18c\uc2dc\ud0a4\uace0, \ub354 \uc2ec\ud560 \uacbd\uc6b0 \ud50c\ub808\uc774\ub97c \uc601\uad6c\ud788 \uae08\uc9c0\ud558\ub294 \ubd88\uc774\uc775\uc744 \ubd80\uc5ec\ud55c\ub2e4. \ube14\ub9ac\uc790\ub4dc\ub294 2016\ub144 7\uc6d4 \uc870\uc900\uc120 \ubcf4\uc815 \ud504\ub85c\uadf8\ub7a8(\uc5d0\uc784\ud575)\uc744 \ube44\ub86f\ud55c \uc81c3\uc790 \uc678\ubd80 \ubcf4\uc815 \ud504\ub85c\uadf8\ub7a8\uc744 \uc0ac\uc6a9\ud558\ub294 \ud50c\ub808\uc774\uc5b4\ub4e4\uc5d0\uac8c \ub300\uaddc\ubaa8 \uc81c\uc7ac\ub97c \uac00\ud558\uc600\ub2e4. \u300a\uc624\ubc84\uc6cc\uce58\u300b \ucd9c\uc2dc \uc774\ud6c4, \ube14\ub9ac\uc790\ub4dc\ub294 \ud50c\ub808\uc774\uc5b4\uc5d0\uac8c \uac8c\uc784\uc5d0\uc11c \ud5c8\uc6a9\ub418\uc9c0 \uc54a\ub294 \uae30\ub2a5\uc744 \uc81c\uacf5\ud558\ub294 \uc6cc\uce58\uc624\ubc84 \ud0c0\uc77c\ub7f0\ud2b8\ub97c \uac1c\ubc1c\ud55c \ubcf4\uc2a4\ub79c\ub4dc \ud575\uc2a4\uc5d0 \ubc95\uc801 \uace0\uc18c\ub97c \ucde8\ud558\uc600\ub2e4.", "paragraph_answer": "\u300a\uc624\ubc84\uc6cc\uce58\u300b\ub294 \uc0c9\ub9f9\uc744 \uc704\ud55c \uc811\uadfc\uc131 \uc124\uc815\uc744 \ud3ec\ud568, \ud3ed\ub113\uc740 \uc0ac\uc6a9\uc790\ub97c \ud655\ubcf4\ud560 \uc218 \uc788\ub294 \uc5ec\uc12f \uac00\uc9c0 \ud2b9\uc9d5\uc744 \ud568\uaed8 \uac1c\ubc1c\ud558\uc600\ub2e4. \uac1c\ubc1c \ub2e8\uacc4\uc5d0\uc11c \uc911\uc694\ud588\ub358 \ubaa9\ud45c \ud558\ub098\ub294 \ud50c\ub808\uc774\uc5b4\ub97c \uc704\ud55c ' \uc804\ud22c\uc758 \uba85\ub8cc\uc131 '\uc73c\ub85c, \ud50c\ub808\uc774\uc5b4\uac00 \uc0c8\ub85c\uc6b4 \uc9c0\uc5ed\uc73c\ub85c \uc774\ub3d9\ud558\uba74 \uc801 \uce90\ub9ad\ud130\uac00 \ub69c\ub837\ud558\uac8c \ubcf4\uc774\ub3c4\ub85d \ud588\ub2e4. \uc774\uac83\uc740 \ud50c\ub808\uc774\uc5b4\uac00 \uc804\uc7a5\uc5d0\uc11c \uc0ac\uc6a9\ud558\ub294 \uc0c9\uc870\uc640 \ucc44\ub3c4 \uc218\uc900\uc758 \ub300\uc870, \uba85\ubc31\ud558\uac8c \ub2e4\ub978 \uc724\uacfd\uc744 \uac16\uac8c \ub9cc\ub4e0 \uce90\ub9ad\ud130\ub4e4\ub85c \ud50c\ub808\uc774\uc5b4\uac00 \uba3c \uac70\ub9ac\uc5d0\uc11c \uc601\uc6c5\uc774 \uc544\uad70\uc778\uc9c0 \uc801\uad70\uc778\uc9c0 \uad6c\ubcc4\ud560 \uc218 \uc788\ub3c4\ub85d \ud568\uc73c\ub85c\uc368 \uac00\ub2a5\ud558\uc600\ub2e4. \ud300\uc740 \uac1c\ubc1c\ub3d9\uc548 \uacbd\uae30 \uc911\uac04\uc5d0 \ud50c\ub808\uc774\uc5b4\uac00 \uc601\uc6c5\uc744 \ubc14\uafc0 \uc218 \uc788\ub3c4\ub85d \ud558\ub294 \uac83\uc774 \uac8c\uc784\ud50c\ub808\uc774\uc5d0 \uc911\uc694\ud55c \uc5ed\ud560\uc744 \ud568\uc744 \uc54c\uc544\ub0c8\ub2e4. \u300a\uc624\ubc84\uc6cc\uce58\u300b\ub294 \ucc98\uc74c\uc5d0 \uc18c\uc561 \uacb0\uc81c\ub098 \ub2e4\uc6b4\ub85c\ub4dc \ucf58\ud150\uce20 \uad6c\uc785\uc774 \ud3ec\ud568\ub41c \ubd80\ubd84 \uc720\ub8cc\ud654 \uac8c\uc784 \ud615\ud0dc\ub85c \ucd9c\uc2dc\ub97c \uacc4\ud68d\ud588\uc73c\ub098, \uc774\ub7ec\ud55c \ubc1c\uacac\uc73c\ub85c \uc778\ud574 \ub2e8\ub3c5 \uad6c\ub9e4\ud615 \uac8c\uc784\uc73c\ub85c \ub178\uc120\uc744 \ubcc0\uacbd\ud558\uc600\ub2e4. \ucf08\ub7ec\ub294 \ud300\uc774 \ud50c\ub808\uc774\uc5b4\uac00 \uc0c1\ud669\uc5d0 \ub530\ub77c \uc5b4\ub290 \uc601\uc6c5\uc73c\ub85c\ub4e0 \uad50\uccb4\ud560 \uc218 \uc788\uae30\ub97c \uc6d0\ud588\uace0, \ud50c\ub808\uc774\uc5b4\ub4e4\uc774 \ud300\uc5d0\uc11c \uc544\ubb34\ub3c4 \uc601\uc6c5\uc744 \uad6c\uc785\ud558\uc9c0 \uc54a\ub294\ub2e4\uba74 \uc18c\uc561 \uacb0\uc81c\ub098 \ub2e4\uc6b4\ub85c\ub4dc \ucf58\ud150\uce20\uac00 \ub2a5\ub825\uc744 \uc81c\ud55c\ud560 \uc218 \uc788\ub294 \uac83\uc5d0 \uac00\uae4c\uc6cc\uc9c4\ub2e4\uace0 \ub9d0\ud558\uc600\ub2e4. \ucf08\ub7ec\ub294 \ub610\ud55c \uc18c\uc561 \uacb0\uc81c \ubc0f \ub2e4\uc6b4\ub85c\ub4dc \ucf58\ud150\uce20\ub85c \uc778\ud574 \uac8c\uc774\uba38\uac00 \ubaa8\ub4e0 \ud50c\ub808\uc774\uc5b4 \ub300\uc2e0\uc5d0 \ub611\uac19\uc740 \ucee8\ud150\uce20\ub97c \uac00\uc9c0\uace0 \uc788\ub294 \uce5c\uad6c\uc640\ub9cc \ud50c\ub808\uc774\ud558\uac8c \ub418\uc5b4 \ud50c\ub808\uc774\uc5b4 \uacf5\ub3d9\uccb4\ub97c \ud574\uccb4\uc2dc\ud0ac \uc218 \uc788\ub2e4\uace0 \uc5b8\uae09\ud558\uc600\ub2e4. \uac1c\ubc1c\uc758 \ub354 \ub192\uc740 \ubaa9\ud45c\ub294 \ub2e4\ub978 \uacbd\uc7c1 \uac8c\uc784 \ud658\uacbd\uc5d0\uc11c \uc885\uc885 \uc77c\uc5b4\ub098\ub294 \ubd80\uc815\uc801 \uc131\ud5a5\uc744 \ud53c\ud558\uace0, \uc774\uc57c\uae30\uac00 \uae0d\uc815\uc801\uc778 \uba54\uc2dc\uc9c0\ub97c \uc8fc\ub3c4\ub85d \ud558\ub294 \uc9c4\uc804\uc5d0 \ub367\ubd99\uc5ec \ubd80\uc815\uc801\uc778 \uc601\ud5a5\uc744 \uc900\ub2e4\uace0 \ub290\ub07c\ub294 \uc694\uc18c\ub97c \uc5c6\uc560\uae30 \uc704\ud574 \uac8c\uc784\ud50c\ub808\uc774 \ub514\uc790\uc778\uc5d0\uc11c \uba85\ud655\ud55c \uc120\ud0dd\uc744 \ud558\ub294 \uac83\uc774\uc5c8\ub2e4. \uc774\ub7ec\ud55c \uc120\ud0dd \uc911 \ud558\ub098\ub294 \ub2e4\uc591\ud55c \ud1b5\uacc4 \uc694\uc57d\uc5d0\uc11c \uc8fd\uc784/\uc8fd\uc74c(kill/death) \ud45c\uc2dc\ub97c \uc5c6\uc560\ub294 \uac83\uc774\uc5c8\uc73c\uba70, \uce90\ud50c\ub7f0\uc740 \"\uc5b4\ub5a4 \uc601\uc6c5\ub4e4\uc740 \ud65c\uc57d\uc744 \uc704\ud574\uc11c \uc8fd\uc784\uc744 \ud544\uc694\ub85c \ud558\uc9c0 \uc54a\ub294\ub2e4\"\ub77c\uace0 \uc5b8\uae09\ud558\uc600\ub2e4. \uce5c\uc120\uc801\uc778 \ud50c\ub808\uc774 \ud658\uacbd\uc744 \uc99d\uc9c4\uc2dc\ud0a4\uae30 \uc704\ud574, \ube14\ub9ac\uc790\ub4dc\ub294 '\ud0c8\uc8fc'(\uacbd\uae30\uac00 \ub05d\ub098\uae30 \uc804\uc5d0 \uc77c\ubd80\ub7ec \ub098\uac00\ub294 \ud589\uc704)\ub97c \ud558\ub294 \ud50c\ub808\uc774\uc5b4\uc5d0\uac8c \uacbd\uae30 \ud6c4 \uacbd\ud5d8\uce58\ub97c \uac10\uc18c\uc2dc\ud0a4\uace0, \ub354 \uc2ec\ud560 \uacbd\uc6b0 \ud50c\ub808\uc774\ub97c \uc601\uad6c\ud788 \uae08\uc9c0\ud558\ub294 \ubd88\uc774\uc775\uc744 \ubd80\uc5ec\ud55c\ub2e4. \ube14\ub9ac\uc790\ub4dc\ub294 2016\ub144 7\uc6d4 \uc870\uc900\uc120 \ubcf4\uc815 \ud504\ub85c\uadf8\ub7a8(\uc5d0\uc784\ud575)\uc744 \ube44\ub86f\ud55c \uc81c3\uc790 \uc678\ubd80 \ubcf4\uc815 \ud504\ub85c\uadf8\ub7a8\uc744 \uc0ac\uc6a9\ud558\ub294 \ud50c\ub808\uc774\uc5b4\ub4e4\uc5d0\uac8c \ub300\uaddc\ubaa8 \uc81c\uc7ac\ub97c \uac00\ud558\uc600\ub2e4. \u300a\uc624\ubc84\uc6cc\uce58\u300b \ucd9c\uc2dc \uc774\ud6c4, \ube14\ub9ac\uc790\ub4dc\ub294 \ud50c\ub808\uc774\uc5b4\uc5d0\uac8c \uac8c\uc784\uc5d0\uc11c \ud5c8\uc6a9\ub418\uc9c0 \uc54a\ub294 \uae30\ub2a5\uc744 \uc81c\uacf5\ud558\ub294 \uc6cc\uce58\uc624\ubc84 \ud0c0\uc77c\ub7f0\ud2b8\ub97c \uac1c\ubc1c\ud55c \ubcf4\uc2a4\ub79c\ub4dc \ud575\uc2a4\uc5d0 \ubc95\uc801 \uace0\uc18c\ub97c \ucde8\ud558\uc600\ub2e4.", "sentence_answer": "\uac1c\ubc1c \ub2e8\uacc4\uc5d0\uc11c \uc911\uc694\ud588\ub358 \ubaa9\ud45c \ud558\ub098\ub294 \ud50c\ub808\uc774\uc5b4\ub97c \uc704\ud55c ' \uc804\ud22c\uc758 \uba85\ub8cc\uc131 '\uc73c\ub85c, \ud50c\ub808\uc774\uc5b4\uac00 \uc0c8\ub85c\uc6b4 \uc9c0\uc5ed\uc73c\ub85c \uc774\ub3d9\ud558\uba74 \uc801 \uce90\ub9ad\ud130\uac00 \ub69c\ub837\ud558\uac8c \ubcf4\uc774\ub3c4\ub85d \ud588\ub2e4.", "paragraph_id": "6524626-15-0", "paragraph_question": "question: \uac1c\ubc1c \ub2e8\uacc4\uc5d0\uc11c \uc911\uc694\ud588\ub358 \ubaa9\ud45c\ub294?, context: \u300a\uc624\ubc84\uc6cc\uce58\u300b\ub294 \uc0c9\ub9f9\uc744 \uc704\ud55c \uc811\uadfc\uc131 \uc124\uc815\uc744 \ud3ec\ud568, \ud3ed\ub113\uc740 \uc0ac\uc6a9\uc790\ub97c \ud655\ubcf4\ud560 \uc218 \uc788\ub294 \uc5ec\uc12f \uac00\uc9c0 \ud2b9\uc9d5\uc744 \ud568\uaed8 \uac1c\ubc1c\ud558\uc600\ub2e4. \uac1c\ubc1c \ub2e8\uacc4\uc5d0\uc11c \uc911\uc694\ud588\ub358 \ubaa9\ud45c \ud558\ub098\ub294 \ud50c\ub808\uc774\uc5b4\ub97c \uc704\ud55c '\uc804\ud22c\uc758 \uba85\ub8cc\uc131'\uc73c\ub85c, \ud50c\ub808\uc774\uc5b4\uac00 \uc0c8\ub85c\uc6b4 \uc9c0\uc5ed\uc73c\ub85c \uc774\ub3d9\ud558\uba74 \uc801 \uce90\ub9ad\ud130\uac00 \ub69c\ub837\ud558\uac8c \ubcf4\uc774\ub3c4\ub85d \ud588\ub2e4. \uc774\uac83\uc740 \ud50c\ub808\uc774\uc5b4\uac00 \uc804\uc7a5\uc5d0\uc11c \uc0ac\uc6a9\ud558\ub294 \uc0c9\uc870\uc640 \ucc44\ub3c4 \uc218\uc900\uc758 \ub300\uc870, \uba85\ubc31\ud558\uac8c \ub2e4\ub978 \uc724\uacfd\uc744 \uac16\uac8c \ub9cc\ub4e0 \uce90\ub9ad\ud130\ub4e4\ub85c \ud50c\ub808\uc774\uc5b4\uac00 \uba3c \uac70\ub9ac\uc5d0\uc11c \uc601\uc6c5\uc774 \uc544\uad70\uc778\uc9c0 \uc801\uad70\uc778\uc9c0 \uad6c\ubcc4\ud560 \uc218 \uc788\ub3c4\ub85d \ud568\uc73c\ub85c\uc368 \uac00\ub2a5\ud558\uc600\ub2e4. \ud300\uc740 \uac1c\ubc1c\ub3d9\uc548 \uacbd\uae30 \uc911\uac04\uc5d0 \ud50c\ub808\uc774\uc5b4\uac00 \uc601\uc6c5\uc744 \ubc14\uafc0 \uc218 \uc788\ub3c4\ub85d \ud558\ub294 \uac83\uc774 \uac8c\uc784\ud50c\ub808\uc774\uc5d0 \uc911\uc694\ud55c \uc5ed\ud560\uc744 \ud568\uc744 \uc54c\uc544\ub0c8\ub2e4. \u300a\uc624\ubc84\uc6cc\uce58\u300b\ub294 \ucc98\uc74c\uc5d0 \uc18c\uc561 \uacb0\uc81c\ub098 \ub2e4\uc6b4\ub85c\ub4dc \ucf58\ud150\uce20 \uad6c\uc785\uc774 \ud3ec\ud568\ub41c \ubd80\ubd84 \uc720\ub8cc\ud654 \uac8c\uc784 \ud615\ud0dc\ub85c \ucd9c\uc2dc\ub97c \uacc4\ud68d\ud588\uc73c\ub098, \uc774\ub7ec\ud55c \ubc1c\uacac\uc73c\ub85c \uc778\ud574 \ub2e8\ub3c5 \uad6c\ub9e4\ud615 \uac8c\uc784\uc73c\ub85c \ub178\uc120\uc744 \ubcc0\uacbd\ud558\uc600\ub2e4. \ucf08\ub7ec\ub294 \ud300\uc774 \ud50c\ub808\uc774\uc5b4\uac00 \uc0c1\ud669\uc5d0 \ub530\ub77c \uc5b4\ub290 \uc601\uc6c5\uc73c\ub85c\ub4e0 \uad50\uccb4\ud560 \uc218 \uc788\uae30\ub97c \uc6d0\ud588\uace0, \ud50c\ub808\uc774\uc5b4\ub4e4\uc774 \ud300\uc5d0\uc11c \uc544\ubb34\ub3c4 \uc601\uc6c5\uc744 \uad6c\uc785\ud558\uc9c0 \uc54a\ub294\ub2e4\uba74 \uc18c\uc561 \uacb0\uc81c\ub098 \ub2e4\uc6b4\ub85c\ub4dc \ucf58\ud150\uce20\uac00 \ub2a5\ub825\uc744 \uc81c\ud55c\ud560 \uc218 \uc788\ub294 \uac83\uc5d0 \uac00\uae4c\uc6cc\uc9c4\ub2e4\uace0 \ub9d0\ud558\uc600\ub2e4. \ucf08\ub7ec\ub294 \ub610\ud55c \uc18c\uc561 \uacb0\uc81c \ubc0f \ub2e4\uc6b4\ub85c\ub4dc \ucf58\ud150\uce20\ub85c \uc778\ud574 \uac8c\uc774\uba38\uac00 \ubaa8\ub4e0 \ud50c\ub808\uc774\uc5b4 \ub300\uc2e0\uc5d0 \ub611\uac19\uc740 \ucee8\ud150\uce20\ub97c \uac00\uc9c0\uace0 \uc788\ub294 \uce5c\uad6c\uc640\ub9cc \ud50c\ub808\uc774\ud558\uac8c \ub418\uc5b4 \ud50c\ub808\uc774\uc5b4 \uacf5\ub3d9\uccb4\ub97c \ud574\uccb4\uc2dc\ud0ac \uc218 \uc788\ub2e4\uace0 \uc5b8\uae09\ud558\uc600\ub2e4. \uac1c\ubc1c\uc758 \ub354 \ub192\uc740 \ubaa9\ud45c\ub294 \ub2e4\ub978 \uacbd\uc7c1 \uac8c\uc784 \ud658\uacbd\uc5d0\uc11c \uc885\uc885 \uc77c\uc5b4\ub098\ub294 \ubd80\uc815\uc801 \uc131\ud5a5\uc744 \ud53c\ud558\uace0, \uc774\uc57c\uae30\uac00 \uae0d\uc815\uc801\uc778 \uba54\uc2dc\uc9c0\ub97c \uc8fc\ub3c4\ub85d \ud558\ub294 \uc9c4\uc804\uc5d0 \ub367\ubd99\uc5ec \ubd80\uc815\uc801\uc778 \uc601\ud5a5\uc744 \uc900\ub2e4\uace0 \ub290\ub07c\ub294 \uc694\uc18c\ub97c \uc5c6\uc560\uae30 \uc704\ud574 \uac8c\uc784\ud50c\ub808\uc774 \ub514\uc790\uc778\uc5d0\uc11c \uba85\ud655\ud55c \uc120\ud0dd\uc744 \ud558\ub294 \uac83\uc774\uc5c8\ub2e4. \uc774\ub7ec\ud55c \uc120\ud0dd \uc911 \ud558\ub098\ub294 \ub2e4\uc591\ud55c \ud1b5\uacc4 \uc694\uc57d\uc5d0\uc11c \uc8fd\uc784/\uc8fd\uc74c(kill/death) \ud45c\uc2dc\ub97c \uc5c6\uc560\ub294 \uac83\uc774\uc5c8\uc73c\uba70, \uce90\ud50c\ub7f0\uc740 \"\uc5b4\ub5a4 \uc601\uc6c5\ub4e4\uc740 \ud65c\uc57d\uc744 \uc704\ud574\uc11c \uc8fd\uc784\uc744 \ud544\uc694\ub85c \ud558\uc9c0 \uc54a\ub294\ub2e4\"\ub77c\uace0 \uc5b8\uae09\ud558\uc600\ub2e4. \uce5c\uc120\uc801\uc778 \ud50c\ub808\uc774 \ud658\uacbd\uc744 \uc99d\uc9c4\uc2dc\ud0a4\uae30 \uc704\ud574, \ube14\ub9ac\uc790\ub4dc\ub294 '\ud0c8\uc8fc'(\uacbd\uae30\uac00 \ub05d\ub098\uae30 \uc804\uc5d0 \uc77c\ubd80\ub7ec \ub098\uac00\ub294 \ud589\uc704)\ub97c \ud558\ub294 \ud50c\ub808\uc774\uc5b4\uc5d0\uac8c \uacbd\uae30 \ud6c4 \uacbd\ud5d8\uce58\ub97c \uac10\uc18c\uc2dc\ud0a4\uace0, \ub354 \uc2ec\ud560 \uacbd\uc6b0 \ud50c\ub808\uc774\ub97c \uc601\uad6c\ud788 \uae08\uc9c0\ud558\ub294 \ubd88\uc774\uc775\uc744 \ubd80\uc5ec\ud55c\ub2e4. \ube14\ub9ac\uc790\ub4dc\ub294 2016\ub144 7\uc6d4 \uc870\uc900\uc120 \ubcf4\uc815 \ud504\ub85c\uadf8\ub7a8(\uc5d0\uc784\ud575)\uc744 \ube44\ub86f\ud55c \uc81c3\uc790 \uc678\ubd80 \ubcf4\uc815 \ud504\ub85c\uadf8\ub7a8\uc744 \uc0ac\uc6a9\ud558\ub294 \ud50c\ub808\uc774\uc5b4\ub4e4\uc5d0\uac8c \ub300\uaddc\ubaa8 \uc81c\uc7ac\ub97c \uac00\ud558\uc600\ub2e4. \u300a\uc624\ubc84\uc6cc\uce58\u300b \ucd9c\uc2dc \uc774\ud6c4, \ube14\ub9ac\uc790\ub4dc\ub294 \ud50c\ub808\uc774\uc5b4\uc5d0\uac8c \uac8c\uc784\uc5d0\uc11c \ud5c8\uc6a9\ub418\uc9c0 \uc54a\ub294 \uae30\ub2a5\uc744 \uc81c\uacf5\ud558\ub294 \uc6cc\uce58\uc624\ubc84 \ud0c0\uc77c\ub7f0\ud2b8\ub97c \uac1c\ubc1c\ud55c \ubcf4\uc2a4\ub79c\ub4dc \ud575\uc2a4\uc5d0 \ubc95\uc801 \uace0\uc18c\ub97c \ucde8\ud558\uc600\ub2e4."} {"question": "2003\ub144 \uc774\ub860\uc801\uc778 \uc0dd\uba85\uccb4 \uac70\uc8fc \uac00\ub2a5 \ud589\uc131\uacc4 \uc911 \uac00\uc7a5 \uc720\ub825\ud55c 30\uac1c \ud56d\uc131\uc744 \ucd94\ub824\ub0b8 \uc0ac\ub78c\uc740?", "paragraph": "\uadf8\ub7ec\ub098 \uc774\ub7ec\ud55c \ube44\uad00\uc801\uc778 \uacb0\uacfc\uc5d0\ub3c4 \uace0\ub798\uc790\ub9ac \ud0c0\uc6b0\uc5d0 \ub300\ud55c \uc5f0\uad6c\uc758 \uc5f4\uae30\ub294 \uc2dd\uc9c0 \uc54a\uc558\ub2e4. 2002\ub144 \ub9c8\uac00\ub81b \ud134\ubd88\uacfc \uc9c8 \ud0c0\ud130\ub294 \u2018\uc0dd\uba85\uc774 \uc0b4 \uc218 \uc788\ub294 \uadfc\uac70\ub9ac \ud56d\uc131\uacc4 \ubaa9\ub85d\u2019(HabCat)\uc744 \ub9cc\ub4e4\uc5c8\ub2e4. \uc774 \ubaa9\ub85d\uc5d0\ub294 1\ub9cc 7\ucc9c \uac1c \uc774\uc0c1\uc758 \uc774\ub860\uc801\uc778 \uc0dd\uba85\uccb4 \uac70\uc8fc \uac00\ub2a5 \ud589\uc131\uacc4\uac00 \uc2e4\ub824 \uc788\ub2e4. 2003\ub144 \ud134\ubd88\uc740 \ud0dc\uc591 \uc8fc\ubcc0 1\ucc9c \uad11\ub144 \uc774\ub0b4 5,000\uac1c\uc758 \ud6c4\ubcf4\ub4e4 \uc911 \uac00\uc7a5 \uc720\ub825\ud55c 30\uac1c \ud56d\uc131\uc744 \ucd94\ub824\ub0c8\uc73c\uba70 \uc5ec\uae30\uc5d0 \uace0\ub798\uc790\ub9ac \ud0c0\uc6b0\uac00 \ud3ec\ud568\ub418\uc5b4 \uc788\ub2e4. \ud134\ubd88\uc740 \uace0\ub798\uc790\ub9ac \ud0c0\uc6b0\ub97c \uc9c0\uad6c\ud615 \ud589\uc131 \ud0d0\uc0ac\uae30\ub97c \ud1b5\ud55c \uc5f0\uad6c\uc5d0 \uc801\ud569\ud55c \ub2e4\uc12f \ubcc4 \uc911 \ud558\ub098\ub85c \uaf3d\uc558\uc73c\uba70, \u201c\uc774\ub4e4 \ub2e4\uc12f \ubcc4\uc740 \ub9cc\uc57d \uc2e0\uc774 \uc6b0\ub9ac \ud589\uc131\uc744 \ub2e4\ub978 \ubcc4 \uc8fc\uc704\uc5d0 \uac00\uc838\ub2e4 \ub193\ub294\ub2e4\uba74 \uac00\uc7a5 \uc0b4\uace0 \uc2f6\uc740 \uacf3\ub4e4\uc774\ub2e4.\u201d\ub77c\uace0 \ub9d0\ud558\uae30\ub3c4 \ud588\ub2e4. \uadf8\ub7ec\ub098 NASA\uc758 \uc9c0\uad6c\ud615 \ud589\uc131 \ud0d0\uc0ac\uae30 \ubbf8\uc158\uc740 \ubbf8 \uc758\ud68c\uac00 \uc608\uc0b0\uc744 \uc0ad\uac10\ud558\uc5ec \ubb34\uae30\ud55c \uc5f0\uae30\ub418\uc5c8\uc73c\uba70, \ud604\uc7ac\ub294 \uc720\ub7fd \uc6b0\uc8fc\uad6d\uc758 \ub2e4\uc708 \uacc4\ud68d\uc774 \uace0\ub798\uc790\ub9ac \ud0c0\uc6b0\ub97c \ud0d0\uc0ac \ub300\uc0c1\uc73c\ub85c \uc9c0\ubaa9\ud558\uace0 \uc788\ub2e4.", "answer": "\ud134\ubd88", "sentence": "2002\ub144 \ub9c8\uac00\ub81b \ud134\ubd88 \uacfc \uc9c8 \ud0c0\ud130\ub294 \u2018\uc0dd\uba85\uc774 \uc0b4 \uc218 \uc788\ub294 \uadfc\uac70\ub9ac \ud56d\uc131\uacc4 \ubaa9\ub85d\u2019(HabCat)\uc744 \ub9cc\ub4e4\uc5c8\ub2e4.", "paragraph_sentence": "\uadf8\ub7ec\ub098 \uc774\ub7ec\ud55c \ube44\uad00\uc801\uc778 \uacb0\uacfc\uc5d0\ub3c4 \uace0\ub798\uc790\ub9ac \ud0c0\uc6b0\uc5d0 \ub300\ud55c \uc5f0\uad6c\uc758 \uc5f4\uae30\ub294 \uc2dd\uc9c0 \uc54a\uc558\ub2e4. 2002\ub144 \ub9c8\uac00\ub81b \ud134\ubd88 \uacfc \uc9c8 \ud0c0\ud130\ub294 \u2018\uc0dd\uba85\uc774 \uc0b4 \uc218 \uc788\ub294 \uadfc\uac70\ub9ac \ud56d\uc131\uacc4 \ubaa9\ub85d\u2019(HabCat)\uc744 \ub9cc\ub4e4\uc5c8\ub2e4. \uc774 \ubaa9\ub85d\uc5d0\ub294 1\ub9cc 7\ucc9c \uac1c \uc774\uc0c1\uc758 \uc774\ub860\uc801\uc778 \uc0dd\uba85\uccb4 \uac70\uc8fc \uac00\ub2a5 \ud589\uc131\uacc4\uac00 \uc2e4\ub824 \uc788\ub2e4. 2003\ub144 \ud134\ubd88\uc740 \ud0dc\uc591 \uc8fc\ubcc0 1\ucc9c \uad11\ub144 \uc774\ub0b4 5,000\uac1c\uc758 \ud6c4\ubcf4\ub4e4 \uc911 \uac00\uc7a5 \uc720\ub825\ud55c 30\uac1c \ud56d\uc131\uc744 \ucd94\ub824\ub0c8\uc73c\uba70 \uc5ec\uae30\uc5d0 \uace0\ub798\uc790\ub9ac \ud0c0\uc6b0\uac00 \ud3ec\ud568\ub418\uc5b4 \uc788\ub2e4. \ud134\ubd88\uc740 \uace0\ub798\uc790\ub9ac \ud0c0\uc6b0\ub97c \uc9c0\uad6c\ud615 \ud589\uc131 \ud0d0\uc0ac\uae30\ub97c \ud1b5\ud55c \uc5f0\uad6c\uc5d0 \uc801\ud569\ud55c \ub2e4\uc12f \ubcc4 \uc911 \ud558\ub098\ub85c \uaf3d\uc558\uc73c\uba70, \u201c\uc774\ub4e4 \ub2e4\uc12f \ubcc4\uc740 \ub9cc\uc57d \uc2e0\uc774 \uc6b0\ub9ac \ud589\uc131\uc744 \ub2e4\ub978 \ubcc4 \uc8fc\uc704\uc5d0 \uac00\uc838\ub2e4 \ub193\ub294\ub2e4\uba74 \uac00\uc7a5 \uc0b4\uace0 \uc2f6\uc740 \uacf3\ub4e4\uc774\ub2e4. \u201d\ub77c\uace0 \ub9d0\ud558\uae30\ub3c4 \ud588\ub2e4. \uadf8\ub7ec\ub098 NASA\uc758 \uc9c0\uad6c\ud615 \ud589\uc131 \ud0d0\uc0ac\uae30 \ubbf8\uc158\uc740 \ubbf8 \uc758\ud68c\uac00 \uc608\uc0b0\uc744 \uc0ad\uac10\ud558\uc5ec \ubb34\uae30\ud55c \uc5f0\uae30\ub418\uc5c8\uc73c\uba70, \ud604\uc7ac\ub294 \uc720\ub7fd \uc6b0\uc8fc\uad6d\uc758 \ub2e4\uc708 \uacc4\ud68d\uc774 \uace0\ub798\uc790\ub9ac \ud0c0\uc6b0\ub97c \ud0d0\uc0ac \ub300\uc0c1\uc73c\ub85c \uc9c0\ubaa9\ud558\uace0 \uc788\ub2e4.", "paragraph_answer": "\uadf8\ub7ec\ub098 \uc774\ub7ec\ud55c \ube44\uad00\uc801\uc778 \uacb0\uacfc\uc5d0\ub3c4 \uace0\ub798\uc790\ub9ac \ud0c0\uc6b0\uc5d0 \ub300\ud55c \uc5f0\uad6c\uc758 \uc5f4\uae30\ub294 \uc2dd\uc9c0 \uc54a\uc558\ub2e4. 2002\ub144 \ub9c8\uac00\ub81b \ud134\ubd88 \uacfc \uc9c8 \ud0c0\ud130\ub294 \u2018\uc0dd\uba85\uc774 \uc0b4 \uc218 \uc788\ub294 \uadfc\uac70\ub9ac \ud56d\uc131\uacc4 \ubaa9\ub85d\u2019(HabCat)\uc744 \ub9cc\ub4e4\uc5c8\ub2e4. \uc774 \ubaa9\ub85d\uc5d0\ub294 1\ub9cc 7\ucc9c \uac1c \uc774\uc0c1\uc758 \uc774\ub860\uc801\uc778 \uc0dd\uba85\uccb4 \uac70\uc8fc \uac00\ub2a5 \ud589\uc131\uacc4\uac00 \uc2e4\ub824 \uc788\ub2e4. 2003\ub144 \ud134\ubd88\uc740 \ud0dc\uc591 \uc8fc\ubcc0 1\ucc9c \uad11\ub144 \uc774\ub0b4 5,000\uac1c\uc758 \ud6c4\ubcf4\ub4e4 \uc911 \uac00\uc7a5 \uc720\ub825\ud55c 30\uac1c \ud56d\uc131\uc744 \ucd94\ub824\ub0c8\uc73c\uba70 \uc5ec\uae30\uc5d0 \uace0\ub798\uc790\ub9ac \ud0c0\uc6b0\uac00 \ud3ec\ud568\ub418\uc5b4 \uc788\ub2e4. \ud134\ubd88\uc740 \uace0\ub798\uc790\ub9ac \ud0c0\uc6b0\ub97c \uc9c0\uad6c\ud615 \ud589\uc131 \ud0d0\uc0ac\uae30\ub97c \ud1b5\ud55c \uc5f0\uad6c\uc5d0 \uc801\ud569\ud55c \ub2e4\uc12f \ubcc4 \uc911 \ud558\ub098\ub85c \uaf3d\uc558\uc73c\uba70, \u201c\uc774\ub4e4 \ub2e4\uc12f \ubcc4\uc740 \ub9cc\uc57d \uc2e0\uc774 \uc6b0\ub9ac \ud589\uc131\uc744 \ub2e4\ub978 \ubcc4 \uc8fc\uc704\uc5d0 \uac00\uc838\ub2e4 \ub193\ub294\ub2e4\uba74 \uac00\uc7a5 \uc0b4\uace0 \uc2f6\uc740 \uacf3\ub4e4\uc774\ub2e4.\u201d\ub77c\uace0 \ub9d0\ud558\uae30\ub3c4 \ud588\ub2e4. \uadf8\ub7ec\ub098 NASA\uc758 \uc9c0\uad6c\ud615 \ud589\uc131 \ud0d0\uc0ac\uae30 \ubbf8\uc158\uc740 \ubbf8 \uc758\ud68c\uac00 \uc608\uc0b0\uc744 \uc0ad\uac10\ud558\uc5ec \ubb34\uae30\ud55c \uc5f0\uae30\ub418\uc5c8\uc73c\uba70, \ud604\uc7ac\ub294 \uc720\ub7fd \uc6b0\uc8fc\uad6d\uc758 \ub2e4\uc708 \uacc4\ud68d\uc774 \uace0\ub798\uc790\ub9ac \ud0c0\uc6b0\ub97c \ud0d0\uc0ac \ub300\uc0c1\uc73c\ub85c \uc9c0\ubaa9\ud558\uace0 \uc788\ub2e4.", "sentence_answer": "2002\ub144 \ub9c8\uac00\ub81b \ud134\ubd88 \uacfc \uc9c8 \ud0c0\ud130\ub294 \u2018\uc0dd\uba85\uc774 \uc0b4 \uc218 \uc788\ub294 \uadfc\uac70\ub9ac \ud56d\uc131\uacc4 \ubaa9\ub85d\u2019(HabCat)\uc744 \ub9cc\ub4e4\uc5c8\ub2e4.", "paragraph_id": "6487118-3-1", "paragraph_question": "question: 2003\ub144 \uc774\ub860\uc801\uc778 \uc0dd\uba85\uccb4 \uac70\uc8fc \uac00\ub2a5 \ud589\uc131\uacc4 \uc911 \uac00\uc7a5 \uc720\ub825\ud55c 30\uac1c \ud56d\uc131\uc744 \ucd94\ub824\ub0b8 \uc0ac\ub78c\uc740?, context: \uadf8\ub7ec\ub098 \uc774\ub7ec\ud55c \ube44\uad00\uc801\uc778 \uacb0\uacfc\uc5d0\ub3c4 \uace0\ub798\uc790\ub9ac \ud0c0\uc6b0\uc5d0 \ub300\ud55c \uc5f0\uad6c\uc758 \uc5f4\uae30\ub294 \uc2dd\uc9c0 \uc54a\uc558\ub2e4. 2002\ub144 \ub9c8\uac00\ub81b \ud134\ubd88\uacfc \uc9c8 \ud0c0\ud130\ub294 \u2018\uc0dd\uba85\uc774 \uc0b4 \uc218 \uc788\ub294 \uadfc\uac70\ub9ac \ud56d\uc131\uacc4 \ubaa9\ub85d\u2019(HabCat)\uc744 \ub9cc\ub4e4\uc5c8\ub2e4. \uc774 \ubaa9\ub85d\uc5d0\ub294 1\ub9cc 7\ucc9c \uac1c \uc774\uc0c1\uc758 \uc774\ub860\uc801\uc778 \uc0dd\uba85\uccb4 \uac70\uc8fc \uac00\ub2a5 \ud589\uc131\uacc4\uac00 \uc2e4\ub824 \uc788\ub2e4. 2003\ub144 \ud134\ubd88\uc740 \ud0dc\uc591 \uc8fc\ubcc0 1\ucc9c \uad11\ub144 \uc774\ub0b4 5,000\uac1c\uc758 \ud6c4\ubcf4\ub4e4 \uc911 \uac00\uc7a5 \uc720\ub825\ud55c 30\uac1c \ud56d\uc131\uc744 \ucd94\ub824\ub0c8\uc73c\uba70 \uc5ec\uae30\uc5d0 \uace0\ub798\uc790\ub9ac \ud0c0\uc6b0\uac00 \ud3ec\ud568\ub418\uc5b4 \uc788\ub2e4. \ud134\ubd88\uc740 \uace0\ub798\uc790\ub9ac \ud0c0\uc6b0\ub97c \uc9c0\uad6c\ud615 \ud589\uc131 \ud0d0\uc0ac\uae30\ub97c \ud1b5\ud55c \uc5f0\uad6c\uc5d0 \uc801\ud569\ud55c \ub2e4\uc12f \ubcc4 \uc911 \ud558\ub098\ub85c \uaf3d\uc558\uc73c\uba70, \u201c\uc774\ub4e4 \ub2e4\uc12f \ubcc4\uc740 \ub9cc\uc57d \uc2e0\uc774 \uc6b0\ub9ac \ud589\uc131\uc744 \ub2e4\ub978 \ubcc4 \uc8fc\uc704\uc5d0 \uac00\uc838\ub2e4 \ub193\ub294\ub2e4\uba74 \uac00\uc7a5 \uc0b4\uace0 \uc2f6\uc740 \uacf3\ub4e4\uc774\ub2e4.\u201d\ub77c\uace0 \ub9d0\ud558\uae30\ub3c4 \ud588\ub2e4. \uadf8\ub7ec\ub098 NASA\uc758 \uc9c0\uad6c\ud615 \ud589\uc131 \ud0d0\uc0ac\uae30 \ubbf8\uc158\uc740 \ubbf8 \uc758\ud68c\uac00 \uc608\uc0b0\uc744 \uc0ad\uac10\ud558\uc5ec \ubb34\uae30\ud55c \uc5f0\uae30\ub418\uc5c8\uc73c\uba70, \ud604\uc7ac\ub294 \uc720\ub7fd \uc6b0\uc8fc\uad6d\uc758 \ub2e4\uc708 \uacc4\ud68d\uc774 \uace0\ub798\uc790\ub9ac \ud0c0\uc6b0\ub97c \ud0d0\uc0ac \ub300\uc0c1\uc73c\ub85c \uc9c0\ubaa9\ud558\uace0 \uc788\ub2e4."} {"question": "\uc5f0\uace8\uc5b4\ub958\uac00 \uac16\uace0 \uc788\ub294 \uac00\uc7a5 \uc911\uc694\ud55c \uba74\uc5ed\uae30\uad00\uc740 \ubb34\uc5c7\uc778\uac00?", "paragraph": "\uc0c1\uc5b4\uc640 \uac00\uc624\ub9ac\uac19\uc740 \uc5f0\uace8\uc5b4\ub958\ub294 \ub354\uc6b1 \ubc1c\ub2ec\ub41c \ud615\ud0dc\uc758 \uba74\uc5ed \uccb4\uacc4\ub97c \uac00\uc9c0\uace0 \uc788\ub2e4. \uc774\ub4e4\uc740 \uc5f0\uace8\uc5b4\ub958\uc5d0 \uc138 \uac1c\uc758 \ud2b9\ud654\ub41c \uae30\uad00\uc744 \uc9c0\ub2cc\ub2e4. \uc0dd\uc2dd\uc120\uc744 \ub458\ub7ec\uc2f8\ub294 \ud3ec\uc720\ub958\uc758 \ubf08\uc640 \ube44\uc2b7\ud55c \uc784\ud30c\uc870\uc9c1\uc778 \uc870\ud608\uae30\uad00, \uc5f0\uace8\uc5b4\ub958\uc758 \uc2dd\ub3c4\uc758 \ubcbd\ub4e4 \uc548\uc5d0 \uc788\ub294 \ub808\uc774\ub515\uc2a4 \uae30\uad00, \uadf8\ub9ac\uace0 \ucc3d\uc790\uc758 \ub098\uc120\ud615 \ud310\ub9c9\uc774 \uadf8\uac83\uc774\ub2e4. \uc774\ub7ec\ud55c \uae30\uad00\ub4e4\uc740 \uc640 \uacfc\ub9bd\uc131 \ubc31\ud608\uad6c, \uc784\ud30c\uad6c, \uadf8\ub9ac\uace0 \uc6d0\ud615\uc9c8 \uc138\ud3ec\uc640 \uac19\uc740 \uc804\ud615\uc801\uc778 \uba74\uc5ed\uc138\ud3ec\ub4e4\uc744 \ud3ec\ud568\ud558\uace0 \uc788\ub2e4. \ub610, \uc774\ub4e4\uc740 \ud655\uc2e4\ud55c \ud749\uc120(\uc601\uc5b4: thymus)\uacfc \uc774\ub4e4\uc758 \uac00\uc7a5 \uc911\uc694\ud55c \uba74\uc5ed\uae30\uad00\uc778 \uc798 \ubc1c\ub2ec\ub41c \ube44\uc7a5(\uc601\uc5b4: spleen)\uc744 \uac16\uace0 \uc788\ub2e4. \uc774 \ube44\uc7a5\uc5d0\ub294 \ub2e4\uc591\ud55c \uc885\ub958\uc758 \uc784\ud30c\uad6c\ub4e4\uacfc, \uc6d0\ud615\uc9c8 \uc138\ud3ec, \uadf8\ub9ac\uace0 \ub300\uc2dd\uc138\ud3ec(\uc601\uc5b4: macrophage)\ub4e4\uc774 \uac1c\ubc1c\ub418\uace0 \uc800\uc7a5\ub41c\ub2e4.", "answer": "\ube44\uc7a5", "sentence": "\ub610, \uc774\ub4e4\uc740 \ud655\uc2e4\ud55c \ud749\uc120(\uc601\uc5b4: thymus)\uacfc \uc774\ub4e4\uc758 \uac00\uc7a5 \uc911\uc694\ud55c \uba74\uc5ed\uae30\uad00\uc778 \uc798 \ubc1c\ub2ec\ub41c \ube44\uc7a5 (\uc601\uc5b4: spleen)", "paragraph_sentence": "\uc0c1\uc5b4\uc640 \uac00\uc624\ub9ac\uac19\uc740 \uc5f0\uace8\uc5b4\ub958\ub294 \ub354\uc6b1 \ubc1c\ub2ec\ub41c \ud615\ud0dc\uc758 \uba74\uc5ed \uccb4\uacc4\ub97c \uac00\uc9c0\uace0 \uc788\ub2e4. \uc774\ub4e4\uc740 \uc5f0\uace8\uc5b4\ub958\uc5d0 \uc138 \uac1c\uc758 \ud2b9\ud654\ub41c \uae30\uad00\uc744 \uc9c0\ub2cc\ub2e4. \uc0dd\uc2dd\uc120\uc744 \ub458\ub7ec\uc2f8\ub294 \ud3ec\uc720\ub958\uc758 \ubf08\uc640 \ube44\uc2b7\ud55c \uc784\ud30c\uc870\uc9c1\uc778 \uc870\ud608\uae30\uad00, \uc5f0\uace8\uc5b4\ub958\uc758 \uc2dd\ub3c4\uc758 \ubcbd\ub4e4 \uc548\uc5d0 \uc788\ub294 \ub808\uc774\ub515\uc2a4 \uae30\uad00, \uadf8\ub9ac\uace0 \ucc3d\uc790\uc758 \ub098\uc120\ud615 \ud310\ub9c9\uc774 \uadf8\uac83\uc774\ub2e4. \uc774\ub7ec\ud55c \uae30\uad00\ub4e4\uc740 \uc640 \uacfc\ub9bd\uc131 \ubc31\ud608\uad6c, \uc784\ud30c\uad6c, \uadf8\ub9ac\uace0 \uc6d0\ud615\uc9c8 \uc138\ud3ec\uc640 \uac19\uc740 \uc804\ud615\uc801\uc778 \uba74\uc5ed\uc138\ud3ec\ub4e4\uc744 \ud3ec\ud568\ud558\uace0 \uc788\ub2e4. \ub610, \uc774\ub4e4\uc740 \ud655\uc2e4\ud55c \ud749\uc120(\uc601\uc5b4: thymus)\uacfc \uc774\ub4e4\uc758 \uac00\uc7a5 \uc911\uc694\ud55c \uba74\uc5ed\uae30\uad00\uc778 \uc798 \ubc1c\ub2ec\ub41c \ube44\uc7a5 (\uc601\uc5b4: spleen) \uc744 \uac16\uace0 \uc788\ub2e4. \uc774 \ube44\uc7a5\uc5d0\ub294 \ub2e4\uc591\ud55c \uc885\ub958\uc758 \uc784\ud30c\uad6c\ub4e4\uacfc, \uc6d0\ud615\uc9c8 \uc138\ud3ec, \uadf8\ub9ac\uace0 \ub300\uc2dd\uc138\ud3ec(\uc601\uc5b4: macrophage)\ub4e4\uc774 \uac1c\ubc1c\ub418\uace0 \uc800\uc7a5\ub41c\ub2e4.", "paragraph_answer": "\uc0c1\uc5b4\uc640 \uac00\uc624\ub9ac\uac19\uc740 \uc5f0\uace8\uc5b4\ub958\ub294 \ub354\uc6b1 \ubc1c\ub2ec\ub41c \ud615\ud0dc\uc758 \uba74\uc5ed \uccb4\uacc4\ub97c \uac00\uc9c0\uace0 \uc788\ub2e4. \uc774\ub4e4\uc740 \uc5f0\uace8\uc5b4\ub958\uc5d0 \uc138 \uac1c\uc758 \ud2b9\ud654\ub41c \uae30\uad00\uc744 \uc9c0\ub2cc\ub2e4. \uc0dd\uc2dd\uc120\uc744 \ub458\ub7ec\uc2f8\ub294 \ud3ec\uc720\ub958\uc758 \ubf08\uc640 \ube44\uc2b7\ud55c \uc784\ud30c\uc870\uc9c1\uc778 \uc870\ud608\uae30\uad00, \uc5f0\uace8\uc5b4\ub958\uc758 \uc2dd\ub3c4\uc758 \ubcbd\ub4e4 \uc548\uc5d0 \uc788\ub294 \ub808\uc774\ub515\uc2a4 \uae30\uad00, \uadf8\ub9ac\uace0 \ucc3d\uc790\uc758 \ub098\uc120\ud615 \ud310\ub9c9\uc774 \uadf8\uac83\uc774\ub2e4. \uc774\ub7ec\ud55c \uae30\uad00\ub4e4\uc740 \uc640 \uacfc\ub9bd\uc131 \ubc31\ud608\uad6c, \uc784\ud30c\uad6c, \uadf8\ub9ac\uace0 \uc6d0\ud615\uc9c8 \uc138\ud3ec\uc640 \uac19\uc740 \uc804\ud615\uc801\uc778 \uba74\uc5ed\uc138\ud3ec\ub4e4\uc744 \ud3ec\ud568\ud558\uace0 \uc788\ub2e4. \ub610, \uc774\ub4e4\uc740 \ud655\uc2e4\ud55c \ud749\uc120(\uc601\uc5b4: thymus)\uacfc \uc774\ub4e4\uc758 \uac00\uc7a5 \uc911\uc694\ud55c \uba74\uc5ed\uae30\uad00\uc778 \uc798 \ubc1c\ub2ec\ub41c \ube44\uc7a5 (\uc601\uc5b4: spleen)\uc744 \uac16\uace0 \uc788\ub2e4. \uc774 \ube44\uc7a5\uc5d0\ub294 \ub2e4\uc591\ud55c \uc885\ub958\uc758 \uc784\ud30c\uad6c\ub4e4\uacfc, \uc6d0\ud615\uc9c8 \uc138\ud3ec, \uadf8\ub9ac\uace0 \ub300\uc2dd\uc138\ud3ec(\uc601\uc5b4: macrophage)\ub4e4\uc774 \uac1c\ubc1c\ub418\uace0 \uc800\uc7a5\ub41c\ub2e4.", "sentence_answer": "\ub610, \uc774\ub4e4\uc740 \ud655\uc2e4\ud55c \ud749\uc120(\uc601\uc5b4: thymus)\uacfc \uc774\ub4e4\uc758 \uac00\uc7a5 \uc911\uc694\ud55c \uba74\uc5ed\uae30\uad00\uc778 \uc798 \ubc1c\ub2ec\ub41c \ube44\uc7a5 (\uc601\uc5b4: spleen)", "paragraph_id": "6505119-10-2", "paragraph_question": "question: \uc5f0\uace8\uc5b4\ub958\uac00 \uac16\uace0 \uc788\ub294 \uac00\uc7a5 \uc911\uc694\ud55c \uba74\uc5ed\uae30\uad00\uc740 \ubb34\uc5c7\uc778\uac00?, context: \uc0c1\uc5b4\uc640 \uac00\uc624\ub9ac\uac19\uc740 \uc5f0\uace8\uc5b4\ub958\ub294 \ub354\uc6b1 \ubc1c\ub2ec\ub41c \ud615\ud0dc\uc758 \uba74\uc5ed \uccb4\uacc4\ub97c \uac00\uc9c0\uace0 \uc788\ub2e4. \uc774\ub4e4\uc740 \uc5f0\uace8\uc5b4\ub958\uc5d0 \uc138 \uac1c\uc758 \ud2b9\ud654\ub41c \uae30\uad00\uc744 \uc9c0\ub2cc\ub2e4. \uc0dd\uc2dd\uc120\uc744 \ub458\ub7ec\uc2f8\ub294 \ud3ec\uc720\ub958\uc758 \ubf08\uc640 \ube44\uc2b7\ud55c \uc784\ud30c\uc870\uc9c1\uc778 \uc870\ud608\uae30\uad00, \uc5f0\uace8\uc5b4\ub958\uc758 \uc2dd\ub3c4\uc758 \ubcbd\ub4e4 \uc548\uc5d0 \uc788\ub294 \ub808\uc774\ub515\uc2a4 \uae30\uad00, \uadf8\ub9ac\uace0 \ucc3d\uc790\uc758 \ub098\uc120\ud615 \ud310\ub9c9\uc774 \uadf8\uac83\uc774\ub2e4. \uc774\ub7ec\ud55c \uae30\uad00\ub4e4\uc740 \uc640 \uacfc\ub9bd\uc131 \ubc31\ud608\uad6c, \uc784\ud30c\uad6c, \uadf8\ub9ac\uace0 \uc6d0\ud615\uc9c8 \uc138\ud3ec\uc640 \uac19\uc740 \uc804\ud615\uc801\uc778 \uba74\uc5ed\uc138\ud3ec\ub4e4\uc744 \ud3ec\ud568\ud558\uace0 \uc788\ub2e4. \ub610, \uc774\ub4e4\uc740 \ud655\uc2e4\ud55c \ud749\uc120(\uc601\uc5b4: thymus)\uacfc \uc774\ub4e4\uc758 \uac00\uc7a5 \uc911\uc694\ud55c \uba74\uc5ed\uae30\uad00\uc778 \uc798 \ubc1c\ub2ec\ub41c \ube44\uc7a5(\uc601\uc5b4: spleen)\uc744 \uac16\uace0 \uc788\ub2e4. \uc774 \ube44\uc7a5\uc5d0\ub294 \ub2e4\uc591\ud55c \uc885\ub958\uc758 \uc784\ud30c\uad6c\ub4e4\uacfc, \uc6d0\ud615\uc9c8 \uc138\ud3ec, \uadf8\ub9ac\uace0 \ub300\uc2dd\uc138\ud3ec(\uc601\uc5b4: macrophage)\ub4e4\uc774 \uac1c\ubc1c\ub418\uace0 \uc800\uc7a5\ub41c\ub2e4."} {"question": "\uace0\uc885 \ud669\uc81c\uc774 \ub300\ud55c\uc81c\uad6d\uc744 \uc218\ub9bd\ud558\uba70 \uc815\ud55c \uc5f0\ud638\ub294?", "paragraph": "\uadfc\ub300 \ud55c\uad6d\uc758 \ubfcc\ub9ac\ub294 1897\ub144\uc5d0 \uc218\ub9bd\ub41c \ub300\ud55c\uc81c\uad6d\uc774\ub2e4. \uace0\uc885 \ud669\uc81c\ub294 \ub098\ub77c \uc774\ub984\uc744 \uc870\uc120(\u671d\u9bae)\uc5d0\uc11c \ub300\ud55c(\u5927\u97d3)\uc73c\ub85c \uace0\uce58\uace0, \uc5f0\ud638\ub97c \uad11\ubb34(\u5149\u6b66)\ub77c \uc815\ud558\uace0, \uc2a4\uc2a4\ub85c \ud669\uc81c\uc758 \uc790\ub9ac\uc5d0 \uc62c\ub790\ub2e4. \ucd5c\ucd08\uc758 \uadfc\ub300\uc801 \ud5cc\ubc95\uc778 \ub300\ud55c\uad6d \uad6d\uc81c\ub97c \ubc18\ud3ec\ud558\uc600\uace0, \ubbf8\uad6d, \uc601\uad6d, \ub3c5\uc77c, \ud504\ub791\uc2a4, \uc774\ud0c8\ub9ac\uc544, \ub7ec\uc2dc\uc544, \uc77c\ubcf8, \uccad\ub098\ub77c \ub4f1\uacfc \uc218\uad50\ud558\uc600\ub2e4. \uad11\ubb34\uac1c\ud601\uc744 \ub2e8\ud589\ud558\uc5ec \uc2e0\uc2dd\uad70\ub300\ub97c \uc124\uce58\ud558\uace0, \uadfc\ub300\uc801 \uc0ac\ubc95\uc81c\ub3c4\ub97c \ub3c4\uc785\ud558\uc600\uc73c\uba70, \uadfc\ub300\uc801 \ud1a0\uc9c0 \uc81c\ub3c4\ub97c \ub3c4\uc785\ud558\uace0, \uc0c1\uacf5\uc5c5\uc744 \uc9c4\ud765\ud558\uace0, \uadfc\ub300\uc801 \ubcd1\uc6d0\uacfc \ud559\uad50 \ubc0f \uc740\ud589 \ub4f1\uc744 \uc124\ub9bd\ud558\uc600\ub2e4. \ud574\uc678\uc5d0 \uc720\ud559\uc0dd\uc744 \ud30c\uacac\ud558\uc5ec \uadfc\ub300 \uc0b0\uc5c5\uae30\uc220\uc744 \uc2b5\ub4dd\ud558\uac8c \ud558\uace0, \uc81c\uc870\u00b7\ucca0\ub3c4\u00b7\uc6b4\uc218 \ub4f1 \uc5ec\ub7ec \ubd84\uc57c\uc5d0\uc11c \uadfc\ub300\uc801 \uae30\uc5c5\uacfc \uacf5\uc7a5\uc744 \uc124\ub9bd\ud558\uc600\ub2e4. \uadf8\ub7ec\ub098 \ub300\ud55c\uc81c\uad6d\uc758 \uc790\uc8fc\uc801 \uadfc\ub300\ud654 \ub178\ub825\uc740 1895\ub144 \uccad\uc77c \uc804\uc7c1\uc73c\ub85c \uc77c\ubcf8\uc758 \uc601\ud5a5\ub825\uc774 \ucee4\uc9c4 \uc774\ub798 \uac04\uc12d\uc744 \ubc1b\uc544\uc654\uc73c\uba70, 1904\ub144\uc5d0\ub294 \ub7ec\uc77c\uc804\uc7c1\uc5d0\uc11c \uc77c\ubcf8\uc774 \uc2b9\ub9ac\ud55c \ub4a4\ub85c \uc77c\ubcf8\uc758 \ud55c\uad6d \ubcd1\ud569\uc774 \ubcf8\uaca9\ud654\ub418\uc5c8\ub2e4. 1905\ub144 \uc744\uc0ac\uc870\uc57d\uc744 \ubb34\ub825\uc801\uc73c\ub85c \uccb4\uacb0\ud558\uc5ec \uc678\uad50\uad8c\uc744 \ubc15\ud0c8\ud558\uace0, 1907\ub144 \uace0\uc885 \ud669\uc81c\ub97c \uac15\uc81c\ub85c \ud1f4\uc704\uc2dc\ud0a4\uace0, 1910\ub144 \ud55c\uc77c \ubcd1\ud569 \uc870\uc57d\uc744 \ud1b5\ud574 \uad6d\uad8c\uc744 \ube7c\uc557\uae30\uba70 \uc77c\uc81c \uac15\uc810\uae30\ub85c \uc804\ud658, \uadfc\ub300\uc801 \uc790\uc8fc\uad6d\uac00\ub77c\ub294 \ubaa9\ud45c\ub294 \uc0ac\ub77c\uc9c0\uace0 \ub9d0\uc558\ub2e4.", "answer": "\uad11\ubb34", "sentence": "\uace0\uc885 \ud669\uc81c\ub294 \ub098\ub77c \uc774\ub984\uc744 \uc870\uc120(\u671d\u9bae)\uc5d0\uc11c \ub300\ud55c(\u5927\u97d3)\uc73c\ub85c \uace0\uce58\uace0, \uc5f0\ud638\ub97c \uad11\ubb34 (\u5149\u6b66)\ub77c \uc815\ud558\uace0, \uc2a4\uc2a4\ub85c \ud669\uc81c\uc758 \uc790\ub9ac\uc5d0 \uc62c\ub790\ub2e4.", "paragraph_sentence": "\uadfc\ub300 \ud55c\uad6d\uc758 \ubfcc\ub9ac\ub294 1897\ub144\uc5d0 \uc218\ub9bd\ub41c \ub300\ud55c\uc81c\uad6d\uc774\ub2e4. \uace0\uc885 \ud669\uc81c\ub294 \ub098\ub77c \uc774\ub984\uc744 \uc870\uc120(\u671d\u9bae)\uc5d0\uc11c \ub300\ud55c(\u5927\u97d3)\uc73c\ub85c \uace0\uce58\uace0, \uc5f0\ud638\ub97c \uad11\ubb34 (\u5149\u6b66)\ub77c \uc815\ud558\uace0, \uc2a4\uc2a4\ub85c \ud669\uc81c\uc758 \uc790\ub9ac\uc5d0 \uc62c\ub790\ub2e4. \ucd5c\ucd08\uc758 \uadfc\ub300\uc801 \ud5cc\ubc95\uc778 \ub300\ud55c\uad6d \uad6d\uc81c\ub97c \ubc18\ud3ec\ud558\uc600\uace0, \ubbf8\uad6d, \uc601\uad6d, \ub3c5\uc77c, \ud504\ub791\uc2a4, \uc774\ud0c8\ub9ac\uc544, \ub7ec\uc2dc\uc544, \uc77c\ubcf8, \uccad\ub098\ub77c \ub4f1\uacfc \uc218\uad50\ud558\uc600\ub2e4. \uad11\ubb34\uac1c\ud601\uc744 \ub2e8\ud589\ud558\uc5ec \uc2e0\uc2dd\uad70\ub300\ub97c \uc124\uce58\ud558\uace0, \uadfc\ub300\uc801 \uc0ac\ubc95\uc81c\ub3c4\ub97c \ub3c4\uc785\ud558\uc600\uc73c\uba70, \uadfc\ub300\uc801 \ud1a0\uc9c0 \uc81c\ub3c4\ub97c \ub3c4\uc785\ud558\uace0, \uc0c1\uacf5\uc5c5\uc744 \uc9c4\ud765\ud558\uace0, \uadfc\ub300\uc801 \ubcd1\uc6d0\uacfc \ud559\uad50 \ubc0f \uc740\ud589 \ub4f1\uc744 \uc124\ub9bd\ud558\uc600\ub2e4. \ud574\uc678\uc5d0 \uc720\ud559\uc0dd\uc744 \ud30c\uacac\ud558\uc5ec \uadfc\ub300 \uc0b0\uc5c5\uae30\uc220\uc744 \uc2b5\ub4dd\ud558\uac8c \ud558\uace0, \uc81c\uc870\u00b7\ucca0\ub3c4\u00b7\uc6b4\uc218 \ub4f1 \uc5ec\ub7ec \ubd84\uc57c\uc5d0\uc11c \uadfc\ub300\uc801 \uae30\uc5c5\uacfc \uacf5\uc7a5\uc744 \uc124\ub9bd\ud558\uc600\ub2e4. \uadf8\ub7ec\ub098 \ub300\ud55c\uc81c\uad6d\uc758 \uc790\uc8fc\uc801 \uadfc\ub300\ud654 \ub178\ub825\uc740 1895\ub144 \uccad\uc77c \uc804\uc7c1\uc73c\ub85c \uc77c\ubcf8\uc758 \uc601\ud5a5\ub825\uc774 \ucee4\uc9c4 \uc774\ub798 \uac04\uc12d\uc744 \ubc1b\uc544\uc654\uc73c\uba70, 1904\ub144\uc5d0\ub294 \ub7ec\uc77c\uc804\uc7c1\uc5d0\uc11c \uc77c\ubcf8\uc774 \uc2b9\ub9ac\ud55c \ub4a4\ub85c \uc77c\ubcf8\uc758 \ud55c\uad6d \ubcd1\ud569\uc774 \ubcf8\uaca9\ud654\ub418\uc5c8\ub2e4. 1905\ub144 \uc744\uc0ac\uc870\uc57d\uc744 \ubb34\ub825\uc801\uc73c\ub85c \uccb4\uacb0\ud558\uc5ec \uc678\uad50\uad8c\uc744 \ubc15\ud0c8\ud558\uace0, 1907\ub144 \uace0\uc885 \ud669\uc81c\ub97c \uac15\uc81c\ub85c \ud1f4\uc704\uc2dc\ud0a4\uace0, 1910\ub144 \ud55c\uc77c \ubcd1\ud569 \uc870\uc57d\uc744 \ud1b5\ud574 \uad6d\uad8c\uc744 \ube7c\uc557\uae30\uba70 \uc77c\uc81c \uac15\uc810\uae30\ub85c \uc804\ud658, \uadfc\ub300\uc801 \uc790\uc8fc\uad6d\uac00\ub77c\ub294 \ubaa9\ud45c\ub294 \uc0ac\ub77c\uc9c0\uace0 \ub9d0\uc558\ub2e4.", "paragraph_answer": "\uadfc\ub300 \ud55c\uad6d\uc758 \ubfcc\ub9ac\ub294 1897\ub144\uc5d0 \uc218\ub9bd\ub41c \ub300\ud55c\uc81c\uad6d\uc774\ub2e4. \uace0\uc885 \ud669\uc81c\ub294 \ub098\ub77c \uc774\ub984\uc744 \uc870\uc120(\u671d\u9bae)\uc5d0\uc11c \ub300\ud55c(\u5927\u97d3)\uc73c\ub85c \uace0\uce58\uace0, \uc5f0\ud638\ub97c \uad11\ubb34 (\u5149\u6b66)\ub77c \uc815\ud558\uace0, \uc2a4\uc2a4\ub85c \ud669\uc81c\uc758 \uc790\ub9ac\uc5d0 \uc62c\ub790\ub2e4. \ucd5c\ucd08\uc758 \uadfc\ub300\uc801 \ud5cc\ubc95\uc778 \ub300\ud55c\uad6d \uad6d\uc81c\ub97c \ubc18\ud3ec\ud558\uc600\uace0, \ubbf8\uad6d, \uc601\uad6d, \ub3c5\uc77c, \ud504\ub791\uc2a4, \uc774\ud0c8\ub9ac\uc544, \ub7ec\uc2dc\uc544, \uc77c\ubcf8, \uccad\ub098\ub77c \ub4f1\uacfc \uc218\uad50\ud558\uc600\ub2e4. \uad11\ubb34\uac1c\ud601\uc744 \ub2e8\ud589\ud558\uc5ec \uc2e0\uc2dd\uad70\ub300\ub97c \uc124\uce58\ud558\uace0, \uadfc\ub300\uc801 \uc0ac\ubc95\uc81c\ub3c4\ub97c \ub3c4\uc785\ud558\uc600\uc73c\uba70, \uadfc\ub300\uc801 \ud1a0\uc9c0 \uc81c\ub3c4\ub97c \ub3c4\uc785\ud558\uace0, \uc0c1\uacf5\uc5c5\uc744 \uc9c4\ud765\ud558\uace0, \uadfc\ub300\uc801 \ubcd1\uc6d0\uacfc \ud559\uad50 \ubc0f \uc740\ud589 \ub4f1\uc744 \uc124\ub9bd\ud558\uc600\ub2e4. \ud574\uc678\uc5d0 \uc720\ud559\uc0dd\uc744 \ud30c\uacac\ud558\uc5ec \uadfc\ub300 \uc0b0\uc5c5\uae30\uc220\uc744 \uc2b5\ub4dd\ud558\uac8c \ud558\uace0, \uc81c\uc870\u00b7\ucca0\ub3c4\u00b7\uc6b4\uc218 \ub4f1 \uc5ec\ub7ec \ubd84\uc57c\uc5d0\uc11c \uadfc\ub300\uc801 \uae30\uc5c5\uacfc \uacf5\uc7a5\uc744 \uc124\ub9bd\ud558\uc600\ub2e4. \uadf8\ub7ec\ub098 \ub300\ud55c\uc81c\uad6d\uc758 \uc790\uc8fc\uc801 \uadfc\ub300\ud654 \ub178\ub825\uc740 1895\ub144 \uccad\uc77c \uc804\uc7c1\uc73c\ub85c \uc77c\ubcf8\uc758 \uc601\ud5a5\ub825\uc774 \ucee4\uc9c4 \uc774\ub798 \uac04\uc12d\uc744 \ubc1b\uc544\uc654\uc73c\uba70, 1904\ub144\uc5d0\ub294 \ub7ec\uc77c\uc804\uc7c1\uc5d0\uc11c \uc77c\ubcf8\uc774 \uc2b9\ub9ac\ud55c \ub4a4\ub85c \uc77c\ubcf8\uc758 \ud55c\uad6d \ubcd1\ud569\uc774 \ubcf8\uaca9\ud654\ub418\uc5c8\ub2e4. 1905\ub144 \uc744\uc0ac\uc870\uc57d\uc744 \ubb34\ub825\uc801\uc73c\ub85c \uccb4\uacb0\ud558\uc5ec \uc678\uad50\uad8c\uc744 \ubc15\ud0c8\ud558\uace0, 1907\ub144 \uace0\uc885 \ud669\uc81c\ub97c \uac15\uc81c\ub85c \ud1f4\uc704\uc2dc\ud0a4\uace0, 1910\ub144 \ud55c\uc77c \ubcd1\ud569 \uc870\uc57d\uc744 \ud1b5\ud574 \uad6d\uad8c\uc744 \ube7c\uc557\uae30\uba70 \uc77c\uc81c \uac15\uc810\uae30\ub85c \uc804\ud658, \uadfc\ub300\uc801 \uc790\uc8fc\uad6d\uac00\ub77c\ub294 \ubaa9\ud45c\ub294 \uc0ac\ub77c\uc9c0\uace0 \ub9d0\uc558\ub2e4.", "sentence_answer": "\uace0\uc885 \ud669\uc81c\ub294 \ub098\ub77c \uc774\ub984\uc744 \uc870\uc120(\u671d\u9bae)\uc5d0\uc11c \ub300\ud55c(\u5927\u97d3)\uc73c\ub85c \uace0\uce58\uace0, \uc5f0\ud638\ub97c \uad11\ubb34 (\u5149\u6b66)\ub77c \uc815\ud558\uace0, \uc2a4\uc2a4\ub85c \ud669\uc81c\uc758 \uc790\ub9ac\uc5d0 \uc62c\ub790\ub2e4.", "paragraph_id": "6485683-4-0", "paragraph_question": "question: \uace0\uc885 \ud669\uc81c\uc774 \ub300\ud55c\uc81c\uad6d\uc744 \uc218\ub9bd\ud558\uba70 \uc815\ud55c \uc5f0\ud638\ub294?, context: \uadfc\ub300 \ud55c\uad6d\uc758 \ubfcc\ub9ac\ub294 1897\ub144\uc5d0 \uc218\ub9bd\ub41c \ub300\ud55c\uc81c\uad6d\uc774\ub2e4. \uace0\uc885 \ud669\uc81c\ub294 \ub098\ub77c \uc774\ub984\uc744 \uc870\uc120(\u671d\u9bae)\uc5d0\uc11c \ub300\ud55c(\u5927\u97d3)\uc73c\ub85c \uace0\uce58\uace0, \uc5f0\ud638\ub97c \uad11\ubb34(\u5149\u6b66)\ub77c \uc815\ud558\uace0, \uc2a4\uc2a4\ub85c \ud669\uc81c\uc758 \uc790\ub9ac\uc5d0 \uc62c\ub790\ub2e4. \ucd5c\ucd08\uc758 \uadfc\ub300\uc801 \ud5cc\ubc95\uc778 \ub300\ud55c\uad6d \uad6d\uc81c\ub97c \ubc18\ud3ec\ud558\uc600\uace0, \ubbf8\uad6d, \uc601\uad6d, \ub3c5\uc77c, \ud504\ub791\uc2a4, \uc774\ud0c8\ub9ac\uc544, \ub7ec\uc2dc\uc544, \uc77c\ubcf8, \uccad\ub098\ub77c \ub4f1\uacfc \uc218\uad50\ud558\uc600\ub2e4. \uad11\ubb34\uac1c\ud601\uc744 \ub2e8\ud589\ud558\uc5ec \uc2e0\uc2dd\uad70\ub300\ub97c \uc124\uce58\ud558\uace0, \uadfc\ub300\uc801 \uc0ac\ubc95\uc81c\ub3c4\ub97c \ub3c4\uc785\ud558\uc600\uc73c\uba70, \uadfc\ub300\uc801 \ud1a0\uc9c0 \uc81c\ub3c4\ub97c \ub3c4\uc785\ud558\uace0, \uc0c1\uacf5\uc5c5\uc744 \uc9c4\ud765\ud558\uace0, \uadfc\ub300\uc801 \ubcd1\uc6d0\uacfc \ud559\uad50 \ubc0f \uc740\ud589 \ub4f1\uc744 \uc124\ub9bd\ud558\uc600\ub2e4. \ud574\uc678\uc5d0 \uc720\ud559\uc0dd\uc744 \ud30c\uacac\ud558\uc5ec \uadfc\ub300 \uc0b0\uc5c5\uae30\uc220\uc744 \uc2b5\ub4dd\ud558\uac8c \ud558\uace0, \uc81c\uc870\u00b7\ucca0\ub3c4\u00b7\uc6b4\uc218 \ub4f1 \uc5ec\ub7ec \ubd84\uc57c\uc5d0\uc11c \uadfc\ub300\uc801 \uae30\uc5c5\uacfc \uacf5\uc7a5\uc744 \uc124\ub9bd\ud558\uc600\ub2e4. \uadf8\ub7ec\ub098 \ub300\ud55c\uc81c\uad6d\uc758 \uc790\uc8fc\uc801 \uadfc\ub300\ud654 \ub178\ub825\uc740 1895\ub144 \uccad\uc77c \uc804\uc7c1\uc73c\ub85c \uc77c\ubcf8\uc758 \uc601\ud5a5\ub825\uc774 \ucee4\uc9c4 \uc774\ub798 \uac04\uc12d\uc744 \ubc1b\uc544\uc654\uc73c\uba70, 1904\ub144\uc5d0\ub294 \ub7ec\uc77c\uc804\uc7c1\uc5d0\uc11c \uc77c\ubcf8\uc774 \uc2b9\ub9ac\ud55c \ub4a4\ub85c \uc77c\ubcf8\uc758 \ud55c\uad6d \ubcd1\ud569\uc774 \ubcf8\uaca9\ud654\ub418\uc5c8\ub2e4. 1905\ub144 \uc744\uc0ac\uc870\uc57d\uc744 \ubb34\ub825\uc801\uc73c\ub85c \uccb4\uacb0\ud558\uc5ec \uc678\uad50\uad8c\uc744 \ubc15\ud0c8\ud558\uace0, 1907\ub144 \uace0\uc885 \ud669\uc81c\ub97c \uac15\uc81c\ub85c \ud1f4\uc704\uc2dc\ud0a4\uace0, 1910\ub144 \ud55c\uc77c \ubcd1\ud569 \uc870\uc57d\uc744 \ud1b5\ud574 \uad6d\uad8c\uc744 \ube7c\uc557\uae30\uba70 \uc77c\uc81c \uac15\uc810\uae30\ub85c \uc804\ud658, \uadfc\ub300\uc801 \uc790\uc8fc\uad6d\uac00\ub77c\ub294 \ubaa9\ud45c\ub294 \uc0ac\ub77c\uc9c0\uace0 \ub9d0\uc558\ub2e4."} {"question": "\uce85\uce74\uc2a4 \uc5d0\ubbf8\ub808\uc774\ud2b8 \ub2e8\uccb4\ub97c \ucc3d\uc124\ud55c \uc778\ubb3c\uc740?", "paragraph": "2010\ub144 \ubaa8\uc2a4\ud06c\ubc14 \uc9c0\ud558\ucca0 \ud14c\ub7ec\ub294 2007\ub144 \uc774\uce58\ucf00\ub9ac\uc57c \uccb4\uccb8 \uacf5\ud654\uad6d\uc744 \uacc4\uc2b9\ud55c \uccb4\uccb8\uc758 \ud14c\ub7ec \ub2e8\uccb4\uc778 \uce85\uce74\uc2a4 \uc5d0\ubbf8\ub808\uc774\ud2b8\uac00 \uc8fc\ub3c4\ud588\ub2e4. \uce85\uce74\uc2a4 \uc5d0\ubbf8\ub808\uc774\ud2b8\uc758 \ucc3d\uc124 \ubc30\uacbd\uc744 \uac70\uc2ac\ub7ec \uc62c\ub77c\uac00\uba74 \uccb4\uccb8\uc758 \ub3c5\ub9bd\uc744 \ub458\ub7ec\uc2fc \uc815\uce58 \uc138\ub825\uc758 \ubd84\uc5f4\uc774 \uc788\ub2e4. 1999\ub144 2\ucc28 \uccb4\uccb8\uc804\uc7c1\uc5d0\uc11c \ub7ec\uc2dc\uc544 \uc5f0\ubc29\uc774 \uc2b9\ub9ac\ud55c \uc9c1\ud6c4, \uccb4\uccb8\uc758 \uc815\uce58\uc138\ub825\uc740 \ub450 \ud30c\ubc8c\ub85c \ub098\ub258\uc5c8\ub2e4. \ub7ec\uc2dc\uc544 \uc5f0\ubc29\uc5d0 \uc794\uc874\ud558\uae38 \ud76c\ub9dd\ud558\ub294 \uc138\ub825\uc740 \ub7ec\uc2dc\uc544\uc758 \uc790\uce58\uacf5\ud654\uad6d\uc73c\ub85c\uc368 \uccb4\uccb8 \uacf5\ud654\uad6d\uc744 \uac74\uad6d, \uc790\uce58\uad8c\uc744 \uc778\uc815\ubc1b\uc558\uc73c\ub098, \ubd84\ub9ac\ub3c5\ub9bd\ud30c\ub294 \uccb4\uccb8\uc758 \uc644\uc804\ud55c \ub3c5\ub9bd\uc744 \ubaa9\ud45c\ub85c \ub7ec\uc2dc\uc544\ub97c \uc0c1\ub300\ub85c \uacc4\uc18d\uc801\uc73c\ub85c \ubb34\uc7a5\ud22c\uc7c1\uc744 \ubc8c\uc600\ub2e4. \uc804\uc7c1\uc774 \uae38\uc5b4\uc9d0\uc5d0 \ub530\ub77c \uc628\uac74\ud30c \uc815\uce58\uc778\uc774\uc5c8\ub358 \ub9c8\uc2a4\ud558\ub3c4\ud504 \ub4f1, \uc8fc\uc694 \uc9c0\ub3c4\uce35\uc744 \uc783\uc73c\uba70 \uc138\ub825 \uc720\uc9c0\uc5d0 \ud070 \ud0c0\uaca9\uc744 \uc785\uc5c8\ub2e4. \uc5ec\uae30\uc5d0 \uccb4\uccb8 \uac15\uacbd\ud30c\uac00 \ubc8c\uc778 \ubbfc\uac04\uc778 \ud14c\ub7ec \uc0ac\uac74\ub4e4\ub85c \uc774\uce58\ucf00\ub9ac\uc57c \uccb4\uccb8 \uc815\ubd80\ub294 \uc624\ud788\ub824 \uad6d\uc81c\uc801\uc73c\ub85c \uc774\ubbf8\uc9c0\uac00 \uc545\ud654\ub418\uba74\uc11c \ubd84\ub9ac\ub3c5\ub9bd\ud30c\ub294 \uc790\uc5f0\uc2a4\ub808 \uc57d\ud654\ub418\uc5c8\ub2e4. \uc774\ub7f0 \uc0c1\ud669\uc5d0\uc11c 2007\ub144 \uc774\uce58\ucf00\ub9ac\uc57c \uccb4\uccb8 \uacf5\ud654\uad6d\uc758 \ub300\ud1b5\ub839\uc774\uc5c8\ub358 \uc774\uce58\ucf00\ub9ac\uc57c \uccb4\uccb8 \uc815\ubd80\ub97c \uc5c6\uc560\uace0 \ub3c4\ucfe0 \uc6b0\ub9c8\ub85c\ud504\ub294 \uc790\uc2e0\uc744 \uc218\uc7a5\uc73c\ub85c \uc120\ucd9c, \uce85\uce74\uc2a4 \uc5d0\ubbf8\ub808\uc774\ud2b8\uc774\ub77c\ub294 \ub2e8\uccb4\ub97c \uc0c8\ub86d\uac8c \ucc3d\uc124\ud588\ub2e4. \ud604\uc7ac \uc774\ub4e4\uc740 \ubd81\uce85\uce74\uc2a4 \uc9c0\uc5ed\uc5d0 \uc774\uc2ac\ub78c \uad6d\uac00\ub97c \uc218\ub9bd\ud558\ub294 \uac83\uc744 \ubaa9\ud45c\ub85c \ub7ec\uc2dc\uc544\uc5d0 \ub300\ud55c \ud14c\ub7ec\ub97c \ubc8c\uc774\uace0 \uc788\uc73c\uba70, \ub7ec\uc2dc\uc544 \ub4f1 \uc5ec\ub7ec \uad6d\uac00\ub294 \uce85\uce74\uc2a4 \uc218\uc7a5\uad6d\uc744 \ud14c\ub7ec \ub2e8\uccb4\ub85c \ubd84\ub958\ud558\uace0 \uc788\ub2e4. 2002\ub144 \uc774\ud6c4 \ud604\uc7ac\uae4c\uc9c0 \uc774\ub4e4\uc774 \ubc8c\uc774\ub294 \ud14c\ub7ec\uc5d0 \uc758\ud574 5000\uba85 \uc774\uc0c1\uc758 \uc0ac\uc0c1\uc790\uac00 \uc18d\ucd9c\ud558\uace0 \uc788\ub2e4.", "answer": "\ub3c4\ucfe0 \uc6b0\ub9c8\ub85c\ud504", "sentence": "\uc774\ub7f0 \uc0c1\ud669\uc5d0\uc11c 2007\ub144 \uc774\uce58\ucf00\ub9ac\uc57c \uccb4\uccb8 \uacf5\ud654\uad6d\uc758 \ub300\ud1b5\ub839\uc774\uc5c8\ub358 \uc774\uce58\ucf00\ub9ac\uc57c \uccb4\uccb8 \uc815\ubd80\ub97c \uc5c6\uc560\uace0 \ub3c4\ucfe0 \uc6b0\ub9c8\ub85c\ud504 \ub294 \uc790\uc2e0\uc744 \uc218\uc7a5\uc73c\ub85c \uc120\ucd9c, \uce85\uce74\uc2a4 \uc5d0\ubbf8\ub808\uc774\ud2b8\uc774\ub77c\ub294 \ub2e8\uccb4\ub97c \uc0c8\ub86d\uac8c \ucc3d\uc124\ud588\ub2e4.", "paragraph_sentence": "2010\ub144 \ubaa8\uc2a4\ud06c\ubc14 \uc9c0\ud558\ucca0 \ud14c\ub7ec\ub294 2007\ub144 \uc774\uce58\ucf00\ub9ac\uc57c \uccb4\uccb8 \uacf5\ud654\uad6d\uc744 \uacc4\uc2b9\ud55c \uccb4\uccb8\uc758 \ud14c\ub7ec \ub2e8\uccb4\uc778 \uce85\uce74\uc2a4 \uc5d0\ubbf8\ub808\uc774\ud2b8\uac00 \uc8fc\ub3c4\ud588\ub2e4. \uce85\uce74\uc2a4 \uc5d0\ubbf8\ub808\uc774\ud2b8\uc758 \ucc3d\uc124 \ubc30\uacbd\uc744 \uac70\uc2ac\ub7ec \uc62c\ub77c\uac00\uba74 \uccb4\uccb8\uc758 \ub3c5\ub9bd\uc744 \ub458\ub7ec\uc2fc \uc815\uce58 \uc138\ub825\uc758 \ubd84\uc5f4\uc774 \uc788\ub2e4. 1999\ub144 2\ucc28 \uccb4\uccb8\uc804\uc7c1\uc5d0\uc11c \ub7ec\uc2dc\uc544 \uc5f0\ubc29\uc774 \uc2b9\ub9ac\ud55c \uc9c1\ud6c4, \uccb4\uccb8\uc758 \uc815\uce58\uc138\ub825\uc740 \ub450 \ud30c\ubc8c\ub85c \ub098\ub258\uc5c8\ub2e4. \ub7ec\uc2dc\uc544 \uc5f0\ubc29\uc5d0 \uc794\uc874\ud558\uae38 \ud76c\ub9dd\ud558\ub294 \uc138\ub825\uc740 \ub7ec\uc2dc\uc544\uc758 \uc790\uce58\uacf5\ud654\uad6d\uc73c\ub85c\uc368 \uccb4\uccb8 \uacf5\ud654\uad6d\uc744 \uac74\uad6d, \uc790\uce58\uad8c\uc744 \uc778\uc815\ubc1b\uc558\uc73c\ub098, \ubd84\ub9ac\ub3c5\ub9bd\ud30c\ub294 \uccb4\uccb8\uc758 \uc644\uc804\ud55c \ub3c5\ub9bd\uc744 \ubaa9\ud45c\ub85c \ub7ec\uc2dc\uc544\ub97c \uc0c1\ub300\ub85c \uacc4\uc18d\uc801\uc73c\ub85c \ubb34\uc7a5\ud22c\uc7c1\uc744 \ubc8c\uc600\ub2e4. \uc804\uc7c1\uc774 \uae38\uc5b4\uc9d0\uc5d0 \ub530\ub77c \uc628\uac74\ud30c \uc815\uce58\uc778\uc774\uc5c8\ub358 \ub9c8\uc2a4\ud558\ub3c4\ud504 \ub4f1, \uc8fc\uc694 \uc9c0\ub3c4\uce35\uc744 \uc783\uc73c\uba70 \uc138\ub825 \uc720\uc9c0\uc5d0 \ud070 \ud0c0\uaca9\uc744 \uc785\uc5c8\ub2e4. \uc5ec\uae30\uc5d0 \uccb4\uccb8 \uac15\uacbd\ud30c\uac00 \ubc8c\uc778 \ubbfc\uac04\uc778 \ud14c\ub7ec \uc0ac\uac74\ub4e4\ub85c \uc774\uce58\ucf00\ub9ac\uc57c \uccb4\uccb8 \uc815\ubd80\ub294 \uc624\ud788\ub824 \uad6d\uc81c\uc801\uc73c\ub85c \uc774\ubbf8\uc9c0\uac00 \uc545\ud654\ub418\uba74\uc11c \ubd84\ub9ac\ub3c5\ub9bd\ud30c\ub294 \uc790\uc5f0\uc2a4\ub808 \uc57d\ud654\ub418\uc5c8\ub2e4. \uc774\ub7f0 \uc0c1\ud669\uc5d0\uc11c 2007\ub144 \uc774\uce58\ucf00\ub9ac\uc57c \uccb4\uccb8 \uacf5\ud654\uad6d\uc758 \ub300\ud1b5\ub839\uc774\uc5c8\ub358 \uc774\uce58\ucf00\ub9ac\uc57c \uccb4\uccb8 \uc815\ubd80\ub97c \uc5c6\uc560\uace0 \ub3c4\ucfe0 \uc6b0\ub9c8\ub85c\ud504 \ub294 \uc790\uc2e0\uc744 \uc218\uc7a5\uc73c\ub85c \uc120\ucd9c, \uce85\uce74\uc2a4 \uc5d0\ubbf8\ub808\uc774\ud2b8\uc774\ub77c\ub294 \ub2e8\uccb4\ub97c \uc0c8\ub86d\uac8c \ucc3d\uc124\ud588\ub2e4. \ud604\uc7ac \uc774\ub4e4\uc740 \ubd81\uce85\uce74\uc2a4 \uc9c0\uc5ed\uc5d0 \uc774\uc2ac\ub78c \uad6d\uac00\ub97c \uc218\ub9bd\ud558\ub294 \uac83\uc744 \ubaa9\ud45c\ub85c \ub7ec\uc2dc\uc544\uc5d0 \ub300\ud55c \ud14c\ub7ec\ub97c \ubc8c\uc774\uace0 \uc788\uc73c\uba70, \ub7ec\uc2dc\uc544 \ub4f1 \uc5ec\ub7ec \uad6d\uac00\ub294 \uce85\uce74\uc2a4 \uc218\uc7a5\uad6d\uc744 \ud14c\ub7ec \ub2e8\uccb4\ub85c \ubd84\ub958\ud558\uace0 \uc788\ub2e4. 2002\ub144 \uc774\ud6c4 \ud604\uc7ac\uae4c\uc9c0 \uc774\ub4e4\uc774 \ubc8c\uc774\ub294 \ud14c\ub7ec\uc5d0 \uc758\ud574 5000\uba85 \uc774\uc0c1\uc758 \uc0ac\uc0c1\uc790\uac00 \uc18d\ucd9c\ud558\uace0 \uc788\ub2e4.", "paragraph_answer": "2010\ub144 \ubaa8\uc2a4\ud06c\ubc14 \uc9c0\ud558\ucca0 \ud14c\ub7ec\ub294 2007\ub144 \uc774\uce58\ucf00\ub9ac\uc57c \uccb4\uccb8 \uacf5\ud654\uad6d\uc744 \uacc4\uc2b9\ud55c \uccb4\uccb8\uc758 \ud14c\ub7ec \ub2e8\uccb4\uc778 \uce85\uce74\uc2a4 \uc5d0\ubbf8\ub808\uc774\ud2b8\uac00 \uc8fc\ub3c4\ud588\ub2e4. \uce85\uce74\uc2a4 \uc5d0\ubbf8\ub808\uc774\ud2b8\uc758 \ucc3d\uc124 \ubc30\uacbd\uc744 \uac70\uc2ac\ub7ec \uc62c\ub77c\uac00\uba74 \uccb4\uccb8\uc758 \ub3c5\ub9bd\uc744 \ub458\ub7ec\uc2fc \uc815\uce58 \uc138\ub825\uc758 \ubd84\uc5f4\uc774 \uc788\ub2e4. 1999\ub144 2\ucc28 \uccb4\uccb8\uc804\uc7c1\uc5d0\uc11c \ub7ec\uc2dc\uc544 \uc5f0\ubc29\uc774 \uc2b9\ub9ac\ud55c \uc9c1\ud6c4, \uccb4\uccb8\uc758 \uc815\uce58\uc138\ub825\uc740 \ub450 \ud30c\ubc8c\ub85c \ub098\ub258\uc5c8\ub2e4. \ub7ec\uc2dc\uc544 \uc5f0\ubc29\uc5d0 \uc794\uc874\ud558\uae38 \ud76c\ub9dd\ud558\ub294 \uc138\ub825\uc740 \ub7ec\uc2dc\uc544\uc758 \uc790\uce58\uacf5\ud654\uad6d\uc73c\ub85c\uc368 \uccb4\uccb8 \uacf5\ud654\uad6d\uc744 \uac74\uad6d, \uc790\uce58\uad8c\uc744 \uc778\uc815\ubc1b\uc558\uc73c\ub098, \ubd84\ub9ac\ub3c5\ub9bd\ud30c\ub294 \uccb4\uccb8\uc758 \uc644\uc804\ud55c \ub3c5\ub9bd\uc744 \ubaa9\ud45c\ub85c \ub7ec\uc2dc\uc544\ub97c \uc0c1\ub300\ub85c \uacc4\uc18d\uc801\uc73c\ub85c \ubb34\uc7a5\ud22c\uc7c1\uc744 \ubc8c\uc600\ub2e4. \uc804\uc7c1\uc774 \uae38\uc5b4\uc9d0\uc5d0 \ub530\ub77c \uc628\uac74\ud30c \uc815\uce58\uc778\uc774\uc5c8\ub358 \ub9c8\uc2a4\ud558\ub3c4\ud504 \ub4f1, \uc8fc\uc694 \uc9c0\ub3c4\uce35\uc744 \uc783\uc73c\uba70 \uc138\ub825 \uc720\uc9c0\uc5d0 \ud070 \ud0c0\uaca9\uc744 \uc785\uc5c8\ub2e4. \uc5ec\uae30\uc5d0 \uccb4\uccb8 \uac15\uacbd\ud30c\uac00 \ubc8c\uc778 \ubbfc\uac04\uc778 \ud14c\ub7ec \uc0ac\uac74\ub4e4\ub85c \uc774\uce58\ucf00\ub9ac\uc57c \uccb4\uccb8 \uc815\ubd80\ub294 \uc624\ud788\ub824 \uad6d\uc81c\uc801\uc73c\ub85c \uc774\ubbf8\uc9c0\uac00 \uc545\ud654\ub418\uba74\uc11c \ubd84\ub9ac\ub3c5\ub9bd\ud30c\ub294 \uc790\uc5f0\uc2a4\ub808 \uc57d\ud654\ub418\uc5c8\ub2e4. \uc774\ub7f0 \uc0c1\ud669\uc5d0\uc11c 2007\ub144 \uc774\uce58\ucf00\ub9ac\uc57c \uccb4\uccb8 \uacf5\ud654\uad6d\uc758 \ub300\ud1b5\ub839\uc774\uc5c8\ub358 \uc774\uce58\ucf00\ub9ac\uc57c \uccb4\uccb8 \uc815\ubd80\ub97c \uc5c6\uc560\uace0 \ub3c4\ucfe0 \uc6b0\ub9c8\ub85c\ud504 \ub294 \uc790\uc2e0\uc744 \uc218\uc7a5\uc73c\ub85c \uc120\ucd9c, \uce85\uce74\uc2a4 \uc5d0\ubbf8\ub808\uc774\ud2b8\uc774\ub77c\ub294 \ub2e8\uccb4\ub97c \uc0c8\ub86d\uac8c \ucc3d\uc124\ud588\ub2e4. \ud604\uc7ac \uc774\ub4e4\uc740 \ubd81\uce85\uce74\uc2a4 \uc9c0\uc5ed\uc5d0 \uc774\uc2ac\ub78c \uad6d\uac00\ub97c \uc218\ub9bd\ud558\ub294 \uac83\uc744 \ubaa9\ud45c\ub85c \ub7ec\uc2dc\uc544\uc5d0 \ub300\ud55c \ud14c\ub7ec\ub97c \ubc8c\uc774\uace0 \uc788\uc73c\uba70, \ub7ec\uc2dc\uc544 \ub4f1 \uc5ec\ub7ec \uad6d\uac00\ub294 \uce85\uce74\uc2a4 \uc218\uc7a5\uad6d\uc744 \ud14c\ub7ec \ub2e8\uccb4\ub85c \ubd84\ub958\ud558\uace0 \uc788\ub2e4. 2002\ub144 \uc774\ud6c4 \ud604\uc7ac\uae4c\uc9c0 \uc774\ub4e4\uc774 \ubc8c\uc774\ub294 \ud14c\ub7ec\uc5d0 \uc758\ud574 5000\uba85 \uc774\uc0c1\uc758 \uc0ac\uc0c1\uc790\uac00 \uc18d\ucd9c\ud558\uace0 \uc788\ub2e4.", "sentence_answer": "\uc774\ub7f0 \uc0c1\ud669\uc5d0\uc11c 2007\ub144 \uc774\uce58\ucf00\ub9ac\uc57c \uccb4\uccb8 \uacf5\ud654\uad6d\uc758 \ub300\ud1b5\ub839\uc774\uc5c8\ub358 \uc774\uce58\ucf00\ub9ac\uc57c \uccb4\uccb8 \uc815\ubd80\ub97c \uc5c6\uc560\uace0 \ub3c4\ucfe0 \uc6b0\ub9c8\ub85c\ud504 \ub294 \uc790\uc2e0\uc744 \uc218\uc7a5\uc73c\ub85c \uc120\ucd9c, \uce85\uce74\uc2a4 \uc5d0\ubbf8\ub808\uc774\ud2b8\uc774\ub77c\ub294 \ub2e8\uccb4\ub97c \uc0c8\ub86d\uac8c \ucc3d\uc124\ud588\ub2e4.", "paragraph_id": "6471101-1-1", "paragraph_question": "question: \uce85\uce74\uc2a4 \uc5d0\ubbf8\ub808\uc774\ud2b8 \ub2e8\uccb4\ub97c \ucc3d\uc124\ud55c \uc778\ubb3c\uc740?, context: 2010\ub144 \ubaa8\uc2a4\ud06c\ubc14 \uc9c0\ud558\ucca0 \ud14c\ub7ec\ub294 2007\ub144 \uc774\uce58\ucf00\ub9ac\uc57c \uccb4\uccb8 \uacf5\ud654\uad6d\uc744 \uacc4\uc2b9\ud55c \uccb4\uccb8\uc758 \ud14c\ub7ec \ub2e8\uccb4\uc778 \uce85\uce74\uc2a4 \uc5d0\ubbf8\ub808\uc774\ud2b8\uac00 \uc8fc\ub3c4\ud588\ub2e4. \uce85\uce74\uc2a4 \uc5d0\ubbf8\ub808\uc774\ud2b8\uc758 \ucc3d\uc124 \ubc30\uacbd\uc744 \uac70\uc2ac\ub7ec \uc62c\ub77c\uac00\uba74 \uccb4\uccb8\uc758 \ub3c5\ub9bd\uc744 \ub458\ub7ec\uc2fc \uc815\uce58 \uc138\ub825\uc758 \ubd84\uc5f4\uc774 \uc788\ub2e4. 1999\ub144 2\ucc28 \uccb4\uccb8\uc804\uc7c1\uc5d0\uc11c \ub7ec\uc2dc\uc544 \uc5f0\ubc29\uc774 \uc2b9\ub9ac\ud55c \uc9c1\ud6c4, \uccb4\uccb8\uc758 \uc815\uce58\uc138\ub825\uc740 \ub450 \ud30c\ubc8c\ub85c \ub098\ub258\uc5c8\ub2e4. \ub7ec\uc2dc\uc544 \uc5f0\ubc29\uc5d0 \uc794\uc874\ud558\uae38 \ud76c\ub9dd\ud558\ub294 \uc138\ub825\uc740 \ub7ec\uc2dc\uc544\uc758 \uc790\uce58\uacf5\ud654\uad6d\uc73c\ub85c\uc368 \uccb4\uccb8 \uacf5\ud654\uad6d\uc744 \uac74\uad6d, \uc790\uce58\uad8c\uc744 \uc778\uc815\ubc1b\uc558\uc73c\ub098, \ubd84\ub9ac\ub3c5\ub9bd\ud30c\ub294 \uccb4\uccb8\uc758 \uc644\uc804\ud55c \ub3c5\ub9bd\uc744 \ubaa9\ud45c\ub85c \ub7ec\uc2dc\uc544\ub97c \uc0c1\ub300\ub85c \uacc4\uc18d\uc801\uc73c\ub85c \ubb34\uc7a5\ud22c\uc7c1\uc744 \ubc8c\uc600\ub2e4. \uc804\uc7c1\uc774 \uae38\uc5b4\uc9d0\uc5d0 \ub530\ub77c \uc628\uac74\ud30c \uc815\uce58\uc778\uc774\uc5c8\ub358 \ub9c8\uc2a4\ud558\ub3c4\ud504 \ub4f1, \uc8fc\uc694 \uc9c0\ub3c4\uce35\uc744 \uc783\uc73c\uba70 \uc138\ub825 \uc720\uc9c0\uc5d0 \ud070 \ud0c0\uaca9\uc744 \uc785\uc5c8\ub2e4. \uc5ec\uae30\uc5d0 \uccb4\uccb8 \uac15\uacbd\ud30c\uac00 \ubc8c\uc778 \ubbfc\uac04\uc778 \ud14c\ub7ec \uc0ac\uac74\ub4e4\ub85c \uc774\uce58\ucf00\ub9ac\uc57c \uccb4\uccb8 \uc815\ubd80\ub294 \uc624\ud788\ub824 \uad6d\uc81c\uc801\uc73c\ub85c \uc774\ubbf8\uc9c0\uac00 \uc545\ud654\ub418\uba74\uc11c \ubd84\ub9ac\ub3c5\ub9bd\ud30c\ub294 \uc790\uc5f0\uc2a4\ub808 \uc57d\ud654\ub418\uc5c8\ub2e4. \uc774\ub7f0 \uc0c1\ud669\uc5d0\uc11c 2007\ub144 \uc774\uce58\ucf00\ub9ac\uc57c \uccb4\uccb8 \uacf5\ud654\uad6d\uc758 \ub300\ud1b5\ub839\uc774\uc5c8\ub358 \uc774\uce58\ucf00\ub9ac\uc57c \uccb4\uccb8 \uc815\ubd80\ub97c \uc5c6\uc560\uace0 \ub3c4\ucfe0 \uc6b0\ub9c8\ub85c\ud504\ub294 \uc790\uc2e0\uc744 \uc218\uc7a5\uc73c\ub85c \uc120\ucd9c, \uce85\uce74\uc2a4 \uc5d0\ubbf8\ub808\uc774\ud2b8\uc774\ub77c\ub294 \ub2e8\uccb4\ub97c \uc0c8\ub86d\uac8c \ucc3d\uc124\ud588\ub2e4. \ud604\uc7ac \uc774\ub4e4\uc740 \ubd81\uce85\uce74\uc2a4 \uc9c0\uc5ed\uc5d0 \uc774\uc2ac\ub78c \uad6d\uac00\ub97c \uc218\ub9bd\ud558\ub294 \uac83\uc744 \ubaa9\ud45c\ub85c \ub7ec\uc2dc\uc544\uc5d0 \ub300\ud55c \ud14c\ub7ec\ub97c \ubc8c\uc774\uace0 \uc788\uc73c\uba70, \ub7ec\uc2dc\uc544 \ub4f1 \uc5ec\ub7ec \uad6d\uac00\ub294 \uce85\uce74\uc2a4 \uc218\uc7a5\uad6d\uc744 \ud14c\ub7ec \ub2e8\uccb4\ub85c \ubd84\ub958\ud558\uace0 \uc788\ub2e4. 2002\ub144 \uc774\ud6c4 \ud604\uc7ac\uae4c\uc9c0 \uc774\ub4e4\uc774 \ubc8c\uc774\ub294 \ud14c\ub7ec\uc5d0 \uc758\ud574 5000\uba85 \uc774\uc0c1\uc758 \uc0ac\uc0c1\uc790\uac00 \uc18d\ucd9c\ud558\uace0 \uc788\ub2e4."} {"question": "\ucd5c\uc9c4\uc2e4\uc758 \uc774\ud63c \ubb38\uc81c\ub85c \uae30\uc874 \uc774\ubbf8\uc9c0\ub97c \uc783\uc5b4, \ub354\uc774\uc0c1 \ucd5c\uc9c4\uc2e4\uc744\n \ucc3e\ub294 \uc0ac\ub78c\uc740 \uc5c6\uc744\uaebc\ub77c\uace0 \ud55c \uae30\uc790\ub294?", "paragraph": "\ucd5c\uc9c4\uc2e4\uc758 \uc5b4\uba38\ub2c8 \uc815\uc625\uc219\uc740 \uc790\uc2e0\uc758 \uc800\uc11c \u300a\uc5c4\ub9c8\uac00, \ubbf8\uc548\ud574 \uadf8\ub9ac\uace0 \uc0ac\ub791\ud574\u300b(2011\ub144)\uc5d0\uc11c \"\uc5b8\ub860\uc740 \uc784\uc2e0\ud55c \uc0ac\ub78c\uc744, \uc544\uc774 \uc5b4\uba38\ub2c8\ub97c \ubc30\ub824\ud558\uc9c0 \uc54a\uc558\ub2e4\"\ub77c\uace0 \uc9c0\uc801\ud558\uba70 \uae30\uc790\ub4e4\uc758 \uc724\ub9ac \uc758\uc2dd\uc5d0 \uc758\uad6c\uc2ec\uc744 \ud45c\ud604\ud588\ub2e4. \ucd5c\uc9c4\uc2e4\uc740 \uc774\ud63c \uacfc\uc815\uc5d0 \uc788\uc744 \ub54c \uc784\uc2e0\ud55c \uc0c1\ud0dc\uc600\ub294\ub370, \uae30\uc790\ub4e4\uc774 \ucd5c\uc9c4\uc2e4\uc758 \uc9d1 \uc55e\uc5d0 \uc9c4\uc744 \uce58\uace0 \uc788\uc5b4\uc11c \ucd5c\uc9c4\uc2e4\uc740 \uc815\uae30 \uac80\uc9c4\uc744 \ubc1b\ub294 \ub0a0 \uc678\uc5d0\ub294 \uac70\uc758 \uc9d1\uc5d0 \uac07\ud600 \uc788\uc5b4\uc57c \ud588\uace0, \uc774\ud63c \uacfc\uc815\uc744 \uc0dd\uc911\uacc4\ud558\ub294 \ub4ef\ud55c \uc120\uc815\uc801\uc774\uace0 \ubb34\ubd84\ubcc4\ud55c \uae30\uc0ac\uc5d0 \uace0\ud1b5\uc744 \uacaa\uae30\ub3c4 \ud558\uc600\ub2e4. \ucd5c\uc9c4\uc2e4\uc740 2\ub144\uc5ec \ub3d9\uc548\uc758 \uc774\ud63c \uacfc\uc815\uc5d0\uc11c \ud5c8\uc704\uc0ac\uc2e4\uc774\ub098 \uc0ac\uc2e4 \uc65c\uace1\uc744 \ubc14\ub85c\uc7a1\uae30 \uc704\ud574 \ubc18\ubc15\uc131\u00b7\ud574\uba85\uc131 \uc778\ud130\ubdf0\ub97c \ud558\uac70\ub098 \ubc95\uc801 \uc870\uce58\ub97c \ucde8\ud558\uae30\ub3c4 \ud558\uc600\ub294\ub370, \uc774\ub294 \uc0ac\uc0dd\ud65c\uc774 \ub354 \uc774\uc0c1 \uc138\uc0c1\uc5d0 \uc54c\ub824\uc9c0\uace0 \uc65c\uace1\ub418\ub294 \uac83\uc744 \ub9c9\uaca0\ub2e4\ub294 \ucde8\uc9c0\uc600\ub2e4. \uadf8 \ubc95\uc801 \uc870\uce58\uc758 \ub300\uc0c1\uc5d0\ub294 \uc5b8\ub860\ub3c4 \ud3ec\ud568\ub418\uc5b4 \uc788\uc5c8\ub2e4. \uc778\ud130\ubdf0\ub97c \uc790\uccad\ud558\uc5ec \ud5c8\uc704\uc0ac\uc2e4\uc744 \uc720\ud3ec\ud55c \uc790\uac00 \uc788\uc5c8\uace0 \uadf8 \uc778\ud130\ubdf0\ub97c \uc9c4\ud589\ud55c \uae30\uc790\ub294 \uadf8 \uc720\ud3ec \ub0b4\uc6a9\uc744 \uadf8\ub300\ub85c \ubc1b\uc544\uc801\uc5b4 \uae30\uc0ac\ud654\ud558\uc600\ub294\ub370, \uc774\ub294 \uae30\uc790\uac00 \ud5c8\uc704\uc0ac\uc2e4\uc720\ud3ec \ud589\uc704\uc5d0 \ub3d9\ucc38\ud55c \uac83\uc774\uba74\uc11c \ud5c8\uc704\uc0ac\uc2e4 \uc720\ud3ec\uc790\uc640 \uae30\uc790 \ubaa8\ub450 \uc774\ud63c\uc740 \ud63c\uc778 \ub2f9\uc0ac\uc790\uc640 \uad6d\uac00\uac00 \ud574\uacb0\ubd10\uc57c \ud558\ub294 \uc0ac\uc548\uc774\ub77c\ub294 \uc0ac\uc2e4\uc744 \ub9dd\uac01\ud55c \ud589\uc704\uc774\uae30\ub3c4 \ud558\uc600\ub2e4. \uc774\ub7f0 \ud5c8\uc704\uc0ac\uc2e4\uc720\ud3ec \ud589\uc704\uc5d0 \ubc29\uc1a1\uc0ac\ub3c4 \ub3d9\ucc38\ud588\ub294\ub370, \uc774\ub97c \ud1b5\ud574 \ud5c8\uc704\uc0ac\uc2e4\uc740 \ud65c\uc790\uc640 \uc601\uc0c1\uc744 \ud1b5\ud574 \ub110\ub9ac \ud37c\uc838 \ub098\uac14\uace0, \uac70\uc758 \uc804 \uad6d\ubbfc\uc801\uc778 \uc778\uc9c0\ub3c4\ub97c \uac00\uc9c4 \ucd5c\uc9c4\uc2e4\uc774 \ubc1b\uc740 \ud0c0\uaca9\uc740 \uc0c1\ub2f9\ud788 \ucef8\uc73c\uba70, \ud6d7\ub0a0 \ucd5c\uc9c4\uc2e4\uc740 \uc774 \ub54c\uc758 \uc0c1\ud669\uc5d0 \ub300\ud574 \"\uad6d\ubbfc\uc744 '\ubc30\uc2ec\uc6d0'\uc73c\ub85c \uc549\ud600\ub193\uace0 2~3\ub144\uac04 \uc774\ud63c \ubb38\uc81c\ub85c \uc2dc\ub044\ub7fd\uac8c \ub5a0\ub4e4\uc5c8\ub2e4. \uadf8 \uacfc\uc815\uc5d0\uc11c \uce58\ubd80\ub97c \uc0dd\uc0dd\ud558\uac8c \ubcf4\uc5ec\uc918 \ub098\ub3c4 \uc544\uc774\ub4e4\ub3c4 \uce58\uba85\uc801\uc778 \uc0c1\ucc98\ub97c \uc785\uc5c8\ub2e4\"\ub77c\uace0 \ub9d0\ud558\uae30\ub3c4 \ud558\uc600\ub2e4. \uadf8 \ub2f9\uc2dc \uc5b8\ub860 \uc0c1\ud669\uc5d0 \ub300\ud574 \uc5f0\ucd9c\uac00 \uc815\uae38\ud654\ub294 \"IT \uac15\uad6d \ud55c\uad6d\uc758 \uc778\ud130\ub137\uacfc \ub09c\ub9bd\ud55c \uc5f0\uc608 \uc800\ub110\ub9ac\uc998\uc740 \uadf8 \ubaa8\ub4e0 \uc0c1\ud669\uc744 \uac70\uc758 \uc2e4\uc2dc\uac04\uc73c\ub85c \uc911\uacc4\ud588\ub2e4\"\ub77c\uace0 \ud45c\ud604\ud588\ub2e4. \uae30\uc790 \uc8fc\uc9c4\uc6b0\ub294 \"\ucd5c\uc9c4\uc2e4\uc740 \uadc0\uc5fd\uace0 \ubc1c\ub784\ud55c \uc5ec\uc131\uc758 \ub300\uba85\uc0ac\uc600\ub2e4. \uc774\ud63c \ub54c\ubb38\uc5d0 \uae54\ub054\ud55c \uc774\ubbf8\uc9c0\uac00 \uc190\uc0c1\ub418\uba74\uc11c \ucd5c\uc9c4\uc2e4\uc740 \ub098\ub77d\uc5d0 \ub5a8\uc5b4\uc84c\ub2e4. \uc5f0\uc608\uacc4\uc5d0\uc11c \uadf8\ub97c \ucc3e\ub294 \uacf3\uc740 \uc544\ubb34 \ub370\ub3c4 \uc5c6\uc5c8\ub2e4. 2004\ub144\uc5d0 MBC\uc758 \ud55c \uad6d\uc7a5\uc740 '\ucd5c\uc9c4\uc2e4\uc740 \ub05d\ub0ac\ub2e4. \uadc0\uc5fd\uace0 \ubc1c\ub784\ud568\uc73c\ub85c \ub5a0\uc624\ub974\ub358 \ucd5c\uc9c4\uc2e4\uc774 \uc774\uc81c\ub294 \uac00\uc815\ud3ed\ub825\uacfc \uc774\ud63c\uc758 \ub300\uba85\uc0ac\uac00 \ub418\uc5c8\ub294\ub370, \ucd5c\uc9c4\uc2e4\uc744 \uc5b4\ub514\uc5d0\ub2e4 \uc4f0\uaca0\ub294\uac00'\ub77c\uace0 \ub9d0\ud588\ub2e4\"\ub77c\uace0 \ub9d0\ud588\ub2e4. \uc774\ub294 \uac00\uc815\ud3ed\ub825\uc774\ub77c\ub294 \ubc94\uc8c4 \ud589\uc704\uc758 \ud53c\ud574\uc790\uc5d0\uac8c \uc624\ud788\ub824 \uadf8 \uc6b4\uc2e0\uc758 \ud3ed\uc744 \uc81c\ud55c\ud558\ub294 \ub300\ud55c\ubbfc\uad6d \ubc29\uc1a1\uacc4\uc758 \uc778\uc2dd \uc218\uc900\uc744 \ub2e8\uc801\uc73c\ub85c \ubcf4\uc5ec\uc8fc\ub294 \uac83\uc774\uc5c8\uace0, \uacb0\ud63c\uc744 \ud558\uba74 \uc774\ud63c\ud560 \uac00\ub2a5\uc131\uc740 \ub298 \uc788\ub294\ub370, \ud569\ubc95\uc801\uc778 \ud589\uc704\uc778 \uc774\ud63c\uc744 \ud588\ub2e4\ub294 \uc774\uc720\ub85c \uc0ac\ud68c\ud65c\ub3d9\uc744 \uc81c\ud55c\ubc1b\uc744 \uc218 \uc788\ub2e4\ub294 \uc0ac\ud68c \ubd84\uc704\uae30\ub97c \ub4dc\ub7ec\ub0b4\ub294 \uac83\uc774\uae30\ub3c4 \ud558\uc600\ub2e4. \ud3ed\ub825\ubc94\uc774 \uc720\uba85\uc778\uc744 \uc0c1\ub300\ub85c \ub300\ub193\uace0 \ud3ed\ub825 \ud589\uc704\ub97c \uc800\uc9c0\ub978 \uac83\uc740 \uc774\ub7f0 \uc0ac\ud68c \ubd84\uc704\uae30\uc640 \ubb34\uad00\ud558\uc9c0 \uc54a\uace0 \uc774\ub7f0 \uc0ac\ud68c \ubd84\uc704\uae30\ub294 \uc624\ud788\ub824 \uac00\uc815\ud3ed\ub825\uc744 \uc870\uc7a5\ud558\ub294 \uba74\uc774 \uc5c6\uc9c0 \uc54a\uc558\ub294\ub370, \ucd5c\uc9c4\uc2e4\uc740 \uc774\ud63c \uc219\ub824 \uae30\uac04\uc744 \uac00\uc9c0\ub294 \ub3d9\uc548\uc5d0 \uc54c\ub824\uc9c4 \uac83\ub9cc\uc73c\ub85c\ub3c4 \uac19\uc740 \uac00\ud574\uc790\ub85c\ubd80\ud130 \ub450 \ubc88\uc758 \uac00\uc815\ud3ed\ub825 \ud589\uc704\ub97c \ub2f9\ud588\uc5c8\uace0, \uccab \ubc88\uc9f8 \ud3ed\ub825 \ud589\uc704\ub294 \uc784\uc2e0 \uc911\uc5d0 \ub2f9\ud55c \uac83\uc774\uae30 \ub54c\ubb38\uc5d0 \ud0dc\uc544\ub3c4 \uadf8 \ud589\uc704\uc758 \ud53c\ud574\uc790\uc600\ub2e4. \uc774\ud63c \uacfc\uc815\uc5d0\uc11c \ucd5c\uc9c4\uc2e4\uc740 \ubbfc\uc0ac\uc0c1\u00b7\ud615\uc0ac\uc0c1 \ud53c\ud574\uc790\uc600\uc9c0\ub9cc, \uac1c\uc778\uc758 \uc544\ud508 \uc0ac\uc0dd\ud65c\uc5d0 \ub300\ud574\uc11c \uadf8\ub9ac\uace0 \uadf8 \ud53c\ud574\uc790\uc5d0 \ub300\ud574\uc11c \ud765\ubbf8 \uc704\uc8fc \uac00\uc2ed\uc73c\ub85c \ud314\uc544\uba39\ub294 \uc77c\ubd80 \uc5b8\ub860\uc758 '\uc0c1\ud63c' \ub4f1\uc73c\ub85c \uc774\uc911\uc73c\ub85c \uace0\ud1b5\uc744 \ubc1b\uc544\uc57c \ud588\ub2e4.", "answer": "\uc8fc\uc9c4\uc6b0", "sentence": "\uae30\uc790 \uc8fc\uc9c4\uc6b0 \ub294 \"\ucd5c\uc9c4\uc2e4\uc740 \uadc0\uc5fd\uace0 \ubc1c\ub784\ud55c \uc5ec\uc131\uc758 \ub300\uba85\uc0ac\uc600\ub2e4.", "paragraph_sentence": "\ucd5c\uc9c4\uc2e4\uc758 \uc5b4\uba38\ub2c8 \uc815\uc625\uc219\uc740 \uc790\uc2e0\uc758 \uc800\uc11c \u300a\uc5c4\ub9c8\uac00, \ubbf8\uc548\ud574 \uadf8\ub9ac\uace0 \uc0ac\ub791\ud574\u300b(2011\ub144)\uc5d0\uc11c \"\uc5b8\ub860\uc740 \uc784\uc2e0\ud55c \uc0ac\ub78c\uc744, \uc544\uc774 \uc5b4\uba38\ub2c8\ub97c \ubc30\ub824\ud558\uc9c0 \uc54a\uc558\ub2e4\"\ub77c\uace0 \uc9c0\uc801\ud558\uba70 \uae30\uc790\ub4e4\uc758 \uc724\ub9ac \uc758\uc2dd\uc5d0 \uc758\uad6c\uc2ec\uc744 \ud45c\ud604\ud588\ub2e4. \ucd5c\uc9c4\uc2e4\uc740 \uc774\ud63c \uacfc\uc815\uc5d0 \uc788\uc744 \ub54c \uc784\uc2e0\ud55c \uc0c1\ud0dc\uc600\ub294\ub370, \uae30\uc790\ub4e4\uc774 \ucd5c\uc9c4\uc2e4\uc758 \uc9d1 \uc55e\uc5d0 \uc9c4\uc744 \uce58\uace0 \uc788\uc5b4\uc11c \ucd5c\uc9c4\uc2e4\uc740 \uc815\uae30 \uac80\uc9c4\uc744 \ubc1b\ub294 \ub0a0 \uc678\uc5d0\ub294 \uac70\uc758 \uc9d1\uc5d0 \uac07\ud600 \uc788\uc5b4\uc57c \ud588\uace0, \uc774\ud63c \uacfc\uc815\uc744 \uc0dd\uc911\uacc4\ud558\ub294 \ub4ef\ud55c \uc120\uc815\uc801\uc774\uace0 \ubb34\ubd84\ubcc4\ud55c \uae30\uc0ac\uc5d0 \uace0\ud1b5\uc744 \uacaa\uae30\ub3c4 \ud558\uc600\ub2e4. \ucd5c\uc9c4\uc2e4\uc740 2\ub144\uc5ec \ub3d9\uc548\uc758 \uc774\ud63c \uacfc\uc815\uc5d0\uc11c \ud5c8\uc704\uc0ac\uc2e4\uc774\ub098 \uc0ac\uc2e4 \uc65c\uace1\uc744 \ubc14\ub85c\uc7a1\uae30 \uc704\ud574 \ubc18\ubc15\uc131\u00b7\ud574\uba85\uc131 \uc778\ud130\ubdf0\ub97c \ud558\uac70\ub098 \ubc95\uc801 \uc870\uce58\ub97c \ucde8\ud558\uae30\ub3c4 \ud558\uc600\ub294\ub370, \uc774\ub294 \uc0ac\uc0dd\ud65c\uc774 \ub354 \uc774\uc0c1 \uc138\uc0c1\uc5d0 \uc54c\ub824\uc9c0\uace0 \uc65c\uace1\ub418\ub294 \uac83\uc744 \ub9c9\uaca0\ub2e4\ub294 \ucde8\uc9c0\uc600\ub2e4. \uadf8 \ubc95\uc801 \uc870\uce58\uc758 \ub300\uc0c1\uc5d0\ub294 \uc5b8\ub860\ub3c4 \ud3ec\ud568\ub418\uc5b4 \uc788\uc5c8\ub2e4. \uc778\ud130\ubdf0\ub97c \uc790\uccad\ud558\uc5ec \ud5c8\uc704\uc0ac\uc2e4\uc744 \uc720\ud3ec\ud55c \uc790\uac00 \uc788\uc5c8\uace0 \uadf8 \uc778\ud130\ubdf0\ub97c \uc9c4\ud589\ud55c \uae30\uc790\ub294 \uadf8 \uc720\ud3ec \ub0b4\uc6a9\uc744 \uadf8\ub300\ub85c \ubc1b\uc544\uc801\uc5b4 \uae30\uc0ac\ud654\ud558\uc600\ub294\ub370, \uc774\ub294 \uae30\uc790\uac00 \ud5c8\uc704\uc0ac\uc2e4\uc720\ud3ec \ud589\uc704\uc5d0 \ub3d9\ucc38\ud55c \uac83\uc774\uba74\uc11c \ud5c8\uc704\uc0ac\uc2e4 \uc720\ud3ec\uc790\uc640 \uae30\uc790 \ubaa8\ub450 \uc774\ud63c\uc740 \ud63c\uc778 \ub2f9\uc0ac\uc790\uc640 \uad6d\uac00\uac00 \ud574\uacb0\ubd10\uc57c \ud558\ub294 \uc0ac\uc548\uc774\ub77c\ub294 \uc0ac\uc2e4\uc744 \ub9dd\uac01\ud55c \ud589\uc704\uc774\uae30\ub3c4 \ud558\uc600\ub2e4. \uc774\ub7f0 \ud5c8\uc704\uc0ac\uc2e4\uc720\ud3ec \ud589\uc704\uc5d0 \ubc29\uc1a1\uc0ac\ub3c4 \ub3d9\ucc38\ud588\ub294\ub370, \uc774\ub97c \ud1b5\ud574 \ud5c8\uc704\uc0ac\uc2e4\uc740 \ud65c\uc790\uc640 \uc601\uc0c1\uc744 \ud1b5\ud574 \ub110\ub9ac \ud37c\uc838 \ub098\uac14\uace0, \uac70\uc758 \uc804 \uad6d\ubbfc\uc801\uc778 \uc778\uc9c0\ub3c4\ub97c \uac00\uc9c4 \ucd5c\uc9c4\uc2e4\uc774 \ubc1b\uc740 \ud0c0\uaca9\uc740 \uc0c1\ub2f9\ud788 \ucef8\uc73c\uba70, \ud6d7\ub0a0 \ucd5c\uc9c4\uc2e4\uc740 \uc774 \ub54c\uc758 \uc0c1\ud669\uc5d0 \ub300\ud574 \"\uad6d\ubbfc\uc744 '\ubc30\uc2ec\uc6d0'\uc73c\ub85c \uc549\ud600\ub193\uace0 2~3\ub144\uac04 \uc774\ud63c \ubb38\uc81c\ub85c \uc2dc\ub044\ub7fd\uac8c \ub5a0\ub4e4\uc5c8\ub2e4. \uadf8 \uacfc\uc815\uc5d0\uc11c \uce58\ubd80\ub97c \uc0dd\uc0dd\ud558\uac8c \ubcf4\uc5ec\uc918 \ub098\ub3c4 \uc544\uc774\ub4e4\ub3c4 \uce58\uba85\uc801\uc778 \uc0c1\ucc98\ub97c \uc785\uc5c8\ub2e4\"\ub77c\uace0 \ub9d0\ud558\uae30\ub3c4 \ud558\uc600\ub2e4. \uadf8 \ub2f9\uc2dc \uc5b8\ub860 \uc0c1\ud669\uc5d0 \ub300\ud574 \uc5f0\ucd9c\uac00 \uc815\uae38\ud654\ub294 \"IT \uac15\uad6d \ud55c\uad6d\uc758 \uc778\ud130\ub137\uacfc \ub09c\ub9bd\ud55c \uc5f0\uc608 \uc800\ub110\ub9ac\uc998\uc740 \uadf8 \ubaa8\ub4e0 \uc0c1\ud669\uc744 \uac70\uc758 \uc2e4\uc2dc\uac04\uc73c\ub85c \uc911\uacc4\ud588\ub2e4\"\ub77c\uace0 \ud45c\ud604\ud588\ub2e4. \uae30\uc790 \uc8fc\uc9c4\uc6b0 \ub294 \"\ucd5c\uc9c4\uc2e4\uc740 \uadc0\uc5fd\uace0 \ubc1c\ub784\ud55c \uc5ec\uc131\uc758 \ub300\uba85\uc0ac\uc600\ub2e4. \uc774\ud63c \ub54c\ubb38\uc5d0 \uae54\ub054\ud55c \uc774\ubbf8\uc9c0\uac00 \uc190\uc0c1\ub418\uba74\uc11c \ucd5c\uc9c4\uc2e4\uc740 \ub098\ub77d\uc5d0 \ub5a8\uc5b4\uc84c\ub2e4. \uc5f0\uc608\uacc4\uc5d0\uc11c \uadf8\ub97c \ucc3e\ub294 \uacf3\uc740 \uc544\ubb34 \ub370\ub3c4 \uc5c6\uc5c8\ub2e4. 2004\ub144\uc5d0 MBC\uc758 \ud55c \uad6d\uc7a5\uc740 '\ucd5c\uc9c4\uc2e4\uc740 \ub05d\ub0ac\ub2e4. \uadc0\uc5fd\uace0 \ubc1c\ub784\ud568\uc73c\ub85c \ub5a0\uc624\ub974\ub358 \ucd5c\uc9c4\uc2e4\uc774 \uc774\uc81c\ub294 \uac00\uc815\ud3ed\ub825\uacfc \uc774\ud63c\uc758 \ub300\uba85\uc0ac\uac00 \ub418\uc5c8\ub294\ub370, \ucd5c\uc9c4\uc2e4\uc744 \uc5b4\ub514\uc5d0\ub2e4 \uc4f0\uaca0\ub294\uac00'\ub77c\uace0 \ub9d0\ud588\ub2e4\"\ub77c\uace0 \ub9d0\ud588\ub2e4. \uc774\ub294 \uac00\uc815\ud3ed\ub825\uc774\ub77c\ub294 \ubc94\uc8c4 \ud589\uc704\uc758 \ud53c\ud574\uc790\uc5d0\uac8c \uc624\ud788\ub824 \uadf8 \uc6b4\uc2e0\uc758 \ud3ed\uc744 \uc81c\ud55c\ud558\ub294 \ub300\ud55c\ubbfc\uad6d \ubc29\uc1a1\uacc4\uc758 \uc778\uc2dd \uc218\uc900\uc744 \ub2e8\uc801\uc73c\ub85c \ubcf4\uc5ec\uc8fc\ub294 \uac83\uc774\uc5c8\uace0, \uacb0\ud63c\uc744 \ud558\uba74 \uc774\ud63c\ud560 \uac00\ub2a5\uc131\uc740 \ub298 \uc788\ub294\ub370, \ud569\ubc95\uc801\uc778 \ud589\uc704\uc778 \uc774\ud63c\uc744 \ud588\ub2e4\ub294 \uc774\uc720\ub85c \uc0ac\ud68c\ud65c\ub3d9\uc744 \uc81c\ud55c\ubc1b\uc744 \uc218 \uc788\ub2e4\ub294 \uc0ac\ud68c \ubd84\uc704\uae30\ub97c \ub4dc\ub7ec\ub0b4\ub294 \uac83\uc774\uae30\ub3c4 \ud558\uc600\ub2e4. \ud3ed\ub825\ubc94\uc774 \uc720\uba85\uc778\uc744 \uc0c1\ub300\ub85c \ub300\ub193\uace0 \ud3ed\ub825 \ud589\uc704\ub97c \uc800\uc9c0\ub978 \uac83\uc740 \uc774\ub7f0 \uc0ac\ud68c \ubd84\uc704\uae30\uc640 \ubb34\uad00\ud558\uc9c0 \uc54a\uace0 \uc774\ub7f0 \uc0ac\ud68c \ubd84\uc704\uae30\ub294 \uc624\ud788\ub824 \uac00\uc815\ud3ed\ub825\uc744 \uc870\uc7a5\ud558\ub294 \uba74\uc774 \uc5c6\uc9c0 \uc54a\uc558\ub294\ub370, \ucd5c\uc9c4\uc2e4\uc740 \uc774\ud63c \uc219\ub824 \uae30\uac04\uc744 \uac00\uc9c0\ub294 \ub3d9\uc548\uc5d0 \uc54c\ub824\uc9c4 \uac83\ub9cc\uc73c\ub85c\ub3c4 \uac19\uc740 \uac00\ud574\uc790\ub85c\ubd80\ud130 \ub450 \ubc88\uc758 \uac00\uc815\ud3ed\ub825 \ud589\uc704\ub97c \ub2f9\ud588\uc5c8\uace0, \uccab \ubc88\uc9f8 \ud3ed\ub825 \ud589\uc704\ub294 \uc784\uc2e0 \uc911\uc5d0 \ub2f9\ud55c \uac83\uc774\uae30 \ub54c\ubb38\uc5d0 \ud0dc\uc544\ub3c4 \uadf8 \ud589\uc704\uc758 \ud53c\ud574\uc790\uc600\ub2e4. \uc774\ud63c \uacfc\uc815\uc5d0\uc11c \ucd5c\uc9c4\uc2e4\uc740 \ubbfc\uc0ac\uc0c1\u00b7\ud615\uc0ac\uc0c1 \ud53c\ud574\uc790\uc600\uc9c0\ub9cc, \uac1c\uc778\uc758 \uc544\ud508 \uc0ac\uc0dd\ud65c\uc5d0 \ub300\ud574\uc11c \uadf8\ub9ac\uace0 \uadf8 \ud53c\ud574\uc790\uc5d0 \ub300\ud574\uc11c \ud765\ubbf8 \uc704\uc8fc \uac00\uc2ed\uc73c\ub85c \ud314\uc544\uba39\ub294 \uc77c\ubd80 \uc5b8\ub860\uc758 '\uc0c1\ud63c' \ub4f1\uc73c\ub85c \uc774\uc911\uc73c\ub85c \uace0\ud1b5\uc744 \ubc1b\uc544\uc57c \ud588\ub2e4.", "paragraph_answer": "\ucd5c\uc9c4\uc2e4\uc758 \uc5b4\uba38\ub2c8 \uc815\uc625\uc219\uc740 \uc790\uc2e0\uc758 \uc800\uc11c \u300a\uc5c4\ub9c8\uac00, \ubbf8\uc548\ud574 \uadf8\ub9ac\uace0 \uc0ac\ub791\ud574\u300b(2011\ub144)\uc5d0\uc11c \"\uc5b8\ub860\uc740 \uc784\uc2e0\ud55c \uc0ac\ub78c\uc744, \uc544\uc774 \uc5b4\uba38\ub2c8\ub97c \ubc30\ub824\ud558\uc9c0 \uc54a\uc558\ub2e4\"\ub77c\uace0 \uc9c0\uc801\ud558\uba70 \uae30\uc790\ub4e4\uc758 \uc724\ub9ac \uc758\uc2dd\uc5d0 \uc758\uad6c\uc2ec\uc744 \ud45c\ud604\ud588\ub2e4. \ucd5c\uc9c4\uc2e4\uc740 \uc774\ud63c \uacfc\uc815\uc5d0 \uc788\uc744 \ub54c \uc784\uc2e0\ud55c \uc0c1\ud0dc\uc600\ub294\ub370, \uae30\uc790\ub4e4\uc774 \ucd5c\uc9c4\uc2e4\uc758 \uc9d1 \uc55e\uc5d0 \uc9c4\uc744 \uce58\uace0 \uc788\uc5b4\uc11c \ucd5c\uc9c4\uc2e4\uc740 \uc815\uae30 \uac80\uc9c4\uc744 \ubc1b\ub294 \ub0a0 \uc678\uc5d0\ub294 \uac70\uc758 \uc9d1\uc5d0 \uac07\ud600 \uc788\uc5b4\uc57c \ud588\uace0, \uc774\ud63c \uacfc\uc815\uc744 \uc0dd\uc911\uacc4\ud558\ub294 \ub4ef\ud55c \uc120\uc815\uc801\uc774\uace0 \ubb34\ubd84\ubcc4\ud55c \uae30\uc0ac\uc5d0 \uace0\ud1b5\uc744 \uacaa\uae30\ub3c4 \ud558\uc600\ub2e4. \ucd5c\uc9c4\uc2e4\uc740 2\ub144\uc5ec \ub3d9\uc548\uc758 \uc774\ud63c \uacfc\uc815\uc5d0\uc11c \ud5c8\uc704\uc0ac\uc2e4\uc774\ub098 \uc0ac\uc2e4 \uc65c\uace1\uc744 \ubc14\ub85c\uc7a1\uae30 \uc704\ud574 \ubc18\ubc15\uc131\u00b7\ud574\uba85\uc131 \uc778\ud130\ubdf0\ub97c \ud558\uac70\ub098 \ubc95\uc801 \uc870\uce58\ub97c \ucde8\ud558\uae30\ub3c4 \ud558\uc600\ub294\ub370, \uc774\ub294 \uc0ac\uc0dd\ud65c\uc774 \ub354 \uc774\uc0c1 \uc138\uc0c1\uc5d0 \uc54c\ub824\uc9c0\uace0 \uc65c\uace1\ub418\ub294 \uac83\uc744 \ub9c9\uaca0\ub2e4\ub294 \ucde8\uc9c0\uc600\ub2e4. \uadf8 \ubc95\uc801 \uc870\uce58\uc758 \ub300\uc0c1\uc5d0\ub294 \uc5b8\ub860\ub3c4 \ud3ec\ud568\ub418\uc5b4 \uc788\uc5c8\ub2e4. \uc778\ud130\ubdf0\ub97c \uc790\uccad\ud558\uc5ec \ud5c8\uc704\uc0ac\uc2e4\uc744 \uc720\ud3ec\ud55c \uc790\uac00 \uc788\uc5c8\uace0 \uadf8 \uc778\ud130\ubdf0\ub97c \uc9c4\ud589\ud55c \uae30\uc790\ub294 \uadf8 \uc720\ud3ec \ub0b4\uc6a9\uc744 \uadf8\ub300\ub85c \ubc1b\uc544\uc801\uc5b4 \uae30\uc0ac\ud654\ud558\uc600\ub294\ub370, \uc774\ub294 \uae30\uc790\uac00 \ud5c8\uc704\uc0ac\uc2e4\uc720\ud3ec \ud589\uc704\uc5d0 \ub3d9\ucc38\ud55c \uac83\uc774\uba74\uc11c \ud5c8\uc704\uc0ac\uc2e4 \uc720\ud3ec\uc790\uc640 \uae30\uc790 \ubaa8\ub450 \uc774\ud63c\uc740 \ud63c\uc778 \ub2f9\uc0ac\uc790\uc640 \uad6d\uac00\uac00 \ud574\uacb0\ubd10\uc57c \ud558\ub294 \uc0ac\uc548\uc774\ub77c\ub294 \uc0ac\uc2e4\uc744 \ub9dd\uac01\ud55c \ud589\uc704\uc774\uae30\ub3c4 \ud558\uc600\ub2e4. \uc774\ub7f0 \ud5c8\uc704\uc0ac\uc2e4\uc720\ud3ec \ud589\uc704\uc5d0 \ubc29\uc1a1\uc0ac\ub3c4 \ub3d9\ucc38\ud588\ub294\ub370, \uc774\ub97c \ud1b5\ud574 \ud5c8\uc704\uc0ac\uc2e4\uc740 \ud65c\uc790\uc640 \uc601\uc0c1\uc744 \ud1b5\ud574 \ub110\ub9ac \ud37c\uc838 \ub098\uac14\uace0, \uac70\uc758 \uc804 \uad6d\ubbfc\uc801\uc778 \uc778\uc9c0\ub3c4\ub97c \uac00\uc9c4 \ucd5c\uc9c4\uc2e4\uc774 \ubc1b\uc740 \ud0c0\uaca9\uc740 \uc0c1\ub2f9\ud788 \ucef8\uc73c\uba70, \ud6d7\ub0a0 \ucd5c\uc9c4\uc2e4\uc740 \uc774 \ub54c\uc758 \uc0c1\ud669\uc5d0 \ub300\ud574 \"\uad6d\ubbfc\uc744 '\ubc30\uc2ec\uc6d0'\uc73c\ub85c \uc549\ud600\ub193\uace0 2~3\ub144\uac04 \uc774\ud63c \ubb38\uc81c\ub85c \uc2dc\ub044\ub7fd\uac8c \ub5a0\ub4e4\uc5c8\ub2e4. \uadf8 \uacfc\uc815\uc5d0\uc11c \uce58\ubd80\ub97c \uc0dd\uc0dd\ud558\uac8c \ubcf4\uc5ec\uc918 \ub098\ub3c4 \uc544\uc774\ub4e4\ub3c4 \uce58\uba85\uc801\uc778 \uc0c1\ucc98\ub97c \uc785\uc5c8\ub2e4\"\ub77c\uace0 \ub9d0\ud558\uae30\ub3c4 \ud558\uc600\ub2e4. \uadf8 \ub2f9\uc2dc \uc5b8\ub860 \uc0c1\ud669\uc5d0 \ub300\ud574 \uc5f0\ucd9c\uac00 \uc815\uae38\ud654\ub294 \"IT \uac15\uad6d \ud55c\uad6d\uc758 \uc778\ud130\ub137\uacfc \ub09c\ub9bd\ud55c \uc5f0\uc608 \uc800\ub110\ub9ac\uc998\uc740 \uadf8 \ubaa8\ub4e0 \uc0c1\ud669\uc744 \uac70\uc758 \uc2e4\uc2dc\uac04\uc73c\ub85c \uc911\uacc4\ud588\ub2e4\"\ub77c\uace0 \ud45c\ud604\ud588\ub2e4. \uae30\uc790 \uc8fc\uc9c4\uc6b0 \ub294 \"\ucd5c\uc9c4\uc2e4\uc740 \uadc0\uc5fd\uace0 \ubc1c\ub784\ud55c \uc5ec\uc131\uc758 \ub300\uba85\uc0ac\uc600\ub2e4. \uc774\ud63c \ub54c\ubb38\uc5d0 \uae54\ub054\ud55c \uc774\ubbf8\uc9c0\uac00 \uc190\uc0c1\ub418\uba74\uc11c \ucd5c\uc9c4\uc2e4\uc740 \ub098\ub77d\uc5d0 \ub5a8\uc5b4\uc84c\ub2e4. \uc5f0\uc608\uacc4\uc5d0\uc11c \uadf8\ub97c \ucc3e\ub294 \uacf3\uc740 \uc544\ubb34 \ub370\ub3c4 \uc5c6\uc5c8\ub2e4. 2004\ub144\uc5d0 MBC\uc758 \ud55c \uad6d\uc7a5\uc740 '\ucd5c\uc9c4\uc2e4\uc740 \ub05d\ub0ac\ub2e4. \uadc0\uc5fd\uace0 \ubc1c\ub784\ud568\uc73c\ub85c \ub5a0\uc624\ub974\ub358 \ucd5c\uc9c4\uc2e4\uc774 \uc774\uc81c\ub294 \uac00\uc815\ud3ed\ub825\uacfc \uc774\ud63c\uc758 \ub300\uba85\uc0ac\uac00 \ub418\uc5c8\ub294\ub370, \ucd5c\uc9c4\uc2e4\uc744 \uc5b4\ub514\uc5d0\ub2e4 \uc4f0\uaca0\ub294\uac00'\ub77c\uace0 \ub9d0\ud588\ub2e4\"\ub77c\uace0 \ub9d0\ud588\ub2e4. \uc774\ub294 \uac00\uc815\ud3ed\ub825\uc774\ub77c\ub294 \ubc94\uc8c4 \ud589\uc704\uc758 \ud53c\ud574\uc790\uc5d0\uac8c \uc624\ud788\ub824 \uadf8 \uc6b4\uc2e0\uc758 \ud3ed\uc744 \uc81c\ud55c\ud558\ub294 \ub300\ud55c\ubbfc\uad6d \ubc29\uc1a1\uacc4\uc758 \uc778\uc2dd \uc218\uc900\uc744 \ub2e8\uc801\uc73c\ub85c \ubcf4\uc5ec\uc8fc\ub294 \uac83\uc774\uc5c8\uace0, \uacb0\ud63c\uc744 \ud558\uba74 \uc774\ud63c\ud560 \uac00\ub2a5\uc131\uc740 \ub298 \uc788\ub294\ub370, \ud569\ubc95\uc801\uc778 \ud589\uc704\uc778 \uc774\ud63c\uc744 \ud588\ub2e4\ub294 \uc774\uc720\ub85c \uc0ac\ud68c\ud65c\ub3d9\uc744 \uc81c\ud55c\ubc1b\uc744 \uc218 \uc788\ub2e4\ub294 \uc0ac\ud68c \ubd84\uc704\uae30\ub97c \ub4dc\ub7ec\ub0b4\ub294 \uac83\uc774\uae30\ub3c4 \ud558\uc600\ub2e4. \ud3ed\ub825\ubc94\uc774 \uc720\uba85\uc778\uc744 \uc0c1\ub300\ub85c \ub300\ub193\uace0 \ud3ed\ub825 \ud589\uc704\ub97c \uc800\uc9c0\ub978 \uac83\uc740 \uc774\ub7f0 \uc0ac\ud68c \ubd84\uc704\uae30\uc640 \ubb34\uad00\ud558\uc9c0 \uc54a\uace0 \uc774\ub7f0 \uc0ac\ud68c \ubd84\uc704\uae30\ub294 \uc624\ud788\ub824 \uac00\uc815\ud3ed\ub825\uc744 \uc870\uc7a5\ud558\ub294 \uba74\uc774 \uc5c6\uc9c0 \uc54a\uc558\ub294\ub370, \ucd5c\uc9c4\uc2e4\uc740 \uc774\ud63c \uc219\ub824 \uae30\uac04\uc744 \uac00\uc9c0\ub294 \ub3d9\uc548\uc5d0 \uc54c\ub824\uc9c4 \uac83\ub9cc\uc73c\ub85c\ub3c4 \uac19\uc740 \uac00\ud574\uc790\ub85c\ubd80\ud130 \ub450 \ubc88\uc758 \uac00\uc815\ud3ed\ub825 \ud589\uc704\ub97c \ub2f9\ud588\uc5c8\uace0, \uccab \ubc88\uc9f8 \ud3ed\ub825 \ud589\uc704\ub294 \uc784\uc2e0 \uc911\uc5d0 \ub2f9\ud55c \uac83\uc774\uae30 \ub54c\ubb38\uc5d0 \ud0dc\uc544\ub3c4 \uadf8 \ud589\uc704\uc758 \ud53c\ud574\uc790\uc600\ub2e4. \uc774\ud63c \uacfc\uc815\uc5d0\uc11c \ucd5c\uc9c4\uc2e4\uc740 \ubbfc\uc0ac\uc0c1\u00b7\ud615\uc0ac\uc0c1 \ud53c\ud574\uc790\uc600\uc9c0\ub9cc, \uac1c\uc778\uc758 \uc544\ud508 \uc0ac\uc0dd\ud65c\uc5d0 \ub300\ud574\uc11c \uadf8\ub9ac\uace0 \uadf8 \ud53c\ud574\uc790\uc5d0 \ub300\ud574\uc11c \ud765\ubbf8 \uc704\uc8fc \uac00\uc2ed\uc73c\ub85c \ud314\uc544\uba39\ub294 \uc77c\ubd80 \uc5b8\ub860\uc758 '\uc0c1\ud63c' \ub4f1\uc73c\ub85c \uc774\uc911\uc73c\ub85c \uace0\ud1b5\uc744 \ubc1b\uc544\uc57c \ud588\ub2e4.", "sentence_answer": "\uae30\uc790 \uc8fc\uc9c4\uc6b0 \ub294 \"\ucd5c\uc9c4\uc2e4\uc740 \uadc0\uc5fd\uace0 \ubc1c\ub784\ud55c \uc5ec\uc131\uc758 \ub300\uba85\uc0ac\uc600\ub2e4.", "paragraph_id": "6560156-11-1", "paragraph_question": "question: \ucd5c\uc9c4\uc2e4\uc758 \uc774\ud63c \ubb38\uc81c\ub85c \uae30\uc874 \uc774\ubbf8\uc9c0\ub97c \uc783\uc5b4, \ub354\uc774\uc0c1 \ucd5c\uc9c4\uc2e4\uc744\n \ucc3e\ub294 \uc0ac\ub78c\uc740 \uc5c6\uc744\uaebc\ub77c\uace0 \ud55c \uae30\uc790\ub294?, context: \ucd5c\uc9c4\uc2e4\uc758 \uc5b4\uba38\ub2c8 \uc815\uc625\uc219\uc740 \uc790\uc2e0\uc758 \uc800\uc11c \u300a\uc5c4\ub9c8\uac00, \ubbf8\uc548\ud574 \uadf8\ub9ac\uace0 \uc0ac\ub791\ud574\u300b(2011\ub144)\uc5d0\uc11c \"\uc5b8\ub860\uc740 \uc784\uc2e0\ud55c \uc0ac\ub78c\uc744, \uc544\uc774 \uc5b4\uba38\ub2c8\ub97c \ubc30\ub824\ud558\uc9c0 \uc54a\uc558\ub2e4\"\ub77c\uace0 \uc9c0\uc801\ud558\uba70 \uae30\uc790\ub4e4\uc758 \uc724\ub9ac \uc758\uc2dd\uc5d0 \uc758\uad6c\uc2ec\uc744 \ud45c\ud604\ud588\ub2e4. \ucd5c\uc9c4\uc2e4\uc740 \uc774\ud63c \uacfc\uc815\uc5d0 \uc788\uc744 \ub54c \uc784\uc2e0\ud55c \uc0c1\ud0dc\uc600\ub294\ub370, \uae30\uc790\ub4e4\uc774 \ucd5c\uc9c4\uc2e4\uc758 \uc9d1 \uc55e\uc5d0 \uc9c4\uc744 \uce58\uace0 \uc788\uc5b4\uc11c \ucd5c\uc9c4\uc2e4\uc740 \uc815\uae30 \uac80\uc9c4\uc744 \ubc1b\ub294 \ub0a0 \uc678\uc5d0\ub294 \uac70\uc758 \uc9d1\uc5d0 \uac07\ud600 \uc788\uc5b4\uc57c \ud588\uace0, \uc774\ud63c \uacfc\uc815\uc744 \uc0dd\uc911\uacc4\ud558\ub294 \ub4ef\ud55c \uc120\uc815\uc801\uc774\uace0 \ubb34\ubd84\ubcc4\ud55c \uae30\uc0ac\uc5d0 \uace0\ud1b5\uc744 \uacaa\uae30\ub3c4 \ud558\uc600\ub2e4. \ucd5c\uc9c4\uc2e4\uc740 2\ub144\uc5ec \ub3d9\uc548\uc758 \uc774\ud63c \uacfc\uc815\uc5d0\uc11c \ud5c8\uc704\uc0ac\uc2e4\uc774\ub098 \uc0ac\uc2e4 \uc65c\uace1\uc744 \ubc14\ub85c\uc7a1\uae30 \uc704\ud574 \ubc18\ubc15\uc131\u00b7\ud574\uba85\uc131 \uc778\ud130\ubdf0\ub97c \ud558\uac70\ub098 \ubc95\uc801 \uc870\uce58\ub97c \ucde8\ud558\uae30\ub3c4 \ud558\uc600\ub294\ub370, \uc774\ub294 \uc0ac\uc0dd\ud65c\uc774 \ub354 \uc774\uc0c1 \uc138\uc0c1\uc5d0 \uc54c\ub824\uc9c0\uace0 \uc65c\uace1\ub418\ub294 \uac83\uc744 \ub9c9\uaca0\ub2e4\ub294 \ucde8\uc9c0\uc600\ub2e4. \uadf8 \ubc95\uc801 \uc870\uce58\uc758 \ub300\uc0c1\uc5d0\ub294 \uc5b8\ub860\ub3c4 \ud3ec\ud568\ub418\uc5b4 \uc788\uc5c8\ub2e4. \uc778\ud130\ubdf0\ub97c \uc790\uccad\ud558\uc5ec \ud5c8\uc704\uc0ac\uc2e4\uc744 \uc720\ud3ec\ud55c \uc790\uac00 \uc788\uc5c8\uace0 \uadf8 \uc778\ud130\ubdf0\ub97c \uc9c4\ud589\ud55c \uae30\uc790\ub294 \uadf8 \uc720\ud3ec \ub0b4\uc6a9\uc744 \uadf8\ub300\ub85c \ubc1b\uc544\uc801\uc5b4 \uae30\uc0ac\ud654\ud558\uc600\ub294\ub370, \uc774\ub294 \uae30\uc790\uac00 \ud5c8\uc704\uc0ac\uc2e4\uc720\ud3ec \ud589\uc704\uc5d0 \ub3d9\ucc38\ud55c \uac83\uc774\uba74\uc11c \ud5c8\uc704\uc0ac\uc2e4 \uc720\ud3ec\uc790\uc640 \uae30\uc790 \ubaa8\ub450 \uc774\ud63c\uc740 \ud63c\uc778 \ub2f9\uc0ac\uc790\uc640 \uad6d\uac00\uac00 \ud574\uacb0\ubd10\uc57c \ud558\ub294 \uc0ac\uc548\uc774\ub77c\ub294 \uc0ac\uc2e4\uc744 \ub9dd\uac01\ud55c \ud589\uc704\uc774\uae30\ub3c4 \ud558\uc600\ub2e4. \uc774\ub7f0 \ud5c8\uc704\uc0ac\uc2e4\uc720\ud3ec \ud589\uc704\uc5d0 \ubc29\uc1a1\uc0ac\ub3c4 \ub3d9\ucc38\ud588\ub294\ub370, \uc774\ub97c \ud1b5\ud574 \ud5c8\uc704\uc0ac\uc2e4\uc740 \ud65c\uc790\uc640 \uc601\uc0c1\uc744 \ud1b5\ud574 \ub110\ub9ac \ud37c\uc838 \ub098\uac14\uace0, \uac70\uc758 \uc804 \uad6d\ubbfc\uc801\uc778 \uc778\uc9c0\ub3c4\ub97c \uac00\uc9c4 \ucd5c\uc9c4\uc2e4\uc774 \ubc1b\uc740 \ud0c0\uaca9\uc740 \uc0c1\ub2f9\ud788 \ucef8\uc73c\uba70, \ud6d7\ub0a0 \ucd5c\uc9c4\uc2e4\uc740 \uc774 \ub54c\uc758 \uc0c1\ud669\uc5d0 \ub300\ud574 \"\uad6d\ubbfc\uc744 '\ubc30\uc2ec\uc6d0'\uc73c\ub85c \uc549\ud600\ub193\uace0 2~3\ub144\uac04 \uc774\ud63c \ubb38\uc81c\ub85c \uc2dc\ub044\ub7fd\uac8c \ub5a0\ub4e4\uc5c8\ub2e4. \uadf8 \uacfc\uc815\uc5d0\uc11c \uce58\ubd80\ub97c \uc0dd\uc0dd\ud558\uac8c \ubcf4\uc5ec\uc918 \ub098\ub3c4 \uc544\uc774\ub4e4\ub3c4 \uce58\uba85\uc801\uc778 \uc0c1\ucc98\ub97c \uc785\uc5c8\ub2e4\"\ub77c\uace0 \ub9d0\ud558\uae30\ub3c4 \ud558\uc600\ub2e4. \uadf8 \ub2f9\uc2dc \uc5b8\ub860 \uc0c1\ud669\uc5d0 \ub300\ud574 \uc5f0\ucd9c\uac00 \uc815\uae38\ud654\ub294 \"IT \uac15\uad6d \ud55c\uad6d\uc758 \uc778\ud130\ub137\uacfc \ub09c\ub9bd\ud55c \uc5f0\uc608 \uc800\ub110\ub9ac\uc998\uc740 \uadf8 \ubaa8\ub4e0 \uc0c1\ud669\uc744 \uac70\uc758 \uc2e4\uc2dc\uac04\uc73c\ub85c \uc911\uacc4\ud588\ub2e4\"\ub77c\uace0 \ud45c\ud604\ud588\ub2e4. \uae30\uc790 \uc8fc\uc9c4\uc6b0\ub294 \"\ucd5c\uc9c4\uc2e4\uc740 \uadc0\uc5fd\uace0 \ubc1c\ub784\ud55c \uc5ec\uc131\uc758 \ub300\uba85\uc0ac\uc600\ub2e4. \uc774\ud63c \ub54c\ubb38\uc5d0 \uae54\ub054\ud55c \uc774\ubbf8\uc9c0\uac00 \uc190\uc0c1\ub418\uba74\uc11c \ucd5c\uc9c4\uc2e4\uc740 \ub098\ub77d\uc5d0 \ub5a8\uc5b4\uc84c\ub2e4. \uc5f0\uc608\uacc4\uc5d0\uc11c \uadf8\ub97c \ucc3e\ub294 \uacf3\uc740 \uc544\ubb34 \ub370\ub3c4 \uc5c6\uc5c8\ub2e4. 2004\ub144\uc5d0 MBC\uc758 \ud55c \uad6d\uc7a5\uc740 '\ucd5c\uc9c4\uc2e4\uc740 \ub05d\ub0ac\ub2e4. \uadc0\uc5fd\uace0 \ubc1c\ub784\ud568\uc73c\ub85c \ub5a0\uc624\ub974\ub358 \ucd5c\uc9c4\uc2e4\uc774 \uc774\uc81c\ub294 \uac00\uc815\ud3ed\ub825\uacfc \uc774\ud63c\uc758 \ub300\uba85\uc0ac\uac00 \ub418\uc5c8\ub294\ub370, \ucd5c\uc9c4\uc2e4\uc744 \uc5b4\ub514\uc5d0\ub2e4 \uc4f0\uaca0\ub294\uac00'\ub77c\uace0 \ub9d0\ud588\ub2e4\"\ub77c\uace0 \ub9d0\ud588\ub2e4. \uc774\ub294 \uac00\uc815\ud3ed\ub825\uc774\ub77c\ub294 \ubc94\uc8c4 \ud589\uc704\uc758 \ud53c\ud574\uc790\uc5d0\uac8c \uc624\ud788\ub824 \uadf8 \uc6b4\uc2e0\uc758 \ud3ed\uc744 \uc81c\ud55c\ud558\ub294 \ub300\ud55c\ubbfc\uad6d \ubc29\uc1a1\uacc4\uc758 \uc778\uc2dd \uc218\uc900\uc744 \ub2e8\uc801\uc73c\ub85c \ubcf4\uc5ec\uc8fc\ub294 \uac83\uc774\uc5c8\uace0, \uacb0\ud63c\uc744 \ud558\uba74 \uc774\ud63c\ud560 \uac00\ub2a5\uc131\uc740 \ub298 \uc788\ub294\ub370, \ud569\ubc95\uc801\uc778 \ud589\uc704\uc778 \uc774\ud63c\uc744 \ud588\ub2e4\ub294 \uc774\uc720\ub85c \uc0ac\ud68c\ud65c\ub3d9\uc744 \uc81c\ud55c\ubc1b\uc744 \uc218 \uc788\ub2e4\ub294 \uc0ac\ud68c \ubd84\uc704\uae30\ub97c \ub4dc\ub7ec\ub0b4\ub294 \uac83\uc774\uae30\ub3c4 \ud558\uc600\ub2e4. \ud3ed\ub825\ubc94\uc774 \uc720\uba85\uc778\uc744 \uc0c1\ub300\ub85c \ub300\ub193\uace0 \ud3ed\ub825 \ud589\uc704\ub97c \uc800\uc9c0\ub978 \uac83\uc740 \uc774\ub7f0 \uc0ac\ud68c \ubd84\uc704\uae30\uc640 \ubb34\uad00\ud558\uc9c0 \uc54a\uace0 \uc774\ub7f0 \uc0ac\ud68c \ubd84\uc704\uae30\ub294 \uc624\ud788\ub824 \uac00\uc815\ud3ed\ub825\uc744 \uc870\uc7a5\ud558\ub294 \uba74\uc774 \uc5c6\uc9c0 \uc54a\uc558\ub294\ub370, \ucd5c\uc9c4\uc2e4\uc740 \uc774\ud63c \uc219\ub824 \uae30\uac04\uc744 \uac00\uc9c0\ub294 \ub3d9\uc548\uc5d0 \uc54c\ub824\uc9c4 \uac83\ub9cc\uc73c\ub85c\ub3c4 \uac19\uc740 \uac00\ud574\uc790\ub85c\ubd80\ud130 \ub450 \ubc88\uc758 \uac00\uc815\ud3ed\ub825 \ud589\uc704\ub97c \ub2f9\ud588\uc5c8\uace0, \uccab \ubc88\uc9f8 \ud3ed\ub825 \ud589\uc704\ub294 \uc784\uc2e0 \uc911\uc5d0 \ub2f9\ud55c \uac83\uc774\uae30 \ub54c\ubb38\uc5d0 \ud0dc\uc544\ub3c4 \uadf8 \ud589\uc704\uc758 \ud53c\ud574\uc790\uc600\ub2e4. \uc774\ud63c \uacfc\uc815\uc5d0\uc11c \ucd5c\uc9c4\uc2e4\uc740 \ubbfc\uc0ac\uc0c1\u00b7\ud615\uc0ac\uc0c1 \ud53c\ud574\uc790\uc600\uc9c0\ub9cc, \uac1c\uc778\uc758 \uc544\ud508 \uc0ac\uc0dd\ud65c\uc5d0 \ub300\ud574\uc11c \uadf8\ub9ac\uace0 \uadf8 \ud53c\ud574\uc790\uc5d0 \ub300\ud574\uc11c \ud765\ubbf8 \uc704\uc8fc \uac00\uc2ed\uc73c\ub85c \ud314\uc544\uba39\ub294 \uc77c\ubd80 \uc5b8\ub860\uc758 '\uc0c1\ud63c' \ub4f1\uc73c\ub85c \uc774\uc911\uc73c\ub85c \uace0\ud1b5\uc744 \ubc1b\uc544\uc57c \ud588\ub2e4."} {"question": "\uc790\uc720 \ud751\uc778\ub4e4\uc774 \ubaa8\uc778 \uc0c8\ub85c\uc6b4 \uc8fc\ub97c \ub9cc\ub4e4\uae30 \uc704\ud574 \uad70\uc0ac\uc801 \ud589\ub3d9\uc744 \uac1c\uc2dc\ud55c \uc774\ub294?", "paragraph": "\uace7 \uadf8\ub294 \ub178\uc608 \uc18c\uc720\uc790\ub4e4\uc5d0 \ub300\ud55c \uacf5\uaca9\uc744 \uac10\ud589\ud558\uae30 \uc704\ud574 \uc9c0\uc9c0\uc790\ub97c \ubaa8\uc558\uace0, \uc5ec\uae30\uc5d0\ub294 \ud130\ube0c\uba3c\ub3c4 \ucc38\uc5ec\ud588\ub2e4. \ub2f9\uc2dc \ud130\ube0c\uba3c\uc740 \u201c\ud130\ube0c\uba3c \uc7a5\uad70\u201d (\uc874 \ube0c\ub77c\uc6b4\uc774 \uadf8\uc5d0\uac8c \ubd99\uc778 \ubcc4\uba85)\uc774\ub77c\ub294 \ubcc4\uba85\uc73c\ub85c \ubd88\ub9ac\uace0 \uc788\uc5c8\ub2e4. \uadf8\uc758 \uc9c0\uc9c0 \uc138\ub825\ub4e4\uc5d0 \uad00\ud55c \uc815\ubcf4\uc640 \ud39c\uc2e4\ubca0\uc774\ub2c8\uc544, \uba54\ub9b4\ub79c\ub4dc, \ub378\ub77c\uc6e8\uc5b4\uc758 \uc8fc \uacbd\uacc4\uc758 \uc790\uc6d0\uc5d0 \ub300\ud55c \uc815\ubcf4\ub294 \ube0c\ub77c\uc6b4\uc774 \uacc4\ud68d\uc744 \uc9dc\ub294\ub370 \uc0c1\ub2f9\ud55c \ub3c4\uc6c0\uc774 \ub418\uc5c8\ub2e4. \ud504\ub808\ub354\ub9ad \ub354\uae00\ub7ec\uc2a4\uc640 \uc70c\ub9ac\uc5c4 \ub85c\uc774\ub4dc \uac1c\ub9ac\uc2a8 \ub4f1\uc758 \ub2e4\ub978 \ub178\uc608 \ud3d0\uc9c0\ub860\uc790\ub4e4\uc740 \uadf8\uc758 \uacc4\ud68d\uc744 \ub2ec\uac00\uc6cc\ud558\uc9c0 \uc54a\uc558\uc73c\ub098, \ube0c\ub77c\uc6b4\uc740 \uc790\uc720 \ud751\uc778\ub4e4\uc774 \ubaa8\uc778 \uc0c8\ub85c\uc6b4 \uc8fc\ub97c \ub9cc\ub4dc\ub294 \uafc8\uc744 \uac16\uace0 \uad70\uc0ac\uc801\uc778 \ud589\ub3d9 \uac1c\uc2dc\ub97c \uc704\ud55c \uc900\ube44\ub97c \ucc28\uadfc\ucc28\uadfc \ud574\ub098\uac14\ub2e4. \uadf8\ub294 \uc77c\ub2e8 \uc804\ud22c\ub97c \ubc8c\uc774\uae30 \uc2dc\uc791\ud558\uba74 \ub2e4\ub978 \ub178\uc608\ub4e4\ub3c4 \uc774\uc5d0 \ud638\uc751\ud558\uc5ec \uc790\uc5f0\uc2a4\ub7fd\uac8c \ub0a8\ucabd\uc73c\ub85c \ubc18\ub780\uc774 \ud655\uc7a5\ub418\uc5b4 \ub098\uac08 \uac83\uc774\ub77c\uace0 \ubbff\uc5c8\ub2e4. \ub2f9\uc2dc \uce90\ub098\ub2e4\uc5d0 \uc0b4\uace0 \uc788\ub358 \uc790\uc720 \ud751\uc778\ub4e4 \uc911 \uadf8\uc758 \uad70\ub300\uc5d0 \ucc38\uc5ec\ud560 \uc774\uac00 \uc5c6\ub294\uc9c0 \ucc3e\uc544\ubd10\ub2ec\ub77c\ub294 \ube0c\ub77c\uc6b4\uc758 \uc694\uccad\uc5d0 \ub530\ub77c \uadf8\ub294 \uce90\ub098\ub2e4 \uc9c0\uc5ed\uc5d0\uc11c \uc9c0\uc6d0\uc790\ub97c \ucc3e\uc558\ub2e4.", "answer": "\uc874 \ube0c\ub77c\uc6b4", "sentence": "\ub2f9\uc2dc \ud130\ube0c\uba3c\uc740 \u201c\ud130\ube0c\uba3c \uc7a5\uad70\u201d ( \uc874 \ube0c\ub77c\uc6b4 \uc774 \uadf8\uc5d0\uac8c \ubd99\uc778 \ubcc4\uba85)", "paragraph_sentence": "\uace7 \uadf8\ub294 \ub178\uc608 \uc18c\uc720\uc790\ub4e4\uc5d0 \ub300\ud55c \uacf5\uaca9\uc744 \uac10\ud589\ud558\uae30 \uc704\ud574 \uc9c0\uc9c0\uc790\ub97c \ubaa8\uc558\uace0, \uc5ec\uae30\uc5d0\ub294 \ud130\ube0c\uba3c\ub3c4 \ucc38\uc5ec\ud588\ub2e4. \ub2f9\uc2dc \ud130\ube0c\uba3c\uc740 \u201c\ud130\ube0c\uba3c \uc7a5\uad70\u201d ( \uc874 \ube0c\ub77c\uc6b4 \uc774 \uadf8\uc5d0\uac8c \ubd99\uc778 \ubcc4\uba85) \uc774\ub77c\ub294 \ubcc4\uba85\uc73c\ub85c \ubd88\ub9ac\uace0 \uc788\uc5c8\ub2e4. \uadf8\uc758 \uc9c0\uc9c0 \uc138\ub825\ub4e4\uc5d0 \uad00\ud55c \uc815\ubcf4\uc640 \ud39c\uc2e4\ubca0\uc774\ub2c8\uc544, \uba54\ub9b4\ub79c\ub4dc, \ub378\ub77c\uc6e8\uc5b4\uc758 \uc8fc \uacbd\uacc4\uc758 \uc790\uc6d0\uc5d0 \ub300\ud55c \uc815\ubcf4\ub294 \ube0c\ub77c\uc6b4\uc774 \uacc4\ud68d\uc744 \uc9dc\ub294\ub370 \uc0c1\ub2f9\ud55c \ub3c4\uc6c0\uc774 \ub418\uc5c8\ub2e4. \ud504\ub808\ub354\ub9ad \ub354\uae00\ub7ec\uc2a4\uc640 \uc70c\ub9ac\uc5c4 \ub85c\uc774\ub4dc \uac1c\ub9ac\uc2a8 \ub4f1\uc758 \ub2e4\ub978 \ub178\uc608 \ud3d0\uc9c0\ub860\uc790\ub4e4\uc740 \uadf8\uc758 \uacc4\ud68d\uc744 \ub2ec\uac00\uc6cc\ud558\uc9c0 \uc54a\uc558\uc73c\ub098, \ube0c\ub77c\uc6b4\uc740 \uc790\uc720 \ud751\uc778\ub4e4\uc774 \ubaa8\uc778 \uc0c8\ub85c\uc6b4 \uc8fc\ub97c \ub9cc\ub4dc\ub294 \uafc8\uc744 \uac16\uace0 \uad70\uc0ac\uc801\uc778 \ud589\ub3d9 \uac1c\uc2dc\ub97c \uc704\ud55c \uc900\ube44\ub97c \ucc28\uadfc\ucc28\uadfc \ud574\ub098\uac14\ub2e4. \uadf8\ub294 \uc77c\ub2e8 \uc804\ud22c\ub97c \ubc8c\uc774\uae30 \uc2dc\uc791\ud558\uba74 \ub2e4\ub978 \ub178\uc608\ub4e4\ub3c4 \uc774\uc5d0 \ud638\uc751\ud558\uc5ec \uc790\uc5f0\uc2a4\ub7fd\uac8c \ub0a8\ucabd\uc73c\ub85c \ubc18\ub780\uc774 \ud655\uc7a5\ub418\uc5b4 \ub098\uac08 \uac83\uc774\ub77c\uace0 \ubbff\uc5c8\ub2e4. \ub2f9\uc2dc \uce90\ub098\ub2e4\uc5d0 \uc0b4\uace0 \uc788\ub358 \uc790\uc720 \ud751\uc778\ub4e4 \uc911 \uadf8\uc758 \uad70\ub300\uc5d0 \ucc38\uc5ec\ud560 \uc774\uac00 \uc5c6\ub294\uc9c0 \ucc3e\uc544\ubd10\ub2ec\ub77c\ub294 \ube0c\ub77c\uc6b4\uc758 \uc694\uccad\uc5d0 \ub530\ub77c \uadf8\ub294 \uce90\ub098\ub2e4 \uc9c0\uc5ed\uc5d0\uc11c \uc9c0\uc6d0\uc790\ub97c \ucc3e\uc558\ub2e4.", "paragraph_answer": "\uace7 \uadf8\ub294 \ub178\uc608 \uc18c\uc720\uc790\ub4e4\uc5d0 \ub300\ud55c \uacf5\uaca9\uc744 \uac10\ud589\ud558\uae30 \uc704\ud574 \uc9c0\uc9c0\uc790\ub97c \ubaa8\uc558\uace0, \uc5ec\uae30\uc5d0\ub294 \ud130\ube0c\uba3c\ub3c4 \ucc38\uc5ec\ud588\ub2e4. \ub2f9\uc2dc \ud130\ube0c\uba3c\uc740 \u201c\ud130\ube0c\uba3c \uc7a5\uad70\u201d ( \uc874 \ube0c\ub77c\uc6b4 \uc774 \uadf8\uc5d0\uac8c \ubd99\uc778 \ubcc4\uba85)\uc774\ub77c\ub294 \ubcc4\uba85\uc73c\ub85c \ubd88\ub9ac\uace0 \uc788\uc5c8\ub2e4. \uadf8\uc758 \uc9c0\uc9c0 \uc138\ub825\ub4e4\uc5d0 \uad00\ud55c \uc815\ubcf4\uc640 \ud39c\uc2e4\ubca0\uc774\ub2c8\uc544, \uba54\ub9b4\ub79c\ub4dc, \ub378\ub77c\uc6e8\uc5b4\uc758 \uc8fc \uacbd\uacc4\uc758 \uc790\uc6d0\uc5d0 \ub300\ud55c \uc815\ubcf4\ub294 \ube0c\ub77c\uc6b4\uc774 \uacc4\ud68d\uc744 \uc9dc\ub294\ub370 \uc0c1\ub2f9\ud55c \ub3c4\uc6c0\uc774 \ub418\uc5c8\ub2e4. \ud504\ub808\ub354\ub9ad \ub354\uae00\ub7ec\uc2a4\uc640 \uc70c\ub9ac\uc5c4 \ub85c\uc774\ub4dc \uac1c\ub9ac\uc2a8 \ub4f1\uc758 \ub2e4\ub978 \ub178\uc608 \ud3d0\uc9c0\ub860\uc790\ub4e4\uc740 \uadf8\uc758 \uacc4\ud68d\uc744 \ub2ec\uac00\uc6cc\ud558\uc9c0 \uc54a\uc558\uc73c\ub098, \ube0c\ub77c\uc6b4\uc740 \uc790\uc720 \ud751\uc778\ub4e4\uc774 \ubaa8\uc778 \uc0c8\ub85c\uc6b4 \uc8fc\ub97c \ub9cc\ub4dc\ub294 \uafc8\uc744 \uac16\uace0 \uad70\uc0ac\uc801\uc778 \ud589\ub3d9 \uac1c\uc2dc\ub97c \uc704\ud55c \uc900\ube44\ub97c \ucc28\uadfc\ucc28\uadfc \ud574\ub098\uac14\ub2e4. \uadf8\ub294 \uc77c\ub2e8 \uc804\ud22c\ub97c \ubc8c\uc774\uae30 \uc2dc\uc791\ud558\uba74 \ub2e4\ub978 \ub178\uc608\ub4e4\ub3c4 \uc774\uc5d0 \ud638\uc751\ud558\uc5ec \uc790\uc5f0\uc2a4\ub7fd\uac8c \ub0a8\ucabd\uc73c\ub85c \ubc18\ub780\uc774 \ud655\uc7a5\ub418\uc5b4 \ub098\uac08 \uac83\uc774\ub77c\uace0 \ubbff\uc5c8\ub2e4. \ub2f9\uc2dc \uce90\ub098\ub2e4\uc5d0 \uc0b4\uace0 \uc788\ub358 \uc790\uc720 \ud751\uc778\ub4e4 \uc911 \uadf8\uc758 \uad70\ub300\uc5d0 \ucc38\uc5ec\ud560 \uc774\uac00 \uc5c6\ub294\uc9c0 \ucc3e\uc544\ubd10\ub2ec\ub77c\ub294 \ube0c\ub77c\uc6b4\uc758 \uc694\uccad\uc5d0 \ub530\ub77c \uadf8\ub294 \uce90\ub098\ub2e4 \uc9c0\uc5ed\uc5d0\uc11c \uc9c0\uc6d0\uc790\ub97c \ucc3e\uc558\ub2e4.", "sentence_answer": "\ub2f9\uc2dc \ud130\ube0c\uba3c\uc740 \u201c\ud130\ube0c\uba3c \uc7a5\uad70\u201d ( \uc874 \ube0c\ub77c\uc6b4 \uc774 \uadf8\uc5d0\uac8c \ubd99\uc778 \ubcc4\uba85)", "paragraph_id": "6570493-19-1", "paragraph_question": "question: \uc790\uc720 \ud751\uc778\ub4e4\uc774 \ubaa8\uc778 \uc0c8\ub85c\uc6b4 \uc8fc\ub97c \ub9cc\ub4e4\uae30 \uc704\ud574 \uad70\uc0ac\uc801 \ud589\ub3d9\uc744 \uac1c\uc2dc\ud55c \uc774\ub294?, context: \uace7 \uadf8\ub294 \ub178\uc608 \uc18c\uc720\uc790\ub4e4\uc5d0 \ub300\ud55c \uacf5\uaca9\uc744 \uac10\ud589\ud558\uae30 \uc704\ud574 \uc9c0\uc9c0\uc790\ub97c \ubaa8\uc558\uace0, \uc5ec\uae30\uc5d0\ub294 \ud130\ube0c\uba3c\ub3c4 \ucc38\uc5ec\ud588\ub2e4. \ub2f9\uc2dc \ud130\ube0c\uba3c\uc740 \u201c\ud130\ube0c\uba3c \uc7a5\uad70\u201d (\uc874 \ube0c\ub77c\uc6b4\uc774 \uadf8\uc5d0\uac8c \ubd99\uc778 \ubcc4\uba85)\uc774\ub77c\ub294 \ubcc4\uba85\uc73c\ub85c \ubd88\ub9ac\uace0 \uc788\uc5c8\ub2e4. \uadf8\uc758 \uc9c0\uc9c0 \uc138\ub825\ub4e4\uc5d0 \uad00\ud55c \uc815\ubcf4\uc640 \ud39c\uc2e4\ubca0\uc774\ub2c8\uc544, \uba54\ub9b4\ub79c\ub4dc, \ub378\ub77c\uc6e8\uc5b4\uc758 \uc8fc \uacbd\uacc4\uc758 \uc790\uc6d0\uc5d0 \ub300\ud55c \uc815\ubcf4\ub294 \ube0c\ub77c\uc6b4\uc774 \uacc4\ud68d\uc744 \uc9dc\ub294\ub370 \uc0c1\ub2f9\ud55c \ub3c4\uc6c0\uc774 \ub418\uc5c8\ub2e4. \ud504\ub808\ub354\ub9ad \ub354\uae00\ub7ec\uc2a4\uc640 \uc70c\ub9ac\uc5c4 \ub85c\uc774\ub4dc \uac1c\ub9ac\uc2a8 \ub4f1\uc758 \ub2e4\ub978 \ub178\uc608 \ud3d0\uc9c0\ub860\uc790\ub4e4\uc740 \uadf8\uc758 \uacc4\ud68d\uc744 \ub2ec\uac00\uc6cc\ud558\uc9c0 \uc54a\uc558\uc73c\ub098, \ube0c\ub77c\uc6b4\uc740 \uc790\uc720 \ud751\uc778\ub4e4\uc774 \ubaa8\uc778 \uc0c8\ub85c\uc6b4 \uc8fc\ub97c \ub9cc\ub4dc\ub294 \uafc8\uc744 \uac16\uace0 \uad70\uc0ac\uc801\uc778 \ud589\ub3d9 \uac1c\uc2dc\ub97c \uc704\ud55c \uc900\ube44\ub97c \ucc28\uadfc\ucc28\uadfc \ud574\ub098\uac14\ub2e4. \uadf8\ub294 \uc77c\ub2e8 \uc804\ud22c\ub97c \ubc8c\uc774\uae30 \uc2dc\uc791\ud558\uba74 \ub2e4\ub978 \ub178\uc608\ub4e4\ub3c4 \uc774\uc5d0 \ud638\uc751\ud558\uc5ec \uc790\uc5f0\uc2a4\ub7fd\uac8c \ub0a8\ucabd\uc73c\ub85c \ubc18\ub780\uc774 \ud655\uc7a5\ub418\uc5b4 \ub098\uac08 \uac83\uc774\ub77c\uace0 \ubbff\uc5c8\ub2e4. \ub2f9\uc2dc \uce90\ub098\ub2e4\uc5d0 \uc0b4\uace0 \uc788\ub358 \uc790\uc720 \ud751\uc778\ub4e4 \uc911 \uadf8\uc758 \uad70\ub300\uc5d0 \ucc38\uc5ec\ud560 \uc774\uac00 \uc5c6\ub294\uc9c0 \ucc3e\uc544\ubd10\ub2ec\ub77c\ub294 \ube0c\ub77c\uc6b4\uc758 \uc694\uccad\uc5d0 \ub530\ub77c \uadf8\ub294 \uce90\ub098\ub2e4 \uc9c0\uc5ed\uc5d0\uc11c \uc9c0\uc6d0\uc790\ub97c \ucc3e\uc558\ub2e4."} {"question": "\ub77c\uc2a8 \ud638\uac00 \ub871\ube44\uce58\uc5d0 \ub3c4\ucc29\ud55c \ub0a0\uc740?", "paragraph": "3\uc6d4 24\uc77c\uc5d0 \uc0ac\uc138\ubcf4\ub97c \ucd9c\ubc1c\ud55c \ub77c\uc2a8 \ud638\ub294 4\uc6d4 6\uc77c\uc5d0 \ub871\ube44\uce58\uc5d0 \ub3c4\ucc29\ud588\ub2e4. \ub77c\uc2a8 \ud638\uac00 \ub871\ube44\uce58\uc5d0 \ub3c4\ucc29\ud55c 4\uc6d4 6\uc77c\uc740 \ucde8\uc5ed\ud55c \uc9c0 23\uc8fc\ub144\uc774 \ub418\ub294 \ub0a0\uc774\uc5c8\ub2e4. 4\uc6d4 6\uc77c \uc774\ud6c4 \ub77c\uc2a8\uc740 \ud1f4\uc5ed \ub54c\uae4c\uc9c0 \ub300\ubd80\ubd84\uc758 \uc2dc\uac04\uc744 \ud56d\uad6c\uc5d0 \uc815\ubc15\ud55c \ucc44 \ubcf4\ub0b4\uac70\ub098 \uce98\ub9ac\ud3ec\ub2c8\uc544 \ub0a8\ucabd \uc791\uc804 \uad6c\uc5ed\uc5d0\uc11c \ub2e4\ub978 \ud568\uc815\ub4e4\uacfc \ud569\ub3d9 \ud6c8\ub828\uc744 \uc9c0\uc6d0\ud558\uba70 \uc9c0\ub0c8\ub2e4. \ub77c\uc2a8 \ud638\uc758 \uac00\uc7a5 \uc911\uc694\ud55c \uc784\ubb34\ub294 1\ub300\uc7a0\uc218\ud568\uadf8\ub8f9 \uc18c\uc18d\uc73c\ub85c\uc11c \ub300\uc7a0\uc218\ud568\uc804 \ud6c8\ub828\uc744 \ud558\ub294 \uac83\uc774\uc5c8\ub2e4. \uc774 \uadf8\ub8f9\uc5d0\ub294 CVS-33 USS \ud0a4\uc0c8\uc9c0, DD-723 USS \uc6d4\ud0a4, DD-754 USS \ud504\ub7ad\ud06c E. \uc5d0\ubc88\uc2a4, DD-787 USS \uc81c\uc784\uc2a4 E. \uce74\uc774, DD-830 USS \uc5d0\ubc84\ub9bf F. \ub77c\uc2a8, DEG-3 USS \uc1fc\ud544\ub4dc, DE-1037 USS \ube0c\ub860\uc288\ud0c0\uc778 \ub4f1\uc73c\ub85c \uad6c\uc131\ub418\uc5b4 \uc788\uc5c8\ub2e4.", "answer": "4\uc6d4 6\uc77c", "sentence": "3\uc6d4 24\uc77c\uc5d0 \uc0ac\uc138\ubcf4\ub97c \ucd9c\ubc1c\ud55c \ub77c\uc2a8 \ud638\ub294 4\uc6d4 6\uc77c \uc5d0 \ub871\ube44\uce58\uc5d0 \ub3c4\ucc29\ud588\ub2e4.", "paragraph_sentence": " 3\uc6d4 24\uc77c\uc5d0 \uc0ac\uc138\ubcf4\ub97c \ucd9c\ubc1c\ud55c \ub77c\uc2a8 \ud638\ub294 4\uc6d4 6\uc77c \uc5d0 \ub871\ube44\uce58\uc5d0 \ub3c4\ucc29\ud588\ub2e4. \ub77c\uc2a8 \ud638\uac00 \ub871\ube44\uce58\uc5d0 \ub3c4\ucc29\ud55c 4\uc6d4 6\uc77c\uc740 \ucde8\uc5ed\ud55c \uc9c0 23\uc8fc\ub144\uc774 \ub418\ub294 \ub0a0\uc774\uc5c8\ub2e4. 4\uc6d4 6\uc77c \uc774\ud6c4 \ub77c\uc2a8\uc740 \ud1f4\uc5ed \ub54c\uae4c\uc9c0 \ub300\ubd80\ubd84\uc758 \uc2dc\uac04\uc744 \ud56d\uad6c\uc5d0 \uc815\ubc15\ud55c \ucc44 \ubcf4\ub0b4\uac70\ub098 \uce98\ub9ac\ud3ec\ub2c8\uc544 \ub0a8\ucabd \uc791\uc804 \uad6c\uc5ed\uc5d0\uc11c \ub2e4\ub978 \ud568\uc815\ub4e4\uacfc \ud569\ub3d9 \ud6c8\ub828\uc744 \uc9c0\uc6d0\ud558\uba70 \uc9c0\ub0c8\ub2e4. \ub77c\uc2a8 \ud638\uc758 \uac00\uc7a5 \uc911\uc694\ud55c \uc784\ubb34\ub294 1\ub300\uc7a0\uc218\ud568\uadf8\ub8f9 \uc18c\uc18d\uc73c\ub85c\uc11c \ub300\uc7a0\uc218\ud568\uc804 \ud6c8\ub828\uc744 \ud558\ub294 \uac83\uc774\uc5c8\ub2e4. \uc774 \uadf8\ub8f9\uc5d0\ub294 CVS-33 USS \ud0a4\uc0c8\uc9c0, DD-723 USS \uc6d4\ud0a4, DD-754 USS \ud504\ub7ad\ud06c E. \uc5d0\ubc88\uc2a4, DD-787 USS \uc81c\uc784\uc2a4 E. \uce74\uc774, DD-830 USS \uc5d0\ubc84\ub9bf F. \ub77c\uc2a8, DEG-3 USS \uc1fc\ud544\ub4dc, DE-1037 USS \ube0c\ub860\uc288\ud0c0\uc778 \ub4f1\uc73c\ub85c \uad6c\uc131\ub418\uc5b4 \uc788\uc5c8\ub2e4.", "paragraph_answer": "3\uc6d4 24\uc77c\uc5d0 \uc0ac\uc138\ubcf4\ub97c \ucd9c\ubc1c\ud55c \ub77c\uc2a8 \ud638\ub294 4\uc6d4 6\uc77c \uc5d0 \ub871\ube44\uce58\uc5d0 \ub3c4\ucc29\ud588\ub2e4. \ub77c\uc2a8 \ud638\uac00 \ub871\ube44\uce58\uc5d0 \ub3c4\ucc29\ud55c 4\uc6d4 6\uc77c\uc740 \ucde8\uc5ed\ud55c \uc9c0 23\uc8fc\ub144\uc774 \ub418\ub294 \ub0a0\uc774\uc5c8\ub2e4. 4\uc6d4 6\uc77c \uc774\ud6c4 \ub77c\uc2a8\uc740 \ud1f4\uc5ed \ub54c\uae4c\uc9c0 \ub300\ubd80\ubd84\uc758 \uc2dc\uac04\uc744 \ud56d\uad6c\uc5d0 \uc815\ubc15\ud55c \ucc44 \ubcf4\ub0b4\uac70\ub098 \uce98\ub9ac\ud3ec\ub2c8\uc544 \ub0a8\ucabd \uc791\uc804 \uad6c\uc5ed\uc5d0\uc11c \ub2e4\ub978 \ud568\uc815\ub4e4\uacfc \ud569\ub3d9 \ud6c8\ub828\uc744 \uc9c0\uc6d0\ud558\uba70 \uc9c0\ub0c8\ub2e4. \ub77c\uc2a8 \ud638\uc758 \uac00\uc7a5 \uc911\uc694\ud55c \uc784\ubb34\ub294 1\ub300\uc7a0\uc218\ud568\uadf8\ub8f9 \uc18c\uc18d\uc73c\ub85c\uc11c \ub300\uc7a0\uc218\ud568\uc804 \ud6c8\ub828\uc744 \ud558\ub294 \uac83\uc774\uc5c8\ub2e4. \uc774 \uadf8\ub8f9\uc5d0\ub294 CVS-33 USS \ud0a4\uc0c8\uc9c0, DD-723 USS \uc6d4\ud0a4, DD-754 USS \ud504\ub7ad\ud06c E. \uc5d0\ubc88\uc2a4, DD-787 USS \uc81c\uc784\uc2a4 E. \uce74\uc774, DD-830 USS \uc5d0\ubc84\ub9bf F. \ub77c\uc2a8, DEG-3 USS \uc1fc\ud544\ub4dc, DE-1037 USS \ube0c\ub860\uc288\ud0c0\uc778 \ub4f1\uc73c\ub85c \uad6c\uc131\ub418\uc5b4 \uc788\uc5c8\ub2e4.", "sentence_answer": "3\uc6d4 24\uc77c\uc5d0 \uc0ac\uc138\ubcf4\ub97c \ucd9c\ubc1c\ud55c \ub77c\uc2a8 \ud638\ub294 4\uc6d4 6\uc77c \uc5d0 \ub871\ube44\uce58\uc5d0 \ub3c4\ucc29\ud588\ub2e4.", "paragraph_id": "6499636-3-2", "paragraph_question": "question: \ub77c\uc2a8 \ud638\uac00 \ub871\ube44\uce58\uc5d0 \ub3c4\ucc29\ud55c \ub0a0\uc740?, context: 3\uc6d4 24\uc77c\uc5d0 \uc0ac\uc138\ubcf4\ub97c \ucd9c\ubc1c\ud55c \ub77c\uc2a8 \ud638\ub294 4\uc6d4 6\uc77c\uc5d0 \ub871\ube44\uce58\uc5d0 \ub3c4\ucc29\ud588\ub2e4. \ub77c\uc2a8 \ud638\uac00 \ub871\ube44\uce58\uc5d0 \ub3c4\ucc29\ud55c 4\uc6d4 6\uc77c\uc740 \ucde8\uc5ed\ud55c \uc9c0 23\uc8fc\ub144\uc774 \ub418\ub294 \ub0a0\uc774\uc5c8\ub2e4. 4\uc6d4 6\uc77c \uc774\ud6c4 \ub77c\uc2a8\uc740 \ud1f4\uc5ed \ub54c\uae4c\uc9c0 \ub300\ubd80\ubd84\uc758 \uc2dc\uac04\uc744 \ud56d\uad6c\uc5d0 \uc815\ubc15\ud55c \ucc44 \ubcf4\ub0b4\uac70\ub098 \uce98\ub9ac\ud3ec\ub2c8\uc544 \ub0a8\ucabd \uc791\uc804 \uad6c\uc5ed\uc5d0\uc11c \ub2e4\ub978 \ud568\uc815\ub4e4\uacfc \ud569\ub3d9 \ud6c8\ub828\uc744 \uc9c0\uc6d0\ud558\uba70 \uc9c0\ub0c8\ub2e4. \ub77c\uc2a8 \ud638\uc758 \uac00\uc7a5 \uc911\uc694\ud55c \uc784\ubb34\ub294 1\ub300\uc7a0\uc218\ud568\uadf8\ub8f9 \uc18c\uc18d\uc73c\ub85c\uc11c \ub300\uc7a0\uc218\ud568\uc804 \ud6c8\ub828\uc744 \ud558\ub294 \uac83\uc774\uc5c8\ub2e4. \uc774 \uadf8\ub8f9\uc5d0\ub294 CVS-33 USS \ud0a4\uc0c8\uc9c0, DD-723 USS \uc6d4\ud0a4, DD-754 USS \ud504\ub7ad\ud06c E. \uc5d0\ubc88\uc2a4, DD-787 USS \uc81c\uc784\uc2a4 E. \uce74\uc774, DD-830 USS \uc5d0\ubc84\ub9bf F. \ub77c\uc2a8, DEG-3 USS \uc1fc\ud544\ub4dc, DE-1037 USS \ube0c\ub860\uc288\ud0c0\uc778 \ub4f1\uc73c\ub85c \uad6c\uc131\ub418\uc5b4 \uc788\uc5c8\ub2e4."} {"question": "\ud53c\uce74\ub808\uc2a4\ud06c \uc18c\uc124\uc774\ub780 \uc5b4\ub290 \ub2e8\uc5b4\uc5d0\uc11c \uc720\ub798\ub41c \uc6a9\uc5b4\uc778\uac00?", "paragraph": "\uc8fc\uc778\uacf5\uc758 \uc131\uaca9\uc744 \uc9c0\uce6d\ud558\ub294 \u2018\ud53c\uce74\ub85c(p\u00edcaro)\u2019\ub77c\ub294 \ub9d0\uc5d0\uc11c \uc5f0\uc720\ub41c \u2018\ud53c\uce74\ub808\uc2a4\ud06c \uc18c\uc124\u2019\uc740 \uc2a4\ud398\uc778 \uc18c\uc124\uc0ac\uc758 \ud55c \ud68d\uc744 \uae0b\ub294 \uc911\uc694\ud55c \uc7a5\ub974\ub77c\uace0 \ub9d0\ud560 \uc218 \uc788\ub2e4. \uc774 \uc2dc\uae30\uc5d0 \uc2a4\ud398\uc778\uc744 \ube44\ub86f\ud55c \uc720\ub7fd\uc758 \ub300\ubd80\ubd84\uc758 \uc18c\uc124\ub4e4\uc774 \uc57d\uac04\uc740 \ud669\ub2f9\ubb34\uacc4\ud55c \ud765\ubbf8 \uc704\uc8fc \uc774\uc57c\uae30\ub97c \ubc97\uc5b4\ub098\uc9c0 \ubabb\ud558\uace0 \uc788\uc5c8\ub2e4. \uc774\ub54c \u2018\ud53c\uce74\ub85c(p\u00edcaro)\u2019 \ud639\uc740 \u2018\ud53c\uce74\ub77c(p\u00edcara)\u2019\ub85c \ubd88\ub9ac\ub294 \uc18c\ub144\uc774\ub098 \uc18c\ub140\uac00 \uacaa\ub294 \uc5ec\ub7ec \uac00\uc9c0 \uc0b6\uc758 \uacbd\ud5d8\ub4e4\uc744 \ud1b5\ud574 \ub2f9\uc2dc \uc0ac\ud68c\uc758 \uc2e4\uc0c1\uc744 \uc0ac\uc2e4\uc801\uc73c\ub85c \ubb18\uc0ac\ud55c \uc774 \uc18c\uc124\uc774 \ub4f1\uc7a5\ud588\ub2e4. \uc774 \uc18c\uc124\uc740 \u2018\uc7ac\ubbf8\uc788\ub294 \uc774\uc57c\uae30\ub098 \ub3c4\ub355\u00b7\uad50\ud6c8\uc801 \uc774\uc57c\uae30\u2019\uc5d0 \uba38\ubb3c\ub7ec \uc788\ub358 \ub2f9\uc2dc \uc18c\uc124\uc758 \ubc94\uc8fc\ub97c \ubc97\uc5b4\ub098 \uc0b6\uc5d0 \ubcf4\ub2e4 \uac00\uae5d\uac8c \uc811\uadfc\ud558\uc5ec \uc0b6\uc758 \uc9c4\uc194\ud55c \ubaa9\uc18c\ub9ac\ub97c \uc804\ub2ec\ud574 \uc8fc\uace0 \uc788\ub2e4\ub294 \uc810\uc5d0\uc11c \ud604\ub300\uc131\uc744 \ud5a5\ud574 \uc9c4\uc77c\ubcf4\ud55c \ubaa8\uc2b5\uc744 \ubcf4\uc5ec\uc8fc\uace0 \uc788\ub2e4.", "answer": "\ud53c\uce74\ub85c", "sentence": "\uc8fc\uc778\uacf5\uc758 \uc131\uaca9\uc744 \uc9c0\uce6d\ud558\ub294 \u2018 \ud53c\uce74\ub85c (p\u00edcaro)", "paragraph_sentence": " \uc8fc\uc778\uacf5\uc758 \uc131\uaca9\uc744 \uc9c0\uce6d\ud558\ub294 \u2018 \ud53c\uce74\ub85c (p\u00edcaro) \u2019\ub77c\ub294 \ub9d0\uc5d0\uc11c \uc5f0\uc720\ub41c \u2018\ud53c\uce74\ub808\uc2a4\ud06c \uc18c\uc124\u2019\uc740 \uc2a4\ud398\uc778 \uc18c\uc124\uc0ac\uc758 \ud55c \ud68d\uc744 \uae0b\ub294 \uc911\uc694\ud55c \uc7a5\ub974\ub77c\uace0 \ub9d0\ud560 \uc218 \uc788\ub2e4. \uc774 \uc2dc\uae30\uc5d0 \uc2a4\ud398\uc778\uc744 \ube44\ub86f\ud55c \uc720\ub7fd\uc758 \ub300\ubd80\ubd84\uc758 \uc18c\uc124\ub4e4\uc774 \uc57d\uac04\uc740 \ud669\ub2f9\ubb34\uacc4\ud55c \ud765\ubbf8 \uc704\uc8fc \uc774\uc57c\uae30\ub97c \ubc97\uc5b4\ub098\uc9c0 \ubabb\ud558\uace0 \uc788\uc5c8\ub2e4. \uc774\ub54c \u2018\ud53c\uce74\ub85c(p\u00edcaro)\u2019 \ud639\uc740 \u2018\ud53c\uce74\ub77c(p\u00edcara)\u2019\ub85c \ubd88\ub9ac\ub294 \uc18c\ub144\uc774\ub098 \uc18c\ub140\uac00 \uacaa\ub294 \uc5ec\ub7ec \uac00\uc9c0 \uc0b6\uc758 \uacbd\ud5d8\ub4e4\uc744 \ud1b5\ud574 \ub2f9\uc2dc \uc0ac\ud68c\uc758 \uc2e4\uc0c1\uc744 \uc0ac\uc2e4\uc801\uc73c\ub85c \ubb18\uc0ac\ud55c \uc774 \uc18c\uc124\uc774 \ub4f1\uc7a5\ud588\ub2e4. \uc774 \uc18c\uc124\uc740 \u2018\uc7ac\ubbf8\uc788\ub294 \uc774\uc57c\uae30\ub098 \ub3c4\ub355\u00b7\uad50\ud6c8\uc801 \uc774\uc57c\uae30\u2019\uc5d0 \uba38\ubb3c\ub7ec \uc788\ub358 \ub2f9\uc2dc \uc18c\uc124\uc758 \ubc94\uc8fc\ub97c \ubc97\uc5b4\ub098 \uc0b6\uc5d0 \ubcf4\ub2e4 \uac00\uae5d\uac8c \uc811\uadfc\ud558\uc5ec \uc0b6\uc758 \uc9c4\uc194\ud55c \ubaa9\uc18c\ub9ac\ub97c \uc804\ub2ec\ud574 \uc8fc\uace0 \uc788\ub2e4\ub294 \uc810\uc5d0\uc11c \ud604\ub300\uc131\uc744 \ud5a5\ud574 \uc9c4\uc77c\ubcf4\ud55c \ubaa8\uc2b5\uc744 \ubcf4\uc5ec\uc8fc\uace0 \uc788\ub2e4.", "paragraph_answer": "\uc8fc\uc778\uacf5\uc758 \uc131\uaca9\uc744 \uc9c0\uce6d\ud558\ub294 \u2018 \ud53c\uce74\ub85c (p\u00edcaro)\u2019\ub77c\ub294 \ub9d0\uc5d0\uc11c \uc5f0\uc720\ub41c \u2018\ud53c\uce74\ub808\uc2a4\ud06c \uc18c\uc124\u2019\uc740 \uc2a4\ud398\uc778 \uc18c\uc124\uc0ac\uc758 \ud55c \ud68d\uc744 \uae0b\ub294 \uc911\uc694\ud55c \uc7a5\ub974\ub77c\uace0 \ub9d0\ud560 \uc218 \uc788\ub2e4. \uc774 \uc2dc\uae30\uc5d0 \uc2a4\ud398\uc778\uc744 \ube44\ub86f\ud55c \uc720\ub7fd\uc758 \ub300\ubd80\ubd84\uc758 \uc18c\uc124\ub4e4\uc774 \uc57d\uac04\uc740 \ud669\ub2f9\ubb34\uacc4\ud55c \ud765\ubbf8 \uc704\uc8fc \uc774\uc57c\uae30\ub97c \ubc97\uc5b4\ub098\uc9c0 \ubabb\ud558\uace0 \uc788\uc5c8\ub2e4. \uc774\ub54c \u2018\ud53c\uce74\ub85c(p\u00edcaro)\u2019 \ud639\uc740 \u2018\ud53c\uce74\ub77c(p\u00edcara)\u2019\ub85c \ubd88\ub9ac\ub294 \uc18c\ub144\uc774\ub098 \uc18c\ub140\uac00 \uacaa\ub294 \uc5ec\ub7ec \uac00\uc9c0 \uc0b6\uc758 \uacbd\ud5d8\ub4e4\uc744 \ud1b5\ud574 \ub2f9\uc2dc \uc0ac\ud68c\uc758 \uc2e4\uc0c1\uc744 \uc0ac\uc2e4\uc801\uc73c\ub85c \ubb18\uc0ac\ud55c \uc774 \uc18c\uc124\uc774 \ub4f1\uc7a5\ud588\ub2e4. \uc774 \uc18c\uc124\uc740 \u2018\uc7ac\ubbf8\uc788\ub294 \uc774\uc57c\uae30\ub098 \ub3c4\ub355\u00b7\uad50\ud6c8\uc801 \uc774\uc57c\uae30\u2019\uc5d0 \uba38\ubb3c\ub7ec \uc788\ub358 \ub2f9\uc2dc \uc18c\uc124\uc758 \ubc94\uc8fc\ub97c \ubc97\uc5b4\ub098 \uc0b6\uc5d0 \ubcf4\ub2e4 \uac00\uae5d\uac8c \uc811\uadfc\ud558\uc5ec \uc0b6\uc758 \uc9c4\uc194\ud55c \ubaa9\uc18c\ub9ac\ub97c \uc804\ub2ec\ud574 \uc8fc\uace0 \uc788\ub2e4\ub294 \uc810\uc5d0\uc11c \ud604\ub300\uc131\uc744 \ud5a5\ud574 \uc9c4\uc77c\ubcf4\ud55c \ubaa8\uc2b5\uc744 \ubcf4\uc5ec\uc8fc\uace0 \uc788\ub2e4.", "sentence_answer": "\uc8fc\uc778\uacf5\uc758 \uc131\uaca9\uc744 \uc9c0\uce6d\ud558\ub294 \u2018 \ud53c\uce74\ub85c (p\u00edcaro)", "paragraph_id": "6528934-1-0", "paragraph_question": "question: \ud53c\uce74\ub808\uc2a4\ud06c \uc18c\uc124\uc774\ub780 \uc5b4\ub290 \ub2e8\uc5b4\uc5d0\uc11c \uc720\ub798\ub41c \uc6a9\uc5b4\uc778\uac00?, context: \uc8fc\uc778\uacf5\uc758 \uc131\uaca9\uc744 \uc9c0\uce6d\ud558\ub294 \u2018\ud53c\uce74\ub85c(p\u00edcaro)\u2019\ub77c\ub294 \ub9d0\uc5d0\uc11c \uc5f0\uc720\ub41c \u2018\ud53c\uce74\ub808\uc2a4\ud06c \uc18c\uc124\u2019\uc740 \uc2a4\ud398\uc778 \uc18c\uc124\uc0ac\uc758 \ud55c \ud68d\uc744 \uae0b\ub294 \uc911\uc694\ud55c \uc7a5\ub974\ub77c\uace0 \ub9d0\ud560 \uc218 \uc788\ub2e4. \uc774 \uc2dc\uae30\uc5d0 \uc2a4\ud398\uc778\uc744 \ube44\ub86f\ud55c \uc720\ub7fd\uc758 \ub300\ubd80\ubd84\uc758 \uc18c\uc124\ub4e4\uc774 \uc57d\uac04\uc740 \ud669\ub2f9\ubb34\uacc4\ud55c \ud765\ubbf8 \uc704\uc8fc \uc774\uc57c\uae30\ub97c \ubc97\uc5b4\ub098\uc9c0 \ubabb\ud558\uace0 \uc788\uc5c8\ub2e4. \uc774\ub54c \u2018\ud53c\uce74\ub85c(p\u00edcaro)\u2019 \ud639\uc740 \u2018\ud53c\uce74\ub77c(p\u00edcara)\u2019\ub85c \ubd88\ub9ac\ub294 \uc18c\ub144\uc774\ub098 \uc18c\ub140\uac00 \uacaa\ub294 \uc5ec\ub7ec \uac00\uc9c0 \uc0b6\uc758 \uacbd\ud5d8\ub4e4\uc744 \ud1b5\ud574 \ub2f9\uc2dc \uc0ac\ud68c\uc758 \uc2e4\uc0c1\uc744 \uc0ac\uc2e4\uc801\uc73c\ub85c \ubb18\uc0ac\ud55c \uc774 \uc18c\uc124\uc774 \ub4f1\uc7a5\ud588\ub2e4. \uc774 \uc18c\uc124\uc740 \u2018\uc7ac\ubbf8\uc788\ub294 \uc774\uc57c\uae30\ub098 \ub3c4\ub355\u00b7\uad50\ud6c8\uc801 \uc774\uc57c\uae30\u2019\uc5d0 \uba38\ubb3c\ub7ec \uc788\ub358 \ub2f9\uc2dc \uc18c\uc124\uc758 \ubc94\uc8fc\ub97c \ubc97\uc5b4\ub098 \uc0b6\uc5d0 \ubcf4\ub2e4 \uac00\uae5d\uac8c \uc811\uadfc\ud558\uc5ec \uc0b6\uc758 \uc9c4\uc194\ud55c \ubaa9\uc18c\ub9ac\ub97c \uc804\ub2ec\ud574 \uc8fc\uace0 \uc788\ub2e4\ub294 \uc810\uc5d0\uc11c \ud604\ub300\uc131\uc744 \ud5a5\ud574 \uc9c4\uc77c\ubcf4\ud55c \ubaa8\uc2b5\uc744 \ubcf4\uc5ec\uc8fc\uace0 \uc788\ub2e4."} {"question": "\uac74\ub96d\uc81c\ub294 \ubd80\ubd84\uc801\uc73c\ub85c\ub098\ub9c8 \ub204\uad6c\uc758 \uc120\uad50\ub97c \ud5c8\ub77d\ud558\uc600\ub098?", "paragraph": "\uac74\ub96d\uc81c\ub294 \ubd80\ubd84\uc801\uc73c\ub85c\ub098\ub9c8 \uc608\uc218\ud68c \uc120\uad50\uc0ac\ub4e4\uc758 \uc120\uad50\ub97c \ud5c8\ub77d\ud558\uc600\uace0 \uc120\uad50\uc0ac\ub4e4\uc740 \uc774\uc5d0 \ub530\ub77c \uad50\uc138\ub97c \ud655\uc7a5\uc2dc\ud0a4\uace0 \uac00\ud1a8\ub9ad\uc73c\ub85c \uac1c\uc885\ud55c \uc2e0\uc790\ub4e4\uc5d0\uac90 \uc870\uc0c1\uc758 \uc81c\uc0ac \ub4f1 \ubcf8\ub798\uc758 \uad00\uc2b5\uc744 \uc5b4\ub290 \uc815\ub3c4 \uc9c0\ud0ac \uc218 \uc788\ub3c4\ub85d \ubc30\ub824\ud558\uc600\ub2e4. \uadf8\ub7ec\ub098 1742\ub144(\uac74\ub96d 7\ub144) \uad50\ud669 \ubca0\ub124\ub515\ud1a0 14\uc138\ub294 \u3008\uac01 \uacbd\uc6b0\uc5d0 \ub530\ub77c\uc11c\u3009(Ex quo Singulari)\ub77c\ub294 \uce59\uc11c\ub97c \ub0b4\ub824 \uac00\ud1a8\ub9ad\uc73c\ub85c \uac1c\uc885\ud55c \uc2e0\uc790\ub4e4\uc774 \ud48d\uc2b5\uc744 \uc720\uc9c0\ud560 \uc218 \uc788\ub358 \uad00\ud589\uc744 \uae08\uc9c0\uc2dc\ud0a4\uace0 1744\ub144(\uac74\ub96d 9\ub144) \ub2e4\uc2dc \ub610\ub2e4\ub978 \uce59\uc11c \u3008\uc634\ub2c8\uc6c0 \uc194\ub9ac\ud0a4\ud22c\ub514\ub214\u3009(Omnium solicitudinum)\ub97c \ub0b4\ub824 \uc774\ub97c \uc7ac\ud655\uc778\ud558\uc600\ub2e4. \uad50\ud669\uc758 \uce59\ub839\uc774 \ub0b4\ub9b0 \ud6c4\ub85c\ub294 \uac00\ud1a8\ub9ad \uc2e0\uc790\uc758 \uc218\uac00 \ub298\uc5b4\ub098\uc9c0 \uc54a\uc558\uace0 \uc608\uc218\ud68c \uc120\uad50\uc0ac\ub4e4\ub3c4 \uc81c\ub300\ub85c \uc120\uad50 \ud65c\ub3d9\uc744 \ud3b4\uc9c0 \ubabb\ud558\uc600\ub2e4. \uadf8 \ud6c4 \uc911\uad6d \ub0b4\uc758 \uac00\ud1a8\ub9ad \uad50\uc138\ub294 \uc870\uc815\uc758 \ud0c4\uc555\uacfc \uc720\ud559\uc790\ub4e4\uc758 \uacf5\uaca9\uc73c\ub85c \uccad\ub098\ub77c \uba78\ub9dd \ub54c\uae4c\uc9c0 \ud06c\uac8c \uc131\uc7a5\ud558\uc9c0 \ubabb\ud558\uc600\ub2e4.", "answer": "\uc608\uc218\ud68c \uc120\uad50\uc0ac\ub4e4", "sentence": "\uac74\ub96d\uc81c\ub294 \ubd80\ubd84\uc801\uc73c\ub85c\ub098\ub9c8 \uc608\uc218\ud68c \uc120\uad50\uc0ac\ub4e4 \uc758 \uc120\uad50\ub97c \ud5c8\ub77d\ud558\uc600\uace0 \uc120\uad50\uc0ac\ub4e4\uc740 \uc774\uc5d0 \ub530\ub77c \uad50\uc138\ub97c \ud655\uc7a5\uc2dc\ud0a4\uace0 \uac00\ud1a8\ub9ad\uc73c\ub85c \uac1c\uc885\ud55c \uc2e0\uc790\ub4e4\uc5d0\uac90 \uc870\uc0c1\uc758 \uc81c\uc0ac \ub4f1 \ubcf8\ub798\uc758 \uad00\uc2b5\uc744 \uc5b4\ub290 \uc815\ub3c4 \uc9c0\ud0ac \uc218 \uc788\ub3c4\ub85d \ubc30\ub824\ud558\uc600\ub2e4.", "paragraph_sentence": " \uac74\ub96d\uc81c\ub294 \ubd80\ubd84\uc801\uc73c\ub85c\ub098\ub9c8 \uc608\uc218\ud68c \uc120\uad50\uc0ac\ub4e4 \uc758 \uc120\uad50\ub97c \ud5c8\ub77d\ud558\uc600\uace0 \uc120\uad50\uc0ac\ub4e4\uc740 \uc774\uc5d0 \ub530\ub77c \uad50\uc138\ub97c \ud655\uc7a5\uc2dc\ud0a4\uace0 \uac00\ud1a8\ub9ad\uc73c\ub85c \uac1c\uc885\ud55c \uc2e0\uc790\ub4e4\uc5d0\uac90 \uc870\uc0c1\uc758 \uc81c\uc0ac \ub4f1 \ubcf8\ub798\uc758 \uad00\uc2b5\uc744 \uc5b4\ub290 \uc815\ub3c4 \uc9c0\ud0ac \uc218 \uc788\ub3c4\ub85d \ubc30\ub824\ud558\uc600\ub2e4. \uadf8\ub7ec\ub098 1742\ub144(\uac74\ub96d 7\ub144) \uad50\ud669 \ubca0\ub124\ub515\ud1a0 14\uc138\ub294 \u3008\uac01 \uacbd\uc6b0\uc5d0 \ub530\ub77c\uc11c\u3009(Ex quo Singulari)\ub77c\ub294 \uce59\uc11c\ub97c \ub0b4\ub824 \uac00\ud1a8\ub9ad\uc73c\ub85c \uac1c\uc885\ud55c \uc2e0\uc790\ub4e4\uc774 \ud48d\uc2b5\uc744 \uc720\uc9c0\ud560 \uc218 \uc788\ub358 \uad00\ud589\uc744 \uae08\uc9c0\uc2dc\ud0a4\uace0 1744\ub144(\uac74\ub96d 9\ub144) \ub2e4\uc2dc \ub610\ub2e4\ub978 \uce59\uc11c \u3008\uc634\ub2c8\uc6c0 \uc194\ub9ac\ud0a4\ud22c\ub514\ub214\u3009(Omnium solicitudinum)\ub97c \ub0b4\ub824 \uc774\ub97c \uc7ac\ud655\uc778\ud558\uc600\ub2e4. \uad50\ud669\uc758 \uce59\ub839\uc774 \ub0b4\ub9b0 \ud6c4\ub85c\ub294 \uac00\ud1a8\ub9ad \uc2e0\uc790\uc758 \uc218\uac00 \ub298\uc5b4\ub098\uc9c0 \uc54a\uc558\uace0 \uc608\uc218\ud68c \uc120\uad50\uc0ac\ub4e4\ub3c4 \uc81c\ub300\ub85c \uc120\uad50 \ud65c\ub3d9\uc744 \ud3b4\uc9c0 \ubabb\ud558\uc600\ub2e4. \uadf8 \ud6c4 \uc911\uad6d \ub0b4\uc758 \uac00\ud1a8\ub9ad \uad50\uc138\ub294 \uc870\uc815\uc758 \ud0c4\uc555\uacfc \uc720\ud559\uc790\ub4e4\uc758 \uacf5\uaca9\uc73c\ub85c \uccad\ub098\ub77c \uba78\ub9dd \ub54c\uae4c\uc9c0 \ud06c\uac8c \uc131\uc7a5\ud558\uc9c0 \ubabb\ud558\uc600\ub2e4.", "paragraph_answer": "\uac74\ub96d\uc81c\ub294 \ubd80\ubd84\uc801\uc73c\ub85c\ub098\ub9c8 \uc608\uc218\ud68c \uc120\uad50\uc0ac\ub4e4 \uc758 \uc120\uad50\ub97c \ud5c8\ub77d\ud558\uc600\uace0 \uc120\uad50\uc0ac\ub4e4\uc740 \uc774\uc5d0 \ub530\ub77c \uad50\uc138\ub97c \ud655\uc7a5\uc2dc\ud0a4\uace0 \uac00\ud1a8\ub9ad\uc73c\ub85c \uac1c\uc885\ud55c \uc2e0\uc790\ub4e4\uc5d0\uac90 \uc870\uc0c1\uc758 \uc81c\uc0ac \ub4f1 \ubcf8\ub798\uc758 \uad00\uc2b5\uc744 \uc5b4\ub290 \uc815\ub3c4 \uc9c0\ud0ac \uc218 \uc788\ub3c4\ub85d \ubc30\ub824\ud558\uc600\ub2e4. \uadf8\ub7ec\ub098 1742\ub144(\uac74\ub96d 7\ub144) \uad50\ud669 \ubca0\ub124\ub515\ud1a0 14\uc138\ub294 \u3008\uac01 \uacbd\uc6b0\uc5d0 \ub530\ub77c\uc11c\u3009(Ex quo Singulari)\ub77c\ub294 \uce59\uc11c\ub97c \ub0b4\ub824 \uac00\ud1a8\ub9ad\uc73c\ub85c \uac1c\uc885\ud55c \uc2e0\uc790\ub4e4\uc774 \ud48d\uc2b5\uc744 \uc720\uc9c0\ud560 \uc218 \uc788\ub358 \uad00\ud589\uc744 \uae08\uc9c0\uc2dc\ud0a4\uace0 1744\ub144(\uac74\ub96d 9\ub144) \ub2e4\uc2dc \ub610\ub2e4\ub978 \uce59\uc11c \u3008\uc634\ub2c8\uc6c0 \uc194\ub9ac\ud0a4\ud22c\ub514\ub214\u3009(Omnium solicitudinum)\ub97c \ub0b4\ub824 \uc774\ub97c \uc7ac\ud655\uc778\ud558\uc600\ub2e4. \uad50\ud669\uc758 \uce59\ub839\uc774 \ub0b4\ub9b0 \ud6c4\ub85c\ub294 \uac00\ud1a8\ub9ad \uc2e0\uc790\uc758 \uc218\uac00 \ub298\uc5b4\ub098\uc9c0 \uc54a\uc558\uace0 \uc608\uc218\ud68c \uc120\uad50\uc0ac\ub4e4\ub3c4 \uc81c\ub300\ub85c \uc120\uad50 \ud65c\ub3d9\uc744 \ud3b4\uc9c0 \ubabb\ud558\uc600\ub2e4. \uadf8 \ud6c4 \uc911\uad6d \ub0b4\uc758 \uac00\ud1a8\ub9ad \uad50\uc138\ub294 \uc870\uc815\uc758 \ud0c4\uc555\uacfc \uc720\ud559\uc790\ub4e4\uc758 \uacf5\uaca9\uc73c\ub85c \uccad\ub098\ub77c \uba78\ub9dd \ub54c\uae4c\uc9c0 \ud06c\uac8c \uc131\uc7a5\ud558\uc9c0 \ubabb\ud558\uc600\ub2e4.", "sentence_answer": "\uac74\ub96d\uc81c\ub294 \ubd80\ubd84\uc801\uc73c\ub85c\ub098\ub9c8 \uc608\uc218\ud68c \uc120\uad50\uc0ac\ub4e4 \uc758 \uc120\uad50\ub97c \ud5c8\ub77d\ud558\uc600\uace0 \uc120\uad50\uc0ac\ub4e4\uc740 \uc774\uc5d0 \ub530\ub77c \uad50\uc138\ub97c \ud655\uc7a5\uc2dc\ud0a4\uace0 \uac00\ud1a8\ub9ad\uc73c\ub85c \uac1c\uc885\ud55c \uc2e0\uc790\ub4e4\uc5d0\uac90 \uc870\uc0c1\uc758 \uc81c\uc0ac \ub4f1 \ubcf8\ub798\uc758 \uad00\uc2b5\uc744 \uc5b4\ub290 \uc815\ub3c4 \uc9c0\ud0ac \uc218 \uc788\ub3c4\ub85d \ubc30\ub824\ud558\uc600\ub2e4.", "paragraph_id": "6472331-24-0", "paragraph_question": "question: \uac74\ub96d\uc81c\ub294 \ubd80\ubd84\uc801\uc73c\ub85c\ub098\ub9c8 \ub204\uad6c\uc758 \uc120\uad50\ub97c \ud5c8\ub77d\ud558\uc600\ub098?, context: \uac74\ub96d\uc81c\ub294 \ubd80\ubd84\uc801\uc73c\ub85c\ub098\ub9c8 \uc608\uc218\ud68c \uc120\uad50\uc0ac\ub4e4\uc758 \uc120\uad50\ub97c \ud5c8\ub77d\ud558\uc600\uace0 \uc120\uad50\uc0ac\ub4e4\uc740 \uc774\uc5d0 \ub530\ub77c \uad50\uc138\ub97c \ud655\uc7a5\uc2dc\ud0a4\uace0 \uac00\ud1a8\ub9ad\uc73c\ub85c \uac1c\uc885\ud55c \uc2e0\uc790\ub4e4\uc5d0\uac90 \uc870\uc0c1\uc758 \uc81c\uc0ac \ub4f1 \ubcf8\ub798\uc758 \uad00\uc2b5\uc744 \uc5b4\ub290 \uc815\ub3c4 \uc9c0\ud0ac \uc218 \uc788\ub3c4\ub85d \ubc30\ub824\ud558\uc600\ub2e4. \uadf8\ub7ec\ub098 1742\ub144(\uac74\ub96d 7\ub144) \uad50\ud669 \ubca0\ub124\ub515\ud1a0 14\uc138\ub294 \u3008\uac01 \uacbd\uc6b0\uc5d0 \ub530\ub77c\uc11c\u3009(Ex quo Singulari)\ub77c\ub294 \uce59\uc11c\ub97c \ub0b4\ub824 \uac00\ud1a8\ub9ad\uc73c\ub85c \uac1c\uc885\ud55c \uc2e0\uc790\ub4e4\uc774 \ud48d\uc2b5\uc744 \uc720\uc9c0\ud560 \uc218 \uc788\ub358 \uad00\ud589\uc744 \uae08\uc9c0\uc2dc\ud0a4\uace0 1744\ub144(\uac74\ub96d 9\ub144) \ub2e4\uc2dc \ub610\ub2e4\ub978 \uce59\uc11c \u3008\uc634\ub2c8\uc6c0 \uc194\ub9ac\ud0a4\ud22c\ub514\ub214\u3009(Omnium solicitudinum)\ub97c \ub0b4\ub824 \uc774\ub97c \uc7ac\ud655\uc778\ud558\uc600\ub2e4. \uad50\ud669\uc758 \uce59\ub839\uc774 \ub0b4\ub9b0 \ud6c4\ub85c\ub294 \uac00\ud1a8\ub9ad \uc2e0\uc790\uc758 \uc218\uac00 \ub298\uc5b4\ub098\uc9c0 \uc54a\uc558\uace0 \uc608\uc218\ud68c \uc120\uad50\uc0ac\ub4e4\ub3c4 \uc81c\ub300\ub85c \uc120\uad50 \ud65c\ub3d9\uc744 \ud3b4\uc9c0 \ubabb\ud558\uc600\ub2e4. \uadf8 \ud6c4 \uc911\uad6d \ub0b4\uc758 \uac00\ud1a8\ub9ad \uad50\uc138\ub294 \uc870\uc815\uc758 \ud0c4\uc555\uacfc \uc720\ud559\uc790\ub4e4\uc758 \uacf5\uaca9\uc73c\ub85c \uccad\ub098\ub77c \uba78\ub9dd \ub54c\uae4c\uc9c0 \ud06c\uac8c \uc131\uc7a5\ud558\uc9c0 \ubabb\ud558\uc600\ub2e4."} {"question": "\uace0\uac01 \ubc1c\uc0ac\ub41c \ubbf8\uc0ac\uc77c\uc740 \ub300\uae30\uad8c \ubc16 \uc5b4\ub514\uae4c\uc9c0 \uce58\uc19f\uac8c \ub418\ub098?", "paragraph": "\ubd81\ud55c\uc740 \ubbf8\uc0ac\uc77c\uc758 \ubc1c\uc0ac \uac01\ub3c4\ub97c \ub192\uc5ec \u2018\uace0\uac01(\u9ad8\u89d2)\u2019\uc73c\ub85c \uc3d8\ub294 \uc2e4\ud5d8\uc744 \ud558\uace0 \uc788\ub2e4. \uc608\ub97c \ub4e4\uc5b4\uc11c \uc27d\uac8c \uc124\uba85\ud558\uc790\uba74 \uacf5\uc744 \uc644\ub9cc\ud55c \ud3ec\ubb3c\uc120\uc744 \uadf8\ub9ac\uac8c \ud558\uba70 \ub358\uc838 \uba40\ub9ac \ub0a0\uc544\uac00\uac8c \ub358\uc9c0\ub294 \uac83\uc774 \uc544\ub2c8\ub77c, \ucd5c\ub300\ud55c \uba38\ub9ac \uc704\ub85c \uacf5\uc744 \ub192\uac8c \ub358\uc838 \uc62c\ub824 \uc790\uae30 \ubc1c \uadfc\ucc98\uc5d0 \ub5a8\uc5b4\uc9c0\uac8c \ud558\ub294 \ubc29\uc2dd\uc774\ub2e4. \uace0\uac01 \ubc1c\uc0ac\ub97c \ud558\uba74 \ubbf8\uc0ac\uc77c\uc740 \uc131\uce35\uad8c\uc744 \uc9c0\ub098 \u2018\ub300\uae30\uad8c \ubc16\u2019\uc758 \uc6b0\uc8fc \uacf5\uac04\uc73c\ub85c \uce58\uc19f\ub294\ub2e4. \uace0\uac01\ub3c4 \ubc1c\uc0ac\ub85c \ubbf8\uc0ac\uc77c\uc774 \uace0\uace0\ub3c4\uae4c\uc9c0 \uce58\uc19f\uc558\ub2e4\uac00 \ub5a8\uc5b4\uc9c8 \ub54c \ubbf8\uc0ac\uc77c\uc758 \uc18d\ub3c4\ub294 \ubcf8\ub798 \uc0ac\uac70\ub9ac\ub97c \ube44\ud589\ud560 \ub54c\ubcf4\ub2e4 \ud6e8\uc52c \ube60\ub974\ub2e4. \ub9cc\uc720\uc778\ub825\uc758 \ubc95\uce59(\uc911\ub825)\uc73c\ub85c \uc778\ud558\uc5ec \ub099\ud558\ud558\ub294 \ubbf8\uc0ac\uc77c\uc5d0 \uac00\uc18d\ub3c4\uac00 \ubd99\uae30 \ub54c\ubb38\uc774\ub2e4. \uc774\ub7ec\ud55c \ub9cc\uc720\uc778\ub825\uc758 \ubc95\uce59\uc744 \uc774\uc6a9\ud55c \uace0\uac01\ub3c4 \ubc1c\uc0ac\ub85c \ubd81\ud55c\uc758 \ubb34\uc218\ub2e8 \ubbf8\uc0ac\uc77c\uc758 \uc18d\ub3c4\ub294 \ub9c8\ud558 10 \uc774\uc0c1\uc758 \uc18d\ub3c4\ub97c \uae30\ub85d\ud588\ub2e4. \ud604\uc7ac \ud55c\uad6d\uc774 \uc8fc\ub825\uc73c\ub85c \uc0ac\uc6a9\ud558\uace0 \uc788\ub294 \ud328\ud2b8\ub9ac\uc5b4\ud2b8 PAC-3 \ub294 \ub9c8\ud558 5 \uc774\uc0c1\uc740 \uc694\uaca9\uc774 \ubd88\uac00\ub2a5\ud558\ub2e4. \ud328\ud2b8\ub9ac\uc5b4\ud2b8\uc758 \ud55c\uacc4\ub294 \ub9c8\ud558 5 \uc774\uae30 \ub54c\ubb38\uc5d0 \ub9c8\ud558 10 \uc758 \uc18d\ub3c4\ub85c \ub0b4\ub824\uaf42\ud788\ub294 \ubb34\uc218\ub2e8 \ubbf8\uc0ac\uc77c\uc740 \uaca9\ucd94\ud560 \uc218 \uc5c6\ub2e4. \uace0\uac01\ub3c4 \ubbf8\uc0ac\uc77c \ubc1c\uc0ac \uc2e4\ud5d8\uc744 \ubc18\ubcf5\uc801\uc73c\ub85c \ud558\uace0 \uc788\ub294 \ubd81\ud55c\uc758 \uc758\ub3c4\ub294 \ud328\ud2b8\ub9ac\uc5b4\ud2b8 PAC-3 \ub4f1\uc73c\ub85c \ubb34\uc7a5\ud55c \ud55c\uad6d\uc758 \ubbf8\uc0ac\uc77c \ubc29\uc5b4\uccb4\uacc4\ub97c \ubb34\ub825\ud654\ud558\uaca0\ub2e4\ub294 \uac83\uc774\ub2e4. \ubc18\uba74\uc5d0, \uc0ac\ub4dc\ub294 \ub9c8\ud558 14 \uae4c\uc9c0 \uc694\uaca9\uc774 \uac00\ub2a5\ud568\uc73c\ub85c \ubb34\uc218\ub2e8\uae09 \uc774\uc0c1\uc758 \ubbf8\uc0ac\uc77c\uc744 \ub9c9\uae30\uc704\ud574\uc11c\ub294 \uc0ac\ub4dc\uac00 \ud544\uc694\ud558\ub2e4. \ubd81\ud55c\uc740 \uac70\uc758 \uc218\uc9c1\uc5d0 \uac00\uae4c\uc6b4 \uace0\uac01\ub3c4 \ubc1c\uc0ac \uc2e4\ud5d8\ub9cc \uc5f0\uc18d\uc73c\ub85c \uc2e4\uc2dc\ud558\uace0 \uc788\ub2e4. \ubd81\ud55c\uc774 \uac70\uc758 \uc218\uc9c1\uc5d0 \uac00\uae4c\uc6b4 \uace0\uac01 \ubbf8\uc0ac\uc77c \ubc1c\uc0ac \uc2e4\ud5d8\uc5d0 \uc9d1\ucc29\ud558\ub294 \uc774\uc720\ub294 \ubbf8\uc0ac\uc77c\uc744 \uc218\uc9c1\uc73c\ub85c \uc62c\ub824\uc11c \ub099\ud558 \ud560 \ub54c \ubbf8\uc0ac\uc77c \ub099\ud558 \uc18d\ub3c4\uc5d0 \ucd5c\ub300\ud55c \uac00\uc18d\ub3c4\ub97c \ubd99\uc5ec \ub9c8\ud558 10 \uc774\uc0c1\uc758 \uc18d\ub3c4\ub97c \ub0b4\uae30 \uc704\ud574\uc11c \uace0\uac01\ub3c4 \ubc1c\uc0ac \uc2e4\ud5d8\uc744 \ud558\ub294 \uac83\uc774\uace0 \ud328\ud2b8\ub9ac\uc5b4\ud2b8\uc758 \ud55c\uacc4\uac00 \ub9c8\ud558 5 \ub77c\ub294 \uac83\uc744 \ubd81\ud55c\ub3c4 \uc774\ubbf8 \uc798 \uc54c\uace0 \uc788\ub294 \uac83\uc774\ub2e4. \uc774\ub7ec\ud55c \uace0\uac01 \ubc1c\uc0ac\uc5d0 \ub300\uc751 \ud558\uae30 \uc704\ud574\uc11c\ub294 \ub9c8\ud558 14 \uae4c\uc9c0 \uc694\uaca9\uc774 \uac00\ub2a5\ud55c \uc0ac\ub4dc\uac00 \ud544\uc694\ud558\ub2e4.", "answer": "\uc6b0\uc8fc", "sentence": "\uace0\uac01 \ubc1c\uc0ac\ub97c \ud558\uba74 \ubbf8\uc0ac\uc77c\uc740 \uc131\uce35\uad8c\uc744 \uc9c0\ub098 \u2018\ub300\uae30\uad8c \ubc16\u2019\uc758 \uc6b0\uc8fc \uacf5\uac04\uc73c\ub85c \uce58\uc19f\ub294\ub2e4.", "paragraph_sentence": "\ubd81\ud55c\uc740 \ubbf8\uc0ac\uc77c\uc758 \ubc1c\uc0ac \uac01\ub3c4\ub97c \ub192\uc5ec \u2018\uace0\uac01(\u9ad8\u89d2)\u2019\uc73c\ub85c \uc3d8\ub294 \uc2e4\ud5d8\uc744 \ud558\uace0 \uc788\ub2e4. \uc608\ub97c \ub4e4\uc5b4\uc11c \uc27d\uac8c \uc124\uba85\ud558\uc790\uba74 \uacf5\uc744 \uc644\ub9cc\ud55c \ud3ec\ubb3c\uc120\uc744 \uadf8\ub9ac\uac8c \ud558\uba70 \ub358\uc838 \uba40\ub9ac \ub0a0\uc544\uac00\uac8c \ub358\uc9c0\ub294 \uac83\uc774 \uc544\ub2c8\ub77c, \ucd5c\ub300\ud55c \uba38\ub9ac \uc704\ub85c \uacf5\uc744 \ub192\uac8c \ub358\uc838 \uc62c\ub824 \uc790\uae30 \ubc1c \uadfc\ucc98\uc5d0 \ub5a8\uc5b4\uc9c0\uac8c \ud558\ub294 \ubc29\uc2dd\uc774\ub2e4. \uace0\uac01 \ubc1c\uc0ac\ub97c \ud558\uba74 \ubbf8\uc0ac\uc77c\uc740 \uc131\uce35\uad8c\uc744 \uc9c0\ub098 \u2018\ub300\uae30\uad8c \ubc16\u2019\uc758 \uc6b0\uc8fc \uacf5\uac04\uc73c\ub85c \uce58\uc19f\ub294\ub2e4. \uace0\uac01\ub3c4 \ubc1c\uc0ac\ub85c \ubbf8\uc0ac\uc77c\uc774 \uace0\uace0\ub3c4\uae4c\uc9c0 \uce58\uc19f\uc558\ub2e4\uac00 \ub5a8\uc5b4\uc9c8 \ub54c \ubbf8\uc0ac\uc77c\uc758 \uc18d\ub3c4\ub294 \ubcf8\ub798 \uc0ac\uac70\ub9ac\ub97c \ube44\ud589\ud560 \ub54c\ubcf4\ub2e4 \ud6e8\uc52c \ube60\ub974\ub2e4. \ub9cc\uc720\uc778\ub825\uc758 \ubc95\uce59(\uc911\ub825)\uc73c\ub85c \uc778\ud558\uc5ec \ub099\ud558\ud558\ub294 \ubbf8\uc0ac\uc77c\uc5d0 \uac00\uc18d\ub3c4\uac00 \ubd99\uae30 \ub54c\ubb38\uc774\ub2e4. \uc774\ub7ec\ud55c \ub9cc\uc720\uc778\ub825\uc758 \ubc95\uce59\uc744 \uc774\uc6a9\ud55c \uace0\uac01\ub3c4 \ubc1c\uc0ac\ub85c \ubd81\ud55c\uc758 \ubb34\uc218\ub2e8 \ubbf8\uc0ac\uc77c\uc758 \uc18d\ub3c4\ub294 \ub9c8\ud558 10 \uc774\uc0c1\uc758 \uc18d\ub3c4\ub97c \uae30\ub85d\ud588\ub2e4. \ud604\uc7ac \ud55c\uad6d\uc774 \uc8fc\ub825\uc73c\ub85c \uc0ac\uc6a9\ud558\uace0 \uc788\ub294 \ud328\ud2b8\ub9ac\uc5b4\ud2b8 PAC-3 \ub294 \ub9c8\ud558 5 \uc774\uc0c1\uc740 \uc694\uaca9\uc774 \ubd88\uac00\ub2a5\ud558\ub2e4. \ud328\ud2b8\ub9ac\uc5b4\ud2b8\uc758 \ud55c\uacc4\ub294 \ub9c8\ud558 5 \uc774\uae30 \ub54c\ubb38\uc5d0 \ub9c8\ud558 10 \uc758 \uc18d\ub3c4\ub85c \ub0b4\ub824\uaf42\ud788\ub294 \ubb34\uc218\ub2e8 \ubbf8\uc0ac\uc77c\uc740 \uaca9\ucd94\ud560 \uc218 \uc5c6\ub2e4. \uace0\uac01\ub3c4 \ubbf8\uc0ac\uc77c \ubc1c\uc0ac \uc2e4\ud5d8\uc744 \ubc18\ubcf5\uc801\uc73c\ub85c \ud558\uace0 \uc788\ub294 \ubd81\ud55c\uc758 \uc758\ub3c4\ub294 \ud328\ud2b8\ub9ac\uc5b4\ud2b8 PAC-3 \ub4f1\uc73c\ub85c \ubb34\uc7a5\ud55c \ud55c\uad6d\uc758 \ubbf8\uc0ac\uc77c \ubc29\uc5b4\uccb4\uacc4\ub97c \ubb34\ub825\ud654\ud558\uaca0\ub2e4\ub294 \uac83\uc774\ub2e4. \ubc18\uba74\uc5d0, \uc0ac\ub4dc\ub294 \ub9c8\ud558 14 \uae4c\uc9c0 \uc694\uaca9\uc774 \uac00\ub2a5\ud568\uc73c\ub85c \ubb34\uc218\ub2e8\uae09 \uc774\uc0c1\uc758 \ubbf8\uc0ac\uc77c\uc744 \ub9c9\uae30\uc704\ud574\uc11c\ub294 \uc0ac\ub4dc\uac00 \ud544\uc694\ud558\ub2e4. \ubd81\ud55c\uc740 \uac70\uc758 \uc218\uc9c1\uc5d0 \uac00\uae4c\uc6b4 \uace0\uac01\ub3c4 \ubc1c\uc0ac \uc2e4\ud5d8\ub9cc \uc5f0\uc18d\uc73c\ub85c \uc2e4\uc2dc\ud558\uace0 \uc788\ub2e4. \ubd81\ud55c\uc774 \uac70\uc758 \uc218\uc9c1\uc5d0 \uac00\uae4c\uc6b4 \uace0\uac01 \ubbf8\uc0ac\uc77c \ubc1c\uc0ac \uc2e4\ud5d8\uc5d0 \uc9d1\ucc29\ud558\ub294 \uc774\uc720\ub294 \ubbf8\uc0ac\uc77c\uc744 \uc218\uc9c1\uc73c\ub85c \uc62c\ub824\uc11c \ub099\ud558 \ud560 \ub54c \ubbf8\uc0ac\uc77c \ub099\ud558 \uc18d\ub3c4\uc5d0 \ucd5c\ub300\ud55c \uac00\uc18d\ub3c4\ub97c \ubd99\uc5ec \ub9c8\ud558 10 \uc774\uc0c1\uc758 \uc18d\ub3c4\ub97c \ub0b4\uae30 \uc704\ud574\uc11c \uace0\uac01\ub3c4 \ubc1c\uc0ac \uc2e4\ud5d8\uc744 \ud558\ub294 \uac83\uc774\uace0 \ud328\ud2b8\ub9ac\uc5b4\ud2b8\uc758 \ud55c\uacc4\uac00 \ub9c8\ud558 5 \ub77c\ub294 \uac83\uc744 \ubd81\ud55c\ub3c4 \uc774\ubbf8 \uc798 \uc54c\uace0 \uc788\ub294 \uac83\uc774\ub2e4. \uc774\ub7ec\ud55c \uace0\uac01 \ubc1c\uc0ac\uc5d0 \ub300\uc751 \ud558\uae30 \uc704\ud574\uc11c\ub294 \ub9c8\ud558 14 \uae4c\uc9c0 \uc694\uaca9\uc774 \uac00\ub2a5\ud55c \uc0ac\ub4dc\uac00 \ud544\uc694\ud558\ub2e4.", "paragraph_answer": "\ubd81\ud55c\uc740 \ubbf8\uc0ac\uc77c\uc758 \ubc1c\uc0ac \uac01\ub3c4\ub97c \ub192\uc5ec \u2018\uace0\uac01(\u9ad8\u89d2)\u2019\uc73c\ub85c \uc3d8\ub294 \uc2e4\ud5d8\uc744 \ud558\uace0 \uc788\ub2e4. \uc608\ub97c \ub4e4\uc5b4\uc11c \uc27d\uac8c \uc124\uba85\ud558\uc790\uba74 \uacf5\uc744 \uc644\ub9cc\ud55c \ud3ec\ubb3c\uc120\uc744 \uadf8\ub9ac\uac8c \ud558\uba70 \ub358\uc838 \uba40\ub9ac \ub0a0\uc544\uac00\uac8c \ub358\uc9c0\ub294 \uac83\uc774 \uc544\ub2c8\ub77c, \ucd5c\ub300\ud55c \uba38\ub9ac \uc704\ub85c \uacf5\uc744 \ub192\uac8c \ub358\uc838 \uc62c\ub824 \uc790\uae30 \ubc1c \uadfc\ucc98\uc5d0 \ub5a8\uc5b4\uc9c0\uac8c \ud558\ub294 \ubc29\uc2dd\uc774\ub2e4. \uace0\uac01 \ubc1c\uc0ac\ub97c \ud558\uba74 \ubbf8\uc0ac\uc77c\uc740 \uc131\uce35\uad8c\uc744 \uc9c0\ub098 \u2018\ub300\uae30\uad8c \ubc16\u2019\uc758 \uc6b0\uc8fc \uacf5\uac04\uc73c\ub85c \uce58\uc19f\ub294\ub2e4. \uace0\uac01\ub3c4 \ubc1c\uc0ac\ub85c \ubbf8\uc0ac\uc77c\uc774 \uace0\uace0\ub3c4\uae4c\uc9c0 \uce58\uc19f\uc558\ub2e4\uac00 \ub5a8\uc5b4\uc9c8 \ub54c \ubbf8\uc0ac\uc77c\uc758 \uc18d\ub3c4\ub294 \ubcf8\ub798 \uc0ac\uac70\ub9ac\ub97c \ube44\ud589\ud560 \ub54c\ubcf4\ub2e4 \ud6e8\uc52c \ube60\ub974\ub2e4. \ub9cc\uc720\uc778\ub825\uc758 \ubc95\uce59(\uc911\ub825)\uc73c\ub85c \uc778\ud558\uc5ec \ub099\ud558\ud558\ub294 \ubbf8\uc0ac\uc77c\uc5d0 \uac00\uc18d\ub3c4\uac00 \ubd99\uae30 \ub54c\ubb38\uc774\ub2e4. \uc774\ub7ec\ud55c \ub9cc\uc720\uc778\ub825\uc758 \ubc95\uce59\uc744 \uc774\uc6a9\ud55c \uace0\uac01\ub3c4 \ubc1c\uc0ac\ub85c \ubd81\ud55c\uc758 \ubb34\uc218\ub2e8 \ubbf8\uc0ac\uc77c\uc758 \uc18d\ub3c4\ub294 \ub9c8\ud558 10 \uc774\uc0c1\uc758 \uc18d\ub3c4\ub97c \uae30\ub85d\ud588\ub2e4. \ud604\uc7ac \ud55c\uad6d\uc774 \uc8fc\ub825\uc73c\ub85c \uc0ac\uc6a9\ud558\uace0 \uc788\ub294 \ud328\ud2b8\ub9ac\uc5b4\ud2b8 PAC-3 \ub294 \ub9c8\ud558 5 \uc774\uc0c1\uc740 \uc694\uaca9\uc774 \ubd88\uac00\ub2a5\ud558\ub2e4. \ud328\ud2b8\ub9ac\uc5b4\ud2b8\uc758 \ud55c\uacc4\ub294 \ub9c8\ud558 5 \uc774\uae30 \ub54c\ubb38\uc5d0 \ub9c8\ud558 10 \uc758 \uc18d\ub3c4\ub85c \ub0b4\ub824\uaf42\ud788\ub294 \ubb34\uc218\ub2e8 \ubbf8\uc0ac\uc77c\uc740 \uaca9\ucd94\ud560 \uc218 \uc5c6\ub2e4. \uace0\uac01\ub3c4 \ubbf8\uc0ac\uc77c \ubc1c\uc0ac \uc2e4\ud5d8\uc744 \ubc18\ubcf5\uc801\uc73c\ub85c \ud558\uace0 \uc788\ub294 \ubd81\ud55c\uc758 \uc758\ub3c4\ub294 \ud328\ud2b8\ub9ac\uc5b4\ud2b8 PAC-3 \ub4f1\uc73c\ub85c \ubb34\uc7a5\ud55c \ud55c\uad6d\uc758 \ubbf8\uc0ac\uc77c \ubc29\uc5b4\uccb4\uacc4\ub97c \ubb34\ub825\ud654\ud558\uaca0\ub2e4\ub294 \uac83\uc774\ub2e4. \ubc18\uba74\uc5d0, \uc0ac\ub4dc\ub294 \ub9c8\ud558 14 \uae4c\uc9c0 \uc694\uaca9\uc774 \uac00\ub2a5\ud568\uc73c\ub85c \ubb34\uc218\ub2e8\uae09 \uc774\uc0c1\uc758 \ubbf8\uc0ac\uc77c\uc744 \ub9c9\uae30\uc704\ud574\uc11c\ub294 \uc0ac\ub4dc\uac00 \ud544\uc694\ud558\ub2e4. \ubd81\ud55c\uc740 \uac70\uc758 \uc218\uc9c1\uc5d0 \uac00\uae4c\uc6b4 \uace0\uac01\ub3c4 \ubc1c\uc0ac \uc2e4\ud5d8\ub9cc \uc5f0\uc18d\uc73c\ub85c \uc2e4\uc2dc\ud558\uace0 \uc788\ub2e4. \ubd81\ud55c\uc774 \uac70\uc758 \uc218\uc9c1\uc5d0 \uac00\uae4c\uc6b4 \uace0\uac01 \ubbf8\uc0ac\uc77c \ubc1c\uc0ac \uc2e4\ud5d8\uc5d0 \uc9d1\ucc29\ud558\ub294 \uc774\uc720\ub294 \ubbf8\uc0ac\uc77c\uc744 \uc218\uc9c1\uc73c\ub85c \uc62c\ub824\uc11c \ub099\ud558 \ud560 \ub54c \ubbf8\uc0ac\uc77c \ub099\ud558 \uc18d\ub3c4\uc5d0 \ucd5c\ub300\ud55c \uac00\uc18d\ub3c4\ub97c \ubd99\uc5ec \ub9c8\ud558 10 \uc774\uc0c1\uc758 \uc18d\ub3c4\ub97c \ub0b4\uae30 \uc704\ud574\uc11c \uace0\uac01\ub3c4 \ubc1c\uc0ac \uc2e4\ud5d8\uc744 \ud558\ub294 \uac83\uc774\uace0 \ud328\ud2b8\ub9ac\uc5b4\ud2b8\uc758 \ud55c\uacc4\uac00 \ub9c8\ud558 5 \ub77c\ub294 \uac83\uc744 \ubd81\ud55c\ub3c4 \uc774\ubbf8 \uc798 \uc54c\uace0 \uc788\ub294 \uac83\uc774\ub2e4. \uc774\ub7ec\ud55c \uace0\uac01 \ubc1c\uc0ac\uc5d0 \ub300\uc751 \ud558\uae30 \uc704\ud574\uc11c\ub294 \ub9c8\ud558 14 \uae4c\uc9c0 \uc694\uaca9\uc774 \uac00\ub2a5\ud55c \uc0ac\ub4dc\uac00 \ud544\uc694\ud558\ub2e4.", "sentence_answer": "\uace0\uac01 \ubc1c\uc0ac\ub97c \ud558\uba74 \ubbf8\uc0ac\uc77c\uc740 \uc131\uce35\uad8c\uc744 \uc9c0\ub098 \u2018\ub300\uae30\uad8c \ubc16\u2019\uc758 \uc6b0\uc8fc \uacf5\uac04\uc73c\ub85c \uce58\uc19f\ub294\ub2e4.", "paragraph_id": "6486360-8-1", "paragraph_question": "question: \uace0\uac01 \ubc1c\uc0ac\ub41c \ubbf8\uc0ac\uc77c\uc740 \ub300\uae30\uad8c \ubc16 \uc5b4\ub514\uae4c\uc9c0 \uce58\uc19f\uac8c \ub418\ub098?, context: \ubd81\ud55c\uc740 \ubbf8\uc0ac\uc77c\uc758 \ubc1c\uc0ac \uac01\ub3c4\ub97c \ub192\uc5ec \u2018\uace0\uac01(\u9ad8\u89d2)\u2019\uc73c\ub85c \uc3d8\ub294 \uc2e4\ud5d8\uc744 \ud558\uace0 \uc788\ub2e4. \uc608\ub97c \ub4e4\uc5b4\uc11c \uc27d\uac8c \uc124\uba85\ud558\uc790\uba74 \uacf5\uc744 \uc644\ub9cc\ud55c \ud3ec\ubb3c\uc120\uc744 \uadf8\ub9ac\uac8c \ud558\uba70 \ub358\uc838 \uba40\ub9ac \ub0a0\uc544\uac00\uac8c \ub358\uc9c0\ub294 \uac83\uc774 \uc544\ub2c8\ub77c, \ucd5c\ub300\ud55c \uba38\ub9ac \uc704\ub85c \uacf5\uc744 \ub192\uac8c \ub358\uc838 \uc62c\ub824 \uc790\uae30 \ubc1c \uadfc\ucc98\uc5d0 \ub5a8\uc5b4\uc9c0\uac8c \ud558\ub294 \ubc29\uc2dd\uc774\ub2e4. \uace0\uac01 \ubc1c\uc0ac\ub97c \ud558\uba74 \ubbf8\uc0ac\uc77c\uc740 \uc131\uce35\uad8c\uc744 \uc9c0\ub098 \u2018\ub300\uae30\uad8c \ubc16\u2019\uc758 \uc6b0\uc8fc \uacf5\uac04\uc73c\ub85c \uce58\uc19f\ub294\ub2e4. \uace0\uac01\ub3c4 \ubc1c\uc0ac\ub85c \ubbf8\uc0ac\uc77c\uc774 \uace0\uace0\ub3c4\uae4c\uc9c0 \uce58\uc19f\uc558\ub2e4\uac00 \ub5a8\uc5b4\uc9c8 \ub54c \ubbf8\uc0ac\uc77c\uc758 \uc18d\ub3c4\ub294 \ubcf8\ub798 \uc0ac\uac70\ub9ac\ub97c \ube44\ud589\ud560 \ub54c\ubcf4\ub2e4 \ud6e8\uc52c \ube60\ub974\ub2e4. \ub9cc\uc720\uc778\ub825\uc758 \ubc95\uce59(\uc911\ub825)\uc73c\ub85c \uc778\ud558\uc5ec \ub099\ud558\ud558\ub294 \ubbf8\uc0ac\uc77c\uc5d0 \uac00\uc18d\ub3c4\uac00 \ubd99\uae30 \ub54c\ubb38\uc774\ub2e4. \uc774\ub7ec\ud55c \ub9cc\uc720\uc778\ub825\uc758 \ubc95\uce59\uc744 \uc774\uc6a9\ud55c \uace0\uac01\ub3c4 \ubc1c\uc0ac\ub85c \ubd81\ud55c\uc758 \ubb34\uc218\ub2e8 \ubbf8\uc0ac\uc77c\uc758 \uc18d\ub3c4\ub294 \ub9c8\ud558 10 \uc774\uc0c1\uc758 \uc18d\ub3c4\ub97c \uae30\ub85d\ud588\ub2e4. \ud604\uc7ac \ud55c\uad6d\uc774 \uc8fc\ub825\uc73c\ub85c \uc0ac\uc6a9\ud558\uace0 \uc788\ub294 \ud328\ud2b8\ub9ac\uc5b4\ud2b8 PAC-3 \ub294 \ub9c8\ud558 5 \uc774\uc0c1\uc740 \uc694\uaca9\uc774 \ubd88\uac00\ub2a5\ud558\ub2e4. \ud328\ud2b8\ub9ac\uc5b4\ud2b8\uc758 \ud55c\uacc4\ub294 \ub9c8\ud558 5 \uc774\uae30 \ub54c\ubb38\uc5d0 \ub9c8\ud558 10 \uc758 \uc18d\ub3c4\ub85c \ub0b4\ub824\uaf42\ud788\ub294 \ubb34\uc218\ub2e8 \ubbf8\uc0ac\uc77c\uc740 \uaca9\ucd94\ud560 \uc218 \uc5c6\ub2e4. \uace0\uac01\ub3c4 \ubbf8\uc0ac\uc77c \ubc1c\uc0ac \uc2e4\ud5d8\uc744 \ubc18\ubcf5\uc801\uc73c\ub85c \ud558\uace0 \uc788\ub294 \ubd81\ud55c\uc758 \uc758\ub3c4\ub294 \ud328\ud2b8\ub9ac\uc5b4\ud2b8 PAC-3 \ub4f1\uc73c\ub85c \ubb34\uc7a5\ud55c \ud55c\uad6d\uc758 \ubbf8\uc0ac\uc77c \ubc29\uc5b4\uccb4\uacc4\ub97c \ubb34\ub825\ud654\ud558\uaca0\ub2e4\ub294 \uac83\uc774\ub2e4. \ubc18\uba74\uc5d0, \uc0ac\ub4dc\ub294 \ub9c8\ud558 14 \uae4c\uc9c0 \uc694\uaca9\uc774 \uac00\ub2a5\ud568\uc73c\ub85c \ubb34\uc218\ub2e8\uae09 \uc774\uc0c1\uc758 \ubbf8\uc0ac\uc77c\uc744 \ub9c9\uae30\uc704\ud574\uc11c\ub294 \uc0ac\ub4dc\uac00 \ud544\uc694\ud558\ub2e4. \ubd81\ud55c\uc740 \uac70\uc758 \uc218\uc9c1\uc5d0 \uac00\uae4c\uc6b4 \uace0\uac01\ub3c4 \ubc1c\uc0ac \uc2e4\ud5d8\ub9cc \uc5f0\uc18d\uc73c\ub85c \uc2e4\uc2dc\ud558\uace0 \uc788\ub2e4. \ubd81\ud55c\uc774 \uac70\uc758 \uc218\uc9c1\uc5d0 \uac00\uae4c\uc6b4 \uace0\uac01 \ubbf8\uc0ac\uc77c \ubc1c\uc0ac \uc2e4\ud5d8\uc5d0 \uc9d1\ucc29\ud558\ub294 \uc774\uc720\ub294 \ubbf8\uc0ac\uc77c\uc744 \uc218\uc9c1\uc73c\ub85c \uc62c\ub824\uc11c \ub099\ud558 \ud560 \ub54c \ubbf8\uc0ac\uc77c \ub099\ud558 \uc18d\ub3c4\uc5d0 \ucd5c\ub300\ud55c \uac00\uc18d\ub3c4\ub97c \ubd99\uc5ec \ub9c8\ud558 10 \uc774\uc0c1\uc758 \uc18d\ub3c4\ub97c \ub0b4\uae30 \uc704\ud574\uc11c \uace0\uac01\ub3c4 \ubc1c\uc0ac \uc2e4\ud5d8\uc744 \ud558\ub294 \uac83\uc774\uace0 \ud328\ud2b8\ub9ac\uc5b4\ud2b8\uc758 \ud55c\uacc4\uac00 \ub9c8\ud558 5 \ub77c\ub294 \uac83\uc744 \ubd81\ud55c\ub3c4 \uc774\ubbf8 \uc798 \uc54c\uace0 \uc788\ub294 \uac83\uc774\ub2e4. \uc774\ub7ec\ud55c \uace0\uac01 \ubc1c\uc0ac\uc5d0 \ub300\uc751 \ud558\uae30 \uc704\ud574\uc11c\ub294 \ub9c8\ud558 14 \uae4c\uc9c0 \uc694\uaca9\uc774 \uac00\ub2a5\ud55c \uc0ac\ub4dc\uac00 \ud544\uc694\ud558\ub2e4."} {"question": "1965\ub144 \ub9c1\uace0\uc2a4\ud0c0\uac00 \ubaa8\ub9b0 \ucf55\uc2a4\uc640 \uacb0\ud63c \ud560 \ub2f9\uc2dc \uadf8\uc5d0\uac8c \ucd5c\ub300\uc758 \uc2a4\ud2b8\ub808\uc2a4\uc640 \uc555\ubc15\uac10\uc744 \uc2ec\uc5b4\uc900 \uac83\uc740?", "paragraph": "1965\ub144 2\uc6d4 11\uc77c \uc2a4\ud0c0\ub294 1962\ub144 \uccab \ub9cc\ub0a8\uc744 \uac00\uc84c\ub358 \ubaa8\ub9b0 \ucf55\uc2a4\uc640 \uacb0\ud63c\ud588\ub2e4. \uc774\ub54c \ube44\ud2c0\ub9c8\ub2c8\uc544\ub294 \uadf8\uc5d0\uac8c \uc0ac\uc0c1 \ucd5c\ub300\uc758 \uc2a4\ud2b8\ub808\uc2a4 \ubc0f \uc555\ubc15\uac10\uc744 \uc2ec\uc5b4\uc92c\ub2e4. \uadf8\ub294 \ubaac\ud2b8\ub9ac\uc62c \uacf5\uc5f0 \uc804 \uc804\ud654\ub85c \uc0b4\uc778\ud611\ubc15\uc744 \ubc1b\uc558\uace0 \uadf8\ub294 \uc790\uae30 \uc2ec\ubc8c\uc988\ub97c \uc138\ub85c\ub85c \uc138\uc6cc \uc554\uc0b4\uc704\ud611\uc5d0\uc11c \ubc29\uc5b4\ud560 \uc6a9\ub3c4\ub85c \ud65c\uc6a9\ud588\ub2e4. \ube44\ud2c0\uc988\uc758 \uba85\uc131\uc5d0\uc11c \ub9d0\ubbf8\uc554\uc740 \uc77c\uad00\ub41c \uc555\ubc15\uc740 \uc774\ub4e4\uc758 \ub77c\uc774\ube0c \uacf5\uc5f0\uc5d0\ub3c4 \uc601\ud5a5\uc744 \ubbf8\ucce4\ub2e4. \"\uc6b0\ub9ac\ub294 \uc810\ucc28 \ub098\uc05c \uc74c\uc545\uac00\ub85c \ubcc0\ubaa8\ud588\uc2b5\ub2c8\ub2e4 ... \uc7a0\uc2dc\ub3c4 \uc274 \uc218 \uc5c6\uc5c8\uc2b5\ub2c8\ub2e4.\"\ub77c\uace0 \uc2a4\ud0c0\ub294 \ub9d0\ud588\ub2e4. \uc544\uc6b8\ub7ec \ub2e4\ub978 \ubc34\ub4dc\uc6d0\uc73c\ub85c\ubd80\ud130 \uc18c\uc678\ub41c\ub2e4\ub294 \ub290\ub08c\uc744 \uac08\uc218\ub85d \ub9ce\uc774 \ub290\ub07c\uac8c \ub418\uc5c8\ub2e4. \uadf8\ub4e4\uc740 \ub2f9\uc2dc \uc2a4\ud0c0\uc758 \uc5f0\uc8fc\uac00 \ud544\uc694\ud558\uc9c0 \uc54a\uc740 \ub85d \uc74c\uc545\uc758 \uc804\ud1b5\uc801 \uacbd\uacc4\ub97c \ub118\ub098\ub4e4\uace0 \uc788\uc5c8\ub2e4. \ub179\uc74c \uc138\uc158\uc5d0\uc11c, \uc2a4\ud0c0\ub294 \ub2e4\ub978 \ubc34\ub4dc\uc6d0\uc774 \uc790\uae30 \uc5c6\uc774 \uc644\ubcbd\ud55c \ud2b8\ub799\uc744 \uc644\uc131\ud560 \uc218\uc2dc\uac04 \ub3d9\uc548 \ub85c\ub4dc \ub9e4\ub2c8\uc800 \ub2d0 \uc560\uc2a4\ud53c\ub110\uacfc \uacf5\uc5f0 \ub9e4\ub2c8\uc800 \ub9ec \uc5d0\ubc18\uc2a4\uc640 \uce74\ud2b8 \uac8c\uc784\uc744 \ud588\ub2e4. \u300a\uba5c\ub85c\ub514 \uba54\uc774\ucee4\u300b\uac00 \ucd9c\uac04\ud55c \ud32c \ud3b8\uc9c0\uc5d0\uc11c \ud55c \ud32c\uc740 \ube44\ud2c0\uc988\uc5d0\uac8c \uc2a4\ud0c0\uac00 \ub354 \ub9ce\uc740 \uace1\uc744 \ubd80\ub974\uac8c \ud574\ub2ec\ub77c\uace0 \uc694\uad6c\ud588\ub2e4. \uc2a4\ud0c0\ub294 \"\ub9e4 \uc568\ubc94\ub9c8\ub2e4 \ud55c \uace1\uc744 \ubd80\ub97c \uc218 \uc788\uc5b4 \uaf64 \ud589\ubcf5\ud588\ub2e4.\"\uace0 \ub2f5\ud588\ub2e4.", "answer": "\ube44\ud2c0\ub9c8\ub2c8\uc544", "sentence": "\uc774\ub54c \ube44\ud2c0\ub9c8\ub2c8\uc544 \ub294 \uadf8\uc5d0\uac8c \uc0ac\uc0c1 \ucd5c\ub300\uc758 \uc2a4\ud2b8\ub808\uc2a4 \ubc0f \uc555\ubc15\uac10\uc744 \uc2ec\uc5b4\uc92c\ub2e4.", "paragraph_sentence": "1965\ub144 2\uc6d4 11\uc77c \uc2a4\ud0c0\ub294 1962\ub144 \uccab \ub9cc\ub0a8\uc744 \uac00\uc84c\ub358 \ubaa8\ub9b0 \ucf55\uc2a4\uc640 \uacb0\ud63c\ud588\ub2e4. \uc774\ub54c \ube44\ud2c0\ub9c8\ub2c8\uc544 \ub294 \uadf8\uc5d0\uac8c \uc0ac\uc0c1 \ucd5c\ub300\uc758 \uc2a4\ud2b8\ub808\uc2a4 \ubc0f \uc555\ubc15\uac10\uc744 \uc2ec\uc5b4\uc92c\ub2e4. \uadf8\ub294 \ubaac\ud2b8\ub9ac\uc62c \uacf5\uc5f0 \uc804 \uc804\ud654\ub85c \uc0b4\uc778\ud611\ubc15\uc744 \ubc1b\uc558\uace0 \uadf8\ub294 \uc790\uae30 \uc2ec\ubc8c\uc988\ub97c \uc138\ub85c\ub85c \uc138\uc6cc \uc554\uc0b4\uc704\ud611\uc5d0\uc11c \ubc29\uc5b4\ud560 \uc6a9\ub3c4\ub85c \ud65c\uc6a9\ud588\ub2e4. \ube44\ud2c0\uc988\uc758 \uba85\uc131\uc5d0\uc11c \ub9d0\ubbf8\uc554\uc740 \uc77c\uad00\ub41c \uc555\ubc15\uc740 \uc774\ub4e4\uc758 \ub77c\uc774\ube0c \uacf5\uc5f0\uc5d0\ub3c4 \uc601\ud5a5\uc744 \ubbf8\ucce4\ub2e4. \"\uc6b0\ub9ac\ub294 \uc810\ucc28 \ub098\uc05c \uc74c\uc545\uac00\ub85c \ubcc0\ubaa8\ud588\uc2b5\ub2c8\ub2e4 ... \uc7a0\uc2dc\ub3c4 \uc274 \uc218 \uc5c6\uc5c8\uc2b5\ub2c8\ub2e4. \"\ub77c\uace0 \uc2a4\ud0c0\ub294 \ub9d0\ud588\ub2e4. \uc544\uc6b8\ub7ec \ub2e4\ub978 \ubc34\ub4dc\uc6d0\uc73c\ub85c\ubd80\ud130 \uc18c\uc678\ub41c\ub2e4\ub294 \ub290\ub08c\uc744 \uac08\uc218\ub85d \ub9ce\uc774 \ub290\ub07c\uac8c \ub418\uc5c8\ub2e4. \uadf8\ub4e4\uc740 \ub2f9\uc2dc \uc2a4\ud0c0\uc758 \uc5f0\uc8fc\uac00 \ud544\uc694\ud558\uc9c0 \uc54a\uc740 \ub85d \uc74c\uc545\uc758 \uc804\ud1b5\uc801 \uacbd\uacc4\ub97c \ub118\ub098\ub4e4\uace0 \uc788\uc5c8\ub2e4. \ub179\uc74c \uc138\uc158\uc5d0\uc11c, \uc2a4\ud0c0\ub294 \ub2e4\ub978 \ubc34\ub4dc\uc6d0\uc774 \uc790\uae30 \uc5c6\uc774 \uc644\ubcbd\ud55c \ud2b8\ub799\uc744 \uc644\uc131\ud560 \uc218\uc2dc\uac04 \ub3d9\uc548 \ub85c\ub4dc \ub9e4\ub2c8\uc800 \ub2d0 \uc560\uc2a4\ud53c\ub110\uacfc \uacf5\uc5f0 \ub9e4\ub2c8\uc800 \ub9ec \uc5d0\ubc18\uc2a4\uc640 \uce74\ud2b8 \uac8c\uc784\uc744 \ud588\ub2e4. \u300a\uba5c\ub85c\ub514 \uba54\uc774\ucee4\u300b\uac00 \ucd9c\uac04\ud55c \ud32c \ud3b8\uc9c0\uc5d0\uc11c \ud55c \ud32c\uc740 \ube44\ud2c0\uc988\uc5d0\uac8c \uc2a4\ud0c0\uac00 \ub354 \ub9ce\uc740 \uace1\uc744 \ubd80\ub974\uac8c \ud574\ub2ec\ub77c\uace0 \uc694\uad6c\ud588\ub2e4. \uc2a4\ud0c0\ub294 \"\ub9e4 \uc568\ubc94\ub9c8\ub2e4 \ud55c \uace1\uc744 \ubd80\ub97c \uc218 \uc788\uc5b4 \uaf64 \ud589\ubcf5\ud588\ub2e4. \"\uace0 \ub2f5\ud588\ub2e4.", "paragraph_answer": "1965\ub144 2\uc6d4 11\uc77c \uc2a4\ud0c0\ub294 1962\ub144 \uccab \ub9cc\ub0a8\uc744 \uac00\uc84c\ub358 \ubaa8\ub9b0 \ucf55\uc2a4\uc640 \uacb0\ud63c\ud588\ub2e4. \uc774\ub54c \ube44\ud2c0\ub9c8\ub2c8\uc544 \ub294 \uadf8\uc5d0\uac8c \uc0ac\uc0c1 \ucd5c\ub300\uc758 \uc2a4\ud2b8\ub808\uc2a4 \ubc0f \uc555\ubc15\uac10\uc744 \uc2ec\uc5b4\uc92c\ub2e4. \uadf8\ub294 \ubaac\ud2b8\ub9ac\uc62c \uacf5\uc5f0 \uc804 \uc804\ud654\ub85c \uc0b4\uc778\ud611\ubc15\uc744 \ubc1b\uc558\uace0 \uadf8\ub294 \uc790\uae30 \uc2ec\ubc8c\uc988\ub97c \uc138\ub85c\ub85c \uc138\uc6cc \uc554\uc0b4\uc704\ud611\uc5d0\uc11c \ubc29\uc5b4\ud560 \uc6a9\ub3c4\ub85c \ud65c\uc6a9\ud588\ub2e4. \ube44\ud2c0\uc988\uc758 \uba85\uc131\uc5d0\uc11c \ub9d0\ubbf8\uc554\uc740 \uc77c\uad00\ub41c \uc555\ubc15\uc740 \uc774\ub4e4\uc758 \ub77c\uc774\ube0c \uacf5\uc5f0\uc5d0\ub3c4 \uc601\ud5a5\uc744 \ubbf8\ucce4\ub2e4. \"\uc6b0\ub9ac\ub294 \uc810\ucc28 \ub098\uc05c \uc74c\uc545\uac00\ub85c \ubcc0\ubaa8\ud588\uc2b5\ub2c8\ub2e4 ... \uc7a0\uc2dc\ub3c4 \uc274 \uc218 \uc5c6\uc5c8\uc2b5\ub2c8\ub2e4.\"\ub77c\uace0 \uc2a4\ud0c0\ub294 \ub9d0\ud588\ub2e4. \uc544\uc6b8\ub7ec \ub2e4\ub978 \ubc34\ub4dc\uc6d0\uc73c\ub85c\ubd80\ud130 \uc18c\uc678\ub41c\ub2e4\ub294 \ub290\ub08c\uc744 \uac08\uc218\ub85d \ub9ce\uc774 \ub290\ub07c\uac8c \ub418\uc5c8\ub2e4. \uadf8\ub4e4\uc740 \ub2f9\uc2dc \uc2a4\ud0c0\uc758 \uc5f0\uc8fc\uac00 \ud544\uc694\ud558\uc9c0 \uc54a\uc740 \ub85d \uc74c\uc545\uc758 \uc804\ud1b5\uc801 \uacbd\uacc4\ub97c \ub118\ub098\ub4e4\uace0 \uc788\uc5c8\ub2e4. \ub179\uc74c \uc138\uc158\uc5d0\uc11c, \uc2a4\ud0c0\ub294 \ub2e4\ub978 \ubc34\ub4dc\uc6d0\uc774 \uc790\uae30 \uc5c6\uc774 \uc644\ubcbd\ud55c \ud2b8\ub799\uc744 \uc644\uc131\ud560 \uc218\uc2dc\uac04 \ub3d9\uc548 \ub85c\ub4dc \ub9e4\ub2c8\uc800 \ub2d0 \uc560\uc2a4\ud53c\ub110\uacfc \uacf5\uc5f0 \ub9e4\ub2c8\uc800 \ub9ec \uc5d0\ubc18\uc2a4\uc640 \uce74\ud2b8 \uac8c\uc784\uc744 \ud588\ub2e4. \u300a\uba5c\ub85c\ub514 \uba54\uc774\ucee4\u300b\uac00 \ucd9c\uac04\ud55c \ud32c \ud3b8\uc9c0\uc5d0\uc11c \ud55c \ud32c\uc740 \ube44\ud2c0\uc988\uc5d0\uac8c \uc2a4\ud0c0\uac00 \ub354 \ub9ce\uc740 \uace1\uc744 \ubd80\ub974\uac8c \ud574\ub2ec\ub77c\uace0 \uc694\uad6c\ud588\ub2e4. \uc2a4\ud0c0\ub294 \"\ub9e4 \uc568\ubc94\ub9c8\ub2e4 \ud55c \uace1\uc744 \ubd80\ub97c \uc218 \uc788\uc5b4 \uaf64 \ud589\ubcf5\ud588\ub2e4.\"\uace0 \ub2f5\ud588\ub2e4.", "sentence_answer": "\uc774\ub54c \ube44\ud2c0\ub9c8\ub2c8\uc544 \ub294 \uadf8\uc5d0\uac8c \uc0ac\uc0c1 \ucd5c\ub300\uc758 \uc2a4\ud2b8\ub808\uc2a4 \ubc0f \uc555\ubc15\uac10\uc744 \uc2ec\uc5b4\uc92c\ub2e4.", "paragraph_id": "6546611-14-1", "paragraph_question": "question: 1965\ub144 \ub9c1\uace0\uc2a4\ud0c0\uac00 \ubaa8\ub9b0 \ucf55\uc2a4\uc640 \uacb0\ud63c \ud560 \ub2f9\uc2dc \uadf8\uc5d0\uac8c \ucd5c\ub300\uc758 \uc2a4\ud2b8\ub808\uc2a4\uc640 \uc555\ubc15\uac10\uc744 \uc2ec\uc5b4\uc900 \uac83\uc740?, context: 1965\ub144 2\uc6d4 11\uc77c \uc2a4\ud0c0\ub294 1962\ub144 \uccab \ub9cc\ub0a8\uc744 \uac00\uc84c\ub358 \ubaa8\ub9b0 \ucf55\uc2a4\uc640 \uacb0\ud63c\ud588\ub2e4. \uc774\ub54c \ube44\ud2c0\ub9c8\ub2c8\uc544\ub294 \uadf8\uc5d0\uac8c \uc0ac\uc0c1 \ucd5c\ub300\uc758 \uc2a4\ud2b8\ub808\uc2a4 \ubc0f \uc555\ubc15\uac10\uc744 \uc2ec\uc5b4\uc92c\ub2e4. \uadf8\ub294 \ubaac\ud2b8\ub9ac\uc62c \uacf5\uc5f0 \uc804 \uc804\ud654\ub85c \uc0b4\uc778\ud611\ubc15\uc744 \ubc1b\uc558\uace0 \uadf8\ub294 \uc790\uae30 \uc2ec\ubc8c\uc988\ub97c \uc138\ub85c\ub85c \uc138\uc6cc \uc554\uc0b4\uc704\ud611\uc5d0\uc11c \ubc29\uc5b4\ud560 \uc6a9\ub3c4\ub85c \ud65c\uc6a9\ud588\ub2e4. \ube44\ud2c0\uc988\uc758 \uba85\uc131\uc5d0\uc11c \ub9d0\ubbf8\uc554\uc740 \uc77c\uad00\ub41c \uc555\ubc15\uc740 \uc774\ub4e4\uc758 \ub77c\uc774\ube0c \uacf5\uc5f0\uc5d0\ub3c4 \uc601\ud5a5\uc744 \ubbf8\ucce4\ub2e4. \"\uc6b0\ub9ac\ub294 \uc810\ucc28 \ub098\uc05c \uc74c\uc545\uac00\ub85c \ubcc0\ubaa8\ud588\uc2b5\ub2c8\ub2e4 ... \uc7a0\uc2dc\ub3c4 \uc274 \uc218 \uc5c6\uc5c8\uc2b5\ub2c8\ub2e4.\"\ub77c\uace0 \uc2a4\ud0c0\ub294 \ub9d0\ud588\ub2e4. \uc544\uc6b8\ub7ec \ub2e4\ub978 \ubc34\ub4dc\uc6d0\uc73c\ub85c\ubd80\ud130 \uc18c\uc678\ub41c\ub2e4\ub294 \ub290\ub08c\uc744 \uac08\uc218\ub85d \ub9ce\uc774 \ub290\ub07c\uac8c \ub418\uc5c8\ub2e4. \uadf8\ub4e4\uc740 \ub2f9\uc2dc \uc2a4\ud0c0\uc758 \uc5f0\uc8fc\uac00 \ud544\uc694\ud558\uc9c0 \uc54a\uc740 \ub85d \uc74c\uc545\uc758 \uc804\ud1b5\uc801 \uacbd\uacc4\ub97c \ub118\ub098\ub4e4\uace0 \uc788\uc5c8\ub2e4. \ub179\uc74c \uc138\uc158\uc5d0\uc11c, \uc2a4\ud0c0\ub294 \ub2e4\ub978 \ubc34\ub4dc\uc6d0\uc774 \uc790\uae30 \uc5c6\uc774 \uc644\ubcbd\ud55c \ud2b8\ub799\uc744 \uc644\uc131\ud560 \uc218\uc2dc\uac04 \ub3d9\uc548 \ub85c\ub4dc \ub9e4\ub2c8\uc800 \ub2d0 \uc560\uc2a4\ud53c\ub110\uacfc \uacf5\uc5f0 \ub9e4\ub2c8\uc800 \ub9ec \uc5d0\ubc18\uc2a4\uc640 \uce74\ud2b8 \uac8c\uc784\uc744 \ud588\ub2e4. \u300a\uba5c\ub85c\ub514 \uba54\uc774\ucee4\u300b\uac00 \ucd9c\uac04\ud55c \ud32c \ud3b8\uc9c0\uc5d0\uc11c \ud55c \ud32c\uc740 \ube44\ud2c0\uc988\uc5d0\uac8c \uc2a4\ud0c0\uac00 \ub354 \ub9ce\uc740 \uace1\uc744 \ubd80\ub974\uac8c \ud574\ub2ec\ub77c\uace0 \uc694\uad6c\ud588\ub2e4. \uc2a4\ud0c0\ub294 \"\ub9e4 \uc568\ubc94\ub9c8\ub2e4 \ud55c \uace1\uc744 \ubd80\ub97c \uc218 \uc788\uc5b4 \uaf64 \ud589\ubcf5\ud588\ub2e4.\"\uace0 \ub2f5\ud588\ub2e4."} {"question": "\ud310\ubb38\uc810 \ub3c4\ub07c \uc0b4\uc778 \uc0ac\uac74\uc73c\ub85c \uc0ac\ub9dd\ud55c \uc720\uc5d4\uad70 \uce21 \uacbd\ube44\uc911\ub300\uc7a5\uc740?", "paragraph": "\uc870\uc120\uc778\ubbfc\uad70 \ubc15\ucca0 \uc911\uc704\uc640 \ub2e4\ub978 \uc7a5\uad50 1\uba85, \uadf8\ub9ac\uace0 15\uba85\uc758 \ubd80\uc0ac\uad00\uacfc \ubcd1\uc774 \ub098\ud0c0\ub098 \uc791\uc5c5 \uc911\uc9c0\ub97c \uc694\uad6c\ud558\uc600\ub2e4. \ubbf8\ub8e8\ub098\ubb34\uc758 \uc704\uce58\uac00 \uc720\uc5d4\uad70 \uce21\uc758 \uad00\ud560\uc5d0 \uc18d\ud588\uae30\uc5d0 \ubcf4\uc218\uc791\uc5c5\uc744 \ud558\ub294\ub370 \ubb38\uc81c\uac00 \uc5c6\ub2e4\uace0 \ud310\ub2e8\ud55c \ubcf4\ub2c8\ud30c\uc2a4 \ub300\uc704\ub294 \uacbd\ube44\uc911\ub300\uc7a5 \uc9c1\uad8c\uc73c\ub85c \uc791\uc5c5\uc744 \uacc4\uc18d \uc9c0\uc2dc\ud558\uc600\ub2e4. \uc778\uadfc \ucd08\uc18c\uc758 \uc778\ubbfc\uad70 \ubd80\uc0ac\uad00\uacfc \ubcd1 20\uc5ec\uba85\uc740 \uacbd\ube44 \ubcd1\ub825\uc744 \uc694\uccad\ubc1b\uace0 \ud2b8\ub7ed\uc744 \uc774\uc6a9\ud558\uc5ec \ub3c4\ucc29\ud558\uc600\ub2e4. \ubc15\ucca0 \uc911\uc704\uc758 \uc791\uc5c5 \uc911\uc9c0 \uc7ac\uc694\uad6c\ub97c \ubcf4\ub2c8\ud30c\uc2a4 \ub300\uc704\uac00 \uac70\ubd80\ud558\uc790 \ubc15\ucca0\uc758 \uacf5\uaca9\uba85\ub839\uc5d0 \ub530\ub77c \uc778\ubbfc\uad70 \ubd80\uc0ac\uad00\uacfc \ubcd1\ub4e4\uc740 \ud2b8\ub7ed\uc5d0 \uc2e4\uc5b4 \uac00\uc9c0\uace0 \uc628 \uace1\uad2d\uc774, \ubabd\ub465\uc774\uc640 \ud568\uaed8 \ub178\ub3d9\uc790\ub4e4\uc774 \uc791\uc5c5\uc5d0 \uc4f0\ub824\uace0 \uac00\uc838\uc654\ub358 \ub3c4\ub07c \ub4f1\uc744 \ube7c\uc557\uc544 \ud718\ub450\ub974\uba70 \uae30\uc2b5\ud558\uc600\ub2e4. \uc774\ub4e4\uc740 \uc720\uc5d4\uad70\uce21 \uc9c0\ud718\uad00\uacfc \uc7a5\ubcd1\ub4e4\uc5d0\uac8c \uc9d1\uc911 \uacf5\uaca9\uc744 \uac00\ud558\uc5ec \uacbd\ube44\uc911\ub300\uc7a5 \uc544\uc11c \ubcf4\ub2c8\ud30c\uc2a4 \ub300\uc704\uc640 \ubc30\ub7ff \uc911\uc704\uac00 \uc774\ub9c8\uc5d0 \uc911\uc0c1\uc744 \uc785\uace0 \uc774\uc1a1 \uc911 \uc0ac\ub9dd\ud558\uc600\uc73c\uba70, \uc8fc\ud55c \ubbf8\uad70 \ubd80\uc0ac\uad00\uacfc \ubcd1 4\uba85, \uad6d\uad70 \uc7a5\uad50\uc640 \ubd80\uc0ac\uad00\uacfc \ubcd1 4\uba85 \ub4f1\uc774 \uc911\uacbd\uc0c1\uc744 \uc785\uc5c8\uace0, \uc720\uc5d4\uad70 \ud2b8\ub7ed 3\ub300\uac00 \ud30c\uc190\ub418\uc5c8\ub2e4.", "answer": "\uc544\uc11c \ubcf4\ub2c8\ud30c\uc2a4 \ub300\uc704", "sentence": "\uc774\ub4e4\uc740 \uc720\uc5d4\uad70\uce21 \uc9c0\ud718\uad00\uacfc \uc7a5\ubcd1\ub4e4\uc5d0\uac8c \uc9d1\uc911 \uacf5\uaca9\uc744 \uac00\ud558\uc5ec \uacbd\ube44\uc911\ub300\uc7a5 \uc544\uc11c \ubcf4\ub2c8\ud30c\uc2a4 \ub300\uc704 \uc640 \ubc30\ub7ff \uc911\uc704\uac00 \uc774\ub9c8\uc5d0 \uc911\uc0c1\uc744 \uc785\uace0 \uc774\uc1a1 \uc911 \uc0ac\ub9dd\ud558\uc600\uc73c\uba70, \uc8fc\ud55c \ubbf8\uad70 \ubd80\uc0ac\uad00\uacfc \ubcd1 4\uba85, \uad6d\uad70 \uc7a5\uad50\uc640 \ubd80\uc0ac\uad00\uacfc \ubcd1 4\uba85 \ub4f1\uc774 \uc911\uacbd\uc0c1\uc744 \uc785\uc5c8\uace0, \uc720\uc5d4\uad70 \ud2b8\ub7ed 3\ub300\uac00 \ud30c\uc190\ub418\uc5c8\ub2e4.", "paragraph_sentence": "\uc870\uc120\uc778\ubbfc\uad70 \ubc15\ucca0 \uc911\uc704\uc640 \ub2e4\ub978 \uc7a5\uad50 1\uba85, \uadf8\ub9ac\uace0 15\uba85\uc758 \ubd80\uc0ac\uad00\uacfc \ubcd1\uc774 \ub098\ud0c0\ub098 \uc791\uc5c5 \uc911\uc9c0\ub97c \uc694\uad6c\ud558\uc600\ub2e4. \ubbf8\ub8e8\ub098\ubb34\uc758 \uc704\uce58\uac00 \uc720\uc5d4\uad70 \uce21\uc758 \uad00\ud560\uc5d0 \uc18d\ud588\uae30\uc5d0 \ubcf4\uc218\uc791\uc5c5\uc744 \ud558\ub294\ub370 \ubb38\uc81c\uac00 \uc5c6\ub2e4\uace0 \ud310\ub2e8\ud55c \ubcf4\ub2c8\ud30c\uc2a4 \ub300\uc704\ub294 \uacbd\ube44\uc911\ub300\uc7a5 \uc9c1\uad8c\uc73c\ub85c \uc791\uc5c5\uc744 \uacc4\uc18d \uc9c0\uc2dc\ud558\uc600\ub2e4. \uc778\uadfc \ucd08\uc18c\uc758 \uc778\ubbfc\uad70 \ubd80\uc0ac\uad00\uacfc \ubcd1 20\uc5ec\uba85\uc740 \uacbd\ube44 \ubcd1\ub825\uc744 \uc694\uccad\ubc1b\uace0 \ud2b8\ub7ed\uc744 \uc774\uc6a9\ud558\uc5ec \ub3c4\ucc29\ud558\uc600\ub2e4. \ubc15\ucca0 \uc911\uc704\uc758 \uc791\uc5c5 \uc911\uc9c0 \uc7ac\uc694\uad6c\ub97c \ubcf4\ub2c8\ud30c\uc2a4 \ub300\uc704\uac00 \uac70\ubd80\ud558\uc790 \ubc15\ucca0\uc758 \uacf5\uaca9\uba85\ub839\uc5d0 \ub530\ub77c \uc778\ubbfc\uad70 \ubd80\uc0ac\uad00\uacfc \ubcd1\ub4e4\uc740 \ud2b8\ub7ed\uc5d0 \uc2e4\uc5b4 \uac00\uc9c0\uace0 \uc628 \uace1\uad2d\uc774, \ubabd\ub465\uc774\uc640 \ud568\uaed8 \ub178\ub3d9\uc790\ub4e4\uc774 \uc791\uc5c5\uc5d0 \uc4f0\ub824\uace0 \uac00\uc838\uc654\ub358 \ub3c4\ub07c \ub4f1\uc744 \ube7c\uc557\uc544 \ud718\ub450\ub974\uba70 \uae30\uc2b5\ud558\uc600\ub2e4. \uc774\ub4e4\uc740 \uc720\uc5d4\uad70\uce21 \uc9c0\ud718\uad00\uacfc \uc7a5\ubcd1\ub4e4\uc5d0\uac8c \uc9d1\uc911 \uacf5\uaca9\uc744 \uac00\ud558\uc5ec \uacbd\ube44\uc911\ub300\uc7a5 \uc544\uc11c \ubcf4\ub2c8\ud30c\uc2a4 \ub300\uc704 \uc640 \ubc30\ub7ff \uc911\uc704\uac00 \uc774\ub9c8\uc5d0 \uc911\uc0c1\uc744 \uc785\uace0 \uc774\uc1a1 \uc911 \uc0ac\ub9dd\ud558\uc600\uc73c\uba70, \uc8fc\ud55c \ubbf8\uad70 \ubd80\uc0ac\uad00\uacfc \ubcd1 4\uba85, \uad6d\uad70 \uc7a5\uad50\uc640 \ubd80\uc0ac\uad00\uacfc \ubcd1 4\uba85 \ub4f1\uc774 \uc911\uacbd\uc0c1\uc744 \uc785\uc5c8\uace0, \uc720\uc5d4\uad70 \ud2b8\ub7ed 3\ub300\uac00 \ud30c\uc190\ub418\uc5c8\ub2e4. ", "paragraph_answer": "\uc870\uc120\uc778\ubbfc\uad70 \ubc15\ucca0 \uc911\uc704\uc640 \ub2e4\ub978 \uc7a5\uad50 1\uba85, \uadf8\ub9ac\uace0 15\uba85\uc758 \ubd80\uc0ac\uad00\uacfc \ubcd1\uc774 \ub098\ud0c0\ub098 \uc791\uc5c5 \uc911\uc9c0\ub97c \uc694\uad6c\ud558\uc600\ub2e4. \ubbf8\ub8e8\ub098\ubb34\uc758 \uc704\uce58\uac00 \uc720\uc5d4\uad70 \uce21\uc758 \uad00\ud560\uc5d0 \uc18d\ud588\uae30\uc5d0 \ubcf4\uc218\uc791\uc5c5\uc744 \ud558\ub294\ub370 \ubb38\uc81c\uac00 \uc5c6\ub2e4\uace0 \ud310\ub2e8\ud55c \ubcf4\ub2c8\ud30c\uc2a4 \ub300\uc704\ub294 \uacbd\ube44\uc911\ub300\uc7a5 \uc9c1\uad8c\uc73c\ub85c \uc791\uc5c5\uc744 \uacc4\uc18d \uc9c0\uc2dc\ud558\uc600\ub2e4. \uc778\uadfc \ucd08\uc18c\uc758 \uc778\ubbfc\uad70 \ubd80\uc0ac\uad00\uacfc \ubcd1 20\uc5ec\uba85\uc740 \uacbd\ube44 \ubcd1\ub825\uc744 \uc694\uccad\ubc1b\uace0 \ud2b8\ub7ed\uc744 \uc774\uc6a9\ud558\uc5ec \ub3c4\ucc29\ud558\uc600\ub2e4. \ubc15\ucca0 \uc911\uc704\uc758 \uc791\uc5c5 \uc911\uc9c0 \uc7ac\uc694\uad6c\ub97c \ubcf4\ub2c8\ud30c\uc2a4 \ub300\uc704\uac00 \uac70\ubd80\ud558\uc790 \ubc15\ucca0\uc758 \uacf5\uaca9\uba85\ub839\uc5d0 \ub530\ub77c \uc778\ubbfc\uad70 \ubd80\uc0ac\uad00\uacfc \ubcd1\ub4e4\uc740 \ud2b8\ub7ed\uc5d0 \uc2e4\uc5b4 \uac00\uc9c0\uace0 \uc628 \uace1\uad2d\uc774, \ubabd\ub465\uc774\uc640 \ud568\uaed8 \ub178\ub3d9\uc790\ub4e4\uc774 \uc791\uc5c5\uc5d0 \uc4f0\ub824\uace0 \uac00\uc838\uc654\ub358 \ub3c4\ub07c \ub4f1\uc744 \ube7c\uc557\uc544 \ud718\ub450\ub974\uba70 \uae30\uc2b5\ud558\uc600\ub2e4. \uc774\ub4e4\uc740 \uc720\uc5d4\uad70\uce21 \uc9c0\ud718\uad00\uacfc \uc7a5\ubcd1\ub4e4\uc5d0\uac8c \uc9d1\uc911 \uacf5\uaca9\uc744 \uac00\ud558\uc5ec \uacbd\ube44\uc911\ub300\uc7a5 \uc544\uc11c \ubcf4\ub2c8\ud30c\uc2a4 \ub300\uc704 \uc640 \ubc30\ub7ff \uc911\uc704\uac00 \uc774\ub9c8\uc5d0 \uc911\uc0c1\uc744 \uc785\uace0 \uc774\uc1a1 \uc911 \uc0ac\ub9dd\ud558\uc600\uc73c\uba70, \uc8fc\ud55c \ubbf8\uad70 \ubd80\uc0ac\uad00\uacfc \ubcd1 4\uba85, \uad6d\uad70 \uc7a5\uad50\uc640 \ubd80\uc0ac\uad00\uacfc \ubcd1 4\uba85 \ub4f1\uc774 \uc911\uacbd\uc0c1\uc744 \uc785\uc5c8\uace0, \uc720\uc5d4\uad70 \ud2b8\ub7ed 3\ub300\uac00 \ud30c\uc190\ub418\uc5c8\ub2e4.", "sentence_answer": "\uc774\ub4e4\uc740 \uc720\uc5d4\uad70\uce21 \uc9c0\ud718\uad00\uacfc \uc7a5\ubcd1\ub4e4\uc5d0\uac8c \uc9d1\uc911 \uacf5\uaca9\uc744 \uac00\ud558\uc5ec \uacbd\ube44\uc911\ub300\uc7a5 \uc544\uc11c \ubcf4\ub2c8\ud30c\uc2a4 \ub300\uc704 \uc640 \ubc30\ub7ff \uc911\uc704\uac00 \uc774\ub9c8\uc5d0 \uc911\uc0c1\uc744 \uc785\uace0 \uc774\uc1a1 \uc911 \uc0ac\ub9dd\ud558\uc600\uc73c\uba70, \uc8fc\ud55c \ubbf8\uad70 \ubd80\uc0ac\uad00\uacfc \ubcd1 4\uba85, \uad6d\uad70 \uc7a5\uad50\uc640 \ubd80\uc0ac\uad00\uacfc \ubcd1 4\uba85 \ub4f1\uc774 \uc911\uacbd\uc0c1\uc744 \uc785\uc5c8\uace0, \uc720\uc5d4\uad70 \ud2b8\ub7ed 3\ub300\uac00 \ud30c\uc190\ub418\uc5c8\ub2e4.", "paragraph_id": "6566656-0-2", "paragraph_question": "question: \ud310\ubb38\uc810 \ub3c4\ub07c \uc0b4\uc778 \uc0ac\uac74\uc73c\ub85c \uc0ac\ub9dd\ud55c \uc720\uc5d4\uad70 \uce21 \uacbd\ube44\uc911\ub300\uc7a5\uc740?, context: \uc870\uc120\uc778\ubbfc\uad70 \ubc15\ucca0 \uc911\uc704\uc640 \ub2e4\ub978 \uc7a5\uad50 1\uba85, \uadf8\ub9ac\uace0 15\uba85\uc758 \ubd80\uc0ac\uad00\uacfc \ubcd1\uc774 \ub098\ud0c0\ub098 \uc791\uc5c5 \uc911\uc9c0\ub97c \uc694\uad6c\ud558\uc600\ub2e4. \ubbf8\ub8e8\ub098\ubb34\uc758 \uc704\uce58\uac00 \uc720\uc5d4\uad70 \uce21\uc758 \uad00\ud560\uc5d0 \uc18d\ud588\uae30\uc5d0 \ubcf4\uc218\uc791\uc5c5\uc744 \ud558\ub294\ub370 \ubb38\uc81c\uac00 \uc5c6\ub2e4\uace0 \ud310\ub2e8\ud55c \ubcf4\ub2c8\ud30c\uc2a4 \ub300\uc704\ub294 \uacbd\ube44\uc911\ub300\uc7a5 \uc9c1\uad8c\uc73c\ub85c \uc791\uc5c5\uc744 \uacc4\uc18d \uc9c0\uc2dc\ud558\uc600\ub2e4. \uc778\uadfc \ucd08\uc18c\uc758 \uc778\ubbfc\uad70 \ubd80\uc0ac\uad00\uacfc \ubcd1 20\uc5ec\uba85\uc740 \uacbd\ube44 \ubcd1\ub825\uc744 \uc694\uccad\ubc1b\uace0 \ud2b8\ub7ed\uc744 \uc774\uc6a9\ud558\uc5ec \ub3c4\ucc29\ud558\uc600\ub2e4. \ubc15\ucca0 \uc911\uc704\uc758 \uc791\uc5c5 \uc911\uc9c0 \uc7ac\uc694\uad6c\ub97c \ubcf4\ub2c8\ud30c\uc2a4 \ub300\uc704\uac00 \uac70\ubd80\ud558\uc790 \ubc15\ucca0\uc758 \uacf5\uaca9\uba85\ub839\uc5d0 \ub530\ub77c \uc778\ubbfc\uad70 \ubd80\uc0ac\uad00\uacfc \ubcd1\ub4e4\uc740 \ud2b8\ub7ed\uc5d0 \uc2e4\uc5b4 \uac00\uc9c0\uace0 \uc628 \uace1\uad2d\uc774, \ubabd\ub465\uc774\uc640 \ud568\uaed8 \ub178\ub3d9\uc790\ub4e4\uc774 \uc791\uc5c5\uc5d0 \uc4f0\ub824\uace0 \uac00\uc838\uc654\ub358 \ub3c4\ub07c \ub4f1\uc744 \ube7c\uc557\uc544 \ud718\ub450\ub974\uba70 \uae30\uc2b5\ud558\uc600\ub2e4. \uc774\ub4e4\uc740 \uc720\uc5d4\uad70\uce21 \uc9c0\ud718\uad00\uacfc \uc7a5\ubcd1\ub4e4\uc5d0\uac8c \uc9d1\uc911 \uacf5\uaca9\uc744 \uac00\ud558\uc5ec \uacbd\ube44\uc911\ub300\uc7a5 \uc544\uc11c \ubcf4\ub2c8\ud30c\uc2a4 \ub300\uc704\uc640 \ubc30\ub7ff \uc911\uc704\uac00 \uc774\ub9c8\uc5d0 \uc911\uc0c1\uc744 \uc785\uace0 \uc774\uc1a1 \uc911 \uc0ac\ub9dd\ud558\uc600\uc73c\uba70, \uc8fc\ud55c \ubbf8\uad70 \ubd80\uc0ac\uad00\uacfc \ubcd1 4\uba85, \uad6d\uad70 \uc7a5\uad50\uc640 \ubd80\uc0ac\uad00\uacfc \ubcd1 4\uba85 \ub4f1\uc774 \uc911\uacbd\uc0c1\uc744 \uc785\uc5c8\uace0, \uc720\uc5d4\uad70 \ud2b8\ub7ed 3\ub300\uac00 \ud30c\uc190\ub418\uc5c8\ub2e4."} {"question": "2016\ub144 \ubc1c\uc0dd\ud55c \uacbd\uc8fc \uc9c0\uc9c4\uc758 \ubc1c\uc0dd \uc9c0\uc5ed\uc740 \uc5b4\ub514\uc778\uac00\uc694?", "paragraph": "2016\ub144 \uacbd\uc8fc \uc9c0\uc9c4\uc740 2016\ub144 9\uc6d4 12\uc77c \ub300\ud55c\ubbfc\uad6d \uacbd\uc0c1\ubd81\ub3c4 \uacbd\uc8fc\uc2dc \ub0a8\ub0a8\uc11c\ucabd 8km \uc9c0\uc5ed\uc5d0\uc11c \ubc1c\uc0dd\ud55c \uc9c0\uc9c4\uc774\ub2e4. \ubbf8\uad6d \uc9c0\uc9c8\uc870\uc0ac\uad6d \uc9d1\uacc4 \uae30\uc900\uc73c\ub85c \uc804\uc9c4 \uaddc\ubaa8\ub294 \ub9ac\ud788\ud130 \uaddc\ubaa8 5.1\uc774\uc5c8\uc73c\uba70, \ubcf8\uc9c4\uc758 \uaddc\ubaa8\ub294 \ub9ac\ud788\ud130 \uaddc\ubaa8 5.8\uc774\uc5c8\ub2e4. \uc804\uc9c4\uc758 \ucd5c\ub300 \uc9c4\ub3c4\ub294 V, \ubcf8\uc9c4\uc758 \ucd5c\ub300 \uc9c4\ub3c4\ub294 VI\uc600\ub2e4. \ubcf8 \uc9c0\uc9c4\uc740 1978\ub144 \ub300\ud55c\ubbfc\uad6d \uc9c0\uc9c4 \uad00\uce21 \uc774\ub798 \uc5ed\ub300 \ucd5c\uac15\uc758 \uc9c0\uc9c4\uc774\ub2e4. \ub300\ud55c\ubbfc\uad6d \ub0b4 \uc721\uc0c1\uc9c0\uc9c4\uc73c\ub85c\ub294 1978\ub144 \ucda9\uccad\ubd81\ub3c4 \uc18d\ub9ac\uc0b0\uc5d0\uc11c \uaddc\ubaa8 5.2, \ucda9\uccad\ub0a8\ub3c4 \ud64d\uc131\uad70\uc5d0\uc11c \uaddc\ubaa8 5.0 \uc758 \uc9c0\uc9c4 \ubc1c\uc0dd \ud6c4 38\ub144\ub9cc\uc758 \ub300\ud615\uc9c0\uc9c4\uc774\ub2e4. \ud55c\ubc18\ub3c4\uc5d0\uc11c\ub294 \ud575\uc2e4\ud5d8 \ub4f1\uc5d0 \uc758\ud55c \uc778\uacf5\uc9c0\uc9c4\uc744 \uc81c\uc678\ud558\uba74 1980\ub144 \uc870\uc120\ubbfc\uc8fc\uc8fc\uc758\uc778\ubbfc\uacf5\ud654\uad6d \ud3c9\uc548\ubd81\ub3c4 \uc758\uc8fc-\uc0ad\uc8fc-\uad6c\uc131 \uc9c0\uc5ed\uc5d0\uc11c \uaddc\ubaa8 5.3\uc758 \uc9c0\uc9c4 \ubc1c\uc0dd\ud6c4 36\ub144\ub9cc\uc758 \ub300\ud615\uc9c0\uc9c4\uc774\ub2e4.", "answer": "\uacbd\uc8fc\uc2dc \ub0a8\ub0a8\uc11c\ucabd 8km \uc9c0\uc5ed", "sentence": "2016\ub144 \uacbd\uc8fc \uc9c0\uc9c4\uc740 2016\ub144 9\uc6d4 12\uc77c \ub300\ud55c\ubbfc\uad6d \uacbd\uc0c1\ubd81\ub3c4 \uacbd\uc8fc\uc2dc \ub0a8\ub0a8\uc11c\ucabd 8km \uc9c0\uc5ed \uc5d0\uc11c \ubc1c\uc0dd\ud55c \uc9c0\uc9c4\uc774\ub2e4.", "paragraph_sentence": " 2016\ub144 \uacbd\uc8fc \uc9c0\uc9c4\uc740 2016\ub144 9\uc6d4 12\uc77c \ub300\ud55c\ubbfc\uad6d \uacbd\uc0c1\ubd81\ub3c4 \uacbd\uc8fc\uc2dc \ub0a8\ub0a8\uc11c\ucabd 8km \uc9c0\uc5ed \uc5d0\uc11c \ubc1c\uc0dd\ud55c \uc9c0\uc9c4\uc774\ub2e4. \ubbf8\uad6d \uc9c0\uc9c8\uc870\uc0ac\uad6d \uc9d1\uacc4 \uae30\uc900\uc73c\ub85c \uc804\uc9c4 \uaddc\ubaa8\ub294 \ub9ac\ud788\ud130 \uaddc\ubaa8 5.1\uc774\uc5c8\uc73c\uba70, \ubcf8\uc9c4\uc758 \uaddc\ubaa8\ub294 \ub9ac\ud788\ud130 \uaddc\ubaa8 5.8\uc774\uc5c8\ub2e4. \uc804\uc9c4\uc758 \ucd5c\ub300 \uc9c4\ub3c4\ub294 V, \ubcf8\uc9c4\uc758 \ucd5c\ub300 \uc9c4\ub3c4\ub294 VI\uc600\ub2e4. \ubcf8 \uc9c0\uc9c4\uc740 1978\ub144 \ub300\ud55c\ubbfc\uad6d \uc9c0\uc9c4 \uad00\uce21 \uc774\ub798 \uc5ed\ub300 \ucd5c\uac15\uc758 \uc9c0\uc9c4\uc774\ub2e4. \ub300\ud55c\ubbfc\uad6d \ub0b4 \uc721\uc0c1\uc9c0\uc9c4\uc73c\ub85c\ub294 1978\ub144 \ucda9\uccad\ubd81\ub3c4 \uc18d\ub9ac\uc0b0\uc5d0\uc11c \uaddc\ubaa8 5.2, \ucda9\uccad\ub0a8\ub3c4 \ud64d\uc131\uad70\uc5d0\uc11c \uaddc\ubaa8 5.0 \uc758 \uc9c0\uc9c4 \ubc1c\uc0dd \ud6c4 38\ub144\ub9cc\uc758 \ub300\ud615\uc9c0\uc9c4\uc774\ub2e4. \ud55c\ubc18\ub3c4\uc5d0\uc11c\ub294 \ud575\uc2e4\ud5d8 \ub4f1\uc5d0 \uc758\ud55c \uc778\uacf5\uc9c0\uc9c4\uc744 \uc81c\uc678\ud558\uba74 1980\ub144 \uc870\uc120\ubbfc\uc8fc\uc8fc\uc758\uc778\ubbfc\uacf5\ud654\uad6d \ud3c9\uc548\ubd81\ub3c4 \uc758\uc8fc-\uc0ad\uc8fc-\uad6c\uc131 \uc9c0\uc5ed\uc5d0\uc11c \uaddc\ubaa8 5.3\uc758 \uc9c0\uc9c4 \ubc1c\uc0dd\ud6c4 36\ub144\ub9cc\uc758 \ub300\ud615\uc9c0\uc9c4\uc774\ub2e4.", "paragraph_answer": "2016\ub144 \uacbd\uc8fc \uc9c0\uc9c4\uc740 2016\ub144 9\uc6d4 12\uc77c \ub300\ud55c\ubbfc\uad6d \uacbd\uc0c1\ubd81\ub3c4 \uacbd\uc8fc\uc2dc \ub0a8\ub0a8\uc11c\ucabd 8km \uc9c0\uc5ed \uc5d0\uc11c \ubc1c\uc0dd\ud55c \uc9c0\uc9c4\uc774\ub2e4. \ubbf8\uad6d \uc9c0\uc9c8\uc870\uc0ac\uad6d \uc9d1\uacc4 \uae30\uc900\uc73c\ub85c \uc804\uc9c4 \uaddc\ubaa8\ub294 \ub9ac\ud788\ud130 \uaddc\ubaa8 5.1\uc774\uc5c8\uc73c\uba70, \ubcf8\uc9c4\uc758 \uaddc\ubaa8\ub294 \ub9ac\ud788\ud130 \uaddc\ubaa8 5.8\uc774\uc5c8\ub2e4. \uc804\uc9c4\uc758 \ucd5c\ub300 \uc9c4\ub3c4\ub294 V, \ubcf8\uc9c4\uc758 \ucd5c\ub300 \uc9c4\ub3c4\ub294 VI\uc600\ub2e4. \ubcf8 \uc9c0\uc9c4\uc740 1978\ub144 \ub300\ud55c\ubbfc\uad6d \uc9c0\uc9c4 \uad00\uce21 \uc774\ub798 \uc5ed\ub300 \ucd5c\uac15\uc758 \uc9c0\uc9c4\uc774\ub2e4. \ub300\ud55c\ubbfc\uad6d \ub0b4 \uc721\uc0c1\uc9c0\uc9c4\uc73c\ub85c\ub294 1978\ub144 \ucda9\uccad\ubd81\ub3c4 \uc18d\ub9ac\uc0b0\uc5d0\uc11c \uaddc\ubaa8 5.2, \ucda9\uccad\ub0a8\ub3c4 \ud64d\uc131\uad70\uc5d0\uc11c \uaddc\ubaa8 5.0 \uc758 \uc9c0\uc9c4 \ubc1c\uc0dd \ud6c4 38\ub144\ub9cc\uc758 \ub300\ud615\uc9c0\uc9c4\uc774\ub2e4. \ud55c\ubc18\ub3c4\uc5d0\uc11c\ub294 \ud575\uc2e4\ud5d8 \ub4f1\uc5d0 \uc758\ud55c \uc778\uacf5\uc9c0\uc9c4\uc744 \uc81c\uc678\ud558\uba74 1980\ub144 \uc870\uc120\ubbfc\uc8fc\uc8fc\uc758\uc778\ubbfc\uacf5\ud654\uad6d \ud3c9\uc548\ubd81\ub3c4 \uc758\uc8fc-\uc0ad\uc8fc-\uad6c\uc131 \uc9c0\uc5ed\uc5d0\uc11c \uaddc\ubaa8 5.3\uc758 \uc9c0\uc9c4 \ubc1c\uc0dd\ud6c4 36\ub144\ub9cc\uc758 \ub300\ud615\uc9c0\uc9c4\uc774\ub2e4.", "sentence_answer": "2016\ub144 \uacbd\uc8fc \uc9c0\uc9c4\uc740 2016\ub144 9\uc6d4 12\uc77c \ub300\ud55c\ubbfc\uad6d \uacbd\uc0c1\ubd81\ub3c4 \uacbd\uc8fc\uc2dc \ub0a8\ub0a8\uc11c\ucabd 8km \uc9c0\uc5ed \uc5d0\uc11c \ubc1c\uc0dd\ud55c \uc9c0\uc9c4\uc774\ub2e4.", "paragraph_id": "6470802-0-0", "paragraph_question": "question: 2016\ub144 \ubc1c\uc0dd\ud55c \uacbd\uc8fc \uc9c0\uc9c4\uc758 \ubc1c\uc0dd \uc9c0\uc5ed\uc740 \uc5b4\ub514\uc778\uac00\uc694?, context: 2016\ub144 \uacbd\uc8fc \uc9c0\uc9c4\uc740 2016\ub144 9\uc6d4 12\uc77c \ub300\ud55c\ubbfc\uad6d \uacbd\uc0c1\ubd81\ub3c4 \uacbd\uc8fc\uc2dc \ub0a8\ub0a8\uc11c\ucabd 8km \uc9c0\uc5ed\uc5d0\uc11c \ubc1c\uc0dd\ud55c \uc9c0\uc9c4\uc774\ub2e4. \ubbf8\uad6d \uc9c0\uc9c8\uc870\uc0ac\uad6d \uc9d1\uacc4 \uae30\uc900\uc73c\ub85c \uc804\uc9c4 \uaddc\ubaa8\ub294 \ub9ac\ud788\ud130 \uaddc\ubaa8 5.1\uc774\uc5c8\uc73c\uba70, \ubcf8\uc9c4\uc758 \uaddc\ubaa8\ub294 \ub9ac\ud788\ud130 \uaddc\ubaa8 5.8\uc774\uc5c8\ub2e4. \uc804\uc9c4\uc758 \ucd5c\ub300 \uc9c4\ub3c4\ub294 V, \ubcf8\uc9c4\uc758 \ucd5c\ub300 \uc9c4\ub3c4\ub294 VI\uc600\ub2e4. \ubcf8 \uc9c0\uc9c4\uc740 1978\ub144 \ub300\ud55c\ubbfc\uad6d \uc9c0\uc9c4 \uad00\uce21 \uc774\ub798 \uc5ed\ub300 \ucd5c\uac15\uc758 \uc9c0\uc9c4\uc774\ub2e4. \ub300\ud55c\ubbfc\uad6d \ub0b4 \uc721\uc0c1\uc9c0\uc9c4\uc73c\ub85c\ub294 1978\ub144 \ucda9\uccad\ubd81\ub3c4 \uc18d\ub9ac\uc0b0\uc5d0\uc11c \uaddc\ubaa8 5.2, \ucda9\uccad\ub0a8\ub3c4 \ud64d\uc131\uad70\uc5d0\uc11c \uaddc\ubaa8 5.0 \uc758 \uc9c0\uc9c4 \ubc1c\uc0dd \ud6c4 38\ub144\ub9cc\uc758 \ub300\ud615\uc9c0\uc9c4\uc774\ub2e4. \ud55c\ubc18\ub3c4\uc5d0\uc11c\ub294 \ud575\uc2e4\ud5d8 \ub4f1\uc5d0 \uc758\ud55c \uc778\uacf5\uc9c0\uc9c4\uc744 \uc81c\uc678\ud558\uba74 1980\ub144 \uc870\uc120\ubbfc\uc8fc\uc8fc\uc758\uc778\ubbfc\uacf5\ud654\uad6d \ud3c9\uc548\ubd81\ub3c4 \uc758\uc8fc-\uc0ad\uc8fc-\uad6c\uc131 \uc9c0\uc5ed\uc5d0\uc11c \uaddc\ubaa8 5.3\uc758 \uc9c0\uc9c4 \ubc1c\uc0dd\ud6c4 36\ub144\ub9cc\uc758 \ub300\ud615\uc9c0\uc9c4\uc774\ub2e4."} {"question": "\uace0\ub824 \uc815\uc885\uc740 \ub204\uad6c\ub85c\ubd80\ud130 \uc655\uc704\ub97c \uc774\uc5b4\ubc1b\uc558\ub294\uac00?", "paragraph": "\uc815\uc885\uc740 \uc784\uae08\uc758 \ud615\uc81c\ub85c \uc655\uc704\ub97c \uc774\uc5b4\ubc1b\uc544 \ubc24\ub0ae\uc73c\ub85c \ub178\ub825\ud558\uc5ec \ub098\ub77c \ub2e4\uc2a4\ub9ac\ub294 \ub3c4\ub9ac\ub97c \uad6c\ud588\uc2b5\ub2c8\ub2e4. \ub54c\ub85c\ub294 \ucd1b\ubd88\uc744 \ubc1d\ud600\ub4e4\uace0 \uc870\uc815\uc758 \uc120\ube44\ub97c \uc811\uacac\ud558\uc600\uace0, \ub610 \uc5b4\ub5a4 \ub54c\ub294 \uc815\uc0ac\uc5d0 \ubc14\ube60\uc11c \ub2a6\uac8c \uc2dd\uc0ac\ud558\uba74\uc11c \ubaa8\ub4e0 \uc815\uc0ac\ub97c \ub4e3\uace0 \uacb0\uc815\ud558\uc600\uc2b5\ub2c8\ub2e4. \uadf8\ub7ec\ubbc0\ub85c \uc989\uc704\ud55c \ucd08\uae30\uc5d0 \uc0ac\ub78c\ub4e4\uc774 \ubaa8\ub450 \uc11c\ub85c \uae30\ubed0\ud558\uc600\uc2b5\ub2c8\ub2e4. \uadf8\ub7f0\ub370 \ub3c4\ucc38\uc744 \uadf8\ub987\ub418\uac8c \ubbff\uac8c \ub418\uc790 \ub3c4\uc74d\uc744 \uc62e\uae30\ub85c \uacb0\uc815\ud558\uc600\uc2b5\ub2c8\ub2e4. \uac8c\ub2e4\uac00 \ucc9c\uc131\uc774 \uad73\uc138\uc5b4 \uace0\uc9d1\uc744 \uad7d\ud788\uc9c0 \uc544\ub2c8\ud588\uace0, \uae09\ubc15\ud558\uac8c \ubc31\uc131\ub4e4\uc744 \uc9d5\ubc1c\ud558\uc5ec \uc5ed\uc0ac(\u5f79\u4e8b)\ub97c \uc77c\uc73c\ud0a4\uace0 \uc0ac\ub78c\ub4e4\uc744 \uc218\uace0\ub86d\uac8c \ud558\ub2c8, \ube44\ub85d \uc784\uae08\uc758 \uc0dd\uac01\uc774 \uc633\ub2e4\uace0 \ud574\ub3c4 \uc0ac\ub78c\ub4e4\uc758 \ub9c8\uc74c\uc740 \uc774\ub97c \ub530\ub974\uc9c0 \uc54a\uc558\uc2b5\ub2c8\ub2e4. \uc6d0\ub9dd\uacfc \ube44\ubc29\uc774 \uc774\ub85c \uc778\ud574 \uc77c\uc5b4\ub0ac\uace0 \uc7ac\ub09c\uc774 \uadf8\ub9bc\uc790\uc640 \uba54\uc544\ub9ac\ucc98\ub7fc \uc7ac\ube68\ub9ac \uc751\ud558\uc5ec \uc11c\uacbd\uc73c\ub85c \ub3c4\uc74d\uc744 \uc62e\uae30\uc9c0\ub3c4 \ubabb\ud558\uace0 \uc784\uae08\uc758 \uc790\ub9ac\ub97c \uc601\uc6d0\ud788 \ub5a0\ub098\uac8c \ub418\uc5c8\uc73c\ub2c8 \ucc38\uc73c\ub85c \ud1b5\ud0c4\ud560 \ub9cc\ud569\ub2c8\ub2e4.", "answer": "\uc784\uae08\uc758 \ud615\uc81c", "sentence": "\uc815\uc885\uc740 \uc784\uae08\uc758 \ud615\uc81c \ub85c \uc655\uc704\ub97c \uc774\uc5b4\ubc1b\uc544 \ubc24\ub0ae\uc73c\ub85c \ub178\ub825\ud558\uc5ec \ub098\ub77c \ub2e4\uc2a4\ub9ac\ub294 \ub3c4\ub9ac\ub97c \uad6c\ud588\uc2b5\ub2c8\ub2e4.", "paragraph_sentence": " \uc815\uc885\uc740 \uc784\uae08\uc758 \ud615\uc81c \ub85c \uc655\uc704\ub97c \uc774\uc5b4\ubc1b\uc544 \ubc24\ub0ae\uc73c\ub85c \ub178\ub825\ud558\uc5ec \ub098\ub77c \ub2e4\uc2a4\ub9ac\ub294 \ub3c4\ub9ac\ub97c \uad6c\ud588\uc2b5\ub2c8\ub2e4. \ub54c\ub85c\ub294 \ucd1b\ubd88\uc744 \ubc1d\ud600\ub4e4\uace0 \uc870\uc815\uc758 \uc120\ube44\ub97c \uc811\uacac\ud558\uc600\uace0, \ub610 \uc5b4\ub5a4 \ub54c\ub294 \uc815\uc0ac\uc5d0 \ubc14\ube60\uc11c \ub2a6\uac8c \uc2dd\uc0ac\ud558\uba74\uc11c \ubaa8\ub4e0 \uc815\uc0ac\ub97c \ub4e3\uace0 \uacb0\uc815\ud558\uc600\uc2b5\ub2c8\ub2e4. \uadf8\ub7ec\ubbc0\ub85c \uc989\uc704\ud55c \ucd08\uae30\uc5d0 \uc0ac\ub78c\ub4e4\uc774 \ubaa8\ub450 \uc11c\ub85c \uae30\ubed0\ud558\uc600\uc2b5\ub2c8\ub2e4. \uadf8\ub7f0\ub370 \ub3c4\ucc38\uc744 \uadf8\ub987\ub418\uac8c \ubbff\uac8c \ub418\uc790 \ub3c4\uc74d\uc744 \uc62e\uae30\ub85c \uacb0\uc815\ud558\uc600\uc2b5\ub2c8\ub2e4. \uac8c\ub2e4\uac00 \ucc9c\uc131\uc774 \uad73\uc138\uc5b4 \uace0\uc9d1\uc744 \uad7d\ud788\uc9c0 \uc544\ub2c8\ud588\uace0, \uae09\ubc15\ud558\uac8c \ubc31\uc131\ub4e4\uc744 \uc9d5\ubc1c\ud558\uc5ec \uc5ed\uc0ac(\u5f79\u4e8b)\ub97c \uc77c\uc73c\ud0a4\uace0 \uc0ac\ub78c\ub4e4\uc744 \uc218\uace0\ub86d\uac8c \ud558\ub2c8, \ube44\ub85d \uc784\uae08\uc758 \uc0dd\uac01\uc774 \uc633\ub2e4\uace0 \ud574\ub3c4 \uc0ac\ub78c\ub4e4\uc758 \ub9c8\uc74c\uc740 \uc774\ub97c \ub530\ub974\uc9c0 \uc54a\uc558\uc2b5\ub2c8\ub2e4. \uc6d0\ub9dd\uacfc \ube44\ubc29\uc774 \uc774\ub85c \uc778\ud574 \uc77c\uc5b4\ub0ac\uace0 \uc7ac\ub09c\uc774 \uadf8\ub9bc\uc790\uc640 \uba54\uc544\ub9ac\ucc98\ub7fc \uc7ac\ube68\ub9ac \uc751\ud558\uc5ec \uc11c\uacbd\uc73c\ub85c \ub3c4\uc74d\uc744 \uc62e\uae30\uc9c0\ub3c4 \ubabb\ud558\uace0 \uc784\uae08\uc758 \uc790\ub9ac\ub97c \uc601\uc6d0\ud788 \ub5a0\ub098\uac8c \ub418\uc5c8\uc73c\ub2c8 \ucc38\uc73c\ub85c \ud1b5\ud0c4\ud560 \ub9cc\ud569\ub2c8\ub2e4.", "paragraph_answer": "\uc815\uc885\uc740 \uc784\uae08\uc758 \ud615\uc81c \ub85c \uc655\uc704\ub97c \uc774\uc5b4\ubc1b\uc544 \ubc24\ub0ae\uc73c\ub85c \ub178\ub825\ud558\uc5ec \ub098\ub77c \ub2e4\uc2a4\ub9ac\ub294 \ub3c4\ub9ac\ub97c \uad6c\ud588\uc2b5\ub2c8\ub2e4. \ub54c\ub85c\ub294 \ucd1b\ubd88\uc744 \ubc1d\ud600\ub4e4\uace0 \uc870\uc815\uc758 \uc120\ube44\ub97c \uc811\uacac\ud558\uc600\uace0, \ub610 \uc5b4\ub5a4 \ub54c\ub294 \uc815\uc0ac\uc5d0 \ubc14\ube60\uc11c \ub2a6\uac8c \uc2dd\uc0ac\ud558\uba74\uc11c \ubaa8\ub4e0 \uc815\uc0ac\ub97c \ub4e3\uace0 \uacb0\uc815\ud558\uc600\uc2b5\ub2c8\ub2e4. \uadf8\ub7ec\ubbc0\ub85c \uc989\uc704\ud55c \ucd08\uae30\uc5d0 \uc0ac\ub78c\ub4e4\uc774 \ubaa8\ub450 \uc11c\ub85c \uae30\ubed0\ud558\uc600\uc2b5\ub2c8\ub2e4. \uadf8\ub7f0\ub370 \ub3c4\ucc38\uc744 \uadf8\ub987\ub418\uac8c \ubbff\uac8c \ub418\uc790 \ub3c4\uc74d\uc744 \uc62e\uae30\ub85c \uacb0\uc815\ud558\uc600\uc2b5\ub2c8\ub2e4. \uac8c\ub2e4\uac00 \ucc9c\uc131\uc774 \uad73\uc138\uc5b4 \uace0\uc9d1\uc744 \uad7d\ud788\uc9c0 \uc544\ub2c8\ud588\uace0, \uae09\ubc15\ud558\uac8c \ubc31\uc131\ub4e4\uc744 \uc9d5\ubc1c\ud558\uc5ec \uc5ed\uc0ac(\u5f79\u4e8b)\ub97c \uc77c\uc73c\ud0a4\uace0 \uc0ac\ub78c\ub4e4\uc744 \uc218\uace0\ub86d\uac8c \ud558\ub2c8, \ube44\ub85d \uc784\uae08\uc758 \uc0dd\uac01\uc774 \uc633\ub2e4\uace0 \ud574\ub3c4 \uc0ac\ub78c\ub4e4\uc758 \ub9c8\uc74c\uc740 \uc774\ub97c \ub530\ub974\uc9c0 \uc54a\uc558\uc2b5\ub2c8\ub2e4. \uc6d0\ub9dd\uacfc \ube44\ubc29\uc774 \uc774\ub85c \uc778\ud574 \uc77c\uc5b4\ub0ac\uace0 \uc7ac\ub09c\uc774 \uadf8\ub9bc\uc790\uc640 \uba54\uc544\ub9ac\ucc98\ub7fc \uc7ac\ube68\ub9ac \uc751\ud558\uc5ec \uc11c\uacbd\uc73c\ub85c \ub3c4\uc74d\uc744 \uc62e\uae30\uc9c0\ub3c4 \ubabb\ud558\uace0 \uc784\uae08\uc758 \uc790\ub9ac\ub97c \uc601\uc6d0\ud788 \ub5a0\ub098\uac8c \ub418\uc5c8\uc73c\ub2c8 \ucc38\uc73c\ub85c \ud1b5\ud0c4\ud560 \ub9cc\ud569\ub2c8\ub2e4.", "sentence_answer": "\uc815\uc885\uc740 \uc784\uae08\uc758 \ud615\uc81c \ub85c \uc655\uc704\ub97c \uc774\uc5b4\ubc1b\uc544 \ubc24\ub0ae\uc73c\ub85c \ub178\ub825\ud558\uc5ec \ub098\ub77c \ub2e4\uc2a4\ub9ac\ub294 \ub3c4\ub9ac\ub97c \uad6c\ud588\uc2b5\ub2c8\ub2e4.", "paragraph_id": "6487169-0-1", "paragraph_question": "question: \uace0\ub824 \uc815\uc885\uc740 \ub204\uad6c\ub85c\ubd80\ud130 \uc655\uc704\ub97c \uc774\uc5b4\ubc1b\uc558\ub294\uac00?, context: \uc815\uc885\uc740 \uc784\uae08\uc758 \ud615\uc81c\ub85c \uc655\uc704\ub97c \uc774\uc5b4\ubc1b\uc544 \ubc24\ub0ae\uc73c\ub85c \ub178\ub825\ud558\uc5ec \ub098\ub77c \ub2e4\uc2a4\ub9ac\ub294 \ub3c4\ub9ac\ub97c \uad6c\ud588\uc2b5\ub2c8\ub2e4. \ub54c\ub85c\ub294 \ucd1b\ubd88\uc744 \ubc1d\ud600\ub4e4\uace0 \uc870\uc815\uc758 \uc120\ube44\ub97c \uc811\uacac\ud558\uc600\uace0, \ub610 \uc5b4\ub5a4 \ub54c\ub294 \uc815\uc0ac\uc5d0 \ubc14\ube60\uc11c \ub2a6\uac8c \uc2dd\uc0ac\ud558\uba74\uc11c \ubaa8\ub4e0 \uc815\uc0ac\ub97c \ub4e3\uace0 \uacb0\uc815\ud558\uc600\uc2b5\ub2c8\ub2e4. \uadf8\ub7ec\ubbc0\ub85c \uc989\uc704\ud55c \ucd08\uae30\uc5d0 \uc0ac\ub78c\ub4e4\uc774 \ubaa8\ub450 \uc11c\ub85c \uae30\ubed0\ud558\uc600\uc2b5\ub2c8\ub2e4. \uadf8\ub7f0\ub370 \ub3c4\ucc38\uc744 \uadf8\ub987\ub418\uac8c \ubbff\uac8c \ub418\uc790 \ub3c4\uc74d\uc744 \uc62e\uae30\ub85c \uacb0\uc815\ud558\uc600\uc2b5\ub2c8\ub2e4. \uac8c\ub2e4\uac00 \ucc9c\uc131\uc774 \uad73\uc138\uc5b4 \uace0\uc9d1\uc744 \uad7d\ud788\uc9c0 \uc544\ub2c8\ud588\uace0, \uae09\ubc15\ud558\uac8c \ubc31\uc131\ub4e4\uc744 \uc9d5\ubc1c\ud558\uc5ec \uc5ed\uc0ac(\u5f79\u4e8b)\ub97c \uc77c\uc73c\ud0a4\uace0 \uc0ac\ub78c\ub4e4\uc744 \uc218\uace0\ub86d\uac8c \ud558\ub2c8, \ube44\ub85d \uc784\uae08\uc758 \uc0dd\uac01\uc774 \uc633\ub2e4\uace0 \ud574\ub3c4 \uc0ac\ub78c\ub4e4\uc758 \ub9c8\uc74c\uc740 \uc774\ub97c \ub530\ub974\uc9c0 \uc54a\uc558\uc2b5\ub2c8\ub2e4. \uc6d0\ub9dd\uacfc \ube44\ubc29\uc774 \uc774\ub85c \uc778\ud574 \uc77c\uc5b4\ub0ac\uace0 \uc7ac\ub09c\uc774 \uadf8\ub9bc\uc790\uc640 \uba54\uc544\ub9ac\ucc98\ub7fc \uc7ac\ube68\ub9ac \uc751\ud558\uc5ec \uc11c\uacbd\uc73c\ub85c \ub3c4\uc74d\uc744 \uc62e\uae30\uc9c0\ub3c4 \ubabb\ud558\uace0 \uc784\uae08\uc758 \uc790\ub9ac\ub97c \uc601\uc6d0\ud788 \ub5a0\ub098\uac8c \ub418\uc5c8\uc73c\ub2c8 \ucc38\uc73c\ub85c \ud1b5\ud0c4\ud560 \ub9cc\ud569\ub2c8\ub2e4."} {"question": "\uc218\ud45c\uc758 \ubc1c\ud589\uc778\uacfc \uc18c\uc9c0\uc778\uc774 \uac70\ub798\ud558\uace0 \uc788\ub294 \uc740\ud589\uc774 \uac19\uc544 \ucd94\uc2ec\uc744 \uc758\ub8b0\ud560 \ub54c \ubcc4\ub3c4 \uad50\ud658\ud45c\uc5d0 \uae30\uc785\ud558\uc5ec \uc81c\ucd9c\ud558\ub294 \uacf3\uc740?", "paragraph": "\ubc1c\ud589\uc778\uacfc \uc18c\uc9c0\uc778\uc774 \ub3d9\uc77c\uc740\ud589\uc5d0 \uac70\ub798\ud558\uace0 \uc788\ub294 \uacbd\uc6b0\uc5d0\ub294 \uc9c1\uc811 \uc9c0\uae09\uc740\ud589\uc5d0 \uc81c\uae30\ud558\uc5ec \uadf8 \uc740\ud589\uc73c\ub85c\ubd80\ud130 \uc9c0\uae09\uc744 \ubc1b\uc9c0\ub9cc \ub300\ubd80\ubd84\uc740 \uc9c0\uae09\uc740\ud589 \uc774\uc678\uc5d0 \ucd94\uc2ec\uc744 \uc758\ub8b0\ud55c\ub2e4. \ud6c4\uc790\uc758 \uacbd\uc6b0 \uc6b0\uc120 \uc758\ub8b0\ub97c \ubc1b\uc740 \uc9c0\uae09\uc740\ud589\ubcc4\ub85c \ubd84\ub958\ud558\uc5ec \uae08\uc561\u00b7\ub9e4\uc218(\u679a\u6578) \ub4f1\uc744 \ubcc4\ub3c4 \uad50\ud658\ud45c\uc5d0 \uae30\uc785\ud558\uc5ec \uc5b4\uc74c\uad50\ud658\uc18c\uc5d0 \uc81c\ucd9c\ud55c\ub2e4. \uc774 \uad50\ud658\uc18c\ub294 \ud2b9\ud788 \uc5b4\uc74c\u00b7\uc218\ud45c\ubc95\uc0c1\uc758 \uc5c4\ubc00\ud55c \uaddc\uc815\uc740 \uc544\ub2c8\uba70(\uc218\ud45c 31\uc870, 69\uc870 \ub4f1), \uc8fc\ub85c \uac70\ub798\uc2e4\ubb34\uc0c1\uc758 \ud544\uc694\uc131\uc5d0\uc11c \ubc95\ubb34\ubd80\ub839(340\ud638, 1990. 3. 6)\uc5d0 \uae30\ud558\uc5ec \ubc95\ubb34\ubd80\uc7a5\uad00\uc774 \uc9c0\uc815\ud55c\ub2e4. \uc608\ucee8\ub300, '\uc11c\uc6b8\uc5b4\uc74c\uad50\ud658\uc18c'\uc758 \uc18c\uc7ac\uc9c0\ub294 '\uc11c\uc6b8\ud2b9\ubcc4\uc2dc'\uc774\uba70, \ud604\uc7ac\ub85c\ub294 \uc11c\uc6b8\ud2b9\ubcc4\uc2dc \uac15\ub0a8\uad6c \uc5ed\uc0bc\ub3d9 717\ubc88\uc9c0 \uae08\uc735\uacb0\uc81c\uc6d0 \ub0b4\uc5d0 \uc124\uce58\ub418\uc5b4 \uc788\ub2e4. \uc5b4\uc74c\uad50\ud658\uc18c\uc5d0 \uc218\ud45c\ub97c \uc81c\uc2dc\ud558\ub294 \uac83\uc740 \uad50\ud658\uc18c\uc5d0 \uac00\uc785\ud55c \uc740\ud589(\uc870\ud569\uc740\ud589)\uc5d0 \ud55c\ud558\uba70 \ub3d9\uc2dc\uc5d0 \uac01 \uc870\ud569\uc740\ud589 \uc55e\uc73c\ub85c \ubc1c\ud589\ub41c \uc218\ud45c(\uc5b4\uc74c)\ub294 \uc774\ub97c \uc804\ubd80 \uad50\ud658\uc5d0 \ud68c\ubd80\ud558\uc5ec \uacb0\uc81c\ud560 \uc758\ubb34\uac00 \uc788\ub2e4. \uc5b4\uc74c\uad50\ud658\uc18c\uc758 \uad50\ud658\uc744 \uacbd\uc6b0\ud558\uc9c0 \uc544\ub2c8\ud55c \uac83\uc740 \ud55c\uad6d\uc740\ud589 \uc55e\uc73c\ub85c \ubc1c\ud589\ub41c \uc870\ud569\uc740\ud589\uc758 \uc218\ud45c\uc640 \uac19\uc774 \uc9c1\uc811 \ud55c\uad6d\uc740\ud589\uc758 \uc870\ud569\uc740\ud589 \ub2f9\uc88c\uc608\uae08\uacc4\uc88c\uc5d0 \uc608\uc785(\u9810\u5165)\ud558\ub294 \uacbd\uc6b0\uc5d0 \ud55c\ud55c\ub2e4(\uc5b4\uad50\uaddc 22\uc870). \uc5b4\uc74c\uad50\ud658\uc18c\ub97c \uc81c\uc2dc\uc7a5\uc18c\ub85c \ud558\ub294 \ub370\ub294 \ub450 \uac00\uc9c0 \uacbd\uc6b0\uac00 \uc788\ub2e4. \uccab\uc9f8\ub294 \uc218\ud45c\uc18c\uc9c0\uc778\uc774 \uc790\uae30\uc758 \uac70\ub798\uc740\ud589\uc5d0 \uadf8 \uc218\ud45c\ub97c \uc608\uae08\ud558\ub294 \uacbd\uc6b0\uc774\uace0, \ub458\uc9f8\ub294 \uc21c\uc804\ud788 \uac70\ub798\uc740\ud589\uc774 \uadf8 \uc218\ud45c\uc0c1\uc758 \uad8c\ub9ac\uc790\ub85c\uc11c \uc5b4\uc74c\uad50\ud658\uc18c\uc5d0 \uc9c0\uae09\uc81c\uc2dc\ub97c \ud558\uace0. \ud6c4\uc790\uc758 \uacbd\uc6b0\ub294 \uadf8 \uac70\ub798\uc740\ud589\uc740 \uc21c\uc804\ud788 \ub300\ub9ac\uc778\uc73c\ub85c\uc11c \uc5b4\uc74c\uad50\ud658\uc18c\uc5d0 \uc9c0\uae09\uc81c\uc2dc\ud568\uc73c\ub85c\uc368 \ucd94\uc2ec\ud558\ub294 \uc704\uc784\uc0ac\ubb34\ub97c \ub2f4\ub2f9\ud55c\ub2e4. \uc5b4\uc74c\uad50\ud658\uc18c\uc5d0 \uac00\uc785\ud55c \uc870\ud569\uc740\ud589\uc740 \uc11c\ub85c \ub2e4\ub978 \uc740\ud589\uc73c\ub85c\ubd80\ud130 \ucd94\uc2ec\ud558\uc5ec \uc628 \uc218\ud45c\ub97c \uc5b4\uc74c\uad50\ud658\uc18c\uc5d0 \uc18c\uc815\uad50\ud658\uc2dc\uac04\uc5d0 \uc81c\uc2dc\uad50\ud658\ud558\uc5ec \uc218\ucde8\ucd1d\uc561(\u53d7\u53d6\u7e3d\u984d)\uc758 \ucc28\uc561\ub9cc\uc744 \uc811\uc218\ud558\uace0 \ud604\uae08\uc5d0 \uc758\ud55c \uc9c0\uae09\uc5d0 \uac08\uc74c\ud55c\ub2e4. \uc870\ud569\uc740\ud589\uc758 \ub300\ud45c\uc790 \uc989 \uad50\ud658\uc6d0\uc740 \uad50\ud658\uc5d0 \ud68c\ubd80\ud560 \uc218\ud45c\ub97c \uc0c1\ub300 \uc740\ud589\ubcc4\ub85c \uad6c\ubd84\ud558\uc5ec \uad50\ud658\uc5b4\uc74c \ucca8\ud45c(\u6dfb\u8868) \ubc0f \uad50\ud658\uc5b4\uc74c \ucc28\uac10\ud45c\uc758 \ub300\ubcc0(\u8cb8\u908a)\uc5d0 \uae08\uc561\uacfc \ub9e4\uc218\ub97c \uae30\uc785\ud558\uc5ec \uad50\ud658\uac1c\uc2dc\uc2dc\uac01 20\ubd84 \uc804\uac00\uc9c0 \ub2f9\uad50\ud658\uc18c\uc5d0 \uc9c0\ucc38\ud574\uc57c \ud55c\ub2e4. \uad50\ud658\uc6d0\uc740 \uc218\ud45c\uc5d0 \uad50\ud658\uc5b4\uc74c \ucca8\ud45c\ub97c \ucca8\ubd80\ud558\uc5ec \uc774\ub97c \uc0c1\ud638 \uad50\ud658\ud558\uace0 \uad50\ud658\uc5b4\uc74c \ucc28\uc561\ud45c\u00b7\uad50\ud658\uc5b4\uc74c\ucc28\uc561 \ub300\uccb4\uccad\uad6c\uc11c\u00b7\uad50\ud658\uc218\uc785\uc5b4\uc74c\uba85\uc138\uc11c \ub4f1\uc744 \uc791\uc131\ud558\uc5ec \uad50\ud658\ubd80\uc7a5\uc744 \uacbd\uc720\ud558\uc5ec \ud55c\uad6d\uc740\ud589\uc5d0 \uc81c\ucd9c\ud55c\ub2e4. \uadf8\ub9ac\ud558\uc5ec \uc218\ud45c\uc758 \uacb0\uc81c\ub294 \uac1c\uac1c\uc758 \uc218\ud45c\uac00 \uc544\ub2c8\uace0 \ubaa8\ub4e0 \uc218\ud45c\ub97c \uc77c\uad04\ud558\uc5ec \ubaa8\ub4e0 \uc740\ud589 \uc0c1\ud638\uac04\uc5d0 \uc9d1\ub2e8\uc801\uc73c\ub85c \ucc28\uac10\uacc4\uc0b0\uc774 \ud589\ud558\uc5ec\uc9c4\ub2e4.", "answer": "\uc5b4\uc74c\uad50\ud658\uc18c", "sentence": "\ud6c4\uc790\uc758 \uacbd\uc6b0 \uc6b0\uc120 \uc758\ub8b0\ub97c \ubc1b\uc740 \uc9c0\uae09\uc740\ud589\ubcc4\ub85c \ubd84\ub958\ud558\uc5ec \uae08\uc561\u00b7\ub9e4\uc218(\u679a\u6578) \ub4f1\uc744 \ubcc4\ub3c4 \uad50\ud658\ud45c\uc5d0 \uae30\uc785\ud558\uc5ec \uc5b4\uc74c\uad50\ud658\uc18c \uc5d0 \uc81c\ucd9c\ud55c\ub2e4.", "paragraph_sentence": "\ubc1c\ud589\uc778\uacfc \uc18c\uc9c0\uc778\uc774 \ub3d9\uc77c\uc740\ud589\uc5d0 \uac70\ub798\ud558\uace0 \uc788\ub294 \uacbd\uc6b0\uc5d0\ub294 \uc9c1\uc811 \uc9c0\uae09\uc740\ud589\uc5d0 \uc81c\uae30\ud558\uc5ec \uadf8 \uc740\ud589\uc73c\ub85c\ubd80\ud130 \uc9c0\uae09\uc744 \ubc1b\uc9c0\ub9cc \ub300\ubd80\ubd84\uc740 \uc9c0\uae09\uc740\ud589 \uc774\uc678\uc5d0 \ucd94\uc2ec\uc744 \uc758\ub8b0\ud55c\ub2e4. \ud6c4\uc790\uc758 \uacbd\uc6b0 \uc6b0\uc120 \uc758\ub8b0\ub97c \ubc1b\uc740 \uc9c0\uae09\uc740\ud589\ubcc4\ub85c \ubd84\ub958\ud558\uc5ec \uae08\uc561\u00b7\ub9e4\uc218(\u679a\u6578) \ub4f1\uc744 \ubcc4\ub3c4 \uad50\ud658\ud45c\uc5d0 \uae30\uc785\ud558\uc5ec \uc5b4\uc74c\uad50\ud658\uc18c \uc5d0 \uc81c\ucd9c\ud55c\ub2e4. \uc774 \uad50\ud658\uc18c\ub294 \ud2b9\ud788 \uc5b4\uc74c\u00b7\uc218\ud45c\ubc95\uc0c1\uc758 \uc5c4\ubc00\ud55c \uaddc\uc815\uc740 \uc544\ub2c8\uba70(\uc218\ud45c 31\uc870, 69\uc870 \ub4f1), \uc8fc\ub85c \uac70\ub798\uc2e4\ubb34\uc0c1\uc758 \ud544\uc694\uc131\uc5d0\uc11c \ubc95\ubb34\ubd80\ub839(340\ud638, 1990. 3. 6)\uc5d0 \uae30\ud558\uc5ec \ubc95\ubb34\ubd80\uc7a5\uad00\uc774 \uc9c0\uc815\ud55c\ub2e4. \uc608\ucee8\ub300, '\uc11c\uc6b8\uc5b4\uc74c\uad50\ud658\uc18c'\uc758 \uc18c\uc7ac\uc9c0\ub294 '\uc11c\uc6b8\ud2b9\ubcc4\uc2dc'\uc774\uba70, \ud604\uc7ac\ub85c\ub294 \uc11c\uc6b8\ud2b9\ubcc4\uc2dc \uac15\ub0a8\uad6c \uc5ed\uc0bc\ub3d9 717\ubc88\uc9c0 \uae08\uc735\uacb0\uc81c\uc6d0 \ub0b4\uc5d0 \uc124\uce58\ub418\uc5b4 \uc788\ub2e4. \uc5b4\uc74c\uad50\ud658\uc18c\uc5d0 \uc218\ud45c\ub97c \uc81c\uc2dc\ud558\ub294 \uac83\uc740 \uad50\ud658\uc18c\uc5d0 \uac00\uc785\ud55c \uc740\ud589(\uc870\ud569\uc740\ud589)\uc5d0 \ud55c\ud558\uba70 \ub3d9\uc2dc\uc5d0 \uac01 \uc870\ud569\uc740\ud589 \uc55e\uc73c\ub85c \ubc1c\ud589\ub41c \uc218\ud45c(\uc5b4\uc74c)\ub294 \uc774\ub97c \uc804\ubd80 \uad50\ud658\uc5d0 \ud68c\ubd80\ud558\uc5ec \uacb0\uc81c\ud560 \uc758\ubb34\uac00 \uc788\ub2e4. \uc5b4\uc74c\uad50\ud658\uc18c\uc758 \uad50\ud658\uc744 \uacbd\uc6b0\ud558\uc9c0 \uc544\ub2c8\ud55c \uac83\uc740 \ud55c\uad6d\uc740\ud589 \uc55e\uc73c\ub85c \ubc1c\ud589\ub41c \uc870\ud569\uc740\ud589\uc758 \uc218\ud45c\uc640 \uac19\uc774 \uc9c1\uc811 \ud55c\uad6d\uc740\ud589\uc758 \uc870\ud569\uc740\ud589 \ub2f9\uc88c\uc608\uae08\uacc4\uc88c\uc5d0 \uc608\uc785(\u9810\u5165)\ud558\ub294 \uacbd\uc6b0\uc5d0 \ud55c\ud55c\ub2e4(\uc5b4\uad50\uaddc 22\uc870). \uc5b4\uc74c\uad50\ud658\uc18c\ub97c \uc81c\uc2dc\uc7a5\uc18c\ub85c \ud558\ub294 \ub370\ub294 \ub450 \uac00\uc9c0 \uacbd\uc6b0\uac00 \uc788\ub2e4. \uccab\uc9f8\ub294 \uc218\ud45c\uc18c\uc9c0\uc778\uc774 \uc790\uae30\uc758 \uac70\ub798\uc740\ud589\uc5d0 \uadf8 \uc218\ud45c\ub97c \uc608\uae08\ud558\ub294 \uacbd\uc6b0\uc774\uace0, \ub458\uc9f8\ub294 \uc21c\uc804\ud788 \uac70\ub798\uc740\ud589\uc774 \uadf8 \uc218\ud45c\uc0c1\uc758 \uad8c\ub9ac\uc790\ub85c\uc11c \uc5b4\uc74c\uad50\ud658\uc18c\uc5d0 \uc9c0\uae09\uc81c\uc2dc\ub97c \ud558\uace0. \ud6c4\uc790\uc758 \uacbd\uc6b0\ub294 \uadf8 \uac70\ub798\uc740\ud589\uc740 \uc21c\uc804\ud788 \ub300\ub9ac\uc778\uc73c\ub85c\uc11c \uc5b4\uc74c\uad50\ud658\uc18c\uc5d0 \uc9c0\uae09\uc81c\uc2dc\ud568\uc73c\ub85c\uc368 \ucd94\uc2ec\ud558\ub294 \uc704\uc784\uc0ac\ubb34\ub97c \ub2f4\ub2f9\ud55c\ub2e4. \uc5b4\uc74c\uad50\ud658\uc18c\uc5d0 \uac00\uc785\ud55c \uc870\ud569\uc740\ud589\uc740 \uc11c\ub85c \ub2e4\ub978 \uc740\ud589\uc73c\ub85c\ubd80\ud130 \ucd94\uc2ec\ud558\uc5ec \uc628 \uc218\ud45c\ub97c \uc5b4\uc74c\uad50\ud658\uc18c\uc5d0 \uc18c\uc815\uad50\ud658\uc2dc\uac04\uc5d0 \uc81c\uc2dc\uad50\ud658\ud558\uc5ec \uc218\ucde8\ucd1d\uc561(\u53d7\u53d6\u7e3d\u984d)\uc758 \ucc28\uc561\ub9cc\uc744 \uc811\uc218\ud558\uace0 \ud604\uae08\uc5d0 \uc758\ud55c \uc9c0\uae09\uc5d0 \uac08\uc74c\ud55c\ub2e4. \uc870\ud569\uc740\ud589\uc758 \ub300\ud45c\uc790 \uc989 \uad50\ud658\uc6d0\uc740 \uad50\ud658\uc5d0 \ud68c\ubd80\ud560 \uc218\ud45c\ub97c \uc0c1\ub300 \uc740\ud589\ubcc4\ub85c \uad6c\ubd84\ud558\uc5ec \uad50\ud658\uc5b4\uc74c \ucca8\ud45c(\u6dfb\u8868) \ubc0f \uad50\ud658\uc5b4\uc74c \ucc28\uac10\ud45c\uc758 \ub300\ubcc0(\u8cb8\u908a)\uc5d0 \uae08\uc561\uacfc \ub9e4\uc218\ub97c \uae30\uc785\ud558\uc5ec \uad50\ud658\uac1c\uc2dc\uc2dc\uac01 20\ubd84 \uc804\uac00\uc9c0 \ub2f9\uad50\ud658\uc18c\uc5d0 \uc9c0\ucc38\ud574\uc57c \ud55c\ub2e4. \uad50\ud658\uc6d0\uc740 \uc218\ud45c\uc5d0 \uad50\ud658\uc5b4\uc74c \ucca8\ud45c\ub97c \ucca8\ubd80\ud558\uc5ec \uc774\ub97c \uc0c1\ud638 \uad50\ud658\ud558\uace0 \uad50\ud658\uc5b4\uc74c \ucc28\uc561\ud45c\u00b7\uad50\ud658\uc5b4\uc74c\ucc28\uc561 \ub300\uccb4\uccad\uad6c\uc11c\u00b7\uad50\ud658\uc218\uc785\uc5b4\uc74c\uba85\uc138\uc11c \ub4f1\uc744 \uc791\uc131\ud558\uc5ec \uad50\ud658\ubd80\uc7a5\uc744 \uacbd\uc720\ud558\uc5ec \ud55c\uad6d\uc740\ud589\uc5d0 \uc81c\ucd9c\ud55c\ub2e4. \uadf8\ub9ac\ud558\uc5ec \uc218\ud45c\uc758 \uacb0\uc81c\ub294 \uac1c\uac1c\uc758 \uc218\ud45c\uac00 \uc544\ub2c8\uace0 \ubaa8\ub4e0 \uc218\ud45c\ub97c \uc77c\uad04\ud558\uc5ec \ubaa8\ub4e0 \uc740\ud589 \uc0c1\ud638\uac04\uc5d0 \uc9d1\ub2e8\uc801\uc73c\ub85c \ucc28\uac10\uacc4\uc0b0\uc774 \ud589\ud558\uc5ec\uc9c4\ub2e4.", "paragraph_answer": "\ubc1c\ud589\uc778\uacfc \uc18c\uc9c0\uc778\uc774 \ub3d9\uc77c\uc740\ud589\uc5d0 \uac70\ub798\ud558\uace0 \uc788\ub294 \uacbd\uc6b0\uc5d0\ub294 \uc9c1\uc811 \uc9c0\uae09\uc740\ud589\uc5d0 \uc81c\uae30\ud558\uc5ec \uadf8 \uc740\ud589\uc73c\ub85c\ubd80\ud130 \uc9c0\uae09\uc744 \ubc1b\uc9c0\ub9cc \ub300\ubd80\ubd84\uc740 \uc9c0\uae09\uc740\ud589 \uc774\uc678\uc5d0 \ucd94\uc2ec\uc744 \uc758\ub8b0\ud55c\ub2e4. \ud6c4\uc790\uc758 \uacbd\uc6b0 \uc6b0\uc120 \uc758\ub8b0\ub97c \ubc1b\uc740 \uc9c0\uae09\uc740\ud589\ubcc4\ub85c \ubd84\ub958\ud558\uc5ec \uae08\uc561\u00b7\ub9e4\uc218(\u679a\u6578) \ub4f1\uc744 \ubcc4\ub3c4 \uad50\ud658\ud45c\uc5d0 \uae30\uc785\ud558\uc5ec \uc5b4\uc74c\uad50\ud658\uc18c \uc5d0 \uc81c\ucd9c\ud55c\ub2e4. \uc774 \uad50\ud658\uc18c\ub294 \ud2b9\ud788 \uc5b4\uc74c\u00b7\uc218\ud45c\ubc95\uc0c1\uc758 \uc5c4\ubc00\ud55c \uaddc\uc815\uc740 \uc544\ub2c8\uba70(\uc218\ud45c 31\uc870, 69\uc870 \ub4f1), \uc8fc\ub85c \uac70\ub798\uc2e4\ubb34\uc0c1\uc758 \ud544\uc694\uc131\uc5d0\uc11c \ubc95\ubb34\ubd80\ub839(340\ud638, 1990. 3. 6)\uc5d0 \uae30\ud558\uc5ec \ubc95\ubb34\ubd80\uc7a5\uad00\uc774 \uc9c0\uc815\ud55c\ub2e4. \uc608\ucee8\ub300, '\uc11c\uc6b8\uc5b4\uc74c\uad50\ud658\uc18c'\uc758 \uc18c\uc7ac\uc9c0\ub294 '\uc11c\uc6b8\ud2b9\ubcc4\uc2dc'\uc774\uba70, \ud604\uc7ac\ub85c\ub294 \uc11c\uc6b8\ud2b9\ubcc4\uc2dc \uac15\ub0a8\uad6c \uc5ed\uc0bc\ub3d9 717\ubc88\uc9c0 \uae08\uc735\uacb0\uc81c\uc6d0 \ub0b4\uc5d0 \uc124\uce58\ub418\uc5b4 \uc788\ub2e4. \uc5b4\uc74c\uad50\ud658\uc18c\uc5d0 \uc218\ud45c\ub97c \uc81c\uc2dc\ud558\ub294 \uac83\uc740 \uad50\ud658\uc18c\uc5d0 \uac00\uc785\ud55c \uc740\ud589(\uc870\ud569\uc740\ud589)\uc5d0 \ud55c\ud558\uba70 \ub3d9\uc2dc\uc5d0 \uac01 \uc870\ud569\uc740\ud589 \uc55e\uc73c\ub85c \ubc1c\ud589\ub41c \uc218\ud45c(\uc5b4\uc74c)\ub294 \uc774\ub97c \uc804\ubd80 \uad50\ud658\uc5d0 \ud68c\ubd80\ud558\uc5ec \uacb0\uc81c\ud560 \uc758\ubb34\uac00 \uc788\ub2e4. \uc5b4\uc74c\uad50\ud658\uc18c\uc758 \uad50\ud658\uc744 \uacbd\uc6b0\ud558\uc9c0 \uc544\ub2c8\ud55c \uac83\uc740 \ud55c\uad6d\uc740\ud589 \uc55e\uc73c\ub85c \ubc1c\ud589\ub41c \uc870\ud569\uc740\ud589\uc758 \uc218\ud45c\uc640 \uac19\uc774 \uc9c1\uc811 \ud55c\uad6d\uc740\ud589\uc758 \uc870\ud569\uc740\ud589 \ub2f9\uc88c\uc608\uae08\uacc4\uc88c\uc5d0 \uc608\uc785(\u9810\u5165)\ud558\ub294 \uacbd\uc6b0\uc5d0 \ud55c\ud55c\ub2e4(\uc5b4\uad50\uaddc 22\uc870). \uc5b4\uc74c\uad50\ud658\uc18c\ub97c \uc81c\uc2dc\uc7a5\uc18c\ub85c \ud558\ub294 \ub370\ub294 \ub450 \uac00\uc9c0 \uacbd\uc6b0\uac00 \uc788\ub2e4. \uccab\uc9f8\ub294 \uc218\ud45c\uc18c\uc9c0\uc778\uc774 \uc790\uae30\uc758 \uac70\ub798\uc740\ud589\uc5d0 \uadf8 \uc218\ud45c\ub97c \uc608\uae08\ud558\ub294 \uacbd\uc6b0\uc774\uace0, \ub458\uc9f8\ub294 \uc21c\uc804\ud788 \uac70\ub798\uc740\ud589\uc774 \uadf8 \uc218\ud45c\uc0c1\uc758 \uad8c\ub9ac\uc790\ub85c\uc11c \uc5b4\uc74c\uad50\ud658\uc18c\uc5d0 \uc9c0\uae09\uc81c\uc2dc\ub97c \ud558\uace0. \ud6c4\uc790\uc758 \uacbd\uc6b0\ub294 \uadf8 \uac70\ub798\uc740\ud589\uc740 \uc21c\uc804\ud788 \ub300\ub9ac\uc778\uc73c\ub85c\uc11c \uc5b4\uc74c\uad50\ud658\uc18c\uc5d0 \uc9c0\uae09\uc81c\uc2dc\ud568\uc73c\ub85c\uc368 \ucd94\uc2ec\ud558\ub294 \uc704\uc784\uc0ac\ubb34\ub97c \ub2f4\ub2f9\ud55c\ub2e4. \uc5b4\uc74c\uad50\ud658\uc18c\uc5d0 \uac00\uc785\ud55c \uc870\ud569\uc740\ud589\uc740 \uc11c\ub85c \ub2e4\ub978 \uc740\ud589\uc73c\ub85c\ubd80\ud130 \ucd94\uc2ec\ud558\uc5ec \uc628 \uc218\ud45c\ub97c \uc5b4\uc74c\uad50\ud658\uc18c\uc5d0 \uc18c\uc815\uad50\ud658\uc2dc\uac04\uc5d0 \uc81c\uc2dc\uad50\ud658\ud558\uc5ec \uc218\ucde8\ucd1d\uc561(\u53d7\u53d6\u7e3d\u984d)\uc758 \ucc28\uc561\ub9cc\uc744 \uc811\uc218\ud558\uace0 \ud604\uae08\uc5d0 \uc758\ud55c \uc9c0\uae09\uc5d0 \uac08\uc74c\ud55c\ub2e4. \uc870\ud569\uc740\ud589\uc758 \ub300\ud45c\uc790 \uc989 \uad50\ud658\uc6d0\uc740 \uad50\ud658\uc5d0 \ud68c\ubd80\ud560 \uc218\ud45c\ub97c \uc0c1\ub300 \uc740\ud589\ubcc4\ub85c \uad6c\ubd84\ud558\uc5ec \uad50\ud658\uc5b4\uc74c \ucca8\ud45c(\u6dfb\u8868) \ubc0f \uad50\ud658\uc5b4\uc74c \ucc28\uac10\ud45c\uc758 \ub300\ubcc0(\u8cb8\u908a)\uc5d0 \uae08\uc561\uacfc \ub9e4\uc218\ub97c \uae30\uc785\ud558\uc5ec \uad50\ud658\uac1c\uc2dc\uc2dc\uac01 20\ubd84 \uc804\uac00\uc9c0 \ub2f9\uad50\ud658\uc18c\uc5d0 \uc9c0\ucc38\ud574\uc57c \ud55c\ub2e4. \uad50\ud658\uc6d0\uc740 \uc218\ud45c\uc5d0 \uad50\ud658\uc5b4\uc74c \ucca8\ud45c\ub97c \ucca8\ubd80\ud558\uc5ec \uc774\ub97c \uc0c1\ud638 \uad50\ud658\ud558\uace0 \uad50\ud658\uc5b4\uc74c \ucc28\uc561\ud45c\u00b7\uad50\ud658\uc5b4\uc74c\ucc28\uc561 \ub300\uccb4\uccad\uad6c\uc11c\u00b7\uad50\ud658\uc218\uc785\uc5b4\uc74c\uba85\uc138\uc11c \ub4f1\uc744 \uc791\uc131\ud558\uc5ec \uad50\ud658\ubd80\uc7a5\uc744 \uacbd\uc720\ud558\uc5ec \ud55c\uad6d\uc740\ud589\uc5d0 \uc81c\ucd9c\ud55c\ub2e4. \uadf8\ub9ac\ud558\uc5ec \uc218\ud45c\uc758 \uacb0\uc81c\ub294 \uac1c\uac1c\uc758 \uc218\ud45c\uac00 \uc544\ub2c8\uace0 \ubaa8\ub4e0 \uc218\ud45c\ub97c \uc77c\uad04\ud558\uc5ec \ubaa8\ub4e0 \uc740\ud589 \uc0c1\ud638\uac04\uc5d0 \uc9d1\ub2e8\uc801\uc73c\ub85c \ucc28\uac10\uacc4\uc0b0\uc774 \ud589\ud558\uc5ec\uc9c4\ub2e4.", "sentence_answer": "\ud6c4\uc790\uc758 \uacbd\uc6b0 \uc6b0\uc120 \uc758\ub8b0\ub97c \ubc1b\uc740 \uc9c0\uae09\uc740\ud589\ubcc4\ub85c \ubd84\ub958\ud558\uc5ec \uae08\uc561\u00b7\ub9e4\uc218(\u679a\u6578) \ub4f1\uc744 \ubcc4\ub3c4 \uad50\ud658\ud45c\uc5d0 \uae30\uc785\ud558\uc5ec \uc5b4\uc74c\uad50\ud658\uc18c \uc5d0 \uc81c\ucd9c\ud55c\ub2e4.", "paragraph_id": "6536952-4-1", "paragraph_question": "question: \uc218\ud45c\uc758 \ubc1c\ud589\uc778\uacfc \uc18c\uc9c0\uc778\uc774 \uac70\ub798\ud558\uace0 \uc788\ub294 \uc740\ud589\uc774 \uac19\uc544 \ucd94\uc2ec\uc744 \uc758\ub8b0\ud560 \ub54c \ubcc4\ub3c4 \uad50\ud658\ud45c\uc5d0 \uae30\uc785\ud558\uc5ec \uc81c\ucd9c\ud558\ub294 \uacf3\uc740?, context: \ubc1c\ud589\uc778\uacfc \uc18c\uc9c0\uc778\uc774 \ub3d9\uc77c\uc740\ud589\uc5d0 \uac70\ub798\ud558\uace0 \uc788\ub294 \uacbd\uc6b0\uc5d0\ub294 \uc9c1\uc811 \uc9c0\uae09\uc740\ud589\uc5d0 \uc81c\uae30\ud558\uc5ec \uadf8 \uc740\ud589\uc73c\ub85c\ubd80\ud130 \uc9c0\uae09\uc744 \ubc1b\uc9c0\ub9cc \ub300\ubd80\ubd84\uc740 \uc9c0\uae09\uc740\ud589 \uc774\uc678\uc5d0 \ucd94\uc2ec\uc744 \uc758\ub8b0\ud55c\ub2e4. \ud6c4\uc790\uc758 \uacbd\uc6b0 \uc6b0\uc120 \uc758\ub8b0\ub97c \ubc1b\uc740 \uc9c0\uae09\uc740\ud589\ubcc4\ub85c \ubd84\ub958\ud558\uc5ec \uae08\uc561\u00b7\ub9e4\uc218(\u679a\u6578) \ub4f1\uc744 \ubcc4\ub3c4 \uad50\ud658\ud45c\uc5d0 \uae30\uc785\ud558\uc5ec \uc5b4\uc74c\uad50\ud658\uc18c\uc5d0 \uc81c\ucd9c\ud55c\ub2e4. \uc774 \uad50\ud658\uc18c\ub294 \ud2b9\ud788 \uc5b4\uc74c\u00b7\uc218\ud45c\ubc95\uc0c1\uc758 \uc5c4\ubc00\ud55c \uaddc\uc815\uc740 \uc544\ub2c8\uba70(\uc218\ud45c 31\uc870, 69\uc870 \ub4f1), \uc8fc\ub85c \uac70\ub798\uc2e4\ubb34\uc0c1\uc758 \ud544\uc694\uc131\uc5d0\uc11c \ubc95\ubb34\ubd80\ub839(340\ud638, 1990. 3. 6)\uc5d0 \uae30\ud558\uc5ec \ubc95\ubb34\ubd80\uc7a5\uad00\uc774 \uc9c0\uc815\ud55c\ub2e4. \uc608\ucee8\ub300, '\uc11c\uc6b8\uc5b4\uc74c\uad50\ud658\uc18c'\uc758 \uc18c\uc7ac\uc9c0\ub294 '\uc11c\uc6b8\ud2b9\ubcc4\uc2dc'\uc774\uba70, \ud604\uc7ac\ub85c\ub294 \uc11c\uc6b8\ud2b9\ubcc4\uc2dc \uac15\ub0a8\uad6c \uc5ed\uc0bc\ub3d9 717\ubc88\uc9c0 \uae08\uc735\uacb0\uc81c\uc6d0 \ub0b4\uc5d0 \uc124\uce58\ub418\uc5b4 \uc788\ub2e4. \uc5b4\uc74c\uad50\ud658\uc18c\uc5d0 \uc218\ud45c\ub97c \uc81c\uc2dc\ud558\ub294 \uac83\uc740 \uad50\ud658\uc18c\uc5d0 \uac00\uc785\ud55c \uc740\ud589(\uc870\ud569\uc740\ud589)\uc5d0 \ud55c\ud558\uba70 \ub3d9\uc2dc\uc5d0 \uac01 \uc870\ud569\uc740\ud589 \uc55e\uc73c\ub85c \ubc1c\ud589\ub41c \uc218\ud45c(\uc5b4\uc74c)\ub294 \uc774\ub97c \uc804\ubd80 \uad50\ud658\uc5d0 \ud68c\ubd80\ud558\uc5ec \uacb0\uc81c\ud560 \uc758\ubb34\uac00 \uc788\ub2e4. \uc5b4\uc74c\uad50\ud658\uc18c\uc758 \uad50\ud658\uc744 \uacbd\uc6b0\ud558\uc9c0 \uc544\ub2c8\ud55c \uac83\uc740 \ud55c\uad6d\uc740\ud589 \uc55e\uc73c\ub85c \ubc1c\ud589\ub41c \uc870\ud569\uc740\ud589\uc758 \uc218\ud45c\uc640 \uac19\uc774 \uc9c1\uc811 \ud55c\uad6d\uc740\ud589\uc758 \uc870\ud569\uc740\ud589 \ub2f9\uc88c\uc608\uae08\uacc4\uc88c\uc5d0 \uc608\uc785(\u9810\u5165)\ud558\ub294 \uacbd\uc6b0\uc5d0 \ud55c\ud55c\ub2e4(\uc5b4\uad50\uaddc 22\uc870). \uc5b4\uc74c\uad50\ud658\uc18c\ub97c \uc81c\uc2dc\uc7a5\uc18c\ub85c \ud558\ub294 \ub370\ub294 \ub450 \uac00\uc9c0 \uacbd\uc6b0\uac00 \uc788\ub2e4. \uccab\uc9f8\ub294 \uc218\ud45c\uc18c\uc9c0\uc778\uc774 \uc790\uae30\uc758 \uac70\ub798\uc740\ud589\uc5d0 \uadf8 \uc218\ud45c\ub97c \uc608\uae08\ud558\ub294 \uacbd\uc6b0\uc774\uace0, \ub458\uc9f8\ub294 \uc21c\uc804\ud788 \uac70\ub798\uc740\ud589\uc774 \uadf8 \uc218\ud45c\uc0c1\uc758 \uad8c\ub9ac\uc790\ub85c\uc11c \uc5b4\uc74c\uad50\ud658\uc18c\uc5d0 \uc9c0\uae09\uc81c\uc2dc\ub97c \ud558\uace0. \ud6c4\uc790\uc758 \uacbd\uc6b0\ub294 \uadf8 \uac70\ub798\uc740\ud589\uc740 \uc21c\uc804\ud788 \ub300\ub9ac\uc778\uc73c\ub85c\uc11c \uc5b4\uc74c\uad50\ud658\uc18c\uc5d0 \uc9c0\uae09\uc81c\uc2dc\ud568\uc73c\ub85c\uc368 \ucd94\uc2ec\ud558\ub294 \uc704\uc784\uc0ac\ubb34\ub97c \ub2f4\ub2f9\ud55c\ub2e4. \uc5b4\uc74c\uad50\ud658\uc18c\uc5d0 \uac00\uc785\ud55c \uc870\ud569\uc740\ud589\uc740 \uc11c\ub85c \ub2e4\ub978 \uc740\ud589\uc73c\ub85c\ubd80\ud130 \ucd94\uc2ec\ud558\uc5ec \uc628 \uc218\ud45c\ub97c \uc5b4\uc74c\uad50\ud658\uc18c\uc5d0 \uc18c\uc815\uad50\ud658\uc2dc\uac04\uc5d0 \uc81c\uc2dc\uad50\ud658\ud558\uc5ec \uc218\ucde8\ucd1d\uc561(\u53d7\u53d6\u7e3d\u984d)\uc758 \ucc28\uc561\ub9cc\uc744 \uc811\uc218\ud558\uace0 \ud604\uae08\uc5d0 \uc758\ud55c \uc9c0\uae09\uc5d0 \uac08\uc74c\ud55c\ub2e4. \uc870\ud569\uc740\ud589\uc758 \ub300\ud45c\uc790 \uc989 \uad50\ud658\uc6d0\uc740 \uad50\ud658\uc5d0 \ud68c\ubd80\ud560 \uc218\ud45c\ub97c \uc0c1\ub300 \uc740\ud589\ubcc4\ub85c \uad6c\ubd84\ud558\uc5ec \uad50\ud658\uc5b4\uc74c \ucca8\ud45c(\u6dfb\u8868) \ubc0f \uad50\ud658\uc5b4\uc74c \ucc28\uac10\ud45c\uc758 \ub300\ubcc0(\u8cb8\u908a)\uc5d0 \uae08\uc561\uacfc \ub9e4\uc218\ub97c \uae30\uc785\ud558\uc5ec \uad50\ud658\uac1c\uc2dc\uc2dc\uac01 20\ubd84 \uc804\uac00\uc9c0 \ub2f9\uad50\ud658\uc18c\uc5d0 \uc9c0\ucc38\ud574\uc57c \ud55c\ub2e4. \uad50\ud658\uc6d0\uc740 \uc218\ud45c\uc5d0 \uad50\ud658\uc5b4\uc74c \ucca8\ud45c\ub97c \ucca8\ubd80\ud558\uc5ec \uc774\ub97c \uc0c1\ud638 \uad50\ud658\ud558\uace0 \uad50\ud658\uc5b4\uc74c \ucc28\uc561\ud45c\u00b7\uad50\ud658\uc5b4\uc74c\ucc28\uc561 \ub300\uccb4\uccad\uad6c\uc11c\u00b7\uad50\ud658\uc218\uc785\uc5b4\uc74c\uba85\uc138\uc11c \ub4f1\uc744 \uc791\uc131\ud558\uc5ec \uad50\ud658\ubd80\uc7a5\uc744 \uacbd\uc720\ud558\uc5ec \ud55c\uad6d\uc740\ud589\uc5d0 \uc81c\ucd9c\ud55c\ub2e4. \uadf8\ub9ac\ud558\uc5ec \uc218\ud45c\uc758 \uacb0\uc81c\ub294 \uac1c\uac1c\uc758 \uc218\ud45c\uac00 \uc544\ub2c8\uace0 \ubaa8\ub4e0 \uc218\ud45c\ub97c \uc77c\uad04\ud558\uc5ec \ubaa8\ub4e0 \uc740\ud589 \uc0c1\ud638\uac04\uc5d0 \uc9d1\ub2e8\uc801\uc73c\ub85c \ucc28\uac10\uacc4\uc0b0\uc774 \ud589\ud558\uc5ec\uc9c4\ub2e4."} {"question": "\uc911\uad6d\uc5d0\uc11c \uc77c\uc5b4\ub0ac\ub358 \uae09\uc9c4\uc801\uc778 \uc0ac\ud68c\uc8fc\uc758 \uc815\ucc45\uc911 \ud558\ub098\ub85c \ub300\ub7c9\uc758 \uc544\uc0ac\uc790\ub97c \ubc1c\uc0dd\uc2dc\ucf30\ub358 \uc6b4\ub3d9\uc740?", "paragraph": "\ud558\uc9c0\ub9cc, \uae09\uc9c4\uc8fc\uc758 \uc815\uce58 \uc6b4\ub3d9\uc774 \uae0d\uc815\uc801\uc778 \ud6a8\uacfc\ub9cc \ubcf4\uc600\ub358 \uac83\uc740 \uc544\ub2c8\uc5c8\ub2e4. \uae09\uc9c4\uc801\uc778 \uc815\uce58 \uc6b4\ub3d9\uc740 \uc790\uce6b\ud558\uba74 \uc0ac\ud68c\uc801 \uc77c\ud0c8 \ud604\uc0c1\uc744 \ub9cc\uc5f0\ud558\uac8c \ud558\ub294 \uc6d0\uc778\uc774 \ub418\uae30\ub3c4 \ud55c\ub2e4. \ub610\ud55c \uadf8 \uacfc\uc815\uc5d0\uc11c \uacaa\ub294 \ud53c\ud574\ub3c4 \ud06c\ub2e4. \uc77c\ub840\ub85c \uc911\uad6d\uc5d0\uc11c \uc77c\uc5b4\ub0ac\ub358 \uae09\uc9c4\uc801\uc778 \uc0ac\ud68c\uc8fc\uc758 \uc815\ucc45\uc778 \ub300\uc57d\uc9c4\uc6b4\ub3d9\uc740 \ub300\ub7c9 \uc544\uc0ac\uc790\ub97c \uc18d\ucd9c\uc2dc\ucf30\ub2e4. \uc624\ub298\ub0a0\uc5d0 \uae09\uc9c4\uc801\uc778 \uc0ac\uc0c1 \uc6b4\ub3d9\uc774 \uc81c\ub300\ub85c \uc81c\uc5b4\ub418\uc9c0 \ubabb\ud560 \uacbd\uc6b0 \uc5b4\ub5a0\ud55c \uc545\uc601\ud5a5\uc744 \ub07c\uce58\ub294 \uc9c0 \uc798 \ubcf4\uc5ec\uc8fc\ub294 \uc0ac\ub840\uc774\ub2e4. \uae09\uc9c4\uc801 \uc790\uc720\uc8fc\uc758\ud654\ub294 \uc790\uc720\uc8fc\uc758 \uacbd\uc81c\uc758 \uae30\ubc18\uc778 \uc790\ubcf8\uc8fc\uc758\ub97c \ub9f9\uc2e0\ud558\ub294 \uacb0\uacfc\ub97c \ub9cc\ub4e4\uc5c8\uc73c\uba70, \uc815\uce58 \uc790\uc720\ud654, \uacbd\uc81c \uc790\uc720\ud654 \uc774\ud6c4 \uacbd\uc81c\uc801 \ubd88\ud3c9\ub4f1 \ubb38\uc81c\uac00 \ub098\ud0c0\ub0ac\ub2e4\ub294 \uac83\uc5d0 \ub300\ud574\uc11c \uc0ac\ud68c\uac00 \uc81c\ub300\ub85c \ub41c \ub300\ucc98\ub97c \ud558\uae30 \uc5b4\ub824\uc6b4 \uc0ac\ud68c\ubb38\ud654\ub97c \ub9cc\ub4e4\uc5b4\ub0c8\ub2e4. \ub9c8\ucc2c\uac00\uc9c0\ub85c, \uae09\uc9c4\uc801 \uacf5\uc0b0\uc8fc\uc758 \uc0ac\uc0c1\uc758 \ub300\ud45c\uaca9\uc778 \ub9c8\ub974\ud06c\uc2a4-\ub808\ub2cc\uc8fc\uc758\ub294 \uc81c\uad6d\uc8fc\uc758 \ubb38\uc81c\uc640 \ub178\ub3d9 \ubb38\uc81c\ub97c \uc804 \uc138\uacc4\uc801\uc73c\ub85c \uad00\uc2ec\uc744 \uac16\uac8c \ud588\uc9c0\ub9cc, \ub3d9\uc2dc\uc5d0 \ub9c8\ub974\ud06c\uc2a4-\ub808\ub2cc\uc8fc\uc758 \ud2b9\uc720\uc758 \uc6d0\ub9ac\uc8fc\uc758\uc801 \uacf5\uc0b0\uc8fc\uc758\uc640 \uae09\uc9c4\uc8fc\uc758\ub294 \uad6d\uac00 \ub0b4\ubd80\uc5d0\uc120 \uac00\ud639\ud55c \ud1b5\uce58, \ucca0\uad8c \ud1b5\uce58\uac00 \uc774\ub904\uc9c0\uba70, \uc0ac\uc2e4\uc0c1 \uadf9\ub2e8\uc8fc\uc758\ub85c \ube60\uc838\ub4dc\ub294 \uc6d0\uc778\uc744 \uc81c\uacf5\ud588\ub2e4. \ub9c8\ub974\ud06c\uc2a4-\ub808\ub2cc\uc8fc\uc758\ub85c\ubd80\ud130 \ub098\uc628 \ubaa8\ud0dd\ub3d9\uc8fc\uc758\ub294 \uae09\uc9c4\uc8fc\uc758\ub97c \ub118\uc740, \uadf9\ub2e8\uc8fc\uc758\ub85c \ub3cc\ubcc0\ud588\ub2e4.", "answer": "\ub300\uc57d\uc9c4\uc6b4\ub3d9", "sentence": "\uc77c\ub840\ub85c \uc911\uad6d\uc5d0\uc11c \uc77c\uc5b4\ub0ac\ub358 \uae09\uc9c4\uc801\uc778 \uc0ac\ud68c\uc8fc\uc758 \uc815\ucc45\uc778 \ub300\uc57d\uc9c4\uc6b4\ub3d9 \uc740 \ub300\ub7c9 \uc544\uc0ac\uc790\ub97c \uc18d\ucd9c\uc2dc\ucf30\ub2e4.", "paragraph_sentence": "\ud558\uc9c0\ub9cc, \uae09\uc9c4\uc8fc\uc758 \uc815\uce58 \uc6b4\ub3d9\uc774 \uae0d\uc815\uc801\uc778 \ud6a8\uacfc\ub9cc \ubcf4\uc600\ub358 \uac83\uc740 \uc544\ub2c8\uc5c8\ub2e4. \uae09\uc9c4\uc801\uc778 \uc815\uce58 \uc6b4\ub3d9\uc740 \uc790\uce6b\ud558\uba74 \uc0ac\ud68c\uc801 \uc77c\ud0c8 \ud604\uc0c1\uc744 \ub9cc\uc5f0\ud558\uac8c \ud558\ub294 \uc6d0\uc778\uc774 \ub418\uae30\ub3c4 \ud55c\ub2e4. \ub610\ud55c \uadf8 \uacfc\uc815\uc5d0\uc11c \uacaa\ub294 \ud53c\ud574\ub3c4 \ud06c\ub2e4. \uc77c\ub840\ub85c \uc911\uad6d\uc5d0\uc11c \uc77c\uc5b4\ub0ac\ub358 \uae09\uc9c4\uc801\uc778 \uc0ac\ud68c\uc8fc\uc758 \uc815\ucc45\uc778 \ub300\uc57d\uc9c4\uc6b4\ub3d9 \uc740 \ub300\ub7c9 \uc544\uc0ac\uc790\ub97c \uc18d\ucd9c\uc2dc\ucf30\ub2e4. \uc624\ub298\ub0a0\uc5d0 \uae09\uc9c4\uc801\uc778 \uc0ac\uc0c1 \uc6b4\ub3d9\uc774 \uc81c\ub300\ub85c \uc81c\uc5b4\ub418\uc9c0 \ubabb\ud560 \uacbd\uc6b0 \uc5b4\ub5a0\ud55c \uc545\uc601\ud5a5\uc744 \ub07c\uce58\ub294 \uc9c0 \uc798 \ubcf4\uc5ec\uc8fc\ub294 \uc0ac\ub840\uc774\ub2e4. \uae09\uc9c4\uc801 \uc790\uc720\uc8fc\uc758\ud654\ub294 \uc790\uc720\uc8fc\uc758 \uacbd\uc81c\uc758 \uae30\ubc18\uc778 \uc790\ubcf8\uc8fc\uc758\ub97c \ub9f9\uc2e0\ud558\ub294 \uacb0\uacfc\ub97c \ub9cc\ub4e4\uc5c8\uc73c\uba70, \uc815\uce58 \uc790\uc720\ud654, \uacbd\uc81c \uc790\uc720\ud654 \uc774\ud6c4 \uacbd\uc81c\uc801 \ubd88\ud3c9\ub4f1 \ubb38\uc81c\uac00 \ub098\ud0c0\ub0ac\ub2e4\ub294 \uac83\uc5d0 \ub300\ud574\uc11c \uc0ac\ud68c\uac00 \uc81c\ub300\ub85c \ub41c \ub300\ucc98\ub97c \ud558\uae30 \uc5b4\ub824\uc6b4 \uc0ac\ud68c\ubb38\ud654\ub97c \ub9cc\ub4e4\uc5b4\ub0c8\ub2e4. \ub9c8\ucc2c\uac00\uc9c0\ub85c, \uae09\uc9c4\uc801 \uacf5\uc0b0\uc8fc\uc758 \uc0ac\uc0c1\uc758 \ub300\ud45c\uaca9\uc778 \ub9c8\ub974\ud06c\uc2a4-\ub808\ub2cc\uc8fc\uc758\ub294 \uc81c\uad6d\uc8fc\uc758 \ubb38\uc81c\uc640 \ub178\ub3d9 \ubb38\uc81c\ub97c \uc804 \uc138\uacc4\uc801\uc73c\ub85c \uad00\uc2ec\uc744 \uac16\uac8c \ud588\uc9c0\ub9cc, \ub3d9\uc2dc\uc5d0 \ub9c8\ub974\ud06c\uc2a4-\ub808\ub2cc\uc8fc\uc758 \ud2b9\uc720\uc758 \uc6d0\ub9ac\uc8fc\uc758\uc801 \uacf5\uc0b0\uc8fc\uc758\uc640 \uae09\uc9c4\uc8fc\uc758\ub294 \uad6d\uac00 \ub0b4\ubd80\uc5d0\uc120 \uac00\ud639\ud55c \ud1b5\uce58, \ucca0\uad8c \ud1b5\uce58\uac00 \uc774\ub904\uc9c0\uba70, \uc0ac\uc2e4\uc0c1 \uadf9\ub2e8\uc8fc\uc758\ub85c \ube60\uc838\ub4dc\ub294 \uc6d0\uc778\uc744 \uc81c\uacf5\ud588\ub2e4. \ub9c8\ub974\ud06c\uc2a4-\ub808\ub2cc\uc8fc\uc758\ub85c\ubd80\ud130 \ub098\uc628 \ubaa8\ud0dd\ub3d9\uc8fc\uc758\ub294 \uae09\uc9c4\uc8fc\uc758\ub97c \ub118\uc740, \uadf9\ub2e8\uc8fc\uc758\ub85c \ub3cc\ubcc0\ud588\ub2e4.", "paragraph_answer": "\ud558\uc9c0\ub9cc, \uae09\uc9c4\uc8fc\uc758 \uc815\uce58 \uc6b4\ub3d9\uc774 \uae0d\uc815\uc801\uc778 \ud6a8\uacfc\ub9cc \ubcf4\uc600\ub358 \uac83\uc740 \uc544\ub2c8\uc5c8\ub2e4. \uae09\uc9c4\uc801\uc778 \uc815\uce58 \uc6b4\ub3d9\uc740 \uc790\uce6b\ud558\uba74 \uc0ac\ud68c\uc801 \uc77c\ud0c8 \ud604\uc0c1\uc744 \ub9cc\uc5f0\ud558\uac8c \ud558\ub294 \uc6d0\uc778\uc774 \ub418\uae30\ub3c4 \ud55c\ub2e4. \ub610\ud55c \uadf8 \uacfc\uc815\uc5d0\uc11c \uacaa\ub294 \ud53c\ud574\ub3c4 \ud06c\ub2e4. \uc77c\ub840\ub85c \uc911\uad6d\uc5d0\uc11c \uc77c\uc5b4\ub0ac\ub358 \uae09\uc9c4\uc801\uc778 \uc0ac\ud68c\uc8fc\uc758 \uc815\ucc45\uc778 \ub300\uc57d\uc9c4\uc6b4\ub3d9 \uc740 \ub300\ub7c9 \uc544\uc0ac\uc790\ub97c \uc18d\ucd9c\uc2dc\ucf30\ub2e4. \uc624\ub298\ub0a0\uc5d0 \uae09\uc9c4\uc801\uc778 \uc0ac\uc0c1 \uc6b4\ub3d9\uc774 \uc81c\ub300\ub85c \uc81c\uc5b4\ub418\uc9c0 \ubabb\ud560 \uacbd\uc6b0 \uc5b4\ub5a0\ud55c \uc545\uc601\ud5a5\uc744 \ub07c\uce58\ub294 \uc9c0 \uc798 \ubcf4\uc5ec\uc8fc\ub294 \uc0ac\ub840\uc774\ub2e4. \uae09\uc9c4\uc801 \uc790\uc720\uc8fc\uc758\ud654\ub294 \uc790\uc720\uc8fc\uc758 \uacbd\uc81c\uc758 \uae30\ubc18\uc778 \uc790\ubcf8\uc8fc\uc758\ub97c \ub9f9\uc2e0\ud558\ub294 \uacb0\uacfc\ub97c \ub9cc\ub4e4\uc5c8\uc73c\uba70, \uc815\uce58 \uc790\uc720\ud654, \uacbd\uc81c \uc790\uc720\ud654 \uc774\ud6c4 \uacbd\uc81c\uc801 \ubd88\ud3c9\ub4f1 \ubb38\uc81c\uac00 \ub098\ud0c0\ub0ac\ub2e4\ub294 \uac83\uc5d0 \ub300\ud574\uc11c \uc0ac\ud68c\uac00 \uc81c\ub300\ub85c \ub41c \ub300\ucc98\ub97c \ud558\uae30 \uc5b4\ub824\uc6b4 \uc0ac\ud68c\ubb38\ud654\ub97c \ub9cc\ub4e4\uc5b4\ub0c8\ub2e4. \ub9c8\ucc2c\uac00\uc9c0\ub85c, \uae09\uc9c4\uc801 \uacf5\uc0b0\uc8fc\uc758 \uc0ac\uc0c1\uc758 \ub300\ud45c\uaca9\uc778 \ub9c8\ub974\ud06c\uc2a4-\ub808\ub2cc\uc8fc\uc758\ub294 \uc81c\uad6d\uc8fc\uc758 \ubb38\uc81c\uc640 \ub178\ub3d9 \ubb38\uc81c\ub97c \uc804 \uc138\uacc4\uc801\uc73c\ub85c \uad00\uc2ec\uc744 \uac16\uac8c \ud588\uc9c0\ub9cc, \ub3d9\uc2dc\uc5d0 \ub9c8\ub974\ud06c\uc2a4-\ub808\ub2cc\uc8fc\uc758 \ud2b9\uc720\uc758 \uc6d0\ub9ac\uc8fc\uc758\uc801 \uacf5\uc0b0\uc8fc\uc758\uc640 \uae09\uc9c4\uc8fc\uc758\ub294 \uad6d\uac00 \ub0b4\ubd80\uc5d0\uc120 \uac00\ud639\ud55c \ud1b5\uce58, \ucca0\uad8c \ud1b5\uce58\uac00 \uc774\ub904\uc9c0\uba70, \uc0ac\uc2e4\uc0c1 \uadf9\ub2e8\uc8fc\uc758\ub85c \ube60\uc838\ub4dc\ub294 \uc6d0\uc778\uc744 \uc81c\uacf5\ud588\ub2e4. \ub9c8\ub974\ud06c\uc2a4-\ub808\ub2cc\uc8fc\uc758\ub85c\ubd80\ud130 \ub098\uc628 \ubaa8\ud0dd\ub3d9\uc8fc\uc758\ub294 \uae09\uc9c4\uc8fc\uc758\ub97c \ub118\uc740, \uadf9\ub2e8\uc8fc\uc758\ub85c \ub3cc\ubcc0\ud588\ub2e4.", "sentence_answer": "\uc77c\ub840\ub85c \uc911\uad6d\uc5d0\uc11c \uc77c\uc5b4\ub0ac\ub358 \uae09\uc9c4\uc801\uc778 \uc0ac\ud68c\uc8fc\uc758 \uc815\ucc45\uc778 \ub300\uc57d\uc9c4\uc6b4\ub3d9 \uc740 \ub300\ub7c9 \uc544\uc0ac\uc790\ub97c \uc18d\ucd9c\uc2dc\ucf30\ub2e4.", "paragraph_id": "6480773-3-0", "paragraph_question": "question: \uc911\uad6d\uc5d0\uc11c \uc77c\uc5b4\ub0ac\ub358 \uae09\uc9c4\uc801\uc778 \uc0ac\ud68c\uc8fc\uc758 \uc815\ucc45\uc911 \ud558\ub098\ub85c \ub300\ub7c9\uc758 \uc544\uc0ac\uc790\ub97c \ubc1c\uc0dd\uc2dc\ucf30\ub358 \uc6b4\ub3d9\uc740?, context: \ud558\uc9c0\ub9cc, \uae09\uc9c4\uc8fc\uc758 \uc815\uce58 \uc6b4\ub3d9\uc774 \uae0d\uc815\uc801\uc778 \ud6a8\uacfc\ub9cc \ubcf4\uc600\ub358 \uac83\uc740 \uc544\ub2c8\uc5c8\ub2e4. \uae09\uc9c4\uc801\uc778 \uc815\uce58 \uc6b4\ub3d9\uc740 \uc790\uce6b\ud558\uba74 \uc0ac\ud68c\uc801 \uc77c\ud0c8 \ud604\uc0c1\uc744 \ub9cc\uc5f0\ud558\uac8c \ud558\ub294 \uc6d0\uc778\uc774 \ub418\uae30\ub3c4 \ud55c\ub2e4. \ub610\ud55c \uadf8 \uacfc\uc815\uc5d0\uc11c \uacaa\ub294 \ud53c\ud574\ub3c4 \ud06c\ub2e4. \uc77c\ub840\ub85c \uc911\uad6d\uc5d0\uc11c \uc77c\uc5b4\ub0ac\ub358 \uae09\uc9c4\uc801\uc778 \uc0ac\ud68c\uc8fc\uc758 \uc815\ucc45\uc778 \ub300\uc57d\uc9c4\uc6b4\ub3d9\uc740 \ub300\ub7c9 \uc544\uc0ac\uc790\ub97c \uc18d\ucd9c\uc2dc\ucf30\ub2e4. \uc624\ub298\ub0a0\uc5d0 \uae09\uc9c4\uc801\uc778 \uc0ac\uc0c1 \uc6b4\ub3d9\uc774 \uc81c\ub300\ub85c \uc81c\uc5b4\ub418\uc9c0 \ubabb\ud560 \uacbd\uc6b0 \uc5b4\ub5a0\ud55c \uc545\uc601\ud5a5\uc744 \ub07c\uce58\ub294 \uc9c0 \uc798 \ubcf4\uc5ec\uc8fc\ub294 \uc0ac\ub840\uc774\ub2e4. \uae09\uc9c4\uc801 \uc790\uc720\uc8fc\uc758\ud654\ub294 \uc790\uc720\uc8fc\uc758 \uacbd\uc81c\uc758 \uae30\ubc18\uc778 \uc790\ubcf8\uc8fc\uc758\ub97c \ub9f9\uc2e0\ud558\ub294 \uacb0\uacfc\ub97c \ub9cc\ub4e4\uc5c8\uc73c\uba70, \uc815\uce58 \uc790\uc720\ud654, \uacbd\uc81c \uc790\uc720\ud654 \uc774\ud6c4 \uacbd\uc81c\uc801 \ubd88\ud3c9\ub4f1 \ubb38\uc81c\uac00 \ub098\ud0c0\ub0ac\ub2e4\ub294 \uac83\uc5d0 \ub300\ud574\uc11c \uc0ac\ud68c\uac00 \uc81c\ub300\ub85c \ub41c \ub300\ucc98\ub97c \ud558\uae30 \uc5b4\ub824\uc6b4 \uc0ac\ud68c\ubb38\ud654\ub97c \ub9cc\ub4e4\uc5b4\ub0c8\ub2e4. \ub9c8\ucc2c\uac00\uc9c0\ub85c, \uae09\uc9c4\uc801 \uacf5\uc0b0\uc8fc\uc758 \uc0ac\uc0c1\uc758 \ub300\ud45c\uaca9\uc778 \ub9c8\ub974\ud06c\uc2a4-\ub808\ub2cc\uc8fc\uc758\ub294 \uc81c\uad6d\uc8fc\uc758 \ubb38\uc81c\uc640 \ub178\ub3d9 \ubb38\uc81c\ub97c \uc804 \uc138\uacc4\uc801\uc73c\ub85c \uad00\uc2ec\uc744 \uac16\uac8c \ud588\uc9c0\ub9cc, \ub3d9\uc2dc\uc5d0 \ub9c8\ub974\ud06c\uc2a4-\ub808\ub2cc\uc8fc\uc758 \ud2b9\uc720\uc758 \uc6d0\ub9ac\uc8fc\uc758\uc801 \uacf5\uc0b0\uc8fc\uc758\uc640 \uae09\uc9c4\uc8fc\uc758\ub294 \uad6d\uac00 \ub0b4\ubd80\uc5d0\uc120 \uac00\ud639\ud55c \ud1b5\uce58, \ucca0\uad8c \ud1b5\uce58\uac00 \uc774\ub904\uc9c0\uba70, \uc0ac\uc2e4\uc0c1 \uadf9\ub2e8\uc8fc\uc758\ub85c \ube60\uc838\ub4dc\ub294 \uc6d0\uc778\uc744 \uc81c\uacf5\ud588\ub2e4. \ub9c8\ub974\ud06c\uc2a4-\ub808\ub2cc\uc8fc\uc758\ub85c\ubd80\ud130 \ub098\uc628 \ubaa8\ud0dd\ub3d9\uc8fc\uc758\ub294 \uae09\uc9c4\uc8fc\uc758\ub97c \ub118\uc740, \uadf9\ub2e8\uc8fc\uc758\ub85c \ub3cc\ubcc0\ud588\ub2e4."} {"question": "\ubd81\ud55c\uc5d0\uc11c \uc9c4\ud589\ub41c \ud68c\ub2f4\uc5d0\uc11c \uad6c\uc131\ub41c \ud569\uc758 \ub0b4\uc6a9\uc740 \ucd1d \uba87\uac1c\uc778\uac00?", "paragraph": "\ud569\uc758 \ub0b4\uc6a9\uc740 \ucd1d 6\uac1c\ud56d\uc73c\ub85c \uad6c\uc131\ub418\uc5c8\ub2e4. \uac00\uc7a5 \uccab \ubc88\uc9f8\ub294 4\uc6d4 \ub9d0 \ud310\ubb38\uc810\uc5d0\uc11c \ub0a8\ubd81\uc815\uc0c1\ud68c\ub2f4\uc744 \uac1c\ucd5c\ud558\uae30\ub85c \ud588\uc73c\uba70 \uadf8 \uc678\uc5d0 \uad70\uc0ac\uc801 \uae34\uc7a5\uc644\ud654\ub97c \uc704\ud574 \uc815\uc0c1 \uac04 \ud56b\ub77c\uc778\uc744 \uc124\uce58\ud558\uba70 \ubd81\ud55c\uc5d0 \ub300\ud55c \uad70\uc0ac\uc801 \uc704\ud611\uc774 \ud574\uc18c\ub418\uace0 \uccb4\uc81c \uc548\uc804\uc774 \ubcf4\uc7a5\ub41c\ub2e4\uba74 \ud575\uc744 \ubcf4\uc720\ud560 \uc774\uc720\uac00 \uc5c6\ub2e4\ub294 \uc810\uc744 \uba85\ubc31\ud788 \ud558\uace0 \ube44\ud575\ud654 \ubb38\uc81c \ud611\uc758 \ub4f1\uc744 \uc704\ud55c \ubd81\ubbf8\ub300\ud654\uc758 \uc6a9\uc758\uac00 \uc788\uc74c\uc744 \ud655\uc778\ud588\ub2e4. \ud2b9\ud788 \ud575\ubb34\uae30\ub97c \ud3ec\ud568\ud55c \uc7ac\ub798\uc2dd \ubb34\uae30\ub97c \ud55c\uad6d\uc744 \ud5a5\ud574 \uc0ac\uc6a9\ud558\uc9c0 \uc54a\uc744 \uac83\uae4c\uc9c0 \ud655\uc57d\ud588\ub2e4. \uc815\uc0c1\ud68c\ub2f4\uc758 \uacbd\uc6b0 \uae40\uc815\uc740\uc774 \uae40\uc5ec\uc815\uc744 \ud1b5\ud574 \uce5c\uc11c\ub97c \uc804\ub2ec\ud558\uae30 \uc774\uc804\ubd80\ud130 \ub17c\uc758\ub418\uc5c8\ub358 \uac83\uc73c\ub85c \ud30c\uc545\ub418\uc5c8\ub2e4. \ud2b9\uc0ac\ub2e8\uc758 \ubc29\ubd81 \uc774\ud2bf\ub0a0 \uc624\uc804 \uccad\uc640\ub300\ub294 \"\uc2e4\ub9dd\uc2a4\ub7fd\uc9c0 \uc54a\uc740 \uacb0\uacfc\uac00 \uc788\uc5c8\ub2e4\"\uba70 \uc0ac\uc804\uc5d0 \ud569\uc758\uac00 \uc788\uc5c8\uc74c\uc744 \uc2dc\uc0ac\ud558\uae30\ub3c4 \ud588\ub2e4. \ubb38\uc7ac\uc778\uc774 \"\ub9c8\uc74c\uc774 \uae09\ud55c \uac83 \uac19\ub2e4. \uc6b0\ubb3c\uac00\uc5d0\uc11c \uc22d\ub289\uc744 \ucc3e\ub294 \uaca9\"\uc774\ub77c\uba70 \uc815\uc0c1\ud68c\ub2f4\uc744 \uc11c\ub450\ub974\uc9c0 \uc54a\uaca0\ub2e4\uace0 \ud588\uc9c0\ub9cc \ubb3c\ubc11 \ub300\ud654\ub97c \uc0c1\ub2f9\ud788 \uc9c4\ucc99\uc2dc\ucf30\uace0 \ubd81\ud55c\uc774 \ube44\ud575\ud654\uc758 \ucd08\uae30 \uc870\uce58\ub97c \uc218\uc6a9\ud558\uba74\uc11c \uc2dc\uae30\ub3c4 \uc608\uc0c1\ubcf4\ub2e4 \ube60\ub974\uac8c \uc9c4\ud589\ub418\uc5c8\ub2e4. \ud55c \ubc1c \ub354 \ub098\uac00\uc11c \ud310\ubb38\uc810 \ubd81\uce21\uacfc \ub0a8\uce21\uc744 \uc624\uac00\ub294 \uc154\ud2c0 \uc815\uc0c1\ud68c\ub2f4\uc774 \uc815\ub840\uc801\uc73c\ub85c \uc774\ub904\uc9c8 \uac00\ub2a5\uc131\uae4c\uc9c0 \ud0c0\uc9c4\ud588\ub2e4.", "answer": "6\uac1c\ud56d", "sentence": "\ud569\uc758 \ub0b4\uc6a9\uc740 \ucd1d 6\uac1c\ud56d \uc73c\ub85c \uad6c\uc131\ub418\uc5c8\ub2e4.", "paragraph_sentence": " \ud569\uc758 \ub0b4\uc6a9\uc740 \ucd1d 6\uac1c\ud56d \uc73c\ub85c \uad6c\uc131\ub418\uc5c8\ub2e4. \uac00\uc7a5 \uccab \ubc88\uc9f8\ub294 4\uc6d4 \ub9d0 \ud310\ubb38\uc810\uc5d0\uc11c \ub0a8\ubd81\uc815\uc0c1\ud68c\ub2f4\uc744 \uac1c\ucd5c\ud558\uae30\ub85c \ud588\uc73c\uba70 \uadf8 \uc678\uc5d0 \uad70\uc0ac\uc801 \uae34\uc7a5\uc644\ud654\ub97c \uc704\ud574 \uc815\uc0c1 \uac04 \ud56b\ub77c\uc778\uc744 \uc124\uce58\ud558\uba70 \ubd81\ud55c\uc5d0 \ub300\ud55c \uad70\uc0ac\uc801 \uc704\ud611\uc774 \ud574\uc18c\ub418\uace0 \uccb4\uc81c \uc548\uc804\uc774 \ubcf4\uc7a5\ub41c\ub2e4\uba74 \ud575\uc744 \ubcf4\uc720\ud560 \uc774\uc720\uac00 \uc5c6\ub2e4\ub294 \uc810\uc744 \uba85\ubc31\ud788 \ud558\uace0 \ube44\ud575\ud654 \ubb38\uc81c \ud611\uc758 \ub4f1\uc744 \uc704\ud55c \ubd81\ubbf8\ub300\ud654\uc758 \uc6a9\uc758\uac00 \uc788\uc74c\uc744 \ud655\uc778\ud588\ub2e4. \ud2b9\ud788 \ud575\ubb34\uae30\ub97c \ud3ec\ud568\ud55c \uc7ac\ub798\uc2dd \ubb34\uae30\ub97c \ud55c\uad6d\uc744 \ud5a5\ud574 \uc0ac\uc6a9\ud558\uc9c0 \uc54a\uc744 \uac83\uae4c\uc9c0 \ud655\uc57d\ud588\ub2e4. \uc815\uc0c1\ud68c\ub2f4\uc758 \uacbd\uc6b0 \uae40\uc815\uc740\uc774 \uae40\uc5ec\uc815\uc744 \ud1b5\ud574 \uce5c\uc11c\ub97c \uc804\ub2ec\ud558\uae30 \uc774\uc804\ubd80\ud130 \ub17c\uc758\ub418\uc5c8\ub358 \uac83\uc73c\ub85c \ud30c\uc545\ub418\uc5c8\ub2e4. \ud2b9\uc0ac\ub2e8\uc758 \ubc29\ubd81 \uc774\ud2bf\ub0a0 \uc624\uc804 \uccad\uc640\ub300\ub294 \"\uc2e4\ub9dd\uc2a4\ub7fd\uc9c0 \uc54a\uc740 \uacb0\uacfc\uac00 \uc788\uc5c8\ub2e4\"\uba70 \uc0ac\uc804\uc5d0 \ud569\uc758\uac00 \uc788\uc5c8\uc74c\uc744 \uc2dc\uc0ac\ud558\uae30\ub3c4 \ud588\ub2e4. \ubb38\uc7ac\uc778\uc774 \"\ub9c8\uc74c\uc774 \uae09\ud55c \uac83 \uac19\ub2e4. \uc6b0\ubb3c\uac00\uc5d0\uc11c \uc22d\ub289\uc744 \ucc3e\ub294 \uaca9\"\uc774\ub77c\uba70 \uc815\uc0c1\ud68c\ub2f4\uc744 \uc11c\ub450\ub974\uc9c0 \uc54a\uaca0\ub2e4\uace0 \ud588\uc9c0\ub9cc \ubb3c\ubc11 \ub300\ud654\ub97c \uc0c1\ub2f9\ud788 \uc9c4\ucc99\uc2dc\ucf30\uace0 \ubd81\ud55c\uc774 \ube44\ud575\ud654\uc758 \ucd08\uae30 \uc870\uce58\ub97c \uc218\uc6a9\ud558\uba74\uc11c \uc2dc\uae30\ub3c4 \uc608\uc0c1\ubcf4\ub2e4 \ube60\ub974\uac8c \uc9c4\ud589\ub418\uc5c8\ub2e4. \ud55c \ubc1c \ub354 \ub098\uac00\uc11c \ud310\ubb38\uc810 \ubd81\uce21\uacfc \ub0a8\uce21\uc744 \uc624\uac00\ub294 \uc154\ud2c0 \uc815\uc0c1\ud68c\ub2f4\uc774 \uc815\ub840\uc801\uc73c\ub85c \uc774\ub904\uc9c8 \uac00\ub2a5\uc131\uae4c\uc9c0 \ud0c0\uc9c4\ud588\ub2e4.", "paragraph_answer": "\ud569\uc758 \ub0b4\uc6a9\uc740 \ucd1d 6\uac1c\ud56d \uc73c\ub85c \uad6c\uc131\ub418\uc5c8\ub2e4. \uac00\uc7a5 \uccab \ubc88\uc9f8\ub294 4\uc6d4 \ub9d0 \ud310\ubb38\uc810\uc5d0\uc11c \ub0a8\ubd81\uc815\uc0c1\ud68c\ub2f4\uc744 \uac1c\ucd5c\ud558\uae30\ub85c \ud588\uc73c\uba70 \uadf8 \uc678\uc5d0 \uad70\uc0ac\uc801 \uae34\uc7a5\uc644\ud654\ub97c \uc704\ud574 \uc815\uc0c1 \uac04 \ud56b\ub77c\uc778\uc744 \uc124\uce58\ud558\uba70 \ubd81\ud55c\uc5d0 \ub300\ud55c \uad70\uc0ac\uc801 \uc704\ud611\uc774 \ud574\uc18c\ub418\uace0 \uccb4\uc81c \uc548\uc804\uc774 \ubcf4\uc7a5\ub41c\ub2e4\uba74 \ud575\uc744 \ubcf4\uc720\ud560 \uc774\uc720\uac00 \uc5c6\ub2e4\ub294 \uc810\uc744 \uba85\ubc31\ud788 \ud558\uace0 \ube44\ud575\ud654 \ubb38\uc81c \ud611\uc758 \ub4f1\uc744 \uc704\ud55c \ubd81\ubbf8\ub300\ud654\uc758 \uc6a9\uc758\uac00 \uc788\uc74c\uc744 \ud655\uc778\ud588\ub2e4. \ud2b9\ud788 \ud575\ubb34\uae30\ub97c \ud3ec\ud568\ud55c \uc7ac\ub798\uc2dd \ubb34\uae30\ub97c \ud55c\uad6d\uc744 \ud5a5\ud574 \uc0ac\uc6a9\ud558\uc9c0 \uc54a\uc744 \uac83\uae4c\uc9c0 \ud655\uc57d\ud588\ub2e4. \uc815\uc0c1\ud68c\ub2f4\uc758 \uacbd\uc6b0 \uae40\uc815\uc740\uc774 \uae40\uc5ec\uc815\uc744 \ud1b5\ud574 \uce5c\uc11c\ub97c \uc804\ub2ec\ud558\uae30 \uc774\uc804\ubd80\ud130 \ub17c\uc758\ub418\uc5c8\ub358 \uac83\uc73c\ub85c \ud30c\uc545\ub418\uc5c8\ub2e4. \ud2b9\uc0ac\ub2e8\uc758 \ubc29\ubd81 \uc774\ud2bf\ub0a0 \uc624\uc804 \uccad\uc640\ub300\ub294 \"\uc2e4\ub9dd\uc2a4\ub7fd\uc9c0 \uc54a\uc740 \uacb0\uacfc\uac00 \uc788\uc5c8\ub2e4\"\uba70 \uc0ac\uc804\uc5d0 \ud569\uc758\uac00 \uc788\uc5c8\uc74c\uc744 \uc2dc\uc0ac\ud558\uae30\ub3c4 \ud588\ub2e4. \ubb38\uc7ac\uc778\uc774 \"\ub9c8\uc74c\uc774 \uae09\ud55c \uac83 \uac19\ub2e4. \uc6b0\ubb3c\uac00\uc5d0\uc11c \uc22d\ub289\uc744 \ucc3e\ub294 \uaca9\"\uc774\ub77c\uba70 \uc815\uc0c1\ud68c\ub2f4\uc744 \uc11c\ub450\ub974\uc9c0 \uc54a\uaca0\ub2e4\uace0 \ud588\uc9c0\ub9cc \ubb3c\ubc11 \ub300\ud654\ub97c \uc0c1\ub2f9\ud788 \uc9c4\ucc99\uc2dc\ucf30\uace0 \ubd81\ud55c\uc774 \ube44\ud575\ud654\uc758 \ucd08\uae30 \uc870\uce58\ub97c \uc218\uc6a9\ud558\uba74\uc11c \uc2dc\uae30\ub3c4 \uc608\uc0c1\ubcf4\ub2e4 \ube60\ub974\uac8c \uc9c4\ud589\ub418\uc5c8\ub2e4. \ud55c \ubc1c \ub354 \ub098\uac00\uc11c \ud310\ubb38\uc810 \ubd81\uce21\uacfc \ub0a8\uce21\uc744 \uc624\uac00\ub294 \uc154\ud2c0 \uc815\uc0c1\ud68c\ub2f4\uc774 \uc815\ub840\uc801\uc73c\ub85c \uc774\ub904\uc9c8 \uac00\ub2a5\uc131\uae4c\uc9c0 \ud0c0\uc9c4\ud588\ub2e4.", "sentence_answer": "\ud569\uc758 \ub0b4\uc6a9\uc740 \ucd1d 6\uac1c\ud56d \uc73c\ub85c \uad6c\uc131\ub418\uc5c8\ub2e4.", "paragraph_id": "6580792-41-0", "paragraph_question": "question: \ubd81\ud55c\uc5d0\uc11c \uc9c4\ud589\ub41c \ud68c\ub2f4\uc5d0\uc11c \uad6c\uc131\ub41c \ud569\uc758 \ub0b4\uc6a9\uc740 \ucd1d \uba87\uac1c\uc778\uac00?, context: \ud569\uc758 \ub0b4\uc6a9\uc740 \ucd1d 6\uac1c\ud56d\uc73c\ub85c \uad6c\uc131\ub418\uc5c8\ub2e4. \uac00\uc7a5 \uccab \ubc88\uc9f8\ub294 4\uc6d4 \ub9d0 \ud310\ubb38\uc810\uc5d0\uc11c \ub0a8\ubd81\uc815\uc0c1\ud68c\ub2f4\uc744 \uac1c\ucd5c\ud558\uae30\ub85c \ud588\uc73c\uba70 \uadf8 \uc678\uc5d0 \uad70\uc0ac\uc801 \uae34\uc7a5\uc644\ud654\ub97c \uc704\ud574 \uc815\uc0c1 \uac04 \ud56b\ub77c\uc778\uc744 \uc124\uce58\ud558\uba70 \ubd81\ud55c\uc5d0 \ub300\ud55c \uad70\uc0ac\uc801 \uc704\ud611\uc774 \ud574\uc18c\ub418\uace0 \uccb4\uc81c \uc548\uc804\uc774 \ubcf4\uc7a5\ub41c\ub2e4\uba74 \ud575\uc744 \ubcf4\uc720\ud560 \uc774\uc720\uac00 \uc5c6\ub2e4\ub294 \uc810\uc744 \uba85\ubc31\ud788 \ud558\uace0 \ube44\ud575\ud654 \ubb38\uc81c \ud611\uc758 \ub4f1\uc744 \uc704\ud55c \ubd81\ubbf8\ub300\ud654\uc758 \uc6a9\uc758\uac00 \uc788\uc74c\uc744 \ud655\uc778\ud588\ub2e4. \ud2b9\ud788 \ud575\ubb34\uae30\ub97c \ud3ec\ud568\ud55c \uc7ac\ub798\uc2dd \ubb34\uae30\ub97c \ud55c\uad6d\uc744 \ud5a5\ud574 \uc0ac\uc6a9\ud558\uc9c0 \uc54a\uc744 \uac83\uae4c\uc9c0 \ud655\uc57d\ud588\ub2e4. \uc815\uc0c1\ud68c\ub2f4\uc758 \uacbd\uc6b0 \uae40\uc815\uc740\uc774 \uae40\uc5ec\uc815\uc744 \ud1b5\ud574 \uce5c\uc11c\ub97c \uc804\ub2ec\ud558\uae30 \uc774\uc804\ubd80\ud130 \ub17c\uc758\ub418\uc5c8\ub358 \uac83\uc73c\ub85c \ud30c\uc545\ub418\uc5c8\ub2e4. \ud2b9\uc0ac\ub2e8\uc758 \ubc29\ubd81 \uc774\ud2bf\ub0a0 \uc624\uc804 \uccad\uc640\ub300\ub294 \"\uc2e4\ub9dd\uc2a4\ub7fd\uc9c0 \uc54a\uc740 \uacb0\uacfc\uac00 \uc788\uc5c8\ub2e4\"\uba70 \uc0ac\uc804\uc5d0 \ud569\uc758\uac00 \uc788\uc5c8\uc74c\uc744 \uc2dc\uc0ac\ud558\uae30\ub3c4 \ud588\ub2e4. \ubb38\uc7ac\uc778\uc774 \"\ub9c8\uc74c\uc774 \uae09\ud55c \uac83 \uac19\ub2e4. \uc6b0\ubb3c\uac00\uc5d0\uc11c \uc22d\ub289\uc744 \ucc3e\ub294 \uaca9\"\uc774\ub77c\uba70 \uc815\uc0c1\ud68c\ub2f4\uc744 \uc11c\ub450\ub974\uc9c0 \uc54a\uaca0\ub2e4\uace0 \ud588\uc9c0\ub9cc \ubb3c\ubc11 \ub300\ud654\ub97c \uc0c1\ub2f9\ud788 \uc9c4\ucc99\uc2dc\ucf30\uace0 \ubd81\ud55c\uc774 \ube44\ud575\ud654\uc758 \ucd08\uae30 \uc870\uce58\ub97c \uc218\uc6a9\ud558\uba74\uc11c \uc2dc\uae30\ub3c4 \uc608\uc0c1\ubcf4\ub2e4 \ube60\ub974\uac8c \uc9c4\ud589\ub418\uc5c8\ub2e4. \ud55c \ubc1c \ub354 \ub098\uac00\uc11c \ud310\ubb38\uc810 \ubd81\uce21\uacfc \ub0a8\uce21\uc744 \uc624\uac00\ub294 \uc154\ud2c0 \uc815\uc0c1\ud68c\ub2f4\uc774 \uc815\ub840\uc801\uc73c\ub85c \uc774\ub904\uc9c8 \uac00\ub2a5\uc131\uae4c\uc9c0 \ud0c0\uc9c4\ud588\ub2e4."} {"question": "\ub780\uc2ac\ub86f\uacfc \uadc0\ub124\ube44\uc5b4 \uc5ec\uc655\uc758 \ubd80\uc801\uc808\ud55c \uad00\uacc4\ub85c \uc778\ud574 \ubd95\uad34\ub41c \uac83\uc740 \ubb34\uc5c7\uc77c\uae4c?", "paragraph": "\uc774 \ud638\uc218\ub294 \ud638\uc218\uc758 \ubd80\uc778\uc758 \uac70\ucc98\uc600\uace0, \ub9c8\uc220\uc0ac, \ub9c8\ubc95\uc0ac\uc758 \ub098\ub77c, \ub9c8\ubc95\uc774 \uac78\ub9b0 \uc544\ubc1c\ub871(Avalon) \uc12c\uc73c\ub85c \ud5a5\ud558\ub294 \uac00\uad50\uc600\ub2e4. \ube44\ube44\uc548 \uc694\uc815\uc740 \ub780\uc2ac\ub86f\uc744 \ubc14\ub2f7 \uc18d \uadf8\uc758 \uad81\uc804\uc73c\ub85c \ub370\ub824\uc628 \uc7a5\ubcf8\uc774\uae30 \ub54c\ubb38\uc5d0, \uadf8\ub97c \uc544\ub4e4\ucc98\ub7fc \uad50\uc721\uc2dc\ucf30\ub2e4. \uc774 \ub54c\ubd80\ud130 \ub780\uc2ac\ub86f \ub4c0 \ub77d((Lancelot du lac, \uc8fc : \ud638\uc218\uc758 \ub780\uc2ac\ub86f)\uc774\ub77c\ub294 \uadf8\uc758 \ubcc4\uba85\uc774 \ubd99\uac8c \ub418\uc5c8\ub2e4. \uc694\uc815\uc73c\ub85c\ubd80\ud130 '\uc655\uc758 \uc544\ub4e4' \ub610\ub294 '\uc544\ub984\ub2e4\uc6b4 \ubc1c\uacac'\uc73c\ub85c \ubd88\ub9b0 \uadf8\ub294 \uadf8\uc758 \ubaa8\ub4e0 \ubfcc\ub9ac\ub97c \uc78a\uac8c \ub418\uc5c8\ub2e4. \uadf8\uc758 \uc80a\uc740 \uc2dc\uc808 \ub3d9\uc548 \uadf8\ub140\ub294 \uadf8\ub97c \uc644\ubcbd\ud55c \uae30\uc0ac\ub85c \uad50\uc721\uc2dc\ucf30\uace0, \uc0ac\ub0e5, \uc74c\uc545, \uacb0\ud22c, \uad81\uc815 \uc608\uc808, \uadc0\uc871\uc758 \uace0\uacb0\ud55c \uc815\uc2e0 \ub4f1\uc744 \uac00\ub974\ucce4\ub2e4. \uadf8\ub294 \uc544\ub354\uc655, \uadc0\ub124\ube44\uc5b4(Gueni\u00e8vre)\uc5ec\uc655\uc744 \ud5a5\ud55c \ucda9\uc131\uc2ec\uc73c\ub85c \ub9e4\uc6b0 \ub9dd\uba85\uc774 \ub192\uc558\uc73c\ub098, \ubd88\ud0c0\uc624\ub974\ub294 \uc0ac\ub791\uc73c\ub85c \uc778\ud558\uc5ec \uc5ec\uc655\uc744 \ubc94\ud558\ub294 \uc2e4\uc218\ub97c \ud558\uc600\uace0, \uc774\ub294 \ucc28\ud6c4 \uc544\ub354\uc655\uc73c\ub85c \ubd80\ud130 \uc6a9\uc11c \ubc1b\uc558\ub2e4. \uadf8\ub7ec\ub098 \uadf8\ub4e4\uc758 \ubd80\uc801\uc808\ud55c \uad00\uacc4\uac00 \uc544\ub354\uc758 \uc655\uad6d\uacfc \uce74\uba5c\ub86f\uc758 \ubd95\uad34\ub97c \ucd08\ub798\ud55c \uc6d0\uc778 \uc911 \ud558\ub098\uac00 \ub418\uc5c8\ub2e4.", "answer": "\uc655\uad6d\uacfc \uce74\uba5c\ub86f", "sentence": "\uadf8\ub7ec\ub098 \uadf8\ub4e4\uc758 \ubd80\uc801\uc808\ud55c \uad00\uacc4\uac00 \uc544\ub354\uc758 \uc655\uad6d\uacfc \uce74\uba5c\ub86f \uc758 \ubd95\uad34\ub97c \ucd08\ub798\ud55c \uc6d0\uc778 \uc911 \ud558\ub098\uac00 \ub418\uc5c8\ub2e4.", "paragraph_sentence": "\uc774 \ud638\uc218\ub294 \ud638\uc218\uc758 \ubd80\uc778\uc758 \uac70\ucc98\uc600\uace0, \ub9c8\uc220\uc0ac, \ub9c8\ubc95\uc0ac\uc758 \ub098\ub77c, \ub9c8\ubc95\uc774 \uac78\ub9b0 \uc544\ubc1c\ub871(Avalon) \uc12c\uc73c\ub85c \ud5a5\ud558\ub294 \uac00\uad50\uc600\ub2e4. \ube44\ube44\uc548 \uc694\uc815\uc740 \ub780\uc2ac\ub86f\uc744 \ubc14\ub2f7 \uc18d \uadf8\uc758 \uad81\uc804\uc73c\ub85c \ub370\ub824\uc628 \uc7a5\ubcf8\uc774\uae30 \ub54c\ubb38\uc5d0, \uadf8\ub97c \uc544\ub4e4\ucc98\ub7fc \uad50\uc721\uc2dc\ucf30\ub2e4. \uc774 \ub54c\ubd80\ud130 \ub780\uc2ac\ub86f \ub4c0 \ub77d((Lancelot du lac, \uc8fc : \ud638\uc218\uc758 \ub780\uc2ac\ub86f)\uc774\ub77c\ub294 \uadf8\uc758 \ubcc4\uba85\uc774 \ubd99\uac8c \ub418\uc5c8\ub2e4. \uc694\uc815\uc73c\ub85c\ubd80\ud130 '\uc655\uc758 \uc544\ub4e4' \ub610\ub294 '\uc544\ub984\ub2e4\uc6b4 \ubc1c\uacac'\uc73c\ub85c \ubd88\ub9b0 \uadf8\ub294 \uadf8\uc758 \ubaa8\ub4e0 \ubfcc\ub9ac\ub97c \uc78a\uac8c \ub418\uc5c8\ub2e4. \uadf8\uc758 \uc80a\uc740 \uc2dc\uc808 \ub3d9\uc548 \uadf8\ub140\ub294 \uadf8\ub97c \uc644\ubcbd\ud55c \uae30\uc0ac\ub85c \uad50\uc721\uc2dc\ucf30\uace0, \uc0ac\ub0e5, \uc74c\uc545, \uacb0\ud22c, \uad81\uc815 \uc608\uc808, \uadc0\uc871\uc758 \uace0\uacb0\ud55c \uc815\uc2e0 \ub4f1\uc744 \uac00\ub974\ucce4\ub2e4. \uadf8\ub294 \uc544\ub354\uc655, \uadc0\ub124\ube44\uc5b4(Gueni\u00e8vre)\uc5ec\uc655\uc744 \ud5a5\ud55c \ucda9\uc131\uc2ec\uc73c\ub85c \ub9e4\uc6b0 \ub9dd\uba85\uc774 \ub192\uc558\uc73c\ub098, \ubd88\ud0c0\uc624\ub974\ub294 \uc0ac\ub791\uc73c\ub85c \uc778\ud558\uc5ec \uc5ec\uc655\uc744 \ubc94\ud558\ub294 \uc2e4\uc218\ub97c \ud558\uc600\uace0, \uc774\ub294 \ucc28\ud6c4 \uc544\ub354\uc655\uc73c\ub85c \ubd80\ud130 \uc6a9\uc11c \ubc1b\uc558\ub2e4. \uadf8\ub7ec\ub098 \uadf8\ub4e4\uc758 \ubd80\uc801\uc808\ud55c \uad00\uacc4\uac00 \uc544\ub354\uc758 \uc655\uad6d\uacfc \uce74\uba5c\ub86f \uc758 \ubd95\uad34\ub97c \ucd08\ub798\ud55c \uc6d0\uc778 \uc911 \ud558\ub098\uac00 \ub418\uc5c8\ub2e4. ", "paragraph_answer": "\uc774 \ud638\uc218\ub294 \ud638\uc218\uc758 \ubd80\uc778\uc758 \uac70\ucc98\uc600\uace0, \ub9c8\uc220\uc0ac, \ub9c8\ubc95\uc0ac\uc758 \ub098\ub77c, \ub9c8\ubc95\uc774 \uac78\ub9b0 \uc544\ubc1c\ub871(Avalon) \uc12c\uc73c\ub85c \ud5a5\ud558\ub294 \uac00\uad50\uc600\ub2e4. \ube44\ube44\uc548 \uc694\uc815\uc740 \ub780\uc2ac\ub86f\uc744 \ubc14\ub2f7 \uc18d \uadf8\uc758 \uad81\uc804\uc73c\ub85c \ub370\ub824\uc628 \uc7a5\ubcf8\uc774\uae30 \ub54c\ubb38\uc5d0, \uadf8\ub97c \uc544\ub4e4\ucc98\ub7fc \uad50\uc721\uc2dc\ucf30\ub2e4. \uc774 \ub54c\ubd80\ud130 \ub780\uc2ac\ub86f \ub4c0 \ub77d((Lancelot du lac, \uc8fc : \ud638\uc218\uc758 \ub780\uc2ac\ub86f)\uc774\ub77c\ub294 \uadf8\uc758 \ubcc4\uba85\uc774 \ubd99\uac8c \ub418\uc5c8\ub2e4. \uc694\uc815\uc73c\ub85c\ubd80\ud130 '\uc655\uc758 \uc544\ub4e4' \ub610\ub294 '\uc544\ub984\ub2e4\uc6b4 \ubc1c\uacac'\uc73c\ub85c \ubd88\ub9b0 \uadf8\ub294 \uadf8\uc758 \ubaa8\ub4e0 \ubfcc\ub9ac\ub97c \uc78a\uac8c \ub418\uc5c8\ub2e4. \uadf8\uc758 \uc80a\uc740 \uc2dc\uc808 \ub3d9\uc548 \uadf8\ub140\ub294 \uadf8\ub97c \uc644\ubcbd\ud55c \uae30\uc0ac\ub85c \uad50\uc721\uc2dc\ucf30\uace0, \uc0ac\ub0e5, \uc74c\uc545, \uacb0\ud22c, \uad81\uc815 \uc608\uc808, \uadc0\uc871\uc758 \uace0\uacb0\ud55c \uc815\uc2e0 \ub4f1\uc744 \uac00\ub974\ucce4\ub2e4. \uadf8\ub294 \uc544\ub354\uc655, \uadc0\ub124\ube44\uc5b4(Gueni\u00e8vre)\uc5ec\uc655\uc744 \ud5a5\ud55c \ucda9\uc131\uc2ec\uc73c\ub85c \ub9e4\uc6b0 \ub9dd\uba85\uc774 \ub192\uc558\uc73c\ub098, \ubd88\ud0c0\uc624\ub974\ub294 \uc0ac\ub791\uc73c\ub85c \uc778\ud558\uc5ec \uc5ec\uc655\uc744 \ubc94\ud558\ub294 \uc2e4\uc218\ub97c \ud558\uc600\uace0, \uc774\ub294 \ucc28\ud6c4 \uc544\ub354\uc655\uc73c\ub85c \ubd80\ud130 \uc6a9\uc11c \ubc1b\uc558\ub2e4. \uadf8\ub7ec\ub098 \uadf8\ub4e4\uc758 \ubd80\uc801\uc808\ud55c \uad00\uacc4\uac00 \uc544\ub354\uc758 \uc655\uad6d\uacfc \uce74\uba5c\ub86f \uc758 \ubd95\uad34\ub97c \ucd08\ub798\ud55c \uc6d0\uc778 \uc911 \ud558\ub098\uac00 \ub418\uc5c8\ub2e4.", "sentence_answer": "\uadf8\ub7ec\ub098 \uadf8\ub4e4\uc758 \ubd80\uc801\uc808\ud55c \uad00\uacc4\uac00 \uc544\ub354\uc758 \uc655\uad6d\uacfc \uce74\uba5c\ub86f \uc758 \ubd95\uad34\ub97c \ucd08\ub798\ud55c \uc6d0\uc778 \uc911 \ud558\ub098\uac00 \ub418\uc5c8\ub2e4.", "paragraph_id": "6534086-3-1", "paragraph_question": "question: \ub780\uc2ac\ub86f\uacfc \uadc0\ub124\ube44\uc5b4 \uc5ec\uc655\uc758 \ubd80\uc801\uc808\ud55c \uad00\uacc4\ub85c \uc778\ud574 \ubd95\uad34\ub41c \uac83\uc740 \ubb34\uc5c7\uc77c\uae4c?, context: \uc774 \ud638\uc218\ub294 \ud638\uc218\uc758 \ubd80\uc778\uc758 \uac70\ucc98\uc600\uace0, \ub9c8\uc220\uc0ac, \ub9c8\ubc95\uc0ac\uc758 \ub098\ub77c, \ub9c8\ubc95\uc774 \uac78\ub9b0 \uc544\ubc1c\ub871(Avalon) \uc12c\uc73c\ub85c \ud5a5\ud558\ub294 \uac00\uad50\uc600\ub2e4. \ube44\ube44\uc548 \uc694\uc815\uc740 \ub780\uc2ac\ub86f\uc744 \ubc14\ub2f7 \uc18d \uadf8\uc758 \uad81\uc804\uc73c\ub85c \ub370\ub824\uc628 \uc7a5\ubcf8\uc774\uae30 \ub54c\ubb38\uc5d0, \uadf8\ub97c \uc544\ub4e4\ucc98\ub7fc \uad50\uc721\uc2dc\ucf30\ub2e4. \uc774 \ub54c\ubd80\ud130 \ub780\uc2ac\ub86f \ub4c0 \ub77d((Lancelot du lac, \uc8fc : \ud638\uc218\uc758 \ub780\uc2ac\ub86f)\uc774\ub77c\ub294 \uadf8\uc758 \ubcc4\uba85\uc774 \ubd99\uac8c \ub418\uc5c8\ub2e4. \uc694\uc815\uc73c\ub85c\ubd80\ud130 '\uc655\uc758 \uc544\ub4e4' \ub610\ub294 '\uc544\ub984\ub2e4\uc6b4 \ubc1c\uacac'\uc73c\ub85c \ubd88\ub9b0 \uadf8\ub294 \uadf8\uc758 \ubaa8\ub4e0 \ubfcc\ub9ac\ub97c \uc78a\uac8c \ub418\uc5c8\ub2e4. \uadf8\uc758 \uc80a\uc740 \uc2dc\uc808 \ub3d9\uc548 \uadf8\ub140\ub294 \uadf8\ub97c \uc644\ubcbd\ud55c \uae30\uc0ac\ub85c \uad50\uc721\uc2dc\ucf30\uace0, \uc0ac\ub0e5, \uc74c\uc545, \uacb0\ud22c, \uad81\uc815 \uc608\uc808, \uadc0\uc871\uc758 \uace0\uacb0\ud55c \uc815\uc2e0 \ub4f1\uc744 \uac00\ub974\ucce4\ub2e4. \uadf8\ub294 \uc544\ub354\uc655, \uadc0\ub124\ube44\uc5b4(Gueni\u00e8vre)\uc5ec\uc655\uc744 \ud5a5\ud55c \ucda9\uc131\uc2ec\uc73c\ub85c \ub9e4\uc6b0 \ub9dd\uba85\uc774 \ub192\uc558\uc73c\ub098, \ubd88\ud0c0\uc624\ub974\ub294 \uc0ac\ub791\uc73c\ub85c \uc778\ud558\uc5ec \uc5ec\uc655\uc744 \ubc94\ud558\ub294 \uc2e4\uc218\ub97c \ud558\uc600\uace0, \uc774\ub294 \ucc28\ud6c4 \uc544\ub354\uc655\uc73c\ub85c \ubd80\ud130 \uc6a9\uc11c \ubc1b\uc558\ub2e4. \uadf8\ub7ec\ub098 \uadf8\ub4e4\uc758 \ubd80\uc801\uc808\ud55c \uad00\uacc4\uac00 \uc544\ub354\uc758 \uc655\uad6d\uacfc \uce74\uba5c\ub86f\uc758 \ubd95\uad34\ub97c \ucd08\ub798\ud55c \uc6d0\uc778 \uc911 \ud558\ub098\uac00 \ub418\uc5c8\ub2e4."} {"question": "\uac10\uc790\ubc2d\uc5d0\uc11c \ud55c\ub2e4\ub808\uc5d0 \ub098\uc624\ub294 \uc218\uc775\uc740 \uc5bc\ub9c8\uc778\uac00?", "paragraph": "\ubc00\uc591\uc2dc \ud558\ub0a8\uc74d\uc5d0 8,000\uc5ec\uba85\uc774 \ub18d\uc0ac\ub97c \uc9c0\uc73c\uba70 \uc9c0\ub0b4\uace0 \uc788\ub2e4. \ub545\uc774 \uae30\ub984\uc838 \uac10\uc790\uc640 \ub538\uae30\uac00 \ub9ce\uc774 \ub09c\ub2e4. \ud558\uc9c0\ub9cc \uc2e0\uacf5\ud56d\uc774 \ub4e4\uc5b4\uc11c\uba74 \ub354\uc774\uc0c1 \ub18d\uc0ac\ub97c \uc9c0\uc744 \uc218 \uc5c6\uac8c \ub41c\ub2e4. \uc774 \ub54c\ubb38\uc5d0 \uc77c\ubd80 \uc8fc\ubbfc\ub4e4\uc740 \uc2e0\uacf5\ud56d \uc720\uce58\ub97c \ubc18\ub300\ud558\uace0 \uc788\uc5c8\ub2e4. \ud55c \uc8fc\ubbfc\uc740 \"\ub18d\uc0ac\uc9d3\uace0 \uc788\ub294\ub370 \ub098\uac00\ub77c\uace0 \ud558\uba74 \ubcf4\uc0c1\ub3c4 \uc5c6\uace0 \ubc14\ub85c \uac70\uc9c0\ub2e4\"\ub77c\uba70 \"\uc6b0\ub9ac\ub294 \ud3c9\uc0c1 \ube44\ud589\uae30 \ud0c8\uc77c \ub450\ubc88\ub3c4 \uc5c6\ub294\ub370\"\ub77c\uba70 \uaca9\ubd84\ud588\ub2e4. \uac10\uc790\ubc2d\uc744 \uc6b4\uc601\ud558\ub294 \ub2e4\ub978 \uc8fc\ubbfc\ub3c4 \"\uac10\uc790\ubc2d\uc5d0\uc11c \ud55c\ub2e4\ub808 200~205\ub9cc\uc6d0\uc529 \uc218\uc775\uc774 \ub098\uc624\ub294\ub370 \uc774\uac83\uc744 \ucca0\uac70\ud558\uba74 \uc815\ubd80\uc5d0\uc11c 200\ub9cc\uc6d0 \uc774\uc0c1 \uc8fc\uaca0\ub098\"\ub77c\uba70 \"\ub2f9\uc7a5 \ub098\uac00\uba74 1~2\ub144\uc740 \uba39\uace0\uc0b4\uaca0\uc9c0\ub9cc \uadf8 \uc774\ud6c4\ub294 \ub300\ucc45\uc774 \uc5c6\ub2e4\"\ub77c\uba70 \ubc18\ub300\uc785\uc7a5\uc744 \ubd84\uba85\ud788 \ud588\ub2e4. \uc2e0\uacf5\ud56d \ud6c4\ubcf4 \uc9c0\uc5ed \uc8fc\ubbfc\ub4e4\uc740 \uc2e0\uacf5\ud56d \uc720\uce58\uc640 \uad00\ub828\ud574\uc11c \uc790\uc2e0\ub4e4\uc758 \uc758\uacac\uc774 \ubc30\uc81c\ub418\uc5c8\ub2e4\ub294 \uc0ac\uc2e4\uc5d0 \ubd84\uac1c\ud558\ub294 \uc0ac\ub78c\ub4e4\ub3c4 \ub9ce\uc558\ub2e4. \uc2e0\uacf5\ud56d \uc720\uce58\uc640 \uad00\ub828\ud574\uc11c \uc8fc\ubbfc\ub4e4\uacfc\uc758 \ub300\ud654\uac00 \ub354 \ud544\uc694\ud558\ub2e4\ub294 \uc9c0\uc801\uc774\ub2e4. \ud558\uc9c0\ub9cc \ubc00\uc591\uce21\uc740 \ud6c4\ubcf4\uc9c0 \uc8fc\ubbfc\ub4e4\uacfc \ub300\ud654\ub97c \ud560 \ub2e8\uacc4\ub294 \uc544\ub2c8\ub77c\uace0 \uc798\ub77c \ub9d0\ud55c\ub2e4. \uc2e0\uacf5\ud56d \uc704\uce58\uac00 \ud655\uc815\ub418\uba74 \uadf8 \ub54c\ubd80\ud130 \uc774\uc57c\uae30\ub97c \ud55c\ub2e4\ub294 \uac83\uc774\ub2e4. \ubd80\uc0b0\uad11\uc5ed\uc2dc\ub294 \"\ud604\uc7ac \uc5b4\uc5c5\uc744 \ud558\uace0 \uc788\ub294 \uc5b4\ubbfc\ub4e4\uc5d0\uac8c\ub294 \uc801\uc808\ud55c \ubcf4\uc0c1\uc774 \ub420 \uc218 \uc788\ub3c4\ub85d \uc2dc\uac00 \uc815\ubd80\uc5d0 \uc694\uad6c\ub97c \ud558\uace0 \uc5b4\ubbfc\ud3b8\uc5d0\uc11c \uc77c\uc744 \ud558\uaca0\ub2e4\"\uace0 \ub9d0\ud588\ub2e4.", "answer": "200~205\ub9cc\uc6d0", "sentence": "\uac10\uc790\ubc2d\uc744 \uc6b4\uc601\ud558\ub294 \ub2e4\ub978 \uc8fc\ubbfc\ub3c4 \"\uac10\uc790\ubc2d\uc5d0\uc11c \ud55c\ub2e4\ub808 200~205\ub9cc\uc6d0 \uc529 \uc218\uc775\uc774 \ub098\uc624\ub294\ub370 \uc774\uac83\uc744 \ucca0\uac70\ud558\uba74 \uc815\ubd80\uc5d0\uc11c 200\ub9cc\uc6d0 \uc774\uc0c1 \uc8fc\uaca0\ub098\"\ub77c\uba70 \"\ub2f9\uc7a5 \ub098\uac00\uba74 1~2\ub144\uc740 \uba39\uace0\uc0b4\uaca0\uc9c0\ub9cc \uadf8 \uc774\ud6c4\ub294 \ub300\ucc45\uc774 \uc5c6\ub2e4\"\ub77c\uba70 \ubc18\ub300\uc785\uc7a5\uc744 \ubd84\uba85\ud788 \ud588\ub2e4.", "paragraph_sentence": "\ubc00\uc591\uc2dc \ud558\ub0a8\uc74d\uc5d0 8,000\uc5ec\uba85\uc774 \ub18d\uc0ac\ub97c \uc9c0\uc73c\uba70 \uc9c0\ub0b4\uace0 \uc788\ub2e4. \ub545\uc774 \uae30\ub984\uc838 \uac10\uc790\uc640 \ub538\uae30\uac00 \ub9ce\uc774 \ub09c\ub2e4. \ud558\uc9c0\ub9cc \uc2e0\uacf5\ud56d\uc774 \ub4e4\uc5b4\uc11c\uba74 \ub354\uc774\uc0c1 \ub18d\uc0ac\ub97c \uc9c0\uc744 \uc218 \uc5c6\uac8c \ub41c\ub2e4. \uc774 \ub54c\ubb38\uc5d0 \uc77c\ubd80 \uc8fc\ubbfc\ub4e4\uc740 \uc2e0\uacf5\ud56d \uc720\uce58\ub97c \ubc18\ub300\ud558\uace0 \uc788\uc5c8\ub2e4. \ud55c \uc8fc\ubbfc\uc740 \"\ub18d\uc0ac\uc9d3\uace0 \uc788\ub294\ub370 \ub098\uac00\ub77c\uace0 \ud558\uba74 \ubcf4\uc0c1\ub3c4 \uc5c6\uace0 \ubc14\ub85c \uac70\uc9c0\ub2e4\"\ub77c\uba70 \"\uc6b0\ub9ac\ub294 \ud3c9\uc0c1 \ube44\ud589\uae30 \ud0c8\uc77c \ub450\ubc88\ub3c4 \uc5c6\ub294\ub370\"\ub77c\uba70 \uaca9\ubd84\ud588\ub2e4. \uac10\uc790\ubc2d\uc744 \uc6b4\uc601\ud558\ub294 \ub2e4\ub978 \uc8fc\ubbfc\ub3c4 \"\uac10\uc790\ubc2d\uc5d0\uc11c \ud55c\ub2e4\ub808 200~205\ub9cc\uc6d0 \uc529 \uc218\uc775\uc774 \ub098\uc624\ub294\ub370 \uc774\uac83\uc744 \ucca0\uac70\ud558\uba74 \uc815\ubd80\uc5d0\uc11c 200\ub9cc\uc6d0 \uc774\uc0c1 \uc8fc\uaca0\ub098\"\ub77c\uba70 \"\ub2f9\uc7a5 \ub098\uac00\uba74 1~2\ub144\uc740 \uba39\uace0\uc0b4\uaca0\uc9c0\ub9cc \uadf8 \uc774\ud6c4\ub294 \ub300\ucc45\uc774 \uc5c6\ub2e4\"\ub77c\uba70 \ubc18\ub300\uc785\uc7a5\uc744 \ubd84\uba85\ud788 \ud588\ub2e4. \uc2e0\uacf5\ud56d \ud6c4\ubcf4 \uc9c0\uc5ed \uc8fc\ubbfc\ub4e4\uc740 \uc2e0\uacf5\ud56d \uc720\uce58\uc640 \uad00\ub828\ud574\uc11c \uc790\uc2e0\ub4e4\uc758 \uc758\uacac\uc774 \ubc30\uc81c\ub418\uc5c8\ub2e4\ub294 \uc0ac\uc2e4\uc5d0 \ubd84\uac1c\ud558\ub294 \uc0ac\ub78c\ub4e4\ub3c4 \ub9ce\uc558\ub2e4. \uc2e0\uacf5\ud56d \uc720\uce58\uc640 \uad00\ub828\ud574\uc11c \uc8fc\ubbfc\ub4e4\uacfc\uc758 \ub300\ud654\uac00 \ub354 \ud544\uc694\ud558\ub2e4\ub294 \uc9c0\uc801\uc774\ub2e4. \ud558\uc9c0\ub9cc \ubc00\uc591\uce21\uc740 \ud6c4\ubcf4\uc9c0 \uc8fc\ubbfc\ub4e4\uacfc \ub300\ud654\ub97c \ud560 \ub2e8\uacc4\ub294 \uc544\ub2c8\ub77c\uace0 \uc798\ub77c \ub9d0\ud55c\ub2e4. \uc2e0\uacf5\ud56d \uc704\uce58\uac00 \ud655\uc815\ub418\uba74 \uadf8 \ub54c\ubd80\ud130 \uc774\uc57c\uae30\ub97c \ud55c\ub2e4\ub294 \uac83\uc774\ub2e4. \ubd80\uc0b0\uad11\uc5ed\uc2dc\ub294 \"\ud604\uc7ac \uc5b4\uc5c5\uc744 \ud558\uace0 \uc788\ub294 \uc5b4\ubbfc\ub4e4\uc5d0\uac8c\ub294 \uc801\uc808\ud55c \ubcf4\uc0c1\uc774 \ub420 \uc218 \uc788\ub3c4\ub85d \uc2dc\uac00 \uc815\ubd80\uc5d0 \uc694\uad6c\ub97c \ud558\uace0 \uc5b4\ubbfc\ud3b8\uc5d0\uc11c \uc77c\uc744 \ud558\uaca0\ub2e4\"\uace0 \ub9d0\ud588\ub2e4.", "paragraph_answer": "\ubc00\uc591\uc2dc \ud558\ub0a8\uc74d\uc5d0 8,000\uc5ec\uba85\uc774 \ub18d\uc0ac\ub97c \uc9c0\uc73c\uba70 \uc9c0\ub0b4\uace0 \uc788\ub2e4. \ub545\uc774 \uae30\ub984\uc838 \uac10\uc790\uc640 \ub538\uae30\uac00 \ub9ce\uc774 \ub09c\ub2e4. \ud558\uc9c0\ub9cc \uc2e0\uacf5\ud56d\uc774 \ub4e4\uc5b4\uc11c\uba74 \ub354\uc774\uc0c1 \ub18d\uc0ac\ub97c \uc9c0\uc744 \uc218 \uc5c6\uac8c \ub41c\ub2e4. \uc774 \ub54c\ubb38\uc5d0 \uc77c\ubd80 \uc8fc\ubbfc\ub4e4\uc740 \uc2e0\uacf5\ud56d \uc720\uce58\ub97c \ubc18\ub300\ud558\uace0 \uc788\uc5c8\ub2e4. \ud55c \uc8fc\ubbfc\uc740 \"\ub18d\uc0ac\uc9d3\uace0 \uc788\ub294\ub370 \ub098\uac00\ub77c\uace0 \ud558\uba74 \ubcf4\uc0c1\ub3c4 \uc5c6\uace0 \ubc14\ub85c \uac70\uc9c0\ub2e4\"\ub77c\uba70 \"\uc6b0\ub9ac\ub294 \ud3c9\uc0c1 \ube44\ud589\uae30 \ud0c8\uc77c \ub450\ubc88\ub3c4 \uc5c6\ub294\ub370\"\ub77c\uba70 \uaca9\ubd84\ud588\ub2e4. \uac10\uc790\ubc2d\uc744 \uc6b4\uc601\ud558\ub294 \ub2e4\ub978 \uc8fc\ubbfc\ub3c4 \"\uac10\uc790\ubc2d\uc5d0\uc11c \ud55c\ub2e4\ub808 200~205\ub9cc\uc6d0 \uc529 \uc218\uc775\uc774 \ub098\uc624\ub294\ub370 \uc774\uac83\uc744 \ucca0\uac70\ud558\uba74 \uc815\ubd80\uc5d0\uc11c 200\ub9cc\uc6d0 \uc774\uc0c1 \uc8fc\uaca0\ub098\"\ub77c\uba70 \"\ub2f9\uc7a5 \ub098\uac00\uba74 1~2\ub144\uc740 \uba39\uace0\uc0b4\uaca0\uc9c0\ub9cc \uadf8 \uc774\ud6c4\ub294 \ub300\ucc45\uc774 \uc5c6\ub2e4\"\ub77c\uba70 \ubc18\ub300\uc785\uc7a5\uc744 \ubd84\uba85\ud788 \ud588\ub2e4. \uc2e0\uacf5\ud56d \ud6c4\ubcf4 \uc9c0\uc5ed \uc8fc\ubbfc\ub4e4\uc740 \uc2e0\uacf5\ud56d \uc720\uce58\uc640 \uad00\ub828\ud574\uc11c \uc790\uc2e0\ub4e4\uc758 \uc758\uacac\uc774 \ubc30\uc81c\ub418\uc5c8\ub2e4\ub294 \uc0ac\uc2e4\uc5d0 \ubd84\uac1c\ud558\ub294 \uc0ac\ub78c\ub4e4\ub3c4 \ub9ce\uc558\ub2e4. \uc2e0\uacf5\ud56d \uc720\uce58\uc640 \uad00\ub828\ud574\uc11c \uc8fc\ubbfc\ub4e4\uacfc\uc758 \ub300\ud654\uac00 \ub354 \ud544\uc694\ud558\ub2e4\ub294 \uc9c0\uc801\uc774\ub2e4. \ud558\uc9c0\ub9cc \ubc00\uc591\uce21\uc740 \ud6c4\ubcf4\uc9c0 \uc8fc\ubbfc\ub4e4\uacfc \ub300\ud654\ub97c \ud560 \ub2e8\uacc4\ub294 \uc544\ub2c8\ub77c\uace0 \uc798\ub77c \ub9d0\ud55c\ub2e4. \uc2e0\uacf5\ud56d \uc704\uce58\uac00 \ud655\uc815\ub418\uba74 \uadf8 \ub54c\ubd80\ud130 \uc774\uc57c\uae30\ub97c \ud55c\ub2e4\ub294 \uac83\uc774\ub2e4. \ubd80\uc0b0\uad11\uc5ed\uc2dc\ub294 \"\ud604\uc7ac \uc5b4\uc5c5\uc744 \ud558\uace0 \uc788\ub294 \uc5b4\ubbfc\ub4e4\uc5d0\uac8c\ub294 \uc801\uc808\ud55c \ubcf4\uc0c1\uc774 \ub420 \uc218 \uc788\ub3c4\ub85d \uc2dc\uac00 \uc815\ubd80\uc5d0 \uc694\uad6c\ub97c \ud558\uace0 \uc5b4\ubbfc\ud3b8\uc5d0\uc11c \uc77c\uc744 \ud558\uaca0\ub2e4\"\uace0 \ub9d0\ud588\ub2e4.", "sentence_answer": "\uac10\uc790\ubc2d\uc744 \uc6b4\uc601\ud558\ub294 \ub2e4\ub978 \uc8fc\ubbfc\ub3c4 \"\uac10\uc790\ubc2d\uc5d0\uc11c \ud55c\ub2e4\ub808 200~205\ub9cc\uc6d0 \uc529 \uc218\uc775\uc774 \ub098\uc624\ub294\ub370 \uc774\uac83\uc744 \ucca0\uac70\ud558\uba74 \uc815\ubd80\uc5d0\uc11c 200\ub9cc\uc6d0 \uc774\uc0c1 \uc8fc\uaca0\ub098\"\ub77c\uba70 \"\ub2f9\uc7a5 \ub098\uac00\uba74 1~2\ub144\uc740 \uba39\uace0\uc0b4\uaca0\uc9c0\ub9cc \uadf8 \uc774\ud6c4\ub294 \ub300\ucc45\uc774 \uc5c6\ub2e4\"\ub77c\uba70 \ubc18\ub300\uc785\uc7a5\uc744 \ubd84\uba85\ud788 \ud588\ub2e4.", "paragraph_id": "6458677-6-1", "paragraph_question": "question: \uac10\uc790\ubc2d\uc5d0\uc11c \ud55c\ub2e4\ub808\uc5d0 \ub098\uc624\ub294 \uc218\uc775\uc740 \uc5bc\ub9c8\uc778\uac00?, context: \ubc00\uc591\uc2dc \ud558\ub0a8\uc74d\uc5d0 8,000\uc5ec\uba85\uc774 \ub18d\uc0ac\ub97c \uc9c0\uc73c\uba70 \uc9c0\ub0b4\uace0 \uc788\ub2e4. \ub545\uc774 \uae30\ub984\uc838 \uac10\uc790\uc640 \ub538\uae30\uac00 \ub9ce\uc774 \ub09c\ub2e4. \ud558\uc9c0\ub9cc \uc2e0\uacf5\ud56d\uc774 \ub4e4\uc5b4\uc11c\uba74 \ub354\uc774\uc0c1 \ub18d\uc0ac\ub97c \uc9c0\uc744 \uc218 \uc5c6\uac8c \ub41c\ub2e4. \uc774 \ub54c\ubb38\uc5d0 \uc77c\ubd80 \uc8fc\ubbfc\ub4e4\uc740 \uc2e0\uacf5\ud56d \uc720\uce58\ub97c \ubc18\ub300\ud558\uace0 \uc788\uc5c8\ub2e4. \ud55c \uc8fc\ubbfc\uc740 \"\ub18d\uc0ac\uc9d3\uace0 \uc788\ub294\ub370 \ub098\uac00\ub77c\uace0 \ud558\uba74 \ubcf4\uc0c1\ub3c4 \uc5c6\uace0 \ubc14\ub85c \uac70\uc9c0\ub2e4\"\ub77c\uba70 \"\uc6b0\ub9ac\ub294 \ud3c9\uc0c1 \ube44\ud589\uae30 \ud0c8\uc77c \ub450\ubc88\ub3c4 \uc5c6\ub294\ub370\"\ub77c\uba70 \uaca9\ubd84\ud588\ub2e4. \uac10\uc790\ubc2d\uc744 \uc6b4\uc601\ud558\ub294 \ub2e4\ub978 \uc8fc\ubbfc\ub3c4 \"\uac10\uc790\ubc2d\uc5d0\uc11c \ud55c\ub2e4\ub808 200~205\ub9cc\uc6d0\uc529 \uc218\uc775\uc774 \ub098\uc624\ub294\ub370 \uc774\uac83\uc744 \ucca0\uac70\ud558\uba74 \uc815\ubd80\uc5d0\uc11c 200\ub9cc\uc6d0 \uc774\uc0c1 \uc8fc\uaca0\ub098\"\ub77c\uba70 \"\ub2f9\uc7a5 \ub098\uac00\uba74 1~2\ub144\uc740 \uba39\uace0\uc0b4\uaca0\uc9c0\ub9cc \uadf8 \uc774\ud6c4\ub294 \ub300\ucc45\uc774 \uc5c6\ub2e4\"\ub77c\uba70 \ubc18\ub300\uc785\uc7a5\uc744 \ubd84\uba85\ud788 \ud588\ub2e4. \uc2e0\uacf5\ud56d \ud6c4\ubcf4 \uc9c0\uc5ed \uc8fc\ubbfc\ub4e4\uc740 \uc2e0\uacf5\ud56d \uc720\uce58\uc640 \uad00\ub828\ud574\uc11c \uc790\uc2e0\ub4e4\uc758 \uc758\uacac\uc774 \ubc30\uc81c\ub418\uc5c8\ub2e4\ub294 \uc0ac\uc2e4\uc5d0 \ubd84\uac1c\ud558\ub294 \uc0ac\ub78c\ub4e4\ub3c4 \ub9ce\uc558\ub2e4. \uc2e0\uacf5\ud56d \uc720\uce58\uc640 \uad00\ub828\ud574\uc11c \uc8fc\ubbfc\ub4e4\uacfc\uc758 \ub300\ud654\uac00 \ub354 \ud544\uc694\ud558\ub2e4\ub294 \uc9c0\uc801\uc774\ub2e4. \ud558\uc9c0\ub9cc \ubc00\uc591\uce21\uc740 \ud6c4\ubcf4\uc9c0 \uc8fc\ubbfc\ub4e4\uacfc \ub300\ud654\ub97c \ud560 \ub2e8\uacc4\ub294 \uc544\ub2c8\ub77c\uace0 \uc798\ub77c \ub9d0\ud55c\ub2e4. \uc2e0\uacf5\ud56d \uc704\uce58\uac00 \ud655\uc815\ub418\uba74 \uadf8 \ub54c\ubd80\ud130 \uc774\uc57c\uae30\ub97c \ud55c\ub2e4\ub294 \uac83\uc774\ub2e4. \ubd80\uc0b0\uad11\uc5ed\uc2dc\ub294 \"\ud604\uc7ac \uc5b4\uc5c5\uc744 \ud558\uace0 \uc788\ub294 \uc5b4\ubbfc\ub4e4\uc5d0\uac8c\ub294 \uc801\uc808\ud55c \ubcf4\uc0c1\uc774 \ub420 \uc218 \uc788\ub3c4\ub85d \uc2dc\uac00 \uc815\ubd80\uc5d0 \uc694\uad6c\ub97c \ud558\uace0 \uc5b4\ubbfc\ud3b8\uc5d0\uc11c \uc77c\uc744 \ud558\uaca0\ub2e4\"\uace0 \ub9d0\ud588\ub2e4."} {"question": "\ub2c8\uccb4\uc758 \uc608\uc220\uac00 \ubcf5\uc74c\uc5d0 \ub530\ub974\uba74 \uc885\uad50\uac00 \uc544\ub2cc \uc608\uc220\uc774 \uc0b6\uc758 \ubb34\uc5c7\uc778\uac00?", "paragraph": "\ubbf8\ud559\uc801 \ud604\ud604\uc73c\ub85c\uc11c\uc758 \uc601\uc6d0 \ud68c\uadc0. \"\uc601\uc6d0 \ud68c\uadc0\"\ub77c\ub294 \uc0dd\uac01\uc740 \ucc28\ub77c\ud22c\uc2a4\ud2b8\ub77c\uc758 \uae30\ubcf8 \uc0dd\uac01\uc774\uace0 \ub2c8\uccb4\uc758 \"\uac00\uc7a5 \uae4a\uc219\ud55c \uacf3\uc5d0 \uc788\ub294 \uc0dd\uac01\"\uc774\ub2e4. \uc601\uc6d0\ud55c \ud68c\uadc0\ub294 \uc624\uc9c1 \uc21c\uac04\uc73c\ub85c\ubd80\ud130\ub9cc \uc774\ud574\ub420 \uc218 \uc788\ub2e4. \"\ubaa9\ud45c \uc5c6\ub294 \uc2dc\uac04\"\uc774\ub77c\ub294 \ubb34\uc2dc\ubb34\uc2dc\ud55c \uc21c\uac04 \uc18d\uc5d0\uc11c \uc778\uac04\uc740 \uc790\uc2e0\uc758 \uc0b6\uc758 \ubcf8\ub798\uc758 \uc784\ubb34\ub97c \uacbd\ud5d8\ud55c\ub2e4. \uc601\uc6d0\ud55c \ud68c\uadc0\uc758 \uc2e0\uc801\uc778 \uc21c\uac04\uc740 \ubbf8\ud559\uc801, \uc989 \ube44\uadf9\uc801-\ub514\uc624\ub2c8\uc18c\uc2a4\uc801 \uc0c1\ud0dc\uc758 \uc21c\uac04\uc774\ub2e4. \uc774 \uc0c1\ud0dc\ub294 \ud604\uc874\uc7ac\uc5d0 \ub300\ud55c \uae0d\uc815(\uc544\ubaa8\ub974 \ud30c\ud2f0)\uc774 \uc774\ub8e8\uc5b4\uc9c0\ub294 \ucd5c\uc0c1\uc758 \uc0c1\ud0dc\uc774\uace0, \uadf8 \uc0c1\ud0dc \uc18d\uc5d0\uc11c \ud5c8\ubb34\uc8fc\uc758, \uc778\uac04\uacfc \ubaa8\ub4e0 \ud604\uc874\uc7ac\uc758 \uacfc\uc789\uc774 \uadf9\ubcf5\ub41c\ub2e4. \ube44\uadf9\uc801-\ub514\uc624\ub2c8\uc18c\uc2a4\uc801 \uc0c1\ud0dc\ub294 \uac00\uc7a5 \uc791\uace0 \ubcf4\uc798\uac83\uc5c6\ub294 \uac83\uc758 \ud68c\uadc0\uc77c\uc9c0\ub77c\ub3c4 \uc601\uc6d0 \ud68c\uadc0\uc5d0 \ub300\ud55c \uc0dd\uac01\uc774 \ubc1b\uc544\ub4e4\uc5ec\uc9c0\ub294 \uc0c1\ud0dc\uc774\ub2e4. \ucd5c\uc0c1\uc758 \uc138\uacc4 \uc644\uc131\uc758 \uc21c\uac04, \ub2c8\uccb4\uac00 \uc5d0\uba38\uc2a8\uc758 \ub9d0\uc744 \ube4c\uc5b4\uc11c \ub9d0\ud558\ub294 \uac83\ucc98\ub7fc, \uc138\uc0c1\uc774 \uc644\ubcbd\ud558\uac8c \ub418\ub294 \"\uac11\uc791\uc2a4\ub7f0 \uc601\uc6d0\"\uc758 \uc2e0\uc801 \uc21c\uac04, \uc6c3\ub294 \uc2e0 \ub514\uc624\ub2c8\uc18c\uc2a4\uac00 \ucda4\uc744 \ucd94\uba74\uc11c \uc6b0\ub9ac \ubab8\uc744 \uad00\ud1b5\ud574 \uc9c0\ub098\uac00\ub294 \ub514\uc624\ub2c8\uc18c\uc2a4\uc801 \uc21c\uac04\uc740 \uc138\uc0c1\uc744 \ubbf8\ud559\uc801 \ud604\uc0c1\uc73c\ub85c \uc601\uc6d0\ud788 \uc815\ub2f9\ud654\ud558\ub294 \uc21c\uac04\uc774\ub2e4. \uc774\ub7f0 \uacbd\ud5d8, \"\uc644\ubcbd\ud55c \uc815\uc624\"\uc758 \uac00\uc7a5 \uace0\uc694\ud55c \uc2dc\uac04\uc5d0 \uc77c\uc5b4\ub098\ub294 \ubc1d\uc740 \ub300\ub0ae\uc758 \"\uc2e0\ube44\uc2a4\ub7ec\uc6b4 \uc9c1\uad00\"\uc73c\ub85c \ub2c8\uccb4\ub294 \ucc28\ub77c\ud22c\uc2a4\ud2b8\ub77c\uc758 \uae30\ubcf8 \uc0dd\uac01\uc774\uae30\ub3c4 \ud55c \uc790\uc2e0\uc758 \ucca0\ud559\ud558\uae30\uc758 \uc6d0\ub798 \ubaa9\uc801\uacfc \uae30\ubcf8 \uacbd\ud5d8\uc744 \ubd84\uba85\ud558\uac8c \ub9d0\ud55c\ub2e4. \ubbf8(\u7f8e) \uc18d\uc5d0\uc11c \ubaa8\uc21c\ub4e4\uc740 \uc5b5\uc81c\ub41c\ub2e4. \uadf8\uac83\uc740 \uc11c\ub85c \ubaa8\uc21c \ub418\ub294 \uac83\uc744 \uc9c0\ubc30\ud558\ub294 \ud798\uc758 \ucd5c\uace0 \ud45c\uc2dd\uc774\ub2e4. \uac11\uc791\uc2a4\ub7f0 \uc601\uc6d0 \ud68c\uadc0\uc758 \uc21c\uac04\uc778 \ubbf8\uc758 \ub514\uc624\ub2c8\uc18c\uc2a4\uc801 \uc21c\uac04 \uc18d\uc5d0\uc11c \uc9c0\ub098\uac04 \uac83\uacfc \ubbf8\ub798\uc758 \uac83\uc758 \ucda9\ub3d9\uacfc \uacf5\uc874\uc758 \uacbd\ud5d8\uc774 \ud604\uc7ac\uc758 \uc21c\uac04\uc5d0\uc11c \uc774\ub8e8\uc5b4\uc9c4\ub2e4. \uc601\uc6d0\uc774 \uc2dc\uac04\uc774 \ub418\uace0, \uc2dc\uac04\uc774 \uc601\uc6d0\uc774 \ub41c\ub2e4. \uc774 \uc21c\uac04\uc5d0 \uc2dc\uac04\uc740 \uac11\uc791\uc2a4\ub7f0 \uc601\uc6d0\uc758 \uc815\uc624-\uc21c\uac04\uc73c\ub85c \uc9c0\uc591\ub41c\ub2e4. \ucc28\ub77c\ud22c\uc2a4\ud2b8\ub77c\ub294 \uc704\ubc84\uba58\uc26c\uc758 \uc9c0\uc704\uc5d0\uc11c\ubd80\ud130 \ud68c\uadc0\ub97c \uac00\ub974\uce5c\ub2e4. \uc704\ubc84\uba58\uc26c\ub294 \"\uc704\ub300\ud55c \ud68c\uadc0\uc758 \ub144\ub3c4\"\uc5d0 \uc138\uc0c1\uc758 \uc2ec\uc5f0\uc744 \ubcf4\uace0 \uacac\ub514\uc5b4 \ub0b8 \uc21c\uac04\uc5d0 \uc790\uc2e0\uc744 \ub118\uc5b4\uc11c\uace0, \ube44\uadf9\uc801-\ub514\uc624\ub2c8\uc18c\uc2a4\uc801 \uc0c1\ud0dc\uc5d0\uc11c \uc790\uc544\ub97c \uc78a\uc5b4\ubc84\ub9ac\ub294 \ucd08\uac1c\uc778\uc801\uc774\uace0 \ucc3d\uc870\uc801\uc778 \uc778\uac04\uc774\ub2e4. \uc704\ubc84\uba58\uc26c\ub294 \"\uc601\uc6d0\uc758 \uc0d8\ubb3c\" \uc18d\uc73c\ub85c \uc21c\uac04\uc758 \ubc88\uac2f\ubd88\uc744 \ub358\uc9c0\ub294 \uc0ac\ub78c\uc774\ub2e4. \uc704\ubc84\uba58\uc26c\ub294 \uadf8\ub9bc\uc790\uac00 \uac00\uc7a5 \uc9e7\uc544\uc9c0\ub294 \uc21c\uac04\uc5d0 \uc601\uc6d0 \ud68c\uadc0\uc758 \"\uc815\uc624\uc758 \uc2ec\uc5f0\" \uc18d\uc73c\ub85c \ub5a8\uc5b4\uc9c0\ub294 \ubc88\uac2f\ubd88\uc774\ub2e4. \uc601\uc6d0 \ud68c\uadc0\uc758 \uc21c\uac04\uc740 \uc544\ud3f4\ub85c\uc801 \ub514\uc624\ub2c8\uc18c\uc2a4, \uc989 \ube44\ub3c4\ub355\uc801\uc778 \uc608\uc220\uac00 \uc2e0 \ub514\uc624\ub2c8\uc18c\uc2a4 \uc790\uadf8\ub808\uc6b0\uc2a4\uac00 \ubbf8\ud559\uc801\uc73c\ub85c \ud604\ud604\ud558\ub294 \uc21c\uac04\uc774\ub2e4. \ub2c8\uccb4\uc758 \uc608\uc220\uac00 \ubcf5\uc74c\uc744 \ub530\ub974\uc790\uba74 \uc885\uad50\uac00 \uc544\ub2c8\ub77c, \uc608\uc220\uc774 \uc0b6\uc758 \"\ud615\uc774\uc0c1\ud559\uc801 \ud65c\ub3d9\"\uc774\ub2e4. \uc608\uc220, \uac00\uc7a5 \"\ucee4\ub2e4\ub780 \uc0b6\uc758 \uc790\uadf9\"\uc774 \uc601\uc6d0 \ud68c\uadc0 \ucca0\ud559\uc758 \uae30\uad00\uc774\ub2e4. \ub2c8\uccb4\uc758 \uc601\uc6d0 \ud68c\uadc0\uc758 \uae30\ubcf8 \uc774\ub860\uc740 \uc608\uc220\uc758 \ud615\uc774\uc0c1\ud559\uc774\uace0, \ubc18(\u53cd)\uadf8\ub9ac\uc2a4\ub3c4\uc801 \uc608\uc220-\uc885\uad50\uc774\uace0, \"\uc608\uc220\uac00-\ud615\uc774\uc0c1\ud559\"\uc774\ub2e4.", "answer": "\ud615\uc774\uc0c1\ud559\uc801 \ud65c\ub3d9", "sentence": "\ub2c8\uccb4\uc758 \uc608\uc220\uac00 \ubcf5\uc74c\uc744 \ub530\ub974\uc790\uba74 \uc885\uad50\uac00 \uc544\ub2c8\ub77c, \uc608\uc220\uc774 \uc0b6\uc758 \" \ud615\uc774\uc0c1\ud559\uc801 \ud65c\ub3d9 \"\uc774\ub2e4.", "paragraph_sentence": "\ubbf8\ud559\uc801 \ud604\ud604\uc73c\ub85c\uc11c\uc758 \uc601\uc6d0 \ud68c\uadc0. \"\uc601\uc6d0 \ud68c\uadc0\"\ub77c\ub294 \uc0dd\uac01\uc740 \ucc28\ub77c\ud22c\uc2a4\ud2b8\ub77c\uc758 \uae30\ubcf8 \uc0dd\uac01\uc774\uace0 \ub2c8\uccb4\uc758 \"\uac00\uc7a5 \uae4a\uc219\ud55c \uacf3\uc5d0 \uc788\ub294 \uc0dd\uac01\"\uc774\ub2e4. \uc601\uc6d0\ud55c \ud68c\uadc0\ub294 \uc624\uc9c1 \uc21c\uac04\uc73c\ub85c\ubd80\ud130\ub9cc \uc774\ud574\ub420 \uc218 \uc788\ub2e4. \"\ubaa9\ud45c \uc5c6\ub294 \uc2dc\uac04\"\uc774\ub77c\ub294 \ubb34\uc2dc\ubb34\uc2dc\ud55c \uc21c\uac04 \uc18d\uc5d0\uc11c \uc778\uac04\uc740 \uc790\uc2e0\uc758 \uc0b6\uc758 \ubcf8\ub798\uc758 \uc784\ubb34\ub97c \uacbd\ud5d8\ud55c\ub2e4. \uc601\uc6d0\ud55c \ud68c\uadc0\uc758 \uc2e0\uc801\uc778 \uc21c\uac04\uc740 \ubbf8\ud559\uc801, \uc989 \ube44\uadf9\uc801-\ub514\uc624\ub2c8\uc18c\uc2a4\uc801 \uc0c1\ud0dc\uc758 \uc21c\uac04\uc774\ub2e4. \uc774 \uc0c1\ud0dc\ub294 \ud604\uc874\uc7ac\uc5d0 \ub300\ud55c \uae0d\uc815(\uc544\ubaa8\ub974 \ud30c\ud2f0)\uc774 \uc774\ub8e8\uc5b4\uc9c0\ub294 \ucd5c\uc0c1\uc758 \uc0c1\ud0dc\uc774\uace0, \uadf8 \uc0c1\ud0dc \uc18d\uc5d0\uc11c \ud5c8\ubb34\uc8fc\uc758, \uc778\uac04\uacfc \ubaa8\ub4e0 \ud604\uc874\uc7ac\uc758 \uacfc\uc789\uc774 \uadf9\ubcf5\ub41c\ub2e4. \ube44\uadf9\uc801-\ub514\uc624\ub2c8\uc18c\uc2a4\uc801 \uc0c1\ud0dc\ub294 \uac00\uc7a5 \uc791\uace0 \ubcf4\uc798\uac83\uc5c6\ub294 \uac83\uc758 \ud68c\uadc0\uc77c\uc9c0\ub77c\ub3c4 \uc601\uc6d0 \ud68c\uadc0\uc5d0 \ub300\ud55c \uc0dd\uac01\uc774 \ubc1b\uc544\ub4e4\uc5ec\uc9c0\ub294 \uc0c1\ud0dc\uc774\ub2e4. \ucd5c\uc0c1\uc758 \uc138\uacc4 \uc644\uc131\uc758 \uc21c\uac04, \ub2c8\uccb4\uac00 \uc5d0\uba38\uc2a8\uc758 \ub9d0\uc744 \ube4c\uc5b4\uc11c \ub9d0\ud558\ub294 \uac83\ucc98\ub7fc, \uc138\uc0c1\uc774 \uc644\ubcbd\ud558\uac8c \ub418\ub294 \"\uac11\uc791\uc2a4\ub7f0 \uc601\uc6d0\"\uc758 \uc2e0\uc801 \uc21c\uac04, \uc6c3\ub294 \uc2e0 \ub514\uc624\ub2c8\uc18c\uc2a4\uac00 \ucda4\uc744 \ucd94\uba74\uc11c \uc6b0\ub9ac \ubab8\uc744 \uad00\ud1b5\ud574 \uc9c0\ub098\uac00\ub294 \ub514\uc624\ub2c8\uc18c\uc2a4\uc801 \uc21c\uac04\uc740 \uc138\uc0c1\uc744 \ubbf8\ud559\uc801 \ud604\uc0c1\uc73c\ub85c \uc601\uc6d0\ud788 \uc815\ub2f9\ud654\ud558\ub294 \uc21c\uac04\uc774\ub2e4. \uc774\ub7f0 \uacbd\ud5d8, \"\uc644\ubcbd\ud55c \uc815\uc624\"\uc758 \uac00\uc7a5 \uace0\uc694\ud55c \uc2dc\uac04\uc5d0 \uc77c\uc5b4\ub098\ub294 \ubc1d\uc740 \ub300\ub0ae\uc758 \"\uc2e0\ube44\uc2a4\ub7ec\uc6b4 \uc9c1\uad00\"\uc73c\ub85c \ub2c8\uccb4\ub294 \ucc28\ub77c\ud22c\uc2a4\ud2b8\ub77c\uc758 \uae30\ubcf8 \uc0dd\uac01\uc774\uae30\ub3c4 \ud55c \uc790\uc2e0\uc758 \ucca0\ud559\ud558\uae30\uc758 \uc6d0\ub798 \ubaa9\uc801\uacfc \uae30\ubcf8 \uacbd\ud5d8\uc744 \ubd84\uba85\ud558\uac8c \ub9d0\ud55c\ub2e4. \ubbf8(\u7f8e) \uc18d\uc5d0\uc11c \ubaa8\uc21c\ub4e4\uc740 \uc5b5\uc81c\ub41c\ub2e4. \uadf8\uac83\uc740 \uc11c\ub85c \ubaa8\uc21c \ub418\ub294 \uac83\uc744 \uc9c0\ubc30\ud558\ub294 \ud798\uc758 \ucd5c\uace0 \ud45c\uc2dd\uc774\ub2e4. \uac11\uc791\uc2a4\ub7f0 \uc601\uc6d0 \ud68c\uadc0\uc758 \uc21c\uac04\uc778 \ubbf8\uc758 \ub514\uc624\ub2c8\uc18c\uc2a4\uc801 \uc21c\uac04 \uc18d\uc5d0\uc11c \uc9c0\ub098\uac04 \uac83\uacfc \ubbf8\ub798\uc758 \uac83\uc758 \ucda9\ub3d9\uacfc \uacf5\uc874\uc758 \uacbd\ud5d8\uc774 \ud604\uc7ac\uc758 \uc21c\uac04\uc5d0\uc11c \uc774\ub8e8\uc5b4\uc9c4\ub2e4. \uc601\uc6d0\uc774 \uc2dc\uac04\uc774 \ub418\uace0, \uc2dc\uac04\uc774 \uc601\uc6d0\uc774 \ub41c\ub2e4. \uc774 \uc21c\uac04\uc5d0 \uc2dc\uac04\uc740 \uac11\uc791\uc2a4\ub7f0 \uc601\uc6d0\uc758 \uc815\uc624-\uc21c\uac04\uc73c\ub85c \uc9c0\uc591\ub41c\ub2e4. \ucc28\ub77c\ud22c\uc2a4\ud2b8\ub77c\ub294 \uc704\ubc84\uba58\uc26c\uc758 \uc9c0\uc704\uc5d0\uc11c\ubd80\ud130 \ud68c\uadc0\ub97c \uac00\ub974\uce5c\ub2e4. \uc704\ubc84\uba58\uc26c\ub294 \"\uc704\ub300\ud55c \ud68c\uadc0\uc758 \ub144\ub3c4\"\uc5d0 \uc138\uc0c1\uc758 \uc2ec\uc5f0\uc744 \ubcf4\uace0 \uacac\ub514\uc5b4 \ub0b8 \uc21c\uac04\uc5d0 \uc790\uc2e0\uc744 \ub118\uc5b4\uc11c\uace0, \ube44\uadf9\uc801-\ub514\uc624\ub2c8\uc18c\uc2a4\uc801 \uc0c1\ud0dc\uc5d0\uc11c \uc790\uc544\ub97c \uc78a\uc5b4\ubc84\ub9ac\ub294 \ucd08\uac1c\uc778\uc801\uc774\uace0 \ucc3d\uc870\uc801\uc778 \uc778\uac04\uc774\ub2e4. \uc704\ubc84\uba58\uc26c\ub294 \"\uc601\uc6d0\uc758 \uc0d8\ubb3c\" \uc18d\uc73c\ub85c \uc21c\uac04\uc758 \ubc88\uac2f\ubd88\uc744 \ub358\uc9c0\ub294 \uc0ac\ub78c\uc774\ub2e4. \uc704\ubc84\uba58\uc26c\ub294 \uadf8\ub9bc\uc790\uac00 \uac00\uc7a5 \uc9e7\uc544\uc9c0\ub294 \uc21c\uac04\uc5d0 \uc601\uc6d0 \ud68c\uadc0\uc758 \"\uc815\uc624\uc758 \uc2ec\uc5f0\" \uc18d\uc73c\ub85c \ub5a8\uc5b4\uc9c0\ub294 \ubc88\uac2f\ubd88\uc774\ub2e4. \uc601\uc6d0 \ud68c\uadc0\uc758 \uc21c\uac04\uc740 \uc544\ud3f4\ub85c\uc801 \ub514\uc624\ub2c8\uc18c\uc2a4, \uc989 \ube44\ub3c4\ub355\uc801\uc778 \uc608\uc220\uac00 \uc2e0 \ub514\uc624\ub2c8\uc18c\uc2a4 \uc790\uadf8\ub808\uc6b0\uc2a4\uac00 \ubbf8\ud559\uc801\uc73c\ub85c \ud604\ud604\ud558\ub294 \uc21c\uac04\uc774\ub2e4. \ub2c8\uccb4\uc758 \uc608\uc220\uac00 \ubcf5\uc74c\uc744 \ub530\ub974\uc790\uba74 \uc885\uad50\uac00 \uc544\ub2c8\ub77c, \uc608\uc220\uc774 \uc0b6\uc758 \" \ud615\uc774\uc0c1\ud559\uc801 \ud65c\ub3d9 \"\uc774\ub2e4. \uc608\uc220, \uac00\uc7a5 \"\ucee4\ub2e4\ub780 \uc0b6\uc758 \uc790\uadf9\"\uc774 \uc601\uc6d0 \ud68c\uadc0 \ucca0\ud559\uc758 \uae30\uad00\uc774\ub2e4. \ub2c8\uccb4\uc758 \uc601\uc6d0 \ud68c\uadc0\uc758 \uae30\ubcf8 \uc774\ub860\uc740 \uc608\uc220\uc758 \ud615\uc774\uc0c1\ud559\uc774\uace0, \ubc18(\u53cd)\uadf8\ub9ac\uc2a4\ub3c4\uc801 \uc608\uc220-\uc885\uad50\uc774\uace0, \"\uc608\uc220\uac00-\ud615\uc774\uc0c1\ud559\"\uc774\ub2e4.", "paragraph_answer": "\ubbf8\ud559\uc801 \ud604\ud604\uc73c\ub85c\uc11c\uc758 \uc601\uc6d0 \ud68c\uadc0. \"\uc601\uc6d0 \ud68c\uadc0\"\ub77c\ub294 \uc0dd\uac01\uc740 \ucc28\ub77c\ud22c\uc2a4\ud2b8\ub77c\uc758 \uae30\ubcf8 \uc0dd\uac01\uc774\uace0 \ub2c8\uccb4\uc758 \"\uac00\uc7a5 \uae4a\uc219\ud55c \uacf3\uc5d0 \uc788\ub294 \uc0dd\uac01\"\uc774\ub2e4. \uc601\uc6d0\ud55c \ud68c\uadc0\ub294 \uc624\uc9c1 \uc21c\uac04\uc73c\ub85c\ubd80\ud130\ub9cc \uc774\ud574\ub420 \uc218 \uc788\ub2e4. \"\ubaa9\ud45c \uc5c6\ub294 \uc2dc\uac04\"\uc774\ub77c\ub294 \ubb34\uc2dc\ubb34\uc2dc\ud55c \uc21c\uac04 \uc18d\uc5d0\uc11c \uc778\uac04\uc740 \uc790\uc2e0\uc758 \uc0b6\uc758 \ubcf8\ub798\uc758 \uc784\ubb34\ub97c \uacbd\ud5d8\ud55c\ub2e4. \uc601\uc6d0\ud55c \ud68c\uadc0\uc758 \uc2e0\uc801\uc778 \uc21c\uac04\uc740 \ubbf8\ud559\uc801, \uc989 \ube44\uadf9\uc801-\ub514\uc624\ub2c8\uc18c\uc2a4\uc801 \uc0c1\ud0dc\uc758 \uc21c\uac04\uc774\ub2e4. \uc774 \uc0c1\ud0dc\ub294 \ud604\uc874\uc7ac\uc5d0 \ub300\ud55c \uae0d\uc815(\uc544\ubaa8\ub974 \ud30c\ud2f0)\uc774 \uc774\ub8e8\uc5b4\uc9c0\ub294 \ucd5c\uc0c1\uc758 \uc0c1\ud0dc\uc774\uace0, \uadf8 \uc0c1\ud0dc \uc18d\uc5d0\uc11c \ud5c8\ubb34\uc8fc\uc758, \uc778\uac04\uacfc \ubaa8\ub4e0 \ud604\uc874\uc7ac\uc758 \uacfc\uc789\uc774 \uadf9\ubcf5\ub41c\ub2e4. \ube44\uadf9\uc801-\ub514\uc624\ub2c8\uc18c\uc2a4\uc801 \uc0c1\ud0dc\ub294 \uac00\uc7a5 \uc791\uace0 \ubcf4\uc798\uac83\uc5c6\ub294 \uac83\uc758 \ud68c\uadc0\uc77c\uc9c0\ub77c\ub3c4 \uc601\uc6d0 \ud68c\uadc0\uc5d0 \ub300\ud55c \uc0dd\uac01\uc774 \ubc1b\uc544\ub4e4\uc5ec\uc9c0\ub294 \uc0c1\ud0dc\uc774\ub2e4. \ucd5c\uc0c1\uc758 \uc138\uacc4 \uc644\uc131\uc758 \uc21c\uac04, \ub2c8\uccb4\uac00 \uc5d0\uba38\uc2a8\uc758 \ub9d0\uc744 \ube4c\uc5b4\uc11c \ub9d0\ud558\ub294 \uac83\ucc98\ub7fc, \uc138\uc0c1\uc774 \uc644\ubcbd\ud558\uac8c \ub418\ub294 \"\uac11\uc791\uc2a4\ub7f0 \uc601\uc6d0\"\uc758 \uc2e0\uc801 \uc21c\uac04, \uc6c3\ub294 \uc2e0 \ub514\uc624\ub2c8\uc18c\uc2a4\uac00 \ucda4\uc744 \ucd94\uba74\uc11c \uc6b0\ub9ac \ubab8\uc744 \uad00\ud1b5\ud574 \uc9c0\ub098\uac00\ub294 \ub514\uc624\ub2c8\uc18c\uc2a4\uc801 \uc21c\uac04\uc740 \uc138\uc0c1\uc744 \ubbf8\ud559\uc801 \ud604\uc0c1\uc73c\ub85c \uc601\uc6d0\ud788 \uc815\ub2f9\ud654\ud558\ub294 \uc21c\uac04\uc774\ub2e4. \uc774\ub7f0 \uacbd\ud5d8, \"\uc644\ubcbd\ud55c \uc815\uc624\"\uc758 \uac00\uc7a5 \uace0\uc694\ud55c \uc2dc\uac04\uc5d0 \uc77c\uc5b4\ub098\ub294 \ubc1d\uc740 \ub300\ub0ae\uc758 \"\uc2e0\ube44\uc2a4\ub7ec\uc6b4 \uc9c1\uad00\"\uc73c\ub85c \ub2c8\uccb4\ub294 \ucc28\ub77c\ud22c\uc2a4\ud2b8\ub77c\uc758 \uae30\ubcf8 \uc0dd\uac01\uc774\uae30\ub3c4 \ud55c \uc790\uc2e0\uc758 \ucca0\ud559\ud558\uae30\uc758 \uc6d0\ub798 \ubaa9\uc801\uacfc \uae30\ubcf8 \uacbd\ud5d8\uc744 \ubd84\uba85\ud558\uac8c \ub9d0\ud55c\ub2e4. \ubbf8(\u7f8e) \uc18d\uc5d0\uc11c \ubaa8\uc21c\ub4e4\uc740 \uc5b5\uc81c\ub41c\ub2e4. \uadf8\uac83\uc740 \uc11c\ub85c \ubaa8\uc21c \ub418\ub294 \uac83\uc744 \uc9c0\ubc30\ud558\ub294 \ud798\uc758 \ucd5c\uace0 \ud45c\uc2dd\uc774\ub2e4. \uac11\uc791\uc2a4\ub7f0 \uc601\uc6d0 \ud68c\uadc0\uc758 \uc21c\uac04\uc778 \ubbf8\uc758 \ub514\uc624\ub2c8\uc18c\uc2a4\uc801 \uc21c\uac04 \uc18d\uc5d0\uc11c \uc9c0\ub098\uac04 \uac83\uacfc \ubbf8\ub798\uc758 \uac83\uc758 \ucda9\ub3d9\uacfc \uacf5\uc874\uc758 \uacbd\ud5d8\uc774 \ud604\uc7ac\uc758 \uc21c\uac04\uc5d0\uc11c \uc774\ub8e8\uc5b4\uc9c4\ub2e4. \uc601\uc6d0\uc774 \uc2dc\uac04\uc774 \ub418\uace0, \uc2dc\uac04\uc774 \uc601\uc6d0\uc774 \ub41c\ub2e4. \uc774 \uc21c\uac04\uc5d0 \uc2dc\uac04\uc740 \uac11\uc791\uc2a4\ub7f0 \uc601\uc6d0\uc758 \uc815\uc624-\uc21c\uac04\uc73c\ub85c \uc9c0\uc591\ub41c\ub2e4. \ucc28\ub77c\ud22c\uc2a4\ud2b8\ub77c\ub294 \uc704\ubc84\uba58\uc26c\uc758 \uc9c0\uc704\uc5d0\uc11c\ubd80\ud130 \ud68c\uadc0\ub97c \uac00\ub974\uce5c\ub2e4. \uc704\ubc84\uba58\uc26c\ub294 \"\uc704\ub300\ud55c \ud68c\uadc0\uc758 \ub144\ub3c4\"\uc5d0 \uc138\uc0c1\uc758 \uc2ec\uc5f0\uc744 \ubcf4\uace0 \uacac\ub514\uc5b4 \ub0b8 \uc21c\uac04\uc5d0 \uc790\uc2e0\uc744 \ub118\uc5b4\uc11c\uace0, \ube44\uadf9\uc801-\ub514\uc624\ub2c8\uc18c\uc2a4\uc801 \uc0c1\ud0dc\uc5d0\uc11c \uc790\uc544\ub97c \uc78a\uc5b4\ubc84\ub9ac\ub294 \ucd08\uac1c\uc778\uc801\uc774\uace0 \ucc3d\uc870\uc801\uc778 \uc778\uac04\uc774\ub2e4. \uc704\ubc84\uba58\uc26c\ub294 \"\uc601\uc6d0\uc758 \uc0d8\ubb3c\" \uc18d\uc73c\ub85c \uc21c\uac04\uc758 \ubc88\uac2f\ubd88\uc744 \ub358\uc9c0\ub294 \uc0ac\ub78c\uc774\ub2e4. \uc704\ubc84\uba58\uc26c\ub294 \uadf8\ub9bc\uc790\uac00 \uac00\uc7a5 \uc9e7\uc544\uc9c0\ub294 \uc21c\uac04\uc5d0 \uc601\uc6d0 \ud68c\uadc0\uc758 \"\uc815\uc624\uc758 \uc2ec\uc5f0\" \uc18d\uc73c\ub85c \ub5a8\uc5b4\uc9c0\ub294 \ubc88\uac2f\ubd88\uc774\ub2e4. \uc601\uc6d0 \ud68c\uadc0\uc758 \uc21c\uac04\uc740 \uc544\ud3f4\ub85c\uc801 \ub514\uc624\ub2c8\uc18c\uc2a4, \uc989 \ube44\ub3c4\ub355\uc801\uc778 \uc608\uc220\uac00 \uc2e0 \ub514\uc624\ub2c8\uc18c\uc2a4 \uc790\uadf8\ub808\uc6b0\uc2a4\uac00 \ubbf8\ud559\uc801\uc73c\ub85c \ud604\ud604\ud558\ub294 \uc21c\uac04\uc774\ub2e4. \ub2c8\uccb4\uc758 \uc608\uc220\uac00 \ubcf5\uc74c\uc744 \ub530\ub974\uc790\uba74 \uc885\uad50\uac00 \uc544\ub2c8\ub77c, \uc608\uc220\uc774 \uc0b6\uc758 \" \ud615\uc774\uc0c1\ud559\uc801 \ud65c\ub3d9 \"\uc774\ub2e4. \uc608\uc220, \uac00\uc7a5 \"\ucee4\ub2e4\ub780 \uc0b6\uc758 \uc790\uadf9\"\uc774 \uc601\uc6d0 \ud68c\uadc0 \ucca0\ud559\uc758 \uae30\uad00\uc774\ub2e4. \ub2c8\uccb4\uc758 \uc601\uc6d0 \ud68c\uadc0\uc758 \uae30\ubcf8 \uc774\ub860\uc740 \uc608\uc220\uc758 \ud615\uc774\uc0c1\ud559\uc774\uace0, \ubc18(\u53cd)\uadf8\ub9ac\uc2a4\ub3c4\uc801 \uc608\uc220-\uc885\uad50\uc774\uace0, \"\uc608\uc220\uac00-\ud615\uc774\uc0c1\ud559\"\uc774\ub2e4.", "sentence_answer": "\ub2c8\uccb4\uc758 \uc608\uc220\uac00 \ubcf5\uc74c\uc744 \ub530\ub974\uc790\uba74 \uc885\uad50\uac00 \uc544\ub2c8\ub77c, \uc608\uc220\uc774 \uc0b6\uc758 \" \ud615\uc774\uc0c1\ud559\uc801 \ud65c\ub3d9 \"\uc774\ub2e4.", "paragraph_id": "6568868-8-2", "paragraph_question": "question: \ub2c8\uccb4\uc758 \uc608\uc220\uac00 \ubcf5\uc74c\uc5d0 \ub530\ub974\uba74 \uc885\uad50\uac00 \uc544\ub2cc \uc608\uc220\uc774 \uc0b6\uc758 \ubb34\uc5c7\uc778\uac00?, context: \ubbf8\ud559\uc801 \ud604\ud604\uc73c\ub85c\uc11c\uc758 \uc601\uc6d0 \ud68c\uadc0. \"\uc601\uc6d0 \ud68c\uadc0\"\ub77c\ub294 \uc0dd\uac01\uc740 \ucc28\ub77c\ud22c\uc2a4\ud2b8\ub77c\uc758 \uae30\ubcf8 \uc0dd\uac01\uc774\uace0 \ub2c8\uccb4\uc758 \"\uac00\uc7a5 \uae4a\uc219\ud55c \uacf3\uc5d0 \uc788\ub294 \uc0dd\uac01\"\uc774\ub2e4. \uc601\uc6d0\ud55c \ud68c\uadc0\ub294 \uc624\uc9c1 \uc21c\uac04\uc73c\ub85c\ubd80\ud130\ub9cc \uc774\ud574\ub420 \uc218 \uc788\ub2e4. \"\ubaa9\ud45c \uc5c6\ub294 \uc2dc\uac04\"\uc774\ub77c\ub294 \ubb34\uc2dc\ubb34\uc2dc\ud55c \uc21c\uac04 \uc18d\uc5d0\uc11c \uc778\uac04\uc740 \uc790\uc2e0\uc758 \uc0b6\uc758 \ubcf8\ub798\uc758 \uc784\ubb34\ub97c \uacbd\ud5d8\ud55c\ub2e4. \uc601\uc6d0\ud55c \ud68c\uadc0\uc758 \uc2e0\uc801\uc778 \uc21c\uac04\uc740 \ubbf8\ud559\uc801, \uc989 \ube44\uadf9\uc801-\ub514\uc624\ub2c8\uc18c\uc2a4\uc801 \uc0c1\ud0dc\uc758 \uc21c\uac04\uc774\ub2e4. \uc774 \uc0c1\ud0dc\ub294 \ud604\uc874\uc7ac\uc5d0 \ub300\ud55c \uae0d\uc815(\uc544\ubaa8\ub974 \ud30c\ud2f0)\uc774 \uc774\ub8e8\uc5b4\uc9c0\ub294 \ucd5c\uc0c1\uc758 \uc0c1\ud0dc\uc774\uace0, \uadf8 \uc0c1\ud0dc \uc18d\uc5d0\uc11c \ud5c8\ubb34\uc8fc\uc758, \uc778\uac04\uacfc \ubaa8\ub4e0 \ud604\uc874\uc7ac\uc758 \uacfc\uc789\uc774 \uadf9\ubcf5\ub41c\ub2e4. \ube44\uadf9\uc801-\ub514\uc624\ub2c8\uc18c\uc2a4\uc801 \uc0c1\ud0dc\ub294 \uac00\uc7a5 \uc791\uace0 \ubcf4\uc798\uac83\uc5c6\ub294 \uac83\uc758 \ud68c\uadc0\uc77c\uc9c0\ub77c\ub3c4 \uc601\uc6d0 \ud68c\uadc0\uc5d0 \ub300\ud55c \uc0dd\uac01\uc774 \ubc1b\uc544\ub4e4\uc5ec\uc9c0\ub294 \uc0c1\ud0dc\uc774\ub2e4. \ucd5c\uc0c1\uc758 \uc138\uacc4 \uc644\uc131\uc758 \uc21c\uac04, \ub2c8\uccb4\uac00 \uc5d0\uba38\uc2a8\uc758 \ub9d0\uc744 \ube4c\uc5b4\uc11c \ub9d0\ud558\ub294 \uac83\ucc98\ub7fc, \uc138\uc0c1\uc774 \uc644\ubcbd\ud558\uac8c \ub418\ub294 \"\uac11\uc791\uc2a4\ub7f0 \uc601\uc6d0\"\uc758 \uc2e0\uc801 \uc21c\uac04, \uc6c3\ub294 \uc2e0 \ub514\uc624\ub2c8\uc18c\uc2a4\uac00 \ucda4\uc744 \ucd94\uba74\uc11c \uc6b0\ub9ac \ubab8\uc744 \uad00\ud1b5\ud574 \uc9c0\ub098\uac00\ub294 \ub514\uc624\ub2c8\uc18c\uc2a4\uc801 \uc21c\uac04\uc740 \uc138\uc0c1\uc744 \ubbf8\ud559\uc801 \ud604\uc0c1\uc73c\ub85c \uc601\uc6d0\ud788 \uc815\ub2f9\ud654\ud558\ub294 \uc21c\uac04\uc774\ub2e4. \uc774\ub7f0 \uacbd\ud5d8, \"\uc644\ubcbd\ud55c \uc815\uc624\"\uc758 \uac00\uc7a5 \uace0\uc694\ud55c \uc2dc\uac04\uc5d0 \uc77c\uc5b4\ub098\ub294 \ubc1d\uc740 \ub300\ub0ae\uc758 \"\uc2e0\ube44\uc2a4\ub7ec\uc6b4 \uc9c1\uad00\"\uc73c\ub85c \ub2c8\uccb4\ub294 \ucc28\ub77c\ud22c\uc2a4\ud2b8\ub77c\uc758 \uae30\ubcf8 \uc0dd\uac01\uc774\uae30\ub3c4 \ud55c \uc790\uc2e0\uc758 \ucca0\ud559\ud558\uae30\uc758 \uc6d0\ub798 \ubaa9\uc801\uacfc \uae30\ubcf8 \uacbd\ud5d8\uc744 \ubd84\uba85\ud558\uac8c \ub9d0\ud55c\ub2e4. \ubbf8(\u7f8e) \uc18d\uc5d0\uc11c \ubaa8\uc21c\ub4e4\uc740 \uc5b5\uc81c\ub41c\ub2e4. \uadf8\uac83\uc740 \uc11c\ub85c \ubaa8\uc21c \ub418\ub294 \uac83\uc744 \uc9c0\ubc30\ud558\ub294 \ud798\uc758 \ucd5c\uace0 \ud45c\uc2dd\uc774\ub2e4. \uac11\uc791\uc2a4\ub7f0 \uc601\uc6d0 \ud68c\uadc0\uc758 \uc21c\uac04\uc778 \ubbf8\uc758 \ub514\uc624\ub2c8\uc18c\uc2a4\uc801 \uc21c\uac04 \uc18d\uc5d0\uc11c \uc9c0\ub098\uac04 \uac83\uacfc \ubbf8\ub798\uc758 \uac83\uc758 \ucda9\ub3d9\uacfc \uacf5\uc874\uc758 \uacbd\ud5d8\uc774 \ud604\uc7ac\uc758 \uc21c\uac04\uc5d0\uc11c \uc774\ub8e8\uc5b4\uc9c4\ub2e4. \uc601\uc6d0\uc774 \uc2dc\uac04\uc774 \ub418\uace0, \uc2dc\uac04\uc774 \uc601\uc6d0\uc774 \ub41c\ub2e4. \uc774 \uc21c\uac04\uc5d0 \uc2dc\uac04\uc740 \uac11\uc791\uc2a4\ub7f0 \uc601\uc6d0\uc758 \uc815\uc624-\uc21c\uac04\uc73c\ub85c \uc9c0\uc591\ub41c\ub2e4. \ucc28\ub77c\ud22c\uc2a4\ud2b8\ub77c\ub294 \uc704\ubc84\uba58\uc26c\uc758 \uc9c0\uc704\uc5d0\uc11c\ubd80\ud130 \ud68c\uadc0\ub97c \uac00\ub974\uce5c\ub2e4. \uc704\ubc84\uba58\uc26c\ub294 \"\uc704\ub300\ud55c \ud68c\uadc0\uc758 \ub144\ub3c4\"\uc5d0 \uc138\uc0c1\uc758 \uc2ec\uc5f0\uc744 \ubcf4\uace0 \uacac\ub514\uc5b4 \ub0b8 \uc21c\uac04\uc5d0 \uc790\uc2e0\uc744 \ub118\uc5b4\uc11c\uace0, \ube44\uadf9\uc801-\ub514\uc624\ub2c8\uc18c\uc2a4\uc801 \uc0c1\ud0dc\uc5d0\uc11c \uc790\uc544\ub97c \uc78a\uc5b4\ubc84\ub9ac\ub294 \ucd08\uac1c\uc778\uc801\uc774\uace0 \ucc3d\uc870\uc801\uc778 \uc778\uac04\uc774\ub2e4. \uc704\ubc84\uba58\uc26c\ub294 \"\uc601\uc6d0\uc758 \uc0d8\ubb3c\" \uc18d\uc73c\ub85c \uc21c\uac04\uc758 \ubc88\uac2f\ubd88\uc744 \ub358\uc9c0\ub294 \uc0ac\ub78c\uc774\ub2e4. \uc704\ubc84\uba58\uc26c\ub294 \uadf8\ub9bc\uc790\uac00 \uac00\uc7a5 \uc9e7\uc544\uc9c0\ub294 \uc21c\uac04\uc5d0 \uc601\uc6d0 \ud68c\uadc0\uc758 \"\uc815\uc624\uc758 \uc2ec\uc5f0\" \uc18d\uc73c\ub85c \ub5a8\uc5b4\uc9c0\ub294 \ubc88\uac2f\ubd88\uc774\ub2e4. \uc601\uc6d0 \ud68c\uadc0\uc758 \uc21c\uac04\uc740 \uc544\ud3f4\ub85c\uc801 \ub514\uc624\ub2c8\uc18c\uc2a4, \uc989 \ube44\ub3c4\ub355\uc801\uc778 \uc608\uc220\uac00 \uc2e0 \ub514\uc624\ub2c8\uc18c\uc2a4 \uc790\uadf8\ub808\uc6b0\uc2a4\uac00 \ubbf8\ud559\uc801\uc73c\ub85c \ud604\ud604\ud558\ub294 \uc21c\uac04\uc774\ub2e4. \ub2c8\uccb4\uc758 \uc608\uc220\uac00 \ubcf5\uc74c\uc744 \ub530\ub974\uc790\uba74 \uc885\uad50\uac00 \uc544\ub2c8\ub77c, \uc608\uc220\uc774 \uc0b6\uc758 \"\ud615\uc774\uc0c1\ud559\uc801 \ud65c\ub3d9\"\uc774\ub2e4. \uc608\uc220, \uac00\uc7a5 \"\ucee4\ub2e4\ub780 \uc0b6\uc758 \uc790\uadf9\"\uc774 \uc601\uc6d0 \ud68c\uadc0 \ucca0\ud559\uc758 \uae30\uad00\uc774\ub2e4. \ub2c8\uccb4\uc758 \uc601\uc6d0 \ud68c\uadc0\uc758 \uae30\ubcf8 \uc774\ub860\uc740 \uc608\uc220\uc758 \ud615\uc774\uc0c1\ud559\uc774\uace0, \ubc18(\u53cd)\uadf8\ub9ac\uc2a4\ub3c4\uc801 \uc608\uc220-\uc885\uad50\uc774\uace0, \"\uc608\uc220\uac00-\ud615\uc774\uc0c1\ud559\"\uc774\ub2e4."} {"question": "6\uc6d4 12\uc77c, \ubca0\uc77c\uc774 \uc790\uc2e0\uc758 50\ubc88\uc9f8 \uad6d\uac00\ub300\ud45c\ud300 \uacbd\uae30\uc5d0\uc11c, \uacb0\uc2b9\uace8\uc744 \ub123\uc5c8\uc744\ub54c\uc758 \uc0c1\ub300 \ud300\uc740?", "paragraph": "2014\ub144 9\uc6d4 9\uc77c, \ubca0\uc77c\uc740 \uc548\ub3c4\ub77c\uc640\uc758 UEFA \uc720\ub85c 2016 \uc608\uc120\uc804\uc5d0\uc11c, \ud55c \uace8\uc740 \uba38\ub9ac\ub85c, \ud55c \uace8\uc740 \ud504\ub9ac\ud0a5\uc73c\ub85c \ub123\uc5c8\ub294\ub370, \uc6e8\uc77c\uc2a4\ub294 \uadf8\uc758 \ub450 \uace8\uc5d0 \ud798\uc785\uc5b4 2-1 \uc5ed\uc804\uc2b9\uc744 \uac70\ub450\uc5c8\ub2e4. \uc774 \uacbd\uae30\uc5d0\uc11c \ub450 \uace8\uc744 \ucd94\uac00\ud574 \ud1b5\uc0b0 14\uace8\uc744 \ub4dd\uc810\ud55c \ubca0\uc77c\uc740 \uc874 \ud558\ud2b8\uc2a8\uacfc \ud568\uaed8 \uc6e8\uc77c\uc2a4 \uad6d\uac00\ub300\ud45c\ud300 \ucd5c\ub2e4 \ub4dd\uc810 \uacf5\ub3d9 10\uc704\uc5d0 \uc62c\ub790\ub2e4. 2015\ub144 3\uc6d4 28\uc77c, \ubca0\uc77c\uc740 3-0\uc73c\ub85c \uc774\uae34 \uc774\uc2a4\ub77c\uc5d8 \uc6d0\uc815 \uacbd\uae30\uc5d0\uc11c\ub3c4 \ub450 \uace8\uc744 \ub123\uc5c8\ub2e4. 6\uc6d4 12\uc77c, \uc790\uc2e0\uc758 50\ubc88\uc9f8 \uad6d\uac00\ub300\ud45c\ud300 \uacbd\uae30\uc5d0\uc11c, \uadf8\ub294 \ubca8\uae30\uc5d0\uc640\uc758 \uc608\uc120 \uc548\ubc29 \uacbd\uae30\uc5d0\uc11c \uacb0\uc2b9\uace8\uc744 \ub123\uc5c8\uace0, \uc6e8\uc77c\uc2a4\ub294 \ubca8\uae30\uc5d0\ub97c \uc81c\uce58\uace0 \uc608\uc120 B\uc870 \uc120\ub450\uc5d0 \uc62c\ub790\ub2e4. 9\uc6d4 3\uc77c, \uadf8\ub294 \ud0a4\ud504\ub85c\uc2a4\uc640\uc758 \uc6d0\uc815 \uacbd\uae30\uc5d0\uc11c \uc7ac\uc988 \ub9ac\ucc98\uc988\uc758 \ud06c\ub85c\uc2a4\ub97c \uba38\ub9ac\ub85c \uc5f0\uacb0\ud574 1-0 \uacb0\uc2b9\uace8\uc744 \ub123\uc5b4 \ubcf8\uc120 \uc9c4\ucd9c\uae4c\uc9c0 3\uc810\ub9cc\uc744 \ub0a8\uaca8\ub450\uc5c8\ub2e4. UEFA \uc720\ub85c 2016\uc758 \ubcf8\uc120 \uc9c4\ucd9c (1958\ub144 FIFA \uc6d4\ub4dc\ucef5 \uc774\ub798 \uc6e8\uc77c\uc2a4\uc758 \uccab \uc8fc\uc694 \ub300\ud68c \ubcf8\uc120 \uc9c4\ucd9c) \uc744 \uc774\ub8e9\ud55c \ud6c4\uc778 2015\ub144 10\uc6d4 13\uc77c, \ubca0\uc77c\uc740 \uc548\ub3c4\ub77c\uc640\uc758 \uce74\ub514\ud504 \uc2dc \uacbd\uae30\uc7a5\uc5d0\uc11c \uc5f4\ub9b0 \uc548\ubc29 \uacbd\uae30\uc5d0\uc11c \ub300\ud68c \uc608\uc120 7\ud638\uace8\uc744 \uae30\ub85d\ud588\ub2e4.", "answer": "\ubca8\uae30\uc5d0", "sentence": "\uadf8\ub294 \ubca8\uae30\uc5d0 \uc640\uc758 \uc608\uc120 \uc548\ubc29 \uacbd\uae30\uc5d0\uc11c \uacb0\uc2b9\uace8\uc744 \ub123\uc5c8\uace0, \uc6e8\uc77c\uc2a4\ub294 \ubca8\uae30\uc5d0\ub97c \uc81c\uce58\uace0 \uc608\uc120 B\uc870 \uc120\ub450\uc5d0 \uc62c\ub790\ub2e4.", "paragraph_sentence": "2014\ub144 9\uc6d4 9\uc77c, \ubca0\uc77c\uc740 \uc548\ub3c4\ub77c\uc640\uc758 UEFA \uc720\ub85c 2016 \uc608\uc120\uc804\uc5d0\uc11c, \ud55c \uace8\uc740 \uba38\ub9ac\ub85c, \ud55c \uace8\uc740 \ud504\ub9ac\ud0a5\uc73c\ub85c \ub123\uc5c8\ub294\ub370, \uc6e8\uc77c\uc2a4\ub294 \uadf8\uc758 \ub450 \uace8\uc5d0 \ud798\uc785\uc5b4 2-1 \uc5ed\uc804\uc2b9\uc744 \uac70\ub450\uc5c8\ub2e4. \uc774 \uacbd\uae30\uc5d0\uc11c \ub450 \uace8\uc744 \ucd94\uac00\ud574 \ud1b5\uc0b0 14\uace8\uc744 \ub4dd\uc810\ud55c \ubca0\uc77c\uc740 \uc874 \ud558\ud2b8\uc2a8\uacfc \ud568\uaed8 \uc6e8\uc77c\uc2a4 \uad6d\uac00\ub300\ud45c\ud300 \ucd5c\ub2e4 \ub4dd\uc810 \uacf5\ub3d9 10\uc704\uc5d0 \uc62c\ub790\ub2e4. 2015\ub144 3\uc6d4 28\uc77c, \ubca0\uc77c\uc740 3-0\uc73c\ub85c \uc774\uae34 \uc774\uc2a4\ub77c\uc5d8 \uc6d0\uc815 \uacbd\uae30\uc5d0\uc11c\ub3c4 \ub450 \uace8\uc744 \ub123\uc5c8\ub2e4. 6\uc6d4 12\uc77c, \uc790\uc2e0\uc758 50\ubc88\uc9f8 \uad6d\uac00\ub300\ud45c\ud300 \uacbd\uae30\uc5d0\uc11c, \uadf8\ub294 \ubca8\uae30\uc5d0 \uc640\uc758 \uc608\uc120 \uc548\ubc29 \uacbd\uae30\uc5d0\uc11c \uacb0\uc2b9\uace8\uc744 \ub123\uc5c8\uace0, \uc6e8\uc77c\uc2a4\ub294 \ubca8\uae30\uc5d0\ub97c \uc81c\uce58\uace0 \uc608\uc120 B\uc870 \uc120\ub450\uc5d0 \uc62c\ub790\ub2e4. 9\uc6d4 3\uc77c, \uadf8\ub294 \ud0a4\ud504\ub85c\uc2a4\uc640\uc758 \uc6d0\uc815 \uacbd\uae30\uc5d0\uc11c \uc7ac\uc988 \ub9ac\ucc98\uc988\uc758 \ud06c\ub85c\uc2a4\ub97c \uba38\ub9ac\ub85c \uc5f0\uacb0\ud574 1-0 \uacb0\uc2b9\uace8\uc744 \ub123\uc5b4 \ubcf8\uc120 \uc9c4\ucd9c\uae4c\uc9c0 3\uc810\ub9cc\uc744 \ub0a8\uaca8\ub450\uc5c8\ub2e4. UEFA \uc720\ub85c 2016\uc758 \ubcf8\uc120 \uc9c4\ucd9c (1958\ub144 FIFA \uc6d4\ub4dc\ucef5 \uc774\ub798 \uc6e8\uc77c\uc2a4\uc758 \uccab \uc8fc\uc694 \ub300\ud68c \ubcf8\uc120 \uc9c4\ucd9c) \uc744 \uc774\ub8e9\ud55c \ud6c4\uc778 2015\ub144 10\uc6d4 13\uc77c, \ubca0\uc77c\uc740 \uc548\ub3c4\ub77c\uc640\uc758 \uce74\ub514\ud504 \uc2dc \uacbd\uae30\uc7a5\uc5d0\uc11c \uc5f4\ub9b0 \uc548\ubc29 \uacbd\uae30\uc5d0\uc11c \ub300\ud68c \uc608\uc120 7\ud638\uace8\uc744 \uae30\ub85d\ud588\ub2e4.", "paragraph_answer": "2014\ub144 9\uc6d4 9\uc77c, \ubca0\uc77c\uc740 \uc548\ub3c4\ub77c\uc640\uc758 UEFA \uc720\ub85c 2016 \uc608\uc120\uc804\uc5d0\uc11c, \ud55c \uace8\uc740 \uba38\ub9ac\ub85c, \ud55c \uace8\uc740 \ud504\ub9ac\ud0a5\uc73c\ub85c \ub123\uc5c8\ub294\ub370, \uc6e8\uc77c\uc2a4\ub294 \uadf8\uc758 \ub450 \uace8\uc5d0 \ud798\uc785\uc5b4 2-1 \uc5ed\uc804\uc2b9\uc744 \uac70\ub450\uc5c8\ub2e4. \uc774 \uacbd\uae30\uc5d0\uc11c \ub450 \uace8\uc744 \ucd94\uac00\ud574 \ud1b5\uc0b0 14\uace8\uc744 \ub4dd\uc810\ud55c \ubca0\uc77c\uc740 \uc874 \ud558\ud2b8\uc2a8\uacfc \ud568\uaed8 \uc6e8\uc77c\uc2a4 \uad6d\uac00\ub300\ud45c\ud300 \ucd5c\ub2e4 \ub4dd\uc810 \uacf5\ub3d9 10\uc704\uc5d0 \uc62c\ub790\ub2e4. 2015\ub144 3\uc6d4 28\uc77c, \ubca0\uc77c\uc740 3-0\uc73c\ub85c \uc774\uae34 \uc774\uc2a4\ub77c\uc5d8 \uc6d0\uc815 \uacbd\uae30\uc5d0\uc11c\ub3c4 \ub450 \uace8\uc744 \ub123\uc5c8\ub2e4. 6\uc6d4 12\uc77c, \uc790\uc2e0\uc758 50\ubc88\uc9f8 \uad6d\uac00\ub300\ud45c\ud300 \uacbd\uae30\uc5d0\uc11c, \uadf8\ub294 \ubca8\uae30\uc5d0 \uc640\uc758 \uc608\uc120 \uc548\ubc29 \uacbd\uae30\uc5d0\uc11c \uacb0\uc2b9\uace8\uc744 \ub123\uc5c8\uace0, \uc6e8\uc77c\uc2a4\ub294 \ubca8\uae30\uc5d0\ub97c \uc81c\uce58\uace0 \uc608\uc120 B\uc870 \uc120\ub450\uc5d0 \uc62c\ub790\ub2e4. 9\uc6d4 3\uc77c, \uadf8\ub294 \ud0a4\ud504\ub85c\uc2a4\uc640\uc758 \uc6d0\uc815 \uacbd\uae30\uc5d0\uc11c \uc7ac\uc988 \ub9ac\ucc98\uc988\uc758 \ud06c\ub85c\uc2a4\ub97c \uba38\ub9ac\ub85c \uc5f0\uacb0\ud574 1-0 \uacb0\uc2b9\uace8\uc744 \ub123\uc5b4 \ubcf8\uc120 \uc9c4\ucd9c\uae4c\uc9c0 3\uc810\ub9cc\uc744 \ub0a8\uaca8\ub450\uc5c8\ub2e4. UEFA \uc720\ub85c 2016\uc758 \ubcf8\uc120 \uc9c4\ucd9c (1958\ub144 FIFA \uc6d4\ub4dc\ucef5 \uc774\ub798 \uc6e8\uc77c\uc2a4\uc758 \uccab \uc8fc\uc694 \ub300\ud68c \ubcf8\uc120 \uc9c4\ucd9c) \uc744 \uc774\ub8e9\ud55c \ud6c4\uc778 2015\ub144 10\uc6d4 13\uc77c, \ubca0\uc77c\uc740 \uc548\ub3c4\ub77c\uc640\uc758 \uce74\ub514\ud504 \uc2dc \uacbd\uae30\uc7a5\uc5d0\uc11c \uc5f4\ub9b0 \uc548\ubc29 \uacbd\uae30\uc5d0\uc11c \ub300\ud68c \uc608\uc120 7\ud638\uace8\uc744 \uae30\ub85d\ud588\ub2e4.", "sentence_answer": "\uadf8\ub294 \ubca8\uae30\uc5d0 \uc640\uc758 \uc608\uc120 \uc548\ubc29 \uacbd\uae30\uc5d0\uc11c \uacb0\uc2b9\uace8\uc744 \ub123\uc5c8\uace0, \uc6e8\uc77c\uc2a4\ub294 \ubca8\uae30\uc5d0\ub97c \uc81c\uce58\uace0 \uc608\uc120 B\uc870 \uc120\ub450\uc5d0 \uc62c\ub790\ub2e4.", "paragraph_id": "5782882-14-1", "paragraph_question": "question: 6\uc6d4 12\uc77c, \ubca0\uc77c\uc774 \uc790\uc2e0\uc758 50\ubc88\uc9f8 \uad6d\uac00\ub300\ud45c\ud300 \uacbd\uae30\uc5d0\uc11c, \uacb0\uc2b9\uace8\uc744 \ub123\uc5c8\uc744\ub54c\uc758 \uc0c1\ub300 \ud300\uc740?, context: 2014\ub144 9\uc6d4 9\uc77c, \ubca0\uc77c\uc740 \uc548\ub3c4\ub77c\uc640\uc758 UEFA \uc720\ub85c 2016 \uc608\uc120\uc804\uc5d0\uc11c, \ud55c \uace8\uc740 \uba38\ub9ac\ub85c, \ud55c \uace8\uc740 \ud504\ub9ac\ud0a5\uc73c\ub85c \ub123\uc5c8\ub294\ub370, \uc6e8\uc77c\uc2a4\ub294 \uadf8\uc758 \ub450 \uace8\uc5d0 \ud798\uc785\uc5b4 2-1 \uc5ed\uc804\uc2b9\uc744 \uac70\ub450\uc5c8\ub2e4. \uc774 \uacbd\uae30\uc5d0\uc11c \ub450 \uace8\uc744 \ucd94\uac00\ud574 \ud1b5\uc0b0 14\uace8\uc744 \ub4dd\uc810\ud55c \ubca0\uc77c\uc740 \uc874 \ud558\ud2b8\uc2a8\uacfc \ud568\uaed8 \uc6e8\uc77c\uc2a4 \uad6d\uac00\ub300\ud45c\ud300 \ucd5c\ub2e4 \ub4dd\uc810 \uacf5\ub3d9 10\uc704\uc5d0 \uc62c\ub790\ub2e4. 2015\ub144 3\uc6d4 28\uc77c, \ubca0\uc77c\uc740 3-0\uc73c\ub85c \uc774\uae34 \uc774\uc2a4\ub77c\uc5d8 \uc6d0\uc815 \uacbd\uae30\uc5d0\uc11c\ub3c4 \ub450 \uace8\uc744 \ub123\uc5c8\ub2e4. 6\uc6d4 12\uc77c, \uc790\uc2e0\uc758 50\ubc88\uc9f8 \uad6d\uac00\ub300\ud45c\ud300 \uacbd\uae30\uc5d0\uc11c, \uadf8\ub294 \ubca8\uae30\uc5d0\uc640\uc758 \uc608\uc120 \uc548\ubc29 \uacbd\uae30\uc5d0\uc11c \uacb0\uc2b9\uace8\uc744 \ub123\uc5c8\uace0, \uc6e8\uc77c\uc2a4\ub294 \ubca8\uae30\uc5d0\ub97c \uc81c\uce58\uace0 \uc608\uc120 B\uc870 \uc120\ub450\uc5d0 \uc62c\ub790\ub2e4. 9\uc6d4 3\uc77c, \uadf8\ub294 \ud0a4\ud504\ub85c\uc2a4\uc640\uc758 \uc6d0\uc815 \uacbd\uae30\uc5d0\uc11c \uc7ac\uc988 \ub9ac\ucc98\uc988\uc758 \ud06c\ub85c\uc2a4\ub97c \uba38\ub9ac\ub85c \uc5f0\uacb0\ud574 1-0 \uacb0\uc2b9\uace8\uc744 \ub123\uc5b4 \ubcf8\uc120 \uc9c4\ucd9c\uae4c\uc9c0 3\uc810\ub9cc\uc744 \ub0a8\uaca8\ub450\uc5c8\ub2e4. UEFA \uc720\ub85c 2016\uc758 \ubcf8\uc120 \uc9c4\ucd9c (1958\ub144 FIFA \uc6d4\ub4dc\ucef5 \uc774\ub798 \uc6e8\uc77c\uc2a4\uc758 \uccab \uc8fc\uc694 \ub300\ud68c \ubcf8\uc120 \uc9c4\ucd9c) \uc744 \uc774\ub8e9\ud55c \ud6c4\uc778 2015\ub144 10\uc6d4 13\uc77c, \ubca0\uc77c\uc740 \uc548\ub3c4\ub77c\uc640\uc758 \uce74\ub514\ud504 \uc2dc \uacbd\uae30\uc7a5\uc5d0\uc11c \uc5f4\ub9b0 \uc548\ubc29 \uacbd\uae30\uc5d0\uc11c \ub300\ud68c \uc608\uc120 7\ud638\uace8\uc744 \uae30\ub85d\ud588\ub2e4."} {"question": "\ub0a8\ubd81\uc774 \uacbd\uc7c1\uc801\uc73c\ub85c \uad70\ube44\ub97c \uc99d\uac15\uc2dc\ud0a4\uae30 \uc2dc\uc791\ud55c \uac83\uc740 \uc5b8\uc81c\ubd80\ud130\uc778\uac00?", "paragraph": "\uc774\uc2b9\ub9cc\uc758 \uc81c1\uacf5\ud654\uad6d\uc758 \ubd81\uc9c4 \ud1b5\uc77c\ub860\uc774 \ub300\ub0a8 \ub3c4\ubc1c\uc744 \ucd09\uc9c4\uc2dc\ucf30\ub2e4\ub294 \uc124\uc774\ub2e4. \uc774\uc2b9\ub9cc\uc740 \uc5ec\ub7ec \ucc28\ub840 \ubb34\ub825 \ubd81\uc9c4 \ud1b5\uc77c\uc744 \ubd80\ub974\uc9d6\uc5c8\uc73c\uba70, \ubbf8\uad6d\uc758 \uad70\uc0ac \uc6d0\uc870\ub97c \uacf5\uacf5\uc5f0\ud788 \uc694\uccad\ud55c \ubc14 \uc788\ub2e4. \uc2e4\uc81c\ub85c 1950\ub144\ub300 \ucd08\ubd80\ud130 \ub0a8\ubd81\uc740 \uacbd\uc7c1\uc801\uc73c\ub85c \uad70\ube44\ub97c \uc99d\uac15\uc2dc\ud0a4\uae30 \uc2dc\uc791\ud588\uace0, \ubbf8\uad6d\uc774 \uad6d\uad70\uc744 \uac15\ud654\uc2dc\ud0a4\uba74 \uc774\uc2b9\ub9cc\uc774 \ubb34\ub825\ud1b5\uc77c\uc744 \ucd94\uad6c\ud560 \uac83\uc774\ub77c\uace0 \ud310\ub2e8, \uc870\uc120\ubbfc\uc8fc\uc8fc\uc758\uc778\ubbfc\uacf5\ud654\uad6d\uc774 \uc608\ubc29\uc801 \uc870\uce58\ub97c \ucde8\ud588\ub2e4\ub294 \uc8fc\uc7a5\uc774\ub2e4. \ud558\uc9c0\ub9cc \ud55c\ubbf8\uacf5\ubaa8\uc124\uc740 \uc704 \ud56d\ubaa9\uc5d0 \uad00\ub828\ub41c \uad6c \uc18c\ub828\uc758 \ubb38\uc11c\uacf5\uac1c\ub85c, \uc81c\uc2dc\ub41c \uc801 \uc788\ub294 \uac00\uc124\ub85c\ub9cc \ub0a8\uac8c \ub418\uc5c8\ub2e4. \uadf8\ub9ac\uace0 2010\ub144 6\uc6d4, 56\ub144\ub9cc\uc5d0 \uacf5\uac1c\ub41c CIA \uadf9\ube44 \ubb38\uc11c\uc5d0\uc11c \ud55c\uad6d \uc804\uc7c1 \ubc1c\ubc1c \ubd88\uacfc \uc5ff\uc0c8\uc804\uc778 6\uc6d4 19\uc77c, \ubd81\ud55c\uc758 \ub0a8\uce68\uac00\ub2a5\uc131\uc774 \ub0ae\uac8c \ud3c9\uac00\ud55c \ubcf4\uace0\uc11c\ub97c \uc791\uc131\ud558\uc600\uace0 \ubd81\ud55c\uc774 \uc18c\ub828\uc758 \ucca0\uc800\ud55c \uc704\uc131\uad6d\uac00\ub85c \ub3c5\uc790\uc801\uc778 \uc804\uc7c1 \uc218\ud589\ub2a5\ub825\uc774 \uc804\ud600 \uc5c6\ub2e4\uace0 \ubcf4\uace0 \uc788\uc5c8\ub2e4.", "answer": "1950\ub144\ub300 \ucd08", "sentence": "\uc2e4\uc81c\ub85c 1950\ub144\ub300 \ucd08 \ubd80\ud130 \ub0a8\ubd81\uc740 \uacbd\uc7c1\uc801\uc73c\ub85c \uad70\ube44\ub97c \uc99d\uac15\uc2dc\ud0a4\uae30 \uc2dc\uc791\ud588\uace0, \ubbf8\uad6d\uc774 \uad6d\uad70\uc744 \uac15\ud654\uc2dc\ud0a4\uba74 \uc774\uc2b9\ub9cc\uc774 \ubb34\ub825\ud1b5\uc77c\uc744 \ucd94\uad6c\ud560 \uac83\uc774\ub77c\uace0 \ud310\ub2e8, \uc870\uc120\ubbfc\uc8fc\uc8fc\uc758\uc778\ubbfc\uacf5\ud654\uad6d\uc774 \uc608\ubc29\uc801 \uc870\uce58\ub97c \ucde8\ud588\ub2e4\ub294 \uc8fc\uc7a5\uc774\ub2e4.", "paragraph_sentence": "\uc774\uc2b9\ub9cc\uc758 \uc81c1\uacf5\ud654\uad6d\uc758 \ubd81\uc9c4 \ud1b5\uc77c\ub860\uc774 \ub300\ub0a8 \ub3c4\ubc1c\uc744 \ucd09\uc9c4\uc2dc\ucf30\ub2e4\ub294 \uc124\uc774\ub2e4. \uc774\uc2b9\ub9cc\uc740 \uc5ec\ub7ec \ucc28\ub840 \ubb34\ub825 \ubd81\uc9c4 \ud1b5\uc77c\uc744 \ubd80\ub974\uc9d6\uc5c8\uc73c\uba70, \ubbf8\uad6d\uc758 \uad70\uc0ac \uc6d0\uc870\ub97c \uacf5\uacf5\uc5f0\ud788 \uc694\uccad\ud55c \ubc14 \uc788\ub2e4. \uc2e4\uc81c\ub85c 1950\ub144\ub300 \ucd08 \ubd80\ud130 \ub0a8\ubd81\uc740 \uacbd\uc7c1\uc801\uc73c\ub85c \uad70\ube44\ub97c \uc99d\uac15\uc2dc\ud0a4\uae30 \uc2dc\uc791\ud588\uace0, \ubbf8\uad6d\uc774 \uad6d\uad70\uc744 \uac15\ud654\uc2dc\ud0a4\uba74 \uc774\uc2b9\ub9cc\uc774 \ubb34\ub825\ud1b5\uc77c\uc744 \ucd94\uad6c\ud560 \uac83\uc774\ub77c\uace0 \ud310\ub2e8, \uc870\uc120\ubbfc\uc8fc\uc8fc\uc758\uc778\ubbfc\uacf5\ud654\uad6d\uc774 \uc608\ubc29\uc801 \uc870\uce58\ub97c \ucde8\ud588\ub2e4\ub294 \uc8fc\uc7a5\uc774\ub2e4. \ud558\uc9c0\ub9cc \ud55c\ubbf8\uacf5\ubaa8\uc124\uc740 \uc704 \ud56d\ubaa9\uc5d0 \uad00\ub828\ub41c \uad6c \uc18c\ub828\uc758 \ubb38\uc11c\uacf5\uac1c\ub85c, \uc81c\uc2dc\ub41c \uc801 \uc788\ub294 \uac00\uc124\ub85c\ub9cc \ub0a8\uac8c \ub418\uc5c8\ub2e4. \uadf8\ub9ac\uace0 2010\ub144 6\uc6d4, 56\ub144\ub9cc\uc5d0 \uacf5\uac1c\ub41c CIA \uadf9\ube44 \ubb38\uc11c\uc5d0\uc11c \ud55c\uad6d \uc804\uc7c1 \ubc1c\ubc1c \ubd88\uacfc \uc5ff\uc0c8\uc804\uc778 6\uc6d4 19\uc77c, \ubd81\ud55c\uc758 \ub0a8\uce68\uac00\ub2a5\uc131\uc774 \ub0ae\uac8c \ud3c9\uac00\ud55c \ubcf4\uace0\uc11c\ub97c \uc791\uc131\ud558\uc600\uace0 \ubd81\ud55c\uc774 \uc18c\ub828\uc758 \ucca0\uc800\ud55c \uc704\uc131\uad6d\uac00\ub85c \ub3c5\uc790\uc801\uc778 \uc804\uc7c1 \uc218\ud589\ub2a5\ub825\uc774 \uc804\ud600 \uc5c6\ub2e4\uace0 \ubcf4\uace0 \uc788\uc5c8\ub2e4.", "paragraph_answer": "\uc774\uc2b9\ub9cc\uc758 \uc81c1\uacf5\ud654\uad6d\uc758 \ubd81\uc9c4 \ud1b5\uc77c\ub860\uc774 \ub300\ub0a8 \ub3c4\ubc1c\uc744 \ucd09\uc9c4\uc2dc\ucf30\ub2e4\ub294 \uc124\uc774\ub2e4. \uc774\uc2b9\ub9cc\uc740 \uc5ec\ub7ec \ucc28\ub840 \ubb34\ub825 \ubd81\uc9c4 \ud1b5\uc77c\uc744 \ubd80\ub974\uc9d6\uc5c8\uc73c\uba70, \ubbf8\uad6d\uc758 \uad70\uc0ac \uc6d0\uc870\ub97c \uacf5\uacf5\uc5f0\ud788 \uc694\uccad\ud55c \ubc14 \uc788\ub2e4. \uc2e4\uc81c\ub85c 1950\ub144\ub300 \ucd08 \ubd80\ud130 \ub0a8\ubd81\uc740 \uacbd\uc7c1\uc801\uc73c\ub85c \uad70\ube44\ub97c \uc99d\uac15\uc2dc\ud0a4\uae30 \uc2dc\uc791\ud588\uace0, \ubbf8\uad6d\uc774 \uad6d\uad70\uc744 \uac15\ud654\uc2dc\ud0a4\uba74 \uc774\uc2b9\ub9cc\uc774 \ubb34\ub825\ud1b5\uc77c\uc744 \ucd94\uad6c\ud560 \uac83\uc774\ub77c\uace0 \ud310\ub2e8, \uc870\uc120\ubbfc\uc8fc\uc8fc\uc758\uc778\ubbfc\uacf5\ud654\uad6d\uc774 \uc608\ubc29\uc801 \uc870\uce58\ub97c \ucde8\ud588\ub2e4\ub294 \uc8fc\uc7a5\uc774\ub2e4. \ud558\uc9c0\ub9cc \ud55c\ubbf8\uacf5\ubaa8\uc124\uc740 \uc704 \ud56d\ubaa9\uc5d0 \uad00\ub828\ub41c \uad6c \uc18c\ub828\uc758 \ubb38\uc11c\uacf5\uac1c\ub85c, \uc81c\uc2dc\ub41c \uc801 \uc788\ub294 \uac00\uc124\ub85c\ub9cc \ub0a8\uac8c \ub418\uc5c8\ub2e4. \uadf8\ub9ac\uace0 2010\ub144 6\uc6d4, 56\ub144\ub9cc\uc5d0 \uacf5\uac1c\ub41c CIA \uadf9\ube44 \ubb38\uc11c\uc5d0\uc11c \ud55c\uad6d \uc804\uc7c1 \ubc1c\ubc1c \ubd88\uacfc \uc5ff\uc0c8\uc804\uc778 6\uc6d4 19\uc77c, \ubd81\ud55c\uc758 \ub0a8\uce68\uac00\ub2a5\uc131\uc774 \ub0ae\uac8c \ud3c9\uac00\ud55c \ubcf4\uace0\uc11c\ub97c \uc791\uc131\ud558\uc600\uace0 \ubd81\ud55c\uc774 \uc18c\ub828\uc758 \ucca0\uc800\ud55c \uc704\uc131\uad6d\uac00\ub85c \ub3c5\uc790\uc801\uc778 \uc804\uc7c1 \uc218\ud589\ub2a5\ub825\uc774 \uc804\ud600 \uc5c6\ub2e4\uace0 \ubcf4\uace0 \uc788\uc5c8\ub2e4.", "sentence_answer": "\uc2e4\uc81c\ub85c 1950\ub144\ub300 \ucd08 \ubd80\ud130 \ub0a8\ubd81\uc740 \uacbd\uc7c1\uc801\uc73c\ub85c \uad70\ube44\ub97c \uc99d\uac15\uc2dc\ud0a4\uae30 \uc2dc\uc791\ud588\uace0, \ubbf8\uad6d\uc774 \uad6d\uad70\uc744 \uac15\ud654\uc2dc\ud0a4\uba74 \uc774\uc2b9\ub9cc\uc774 \ubb34\ub825\ud1b5\uc77c\uc744 \ucd94\uad6c\ud560 \uac83\uc774\ub77c\uace0 \ud310\ub2e8, \uc870\uc120\ubbfc\uc8fc\uc8fc\uc758\uc778\ubbfc\uacf5\ud654\uad6d\uc774 \uc608\ubc29\uc801 \uc870\uce58\ub97c \ucde8\ud588\ub2e4\ub294 \uc8fc\uc7a5\uc774\ub2e4.", "paragraph_id": "6569867-23-1", "paragraph_question": "question: \ub0a8\ubd81\uc774 \uacbd\uc7c1\uc801\uc73c\ub85c \uad70\ube44\ub97c \uc99d\uac15\uc2dc\ud0a4\uae30 \uc2dc\uc791\ud55c \uac83\uc740 \uc5b8\uc81c\ubd80\ud130\uc778\uac00?, context: \uc774\uc2b9\ub9cc\uc758 \uc81c1\uacf5\ud654\uad6d\uc758 \ubd81\uc9c4 \ud1b5\uc77c\ub860\uc774 \ub300\ub0a8 \ub3c4\ubc1c\uc744 \ucd09\uc9c4\uc2dc\ucf30\ub2e4\ub294 \uc124\uc774\ub2e4. \uc774\uc2b9\ub9cc\uc740 \uc5ec\ub7ec \ucc28\ub840 \ubb34\ub825 \ubd81\uc9c4 \ud1b5\uc77c\uc744 \ubd80\ub974\uc9d6\uc5c8\uc73c\uba70, \ubbf8\uad6d\uc758 \uad70\uc0ac \uc6d0\uc870\ub97c \uacf5\uacf5\uc5f0\ud788 \uc694\uccad\ud55c \ubc14 \uc788\ub2e4. \uc2e4\uc81c\ub85c 1950\ub144\ub300 \ucd08\ubd80\ud130 \ub0a8\ubd81\uc740 \uacbd\uc7c1\uc801\uc73c\ub85c \uad70\ube44\ub97c \uc99d\uac15\uc2dc\ud0a4\uae30 \uc2dc\uc791\ud588\uace0, \ubbf8\uad6d\uc774 \uad6d\uad70\uc744 \uac15\ud654\uc2dc\ud0a4\uba74 \uc774\uc2b9\ub9cc\uc774 \ubb34\ub825\ud1b5\uc77c\uc744 \ucd94\uad6c\ud560 \uac83\uc774\ub77c\uace0 \ud310\ub2e8, \uc870\uc120\ubbfc\uc8fc\uc8fc\uc758\uc778\ubbfc\uacf5\ud654\uad6d\uc774 \uc608\ubc29\uc801 \uc870\uce58\ub97c \ucde8\ud588\ub2e4\ub294 \uc8fc\uc7a5\uc774\ub2e4. \ud558\uc9c0\ub9cc \ud55c\ubbf8\uacf5\ubaa8\uc124\uc740 \uc704 \ud56d\ubaa9\uc5d0 \uad00\ub828\ub41c \uad6c \uc18c\ub828\uc758 \ubb38\uc11c\uacf5\uac1c\ub85c, \uc81c\uc2dc\ub41c \uc801 \uc788\ub294 \uac00\uc124\ub85c\ub9cc \ub0a8\uac8c \ub418\uc5c8\ub2e4. \uadf8\ub9ac\uace0 2010\ub144 6\uc6d4, 56\ub144\ub9cc\uc5d0 \uacf5\uac1c\ub41c CIA \uadf9\ube44 \ubb38\uc11c\uc5d0\uc11c \ud55c\uad6d \uc804\uc7c1 \ubc1c\ubc1c \ubd88\uacfc \uc5ff\uc0c8\uc804\uc778 6\uc6d4 19\uc77c, \ubd81\ud55c\uc758 \ub0a8\uce68\uac00\ub2a5\uc131\uc774 \ub0ae\uac8c \ud3c9\uac00\ud55c \ubcf4\uace0\uc11c\ub97c \uc791\uc131\ud558\uc600\uace0 \ubd81\ud55c\uc774 \uc18c\ub828\uc758 \ucca0\uc800\ud55c \uc704\uc131\uad6d\uac00\ub85c \ub3c5\uc790\uc801\uc778 \uc804\uc7c1 \uc218\ud589\ub2a5\ub825\uc774 \uc804\ud600 \uc5c6\ub2e4\uace0 \ubcf4\uace0 \uc788\uc5c8\ub2e4."} {"question": "\uc0d8\ud504\ub7ec\uc2a4\uac00 \uc0dd\uc560 \uccab \uc138\uacc4 \ub7ad\ud0b9 1\uc704\ub97c \ud55c \ud574\ub294?", "paragraph": "1993\ub144 \ucd08 \ud638\uc8fc \uc624\ud508\uc5d0\uc11c 4\uac15\uc5d0 \uc62c\ub790\uc73c\uba70 \ud504\ub791\uc2a4 \uc624\ud508\uc5d0\uc11c\ub294 \uc774\uc804 \ud574\uc5d0 \uc774\uc5b4 \ub2e4\uc2dc \ud55c \ubc88 8\uac15\uc5d0 \uc62c\ub790\ub2e4. \uadf8\ub9ac\uace0 1993\ub144 4\uc6d4 \uc0dd\uc560 \ucd5c\ucd08\ub85c \uc138\uacc4 \ub7ad\ud0b9 1\uc704\uc5d0 \uc62c\ub790\ub2e4. \ub2f9\uc2dc \uc0d8\ud504\ub7ec\uc2a4\ub294 1990\ub144 US \uc624\ud508 \uc6b0\uc2b9 \uc774\ud6c4 \uadf8\ub79c\ub4dc \uc2ac\ub7a8 \ud0c0\uc774\ud2c0\uc774 \uc5c6\ub294 \uc0c1\ud0dc\uc600\uae30 \ub54c\ubb38\uc5d0 \uadf8\uac00 \uc9c4\uc815\ud55c \uc138\uacc4 \ub7ad\ud0b9 1\uc704\ub77c\uace0 \ud560 \uc218 \uc788\ub294\uc9c0\uc5d0 \ub300\ud574 \ub2e4\uc18c \ub17c\ub780\uc774 \uc788\uc5c8\ub2e4. \uadf8\ub7ec\ub098 \uadf8\ub294 \uc138 \ub2ec \ud6c4 \uc714\ube14\ub358 \uacb0\uc2b9\uc5d0\uc11c \uc804 \uc138\uacc4 \ub7ad\ud0b9 1\uc704\uc600\ub358 \uc9d0 \ucfe0\ub9ac\uc5b4\ub97c \uaebe\uace0 \uc6b0\uc2b9\ud558\uba74\uc11c \uadf8\ub7ec\ud55c \ub17c\ub780\uc744 \uc7a0\uc7ac\uc6e0\ub2e4. \uadf8 \uc5ec\uc138\ub97c \ubab0\uc544 \uac19\uc740 \ud574 US \uc624\ud508\uc5d0\uc11c\ub3c4 \uc6b0\uc2b9\ud558\uba74\uc11c \uc0dd\uc560 \ub450 \ubc88\uc9f8 US \uc624\ud508 \ud0c0\uc774\ud2c0\uc744 \ud68d\ub4dd\ud588\ub2e4. \uadf8\ub294 \uc774 \ud574 \uc5f0\ub9d0 ATP \ud22c\uc5b4 \uc0ac\uc0c1 \ucd5c\ucd08\ub85c \ud55c \uc2dc\uc98c \ucc9c \uac1c \uc774\uc0c1\uc758 \uc5d0\uc774\uc2a4\ub97c \uae30\ub85d\ud558\uba74\uc11c \ubd80\ub3d9\uc758 1\uc704\ub85c \uc2dc\uc98c\uc744 \ub9c8\uac10\ud588\ub2e4.", "answer": "1993\ub144", "sentence": "1993\ub144 \ucd08 \ud638\uc8fc \uc624\ud508\uc5d0\uc11c 4\uac15\uc5d0 \uc62c\ub790\uc73c\uba70 \ud504\ub791\uc2a4 \uc624\ud508\uc5d0\uc11c\ub294 \uc774\uc804 \ud574\uc5d0 \uc774\uc5b4 \ub2e4\uc2dc \ud55c \ubc88 8\uac15\uc5d0 \uc62c\ub790\ub2e4.", "paragraph_sentence": " 1993\ub144 \ucd08 \ud638\uc8fc \uc624\ud508\uc5d0\uc11c 4\uac15\uc5d0 \uc62c\ub790\uc73c\uba70 \ud504\ub791\uc2a4 \uc624\ud508\uc5d0\uc11c\ub294 \uc774\uc804 \ud574\uc5d0 \uc774\uc5b4 \ub2e4\uc2dc \ud55c \ubc88 8\uac15\uc5d0 \uc62c\ub790\ub2e4. \uadf8\ub9ac\uace0 1993\ub144 4\uc6d4 \uc0dd\uc560 \ucd5c\ucd08\ub85c \uc138\uacc4 \ub7ad\ud0b9 1\uc704\uc5d0 \uc62c\ub790\ub2e4. \ub2f9\uc2dc \uc0d8\ud504\ub7ec\uc2a4\ub294 1990\ub144 US \uc624\ud508 \uc6b0\uc2b9 \uc774\ud6c4 \uadf8\ub79c\ub4dc \uc2ac\ub7a8 \ud0c0\uc774\ud2c0\uc774 \uc5c6\ub294 \uc0c1\ud0dc\uc600\uae30 \ub54c\ubb38\uc5d0 \uadf8\uac00 \uc9c4\uc815\ud55c \uc138\uacc4 \ub7ad\ud0b9 1\uc704\ub77c\uace0 \ud560 \uc218 \uc788\ub294\uc9c0\uc5d0 \ub300\ud574 \ub2e4\uc18c \ub17c\ub780\uc774 \uc788\uc5c8\ub2e4. \uadf8\ub7ec\ub098 \uadf8\ub294 \uc138 \ub2ec \ud6c4 \uc714\ube14\ub358 \uacb0\uc2b9\uc5d0\uc11c \uc804 \uc138\uacc4 \ub7ad\ud0b9 1\uc704\uc600\ub358 \uc9d0 \ucfe0\ub9ac\uc5b4\ub97c \uaebe\uace0 \uc6b0\uc2b9\ud558\uba74\uc11c \uadf8\ub7ec\ud55c \ub17c\ub780\uc744 \uc7a0\uc7ac\uc6e0\ub2e4. \uadf8 \uc5ec\uc138\ub97c \ubab0\uc544 \uac19\uc740 \ud574 US \uc624\ud508\uc5d0\uc11c\ub3c4 \uc6b0\uc2b9\ud558\uba74\uc11c \uc0dd\uc560 \ub450 \ubc88\uc9f8 US \uc624\ud508 \ud0c0\uc774\ud2c0\uc744 \ud68d\ub4dd\ud588\ub2e4. \uadf8\ub294 \uc774 \ud574 \uc5f0\ub9d0 ATP \ud22c\uc5b4 \uc0ac\uc0c1 \ucd5c\ucd08\ub85c \ud55c \uc2dc\uc98c \ucc9c \uac1c \uc774\uc0c1\uc758 \uc5d0\uc774\uc2a4\ub97c \uae30\ub85d\ud558\uba74\uc11c \ubd80\ub3d9\uc758 1\uc704\ub85c \uc2dc\uc98c\uc744 \ub9c8\uac10\ud588\ub2e4.", "paragraph_answer": " 1993\ub144 \ucd08 \ud638\uc8fc \uc624\ud508\uc5d0\uc11c 4\uac15\uc5d0 \uc62c\ub790\uc73c\uba70 \ud504\ub791\uc2a4 \uc624\ud508\uc5d0\uc11c\ub294 \uc774\uc804 \ud574\uc5d0 \uc774\uc5b4 \ub2e4\uc2dc \ud55c \ubc88 8\uac15\uc5d0 \uc62c\ub790\ub2e4. \uadf8\ub9ac\uace0 1993\ub144 4\uc6d4 \uc0dd\uc560 \ucd5c\ucd08\ub85c \uc138\uacc4 \ub7ad\ud0b9 1\uc704\uc5d0 \uc62c\ub790\ub2e4. \ub2f9\uc2dc \uc0d8\ud504\ub7ec\uc2a4\ub294 1990\ub144 US \uc624\ud508 \uc6b0\uc2b9 \uc774\ud6c4 \uadf8\ub79c\ub4dc \uc2ac\ub7a8 \ud0c0\uc774\ud2c0\uc774 \uc5c6\ub294 \uc0c1\ud0dc\uc600\uae30 \ub54c\ubb38\uc5d0 \uadf8\uac00 \uc9c4\uc815\ud55c \uc138\uacc4 \ub7ad\ud0b9 1\uc704\ub77c\uace0 \ud560 \uc218 \uc788\ub294\uc9c0\uc5d0 \ub300\ud574 \ub2e4\uc18c \ub17c\ub780\uc774 \uc788\uc5c8\ub2e4. \uadf8\ub7ec\ub098 \uadf8\ub294 \uc138 \ub2ec \ud6c4 \uc714\ube14\ub358 \uacb0\uc2b9\uc5d0\uc11c \uc804 \uc138\uacc4 \ub7ad\ud0b9 1\uc704\uc600\ub358 \uc9d0 \ucfe0\ub9ac\uc5b4\ub97c \uaebe\uace0 \uc6b0\uc2b9\ud558\uba74\uc11c \uadf8\ub7ec\ud55c \ub17c\ub780\uc744 \uc7a0\uc7ac\uc6e0\ub2e4. \uadf8 \uc5ec\uc138\ub97c \ubab0\uc544 \uac19\uc740 \ud574 US \uc624\ud508\uc5d0\uc11c\ub3c4 \uc6b0\uc2b9\ud558\uba74\uc11c \uc0dd\uc560 \ub450 \ubc88\uc9f8 US \uc624\ud508 \ud0c0\uc774\ud2c0\uc744 \ud68d\ub4dd\ud588\ub2e4. \uadf8\ub294 \uc774 \ud574 \uc5f0\ub9d0 ATP \ud22c\uc5b4 \uc0ac\uc0c1 \ucd5c\ucd08\ub85c \ud55c \uc2dc\uc98c \ucc9c \uac1c \uc774\uc0c1\uc758 \uc5d0\uc774\uc2a4\ub97c \uae30\ub85d\ud558\uba74\uc11c \ubd80\ub3d9\uc758 1\uc704\ub85c \uc2dc\uc98c\uc744 \ub9c8\uac10\ud588\ub2e4.", "sentence_answer": " 1993\ub144 \ucd08 \ud638\uc8fc \uc624\ud508\uc5d0\uc11c 4\uac15\uc5d0 \uc62c\ub790\uc73c\uba70 \ud504\ub791\uc2a4 \uc624\ud508\uc5d0\uc11c\ub294 \uc774\uc804 \ud574\uc5d0 \uc774\uc5b4 \ub2e4\uc2dc \ud55c \ubc88 8\uac15\uc5d0 \uc62c\ub790\ub2e4.", "paragraph_id": "6568924-5-0", "paragraph_question": "question: \uc0d8\ud504\ub7ec\uc2a4\uac00 \uc0dd\uc560 \uccab \uc138\uacc4 \ub7ad\ud0b9 1\uc704\ub97c \ud55c \ud574\ub294?, context: 1993\ub144 \ucd08 \ud638\uc8fc \uc624\ud508\uc5d0\uc11c 4\uac15\uc5d0 \uc62c\ub790\uc73c\uba70 \ud504\ub791\uc2a4 \uc624\ud508\uc5d0\uc11c\ub294 \uc774\uc804 \ud574\uc5d0 \uc774\uc5b4 \ub2e4\uc2dc \ud55c \ubc88 8\uac15\uc5d0 \uc62c\ub790\ub2e4. \uadf8\ub9ac\uace0 1993\ub144 4\uc6d4 \uc0dd\uc560 \ucd5c\ucd08\ub85c \uc138\uacc4 \ub7ad\ud0b9 1\uc704\uc5d0 \uc62c\ub790\ub2e4. \ub2f9\uc2dc \uc0d8\ud504\ub7ec\uc2a4\ub294 1990\ub144 US \uc624\ud508 \uc6b0\uc2b9 \uc774\ud6c4 \uadf8\ub79c\ub4dc \uc2ac\ub7a8 \ud0c0\uc774\ud2c0\uc774 \uc5c6\ub294 \uc0c1\ud0dc\uc600\uae30 \ub54c\ubb38\uc5d0 \uadf8\uac00 \uc9c4\uc815\ud55c \uc138\uacc4 \ub7ad\ud0b9 1\uc704\ub77c\uace0 \ud560 \uc218 \uc788\ub294\uc9c0\uc5d0 \ub300\ud574 \ub2e4\uc18c \ub17c\ub780\uc774 \uc788\uc5c8\ub2e4. \uadf8\ub7ec\ub098 \uadf8\ub294 \uc138 \ub2ec \ud6c4 \uc714\ube14\ub358 \uacb0\uc2b9\uc5d0\uc11c \uc804 \uc138\uacc4 \ub7ad\ud0b9 1\uc704\uc600\ub358 \uc9d0 \ucfe0\ub9ac\uc5b4\ub97c \uaebe\uace0 \uc6b0\uc2b9\ud558\uba74\uc11c \uadf8\ub7ec\ud55c \ub17c\ub780\uc744 \uc7a0\uc7ac\uc6e0\ub2e4. \uadf8 \uc5ec\uc138\ub97c \ubab0\uc544 \uac19\uc740 \ud574 US \uc624\ud508\uc5d0\uc11c\ub3c4 \uc6b0\uc2b9\ud558\uba74\uc11c \uc0dd\uc560 \ub450 \ubc88\uc9f8 US \uc624\ud508 \ud0c0\uc774\ud2c0\uc744 \ud68d\ub4dd\ud588\ub2e4. \uadf8\ub294 \uc774 \ud574 \uc5f0\ub9d0 ATP \ud22c\uc5b4 \uc0ac\uc0c1 \ucd5c\ucd08\ub85c \ud55c \uc2dc\uc98c \ucc9c \uac1c \uc774\uc0c1\uc758 \uc5d0\uc774\uc2a4\ub97c \uae30\ub85d\ud558\uba74\uc11c \ubd80\ub3d9\uc758 1\uc704\ub85c \uc2dc\uc98c\uc744 \ub9c8\uac10\ud588\ub2e4."} {"question": "LG V30\uc5d0 \ud0d1\uc7ac\ub41c \ub514\uc2a4\ud50c\ub808\uc774\ub294?", "paragraph": "LG V30\ub294 LG\uc804\uc790 \uc2a4\ub9c8\ud2b8\ud3f0 \ucd5c\ucd08\ub85c LG\ub514\uc2a4\ud50c\ub808\uc774\uc5d0\uc11c \uac1c\ubc1c\ud55c \uc62c\ub808\ub4dc \ud480\ube44\uc804 (OLED FullVision) \ub514\uc2a4\ud50c\ub808\uc774\ub97c \ud0d1\uc7ac\ud588\ub2e4. LG G6\uc640 \uac19\uc774 18:9\uc758 \ud654\uba74\ube44\ub97c \uc9c0\uc6d0\ud558\uba70 \uc804\uc791\uc778 LG V20\uc5d0 \ube44\ud574 \uc0c1\ud558\ub2e8 \ubca0\uc824\uacfc \uc88c\uc6b0 \ubca0\uc824\uc774 \uc904\uc5b4\ub4e4\uc5c8\ub2e4. LCD\ub97c \ud0d1\uc7ac\ud55c LG G6\ubcf4\ub2e4 \uba85\uc554\ube44\uc640 \uc0c9 \ud45c\ud604\ub825\uc774 \ud5a5\uc0c1\ub418\uc5c8\ub2e4. OLED\ub294 \ubc31\ub77c\uc774\ud2b8 \uc870\uba85\uc774 \ud544\uc694\uc5c6\uae30 \ub54c\ubb38\uc5d0, G6 \ub300\ube44 \ub354 \uc587\uc740 \ub450\uaed8 \uac00\ubcbc\uc6b4 \ubb34\uac8c\ub97c \uad6c\ud604\ud588\ub2e4. \ubca0\uc824 \ubca4\ub529 \uae30\uc220\uc744 \uc801\uc6a9\ud574, G6\ubcf4\ub2e4 \ud558\ub2e8 \ubca0\uc824 \ud06c\uae30\uac00 \uc904\uc5b4\ub4e4\uc5c8\uace0 \ubcf8\uccb4\uc758 \uce21\uba74\uc744 \uace1\uba74\uc73c\ub85c \uc124\uacc4\ud588\ub2e4. LCD\ubcf4\ub2e4 \uc751\ub2f5 \uc18d\ub3c4\uac00 \ube60\ub974\ub2e4. \uc804\uba74\uc758 LG \ub85c\uace0\uac00 \ud6c4\uba74\uc73c\ub85c \uc774\ub3d9\ud588\ub2e4. HDR 10\uacfc \ub3cc\ube44 \ube44\uc804\uc744 \uc9c0\uc6d0\ud558\uba70, \ud50c\ub77c\uc2a4\ud2f1 \uae30\ud310 \uc704\uc5d0 \ud654\uc18c\ub97c \ubc30\uce58\ud55c 6\uc138\ub300 POLED \uae30\ubc18\uc774\uae30 \ub54c\ubb38\uc5d0 \ucda9\uaca9\uc5d0 \uac15\ud558\ub2e4\uace0 \ud55c\ub2e4. \ud558\uc9c0\ub9cc, \uc774\ub85c \uc778\ud574 \uc138\ucee8\ub4dc \uc2a4\ud06c\ub9b0\uc740 \ud0d1\uc7ac\ub418\uc9c0 \uc54a\uc558\ub2e4.", "answer": "\uc62c\ub808\ub4dc \ud480\ube44\uc804", "sentence": "LG V30\ub294 LG\uc804\uc790 \uc2a4\ub9c8\ud2b8\ud3f0 \ucd5c\ucd08\ub85c LG\ub514\uc2a4\ud50c\ub808\uc774\uc5d0\uc11c \uac1c\ubc1c\ud55c \uc62c\ub808\ub4dc \ud480\ube44\uc804 (OLED FullVision) \ub514\uc2a4\ud50c\ub808\uc774\ub97c \ud0d1\uc7ac\ud588\ub2e4.", "paragraph_sentence": " LG V30\ub294 LG\uc804\uc790 \uc2a4\ub9c8\ud2b8\ud3f0 \ucd5c\ucd08\ub85c LG\ub514\uc2a4\ud50c\ub808\uc774\uc5d0\uc11c \uac1c\ubc1c\ud55c \uc62c\ub808\ub4dc \ud480\ube44\uc804 (OLED FullVision) \ub514\uc2a4\ud50c\ub808\uc774\ub97c \ud0d1\uc7ac\ud588\ub2e4. LG G6\uc640 \uac19\uc774 18:9\uc758 \ud654\uba74\ube44\ub97c \uc9c0\uc6d0\ud558\uba70 \uc804\uc791\uc778 LG V20\uc5d0 \ube44\ud574 \uc0c1\ud558\ub2e8 \ubca0\uc824\uacfc \uc88c\uc6b0 \ubca0\uc824\uc774 \uc904\uc5b4\ub4e4\uc5c8\ub2e4. LCD\ub97c \ud0d1\uc7ac\ud55c LG G6\ubcf4\ub2e4 \uba85\uc554\ube44\uc640 \uc0c9 \ud45c\ud604\ub825\uc774 \ud5a5\uc0c1\ub418\uc5c8\ub2e4. OLED\ub294 \ubc31\ub77c\uc774\ud2b8 \uc870\uba85\uc774 \ud544\uc694\uc5c6\uae30 \ub54c\ubb38\uc5d0, G6 \ub300\ube44 \ub354 \uc587\uc740 \ub450\uaed8 \uac00\ubcbc\uc6b4 \ubb34\uac8c\ub97c \uad6c\ud604\ud588\ub2e4. \ubca0\uc824 \ubca4\ub529 \uae30\uc220\uc744 \uc801\uc6a9\ud574, G6\ubcf4\ub2e4 \ud558\ub2e8 \ubca0\uc824 \ud06c\uae30\uac00 \uc904\uc5b4\ub4e4\uc5c8\uace0 \ubcf8\uccb4\uc758 \uce21\uba74\uc744 \uace1\uba74\uc73c\ub85c \uc124\uacc4\ud588\ub2e4. LCD\ubcf4\ub2e4 \uc751\ub2f5 \uc18d\ub3c4\uac00 \ube60\ub974\ub2e4. \uc804\uba74\uc758 LG \ub85c\uace0\uac00 \ud6c4\uba74\uc73c\ub85c \uc774\ub3d9\ud588\ub2e4. HDR 10\uacfc \ub3cc\ube44 \ube44\uc804\uc744 \uc9c0\uc6d0\ud558\uba70, \ud50c\ub77c\uc2a4\ud2f1 \uae30\ud310 \uc704\uc5d0 \ud654\uc18c\ub97c \ubc30\uce58\ud55c 6\uc138\ub300 POLED \uae30\ubc18\uc774\uae30 \ub54c\ubb38\uc5d0 \ucda9\uaca9\uc5d0 \uac15\ud558\ub2e4\uace0 \ud55c\ub2e4. \ud558\uc9c0\ub9cc, \uc774\ub85c \uc778\ud574 \uc138\ucee8\ub4dc \uc2a4\ud06c\ub9b0\uc740 \ud0d1\uc7ac\ub418\uc9c0 \uc54a\uc558\ub2e4.", "paragraph_answer": "LG V30\ub294 LG\uc804\uc790 \uc2a4\ub9c8\ud2b8\ud3f0 \ucd5c\ucd08\ub85c LG\ub514\uc2a4\ud50c\ub808\uc774\uc5d0\uc11c \uac1c\ubc1c\ud55c \uc62c\ub808\ub4dc \ud480\ube44\uc804 (OLED FullVision) \ub514\uc2a4\ud50c\ub808\uc774\ub97c \ud0d1\uc7ac\ud588\ub2e4. LG G6\uc640 \uac19\uc774 18:9\uc758 \ud654\uba74\ube44\ub97c \uc9c0\uc6d0\ud558\uba70 \uc804\uc791\uc778 LG V20\uc5d0 \ube44\ud574 \uc0c1\ud558\ub2e8 \ubca0\uc824\uacfc \uc88c\uc6b0 \ubca0\uc824\uc774 \uc904\uc5b4\ub4e4\uc5c8\ub2e4. LCD\ub97c \ud0d1\uc7ac\ud55c LG G6\ubcf4\ub2e4 \uba85\uc554\ube44\uc640 \uc0c9 \ud45c\ud604\ub825\uc774 \ud5a5\uc0c1\ub418\uc5c8\ub2e4. OLED\ub294 \ubc31\ub77c\uc774\ud2b8 \uc870\uba85\uc774 \ud544\uc694\uc5c6\uae30 \ub54c\ubb38\uc5d0, G6 \ub300\ube44 \ub354 \uc587\uc740 \ub450\uaed8 \uac00\ubcbc\uc6b4 \ubb34\uac8c\ub97c \uad6c\ud604\ud588\ub2e4. \ubca0\uc824 \ubca4\ub529 \uae30\uc220\uc744 \uc801\uc6a9\ud574, G6\ubcf4\ub2e4 \ud558\ub2e8 \ubca0\uc824 \ud06c\uae30\uac00 \uc904\uc5b4\ub4e4\uc5c8\uace0 \ubcf8\uccb4\uc758 \uce21\uba74\uc744 \uace1\uba74\uc73c\ub85c \uc124\uacc4\ud588\ub2e4. LCD\ubcf4\ub2e4 \uc751\ub2f5 \uc18d\ub3c4\uac00 \ube60\ub974\ub2e4. \uc804\uba74\uc758 LG \ub85c\uace0\uac00 \ud6c4\uba74\uc73c\ub85c \uc774\ub3d9\ud588\ub2e4. HDR 10\uacfc \ub3cc\ube44 \ube44\uc804\uc744 \uc9c0\uc6d0\ud558\uba70, \ud50c\ub77c\uc2a4\ud2f1 \uae30\ud310 \uc704\uc5d0 \ud654\uc18c\ub97c \ubc30\uce58\ud55c 6\uc138\ub300 POLED \uae30\ubc18\uc774\uae30 \ub54c\ubb38\uc5d0 \ucda9\uaca9\uc5d0 \uac15\ud558\ub2e4\uace0 \ud55c\ub2e4. \ud558\uc9c0\ub9cc, \uc774\ub85c \uc778\ud574 \uc138\ucee8\ub4dc \uc2a4\ud06c\ub9b0\uc740 \ud0d1\uc7ac\ub418\uc9c0 \uc54a\uc558\ub2e4.", "sentence_answer": "LG V30\ub294 LG\uc804\uc790 \uc2a4\ub9c8\ud2b8\ud3f0 \ucd5c\ucd08\ub85c LG\ub514\uc2a4\ud50c\ub808\uc774\uc5d0\uc11c \uac1c\ubc1c\ud55c \uc62c\ub808\ub4dc \ud480\ube44\uc804 (OLED FullVision) \ub514\uc2a4\ud50c\ub808\uc774\ub97c \ud0d1\uc7ac\ud588\ub2e4.", "paragraph_id": "6500238-1-0", "paragraph_question": "question: LG V30\uc5d0 \ud0d1\uc7ac\ub41c \ub514\uc2a4\ud50c\ub808\uc774\ub294?, context: LG V30\ub294 LG\uc804\uc790 \uc2a4\ub9c8\ud2b8\ud3f0 \ucd5c\ucd08\ub85c LG\ub514\uc2a4\ud50c\ub808\uc774\uc5d0\uc11c \uac1c\ubc1c\ud55c \uc62c\ub808\ub4dc \ud480\ube44\uc804 (OLED FullVision) \ub514\uc2a4\ud50c\ub808\uc774\ub97c \ud0d1\uc7ac\ud588\ub2e4. LG G6\uc640 \uac19\uc774 18:9\uc758 \ud654\uba74\ube44\ub97c \uc9c0\uc6d0\ud558\uba70 \uc804\uc791\uc778 LG V20\uc5d0 \ube44\ud574 \uc0c1\ud558\ub2e8 \ubca0\uc824\uacfc \uc88c\uc6b0 \ubca0\uc824\uc774 \uc904\uc5b4\ub4e4\uc5c8\ub2e4. LCD\ub97c \ud0d1\uc7ac\ud55c LG G6\ubcf4\ub2e4 \uba85\uc554\ube44\uc640 \uc0c9 \ud45c\ud604\ub825\uc774 \ud5a5\uc0c1\ub418\uc5c8\ub2e4. OLED\ub294 \ubc31\ub77c\uc774\ud2b8 \uc870\uba85\uc774 \ud544\uc694\uc5c6\uae30 \ub54c\ubb38\uc5d0, G6 \ub300\ube44 \ub354 \uc587\uc740 \ub450\uaed8 \uac00\ubcbc\uc6b4 \ubb34\uac8c\ub97c \uad6c\ud604\ud588\ub2e4. \ubca0\uc824 \ubca4\ub529 \uae30\uc220\uc744 \uc801\uc6a9\ud574, G6\ubcf4\ub2e4 \ud558\ub2e8 \ubca0\uc824 \ud06c\uae30\uac00 \uc904\uc5b4\ub4e4\uc5c8\uace0 \ubcf8\uccb4\uc758 \uce21\uba74\uc744 \uace1\uba74\uc73c\ub85c \uc124\uacc4\ud588\ub2e4. LCD\ubcf4\ub2e4 \uc751\ub2f5 \uc18d\ub3c4\uac00 \ube60\ub974\ub2e4. \uc804\uba74\uc758 LG \ub85c\uace0\uac00 \ud6c4\uba74\uc73c\ub85c \uc774\ub3d9\ud588\ub2e4. HDR 10\uacfc \ub3cc\ube44 \ube44\uc804\uc744 \uc9c0\uc6d0\ud558\uba70, \ud50c\ub77c\uc2a4\ud2f1 \uae30\ud310 \uc704\uc5d0 \ud654\uc18c\ub97c \ubc30\uce58\ud55c 6\uc138\ub300 POLED \uae30\ubc18\uc774\uae30 \ub54c\ubb38\uc5d0 \ucda9\uaca9\uc5d0 \uac15\ud558\ub2e4\uace0 \ud55c\ub2e4. \ud558\uc9c0\ub9cc, \uc774\ub85c \uc778\ud574 \uc138\ucee8\ub4dc \uc2a4\ud06c\ub9b0\uc740 \ud0d1\uc7ac\ub418\uc9c0 \uc54a\uc558\ub2e4."} {"question": "\uce74\ub97c \ub300\uacf5\uc744 \ub300\uc2e0\ud574 \uc0c8\ub86d\uac8c \uc624\uc2a4\ud2b8\ub9ac\uc544\uad70\uc758 \uc0ac\ub839\uad00\uc774 \ub41c \uc778\ubb3c\uc740 \ub204\uad6c\uc778\uac00?", "paragraph": "\uc624\uc2a4\ud2b8\ub9ac\uc544 \ud669\uc81c\uc758 \ub3d9\uc0dd, \uce74\ub97c \ub300\uacf5(Archduke Charles)\uc740 \uc624\uc2a4\ud2b8\ub9ac\uc544\uad70\uc758 \uacb0\uc815\uad8c\uc744 \ub2f4\ub2f9\ud558\ub294 \uad70\uc0ac\uc801, \uc815\ucc45\uc801 \uc704\uc6d0\ud68c\uc778 \ud638\ud504\ud06c\ub9ac\ud06c\uc2a4\ub77c\ud2b8(Hofkriegsrat;\ucd5c\uace0\uad70\uc0ac\uc704\uc6d0\ud68c)\ub85c\ubd80\ud130 \uacb0\uc815\uad8c\uc744 \ube7c\uc557\uc740 \ud6c4, 1801\ub144\ubd80\ud130 \uc624\uc2a4\ud2b8\ub9ac\uc544\uad70\uc744 \uc7ac\uac74\ud558\uae30 \uc2dc\uc791\ud588\ub2e4. \uce74\ub97c\uc740 \uc624\uc2a4\ud2b8\ub9ac\uc544\uc758 \uac00\uc7a5 \ub6f0\uc5b4\ub09c \uc57c\uc804 \uc9c0\ud718\uad00\uc774\uc5c8\uc9c0\ub9cc, \uadf8\ub7ec\ub098 \uadf8\ub294 \ud669\uc2e4\uc5d0\uc11c \uc778\uae30\uac00 \uc5c6\uc5b4 \ub9ce\uc740 \uc601\ud5a5\ub825\uc744 \uc783\uace0, \uadf8\uc758 \ucda9\uace0\ub97c \ubb34\uc2dc\ud558\uace0, \uc624\uc2a4\ud2b8\ub9ac\uc544\uac00 \ud504\ub791\uc2a4\uc640 \uc804\uc7c1\uc744 \uc2dc\uc791\ud588\ub2e4. \uce74\ub97c \ub9c8\ud06c(Karl Mack)\uac00 \uadf8\ub97c \ub300\uc2e0\ud574 \uc0c8\ub86d\uac8c \uc624\uc2a4\ud2b8\ub9ac\uc544\uad70\uc758 \uc0ac\ub839\uad00\uc774 \ub418\uc5b4 \uc804\uc7c1 \uc804\uc57c\uc5d0 \uae30\uc874\uc758 \uac01\uac01 6\uac1c\uc758 \uc911\ub300\ub85c \uad6c\uc131\ub41c 3\uac1c \ub300\ub300\uc758 \uad6c\uc131\ubcf4\ub2e4\ub294 \uac01\uac01 4\uac1c \uc911\ub300\ub85c \uad6c\uc131\ub41c 4\uac1c \ub300\ub300\ub85c \uc5f0\ub300\ub97c \uad6c\uc131\ud558\ub294 \uac1c\ud601\uc744 \ud558\uc5ec \ubcf4\ubcd1\ub300\ub97c \uc124\ub9bd\ud558\uc600\ub2e4. \uac11\uc791\uc2a4\ub7f0 \ubcc0\ud654\ub294 \uc7a5\uad50\ub4e4\uc758 \ud6c8\ub828\uc774 \ucda9\ubd84\ud558\uc9c0 \ubabb\ud588\uace0, \uc774\ub7f0 \uacb0\uacfc\uc640 \uc0c8\ub85c\uc6b4 \ubd80\ub300\ud3b8\uc131\uc740 \uc624\uc2a4\ud2b8\ub9ac\uc544\uad70\uc774 \uc774\uc81c\uae4c\uc9c0 \ud574\uc654\ub358 \uac83\ubcf4\ub2e4 \uc624\ud788\ub824 \ub354 \uc548 \uc88b\uc740 \uacb0\uacfc\ub97c \ubd88\ub7ec\uc654\ub2e4. \uc624\uc2a4\ud2b8\ub9ac\uc544 \uae30\ubcd1\uc740 \ub2f9\uc2dc \uc720\ub7fd\uc5d0\uc11c \ucd5c\uac15\uc758 \uae30\ubcd1\uc73c\ub85c \uc5ec\uaca8\uc84c\uc73c\ub098, \ub9ce\uc740 \uae30\ubcd1\ubd84\uacac\ub300\ub97c \ub2e4\uc591\ud55c \ubcf4\ubcd1\ud3b8\uc131\uc5d0 \uc9d1\uc5b4\ub123\ub294 \uac83\uc740 \ubcf4\ubcd1\uacfc \uae30\ubcd1\uc774 \uac01\uac01 \ubc00\uc9d1\ud55c \ubd80\ub300\ub97c \uad6c\uc131\ud55c \ud504\ub791\uc2a4\uad70\uc5d0 \ube44\ud574 \ubcc4\ub85c \ud6a8\uacfc\uc801\uc774\uc9c0 \ubabb\ud588\ub2e4.", "answer": "\uce74\ub97c \ub9c8\ud06c", "sentence": "\uce74\ub97c \ub9c8\ud06c (Karl Mack)\uac00 \uadf8\ub97c \ub300\uc2e0\ud574 \uc0c8\ub86d\uac8c \uc624\uc2a4\ud2b8\ub9ac\uc544\uad70\uc758 \uc0ac\ub839\uad00\uc774 \ub418\uc5b4 \uc804\uc7c1 \uc804\uc57c\uc5d0 \uae30\uc874\uc758 \uac01\uac01 6\uac1c\uc758 \uc911\ub300\ub85c \uad6c\uc131\ub41c 3\uac1c \ub300\ub300\uc758 \uad6c\uc131\ubcf4\ub2e4\ub294 \uac01\uac01 4\uac1c \uc911\ub300\ub85c \uad6c\uc131\ub41c 4\uac1c \ub300\ub300\ub85c \uc5f0\ub300\ub97c \uad6c\uc131\ud558\ub294 \uac1c\ud601\uc744 \ud558\uc5ec \ubcf4\ubcd1\ub300\ub97c \uc124\ub9bd\ud558\uc600\ub2e4.", "paragraph_sentence": "\uc624\uc2a4\ud2b8\ub9ac\uc544 \ud669\uc81c\uc758 \ub3d9\uc0dd, \uce74\ub97c \ub300\uacf5(Archduke Charles)\uc740 \uc624\uc2a4\ud2b8\ub9ac\uc544\uad70\uc758 \uacb0\uc815\uad8c\uc744 \ub2f4\ub2f9\ud558\ub294 \uad70\uc0ac\uc801, \uc815\ucc45\uc801 \uc704\uc6d0\ud68c\uc778 \ud638\ud504\ud06c\ub9ac\ud06c\uc2a4\ub77c\ud2b8(Hofkriegsrat;\ucd5c\uace0\uad70\uc0ac\uc704\uc6d0\ud68c)\ub85c\ubd80\ud130 \uacb0\uc815\uad8c\uc744 \ube7c\uc557\uc740 \ud6c4, 1801\ub144\ubd80\ud130 \uc624\uc2a4\ud2b8\ub9ac\uc544\uad70\uc744 \uc7ac\uac74\ud558\uae30 \uc2dc\uc791\ud588\ub2e4. \uce74\ub97c\uc740 \uc624\uc2a4\ud2b8\ub9ac\uc544\uc758 \uac00\uc7a5 \ub6f0\uc5b4\ub09c \uc57c\uc804 \uc9c0\ud718\uad00\uc774\uc5c8\uc9c0\ub9cc, \uadf8\ub7ec\ub098 \uadf8\ub294 \ud669\uc2e4\uc5d0\uc11c \uc778\uae30\uac00 \uc5c6\uc5b4 \ub9ce\uc740 \uc601\ud5a5\ub825\uc744 \uc783\uace0, \uadf8\uc758 \ucda9\uace0\ub97c \ubb34\uc2dc\ud558\uace0, \uc624\uc2a4\ud2b8\ub9ac\uc544\uac00 \ud504\ub791\uc2a4\uc640 \uc804\uc7c1\uc744 \uc2dc\uc791\ud588\ub2e4. \uce74\ub97c \ub9c8\ud06c (Karl Mack)\uac00 \uadf8\ub97c \ub300\uc2e0\ud574 \uc0c8\ub86d\uac8c \uc624\uc2a4\ud2b8\ub9ac\uc544\uad70\uc758 \uc0ac\ub839\uad00\uc774 \ub418\uc5b4 \uc804\uc7c1 \uc804\uc57c\uc5d0 \uae30\uc874\uc758 \uac01\uac01 6\uac1c\uc758 \uc911\ub300\ub85c \uad6c\uc131\ub41c 3\uac1c \ub300\ub300\uc758 \uad6c\uc131\ubcf4\ub2e4\ub294 \uac01\uac01 4\uac1c \uc911\ub300\ub85c \uad6c\uc131\ub41c 4\uac1c \ub300\ub300\ub85c \uc5f0\ub300\ub97c \uad6c\uc131\ud558\ub294 \uac1c\ud601\uc744 \ud558\uc5ec \ubcf4\ubcd1\ub300\ub97c \uc124\ub9bd\ud558\uc600\ub2e4. \uac11\uc791\uc2a4\ub7f0 \ubcc0\ud654\ub294 \uc7a5\uad50\ub4e4\uc758 \ud6c8\ub828\uc774 \ucda9\ubd84\ud558\uc9c0 \ubabb\ud588\uace0, \uc774\ub7f0 \uacb0\uacfc\uc640 \uc0c8\ub85c\uc6b4 \ubd80\ub300\ud3b8\uc131\uc740 \uc624\uc2a4\ud2b8\ub9ac\uc544\uad70\uc774 \uc774\uc81c\uae4c\uc9c0 \ud574\uc654\ub358 \uac83\ubcf4\ub2e4 \uc624\ud788\ub824 \ub354 \uc548 \uc88b\uc740 \uacb0\uacfc\ub97c \ubd88\ub7ec\uc654\ub2e4. \uc624\uc2a4\ud2b8\ub9ac\uc544 \uae30\ubcd1\uc740 \ub2f9\uc2dc \uc720\ub7fd\uc5d0\uc11c \ucd5c\uac15\uc758 \uae30\ubcd1\uc73c\ub85c \uc5ec\uaca8\uc84c\uc73c\ub098, \ub9ce\uc740 \uae30\ubcd1\ubd84\uacac\ub300\ub97c \ub2e4\uc591\ud55c \ubcf4\ubcd1\ud3b8\uc131\uc5d0 \uc9d1\uc5b4\ub123\ub294 \uac83\uc740 \ubcf4\ubcd1\uacfc \uae30\ubcd1\uc774 \uac01\uac01 \ubc00\uc9d1\ud55c \ubd80\ub300\ub97c \uad6c\uc131\ud55c \ud504\ub791\uc2a4\uad70\uc5d0 \ube44\ud574 \ubcc4\ub85c \ud6a8\uacfc\uc801\uc774\uc9c0 \ubabb\ud588\ub2e4.", "paragraph_answer": "\uc624\uc2a4\ud2b8\ub9ac\uc544 \ud669\uc81c\uc758 \ub3d9\uc0dd, \uce74\ub97c \ub300\uacf5(Archduke Charles)\uc740 \uc624\uc2a4\ud2b8\ub9ac\uc544\uad70\uc758 \uacb0\uc815\uad8c\uc744 \ub2f4\ub2f9\ud558\ub294 \uad70\uc0ac\uc801, \uc815\ucc45\uc801 \uc704\uc6d0\ud68c\uc778 \ud638\ud504\ud06c\ub9ac\ud06c\uc2a4\ub77c\ud2b8(Hofkriegsrat;\ucd5c\uace0\uad70\uc0ac\uc704\uc6d0\ud68c)\ub85c\ubd80\ud130 \uacb0\uc815\uad8c\uc744 \ube7c\uc557\uc740 \ud6c4, 1801\ub144\ubd80\ud130 \uc624\uc2a4\ud2b8\ub9ac\uc544\uad70\uc744 \uc7ac\uac74\ud558\uae30 \uc2dc\uc791\ud588\ub2e4. \uce74\ub97c\uc740 \uc624\uc2a4\ud2b8\ub9ac\uc544\uc758 \uac00\uc7a5 \ub6f0\uc5b4\ub09c \uc57c\uc804 \uc9c0\ud718\uad00\uc774\uc5c8\uc9c0\ub9cc, \uadf8\ub7ec\ub098 \uadf8\ub294 \ud669\uc2e4\uc5d0\uc11c \uc778\uae30\uac00 \uc5c6\uc5b4 \ub9ce\uc740 \uc601\ud5a5\ub825\uc744 \uc783\uace0, \uadf8\uc758 \ucda9\uace0\ub97c \ubb34\uc2dc\ud558\uace0, \uc624\uc2a4\ud2b8\ub9ac\uc544\uac00 \ud504\ub791\uc2a4\uc640 \uc804\uc7c1\uc744 \uc2dc\uc791\ud588\ub2e4. \uce74\ub97c \ub9c8\ud06c (Karl Mack)\uac00 \uadf8\ub97c \ub300\uc2e0\ud574 \uc0c8\ub86d\uac8c \uc624\uc2a4\ud2b8\ub9ac\uc544\uad70\uc758 \uc0ac\ub839\uad00\uc774 \ub418\uc5b4 \uc804\uc7c1 \uc804\uc57c\uc5d0 \uae30\uc874\uc758 \uac01\uac01 6\uac1c\uc758 \uc911\ub300\ub85c \uad6c\uc131\ub41c 3\uac1c \ub300\ub300\uc758 \uad6c\uc131\ubcf4\ub2e4\ub294 \uac01\uac01 4\uac1c \uc911\ub300\ub85c \uad6c\uc131\ub41c 4\uac1c \ub300\ub300\ub85c \uc5f0\ub300\ub97c \uad6c\uc131\ud558\ub294 \uac1c\ud601\uc744 \ud558\uc5ec \ubcf4\ubcd1\ub300\ub97c \uc124\ub9bd\ud558\uc600\ub2e4. \uac11\uc791\uc2a4\ub7f0 \ubcc0\ud654\ub294 \uc7a5\uad50\ub4e4\uc758 \ud6c8\ub828\uc774 \ucda9\ubd84\ud558\uc9c0 \ubabb\ud588\uace0, \uc774\ub7f0 \uacb0\uacfc\uc640 \uc0c8\ub85c\uc6b4 \ubd80\ub300\ud3b8\uc131\uc740 \uc624\uc2a4\ud2b8\ub9ac\uc544\uad70\uc774 \uc774\uc81c\uae4c\uc9c0 \ud574\uc654\ub358 \uac83\ubcf4\ub2e4 \uc624\ud788\ub824 \ub354 \uc548 \uc88b\uc740 \uacb0\uacfc\ub97c \ubd88\ub7ec\uc654\ub2e4. \uc624\uc2a4\ud2b8\ub9ac\uc544 \uae30\ubcd1\uc740 \ub2f9\uc2dc \uc720\ub7fd\uc5d0\uc11c \ucd5c\uac15\uc758 \uae30\ubcd1\uc73c\ub85c \uc5ec\uaca8\uc84c\uc73c\ub098, \ub9ce\uc740 \uae30\ubcd1\ubd84\uacac\ub300\ub97c \ub2e4\uc591\ud55c \ubcf4\ubcd1\ud3b8\uc131\uc5d0 \uc9d1\uc5b4\ub123\ub294 \uac83\uc740 \ubcf4\ubcd1\uacfc \uae30\ubcd1\uc774 \uac01\uac01 \ubc00\uc9d1\ud55c \ubd80\ub300\ub97c \uad6c\uc131\ud55c \ud504\ub791\uc2a4\uad70\uc5d0 \ube44\ud574 \ubcc4\ub85c \ud6a8\uacfc\uc801\uc774\uc9c0 \ubabb\ud588\ub2e4.", "sentence_answer": " \uce74\ub97c \ub9c8\ud06c (Karl Mack)\uac00 \uadf8\ub97c \ub300\uc2e0\ud574 \uc0c8\ub86d\uac8c \uc624\uc2a4\ud2b8\ub9ac\uc544\uad70\uc758 \uc0ac\ub839\uad00\uc774 \ub418\uc5b4 \uc804\uc7c1 \uc804\uc57c\uc5d0 \uae30\uc874\uc758 \uac01\uac01 6\uac1c\uc758 \uc911\ub300\ub85c \uad6c\uc131\ub41c 3\uac1c \ub300\ub300\uc758 \uad6c\uc131\ubcf4\ub2e4\ub294 \uac01\uac01 4\uac1c \uc911\ub300\ub85c \uad6c\uc131\ub41c 4\uac1c \ub300\ub300\ub85c \uc5f0\ub300\ub97c \uad6c\uc131\ud558\ub294 \uac1c\ud601\uc744 \ud558\uc5ec \ubcf4\ubcd1\ub300\ub97c \uc124\ub9bd\ud558\uc600\ub2e4.", "paragraph_id": "6533833-5-0", "paragraph_question": "question: \uce74\ub97c \ub300\uacf5\uc744 \ub300\uc2e0\ud574 \uc0c8\ub86d\uac8c \uc624\uc2a4\ud2b8\ub9ac\uc544\uad70\uc758 \uc0ac\ub839\uad00\uc774 \ub41c \uc778\ubb3c\uc740 \ub204\uad6c\uc778\uac00?, context: \uc624\uc2a4\ud2b8\ub9ac\uc544 \ud669\uc81c\uc758 \ub3d9\uc0dd, \uce74\ub97c \ub300\uacf5(Archduke Charles)\uc740 \uc624\uc2a4\ud2b8\ub9ac\uc544\uad70\uc758 \uacb0\uc815\uad8c\uc744 \ub2f4\ub2f9\ud558\ub294 \uad70\uc0ac\uc801, \uc815\ucc45\uc801 \uc704\uc6d0\ud68c\uc778 \ud638\ud504\ud06c\ub9ac\ud06c\uc2a4\ub77c\ud2b8(Hofkriegsrat;\ucd5c\uace0\uad70\uc0ac\uc704\uc6d0\ud68c)\ub85c\ubd80\ud130 \uacb0\uc815\uad8c\uc744 \ube7c\uc557\uc740 \ud6c4, 1801\ub144\ubd80\ud130 \uc624\uc2a4\ud2b8\ub9ac\uc544\uad70\uc744 \uc7ac\uac74\ud558\uae30 \uc2dc\uc791\ud588\ub2e4. \uce74\ub97c\uc740 \uc624\uc2a4\ud2b8\ub9ac\uc544\uc758 \uac00\uc7a5 \ub6f0\uc5b4\ub09c \uc57c\uc804 \uc9c0\ud718\uad00\uc774\uc5c8\uc9c0\ub9cc, \uadf8\ub7ec\ub098 \uadf8\ub294 \ud669\uc2e4\uc5d0\uc11c \uc778\uae30\uac00 \uc5c6\uc5b4 \ub9ce\uc740 \uc601\ud5a5\ub825\uc744 \uc783\uace0, \uadf8\uc758 \ucda9\uace0\ub97c \ubb34\uc2dc\ud558\uace0, \uc624\uc2a4\ud2b8\ub9ac\uc544\uac00 \ud504\ub791\uc2a4\uc640 \uc804\uc7c1\uc744 \uc2dc\uc791\ud588\ub2e4. \uce74\ub97c \ub9c8\ud06c(Karl Mack)\uac00 \uadf8\ub97c \ub300\uc2e0\ud574 \uc0c8\ub86d\uac8c \uc624\uc2a4\ud2b8\ub9ac\uc544\uad70\uc758 \uc0ac\ub839\uad00\uc774 \ub418\uc5b4 \uc804\uc7c1 \uc804\uc57c\uc5d0 \uae30\uc874\uc758 \uac01\uac01 6\uac1c\uc758 \uc911\ub300\ub85c \uad6c\uc131\ub41c 3\uac1c \ub300\ub300\uc758 \uad6c\uc131\ubcf4\ub2e4\ub294 \uac01\uac01 4\uac1c \uc911\ub300\ub85c \uad6c\uc131\ub41c 4\uac1c \ub300\ub300\ub85c \uc5f0\ub300\ub97c \uad6c\uc131\ud558\ub294 \uac1c\ud601\uc744 \ud558\uc5ec \ubcf4\ubcd1\ub300\ub97c \uc124\ub9bd\ud558\uc600\ub2e4. \uac11\uc791\uc2a4\ub7f0 \ubcc0\ud654\ub294 \uc7a5\uad50\ub4e4\uc758 \ud6c8\ub828\uc774 \ucda9\ubd84\ud558\uc9c0 \ubabb\ud588\uace0, \uc774\ub7f0 \uacb0\uacfc\uc640 \uc0c8\ub85c\uc6b4 \ubd80\ub300\ud3b8\uc131\uc740 \uc624\uc2a4\ud2b8\ub9ac\uc544\uad70\uc774 \uc774\uc81c\uae4c\uc9c0 \ud574\uc654\ub358 \uac83\ubcf4\ub2e4 \uc624\ud788\ub824 \ub354 \uc548 \uc88b\uc740 \uacb0\uacfc\ub97c \ubd88\ub7ec\uc654\ub2e4. \uc624\uc2a4\ud2b8\ub9ac\uc544 \uae30\ubcd1\uc740 \ub2f9\uc2dc \uc720\ub7fd\uc5d0\uc11c \ucd5c\uac15\uc758 \uae30\ubcd1\uc73c\ub85c \uc5ec\uaca8\uc84c\uc73c\ub098, \ub9ce\uc740 \uae30\ubcd1\ubd84\uacac\ub300\ub97c \ub2e4\uc591\ud55c \ubcf4\ubcd1\ud3b8\uc131\uc5d0 \uc9d1\uc5b4\ub123\ub294 \uac83\uc740 \ubcf4\ubcd1\uacfc \uae30\ubcd1\uc774 \uac01\uac01 \ubc00\uc9d1\ud55c \ubd80\ub300\ub97c \uad6c\uc131\ud55c \ud504\ub791\uc2a4\uad70\uc5d0 \ube44\ud574 \ubcc4\ub85c \ud6a8\uacfc\uc801\uc774\uc9c0 \ubabb\ud588\ub2e4."} {"question": "\uc774 \uc778\ubb3c\uc774 2\ub300 \ud68c\uc7a5\uc73c\ub85c \uc120\ucd9c\ub418\uc5c8\uc73c\uba70 \uc7ac\uc57c\uccad\ub144\ub2e8\uccb4\uc758 \ud6a8\uc2dc\ub77c \ubd88\ub9ac\ub294 \uc758\ud68c\uc758 \uc774\ub984\uc740?", "paragraph": "1964\ub144\uc5d0 \ub18d\ucd0c\uc6b4\ub3d9\uac00\ub97c \uafc8\uafb8\uba70 \uc911\uc559\ub300 \ub18d\ucd0c\uc0ac\ud68c\uac1c\ubc1c\ud559\uacfc\uc5d0 \uc785\ud559\ud558\uc600\uc73c\ub098 \ud55c\uc77c\ud611\uc815\uc744 \ubc18\ub300\ud558\ub294 6.3 \ud56d\uc7c1\uc744 \uc8fc\ub3c4\ud55c \ud610\uc758\ub85c \uad6c\uc18d\ub418\uc5b4 \uc81c\uc801\ub418\uc5c8\ub2e4. \uc774\ud6c4 \uad6d\ubbfc\uc0b0\uc5c5\ud559\uad50 \ub18d\uc5c5\uacbd\uc601\ud559\uacfc\uc5d0 \uc785\ud559\ud558\uc5ec \uc878\uc5c5\ud558\uc600\uace0 \uace0\ub824\ub300 \uad50\uc721\ub300\ud559\uc6d0\uc744 \uc878\uc5c5\ud588\ub2e4. \uc774\ud6c4 \ub2e4\uc2dc \uc911\uc559\ub300 \uacbd\uc81c\ud559\uacfc\uc5d0 \ubcf5\ud559\ud558\uc5ec 1996\ub144\uc5d0 \uc878\uc5c5\ud558\uc600\ub2e4. \uad70 \ubcf5\ubb34 \uc911\uc5d0 \ud30c\uacac\uad50\uc0ac \uc2dc\ud5d8\uc5d0 \ud569\uaca9\ud558\uc5ec \uc804\uc5ed \ud6c4\uc5d0 \uad50\uc0ac\uc0dd\ud65c\uc744 \uc2dc\uc791\ud588\uc73c\uba70 1973\ub144\uc5d0\ub294 \uc7a5\ud6c8\uace0\uc5d0\uc11c, 1979\ub144\uc5d0\ub294 \uc1a1\uace1\uc5ec\uace0\uc5d0\uc11c \uad6d\uc5b4 \uad50\uc0ac\ub85c \uc7ac\uc9c1\ud588\ub2e4. \ud568\uc11d\ud5cc, \uacc4\ud6c8\uc81c, \uae40\uc218\ud658 \ub4f1\uacfc \ud568\uaed8 \ubbfc\uc8fc\uc218\ud638\uad6d\ubbfc\ud611\uc758\ud68c\ub97c \uacb0\uc131\ud558\uc600\uace0, \uc7ac\uc57c\uccad\ub144\ub2e8\uccb4\uc758 \ud6a8\uc2dc\uc778 \ubbfc\uc8fc\uc218\ud638\uccad\ub144\ud611\uc758\ud68c\uc758 2\ub300 \ud68c\uc7a5\uc73c\ub85c \uc120\ucd9c\ub418\uc5c8\ub2e4. \uae40\uc218\ud658\uc73c\ub85c\ubd80\ud130 \uacbd\ubd81\uac00\ud1a8\ub9ad\ub18d\ubbfc\ud68c \uae30\ub3c4\ud68c\uc5d0\uc11c \uc778\uad8c \ubb38\uc81c\ub97c \uac15\uc5f0\ud574\ub2ec\ub77c\uace0 \uc694\uccad\uc744 \ubc1b\uc544 \ud2b9\uac15\uc744 \ud588\ub294\ub370 \uc774\ud6c4 \uae34\uae09\uc870\uce58 9\ud638 \uc704\ubc18\uc73c\ub85c \uad6c\uc18d\ub418\uc5c8\ub2e4.", "answer": "\ubbfc\uc8fc\uc218\ud638\uccad\ub144\ud611\uc758\ud68c\uc758", "sentence": "\uc7ac\uc57c\uccad\ub144\ub2e8\uccb4\uc758 \ud6a8\uc2dc\uc778 \ubbfc\uc8fc\uc218\ud638\uccad\ub144\ud611\uc758\ud68c\uc758 2\ub300 \ud68c\uc7a5\uc73c\ub85c \uc120\ucd9c\ub418\uc5c8\ub2e4.", "paragraph_sentence": "1964\ub144\uc5d0 \ub18d\ucd0c\uc6b4\ub3d9\uac00\ub97c \uafc8\uafb8\uba70 \uc911\uc559\ub300 \ub18d\ucd0c\uc0ac\ud68c\uac1c\ubc1c\ud559\uacfc\uc5d0 \uc785\ud559\ud558\uc600\uc73c\ub098 \ud55c\uc77c\ud611\uc815\uc744 \ubc18\ub300\ud558\ub294 6.3 \ud56d\uc7c1\uc744 \uc8fc\ub3c4\ud55c \ud610\uc758\ub85c \uad6c\uc18d\ub418\uc5b4 \uc81c\uc801\ub418\uc5c8\ub2e4. \uc774\ud6c4 \uad6d\ubbfc\uc0b0\uc5c5\ud559\uad50 \ub18d\uc5c5\uacbd\uc601\ud559\uacfc\uc5d0 \uc785\ud559\ud558\uc5ec \uc878\uc5c5\ud558\uc600\uace0 \uace0\ub824\ub300 \uad50\uc721\ub300\ud559\uc6d0\uc744 \uc878\uc5c5\ud588\ub2e4. \uc774\ud6c4 \ub2e4\uc2dc \uc911\uc559\ub300 \uacbd\uc81c\ud559\uacfc\uc5d0 \ubcf5\ud559\ud558\uc5ec 1996\ub144\uc5d0 \uc878\uc5c5\ud558\uc600\ub2e4. \uad70 \ubcf5\ubb34 \uc911\uc5d0 \ud30c\uacac\uad50\uc0ac \uc2dc\ud5d8\uc5d0 \ud569\uaca9\ud558\uc5ec \uc804\uc5ed \ud6c4\uc5d0 \uad50\uc0ac\uc0dd\ud65c\uc744 \uc2dc\uc791\ud588\uc73c\uba70 1973\ub144\uc5d0\ub294 \uc7a5\ud6c8\uace0\uc5d0\uc11c, 1979\ub144\uc5d0\ub294 \uc1a1\uace1\uc5ec\uace0\uc5d0\uc11c \uad6d\uc5b4 \uad50\uc0ac\ub85c \uc7ac\uc9c1\ud588\ub2e4. \ud568\uc11d\ud5cc, \uacc4\ud6c8\uc81c, \uae40\uc218\ud658 \ub4f1\uacfc \ud568\uaed8 \ubbfc\uc8fc\uc218\ud638\uad6d\ubbfc\ud611\uc758\ud68c\ub97c \uacb0\uc131\ud558\uc600\uace0, \uc7ac\uc57c\uccad\ub144\ub2e8\uccb4\uc758 \ud6a8\uc2dc\uc778 \ubbfc\uc8fc\uc218\ud638\uccad\ub144\ud611\uc758\ud68c\uc758 2\ub300 \ud68c\uc7a5\uc73c\ub85c \uc120\ucd9c\ub418\uc5c8\ub2e4. \uae40\uc218\ud658\uc73c\ub85c\ubd80\ud130 \uacbd\ubd81\uac00\ud1a8\ub9ad\ub18d\ubbfc\ud68c \uae30\ub3c4\ud68c\uc5d0\uc11c \uc778\uad8c \ubb38\uc81c\ub97c \uac15\uc5f0\ud574\ub2ec\ub77c\uace0 \uc694\uccad\uc744 \ubc1b\uc544 \ud2b9\uac15\uc744 \ud588\ub294\ub370 \uc774\ud6c4 \uae34\uae09\uc870\uce58 9\ud638 \uc704\ubc18\uc73c\ub85c \uad6c\uc18d\ub418\uc5c8\ub2e4.", "paragraph_answer": "1964\ub144\uc5d0 \ub18d\ucd0c\uc6b4\ub3d9\uac00\ub97c \uafc8\uafb8\uba70 \uc911\uc559\ub300 \ub18d\ucd0c\uc0ac\ud68c\uac1c\ubc1c\ud559\uacfc\uc5d0 \uc785\ud559\ud558\uc600\uc73c\ub098 \ud55c\uc77c\ud611\uc815\uc744 \ubc18\ub300\ud558\ub294 6.3 \ud56d\uc7c1\uc744 \uc8fc\ub3c4\ud55c \ud610\uc758\ub85c \uad6c\uc18d\ub418\uc5b4 \uc81c\uc801\ub418\uc5c8\ub2e4. \uc774\ud6c4 \uad6d\ubbfc\uc0b0\uc5c5\ud559\uad50 \ub18d\uc5c5\uacbd\uc601\ud559\uacfc\uc5d0 \uc785\ud559\ud558\uc5ec \uc878\uc5c5\ud558\uc600\uace0 \uace0\ub824\ub300 \uad50\uc721\ub300\ud559\uc6d0\uc744 \uc878\uc5c5\ud588\ub2e4. \uc774\ud6c4 \ub2e4\uc2dc \uc911\uc559\ub300 \uacbd\uc81c\ud559\uacfc\uc5d0 \ubcf5\ud559\ud558\uc5ec 1996\ub144\uc5d0 \uc878\uc5c5\ud558\uc600\ub2e4. \uad70 \ubcf5\ubb34 \uc911\uc5d0 \ud30c\uacac\uad50\uc0ac \uc2dc\ud5d8\uc5d0 \ud569\uaca9\ud558\uc5ec \uc804\uc5ed \ud6c4\uc5d0 \uad50\uc0ac\uc0dd\ud65c\uc744 \uc2dc\uc791\ud588\uc73c\uba70 1973\ub144\uc5d0\ub294 \uc7a5\ud6c8\uace0\uc5d0\uc11c, 1979\ub144\uc5d0\ub294 \uc1a1\uace1\uc5ec\uace0\uc5d0\uc11c \uad6d\uc5b4 \uad50\uc0ac\ub85c \uc7ac\uc9c1\ud588\ub2e4. \ud568\uc11d\ud5cc, \uacc4\ud6c8\uc81c, \uae40\uc218\ud658 \ub4f1\uacfc \ud568\uaed8 \ubbfc\uc8fc\uc218\ud638\uad6d\ubbfc\ud611\uc758\ud68c\ub97c \uacb0\uc131\ud558\uc600\uace0, \uc7ac\uc57c\uccad\ub144\ub2e8\uccb4\uc758 \ud6a8\uc2dc\uc778 \ubbfc\uc8fc\uc218\ud638\uccad\ub144\ud611\uc758\ud68c\uc758 2\ub300 \ud68c\uc7a5\uc73c\ub85c \uc120\ucd9c\ub418\uc5c8\ub2e4. \uae40\uc218\ud658\uc73c\ub85c\ubd80\ud130 \uacbd\ubd81\uac00\ud1a8\ub9ad\ub18d\ubbfc\ud68c \uae30\ub3c4\ud68c\uc5d0\uc11c \uc778\uad8c \ubb38\uc81c\ub97c \uac15\uc5f0\ud574\ub2ec\ub77c\uace0 \uc694\uccad\uc744 \ubc1b\uc544 \ud2b9\uac15\uc744 \ud588\ub294\ub370 \uc774\ud6c4 \uae34\uae09\uc870\uce58 9\ud638 \uc704\ubc18\uc73c\ub85c \uad6c\uc18d\ub418\uc5c8\ub2e4.", "sentence_answer": "\uc7ac\uc57c\uccad\ub144\ub2e8\uccb4\uc758 \ud6a8\uc2dc\uc778 \ubbfc\uc8fc\uc218\ud638\uccad\ub144\ud611\uc758\ud68c\uc758 2\ub300 \ud68c\uc7a5\uc73c\ub85c \uc120\ucd9c\ub418\uc5c8\ub2e4.", "paragraph_id": "6547718-0-2", "paragraph_question": "question: \uc774 \uc778\ubb3c\uc774 2\ub300 \ud68c\uc7a5\uc73c\ub85c \uc120\ucd9c\ub418\uc5c8\uc73c\uba70 \uc7ac\uc57c\uccad\ub144\ub2e8\uccb4\uc758 \ud6a8\uc2dc\ub77c \ubd88\ub9ac\ub294 \uc758\ud68c\uc758 \uc774\ub984\uc740?, context: 1964\ub144\uc5d0 \ub18d\ucd0c\uc6b4\ub3d9\uac00\ub97c \uafc8\uafb8\uba70 \uc911\uc559\ub300 \ub18d\ucd0c\uc0ac\ud68c\uac1c\ubc1c\ud559\uacfc\uc5d0 \uc785\ud559\ud558\uc600\uc73c\ub098 \ud55c\uc77c\ud611\uc815\uc744 \ubc18\ub300\ud558\ub294 6.3 \ud56d\uc7c1\uc744 \uc8fc\ub3c4\ud55c \ud610\uc758\ub85c \uad6c\uc18d\ub418\uc5b4 \uc81c\uc801\ub418\uc5c8\ub2e4. \uc774\ud6c4 \uad6d\ubbfc\uc0b0\uc5c5\ud559\uad50 \ub18d\uc5c5\uacbd\uc601\ud559\uacfc\uc5d0 \uc785\ud559\ud558\uc5ec \uc878\uc5c5\ud558\uc600\uace0 \uace0\ub824\ub300 \uad50\uc721\ub300\ud559\uc6d0\uc744 \uc878\uc5c5\ud588\ub2e4. \uc774\ud6c4 \ub2e4\uc2dc \uc911\uc559\ub300 \uacbd\uc81c\ud559\uacfc\uc5d0 \ubcf5\ud559\ud558\uc5ec 1996\ub144\uc5d0 \uc878\uc5c5\ud558\uc600\ub2e4. \uad70 \ubcf5\ubb34 \uc911\uc5d0 \ud30c\uacac\uad50\uc0ac \uc2dc\ud5d8\uc5d0 \ud569\uaca9\ud558\uc5ec \uc804\uc5ed \ud6c4\uc5d0 \uad50\uc0ac\uc0dd\ud65c\uc744 \uc2dc\uc791\ud588\uc73c\uba70 1973\ub144\uc5d0\ub294 \uc7a5\ud6c8\uace0\uc5d0\uc11c, 1979\ub144\uc5d0\ub294 \uc1a1\uace1\uc5ec\uace0\uc5d0\uc11c \uad6d\uc5b4 \uad50\uc0ac\ub85c \uc7ac\uc9c1\ud588\ub2e4. \ud568\uc11d\ud5cc, \uacc4\ud6c8\uc81c, \uae40\uc218\ud658 \ub4f1\uacfc \ud568\uaed8 \ubbfc\uc8fc\uc218\ud638\uad6d\ubbfc\ud611\uc758\ud68c\ub97c \uacb0\uc131\ud558\uc600\uace0, \uc7ac\uc57c\uccad\ub144\ub2e8\uccb4\uc758 \ud6a8\uc2dc\uc778 \ubbfc\uc8fc\uc218\ud638\uccad\ub144\ud611\uc758\ud68c\uc758 2\ub300 \ud68c\uc7a5\uc73c\ub85c \uc120\ucd9c\ub418\uc5c8\ub2e4. \uae40\uc218\ud658\uc73c\ub85c\ubd80\ud130 \uacbd\ubd81\uac00\ud1a8\ub9ad\ub18d\ubbfc\ud68c \uae30\ub3c4\ud68c\uc5d0\uc11c \uc778\uad8c \ubb38\uc81c\ub97c \uac15\uc5f0\ud574\ub2ec\ub77c\uace0 \uc694\uccad\uc744 \ubc1b\uc544 \ud2b9\uac15\uc744 \ud588\ub294\ub370 \uc774\ud6c4 \uae34\uae09\uc870\uce58 9\ud638 \uc704\ubc18\uc73c\ub85c \uad6c\uc18d\ub418\uc5c8\ub2e4."} {"question": "\ubd81\ud55c\uc758 \uc5f0\ud3c9\ub3c4 \ud3ec\uaca9\uc740 \uc804\uc7c1\ub3c4\ubc1c \ud589\uc704\ub85c \uac04\uc8fc\ub420 \uc218 \uc788\uc74c\uc744 \uba85\uc2dc\ud55c \uac83\uc740?", "paragraph": "\ub300\ud55c\ubcc0\ud638\uc0ac\ud611\ud68c \ub4f1\uc5d0 \uc758\ud558\uba74 \uc870\uc120\ubbfc\uc8fc\uc8fc\uc758\uc778\ubbfc\uacf5\ud654\uad6d\uc774 \uc5f0\ud3c9\ub3c4\uc5d0 \uc9c1\uc811 \ub300\ud3ec\uacf5\uaca9\uc744 \ud55c \uac83\uc740 \uc720\uc5d4 \ud5cc\uc7a5\uacfc \uad6d\uc81c\ubc95\uc744 \uc704\ubc30\ud55c \uc804\uc7c1\ub3c4\ubc1c \ud589\uc704\ub85c \uac04\uc8fc\ub41c\ub2e4. \uc774\uac19\uc740 \uad70\uc0ac\ub3c4\ubc1c\uc740 \ubb34\ub825\ud589\uc0ac\ub97c \uae08\uc9c0\ud558\ub294 \uc720\uc5d4 \ud5cc\uc7a5 \uc81c2\uc870 4\ud56d\uacfc \uc815\uc804\ud611\uc815 \uc704\ubc18\uc5d0 \ud574\ub2f9\ud55c\ub2e4. \uc720\uc5d4 \ud5cc\uc7a5 51\uc870\uac00 \uac1c\ubcc4 \uad6d\uac00\uc758 \uc790\uc704\uad8c(right of self-defense)\ud589\uc0ac\ub97c \uaddc\uc815\ud558\uace0 \uc788\uc9c0\ub9cc \ub300\ud55c\ubbfc\uad6d \uad6d\uad70\uc774 \uc790\uae30 \uc601\uc5ed\ub0b4\uc5d0\uc11c \uc790\uccb4 \ubc29\uc704\ud6c8\ub828\uc744 \ud558\uace0 \uc788\uc5c8\ub358 \uc0c1\ud669\uc5d0\uc11c \ubd81\ud55c \uad70\uc774 122mm \ubc29\uc0ac\ud3ec\uae4c\uc9c0 \ub3d9\uc6d0\ud558\uc5ec \ubb34\ubc29\ube44 \uc0c1\ud0dc\uc758 \ubbfc\uac04\uc778\uc9c0\uc5ed\uc744 \ubb34\ucc28\ubcc4 \ud3ec\uaca9\uc744 \uac00\ud55c \ud589\uc704\ub294 \uc790\uc704\uad8c\uc5d0 \ud574\ub2f9\ud55c\ub2e4\uace0 \ubcf4\uae30 \uc5b4\ub835\ub2e4. \ub530\ub77c\uc11c \ub300\ud55c\ubbfc\uad6d \uc815\ubd80\ub294 \uc870\uc120\ubbfc\uc8fc\uc8fc\uc758\uc778\ubbfc\uacf5\ud654\uad6d\uc758 \uc774\ubc88 \uc5f0\ud3c9\ub3c4 \ud3ec\uaca9\uacf5\uaca9\uc744 \uc720\uc5d4 \uc548\uc804\ubcf4\uc7a5\uc774\uc0ac\ud68c\uc5d0 \ud68c\ubd80\ud558\uc5ec \ucc45\uc784\uc744 \ubb3c\uc744 \uc218\ub3c4 \uc788\ub2e4.", "answer": "\uc720\uc5d4 \ud5cc\uc7a5\uacfc \uad6d\uc81c\ubc95", "sentence": "\ub300\ud55c\ubcc0\ud638\uc0ac\ud611\ud68c \ub4f1\uc5d0 \uc758\ud558\uba74 \uc870\uc120\ubbfc\uc8fc\uc8fc\uc758\uc778\ubbfc\uacf5\ud654\uad6d\uc774 \uc5f0\ud3c9\ub3c4\uc5d0 \uc9c1\uc811 \ub300\ud3ec\uacf5\uaca9\uc744 \ud55c \uac83\uc740 \uc720\uc5d4 \ud5cc\uc7a5\uacfc \uad6d\uc81c\ubc95 \uc744 \uc704\ubc30\ud55c \uc804\uc7c1\ub3c4\ubc1c \ud589\uc704\ub85c \uac04\uc8fc\ub41c\ub2e4.", "paragraph_sentence": " \ub300\ud55c\ubcc0\ud638\uc0ac\ud611\ud68c \ub4f1\uc5d0 \uc758\ud558\uba74 \uc870\uc120\ubbfc\uc8fc\uc8fc\uc758\uc778\ubbfc\uacf5\ud654\uad6d\uc774 \uc5f0\ud3c9\ub3c4\uc5d0 \uc9c1\uc811 \ub300\ud3ec\uacf5\uaca9\uc744 \ud55c \uac83\uc740 \uc720\uc5d4 \ud5cc\uc7a5\uacfc \uad6d\uc81c\ubc95 \uc744 \uc704\ubc30\ud55c \uc804\uc7c1\ub3c4\ubc1c \ud589\uc704\ub85c \uac04\uc8fc\ub41c\ub2e4. \uc774\uac19\uc740 \uad70\uc0ac\ub3c4\ubc1c\uc740 \ubb34\ub825\ud589\uc0ac\ub97c \uae08\uc9c0\ud558\ub294 \uc720\uc5d4 \ud5cc\uc7a5 \uc81c2\uc870 4\ud56d\uacfc \uc815\uc804\ud611\uc815 \uc704\ubc18\uc5d0 \ud574\ub2f9\ud55c\ub2e4. \uc720\uc5d4 \ud5cc\uc7a5 51\uc870\uac00 \uac1c\ubcc4 \uad6d\uac00\uc758 \uc790\uc704\uad8c(right of self-defense)\ud589\uc0ac\ub97c \uaddc\uc815\ud558\uace0 \uc788\uc9c0\ub9cc \ub300\ud55c\ubbfc\uad6d \uad6d\uad70\uc774 \uc790\uae30 \uc601\uc5ed\ub0b4\uc5d0\uc11c \uc790\uccb4 \ubc29\uc704\ud6c8\ub828\uc744 \ud558\uace0 \uc788\uc5c8\ub358 \uc0c1\ud669\uc5d0\uc11c \ubd81\ud55c \uad70\uc774 122mm \ubc29\uc0ac\ud3ec\uae4c\uc9c0 \ub3d9\uc6d0\ud558\uc5ec \ubb34\ubc29\ube44 \uc0c1\ud0dc\uc758 \ubbfc\uac04\uc778\uc9c0\uc5ed\uc744 \ubb34\ucc28\ubcc4 \ud3ec\uaca9\uc744 \uac00\ud55c \ud589\uc704\ub294 \uc790\uc704\uad8c\uc5d0 \ud574\ub2f9\ud55c\ub2e4\uace0 \ubcf4\uae30 \uc5b4\ub835\ub2e4. \ub530\ub77c\uc11c \ub300\ud55c\ubbfc\uad6d \uc815\ubd80\ub294 \uc870\uc120\ubbfc\uc8fc\uc8fc\uc758\uc778\ubbfc\uacf5\ud654\uad6d\uc758 \uc774\ubc88 \uc5f0\ud3c9\ub3c4 \ud3ec\uaca9\uacf5\uaca9\uc744 \uc720\uc5d4 \uc548\uc804\ubcf4\uc7a5\uc774\uc0ac\ud68c\uc5d0 \ud68c\ubd80\ud558\uc5ec \ucc45\uc784\uc744 \ubb3c\uc744 \uc218\ub3c4 \uc788\ub2e4.", "paragraph_answer": "\ub300\ud55c\ubcc0\ud638\uc0ac\ud611\ud68c \ub4f1\uc5d0 \uc758\ud558\uba74 \uc870\uc120\ubbfc\uc8fc\uc8fc\uc758\uc778\ubbfc\uacf5\ud654\uad6d\uc774 \uc5f0\ud3c9\ub3c4\uc5d0 \uc9c1\uc811 \ub300\ud3ec\uacf5\uaca9\uc744 \ud55c \uac83\uc740 \uc720\uc5d4 \ud5cc\uc7a5\uacfc \uad6d\uc81c\ubc95 \uc744 \uc704\ubc30\ud55c \uc804\uc7c1\ub3c4\ubc1c \ud589\uc704\ub85c \uac04\uc8fc\ub41c\ub2e4. \uc774\uac19\uc740 \uad70\uc0ac\ub3c4\ubc1c\uc740 \ubb34\ub825\ud589\uc0ac\ub97c \uae08\uc9c0\ud558\ub294 \uc720\uc5d4 \ud5cc\uc7a5 \uc81c2\uc870 4\ud56d\uacfc \uc815\uc804\ud611\uc815 \uc704\ubc18\uc5d0 \ud574\ub2f9\ud55c\ub2e4. \uc720\uc5d4 \ud5cc\uc7a5 51\uc870\uac00 \uac1c\ubcc4 \uad6d\uac00\uc758 \uc790\uc704\uad8c(right of self-defense)\ud589\uc0ac\ub97c \uaddc\uc815\ud558\uace0 \uc788\uc9c0\ub9cc \ub300\ud55c\ubbfc\uad6d \uad6d\uad70\uc774 \uc790\uae30 \uc601\uc5ed\ub0b4\uc5d0\uc11c \uc790\uccb4 \ubc29\uc704\ud6c8\ub828\uc744 \ud558\uace0 \uc788\uc5c8\ub358 \uc0c1\ud669\uc5d0\uc11c \ubd81\ud55c \uad70\uc774 122mm \ubc29\uc0ac\ud3ec\uae4c\uc9c0 \ub3d9\uc6d0\ud558\uc5ec \ubb34\ubc29\ube44 \uc0c1\ud0dc\uc758 \ubbfc\uac04\uc778\uc9c0\uc5ed\uc744 \ubb34\ucc28\ubcc4 \ud3ec\uaca9\uc744 \uac00\ud55c \ud589\uc704\ub294 \uc790\uc704\uad8c\uc5d0 \ud574\ub2f9\ud55c\ub2e4\uace0 \ubcf4\uae30 \uc5b4\ub835\ub2e4. \ub530\ub77c\uc11c \ub300\ud55c\ubbfc\uad6d \uc815\ubd80\ub294 \uc870\uc120\ubbfc\uc8fc\uc8fc\uc758\uc778\ubbfc\uacf5\ud654\uad6d\uc758 \uc774\ubc88 \uc5f0\ud3c9\ub3c4 \ud3ec\uaca9\uacf5\uaca9\uc744 \uc720\uc5d4 \uc548\uc804\ubcf4\uc7a5\uc774\uc0ac\ud68c\uc5d0 \ud68c\ubd80\ud558\uc5ec \ucc45\uc784\uc744 \ubb3c\uc744 \uc218\ub3c4 \uc788\ub2e4.", "sentence_answer": "\ub300\ud55c\ubcc0\ud638\uc0ac\ud611\ud68c \ub4f1\uc5d0 \uc758\ud558\uba74 \uc870\uc120\ubbfc\uc8fc\uc8fc\uc758\uc778\ubbfc\uacf5\ud654\uad6d\uc774 \uc5f0\ud3c9\ub3c4\uc5d0 \uc9c1\uc811 \ub300\ud3ec\uacf5\uaca9\uc744 \ud55c \uac83\uc740 \uc720\uc5d4 \ud5cc\uc7a5\uacfc \uad6d\uc81c\ubc95 \uc744 \uc704\ubc30\ud55c \uc804\uc7c1\ub3c4\ubc1c \ud589\uc704\ub85c \uac04\uc8fc\ub41c\ub2e4.", "paragraph_id": "6587498-5-1", "paragraph_question": "question: \ubd81\ud55c\uc758 \uc5f0\ud3c9\ub3c4 \ud3ec\uaca9\uc740 \uc804\uc7c1\ub3c4\ubc1c \ud589\uc704\ub85c \uac04\uc8fc\ub420 \uc218 \uc788\uc74c\uc744 \uba85\uc2dc\ud55c \uac83\uc740?, context: \ub300\ud55c\ubcc0\ud638\uc0ac\ud611\ud68c \ub4f1\uc5d0 \uc758\ud558\uba74 \uc870\uc120\ubbfc\uc8fc\uc8fc\uc758\uc778\ubbfc\uacf5\ud654\uad6d\uc774 \uc5f0\ud3c9\ub3c4\uc5d0 \uc9c1\uc811 \ub300\ud3ec\uacf5\uaca9\uc744 \ud55c \uac83\uc740 \uc720\uc5d4 \ud5cc\uc7a5\uacfc \uad6d\uc81c\ubc95\uc744 \uc704\ubc30\ud55c \uc804\uc7c1\ub3c4\ubc1c \ud589\uc704\ub85c \uac04\uc8fc\ub41c\ub2e4. \uc774\uac19\uc740 \uad70\uc0ac\ub3c4\ubc1c\uc740 \ubb34\ub825\ud589\uc0ac\ub97c \uae08\uc9c0\ud558\ub294 \uc720\uc5d4 \ud5cc\uc7a5 \uc81c2\uc870 4\ud56d\uacfc \uc815\uc804\ud611\uc815 \uc704\ubc18\uc5d0 \ud574\ub2f9\ud55c\ub2e4. \uc720\uc5d4 \ud5cc\uc7a5 51\uc870\uac00 \uac1c\ubcc4 \uad6d\uac00\uc758 \uc790\uc704\uad8c(right of self-defense)\ud589\uc0ac\ub97c \uaddc\uc815\ud558\uace0 \uc788\uc9c0\ub9cc \ub300\ud55c\ubbfc\uad6d \uad6d\uad70\uc774 \uc790\uae30 \uc601\uc5ed\ub0b4\uc5d0\uc11c \uc790\uccb4 \ubc29\uc704\ud6c8\ub828\uc744 \ud558\uace0 \uc788\uc5c8\ub358 \uc0c1\ud669\uc5d0\uc11c \ubd81\ud55c \uad70\uc774 122mm \ubc29\uc0ac\ud3ec\uae4c\uc9c0 \ub3d9\uc6d0\ud558\uc5ec \ubb34\ubc29\ube44 \uc0c1\ud0dc\uc758 \ubbfc\uac04\uc778\uc9c0\uc5ed\uc744 \ubb34\ucc28\ubcc4 \ud3ec\uaca9\uc744 \uac00\ud55c \ud589\uc704\ub294 \uc790\uc704\uad8c\uc5d0 \ud574\ub2f9\ud55c\ub2e4\uace0 \ubcf4\uae30 \uc5b4\ub835\ub2e4. \ub530\ub77c\uc11c \ub300\ud55c\ubbfc\uad6d \uc815\ubd80\ub294 \uc870\uc120\ubbfc\uc8fc\uc8fc\uc758\uc778\ubbfc\uacf5\ud654\uad6d\uc758 \uc774\ubc88 \uc5f0\ud3c9\ub3c4 \ud3ec\uaca9\uacf5\uaca9\uc744 \uc720\uc5d4 \uc548\uc804\ubcf4\uc7a5\uc774\uc0ac\ud68c\uc5d0 \ud68c\ubd80\ud558\uc5ec \ucc45\uc784\uc744 \ubb3c\uc744 \uc218\ub3c4 \uc788\ub2e4."} {"question": "\ubc31\uc81c \uc0c1\uc655\uc740 \uad6d\ud638\ub97c \ubb34\uc5c7\uc73c\ub85c \ubc14\uafb8\uc5c8\ub294\uac00?", "paragraph": "\ubd81\ub3d9\ucabd\uc5d0\uc11c \ub0a8\uc11c\ucabd\uc73c\ub85c \uae08\uac15\uc774 \ud750\ub974\uba70, \ube44\uc625\ud55c \ucda9\uc801\ud3c9\uc57c\ub97c \ud615\uc131\ud55c\ub2e4. \ubd80\uc5ec\ub294 \uc218\ub824\ud55c \uc790\uc5f0\uacbd\uad00\uacfc \ud568\uaed8 \ubc31\uc81c \ubb38\ud654\uc758 \uc720\ubb3c\u00b7\uc720\uc801\uc774 \uc0b0\uc7ac\ud574 \uc788\ub294 \ubb38\ud654 \uad00\uad11\uc9c0\uc774\ub2e4. \ubc31\uc81c\uc2dc\ub300 \ubd80\uc5ec\uc758 \uba85\uce6d\uc740 \uc0ac\ube44\uc774\uba70, \ubc31\uc81c \uc131\uc655\uc774 \uad6d\ud638\ub97c \ub0a8\ubd80\uc5ec(538\ub144)\ub85c \ubc14\uafb8\uace0 \uc6c5\uc9c4\uc5d0\uc11c \uc0ac\ube44\ub85c \ucc9c\ub3c4\ud55c \uc774\ud6c4 538\ub144\ubd80\ud130 660\ub144\uae4c\uc9c0 \ubc31\uc81c \uc218\ub3c4\uc600\ub2e4. \ub9e4\ub144 10\uc6d4 \uacf5\uc8fc\uc2dc\uc640 \ud568\uaed8 \ubc31\uc81c\ubb38\ud654\uc81c\ub97c \uac1c\ucd5c\ud55c\ub2e4. \uba85\uc18c\ub85c\ub294 \ubc31\uc81c\uc2dc\ub300 \ubb38\ud654\uc7ac\uc778 \ubd80\uc18c\uc0b0\uc131\uacfc \ubc31\uc81c\uc758 \ubcc4\uad81 \uc5f0\ubabb\uc774\uc5c8\ub358 \uad81\ub0a8\uc9c0 \ub4f1\uc774 \uc788\uace0, \ubc31\uc81c\ubb38\ud654\ub2e8\uc9c0\uac00 \uc870\uc131\ub418\uc5b4 \uc788\ub2e4. \ubd80\uc5ec\uc758 \uad00\ubd81\ub9ac \uc720\uc801\u00b7\ubd80\uc18c\uc0b0\uc131\uacfc \ub2a5\uc0b0\ub9ac \uace0\ubd84\uad70, \uc815\ub9bc\uc0ac\uc9c0\uc640 \ubd80\uc5ec \ub098\uc131\uc758 \ub4f1 4\uacf3\uc758 \ubc31\uc81c\uc5ed\uc0ac\uc720\uc801\uc9c0\uad6c\uac00 \uc720\ub124\uc2a4\ucf54 \uc138\uacc4\ubb38\ud654\uc720\uc0b0\uc73c\ub85c \ub4f1\uc7ac\ub418\uc5c8\ub2e4. \uad70\uccad \uc18c\uc7ac\uc9c0\ub294 \ubd80\uc5ec\uc74d\uc774\uace0, \ud589\uc815\uad6c\uc5ed\uc740 1\uc74d 15\uba74\uc774\ub2e4.", "answer": "\ub0a8\ubd80\uc5ec", "sentence": "\ubc31\uc81c\uc2dc\ub300 \ubd80\uc5ec\uc758 \uba85\uce6d\uc740 \uc0ac\ube44\uc774\uba70, \ubc31\uc81c \uc131\uc655\uc774 \uad6d\ud638\ub97c \ub0a8\ubd80\uc5ec (538\ub144)\ub85c \ubc14\uafb8\uace0 \uc6c5\uc9c4\uc5d0\uc11c \uc0ac\ube44\ub85c \ucc9c\ub3c4\ud55c \uc774\ud6c4 538\ub144\ubd80\ud130 660\ub144\uae4c\uc9c0 \ubc31\uc81c \uc218\ub3c4\uc600\ub2e4.", "paragraph_sentence": "\ubd81\ub3d9\ucabd\uc5d0\uc11c \ub0a8\uc11c\ucabd\uc73c\ub85c \uae08\uac15\uc774 \ud750\ub974\uba70, \ube44\uc625\ud55c \ucda9\uc801\ud3c9\uc57c\ub97c \ud615\uc131\ud55c\ub2e4. \ubd80\uc5ec\ub294 \uc218\ub824\ud55c \uc790\uc5f0\uacbd\uad00\uacfc \ud568\uaed8 \ubc31\uc81c \ubb38\ud654\uc758 \uc720\ubb3c\u00b7\uc720\uc801\uc774 \uc0b0\uc7ac\ud574 \uc788\ub294 \ubb38\ud654 \uad00\uad11\uc9c0\uc774\ub2e4. \ubc31\uc81c\uc2dc\ub300 \ubd80\uc5ec\uc758 \uba85\uce6d\uc740 \uc0ac\ube44\uc774\uba70, \ubc31\uc81c \uc131\uc655\uc774 \uad6d\ud638\ub97c \ub0a8\ubd80\uc5ec (538\ub144)\ub85c \ubc14\uafb8\uace0 \uc6c5\uc9c4\uc5d0\uc11c \uc0ac\ube44\ub85c \ucc9c\ub3c4\ud55c \uc774\ud6c4 538\ub144\ubd80\ud130 660\ub144\uae4c\uc9c0 \ubc31\uc81c \uc218\ub3c4\uc600\ub2e4. \ub9e4\ub144 10\uc6d4 \uacf5\uc8fc\uc2dc\uc640 \ud568\uaed8 \ubc31\uc81c\ubb38\ud654\uc81c\ub97c \uac1c\ucd5c\ud55c\ub2e4. \uba85\uc18c\ub85c\ub294 \ubc31\uc81c\uc2dc\ub300 \ubb38\ud654\uc7ac\uc778 \ubd80\uc18c\uc0b0\uc131\uacfc \ubc31\uc81c\uc758 \ubcc4\uad81 \uc5f0\ubabb\uc774\uc5c8\ub358 \uad81\ub0a8\uc9c0 \ub4f1\uc774 \uc788\uace0, \ubc31\uc81c\ubb38\ud654\ub2e8\uc9c0\uac00 \uc870\uc131\ub418\uc5b4 \uc788\ub2e4. \ubd80\uc5ec\uc758 \uad00\ubd81\ub9ac \uc720\uc801\u00b7\ubd80\uc18c\uc0b0\uc131\uacfc \ub2a5\uc0b0\ub9ac \uace0\ubd84\uad70, \uc815\ub9bc\uc0ac\uc9c0\uc640 \ubd80\uc5ec \ub098\uc131\uc758 \ub4f1 4\uacf3\uc758 \ubc31\uc81c\uc5ed\uc0ac\uc720\uc801\uc9c0\uad6c\uac00 \uc720\ub124\uc2a4\ucf54 \uc138\uacc4\ubb38\ud654\uc720\uc0b0\uc73c\ub85c \ub4f1\uc7ac\ub418\uc5c8\ub2e4. \uad70\uccad \uc18c\uc7ac\uc9c0\ub294 \ubd80\uc5ec\uc74d\uc774\uace0, \ud589\uc815\uad6c\uc5ed\uc740 1\uc74d 15\uba74\uc774\ub2e4.", "paragraph_answer": "\ubd81\ub3d9\ucabd\uc5d0\uc11c \ub0a8\uc11c\ucabd\uc73c\ub85c \uae08\uac15\uc774 \ud750\ub974\uba70, \ube44\uc625\ud55c \ucda9\uc801\ud3c9\uc57c\ub97c \ud615\uc131\ud55c\ub2e4. \ubd80\uc5ec\ub294 \uc218\ub824\ud55c \uc790\uc5f0\uacbd\uad00\uacfc \ud568\uaed8 \ubc31\uc81c \ubb38\ud654\uc758 \uc720\ubb3c\u00b7\uc720\uc801\uc774 \uc0b0\uc7ac\ud574 \uc788\ub294 \ubb38\ud654 \uad00\uad11\uc9c0\uc774\ub2e4. \ubc31\uc81c\uc2dc\ub300 \ubd80\uc5ec\uc758 \uba85\uce6d\uc740 \uc0ac\ube44\uc774\uba70, \ubc31\uc81c \uc131\uc655\uc774 \uad6d\ud638\ub97c \ub0a8\ubd80\uc5ec (538\ub144)\ub85c \ubc14\uafb8\uace0 \uc6c5\uc9c4\uc5d0\uc11c \uc0ac\ube44\ub85c \ucc9c\ub3c4\ud55c \uc774\ud6c4 538\ub144\ubd80\ud130 660\ub144\uae4c\uc9c0 \ubc31\uc81c \uc218\ub3c4\uc600\ub2e4. \ub9e4\ub144 10\uc6d4 \uacf5\uc8fc\uc2dc\uc640 \ud568\uaed8 \ubc31\uc81c\ubb38\ud654\uc81c\ub97c \uac1c\ucd5c\ud55c\ub2e4. \uba85\uc18c\ub85c\ub294 \ubc31\uc81c\uc2dc\ub300 \ubb38\ud654\uc7ac\uc778 \ubd80\uc18c\uc0b0\uc131\uacfc \ubc31\uc81c\uc758 \ubcc4\uad81 \uc5f0\ubabb\uc774\uc5c8\ub358 \uad81\ub0a8\uc9c0 \ub4f1\uc774 \uc788\uace0, \ubc31\uc81c\ubb38\ud654\ub2e8\uc9c0\uac00 \uc870\uc131\ub418\uc5b4 \uc788\ub2e4. \ubd80\uc5ec\uc758 \uad00\ubd81\ub9ac \uc720\uc801\u00b7\ubd80\uc18c\uc0b0\uc131\uacfc \ub2a5\uc0b0\ub9ac \uace0\ubd84\uad70, \uc815\ub9bc\uc0ac\uc9c0\uc640 \ubd80\uc5ec \ub098\uc131\uc758 \ub4f1 4\uacf3\uc758 \ubc31\uc81c\uc5ed\uc0ac\uc720\uc801\uc9c0\uad6c\uac00 \uc720\ub124\uc2a4\ucf54 \uc138\uacc4\ubb38\ud654\uc720\uc0b0\uc73c\ub85c \ub4f1\uc7ac\ub418\uc5c8\ub2e4. \uad70\uccad \uc18c\uc7ac\uc9c0\ub294 \ubd80\uc5ec\uc74d\uc774\uace0, \ud589\uc815\uad6c\uc5ed\uc740 1\uc74d 15\uba74\uc774\ub2e4.", "sentence_answer": "\ubc31\uc81c\uc2dc\ub300 \ubd80\uc5ec\uc758 \uba85\uce6d\uc740 \uc0ac\ube44\uc774\uba70, \ubc31\uc81c \uc131\uc655\uc774 \uad6d\ud638\ub97c \ub0a8\ubd80\uc5ec (538\ub144)\ub85c \ubc14\uafb8\uace0 \uc6c5\uc9c4\uc5d0\uc11c \uc0ac\ube44\ub85c \ucc9c\ub3c4\ud55c \uc774\ud6c4 538\ub144\ubd80\ud130 660\ub144\uae4c\uc9c0 \ubc31\uc81c \uc218\ub3c4\uc600\ub2e4.", "paragraph_id": "6582301-0-1", "paragraph_question": "question: \ubc31\uc81c \uc0c1\uc655\uc740 \uad6d\ud638\ub97c \ubb34\uc5c7\uc73c\ub85c \ubc14\uafb8\uc5c8\ub294\uac00?, context: \ubd81\ub3d9\ucabd\uc5d0\uc11c \ub0a8\uc11c\ucabd\uc73c\ub85c \uae08\uac15\uc774 \ud750\ub974\uba70, \ube44\uc625\ud55c \ucda9\uc801\ud3c9\uc57c\ub97c \ud615\uc131\ud55c\ub2e4. \ubd80\uc5ec\ub294 \uc218\ub824\ud55c \uc790\uc5f0\uacbd\uad00\uacfc \ud568\uaed8 \ubc31\uc81c \ubb38\ud654\uc758 \uc720\ubb3c\u00b7\uc720\uc801\uc774 \uc0b0\uc7ac\ud574 \uc788\ub294 \ubb38\ud654 \uad00\uad11\uc9c0\uc774\ub2e4. \ubc31\uc81c\uc2dc\ub300 \ubd80\uc5ec\uc758 \uba85\uce6d\uc740 \uc0ac\ube44\uc774\uba70, \ubc31\uc81c \uc131\uc655\uc774 \uad6d\ud638\ub97c \ub0a8\ubd80\uc5ec(538\ub144)\ub85c \ubc14\uafb8\uace0 \uc6c5\uc9c4\uc5d0\uc11c \uc0ac\ube44\ub85c \ucc9c\ub3c4\ud55c \uc774\ud6c4 538\ub144\ubd80\ud130 660\ub144\uae4c\uc9c0 \ubc31\uc81c \uc218\ub3c4\uc600\ub2e4. \ub9e4\ub144 10\uc6d4 \uacf5\uc8fc\uc2dc\uc640 \ud568\uaed8 \ubc31\uc81c\ubb38\ud654\uc81c\ub97c \uac1c\ucd5c\ud55c\ub2e4. \uba85\uc18c\ub85c\ub294 \ubc31\uc81c\uc2dc\ub300 \ubb38\ud654\uc7ac\uc778 \ubd80\uc18c\uc0b0\uc131\uacfc \ubc31\uc81c\uc758 \ubcc4\uad81 \uc5f0\ubabb\uc774\uc5c8\ub358 \uad81\ub0a8\uc9c0 \ub4f1\uc774 \uc788\uace0, \ubc31\uc81c\ubb38\ud654\ub2e8\uc9c0\uac00 \uc870\uc131\ub418\uc5b4 \uc788\ub2e4. \ubd80\uc5ec\uc758 \uad00\ubd81\ub9ac \uc720\uc801\u00b7\ubd80\uc18c\uc0b0\uc131\uacfc \ub2a5\uc0b0\ub9ac \uace0\ubd84\uad70, \uc815\ub9bc\uc0ac\uc9c0\uc640 \ubd80\uc5ec \ub098\uc131\uc758 \ub4f1 4\uacf3\uc758 \ubc31\uc81c\uc5ed\uc0ac\uc720\uc801\uc9c0\uad6c\uac00 \uc720\ub124\uc2a4\ucf54 \uc138\uacc4\ubb38\ud654\uc720\uc0b0\uc73c\ub85c \ub4f1\uc7ac\ub418\uc5c8\ub2e4. \uad70\uccad \uc18c\uc7ac\uc9c0\ub294 \ubd80\uc5ec\uc74d\uc774\uace0, \ud589\uc815\uad6c\uc5ed\uc740 1\uc74d 15\uba74\uc774\ub2e4."} {"question": "\uc804\uad6d \ud55c\uc13c\ubcd1 \ud658\uc790\ucd0c \ubc29\ubb38\ud558\uc5ec \uc790\ud65c\uc0ac\uc5c5\uc744 \uc9c0\uc6d0\ud55c \uc778\ubb3c\ub85c 1968\ub144 \uc11c\uc6b8\ub300\ud559\uad50 \uae30\uc220\uc0ac\uc758 \uc815\uc601\uc0ac\uc758 \uc124\uce58\ub97c \uc8fc\uad00\ud55c \uc0ac\ub78c\uc740 \ub204\uad6c\uc778\uac00?", "paragraph": "\uc721\uc601\uc218\ub294 \uc804\uad6d \ud55c\uc13c\ubcd1 \ud658\uc790(\ub098\ud658\uc790)\ucd0c\uc744 \uc9c1\uc811 \ubc29\ubb38\ud558\uc5ec \uc790\ud65c\uc0ac\uc5c5\uc744 \uc9c0\uc6d0\ud558\uc600\uc73c\uba70, 1964\ub144 9\uc6d4\ubd80\ud130 \ubca0\ud2b8\ub0a8\uc5d0 \ud30c\ubcd1\ub41c \ud30c\uc6d4 \uc7a5\ubcd1 \uac00\uc871\ub4e4\uc744 \ucc3e\uc544 \uc704\ub85c, \uc704\ubb38\ud558\uc600\ub2e4. \ub610\ud55c \ud48d\uc218\ud574 \ud604\uc7a5\uc5d0 \ube44\ub97c \ub9de\uc73c\uba70 \ub2ec\ub824\uac00 \uc7ac\ub09c\ub2f9\ud55c \uc0ac\ub78c\ub4e4\uc744 \uc704\ubb38\ud558\uc600\ub2e4. 1968\ub144 \uc11c\uc6b8\ub300\ud559\uad50\uc5d0 \uae30\uc219\uc0ac \uc815\uc601\uc0ac\uc758 \uc124\uce58\ub97c \uc8fc\uad00\ud588\uace0 1969\ub144 4\uc6d4 \uc5b4\ub9b0\uc774 \ubcf5\uc9c0\uc7ac\ub2e8\uc778 \uc721\uc601\uc7ac\ub2e8\uc744 \uc124\ub9bd\ud558\uc600\uc73c\uba70 \uc5b4\ub9b0\uc774\ub0a0\uc5d0 \ub9de\ucdb0 \uc5b4\ub9b0\uc774\ub300\uacf5\uc6d0, \uc5b4\ub9b0\uc774\ud68c\uad00\uc758 \uac74\ub9bd\uc744 \uc8fc\ub3c4\ud558\uc600\ub2e4. 1972\ub144\uc5d0\ub294 \ubd80\uc0b0 \uc5b4\ub9b0\uc774\ud68c\uad00\uc744 \uae30\uacf5\ud558\uc600\ub2e4. \uc721\uc601\uc218\ub294 \uc9c1\uc811 \uc18c\ub144\uc18c\ub140 \uc7a1\uc9c0 \u2018\uc5b4\uae68\ub3d9\ubb34\u2019\ub97c \ubc1c\uac04\ud558\uc5ec \ub18d\uc5b4\ucd0c \uc5b4\ub9b0\uc774\uc5d0\uac8c\uae4c\uc9c0 \ubc30\ud3ec\ud558\uace0, 1973\ub144 \ubd88\uc6b0 \uccad\uc18c\ub144\uacfc \ube48\uc790\ub4e4\uc744 \ub300\uc0c1\uc73c\ub85c \ud55c \uc9c1\uc5c5\ud6c8\ub828 \uae30\uad00 \uc815\uc218\uc9c1\uc5c5\ud6c8\ub828\uc6d0\uc744 \uc124\uce58\ud558\uc600\ub2e4. \uc721\uc601\uc218\uac00 \uc774\ub807\ub4ef \ud65c\ubc1c\ud55c \uc0ac\ud68c\ubd09\uc0ac\ub97c \ud55c \uc774\uc720\ub294 \uc9c1\uc5c5\uc744 \uac16\ub3c4\ub85d \ud558\ub294 \uc9c1\uc5c5\ud6c8\ub828 \uc704\uc8fc\uc758 \uc0ac\ud68c\ubcf5\uc9c0\uac00 \ube48\uace4\ud1f4\uce58\uc758 \uae38\uc784\uc744 \uc54c\uace0 \uc788\uc5c8\uae30 \ub54c\ubb38\uc774\ub2e4.", "answer": "\uc721\uc601\uc218", "sentence": "\uc721\uc601\uc218 \ub294 \uc804\uad6d \ud55c\uc13c\ubcd1 \ud658\uc790(\ub098\ud658\uc790)\ucd0c\uc744 \uc9c1\uc811 \ubc29\ubb38\ud558\uc5ec \uc790\ud65c\uc0ac\uc5c5\uc744 \uc9c0\uc6d0\ud558\uc600\uc73c\uba70, 1964\ub144 9\uc6d4\ubd80\ud130 \ubca0\ud2b8\ub0a8\uc5d0 \ud30c\ubcd1\ub41c \ud30c\uc6d4 \uc7a5\ubcd1 \uac00\uc871\ub4e4\uc744 \ucc3e\uc544 \uc704\ub85c, \uc704\ubb38\ud558\uc600\ub2e4.", "paragraph_sentence": " \uc721\uc601\uc218 \ub294 \uc804\uad6d \ud55c\uc13c\ubcd1 \ud658\uc790(\ub098\ud658\uc790)\ucd0c\uc744 \uc9c1\uc811 \ubc29\ubb38\ud558\uc5ec \uc790\ud65c\uc0ac\uc5c5\uc744 \uc9c0\uc6d0\ud558\uc600\uc73c\uba70, 1964\ub144 9\uc6d4\ubd80\ud130 \ubca0\ud2b8\ub0a8\uc5d0 \ud30c\ubcd1\ub41c \ud30c\uc6d4 \uc7a5\ubcd1 \uac00\uc871\ub4e4\uc744 \ucc3e\uc544 \uc704\ub85c, \uc704\ubb38\ud558\uc600\ub2e4. \ub610\ud55c \ud48d\uc218\ud574 \ud604\uc7a5\uc5d0 \ube44\ub97c \ub9de\uc73c\uba70 \ub2ec\ub824\uac00 \uc7ac\ub09c\ub2f9\ud55c \uc0ac\ub78c\ub4e4\uc744 \uc704\ubb38\ud558\uc600\ub2e4. 1968\ub144 \uc11c\uc6b8\ub300\ud559\uad50\uc5d0 \uae30\uc219\uc0ac \uc815\uc601\uc0ac\uc758 \uc124\uce58\ub97c \uc8fc\uad00\ud588\uace0 1969\ub144 4\uc6d4 \uc5b4\ub9b0\uc774 \ubcf5\uc9c0\uc7ac\ub2e8\uc778 \uc721\uc601\uc7ac\ub2e8\uc744 \uc124\ub9bd\ud558\uc600\uc73c\uba70 \uc5b4\ub9b0\uc774\ub0a0\uc5d0 \ub9de\ucdb0 \uc5b4\ub9b0\uc774\ub300\uacf5\uc6d0, \uc5b4\ub9b0\uc774\ud68c\uad00\uc758 \uac74\ub9bd\uc744 \uc8fc\ub3c4\ud558\uc600\ub2e4. 1972\ub144\uc5d0\ub294 \ubd80\uc0b0 \uc5b4\ub9b0\uc774\ud68c\uad00\uc744 \uae30\uacf5\ud558\uc600\ub2e4. \uc721\uc601\uc218\ub294 \uc9c1\uc811 \uc18c\ub144\uc18c\ub140 \uc7a1\uc9c0 \u2018\uc5b4\uae68\ub3d9\ubb34\u2019\ub97c \ubc1c\uac04\ud558\uc5ec \ub18d\uc5b4\ucd0c \uc5b4\ub9b0\uc774\uc5d0\uac8c\uae4c\uc9c0 \ubc30\ud3ec\ud558\uace0, 1973\ub144 \ubd88\uc6b0 \uccad\uc18c\ub144\uacfc \ube48\uc790\ub4e4\uc744 \ub300\uc0c1\uc73c\ub85c \ud55c \uc9c1\uc5c5\ud6c8\ub828 \uae30\uad00 \uc815\uc218\uc9c1\uc5c5\ud6c8\ub828\uc6d0\uc744 \uc124\uce58\ud558\uc600\ub2e4. \uc721\uc601\uc218\uac00 \uc774\ub807\ub4ef \ud65c\ubc1c\ud55c \uc0ac\ud68c\ubd09\uc0ac\ub97c \ud55c \uc774\uc720\ub294 \uc9c1\uc5c5\uc744 \uac16\ub3c4\ub85d \ud558\ub294 \uc9c1\uc5c5\ud6c8\ub828 \uc704\uc8fc\uc758 \uc0ac\ud68c\ubcf5\uc9c0\uac00 \ube48\uace4\ud1f4\uce58\uc758 \uae38\uc784\uc744 \uc54c\uace0 \uc788\uc5c8\uae30 \ub54c\ubb38\uc774\ub2e4.", "paragraph_answer": " \uc721\uc601\uc218 \ub294 \uc804\uad6d \ud55c\uc13c\ubcd1 \ud658\uc790(\ub098\ud658\uc790)\ucd0c\uc744 \uc9c1\uc811 \ubc29\ubb38\ud558\uc5ec \uc790\ud65c\uc0ac\uc5c5\uc744 \uc9c0\uc6d0\ud558\uc600\uc73c\uba70, 1964\ub144 9\uc6d4\ubd80\ud130 \ubca0\ud2b8\ub0a8\uc5d0 \ud30c\ubcd1\ub41c \ud30c\uc6d4 \uc7a5\ubcd1 \uac00\uc871\ub4e4\uc744 \ucc3e\uc544 \uc704\ub85c, \uc704\ubb38\ud558\uc600\ub2e4. \ub610\ud55c \ud48d\uc218\ud574 \ud604\uc7a5\uc5d0 \ube44\ub97c \ub9de\uc73c\uba70 \ub2ec\ub824\uac00 \uc7ac\ub09c\ub2f9\ud55c \uc0ac\ub78c\ub4e4\uc744 \uc704\ubb38\ud558\uc600\ub2e4. 1968\ub144 \uc11c\uc6b8\ub300\ud559\uad50\uc5d0 \uae30\uc219\uc0ac \uc815\uc601\uc0ac\uc758 \uc124\uce58\ub97c \uc8fc\uad00\ud588\uace0 1969\ub144 4\uc6d4 \uc5b4\ub9b0\uc774 \ubcf5\uc9c0\uc7ac\ub2e8\uc778 \uc721\uc601\uc7ac\ub2e8\uc744 \uc124\ub9bd\ud558\uc600\uc73c\uba70 \uc5b4\ub9b0\uc774\ub0a0\uc5d0 \ub9de\ucdb0 \uc5b4\ub9b0\uc774\ub300\uacf5\uc6d0, \uc5b4\ub9b0\uc774\ud68c\uad00\uc758 \uac74\ub9bd\uc744 \uc8fc\ub3c4\ud558\uc600\ub2e4. 1972\ub144\uc5d0\ub294 \ubd80\uc0b0 \uc5b4\ub9b0\uc774\ud68c\uad00\uc744 \uae30\uacf5\ud558\uc600\ub2e4. \uc721\uc601\uc218\ub294 \uc9c1\uc811 \uc18c\ub144\uc18c\ub140 \uc7a1\uc9c0 \u2018\uc5b4\uae68\ub3d9\ubb34\u2019\ub97c \ubc1c\uac04\ud558\uc5ec \ub18d\uc5b4\ucd0c \uc5b4\ub9b0\uc774\uc5d0\uac8c\uae4c\uc9c0 \ubc30\ud3ec\ud558\uace0, 1973\ub144 \ubd88\uc6b0 \uccad\uc18c\ub144\uacfc \ube48\uc790\ub4e4\uc744 \ub300\uc0c1\uc73c\ub85c \ud55c \uc9c1\uc5c5\ud6c8\ub828 \uae30\uad00 \uc815\uc218\uc9c1\uc5c5\ud6c8\ub828\uc6d0\uc744 \uc124\uce58\ud558\uc600\ub2e4. \uc721\uc601\uc218\uac00 \uc774\ub807\ub4ef \ud65c\ubc1c\ud55c \uc0ac\ud68c\ubd09\uc0ac\ub97c \ud55c \uc774\uc720\ub294 \uc9c1\uc5c5\uc744 \uac16\ub3c4\ub85d \ud558\ub294 \uc9c1\uc5c5\ud6c8\ub828 \uc704\uc8fc\uc758 \uc0ac\ud68c\ubcf5\uc9c0\uac00 \ube48\uace4\ud1f4\uce58\uc758 \uae38\uc784\uc744 \uc54c\uace0 \uc788\uc5c8\uae30 \ub54c\ubb38\uc774\ub2e4.", "sentence_answer": " \uc721\uc601\uc218 \ub294 \uc804\uad6d \ud55c\uc13c\ubcd1 \ud658\uc790(\ub098\ud658\uc790)\ucd0c\uc744 \uc9c1\uc811 \ubc29\ubb38\ud558\uc5ec \uc790\ud65c\uc0ac\uc5c5\uc744 \uc9c0\uc6d0\ud558\uc600\uc73c\uba70, 1964\ub144 9\uc6d4\ubd80\ud130 \ubca0\ud2b8\ub0a8\uc5d0 \ud30c\ubcd1\ub41c \ud30c\uc6d4 \uc7a5\ubcd1 \uac00\uc871\ub4e4\uc744 \ucc3e\uc544 \uc704\ub85c, \uc704\ubb38\ud558\uc600\ub2e4.", "paragraph_id": "5834598-0-0", "paragraph_question": "question: \uc804\uad6d \ud55c\uc13c\ubcd1 \ud658\uc790\ucd0c \ubc29\ubb38\ud558\uc5ec \uc790\ud65c\uc0ac\uc5c5\uc744 \uc9c0\uc6d0\ud55c \uc778\ubb3c\ub85c 1968\ub144 \uc11c\uc6b8\ub300\ud559\uad50 \uae30\uc220\uc0ac\uc758 \uc815\uc601\uc0ac\uc758 \uc124\uce58\ub97c \uc8fc\uad00\ud55c \uc0ac\ub78c\uc740 \ub204\uad6c\uc778\uac00?, context: \uc721\uc601\uc218\ub294 \uc804\uad6d \ud55c\uc13c\ubcd1 \ud658\uc790(\ub098\ud658\uc790)\ucd0c\uc744 \uc9c1\uc811 \ubc29\ubb38\ud558\uc5ec \uc790\ud65c\uc0ac\uc5c5\uc744 \uc9c0\uc6d0\ud558\uc600\uc73c\uba70, 1964\ub144 9\uc6d4\ubd80\ud130 \ubca0\ud2b8\ub0a8\uc5d0 \ud30c\ubcd1\ub41c \ud30c\uc6d4 \uc7a5\ubcd1 \uac00\uc871\ub4e4\uc744 \ucc3e\uc544 \uc704\ub85c, \uc704\ubb38\ud558\uc600\ub2e4. \ub610\ud55c \ud48d\uc218\ud574 \ud604\uc7a5\uc5d0 \ube44\ub97c \ub9de\uc73c\uba70 \ub2ec\ub824\uac00 \uc7ac\ub09c\ub2f9\ud55c \uc0ac\ub78c\ub4e4\uc744 \uc704\ubb38\ud558\uc600\ub2e4. 1968\ub144 \uc11c\uc6b8\ub300\ud559\uad50\uc5d0 \uae30\uc219\uc0ac \uc815\uc601\uc0ac\uc758 \uc124\uce58\ub97c \uc8fc\uad00\ud588\uace0 1969\ub144 4\uc6d4 \uc5b4\ub9b0\uc774 \ubcf5\uc9c0\uc7ac\ub2e8\uc778 \uc721\uc601\uc7ac\ub2e8\uc744 \uc124\ub9bd\ud558\uc600\uc73c\uba70 \uc5b4\ub9b0\uc774\ub0a0\uc5d0 \ub9de\ucdb0 \uc5b4\ub9b0\uc774\ub300\uacf5\uc6d0, \uc5b4\ub9b0\uc774\ud68c\uad00\uc758 \uac74\ub9bd\uc744 \uc8fc\ub3c4\ud558\uc600\ub2e4. 1972\ub144\uc5d0\ub294 \ubd80\uc0b0 \uc5b4\ub9b0\uc774\ud68c\uad00\uc744 \uae30\uacf5\ud558\uc600\ub2e4. \uc721\uc601\uc218\ub294 \uc9c1\uc811 \uc18c\ub144\uc18c\ub140 \uc7a1\uc9c0 \u2018\uc5b4\uae68\ub3d9\ubb34\u2019\ub97c \ubc1c\uac04\ud558\uc5ec \ub18d\uc5b4\ucd0c \uc5b4\ub9b0\uc774\uc5d0\uac8c\uae4c\uc9c0 \ubc30\ud3ec\ud558\uace0, 1973\ub144 \ubd88\uc6b0 \uccad\uc18c\ub144\uacfc \ube48\uc790\ub4e4\uc744 \ub300\uc0c1\uc73c\ub85c \ud55c \uc9c1\uc5c5\ud6c8\ub828 \uae30\uad00 \uc815\uc218\uc9c1\uc5c5\ud6c8\ub828\uc6d0\uc744 \uc124\uce58\ud558\uc600\ub2e4. \uc721\uc601\uc218\uac00 \uc774\ub807\ub4ef \ud65c\ubc1c\ud55c \uc0ac\ud68c\ubd09\uc0ac\ub97c \ud55c \uc774\uc720\ub294 \uc9c1\uc5c5\uc744 \uac16\ub3c4\ub85d \ud558\ub294 \uc9c1\uc5c5\ud6c8\ub828 \uc704\uc8fc\uc758 \uc0ac\ud68c\ubcf5\uc9c0\uac00 \ube48\uace4\ud1f4\uce58\uc758 \uae38\uc784\uc744 \uc54c\uace0 \uc788\uc5c8\uae30 \ub54c\ubb38\uc774\ub2e4."} {"question": "1947\ub144 \ud55c \ucd08\ub4f1\ud559\uad50\uc5d0\uc11c \uc5b4\ub9b0\uc544\uc774\uac00 \ub9d0\ubc1c\uad7d\uc5d0 \ucc28\uc774\ub294 \uc77c\uc774 \ubc8c\uc5b4\uc84c\ub294\ub370 \uc5b4\ub5a4 \uae30\ub150\uc2dd\uc5d0\uc11c \ubc8c\uc5b4\uc84c\ub294\uac00?", "paragraph": "\uadf8\ub7ec\ub098, 1947\ub144 \uc81c\uc8fc \ubd81\ucd08\ub4f1\ud559\uad50 3.1\uc808 \uae30\ub150\uc2dd\uc5d0\uc11c \uae30\ub9c8\uacbd\uad00\uc758 \ub9d0\ubc1c\uad7d\uc5d0 \uc5b4\ub9b0\uc544\uc774\uac00 \uce58\uc774\ub294 \uc77c\uc774 \ubc8c\uc5b4\uc84c\uace0, \uc774\ub97c \ubcf8 \uc2dc\uc704\uad70\uc911\ub4e4\uc740 \uae30\ub9c8\uacbd\uad00\uc5d0\uac8c \ub3cc\uc744 \ub358\uc9c0\uace0 \uc57c\uc720\ub97c \ubcf4\ub0b4\uba70 \uacbd\ucc30\uc11c\uae4c\uc9c0 \ucad3\uc544\uac14\ub2e4. \uadf8\ub7f0\ub370 \uacbd\ucc30\uc774 \uc774\ub97c \uacbd\ucc30\uc11c \uc2b5\uaca9\uc73c\ub85c \uc624\uc778\ud558\uc5ec \uc2dc\uc704\ub300\uc5d0\uac8c \ubc1c\ud3ec\ud574 6\uba85\uc774 \uc0ac\ub9dd\ud558\uace0 6\uba85\uc774 \uc911\uc0c1\uc744 \uc785\uc5c8\ub2e4. \ubc1c\ud3ec\uc0ac\uac74\uc758 \uc804\ubaa8\ub97c \ubaa8\ub974\ub358 \ubbf8\uad70\uc815 \ub2f9\uad6d\uc740 \uc774 \ubc1c\ud3ec\uc0ac\uac74\uc744 \uc798\ubabb\uc744 \uc2dc\uc778\ud558\uba74\uc11c\ub3c4 \uc815\ub2f9\ubc29\uc704\ub85c \uc8fc\uc7a5\ud558\uace0 \uc0ac\uac74\uc744 '\uc2dc\uc704\ub300\uc5d0 \uc758\ud55c \uacbd\ucc30\uc11c \uc2b5\uaca9\uc0ac\uac74'\uc73c\ub85c \uaddc\uc815\uc9d3\uace0 3-1\uc808 \uae30\ub150 \ud589\uc0ac\ub97c \uc900\ube44\ud55c \uc0ac\ub78c\ub4e4\uc744 \uc5f0\ud589\ud558\uae30 \uc2dc\uc791\ud588\ub2e4. \ud55c\ud3b8 \uacbd\ubb34\ubd80\uc5d0\uc11c\ub294 3\ub9cc\uc5ec \uc2dc\uc704\uad70\uc911\uc774 \uacbd\ucc30\uc11c\ub97c \ud3ec\uc704 \uc2b5\uaca9\ud558\ub824\uace0 \ud588\uae30\uc5d0 \ubd88\uac00\ud53c\ud558\uac8c \ubc1c\ud3ec\ud588\ub2e4\uace0 \ud574\uba85\ud558\uba74\uc11c \ubbfc\uc2ec\uc774 \ub4e4\ub053\uc5c8\ub2e4. \uc774\uc5d0 \ub0a8\ub85c\ub2f9\uc740 \uc774\ub7f0 \ubbfc\uc2ec\uc758 \ud750\ub984\uc744 \ub193\uce58\uc9c0 \uc54a\uace0 \uc870\uc9c1\uc801\uc778 \ubc18\uacbd\ud65c\ub3d9\uc744 \uc804\uac1c\ud588\ub2e4. \ucc98\uc74c\uc5d0\ub294 \uc804\ub2e8\uc9c0\ub97c \ubd99\uc774\ub294 \uc77c\uacfc \uc0ac\uc0c1\uc790 \uad6c\ud638\uae08 \ubaa8\uae08\uc6b4\ub3d9\uc744 \ubc8c\uc600\ub2e4.", "answer": "3.1\uc808 \uae30\ub150\uc2dd", "sentence": "\uadf8\ub7ec\ub098, 1947\ub144 \uc81c\uc8fc \ubd81\ucd08\ub4f1\ud559\uad50 3.1\uc808 \uae30\ub150\uc2dd \uc5d0\uc11c \uae30\ub9c8\uacbd\uad00\uc758 \ub9d0\ubc1c\uad7d\uc5d0 \uc5b4\ub9b0\uc544\uc774\uac00 \uce58\uc774\ub294 \uc77c\uc774 \ubc8c\uc5b4\uc84c\uace0, \uc774\ub97c \ubcf8 \uc2dc\uc704\uad70\uc911\ub4e4\uc740 \uae30\ub9c8\uacbd\uad00\uc5d0\uac8c \ub3cc\uc744 \ub358\uc9c0\uace0 \uc57c\uc720\ub97c \ubcf4\ub0b4\uba70 \uacbd\ucc30\uc11c\uae4c\uc9c0 \ucad3\uc544\uac14\ub2e4.", "paragraph_sentence": " \uadf8\ub7ec\ub098, 1947\ub144 \uc81c\uc8fc \ubd81\ucd08\ub4f1\ud559\uad50 3.1\uc808 \uae30\ub150\uc2dd \uc5d0\uc11c \uae30\ub9c8\uacbd\uad00\uc758 \ub9d0\ubc1c\uad7d\uc5d0 \uc5b4\ub9b0\uc544\uc774\uac00 \uce58\uc774\ub294 \uc77c\uc774 \ubc8c\uc5b4\uc84c\uace0, \uc774\ub97c \ubcf8 \uc2dc\uc704\uad70\uc911\ub4e4\uc740 \uae30\ub9c8\uacbd\uad00\uc5d0\uac8c \ub3cc\uc744 \ub358\uc9c0\uace0 \uc57c\uc720\ub97c \ubcf4\ub0b4\uba70 \uacbd\ucc30\uc11c\uae4c\uc9c0 \ucad3\uc544\uac14\ub2e4. \uadf8\ub7f0\ub370 \uacbd\ucc30\uc774 \uc774\ub97c \uacbd\ucc30\uc11c \uc2b5\uaca9\uc73c\ub85c \uc624\uc778\ud558\uc5ec \uc2dc\uc704\ub300\uc5d0\uac8c \ubc1c\ud3ec\ud574 6\uba85\uc774 \uc0ac\ub9dd\ud558\uace0 6\uba85\uc774 \uc911\uc0c1\uc744 \uc785\uc5c8\ub2e4. \ubc1c\ud3ec\uc0ac\uac74\uc758 \uc804\ubaa8\ub97c \ubaa8\ub974\ub358 \ubbf8\uad70\uc815 \ub2f9\uad6d\uc740 \uc774 \ubc1c\ud3ec\uc0ac\uac74\uc744 \uc798\ubabb\uc744 \uc2dc\uc778\ud558\uba74\uc11c\ub3c4 \uc815\ub2f9\ubc29\uc704\ub85c \uc8fc\uc7a5\ud558\uace0 \uc0ac\uac74\uc744 '\uc2dc\uc704\ub300\uc5d0 \uc758\ud55c \uacbd\ucc30\uc11c \uc2b5\uaca9\uc0ac\uac74'\uc73c\ub85c \uaddc\uc815\uc9d3\uace0 3-1\uc808 \uae30\ub150 \ud589\uc0ac\ub97c \uc900\ube44\ud55c \uc0ac\ub78c\ub4e4\uc744 \uc5f0\ud589\ud558\uae30 \uc2dc\uc791\ud588\ub2e4. \ud55c\ud3b8 \uacbd\ubb34\ubd80\uc5d0\uc11c\ub294 3\ub9cc\uc5ec \uc2dc\uc704\uad70\uc911\uc774 \uacbd\ucc30\uc11c\ub97c \ud3ec\uc704 \uc2b5\uaca9\ud558\ub824\uace0 \ud588\uae30\uc5d0 \ubd88\uac00\ud53c\ud558\uac8c \ubc1c\ud3ec\ud588\ub2e4\uace0 \ud574\uba85\ud558\uba74\uc11c \ubbfc\uc2ec\uc774 \ub4e4\ub053\uc5c8\ub2e4. \uc774\uc5d0 \ub0a8\ub85c\ub2f9\uc740 \uc774\ub7f0 \ubbfc\uc2ec\uc758 \ud750\ub984\uc744 \ub193\uce58\uc9c0 \uc54a\uace0 \uc870\uc9c1\uc801\uc778 \ubc18\uacbd\ud65c\ub3d9\uc744 \uc804\uac1c\ud588\ub2e4. \ucc98\uc74c\uc5d0\ub294 \uc804\ub2e8\uc9c0\ub97c \ubd99\uc774\ub294 \uc77c\uacfc \uc0ac\uc0c1\uc790 \uad6c\ud638\uae08 \ubaa8\uae08\uc6b4\ub3d9\uc744 \ubc8c\uc600\ub2e4.", "paragraph_answer": "\uadf8\ub7ec\ub098, 1947\ub144 \uc81c\uc8fc \ubd81\ucd08\ub4f1\ud559\uad50 3.1\uc808 \uae30\ub150\uc2dd \uc5d0\uc11c \uae30\ub9c8\uacbd\uad00\uc758 \ub9d0\ubc1c\uad7d\uc5d0 \uc5b4\ub9b0\uc544\uc774\uac00 \uce58\uc774\ub294 \uc77c\uc774 \ubc8c\uc5b4\uc84c\uace0, \uc774\ub97c \ubcf8 \uc2dc\uc704\uad70\uc911\ub4e4\uc740 \uae30\ub9c8\uacbd\uad00\uc5d0\uac8c \ub3cc\uc744 \ub358\uc9c0\uace0 \uc57c\uc720\ub97c \ubcf4\ub0b4\uba70 \uacbd\ucc30\uc11c\uae4c\uc9c0 \ucad3\uc544\uac14\ub2e4. \uadf8\ub7f0\ub370 \uacbd\ucc30\uc774 \uc774\ub97c \uacbd\ucc30\uc11c \uc2b5\uaca9\uc73c\ub85c \uc624\uc778\ud558\uc5ec \uc2dc\uc704\ub300\uc5d0\uac8c \ubc1c\ud3ec\ud574 6\uba85\uc774 \uc0ac\ub9dd\ud558\uace0 6\uba85\uc774 \uc911\uc0c1\uc744 \uc785\uc5c8\ub2e4. \ubc1c\ud3ec\uc0ac\uac74\uc758 \uc804\ubaa8\ub97c \ubaa8\ub974\ub358 \ubbf8\uad70\uc815 \ub2f9\uad6d\uc740 \uc774 \ubc1c\ud3ec\uc0ac\uac74\uc744 \uc798\ubabb\uc744 \uc2dc\uc778\ud558\uba74\uc11c\ub3c4 \uc815\ub2f9\ubc29\uc704\ub85c \uc8fc\uc7a5\ud558\uace0 \uc0ac\uac74\uc744 '\uc2dc\uc704\ub300\uc5d0 \uc758\ud55c \uacbd\ucc30\uc11c \uc2b5\uaca9\uc0ac\uac74'\uc73c\ub85c \uaddc\uc815\uc9d3\uace0 3-1\uc808 \uae30\ub150 \ud589\uc0ac\ub97c \uc900\ube44\ud55c \uc0ac\ub78c\ub4e4\uc744 \uc5f0\ud589\ud558\uae30 \uc2dc\uc791\ud588\ub2e4. \ud55c\ud3b8 \uacbd\ubb34\ubd80\uc5d0\uc11c\ub294 3\ub9cc\uc5ec \uc2dc\uc704\uad70\uc911\uc774 \uacbd\ucc30\uc11c\ub97c \ud3ec\uc704 \uc2b5\uaca9\ud558\ub824\uace0 \ud588\uae30\uc5d0 \ubd88\uac00\ud53c\ud558\uac8c \ubc1c\ud3ec\ud588\ub2e4\uace0 \ud574\uba85\ud558\uba74\uc11c \ubbfc\uc2ec\uc774 \ub4e4\ub053\uc5c8\ub2e4. \uc774\uc5d0 \ub0a8\ub85c\ub2f9\uc740 \uc774\ub7f0 \ubbfc\uc2ec\uc758 \ud750\ub984\uc744 \ub193\uce58\uc9c0 \uc54a\uace0 \uc870\uc9c1\uc801\uc778 \ubc18\uacbd\ud65c\ub3d9\uc744 \uc804\uac1c\ud588\ub2e4. \ucc98\uc74c\uc5d0\ub294 \uc804\ub2e8\uc9c0\ub97c \ubd99\uc774\ub294 \uc77c\uacfc \uc0ac\uc0c1\uc790 \uad6c\ud638\uae08 \ubaa8\uae08\uc6b4\ub3d9\uc744 \ubc8c\uc600\ub2e4.", "sentence_answer": "\uadf8\ub7ec\ub098, 1947\ub144 \uc81c\uc8fc \ubd81\ucd08\ub4f1\ud559\uad50 3.1\uc808 \uae30\ub150\uc2dd \uc5d0\uc11c \uae30\ub9c8\uacbd\uad00\uc758 \ub9d0\ubc1c\uad7d\uc5d0 \uc5b4\ub9b0\uc544\uc774\uac00 \uce58\uc774\ub294 \uc77c\uc774 \ubc8c\uc5b4\uc84c\uace0, \uc774\ub97c \ubcf8 \uc2dc\uc704\uad70\uc911\ub4e4\uc740 \uae30\ub9c8\uacbd\uad00\uc5d0\uac8c \ub3cc\uc744 \ub358\uc9c0\uace0 \uc57c\uc720\ub97c \ubcf4\ub0b4\uba70 \uacbd\ucc30\uc11c\uae4c\uc9c0 \ucad3\uc544\uac14\ub2e4.", "paragraph_id": "5871990-2-0", "paragraph_question": "question: 1947\ub144 \ud55c \ucd08\ub4f1\ud559\uad50\uc5d0\uc11c \uc5b4\ub9b0\uc544\uc774\uac00 \ub9d0\ubc1c\uad7d\uc5d0 \ucc28\uc774\ub294 \uc77c\uc774 \ubc8c\uc5b4\uc84c\ub294\ub370 \uc5b4\ub5a4 \uae30\ub150\uc2dd\uc5d0\uc11c \ubc8c\uc5b4\uc84c\ub294\uac00?, context: \uadf8\ub7ec\ub098, 1947\ub144 \uc81c\uc8fc \ubd81\ucd08\ub4f1\ud559\uad50 3.1\uc808 \uae30\ub150\uc2dd\uc5d0\uc11c \uae30\ub9c8\uacbd\uad00\uc758 \ub9d0\ubc1c\uad7d\uc5d0 \uc5b4\ub9b0\uc544\uc774\uac00 \uce58\uc774\ub294 \uc77c\uc774 \ubc8c\uc5b4\uc84c\uace0, \uc774\ub97c \ubcf8 \uc2dc\uc704\uad70\uc911\ub4e4\uc740 \uae30\ub9c8\uacbd\uad00\uc5d0\uac8c \ub3cc\uc744 \ub358\uc9c0\uace0 \uc57c\uc720\ub97c \ubcf4\ub0b4\uba70 \uacbd\ucc30\uc11c\uae4c\uc9c0 \ucad3\uc544\uac14\ub2e4. \uadf8\ub7f0\ub370 \uacbd\ucc30\uc774 \uc774\ub97c \uacbd\ucc30\uc11c \uc2b5\uaca9\uc73c\ub85c \uc624\uc778\ud558\uc5ec \uc2dc\uc704\ub300\uc5d0\uac8c \ubc1c\ud3ec\ud574 6\uba85\uc774 \uc0ac\ub9dd\ud558\uace0 6\uba85\uc774 \uc911\uc0c1\uc744 \uc785\uc5c8\ub2e4. \ubc1c\ud3ec\uc0ac\uac74\uc758 \uc804\ubaa8\ub97c \ubaa8\ub974\ub358 \ubbf8\uad70\uc815 \ub2f9\uad6d\uc740 \uc774 \ubc1c\ud3ec\uc0ac\uac74\uc744 \uc798\ubabb\uc744 \uc2dc\uc778\ud558\uba74\uc11c\ub3c4 \uc815\ub2f9\ubc29\uc704\ub85c \uc8fc\uc7a5\ud558\uace0 \uc0ac\uac74\uc744 '\uc2dc\uc704\ub300\uc5d0 \uc758\ud55c \uacbd\ucc30\uc11c \uc2b5\uaca9\uc0ac\uac74'\uc73c\ub85c \uaddc\uc815\uc9d3\uace0 3-1\uc808 \uae30\ub150 \ud589\uc0ac\ub97c \uc900\ube44\ud55c \uc0ac\ub78c\ub4e4\uc744 \uc5f0\ud589\ud558\uae30 \uc2dc\uc791\ud588\ub2e4. \ud55c\ud3b8 \uacbd\ubb34\ubd80\uc5d0\uc11c\ub294 3\ub9cc\uc5ec \uc2dc\uc704\uad70\uc911\uc774 \uacbd\ucc30\uc11c\ub97c \ud3ec\uc704 \uc2b5\uaca9\ud558\ub824\uace0 \ud588\uae30\uc5d0 \ubd88\uac00\ud53c\ud558\uac8c \ubc1c\ud3ec\ud588\ub2e4\uace0 \ud574\uba85\ud558\uba74\uc11c \ubbfc\uc2ec\uc774 \ub4e4\ub053\uc5c8\ub2e4. \uc774\uc5d0 \ub0a8\ub85c\ub2f9\uc740 \uc774\ub7f0 \ubbfc\uc2ec\uc758 \ud750\ub984\uc744 \ub193\uce58\uc9c0 \uc54a\uace0 \uc870\uc9c1\uc801\uc778 \ubc18\uacbd\ud65c\ub3d9\uc744 \uc804\uac1c\ud588\ub2e4. \ucc98\uc74c\uc5d0\ub294 \uc804\ub2e8\uc9c0\ub97c \ubd99\uc774\ub294 \uc77c\uacfc \uc0ac\uc0c1\uc790 \uad6c\ud638\uae08 \ubaa8\uae08\uc6b4\ub3d9\uc744 \ubc8c\uc600\ub2e4."} {"question": "\ucd1d\uc5ec\ud559\uc0dd\ud68c\uc758 \uba85\uce6d\uc740?", "paragraph": "\ucd1d\uc5ec\ud559\uc0dd\ud68c\ub294 \uc5ec\uc131 \ubb38\uc81c\ub97c \uace0\ubbfc\ud558\uace0 \ub0a8\uc131\uc911\uc2ec\uc801 \ub300\ud559 \ubb38\ud654\ub97c \uc5ec\uc131\uc8fc\uc758\uc801\uc73c\ub85c \uc7ac\uad6c\uc131\ud558\uae30 \uc704\ud574\uc11c \ud65c\ub3d9\ud558\uace0 \uc788\ub2e4. \ucd1d\uc5ec\ud559\uc0dd\ud68c\ub294 1988\ub144 3\uc6d4\uc5d0 \uae30\uc874 \ucd1d\ud559\uc0dd\ud68c \uc0b0\ud558 \uc5ec\ud559\uc0dd\ubd80\uc758 \ud55c\uacc4\uc640 \ud559\ub0b4\uc5d0\uc11c \uc18c\uc218\uc758 \uc704\uce58\uc5d0 \uc788\ub358 \uc5ec\ud559\uc0dd \ubb38\uc81c\ub97c \uadf9\ubcf5\ud558\uace0\uc790 \ubc1c\uc871\ud558\uc600\ub2e4. \ub9e4\ub144 \uc120\uac70\ub97c \ud1b5\ud574 \ucd1d\uc5ec\ud559\uc0dd\ud68c\uac00 \uc120\ucd9c\ub418\uace0 \uc788\uc73c\uba70 2016\ub144 \uae30\uc900\uc73c\ub85c \uc81c27\ub300 \ucd1d\uc5ec\ud559\uc0dd\ud68c \u2018\uc787\ub2e4\u2019\uac00 \ud65c\ub3d9\ud558\uace0 \uc788\ub2e4. \ucd1d\uc5ec\ud559\uc0dd\ud68c\ub294 \ud0c4\uc0dd \uc774\ub798 \uc704\uc0c1\uc774 \ubaa8\ud638\ud558\ub2e4\ub294 \ube44\ud310\uc744 \ubc1b\uc544\uc654\uc9c0\ub9cc \ub9e4\ub144 \ub0a8\uc131 \uc911\uc2ec\uc758 \ucea0\ud37c\uc2a4 \ubb38\ud654\ub97c \uadf9\ubcf5\ud558\uae30 \uc704\ud55c \ud589\uc0ac\ub97c \uc5f4\uc5b4\uc654\uace0 2000\ub144 2\uc6d4\ubd80\ud130\ub294 \u2018\uc0c8\ub0b4\uae30\ubc30\uc6c0\ud130 \ubc18\uc131\ud3ed\ub825 \ub0b4\uaddc\u2019\ub97c \uc81c\uc815\ud574 \uc2dc\ud589\ud558\uae30\ub3c4 \ud558\uc600\ub2e4. \uc774\uc640 \ub354\ubd88\uc5b4 \uc5ec\ud559\uc0dd\ucc98\ub294 1956\ub144\uc5d0 \uc124\ub9bd\ub41c \uc774\ub798 \uc131\ud3ed\ub825\uc0c1\ub2f4\uc2e4 \uac1c\uc124, \ubc18\uc131\ud3ed\ub825\ud559\uce59 \uc81c\uc815 \ub4f1 \ub0a8\ub140\ud3c9\ub4f1\uc801 \uad50\uc721 \ud658\uacbd\uc744 \ub9c8\ub828\ud558\uae30 \uc704\ud574 \ub178\ub825\ud558\uc600\ub2e4. 2002\ub144\uc5d0\ub294 \uc5ec\uc131\uc778\ub825\uac1c\ubc1c\uc5f0\uad6c\uc6d0\uc774 \ubc1c\uc871\ub418\uc5b4 \uc5ec\ud559\uc0dd\ub4e4\uc758 \uc0ac\ud68c \uc9c4\ucd9c \ub4f1\uc744 \uc704\ud55c \ud559\ub0b4\uc678\uc801\uc778 \ud65c\ub3d9\uc744 \uc218\ud589\ud558\uace0 \uc788\ub2e4.", "answer": "\uc787\ub2e4", "sentence": "\ub9e4\ub144 \uc120\uac70\ub97c \ud1b5\ud574 \ucd1d\uc5ec\ud559\uc0dd\ud68c\uac00 \uc120\ucd9c\ub418\uace0 \uc788\uc73c\uba70 2016\ub144 \uae30\uc900\uc73c\ub85c \uc81c27\ub300 \ucd1d\uc5ec\ud559\uc0dd\ud68c \u2018 \uc787\ub2e4 \u2019\uac00 \ud65c\ub3d9\ud558\uace0 \uc788\ub2e4.", "paragraph_sentence": "\ucd1d\uc5ec\ud559\uc0dd\ud68c\ub294 \uc5ec\uc131 \ubb38\uc81c\ub97c \uace0\ubbfc\ud558\uace0 \ub0a8\uc131\uc911\uc2ec\uc801 \ub300\ud559 \ubb38\ud654\ub97c \uc5ec\uc131\uc8fc\uc758\uc801\uc73c\ub85c \uc7ac\uad6c\uc131\ud558\uae30 \uc704\ud574\uc11c \ud65c\ub3d9\ud558\uace0 \uc788\ub2e4. \ucd1d\uc5ec\ud559\uc0dd\ud68c\ub294 1988\ub144 3\uc6d4\uc5d0 \uae30\uc874 \ucd1d\ud559\uc0dd\ud68c \uc0b0\ud558 \uc5ec\ud559\uc0dd\ubd80\uc758 \ud55c\uacc4\uc640 \ud559\ub0b4\uc5d0\uc11c \uc18c\uc218\uc758 \uc704\uce58\uc5d0 \uc788\ub358 \uc5ec\ud559\uc0dd \ubb38\uc81c\ub97c \uadf9\ubcf5\ud558\uace0\uc790 \ubc1c\uc871\ud558\uc600\ub2e4. \ub9e4\ub144 \uc120\uac70\ub97c \ud1b5\ud574 \ucd1d\uc5ec\ud559\uc0dd\ud68c\uac00 \uc120\ucd9c\ub418\uace0 \uc788\uc73c\uba70 2016\ub144 \uae30\uc900\uc73c\ub85c \uc81c27\ub300 \ucd1d\uc5ec\ud559\uc0dd\ud68c \u2018 \uc787\ub2e4 \u2019\uac00 \ud65c\ub3d9\ud558\uace0 \uc788\ub2e4. \ucd1d\uc5ec\ud559\uc0dd\ud68c\ub294 \ud0c4\uc0dd \uc774\ub798 \uc704\uc0c1\uc774 \ubaa8\ud638\ud558\ub2e4\ub294 \ube44\ud310\uc744 \ubc1b\uc544\uc654\uc9c0\ub9cc \ub9e4\ub144 \ub0a8\uc131 \uc911\uc2ec\uc758 \ucea0\ud37c\uc2a4 \ubb38\ud654\ub97c \uadf9\ubcf5\ud558\uae30 \uc704\ud55c \ud589\uc0ac\ub97c \uc5f4\uc5b4\uc654\uace0 2000\ub144 2\uc6d4\ubd80\ud130\ub294 \u2018\uc0c8\ub0b4\uae30\ubc30\uc6c0\ud130 \ubc18\uc131\ud3ed\ub825 \ub0b4\uaddc\u2019\ub97c \uc81c\uc815\ud574 \uc2dc\ud589\ud558\uae30\ub3c4 \ud558\uc600\ub2e4. \uc774\uc640 \ub354\ubd88\uc5b4 \uc5ec\ud559\uc0dd\ucc98\ub294 1956\ub144\uc5d0 \uc124\ub9bd\ub41c \uc774\ub798 \uc131\ud3ed\ub825\uc0c1\ub2f4\uc2e4 \uac1c\uc124, \ubc18\uc131\ud3ed\ub825\ud559\uce59 \uc81c\uc815 \ub4f1 \ub0a8\ub140\ud3c9\ub4f1\uc801 \uad50\uc721 \ud658\uacbd\uc744 \ub9c8\ub828\ud558\uae30 \uc704\ud574 \ub178\ub825\ud558\uc600\ub2e4. 2002\ub144\uc5d0\ub294 \uc5ec\uc131\uc778\ub825\uac1c\ubc1c\uc5f0\uad6c\uc6d0\uc774 \ubc1c\uc871\ub418\uc5b4 \uc5ec\ud559\uc0dd\ub4e4\uc758 \uc0ac\ud68c \uc9c4\ucd9c \ub4f1\uc744 \uc704\ud55c \ud559\ub0b4\uc678\uc801\uc778 \ud65c\ub3d9\uc744 \uc218\ud589\ud558\uace0 \uc788\ub2e4.", "paragraph_answer": "\ucd1d\uc5ec\ud559\uc0dd\ud68c\ub294 \uc5ec\uc131 \ubb38\uc81c\ub97c \uace0\ubbfc\ud558\uace0 \ub0a8\uc131\uc911\uc2ec\uc801 \ub300\ud559 \ubb38\ud654\ub97c \uc5ec\uc131\uc8fc\uc758\uc801\uc73c\ub85c \uc7ac\uad6c\uc131\ud558\uae30 \uc704\ud574\uc11c \ud65c\ub3d9\ud558\uace0 \uc788\ub2e4. \ucd1d\uc5ec\ud559\uc0dd\ud68c\ub294 1988\ub144 3\uc6d4\uc5d0 \uae30\uc874 \ucd1d\ud559\uc0dd\ud68c \uc0b0\ud558 \uc5ec\ud559\uc0dd\ubd80\uc758 \ud55c\uacc4\uc640 \ud559\ub0b4\uc5d0\uc11c \uc18c\uc218\uc758 \uc704\uce58\uc5d0 \uc788\ub358 \uc5ec\ud559\uc0dd \ubb38\uc81c\ub97c \uadf9\ubcf5\ud558\uace0\uc790 \ubc1c\uc871\ud558\uc600\ub2e4. \ub9e4\ub144 \uc120\uac70\ub97c \ud1b5\ud574 \ucd1d\uc5ec\ud559\uc0dd\ud68c\uac00 \uc120\ucd9c\ub418\uace0 \uc788\uc73c\uba70 2016\ub144 \uae30\uc900\uc73c\ub85c \uc81c27\ub300 \ucd1d\uc5ec\ud559\uc0dd\ud68c \u2018 \uc787\ub2e4 \u2019\uac00 \ud65c\ub3d9\ud558\uace0 \uc788\ub2e4. \ucd1d\uc5ec\ud559\uc0dd\ud68c\ub294 \ud0c4\uc0dd \uc774\ub798 \uc704\uc0c1\uc774 \ubaa8\ud638\ud558\ub2e4\ub294 \ube44\ud310\uc744 \ubc1b\uc544\uc654\uc9c0\ub9cc \ub9e4\ub144 \ub0a8\uc131 \uc911\uc2ec\uc758 \ucea0\ud37c\uc2a4 \ubb38\ud654\ub97c \uadf9\ubcf5\ud558\uae30 \uc704\ud55c \ud589\uc0ac\ub97c \uc5f4\uc5b4\uc654\uace0 2000\ub144 2\uc6d4\ubd80\ud130\ub294 \u2018\uc0c8\ub0b4\uae30\ubc30\uc6c0\ud130 \ubc18\uc131\ud3ed\ub825 \ub0b4\uaddc\u2019\ub97c \uc81c\uc815\ud574 \uc2dc\ud589\ud558\uae30\ub3c4 \ud558\uc600\ub2e4. \uc774\uc640 \ub354\ubd88\uc5b4 \uc5ec\ud559\uc0dd\ucc98\ub294 1956\ub144\uc5d0 \uc124\ub9bd\ub41c \uc774\ub798 \uc131\ud3ed\ub825\uc0c1\ub2f4\uc2e4 \uac1c\uc124, \ubc18\uc131\ud3ed\ub825\ud559\uce59 \uc81c\uc815 \ub4f1 \ub0a8\ub140\ud3c9\ub4f1\uc801 \uad50\uc721 \ud658\uacbd\uc744 \ub9c8\ub828\ud558\uae30 \uc704\ud574 \ub178\ub825\ud558\uc600\ub2e4. 2002\ub144\uc5d0\ub294 \uc5ec\uc131\uc778\ub825\uac1c\ubc1c\uc5f0\uad6c\uc6d0\uc774 \ubc1c\uc871\ub418\uc5b4 \uc5ec\ud559\uc0dd\ub4e4\uc758 \uc0ac\ud68c \uc9c4\ucd9c \ub4f1\uc744 \uc704\ud55c \ud559\ub0b4\uc678\uc801\uc778 \ud65c\ub3d9\uc744 \uc218\ud589\ud558\uace0 \uc788\ub2e4.", "sentence_answer": "\ub9e4\ub144 \uc120\uac70\ub97c \ud1b5\ud574 \ucd1d\uc5ec\ud559\uc0dd\ud68c\uac00 \uc120\ucd9c\ub418\uace0 \uc788\uc73c\uba70 2016\ub144 \uae30\uc900\uc73c\ub85c \uc81c27\ub300 \ucd1d\uc5ec\ud559\uc0dd\ud68c \u2018 \uc787\ub2e4 \u2019\uac00 \ud65c\ub3d9\ud558\uace0 \uc788\ub2e4.", "paragraph_id": "6458470-12-1", "paragraph_question": "question: \ucd1d\uc5ec\ud559\uc0dd\ud68c\uc758 \uba85\uce6d\uc740?, context: \ucd1d\uc5ec\ud559\uc0dd\ud68c\ub294 \uc5ec\uc131 \ubb38\uc81c\ub97c \uace0\ubbfc\ud558\uace0 \ub0a8\uc131\uc911\uc2ec\uc801 \ub300\ud559 \ubb38\ud654\ub97c \uc5ec\uc131\uc8fc\uc758\uc801\uc73c\ub85c \uc7ac\uad6c\uc131\ud558\uae30 \uc704\ud574\uc11c \ud65c\ub3d9\ud558\uace0 \uc788\ub2e4. \ucd1d\uc5ec\ud559\uc0dd\ud68c\ub294 1988\ub144 3\uc6d4\uc5d0 \uae30\uc874 \ucd1d\ud559\uc0dd\ud68c \uc0b0\ud558 \uc5ec\ud559\uc0dd\ubd80\uc758 \ud55c\uacc4\uc640 \ud559\ub0b4\uc5d0\uc11c \uc18c\uc218\uc758 \uc704\uce58\uc5d0 \uc788\ub358 \uc5ec\ud559\uc0dd \ubb38\uc81c\ub97c \uadf9\ubcf5\ud558\uace0\uc790 \ubc1c\uc871\ud558\uc600\ub2e4. \ub9e4\ub144 \uc120\uac70\ub97c \ud1b5\ud574 \ucd1d\uc5ec\ud559\uc0dd\ud68c\uac00 \uc120\ucd9c\ub418\uace0 \uc788\uc73c\uba70 2016\ub144 \uae30\uc900\uc73c\ub85c \uc81c27\ub300 \ucd1d\uc5ec\ud559\uc0dd\ud68c \u2018\uc787\ub2e4\u2019\uac00 \ud65c\ub3d9\ud558\uace0 \uc788\ub2e4. \ucd1d\uc5ec\ud559\uc0dd\ud68c\ub294 \ud0c4\uc0dd \uc774\ub798 \uc704\uc0c1\uc774 \ubaa8\ud638\ud558\ub2e4\ub294 \ube44\ud310\uc744 \ubc1b\uc544\uc654\uc9c0\ub9cc \ub9e4\ub144 \ub0a8\uc131 \uc911\uc2ec\uc758 \ucea0\ud37c\uc2a4 \ubb38\ud654\ub97c \uadf9\ubcf5\ud558\uae30 \uc704\ud55c \ud589\uc0ac\ub97c \uc5f4\uc5b4\uc654\uace0 2000\ub144 2\uc6d4\ubd80\ud130\ub294 \u2018\uc0c8\ub0b4\uae30\ubc30\uc6c0\ud130 \ubc18\uc131\ud3ed\ub825 \ub0b4\uaddc\u2019\ub97c \uc81c\uc815\ud574 \uc2dc\ud589\ud558\uae30\ub3c4 \ud558\uc600\ub2e4. \uc774\uc640 \ub354\ubd88\uc5b4 \uc5ec\ud559\uc0dd\ucc98\ub294 1956\ub144\uc5d0 \uc124\ub9bd\ub41c \uc774\ub798 \uc131\ud3ed\ub825\uc0c1\ub2f4\uc2e4 \uac1c\uc124, \ubc18\uc131\ud3ed\ub825\ud559\uce59 \uc81c\uc815 \ub4f1 \ub0a8\ub140\ud3c9\ub4f1\uc801 \uad50\uc721 \ud658\uacbd\uc744 \ub9c8\ub828\ud558\uae30 \uc704\ud574 \ub178\ub825\ud558\uc600\ub2e4. 2002\ub144\uc5d0\ub294 \uc5ec\uc131\uc778\ub825\uac1c\ubc1c\uc5f0\uad6c\uc6d0\uc774 \ubc1c\uc871\ub418\uc5b4 \uc5ec\ud559\uc0dd\ub4e4\uc758 \uc0ac\ud68c \uc9c4\ucd9c \ub4f1\uc744 \uc704\ud55c \ud559\ub0b4\uc678\uc801\uc778 \ud65c\ub3d9\uc744 \uc218\ud589\ud558\uace0 \uc788\ub2e4."} {"question": "\uc2e0\ud765\ud559\uad50 \ud559\uc0dd\ub4e4\uc740 \uc2dc\uc704\ub97c \uc8fc\ub3c4\ud588\ub2e4\ub294 \ud610\uc758\ub85c \ubaa8\ub450 \uc2e4\ud615 \uba87\ub144\uc744 \uc5b8\ub3c4 \ubc1b\uc558\ub294\uac00?", "paragraph": "\"3.13 \ub9cc\uc138\uc6b4\ub3d9\" \uc740 1919\ub144 3.13\uc77c \uc804\uc8fc\uc5d0\uc11c \uc77c\uc5b4\ub09c \ub9cc\uc138\uc6b4\ub3d9\uc774\ub2e4. \uc9c0\uc5ed \uc778\uc0ac\ub4e4\uacfc \ud568\uaed8 \uc2e0\ud765\ud559\uad50 \ud559\uc0dd\ub4e4\uc774 \uc8fc\ub3c4\uc801\uc778 \uc5ed\ud560\uc744 \ud558\uba70, \ub9cc\uc138\uc6b4\ub3d9\uc744 \uc774\ub04c\uc5c8\ub2e4. \ubc15\ud0dc\ub828, \uae40\uc2e0\uadf9 \ub4f1 \uc804\uc8fc \uc9c0\ub3c4\uc790\ub4e4\uc740 \uad70\uc0b0\uc5d0\uc11c 4\uc77c\uacfc 5\uc77c \ub3c5\ub9bd\ub9cc\uc138 \uc2dc\uc704\uac00 \uac10\ud589\ub410\ub2e4\ub294 \uc18c\uc2dd\uc5d0 \ub4e3\uace0 \uc900\ube44\ud558\uace0 \uc788\uc5c8\ub2e4. \ucc9c\ub3c4\uad50\uc640 \ubc15\ud0dc\ub828 \uc2e0\uac04\ud68c \ucd1d\ubb34\uc9d1\uc5d0\uc11c \ud544\uc694\ud55c \ud0dc\uadf9\uae30\ub97c \uc778\uc1c4\ud558\uae30\ub85c \ud588\uc5c8\ub2e4. \uc11c\uc6b8\uc744 \ube44\ub86f\ud55c \ub2e4\ub978 \uc9c0\ubc29\uc5d0\uc11c \uc2dc\uc704\uac00 \uacc4\uc18d\ub418\uc790 \uc77c\ubcf8\uacbd\ucc30\uc740 \uc2e0\ud765\ud559\uad50\uc640 \uae30\uc804\ud559\uad50\ub97c \ube44\ub86f\ud55c \uc804\uc8fc\uc2dc\ub0b4 \ud559\uad50\uc5d0 \uac15\uc81c \ubc29\ud559\uc870\uce58\ub97c \ucde8\ud588\ub2e4. \uc774\uc5d0 \ucd5c\uc885\uc0bc \ub4f1 \uc2e0\ud765\ud559\uad50 \ud559\uc0dd 5\uba85\uc740 \ubc24\uc744 \uc774\uc6a9\ud574 \uc2e0\ud765\ud559\uad50 \uc9c0\ud558\uc2e4\uc5d0\uc11c \ud0dc\uadf9\uae30 \ub4f1 \uc778\uc1c4\ubb3c\uc744 \ub9cc\ub4e4\uc5c8\ub2e4. \uc900\ube44\ub97c \ub9c8\uce5c \uc774\ub4e4\uc740 13\uc77c \uc7a5\ud130\ub85c \ubaa8\uc774\uae30 \uc2dc\uc791\ud588\uace0, \ucc44\uc18c\uac00\ub9c8\ub2c8\ub85c \uc704\uc7a5\ud55c \ud0dc\uadf9\uae30\ub97c \uc7a5\ud130\ub85c \uc2e4\uc5b4 \ub098\ub974\uace0 \uac70\uc0ac \uc9c1\uc804 \uc2dc\uc7a5 \uc785\uad6c\uc778 \uc644\uc0b0\ub3d9\uacfc \uc804\uc8fc\uad50 \uac74\ub108\ud3b8\uc5d0\uc11c \uad70\uc911\ub4e4\uc5d0\uac8c \uc740\ubc00\ud788 \ubc30\ubd80\ud588\ub2e4. \ub0ae 12\uc2dc20\ubd84\uaed8 \uc2e0\ud765\ud559\uad50\uc640 \uae30\uc804\ud559\uad50 \ud559\uc0dd \ubc0f \ucc9c\ub3c4\uad50\ub3c4 \ub4f1\uc740 \ud0dc\uadf9\uae30\ub97c \ub4e4\uace0 \ub9cc\uc138\ub97c \ubd88\ub800\ub2e4. \ub0a8\ubb38 \ubc16 \uc2dc\uc7a5, \uc81c2\ubcf4\ud1b5\ud559\uad50(\ud604 \uc644\uc0b0\ucd08\ub4f1\ud559\uad50)\uc5d0\uc11c \ubaa8\uc5ec \uc778\uc1c4\ubb3c\uc744 \ubfcc\ub9ac\uba70 \uc2dc\uac00\uc9c0\ub85c \uad6c\ubcf4\ub85c \ud589\uc9c4\ud588\ub2e4. \uc2dc\uc704\ub294 \uc624\ud6c4 11\uc2dc\uae4c\uc9c0 \uc11c\ub108\ucc28\ub840 \uacc4\uc18d\ub410\ub2e4. \ub610 \ub2e4\uc74c\ub0a0 \uc624\ud6c4 3\uc2dc\uc5d0\ub3c4 \uad70\uc911\uc774 \ubaa8\uc5ec \ub9cc\uc138\ub97c \ubd88\ub800\ub2e4. \uc774\ud6c4 \uace0\ud615\uc9c4, \ub0a8\uad81\ud604, \uae40\ubcd1\ud559, \uae40\uc810\uc1e0, \uc774\uae30\uace4, \uae40\uacbd\uc2e0 \ub4f1 \uc2e0\ud765\ud559\uad50 \ud559\uc0dd\ub4e4\uc740 \uc2dc\uc704\ub97c \uc8fc\ub3c4\ud588\ub2e4\ub294 \ud610\uc758\ub85c \ubaa8\ub450 \uc2e4\ud615 1\ub144\uc744 \uc5b8\ub3c4 \ubc1b\uc558\ub2e4. \uc774\uc678 \uc2e0\ud765\ud559\uad50 \ud559\uc0dd 3\uba85\uc740 \uc77c\uc81c\uc758 \uace0\ubb38\uc5d0 \uc625\uc0ac\ud55c \uac83\uc73c\ub85c \uc54c\ub824\uc84c\ub2e4. \ub610 \uc2dc\uc704\ub97c \uc9c0\ub3c4\ud55c \uae40\uc778\uc804 \ubaa9\uc0ac\ub294 \uc774\ud6c4 \uc911\uad6d \uc0c1\ud574\ub85c \uac70\ucc98\ub97c \uc62e\uaca8 \uc784\uc2dc\uc815\ubd80\uc5d0\uc11c \ud65c\ub3d9\ud588\ub2e4. \ud604\uc7ac \uc2e0\ud765\ud559\uad50 \uad50\ubb38 \uc606\uc5d0 \ub9cc\uc138\uc6b4\ub3d9 \uae30\ub150\ube44\uac00 \uc138\uc6cc\uc838 \uc788\ub2e4.", "answer": "1\ub144", "sentence": "\uc774\ud6c4 \uace0\ud615\uc9c4, \ub0a8\uad81\ud604, \uae40\ubcd1\ud559, \uae40\uc810\uc1e0, \uc774\uae30\uace4, \uae40\uacbd\uc2e0 \ub4f1 \uc2e0\ud765\ud559\uad50 \ud559\uc0dd\ub4e4\uc740 \uc2dc\uc704\ub97c \uc8fc\ub3c4\ud588\ub2e4\ub294 \ud610\uc758\ub85c \ubaa8\ub450 \uc2e4\ud615 1\ub144 \uc744 \uc5b8\ub3c4 \ubc1b\uc558\ub2e4.", "paragraph_sentence": "\"3.13 \ub9cc\uc138\uc6b4\ub3d9\" \uc740 1919\ub144 3.13\uc77c \uc804\uc8fc\uc5d0\uc11c \uc77c\uc5b4\ub09c \ub9cc\uc138\uc6b4\ub3d9\uc774\ub2e4. \uc9c0\uc5ed \uc778\uc0ac\ub4e4\uacfc \ud568\uaed8 \uc2e0\ud765\ud559\uad50 \ud559\uc0dd\ub4e4\uc774 \uc8fc\ub3c4\uc801\uc778 \uc5ed\ud560\uc744 \ud558\uba70, \ub9cc\uc138\uc6b4\ub3d9\uc744 \uc774\ub04c\uc5c8\ub2e4. \ubc15\ud0dc\ub828, \uae40\uc2e0\uadf9 \ub4f1 \uc804\uc8fc \uc9c0\ub3c4\uc790\ub4e4\uc740 \uad70\uc0b0\uc5d0\uc11c 4\uc77c\uacfc 5\uc77c \ub3c5\ub9bd\ub9cc\uc138 \uc2dc\uc704\uac00 \uac10\ud589\ub410\ub2e4\ub294 \uc18c\uc2dd\uc5d0 \ub4e3\uace0 \uc900\ube44\ud558\uace0 \uc788\uc5c8\ub2e4. \ucc9c\ub3c4\uad50\uc640 \ubc15\ud0dc\ub828 \uc2e0\uac04\ud68c \ucd1d\ubb34\uc9d1\uc5d0\uc11c \ud544\uc694\ud55c \ud0dc\uadf9\uae30\ub97c \uc778\uc1c4\ud558\uae30\ub85c \ud588\uc5c8\ub2e4. \uc11c\uc6b8\uc744 \ube44\ub86f\ud55c \ub2e4\ub978 \uc9c0\ubc29\uc5d0\uc11c \uc2dc\uc704\uac00 \uacc4\uc18d\ub418\uc790 \uc77c\ubcf8\uacbd\ucc30\uc740 \uc2e0\ud765\ud559\uad50\uc640 \uae30\uc804\ud559\uad50\ub97c \ube44\ub86f\ud55c \uc804\uc8fc\uc2dc\ub0b4 \ud559\uad50\uc5d0 \uac15\uc81c \ubc29\ud559\uc870\uce58\ub97c \ucde8\ud588\ub2e4. \uc774\uc5d0 \ucd5c\uc885\uc0bc \ub4f1 \uc2e0\ud765\ud559\uad50 \ud559\uc0dd 5\uba85\uc740 \ubc24\uc744 \uc774\uc6a9\ud574 \uc2e0\ud765\ud559\uad50 \uc9c0\ud558\uc2e4\uc5d0\uc11c \ud0dc\uadf9\uae30 \ub4f1 \uc778\uc1c4\ubb3c\uc744 \ub9cc\ub4e4\uc5c8\ub2e4. \uc900\ube44\ub97c \ub9c8\uce5c \uc774\ub4e4\uc740 13\uc77c \uc7a5\ud130\ub85c \ubaa8\uc774\uae30 \uc2dc\uc791\ud588\uace0, \ucc44\uc18c\uac00\ub9c8\ub2c8\ub85c \uc704\uc7a5\ud55c \ud0dc\uadf9\uae30\ub97c \uc7a5\ud130\ub85c \uc2e4\uc5b4 \ub098\ub974\uace0 \uac70\uc0ac \uc9c1\uc804 \uc2dc\uc7a5 \uc785\uad6c\uc778 \uc644\uc0b0\ub3d9\uacfc \uc804\uc8fc\uad50 \uac74\ub108\ud3b8\uc5d0\uc11c \uad70\uc911\ub4e4\uc5d0\uac8c \uc740\ubc00\ud788 \ubc30\ubd80\ud588\ub2e4. \ub0ae 12\uc2dc20\ubd84\uaed8 \uc2e0\ud765\ud559\uad50\uc640 \uae30\uc804\ud559\uad50 \ud559\uc0dd \ubc0f \ucc9c\ub3c4\uad50\ub3c4 \ub4f1\uc740 \ud0dc\uadf9\uae30\ub97c \ub4e4\uace0 \ub9cc\uc138\ub97c \ubd88\ub800\ub2e4. \ub0a8\ubb38 \ubc16 \uc2dc\uc7a5, \uc81c2\ubcf4\ud1b5\ud559\uad50(\ud604 \uc644\uc0b0\ucd08\ub4f1\ud559\uad50)\uc5d0\uc11c \ubaa8\uc5ec \uc778\uc1c4\ubb3c\uc744 \ubfcc\ub9ac\uba70 \uc2dc\uac00\uc9c0\ub85c \uad6c\ubcf4\ub85c \ud589\uc9c4\ud588\ub2e4. \uc2dc\uc704\ub294 \uc624\ud6c4 11\uc2dc\uae4c\uc9c0 \uc11c\ub108\ucc28\ub840 \uacc4\uc18d\ub410\ub2e4. \ub610 \ub2e4\uc74c\ub0a0 \uc624\ud6c4 3\uc2dc\uc5d0\ub3c4 \uad70\uc911\uc774 \ubaa8\uc5ec \ub9cc\uc138\ub97c \ubd88\ub800\ub2e4. \uc774\ud6c4 \uace0\ud615\uc9c4, \ub0a8\uad81\ud604, \uae40\ubcd1\ud559, \uae40\uc810\uc1e0, \uc774\uae30\uace4, \uae40\uacbd\uc2e0 \ub4f1 \uc2e0\ud765\ud559\uad50 \ud559\uc0dd\ub4e4\uc740 \uc2dc\uc704\ub97c \uc8fc\ub3c4\ud588\ub2e4\ub294 \ud610\uc758\ub85c \ubaa8\ub450 \uc2e4\ud615 1\ub144 \uc744 \uc5b8\ub3c4 \ubc1b\uc558\ub2e4. \uc774\uc678 \uc2e0\ud765\ud559\uad50 \ud559\uc0dd 3\uba85\uc740 \uc77c\uc81c\uc758 \uace0\ubb38\uc5d0 \uc625\uc0ac\ud55c \uac83\uc73c\ub85c \uc54c\ub824\uc84c\ub2e4. \ub610 \uc2dc\uc704\ub97c \uc9c0\ub3c4\ud55c \uae40\uc778\uc804 \ubaa9\uc0ac\ub294 \uc774\ud6c4 \uc911\uad6d \uc0c1\ud574\ub85c \uac70\ucc98\ub97c \uc62e\uaca8 \uc784\uc2dc\uc815\ubd80\uc5d0\uc11c \ud65c\ub3d9\ud588\ub2e4. \ud604\uc7ac \uc2e0\ud765\ud559\uad50 \uad50\ubb38 \uc606\uc5d0 \ub9cc\uc138\uc6b4\ub3d9 \uae30\ub150\ube44\uac00 \uc138\uc6cc\uc838 \uc788\ub2e4.", "paragraph_answer": "\"3.13 \ub9cc\uc138\uc6b4\ub3d9\" \uc740 1919\ub144 3.13\uc77c \uc804\uc8fc\uc5d0\uc11c \uc77c\uc5b4\ub09c \ub9cc\uc138\uc6b4\ub3d9\uc774\ub2e4. \uc9c0\uc5ed \uc778\uc0ac\ub4e4\uacfc \ud568\uaed8 \uc2e0\ud765\ud559\uad50 \ud559\uc0dd\ub4e4\uc774 \uc8fc\ub3c4\uc801\uc778 \uc5ed\ud560\uc744 \ud558\uba70, \ub9cc\uc138\uc6b4\ub3d9\uc744 \uc774\ub04c\uc5c8\ub2e4. \ubc15\ud0dc\ub828, \uae40\uc2e0\uadf9 \ub4f1 \uc804\uc8fc \uc9c0\ub3c4\uc790\ub4e4\uc740 \uad70\uc0b0\uc5d0\uc11c 4\uc77c\uacfc 5\uc77c \ub3c5\ub9bd\ub9cc\uc138 \uc2dc\uc704\uac00 \uac10\ud589\ub410\ub2e4\ub294 \uc18c\uc2dd\uc5d0 \ub4e3\uace0 \uc900\ube44\ud558\uace0 \uc788\uc5c8\ub2e4. \ucc9c\ub3c4\uad50\uc640 \ubc15\ud0dc\ub828 \uc2e0\uac04\ud68c \ucd1d\ubb34\uc9d1\uc5d0\uc11c \ud544\uc694\ud55c \ud0dc\uadf9\uae30\ub97c \uc778\uc1c4\ud558\uae30\ub85c \ud588\uc5c8\ub2e4. \uc11c\uc6b8\uc744 \ube44\ub86f\ud55c \ub2e4\ub978 \uc9c0\ubc29\uc5d0\uc11c \uc2dc\uc704\uac00 \uacc4\uc18d\ub418\uc790 \uc77c\ubcf8\uacbd\ucc30\uc740 \uc2e0\ud765\ud559\uad50\uc640 \uae30\uc804\ud559\uad50\ub97c \ube44\ub86f\ud55c \uc804\uc8fc\uc2dc\ub0b4 \ud559\uad50\uc5d0 \uac15\uc81c \ubc29\ud559\uc870\uce58\ub97c \ucde8\ud588\ub2e4. \uc774\uc5d0 \ucd5c\uc885\uc0bc \ub4f1 \uc2e0\ud765\ud559\uad50 \ud559\uc0dd 5\uba85\uc740 \ubc24\uc744 \uc774\uc6a9\ud574 \uc2e0\ud765\ud559\uad50 \uc9c0\ud558\uc2e4\uc5d0\uc11c \ud0dc\uadf9\uae30 \ub4f1 \uc778\uc1c4\ubb3c\uc744 \ub9cc\ub4e4\uc5c8\ub2e4. \uc900\ube44\ub97c \ub9c8\uce5c \uc774\ub4e4\uc740 13\uc77c \uc7a5\ud130\ub85c \ubaa8\uc774\uae30 \uc2dc\uc791\ud588\uace0, \ucc44\uc18c\uac00\ub9c8\ub2c8\ub85c \uc704\uc7a5\ud55c \ud0dc\uadf9\uae30\ub97c \uc7a5\ud130\ub85c \uc2e4\uc5b4 \ub098\ub974\uace0 \uac70\uc0ac \uc9c1\uc804 \uc2dc\uc7a5 \uc785\uad6c\uc778 \uc644\uc0b0\ub3d9\uacfc \uc804\uc8fc\uad50 \uac74\ub108\ud3b8\uc5d0\uc11c \uad70\uc911\ub4e4\uc5d0\uac8c \uc740\ubc00\ud788 \ubc30\ubd80\ud588\ub2e4. \ub0ae 12\uc2dc20\ubd84\uaed8 \uc2e0\ud765\ud559\uad50\uc640 \uae30\uc804\ud559\uad50 \ud559\uc0dd \ubc0f \ucc9c\ub3c4\uad50\ub3c4 \ub4f1\uc740 \ud0dc\uadf9\uae30\ub97c \ub4e4\uace0 \ub9cc\uc138\ub97c \ubd88\ub800\ub2e4. \ub0a8\ubb38 \ubc16 \uc2dc\uc7a5, \uc81c2\ubcf4\ud1b5\ud559\uad50(\ud604 \uc644\uc0b0\ucd08\ub4f1\ud559\uad50)\uc5d0\uc11c \ubaa8\uc5ec \uc778\uc1c4\ubb3c\uc744 \ubfcc\ub9ac\uba70 \uc2dc\uac00\uc9c0\ub85c \uad6c\ubcf4\ub85c \ud589\uc9c4\ud588\ub2e4. \uc2dc\uc704\ub294 \uc624\ud6c4 11\uc2dc\uae4c\uc9c0 \uc11c\ub108\ucc28\ub840 \uacc4\uc18d\ub410\ub2e4. \ub610 \ub2e4\uc74c\ub0a0 \uc624\ud6c4 3\uc2dc\uc5d0\ub3c4 \uad70\uc911\uc774 \ubaa8\uc5ec \ub9cc\uc138\ub97c \ubd88\ub800\ub2e4. \uc774\ud6c4 \uace0\ud615\uc9c4, \ub0a8\uad81\ud604, \uae40\ubcd1\ud559, \uae40\uc810\uc1e0, \uc774\uae30\uace4, \uae40\uacbd\uc2e0 \ub4f1 \uc2e0\ud765\ud559\uad50 \ud559\uc0dd\ub4e4\uc740 \uc2dc\uc704\ub97c \uc8fc\ub3c4\ud588\ub2e4\ub294 \ud610\uc758\ub85c \ubaa8\ub450 \uc2e4\ud615 1\ub144 \uc744 \uc5b8\ub3c4 \ubc1b\uc558\ub2e4. \uc774\uc678 \uc2e0\ud765\ud559\uad50 \ud559\uc0dd 3\uba85\uc740 \uc77c\uc81c\uc758 \uace0\ubb38\uc5d0 \uc625\uc0ac\ud55c \uac83\uc73c\ub85c \uc54c\ub824\uc84c\ub2e4. \ub610 \uc2dc\uc704\ub97c \uc9c0\ub3c4\ud55c \uae40\uc778\uc804 \ubaa9\uc0ac\ub294 \uc774\ud6c4 \uc911\uad6d \uc0c1\ud574\ub85c \uac70\ucc98\ub97c \uc62e\uaca8 \uc784\uc2dc\uc815\ubd80\uc5d0\uc11c \ud65c\ub3d9\ud588\ub2e4. \ud604\uc7ac \uc2e0\ud765\ud559\uad50 \uad50\ubb38 \uc606\uc5d0 \ub9cc\uc138\uc6b4\ub3d9 \uae30\ub150\ube44\uac00 \uc138\uc6cc\uc838 \uc788\ub2e4.", "sentence_answer": "\uc774\ud6c4 \uace0\ud615\uc9c4, \ub0a8\uad81\ud604, \uae40\ubcd1\ud559, \uae40\uc810\uc1e0, \uc774\uae30\uace4, \uae40\uacbd\uc2e0 \ub4f1 \uc2e0\ud765\ud559\uad50 \ud559\uc0dd\ub4e4\uc740 \uc2dc\uc704\ub97c \uc8fc\ub3c4\ud588\ub2e4\ub294 \ud610\uc758\ub85c \ubaa8\ub450 \uc2e4\ud615 1\ub144 \uc744 \uc5b8\ub3c4 \ubc1b\uc558\ub2e4.", "paragraph_id": "6549838-0-1", "paragraph_question": "question: \uc2e0\ud765\ud559\uad50 \ud559\uc0dd\ub4e4\uc740 \uc2dc\uc704\ub97c \uc8fc\ub3c4\ud588\ub2e4\ub294 \ud610\uc758\ub85c \ubaa8\ub450 \uc2e4\ud615 \uba87\ub144\uc744 \uc5b8\ub3c4 \ubc1b\uc558\ub294\uac00?, context: \"3.13 \ub9cc\uc138\uc6b4\ub3d9\" \uc740 1919\ub144 3.13\uc77c \uc804\uc8fc\uc5d0\uc11c \uc77c\uc5b4\ub09c \ub9cc\uc138\uc6b4\ub3d9\uc774\ub2e4. \uc9c0\uc5ed \uc778\uc0ac\ub4e4\uacfc \ud568\uaed8 \uc2e0\ud765\ud559\uad50 \ud559\uc0dd\ub4e4\uc774 \uc8fc\ub3c4\uc801\uc778 \uc5ed\ud560\uc744 \ud558\uba70, \ub9cc\uc138\uc6b4\ub3d9\uc744 \uc774\ub04c\uc5c8\ub2e4. \ubc15\ud0dc\ub828, \uae40\uc2e0\uadf9 \ub4f1 \uc804\uc8fc \uc9c0\ub3c4\uc790\ub4e4\uc740 \uad70\uc0b0\uc5d0\uc11c 4\uc77c\uacfc 5\uc77c \ub3c5\ub9bd\ub9cc\uc138 \uc2dc\uc704\uac00 \uac10\ud589\ub410\ub2e4\ub294 \uc18c\uc2dd\uc5d0 \ub4e3\uace0 \uc900\ube44\ud558\uace0 \uc788\uc5c8\ub2e4. \ucc9c\ub3c4\uad50\uc640 \ubc15\ud0dc\ub828 \uc2e0\uac04\ud68c \ucd1d\ubb34\uc9d1\uc5d0\uc11c \ud544\uc694\ud55c \ud0dc\uadf9\uae30\ub97c \uc778\uc1c4\ud558\uae30\ub85c \ud588\uc5c8\ub2e4. \uc11c\uc6b8\uc744 \ube44\ub86f\ud55c \ub2e4\ub978 \uc9c0\ubc29\uc5d0\uc11c \uc2dc\uc704\uac00 \uacc4\uc18d\ub418\uc790 \uc77c\ubcf8\uacbd\ucc30\uc740 \uc2e0\ud765\ud559\uad50\uc640 \uae30\uc804\ud559\uad50\ub97c \ube44\ub86f\ud55c \uc804\uc8fc\uc2dc\ub0b4 \ud559\uad50\uc5d0 \uac15\uc81c \ubc29\ud559\uc870\uce58\ub97c \ucde8\ud588\ub2e4. \uc774\uc5d0 \ucd5c\uc885\uc0bc \ub4f1 \uc2e0\ud765\ud559\uad50 \ud559\uc0dd 5\uba85\uc740 \ubc24\uc744 \uc774\uc6a9\ud574 \uc2e0\ud765\ud559\uad50 \uc9c0\ud558\uc2e4\uc5d0\uc11c \ud0dc\uadf9\uae30 \ub4f1 \uc778\uc1c4\ubb3c\uc744 \ub9cc\ub4e4\uc5c8\ub2e4. \uc900\ube44\ub97c \ub9c8\uce5c \uc774\ub4e4\uc740 13\uc77c \uc7a5\ud130\ub85c \ubaa8\uc774\uae30 \uc2dc\uc791\ud588\uace0, \ucc44\uc18c\uac00\ub9c8\ub2c8\ub85c \uc704\uc7a5\ud55c \ud0dc\uadf9\uae30\ub97c \uc7a5\ud130\ub85c \uc2e4\uc5b4 \ub098\ub974\uace0 \uac70\uc0ac \uc9c1\uc804 \uc2dc\uc7a5 \uc785\uad6c\uc778 \uc644\uc0b0\ub3d9\uacfc \uc804\uc8fc\uad50 \uac74\ub108\ud3b8\uc5d0\uc11c \uad70\uc911\ub4e4\uc5d0\uac8c \uc740\ubc00\ud788 \ubc30\ubd80\ud588\ub2e4. \ub0ae 12\uc2dc20\ubd84\uaed8 \uc2e0\ud765\ud559\uad50\uc640 \uae30\uc804\ud559\uad50 \ud559\uc0dd \ubc0f \ucc9c\ub3c4\uad50\ub3c4 \ub4f1\uc740 \ud0dc\uadf9\uae30\ub97c \ub4e4\uace0 \ub9cc\uc138\ub97c \ubd88\ub800\ub2e4. \ub0a8\ubb38 \ubc16 \uc2dc\uc7a5, \uc81c2\ubcf4\ud1b5\ud559\uad50(\ud604 \uc644\uc0b0\ucd08\ub4f1\ud559\uad50)\uc5d0\uc11c \ubaa8\uc5ec \uc778\uc1c4\ubb3c\uc744 \ubfcc\ub9ac\uba70 \uc2dc\uac00\uc9c0\ub85c \uad6c\ubcf4\ub85c \ud589\uc9c4\ud588\ub2e4. \uc2dc\uc704\ub294 \uc624\ud6c4 11\uc2dc\uae4c\uc9c0 \uc11c\ub108\ucc28\ub840 \uacc4\uc18d\ub410\ub2e4. \ub610 \ub2e4\uc74c\ub0a0 \uc624\ud6c4 3\uc2dc\uc5d0\ub3c4 \uad70\uc911\uc774 \ubaa8\uc5ec \ub9cc\uc138\ub97c \ubd88\ub800\ub2e4. \uc774\ud6c4 \uace0\ud615\uc9c4, \ub0a8\uad81\ud604, \uae40\ubcd1\ud559, \uae40\uc810\uc1e0, \uc774\uae30\uace4, \uae40\uacbd\uc2e0 \ub4f1 \uc2e0\ud765\ud559\uad50 \ud559\uc0dd\ub4e4\uc740 \uc2dc\uc704\ub97c \uc8fc\ub3c4\ud588\ub2e4\ub294 \ud610\uc758\ub85c \ubaa8\ub450 \uc2e4\ud615 1\ub144\uc744 \uc5b8\ub3c4 \ubc1b\uc558\ub2e4. \uc774\uc678 \uc2e0\ud765\ud559\uad50 \ud559\uc0dd 3\uba85\uc740 \uc77c\uc81c\uc758 \uace0\ubb38\uc5d0 \uc625\uc0ac\ud55c \uac83\uc73c\ub85c \uc54c\ub824\uc84c\ub2e4. \ub610 \uc2dc\uc704\ub97c \uc9c0\ub3c4\ud55c \uae40\uc778\uc804 \ubaa9\uc0ac\ub294 \uc774\ud6c4 \uc911\uad6d \uc0c1\ud574\ub85c \uac70\ucc98\ub97c \uc62e\uaca8 \uc784\uc2dc\uc815\ubd80\uc5d0\uc11c \ud65c\ub3d9\ud588\ub2e4. \ud604\uc7ac \uc2e0\ud765\ud559\uad50 \uad50\ubb38 \uc606\uc5d0 \ub9cc\uc138\uc6b4\ub3d9 \uae30\ub150\ube44\uac00 \uc138\uc6cc\uc838 \uc788\ub2e4."} {"question": "Ordinary Men: Reserve Police Battalion 101 and the Final Solution in Poland\ub97c \uc4f4 \ud640\ub85c\ucf54\uc2a4\ud2b8 \uc0ac\ud559\uc790\ub294?", "paragraph": "1992\ub144\uc5d0 \ubc1c\uac04\ub41c \ud640\ub85c\ucf54\uc2a4\ud2b8 \uc0ac\ud559\uc790 Christopher Browning\uc758 \uc800\uc11c \u201cOrdinary Men: Reserve Police Battalion 101 and the Final Solution in Poland\u201d\uc5d0\uc11c \uadf8\ub294 \uc720\ub300\uc778\ub4e4\uc744 \uccb4\ud3ec\ud558\uace0 \ud559\uc0b4\ud558\uba70 \uc218\uc6a9\uc18c\ub85c \uc774\uc1a1\ud558\ub358 \uc784\ubb34\ub97c \uc218\ud589\ud55c \uacbd\ucc30\uc870\uc9c1\uc744 \ub2e4\ub918\ub2e4. 101\ubd80\ub300\ub77c \ubd88\ub9b0 \uc774 \uacbd\ucc30\uc870\uc9c1\uc758 \ub300\uc6d0\ub4e4\uc740 \ub300\ubd80\ubd84\uc774 \ubcd1\uc5ed\uc5d0 \ubd80\uc801\ud569\ud55c \ud568\ubd80\ub974\ud06c \ub178\ub3d9\uacc4\uae09 \ucd9c\uc2e0\uc758 \ub3d9\ud5a5 \uc911\ub144 \ub0a8\uc131\ub4e4\ub85c, \ud559\uc0b4\uc5d0 \ub300\ud55c \uc5b4\ub5a0\ud55c \uad50\uc721\ub3c4 \ubc1b\uc9c0 \uc54a\uc558\uc5c8\ub2e4. \uadf8\ub9ac\uace0 \ubd80\ub300\uc758 \uc9c0\ud718\uad00\uc740 \ud559\uc0b4\uc784\ubb34\uac00 \uc8fc\uc5b4\uc9c0\uba74 \uc774\uc5d0 \ub300\ud574 \ubd88\ucf8c\uac10\uc744 \uac00\uc9c0\ub294 \ubd80\ub300\uc6d0\ub4e4\uc5d0\uac8c\ub294 \uc9c1\uc811\uc801\uc73c\ub85c \ucc38\uc5ec\ud558\uc9c0 \uc54a\uc744 \uc120\ud0dd\uad8c\uc744 \uc8fc\uc5c8\ub2e4. \ub300\uc2e0\uc5d0 \ud559\uc0b4\ud604\uc7a5\ud1b5\uc81c\uc640 \uc800\uc9c0\uc120\ud615\uc131 \ub4f1\uc758 \uac04\uc811\uc801\uc778 \uc791\uc5c5\uc5d0\ub9cc \ucc38\uc5ec\ud558\ub3c4\ub85d \ud558\uc600\ub2e4. \uadf8\ub9ac\uace0 500\uc5ec\uba85\uc758 \ubd80\ub300\uc6d0 \uc911 15\uba85 \ubbf8\ub9cc\uc758 \ub300\uc6d0\ub9cc\uc774 \uc774\ub97c \uc120\ud0dd\ud558\uc600\uace0 \uc808\ub300\ub2e4\uc218\ub294 \ud559\uc0b4\uc5d0 \uc801\uadf9\uc801\uc73c\ub85c \ub3d9\ucc38\ud588\ub2e4.", "answer": "Christopher Browning", "sentence": "1992\ub144\uc5d0 \ubc1c\uac04\ub41c \ud640\ub85c\ucf54\uc2a4\ud2b8 \uc0ac\ud559\uc790 Christopher Browning \uc758 \uc800\uc11c \u201cOrdinary Men: Reserve Police Battalion 101 and the Final Solution in Poland\u201d\uc5d0\uc11c \uadf8\ub294 \uc720\ub300\uc778\ub4e4\uc744 \uccb4\ud3ec\ud558\uace0 \ud559\uc0b4\ud558\uba70 \uc218\uc6a9\uc18c\ub85c \uc774\uc1a1\ud558\ub358 \uc784\ubb34\ub97c \uc218\ud589\ud55c \uacbd\ucc30\uc870\uc9c1\uc744 \ub2e4\ub918\ub2e4.", "paragraph_sentence": " 1992\ub144\uc5d0 \ubc1c\uac04\ub41c \ud640\ub85c\ucf54\uc2a4\ud2b8 \uc0ac\ud559\uc790 Christopher Browning \uc758 \uc800\uc11c \u201cOrdinary Men: Reserve Police Battalion 101 and the Final Solution in Poland\u201d\uc5d0\uc11c \uadf8\ub294 \uc720\ub300\uc778\ub4e4\uc744 \uccb4\ud3ec\ud558\uace0 \ud559\uc0b4\ud558\uba70 \uc218\uc6a9\uc18c\ub85c \uc774\uc1a1\ud558\ub358 \uc784\ubb34\ub97c \uc218\ud589\ud55c \uacbd\ucc30\uc870\uc9c1\uc744 \ub2e4\ub918\ub2e4. 101\ubd80\ub300\ub77c \ubd88\ub9b0 \uc774 \uacbd\ucc30\uc870\uc9c1\uc758 \ub300\uc6d0\ub4e4\uc740 \ub300\ubd80\ubd84\uc774 \ubcd1\uc5ed\uc5d0 \ubd80\uc801\ud569\ud55c \ud568\ubd80\ub974\ud06c \ub178\ub3d9\uacc4\uae09 \ucd9c\uc2e0\uc758 \ub3d9\ud5a5 \uc911\ub144 \ub0a8\uc131\ub4e4\ub85c, \ud559\uc0b4\uc5d0 \ub300\ud55c \uc5b4\ub5a0\ud55c \uad50\uc721\ub3c4 \ubc1b\uc9c0 \uc54a\uc558\uc5c8\ub2e4. \uadf8\ub9ac\uace0 \ubd80\ub300\uc758 \uc9c0\ud718\uad00\uc740 \ud559\uc0b4\uc784\ubb34\uac00 \uc8fc\uc5b4\uc9c0\uba74 \uc774\uc5d0 \ub300\ud574 \ubd88\ucf8c\uac10\uc744 \uac00\uc9c0\ub294 \ubd80\ub300\uc6d0\ub4e4\uc5d0\uac8c\ub294 \uc9c1\uc811\uc801\uc73c\ub85c \ucc38\uc5ec\ud558\uc9c0 \uc54a\uc744 \uc120\ud0dd\uad8c\uc744 \uc8fc\uc5c8\ub2e4. \ub300\uc2e0\uc5d0 \ud559\uc0b4\ud604\uc7a5\ud1b5\uc81c\uc640 \uc800\uc9c0\uc120\ud615\uc131 \ub4f1\uc758 \uac04\uc811\uc801\uc778 \uc791\uc5c5\uc5d0\ub9cc \ucc38\uc5ec\ud558\ub3c4\ub85d \ud558\uc600\ub2e4. \uadf8\ub9ac\uace0 500\uc5ec\uba85\uc758 \ubd80\ub300\uc6d0 \uc911 15\uba85 \ubbf8\ub9cc\uc758 \ub300\uc6d0\ub9cc\uc774 \uc774\ub97c \uc120\ud0dd\ud558\uc600\uace0 \uc808\ub300\ub2e4\uc218\ub294 \ud559\uc0b4\uc5d0 \uc801\uadf9\uc801\uc73c\ub85c \ub3d9\ucc38\ud588\ub2e4.", "paragraph_answer": "1992\ub144\uc5d0 \ubc1c\uac04\ub41c \ud640\ub85c\ucf54\uc2a4\ud2b8 \uc0ac\ud559\uc790 Christopher Browning \uc758 \uc800\uc11c \u201cOrdinary Men: Reserve Police Battalion 101 and the Final Solution in Poland\u201d\uc5d0\uc11c \uadf8\ub294 \uc720\ub300\uc778\ub4e4\uc744 \uccb4\ud3ec\ud558\uace0 \ud559\uc0b4\ud558\uba70 \uc218\uc6a9\uc18c\ub85c \uc774\uc1a1\ud558\ub358 \uc784\ubb34\ub97c \uc218\ud589\ud55c \uacbd\ucc30\uc870\uc9c1\uc744 \ub2e4\ub918\ub2e4. 101\ubd80\ub300\ub77c \ubd88\ub9b0 \uc774 \uacbd\ucc30\uc870\uc9c1\uc758 \ub300\uc6d0\ub4e4\uc740 \ub300\ubd80\ubd84\uc774 \ubcd1\uc5ed\uc5d0 \ubd80\uc801\ud569\ud55c \ud568\ubd80\ub974\ud06c \ub178\ub3d9\uacc4\uae09 \ucd9c\uc2e0\uc758 \ub3d9\ud5a5 \uc911\ub144 \ub0a8\uc131\ub4e4\ub85c, \ud559\uc0b4\uc5d0 \ub300\ud55c \uc5b4\ub5a0\ud55c \uad50\uc721\ub3c4 \ubc1b\uc9c0 \uc54a\uc558\uc5c8\ub2e4. \uadf8\ub9ac\uace0 \ubd80\ub300\uc758 \uc9c0\ud718\uad00\uc740 \ud559\uc0b4\uc784\ubb34\uac00 \uc8fc\uc5b4\uc9c0\uba74 \uc774\uc5d0 \ub300\ud574 \ubd88\ucf8c\uac10\uc744 \uac00\uc9c0\ub294 \ubd80\ub300\uc6d0\ub4e4\uc5d0\uac8c\ub294 \uc9c1\uc811\uc801\uc73c\ub85c \ucc38\uc5ec\ud558\uc9c0 \uc54a\uc744 \uc120\ud0dd\uad8c\uc744 \uc8fc\uc5c8\ub2e4. \ub300\uc2e0\uc5d0 \ud559\uc0b4\ud604\uc7a5\ud1b5\uc81c\uc640 \uc800\uc9c0\uc120\ud615\uc131 \ub4f1\uc758 \uac04\uc811\uc801\uc778 \uc791\uc5c5\uc5d0\ub9cc \ucc38\uc5ec\ud558\ub3c4\ub85d \ud558\uc600\ub2e4. \uadf8\ub9ac\uace0 500\uc5ec\uba85\uc758 \ubd80\ub300\uc6d0 \uc911 15\uba85 \ubbf8\ub9cc\uc758 \ub300\uc6d0\ub9cc\uc774 \uc774\ub97c \uc120\ud0dd\ud558\uc600\uace0 \uc808\ub300\ub2e4\uc218\ub294 \ud559\uc0b4\uc5d0 \uc801\uadf9\uc801\uc73c\ub85c \ub3d9\ucc38\ud588\ub2e4.", "sentence_answer": "1992\ub144\uc5d0 \ubc1c\uac04\ub41c \ud640\ub85c\ucf54\uc2a4\ud2b8 \uc0ac\ud559\uc790 Christopher Browning \uc758 \uc800\uc11c \u201cOrdinary Men: Reserve Police Battalion 101 and the Final Solution in Poland\u201d\uc5d0\uc11c \uadf8\ub294 \uc720\ub300\uc778\ub4e4\uc744 \uccb4\ud3ec\ud558\uace0 \ud559\uc0b4\ud558\uba70 \uc218\uc6a9\uc18c\ub85c \uc774\uc1a1\ud558\ub358 \uc784\ubb34\ub97c \uc218\ud589\ud55c \uacbd\ucc30\uc870\uc9c1\uc744 \ub2e4\ub918\ub2e4.", "paragraph_id": "6571961-19-0", "paragraph_question": "question: Ordinary Men: Reserve Police Battalion 101 and the Final Solution in Poland\ub97c \uc4f4 \ud640\ub85c\ucf54\uc2a4\ud2b8 \uc0ac\ud559\uc790\ub294?, context: 1992\ub144\uc5d0 \ubc1c\uac04\ub41c \ud640\ub85c\ucf54\uc2a4\ud2b8 \uc0ac\ud559\uc790 Christopher Browning\uc758 \uc800\uc11c \u201cOrdinary Men: Reserve Police Battalion 101 and the Final Solution in Poland\u201d\uc5d0\uc11c \uadf8\ub294 \uc720\ub300\uc778\ub4e4\uc744 \uccb4\ud3ec\ud558\uace0 \ud559\uc0b4\ud558\uba70 \uc218\uc6a9\uc18c\ub85c \uc774\uc1a1\ud558\ub358 \uc784\ubb34\ub97c \uc218\ud589\ud55c \uacbd\ucc30\uc870\uc9c1\uc744 \ub2e4\ub918\ub2e4. 101\ubd80\ub300\ub77c \ubd88\ub9b0 \uc774 \uacbd\ucc30\uc870\uc9c1\uc758 \ub300\uc6d0\ub4e4\uc740 \ub300\ubd80\ubd84\uc774 \ubcd1\uc5ed\uc5d0 \ubd80\uc801\ud569\ud55c \ud568\ubd80\ub974\ud06c \ub178\ub3d9\uacc4\uae09 \ucd9c\uc2e0\uc758 \ub3d9\ud5a5 \uc911\ub144 \ub0a8\uc131\ub4e4\ub85c, \ud559\uc0b4\uc5d0 \ub300\ud55c \uc5b4\ub5a0\ud55c \uad50\uc721\ub3c4 \ubc1b\uc9c0 \uc54a\uc558\uc5c8\ub2e4. \uadf8\ub9ac\uace0 \ubd80\ub300\uc758 \uc9c0\ud718\uad00\uc740 \ud559\uc0b4\uc784\ubb34\uac00 \uc8fc\uc5b4\uc9c0\uba74 \uc774\uc5d0 \ub300\ud574 \ubd88\ucf8c\uac10\uc744 \uac00\uc9c0\ub294 \ubd80\ub300\uc6d0\ub4e4\uc5d0\uac8c\ub294 \uc9c1\uc811\uc801\uc73c\ub85c \ucc38\uc5ec\ud558\uc9c0 \uc54a\uc744 \uc120\ud0dd\uad8c\uc744 \uc8fc\uc5c8\ub2e4. \ub300\uc2e0\uc5d0 \ud559\uc0b4\ud604\uc7a5\ud1b5\uc81c\uc640 \uc800\uc9c0\uc120\ud615\uc131 \ub4f1\uc758 \uac04\uc811\uc801\uc778 \uc791\uc5c5\uc5d0\ub9cc \ucc38\uc5ec\ud558\ub3c4\ub85d \ud558\uc600\ub2e4. \uadf8\ub9ac\uace0 500\uc5ec\uba85\uc758 \ubd80\ub300\uc6d0 \uc911 15\uba85 \ubbf8\ub9cc\uc758 \ub300\uc6d0\ub9cc\uc774 \uc774\ub97c \uc120\ud0dd\ud558\uc600\uace0 \uc808\ub300\ub2e4\uc218\ub294 \ud559\uc0b4\uc5d0 \uc801\uadf9\uc801\uc73c\ub85c \ub3d9\ucc38\ud588\ub2e4."} {"question": "\uacc4\uc8fc\uc5d0 \uc0ac\uc6a9\ub418\ub294 \ubc30\ud134 \ubb34\uac8c\ub294 \uba87 \uadf8\ub7a8 \uc774\uc0c1\uc778\uac00?", "paragraph": "\uc774\uc5b4\ub2ec\ub9ac\uae30\ub294 \uac01 \uac1c\uc778\uc758 \uc18d\ub825\uc744 \ud569\ub9ac\uc801\uc73c\ub85c \uc5f0\uacb0\uc2dc\ud0a8 \ub2e8\uccb4 \uc6b4\ub3d9\uc73c\ub85c, \uacc4\uc8fc(\u7e7c\u8d70)\ub77c\uace0\ub3c4 \ud55c\ub2e4. \uc120\uc218 \uac1c\uac1c\uc778\ubcf4\ub2e4 4\uba85\uc758 \ud611\ub3d9\uc2ec\uacfc \ub2e8\uacb0\uc2ec\uc774 \uc694\uad6c\ub418\uba70, 400m\u00b7800m\u00b71500m \uc774\uc5b4\ub2ec\ub9ac\uae30\uac00 \uc788\ub2e4. \uac00\uc7a5 \ube60\ub978 \uc120\uc218\ub97c \ub9c8\uc9c0\ub9c9 \uc8fc\uc790\ub85c, \uadf8 \ub2e4\uc74c \ube60\ub978 \uc8fc\uc790\ub97c \uccab \ubc88\uc9f8 \uc8fc\uc790\ub85c \ud558\uace0, \ub450 \ubc88\uc9f8\u00b7\uc138 \ubc88\uc9f8 \uc8fc\uc790\ub294 \uc801\ub2f9\ud788 \ubc30\uce58\ud55c\ub2e4. \ucc98\uc74c \uc8fc\uc790\uac00 \uac70\ub9ac\ub97c \ubc8c\ub9b0 \ub2e4\uc74c, \ub450 \ubc88\uc9f8\u00b7\uc138 \ubc88\uc9f8 \uc8fc\uc790\uac00 \ub9ac\ub4dc\ub97c \ub2f9\ud558\ub354\ub77c\ub3c4 \ub9c8\uc9c0\ub9c9 \uc8fc\uc790\uac00 \ub9cc\ud68c\ub97c \ud560 \uc218 \uc788\uae30 \ub54c\ubb38\uc774\ub2e4. \ub2e4\uc74c\uc5d0\ub294 \ubc30\ud134\uc744 \uc8fc\uace0 \ubc1b\ub294 \ubc95\uc774 \uc911\uc694\ud558\ub2e4. \ubc30\ud134\uc740 \ub2e4\uc74c \uc8fc\uc790\uac00 \ubc1b\uae30 \uc27d\ub3c4\ub85d \uc804\uccb4 \uae38\uc774\uc758 2/3\ub97c \ub0a8\uae30\uace0 \uadf8 \uc544\ub7ab\ubd80\ubd84\uc744 \uc7a1\ub294\ub2e4. \ubc30\ud134\uc744 \ubc1b\uc744 \uc8fc\uc790\uac00 \uc900\ube44 \ub3d9\uc791\uc744 \ud560 \uc218 \uc788\ub294 \uac70\ub9ac\ub294 10m\uc758 \uc900\ube44 \uc9c0\uc5ed\uc744 \ud3ec\ud568\ud558\uc5ec 30m\uc774\uc9c0\ub9cc \ubc30\ud134\uc744 \uc8fc\uace0 \ubc1b\ub294 \ub3d9\uc791\uc740 \ubc18\ub4dc\uc2dc 20m \ubc30\ud134 \ud130\uce58 \uc874\uc5d0\uc11c \ud574\uc57c \ud55c\ub2e4. \ubc30\ud134\uc744 \uc8fc\ub294 \uc8fc\uc790\uc640 \ubc1b\ub294 \uc8fc\uc790\uc758 \uc18d\ub3c4\uac00 \uc77c\uce58\ud558\ub294 \uc9c0\uc810\uc5d0\uc11c \ubc30\ud134\uc744 \uc8fc\uace0 \ubc1b\ub294\uac83\uc774 \uac00\uc7a5 \ud6a8\uacfc\uc801\uc774\ub2e4. \ubc30\ud134\uc740 \uae38\uc774 28~30 cm, \ubb34\uac8c 50g \uc774\uc0c1\uc774\ub2e4.", "answer": "50g", "sentence": "\ubc30\ud134\uc740 \uae38\uc774 28~30 cm, \ubb34\uac8c 50g \uc774\uc0c1\uc774\ub2e4.", "paragraph_sentence": "\uc774\uc5b4\ub2ec\ub9ac\uae30\ub294 \uac01 \uac1c\uc778\uc758 \uc18d\ub825\uc744 \ud569\ub9ac\uc801\uc73c\ub85c \uc5f0\uacb0\uc2dc\ud0a8 \ub2e8\uccb4 \uc6b4\ub3d9\uc73c\ub85c, \uacc4\uc8fc(\u7e7c\u8d70)\ub77c\uace0\ub3c4 \ud55c\ub2e4. \uc120\uc218 \uac1c\uac1c\uc778\ubcf4\ub2e4 4\uba85\uc758 \ud611\ub3d9\uc2ec\uacfc \ub2e8\uacb0\uc2ec\uc774 \uc694\uad6c\ub418\uba70, 400m\u00b7800m\u00b71500m \uc774\uc5b4\ub2ec\ub9ac\uae30\uac00 \uc788\ub2e4. \uac00\uc7a5 \ube60\ub978 \uc120\uc218\ub97c \ub9c8\uc9c0\ub9c9 \uc8fc\uc790\ub85c, \uadf8 \ub2e4\uc74c \ube60\ub978 \uc8fc\uc790\ub97c \uccab \ubc88\uc9f8 \uc8fc\uc790\ub85c \ud558\uace0, \ub450 \ubc88\uc9f8\u00b7\uc138 \ubc88\uc9f8 \uc8fc\uc790\ub294 \uc801\ub2f9\ud788 \ubc30\uce58\ud55c\ub2e4. \ucc98\uc74c \uc8fc\uc790\uac00 \uac70\ub9ac\ub97c \ubc8c\ub9b0 \ub2e4\uc74c, \ub450 \ubc88\uc9f8\u00b7\uc138 \ubc88\uc9f8 \uc8fc\uc790\uac00 \ub9ac\ub4dc\ub97c \ub2f9\ud558\ub354\ub77c\ub3c4 \ub9c8\uc9c0\ub9c9 \uc8fc\uc790\uac00 \ub9cc\ud68c\ub97c \ud560 \uc218 \uc788\uae30 \ub54c\ubb38\uc774\ub2e4. \ub2e4\uc74c\uc5d0\ub294 \ubc30\ud134\uc744 \uc8fc\uace0 \ubc1b\ub294 \ubc95\uc774 \uc911\uc694\ud558\ub2e4. \ubc30\ud134\uc740 \ub2e4\uc74c \uc8fc\uc790\uac00 \ubc1b\uae30 \uc27d\ub3c4\ub85d \uc804\uccb4 \uae38\uc774\uc758 2/3\ub97c \ub0a8\uae30\uace0 \uadf8 \uc544\ub7ab\ubd80\ubd84\uc744 \uc7a1\ub294\ub2e4. \ubc30\ud134\uc744 \ubc1b\uc744 \uc8fc\uc790\uac00 \uc900\ube44 \ub3d9\uc791\uc744 \ud560 \uc218 \uc788\ub294 \uac70\ub9ac\ub294 10m\uc758 \uc900\ube44 \uc9c0\uc5ed\uc744 \ud3ec\ud568\ud558\uc5ec 30m\uc774\uc9c0\ub9cc \ubc30\ud134\uc744 \uc8fc\uace0 \ubc1b\ub294 \ub3d9\uc791\uc740 \ubc18\ub4dc\uc2dc 20m \ubc30\ud134 \ud130\uce58 \uc874\uc5d0\uc11c \ud574\uc57c \ud55c\ub2e4. \ubc30\ud134\uc744 \uc8fc\ub294 \uc8fc\uc790\uc640 \ubc1b\ub294 \uc8fc\uc790\uc758 \uc18d\ub3c4\uac00 \uc77c\uce58\ud558\ub294 \uc9c0\uc810\uc5d0\uc11c \ubc30\ud134\uc744 \uc8fc\uace0 \ubc1b\ub294\uac83\uc774 \uac00\uc7a5 \ud6a8\uacfc\uc801\uc774\ub2e4. \ubc30\ud134\uc740 \uae38\uc774 28~30 cm, \ubb34\uac8c 50g \uc774\uc0c1\uc774\ub2e4. ", "paragraph_answer": "\uc774\uc5b4\ub2ec\ub9ac\uae30\ub294 \uac01 \uac1c\uc778\uc758 \uc18d\ub825\uc744 \ud569\ub9ac\uc801\uc73c\ub85c \uc5f0\uacb0\uc2dc\ud0a8 \ub2e8\uccb4 \uc6b4\ub3d9\uc73c\ub85c, \uacc4\uc8fc(\u7e7c\u8d70)\ub77c\uace0\ub3c4 \ud55c\ub2e4. \uc120\uc218 \uac1c\uac1c\uc778\ubcf4\ub2e4 4\uba85\uc758 \ud611\ub3d9\uc2ec\uacfc \ub2e8\uacb0\uc2ec\uc774 \uc694\uad6c\ub418\uba70, 400m\u00b7800m\u00b71500m \uc774\uc5b4\ub2ec\ub9ac\uae30\uac00 \uc788\ub2e4. \uac00\uc7a5 \ube60\ub978 \uc120\uc218\ub97c \ub9c8\uc9c0\ub9c9 \uc8fc\uc790\ub85c, \uadf8 \ub2e4\uc74c \ube60\ub978 \uc8fc\uc790\ub97c \uccab \ubc88\uc9f8 \uc8fc\uc790\ub85c \ud558\uace0, \ub450 \ubc88\uc9f8\u00b7\uc138 \ubc88\uc9f8 \uc8fc\uc790\ub294 \uc801\ub2f9\ud788 \ubc30\uce58\ud55c\ub2e4. \ucc98\uc74c \uc8fc\uc790\uac00 \uac70\ub9ac\ub97c \ubc8c\ub9b0 \ub2e4\uc74c, \ub450 \ubc88\uc9f8\u00b7\uc138 \ubc88\uc9f8 \uc8fc\uc790\uac00 \ub9ac\ub4dc\ub97c \ub2f9\ud558\ub354\ub77c\ub3c4 \ub9c8\uc9c0\ub9c9 \uc8fc\uc790\uac00 \ub9cc\ud68c\ub97c \ud560 \uc218 \uc788\uae30 \ub54c\ubb38\uc774\ub2e4. \ub2e4\uc74c\uc5d0\ub294 \ubc30\ud134\uc744 \uc8fc\uace0 \ubc1b\ub294 \ubc95\uc774 \uc911\uc694\ud558\ub2e4. \ubc30\ud134\uc740 \ub2e4\uc74c \uc8fc\uc790\uac00 \ubc1b\uae30 \uc27d\ub3c4\ub85d \uc804\uccb4 \uae38\uc774\uc758 2/3\ub97c \ub0a8\uae30\uace0 \uadf8 \uc544\ub7ab\ubd80\ubd84\uc744 \uc7a1\ub294\ub2e4. \ubc30\ud134\uc744 \ubc1b\uc744 \uc8fc\uc790\uac00 \uc900\ube44 \ub3d9\uc791\uc744 \ud560 \uc218 \uc788\ub294 \uac70\ub9ac\ub294 10m\uc758 \uc900\ube44 \uc9c0\uc5ed\uc744 \ud3ec\ud568\ud558\uc5ec 30m\uc774\uc9c0\ub9cc \ubc30\ud134\uc744 \uc8fc\uace0 \ubc1b\ub294 \ub3d9\uc791\uc740 \ubc18\ub4dc\uc2dc 20m \ubc30\ud134 \ud130\uce58 \uc874\uc5d0\uc11c \ud574\uc57c \ud55c\ub2e4. \ubc30\ud134\uc744 \uc8fc\ub294 \uc8fc\uc790\uc640 \ubc1b\ub294 \uc8fc\uc790\uc758 \uc18d\ub3c4\uac00 \uc77c\uce58\ud558\ub294 \uc9c0\uc810\uc5d0\uc11c \ubc30\ud134\uc744 \uc8fc\uace0 \ubc1b\ub294\uac83\uc774 \uac00\uc7a5 \ud6a8\uacfc\uc801\uc774\ub2e4. \ubc30\ud134\uc740 \uae38\uc774 28~30 cm, \ubb34\uac8c 50g \uc774\uc0c1\uc774\ub2e4.", "sentence_answer": "\ubc30\ud134\uc740 \uae38\uc774 28~30 cm, \ubb34\uac8c 50g \uc774\uc0c1\uc774\ub2e4.", "paragraph_id": "6543634-2-2", "paragraph_question": "question: \uacc4\uc8fc\uc5d0 \uc0ac\uc6a9\ub418\ub294 \ubc30\ud134 \ubb34\uac8c\ub294 \uba87 \uadf8\ub7a8 \uc774\uc0c1\uc778\uac00?, context: \uc774\uc5b4\ub2ec\ub9ac\uae30\ub294 \uac01 \uac1c\uc778\uc758 \uc18d\ub825\uc744 \ud569\ub9ac\uc801\uc73c\ub85c \uc5f0\uacb0\uc2dc\ud0a8 \ub2e8\uccb4 \uc6b4\ub3d9\uc73c\ub85c, \uacc4\uc8fc(\u7e7c\u8d70)\ub77c\uace0\ub3c4 \ud55c\ub2e4. \uc120\uc218 \uac1c\uac1c\uc778\ubcf4\ub2e4 4\uba85\uc758 \ud611\ub3d9\uc2ec\uacfc \ub2e8\uacb0\uc2ec\uc774 \uc694\uad6c\ub418\uba70, 400m\u00b7800m\u00b71500m \uc774\uc5b4\ub2ec\ub9ac\uae30\uac00 \uc788\ub2e4. \uac00\uc7a5 \ube60\ub978 \uc120\uc218\ub97c \ub9c8\uc9c0\ub9c9 \uc8fc\uc790\ub85c, \uadf8 \ub2e4\uc74c \ube60\ub978 \uc8fc\uc790\ub97c \uccab \ubc88\uc9f8 \uc8fc\uc790\ub85c \ud558\uace0, \ub450 \ubc88\uc9f8\u00b7\uc138 \ubc88\uc9f8 \uc8fc\uc790\ub294 \uc801\ub2f9\ud788 \ubc30\uce58\ud55c\ub2e4. \ucc98\uc74c \uc8fc\uc790\uac00 \uac70\ub9ac\ub97c \ubc8c\ub9b0 \ub2e4\uc74c, \ub450 \ubc88\uc9f8\u00b7\uc138 \ubc88\uc9f8 \uc8fc\uc790\uac00 \ub9ac\ub4dc\ub97c \ub2f9\ud558\ub354\ub77c\ub3c4 \ub9c8\uc9c0\ub9c9 \uc8fc\uc790\uac00 \ub9cc\ud68c\ub97c \ud560 \uc218 \uc788\uae30 \ub54c\ubb38\uc774\ub2e4. \ub2e4\uc74c\uc5d0\ub294 \ubc30\ud134\uc744 \uc8fc\uace0 \ubc1b\ub294 \ubc95\uc774 \uc911\uc694\ud558\ub2e4. \ubc30\ud134\uc740 \ub2e4\uc74c \uc8fc\uc790\uac00 \ubc1b\uae30 \uc27d\ub3c4\ub85d \uc804\uccb4 \uae38\uc774\uc758 2/3\ub97c \ub0a8\uae30\uace0 \uadf8 \uc544\ub7ab\ubd80\ubd84\uc744 \uc7a1\ub294\ub2e4. \ubc30\ud134\uc744 \ubc1b\uc744 \uc8fc\uc790\uac00 \uc900\ube44 \ub3d9\uc791\uc744 \ud560 \uc218 \uc788\ub294 \uac70\ub9ac\ub294 10m\uc758 \uc900\ube44 \uc9c0\uc5ed\uc744 \ud3ec\ud568\ud558\uc5ec 30m\uc774\uc9c0\ub9cc \ubc30\ud134\uc744 \uc8fc\uace0 \ubc1b\ub294 \ub3d9\uc791\uc740 \ubc18\ub4dc\uc2dc 20m \ubc30\ud134 \ud130\uce58 \uc874\uc5d0\uc11c \ud574\uc57c \ud55c\ub2e4. \ubc30\ud134\uc744 \uc8fc\ub294 \uc8fc\uc790\uc640 \ubc1b\ub294 \uc8fc\uc790\uc758 \uc18d\ub3c4\uac00 \uc77c\uce58\ud558\ub294 \uc9c0\uc810\uc5d0\uc11c \ubc30\ud134\uc744 \uc8fc\uace0 \ubc1b\ub294\uac83\uc774 \uac00\uc7a5 \ud6a8\uacfc\uc801\uc774\ub2e4. \ubc30\ud134\uc740 \uae38\uc774 28~30 cm, \ubb34\uac8c 50g \uc774\uc0c1\uc774\ub2e4."} {"question": "\uccbc\uc2dc\uc640\uc758 \uce5c\uc120 \uacbd\uae30\uc5d0\uc11c \uc778\uc800\ub9ac \ud0c0\uc784\uc5d0 \ubca0\ucef4\uc758 \uc624\ub978\ucabd \ubc1c\uc744 \ucc0d\uc740 \uc120\uc218\ub294?", "paragraph": "7\uc6d4 21\uc77c \ud1a0\uc694\uc77c \uc624\ud6c4, \uc544\uc9c1 \ud55c \ub2ec \uc804\uc5d0 \ub77c \ub9ac\uac00 \uc2dc\uc98c \ucd5c\uc885\uc804\uc5d0\uc11c \ubd80\uc0c1\ub2f9\ud55c \uc67c\ucabd \ubb34\ub98e \uc7ac\ud65c\uc5d0 \ub4e4\uc5b4\uac04 \uc640\uc911\uc5d0\ub3c4, \ubca0\ucef4\uc740 \uac24\ub7ed\uc2dc \ub370\ubdd4\uc804\uc744 \uce58\ub800\ub294\ub370, \uadf8\ub294 0-1\ub85c \ud328\ud55c \uccbc\uc2dc\uc640\uc758 \uc6d4\ub4dc \uc2dc\ub9ac\uc988 \uc624\ube0c \uc0ac\ucee4 \uce5c\uc120 \uacbd\uae30\uc5d0\uc11c 78\ubd84\uc5d0 \uc568\ub7f0 \uace0\ub4e0\uacfc \uad50\uccb4\ub418\uc5b4 \ub4e4\uc5b4\uac00 \ucd9c\uc804\ud588\ub2e4. \ud5d0\ub9ac\uc6b0\ub4dc \uc720\uba85\uc778\uc0ac\uc778 \ud1b0 \ud06c\ub8e8\uc988, \ucf00\uc774\ud2f0 \ud648\uc988, \uc5d0\ubc14 \ub871\uace0\ub9ac\uc544, \uc544\ub110\ub4dc \uc288\uc6cc\uc81c\ub124\uac70 \uce98\ub9ac\ud3ec\ub2c8\uc544 \uc8fc\uc9c0\uc0ac, \uadf8\ub9ac\uace0 \ub4dc\ub8e8 \ucf00\ub9ac \ub4f1\uc744 \ud3ec\ud568\ud55c \ud648\ub514\ud3ec \uc13c\ud130\uc758 \ub9cc\uc6d0 \uad00\uc911 \uc55e\uc5d0\uc11c \uacbd\uae30\ub294 ESPN \ubcf8\ubc29\uc1a1\uc73c\ub85c \uc0dd\uc911\uacc4\ub418\uc5c8\ub2e4. \uadf8\ub7ec\ub098, \uc138\uacc4\uad6c\uae09 \ucd95\uad6c \uac70\ubb3c\ub4e4\uc778 \uc548\ub4dc\ub9ac \uc181\uccb8\ucf54, \ub514\ub514\uc5d0 \ub4dc\ub85c\uadf8\ubc14, \ubbf8\ud558\uc5d8 \ubc1c\ub77c\ud06c, \uadf8\ub9ac\uace0 \ud504\ub7ad\ud06c \ub7a8\ud37c\ub4dc \ub4f1\uc774 \ucd9c\uc804\ud588\uc9c0\ub9cc, \ubbf8\uad6d \uc911\uacc4 \uce74\uba54\ub77c\ub294 \ud6c4\ubcf4\ub85c \ub300\ubd80\ubd84 \uc2dc\uac04\uc744 \ub300\uae30\ud558\ub358 \ubca0\ucef4\uc5d0 \uc9d1\uc911\ud588\uace0, \ud604\uc7a5 \uc911\uacc4\ub294 \ub4b7\uc804\uc774\uc5c8\ub2e4. \uacbd\uae30 \ucd94\uac00 \uc2dc\uac04\uc740 \uc9e7\uac8c \uc9c4\ud589\ub418\uc5c8\ub2e4, \uc774\ub294 \ubd80\uc0c1\ub2f9\ud55c \ubca0\ucef4\uc774 \uc2a4\ud2f0\ube0c \uc2dc\ub4dc\uc6f0\uc5d0\uac8c \uac78\ub824\ub118\uc9c4 \uac83\uc73c\ub85c \uc9c4\ud589\ub418\uc5c8\uace0, \uc2dc\ub4dc\uc6f0\uc758 \ucd95\uad6c\ud654 \uc9d5\uc740 \ubca0\ucef4\uc758 \uc624\ub978\ucabd \ubc1c\uc744 \ucc0d\uc5c8\uace0, \ubca0\ucef4\uc740 \uacf5\uc911\uc5d0 \uc911 \ub5a0 \uacbd\uae30\uc7a5 \ud45c\uba74\uc5d0 \ub110\ubd80\ub7ec\uc84c\ub2e4. \ube44\ub85d \uc804\uc5d0 \ub2f9\ud55c \ubd80\uc0c1\uc740 \ud06c\uac8c \uc545\ud654\ub418\uc9c0 \uc54a\uc558\uc9c0\ub9cc, \ubca0\ucef4\uc758 \uc7ac\ud65c \uae30\uac04\uc740 1\uc8fc \ub354 \uae38\uc5b4\uc84c\ub2e4. ESPN\uc758 \ubca0\ucef4 \ub370\ubdd4\uc804\uc740 1.0 TV \ud3c9\uc810\uc744 \ubc1b\uc558\ub294\ub370, \uadf8 \ub73b\uc740 \ud3c9\uade0 \ubbf8\uad6d\uc758 947,000\uac00\uad6c\uac00 \ud154\ub808\ube44\uc804\uc744 \ud1b5\ud574 \uc2dc\uccad\ud588\ub2e4\ub294 \uac83\uc73c\ub85c, \uc804\uad6d\uc801\uc73c\ub85c \ub9e4\uccb4\uc758 \uad00\uc2ec\uc744 \uae01\uc5b4\ubaa8\uc73c\uace0 ESPN\uc758 2\uc8fc\uc5d0 \uac78\uccd0 \uc2ec\ud608\uc744 \uae30\uc6b8\uc778 \ud64d\ubcf4\uc5d0 \ube44\ud558\uba74 \uc2e4\ub9dd\uc2a4\ub7ec\uc6b4 \uc218\uce58\uc600\ub2e4. \ubbf8\uad6d\uc758 \ud0c0 \uc885\ubaa9 \ub610\ud55c \uadf8 \uc8fc \uc8fc\ub9d0\uc5d0 \uc911\uacc4\ub418\uc5c8\uae30\uc5d0, \ubca0\ucef4 \uacbd\uae30\uc758 \uc2dc\uccad\ub960\uc744 \ube0c\ub9ac\ud2f0\uc2dc \uc624\ud508 \uace8\ud504 \ub300\ud68c, \uc815\uaddc \uc2dc\uc98c \uc804\uad6d \uc57c\uad6c \uacbd\uae30, \uadf8\ub9ac\uace0 \uc778\ub514 \ub808\uc774\uc2f1 \ub9ac\uadf8\uc758 \ud63c\ub2e4 200 \uc790\ub3d9\ucc28 \uacbd\uc8fc\ubcf4\ub2e4 \uc801\uc740 \uc2dc\uccad\uc728\uc744 \uae30\ub85d\ud588\ub2e4.", "answer": "\uc2a4\ud2f0\ube0c \uc2dc\ub4dc\uc6f0", "sentence": "\uacbd\uae30 \ucd94\uac00 \uc2dc\uac04\uc740 \uc9e7\uac8c \uc9c4\ud589\ub418\uc5c8\ub2e4, \uc774\ub294 \ubd80\uc0c1\ub2f9\ud55c \ubca0\ucef4\uc774 \uc2a4\ud2f0\ube0c \uc2dc\ub4dc\uc6f0 \uc5d0\uac8c \uac78\ub824\ub118\uc9c4 \uac83\uc73c\ub85c \uc9c4\ud589\ub418\uc5c8\uace0, \uc2dc\ub4dc\uc6f0\uc758 \ucd95\uad6c\ud654 \uc9d5\uc740 \ubca0\ucef4\uc758 \uc624\ub978\ucabd \ubc1c\uc744 \ucc0d\uc5c8\uace0, \ubca0\ucef4\uc740 \uacf5\uc911\uc5d0 \uc911 \ub5a0 \uacbd\uae30\uc7a5 \ud45c\uba74\uc5d0 \ub110\ubd80\ub7ec\uc84c\ub2e4.", "paragraph_sentence": "7\uc6d4 21\uc77c \ud1a0\uc694\uc77c \uc624\ud6c4, \uc544\uc9c1 \ud55c \ub2ec \uc804\uc5d0 \ub77c \ub9ac\uac00 \uc2dc\uc98c \ucd5c\uc885\uc804\uc5d0\uc11c \ubd80\uc0c1\ub2f9\ud55c \uc67c\ucabd \ubb34\ub98e \uc7ac\ud65c\uc5d0 \ub4e4\uc5b4\uac04 \uc640\uc911\uc5d0\ub3c4, \ubca0\ucef4\uc740 \uac24\ub7ed\uc2dc \ub370\ubdd4\uc804\uc744 \uce58\ub800\ub294\ub370, \uadf8\ub294 0-1\ub85c \ud328\ud55c \uccbc\uc2dc\uc640\uc758 \uc6d4\ub4dc \uc2dc\ub9ac\uc988 \uc624\ube0c \uc0ac\ucee4 \uce5c\uc120 \uacbd\uae30\uc5d0\uc11c 78\ubd84\uc5d0 \uc568\ub7f0 \uace0\ub4e0\uacfc \uad50\uccb4\ub418\uc5b4 \ub4e4\uc5b4\uac00 \ucd9c\uc804\ud588\ub2e4. \ud5d0\ub9ac\uc6b0\ub4dc \uc720\uba85\uc778\uc0ac\uc778 \ud1b0 \ud06c\ub8e8\uc988, \ucf00\uc774\ud2f0 \ud648\uc988, \uc5d0\ubc14 \ub871\uace0\ub9ac\uc544, \uc544\ub110\ub4dc \uc288\uc6cc\uc81c\ub124\uac70 \uce98\ub9ac\ud3ec\ub2c8\uc544 \uc8fc\uc9c0\uc0ac, \uadf8\ub9ac\uace0 \ub4dc\ub8e8 \ucf00\ub9ac \ub4f1\uc744 \ud3ec\ud568\ud55c \ud648\ub514\ud3ec \uc13c\ud130\uc758 \ub9cc\uc6d0 \uad00\uc911 \uc55e\uc5d0\uc11c \uacbd\uae30\ub294 ESPN \ubcf8\ubc29\uc1a1\uc73c\ub85c \uc0dd\uc911\uacc4\ub418\uc5c8\ub2e4. \uadf8\ub7ec\ub098, \uc138\uacc4\uad6c\uae09 \ucd95\uad6c \uac70\ubb3c\ub4e4\uc778 \uc548\ub4dc\ub9ac \uc181\uccb8\ucf54, \ub514\ub514\uc5d0 \ub4dc\ub85c\uadf8\ubc14, \ubbf8\ud558\uc5d8 \ubc1c\ub77c\ud06c, \uadf8\ub9ac\uace0 \ud504\ub7ad\ud06c \ub7a8\ud37c\ub4dc \ub4f1\uc774 \ucd9c\uc804\ud588\uc9c0\ub9cc, \ubbf8\uad6d \uc911\uacc4 \uce74\uba54\ub77c\ub294 \ud6c4\ubcf4\ub85c \ub300\ubd80\ubd84 \uc2dc\uac04\uc744 \ub300\uae30\ud558\ub358 \ubca0\ucef4\uc5d0 \uc9d1\uc911\ud588\uace0, \ud604\uc7a5 \uc911\uacc4\ub294 \ub4b7\uc804\uc774\uc5c8\ub2e4. \uacbd\uae30 \ucd94\uac00 \uc2dc\uac04\uc740 \uc9e7\uac8c \uc9c4\ud589\ub418\uc5c8\ub2e4, \uc774\ub294 \ubd80\uc0c1\ub2f9\ud55c \ubca0\ucef4\uc774 \uc2a4\ud2f0\ube0c \uc2dc\ub4dc\uc6f0 \uc5d0\uac8c \uac78\ub824\ub118\uc9c4 \uac83\uc73c\ub85c \uc9c4\ud589\ub418\uc5c8\uace0, \uc2dc\ub4dc\uc6f0\uc758 \ucd95\uad6c\ud654 \uc9d5\uc740 \ubca0\ucef4\uc758 \uc624\ub978\ucabd \ubc1c\uc744 \ucc0d\uc5c8\uace0, \ubca0\ucef4\uc740 \uacf5\uc911\uc5d0 \uc911 \ub5a0 \uacbd\uae30\uc7a5 \ud45c\uba74\uc5d0 \ub110\ubd80\ub7ec\uc84c\ub2e4. \ube44\ub85d \uc804\uc5d0 \ub2f9\ud55c \ubd80\uc0c1\uc740 \ud06c\uac8c \uc545\ud654\ub418\uc9c0 \uc54a\uc558\uc9c0\ub9cc, \ubca0\ucef4\uc758 \uc7ac\ud65c \uae30\uac04\uc740 1\uc8fc \ub354 \uae38\uc5b4\uc84c\ub2e4. ESPN\uc758 \ubca0\ucef4 \ub370\ubdd4\uc804\uc740 1.0 TV \ud3c9\uc810\uc744 \ubc1b\uc558\ub294\ub370, \uadf8 \ub73b\uc740 \ud3c9\uade0 \ubbf8\uad6d\uc758 947,000\uac00\uad6c\uac00 \ud154\ub808\ube44\uc804\uc744 \ud1b5\ud574 \uc2dc\uccad\ud588\ub2e4\ub294 \uac83\uc73c\ub85c, \uc804\uad6d\uc801\uc73c\ub85c \ub9e4\uccb4\uc758 \uad00\uc2ec\uc744 \uae01\uc5b4\ubaa8\uc73c\uace0 ESPN\uc758 2\uc8fc\uc5d0 \uac78\uccd0 \uc2ec\ud608\uc744 \uae30\uc6b8\uc778 \ud64d\ubcf4\uc5d0 \ube44\ud558\uba74 \uc2e4\ub9dd\uc2a4\ub7ec\uc6b4 \uc218\uce58\uc600\ub2e4. \ubbf8\uad6d\uc758 \ud0c0 \uc885\ubaa9 \ub610\ud55c \uadf8 \uc8fc \uc8fc\ub9d0\uc5d0 \uc911\uacc4\ub418\uc5c8\uae30\uc5d0, \ubca0\ucef4 \uacbd\uae30\uc758 \uc2dc\uccad\ub960\uc744 \ube0c\ub9ac\ud2f0\uc2dc \uc624\ud508 \uace8\ud504 \ub300\ud68c, \uc815\uaddc \uc2dc\uc98c \uc804\uad6d \uc57c\uad6c \uacbd\uae30, \uadf8\ub9ac\uace0 \uc778\ub514 \ub808\uc774\uc2f1 \ub9ac\uadf8\uc758 \ud63c\ub2e4 200 \uc790\ub3d9\ucc28 \uacbd\uc8fc\ubcf4\ub2e4 \uc801\uc740 \uc2dc\uccad\uc728\uc744 \uae30\ub85d\ud588\ub2e4.", "paragraph_answer": "7\uc6d4 21\uc77c \ud1a0\uc694\uc77c \uc624\ud6c4, \uc544\uc9c1 \ud55c \ub2ec \uc804\uc5d0 \ub77c \ub9ac\uac00 \uc2dc\uc98c \ucd5c\uc885\uc804\uc5d0\uc11c \ubd80\uc0c1\ub2f9\ud55c \uc67c\ucabd \ubb34\ub98e \uc7ac\ud65c\uc5d0 \ub4e4\uc5b4\uac04 \uc640\uc911\uc5d0\ub3c4, \ubca0\ucef4\uc740 \uac24\ub7ed\uc2dc \ub370\ubdd4\uc804\uc744 \uce58\ub800\ub294\ub370, \uadf8\ub294 0-1\ub85c \ud328\ud55c \uccbc\uc2dc\uc640\uc758 \uc6d4\ub4dc \uc2dc\ub9ac\uc988 \uc624\ube0c \uc0ac\ucee4 \uce5c\uc120 \uacbd\uae30\uc5d0\uc11c 78\ubd84\uc5d0 \uc568\ub7f0 \uace0\ub4e0\uacfc \uad50\uccb4\ub418\uc5b4 \ub4e4\uc5b4\uac00 \ucd9c\uc804\ud588\ub2e4. \ud5d0\ub9ac\uc6b0\ub4dc \uc720\uba85\uc778\uc0ac\uc778 \ud1b0 \ud06c\ub8e8\uc988, \ucf00\uc774\ud2f0 \ud648\uc988, \uc5d0\ubc14 \ub871\uace0\ub9ac\uc544, \uc544\ub110\ub4dc \uc288\uc6cc\uc81c\ub124\uac70 \uce98\ub9ac\ud3ec\ub2c8\uc544 \uc8fc\uc9c0\uc0ac, \uadf8\ub9ac\uace0 \ub4dc\ub8e8 \ucf00\ub9ac \ub4f1\uc744 \ud3ec\ud568\ud55c \ud648\ub514\ud3ec \uc13c\ud130\uc758 \ub9cc\uc6d0 \uad00\uc911 \uc55e\uc5d0\uc11c \uacbd\uae30\ub294 ESPN \ubcf8\ubc29\uc1a1\uc73c\ub85c \uc0dd\uc911\uacc4\ub418\uc5c8\ub2e4. \uadf8\ub7ec\ub098, \uc138\uacc4\uad6c\uae09 \ucd95\uad6c \uac70\ubb3c\ub4e4\uc778 \uc548\ub4dc\ub9ac \uc181\uccb8\ucf54, \ub514\ub514\uc5d0 \ub4dc\ub85c\uadf8\ubc14, \ubbf8\ud558\uc5d8 \ubc1c\ub77c\ud06c, \uadf8\ub9ac\uace0 \ud504\ub7ad\ud06c \ub7a8\ud37c\ub4dc \ub4f1\uc774 \ucd9c\uc804\ud588\uc9c0\ub9cc, \ubbf8\uad6d \uc911\uacc4 \uce74\uba54\ub77c\ub294 \ud6c4\ubcf4\ub85c \ub300\ubd80\ubd84 \uc2dc\uac04\uc744 \ub300\uae30\ud558\ub358 \ubca0\ucef4\uc5d0 \uc9d1\uc911\ud588\uace0, \ud604\uc7a5 \uc911\uacc4\ub294 \ub4b7\uc804\uc774\uc5c8\ub2e4. \uacbd\uae30 \ucd94\uac00 \uc2dc\uac04\uc740 \uc9e7\uac8c \uc9c4\ud589\ub418\uc5c8\ub2e4, \uc774\ub294 \ubd80\uc0c1\ub2f9\ud55c \ubca0\ucef4\uc774 \uc2a4\ud2f0\ube0c \uc2dc\ub4dc\uc6f0 \uc5d0\uac8c \uac78\ub824\ub118\uc9c4 \uac83\uc73c\ub85c \uc9c4\ud589\ub418\uc5c8\uace0, \uc2dc\ub4dc\uc6f0\uc758 \ucd95\uad6c\ud654 \uc9d5\uc740 \ubca0\ucef4\uc758 \uc624\ub978\ucabd \ubc1c\uc744 \ucc0d\uc5c8\uace0, \ubca0\ucef4\uc740 \uacf5\uc911\uc5d0 \uc911 \ub5a0 \uacbd\uae30\uc7a5 \ud45c\uba74\uc5d0 \ub110\ubd80\ub7ec\uc84c\ub2e4. \ube44\ub85d \uc804\uc5d0 \ub2f9\ud55c \ubd80\uc0c1\uc740 \ud06c\uac8c \uc545\ud654\ub418\uc9c0 \uc54a\uc558\uc9c0\ub9cc, \ubca0\ucef4\uc758 \uc7ac\ud65c \uae30\uac04\uc740 1\uc8fc \ub354 \uae38\uc5b4\uc84c\ub2e4. ESPN\uc758 \ubca0\ucef4 \ub370\ubdd4\uc804\uc740 1.0 TV \ud3c9\uc810\uc744 \ubc1b\uc558\ub294\ub370, \uadf8 \ub73b\uc740 \ud3c9\uade0 \ubbf8\uad6d\uc758 947,000\uac00\uad6c\uac00 \ud154\ub808\ube44\uc804\uc744 \ud1b5\ud574 \uc2dc\uccad\ud588\ub2e4\ub294 \uac83\uc73c\ub85c, \uc804\uad6d\uc801\uc73c\ub85c \ub9e4\uccb4\uc758 \uad00\uc2ec\uc744 \uae01\uc5b4\ubaa8\uc73c\uace0 ESPN\uc758 2\uc8fc\uc5d0 \uac78\uccd0 \uc2ec\ud608\uc744 \uae30\uc6b8\uc778 \ud64d\ubcf4\uc5d0 \ube44\ud558\uba74 \uc2e4\ub9dd\uc2a4\ub7ec\uc6b4 \uc218\uce58\uc600\ub2e4. \ubbf8\uad6d\uc758 \ud0c0 \uc885\ubaa9 \ub610\ud55c \uadf8 \uc8fc \uc8fc\ub9d0\uc5d0 \uc911\uacc4\ub418\uc5c8\uae30\uc5d0, \ubca0\ucef4 \uacbd\uae30\uc758 \uc2dc\uccad\ub960\uc744 \ube0c\ub9ac\ud2f0\uc2dc \uc624\ud508 \uace8\ud504 \ub300\ud68c, \uc815\uaddc \uc2dc\uc98c \uc804\uad6d \uc57c\uad6c \uacbd\uae30, \uadf8\ub9ac\uace0 \uc778\ub514 \ub808\uc774\uc2f1 \ub9ac\uadf8\uc758 \ud63c\ub2e4 200 \uc790\ub3d9\ucc28 \uacbd\uc8fc\ubcf4\ub2e4 \uc801\uc740 \uc2dc\uccad\uc728\uc744 \uae30\ub85d\ud588\ub2e4.", "sentence_answer": "\uacbd\uae30 \ucd94\uac00 \uc2dc\uac04\uc740 \uc9e7\uac8c \uc9c4\ud589\ub418\uc5c8\ub2e4, \uc774\ub294 \ubd80\uc0c1\ub2f9\ud55c \ubca0\ucef4\uc774 \uc2a4\ud2f0\ube0c \uc2dc\ub4dc\uc6f0 \uc5d0\uac8c \uac78\ub824\ub118\uc9c4 \uac83\uc73c\ub85c \uc9c4\ud589\ub418\uc5c8\uace0, \uc2dc\ub4dc\uc6f0\uc758 \ucd95\uad6c\ud654 \uc9d5\uc740 \ubca0\ucef4\uc758 \uc624\ub978\ucabd \ubc1c\uc744 \ucc0d\uc5c8\uace0, \ubca0\ucef4\uc740 \uacf5\uc911\uc5d0 \uc911 \ub5a0 \uacbd\uae30\uc7a5 \ud45c\uba74\uc5d0 \ub110\ubd80\ub7ec\uc84c\ub2e4.", "paragraph_id": "6528559-20-1", "paragraph_question": "question: \uccbc\uc2dc\uc640\uc758 \uce5c\uc120 \uacbd\uae30\uc5d0\uc11c \uc778\uc800\ub9ac \ud0c0\uc784\uc5d0 \ubca0\ucef4\uc758 \uc624\ub978\ucabd \ubc1c\uc744 \ucc0d\uc740 \uc120\uc218\ub294?, context: 7\uc6d4 21\uc77c \ud1a0\uc694\uc77c \uc624\ud6c4, \uc544\uc9c1 \ud55c \ub2ec \uc804\uc5d0 \ub77c \ub9ac\uac00 \uc2dc\uc98c \ucd5c\uc885\uc804\uc5d0\uc11c \ubd80\uc0c1\ub2f9\ud55c \uc67c\ucabd \ubb34\ub98e \uc7ac\ud65c\uc5d0 \ub4e4\uc5b4\uac04 \uc640\uc911\uc5d0\ub3c4, \ubca0\ucef4\uc740 \uac24\ub7ed\uc2dc \ub370\ubdd4\uc804\uc744 \uce58\ub800\ub294\ub370, \uadf8\ub294 0-1\ub85c \ud328\ud55c \uccbc\uc2dc\uc640\uc758 \uc6d4\ub4dc \uc2dc\ub9ac\uc988 \uc624\ube0c \uc0ac\ucee4 \uce5c\uc120 \uacbd\uae30\uc5d0\uc11c 78\ubd84\uc5d0 \uc568\ub7f0 \uace0\ub4e0\uacfc \uad50\uccb4\ub418\uc5b4 \ub4e4\uc5b4\uac00 \ucd9c\uc804\ud588\ub2e4. \ud5d0\ub9ac\uc6b0\ub4dc \uc720\uba85\uc778\uc0ac\uc778 \ud1b0 \ud06c\ub8e8\uc988, \ucf00\uc774\ud2f0 \ud648\uc988, \uc5d0\ubc14 \ub871\uace0\ub9ac\uc544, \uc544\ub110\ub4dc \uc288\uc6cc\uc81c\ub124\uac70 \uce98\ub9ac\ud3ec\ub2c8\uc544 \uc8fc\uc9c0\uc0ac, \uadf8\ub9ac\uace0 \ub4dc\ub8e8 \ucf00\ub9ac \ub4f1\uc744 \ud3ec\ud568\ud55c \ud648\ub514\ud3ec \uc13c\ud130\uc758 \ub9cc\uc6d0 \uad00\uc911 \uc55e\uc5d0\uc11c \uacbd\uae30\ub294 ESPN \ubcf8\ubc29\uc1a1\uc73c\ub85c \uc0dd\uc911\uacc4\ub418\uc5c8\ub2e4. \uadf8\ub7ec\ub098, \uc138\uacc4\uad6c\uae09 \ucd95\uad6c \uac70\ubb3c\ub4e4\uc778 \uc548\ub4dc\ub9ac \uc181\uccb8\ucf54, \ub514\ub514\uc5d0 \ub4dc\ub85c\uadf8\ubc14, \ubbf8\ud558\uc5d8 \ubc1c\ub77c\ud06c, \uadf8\ub9ac\uace0 \ud504\ub7ad\ud06c \ub7a8\ud37c\ub4dc \ub4f1\uc774 \ucd9c\uc804\ud588\uc9c0\ub9cc, \ubbf8\uad6d \uc911\uacc4 \uce74\uba54\ub77c\ub294 \ud6c4\ubcf4\ub85c \ub300\ubd80\ubd84 \uc2dc\uac04\uc744 \ub300\uae30\ud558\ub358 \ubca0\ucef4\uc5d0 \uc9d1\uc911\ud588\uace0, \ud604\uc7a5 \uc911\uacc4\ub294 \ub4b7\uc804\uc774\uc5c8\ub2e4. \uacbd\uae30 \ucd94\uac00 \uc2dc\uac04\uc740 \uc9e7\uac8c \uc9c4\ud589\ub418\uc5c8\ub2e4, \uc774\ub294 \ubd80\uc0c1\ub2f9\ud55c \ubca0\ucef4\uc774 \uc2a4\ud2f0\ube0c \uc2dc\ub4dc\uc6f0\uc5d0\uac8c \uac78\ub824\ub118\uc9c4 \uac83\uc73c\ub85c \uc9c4\ud589\ub418\uc5c8\uace0, \uc2dc\ub4dc\uc6f0\uc758 \ucd95\uad6c\ud654 \uc9d5\uc740 \ubca0\ucef4\uc758 \uc624\ub978\ucabd \ubc1c\uc744 \ucc0d\uc5c8\uace0, \ubca0\ucef4\uc740 \uacf5\uc911\uc5d0 \uc911 \ub5a0 \uacbd\uae30\uc7a5 \ud45c\uba74\uc5d0 \ub110\ubd80\ub7ec\uc84c\ub2e4. \ube44\ub85d \uc804\uc5d0 \ub2f9\ud55c \ubd80\uc0c1\uc740 \ud06c\uac8c \uc545\ud654\ub418\uc9c0 \uc54a\uc558\uc9c0\ub9cc, \ubca0\ucef4\uc758 \uc7ac\ud65c \uae30\uac04\uc740 1\uc8fc \ub354 \uae38\uc5b4\uc84c\ub2e4. ESPN\uc758 \ubca0\ucef4 \ub370\ubdd4\uc804\uc740 1.0 TV \ud3c9\uc810\uc744 \ubc1b\uc558\ub294\ub370, \uadf8 \ub73b\uc740 \ud3c9\uade0 \ubbf8\uad6d\uc758 947,000\uac00\uad6c\uac00 \ud154\ub808\ube44\uc804\uc744 \ud1b5\ud574 \uc2dc\uccad\ud588\ub2e4\ub294 \uac83\uc73c\ub85c, \uc804\uad6d\uc801\uc73c\ub85c \ub9e4\uccb4\uc758 \uad00\uc2ec\uc744 \uae01\uc5b4\ubaa8\uc73c\uace0 ESPN\uc758 2\uc8fc\uc5d0 \uac78\uccd0 \uc2ec\ud608\uc744 \uae30\uc6b8\uc778 \ud64d\ubcf4\uc5d0 \ube44\ud558\uba74 \uc2e4\ub9dd\uc2a4\ub7ec\uc6b4 \uc218\uce58\uc600\ub2e4. \ubbf8\uad6d\uc758 \ud0c0 \uc885\ubaa9 \ub610\ud55c \uadf8 \uc8fc \uc8fc\ub9d0\uc5d0 \uc911\uacc4\ub418\uc5c8\uae30\uc5d0, \ubca0\ucef4 \uacbd\uae30\uc758 \uc2dc\uccad\ub960\uc744 \ube0c\ub9ac\ud2f0\uc2dc \uc624\ud508 \uace8\ud504 \ub300\ud68c, \uc815\uaddc \uc2dc\uc98c \uc804\uad6d \uc57c\uad6c \uacbd\uae30, \uadf8\ub9ac\uace0 \uc778\ub514 \ub808\uc774\uc2f1 \ub9ac\uadf8\uc758 \ud63c\ub2e4 200 \uc790\ub3d9\ucc28 \uacbd\uc8fc\ubcf4\ub2e4 \uc801\uc740 \uc2dc\uccad\uc728\uc744 \uae30\ub85d\ud588\ub2e4."} {"question": "\ub958\uc900\uc5f4\uc774 \uc601\ud654 \uae00\ub85c\ub9ac\ub370\uc774\ub85c \uc778\ud574 \ucd08\uccad\ub41c \uc601\ud654\uc81c \uc774\ub984\uc740?", "paragraph": "\ub958\uc900\uc5f4\uc740 2015\ub144 6\uc6d4, KBS 2TV \ub4dc\ub77c\ub9c8 \u300a\ud504\ub85c\ub4c0\uc0ac\u300b\uc5d0\uc11c \ubc31\uc2b9\ucc2c(\uae40\uc218\ud604)\uc758 \ubc29\uc1a1\uad6d \uc785\uc0ac \ub3d9\uae30 \uc8fc\uc885\ud604 \uc5ed\ud560\uc744 \uc5f0\uae30\ud588\ub2e4. 2015\ub144 7\uc6d4, \ub958\uc900\uc5f4\uc740 \uc528\uc81c\uc2a4 \uc5d4\ud130\ud14c\uc778\uba3c\ud2b8\uc640 \ub9e4\ub2c8\uc9c0\uba3c\ud2b8 \uc804\uc18d \uacc4\uc57d\uc744 \ub9fa\uc5c8\ub2e4. 2015\ub144 10\uc6d4 \ub958\uc900\uc5f4\uc740 \ucd5c\uc815\uc5f4 \uac10\ub3c5\uc758 \uc601\ud654 \u300a\uae00\ub85c\ub9ac\ub370\uc774\u300b\uc5d0\uc11c \uc785\ub300\ud558\ub294 \uce5c\uad6c\ub97c \ubc30\uc6c5\ud558\ub7ec \uc5ec\ud589\uc744 \ub5a0\ub0ac\ub2e4\uac00 \ub73b\ubc16\uc758 \uc0ac\uac74\uc744 \ub9c8\uc8fc\ud558\uac8c \ub41c \ub124 \uba85\uc758 \uc2a4\ubb34 \uc0b4 \uce5c\uad6c\ub4e4 \uc911\uc5d0\uc11c \uc704\uae30\uc758 \uc21c\uac04 \uacb0\uc815\uc801 \uc120\ud0dd\uc744 \uc81c\uc548\ud558\ub294 \uc9c0\uacf5 \uc5ed\ud560\uc744 \uc5f0\uae30\ud55c \uc774 \uc601\ud654\ub85c \uc81c20\ud68c \ubd80\uc0b0\uad6d\uc81c\uc601\ud654\uc81c\uc758 \ud55c\uad6d\uc601\ud654\uc758 \uc624\ub298 - \ud30c\ub178\ub77c\ub9c8 \uc139\uc158\ubd80\ubb38\uc5d0 \ucd08\uccad\ub418\uba74\uc11c, \ucd9c\uc5f0 \ubc30\uc6b0 \uc790\uaca9\uc73c\ub85c \ub808\ub4dc\uce74\ud3ab\uc744 \ubc1f\uc558\ub2e4. 2015\ub144 12\uc6d4, \u300a\uc18c\uc15c\ud3ec\ube44\uc544\u300b\ub85c KT&G \uc0c1\uc0c1\ub9c8\ub2f9 \uc2dc\ub124\ub9c8\uac00 2015\ub144 \uc2a4\ud06c\ub9b0\uc5d0\uc11c \uac00\uc7a5 \ubc18\uc9dd\uc778 \uc2a4\ud0c0\uc640 2016\ub144 \uae30\ub300\uc8fc\ub97c \uc18c\uac1c\ud558\ub294 \uc5f0\ub9d0\uae30\ud68d\uc804 \u30082015 \uc528\ub124 \uc544\uc774\ucf58: KT&G \uc0c1\uc0c1\ub9c8\ub2f9 \ubc30\uc6b0\uae30\ud68d\uc804\u3009\uc5d0\uc11c \uc2a4\ud06c\ub9b0\uc758 \uc0c8\ub85c\uc6b4 \uc2a4\ud0c0\uc778 \u3008\uc528\ub124 \uc544\uc774\ucf58\u3009\uc5d0 \uc120\uc815\ub418\uae30\ub3c4 \ud558\uc600\ub2e4. 2015\ub144 5\uc6d4 18\uc77c, \ub958\uc900\uc5f4\uc774 TvN \uae08\ud1a0\ub4dc\ub77c\ub9c8 \u300a\uc751\ub2f5\ud558\ub77c 1988\u300b\uc758 \uae40\uc815\ud658 \uc5ed\ud560\uc5d0 \uce90\uc2a4\ud305\ub41c \uac83\uc774 \ubc1c\ud45c\ub418\uc5c8\ub2e4. \uc774 \ub4dc\ub77c\ub9c8\ub294 2015\ub144 11\uc6d4 \ucd08\uc5d0 \uccab \ubc29\uc601 \ub418\uc5c8\uace0, \ub958\uc900\uc5f4\uc740 \uadf9 \uc911 \uac89\ubaa8\uc2b5\uc740 \ubb34\uc2ec\ud55c \ub4ef\ud558\uc9c0\ub9cc \ub0b4\uba74\uc740 \ub530\ub73b\ud558\uace0 \uc18d \uae4a\uc740 \uce90\ub9ad\ud130\ub97c \uc5f0\uae30\ud588\uace0, \uc2e0\uc778\ub2f5\uc9c0 \uc54a\uc740 \uc548\uc815\ub41c \uc5f0\uae30\ub825\uacfc \uc5ec\uc2ec\uc744 \uc0ac\ub85c\uc7a1\ub294 \ub9e4\ub825\uc73c\ub85c \uc8fc\ubaa9\uc744 \ubc1b\uc73c\uba70 \ub9ce\uc740 \uc0ac\ub791\uc744 \ubc1b\uc558\ub2e4. \uc774 \uc791\ud488\uc73c\ub85c \uc81c52\ud68c \ubc31\uc0c1\uc608\uc220\ub300\uc0c1 TV\ubd80\ubb38 \uc2e0\uc778\uc0c1\uc744 \uc218\uc0c1\ud588\ub2e4.", "answer": "\ubd80\uc0b0\uad6d\uc81c\uc601\ud654\uc81c", "sentence": "2015\ub144 10\uc6d4 \ub958\uc900\uc5f4\uc740 \ucd5c\uc815\uc5f4 \uac10\ub3c5\uc758 \uc601\ud654 \u300a\uae00\ub85c\ub9ac\ub370\uc774\u300b\uc5d0\uc11c \uc785\ub300\ud558\ub294 \uce5c\uad6c\ub97c \ubc30\uc6c5\ud558\ub7ec \uc5ec\ud589\uc744 \ub5a0\ub0ac\ub2e4\uac00 \ub73b\ubc16\uc758 \uc0ac\uac74\uc744 \ub9c8\uc8fc\ud558\uac8c \ub41c \ub124 \uba85\uc758 \uc2a4\ubb34 \uc0b4 \uce5c\uad6c\ub4e4 \uc911\uc5d0\uc11c \uc704\uae30\uc758 \uc21c\uac04 \uacb0\uc815\uc801 \uc120\ud0dd\uc744 \uc81c\uc548\ud558\ub294 \uc9c0\uacf5 \uc5ed\ud560\uc744 \uc5f0\uae30\ud55c \uc774 \uc601\ud654\ub85c \uc81c20\ud68c \ubd80\uc0b0\uad6d\uc81c\uc601\ud654\uc81c \uc758 \ud55c\uad6d\uc601\ud654\uc758 \uc624\ub298 - \ud30c\ub178\ub77c\ub9c8 \uc139\uc158\ubd80\ubb38\uc5d0 \ucd08\uccad\ub418\uba74\uc11c, \ucd9c\uc5f0 \ubc30\uc6b0 \uc790\uaca9\uc73c\ub85c \ub808\ub4dc\uce74\ud3ab\uc744 \ubc1f\uc558\ub2e4.", "paragraph_sentence": "\ub958\uc900\uc5f4\uc740 2015\ub144 6\uc6d4, KBS 2TV \ub4dc\ub77c\ub9c8 \u300a\ud504\ub85c\ub4c0\uc0ac\u300b\uc5d0\uc11c \ubc31\uc2b9\ucc2c(\uae40\uc218\ud604)\uc758 \ubc29\uc1a1\uad6d \uc785\uc0ac \ub3d9\uae30 \uc8fc\uc885\ud604 \uc5ed\ud560\uc744 \uc5f0\uae30\ud588\ub2e4. 2015\ub144 7\uc6d4, \ub958\uc900\uc5f4\uc740 \uc528\uc81c\uc2a4 \uc5d4\ud130\ud14c\uc778\uba3c\ud2b8\uc640 \ub9e4\ub2c8\uc9c0\uba3c\ud2b8 \uc804\uc18d \uacc4\uc57d\uc744 \ub9fa\uc5c8\ub2e4. 2015\ub144 10\uc6d4 \ub958\uc900\uc5f4\uc740 \ucd5c\uc815\uc5f4 \uac10\ub3c5\uc758 \uc601\ud654 \u300a\uae00\ub85c\ub9ac\ub370\uc774\u300b\uc5d0\uc11c \uc785\ub300\ud558\ub294 \uce5c\uad6c\ub97c \ubc30\uc6c5\ud558\ub7ec \uc5ec\ud589\uc744 \ub5a0\ub0ac\ub2e4\uac00 \ub73b\ubc16\uc758 \uc0ac\uac74\uc744 \ub9c8\uc8fc\ud558\uac8c \ub41c \ub124 \uba85\uc758 \uc2a4\ubb34 \uc0b4 \uce5c\uad6c\ub4e4 \uc911\uc5d0\uc11c \uc704\uae30\uc758 \uc21c\uac04 \uacb0\uc815\uc801 \uc120\ud0dd\uc744 \uc81c\uc548\ud558\ub294 \uc9c0\uacf5 \uc5ed\ud560\uc744 \uc5f0\uae30\ud55c \uc774 \uc601\ud654\ub85c \uc81c20\ud68c \ubd80\uc0b0\uad6d\uc81c\uc601\ud654\uc81c \uc758 \ud55c\uad6d\uc601\ud654\uc758 \uc624\ub298 - \ud30c\ub178\ub77c\ub9c8 \uc139\uc158\ubd80\ubb38\uc5d0 \ucd08\uccad\ub418\uba74\uc11c, \ucd9c\uc5f0 \ubc30\uc6b0 \uc790\uaca9\uc73c\ub85c \ub808\ub4dc\uce74\ud3ab\uc744 \ubc1f\uc558\ub2e4. 2015\ub144 12\uc6d4, \u300a\uc18c\uc15c\ud3ec\ube44\uc544\u300b\ub85c KT&G \uc0c1\uc0c1\ub9c8\ub2f9 \uc2dc\ub124\ub9c8\uac00 2015\ub144 \uc2a4\ud06c\ub9b0\uc5d0\uc11c \uac00\uc7a5 \ubc18\uc9dd\uc778 \uc2a4\ud0c0\uc640 2016\ub144 \uae30\ub300\uc8fc\ub97c \uc18c\uac1c\ud558\ub294 \uc5f0\ub9d0\uae30\ud68d\uc804 \u30082015 \uc528\ub124 \uc544\uc774\ucf58: KT&G \uc0c1\uc0c1\ub9c8\ub2f9 \ubc30\uc6b0\uae30\ud68d\uc804\u3009\uc5d0\uc11c \uc2a4\ud06c\ub9b0\uc758 \uc0c8\ub85c\uc6b4 \uc2a4\ud0c0\uc778 \u3008\uc528\ub124 \uc544\uc774\ucf58\u3009\uc5d0 \uc120\uc815\ub418\uae30\ub3c4 \ud558\uc600\ub2e4. 2015\ub144 5\uc6d4 18\uc77c, \ub958\uc900\uc5f4\uc774 TvN \uae08\ud1a0\ub4dc\ub77c\ub9c8 \u300a\uc751\ub2f5\ud558\ub77c 1988\u300b\uc758 \uae40\uc815\ud658 \uc5ed\ud560\uc5d0 \uce90\uc2a4\ud305\ub41c \uac83\uc774 \ubc1c\ud45c\ub418\uc5c8\ub2e4. \uc774 \ub4dc\ub77c\ub9c8\ub294 2015\ub144 11\uc6d4 \ucd08\uc5d0 \uccab \ubc29\uc601 \ub418\uc5c8\uace0, \ub958\uc900\uc5f4\uc740 \uadf9 \uc911 \uac89\ubaa8\uc2b5\uc740 \ubb34\uc2ec\ud55c \ub4ef\ud558\uc9c0\ub9cc \ub0b4\uba74\uc740 \ub530\ub73b\ud558\uace0 \uc18d \uae4a\uc740 \uce90\ub9ad\ud130\ub97c \uc5f0\uae30\ud588\uace0, \uc2e0\uc778\ub2f5\uc9c0 \uc54a\uc740 \uc548\uc815\ub41c \uc5f0\uae30\ub825\uacfc \uc5ec\uc2ec\uc744 \uc0ac\ub85c\uc7a1\ub294 \ub9e4\ub825\uc73c\ub85c \uc8fc\ubaa9\uc744 \ubc1b\uc73c\uba70 \ub9ce\uc740 \uc0ac\ub791\uc744 \ubc1b\uc558\ub2e4. \uc774 \uc791\ud488\uc73c\ub85c \uc81c52\ud68c \ubc31\uc0c1\uc608\uc220\ub300\uc0c1 TV\ubd80\ubb38 \uc2e0\uc778\uc0c1\uc744 \uc218\uc0c1\ud588\ub2e4.", "paragraph_answer": "\ub958\uc900\uc5f4\uc740 2015\ub144 6\uc6d4, KBS 2TV \ub4dc\ub77c\ub9c8 \u300a\ud504\ub85c\ub4c0\uc0ac\u300b\uc5d0\uc11c \ubc31\uc2b9\ucc2c(\uae40\uc218\ud604)\uc758 \ubc29\uc1a1\uad6d \uc785\uc0ac \ub3d9\uae30 \uc8fc\uc885\ud604 \uc5ed\ud560\uc744 \uc5f0\uae30\ud588\ub2e4. 2015\ub144 7\uc6d4, \ub958\uc900\uc5f4\uc740 \uc528\uc81c\uc2a4 \uc5d4\ud130\ud14c\uc778\uba3c\ud2b8\uc640 \ub9e4\ub2c8\uc9c0\uba3c\ud2b8 \uc804\uc18d \uacc4\uc57d\uc744 \ub9fa\uc5c8\ub2e4. 2015\ub144 10\uc6d4 \ub958\uc900\uc5f4\uc740 \ucd5c\uc815\uc5f4 \uac10\ub3c5\uc758 \uc601\ud654 \u300a\uae00\ub85c\ub9ac\ub370\uc774\u300b\uc5d0\uc11c \uc785\ub300\ud558\ub294 \uce5c\uad6c\ub97c \ubc30\uc6c5\ud558\ub7ec \uc5ec\ud589\uc744 \ub5a0\ub0ac\ub2e4\uac00 \ub73b\ubc16\uc758 \uc0ac\uac74\uc744 \ub9c8\uc8fc\ud558\uac8c \ub41c \ub124 \uba85\uc758 \uc2a4\ubb34 \uc0b4 \uce5c\uad6c\ub4e4 \uc911\uc5d0\uc11c \uc704\uae30\uc758 \uc21c\uac04 \uacb0\uc815\uc801 \uc120\ud0dd\uc744 \uc81c\uc548\ud558\ub294 \uc9c0\uacf5 \uc5ed\ud560\uc744 \uc5f0\uae30\ud55c \uc774 \uc601\ud654\ub85c \uc81c20\ud68c \ubd80\uc0b0\uad6d\uc81c\uc601\ud654\uc81c \uc758 \ud55c\uad6d\uc601\ud654\uc758 \uc624\ub298 - \ud30c\ub178\ub77c\ub9c8 \uc139\uc158\ubd80\ubb38\uc5d0 \ucd08\uccad\ub418\uba74\uc11c, \ucd9c\uc5f0 \ubc30\uc6b0 \uc790\uaca9\uc73c\ub85c \ub808\ub4dc\uce74\ud3ab\uc744 \ubc1f\uc558\ub2e4. 2015\ub144 12\uc6d4, \u300a\uc18c\uc15c\ud3ec\ube44\uc544\u300b\ub85c KT&G \uc0c1\uc0c1\ub9c8\ub2f9 \uc2dc\ub124\ub9c8\uac00 2015\ub144 \uc2a4\ud06c\ub9b0\uc5d0\uc11c \uac00\uc7a5 \ubc18\uc9dd\uc778 \uc2a4\ud0c0\uc640 2016\ub144 \uae30\ub300\uc8fc\ub97c \uc18c\uac1c\ud558\ub294 \uc5f0\ub9d0\uae30\ud68d\uc804 \u30082015 \uc528\ub124 \uc544\uc774\ucf58: KT&G \uc0c1\uc0c1\ub9c8\ub2f9 \ubc30\uc6b0\uae30\ud68d\uc804\u3009\uc5d0\uc11c \uc2a4\ud06c\ub9b0\uc758 \uc0c8\ub85c\uc6b4 \uc2a4\ud0c0\uc778 \u3008\uc528\ub124 \uc544\uc774\ucf58\u3009\uc5d0 \uc120\uc815\ub418\uae30\ub3c4 \ud558\uc600\ub2e4. 2015\ub144 5\uc6d4 18\uc77c, \ub958\uc900\uc5f4\uc774 TvN \uae08\ud1a0\ub4dc\ub77c\ub9c8 \u300a\uc751\ub2f5\ud558\ub77c 1988\u300b\uc758 \uae40\uc815\ud658 \uc5ed\ud560\uc5d0 \uce90\uc2a4\ud305\ub41c \uac83\uc774 \ubc1c\ud45c\ub418\uc5c8\ub2e4. \uc774 \ub4dc\ub77c\ub9c8\ub294 2015\ub144 11\uc6d4 \ucd08\uc5d0 \uccab \ubc29\uc601 \ub418\uc5c8\uace0, \ub958\uc900\uc5f4\uc740 \uadf9 \uc911 \uac89\ubaa8\uc2b5\uc740 \ubb34\uc2ec\ud55c \ub4ef\ud558\uc9c0\ub9cc \ub0b4\uba74\uc740 \ub530\ub73b\ud558\uace0 \uc18d \uae4a\uc740 \uce90\ub9ad\ud130\ub97c \uc5f0\uae30\ud588\uace0, \uc2e0\uc778\ub2f5\uc9c0 \uc54a\uc740 \uc548\uc815\ub41c \uc5f0\uae30\ub825\uacfc \uc5ec\uc2ec\uc744 \uc0ac\ub85c\uc7a1\ub294 \ub9e4\ub825\uc73c\ub85c \uc8fc\ubaa9\uc744 \ubc1b\uc73c\uba70 \ub9ce\uc740 \uc0ac\ub791\uc744 \ubc1b\uc558\ub2e4. \uc774 \uc791\ud488\uc73c\ub85c \uc81c52\ud68c \ubc31\uc0c1\uc608\uc220\ub300\uc0c1 TV\ubd80\ubb38 \uc2e0\uc778\uc0c1\uc744 \uc218\uc0c1\ud588\ub2e4.", "sentence_answer": "2015\ub144 10\uc6d4 \ub958\uc900\uc5f4\uc740 \ucd5c\uc815\uc5f4 \uac10\ub3c5\uc758 \uc601\ud654 \u300a\uae00\ub85c\ub9ac\ub370\uc774\u300b\uc5d0\uc11c \uc785\ub300\ud558\ub294 \uce5c\uad6c\ub97c \ubc30\uc6c5\ud558\ub7ec \uc5ec\ud589\uc744 \ub5a0\ub0ac\ub2e4\uac00 \ub73b\ubc16\uc758 \uc0ac\uac74\uc744 \ub9c8\uc8fc\ud558\uac8c \ub41c \ub124 \uba85\uc758 \uc2a4\ubb34 \uc0b4 \uce5c\uad6c\ub4e4 \uc911\uc5d0\uc11c \uc704\uae30\uc758 \uc21c\uac04 \uacb0\uc815\uc801 \uc120\ud0dd\uc744 \uc81c\uc548\ud558\ub294 \uc9c0\uacf5 \uc5ed\ud560\uc744 \uc5f0\uae30\ud55c \uc774 \uc601\ud654\ub85c \uc81c20\ud68c \ubd80\uc0b0\uad6d\uc81c\uc601\ud654\uc81c \uc758 \ud55c\uad6d\uc601\ud654\uc758 \uc624\ub298 - \ud30c\ub178\ub77c\ub9c8 \uc139\uc158\ubd80\ubb38\uc5d0 \ucd08\uccad\ub418\uba74\uc11c, \ucd9c\uc5f0 \ubc30\uc6b0 \uc790\uaca9\uc73c\ub85c \ub808\ub4dc\uce74\ud3ab\uc744 \ubc1f\uc558\ub2e4.", "paragraph_id": "6498117-2-1", "paragraph_question": "question: \ub958\uc900\uc5f4\uc774 \uc601\ud654 \uae00\ub85c\ub9ac\ub370\uc774\ub85c \uc778\ud574 \ucd08\uccad\ub41c \uc601\ud654\uc81c \uc774\ub984\uc740?, context: \ub958\uc900\uc5f4\uc740 2015\ub144 6\uc6d4, KBS 2TV \ub4dc\ub77c\ub9c8 \u300a\ud504\ub85c\ub4c0\uc0ac\u300b\uc5d0\uc11c \ubc31\uc2b9\ucc2c(\uae40\uc218\ud604)\uc758 \ubc29\uc1a1\uad6d \uc785\uc0ac \ub3d9\uae30 \uc8fc\uc885\ud604 \uc5ed\ud560\uc744 \uc5f0\uae30\ud588\ub2e4. 2015\ub144 7\uc6d4, \ub958\uc900\uc5f4\uc740 \uc528\uc81c\uc2a4 \uc5d4\ud130\ud14c\uc778\uba3c\ud2b8\uc640 \ub9e4\ub2c8\uc9c0\uba3c\ud2b8 \uc804\uc18d \uacc4\uc57d\uc744 \ub9fa\uc5c8\ub2e4. 2015\ub144 10\uc6d4 \ub958\uc900\uc5f4\uc740 \ucd5c\uc815\uc5f4 \uac10\ub3c5\uc758 \uc601\ud654 \u300a\uae00\ub85c\ub9ac\ub370\uc774\u300b\uc5d0\uc11c \uc785\ub300\ud558\ub294 \uce5c\uad6c\ub97c \ubc30\uc6c5\ud558\ub7ec \uc5ec\ud589\uc744 \ub5a0\ub0ac\ub2e4\uac00 \ub73b\ubc16\uc758 \uc0ac\uac74\uc744 \ub9c8\uc8fc\ud558\uac8c \ub41c \ub124 \uba85\uc758 \uc2a4\ubb34 \uc0b4 \uce5c\uad6c\ub4e4 \uc911\uc5d0\uc11c \uc704\uae30\uc758 \uc21c\uac04 \uacb0\uc815\uc801 \uc120\ud0dd\uc744 \uc81c\uc548\ud558\ub294 \uc9c0\uacf5 \uc5ed\ud560\uc744 \uc5f0\uae30\ud55c \uc774 \uc601\ud654\ub85c \uc81c20\ud68c \ubd80\uc0b0\uad6d\uc81c\uc601\ud654\uc81c\uc758 \ud55c\uad6d\uc601\ud654\uc758 \uc624\ub298 - \ud30c\ub178\ub77c\ub9c8 \uc139\uc158\ubd80\ubb38\uc5d0 \ucd08\uccad\ub418\uba74\uc11c, \ucd9c\uc5f0 \ubc30\uc6b0 \uc790\uaca9\uc73c\ub85c \ub808\ub4dc\uce74\ud3ab\uc744 \ubc1f\uc558\ub2e4. 2015\ub144 12\uc6d4, \u300a\uc18c\uc15c\ud3ec\ube44\uc544\u300b\ub85c KT&G \uc0c1\uc0c1\ub9c8\ub2f9 \uc2dc\ub124\ub9c8\uac00 2015\ub144 \uc2a4\ud06c\ub9b0\uc5d0\uc11c \uac00\uc7a5 \ubc18\uc9dd\uc778 \uc2a4\ud0c0\uc640 2016\ub144 \uae30\ub300\uc8fc\ub97c \uc18c\uac1c\ud558\ub294 \uc5f0\ub9d0\uae30\ud68d\uc804 \u30082015 \uc528\ub124 \uc544\uc774\ucf58: KT&G \uc0c1\uc0c1\ub9c8\ub2f9 \ubc30\uc6b0\uae30\ud68d\uc804\u3009\uc5d0\uc11c \uc2a4\ud06c\ub9b0\uc758 \uc0c8\ub85c\uc6b4 \uc2a4\ud0c0\uc778 \u3008\uc528\ub124 \uc544\uc774\ucf58\u3009\uc5d0 \uc120\uc815\ub418\uae30\ub3c4 \ud558\uc600\ub2e4. 2015\ub144 5\uc6d4 18\uc77c, \ub958\uc900\uc5f4\uc774 TvN \uae08\ud1a0\ub4dc\ub77c\ub9c8 \u300a\uc751\ub2f5\ud558\ub77c 1988\u300b\uc758 \uae40\uc815\ud658 \uc5ed\ud560\uc5d0 \uce90\uc2a4\ud305\ub41c \uac83\uc774 \ubc1c\ud45c\ub418\uc5c8\ub2e4. \uc774 \ub4dc\ub77c\ub9c8\ub294 2015\ub144 11\uc6d4 \ucd08\uc5d0 \uccab \ubc29\uc601 \ub418\uc5c8\uace0, \ub958\uc900\uc5f4\uc740 \uadf9 \uc911 \uac89\ubaa8\uc2b5\uc740 \ubb34\uc2ec\ud55c \ub4ef\ud558\uc9c0\ub9cc \ub0b4\uba74\uc740 \ub530\ub73b\ud558\uace0 \uc18d \uae4a\uc740 \uce90\ub9ad\ud130\ub97c \uc5f0\uae30\ud588\uace0, \uc2e0\uc778\ub2f5\uc9c0 \uc54a\uc740 \uc548\uc815\ub41c \uc5f0\uae30\ub825\uacfc \uc5ec\uc2ec\uc744 \uc0ac\ub85c\uc7a1\ub294 \ub9e4\ub825\uc73c\ub85c \uc8fc\ubaa9\uc744 \ubc1b\uc73c\uba70 \ub9ce\uc740 \uc0ac\ub791\uc744 \ubc1b\uc558\ub2e4. \uc774 \uc791\ud488\uc73c\ub85c \uc81c52\ud68c \ubc31\uc0c1\uc608\uc220\ub300\uc0c1 TV\ubd80\ubb38 \uc2e0\uc778\uc0c1\uc744 \uc218\uc0c1\ud588\ub2e4."} {"question": "\ub3c5\uc77c\uc774 \uc5b8\uc81c\ubd80\ud130 \uc801\uadf9\uc801\uc73c\ub85c \uc774\ubbfc\uc744 \ubc1b\uc544\ub4e4\uc600\ub098?", "paragraph": "\uc720\ub7fd\uc758 \ub300\ubd80\ubd84 \uad6d\uac00\ub4e4\uc774 \uadf8\ub7ec\ud558\ub4ef \ub3c5\uc77c\ub3c4 \uc81c 2\ucc28 \uc138\uacc4\ub300\uc804 \uc774\ud6c4 \uc801\uadf9\uc801\uc73c\ub85c \uc774\ubbfc\uc744 \ubc1b\uc544\ub4e4\uc774\uba74\uc11c \ud55c\uad6d\uc5d0 \ube44\ud574 \uc815\ucc29\ud654 \ub41c \ub2e4\ubb38\ud654 \uc0ac\ud68c\ub97c \uc774\ub8e8\uace0 \uc788\ub2e4. \ub3c5\uc77c\uc815\ubd80\uc640 \uc5f0\ubc29\uc8fc\ub294 '\ubb38\ud654' \ub77c\ub294 \uc544\uc774\ucf58\uc744 \ub0b4\uc138\uc6cc \ubb38\ud654\ud3b8\uacac\uc744 \uc904\uc774\uae30 \uc704\ud574 \ub2e4\ubbfc\uc871\uc758 \ubb38\ud654\ub97c \ud3ec\uc6a9\ud558\ub294 \uc9c0\uc18d\uc801\uc778 \ucd1d\uccb4\uc801 \uc0ac\ud68c\uc735\ud569 \uc815\ucc45\uc744 \ud3bc\uce58\uace0 \uc788\ub2e4. \ub9e4\ub144 \uc678\uad6d\uc778\uc744 \ubc14\ub77c\ubcf4\ub294 \ub3c5\uc77c\uc778\uacfc \uc774\ubbfc\uc790 \uc790\uc2e0\ub4e4\uc758 \uc2dc\uc791\uc801\uc778 \uc778\uc2dd\ub3c4 \ub9ce\uc774 \ub2ec\ub77c\uc9c0\uace0 \uc788\uace0 \ub2e4\ub978 \uad6d\uac00\ub4e4\ub85c\ubd80\ud130 \ub098\ub984 \uc131\uacf5\uc801\uc778 \uc774\ubbfc \uc815\ucc45\uc784\uc744 \uc778\uc815\ubc1b\uace0 \uc788\ub2e4. \uadf8\ub7fc\uc5d0\ub3c4 \ubd88\uad6c\ud558\uace0 \uc77c\ubd80\uc5d0\uc11c\ub294 \uc5ec\uc804\ud788 \ub3c5\uc77c\ubb38\ud654\uc5d0 \ub300\ud55c \uae30\ubcf8\uc801 \uc774\ud574\ub97c \uac00\uc9c4 \uc774\ubbfc\uc790\ub4e4\uc744 \uc218\uc6a9\ud558\uace0\uc790 \ud558\ub294 \uc9d1\ub2e8\uacfc \uc774\ub97c \uad6d\uac00 \uc774\uae30\uc8fc\uc758\ub85c \ub2e8\uc815\ud558\uace0 \uac70\ubd80\ud558\ub294 \uc9d1\ub2e8 \uc0ac\uc774\uc5d0 \ucca8\uc608\ud55c \ub300\ub9bd \uac01\uc744 \uc138\uc6b0\uace0 \uc788\ub294 \uac83\ub3c4 \uc548\ud0c0\uae4c\uc6b4 \uc0ac\uc2e4\uc774\ub2e4. \ub3c5\uc77c\uc740 \ubb38\ud654\uc801 \ucc28\uc774\ub97c '\ub2e4\ub984'\uc774 \uc544\ub2cc '\ud2b9\uc131' \ub610\ub294 '\uac1c\uc131'\uc73c\ub85c \uc2b9\ud654\uc2dc\ud0a4\uae30 \uc704\ud55c \uc778\uc2dd\ubcc0\ud654\ub97c \uc704\ud574 \uc5b4\ub9b4 \ub54c\ubd80\ud130 \ub2e4\uc591\ud55c \ubb38\ud654\uc608\uc220\uad50\uc721\uc744 \uccb4\ud5d8\ud560 \uc218 \uc788\ub294 \uae30\ud68c\ub97c \uc81c\uacf5\ud55c\ub2e4. \ub3c5\uc77c \uc774\ubbfc\uc790\ub4e4\uacfc \uc9c0\uc5ed \uc0ac\ud68c \uac04\uc758 \ubcf4\uc774\uc9c0 \uc54a\ub294 \uac04\uaca9\uae4c\uc9c0 \uc904\uc5ec\uc57c \ud55c\ub2e4\ub294 \uc774\ubbfc\uc790 \uc815\ucc45\uc758 \uc911\uc694\uc131\uc774 \ubd80\uac01\ub418\ub294 \uc774\uc720\uac00 \ubc14\ub85c \uc5ec\uae30\uc5d0 \uc788\ub2e4", "answer": "\uc81c 2\ucc28 \uc138\uacc4\ub300\uc804 \uc774\ud6c4", "sentence": "\uc720\ub7fd\uc758 \ub300\ubd80\ubd84 \uad6d\uac00\ub4e4\uc774 \uadf8\ub7ec\ud558\ub4ef \ub3c5\uc77c\ub3c4 \uc81c 2\ucc28 \uc138\uacc4\ub300\uc804 \uc774\ud6c4 \uc801\uadf9\uc801\uc73c\ub85c \uc774\ubbfc\uc744 \ubc1b\uc544\ub4e4\uc774\uba74\uc11c \ud55c\uad6d\uc5d0 \ube44\ud574 \uc815\ucc29\ud654 \ub41c \ub2e4\ubb38\ud654 \uc0ac\ud68c\ub97c \uc774\ub8e8\uace0 \uc788\ub2e4.", "paragraph_sentence": " \uc720\ub7fd\uc758 \ub300\ubd80\ubd84 \uad6d\uac00\ub4e4\uc774 \uadf8\ub7ec\ud558\ub4ef \ub3c5\uc77c\ub3c4 \uc81c 2\ucc28 \uc138\uacc4\ub300\uc804 \uc774\ud6c4 \uc801\uadf9\uc801\uc73c\ub85c \uc774\ubbfc\uc744 \ubc1b\uc544\ub4e4\uc774\uba74\uc11c \ud55c\uad6d\uc5d0 \ube44\ud574 \uc815\ucc29\ud654 \ub41c \ub2e4\ubb38\ud654 \uc0ac\ud68c\ub97c \uc774\ub8e8\uace0 \uc788\ub2e4. \ub3c5\uc77c\uc815\ubd80\uc640 \uc5f0\ubc29\uc8fc\ub294 '\ubb38\ud654' \ub77c\ub294 \uc544\uc774\ucf58\uc744 \ub0b4\uc138\uc6cc \ubb38\ud654\ud3b8\uacac\uc744 \uc904\uc774\uae30 \uc704\ud574 \ub2e4\ubbfc\uc871\uc758 \ubb38\ud654\ub97c \ud3ec\uc6a9\ud558\ub294 \uc9c0\uc18d\uc801\uc778 \ucd1d\uccb4\uc801 \uc0ac\ud68c\uc735\ud569 \uc815\ucc45\uc744 \ud3bc\uce58\uace0 \uc788\ub2e4. \ub9e4\ub144 \uc678\uad6d\uc778\uc744 \ubc14\ub77c\ubcf4\ub294 \ub3c5\uc77c\uc778\uacfc \uc774\ubbfc\uc790 \uc790\uc2e0\ub4e4\uc758 \uc2dc\uc791\uc801\uc778 \uc778\uc2dd\ub3c4 \ub9ce\uc774 \ub2ec\ub77c\uc9c0\uace0 \uc788\uace0 \ub2e4\ub978 \uad6d\uac00\ub4e4\ub85c\ubd80\ud130 \ub098\ub984 \uc131\uacf5\uc801\uc778 \uc774\ubbfc \uc815\ucc45\uc784\uc744 \uc778\uc815\ubc1b\uace0 \uc788\ub2e4. \uadf8\ub7fc\uc5d0\ub3c4 \ubd88\uad6c\ud558\uace0 \uc77c\ubd80\uc5d0\uc11c\ub294 \uc5ec\uc804\ud788 \ub3c5\uc77c\ubb38\ud654\uc5d0 \ub300\ud55c \uae30\ubcf8\uc801 \uc774\ud574\ub97c \uac00\uc9c4 \uc774\ubbfc\uc790\ub4e4\uc744 \uc218\uc6a9\ud558\uace0\uc790 \ud558\ub294 \uc9d1\ub2e8\uacfc \uc774\ub97c \uad6d\uac00 \uc774\uae30\uc8fc\uc758\ub85c \ub2e8\uc815\ud558\uace0 \uac70\ubd80\ud558\ub294 \uc9d1\ub2e8 \uc0ac\uc774\uc5d0 \ucca8\uc608\ud55c \ub300\ub9bd \uac01\uc744 \uc138\uc6b0\uace0 \uc788\ub294 \uac83\ub3c4 \uc548\ud0c0\uae4c\uc6b4 \uc0ac\uc2e4\uc774\ub2e4. \ub3c5\uc77c\uc740 \ubb38\ud654\uc801 \ucc28\uc774\ub97c '\ub2e4\ub984'\uc774 \uc544\ub2cc '\ud2b9\uc131' \ub610\ub294 '\uac1c\uc131'\uc73c\ub85c \uc2b9\ud654\uc2dc\ud0a4\uae30 \uc704\ud55c \uc778\uc2dd\ubcc0\ud654\ub97c \uc704\ud574 \uc5b4\ub9b4 \ub54c\ubd80\ud130 \ub2e4\uc591\ud55c \ubb38\ud654\uc608\uc220\uad50\uc721\uc744 \uccb4\ud5d8\ud560 \uc218 \uc788\ub294 \uae30\ud68c\ub97c \uc81c\uacf5\ud55c\ub2e4. \ub3c5\uc77c \uc774\ubbfc\uc790\ub4e4\uacfc \uc9c0\uc5ed \uc0ac\ud68c \uac04\uc758 \ubcf4\uc774\uc9c0 \uc54a\ub294 \uac04\uaca9\uae4c\uc9c0 \uc904\uc5ec\uc57c \ud55c\ub2e4\ub294 \uc774\ubbfc\uc790 \uc815\ucc45\uc758 \uc911\uc694\uc131\uc774 \ubd80\uac01\ub418\ub294 \uc774\uc720\uac00 \ubc14\ub85c \uc5ec\uae30\uc5d0 \uc788\ub2e4", "paragraph_answer": "\uc720\ub7fd\uc758 \ub300\ubd80\ubd84 \uad6d\uac00\ub4e4\uc774 \uadf8\ub7ec\ud558\ub4ef \ub3c5\uc77c\ub3c4 \uc81c 2\ucc28 \uc138\uacc4\ub300\uc804 \uc774\ud6c4 \uc801\uadf9\uc801\uc73c\ub85c \uc774\ubbfc\uc744 \ubc1b\uc544\ub4e4\uc774\uba74\uc11c \ud55c\uad6d\uc5d0 \ube44\ud574 \uc815\ucc29\ud654 \ub41c \ub2e4\ubb38\ud654 \uc0ac\ud68c\ub97c \uc774\ub8e8\uace0 \uc788\ub2e4. \ub3c5\uc77c\uc815\ubd80\uc640 \uc5f0\ubc29\uc8fc\ub294 '\ubb38\ud654' \ub77c\ub294 \uc544\uc774\ucf58\uc744 \ub0b4\uc138\uc6cc \ubb38\ud654\ud3b8\uacac\uc744 \uc904\uc774\uae30 \uc704\ud574 \ub2e4\ubbfc\uc871\uc758 \ubb38\ud654\ub97c \ud3ec\uc6a9\ud558\ub294 \uc9c0\uc18d\uc801\uc778 \ucd1d\uccb4\uc801 \uc0ac\ud68c\uc735\ud569 \uc815\ucc45\uc744 \ud3bc\uce58\uace0 \uc788\ub2e4. \ub9e4\ub144 \uc678\uad6d\uc778\uc744 \ubc14\ub77c\ubcf4\ub294 \ub3c5\uc77c\uc778\uacfc \uc774\ubbfc\uc790 \uc790\uc2e0\ub4e4\uc758 \uc2dc\uc791\uc801\uc778 \uc778\uc2dd\ub3c4 \ub9ce\uc774 \ub2ec\ub77c\uc9c0\uace0 \uc788\uace0 \ub2e4\ub978 \uad6d\uac00\ub4e4\ub85c\ubd80\ud130 \ub098\ub984 \uc131\uacf5\uc801\uc778 \uc774\ubbfc \uc815\ucc45\uc784\uc744 \uc778\uc815\ubc1b\uace0 \uc788\ub2e4. \uadf8\ub7fc\uc5d0\ub3c4 \ubd88\uad6c\ud558\uace0 \uc77c\ubd80\uc5d0\uc11c\ub294 \uc5ec\uc804\ud788 \ub3c5\uc77c\ubb38\ud654\uc5d0 \ub300\ud55c \uae30\ubcf8\uc801 \uc774\ud574\ub97c \uac00\uc9c4 \uc774\ubbfc\uc790\ub4e4\uc744 \uc218\uc6a9\ud558\uace0\uc790 \ud558\ub294 \uc9d1\ub2e8\uacfc \uc774\ub97c \uad6d\uac00 \uc774\uae30\uc8fc\uc758\ub85c \ub2e8\uc815\ud558\uace0 \uac70\ubd80\ud558\ub294 \uc9d1\ub2e8 \uc0ac\uc774\uc5d0 \ucca8\uc608\ud55c \ub300\ub9bd \uac01\uc744 \uc138\uc6b0\uace0 \uc788\ub294 \uac83\ub3c4 \uc548\ud0c0\uae4c\uc6b4 \uc0ac\uc2e4\uc774\ub2e4. \ub3c5\uc77c\uc740 \ubb38\ud654\uc801 \ucc28\uc774\ub97c '\ub2e4\ub984'\uc774 \uc544\ub2cc '\ud2b9\uc131' \ub610\ub294 '\uac1c\uc131'\uc73c\ub85c \uc2b9\ud654\uc2dc\ud0a4\uae30 \uc704\ud55c \uc778\uc2dd\ubcc0\ud654\ub97c \uc704\ud574 \uc5b4\ub9b4 \ub54c\ubd80\ud130 \ub2e4\uc591\ud55c \ubb38\ud654\uc608\uc220\uad50\uc721\uc744 \uccb4\ud5d8\ud560 \uc218 \uc788\ub294 \uae30\ud68c\ub97c \uc81c\uacf5\ud55c\ub2e4. \ub3c5\uc77c \uc774\ubbfc\uc790\ub4e4\uacfc \uc9c0\uc5ed \uc0ac\ud68c \uac04\uc758 \ubcf4\uc774\uc9c0 \uc54a\ub294 \uac04\uaca9\uae4c\uc9c0 \uc904\uc5ec\uc57c \ud55c\ub2e4\ub294 \uc774\ubbfc\uc790 \uc815\ucc45\uc758 \uc911\uc694\uc131\uc774 \ubd80\uac01\ub418\ub294 \uc774\uc720\uac00 \ubc14\ub85c \uc5ec\uae30\uc5d0 \uc788\ub2e4", "sentence_answer": "\uc720\ub7fd\uc758 \ub300\ubd80\ubd84 \uad6d\uac00\ub4e4\uc774 \uadf8\ub7ec\ud558\ub4ef \ub3c5\uc77c\ub3c4 \uc81c 2\ucc28 \uc138\uacc4\ub300\uc804 \uc774\ud6c4 \uc801\uadf9\uc801\uc73c\ub85c \uc774\ubbfc\uc744 \ubc1b\uc544\ub4e4\uc774\uba74\uc11c \ud55c\uad6d\uc5d0 \ube44\ud574 \uc815\ucc29\ud654 \ub41c \ub2e4\ubb38\ud654 \uc0ac\ud68c\ub97c \uc774\ub8e8\uace0 \uc788\ub2e4.", "paragraph_id": "6046529-9-0", "paragraph_question": "question: \ub3c5\uc77c\uc774 \uc5b8\uc81c\ubd80\ud130 \uc801\uadf9\uc801\uc73c\ub85c \uc774\ubbfc\uc744 \ubc1b\uc544\ub4e4\uc600\ub098?, context: \uc720\ub7fd\uc758 \ub300\ubd80\ubd84 \uad6d\uac00\ub4e4\uc774 \uadf8\ub7ec\ud558\ub4ef \ub3c5\uc77c\ub3c4 \uc81c 2\ucc28 \uc138\uacc4\ub300\uc804 \uc774\ud6c4 \uc801\uadf9\uc801\uc73c\ub85c \uc774\ubbfc\uc744 \ubc1b\uc544\ub4e4\uc774\uba74\uc11c \ud55c\uad6d\uc5d0 \ube44\ud574 \uc815\ucc29\ud654 \ub41c \ub2e4\ubb38\ud654 \uc0ac\ud68c\ub97c \uc774\ub8e8\uace0 \uc788\ub2e4. \ub3c5\uc77c\uc815\ubd80\uc640 \uc5f0\ubc29\uc8fc\ub294 '\ubb38\ud654' \ub77c\ub294 \uc544\uc774\ucf58\uc744 \ub0b4\uc138\uc6cc \ubb38\ud654\ud3b8\uacac\uc744 \uc904\uc774\uae30 \uc704\ud574 \ub2e4\ubbfc\uc871\uc758 \ubb38\ud654\ub97c \ud3ec\uc6a9\ud558\ub294 \uc9c0\uc18d\uc801\uc778 \ucd1d\uccb4\uc801 \uc0ac\ud68c\uc735\ud569 \uc815\ucc45\uc744 \ud3bc\uce58\uace0 \uc788\ub2e4. \ub9e4\ub144 \uc678\uad6d\uc778\uc744 \ubc14\ub77c\ubcf4\ub294 \ub3c5\uc77c\uc778\uacfc \uc774\ubbfc\uc790 \uc790\uc2e0\ub4e4\uc758 \uc2dc\uc791\uc801\uc778 \uc778\uc2dd\ub3c4 \ub9ce\uc774 \ub2ec\ub77c\uc9c0\uace0 \uc788\uace0 \ub2e4\ub978 \uad6d\uac00\ub4e4\ub85c\ubd80\ud130 \ub098\ub984 \uc131\uacf5\uc801\uc778 \uc774\ubbfc \uc815\ucc45\uc784\uc744 \uc778\uc815\ubc1b\uace0 \uc788\ub2e4. \uadf8\ub7fc\uc5d0\ub3c4 \ubd88\uad6c\ud558\uace0 \uc77c\ubd80\uc5d0\uc11c\ub294 \uc5ec\uc804\ud788 \ub3c5\uc77c\ubb38\ud654\uc5d0 \ub300\ud55c \uae30\ubcf8\uc801 \uc774\ud574\ub97c \uac00\uc9c4 \uc774\ubbfc\uc790\ub4e4\uc744 \uc218\uc6a9\ud558\uace0\uc790 \ud558\ub294 \uc9d1\ub2e8\uacfc \uc774\ub97c \uad6d\uac00 \uc774\uae30\uc8fc\uc758\ub85c \ub2e8\uc815\ud558\uace0 \uac70\ubd80\ud558\ub294 \uc9d1\ub2e8 \uc0ac\uc774\uc5d0 \ucca8\uc608\ud55c \ub300\ub9bd \uac01\uc744 \uc138\uc6b0\uace0 \uc788\ub294 \uac83\ub3c4 \uc548\ud0c0\uae4c\uc6b4 \uc0ac\uc2e4\uc774\ub2e4. \ub3c5\uc77c\uc740 \ubb38\ud654\uc801 \ucc28\uc774\ub97c '\ub2e4\ub984'\uc774 \uc544\ub2cc '\ud2b9\uc131' \ub610\ub294 '\uac1c\uc131'\uc73c\ub85c \uc2b9\ud654\uc2dc\ud0a4\uae30 \uc704\ud55c \uc778\uc2dd\ubcc0\ud654\ub97c \uc704\ud574 \uc5b4\ub9b4 \ub54c\ubd80\ud130 \ub2e4\uc591\ud55c \ubb38\ud654\uc608\uc220\uad50\uc721\uc744 \uccb4\ud5d8\ud560 \uc218 \uc788\ub294 \uae30\ud68c\ub97c \uc81c\uacf5\ud55c\ub2e4. \ub3c5\uc77c \uc774\ubbfc\uc790\ub4e4\uacfc \uc9c0\uc5ed \uc0ac\ud68c \uac04\uc758 \ubcf4\uc774\uc9c0 \uc54a\ub294 \uac04\uaca9\uae4c\uc9c0 \uc904\uc5ec\uc57c \ud55c\ub2e4\ub294 \uc774\ubbfc\uc790 \uc815\ucc45\uc758 \uc911\uc694\uc131\uc774 \ubd80\uac01\ub418\ub294 \uc774\uc720\uac00 \ubc14\ub85c \uc5ec\uae30\uc5d0 \uc788\ub2e4"} {"question": "\ud53c\ub378 \uce74\uc2a4\ud2b8\ub85c\uc758 \ud601\uba85\uad70\uc740 \ucfe0\ubc14 \ub0b4\uc5d0\uc11c \uc5b4\ub514\uc5d0 \uac8c\ub9b4\ub77c \uae30\uc9c0\ub97c \uad6c\ucd95\ud558\uc600\ub294\uac00?", "paragraph": "\uadf8\ub294 \uc911\ub0a8\ubbf8, \ucfe0\ubc14\uc5d0\uc11c \ub9dd\uba85\ud55c \uc0ac\ud68c\uc8fc\uc758\uc790 \ubc0f \uc88c\ud30c\ubbfc\uc871\uc8fc\uc758\uc790\ub4e4\uc744 \ubaa8\uc544 \uc57d 800\uba85\uc758 \ud601\uba85\uad70\uc744 \ubaa8\uc73c\uac8c \ub41c\ub2e4. \uc774 \uacfc\uc815\uc5d0\uc11c \ucfe0\ubc14 \ubbfc\uc911\ub4e4\uc758 \uc9c0\uc9c0\ub97c \uc5bb\uc5b4 \uae08\uc804\uc801 \ub3c4\uc6c0\uc744 \ubc1b\uc544 \uad70\uc0ac\ub97c \uc720\uc9c0\ud560 \uc218 \uc788\uc5c8\ub2e4. \uadf8\ub9ac\uace0 1956\ub144 \uadf8\ub780\ub9c8 \ud568\uc120\uc744 \ud0c0\uace0, \ucfe0\ubc14\uc5d0 \uc0c1\ub959, \ubc14\ud2f0\uc2a4\ud0c0 \uc815\ubd80\uad70\uacfc \ub0b4\uc804\uc744 \uce58\ub8e8\uc5c8\ub2e4. \ubc14\ud2f0\uc2a4\ud0c0 \uc815\ubd80\uc758 \uacf5\uad70 \ub54c\ubb38\uc5d0, \uc0c1\ub959 \ub3c4\uc911 \uc218\ub9ce\uc740 \ubcd1\uc0ac\ub97c \uc783\uc5c8\uc9c0\ub9cc, \uc0c1\ub959\ud55c \ud6c4 \ubbfc\uc911\uc758 \uc9c0\uc9c0\ub97c \ubc1b\uc544, \ucfe0\ubc14 \ud601\uba85\uad70\uc758 \uc218\ub294 \uae09\uc99d\ud558\uc600\ub2e4. \uc774\uc5d0 \uc704\uae30\uac10\uc744 \ub290\ub080 \ubc14\ud2f0\uc2a4\ud0c0\ub294 \ucfe0\ubc14 \ud601\uba85\uad70\uc5d0 \ube44\ud574 \uc6d4\ub4f1\ud55c \uc7a5\ube44\ub97c \uac16\ucd98 \ub300\ub300\uae09(17\ubcf4\ubcd1\ub300\ub300) \uaddc\ubaa8\uc758 \ubcd1\uc0ac\ub4e4\uc744 \ud30c\uacac\ud558\uc9c0\ub9cc, \ucfe0\ubc14 \ud601\uba85\uad70\uc740 \uc218\uc801 \uc5f4\uc138\uc784\uc5d0\ub3c4 \ubd88\uad6c\ud558\uace0, \ubc14\ud2f0\uc2a4\ud0c0 \uc815\ubd80\uad70\uc744 \ubcc4 \ud0c8 \uc5c6\uc774 \ud574\uce58\uc6e0\ub2e4. \ud53c\ub378 \uce74\uc2a4\ud2b8\ub85c\uc758 \ud601\uba85\uad70\uc740 \ucfe0\ubc14 \ub0b4\uc5d0\uc11c \uac8c\ub9b4\ub77c \uae30\uc9c0\ub97c \uad6c\ucd95\ud558\uae30 \uc704\ud574 \ube44\uad50\uc801 \uc0b0\uc138\uac00 \ud5d8\ud55c \uc2dc\uc5d0\ub77c \ub9c8\uc5d0\uc2a4\ud2b8\ub77c\uc5d0 \uae30\uc9c0\ub97c \uad6c\ucd95\ud558\uc600\ub2e4. \uadf8\ub9ac\uace0 \ud53c\ub378 \uce74\uc2a4\ud2b8\ub85c\uc758 \ud601\uba85\uad70\uc740 \uc8fc\uae30\uc801\uc73c\ub85c \ub3c4\uc2dc\uc5d0 \ud601\uba85\uc5d0 \ucc38\uac00\ud558\uc790\ub294 \uc804\ub2e8\uc9c0\ub97c \ub3cc\ub9ac\uba74\uc11c \ud601\uba85\uad70 \ubcd1\uc0ac\ub4e4\uc758 \uc218\ub97c \uc99d\uac00\uc2dc\ucf30\ub2e4.", "answer": "\uc2dc\uc5d0\ub77c \ub9c8\uc5d0\uc2a4\ud2b8\ub77c", "sentence": "\ud53c\ub378 \uce74\uc2a4\ud2b8\ub85c\uc758 \ud601\uba85\uad70\uc740 \ucfe0\ubc14 \ub0b4\uc5d0\uc11c \uac8c\ub9b4\ub77c \uae30\uc9c0\ub97c \uad6c\ucd95\ud558\uae30 \uc704\ud574 \ube44\uad50\uc801 \uc0b0\uc138\uac00 \ud5d8\ud55c \uc2dc\uc5d0\ub77c \ub9c8\uc5d0\uc2a4\ud2b8\ub77c \uc5d0", "paragraph_sentence": "\uadf8\ub294 \uc911\ub0a8\ubbf8, \ucfe0\ubc14\uc5d0\uc11c \ub9dd\uba85\ud55c \uc0ac\ud68c\uc8fc\uc758\uc790 \ubc0f \uc88c\ud30c\ubbfc\uc871\uc8fc\uc758\uc790\ub4e4\uc744 \ubaa8\uc544 \uc57d 800\uba85\uc758 \ud601\uba85\uad70\uc744 \ubaa8\uc73c\uac8c \ub41c\ub2e4. \uc774 \uacfc\uc815\uc5d0\uc11c \ucfe0\ubc14 \ubbfc\uc911\ub4e4\uc758 \uc9c0\uc9c0\ub97c \uc5bb\uc5b4 \uae08\uc804\uc801 \ub3c4\uc6c0\uc744 \ubc1b\uc544 \uad70\uc0ac\ub97c \uc720\uc9c0\ud560 \uc218 \uc788\uc5c8\ub2e4. \uadf8\ub9ac\uace0 1956\ub144 \uadf8\ub780\ub9c8 \ud568\uc120\uc744 \ud0c0\uace0, \ucfe0\ubc14\uc5d0 \uc0c1\ub959, \ubc14\ud2f0\uc2a4\ud0c0 \uc815\ubd80\uad70\uacfc \ub0b4\uc804\uc744 \uce58\ub8e8\uc5c8\ub2e4. \ubc14\ud2f0\uc2a4\ud0c0 \uc815\ubd80\uc758 \uacf5\uad70 \ub54c\ubb38\uc5d0, \uc0c1\ub959 \ub3c4\uc911 \uc218\ub9ce\uc740 \ubcd1\uc0ac\ub97c \uc783\uc5c8\uc9c0\ub9cc, \uc0c1\ub959\ud55c \ud6c4 \ubbfc\uc911\uc758 \uc9c0\uc9c0\ub97c \ubc1b\uc544, \ucfe0\ubc14 \ud601\uba85\uad70\uc758 \uc218\ub294 \uae09\uc99d\ud558\uc600\ub2e4. \uc774\uc5d0 \uc704\uae30\uac10\uc744 \ub290\ub080 \ubc14\ud2f0\uc2a4\ud0c0\ub294 \ucfe0\ubc14 \ud601\uba85\uad70\uc5d0 \ube44\ud574 \uc6d4\ub4f1\ud55c \uc7a5\ube44\ub97c \uac16\ucd98 \ub300\ub300\uae09(17\ubcf4\ubcd1\ub300\ub300) \uaddc\ubaa8\uc758 \ubcd1\uc0ac\ub4e4\uc744 \ud30c\uacac\ud558\uc9c0\ub9cc, \ucfe0\ubc14 \ud601\uba85\uad70\uc740 \uc218\uc801 \uc5f4\uc138\uc784\uc5d0\ub3c4 \ubd88\uad6c\ud558\uace0, \ubc14\ud2f0\uc2a4\ud0c0 \uc815\ubd80\uad70\uc744 \ubcc4 \ud0c8 \uc5c6\uc774 \ud574\uce58\uc6e0\ub2e4. \ud53c\ub378 \uce74\uc2a4\ud2b8\ub85c\uc758 \ud601\uba85\uad70\uc740 \ucfe0\ubc14 \ub0b4\uc5d0\uc11c \uac8c\ub9b4\ub77c \uae30\uc9c0\ub97c \uad6c\ucd95\ud558\uae30 \uc704\ud574 \ube44\uad50\uc801 \uc0b0\uc138\uac00 \ud5d8\ud55c \uc2dc\uc5d0\ub77c \ub9c8\uc5d0\uc2a4\ud2b8\ub77c \uc5d0 \uae30\uc9c0\ub97c \uad6c\ucd95\ud558\uc600\ub2e4. \uadf8\ub9ac\uace0 \ud53c\ub378 \uce74\uc2a4\ud2b8\ub85c\uc758 \ud601\uba85\uad70\uc740 \uc8fc\uae30\uc801\uc73c\ub85c \ub3c4\uc2dc\uc5d0 \ud601\uba85\uc5d0 \ucc38\uac00\ud558\uc790\ub294 \uc804\ub2e8\uc9c0\ub97c \ub3cc\ub9ac\uba74\uc11c \ud601\uba85\uad70 \ubcd1\uc0ac\ub4e4\uc758 \uc218\ub97c \uc99d\uac00\uc2dc\ucf30\ub2e4.", "paragraph_answer": "\uadf8\ub294 \uc911\ub0a8\ubbf8, \ucfe0\ubc14\uc5d0\uc11c \ub9dd\uba85\ud55c \uc0ac\ud68c\uc8fc\uc758\uc790 \ubc0f \uc88c\ud30c\ubbfc\uc871\uc8fc\uc758\uc790\ub4e4\uc744 \ubaa8\uc544 \uc57d 800\uba85\uc758 \ud601\uba85\uad70\uc744 \ubaa8\uc73c\uac8c \ub41c\ub2e4. \uc774 \uacfc\uc815\uc5d0\uc11c \ucfe0\ubc14 \ubbfc\uc911\ub4e4\uc758 \uc9c0\uc9c0\ub97c \uc5bb\uc5b4 \uae08\uc804\uc801 \ub3c4\uc6c0\uc744 \ubc1b\uc544 \uad70\uc0ac\ub97c \uc720\uc9c0\ud560 \uc218 \uc788\uc5c8\ub2e4. \uadf8\ub9ac\uace0 1956\ub144 \uadf8\ub780\ub9c8 \ud568\uc120\uc744 \ud0c0\uace0, \ucfe0\ubc14\uc5d0 \uc0c1\ub959, \ubc14\ud2f0\uc2a4\ud0c0 \uc815\ubd80\uad70\uacfc \ub0b4\uc804\uc744 \uce58\ub8e8\uc5c8\ub2e4. \ubc14\ud2f0\uc2a4\ud0c0 \uc815\ubd80\uc758 \uacf5\uad70 \ub54c\ubb38\uc5d0, \uc0c1\ub959 \ub3c4\uc911 \uc218\ub9ce\uc740 \ubcd1\uc0ac\ub97c \uc783\uc5c8\uc9c0\ub9cc, \uc0c1\ub959\ud55c \ud6c4 \ubbfc\uc911\uc758 \uc9c0\uc9c0\ub97c \ubc1b\uc544, \ucfe0\ubc14 \ud601\uba85\uad70\uc758 \uc218\ub294 \uae09\uc99d\ud558\uc600\ub2e4. \uc774\uc5d0 \uc704\uae30\uac10\uc744 \ub290\ub080 \ubc14\ud2f0\uc2a4\ud0c0\ub294 \ucfe0\ubc14 \ud601\uba85\uad70\uc5d0 \ube44\ud574 \uc6d4\ub4f1\ud55c \uc7a5\ube44\ub97c \uac16\ucd98 \ub300\ub300\uae09(17\ubcf4\ubcd1\ub300\ub300) \uaddc\ubaa8\uc758 \ubcd1\uc0ac\ub4e4\uc744 \ud30c\uacac\ud558\uc9c0\ub9cc, \ucfe0\ubc14 \ud601\uba85\uad70\uc740 \uc218\uc801 \uc5f4\uc138\uc784\uc5d0\ub3c4 \ubd88\uad6c\ud558\uace0, \ubc14\ud2f0\uc2a4\ud0c0 \uc815\ubd80\uad70\uc744 \ubcc4 \ud0c8 \uc5c6\uc774 \ud574\uce58\uc6e0\ub2e4. \ud53c\ub378 \uce74\uc2a4\ud2b8\ub85c\uc758 \ud601\uba85\uad70\uc740 \ucfe0\ubc14 \ub0b4\uc5d0\uc11c \uac8c\ub9b4\ub77c \uae30\uc9c0\ub97c \uad6c\ucd95\ud558\uae30 \uc704\ud574 \ube44\uad50\uc801 \uc0b0\uc138\uac00 \ud5d8\ud55c \uc2dc\uc5d0\ub77c \ub9c8\uc5d0\uc2a4\ud2b8\ub77c \uc5d0 \uae30\uc9c0\ub97c \uad6c\ucd95\ud558\uc600\ub2e4. \uadf8\ub9ac\uace0 \ud53c\ub378 \uce74\uc2a4\ud2b8\ub85c\uc758 \ud601\uba85\uad70\uc740 \uc8fc\uae30\uc801\uc73c\ub85c \ub3c4\uc2dc\uc5d0 \ud601\uba85\uc5d0 \ucc38\uac00\ud558\uc790\ub294 \uc804\ub2e8\uc9c0\ub97c \ub3cc\ub9ac\uba74\uc11c \ud601\uba85\uad70 \ubcd1\uc0ac\ub4e4\uc758 \uc218\ub97c \uc99d\uac00\uc2dc\ucf30\ub2e4.", "sentence_answer": "\ud53c\ub378 \uce74\uc2a4\ud2b8\ub85c\uc758 \ud601\uba85\uad70\uc740 \ucfe0\ubc14 \ub0b4\uc5d0\uc11c \uac8c\ub9b4\ub77c \uae30\uc9c0\ub97c \uad6c\ucd95\ud558\uae30 \uc704\ud574 \ube44\uad50\uc801 \uc0b0\uc138\uac00 \ud5d8\ud55c \uc2dc\uc5d0\ub77c \ub9c8\uc5d0\uc2a4\ud2b8\ub77c \uc5d0", "paragraph_id": "6568902-11-0", "paragraph_question": "question: \ud53c\ub378 \uce74\uc2a4\ud2b8\ub85c\uc758 \ud601\uba85\uad70\uc740 \ucfe0\ubc14 \ub0b4\uc5d0\uc11c \uc5b4\ub514\uc5d0 \uac8c\ub9b4\ub77c \uae30\uc9c0\ub97c \uad6c\ucd95\ud558\uc600\ub294\uac00?, context: \uadf8\ub294 \uc911\ub0a8\ubbf8, \ucfe0\ubc14\uc5d0\uc11c \ub9dd\uba85\ud55c \uc0ac\ud68c\uc8fc\uc758\uc790 \ubc0f \uc88c\ud30c\ubbfc\uc871\uc8fc\uc758\uc790\ub4e4\uc744 \ubaa8\uc544 \uc57d 800\uba85\uc758 \ud601\uba85\uad70\uc744 \ubaa8\uc73c\uac8c \ub41c\ub2e4. \uc774 \uacfc\uc815\uc5d0\uc11c \ucfe0\ubc14 \ubbfc\uc911\ub4e4\uc758 \uc9c0\uc9c0\ub97c \uc5bb\uc5b4 \uae08\uc804\uc801 \ub3c4\uc6c0\uc744 \ubc1b\uc544 \uad70\uc0ac\ub97c \uc720\uc9c0\ud560 \uc218 \uc788\uc5c8\ub2e4. \uadf8\ub9ac\uace0 1956\ub144 \uadf8\ub780\ub9c8 \ud568\uc120\uc744 \ud0c0\uace0, \ucfe0\ubc14\uc5d0 \uc0c1\ub959, \ubc14\ud2f0\uc2a4\ud0c0 \uc815\ubd80\uad70\uacfc \ub0b4\uc804\uc744 \uce58\ub8e8\uc5c8\ub2e4. \ubc14\ud2f0\uc2a4\ud0c0 \uc815\ubd80\uc758 \uacf5\uad70 \ub54c\ubb38\uc5d0, \uc0c1\ub959 \ub3c4\uc911 \uc218\ub9ce\uc740 \ubcd1\uc0ac\ub97c \uc783\uc5c8\uc9c0\ub9cc, \uc0c1\ub959\ud55c \ud6c4 \ubbfc\uc911\uc758 \uc9c0\uc9c0\ub97c \ubc1b\uc544, \ucfe0\ubc14 \ud601\uba85\uad70\uc758 \uc218\ub294 \uae09\uc99d\ud558\uc600\ub2e4. \uc774\uc5d0 \uc704\uae30\uac10\uc744 \ub290\ub080 \ubc14\ud2f0\uc2a4\ud0c0\ub294 \ucfe0\ubc14 \ud601\uba85\uad70\uc5d0 \ube44\ud574 \uc6d4\ub4f1\ud55c \uc7a5\ube44\ub97c \uac16\ucd98 \ub300\ub300\uae09(17\ubcf4\ubcd1\ub300\ub300) \uaddc\ubaa8\uc758 \ubcd1\uc0ac\ub4e4\uc744 \ud30c\uacac\ud558\uc9c0\ub9cc, \ucfe0\ubc14 \ud601\uba85\uad70\uc740 \uc218\uc801 \uc5f4\uc138\uc784\uc5d0\ub3c4 \ubd88\uad6c\ud558\uace0, \ubc14\ud2f0\uc2a4\ud0c0 \uc815\ubd80\uad70\uc744 \ubcc4 \ud0c8 \uc5c6\uc774 \ud574\uce58\uc6e0\ub2e4. \ud53c\ub378 \uce74\uc2a4\ud2b8\ub85c\uc758 \ud601\uba85\uad70\uc740 \ucfe0\ubc14 \ub0b4\uc5d0\uc11c \uac8c\ub9b4\ub77c \uae30\uc9c0\ub97c \uad6c\ucd95\ud558\uae30 \uc704\ud574 \ube44\uad50\uc801 \uc0b0\uc138\uac00 \ud5d8\ud55c \uc2dc\uc5d0\ub77c \ub9c8\uc5d0\uc2a4\ud2b8\ub77c\uc5d0 \uae30\uc9c0\ub97c \uad6c\ucd95\ud558\uc600\ub2e4. \uadf8\ub9ac\uace0 \ud53c\ub378 \uce74\uc2a4\ud2b8\ub85c\uc758 \ud601\uba85\uad70\uc740 \uc8fc\uae30\uc801\uc73c\ub85c \ub3c4\uc2dc\uc5d0 \ud601\uba85\uc5d0 \ucc38\uac00\ud558\uc790\ub294 \uc804\ub2e8\uc9c0\ub97c \ub3cc\ub9ac\uba74\uc11c \ud601\uba85\uad70 \ubcd1\uc0ac\ub4e4\uc758 \uc218\ub97c \uc99d\uac00\uc2dc\ucf30\ub2e4."} {"question": "\uc774\uc9d1\ud2b8\uc5d0\uc11c \uc0ac\ub78c\uacfc \uc2e0 \uc0ac\uc774\uc758 \ub9e4\uac1c\uc790 \uc5ed\ud560\uc744 \ud558\ub294 \uc0ac\ub78c\uc740?", "paragraph": "\uace0\ub300 \uc774\uc9d1\ud2b8 \uc778\ub4e4\uc740 \uc655\uad8c\ub3c4 \uc790\uc5f0\uc758 \ud798\uc774\ub77c \ubcf4\uc558\ub2e4. \ud30c\ub77c\uc624\ub294 \uc778\uac04\uc774\uc790 \uc2e0\uc774 \uc778\uac04\uc758 \uc721\uccb4\ub85c \uc778\uac04 \uc138\uc0c1\uc5d0 \ub0b4\ub824\uc628 \uc2e0\uc774\ub77c\uace0 \uc0dd\uac01\ud558\uc600\ub2e4. \uadf8 \uacb0\uacfc \ud30c\ub77c\uc624\ub294 \uc774\uc9d1\ud2b8\uc5d0\uc11c \uc0ac\ub78c\uacfc \uc2e0 \uc0ac\uc774\uc758 \ub9e4\uac1c\uc790 \uc5ed\ud560\uc744 \ud558\uc600\ub2e4. \uadf8\ub294 \ub9c8\ud2b8\uc758 \ubc95\uce59\uc744 \ubc1b\ub4e4\uc5b4\uc57c \ud588\uc73c\uba70, \uc801\uc73c\ub85c\ubd80\ud130 \uad6d\uac00\ub97c \uc9c0\ud0a4\uace0, \uacf5\uc815\ud574\uc57c \ud558\uace0, \ubc31\uc131 \uac04 \uac08\ub4f1\uc744 \ud574\uacb0\ud574\uc57c \ud558\uba70, \uc2dd\ub7c9\uc758 \uacf5\uae09\uc744 \ubcf4\uc7a5\ud574\uc57c \ud558\uace0, \uc2e0\uc804 \uac74\ucd95\uacfc \ubd09\ud5cc\uc744 \ud1b5\ud558\uc5ec \uc2e0\uc744 \ub2ec\ub798\ub294 \uc784\ubb34\ub97c \ud558\uc5ec\uc57c \ud588\ub2e4. \uadf8 \uc774\uc720\uc5d0\uc11c \uc2e0\uc804 \ubcbd\ud654\uc5d0\uc11c\ub294 \uc885\uc885 \ub9c8\ud2b8\uc758 \uc0c1\uc9d5\uc744 \ubc1b\ub4dc\ub294 \ud30c\ub77c\uc624\ub4e4\uc758 \ubaa8\uc2b5\uc744 \uc0b4\ud3b4\ubcfc \uc218 \uc788\ub2e4. \uadf8\ub798\uc11c \uc774\uc9d1\ud2b8 \uc2e0\ud559\uc801\uc73c\ub85c \uadf8\ub294 \uc804 \uc138\uacc4\uc758 \ud1b5\uce58\uad8c\uc744 \uac00\uc84c\uc73c\uba70, \uc774\uc9d1\ud2b8\ubfd0\ub9cc \uc544\ub2c8\ub77c \ud0c0\uad6d\uc5d0\uc11c\ub3c4 \"\uc655\"\uc774\ub77c\ub294 \ub2e8\uc5b4\ub294 \ud30c\ub77c\uc624\ub9cc\uc774 \uc4f8 \uc218 \uc788\uc5c8\ub2e4.", "answer": "\ud30c\ub77c\uc624", "sentence": "\ud30c\ub77c\uc624 \ub294 \uc778\uac04\uc774\uc790 \uc2e0\uc774 \uc778\uac04\uc758 \uc721\uccb4\ub85c \uc778\uac04 \uc138\uc0c1\uc5d0 \ub0b4\ub824\uc628 \uc2e0\uc774\ub77c\uace0 \uc0dd\uac01\ud558\uc600\ub2e4.", "paragraph_sentence": "\uace0\ub300 \uc774\uc9d1\ud2b8 \uc778\ub4e4\uc740 \uc655\uad8c\ub3c4 \uc790\uc5f0\uc758 \ud798\uc774\ub77c \ubcf4\uc558\ub2e4. \ud30c\ub77c\uc624 \ub294 \uc778\uac04\uc774\uc790 \uc2e0\uc774 \uc778\uac04\uc758 \uc721\uccb4\ub85c \uc778\uac04 \uc138\uc0c1\uc5d0 \ub0b4\ub824\uc628 \uc2e0\uc774\ub77c\uace0 \uc0dd\uac01\ud558\uc600\ub2e4. \uadf8 \uacb0\uacfc \ud30c\ub77c\uc624\ub294 \uc774\uc9d1\ud2b8\uc5d0\uc11c \uc0ac\ub78c\uacfc \uc2e0 \uc0ac\uc774\uc758 \ub9e4\uac1c\uc790 \uc5ed\ud560\uc744 \ud558\uc600\ub2e4. \uadf8\ub294 \ub9c8\ud2b8\uc758 \ubc95\uce59\uc744 \ubc1b\ub4e4\uc5b4\uc57c \ud588\uc73c\uba70, \uc801\uc73c\ub85c\ubd80\ud130 \uad6d\uac00\ub97c \uc9c0\ud0a4\uace0, \uacf5\uc815\ud574\uc57c \ud558\uace0, \ubc31\uc131 \uac04 \uac08\ub4f1\uc744 \ud574\uacb0\ud574\uc57c \ud558\uba70, \uc2dd\ub7c9\uc758 \uacf5\uae09\uc744 \ubcf4\uc7a5\ud574\uc57c \ud558\uace0, \uc2e0\uc804 \uac74\ucd95\uacfc \ubd09\ud5cc\uc744 \ud1b5\ud558\uc5ec \uc2e0\uc744 \ub2ec\ub798\ub294 \uc784\ubb34\ub97c \ud558\uc5ec\uc57c \ud588\ub2e4. \uadf8 \uc774\uc720\uc5d0\uc11c \uc2e0\uc804 \ubcbd\ud654\uc5d0\uc11c\ub294 \uc885\uc885 \ub9c8\ud2b8\uc758 \uc0c1\uc9d5\uc744 \ubc1b\ub4dc\ub294 \ud30c\ub77c\uc624\ub4e4\uc758 \ubaa8\uc2b5\uc744 \uc0b4\ud3b4\ubcfc \uc218 \uc788\ub2e4. \uadf8\ub798\uc11c \uc774\uc9d1\ud2b8 \uc2e0\ud559\uc801\uc73c\ub85c \uadf8\ub294 \uc804 \uc138\uacc4\uc758 \ud1b5\uce58\uad8c\uc744 \uac00\uc84c\uc73c\uba70, \uc774\uc9d1\ud2b8\ubfd0\ub9cc \uc544\ub2c8\ub77c \ud0c0\uad6d\uc5d0\uc11c\ub3c4 \"\uc655\"\uc774\ub77c\ub294 \ub2e8\uc5b4\ub294 \ud30c\ub77c\uc624\ub9cc\uc774 \uc4f8 \uc218 \uc788\uc5c8\ub2e4.", "paragraph_answer": "\uace0\ub300 \uc774\uc9d1\ud2b8 \uc778\ub4e4\uc740 \uc655\uad8c\ub3c4 \uc790\uc5f0\uc758 \ud798\uc774\ub77c \ubcf4\uc558\ub2e4. \ud30c\ub77c\uc624 \ub294 \uc778\uac04\uc774\uc790 \uc2e0\uc774 \uc778\uac04\uc758 \uc721\uccb4\ub85c \uc778\uac04 \uc138\uc0c1\uc5d0 \ub0b4\ub824\uc628 \uc2e0\uc774\ub77c\uace0 \uc0dd\uac01\ud558\uc600\ub2e4. \uadf8 \uacb0\uacfc \ud30c\ub77c\uc624\ub294 \uc774\uc9d1\ud2b8\uc5d0\uc11c \uc0ac\ub78c\uacfc \uc2e0 \uc0ac\uc774\uc758 \ub9e4\uac1c\uc790 \uc5ed\ud560\uc744 \ud558\uc600\ub2e4. \uadf8\ub294 \ub9c8\ud2b8\uc758 \ubc95\uce59\uc744 \ubc1b\ub4e4\uc5b4\uc57c \ud588\uc73c\uba70, \uc801\uc73c\ub85c\ubd80\ud130 \uad6d\uac00\ub97c \uc9c0\ud0a4\uace0, \uacf5\uc815\ud574\uc57c \ud558\uace0, \ubc31\uc131 \uac04 \uac08\ub4f1\uc744 \ud574\uacb0\ud574\uc57c \ud558\uba70, \uc2dd\ub7c9\uc758 \uacf5\uae09\uc744 \ubcf4\uc7a5\ud574\uc57c \ud558\uace0, \uc2e0\uc804 \uac74\ucd95\uacfc \ubd09\ud5cc\uc744 \ud1b5\ud558\uc5ec \uc2e0\uc744 \ub2ec\ub798\ub294 \uc784\ubb34\ub97c \ud558\uc5ec\uc57c \ud588\ub2e4. \uadf8 \uc774\uc720\uc5d0\uc11c \uc2e0\uc804 \ubcbd\ud654\uc5d0\uc11c\ub294 \uc885\uc885 \ub9c8\ud2b8\uc758 \uc0c1\uc9d5\uc744 \ubc1b\ub4dc\ub294 \ud30c\ub77c\uc624\ub4e4\uc758 \ubaa8\uc2b5\uc744 \uc0b4\ud3b4\ubcfc \uc218 \uc788\ub2e4. \uadf8\ub798\uc11c \uc774\uc9d1\ud2b8 \uc2e0\ud559\uc801\uc73c\ub85c \uadf8\ub294 \uc804 \uc138\uacc4\uc758 \ud1b5\uce58\uad8c\uc744 \uac00\uc84c\uc73c\uba70, \uc774\uc9d1\ud2b8\ubfd0\ub9cc \uc544\ub2c8\ub77c \ud0c0\uad6d\uc5d0\uc11c\ub3c4 \"\uc655\"\uc774\ub77c\ub294 \ub2e8\uc5b4\ub294 \ud30c\ub77c\uc624\ub9cc\uc774 \uc4f8 \uc218 \uc788\uc5c8\ub2e4.", "sentence_answer": " \ud30c\ub77c\uc624 \ub294 \uc778\uac04\uc774\uc790 \uc2e0\uc774 \uc778\uac04\uc758 \uc721\uccb4\ub85c \uc778\uac04 \uc138\uc0c1\uc5d0 \ub0b4\ub824\uc628 \uc2e0\uc774\ub77c\uace0 \uc0dd\uac01\ud558\uc600\ub2e4.", "paragraph_id": "6657788-7-0", "paragraph_question": "question: \uc774\uc9d1\ud2b8\uc5d0\uc11c \uc0ac\ub78c\uacfc \uc2e0 \uc0ac\uc774\uc758 \ub9e4\uac1c\uc790 \uc5ed\ud560\uc744 \ud558\ub294 \uc0ac\ub78c\uc740?, context: \uace0\ub300 \uc774\uc9d1\ud2b8 \uc778\ub4e4\uc740 \uc655\uad8c\ub3c4 \uc790\uc5f0\uc758 \ud798\uc774\ub77c \ubcf4\uc558\ub2e4. \ud30c\ub77c\uc624\ub294 \uc778\uac04\uc774\uc790 \uc2e0\uc774 \uc778\uac04\uc758 \uc721\uccb4\ub85c \uc778\uac04 \uc138\uc0c1\uc5d0 \ub0b4\ub824\uc628 \uc2e0\uc774\ub77c\uace0 \uc0dd\uac01\ud558\uc600\ub2e4. \uadf8 \uacb0\uacfc \ud30c\ub77c\uc624\ub294 \uc774\uc9d1\ud2b8\uc5d0\uc11c \uc0ac\ub78c\uacfc \uc2e0 \uc0ac\uc774\uc758 \ub9e4\uac1c\uc790 \uc5ed\ud560\uc744 \ud558\uc600\ub2e4. \uadf8\ub294 \ub9c8\ud2b8\uc758 \ubc95\uce59\uc744 \ubc1b\ub4e4\uc5b4\uc57c \ud588\uc73c\uba70, \uc801\uc73c\ub85c\ubd80\ud130 \uad6d\uac00\ub97c \uc9c0\ud0a4\uace0, \uacf5\uc815\ud574\uc57c \ud558\uace0, \ubc31\uc131 \uac04 \uac08\ub4f1\uc744 \ud574\uacb0\ud574\uc57c \ud558\uba70, \uc2dd\ub7c9\uc758 \uacf5\uae09\uc744 \ubcf4\uc7a5\ud574\uc57c \ud558\uace0, \uc2e0\uc804 \uac74\ucd95\uacfc \ubd09\ud5cc\uc744 \ud1b5\ud558\uc5ec \uc2e0\uc744 \ub2ec\ub798\ub294 \uc784\ubb34\ub97c \ud558\uc5ec\uc57c \ud588\ub2e4. \uadf8 \uc774\uc720\uc5d0\uc11c \uc2e0\uc804 \ubcbd\ud654\uc5d0\uc11c\ub294 \uc885\uc885 \ub9c8\ud2b8\uc758 \uc0c1\uc9d5\uc744 \ubc1b\ub4dc\ub294 \ud30c\ub77c\uc624\ub4e4\uc758 \ubaa8\uc2b5\uc744 \uc0b4\ud3b4\ubcfc \uc218 \uc788\ub2e4. \uadf8\ub798\uc11c \uc774\uc9d1\ud2b8 \uc2e0\ud559\uc801\uc73c\ub85c \uadf8\ub294 \uc804 \uc138\uacc4\uc758 \ud1b5\uce58\uad8c\uc744 \uac00\uc84c\uc73c\uba70, \uc774\uc9d1\ud2b8\ubfd0\ub9cc \uc544\ub2c8\ub77c \ud0c0\uad6d\uc5d0\uc11c\ub3c4 \"\uc655\"\uc774\ub77c\ub294 \ub2e8\uc5b4\ub294 \ud30c\ub77c\uc624\ub9cc\uc774 \uc4f8 \uc218 \uc788\uc5c8\ub2e4."} {"question": "\uc789\uc5b4\uc5d0\uac8c \ubc1c\ubcd1\ud558\ub294 \ubc14\uc774\ub7ec\uc2a4\uc758 \uc774\ub984\uc740?", "paragraph": "\ub2e4\ub978 \ub3d9\ubb3c\ub4e4 \ucc98\ub7fc \uc5b4\ub958 \uc5ed\uc2dc \uc9c8\ubcd1\ub4e4\uacfc \uae30\uc0dd\ucda9\ub4e4\ub85c\ubd80\ud130 \uace0\ud1b5\uc744 \uacaa\ub294\ub2e4. \uc9c8\ubcd1\uc744 \ub9c9\uae30\uc704\ud574 \uc5b4\ub958\ub4e4\uc740 \ub2e4\uc591\ud55c \ubc29\uc5b4\uccb4\uacc4\ub97c \uac16\uace0\uc788\ub2e4. \ud45c\ud53c\uc5d0 \uc758\ud574 \ubcf4\ud638\ub418\ub294, \ubbf8\uc0dd\ubb3c(\uc601\uc5b4: microorganism)\ub4e4\uc744 \uc7a1\uac70\ub098 \uadf8\ub4e4\uc758 \uc131\uc7a5\uc744 \uc5b5\uc81c\ud558\ub294 \uc810\uc561\uce35(\uc601\uc5b4: mucus layer)\uc744 \ube44\ub86f\ud55c, \ud53c\ubd80\uc640 \ube44\ub298\uc774 \ud3c9\uc0c1\uc801\uc778 \ubc29\uc5b4\uccb4\uacc4\ub4e4\uc774\ub2e4. \ubcd1\uc6d0\uade0(\uc601\uc5b4: pathogen)\ub4e4\uc774 \uc774\ub7ec\ud55c \ubc29\uc5b4\uccb4\uacc4\ub97c \ub6ab\uace0 \uce68\ud22c\ud558\uba74, \ubb3c\uace0\uae30\ub4e4\uc740 , \uac10\uc5fc\ub41c \uacf3\uc744 \ud5a5\ud574 \ud608\ub958\ub97c \uc99d\uac00\uc2dc\ud0b4\uc73c\ub85c\uc368 \ubcd1\uc6d0\uade0\uc744 \uc0c1\ub300\ud558\uae30 \uc704\ud55c \ubc31\ud608\uad6c\ub97c \uc804\ub2ec\ud558\ub294, \uc5fc\uc99d\ubc18\uc751\uc744 \uc2dc\uc791\ud560 \uc218 \uc788\ub2e4. \ud2b9\uc815\ud55c \ubc29\uc5b4\uccb4\uacc4\ub294 \ubb3c\uace0\uae30\uc758 \uc2e0\uccb4\uc5d0 \uc758\ud574 \ud310\ubcc4\ub41c \ud2b9\uc815\ud55c \ubcd1\uc6d0\uade0\uc5d0 \ubc18\uc751\ud55c\ub2e4. \uc608\ub85c, \uba74\uc5ed\ubc18\uc751\uc774 \uc788\ub2e4. \ucd5c\uadfc\uc5d0, \ubc31\uc2e0\uc774 \uc218\uc0b0\uc591\uc2dd(\uc601\uc5b4: aguaculture)\uacfc \uad00\uc0c1\uc6a9 \ubb3c\uace0\uae30 \uc0ac\uc721\uc5d0 \ub110\ub9ac \uc0ac\uc6a9\ub418\uace0 \uc788\ub2e4. \uc608\ub85c\ub294 \uc789\uc5b4\uc5d0\uac8c \ubc1c\ubcd1\ud558\ub294 \uc789\uc5b4\ud3ec\uc9c4 \ubc14\uc774\ub7ec\uc2a4\uc640, \uc591\uc2dd\ub41c \uc5f0\uc5b4\uc5d0\uc11c \uc0ac\uc6a9\ub418\ub294 \uc808\uc885\uc99d(\uc601\uc5b4: furunculosis)\ubc31\uc2e0\uc774 \uc788\ub2e4.", "answer": "\uc789\uc5b4\ud3ec\uc9c4 \ubc14\uc774\ub7ec\uc2a4", "sentence": "\uc608\ub85c\ub294 \uc789\uc5b4\uc5d0\uac8c \ubc1c\ubcd1\ud558\ub294 \uc789\uc5b4\ud3ec\uc9c4 \ubc14\uc774\ub7ec\uc2a4 \uc640, \uc591\uc2dd\ub41c \uc5f0\uc5b4\uc5d0\uc11c \uc0ac\uc6a9\ub418\ub294 \uc808\uc885\uc99d(\uc601\uc5b4: furunculosis)\ubc31\uc2e0\uc774 \uc788\ub2e4.", "paragraph_sentence": "\ub2e4\ub978 \ub3d9\ubb3c\ub4e4 \ucc98\ub7fc \uc5b4\ub958 \uc5ed\uc2dc \uc9c8\ubcd1\ub4e4\uacfc \uae30\uc0dd\ucda9\ub4e4\ub85c\ubd80\ud130 \uace0\ud1b5\uc744 \uacaa\ub294\ub2e4. \uc9c8\ubcd1\uc744 \ub9c9\uae30\uc704\ud574 \uc5b4\ub958\ub4e4\uc740 \ub2e4\uc591\ud55c \ubc29\uc5b4\uccb4\uacc4\ub97c \uac16\uace0\uc788\ub2e4. \ud45c\ud53c\uc5d0 \uc758\ud574 \ubcf4\ud638\ub418\ub294, \ubbf8\uc0dd\ubb3c(\uc601\uc5b4: microorganism)\ub4e4\uc744 \uc7a1\uac70\ub098 \uadf8\ub4e4\uc758 \uc131\uc7a5\uc744 \uc5b5\uc81c\ud558\ub294 \uc810\uc561\uce35(\uc601\uc5b4: mucus layer)\uc744 \ube44\ub86f\ud55c, \ud53c\ubd80\uc640 \ube44\ub298\uc774 \ud3c9\uc0c1\uc801\uc778 \ubc29\uc5b4\uccb4\uacc4\ub4e4\uc774\ub2e4. \ubcd1\uc6d0\uade0(\uc601\uc5b4: pathogen)\ub4e4\uc774 \uc774\ub7ec\ud55c \ubc29\uc5b4\uccb4\uacc4\ub97c \ub6ab\uace0 \uce68\ud22c\ud558\uba74, \ubb3c\uace0\uae30\ub4e4\uc740 , \uac10\uc5fc\ub41c \uacf3\uc744 \ud5a5\ud574 \ud608\ub958\ub97c \uc99d\uac00\uc2dc\ud0b4\uc73c\ub85c\uc368 \ubcd1\uc6d0\uade0\uc744 \uc0c1\ub300\ud558\uae30 \uc704\ud55c \ubc31\ud608\uad6c\ub97c \uc804\ub2ec\ud558\ub294, \uc5fc\uc99d\ubc18\uc751\uc744 \uc2dc\uc791\ud560 \uc218 \uc788\ub2e4. \ud2b9\uc815\ud55c \ubc29\uc5b4\uccb4\uacc4\ub294 \ubb3c\uace0\uae30\uc758 \uc2e0\uccb4\uc5d0 \uc758\ud574 \ud310\ubcc4\ub41c \ud2b9\uc815\ud55c \ubcd1\uc6d0\uade0\uc5d0 \ubc18\uc751\ud55c\ub2e4. \uc608\ub85c, \uba74\uc5ed\ubc18\uc751\uc774 \uc788\ub2e4. \ucd5c\uadfc\uc5d0, \ubc31\uc2e0\uc774 \uc218\uc0b0\uc591\uc2dd(\uc601\uc5b4: aguaculture)\uacfc \uad00\uc0c1\uc6a9 \ubb3c\uace0\uae30 \uc0ac\uc721\uc5d0 \ub110\ub9ac \uc0ac\uc6a9\ub418\uace0 \uc788\ub2e4. \uc608\ub85c\ub294 \uc789\uc5b4\uc5d0\uac8c \ubc1c\ubcd1\ud558\ub294 \uc789\uc5b4\ud3ec\uc9c4 \ubc14\uc774\ub7ec\uc2a4 \uc640, \uc591\uc2dd\ub41c \uc5f0\uc5b4\uc5d0\uc11c \uc0ac\uc6a9\ub418\ub294 \uc808\uc885\uc99d(\uc601\uc5b4: furunculosis)\ubc31\uc2e0\uc774 \uc788\ub2e4. ", "paragraph_answer": "\ub2e4\ub978 \ub3d9\ubb3c\ub4e4 \ucc98\ub7fc \uc5b4\ub958 \uc5ed\uc2dc \uc9c8\ubcd1\ub4e4\uacfc \uae30\uc0dd\ucda9\ub4e4\ub85c\ubd80\ud130 \uace0\ud1b5\uc744 \uacaa\ub294\ub2e4. \uc9c8\ubcd1\uc744 \ub9c9\uae30\uc704\ud574 \uc5b4\ub958\ub4e4\uc740 \ub2e4\uc591\ud55c \ubc29\uc5b4\uccb4\uacc4\ub97c \uac16\uace0\uc788\ub2e4. \ud45c\ud53c\uc5d0 \uc758\ud574 \ubcf4\ud638\ub418\ub294, \ubbf8\uc0dd\ubb3c(\uc601\uc5b4: microorganism)\ub4e4\uc744 \uc7a1\uac70\ub098 \uadf8\ub4e4\uc758 \uc131\uc7a5\uc744 \uc5b5\uc81c\ud558\ub294 \uc810\uc561\uce35(\uc601\uc5b4: mucus layer)\uc744 \ube44\ub86f\ud55c, \ud53c\ubd80\uc640 \ube44\ub298\uc774 \ud3c9\uc0c1\uc801\uc778 \ubc29\uc5b4\uccb4\uacc4\ub4e4\uc774\ub2e4. \ubcd1\uc6d0\uade0(\uc601\uc5b4: pathogen)\ub4e4\uc774 \uc774\ub7ec\ud55c \ubc29\uc5b4\uccb4\uacc4\ub97c \ub6ab\uace0 \uce68\ud22c\ud558\uba74, \ubb3c\uace0\uae30\ub4e4\uc740 , \uac10\uc5fc\ub41c \uacf3\uc744 \ud5a5\ud574 \ud608\ub958\ub97c \uc99d\uac00\uc2dc\ud0b4\uc73c\ub85c\uc368 \ubcd1\uc6d0\uade0\uc744 \uc0c1\ub300\ud558\uae30 \uc704\ud55c \ubc31\ud608\uad6c\ub97c \uc804\ub2ec\ud558\ub294, \uc5fc\uc99d\ubc18\uc751\uc744 \uc2dc\uc791\ud560 \uc218 \uc788\ub2e4. \ud2b9\uc815\ud55c \ubc29\uc5b4\uccb4\uacc4\ub294 \ubb3c\uace0\uae30\uc758 \uc2e0\uccb4\uc5d0 \uc758\ud574 \ud310\ubcc4\ub41c \ud2b9\uc815\ud55c \ubcd1\uc6d0\uade0\uc5d0 \ubc18\uc751\ud55c\ub2e4. \uc608\ub85c, \uba74\uc5ed\ubc18\uc751\uc774 \uc788\ub2e4. \ucd5c\uadfc\uc5d0, \ubc31\uc2e0\uc774 \uc218\uc0b0\uc591\uc2dd(\uc601\uc5b4: aguaculture)\uacfc \uad00\uc0c1\uc6a9 \ubb3c\uace0\uae30 \uc0ac\uc721\uc5d0 \ub110\ub9ac \uc0ac\uc6a9\ub418\uace0 \uc788\ub2e4. \uc608\ub85c\ub294 \uc789\uc5b4\uc5d0\uac8c \ubc1c\ubcd1\ud558\ub294 \uc789\uc5b4\ud3ec\uc9c4 \ubc14\uc774\ub7ec\uc2a4 \uc640, \uc591\uc2dd\ub41c \uc5f0\uc5b4\uc5d0\uc11c \uc0ac\uc6a9\ub418\ub294 \uc808\uc885\uc99d(\uc601\uc5b4: furunculosis)\ubc31\uc2e0\uc774 \uc788\ub2e4.", "sentence_answer": "\uc608\ub85c\ub294 \uc789\uc5b4\uc5d0\uac8c \ubc1c\ubcd1\ud558\ub294 \uc789\uc5b4\ud3ec\uc9c4 \ubc14\uc774\ub7ec\uc2a4 \uc640, \uc591\uc2dd\ub41c \uc5f0\uc5b4\uc5d0\uc11c \uc0ac\uc6a9\ub418\ub294 \uc808\uc885\uc99d(\uc601\uc5b4: furunculosis)\ubc31\uc2e0\uc774 \uc788\ub2e4.", "paragraph_id": "6412467-12-1", "paragraph_question": "question: \uc789\uc5b4\uc5d0\uac8c \ubc1c\ubcd1\ud558\ub294 \ubc14\uc774\ub7ec\uc2a4\uc758 \uc774\ub984\uc740?, context: \ub2e4\ub978 \ub3d9\ubb3c\ub4e4 \ucc98\ub7fc \uc5b4\ub958 \uc5ed\uc2dc \uc9c8\ubcd1\ub4e4\uacfc \uae30\uc0dd\ucda9\ub4e4\ub85c\ubd80\ud130 \uace0\ud1b5\uc744 \uacaa\ub294\ub2e4. \uc9c8\ubcd1\uc744 \ub9c9\uae30\uc704\ud574 \uc5b4\ub958\ub4e4\uc740 \ub2e4\uc591\ud55c \ubc29\uc5b4\uccb4\uacc4\ub97c \uac16\uace0\uc788\ub2e4. \ud45c\ud53c\uc5d0 \uc758\ud574 \ubcf4\ud638\ub418\ub294, \ubbf8\uc0dd\ubb3c(\uc601\uc5b4: microorganism)\ub4e4\uc744 \uc7a1\uac70\ub098 \uadf8\ub4e4\uc758 \uc131\uc7a5\uc744 \uc5b5\uc81c\ud558\ub294 \uc810\uc561\uce35(\uc601\uc5b4: mucus layer)\uc744 \ube44\ub86f\ud55c, \ud53c\ubd80\uc640 \ube44\ub298\uc774 \ud3c9\uc0c1\uc801\uc778 \ubc29\uc5b4\uccb4\uacc4\ub4e4\uc774\ub2e4. \ubcd1\uc6d0\uade0(\uc601\uc5b4: pathogen)\ub4e4\uc774 \uc774\ub7ec\ud55c \ubc29\uc5b4\uccb4\uacc4\ub97c \ub6ab\uace0 \uce68\ud22c\ud558\uba74, \ubb3c\uace0\uae30\ub4e4\uc740 , \uac10\uc5fc\ub41c \uacf3\uc744 \ud5a5\ud574 \ud608\ub958\ub97c \uc99d\uac00\uc2dc\ud0b4\uc73c\ub85c\uc368 \ubcd1\uc6d0\uade0\uc744 \uc0c1\ub300\ud558\uae30 \uc704\ud55c \ubc31\ud608\uad6c\ub97c \uc804\ub2ec\ud558\ub294, \uc5fc\uc99d\ubc18\uc751\uc744 \uc2dc\uc791\ud560 \uc218 \uc788\ub2e4. \ud2b9\uc815\ud55c \ubc29\uc5b4\uccb4\uacc4\ub294 \ubb3c\uace0\uae30\uc758 \uc2e0\uccb4\uc5d0 \uc758\ud574 \ud310\ubcc4\ub41c \ud2b9\uc815\ud55c \ubcd1\uc6d0\uade0\uc5d0 \ubc18\uc751\ud55c\ub2e4. \uc608\ub85c, \uba74\uc5ed\ubc18\uc751\uc774 \uc788\ub2e4. \ucd5c\uadfc\uc5d0, \ubc31\uc2e0\uc774 \uc218\uc0b0\uc591\uc2dd(\uc601\uc5b4: aguaculture)\uacfc \uad00\uc0c1\uc6a9 \ubb3c\uace0\uae30 \uc0ac\uc721\uc5d0 \ub110\ub9ac \uc0ac\uc6a9\ub418\uace0 \uc788\ub2e4. \uc608\ub85c\ub294 \uc789\uc5b4\uc5d0\uac8c \ubc1c\ubcd1\ud558\ub294 \uc789\uc5b4\ud3ec\uc9c4 \ubc14\uc774\ub7ec\uc2a4\uc640, \uc591\uc2dd\ub41c \uc5f0\uc5b4\uc5d0\uc11c \uc0ac\uc6a9\ub418\ub294 \uc808\uc885\uc99d(\uc601\uc5b4: furunculosis)\ubc31\uc2e0\uc774 \uc788\ub2e4."} {"question": "single ladies\ub294 VH1\uac00 \uc120\uc815\ud55c 2000\ub144\ub300 \uac00\uc7a5 \uc704\ub300\ud55c \ub178\ub798 100 \uc911 \uba87 \uc704\uc5d0 \uc62c\ub790\ub294\uac00?", "paragraph": "Eye Weekly\uc758 \ud3c9\ub860\uac00\ub4e4\uc774 \uc120\uc815\ud55c 2008\ub144 \ucd5c\uace0\uc758 \uc2f1\uae00 \uc21c\uc704\uc5d0\uc11c \u3008Single Ladies\u3009 \ub294 6\uc704\uc5d0 \uc120\uc815\ub418\uc5c8\uc73c\uba70, About.com\uc758 \ub9c8\ud06c \uc5d0\ub4dc\uc6cc\ub4dc \ub124\ub85c\uac00 \ubf51\uc740 2008\ub144 \ucd5c\uace0\uc758 \uc54c\uc564\ube44 \ub178\ub798 \uc21c\uc704\uc5d0\uc11c 6\uc704\uc5d0 \uc120\uc815\ub418\uc5c8\ub2e4. \u300a\ube4c\ub9ac\uc9c0 \ubcf4\uc774\uc2a4\u300b\uc758 \uc5f0\ub9d0 Pazz & Jop \uc2f1\uae00 \ubaa9\ub85d\uc5d0\uc11c \u3008Single Ladies\u3009\ub294 2008\ub144\uc5d0\ub294 3\uc704, 2009\ub144\uc5d0\ub294 40\uc704\uc5d0 \uc62c\ub790\ub2e4. \uac8c\ub2e4\uac00 Maurice Joshua Club Mix \ubc84\uc804\ub3c4 2008\ub144\uc5d0 443\uc704\ub85c \ubaa9\ub85d\uc5d0 \uc62c\ub77c\uac14\ub2e4. \ube14\ub799 \uc5d4\ud130\ud14c\uc778\uba3c\ud2b8 \ud154\ub808\ube44\uc804 (BET)\uc774 \uc120\uc815\ud55c 2000\ub144\ub300 \ucd5c\uace0\uc758 \ub178\ub798 \ubaa9\ub85d\uc5d0\ub3c4 \uc774\ub984\uc744 \uc62c\ub838\ub2e4. \u300a\ubcf4\uc2a4\ud134 \uae00\ub85c\ube0c\u300b\uc758 \uc0ac\ub77c \ub85c\ub9cc\uc740 \uc9c0\ub09c 10\ub144\uac04 \uac00\uc7a5 \ub9e4\ub825\uc801\uc778 \ub178\ub798 4\uc704\uc5d0 \uc120\uc815\ud558\uba70 \"\ube44\uc698\uc138\ub294 \ub9ac\uc5b4\ud0c0\ub4dc\uc640 \ud568\uaed8 \uac04\ub2e8\ud558\uac8c \uafb8\ubc08\uc73c\ub85c\uc368 \ud558\ub098\uc758 \uc139\uc2dc\ud558\uba74\uc11c\ub3c4 \uad6c\uc2dd \ub290\ub08c\uc774 \ub098\ub294 \ud328\ud0a4\uc9c0\ub97c \ub9cc\ub4e4\uc5b4\ub0c8\ub2e4. \ubba4\uc9c1\ube44\ub514\uc624\ub294 \uc804 \uc138\uacc4\uc5d0 \ucda4\uacfc \uc720\ud29c\ube0c\uc758 \ud328\ub7ec\ub514 \uc5f4\ud48d\uc744 \uc77c\uc73c\ucf30\ub2e4\"\uace0 \uc37c\ub2e4. VH1\uc758 2000\ub144\ub300 \uac00\uc7a5 \uc704\ub300\ud55c \ub178\ub798 100\uc704 \uc21c\uc704\uc5d0\ub3c4 16\uc704\uc5d0 \uc62c\ub790\ub2e4. \ucc99 \ud074\ub85c\uc2a4\ud130\ub9e8\uc758 2009\ub144 \ucc45 Eating the Dinosaur\uc5d0\uc11c \"\uc8fc\uc7a5\ud558\uac74\ub300 \ub3c4\uc2dc\uc758 \ucc98\ub140 \ud30c\ud2f0\ub97c \ub178\ub9b0 \uba85\ubc31\ud55c \uccab \ub178\ub798\uc774\ub2e4\"\ub77c\uba70 \u3008Single Ladies\u3009\uc5d0 \ub300\ud574 \uc37c\ub2e4.", "answer": "16\uc704", "sentence": "2000\ub144\ub300 \uac00\uc7a5 \uc704\ub300\ud55c \ub178\ub798 100\uc704 \uc21c\uc704\uc5d0\ub3c4 16\uc704 \uc5d0 \uc62c\ub790\ub2e4.", "paragraph_sentence": "Eye Weekly\uc758 \ud3c9\ub860\uac00\ub4e4\uc774 \uc120\uc815\ud55c 2008\ub144 \ucd5c\uace0\uc758 \uc2f1\uae00 \uc21c\uc704\uc5d0\uc11c \u3008Single Ladies\u3009 \ub294 6\uc704\uc5d0 \uc120\uc815\ub418\uc5c8\uc73c\uba70, About.com\uc758 \ub9c8\ud06c \uc5d0\ub4dc\uc6cc\ub4dc \ub124\ub85c\uac00 \ubf51\uc740 2008\ub144 \ucd5c\uace0\uc758 \uc54c\uc564\ube44 \ub178\ub798 \uc21c\uc704\uc5d0\uc11c 6\uc704\uc5d0 \uc120\uc815\ub418\uc5c8\ub2e4. \u300a\ube4c\ub9ac\uc9c0 \ubcf4\uc774\uc2a4\u300b\uc758 \uc5f0\ub9d0 Pazz & Jop \uc2f1\uae00 \ubaa9\ub85d\uc5d0\uc11c \u3008Single Ladies\u3009\ub294 2008\ub144\uc5d0\ub294 3\uc704, 2009\ub144\uc5d0\ub294 40\uc704\uc5d0 \uc62c\ub790\ub2e4. \uac8c\ub2e4\uac00 Maurice Joshua Club Mix \ubc84\uc804\ub3c4 2008\ub144\uc5d0 443\uc704\ub85c \ubaa9\ub85d\uc5d0 \uc62c\ub77c\uac14\ub2e4. \ube14\ub799 \uc5d4\ud130\ud14c\uc778\uba3c\ud2b8 \ud154\ub808\ube44\uc804 (BET)\uc774 \uc120\uc815\ud55c 2000\ub144\ub300 \ucd5c\uace0\uc758 \ub178\ub798 \ubaa9\ub85d\uc5d0\ub3c4 \uc774\ub984\uc744 \uc62c\ub838\ub2e4. \u300a\ubcf4\uc2a4\ud134 \uae00\ub85c\ube0c\u300b\uc758 \uc0ac\ub77c \ub85c\ub9cc\uc740 \uc9c0\ub09c 10\ub144\uac04 \uac00\uc7a5 \ub9e4\ub825\uc801\uc778 \ub178\ub798 4\uc704\uc5d0 \uc120\uc815\ud558\uba70 \"\ube44\uc698\uc138\ub294 \ub9ac\uc5b4\ud0c0\ub4dc\uc640 \ud568\uaed8 \uac04\ub2e8\ud558\uac8c \uafb8\ubc08\uc73c\ub85c\uc368 \ud558\ub098\uc758 \uc139\uc2dc\ud558\uba74\uc11c\ub3c4 \uad6c\uc2dd \ub290\ub08c\uc774 \ub098\ub294 \ud328\ud0a4\uc9c0\ub97c \ub9cc\ub4e4\uc5b4\ub0c8\ub2e4. \ubba4\uc9c1\ube44\ub514\uc624\ub294 \uc804 \uc138\uacc4\uc5d0 \ucda4\uacfc \uc720\ud29c\ube0c\uc758 \ud328\ub7ec\ub514 \uc5f4\ud48d\uc744 \uc77c\uc73c\ucf30\ub2e4\"\uace0 \uc37c\ub2e4. VH1\uc758 2000\ub144\ub300 \uac00\uc7a5 \uc704\ub300\ud55c \ub178\ub798 100\uc704 \uc21c\uc704\uc5d0\ub3c4 16\uc704 \uc5d0 \uc62c\ub790\ub2e4. \ucc99 \ud074\ub85c\uc2a4\ud130\ub9e8\uc758 2009\ub144 \ucc45 Eating the Dinosaur\uc5d0\uc11c \"\uc8fc\uc7a5\ud558\uac74\ub300 \ub3c4\uc2dc\uc758 \ucc98\ub140 \ud30c\ud2f0\ub97c \ub178\ub9b0 \uba85\ubc31\ud55c \uccab \ub178\ub798\uc774\ub2e4\"\ub77c\uba70 \u3008Single Ladies\u3009\uc5d0 \ub300\ud574 \uc37c\ub2e4.", "paragraph_answer": "Eye Weekly\uc758 \ud3c9\ub860\uac00\ub4e4\uc774 \uc120\uc815\ud55c 2008\ub144 \ucd5c\uace0\uc758 \uc2f1\uae00 \uc21c\uc704\uc5d0\uc11c \u3008Single Ladies\u3009 \ub294 6\uc704\uc5d0 \uc120\uc815\ub418\uc5c8\uc73c\uba70, About.com\uc758 \ub9c8\ud06c \uc5d0\ub4dc\uc6cc\ub4dc \ub124\ub85c\uac00 \ubf51\uc740 2008\ub144 \ucd5c\uace0\uc758 \uc54c\uc564\ube44 \ub178\ub798 \uc21c\uc704\uc5d0\uc11c 6\uc704\uc5d0 \uc120\uc815\ub418\uc5c8\ub2e4. \u300a\ube4c\ub9ac\uc9c0 \ubcf4\uc774\uc2a4\u300b\uc758 \uc5f0\ub9d0 Pazz & Jop \uc2f1\uae00 \ubaa9\ub85d\uc5d0\uc11c \u3008Single Ladies\u3009\ub294 2008\ub144\uc5d0\ub294 3\uc704, 2009\ub144\uc5d0\ub294 40\uc704\uc5d0 \uc62c\ub790\ub2e4. \uac8c\ub2e4\uac00 Maurice Joshua Club Mix \ubc84\uc804\ub3c4 2008\ub144\uc5d0 443\uc704\ub85c \ubaa9\ub85d\uc5d0 \uc62c\ub77c\uac14\ub2e4. \ube14\ub799 \uc5d4\ud130\ud14c\uc778\uba3c\ud2b8 \ud154\ub808\ube44\uc804 (BET)\uc774 \uc120\uc815\ud55c 2000\ub144\ub300 \ucd5c\uace0\uc758 \ub178\ub798 \ubaa9\ub85d\uc5d0\ub3c4 \uc774\ub984\uc744 \uc62c\ub838\ub2e4. \u300a\ubcf4\uc2a4\ud134 \uae00\ub85c\ube0c\u300b\uc758 \uc0ac\ub77c \ub85c\ub9cc\uc740 \uc9c0\ub09c 10\ub144\uac04 \uac00\uc7a5 \ub9e4\ub825\uc801\uc778 \ub178\ub798 4\uc704\uc5d0 \uc120\uc815\ud558\uba70 \"\ube44\uc698\uc138\ub294 \ub9ac\uc5b4\ud0c0\ub4dc\uc640 \ud568\uaed8 \uac04\ub2e8\ud558\uac8c \uafb8\ubc08\uc73c\ub85c\uc368 \ud558\ub098\uc758 \uc139\uc2dc\ud558\uba74\uc11c\ub3c4 \uad6c\uc2dd \ub290\ub08c\uc774 \ub098\ub294 \ud328\ud0a4\uc9c0\ub97c \ub9cc\ub4e4\uc5b4\ub0c8\ub2e4. \ubba4\uc9c1\ube44\ub514\uc624\ub294 \uc804 \uc138\uacc4\uc5d0 \ucda4\uacfc \uc720\ud29c\ube0c\uc758 \ud328\ub7ec\ub514 \uc5f4\ud48d\uc744 \uc77c\uc73c\ucf30\ub2e4\"\uace0 \uc37c\ub2e4. VH1\uc758 2000\ub144\ub300 \uac00\uc7a5 \uc704\ub300\ud55c \ub178\ub798 100\uc704 \uc21c\uc704\uc5d0\ub3c4 16\uc704 \uc5d0 \uc62c\ub790\ub2e4. \ucc99 \ud074\ub85c\uc2a4\ud130\ub9e8\uc758 2009\ub144 \ucc45 Eating the Dinosaur\uc5d0\uc11c \"\uc8fc\uc7a5\ud558\uac74\ub300 \ub3c4\uc2dc\uc758 \ucc98\ub140 \ud30c\ud2f0\ub97c \ub178\ub9b0 \uba85\ubc31\ud55c \uccab \ub178\ub798\uc774\ub2e4\"\ub77c\uba70 \u3008Single Ladies\u3009\uc5d0 \ub300\ud574 \uc37c\ub2e4.", "sentence_answer": "2000\ub144\ub300 \uac00\uc7a5 \uc704\ub300\ud55c \ub178\ub798 100\uc704 \uc21c\uc704\uc5d0\ub3c4 16\uc704 \uc5d0 \uc62c\ub790\ub2e4.", "paragraph_id": "6488992-12-1", "paragraph_question": "question: single ladies\ub294 VH1\uac00 \uc120\uc815\ud55c 2000\ub144\ub300 \uac00\uc7a5 \uc704\ub300\ud55c \ub178\ub798 100 \uc911 \uba87 \uc704\uc5d0 \uc62c\ub790\ub294\uac00?, context: Eye Weekly\uc758 \ud3c9\ub860\uac00\ub4e4\uc774 \uc120\uc815\ud55c 2008\ub144 \ucd5c\uace0\uc758 \uc2f1\uae00 \uc21c\uc704\uc5d0\uc11c \u3008Single Ladies\u3009 \ub294 6\uc704\uc5d0 \uc120\uc815\ub418\uc5c8\uc73c\uba70, About.com\uc758 \ub9c8\ud06c \uc5d0\ub4dc\uc6cc\ub4dc \ub124\ub85c\uac00 \ubf51\uc740 2008\ub144 \ucd5c\uace0\uc758 \uc54c\uc564\ube44 \ub178\ub798 \uc21c\uc704\uc5d0\uc11c 6\uc704\uc5d0 \uc120\uc815\ub418\uc5c8\ub2e4. \u300a\ube4c\ub9ac\uc9c0 \ubcf4\uc774\uc2a4\u300b\uc758 \uc5f0\ub9d0 Pazz & Jop \uc2f1\uae00 \ubaa9\ub85d\uc5d0\uc11c \u3008Single Ladies\u3009\ub294 2008\ub144\uc5d0\ub294 3\uc704, 2009\ub144\uc5d0\ub294 40\uc704\uc5d0 \uc62c\ub790\ub2e4. \uac8c\ub2e4\uac00 Maurice Joshua Club Mix \ubc84\uc804\ub3c4 2008\ub144\uc5d0 443\uc704\ub85c \ubaa9\ub85d\uc5d0 \uc62c\ub77c\uac14\ub2e4. \ube14\ub799 \uc5d4\ud130\ud14c\uc778\uba3c\ud2b8 \ud154\ub808\ube44\uc804 (BET)\uc774 \uc120\uc815\ud55c 2000\ub144\ub300 \ucd5c\uace0\uc758 \ub178\ub798 \ubaa9\ub85d\uc5d0\ub3c4 \uc774\ub984\uc744 \uc62c\ub838\ub2e4. \u300a\ubcf4\uc2a4\ud134 \uae00\ub85c\ube0c\u300b\uc758 \uc0ac\ub77c \ub85c\ub9cc\uc740 \uc9c0\ub09c 10\ub144\uac04 \uac00\uc7a5 \ub9e4\ub825\uc801\uc778 \ub178\ub798 4\uc704\uc5d0 \uc120\uc815\ud558\uba70 \"\ube44\uc698\uc138\ub294 \ub9ac\uc5b4\ud0c0\ub4dc\uc640 \ud568\uaed8 \uac04\ub2e8\ud558\uac8c \uafb8\ubc08\uc73c\ub85c\uc368 \ud558\ub098\uc758 \uc139\uc2dc\ud558\uba74\uc11c\ub3c4 \uad6c\uc2dd \ub290\ub08c\uc774 \ub098\ub294 \ud328\ud0a4\uc9c0\ub97c \ub9cc\ub4e4\uc5b4\ub0c8\ub2e4. \ubba4\uc9c1\ube44\ub514\uc624\ub294 \uc804 \uc138\uacc4\uc5d0 \ucda4\uacfc \uc720\ud29c\ube0c\uc758 \ud328\ub7ec\ub514 \uc5f4\ud48d\uc744 \uc77c\uc73c\ucf30\ub2e4\"\uace0 \uc37c\ub2e4. VH1\uc758 2000\ub144\ub300 \uac00\uc7a5 \uc704\ub300\ud55c \ub178\ub798 100\uc704 \uc21c\uc704\uc5d0\ub3c4 16\uc704\uc5d0 \uc62c\ub790\ub2e4. \ucc99 \ud074\ub85c\uc2a4\ud130\ub9e8\uc758 2009\ub144 \ucc45 Eating the Dinosaur\uc5d0\uc11c \"\uc8fc\uc7a5\ud558\uac74\ub300 \ub3c4\uc2dc\uc758 \ucc98\ub140 \ud30c\ud2f0\ub97c \ub178\ub9b0 \uba85\ubc31\ud55c \uccab \ub178\ub798\uc774\ub2e4\"\ub77c\uba70 \u3008Single Ladies\u3009\uc5d0 \ub300\ud574 \uc37c\ub2e4."} {"question": "\ud130\ucee4\uc758 \uc785\ub300 \uacc4\ud68d\uacfc\ub294 \ub2e4\ub974\uac8c \uc2dc\uc98c \ud480\ud0c0\uc784\uc744 \uc18c\ud654\ud55c \uadf8\ub294 5\uc6d4 16\uc77c\uae4c\uc9c0 403\uc758 \ud0c0\uc728\ub85c \uc544\uba54\ub9ac\uce78 \ub9ac\uadf8 \uba87 \uc704\ub97c \ub2ec\ub838\ub098?", "paragraph": "1944\ub144 \uc2dc\uc98c \uc804 \ud574\uad70 \uc785\ub300\ub97c \uc704\ud574\uc11c \ubc1b\uc740 \uc2e0\uccb4 \uac80\uc0ac\ub97c \ud1b5\uacfc\ud588\ub2e4. \uadf8\ud574 \ubc14\ub85c \ub098\ub77c\uc758 \ubd80\ub984\uc744 \ubc1b\uace0 \uc785\ub300\ud560 \uc608\uc815\uc774\uc5c8\uc9c0\ub9cc, \uacc4\ud68d\uacfc\ub294 \ub2e4\ub974\uac8c \ud480\ud0c0\uc784\uc744 \uc18c\ud654\ud588\ub2e4. \uc2dc\uc98c \uccab \ud55c \ub2ec\uc740 \ucf8c\uc870\uc758 \ud0c0\uaca9\uc744 \uc120\ubcf4\uc774\uba70, 5\uc6d4 16\uc77c\uae4c\uc9c0 .403\uc758 \ud0c0\uc728\ub85c \uc544\uba54\ub9ac\uce78 \ub9ac\uadf8 1\uc704\ub97c \ub2ec\ub838\ub2e4. \ub2e4\ubc29\uba74\uc758 \ud65c\uc57d\uc5d0 \uc5ec\ub7ec \uc57c\uad6c \uad00\uacc4\uc790\ub4e4\ub3c4 \uadf8\uc5d0\uac8c \uc88b\uc740 \ud3c9\uac00\ub97c \uc544\ub07c\uc9c0 \uc54a\uc558\ub2e4. \uc2a4\ud3ec\uce20\uae30\uc790 \ud504\ub808\ub4dc \ub9ac\uc5e1\uc740 \uc62c\ud574 \uc7a0\uc7ac\ub825\uc744 \ud130\ub728\ub9b0 \uc120\uc218\ub85c \ud130\ucee4\ub97c \uaf3d\uc558\uace0, \uc9c0\ubbf8 \ub2e4\uc774\ud06c\uc2a4 \uac10\ub3c5\uc740 \uadf8\ub97c \ub450\uace0 \uc544\uba54\ub9ac\uce78 \ub9ac\uadf8\uc5d0\uc11c \ucd5c\uace0\uc758 \uc678\uc57c\uc218 \uc911 \ud558\ub098\ub85c \uc774\uc57c\uae30\ud588\ub2e4. 6\uc6d4\uae4c\uc9c0 \uc544\uba54\ub9ac\uce78 \ub9ac\uadf8\uc5d0\uc11c\ub294 \ud130\ucee4\uac00 \ud0c0\uc728 .369\ub85c, \ub0b4\uc154\ub110 \ub9ac\uadf8\uc5d0\uc11c\ub294 \ub515\uc2dc \uc6cc\ucee4\uac00 \ud0c0\uc728 .377\ub85c \uac01 \ub9ac\uadf8\uc5d0\uc11c \ud0c0\uaca9\uc655 \uacbd\uc7c1\uc744 \uc774\ub04c\uc5c8\ub2e4. \uc774\ub7ec\ud55c \ud65c\uc57d\uc73c\ub85c 1944\ub144 \uc62c\uc2a4\ud0c0 \uc120\uc218 \uba85\ub2e8\uc5d0 \uacbd\ub825 \ucd5c\ucd08\uc774\uc790 \ub9c8\uc9c0\ub9c9\uc73c\ub85c \uc774\ub984\uc744 \uc62c\ub838\uc73c\uba70, \uc62c\uc2a4\ud0c0\uc804\uc5d0\uc11c \ub9ac\ub4dc \uc624\ud504\ub85c \ub098\uc11c 4\ud0c0\uc218 \ubb34\uc548\ud0c0\ub97c \uae30\ub85d\ud588\ub2e4.", "answer": "1\uc704", "sentence": "\ub9ac\uadf8 1\uc704 \ub97c \ub2ec\ub838\ub2e4.", "paragraph_sentence": "1944\ub144 \uc2dc\uc98c \uc804 \ud574\uad70 \uc785\ub300\ub97c \uc704\ud574\uc11c \ubc1b\uc740 \uc2e0\uccb4 \uac80\uc0ac\ub97c \ud1b5\uacfc\ud588\ub2e4. \uadf8\ud574 \ubc14\ub85c \ub098\ub77c\uc758 \ubd80\ub984\uc744 \ubc1b\uace0 \uc785\ub300\ud560 \uc608\uc815\uc774\uc5c8\uc9c0\ub9cc, \uacc4\ud68d\uacfc\ub294 \ub2e4\ub974\uac8c \ud480\ud0c0\uc784\uc744 \uc18c\ud654\ud588\ub2e4. \uc2dc\uc98c \uccab \ud55c \ub2ec\uc740 \ucf8c\uc870\uc758 \ud0c0\uaca9\uc744 \uc120\ubcf4\uc774\uba70, 5\uc6d4 16\uc77c\uae4c\uc9c0 .403\uc758 \ud0c0\uc728\ub85c \uc544\uba54\ub9ac\uce78 \ub9ac\uadf8 1\uc704 \ub97c \ub2ec\ub838\ub2e4. \ub2e4\ubc29\uba74\uc758 \ud65c\uc57d\uc5d0 \uc5ec\ub7ec \uc57c\uad6c \uad00\uacc4\uc790\ub4e4\ub3c4 \uadf8\uc5d0\uac8c \uc88b\uc740 \ud3c9\uac00\ub97c \uc544\ub07c\uc9c0 \uc54a\uc558\ub2e4. \uc2a4\ud3ec\uce20\uae30\uc790 \ud504\ub808\ub4dc \ub9ac\uc5e1\uc740 \uc62c\ud574 \uc7a0\uc7ac\ub825\uc744 \ud130\ub728\ub9b0 \uc120\uc218\ub85c \ud130\ucee4\ub97c \uaf3d\uc558\uace0, \uc9c0\ubbf8 \ub2e4\uc774\ud06c\uc2a4 \uac10\ub3c5\uc740 \uadf8\ub97c \ub450\uace0 \uc544\uba54\ub9ac\uce78 \ub9ac\uadf8\uc5d0\uc11c \ucd5c\uace0\uc758 \uc678\uc57c\uc218 \uc911 \ud558\ub098\ub85c \uc774\uc57c\uae30\ud588\ub2e4. 6\uc6d4\uae4c\uc9c0 \uc544\uba54\ub9ac\uce78 \ub9ac\uadf8\uc5d0\uc11c\ub294 \ud130\ucee4\uac00 \ud0c0\uc728 .369\ub85c, \ub0b4\uc154\ub110 \ub9ac\uadf8\uc5d0\uc11c\ub294 \ub515\uc2dc \uc6cc\ucee4\uac00 \ud0c0\uc728 .377\ub85c \uac01 \ub9ac\uadf8\uc5d0\uc11c \ud0c0\uaca9\uc655 \uacbd\uc7c1\uc744 \uc774\ub04c\uc5c8\ub2e4. \uc774\ub7ec\ud55c \ud65c\uc57d\uc73c\ub85c 1944\ub144 \uc62c\uc2a4\ud0c0 \uc120\uc218 \uba85\ub2e8\uc5d0 \uacbd\ub825 \ucd5c\ucd08\uc774\uc790 \ub9c8\uc9c0\ub9c9\uc73c\ub85c \uc774\ub984\uc744 \uc62c\ub838\uc73c\uba70, \uc62c\uc2a4\ud0c0\uc804\uc5d0\uc11c \ub9ac\ub4dc \uc624\ud504\ub85c \ub098\uc11c 4\ud0c0\uc218 \ubb34\uc548\ud0c0\ub97c \uae30\ub85d\ud588\ub2e4.", "paragraph_answer": "1944\ub144 \uc2dc\uc98c \uc804 \ud574\uad70 \uc785\ub300\ub97c \uc704\ud574\uc11c \ubc1b\uc740 \uc2e0\uccb4 \uac80\uc0ac\ub97c \ud1b5\uacfc\ud588\ub2e4. \uadf8\ud574 \ubc14\ub85c \ub098\ub77c\uc758 \ubd80\ub984\uc744 \ubc1b\uace0 \uc785\ub300\ud560 \uc608\uc815\uc774\uc5c8\uc9c0\ub9cc, \uacc4\ud68d\uacfc\ub294 \ub2e4\ub974\uac8c \ud480\ud0c0\uc784\uc744 \uc18c\ud654\ud588\ub2e4. \uc2dc\uc98c \uccab \ud55c \ub2ec\uc740 \ucf8c\uc870\uc758 \ud0c0\uaca9\uc744 \uc120\ubcf4\uc774\uba70, 5\uc6d4 16\uc77c\uae4c\uc9c0 .403\uc758 \ud0c0\uc728\ub85c \uc544\uba54\ub9ac\uce78 \ub9ac\uadf8 1\uc704 \ub97c \ub2ec\ub838\ub2e4. \ub2e4\ubc29\uba74\uc758 \ud65c\uc57d\uc5d0 \uc5ec\ub7ec \uc57c\uad6c \uad00\uacc4\uc790\ub4e4\ub3c4 \uadf8\uc5d0\uac8c \uc88b\uc740 \ud3c9\uac00\ub97c \uc544\ub07c\uc9c0 \uc54a\uc558\ub2e4. \uc2a4\ud3ec\uce20\uae30\uc790 \ud504\ub808\ub4dc \ub9ac\uc5e1\uc740 \uc62c\ud574 \uc7a0\uc7ac\ub825\uc744 \ud130\ub728\ub9b0 \uc120\uc218\ub85c \ud130\ucee4\ub97c \uaf3d\uc558\uace0, \uc9c0\ubbf8 \ub2e4\uc774\ud06c\uc2a4 \uac10\ub3c5\uc740 \uadf8\ub97c \ub450\uace0 \uc544\uba54\ub9ac\uce78 \ub9ac\uadf8\uc5d0\uc11c \ucd5c\uace0\uc758 \uc678\uc57c\uc218 \uc911 \ud558\ub098\ub85c \uc774\uc57c\uae30\ud588\ub2e4. 6\uc6d4\uae4c\uc9c0 \uc544\uba54\ub9ac\uce78 \ub9ac\uadf8\uc5d0\uc11c\ub294 \ud130\ucee4\uac00 \ud0c0\uc728 .369\ub85c, \ub0b4\uc154\ub110 \ub9ac\uadf8\uc5d0\uc11c\ub294 \ub515\uc2dc \uc6cc\ucee4\uac00 \ud0c0\uc728 .377\ub85c \uac01 \ub9ac\uadf8\uc5d0\uc11c \ud0c0\uaca9\uc655 \uacbd\uc7c1\uc744 \uc774\ub04c\uc5c8\ub2e4. \uc774\ub7ec\ud55c \ud65c\uc57d\uc73c\ub85c 1944\ub144 \uc62c\uc2a4\ud0c0 \uc120\uc218 \uba85\ub2e8\uc5d0 \uacbd\ub825 \ucd5c\ucd08\uc774\uc790 \ub9c8\uc9c0\ub9c9\uc73c\ub85c \uc774\ub984\uc744 \uc62c\ub838\uc73c\uba70, \uc62c\uc2a4\ud0c0\uc804\uc5d0\uc11c \ub9ac\ub4dc \uc624\ud504\ub85c \ub098\uc11c 4\ud0c0\uc218 \ubb34\uc548\ud0c0\ub97c \uae30\ub85d\ud588\ub2e4.", "sentence_answer": "\ub9ac\uadf8 1\uc704 \ub97c \ub2ec\ub838\ub2e4.", "paragraph_id": "6511496-3-1", "paragraph_question": "question: \ud130\ucee4\uc758 \uc785\ub300 \uacc4\ud68d\uacfc\ub294 \ub2e4\ub974\uac8c \uc2dc\uc98c \ud480\ud0c0\uc784\uc744 \uc18c\ud654\ud55c \uadf8\ub294 5\uc6d4 16\uc77c\uae4c\uc9c0 403\uc758 \ud0c0\uc728\ub85c \uc544\uba54\ub9ac\uce78 \ub9ac\uadf8 \uba87 \uc704\ub97c \ub2ec\ub838\ub098?, context: 1944\ub144 \uc2dc\uc98c \uc804 \ud574\uad70 \uc785\ub300\ub97c \uc704\ud574\uc11c \ubc1b\uc740 \uc2e0\uccb4 \uac80\uc0ac\ub97c \ud1b5\uacfc\ud588\ub2e4. \uadf8\ud574 \ubc14\ub85c \ub098\ub77c\uc758 \ubd80\ub984\uc744 \ubc1b\uace0 \uc785\ub300\ud560 \uc608\uc815\uc774\uc5c8\uc9c0\ub9cc, \uacc4\ud68d\uacfc\ub294 \ub2e4\ub974\uac8c \ud480\ud0c0\uc784\uc744 \uc18c\ud654\ud588\ub2e4. \uc2dc\uc98c \uccab \ud55c \ub2ec\uc740 \ucf8c\uc870\uc758 \ud0c0\uaca9\uc744 \uc120\ubcf4\uc774\uba70, 5\uc6d4 16\uc77c\uae4c\uc9c0 .403\uc758 \ud0c0\uc728\ub85c \uc544\uba54\ub9ac\uce78 \ub9ac\uadf8 1\uc704\ub97c \ub2ec\ub838\ub2e4. \ub2e4\ubc29\uba74\uc758 \ud65c\uc57d\uc5d0 \uc5ec\ub7ec \uc57c\uad6c \uad00\uacc4\uc790\ub4e4\ub3c4 \uadf8\uc5d0\uac8c \uc88b\uc740 \ud3c9\uac00\ub97c \uc544\ub07c\uc9c0 \uc54a\uc558\ub2e4. \uc2a4\ud3ec\uce20\uae30\uc790 \ud504\ub808\ub4dc \ub9ac\uc5e1\uc740 \uc62c\ud574 \uc7a0\uc7ac\ub825\uc744 \ud130\ub728\ub9b0 \uc120\uc218\ub85c \ud130\ucee4\ub97c \uaf3d\uc558\uace0, \uc9c0\ubbf8 \ub2e4\uc774\ud06c\uc2a4 \uac10\ub3c5\uc740 \uadf8\ub97c \ub450\uace0 \uc544\uba54\ub9ac\uce78 \ub9ac\uadf8\uc5d0\uc11c \ucd5c\uace0\uc758 \uc678\uc57c\uc218 \uc911 \ud558\ub098\ub85c \uc774\uc57c\uae30\ud588\ub2e4. 6\uc6d4\uae4c\uc9c0 \uc544\uba54\ub9ac\uce78 \ub9ac\uadf8\uc5d0\uc11c\ub294 \ud130\ucee4\uac00 \ud0c0\uc728 .369\ub85c, \ub0b4\uc154\ub110 \ub9ac\uadf8\uc5d0\uc11c\ub294 \ub515\uc2dc \uc6cc\ucee4\uac00 \ud0c0\uc728 .377\ub85c \uac01 \ub9ac\uadf8\uc5d0\uc11c \ud0c0\uaca9\uc655 \uacbd\uc7c1\uc744 \uc774\ub04c\uc5c8\ub2e4. \uc774\ub7ec\ud55c \ud65c\uc57d\uc73c\ub85c 1944\ub144 \uc62c\uc2a4\ud0c0 \uc120\uc218 \uba85\ub2e8\uc5d0 \uacbd\ub825 \ucd5c\ucd08\uc774\uc790 \ub9c8\uc9c0\ub9c9\uc73c\ub85c \uc774\ub984\uc744 \uc62c\ub838\uc73c\uba70, \uc62c\uc2a4\ud0c0\uc804\uc5d0\uc11c \ub9ac\ub4dc \uc624\ud504\ub85c \ub098\uc11c 4\ud0c0\uc218 \ubb34\uc548\ud0c0\ub97c \uae30\ub85d\ud588\ub2e4."} {"question": "\uba85\ub098\ub77c \ub54c\uc758 \ub300\uc2e0\ub4e4\uc744 \uc9c0\uce6d\ud558\ub294 \ub2e8\uc5b4\ub294 \ubb34\uc5c7\uc778\uac00?", "paragraph": "\uadf8\uc758 \uc815\ucc45\uc73c\ub85c \uad6d\uac00\ub294 \ub354\uc6b1 \uc548\uc815\uc774 \ub418\uc5c8\uace0 \ub0b4\uc2e4 \uc5ed\uc2dc \ud2bc\ud2bc\ud574\uc84c\ub2e4. \ub610\ud55c \uc7ac\uc815 \uac1c\ud601\uc744 \ud1b5\ud558\uc5ec \uae30\uac15 \ub2e8\uc18d\uacfc \uc7ac\uc815 \uc815\ube44\ub97c \uc77c\uac70\uc5d0 \uc2e4\ud589\ud558\ub294 \uc815\ucc45\uc744 \ucd94\uc9c4\ud568\uacfc \ub354\ubd88\uc5b4 \uc870\uc138 \uc81c\ub3c4 \uc790\uccb4\uc5d0\ub3c4 \uac1c\ud601\uc744 \ub2e8\ud589\ud588\ub2e4. \ub610\ud55c \uad70\uae30\ucc98\ub97c \uc138\uc6b0\uace0 \ud669\uad8c\uc744 \ub354\uc6b1 \uac15\ud654, \uc7ac\uc0c1\ub4e4\uc758 \uc815\uce58 \ubc1c\uc5b8\uad8c\uc744 \uaddc\uc81c\ud558\uc600\uace0 \uc7ac\uc0c1\ub4e4\uc758 \uc815\uce58 \ucc38\uc5ec\ub97c \uaddc\uc81c\ud55c \ub300\uc2e0 \uc790\uc2e0\uc5d0\uac90 \uc7ac\uc0c1\ub4e4\uc774 \ubcf8\ub798 \uacb0\uc7ac\ud558\ub358 \ubb38\uc11c\uc758 \uc591\uae4c\uc9c0 \ud569\ud558\uc5ec \uc5b4\ub9c8\uc5b4\ub9c8\ud558\uac8c \ub9ce\uc740 \uc591\uc758 \ubb38\uc11c\ub97c \uac80\ud1a0, \uc774\uc5d0 \uc77c\uc77c\uc774 \ub2f5\ud558\uc600\uace0 \ud558\uae09\uc758 \uc9c0\ubc29\uad00\uc774\ub77c\ub3c4 \uc790\uc2e0\uc5d0\uac8c \uc0c1\uc18c\ub97c \uc62c\ub9ac\uba74 \uc774 \uc5ed\uc2dc \ubc1b\uc544\uc8fc\uc5b4 \uc8fc\ud544\ub85c \uc368\uc11c \ubcf4\ub0b4\uc8fc\uc5b4\uc11c \uba85\uad70\uc758 \ubaa8\uc2b5\uc744 \ubcf4\uc5ec\uc8fc\uc5c8\ub2e4. \uc7ac\uc0c1\ub4e4\uacfc \ub300\uc2e0\ub4e4\uc744 \uad70\uae30\ub300\uc2e0\uc73c\ub85c \uc0bc\uace0 \uac19\uc774 \uc815\uc0ac\ub97c \uc758\ub17c\ud558\uc600\uc73c\ub098 \uc774\ubbf8 \uadf8\ub4e4\uc758 \uad8c\uc138\ub294 \uba85\ub098\ub77c \ub54c\uc758 \ub300\uc2e0\ub4e4\uc778 \ub0b4\uac01\ub300\ud559\uc0ac(\u5167\u95a3\u5927\u5b78\u58eb)\uc5d0 \ube44\ud558\uc5ec \ud06c\uac8c \ucd95\uc18c\ub418\uc5b4 \uc788\uc5c8\uae30\uc5d0 \uc2e0\uad8c\uc740 \ud06c\uac8c \uc704\ucd95\ub418\uc5c8\ub2e4. \uac15\ud76c\uc81c \ub54c \uc77c\uc5b4\ub09c \ubb38\uc790\uc758 \uc625\uc744 \ub2e4\uc2dc \ub300\ub300\uc801\uc73c\ub85c \uc2e4\uc2dc\ud558\uc5ec \uccad\ub098\ub77c\uc758 \uc815\ud1b5\uc131\uc5d0 \ubc18\ub300\ud558\ub294 \ud559\uc790\ub4e4\uacfc \ud55c\uc871\uc744 \uc5c4\uc815\ud788 \ub2e4\uc2a4\ub838\uc73c\uba70 \uc9c0\ubc29\uad00\ub4e4\uc5d0 \ub300\ud55c \ucca0\uc800\ud55c \uac10\uc2dc\ub85c \ubd80\uc815\ubd80\ud328\ub97c \ud06c\uac8c \uc904\uc600\ub2e4. \uac15\ud76c\uc81c\uac00 \uc8fd\uc740 \uc9c1\ud6c4 \uad70\uc0ac\ub4e4\uc744 \ub3d9\uc6d0\ud558\uc5ec \ud669\uc81c\uc5d0 \uc62c\ub790\uae30 \ub54c\ubb38\uc5d0 \uce5c\uc704 \ucfe0\ub370\ud0c0\ub97c \uc77c\uc73c\ucf1c \ud669\uc704\ub97c \ucc2c\ud0c8\ud55c \uad8c\ub825 \uc9c0\ud5a5\uc801 \ub3c5\uc7ac\uc790\ub77c\ub294 \ud3c9\ud310\ub3c4 \uc788\uc9c0\ub9cc, \uc9c0\uce60 \uc904 \ubaa8\ub974\uace0 \ub2a6\uc740 \ubc24\uae4c\uc9c0 \uc815\uce58\uc5d0 \ubab0\ub450\ud55c \ud6cc\ub96d\ud55c \uad70\uc8fc\ub77c\ub294 \ud3c9\uac00\ub3c4 \ub3d9\uc2dc\uc5d0 \ubc1b\uace0 \uc788\ub2e4..", "answer": "\ub0b4\uac01\ub300\ud559\uc0ac", "sentence": "\uc7ac\uc0c1\ub4e4\uacfc \ub300\uc2e0\ub4e4\uc744 \uad70\uae30\ub300\uc2e0\uc73c\ub85c \uc0bc\uace0 \uac19\uc774 \uc815\uc0ac\ub97c \uc758\ub17c\ud558\uc600\uc73c\ub098 \uc774\ubbf8 \uadf8\ub4e4\uc758 \uad8c\uc138\ub294 \uba85\ub098\ub77c \ub54c\uc758 \ub300\uc2e0\ub4e4\uc778 \ub0b4\uac01\ub300\ud559\uc0ac (\u5167\u95a3\u5927\u5b78\u58eb)", "paragraph_sentence": "\uadf8\uc758 \uc815\ucc45\uc73c\ub85c \uad6d\uac00\ub294 \ub354\uc6b1 \uc548\uc815\uc774 \ub418\uc5c8\uace0 \ub0b4\uc2e4 \uc5ed\uc2dc \ud2bc\ud2bc\ud574\uc84c\ub2e4. \ub610\ud55c \uc7ac\uc815 \uac1c\ud601\uc744 \ud1b5\ud558\uc5ec \uae30\uac15 \ub2e8\uc18d\uacfc \uc7ac\uc815 \uc815\ube44\ub97c \uc77c\uac70\uc5d0 \uc2e4\ud589\ud558\ub294 \uc815\ucc45\uc744 \ucd94\uc9c4\ud568\uacfc \ub354\ubd88\uc5b4 \uc870\uc138 \uc81c\ub3c4 \uc790\uccb4\uc5d0\ub3c4 \uac1c\ud601\uc744 \ub2e8\ud589\ud588\ub2e4. \ub610\ud55c \uad70\uae30\ucc98\ub97c \uc138\uc6b0\uace0 \ud669\uad8c\uc744 \ub354\uc6b1 \uac15\ud654, \uc7ac\uc0c1\ub4e4\uc758 \uc815\uce58 \ubc1c\uc5b8\uad8c\uc744 \uaddc\uc81c\ud558\uc600\uace0 \uc7ac\uc0c1\ub4e4\uc758 \uc815\uce58 \ucc38\uc5ec\ub97c \uaddc\uc81c\ud55c \ub300\uc2e0 \uc790\uc2e0\uc5d0\uac90 \uc7ac\uc0c1\ub4e4\uc774 \ubcf8\ub798 \uacb0\uc7ac\ud558\ub358 \ubb38\uc11c\uc758 \uc591\uae4c\uc9c0 \ud569\ud558\uc5ec \uc5b4\ub9c8\uc5b4\ub9c8\ud558\uac8c \ub9ce\uc740 \uc591\uc758 \ubb38\uc11c\ub97c \uac80\ud1a0, \uc774\uc5d0 \uc77c\uc77c\uc774 \ub2f5\ud558\uc600\uace0 \ud558\uae09\uc758 \uc9c0\ubc29\uad00\uc774\ub77c\ub3c4 \uc790\uc2e0\uc5d0\uac8c \uc0c1\uc18c\ub97c \uc62c\ub9ac\uba74 \uc774 \uc5ed\uc2dc \ubc1b\uc544\uc8fc\uc5b4 \uc8fc\ud544\ub85c \uc368\uc11c \ubcf4\ub0b4\uc8fc\uc5b4\uc11c \uba85\uad70\uc758 \ubaa8\uc2b5\uc744 \ubcf4\uc5ec\uc8fc\uc5c8\ub2e4. \uc7ac\uc0c1\ub4e4\uacfc \ub300\uc2e0\ub4e4\uc744 \uad70\uae30\ub300\uc2e0\uc73c\ub85c \uc0bc\uace0 \uac19\uc774 \uc815\uc0ac\ub97c \uc758\ub17c\ud558\uc600\uc73c\ub098 \uc774\ubbf8 \uadf8\ub4e4\uc758 \uad8c\uc138\ub294 \uba85\ub098\ub77c \ub54c\uc758 \ub300\uc2e0\ub4e4\uc778 \ub0b4\uac01\ub300\ud559\uc0ac (\u5167\u95a3\u5927\u5b78\u58eb) \uc5d0 \ube44\ud558\uc5ec \ud06c\uac8c \ucd95\uc18c\ub418\uc5b4 \uc788\uc5c8\uae30\uc5d0 \uc2e0\uad8c\uc740 \ud06c\uac8c \uc704\ucd95\ub418\uc5c8\ub2e4. \uac15\ud76c\uc81c \ub54c \uc77c\uc5b4\ub09c \ubb38\uc790\uc758 \uc625\uc744 \ub2e4\uc2dc \ub300\ub300\uc801\uc73c\ub85c \uc2e4\uc2dc\ud558\uc5ec \uccad\ub098\ub77c\uc758 \uc815\ud1b5\uc131\uc5d0 \ubc18\ub300\ud558\ub294 \ud559\uc790\ub4e4\uacfc \ud55c\uc871\uc744 \uc5c4\uc815\ud788 \ub2e4\uc2a4\ub838\uc73c\uba70 \uc9c0\ubc29\uad00\ub4e4\uc5d0 \ub300\ud55c \ucca0\uc800\ud55c \uac10\uc2dc\ub85c \ubd80\uc815\ubd80\ud328\ub97c \ud06c\uac8c \uc904\uc600\ub2e4. \uac15\ud76c\uc81c\uac00 \uc8fd\uc740 \uc9c1\ud6c4 \uad70\uc0ac\ub4e4\uc744 \ub3d9\uc6d0\ud558\uc5ec \ud669\uc81c\uc5d0 \uc62c\ub790\uae30 \ub54c\ubb38\uc5d0 \uce5c\uc704 \ucfe0\ub370\ud0c0\ub97c \uc77c\uc73c\ucf1c \ud669\uc704\ub97c \ucc2c\ud0c8\ud55c \uad8c\ub825 \uc9c0\ud5a5\uc801 \ub3c5\uc7ac\uc790\ub77c\ub294 \ud3c9\ud310\ub3c4 \uc788\uc9c0\ub9cc, \uc9c0\uce60 \uc904 \ubaa8\ub974\uace0 \ub2a6\uc740 \ubc24\uae4c\uc9c0 \uc815\uce58\uc5d0 \ubab0\ub450\ud55c \ud6cc\ub96d\ud55c \uad70\uc8fc\ub77c\ub294 \ud3c9\uac00\ub3c4 \ub3d9\uc2dc\uc5d0 \ubc1b\uace0 \uc788\ub2e4 ..", "paragraph_answer": "\uadf8\uc758 \uc815\ucc45\uc73c\ub85c \uad6d\uac00\ub294 \ub354\uc6b1 \uc548\uc815\uc774 \ub418\uc5c8\uace0 \ub0b4\uc2e4 \uc5ed\uc2dc \ud2bc\ud2bc\ud574\uc84c\ub2e4. \ub610\ud55c \uc7ac\uc815 \uac1c\ud601\uc744 \ud1b5\ud558\uc5ec \uae30\uac15 \ub2e8\uc18d\uacfc \uc7ac\uc815 \uc815\ube44\ub97c \uc77c\uac70\uc5d0 \uc2e4\ud589\ud558\ub294 \uc815\ucc45\uc744 \ucd94\uc9c4\ud568\uacfc \ub354\ubd88\uc5b4 \uc870\uc138 \uc81c\ub3c4 \uc790\uccb4\uc5d0\ub3c4 \uac1c\ud601\uc744 \ub2e8\ud589\ud588\ub2e4. \ub610\ud55c \uad70\uae30\ucc98\ub97c \uc138\uc6b0\uace0 \ud669\uad8c\uc744 \ub354\uc6b1 \uac15\ud654, \uc7ac\uc0c1\ub4e4\uc758 \uc815\uce58 \ubc1c\uc5b8\uad8c\uc744 \uaddc\uc81c\ud558\uc600\uace0 \uc7ac\uc0c1\ub4e4\uc758 \uc815\uce58 \ucc38\uc5ec\ub97c \uaddc\uc81c\ud55c \ub300\uc2e0 \uc790\uc2e0\uc5d0\uac90 \uc7ac\uc0c1\ub4e4\uc774 \ubcf8\ub798 \uacb0\uc7ac\ud558\ub358 \ubb38\uc11c\uc758 \uc591\uae4c\uc9c0 \ud569\ud558\uc5ec \uc5b4\ub9c8\uc5b4\ub9c8\ud558\uac8c \ub9ce\uc740 \uc591\uc758 \ubb38\uc11c\ub97c \uac80\ud1a0, \uc774\uc5d0 \uc77c\uc77c\uc774 \ub2f5\ud558\uc600\uace0 \ud558\uae09\uc758 \uc9c0\ubc29\uad00\uc774\ub77c\ub3c4 \uc790\uc2e0\uc5d0\uac8c \uc0c1\uc18c\ub97c \uc62c\ub9ac\uba74 \uc774 \uc5ed\uc2dc \ubc1b\uc544\uc8fc\uc5b4 \uc8fc\ud544\ub85c \uc368\uc11c \ubcf4\ub0b4\uc8fc\uc5b4\uc11c \uba85\uad70\uc758 \ubaa8\uc2b5\uc744 \ubcf4\uc5ec\uc8fc\uc5c8\ub2e4. \uc7ac\uc0c1\ub4e4\uacfc \ub300\uc2e0\ub4e4\uc744 \uad70\uae30\ub300\uc2e0\uc73c\ub85c \uc0bc\uace0 \uac19\uc774 \uc815\uc0ac\ub97c \uc758\ub17c\ud558\uc600\uc73c\ub098 \uc774\ubbf8 \uadf8\ub4e4\uc758 \uad8c\uc138\ub294 \uba85\ub098\ub77c \ub54c\uc758 \ub300\uc2e0\ub4e4\uc778 \ub0b4\uac01\ub300\ud559\uc0ac (\u5167\u95a3\u5927\u5b78\u58eb)\uc5d0 \ube44\ud558\uc5ec \ud06c\uac8c \ucd95\uc18c\ub418\uc5b4 \uc788\uc5c8\uae30\uc5d0 \uc2e0\uad8c\uc740 \ud06c\uac8c \uc704\ucd95\ub418\uc5c8\ub2e4. \uac15\ud76c\uc81c \ub54c \uc77c\uc5b4\ub09c \ubb38\uc790\uc758 \uc625\uc744 \ub2e4\uc2dc \ub300\ub300\uc801\uc73c\ub85c \uc2e4\uc2dc\ud558\uc5ec \uccad\ub098\ub77c\uc758 \uc815\ud1b5\uc131\uc5d0 \ubc18\ub300\ud558\ub294 \ud559\uc790\ub4e4\uacfc \ud55c\uc871\uc744 \uc5c4\uc815\ud788 \ub2e4\uc2a4\ub838\uc73c\uba70 \uc9c0\ubc29\uad00\ub4e4\uc5d0 \ub300\ud55c \ucca0\uc800\ud55c \uac10\uc2dc\ub85c \ubd80\uc815\ubd80\ud328\ub97c \ud06c\uac8c \uc904\uc600\ub2e4. \uac15\ud76c\uc81c\uac00 \uc8fd\uc740 \uc9c1\ud6c4 \uad70\uc0ac\ub4e4\uc744 \ub3d9\uc6d0\ud558\uc5ec \ud669\uc81c\uc5d0 \uc62c\ub790\uae30 \ub54c\ubb38\uc5d0 \uce5c\uc704 \ucfe0\ub370\ud0c0\ub97c \uc77c\uc73c\ucf1c \ud669\uc704\ub97c \ucc2c\ud0c8\ud55c \uad8c\ub825 \uc9c0\ud5a5\uc801 \ub3c5\uc7ac\uc790\ub77c\ub294 \ud3c9\ud310\ub3c4 \uc788\uc9c0\ub9cc, \uc9c0\uce60 \uc904 \ubaa8\ub974\uace0 \ub2a6\uc740 \ubc24\uae4c\uc9c0 \uc815\uce58\uc5d0 \ubab0\ub450\ud55c \ud6cc\ub96d\ud55c \uad70\uc8fc\ub77c\ub294 \ud3c9\uac00\ub3c4 \ub3d9\uc2dc\uc5d0 \ubc1b\uace0 \uc788\ub2e4..", "sentence_answer": "\uc7ac\uc0c1\ub4e4\uacfc \ub300\uc2e0\ub4e4\uc744 \uad70\uae30\ub300\uc2e0\uc73c\ub85c \uc0bc\uace0 \uac19\uc774 \uc815\uc0ac\ub97c \uc758\ub17c\ud558\uc600\uc73c\ub098 \uc774\ubbf8 \uadf8\ub4e4\uc758 \uad8c\uc138\ub294 \uba85\ub098\ub77c \ub54c\uc758 \ub300\uc2e0\ub4e4\uc778 \ub0b4\uac01\ub300\ud559\uc0ac (\u5167\u95a3\u5927\u5b78\u58eb)", "paragraph_id": "6541455-2-1", "paragraph_question": "question: \uba85\ub098\ub77c \ub54c\uc758 \ub300\uc2e0\ub4e4\uc744 \uc9c0\uce6d\ud558\ub294 \ub2e8\uc5b4\ub294 \ubb34\uc5c7\uc778\uac00?, context: \uadf8\uc758 \uc815\ucc45\uc73c\ub85c \uad6d\uac00\ub294 \ub354\uc6b1 \uc548\uc815\uc774 \ub418\uc5c8\uace0 \ub0b4\uc2e4 \uc5ed\uc2dc \ud2bc\ud2bc\ud574\uc84c\ub2e4. \ub610\ud55c \uc7ac\uc815 \uac1c\ud601\uc744 \ud1b5\ud558\uc5ec \uae30\uac15 \ub2e8\uc18d\uacfc \uc7ac\uc815 \uc815\ube44\ub97c \uc77c\uac70\uc5d0 \uc2e4\ud589\ud558\ub294 \uc815\ucc45\uc744 \ucd94\uc9c4\ud568\uacfc \ub354\ubd88\uc5b4 \uc870\uc138 \uc81c\ub3c4 \uc790\uccb4\uc5d0\ub3c4 \uac1c\ud601\uc744 \ub2e8\ud589\ud588\ub2e4. \ub610\ud55c \uad70\uae30\ucc98\ub97c \uc138\uc6b0\uace0 \ud669\uad8c\uc744 \ub354\uc6b1 \uac15\ud654, \uc7ac\uc0c1\ub4e4\uc758 \uc815\uce58 \ubc1c\uc5b8\uad8c\uc744 \uaddc\uc81c\ud558\uc600\uace0 \uc7ac\uc0c1\ub4e4\uc758 \uc815\uce58 \ucc38\uc5ec\ub97c \uaddc\uc81c\ud55c \ub300\uc2e0 \uc790\uc2e0\uc5d0\uac90 \uc7ac\uc0c1\ub4e4\uc774 \ubcf8\ub798 \uacb0\uc7ac\ud558\ub358 \ubb38\uc11c\uc758 \uc591\uae4c\uc9c0 \ud569\ud558\uc5ec \uc5b4\ub9c8\uc5b4\ub9c8\ud558\uac8c \ub9ce\uc740 \uc591\uc758 \ubb38\uc11c\ub97c \uac80\ud1a0, \uc774\uc5d0 \uc77c\uc77c\uc774 \ub2f5\ud558\uc600\uace0 \ud558\uae09\uc758 \uc9c0\ubc29\uad00\uc774\ub77c\ub3c4 \uc790\uc2e0\uc5d0\uac8c \uc0c1\uc18c\ub97c \uc62c\ub9ac\uba74 \uc774 \uc5ed\uc2dc \ubc1b\uc544\uc8fc\uc5b4 \uc8fc\ud544\ub85c \uc368\uc11c \ubcf4\ub0b4\uc8fc\uc5b4\uc11c \uba85\uad70\uc758 \ubaa8\uc2b5\uc744 \ubcf4\uc5ec\uc8fc\uc5c8\ub2e4. \uc7ac\uc0c1\ub4e4\uacfc \ub300\uc2e0\ub4e4\uc744 \uad70\uae30\ub300\uc2e0\uc73c\ub85c \uc0bc\uace0 \uac19\uc774 \uc815\uc0ac\ub97c \uc758\ub17c\ud558\uc600\uc73c\ub098 \uc774\ubbf8 \uadf8\ub4e4\uc758 \uad8c\uc138\ub294 \uba85\ub098\ub77c \ub54c\uc758 \ub300\uc2e0\ub4e4\uc778 \ub0b4\uac01\ub300\ud559\uc0ac(\u5167\u95a3\u5927\u5b78\u58eb)\uc5d0 \ube44\ud558\uc5ec \ud06c\uac8c \ucd95\uc18c\ub418\uc5b4 \uc788\uc5c8\uae30\uc5d0 \uc2e0\uad8c\uc740 \ud06c\uac8c \uc704\ucd95\ub418\uc5c8\ub2e4. \uac15\ud76c\uc81c \ub54c \uc77c\uc5b4\ub09c \ubb38\uc790\uc758 \uc625\uc744 \ub2e4\uc2dc \ub300\ub300\uc801\uc73c\ub85c \uc2e4\uc2dc\ud558\uc5ec \uccad\ub098\ub77c\uc758 \uc815\ud1b5\uc131\uc5d0 \ubc18\ub300\ud558\ub294 \ud559\uc790\ub4e4\uacfc \ud55c\uc871\uc744 \uc5c4\uc815\ud788 \ub2e4\uc2a4\ub838\uc73c\uba70 \uc9c0\ubc29\uad00\ub4e4\uc5d0 \ub300\ud55c \ucca0\uc800\ud55c \uac10\uc2dc\ub85c \ubd80\uc815\ubd80\ud328\ub97c \ud06c\uac8c \uc904\uc600\ub2e4. \uac15\ud76c\uc81c\uac00 \uc8fd\uc740 \uc9c1\ud6c4 \uad70\uc0ac\ub4e4\uc744 \ub3d9\uc6d0\ud558\uc5ec \ud669\uc81c\uc5d0 \uc62c\ub790\uae30 \ub54c\ubb38\uc5d0 \uce5c\uc704 \ucfe0\ub370\ud0c0\ub97c \uc77c\uc73c\ucf1c \ud669\uc704\ub97c \ucc2c\ud0c8\ud55c \uad8c\ub825 \uc9c0\ud5a5\uc801 \ub3c5\uc7ac\uc790\ub77c\ub294 \ud3c9\ud310\ub3c4 \uc788\uc9c0\ub9cc, \uc9c0\uce60 \uc904 \ubaa8\ub974\uace0 \ub2a6\uc740 \ubc24\uae4c\uc9c0 \uc815\uce58\uc5d0 \ubab0\ub450\ud55c \ud6cc\ub96d\ud55c \uad70\uc8fc\ub77c\ub294 \ud3c9\uac00\ub3c4 \ub3d9\uc2dc\uc5d0 \ubc1b\uace0 \uc788\ub2e4.."} {"question": "\ud504\ub7ad\ud06c \ub85c\uc774\ub4dc \ub77c\uc774\ud2b8\ub294 \uc5b4\ub5a4 \uac74\ucd95 \uc124\uacc4 \uc6b4\ub3d9\uc744 \uc2dc\uc791\ud588\ub294\uac00?", "paragraph": "\ub3c4\uc2dc\ub4e4\uc774 \uc18c\uc704 \ud669\uae08\uc2dc\ub300(Golden Age)\ub85c \uc811\uc5b4\ub4e4\uba74\uc11c \ub9c8\ucc9c\ub8e8\uac00 \ub4f1\uc7a5\ud574 \ubbf8\uad6d \uc0ac\ud68c\uc758 \uae30\uc220\uc801 \uc5c5\uc801\uacfc \uc790\uc2e0\uac10\uc744 \uc0c1\uc9d5\uc801\uc73c\ub85c \ub098\ud0c0\ub0c8\ub2e4. \uc70c\ub9ac\uc5c4 \ub7ec \ubc30\ub7f0 \uc81c\ub2c8\uc640 \ub8e8\uc774\uc2a4 \uc124\ub9ac\ubc88\uc740 \uc2dc\uce74\uace0 \ucd9c\uc2e0\uc758 \uac74\ucd95\uac00\ub85c, \uadf8\ub7ec\ud55c \uac74\ucd95\uc591\uc2dd\uc758 \uc120\uad6c\uc790\ub4e4\uc774\ub2e4. \uc124\ub9ac\ubc88\uc758 \uc81c\uc790\uc778 \ud504\ub7ad\ud06c \ub85c\uc774\ub4dc \ub77c\uc774\ud2b8(1869~1959)\ub294 \ud504\ub808\ub9ac \ud559\ud30c(Prairie School)\ub85c \uc54c\ub824\uc9c4 \uc0c8\ub85c\uc6b4 \uac74\ucd95\uc124\uacc4\uc6b4\ub3d9\uc744 \uc2dc\uc791\ud574 20\uc138\uae30\ucd08\uc5d0 \ubbf8\uad6d \uc804\uc5ed\uc5d0 \uac78\uccd0 \ud3ed\ub113\uc740 \uc601\ud5a5\uc744 \ub07c\ucce4\ub2e4. \ud6c4\ub300\uc758 \uc8fc\uc694 \ubbf8\uad6d\uac74\ucd95\uac00\ub85c\ub294 \ub3c5\uc77c \ud0dc\uc0dd\uc758 \ub8e8\ud2b8\ube44\ud788 \ubbf8\uc2a4 \ud310 \ub370\uc5b4\ub974 \ub85c\uc5b4, \uadf8\uc758 \uc81c\uc790\ub4e4\uc778 \ud544\ub9bd \uc874\uc2a8, \ub85c\ubc84\ud2b8 \ubca4\ud22c\ub9ac \ub4f1\uc774 \uc788\ub2e4. 1825~1870\ub144\uc5d0 \uc720\ud589\ud588\uc73c\uba70 \ud669\uc57c \uadf8 \uc790\uccb4\ub97c \uc18c\uc7ac\ub85c \uc0bc\uc558\ub358 \ud5c8\ub4dc\uc2a8 \uac15\ud30c(Hudson River School)\ub294 \ubbf8\uad6d \ubbf8\uc220\uc5d0\uc11c \uc77c\uc5b4\ub09c \ucd5c\ucd08\uc758 \ud1a0\ucc29\ud654 \uc6b4\ub3d9\uc73c\ub85c \uc5ec\uaca8\uc9c4\ub2e4. \uc544\uc154 B. \ub4c0\ub79c\ub4dc\uac00 \uadf8 \ub300\ud45c\uc801\uc778 \ud654\uac00\ub2e4. \ub3c4\uc2dc\uc0dd\ud65c\uc740 \uc5d0\ub4dc\uc6cc\ub4dc \ud638\ud37c \uac19\uc740 \uc0ac\uc2e4\uc8fc\uc758\ud30c\ub098 \uc2a4\ud29c\uc5b4\ud2b8 \ub370\uc774\ube44\uc2a4 \uac19\uc740 \ucd94\uc0c1\ud30c \ud654\uac00\ub97c \ub9c9\ub860\ud558\uace0 20\uc138\uae30\uc758 \ub9ce\uc740 \uc608\uc220\uac00\ub4e4\uc5d0\uac8c \uc601\uac10\uc744 \ubd88\uc5b4\ub123\uc5c8\ub2e4. 20\uc138\uae30 \uc911\uc5fd \ucd94\uc0c1\ud45c\ud604\uc8fc\uc758(Abstract Expressionism)\ub85c \uc54c\ub824\uc9c4 \ubbf8\uad6d\uc758 \ud68c\ud654\uc591\uc2dd\uc774 \uacb0\uacfc\uc801\uc73c\ub85c \uc11c\uc591\ubbf8\uc220\uc744 \uc555\ub3c4\ud558\uac8c \ub418\uc5c8\ub2e4. \uc911\uc2ec\uc778\ubb3c\uc740 \uc7ad\uc2a8 \ud3f4\ub7ed\uacfc \ub85c\ubc84\ud2b8 \uba38\ub354\uc6f0 \ub4f1\uc774\uc5c8\ub2e4. \ucd94\uc0c1\ud45c\ud604\uc8fc\uc758\uc758 \uc704\uc138\ub3c4 1960\ub144\ub300\uc5d0 \ub610\ub2e4\ub978 \ubbf8\uad6d\uc801 \uc591\uc2dd\uc778 \ud31d \uc544\ud2b8(Pop Art)\uc758 \ub4f1\uc7a5\uc73c\ub85c \uc1e0\ud1f4\ud588\ub2e4. \uc564\ub514 \uc6cc\ud640\uacfc \ub85c\ubc84\ud2b8 \ub77c\uc6b0\uc174\ubc84\uadf8 \ub4f1\uc758 \ud31d \uc544\ud2f0\uc2a4\ud2b8\ub4e4\uc740 \ub300\uc911\ubb38\ud654\uc640 \uc0c1\uc5c5\uc5d0\uc11c \uc790\uc720\ub86d\uac8c \uc18c\uc7ac\ub97c \uc120\ud0dd\ud588\ub2e4.", "answer": "\ud504\ub808\ub9ac \ud559\ud30c", "sentence": "\uc124\ub9ac\ubc88\uc758 \uc81c\uc790\uc778 \ud504\ub7ad\ud06c \ub85c\uc774\ub4dc \ub77c\uc774\ud2b8(1869~1959)\ub294 \ud504\ub808\ub9ac \ud559\ud30c (Prairie School)", "paragraph_sentence": "\ub3c4\uc2dc\ub4e4\uc774 \uc18c\uc704 \ud669\uae08\uc2dc\ub300(Golden Age)\ub85c \uc811\uc5b4\ub4e4\uba74\uc11c \ub9c8\ucc9c\ub8e8\uac00 \ub4f1\uc7a5\ud574 \ubbf8\uad6d \uc0ac\ud68c\uc758 \uae30\uc220\uc801 \uc5c5\uc801\uacfc \uc790\uc2e0\uac10\uc744 \uc0c1\uc9d5\uc801\uc73c\ub85c \ub098\ud0c0\ub0c8\ub2e4. \uc70c\ub9ac\uc5c4 \ub7ec \ubc30\ub7f0 \uc81c\ub2c8\uc640 \ub8e8\uc774\uc2a4 \uc124\ub9ac\ubc88\uc740 \uc2dc\uce74\uace0 \ucd9c\uc2e0\uc758 \uac74\ucd95\uac00\ub85c, \uadf8\ub7ec\ud55c \uac74\ucd95\uc591\uc2dd\uc758 \uc120\uad6c\uc790\ub4e4\uc774\ub2e4. \uc124\ub9ac\ubc88\uc758 \uc81c\uc790\uc778 \ud504\ub7ad\ud06c \ub85c\uc774\ub4dc \ub77c\uc774\ud2b8(1869~1959)\ub294 \ud504\ub808\ub9ac \ud559\ud30c (Prairie School) \ub85c \uc54c\ub824\uc9c4 \uc0c8\ub85c\uc6b4 \uac74\ucd95\uc124\uacc4\uc6b4\ub3d9\uc744 \uc2dc\uc791\ud574 20\uc138\uae30\ucd08\uc5d0 \ubbf8\uad6d \uc804\uc5ed\uc5d0 \uac78\uccd0 \ud3ed\ub113\uc740 \uc601\ud5a5\uc744 \ub07c\ucce4\ub2e4. \ud6c4\ub300\uc758 \uc8fc\uc694 \ubbf8\uad6d\uac74\ucd95\uac00\ub85c\ub294 \ub3c5\uc77c \ud0dc\uc0dd\uc758 \ub8e8\ud2b8\ube44\ud788 \ubbf8\uc2a4 \ud310 \ub370\uc5b4\ub974 \ub85c\uc5b4, \uadf8\uc758 \uc81c\uc790\ub4e4\uc778 \ud544\ub9bd \uc874\uc2a8, \ub85c\ubc84\ud2b8 \ubca4\ud22c\ub9ac \ub4f1\uc774 \uc788\ub2e4. 1825~1870\ub144\uc5d0 \uc720\ud589\ud588\uc73c\uba70 \ud669\uc57c \uadf8 \uc790\uccb4\ub97c \uc18c\uc7ac\ub85c \uc0bc\uc558\ub358 \ud5c8\ub4dc\uc2a8 \uac15\ud30c(Hudson River School)\ub294 \ubbf8\uad6d \ubbf8\uc220\uc5d0\uc11c \uc77c\uc5b4\ub09c \ucd5c\ucd08\uc758 \ud1a0\ucc29\ud654 \uc6b4\ub3d9\uc73c\ub85c \uc5ec\uaca8\uc9c4\ub2e4. \uc544\uc154 B. \ub4c0\ub79c\ub4dc\uac00 \uadf8 \ub300\ud45c\uc801\uc778 \ud654\uac00\ub2e4. \ub3c4\uc2dc\uc0dd\ud65c\uc740 \uc5d0\ub4dc\uc6cc\ub4dc \ud638\ud37c \uac19\uc740 \uc0ac\uc2e4\uc8fc\uc758\ud30c\ub098 \uc2a4\ud29c\uc5b4\ud2b8 \ub370\uc774\ube44\uc2a4 \uac19\uc740 \ucd94\uc0c1\ud30c \ud654\uac00\ub97c \ub9c9\ub860\ud558\uace0 20\uc138\uae30\uc758 \ub9ce\uc740 \uc608\uc220\uac00\ub4e4\uc5d0\uac8c \uc601\uac10\uc744 \ubd88\uc5b4\ub123\uc5c8\ub2e4. 20\uc138\uae30 \uc911\uc5fd \ucd94\uc0c1\ud45c\ud604\uc8fc\uc758(Abstract Expressionism)\ub85c \uc54c\ub824\uc9c4 \ubbf8\uad6d\uc758 \ud68c\ud654\uc591\uc2dd\uc774 \uacb0\uacfc\uc801\uc73c\ub85c \uc11c\uc591\ubbf8\uc220\uc744 \uc555\ub3c4\ud558\uac8c \ub418\uc5c8\ub2e4. \uc911\uc2ec\uc778\ubb3c\uc740 \uc7ad\uc2a8 \ud3f4\ub7ed\uacfc \ub85c\ubc84\ud2b8 \uba38\ub354\uc6f0 \ub4f1\uc774\uc5c8\ub2e4. \ucd94\uc0c1\ud45c\ud604\uc8fc\uc758\uc758 \uc704\uc138\ub3c4 1960\ub144\ub300\uc5d0 \ub610\ub2e4\ub978 \ubbf8\uad6d\uc801 \uc591\uc2dd\uc778 \ud31d \uc544\ud2b8(Pop Art)\uc758 \ub4f1\uc7a5\uc73c\ub85c \uc1e0\ud1f4\ud588\ub2e4. \uc564\ub514 \uc6cc\ud640\uacfc \ub85c\ubc84\ud2b8 \ub77c\uc6b0\uc174\ubc84\uadf8 \ub4f1\uc758 \ud31d \uc544\ud2f0\uc2a4\ud2b8\ub4e4\uc740 \ub300\uc911\ubb38\ud654\uc640 \uc0c1\uc5c5\uc5d0\uc11c \uc790\uc720\ub86d\uac8c \uc18c\uc7ac\ub97c \uc120\ud0dd\ud588\ub2e4.", "paragraph_answer": "\ub3c4\uc2dc\ub4e4\uc774 \uc18c\uc704 \ud669\uae08\uc2dc\ub300(Golden Age)\ub85c \uc811\uc5b4\ub4e4\uba74\uc11c \ub9c8\ucc9c\ub8e8\uac00 \ub4f1\uc7a5\ud574 \ubbf8\uad6d \uc0ac\ud68c\uc758 \uae30\uc220\uc801 \uc5c5\uc801\uacfc \uc790\uc2e0\uac10\uc744 \uc0c1\uc9d5\uc801\uc73c\ub85c \ub098\ud0c0\ub0c8\ub2e4. \uc70c\ub9ac\uc5c4 \ub7ec \ubc30\ub7f0 \uc81c\ub2c8\uc640 \ub8e8\uc774\uc2a4 \uc124\ub9ac\ubc88\uc740 \uc2dc\uce74\uace0 \ucd9c\uc2e0\uc758 \uac74\ucd95\uac00\ub85c, \uadf8\ub7ec\ud55c \uac74\ucd95\uc591\uc2dd\uc758 \uc120\uad6c\uc790\ub4e4\uc774\ub2e4. \uc124\ub9ac\ubc88\uc758 \uc81c\uc790\uc778 \ud504\ub7ad\ud06c \ub85c\uc774\ub4dc \ub77c\uc774\ud2b8(1869~1959)\ub294 \ud504\ub808\ub9ac \ud559\ud30c (Prairie School)\ub85c \uc54c\ub824\uc9c4 \uc0c8\ub85c\uc6b4 \uac74\ucd95\uc124\uacc4\uc6b4\ub3d9\uc744 \uc2dc\uc791\ud574 20\uc138\uae30\ucd08\uc5d0 \ubbf8\uad6d \uc804\uc5ed\uc5d0 \uac78\uccd0 \ud3ed\ub113\uc740 \uc601\ud5a5\uc744 \ub07c\ucce4\ub2e4. \ud6c4\ub300\uc758 \uc8fc\uc694 \ubbf8\uad6d\uac74\ucd95\uac00\ub85c\ub294 \ub3c5\uc77c \ud0dc\uc0dd\uc758 \ub8e8\ud2b8\ube44\ud788 \ubbf8\uc2a4 \ud310 \ub370\uc5b4\ub974 \ub85c\uc5b4, \uadf8\uc758 \uc81c\uc790\ub4e4\uc778 \ud544\ub9bd \uc874\uc2a8, \ub85c\ubc84\ud2b8 \ubca4\ud22c\ub9ac \ub4f1\uc774 \uc788\ub2e4. 1825~1870\ub144\uc5d0 \uc720\ud589\ud588\uc73c\uba70 \ud669\uc57c \uadf8 \uc790\uccb4\ub97c \uc18c\uc7ac\ub85c \uc0bc\uc558\ub358 \ud5c8\ub4dc\uc2a8 \uac15\ud30c(Hudson River School)\ub294 \ubbf8\uad6d \ubbf8\uc220\uc5d0\uc11c \uc77c\uc5b4\ub09c \ucd5c\ucd08\uc758 \ud1a0\ucc29\ud654 \uc6b4\ub3d9\uc73c\ub85c \uc5ec\uaca8\uc9c4\ub2e4. \uc544\uc154 B. \ub4c0\ub79c\ub4dc\uac00 \uadf8 \ub300\ud45c\uc801\uc778 \ud654\uac00\ub2e4. \ub3c4\uc2dc\uc0dd\ud65c\uc740 \uc5d0\ub4dc\uc6cc\ub4dc \ud638\ud37c \uac19\uc740 \uc0ac\uc2e4\uc8fc\uc758\ud30c\ub098 \uc2a4\ud29c\uc5b4\ud2b8 \ub370\uc774\ube44\uc2a4 \uac19\uc740 \ucd94\uc0c1\ud30c \ud654\uac00\ub97c \ub9c9\ub860\ud558\uace0 20\uc138\uae30\uc758 \ub9ce\uc740 \uc608\uc220\uac00\ub4e4\uc5d0\uac8c \uc601\uac10\uc744 \ubd88\uc5b4\ub123\uc5c8\ub2e4. 20\uc138\uae30 \uc911\uc5fd \ucd94\uc0c1\ud45c\ud604\uc8fc\uc758(Abstract Expressionism)\ub85c \uc54c\ub824\uc9c4 \ubbf8\uad6d\uc758 \ud68c\ud654\uc591\uc2dd\uc774 \uacb0\uacfc\uc801\uc73c\ub85c \uc11c\uc591\ubbf8\uc220\uc744 \uc555\ub3c4\ud558\uac8c \ub418\uc5c8\ub2e4. \uc911\uc2ec\uc778\ubb3c\uc740 \uc7ad\uc2a8 \ud3f4\ub7ed\uacfc \ub85c\ubc84\ud2b8 \uba38\ub354\uc6f0 \ub4f1\uc774\uc5c8\ub2e4. \ucd94\uc0c1\ud45c\ud604\uc8fc\uc758\uc758 \uc704\uc138\ub3c4 1960\ub144\ub300\uc5d0 \ub610\ub2e4\ub978 \ubbf8\uad6d\uc801 \uc591\uc2dd\uc778 \ud31d \uc544\ud2b8(Pop Art)\uc758 \ub4f1\uc7a5\uc73c\ub85c \uc1e0\ud1f4\ud588\ub2e4. \uc564\ub514 \uc6cc\ud640\uacfc \ub85c\ubc84\ud2b8 \ub77c\uc6b0\uc174\ubc84\uadf8 \ub4f1\uc758 \ud31d \uc544\ud2f0\uc2a4\ud2b8\ub4e4\uc740 \ub300\uc911\ubb38\ud654\uc640 \uc0c1\uc5c5\uc5d0\uc11c \uc790\uc720\ub86d\uac8c \uc18c\uc7ac\ub97c \uc120\ud0dd\ud588\ub2e4.", "sentence_answer": "\uc124\ub9ac\ubc88\uc758 \uc81c\uc790\uc778 \ud504\ub7ad\ud06c \ub85c\uc774\ub4dc \ub77c\uc774\ud2b8(1869~1959)\ub294 \ud504\ub808\ub9ac \ud559\ud30c (Prairie School)", "paragraph_id": "6412933-34-1", "paragraph_question": "question: \ud504\ub7ad\ud06c \ub85c\uc774\ub4dc \ub77c\uc774\ud2b8\ub294 \uc5b4\ub5a4 \uac74\ucd95 \uc124\uacc4 \uc6b4\ub3d9\uc744 \uc2dc\uc791\ud588\ub294\uac00?, context: \ub3c4\uc2dc\ub4e4\uc774 \uc18c\uc704 \ud669\uae08\uc2dc\ub300(Golden Age)\ub85c \uc811\uc5b4\ub4e4\uba74\uc11c \ub9c8\ucc9c\ub8e8\uac00 \ub4f1\uc7a5\ud574 \ubbf8\uad6d \uc0ac\ud68c\uc758 \uae30\uc220\uc801 \uc5c5\uc801\uacfc \uc790\uc2e0\uac10\uc744 \uc0c1\uc9d5\uc801\uc73c\ub85c \ub098\ud0c0\ub0c8\ub2e4. \uc70c\ub9ac\uc5c4 \ub7ec \ubc30\ub7f0 \uc81c\ub2c8\uc640 \ub8e8\uc774\uc2a4 \uc124\ub9ac\ubc88\uc740 \uc2dc\uce74\uace0 \ucd9c\uc2e0\uc758 \uac74\ucd95\uac00\ub85c, \uadf8\ub7ec\ud55c \uac74\ucd95\uc591\uc2dd\uc758 \uc120\uad6c\uc790\ub4e4\uc774\ub2e4. \uc124\ub9ac\ubc88\uc758 \uc81c\uc790\uc778 \ud504\ub7ad\ud06c \ub85c\uc774\ub4dc \ub77c\uc774\ud2b8(1869~1959)\ub294 \ud504\ub808\ub9ac \ud559\ud30c(Prairie School)\ub85c \uc54c\ub824\uc9c4 \uc0c8\ub85c\uc6b4 \uac74\ucd95\uc124\uacc4\uc6b4\ub3d9\uc744 \uc2dc\uc791\ud574 20\uc138\uae30\ucd08\uc5d0 \ubbf8\uad6d \uc804\uc5ed\uc5d0 \uac78\uccd0 \ud3ed\ub113\uc740 \uc601\ud5a5\uc744 \ub07c\ucce4\ub2e4. \ud6c4\ub300\uc758 \uc8fc\uc694 \ubbf8\uad6d\uac74\ucd95\uac00\ub85c\ub294 \ub3c5\uc77c \ud0dc\uc0dd\uc758 \ub8e8\ud2b8\ube44\ud788 \ubbf8\uc2a4 \ud310 \ub370\uc5b4\ub974 \ub85c\uc5b4, \uadf8\uc758 \uc81c\uc790\ub4e4\uc778 \ud544\ub9bd \uc874\uc2a8, \ub85c\ubc84\ud2b8 \ubca4\ud22c\ub9ac \ub4f1\uc774 \uc788\ub2e4. 1825~1870\ub144\uc5d0 \uc720\ud589\ud588\uc73c\uba70 \ud669\uc57c \uadf8 \uc790\uccb4\ub97c \uc18c\uc7ac\ub85c \uc0bc\uc558\ub358 \ud5c8\ub4dc\uc2a8 \uac15\ud30c(Hudson River School)\ub294 \ubbf8\uad6d \ubbf8\uc220\uc5d0\uc11c \uc77c\uc5b4\ub09c \ucd5c\ucd08\uc758 \ud1a0\ucc29\ud654 \uc6b4\ub3d9\uc73c\ub85c \uc5ec\uaca8\uc9c4\ub2e4. \uc544\uc154 B. \ub4c0\ub79c\ub4dc\uac00 \uadf8 \ub300\ud45c\uc801\uc778 \ud654\uac00\ub2e4. \ub3c4\uc2dc\uc0dd\ud65c\uc740 \uc5d0\ub4dc\uc6cc\ub4dc \ud638\ud37c \uac19\uc740 \uc0ac\uc2e4\uc8fc\uc758\ud30c\ub098 \uc2a4\ud29c\uc5b4\ud2b8 \ub370\uc774\ube44\uc2a4 \uac19\uc740 \ucd94\uc0c1\ud30c \ud654\uac00\ub97c \ub9c9\ub860\ud558\uace0 20\uc138\uae30\uc758 \ub9ce\uc740 \uc608\uc220\uac00\ub4e4\uc5d0\uac8c \uc601\uac10\uc744 \ubd88\uc5b4\ub123\uc5c8\ub2e4. 20\uc138\uae30 \uc911\uc5fd \ucd94\uc0c1\ud45c\ud604\uc8fc\uc758(Abstract Expressionism)\ub85c \uc54c\ub824\uc9c4 \ubbf8\uad6d\uc758 \ud68c\ud654\uc591\uc2dd\uc774 \uacb0\uacfc\uc801\uc73c\ub85c \uc11c\uc591\ubbf8\uc220\uc744 \uc555\ub3c4\ud558\uac8c \ub418\uc5c8\ub2e4. \uc911\uc2ec\uc778\ubb3c\uc740 \uc7ad\uc2a8 \ud3f4\ub7ed\uacfc \ub85c\ubc84\ud2b8 \uba38\ub354\uc6f0 \ub4f1\uc774\uc5c8\ub2e4. \ucd94\uc0c1\ud45c\ud604\uc8fc\uc758\uc758 \uc704\uc138\ub3c4 1960\ub144\ub300\uc5d0 \ub610\ub2e4\ub978 \ubbf8\uad6d\uc801 \uc591\uc2dd\uc778 \ud31d \uc544\ud2b8(Pop Art)\uc758 \ub4f1\uc7a5\uc73c\ub85c \uc1e0\ud1f4\ud588\ub2e4. \uc564\ub514 \uc6cc\ud640\uacfc \ub85c\ubc84\ud2b8 \ub77c\uc6b0\uc174\ubc84\uadf8 \ub4f1\uc758 \ud31d \uc544\ud2f0\uc2a4\ud2b8\ub4e4\uc740 \ub300\uc911\ubb38\ud654\uc640 \uc0c1\uc5c5\uc5d0\uc11c \uc790\uc720\ub86d\uac8c \uc18c\uc7ac\ub97c \uc120\ud0dd\ud588\ub2e4."} {"question": "\uce74\ub274\ub3c4\uac00 20\uc138\uae30 \ucd08\ubd80\ud130 \uac70\uc8fc\ud588\ub358 \uc9c0\uc5ed\uc740?", "paragraph": "\uc601\ud654\uc758 \uc608\uc220\uc131\uc744 \uc774\ub860\uc801\uc73c\ub85c \ucc98\uc74c\uc73c\ub85c \uc8fc\uc7a5\ud55c \uc0ac\ub78c\uc740 \uc774\ud0c8\ub9ac\uc544\uc778 \ub9ac\uce58\uc624\ub2c8 \uce74\ub274\ub3c4(1879-1923)\ub77c \ud55c\ub2e4. \uc601\ud654\ub294 \ubcf8\uc9c8\uc801\uc73c\ub85c '\ube5b\uc758 \ud39c'\uc73c\ub85c \uadf8\ub824\uc9c0\uace0 \uc601\uc0c1\uc73c\ub85c \ub9cc\ub4e4\uc5b4\uc9c4 \uc2dc\uac01(\u8996\u89ba)\uc758 \ub4dc\ub77c\ub9c8\ub77c\uace0 \uadf8\uac00 \uc8fc\uc7a5\ud55c \uac83\uc740 1911\ub144\uc774\uc5c8\ub2e4. \uadf8\ub294 \uc608\uc220\uc758 \uae30\ubcf8\uc740 \uac74\ucd95\uacfc \uc74c\uc545\uc774\uba70, \uc774\ub97c \ubcf4\ucda9\ud558\ub294 \uac83\uc73c\ub85c\uc11c \uadf8\ub9bc\uacfc \uc870\uac01\uacfc \uc2dc\uc640 \ubb34\uc6a9\uc744 \ub4e4\uc5c8\uace0, \uc774 \ubaa8\ub4e0 \uac83\uc744 \ud3ec\ud568\ud558\ub294 \uac83\uc774 \uc601\ud654\uc774\uba70 \uc6c0\uc9c1\uc784\uc758 \uc870\ud615 \uc608\uc220\uc774\ub77c \ud558\uc5ec \uc601\ud654\ub97c \uc81c7\uc758 \uc608\uc220\uc774\ub77c\uace0 \uc8fc\uc7a5\ud588\ub2e4. \uce74\ub274\ub3c4\ub294 20\uc138\uae30 \ucd08\uae30\ubd80\ud130 \ud30c\ub9ac\uc5d0 \uc0b4\uba74\uc11c \uc544\ud3f4\ub9ac\ub124\ub974, \ud53c\uce87, \uc2a4\ud2b8\ub77c\ube48\uc2a4\ud0a4, \ub808\uc81c \uac19\uc740 \uc608\uc220\uac00\ub4e4\uacfc \uce5c\ubd84\uc744 \uac16\uace0 \uc788\ub358 \uc0ac\ub78c\uc73c\ub85c, \ub4a4\uc5d0 \ud504\ub791\uc2a4\uc758 \uc601\ud654 \uc774\ub860\uac00\uc5d0\uac8c \ud070 \uc601\ud5a5\uc744 \uc8fc\uc5c8\ub2e4. \uce74\ub274\ub3c4\ub294 \uc601\ud654\uc758 \uc5f0\uadf9\uc131\uc744 \ubd80\uc815\ud568\uc73c\ub85c\uc368 \uc790\uae30\uc758 \uc8fc\uc7a5\uc744 \ub9e4\uc6b0 \ubd84\uba85\ud558\uac8c \ud558\uc600\uace0, \uadf8\ub7f0 \ub9cc\ud07c \uc601\ud654\uc758 \uc601\uc0c1\uc131(\u6620\u50cf\u6027)\uc758 \uc21c\uc218\uc131\uc5d0\ub9cc \uadf8 \ud45c\ud604\ub825\uc744 \uc81c\uc57d\ud588\ub2e4\uace0 \ud560 \uc218 \uc788\uc73c\uba70, \uadf8\uac00 '\uc601\ud654\uc2dc(\u6620\u7575\u8a69)\uc758 \uc804\ub3c4\uc0ac(\u50b3\u9053\u5e2b)'(\uc7a5\uc5d0\ud504\uc2a4\ud0d5)\ub77c\uace0 \ubd88\ub9ac\ub294 \uae4c\ub2ed\uc740 \uc774\ub7f0 \ub370\uc5d0 \uc788\ub2e4.", "answer": "\ud30c\ub9ac", "sentence": "\uce74\ub274\ub3c4\ub294 20\uc138\uae30 \ucd08\uae30\ubd80\ud130 \ud30c\ub9ac \uc5d0 \uc0b4\uba74\uc11c \uc544\ud3f4\ub9ac\ub124\ub974, \ud53c\uce87, \uc2a4\ud2b8\ub77c\ube48\uc2a4\ud0a4, \ub808\uc81c \uac19\uc740 \uc608\uc220\uac00\ub4e4\uacfc \uce5c\ubd84\uc744 \uac16\uace0 \uc788\ub358 \uc0ac\ub78c\uc73c\ub85c, \ub4a4\uc5d0 \ud504\ub791\uc2a4\uc758 \uc601\ud654 \uc774\ub860\uac00\uc5d0\uac8c \ud070 \uc601\ud5a5\uc744 \uc8fc\uc5c8\ub2e4.", "paragraph_sentence": "\uc601\ud654\uc758 \uc608\uc220\uc131\uc744 \uc774\ub860\uc801\uc73c\ub85c \ucc98\uc74c\uc73c\ub85c \uc8fc\uc7a5\ud55c \uc0ac\ub78c\uc740 \uc774\ud0c8\ub9ac\uc544\uc778 \ub9ac\uce58\uc624\ub2c8 \uce74\ub274\ub3c4(1879-1923)\ub77c \ud55c\ub2e4. \uc601\ud654\ub294 \ubcf8\uc9c8\uc801\uc73c\ub85c '\ube5b\uc758 \ud39c'\uc73c\ub85c \uadf8\ub824\uc9c0\uace0 \uc601\uc0c1\uc73c\ub85c \ub9cc\ub4e4\uc5b4\uc9c4 \uc2dc\uac01(\u8996\u89ba)\uc758 \ub4dc\ub77c\ub9c8\ub77c\uace0 \uadf8\uac00 \uc8fc\uc7a5\ud55c \uac83\uc740 1911\ub144\uc774\uc5c8\ub2e4. \uadf8\ub294 \uc608\uc220\uc758 \uae30\ubcf8\uc740 \uac74\ucd95\uacfc \uc74c\uc545\uc774\uba70, \uc774\ub97c \ubcf4\ucda9\ud558\ub294 \uac83\uc73c\ub85c\uc11c \uadf8\ub9bc\uacfc \uc870\uac01\uacfc \uc2dc\uc640 \ubb34\uc6a9\uc744 \ub4e4\uc5c8\uace0, \uc774 \ubaa8\ub4e0 \uac83\uc744 \ud3ec\ud568\ud558\ub294 \uac83\uc774 \uc601\ud654\uc774\uba70 \uc6c0\uc9c1\uc784\uc758 \uc870\ud615 \uc608\uc220\uc774\ub77c \ud558\uc5ec \uc601\ud654\ub97c \uc81c7\uc758 \uc608\uc220\uc774\ub77c\uace0 \uc8fc\uc7a5\ud588\ub2e4. \uce74\ub274\ub3c4\ub294 20\uc138\uae30 \ucd08\uae30\ubd80\ud130 \ud30c\ub9ac \uc5d0 \uc0b4\uba74\uc11c \uc544\ud3f4\ub9ac\ub124\ub974, \ud53c\uce87, \uc2a4\ud2b8\ub77c\ube48\uc2a4\ud0a4, \ub808\uc81c \uac19\uc740 \uc608\uc220\uac00\ub4e4\uacfc \uce5c\ubd84\uc744 \uac16\uace0 \uc788\ub358 \uc0ac\ub78c\uc73c\ub85c, \ub4a4\uc5d0 \ud504\ub791\uc2a4\uc758 \uc601\ud654 \uc774\ub860\uac00\uc5d0\uac8c \ud070 \uc601\ud5a5\uc744 \uc8fc\uc5c8\ub2e4. \uce74\ub274\ub3c4\ub294 \uc601\ud654\uc758 \uc5f0\uadf9\uc131\uc744 \ubd80\uc815\ud568\uc73c\ub85c\uc368 \uc790\uae30\uc758 \uc8fc\uc7a5\uc744 \ub9e4\uc6b0 \ubd84\uba85\ud558\uac8c \ud558\uc600\uace0, \uadf8\ub7f0 \ub9cc\ud07c \uc601\ud654\uc758 \uc601\uc0c1\uc131(\u6620\u50cf\u6027)\uc758 \uc21c\uc218\uc131\uc5d0\ub9cc \uadf8 \ud45c\ud604\ub825\uc744 \uc81c\uc57d\ud588\ub2e4\uace0 \ud560 \uc218 \uc788\uc73c\uba70, \uadf8\uac00 '\uc601\ud654\uc2dc(\u6620\u7575\u8a69)\uc758 \uc804\ub3c4\uc0ac(\u50b3\u9053\u5e2b)'(\uc7a5\uc5d0\ud504\uc2a4\ud0d5)\ub77c\uace0 \ubd88\ub9ac\ub294 \uae4c\ub2ed\uc740 \uc774\ub7f0 \ub370\uc5d0 \uc788\ub2e4.", "paragraph_answer": "\uc601\ud654\uc758 \uc608\uc220\uc131\uc744 \uc774\ub860\uc801\uc73c\ub85c \ucc98\uc74c\uc73c\ub85c \uc8fc\uc7a5\ud55c \uc0ac\ub78c\uc740 \uc774\ud0c8\ub9ac\uc544\uc778 \ub9ac\uce58\uc624\ub2c8 \uce74\ub274\ub3c4(1879-1923)\ub77c \ud55c\ub2e4. \uc601\ud654\ub294 \ubcf8\uc9c8\uc801\uc73c\ub85c '\ube5b\uc758 \ud39c'\uc73c\ub85c \uadf8\ub824\uc9c0\uace0 \uc601\uc0c1\uc73c\ub85c \ub9cc\ub4e4\uc5b4\uc9c4 \uc2dc\uac01(\u8996\u89ba)\uc758 \ub4dc\ub77c\ub9c8\ub77c\uace0 \uadf8\uac00 \uc8fc\uc7a5\ud55c \uac83\uc740 1911\ub144\uc774\uc5c8\ub2e4. \uadf8\ub294 \uc608\uc220\uc758 \uae30\ubcf8\uc740 \uac74\ucd95\uacfc \uc74c\uc545\uc774\uba70, \uc774\ub97c \ubcf4\ucda9\ud558\ub294 \uac83\uc73c\ub85c\uc11c \uadf8\ub9bc\uacfc \uc870\uac01\uacfc \uc2dc\uc640 \ubb34\uc6a9\uc744 \ub4e4\uc5c8\uace0, \uc774 \ubaa8\ub4e0 \uac83\uc744 \ud3ec\ud568\ud558\ub294 \uac83\uc774 \uc601\ud654\uc774\uba70 \uc6c0\uc9c1\uc784\uc758 \uc870\ud615 \uc608\uc220\uc774\ub77c \ud558\uc5ec \uc601\ud654\ub97c \uc81c7\uc758 \uc608\uc220\uc774\ub77c\uace0 \uc8fc\uc7a5\ud588\ub2e4. \uce74\ub274\ub3c4\ub294 20\uc138\uae30 \ucd08\uae30\ubd80\ud130 \ud30c\ub9ac \uc5d0 \uc0b4\uba74\uc11c \uc544\ud3f4\ub9ac\ub124\ub974, \ud53c\uce87, \uc2a4\ud2b8\ub77c\ube48\uc2a4\ud0a4, \ub808\uc81c \uac19\uc740 \uc608\uc220\uac00\ub4e4\uacfc \uce5c\ubd84\uc744 \uac16\uace0 \uc788\ub358 \uc0ac\ub78c\uc73c\ub85c, \ub4a4\uc5d0 \ud504\ub791\uc2a4\uc758 \uc601\ud654 \uc774\ub860\uac00\uc5d0\uac8c \ud070 \uc601\ud5a5\uc744 \uc8fc\uc5c8\ub2e4. \uce74\ub274\ub3c4\ub294 \uc601\ud654\uc758 \uc5f0\uadf9\uc131\uc744 \ubd80\uc815\ud568\uc73c\ub85c\uc368 \uc790\uae30\uc758 \uc8fc\uc7a5\uc744 \ub9e4\uc6b0 \ubd84\uba85\ud558\uac8c \ud558\uc600\uace0, \uadf8\ub7f0 \ub9cc\ud07c \uc601\ud654\uc758 \uc601\uc0c1\uc131(\u6620\u50cf\u6027)\uc758 \uc21c\uc218\uc131\uc5d0\ub9cc \uadf8 \ud45c\ud604\ub825\uc744 \uc81c\uc57d\ud588\ub2e4\uace0 \ud560 \uc218 \uc788\uc73c\uba70, \uadf8\uac00 '\uc601\ud654\uc2dc(\u6620\u7575\u8a69)\uc758 \uc804\ub3c4\uc0ac(\u50b3\u9053\u5e2b)'(\uc7a5\uc5d0\ud504\uc2a4\ud0d5)\ub77c\uace0 \ubd88\ub9ac\ub294 \uae4c\ub2ed\uc740 \uc774\ub7f0 \ub370\uc5d0 \uc788\ub2e4.", "sentence_answer": "\uce74\ub274\ub3c4\ub294 20\uc138\uae30 \ucd08\uae30\ubd80\ud130 \ud30c\ub9ac \uc5d0 \uc0b4\uba74\uc11c \uc544\ud3f4\ub9ac\ub124\ub974, \ud53c\uce87, \uc2a4\ud2b8\ub77c\ube48\uc2a4\ud0a4, \ub808\uc81c \uac19\uc740 \uc608\uc220\uac00\ub4e4\uacfc \uce5c\ubd84\uc744 \uac16\uace0 \uc788\ub358 \uc0ac\ub78c\uc73c\ub85c, \ub4a4\uc5d0 \ud504\ub791\uc2a4\uc758 \uc601\ud654 \uc774\ub860\uac00\uc5d0\uac8c \ud070 \uc601\ud5a5\uc744 \uc8fc\uc5c8\ub2e4.", "paragraph_id": "6541178-0-2", "paragraph_question": "question: \uce74\ub274\ub3c4\uac00 20\uc138\uae30 \ucd08\ubd80\ud130 \uac70\uc8fc\ud588\ub358 \uc9c0\uc5ed\uc740?, context: \uc601\ud654\uc758 \uc608\uc220\uc131\uc744 \uc774\ub860\uc801\uc73c\ub85c \ucc98\uc74c\uc73c\ub85c \uc8fc\uc7a5\ud55c \uc0ac\ub78c\uc740 \uc774\ud0c8\ub9ac\uc544\uc778 \ub9ac\uce58\uc624\ub2c8 \uce74\ub274\ub3c4(1879-1923)\ub77c \ud55c\ub2e4. \uc601\ud654\ub294 \ubcf8\uc9c8\uc801\uc73c\ub85c '\ube5b\uc758 \ud39c'\uc73c\ub85c \uadf8\ub824\uc9c0\uace0 \uc601\uc0c1\uc73c\ub85c \ub9cc\ub4e4\uc5b4\uc9c4 \uc2dc\uac01(\u8996\u89ba)\uc758 \ub4dc\ub77c\ub9c8\ub77c\uace0 \uadf8\uac00 \uc8fc\uc7a5\ud55c \uac83\uc740 1911\ub144\uc774\uc5c8\ub2e4. \uadf8\ub294 \uc608\uc220\uc758 \uae30\ubcf8\uc740 \uac74\ucd95\uacfc \uc74c\uc545\uc774\uba70, \uc774\ub97c \ubcf4\ucda9\ud558\ub294 \uac83\uc73c\ub85c\uc11c \uadf8\ub9bc\uacfc \uc870\uac01\uacfc \uc2dc\uc640 \ubb34\uc6a9\uc744 \ub4e4\uc5c8\uace0, \uc774 \ubaa8\ub4e0 \uac83\uc744 \ud3ec\ud568\ud558\ub294 \uac83\uc774 \uc601\ud654\uc774\uba70 \uc6c0\uc9c1\uc784\uc758 \uc870\ud615 \uc608\uc220\uc774\ub77c \ud558\uc5ec \uc601\ud654\ub97c \uc81c7\uc758 \uc608\uc220\uc774\ub77c\uace0 \uc8fc\uc7a5\ud588\ub2e4. \uce74\ub274\ub3c4\ub294 20\uc138\uae30 \ucd08\uae30\ubd80\ud130 \ud30c\ub9ac\uc5d0 \uc0b4\uba74\uc11c \uc544\ud3f4\ub9ac\ub124\ub974, \ud53c\uce87, \uc2a4\ud2b8\ub77c\ube48\uc2a4\ud0a4, \ub808\uc81c \uac19\uc740 \uc608\uc220\uac00\ub4e4\uacfc \uce5c\ubd84\uc744 \uac16\uace0 \uc788\ub358 \uc0ac\ub78c\uc73c\ub85c, \ub4a4\uc5d0 \ud504\ub791\uc2a4\uc758 \uc601\ud654 \uc774\ub860\uac00\uc5d0\uac8c \ud070 \uc601\ud5a5\uc744 \uc8fc\uc5c8\ub2e4. \uce74\ub274\ub3c4\ub294 \uc601\ud654\uc758 \uc5f0\uadf9\uc131\uc744 \ubd80\uc815\ud568\uc73c\ub85c\uc368 \uc790\uae30\uc758 \uc8fc\uc7a5\uc744 \ub9e4\uc6b0 \ubd84\uba85\ud558\uac8c \ud558\uc600\uace0, \uadf8\ub7f0 \ub9cc\ud07c \uc601\ud654\uc758 \uc601\uc0c1\uc131(\u6620\u50cf\u6027)\uc758 \uc21c\uc218\uc131\uc5d0\ub9cc \uadf8 \ud45c\ud604\ub825\uc744 \uc81c\uc57d\ud588\ub2e4\uace0 \ud560 \uc218 \uc788\uc73c\uba70, \uadf8\uac00 '\uc601\ud654\uc2dc(\u6620\u7575\u8a69)\uc758 \uc804\ub3c4\uc0ac(\u50b3\u9053\u5e2b)'(\uc7a5\uc5d0\ud504\uc2a4\ud0d5)\ub77c\uace0 \ubd88\ub9ac\ub294 \uae4c\ub2ed\uc740 \uc774\ub7f0 \ub370\uc5d0 \uc788\ub2e4."} {"question": "\ud45c\ucc3d\uc6d0\uc774 \uacbd\ucc30\uad00\uc5d0 \uc784\uc6a9\ub41c \uc5f0\ub3c4\ub294?", "paragraph": "1989\ub144 \uacbd\ucc30\ub300\ud559 \uc878\uc5c5\uacfc \ub3d9\uc2dc\uc5d0 \uacbd\ucc30\uad00\uc5d0 \uc784\uc6a9\ub418\uc5c8\ub2e4. \uacbd\ucc30\uad00 \uc7ac\uc9c1 \uc911 1989\ub144\ubd80\ud130 1993\ub144\uae4c\uc9c0\ub294 \uc758\ubb34\ubcf5\ubb34\ub85c\uc368 \uc804\uacbd \uc18c\ub300\uc7a5\uc744 \ud55c\ud6c4 \uacbd\ucc30\uad00\uc73c\ub85c \uadfc\ubb34\ud558\uc600\uace0 1993\ubd80\ud1301997\ub144\uae4c\uc9c0\ub294 \uc601\uad6d\uc5d0\uc11c \uc11d\uc0ac\ud559\uc704\uc640 \ubc15\uc0ac\ud559\uc704\ub97c \ucde8\ub4dd\ud558\uc600\ub2e4. 1998\ub144 \uad11\uc6b4\ub300\ud559\uad50\uc640 \ud55c\uad6d\uc678\uad6d\uc5b4\ub300\ud559\uad50 \uad50\uc218\ub85c \ucd9c\uac15\ud558\uace0, \uacbd\ucc30\uad00\uc5d0\uc11c \ud1f4\uc9c1\ud55c \uc774\ud6c4\uc5d0\ub294 \uacbd\ucc30\ub300\ud559\uc758 \uac15\uc0ac\uac00 \ub418\uc5c8\uc73c\uba70 \uc5f0\uc138\ub300\ud559\uad50, \uc544\uc8fc\ub300\ud559\uad50, \uacbd\uae30\ub300\ud559\uad50 \ub4f1\uc5d0 \ucd9c\uac15\ud558\uc600\ub2e4. 2001\ub144 \uacbd\ucc30\ub300\ud559 \uc870\uad50\uc218\uac00 \ub418\uc5c8\uace0, 2012\ub144 \uacbd\ucc30\ub300\ud559 \uc815\uad50\uc218\uac00 \ub418\uc5c8\ub2e4. 2012\ub144 12\uc6d4, \ub300\ud55c\ubbfc\uad6d \uad6d\uac00\uc815\ubcf4\uc6d0 \uc5ec\ub860 \uc870\uc791 \uc0ac\uac74\uc5d0 \ub300\ud55c \uacac\ud574\ub97c \ud45c\uba85\ud558\ub294 \uacfc\uc815\uc5d0\uc11c '\uacbd\ucc30\ub300\uc758 \uc815\uce58\uc801 \uc911\ub9bd\uc131\uc744 \uce68\ud574\ud560 \uc218 \uc788\ub2e4'\ub294 \uc774\uc720\ub85c \uacbd\ucc30\ub300\ud559 \uad50\uc218\uc9c1\uc744 \uc0ac\ud1f4\ud588\ub2e4.", "answer": "1989\ub144", "sentence": "1989\ub144 \uacbd\ucc30\ub300\ud559 \uc878\uc5c5\uacfc \ub3d9\uc2dc\uc5d0 \uacbd\ucc30\uad00\uc5d0 \uc784\uc6a9\ub418\uc5c8\ub2e4.", "paragraph_sentence": " 1989\ub144 \uacbd\ucc30\ub300\ud559 \uc878\uc5c5\uacfc \ub3d9\uc2dc\uc5d0 \uacbd\ucc30\uad00\uc5d0 \uc784\uc6a9\ub418\uc5c8\ub2e4. \uacbd\ucc30\uad00 \uc7ac\uc9c1 \uc911 1989\ub144\ubd80\ud130 1993\ub144\uae4c\uc9c0\ub294 \uc758\ubb34\ubcf5\ubb34\ub85c\uc368 \uc804\uacbd \uc18c\ub300\uc7a5\uc744 \ud55c\ud6c4 \uacbd\ucc30\uad00\uc73c\ub85c \uadfc\ubb34\ud558\uc600\uace0 1993\ubd80\ud1301997\ub144\uae4c\uc9c0\ub294 \uc601\uad6d\uc5d0\uc11c \uc11d\uc0ac\ud559\uc704\uc640 \ubc15\uc0ac\ud559\uc704\ub97c \ucde8\ub4dd\ud558\uc600\ub2e4. 1998\ub144 \uad11\uc6b4\ub300\ud559\uad50\uc640 \ud55c\uad6d\uc678\uad6d\uc5b4\ub300\ud559\uad50 \uad50\uc218\ub85c \ucd9c\uac15\ud558\uace0, \uacbd\ucc30\uad00\uc5d0\uc11c \ud1f4\uc9c1\ud55c \uc774\ud6c4\uc5d0\ub294 \uacbd\ucc30\ub300\ud559\uc758 \uac15\uc0ac\uac00 \ub418\uc5c8\uc73c\uba70 \uc5f0\uc138\ub300\ud559\uad50, \uc544\uc8fc\ub300\ud559\uad50, \uacbd\uae30\ub300\ud559\uad50 \ub4f1\uc5d0 \ucd9c\uac15\ud558\uc600\ub2e4. 2001\ub144 \uacbd\ucc30\ub300\ud559 \uc870\uad50\uc218\uac00 \ub418\uc5c8\uace0, 2012\ub144 \uacbd\ucc30\ub300\ud559 \uc815\uad50\uc218\uac00 \ub418\uc5c8\ub2e4. 2012\ub144 12\uc6d4, \ub300\ud55c\ubbfc\uad6d \uad6d\uac00\uc815\ubcf4\uc6d0 \uc5ec\ub860 \uc870\uc791 \uc0ac\uac74\uc5d0 \ub300\ud55c \uacac\ud574\ub97c \ud45c\uba85\ud558\ub294 \uacfc\uc815\uc5d0\uc11c '\uacbd\ucc30\ub300\uc758 \uc815\uce58\uc801 \uc911\ub9bd\uc131\uc744 \uce68\ud574\ud560 \uc218 \uc788\ub2e4'\ub294 \uc774\uc720\ub85c \uacbd\ucc30\ub300\ud559 \uad50\uc218\uc9c1\uc744 \uc0ac\ud1f4\ud588\ub2e4.", "paragraph_answer": " 1989\ub144 \uacbd\ucc30\ub300\ud559 \uc878\uc5c5\uacfc \ub3d9\uc2dc\uc5d0 \uacbd\ucc30\uad00\uc5d0 \uc784\uc6a9\ub418\uc5c8\ub2e4. \uacbd\ucc30\uad00 \uc7ac\uc9c1 \uc911 1989\ub144\ubd80\ud130 1993\ub144\uae4c\uc9c0\ub294 \uc758\ubb34\ubcf5\ubb34\ub85c\uc368 \uc804\uacbd \uc18c\ub300\uc7a5\uc744 \ud55c\ud6c4 \uacbd\ucc30\uad00\uc73c\ub85c \uadfc\ubb34\ud558\uc600\uace0 1993\ubd80\ud1301997\ub144\uae4c\uc9c0\ub294 \uc601\uad6d\uc5d0\uc11c \uc11d\uc0ac\ud559\uc704\uc640 \ubc15\uc0ac\ud559\uc704\ub97c \ucde8\ub4dd\ud558\uc600\ub2e4. 1998\ub144 \uad11\uc6b4\ub300\ud559\uad50\uc640 \ud55c\uad6d\uc678\uad6d\uc5b4\ub300\ud559\uad50 \uad50\uc218\ub85c \ucd9c\uac15\ud558\uace0, \uacbd\ucc30\uad00\uc5d0\uc11c \ud1f4\uc9c1\ud55c \uc774\ud6c4\uc5d0\ub294 \uacbd\ucc30\ub300\ud559\uc758 \uac15\uc0ac\uac00 \ub418\uc5c8\uc73c\uba70 \uc5f0\uc138\ub300\ud559\uad50, \uc544\uc8fc\ub300\ud559\uad50, \uacbd\uae30\ub300\ud559\uad50 \ub4f1\uc5d0 \ucd9c\uac15\ud558\uc600\ub2e4. 2001\ub144 \uacbd\ucc30\ub300\ud559 \uc870\uad50\uc218\uac00 \ub418\uc5c8\uace0, 2012\ub144 \uacbd\ucc30\ub300\ud559 \uc815\uad50\uc218\uac00 \ub418\uc5c8\ub2e4. 2012\ub144 12\uc6d4, \ub300\ud55c\ubbfc\uad6d \uad6d\uac00\uc815\ubcf4\uc6d0 \uc5ec\ub860 \uc870\uc791 \uc0ac\uac74\uc5d0 \ub300\ud55c \uacac\ud574\ub97c \ud45c\uba85\ud558\ub294 \uacfc\uc815\uc5d0\uc11c '\uacbd\ucc30\ub300\uc758 \uc815\uce58\uc801 \uc911\ub9bd\uc131\uc744 \uce68\ud574\ud560 \uc218 \uc788\ub2e4'\ub294 \uc774\uc720\ub85c \uacbd\ucc30\ub300\ud559 \uad50\uc218\uc9c1\uc744 \uc0ac\ud1f4\ud588\ub2e4.", "sentence_answer": " 1989\ub144 \uacbd\ucc30\ub300\ud559 \uc878\uc5c5\uacfc \ub3d9\uc2dc\uc5d0 \uacbd\ucc30\uad00\uc5d0 \uc784\uc6a9\ub418\uc5c8\ub2e4.", "paragraph_id": "6568397-0-0", "paragraph_question": "question: \ud45c\ucc3d\uc6d0\uc774 \uacbd\ucc30\uad00\uc5d0 \uc784\uc6a9\ub41c \uc5f0\ub3c4\ub294?, context: 1989\ub144 \uacbd\ucc30\ub300\ud559 \uc878\uc5c5\uacfc \ub3d9\uc2dc\uc5d0 \uacbd\ucc30\uad00\uc5d0 \uc784\uc6a9\ub418\uc5c8\ub2e4. \uacbd\ucc30\uad00 \uc7ac\uc9c1 \uc911 1989\ub144\ubd80\ud130 1993\ub144\uae4c\uc9c0\ub294 \uc758\ubb34\ubcf5\ubb34\ub85c\uc368 \uc804\uacbd \uc18c\ub300\uc7a5\uc744 \ud55c\ud6c4 \uacbd\ucc30\uad00\uc73c\ub85c \uadfc\ubb34\ud558\uc600\uace0 1993\ubd80\ud1301997\ub144\uae4c\uc9c0\ub294 \uc601\uad6d\uc5d0\uc11c \uc11d\uc0ac\ud559\uc704\uc640 \ubc15\uc0ac\ud559\uc704\ub97c \ucde8\ub4dd\ud558\uc600\ub2e4. 1998\ub144 \uad11\uc6b4\ub300\ud559\uad50\uc640 \ud55c\uad6d\uc678\uad6d\uc5b4\ub300\ud559\uad50 \uad50\uc218\ub85c \ucd9c\uac15\ud558\uace0, \uacbd\ucc30\uad00\uc5d0\uc11c \ud1f4\uc9c1\ud55c \uc774\ud6c4\uc5d0\ub294 \uacbd\ucc30\ub300\ud559\uc758 \uac15\uc0ac\uac00 \ub418\uc5c8\uc73c\uba70 \uc5f0\uc138\ub300\ud559\uad50, \uc544\uc8fc\ub300\ud559\uad50, \uacbd\uae30\ub300\ud559\uad50 \ub4f1\uc5d0 \ucd9c\uac15\ud558\uc600\ub2e4. 2001\ub144 \uacbd\ucc30\ub300\ud559 \uc870\uad50\uc218\uac00 \ub418\uc5c8\uace0, 2012\ub144 \uacbd\ucc30\ub300\ud559 \uc815\uad50\uc218\uac00 \ub418\uc5c8\ub2e4. 2012\ub144 12\uc6d4, \ub300\ud55c\ubbfc\uad6d \uad6d\uac00\uc815\ubcf4\uc6d0 \uc5ec\ub860 \uc870\uc791 \uc0ac\uac74\uc5d0 \ub300\ud55c \uacac\ud574\ub97c \ud45c\uba85\ud558\ub294 \uacfc\uc815\uc5d0\uc11c '\uacbd\ucc30\ub300\uc758 \uc815\uce58\uc801 \uc911\ub9bd\uc131\uc744 \uce68\ud574\ud560 \uc218 \uc788\ub2e4'\ub294 \uc774\uc720\ub85c \uacbd\ucc30\ub300\ud559 \uad50\uc218\uc9c1\uc744 \uc0ac\ud1f4\ud588\ub2e4."} {"question": "2009\ub144 8\uc6d4 10\uc77c \ub370\uc774\ube0c\uac00 \uc2dc\uc560\ud2c0 \uacf5\uc5f0 \uc911 \uc785\uc740 \ubd80\uc0c1\uc740?", "paragraph": "2009\ub144 5\uc6d4 12\uc77c, \ub514\ud398\uc26c \ubaa8\ub4dc\uac00 Universe \ud22c\uc5b4\ub85c \uadf8\ub9ac\uc2a4\uc758 \uc544\ud14c\ub124\uc5d0\uc11c \ubb34\ub300\uc5d0 \uc11c\uae30 \ubc14\ub85c \uc9c1\uc804, \ub370\uc774\ube0c\ub294 \uadf8\uc758 \ub300\uae30\uc2e4\uc5d0\uc11c \uc4f0\ub7ec\uc84c\ub2e4. \uadf8\ub294 \ubcd1\uc6d0\uc73c\ub85c \uae09\ud788 \uc774\uc1a1\ub418\uc5c8\uace0, \uc704\uc7a5\uc5fc \uc9c4\ub2e8\uc744 \ubc1b\ub294\ub2e4. \ucd94\uac00 \uac80\uc0ac\ub85c \uadf8\uc758 \ubc29\uad11\uc5d0\uc11c \uc2ec\ud558\uc9c0\ub294 \uc54a\uc740 \uc545\uc131 \uc885\uc591\uc774 \ubc1c\uacac\ub418\uc5b4 \uc218\ub9ce\uc740 \uc77c\uc815\ub4e4\uc774 \uc5f0\uae30\ub418\uac70\ub098 \ucde8\uc18c\ub418\uc5c8\ub2e4. \uadf8\uc758 \uc885\uc591\uc740 \uc131\uacf5\uc801\uc73c\ub85c \uc808\uc81c\ub418\uc5c8\ub2e4. \uc758\uc0ac\uc758 \uc9c0\uc2dc\ub85c \uc5bc\ub9c8 \uac04\uc758 \ud68c\ubcf5 \uae30\uac04\uc744 \uac70\uce5c \ud6c4, 6\uc6d4 8\uc77c \ub3c5\uc77c \ub77c\uc774\ud504\uce58\ud788\uc5d0\uc11c \ud22c\uc5b4\uac00 \uc7ac\uac1c\ub418\uc5c8\ub2e4. \uc774\ud6c4 2009\ub144 7\uc6d4 9\uc77c \uc2a4\ud398\uc778 \ube4c\ubc14\uc624 \uacf5\uc5f0 \uc911 \uadf8\uc758 \uc885\uc544\ub9ac \uadfc\uc721\uc774 \ud30c\uc5f4\ub418\uc5b4 \ub450 \uac1c\uc758 \uacf5\uc5f0\uc774 \ucde8\uc18c\ub418\uc5c8\ub2e4. 2\uc8fc \uac04\uc758 \ud734\uc2dd \ud6c4 \ub514\ud398\uc26c \ubaa8\ub4dc\ub294 \ubd81\ubbf8 \uc77c\uc815\uc744 \uc704\ud574 \ud22c\uc5b4\ub97c \uc7ac\uac1c\ud588\ub2e4. 2009\ub144 8\uc6d4 10\uc77c \uc2dc\uc560\ud2c0\uc5d0\uc11c\uc758 \uacf5\uc5f0 \uc911, \uadf8\ub294 \uc774\ubc88\uc5d0\ub294 \uc131\ub300 \ubd80\uc0c1\uc744 \uc785\ub294\ub2e4. \uc758\uc0ac\ub4e4\uc740 \uadf8\uc5d0\uac8c \ud734\uc2dd\uc744 \uad8c\ud588\uc73c\uba70 \uc774\ub85c \uc778\ud574 \ub610 2\uac1c\uc758 \uacf5\uc5f0\uc774 \ucde8\uc18c\ub418\uc5c8\ub2e4. \ub514\ud398\uc26c \ubaa8\ub4dc\ub294 8\uc6d4 16\uc77c \ub85c\uc2a4\uc564\uc824\ub808\uc2a4\uc5d0\uc11c \ubb34\ub300\uc5d0 \ubcf5\uadc0\ud558\uac8c \ub41c\ub2e4. \ud32c\ub4e4\uacfc \uc5b8\ub860\uc5d0 \ub530\ub974\uba74, \uadf8\uc758 \uac74\uac15 \uc0c1\uc758 \ucc28\uc9c8\uc5d0\ub3c4 \ubd88\uad6c\ud558\uace0 \uacf5\uc5f0\uc740 \uc5ec\ub290 \ub54c\ub9cc\ud07c\uc774\ub098 \ud6cc\ub96d\ud588\ub2e4\uace0 \ud55c\ub2e4. 2009\ub144 10\uc6d4 \ub77c\ud2f4 \uc544\uba54\ub9ac\uce74 \ud22c\uc5b4\uc758 \uba55\uc2dc\ucf54, \ucf54\uc2a4\ud0c0 \ub9ac\uce74, \ucf5c\ub86c\ube44\uc544, \ud398\ub8e8, \uce60\ub808 \uadf8\ub9ac\uace0 \uc544\ub974\ud5e8\ud2f0\ub098 \uacf5\uc5f0\uc5d0\uc11c \uadf8\ub294 \uc5b4\ub5a0\ud55c \uc774\uc0c1 \uc99d\ud6c4\ub3c4 \ubcf4\uc774\uc9c0 \uc54a\uc558\ub2e4\uace0 \ud55c\ub2e4.", "answer": "\uc131\ub300 \ubd80\uc0c1", "sentence": "2009\ub144 8\uc6d4 10\uc77c \uc2dc\uc560\ud2c0\uc5d0\uc11c\uc758 \uacf5\uc5f0 \uc911, \uadf8\ub294 \uc774\ubc88\uc5d0\ub294 \uc131\ub300 \ubd80\uc0c1 \uc744 \uc785\ub294\ub2e4.", "paragraph_sentence": "2009\ub144 5\uc6d4 12\uc77c, \ub514\ud398\uc26c \ubaa8\ub4dc\uac00 Universe \ud22c\uc5b4\ub85c \uadf8\ub9ac\uc2a4\uc758 \uc544\ud14c\ub124\uc5d0\uc11c \ubb34\ub300\uc5d0 \uc11c\uae30 \ubc14\ub85c \uc9c1\uc804, \ub370\uc774\ube0c\ub294 \uadf8\uc758 \ub300\uae30\uc2e4\uc5d0\uc11c \uc4f0\ub7ec\uc84c\ub2e4. \uadf8\ub294 \ubcd1\uc6d0\uc73c\ub85c \uae09\ud788 \uc774\uc1a1\ub418\uc5c8\uace0, \uc704\uc7a5\uc5fc \uc9c4\ub2e8\uc744 \ubc1b\ub294\ub2e4. \ucd94\uac00 \uac80\uc0ac\ub85c \uadf8\uc758 \ubc29\uad11\uc5d0\uc11c \uc2ec\ud558\uc9c0\ub294 \uc54a\uc740 \uc545\uc131 \uc885\uc591\uc774 \ubc1c\uacac\ub418\uc5b4 \uc218\ub9ce\uc740 \uc77c\uc815\ub4e4\uc774 \uc5f0\uae30\ub418\uac70\ub098 \ucde8\uc18c\ub418\uc5c8\ub2e4. \uadf8\uc758 \uc885\uc591\uc740 \uc131\uacf5\uc801\uc73c\ub85c \uc808\uc81c\ub418\uc5c8\ub2e4. \uc758\uc0ac\uc758 \uc9c0\uc2dc\ub85c \uc5bc\ub9c8 \uac04\uc758 \ud68c\ubcf5 \uae30\uac04\uc744 \uac70\uce5c \ud6c4, 6\uc6d4 8\uc77c \ub3c5\uc77c \ub77c\uc774\ud504\uce58\ud788\uc5d0\uc11c \ud22c\uc5b4\uac00 \uc7ac\uac1c\ub418\uc5c8\ub2e4. \uc774\ud6c4 2009\ub144 7\uc6d4 9\uc77c \uc2a4\ud398\uc778 \ube4c\ubc14\uc624 \uacf5\uc5f0 \uc911 \uadf8\uc758 \uc885\uc544\ub9ac \uadfc\uc721\uc774 \ud30c\uc5f4\ub418\uc5b4 \ub450 \uac1c\uc758 \uacf5\uc5f0\uc774 \ucde8\uc18c\ub418\uc5c8\ub2e4. 2\uc8fc \uac04\uc758 \ud734\uc2dd \ud6c4 \ub514\ud398\uc26c \ubaa8\ub4dc\ub294 \ubd81\ubbf8 \uc77c\uc815\uc744 \uc704\ud574 \ud22c\uc5b4\ub97c \uc7ac\uac1c\ud588\ub2e4. 2009\ub144 8\uc6d4 10\uc77c \uc2dc\uc560\ud2c0\uc5d0\uc11c\uc758 \uacf5\uc5f0 \uc911, \uadf8\ub294 \uc774\ubc88\uc5d0\ub294 \uc131\ub300 \ubd80\uc0c1 \uc744 \uc785\ub294\ub2e4. \uc758\uc0ac\ub4e4\uc740 \uadf8\uc5d0\uac8c \ud734\uc2dd\uc744 \uad8c\ud588\uc73c\uba70 \uc774\ub85c \uc778\ud574 \ub610 2\uac1c\uc758 \uacf5\uc5f0\uc774 \ucde8\uc18c\ub418\uc5c8\ub2e4. \ub514\ud398\uc26c \ubaa8\ub4dc\ub294 8\uc6d4 16\uc77c \ub85c\uc2a4\uc564\uc824\ub808\uc2a4\uc5d0\uc11c \ubb34\ub300\uc5d0 \ubcf5\uadc0\ud558\uac8c \ub41c\ub2e4. \ud32c\ub4e4\uacfc \uc5b8\ub860\uc5d0 \ub530\ub974\uba74, \uadf8\uc758 \uac74\uac15 \uc0c1\uc758 \ucc28\uc9c8\uc5d0\ub3c4 \ubd88\uad6c\ud558\uace0 \uacf5\uc5f0\uc740 \uc5ec\ub290 \ub54c\ub9cc\ud07c\uc774\ub098 \ud6cc\ub96d\ud588\ub2e4\uace0 \ud55c\ub2e4. 2009\ub144 10\uc6d4 \ub77c\ud2f4 \uc544\uba54\ub9ac\uce74 \ud22c\uc5b4\uc758 \uba55\uc2dc\ucf54, \ucf54\uc2a4\ud0c0 \ub9ac\uce74, \ucf5c\ub86c\ube44\uc544, \ud398\ub8e8, \uce60\ub808 \uadf8\ub9ac\uace0 \uc544\ub974\ud5e8\ud2f0\ub098 \uacf5\uc5f0\uc5d0\uc11c \uadf8\ub294 \uc5b4\ub5a0\ud55c \uc774\uc0c1 \uc99d\ud6c4\ub3c4 \ubcf4\uc774\uc9c0 \uc54a\uc558\ub2e4\uace0 \ud55c\ub2e4.", "paragraph_answer": "2009\ub144 5\uc6d4 12\uc77c, \ub514\ud398\uc26c \ubaa8\ub4dc\uac00 Universe \ud22c\uc5b4\ub85c \uadf8\ub9ac\uc2a4\uc758 \uc544\ud14c\ub124\uc5d0\uc11c \ubb34\ub300\uc5d0 \uc11c\uae30 \ubc14\ub85c \uc9c1\uc804, \ub370\uc774\ube0c\ub294 \uadf8\uc758 \ub300\uae30\uc2e4\uc5d0\uc11c \uc4f0\ub7ec\uc84c\ub2e4. \uadf8\ub294 \ubcd1\uc6d0\uc73c\ub85c \uae09\ud788 \uc774\uc1a1\ub418\uc5c8\uace0, \uc704\uc7a5\uc5fc \uc9c4\ub2e8\uc744 \ubc1b\ub294\ub2e4. \ucd94\uac00 \uac80\uc0ac\ub85c \uadf8\uc758 \ubc29\uad11\uc5d0\uc11c \uc2ec\ud558\uc9c0\ub294 \uc54a\uc740 \uc545\uc131 \uc885\uc591\uc774 \ubc1c\uacac\ub418\uc5b4 \uc218\ub9ce\uc740 \uc77c\uc815\ub4e4\uc774 \uc5f0\uae30\ub418\uac70\ub098 \ucde8\uc18c\ub418\uc5c8\ub2e4. \uadf8\uc758 \uc885\uc591\uc740 \uc131\uacf5\uc801\uc73c\ub85c \uc808\uc81c\ub418\uc5c8\ub2e4. \uc758\uc0ac\uc758 \uc9c0\uc2dc\ub85c \uc5bc\ub9c8 \uac04\uc758 \ud68c\ubcf5 \uae30\uac04\uc744 \uac70\uce5c \ud6c4, 6\uc6d4 8\uc77c \ub3c5\uc77c \ub77c\uc774\ud504\uce58\ud788\uc5d0\uc11c \ud22c\uc5b4\uac00 \uc7ac\uac1c\ub418\uc5c8\ub2e4. \uc774\ud6c4 2009\ub144 7\uc6d4 9\uc77c \uc2a4\ud398\uc778 \ube4c\ubc14\uc624 \uacf5\uc5f0 \uc911 \uadf8\uc758 \uc885\uc544\ub9ac \uadfc\uc721\uc774 \ud30c\uc5f4\ub418\uc5b4 \ub450 \uac1c\uc758 \uacf5\uc5f0\uc774 \ucde8\uc18c\ub418\uc5c8\ub2e4. 2\uc8fc \uac04\uc758 \ud734\uc2dd \ud6c4 \ub514\ud398\uc26c \ubaa8\ub4dc\ub294 \ubd81\ubbf8 \uc77c\uc815\uc744 \uc704\ud574 \ud22c\uc5b4\ub97c \uc7ac\uac1c\ud588\ub2e4. 2009\ub144 8\uc6d4 10\uc77c \uc2dc\uc560\ud2c0\uc5d0\uc11c\uc758 \uacf5\uc5f0 \uc911, \uadf8\ub294 \uc774\ubc88\uc5d0\ub294 \uc131\ub300 \ubd80\uc0c1 \uc744 \uc785\ub294\ub2e4. \uc758\uc0ac\ub4e4\uc740 \uadf8\uc5d0\uac8c \ud734\uc2dd\uc744 \uad8c\ud588\uc73c\uba70 \uc774\ub85c \uc778\ud574 \ub610 2\uac1c\uc758 \uacf5\uc5f0\uc774 \ucde8\uc18c\ub418\uc5c8\ub2e4. \ub514\ud398\uc26c \ubaa8\ub4dc\ub294 8\uc6d4 16\uc77c \ub85c\uc2a4\uc564\uc824\ub808\uc2a4\uc5d0\uc11c \ubb34\ub300\uc5d0 \ubcf5\uadc0\ud558\uac8c \ub41c\ub2e4. \ud32c\ub4e4\uacfc \uc5b8\ub860\uc5d0 \ub530\ub974\uba74, \uadf8\uc758 \uac74\uac15 \uc0c1\uc758 \ucc28\uc9c8\uc5d0\ub3c4 \ubd88\uad6c\ud558\uace0 \uacf5\uc5f0\uc740 \uc5ec\ub290 \ub54c\ub9cc\ud07c\uc774\ub098 \ud6cc\ub96d\ud588\ub2e4\uace0 \ud55c\ub2e4. 2009\ub144 10\uc6d4 \ub77c\ud2f4 \uc544\uba54\ub9ac\uce74 \ud22c\uc5b4\uc758 \uba55\uc2dc\ucf54, \ucf54\uc2a4\ud0c0 \ub9ac\uce74, \ucf5c\ub86c\ube44\uc544, \ud398\ub8e8, \uce60\ub808 \uadf8\ub9ac\uace0 \uc544\ub974\ud5e8\ud2f0\ub098 \uacf5\uc5f0\uc5d0\uc11c \uadf8\ub294 \uc5b4\ub5a0\ud55c \uc774\uc0c1 \uc99d\ud6c4\ub3c4 \ubcf4\uc774\uc9c0 \uc54a\uc558\ub2e4\uace0 \ud55c\ub2e4.", "sentence_answer": "2009\ub144 8\uc6d4 10\uc77c \uc2dc\uc560\ud2c0\uc5d0\uc11c\uc758 \uacf5\uc5f0 \uc911, \uadf8\ub294 \uc774\ubc88\uc5d0\ub294 \uc131\ub300 \ubd80\uc0c1 \uc744 \uc785\ub294\ub2e4.", "paragraph_id": "6578306-4-2", "paragraph_question": "question: 2009\ub144 8\uc6d4 10\uc77c \ub370\uc774\ube0c\uac00 \uc2dc\uc560\ud2c0 \uacf5\uc5f0 \uc911 \uc785\uc740 \ubd80\uc0c1\uc740?, context: 2009\ub144 5\uc6d4 12\uc77c, \ub514\ud398\uc26c \ubaa8\ub4dc\uac00 Universe \ud22c\uc5b4\ub85c \uadf8\ub9ac\uc2a4\uc758 \uc544\ud14c\ub124\uc5d0\uc11c \ubb34\ub300\uc5d0 \uc11c\uae30 \ubc14\ub85c \uc9c1\uc804, \ub370\uc774\ube0c\ub294 \uadf8\uc758 \ub300\uae30\uc2e4\uc5d0\uc11c \uc4f0\ub7ec\uc84c\ub2e4. \uadf8\ub294 \ubcd1\uc6d0\uc73c\ub85c \uae09\ud788 \uc774\uc1a1\ub418\uc5c8\uace0, \uc704\uc7a5\uc5fc \uc9c4\ub2e8\uc744 \ubc1b\ub294\ub2e4. \ucd94\uac00 \uac80\uc0ac\ub85c \uadf8\uc758 \ubc29\uad11\uc5d0\uc11c \uc2ec\ud558\uc9c0\ub294 \uc54a\uc740 \uc545\uc131 \uc885\uc591\uc774 \ubc1c\uacac\ub418\uc5b4 \uc218\ub9ce\uc740 \uc77c\uc815\ub4e4\uc774 \uc5f0\uae30\ub418\uac70\ub098 \ucde8\uc18c\ub418\uc5c8\ub2e4. \uadf8\uc758 \uc885\uc591\uc740 \uc131\uacf5\uc801\uc73c\ub85c \uc808\uc81c\ub418\uc5c8\ub2e4. \uc758\uc0ac\uc758 \uc9c0\uc2dc\ub85c \uc5bc\ub9c8 \uac04\uc758 \ud68c\ubcf5 \uae30\uac04\uc744 \uac70\uce5c \ud6c4, 6\uc6d4 8\uc77c \ub3c5\uc77c \ub77c\uc774\ud504\uce58\ud788\uc5d0\uc11c \ud22c\uc5b4\uac00 \uc7ac\uac1c\ub418\uc5c8\ub2e4. \uc774\ud6c4 2009\ub144 7\uc6d4 9\uc77c \uc2a4\ud398\uc778 \ube4c\ubc14\uc624 \uacf5\uc5f0 \uc911 \uadf8\uc758 \uc885\uc544\ub9ac \uadfc\uc721\uc774 \ud30c\uc5f4\ub418\uc5b4 \ub450 \uac1c\uc758 \uacf5\uc5f0\uc774 \ucde8\uc18c\ub418\uc5c8\ub2e4. 2\uc8fc \uac04\uc758 \ud734\uc2dd \ud6c4 \ub514\ud398\uc26c \ubaa8\ub4dc\ub294 \ubd81\ubbf8 \uc77c\uc815\uc744 \uc704\ud574 \ud22c\uc5b4\ub97c \uc7ac\uac1c\ud588\ub2e4. 2009\ub144 8\uc6d4 10\uc77c \uc2dc\uc560\ud2c0\uc5d0\uc11c\uc758 \uacf5\uc5f0 \uc911, \uadf8\ub294 \uc774\ubc88\uc5d0\ub294 \uc131\ub300 \ubd80\uc0c1\uc744 \uc785\ub294\ub2e4. \uc758\uc0ac\ub4e4\uc740 \uadf8\uc5d0\uac8c \ud734\uc2dd\uc744 \uad8c\ud588\uc73c\uba70 \uc774\ub85c \uc778\ud574 \ub610 2\uac1c\uc758 \uacf5\uc5f0\uc774 \ucde8\uc18c\ub418\uc5c8\ub2e4. \ub514\ud398\uc26c \ubaa8\ub4dc\ub294 8\uc6d4 16\uc77c \ub85c\uc2a4\uc564\uc824\ub808\uc2a4\uc5d0\uc11c \ubb34\ub300\uc5d0 \ubcf5\uadc0\ud558\uac8c \ub41c\ub2e4. \ud32c\ub4e4\uacfc \uc5b8\ub860\uc5d0 \ub530\ub974\uba74, \uadf8\uc758 \uac74\uac15 \uc0c1\uc758 \ucc28\uc9c8\uc5d0\ub3c4 \ubd88\uad6c\ud558\uace0 \uacf5\uc5f0\uc740 \uc5ec\ub290 \ub54c\ub9cc\ud07c\uc774\ub098 \ud6cc\ub96d\ud588\ub2e4\uace0 \ud55c\ub2e4. 2009\ub144 10\uc6d4 \ub77c\ud2f4 \uc544\uba54\ub9ac\uce74 \ud22c\uc5b4\uc758 \uba55\uc2dc\ucf54, \ucf54\uc2a4\ud0c0 \ub9ac\uce74, \ucf5c\ub86c\ube44\uc544, \ud398\ub8e8, \uce60\ub808 \uadf8\ub9ac\uace0 \uc544\ub974\ud5e8\ud2f0\ub098 \uacf5\uc5f0\uc5d0\uc11c \uadf8\ub294 \uc5b4\ub5a0\ud55c \uc774\uc0c1 \uc99d\ud6c4\ub3c4 \ubcf4\uc774\uc9c0 \uc54a\uc558\ub2e4\uace0 \ud55c\ub2e4."} {"question": "\uc0ac\ub4dc \ubc30\uce58\uc5d0 \ub300\ud55c \uc804\ub7b5\uc801\ud658\uacbd\uc601\ud5a5\ud3c9\uac00\ub97c \uc704\ud574\uc11c\ub294, \ub9de\uad50\ud658 \ud615\uc2dd\uc758 \ubd80\uc9c0\ub97c \uc5b4\ub5a4 \ubc29\uc2dd\uc73c\ub85c \uc7ac \ud655\ubcf4\ud574\uc57c \ud558\ub098?", "paragraph": "\ud658\uacbd\uc601\ud5a5\ud3c9\uac00 \ubb38\uc81c\ub97c \uccad\uc640\ub300\uac00 \uc6d0\ud558\ub294 \ub300\ub85c \ud560 \uc218 \uc788\ub294 \ud569\ubc95\uc801\uc778 \ubc29\ubc95\uc740 \uc0ac\ub4dc \ubc30\uce58 \uacc4\ud68d\uc744 \uc804\uba74 \ubc31\uc9c0\ud654\ud558\uace0 \uc804\ub7b5\ud658\uacbd\uc601\ud5a5\ud3c9\uac00\ub97c \ubaa8\ub450 \ubc1b\uc744 \uc218 \uc788\ub3c4\ub85d \ud558\ub294 \uac83\uc774\uc9c0\ub9cc \uc774\ub97c \uc704\ud574\uc11c\ub294 \uc774\ubbf8 \uc57c\uc804 \ubc30\uce58\ub41c \ub808\uc774\ub354\uc640 \ubc1c\uc0ac\ub300 \ub4f1 \uc0ac\ub4dc \uc7a5\ube44\ub4e4\uc744 \uaebc\ub0b4\uc57c \ud558\uace0, \ubd80\uc9c0\ub3c4 \ub86f\ub370\uc640\uc758 \ud1a0\uc9c0 \ub9de\uad50\ud658\uc774 \uc544\ub2cc \ub9e4\ub9e4 \ubc29\uc2dd\uc73c\ub85c \ub2e4\uc2dc \ud655\ubcf4\ud574\uc57c \ud558\ub294 \ub4f1 \ubcf5\uc7a1\ud55c \ubb38\uc81c\uac00 \uc5ec\ub7ff \ubc1c\uc0dd\ud55c\ub2e4. \uccad\uc640\ub300\uc5d0\uc11c \uc694\uad6c\ud558\ub294 \"\ud604 \ubc30\uce58 \uc0c1\ud0dc\ub294 \uc720\uc9c0\ud558\uba74\uc11c \uc804\ub7b5\ud658\uacbd\uc601\ud5a5\ud3c9\uac00\ub97c \uc81c\ub300\ub85c \uac70\uce58\ub77c\"\ub294 \uac83\uc774 \ubc95\uc801\uc73c\ub85c \ubd88\uac00\ub2a5\ud55c \uc774\uc720\ub294 \ud658\uacbd\uc601\ud5a5\ud3c9\uac00\ubc95\uacfc \uc2dc\ud589\ub839\uc5d0 \ub530\ub974\uba74 \uad6d\ubc29\uc7a5\uad00\uc774 \uc0ac\uc5c5\uacc4\ud68d\uc744 \uc2b9\uc778\ud558\uae30 \uc804, \uc989 \ubd80\uc9c0\ub97c \ud655\ubcf4\ud558\uae30 \uc804 \uacc4\ud68d \ub2e8\uacc4\uc5d0 \uc2e4\uc2dc\ud558\ub294 \uac83\uc774\uace0 \uc0ac\ub4dc\ub294 \uadf8 \ub2e8\uacc4\ub97c \ud55c\ucc38 \uc9c0\ub098 \uc774\ubbf8 \ubc30\uce58\u00b7\uc6b4\uc6a9\ub418\uace0 \uc788\uc73c\uba70 \uc0ac\ub4dc \ubd80\uc9c0\ucc98\ub7fc \ub9e4\uc785\uc774\ub098 \uc218\uc6a9 \ubc29\uc2dd\uc774 \uc544\ub2cc, \ud1a0\uc9c0 \ub9de\uad50\ud658 \ubc29\uc2dd\uc73c\ub85c \ud655\ubcf4\ub41c \ub545\uc758 \uacbd\uc6b0\uc5d0\ub294 \uc544\uc608 \uc804\ub7b5\ud658\uacbd\uc601\ud5a5\ud3c9\uac00 \ub300\uc0c1\uc774 \uc544\ub2c8\ub77c\ub294 \uac83\uc774 \uad6d\ubc29\ubd80 \uc124\uba85\uc774\ub2e4.", "answer": "\ub9e4\ub9e4", "sentence": "\ud658\uacbd\uc601\ud5a5\ud3c9\uac00 \ubb38\uc81c\ub97c \uccad\uc640\ub300\uac00 \uc6d0\ud558\ub294 \ub300\ub85c \ud560 \uc218 \uc788\ub294 \ud569\ubc95\uc801\uc778 \ubc29\ubc95\uc740 \uc0ac\ub4dc \ubc30\uce58 \uacc4\ud68d\uc744 \uc804\uba74 \ubc31\uc9c0\ud654\ud558\uace0 \uc804\ub7b5\ud658\uacbd\uc601\ud5a5\ud3c9\uac00\ub97c \ubaa8\ub450 \ubc1b\uc744 \uc218 \uc788\ub3c4\ub85d \ud558\ub294 \uac83\uc774\uc9c0\ub9cc \uc774\ub97c \uc704\ud574\uc11c\ub294 \uc774\ubbf8 \uc57c\uc804 \ubc30\uce58\ub41c \ub808\uc774\ub354\uc640 \ubc1c\uc0ac\ub300 \ub4f1 \uc0ac\ub4dc \uc7a5\ube44\ub4e4\uc744 \uaebc\ub0b4\uc57c \ud558\uace0, \ubd80\uc9c0\ub3c4 \ub86f\ub370\uc640\uc758 \ud1a0\uc9c0 \ub9de\uad50\ud658\uc774 \uc544\ub2cc \ub9e4\ub9e4 \ubc29\uc2dd\uc73c\ub85c \ub2e4\uc2dc \ud655\ubcf4\ud574\uc57c \ud558\ub294 \ub4f1 \ubcf5\uc7a1\ud55c \ubb38\uc81c\uac00 \uc5ec\ub7ff \ubc1c\uc0dd\ud55c\ub2e4.", "paragraph_sentence": " \ud658\uacbd\uc601\ud5a5\ud3c9\uac00 \ubb38\uc81c\ub97c \uccad\uc640\ub300\uac00 \uc6d0\ud558\ub294 \ub300\ub85c \ud560 \uc218 \uc788\ub294 \ud569\ubc95\uc801\uc778 \ubc29\ubc95\uc740 \uc0ac\ub4dc \ubc30\uce58 \uacc4\ud68d\uc744 \uc804\uba74 \ubc31\uc9c0\ud654\ud558\uace0 \uc804\ub7b5\ud658\uacbd\uc601\ud5a5\ud3c9\uac00\ub97c \ubaa8\ub450 \ubc1b\uc744 \uc218 \uc788\ub3c4\ub85d \ud558\ub294 \uac83\uc774\uc9c0\ub9cc \uc774\ub97c \uc704\ud574\uc11c\ub294 \uc774\ubbf8 \uc57c\uc804 \ubc30\uce58\ub41c \ub808\uc774\ub354\uc640 \ubc1c\uc0ac\ub300 \ub4f1 \uc0ac\ub4dc \uc7a5\ube44\ub4e4\uc744 \uaebc\ub0b4\uc57c \ud558\uace0, \ubd80\uc9c0\ub3c4 \ub86f\ub370\uc640\uc758 \ud1a0\uc9c0 \ub9de\uad50\ud658\uc774 \uc544\ub2cc \ub9e4\ub9e4 \ubc29\uc2dd\uc73c\ub85c \ub2e4\uc2dc \ud655\ubcf4\ud574\uc57c \ud558\ub294 \ub4f1 \ubcf5\uc7a1\ud55c \ubb38\uc81c\uac00 \uc5ec\ub7ff \ubc1c\uc0dd\ud55c\ub2e4. \uccad\uc640\ub300\uc5d0\uc11c \uc694\uad6c\ud558\ub294 \"\ud604 \ubc30\uce58 \uc0c1\ud0dc\ub294 \uc720\uc9c0\ud558\uba74\uc11c \uc804\ub7b5\ud658\uacbd\uc601\ud5a5\ud3c9\uac00\ub97c \uc81c\ub300\ub85c \uac70\uce58\ub77c\"\ub294 \uac83\uc774 \ubc95\uc801\uc73c\ub85c \ubd88\uac00\ub2a5\ud55c \uc774\uc720\ub294 \ud658\uacbd\uc601\ud5a5\ud3c9\uac00\ubc95\uacfc \uc2dc\ud589\ub839\uc5d0 \ub530\ub974\uba74 \uad6d\ubc29\uc7a5\uad00\uc774 \uc0ac\uc5c5\uacc4\ud68d\uc744 \uc2b9\uc778\ud558\uae30 \uc804, \uc989 \ubd80\uc9c0\ub97c \ud655\ubcf4\ud558\uae30 \uc804 \uacc4\ud68d \ub2e8\uacc4\uc5d0 \uc2e4\uc2dc\ud558\ub294 \uac83\uc774\uace0 \uc0ac\ub4dc\ub294 \uadf8 \ub2e8\uacc4\ub97c \ud55c\ucc38 \uc9c0\ub098 \uc774\ubbf8 \ubc30\uce58\u00b7\uc6b4\uc6a9\ub418\uace0 \uc788\uc73c\uba70 \uc0ac\ub4dc \ubd80\uc9c0\ucc98\ub7fc \ub9e4\uc785\uc774\ub098 \uc218\uc6a9 \ubc29\uc2dd\uc774 \uc544\ub2cc, \ud1a0\uc9c0 \ub9de\uad50\ud658 \ubc29\uc2dd\uc73c\ub85c \ud655\ubcf4\ub41c \ub545\uc758 \uacbd\uc6b0\uc5d0\ub294 \uc544\uc608 \uc804\ub7b5\ud658\uacbd\uc601\ud5a5\ud3c9\uac00 \ub300\uc0c1\uc774 \uc544\ub2c8\ub77c\ub294 \uac83\uc774 \uad6d\ubc29\ubd80 \uc124\uba85\uc774\ub2e4.", "paragraph_answer": "\ud658\uacbd\uc601\ud5a5\ud3c9\uac00 \ubb38\uc81c\ub97c \uccad\uc640\ub300\uac00 \uc6d0\ud558\ub294 \ub300\ub85c \ud560 \uc218 \uc788\ub294 \ud569\ubc95\uc801\uc778 \ubc29\ubc95\uc740 \uc0ac\ub4dc \ubc30\uce58 \uacc4\ud68d\uc744 \uc804\uba74 \ubc31\uc9c0\ud654\ud558\uace0 \uc804\ub7b5\ud658\uacbd\uc601\ud5a5\ud3c9\uac00\ub97c \ubaa8\ub450 \ubc1b\uc744 \uc218 \uc788\ub3c4\ub85d \ud558\ub294 \uac83\uc774\uc9c0\ub9cc \uc774\ub97c \uc704\ud574\uc11c\ub294 \uc774\ubbf8 \uc57c\uc804 \ubc30\uce58\ub41c \ub808\uc774\ub354\uc640 \ubc1c\uc0ac\ub300 \ub4f1 \uc0ac\ub4dc \uc7a5\ube44\ub4e4\uc744 \uaebc\ub0b4\uc57c \ud558\uace0, \ubd80\uc9c0\ub3c4 \ub86f\ub370\uc640\uc758 \ud1a0\uc9c0 \ub9de\uad50\ud658\uc774 \uc544\ub2cc \ub9e4\ub9e4 \ubc29\uc2dd\uc73c\ub85c \ub2e4\uc2dc \ud655\ubcf4\ud574\uc57c \ud558\ub294 \ub4f1 \ubcf5\uc7a1\ud55c \ubb38\uc81c\uac00 \uc5ec\ub7ff \ubc1c\uc0dd\ud55c\ub2e4. \uccad\uc640\ub300\uc5d0\uc11c \uc694\uad6c\ud558\ub294 \"\ud604 \ubc30\uce58 \uc0c1\ud0dc\ub294 \uc720\uc9c0\ud558\uba74\uc11c \uc804\ub7b5\ud658\uacbd\uc601\ud5a5\ud3c9\uac00\ub97c \uc81c\ub300\ub85c \uac70\uce58\ub77c\"\ub294 \uac83\uc774 \ubc95\uc801\uc73c\ub85c \ubd88\uac00\ub2a5\ud55c \uc774\uc720\ub294 \ud658\uacbd\uc601\ud5a5\ud3c9\uac00\ubc95\uacfc \uc2dc\ud589\ub839\uc5d0 \ub530\ub974\uba74 \uad6d\ubc29\uc7a5\uad00\uc774 \uc0ac\uc5c5\uacc4\ud68d\uc744 \uc2b9\uc778\ud558\uae30 \uc804, \uc989 \ubd80\uc9c0\ub97c \ud655\ubcf4\ud558\uae30 \uc804 \uacc4\ud68d \ub2e8\uacc4\uc5d0 \uc2e4\uc2dc\ud558\ub294 \uac83\uc774\uace0 \uc0ac\ub4dc\ub294 \uadf8 \ub2e8\uacc4\ub97c \ud55c\ucc38 \uc9c0\ub098 \uc774\ubbf8 \ubc30\uce58\u00b7\uc6b4\uc6a9\ub418\uace0 \uc788\uc73c\uba70 \uc0ac\ub4dc \ubd80\uc9c0\ucc98\ub7fc \ub9e4\uc785\uc774\ub098 \uc218\uc6a9 \ubc29\uc2dd\uc774 \uc544\ub2cc, \ud1a0\uc9c0 \ub9de\uad50\ud658 \ubc29\uc2dd\uc73c\ub85c \ud655\ubcf4\ub41c \ub545\uc758 \uacbd\uc6b0\uc5d0\ub294 \uc544\uc608 \uc804\ub7b5\ud658\uacbd\uc601\ud5a5\ud3c9\uac00 \ub300\uc0c1\uc774 \uc544\ub2c8\ub77c\ub294 \uac83\uc774 \uad6d\ubc29\ubd80 \uc124\uba85\uc774\ub2e4.", "sentence_answer": "\ud658\uacbd\uc601\ud5a5\ud3c9\uac00 \ubb38\uc81c\ub97c \uccad\uc640\ub300\uac00 \uc6d0\ud558\ub294 \ub300\ub85c \ud560 \uc218 \uc788\ub294 \ud569\ubc95\uc801\uc778 \ubc29\ubc95\uc740 \uc0ac\ub4dc \ubc30\uce58 \uacc4\ud68d\uc744 \uc804\uba74 \ubc31\uc9c0\ud654\ud558\uace0 \uc804\ub7b5\ud658\uacbd\uc601\ud5a5\ud3c9\uac00\ub97c \ubaa8\ub450 \ubc1b\uc744 \uc218 \uc788\ub3c4\ub85d \ud558\ub294 \uac83\uc774\uc9c0\ub9cc \uc774\ub97c \uc704\ud574\uc11c\ub294 \uc774\ubbf8 \uc57c\uc804 \ubc30\uce58\ub41c \ub808\uc774\ub354\uc640 \ubc1c\uc0ac\ub300 \ub4f1 \uc0ac\ub4dc \uc7a5\ube44\ub4e4\uc744 \uaebc\ub0b4\uc57c \ud558\uace0, \ubd80\uc9c0\ub3c4 \ub86f\ub370\uc640\uc758 \ud1a0\uc9c0 \ub9de\uad50\ud658\uc774 \uc544\ub2cc \ub9e4\ub9e4 \ubc29\uc2dd\uc73c\ub85c \ub2e4\uc2dc \ud655\ubcf4\ud574\uc57c \ud558\ub294 \ub4f1 \ubcf5\uc7a1\ud55c \ubb38\uc81c\uac00 \uc5ec\ub7ff \ubc1c\uc0dd\ud55c\ub2e4.", "paragraph_id": "6486360-34-0", "paragraph_question": "question: \uc0ac\ub4dc \ubc30\uce58\uc5d0 \ub300\ud55c \uc804\ub7b5\uc801\ud658\uacbd\uc601\ud5a5\ud3c9\uac00\ub97c \uc704\ud574\uc11c\ub294, \ub9de\uad50\ud658 \ud615\uc2dd\uc758 \ubd80\uc9c0\ub97c \uc5b4\ub5a4 \ubc29\uc2dd\uc73c\ub85c \uc7ac \ud655\ubcf4\ud574\uc57c \ud558\ub098?, context: \ud658\uacbd\uc601\ud5a5\ud3c9\uac00 \ubb38\uc81c\ub97c \uccad\uc640\ub300\uac00 \uc6d0\ud558\ub294 \ub300\ub85c \ud560 \uc218 \uc788\ub294 \ud569\ubc95\uc801\uc778 \ubc29\ubc95\uc740 \uc0ac\ub4dc \ubc30\uce58 \uacc4\ud68d\uc744 \uc804\uba74 \ubc31\uc9c0\ud654\ud558\uace0 \uc804\ub7b5\ud658\uacbd\uc601\ud5a5\ud3c9\uac00\ub97c \ubaa8\ub450 \ubc1b\uc744 \uc218 \uc788\ub3c4\ub85d \ud558\ub294 \uac83\uc774\uc9c0\ub9cc \uc774\ub97c \uc704\ud574\uc11c\ub294 \uc774\ubbf8 \uc57c\uc804 \ubc30\uce58\ub41c \ub808\uc774\ub354\uc640 \ubc1c\uc0ac\ub300 \ub4f1 \uc0ac\ub4dc \uc7a5\ube44\ub4e4\uc744 \uaebc\ub0b4\uc57c \ud558\uace0, \ubd80\uc9c0\ub3c4 \ub86f\ub370\uc640\uc758 \ud1a0\uc9c0 \ub9de\uad50\ud658\uc774 \uc544\ub2cc \ub9e4\ub9e4 \ubc29\uc2dd\uc73c\ub85c \ub2e4\uc2dc \ud655\ubcf4\ud574\uc57c \ud558\ub294 \ub4f1 \ubcf5\uc7a1\ud55c \ubb38\uc81c\uac00 \uc5ec\ub7ff \ubc1c\uc0dd\ud55c\ub2e4. \uccad\uc640\ub300\uc5d0\uc11c \uc694\uad6c\ud558\ub294 \"\ud604 \ubc30\uce58 \uc0c1\ud0dc\ub294 \uc720\uc9c0\ud558\uba74\uc11c \uc804\ub7b5\ud658\uacbd\uc601\ud5a5\ud3c9\uac00\ub97c \uc81c\ub300\ub85c \uac70\uce58\ub77c\"\ub294 \uac83\uc774 \ubc95\uc801\uc73c\ub85c \ubd88\uac00\ub2a5\ud55c \uc774\uc720\ub294 \ud658\uacbd\uc601\ud5a5\ud3c9\uac00\ubc95\uacfc \uc2dc\ud589\ub839\uc5d0 \ub530\ub974\uba74 \uad6d\ubc29\uc7a5\uad00\uc774 \uc0ac\uc5c5\uacc4\ud68d\uc744 \uc2b9\uc778\ud558\uae30 \uc804, \uc989 \ubd80\uc9c0\ub97c \ud655\ubcf4\ud558\uae30 \uc804 \uacc4\ud68d \ub2e8\uacc4\uc5d0 \uc2e4\uc2dc\ud558\ub294 \uac83\uc774\uace0 \uc0ac\ub4dc\ub294 \uadf8 \ub2e8\uacc4\ub97c \ud55c\ucc38 \uc9c0\ub098 \uc774\ubbf8 \ubc30\uce58\u00b7\uc6b4\uc6a9\ub418\uace0 \uc788\uc73c\uba70 \uc0ac\ub4dc \ubd80\uc9c0\ucc98\ub7fc \ub9e4\uc785\uc774\ub098 \uc218\uc6a9 \ubc29\uc2dd\uc774 \uc544\ub2cc, \ud1a0\uc9c0 \ub9de\uad50\ud658 \ubc29\uc2dd\uc73c\ub85c \ud655\ubcf4\ub41c \ub545\uc758 \uacbd\uc6b0\uc5d0\ub294 \uc544\uc608 \uc804\ub7b5\ud658\uacbd\uc601\ud5a5\ud3c9\uac00 \ub300\uc0c1\uc774 \uc544\ub2c8\ub77c\ub294 \uac83\uc774 \uad6d\ubc29\ubd80 \uc124\uba85\uc774\ub2e4."} {"question": "\uc694\ubbf8\uc6b0\ub9ac \uc785\ub2e8\ud6c4 \uc815\ubbfc\ucca0\uc758 \uccab \ub4f1\ud310\ud55c \uacbd\uae30\uc758 \uc0c1\ub300\ud3b8\uc740 \ub204\uad6c\uc778\uac00?", "paragraph": "2000\ub144 \uc77c\ubcf8 \ud504\ub85c \uc57c\uad6c \uc694\ubbf8\uc6b0\ub9ac \uc790\uc774\uc5b8\uce20\uc5d0 \uc785\ub2e8\ud558\uc5ec 2\uc2dc\uc98c\uc744 \ubcf4\ub0b8\ub2e4. \uad6d\ub0b4\uc5d0\uc11c\ub294 \ub9ac\uadf8 \ucd5c\uace0 \uc218\uc900\uc758 \uc5d0\uc774\uc2a4\uc600\uc9c0\ub9cc \uc77c\ubcf8 \ub9ac\uadf8\uc5d0\uc11c\ub294 \ud604\uc9c0 \uc801\uc751 \uc2e4\ud328\uc640 \ub9ac\uadf8\uaca9\ucc28\ub85c \uc778\ud574 \ub69c\ub837\ud55c \uc131\uacfc\ub97c \uc62c\ub9ac\uc9c0 \ubabb\ud588\ub2e4. \uc77c\ubcf8 \ub9ac\uadf8\uc5d0\uc11c \uadf8\ub294 2\uc2dc\uc98c \ub3d9\uc548 3\uc2b9 2\ud328\uc758 \ucd08\ub77c\ud55c \uc131\uc801\uc744 \uac70\ub450\uc5c8\uace0, \ubd80\uc9c4\ud588\ub358 \uc8fc\uc694\ud55c \uc6d0\uc778\uc73c\ub85c\ub294 \uae09\uaca9\ud55c \uad6c\uc704 \uc800\ud558, \ud314\uafc8\uce58 \ud1b5\uc99d\uc774\uc5c8\ub2e4. \uadf8\ub807\uc9c0\ub9cc \uc774 \uc548\ud0c0\uae4c\uc6b4 \uacb0\uacfc\uc5d0\ub294 \uc5ec\ub7ec \ub17c\ub780\uc774 \uc788\ub294\ub370 \uc2e4\uc81c\ub85c \uc815\ubbfc\ucca0\uc758 \uc694\ubbf8\uc6b0\ub9ac \uc785\ub2e8 \ud6c4 \uccab \ub4f1\ud310\uc740 \uc57c\ucfe0\ub974\ud2b8\ub97c \uc0c1\ub300\ub85c 7\uc774\ub2dd 1\uc2e4\uc810 \uc2b9\uc73c\ub85c \ud638\ud22c\ud558\uc600\uc73c\ub098 \uacbd\uae30 \uc9c1\ud6c4 2\uad70 \ud1b5\ubcf4\ub97c \ubc1b\uace0, \ub2e4\uc2dc \uc62c\ub77c\uc628 1\uad70 \uacbd\uae30\uc5d0\uc11c \uc694\ucf54\ud558\ub9c8\ub97c \uc0c1\ub300\ub85c 9\uc774\ub2dd \ubb34\uc2e4\uc810\uc73c\ub85c \uc644\ubd09\uc2b9 \ud558\uc600\uc73c\ub098 \uc774\ud6c4\ub85c\ub3c4 \ub4f1\ud310\uae30\ud68c\ub294 \uc880\ucc98\ub7fc \uc8fc\uc5b4\uc9c0\uc9c0 \uc54a\uc558\ub2e4. \uadf8 \ud6c4 \uad6d\ub0b4 \ubcf5\uadc0 20\uc644\ubd09 \ub2ec\uc131 \uc778\ud130\ubdf0\uc5d0\uc11c \"\uc694\ubbf8\uc6b0\ub9ac\uc758 \uc2dc\uc2a4\ud15c\uc774 \ucc38 \uc774\uc0c1\ud588\ub2e4.\"\ub77c\uace0 \ud68c\uace0\ud55c \ubc14 \uc788\ub2e4.", "answer": "\uc57c\ucfe0\ub974\ud2b8", "sentence": "\uadf8\ub807\uc9c0\ub9cc \uc774 \uc548\ud0c0\uae4c\uc6b4 \uacb0\uacfc\uc5d0\ub294 \uc5ec\ub7ec \ub17c\ub780\uc774 \uc788\ub294\ub370 \uc2e4\uc81c\ub85c \uc815\ubbfc\ucca0\uc758 \uc694\ubbf8\uc6b0\ub9ac \uc785\ub2e8 \ud6c4 \uccab \ub4f1\ud310\uc740 \uc57c\ucfe0\ub974\ud2b8 \ub97c \uc0c1\ub300\ub85c 7\uc774\ub2dd 1\uc2e4\uc810 \uc2b9\uc73c\ub85c \ud638\ud22c\ud558\uc600\uc73c\ub098 \uacbd\uae30 \uc9c1\ud6c4 2\uad70 \ud1b5\ubcf4\ub97c \ubc1b\uace0, \ub2e4\uc2dc \uc62c\ub77c\uc628 1\uad70 \uacbd\uae30\uc5d0\uc11c \uc694\ucf54\ud558\ub9c8\ub97c \uc0c1\ub300\ub85c 9\uc774\ub2dd \ubb34\uc2e4\uc810\uc73c\ub85c \uc644\ubd09\uc2b9 \ud558\uc600\uc73c\ub098 \uc774\ud6c4\ub85c\ub3c4 \ub4f1\ud310\uae30\ud68c\ub294 \uc880\ucc98\ub7fc \uc8fc\uc5b4\uc9c0\uc9c0 \uc54a\uc558\ub2e4.", "paragraph_sentence": "2000\ub144 \uc77c\ubcf8 \ud504\ub85c \uc57c\uad6c \uc694\ubbf8\uc6b0\ub9ac \uc790\uc774\uc5b8\uce20\uc5d0 \uc785\ub2e8\ud558\uc5ec 2\uc2dc\uc98c\uc744 \ubcf4\ub0b8\ub2e4. \uad6d\ub0b4\uc5d0\uc11c\ub294 \ub9ac\uadf8 \ucd5c\uace0 \uc218\uc900\uc758 \uc5d0\uc774\uc2a4\uc600\uc9c0\ub9cc \uc77c\ubcf8 \ub9ac\uadf8\uc5d0\uc11c\ub294 \ud604\uc9c0 \uc801\uc751 \uc2e4\ud328\uc640 \ub9ac\uadf8\uaca9\ucc28\ub85c \uc778\ud574 \ub69c\ub837\ud55c \uc131\uacfc\ub97c \uc62c\ub9ac\uc9c0 \ubabb\ud588\ub2e4. \uc77c\ubcf8 \ub9ac\uadf8\uc5d0\uc11c \uadf8\ub294 2\uc2dc\uc98c \ub3d9\uc548 3\uc2b9 2\ud328\uc758 \ucd08\ub77c\ud55c \uc131\uc801\uc744 \uac70\ub450\uc5c8\uace0, \ubd80\uc9c4\ud588\ub358 \uc8fc\uc694\ud55c \uc6d0\uc778\uc73c\ub85c\ub294 \uae09\uaca9\ud55c \uad6c\uc704 \uc800\ud558, \ud314\uafc8\uce58 \ud1b5\uc99d\uc774\uc5c8\ub2e4. \uadf8\ub807\uc9c0\ub9cc \uc774 \uc548\ud0c0\uae4c\uc6b4 \uacb0\uacfc\uc5d0\ub294 \uc5ec\ub7ec \ub17c\ub780\uc774 \uc788\ub294\ub370 \uc2e4\uc81c\ub85c \uc815\ubbfc\ucca0\uc758 \uc694\ubbf8\uc6b0\ub9ac \uc785\ub2e8 \ud6c4 \uccab \ub4f1\ud310\uc740 \uc57c\ucfe0\ub974\ud2b8 \ub97c \uc0c1\ub300\ub85c 7\uc774\ub2dd 1\uc2e4\uc810 \uc2b9\uc73c\ub85c \ud638\ud22c\ud558\uc600\uc73c\ub098 \uacbd\uae30 \uc9c1\ud6c4 2\uad70 \ud1b5\ubcf4\ub97c \ubc1b\uace0, \ub2e4\uc2dc \uc62c\ub77c\uc628 1\uad70 \uacbd\uae30\uc5d0\uc11c \uc694\ucf54\ud558\ub9c8\ub97c \uc0c1\ub300\ub85c 9\uc774\ub2dd \ubb34\uc2e4\uc810\uc73c\ub85c \uc644\ubd09\uc2b9 \ud558\uc600\uc73c\ub098 \uc774\ud6c4\ub85c\ub3c4 \ub4f1\ud310\uae30\ud68c\ub294 \uc880\ucc98\ub7fc \uc8fc\uc5b4\uc9c0\uc9c0 \uc54a\uc558\ub2e4. \uadf8 \ud6c4 \uad6d\ub0b4 \ubcf5\uadc0 20\uc644\ubd09 \ub2ec\uc131 \uc778\ud130\ubdf0\uc5d0\uc11c \"\uc694\ubbf8\uc6b0\ub9ac\uc758 \uc2dc\uc2a4\ud15c\uc774 \ucc38 \uc774\uc0c1\ud588\ub2e4. \"\ub77c\uace0 \ud68c\uace0\ud55c \ubc14 \uc788\ub2e4.", "paragraph_answer": "2000\ub144 \uc77c\ubcf8 \ud504\ub85c \uc57c\uad6c \uc694\ubbf8\uc6b0\ub9ac \uc790\uc774\uc5b8\uce20\uc5d0 \uc785\ub2e8\ud558\uc5ec 2\uc2dc\uc98c\uc744 \ubcf4\ub0b8\ub2e4. \uad6d\ub0b4\uc5d0\uc11c\ub294 \ub9ac\uadf8 \ucd5c\uace0 \uc218\uc900\uc758 \uc5d0\uc774\uc2a4\uc600\uc9c0\ub9cc \uc77c\ubcf8 \ub9ac\uadf8\uc5d0\uc11c\ub294 \ud604\uc9c0 \uc801\uc751 \uc2e4\ud328\uc640 \ub9ac\uadf8\uaca9\ucc28\ub85c \uc778\ud574 \ub69c\ub837\ud55c \uc131\uacfc\ub97c \uc62c\ub9ac\uc9c0 \ubabb\ud588\ub2e4. \uc77c\ubcf8 \ub9ac\uadf8\uc5d0\uc11c \uadf8\ub294 2\uc2dc\uc98c \ub3d9\uc548 3\uc2b9 2\ud328\uc758 \ucd08\ub77c\ud55c \uc131\uc801\uc744 \uac70\ub450\uc5c8\uace0, \ubd80\uc9c4\ud588\ub358 \uc8fc\uc694\ud55c \uc6d0\uc778\uc73c\ub85c\ub294 \uae09\uaca9\ud55c \uad6c\uc704 \uc800\ud558, \ud314\uafc8\uce58 \ud1b5\uc99d\uc774\uc5c8\ub2e4. \uadf8\ub807\uc9c0\ub9cc \uc774 \uc548\ud0c0\uae4c\uc6b4 \uacb0\uacfc\uc5d0\ub294 \uc5ec\ub7ec \ub17c\ub780\uc774 \uc788\ub294\ub370 \uc2e4\uc81c\ub85c \uc815\ubbfc\ucca0\uc758 \uc694\ubbf8\uc6b0\ub9ac \uc785\ub2e8 \ud6c4 \uccab \ub4f1\ud310\uc740 \uc57c\ucfe0\ub974\ud2b8 \ub97c \uc0c1\ub300\ub85c 7\uc774\ub2dd 1\uc2e4\uc810 \uc2b9\uc73c\ub85c \ud638\ud22c\ud558\uc600\uc73c\ub098 \uacbd\uae30 \uc9c1\ud6c4 2\uad70 \ud1b5\ubcf4\ub97c \ubc1b\uace0, \ub2e4\uc2dc \uc62c\ub77c\uc628 1\uad70 \uacbd\uae30\uc5d0\uc11c \uc694\ucf54\ud558\ub9c8\ub97c \uc0c1\ub300\ub85c 9\uc774\ub2dd \ubb34\uc2e4\uc810\uc73c\ub85c \uc644\ubd09\uc2b9 \ud558\uc600\uc73c\ub098 \uc774\ud6c4\ub85c\ub3c4 \ub4f1\ud310\uae30\ud68c\ub294 \uc880\ucc98\ub7fc \uc8fc\uc5b4\uc9c0\uc9c0 \uc54a\uc558\ub2e4. \uadf8 \ud6c4 \uad6d\ub0b4 \ubcf5\uadc0 20\uc644\ubd09 \ub2ec\uc131 \uc778\ud130\ubdf0\uc5d0\uc11c \"\uc694\ubbf8\uc6b0\ub9ac\uc758 \uc2dc\uc2a4\ud15c\uc774 \ucc38 \uc774\uc0c1\ud588\ub2e4.\"\ub77c\uace0 \ud68c\uace0\ud55c \ubc14 \uc788\ub2e4.", "sentence_answer": "\uadf8\ub807\uc9c0\ub9cc \uc774 \uc548\ud0c0\uae4c\uc6b4 \uacb0\uacfc\uc5d0\ub294 \uc5ec\ub7ec \ub17c\ub780\uc774 \uc788\ub294\ub370 \uc2e4\uc81c\ub85c \uc815\ubbfc\ucca0\uc758 \uc694\ubbf8\uc6b0\ub9ac \uc785\ub2e8 \ud6c4 \uccab \ub4f1\ud310\uc740 \uc57c\ucfe0\ub974\ud2b8 \ub97c \uc0c1\ub300\ub85c 7\uc774\ub2dd 1\uc2e4\uc810 \uc2b9\uc73c\ub85c \ud638\ud22c\ud558\uc600\uc73c\ub098 \uacbd\uae30 \uc9c1\ud6c4 2\uad70 \ud1b5\ubcf4\ub97c \ubc1b\uace0, \ub2e4\uc2dc \uc62c\ub77c\uc628 1\uad70 \uacbd\uae30\uc5d0\uc11c \uc694\ucf54\ud558\ub9c8\ub97c \uc0c1\ub300\ub85c 9\uc774\ub2dd \ubb34\uc2e4\uc810\uc73c\ub85c \uc644\ubd09\uc2b9 \ud558\uc600\uc73c\ub098 \uc774\ud6c4\ub85c\ub3c4 \ub4f1\ud310\uae30\ud68c\ub294 \uc880\ucc98\ub7fc \uc8fc\uc5b4\uc9c0\uc9c0 \uc54a\uc558\ub2e4.", "paragraph_id": "6550099-2-2", "paragraph_question": "question: \uc694\ubbf8\uc6b0\ub9ac \uc785\ub2e8\ud6c4 \uc815\ubbfc\ucca0\uc758 \uccab \ub4f1\ud310\ud55c \uacbd\uae30\uc758 \uc0c1\ub300\ud3b8\uc740 \ub204\uad6c\uc778\uac00?, context: 2000\ub144 \uc77c\ubcf8 \ud504\ub85c \uc57c\uad6c \uc694\ubbf8\uc6b0\ub9ac \uc790\uc774\uc5b8\uce20\uc5d0 \uc785\ub2e8\ud558\uc5ec 2\uc2dc\uc98c\uc744 \ubcf4\ub0b8\ub2e4. \uad6d\ub0b4\uc5d0\uc11c\ub294 \ub9ac\uadf8 \ucd5c\uace0 \uc218\uc900\uc758 \uc5d0\uc774\uc2a4\uc600\uc9c0\ub9cc \uc77c\ubcf8 \ub9ac\uadf8\uc5d0\uc11c\ub294 \ud604\uc9c0 \uc801\uc751 \uc2e4\ud328\uc640 \ub9ac\uadf8\uaca9\ucc28\ub85c \uc778\ud574 \ub69c\ub837\ud55c \uc131\uacfc\ub97c \uc62c\ub9ac\uc9c0 \ubabb\ud588\ub2e4. \uc77c\ubcf8 \ub9ac\uadf8\uc5d0\uc11c \uadf8\ub294 2\uc2dc\uc98c \ub3d9\uc548 3\uc2b9 2\ud328\uc758 \ucd08\ub77c\ud55c \uc131\uc801\uc744 \uac70\ub450\uc5c8\uace0, \ubd80\uc9c4\ud588\ub358 \uc8fc\uc694\ud55c \uc6d0\uc778\uc73c\ub85c\ub294 \uae09\uaca9\ud55c \uad6c\uc704 \uc800\ud558, \ud314\uafc8\uce58 \ud1b5\uc99d\uc774\uc5c8\ub2e4. \uadf8\ub807\uc9c0\ub9cc \uc774 \uc548\ud0c0\uae4c\uc6b4 \uacb0\uacfc\uc5d0\ub294 \uc5ec\ub7ec \ub17c\ub780\uc774 \uc788\ub294\ub370 \uc2e4\uc81c\ub85c \uc815\ubbfc\ucca0\uc758 \uc694\ubbf8\uc6b0\ub9ac \uc785\ub2e8 \ud6c4 \uccab \ub4f1\ud310\uc740 \uc57c\ucfe0\ub974\ud2b8\ub97c \uc0c1\ub300\ub85c 7\uc774\ub2dd 1\uc2e4\uc810 \uc2b9\uc73c\ub85c \ud638\ud22c\ud558\uc600\uc73c\ub098 \uacbd\uae30 \uc9c1\ud6c4 2\uad70 \ud1b5\ubcf4\ub97c \ubc1b\uace0, \ub2e4\uc2dc \uc62c\ub77c\uc628 1\uad70 \uacbd\uae30\uc5d0\uc11c \uc694\ucf54\ud558\ub9c8\ub97c \uc0c1\ub300\ub85c 9\uc774\ub2dd \ubb34\uc2e4\uc810\uc73c\ub85c \uc644\ubd09\uc2b9 \ud558\uc600\uc73c\ub098 \uc774\ud6c4\ub85c\ub3c4 \ub4f1\ud310\uae30\ud68c\ub294 \uc880\ucc98\ub7fc \uc8fc\uc5b4\uc9c0\uc9c0 \uc54a\uc558\ub2e4. \uadf8 \ud6c4 \uad6d\ub0b4 \ubcf5\uadc0 20\uc644\ubd09 \ub2ec\uc131 \uc778\ud130\ubdf0\uc5d0\uc11c \"\uc694\ubbf8\uc6b0\ub9ac\uc758 \uc2dc\uc2a4\ud15c\uc774 \ucc38 \uc774\uc0c1\ud588\ub2e4.\"\ub77c\uace0 \ud68c\uace0\ud55c \ubc14 \uc788\ub2e4."} {"question": "\ud654\uac00 \ud55c\uc2a4 \ub9ac\ud788\ud130\uac00 \uc870\ud615\uc801\uc778 \uc2e4\ud5d8\uc601\ud654\ub97c \uc5b8\uc81c \ubc1c\ud45c\ud558\uc600\ub294\uac00?", "paragraph": "\uadf8\ub7ec\ub098 \uc81c1\ucc28 \ub300\uc804\uc758 \ud328\uc804 \ud6c4\uc5d0 \ub3c5\uc77c\uc758 \uacbd\uc81c\uc801\u00b7\uc0ac\ud68c\uc801 \ud63c\ub780\uc774 \ud45c\ud604\uc8fc\uc758\ub97c \ud06c\uac8c \uc721\uc131\ud558\ub294 \uc694\uc778\uc744 \uc774\ub8e8\uace0 \uc788\ub2e4\ub294 \uc810\uc5d0\uc11c \ud504\ub791\uc2a4\uc758 \uc601\ud654\uc774\ub860\uc5d0 \ube44\ud574 \uc0ac\ud68c\uc801\uc774\uace0\ub3c4 \uc2e4\ucc9c\uc801\uc778 \uc131\uaca9\uc774 \ud55c\uce35 \uac15\ub825\ud558\uac8c \ub4dc\ub7ec\ub098 \uc788\ub2e4\uace0 \ud560 \uc218 \uc788\ub2e4. \ub3c5\uc77c\uc758 \ud45c\ud604\uc8fc\uc758\ub294 \ubb38\ud559\uacfc \uc5f0\uadf9\uc5d0\uc11c \uc601\ud654\uc5d0 \ubbf8\ucce4\uc73c\ub098 \uc774\ub860\ud654\ub418\uae30 \uc804\uc5d0 <\uc774\uae30\uc8fc\uc758\uc790>(1919)\uc640 \uac19\uc740 \uac78\uc791\uc774 \ub098\uc640 \uc788\uc5c8\ub2e4. \uadf8\ub9ac\uace0 1921\ub144\uc5d0 \ud654\uac00 \ud55c\uc2a4 \ub9ac\ud788\ud130(Hans Richter)\uac00 \ube44\ud0b9 \uc5d0\uac94\ub9c1\uacfc \ucd94\uc0c1\uc801\uc778 \ud654\uc0c1(\u7575\u50cf)\uc758 \uc5f0\uc18d\uc73c\ub85c \uc774\ub8e8\uc5b4\uc9c4 \uc21c\uc218\ud55c \uc870\ud615\uc801\uc778 \uc2e4\ud5d8\uc601\ud654\ub97c \ubc1c\ud45c\ud558\uc5ec, \uae09\uc9c4\uc801\uc778 \uc131\uaca9\uc740 \ub354\uc6b1 \uac15\ub825\ud574\uc84c\ub2e4. \uc774 \ub3c5\uc77c\uc758 \ud45c\ud604\uc8fc\uc758 \uc601\ud654\uc758 \uc774\ub860\uc740 \ub2f9\uc2dc\uc5d0\ub294 \uccb4\uacc4\ud654\ub418\uc5b4 \uc788\uc9c0 \uc54a\uc558\uc73c\uba70, \uadf8 \uc120\uad6c\uc790\uc778 \ub9ac\ud788\ud130\ub294 \ud6c4\ub144\uc5d0 \uc774 \uc2e4\ud5d8\uc740 \uc785\uccb4\uc8fc\uc758 \ubc29\ud5a5\uc744 \ubc1c\uc804\uc2dc\ud0a4\uae30 \uc704\ud558\uc5ec \uc601\ud654\ub77c\ub294 \ubbf8\ub514\uc5c4\uc744 \ud544\uc5f0\uc801\uc73c\ub85c \uc0ac\uc6a9\ud558\uac8c \ub41c \uac83\uc774\uace0, \"\uc6b0\ub9ac\ub294 \uc601\ud654 \uc790\uccb4\uc5d0\ub294 \uad00\uc2ec\uc774 \uc5c6\uc5c8\ub2e4. \ub610 \uc601\ud654\ub97c \uc774\uc6a9\ud560 \uac83\ub3c4 \uc0dd\uac01\ud558\uc9c0 \uc54a\uc558\ub2e4. \uc0c8\ub85c\uc6b4 \uacbd\ud5d8\uc774 \ub17c\ub9ac\uc801\uc73c\ub85c \ud544\uc5f0\uc801\uc73c\ub85c \uae30\ub300\ud558\uc9c0 \uc54a\uc740 \uac00\uc6b4\ub370 \ub098\ud0c0\ub0ac\ub2e4\"\uace0 \ub9d0\ud588\ub2e4.", "answer": "1921\ub144", "sentence": "\uadf8\ub9ac\uace0 1921\ub144 \uc5d0", "paragraph_sentence": "\uadf8\ub7ec\ub098 \uc81c1\ucc28 \ub300\uc804\uc758 \ud328\uc804 \ud6c4\uc5d0 \ub3c5\uc77c\uc758 \uacbd\uc81c\uc801\u00b7\uc0ac\ud68c\uc801 \ud63c\ub780\uc774 \ud45c\ud604\uc8fc\uc758\ub97c \ud06c\uac8c \uc721\uc131\ud558\ub294 \uc694\uc778\uc744 \uc774\ub8e8\uace0 \uc788\ub2e4\ub294 \uc810\uc5d0\uc11c \ud504\ub791\uc2a4\uc758 \uc601\ud654\uc774\ub860\uc5d0 \ube44\ud574 \uc0ac\ud68c\uc801\uc774\uace0\ub3c4 \uc2e4\ucc9c\uc801\uc778 \uc131\uaca9\uc774 \ud55c\uce35 \uac15\ub825\ud558\uac8c \ub4dc\ub7ec\ub098 \uc788\ub2e4\uace0 \ud560 \uc218 \uc788\ub2e4. \ub3c5\uc77c\uc758 \ud45c\ud604\uc8fc\uc758\ub294 \ubb38\ud559\uacfc \uc5f0\uadf9\uc5d0\uc11c \uc601\ud654\uc5d0 \ubbf8\ucce4\uc73c\ub098 \uc774\ub860\ud654\ub418\uae30 \uc804\uc5d0 <\uc774\uae30\uc8fc\uc758\uc790>(1919)\uc640 \uac19\uc740 \uac78\uc791\uc774 \ub098\uc640 \uc788\uc5c8\ub2e4. \uadf8\ub9ac\uace0 1921\ub144 \uc5d0 \ud654\uac00 \ud55c\uc2a4 \ub9ac\ud788\ud130(Hans Richter)\uac00 \ube44\ud0b9 \uc5d0\uac94\ub9c1\uacfc \ucd94\uc0c1\uc801\uc778 \ud654\uc0c1(\u7575\u50cf)\uc758 \uc5f0\uc18d\uc73c\ub85c \uc774\ub8e8\uc5b4\uc9c4 \uc21c\uc218\ud55c \uc870\ud615\uc801\uc778 \uc2e4\ud5d8\uc601\ud654\ub97c \ubc1c\ud45c\ud558\uc5ec, \uae09\uc9c4\uc801\uc778 \uc131\uaca9\uc740 \ub354\uc6b1 \uac15\ub825\ud574\uc84c\ub2e4. \uc774 \ub3c5\uc77c\uc758 \ud45c\ud604\uc8fc\uc758 \uc601\ud654\uc758 \uc774\ub860\uc740 \ub2f9\uc2dc\uc5d0\ub294 \uccb4\uacc4\ud654\ub418\uc5b4 \uc788\uc9c0 \uc54a\uc558\uc73c\uba70, \uadf8 \uc120\uad6c\uc790\uc778 \ub9ac\ud788\ud130\ub294 \ud6c4\ub144\uc5d0 \uc774 \uc2e4\ud5d8\uc740 \uc785\uccb4\uc8fc\uc758 \ubc29\ud5a5\uc744 \ubc1c\uc804\uc2dc\ud0a4\uae30 \uc704\ud558\uc5ec \uc601\ud654\ub77c\ub294 \ubbf8\ub514\uc5c4\uc744 \ud544\uc5f0\uc801\uc73c\ub85c \uc0ac\uc6a9\ud558\uac8c \ub41c \uac83\uc774\uace0, \"\uc6b0\ub9ac\ub294 \uc601\ud654 \uc790\uccb4\uc5d0\ub294 \uad00\uc2ec\uc774 \uc5c6\uc5c8\ub2e4. \ub610 \uc601\ud654\ub97c \uc774\uc6a9\ud560 \uac83\ub3c4 \uc0dd\uac01\ud558\uc9c0 \uc54a\uc558\ub2e4. \uc0c8\ub85c\uc6b4 \uacbd\ud5d8\uc774 \ub17c\ub9ac\uc801\uc73c\ub85c \ud544\uc5f0\uc801\uc73c\ub85c \uae30\ub300\ud558\uc9c0 \uc54a\uc740 \uac00\uc6b4\ub370 \ub098\ud0c0\ub0ac\ub2e4\"\uace0 \ub9d0\ud588\ub2e4.", "paragraph_answer": "\uadf8\ub7ec\ub098 \uc81c1\ucc28 \ub300\uc804\uc758 \ud328\uc804 \ud6c4\uc5d0 \ub3c5\uc77c\uc758 \uacbd\uc81c\uc801\u00b7\uc0ac\ud68c\uc801 \ud63c\ub780\uc774 \ud45c\ud604\uc8fc\uc758\ub97c \ud06c\uac8c \uc721\uc131\ud558\ub294 \uc694\uc778\uc744 \uc774\ub8e8\uace0 \uc788\ub2e4\ub294 \uc810\uc5d0\uc11c \ud504\ub791\uc2a4\uc758 \uc601\ud654\uc774\ub860\uc5d0 \ube44\ud574 \uc0ac\ud68c\uc801\uc774\uace0\ub3c4 \uc2e4\ucc9c\uc801\uc778 \uc131\uaca9\uc774 \ud55c\uce35 \uac15\ub825\ud558\uac8c \ub4dc\ub7ec\ub098 \uc788\ub2e4\uace0 \ud560 \uc218 \uc788\ub2e4. \ub3c5\uc77c\uc758 \ud45c\ud604\uc8fc\uc758\ub294 \ubb38\ud559\uacfc \uc5f0\uadf9\uc5d0\uc11c \uc601\ud654\uc5d0 \ubbf8\ucce4\uc73c\ub098 \uc774\ub860\ud654\ub418\uae30 \uc804\uc5d0 <\uc774\uae30\uc8fc\uc758\uc790>(1919)\uc640 \uac19\uc740 \uac78\uc791\uc774 \ub098\uc640 \uc788\uc5c8\ub2e4. \uadf8\ub9ac\uace0 1921\ub144 \uc5d0 \ud654\uac00 \ud55c\uc2a4 \ub9ac\ud788\ud130(Hans Richter)\uac00 \ube44\ud0b9 \uc5d0\uac94\ub9c1\uacfc \ucd94\uc0c1\uc801\uc778 \ud654\uc0c1(\u7575\u50cf)\uc758 \uc5f0\uc18d\uc73c\ub85c \uc774\ub8e8\uc5b4\uc9c4 \uc21c\uc218\ud55c \uc870\ud615\uc801\uc778 \uc2e4\ud5d8\uc601\ud654\ub97c \ubc1c\ud45c\ud558\uc5ec, \uae09\uc9c4\uc801\uc778 \uc131\uaca9\uc740 \ub354\uc6b1 \uac15\ub825\ud574\uc84c\ub2e4. \uc774 \ub3c5\uc77c\uc758 \ud45c\ud604\uc8fc\uc758 \uc601\ud654\uc758 \uc774\ub860\uc740 \ub2f9\uc2dc\uc5d0\ub294 \uccb4\uacc4\ud654\ub418\uc5b4 \uc788\uc9c0 \uc54a\uc558\uc73c\uba70, \uadf8 \uc120\uad6c\uc790\uc778 \ub9ac\ud788\ud130\ub294 \ud6c4\ub144\uc5d0 \uc774 \uc2e4\ud5d8\uc740 \uc785\uccb4\uc8fc\uc758 \ubc29\ud5a5\uc744 \ubc1c\uc804\uc2dc\ud0a4\uae30 \uc704\ud558\uc5ec \uc601\ud654\ub77c\ub294 \ubbf8\ub514\uc5c4\uc744 \ud544\uc5f0\uc801\uc73c\ub85c \uc0ac\uc6a9\ud558\uac8c \ub41c \uac83\uc774\uace0, \"\uc6b0\ub9ac\ub294 \uc601\ud654 \uc790\uccb4\uc5d0\ub294 \uad00\uc2ec\uc774 \uc5c6\uc5c8\ub2e4. \ub610 \uc601\ud654\ub97c \uc774\uc6a9\ud560 \uac83\ub3c4 \uc0dd\uac01\ud558\uc9c0 \uc54a\uc558\ub2e4. \uc0c8\ub85c\uc6b4 \uacbd\ud5d8\uc774 \ub17c\ub9ac\uc801\uc73c\ub85c \ud544\uc5f0\uc801\uc73c\ub85c \uae30\ub300\ud558\uc9c0 \uc54a\uc740 \uac00\uc6b4\ub370 \ub098\ud0c0\ub0ac\ub2e4\"\uace0 \ub9d0\ud588\ub2e4.", "sentence_answer": "\uadf8\ub9ac\uace0 1921\ub144 \uc5d0", "paragraph_id": "6465647-2-1", "paragraph_question": "question: \ud654\uac00 \ud55c\uc2a4 \ub9ac\ud788\ud130\uac00 \uc870\ud615\uc801\uc778 \uc2e4\ud5d8\uc601\ud654\ub97c \uc5b8\uc81c \ubc1c\ud45c\ud558\uc600\ub294\uac00?, context: \uadf8\ub7ec\ub098 \uc81c1\ucc28 \ub300\uc804\uc758 \ud328\uc804 \ud6c4\uc5d0 \ub3c5\uc77c\uc758 \uacbd\uc81c\uc801\u00b7\uc0ac\ud68c\uc801 \ud63c\ub780\uc774 \ud45c\ud604\uc8fc\uc758\ub97c \ud06c\uac8c \uc721\uc131\ud558\ub294 \uc694\uc778\uc744 \uc774\ub8e8\uace0 \uc788\ub2e4\ub294 \uc810\uc5d0\uc11c \ud504\ub791\uc2a4\uc758 \uc601\ud654\uc774\ub860\uc5d0 \ube44\ud574 \uc0ac\ud68c\uc801\uc774\uace0\ub3c4 \uc2e4\ucc9c\uc801\uc778 \uc131\uaca9\uc774 \ud55c\uce35 \uac15\ub825\ud558\uac8c \ub4dc\ub7ec\ub098 \uc788\ub2e4\uace0 \ud560 \uc218 \uc788\ub2e4. \ub3c5\uc77c\uc758 \ud45c\ud604\uc8fc\uc758\ub294 \ubb38\ud559\uacfc \uc5f0\uadf9\uc5d0\uc11c \uc601\ud654\uc5d0 \ubbf8\ucce4\uc73c\ub098 \uc774\ub860\ud654\ub418\uae30 \uc804\uc5d0 <\uc774\uae30\uc8fc\uc758\uc790>(1919)\uc640 \uac19\uc740 \uac78\uc791\uc774 \ub098\uc640 \uc788\uc5c8\ub2e4. \uadf8\ub9ac\uace0 1921\ub144\uc5d0 \ud654\uac00 \ud55c\uc2a4 \ub9ac\ud788\ud130(Hans Richter)\uac00 \ube44\ud0b9 \uc5d0\uac94\ub9c1\uacfc \ucd94\uc0c1\uc801\uc778 \ud654\uc0c1(\u7575\u50cf)\uc758 \uc5f0\uc18d\uc73c\ub85c \uc774\ub8e8\uc5b4\uc9c4 \uc21c\uc218\ud55c \uc870\ud615\uc801\uc778 \uc2e4\ud5d8\uc601\ud654\ub97c \ubc1c\ud45c\ud558\uc5ec, \uae09\uc9c4\uc801\uc778 \uc131\uaca9\uc740 \ub354\uc6b1 \uac15\ub825\ud574\uc84c\ub2e4. \uc774 \ub3c5\uc77c\uc758 \ud45c\ud604\uc8fc\uc758 \uc601\ud654\uc758 \uc774\ub860\uc740 \ub2f9\uc2dc\uc5d0\ub294 \uccb4\uacc4\ud654\ub418\uc5b4 \uc788\uc9c0 \uc54a\uc558\uc73c\uba70, \uadf8 \uc120\uad6c\uc790\uc778 \ub9ac\ud788\ud130\ub294 \ud6c4\ub144\uc5d0 \uc774 \uc2e4\ud5d8\uc740 \uc785\uccb4\uc8fc\uc758 \ubc29\ud5a5\uc744 \ubc1c\uc804\uc2dc\ud0a4\uae30 \uc704\ud558\uc5ec \uc601\ud654\ub77c\ub294 \ubbf8\ub514\uc5c4\uc744 \ud544\uc5f0\uc801\uc73c\ub85c \uc0ac\uc6a9\ud558\uac8c \ub41c \uac83\uc774\uace0, \"\uc6b0\ub9ac\ub294 \uc601\ud654 \uc790\uccb4\uc5d0\ub294 \uad00\uc2ec\uc774 \uc5c6\uc5c8\ub2e4. \ub610 \uc601\ud654\ub97c \uc774\uc6a9\ud560 \uac83\ub3c4 \uc0dd\uac01\ud558\uc9c0 \uc54a\uc558\ub2e4. \uc0c8\ub85c\uc6b4 \uacbd\ud5d8\uc774 \ub17c\ub9ac\uc801\uc73c\ub85c \ud544\uc5f0\uc801\uc73c\ub85c \uae30\ub300\ud558\uc9c0 \uc54a\uc740 \uac00\uc6b4\ub370 \ub098\ud0c0\ub0ac\ub2e4\"\uace0 \ub9d0\ud588\ub2e4."} {"question": "\ubca0\ucef4\uc774 \ubaa9\uc5d0 \ub450\ub978 \uae08\uc0c9\uacfc \ub179\uc0c9\uc73c\ub85c \ub41c \ubaa9\ub3c4\ub9ac\ub97c \ud1b5\ud574 \ub9e8\uccb4\uc2a4\ud130 \uc720\ub098\uc774\ud2f0\ub4dc \ud32c\ub4e4\uc774 \ub300\ud56d\ud55c \uad6c\ub2e8\uc8fc\ub294?", "paragraph": "2009\ub144 11\uc6d4, \uba54\uc774\uc800 \ub9ac\uadf8 \uc0ac\ucee4 2009 \uc2dc\uc98c\uc774 \uc885\ub8cc\ub41c \ud6c4, \ubca0\ucef4\uc774 2010\ub144 1\uc6d4\ubd80\ud130 \ub610\ub2e4\uc2dc \ubc00\ub780\uc73c\ub85c \uc784\ub300\ub97c \ub5a0\ub098\ub294 \uac83\uc774 \ud655\uc815\ub418\uc5c8\ub2e4. 2010\ub144 1\uc6d4 6\uc77c, \ubca0\ucef4\uc740 \ubc00\ub780 \ubcf5\uadc0\uc804\uc5d0\uc11c \uac1c\uc120\ud588\ub294\ub370, \uc81c\ub178\uc544\uc804\uc5d0 75\ubd84 \ucd9c\uc804\ud574 5-2 \uc2b9\ub9ac\uc5d0 \uc77c\uc870\ud588\ub2e4. 2010\ub144 2\uc6d4 16\uc77c, \ubca0\ucef4\uc740 \ub9e8\uccb4\uc2a4\ud130 \uc720\ub098\uc774\ud2f0\ub4dc\ub97c 2003\ub144\uc5d0 \ub5a0\ub09c \uc774\ub798 \ucc98\uc74c\uc73c\ub85c \uc804 \uc18c\uc18d \uad6c\ub2e8\uc744 \uc0c1\ub300\ub85c \uacbd\uae30\ub97c \uce58\ub800\ub2e4. \uadf8\ub294 \uc0b0 \uc2dc\ub85c\uc5d0\uc11c \ubc8c\uc5b4\uc9c4 \uacbd\uae30\uc5d0 76\ubd84 \ucd9c\uc804\ud574 \ud074\ub77c\ub80c\uc11c \uc138\uc774\ub3c4\ub974\ud504\uc640 \uad50\uccb4\ub418\uc5b4 \ub098\uac00\uae30 \uc804\uae4c\uc9c0 \ub6f0\uc5c8\uace0, \uacbd\uae30\ub294 \ub9e8\uccb4\uc2a4\ud130 \uc720\ub098\uc774\ud2f0\ub4dc\uc5d0 2-3\uc73c\ub85c \ud328\ud55c\ub290 \uac83\uc73c\ub85c \ub05d\ub0ac\ub2e4. 2010\ub144 3\uc6d4 10\uc77c, \ubca0\ucef4\uc740 2\ucc28\uc804\uc5d0\uc11c \uc62c\ub4dc \ud2b8\ub798\ud37c\ub4dc \ubcf5\uadc0\uc804\uc744 \uce58\ub800\ub2e4. \uadf8\ub294 \uc120\ubc1c\ub85c \ucd9c\uc804\ud558\uc9c0 \uc54a\uc558\uc9c0\ub9cc, 64\ubd84\uc5d0 \uc774\ub0d0\uce58\uc624 \uc544\ubc14\ud14c\uc640 \uad50\uccb4\ub418\uc5b4 \ub4e4\uc5b4\uac14\uace0, \ub9e8\uccb4\uc2a4\ud130 \uc720\ub098\uc774\ud2f0\ub4dc \uc9c0\uc9c0\uc790\ub4e4\uc774 \uadf8\ub97c \uae0d\uc815\uc801\uc73c\ub85c \ud658\uc601\ud574\uc8fc\uc5c8\ub2e4. \uacbd\uae30\ub294 \uc720\ub098\uc774\ud2f0\ub4dc\uc5d0 0-3 \uc644\uc804\ud788 \ubc00\ub9b0 \uac00\uc6b4\ub370\uc5d0 \uad50\uccb4\ub85c \ub4e4\uc5b4\uac14\uace0, \uc2b9\ubd80\ub294 \uc774\ubbf8 \uacb0\uc815\uc774 \ub098 \uc788\uc5c8\ub2e4. \uc774 \uacbd\uae30\ub294 \ubca0\ucef4\uc774 \uc62c\ub4dc \ud2b8\ub798\ud37c\ub4dc\uc5d0\uc11c \ub9e8\uccb4\uc2a4\ud130 \uc720\ub098\uc774\ud2f0\ub4dc\ub97c \uc0c1\ub300\ub85c \uce58\ub978 \uccab \uacbd\uae30\ub85c, \uadf8\ub294 \uc218 \ucc28\ub840 \uce21\uba74\uacfc \uad6c\uc11d\uc5d0\uc11c \ub4dd\uc810 \uae30\ud68c\ub97c \ucc3d\ucd9c\ud588\uc9c0\ub9cc, \ub9e8\uccb4\uc2a4\ud130 \uc720\ub098\uc774\ud2f0\ub4dc\uac00 \ubc00\ub780\uc744 \uc644\uc804\ud788 \uc81c\uc555\ud558\uba74\uc11c 0-4, \ud569\uacc4 2-7 \uc644\ud328\ub97c \ub2f9\ud588\ub2e4. \uacbd\uae30 \uc885\ub8cc\ub97c \uc54c\ub9ac\ub294 \ud638\uac01 \uc18c\ub9ac\uac00 \ub098\uace0, \uadf8\ub294 \ub9ec\ucef4 \uae00\ub808\uc774\uc800 \uad6c\ub2e8\uc8fc\uc5d0 \ubc18\ub300 \uc2dc\uc704\ub97c \ubc8c\uc774\ub294 \ub9e8\uccb4\uc2a4\ud130 \uc720\ub098\uc774\ud2f0\ub4dc \uc9c0\uc9c0\uc790\ub4e4\uc774 \uc900 \ub179\uc0c9\uacfc \uae08\uc0c9\uc73c\ub85c \ub41c \ubaa9\ub3c4\ub9ac\ub97c \ubaa9\uc5d0 \ub450\ub974\uba74\uc11c \uc0ac\uc18c\ud55c \ub17c\ub780\uc758 \ub300\uc0c1\uc774 \ub418\uc5c8\ub2e4. \uae00\ub808\uc774\uc800\uc5d0 \ub300\ud56d\ud558\ub294 \ud32c\ub4e4\uc758 \uc2dc\uc704\uac00 2010\ub144\uc5d0 \ub9e8\uccb4\uc2a4\ud130 \uc720\ub098\uc774\ud2f0\ub4dc \uc9c0\uc9c0\uc790 \uc870\ud569\uc73c\ub85c \uc870\uc9c1\ub418\uc5b4 \ud798\uc774 \uc2e4\ub838\ub294\ub370, \ub179\uc0c9\uacfc \uae08\uc0c9\uc73c\ub85c \ub41c \ubaa9\ub3c4\ub9ac\uac00 \ubc18 \uae00\ub808\uc774\uc800\uc758 \uc0c1\uc9d5\ubb3c\uc774\uc5c8\uace0, \ubca0\ucef4\uc740 \uc774 \uacb0\uc815\uc5d0 \uacf5\uac1c\uc801\uc73c\ub85c \ub3d9\ud589\uc744 \ud45c\uc758\ud588\ub2e4\uace0 \ub2e4\uc218\ub294 \ub118\uaca8\uc9da\uc5c8\ub2e4. \uadf8\ub7ec\ub098, \ubca0\ucef4\uc740 \ub098\uc911\uc5d0 \uc774 \ud589\ub3d9\uc5d0 \ub300\ud574 \uc9c8\ubb38\uc744 \ubc1b\uc558\uc744 \ub54c \uc2dc\uc704\ub294 \uc790\uc2e0\uacfc \uad00\ub828\uc774 \uc5c6\ub294 \uc77c\uc774\ub77c\uace0 \ub300\ub2f5\ud588\ub2e4.", "answer": "\ub9ec\ucef4 \uae00\ub808\uc774\uc800", "sentence": "\uadf8\ub294 \ub9ec\ucef4 \uae00\ub808\uc774\uc800 \uad6c\ub2e8\uc8fc\uc5d0 \ubc18\ub300 \uc2dc\uc704\ub97c \ubc8c\uc774\ub294 \ub9e8\uccb4\uc2a4\ud130 \uc720\ub098\uc774\ud2f0\ub4dc \uc9c0\uc9c0\uc790\ub4e4\uc774 \uc900 \ub179\uc0c9\uacfc \uae08\uc0c9\uc73c\ub85c \ub41c \ubaa9\ub3c4\ub9ac\ub97c \ubaa9\uc5d0 \ub450\ub974\uba74\uc11c \uc0ac\uc18c\ud55c \ub17c\ub780\uc758 \ub300\uc0c1\uc774 \ub418\uc5c8\ub2e4.", "paragraph_sentence": "2009\ub144 11\uc6d4, \uba54\uc774\uc800 \ub9ac\uadf8 \uc0ac\ucee4 2009 \uc2dc\uc98c\uc774 \uc885\ub8cc\ub41c \ud6c4, \ubca0\ucef4\uc774 2010\ub144 1\uc6d4\ubd80\ud130 \ub610\ub2e4\uc2dc \ubc00\ub780\uc73c\ub85c \uc784\ub300\ub97c \ub5a0\ub098\ub294 \uac83\uc774 \ud655\uc815\ub418\uc5c8\ub2e4. 2010\ub144 1\uc6d4 6\uc77c, \ubca0\ucef4\uc740 \ubc00\ub780 \ubcf5\uadc0\uc804\uc5d0\uc11c \uac1c\uc120\ud588\ub294\ub370, \uc81c\ub178\uc544\uc804\uc5d0 75\ubd84 \ucd9c\uc804\ud574 5-2 \uc2b9\ub9ac\uc5d0 \uc77c\uc870\ud588\ub2e4. 2010\ub144 2\uc6d4 16\uc77c, \ubca0\ucef4\uc740 \ub9e8\uccb4\uc2a4\ud130 \uc720\ub098\uc774\ud2f0\ub4dc\ub97c 2003\ub144\uc5d0 \ub5a0\ub09c \uc774\ub798 \ucc98\uc74c\uc73c\ub85c \uc804 \uc18c\uc18d \uad6c\ub2e8\uc744 \uc0c1\ub300\ub85c \uacbd\uae30\ub97c \uce58\ub800\ub2e4. \uadf8\ub294 \uc0b0 \uc2dc\ub85c\uc5d0\uc11c \ubc8c\uc5b4\uc9c4 \uacbd\uae30\uc5d0 76\ubd84 \ucd9c\uc804\ud574 \ud074\ub77c\ub80c\uc11c \uc138\uc774\ub3c4\ub974\ud504\uc640 \uad50\uccb4\ub418\uc5b4 \ub098\uac00\uae30 \uc804\uae4c\uc9c0 \ub6f0\uc5c8\uace0, \uacbd\uae30\ub294 \ub9e8\uccb4\uc2a4\ud130 \uc720\ub098\uc774\ud2f0\ub4dc\uc5d0 2-3\uc73c\ub85c \ud328\ud55c\ub290 \uac83\uc73c\ub85c \ub05d\ub0ac\ub2e4. 2010\ub144 3\uc6d4 10\uc77c, \ubca0\ucef4\uc740 2\ucc28\uc804\uc5d0\uc11c \uc62c\ub4dc \ud2b8\ub798\ud37c\ub4dc \ubcf5\uadc0\uc804\uc744 \uce58\ub800\ub2e4. \uadf8\ub294 \uc120\ubc1c\ub85c \ucd9c\uc804\ud558\uc9c0 \uc54a\uc558\uc9c0\ub9cc, 64\ubd84\uc5d0 \uc774\ub0d0\uce58\uc624 \uc544\ubc14\ud14c\uc640 \uad50\uccb4\ub418\uc5b4 \ub4e4\uc5b4\uac14\uace0, \ub9e8\uccb4\uc2a4\ud130 \uc720\ub098\uc774\ud2f0\ub4dc \uc9c0\uc9c0\uc790\ub4e4\uc774 \uadf8\ub97c \uae0d\uc815\uc801\uc73c\ub85c \ud658\uc601\ud574\uc8fc\uc5c8\ub2e4. \uacbd\uae30\ub294 \uc720\ub098\uc774\ud2f0\ub4dc\uc5d0 0-3 \uc644\uc804\ud788 \ubc00\ub9b0 \uac00\uc6b4\ub370\uc5d0 \uad50\uccb4\ub85c \ub4e4\uc5b4\uac14\uace0, \uc2b9\ubd80\ub294 \uc774\ubbf8 \uacb0\uc815\uc774 \ub098 \uc788\uc5c8\ub2e4. \uc774 \uacbd\uae30\ub294 \ubca0\ucef4\uc774 \uc62c\ub4dc \ud2b8\ub798\ud37c\ub4dc\uc5d0\uc11c \ub9e8\uccb4\uc2a4\ud130 \uc720\ub098\uc774\ud2f0\ub4dc\ub97c \uc0c1\ub300\ub85c \uce58\ub978 \uccab \uacbd\uae30\ub85c, \uadf8\ub294 \uc218 \ucc28\ub840 \uce21\uba74\uacfc \uad6c\uc11d\uc5d0\uc11c \ub4dd\uc810 \uae30\ud68c\ub97c \ucc3d\ucd9c\ud588\uc9c0\ub9cc, \ub9e8\uccb4\uc2a4\ud130 \uc720\ub098\uc774\ud2f0\ub4dc\uac00 \ubc00\ub780\uc744 \uc644\uc804\ud788 \uc81c\uc555\ud558\uba74\uc11c 0-4, \ud569\uacc4 2-7 \uc644\ud328\ub97c \ub2f9\ud588\ub2e4. \uacbd\uae30 \uc885\ub8cc\ub97c \uc54c\ub9ac\ub294 \ud638\uac01 \uc18c\ub9ac\uac00 \ub098\uace0, \uadf8\ub294 \ub9ec\ucef4 \uae00\ub808\uc774\uc800 \uad6c\ub2e8\uc8fc\uc5d0 \ubc18\ub300 \uc2dc\uc704\ub97c \ubc8c\uc774\ub294 \ub9e8\uccb4\uc2a4\ud130 \uc720\ub098\uc774\ud2f0\ub4dc \uc9c0\uc9c0\uc790\ub4e4\uc774 \uc900 \ub179\uc0c9\uacfc \uae08\uc0c9\uc73c\ub85c \ub41c \ubaa9\ub3c4\ub9ac\ub97c \ubaa9\uc5d0 \ub450\ub974\uba74\uc11c \uc0ac\uc18c\ud55c \ub17c\ub780\uc758 \ub300\uc0c1\uc774 \ub418\uc5c8\ub2e4. \uae00\ub808\uc774\uc800\uc5d0 \ub300\ud56d\ud558\ub294 \ud32c\ub4e4\uc758 \uc2dc\uc704\uac00 2010\ub144\uc5d0 \ub9e8\uccb4\uc2a4\ud130 \uc720\ub098\uc774\ud2f0\ub4dc \uc9c0\uc9c0\uc790 \uc870\ud569\uc73c\ub85c \uc870\uc9c1\ub418\uc5b4 \ud798\uc774 \uc2e4\ub838\ub294\ub370, \ub179\uc0c9\uacfc \uae08\uc0c9\uc73c\ub85c \ub41c \ubaa9\ub3c4\ub9ac\uac00 \ubc18 \uae00\ub808\uc774\uc800\uc758 \uc0c1\uc9d5\ubb3c\uc774\uc5c8\uace0, \ubca0\ucef4\uc740 \uc774 \uacb0\uc815\uc5d0 \uacf5\uac1c\uc801\uc73c\ub85c \ub3d9\ud589\uc744 \ud45c\uc758\ud588\ub2e4\uace0 \ub2e4\uc218\ub294 \ub118\uaca8\uc9da\uc5c8\ub2e4. \uadf8\ub7ec\ub098, \ubca0\ucef4\uc740 \ub098\uc911\uc5d0 \uc774 \ud589\ub3d9\uc5d0 \ub300\ud574 \uc9c8\ubb38\uc744 \ubc1b\uc558\uc744 \ub54c \uc2dc\uc704\ub294 \uc790\uc2e0\uacfc \uad00\ub828\uc774 \uc5c6\ub294 \uc77c\uc774\ub77c\uace0 \ub300\ub2f5\ud588\ub2e4.", "paragraph_answer": "2009\ub144 11\uc6d4, \uba54\uc774\uc800 \ub9ac\uadf8 \uc0ac\ucee4 2009 \uc2dc\uc98c\uc774 \uc885\ub8cc\ub41c \ud6c4, \ubca0\ucef4\uc774 2010\ub144 1\uc6d4\ubd80\ud130 \ub610\ub2e4\uc2dc \ubc00\ub780\uc73c\ub85c \uc784\ub300\ub97c \ub5a0\ub098\ub294 \uac83\uc774 \ud655\uc815\ub418\uc5c8\ub2e4. 2010\ub144 1\uc6d4 6\uc77c, \ubca0\ucef4\uc740 \ubc00\ub780 \ubcf5\uadc0\uc804\uc5d0\uc11c \uac1c\uc120\ud588\ub294\ub370, \uc81c\ub178\uc544\uc804\uc5d0 75\ubd84 \ucd9c\uc804\ud574 5-2 \uc2b9\ub9ac\uc5d0 \uc77c\uc870\ud588\ub2e4. 2010\ub144 2\uc6d4 16\uc77c, \ubca0\ucef4\uc740 \ub9e8\uccb4\uc2a4\ud130 \uc720\ub098\uc774\ud2f0\ub4dc\ub97c 2003\ub144\uc5d0 \ub5a0\ub09c \uc774\ub798 \ucc98\uc74c\uc73c\ub85c \uc804 \uc18c\uc18d \uad6c\ub2e8\uc744 \uc0c1\ub300\ub85c \uacbd\uae30\ub97c \uce58\ub800\ub2e4. \uadf8\ub294 \uc0b0 \uc2dc\ub85c\uc5d0\uc11c \ubc8c\uc5b4\uc9c4 \uacbd\uae30\uc5d0 76\ubd84 \ucd9c\uc804\ud574 \ud074\ub77c\ub80c\uc11c \uc138\uc774\ub3c4\ub974\ud504\uc640 \uad50\uccb4\ub418\uc5b4 \ub098\uac00\uae30 \uc804\uae4c\uc9c0 \ub6f0\uc5c8\uace0, \uacbd\uae30\ub294 \ub9e8\uccb4\uc2a4\ud130 \uc720\ub098\uc774\ud2f0\ub4dc\uc5d0 2-3\uc73c\ub85c \ud328\ud55c\ub290 \uac83\uc73c\ub85c \ub05d\ub0ac\ub2e4. 2010\ub144 3\uc6d4 10\uc77c, \ubca0\ucef4\uc740 2\ucc28\uc804\uc5d0\uc11c \uc62c\ub4dc \ud2b8\ub798\ud37c\ub4dc \ubcf5\uadc0\uc804\uc744 \uce58\ub800\ub2e4. \uadf8\ub294 \uc120\ubc1c\ub85c \ucd9c\uc804\ud558\uc9c0 \uc54a\uc558\uc9c0\ub9cc, 64\ubd84\uc5d0 \uc774\ub0d0\uce58\uc624 \uc544\ubc14\ud14c\uc640 \uad50\uccb4\ub418\uc5b4 \ub4e4\uc5b4\uac14\uace0, \ub9e8\uccb4\uc2a4\ud130 \uc720\ub098\uc774\ud2f0\ub4dc \uc9c0\uc9c0\uc790\ub4e4\uc774 \uadf8\ub97c \uae0d\uc815\uc801\uc73c\ub85c \ud658\uc601\ud574\uc8fc\uc5c8\ub2e4. \uacbd\uae30\ub294 \uc720\ub098\uc774\ud2f0\ub4dc\uc5d0 0-3 \uc644\uc804\ud788 \ubc00\ub9b0 \uac00\uc6b4\ub370\uc5d0 \uad50\uccb4\ub85c \ub4e4\uc5b4\uac14\uace0, \uc2b9\ubd80\ub294 \uc774\ubbf8 \uacb0\uc815\uc774 \ub098 \uc788\uc5c8\ub2e4. \uc774 \uacbd\uae30\ub294 \ubca0\ucef4\uc774 \uc62c\ub4dc \ud2b8\ub798\ud37c\ub4dc\uc5d0\uc11c \ub9e8\uccb4\uc2a4\ud130 \uc720\ub098\uc774\ud2f0\ub4dc\ub97c \uc0c1\ub300\ub85c \uce58\ub978 \uccab \uacbd\uae30\ub85c, \uadf8\ub294 \uc218 \ucc28\ub840 \uce21\uba74\uacfc \uad6c\uc11d\uc5d0\uc11c \ub4dd\uc810 \uae30\ud68c\ub97c \ucc3d\ucd9c\ud588\uc9c0\ub9cc, \ub9e8\uccb4\uc2a4\ud130 \uc720\ub098\uc774\ud2f0\ub4dc\uac00 \ubc00\ub780\uc744 \uc644\uc804\ud788 \uc81c\uc555\ud558\uba74\uc11c 0-4, \ud569\uacc4 2-7 \uc644\ud328\ub97c \ub2f9\ud588\ub2e4. \uacbd\uae30 \uc885\ub8cc\ub97c \uc54c\ub9ac\ub294 \ud638\uac01 \uc18c\ub9ac\uac00 \ub098\uace0, \uadf8\ub294 \ub9ec\ucef4 \uae00\ub808\uc774\uc800 \uad6c\ub2e8\uc8fc\uc5d0 \ubc18\ub300 \uc2dc\uc704\ub97c \ubc8c\uc774\ub294 \ub9e8\uccb4\uc2a4\ud130 \uc720\ub098\uc774\ud2f0\ub4dc \uc9c0\uc9c0\uc790\ub4e4\uc774 \uc900 \ub179\uc0c9\uacfc \uae08\uc0c9\uc73c\ub85c \ub41c \ubaa9\ub3c4\ub9ac\ub97c \ubaa9\uc5d0 \ub450\ub974\uba74\uc11c \uc0ac\uc18c\ud55c \ub17c\ub780\uc758 \ub300\uc0c1\uc774 \ub418\uc5c8\ub2e4. \uae00\ub808\uc774\uc800\uc5d0 \ub300\ud56d\ud558\ub294 \ud32c\ub4e4\uc758 \uc2dc\uc704\uac00 2010\ub144\uc5d0 \ub9e8\uccb4\uc2a4\ud130 \uc720\ub098\uc774\ud2f0\ub4dc \uc9c0\uc9c0\uc790 \uc870\ud569\uc73c\ub85c \uc870\uc9c1\ub418\uc5b4 \ud798\uc774 \uc2e4\ub838\ub294\ub370, \ub179\uc0c9\uacfc \uae08\uc0c9\uc73c\ub85c \ub41c \ubaa9\ub3c4\ub9ac\uac00 \ubc18 \uae00\ub808\uc774\uc800\uc758 \uc0c1\uc9d5\ubb3c\uc774\uc5c8\uace0, \ubca0\ucef4\uc740 \uc774 \uacb0\uc815\uc5d0 \uacf5\uac1c\uc801\uc73c\ub85c \ub3d9\ud589\uc744 \ud45c\uc758\ud588\ub2e4\uace0 \ub2e4\uc218\ub294 \ub118\uaca8\uc9da\uc5c8\ub2e4. \uadf8\ub7ec\ub098, \ubca0\ucef4\uc740 \ub098\uc911\uc5d0 \uc774 \ud589\ub3d9\uc5d0 \ub300\ud574 \uc9c8\ubb38\uc744 \ubc1b\uc558\uc744 \ub54c \uc2dc\uc704\ub294 \uc790\uc2e0\uacfc \uad00\ub828\uc774 \uc5c6\ub294 \uc77c\uc774\ub77c\uace0 \ub300\ub2f5\ud588\ub2e4.", "sentence_answer": "\uadf8\ub294 \ub9ec\ucef4 \uae00\ub808\uc774\uc800 \uad6c\ub2e8\uc8fc\uc5d0 \ubc18\ub300 \uc2dc\uc704\ub97c \ubc8c\uc774\ub294 \ub9e8\uccb4\uc2a4\ud130 \uc720\ub098\uc774\ud2f0\ub4dc \uc9c0\uc9c0\uc790\ub4e4\uc774 \uc900 \ub179\uc0c9\uacfc \uae08\uc0c9\uc73c\ub85c \ub41c \ubaa9\ub3c4\ub9ac\ub97c \ubaa9\uc5d0 \ub450\ub974\uba74\uc11c \uc0ac\uc18c\ud55c \ub17c\ub780\uc758 \ub300\uc0c1\uc774 \ub418\uc5c8\ub2e4.", "paragraph_id": "6528559-27-0", "paragraph_question": "question: \ubca0\ucef4\uc774 \ubaa9\uc5d0 \ub450\ub978 \uae08\uc0c9\uacfc \ub179\uc0c9\uc73c\ub85c \ub41c \ubaa9\ub3c4\ub9ac\ub97c \ud1b5\ud574 \ub9e8\uccb4\uc2a4\ud130 \uc720\ub098\uc774\ud2f0\ub4dc \ud32c\ub4e4\uc774 \ub300\ud56d\ud55c \uad6c\ub2e8\uc8fc\ub294?, context: 2009\ub144 11\uc6d4, \uba54\uc774\uc800 \ub9ac\uadf8 \uc0ac\ucee4 2009 \uc2dc\uc98c\uc774 \uc885\ub8cc\ub41c \ud6c4, \ubca0\ucef4\uc774 2010\ub144 1\uc6d4\ubd80\ud130 \ub610\ub2e4\uc2dc \ubc00\ub780\uc73c\ub85c \uc784\ub300\ub97c \ub5a0\ub098\ub294 \uac83\uc774 \ud655\uc815\ub418\uc5c8\ub2e4. 2010\ub144 1\uc6d4 6\uc77c, \ubca0\ucef4\uc740 \ubc00\ub780 \ubcf5\uadc0\uc804\uc5d0\uc11c \uac1c\uc120\ud588\ub294\ub370, \uc81c\ub178\uc544\uc804\uc5d0 75\ubd84 \ucd9c\uc804\ud574 5-2 \uc2b9\ub9ac\uc5d0 \uc77c\uc870\ud588\ub2e4. 2010\ub144 2\uc6d4 16\uc77c, \ubca0\ucef4\uc740 \ub9e8\uccb4\uc2a4\ud130 \uc720\ub098\uc774\ud2f0\ub4dc\ub97c 2003\ub144\uc5d0 \ub5a0\ub09c \uc774\ub798 \ucc98\uc74c\uc73c\ub85c \uc804 \uc18c\uc18d \uad6c\ub2e8\uc744 \uc0c1\ub300\ub85c \uacbd\uae30\ub97c \uce58\ub800\ub2e4. \uadf8\ub294 \uc0b0 \uc2dc\ub85c\uc5d0\uc11c \ubc8c\uc5b4\uc9c4 \uacbd\uae30\uc5d0 76\ubd84 \ucd9c\uc804\ud574 \ud074\ub77c\ub80c\uc11c \uc138\uc774\ub3c4\ub974\ud504\uc640 \uad50\uccb4\ub418\uc5b4 \ub098\uac00\uae30 \uc804\uae4c\uc9c0 \ub6f0\uc5c8\uace0, \uacbd\uae30\ub294 \ub9e8\uccb4\uc2a4\ud130 \uc720\ub098\uc774\ud2f0\ub4dc\uc5d0 2-3\uc73c\ub85c \ud328\ud55c\ub290 \uac83\uc73c\ub85c \ub05d\ub0ac\ub2e4. 2010\ub144 3\uc6d4 10\uc77c, \ubca0\ucef4\uc740 2\ucc28\uc804\uc5d0\uc11c \uc62c\ub4dc \ud2b8\ub798\ud37c\ub4dc \ubcf5\uadc0\uc804\uc744 \uce58\ub800\ub2e4. \uadf8\ub294 \uc120\ubc1c\ub85c \ucd9c\uc804\ud558\uc9c0 \uc54a\uc558\uc9c0\ub9cc, 64\ubd84\uc5d0 \uc774\ub0d0\uce58\uc624 \uc544\ubc14\ud14c\uc640 \uad50\uccb4\ub418\uc5b4 \ub4e4\uc5b4\uac14\uace0, \ub9e8\uccb4\uc2a4\ud130 \uc720\ub098\uc774\ud2f0\ub4dc \uc9c0\uc9c0\uc790\ub4e4\uc774 \uadf8\ub97c \uae0d\uc815\uc801\uc73c\ub85c \ud658\uc601\ud574\uc8fc\uc5c8\ub2e4. \uacbd\uae30\ub294 \uc720\ub098\uc774\ud2f0\ub4dc\uc5d0 0-3 \uc644\uc804\ud788 \ubc00\ub9b0 \uac00\uc6b4\ub370\uc5d0 \uad50\uccb4\ub85c \ub4e4\uc5b4\uac14\uace0, \uc2b9\ubd80\ub294 \uc774\ubbf8 \uacb0\uc815\uc774 \ub098 \uc788\uc5c8\ub2e4. \uc774 \uacbd\uae30\ub294 \ubca0\ucef4\uc774 \uc62c\ub4dc \ud2b8\ub798\ud37c\ub4dc\uc5d0\uc11c \ub9e8\uccb4\uc2a4\ud130 \uc720\ub098\uc774\ud2f0\ub4dc\ub97c \uc0c1\ub300\ub85c \uce58\ub978 \uccab \uacbd\uae30\ub85c, \uadf8\ub294 \uc218 \ucc28\ub840 \uce21\uba74\uacfc \uad6c\uc11d\uc5d0\uc11c \ub4dd\uc810 \uae30\ud68c\ub97c \ucc3d\ucd9c\ud588\uc9c0\ub9cc, \ub9e8\uccb4\uc2a4\ud130 \uc720\ub098\uc774\ud2f0\ub4dc\uac00 \ubc00\ub780\uc744 \uc644\uc804\ud788 \uc81c\uc555\ud558\uba74\uc11c 0-4, \ud569\uacc4 2-7 \uc644\ud328\ub97c \ub2f9\ud588\ub2e4. \uacbd\uae30 \uc885\ub8cc\ub97c \uc54c\ub9ac\ub294 \ud638\uac01 \uc18c\ub9ac\uac00 \ub098\uace0, \uadf8\ub294 \ub9ec\ucef4 \uae00\ub808\uc774\uc800 \uad6c\ub2e8\uc8fc\uc5d0 \ubc18\ub300 \uc2dc\uc704\ub97c \ubc8c\uc774\ub294 \ub9e8\uccb4\uc2a4\ud130 \uc720\ub098\uc774\ud2f0\ub4dc \uc9c0\uc9c0\uc790\ub4e4\uc774 \uc900 \ub179\uc0c9\uacfc \uae08\uc0c9\uc73c\ub85c \ub41c \ubaa9\ub3c4\ub9ac\ub97c \ubaa9\uc5d0 \ub450\ub974\uba74\uc11c \uc0ac\uc18c\ud55c \ub17c\ub780\uc758 \ub300\uc0c1\uc774 \ub418\uc5c8\ub2e4. \uae00\ub808\uc774\uc800\uc5d0 \ub300\ud56d\ud558\ub294 \ud32c\ub4e4\uc758 \uc2dc\uc704\uac00 2010\ub144\uc5d0 \ub9e8\uccb4\uc2a4\ud130 \uc720\ub098\uc774\ud2f0\ub4dc \uc9c0\uc9c0\uc790 \uc870\ud569\uc73c\ub85c \uc870\uc9c1\ub418\uc5b4 \ud798\uc774 \uc2e4\ub838\ub294\ub370, \ub179\uc0c9\uacfc \uae08\uc0c9\uc73c\ub85c \ub41c \ubaa9\ub3c4\ub9ac\uac00 \ubc18 \uae00\ub808\uc774\uc800\uc758 \uc0c1\uc9d5\ubb3c\uc774\uc5c8\uace0, \ubca0\ucef4\uc740 \uc774 \uacb0\uc815\uc5d0 \uacf5\uac1c\uc801\uc73c\ub85c \ub3d9\ud589\uc744 \ud45c\uc758\ud588\ub2e4\uace0 \ub2e4\uc218\ub294 \ub118\uaca8\uc9da\uc5c8\ub2e4. \uadf8\ub7ec\ub098, \ubca0\ucef4\uc740 \ub098\uc911\uc5d0 \uc774 \ud589\ub3d9\uc5d0 \ub300\ud574 \uc9c8\ubb38\uc744 \ubc1b\uc558\uc744 \ub54c \uc2dc\uc704\ub294 \uc790\uc2e0\uacfc \uad00\ub828\uc774 \uc5c6\ub294 \uc77c\uc774\ub77c\uace0 \ub300\ub2f5\ud588\ub2e4."} {"question": "\uce5c\ub178\ud30c\uc778 \uac1c\ud601\ud30c \ub2f9\uc6d0\ub4e4\uc774 \ud0c8\ub2f9 \ud6c4 \ud55c\ub098\ub77c\ub2f9, \uac1c\ud601\uad6d\ubbfc\uc815\ub2f9\uacfc \ud568\uaed8 \ucc3d\uc124\ud55c \ub2f9\uc758 \uc774\ub984\uc740?", "paragraph": "\uadf8\ub7ec\ub098 \ub0b4\uac01\uc81c \uac1c\ud5cc \ubb38\uc81c\ub97c \ub450\uace0 \uc5ec\uad8c \ubd84\uc5f4\uc774 \uc788\uc5c8\uace0, \uc5ec\ub7ec \uc2a4\uce94\ub4e4\uc774 \ud130\uc838 \ubbfc\uc2ec \uc774\ubc18\uc774 \uc77c\uc5b4\ub0ac\ub2e4. \uc0c8\uc815\uce58\uad6d\ubbfc\ud68c\uc758\ub294 2000\ub144 \uc0c8\ucc9c\ub144\ubbfc\uc8fc\ub2f9\uc73c\ub85c \uac1c\ud3b8\ub418\uc5b4 \ud574\uccb4\ub418\uc5c8\uace0, \uc774\uc5b4 \ucd1d\uc120\uc5d0\uc11c \uc758\uc11d\uc218\ub294 \ub298\uc5b4\ub0ac\uc73c\ub098 \ud55c\ub098\ub77c\ub2f9\uc5d0 \ubc00\ub824 \ud328\ud588\ub2e4. \uc774\ud6c4 \uae40\ub300\uc911\uc740 \ub9ac\ub354\uc2ed\uc744 \uc783\uc5c8\uace0, \uacf5\ub3d9\uc5ec\ub2f9 \uacf5\uc870 \ud30c\uae30\uae4c\uc9c0 \uc77c\uc5b4\ub098\uba74\uc11c \uc815\uad8c\uc758 \uad6d\uc815 \uc6b4\uc601\uc740 \uc5b4\ub824\uc6c0\uc744 \uacaa\uac8c \ub418\uc5c8\ub2e4. \ubbfc\uc8fc\ub2f9\uc740 2002\ub144 \uc9c0\ubc29\uc120\uac70\uc5d0\uc11c \ud328\ud588\uace0, \uacb0\uad6d \uae40\ub300\uc911\uc740 \ud0c8\ub2f9\ud558\uc600\ub2e4(2000\ub144 \ucd1d\uc120 \uc804 \uc0c8\ucc9c\ub144\ubbfc\uc8fc\ub2f9 \ucc3d\ub2f9). \uadf8\ub7ec\ub098 \ub3d9\uad50\ub3d9\uacc4\ub294 \uc5ec\uc804\ud788 \ub2f9\uad8c\uc744 \uc7a5\uc545\ud558\uace0 \uc788\uc5c8\ub2e4. \ud558\uc9c0\ub9cc \uac19\uc740 \ud574 \ub300\uc120 \ud6c4\ubcf4\ub97c \uc120\ucd9c\ud558\uae30 \uc704\ud55c \uad6d\ubbfc \uacbd\uc120\uc81c\uac00 \ub3d9\uad50\ub3d9\uacc4\uc758 \uc808\ub300\uc801 \uad8c\uc704\uac00 \ud754\ub4e4\ub9ac\uae30 \uc2dc\uc791\ud558\ub294\ub370, \ub2f9\uc2dc '\ub178\ud48d'\uc758 \uacb0\uacfc\ub85c \ub178\ubb34\ud604\uc758 \uc9c0\uc9c0\uc728\uc774 \uae09\uc0c1\uc2b9\ud558\uace0 \ub3d9\uad50\ub3d9\uacc4 \ud6c4\ubcf4 \ud55c\ud654\uac11\uc740 \ub099\uc120\ud55c \uac83\uc774\ub2e4. \uacb0\uad6d, \ub300\uc120 \ud6c4\ubcf4\ub85c\ub294 \ub178\ubb34\ud604\uc774 \uc120\ucd9c\ub418\uc5c8\ub2e4. \ub300\uc120 \uacb0\uacfc, \ub178\ubb34\ud604\uc758 \ub2f9\uc120\uc73c\ub85c \uc0c8\ucc9c\ub144\ubbfc\uc8fc\ub2f9\uc740 \uc815\uad8c\uc744 \uc5f0\uc7a5\ud588\uc73c\ub098, \ub2f9\uc2dc \ub2f9\uad8c\uc744 \uc7a5\uc545\ud558\uace0 \uc788\uc5c8\ub358 \ub3d9\uad50\ub3d9\uacc4\uc640 \ub178\ubb34\ud604 \uce21 \uac04\uc758 \uad00\uacc4 \uc545\ud654\ub294 \uacc4\uc18d\ub418\uc5c8\uace0, \uc774 \uc640\uc911\uc5d0 \uc18c\uc704 '\uce5c\ub178'\ub77c\uace0 \ubd88\ub9ac\ub358 \uac1c\ud601\ud30c \ub2f9\uc6d0\ub4e4\uc774 \ud0c8\ub2f9\ud558\uc600\ub2e4. \uc774\ub4e4\uc740 \ud55c\ub098\ub77c\ub2f9 \ud0c8\ub2f9\ud30c \uc77c\ubd80, \uac1c\ud601\uad6d\ubbfc\uc815\ub2f9\uacfc \uc5f0\ub300\ud55c \ub4a4 \uc5f4\ub9b0\uc6b0\ub9ac\ub2f9\uc744 \ucc3d\ub2f9\ud588\ub2e4.", "answer": "\uc5f4\ub9b0\uc6b0\ub9ac\ub2f9", "sentence": "\uc774\ub4e4\uc740 \ud55c\ub098\ub77c\ub2f9 \ud0c8\ub2f9\ud30c \uc77c\ubd80, \uac1c\ud601\uad6d\ubbfc\uc815\ub2f9\uacfc \uc5f0\ub300\ud55c \ub4a4 \uc5f4\ub9b0\uc6b0\ub9ac\ub2f9 \uc744 \ucc3d\ub2f9\ud588\ub2e4.", "paragraph_sentence": "\uadf8\ub7ec\ub098 \ub0b4\uac01\uc81c \uac1c\ud5cc \ubb38\uc81c\ub97c \ub450\uace0 \uc5ec\uad8c \ubd84\uc5f4\uc774 \uc788\uc5c8\uace0, \uc5ec\ub7ec \uc2a4\uce94\ub4e4\uc774 \ud130\uc838 \ubbfc\uc2ec \uc774\ubc18\uc774 \uc77c\uc5b4\ub0ac\ub2e4. \uc0c8\uc815\uce58\uad6d\ubbfc\ud68c\uc758\ub294 2000\ub144 \uc0c8\ucc9c\ub144\ubbfc\uc8fc\ub2f9\uc73c\ub85c \uac1c\ud3b8\ub418\uc5b4 \ud574\uccb4\ub418\uc5c8\uace0, \uc774\uc5b4 \ucd1d\uc120\uc5d0\uc11c \uc758\uc11d\uc218\ub294 \ub298\uc5b4\ub0ac\uc73c\ub098 \ud55c\ub098\ub77c\ub2f9\uc5d0 \ubc00\ub824 \ud328\ud588\ub2e4. \uc774\ud6c4 \uae40\ub300\uc911\uc740 \ub9ac\ub354\uc2ed\uc744 \uc783\uc5c8\uace0, \uacf5\ub3d9\uc5ec\ub2f9 \uacf5\uc870 \ud30c\uae30\uae4c\uc9c0 \uc77c\uc5b4\ub098\uba74\uc11c \uc815\uad8c\uc758 \uad6d\uc815 \uc6b4\uc601\uc740 \uc5b4\ub824\uc6c0\uc744 \uacaa\uac8c \ub418\uc5c8\ub2e4. \ubbfc\uc8fc\ub2f9\uc740 2002\ub144 \uc9c0\ubc29\uc120\uac70\uc5d0\uc11c \ud328\ud588\uace0, \uacb0\uad6d \uae40\ub300\uc911\uc740 \ud0c8\ub2f9\ud558\uc600\ub2e4(2000\ub144 \ucd1d\uc120 \uc804 \uc0c8\ucc9c\ub144\ubbfc\uc8fc\ub2f9 \ucc3d\ub2f9). \uadf8\ub7ec\ub098 \ub3d9\uad50\ub3d9\uacc4\ub294 \uc5ec\uc804\ud788 \ub2f9\uad8c\uc744 \uc7a5\uc545\ud558\uace0 \uc788\uc5c8\ub2e4. \ud558\uc9c0\ub9cc \uac19\uc740 \ud574 \ub300\uc120 \ud6c4\ubcf4\ub97c \uc120\ucd9c\ud558\uae30 \uc704\ud55c \uad6d\ubbfc \uacbd\uc120\uc81c\uac00 \ub3d9\uad50\ub3d9\uacc4\uc758 \uc808\ub300\uc801 \uad8c\uc704\uac00 \ud754\ub4e4\ub9ac\uae30 \uc2dc\uc791\ud558\ub294\ub370, \ub2f9\uc2dc '\ub178\ud48d'\uc758 \uacb0\uacfc\ub85c \ub178\ubb34\ud604\uc758 \uc9c0\uc9c0\uc728\uc774 \uae09\uc0c1\uc2b9\ud558\uace0 \ub3d9\uad50\ub3d9\uacc4 \ud6c4\ubcf4 \ud55c\ud654\uac11\uc740 \ub099\uc120\ud55c \uac83\uc774\ub2e4. \uacb0\uad6d, \ub300\uc120 \ud6c4\ubcf4\ub85c\ub294 \ub178\ubb34\ud604\uc774 \uc120\ucd9c\ub418\uc5c8\ub2e4. \ub300\uc120 \uacb0\uacfc, \ub178\ubb34\ud604\uc758 \ub2f9\uc120\uc73c\ub85c \uc0c8\ucc9c\ub144\ubbfc\uc8fc\ub2f9\uc740 \uc815\uad8c\uc744 \uc5f0\uc7a5\ud588\uc73c\ub098, \ub2f9\uc2dc \ub2f9\uad8c\uc744 \uc7a5\uc545\ud558\uace0 \uc788\uc5c8\ub358 \ub3d9\uad50\ub3d9\uacc4\uc640 \ub178\ubb34\ud604 \uce21 \uac04\uc758 \uad00\uacc4 \uc545\ud654\ub294 \uacc4\uc18d\ub418\uc5c8\uace0, \uc774 \uc640\uc911\uc5d0 \uc18c\uc704 '\uce5c\ub178'\ub77c\uace0 \ubd88\ub9ac\ub358 \uac1c\ud601\ud30c \ub2f9\uc6d0\ub4e4\uc774 \ud0c8\ub2f9\ud558\uc600\ub2e4. \uc774\ub4e4\uc740 \ud55c\ub098\ub77c\ub2f9 \ud0c8\ub2f9\ud30c \uc77c\ubd80, \uac1c\ud601\uad6d\ubbfc\uc815\ub2f9\uacfc \uc5f0\ub300\ud55c \ub4a4 \uc5f4\ub9b0\uc6b0\ub9ac\ub2f9 \uc744 \ucc3d\ub2f9\ud588\ub2e4. ", "paragraph_answer": "\uadf8\ub7ec\ub098 \ub0b4\uac01\uc81c \uac1c\ud5cc \ubb38\uc81c\ub97c \ub450\uace0 \uc5ec\uad8c \ubd84\uc5f4\uc774 \uc788\uc5c8\uace0, \uc5ec\ub7ec \uc2a4\uce94\ub4e4\uc774 \ud130\uc838 \ubbfc\uc2ec \uc774\ubc18\uc774 \uc77c\uc5b4\ub0ac\ub2e4. \uc0c8\uc815\uce58\uad6d\ubbfc\ud68c\uc758\ub294 2000\ub144 \uc0c8\ucc9c\ub144\ubbfc\uc8fc\ub2f9\uc73c\ub85c \uac1c\ud3b8\ub418\uc5b4 \ud574\uccb4\ub418\uc5c8\uace0, \uc774\uc5b4 \ucd1d\uc120\uc5d0\uc11c \uc758\uc11d\uc218\ub294 \ub298\uc5b4\ub0ac\uc73c\ub098 \ud55c\ub098\ub77c\ub2f9\uc5d0 \ubc00\ub824 \ud328\ud588\ub2e4. \uc774\ud6c4 \uae40\ub300\uc911\uc740 \ub9ac\ub354\uc2ed\uc744 \uc783\uc5c8\uace0, \uacf5\ub3d9\uc5ec\ub2f9 \uacf5\uc870 \ud30c\uae30\uae4c\uc9c0 \uc77c\uc5b4\ub098\uba74\uc11c \uc815\uad8c\uc758 \uad6d\uc815 \uc6b4\uc601\uc740 \uc5b4\ub824\uc6c0\uc744 \uacaa\uac8c \ub418\uc5c8\ub2e4. \ubbfc\uc8fc\ub2f9\uc740 2002\ub144 \uc9c0\ubc29\uc120\uac70\uc5d0\uc11c \ud328\ud588\uace0, \uacb0\uad6d \uae40\ub300\uc911\uc740 \ud0c8\ub2f9\ud558\uc600\ub2e4(2000\ub144 \ucd1d\uc120 \uc804 \uc0c8\ucc9c\ub144\ubbfc\uc8fc\ub2f9 \ucc3d\ub2f9). \uadf8\ub7ec\ub098 \ub3d9\uad50\ub3d9\uacc4\ub294 \uc5ec\uc804\ud788 \ub2f9\uad8c\uc744 \uc7a5\uc545\ud558\uace0 \uc788\uc5c8\ub2e4. \ud558\uc9c0\ub9cc \uac19\uc740 \ud574 \ub300\uc120 \ud6c4\ubcf4\ub97c \uc120\ucd9c\ud558\uae30 \uc704\ud55c \uad6d\ubbfc \uacbd\uc120\uc81c\uac00 \ub3d9\uad50\ub3d9\uacc4\uc758 \uc808\ub300\uc801 \uad8c\uc704\uac00 \ud754\ub4e4\ub9ac\uae30 \uc2dc\uc791\ud558\ub294\ub370, \ub2f9\uc2dc '\ub178\ud48d'\uc758 \uacb0\uacfc\ub85c \ub178\ubb34\ud604\uc758 \uc9c0\uc9c0\uc728\uc774 \uae09\uc0c1\uc2b9\ud558\uace0 \ub3d9\uad50\ub3d9\uacc4 \ud6c4\ubcf4 \ud55c\ud654\uac11\uc740 \ub099\uc120\ud55c \uac83\uc774\ub2e4. \uacb0\uad6d, \ub300\uc120 \ud6c4\ubcf4\ub85c\ub294 \ub178\ubb34\ud604\uc774 \uc120\ucd9c\ub418\uc5c8\ub2e4. \ub300\uc120 \uacb0\uacfc, \ub178\ubb34\ud604\uc758 \ub2f9\uc120\uc73c\ub85c \uc0c8\ucc9c\ub144\ubbfc\uc8fc\ub2f9\uc740 \uc815\uad8c\uc744 \uc5f0\uc7a5\ud588\uc73c\ub098, \ub2f9\uc2dc \ub2f9\uad8c\uc744 \uc7a5\uc545\ud558\uace0 \uc788\uc5c8\ub358 \ub3d9\uad50\ub3d9\uacc4\uc640 \ub178\ubb34\ud604 \uce21 \uac04\uc758 \uad00\uacc4 \uc545\ud654\ub294 \uacc4\uc18d\ub418\uc5c8\uace0, \uc774 \uc640\uc911\uc5d0 \uc18c\uc704 '\uce5c\ub178'\ub77c\uace0 \ubd88\ub9ac\ub358 \uac1c\ud601\ud30c \ub2f9\uc6d0\ub4e4\uc774 \ud0c8\ub2f9\ud558\uc600\ub2e4. \uc774\ub4e4\uc740 \ud55c\ub098\ub77c\ub2f9 \ud0c8\ub2f9\ud30c \uc77c\ubd80, \uac1c\ud601\uad6d\ubbfc\uc815\ub2f9\uacfc \uc5f0\ub300\ud55c \ub4a4 \uc5f4\ub9b0\uc6b0\ub9ac\ub2f9 \uc744 \ucc3d\ub2f9\ud588\ub2e4.", "sentence_answer": "\uc774\ub4e4\uc740 \ud55c\ub098\ub77c\ub2f9 \ud0c8\ub2f9\ud30c \uc77c\ubd80, \uac1c\ud601\uad6d\ubbfc\uc815\ub2f9\uacfc \uc5f0\ub300\ud55c \ub4a4 \uc5f4\ub9b0\uc6b0\ub9ac\ub2f9 \uc744 \ucc3d\ub2f9\ud588\ub2e4.", "paragraph_id": "6290061-3-2", "paragraph_question": "question: \uce5c\ub178\ud30c\uc778 \uac1c\ud601\ud30c \ub2f9\uc6d0\ub4e4\uc774 \ud0c8\ub2f9 \ud6c4 \ud55c\ub098\ub77c\ub2f9, \uac1c\ud601\uad6d\ubbfc\uc815\ub2f9\uacfc \ud568\uaed8 \ucc3d\uc124\ud55c \ub2f9\uc758 \uc774\ub984\uc740?, context: \uadf8\ub7ec\ub098 \ub0b4\uac01\uc81c \uac1c\ud5cc \ubb38\uc81c\ub97c \ub450\uace0 \uc5ec\uad8c \ubd84\uc5f4\uc774 \uc788\uc5c8\uace0, \uc5ec\ub7ec \uc2a4\uce94\ub4e4\uc774 \ud130\uc838 \ubbfc\uc2ec \uc774\ubc18\uc774 \uc77c\uc5b4\ub0ac\ub2e4. \uc0c8\uc815\uce58\uad6d\ubbfc\ud68c\uc758\ub294 2000\ub144 \uc0c8\ucc9c\ub144\ubbfc\uc8fc\ub2f9\uc73c\ub85c \uac1c\ud3b8\ub418\uc5b4 \ud574\uccb4\ub418\uc5c8\uace0, \uc774\uc5b4 \ucd1d\uc120\uc5d0\uc11c \uc758\uc11d\uc218\ub294 \ub298\uc5b4\ub0ac\uc73c\ub098 \ud55c\ub098\ub77c\ub2f9\uc5d0 \ubc00\ub824 \ud328\ud588\ub2e4. \uc774\ud6c4 \uae40\ub300\uc911\uc740 \ub9ac\ub354\uc2ed\uc744 \uc783\uc5c8\uace0, \uacf5\ub3d9\uc5ec\ub2f9 \uacf5\uc870 \ud30c\uae30\uae4c\uc9c0 \uc77c\uc5b4\ub098\uba74\uc11c \uc815\uad8c\uc758 \uad6d\uc815 \uc6b4\uc601\uc740 \uc5b4\ub824\uc6c0\uc744 \uacaa\uac8c \ub418\uc5c8\ub2e4. \ubbfc\uc8fc\ub2f9\uc740 2002\ub144 \uc9c0\ubc29\uc120\uac70\uc5d0\uc11c \ud328\ud588\uace0, \uacb0\uad6d \uae40\ub300\uc911\uc740 \ud0c8\ub2f9\ud558\uc600\ub2e4(2000\ub144 \ucd1d\uc120 \uc804 \uc0c8\ucc9c\ub144\ubbfc\uc8fc\ub2f9 \ucc3d\ub2f9). \uadf8\ub7ec\ub098 \ub3d9\uad50\ub3d9\uacc4\ub294 \uc5ec\uc804\ud788 \ub2f9\uad8c\uc744 \uc7a5\uc545\ud558\uace0 \uc788\uc5c8\ub2e4. \ud558\uc9c0\ub9cc \uac19\uc740 \ud574 \ub300\uc120 \ud6c4\ubcf4\ub97c \uc120\ucd9c\ud558\uae30 \uc704\ud55c \uad6d\ubbfc \uacbd\uc120\uc81c\uac00 \ub3d9\uad50\ub3d9\uacc4\uc758 \uc808\ub300\uc801 \uad8c\uc704\uac00 \ud754\ub4e4\ub9ac\uae30 \uc2dc\uc791\ud558\ub294\ub370, \ub2f9\uc2dc '\ub178\ud48d'\uc758 \uacb0\uacfc\ub85c \ub178\ubb34\ud604\uc758 \uc9c0\uc9c0\uc728\uc774 \uae09\uc0c1\uc2b9\ud558\uace0 \ub3d9\uad50\ub3d9\uacc4 \ud6c4\ubcf4 \ud55c\ud654\uac11\uc740 \ub099\uc120\ud55c \uac83\uc774\ub2e4. \uacb0\uad6d, \ub300\uc120 \ud6c4\ubcf4\ub85c\ub294 \ub178\ubb34\ud604\uc774 \uc120\ucd9c\ub418\uc5c8\ub2e4. \ub300\uc120 \uacb0\uacfc, \ub178\ubb34\ud604\uc758 \ub2f9\uc120\uc73c\ub85c \uc0c8\ucc9c\ub144\ubbfc\uc8fc\ub2f9\uc740 \uc815\uad8c\uc744 \uc5f0\uc7a5\ud588\uc73c\ub098, \ub2f9\uc2dc \ub2f9\uad8c\uc744 \uc7a5\uc545\ud558\uace0 \uc788\uc5c8\ub358 \ub3d9\uad50\ub3d9\uacc4\uc640 \ub178\ubb34\ud604 \uce21 \uac04\uc758 \uad00\uacc4 \uc545\ud654\ub294 \uacc4\uc18d\ub418\uc5c8\uace0, \uc774 \uc640\uc911\uc5d0 \uc18c\uc704 '\uce5c\ub178'\ub77c\uace0 \ubd88\ub9ac\ub358 \uac1c\ud601\ud30c \ub2f9\uc6d0\ub4e4\uc774 \ud0c8\ub2f9\ud558\uc600\ub2e4. \uc774\ub4e4\uc740 \ud55c\ub098\ub77c\ub2f9 \ud0c8\ub2f9\ud30c \uc77c\ubd80, \uac1c\ud601\uad6d\ubbfc\uc815\ub2f9\uacfc \uc5f0\ub300\ud55c \ub4a4 \uc5f4\ub9b0\uc6b0\ub9ac\ub2f9\uc744 \ucc3d\ub2f9\ud588\ub2e4."} {"question": "\uc724\uc774\uc789 \ud669\ud0dc\uc790\ub85c \uc788\uc744 \ub2f9\uc2dc \ud30c\ubc8c\uc5d0 \ucc38\uac00\ud588\ub358 \ud669\uc790\ub294 \uc724\uc789\uc744 \ud3ec\ud568\ud574 \ubaa8\ub450 \uba87 \uba85\uc774\uc5c8\ub294\uac00?", "paragraph": "\uc724\uc789\uc740 20\ub300\uae4c\uc9c0\ub9cc \ud574\ub3c4 \uc544\ubc84\uc9c0\uc758 \ub73b\uc744 \ubc1b\ub4e4\uace0 \ube48\ud2c8\uc5c6\uc774 \uc77c\uc744 \ucc98\ub9ac\ud558\uc600\uc73c\ub098 30\ub300\uac00 \ub118\uc5b4\uc11c\ubd80\ud130 \uc544\ubc84\uc9c0\uc758 \uae30\ub300\uc5d0 \ubd80\uc751\ud558\uc9c0 \ubabb\ud558\uace0 \uc8fc\uc0c9\uc7a1\uae30\ub97c \uc88b\uc544\ud558\uc600\ub2e4. \uc18d\uc73c\ub85c\ub294 \uc724\uc789\uc744 \ub9e4\uc6b0 \uc544\uaf08\uc73c\ub098 \uac89\uc73c\ub85c \ub0b4\uc0c9\uc744 \ud558\uc9c0 \ubabb\ud55c \uc544\ubc84\uc9c0\uc758 \ub0c9\uc815\ud568\uacfc \ud639\ub3c5\ud568, \uc5b4\uba38\ub2c8\ub97c \ud0dc\uc5b4\ub098\uc790\ub9c8\uc790 \uc5ec\uc75c \ubaa8\uc815\uc758 \uac08\ub9dd, \uadf8\ub9ac\uace0 \ud615\uc81c\ub4e4\uac04\uc758 \uc554\ud22c \ub4f1\uc73c\ub85c \uc778\ud574 \uc810\uc810 \ud0c0\ub77d\ud558\uae30 \uc2dc\uc791\ud55c \uc724\uc789\uc5d0\uac8c \uc2e4\ub9dd\ud55c \uac15\ud76c\uc81c\ub294 3\ud669\uc790 \uc724\uc9c0, 4\ud669\uc790 \uc724\uc9c4, 8\ud669\uc790 \uc724\uc0ac \ub4f1 \ub2e4\ub978 \ud669\uc790\ub4e4\uc5d0\uac8c \uac01\uae30 \ubd80\uc11c\ub97c \ucc45\uc784\uc9c0\uace0 \ub3c4\ub9e1\uac8c \ud558\uc600\ub2e4. \ub2e4\ub978 \ud669\uc790\ub4e4\uc740 \ubaa8\ub450 \uc721\ubd80\ub97c \uad00\ub9ac\ud558\uc5ec \uac15\ud76c\uc81c\uc758 \uc2e0\uc784\uc744 \uc5bb\uace0 \uad70\uc655, \uce5c\uc655\uc73c\ub85c \uc2b9\uc2b9\uc7a5\uad6c\ud558\uc600\uc73c\ub098, \uc724\uc789\uc740 \uc5b4\ub5a0\ud55c \ubd80\uc11c\ub3c4 \ub9e1\uc9c0 \uc54a\uc740 \ucc44, \ubd80\ud669\uc758 \ub208\ubc16\uc5d0 \ub098\uace0 \uc815\uc2e0\uc9c8\ud658\uc5d0 \uac00\uae4c\uc6b4 \ube44\ud589\uae4c\uc9c0 \uc77c\uc0bc\uc558\ub2e4 \ud55c\ub2e4. \uadf8\ub9ac\uace0 \uc774\ubbf8 \uc870\uc815\uc740 \ud669\uc790\ub4e4\uacfc \uc2e0\ub8cc\ub4e4\uc758 \uc57c\uc2ec\uc73c\ub85c \uc778\ud574 \uc0ac\ubd84\uc624\uc5f4\uc774 \ub418\uc5c8\ub294\ub370, \uc774 \uc911 \uc724\uc2dc\ub294 \uc790\uc2e0\uc774 \uc7a5\uc790\uc778\ub370 \uc5b8\uc81c\ub098 \ucc28\ub0a8\uc774\uba70 \ud669\ud0dc\uc790\uc778 \uc724\uc789\uc758 \ub4a4\uc5d0 \uc11c\uc57c \ub418\uace0 \uc724\uc789\uc5d0\uac8c \uba38\ub9ac\ub97c \uc870\uc544\ub9ac\uace0 \uc808\uc744 \ud574\uc57c \ud588\uc73c\uba70 \uc790\uc2e0\uc774 \uc544\uc6b0\ub4e4\uc5d0\uac8c \u201c\ud615\ub2d8\u201d\uc774\ub77c \ubd88\ub9ac\ub294 \ubc18\uba74 \uc724\uc789\uc740 \uc774\ubcf5 \ud615\uc778 \uc724\uc2dc\ub97c \ube44\ub86f\ud55c \ud615\uc81c\ub4e4\uc5d0\uac8c \u201c\ud669\ud0dc\uc790 \uc804\ud558\u201d\ub77c\ub294 \uc874\uce6d\uc73c\ub85c \ubd88\ub9ac\ub294 \uac83\uc774 \ud070 \ubd88\ub9cc\uc774\ub77c \ubc18\uc724\uc789 \uc138\ub825\uc5d0 \uc801\uadf9 \uac00\ub2f4\ud558\uc600\ub2e4. \uc724\uc789\uc774 \ud669\ud0dc\uc790\ub85c \uc788\uc744 \ub2f9\uc2dc \uc724\uc789\uc744 \ud3ec\ud568\ud55c 9\uba85\uc758 \ud669\uc790\uac00 \ud30c\ubc8c\uc5d0 \ucc38\uc5ec\ud558\uc600\ub294\ub370 \uadf8 \uacc4\ubcf4\uc640 \ud30c\ubc8c\uc5d0 \ucc38\uac00\ud55c \uc8fc\uc694 \ub300\uc2e0\ub4e4\uc744 \ubcf4\uba74 \uc774\ub7ec\ud558\ub2e4.", "answer": "9\uba85", "sentence": "\uc724\uc789\uc774 \ud669\ud0dc\uc790\ub85c \uc788\uc744 \ub2f9\uc2dc \uc724\uc789\uc744 \ud3ec\ud568\ud55c 9\uba85 \uc758 \ud669\uc790\uac00 \ud30c\ubc8c\uc5d0 \ucc38\uc5ec\ud558\uc600\ub294\ub370 \uadf8 \uacc4\ubcf4\uc640 \ud30c\ubc8c\uc5d0 \ucc38\uac00\ud55c \uc8fc\uc694 \ub300\uc2e0\ub4e4\uc744 \ubcf4\uba74 \uc774\ub7ec\ud558\ub2e4.", "paragraph_sentence": "\uc724\uc789\uc740 20\ub300\uae4c\uc9c0\ub9cc \ud574\ub3c4 \uc544\ubc84\uc9c0\uc758 \ub73b\uc744 \ubc1b\ub4e4\uace0 \ube48\ud2c8\uc5c6\uc774 \uc77c\uc744 \ucc98\ub9ac\ud558\uc600\uc73c\ub098 30\ub300\uac00 \ub118\uc5b4\uc11c\ubd80\ud130 \uc544\ubc84\uc9c0\uc758 \uae30\ub300\uc5d0 \ubd80\uc751\ud558\uc9c0 \ubabb\ud558\uace0 \uc8fc\uc0c9\uc7a1\uae30\ub97c \uc88b\uc544\ud558\uc600\ub2e4. \uc18d\uc73c\ub85c\ub294 \uc724\uc789\uc744 \ub9e4\uc6b0 \uc544\uaf08\uc73c\ub098 \uac89\uc73c\ub85c \ub0b4\uc0c9\uc744 \ud558\uc9c0 \ubabb\ud55c \uc544\ubc84\uc9c0\uc758 \ub0c9\uc815\ud568\uacfc \ud639\ub3c5\ud568, \uc5b4\uba38\ub2c8\ub97c \ud0dc\uc5b4\ub098\uc790\ub9c8\uc790 \uc5ec\uc75c \ubaa8\uc815\uc758 \uac08\ub9dd, \uadf8\ub9ac\uace0 \ud615\uc81c\ub4e4\uac04\uc758 \uc554\ud22c \ub4f1\uc73c\ub85c \uc778\ud574 \uc810\uc810 \ud0c0\ub77d\ud558\uae30 \uc2dc\uc791\ud55c \uc724\uc789\uc5d0\uac8c \uc2e4\ub9dd\ud55c \uac15\ud76c\uc81c\ub294 3\ud669\uc790 \uc724\uc9c0, 4\ud669\uc790 \uc724\uc9c4, 8\ud669\uc790 \uc724\uc0ac \ub4f1 \ub2e4\ub978 \ud669\uc790\ub4e4\uc5d0\uac8c \uac01\uae30 \ubd80\uc11c\ub97c \ucc45\uc784\uc9c0\uace0 \ub3c4\ub9e1\uac8c \ud558\uc600\ub2e4. \ub2e4\ub978 \ud669\uc790\ub4e4\uc740 \ubaa8\ub450 \uc721\ubd80\ub97c \uad00\ub9ac\ud558\uc5ec \uac15\ud76c\uc81c\uc758 \uc2e0\uc784\uc744 \uc5bb\uace0 \uad70\uc655, \uce5c\uc655\uc73c\ub85c \uc2b9\uc2b9\uc7a5\uad6c\ud558\uc600\uc73c\ub098, \uc724\uc789\uc740 \uc5b4\ub5a0\ud55c \ubd80\uc11c\ub3c4 \ub9e1\uc9c0 \uc54a\uc740 \ucc44, \ubd80\ud669\uc758 \ub208\ubc16\uc5d0 \ub098\uace0 \uc815\uc2e0\uc9c8\ud658\uc5d0 \uac00\uae4c\uc6b4 \ube44\ud589\uae4c\uc9c0 \uc77c\uc0bc\uc558\ub2e4 \ud55c\ub2e4. \uadf8\ub9ac\uace0 \uc774\ubbf8 \uc870\uc815\uc740 \ud669\uc790\ub4e4\uacfc \uc2e0\ub8cc\ub4e4\uc758 \uc57c\uc2ec\uc73c\ub85c \uc778\ud574 \uc0ac\ubd84\uc624\uc5f4\uc774 \ub418\uc5c8\ub294\ub370, \uc774 \uc911 \uc724\uc2dc\ub294 \uc790\uc2e0\uc774 \uc7a5\uc790\uc778\ub370 \uc5b8\uc81c\ub098 \ucc28\ub0a8\uc774\uba70 \ud669\ud0dc\uc790\uc778 \uc724\uc789\uc758 \ub4a4\uc5d0 \uc11c\uc57c \ub418\uace0 \uc724\uc789\uc5d0\uac8c \uba38\ub9ac\ub97c \uc870\uc544\ub9ac\uace0 \uc808\uc744 \ud574\uc57c \ud588\uc73c\uba70 \uc790\uc2e0\uc774 \uc544\uc6b0\ub4e4\uc5d0\uac8c \u201c\ud615\ub2d8\u201d\uc774\ub77c \ubd88\ub9ac\ub294 \ubc18\uba74 \uc724\uc789\uc740 \uc774\ubcf5 \ud615\uc778 \uc724\uc2dc\ub97c \ube44\ub86f\ud55c \ud615\uc81c\ub4e4\uc5d0\uac8c \u201c\ud669\ud0dc\uc790 \uc804\ud558\u201d\ub77c\ub294 \uc874\uce6d\uc73c\ub85c \ubd88\ub9ac\ub294 \uac83\uc774 \ud070 \ubd88\ub9cc\uc774\ub77c \ubc18\uc724\uc789 \uc138\ub825\uc5d0 \uc801\uadf9 \uac00\ub2f4\ud558\uc600\ub2e4. \uc724\uc789\uc774 \ud669\ud0dc\uc790\ub85c \uc788\uc744 \ub2f9\uc2dc \uc724\uc789\uc744 \ud3ec\ud568\ud55c 9\uba85 \uc758 \ud669\uc790\uac00 \ud30c\ubc8c\uc5d0 \ucc38\uc5ec\ud558\uc600\ub294\ub370 \uadf8 \uacc4\ubcf4\uc640 \ud30c\ubc8c\uc5d0 \ucc38\uac00\ud55c \uc8fc\uc694 \ub300\uc2e0\ub4e4\uc744 \ubcf4\uba74 \uc774\ub7ec\ud558\ub2e4. ", "paragraph_answer": "\uc724\uc789\uc740 20\ub300\uae4c\uc9c0\ub9cc \ud574\ub3c4 \uc544\ubc84\uc9c0\uc758 \ub73b\uc744 \ubc1b\ub4e4\uace0 \ube48\ud2c8\uc5c6\uc774 \uc77c\uc744 \ucc98\ub9ac\ud558\uc600\uc73c\ub098 30\ub300\uac00 \ub118\uc5b4\uc11c\ubd80\ud130 \uc544\ubc84\uc9c0\uc758 \uae30\ub300\uc5d0 \ubd80\uc751\ud558\uc9c0 \ubabb\ud558\uace0 \uc8fc\uc0c9\uc7a1\uae30\ub97c \uc88b\uc544\ud558\uc600\ub2e4. \uc18d\uc73c\ub85c\ub294 \uc724\uc789\uc744 \ub9e4\uc6b0 \uc544\uaf08\uc73c\ub098 \uac89\uc73c\ub85c \ub0b4\uc0c9\uc744 \ud558\uc9c0 \ubabb\ud55c \uc544\ubc84\uc9c0\uc758 \ub0c9\uc815\ud568\uacfc \ud639\ub3c5\ud568, \uc5b4\uba38\ub2c8\ub97c \ud0dc\uc5b4\ub098\uc790\ub9c8\uc790 \uc5ec\uc75c \ubaa8\uc815\uc758 \uac08\ub9dd, \uadf8\ub9ac\uace0 \ud615\uc81c\ub4e4\uac04\uc758 \uc554\ud22c \ub4f1\uc73c\ub85c \uc778\ud574 \uc810\uc810 \ud0c0\ub77d\ud558\uae30 \uc2dc\uc791\ud55c \uc724\uc789\uc5d0\uac8c \uc2e4\ub9dd\ud55c \uac15\ud76c\uc81c\ub294 3\ud669\uc790 \uc724\uc9c0, 4\ud669\uc790 \uc724\uc9c4, 8\ud669\uc790 \uc724\uc0ac \ub4f1 \ub2e4\ub978 \ud669\uc790\ub4e4\uc5d0\uac8c \uac01\uae30 \ubd80\uc11c\ub97c \ucc45\uc784\uc9c0\uace0 \ub3c4\ub9e1\uac8c \ud558\uc600\ub2e4. \ub2e4\ub978 \ud669\uc790\ub4e4\uc740 \ubaa8\ub450 \uc721\ubd80\ub97c \uad00\ub9ac\ud558\uc5ec \uac15\ud76c\uc81c\uc758 \uc2e0\uc784\uc744 \uc5bb\uace0 \uad70\uc655, \uce5c\uc655\uc73c\ub85c \uc2b9\uc2b9\uc7a5\uad6c\ud558\uc600\uc73c\ub098, \uc724\uc789\uc740 \uc5b4\ub5a0\ud55c \ubd80\uc11c\ub3c4 \ub9e1\uc9c0 \uc54a\uc740 \ucc44, \ubd80\ud669\uc758 \ub208\ubc16\uc5d0 \ub098\uace0 \uc815\uc2e0\uc9c8\ud658\uc5d0 \uac00\uae4c\uc6b4 \ube44\ud589\uae4c\uc9c0 \uc77c\uc0bc\uc558\ub2e4 \ud55c\ub2e4. \uadf8\ub9ac\uace0 \uc774\ubbf8 \uc870\uc815\uc740 \ud669\uc790\ub4e4\uacfc \uc2e0\ub8cc\ub4e4\uc758 \uc57c\uc2ec\uc73c\ub85c \uc778\ud574 \uc0ac\ubd84\uc624\uc5f4\uc774 \ub418\uc5c8\ub294\ub370, \uc774 \uc911 \uc724\uc2dc\ub294 \uc790\uc2e0\uc774 \uc7a5\uc790\uc778\ub370 \uc5b8\uc81c\ub098 \ucc28\ub0a8\uc774\uba70 \ud669\ud0dc\uc790\uc778 \uc724\uc789\uc758 \ub4a4\uc5d0 \uc11c\uc57c \ub418\uace0 \uc724\uc789\uc5d0\uac8c \uba38\ub9ac\ub97c \uc870\uc544\ub9ac\uace0 \uc808\uc744 \ud574\uc57c \ud588\uc73c\uba70 \uc790\uc2e0\uc774 \uc544\uc6b0\ub4e4\uc5d0\uac8c \u201c\ud615\ub2d8\u201d\uc774\ub77c \ubd88\ub9ac\ub294 \ubc18\uba74 \uc724\uc789\uc740 \uc774\ubcf5 \ud615\uc778 \uc724\uc2dc\ub97c \ube44\ub86f\ud55c \ud615\uc81c\ub4e4\uc5d0\uac8c \u201c\ud669\ud0dc\uc790 \uc804\ud558\u201d\ub77c\ub294 \uc874\uce6d\uc73c\ub85c \ubd88\ub9ac\ub294 \uac83\uc774 \ud070 \ubd88\ub9cc\uc774\ub77c \ubc18\uc724\uc789 \uc138\ub825\uc5d0 \uc801\uadf9 \uac00\ub2f4\ud558\uc600\ub2e4. \uc724\uc789\uc774 \ud669\ud0dc\uc790\ub85c \uc788\uc744 \ub2f9\uc2dc \uc724\uc789\uc744 \ud3ec\ud568\ud55c 9\uba85 \uc758 \ud669\uc790\uac00 \ud30c\ubc8c\uc5d0 \ucc38\uc5ec\ud558\uc600\ub294\ub370 \uadf8 \uacc4\ubcf4\uc640 \ud30c\ubc8c\uc5d0 \ucc38\uac00\ud55c \uc8fc\uc694 \ub300\uc2e0\ub4e4\uc744 \ubcf4\uba74 \uc774\ub7ec\ud558\ub2e4.", "sentence_answer": "\uc724\uc789\uc774 \ud669\ud0dc\uc790\ub85c \uc788\uc744 \ub2f9\uc2dc \uc724\uc789\uc744 \ud3ec\ud568\ud55c 9\uba85 \uc758 \ud669\uc790\uac00 \ud30c\ubc8c\uc5d0 \ucc38\uc5ec\ud558\uc600\ub294\ub370 \uadf8 \uacc4\ubcf4\uc640 \ud30c\ubc8c\uc5d0 \ucc38\uac00\ud55c \uc8fc\uc694 \ub300\uc2e0\ub4e4\uc744 \ubcf4\uba74 \uc774\ub7ec\ud558\ub2e4.", "paragraph_id": "6486899-31-1", "paragraph_question": "question: \uc724\uc774\uc789 \ud669\ud0dc\uc790\ub85c \uc788\uc744 \ub2f9\uc2dc \ud30c\ubc8c\uc5d0 \ucc38\uac00\ud588\ub358 \ud669\uc790\ub294 \uc724\uc789\uc744 \ud3ec\ud568\ud574 \ubaa8\ub450 \uba87 \uba85\uc774\uc5c8\ub294\uac00?, context: \uc724\uc789\uc740 20\ub300\uae4c\uc9c0\ub9cc \ud574\ub3c4 \uc544\ubc84\uc9c0\uc758 \ub73b\uc744 \ubc1b\ub4e4\uace0 \ube48\ud2c8\uc5c6\uc774 \uc77c\uc744 \ucc98\ub9ac\ud558\uc600\uc73c\ub098 30\ub300\uac00 \ub118\uc5b4\uc11c\ubd80\ud130 \uc544\ubc84\uc9c0\uc758 \uae30\ub300\uc5d0 \ubd80\uc751\ud558\uc9c0 \ubabb\ud558\uace0 \uc8fc\uc0c9\uc7a1\uae30\ub97c \uc88b\uc544\ud558\uc600\ub2e4. \uc18d\uc73c\ub85c\ub294 \uc724\uc789\uc744 \ub9e4\uc6b0 \uc544\uaf08\uc73c\ub098 \uac89\uc73c\ub85c \ub0b4\uc0c9\uc744 \ud558\uc9c0 \ubabb\ud55c \uc544\ubc84\uc9c0\uc758 \ub0c9\uc815\ud568\uacfc \ud639\ub3c5\ud568, \uc5b4\uba38\ub2c8\ub97c \ud0dc\uc5b4\ub098\uc790\ub9c8\uc790 \uc5ec\uc75c \ubaa8\uc815\uc758 \uac08\ub9dd, \uadf8\ub9ac\uace0 \ud615\uc81c\ub4e4\uac04\uc758 \uc554\ud22c \ub4f1\uc73c\ub85c \uc778\ud574 \uc810\uc810 \ud0c0\ub77d\ud558\uae30 \uc2dc\uc791\ud55c \uc724\uc789\uc5d0\uac8c \uc2e4\ub9dd\ud55c \uac15\ud76c\uc81c\ub294 3\ud669\uc790 \uc724\uc9c0, 4\ud669\uc790 \uc724\uc9c4, 8\ud669\uc790 \uc724\uc0ac \ub4f1 \ub2e4\ub978 \ud669\uc790\ub4e4\uc5d0\uac8c \uac01\uae30 \ubd80\uc11c\ub97c \ucc45\uc784\uc9c0\uace0 \ub3c4\ub9e1\uac8c \ud558\uc600\ub2e4. \ub2e4\ub978 \ud669\uc790\ub4e4\uc740 \ubaa8\ub450 \uc721\ubd80\ub97c \uad00\ub9ac\ud558\uc5ec \uac15\ud76c\uc81c\uc758 \uc2e0\uc784\uc744 \uc5bb\uace0 \uad70\uc655, \uce5c\uc655\uc73c\ub85c \uc2b9\uc2b9\uc7a5\uad6c\ud558\uc600\uc73c\ub098, \uc724\uc789\uc740 \uc5b4\ub5a0\ud55c \ubd80\uc11c\ub3c4 \ub9e1\uc9c0 \uc54a\uc740 \ucc44, \ubd80\ud669\uc758 \ub208\ubc16\uc5d0 \ub098\uace0 \uc815\uc2e0\uc9c8\ud658\uc5d0 \uac00\uae4c\uc6b4 \ube44\ud589\uae4c\uc9c0 \uc77c\uc0bc\uc558\ub2e4 \ud55c\ub2e4. \uadf8\ub9ac\uace0 \uc774\ubbf8 \uc870\uc815\uc740 \ud669\uc790\ub4e4\uacfc \uc2e0\ub8cc\ub4e4\uc758 \uc57c\uc2ec\uc73c\ub85c \uc778\ud574 \uc0ac\ubd84\uc624\uc5f4\uc774 \ub418\uc5c8\ub294\ub370, \uc774 \uc911 \uc724\uc2dc\ub294 \uc790\uc2e0\uc774 \uc7a5\uc790\uc778\ub370 \uc5b8\uc81c\ub098 \ucc28\ub0a8\uc774\uba70 \ud669\ud0dc\uc790\uc778 \uc724\uc789\uc758 \ub4a4\uc5d0 \uc11c\uc57c \ub418\uace0 \uc724\uc789\uc5d0\uac8c \uba38\ub9ac\ub97c \uc870\uc544\ub9ac\uace0 \uc808\uc744 \ud574\uc57c \ud588\uc73c\uba70 \uc790\uc2e0\uc774 \uc544\uc6b0\ub4e4\uc5d0\uac8c \u201c\ud615\ub2d8\u201d\uc774\ub77c \ubd88\ub9ac\ub294 \ubc18\uba74 \uc724\uc789\uc740 \uc774\ubcf5 \ud615\uc778 \uc724\uc2dc\ub97c \ube44\ub86f\ud55c \ud615\uc81c\ub4e4\uc5d0\uac8c \u201c\ud669\ud0dc\uc790 \uc804\ud558\u201d\ub77c\ub294 \uc874\uce6d\uc73c\ub85c \ubd88\ub9ac\ub294 \uac83\uc774 \ud070 \ubd88\ub9cc\uc774\ub77c \ubc18\uc724\uc789 \uc138\ub825\uc5d0 \uc801\uadf9 \uac00\ub2f4\ud558\uc600\ub2e4. \uc724\uc789\uc774 \ud669\ud0dc\uc790\ub85c \uc788\uc744 \ub2f9\uc2dc \uc724\uc789\uc744 \ud3ec\ud568\ud55c 9\uba85\uc758 \ud669\uc790\uac00 \ud30c\ubc8c\uc5d0 \ucc38\uc5ec\ud558\uc600\ub294\ub370 \uadf8 \uacc4\ubcf4\uc640 \ud30c\ubc8c\uc5d0 \ucc38\uac00\ud55c \uc8fc\uc694 \ub300\uc2e0\ub4e4\uc744 \ubcf4\uba74 \uc774\ub7ec\ud558\ub2e4."} {"question": "\ud574\ubc29 \uc804\uae4c\uc9c0 \uc870\uc120\uc758 \uc720\uc77c\ud55c \uace0\ub4f1\uacf5\uc5c5\uae30\uad00\uc774\uc5c8\ub358 \ud559\uad50\ub294?", "paragraph": "\uacbd\uc131\uace0\ub4f1\uacf5\uc5c5\ud559\uad50\ub294 1940\ub144\ub300\uae4c\uc9c0 \uc870\uc120\uc758 \uc720\uc77c\ud55c \uace0\ub4f1\uacf5\uc5c5\uae30\uad00\uc774\uc5c8\uae30 \ub54c\ubb38\uc5d0 \ub2e4\ub978 \uc804\ubb38\ud559\uad50\uc5d0 \ube44\ud574 \ud2b9\uc218\ud55c \ud559\uad50\ub85c \uc5ec\uaca8\uc84c\ub2e4. \uc870\uc120\uad50\uc721\ub839 \uac1c\uc815 \uc774\uc804\uc5d0\ub294 \uc870\uc120\uc778\uc744 \uc804\uccb4\uc815\uc6d0\uc5d0\uc11c 2/3\uac00\ub7c9 \uc120\ubc1c\ud55c\ub2e4\ub294 \ub0b4\uaddc\uac00 \uc788\uc5b4 \uc870\uc120\uc778\uc758 \uc785\ud559 \uacbd\uc7c1\uc774 \uce58\uc5f4\ud558\uc9c0 \uc54a\uc558\uc9c0\ub9cc, \uad50\uc721\ub839 \uac1c\uc815 \uc774\ud6c4 \uadf8\ub7ec\ud55c \ub0b4\uaddc\uac00 \ud3d0\uc9c0\ub418\uc5b4 \uc785\uc2dc \uacbd\uc7c1\ub960\uc740 13:1\uc5d0\uc11c 27:1\uae4c\uc9c0 \ud3ed\uc99d\ud558\uac8c \ub418\uc5c8\uace0 \uc870\uc120\uc778\uc758 \uc785\ud559\uc774 \ub9e4\uc6b0 \uc5b4\ub824\uc6cc\uc84c\ub2e4. \uc774 \ub54c\ubb38\uc5d0 1935\ub144\uc758 \uc785\uc2dc\uc5d0\uc11c\ub294 \ud1a0\ubaa9\uacfc\uc5d0 \uc870\uc120\uc778 \ud559\uc0dd\uc774 \ub2e8 1\uba85\ub9cc \ud569\uaca9\ud558\uae30\ub3c4 \ud588\ub2e4. \ub2f9\uc2dc \uad50\uc721\uacc4\uc5d0\uc11c\ub294 \uacbd\uc131\uace0\ub4f1\uc0c1\uc5c5\ud559\uad50(\uacbd\uc131\uace0\uc0c1)\uc758 \uc870\uc120\uc778 \uc785\ud559\ub09c\uacfc \ud568\uaed8 \uacbd\uc131\uace0\uacf5\uacfc \uacbd\uc131\uace0\uc0c1\uc758 \uc785\uc2dc\uc81c\ub3c4\ub97c \uc870\uc120\uc778\uc5d0\uac8c \uc720\ub9ac\ud558\uac8c \uc2dc\uc815\ud574\uc57c \ud55c\ub2e4\ub294 \uc758\uacac\uc774 \uc788\uc5c8\uc73c\ub098, \uacbd\uc131\uace0\ub4f1\uacf5\uc5c5\ud559\uad50\uc640 \uc870\uc120\ucd1d\ub3c5\ubd80 \ud559\ubb34\uad6d\uc5d0\uc11c \ubc1b\uc544\ub4e4\uc774\uc9c0 \uc54a\uc558\ub2e4.", "answer": "\uacbd\uc131\uace0\ub4f1\uacf5\uc5c5\ud559\uad50", "sentence": "\uacbd\uc131\uace0\ub4f1\uacf5\uc5c5\ud559\uad50 \ub294 1940\ub144\ub300\uae4c\uc9c0 \uc870\uc120\uc758 \uc720\uc77c\ud55c \uace0\ub4f1\uacf5\uc5c5\uae30\uad00\uc774\uc5c8\uae30 \ub54c\ubb38\uc5d0 \ub2e4\ub978 \uc804\ubb38\ud559\uad50\uc5d0 \ube44\ud574 \ud2b9\uc218\ud55c \ud559\uad50\ub85c \uc5ec\uaca8\uc84c\ub2e4.", "paragraph_sentence": " \uacbd\uc131\uace0\ub4f1\uacf5\uc5c5\ud559\uad50 \ub294 1940\ub144\ub300\uae4c\uc9c0 \uc870\uc120\uc758 \uc720\uc77c\ud55c \uace0\ub4f1\uacf5\uc5c5\uae30\uad00\uc774\uc5c8\uae30 \ub54c\ubb38\uc5d0 \ub2e4\ub978 \uc804\ubb38\ud559\uad50\uc5d0 \ube44\ud574 \ud2b9\uc218\ud55c \ud559\uad50\ub85c \uc5ec\uaca8\uc84c\ub2e4. \uc870\uc120\uad50\uc721\ub839 \uac1c\uc815 \uc774\uc804\uc5d0\ub294 \uc870\uc120\uc778\uc744 \uc804\uccb4\uc815\uc6d0\uc5d0\uc11c 2/3\uac00\ub7c9 \uc120\ubc1c\ud55c\ub2e4\ub294 \ub0b4\uaddc\uac00 \uc788\uc5b4 \uc870\uc120\uc778\uc758 \uc785\ud559 \uacbd\uc7c1\uc774 \uce58\uc5f4\ud558\uc9c0 \uc54a\uc558\uc9c0\ub9cc, \uad50\uc721\ub839 \uac1c\uc815 \uc774\ud6c4 \uadf8\ub7ec\ud55c \ub0b4\uaddc\uac00 \ud3d0\uc9c0\ub418\uc5b4 \uc785\uc2dc \uacbd\uc7c1\ub960\uc740 13:1\uc5d0\uc11c 27:1\uae4c\uc9c0 \ud3ed\uc99d\ud558\uac8c \ub418\uc5c8\uace0 \uc870\uc120\uc778\uc758 \uc785\ud559\uc774 \ub9e4\uc6b0 \uc5b4\ub824\uc6cc\uc84c\ub2e4. \uc774 \ub54c\ubb38\uc5d0 1935\ub144\uc758 \uc785\uc2dc\uc5d0\uc11c\ub294 \ud1a0\ubaa9\uacfc\uc5d0 \uc870\uc120\uc778 \ud559\uc0dd\uc774 \ub2e8 1\uba85\ub9cc \ud569\uaca9\ud558\uae30\ub3c4 \ud588\ub2e4. \ub2f9\uc2dc \uad50\uc721\uacc4\uc5d0\uc11c\ub294 \uacbd\uc131\uace0\ub4f1\uc0c1\uc5c5\ud559\uad50(\uacbd\uc131\uace0\uc0c1)\uc758 \uc870\uc120\uc778 \uc785\ud559\ub09c\uacfc \ud568\uaed8 \uacbd\uc131\uace0\uacf5\uacfc \uacbd\uc131\uace0\uc0c1\uc758 \uc785\uc2dc\uc81c\ub3c4\ub97c \uc870\uc120\uc778\uc5d0\uac8c \uc720\ub9ac\ud558\uac8c \uc2dc\uc815\ud574\uc57c \ud55c\ub2e4\ub294 \uc758\uacac\uc774 \uc788\uc5c8\uc73c\ub098, \uacbd\uc131\uace0\ub4f1\uacf5\uc5c5\ud559\uad50\uc640 \uc870\uc120\ucd1d\ub3c5\ubd80 \ud559\ubb34\uad6d\uc5d0\uc11c \ubc1b\uc544\ub4e4\uc774\uc9c0 \uc54a\uc558\ub2e4.", "paragraph_answer": " \uacbd\uc131\uace0\ub4f1\uacf5\uc5c5\ud559\uad50 \ub294 1940\ub144\ub300\uae4c\uc9c0 \uc870\uc120\uc758 \uc720\uc77c\ud55c \uace0\ub4f1\uacf5\uc5c5\uae30\uad00\uc774\uc5c8\uae30 \ub54c\ubb38\uc5d0 \ub2e4\ub978 \uc804\ubb38\ud559\uad50\uc5d0 \ube44\ud574 \ud2b9\uc218\ud55c \ud559\uad50\ub85c \uc5ec\uaca8\uc84c\ub2e4. \uc870\uc120\uad50\uc721\ub839 \uac1c\uc815 \uc774\uc804\uc5d0\ub294 \uc870\uc120\uc778\uc744 \uc804\uccb4\uc815\uc6d0\uc5d0\uc11c 2/3\uac00\ub7c9 \uc120\ubc1c\ud55c\ub2e4\ub294 \ub0b4\uaddc\uac00 \uc788\uc5b4 \uc870\uc120\uc778\uc758 \uc785\ud559 \uacbd\uc7c1\uc774 \uce58\uc5f4\ud558\uc9c0 \uc54a\uc558\uc9c0\ub9cc, \uad50\uc721\ub839 \uac1c\uc815 \uc774\ud6c4 \uadf8\ub7ec\ud55c \ub0b4\uaddc\uac00 \ud3d0\uc9c0\ub418\uc5b4 \uc785\uc2dc \uacbd\uc7c1\ub960\uc740 13:1\uc5d0\uc11c 27:1\uae4c\uc9c0 \ud3ed\uc99d\ud558\uac8c \ub418\uc5c8\uace0 \uc870\uc120\uc778\uc758 \uc785\ud559\uc774 \ub9e4\uc6b0 \uc5b4\ub824\uc6cc\uc84c\ub2e4. \uc774 \ub54c\ubb38\uc5d0 1935\ub144\uc758 \uc785\uc2dc\uc5d0\uc11c\ub294 \ud1a0\ubaa9\uacfc\uc5d0 \uc870\uc120\uc778 \ud559\uc0dd\uc774 \ub2e8 1\uba85\ub9cc \ud569\uaca9\ud558\uae30\ub3c4 \ud588\ub2e4. \ub2f9\uc2dc \uad50\uc721\uacc4\uc5d0\uc11c\ub294 \uacbd\uc131\uace0\ub4f1\uc0c1\uc5c5\ud559\uad50(\uacbd\uc131\uace0\uc0c1)\uc758 \uc870\uc120\uc778 \uc785\ud559\ub09c\uacfc \ud568\uaed8 \uacbd\uc131\uace0\uacf5\uacfc \uacbd\uc131\uace0\uc0c1\uc758 \uc785\uc2dc\uc81c\ub3c4\ub97c \uc870\uc120\uc778\uc5d0\uac8c \uc720\ub9ac\ud558\uac8c \uc2dc\uc815\ud574\uc57c \ud55c\ub2e4\ub294 \uc758\uacac\uc774 \uc788\uc5c8\uc73c\ub098, \uacbd\uc131\uace0\ub4f1\uacf5\uc5c5\ud559\uad50\uc640 \uc870\uc120\ucd1d\ub3c5\ubd80 \ud559\ubb34\uad6d\uc5d0\uc11c \ubc1b\uc544\ub4e4\uc774\uc9c0 \uc54a\uc558\ub2e4.", "sentence_answer": " \uacbd\uc131\uace0\ub4f1\uacf5\uc5c5\ud559\uad50 \ub294 1940\ub144\ub300\uae4c\uc9c0 \uc870\uc120\uc758 \uc720\uc77c\ud55c \uace0\ub4f1\uacf5\uc5c5\uae30\uad00\uc774\uc5c8\uae30 \ub54c\ubb38\uc5d0 \ub2e4\ub978 \uc804\ubb38\ud559\uad50\uc5d0 \ube44\ud574 \ud2b9\uc218\ud55c \ud559\uad50\ub85c \uc5ec\uaca8\uc84c\ub2e4.", "paragraph_id": "6194880-2-0", "paragraph_question": "question: \ud574\ubc29 \uc804\uae4c\uc9c0 \uc870\uc120\uc758 \uc720\uc77c\ud55c \uace0\ub4f1\uacf5\uc5c5\uae30\uad00\uc774\uc5c8\ub358 \ud559\uad50\ub294?, context: \uacbd\uc131\uace0\ub4f1\uacf5\uc5c5\ud559\uad50\ub294 1940\ub144\ub300\uae4c\uc9c0 \uc870\uc120\uc758 \uc720\uc77c\ud55c \uace0\ub4f1\uacf5\uc5c5\uae30\uad00\uc774\uc5c8\uae30 \ub54c\ubb38\uc5d0 \ub2e4\ub978 \uc804\ubb38\ud559\uad50\uc5d0 \ube44\ud574 \ud2b9\uc218\ud55c \ud559\uad50\ub85c \uc5ec\uaca8\uc84c\ub2e4. \uc870\uc120\uad50\uc721\ub839 \uac1c\uc815 \uc774\uc804\uc5d0\ub294 \uc870\uc120\uc778\uc744 \uc804\uccb4\uc815\uc6d0\uc5d0\uc11c 2/3\uac00\ub7c9 \uc120\ubc1c\ud55c\ub2e4\ub294 \ub0b4\uaddc\uac00 \uc788\uc5b4 \uc870\uc120\uc778\uc758 \uc785\ud559 \uacbd\uc7c1\uc774 \uce58\uc5f4\ud558\uc9c0 \uc54a\uc558\uc9c0\ub9cc, \uad50\uc721\ub839 \uac1c\uc815 \uc774\ud6c4 \uadf8\ub7ec\ud55c \ub0b4\uaddc\uac00 \ud3d0\uc9c0\ub418\uc5b4 \uc785\uc2dc \uacbd\uc7c1\ub960\uc740 13:1\uc5d0\uc11c 27:1\uae4c\uc9c0 \ud3ed\uc99d\ud558\uac8c \ub418\uc5c8\uace0 \uc870\uc120\uc778\uc758 \uc785\ud559\uc774 \ub9e4\uc6b0 \uc5b4\ub824\uc6cc\uc84c\ub2e4. \uc774 \ub54c\ubb38\uc5d0 1935\ub144\uc758 \uc785\uc2dc\uc5d0\uc11c\ub294 \ud1a0\ubaa9\uacfc\uc5d0 \uc870\uc120\uc778 \ud559\uc0dd\uc774 \ub2e8 1\uba85\ub9cc \ud569\uaca9\ud558\uae30\ub3c4 \ud588\ub2e4. \ub2f9\uc2dc \uad50\uc721\uacc4\uc5d0\uc11c\ub294 \uacbd\uc131\uace0\ub4f1\uc0c1\uc5c5\ud559\uad50(\uacbd\uc131\uace0\uc0c1)\uc758 \uc870\uc120\uc778 \uc785\ud559\ub09c\uacfc \ud568\uaed8 \uacbd\uc131\uace0\uacf5\uacfc \uacbd\uc131\uace0\uc0c1\uc758 \uc785\uc2dc\uc81c\ub3c4\ub97c \uc870\uc120\uc778\uc5d0\uac8c \uc720\ub9ac\ud558\uac8c \uc2dc\uc815\ud574\uc57c \ud55c\ub2e4\ub294 \uc758\uacac\uc774 \uc788\uc5c8\uc73c\ub098, \uacbd\uc131\uace0\ub4f1\uacf5\uc5c5\ud559\uad50\uc640 \uc870\uc120\ucd1d\ub3c5\ubd80 \ud559\ubb34\uad6d\uc5d0\uc11c \ubc1b\uc544\ub4e4\uc774\uc9c0 \uc54a\uc558\ub2e4."} {"question": "1403\ub144 \uc601\ub77d\uc81c\uac00 \ub9cc\uc8fc\uc758 \uc5ec\uc9c4\uc871\uc744 \ud1b5\ud560\ud558\uae30 \uc704\ud574 \ubc31\ub450\uc0b0 \ubd81\ucabd\uc5d0 \uc124\uce58\ud55c \uac83\uc740?", "paragraph": "\ub610\ud55c \ub3d9\ubd81\uc9c0\ubc29\uc5d0 \uc131\uc744 \uc313\uace0 \uc5ec\uc9c4\uc871\uc744 \uacf5\ub7b5\ud558\ub294 \ud55c\ud3b8 \uc5ec\uc9c4\uc871\uc744 \ud1b5\uc81c\ud558\ub294 \ud1b5\uc81c\uae30\uad00\uc758 \uc124\uce58\ub97c \ucd94\uc9c4\ud55c\ub2e4. 1403\ub144 \ub9cc\uc8fc\uc758 \uc5ec\uc9c4\uc871\uc744 \ud1b5\ud560\ud558\uae30 \uc704\ud558\uc5ec \ubc31\ub450\uc0b0 \ubd81\ucabd\uc5d0 \uac74\uc8fc\uc704(\u5efa\u5dde\u885b)\ub97c \uc124\uce58\ud588\ub294\ub370, \uac74\uc8fc\uc704\uc758 \uc124\uce58\uc7a5\uc18c\ub294 \uac74\uc8fc \uc9c0\ub9b0 \uc131 \ubd80\uadfc\uc758 \ud718\ubc1c\ucc9c(\u8f1d\u767c\u5ddd) \uc0c1\ub958\uc5d0 \uc788\ub294 \ubd81\uc0b0\uc131\uc790(\u5317\u5c71\u57ce\u5b50)\uc600\ub2e4. \uadf8\ub7ec\ub098 \uc5ec\uc9c4\uc871\uc758 \ubd80\ub77d\uc740 \ub2e4\uc591\ud588\uace0, 1411\ub144\uc5d0\ub294 \ud5e4\uc774\ub8fd\uac15(\u9ed1\u9f8d\u6c5f) \ud558\ub958\uc5d0 \ub204\ub974\uac04 \ub3c4\uc9c0\ud718\uc0ac\uc0ac(\u5974\u5152\u5e72\u90fd\u6307\u63ee\u4f7f\u53f8)\ub97c \ub450\uc5c8\ub2e4. \uac74\uc8fc\uc704\u00b7\uc6b0\uc790\uc704(\u5140\u8005\u885b)\u00b7\ub204\ub974\uac04\uc704(\u5974\u5152\u5e72\u885b)\ub97c \uc77c\uad04 \ud1b5\uc81c\ud558\uae30 \uc704\ud558\uc5ec \uc601\ub77d\uc81c\ub294 1411\ub144 \ud0dc\uac10(\u592a\u76e3) \uc774\uc2dc\ud558(\u8d64\u5931\u54c8) \ub4f1\uc5d0\uac8c \uba85\ud558\uc5ec \uad70\ubcd1 \uc57d 1,000\uc744 \uc778\uc194\ud558\uace0 25\ucc99\uc758 \uc120\ubc15\uc73c\ub85c \uc479\ud654 \uac15(\u677e\u82b1\u6c5f)\u00b7\ud5e4\uc774\ub8fd\uac15\uc774 \ub9cc\ub098\ub294 \ud558\ub958\uc9c0\uc810\uc5d0 \ud589\uc815\uad00\uccad\uc778 \ub3c4\uc0ac(\u90fd\u53f8)\ub97c \uc124\uce58\ud558\uc5ec 3\uac1c\uc758 \uc5ec\uc9c4 \ubd80\ub77d\uc744 \uac10\uc2dc, \ud1b5\uc81c\ud558\uc600\ub2e4.", "answer": "\uac74\uc8fc\uc704(\u5efa\u5dde\u885b)", "sentence": "1403\ub144 \ub9cc\uc8fc\uc758 \uc5ec\uc9c4\uc871\uc744 \ud1b5\ud560\ud558\uae30 \uc704\ud558\uc5ec \ubc31\ub450\uc0b0 \ubd81\ucabd\uc5d0 \uac74\uc8fc\uc704(\u5efa\u5dde\u885b) \ub97c \uc124\uce58\ud588\ub294\ub370, \uac74\uc8fc\uc704\uc758 \uc124\uce58\uc7a5\uc18c\ub294 \uac74\uc8fc \uc9c0\ub9b0 \uc131 \ubd80\uadfc\uc758 \ud718\ubc1c\ucc9c(\u8f1d\u767c\u5ddd) \uc0c1\ub958\uc5d0 \uc788\ub294 \ubd81\uc0b0\uc131\uc790(\u5317\u5c71\u57ce\u5b50)\uc600\ub2e4.", "paragraph_sentence": "\ub610\ud55c \ub3d9\ubd81\uc9c0\ubc29\uc5d0 \uc131\uc744 \uc313\uace0 \uc5ec\uc9c4\uc871\uc744 \uacf5\ub7b5\ud558\ub294 \ud55c\ud3b8 \uc5ec\uc9c4\uc871\uc744 \ud1b5\uc81c\ud558\ub294 \ud1b5\uc81c\uae30\uad00\uc758 \uc124\uce58\ub97c \ucd94\uc9c4\ud55c\ub2e4. 1403\ub144 \ub9cc\uc8fc\uc758 \uc5ec\uc9c4\uc871\uc744 \ud1b5\ud560\ud558\uae30 \uc704\ud558\uc5ec \ubc31\ub450\uc0b0 \ubd81\ucabd\uc5d0 \uac74\uc8fc\uc704(\u5efa\u5dde\u885b) \ub97c \uc124\uce58\ud588\ub294\ub370, \uac74\uc8fc\uc704\uc758 \uc124\uce58\uc7a5\uc18c\ub294 \uac74\uc8fc \uc9c0\ub9b0 \uc131 \ubd80\uadfc\uc758 \ud718\ubc1c\ucc9c(\u8f1d\u767c\u5ddd) \uc0c1\ub958\uc5d0 \uc788\ub294 \ubd81\uc0b0\uc131\uc790(\u5317\u5c71\u57ce\u5b50)\uc600\ub2e4. \uadf8\ub7ec\ub098 \uc5ec\uc9c4\uc871\uc758 \ubd80\ub77d\uc740 \ub2e4\uc591\ud588\uace0, 1411\ub144\uc5d0\ub294 \ud5e4\uc774\ub8fd\uac15(\u9ed1\u9f8d\u6c5f) \ud558\ub958\uc5d0 \ub204\ub974\uac04 \ub3c4\uc9c0\ud718\uc0ac\uc0ac(\u5974\u5152\u5e72\u90fd\u6307\u63ee\u4f7f\u53f8)\ub97c \ub450\uc5c8\ub2e4. \uac74\uc8fc\uc704\u00b7\uc6b0\uc790\uc704(\u5140\u8005\u885b)\u00b7\ub204\ub974\uac04\uc704(\u5974\u5152\u5e72\u885b)\ub97c \uc77c\uad04 \ud1b5\uc81c\ud558\uae30 \uc704\ud558\uc5ec \uc601\ub77d\uc81c\ub294 1411\ub144 \ud0dc\uac10(\u592a\u76e3) \uc774\uc2dc\ud558(\u8d64\u5931\u54c8) \ub4f1\uc5d0\uac8c \uba85\ud558\uc5ec \uad70\ubcd1 \uc57d 1,000\uc744 \uc778\uc194\ud558\uace0 25\ucc99\uc758 \uc120\ubc15\uc73c\ub85c \uc479\ud654 \uac15(\u677e\u82b1\u6c5f)\u00b7\ud5e4\uc774\ub8fd\uac15\uc774 \ub9cc\ub098\ub294 \ud558\ub958\uc9c0\uc810\uc5d0 \ud589\uc815\uad00\uccad\uc778 \ub3c4\uc0ac(\u90fd\u53f8)\ub97c \uc124\uce58\ud558\uc5ec 3\uac1c\uc758 \uc5ec\uc9c4 \ubd80\ub77d\uc744 \uac10\uc2dc, \ud1b5\uc81c\ud558\uc600\ub2e4.", "paragraph_answer": "\ub610\ud55c \ub3d9\ubd81\uc9c0\ubc29\uc5d0 \uc131\uc744 \uc313\uace0 \uc5ec\uc9c4\uc871\uc744 \uacf5\ub7b5\ud558\ub294 \ud55c\ud3b8 \uc5ec\uc9c4\uc871\uc744 \ud1b5\uc81c\ud558\ub294 \ud1b5\uc81c\uae30\uad00\uc758 \uc124\uce58\ub97c \ucd94\uc9c4\ud55c\ub2e4. 1403\ub144 \ub9cc\uc8fc\uc758 \uc5ec\uc9c4\uc871\uc744 \ud1b5\ud560\ud558\uae30 \uc704\ud558\uc5ec \ubc31\ub450\uc0b0 \ubd81\ucabd\uc5d0 \uac74\uc8fc\uc704(\u5efa\u5dde\u885b) \ub97c \uc124\uce58\ud588\ub294\ub370, \uac74\uc8fc\uc704\uc758 \uc124\uce58\uc7a5\uc18c\ub294 \uac74\uc8fc \uc9c0\ub9b0 \uc131 \ubd80\uadfc\uc758 \ud718\ubc1c\ucc9c(\u8f1d\u767c\u5ddd) \uc0c1\ub958\uc5d0 \uc788\ub294 \ubd81\uc0b0\uc131\uc790(\u5317\u5c71\u57ce\u5b50)\uc600\ub2e4. \uadf8\ub7ec\ub098 \uc5ec\uc9c4\uc871\uc758 \ubd80\ub77d\uc740 \ub2e4\uc591\ud588\uace0, 1411\ub144\uc5d0\ub294 \ud5e4\uc774\ub8fd\uac15(\u9ed1\u9f8d\u6c5f) \ud558\ub958\uc5d0 \ub204\ub974\uac04 \ub3c4\uc9c0\ud718\uc0ac\uc0ac(\u5974\u5152\u5e72\u90fd\u6307\u63ee\u4f7f\u53f8)\ub97c \ub450\uc5c8\ub2e4. \uac74\uc8fc\uc704\u00b7\uc6b0\uc790\uc704(\u5140\u8005\u885b)\u00b7\ub204\ub974\uac04\uc704(\u5974\u5152\u5e72\u885b)\ub97c \uc77c\uad04 \ud1b5\uc81c\ud558\uae30 \uc704\ud558\uc5ec \uc601\ub77d\uc81c\ub294 1411\ub144 \ud0dc\uac10(\u592a\u76e3) \uc774\uc2dc\ud558(\u8d64\u5931\u54c8) \ub4f1\uc5d0\uac8c \uba85\ud558\uc5ec \uad70\ubcd1 \uc57d 1,000\uc744 \uc778\uc194\ud558\uace0 25\ucc99\uc758 \uc120\ubc15\uc73c\ub85c \uc479\ud654 \uac15(\u677e\u82b1\u6c5f)\u00b7\ud5e4\uc774\ub8fd\uac15\uc774 \ub9cc\ub098\ub294 \ud558\ub958\uc9c0\uc810\uc5d0 \ud589\uc815\uad00\uccad\uc778 \ub3c4\uc0ac(\u90fd\u53f8)\ub97c \uc124\uce58\ud558\uc5ec 3\uac1c\uc758 \uc5ec\uc9c4 \ubd80\ub77d\uc744 \uac10\uc2dc, \ud1b5\uc81c\ud558\uc600\ub2e4.", "sentence_answer": "1403\ub144 \ub9cc\uc8fc\uc758 \uc5ec\uc9c4\uc871\uc744 \ud1b5\ud560\ud558\uae30 \uc704\ud558\uc5ec \ubc31\ub450\uc0b0 \ubd81\ucabd\uc5d0 \uac74\uc8fc\uc704(\u5efa\u5dde\u885b) \ub97c \uc124\uce58\ud588\ub294\ub370, \uac74\uc8fc\uc704\uc758 \uc124\uce58\uc7a5\uc18c\ub294 \uac74\uc8fc \uc9c0\ub9b0 \uc131 \ubd80\uadfc\uc758 \ud718\ubc1c\ucc9c(\u8f1d\u767c\u5ddd) \uc0c1\ub958\uc5d0 \uc788\ub294 \ubd81\uc0b0\uc131\uc790(\u5317\u5c71\u57ce\u5b50)\uc600\ub2e4.", "paragraph_id": "6524266-4-0", "paragraph_question": "question: 1403\ub144 \uc601\ub77d\uc81c\uac00 \ub9cc\uc8fc\uc758 \uc5ec\uc9c4\uc871\uc744 \ud1b5\ud560\ud558\uae30 \uc704\ud574 \ubc31\ub450\uc0b0 \ubd81\ucabd\uc5d0 \uc124\uce58\ud55c \uac83\uc740?, context: \ub610\ud55c \ub3d9\ubd81\uc9c0\ubc29\uc5d0 \uc131\uc744 \uc313\uace0 \uc5ec\uc9c4\uc871\uc744 \uacf5\ub7b5\ud558\ub294 \ud55c\ud3b8 \uc5ec\uc9c4\uc871\uc744 \ud1b5\uc81c\ud558\ub294 \ud1b5\uc81c\uae30\uad00\uc758 \uc124\uce58\ub97c \ucd94\uc9c4\ud55c\ub2e4. 1403\ub144 \ub9cc\uc8fc\uc758 \uc5ec\uc9c4\uc871\uc744 \ud1b5\ud560\ud558\uae30 \uc704\ud558\uc5ec \ubc31\ub450\uc0b0 \ubd81\ucabd\uc5d0 \uac74\uc8fc\uc704(\u5efa\u5dde\u885b)\ub97c \uc124\uce58\ud588\ub294\ub370, \uac74\uc8fc\uc704\uc758 \uc124\uce58\uc7a5\uc18c\ub294 \uac74\uc8fc \uc9c0\ub9b0 \uc131 \ubd80\uadfc\uc758 \ud718\ubc1c\ucc9c(\u8f1d\u767c\u5ddd) \uc0c1\ub958\uc5d0 \uc788\ub294 \ubd81\uc0b0\uc131\uc790(\u5317\u5c71\u57ce\u5b50)\uc600\ub2e4. \uadf8\ub7ec\ub098 \uc5ec\uc9c4\uc871\uc758 \ubd80\ub77d\uc740 \ub2e4\uc591\ud588\uace0, 1411\ub144\uc5d0\ub294 \ud5e4\uc774\ub8fd\uac15(\u9ed1\u9f8d\u6c5f) \ud558\ub958\uc5d0 \ub204\ub974\uac04 \ub3c4\uc9c0\ud718\uc0ac\uc0ac(\u5974\u5152\u5e72\u90fd\u6307\u63ee\u4f7f\u53f8)\ub97c \ub450\uc5c8\ub2e4. \uac74\uc8fc\uc704\u00b7\uc6b0\uc790\uc704(\u5140\u8005\u885b)\u00b7\ub204\ub974\uac04\uc704(\u5974\u5152\u5e72\u885b)\ub97c \uc77c\uad04 \ud1b5\uc81c\ud558\uae30 \uc704\ud558\uc5ec \uc601\ub77d\uc81c\ub294 1411\ub144 \ud0dc\uac10(\u592a\u76e3) \uc774\uc2dc\ud558(\u8d64\u5931\u54c8) \ub4f1\uc5d0\uac8c \uba85\ud558\uc5ec \uad70\ubcd1 \uc57d 1,000\uc744 \uc778\uc194\ud558\uace0 25\ucc99\uc758 \uc120\ubc15\uc73c\ub85c \uc479\ud654 \uac15(\u677e\u82b1\u6c5f)\u00b7\ud5e4\uc774\ub8fd\uac15\uc774 \ub9cc\ub098\ub294 \ud558\ub958\uc9c0\uc810\uc5d0 \ud589\uc815\uad00\uccad\uc778 \ub3c4\uc0ac(\u90fd\u53f8)\ub97c \uc124\uce58\ud558\uc5ec 3\uac1c\uc758 \uc5ec\uc9c4 \ubd80\ub77d\uc744 \uac10\uc2dc, \ud1b5\uc81c\ud558\uc600\ub2e4."} {"question": "\uc601\uad6d\uacfc\uc758 \uad50\uc5ed\uc774 \uc2dc\uc791\ub41c \ud6c4 \ubcf8\uaca9\uc801\uc73c\ub85c \uccad\ub098\ub77c\uc5d0 \ubc18\uc785\ub41c \ubb3c\ud488\uc740 \ubb34\uc5c7\uc778\uac00?", "paragraph": "\uc774 \uc2dc\uae30\uc5d0 \uc544\ud3b8\ub3c4 \ubcf8\uaca9\uc801\uc73c\ub85c \ub4e4\uc5b4\uc624\uae30 \uc2dc\uc791\ud588\ub294\ub370 \uc601\uad6d\uacfc\uc758 \uad50\uc5ed\uc774 \uc2dc\uc791\ub41c \uc774\ud6c4 \uc601\uad6d \uc0c1\uc778\ub4e4\uc740 \ub3c8\uc744 \ub354 \ubc8c\uae30 \uc704\ud574 \uc778\ub3c4\uc5d0\uc11c \uc544\ud3b8\uc744 \uac00\uc838\uc640 \uc911\uad6d\uc5d0 \ud314\uae30 \uc2dc\uc791\ud558\uc600\ub2e4. \ubcf8\ub798 1729\ub144(\uc639\uc815 7\ub144)\uc5d0 \uc774\ubbf8 \uc544\ud3b8 \uae08\uc9c0\ub839\uc774 \ub0b4\ub9ac\uace0 2\ub144 \ub4a4\uc778 1731\ub144(\uc639\uc815 9\ub144)\uc5d0\ub294 \uc544\ud3b8 \ud761\uc5f0\uc790\uc640 \uc544\ud3b8 \ud310\ub9e4\uc790\ub97c \uac00\ucc28\uc5c6\uc774 \ucc98\ud615\ud558\ub294 \ub4f1\uc758 \ub2e8\uc18d\uc744 \ub300\ud3ed \uac15\ud654\ud558\ub294 \ub4f1 \uac15\uacbd\ucc45\uc744 \uc37c\uc73c\ub098 \uac74\ub96d\uc81c\uc758 \uce58\uc138\ub97c \uac70\uce58\uba74\uc11c \uc815\ucc45\ub4e4\uc774 \ub290\uc2a8\ud574\uc9c0\uace0 \uc9c0\ubc29\uc758 \uad00\ub9ac\ub4e4 \uc5ed\uc2dc \uc601\uad6d \uc0c1\uc778\ub4e4\uc5d0\uac8c \ub1cc\ubb3c\uc744 \ubc1b\uc73c\uba74\uc11c \uc544\ud3b8\uc5d0 \ub300\ud55c \uc81c\uc7ac\uac00 \ud06c\uac8c \ub290\uc2a8\ud574\uc838 \uc544\ud3b8\uc774 \ub2e4\uc2dc \ub4e4\uc5b4\uc624\uae30 \uc2dc\uc791\ud558\uc600\uace0 \uac00\uacbd\uc81c \uce58\uc138 \ub54c\uc5d0\ub294 \ubbfc\uac04\uc740 \ubb3c\ub860 \uc911\uc559\uacfc \uc9c0\ubc29 \uad00\ub9ac\ub4e4\ub3c4 \uc544\ud3b8\uc744 \ud761\uc5f0\ud558\uc600\ub2e4. \uc544\ud3b8 \uc218\uc785\ub7c9\uc740 1816\ub144(\uac00\uacbd 21\ub144) 5000\uc0c1\uc790\ub97c \uc2dc\uc791\uc73c\ub85c \uac00\uacbd\uc81c \ub9d0\ub144\uc778 1820\ub144(\uac00\uacbd 25\ub144)\uc5d0\ub294 \uc774\ubbf8 1\ub9cc \uc0c1\uc790\uc5d0 \uc721\ubc15\ud560 \uc815\ub3c4\ub85c \ub9ce\uc740 \uc591\uc758 \uc544\ud3b8\uc774 \ub4e4\uc5b4\uc654\ub2e4.", "answer": "\uc544\ud3b8", "sentence": "\uc774 \uc2dc\uae30\uc5d0 \uc544\ud3b8 \ub3c4 \ubcf8\uaca9\uc801\uc73c\ub85c \ub4e4\uc5b4\uc624\uae30 \uc2dc\uc791\ud588\ub294\ub370 \uc601\uad6d\uacfc\uc758 \uad50\uc5ed\uc774 \uc2dc\uc791\ub41c \uc774\ud6c4 \uc601\uad6d \uc0c1\uc778\ub4e4\uc740 \ub3c8\uc744 \ub354 \ubc8c\uae30 \uc704\ud574 \uc778\ub3c4\uc5d0\uc11c \uc544\ud3b8\uc744 \uac00\uc838\uc640 \uc911\uad6d\uc5d0 \ud314\uae30 \uc2dc\uc791\ud558\uc600\ub2e4.", "paragraph_sentence": " \uc774 \uc2dc\uae30\uc5d0 \uc544\ud3b8 \ub3c4 \ubcf8\uaca9\uc801\uc73c\ub85c \ub4e4\uc5b4\uc624\uae30 \uc2dc\uc791\ud588\ub294\ub370 \uc601\uad6d\uacfc\uc758 \uad50\uc5ed\uc774 \uc2dc\uc791\ub41c \uc774\ud6c4 \uc601\uad6d \uc0c1\uc778\ub4e4\uc740 \ub3c8\uc744 \ub354 \ubc8c\uae30 \uc704\ud574 \uc778\ub3c4\uc5d0\uc11c \uc544\ud3b8\uc744 \uac00\uc838\uc640 \uc911\uad6d\uc5d0 \ud314\uae30 \uc2dc\uc791\ud558\uc600\ub2e4. \ubcf8\ub798 1729\ub144(\uc639\uc815 7\ub144)\uc5d0 \uc774\ubbf8 \uc544\ud3b8 \uae08\uc9c0\ub839\uc774 \ub0b4\ub9ac\uace0 2\ub144 \ub4a4\uc778 1731\ub144(\uc639\uc815 9\ub144)\uc5d0\ub294 \uc544\ud3b8 \ud761\uc5f0\uc790\uc640 \uc544\ud3b8 \ud310\ub9e4\uc790\ub97c \uac00\ucc28\uc5c6\uc774 \ucc98\ud615\ud558\ub294 \ub4f1\uc758 \ub2e8\uc18d\uc744 \ub300\ud3ed \uac15\ud654\ud558\ub294 \ub4f1 \uac15\uacbd\ucc45\uc744 \uc37c\uc73c\ub098 \uac74\ub96d\uc81c\uc758 \uce58\uc138\ub97c \uac70\uce58\uba74\uc11c \uc815\ucc45\ub4e4\uc774 \ub290\uc2a8\ud574\uc9c0\uace0 \uc9c0\ubc29\uc758 \uad00\ub9ac\ub4e4 \uc5ed\uc2dc \uc601\uad6d \uc0c1\uc778\ub4e4\uc5d0\uac8c \ub1cc\ubb3c\uc744 \ubc1b\uc73c\uba74\uc11c \uc544\ud3b8\uc5d0 \ub300\ud55c \uc81c\uc7ac\uac00 \ud06c\uac8c \ub290\uc2a8\ud574\uc838 \uc544\ud3b8\uc774 \ub2e4\uc2dc \ub4e4\uc5b4\uc624\uae30 \uc2dc\uc791\ud558\uc600\uace0 \uac00\uacbd\uc81c \uce58\uc138 \ub54c\uc5d0\ub294 \ubbfc\uac04\uc740 \ubb3c\ub860 \uc911\uc559\uacfc \uc9c0\ubc29 \uad00\ub9ac\ub4e4\ub3c4 \uc544\ud3b8\uc744 \ud761\uc5f0\ud558\uc600\ub2e4. \uc544\ud3b8 \uc218\uc785\ub7c9\uc740 1816\ub144(\uac00\uacbd 21\ub144) 5000\uc0c1\uc790\ub97c \uc2dc\uc791\uc73c\ub85c \uac00\uacbd\uc81c \ub9d0\ub144\uc778 1820\ub144(\uac00\uacbd 25\ub144)\uc5d0\ub294 \uc774\ubbf8 1\ub9cc \uc0c1\uc790\uc5d0 \uc721\ubc15\ud560 \uc815\ub3c4\ub85c \ub9ce\uc740 \uc591\uc758 \uc544\ud3b8\uc774 \ub4e4\uc5b4\uc654\ub2e4.", "paragraph_answer": "\uc774 \uc2dc\uae30\uc5d0 \uc544\ud3b8 \ub3c4 \ubcf8\uaca9\uc801\uc73c\ub85c \ub4e4\uc5b4\uc624\uae30 \uc2dc\uc791\ud588\ub294\ub370 \uc601\uad6d\uacfc\uc758 \uad50\uc5ed\uc774 \uc2dc\uc791\ub41c \uc774\ud6c4 \uc601\uad6d \uc0c1\uc778\ub4e4\uc740 \ub3c8\uc744 \ub354 \ubc8c\uae30 \uc704\ud574 \uc778\ub3c4\uc5d0\uc11c \uc544\ud3b8\uc744 \uac00\uc838\uc640 \uc911\uad6d\uc5d0 \ud314\uae30 \uc2dc\uc791\ud558\uc600\ub2e4. \ubcf8\ub798 1729\ub144(\uc639\uc815 7\ub144)\uc5d0 \uc774\ubbf8 \uc544\ud3b8 \uae08\uc9c0\ub839\uc774 \ub0b4\ub9ac\uace0 2\ub144 \ub4a4\uc778 1731\ub144(\uc639\uc815 9\ub144)\uc5d0\ub294 \uc544\ud3b8 \ud761\uc5f0\uc790\uc640 \uc544\ud3b8 \ud310\ub9e4\uc790\ub97c \uac00\ucc28\uc5c6\uc774 \ucc98\ud615\ud558\ub294 \ub4f1\uc758 \ub2e8\uc18d\uc744 \ub300\ud3ed \uac15\ud654\ud558\ub294 \ub4f1 \uac15\uacbd\ucc45\uc744 \uc37c\uc73c\ub098 \uac74\ub96d\uc81c\uc758 \uce58\uc138\ub97c \uac70\uce58\uba74\uc11c \uc815\ucc45\ub4e4\uc774 \ub290\uc2a8\ud574\uc9c0\uace0 \uc9c0\ubc29\uc758 \uad00\ub9ac\ub4e4 \uc5ed\uc2dc \uc601\uad6d \uc0c1\uc778\ub4e4\uc5d0\uac8c \ub1cc\ubb3c\uc744 \ubc1b\uc73c\uba74\uc11c \uc544\ud3b8\uc5d0 \ub300\ud55c \uc81c\uc7ac\uac00 \ud06c\uac8c \ub290\uc2a8\ud574\uc838 \uc544\ud3b8\uc774 \ub2e4\uc2dc \ub4e4\uc5b4\uc624\uae30 \uc2dc\uc791\ud558\uc600\uace0 \uac00\uacbd\uc81c \uce58\uc138 \ub54c\uc5d0\ub294 \ubbfc\uac04\uc740 \ubb3c\ub860 \uc911\uc559\uacfc \uc9c0\ubc29 \uad00\ub9ac\ub4e4\ub3c4 \uc544\ud3b8\uc744 \ud761\uc5f0\ud558\uc600\ub2e4. \uc544\ud3b8 \uc218\uc785\ub7c9\uc740 1816\ub144(\uac00\uacbd 21\ub144) 5000\uc0c1\uc790\ub97c \uc2dc\uc791\uc73c\ub85c \uac00\uacbd\uc81c \ub9d0\ub144\uc778 1820\ub144(\uac00\uacbd 25\ub144)\uc5d0\ub294 \uc774\ubbf8 1\ub9cc \uc0c1\uc790\uc5d0 \uc721\ubc15\ud560 \uc815\ub3c4\ub85c \ub9ce\uc740 \uc591\uc758 \uc544\ud3b8\uc774 \ub4e4\uc5b4\uc654\ub2e4.", "sentence_answer": "\uc774 \uc2dc\uae30\uc5d0 \uc544\ud3b8 \ub3c4 \ubcf8\uaca9\uc801\uc73c\ub85c \ub4e4\uc5b4\uc624\uae30 \uc2dc\uc791\ud588\ub294\ub370 \uc601\uad6d\uacfc\uc758 \uad50\uc5ed\uc774 \uc2dc\uc791\ub41c \uc774\ud6c4 \uc601\uad6d \uc0c1\uc778\ub4e4\uc740 \ub3c8\uc744 \ub354 \ubc8c\uae30 \uc704\ud574 \uc778\ub3c4\uc5d0\uc11c \uc544\ud3b8\uc744 \uac00\uc838\uc640 \uc911\uad6d\uc5d0 \ud314\uae30 \uc2dc\uc791\ud558\uc600\ub2e4.", "paragraph_id": "6472899-10-0", "paragraph_question": "question: \uc601\uad6d\uacfc\uc758 \uad50\uc5ed\uc774 \uc2dc\uc791\ub41c \ud6c4 \ubcf8\uaca9\uc801\uc73c\ub85c \uccad\ub098\ub77c\uc5d0 \ubc18\uc785\ub41c \ubb3c\ud488\uc740 \ubb34\uc5c7\uc778\uac00?, context: \uc774 \uc2dc\uae30\uc5d0 \uc544\ud3b8\ub3c4 \ubcf8\uaca9\uc801\uc73c\ub85c \ub4e4\uc5b4\uc624\uae30 \uc2dc\uc791\ud588\ub294\ub370 \uc601\uad6d\uacfc\uc758 \uad50\uc5ed\uc774 \uc2dc\uc791\ub41c \uc774\ud6c4 \uc601\uad6d \uc0c1\uc778\ub4e4\uc740 \ub3c8\uc744 \ub354 \ubc8c\uae30 \uc704\ud574 \uc778\ub3c4\uc5d0\uc11c \uc544\ud3b8\uc744 \uac00\uc838\uc640 \uc911\uad6d\uc5d0 \ud314\uae30 \uc2dc\uc791\ud558\uc600\ub2e4. \ubcf8\ub798 1729\ub144(\uc639\uc815 7\ub144)\uc5d0 \uc774\ubbf8 \uc544\ud3b8 \uae08\uc9c0\ub839\uc774 \ub0b4\ub9ac\uace0 2\ub144 \ub4a4\uc778 1731\ub144(\uc639\uc815 9\ub144)\uc5d0\ub294 \uc544\ud3b8 \ud761\uc5f0\uc790\uc640 \uc544\ud3b8 \ud310\ub9e4\uc790\ub97c \uac00\ucc28\uc5c6\uc774 \ucc98\ud615\ud558\ub294 \ub4f1\uc758 \ub2e8\uc18d\uc744 \ub300\ud3ed \uac15\ud654\ud558\ub294 \ub4f1 \uac15\uacbd\ucc45\uc744 \uc37c\uc73c\ub098 \uac74\ub96d\uc81c\uc758 \uce58\uc138\ub97c \uac70\uce58\uba74\uc11c \uc815\ucc45\ub4e4\uc774 \ub290\uc2a8\ud574\uc9c0\uace0 \uc9c0\ubc29\uc758 \uad00\ub9ac\ub4e4 \uc5ed\uc2dc \uc601\uad6d \uc0c1\uc778\ub4e4\uc5d0\uac8c \ub1cc\ubb3c\uc744 \ubc1b\uc73c\uba74\uc11c \uc544\ud3b8\uc5d0 \ub300\ud55c \uc81c\uc7ac\uac00 \ud06c\uac8c \ub290\uc2a8\ud574\uc838 \uc544\ud3b8\uc774 \ub2e4\uc2dc \ub4e4\uc5b4\uc624\uae30 \uc2dc\uc791\ud558\uc600\uace0 \uac00\uacbd\uc81c \uce58\uc138 \ub54c\uc5d0\ub294 \ubbfc\uac04\uc740 \ubb3c\ub860 \uc911\uc559\uacfc \uc9c0\ubc29 \uad00\ub9ac\ub4e4\ub3c4 \uc544\ud3b8\uc744 \ud761\uc5f0\ud558\uc600\ub2e4. \uc544\ud3b8 \uc218\uc785\ub7c9\uc740 1816\ub144(\uac00\uacbd 21\ub144) 5000\uc0c1\uc790\ub97c \uc2dc\uc791\uc73c\ub85c \uac00\uacbd\uc81c \ub9d0\ub144\uc778 1820\ub144(\uac00\uacbd 25\ub144)\uc5d0\ub294 \uc774\ubbf8 1\ub9cc \uc0c1\uc790\uc5d0 \uc721\ubc15\ud560 \uc815\ub3c4\ub85c \ub9ce\uc740 \uc591\uc758 \uc544\ud3b8\uc774 \ub4e4\uc5b4\uc654\ub2e4."} {"question": "16\uc138 \uc774\ud558\uc758 \uc5ec\uc131 \uccad\uc18c\ub144\ub4e4\uc774 \uac80\uc740 \uc18d\uc637\uc744 \uad6c\uc785\ud558\uc5ec \ud56b \ud32c\uce20\ucc98\ub7fc \uc785\ub294 \uac83\uc740 \uac00\uac00\uc758 \uc2a4\ud0c0\uc77c\uc744 \ubcf8\ub72c \uac83\uc774\ub77c\uace0 \ub9d0\ud55c \uc774\ub294?", "paragraph": "\uadf8\ub140\uc758 \uc774\ubbf8\uc9c0\uc640 \ud328\uc158 \uac10\uac01\uc740 \ud328\ub9ac\uc2a4 \ud790\ud2bc\uacfc \ub2c8\ucf5c \ub9ac\uce58\uc640 \uac19\uc740 \uba85\uc0ac\ub4e4\uc5d0\uac8c \ub9ce\uc740 \uc601\ud5a5\uc744 \uc8fc\uc5c8\ub2e4. \ucd5c\uadfc\uc758 \uc5f0\uad6c\uc5d0\uc11c\ub294 \uac00\uac00\uc758 \uc601\ud5a5\uc73c\ub85c \uc601\uad6d\uc758 \ub2c8\ucee4\uc2a4 \uc18c\ube44\uac00 25% \uc99d\uac00\ud558\uc600\ub2e4\ub294 \uacb0\uacfc\uac00 \ubc1c\ud45c\ub418\uc5c8\ub2e4. \ub780\uc81c\ub9ac \uc804\ubb38\uac00 \uc544\ub124\ud2b8 \uc6cc\ubc84\ud2bc\uc5d0 \ub530\ub974\uba74, 16\uc138 \uc774\ud558\uc758 \uc5ec\uc131 \uccad\uc18c\ub144\ub4e4\uc774 \uac80\uc740 \uc18d\uc637\uc744 \uad6c\uc785\ud558\uc5ec \ud56b \ud32c\uce20\ucc98\ub7fc \uc785\ub294 \uac83\uc740 \uac00\uac00\uc758 \uc2a4\ud0c0\uc77c\uc744 \ubcf8\ub72c \uac83\uc774\ub77c\uace0 \ud55c\ub2e4. \ud55c\ud3b8, 2010\ub144 9\uc6d4 \uc5f4\ub9b0 2010 MTV \ube44\ub514\uc624 \ubba4\uc9c1 \uc5b4\uc6cc\ub4dc\uc5d0\uc11c \uc0dd\uace0\uae30 \ub4dc\ub808\uc2a4\ub97c \uc785\uace0 \ub098\uc640 \uc138\uacc4\uc801\uc73c\ub85c \ud070 \ud654\uc81c\uac00 \ub418\uc5c8\ub294\ub370, \uc774 \uc0dd\uace0\uae30 \ub4dc\ub808\uc2a4\ub294 \ub274\uc695\ud0c0\uc784\uc2a4\uac00 \uc120\uc815\ud55c 2010\ub144 \uc62c\ud574\uc758 \uc544\uc774\ub514\uc5b4\ub85c \uc120\uc815\ub418\uc5c8\uace0, \ubbf8\uad6d \ub85c\ud070\ub864 \uba85\uc608\uc758 \uc804\ub2f9\uc5d0 \ubc15\uc81c\ub97c \uac70\uccd0 \uac00\uac00\uc758 \uc5b4\ub9b0 \uc2dc\uc808 \ud53c\uc544\ub178\uc640 \ud568\uaed8 \"Women Who Rock: Vision, Passion, Power\"\ub77c\ub294 \uc774\ub984\uc73c\ub85c \ubcf4\uc874\ud558\uace0\uc788\ub2e4.", "answer": "\uc544\ub124\ud2b8 \uc6cc\ubc84\ud2bc", "sentence": "\ub780\uc81c\ub9ac \uc804\ubb38\uac00 \uc544\ub124\ud2b8 \uc6cc\ubc84\ud2bc \uc5d0", "paragraph_sentence": "\uadf8\ub140\uc758 \uc774\ubbf8\uc9c0\uc640 \ud328\uc158 \uac10\uac01\uc740 \ud328\ub9ac\uc2a4 \ud790\ud2bc\uacfc \ub2c8\ucf5c \ub9ac\uce58\uc640 \uac19\uc740 \uba85\uc0ac\ub4e4\uc5d0\uac8c \ub9ce\uc740 \uc601\ud5a5\uc744 \uc8fc\uc5c8\ub2e4. \ucd5c\uadfc\uc758 \uc5f0\uad6c\uc5d0\uc11c\ub294 \uac00\uac00\uc758 \uc601\ud5a5\uc73c\ub85c \uc601\uad6d\uc758 \ub2c8\ucee4\uc2a4 \uc18c\ube44\uac00 25% \uc99d\uac00\ud558\uc600\ub2e4\ub294 \uacb0\uacfc\uac00 \ubc1c\ud45c\ub418\uc5c8\ub2e4. \ub780\uc81c\ub9ac \uc804\ubb38\uac00 \uc544\ub124\ud2b8 \uc6cc\ubc84\ud2bc \uc5d0 \ub530\ub974\uba74, 16\uc138 \uc774\ud558\uc758 \uc5ec\uc131 \uccad\uc18c\ub144\ub4e4\uc774 \uac80\uc740 \uc18d\uc637\uc744 \uad6c\uc785\ud558\uc5ec \ud56b \ud32c\uce20\ucc98\ub7fc \uc785\ub294 \uac83\uc740 \uac00\uac00\uc758 \uc2a4\ud0c0\uc77c\uc744 \ubcf8\ub72c \uac83\uc774\ub77c\uace0 \ud55c\ub2e4. \ud55c\ud3b8, 2010\ub144 9\uc6d4 \uc5f4\ub9b0 2010 MTV \ube44\ub514\uc624 \ubba4\uc9c1 \uc5b4\uc6cc\ub4dc\uc5d0\uc11c \uc0dd\uace0\uae30 \ub4dc\ub808\uc2a4\ub97c \uc785\uace0 \ub098\uc640 \uc138\uacc4\uc801\uc73c\ub85c \ud070 \ud654\uc81c\uac00 \ub418\uc5c8\ub294\ub370, \uc774 \uc0dd\uace0\uae30 \ub4dc\ub808\uc2a4\ub294 \ub274\uc695\ud0c0\uc784\uc2a4\uac00 \uc120\uc815\ud55c 2010\ub144 \uc62c\ud574\uc758 \uc544\uc774\ub514\uc5b4\ub85c \uc120\uc815\ub418\uc5c8\uace0, \ubbf8\uad6d \ub85c\ud070\ub864 \uba85\uc608\uc758 \uc804\ub2f9\uc5d0 \ubc15\uc81c\ub97c \uac70\uccd0 \uac00\uac00\uc758 \uc5b4\ub9b0 \uc2dc\uc808 \ud53c\uc544\ub178\uc640 \ud568\uaed8 \"Women Who Rock: Vision, Passion, Power\"\ub77c\ub294 \uc774\ub984\uc73c\ub85c \ubcf4\uc874\ud558\uace0\uc788\ub2e4.", "paragraph_answer": "\uadf8\ub140\uc758 \uc774\ubbf8\uc9c0\uc640 \ud328\uc158 \uac10\uac01\uc740 \ud328\ub9ac\uc2a4 \ud790\ud2bc\uacfc \ub2c8\ucf5c \ub9ac\uce58\uc640 \uac19\uc740 \uba85\uc0ac\ub4e4\uc5d0\uac8c \ub9ce\uc740 \uc601\ud5a5\uc744 \uc8fc\uc5c8\ub2e4. \ucd5c\uadfc\uc758 \uc5f0\uad6c\uc5d0\uc11c\ub294 \uac00\uac00\uc758 \uc601\ud5a5\uc73c\ub85c \uc601\uad6d\uc758 \ub2c8\ucee4\uc2a4 \uc18c\ube44\uac00 25% \uc99d\uac00\ud558\uc600\ub2e4\ub294 \uacb0\uacfc\uac00 \ubc1c\ud45c\ub418\uc5c8\ub2e4. \ub780\uc81c\ub9ac \uc804\ubb38\uac00 \uc544\ub124\ud2b8 \uc6cc\ubc84\ud2bc \uc5d0 \ub530\ub974\uba74, 16\uc138 \uc774\ud558\uc758 \uc5ec\uc131 \uccad\uc18c\ub144\ub4e4\uc774 \uac80\uc740 \uc18d\uc637\uc744 \uad6c\uc785\ud558\uc5ec \ud56b \ud32c\uce20\ucc98\ub7fc \uc785\ub294 \uac83\uc740 \uac00\uac00\uc758 \uc2a4\ud0c0\uc77c\uc744 \ubcf8\ub72c \uac83\uc774\ub77c\uace0 \ud55c\ub2e4. \ud55c\ud3b8, 2010\ub144 9\uc6d4 \uc5f4\ub9b0 2010 MTV \ube44\ub514\uc624 \ubba4\uc9c1 \uc5b4\uc6cc\ub4dc\uc5d0\uc11c \uc0dd\uace0\uae30 \ub4dc\ub808\uc2a4\ub97c \uc785\uace0 \ub098\uc640 \uc138\uacc4\uc801\uc73c\ub85c \ud070 \ud654\uc81c\uac00 \ub418\uc5c8\ub294\ub370, \uc774 \uc0dd\uace0\uae30 \ub4dc\ub808\uc2a4\ub294 \ub274\uc695\ud0c0\uc784\uc2a4\uac00 \uc120\uc815\ud55c 2010\ub144 \uc62c\ud574\uc758 \uc544\uc774\ub514\uc5b4\ub85c \uc120\uc815\ub418\uc5c8\uace0, \ubbf8\uad6d \ub85c\ud070\ub864 \uba85\uc608\uc758 \uc804\ub2f9\uc5d0 \ubc15\uc81c\ub97c \uac70\uccd0 \uac00\uac00\uc758 \uc5b4\ub9b0 \uc2dc\uc808 \ud53c\uc544\ub178\uc640 \ud568\uaed8 \"Women Who Rock: Vision, Passion, Power\"\ub77c\ub294 \uc774\ub984\uc73c\ub85c \ubcf4\uc874\ud558\uace0\uc788\ub2e4.", "sentence_answer": "\ub780\uc81c\ub9ac \uc804\ubb38\uac00 \uc544\ub124\ud2b8 \uc6cc\ubc84\ud2bc \uc5d0", "paragraph_id": "6513681-26-1", "paragraph_question": "question: 16\uc138 \uc774\ud558\uc758 \uc5ec\uc131 \uccad\uc18c\ub144\ub4e4\uc774 \uac80\uc740 \uc18d\uc637\uc744 \uad6c\uc785\ud558\uc5ec \ud56b \ud32c\uce20\ucc98\ub7fc \uc785\ub294 \uac83\uc740 \uac00\uac00\uc758 \uc2a4\ud0c0\uc77c\uc744 \ubcf8\ub72c \uac83\uc774\ub77c\uace0 \ub9d0\ud55c \uc774\ub294?, context: \uadf8\ub140\uc758 \uc774\ubbf8\uc9c0\uc640 \ud328\uc158 \uac10\uac01\uc740 \ud328\ub9ac\uc2a4 \ud790\ud2bc\uacfc \ub2c8\ucf5c \ub9ac\uce58\uc640 \uac19\uc740 \uba85\uc0ac\ub4e4\uc5d0\uac8c \ub9ce\uc740 \uc601\ud5a5\uc744 \uc8fc\uc5c8\ub2e4. \ucd5c\uadfc\uc758 \uc5f0\uad6c\uc5d0\uc11c\ub294 \uac00\uac00\uc758 \uc601\ud5a5\uc73c\ub85c \uc601\uad6d\uc758 \ub2c8\ucee4\uc2a4 \uc18c\ube44\uac00 25% \uc99d\uac00\ud558\uc600\ub2e4\ub294 \uacb0\uacfc\uac00 \ubc1c\ud45c\ub418\uc5c8\ub2e4. \ub780\uc81c\ub9ac \uc804\ubb38\uac00 \uc544\ub124\ud2b8 \uc6cc\ubc84\ud2bc\uc5d0 \ub530\ub974\uba74, 16\uc138 \uc774\ud558\uc758 \uc5ec\uc131 \uccad\uc18c\ub144\ub4e4\uc774 \uac80\uc740 \uc18d\uc637\uc744 \uad6c\uc785\ud558\uc5ec \ud56b \ud32c\uce20\ucc98\ub7fc \uc785\ub294 \uac83\uc740 \uac00\uac00\uc758 \uc2a4\ud0c0\uc77c\uc744 \ubcf8\ub72c \uac83\uc774\ub77c\uace0 \ud55c\ub2e4. \ud55c\ud3b8, 2010\ub144 9\uc6d4 \uc5f4\ub9b0 2010 MTV \ube44\ub514\uc624 \ubba4\uc9c1 \uc5b4\uc6cc\ub4dc\uc5d0\uc11c \uc0dd\uace0\uae30 \ub4dc\ub808\uc2a4\ub97c \uc785\uace0 \ub098\uc640 \uc138\uacc4\uc801\uc73c\ub85c \ud070 \ud654\uc81c\uac00 \ub418\uc5c8\ub294\ub370, \uc774 \uc0dd\uace0\uae30 \ub4dc\ub808\uc2a4\ub294 \ub274\uc695\ud0c0\uc784\uc2a4\uac00 \uc120\uc815\ud55c 2010\ub144 \uc62c\ud574\uc758 \uc544\uc774\ub514\uc5b4\ub85c \uc120\uc815\ub418\uc5c8\uace0, \ubbf8\uad6d \ub85c\ud070\ub864 \uba85\uc608\uc758 \uc804\ub2f9\uc5d0 \ubc15\uc81c\ub97c \uac70\uccd0 \uac00\uac00\uc758 \uc5b4\ub9b0 \uc2dc\uc808 \ud53c\uc544\ub178\uc640 \ud568\uaed8 \"Women Who Rock: Vision, Passion, Power\"\ub77c\ub294 \uc774\ub984\uc73c\ub85c \ubcf4\uc874\ud558\uace0\uc788\ub2e4."} {"question": "\uc720\ub85c\ud30c\uc758 \ud45c\uba74\uc744 \ud3c9\uba74\uc774\ub098 \uad6c\ub85c \uacc4\uc0b0\ud558\uc5ec \ubb34\uac8c\ub97c \uc90c\uc5d0 \ub530\ub77c, \ud0c4\uc131\ub825\uc774 \uc0dd\uaca8 \uad74\uace1\uc774 \uc0dd\uaca8\ub098\ub294 \ud604\uc0c1\uc744 \ubd84\uc11d\ud558\ub294 \uac83\uc740?", "paragraph": "\uc587\uc740 \uc5bc\uc74c \uc774\ub860(Thin Ice model)\uc740 \uc720\ub85c\ud30c\uc758 \uc5bc\uc74c \uc9c0\uac01\uc774 \uae30\uaecf\ud574\uc57c \uba87 \ud0ac\ub85c\ubbf8\ud130\ubc16\uc5d0 \ub418\uc9c0 \uc54a\uc744 \uac83\uc774\ub77c\uace0 \uc608\uce21\ud55c\ub2e4. \uadf8\ub7ec\ub098 \ub300\ubd80\ubd84\uc758 \ud589\uc131 \uc9c0\uc9c8\ud559\uc790\ub4e4\uc740 \uc774 \uc774\ub860\uc774 \ubaa9\uc131\uc758 \uc870\uc11d\ub825\uc744 \uc9c0\uac01\uc758 \ub9e8 \uc717\uce35\uc5d0\uc11c\ub9cc \uacc4\uc0b0\ud588\ub2e4\uace0 \uc598\uae30\ud55c\ub2e4. \uc77c\ub840\ub85c \uad74\uace1 \ubd84\uc11d\uc774 \uc788\ub294\ub370, \uc774\ub294 \uc720\ub85c\ud30c\uc758 \ud45c\uba74\uc744 \ud3c9\uba74\uc774\ub098 \uad6c\ub85c \uacc4\uc0b0\ud558\uc5ec \ubb34\uac8c\ub97c \uc90c\uc5d0 \ub530\ub77c, \ud0c4\uc131\ub825\uc774 \uc0dd\uaca8 \uad74\uace1\uc774 \uc0dd\uaca8\ub098\ub294 \ud604\uc0c1\uc744 \ubd84\uc11d\ud558\ub294 \uac83\uc774\ub2e4. \uc587\uc740 \uc5bc\uc74c \uc774\ub860\uc740 \ud45c\uba74\uc5d0\uc11c \uad74\uace1\uc774 \uc77c\uc5b4\ub098\ub294 \ubd80\ubd84\uc774 200 m \uc815\ub3c4\ub85c \uc587\ub2e4\uace0 \uc608\uc0c1\ud55c\ub2e4. \ub9cc\uc57d \uc815\ub9d0 \uc720\ub85c\ud30c\uc758 \uc5bc\uc74c \uc9c0\uac01\uc774 \uba87 \ud0ac\ub85c\ubbf8\ud130\ubc16\uc5d0 \ub418\uc9c0 \uc54a\ub294\ub2e4\uba74, \uc587\uc740 \uc5bc\uc74c \uc774\ub860\uc740 \uc9c0\ud558\uc758 \uc561\uccb4\uac00 \ud45c\uba74\uacfc \uc8fc\uae30\uc801\uc73c\ub85c \uc811\ucd09\ud558\uc5ec \ud63c\ub780 \uc9c0\ud615\uc758 \uc0dd\uc131\uc744 \uc720\ubc1c\ud558\uace0, \ud45c\uba74\uc758 \uc120\ub4e4\uc744 \uc0dd\uaca8\ub098\uac8c \ud560 \uc218 \uc788\ub2e4\uace0 \uc608\uce21\ud55c\ub2e4.", "answer": "\uad74\uace1 \ubd84\uc11d", "sentence": "\uc77c\ub840\ub85c \uad74\uace1 \ubd84\uc11d \uc774 \uc788\ub294\ub370, \uc774\ub294 \uc720\ub85c\ud30c\uc758 \ud45c\uba74\uc744 \ud3c9\uba74\uc774\ub098 \uad6c\ub85c \uacc4\uc0b0\ud558\uc5ec \ubb34\uac8c\ub97c \uc90c\uc5d0 \ub530\ub77c, \ud0c4\uc131\ub825\uc774 \uc0dd\uaca8 \uad74\uace1\uc774 \uc0dd\uaca8\ub098\ub294 \ud604\uc0c1\uc744 \ubd84\uc11d\ud558\ub294 \uac83\uc774\ub2e4.", "paragraph_sentence": "\uc587\uc740 \uc5bc\uc74c \uc774\ub860(Thin Ice model)\uc740 \uc720\ub85c\ud30c\uc758 \uc5bc\uc74c \uc9c0\uac01\uc774 \uae30\uaecf\ud574\uc57c \uba87 \ud0ac\ub85c\ubbf8\ud130\ubc16\uc5d0 \ub418\uc9c0 \uc54a\uc744 \uac83\uc774\ub77c\uace0 \uc608\uce21\ud55c\ub2e4. \uadf8\ub7ec\ub098 \ub300\ubd80\ubd84\uc758 \ud589\uc131 \uc9c0\uc9c8\ud559\uc790\ub4e4\uc740 \uc774 \uc774\ub860\uc774 \ubaa9\uc131\uc758 \uc870\uc11d\ub825\uc744 \uc9c0\uac01\uc758 \ub9e8 \uc717\uce35\uc5d0\uc11c\ub9cc \uacc4\uc0b0\ud588\ub2e4\uace0 \uc598\uae30\ud55c\ub2e4. \uc77c\ub840\ub85c \uad74\uace1 \ubd84\uc11d \uc774 \uc788\ub294\ub370, \uc774\ub294 \uc720\ub85c\ud30c\uc758 \ud45c\uba74\uc744 \ud3c9\uba74\uc774\ub098 \uad6c\ub85c \uacc4\uc0b0\ud558\uc5ec \ubb34\uac8c\ub97c \uc90c\uc5d0 \ub530\ub77c, \ud0c4\uc131\ub825\uc774 \uc0dd\uaca8 \uad74\uace1\uc774 \uc0dd\uaca8\ub098\ub294 \ud604\uc0c1\uc744 \ubd84\uc11d\ud558\ub294 \uac83\uc774\ub2e4. \uc587\uc740 \uc5bc\uc74c \uc774\ub860\uc740 \ud45c\uba74\uc5d0\uc11c \uad74\uace1\uc774 \uc77c\uc5b4\ub098\ub294 \ubd80\ubd84\uc774 200 m \uc815\ub3c4\ub85c \uc587\ub2e4\uace0 \uc608\uc0c1\ud55c\ub2e4. \ub9cc\uc57d \uc815\ub9d0 \uc720\ub85c\ud30c\uc758 \uc5bc\uc74c \uc9c0\uac01\uc774 \uba87 \ud0ac\ub85c\ubbf8\ud130\ubc16\uc5d0 \ub418\uc9c0 \uc54a\ub294\ub2e4\uba74, \uc587\uc740 \uc5bc\uc74c \uc774\ub860\uc740 \uc9c0\ud558\uc758 \uc561\uccb4\uac00 \ud45c\uba74\uacfc \uc8fc\uae30\uc801\uc73c\ub85c \uc811\ucd09\ud558\uc5ec \ud63c\ub780 \uc9c0\ud615\uc758 \uc0dd\uc131\uc744 \uc720\ubc1c\ud558\uace0, \ud45c\uba74\uc758 \uc120\ub4e4\uc744 \uc0dd\uaca8\ub098\uac8c \ud560 \uc218 \uc788\ub2e4\uace0 \uc608\uce21\ud55c\ub2e4.", "paragraph_answer": "\uc587\uc740 \uc5bc\uc74c \uc774\ub860(Thin Ice model)\uc740 \uc720\ub85c\ud30c\uc758 \uc5bc\uc74c \uc9c0\uac01\uc774 \uae30\uaecf\ud574\uc57c \uba87 \ud0ac\ub85c\ubbf8\ud130\ubc16\uc5d0 \ub418\uc9c0 \uc54a\uc744 \uac83\uc774\ub77c\uace0 \uc608\uce21\ud55c\ub2e4. \uadf8\ub7ec\ub098 \ub300\ubd80\ubd84\uc758 \ud589\uc131 \uc9c0\uc9c8\ud559\uc790\ub4e4\uc740 \uc774 \uc774\ub860\uc774 \ubaa9\uc131\uc758 \uc870\uc11d\ub825\uc744 \uc9c0\uac01\uc758 \ub9e8 \uc717\uce35\uc5d0\uc11c\ub9cc \uacc4\uc0b0\ud588\ub2e4\uace0 \uc598\uae30\ud55c\ub2e4. \uc77c\ub840\ub85c \uad74\uace1 \ubd84\uc11d \uc774 \uc788\ub294\ub370, \uc774\ub294 \uc720\ub85c\ud30c\uc758 \ud45c\uba74\uc744 \ud3c9\uba74\uc774\ub098 \uad6c\ub85c \uacc4\uc0b0\ud558\uc5ec \ubb34\uac8c\ub97c \uc90c\uc5d0 \ub530\ub77c, \ud0c4\uc131\ub825\uc774 \uc0dd\uaca8 \uad74\uace1\uc774 \uc0dd\uaca8\ub098\ub294 \ud604\uc0c1\uc744 \ubd84\uc11d\ud558\ub294 \uac83\uc774\ub2e4. \uc587\uc740 \uc5bc\uc74c \uc774\ub860\uc740 \ud45c\uba74\uc5d0\uc11c \uad74\uace1\uc774 \uc77c\uc5b4\ub098\ub294 \ubd80\ubd84\uc774 200 m \uc815\ub3c4\ub85c \uc587\ub2e4\uace0 \uc608\uc0c1\ud55c\ub2e4. \ub9cc\uc57d \uc815\ub9d0 \uc720\ub85c\ud30c\uc758 \uc5bc\uc74c \uc9c0\uac01\uc774 \uba87 \ud0ac\ub85c\ubbf8\ud130\ubc16\uc5d0 \ub418\uc9c0 \uc54a\ub294\ub2e4\uba74, \uc587\uc740 \uc5bc\uc74c \uc774\ub860\uc740 \uc9c0\ud558\uc758 \uc561\uccb4\uac00 \ud45c\uba74\uacfc \uc8fc\uae30\uc801\uc73c\ub85c \uc811\ucd09\ud558\uc5ec \ud63c\ub780 \uc9c0\ud615\uc758 \uc0dd\uc131\uc744 \uc720\ubc1c\ud558\uace0, \ud45c\uba74\uc758 \uc120\ub4e4\uc744 \uc0dd\uaca8\ub098\uac8c \ud560 \uc218 \uc788\ub2e4\uace0 \uc608\uce21\ud55c\ub2e4.", "sentence_answer": "\uc77c\ub840\ub85c \uad74\uace1 \ubd84\uc11d \uc774 \uc788\ub294\ub370, \uc774\ub294 \uc720\ub85c\ud30c\uc758 \ud45c\uba74\uc744 \ud3c9\uba74\uc774\ub098 \uad6c\ub85c \uacc4\uc0b0\ud558\uc5ec \ubb34\uac8c\ub97c \uc90c\uc5d0 \ub530\ub77c, \ud0c4\uc131\ub825\uc774 \uc0dd\uaca8 \uad74\uace1\uc774 \uc0dd\uaca8\ub098\ub294 \ud604\uc0c1\uc744 \ubd84\uc11d\ud558\ub294 \uac83\uc774\ub2e4.", "paragraph_id": "6527070-7-1", "paragraph_question": "question: \uc720\ub85c\ud30c\uc758 \ud45c\uba74\uc744 \ud3c9\uba74\uc774\ub098 \uad6c\ub85c \uacc4\uc0b0\ud558\uc5ec \ubb34\uac8c\ub97c \uc90c\uc5d0 \ub530\ub77c, \ud0c4\uc131\ub825\uc774 \uc0dd\uaca8 \uad74\uace1\uc774 \uc0dd\uaca8\ub098\ub294 \ud604\uc0c1\uc744 \ubd84\uc11d\ud558\ub294 \uac83\uc740?, context: \uc587\uc740 \uc5bc\uc74c \uc774\ub860(Thin Ice model)\uc740 \uc720\ub85c\ud30c\uc758 \uc5bc\uc74c \uc9c0\uac01\uc774 \uae30\uaecf\ud574\uc57c \uba87 \ud0ac\ub85c\ubbf8\ud130\ubc16\uc5d0 \ub418\uc9c0 \uc54a\uc744 \uac83\uc774\ub77c\uace0 \uc608\uce21\ud55c\ub2e4. \uadf8\ub7ec\ub098 \ub300\ubd80\ubd84\uc758 \ud589\uc131 \uc9c0\uc9c8\ud559\uc790\ub4e4\uc740 \uc774 \uc774\ub860\uc774 \ubaa9\uc131\uc758 \uc870\uc11d\ub825\uc744 \uc9c0\uac01\uc758 \ub9e8 \uc717\uce35\uc5d0\uc11c\ub9cc \uacc4\uc0b0\ud588\ub2e4\uace0 \uc598\uae30\ud55c\ub2e4. \uc77c\ub840\ub85c \uad74\uace1 \ubd84\uc11d\uc774 \uc788\ub294\ub370, \uc774\ub294 \uc720\ub85c\ud30c\uc758 \ud45c\uba74\uc744 \ud3c9\uba74\uc774\ub098 \uad6c\ub85c \uacc4\uc0b0\ud558\uc5ec \ubb34\uac8c\ub97c \uc90c\uc5d0 \ub530\ub77c, \ud0c4\uc131\ub825\uc774 \uc0dd\uaca8 \uad74\uace1\uc774 \uc0dd\uaca8\ub098\ub294 \ud604\uc0c1\uc744 \ubd84\uc11d\ud558\ub294 \uac83\uc774\ub2e4. \uc587\uc740 \uc5bc\uc74c \uc774\ub860\uc740 \ud45c\uba74\uc5d0\uc11c \uad74\uace1\uc774 \uc77c\uc5b4\ub098\ub294 \ubd80\ubd84\uc774 200 m \uc815\ub3c4\ub85c \uc587\ub2e4\uace0 \uc608\uc0c1\ud55c\ub2e4. \ub9cc\uc57d \uc815\ub9d0 \uc720\ub85c\ud30c\uc758 \uc5bc\uc74c \uc9c0\uac01\uc774 \uba87 \ud0ac\ub85c\ubbf8\ud130\ubc16\uc5d0 \ub418\uc9c0 \uc54a\ub294\ub2e4\uba74, \uc587\uc740 \uc5bc\uc74c \uc774\ub860\uc740 \uc9c0\ud558\uc758 \uc561\uccb4\uac00 \ud45c\uba74\uacfc \uc8fc\uae30\uc801\uc73c\ub85c \uc811\ucd09\ud558\uc5ec \ud63c\ub780 \uc9c0\ud615\uc758 \uc0dd\uc131\uc744 \uc720\ubc1c\ud558\uace0, \ud45c\uba74\uc758 \uc120\ub4e4\uc744 \uc0dd\uaca8\ub098\uac8c \ud560 \uc218 \uc788\ub2e4\uace0 \uc608\uce21\ud55c\ub2e4."} {"question": "\uc81c\uc774 \uc62c\uc0e8\uc2a4\ud0a4\ub294 \uc5b4\ub290 \ub300\ud559\uc758 \uad50\uc218\uc778\uac00?", "paragraph": "\ud558\ubc84\ub4dc \ub300\ud559\uc758 \uc778\uc2dd \uacfc\ud559\uc790 \uc2a4\ud2f0\ube10 \ud540\ucee4\uc640 \uc804\uc0b0\ud559\uc790 \ubbf8\uce58 \uce74\ud3ec\uc640 \uac19\uc740 \uba87\uba87 \uc800\uba85\ud55c \uc0ac\uc0c1\uac00\ub4e4\uc740 '\ub85c\ubd07\uc774 \uc131\uacf5\uc801\uc73c\ub85c \uc778\uac04\uc744 \ud749\ub0b4\ub0b4\ub294 \uac83'\uc5d0 \ub300\ud574 \ud68c\uc758\uc801\uc778 \uc758\uacac\uc744 \ubcf4\uc600\ub2e4. \uacf5\uc911\ubcf4\uac74\ud559\uc790 \uc81c\uc774 \uc62c\uc0e8\uc2a4\ud0a4 \ubbf8\uad6d \uc77c\ub9ac\ub178\uc774\ub300 \uad50\uc218\ub294\u201c\uc5ed\uc0ac\uc0c1 \uc601\uc0dd\uc744 \ucd94\uad6c\ud588\ub358 \ubaa8\ub4e0 \uc774\ub4e4\uc740 \ub2e4 \uc8fd\uc5c8\uc73c\uba70 \uc218\ub9ce\uc740 \uacfc\ud559\uc790\ub4e4\uc774 \uc608\uc804\ubd80\ud130 \ub178\ud654 \ubc29\uc9c0\uc758 \uafc8\uc774 \uc774\ub904\uc9c4\ub2e4\uace0 \ub9d0\ud574\uc654\ub2e4\u201d\uba70 \u201c\uacfc\ud559\uc758 \ubc1c\ub2ec\ub85c \uc0dd\uba85\uc774 \uc5f0\uc7a5\ub418\ub294 \uac83\uc740 \uc0ac\uc2e4\uc774\uc9c0\ub9cc \ub4dc\ub77c\ub9c8\ud2f1\ud55c \ubcc0\ud654\ub294 \uac70\uc758 \uc5c6\uc744 \uac83\u201d\uc774\ub77c\uace0 \ub9d0\ud55c\ub2e4.[9] \uadf8\ub294 \uacfc\ud559\uc5d0 \uc758\ud574 \uc778\uac04\uc774 \uc601\uc0dd\ud55c\ub2e4\ub294 \ub9d0\uc740 \ub9e4\ub825\uc801\uc73c\ub85c \ub4e4\ub9ac\uc9c0\ub9cc \uc778\uac04 \uc720\uc804\uc790\ub97c \ud391\ud06c\ub09c \uc790\ub3d9\ucc28 \ud0c0\uc774\uc5b4\ub97c \uac08\uc544 \ub07c\uc6b0\ub4ef \ud560 \uc218 \uc5c6\ub2e4\uace0 \uc124\uba85\ud558\uba74\uc11c \uacfc\ud559\uc744 \ub9cc\ub2a5\uc73c\ub85c \ubc14\ub77c\ubcf4\uace0, \uacfc\ud559\uc774 \uc778\uac04\uc744 \uc816\uacfc \uafc0\uc774 \ud750\ub974\ub294 \ub545\uc73c\ub85c \ub370\ub824\uac08 \uac83\uc73c\ub85c \uae30\ub300\ud55c\ub2e4\uba74 \uc774\uac83\uc740 \uacfc\ud559\uc8fc\uc758\ub77c\ub294 \ub610 \ud558\ub098\uc758 \uc6b0\uc0c1\uc774 \ub418\uace0 \ub9d0 \uac83\uc774\ub77c \uc8fc\uc7a5\ud588\ub2e4.", "answer": "\uc77c\ub9ac\ub178\uc774\ub300", "sentence": "\uacf5\uc911\ubcf4\uac74\ud559\uc790 \uc81c\uc774 \uc62c\uc0e8\uc2a4\ud0a4 \ubbf8\uad6d \uc77c\ub9ac\ub178\uc774\ub300 \uad50\uc218\ub294\u201c\uc5ed\uc0ac\uc0c1 \uc601\uc0dd\uc744 \ucd94\uad6c\ud588\ub358 \ubaa8\ub4e0 \uc774\ub4e4\uc740 \ub2e4 \uc8fd\uc5c8\uc73c\uba70 \uc218\ub9ce\uc740 \uacfc\ud559\uc790\ub4e4\uc774 \uc608\uc804\ubd80\ud130 \ub178\ud654 \ubc29\uc9c0\uc758 \uafc8\uc774 \uc774\ub904\uc9c4\ub2e4\uace0 \ub9d0\ud574\uc654\ub2e4\u201d\uba70 \u201c\uacfc\ud559\uc758 \ubc1c\ub2ec\ub85c \uc0dd\uba85\uc774 \uc5f0\uc7a5\ub418\ub294 \uac83\uc740 \uc0ac\uc2e4\uc774\uc9c0\ub9cc \ub4dc\ub77c\ub9c8\ud2f1\ud55c \ubcc0\ud654\ub294 \uac70\uc758 \uc5c6\uc744 \uac83\u201d\uc774\ub77c\uace0 \ub9d0\ud55c\ub2e4.[9]", "paragraph_sentence": "\ud558\ubc84\ub4dc \ub300\ud559\uc758 \uc778\uc2dd \uacfc\ud559\uc790 \uc2a4\ud2f0\ube10 \ud540\ucee4\uc640 \uc804\uc0b0\ud559\uc790 \ubbf8\uce58 \uce74\ud3ec\uc640 \uac19\uc740 \uba87\uba87 \uc800\uba85\ud55c \uc0ac\uc0c1\uac00\ub4e4\uc740 '\ub85c\ubd07\uc774 \uc131\uacf5\uc801\uc73c\ub85c \uc778\uac04\uc744 \ud749\ub0b4\ub0b4\ub294 \uac83'\uc5d0 \ub300\ud574 \ud68c\uc758\uc801\uc778 \uc758\uacac\uc744 \ubcf4\uc600\ub2e4. \uacf5\uc911\ubcf4\uac74\ud559\uc790 \uc81c\uc774 \uc62c\uc0e8\uc2a4\ud0a4 \ubbf8\uad6d \uc77c\ub9ac\ub178\uc774\ub300 \uad50\uc218\ub294\u201c\uc5ed\uc0ac\uc0c1 \uc601\uc0dd\uc744 \ucd94\uad6c\ud588\ub358 \ubaa8\ub4e0 \uc774\ub4e4\uc740 \ub2e4 \uc8fd\uc5c8\uc73c\uba70 \uc218\ub9ce\uc740 \uacfc\ud559\uc790\ub4e4\uc774 \uc608\uc804\ubd80\ud130 \ub178\ud654 \ubc29\uc9c0\uc758 \uafc8\uc774 \uc774\ub904\uc9c4\ub2e4\uace0 \ub9d0\ud574\uc654\ub2e4\u201d\uba70 \u201c\uacfc\ud559\uc758 \ubc1c\ub2ec\ub85c \uc0dd\uba85\uc774 \uc5f0\uc7a5\ub418\ub294 \uac83\uc740 \uc0ac\uc2e4\uc774\uc9c0\ub9cc \ub4dc\ub77c\ub9c8\ud2f1\ud55c \ubcc0\ud654\ub294 \uac70\uc758 \uc5c6\uc744 \uac83\u201d\uc774\ub77c\uace0 \ub9d0\ud55c\ub2e4.[9] \uadf8\ub294 \uacfc\ud559\uc5d0 \uc758\ud574 \uc778\uac04\uc774 \uc601\uc0dd\ud55c\ub2e4\ub294 \ub9d0\uc740 \ub9e4\ub825\uc801\uc73c\ub85c \ub4e4\ub9ac\uc9c0\ub9cc \uc778\uac04 \uc720\uc804\uc790\ub97c \ud391\ud06c\ub09c \uc790\ub3d9\ucc28 \ud0c0\uc774\uc5b4\ub97c \uac08\uc544 \ub07c\uc6b0\ub4ef \ud560 \uc218 \uc5c6\ub2e4\uace0 \uc124\uba85\ud558\uba74\uc11c \uacfc\ud559\uc744 \ub9cc\ub2a5\uc73c\ub85c \ubc14\ub77c\ubcf4\uace0, \uacfc\ud559\uc774 \uc778\uac04\uc744 \uc816\uacfc \uafc0\uc774 \ud750\ub974\ub294 \ub545\uc73c\ub85c \ub370\ub824\uac08 \uac83\uc73c\ub85c \uae30\ub300\ud55c\ub2e4\uba74 \uc774\uac83\uc740 \uacfc\ud559\uc8fc\uc758\ub77c\ub294 \ub610 \ud558\ub098\uc758 \uc6b0\uc0c1\uc774 \ub418\uace0 \ub9d0 \uac83\uc774\ub77c \uc8fc\uc7a5\ud588\ub2e4.", "paragraph_answer": "\ud558\ubc84\ub4dc \ub300\ud559\uc758 \uc778\uc2dd \uacfc\ud559\uc790 \uc2a4\ud2f0\ube10 \ud540\ucee4\uc640 \uc804\uc0b0\ud559\uc790 \ubbf8\uce58 \uce74\ud3ec\uc640 \uac19\uc740 \uba87\uba87 \uc800\uba85\ud55c \uc0ac\uc0c1\uac00\ub4e4\uc740 '\ub85c\ubd07\uc774 \uc131\uacf5\uc801\uc73c\ub85c \uc778\uac04\uc744 \ud749\ub0b4\ub0b4\ub294 \uac83'\uc5d0 \ub300\ud574 \ud68c\uc758\uc801\uc778 \uc758\uacac\uc744 \ubcf4\uc600\ub2e4. \uacf5\uc911\ubcf4\uac74\ud559\uc790 \uc81c\uc774 \uc62c\uc0e8\uc2a4\ud0a4 \ubbf8\uad6d \uc77c\ub9ac\ub178\uc774\ub300 \uad50\uc218\ub294\u201c\uc5ed\uc0ac\uc0c1 \uc601\uc0dd\uc744 \ucd94\uad6c\ud588\ub358 \ubaa8\ub4e0 \uc774\ub4e4\uc740 \ub2e4 \uc8fd\uc5c8\uc73c\uba70 \uc218\ub9ce\uc740 \uacfc\ud559\uc790\ub4e4\uc774 \uc608\uc804\ubd80\ud130 \ub178\ud654 \ubc29\uc9c0\uc758 \uafc8\uc774 \uc774\ub904\uc9c4\ub2e4\uace0 \ub9d0\ud574\uc654\ub2e4\u201d\uba70 \u201c\uacfc\ud559\uc758 \ubc1c\ub2ec\ub85c \uc0dd\uba85\uc774 \uc5f0\uc7a5\ub418\ub294 \uac83\uc740 \uc0ac\uc2e4\uc774\uc9c0\ub9cc \ub4dc\ub77c\ub9c8\ud2f1\ud55c \ubcc0\ud654\ub294 \uac70\uc758 \uc5c6\uc744 \uac83\u201d\uc774\ub77c\uace0 \ub9d0\ud55c\ub2e4.[9] \uadf8\ub294 \uacfc\ud559\uc5d0 \uc758\ud574 \uc778\uac04\uc774 \uc601\uc0dd\ud55c\ub2e4\ub294 \ub9d0\uc740 \ub9e4\ub825\uc801\uc73c\ub85c \ub4e4\ub9ac\uc9c0\ub9cc \uc778\uac04 \uc720\uc804\uc790\ub97c \ud391\ud06c\ub09c \uc790\ub3d9\ucc28 \ud0c0\uc774\uc5b4\ub97c \uac08\uc544 \ub07c\uc6b0\ub4ef \ud560 \uc218 \uc5c6\ub2e4\uace0 \uc124\uba85\ud558\uba74\uc11c \uacfc\ud559\uc744 \ub9cc\ub2a5\uc73c\ub85c \ubc14\ub77c\ubcf4\uace0, \uacfc\ud559\uc774 \uc778\uac04\uc744 \uc816\uacfc \uafc0\uc774 \ud750\ub974\ub294 \ub545\uc73c\ub85c \ub370\ub824\uac08 \uac83\uc73c\ub85c \uae30\ub300\ud55c\ub2e4\uba74 \uc774\uac83\uc740 \uacfc\ud559\uc8fc\uc758\ub77c\ub294 \ub610 \ud558\ub098\uc758 \uc6b0\uc0c1\uc774 \ub418\uace0 \ub9d0 \uac83\uc774\ub77c \uc8fc\uc7a5\ud588\ub2e4.", "sentence_answer": "\uacf5\uc911\ubcf4\uac74\ud559\uc790 \uc81c\uc774 \uc62c\uc0e8\uc2a4\ud0a4 \ubbf8\uad6d \uc77c\ub9ac\ub178\uc774\ub300 \uad50\uc218\ub294\u201c\uc5ed\uc0ac\uc0c1 \uc601\uc0dd\uc744 \ucd94\uad6c\ud588\ub358 \ubaa8\ub4e0 \uc774\ub4e4\uc740 \ub2e4 \uc8fd\uc5c8\uc73c\uba70 \uc218\ub9ce\uc740 \uacfc\ud559\uc790\ub4e4\uc774 \uc608\uc804\ubd80\ud130 \ub178\ud654 \ubc29\uc9c0\uc758 \uafc8\uc774 \uc774\ub904\uc9c4\ub2e4\uace0 \ub9d0\ud574\uc654\ub2e4\u201d\uba70 \u201c\uacfc\ud559\uc758 \ubc1c\ub2ec\ub85c \uc0dd\uba85\uc774 \uc5f0\uc7a5\ub418\ub294 \uac83\uc740 \uc0ac\uc2e4\uc774\uc9c0\ub9cc \ub4dc\ub77c\ub9c8\ud2f1\ud55c \ubcc0\ud654\ub294 \uac70\uc758 \uc5c6\uc744 \uac83\u201d\uc774\ub77c\uace0 \ub9d0\ud55c\ub2e4.[9]", "paragraph_id": "6480893-7-1", "paragraph_question": "question: \uc81c\uc774 \uc62c\uc0e8\uc2a4\ud0a4\ub294 \uc5b4\ub290 \ub300\ud559\uc758 \uad50\uc218\uc778\uac00?, context: \ud558\ubc84\ub4dc \ub300\ud559\uc758 \uc778\uc2dd \uacfc\ud559\uc790 \uc2a4\ud2f0\ube10 \ud540\ucee4\uc640 \uc804\uc0b0\ud559\uc790 \ubbf8\uce58 \uce74\ud3ec\uc640 \uac19\uc740 \uba87\uba87 \uc800\uba85\ud55c \uc0ac\uc0c1\uac00\ub4e4\uc740 '\ub85c\ubd07\uc774 \uc131\uacf5\uc801\uc73c\ub85c \uc778\uac04\uc744 \ud749\ub0b4\ub0b4\ub294 \uac83'\uc5d0 \ub300\ud574 \ud68c\uc758\uc801\uc778 \uc758\uacac\uc744 \ubcf4\uc600\ub2e4. \uacf5\uc911\ubcf4\uac74\ud559\uc790 \uc81c\uc774 \uc62c\uc0e8\uc2a4\ud0a4 \ubbf8\uad6d \uc77c\ub9ac\ub178\uc774\ub300 \uad50\uc218\ub294\u201c\uc5ed\uc0ac\uc0c1 \uc601\uc0dd\uc744 \ucd94\uad6c\ud588\ub358 \ubaa8\ub4e0 \uc774\ub4e4\uc740 \ub2e4 \uc8fd\uc5c8\uc73c\uba70 \uc218\ub9ce\uc740 \uacfc\ud559\uc790\ub4e4\uc774 \uc608\uc804\ubd80\ud130 \ub178\ud654 \ubc29\uc9c0\uc758 \uafc8\uc774 \uc774\ub904\uc9c4\ub2e4\uace0 \ub9d0\ud574\uc654\ub2e4\u201d\uba70 \u201c\uacfc\ud559\uc758 \ubc1c\ub2ec\ub85c \uc0dd\uba85\uc774 \uc5f0\uc7a5\ub418\ub294 \uac83\uc740 \uc0ac\uc2e4\uc774\uc9c0\ub9cc \ub4dc\ub77c\ub9c8\ud2f1\ud55c \ubcc0\ud654\ub294 \uac70\uc758 \uc5c6\uc744 \uac83\u201d\uc774\ub77c\uace0 \ub9d0\ud55c\ub2e4.[9] \uadf8\ub294 \uacfc\ud559\uc5d0 \uc758\ud574 \uc778\uac04\uc774 \uc601\uc0dd\ud55c\ub2e4\ub294 \ub9d0\uc740 \ub9e4\ub825\uc801\uc73c\ub85c \ub4e4\ub9ac\uc9c0\ub9cc \uc778\uac04 \uc720\uc804\uc790\ub97c \ud391\ud06c\ub09c \uc790\ub3d9\ucc28 \ud0c0\uc774\uc5b4\ub97c \uac08\uc544 \ub07c\uc6b0\ub4ef \ud560 \uc218 \uc5c6\ub2e4\uace0 \uc124\uba85\ud558\uba74\uc11c \uacfc\ud559\uc744 \ub9cc\ub2a5\uc73c\ub85c \ubc14\ub77c\ubcf4\uace0, \uacfc\ud559\uc774 \uc778\uac04\uc744 \uc816\uacfc \uafc0\uc774 \ud750\ub974\ub294 \ub545\uc73c\ub85c \ub370\ub824\uac08 \uac83\uc73c\ub85c \uae30\ub300\ud55c\ub2e4\uba74 \uc774\uac83\uc740 \uacfc\ud559\uc8fc\uc758\ub77c\ub294 \ub610 \ud558\ub098\uc758 \uc6b0\uc0c1\uc774 \ub418\uace0 \ub9d0 \uac83\uc774\ub77c \uc8fc\uc7a5\ud588\ub2e4."} {"question": "\uc218\ud559\uc744 \ud0d0\uad6c\ud588\uc73c\uba70, \ucd5c\ucd08\ub85c \uc9c0\uad6c\uac00 \uc6d0\ud615\uc774\ub77c\ub294 \uac00\uc124\uc744 \uc81c\uc548\ud55c \ud559\ud30c\ub294?", "paragraph": "\uc18c\ud06c\ub77c\ud14c\uc2a4 \uc774\uc804 \ucca0\ud559\uc790 \ub85c \uc54c\ub824\uc9c4 \uace0\ub300 \uadf8\ub9ac\uc2a4\uc758 \ucca0\ud559\uc790\ub4e4\uc740 \uadf8\ub9ac\uc2a4 \uc2e0\ud654\uc5d0 \ub4f1\uc7a5\ud558\ub294 \u201c\uc778\uac04\uc774 \uc0b4\uace0 \uc788\ub294 \uccb4\uacc4\ud654\ub41c \uc6b0\uc8fc\uac00 \uc5b4\ub5bb\uac8c \ud615\uc131\ub418\uc5c8\ub294\uac00\u201d\ub77c\ub294 \ubb3c\uc74c\uc5d0 \ub300\ud55c \ub2f5\uc744 \ucc3e\uc73c\ub824 \ub178\ub825\ud558\uc600\ub2e4. \uc18c\ud06c\ub77c\ud14c\uc2a4 \uc774\uc804 \ucca0\ud559\uc790\uc774\uba74\uc11c \u2018\uacfc\ud559\uc758 \uc544\ubc84\uc9c0\u2019\ub85c \ubd88\ub9ac\ub294 \ud0c8\ub808\uc2a4\ub294 \uc790\uc5f0\ud604\uc0c1\uc758 \uc124\uba85\uc744 \uc704\ud574 \u201c\ubaa8\ub4e0 \uac83\uc740 \ubb3c\uc774\ub2e4.\u201d\uc640 \uac19\uc740 \ucd5c\ucd08\ub85c \ucd08\uc790\uc5f0\uc801\uc774\uc9c0 \uc54a\uc740 \uc124\uba85\uc744 \uc81c\uc2dc\ud558\uc600\ub2e4. \ud0c8\ub808\uc2a4\uc758 \uc81c\uc790\uc778 \ud53c\ud0c0\uace0\ub77c\uc2a4\ub294 \ud53c\ud0c0\uace0\ub77c\uc2a4 \ud559\ud30c\ub97c \ud615\uc131\ud558\uc600\ub2e4. \uc774 \ud559\ud30c\ub294 \uc218\ud559\uc744 \ud0d0\uad6c\ud588\uc73c\uba70, \ucd5c\ucd08\ub85c \uc9c0\uad6c\uac00 \uc6d0\ud615\uc774\ub77c\ub294 \uac00\uc124\uc744 \uc81c\uc548\ud558\uc600\ub2e4. \ub808\uc6b0\ud0a4\ud3ec\uc2a4\ub294 \uc6d0\uc790\uc124\uc744 \uc8fc\uc7a5\ud558\uc600\ub2e4. \uc774 \ud559\uc124\uc740 \uc6d0\uc790\ub77c\ub294 \ub098\ub204\uc5b4\uc9c0\uc9c0 \uc54a\uace0, \uc18c\uba78\ud558\uc9c0 \uc54a\ub294 \ubb3c\uc9c8\uc744 \uac00\uc815\ud558\uace0 \uc788\uc73c\uba70, \ub808\uc6b0\ud0a4\ud3ec\uc2a4\uc758 \uc81c\uc790 \ub370\ubaa8\ud06c\ub9ac\ud1a0\uc2a4\uc5d0 \uc758\ud574 \uc815\uad50\ud654\ub41c\ub2e4.", "answer": "\ud53c\ud0c0\uace0\ub77c\uc2a4 \ud559\ud30c", "sentence": "\ud0c8\ub808\uc2a4\uc758 \uc81c\uc790\uc778 \ud53c\ud0c0\uace0\ub77c\uc2a4\ub294 \ud53c\ud0c0\uace0\ub77c\uc2a4 \ud559\ud30c \ub97c \ud615\uc131\ud558\uc600\ub2e4.", "paragraph_sentence": "\uc18c\ud06c\ub77c\ud14c\uc2a4 \uc774\uc804 \ucca0\ud559\uc790 \ub85c \uc54c\ub824\uc9c4 \uace0\ub300 \uadf8\ub9ac\uc2a4\uc758 \ucca0\ud559\uc790\ub4e4\uc740 \uadf8\ub9ac\uc2a4 \uc2e0\ud654\uc5d0 \ub4f1\uc7a5\ud558\ub294 \u201c\uc778\uac04\uc774 \uc0b4\uace0 \uc788\ub294 \uccb4\uacc4\ud654\ub41c \uc6b0\uc8fc\uac00 \uc5b4\ub5bb\uac8c \ud615\uc131\ub418\uc5c8\ub294\uac00\u201d\ub77c\ub294 \ubb3c\uc74c\uc5d0 \ub300\ud55c \ub2f5\uc744 \ucc3e\uc73c\ub824 \ub178\ub825\ud558\uc600\ub2e4. \uc18c\ud06c\ub77c\ud14c\uc2a4 \uc774\uc804 \ucca0\ud559\uc790\uc774\uba74\uc11c \u2018\uacfc\ud559\uc758 \uc544\ubc84\uc9c0\u2019\ub85c \ubd88\ub9ac\ub294 \ud0c8\ub808\uc2a4\ub294 \uc790\uc5f0\ud604\uc0c1\uc758 \uc124\uba85\uc744 \uc704\ud574 \u201c\ubaa8\ub4e0 \uac83\uc740 \ubb3c\uc774\ub2e4. \u201d\uc640 \uac19\uc740 \ucd5c\ucd08\ub85c \ucd08\uc790\uc5f0\uc801\uc774\uc9c0 \uc54a\uc740 \uc124\uba85\uc744 \uc81c\uc2dc\ud558\uc600\ub2e4. \ud0c8\ub808\uc2a4\uc758 \uc81c\uc790\uc778 \ud53c\ud0c0\uace0\ub77c\uc2a4\ub294 \ud53c\ud0c0\uace0\ub77c\uc2a4 \ud559\ud30c \ub97c \ud615\uc131\ud558\uc600\ub2e4. \uc774 \ud559\ud30c\ub294 \uc218\ud559\uc744 \ud0d0\uad6c\ud588\uc73c\uba70, \ucd5c\ucd08\ub85c \uc9c0\uad6c\uac00 \uc6d0\ud615\uc774\ub77c\ub294 \uac00\uc124\uc744 \uc81c\uc548\ud558\uc600\ub2e4. \ub808\uc6b0\ud0a4\ud3ec\uc2a4\ub294 \uc6d0\uc790\uc124\uc744 \uc8fc\uc7a5\ud558\uc600\ub2e4. \uc774 \ud559\uc124\uc740 \uc6d0\uc790\ub77c\ub294 \ub098\ub204\uc5b4\uc9c0\uc9c0 \uc54a\uace0, \uc18c\uba78\ud558\uc9c0 \uc54a\ub294 \ubb3c\uc9c8\uc744 \uac00\uc815\ud558\uace0 \uc788\uc73c\uba70, \ub808\uc6b0\ud0a4\ud3ec\uc2a4\uc758 \uc81c\uc790 \ub370\ubaa8\ud06c\ub9ac\ud1a0\uc2a4\uc5d0 \uc758\ud574 \uc815\uad50\ud654\ub41c\ub2e4.", "paragraph_answer": "\uc18c\ud06c\ub77c\ud14c\uc2a4 \uc774\uc804 \ucca0\ud559\uc790 \ub85c \uc54c\ub824\uc9c4 \uace0\ub300 \uadf8\ub9ac\uc2a4\uc758 \ucca0\ud559\uc790\ub4e4\uc740 \uadf8\ub9ac\uc2a4 \uc2e0\ud654\uc5d0 \ub4f1\uc7a5\ud558\ub294 \u201c\uc778\uac04\uc774 \uc0b4\uace0 \uc788\ub294 \uccb4\uacc4\ud654\ub41c \uc6b0\uc8fc\uac00 \uc5b4\ub5bb\uac8c \ud615\uc131\ub418\uc5c8\ub294\uac00\u201d\ub77c\ub294 \ubb3c\uc74c\uc5d0 \ub300\ud55c \ub2f5\uc744 \ucc3e\uc73c\ub824 \ub178\ub825\ud558\uc600\ub2e4. \uc18c\ud06c\ub77c\ud14c\uc2a4 \uc774\uc804 \ucca0\ud559\uc790\uc774\uba74\uc11c \u2018\uacfc\ud559\uc758 \uc544\ubc84\uc9c0\u2019\ub85c \ubd88\ub9ac\ub294 \ud0c8\ub808\uc2a4\ub294 \uc790\uc5f0\ud604\uc0c1\uc758 \uc124\uba85\uc744 \uc704\ud574 \u201c\ubaa8\ub4e0 \uac83\uc740 \ubb3c\uc774\ub2e4.\u201d\uc640 \uac19\uc740 \ucd5c\ucd08\ub85c \ucd08\uc790\uc5f0\uc801\uc774\uc9c0 \uc54a\uc740 \uc124\uba85\uc744 \uc81c\uc2dc\ud558\uc600\ub2e4. \ud0c8\ub808\uc2a4\uc758 \uc81c\uc790\uc778 \ud53c\ud0c0\uace0\ub77c\uc2a4\ub294 \ud53c\ud0c0\uace0\ub77c\uc2a4 \ud559\ud30c \ub97c \ud615\uc131\ud558\uc600\ub2e4. \uc774 \ud559\ud30c\ub294 \uc218\ud559\uc744 \ud0d0\uad6c\ud588\uc73c\uba70, \ucd5c\ucd08\ub85c \uc9c0\uad6c\uac00 \uc6d0\ud615\uc774\ub77c\ub294 \uac00\uc124\uc744 \uc81c\uc548\ud558\uc600\ub2e4. \ub808\uc6b0\ud0a4\ud3ec\uc2a4\ub294 \uc6d0\uc790\uc124\uc744 \uc8fc\uc7a5\ud558\uc600\ub2e4. \uc774 \ud559\uc124\uc740 \uc6d0\uc790\ub77c\ub294 \ub098\ub204\uc5b4\uc9c0\uc9c0 \uc54a\uace0, \uc18c\uba78\ud558\uc9c0 \uc54a\ub294 \ubb3c\uc9c8\uc744 \uac00\uc815\ud558\uace0 \uc788\uc73c\uba70, \ub808\uc6b0\ud0a4\ud3ec\uc2a4\uc758 \uc81c\uc790 \ub370\ubaa8\ud06c\ub9ac\ud1a0\uc2a4\uc5d0 \uc758\ud574 \uc815\uad50\ud654\ub41c\ub2e4.", "sentence_answer": "\ud0c8\ub808\uc2a4\uc758 \uc81c\uc790\uc778 \ud53c\ud0c0\uace0\ub77c\uc2a4\ub294 \ud53c\ud0c0\uace0\ub77c\uc2a4 \ud559\ud30c \ub97c \ud615\uc131\ud558\uc600\ub2e4.", "paragraph_id": "6489929-3-1", "paragraph_question": "question: \uc218\ud559\uc744 \ud0d0\uad6c\ud588\uc73c\uba70, \ucd5c\ucd08\ub85c \uc9c0\uad6c\uac00 \uc6d0\ud615\uc774\ub77c\ub294 \uac00\uc124\uc744 \uc81c\uc548\ud55c \ud559\ud30c\ub294?, context: \uc18c\ud06c\ub77c\ud14c\uc2a4 \uc774\uc804 \ucca0\ud559\uc790 \ub85c \uc54c\ub824\uc9c4 \uace0\ub300 \uadf8\ub9ac\uc2a4\uc758 \ucca0\ud559\uc790\ub4e4\uc740 \uadf8\ub9ac\uc2a4 \uc2e0\ud654\uc5d0 \ub4f1\uc7a5\ud558\ub294 \u201c\uc778\uac04\uc774 \uc0b4\uace0 \uc788\ub294 \uccb4\uacc4\ud654\ub41c \uc6b0\uc8fc\uac00 \uc5b4\ub5bb\uac8c \ud615\uc131\ub418\uc5c8\ub294\uac00\u201d\ub77c\ub294 \ubb3c\uc74c\uc5d0 \ub300\ud55c \ub2f5\uc744 \ucc3e\uc73c\ub824 \ub178\ub825\ud558\uc600\ub2e4. \uc18c\ud06c\ub77c\ud14c\uc2a4 \uc774\uc804 \ucca0\ud559\uc790\uc774\uba74\uc11c \u2018\uacfc\ud559\uc758 \uc544\ubc84\uc9c0\u2019\ub85c \ubd88\ub9ac\ub294 \ud0c8\ub808\uc2a4\ub294 \uc790\uc5f0\ud604\uc0c1\uc758 \uc124\uba85\uc744 \uc704\ud574 \u201c\ubaa8\ub4e0 \uac83\uc740 \ubb3c\uc774\ub2e4.\u201d\uc640 \uac19\uc740 \ucd5c\ucd08\ub85c \ucd08\uc790\uc5f0\uc801\uc774\uc9c0 \uc54a\uc740 \uc124\uba85\uc744 \uc81c\uc2dc\ud558\uc600\ub2e4. \ud0c8\ub808\uc2a4\uc758 \uc81c\uc790\uc778 \ud53c\ud0c0\uace0\ub77c\uc2a4\ub294 \ud53c\ud0c0\uace0\ub77c\uc2a4 \ud559\ud30c\ub97c \ud615\uc131\ud558\uc600\ub2e4. \uc774 \ud559\ud30c\ub294 \uc218\ud559\uc744 \ud0d0\uad6c\ud588\uc73c\uba70, \ucd5c\ucd08\ub85c \uc9c0\uad6c\uac00 \uc6d0\ud615\uc774\ub77c\ub294 \uac00\uc124\uc744 \uc81c\uc548\ud558\uc600\ub2e4. \ub808\uc6b0\ud0a4\ud3ec\uc2a4\ub294 \uc6d0\uc790\uc124\uc744 \uc8fc\uc7a5\ud558\uc600\ub2e4. \uc774 \ud559\uc124\uc740 \uc6d0\uc790\ub77c\ub294 \ub098\ub204\uc5b4\uc9c0\uc9c0 \uc54a\uace0, \uc18c\uba78\ud558\uc9c0 \uc54a\ub294 \ubb3c\uc9c8\uc744 \uac00\uc815\ud558\uace0 \uc788\uc73c\uba70, \ub808\uc6b0\ud0a4\ud3ec\uc2a4\uc758 \uc81c\uc790 \ub370\ubaa8\ud06c\ub9ac\ud1a0\uc2a4\uc5d0 \uc758\ud574 \uc815\uad50\ud654\ub41c\ub2e4."} {"question": "\uc7a5\uc6d0\uc0bc\uc740 \ub6f0\uc5b4\ub09c \uae30\uad50\ud30c \ud22c\uc218\ub85c \uad6d\uac00\ub300\ud45c\ub85c\ub3c4 \ub6f0\uc5c8\ub294\ub370 2008\ub144 \uc62c\ub9bc\ud53d \ub2f9\uc2dc\uc5d0 \ucf5c\ub4dc \uac8c\uc784 \uc644\ubd09\uc2b9\uc744 \uac70\ub454 \uacbd\uae30\ub2e4 \uc788\ub2e4. \uc774\ub54c \uc0c1\ub300 \ub098\ub77c\ub294 \uc5b4\ub290 \ub098\ub77c\uc600\ub098", "paragraph": "2008\ub144 11\uc6d4 14\uc77c, \ud788\uc5b4\ub85c\uc988\uc640 \uc0bc\uc131 \ub77c\uc774\uc628\uc988 \ub294 \ub300\ud55c\ubbfc\uad6d \uc57c\uad6c\uacc4\ub97c \uae5c\uc9dd \ub180\ub798\ud0a8 \ub300\ud615 \ud2b8\ub808\uc774\ub4dc\ub97c \uc131\uc0ac\uc2dc\ucf30\ub2e4. \ud2b8\ub808\uc774\ub4dc \ub0b4\uc6a9\uc740 \ud788\uc5b4\ub85c\uc988\uac00 \ud300\uc758 \uc88c\uc644 \uc5d0\uc774\uc2a4\uc778 \uc7a5\uc6d0\uc0bc\uc744 \uc0bc\uc131\uc5d0 \ub0b4\uc8fc\ub294 \ub300\uc2e0, \uc0bc\uc131\uc740 \uc88c\uc644 \ud22c\uc218 \ubc15\uc131\ud6c8\uacfc \uc774\uc801\ub8cc 30\uc5b5 \uc6d0\uc744 \ud788\uc5b4\ub85c\uc988\uc5d0 \uac74\ub0b4\ub294 \uac83\uc774\uc5c8\ub2e4. \uc7a5\uc6d0\uc0bc\uc740 \ud604\ub300 \uc720\ub2c8\ucf58\uc2a4\uc5d0\uc11c 2006\ub144\uc5d0 \ub370\ubdd4\ud55c \uc88c\uc644 \uae30\uad50\ud30c \ud22c\uc218\ub85c \ub370\ubdd4 \uc2dc\uc98c\uc5d0 12\uc2b9\uc5d0 \ud3c9\uade0\uc790\ucc45\uc810 2.85\ub97c \uae30\ub85d\ud558\uba70 \ud654\ub824\ud558\uac8c \ub370\ubdd4\ud558\uc600\uace0, \ub610\ud55c 2006\ub144 \uc544\uc2dc\uc548 \uac8c\uc784\uacfc 2008\ub144 \ud558\uacc4 \uc62c\ub9bc\ud53d\uc5d0\ub3c4 \ub300\ud55c\ubbfc\uad6d \uc57c\uad6c \uad6d\uac00\ub300\ud45c\ud300\uc5d0 \ubc1c\ud0c1\ub418\uc5b4 \ub124\ub35c\ub780\ub4dc \uc804\uc5d0\uc11c 8\uc774\ub2dd \ub3d9\uc548 \ubb34\uc2e4\uc810\uc73c\ub85c \ucf5c\ub4dc \uac8c\uc784 \uc644\ubd09\uc2b9\uc744 \uac70\ub450\ub294 \ub4f1\uc758 \ud65c\uc57d\uc744 \ubcf4\uc600\uace0, \ub610\ud55c \uc62c\ub9bc\ud53d \uae08\uba54\ub2ec\ub85c \ubcd1\uc5ed \ud61c\ud0dd\ub3c4 \ubc1b\uc740 \uc5d0\uc774\uc2a4 \ud22c\uc218\uc600\ub2e4.", "answer": "\ub124\ub35c\ub780\ub4dc", "sentence": "\uc7a5\uc6d0\uc0bc\uc740 \ud604\ub300 \uc720\ub2c8\ucf58\uc2a4\uc5d0\uc11c 2006\ub144\uc5d0 \ub370\ubdd4\ud55c \uc88c\uc644 \uae30\uad50\ud30c \ud22c\uc218\ub85c \ub370\ubdd4 \uc2dc\uc98c\uc5d0 12\uc2b9\uc5d0 \ud3c9\uade0\uc790\ucc45\uc810 2.85\ub97c \uae30\ub85d\ud558\uba70 \ud654\ub824\ud558\uac8c \ub370\ubdd4\ud558\uc600\uace0, \ub610\ud55c 2006\ub144 \uc544\uc2dc\uc548 \uac8c\uc784\uacfc 2008\ub144 \ud558\uacc4 \uc62c\ub9bc\ud53d\uc5d0\ub3c4 \ub300\ud55c\ubbfc\uad6d \uc57c\uad6c \uad6d\uac00\ub300\ud45c\ud300\uc5d0 \ubc1c\ud0c1\ub418\uc5b4 \ub124\ub35c\ub780\ub4dc \uc804\uc5d0\uc11c 8\uc774\ub2dd \ub3d9\uc548 \ubb34\uc2e4\uc810\uc73c\ub85c \ucf5c\ub4dc \uac8c\uc784 \uc644\ubd09\uc2b9\uc744 \uac70\ub450\ub294 \ub4f1\uc758 \ud65c\uc57d\uc744 \ubcf4\uc600\uace0, \ub610\ud55c \uc62c\ub9bc\ud53d \uae08\uba54\ub2ec\ub85c \ubcd1\uc5ed \ud61c\ud0dd\ub3c4 \ubc1b\uc740 \uc5d0\uc774\uc2a4 \ud22c\uc218\uc600\ub2e4.", "paragraph_sentence": "2008\ub144 11\uc6d4 14\uc77c, \ud788\uc5b4\ub85c\uc988\uc640 \uc0bc\uc131 \ub77c\uc774\uc628\uc988 \ub294 \ub300\ud55c\ubbfc\uad6d \uc57c\uad6c\uacc4\ub97c \uae5c\uc9dd \ub180\ub798\ud0a8 \ub300\ud615 \ud2b8\ub808\uc774\ub4dc\ub97c \uc131\uc0ac\uc2dc\ucf30\ub2e4. \ud2b8\ub808\uc774\ub4dc \ub0b4\uc6a9\uc740 \ud788\uc5b4\ub85c\uc988\uac00 \ud300\uc758 \uc88c\uc644 \uc5d0\uc774\uc2a4\uc778 \uc7a5\uc6d0\uc0bc\uc744 \uc0bc\uc131\uc5d0 \ub0b4\uc8fc\ub294 \ub300\uc2e0, \uc0bc\uc131\uc740 \uc88c\uc644 \ud22c\uc218 \ubc15\uc131\ud6c8\uacfc \uc774\uc801\ub8cc 30\uc5b5 \uc6d0\uc744 \ud788\uc5b4\ub85c\uc988\uc5d0 \uac74\ub0b4\ub294 \uac83\uc774\uc5c8\ub2e4. \uc7a5\uc6d0\uc0bc\uc740 \ud604\ub300 \uc720\ub2c8\ucf58\uc2a4\uc5d0\uc11c 2006\ub144\uc5d0 \ub370\ubdd4\ud55c \uc88c\uc644 \uae30\uad50\ud30c \ud22c\uc218\ub85c \ub370\ubdd4 \uc2dc\uc98c\uc5d0 12\uc2b9\uc5d0 \ud3c9\uade0\uc790\ucc45\uc810 2.85\ub97c \uae30\ub85d\ud558\uba70 \ud654\ub824\ud558\uac8c \ub370\ubdd4\ud558\uc600\uace0, \ub610\ud55c 2006\ub144 \uc544\uc2dc\uc548 \uac8c\uc784\uacfc 2008\ub144 \ud558\uacc4 \uc62c\ub9bc\ud53d\uc5d0\ub3c4 \ub300\ud55c\ubbfc\uad6d \uc57c\uad6c \uad6d\uac00\ub300\ud45c\ud300\uc5d0 \ubc1c\ud0c1\ub418\uc5b4 \ub124\ub35c\ub780\ub4dc \uc804\uc5d0\uc11c 8\uc774\ub2dd \ub3d9\uc548 \ubb34\uc2e4\uc810\uc73c\ub85c \ucf5c\ub4dc \uac8c\uc784 \uc644\ubd09\uc2b9\uc744 \uac70\ub450\ub294 \ub4f1\uc758 \ud65c\uc57d\uc744 \ubcf4\uc600\uace0, \ub610\ud55c \uc62c\ub9bc\ud53d \uae08\uba54\ub2ec\ub85c \ubcd1\uc5ed \ud61c\ud0dd\ub3c4 \ubc1b\uc740 \uc5d0\uc774\uc2a4 \ud22c\uc218\uc600\ub2e4. ", "paragraph_answer": "2008\ub144 11\uc6d4 14\uc77c, \ud788\uc5b4\ub85c\uc988\uc640 \uc0bc\uc131 \ub77c\uc774\uc628\uc988 \ub294 \ub300\ud55c\ubbfc\uad6d \uc57c\uad6c\uacc4\ub97c \uae5c\uc9dd \ub180\ub798\ud0a8 \ub300\ud615 \ud2b8\ub808\uc774\ub4dc\ub97c \uc131\uc0ac\uc2dc\ucf30\ub2e4. \ud2b8\ub808\uc774\ub4dc \ub0b4\uc6a9\uc740 \ud788\uc5b4\ub85c\uc988\uac00 \ud300\uc758 \uc88c\uc644 \uc5d0\uc774\uc2a4\uc778 \uc7a5\uc6d0\uc0bc\uc744 \uc0bc\uc131\uc5d0 \ub0b4\uc8fc\ub294 \ub300\uc2e0, \uc0bc\uc131\uc740 \uc88c\uc644 \ud22c\uc218 \ubc15\uc131\ud6c8\uacfc \uc774\uc801\ub8cc 30\uc5b5 \uc6d0\uc744 \ud788\uc5b4\ub85c\uc988\uc5d0 \uac74\ub0b4\ub294 \uac83\uc774\uc5c8\ub2e4. \uc7a5\uc6d0\uc0bc\uc740 \ud604\ub300 \uc720\ub2c8\ucf58\uc2a4\uc5d0\uc11c 2006\ub144\uc5d0 \ub370\ubdd4\ud55c \uc88c\uc644 \uae30\uad50\ud30c \ud22c\uc218\ub85c \ub370\ubdd4 \uc2dc\uc98c\uc5d0 12\uc2b9\uc5d0 \ud3c9\uade0\uc790\ucc45\uc810 2.85\ub97c \uae30\ub85d\ud558\uba70 \ud654\ub824\ud558\uac8c \ub370\ubdd4\ud558\uc600\uace0, \ub610\ud55c 2006\ub144 \uc544\uc2dc\uc548 \uac8c\uc784\uacfc 2008\ub144 \ud558\uacc4 \uc62c\ub9bc\ud53d\uc5d0\ub3c4 \ub300\ud55c\ubbfc\uad6d \uc57c\uad6c \uad6d\uac00\ub300\ud45c\ud300\uc5d0 \ubc1c\ud0c1\ub418\uc5b4 \ub124\ub35c\ub780\ub4dc \uc804\uc5d0\uc11c 8\uc774\ub2dd \ub3d9\uc548 \ubb34\uc2e4\uc810\uc73c\ub85c \ucf5c\ub4dc \uac8c\uc784 \uc644\ubd09\uc2b9\uc744 \uac70\ub450\ub294 \ub4f1\uc758 \ud65c\uc57d\uc744 \ubcf4\uc600\uace0, \ub610\ud55c \uc62c\ub9bc\ud53d \uae08\uba54\ub2ec\ub85c \ubcd1\uc5ed \ud61c\ud0dd\ub3c4 \ubc1b\uc740 \uc5d0\uc774\uc2a4 \ud22c\uc218\uc600\ub2e4.", "sentence_answer": "\uc7a5\uc6d0\uc0bc\uc740 \ud604\ub300 \uc720\ub2c8\ucf58\uc2a4\uc5d0\uc11c 2006\ub144\uc5d0 \ub370\ubdd4\ud55c \uc88c\uc644 \uae30\uad50\ud30c \ud22c\uc218\ub85c \ub370\ubdd4 \uc2dc\uc98c\uc5d0 12\uc2b9\uc5d0 \ud3c9\uade0\uc790\ucc45\uc810 2.85\ub97c \uae30\ub85d\ud558\uba70 \ud654\ub824\ud558\uac8c \ub370\ubdd4\ud558\uc600\uace0, \ub610\ud55c 2006\ub144 \uc544\uc2dc\uc548 \uac8c\uc784\uacfc 2008\ub144 \ud558\uacc4 \uc62c\ub9bc\ud53d\uc5d0\ub3c4 \ub300\ud55c\ubbfc\uad6d \uc57c\uad6c \uad6d\uac00\ub300\ud45c\ud300\uc5d0 \ubc1c\ud0c1\ub418\uc5b4 \ub124\ub35c\ub780\ub4dc \uc804\uc5d0\uc11c 8\uc774\ub2dd \ub3d9\uc548 \ubb34\uc2e4\uc810\uc73c\ub85c \ucf5c\ub4dc \uac8c\uc784 \uc644\ubd09\uc2b9\uc744 \uac70\ub450\ub294 \ub4f1\uc758 \ud65c\uc57d\uc744 \ubcf4\uc600\uace0, \ub610\ud55c \uc62c\ub9bc\ud53d \uae08\uba54\ub2ec\ub85c \ubcd1\uc5ed \ud61c\ud0dd\ub3c4 \ubc1b\uc740 \uc5d0\uc774\uc2a4 \ud22c\uc218\uc600\ub2e4.", "paragraph_id": "6024902-2-1", "paragraph_question": "question: \uc7a5\uc6d0\uc0bc\uc740 \ub6f0\uc5b4\ub09c \uae30\uad50\ud30c \ud22c\uc218\ub85c \uad6d\uac00\ub300\ud45c\ub85c\ub3c4 \ub6f0\uc5c8\ub294\ub370 2008\ub144 \uc62c\ub9bc\ud53d \ub2f9\uc2dc\uc5d0 \ucf5c\ub4dc \uac8c\uc784 \uc644\ubd09\uc2b9\uc744 \uac70\ub454 \uacbd\uae30\ub2e4 \uc788\ub2e4. \uc774\ub54c \uc0c1\ub300 \ub098\ub77c\ub294 \uc5b4\ub290 \ub098\ub77c\uc600\ub098, context: 2008\ub144 11\uc6d4 14\uc77c, \ud788\uc5b4\ub85c\uc988\uc640 \uc0bc\uc131 \ub77c\uc774\uc628\uc988 \ub294 \ub300\ud55c\ubbfc\uad6d \uc57c\uad6c\uacc4\ub97c \uae5c\uc9dd \ub180\ub798\ud0a8 \ub300\ud615 \ud2b8\ub808\uc774\ub4dc\ub97c \uc131\uc0ac\uc2dc\ucf30\ub2e4. \ud2b8\ub808\uc774\ub4dc \ub0b4\uc6a9\uc740 \ud788\uc5b4\ub85c\uc988\uac00 \ud300\uc758 \uc88c\uc644 \uc5d0\uc774\uc2a4\uc778 \uc7a5\uc6d0\uc0bc\uc744 \uc0bc\uc131\uc5d0 \ub0b4\uc8fc\ub294 \ub300\uc2e0, \uc0bc\uc131\uc740 \uc88c\uc644 \ud22c\uc218 \ubc15\uc131\ud6c8\uacfc \uc774\uc801\ub8cc 30\uc5b5 \uc6d0\uc744 \ud788\uc5b4\ub85c\uc988\uc5d0 \uac74\ub0b4\ub294 \uac83\uc774\uc5c8\ub2e4. \uc7a5\uc6d0\uc0bc\uc740 \ud604\ub300 \uc720\ub2c8\ucf58\uc2a4\uc5d0\uc11c 2006\ub144\uc5d0 \ub370\ubdd4\ud55c \uc88c\uc644 \uae30\uad50\ud30c \ud22c\uc218\ub85c \ub370\ubdd4 \uc2dc\uc98c\uc5d0 12\uc2b9\uc5d0 \ud3c9\uade0\uc790\ucc45\uc810 2.85\ub97c \uae30\ub85d\ud558\uba70 \ud654\ub824\ud558\uac8c \ub370\ubdd4\ud558\uc600\uace0, \ub610\ud55c 2006\ub144 \uc544\uc2dc\uc548 \uac8c\uc784\uacfc 2008\ub144 \ud558\uacc4 \uc62c\ub9bc\ud53d\uc5d0\ub3c4 \ub300\ud55c\ubbfc\uad6d \uc57c\uad6c \uad6d\uac00\ub300\ud45c\ud300\uc5d0 \ubc1c\ud0c1\ub418\uc5b4 \ub124\ub35c\ub780\ub4dc \uc804\uc5d0\uc11c 8\uc774\ub2dd \ub3d9\uc548 \ubb34\uc2e4\uc810\uc73c\ub85c \ucf5c\ub4dc \uac8c\uc784 \uc644\ubd09\uc2b9\uc744 \uac70\ub450\ub294 \ub4f1\uc758 \ud65c\uc57d\uc744 \ubcf4\uc600\uace0, \ub610\ud55c \uc62c\ub9bc\ud53d \uae08\uba54\ub2ec\ub85c \ubcd1\uc5ed \ud61c\ud0dd\ub3c4 \ubc1b\uc740 \uc5d0\uc774\uc2a4 \ud22c\uc218\uc600\ub2e4."} {"question": "\uae40\uc2b9\uaddc \uad6d\uc815\uc6d0\uc7a5 \ud6c4\uc784\uc758 \uc774\ub984\uc740?", "paragraph": "\ud55c\ud3b8 \uc218\uc0ac\uacfc\uc815\uc5d0\uc11c \uacf5\uc548\ub2f9\uad6d\uc774 \uc0ac\ubb34\uc2e4\uc744 \uc555\uc218\uc218\uc0c9\ud558\uba74\uc11c \ubbfc\uc8fc\ub178\ub3d9\ub2f9 \ucd5c\uae30\uc601 \uc0ac\ubb34\ubd80\ucd1d\uc7a5\uacfc \uc2dc\ubbfc\ub2e8\uccb4 \uad00\uacc4\uc790 \ub4f1\uc758 \uc774\ub984\uc774 \uc801\ud78c \uba54\ubaa8\ub97c \ubc1c\uacac\ud558\uba74\uc11c \uc0ac\uac74\uc758 \uaddc\ubaa8\uac00 \ud655\ub300\ub418\uae30 \uc2dc\uc791\ud588\uc73c\uba70, \ub2f9\uad6d\uc758 \ud5c8\ub77d \uc5c6\uc774 \ubd81\uc870\uc120\uc744 \uc138 \ucc28\ub840 \ubc29\ubb38\ud55c \uc804\ub825\uc774 \uc788\ub294 \uc7a5\ubbfc\ud638\uac00 \uace0\uc815\uac04\ucca9\uc73c\ub85c \ud65c\ub3d9\ud55c \uac83\uc73c\ub85c \uc758\uc2ec\uc744 \ubc1b\uc73c\uba74\uc11c \uc774 \uc0ac\uac74\uc758 \uac00\uc7a5 \ud575\uc2ec\uc801\uc778 \uc778\ubb3c\ub85c \ub5a0\uc624\ub974\uac8c \ub418\uc5c8\ub2e4. \uacb0\uad6d \uc7a5\ubbfc\ud638\uac00 \ubc29\ubd81 \ub2f9\uc2dc \ucda9\uc131\uc11c\uc57d\uc744 \ud558\uace0 \ubd81\uc870\uc120 \ub178\ub3d9\ub2f9\uc5d0 \uac00\uc785\ud588\ub2e4\ub294 \uc0ac\uc2e4\uc774 \ub4dc\ub7ec\ub0ac\uc73c\uba70, \uac19\uc774 \ud65c\ub3d9\ud55c \ud610\uc758\ub85c \ubbfc\uc8fc\ub178\ub3d9\ub2f9 \uc0ac\ubb34\ubd80\ucd1d\uc7a5 \ucd5c\uae30\uc601 \uacfc \uc7a5\ubbfc\ud638\uc758 \ud68c\uc0ac \uc9c1\uc6d0 \uc774\uc9c4\uac15\uc774 \ucd94\uac00\ub85c \uad6c\uc18d\ub418\uc5c8\ub2e4. \ub610\ud55c \uc774 \uc0ac\uac74\uc73c\ub85c \uc778\ud574 \uae40\uc2b9\uaddc \ub2f9\uc2dc \uad6d\uac00\uc815\ubcf4\uc6d0\uc7a5\uc774 \uc218\uc0ac\uc9c0\uc18d\uc5d0 \ubd80\ub2f4\uc744 \ub290\uaef4 \uad6d\uac00\uc815\ubcf4\uc6d0\uc7a5\uc744 \uc0ac\uc9c1\ud558\uac8c \ub41c\ub2e4.\uadf8 \uc774\ud6c4 \uae40\uc2b9\uaddc \uad6d\uc815\uc6d0\uc7a5\uc740 \uc0ac\uc784\uc758\uc0ac\ub97c \ud45c\uba85\ud558\uace0 \uc790\ub9ac\uc5d0\uc11c \ubb3c\ub7ec\ub098\uac8c \ub418\uc5c8\ub2e4. \uc774 \ud6c4 \uae40\ub9cc\ubcf5 \ucc28\uc7a5\uc774 \uad6d\uc815\uc6d0\uc7a5 \uc790\ub9ac\uc5d0 \uc624\ub974\uac8c \ub41c\ub2e4", "answer": "\uae40\ub9cc\ubcf5", "sentence": "\uc774 \ud6c4 \uae40\ub9cc\ubcf5 \ucc28\uc7a5\uc774 \uad6d\uc815\uc6d0\uc7a5 \uc790\ub9ac\uc5d0 \uc624\ub974\uac8c \ub41c\ub2e4", "paragraph_sentence": "\ud55c\ud3b8 \uc218\uc0ac\uacfc\uc815\uc5d0\uc11c \uacf5\uc548\ub2f9\uad6d\uc774 \uc0ac\ubb34\uc2e4\uc744 \uc555\uc218\uc218\uc0c9\ud558\uba74\uc11c \ubbfc\uc8fc\ub178\ub3d9\ub2f9 \ucd5c\uae30\uc601 \uc0ac\ubb34\ubd80\ucd1d\uc7a5\uacfc \uc2dc\ubbfc\ub2e8\uccb4 \uad00\uacc4\uc790 \ub4f1\uc758 \uc774\ub984\uc774 \uc801\ud78c \uba54\ubaa8\ub97c \ubc1c\uacac\ud558\uba74\uc11c \uc0ac\uac74\uc758 \uaddc\ubaa8\uac00 \ud655\ub300\ub418\uae30 \uc2dc\uc791\ud588\uc73c\uba70, \ub2f9\uad6d\uc758 \ud5c8\ub77d \uc5c6\uc774 \ubd81\uc870\uc120\uc744 \uc138 \ucc28\ub840 \ubc29\ubb38\ud55c \uc804\ub825\uc774 \uc788\ub294 \uc7a5\ubbfc\ud638\uac00 \uace0\uc815\uac04\ucca9\uc73c\ub85c \ud65c\ub3d9\ud55c \uac83\uc73c\ub85c \uc758\uc2ec\uc744 \ubc1b\uc73c\uba74\uc11c \uc774 \uc0ac\uac74\uc758 \uac00\uc7a5 \ud575\uc2ec\uc801\uc778 \uc778\ubb3c\ub85c \ub5a0\uc624\ub974\uac8c \ub418\uc5c8\ub2e4. \uacb0\uad6d \uc7a5\ubbfc\ud638\uac00 \ubc29\ubd81 \ub2f9\uc2dc \ucda9\uc131\uc11c\uc57d\uc744 \ud558\uace0 \ubd81\uc870\uc120 \ub178\ub3d9\ub2f9\uc5d0 \uac00\uc785\ud588\ub2e4\ub294 \uc0ac\uc2e4\uc774 \ub4dc\ub7ec\ub0ac\uc73c\uba70, \uac19\uc774 \ud65c\ub3d9\ud55c \ud610\uc758\ub85c \ubbfc\uc8fc\ub178\ub3d9\ub2f9 \uc0ac\ubb34\ubd80\ucd1d\uc7a5 \ucd5c\uae30\uc601 \uacfc \uc7a5\ubbfc\ud638\uc758 \ud68c\uc0ac \uc9c1\uc6d0 \uc774\uc9c4\uac15\uc774 \ucd94\uac00\ub85c \uad6c\uc18d\ub418\uc5c8\ub2e4. \ub610\ud55c \uc774 \uc0ac\uac74\uc73c\ub85c \uc778\ud574 \uae40\uc2b9\uaddc \ub2f9\uc2dc \uad6d\uac00\uc815\ubcf4\uc6d0\uc7a5\uc774 \uc218\uc0ac\uc9c0\uc18d\uc5d0 \ubd80\ub2f4\uc744 \ub290\uaef4 \uad6d\uac00\uc815\ubcf4\uc6d0\uc7a5\uc744 \uc0ac\uc9c1\ud558\uac8c \ub41c\ub2e4. \uadf8 \uc774\ud6c4 \uae40\uc2b9\uaddc \uad6d\uc815\uc6d0\uc7a5\uc740 \uc0ac\uc784\uc758\uc0ac\ub97c \ud45c\uba85\ud558\uace0 \uc790\ub9ac\uc5d0\uc11c \ubb3c\ub7ec\ub098\uac8c \ub418\uc5c8\ub2e4. \uc774 \ud6c4 \uae40\ub9cc\ubcf5 \ucc28\uc7a5\uc774 \uad6d\uc815\uc6d0\uc7a5 \uc790\ub9ac\uc5d0 \uc624\ub974\uac8c \ub41c\ub2e4 ", "paragraph_answer": "\ud55c\ud3b8 \uc218\uc0ac\uacfc\uc815\uc5d0\uc11c \uacf5\uc548\ub2f9\uad6d\uc774 \uc0ac\ubb34\uc2e4\uc744 \uc555\uc218\uc218\uc0c9\ud558\uba74\uc11c \ubbfc\uc8fc\ub178\ub3d9\ub2f9 \ucd5c\uae30\uc601 \uc0ac\ubb34\ubd80\ucd1d\uc7a5\uacfc \uc2dc\ubbfc\ub2e8\uccb4 \uad00\uacc4\uc790 \ub4f1\uc758 \uc774\ub984\uc774 \uc801\ud78c \uba54\ubaa8\ub97c \ubc1c\uacac\ud558\uba74\uc11c \uc0ac\uac74\uc758 \uaddc\ubaa8\uac00 \ud655\ub300\ub418\uae30 \uc2dc\uc791\ud588\uc73c\uba70, \ub2f9\uad6d\uc758 \ud5c8\ub77d \uc5c6\uc774 \ubd81\uc870\uc120\uc744 \uc138 \ucc28\ub840 \ubc29\ubb38\ud55c \uc804\ub825\uc774 \uc788\ub294 \uc7a5\ubbfc\ud638\uac00 \uace0\uc815\uac04\ucca9\uc73c\ub85c \ud65c\ub3d9\ud55c \uac83\uc73c\ub85c \uc758\uc2ec\uc744 \ubc1b\uc73c\uba74\uc11c \uc774 \uc0ac\uac74\uc758 \uac00\uc7a5 \ud575\uc2ec\uc801\uc778 \uc778\ubb3c\ub85c \ub5a0\uc624\ub974\uac8c \ub418\uc5c8\ub2e4. \uacb0\uad6d \uc7a5\ubbfc\ud638\uac00 \ubc29\ubd81 \ub2f9\uc2dc \ucda9\uc131\uc11c\uc57d\uc744 \ud558\uace0 \ubd81\uc870\uc120 \ub178\ub3d9\ub2f9\uc5d0 \uac00\uc785\ud588\ub2e4\ub294 \uc0ac\uc2e4\uc774 \ub4dc\ub7ec\ub0ac\uc73c\uba70, \uac19\uc774 \ud65c\ub3d9\ud55c \ud610\uc758\ub85c \ubbfc\uc8fc\ub178\ub3d9\ub2f9 \uc0ac\ubb34\ubd80\ucd1d\uc7a5 \ucd5c\uae30\uc601 \uacfc \uc7a5\ubbfc\ud638\uc758 \ud68c\uc0ac \uc9c1\uc6d0 \uc774\uc9c4\uac15\uc774 \ucd94\uac00\ub85c \uad6c\uc18d\ub418\uc5c8\ub2e4. \ub610\ud55c \uc774 \uc0ac\uac74\uc73c\ub85c \uc778\ud574 \uae40\uc2b9\uaddc \ub2f9\uc2dc \uad6d\uac00\uc815\ubcf4\uc6d0\uc7a5\uc774 \uc218\uc0ac\uc9c0\uc18d\uc5d0 \ubd80\ub2f4\uc744 \ub290\uaef4 \uad6d\uac00\uc815\ubcf4\uc6d0\uc7a5\uc744 \uc0ac\uc9c1\ud558\uac8c \ub41c\ub2e4.\uadf8 \uc774\ud6c4 \uae40\uc2b9\uaddc \uad6d\uc815\uc6d0\uc7a5\uc740 \uc0ac\uc784\uc758\uc0ac\ub97c \ud45c\uba85\ud558\uace0 \uc790\ub9ac\uc5d0\uc11c \ubb3c\ub7ec\ub098\uac8c \ub418\uc5c8\ub2e4. \uc774 \ud6c4 \uae40\ub9cc\ubcf5 \ucc28\uc7a5\uc774 \uad6d\uc815\uc6d0\uc7a5 \uc790\ub9ac\uc5d0 \uc624\ub974\uac8c \ub41c\ub2e4", "sentence_answer": "\uc774 \ud6c4 \uae40\ub9cc\ubcf5 \ucc28\uc7a5\uc774 \uad6d\uc815\uc6d0\uc7a5 \uc790\ub9ac\uc5d0 \uc624\ub974\uac8c \ub41c\ub2e4", "paragraph_id": "6548831-2-1", "paragraph_question": "question: \uae40\uc2b9\uaddc \uad6d\uc815\uc6d0\uc7a5 \ud6c4\uc784\uc758 \uc774\ub984\uc740?, context: \ud55c\ud3b8 \uc218\uc0ac\uacfc\uc815\uc5d0\uc11c \uacf5\uc548\ub2f9\uad6d\uc774 \uc0ac\ubb34\uc2e4\uc744 \uc555\uc218\uc218\uc0c9\ud558\uba74\uc11c \ubbfc\uc8fc\ub178\ub3d9\ub2f9 \ucd5c\uae30\uc601 \uc0ac\ubb34\ubd80\ucd1d\uc7a5\uacfc \uc2dc\ubbfc\ub2e8\uccb4 \uad00\uacc4\uc790 \ub4f1\uc758 \uc774\ub984\uc774 \uc801\ud78c \uba54\ubaa8\ub97c \ubc1c\uacac\ud558\uba74\uc11c \uc0ac\uac74\uc758 \uaddc\ubaa8\uac00 \ud655\ub300\ub418\uae30 \uc2dc\uc791\ud588\uc73c\uba70, \ub2f9\uad6d\uc758 \ud5c8\ub77d \uc5c6\uc774 \ubd81\uc870\uc120\uc744 \uc138 \ucc28\ub840 \ubc29\ubb38\ud55c \uc804\ub825\uc774 \uc788\ub294 \uc7a5\ubbfc\ud638\uac00 \uace0\uc815\uac04\ucca9\uc73c\ub85c \ud65c\ub3d9\ud55c \uac83\uc73c\ub85c \uc758\uc2ec\uc744 \ubc1b\uc73c\uba74\uc11c \uc774 \uc0ac\uac74\uc758 \uac00\uc7a5 \ud575\uc2ec\uc801\uc778 \uc778\ubb3c\ub85c \ub5a0\uc624\ub974\uac8c \ub418\uc5c8\ub2e4. \uacb0\uad6d \uc7a5\ubbfc\ud638\uac00 \ubc29\ubd81 \ub2f9\uc2dc \ucda9\uc131\uc11c\uc57d\uc744 \ud558\uace0 \ubd81\uc870\uc120 \ub178\ub3d9\ub2f9\uc5d0 \uac00\uc785\ud588\ub2e4\ub294 \uc0ac\uc2e4\uc774 \ub4dc\ub7ec\ub0ac\uc73c\uba70, \uac19\uc774 \ud65c\ub3d9\ud55c \ud610\uc758\ub85c \ubbfc\uc8fc\ub178\ub3d9\ub2f9 \uc0ac\ubb34\ubd80\ucd1d\uc7a5 \ucd5c\uae30\uc601 \uacfc \uc7a5\ubbfc\ud638\uc758 \ud68c\uc0ac \uc9c1\uc6d0 \uc774\uc9c4\uac15\uc774 \ucd94\uac00\ub85c \uad6c\uc18d\ub418\uc5c8\ub2e4. \ub610\ud55c \uc774 \uc0ac\uac74\uc73c\ub85c \uc778\ud574 \uae40\uc2b9\uaddc \ub2f9\uc2dc \uad6d\uac00\uc815\ubcf4\uc6d0\uc7a5\uc774 \uc218\uc0ac\uc9c0\uc18d\uc5d0 \ubd80\ub2f4\uc744 \ub290\uaef4 \uad6d\uac00\uc815\ubcf4\uc6d0\uc7a5\uc744 \uc0ac\uc9c1\ud558\uac8c \ub41c\ub2e4.\uadf8 \uc774\ud6c4 \uae40\uc2b9\uaddc \uad6d\uc815\uc6d0\uc7a5\uc740 \uc0ac\uc784\uc758\uc0ac\ub97c \ud45c\uba85\ud558\uace0 \uc790\ub9ac\uc5d0\uc11c \ubb3c\ub7ec\ub098\uac8c \ub418\uc5c8\ub2e4. \uc774 \ud6c4 \uae40\ub9cc\ubcf5 \ucc28\uc7a5\uc774 \uad6d\uc815\uc6d0\uc7a5 \uc790\ub9ac\uc5d0 \uc624\ub974\uac8c \ub41c\ub2e4"} {"question": "\uc6d0\uc815\ud654\uc640 \ub0b4\uc5f0 \uad00\uacc4\ub97c \uc720\uc9c0\ud55c \ub300\uc704\ub294 \uc5b4\ub5a4 \ud310\uacb0\uc744 \ubc1b\uc558\ub294\uac00?", "paragraph": "1974\ub144\uc5d0 \ud568\uacbd\ubd81\ub3c4 \uccad\uc9c4\uc2dc\uc5d0\uc11c \ud0dc\uc5b4\ub0ac\ub2e4. 1988\ub144, \uace0\ubb34\uc0b0\uc5ec\uc790\uace0\ub4f1\uc911\ud559\uad50 4\ud559\ub144 \ub54c\ub294 \ud559\uc5c5 \uc131\uc801\uc774 \uc6b0\uc218\ud558\uc5ec '\uc774\uc911 \uc601\uc608 \ubd89\uc740\uae30 \ud718\uc7a5'\uc744 \ubc1b\uc558\uace0 15\uc138 \ub54c \uae40\uc77c\uc131\uc0ac\ud68c\uc8fc\uc758\uccad\ub144\ub3d9\ub9f9\uc5d0 \ubc1c\ud0c1\ub3fc \uacf5\uc791\uc6d0\uc744 \uc591\uc131\ud558\ub294 \ud559\uad50\uc778 \uae08\uc131\uc815\uce58\uad70\uc0ac\ub300\ud559\uc5d0\uc11c \uad50\uc721\ubc1b\uc558\uc73c\ub098 1992\ub144 \uba38\ub9ac \ubd80\uc0c1\uc73c\ub85c \uc18c\uc18d\ub41c \ud2b9\uc218\ubd80\ub300\uc5d0\uc11c \uc758\ubcd1\uc804\uc5ed\ud588\ub2e4. 1998\ub144\ubd80\ud130 \uad6d\uac00\uc548\uc804\ubcf4\uc704\ubd80\uc5d0 \ud3ec\uc12d\ub418\uc5b4 \uacf5\uc791 \ud65c\ub3d9\uc744 \uc2dc\uc791. \ud558\uc600\ub2e4\uac00, 2008\ub144 7\uc6d4 15\uc77c\uc5d0 \ub300\ud55c\ubbfc\uad6d\uc758 \uad70\uacbd \ud569\ub3d9 \uc0ac\ubc95 \ub2f9\uad6d\uc5d0 \uad6d\uac00\ubcf4\uc548\ubc95 \uc704\ubc18\uc73c\ub85c \uc77c\ubcf8\uc5d0\uc11c \uadc0\uad6d \uc9c1\ud6c4 \uccb4\ud3ec\ub418\uc5c8\ub2e4. \uc774\uc640 \ub3d9\uc2dc\uc5d0, \uacc4\ubd80 \uae40\ub3d9\uc21c\uacfc \ud55c\uad6d\uad70 \ub0b4\uc5d0\uc11c \ud611\ub825\ud55c \uc0ac\ub78c\ub4e4\ub3c4 \uccb4\ud3ec\ub418\uc5c8\ub2e4. \uadf8\ub9ac\uace0 \uc6d0\uc815\ud654\ub294 \ubaa8 \ubd80\ub300 \uc815\ud6c8\uc7a5\uad50\uc778 \ub300\uc704 \ud669 \ubaa8(30)\uc528\uc640 \ub0b4\uc5f0 \uad00\uacc4\ub97c \uc720\uc9c0\ud558\uba74\uc11c \ub300\uc704(\uc9c4) \ud669 \ubaa8\uc528\ub97c \uc774\uc6a9\ud574 \uad70\uc0ac \uae30\ubc00\uc744 \ube7c\ub0b4\uc5c8\uace0 \ub300\uc704(\uc9c4) \ud669 \ubaa8\uc528\ub294 \uc6d0\uc815\ud654\uac00 \uc790\uc2e0\ubcf4\ub2e4 7\uc0b4\uc774\ub098 \uc5f0\uc0c1\uc778 \ub370\ub2e4\uac00 \uac04\ucca9\uc774\ub77c\ub294 \uc0ac\uc2e4\uc744 \uc54c\uba74\uc11c\ub3c4 \ub0b4\uc5f0 \uad00\uacc4\ub97c \uc720\uc9c0 \ud558\uc600\ub2e4. \uacb0\uad6d, \uc774 \uc0ac\uac74\uc73c\ub85c \ub300\uc704(\uc9c4) \ud669 \ubaa8(30)\uc528\ub294 \ud30c\uba74\ub418\uc5b4 \ubd88\uba85\uc608 \ud1f4\uc5ed \ub418\uba74\uc11c \uad6c\uc18d\uae30\uc18c\ub418\uc5b4 2009\ub144 \ub300\ubc95\uc6d0\uc5d0\uc11c \uc9d5\uc5ed 3\ub1446\uc6d4\uc758 \uc2e4\ud615\uc744 \ud655\uc815\ud310\uacb0 \ubc1b\uc558\ub2e4.", "answer": "\uc9d5\uc5ed 3\ub1446\uc6d4", "sentence": "\uc774 \uc0ac\uac74\uc73c\ub85c \ub300\uc704(\uc9c4) \ud669 \ubaa8(30)\uc528\ub294 \ud30c\uba74\ub418\uc5b4 \ubd88\uba85\uc608 \ud1f4\uc5ed \ub418\uba74\uc11c \uad6c\uc18d\uae30\uc18c\ub418\uc5b4 2009\ub144 \ub300\ubc95\uc6d0\uc5d0\uc11c \uc9d5\uc5ed 3\ub1446\uc6d4 \uc758 \uc2e4\ud615\uc744 \ud655\uc815\ud310\uacb0 \ubc1b\uc558\ub2e4.", "paragraph_sentence": "1974\ub144\uc5d0 \ud568\uacbd\ubd81\ub3c4 \uccad\uc9c4\uc2dc\uc5d0\uc11c \ud0dc\uc5b4\ub0ac\ub2e4. 1988\ub144, \uace0\ubb34\uc0b0\uc5ec\uc790\uace0\ub4f1\uc911\ud559\uad50 4\ud559\ub144 \ub54c\ub294 \ud559\uc5c5 \uc131\uc801\uc774 \uc6b0\uc218\ud558\uc5ec '\uc774\uc911 \uc601\uc608 \ubd89\uc740\uae30 \ud718\uc7a5'\uc744 \ubc1b\uc558\uace0 15\uc138 \ub54c \uae40\uc77c\uc131\uc0ac\ud68c\uc8fc\uc758\uccad\ub144\ub3d9\ub9f9\uc5d0 \ubc1c\ud0c1\ub3fc \uacf5\uc791\uc6d0\uc744 \uc591\uc131\ud558\ub294 \ud559\uad50\uc778 \uae08\uc131\uc815\uce58\uad70\uc0ac\ub300\ud559\uc5d0\uc11c \uad50\uc721\ubc1b\uc558\uc73c\ub098 1992\ub144 \uba38\ub9ac \ubd80\uc0c1\uc73c\ub85c \uc18c\uc18d\ub41c \ud2b9\uc218\ubd80\ub300\uc5d0\uc11c \uc758\ubcd1\uc804\uc5ed\ud588\ub2e4. 1998\ub144\ubd80\ud130 \uad6d\uac00\uc548\uc804\ubcf4\uc704\ubd80\uc5d0 \ud3ec\uc12d\ub418\uc5b4 \uacf5\uc791 \ud65c\ub3d9\uc744 \uc2dc\uc791. \ud558\uc600\ub2e4\uac00, 2008\ub144 7\uc6d4 15\uc77c\uc5d0 \ub300\ud55c\ubbfc\uad6d\uc758 \uad70\uacbd \ud569\ub3d9 \uc0ac\ubc95 \ub2f9\uad6d\uc5d0 \uad6d\uac00\ubcf4\uc548\ubc95 \uc704\ubc18\uc73c\ub85c \uc77c\ubcf8\uc5d0\uc11c \uadc0\uad6d \uc9c1\ud6c4 \uccb4\ud3ec\ub418\uc5c8\ub2e4. \uc774\uc640 \ub3d9\uc2dc\uc5d0, \uacc4\ubd80 \uae40\ub3d9\uc21c\uacfc \ud55c\uad6d\uad70 \ub0b4\uc5d0\uc11c \ud611\ub825\ud55c \uc0ac\ub78c\ub4e4\ub3c4 \uccb4\ud3ec\ub418\uc5c8\ub2e4. \uadf8\ub9ac\uace0 \uc6d0\uc815\ud654\ub294 \ubaa8 \ubd80\ub300 \uc815\ud6c8\uc7a5\uad50\uc778 \ub300\uc704 \ud669 \ubaa8(30)\uc528\uc640 \ub0b4\uc5f0 \uad00\uacc4\ub97c \uc720\uc9c0\ud558\uba74\uc11c \ub300\uc704(\uc9c4) \ud669 \ubaa8\uc528\ub97c \uc774\uc6a9\ud574 \uad70\uc0ac \uae30\ubc00\uc744 \ube7c\ub0b4\uc5c8\uace0 \ub300\uc704(\uc9c4) \ud669 \ubaa8\uc528\ub294 \uc6d0\uc815\ud654\uac00 \uc790\uc2e0\ubcf4\ub2e4 7\uc0b4\uc774\ub098 \uc5f0\uc0c1\uc778 \ub370\ub2e4\uac00 \uac04\ucca9\uc774\ub77c\ub294 \uc0ac\uc2e4\uc744 \uc54c\uba74\uc11c\ub3c4 \ub0b4\uc5f0 \uad00\uacc4\ub97c \uc720\uc9c0 \ud558\uc600\ub2e4. \uacb0\uad6d, \uc774 \uc0ac\uac74\uc73c\ub85c \ub300\uc704(\uc9c4) \ud669 \ubaa8(30)\uc528\ub294 \ud30c\uba74\ub418\uc5b4 \ubd88\uba85\uc608 \ud1f4\uc5ed \ub418\uba74\uc11c \uad6c\uc18d\uae30\uc18c\ub418\uc5b4 2009\ub144 \ub300\ubc95\uc6d0\uc5d0\uc11c \uc9d5\uc5ed 3\ub1446\uc6d4 \uc758 \uc2e4\ud615\uc744 \ud655\uc815\ud310\uacb0 \ubc1b\uc558\ub2e4. ", "paragraph_answer": "1974\ub144\uc5d0 \ud568\uacbd\ubd81\ub3c4 \uccad\uc9c4\uc2dc\uc5d0\uc11c \ud0dc\uc5b4\ub0ac\ub2e4. 1988\ub144, \uace0\ubb34\uc0b0\uc5ec\uc790\uace0\ub4f1\uc911\ud559\uad50 4\ud559\ub144 \ub54c\ub294 \ud559\uc5c5 \uc131\uc801\uc774 \uc6b0\uc218\ud558\uc5ec '\uc774\uc911 \uc601\uc608 \ubd89\uc740\uae30 \ud718\uc7a5'\uc744 \ubc1b\uc558\uace0 15\uc138 \ub54c \uae40\uc77c\uc131\uc0ac\ud68c\uc8fc\uc758\uccad\ub144\ub3d9\ub9f9\uc5d0 \ubc1c\ud0c1\ub3fc \uacf5\uc791\uc6d0\uc744 \uc591\uc131\ud558\ub294 \ud559\uad50\uc778 \uae08\uc131\uc815\uce58\uad70\uc0ac\ub300\ud559\uc5d0\uc11c \uad50\uc721\ubc1b\uc558\uc73c\ub098 1992\ub144 \uba38\ub9ac \ubd80\uc0c1\uc73c\ub85c \uc18c\uc18d\ub41c \ud2b9\uc218\ubd80\ub300\uc5d0\uc11c \uc758\ubcd1\uc804\uc5ed\ud588\ub2e4. 1998\ub144\ubd80\ud130 \uad6d\uac00\uc548\uc804\ubcf4\uc704\ubd80\uc5d0 \ud3ec\uc12d\ub418\uc5b4 \uacf5\uc791 \ud65c\ub3d9\uc744 \uc2dc\uc791. \ud558\uc600\ub2e4\uac00, 2008\ub144 7\uc6d4 15\uc77c\uc5d0 \ub300\ud55c\ubbfc\uad6d\uc758 \uad70\uacbd \ud569\ub3d9 \uc0ac\ubc95 \ub2f9\uad6d\uc5d0 \uad6d\uac00\ubcf4\uc548\ubc95 \uc704\ubc18\uc73c\ub85c \uc77c\ubcf8\uc5d0\uc11c \uadc0\uad6d \uc9c1\ud6c4 \uccb4\ud3ec\ub418\uc5c8\ub2e4. \uc774\uc640 \ub3d9\uc2dc\uc5d0, \uacc4\ubd80 \uae40\ub3d9\uc21c\uacfc \ud55c\uad6d\uad70 \ub0b4\uc5d0\uc11c \ud611\ub825\ud55c \uc0ac\ub78c\ub4e4\ub3c4 \uccb4\ud3ec\ub418\uc5c8\ub2e4. \uadf8\ub9ac\uace0 \uc6d0\uc815\ud654\ub294 \ubaa8 \ubd80\ub300 \uc815\ud6c8\uc7a5\uad50\uc778 \ub300\uc704 \ud669 \ubaa8(30)\uc528\uc640 \ub0b4\uc5f0 \uad00\uacc4\ub97c \uc720\uc9c0\ud558\uba74\uc11c \ub300\uc704(\uc9c4) \ud669 \ubaa8\uc528\ub97c \uc774\uc6a9\ud574 \uad70\uc0ac \uae30\ubc00\uc744 \ube7c\ub0b4\uc5c8\uace0 \ub300\uc704(\uc9c4) \ud669 \ubaa8\uc528\ub294 \uc6d0\uc815\ud654\uac00 \uc790\uc2e0\ubcf4\ub2e4 7\uc0b4\uc774\ub098 \uc5f0\uc0c1\uc778 \ub370\ub2e4\uac00 \uac04\ucca9\uc774\ub77c\ub294 \uc0ac\uc2e4\uc744 \uc54c\uba74\uc11c\ub3c4 \ub0b4\uc5f0 \uad00\uacc4\ub97c \uc720\uc9c0 \ud558\uc600\ub2e4. \uacb0\uad6d, \uc774 \uc0ac\uac74\uc73c\ub85c \ub300\uc704(\uc9c4) \ud669 \ubaa8(30)\uc528\ub294 \ud30c\uba74\ub418\uc5b4 \ubd88\uba85\uc608 \ud1f4\uc5ed \ub418\uba74\uc11c \uad6c\uc18d\uae30\uc18c\ub418\uc5b4 2009\ub144 \ub300\ubc95\uc6d0\uc5d0\uc11c \uc9d5\uc5ed 3\ub1446\uc6d4 \uc758 \uc2e4\ud615\uc744 \ud655\uc815\ud310\uacb0 \ubc1b\uc558\ub2e4.", "sentence_answer": "\uc774 \uc0ac\uac74\uc73c\ub85c \ub300\uc704(\uc9c4) \ud669 \ubaa8(30)\uc528\ub294 \ud30c\uba74\ub418\uc5b4 \ubd88\uba85\uc608 \ud1f4\uc5ed \ub418\uba74\uc11c \uad6c\uc18d\uae30\uc18c\ub418\uc5b4 2009\ub144 \ub300\ubc95\uc6d0\uc5d0\uc11c \uc9d5\uc5ed 3\ub1446\uc6d4 \uc758 \uc2e4\ud615\uc744 \ud655\uc815\ud310\uacb0 \ubc1b\uc558\ub2e4.", "paragraph_id": "5826881-0-2", "paragraph_question": "question: \uc6d0\uc815\ud654\uc640 \ub0b4\uc5f0 \uad00\uacc4\ub97c \uc720\uc9c0\ud55c \ub300\uc704\ub294 \uc5b4\ub5a4 \ud310\uacb0\uc744 \ubc1b\uc558\ub294\uac00?, context: 1974\ub144\uc5d0 \ud568\uacbd\ubd81\ub3c4 \uccad\uc9c4\uc2dc\uc5d0\uc11c \ud0dc\uc5b4\ub0ac\ub2e4. 1988\ub144, \uace0\ubb34\uc0b0\uc5ec\uc790\uace0\ub4f1\uc911\ud559\uad50 4\ud559\ub144 \ub54c\ub294 \ud559\uc5c5 \uc131\uc801\uc774 \uc6b0\uc218\ud558\uc5ec '\uc774\uc911 \uc601\uc608 \ubd89\uc740\uae30 \ud718\uc7a5'\uc744 \ubc1b\uc558\uace0 15\uc138 \ub54c \uae40\uc77c\uc131\uc0ac\ud68c\uc8fc\uc758\uccad\ub144\ub3d9\ub9f9\uc5d0 \ubc1c\ud0c1\ub3fc \uacf5\uc791\uc6d0\uc744 \uc591\uc131\ud558\ub294 \ud559\uad50\uc778 \uae08\uc131\uc815\uce58\uad70\uc0ac\ub300\ud559\uc5d0\uc11c \uad50\uc721\ubc1b\uc558\uc73c\ub098 1992\ub144 \uba38\ub9ac \ubd80\uc0c1\uc73c\ub85c \uc18c\uc18d\ub41c \ud2b9\uc218\ubd80\ub300\uc5d0\uc11c \uc758\ubcd1\uc804\uc5ed\ud588\ub2e4. 1998\ub144\ubd80\ud130 \uad6d\uac00\uc548\uc804\ubcf4\uc704\ubd80\uc5d0 \ud3ec\uc12d\ub418\uc5b4 \uacf5\uc791 \ud65c\ub3d9\uc744 \uc2dc\uc791. \ud558\uc600\ub2e4\uac00, 2008\ub144 7\uc6d4 15\uc77c\uc5d0 \ub300\ud55c\ubbfc\uad6d\uc758 \uad70\uacbd \ud569\ub3d9 \uc0ac\ubc95 \ub2f9\uad6d\uc5d0 \uad6d\uac00\ubcf4\uc548\ubc95 \uc704\ubc18\uc73c\ub85c \uc77c\ubcf8\uc5d0\uc11c \uadc0\uad6d \uc9c1\ud6c4 \uccb4\ud3ec\ub418\uc5c8\ub2e4. \uc774\uc640 \ub3d9\uc2dc\uc5d0, \uacc4\ubd80 \uae40\ub3d9\uc21c\uacfc \ud55c\uad6d\uad70 \ub0b4\uc5d0\uc11c \ud611\ub825\ud55c \uc0ac\ub78c\ub4e4\ub3c4 \uccb4\ud3ec\ub418\uc5c8\ub2e4. \uadf8\ub9ac\uace0 \uc6d0\uc815\ud654\ub294 \ubaa8 \ubd80\ub300 \uc815\ud6c8\uc7a5\uad50\uc778 \ub300\uc704 \ud669 \ubaa8(30)\uc528\uc640 \ub0b4\uc5f0 \uad00\uacc4\ub97c \uc720\uc9c0\ud558\uba74\uc11c \ub300\uc704(\uc9c4) \ud669 \ubaa8\uc528\ub97c \uc774\uc6a9\ud574 \uad70\uc0ac \uae30\ubc00\uc744 \ube7c\ub0b4\uc5c8\uace0 \ub300\uc704(\uc9c4) \ud669 \ubaa8\uc528\ub294 \uc6d0\uc815\ud654\uac00 \uc790\uc2e0\ubcf4\ub2e4 7\uc0b4\uc774\ub098 \uc5f0\uc0c1\uc778 \ub370\ub2e4\uac00 \uac04\ucca9\uc774\ub77c\ub294 \uc0ac\uc2e4\uc744 \uc54c\uba74\uc11c\ub3c4 \ub0b4\uc5f0 \uad00\uacc4\ub97c \uc720\uc9c0 \ud558\uc600\ub2e4. \uacb0\uad6d, \uc774 \uc0ac\uac74\uc73c\ub85c \ub300\uc704(\uc9c4) \ud669 \ubaa8(30)\uc528\ub294 \ud30c\uba74\ub418\uc5b4 \ubd88\uba85\uc608 \ud1f4\uc5ed \ub418\uba74\uc11c \uad6c\uc18d\uae30\uc18c\ub418\uc5b4 2009\ub144 \ub300\ubc95\uc6d0\uc5d0\uc11c \uc9d5\uc5ed 3\ub1446\uc6d4\uc758 \uc2e4\ud615\uc744 \ud655\uc815\ud310\uacb0 \ubc1b\uc558\ub2e4."} {"question": "1923\ub144 \uc789\uae00\ub79c\ub4dc FA\ucef5 \uacb0\uc2b9\uc804 \uc804\ubc18 11\ubd84 \uacbd\uc5d0 \uacbd\uae30\uac00 \uc7a0\uc2dc \uc911\ub2e8\ub41c \uc774\uc720\ub294?", "paragraph": "\uc591 \ud300 \ubaa8\ub450 \uadf8 \ub2f9\uc2dc\uc5d0 \uac00\uc7a5 \uc804\ud615\uc801\uc778 \ud3ec\uba54\uc774\uc158\uc744 \ub4e4\uace0 \ub098\uc654\ub294\ub370, \uc804\uc220\uc758 \uae30\ubcf8 \ud615\ud0dc\ub294 2\uba85\uc758 \ud480\ubc31\uc774 \ub4a4\uc5d0 \uc704\uce58\ud558\uace0 \ud55c \uba85\uc758 \uc13c\ud130 \ud558\ud504\uc640 \ub450 \uba85\uc758 \uc719 \ud558\ud504\ub85c \uad6c\uc131\ub41c 3\uba85\uc758 \ud558\ud504\ubc31\uc774 \uadf8 \uc55e\uc5d0 \uc11c\uc11c \uc218\ube44\ub97c \ub2f4\ub2f9\ud558\uace0 \ub9c8\uc9c0\ub9c9\uc73c\ub85c 2\uba85\uc758 \uc544\uc6c3\uc0ac\uc774\ub4dc \ud3ec\uc6cc\ub4dc, \ub450 \uba85\uc758 \uc778\uc0ac\uc774\ub4dc \ud3ec\uc6cc\ub4dc, \uadf8\ub9ac\uace0 \ud55c \uba85\uc758 \uc13c\ud130 \ud3ec\uc6cc\ub4dc\ub85c \uad6c\uc131\ub41c 5\uba85\uc758 \ud3ec\uc6cc\ub4dc\uac00 \uacf5\uaca9\uc744 \ub9e1\ub294 \uac83\uc774\ub2e4. \uc6e8\uc2a4\ud2b8 \ud584\uc740 \ub450 \ube60\ub978 \uc544\uc6c3\uc0ac\uc774\ub4dc \ud3ec\uc6cc\ub4dc\uc778 \ub515 \ub9ac\ucc28\uc988\uc640 \uc9c0\ubbf8 \ub7ec\ud3a0\uc744 \uc911\uc2ec\uc73c\ub85c \ud558\ub294 \uc804\ub7b5\uc744 \uad6c\uc131\ud588\uc73c\ub098 \ubcfc\ud134\uc740 \uc2dc\uc791\ud558\uc790\ub9c8\uc790 \uadf8 \ub450 \uc120\uc218\ub97c \uc798 \uc6c0\uc9c1\uc774\uc9c0 \ubabb\ud558\uac8c \ubb36\uc5b4\ub450\uc5c8\uace0, \ub610\ud55c \uadf8 \ub450 \uc120\uc218\uac00 \uacf5\uc744 \uc7a1\uc790\ub9c8\uc790 \ub2ec\ub824\ub4e4\uc5b4 \ubc29\ud574\ud558\ub294 \ub9de\ucda4\uc804\ub7b5\uc744 \uc37c\ub2e4. \uacbd\uae30 \uc2dc\uc791 2\ubd84 \ud6c4 \uc6e8\uc2a4\ud2b8 \ud584\uc758 \ud558\ud504\ubc31\uc778 \uc7ad \ud2b8\ub808\uc0ac\ub358\uc774 \uc2a4\ub85c\uc778\uc744 \ub358\uc9c4 \ud6c4\uc5d0 \uad00\uc911\ub4e4\uc5d0 \ud729\uc4f8\ub824 \uadf8\ub77c\uc6b4\ub4dc\ub85c \ubcf5\uadc0\ud558\uc9c0 \ubabb\ud558\ub294 \uc0c1\ud669\uc774 \ubc8c\uc5b4\uc84c\ub2e4. \uc774 \uae30\ud68c\ub97c \ub193\uce58\uc9c0 \uc54a\uc740 \ubcfc\ud134\uc758 \ub370\uc774\ube44\ub4dc \uc7ad\uc774 \uc120\uc218 \uc22b\uc790\uc758 \uc6b0\uc704\ub97c \uc810\ud55c \uc0c1\ud669\uc5d0\uc11c \uc288\ud305\uc744 \ud558\uc600\ub2e4. \uc6e8\uc2a4\ud2b8 \ud584\uc758 \uace8\ud0a4\ud37c \ud14c\ub4dc \ud5c8\ud504\ud2bc\uc774 \uc774 \uc29b\uc744 \ub9c9\uc9c0 \ubabb\ud558\uace0 \uc2e4\uc810\uc744 \ud5c8\uc6a9\ud574 \ubcfc\ud134\uc774 \uc55e\uc11c\ub098\uac00\uae30 \uc2dc\uc791\ud588\ub2e4. \uadf8\ub54c \uadf8 \uacf5\uc774 \ub2e4\ub978 \uad00\uc911\ub4e4\uc5d0 \ubc00\ub824\ub0b4\ub824\uc640 \uace8\ub124\ud2b8\uc5d0 \ubc14\uc9dd \ubd99\uc5b4\uc788\ub358 \ud55c \uad00\uc911\uc744 \uac15\ud0c0\ud558\uba74\uc11c \uadf8 \uad00\uc911\uc774 \ubb34\uc758\uc2dd \uc0c1\ud0dc\uac00 \ub418\uc5b4\ubc84\ub838\ub2e4. \uc2e4\uc810 3\ubd84 \ud6c4 \uc6e8\uc2a4\ud2b8 \ud584\uc758 \ube45 \uc653\uc2a8\uc774 \ubcfc\ud134\uc758 \uace8\ubb38 \ubc14\ub85c \uc55e\uc5d0\uc11c \uacf5\uc744 \uc7a1\uc544 \ucc9c\uae08\uac19\uc740 \uae30\ud68c\ub97c \uc7a1\uc558\uc73c\ub098 \uc29b\uc774 \ud06c\ub85c\uc2a4\ubc14\ub97c \ub118\uc5b4\uac00\ubc84\ub838\ub2e4. \uc804\ubc18 11\ubd84 \uacbd\uc5d0\ub294 \ub610\ub2e4\uc2dc \uad00\uc911\ub4e4\uc774 \uc55e\uc73c\ub85c \ubab0\ub9ac\uba74\uc11c \ub2e4\uc218\uc758 \ud32c\ub4e4\uc774 \uadf8\ub77c\uc6b4\ub4dc \uc548\uc73c\ub85c \uc9c4\uc785\ud558\ub294 \ubc14\ub78c\uc5d0 \ubd80\uc2ec\ub4e4\uc744 \uc704\ud574 \ub9d0\uc744 \ud0c4 \uacbd\ucc30\ub4e4\uc774 \uc0c1\ud669\uc744 \uc218\uc2b5\ud560 \ub54c\uae4c\uc9c0 \uacbd\uae30\uac00 \uc9c0\uc5f0\ub418\uc5c8\ub2e4. \uc120\uc218\ub4e4\uc774 \uacbd\uae30 \uc7ac\uac1c\ub97c \uae30\ub2e4\ub9ac\ub294 \ub3d9\uc548 \uba87\uba87 \uad00\uc911\ub4e4\uc740 \ub9ce\uc740 \uad00\uc911 \uc18d\uc5d0\uc11c \ub2f9\ud55c \ubd80\uc0c1 \ub54c\ubb38\uc5d0 \uc601\uad6d \uc801\uc2ed\uc790\uc758 \uc751\uae09\ucc98\uce58\uac00 \ud544\uc694\ud558\uae30\ub3c4 \ud588\ub2e4. \uacbd\ucc30\ub4e4\uc740 \uadf8\ub77c\uc6b4\ub4dc \uc8fc\ubcc0\uc744 \ub3cc\uba70 \ub354\uc774\uc0c1 \uad00\uc911\ub4e4\uc774 \uc120\uc744 \ub118\uc5b4 \uc9c4\uc785\ud558\uc9c0 \ubabb\ud558\ub3c4\ub85d \ud588\uace0 \uadf8 \uc0ac\uc774 \uacbd\uae30\uac00 \uc7ac\uac1c\ub418\uc5c8\ub2e4.", "answer": "\ub2e4\uc218\uc758 \ud32c\ub4e4\uc774 \uadf8\ub77c\uc6b4\ub4dc \uc548\uc73c\ub85c \uc9c4\uc785", "sentence": "\uc804\ubc18 11\ubd84 \uacbd\uc5d0\ub294 \ub610\ub2e4\uc2dc \uad00\uc911\ub4e4\uc774 \uc55e\uc73c\ub85c \ubab0\ub9ac\uba74\uc11c \ub2e4\uc218\uc758 \ud32c\ub4e4\uc774 \uadf8\ub77c\uc6b4\ub4dc \uc548\uc73c\ub85c \uc9c4\uc785 \ud558\ub294 \ubc14\ub78c\uc5d0 \ubd80\uc2ec\ub4e4\uc744 \uc704\ud574 \ub9d0\uc744 \ud0c4 \uacbd\ucc30\ub4e4\uc774 \uc0c1\ud669\uc744 \uc218\uc2b5\ud560 \ub54c\uae4c\uc9c0 \uacbd\uae30\uac00 \uc9c0\uc5f0\ub418\uc5c8\ub2e4.", "paragraph_sentence": "\uc591 \ud300 \ubaa8\ub450 \uadf8 \ub2f9\uc2dc\uc5d0 \uac00\uc7a5 \uc804\ud615\uc801\uc778 \ud3ec\uba54\uc774\uc158\uc744 \ub4e4\uace0 \ub098\uc654\ub294\ub370, \uc804\uc220\uc758 \uae30\ubcf8 \ud615\ud0dc\ub294 2\uba85\uc758 \ud480\ubc31\uc774 \ub4a4\uc5d0 \uc704\uce58\ud558\uace0 \ud55c \uba85\uc758 \uc13c\ud130 \ud558\ud504\uc640 \ub450 \uba85\uc758 \uc719 \ud558\ud504\ub85c \uad6c\uc131\ub41c 3\uba85\uc758 \ud558\ud504\ubc31\uc774 \uadf8 \uc55e\uc5d0 \uc11c\uc11c \uc218\ube44\ub97c \ub2f4\ub2f9\ud558\uace0 \ub9c8\uc9c0\ub9c9\uc73c\ub85c 2\uba85\uc758 \uc544\uc6c3\uc0ac\uc774\ub4dc \ud3ec\uc6cc\ub4dc, \ub450 \uba85\uc758 \uc778\uc0ac\uc774\ub4dc \ud3ec\uc6cc\ub4dc, \uadf8\ub9ac\uace0 \ud55c \uba85\uc758 \uc13c\ud130 \ud3ec\uc6cc\ub4dc\ub85c \uad6c\uc131\ub41c 5\uba85\uc758 \ud3ec\uc6cc\ub4dc\uac00 \uacf5\uaca9\uc744 \ub9e1\ub294 \uac83\uc774\ub2e4. \uc6e8\uc2a4\ud2b8 \ud584\uc740 \ub450 \ube60\ub978 \uc544\uc6c3\uc0ac\uc774\ub4dc \ud3ec\uc6cc\ub4dc\uc778 \ub515 \ub9ac\ucc28\uc988\uc640 \uc9c0\ubbf8 \ub7ec\ud3a0\uc744 \uc911\uc2ec\uc73c\ub85c \ud558\ub294 \uc804\ub7b5\uc744 \uad6c\uc131\ud588\uc73c\ub098 \ubcfc\ud134\uc740 \uc2dc\uc791\ud558\uc790\ub9c8\uc790 \uadf8 \ub450 \uc120\uc218\ub97c \uc798 \uc6c0\uc9c1\uc774\uc9c0 \ubabb\ud558\uac8c \ubb36\uc5b4\ub450\uc5c8\uace0, \ub610\ud55c \uadf8 \ub450 \uc120\uc218\uac00 \uacf5\uc744 \uc7a1\uc790\ub9c8\uc790 \ub2ec\ub824\ub4e4\uc5b4 \ubc29\ud574\ud558\ub294 \ub9de\ucda4\uc804\ub7b5\uc744 \uc37c\ub2e4. \uacbd\uae30 \uc2dc\uc791 2\ubd84 \ud6c4 \uc6e8\uc2a4\ud2b8 \ud584\uc758 \ud558\ud504\ubc31\uc778 \uc7ad \ud2b8\ub808\uc0ac\ub358\uc774 \uc2a4\ub85c\uc778\uc744 \ub358\uc9c4 \ud6c4\uc5d0 \uad00\uc911\ub4e4\uc5d0 \ud729\uc4f8\ub824 \uadf8\ub77c\uc6b4\ub4dc\ub85c \ubcf5\uadc0\ud558\uc9c0 \ubabb\ud558\ub294 \uc0c1\ud669\uc774 \ubc8c\uc5b4\uc84c\ub2e4. \uc774 \uae30\ud68c\ub97c \ub193\uce58\uc9c0 \uc54a\uc740 \ubcfc\ud134\uc758 \ub370\uc774\ube44\ub4dc \uc7ad\uc774 \uc120\uc218 \uc22b\uc790\uc758 \uc6b0\uc704\ub97c \uc810\ud55c \uc0c1\ud669\uc5d0\uc11c \uc288\ud305\uc744 \ud558\uc600\ub2e4. \uc6e8\uc2a4\ud2b8 \ud584\uc758 \uace8\ud0a4\ud37c \ud14c\ub4dc \ud5c8\ud504\ud2bc\uc774 \uc774 \uc29b\uc744 \ub9c9\uc9c0 \ubabb\ud558\uace0 \uc2e4\uc810\uc744 \ud5c8\uc6a9\ud574 \ubcfc\ud134\uc774 \uc55e\uc11c\ub098\uac00\uae30 \uc2dc\uc791\ud588\ub2e4. \uadf8\ub54c \uadf8 \uacf5\uc774 \ub2e4\ub978 \uad00\uc911\ub4e4\uc5d0 \ubc00\ub824\ub0b4\ub824\uc640 \uace8\ub124\ud2b8\uc5d0 \ubc14\uc9dd \ubd99\uc5b4\uc788\ub358 \ud55c \uad00\uc911\uc744 \uac15\ud0c0\ud558\uba74\uc11c \uadf8 \uad00\uc911\uc774 \ubb34\uc758\uc2dd \uc0c1\ud0dc\uac00 \ub418\uc5b4\ubc84\ub838\ub2e4. \uc2e4\uc810 3\ubd84 \ud6c4 \uc6e8\uc2a4\ud2b8 \ud584\uc758 \ube45 \uc653\uc2a8\uc774 \ubcfc\ud134\uc758 \uace8\ubb38 \ubc14\ub85c \uc55e\uc5d0\uc11c \uacf5\uc744 \uc7a1\uc544 \ucc9c\uae08\uac19\uc740 \uae30\ud68c\ub97c \uc7a1\uc558\uc73c\ub098 \uc29b\uc774 \ud06c\ub85c\uc2a4\ubc14\ub97c \ub118\uc5b4\uac00\ubc84\ub838\ub2e4. \uc804\ubc18 11\ubd84 \uacbd\uc5d0\ub294 \ub610\ub2e4\uc2dc \uad00\uc911\ub4e4\uc774 \uc55e\uc73c\ub85c \ubab0\ub9ac\uba74\uc11c \ub2e4\uc218\uc758 \ud32c\ub4e4\uc774 \uadf8\ub77c\uc6b4\ub4dc \uc548\uc73c\ub85c \uc9c4\uc785 \ud558\ub294 \ubc14\ub78c\uc5d0 \ubd80\uc2ec\ub4e4\uc744 \uc704\ud574 \ub9d0\uc744 \ud0c4 \uacbd\ucc30\ub4e4\uc774 \uc0c1\ud669\uc744 \uc218\uc2b5\ud560 \ub54c\uae4c\uc9c0 \uacbd\uae30\uac00 \uc9c0\uc5f0\ub418\uc5c8\ub2e4. \uc120\uc218\ub4e4\uc774 \uacbd\uae30 \uc7ac\uac1c\ub97c \uae30\ub2e4\ub9ac\ub294 \ub3d9\uc548 \uba87\uba87 \uad00\uc911\ub4e4\uc740 \ub9ce\uc740 \uad00\uc911 \uc18d\uc5d0\uc11c \ub2f9\ud55c \ubd80\uc0c1 \ub54c\ubb38\uc5d0 \uc601\uad6d \uc801\uc2ed\uc790\uc758 \uc751\uae09\ucc98\uce58\uac00 \ud544\uc694\ud558\uae30\ub3c4 \ud588\ub2e4. \uacbd\ucc30\ub4e4\uc740 \uadf8\ub77c\uc6b4\ub4dc \uc8fc\ubcc0\uc744 \ub3cc\uba70 \ub354\uc774\uc0c1 \uad00\uc911\ub4e4\uc774 \uc120\uc744 \ub118\uc5b4 \uc9c4\uc785\ud558\uc9c0 \ubabb\ud558\ub3c4\ub85d \ud588\uace0 \uadf8 \uc0ac\uc774 \uacbd\uae30\uac00 \uc7ac\uac1c\ub418\uc5c8\ub2e4.", "paragraph_answer": "\uc591 \ud300 \ubaa8\ub450 \uadf8 \ub2f9\uc2dc\uc5d0 \uac00\uc7a5 \uc804\ud615\uc801\uc778 \ud3ec\uba54\uc774\uc158\uc744 \ub4e4\uace0 \ub098\uc654\ub294\ub370, \uc804\uc220\uc758 \uae30\ubcf8 \ud615\ud0dc\ub294 2\uba85\uc758 \ud480\ubc31\uc774 \ub4a4\uc5d0 \uc704\uce58\ud558\uace0 \ud55c \uba85\uc758 \uc13c\ud130 \ud558\ud504\uc640 \ub450 \uba85\uc758 \uc719 \ud558\ud504\ub85c \uad6c\uc131\ub41c 3\uba85\uc758 \ud558\ud504\ubc31\uc774 \uadf8 \uc55e\uc5d0 \uc11c\uc11c \uc218\ube44\ub97c \ub2f4\ub2f9\ud558\uace0 \ub9c8\uc9c0\ub9c9\uc73c\ub85c 2\uba85\uc758 \uc544\uc6c3\uc0ac\uc774\ub4dc \ud3ec\uc6cc\ub4dc, \ub450 \uba85\uc758 \uc778\uc0ac\uc774\ub4dc \ud3ec\uc6cc\ub4dc, \uadf8\ub9ac\uace0 \ud55c \uba85\uc758 \uc13c\ud130 \ud3ec\uc6cc\ub4dc\ub85c \uad6c\uc131\ub41c 5\uba85\uc758 \ud3ec\uc6cc\ub4dc\uac00 \uacf5\uaca9\uc744 \ub9e1\ub294 \uac83\uc774\ub2e4. \uc6e8\uc2a4\ud2b8 \ud584\uc740 \ub450 \ube60\ub978 \uc544\uc6c3\uc0ac\uc774\ub4dc \ud3ec\uc6cc\ub4dc\uc778 \ub515 \ub9ac\ucc28\uc988\uc640 \uc9c0\ubbf8 \ub7ec\ud3a0\uc744 \uc911\uc2ec\uc73c\ub85c \ud558\ub294 \uc804\ub7b5\uc744 \uad6c\uc131\ud588\uc73c\ub098 \ubcfc\ud134\uc740 \uc2dc\uc791\ud558\uc790\ub9c8\uc790 \uadf8 \ub450 \uc120\uc218\ub97c \uc798 \uc6c0\uc9c1\uc774\uc9c0 \ubabb\ud558\uac8c \ubb36\uc5b4\ub450\uc5c8\uace0, \ub610\ud55c \uadf8 \ub450 \uc120\uc218\uac00 \uacf5\uc744 \uc7a1\uc790\ub9c8\uc790 \ub2ec\ub824\ub4e4\uc5b4 \ubc29\ud574\ud558\ub294 \ub9de\ucda4\uc804\ub7b5\uc744 \uc37c\ub2e4. \uacbd\uae30 \uc2dc\uc791 2\ubd84 \ud6c4 \uc6e8\uc2a4\ud2b8 \ud584\uc758 \ud558\ud504\ubc31\uc778 \uc7ad \ud2b8\ub808\uc0ac\ub358\uc774 \uc2a4\ub85c\uc778\uc744 \ub358\uc9c4 \ud6c4\uc5d0 \uad00\uc911\ub4e4\uc5d0 \ud729\uc4f8\ub824 \uadf8\ub77c\uc6b4\ub4dc\ub85c \ubcf5\uadc0\ud558\uc9c0 \ubabb\ud558\ub294 \uc0c1\ud669\uc774 \ubc8c\uc5b4\uc84c\ub2e4. \uc774 \uae30\ud68c\ub97c \ub193\uce58\uc9c0 \uc54a\uc740 \ubcfc\ud134\uc758 \ub370\uc774\ube44\ub4dc \uc7ad\uc774 \uc120\uc218 \uc22b\uc790\uc758 \uc6b0\uc704\ub97c \uc810\ud55c \uc0c1\ud669\uc5d0\uc11c \uc288\ud305\uc744 \ud558\uc600\ub2e4. \uc6e8\uc2a4\ud2b8 \ud584\uc758 \uace8\ud0a4\ud37c \ud14c\ub4dc \ud5c8\ud504\ud2bc\uc774 \uc774 \uc29b\uc744 \ub9c9\uc9c0 \ubabb\ud558\uace0 \uc2e4\uc810\uc744 \ud5c8\uc6a9\ud574 \ubcfc\ud134\uc774 \uc55e\uc11c\ub098\uac00\uae30 \uc2dc\uc791\ud588\ub2e4. \uadf8\ub54c \uadf8 \uacf5\uc774 \ub2e4\ub978 \uad00\uc911\ub4e4\uc5d0 \ubc00\ub824\ub0b4\ub824\uc640 \uace8\ub124\ud2b8\uc5d0 \ubc14\uc9dd \ubd99\uc5b4\uc788\ub358 \ud55c \uad00\uc911\uc744 \uac15\ud0c0\ud558\uba74\uc11c \uadf8 \uad00\uc911\uc774 \ubb34\uc758\uc2dd \uc0c1\ud0dc\uac00 \ub418\uc5b4\ubc84\ub838\ub2e4. \uc2e4\uc810 3\ubd84 \ud6c4 \uc6e8\uc2a4\ud2b8 \ud584\uc758 \ube45 \uc653\uc2a8\uc774 \ubcfc\ud134\uc758 \uace8\ubb38 \ubc14\ub85c \uc55e\uc5d0\uc11c \uacf5\uc744 \uc7a1\uc544 \ucc9c\uae08\uac19\uc740 \uae30\ud68c\ub97c \uc7a1\uc558\uc73c\ub098 \uc29b\uc774 \ud06c\ub85c\uc2a4\ubc14\ub97c \ub118\uc5b4\uac00\ubc84\ub838\ub2e4. \uc804\ubc18 11\ubd84 \uacbd\uc5d0\ub294 \ub610\ub2e4\uc2dc \uad00\uc911\ub4e4\uc774 \uc55e\uc73c\ub85c \ubab0\ub9ac\uba74\uc11c \ub2e4\uc218\uc758 \ud32c\ub4e4\uc774 \uadf8\ub77c\uc6b4\ub4dc \uc548\uc73c\ub85c \uc9c4\uc785 \ud558\ub294 \ubc14\ub78c\uc5d0 \ubd80\uc2ec\ub4e4\uc744 \uc704\ud574 \ub9d0\uc744 \ud0c4 \uacbd\ucc30\ub4e4\uc774 \uc0c1\ud669\uc744 \uc218\uc2b5\ud560 \ub54c\uae4c\uc9c0 \uacbd\uae30\uac00 \uc9c0\uc5f0\ub418\uc5c8\ub2e4. \uc120\uc218\ub4e4\uc774 \uacbd\uae30 \uc7ac\uac1c\ub97c \uae30\ub2e4\ub9ac\ub294 \ub3d9\uc548 \uba87\uba87 \uad00\uc911\ub4e4\uc740 \ub9ce\uc740 \uad00\uc911 \uc18d\uc5d0\uc11c \ub2f9\ud55c \ubd80\uc0c1 \ub54c\ubb38\uc5d0 \uc601\uad6d \uc801\uc2ed\uc790\uc758 \uc751\uae09\ucc98\uce58\uac00 \ud544\uc694\ud558\uae30\ub3c4 \ud588\ub2e4. \uacbd\ucc30\ub4e4\uc740 \uadf8\ub77c\uc6b4\ub4dc \uc8fc\ubcc0\uc744 \ub3cc\uba70 \ub354\uc774\uc0c1 \uad00\uc911\ub4e4\uc774 \uc120\uc744 \ub118\uc5b4 \uc9c4\uc785\ud558\uc9c0 \ubabb\ud558\ub3c4\ub85d \ud588\uace0 \uadf8 \uc0ac\uc774 \uacbd\uae30\uac00 \uc7ac\uac1c\ub418\uc5c8\ub2e4.", "sentence_answer": "\uc804\ubc18 11\ubd84 \uacbd\uc5d0\ub294 \ub610\ub2e4\uc2dc \uad00\uc911\ub4e4\uc774 \uc55e\uc73c\ub85c \ubab0\ub9ac\uba74\uc11c \ub2e4\uc218\uc758 \ud32c\ub4e4\uc774 \uadf8\ub77c\uc6b4\ub4dc \uc548\uc73c\ub85c \uc9c4\uc785 \ud558\ub294 \ubc14\ub78c\uc5d0 \ubd80\uc2ec\ub4e4\uc744 \uc704\ud574 \ub9d0\uc744 \ud0c4 \uacbd\ucc30\ub4e4\uc774 \uc0c1\ud669\uc744 \uc218\uc2b5\ud560 \ub54c\uae4c\uc9c0 \uacbd\uae30\uac00 \uc9c0\uc5f0\ub418\uc5c8\ub2e4.", "paragraph_id": "5782571-6-1", "paragraph_question": "question: 1923\ub144 \uc789\uae00\ub79c\ub4dc FA\ucef5 \uacb0\uc2b9\uc804 \uc804\ubc18 11\ubd84 \uacbd\uc5d0 \uacbd\uae30\uac00 \uc7a0\uc2dc \uc911\ub2e8\ub41c \uc774\uc720\ub294?, context: \uc591 \ud300 \ubaa8\ub450 \uadf8 \ub2f9\uc2dc\uc5d0 \uac00\uc7a5 \uc804\ud615\uc801\uc778 \ud3ec\uba54\uc774\uc158\uc744 \ub4e4\uace0 \ub098\uc654\ub294\ub370, \uc804\uc220\uc758 \uae30\ubcf8 \ud615\ud0dc\ub294 2\uba85\uc758 \ud480\ubc31\uc774 \ub4a4\uc5d0 \uc704\uce58\ud558\uace0 \ud55c \uba85\uc758 \uc13c\ud130 \ud558\ud504\uc640 \ub450 \uba85\uc758 \uc719 \ud558\ud504\ub85c \uad6c\uc131\ub41c 3\uba85\uc758 \ud558\ud504\ubc31\uc774 \uadf8 \uc55e\uc5d0 \uc11c\uc11c \uc218\ube44\ub97c \ub2f4\ub2f9\ud558\uace0 \ub9c8\uc9c0\ub9c9\uc73c\ub85c 2\uba85\uc758 \uc544\uc6c3\uc0ac\uc774\ub4dc \ud3ec\uc6cc\ub4dc, \ub450 \uba85\uc758 \uc778\uc0ac\uc774\ub4dc \ud3ec\uc6cc\ub4dc, \uadf8\ub9ac\uace0 \ud55c \uba85\uc758 \uc13c\ud130 \ud3ec\uc6cc\ub4dc\ub85c \uad6c\uc131\ub41c 5\uba85\uc758 \ud3ec\uc6cc\ub4dc\uac00 \uacf5\uaca9\uc744 \ub9e1\ub294 \uac83\uc774\ub2e4. \uc6e8\uc2a4\ud2b8 \ud584\uc740 \ub450 \ube60\ub978 \uc544\uc6c3\uc0ac\uc774\ub4dc \ud3ec\uc6cc\ub4dc\uc778 \ub515 \ub9ac\ucc28\uc988\uc640 \uc9c0\ubbf8 \ub7ec\ud3a0\uc744 \uc911\uc2ec\uc73c\ub85c \ud558\ub294 \uc804\ub7b5\uc744 \uad6c\uc131\ud588\uc73c\ub098 \ubcfc\ud134\uc740 \uc2dc\uc791\ud558\uc790\ub9c8\uc790 \uadf8 \ub450 \uc120\uc218\ub97c \uc798 \uc6c0\uc9c1\uc774\uc9c0 \ubabb\ud558\uac8c \ubb36\uc5b4\ub450\uc5c8\uace0, \ub610\ud55c \uadf8 \ub450 \uc120\uc218\uac00 \uacf5\uc744 \uc7a1\uc790\ub9c8\uc790 \ub2ec\ub824\ub4e4\uc5b4 \ubc29\ud574\ud558\ub294 \ub9de\ucda4\uc804\ub7b5\uc744 \uc37c\ub2e4. \uacbd\uae30 \uc2dc\uc791 2\ubd84 \ud6c4 \uc6e8\uc2a4\ud2b8 \ud584\uc758 \ud558\ud504\ubc31\uc778 \uc7ad \ud2b8\ub808\uc0ac\ub358\uc774 \uc2a4\ub85c\uc778\uc744 \ub358\uc9c4 \ud6c4\uc5d0 \uad00\uc911\ub4e4\uc5d0 \ud729\uc4f8\ub824 \uadf8\ub77c\uc6b4\ub4dc\ub85c \ubcf5\uadc0\ud558\uc9c0 \ubabb\ud558\ub294 \uc0c1\ud669\uc774 \ubc8c\uc5b4\uc84c\ub2e4. \uc774 \uae30\ud68c\ub97c \ub193\uce58\uc9c0 \uc54a\uc740 \ubcfc\ud134\uc758 \ub370\uc774\ube44\ub4dc \uc7ad\uc774 \uc120\uc218 \uc22b\uc790\uc758 \uc6b0\uc704\ub97c \uc810\ud55c \uc0c1\ud669\uc5d0\uc11c \uc288\ud305\uc744 \ud558\uc600\ub2e4. \uc6e8\uc2a4\ud2b8 \ud584\uc758 \uace8\ud0a4\ud37c \ud14c\ub4dc \ud5c8\ud504\ud2bc\uc774 \uc774 \uc29b\uc744 \ub9c9\uc9c0 \ubabb\ud558\uace0 \uc2e4\uc810\uc744 \ud5c8\uc6a9\ud574 \ubcfc\ud134\uc774 \uc55e\uc11c\ub098\uac00\uae30 \uc2dc\uc791\ud588\ub2e4. \uadf8\ub54c \uadf8 \uacf5\uc774 \ub2e4\ub978 \uad00\uc911\ub4e4\uc5d0 \ubc00\ub824\ub0b4\ub824\uc640 \uace8\ub124\ud2b8\uc5d0 \ubc14\uc9dd \ubd99\uc5b4\uc788\ub358 \ud55c \uad00\uc911\uc744 \uac15\ud0c0\ud558\uba74\uc11c \uadf8 \uad00\uc911\uc774 \ubb34\uc758\uc2dd \uc0c1\ud0dc\uac00 \ub418\uc5b4\ubc84\ub838\ub2e4. \uc2e4\uc810 3\ubd84 \ud6c4 \uc6e8\uc2a4\ud2b8 \ud584\uc758 \ube45 \uc653\uc2a8\uc774 \ubcfc\ud134\uc758 \uace8\ubb38 \ubc14\ub85c \uc55e\uc5d0\uc11c \uacf5\uc744 \uc7a1\uc544 \ucc9c\uae08\uac19\uc740 \uae30\ud68c\ub97c \uc7a1\uc558\uc73c\ub098 \uc29b\uc774 \ud06c\ub85c\uc2a4\ubc14\ub97c \ub118\uc5b4\uac00\ubc84\ub838\ub2e4. \uc804\ubc18 11\ubd84 \uacbd\uc5d0\ub294 \ub610\ub2e4\uc2dc \uad00\uc911\ub4e4\uc774 \uc55e\uc73c\ub85c \ubab0\ub9ac\uba74\uc11c \ub2e4\uc218\uc758 \ud32c\ub4e4\uc774 \uadf8\ub77c\uc6b4\ub4dc \uc548\uc73c\ub85c \uc9c4\uc785\ud558\ub294 \ubc14\ub78c\uc5d0 \ubd80\uc2ec\ub4e4\uc744 \uc704\ud574 \ub9d0\uc744 \ud0c4 \uacbd\ucc30\ub4e4\uc774 \uc0c1\ud669\uc744 \uc218\uc2b5\ud560 \ub54c\uae4c\uc9c0 \uacbd\uae30\uac00 \uc9c0\uc5f0\ub418\uc5c8\ub2e4. \uc120\uc218\ub4e4\uc774 \uacbd\uae30 \uc7ac\uac1c\ub97c \uae30\ub2e4\ub9ac\ub294 \ub3d9\uc548 \uba87\uba87 \uad00\uc911\ub4e4\uc740 \ub9ce\uc740 \uad00\uc911 \uc18d\uc5d0\uc11c \ub2f9\ud55c \ubd80\uc0c1 \ub54c\ubb38\uc5d0 \uc601\uad6d \uc801\uc2ed\uc790\uc758 \uc751\uae09\ucc98\uce58\uac00 \ud544\uc694\ud558\uae30\ub3c4 \ud588\ub2e4. \uacbd\ucc30\ub4e4\uc740 \uadf8\ub77c\uc6b4\ub4dc \uc8fc\ubcc0\uc744 \ub3cc\uba70 \ub354\uc774\uc0c1 \uad00\uc911\ub4e4\uc774 \uc120\uc744 \ub118\uc5b4 \uc9c4\uc785\ud558\uc9c0 \ubabb\ud558\ub3c4\ub85d \ud588\uace0 \uadf8 \uc0ac\uc774 \uacbd\uae30\uac00 \uc7ac\uac1c\ub418\uc5c8\ub2e4."} {"question": "\uce7c 10\uc138 \uad6c\uc2a4\ud0c0\ube0c\uac00 \ub77c\uc9c0\ube44\uc6b0 \uac00\ubb38\uc5d0\uac8c \ub9ac\ud22c\uc544\ub2c8\uc544 \uacf5\uad6d\uc758 \uc77c\ubd80\ub97c \ub5bc\uc5b4\uc8fc\uace0, \ub098\uba38\uc9c0\ub97c \uc2a4\uc6e8\ub374\uc758 \uc18d\uad6d\uc73c\ub85c \uc0bc\uc740 \uc0ac\uac74\uc744 \ubb34\uc5c7\uc774\ub77c\uace0 \ud558\ub294\uac00?", "paragraph": "\ub450\uba85\uc758 \ud3f4\ub780\ub4dc-\ub9ac\ud22c\uc544\ub2c8\uc544 \uadc0\uc871 \uc57c\ub204\uc2dc \ub77c\uc9c0\ube44\uc6b0(Janusz Radziwi\u0142\u0142) \uacf5\uacfc \ubcf4\uad6c\uc2a4\uc640\ud504 \ub77c\uc9c0\ube44\uc6b0(Bogus\u0142aw Radziwi\u0142\u0142) \uacf5\uc740 \ud63c\ub780\ud55c \uc5f0\ubc29\uc5d0 \ub0b4\ubd80\ubd84\uc7c1\uc744 \uc77c\uc73c\ucf30\uace0, \ud3f4\ub780\ub4dc-\ub9ac\ud22c\uc544\ub2c8\uc544 \uc5f0\ud569(Polish-Lithuanian union)\uccb4\uc81c\uc640 \uc5f0\ubc29\uc815\ubd80\ub97c \ubd95\uad34\uc2dc\ud0ac \ubaa9\uc801\uc73c\ub85c \uc2a4\uc6e8\ub374 \uc655 \uce7c 10\uc138 \uad6c\uc2a4\ud0c0\ube0c\uc640 \uad50\uc12d\uc744 \uac1c\uc2dc\ud588\ub2e4. \uadf8\ub4e4\uc740 \ucf00\ub2e4\uc774\ub124\uc774(K\u0117dainiai)\uc5d0\uc11c \ud569\ubcd1\uc5d0 \uad00\ud55c \uc870\uc57d\uc5d0 \uc870\uc778\ud588\uc73c\ub098, \uc774 \uc870\uc57d\uc5d0\ub294 \ub9ac\ud22c\uc544\ub2c8\uc544 \ub300\uacf5\uad6d\uc5d0\uc11c 2\uac1c\uc758 \uacf5\uad6d\uc744 \ubd84\ud560 \ub3c5\ub9bd\uc2dc\ucf1c \ub77c\uc9c0\ube44\uc6b0 \uac00\ubb38\uc758 \uacf5\ub4e4\uc5d0\uac8c \uc591\ub3c4\ud558\uace0, \ub300\uacf5\uad6d\ub839\uc758 \ub0a8\uc740 \uc9c0\uc5ed\uc740 \uc2a4\uc6e8\ub374\uc758 \uc18d\uad6d(vassal)\uc774 \ub41c\ub2e4\ub294 \uac83\uc774 \uc57d\uc18d\ub418\uc5c8\ub2e4(\ucf00\ub2e4\uc774\ub124\uc774 \ud569\ubcd1(Union of K\u0117dainiai).", "answer": "\ucf00\ub2e4\uc774\ub124\uc774 \ud569\ubcd1", "sentence": "\uadf8\ub4e4\uc740 \ucf00\ub2e4\uc774\ub124\uc774(K\u0117dainiai)\uc5d0\uc11c \ud569\ubcd1\uc5d0 \uad00\ud55c \uc870\uc57d\uc5d0 \uc870\uc778\ud588\uc73c\ub098, \uc774 \uc870\uc57d\uc5d0\ub294 \ub9ac\ud22c\uc544\ub2c8\uc544 \ub300\uacf5\uad6d\uc5d0\uc11c 2\uac1c\uc758 \uacf5\uad6d\uc744 \ubd84\ud560 \ub3c5\ub9bd\uc2dc\ucf1c \ub77c\uc9c0\ube44\uc6b0 \uac00\ubb38\uc758 \uacf5\ub4e4\uc5d0\uac8c \uc591\ub3c4\ud558\uace0, \ub300\uacf5\uad6d\ub839\uc758 \ub0a8\uc740 \uc9c0\uc5ed\uc740 \uc2a4\uc6e8\ub374\uc758 \uc18d\uad6d(vassal)\uc774 \ub41c\ub2e4\ub294 \uac83\uc774 \uc57d\uc18d\ub418\uc5c8\ub2e4( \ucf00\ub2e4\uc774\ub124\uc774 \ud569\ubcd1 (Union of K\u0117dainiai)", "paragraph_sentence": "\ub450\uba85\uc758 \ud3f4\ub780\ub4dc-\ub9ac\ud22c\uc544\ub2c8\uc544 \uadc0\uc871 \uc57c\ub204\uc2dc \ub77c\uc9c0\ube44\uc6b0(Janusz Radziwi\u0142\u0142) \uacf5\uacfc \ubcf4\uad6c\uc2a4\uc640\ud504 \ub77c\uc9c0\ube44\uc6b0(Bogus\u0142aw Radziwi\u0142\u0142) \uacf5\uc740 \ud63c\ub780\ud55c \uc5f0\ubc29\uc5d0 \ub0b4\ubd80\ubd84\uc7c1\uc744 \uc77c\uc73c\ucf30\uace0, \ud3f4\ub780\ub4dc-\ub9ac\ud22c\uc544\ub2c8\uc544 \uc5f0\ud569(Polish-Lithuanian union)\uccb4\uc81c\uc640 \uc5f0\ubc29\uc815\ubd80\ub97c \ubd95\uad34\uc2dc\ud0ac \ubaa9\uc801\uc73c\ub85c \uc2a4\uc6e8\ub374 \uc655 \uce7c 10\uc138 \uad6c\uc2a4\ud0c0\ube0c\uc640 \uad50\uc12d\uc744 \uac1c\uc2dc\ud588\ub2e4. \uadf8\ub4e4\uc740 \ucf00\ub2e4\uc774\ub124\uc774(K\u0117dainiai)\uc5d0\uc11c \ud569\ubcd1\uc5d0 \uad00\ud55c \uc870\uc57d\uc5d0 \uc870\uc778\ud588\uc73c\ub098, \uc774 \uc870\uc57d\uc5d0\ub294 \ub9ac\ud22c\uc544\ub2c8\uc544 \ub300\uacf5\uad6d\uc5d0\uc11c 2\uac1c\uc758 \uacf5\uad6d\uc744 \ubd84\ud560 \ub3c5\ub9bd\uc2dc\ucf1c \ub77c\uc9c0\ube44\uc6b0 \uac00\ubb38\uc758 \uacf5\ub4e4\uc5d0\uac8c \uc591\ub3c4\ud558\uace0, \ub300\uacf5\uad6d\ub839\uc758 \ub0a8\uc740 \uc9c0\uc5ed\uc740 \uc2a4\uc6e8\ub374\uc758 \uc18d\uad6d(vassal)\uc774 \ub41c\ub2e4\ub294 \uac83\uc774 \uc57d\uc18d\ub418\uc5c8\ub2e4( \ucf00\ub2e4\uc774\ub124\uc774 \ud569\ubcd1 (Union of K\u0117dainiai) .", "paragraph_answer": "\ub450\uba85\uc758 \ud3f4\ub780\ub4dc-\ub9ac\ud22c\uc544\ub2c8\uc544 \uadc0\uc871 \uc57c\ub204\uc2dc \ub77c\uc9c0\ube44\uc6b0(Janusz Radziwi\u0142\u0142) \uacf5\uacfc \ubcf4\uad6c\uc2a4\uc640\ud504 \ub77c\uc9c0\ube44\uc6b0(Bogus\u0142aw Radziwi\u0142\u0142) \uacf5\uc740 \ud63c\ub780\ud55c \uc5f0\ubc29\uc5d0 \ub0b4\ubd80\ubd84\uc7c1\uc744 \uc77c\uc73c\ucf30\uace0, \ud3f4\ub780\ub4dc-\ub9ac\ud22c\uc544\ub2c8\uc544 \uc5f0\ud569(Polish-Lithuanian union)\uccb4\uc81c\uc640 \uc5f0\ubc29\uc815\ubd80\ub97c \ubd95\uad34\uc2dc\ud0ac \ubaa9\uc801\uc73c\ub85c \uc2a4\uc6e8\ub374 \uc655 \uce7c 10\uc138 \uad6c\uc2a4\ud0c0\ube0c\uc640 \uad50\uc12d\uc744 \uac1c\uc2dc\ud588\ub2e4. \uadf8\ub4e4\uc740 \ucf00\ub2e4\uc774\ub124\uc774(K\u0117dainiai)\uc5d0\uc11c \ud569\ubcd1\uc5d0 \uad00\ud55c \uc870\uc57d\uc5d0 \uc870\uc778\ud588\uc73c\ub098, \uc774 \uc870\uc57d\uc5d0\ub294 \ub9ac\ud22c\uc544\ub2c8\uc544 \ub300\uacf5\uad6d\uc5d0\uc11c 2\uac1c\uc758 \uacf5\uad6d\uc744 \ubd84\ud560 \ub3c5\ub9bd\uc2dc\ucf1c \ub77c\uc9c0\ube44\uc6b0 \uac00\ubb38\uc758 \uacf5\ub4e4\uc5d0\uac8c \uc591\ub3c4\ud558\uace0, \ub300\uacf5\uad6d\ub839\uc758 \ub0a8\uc740 \uc9c0\uc5ed\uc740 \uc2a4\uc6e8\ub374\uc758 \uc18d\uad6d(vassal)\uc774 \ub41c\ub2e4\ub294 \uac83\uc774 \uc57d\uc18d\ub418\uc5c8\ub2e4( \ucf00\ub2e4\uc774\ub124\uc774 \ud569\ubcd1 (Union of K\u0117dainiai).", "sentence_answer": "\uadf8\ub4e4\uc740 \ucf00\ub2e4\uc774\ub124\uc774(K\u0117dainiai)\uc5d0\uc11c \ud569\ubcd1\uc5d0 \uad00\ud55c \uc870\uc57d\uc5d0 \uc870\uc778\ud588\uc73c\ub098, \uc774 \uc870\uc57d\uc5d0\ub294 \ub9ac\ud22c\uc544\ub2c8\uc544 \ub300\uacf5\uad6d\uc5d0\uc11c 2\uac1c\uc758 \uacf5\uad6d\uc744 \ubd84\ud560 \ub3c5\ub9bd\uc2dc\ucf1c \ub77c\uc9c0\ube44\uc6b0 \uac00\ubb38\uc758 \uacf5\ub4e4\uc5d0\uac8c \uc591\ub3c4\ud558\uace0, \ub300\uacf5\uad6d\ub839\uc758 \ub0a8\uc740 \uc9c0\uc5ed\uc740 \uc2a4\uc6e8\ub374\uc758 \uc18d\uad6d(vassal)\uc774 \ub41c\ub2e4\ub294 \uac83\uc774 \uc57d\uc18d\ub418\uc5c8\ub2e4( \ucf00\ub2e4\uc774\ub124\uc774 \ud569\ubcd1 (Union of K\u0117dainiai)", "paragraph_id": "6486681-2-1", "paragraph_question": "question: \uce7c 10\uc138 \uad6c\uc2a4\ud0c0\ube0c\uac00 \ub77c\uc9c0\ube44\uc6b0 \uac00\ubb38\uc5d0\uac8c \ub9ac\ud22c\uc544\ub2c8\uc544 \uacf5\uad6d\uc758 \uc77c\ubd80\ub97c \ub5bc\uc5b4\uc8fc\uace0, \ub098\uba38\uc9c0\ub97c \uc2a4\uc6e8\ub374\uc758 \uc18d\uad6d\uc73c\ub85c \uc0bc\uc740 \uc0ac\uac74\uc744 \ubb34\uc5c7\uc774\ub77c\uace0 \ud558\ub294\uac00?, context: \ub450\uba85\uc758 \ud3f4\ub780\ub4dc-\ub9ac\ud22c\uc544\ub2c8\uc544 \uadc0\uc871 \uc57c\ub204\uc2dc \ub77c\uc9c0\ube44\uc6b0(Janusz Radziwi\u0142\u0142) \uacf5\uacfc \ubcf4\uad6c\uc2a4\uc640\ud504 \ub77c\uc9c0\ube44\uc6b0(Bogus\u0142aw Radziwi\u0142\u0142) \uacf5\uc740 \ud63c\ub780\ud55c \uc5f0\ubc29\uc5d0 \ub0b4\ubd80\ubd84\uc7c1\uc744 \uc77c\uc73c\ucf30\uace0, \ud3f4\ub780\ub4dc-\ub9ac\ud22c\uc544\ub2c8\uc544 \uc5f0\ud569(Polish-Lithuanian union)\uccb4\uc81c\uc640 \uc5f0\ubc29\uc815\ubd80\ub97c \ubd95\uad34\uc2dc\ud0ac \ubaa9\uc801\uc73c\ub85c \uc2a4\uc6e8\ub374 \uc655 \uce7c 10\uc138 \uad6c\uc2a4\ud0c0\ube0c\uc640 \uad50\uc12d\uc744 \uac1c\uc2dc\ud588\ub2e4. \uadf8\ub4e4\uc740 \ucf00\ub2e4\uc774\ub124\uc774(K\u0117dainiai)\uc5d0\uc11c \ud569\ubcd1\uc5d0 \uad00\ud55c \uc870\uc57d\uc5d0 \uc870\uc778\ud588\uc73c\ub098, \uc774 \uc870\uc57d\uc5d0\ub294 \ub9ac\ud22c\uc544\ub2c8\uc544 \ub300\uacf5\uad6d\uc5d0\uc11c 2\uac1c\uc758 \uacf5\uad6d\uc744 \ubd84\ud560 \ub3c5\ub9bd\uc2dc\ucf1c \ub77c\uc9c0\ube44\uc6b0 \uac00\ubb38\uc758 \uacf5\ub4e4\uc5d0\uac8c \uc591\ub3c4\ud558\uace0, \ub300\uacf5\uad6d\ub839\uc758 \ub0a8\uc740 \uc9c0\uc5ed\uc740 \uc2a4\uc6e8\ub374\uc758 \uc18d\uad6d(vassal)\uc774 \ub41c\ub2e4\ub294 \uac83\uc774 \uc57d\uc18d\ub418\uc5c8\ub2e4(\ucf00\ub2e4\uc774\ub124\uc774 \ud569\ubcd1(Union of K\u0117dainiai)."} {"question": "1978\ub144 10\uc6d4 \uc774\ub8e8\uc5b4\uc84c\ub358 \ubb34\uc5c7\uc73c\ub85c \uc778\ud574 \uc57c\uac04\uc5f4\ucc28 \uc678\uc758 \ubaa8\ub4e0 \ubcf4\ud1b5 \uc5f4\ucc28\uac00 \uc815\ucc28\ud558\uac8c \ub418\uc5c8\ub294\uac00?", "paragraph": "\uadf8\ub9ac\ud558\uc5ec 1955\ub144\ubd80\ud130 \uacc4\uc18d\ub418\uc5b4 \uc628 \uc8fc\ubbfc\ub4e4\uc758 \uc9c4\uc815\uc73c\ub85c 1959\ub144 1\uc6d4 \uc77c\ubcf8\uad6d\uc720\ucca0\ub3c4\ub294 \uc544\ub9c8\ub8e8\ubca0 \ucd0c \ubd80\uadfc\uc5d0\uc11c \ubc14\ub85c \uc774\uc6a9\ud560 \uc218 \uc788\ub294 \uc5ed\uc744 \uc124\uce58\ud558\uae30\ub85c \uacb0\uc815\ud588\ub2e4. \uc774 \uacfc\uc815\uc5d0\uc11c \ub9c8\uc744\uc758 \uc544\uc774\ub4e4\uc774 \uc5ed\uae4c\uc9c0 \uc774\uc5b4\uc9c0\ub294 \uc9c4\uc785\ub85c\uc640 \uc2b9\uac15\uc7a5\uc744 \ub9cc\ub4dc\ub294 \ub370 \ud544\uc694\ud55c \ub3cc\ub4e4\uc744 \ubc14\ub2f7\uac00\uc5d0\uc11c \uce90\ub0b4\uc624\ub294 \uac83\uc744 \uac70\ub4e4\uae30\ub3c4 \ud588\ub2e4. 1959\ub144 4\uc6d4 16\uc77c \uc544\ub9c8\ub8e8\ubca0 \uc5ed\uc774 \ub2e4\ub9ac\uc758 \uc11c\ucabd\uc5d0 \uac1c\uc5c5\ud588\ub2e4. \uac1c\uc5c5\ud560 \ubb34\ub835\uc5d0\ub294 \ud1b5\uadfc \uc2dc\uac04\ub300\uc5d0\ub9cc \uc5f4\ucc28\ub4e4\uc774 \uc130\uc73c\uba70 \ub0ae \uc2dc\uac04\ub300\uc5d0\ub294 \ubaa8\ub4e0 \uc5f4\ucc28\uac00 \ud1b5\uacfc\ud588\ub2e4. \uadf8\ub7ec\ub098 \ucc28\ucc28 \uc815\ucc28 \ud3b8\uc218\uac00 \ub298\uc5b4\ub098\ub2e4\uac00 1978\ub144 10\uc6d4\uc5d0 \uc774\ub8e8\uc5b4\uc9c4 \uc2dc\uac04\ud45c \uac1c\uc815 \ub54c\ubd80\ud130 \uc57c\uac04\uc5f4\ucc28\ub97c \uc81c\uc678\ud55c \ubaa8\ub4e0 \ubcf4\ud1b5 \uc5f4\ucc28\uac00 \uc815\ucc28\ud558\uac8c \ub418\uc5c8\ub2e4(1985\ub144 3\uc6d4 \uc2dc\uac04\ud45c \uac1c\uc815\uc73c\ub85c \uc57c\uac04 \ubcf4\ud1b5 \uc5f4\ucc28\ub294 \ud3d0\uc9c0\ub418\uc5c8\ub2e4.).", "answer": "\uc2dc\uac04\ud45c \uac1c\uc815", "sentence": "\uadf8\ub7ec\ub098 \ucc28\ucc28 \uc815\ucc28 \ud3b8\uc218\uac00 \ub298\uc5b4\ub098\ub2e4\uac00 1978\ub144 10\uc6d4\uc5d0 \uc774\ub8e8\uc5b4\uc9c4 \uc2dc\uac04\ud45c \uac1c\uc815 \ub54c\ubd80\ud130 \uc57c\uac04\uc5f4\ucc28\ub97c \uc81c\uc678\ud55c \ubaa8\ub4e0 \ubcf4\ud1b5 \uc5f4\ucc28\uac00 \uc815\ucc28\ud558\uac8c \ub418\uc5c8\ub2e4(1985\ub144 3\uc6d4 \uc2dc\uac04\ud45c \uac1c\uc815\uc73c\ub85c \uc57c\uac04 \ubcf4\ud1b5 \uc5f4\ucc28\ub294 \ud3d0\uc9c0\ub418\uc5c8\ub2e4.", "paragraph_sentence": "\uadf8\ub9ac\ud558\uc5ec 1955\ub144\ubd80\ud130 \uacc4\uc18d\ub418\uc5b4 \uc628 \uc8fc\ubbfc\ub4e4\uc758 \uc9c4\uc815\uc73c\ub85c 1959\ub144 1\uc6d4 \uc77c\ubcf8\uad6d\uc720\ucca0\ub3c4\ub294 \uc544\ub9c8\ub8e8\ubca0 \ucd0c \ubd80\uadfc\uc5d0\uc11c \ubc14\ub85c \uc774\uc6a9\ud560 \uc218 \uc788\ub294 \uc5ed\uc744 \uc124\uce58\ud558\uae30\ub85c \uacb0\uc815\ud588\ub2e4. \uc774 \uacfc\uc815\uc5d0\uc11c \ub9c8\uc744\uc758 \uc544\uc774\ub4e4\uc774 \uc5ed\uae4c\uc9c0 \uc774\uc5b4\uc9c0\ub294 \uc9c4\uc785\ub85c\uc640 \uc2b9\uac15\uc7a5\uc744 \ub9cc\ub4dc\ub294 \ub370 \ud544\uc694\ud55c \ub3cc\ub4e4\uc744 \ubc14\ub2f7\uac00\uc5d0\uc11c \uce90\ub0b4\uc624\ub294 \uac83\uc744 \uac70\ub4e4\uae30\ub3c4 \ud588\ub2e4. 1959\ub144 4\uc6d4 16\uc77c \uc544\ub9c8\ub8e8\ubca0 \uc5ed\uc774 \ub2e4\ub9ac\uc758 \uc11c\ucabd\uc5d0 \uac1c\uc5c5\ud588\ub2e4. \uac1c\uc5c5\ud560 \ubb34\ub835\uc5d0\ub294 \ud1b5\uadfc \uc2dc\uac04\ub300\uc5d0\ub9cc \uc5f4\ucc28\ub4e4\uc774 \uc130\uc73c\uba70 \ub0ae \uc2dc\uac04\ub300\uc5d0\ub294 \ubaa8\ub4e0 \uc5f4\ucc28\uac00 \ud1b5\uacfc\ud588\ub2e4. \uadf8\ub7ec\ub098 \ucc28\ucc28 \uc815\ucc28 \ud3b8\uc218\uac00 \ub298\uc5b4\ub098\ub2e4\uac00 1978\ub144 10\uc6d4\uc5d0 \uc774\ub8e8\uc5b4\uc9c4 \uc2dc\uac04\ud45c \uac1c\uc815 \ub54c\ubd80\ud130 \uc57c\uac04\uc5f4\ucc28\ub97c \uc81c\uc678\ud55c \ubaa8\ub4e0 \ubcf4\ud1b5 \uc5f4\ucc28\uac00 \uc815\ucc28\ud558\uac8c \ub418\uc5c8\ub2e4(1985\ub144 3\uc6d4 \uc2dc\uac04\ud45c \uac1c\uc815\uc73c\ub85c \uc57c\uac04 \ubcf4\ud1b5 \uc5f4\ucc28\ub294 \ud3d0\uc9c0\ub418\uc5c8\ub2e4. ) .", "paragraph_answer": "\uadf8\ub9ac\ud558\uc5ec 1955\ub144\ubd80\ud130 \uacc4\uc18d\ub418\uc5b4 \uc628 \uc8fc\ubbfc\ub4e4\uc758 \uc9c4\uc815\uc73c\ub85c 1959\ub144 1\uc6d4 \uc77c\ubcf8\uad6d\uc720\ucca0\ub3c4\ub294 \uc544\ub9c8\ub8e8\ubca0 \ucd0c \ubd80\uadfc\uc5d0\uc11c \ubc14\ub85c \uc774\uc6a9\ud560 \uc218 \uc788\ub294 \uc5ed\uc744 \uc124\uce58\ud558\uae30\ub85c \uacb0\uc815\ud588\ub2e4. \uc774 \uacfc\uc815\uc5d0\uc11c \ub9c8\uc744\uc758 \uc544\uc774\ub4e4\uc774 \uc5ed\uae4c\uc9c0 \uc774\uc5b4\uc9c0\ub294 \uc9c4\uc785\ub85c\uc640 \uc2b9\uac15\uc7a5\uc744 \ub9cc\ub4dc\ub294 \ub370 \ud544\uc694\ud55c \ub3cc\ub4e4\uc744 \ubc14\ub2f7\uac00\uc5d0\uc11c \uce90\ub0b4\uc624\ub294 \uac83\uc744 \uac70\ub4e4\uae30\ub3c4 \ud588\ub2e4. 1959\ub144 4\uc6d4 16\uc77c \uc544\ub9c8\ub8e8\ubca0 \uc5ed\uc774 \ub2e4\ub9ac\uc758 \uc11c\ucabd\uc5d0 \uac1c\uc5c5\ud588\ub2e4. \uac1c\uc5c5\ud560 \ubb34\ub835\uc5d0\ub294 \ud1b5\uadfc \uc2dc\uac04\ub300\uc5d0\ub9cc \uc5f4\ucc28\ub4e4\uc774 \uc130\uc73c\uba70 \ub0ae \uc2dc\uac04\ub300\uc5d0\ub294 \ubaa8\ub4e0 \uc5f4\ucc28\uac00 \ud1b5\uacfc\ud588\ub2e4. \uadf8\ub7ec\ub098 \ucc28\ucc28 \uc815\ucc28 \ud3b8\uc218\uac00 \ub298\uc5b4\ub098\ub2e4\uac00 1978\ub144 10\uc6d4\uc5d0 \uc774\ub8e8\uc5b4\uc9c4 \uc2dc\uac04\ud45c \uac1c\uc815 \ub54c\ubd80\ud130 \uc57c\uac04\uc5f4\ucc28\ub97c \uc81c\uc678\ud55c \ubaa8\ub4e0 \ubcf4\ud1b5 \uc5f4\ucc28\uac00 \uc815\ucc28\ud558\uac8c \ub418\uc5c8\ub2e4(1985\ub144 3\uc6d4 \uc2dc\uac04\ud45c \uac1c\uc815\uc73c\ub85c \uc57c\uac04 \ubcf4\ud1b5 \uc5f4\ucc28\ub294 \ud3d0\uc9c0\ub418\uc5c8\ub2e4.).", "sentence_answer": "\uadf8\ub7ec\ub098 \ucc28\ucc28 \uc815\ucc28 \ud3b8\uc218\uac00 \ub298\uc5b4\ub098\ub2e4\uac00 1978\ub144 10\uc6d4\uc5d0 \uc774\ub8e8\uc5b4\uc9c4 \uc2dc\uac04\ud45c \uac1c\uc815 \ub54c\ubd80\ud130 \uc57c\uac04\uc5f4\ucc28\ub97c \uc81c\uc678\ud55c \ubaa8\ub4e0 \ubcf4\ud1b5 \uc5f4\ucc28\uac00 \uc815\ucc28\ud558\uac8c \ub418\uc5c8\ub2e4(1985\ub144 3\uc6d4 \uc2dc\uac04\ud45c \uac1c\uc815\uc73c\ub85c \uc57c\uac04 \ubcf4\ud1b5 \uc5f4\ucc28\ub294 \ud3d0\uc9c0\ub418\uc5c8\ub2e4.", "paragraph_id": "6533582-13-1", "paragraph_question": "question: 1978\ub144 10\uc6d4 \uc774\ub8e8\uc5b4\uc84c\ub358 \ubb34\uc5c7\uc73c\ub85c \uc778\ud574 \uc57c\uac04\uc5f4\ucc28 \uc678\uc758 \ubaa8\ub4e0 \ubcf4\ud1b5 \uc5f4\ucc28\uac00 \uc815\ucc28\ud558\uac8c \ub418\uc5c8\ub294\uac00?, context: \uadf8\ub9ac\ud558\uc5ec 1955\ub144\ubd80\ud130 \uacc4\uc18d\ub418\uc5b4 \uc628 \uc8fc\ubbfc\ub4e4\uc758 \uc9c4\uc815\uc73c\ub85c 1959\ub144 1\uc6d4 \uc77c\ubcf8\uad6d\uc720\ucca0\ub3c4\ub294 \uc544\ub9c8\ub8e8\ubca0 \ucd0c \ubd80\uadfc\uc5d0\uc11c \ubc14\ub85c \uc774\uc6a9\ud560 \uc218 \uc788\ub294 \uc5ed\uc744 \uc124\uce58\ud558\uae30\ub85c \uacb0\uc815\ud588\ub2e4. \uc774 \uacfc\uc815\uc5d0\uc11c \ub9c8\uc744\uc758 \uc544\uc774\ub4e4\uc774 \uc5ed\uae4c\uc9c0 \uc774\uc5b4\uc9c0\ub294 \uc9c4\uc785\ub85c\uc640 \uc2b9\uac15\uc7a5\uc744 \ub9cc\ub4dc\ub294 \ub370 \ud544\uc694\ud55c \ub3cc\ub4e4\uc744 \ubc14\ub2f7\uac00\uc5d0\uc11c \uce90\ub0b4\uc624\ub294 \uac83\uc744 \uac70\ub4e4\uae30\ub3c4 \ud588\ub2e4. 1959\ub144 4\uc6d4 16\uc77c \uc544\ub9c8\ub8e8\ubca0 \uc5ed\uc774 \ub2e4\ub9ac\uc758 \uc11c\ucabd\uc5d0 \uac1c\uc5c5\ud588\ub2e4. \uac1c\uc5c5\ud560 \ubb34\ub835\uc5d0\ub294 \ud1b5\uadfc \uc2dc\uac04\ub300\uc5d0\ub9cc \uc5f4\ucc28\ub4e4\uc774 \uc130\uc73c\uba70 \ub0ae \uc2dc\uac04\ub300\uc5d0\ub294 \ubaa8\ub4e0 \uc5f4\ucc28\uac00 \ud1b5\uacfc\ud588\ub2e4. \uadf8\ub7ec\ub098 \ucc28\ucc28 \uc815\ucc28 \ud3b8\uc218\uac00 \ub298\uc5b4\ub098\ub2e4\uac00 1978\ub144 10\uc6d4\uc5d0 \uc774\ub8e8\uc5b4\uc9c4 \uc2dc\uac04\ud45c \uac1c\uc815 \ub54c\ubd80\ud130 \uc57c\uac04\uc5f4\ucc28\ub97c \uc81c\uc678\ud55c \ubaa8\ub4e0 \ubcf4\ud1b5 \uc5f4\ucc28\uac00 \uc815\ucc28\ud558\uac8c \ub418\uc5c8\ub2e4(1985\ub144 3\uc6d4 \uc2dc\uac04\ud45c \uac1c\uc815\uc73c\ub85c \uc57c\uac04 \ubcf4\ud1b5 \uc5f4\ucc28\ub294 \ud3d0\uc9c0\ub418\uc5c8\ub2e4.)."} {"question": "\uc18c\ub828 \uad70\uc815\uccad \uc0ac\ub839\uad00\uc740 \ub204\uad6c\uc600\ub098?", "paragraph": "1946\ub144 3\uc6d4 1\uc77c '3.1.\uae30\ubbf8\ub144\ub3c5\ub9bd\uc120\uc5b8 \uae30\ub150\uc0ac\uc5c5\ud68c' \uace0\ubb38\uc5d0 \ucde8\uc784\ud558\uc600\uace0, 3\uc6d4\uc5d0 \ubbf8\u00b7\uc18c\uacf5\uc704\uac00 \uac1c\ucd5c \uc608\uc815\ub418\uc790, \uadf8\ub294 \ubbf8\uc18c\uacf5\ub3d9\uc704\uc6d0\ud68c\uc758 \uc131\uacf5\uc744 \uae30\uc6d0\ud558\ub294 \uc131\uba85\uc11c\ub97c \ubc1c\ud45c\ud588\ub2e4. 3\uc6d4 7\uc77c \uc18c\ub828 \uad70\uc815\uccad \uc0ac\ub839\uad00 \uc2a4\ud2f0\ucf54\ud504\uac00 \uc9e0 \ucd08\uc548\uc5d0\uc11c \uadf8\ub294 \ud5a5\ud6c4 \uc218\ub9bd\ub420 \uc815\ubd80\uc758 \ubd80\uc218\uc0c1\uc73c\ub85c \uc9c0\ubaa9\ub418\uae30\ub3c4 \ud588\ub2e4. 1946\ub144 3\uc6d4 19\uc77c \ubbfc\uc8fc\uc758\uc6d0 \uc758\uc7a5 \uc9c1\ubb34\ub300\ub9ac\uc5d0 \uc120\ucd9c, 3\uc6d4 20\uc77c \ubbf8\uc18c\uacf5\uc704 \uac1c\ucd5c\ub54c \ubbf8\uad6d\uc740 \uc774\uc2b9\ub9cc\uc758 \ubc18\uc18c\ubc18\ud0c1\uc73c\ub85c \uc778\ud574, \uc774\uc2b9\ub9cc\uc774 \ud68c\uc758 \uc804\uba74\uc5d0 \ub098\uc624\ub294 \uac83\uc744 \uaebc\ub838\uace0, \uae40\uaddc\uc2dd\uc774 \ub300\uc2e0 \ubbfc\uc8fc\uc758\uc6d0 \uc758\uc7a5\ub300\ub9ac\ub85c \ubbf8\uc18c\uacf5\uc704\uc5d0 \ucc38\uc11d\ud558\uc600\ub2e4. 46\ub144 3\uc6d4 24\uc77c \ubbf8\uc18c\uacf5\uc704\uc5d0 \ub300\ucc98\ud558\uae30 \uc704\ud55c \ubbfc\uc8fc\uc758\uc6d0\ub0b4 \ubbf8\uc18c\uacf5\uc704 \uad50\uc12d\ub2e8\uc744 \uad6c\uc131\ud558\uc5ec \ub2e8\uc7a5\uc5d0 \ucde8\uc784\ud588\ub2e4. 3\uc6d4 19\uc77c \uc774\uc2b9\ub9cc\uc774 \ubbfc\uc8fc\uc758\uc6d0 \uc758\uc7a5\uc9c1\uc744 \uc0ac\ud1f4\ud588\ub2e4. \ud558\uac8c \ub418\uc5b4 \uae40\uaddc\uc2dd\uc740 \ubbfc\uc8fc\uc758\uc6d0 \uc758\uc7a5 \ub300\ub9ac\uc5d0 \ucde8\uc784\ud588\ub2e4. \uc774\uc2b9\ub9cc\uc758 \uc0ac\ud1f4 \uc774\uc720\ub294 \ud45c\uba74\uc0c1 \uc774\uc720\ub294 \uac74\uac15\ubb38\uc81c\uc600\uc9c0\ub9cc, \uc0ac\uc2e4\uc740 \ubbf8\uad70\uc815\uc774 \ubbf8\uc18c\uacf5\uc704 \uac1c\ucd5c\ub97c \uc55e\ub450\uace0 \ucca0\uc800\ud55c \ubc18\uc18c(\u53cd\u8607)\uc8fc\uc758\uc790\uc774\uba70 \uc2e0\ud0c1\ud1b5\uce58\uc548\uc744 \uaca9\ub82c\ud788 \ube44\ud310\ud574 \uc628 \uc774\uc2b9\ub9cc\uc744 \uc815\uce58 \uc77c\uc120\uc5d0\uc11c \ubc30\uc81c\ud560 \ud544\uc694\ub97c \ub290\uaf08\uae30 \ub54c\ubb38\uc774\uc5c8\ub2e4. \ubbf8\uad70\uc815\uc740 \uc758\uc7a5 \ub300\ub9ac\uc5d0 \uae40\uaddc\uc2dd\uc744 \uc549\ud614\ub358 \uac83\uc774\ub2e4.", "answer": "\uc2a4\ud2f0\ucf54\ud504", "sentence": "3\uc6d4 7\uc77c \uc18c\ub828 \uad70\uc815\uccad \uc0ac\ub839\uad00 \uc2a4\ud2f0\ucf54\ud504 \uac00 \uc9e0 \ucd08\uc548\uc5d0\uc11c \uadf8\ub294 \ud5a5\ud6c4 \uc218\ub9bd\ub420 \uc815\ubd80\uc758 \ubd80\uc218\uc0c1\uc73c\ub85c \uc9c0\ubaa9\ub418\uae30\ub3c4 \ud588\ub2e4.", "paragraph_sentence": "1946\ub144 3\uc6d4 1\uc77c '3.1.\uae30\ubbf8\ub144\ub3c5\ub9bd\uc120\uc5b8 \uae30\ub150\uc0ac\uc5c5\ud68c' \uace0\ubb38\uc5d0 \ucde8\uc784\ud558\uc600\uace0, 3\uc6d4\uc5d0 \ubbf8\u00b7\uc18c\uacf5\uc704\uac00 \uac1c\ucd5c \uc608\uc815\ub418\uc790, \uadf8\ub294 \ubbf8\uc18c\uacf5\ub3d9\uc704\uc6d0\ud68c\uc758 \uc131\uacf5\uc744 \uae30\uc6d0\ud558\ub294 \uc131\uba85\uc11c\ub97c \ubc1c\ud45c\ud588\ub2e4. 3\uc6d4 7\uc77c \uc18c\ub828 \uad70\uc815\uccad \uc0ac\ub839\uad00 \uc2a4\ud2f0\ucf54\ud504 \uac00 \uc9e0 \ucd08\uc548\uc5d0\uc11c \uadf8\ub294 \ud5a5\ud6c4 \uc218\ub9bd\ub420 \uc815\ubd80\uc758 \ubd80\uc218\uc0c1\uc73c\ub85c \uc9c0\ubaa9\ub418\uae30\ub3c4 \ud588\ub2e4. 1946\ub144 3\uc6d4 19\uc77c \ubbfc\uc8fc\uc758\uc6d0 \uc758\uc7a5 \uc9c1\ubb34\ub300\ub9ac\uc5d0 \uc120\ucd9c, 3\uc6d4 20\uc77c \ubbf8\uc18c\uacf5\uc704 \uac1c\ucd5c\ub54c \ubbf8\uad6d\uc740 \uc774\uc2b9\ub9cc\uc758 \ubc18\uc18c\ubc18\ud0c1\uc73c\ub85c \uc778\ud574, \uc774\uc2b9\ub9cc\uc774 \ud68c\uc758 \uc804\uba74\uc5d0 \ub098\uc624\ub294 \uac83\uc744 \uaebc\ub838\uace0, \uae40\uaddc\uc2dd\uc774 \ub300\uc2e0 \ubbfc\uc8fc\uc758\uc6d0 \uc758\uc7a5\ub300\ub9ac\ub85c \ubbf8\uc18c\uacf5\uc704\uc5d0 \ucc38\uc11d\ud558\uc600\ub2e4. 46\ub144 3\uc6d4 24\uc77c \ubbf8\uc18c\uacf5\uc704\uc5d0 \ub300\ucc98\ud558\uae30 \uc704\ud55c \ubbfc\uc8fc\uc758\uc6d0\ub0b4 \ubbf8\uc18c\uacf5\uc704 \uad50\uc12d\ub2e8\uc744 \uad6c\uc131\ud558\uc5ec \ub2e8\uc7a5\uc5d0 \ucde8\uc784\ud588\ub2e4. 3\uc6d4 19\uc77c \uc774\uc2b9\ub9cc\uc774 \ubbfc\uc8fc\uc758\uc6d0 \uc758\uc7a5\uc9c1\uc744 \uc0ac\ud1f4\ud588\ub2e4. \ud558\uac8c \ub418\uc5b4 \uae40\uaddc\uc2dd\uc740 \ubbfc\uc8fc\uc758\uc6d0 \uc758\uc7a5 \ub300\ub9ac\uc5d0 \ucde8\uc784\ud588\ub2e4. \uc774\uc2b9\ub9cc\uc758 \uc0ac\ud1f4 \uc774\uc720\ub294 \ud45c\uba74\uc0c1 \uc774\uc720\ub294 \uac74\uac15\ubb38\uc81c\uc600\uc9c0\ub9cc, \uc0ac\uc2e4\uc740 \ubbf8\uad70\uc815\uc774 \ubbf8\uc18c\uacf5\uc704 \uac1c\ucd5c\ub97c \uc55e\ub450\uace0 \ucca0\uc800\ud55c \ubc18\uc18c(\u53cd\u8607)\uc8fc\uc758\uc790\uc774\uba70 \uc2e0\ud0c1\ud1b5\uce58\uc548\uc744 \uaca9\ub82c\ud788 \ube44\ud310\ud574 \uc628 \uc774\uc2b9\ub9cc\uc744 \uc815\uce58 \uc77c\uc120\uc5d0\uc11c \ubc30\uc81c\ud560 \ud544\uc694\ub97c \ub290\uaf08\uae30 \ub54c\ubb38\uc774\uc5c8\ub2e4. \ubbf8\uad70\uc815\uc740 \uc758\uc7a5 \ub300\ub9ac\uc5d0 \uae40\uaddc\uc2dd\uc744 \uc549\ud614\ub358 \uac83\uc774\ub2e4.", "paragraph_answer": "1946\ub144 3\uc6d4 1\uc77c '3.1.\uae30\ubbf8\ub144\ub3c5\ub9bd\uc120\uc5b8 \uae30\ub150\uc0ac\uc5c5\ud68c' \uace0\ubb38\uc5d0 \ucde8\uc784\ud558\uc600\uace0, 3\uc6d4\uc5d0 \ubbf8\u00b7\uc18c\uacf5\uc704\uac00 \uac1c\ucd5c \uc608\uc815\ub418\uc790, \uadf8\ub294 \ubbf8\uc18c\uacf5\ub3d9\uc704\uc6d0\ud68c\uc758 \uc131\uacf5\uc744 \uae30\uc6d0\ud558\ub294 \uc131\uba85\uc11c\ub97c \ubc1c\ud45c\ud588\ub2e4. 3\uc6d4 7\uc77c \uc18c\ub828 \uad70\uc815\uccad \uc0ac\ub839\uad00 \uc2a4\ud2f0\ucf54\ud504 \uac00 \uc9e0 \ucd08\uc548\uc5d0\uc11c \uadf8\ub294 \ud5a5\ud6c4 \uc218\ub9bd\ub420 \uc815\ubd80\uc758 \ubd80\uc218\uc0c1\uc73c\ub85c \uc9c0\ubaa9\ub418\uae30\ub3c4 \ud588\ub2e4. 1946\ub144 3\uc6d4 19\uc77c \ubbfc\uc8fc\uc758\uc6d0 \uc758\uc7a5 \uc9c1\ubb34\ub300\ub9ac\uc5d0 \uc120\ucd9c, 3\uc6d4 20\uc77c \ubbf8\uc18c\uacf5\uc704 \uac1c\ucd5c\ub54c \ubbf8\uad6d\uc740 \uc774\uc2b9\ub9cc\uc758 \ubc18\uc18c\ubc18\ud0c1\uc73c\ub85c \uc778\ud574, \uc774\uc2b9\ub9cc\uc774 \ud68c\uc758 \uc804\uba74\uc5d0 \ub098\uc624\ub294 \uac83\uc744 \uaebc\ub838\uace0, \uae40\uaddc\uc2dd\uc774 \ub300\uc2e0 \ubbfc\uc8fc\uc758\uc6d0 \uc758\uc7a5\ub300\ub9ac\ub85c \ubbf8\uc18c\uacf5\uc704\uc5d0 \ucc38\uc11d\ud558\uc600\ub2e4. 46\ub144 3\uc6d4 24\uc77c \ubbf8\uc18c\uacf5\uc704\uc5d0 \ub300\ucc98\ud558\uae30 \uc704\ud55c \ubbfc\uc8fc\uc758\uc6d0\ub0b4 \ubbf8\uc18c\uacf5\uc704 \uad50\uc12d\ub2e8\uc744 \uad6c\uc131\ud558\uc5ec \ub2e8\uc7a5\uc5d0 \ucde8\uc784\ud588\ub2e4. 3\uc6d4 19\uc77c \uc774\uc2b9\ub9cc\uc774 \ubbfc\uc8fc\uc758\uc6d0 \uc758\uc7a5\uc9c1\uc744 \uc0ac\ud1f4\ud588\ub2e4. \ud558\uac8c \ub418\uc5b4 \uae40\uaddc\uc2dd\uc740 \ubbfc\uc8fc\uc758\uc6d0 \uc758\uc7a5 \ub300\ub9ac\uc5d0 \ucde8\uc784\ud588\ub2e4. \uc774\uc2b9\ub9cc\uc758 \uc0ac\ud1f4 \uc774\uc720\ub294 \ud45c\uba74\uc0c1 \uc774\uc720\ub294 \uac74\uac15\ubb38\uc81c\uc600\uc9c0\ub9cc, \uc0ac\uc2e4\uc740 \ubbf8\uad70\uc815\uc774 \ubbf8\uc18c\uacf5\uc704 \uac1c\ucd5c\ub97c \uc55e\ub450\uace0 \ucca0\uc800\ud55c \ubc18\uc18c(\u53cd\u8607)\uc8fc\uc758\uc790\uc774\uba70 \uc2e0\ud0c1\ud1b5\uce58\uc548\uc744 \uaca9\ub82c\ud788 \ube44\ud310\ud574 \uc628 \uc774\uc2b9\ub9cc\uc744 \uc815\uce58 \uc77c\uc120\uc5d0\uc11c \ubc30\uc81c\ud560 \ud544\uc694\ub97c \ub290\uaf08\uae30 \ub54c\ubb38\uc774\uc5c8\ub2e4. \ubbf8\uad70\uc815\uc740 \uc758\uc7a5 \ub300\ub9ac\uc5d0 \uae40\uaddc\uc2dd\uc744 \uc549\ud614\ub358 \uac83\uc774\ub2e4.", "sentence_answer": "3\uc6d4 7\uc77c \uc18c\ub828 \uad70\uc815\uccad \uc0ac\ub839\uad00 \uc2a4\ud2f0\ucf54\ud504 \uac00 \uc9e0 \ucd08\uc548\uc5d0\uc11c \uadf8\ub294 \ud5a5\ud6c4 \uc218\ub9bd\ub420 \uc815\ubd80\uc758 \ubd80\uc218\uc0c1\uc73c\ub85c \uc9c0\ubaa9\ub418\uae30\ub3c4 \ud588\ub2e4.", "paragraph_id": "5991262-28-1", "paragraph_question": "question: \uc18c\ub828 \uad70\uc815\uccad \uc0ac\ub839\uad00\uc740 \ub204\uad6c\uc600\ub098?, context: 1946\ub144 3\uc6d4 1\uc77c '3.1.\uae30\ubbf8\ub144\ub3c5\ub9bd\uc120\uc5b8 \uae30\ub150\uc0ac\uc5c5\ud68c' \uace0\ubb38\uc5d0 \ucde8\uc784\ud558\uc600\uace0, 3\uc6d4\uc5d0 \ubbf8\u00b7\uc18c\uacf5\uc704\uac00 \uac1c\ucd5c \uc608\uc815\ub418\uc790, \uadf8\ub294 \ubbf8\uc18c\uacf5\ub3d9\uc704\uc6d0\ud68c\uc758 \uc131\uacf5\uc744 \uae30\uc6d0\ud558\ub294 \uc131\uba85\uc11c\ub97c \ubc1c\ud45c\ud588\ub2e4. 3\uc6d4 7\uc77c \uc18c\ub828 \uad70\uc815\uccad \uc0ac\ub839\uad00 \uc2a4\ud2f0\ucf54\ud504\uac00 \uc9e0 \ucd08\uc548\uc5d0\uc11c \uadf8\ub294 \ud5a5\ud6c4 \uc218\ub9bd\ub420 \uc815\ubd80\uc758 \ubd80\uc218\uc0c1\uc73c\ub85c \uc9c0\ubaa9\ub418\uae30\ub3c4 \ud588\ub2e4. 1946\ub144 3\uc6d4 19\uc77c \ubbfc\uc8fc\uc758\uc6d0 \uc758\uc7a5 \uc9c1\ubb34\ub300\ub9ac\uc5d0 \uc120\ucd9c, 3\uc6d4 20\uc77c \ubbf8\uc18c\uacf5\uc704 \uac1c\ucd5c\ub54c \ubbf8\uad6d\uc740 \uc774\uc2b9\ub9cc\uc758 \ubc18\uc18c\ubc18\ud0c1\uc73c\ub85c \uc778\ud574, \uc774\uc2b9\ub9cc\uc774 \ud68c\uc758 \uc804\uba74\uc5d0 \ub098\uc624\ub294 \uac83\uc744 \uaebc\ub838\uace0, \uae40\uaddc\uc2dd\uc774 \ub300\uc2e0 \ubbfc\uc8fc\uc758\uc6d0 \uc758\uc7a5\ub300\ub9ac\ub85c \ubbf8\uc18c\uacf5\uc704\uc5d0 \ucc38\uc11d\ud558\uc600\ub2e4. 46\ub144 3\uc6d4 24\uc77c \ubbf8\uc18c\uacf5\uc704\uc5d0 \ub300\ucc98\ud558\uae30 \uc704\ud55c \ubbfc\uc8fc\uc758\uc6d0\ub0b4 \ubbf8\uc18c\uacf5\uc704 \uad50\uc12d\ub2e8\uc744 \uad6c\uc131\ud558\uc5ec \ub2e8\uc7a5\uc5d0 \ucde8\uc784\ud588\ub2e4. 3\uc6d4 19\uc77c \uc774\uc2b9\ub9cc\uc774 \ubbfc\uc8fc\uc758\uc6d0 \uc758\uc7a5\uc9c1\uc744 \uc0ac\ud1f4\ud588\ub2e4. \ud558\uac8c \ub418\uc5b4 \uae40\uaddc\uc2dd\uc740 \ubbfc\uc8fc\uc758\uc6d0 \uc758\uc7a5 \ub300\ub9ac\uc5d0 \ucde8\uc784\ud588\ub2e4. \uc774\uc2b9\ub9cc\uc758 \uc0ac\ud1f4 \uc774\uc720\ub294 \ud45c\uba74\uc0c1 \uc774\uc720\ub294 \uac74\uac15\ubb38\uc81c\uc600\uc9c0\ub9cc, \uc0ac\uc2e4\uc740 \ubbf8\uad70\uc815\uc774 \ubbf8\uc18c\uacf5\uc704 \uac1c\ucd5c\ub97c \uc55e\ub450\uace0 \ucca0\uc800\ud55c \ubc18\uc18c(\u53cd\u8607)\uc8fc\uc758\uc790\uc774\uba70 \uc2e0\ud0c1\ud1b5\uce58\uc548\uc744 \uaca9\ub82c\ud788 \ube44\ud310\ud574 \uc628 \uc774\uc2b9\ub9cc\uc744 \uc815\uce58 \uc77c\uc120\uc5d0\uc11c \ubc30\uc81c\ud560 \ud544\uc694\ub97c \ub290\uaf08\uae30 \ub54c\ubb38\uc774\uc5c8\ub2e4. \ubbf8\uad70\uc815\uc740 \uc758\uc7a5 \ub300\ub9ac\uc5d0 \uae40\uaddc\uc2dd\uc744 \uc549\ud614\ub358 \uac83\uc774\ub2e4."} {"question": "\uac00\uc218 \ube44\uac00 \uc2e0\ubcd1 \ud6c8\ub828\uc9c0\ub85c 5\uc0ac\ub2e8 \uc5f4\uc1e0\ubd80\ub300\ub97c \ubc30\uce58\ubc1b\uc740 \uc2dc\uae30\ub294?", "paragraph": "\uadf8\ub294 \uc758\uc815\ubd80 306 \ubcf4\ucda9\ub300\uc5d0\uc11c \uae30\ucd08\uad70\uc0ac\ud6c8\ub828\uc744 \ubc1b\uace0 2011\ub144 10\uc6d4 14\uc77c \uc2e0\ubcd1 \ud6c8\ub828\uc9c0\ub294 5\uc0ac\ub2e8 \uc5f4\uc1e0\ubd80\ub300\ub85c \ubc30\uce58\ubc1b\uc558\ub2e4. 11\uc6d4 17\uc77c \ube44\ub294 \ud2b9\uae09\uc804\uc0ac\ub85c \ubf51\ud78c \uac83\uc740 \ubb3c\ub860 \uc0ac\ub2e8\uc7a5 \ud45c\ucc3d\uc778 \u2018\uc874\uc911\uc0c1\u2019\ub3c4 \ubc1b\uc558\ub2e4. \uad70 \uad00\uacc4\uc790\ub294 \u201c\ube44\uac00 \ub6f0\uc5b4\ub09c \uc0ac\uaca9 \uc2e4\ub825\uacfc \uccb4\ub825, \uc194\uc120\uc218\ubc94\ud558\ub294 \uc0dd\ud65c\ud0dc\ub3c4\ub85c \ub2e4\ub978 \ubcd1\uc0ac\ub4e4\uc758 \uadc0\uac10\uc774 \ub410\ub2e4.\u201d\ub77c\uace0 \ub9d0\ud588\ub2e4. \ube44\ub294 \uc2e0\ubcd1 \uc790\uce58\ub300\uc5d0\uc11c \uc911\ub300\uc7a5 \ud6c8\ub828\ubcd1\uc744 \ub9e1\uae30\ub3c4 \ud588\ub2e4. \uc774\ub0a0 \ud1f4\uc18c\uc2dd\uc744 \ub9c8\uce5c \ube44\ub294 18\uc77c \uc81c2 \uc2e0\ubcd1\uad50\uc721\ub300\uc5d0 \uc785\uc18c\ud574 \uc55e\uc73c\ub85c 3\uc8fc\uac04 \uc2ec\ud654 \uad50\uc721\uc744 \uac70\uce5c \ub4a4 \ub2e4\uc74c \ub2ec 9\uc77c 5\uc0ac\ub2e8 \uc2e0\ubcd1\uad50\uc721\ub300 \uc870\uad50\ub85c \ubcf5\ubb34\ud558\ub2e4\uac00 \uad6d\ubc29\ubd80 \ud64d\ubcf4\uc9c0\uc6d0\ub2e8\uc73c\ub85c \ubcf4\uc9c1\uc774 \ubcc0\uacbd\ub418\uc5c8\ub2e4. \ud0c8\ubaa8\ubcf4\ud589\uc73c\ub85c 2013\ub144 1\uc6d4 7\uc77c\uac04\uc758 \uadfc\uc2e0 \ucc98\ubd84\uc744 \ubc1b\uc558\uc73c\uba70 2013\ub144 7\uc6d4 10\uc77c\uc5d0 \uc81c\ub300\ud558\uc600\ub2e4. \ube44\ub294 \uad70\uc785\ub300\ud6c4 \ud050\ube0c \uc5d4\ud130\ud14c\uc778\uba3c\ud2b8\ub85c \uc18c\uc18d\ub418\uc5c8\uc73c\ub098 \ud604\uc7ac\ub294 \ub808\uc778\ucef4\ud37c\ub2c8 1\uc778 \uae30\ud68d\uc0ac\uc5d0 \uc18c\uc18d\ub418\uc5b4\uc788\ub2e4.", "answer": "2011\ub144 10\uc6d4 14\uc77c", "sentence": "\uadf8\ub294 \uc758\uc815\ubd80 306 \ubcf4\ucda9\ub300\uc5d0\uc11c \uae30\ucd08\uad70\uc0ac\ud6c8\ub828\uc744 \ubc1b\uace0 2011\ub144 10\uc6d4 14\uc77c \uc2e0\ubcd1 \ud6c8\ub828\uc9c0\ub294 5\uc0ac\ub2e8 \uc5f4\uc1e0\ubd80\ub300\ub85c \ubc30\uce58\ubc1b\uc558\ub2e4.", "paragraph_sentence": " \uadf8\ub294 \uc758\uc815\ubd80 306 \ubcf4\ucda9\ub300\uc5d0\uc11c \uae30\ucd08\uad70\uc0ac\ud6c8\ub828\uc744 \ubc1b\uace0 2011\ub144 10\uc6d4 14\uc77c \uc2e0\ubcd1 \ud6c8\ub828\uc9c0\ub294 5\uc0ac\ub2e8 \uc5f4\uc1e0\ubd80\ub300\ub85c \ubc30\uce58\ubc1b\uc558\ub2e4. 11\uc6d4 17\uc77c \ube44\ub294 \ud2b9\uae09\uc804\uc0ac\ub85c \ubf51\ud78c \uac83\uc740 \ubb3c\ub860 \uc0ac\ub2e8\uc7a5 \ud45c\ucc3d\uc778 \u2018\uc874\uc911\uc0c1\u2019\ub3c4 \ubc1b\uc558\ub2e4. \uad70 \uad00\uacc4\uc790\ub294 \u201c\ube44\uac00 \ub6f0\uc5b4\ub09c \uc0ac\uaca9 \uc2e4\ub825\uacfc \uccb4\ub825, \uc194\uc120\uc218\ubc94\ud558\ub294 \uc0dd\ud65c\ud0dc\ub3c4\ub85c \ub2e4\ub978 \ubcd1\uc0ac\ub4e4\uc758 \uadc0\uac10\uc774 \ub410\ub2e4. \u201d\ub77c\uace0 \ub9d0\ud588\ub2e4. \ube44\ub294 \uc2e0\ubcd1 \uc790\uce58\ub300\uc5d0\uc11c \uc911\ub300\uc7a5 \ud6c8\ub828\ubcd1\uc744 \ub9e1\uae30\ub3c4 \ud588\ub2e4. \uc774\ub0a0 \ud1f4\uc18c\uc2dd\uc744 \ub9c8\uce5c \ube44\ub294 18\uc77c \uc81c2 \uc2e0\ubcd1\uad50\uc721\ub300\uc5d0 \uc785\uc18c\ud574 \uc55e\uc73c\ub85c 3\uc8fc\uac04 \uc2ec\ud654 \uad50\uc721\uc744 \uac70\uce5c \ub4a4 \ub2e4\uc74c \ub2ec 9\uc77c 5\uc0ac\ub2e8 \uc2e0\ubcd1\uad50\uc721\ub300 \uc870\uad50\ub85c \ubcf5\ubb34\ud558\ub2e4\uac00 \uad6d\ubc29\ubd80 \ud64d\ubcf4\uc9c0\uc6d0\ub2e8\uc73c\ub85c \ubcf4\uc9c1\uc774 \ubcc0\uacbd\ub418\uc5c8\ub2e4. \ud0c8\ubaa8\ubcf4\ud589\uc73c\ub85c 2013\ub144 1\uc6d4 7\uc77c\uac04\uc758 \uadfc\uc2e0 \ucc98\ubd84\uc744 \ubc1b\uc558\uc73c\uba70 2013\ub144 7\uc6d4 10\uc77c\uc5d0 \uc81c\ub300\ud558\uc600\ub2e4. \ube44\ub294 \uad70\uc785\ub300\ud6c4 \ud050\ube0c \uc5d4\ud130\ud14c\uc778\uba3c\ud2b8\ub85c \uc18c\uc18d\ub418\uc5c8\uc73c\ub098 \ud604\uc7ac\ub294 \ub808\uc778\ucef4\ud37c\ub2c8 1\uc778 \uae30\ud68d\uc0ac\uc5d0 \uc18c\uc18d\ub418\uc5b4\uc788\ub2e4.", "paragraph_answer": "\uadf8\ub294 \uc758\uc815\ubd80 306 \ubcf4\ucda9\ub300\uc5d0\uc11c \uae30\ucd08\uad70\uc0ac\ud6c8\ub828\uc744 \ubc1b\uace0 2011\ub144 10\uc6d4 14\uc77c \uc2e0\ubcd1 \ud6c8\ub828\uc9c0\ub294 5\uc0ac\ub2e8 \uc5f4\uc1e0\ubd80\ub300\ub85c \ubc30\uce58\ubc1b\uc558\ub2e4. 11\uc6d4 17\uc77c \ube44\ub294 \ud2b9\uae09\uc804\uc0ac\ub85c \ubf51\ud78c \uac83\uc740 \ubb3c\ub860 \uc0ac\ub2e8\uc7a5 \ud45c\ucc3d\uc778 \u2018\uc874\uc911\uc0c1\u2019\ub3c4 \ubc1b\uc558\ub2e4. \uad70 \uad00\uacc4\uc790\ub294 \u201c\ube44\uac00 \ub6f0\uc5b4\ub09c \uc0ac\uaca9 \uc2e4\ub825\uacfc \uccb4\ub825, \uc194\uc120\uc218\ubc94\ud558\ub294 \uc0dd\ud65c\ud0dc\ub3c4\ub85c \ub2e4\ub978 \ubcd1\uc0ac\ub4e4\uc758 \uadc0\uac10\uc774 \ub410\ub2e4.\u201d\ub77c\uace0 \ub9d0\ud588\ub2e4. \ube44\ub294 \uc2e0\ubcd1 \uc790\uce58\ub300\uc5d0\uc11c \uc911\ub300\uc7a5 \ud6c8\ub828\ubcd1\uc744 \ub9e1\uae30\ub3c4 \ud588\ub2e4. \uc774\ub0a0 \ud1f4\uc18c\uc2dd\uc744 \ub9c8\uce5c \ube44\ub294 18\uc77c \uc81c2 \uc2e0\ubcd1\uad50\uc721\ub300\uc5d0 \uc785\uc18c\ud574 \uc55e\uc73c\ub85c 3\uc8fc\uac04 \uc2ec\ud654 \uad50\uc721\uc744 \uac70\uce5c \ub4a4 \ub2e4\uc74c \ub2ec 9\uc77c 5\uc0ac\ub2e8 \uc2e0\ubcd1\uad50\uc721\ub300 \uc870\uad50\ub85c \ubcf5\ubb34\ud558\ub2e4\uac00 \uad6d\ubc29\ubd80 \ud64d\ubcf4\uc9c0\uc6d0\ub2e8\uc73c\ub85c \ubcf4\uc9c1\uc774 \ubcc0\uacbd\ub418\uc5c8\ub2e4. \ud0c8\ubaa8\ubcf4\ud589\uc73c\ub85c 2013\ub144 1\uc6d4 7\uc77c\uac04\uc758 \uadfc\uc2e0 \ucc98\ubd84\uc744 \ubc1b\uc558\uc73c\uba70 2013\ub144 7\uc6d4 10\uc77c\uc5d0 \uc81c\ub300\ud558\uc600\ub2e4. \ube44\ub294 \uad70\uc785\ub300\ud6c4 \ud050\ube0c \uc5d4\ud130\ud14c\uc778\uba3c\ud2b8\ub85c \uc18c\uc18d\ub418\uc5c8\uc73c\ub098 \ud604\uc7ac\ub294 \ub808\uc778\ucef4\ud37c\ub2c8 1\uc778 \uae30\ud68d\uc0ac\uc5d0 \uc18c\uc18d\ub418\uc5b4\uc788\ub2e4.", "sentence_answer": "\uadf8\ub294 \uc758\uc815\ubd80 306 \ubcf4\ucda9\ub300\uc5d0\uc11c \uae30\ucd08\uad70\uc0ac\ud6c8\ub828\uc744 \ubc1b\uace0 2011\ub144 10\uc6d4 14\uc77c \uc2e0\ubcd1 \ud6c8\ub828\uc9c0\ub294 5\uc0ac\ub2e8 \uc5f4\uc1e0\ubd80\ub300\ub85c \ubc30\uce58\ubc1b\uc558\ub2e4.", "paragraph_id": "6509888-2-1", "paragraph_question": "question: \uac00\uc218 \ube44\uac00 \uc2e0\ubcd1 \ud6c8\ub828\uc9c0\ub85c 5\uc0ac\ub2e8 \uc5f4\uc1e0\ubd80\ub300\ub97c \ubc30\uce58\ubc1b\uc740 \uc2dc\uae30\ub294?, context: \uadf8\ub294 \uc758\uc815\ubd80 306 \ubcf4\ucda9\ub300\uc5d0\uc11c \uae30\ucd08\uad70\uc0ac\ud6c8\ub828\uc744 \ubc1b\uace0 2011\ub144 10\uc6d4 14\uc77c \uc2e0\ubcd1 \ud6c8\ub828\uc9c0\ub294 5\uc0ac\ub2e8 \uc5f4\uc1e0\ubd80\ub300\ub85c \ubc30\uce58\ubc1b\uc558\ub2e4. 11\uc6d4 17\uc77c \ube44\ub294 \ud2b9\uae09\uc804\uc0ac\ub85c \ubf51\ud78c \uac83\uc740 \ubb3c\ub860 \uc0ac\ub2e8\uc7a5 \ud45c\ucc3d\uc778 \u2018\uc874\uc911\uc0c1\u2019\ub3c4 \ubc1b\uc558\ub2e4. \uad70 \uad00\uacc4\uc790\ub294 \u201c\ube44\uac00 \ub6f0\uc5b4\ub09c \uc0ac\uaca9 \uc2e4\ub825\uacfc \uccb4\ub825, \uc194\uc120\uc218\ubc94\ud558\ub294 \uc0dd\ud65c\ud0dc\ub3c4\ub85c \ub2e4\ub978 \ubcd1\uc0ac\ub4e4\uc758 \uadc0\uac10\uc774 \ub410\ub2e4.\u201d\ub77c\uace0 \ub9d0\ud588\ub2e4. \ube44\ub294 \uc2e0\ubcd1 \uc790\uce58\ub300\uc5d0\uc11c \uc911\ub300\uc7a5 \ud6c8\ub828\ubcd1\uc744 \ub9e1\uae30\ub3c4 \ud588\ub2e4. \uc774\ub0a0 \ud1f4\uc18c\uc2dd\uc744 \ub9c8\uce5c \ube44\ub294 18\uc77c \uc81c2 \uc2e0\ubcd1\uad50\uc721\ub300\uc5d0 \uc785\uc18c\ud574 \uc55e\uc73c\ub85c 3\uc8fc\uac04 \uc2ec\ud654 \uad50\uc721\uc744 \uac70\uce5c \ub4a4 \ub2e4\uc74c \ub2ec 9\uc77c 5\uc0ac\ub2e8 \uc2e0\ubcd1\uad50\uc721\ub300 \uc870\uad50\ub85c \ubcf5\ubb34\ud558\ub2e4\uac00 \uad6d\ubc29\ubd80 \ud64d\ubcf4\uc9c0\uc6d0\ub2e8\uc73c\ub85c \ubcf4\uc9c1\uc774 \ubcc0\uacbd\ub418\uc5c8\ub2e4. \ud0c8\ubaa8\ubcf4\ud589\uc73c\ub85c 2013\ub144 1\uc6d4 7\uc77c\uac04\uc758 \uadfc\uc2e0 \ucc98\ubd84\uc744 \ubc1b\uc558\uc73c\uba70 2013\ub144 7\uc6d4 10\uc77c\uc5d0 \uc81c\ub300\ud558\uc600\ub2e4. \ube44\ub294 \uad70\uc785\ub300\ud6c4 \ud050\ube0c \uc5d4\ud130\ud14c\uc778\uba3c\ud2b8\ub85c \uc18c\uc18d\ub418\uc5c8\uc73c\ub098 \ud604\uc7ac\ub294 \ub808\uc778\ucef4\ud37c\ub2c8 1\uc778 \uae30\ud68d\uc0ac\uc5d0 \uc18c\uc18d\ub418\uc5b4\uc788\ub2e4."} {"question": "\uc0bc\uc5fd\ucda9\uc758 \uc8fc\uc694 \uc704\uae30\uc2dc\uae30\ub294 \uc5b8\uc81c\uc778\uac00?", "paragraph": "\ud558\ubd80 \uce84\ube0c\ub9ac\uc544\uae30 \uc9c0\uce35\uc5d0\uc11c \uc0bc\uc5fd\ucda9\uc774 \ucc98\uc74c \ucd9c\ud604\ud55c \uc9c1\ud6c4\uc5d0 \uc0bc\uc5fd\ucda9\uc740 \uae09\uaca9\ud788 \ub2e4\uc591\ud654\ud574\uc11c \ub808\ub4dc\ub9ac\ud0a4\ub2e4, \ud504\ud2f0\ucf54\ud30c\ub9ac\ub2e4, \uc544\uadf8\ub178\uc2a4\ud2f0\ub2e4 \uadf8\ub9ac\uace0 \ucf54\ub9ac\ub125\uc18c\ud0a4\ub2e4 \ub4f1 \uce84\ube0c\ub9ac\uc544\uae30\ub97c \ud2b9\uc9d5\uc9d3\ub294 \uc8fc\uc694 \ubaa9\ub4e4\uc774 \uc0dd\uaca8\ub0ac\ub2e4. \uc0bc\uc5fd\ucda9 \ud654\uc11d \uae30\ub85d\uc758 \uccab \uc8fc\uc694 \uc704\uae30\ub294 \uce84\ube0c\ub9ac\uc544\uae30 \uc911\uae30\uc5d0 \uc788\uc5c8\ub2e4. \uc5ec\uae30\uc5d0\uc11c \uc0b4\uc544\ub0a8\uc740 \uc0bc\uc5fd\ucda9 \ubaa9\ub4e4\uc740 \uba38\ub9ac\uc640 \uaf2c\ub9ac \ud06c\uae30\uac00 \ube44\uc2b7\ud558\uac70\ub098, \uaf2c\ub9ac\uac00 \ub354 \ud070 \ud615\ud0dc\ub97c \uac00\uc9c0\uace0 \uc788\uc5c8\uc73c\uba70 \uc678\uace8\uaca9\uc5d0 \ub354 \ub450\uaebc\uc6b4 \ud050\ud2f0\ud074 \uce35\uc744 \uac00\uc9c0\uace0 \uc788\uc5b4 \ud3ec\uc2dd\uc790\ub85c\ubd80\ud130 \ubab8\uc744 \ub354 \uc798 \ubcf4\ud638\ud560 \uc218 \uc788\uc5c8\ub2e4 (\uac00\uc2b4 \ud56d\ubaa9\uc744 \ubcfc \uac83). \uce84\ube0c\ub9ac\uc544\uae30 \ub9d0\uc758 \ub300\ub7c9\uba78\uc885 \uc0ac\uac74\uc73c\ub85c \uc0bc\uc5fd\ucda9 \ub3d9\ubb3c\uad70\uc5d0\ub294 \ud070 \ubcc0\ud654\uac00 \ucc3e\uc544 \uc654\ub2e4. \uac70\uc758 \ubaa8\ub4e0 \ub808\ub4dc\ub9ac\ud0a4\ub2e4\ubaa9(\uc62c\ub808\ub12c\ub85c\uc774\ub4dc\ub97c \ud3ec\ud568\ud558\uc5ec)\uacfc \ub300\ubd80\ubd84\uc758 \uce84\ube0c\ub9ac\uc544\uae30 \ud6c4\uae30 \uc0bc\uc5fd\ucda9\ub4e4\uc774 \uba78\uc885\ud588\ub2e4. \uba78\uc885\uacfc \uac19\uc740 \uc2dc\uae30\uc5d0 \ub85c\ub80c\uc2dc\uc544\uc758 \ub300\ub959\ubd95 \uc9c0\uc5ed\uc5d0\uc11c \uc0bc\uc5fd\ucda9\uc758 \uc804\ubc18\uc801\uc778 \uc1e0\ud1f4\uac00 \uacc4\uc18d\ub41c \uac83\uc744 \ubcf4\uba74 \ud658\uacbd\uc5d0 \uae09\uaca9\ud55c \ubcc0\ud654\uac00 \uc788\uc5c8\ub358 \uac83\uc73c\ub85c \ubcf4\uc778\ub2e4.", "answer": "\uce84\ube0c\ub9ac\uc544\uae30 \uc911\uae30", "sentence": "\uc0bc\uc5fd\ucda9 \ud654\uc11d \uae30\ub85d\uc758 \uccab \uc8fc\uc694 \uc704\uae30\ub294 \uce84\ube0c\ub9ac\uc544\uae30 \uc911\uae30 \uc5d0 \uc788\uc5c8\ub2e4.", "paragraph_sentence": "\ud558\ubd80 \uce84\ube0c\ub9ac\uc544\uae30 \uc9c0\uce35\uc5d0\uc11c \uc0bc\uc5fd\ucda9\uc774 \ucc98\uc74c \ucd9c\ud604\ud55c \uc9c1\ud6c4\uc5d0 \uc0bc\uc5fd\ucda9\uc740 \uae09\uaca9\ud788 \ub2e4\uc591\ud654\ud574\uc11c \ub808\ub4dc\ub9ac\ud0a4\ub2e4, \ud504\ud2f0\ucf54\ud30c\ub9ac\ub2e4, \uc544\uadf8\ub178\uc2a4\ud2f0\ub2e4 \uadf8\ub9ac\uace0 \ucf54\ub9ac\ub125\uc18c\ud0a4\ub2e4 \ub4f1 \uce84\ube0c\ub9ac\uc544\uae30\ub97c \ud2b9\uc9d5\uc9d3\ub294 \uc8fc\uc694 \ubaa9\ub4e4\uc774 \uc0dd\uaca8\ub0ac\ub2e4. \uc0bc\uc5fd\ucda9 \ud654\uc11d \uae30\ub85d\uc758 \uccab \uc8fc\uc694 \uc704\uae30\ub294 \uce84\ube0c\ub9ac\uc544\uae30 \uc911\uae30 \uc5d0 \uc788\uc5c8\ub2e4. \uc5ec\uae30\uc5d0\uc11c \uc0b4\uc544\ub0a8\uc740 \uc0bc\uc5fd\ucda9 \ubaa9\ub4e4\uc740 \uba38\ub9ac\uc640 \uaf2c\ub9ac \ud06c\uae30\uac00 \ube44\uc2b7\ud558\uac70\ub098, \uaf2c\ub9ac\uac00 \ub354 \ud070 \ud615\ud0dc\ub97c \uac00\uc9c0\uace0 \uc788\uc5c8\uc73c\uba70 \uc678\uace8\uaca9\uc5d0 \ub354 \ub450\uaebc\uc6b4 \ud050\ud2f0\ud074 \uce35\uc744 \uac00\uc9c0\uace0 \uc788\uc5b4 \ud3ec\uc2dd\uc790\ub85c\ubd80\ud130 \ubab8\uc744 \ub354 \uc798 \ubcf4\ud638\ud560 \uc218 \uc788\uc5c8\ub2e4 (\uac00\uc2b4 \ud56d\ubaa9\uc744 \ubcfc \uac83). \uce84\ube0c\ub9ac\uc544\uae30 \ub9d0\uc758 \ub300\ub7c9\uba78\uc885 \uc0ac\uac74\uc73c\ub85c \uc0bc\uc5fd\ucda9 \ub3d9\ubb3c\uad70\uc5d0\ub294 \ud070 \ubcc0\ud654\uac00 \ucc3e\uc544 \uc654\ub2e4. \uac70\uc758 \ubaa8\ub4e0 \ub808\ub4dc\ub9ac\ud0a4\ub2e4\ubaa9(\uc62c\ub808\ub12c\ub85c\uc774\ub4dc\ub97c \ud3ec\ud568\ud558\uc5ec)\uacfc \ub300\ubd80\ubd84\uc758 \uce84\ube0c\ub9ac\uc544\uae30 \ud6c4\uae30 \uc0bc\uc5fd\ucda9\ub4e4\uc774 \uba78\uc885\ud588\ub2e4. \uba78\uc885\uacfc \uac19\uc740 \uc2dc\uae30\uc5d0 \ub85c\ub80c\uc2dc\uc544\uc758 \ub300\ub959\ubd95 \uc9c0\uc5ed\uc5d0\uc11c \uc0bc\uc5fd\ucda9\uc758 \uc804\ubc18\uc801\uc778 \uc1e0\ud1f4\uac00 \uacc4\uc18d\ub41c \uac83\uc744 \ubcf4\uba74 \ud658\uacbd\uc5d0 \uae09\uaca9\ud55c \ubcc0\ud654\uac00 \uc788\uc5c8\ub358 \uac83\uc73c\ub85c \ubcf4\uc778\ub2e4.", "paragraph_answer": "\ud558\ubd80 \uce84\ube0c\ub9ac\uc544\uae30 \uc9c0\uce35\uc5d0\uc11c \uc0bc\uc5fd\ucda9\uc774 \ucc98\uc74c \ucd9c\ud604\ud55c \uc9c1\ud6c4\uc5d0 \uc0bc\uc5fd\ucda9\uc740 \uae09\uaca9\ud788 \ub2e4\uc591\ud654\ud574\uc11c \ub808\ub4dc\ub9ac\ud0a4\ub2e4, \ud504\ud2f0\ucf54\ud30c\ub9ac\ub2e4, \uc544\uadf8\ub178\uc2a4\ud2f0\ub2e4 \uadf8\ub9ac\uace0 \ucf54\ub9ac\ub125\uc18c\ud0a4\ub2e4 \ub4f1 \uce84\ube0c\ub9ac\uc544\uae30\ub97c \ud2b9\uc9d5\uc9d3\ub294 \uc8fc\uc694 \ubaa9\ub4e4\uc774 \uc0dd\uaca8\ub0ac\ub2e4. \uc0bc\uc5fd\ucda9 \ud654\uc11d \uae30\ub85d\uc758 \uccab \uc8fc\uc694 \uc704\uae30\ub294 \uce84\ube0c\ub9ac\uc544\uae30 \uc911\uae30 \uc5d0 \uc788\uc5c8\ub2e4. \uc5ec\uae30\uc5d0\uc11c \uc0b4\uc544\ub0a8\uc740 \uc0bc\uc5fd\ucda9 \ubaa9\ub4e4\uc740 \uba38\ub9ac\uc640 \uaf2c\ub9ac \ud06c\uae30\uac00 \ube44\uc2b7\ud558\uac70\ub098, \uaf2c\ub9ac\uac00 \ub354 \ud070 \ud615\ud0dc\ub97c \uac00\uc9c0\uace0 \uc788\uc5c8\uc73c\uba70 \uc678\uace8\uaca9\uc5d0 \ub354 \ub450\uaebc\uc6b4 \ud050\ud2f0\ud074 \uce35\uc744 \uac00\uc9c0\uace0 \uc788\uc5b4 \ud3ec\uc2dd\uc790\ub85c\ubd80\ud130 \ubab8\uc744 \ub354 \uc798 \ubcf4\ud638\ud560 \uc218 \uc788\uc5c8\ub2e4 (\uac00\uc2b4 \ud56d\ubaa9\uc744 \ubcfc \uac83). \uce84\ube0c\ub9ac\uc544\uae30 \ub9d0\uc758 \ub300\ub7c9\uba78\uc885 \uc0ac\uac74\uc73c\ub85c \uc0bc\uc5fd\ucda9 \ub3d9\ubb3c\uad70\uc5d0\ub294 \ud070 \ubcc0\ud654\uac00 \ucc3e\uc544 \uc654\ub2e4. \uac70\uc758 \ubaa8\ub4e0 \ub808\ub4dc\ub9ac\ud0a4\ub2e4\ubaa9(\uc62c\ub808\ub12c\ub85c\uc774\ub4dc\ub97c \ud3ec\ud568\ud558\uc5ec)\uacfc \ub300\ubd80\ubd84\uc758 \uce84\ube0c\ub9ac\uc544\uae30 \ud6c4\uae30 \uc0bc\uc5fd\ucda9\ub4e4\uc774 \uba78\uc885\ud588\ub2e4. \uba78\uc885\uacfc \uac19\uc740 \uc2dc\uae30\uc5d0 \ub85c\ub80c\uc2dc\uc544\uc758 \ub300\ub959\ubd95 \uc9c0\uc5ed\uc5d0\uc11c \uc0bc\uc5fd\ucda9\uc758 \uc804\ubc18\uc801\uc778 \uc1e0\ud1f4\uac00 \uacc4\uc18d\ub41c \uac83\uc744 \ubcf4\uba74 \ud658\uacbd\uc5d0 \uae09\uaca9\ud55c \ubcc0\ud654\uac00 \uc788\uc5c8\ub358 \uac83\uc73c\ub85c \ubcf4\uc778\ub2e4.", "sentence_answer": "\uc0bc\uc5fd\ucda9 \ud654\uc11d \uae30\ub85d\uc758 \uccab \uc8fc\uc694 \uc704\uae30\ub294 \uce84\ube0c\ub9ac\uc544\uae30 \uc911\uae30 \uc5d0 \uc788\uc5c8\ub2e4.", "paragraph_id": "6583774-4-1", "paragraph_question": "question: \uc0bc\uc5fd\ucda9\uc758 \uc8fc\uc694 \uc704\uae30\uc2dc\uae30\ub294 \uc5b8\uc81c\uc778\uac00?, context: \ud558\ubd80 \uce84\ube0c\ub9ac\uc544\uae30 \uc9c0\uce35\uc5d0\uc11c \uc0bc\uc5fd\ucda9\uc774 \ucc98\uc74c \ucd9c\ud604\ud55c \uc9c1\ud6c4\uc5d0 \uc0bc\uc5fd\ucda9\uc740 \uae09\uaca9\ud788 \ub2e4\uc591\ud654\ud574\uc11c \ub808\ub4dc\ub9ac\ud0a4\ub2e4, \ud504\ud2f0\ucf54\ud30c\ub9ac\ub2e4, \uc544\uadf8\ub178\uc2a4\ud2f0\ub2e4 \uadf8\ub9ac\uace0 \ucf54\ub9ac\ub125\uc18c\ud0a4\ub2e4 \ub4f1 \uce84\ube0c\ub9ac\uc544\uae30\ub97c \ud2b9\uc9d5\uc9d3\ub294 \uc8fc\uc694 \ubaa9\ub4e4\uc774 \uc0dd\uaca8\ub0ac\ub2e4. \uc0bc\uc5fd\ucda9 \ud654\uc11d \uae30\ub85d\uc758 \uccab \uc8fc\uc694 \uc704\uae30\ub294 \uce84\ube0c\ub9ac\uc544\uae30 \uc911\uae30\uc5d0 \uc788\uc5c8\ub2e4. \uc5ec\uae30\uc5d0\uc11c \uc0b4\uc544\ub0a8\uc740 \uc0bc\uc5fd\ucda9 \ubaa9\ub4e4\uc740 \uba38\ub9ac\uc640 \uaf2c\ub9ac \ud06c\uae30\uac00 \ube44\uc2b7\ud558\uac70\ub098, \uaf2c\ub9ac\uac00 \ub354 \ud070 \ud615\ud0dc\ub97c \uac00\uc9c0\uace0 \uc788\uc5c8\uc73c\uba70 \uc678\uace8\uaca9\uc5d0 \ub354 \ub450\uaebc\uc6b4 \ud050\ud2f0\ud074 \uce35\uc744 \uac00\uc9c0\uace0 \uc788\uc5b4 \ud3ec\uc2dd\uc790\ub85c\ubd80\ud130 \ubab8\uc744 \ub354 \uc798 \ubcf4\ud638\ud560 \uc218 \uc788\uc5c8\ub2e4 (\uac00\uc2b4 \ud56d\ubaa9\uc744 \ubcfc \uac83). \uce84\ube0c\ub9ac\uc544\uae30 \ub9d0\uc758 \ub300\ub7c9\uba78\uc885 \uc0ac\uac74\uc73c\ub85c \uc0bc\uc5fd\ucda9 \ub3d9\ubb3c\uad70\uc5d0\ub294 \ud070 \ubcc0\ud654\uac00 \ucc3e\uc544 \uc654\ub2e4. \uac70\uc758 \ubaa8\ub4e0 \ub808\ub4dc\ub9ac\ud0a4\ub2e4\ubaa9(\uc62c\ub808\ub12c\ub85c\uc774\ub4dc\ub97c \ud3ec\ud568\ud558\uc5ec)\uacfc \ub300\ubd80\ubd84\uc758 \uce84\ube0c\ub9ac\uc544\uae30 \ud6c4\uae30 \uc0bc\uc5fd\ucda9\ub4e4\uc774 \uba78\uc885\ud588\ub2e4. \uba78\uc885\uacfc \uac19\uc740 \uc2dc\uae30\uc5d0 \ub85c\ub80c\uc2dc\uc544\uc758 \ub300\ub959\ubd95 \uc9c0\uc5ed\uc5d0\uc11c \uc0bc\uc5fd\ucda9\uc758 \uc804\ubc18\uc801\uc778 \uc1e0\ud1f4\uac00 \uacc4\uc18d\ub41c \uac83\uc744 \ubcf4\uba74 \ud658\uacbd\uc5d0 \uae09\uaca9\ud55c \ubcc0\ud654\uac00 \uc788\uc5c8\ub358 \uac83\uc73c\ub85c \ubcf4\uc778\ub2e4."} {"question": "\ucf54\ub9ac\ub3c4\ub77c\uc2a4 \uc544\uc5d0\ub124\uc6b0\uc2a4\ub294 \ubcf4\ud1b5 \uba87 \ub144\uc744 \uc0ac\ub294\uac00?", "paragraph": "\ub300\ubd80\ubd84 \ub625\uc744 \uba39\ub294 \ubb3c\uace0\uae30\ub098 \uc4f0\ub808\uae30\ub97c \uc8fc\uc6cc\uba39\ub294 \ubb3c\uace0\uae30\ub77c\uace0 \ubd88\ub9ac\ub294\ub370 \uc0ac\uc2e4\uc740 \ubc14\ub2e5\uc5d0 \ub5a8\uc5b4\uc9c4 \uba39\uc774\ub97c \ucc3e\ub294 \uac83\uc774\ub2e4. \uc218\ucef7 \uc131\uccb4\uc758 \uae38\uc774\ub294 6.5cm \uace0 \uc554\ucef7 \uc131\uccb4\ub294 \uc870\uae08 \ucee4\uc11c 7cm \uc815\ub3c4\uac00 \ub41c\ub2e4. \ubcf4\ud1b5 10\ub144 \uc815\ub3c4\ub97c \uc0b4\uba70, \ub178\ub797\uac70\ub098 \ud551\ud06c\ube5b\uc758 \ubab8\ud1b5, \ud558\uc580 \ubc30, \uccad\ud68c\uc0c9 \uba38\ub9ac\uc640 \ub4f1\uc744 \uac00\uc9c0\uace0 \uc788\ub2e4. \ub2e4\ub978 \ucf54\ub9ac\ub3c4\ub77c\uc2a4\ub4e4\uacfc \uac19\uc774 \uc790\uae30\ubc29\uc5b4\uc6a9\uc73c\ub85c \ub4f1\uc9c0\ub290\ub7ec\ubbf8, \uac00\uc2b4\uc9c0\ub290\ub7ec\ubbf8\uc640 \uae30\ub984\uc9c0\ub290\ub7ec\ubbf8\uc5d0 \uc57d\ud55c \ub3c5\uc744 \uac00\uc9c4 \uac00\uc2dc\ub97c \uac00\uc9c0\uace0 \uc788\ub2e4. \uac08\uc0c9\uc744 \ub764 \uc624\ub80c\uc9c0\uc0c9\uc774 \uc8fc\ub85c \uba38\ub9ac \uc704\uc5d0 \uc788\uc73c\uba70 \uc704\uc5d0\uc11c \ubd24\uc744 \ub54c \uc774\uac83\uc740 \uc774 \uc885\uc758 \uac00\uc7a5 \ud2b9\ubcc4\ud55c \ud2b9\uc9d5\uc774\ub2e4. \uadf8\ub9ac\uace0 \uc8fc\ub85c \uac74\uac15\ud560 \ub54c\ub294 \uccad\ub3d9 \uc0c9\uae54\uc744 \ub764\ub2e4. \uac00\ub054\uc529 \uc218\uba74\uc73c\ub85c \ub2e4\uac00\uc640 \uacf5\uae30\ub97c \uc0bc\ucf1c \uc7a5\ub0b4\ud638\ud761\uc744 \ud55c\ub2e4. \ubaa8\ub4e0 \ucf54\ub9ac\ub3c4\ub77c\uc2a4\ub97c \ud3ec\ud568\ud558\uc5ec \uc5f4\ub300\uc5b4\uc911\uc5d0\uc11c \uac00\uc7a5 \uac15\uc778\ud55c \uc0dd\uba85\ub825\uc744 \uc790\ub791\ud55c\ub2e4.", "answer": "10\ub144", "sentence": "\ubcf4\ud1b5 10\ub144 \uc815\ub3c4\ub97c \uc0b4\uba70, \ub178\ub797\uac70\ub098 \ud551\ud06c\ube5b\uc758 \ubab8\ud1b5, \ud558\uc580 \ubc30, \uccad\ud68c\uc0c9 \uba38\ub9ac\uc640 \ub4f1\uc744 \uac00\uc9c0\uace0 \uc788\ub2e4.", "paragraph_sentence": "\ub300\ubd80\ubd84 \ub625\uc744 \uba39\ub294 \ubb3c\uace0\uae30\ub098 \uc4f0\ub808\uae30\ub97c \uc8fc\uc6cc\uba39\ub294 \ubb3c\uace0\uae30\ub77c\uace0 \ubd88\ub9ac\ub294\ub370 \uc0ac\uc2e4\uc740 \ubc14\ub2e5\uc5d0 \ub5a8\uc5b4\uc9c4 \uba39\uc774\ub97c \ucc3e\ub294 \uac83\uc774\ub2e4. \uc218\ucef7 \uc131\uccb4\uc758 \uae38\uc774\ub294 6.5cm \uace0 \uc554\ucef7 \uc131\uccb4\ub294 \uc870\uae08 \ucee4\uc11c 7cm \uc815\ub3c4\uac00 \ub41c\ub2e4. \ubcf4\ud1b5 10\ub144 \uc815\ub3c4\ub97c \uc0b4\uba70, \ub178\ub797\uac70\ub098 \ud551\ud06c\ube5b\uc758 \ubab8\ud1b5, \ud558\uc580 \ubc30, \uccad\ud68c\uc0c9 \uba38\ub9ac\uc640 \ub4f1\uc744 \uac00\uc9c0\uace0 \uc788\ub2e4. \ub2e4\ub978 \ucf54\ub9ac\ub3c4\ub77c\uc2a4\ub4e4\uacfc \uac19\uc774 \uc790\uae30\ubc29\uc5b4\uc6a9\uc73c\ub85c \ub4f1\uc9c0\ub290\ub7ec\ubbf8, \uac00\uc2b4\uc9c0\ub290\ub7ec\ubbf8\uc640 \uae30\ub984\uc9c0\ub290\ub7ec\ubbf8\uc5d0 \uc57d\ud55c \ub3c5\uc744 \uac00\uc9c4 \uac00\uc2dc\ub97c \uac00\uc9c0\uace0 \uc788\ub2e4. \uac08\uc0c9\uc744 \ub764 \uc624\ub80c\uc9c0\uc0c9\uc774 \uc8fc\ub85c \uba38\ub9ac \uc704\uc5d0 \uc788\uc73c\uba70 \uc704\uc5d0\uc11c \ubd24\uc744 \ub54c \uc774\uac83\uc740 \uc774 \uc885\uc758 \uac00\uc7a5 \ud2b9\ubcc4\ud55c \ud2b9\uc9d5\uc774\ub2e4. \uadf8\ub9ac\uace0 \uc8fc\ub85c \uac74\uac15\ud560 \ub54c\ub294 \uccad\ub3d9 \uc0c9\uae54\uc744 \ub764\ub2e4. \uac00\ub054\uc529 \uc218\uba74\uc73c\ub85c \ub2e4\uac00\uc640 \uacf5\uae30\ub97c \uc0bc\ucf1c \uc7a5\ub0b4\ud638\ud761\uc744 \ud55c\ub2e4. \ubaa8\ub4e0 \ucf54\ub9ac\ub3c4\ub77c\uc2a4\ub97c \ud3ec\ud568\ud558\uc5ec \uc5f4\ub300\uc5b4\uc911\uc5d0\uc11c \uac00\uc7a5 \uac15\uc778\ud55c \uc0dd\uba85\ub825\uc744 \uc790\ub791\ud55c\ub2e4.", "paragraph_answer": "\ub300\ubd80\ubd84 \ub625\uc744 \uba39\ub294 \ubb3c\uace0\uae30\ub098 \uc4f0\ub808\uae30\ub97c \uc8fc\uc6cc\uba39\ub294 \ubb3c\uace0\uae30\ub77c\uace0 \ubd88\ub9ac\ub294\ub370 \uc0ac\uc2e4\uc740 \ubc14\ub2e5\uc5d0 \ub5a8\uc5b4\uc9c4 \uba39\uc774\ub97c \ucc3e\ub294 \uac83\uc774\ub2e4. \uc218\ucef7 \uc131\uccb4\uc758 \uae38\uc774\ub294 6.5cm \uace0 \uc554\ucef7 \uc131\uccb4\ub294 \uc870\uae08 \ucee4\uc11c 7cm \uc815\ub3c4\uac00 \ub41c\ub2e4. \ubcf4\ud1b5 10\ub144 \uc815\ub3c4\ub97c \uc0b4\uba70, \ub178\ub797\uac70\ub098 \ud551\ud06c\ube5b\uc758 \ubab8\ud1b5, \ud558\uc580 \ubc30, \uccad\ud68c\uc0c9 \uba38\ub9ac\uc640 \ub4f1\uc744 \uac00\uc9c0\uace0 \uc788\ub2e4. \ub2e4\ub978 \ucf54\ub9ac\ub3c4\ub77c\uc2a4\ub4e4\uacfc \uac19\uc774 \uc790\uae30\ubc29\uc5b4\uc6a9\uc73c\ub85c \ub4f1\uc9c0\ub290\ub7ec\ubbf8, \uac00\uc2b4\uc9c0\ub290\ub7ec\ubbf8\uc640 \uae30\ub984\uc9c0\ub290\ub7ec\ubbf8\uc5d0 \uc57d\ud55c \ub3c5\uc744 \uac00\uc9c4 \uac00\uc2dc\ub97c \uac00\uc9c0\uace0 \uc788\ub2e4. \uac08\uc0c9\uc744 \ub764 \uc624\ub80c\uc9c0\uc0c9\uc774 \uc8fc\ub85c \uba38\ub9ac \uc704\uc5d0 \uc788\uc73c\uba70 \uc704\uc5d0\uc11c \ubd24\uc744 \ub54c \uc774\uac83\uc740 \uc774 \uc885\uc758 \uac00\uc7a5 \ud2b9\ubcc4\ud55c \ud2b9\uc9d5\uc774\ub2e4. \uadf8\ub9ac\uace0 \uc8fc\ub85c \uac74\uac15\ud560 \ub54c\ub294 \uccad\ub3d9 \uc0c9\uae54\uc744 \ub764\ub2e4. \uac00\ub054\uc529 \uc218\uba74\uc73c\ub85c \ub2e4\uac00\uc640 \uacf5\uae30\ub97c \uc0bc\ucf1c \uc7a5\ub0b4\ud638\ud761\uc744 \ud55c\ub2e4. \ubaa8\ub4e0 \ucf54\ub9ac\ub3c4\ub77c\uc2a4\ub97c \ud3ec\ud568\ud558\uc5ec \uc5f4\ub300\uc5b4\uc911\uc5d0\uc11c \uac00\uc7a5 \uac15\uc778\ud55c \uc0dd\uba85\ub825\uc744 \uc790\ub791\ud55c\ub2e4.", "sentence_answer": "\ubcf4\ud1b5 10\ub144 \uc815\ub3c4\ub97c \uc0b4\uba70, \ub178\ub797\uac70\ub098 \ud551\ud06c\ube5b\uc758 \ubab8\ud1b5, \ud558\uc580 \ubc30, \uccad\ud68c\uc0c9 \uba38\ub9ac\uc640 \ub4f1\uc744 \uac00\uc9c0\uace0 \uc788\ub2e4.", "paragraph_id": "5906299-0-0", "paragraph_question": "question: \ucf54\ub9ac\ub3c4\ub77c\uc2a4 \uc544\uc5d0\ub124\uc6b0\uc2a4\ub294 \ubcf4\ud1b5 \uba87 \ub144\uc744 \uc0ac\ub294\uac00?, context: \ub300\ubd80\ubd84 \ub625\uc744 \uba39\ub294 \ubb3c\uace0\uae30\ub098 \uc4f0\ub808\uae30\ub97c \uc8fc\uc6cc\uba39\ub294 \ubb3c\uace0\uae30\ub77c\uace0 \ubd88\ub9ac\ub294\ub370 \uc0ac\uc2e4\uc740 \ubc14\ub2e5\uc5d0 \ub5a8\uc5b4\uc9c4 \uba39\uc774\ub97c \ucc3e\ub294 \uac83\uc774\ub2e4. \uc218\ucef7 \uc131\uccb4\uc758 \uae38\uc774\ub294 6.5cm \uace0 \uc554\ucef7 \uc131\uccb4\ub294 \uc870\uae08 \ucee4\uc11c 7cm \uc815\ub3c4\uac00 \ub41c\ub2e4. \ubcf4\ud1b5 10\ub144 \uc815\ub3c4\ub97c \uc0b4\uba70, \ub178\ub797\uac70\ub098 \ud551\ud06c\ube5b\uc758 \ubab8\ud1b5, \ud558\uc580 \ubc30, \uccad\ud68c\uc0c9 \uba38\ub9ac\uc640 \ub4f1\uc744 \uac00\uc9c0\uace0 \uc788\ub2e4. \ub2e4\ub978 \ucf54\ub9ac\ub3c4\ub77c\uc2a4\ub4e4\uacfc \uac19\uc774 \uc790\uae30\ubc29\uc5b4\uc6a9\uc73c\ub85c \ub4f1\uc9c0\ub290\ub7ec\ubbf8, \uac00\uc2b4\uc9c0\ub290\ub7ec\ubbf8\uc640 \uae30\ub984\uc9c0\ub290\ub7ec\ubbf8\uc5d0 \uc57d\ud55c \ub3c5\uc744 \uac00\uc9c4 \uac00\uc2dc\ub97c \uac00\uc9c0\uace0 \uc788\ub2e4. \uac08\uc0c9\uc744 \ub764 \uc624\ub80c\uc9c0\uc0c9\uc774 \uc8fc\ub85c \uba38\ub9ac \uc704\uc5d0 \uc788\uc73c\uba70 \uc704\uc5d0\uc11c \ubd24\uc744 \ub54c \uc774\uac83\uc740 \uc774 \uc885\uc758 \uac00\uc7a5 \ud2b9\ubcc4\ud55c \ud2b9\uc9d5\uc774\ub2e4. \uadf8\ub9ac\uace0 \uc8fc\ub85c \uac74\uac15\ud560 \ub54c\ub294 \uccad\ub3d9 \uc0c9\uae54\uc744 \ub764\ub2e4. \uac00\ub054\uc529 \uc218\uba74\uc73c\ub85c \ub2e4\uac00\uc640 \uacf5\uae30\ub97c \uc0bc\ucf1c \uc7a5\ub0b4\ud638\ud761\uc744 \ud55c\ub2e4. \ubaa8\ub4e0 \ucf54\ub9ac\ub3c4\ub77c\uc2a4\ub97c \ud3ec\ud568\ud558\uc5ec \uc5f4\ub300\uc5b4\uc911\uc5d0\uc11c \uac00\uc7a5 \uac15\uc778\ud55c \uc0dd\uba85\ub825\uc744 \uc790\ub791\ud55c\ub2e4."} {"question": "\ud574\ubcd1\ub300 \ucd08\ub300 \uc9c0\ud718\uad00\uc758 \uc774\ub984\uc740?", "paragraph": "\ub300\ud55c\ubbfc\uad6d \ud574\ubcd1\ub300\ub294 \ub300\ud55c\ubbfc\uad6d \ud574\uad70 \uc608\ud558\uc5d0 \ud3b8\uc131\ub418\uc5b4 \uc788\ub294 \uad70\uc73c\ub85c\uc11c \uad6d\uac00 \uc804\ub7b5 \uae30\ub3d9\uad70\uc73c\ub85c\uc11c \ud574\ubcd1\ub300\ub294 \uc0c1\ub959\uc791\uc804\uc744 \uc218\ud589\ud55c\ub2e4. \ub300\ud55c\ubbfc\uad6d \ud574\uad70(\uc608\ud558 \ud574\ubcd1\ub300 \ud3ec\ud568)\uc740 \ubcd1\ub825 \uc57d 69,000\uba85, \uc7a0\uc218\ud568 \uc57d 20 \ucc99, \uc804\ud22c\ud568\uc815 \uc57d 140\ucc99, \uc9c0\uc6d0\ud568\uc815 20\ucc99, \ud5ec\uae30/\ud574\uc0c1\ucd08\uacc4\uae30 \uc57d 50\ub300, K1A1 \uc804\ucc28\uc640 K9 \uc790\uc8fc\ud3ec, \uc0c1\ub959\ub3cc\uaca9\uc7a5\uac11\ucc28(KAAVP7A1) \ub4f1\uc758 \uae30\uac11 \ucc28\ub7c9, \uc0c1\ub959 \uc7a5\ube44\ub97c \ubcf4\uc720\ud558\uace0 \uc788\ub2e4. \ub300\ud55c\ubbfc\uad6d \ud574\ubcd1\ub300\ub294 1949\ub144 4\uc6d4 15\uc77c \uacbd\uc0c1\ub0a8\ub3c4 \uc9c4\ud574\uc2dc\uc5d0\uc11c \ucd08\ub300 \uc9c0\ud718\uad00\uc5d0 \uc2e0\ud604\uc900 \uc911\ub839\uc774 \uc784\uba85\ub418\uace0, \ud574\uad70 \uc7a5\uad50 26\uba85, \ubd80\uc0ac\uad00 54\uba85, \ubcd1 300\uba85\uc73c\ub85c \ucc3d\uc124\ub418\uc5c8\ub2e4. \ud574\uad70 \uc608\ud558\uc758 \uad6d\uac00 \uc804\ub7b5\uae30\ub3d9\uad70\uc73c\ub85c\uc11c \uc0c1\ub959 \uc791\uc804\uc744 \uc8fc\uc784\ubb34\ub85c \ud558\uba70, \uadf8 \uc678 \uae40\ud3ec, \uac15\ud654, \ud3ec\ud56d, \uacbd\uc8fc, \uc9c4\ud574, \uc81c\uc8fc, \ub3c4\uc11c\uc9c0\uc5ed \ub4f1 \ubc29\uc5b4, \uc0c1\ub959\uc791\uc804\uc744 \ud558\uace0, \uc608\ube44\uad70 \uad50\uc721 \ubc0f \ud6c8\ub828 \ub4f1\uc758 \uc784\ubb34\ub3c4 \uc218\ud589\ud55c\ub2e4. 2014\ub144 \ud604\uc7ac, 2\uac1c \uc0ac\ub2e8\uacfc 1\uac1c \uc5ec\ub2e8\uc744 \ubcf4\uc720\ud558\uace0 \uc788\uace0, \uc774\uc678\uc5d0\ub3c4 \uc5f0\ud3c9\ub3c4\uc640 \uc9c4\ud574, \uc81c\uc8fc\ub3c4 \ubc0f \uae30\ud0c0 \uc5ec\ub7ec \ub3c4\uc11c \uc9c0\uc5ed\ub4e4\uc5d0\ub3c4 \ud574\ubcd1 \ubd80\ub300\ub4e4\uc744 \uc8fc\ub454\uc2dc\ud0a4\uace0 \uc788\ub2e4.", "answer": "\uc2e0\ud604\uc900", "sentence": "\ub300\ud55c\ubbfc\uad6d \ud574\ubcd1\ub300\ub294 1949\ub144 4\uc6d4 15\uc77c \uacbd\uc0c1\ub0a8\ub3c4 \uc9c4\ud574\uc2dc\uc5d0\uc11c \ucd08\ub300 \uc9c0\ud718\uad00\uc5d0 \uc2e0\ud604\uc900 \uc911\ub839\uc774 \uc784\uba85\ub418\uace0, \ud574\uad70 \uc7a5\uad50 26\uba85, \ubd80\uc0ac\uad00 54\uba85, \ubcd1 300\uba85\uc73c\ub85c \ucc3d\uc124\ub418\uc5c8\ub2e4.", "paragraph_sentence": "\ub300\ud55c\ubbfc\uad6d \ud574\ubcd1\ub300\ub294 \ub300\ud55c\ubbfc\uad6d \ud574\uad70 \uc608\ud558\uc5d0 \ud3b8\uc131\ub418\uc5b4 \uc788\ub294 \uad70\uc73c\ub85c\uc11c \uad6d\uac00 \uc804\ub7b5 \uae30\ub3d9\uad70\uc73c\ub85c\uc11c \ud574\ubcd1\ub300\ub294 \uc0c1\ub959\uc791\uc804\uc744 \uc218\ud589\ud55c\ub2e4. \ub300\ud55c\ubbfc\uad6d \ud574\uad70(\uc608\ud558 \ud574\ubcd1\ub300 \ud3ec\ud568)\uc740 \ubcd1\ub825 \uc57d 69,000\uba85, \uc7a0\uc218\ud568 \uc57d 20 \ucc99, \uc804\ud22c\ud568\uc815 \uc57d 140\ucc99, \uc9c0\uc6d0\ud568\uc815 20\ucc99, \ud5ec\uae30/\ud574\uc0c1\ucd08\uacc4\uae30 \uc57d 50\ub300, K1A1 \uc804\ucc28\uc640 K9 \uc790\uc8fc\ud3ec, \uc0c1\ub959\ub3cc\uaca9\uc7a5\uac11\ucc28(KAAVP7A1) \ub4f1\uc758 \uae30\uac11 \ucc28\ub7c9, \uc0c1\ub959 \uc7a5\ube44\ub97c \ubcf4\uc720\ud558\uace0 \uc788\ub2e4. \ub300\ud55c\ubbfc\uad6d \ud574\ubcd1\ub300\ub294 1949\ub144 4\uc6d4 15\uc77c \uacbd\uc0c1\ub0a8\ub3c4 \uc9c4\ud574\uc2dc\uc5d0\uc11c \ucd08\ub300 \uc9c0\ud718\uad00\uc5d0 \uc2e0\ud604\uc900 \uc911\ub839\uc774 \uc784\uba85\ub418\uace0, \ud574\uad70 \uc7a5\uad50 26\uba85, \ubd80\uc0ac\uad00 54\uba85, \ubcd1 300\uba85\uc73c\ub85c \ucc3d\uc124\ub418\uc5c8\ub2e4. \ud574\uad70 \uc608\ud558\uc758 \uad6d\uac00 \uc804\ub7b5\uae30\ub3d9\uad70\uc73c\ub85c\uc11c \uc0c1\ub959 \uc791\uc804\uc744 \uc8fc\uc784\ubb34\ub85c \ud558\uba70, \uadf8 \uc678 \uae40\ud3ec, \uac15\ud654, \ud3ec\ud56d, \uacbd\uc8fc, \uc9c4\ud574, \uc81c\uc8fc, \ub3c4\uc11c\uc9c0\uc5ed \ub4f1 \ubc29\uc5b4, \uc0c1\ub959\uc791\uc804\uc744 \ud558\uace0, \uc608\ube44\uad70 \uad50\uc721 \ubc0f \ud6c8\ub828 \ub4f1\uc758 \uc784\ubb34\ub3c4 \uc218\ud589\ud55c\ub2e4. 2014\ub144 \ud604\uc7ac, 2\uac1c \uc0ac\ub2e8\uacfc 1\uac1c \uc5ec\ub2e8\uc744 \ubcf4\uc720\ud558\uace0 \uc788\uace0, \uc774\uc678\uc5d0\ub3c4 \uc5f0\ud3c9\ub3c4\uc640 \uc9c4\ud574, \uc81c\uc8fc\ub3c4 \ubc0f \uae30\ud0c0 \uc5ec\ub7ec \ub3c4\uc11c \uc9c0\uc5ed\ub4e4\uc5d0\ub3c4 \ud574\ubcd1 \ubd80\ub300\ub4e4\uc744 \uc8fc\ub454\uc2dc\ud0a4\uace0 \uc788\ub2e4.", "paragraph_answer": "\ub300\ud55c\ubbfc\uad6d \ud574\ubcd1\ub300\ub294 \ub300\ud55c\ubbfc\uad6d \ud574\uad70 \uc608\ud558\uc5d0 \ud3b8\uc131\ub418\uc5b4 \uc788\ub294 \uad70\uc73c\ub85c\uc11c \uad6d\uac00 \uc804\ub7b5 \uae30\ub3d9\uad70\uc73c\ub85c\uc11c \ud574\ubcd1\ub300\ub294 \uc0c1\ub959\uc791\uc804\uc744 \uc218\ud589\ud55c\ub2e4. \ub300\ud55c\ubbfc\uad6d \ud574\uad70(\uc608\ud558 \ud574\ubcd1\ub300 \ud3ec\ud568)\uc740 \ubcd1\ub825 \uc57d 69,000\uba85, \uc7a0\uc218\ud568 \uc57d 20 \ucc99, \uc804\ud22c\ud568\uc815 \uc57d 140\ucc99, \uc9c0\uc6d0\ud568\uc815 20\ucc99, \ud5ec\uae30/\ud574\uc0c1\ucd08\uacc4\uae30 \uc57d 50\ub300, K1A1 \uc804\ucc28\uc640 K9 \uc790\uc8fc\ud3ec, \uc0c1\ub959\ub3cc\uaca9\uc7a5\uac11\ucc28(KAAVP7A1) \ub4f1\uc758 \uae30\uac11 \ucc28\ub7c9, \uc0c1\ub959 \uc7a5\ube44\ub97c \ubcf4\uc720\ud558\uace0 \uc788\ub2e4. \ub300\ud55c\ubbfc\uad6d \ud574\ubcd1\ub300\ub294 1949\ub144 4\uc6d4 15\uc77c \uacbd\uc0c1\ub0a8\ub3c4 \uc9c4\ud574\uc2dc\uc5d0\uc11c \ucd08\ub300 \uc9c0\ud718\uad00\uc5d0 \uc2e0\ud604\uc900 \uc911\ub839\uc774 \uc784\uba85\ub418\uace0, \ud574\uad70 \uc7a5\uad50 26\uba85, \ubd80\uc0ac\uad00 54\uba85, \ubcd1 300\uba85\uc73c\ub85c \ucc3d\uc124\ub418\uc5c8\ub2e4. \ud574\uad70 \uc608\ud558\uc758 \uad6d\uac00 \uc804\ub7b5\uae30\ub3d9\uad70\uc73c\ub85c\uc11c \uc0c1\ub959 \uc791\uc804\uc744 \uc8fc\uc784\ubb34\ub85c \ud558\uba70, \uadf8 \uc678 \uae40\ud3ec, \uac15\ud654, \ud3ec\ud56d, \uacbd\uc8fc, \uc9c4\ud574, \uc81c\uc8fc, \ub3c4\uc11c\uc9c0\uc5ed \ub4f1 \ubc29\uc5b4, \uc0c1\ub959\uc791\uc804\uc744 \ud558\uace0, \uc608\ube44\uad70 \uad50\uc721 \ubc0f \ud6c8\ub828 \ub4f1\uc758 \uc784\ubb34\ub3c4 \uc218\ud589\ud55c\ub2e4. 2014\ub144 \ud604\uc7ac, 2\uac1c \uc0ac\ub2e8\uacfc 1\uac1c \uc5ec\ub2e8\uc744 \ubcf4\uc720\ud558\uace0 \uc788\uace0, \uc774\uc678\uc5d0\ub3c4 \uc5f0\ud3c9\ub3c4\uc640 \uc9c4\ud574, \uc81c\uc8fc\ub3c4 \ubc0f \uae30\ud0c0 \uc5ec\ub7ec \ub3c4\uc11c \uc9c0\uc5ed\ub4e4\uc5d0\ub3c4 \ud574\ubcd1 \ubd80\ub300\ub4e4\uc744 \uc8fc\ub454\uc2dc\ud0a4\uace0 \uc788\ub2e4.", "sentence_answer": "\ub300\ud55c\ubbfc\uad6d \ud574\ubcd1\ub300\ub294 1949\ub144 4\uc6d4 15\uc77c \uacbd\uc0c1\ub0a8\ub3c4 \uc9c4\ud574\uc2dc\uc5d0\uc11c \ucd08\ub300 \uc9c0\ud718\uad00\uc5d0 \uc2e0\ud604\uc900 \uc911\ub839\uc774 \uc784\uba85\ub418\uace0, \ud574\uad70 \uc7a5\uad50 26\uba85, \ubd80\uc0ac\uad00 54\uba85, \ubcd1 300\uba85\uc73c\ub85c \ucc3d\uc124\ub418\uc5c8\ub2e4.", "paragraph_id": "6031861-24-1", "paragraph_question": "question: \ud574\ubcd1\ub300 \ucd08\ub300 \uc9c0\ud718\uad00\uc758 \uc774\ub984\uc740?, context: \ub300\ud55c\ubbfc\uad6d \ud574\ubcd1\ub300\ub294 \ub300\ud55c\ubbfc\uad6d \ud574\uad70 \uc608\ud558\uc5d0 \ud3b8\uc131\ub418\uc5b4 \uc788\ub294 \uad70\uc73c\ub85c\uc11c \uad6d\uac00 \uc804\ub7b5 \uae30\ub3d9\uad70\uc73c\ub85c\uc11c \ud574\ubcd1\ub300\ub294 \uc0c1\ub959\uc791\uc804\uc744 \uc218\ud589\ud55c\ub2e4. \ub300\ud55c\ubbfc\uad6d \ud574\uad70(\uc608\ud558 \ud574\ubcd1\ub300 \ud3ec\ud568)\uc740 \ubcd1\ub825 \uc57d 69,000\uba85, \uc7a0\uc218\ud568 \uc57d 20 \ucc99, \uc804\ud22c\ud568\uc815 \uc57d 140\ucc99, \uc9c0\uc6d0\ud568\uc815 20\ucc99, \ud5ec\uae30/\ud574\uc0c1\ucd08\uacc4\uae30 \uc57d 50\ub300, K1A1 \uc804\ucc28\uc640 K9 \uc790\uc8fc\ud3ec, \uc0c1\ub959\ub3cc\uaca9\uc7a5\uac11\ucc28(KAAVP7A1) \ub4f1\uc758 \uae30\uac11 \ucc28\ub7c9, \uc0c1\ub959 \uc7a5\ube44\ub97c \ubcf4\uc720\ud558\uace0 \uc788\ub2e4. \ub300\ud55c\ubbfc\uad6d \ud574\ubcd1\ub300\ub294 1949\ub144 4\uc6d4 15\uc77c \uacbd\uc0c1\ub0a8\ub3c4 \uc9c4\ud574\uc2dc\uc5d0\uc11c \ucd08\ub300 \uc9c0\ud718\uad00\uc5d0 \uc2e0\ud604\uc900 \uc911\ub839\uc774 \uc784\uba85\ub418\uace0, \ud574\uad70 \uc7a5\uad50 26\uba85, \ubd80\uc0ac\uad00 54\uba85, \ubcd1 300\uba85\uc73c\ub85c \ucc3d\uc124\ub418\uc5c8\ub2e4. \ud574\uad70 \uc608\ud558\uc758 \uad6d\uac00 \uc804\ub7b5\uae30\ub3d9\uad70\uc73c\ub85c\uc11c \uc0c1\ub959 \uc791\uc804\uc744 \uc8fc\uc784\ubb34\ub85c \ud558\uba70, \uadf8 \uc678 \uae40\ud3ec, \uac15\ud654, \ud3ec\ud56d, \uacbd\uc8fc, \uc9c4\ud574, \uc81c\uc8fc, \ub3c4\uc11c\uc9c0\uc5ed \ub4f1 \ubc29\uc5b4, \uc0c1\ub959\uc791\uc804\uc744 \ud558\uace0, \uc608\ube44\uad70 \uad50\uc721 \ubc0f \ud6c8\ub828 \ub4f1\uc758 \uc784\ubb34\ub3c4 \uc218\ud589\ud55c\ub2e4. 2014\ub144 \ud604\uc7ac, 2\uac1c \uc0ac\ub2e8\uacfc 1\uac1c \uc5ec\ub2e8\uc744 \ubcf4\uc720\ud558\uace0 \uc788\uace0, \uc774\uc678\uc5d0\ub3c4 \uc5f0\ud3c9\ub3c4\uc640 \uc9c4\ud574, \uc81c\uc8fc\ub3c4 \ubc0f \uae30\ud0c0 \uc5ec\ub7ec \ub3c4\uc11c \uc9c0\uc5ed\ub4e4\uc5d0\ub3c4 \ud574\ubcd1 \ubd80\ub300\ub4e4\uc744 \uc8fc\ub454\uc2dc\ud0a4\uace0 \uc788\ub2e4."} {"question": "\ud45c\ucc3d\uc6d0\uc5d0 \ub530\ub974\uba74 \ubc15\uadfc\ud61c \ud0c4\ud575 \ucd1b\ubd88 \uc9d1\ud68c \ub2f9\uc2dc \uc5ec\ub860\uc870\uc0ac \uacb0\uacfc \uad6d\ubbfc\uc758 \uba87 \ud504\ub85c\uac00 \uc989\uc2dc \ud0c4\ud575\uc744 \uc694\uad6c\ud558\uace0 \uc788\ub2e4\uace0 \ud558\uc600\ub294\uac00?", "paragraph": "\ub300\ud55c\ubbfc\uad6d \uc81c18\ub300 \ud604\uc9c1 \ub300\ud1b5\ub839 \ubc15\uadfc\ud61c\ub294 \ubc94\uc8c4 \ud53c\uc758\uc790\uc785\ub2c8\ub2e4. \uadf8\uac83\ub3c4 \uad6d\uac00\uad8c\ub825\uc744 \uc0ac\uc720\ud654\ud574\uc11c \uc0ac\uc778\ub4e4\uc758 \ud638\uc8fc\uba38\ub2c8\uc5d0 \uad6d\uac00\uc138\uae08\uc744 \ud138\uc5b4 \ub123\uc740 \uac00\uc7a5 \uc9c8 \ub098\uc05c \ubc94\uc8c4\uc790\uc785\ub2c8\ub2e4. \uadf8\ub7f0\ub370 \uadf8\ub7f0 \ubc94\uc8c4 \ud53c\uc758\uc790\uac00 \uc9c0\uae08\uae4c\uc9c0 \uad6d\uac00\uad8c\ub825\uc744 \ud2c0\uc5b4\uc954 \ucc44 \uadf8 \uad6d\uac00 \uad8c\ub825\uc744 \uc790\uae30 \uac1c\uc778\uc758 \ubc29\uc5b4\uc640 \ubcf4\ud638\ub97c \uc704\ud574 \uc0ac\uc6a9\ud558\uace0 \uc788\uc2b5\ub2c8\ub2e4. \ub300\ud55c\ubbfc\uad6d \uad6d\ubbfc\ub4e4\uaed8\uc11c\ub294 \uc774 \uc0c1\ud669\uc744 \ub3c4\uc800\ud788 \ubc1b\uc544\ub4e4\uc77c \uc218\uac00 \uc5c6\ub2e4\uace0 \ud558\uc2ed\ub2c8\ub2e4. \ub9e4\uc8fc 100\ub9cc \uba85\uc774 \ub118\ub294 \uad6d\ubbfc\uaed8\uc11c \ucc28\uac00\uc6b4 \uad11\uc7a5\uc5d0 \ubaa8\uc5ec \ucd1b\ubd88\uc744 \ub4e4\uace0 \uacc4\uc2ed\ub2c8\ub2e4. \uc9c0\uae08 \uc5ec\ub860\uc870\uc0ac \uacb0\uacfc\ub294 80% \uc774\uc0c1\uc758 \uad6d\ubbfc\uc774 \uc989\uc2dc \ubc15\uadfc\ud61c \ub300\ud1b5\ub839\uc744 \ud0c4\ud575\ud558\ub77c\uace0 \uc694\uad6c\ud558\uace0 \uc788\uc2b5\ub2c8\ub2e4. \uad6d\ud68c\uc5d0 \uc694\uad6c\ud558\uace0 \uc788\uc2b5\ub2c8\ub2e4. \uadf8\ub7f0\ub370 \uc9c0\uae08 \uad6d\ud68c\ub294 \ubb34\uc5c7 \ud558\uace0 \uc788\uc2b5\ub2c8\uae4c? \uad6d\ubbfc\ub4e4\uc740 \uad6d\ud68c\ub97c \uc9c1\ubb34\uc720\uae30\ub85c \uace0\ubc1c\ud558\uace0 \uc788\uc2b5\ub2c8\ub2e4. (\uc911\ub7b5) \uadf8\ub807\uac8c \uc18c\ub9ac \ub192\uc5ec \ubc15\uadfc\ud61c \ub300\ud1b5\ub839\uc744 1\ubd84 1\ucd08\ub3c4 \uadf8 \uc790\ub9ac\uc5d0 \ub450\uc5b4\uc11c\ub294 \uc548 \ub41c\ub2e4\ub77c\uace0 \uc8fc\uc7a5\ud558\ub358, \uad6d\uac00\uc640 \uad6d\ubbfc\uc744 \uc704\ud574\uc11c \ubc15\uadfc\ud61c \ub300\ud1b5\ub839\uc740 \ubc14\ub85c \ub0b4\ub824\uc640\uc57c \ud55c\ub2e4\uace0 \uc694\uad6c\ud558\uace0 \uc8fc\uc7a5\ud558\ub358 \uc0c8\ub204\ub9ac\ub2f9 \uc758\uc6d0\ub4e4\uc758 \uc785\uc7a5\uc774 \ubc14\ub00c\uc5c8\uc2b5\ub2c8\ub2e4.\uc57c\ub2f9 \uc77c\ubd80\uc5d0\uc11c\ub3c4 \uc758\uacac\uc758 \ubcc0\ud654\uac00 \uac10\uc9c0\ub418\uace0 \ubcf4\ub3c4\ub418\uae30 \uc2dc\uc791\ud588\uc2b5\ub2c8\ub2e4. \uc800\ub294 \uac00\ub9cc\ud788 \uc788\uc744 \uc218 \uc5c6\uc5c8\uc2b5\ub2c8\ub2e4. \ub300\ud55c\ubbfc\uad6d \uad6d\ud68c\uc758\uc6d0\uc758 \ud55c \uc0ac\ub78c\uc73c\ub85c\uc11c, \uad6d\ubbfc\uc774 \ubf51\uc544 \uc8fc\uc2e0 \uad6d\ubbfc\uc758 \ub300\ud45c\ub85c\uc11c \uc800\ub3c4 \uadf8 \uc9c1\ubb34\uc720\uae30\uc758 \ud589\ub82c\uc5d0 \uacf5\ubc94\uc73c\ub85c \ub3d9\ucc38\ud560 \uc218 \uc5c6\uc5c8\uc2b5\ub2c8\ub2e4. \uadf8\ub798\uc11c \uc81c\uac00 \ud655\uc778\ud558\uace0 \ud655\ubcf4\ud55c \uacf5\uc801\uc778 \uc790\ub8cc\ub4e4\uc744 \ud1b5\ud574 \uc758\uc6d0 \uc5ec\ub7ec\ubd84 \ud55c \ubd84 \ud55c \ubd84\uc758 \ud0c4\ud575\uc5d0 \ub300\ud55c \ucc2c\ubc18 \uc785\uc7a5\uc744 \uad6d\ubbfc \uc5ec\ub7ec\ubd84\uacfc \uacf5\uc720\ud558\uace0 \uc788\uc2b5\ub2c8\ub2e4. \uadf8 \uacb0\uacfc \ub9ce\uc740 \uc758\uc6d0\ub4e4\uaed8\uc11c \uc5ec\ub7ec \uad6d\ubbfc\ub4e4\uc758 \uc804\ud654 \uc5f0\ub77d\uc744 \ubc1b\uc73c\uc2dc\uace0 \ubd88\ud3b8\ud558\ub2e4\ub294 \uc774\uc57c\uae30\ub97c \ub4e4\uc5c8\uc2b5\ub2c8\ub2e4. \uadf8 \ubd80\ubd84\uc5d0 \ub300\ud574\uc11c\ub294 \uc9c4\uc2ec\uc73c\ub85c \uc0ac\uacfc\uc758 \ub9d0\uc500\uc744 \ub4dc\ub9ac\uaca0\uc2b5\ub2c8\ub2e4. \uc544\uc6b8\ub7ec \uc624\ub298 \uc548\uc804\ud589\uc815\uc704\uc6d0\ud68c \ud68c\uc758 \ub3c4\uc911 \uc81c\uac00 \ud3c9\uc18c \uc88b\uc544\ud558\uace0 \uc0ac\ub791\ud558\uace0 \uc874\uacbd\ud558\ub294 \uc7a5\uc81c\uc6d0 \uc758\uc6d0\uacfc \uac10\uc815\uc801\uc778 \uc2f8\uc6c0\uae4c\uc9c0 \ud558\ub294 \uc0c1\ud669\uc5d0 \ucc98\ud588\uc2b5\ub2c8\ub2e4. \uadf8 \ubd80\ubd84 \uc5ed\uc2dc \uac1c\uc778\uc801\uc73c\ub85c \uc7a5\uc81c\uc6d0 \uc758\uc6d0\uaed8 \uacf5\uac1c\uc801\uc73c\ub85c \uc0ac\uacfc\ub4dc\ub9ac\uaca0\uc2b5\ub2c8\ub2e4. \ud558\uc9c0\ub9cc, \ud558\uc9c0\ub9cc, \uad6d\ud68c\uac00 \uc81c \uae30\ub2a5\uc744 \ud560 \ub54c\uae4c\uc9c0, \ubc15\uadfc\ud61c \ub300\ud1b5\ub839\uc5d0 \ub300\ud55c \ud0c4\ud575\uc18c\ucd94\uc548\uc5d0, \uadf8 \ubc1c\uc758\uc548\uc5d0 \ub204\uac00 \uc11c\uba85\ud558\uace0 \ub204\uac00 \uc11c\uba85\ud558\uc9c0 \uc54a\ub294\uc9c0, \ub204\uad6c\uc758 \uc11c\uba85 \ub54c\ubb38\uc5d0, \ub204\uad6c\uc758 \uc11c\uba85 \ubd88\ucc38 \ub54c\ubb38\uc5d0 \ud0c4\ud575\uc548 \ubc1c\uc758\uac00 \uc774\ub8e8\uc5b4\uc9c0\uc9c0 \ubabb\ud558\ub294\uc9c0 \uadf8\ub9ac\uace0 \uadf8 \ud0c4\ud575\uc18c\ucd94\uc548\uc5d0 \ub300\ud574\uc11c \ub204\uad6c\uc758 \ubd88\ucc38\uc73c\ub85c \uc778\ud574 \uc758\uacb0\uc774 \uc774\ub8e8\uc5b4\uc9c0\uc9c0 \ubabb\ud558\ub294\uc9c0\ub294 \ubd84\uba85\ud788 \uc81c\uac00 \ub05d\uae4c\uc9c0 \uad6d\ubbfc\uacfc \uacf5\uc720\ud574\uc57c \ud558\uaca0\uc2b5\ub2c8\ub2e4. \uadf8\uc5d0 \ub300\ud55c \ucc45\uc784\uc740 \ubd84\uba85\ud558\uace0 \uba85\ud655\ud558\uac8c \uc9c0\uaca0\uc2b5\ub2c8\ub2e4. \uc874\uacbd\ud558\uace0 \uc0ac\ub791\ud558\ub294 \ub3d9\ub8cc \uc758\uc6d0 \uc5ec\ub7ec\ubd84! \ubd80\ub514 \uad6d\ubbfc\uc758 \uc774 \uc544\ud508 \uc18c\ub9ac\ub97c \uc678\uba74\ud558\uc9c0 \ub9c8\uc2dc\uace0 \ubc15\uadfc\ud61c \ub300\ud1b5\ub839 \ud5cc\ubc95 \uc808\ucc28\uc5d0 \ub530\ub77c \ud0c4\ud575\uc744 \uc758\uacb0\ud560 \uc218 \uc788\ub3c4\ub85d \ucc38\uc5ec\ud558\uace0 \ub3c4\uc640\uc8fc\uc2dc\uae30 \ubc14\ub78d\ub2c8\ub2e4.", "answer": "80%", "sentence": "\uc9c0\uae08 \uc5ec\ub860\uc870\uc0ac \uacb0\uacfc\ub294 80% \uc774\uc0c1\uc758 \uad6d\ubbfc\uc774 \uc989\uc2dc \ubc15\uadfc\ud61c \ub300\ud1b5\ub839\uc744 \ud0c4\ud575\ud558\ub77c\uace0 \uc694\uad6c\ud558\uace0 \uc788\uc2b5\ub2c8\ub2e4.", "paragraph_sentence": "\ub300\ud55c\ubbfc\uad6d \uc81c18\ub300 \ud604\uc9c1 \ub300\ud1b5\ub839 \ubc15\uadfc\ud61c\ub294 \ubc94\uc8c4 \ud53c\uc758\uc790\uc785\ub2c8\ub2e4. \uadf8\uac83\ub3c4 \uad6d\uac00\uad8c\ub825\uc744 \uc0ac\uc720\ud654\ud574\uc11c \uc0ac\uc778\ub4e4\uc758 \ud638\uc8fc\uba38\ub2c8\uc5d0 \uad6d\uac00\uc138\uae08\uc744 \ud138\uc5b4 \ub123\uc740 \uac00\uc7a5 \uc9c8 \ub098\uc05c \ubc94\uc8c4\uc790\uc785\ub2c8\ub2e4. \uadf8\ub7f0\ub370 \uadf8\ub7f0 \ubc94\uc8c4 \ud53c\uc758\uc790\uac00 \uc9c0\uae08\uae4c\uc9c0 \uad6d\uac00\uad8c\ub825\uc744 \ud2c0\uc5b4\uc954 \ucc44 \uadf8 \uad6d\uac00 \uad8c\ub825\uc744 \uc790\uae30 \uac1c\uc778\uc758 \ubc29\uc5b4\uc640 \ubcf4\ud638\ub97c \uc704\ud574 \uc0ac\uc6a9\ud558\uace0 \uc788\uc2b5\ub2c8\ub2e4. \ub300\ud55c\ubbfc\uad6d \uad6d\ubbfc\ub4e4\uaed8\uc11c\ub294 \uc774 \uc0c1\ud669\uc744 \ub3c4\uc800\ud788 \ubc1b\uc544\ub4e4\uc77c \uc218\uac00 \uc5c6\ub2e4\uace0 \ud558\uc2ed\ub2c8\ub2e4. \ub9e4\uc8fc 100\ub9cc \uba85\uc774 \ub118\ub294 \uad6d\ubbfc\uaed8\uc11c \ucc28\uac00\uc6b4 \uad11\uc7a5\uc5d0 \ubaa8\uc5ec \ucd1b\ubd88\uc744 \ub4e4\uace0 \uacc4\uc2ed\ub2c8\ub2e4. \uc9c0\uae08 \uc5ec\ub860\uc870\uc0ac \uacb0\uacfc\ub294 80% \uc774\uc0c1\uc758 \uad6d\ubbfc\uc774 \uc989\uc2dc \ubc15\uadfc\ud61c \ub300\ud1b5\ub839\uc744 \ud0c4\ud575\ud558\ub77c\uace0 \uc694\uad6c\ud558\uace0 \uc788\uc2b5\ub2c8\ub2e4. \uad6d\ud68c\uc5d0 \uc694\uad6c\ud558\uace0 \uc788\uc2b5\ub2c8\ub2e4. \uadf8\ub7f0\ub370 \uc9c0\uae08 \uad6d\ud68c\ub294 \ubb34\uc5c7 \ud558\uace0 \uc788\uc2b5\ub2c8\uae4c? \uad6d\ubbfc\ub4e4\uc740 \uad6d\ud68c\ub97c \uc9c1\ubb34\uc720\uae30\ub85c \uace0\ubc1c\ud558\uace0 \uc788\uc2b5\ub2c8\ub2e4. (\uc911\ub7b5) \uadf8\ub807\uac8c \uc18c\ub9ac \ub192\uc5ec \ubc15\uadfc\ud61c \ub300\ud1b5\ub839\uc744 1\ubd84 1\ucd08\ub3c4 \uadf8 \uc790\ub9ac\uc5d0 \ub450\uc5b4\uc11c\ub294 \uc548 \ub41c\ub2e4\ub77c\uace0 \uc8fc\uc7a5\ud558\ub358, \uad6d\uac00\uc640 \uad6d\ubbfc\uc744 \uc704\ud574\uc11c \ubc15\uadfc\ud61c \ub300\ud1b5\ub839\uc740 \ubc14\ub85c \ub0b4\ub824\uc640\uc57c \ud55c\ub2e4\uace0 \uc694\uad6c\ud558\uace0 \uc8fc\uc7a5\ud558\ub358 \uc0c8\ub204\ub9ac\ub2f9 \uc758\uc6d0\ub4e4\uc758 \uc785\uc7a5\uc774 \ubc14\ub00c\uc5c8\uc2b5\ub2c8\ub2e4.\uc57c\ub2f9 \uc77c\ubd80\uc5d0\uc11c\ub3c4 \uc758\uacac\uc758 \ubcc0\ud654\uac00 \uac10\uc9c0\ub418\uace0 \ubcf4\ub3c4\ub418\uae30 \uc2dc\uc791\ud588\uc2b5\ub2c8\ub2e4. \uc800\ub294 \uac00\ub9cc\ud788 \uc788\uc744 \uc218 \uc5c6\uc5c8\uc2b5\ub2c8\ub2e4. \ub300\ud55c\ubbfc\uad6d \uad6d\ud68c\uc758\uc6d0\uc758 \ud55c \uc0ac\ub78c\uc73c\ub85c\uc11c, \uad6d\ubbfc\uc774 \ubf51\uc544 \uc8fc\uc2e0 \uad6d\ubbfc\uc758 \ub300\ud45c\ub85c\uc11c \uc800\ub3c4 \uadf8 \uc9c1\ubb34\uc720\uae30\uc758 \ud589\ub82c\uc5d0 \uacf5\ubc94\uc73c\ub85c \ub3d9\ucc38\ud560 \uc218 \uc5c6\uc5c8\uc2b5\ub2c8\ub2e4. \uadf8\ub798\uc11c \uc81c\uac00 \ud655\uc778\ud558\uace0 \ud655\ubcf4\ud55c \uacf5\uc801\uc778 \uc790\ub8cc\ub4e4\uc744 \ud1b5\ud574 \uc758\uc6d0 \uc5ec\ub7ec\ubd84 \ud55c \ubd84 \ud55c \ubd84\uc758 \ud0c4\ud575\uc5d0 \ub300\ud55c \ucc2c\ubc18 \uc785\uc7a5\uc744 \uad6d\ubbfc \uc5ec\ub7ec\ubd84\uacfc \uacf5\uc720\ud558\uace0 \uc788\uc2b5\ub2c8\ub2e4. \uadf8 \uacb0\uacfc \ub9ce\uc740 \uc758\uc6d0\ub4e4\uaed8\uc11c \uc5ec\ub7ec \uad6d\ubbfc\ub4e4\uc758 \uc804\ud654 \uc5f0\ub77d\uc744 \ubc1b\uc73c\uc2dc\uace0 \ubd88\ud3b8\ud558\ub2e4\ub294 \uc774\uc57c\uae30\ub97c \ub4e4\uc5c8\uc2b5\ub2c8\ub2e4. \uadf8 \ubd80\ubd84\uc5d0 \ub300\ud574\uc11c\ub294 \uc9c4\uc2ec\uc73c\ub85c \uc0ac\uacfc\uc758 \ub9d0\uc500\uc744 \ub4dc\ub9ac\uaca0\uc2b5\ub2c8\ub2e4. \uc544\uc6b8\ub7ec \uc624\ub298 \uc548\uc804\ud589\uc815\uc704\uc6d0\ud68c \ud68c\uc758 \ub3c4\uc911 \uc81c\uac00 \ud3c9\uc18c \uc88b\uc544\ud558\uace0 \uc0ac\ub791\ud558\uace0 \uc874\uacbd\ud558\ub294 \uc7a5\uc81c\uc6d0 \uc758\uc6d0\uacfc \uac10\uc815\uc801\uc778 \uc2f8\uc6c0\uae4c\uc9c0 \ud558\ub294 \uc0c1\ud669\uc5d0 \ucc98\ud588\uc2b5\ub2c8\ub2e4. \uadf8 \ubd80\ubd84 \uc5ed\uc2dc \uac1c\uc778\uc801\uc73c\ub85c \uc7a5\uc81c\uc6d0 \uc758\uc6d0\uaed8 \uacf5\uac1c\uc801\uc73c\ub85c \uc0ac\uacfc\ub4dc\ub9ac\uaca0\uc2b5\ub2c8\ub2e4. \ud558\uc9c0\ub9cc, \ud558\uc9c0\ub9cc, \uad6d\ud68c\uac00 \uc81c \uae30\ub2a5\uc744 \ud560 \ub54c\uae4c\uc9c0, \ubc15\uadfc\ud61c \ub300\ud1b5\ub839\uc5d0 \ub300\ud55c \ud0c4\ud575\uc18c\ucd94\uc548\uc5d0, \uadf8 \ubc1c\uc758\uc548\uc5d0 \ub204\uac00 \uc11c\uba85\ud558\uace0 \ub204\uac00 \uc11c\uba85\ud558\uc9c0 \uc54a\ub294\uc9c0, \ub204\uad6c\uc758 \uc11c\uba85 \ub54c\ubb38\uc5d0, \ub204\uad6c\uc758 \uc11c\uba85 \ubd88\ucc38 \ub54c\ubb38\uc5d0 \ud0c4\ud575\uc548 \ubc1c\uc758\uac00 \uc774\ub8e8\uc5b4\uc9c0\uc9c0 \ubabb\ud558\ub294\uc9c0 \uadf8\ub9ac\uace0 \uadf8 \ud0c4\ud575\uc18c\ucd94\uc548\uc5d0 \ub300\ud574\uc11c \ub204\uad6c\uc758 \ubd88\ucc38\uc73c\ub85c \uc778\ud574 \uc758\uacb0\uc774 \uc774\ub8e8\uc5b4\uc9c0\uc9c0 \ubabb\ud558\ub294\uc9c0\ub294 \ubd84\uba85\ud788 \uc81c\uac00 \ub05d\uae4c\uc9c0 \uad6d\ubbfc\uacfc \uacf5\uc720\ud574\uc57c \ud558\uaca0\uc2b5\ub2c8\ub2e4. \uadf8\uc5d0 \ub300\ud55c \ucc45\uc784\uc740 \ubd84\uba85\ud558\uace0 \uba85\ud655\ud558\uac8c \uc9c0\uaca0\uc2b5\ub2c8\ub2e4. \uc874\uacbd\ud558\uace0 \uc0ac\ub791\ud558\ub294 \ub3d9\ub8cc \uc758\uc6d0 \uc5ec\ub7ec\ubd84! \ubd80\ub514 \uad6d\ubbfc\uc758 \uc774 \uc544\ud508 \uc18c\ub9ac\ub97c \uc678\uba74\ud558\uc9c0 \ub9c8\uc2dc\uace0 \ubc15\uadfc\ud61c \ub300\ud1b5\ub839 \ud5cc\ubc95 \uc808\ucc28\uc5d0 \ub530\ub77c \ud0c4\ud575\uc744 \uc758\uacb0\ud560 \uc218 \uc788\ub3c4\ub85d \ucc38\uc5ec\ud558\uace0 \ub3c4\uc640\uc8fc\uc2dc\uae30 \ubc14\ub78d\ub2c8\ub2e4.", "paragraph_answer": "\ub300\ud55c\ubbfc\uad6d \uc81c18\ub300 \ud604\uc9c1 \ub300\ud1b5\ub839 \ubc15\uadfc\ud61c\ub294 \ubc94\uc8c4 \ud53c\uc758\uc790\uc785\ub2c8\ub2e4. \uadf8\uac83\ub3c4 \uad6d\uac00\uad8c\ub825\uc744 \uc0ac\uc720\ud654\ud574\uc11c \uc0ac\uc778\ub4e4\uc758 \ud638\uc8fc\uba38\ub2c8\uc5d0 \uad6d\uac00\uc138\uae08\uc744 \ud138\uc5b4 \ub123\uc740 \uac00\uc7a5 \uc9c8 \ub098\uc05c \ubc94\uc8c4\uc790\uc785\ub2c8\ub2e4. \uadf8\ub7f0\ub370 \uadf8\ub7f0 \ubc94\uc8c4 \ud53c\uc758\uc790\uac00 \uc9c0\uae08\uae4c\uc9c0 \uad6d\uac00\uad8c\ub825\uc744 \ud2c0\uc5b4\uc954 \ucc44 \uadf8 \uad6d\uac00 \uad8c\ub825\uc744 \uc790\uae30 \uac1c\uc778\uc758 \ubc29\uc5b4\uc640 \ubcf4\ud638\ub97c \uc704\ud574 \uc0ac\uc6a9\ud558\uace0 \uc788\uc2b5\ub2c8\ub2e4. \ub300\ud55c\ubbfc\uad6d \uad6d\ubbfc\ub4e4\uaed8\uc11c\ub294 \uc774 \uc0c1\ud669\uc744 \ub3c4\uc800\ud788 \ubc1b\uc544\ub4e4\uc77c \uc218\uac00 \uc5c6\ub2e4\uace0 \ud558\uc2ed\ub2c8\ub2e4. \ub9e4\uc8fc 100\ub9cc \uba85\uc774 \ub118\ub294 \uad6d\ubbfc\uaed8\uc11c \ucc28\uac00\uc6b4 \uad11\uc7a5\uc5d0 \ubaa8\uc5ec \ucd1b\ubd88\uc744 \ub4e4\uace0 \uacc4\uc2ed\ub2c8\ub2e4. \uc9c0\uae08 \uc5ec\ub860\uc870\uc0ac \uacb0\uacfc\ub294 80% \uc774\uc0c1\uc758 \uad6d\ubbfc\uc774 \uc989\uc2dc \ubc15\uadfc\ud61c \ub300\ud1b5\ub839\uc744 \ud0c4\ud575\ud558\ub77c\uace0 \uc694\uad6c\ud558\uace0 \uc788\uc2b5\ub2c8\ub2e4. \uad6d\ud68c\uc5d0 \uc694\uad6c\ud558\uace0 \uc788\uc2b5\ub2c8\ub2e4. \uadf8\ub7f0\ub370 \uc9c0\uae08 \uad6d\ud68c\ub294 \ubb34\uc5c7 \ud558\uace0 \uc788\uc2b5\ub2c8\uae4c? \uad6d\ubbfc\ub4e4\uc740 \uad6d\ud68c\ub97c \uc9c1\ubb34\uc720\uae30\ub85c \uace0\ubc1c\ud558\uace0 \uc788\uc2b5\ub2c8\ub2e4. (\uc911\ub7b5) \uadf8\ub807\uac8c \uc18c\ub9ac \ub192\uc5ec \ubc15\uadfc\ud61c \ub300\ud1b5\ub839\uc744 1\ubd84 1\ucd08\ub3c4 \uadf8 \uc790\ub9ac\uc5d0 \ub450\uc5b4\uc11c\ub294 \uc548 \ub41c\ub2e4\ub77c\uace0 \uc8fc\uc7a5\ud558\ub358, \uad6d\uac00\uc640 \uad6d\ubbfc\uc744 \uc704\ud574\uc11c \ubc15\uadfc\ud61c \ub300\ud1b5\ub839\uc740 \ubc14\ub85c \ub0b4\ub824\uc640\uc57c \ud55c\ub2e4\uace0 \uc694\uad6c\ud558\uace0 \uc8fc\uc7a5\ud558\ub358 \uc0c8\ub204\ub9ac\ub2f9 \uc758\uc6d0\ub4e4\uc758 \uc785\uc7a5\uc774 \ubc14\ub00c\uc5c8\uc2b5\ub2c8\ub2e4.\uc57c\ub2f9 \uc77c\ubd80\uc5d0\uc11c\ub3c4 \uc758\uacac\uc758 \ubcc0\ud654\uac00 \uac10\uc9c0\ub418\uace0 \ubcf4\ub3c4\ub418\uae30 \uc2dc\uc791\ud588\uc2b5\ub2c8\ub2e4. \uc800\ub294 \uac00\ub9cc\ud788 \uc788\uc744 \uc218 \uc5c6\uc5c8\uc2b5\ub2c8\ub2e4. \ub300\ud55c\ubbfc\uad6d \uad6d\ud68c\uc758\uc6d0\uc758 \ud55c \uc0ac\ub78c\uc73c\ub85c\uc11c, \uad6d\ubbfc\uc774 \ubf51\uc544 \uc8fc\uc2e0 \uad6d\ubbfc\uc758 \ub300\ud45c\ub85c\uc11c \uc800\ub3c4 \uadf8 \uc9c1\ubb34\uc720\uae30\uc758 \ud589\ub82c\uc5d0 \uacf5\ubc94\uc73c\ub85c \ub3d9\ucc38\ud560 \uc218 \uc5c6\uc5c8\uc2b5\ub2c8\ub2e4. \uadf8\ub798\uc11c \uc81c\uac00 \ud655\uc778\ud558\uace0 \ud655\ubcf4\ud55c \uacf5\uc801\uc778 \uc790\ub8cc\ub4e4\uc744 \ud1b5\ud574 \uc758\uc6d0 \uc5ec\ub7ec\ubd84 \ud55c \ubd84 \ud55c \ubd84\uc758 \ud0c4\ud575\uc5d0 \ub300\ud55c \ucc2c\ubc18 \uc785\uc7a5\uc744 \uad6d\ubbfc \uc5ec\ub7ec\ubd84\uacfc \uacf5\uc720\ud558\uace0 \uc788\uc2b5\ub2c8\ub2e4. \uadf8 \uacb0\uacfc \ub9ce\uc740 \uc758\uc6d0\ub4e4\uaed8\uc11c \uc5ec\ub7ec \uad6d\ubbfc\ub4e4\uc758 \uc804\ud654 \uc5f0\ub77d\uc744 \ubc1b\uc73c\uc2dc\uace0 \ubd88\ud3b8\ud558\ub2e4\ub294 \uc774\uc57c\uae30\ub97c \ub4e4\uc5c8\uc2b5\ub2c8\ub2e4. \uadf8 \ubd80\ubd84\uc5d0 \ub300\ud574\uc11c\ub294 \uc9c4\uc2ec\uc73c\ub85c \uc0ac\uacfc\uc758 \ub9d0\uc500\uc744 \ub4dc\ub9ac\uaca0\uc2b5\ub2c8\ub2e4. \uc544\uc6b8\ub7ec \uc624\ub298 \uc548\uc804\ud589\uc815\uc704\uc6d0\ud68c \ud68c\uc758 \ub3c4\uc911 \uc81c\uac00 \ud3c9\uc18c \uc88b\uc544\ud558\uace0 \uc0ac\ub791\ud558\uace0 \uc874\uacbd\ud558\ub294 \uc7a5\uc81c\uc6d0 \uc758\uc6d0\uacfc \uac10\uc815\uc801\uc778 \uc2f8\uc6c0\uae4c\uc9c0 \ud558\ub294 \uc0c1\ud669\uc5d0 \ucc98\ud588\uc2b5\ub2c8\ub2e4. \uadf8 \ubd80\ubd84 \uc5ed\uc2dc \uac1c\uc778\uc801\uc73c\ub85c \uc7a5\uc81c\uc6d0 \uc758\uc6d0\uaed8 \uacf5\uac1c\uc801\uc73c\ub85c \uc0ac\uacfc\ub4dc\ub9ac\uaca0\uc2b5\ub2c8\ub2e4. \ud558\uc9c0\ub9cc, \ud558\uc9c0\ub9cc, \uad6d\ud68c\uac00 \uc81c \uae30\ub2a5\uc744 \ud560 \ub54c\uae4c\uc9c0, \ubc15\uadfc\ud61c \ub300\ud1b5\ub839\uc5d0 \ub300\ud55c \ud0c4\ud575\uc18c\ucd94\uc548\uc5d0, \uadf8 \ubc1c\uc758\uc548\uc5d0 \ub204\uac00 \uc11c\uba85\ud558\uace0 \ub204\uac00 \uc11c\uba85\ud558\uc9c0 \uc54a\ub294\uc9c0, \ub204\uad6c\uc758 \uc11c\uba85 \ub54c\ubb38\uc5d0, \ub204\uad6c\uc758 \uc11c\uba85 \ubd88\ucc38 \ub54c\ubb38\uc5d0 \ud0c4\ud575\uc548 \ubc1c\uc758\uac00 \uc774\ub8e8\uc5b4\uc9c0\uc9c0 \ubabb\ud558\ub294\uc9c0 \uadf8\ub9ac\uace0 \uadf8 \ud0c4\ud575\uc18c\ucd94\uc548\uc5d0 \ub300\ud574\uc11c \ub204\uad6c\uc758 \ubd88\ucc38\uc73c\ub85c \uc778\ud574 \uc758\uacb0\uc774 \uc774\ub8e8\uc5b4\uc9c0\uc9c0 \ubabb\ud558\ub294\uc9c0\ub294 \ubd84\uba85\ud788 \uc81c\uac00 \ub05d\uae4c\uc9c0 \uad6d\ubbfc\uacfc \uacf5\uc720\ud574\uc57c \ud558\uaca0\uc2b5\ub2c8\ub2e4. \uadf8\uc5d0 \ub300\ud55c \ucc45\uc784\uc740 \ubd84\uba85\ud558\uace0 \uba85\ud655\ud558\uac8c \uc9c0\uaca0\uc2b5\ub2c8\ub2e4. \uc874\uacbd\ud558\uace0 \uc0ac\ub791\ud558\ub294 \ub3d9\ub8cc \uc758\uc6d0 \uc5ec\ub7ec\ubd84! \ubd80\ub514 \uad6d\ubbfc\uc758 \uc774 \uc544\ud508 \uc18c\ub9ac\ub97c \uc678\uba74\ud558\uc9c0 \ub9c8\uc2dc\uace0 \ubc15\uadfc\ud61c \ub300\ud1b5\ub839 \ud5cc\ubc95 \uc808\ucc28\uc5d0 \ub530\ub77c \ud0c4\ud575\uc744 \uc758\uacb0\ud560 \uc218 \uc788\ub3c4\ub85d \ucc38\uc5ec\ud558\uace0 \ub3c4\uc640\uc8fc\uc2dc\uae30 \ubc14\ub78d\ub2c8\ub2e4.", "sentence_answer": "\uc9c0\uae08 \uc5ec\ub860\uc870\uc0ac \uacb0\uacfc\ub294 80% \uc774\uc0c1\uc758 \uad6d\ubbfc\uc774 \uc989\uc2dc \ubc15\uadfc\ud61c \ub300\ud1b5\ub839\uc744 \ud0c4\ud575\ud558\ub77c\uace0 \uc694\uad6c\ud558\uace0 \uc788\uc2b5\ub2c8\ub2e4.", "paragraph_id": "6568380-3-0", "paragraph_question": "question: \ud45c\ucc3d\uc6d0\uc5d0 \ub530\ub974\uba74 \ubc15\uadfc\ud61c \ud0c4\ud575 \ucd1b\ubd88 \uc9d1\ud68c \ub2f9\uc2dc \uc5ec\ub860\uc870\uc0ac \uacb0\uacfc \uad6d\ubbfc\uc758 \uba87 \ud504\ub85c\uac00 \uc989\uc2dc \ud0c4\ud575\uc744 \uc694\uad6c\ud558\uace0 \uc788\ub2e4\uace0 \ud558\uc600\ub294\uac00?, context: \ub300\ud55c\ubbfc\uad6d \uc81c18\ub300 \ud604\uc9c1 \ub300\ud1b5\ub839 \ubc15\uadfc\ud61c\ub294 \ubc94\uc8c4 \ud53c\uc758\uc790\uc785\ub2c8\ub2e4. \uadf8\uac83\ub3c4 \uad6d\uac00\uad8c\ub825\uc744 \uc0ac\uc720\ud654\ud574\uc11c \uc0ac\uc778\ub4e4\uc758 \ud638\uc8fc\uba38\ub2c8\uc5d0 \uad6d\uac00\uc138\uae08\uc744 \ud138\uc5b4 \ub123\uc740 \uac00\uc7a5 \uc9c8 \ub098\uc05c \ubc94\uc8c4\uc790\uc785\ub2c8\ub2e4. \uadf8\ub7f0\ub370 \uadf8\ub7f0 \ubc94\uc8c4 \ud53c\uc758\uc790\uac00 \uc9c0\uae08\uae4c\uc9c0 \uad6d\uac00\uad8c\ub825\uc744 \ud2c0\uc5b4\uc954 \ucc44 \uadf8 \uad6d\uac00 \uad8c\ub825\uc744 \uc790\uae30 \uac1c\uc778\uc758 \ubc29\uc5b4\uc640 \ubcf4\ud638\ub97c \uc704\ud574 \uc0ac\uc6a9\ud558\uace0 \uc788\uc2b5\ub2c8\ub2e4. \ub300\ud55c\ubbfc\uad6d \uad6d\ubbfc\ub4e4\uaed8\uc11c\ub294 \uc774 \uc0c1\ud669\uc744 \ub3c4\uc800\ud788 \ubc1b\uc544\ub4e4\uc77c \uc218\uac00 \uc5c6\ub2e4\uace0 \ud558\uc2ed\ub2c8\ub2e4. \ub9e4\uc8fc 100\ub9cc \uba85\uc774 \ub118\ub294 \uad6d\ubbfc\uaed8\uc11c \ucc28\uac00\uc6b4 \uad11\uc7a5\uc5d0 \ubaa8\uc5ec \ucd1b\ubd88\uc744 \ub4e4\uace0 \uacc4\uc2ed\ub2c8\ub2e4. \uc9c0\uae08 \uc5ec\ub860\uc870\uc0ac \uacb0\uacfc\ub294 80% \uc774\uc0c1\uc758 \uad6d\ubbfc\uc774 \uc989\uc2dc \ubc15\uadfc\ud61c \ub300\ud1b5\ub839\uc744 \ud0c4\ud575\ud558\ub77c\uace0 \uc694\uad6c\ud558\uace0 \uc788\uc2b5\ub2c8\ub2e4. \uad6d\ud68c\uc5d0 \uc694\uad6c\ud558\uace0 \uc788\uc2b5\ub2c8\ub2e4. \uadf8\ub7f0\ub370 \uc9c0\uae08 \uad6d\ud68c\ub294 \ubb34\uc5c7 \ud558\uace0 \uc788\uc2b5\ub2c8\uae4c? \uad6d\ubbfc\ub4e4\uc740 \uad6d\ud68c\ub97c \uc9c1\ubb34\uc720\uae30\ub85c \uace0\ubc1c\ud558\uace0 \uc788\uc2b5\ub2c8\ub2e4. (\uc911\ub7b5) \uadf8\ub807\uac8c \uc18c\ub9ac \ub192\uc5ec \ubc15\uadfc\ud61c \ub300\ud1b5\ub839\uc744 1\ubd84 1\ucd08\ub3c4 \uadf8 \uc790\ub9ac\uc5d0 \ub450\uc5b4\uc11c\ub294 \uc548 \ub41c\ub2e4\ub77c\uace0 \uc8fc\uc7a5\ud558\ub358, \uad6d\uac00\uc640 \uad6d\ubbfc\uc744 \uc704\ud574\uc11c \ubc15\uadfc\ud61c \ub300\ud1b5\ub839\uc740 \ubc14\ub85c \ub0b4\ub824\uc640\uc57c \ud55c\ub2e4\uace0 \uc694\uad6c\ud558\uace0 \uc8fc\uc7a5\ud558\ub358 \uc0c8\ub204\ub9ac\ub2f9 \uc758\uc6d0\ub4e4\uc758 \uc785\uc7a5\uc774 \ubc14\ub00c\uc5c8\uc2b5\ub2c8\ub2e4.\uc57c\ub2f9 \uc77c\ubd80\uc5d0\uc11c\ub3c4 \uc758\uacac\uc758 \ubcc0\ud654\uac00 \uac10\uc9c0\ub418\uace0 \ubcf4\ub3c4\ub418\uae30 \uc2dc\uc791\ud588\uc2b5\ub2c8\ub2e4. \uc800\ub294 \uac00\ub9cc\ud788 \uc788\uc744 \uc218 \uc5c6\uc5c8\uc2b5\ub2c8\ub2e4. \ub300\ud55c\ubbfc\uad6d \uad6d\ud68c\uc758\uc6d0\uc758 \ud55c \uc0ac\ub78c\uc73c\ub85c\uc11c, \uad6d\ubbfc\uc774 \ubf51\uc544 \uc8fc\uc2e0 \uad6d\ubbfc\uc758 \ub300\ud45c\ub85c\uc11c \uc800\ub3c4 \uadf8 \uc9c1\ubb34\uc720\uae30\uc758 \ud589\ub82c\uc5d0 \uacf5\ubc94\uc73c\ub85c \ub3d9\ucc38\ud560 \uc218 \uc5c6\uc5c8\uc2b5\ub2c8\ub2e4. \uadf8\ub798\uc11c \uc81c\uac00 \ud655\uc778\ud558\uace0 \ud655\ubcf4\ud55c \uacf5\uc801\uc778 \uc790\ub8cc\ub4e4\uc744 \ud1b5\ud574 \uc758\uc6d0 \uc5ec\ub7ec\ubd84 \ud55c \ubd84 \ud55c \ubd84\uc758 \ud0c4\ud575\uc5d0 \ub300\ud55c \ucc2c\ubc18 \uc785\uc7a5\uc744 \uad6d\ubbfc \uc5ec\ub7ec\ubd84\uacfc \uacf5\uc720\ud558\uace0 \uc788\uc2b5\ub2c8\ub2e4. \uadf8 \uacb0\uacfc \ub9ce\uc740 \uc758\uc6d0\ub4e4\uaed8\uc11c \uc5ec\ub7ec \uad6d\ubbfc\ub4e4\uc758 \uc804\ud654 \uc5f0\ub77d\uc744 \ubc1b\uc73c\uc2dc\uace0 \ubd88\ud3b8\ud558\ub2e4\ub294 \uc774\uc57c\uae30\ub97c \ub4e4\uc5c8\uc2b5\ub2c8\ub2e4. \uadf8 \ubd80\ubd84\uc5d0 \ub300\ud574\uc11c\ub294 \uc9c4\uc2ec\uc73c\ub85c \uc0ac\uacfc\uc758 \ub9d0\uc500\uc744 \ub4dc\ub9ac\uaca0\uc2b5\ub2c8\ub2e4. \uc544\uc6b8\ub7ec \uc624\ub298 \uc548\uc804\ud589\uc815\uc704\uc6d0\ud68c \ud68c\uc758 \ub3c4\uc911 \uc81c\uac00 \ud3c9\uc18c \uc88b\uc544\ud558\uace0 \uc0ac\ub791\ud558\uace0 \uc874\uacbd\ud558\ub294 \uc7a5\uc81c\uc6d0 \uc758\uc6d0\uacfc \uac10\uc815\uc801\uc778 \uc2f8\uc6c0\uae4c\uc9c0 \ud558\ub294 \uc0c1\ud669\uc5d0 \ucc98\ud588\uc2b5\ub2c8\ub2e4. \uadf8 \ubd80\ubd84 \uc5ed\uc2dc \uac1c\uc778\uc801\uc73c\ub85c \uc7a5\uc81c\uc6d0 \uc758\uc6d0\uaed8 \uacf5\uac1c\uc801\uc73c\ub85c \uc0ac\uacfc\ub4dc\ub9ac\uaca0\uc2b5\ub2c8\ub2e4. \ud558\uc9c0\ub9cc, \ud558\uc9c0\ub9cc, \uad6d\ud68c\uac00 \uc81c \uae30\ub2a5\uc744 \ud560 \ub54c\uae4c\uc9c0, \ubc15\uadfc\ud61c \ub300\ud1b5\ub839\uc5d0 \ub300\ud55c \ud0c4\ud575\uc18c\ucd94\uc548\uc5d0, \uadf8 \ubc1c\uc758\uc548\uc5d0 \ub204\uac00 \uc11c\uba85\ud558\uace0 \ub204\uac00 \uc11c\uba85\ud558\uc9c0 \uc54a\ub294\uc9c0, \ub204\uad6c\uc758 \uc11c\uba85 \ub54c\ubb38\uc5d0, \ub204\uad6c\uc758 \uc11c\uba85 \ubd88\ucc38 \ub54c\ubb38\uc5d0 \ud0c4\ud575\uc548 \ubc1c\uc758\uac00 \uc774\ub8e8\uc5b4\uc9c0\uc9c0 \ubabb\ud558\ub294\uc9c0 \uadf8\ub9ac\uace0 \uadf8 \ud0c4\ud575\uc18c\ucd94\uc548\uc5d0 \ub300\ud574\uc11c \ub204\uad6c\uc758 \ubd88\ucc38\uc73c\ub85c \uc778\ud574 \uc758\uacb0\uc774 \uc774\ub8e8\uc5b4\uc9c0\uc9c0 \ubabb\ud558\ub294\uc9c0\ub294 \ubd84\uba85\ud788 \uc81c\uac00 \ub05d\uae4c\uc9c0 \uad6d\ubbfc\uacfc \uacf5\uc720\ud574\uc57c \ud558\uaca0\uc2b5\ub2c8\ub2e4. \uadf8\uc5d0 \ub300\ud55c \ucc45\uc784\uc740 \ubd84\uba85\ud558\uace0 \uba85\ud655\ud558\uac8c \uc9c0\uaca0\uc2b5\ub2c8\ub2e4. \uc874\uacbd\ud558\uace0 \uc0ac\ub791\ud558\ub294 \ub3d9\ub8cc \uc758\uc6d0 \uc5ec\ub7ec\ubd84! \ubd80\ub514 \uad6d\ubbfc\uc758 \uc774 \uc544\ud508 \uc18c\ub9ac\ub97c \uc678\uba74\ud558\uc9c0 \ub9c8\uc2dc\uace0 \ubc15\uadfc\ud61c \ub300\ud1b5\ub839 \ud5cc\ubc95 \uc808\ucc28\uc5d0 \ub530\ub77c \ud0c4\ud575\uc744 \uc758\uacb0\ud560 \uc218 \uc788\ub3c4\ub85d \ucc38\uc5ec\ud558\uace0 \ub3c4\uc640\uc8fc\uc2dc\uae30 \ubc14\ub78d\ub2c8\ub2e4."} {"question": "2010\ub144\uc5d0\ub294 \uac00\uc218 \uace0 \uae40\ud604\uc2dd\uc758 \ucd94\ubaa8 \uc601\ud654\ub85c \uae30\ud68d\ub41c \uc601\ud654\ub294", "paragraph": "\uc601\ud654\uc5d0 \ub9e4\uc9c4\ud558\ub358 \uc784\ucc3d\uc815\uc740 2008\ub144\uc5d0\ub294 \uacc4\uc57d\ud588\ub358 \uc601\ud654\uac00 \ubb34\uae30\ud55c \uc81c\uc791 \uc5f0\uae30\ub418\uba70 1\ub144\uac04 \uacf5\ubc31\uae30\ub97c \uac00\uc9c0\uae30\ub3c4 \ud588\uc73c\ub098, 2009\ub144 3\uc6d4 10\uc77c 6\ub144 \ub9cc\uc5d0 \uc815\uaddc 11\uc9d1 \u300aReturn To My World\u300b\ub97c \ubc1c\ud558 \ubcf5\uadc0 \uba85\ub791\uc528\uc5b4\ud130 \uc218\ubc15\uc758 \ubba4\uc9c0\uceec <\ube68\ub798 >\uc5d0 \uc194\ub871\uace0 \uc5ed\uc73c\ub85c \ubba4\uc9c0\uceec\uc5d0 \ucd9c\uc5f0\ud558\uae30\ub3c4 \ud588\ub2e4. \uac19\uc740 \ud574\uc5d0 \uae40\ucc3d\ub82c, \uc774\ud558\ub298\uacfc \ud568\uaed8 \ucc3d\ub2e8\uba64\ubc84\ub85c\uc368 KBS \ucc9c\ud558\ubb34\uc801\uc57c\uad6c\ub2e8 \ucd9c\uc5f0\ud558\uc600\uc73c\ub098, 2010\ub144 6\uc6d4 \ucd5c\uc885 \ud558\ucc28\ud558\uc600\ub2e4. 2009\ub144 \uc601\ud654 \u300a\uccad\ub2f4\ubcf4\uc0b4\u300b\uc774 \uac1c\ubd09\ub418\uc5c8\uace0 2010\ub144 \ubca0\uc2a4\ud2b8 \uc568\ubc94&\ub9ac\uba54\uc774\ud06c \uc568\ubc94\uc778 \u300aRemind\u300b\ub97c \ubc1c\ub9e4\ud558\uc600\ub2e4. 2010\ub144\uc5d0\ub294 \uac00\uc218 \u6545 \uae40\ud604\uc2dd\uc758 \ucd94\ubaa8 \uc601\ud654\ub85c \uae30\ud68d, \uc81c\uc791\ud55c \u300a\ube44\ucc98\ub7fc \uc74c\uc545\ucc98\ub7fc\u300b\uc740 \ucd2c\uc601\uc744 \ub9c8\uce5c \ub4a4 \ud6c4\ubc18 \uc791\uc5c5 \uc911 \u6545 \uae40\ub0a8\uacbd \uac10\ub3c5\uc758 \uac11\uc791\uc2a4\ub7f0 \uc0ac\ub9dd\uc73c\ub85c \uac1c\ubd09\uc774 \ubbf8\ub904\uc9c0\ub2e4 2012\ub144 4\uc6d4 \uac1c\ubd09\ud558\ub824 \ud588\uc73c\ub098 \ubb34\uae30\ud55c \uc5f0\uae30\ub418\uc5c8\ub2e4. \uac19\uc740 \ud574 11\uc6d4\uc5d0\ub294 \uc5c4\uc9c0\uc6d0\uacfc \ud568\uaed8 \uc8fc\uc5f0\ud55c \uc601\ud654\u300a\ubd88\ub7c9\ub0a8\ub140\u300b\uac00 \uac1c\ubd09\ub418\uc5c8\ub2e4. 2011\ub144 \u300a\uc0ac\ub791\uc774 \ubb34\uc11c\uc6cc\u300b, 2012\ub144 \u300a\uacf5\ubaa8\uc790\ub4e4\u300b\uc5d0 \ucd9c\uc5f0 \ud558\uc600\uc73c\uba70 \ub610\ud55c \uc601\ud654 \u300a\ucc3d\uc218\u300b\uc758 \uc8fc\uc5f0\uc744 \ub9e1\uc558\ub2e4. \ud574\ub2f9 \uc791\ud488\uc740 2012\ub144 \uc81c17\ud68c \ubd80\uc0b0\uad6d\uc81c\uc601\ud654\uc81c\uc5d0 \ucd08\uccad \ubc0f \uc0c1\uc601\ub418\uc5c8\uc73c\uba70 \uc774\ud6c4 2013\ub144 11\uc6d4 \uc815\uc2dd \uac1c\ubd09\ud558\uc600\ub2e4. 2015\ub144\uc5d0\ub294 \u300a\uce58\uc678\ubc95\uad8c\u300b\uc774 \uac1c\ubd09\ud588\uc9c0\ub9cc \uc88b\uc740 \uc131\uc801\uc744 \uac70\ub450\uc9c0\ub294 \ubabb\ud558\uc600\ub2e4.", "answer": "\ube44\ucc98\ub7fc \uc74c\uc545\ucc98\ub7fc", "sentence": "2010\ub144\uc5d0\ub294 \uac00\uc218 \u6545 \uae40\ud604\uc2dd\uc758 \ucd94\ubaa8 \uc601\ud654\ub85c \uae30\ud68d, \uc81c\uc791\ud55c \u300a \ube44\ucc98\ub7fc \uc74c\uc545\ucc98\ub7fc \u300b\uc740 \ucd2c\uc601\uc744 \ub9c8\uce5c \ub4a4 \ud6c4\ubc18 \uc791\uc5c5 \uc911 \u6545 \uae40\ub0a8\uacbd \uac10\ub3c5\uc758 \uac11\uc791\uc2a4\ub7f0 \uc0ac\ub9dd\uc73c\ub85c \uac1c\ubd09\uc774 \ubbf8\ub904\uc9c0\ub2e4 2012\ub144 4\uc6d4 \uac1c\ubd09\ud558\ub824 \ud588\uc73c\ub098 \ubb34\uae30\ud55c \uc5f0\uae30\ub418\uc5c8\ub2e4.", "paragraph_sentence": "\uc601\ud654\uc5d0 \ub9e4\uc9c4\ud558\ub358 \uc784\ucc3d\uc815\uc740 2008\ub144\uc5d0\ub294 \uacc4\uc57d\ud588\ub358 \uc601\ud654\uac00 \ubb34\uae30\ud55c \uc81c\uc791 \uc5f0\uae30\ub418\uba70 1\ub144\uac04 \uacf5\ubc31\uae30\ub97c \uac00\uc9c0\uae30\ub3c4 \ud588\uc73c\ub098, 2009\ub144 3\uc6d4 10\uc77c 6\ub144 \ub9cc\uc5d0 \uc815\uaddc 11\uc9d1 \u300aReturn To My World\u300b\ub97c \ubc1c\ud558 \ubcf5\uadc0 \uba85\ub791\uc528\uc5b4\ud130 \uc218\ubc15\uc758 \ubba4\uc9c0\uceec <\ube68\ub798 >\uc5d0 \uc194\ub871\uace0 \uc5ed\uc73c\ub85c \ubba4\uc9c0\uceec\uc5d0 \ucd9c\uc5f0\ud558\uae30\ub3c4 \ud588\ub2e4. \uac19\uc740 \ud574\uc5d0 \uae40\ucc3d\ub82c, \uc774\ud558\ub298\uacfc \ud568\uaed8 \ucc3d\ub2e8\uba64\ubc84\ub85c\uc368 KBS \ucc9c\ud558\ubb34\uc801\uc57c\uad6c\ub2e8 \ucd9c\uc5f0\ud558\uc600\uc73c\ub098, 2010\ub144 6\uc6d4 \ucd5c\uc885 \ud558\ucc28\ud558\uc600\ub2e4. 2009\ub144 \uc601\ud654 \u300a\uccad\ub2f4\ubcf4\uc0b4\u300b\uc774 \uac1c\ubd09\ub418\uc5c8\uace0 2010\ub144 \ubca0\uc2a4\ud2b8 \uc568\ubc94&\ub9ac\uba54\uc774\ud06c \uc568\ubc94\uc778 \u300aRemind\u300b\ub97c \ubc1c\ub9e4\ud558\uc600\ub2e4. 2010\ub144\uc5d0\ub294 \uac00\uc218 \u6545 \uae40\ud604\uc2dd\uc758 \ucd94\ubaa8 \uc601\ud654\ub85c \uae30\ud68d, \uc81c\uc791\ud55c \u300a \ube44\ucc98\ub7fc \uc74c\uc545\ucc98\ub7fc \u300b\uc740 \ucd2c\uc601\uc744 \ub9c8\uce5c \ub4a4 \ud6c4\ubc18 \uc791\uc5c5 \uc911 \u6545 \uae40\ub0a8\uacbd \uac10\ub3c5\uc758 \uac11\uc791\uc2a4\ub7f0 \uc0ac\ub9dd\uc73c\ub85c \uac1c\ubd09\uc774 \ubbf8\ub904\uc9c0\ub2e4 2012\ub144 4\uc6d4 \uac1c\ubd09\ud558\ub824 \ud588\uc73c\ub098 \ubb34\uae30\ud55c \uc5f0\uae30\ub418\uc5c8\ub2e4. \uac19\uc740 \ud574 11\uc6d4\uc5d0\ub294 \uc5c4\uc9c0\uc6d0\uacfc \ud568\uaed8 \uc8fc\uc5f0\ud55c \uc601\ud654\u300a\ubd88\ub7c9\ub0a8\ub140\u300b\uac00 \uac1c\ubd09\ub418\uc5c8\ub2e4. 2011\ub144 \u300a\uc0ac\ub791\uc774 \ubb34\uc11c\uc6cc\u300b, 2012\ub144 \u300a\uacf5\ubaa8\uc790\ub4e4\u300b\uc5d0 \ucd9c\uc5f0 \ud558\uc600\uc73c\uba70 \ub610\ud55c \uc601\ud654 \u300a\ucc3d\uc218\u300b\uc758 \uc8fc\uc5f0\uc744 \ub9e1\uc558\ub2e4. \ud574\ub2f9 \uc791\ud488\uc740 2012\ub144 \uc81c17\ud68c \ubd80\uc0b0\uad6d\uc81c\uc601\ud654\uc81c\uc5d0 \ucd08\uccad \ubc0f \uc0c1\uc601\ub418\uc5c8\uc73c\uba70 \uc774\ud6c4 2013\ub144 11\uc6d4 \uc815\uc2dd \uac1c\ubd09\ud558\uc600\ub2e4. 2015\ub144\uc5d0\ub294 \u300a\uce58\uc678\ubc95\uad8c\u300b\uc774 \uac1c\ubd09\ud588\uc9c0\ub9cc \uc88b\uc740 \uc131\uc801\uc744 \uac70\ub450\uc9c0\ub294 \ubabb\ud558\uc600\ub2e4.", "paragraph_answer": "\uc601\ud654\uc5d0 \ub9e4\uc9c4\ud558\ub358 \uc784\ucc3d\uc815\uc740 2008\ub144\uc5d0\ub294 \uacc4\uc57d\ud588\ub358 \uc601\ud654\uac00 \ubb34\uae30\ud55c \uc81c\uc791 \uc5f0\uae30\ub418\uba70 1\ub144\uac04 \uacf5\ubc31\uae30\ub97c \uac00\uc9c0\uae30\ub3c4 \ud588\uc73c\ub098, 2009\ub144 3\uc6d4 10\uc77c 6\ub144 \ub9cc\uc5d0 \uc815\uaddc 11\uc9d1 \u300aReturn To My World\u300b\ub97c \ubc1c\ud558 \ubcf5\uadc0 \uba85\ub791\uc528\uc5b4\ud130 \uc218\ubc15\uc758 \ubba4\uc9c0\uceec <\ube68\ub798 >\uc5d0 \uc194\ub871\uace0 \uc5ed\uc73c\ub85c \ubba4\uc9c0\uceec\uc5d0 \ucd9c\uc5f0\ud558\uae30\ub3c4 \ud588\ub2e4. \uac19\uc740 \ud574\uc5d0 \uae40\ucc3d\ub82c, \uc774\ud558\ub298\uacfc \ud568\uaed8 \ucc3d\ub2e8\uba64\ubc84\ub85c\uc368 KBS \ucc9c\ud558\ubb34\uc801\uc57c\uad6c\ub2e8 \ucd9c\uc5f0\ud558\uc600\uc73c\ub098, 2010\ub144 6\uc6d4 \ucd5c\uc885 \ud558\ucc28\ud558\uc600\ub2e4. 2009\ub144 \uc601\ud654 \u300a\uccad\ub2f4\ubcf4\uc0b4\u300b\uc774 \uac1c\ubd09\ub418\uc5c8\uace0 2010\ub144 \ubca0\uc2a4\ud2b8 \uc568\ubc94&\ub9ac\uba54\uc774\ud06c \uc568\ubc94\uc778 \u300aRemind\u300b\ub97c \ubc1c\ub9e4\ud558\uc600\ub2e4. 2010\ub144\uc5d0\ub294 \uac00\uc218 \u6545 \uae40\ud604\uc2dd\uc758 \ucd94\ubaa8 \uc601\ud654\ub85c \uae30\ud68d, \uc81c\uc791\ud55c \u300a \ube44\ucc98\ub7fc \uc74c\uc545\ucc98\ub7fc \u300b\uc740 \ucd2c\uc601\uc744 \ub9c8\uce5c \ub4a4 \ud6c4\ubc18 \uc791\uc5c5 \uc911 \u6545 \uae40\ub0a8\uacbd \uac10\ub3c5\uc758 \uac11\uc791\uc2a4\ub7f0 \uc0ac\ub9dd\uc73c\ub85c \uac1c\ubd09\uc774 \ubbf8\ub904\uc9c0\ub2e4 2012\ub144 4\uc6d4 \uac1c\ubd09\ud558\ub824 \ud588\uc73c\ub098 \ubb34\uae30\ud55c \uc5f0\uae30\ub418\uc5c8\ub2e4. \uac19\uc740 \ud574 11\uc6d4\uc5d0\ub294 \uc5c4\uc9c0\uc6d0\uacfc \ud568\uaed8 \uc8fc\uc5f0\ud55c \uc601\ud654\u300a\ubd88\ub7c9\ub0a8\ub140\u300b\uac00 \uac1c\ubd09\ub418\uc5c8\ub2e4. 2011\ub144 \u300a\uc0ac\ub791\uc774 \ubb34\uc11c\uc6cc\u300b, 2012\ub144 \u300a\uacf5\ubaa8\uc790\ub4e4\u300b\uc5d0 \ucd9c\uc5f0 \ud558\uc600\uc73c\uba70 \ub610\ud55c \uc601\ud654 \u300a\ucc3d\uc218\u300b\uc758 \uc8fc\uc5f0\uc744 \ub9e1\uc558\ub2e4. \ud574\ub2f9 \uc791\ud488\uc740 2012\ub144 \uc81c17\ud68c \ubd80\uc0b0\uad6d\uc81c\uc601\ud654\uc81c\uc5d0 \ucd08\uccad \ubc0f \uc0c1\uc601\ub418\uc5c8\uc73c\uba70 \uc774\ud6c4 2013\ub144 11\uc6d4 \uc815\uc2dd \uac1c\ubd09\ud558\uc600\ub2e4. 2015\ub144\uc5d0\ub294 \u300a\uce58\uc678\ubc95\uad8c\u300b\uc774 \uac1c\ubd09\ud588\uc9c0\ub9cc \uc88b\uc740 \uc131\uc801\uc744 \uac70\ub450\uc9c0\ub294 \ubabb\ud558\uc600\ub2e4.", "sentence_answer": "2010\ub144\uc5d0\ub294 \uac00\uc218 \u6545 \uae40\ud604\uc2dd\uc758 \ucd94\ubaa8 \uc601\ud654\ub85c \uae30\ud68d, \uc81c\uc791\ud55c \u300a \ube44\ucc98\ub7fc \uc74c\uc545\ucc98\ub7fc \u300b\uc740 \ucd2c\uc601\uc744 \ub9c8\uce5c \ub4a4 \ud6c4\ubc18 \uc791\uc5c5 \uc911 \u6545 \uae40\ub0a8\uacbd \uac10\ub3c5\uc758 \uac11\uc791\uc2a4\ub7f0 \uc0ac\ub9dd\uc73c\ub85c \uac1c\ubd09\uc774 \ubbf8\ub904\uc9c0\ub2e4 2012\ub144 4\uc6d4 \uac1c\ubd09\ud558\ub824 \ud588\uc73c\ub098 \ubb34\uae30\ud55c \uc5f0\uae30\ub418\uc5c8\ub2e4.", "paragraph_id": "5856097-1-1", "paragraph_question": "question: 2010\ub144\uc5d0\ub294 \uac00\uc218 \uace0 \uae40\ud604\uc2dd\uc758 \ucd94\ubaa8 \uc601\ud654\ub85c \uae30\ud68d\ub41c \uc601\ud654\ub294, context: \uc601\ud654\uc5d0 \ub9e4\uc9c4\ud558\ub358 \uc784\ucc3d\uc815\uc740 2008\ub144\uc5d0\ub294 \uacc4\uc57d\ud588\ub358 \uc601\ud654\uac00 \ubb34\uae30\ud55c \uc81c\uc791 \uc5f0\uae30\ub418\uba70 1\ub144\uac04 \uacf5\ubc31\uae30\ub97c \uac00\uc9c0\uae30\ub3c4 \ud588\uc73c\ub098, 2009\ub144 3\uc6d4 10\uc77c 6\ub144 \ub9cc\uc5d0 \uc815\uaddc 11\uc9d1 \u300aReturn To My World\u300b\ub97c \ubc1c\ud558 \ubcf5\uadc0 \uba85\ub791\uc528\uc5b4\ud130 \uc218\ubc15\uc758 \ubba4\uc9c0\uceec <\ube68\ub798 >\uc5d0 \uc194\ub871\uace0 \uc5ed\uc73c\ub85c \ubba4\uc9c0\uceec\uc5d0 \ucd9c\uc5f0\ud558\uae30\ub3c4 \ud588\ub2e4. \uac19\uc740 \ud574\uc5d0 \uae40\ucc3d\ub82c, \uc774\ud558\ub298\uacfc \ud568\uaed8 \ucc3d\ub2e8\uba64\ubc84\ub85c\uc368 KBS \ucc9c\ud558\ubb34\uc801\uc57c\uad6c\ub2e8 \ucd9c\uc5f0\ud558\uc600\uc73c\ub098, 2010\ub144 6\uc6d4 \ucd5c\uc885 \ud558\ucc28\ud558\uc600\ub2e4. 2009\ub144 \uc601\ud654 \u300a\uccad\ub2f4\ubcf4\uc0b4\u300b\uc774 \uac1c\ubd09\ub418\uc5c8\uace0 2010\ub144 \ubca0\uc2a4\ud2b8 \uc568\ubc94&\ub9ac\uba54\uc774\ud06c \uc568\ubc94\uc778 \u300aRemind\u300b\ub97c \ubc1c\ub9e4\ud558\uc600\ub2e4. 2010\ub144\uc5d0\ub294 \uac00\uc218 \u6545 \uae40\ud604\uc2dd\uc758 \ucd94\ubaa8 \uc601\ud654\ub85c \uae30\ud68d, \uc81c\uc791\ud55c \u300a\ube44\ucc98\ub7fc \uc74c\uc545\ucc98\ub7fc\u300b\uc740 \ucd2c\uc601\uc744 \ub9c8\uce5c \ub4a4 \ud6c4\ubc18 \uc791\uc5c5 \uc911 \u6545 \uae40\ub0a8\uacbd \uac10\ub3c5\uc758 \uac11\uc791\uc2a4\ub7f0 \uc0ac\ub9dd\uc73c\ub85c \uac1c\ubd09\uc774 \ubbf8\ub904\uc9c0\ub2e4 2012\ub144 4\uc6d4 \uac1c\ubd09\ud558\ub824 \ud588\uc73c\ub098 \ubb34\uae30\ud55c \uc5f0\uae30\ub418\uc5c8\ub2e4. \uac19\uc740 \ud574 11\uc6d4\uc5d0\ub294 \uc5c4\uc9c0\uc6d0\uacfc \ud568\uaed8 \uc8fc\uc5f0\ud55c \uc601\ud654\u300a\ubd88\ub7c9\ub0a8\ub140\u300b\uac00 \uac1c\ubd09\ub418\uc5c8\ub2e4. 2011\ub144 \u300a\uc0ac\ub791\uc774 \ubb34\uc11c\uc6cc\u300b, 2012\ub144 \u300a\uacf5\ubaa8\uc790\ub4e4\u300b\uc5d0 \ucd9c\uc5f0 \ud558\uc600\uc73c\uba70 \ub610\ud55c \uc601\ud654 \u300a\ucc3d\uc218\u300b\uc758 \uc8fc\uc5f0\uc744 \ub9e1\uc558\ub2e4. \ud574\ub2f9 \uc791\ud488\uc740 2012\ub144 \uc81c17\ud68c \ubd80\uc0b0\uad6d\uc81c\uc601\ud654\uc81c\uc5d0 \ucd08\uccad \ubc0f \uc0c1\uc601\ub418\uc5c8\uc73c\uba70 \uc774\ud6c4 2013\ub144 11\uc6d4 \uc815\uc2dd \uac1c\ubd09\ud558\uc600\ub2e4. 2015\ub144\uc5d0\ub294 \u300a\uce58\uc678\ubc95\uad8c\u300b\uc774 \uac1c\ubd09\ud588\uc9c0\ub9cc \uc88b\uc740 \uc131\uc801\uc744 \uac70\ub450\uc9c0\ub294 \ubabb\ud558\uc600\ub2e4."} {"question": "\uc6b4\ub3d9\uc744 \ud560 \uacbd\uc6b0 \ubc29\ucd9c\ub418\ub294 \uc694\uc18c\ub85c \ub274\ub7f0\uacfc \uc2dc\ub0c5\uc2a4\uc758 \ubc1c\ub2ec\uc744 \uc99d\uac00\uc2dc\ud0a4\ub294 \ubc29\ucd9c \uc694\uc18c\ub294 \ubb34\uc5c7\uc778\uac00?", "paragraph": "\ud0d0\uc0c9\ud558\uac70\ub098 \uac16\uace0 \ub180 \uba87 \uac00\uc9c0 \uc7a5\ub09c\uac10\ub4e4\uc774 \uc788\ub294 \ub354 \ud070 \uc950\uc7a5\uc5d0\uc11c, \uc5f4 \ub9c8\ub9ac \uac00\ub7c9\uc758 \uc950\ub4e4\uacfc \ud568\uaed8 \uc0ac\ub294 \uc950\ub294 \ub354 \ub450\uaebc\uc6b4 \ud53c\uc9c8, \ub354 \ub9ce\uc740 \uac00\uc9c0\ub3cc\uae30 \uac00\uc9c0\ub4e4, \uadf8\ub9ac\uace0 \ub9ce\uc740 \ud559\uc2b5 \uac80\uc0ac\uc5d0\uc11c \ub354 \ud5a5\uc0c1\ub41c \uc218\ud589\uc744 \ubcf4\uc778\ub2e4(Rosenzweig & Bennett 1996). \uc774\ub7ec\ud55c \uc591\uc0c1\uc740 \uc6b4\ub3d9\uc5d0 \uc758\ud55c \uacb0\uacfc\uc774\ub2e4. \uc989 \uc9d1\ub2e8 \uc950\uc7a5\uc5d0\uc11c \uc0ac\ub294 \uc950\ub4e4\uc774 \ud6e8\uc52c \ub354 \ub9ce\uc774 \ub3cc\uc544\ub2e4\ub2c8\uae30 \ub54c\ubb38\uc774\ub2e4. \ub2e4\ub978 \uc544\ubb34\ub7f0 \ud48d\uc694\ud654 \uacbd\ud5d8 \uc5c6\uc774 \ub2e8\uc21c\ud788 \ub2e4\ub78c\uc950 \uccc7\ubc14\ud034 \ub3cc\ub9ac\uae30\ub9cc\uc744 \ud574\ub3c4 \uc950\uc758 \ub1cc\uc5d0\ub294 \uc0c1\ub2f9\ud55c \uc720\uc775\uc774 \uc0dd\uae34\ub2e4(van Praag, Kempermann, & Gage 1999). \uc6b4\ub3d9\uc740 \uc2e0\uacbd\uc601\uc591\uc694\uc18c\ub4e4\uc744 \ubc29\ucd9c\uc2dc\ucf1c \ub274\ub7f0\uacfc \uc2dc\ub0c5\uc2a4\uc758 \ubc1c\ub2ec\uc744 \uc99d\uac00\uc2dc\ud0a4\ub294\ub370, \uadf8\ub7ec\ud55c \uc6b4\ub3d9\uc774 \uad73\uc774 \uc2ec\ud55c \uc815\ub3c4\uc758 \uac83\uc774\uc5b4\uc57c \ud560 \ud544\uc694\ub294 \uc5c6\ub2e4(Trejo, Carro, & Torres-Aleman 2001, van Praag, Kempermann, & Gage 2000). \uc774\ub97c \uc778\uac04\uc5d0\uac8c \uc801\uc6a9\ud558\uba74 \"\ub2f9\uc2e0\uc758 \ub1cc\ub97c \uc704\ud574\uc11c \uc6b4\ub3d9\uc774 \ud544\uc694\ud558\ub2e4\" \ub294 \ucda9\uace0\ub294 \ud2b9\ud788 \ub178\uc778\ub4e4\uc5d0\uac8c \uc911\uc694\ud558\uac8c \ub41c\ub2e4. \ud48d\uc694\ub85c\uc6b4 \ud658\uacbd\uc740 \ub9ce\uc740 \ub2e4\uc591\ud55c \uc885\ub4e4\uc5d0\uac8c\uc11c \ucd95\uc0c9\uacfc \uac00\uc9c0\ub3cc\uae30\uc758 \ubc1c\uc544\ub97c \ucd09\uc9c4\ud55c\ub2e4(Coss, Brandon, & Globus 1980). \ub9ce\uc740 \ud559\ubb38\uc801 \uad50\uc721\uc744 \ubc1b\uc740 \uc0ac\ub78c\ub4e4\uc774 \uc815\uc2dd \uad50\uc721\uc744 \ub35c \ubc1b\uc740 \uc0ac\ub78c\ub4e4\ubcf4\ub2e4 \ub354 \uae38\uace0 \ub113\uac8c \ubd84\ud3ec\ub41c \uac00\uc9c0\ub3cc\uae30\ub4e4\uc744 \uc9c0\ub2c8\ub294 \uacbd\ud5a5\uc774 \uc788\ub2e4(Jacobs, Schall, & Scheibel 1993). \uc774\uc5d0 \ub300\ud574\uc11c\ub294 \ub450 \uac00\uc9c0 \uc124\uba85\uc774 \uac00\ub2a5\ud558\ub2e4. \uccab \ubc88\uc9f8 \uac00\ub2a5\uc131\uc740 \ud559\uc2b5\uc774 \uc2e4\uc81c\ub85c \uac00\uc9c0\ub3cc\uae30\uc758 \ubd84\uc9c0\ud654\ub97c \uc99d\uac00\uc2dc\ud0ac \uc218 \uc788\ub2e4\ub294 \uc124\uba85\uc774\ub2e4. \ub2e4\ub978 \uac00\ub2a5\uc131\uc740 \ub113\uac8c \ud37c\uc9c4 \uac00\uc9c0\ub3cc\uae30\ub97c \uc774\ubbf8 \ubcf4\uc720\ud558\uace0 \uc788\ub294 \uc0ac\ub78c\ub4e4\uc774 \ud559\uad50\uc5d0\uc11c \ub354 \uc88b\uc740 \uc131\uc801\uc744 \uc62c\ub9ac\uace0, \ub530\ub77c\uc11c \ub354 \ub9ce\uc740 \uad50\uc721\uc744 \ubc1b\ub294\ub2e4\ub294 \uac83\uc774\ub2e4.", "answer": "\uc2e0\uacbd\uc601\uc591\uc694\uc18c", "sentence": "\uc6b4\ub3d9\uc740 \uc2e0\uacbd\uc601\uc591\uc694\uc18c \ub4e4\uc744 \ubc29\ucd9c\uc2dc\ucf1c \ub274\ub7f0\uacfc \uc2dc\ub0c5\uc2a4\uc758 \ubc1c\ub2ec\uc744 \uc99d\uac00\uc2dc\ud0a4\ub294\ub370, \uadf8\ub7ec\ud55c \uc6b4\ub3d9\uc774 \uad73\uc774 \uc2ec\ud55c \uc815\ub3c4\uc758 \uac83\uc774\uc5b4\uc57c \ud560 \ud544\uc694\ub294 \uc5c6\ub2e4(Trejo, Carro, & Torres-Aleman 2001, van Praag, Kempermann, & Gage 2000).", "paragraph_sentence": "\ud0d0\uc0c9\ud558\uac70\ub098 \uac16\uace0 \ub180 \uba87 \uac00\uc9c0 \uc7a5\ub09c\uac10\ub4e4\uc774 \uc788\ub294 \ub354 \ud070 \uc950\uc7a5\uc5d0\uc11c, \uc5f4 \ub9c8\ub9ac \uac00\ub7c9\uc758 \uc950\ub4e4\uacfc \ud568\uaed8 \uc0ac\ub294 \uc950\ub294 \ub354 \ub450\uaebc\uc6b4 \ud53c\uc9c8, \ub354 \ub9ce\uc740 \uac00\uc9c0\ub3cc\uae30 \uac00\uc9c0\ub4e4, \uadf8\ub9ac\uace0 \ub9ce\uc740 \ud559\uc2b5 \uac80\uc0ac\uc5d0\uc11c \ub354 \ud5a5\uc0c1\ub41c \uc218\ud589\uc744 \ubcf4\uc778\ub2e4(Rosenzweig & Bennett 1996). \uc774\ub7ec\ud55c \uc591\uc0c1\uc740 \uc6b4\ub3d9\uc5d0 \uc758\ud55c \uacb0\uacfc\uc774\ub2e4. \uc989 \uc9d1\ub2e8 \uc950\uc7a5\uc5d0\uc11c \uc0ac\ub294 \uc950\ub4e4\uc774 \ud6e8\uc52c \ub354 \ub9ce\uc774 \ub3cc\uc544\ub2e4\ub2c8\uae30 \ub54c\ubb38\uc774\ub2e4. \ub2e4\ub978 \uc544\ubb34\ub7f0 \ud48d\uc694\ud654 \uacbd\ud5d8 \uc5c6\uc774 \ub2e8\uc21c\ud788 \ub2e4\ub78c\uc950 \uccc7\ubc14\ud034 \ub3cc\ub9ac\uae30\ub9cc\uc744 \ud574\ub3c4 \uc950\uc758 \ub1cc\uc5d0\ub294 \uc0c1\ub2f9\ud55c \uc720\uc775\uc774 \uc0dd\uae34\ub2e4(van Praag, Kempermann, & Gage 1999). \uc6b4\ub3d9\uc740 \uc2e0\uacbd\uc601\uc591\uc694\uc18c \ub4e4\uc744 \ubc29\ucd9c\uc2dc\ucf1c \ub274\ub7f0\uacfc \uc2dc\ub0c5\uc2a4\uc758 \ubc1c\ub2ec\uc744 \uc99d\uac00\uc2dc\ud0a4\ub294\ub370, \uadf8\ub7ec\ud55c \uc6b4\ub3d9\uc774 \uad73\uc774 \uc2ec\ud55c \uc815\ub3c4\uc758 \uac83\uc774\uc5b4\uc57c \ud560 \ud544\uc694\ub294 \uc5c6\ub2e4(Trejo, Carro, & Torres-Aleman 2001, van Praag, Kempermann, & Gage 2000). \uc774\ub97c \uc778\uac04\uc5d0\uac8c \uc801\uc6a9\ud558\uba74 \"\ub2f9\uc2e0\uc758 \ub1cc\ub97c \uc704\ud574\uc11c \uc6b4\ub3d9\uc774 \ud544\uc694\ud558\ub2e4\" \ub294 \ucda9\uace0\ub294 \ud2b9\ud788 \ub178\uc778\ub4e4\uc5d0\uac8c \uc911\uc694\ud558\uac8c \ub41c\ub2e4. \ud48d\uc694\ub85c\uc6b4 \ud658\uacbd\uc740 \ub9ce\uc740 \ub2e4\uc591\ud55c \uc885\ub4e4\uc5d0\uac8c\uc11c \ucd95\uc0c9\uacfc \uac00\uc9c0\ub3cc\uae30\uc758 \ubc1c\uc544\ub97c \ucd09\uc9c4\ud55c\ub2e4(Coss, Brandon, & Globus 1980). \ub9ce\uc740 \ud559\ubb38\uc801 \uad50\uc721\uc744 \ubc1b\uc740 \uc0ac\ub78c\ub4e4\uc774 \uc815\uc2dd \uad50\uc721\uc744 \ub35c \ubc1b\uc740 \uc0ac\ub78c\ub4e4\ubcf4\ub2e4 \ub354 \uae38\uace0 \ub113\uac8c \ubd84\ud3ec\ub41c \uac00\uc9c0\ub3cc\uae30\ub4e4\uc744 \uc9c0\ub2c8\ub294 \uacbd\ud5a5\uc774 \uc788\ub2e4(Jacobs, Schall, & Scheibel 1993). \uc774\uc5d0 \ub300\ud574\uc11c\ub294 \ub450 \uac00\uc9c0 \uc124\uba85\uc774 \uac00\ub2a5\ud558\ub2e4. \uccab \ubc88\uc9f8 \uac00\ub2a5\uc131\uc740 \ud559\uc2b5\uc774 \uc2e4\uc81c\ub85c \uac00\uc9c0\ub3cc\uae30\uc758 \ubd84\uc9c0\ud654\ub97c \uc99d\uac00\uc2dc\ud0ac \uc218 \uc788\ub2e4\ub294 \uc124\uba85\uc774\ub2e4. \ub2e4\ub978 \uac00\ub2a5\uc131\uc740 \ub113\uac8c \ud37c\uc9c4 \uac00\uc9c0\ub3cc\uae30\ub97c \uc774\ubbf8 \ubcf4\uc720\ud558\uace0 \uc788\ub294 \uc0ac\ub78c\ub4e4\uc774 \ud559\uad50\uc5d0\uc11c \ub354 \uc88b\uc740 \uc131\uc801\uc744 \uc62c\ub9ac\uace0, \ub530\ub77c\uc11c \ub354 \ub9ce\uc740 \uad50\uc721\uc744 \ubc1b\ub294\ub2e4\ub294 \uac83\uc774\ub2e4.", "paragraph_answer": "\ud0d0\uc0c9\ud558\uac70\ub098 \uac16\uace0 \ub180 \uba87 \uac00\uc9c0 \uc7a5\ub09c\uac10\ub4e4\uc774 \uc788\ub294 \ub354 \ud070 \uc950\uc7a5\uc5d0\uc11c, \uc5f4 \ub9c8\ub9ac \uac00\ub7c9\uc758 \uc950\ub4e4\uacfc \ud568\uaed8 \uc0ac\ub294 \uc950\ub294 \ub354 \ub450\uaebc\uc6b4 \ud53c\uc9c8, \ub354 \ub9ce\uc740 \uac00\uc9c0\ub3cc\uae30 \uac00\uc9c0\ub4e4, \uadf8\ub9ac\uace0 \ub9ce\uc740 \ud559\uc2b5 \uac80\uc0ac\uc5d0\uc11c \ub354 \ud5a5\uc0c1\ub41c \uc218\ud589\uc744 \ubcf4\uc778\ub2e4(Rosenzweig & Bennett 1996). \uc774\ub7ec\ud55c \uc591\uc0c1\uc740 \uc6b4\ub3d9\uc5d0 \uc758\ud55c \uacb0\uacfc\uc774\ub2e4. \uc989 \uc9d1\ub2e8 \uc950\uc7a5\uc5d0\uc11c \uc0ac\ub294 \uc950\ub4e4\uc774 \ud6e8\uc52c \ub354 \ub9ce\uc774 \ub3cc\uc544\ub2e4\ub2c8\uae30 \ub54c\ubb38\uc774\ub2e4. \ub2e4\ub978 \uc544\ubb34\ub7f0 \ud48d\uc694\ud654 \uacbd\ud5d8 \uc5c6\uc774 \ub2e8\uc21c\ud788 \ub2e4\ub78c\uc950 \uccc7\ubc14\ud034 \ub3cc\ub9ac\uae30\ub9cc\uc744 \ud574\ub3c4 \uc950\uc758 \ub1cc\uc5d0\ub294 \uc0c1\ub2f9\ud55c \uc720\uc775\uc774 \uc0dd\uae34\ub2e4(van Praag, Kempermann, & Gage 1999). \uc6b4\ub3d9\uc740 \uc2e0\uacbd\uc601\uc591\uc694\uc18c \ub4e4\uc744 \ubc29\ucd9c\uc2dc\ucf1c \ub274\ub7f0\uacfc \uc2dc\ub0c5\uc2a4\uc758 \ubc1c\ub2ec\uc744 \uc99d\uac00\uc2dc\ud0a4\ub294\ub370, \uadf8\ub7ec\ud55c \uc6b4\ub3d9\uc774 \uad73\uc774 \uc2ec\ud55c \uc815\ub3c4\uc758 \uac83\uc774\uc5b4\uc57c \ud560 \ud544\uc694\ub294 \uc5c6\ub2e4(Trejo, Carro, & Torres-Aleman 2001, van Praag, Kempermann, & Gage 2000). \uc774\ub97c \uc778\uac04\uc5d0\uac8c \uc801\uc6a9\ud558\uba74 \"\ub2f9\uc2e0\uc758 \ub1cc\ub97c \uc704\ud574\uc11c \uc6b4\ub3d9\uc774 \ud544\uc694\ud558\ub2e4\" \ub294 \ucda9\uace0\ub294 \ud2b9\ud788 \ub178\uc778\ub4e4\uc5d0\uac8c \uc911\uc694\ud558\uac8c \ub41c\ub2e4. \ud48d\uc694\ub85c\uc6b4 \ud658\uacbd\uc740 \ub9ce\uc740 \ub2e4\uc591\ud55c \uc885\ub4e4\uc5d0\uac8c\uc11c \ucd95\uc0c9\uacfc \uac00\uc9c0\ub3cc\uae30\uc758 \ubc1c\uc544\ub97c \ucd09\uc9c4\ud55c\ub2e4(Coss, Brandon, & Globus 1980). \ub9ce\uc740 \ud559\ubb38\uc801 \uad50\uc721\uc744 \ubc1b\uc740 \uc0ac\ub78c\ub4e4\uc774 \uc815\uc2dd \uad50\uc721\uc744 \ub35c \ubc1b\uc740 \uc0ac\ub78c\ub4e4\ubcf4\ub2e4 \ub354 \uae38\uace0 \ub113\uac8c \ubd84\ud3ec\ub41c \uac00\uc9c0\ub3cc\uae30\ub4e4\uc744 \uc9c0\ub2c8\ub294 \uacbd\ud5a5\uc774 \uc788\ub2e4(Jacobs, Schall, & Scheibel 1993). \uc774\uc5d0 \ub300\ud574\uc11c\ub294 \ub450 \uac00\uc9c0 \uc124\uba85\uc774 \uac00\ub2a5\ud558\ub2e4. \uccab \ubc88\uc9f8 \uac00\ub2a5\uc131\uc740 \ud559\uc2b5\uc774 \uc2e4\uc81c\ub85c \uac00\uc9c0\ub3cc\uae30\uc758 \ubd84\uc9c0\ud654\ub97c \uc99d\uac00\uc2dc\ud0ac \uc218 \uc788\ub2e4\ub294 \uc124\uba85\uc774\ub2e4. \ub2e4\ub978 \uac00\ub2a5\uc131\uc740 \ub113\uac8c \ud37c\uc9c4 \uac00\uc9c0\ub3cc\uae30\ub97c \uc774\ubbf8 \ubcf4\uc720\ud558\uace0 \uc788\ub294 \uc0ac\ub78c\ub4e4\uc774 \ud559\uad50\uc5d0\uc11c \ub354 \uc88b\uc740 \uc131\uc801\uc744 \uc62c\ub9ac\uace0, \ub530\ub77c\uc11c \ub354 \ub9ce\uc740 \uad50\uc721\uc744 \ubc1b\ub294\ub2e4\ub294 \uac83\uc774\ub2e4.", "sentence_answer": "\uc6b4\ub3d9\uc740 \uc2e0\uacbd\uc601\uc591\uc694\uc18c \ub4e4\uc744 \ubc29\ucd9c\uc2dc\ucf1c \ub274\ub7f0\uacfc \uc2dc\ub0c5\uc2a4\uc758 \ubc1c\ub2ec\uc744 \uc99d\uac00\uc2dc\ud0a4\ub294\ub370, \uadf8\ub7ec\ud55c \uc6b4\ub3d9\uc774 \uad73\uc774 \uc2ec\ud55c \uc815\ub3c4\uc758 \uac83\uc774\uc5b4\uc57c \ud560 \ud544\uc694\ub294 \uc5c6\ub2e4(Trejo, Carro, & Torres-Aleman 2001, van Praag, Kempermann, & Gage 2000).", "paragraph_id": "6457416-2-1", "paragraph_question": "question: \uc6b4\ub3d9\uc744 \ud560 \uacbd\uc6b0 \ubc29\ucd9c\ub418\ub294 \uc694\uc18c\ub85c \ub274\ub7f0\uacfc \uc2dc\ub0c5\uc2a4\uc758 \ubc1c\ub2ec\uc744 \uc99d\uac00\uc2dc\ud0a4\ub294 \ubc29\ucd9c \uc694\uc18c\ub294 \ubb34\uc5c7\uc778\uac00?, context: \ud0d0\uc0c9\ud558\uac70\ub098 \uac16\uace0 \ub180 \uba87 \uac00\uc9c0 \uc7a5\ub09c\uac10\ub4e4\uc774 \uc788\ub294 \ub354 \ud070 \uc950\uc7a5\uc5d0\uc11c, \uc5f4 \ub9c8\ub9ac \uac00\ub7c9\uc758 \uc950\ub4e4\uacfc \ud568\uaed8 \uc0ac\ub294 \uc950\ub294 \ub354 \ub450\uaebc\uc6b4 \ud53c\uc9c8, \ub354 \ub9ce\uc740 \uac00\uc9c0\ub3cc\uae30 \uac00\uc9c0\ub4e4, \uadf8\ub9ac\uace0 \ub9ce\uc740 \ud559\uc2b5 \uac80\uc0ac\uc5d0\uc11c \ub354 \ud5a5\uc0c1\ub41c \uc218\ud589\uc744 \ubcf4\uc778\ub2e4(Rosenzweig & Bennett 1996). \uc774\ub7ec\ud55c \uc591\uc0c1\uc740 \uc6b4\ub3d9\uc5d0 \uc758\ud55c \uacb0\uacfc\uc774\ub2e4. \uc989 \uc9d1\ub2e8 \uc950\uc7a5\uc5d0\uc11c \uc0ac\ub294 \uc950\ub4e4\uc774 \ud6e8\uc52c \ub354 \ub9ce\uc774 \ub3cc\uc544\ub2e4\ub2c8\uae30 \ub54c\ubb38\uc774\ub2e4. \ub2e4\ub978 \uc544\ubb34\ub7f0 \ud48d\uc694\ud654 \uacbd\ud5d8 \uc5c6\uc774 \ub2e8\uc21c\ud788 \ub2e4\ub78c\uc950 \uccc7\ubc14\ud034 \ub3cc\ub9ac\uae30\ub9cc\uc744 \ud574\ub3c4 \uc950\uc758 \ub1cc\uc5d0\ub294 \uc0c1\ub2f9\ud55c \uc720\uc775\uc774 \uc0dd\uae34\ub2e4(van Praag, Kempermann, & Gage 1999). \uc6b4\ub3d9\uc740 \uc2e0\uacbd\uc601\uc591\uc694\uc18c\ub4e4\uc744 \ubc29\ucd9c\uc2dc\ucf1c \ub274\ub7f0\uacfc \uc2dc\ub0c5\uc2a4\uc758 \ubc1c\ub2ec\uc744 \uc99d\uac00\uc2dc\ud0a4\ub294\ub370, \uadf8\ub7ec\ud55c \uc6b4\ub3d9\uc774 \uad73\uc774 \uc2ec\ud55c \uc815\ub3c4\uc758 \uac83\uc774\uc5b4\uc57c \ud560 \ud544\uc694\ub294 \uc5c6\ub2e4(Trejo, Carro, & Torres-Aleman 2001, van Praag, Kempermann, & Gage 2000). \uc774\ub97c \uc778\uac04\uc5d0\uac8c \uc801\uc6a9\ud558\uba74 \"\ub2f9\uc2e0\uc758 \ub1cc\ub97c \uc704\ud574\uc11c \uc6b4\ub3d9\uc774 \ud544\uc694\ud558\ub2e4\" \ub294 \ucda9\uace0\ub294 \ud2b9\ud788 \ub178\uc778\ub4e4\uc5d0\uac8c \uc911\uc694\ud558\uac8c \ub41c\ub2e4. \ud48d\uc694\ub85c\uc6b4 \ud658\uacbd\uc740 \ub9ce\uc740 \ub2e4\uc591\ud55c \uc885\ub4e4\uc5d0\uac8c\uc11c \ucd95\uc0c9\uacfc \uac00\uc9c0\ub3cc\uae30\uc758 \ubc1c\uc544\ub97c \ucd09\uc9c4\ud55c\ub2e4(Coss, Brandon, & Globus 1980). \ub9ce\uc740 \ud559\ubb38\uc801 \uad50\uc721\uc744 \ubc1b\uc740 \uc0ac\ub78c\ub4e4\uc774 \uc815\uc2dd \uad50\uc721\uc744 \ub35c \ubc1b\uc740 \uc0ac\ub78c\ub4e4\ubcf4\ub2e4 \ub354 \uae38\uace0 \ub113\uac8c \ubd84\ud3ec\ub41c \uac00\uc9c0\ub3cc\uae30\ub4e4\uc744 \uc9c0\ub2c8\ub294 \uacbd\ud5a5\uc774 \uc788\ub2e4(Jacobs, Schall, & Scheibel 1993). \uc774\uc5d0 \ub300\ud574\uc11c\ub294 \ub450 \uac00\uc9c0 \uc124\uba85\uc774 \uac00\ub2a5\ud558\ub2e4. \uccab \ubc88\uc9f8 \uac00\ub2a5\uc131\uc740 \ud559\uc2b5\uc774 \uc2e4\uc81c\ub85c \uac00\uc9c0\ub3cc\uae30\uc758 \ubd84\uc9c0\ud654\ub97c \uc99d\uac00\uc2dc\ud0ac \uc218 \uc788\ub2e4\ub294 \uc124\uba85\uc774\ub2e4. \ub2e4\ub978 \uac00\ub2a5\uc131\uc740 \ub113\uac8c \ud37c\uc9c4 \uac00\uc9c0\ub3cc\uae30\ub97c \uc774\ubbf8 \ubcf4\uc720\ud558\uace0 \uc788\ub294 \uc0ac\ub78c\ub4e4\uc774 \ud559\uad50\uc5d0\uc11c \ub354 \uc88b\uc740 \uc131\uc801\uc744 \uc62c\ub9ac\uace0, \ub530\ub77c\uc11c \ub354 \ub9ce\uc740 \uad50\uc721\uc744 \ubc1b\ub294\ub2e4\ub294 \uac83\uc774\ub2e4."} {"question": "2010\ub144 \uc140\ud0c0 \ube44\uace0\uc5d0 \uc0c8\ub86d\uac8c \ubd80\uc784\ud55c \uac10\ub3c5\uc740?", "paragraph": "\uc140\ud0c0\ub294 \ucc9c\ubb38\ud559\uc801\uc778 \ubd80\ucc44\uc5d0 \ub530\ub978 \uc120\uc218 \uc601\uc785\uc758 \uc2e4\ud328, \uc8fc\ucd95\uc120\uc218\ub4e4\uc758 \ub178\uc1e0\ud654\ub85c \uc778\ud574 2006/07 \uc2dc\uc98c \ubd88\uc6b4\uc758 \uac15\ub4f1\uc744 \ub9de\uc774\ud588\ub2e4. \uc774\ud6c4 5\uc2dc\uc98c\uc744 \uc138\uad70\ub2e4 \ub9ac\uac00\uc5d0\uc11c \ubcf4\ub0b8 \uc140\ud0c0 \ube44\uace0\ub294 2010\ub144 \ub77c\ud30c\uc5d8 \ubca0\ub2c8\ud14c\uc2a4 \uc544\ub798\uc5d0\uc11c \ub9ac\ubc84\ud480 \uc218\uc11d \ucf54\uce58 \uc5ed\ud560\ub3c4 \uc218\ud589\ud588\ub358 \ud30c\ucf54 \uc5d0\ub808\ub77c \uac10\ub3c5 \ubd80\uc784 \ud6c4 \ucc28\uadfc\ucc28\uadfc \uc804\ub825\uc744 \ub2e4\uc838\ub098\uac14\uace0, \uc720\uc2a4\ud300\uc5d0 \uc0c1\ub2f9\ud788 \ub9ce\uc740 \uc2e0\uacbd\uc744 \uc3df\uc558\ub2e4. \uc2b9\uaca9 \uc5ed\uc2dc \uc9c0\ub09c 5\ub144\ub3d9\uc548\uc758 \ub178\ub825\uc774 \uacb0\uc2e4\uc744 \ub9fa\uc5c8\ub2e4\uace0 \ud560 \uc218\uc788\ub2e4. 1\uad70\uc758 80%\uac00 \uc140\ud0c0\uc758 \uc5f0\uace0\uc9c0 \uc9c0\ubc29\uc778 \uac08\ub9ac\uc2dc\uc544 \ucd9c\uc2e0\uc774\uc600\uace0 \ud604\uc7ac \ubca0\uc2a4\ud2b8 11\uc758 \uc808\ubc18\uc774 \uc720\uc2a4\ud300 \ucd9c\uc2e0\uc774\uac70\ub098 \uac08\ub9ac\uc2dc\uc544 \uc9c0\ubc29 \ucd9c\uc2e0\uc774\ub2e4. \uc140\ud0c0\uc758 \uc720\uc2a4\ud798\uc740 \uc11c\uc11c\ud788 \ube5b\uc744 \ubc1c\ud558\uace0 \uc788\ub294\ub370, 2011/12 \uc2dc\uc98c \ub77c\uc694\uc5d0\uc11c \uc88b\uc740 \ud65c\uc57d\uc744 \ubcf4\uc774\uba70 12/13\uc2dc\uc98c\uc5d0 \uc2a4\uc644\uc9c0 \uc2dc\ud2f0\ub85c \uc774\uc801\ud55c \ubbf8\ucd94 \uc5ed\uc2dc \uc140\ud0c0\uc5d0\uc11c \ud504\ub85c \uc0dd\ud65c\uc744 \uc2dc\uc791\ud588\ub2e4.", "answer": "\ud30c\ucf54 \uc5d0\ub808\ub77c", "sentence": "\uc774\ud6c4 5\uc2dc\uc98c\uc744 \uc138\uad70\ub2e4 \ub9ac\uac00\uc5d0\uc11c \ubcf4\ub0b8 \uc140\ud0c0 \ube44\uace0\ub294 2010\ub144 \ub77c\ud30c\uc5d8 \ubca0\ub2c8\ud14c\uc2a4 \uc544\ub798\uc5d0\uc11c \ub9ac\ubc84\ud480 \uc218\uc11d \ucf54\uce58 \uc5ed\ud560\ub3c4 \uc218\ud589\ud588\ub358 \ud30c\ucf54 \uc5d0\ub808\ub77c \uac10\ub3c5 \ubd80\uc784 \ud6c4 \ucc28\uadfc\ucc28\uadfc \uc804\ub825\uc744 \ub2e4\uc838\ub098\uac14\uace0, \uc720\uc2a4\ud300\uc5d0 \uc0c1\ub2f9\ud788 \ub9ce\uc740 \uc2e0\uacbd\uc744 \uc3df\uc558\ub2e4.", "paragraph_sentence": "\uc140\ud0c0\ub294 \ucc9c\ubb38\ud559\uc801\uc778 \ubd80\ucc44\uc5d0 \ub530\ub978 \uc120\uc218 \uc601\uc785\uc758 \uc2e4\ud328, \uc8fc\ucd95\uc120\uc218\ub4e4\uc758 \ub178\uc1e0\ud654\ub85c \uc778\ud574 2006/07 \uc2dc\uc98c \ubd88\uc6b4\uc758 \uac15\ub4f1\uc744 \ub9de\uc774\ud588\ub2e4. \uc774\ud6c4 5\uc2dc\uc98c\uc744 \uc138\uad70\ub2e4 \ub9ac\uac00\uc5d0\uc11c \ubcf4\ub0b8 \uc140\ud0c0 \ube44\uace0\ub294 2010\ub144 \ub77c\ud30c\uc5d8 \ubca0\ub2c8\ud14c\uc2a4 \uc544\ub798\uc5d0\uc11c \ub9ac\ubc84\ud480 \uc218\uc11d \ucf54\uce58 \uc5ed\ud560\ub3c4 \uc218\ud589\ud588\ub358 \ud30c\ucf54 \uc5d0\ub808\ub77c \uac10\ub3c5 \ubd80\uc784 \ud6c4 \ucc28\uadfc\ucc28\uadfc \uc804\ub825\uc744 \ub2e4\uc838\ub098\uac14\uace0, \uc720\uc2a4\ud300\uc5d0 \uc0c1\ub2f9\ud788 \ub9ce\uc740 \uc2e0\uacbd\uc744 \uc3df\uc558\ub2e4. \uc2b9\uaca9 \uc5ed\uc2dc \uc9c0\ub09c 5\ub144\ub3d9\uc548\uc758 \ub178\ub825\uc774 \uacb0\uc2e4\uc744 \ub9fa\uc5c8\ub2e4\uace0 \ud560 \uc218\uc788\ub2e4. 1\uad70\uc758 80%\uac00 \uc140\ud0c0\uc758 \uc5f0\uace0\uc9c0 \uc9c0\ubc29\uc778 \uac08\ub9ac\uc2dc\uc544 \ucd9c\uc2e0\uc774\uc600\uace0 \ud604\uc7ac \ubca0\uc2a4\ud2b8 11\uc758 \uc808\ubc18\uc774 \uc720\uc2a4\ud300 \ucd9c\uc2e0\uc774\uac70\ub098 \uac08\ub9ac\uc2dc\uc544 \uc9c0\ubc29 \ucd9c\uc2e0\uc774\ub2e4. \uc140\ud0c0\uc758 \uc720\uc2a4\ud798\uc740 \uc11c\uc11c\ud788 \ube5b\uc744 \ubc1c\ud558\uace0 \uc788\ub294\ub370, 2011/12 \uc2dc\uc98c \ub77c\uc694\uc5d0\uc11c \uc88b\uc740 \ud65c\uc57d\uc744 \ubcf4\uc774\uba70 12/13\uc2dc\uc98c\uc5d0 \uc2a4\uc644\uc9c0 \uc2dc\ud2f0\ub85c \uc774\uc801\ud55c \ubbf8\ucd94 \uc5ed\uc2dc \uc140\ud0c0\uc5d0\uc11c \ud504\ub85c \uc0dd\ud65c\uc744 \uc2dc\uc791\ud588\ub2e4.", "paragraph_answer": "\uc140\ud0c0\ub294 \ucc9c\ubb38\ud559\uc801\uc778 \ubd80\ucc44\uc5d0 \ub530\ub978 \uc120\uc218 \uc601\uc785\uc758 \uc2e4\ud328, \uc8fc\ucd95\uc120\uc218\ub4e4\uc758 \ub178\uc1e0\ud654\ub85c \uc778\ud574 2006/07 \uc2dc\uc98c \ubd88\uc6b4\uc758 \uac15\ub4f1\uc744 \ub9de\uc774\ud588\ub2e4. \uc774\ud6c4 5\uc2dc\uc98c\uc744 \uc138\uad70\ub2e4 \ub9ac\uac00\uc5d0\uc11c \ubcf4\ub0b8 \uc140\ud0c0 \ube44\uace0\ub294 2010\ub144 \ub77c\ud30c\uc5d8 \ubca0\ub2c8\ud14c\uc2a4 \uc544\ub798\uc5d0\uc11c \ub9ac\ubc84\ud480 \uc218\uc11d \ucf54\uce58 \uc5ed\ud560\ub3c4 \uc218\ud589\ud588\ub358 \ud30c\ucf54 \uc5d0\ub808\ub77c \uac10\ub3c5 \ubd80\uc784 \ud6c4 \ucc28\uadfc\ucc28\uadfc \uc804\ub825\uc744 \ub2e4\uc838\ub098\uac14\uace0, \uc720\uc2a4\ud300\uc5d0 \uc0c1\ub2f9\ud788 \ub9ce\uc740 \uc2e0\uacbd\uc744 \uc3df\uc558\ub2e4. \uc2b9\uaca9 \uc5ed\uc2dc \uc9c0\ub09c 5\ub144\ub3d9\uc548\uc758 \ub178\ub825\uc774 \uacb0\uc2e4\uc744 \ub9fa\uc5c8\ub2e4\uace0 \ud560 \uc218\uc788\ub2e4. 1\uad70\uc758 80%\uac00 \uc140\ud0c0\uc758 \uc5f0\uace0\uc9c0 \uc9c0\ubc29\uc778 \uac08\ub9ac\uc2dc\uc544 \ucd9c\uc2e0\uc774\uc600\uace0 \ud604\uc7ac \ubca0\uc2a4\ud2b8 11\uc758 \uc808\ubc18\uc774 \uc720\uc2a4\ud300 \ucd9c\uc2e0\uc774\uac70\ub098 \uac08\ub9ac\uc2dc\uc544 \uc9c0\ubc29 \ucd9c\uc2e0\uc774\ub2e4. \uc140\ud0c0\uc758 \uc720\uc2a4\ud798\uc740 \uc11c\uc11c\ud788 \ube5b\uc744 \ubc1c\ud558\uace0 \uc788\ub294\ub370, 2011/12 \uc2dc\uc98c \ub77c\uc694\uc5d0\uc11c \uc88b\uc740 \ud65c\uc57d\uc744 \ubcf4\uc774\uba70 12/13\uc2dc\uc98c\uc5d0 \uc2a4\uc644\uc9c0 \uc2dc\ud2f0\ub85c \uc774\uc801\ud55c \ubbf8\ucd94 \uc5ed\uc2dc \uc140\ud0c0\uc5d0\uc11c \ud504\ub85c \uc0dd\ud65c\uc744 \uc2dc\uc791\ud588\ub2e4.", "sentence_answer": "\uc774\ud6c4 5\uc2dc\uc98c\uc744 \uc138\uad70\ub2e4 \ub9ac\uac00\uc5d0\uc11c \ubcf4\ub0b8 \uc140\ud0c0 \ube44\uace0\ub294 2010\ub144 \ub77c\ud30c\uc5d8 \ubca0\ub2c8\ud14c\uc2a4 \uc544\ub798\uc5d0\uc11c \ub9ac\ubc84\ud480 \uc218\uc11d \ucf54\uce58 \uc5ed\ud560\ub3c4 \uc218\ud589\ud588\ub358 \ud30c\ucf54 \uc5d0\ub808\ub77c \uac10\ub3c5 \ubd80\uc784 \ud6c4 \ucc28\uadfc\ucc28\uadfc \uc804\ub825\uc744 \ub2e4\uc838\ub098\uac14\uace0, \uc720\uc2a4\ud300\uc5d0 \uc0c1\ub2f9\ud788 \ub9ce\uc740 \uc2e0\uacbd\uc744 \uc3df\uc558\ub2e4.", "paragraph_id": "6518223-6-0", "paragraph_question": "question: 2010\ub144 \uc140\ud0c0 \ube44\uace0\uc5d0 \uc0c8\ub86d\uac8c \ubd80\uc784\ud55c \uac10\ub3c5\uc740?, context: \uc140\ud0c0\ub294 \ucc9c\ubb38\ud559\uc801\uc778 \ubd80\ucc44\uc5d0 \ub530\ub978 \uc120\uc218 \uc601\uc785\uc758 \uc2e4\ud328, \uc8fc\ucd95\uc120\uc218\ub4e4\uc758 \ub178\uc1e0\ud654\ub85c \uc778\ud574 2006/07 \uc2dc\uc98c \ubd88\uc6b4\uc758 \uac15\ub4f1\uc744 \ub9de\uc774\ud588\ub2e4. \uc774\ud6c4 5\uc2dc\uc98c\uc744 \uc138\uad70\ub2e4 \ub9ac\uac00\uc5d0\uc11c \ubcf4\ub0b8 \uc140\ud0c0 \ube44\uace0\ub294 2010\ub144 \ub77c\ud30c\uc5d8 \ubca0\ub2c8\ud14c\uc2a4 \uc544\ub798\uc5d0\uc11c \ub9ac\ubc84\ud480 \uc218\uc11d \ucf54\uce58 \uc5ed\ud560\ub3c4 \uc218\ud589\ud588\ub358 \ud30c\ucf54 \uc5d0\ub808\ub77c \uac10\ub3c5 \ubd80\uc784 \ud6c4 \ucc28\uadfc\ucc28\uadfc \uc804\ub825\uc744 \ub2e4\uc838\ub098\uac14\uace0, \uc720\uc2a4\ud300\uc5d0 \uc0c1\ub2f9\ud788 \ub9ce\uc740 \uc2e0\uacbd\uc744 \uc3df\uc558\ub2e4. \uc2b9\uaca9 \uc5ed\uc2dc \uc9c0\ub09c 5\ub144\ub3d9\uc548\uc758 \ub178\ub825\uc774 \uacb0\uc2e4\uc744 \ub9fa\uc5c8\ub2e4\uace0 \ud560 \uc218\uc788\ub2e4. 1\uad70\uc758 80%\uac00 \uc140\ud0c0\uc758 \uc5f0\uace0\uc9c0 \uc9c0\ubc29\uc778 \uac08\ub9ac\uc2dc\uc544 \ucd9c\uc2e0\uc774\uc600\uace0 \ud604\uc7ac \ubca0\uc2a4\ud2b8 11\uc758 \uc808\ubc18\uc774 \uc720\uc2a4\ud300 \ucd9c\uc2e0\uc774\uac70\ub098 \uac08\ub9ac\uc2dc\uc544 \uc9c0\ubc29 \ucd9c\uc2e0\uc774\ub2e4. \uc140\ud0c0\uc758 \uc720\uc2a4\ud798\uc740 \uc11c\uc11c\ud788 \ube5b\uc744 \ubc1c\ud558\uace0 \uc788\ub294\ub370, 2011/12 \uc2dc\uc98c \ub77c\uc694\uc5d0\uc11c \uc88b\uc740 \ud65c\uc57d\uc744 \ubcf4\uc774\uba70 12/13\uc2dc\uc98c\uc5d0 \uc2a4\uc644\uc9c0 \uc2dc\ud2f0\ub85c \uc774\uc801\ud55c \ubbf8\ucd94 \uc5ed\uc2dc \uc140\ud0c0\uc5d0\uc11c \ud504\ub85c \uc0dd\ud65c\uc744 \uc2dc\uc791\ud588\ub2e4."} {"question": "1990\ub144 \ud45c\ucc3d\uc6d0\uc774 \uc9c0\uc5ed\uae30\ub3d9\ub300 \uc18c\ub300\uc7a5\uc744 \ub9e1\uc558\ub358 \uc9c0\uc5ed\uc740?", "paragraph": "1989\ub144 \uacbd\ucc30\ub300\ud559\uc744 \uc878\uc5c5\ud558\uace0 \uacbd\ucc30\uad00\uc5d0 \uc784\uc6a9\ub418\uc5b4 \uc81c\uc8fc\ub3c4 \uc911\ubb38\uc9c0\uc5ed \uc804\uacbd\ub300 \uc18c\ub300\uc7a5, 1990\ub144 \ud654\uc131\uad70 \uc9c0\uc5ed\uae30\ub3d9\ub300 \uc18c\ub300\uc7a5 \ub4f1\uc744 \uac70\uccd0 1991\ub144 \ubd80\ucc9c\uacbd\ucc30\uc11c \ud615\uc0ac\uacfc \ud615\uc0ac, 1992\ub144 \uacbd\uae30\ub3c4 \uc9c0\ubc29\uacbd\ucc30\uccad \ubcf4\uc548\uacfc \uc678\uc0ac\uacc4 \ud615\uc0ac, 1993\ub144 \ubcf4\uc548\uacfc \uc678\uc0ac\uacc4 \uc8fc\uc784\uc774 \ub418\uc5c8\ub2e4. 1993\ub144 5\uc6d4 \uad6d\ube44\uc7a5\ud559\uc0dd\uc73c\ub85c \uc601\uad6d\uc73c\ub85c \uc720\ud559\uc744 \uac14\uace0 \uc5d1\uc11c\ud130 \ub300\ud559\uad50\uc5d0\uc11c \uacbd\ucc30\ud559\uc744 \uc218\ud559\ud558\uc600\ub2e4. 1997\ub144 12\uc6d4 \uc601\uad6d\uc5d0\uc11c \uadc0\uad6d\ud558\uc600\uace0, 1998\ub144 \uacbd\ucc30\uccad \uc81c\ub3c4\uac1c\uc120\uae30\ud68d\ub2e8 \uc5f0\uad6c\ub2f4\ub2f9\uad00, \uacbd\ucc30\ub300\ud559 \uad50\uad00 \ub4f1\uc744 \uc9c0\ub0b4\uace0 1999\ub144 \ud1f4\uc9c1\ud558\uc600\ub2e4. 1998\ub144\ubd80\ud130\ub294 \uad11\uc6b4\ub300\ud559\uad50, \ud55c\uad6d\uc678\uad6d\uc5b4\ub300\ud559\uad50 \ubc94\uc8c4\uc2ec\ub9ac\ud559 \uad50\uc218\ub85c \ucd9c\uac15\ud558\uc600\uace0, 1999\ub144 \uacbd\ucc30\uad00\uc5d0\uc11c \uc0ac\ud1f4\ud55c \ub4a4\ub85c\ub294 \uacbd\ucc30\ub300\ud559 \uc804\uc784\uac15\uc0ac\uac00 \ub418\uc5c8\uc73c\uba70, \uc5f0\uc138\ub300\ud559\uad50, \uc544\uc8fc\ub300\ud559\uad50, \uacbd\uae30\ub300\ud559\uad50 \ub4f1\uc5d0\ub3c4 \ucd9c\uac15\ud558\uc600\ub2e4. 2001\ub144\ubd80\ud130 \uacbd\ucc30\ub300\ud559 \uc870\uad50\uc218\uac00 \ub418\uace0 2012\ub144 \uacbd\ucc30\ub300\ud559 \uc815\uad50\uc218\uac00 \ub418\uc5c8\ub2e4.", "answer": "\ud654\uc131\uad70", "sentence": "1989\ub144 \uacbd\ucc30\ub300\ud559\uc744 \uc878\uc5c5\ud558\uace0 \uacbd\ucc30\uad00\uc5d0 \uc784\uc6a9\ub418\uc5b4 \uc81c\uc8fc\ub3c4 \uc911\ubb38\uc9c0\uc5ed \uc804\uacbd\ub300 \uc18c\ub300\uc7a5, 1990\ub144 \ud654\uc131\uad70 \uc9c0\uc5ed\uae30\ub3d9\ub300 \uc18c\ub300\uc7a5 \ub4f1\uc744 \uac70\uccd0 1991\ub144 \ubd80\ucc9c\uacbd\ucc30\uc11c \ud615\uc0ac\uacfc \ud615\uc0ac, 1992\ub144 \uacbd\uae30\ub3c4 \uc9c0\ubc29\uacbd\ucc30\uccad \ubcf4\uc548\uacfc \uc678\uc0ac\uacc4 \ud615\uc0ac, 1993\ub144 \ubcf4\uc548\uacfc \uc678\uc0ac\uacc4 \uc8fc\uc784\uc774 \ub418\uc5c8\ub2e4.", "paragraph_sentence": " 1989\ub144 \uacbd\ucc30\ub300\ud559\uc744 \uc878\uc5c5\ud558\uace0 \uacbd\ucc30\uad00\uc5d0 \uc784\uc6a9\ub418\uc5b4 \uc81c\uc8fc\ub3c4 \uc911\ubb38\uc9c0\uc5ed \uc804\uacbd\ub300 \uc18c\ub300\uc7a5, 1990\ub144 \ud654\uc131\uad70 \uc9c0\uc5ed\uae30\ub3d9\ub300 \uc18c\ub300\uc7a5 \ub4f1\uc744 \uac70\uccd0 1991\ub144 \ubd80\ucc9c\uacbd\ucc30\uc11c \ud615\uc0ac\uacfc \ud615\uc0ac, 1992\ub144 \uacbd\uae30\ub3c4 \uc9c0\ubc29\uacbd\ucc30\uccad \ubcf4\uc548\uacfc \uc678\uc0ac\uacc4 \ud615\uc0ac, 1993\ub144 \ubcf4\uc548\uacfc \uc678\uc0ac\uacc4 \uc8fc\uc784\uc774 \ub418\uc5c8\ub2e4. 1993\ub144 5\uc6d4 \uad6d\ube44\uc7a5\ud559\uc0dd\uc73c\ub85c \uc601\uad6d\uc73c\ub85c \uc720\ud559\uc744 \uac14\uace0 \uc5d1\uc11c\ud130 \ub300\ud559\uad50\uc5d0\uc11c \uacbd\ucc30\ud559\uc744 \uc218\ud559\ud558\uc600\ub2e4. 1997\ub144 12\uc6d4 \uc601\uad6d\uc5d0\uc11c \uadc0\uad6d\ud558\uc600\uace0, 1998\ub144 \uacbd\ucc30\uccad \uc81c\ub3c4\uac1c\uc120\uae30\ud68d\ub2e8 \uc5f0\uad6c\ub2f4\ub2f9\uad00, \uacbd\ucc30\ub300\ud559 \uad50\uad00 \ub4f1\uc744 \uc9c0\ub0b4\uace0 1999\ub144 \ud1f4\uc9c1\ud558\uc600\ub2e4. 1998\ub144\ubd80\ud130\ub294 \uad11\uc6b4\ub300\ud559\uad50, \ud55c\uad6d\uc678\uad6d\uc5b4\ub300\ud559\uad50 \ubc94\uc8c4\uc2ec\ub9ac\ud559 \uad50\uc218\ub85c \ucd9c\uac15\ud558\uc600\uace0, 1999\ub144 \uacbd\ucc30\uad00\uc5d0\uc11c \uc0ac\ud1f4\ud55c \ub4a4\ub85c\ub294 \uacbd\ucc30\ub300\ud559 \uc804\uc784\uac15\uc0ac\uac00 \ub418\uc5c8\uc73c\uba70, \uc5f0\uc138\ub300\ud559\uad50, \uc544\uc8fc\ub300\ud559\uad50, \uacbd\uae30\ub300\ud559\uad50 \ub4f1\uc5d0\ub3c4 \ucd9c\uac15\ud558\uc600\ub2e4. 2001\ub144\ubd80\ud130 \uacbd\ucc30\ub300\ud559 \uc870\uad50\uc218\uac00 \ub418\uace0 2012\ub144 \uacbd\ucc30\ub300\ud559 \uc815\uad50\uc218\uac00 \ub418\uc5c8\ub2e4.", "paragraph_answer": "1989\ub144 \uacbd\ucc30\ub300\ud559\uc744 \uc878\uc5c5\ud558\uace0 \uacbd\ucc30\uad00\uc5d0 \uc784\uc6a9\ub418\uc5b4 \uc81c\uc8fc\ub3c4 \uc911\ubb38\uc9c0\uc5ed \uc804\uacbd\ub300 \uc18c\ub300\uc7a5, 1990\ub144 \ud654\uc131\uad70 \uc9c0\uc5ed\uae30\ub3d9\ub300 \uc18c\ub300\uc7a5 \ub4f1\uc744 \uac70\uccd0 1991\ub144 \ubd80\ucc9c\uacbd\ucc30\uc11c \ud615\uc0ac\uacfc \ud615\uc0ac, 1992\ub144 \uacbd\uae30\ub3c4 \uc9c0\ubc29\uacbd\ucc30\uccad \ubcf4\uc548\uacfc \uc678\uc0ac\uacc4 \ud615\uc0ac, 1993\ub144 \ubcf4\uc548\uacfc \uc678\uc0ac\uacc4 \uc8fc\uc784\uc774 \ub418\uc5c8\ub2e4. 1993\ub144 5\uc6d4 \uad6d\ube44\uc7a5\ud559\uc0dd\uc73c\ub85c \uc601\uad6d\uc73c\ub85c \uc720\ud559\uc744 \uac14\uace0 \uc5d1\uc11c\ud130 \ub300\ud559\uad50\uc5d0\uc11c \uacbd\ucc30\ud559\uc744 \uc218\ud559\ud558\uc600\ub2e4. 1997\ub144 12\uc6d4 \uc601\uad6d\uc5d0\uc11c \uadc0\uad6d\ud558\uc600\uace0, 1998\ub144 \uacbd\ucc30\uccad \uc81c\ub3c4\uac1c\uc120\uae30\ud68d\ub2e8 \uc5f0\uad6c\ub2f4\ub2f9\uad00, \uacbd\ucc30\ub300\ud559 \uad50\uad00 \ub4f1\uc744 \uc9c0\ub0b4\uace0 1999\ub144 \ud1f4\uc9c1\ud558\uc600\ub2e4. 1998\ub144\ubd80\ud130\ub294 \uad11\uc6b4\ub300\ud559\uad50, \ud55c\uad6d\uc678\uad6d\uc5b4\ub300\ud559\uad50 \ubc94\uc8c4\uc2ec\ub9ac\ud559 \uad50\uc218\ub85c \ucd9c\uac15\ud558\uc600\uace0, 1999\ub144 \uacbd\ucc30\uad00\uc5d0\uc11c \uc0ac\ud1f4\ud55c \ub4a4\ub85c\ub294 \uacbd\ucc30\ub300\ud559 \uc804\uc784\uac15\uc0ac\uac00 \ub418\uc5c8\uc73c\uba70, \uc5f0\uc138\ub300\ud559\uad50, \uc544\uc8fc\ub300\ud559\uad50, \uacbd\uae30\ub300\ud559\uad50 \ub4f1\uc5d0\ub3c4 \ucd9c\uac15\ud558\uc600\ub2e4. 2001\ub144\ubd80\ud130 \uacbd\ucc30\ub300\ud559 \uc870\uad50\uc218\uac00 \ub418\uace0 2012\ub144 \uacbd\ucc30\ub300\ud559 \uc815\uad50\uc218\uac00 \ub418\uc5c8\ub2e4.", "sentence_answer": "1989\ub144 \uacbd\ucc30\ub300\ud559\uc744 \uc878\uc5c5\ud558\uace0 \uacbd\ucc30\uad00\uc5d0 \uc784\uc6a9\ub418\uc5b4 \uc81c\uc8fc\ub3c4 \uc911\ubb38\uc9c0\uc5ed \uc804\uacbd\ub300 \uc18c\ub300\uc7a5, 1990\ub144 \ud654\uc131\uad70 \uc9c0\uc5ed\uae30\ub3d9\ub300 \uc18c\ub300\uc7a5 \ub4f1\uc744 \uac70\uccd0 1991\ub144 \ubd80\ucc9c\uacbd\ucc30\uc11c \ud615\uc0ac\uacfc \ud615\uc0ac, 1992\ub144 \uacbd\uae30\ub3c4 \uc9c0\ubc29\uacbd\ucc30\uccad \ubcf4\uc548\uacfc \uc678\uc0ac\uacc4 \ud615\uc0ac, 1993\ub144 \ubcf4\uc548\uacfc \uc678\uc0ac\uacc4 \uc8fc\uc784\uc774 \ub418\uc5c8\ub2e4.", "paragraph_id": "6568380-1-0", "paragraph_question": "question: 1990\ub144 \ud45c\ucc3d\uc6d0\uc774 \uc9c0\uc5ed\uae30\ub3d9\ub300 \uc18c\ub300\uc7a5\uc744 \ub9e1\uc558\ub358 \uc9c0\uc5ed\uc740?, context: 1989\ub144 \uacbd\ucc30\ub300\ud559\uc744 \uc878\uc5c5\ud558\uace0 \uacbd\ucc30\uad00\uc5d0 \uc784\uc6a9\ub418\uc5b4 \uc81c\uc8fc\ub3c4 \uc911\ubb38\uc9c0\uc5ed \uc804\uacbd\ub300 \uc18c\ub300\uc7a5, 1990\ub144 \ud654\uc131\uad70 \uc9c0\uc5ed\uae30\ub3d9\ub300 \uc18c\ub300\uc7a5 \ub4f1\uc744 \uac70\uccd0 1991\ub144 \ubd80\ucc9c\uacbd\ucc30\uc11c \ud615\uc0ac\uacfc \ud615\uc0ac, 1992\ub144 \uacbd\uae30\ub3c4 \uc9c0\ubc29\uacbd\ucc30\uccad \ubcf4\uc548\uacfc \uc678\uc0ac\uacc4 \ud615\uc0ac, 1993\ub144 \ubcf4\uc548\uacfc \uc678\uc0ac\uacc4 \uc8fc\uc784\uc774 \ub418\uc5c8\ub2e4. 1993\ub144 5\uc6d4 \uad6d\ube44\uc7a5\ud559\uc0dd\uc73c\ub85c \uc601\uad6d\uc73c\ub85c \uc720\ud559\uc744 \uac14\uace0 \uc5d1\uc11c\ud130 \ub300\ud559\uad50\uc5d0\uc11c \uacbd\ucc30\ud559\uc744 \uc218\ud559\ud558\uc600\ub2e4. 1997\ub144 12\uc6d4 \uc601\uad6d\uc5d0\uc11c \uadc0\uad6d\ud558\uc600\uace0, 1998\ub144 \uacbd\ucc30\uccad \uc81c\ub3c4\uac1c\uc120\uae30\ud68d\ub2e8 \uc5f0\uad6c\ub2f4\ub2f9\uad00, \uacbd\ucc30\ub300\ud559 \uad50\uad00 \ub4f1\uc744 \uc9c0\ub0b4\uace0 1999\ub144 \ud1f4\uc9c1\ud558\uc600\ub2e4. 1998\ub144\ubd80\ud130\ub294 \uad11\uc6b4\ub300\ud559\uad50, \ud55c\uad6d\uc678\uad6d\uc5b4\ub300\ud559\uad50 \ubc94\uc8c4\uc2ec\ub9ac\ud559 \uad50\uc218\ub85c \ucd9c\uac15\ud558\uc600\uace0, 1999\ub144 \uacbd\ucc30\uad00\uc5d0\uc11c \uc0ac\ud1f4\ud55c \ub4a4\ub85c\ub294 \uacbd\ucc30\ub300\ud559 \uc804\uc784\uac15\uc0ac\uac00 \ub418\uc5c8\uc73c\uba70, \uc5f0\uc138\ub300\ud559\uad50, \uc544\uc8fc\ub300\ud559\uad50, \uacbd\uae30\ub300\ud559\uad50 \ub4f1\uc5d0\ub3c4 \ucd9c\uac15\ud558\uc600\ub2e4. 2001\ub144\ubd80\ud130 \uacbd\ucc30\ub300\ud559 \uc870\uad50\uc218\uac00 \ub418\uace0 2012\ub144 \uacbd\ucc30\ub300\ud559 \uc815\uad50\uc218\uac00 \ub418\uc5c8\ub2e4."} {"question": "\uae40\uaddc\uc2dd\uc740 \ub204\uad6c\uc5d0\uac8c \uc0ac\ud1f4\ud558\ub77c\uace0 \uc555\ub825\uc744 \ud589\uc0ac\ud588\ub098?", "paragraph": "\uad70\uc815\uc758 \uc88c\uc6b0 \ud569\uc791 \uc815\ucc45\uc5d0 \ub530\ub77c \ub450\ub4dc\ub7ec\uc9c0\uac8c \ub41c \uc0ac\ub78c\uc774 \uae40\uaddc\uc2dd\uc774\uc5c8\ub2e4. \uadf8\ub7ec\ub098 \uae40\uaddc\uc2dd\uc740 \ub2f9\uc2dc \uce58\uc548\uc758 \ucc45\uc784\uc744 \uc9c0\uace0 \uc788\ub358 \uc870\ubcd1\uc625\uacfc \uc7a5\ud0dd\uc0c1\uc774 \uc5ec\uc804\ud788 \uc774\uc2b9\ub9cc\uc744 \uc9c0\uc9c0\ud558\ub294 \ub370 \ub300\ud558\uc5ec \ubab9\uc2dc \ubabb\ub9c8\ub545\ud558\uac8c \uc0dd\uac01\ud558\uace0 \uc788\uc5c8\ub2e4. \ud558\ub8e8\ub294 \uc7a5\ud0dd\uc0c1\uc774 \uae40\uaddc\uc2dd\uc744 \ub9cc\ub0ac\ub2e4. \uae40\uaddc\uc2dd\uc740 \uc7a5\ud0dd\uc0c1\uc5d0\uac8c \"\uc7a5 \ucd1d\uac10\uc740 \uc774\uc81c \uc0ac\ud45c\ub97c \ub0b4\ub294 \uac83\uc774 \uc5b4\ub5bb\uaca0\uc18c? \uadf8\ub807\uc9c0 \uc54a\uc544\ub3c4 \uace7 \uac08\ub9ac\uac8c \ub420 \ud130\uc778\ub370?\" \ub77c\uace0 \ub9d0\ud558\uc600\ub2e4. \uc774 \ub9d0\uc744 \ub4e3\uace0 \uc7a5\ud0dd\uc0c1\uc740 \ud558\uc9c0 \uc911\uc7a5\uc774 \uae40\uaddc\uc2dd\uc5d0\uac8c \ubb34\uc2a8 \ub9d0\uc744 \ud558\uc600\uc744 \uac83\uc774\ub77c\uace0 \uc0dd\uac01\ud558\uc5ec \ud558\uc9c0 \uc911\uc7a5\uc744 \ub9cc\ub0ac\ub2e4. \uadf8\ub9ac\uace0 \ud558\uc9c0 \uc911\uc7a5\uc5d0\uac8c \uc774\ub7ec\ud55c \uc774\uc57c\uae30\ub97c \ud558\uc600\ub354\ub2c8 \ud558\uc9c0\ub294 \"\ub2f9\uc2e0\uacfc \uc870 \ubd80\uc7a5\uc740 \uc6b0\ub9ac \uad6d\ubb34\uc131\uc5d0\uc11c \uc808\ub300 \uc2e0\uc784\ud558\uace0 \uc788\ub294 \uc0ac\ub78c\uc778\ub370 \uc5b4\ucc0c \uc77c\uac1c \uc0ac\ub839\uad00\uc778 \ub0b4\uac00 \ub9c8\uc74c\ub300\ub85c \ud560 \uc218 \uc788\uaca0\uc18c?\"\ub77c\uace0 \ub300\ub2f5\ud558\ub294 \uac83\uc774\uc5c8\ub2e4. \uace0 \ud558\uc600\ub2e4. \uc7a5\ud0dd\uc0c1\uc740 '\uc6b0\ub9ac\uac00 \uae40 \ubc15\uc0ac\ub97c \uc9c0\uc9c0\ud558\uc9c0 \uc54a\ub294 \ub370\ub294 \uc801\uc774 \ubd88\ub9cc\uc744 \uac00\uc9c0\uace0 \uc788\ub358 \ud558\uc9c0 \uc911\uc7a5\uc774\uc5c8\ub2e4. \uadf8\ub7ec\ub098 \uc6b0\ub9ac\ub97c \ub2e4\ub978 \uc778\ubb3c\ub85c \uac08\uc544\uce58\uc6b4\ub2e4\uba74 \uce58\uc548\uc744 \uc720\uc9c0\ud558\uae30\uac00 \uc5b4\ub824\uc6b0\ub9ac\ub77c\ub294 \uc0ac\uc2e4\uc744 \uad6c\ub294 \uc798 \uc54c\uace0 \uc788\uc5c8\ub2e4.'\uace0 \ud588\ub2e4. \ud55c\ud3b8 \uae40\uaddc\uc2dd\uc740 \uc7a5\ud0dd\uc0c1\ub354\ub7ec \uc0ac\ud1f4\ud558\ub77c\ub294 \uc555\ub825\uc744 \uac00\ud588\uc9c0\ub9cc \uc7a5\ud0dd\uc0c1\uc740 \ubb35\uc0b4\ud574\ubc84\ub838\ub2e4.", "answer": "\uc7a5\ud0dd\uc0c1", "sentence": "\uadf8\ub7ec\ub098 \uae40\uaddc\uc2dd\uc740 \ub2f9\uc2dc \uce58\uc548\uc758 \ucc45\uc784\uc744 \uc9c0\uace0 \uc788\ub358 \uc870\ubcd1\uc625\uacfc \uc7a5\ud0dd\uc0c1 \uc774 \uc5ec\uc804\ud788 \uc774\uc2b9\ub9cc\uc744 \uc9c0\uc9c0\ud558\ub294 \ub370 \ub300\ud558\uc5ec \ubab9\uc2dc \ubabb\ub9c8\ub545\ud558\uac8c \uc0dd\uac01\ud558\uace0 \uc788\uc5c8\ub2e4.", "paragraph_sentence": "\uad70\uc815\uc758 \uc88c\uc6b0 \ud569\uc791 \uc815\ucc45\uc5d0 \ub530\ub77c \ub450\ub4dc\ub7ec\uc9c0\uac8c \ub41c \uc0ac\ub78c\uc774 \uae40\uaddc\uc2dd\uc774\uc5c8\ub2e4. \uadf8\ub7ec\ub098 \uae40\uaddc\uc2dd\uc740 \ub2f9\uc2dc \uce58\uc548\uc758 \ucc45\uc784\uc744 \uc9c0\uace0 \uc788\ub358 \uc870\ubcd1\uc625\uacfc \uc7a5\ud0dd\uc0c1 \uc774 \uc5ec\uc804\ud788 \uc774\uc2b9\ub9cc\uc744 \uc9c0\uc9c0\ud558\ub294 \ub370 \ub300\ud558\uc5ec \ubab9\uc2dc \ubabb\ub9c8\ub545\ud558\uac8c \uc0dd\uac01\ud558\uace0 \uc788\uc5c8\ub2e4. \ud558\ub8e8\ub294 \uc7a5\ud0dd\uc0c1\uc774 \uae40\uaddc\uc2dd\uc744 \ub9cc\ub0ac\ub2e4. \uae40\uaddc\uc2dd\uc740 \uc7a5\ud0dd\uc0c1\uc5d0\uac8c \"\uc7a5 \ucd1d\uac10\uc740 \uc774\uc81c \uc0ac\ud45c\ub97c \ub0b4\ub294 \uac83\uc774 \uc5b4\ub5bb\uaca0\uc18c? \uadf8\ub807\uc9c0 \uc54a\uc544\ub3c4 \uace7 \uac08\ub9ac\uac8c \ub420 \ud130\uc778\ub370?\" \ub77c\uace0 \ub9d0\ud558\uc600\ub2e4. \uc774 \ub9d0\uc744 \ub4e3\uace0 \uc7a5\ud0dd\uc0c1\uc740 \ud558\uc9c0 \uc911\uc7a5\uc774 \uae40\uaddc\uc2dd\uc5d0\uac8c \ubb34\uc2a8 \ub9d0\uc744 \ud558\uc600\uc744 \uac83\uc774\ub77c\uace0 \uc0dd\uac01\ud558\uc5ec \ud558\uc9c0 \uc911\uc7a5\uc744 \ub9cc\ub0ac\ub2e4. \uadf8\ub9ac\uace0 \ud558\uc9c0 \uc911\uc7a5\uc5d0\uac8c \uc774\ub7ec\ud55c \uc774\uc57c\uae30\ub97c \ud558\uc600\ub354\ub2c8 \ud558\uc9c0\ub294 \"\ub2f9\uc2e0\uacfc \uc870 \ubd80\uc7a5\uc740 \uc6b0\ub9ac \uad6d\ubb34\uc131\uc5d0\uc11c \uc808\ub300 \uc2e0\uc784\ud558\uace0 \uc788\ub294 \uc0ac\ub78c\uc778\ub370 \uc5b4\ucc0c \uc77c\uac1c \uc0ac\ub839\uad00\uc778 \ub0b4\uac00 \ub9c8\uc74c\ub300\ub85c \ud560 \uc218 \uc788\uaca0\uc18c?\"\ub77c\uace0 \ub300\ub2f5\ud558\ub294 \uac83\uc774\uc5c8\ub2e4. \uace0 \ud558\uc600\ub2e4. \uc7a5\ud0dd\uc0c1\uc740 '\uc6b0\ub9ac\uac00 \uae40 \ubc15\uc0ac\ub97c \uc9c0\uc9c0\ud558\uc9c0 \uc54a\ub294 \ub370\ub294 \uc801\uc774 \ubd88\ub9cc\uc744 \uac00\uc9c0\uace0 \uc788\ub358 \ud558\uc9c0 \uc911\uc7a5\uc774\uc5c8\ub2e4. \uadf8\ub7ec\ub098 \uc6b0\ub9ac\ub97c \ub2e4\ub978 \uc778\ubb3c\ub85c \uac08\uc544\uce58\uc6b4\ub2e4\uba74 \uce58\uc548\uc744 \uc720\uc9c0\ud558\uae30\uac00 \uc5b4\ub824\uc6b0\ub9ac\ub77c\ub294 \uc0ac\uc2e4\uc744 \uad6c\ub294 \uc798 \uc54c\uace0 \uc788\uc5c8\ub2e4. '\uace0 \ud588\ub2e4. \ud55c\ud3b8 \uae40\uaddc\uc2dd\uc740 \uc7a5\ud0dd\uc0c1\ub354\ub7ec \uc0ac\ud1f4\ud558\ub77c\ub294 \uc555\ub825\uc744 \uac00\ud588\uc9c0\ub9cc \uc7a5\ud0dd\uc0c1\uc740 \ubb35\uc0b4\ud574\ubc84\ub838\ub2e4.", "paragraph_answer": "\uad70\uc815\uc758 \uc88c\uc6b0 \ud569\uc791 \uc815\ucc45\uc5d0 \ub530\ub77c \ub450\ub4dc\ub7ec\uc9c0\uac8c \ub41c \uc0ac\ub78c\uc774 \uae40\uaddc\uc2dd\uc774\uc5c8\ub2e4. \uadf8\ub7ec\ub098 \uae40\uaddc\uc2dd\uc740 \ub2f9\uc2dc \uce58\uc548\uc758 \ucc45\uc784\uc744 \uc9c0\uace0 \uc788\ub358 \uc870\ubcd1\uc625\uacfc \uc7a5\ud0dd\uc0c1 \uc774 \uc5ec\uc804\ud788 \uc774\uc2b9\ub9cc\uc744 \uc9c0\uc9c0\ud558\ub294 \ub370 \ub300\ud558\uc5ec \ubab9\uc2dc \ubabb\ub9c8\ub545\ud558\uac8c \uc0dd\uac01\ud558\uace0 \uc788\uc5c8\ub2e4. \ud558\ub8e8\ub294 \uc7a5\ud0dd\uc0c1\uc774 \uae40\uaddc\uc2dd\uc744 \ub9cc\ub0ac\ub2e4. \uae40\uaddc\uc2dd\uc740 \uc7a5\ud0dd\uc0c1\uc5d0\uac8c \"\uc7a5 \ucd1d\uac10\uc740 \uc774\uc81c \uc0ac\ud45c\ub97c \ub0b4\ub294 \uac83\uc774 \uc5b4\ub5bb\uaca0\uc18c? \uadf8\ub807\uc9c0 \uc54a\uc544\ub3c4 \uace7 \uac08\ub9ac\uac8c \ub420 \ud130\uc778\ub370?\" \ub77c\uace0 \ub9d0\ud558\uc600\ub2e4. \uc774 \ub9d0\uc744 \ub4e3\uace0 \uc7a5\ud0dd\uc0c1\uc740 \ud558\uc9c0 \uc911\uc7a5\uc774 \uae40\uaddc\uc2dd\uc5d0\uac8c \ubb34\uc2a8 \ub9d0\uc744 \ud558\uc600\uc744 \uac83\uc774\ub77c\uace0 \uc0dd\uac01\ud558\uc5ec \ud558\uc9c0 \uc911\uc7a5\uc744 \ub9cc\ub0ac\ub2e4. \uadf8\ub9ac\uace0 \ud558\uc9c0 \uc911\uc7a5\uc5d0\uac8c \uc774\ub7ec\ud55c \uc774\uc57c\uae30\ub97c \ud558\uc600\ub354\ub2c8 \ud558\uc9c0\ub294 \"\ub2f9\uc2e0\uacfc \uc870 \ubd80\uc7a5\uc740 \uc6b0\ub9ac \uad6d\ubb34\uc131\uc5d0\uc11c \uc808\ub300 \uc2e0\uc784\ud558\uace0 \uc788\ub294 \uc0ac\ub78c\uc778\ub370 \uc5b4\ucc0c \uc77c\uac1c \uc0ac\ub839\uad00\uc778 \ub0b4\uac00 \ub9c8\uc74c\ub300\ub85c \ud560 \uc218 \uc788\uaca0\uc18c?\"\ub77c\uace0 \ub300\ub2f5\ud558\ub294 \uac83\uc774\uc5c8\ub2e4. \uace0 \ud558\uc600\ub2e4. \uc7a5\ud0dd\uc0c1\uc740 '\uc6b0\ub9ac\uac00 \uae40 \ubc15\uc0ac\ub97c \uc9c0\uc9c0\ud558\uc9c0 \uc54a\ub294 \ub370\ub294 \uc801\uc774 \ubd88\ub9cc\uc744 \uac00\uc9c0\uace0 \uc788\ub358 \ud558\uc9c0 \uc911\uc7a5\uc774\uc5c8\ub2e4. \uadf8\ub7ec\ub098 \uc6b0\ub9ac\ub97c \ub2e4\ub978 \uc778\ubb3c\ub85c \uac08\uc544\uce58\uc6b4\ub2e4\uba74 \uce58\uc548\uc744 \uc720\uc9c0\ud558\uae30\uac00 \uc5b4\ub824\uc6b0\ub9ac\ub77c\ub294 \uc0ac\uc2e4\uc744 \uad6c\ub294 \uc798 \uc54c\uace0 \uc788\uc5c8\ub2e4.'\uace0 \ud588\ub2e4. \ud55c\ud3b8 \uae40\uaddc\uc2dd\uc740 \uc7a5\ud0dd\uc0c1\ub354\ub7ec \uc0ac\ud1f4\ud558\ub77c\ub294 \uc555\ub825\uc744 \uac00\ud588\uc9c0\ub9cc \uc7a5\ud0dd\uc0c1\uc740 \ubb35\uc0b4\ud574\ubc84\ub838\ub2e4.", "sentence_answer": "\uadf8\ub7ec\ub098 \uae40\uaddc\uc2dd\uc740 \ub2f9\uc2dc \uce58\uc548\uc758 \ucc45\uc784\uc744 \uc9c0\uace0 \uc788\ub358 \uc870\ubcd1\uc625\uacfc \uc7a5\ud0dd\uc0c1 \uc774 \uc5ec\uc804\ud788 \uc774\uc2b9\ub9cc\uc744 \uc9c0\uc9c0\ud558\ub294 \ub370 \ub300\ud558\uc5ec \ubab9\uc2dc \ubabb\ub9c8\ub545\ud558\uac8c \uc0dd\uac01\ud558\uace0 \uc788\uc5c8\ub2e4.", "paragraph_id": "5991262-34-0", "paragraph_question": "question: \uae40\uaddc\uc2dd\uc740 \ub204\uad6c\uc5d0\uac8c \uc0ac\ud1f4\ud558\ub77c\uace0 \uc555\ub825\uc744 \ud589\uc0ac\ud588\ub098?, context: \uad70\uc815\uc758 \uc88c\uc6b0 \ud569\uc791 \uc815\ucc45\uc5d0 \ub530\ub77c \ub450\ub4dc\ub7ec\uc9c0\uac8c \ub41c \uc0ac\ub78c\uc774 \uae40\uaddc\uc2dd\uc774\uc5c8\ub2e4. \uadf8\ub7ec\ub098 \uae40\uaddc\uc2dd\uc740 \ub2f9\uc2dc \uce58\uc548\uc758 \ucc45\uc784\uc744 \uc9c0\uace0 \uc788\ub358 \uc870\ubcd1\uc625\uacfc \uc7a5\ud0dd\uc0c1\uc774 \uc5ec\uc804\ud788 \uc774\uc2b9\ub9cc\uc744 \uc9c0\uc9c0\ud558\ub294 \ub370 \ub300\ud558\uc5ec \ubab9\uc2dc \ubabb\ub9c8\ub545\ud558\uac8c \uc0dd\uac01\ud558\uace0 \uc788\uc5c8\ub2e4. \ud558\ub8e8\ub294 \uc7a5\ud0dd\uc0c1\uc774 \uae40\uaddc\uc2dd\uc744 \ub9cc\ub0ac\ub2e4. \uae40\uaddc\uc2dd\uc740 \uc7a5\ud0dd\uc0c1\uc5d0\uac8c \"\uc7a5 \ucd1d\uac10\uc740 \uc774\uc81c \uc0ac\ud45c\ub97c \ub0b4\ub294 \uac83\uc774 \uc5b4\ub5bb\uaca0\uc18c? \uadf8\ub807\uc9c0 \uc54a\uc544\ub3c4 \uace7 \uac08\ub9ac\uac8c \ub420 \ud130\uc778\ub370?\" \ub77c\uace0 \ub9d0\ud558\uc600\ub2e4. \uc774 \ub9d0\uc744 \ub4e3\uace0 \uc7a5\ud0dd\uc0c1\uc740 \ud558\uc9c0 \uc911\uc7a5\uc774 \uae40\uaddc\uc2dd\uc5d0\uac8c \ubb34\uc2a8 \ub9d0\uc744 \ud558\uc600\uc744 \uac83\uc774\ub77c\uace0 \uc0dd\uac01\ud558\uc5ec \ud558\uc9c0 \uc911\uc7a5\uc744 \ub9cc\ub0ac\ub2e4. \uadf8\ub9ac\uace0 \ud558\uc9c0 \uc911\uc7a5\uc5d0\uac8c \uc774\ub7ec\ud55c \uc774\uc57c\uae30\ub97c \ud558\uc600\ub354\ub2c8 \ud558\uc9c0\ub294 \"\ub2f9\uc2e0\uacfc \uc870 \ubd80\uc7a5\uc740 \uc6b0\ub9ac \uad6d\ubb34\uc131\uc5d0\uc11c \uc808\ub300 \uc2e0\uc784\ud558\uace0 \uc788\ub294 \uc0ac\ub78c\uc778\ub370 \uc5b4\ucc0c \uc77c\uac1c \uc0ac\ub839\uad00\uc778 \ub0b4\uac00 \ub9c8\uc74c\ub300\ub85c \ud560 \uc218 \uc788\uaca0\uc18c?\"\ub77c\uace0 \ub300\ub2f5\ud558\ub294 \uac83\uc774\uc5c8\ub2e4. \uace0 \ud558\uc600\ub2e4. \uc7a5\ud0dd\uc0c1\uc740 '\uc6b0\ub9ac\uac00 \uae40 \ubc15\uc0ac\ub97c \uc9c0\uc9c0\ud558\uc9c0 \uc54a\ub294 \ub370\ub294 \uc801\uc774 \ubd88\ub9cc\uc744 \uac00\uc9c0\uace0 \uc788\ub358 \ud558\uc9c0 \uc911\uc7a5\uc774\uc5c8\ub2e4. \uadf8\ub7ec\ub098 \uc6b0\ub9ac\ub97c \ub2e4\ub978 \uc778\ubb3c\ub85c \uac08\uc544\uce58\uc6b4\ub2e4\uba74 \uce58\uc548\uc744 \uc720\uc9c0\ud558\uae30\uac00 \uc5b4\ub824\uc6b0\ub9ac\ub77c\ub294 \uc0ac\uc2e4\uc744 \uad6c\ub294 \uc798 \uc54c\uace0 \uc788\uc5c8\ub2e4.'\uace0 \ud588\ub2e4. \ud55c\ud3b8 \uae40\uaddc\uc2dd\uc740 \uc7a5\ud0dd\uc0c1\ub354\ub7ec \uc0ac\ud1f4\ud558\ub77c\ub294 \uc555\ub825\uc744 \uac00\ud588\uc9c0\ub9cc \uc7a5\ud0dd\uc0c1\uc740 \ubb35\uc0b4\ud574\ubc84\ub838\ub2e4."} {"question": "'\ud638\uc8fc\uad6d\uc790'\ub77c\ub294 \ub2c9\ub124\uc784\uc744 \uc0ac\uc6a9\ud558\ub358 \uc0ac\ub78c\uc774 \uac80\uac70\ub41c \uc77c\uc790\ub294?", "paragraph": "2017\ub144 11\uc6d4 19\uc77c, \uc624\uc2a4\ud2b8\ub808\uc77c\ub9ac\uc544 \ub2e4\uc708\uc5d0 \uc0b4\uba74\uc11c '\ud638\uc8fc\uad6d\uc790'\ub77c\ub294 \ub2c9\ub124\uc784\uc73c\ub85c \uc6cc\ub9c8\ub4dc\uc5d0\uc11c \ud65c\ub3d9\ud558\ub358 \ub300\ud55c\ubbfc\uad6d \uad6d\uc801 \uc5ec\uc131 \uc720\ud29c\ubc84\uac00 \uc624\uc2a4\ud2b8\ub808\uc77c\ub9ac\uc544(\ud638\uc8fc) \ub0a8\uc790 \uc5b4\ub9b0\uc774\uc5d0\uac8c \uc218\uba74\uc81c\ub97c \uba39\uc778 \ub4a4 \uc131\ud3ed\ud589\uc744 \ud588\ub2e4\ub294 \uae00\uacfc \ub3d9\uc601\uc0c1\uc744 \uc6cc\ub9c8\ub4dc\uc5d0 \uac8c\uc7ac\ud558\uc600\ub2e4. \uc774\uc5d0 \uc2ec\uac01\uc131\uc744 \ub290\ub080 \uc0d8 \ud574\ubc0d\ud134\uacfc \ub300\ud55c\ubbfc\uad6d \ub124\ud2f0\uc98c\ub4e4\uc774 \uc624\uc2a4\ud2b8\ub808\uc77c\ub9ac\uc544(\ud638\uc8fc) \uc5f0\ubc29 \uacbd\ucc30\uc5d0 \uc2e0\uace0\ud558\uc5ec, \uc6a9\uc758\uc790\ub294 \uc624\uc2a4\ud2b8\ub808\uc77c\ub9ac\uc544(\ud638\uc8fc) \ud604\uc9c0 \uc2dc\uac04\uc73c\ub85c 11\uc6d4 20\uc77c \uc624\uc804\uc5d0 \uc720\ud29c\ube0c \ubc29\uc1a1 \ub3c4\uc911\uc5d0 \uac80\uac70\ub418\uc5c8\ub2e4. \uc624\uc2a4\ud2b8\ub808\uc77c\ub9ac\uc544(\ud638\uc8fc) \uc5f0\ubc29 \uacbd\ucc30\uc5d0 \ub530\ub974\uba74, \uacbd\ucc30\uccad \uc18c\uc18d \uc544\ub3d9 \ucc29\ucde8 \uc608\ubc29\ud300\uc774 \ud574\ub2f9 \uc6a9\uc758\uc790\ub97c \uac80\uac70\ud558\uc600\uc73c\uba70, \ubd81\ubd80 \uc9c0\ubc29 \ud615\ubc95 \uc81c125\uc870\uc5d0 \ubc18\ud558\ub294 \uc544\ub3d9 \ucc29\ucde8\ubb3c\uc744 \uc81c\uc791\ud55c \ud610\uc758\ub85c \uae30\uc18c\ub420 \uc608\uc815\uc774\ub77c\uace0 \ubc1d\ud614\ub2e4.", "answer": "11\uc6d4 20\uc77c", "sentence": "\uc774\uc5d0 \uc2ec\uac01\uc131\uc744 \ub290\ub080 \uc0d8 \ud574\ubc0d\ud134\uacfc \ub300\ud55c\ubbfc\uad6d \ub124\ud2f0\uc98c\ub4e4\uc774 \uc624\uc2a4\ud2b8\ub808\uc77c\ub9ac\uc544(\ud638\uc8fc) \uc5f0\ubc29 \uacbd\ucc30\uc5d0 \uc2e0\uace0\ud558\uc5ec, \uc6a9\uc758\uc790\ub294 \uc624\uc2a4\ud2b8\ub808\uc77c\ub9ac\uc544(\ud638\uc8fc) \ud604\uc9c0 \uc2dc\uac04\uc73c\ub85c 11\uc6d4 20\uc77c \uc624\uc804\uc5d0 \uc720\ud29c\ube0c \ubc29\uc1a1 \ub3c4\uc911\uc5d0 \uac80\uac70\ub418\uc5c8\ub2e4.", "paragraph_sentence": "2017\ub144 11\uc6d4 19\uc77c, \uc624\uc2a4\ud2b8\ub808\uc77c\ub9ac\uc544 \ub2e4\uc708\uc5d0 \uc0b4\uba74\uc11c '\ud638\uc8fc\uad6d\uc790'\ub77c\ub294 \ub2c9\ub124\uc784\uc73c\ub85c \uc6cc\ub9c8\ub4dc\uc5d0\uc11c \ud65c\ub3d9\ud558\ub358 \ub300\ud55c\ubbfc\uad6d \uad6d\uc801 \uc5ec\uc131 \uc720\ud29c\ubc84\uac00 \uc624\uc2a4\ud2b8\ub808\uc77c\ub9ac\uc544(\ud638\uc8fc) \ub0a8\uc790 \uc5b4\ub9b0\uc774\uc5d0\uac8c \uc218\uba74\uc81c\ub97c \uba39\uc778 \ub4a4 \uc131\ud3ed\ud589\uc744 \ud588\ub2e4\ub294 \uae00\uacfc \ub3d9\uc601\uc0c1\uc744 \uc6cc\ub9c8\ub4dc\uc5d0 \uac8c\uc7ac\ud558\uc600\ub2e4. \uc774\uc5d0 \uc2ec\uac01\uc131\uc744 \ub290\ub080 \uc0d8 \ud574\ubc0d\ud134\uacfc \ub300\ud55c\ubbfc\uad6d \ub124\ud2f0\uc98c\ub4e4\uc774 \uc624\uc2a4\ud2b8\ub808\uc77c\ub9ac\uc544(\ud638\uc8fc) \uc5f0\ubc29 \uacbd\ucc30\uc5d0 \uc2e0\uace0\ud558\uc5ec, \uc6a9\uc758\uc790\ub294 \uc624\uc2a4\ud2b8\ub808\uc77c\ub9ac\uc544(\ud638\uc8fc) \ud604\uc9c0 \uc2dc\uac04\uc73c\ub85c 11\uc6d4 20\uc77c \uc624\uc804\uc5d0 \uc720\ud29c\ube0c \ubc29\uc1a1 \ub3c4\uc911\uc5d0 \uac80\uac70\ub418\uc5c8\ub2e4. \uc624\uc2a4\ud2b8\ub808\uc77c\ub9ac\uc544(\ud638\uc8fc) \uc5f0\ubc29 \uacbd\ucc30\uc5d0 \ub530\ub974\uba74, \uacbd\ucc30\uccad \uc18c\uc18d \uc544\ub3d9 \ucc29\ucde8 \uc608\ubc29\ud300\uc774 \ud574\ub2f9 \uc6a9\uc758\uc790\ub97c \uac80\uac70\ud558\uc600\uc73c\uba70, \ubd81\ubd80 \uc9c0\ubc29 \ud615\ubc95 \uc81c125\uc870\uc5d0 \ubc18\ud558\ub294 \uc544\ub3d9 \ucc29\ucde8\ubb3c\uc744 \uc81c\uc791\ud55c \ud610\uc758\ub85c \uae30\uc18c\ub420 \uc608\uc815\uc774\ub77c\uace0 \ubc1d\ud614\ub2e4.", "paragraph_answer": "2017\ub144 11\uc6d4 19\uc77c, \uc624\uc2a4\ud2b8\ub808\uc77c\ub9ac\uc544 \ub2e4\uc708\uc5d0 \uc0b4\uba74\uc11c '\ud638\uc8fc\uad6d\uc790'\ub77c\ub294 \ub2c9\ub124\uc784\uc73c\ub85c \uc6cc\ub9c8\ub4dc\uc5d0\uc11c \ud65c\ub3d9\ud558\ub358 \ub300\ud55c\ubbfc\uad6d \uad6d\uc801 \uc5ec\uc131 \uc720\ud29c\ubc84\uac00 \uc624\uc2a4\ud2b8\ub808\uc77c\ub9ac\uc544(\ud638\uc8fc) \ub0a8\uc790 \uc5b4\ub9b0\uc774\uc5d0\uac8c \uc218\uba74\uc81c\ub97c \uba39\uc778 \ub4a4 \uc131\ud3ed\ud589\uc744 \ud588\ub2e4\ub294 \uae00\uacfc \ub3d9\uc601\uc0c1\uc744 \uc6cc\ub9c8\ub4dc\uc5d0 \uac8c\uc7ac\ud558\uc600\ub2e4. \uc774\uc5d0 \uc2ec\uac01\uc131\uc744 \ub290\ub080 \uc0d8 \ud574\ubc0d\ud134\uacfc \ub300\ud55c\ubbfc\uad6d \ub124\ud2f0\uc98c\ub4e4\uc774 \uc624\uc2a4\ud2b8\ub808\uc77c\ub9ac\uc544(\ud638\uc8fc) \uc5f0\ubc29 \uacbd\ucc30\uc5d0 \uc2e0\uace0\ud558\uc5ec, \uc6a9\uc758\uc790\ub294 \uc624\uc2a4\ud2b8\ub808\uc77c\ub9ac\uc544(\ud638\uc8fc) \ud604\uc9c0 \uc2dc\uac04\uc73c\ub85c 11\uc6d4 20\uc77c \uc624\uc804\uc5d0 \uc720\ud29c\ube0c \ubc29\uc1a1 \ub3c4\uc911\uc5d0 \uac80\uac70\ub418\uc5c8\ub2e4. \uc624\uc2a4\ud2b8\ub808\uc77c\ub9ac\uc544(\ud638\uc8fc) \uc5f0\ubc29 \uacbd\ucc30\uc5d0 \ub530\ub974\uba74, \uacbd\ucc30\uccad \uc18c\uc18d \uc544\ub3d9 \ucc29\ucde8 \uc608\ubc29\ud300\uc774 \ud574\ub2f9 \uc6a9\uc758\uc790\ub97c \uac80\uac70\ud558\uc600\uc73c\uba70, \ubd81\ubd80 \uc9c0\ubc29 \ud615\ubc95 \uc81c125\uc870\uc5d0 \ubc18\ud558\ub294 \uc544\ub3d9 \ucc29\ucde8\ubb3c\uc744 \uc81c\uc791\ud55c \ud610\uc758\ub85c \uae30\uc18c\ub420 \uc608\uc815\uc774\ub77c\uace0 \ubc1d\ud614\ub2e4.", "sentence_answer": "\uc774\uc5d0 \uc2ec\uac01\uc131\uc744 \ub290\ub080 \uc0d8 \ud574\ubc0d\ud134\uacfc \ub300\ud55c\ubbfc\uad6d \ub124\ud2f0\uc98c\ub4e4\uc774 \uc624\uc2a4\ud2b8\ub808\uc77c\ub9ac\uc544(\ud638\uc8fc) \uc5f0\ubc29 \uacbd\ucc30\uc5d0 \uc2e0\uace0\ud558\uc5ec, \uc6a9\uc758\uc790\ub294 \uc624\uc2a4\ud2b8\ub808\uc77c\ub9ac\uc544(\ud638\uc8fc) \ud604\uc9c0 \uc2dc\uac04\uc73c\ub85c 11\uc6d4 20\uc77c \uc624\uc804\uc5d0 \uc720\ud29c\ube0c \ubc29\uc1a1 \ub3c4\uc911\uc5d0 \uac80\uac70\ub418\uc5c8\ub2e4.", "paragraph_id": "6605492-3-1", "paragraph_question": "question: '\ud638\uc8fc\uad6d\uc790'\ub77c\ub294 \ub2c9\ub124\uc784\uc744 \uc0ac\uc6a9\ud558\ub358 \uc0ac\ub78c\uc774 \uac80\uac70\ub41c \uc77c\uc790\ub294?, context: 2017\ub144 11\uc6d4 19\uc77c, \uc624\uc2a4\ud2b8\ub808\uc77c\ub9ac\uc544 \ub2e4\uc708\uc5d0 \uc0b4\uba74\uc11c '\ud638\uc8fc\uad6d\uc790'\ub77c\ub294 \ub2c9\ub124\uc784\uc73c\ub85c \uc6cc\ub9c8\ub4dc\uc5d0\uc11c \ud65c\ub3d9\ud558\ub358 \ub300\ud55c\ubbfc\uad6d \uad6d\uc801 \uc5ec\uc131 \uc720\ud29c\ubc84\uac00 \uc624\uc2a4\ud2b8\ub808\uc77c\ub9ac\uc544(\ud638\uc8fc) \ub0a8\uc790 \uc5b4\ub9b0\uc774\uc5d0\uac8c \uc218\uba74\uc81c\ub97c \uba39\uc778 \ub4a4 \uc131\ud3ed\ud589\uc744 \ud588\ub2e4\ub294 \uae00\uacfc \ub3d9\uc601\uc0c1\uc744 \uc6cc\ub9c8\ub4dc\uc5d0 \uac8c\uc7ac\ud558\uc600\ub2e4. \uc774\uc5d0 \uc2ec\uac01\uc131\uc744 \ub290\ub080 \uc0d8 \ud574\ubc0d\ud134\uacfc \ub300\ud55c\ubbfc\uad6d \ub124\ud2f0\uc98c\ub4e4\uc774 \uc624\uc2a4\ud2b8\ub808\uc77c\ub9ac\uc544(\ud638\uc8fc) \uc5f0\ubc29 \uacbd\ucc30\uc5d0 \uc2e0\uace0\ud558\uc5ec, \uc6a9\uc758\uc790\ub294 \uc624\uc2a4\ud2b8\ub808\uc77c\ub9ac\uc544(\ud638\uc8fc) \ud604\uc9c0 \uc2dc\uac04\uc73c\ub85c 11\uc6d4 20\uc77c \uc624\uc804\uc5d0 \uc720\ud29c\ube0c \ubc29\uc1a1 \ub3c4\uc911\uc5d0 \uac80\uac70\ub418\uc5c8\ub2e4. \uc624\uc2a4\ud2b8\ub808\uc77c\ub9ac\uc544(\ud638\uc8fc) \uc5f0\ubc29 \uacbd\ucc30\uc5d0 \ub530\ub974\uba74, \uacbd\ucc30\uccad \uc18c\uc18d \uc544\ub3d9 \ucc29\ucde8 \uc608\ubc29\ud300\uc774 \ud574\ub2f9 \uc6a9\uc758\uc790\ub97c \uac80\uac70\ud558\uc600\uc73c\uba70, \ubd81\ubd80 \uc9c0\ubc29 \ud615\ubc95 \uc81c125\uc870\uc5d0 \ubc18\ud558\ub294 \uc544\ub3d9 \ucc29\ucde8\ubb3c\uc744 \uc81c\uc791\ud55c \ud610\uc758\ub85c \uae30\uc18c\ub420 \uc608\uc815\uc774\ub77c\uace0 \ubc1d\ud614\ub2e4."} {"question": "\ub8e8 \uac8c\ub9ad \ub300\uc2e0 \uc568\uc2a4\uc6cc\uc2a4 \ubca0\uc774\ube0c \ub2ec\uadf8\ub80c\uc744 1\ub8e8\uc218\ub85c \uae30\uc6a9\ud55c \uac10\ub3c5\uc740?", "paragraph": "\uacbd\uae30 \ud734\uc2dd\uc77c\uc774\uc5c8\ub358 5\uc6d4 1\uc77c\uc774 \uc9c0\ub098 \ub2e4\uc74c\ub0a0\uc778 5\uc6d4 2\uc77c, \ud0c0\uc774\uac70\uc2a4\uc640\uc758 \ub514\ud2b8\ub85c\uc774\ud2b8 \uc6d0\uc815 \uacbd\uae30\uac00 \uc2dc\uc791\ud558\uae30 \uc804 \ub8e8 \uac8c\ub9ad\uc740 \ub9e4\uce74\uc2dc \uac10\ub3c5\uc5d0\uac8c \ub2e4\uac00\uac00\uc11c \u201c\uac10\ub3c5\ub2d8, \uc624\ub298\uc740 \ubca4\uce58\uc5d0 \uc788\uc744\uac8c\uc694.\u201d\ub77c\uace0 \ub9d0\ud588\uace0, \uac10\ub3c5\uc740 \u201c\uadf8\uac8c \ud300\uc744 \uc704\ud55c \uae38\uc774\ub77c\uace0 \uc0dd\uac01\ud55c\ub2e4\uba74\u201d\uc774\ub77c\uace0 \ub9d0\ud588\ub2e4. \ub9e4\uce74\uc2dc\ub294 \ub9c8\uc9c0\ubabb\ud574 \ud5c8\ub77d\uc744 \ud588\uace0 \uc568\uc2a4\uc6cc\uc2a4 \"\ubca0\uc774\ube0c\" \ub2ec\uadf8\ub80c\uc744 1\ub8e8\uc218\ub85c \uae30\uc6a9\ud588\ub2e4. \uadf8\ub7ec\uba74\uc11c \ub9d0\ud558\uae38 \u201c\uc5b8\uc81c\ub77c\ub3c4 \ub2e4\uc2dc \uacbd\uae30\uc5d0 \ucd9c\uc7a5\ud558\uace0 \uc2f6\uc73c\uba74 \ub9d0\ud574\ub77c, 1\ub8e8\ub294 \ud56d\uc0c1 \ub108\uc758 \uac83\uc774\ub2e4.\u201d \uacbd\uae30 \uc2dc\uc791 \uc804, \ud300\uc758 \uc8fc\uc7a5\uc774 \uc120\ubc1c \uc120\uc218 \uba85\ub2e8\uc744 \uc8fc\uc2ec\uc5d0\uac8c \uac00\uc838\ub2e4 \uc8fc\ub294 \uad00\ub840\ub300\ub85c \uac8c\ub9ad\uc740 \uc591\ud0a4\uc2a4 \ud300\uc758 \uc120\ubc1c \uc120\uc218 \uba85\ub2e8\uc744 \uc8fc\uc2ec\uc5d0\uac8c \uac74\ub124\uc8fc\uc5c8\ub2e4. \ud558\uc9c0\ub9cc \uadf8\ub0a0 \ucd9c\uc7a5 \uba85\ub2e8\uc5d0\uc11c \uc815\uc791 \uac8c\ub9ad\uc758 \uc774\ub984\uc740 \ucc3e\uc544\ubcfc \uc218\uac00 \uc5c6\uc5c8\ub2e4. 14\ub144\uac04 \uc774\uc5b4\uc654\ub358 \uc5f0\uc18d\ucd9c\uc7a5\uae30\ub85d\uc744 \ub9c8\uac10\ud558\ub294 \ub8e8 \uac8c\ub9ad\uc774 \ube60\uc9c4 \ucd9c\uc7a5\uba85\ub2e8\uc740 \uc2ec\ud310\ub4e4\uc744 \uae5c\uc9dd \ub180\ub77c\uac8c \ud588\ub2e4. \uacbd\uae30\uac00 \uc2dc\uc791\ub418\uae30 \uc804 \ube0c\ub9ad\uc2a4 \uc2a4\ud0c0\ub514\uc6c0(Briggs Stadium, \ub2f9\uc2dc \ub514\ud2b8\ub85c\uc774\ud2b8 \ud0c0\uc774\uac70\uc2a4 \ud648\uad6c\uc7a5)\uc758 \uc7a5\ub0b4 \uc544\ub098\uc6b4\uc11c\uac00 \ud32c\ub4e4\uc5d0\uac8c \ub9d0\ud588\ub2e4.", "answer": "\ub9e4\uce74\uc2dc", "sentence": "\uacbd\uae30 \ud734\uc2dd\uc77c\uc774\uc5c8\ub358 5\uc6d4 1\uc77c\uc774 \uc9c0\ub098 \ub2e4\uc74c\ub0a0\uc778 5\uc6d4 2\uc77c, \ud0c0\uc774\uac70\uc2a4\uc640\uc758 \ub514\ud2b8\ub85c\uc774\ud2b8 \uc6d0\uc815 \uacbd\uae30\uac00 \uc2dc\uc791\ud558\uae30 \uc804 \ub8e8 \uac8c\ub9ad\uc740 \ub9e4\uce74\uc2dc \uac10\ub3c5\uc5d0\uac8c \ub2e4\uac00\uac00\uc11c \u201c\uac10\ub3c5\ub2d8, \uc624\ub298\uc740 \ubca4\uce58\uc5d0 \uc788\uc744\uac8c\uc694.", "paragraph_sentence": " \uacbd\uae30 \ud734\uc2dd\uc77c\uc774\uc5c8\ub358 5\uc6d4 1\uc77c\uc774 \uc9c0\ub098 \ub2e4\uc74c\ub0a0\uc778 5\uc6d4 2\uc77c, \ud0c0\uc774\uac70\uc2a4\uc640\uc758 \ub514\ud2b8\ub85c\uc774\ud2b8 \uc6d0\uc815 \uacbd\uae30\uac00 \uc2dc\uc791\ud558\uae30 \uc804 \ub8e8 \uac8c\ub9ad\uc740 \ub9e4\uce74\uc2dc \uac10\ub3c5\uc5d0\uac8c \ub2e4\uac00\uac00\uc11c \u201c\uac10\ub3c5\ub2d8, \uc624\ub298\uc740 \ubca4\uce58\uc5d0 \uc788\uc744\uac8c\uc694. \u201d\ub77c\uace0 \ub9d0\ud588\uace0, \uac10\ub3c5\uc740 \u201c\uadf8\uac8c \ud300\uc744 \uc704\ud55c \uae38\uc774\ub77c\uace0 \uc0dd\uac01\ud55c\ub2e4\uba74\u201d\uc774\ub77c\uace0 \ub9d0\ud588\ub2e4. \ub9e4\uce74\uc2dc\ub294 \ub9c8\uc9c0\ubabb\ud574 \ud5c8\ub77d\uc744 \ud588\uace0 \uc568\uc2a4\uc6cc\uc2a4 \"\ubca0\uc774\ube0c\" \ub2ec\uadf8\ub80c\uc744 1\ub8e8\uc218\ub85c \uae30\uc6a9\ud588\ub2e4. \uadf8\ub7ec\uba74\uc11c \ub9d0\ud558\uae38 \u201c\uc5b8\uc81c\ub77c\ub3c4 \ub2e4\uc2dc \uacbd\uae30\uc5d0 \ucd9c\uc7a5\ud558\uace0 \uc2f6\uc73c\uba74 \ub9d0\ud574\ub77c, 1\ub8e8\ub294 \ud56d\uc0c1 \ub108\uc758 \uac83\uc774\ub2e4. \u201d \uacbd\uae30 \uc2dc\uc791 \uc804, \ud300\uc758 \uc8fc\uc7a5\uc774 \uc120\ubc1c \uc120\uc218 \uba85\ub2e8\uc744 \uc8fc\uc2ec\uc5d0\uac8c \uac00\uc838\ub2e4 \uc8fc\ub294 \uad00\ub840\ub300\ub85c \uac8c\ub9ad\uc740 \uc591\ud0a4\uc2a4 \ud300\uc758 \uc120\ubc1c \uc120\uc218 \uba85\ub2e8\uc744 \uc8fc\uc2ec\uc5d0\uac8c \uac74\ub124\uc8fc\uc5c8\ub2e4. \ud558\uc9c0\ub9cc \uadf8\ub0a0 \ucd9c\uc7a5 \uba85\ub2e8\uc5d0\uc11c \uc815\uc791 \uac8c\ub9ad\uc758 \uc774\ub984\uc740 \ucc3e\uc544\ubcfc \uc218\uac00 \uc5c6\uc5c8\ub2e4. 14\ub144\uac04 \uc774\uc5b4\uc654\ub358 \uc5f0\uc18d\ucd9c\uc7a5\uae30\ub85d\uc744 \ub9c8\uac10\ud558\ub294 \ub8e8 \uac8c\ub9ad\uc774 \ube60\uc9c4 \ucd9c\uc7a5\uba85\ub2e8\uc740 \uc2ec\ud310\ub4e4\uc744 \uae5c\uc9dd \ub180\ub77c\uac8c \ud588\ub2e4. \uacbd\uae30\uac00 \uc2dc\uc791\ub418\uae30 \uc804 \ube0c\ub9ad\uc2a4 \uc2a4\ud0c0\ub514\uc6c0(Briggs Stadium, \ub2f9\uc2dc \ub514\ud2b8\ub85c\uc774\ud2b8 \ud0c0\uc774\uac70\uc2a4 \ud648\uad6c\uc7a5)\uc758 \uc7a5\ub0b4 \uc544\ub098\uc6b4\uc11c\uac00 \ud32c\ub4e4\uc5d0\uac8c \ub9d0\ud588\ub2e4.", "paragraph_answer": "\uacbd\uae30 \ud734\uc2dd\uc77c\uc774\uc5c8\ub358 5\uc6d4 1\uc77c\uc774 \uc9c0\ub098 \ub2e4\uc74c\ub0a0\uc778 5\uc6d4 2\uc77c, \ud0c0\uc774\uac70\uc2a4\uc640\uc758 \ub514\ud2b8\ub85c\uc774\ud2b8 \uc6d0\uc815 \uacbd\uae30\uac00 \uc2dc\uc791\ud558\uae30 \uc804 \ub8e8 \uac8c\ub9ad\uc740 \ub9e4\uce74\uc2dc \uac10\ub3c5\uc5d0\uac8c \ub2e4\uac00\uac00\uc11c \u201c\uac10\ub3c5\ub2d8, \uc624\ub298\uc740 \ubca4\uce58\uc5d0 \uc788\uc744\uac8c\uc694.\u201d\ub77c\uace0 \ub9d0\ud588\uace0, \uac10\ub3c5\uc740 \u201c\uadf8\uac8c \ud300\uc744 \uc704\ud55c \uae38\uc774\ub77c\uace0 \uc0dd\uac01\ud55c\ub2e4\uba74\u201d\uc774\ub77c\uace0 \ub9d0\ud588\ub2e4. \ub9e4\uce74\uc2dc\ub294 \ub9c8\uc9c0\ubabb\ud574 \ud5c8\ub77d\uc744 \ud588\uace0 \uc568\uc2a4\uc6cc\uc2a4 \"\ubca0\uc774\ube0c\" \ub2ec\uadf8\ub80c\uc744 1\ub8e8\uc218\ub85c \uae30\uc6a9\ud588\ub2e4. \uadf8\ub7ec\uba74\uc11c \ub9d0\ud558\uae38 \u201c\uc5b8\uc81c\ub77c\ub3c4 \ub2e4\uc2dc \uacbd\uae30\uc5d0 \ucd9c\uc7a5\ud558\uace0 \uc2f6\uc73c\uba74 \ub9d0\ud574\ub77c, 1\ub8e8\ub294 \ud56d\uc0c1 \ub108\uc758 \uac83\uc774\ub2e4.\u201d \uacbd\uae30 \uc2dc\uc791 \uc804, \ud300\uc758 \uc8fc\uc7a5\uc774 \uc120\ubc1c \uc120\uc218 \uba85\ub2e8\uc744 \uc8fc\uc2ec\uc5d0\uac8c \uac00\uc838\ub2e4 \uc8fc\ub294 \uad00\ub840\ub300\ub85c \uac8c\ub9ad\uc740 \uc591\ud0a4\uc2a4 \ud300\uc758 \uc120\ubc1c \uc120\uc218 \uba85\ub2e8\uc744 \uc8fc\uc2ec\uc5d0\uac8c \uac74\ub124\uc8fc\uc5c8\ub2e4. \ud558\uc9c0\ub9cc \uadf8\ub0a0 \ucd9c\uc7a5 \uba85\ub2e8\uc5d0\uc11c \uc815\uc791 \uac8c\ub9ad\uc758 \uc774\ub984\uc740 \ucc3e\uc544\ubcfc \uc218\uac00 \uc5c6\uc5c8\ub2e4. 14\ub144\uac04 \uc774\uc5b4\uc654\ub358 \uc5f0\uc18d\ucd9c\uc7a5\uae30\ub85d\uc744 \ub9c8\uac10\ud558\ub294 \ub8e8 \uac8c\ub9ad\uc774 \ube60\uc9c4 \ucd9c\uc7a5\uba85\ub2e8\uc740 \uc2ec\ud310\ub4e4\uc744 \uae5c\uc9dd \ub180\ub77c\uac8c \ud588\ub2e4. \uacbd\uae30\uac00 \uc2dc\uc791\ub418\uae30 \uc804 \ube0c\ub9ad\uc2a4 \uc2a4\ud0c0\ub514\uc6c0(Briggs Stadium, \ub2f9\uc2dc \ub514\ud2b8\ub85c\uc774\ud2b8 \ud0c0\uc774\uac70\uc2a4 \ud648\uad6c\uc7a5)\uc758 \uc7a5\ub0b4 \uc544\ub098\uc6b4\uc11c\uac00 \ud32c\ub4e4\uc5d0\uac8c \ub9d0\ud588\ub2e4.", "sentence_answer": "\uacbd\uae30 \ud734\uc2dd\uc77c\uc774\uc5c8\ub358 5\uc6d4 1\uc77c\uc774 \uc9c0\ub098 \ub2e4\uc74c\ub0a0\uc778 5\uc6d4 2\uc77c, \ud0c0\uc774\uac70\uc2a4\uc640\uc758 \ub514\ud2b8\ub85c\uc774\ud2b8 \uc6d0\uc815 \uacbd\uae30\uac00 \uc2dc\uc791\ud558\uae30 \uc804 \ub8e8 \uac8c\ub9ad\uc740 \ub9e4\uce74\uc2dc \uac10\ub3c5\uc5d0\uac8c \ub2e4\uac00\uac00\uc11c \u201c\uac10\ub3c5\ub2d8, \uc624\ub298\uc740 \ubca4\uce58\uc5d0 \uc788\uc744\uac8c\uc694.", "paragraph_id": "6602417-7-0", "paragraph_question": "question: \ub8e8 \uac8c\ub9ad \ub300\uc2e0 \uc568\uc2a4\uc6cc\uc2a4 \ubca0\uc774\ube0c \ub2ec\uadf8\ub80c\uc744 1\ub8e8\uc218\ub85c \uae30\uc6a9\ud55c \uac10\ub3c5\uc740?, context: \uacbd\uae30 \ud734\uc2dd\uc77c\uc774\uc5c8\ub358 5\uc6d4 1\uc77c\uc774 \uc9c0\ub098 \ub2e4\uc74c\ub0a0\uc778 5\uc6d4 2\uc77c, \ud0c0\uc774\uac70\uc2a4\uc640\uc758 \ub514\ud2b8\ub85c\uc774\ud2b8 \uc6d0\uc815 \uacbd\uae30\uac00 \uc2dc\uc791\ud558\uae30 \uc804 \ub8e8 \uac8c\ub9ad\uc740 \ub9e4\uce74\uc2dc \uac10\ub3c5\uc5d0\uac8c \ub2e4\uac00\uac00\uc11c \u201c\uac10\ub3c5\ub2d8, \uc624\ub298\uc740 \ubca4\uce58\uc5d0 \uc788\uc744\uac8c\uc694.\u201d\ub77c\uace0 \ub9d0\ud588\uace0, \uac10\ub3c5\uc740 \u201c\uadf8\uac8c \ud300\uc744 \uc704\ud55c \uae38\uc774\ub77c\uace0 \uc0dd\uac01\ud55c\ub2e4\uba74\u201d\uc774\ub77c\uace0 \ub9d0\ud588\ub2e4. \ub9e4\uce74\uc2dc\ub294 \ub9c8\uc9c0\ubabb\ud574 \ud5c8\ub77d\uc744 \ud588\uace0 \uc568\uc2a4\uc6cc\uc2a4 \"\ubca0\uc774\ube0c\" \ub2ec\uadf8\ub80c\uc744 1\ub8e8\uc218\ub85c \uae30\uc6a9\ud588\ub2e4. \uadf8\ub7ec\uba74\uc11c \ub9d0\ud558\uae38 \u201c\uc5b8\uc81c\ub77c\ub3c4 \ub2e4\uc2dc \uacbd\uae30\uc5d0 \ucd9c\uc7a5\ud558\uace0 \uc2f6\uc73c\uba74 \ub9d0\ud574\ub77c, 1\ub8e8\ub294 \ud56d\uc0c1 \ub108\uc758 \uac83\uc774\ub2e4.\u201d \uacbd\uae30 \uc2dc\uc791 \uc804, \ud300\uc758 \uc8fc\uc7a5\uc774 \uc120\ubc1c \uc120\uc218 \uba85\ub2e8\uc744 \uc8fc\uc2ec\uc5d0\uac8c \uac00\uc838\ub2e4 \uc8fc\ub294 \uad00\ub840\ub300\ub85c \uac8c\ub9ad\uc740 \uc591\ud0a4\uc2a4 \ud300\uc758 \uc120\ubc1c \uc120\uc218 \uba85\ub2e8\uc744 \uc8fc\uc2ec\uc5d0\uac8c \uac74\ub124\uc8fc\uc5c8\ub2e4. \ud558\uc9c0\ub9cc \uadf8\ub0a0 \ucd9c\uc7a5 \uba85\ub2e8\uc5d0\uc11c \uc815\uc791 \uac8c\ub9ad\uc758 \uc774\ub984\uc740 \ucc3e\uc544\ubcfc \uc218\uac00 \uc5c6\uc5c8\ub2e4. 14\ub144\uac04 \uc774\uc5b4\uc654\ub358 \uc5f0\uc18d\ucd9c\uc7a5\uae30\ub85d\uc744 \ub9c8\uac10\ud558\ub294 \ub8e8 \uac8c\ub9ad\uc774 \ube60\uc9c4 \ucd9c\uc7a5\uba85\ub2e8\uc740 \uc2ec\ud310\ub4e4\uc744 \uae5c\uc9dd \ub180\ub77c\uac8c \ud588\ub2e4. \uacbd\uae30\uac00 \uc2dc\uc791\ub418\uae30 \uc804 \ube0c\ub9ad\uc2a4 \uc2a4\ud0c0\ub514\uc6c0(Briggs Stadium, \ub2f9\uc2dc \ub514\ud2b8\ub85c\uc774\ud2b8 \ud0c0\uc774\uac70\uc2a4 \ud648\uad6c\uc7a5)\uc758 \uc7a5\ub0b4 \uc544\ub098\uc6b4\uc11c\uac00 \ud32c\ub4e4\uc5d0\uac8c \ub9d0\ud588\ub2e4."} {"question": "\ubcf4\uc548\uc0ac\ub839\uad00\uc758 \ud6c4\uc784\uc73c\ub85c \uc784\uba85\ub41c \ub178\ud0dc\uc6b0\uac00 \ub9e1\uace0\uc788\ub358 \uc9c1\ucc45\uc740?", "paragraph": "1980\ub144 8\uc6d4 6\uc77c \uc721\uad70 \ub300\uc7a5(\u5927\u5c07)\uc73c\ub85c \uc9c4\uae09\ud55c \uc804\ub450\ud658\uc740 \uace7 \ub300\ud1b5\ub839 \uc120\uac70 \ucd9c\ub9c8\ub97c \uc120\uc5b8\ud588\ub2e4. \ubcf4\uc548\uc0ac\uc5d0 \uc758\ud574 \uac80\uc5f4\uacfc \uc870\uc885\uc744 \ubc1b\uc558\ub358 \ud55c\uad6d \uc5b8\ub860\uc740 \ubbf8\uad6d\uc758 \ub85c\uc2a4\uc564\uc824\ub808\uc2a4 \ud0c0\uc784\uc2a4\uac00 \uc804\ub450\ud658 \uc7a5\uad70\uc744 \uc9c0\uc9c0\ud55c\ub2e4\uace0 \ubcf4\ub3c4\ud588\ub2e4. \ud558\uc9c0\ub9cc \uc774\ub294 \ubbf8\uad6d \uc778\uc0ac\ub4e4\uc758 \ubc1c\uc5b8\uc758 \uc9c4\uc758\ub97c \ud6fc\uc190\ud55c \uc624\ubcf4\uc600\ub2e4. 8\uc6d4 16\uc77c \ucd5c\uaddc\ud558 \ub300\ud1b5\ub839\uc774 \uc0ac\uc784\ud588\uace0 8\uc6d4 22\uc77c \uc804\ub450\ud658\uc774 \uad70\ub300\uc5d0\uc11c \uc804\uc5ed\ud588\ub2e4. 8\uc6d4 23\uc77c \uc804\ub450\ud658\uc740 \ubcf4\uc548\uc0ac\ub839\uad00\uc5d0\uc11c \uc0ac\uc784\ud558\uace0 \ub2f9\uc2dc \uc218\ub3c4\uacbd\ube44\uc0ac\ub839\uad00\uc774\ub358 \ub178\ud0dc\uc6b0\uac00 \ud6c4\uc784\uc73c\ub85c \uc784\uba85\ub418\uc5c8\ub2e4. 8\uc6d4 25\uc77c \ud544\ub9ac\ud540\uc758 \uc774\uba5c\ub2e4 \ub9c8\ub974\ucf54\uc2a4\ub294 \ud3c9\ud654\ub97c \uc0ac\ub791\ud558\ub294 \uc0ac\ub78c\uc740 \uc804\ub450\ud658\uc744 \uc9c0\uc9c0\ud574\uc57c \ud55c\ub2e4\uace0 \ud588\ub2e4. 8\uc6d4 27\uc77c \uc7a5\ucda9\uccb4\uc721\uad00\uc5d0\uc11c \ud1b5\uc77c\uc8fc\uccb4\uad6d\ubbfc\ud68c\uc758 \ub300\uc758\uc6d0\uc5d0 \uc758\ud574 \uc81c11\ub300 \ub300\ud1b5\ub839\uc73c\ub85c \uc120\ucd9c\ub41c \ud6c4 9\uc6d4 1\uc77c \uc7a0\uc2e4\uccb4\uc721\uad00\uc5d0\uc11c \uc81c11\ub300 \ub300\ud1b5\ub839\uc5d0 \ucde8\uc784\ud588\ub2e4.", "answer": "\uc218\ub3c4\uacbd\ube44\uc0ac\ub839\uad00", "sentence": "8\uc6d4 23\uc77c \uc804\ub450\ud658\uc740 \ubcf4\uc548\uc0ac\ub839\uad00\uc5d0\uc11c \uc0ac\uc784\ud558\uace0 \ub2f9\uc2dc \uc218\ub3c4\uacbd\ube44\uc0ac\ub839\uad00 \uc774\ub358 \ub178\ud0dc\uc6b0\uac00 \ud6c4\uc784\uc73c\ub85c \uc784\uba85\ub418\uc5c8\ub2e4.", "paragraph_sentence": "1980\ub144 8\uc6d4 6\uc77c \uc721\uad70 \ub300\uc7a5(\u5927\u5c07)\uc73c\ub85c \uc9c4\uae09\ud55c \uc804\ub450\ud658\uc740 \uace7 \ub300\ud1b5\ub839 \uc120\uac70 \ucd9c\ub9c8\ub97c \uc120\uc5b8\ud588\ub2e4. \ubcf4\uc548\uc0ac\uc5d0 \uc758\ud574 \uac80\uc5f4\uacfc \uc870\uc885\uc744 \ubc1b\uc558\ub358 \ud55c\uad6d \uc5b8\ub860\uc740 \ubbf8\uad6d\uc758 \ub85c\uc2a4\uc564\uc824\ub808\uc2a4 \ud0c0\uc784\uc2a4\uac00 \uc804\ub450\ud658 \uc7a5\uad70\uc744 \uc9c0\uc9c0\ud55c\ub2e4\uace0 \ubcf4\ub3c4\ud588\ub2e4. \ud558\uc9c0\ub9cc \uc774\ub294 \ubbf8\uad6d \uc778\uc0ac\ub4e4\uc758 \ubc1c\uc5b8\uc758 \uc9c4\uc758\ub97c \ud6fc\uc190\ud55c \uc624\ubcf4\uc600\ub2e4. 8\uc6d4 16\uc77c \ucd5c\uaddc\ud558 \ub300\ud1b5\ub839\uc774 \uc0ac\uc784\ud588\uace0 8\uc6d4 22\uc77c \uc804\ub450\ud658\uc774 \uad70\ub300\uc5d0\uc11c \uc804\uc5ed\ud588\ub2e4. 8\uc6d4 23\uc77c \uc804\ub450\ud658\uc740 \ubcf4\uc548\uc0ac\ub839\uad00\uc5d0\uc11c \uc0ac\uc784\ud558\uace0 \ub2f9\uc2dc \uc218\ub3c4\uacbd\ube44\uc0ac\ub839\uad00 \uc774\ub358 \ub178\ud0dc\uc6b0\uac00 \ud6c4\uc784\uc73c\ub85c \uc784\uba85\ub418\uc5c8\ub2e4. 8\uc6d4 25\uc77c \ud544\ub9ac\ud540\uc758 \uc774\uba5c\ub2e4 \ub9c8\ub974\ucf54\uc2a4\ub294 \ud3c9\ud654\ub97c \uc0ac\ub791\ud558\ub294 \uc0ac\ub78c\uc740 \uc804\ub450\ud658\uc744 \uc9c0\uc9c0\ud574\uc57c \ud55c\ub2e4\uace0 \ud588\ub2e4. 8\uc6d4 27\uc77c \uc7a5\ucda9\uccb4\uc721\uad00\uc5d0\uc11c \ud1b5\uc77c\uc8fc\uccb4\uad6d\ubbfc\ud68c\uc758 \ub300\uc758\uc6d0\uc5d0 \uc758\ud574 \uc81c11\ub300 \ub300\ud1b5\ub839\uc73c\ub85c \uc120\ucd9c\ub41c \ud6c4 9\uc6d4 1\uc77c \uc7a0\uc2e4\uccb4\uc721\uad00\uc5d0\uc11c \uc81c11\ub300 \ub300\ud1b5\ub839\uc5d0 \ucde8\uc784\ud588\ub2e4.", "paragraph_answer": "1980\ub144 8\uc6d4 6\uc77c \uc721\uad70 \ub300\uc7a5(\u5927\u5c07)\uc73c\ub85c \uc9c4\uae09\ud55c \uc804\ub450\ud658\uc740 \uace7 \ub300\ud1b5\ub839 \uc120\uac70 \ucd9c\ub9c8\ub97c \uc120\uc5b8\ud588\ub2e4. \ubcf4\uc548\uc0ac\uc5d0 \uc758\ud574 \uac80\uc5f4\uacfc \uc870\uc885\uc744 \ubc1b\uc558\ub358 \ud55c\uad6d \uc5b8\ub860\uc740 \ubbf8\uad6d\uc758 \ub85c\uc2a4\uc564\uc824\ub808\uc2a4 \ud0c0\uc784\uc2a4\uac00 \uc804\ub450\ud658 \uc7a5\uad70\uc744 \uc9c0\uc9c0\ud55c\ub2e4\uace0 \ubcf4\ub3c4\ud588\ub2e4. \ud558\uc9c0\ub9cc \uc774\ub294 \ubbf8\uad6d \uc778\uc0ac\ub4e4\uc758 \ubc1c\uc5b8\uc758 \uc9c4\uc758\ub97c \ud6fc\uc190\ud55c \uc624\ubcf4\uc600\ub2e4. 8\uc6d4 16\uc77c \ucd5c\uaddc\ud558 \ub300\ud1b5\ub839\uc774 \uc0ac\uc784\ud588\uace0 8\uc6d4 22\uc77c \uc804\ub450\ud658\uc774 \uad70\ub300\uc5d0\uc11c \uc804\uc5ed\ud588\ub2e4. 8\uc6d4 23\uc77c \uc804\ub450\ud658\uc740 \ubcf4\uc548\uc0ac\ub839\uad00\uc5d0\uc11c \uc0ac\uc784\ud558\uace0 \ub2f9\uc2dc \uc218\ub3c4\uacbd\ube44\uc0ac\ub839\uad00 \uc774\ub358 \ub178\ud0dc\uc6b0\uac00 \ud6c4\uc784\uc73c\ub85c \uc784\uba85\ub418\uc5c8\ub2e4. 8\uc6d4 25\uc77c \ud544\ub9ac\ud540\uc758 \uc774\uba5c\ub2e4 \ub9c8\ub974\ucf54\uc2a4\ub294 \ud3c9\ud654\ub97c \uc0ac\ub791\ud558\ub294 \uc0ac\ub78c\uc740 \uc804\ub450\ud658\uc744 \uc9c0\uc9c0\ud574\uc57c \ud55c\ub2e4\uace0 \ud588\ub2e4. 8\uc6d4 27\uc77c \uc7a5\ucda9\uccb4\uc721\uad00\uc5d0\uc11c \ud1b5\uc77c\uc8fc\uccb4\uad6d\ubbfc\ud68c\uc758 \ub300\uc758\uc6d0\uc5d0 \uc758\ud574 \uc81c11\ub300 \ub300\ud1b5\ub839\uc73c\ub85c \uc120\ucd9c\ub41c \ud6c4 9\uc6d4 1\uc77c \uc7a0\uc2e4\uccb4\uc721\uad00\uc5d0\uc11c \uc81c11\ub300 \ub300\ud1b5\ub839\uc5d0 \ucde8\uc784\ud588\ub2e4.", "sentence_answer": "8\uc6d4 23\uc77c \uc804\ub450\ud658\uc740 \ubcf4\uc548\uc0ac\ub839\uad00\uc5d0\uc11c \uc0ac\uc784\ud558\uace0 \ub2f9\uc2dc \uc218\ub3c4\uacbd\ube44\uc0ac\ub839\uad00 \uc774\ub358 \ub178\ud0dc\uc6b0\uac00 \ud6c4\uc784\uc73c\ub85c \uc784\uba85\ub418\uc5c8\ub2e4.", "paragraph_id": "6657628-15-1", "paragraph_question": "question: \ubcf4\uc548\uc0ac\ub839\uad00\uc758 \ud6c4\uc784\uc73c\ub85c \uc784\uba85\ub41c \ub178\ud0dc\uc6b0\uac00 \ub9e1\uace0\uc788\ub358 \uc9c1\ucc45\uc740?, context: 1980\ub144 8\uc6d4 6\uc77c \uc721\uad70 \ub300\uc7a5(\u5927\u5c07)\uc73c\ub85c \uc9c4\uae09\ud55c \uc804\ub450\ud658\uc740 \uace7 \ub300\ud1b5\ub839 \uc120\uac70 \ucd9c\ub9c8\ub97c \uc120\uc5b8\ud588\ub2e4. \ubcf4\uc548\uc0ac\uc5d0 \uc758\ud574 \uac80\uc5f4\uacfc \uc870\uc885\uc744 \ubc1b\uc558\ub358 \ud55c\uad6d \uc5b8\ub860\uc740 \ubbf8\uad6d\uc758 \ub85c\uc2a4\uc564\uc824\ub808\uc2a4 \ud0c0\uc784\uc2a4\uac00 \uc804\ub450\ud658 \uc7a5\uad70\uc744 \uc9c0\uc9c0\ud55c\ub2e4\uace0 \ubcf4\ub3c4\ud588\ub2e4. \ud558\uc9c0\ub9cc \uc774\ub294 \ubbf8\uad6d \uc778\uc0ac\ub4e4\uc758 \ubc1c\uc5b8\uc758 \uc9c4\uc758\ub97c \ud6fc\uc190\ud55c \uc624\ubcf4\uc600\ub2e4. 8\uc6d4 16\uc77c \ucd5c\uaddc\ud558 \ub300\ud1b5\ub839\uc774 \uc0ac\uc784\ud588\uace0 8\uc6d4 22\uc77c \uc804\ub450\ud658\uc774 \uad70\ub300\uc5d0\uc11c \uc804\uc5ed\ud588\ub2e4. 8\uc6d4 23\uc77c \uc804\ub450\ud658\uc740 \ubcf4\uc548\uc0ac\ub839\uad00\uc5d0\uc11c \uc0ac\uc784\ud558\uace0 \ub2f9\uc2dc \uc218\ub3c4\uacbd\ube44\uc0ac\ub839\uad00\uc774\ub358 \ub178\ud0dc\uc6b0\uac00 \ud6c4\uc784\uc73c\ub85c \uc784\uba85\ub418\uc5c8\ub2e4. 8\uc6d4 25\uc77c \ud544\ub9ac\ud540\uc758 \uc774\uba5c\ub2e4 \ub9c8\ub974\ucf54\uc2a4\ub294 \ud3c9\ud654\ub97c \uc0ac\ub791\ud558\ub294 \uc0ac\ub78c\uc740 \uc804\ub450\ud658\uc744 \uc9c0\uc9c0\ud574\uc57c \ud55c\ub2e4\uace0 \ud588\ub2e4. 8\uc6d4 27\uc77c \uc7a5\ucda9\uccb4\uc721\uad00\uc5d0\uc11c \ud1b5\uc77c\uc8fc\uccb4\uad6d\ubbfc\ud68c\uc758 \ub300\uc758\uc6d0\uc5d0 \uc758\ud574 \uc81c11\ub300 \ub300\ud1b5\ub839\uc73c\ub85c \uc120\ucd9c\ub41c \ud6c4 9\uc6d4 1\uc77c \uc7a0\uc2e4\uccb4\uc721\uad00\uc5d0\uc11c \uc81c11\ub300 \ub300\ud1b5\ub839\uc5d0 \ucde8\uc784\ud588\ub2e4."} {"question": "\uae40\ub300\uc911\uc774 \ub178\ubca8 \ud3c9\ud654\uc0c1\uc744 \uc218\uc0c1\ud55c \uc5f0\ub3c4\ub294?", "paragraph": "\uc804\ub0a8 \uc2e0\uc548\uc5d0\uc11c \ud0dc\uc5b4\ub0ac\uace0, \uc81c6\u00b77\u00b78\u00b713\u00b714\ub300 \uad6d\ud68c\uc758\uc6d0\uc744 \uc9c0\ub0c8\ub2e4. \uad70\ubd80 \uc815\uad8c\uc758 \uc704\ud611\uc73c\ub85c \uc5ec\ub7ec \ubc88 \uc8fd\uc744 \uace0\ube44\ub97c \ub118\uae30\uba74\uc11c \uae40\uc601\uc0bc\uacfc \ud568\uaed8 \uad6d\ub0b4\uc678\uc5d0\uc11c \uc624\ub7ab\ub3d9\uc548 \ubbfc\uc8fc \uc9c4\uc601\uc758 \uc9c0\ub3c4\uc790\ub85c \ud65c\ub3d9\ud558\uba70 \uad70\uc0ac \uc815\uad8c\uc5d0 \ud56d\uac70\ud588\ub2e4. \uae40\ub300\uc911\uc740 \uad70\ubd80 \uc815\uad8c\uc73c\ub85c\ubd80\ud130 \ub0a9\uce58\uc640 \uac00\ud0dd\uc5f0\uae08, \ud22c\uc625 \ub4f1\uc758 \uc5ec\ub7ec \ud0c4\uc555\uc744 \ubc1b\uc558\ub2e4. 1987\ub144 6\uc6d4 \ubbfc\uc8fc \ud56d\uc7c1 \uc774\ud6c4\uc5d0\ub294 \ud1b5\uc77c\ubbfc\uc8fc\ub2f9\uc758 \uc0c1\uc784\uace0\ubb38\uc73c\ub85c \ud65c\ub3d9\ud558\uba70 \ubbfc\uc8fc\ud654\ucd94\uc9c4\ud611\uc758\ud68c\ub97c \uad6c\uc131\ud574 \uc774\ub978\ubc14 \ubbfc\uc8fc \uc9c4\uc601\uc744 \uad6c\ucd95\ud558\uc600\ub2e4. \ub300\ub0b4\uc678\uc801\uc73c\ub85c\ub294 \ubbfc\uc8fc\ud654\ub97c \uc9c0\uc9c0\ud588\uc73c\ub098, \uc778\uad8c \ud5a5\uc0c1\uacfc \ub0a8\ubd81 \uad00\uacc4\uc758 \uc9c4\uc804\uc5d0 \uae30\uc5ec\ud55c \uacf5\ub85c\ub85c 2000\ub144 \uc784\uae30 \uc911\uc5d0 \ud55c\uad6d\uc778 \uc5ed\uc0ac\uc0c1 \ucd5c\ucd08\ub85c \ub178\ubca8 \ud3c9\ud654\uc0c1\uc744 \uc218\uc0c1\ud558\uc600\ub2e4. 2000\ub144 \ub178\ub974\uc6e8\uc774 \ub77c\ud504\ud1a0(Rafto) \uc778\uad8c\uc0c1, 1998\ub144 \ubb34\uad81\ud654 \ub300\ud6c8\uc7a5, 1998\ub144 \uad6d\uc81c\uc778\uad8c\uc5f0\ub9f9 \uc778\uad8c\uc0c1, 1999\ub144 \ubbf8\uad6d \ud544\ub77c\ub378\ud53c\uc544\uc2dc \uc790\uc720\uc758 \uba54\ub2ec, \ubd81\ubbf8\uc8fc \ud55c\uad6d\uc778\uad8c\uc5f0\ud569 \uc778\uad8c\uc0c1, \ubbf8\uad6d \uc870\uc9c0 \ubbf8\ub2c8(George Meany) \uc778\uad8c\uc0c1, \ube0c\ub8e8\ub178 \ud06c\ub77c\uc774\uc2a4\ud0a4(Bruno Kreisky) \uc778\uad8c\uc0c1\uc744 \uc218\uc0c1\ud558\uc600\ub2e4. \uc5f0\uc124\uc5d0 \ub2a5\ud558\uc600\uc73c\uba70, \uad6d\ud68c\uc5d0\uc11c \uac00\uc7a5 \uc624\ub798 \uc5f0\uc124\ud55c \uae30\ub85d\uc73c\ub85c \uae30\ub124\uc2a4\ubd81 \uc99d\uc11c\ub97c \ubc1b\uc558\ub2e4. \ucd94\uc6b4 \uaca8\uc6b8\uc5d0\ub3c4 \uc628\uac16 \ud48d\uc0c1(\u98a8\u971c)\uc744 \ucc38\uace0 \uc774\uaca8\ub0b4\ub294 \uc778\ub3d9\ucd08(\u5fcd\u51ac\u8349)\ub85c \ube44\uc720\ub418\uc5b4 \ubd88\ub9ac\uae30\ub3c4 \ud558\uc600\ub2e4.", "answer": "2000\ub144", "sentence": "\ub300\ub0b4\uc678\uc801\uc73c\ub85c\ub294 \ubbfc\uc8fc\ud654\ub97c \uc9c0\uc9c0\ud588\uc73c\ub098, \uc778\uad8c \ud5a5\uc0c1\uacfc \ub0a8\ubd81 \uad00\uacc4\uc758 \uc9c4\uc804\uc5d0 \uae30\uc5ec\ud55c \uacf5\ub85c\ub85c 2000\ub144 \uc784\uae30 \uc911\uc5d0 \ud55c\uad6d\uc778 \uc5ed\uc0ac\uc0c1 \ucd5c\ucd08\ub85c \ub178\ubca8 \ud3c9\ud654\uc0c1\uc744 \uc218\uc0c1\ud558\uc600\ub2e4.", "paragraph_sentence": "\uc804\ub0a8 \uc2e0\uc548\uc5d0\uc11c \ud0dc\uc5b4\ub0ac\uace0, \uc81c6\u00b77\u00b78\u00b713\u00b714\ub300 \uad6d\ud68c\uc758\uc6d0\uc744 \uc9c0\ub0c8\ub2e4. \uad70\ubd80 \uc815\uad8c\uc758 \uc704\ud611\uc73c\ub85c \uc5ec\ub7ec \ubc88 \uc8fd\uc744 \uace0\ube44\ub97c \ub118\uae30\uba74\uc11c \uae40\uc601\uc0bc\uacfc \ud568\uaed8 \uad6d\ub0b4\uc678\uc5d0\uc11c \uc624\ub7ab\ub3d9\uc548 \ubbfc\uc8fc \uc9c4\uc601\uc758 \uc9c0\ub3c4\uc790\ub85c \ud65c\ub3d9\ud558\uba70 \uad70\uc0ac \uc815\uad8c\uc5d0 \ud56d\uac70\ud588\ub2e4. \uae40\ub300\uc911\uc740 \uad70\ubd80 \uc815\uad8c\uc73c\ub85c\ubd80\ud130 \ub0a9\uce58\uc640 \uac00\ud0dd\uc5f0\uae08, \ud22c\uc625 \ub4f1\uc758 \uc5ec\ub7ec \ud0c4\uc555\uc744 \ubc1b\uc558\ub2e4. 1987\ub144 6\uc6d4 \ubbfc\uc8fc \ud56d\uc7c1 \uc774\ud6c4\uc5d0\ub294 \ud1b5\uc77c\ubbfc\uc8fc\ub2f9\uc758 \uc0c1\uc784\uace0\ubb38\uc73c\ub85c \ud65c\ub3d9\ud558\uba70 \ubbfc\uc8fc\ud654\ucd94\uc9c4\ud611\uc758\ud68c\ub97c \uad6c\uc131\ud574 \uc774\ub978\ubc14 \ubbfc\uc8fc \uc9c4\uc601\uc744 \uad6c\ucd95\ud558\uc600\ub2e4. \ub300\ub0b4\uc678\uc801\uc73c\ub85c\ub294 \ubbfc\uc8fc\ud654\ub97c \uc9c0\uc9c0\ud588\uc73c\ub098, \uc778\uad8c \ud5a5\uc0c1\uacfc \ub0a8\ubd81 \uad00\uacc4\uc758 \uc9c4\uc804\uc5d0 \uae30\uc5ec\ud55c \uacf5\ub85c\ub85c 2000\ub144 \uc784\uae30 \uc911\uc5d0 \ud55c\uad6d\uc778 \uc5ed\uc0ac\uc0c1 \ucd5c\ucd08\ub85c \ub178\ubca8 \ud3c9\ud654\uc0c1\uc744 \uc218\uc0c1\ud558\uc600\ub2e4. 2000\ub144 \ub178\ub974\uc6e8\uc774 \ub77c\ud504\ud1a0(Rafto) \uc778\uad8c\uc0c1, 1998\ub144 \ubb34\uad81\ud654 \ub300\ud6c8\uc7a5, 1998\ub144 \uad6d\uc81c\uc778\uad8c\uc5f0\ub9f9 \uc778\uad8c\uc0c1, 1999\ub144 \ubbf8\uad6d \ud544\ub77c\ub378\ud53c\uc544\uc2dc \uc790\uc720\uc758 \uba54\ub2ec, \ubd81\ubbf8\uc8fc \ud55c\uad6d\uc778\uad8c\uc5f0\ud569 \uc778\uad8c\uc0c1, \ubbf8\uad6d \uc870\uc9c0 \ubbf8\ub2c8(George Meany) \uc778\uad8c\uc0c1, \ube0c\ub8e8\ub178 \ud06c\ub77c\uc774\uc2a4\ud0a4(Bruno Kreisky) \uc778\uad8c\uc0c1\uc744 \uc218\uc0c1\ud558\uc600\ub2e4. \uc5f0\uc124\uc5d0 \ub2a5\ud558\uc600\uc73c\uba70, \uad6d\ud68c\uc5d0\uc11c \uac00\uc7a5 \uc624\ub798 \uc5f0\uc124\ud55c \uae30\ub85d\uc73c\ub85c \uae30\ub124\uc2a4\ubd81 \uc99d\uc11c\ub97c \ubc1b\uc558\ub2e4. \ucd94\uc6b4 \uaca8\uc6b8\uc5d0\ub3c4 \uc628\uac16 \ud48d\uc0c1(\u98a8\u971c)\uc744 \ucc38\uace0 \uc774\uaca8\ub0b4\ub294 \uc778\ub3d9\ucd08(\u5fcd\u51ac\u8349)\ub85c \ube44\uc720\ub418\uc5b4 \ubd88\ub9ac\uae30\ub3c4 \ud558\uc600\ub2e4.", "paragraph_answer": "\uc804\ub0a8 \uc2e0\uc548\uc5d0\uc11c \ud0dc\uc5b4\ub0ac\uace0, \uc81c6\u00b77\u00b78\u00b713\u00b714\ub300 \uad6d\ud68c\uc758\uc6d0\uc744 \uc9c0\ub0c8\ub2e4. \uad70\ubd80 \uc815\uad8c\uc758 \uc704\ud611\uc73c\ub85c \uc5ec\ub7ec \ubc88 \uc8fd\uc744 \uace0\ube44\ub97c \ub118\uae30\uba74\uc11c \uae40\uc601\uc0bc\uacfc \ud568\uaed8 \uad6d\ub0b4\uc678\uc5d0\uc11c \uc624\ub7ab\ub3d9\uc548 \ubbfc\uc8fc \uc9c4\uc601\uc758 \uc9c0\ub3c4\uc790\ub85c \ud65c\ub3d9\ud558\uba70 \uad70\uc0ac \uc815\uad8c\uc5d0 \ud56d\uac70\ud588\ub2e4. \uae40\ub300\uc911\uc740 \uad70\ubd80 \uc815\uad8c\uc73c\ub85c\ubd80\ud130 \ub0a9\uce58\uc640 \uac00\ud0dd\uc5f0\uae08, \ud22c\uc625 \ub4f1\uc758 \uc5ec\ub7ec \ud0c4\uc555\uc744 \ubc1b\uc558\ub2e4. 1987\ub144 6\uc6d4 \ubbfc\uc8fc \ud56d\uc7c1 \uc774\ud6c4\uc5d0\ub294 \ud1b5\uc77c\ubbfc\uc8fc\ub2f9\uc758 \uc0c1\uc784\uace0\ubb38\uc73c\ub85c \ud65c\ub3d9\ud558\uba70 \ubbfc\uc8fc\ud654\ucd94\uc9c4\ud611\uc758\ud68c\ub97c \uad6c\uc131\ud574 \uc774\ub978\ubc14 \ubbfc\uc8fc \uc9c4\uc601\uc744 \uad6c\ucd95\ud558\uc600\ub2e4. \ub300\ub0b4\uc678\uc801\uc73c\ub85c\ub294 \ubbfc\uc8fc\ud654\ub97c \uc9c0\uc9c0\ud588\uc73c\ub098, \uc778\uad8c \ud5a5\uc0c1\uacfc \ub0a8\ubd81 \uad00\uacc4\uc758 \uc9c4\uc804\uc5d0 \uae30\uc5ec\ud55c \uacf5\ub85c\ub85c 2000\ub144 \uc784\uae30 \uc911\uc5d0 \ud55c\uad6d\uc778 \uc5ed\uc0ac\uc0c1 \ucd5c\ucd08\ub85c \ub178\ubca8 \ud3c9\ud654\uc0c1\uc744 \uc218\uc0c1\ud558\uc600\ub2e4. 2000\ub144 \ub178\ub974\uc6e8\uc774 \ub77c\ud504\ud1a0(Rafto) \uc778\uad8c\uc0c1, 1998\ub144 \ubb34\uad81\ud654 \ub300\ud6c8\uc7a5, 1998\ub144 \uad6d\uc81c\uc778\uad8c\uc5f0\ub9f9 \uc778\uad8c\uc0c1, 1999\ub144 \ubbf8\uad6d \ud544\ub77c\ub378\ud53c\uc544\uc2dc \uc790\uc720\uc758 \uba54\ub2ec, \ubd81\ubbf8\uc8fc \ud55c\uad6d\uc778\uad8c\uc5f0\ud569 \uc778\uad8c\uc0c1, \ubbf8\uad6d \uc870\uc9c0 \ubbf8\ub2c8(George Meany) \uc778\uad8c\uc0c1, \ube0c\ub8e8\ub178 \ud06c\ub77c\uc774\uc2a4\ud0a4(Bruno Kreisky) \uc778\uad8c\uc0c1\uc744 \uc218\uc0c1\ud558\uc600\ub2e4. \uc5f0\uc124\uc5d0 \ub2a5\ud558\uc600\uc73c\uba70, \uad6d\ud68c\uc5d0\uc11c \uac00\uc7a5 \uc624\ub798 \uc5f0\uc124\ud55c \uae30\ub85d\uc73c\ub85c \uae30\ub124\uc2a4\ubd81 \uc99d\uc11c\ub97c \ubc1b\uc558\ub2e4. \ucd94\uc6b4 \uaca8\uc6b8\uc5d0\ub3c4 \uc628\uac16 \ud48d\uc0c1(\u98a8\u971c)\uc744 \ucc38\uace0 \uc774\uaca8\ub0b4\ub294 \uc778\ub3d9\ucd08(\u5fcd\u51ac\u8349)\ub85c \ube44\uc720\ub418\uc5b4 \ubd88\ub9ac\uae30\ub3c4 \ud558\uc600\ub2e4.", "sentence_answer": "\ub300\ub0b4\uc678\uc801\uc73c\ub85c\ub294 \ubbfc\uc8fc\ud654\ub97c \uc9c0\uc9c0\ud588\uc73c\ub098, \uc778\uad8c \ud5a5\uc0c1\uacfc \ub0a8\ubd81 \uad00\uacc4\uc758 \uc9c4\uc804\uc5d0 \uae30\uc5ec\ud55c \uacf5\ub85c\ub85c 2000\ub144 \uc784\uae30 \uc911\uc5d0 \ud55c\uad6d\uc778 \uc5ed\uc0ac\uc0c1 \ucd5c\ucd08\ub85c \ub178\ubca8 \ud3c9\ud654\uc0c1\uc744 \uc218\uc0c1\ud558\uc600\ub2e4.", "paragraph_id": "6656766-0-2", "paragraph_question": "question: \uae40\ub300\uc911\uc774 \ub178\ubca8 \ud3c9\ud654\uc0c1\uc744 \uc218\uc0c1\ud55c \uc5f0\ub3c4\ub294?, context: \uc804\ub0a8 \uc2e0\uc548\uc5d0\uc11c \ud0dc\uc5b4\ub0ac\uace0, \uc81c6\u00b77\u00b78\u00b713\u00b714\ub300 \uad6d\ud68c\uc758\uc6d0\uc744 \uc9c0\ub0c8\ub2e4. \uad70\ubd80 \uc815\uad8c\uc758 \uc704\ud611\uc73c\ub85c \uc5ec\ub7ec \ubc88 \uc8fd\uc744 \uace0\ube44\ub97c \ub118\uae30\uba74\uc11c \uae40\uc601\uc0bc\uacfc \ud568\uaed8 \uad6d\ub0b4\uc678\uc5d0\uc11c \uc624\ub7ab\ub3d9\uc548 \ubbfc\uc8fc \uc9c4\uc601\uc758 \uc9c0\ub3c4\uc790\ub85c \ud65c\ub3d9\ud558\uba70 \uad70\uc0ac \uc815\uad8c\uc5d0 \ud56d\uac70\ud588\ub2e4. \uae40\ub300\uc911\uc740 \uad70\ubd80 \uc815\uad8c\uc73c\ub85c\ubd80\ud130 \ub0a9\uce58\uc640 \uac00\ud0dd\uc5f0\uae08, \ud22c\uc625 \ub4f1\uc758 \uc5ec\ub7ec \ud0c4\uc555\uc744 \ubc1b\uc558\ub2e4. 1987\ub144 6\uc6d4 \ubbfc\uc8fc \ud56d\uc7c1 \uc774\ud6c4\uc5d0\ub294 \ud1b5\uc77c\ubbfc\uc8fc\ub2f9\uc758 \uc0c1\uc784\uace0\ubb38\uc73c\ub85c \ud65c\ub3d9\ud558\uba70 \ubbfc\uc8fc\ud654\ucd94\uc9c4\ud611\uc758\ud68c\ub97c \uad6c\uc131\ud574 \uc774\ub978\ubc14 \ubbfc\uc8fc \uc9c4\uc601\uc744 \uad6c\ucd95\ud558\uc600\ub2e4. \ub300\ub0b4\uc678\uc801\uc73c\ub85c\ub294 \ubbfc\uc8fc\ud654\ub97c \uc9c0\uc9c0\ud588\uc73c\ub098, \uc778\uad8c \ud5a5\uc0c1\uacfc \ub0a8\ubd81 \uad00\uacc4\uc758 \uc9c4\uc804\uc5d0 \uae30\uc5ec\ud55c \uacf5\ub85c\ub85c 2000\ub144 \uc784\uae30 \uc911\uc5d0 \ud55c\uad6d\uc778 \uc5ed\uc0ac\uc0c1 \ucd5c\ucd08\ub85c \ub178\ubca8 \ud3c9\ud654\uc0c1\uc744 \uc218\uc0c1\ud558\uc600\ub2e4. 2000\ub144 \ub178\ub974\uc6e8\uc774 \ub77c\ud504\ud1a0(Rafto) \uc778\uad8c\uc0c1, 1998\ub144 \ubb34\uad81\ud654 \ub300\ud6c8\uc7a5, 1998\ub144 \uad6d\uc81c\uc778\uad8c\uc5f0\ub9f9 \uc778\uad8c\uc0c1, 1999\ub144 \ubbf8\uad6d \ud544\ub77c\ub378\ud53c\uc544\uc2dc \uc790\uc720\uc758 \uba54\ub2ec, \ubd81\ubbf8\uc8fc \ud55c\uad6d\uc778\uad8c\uc5f0\ud569 \uc778\uad8c\uc0c1, \ubbf8\uad6d \uc870\uc9c0 \ubbf8\ub2c8(George Meany) \uc778\uad8c\uc0c1, \ube0c\ub8e8\ub178 \ud06c\ub77c\uc774\uc2a4\ud0a4(Bruno Kreisky) \uc778\uad8c\uc0c1\uc744 \uc218\uc0c1\ud558\uc600\ub2e4. \uc5f0\uc124\uc5d0 \ub2a5\ud558\uc600\uc73c\uba70, \uad6d\ud68c\uc5d0\uc11c \uac00\uc7a5 \uc624\ub798 \uc5f0\uc124\ud55c \uae30\ub85d\uc73c\ub85c \uae30\ub124\uc2a4\ubd81 \uc99d\uc11c\ub97c \ubc1b\uc558\ub2e4. \ucd94\uc6b4 \uaca8\uc6b8\uc5d0\ub3c4 \uc628\uac16 \ud48d\uc0c1(\u98a8\u971c)\uc744 \ucc38\uace0 \uc774\uaca8\ub0b4\ub294 \uc778\ub3d9\ucd08(\u5fcd\u51ac\u8349)\ub85c \ube44\uc720\ub418\uc5b4 \ubd88\ub9ac\uae30\ub3c4 \ud558\uc600\ub2e4."} {"question": "\uccad\uc18c\ub144 \uc62c\ub9bc\ud53d\uc774 \ucc98\uc74c \uac1c\ucd5c\ub41c \uac83\uc740 \uc5b8\uc81c\uc778\uac00?", "paragraph": "2010\ub144\uc5d0 \uccab\ub300\ud68c\uac00 \uac1c\ucd5c\ub418\uba70 14~18\uc138\uac00 \ucc38\uac00\ud558\ub294 \ub300\ud68c\uc774\ub2e4. \uccad\uc18c\ub144 \uc62c\ub9bc\ud53d\uc740 IOC\uc704\uc6d0\uc7a5\uc778 \uc790\ud06c \ub85c\uac8c\uac00 2001\ub144\uc5d0 \uace0\uc548\ud55c \uac83\uc73c\ub85c\uc368 2007\ub144 \uc81c119\ucc28 IOC\ucd1d\ud68c\ub54c \uc2b9\uc778\ub418\uc5c8\ub2e4. \uc81c1\ud68c \ud558\uacc4 \uccad\uc18c\ub144 \uc62c\ub9bc\ud53d\uc740 2010\ub144 \uc2f1\uac00\ud3ec\ub974\uc5d0\uc11c \uac1c\ucd5c\ub418\uc5c8\uc73c\uba70 \uc81c1\ud68c \ub3d9\uacc4 \uccad\uc18c\ub144 \uc62c\ub9bc\ud53d\uc740 2\ub144\ud6c4\uc778 2012\ub144 \uc624\uc2a4\ud2b8\ub9ac\uc544\uc758 \uc778\uc2a4\ube0c\ub8e8\ud06c\uc5d0\uc11c \uc5f4\ub838\ub2e4. \uc774 \uc62c\ub9bc\ud53d\uc740 \uc131\uc778 \uc62c\ub9bc\ud53d\ubcf4\ub2e4 \uc9e7\uc740 \uae30\uac04 \ub0b4\uc5d0 \uc9c4\ud589\ub418\uba70 \ud558\uacc4 \uc62c\ub9bc\ud53d\uc740 12\uc77c, \ub3d9\uacc4 \uc62c\ub9bc\ud53d\uc740 9\uc77c\uac04 \uc5f4\ub9b0\ub2e4. IOC\ub294 \ud558\uacc4 \uc720\uc2a4 \uc62c\ub9bc\ud53d\uc5d0 3,500 \uba85\uc758 \uc120\uc218\ub2e8\uacfc 875 \uba85\uc758 \uc784\uc9c1\uc6d0, \ub3d9\uacc4 \uc720\uc2a4 \uc62c\ub9bc\ud53d\uc5d0\ub294 970 \uba85\uc758 \uc120\uc218\ub2e8\uacfc 580 \uba85\uc758 \uc784\uc9c1\uc6d0\uc774 \ucc38\uc5ec\ud55c\ub2e4\uace0 \ub9d0\ud588\ub2e4. \uacbd\uae30\uc885\ubaa9\uc740 \uc131\uc778 \uc62c\ub9bc\ud53d\uacfc \ub3d9\uc77c\ud558\ub098 \uc138\ubd80\uc885\ubaa9\uc5d0 \uc788\uc5b4\uc11c\ub294 \uc81c\uc678\ub418\ub294 \uc885\ubaa9\uc774 \uc788\ub2e4.", "answer": "2010\ub144", "sentence": "2010\ub144 \uc5d0 \uccab\ub300\ud68c\uac00 \uac1c\ucd5c\ub418\uba70 14~18\uc138\uac00 \ucc38\uac00\ud558\ub294 \ub300\ud68c\uc774\ub2e4.", "paragraph_sentence": " 2010\ub144 \uc5d0 \uccab\ub300\ud68c\uac00 \uac1c\ucd5c\ub418\uba70 14~18\uc138\uac00 \ucc38\uac00\ud558\ub294 \ub300\ud68c\uc774\ub2e4. \uccad\uc18c\ub144 \uc62c\ub9bc\ud53d\uc740 IOC\uc704\uc6d0\uc7a5\uc778 \uc790\ud06c \ub85c\uac8c\uac00 2001\ub144\uc5d0 \uace0\uc548\ud55c \uac83\uc73c\ub85c\uc368 2007\ub144 \uc81c119\ucc28 IOC\ucd1d\ud68c\ub54c \uc2b9\uc778\ub418\uc5c8\ub2e4. \uc81c1\ud68c \ud558\uacc4 \uccad\uc18c\ub144 \uc62c\ub9bc\ud53d\uc740 2010\ub144 \uc2f1\uac00\ud3ec\ub974\uc5d0\uc11c \uac1c\ucd5c\ub418\uc5c8\uc73c\uba70 \uc81c1\ud68c \ub3d9\uacc4 \uccad\uc18c\ub144 \uc62c\ub9bc\ud53d\uc740 2\ub144\ud6c4\uc778 2012\ub144 \uc624\uc2a4\ud2b8\ub9ac\uc544\uc758 \uc778\uc2a4\ube0c\ub8e8\ud06c\uc5d0\uc11c \uc5f4\ub838\ub2e4. \uc774 \uc62c\ub9bc\ud53d\uc740 \uc131\uc778 \uc62c\ub9bc\ud53d\ubcf4\ub2e4 \uc9e7\uc740 \uae30\uac04 \ub0b4\uc5d0 \uc9c4\ud589\ub418\uba70 \ud558\uacc4 \uc62c\ub9bc\ud53d\uc740 12\uc77c, \ub3d9\uacc4 \uc62c\ub9bc\ud53d\uc740 9\uc77c\uac04 \uc5f4\ub9b0\ub2e4. IOC\ub294 \ud558\uacc4 \uc720\uc2a4 \uc62c\ub9bc\ud53d\uc5d0 3,500 \uba85\uc758 \uc120\uc218\ub2e8\uacfc 875 \uba85\uc758 \uc784\uc9c1\uc6d0, \ub3d9\uacc4 \uc720\uc2a4 \uc62c\ub9bc\ud53d\uc5d0\ub294 970 \uba85\uc758 \uc120\uc218\ub2e8\uacfc 580 \uba85\uc758 \uc784\uc9c1\uc6d0\uc774 \ucc38\uc5ec\ud55c\ub2e4\uace0 \ub9d0\ud588\ub2e4. \uacbd\uae30\uc885\ubaa9\uc740 \uc131\uc778 \uc62c\ub9bc\ud53d\uacfc \ub3d9\uc77c\ud558\ub098 \uc138\ubd80\uc885\ubaa9\uc5d0 \uc788\uc5b4\uc11c\ub294 \uc81c\uc678\ub418\ub294 \uc885\ubaa9\uc774 \uc788\ub2e4.", "paragraph_answer": " 2010\ub144 \uc5d0 \uccab\ub300\ud68c\uac00 \uac1c\ucd5c\ub418\uba70 14~18\uc138\uac00 \ucc38\uac00\ud558\ub294 \ub300\ud68c\uc774\ub2e4. \uccad\uc18c\ub144 \uc62c\ub9bc\ud53d\uc740 IOC\uc704\uc6d0\uc7a5\uc778 \uc790\ud06c \ub85c\uac8c\uac00 2001\ub144\uc5d0 \uace0\uc548\ud55c \uac83\uc73c\ub85c\uc368 2007\ub144 \uc81c119\ucc28 IOC\ucd1d\ud68c\ub54c \uc2b9\uc778\ub418\uc5c8\ub2e4. \uc81c1\ud68c \ud558\uacc4 \uccad\uc18c\ub144 \uc62c\ub9bc\ud53d\uc740 2010\ub144 \uc2f1\uac00\ud3ec\ub974\uc5d0\uc11c \uac1c\ucd5c\ub418\uc5c8\uc73c\uba70 \uc81c1\ud68c \ub3d9\uacc4 \uccad\uc18c\ub144 \uc62c\ub9bc\ud53d\uc740 2\ub144\ud6c4\uc778 2012\ub144 \uc624\uc2a4\ud2b8\ub9ac\uc544\uc758 \uc778\uc2a4\ube0c\ub8e8\ud06c\uc5d0\uc11c \uc5f4\ub838\ub2e4. \uc774 \uc62c\ub9bc\ud53d\uc740 \uc131\uc778 \uc62c\ub9bc\ud53d\ubcf4\ub2e4 \uc9e7\uc740 \uae30\uac04 \ub0b4\uc5d0 \uc9c4\ud589\ub418\uba70 \ud558\uacc4 \uc62c\ub9bc\ud53d\uc740 12\uc77c, \ub3d9\uacc4 \uc62c\ub9bc\ud53d\uc740 9\uc77c\uac04 \uc5f4\ub9b0\ub2e4. IOC\ub294 \ud558\uacc4 \uc720\uc2a4 \uc62c\ub9bc\ud53d\uc5d0 3,500 \uba85\uc758 \uc120\uc218\ub2e8\uacfc 875 \uba85\uc758 \uc784\uc9c1\uc6d0, \ub3d9\uacc4 \uc720\uc2a4 \uc62c\ub9bc\ud53d\uc5d0\ub294 970 \uba85\uc758 \uc120\uc218\ub2e8\uacfc 580 \uba85\uc758 \uc784\uc9c1\uc6d0\uc774 \ucc38\uc5ec\ud55c\ub2e4\uace0 \ub9d0\ud588\ub2e4. \uacbd\uae30\uc885\ubaa9\uc740 \uc131\uc778 \uc62c\ub9bc\ud53d\uacfc \ub3d9\uc77c\ud558\ub098 \uc138\ubd80\uc885\ubaa9\uc5d0 \uc788\uc5b4\uc11c\ub294 \uc81c\uc678\ub418\ub294 \uc885\ubaa9\uc774 \uc788\ub2e4.", "sentence_answer": " 2010\ub144 \uc5d0 \uccab\ub300\ud68c\uac00 \uac1c\ucd5c\ub418\uba70 14~18\uc138\uac00 \ucc38\uac00\ud558\ub294 \ub300\ud68c\uc774\ub2e4.", "paragraph_id": "6588307-10-0", "paragraph_question": "question: \uccad\uc18c\ub144 \uc62c\ub9bc\ud53d\uc774 \ucc98\uc74c \uac1c\ucd5c\ub41c \uac83\uc740 \uc5b8\uc81c\uc778\uac00?, context: 2010\ub144\uc5d0 \uccab\ub300\ud68c\uac00 \uac1c\ucd5c\ub418\uba70 14~18\uc138\uac00 \ucc38\uac00\ud558\ub294 \ub300\ud68c\uc774\ub2e4. \uccad\uc18c\ub144 \uc62c\ub9bc\ud53d\uc740 IOC\uc704\uc6d0\uc7a5\uc778 \uc790\ud06c \ub85c\uac8c\uac00 2001\ub144\uc5d0 \uace0\uc548\ud55c \uac83\uc73c\ub85c\uc368 2007\ub144 \uc81c119\ucc28 IOC\ucd1d\ud68c\ub54c \uc2b9\uc778\ub418\uc5c8\ub2e4. \uc81c1\ud68c \ud558\uacc4 \uccad\uc18c\ub144 \uc62c\ub9bc\ud53d\uc740 2010\ub144 \uc2f1\uac00\ud3ec\ub974\uc5d0\uc11c \uac1c\ucd5c\ub418\uc5c8\uc73c\uba70 \uc81c1\ud68c \ub3d9\uacc4 \uccad\uc18c\ub144 \uc62c\ub9bc\ud53d\uc740 2\ub144\ud6c4\uc778 2012\ub144 \uc624\uc2a4\ud2b8\ub9ac\uc544\uc758 \uc778\uc2a4\ube0c\ub8e8\ud06c\uc5d0\uc11c \uc5f4\ub838\ub2e4. \uc774 \uc62c\ub9bc\ud53d\uc740 \uc131\uc778 \uc62c\ub9bc\ud53d\ubcf4\ub2e4 \uc9e7\uc740 \uae30\uac04 \ub0b4\uc5d0 \uc9c4\ud589\ub418\uba70 \ud558\uacc4 \uc62c\ub9bc\ud53d\uc740 12\uc77c, \ub3d9\uacc4 \uc62c\ub9bc\ud53d\uc740 9\uc77c\uac04 \uc5f4\ub9b0\ub2e4. IOC\ub294 \ud558\uacc4 \uc720\uc2a4 \uc62c\ub9bc\ud53d\uc5d0 3,500 \uba85\uc758 \uc120\uc218\ub2e8\uacfc 875 \uba85\uc758 \uc784\uc9c1\uc6d0, \ub3d9\uacc4 \uc720\uc2a4 \uc62c\ub9bc\ud53d\uc5d0\ub294 970 \uba85\uc758 \uc120\uc218\ub2e8\uacfc 580 \uba85\uc758 \uc784\uc9c1\uc6d0\uc774 \ucc38\uc5ec\ud55c\ub2e4\uace0 \ub9d0\ud588\ub2e4. \uacbd\uae30\uc885\ubaa9\uc740 \uc131\uc778 \uc62c\ub9bc\ud53d\uacfc \ub3d9\uc77c\ud558\ub098 \uc138\ubd80\uc885\ubaa9\uc5d0 \uc788\uc5b4\uc11c\ub294 \uc81c\uc678\ub418\ub294 \uc885\ubaa9\uc774 \uc788\ub2e4."} {"question": "\uc625\ud0c0\ube44\uc544\ub204\uc2a4\uac00 \ucd94\ubc29 \uba85\ub839\ubc95 \uc81c\uc815\uc744 \ubc18\ub300\ud558\uba74\uc11c\uae4c\uc9c0 \uc9c0\ud0a4\uace0 \uc2f6\uc5c8\ub358 \uc0ac\ub78c\uc740 \ub204\uad6c\uc778\uac00?", "paragraph": "\uae30\uc6d0\uc804 43\ub144 10\uc6d4, \ubcfc\ub85c\ub0d0 \uadfc\ucc98\uc5d0\uc11c \uc5f4\ub9b0 \ud68c\ub2f4\uc5d0\uc11c \uc625\ud0c0\ube44\uc544\ub204\uc2a4, \uc548\ud1a0\ub2c8\uc6b0\uc2a4, \ub808\ud53c\ub450\uc2a4\ub294 \uad70\uc0ac \ub3c5\uc7ac \uccb4\uc81c\uc778 \uc81c2\ucc28 \uc0bc\ub450 \uc815\uce58\ub97c \uacb0\uc131\ud558\uc600\ub2e4. \uc774 5\ub144\uac04\uc758 \uba85\ubc31\ud55c \uc6d4\uad8c \ud589\uc704\ub294 \ube44\uacf5\uc2dd\uc801\uc774\uc5c8\ub358 \uadf8\ub098\uc774\uc6b0\uc2a4 \ud3fc\ud398\uc774\uc6b0\uc2a4 \ub9c8\uadf8\ub204\uc2a4, \uc728\ub9ac\uc6b0\uc2a4 \uce74\uc774\uc0ac\ub974, \ub9c8\ub974\ucfe0\uc2a4 \ub9ac\ud0a4\ub2c8\uc6b0\uc2a4 \ud06c\ub77c\uc218\uc2a4\uc758 \uc81c1\ucc28 \uc0bc\ub450 \uc815\uce58\uc640 \ub2ec\ub9ac \ud3c9\ubbfc\ub4e4\uc5d0\uac8c \ud1b5\uacfc\ub41c \ubc95\uc548\uc5d0 \ub530\ub77c \ud06c\uac8c \uc9c0\uc9c0\ubc1b\uc558\ub2e4. 3\uba85\uc758 \uc9d1\uc815\uad00\uc740 300\uba85\uc758 \uc6d0\ub85c\uc6d0 \uc758\uc6d0\uacfc 2,000\uba85\uc758 \uae30\uc0ac \uacc4\uae09 \ucd9c\uc2e0\uc744 \ubc94\ubc95\uc790\ub85c \uaddc\uc815\ud558\uace0 \ucd94\ubc29 \uba85\ub839\uc744 \ub0b4\ub838\uc73c\uba70 \uadf8\ub4e4\uc758 \uc7ac\uc0b0\uc744 \ubab0\uc218\ud558\uace0 \ub9cc\uc57d \uadf8\ub4e4\uc774 \uad6d\uc678 \ud0c8\ucd9c\uc744 \uae30\ub3c4\ud558\uba74 \uadf8\ub4e4\uc744 \uc7a1\uc544 \ucc98\ud615\ud558\uc600\ub2e4.\uc138 \uc9d1\uc815\uad00\ub4e4\uc740 \uce74\uc774\uc0ac\ub974\uc758 \uc554\uc0b4\uc790\uc778 \ub9c8\ub974\ucfe0\uc2a4 \ube0c\ub8e8\ud22c\uc2a4, \uce74\uc2dc\uc6b0\uc2a4 \ub871\uae30\ub204\uc2a4\uc640 \uace7 \uc2f8\uc6b0\uae30 \uc704\ud574 \uc790\uc2e0\ub4e4\uc758 \ubcd1\uc0ac\ub4e4\uc758 \uc6d4\uae09\uc744 \uc62c\ub824\uc8fc\uc5b4\uc57c \ud588\uae30 \ub54c\ubb38\uc5d0 \uc774\ub4e4\uc740 \uc774\uc5d0 \ud06c\uac8c \uc790\uadf9\ubc1b\uc544 \uc774 \ubc95\uc548\uc744 \ub9cc\ub4e4\uc5c8\ub2e4. 3\uba85\uc758 \uc9d1\uc815\uad00\uc774 \uc554\uc0b4\uc790\ub4e4\uc758 \uc720\uc0b0\uacfc \uc7ac\uc0b0\uc744 \ubab0\uc218\ud558\ub294 \ub3d9\uc548 \uc554\uc0b4\uc790\ub4e4\uc5d0\uac8c \ub0b4\uac74 \ud604\uc0c1\uae08\uc740 \ub85c\ub9c8 \uc2dc\ubbfc\uc744 \ud06c\uac8c \uace0\ubb34\uc2dc\ucf30\ub2e4. \uc138 \uc9d1\uc815\uad00\uc758 \uc774\ub7ec\ud55c \uc218\ub2e8\uc740 \uc554\uc0b4\uc790\ub4e4\uacfc \uce5c\ubd84\uc774 \uc788\ub294 \uc0ac\ub78c\ubfd0\ub9cc \uc544\ub2c8\ub77c \ub354 \ub9ce\uc740 \uc0ac\ub78c\uc744 \uc219\uccad\ud558\uc600\ub2e4. \uc625\ud0c0\ube44\uc544\ub204\uc2a4\ub294 \ucc98\uc74c\uc5d0\ub294 \ucd94\ubc29 \uba85\ub839\ubc95\uc744 \uc81c\uc815\ud558\ub294 \uac83\uc5d0 \ubc18\ub300\ud558\uc600\ub294\ub370 \ucd94\ubc29 \uba85\ub839\uc744 \ubc1b\uc740 \uc790\uc2e0\uc758 \uc0c8 \uc870\ub825\uc790, \ud0a4\ucf00\ub85c\uc758 \ubaa9\uc228\uc744 \uc9c0\ud0a4\uace0 \uc2f6\uc5c8\uae30 \ub54c\ubb38\uc774\uc5c8\ub2e4. \uadf8\ub7ec\ub098 \uc548\ud1a0\ub2c8\uc6b0\uc2a4\uc758 \ud0a4\ucf00\ub85c\uc5d0 \ub300\ud55c \uc99d\uc624\ub294 \ub9e4\uc6b0 \ucee4\uc11c \uacb0\uad6d \ud0a4\ucf00\ub85c\ub3c4 \uc219\uccad\uc758 \ud76c\uc0dd\uc790\uac00 \ub418\uc5c8\ub2e4. \ub9ce\uc740 \uc218\uc758 \uacf5\ud654\ud30c \uc6d0\ub85c\uc6d0 \uc758\uc6d0\uc774 \uc8fd\uc5c8\uace0, 3\uba85\uc758 \uc9d1\uc815\uad00\uc740 \uc790\uc2e0\uc758 \uc9c0\uc9c0\uc790\ub4e4\ub85c \uc6d0\ub85c\uc6d0\uc758 \ube48\uc790\ub9ac\ub97c \ucc44\uc6e0\ub2e4. 20\uc138\uae30\uc758 \uc5ed\uc0ac\ud559\uc790 \ub85c\ub110\ub4dc \uc0ac\uc784\uc740 \uc774\ub97c \ub85c\ub9c8 \ud601\uba85(Roman revolution)\uc774\ub77c \uba85\uba85\ud558\uc600\ub294\ub370, \uc774 \uc0ac\uac74\uc774 \uad6c\uc138\ub300 \uc6d0\ub85c\uc6d0 \uc758\uc6d0\ub4e4\uc744 \ubab0\uc544\ub0b4\uace0 \uc0c8\ub85c\uc6b4 \uc815\uce58 \uc138\ub825\uc744 \uad6c\ucd95\ud558\uc5ec \ub4b7\ub0a0 \uc544\uc6b0\uad6c\uc2a4\ud22c\uc2a4 \uc2dc\ub300\uc758 \uc6d0\ub3d9\ub825\uc774 \ub418\uc5c8\uae30 \ub54c\ubb38\uc774\ub2e4.", "answer": "\ud0a4\ucf00\ub85c", "sentence": "\uc625\ud0c0\ube44\uc544\ub204\uc2a4\ub294 \ucc98\uc74c\uc5d0\ub294 \ucd94\ubc29 \uba85\ub839\ubc95\uc744 \uc81c\uc815\ud558\ub294 \uac83\uc5d0 \ubc18\ub300\ud558\uc600\ub294\ub370 \ucd94\ubc29 \uba85\ub839\uc744 \ubc1b\uc740 \uc790\uc2e0\uc758 \uc0c8 \uc870\ub825\uc790, \ud0a4\ucf00\ub85c \uc758 \ubaa9\uc228\uc744 \uc9c0\ud0a4\uace0 \uc2f6\uc5c8\uae30 \ub54c\ubb38\uc774\uc5c8\ub2e4.", "paragraph_sentence": "\uae30\uc6d0\uc804 43\ub144 10\uc6d4, \ubcfc\ub85c\ub0d0 \uadfc\ucc98\uc5d0\uc11c \uc5f4\ub9b0 \ud68c\ub2f4\uc5d0\uc11c \uc625\ud0c0\ube44\uc544\ub204\uc2a4, \uc548\ud1a0\ub2c8\uc6b0\uc2a4, \ub808\ud53c\ub450\uc2a4\ub294 \uad70\uc0ac \ub3c5\uc7ac \uccb4\uc81c\uc778 \uc81c2\ucc28 \uc0bc\ub450 \uc815\uce58\ub97c \uacb0\uc131\ud558\uc600\ub2e4. \uc774 5\ub144\uac04\uc758 \uba85\ubc31\ud55c \uc6d4\uad8c \ud589\uc704\ub294 \ube44\uacf5\uc2dd\uc801\uc774\uc5c8\ub358 \uadf8\ub098\uc774\uc6b0\uc2a4 \ud3fc\ud398\uc774\uc6b0\uc2a4 \ub9c8\uadf8\ub204\uc2a4, \uc728\ub9ac\uc6b0\uc2a4 \uce74\uc774\uc0ac\ub974, \ub9c8\ub974\ucfe0\uc2a4 \ub9ac\ud0a4\ub2c8\uc6b0\uc2a4 \ud06c\ub77c\uc218\uc2a4\uc758 \uc81c1\ucc28 \uc0bc\ub450 \uc815\uce58\uc640 \ub2ec\ub9ac \ud3c9\ubbfc\ub4e4\uc5d0\uac8c \ud1b5\uacfc\ub41c \ubc95\uc548\uc5d0 \ub530\ub77c \ud06c\uac8c \uc9c0\uc9c0\ubc1b\uc558\ub2e4. 3\uba85\uc758 \uc9d1\uc815\uad00\uc740 300\uba85\uc758 \uc6d0\ub85c\uc6d0 \uc758\uc6d0\uacfc 2,000\uba85\uc758 \uae30\uc0ac \uacc4\uae09 \ucd9c\uc2e0\uc744 \ubc94\ubc95\uc790\ub85c \uaddc\uc815\ud558\uace0 \ucd94\ubc29 \uba85\ub839\uc744 \ub0b4\ub838\uc73c\uba70 \uadf8\ub4e4\uc758 \uc7ac\uc0b0\uc744 \ubab0\uc218\ud558\uace0 \ub9cc\uc57d \uadf8\ub4e4\uc774 \uad6d\uc678 \ud0c8\ucd9c\uc744 \uae30\ub3c4\ud558\uba74 \uadf8\ub4e4\uc744 \uc7a1\uc544 \ucc98\ud615\ud558\uc600\ub2e4. \uc138 \uc9d1\uc815\uad00\ub4e4\uc740 \uce74\uc774\uc0ac\ub974\uc758 \uc554\uc0b4\uc790\uc778 \ub9c8\ub974\ucfe0\uc2a4 \ube0c\ub8e8\ud22c\uc2a4, \uce74\uc2dc\uc6b0\uc2a4 \ub871\uae30\ub204\uc2a4\uc640 \uace7 \uc2f8\uc6b0\uae30 \uc704\ud574 \uc790\uc2e0\ub4e4\uc758 \ubcd1\uc0ac\ub4e4\uc758 \uc6d4\uae09\uc744 \uc62c\ub824\uc8fc\uc5b4\uc57c \ud588\uae30 \ub54c\ubb38\uc5d0 \uc774\ub4e4\uc740 \uc774\uc5d0 \ud06c\uac8c \uc790\uadf9\ubc1b\uc544 \uc774 \ubc95\uc548\uc744 \ub9cc\ub4e4\uc5c8\ub2e4. 3\uba85\uc758 \uc9d1\uc815\uad00\uc774 \uc554\uc0b4\uc790\ub4e4\uc758 \uc720\uc0b0\uacfc \uc7ac\uc0b0\uc744 \ubab0\uc218\ud558\ub294 \ub3d9\uc548 \uc554\uc0b4\uc790\ub4e4\uc5d0\uac8c \ub0b4\uac74 \ud604\uc0c1\uae08\uc740 \ub85c\ub9c8 \uc2dc\ubbfc\uc744 \ud06c\uac8c \uace0\ubb34\uc2dc\ucf30\ub2e4. \uc138 \uc9d1\uc815\uad00\uc758 \uc774\ub7ec\ud55c \uc218\ub2e8\uc740 \uc554\uc0b4\uc790\ub4e4\uacfc \uce5c\ubd84\uc774 \uc788\ub294 \uc0ac\ub78c\ubfd0\ub9cc \uc544\ub2c8\ub77c \ub354 \ub9ce\uc740 \uc0ac\ub78c\uc744 \uc219\uccad\ud558\uc600\ub2e4. \uc625\ud0c0\ube44\uc544\ub204\uc2a4\ub294 \ucc98\uc74c\uc5d0\ub294 \ucd94\ubc29 \uba85\ub839\ubc95\uc744 \uc81c\uc815\ud558\ub294 \uac83\uc5d0 \ubc18\ub300\ud558\uc600\ub294\ub370 \ucd94\ubc29 \uba85\ub839\uc744 \ubc1b\uc740 \uc790\uc2e0\uc758 \uc0c8 \uc870\ub825\uc790, \ud0a4\ucf00\ub85c \uc758 \ubaa9\uc228\uc744 \uc9c0\ud0a4\uace0 \uc2f6\uc5c8\uae30 \ub54c\ubb38\uc774\uc5c8\ub2e4. \uadf8\ub7ec\ub098 \uc548\ud1a0\ub2c8\uc6b0\uc2a4\uc758 \ud0a4\ucf00\ub85c\uc5d0 \ub300\ud55c \uc99d\uc624\ub294 \ub9e4\uc6b0 \ucee4\uc11c \uacb0\uad6d \ud0a4\ucf00\ub85c\ub3c4 \uc219\uccad\uc758 \ud76c\uc0dd\uc790\uac00 \ub418\uc5c8\ub2e4. \ub9ce\uc740 \uc218\uc758 \uacf5\ud654\ud30c \uc6d0\ub85c\uc6d0 \uc758\uc6d0\uc774 \uc8fd\uc5c8\uace0, 3\uba85\uc758 \uc9d1\uc815\uad00\uc740 \uc790\uc2e0\uc758 \uc9c0\uc9c0\uc790\ub4e4\ub85c \uc6d0\ub85c\uc6d0\uc758 \ube48\uc790\ub9ac\ub97c \ucc44\uc6e0\ub2e4. 20\uc138\uae30\uc758 \uc5ed\uc0ac\ud559\uc790 \ub85c\ub110\ub4dc \uc0ac\uc784\uc740 \uc774\ub97c \ub85c\ub9c8 \ud601\uba85(Roman revolution)\uc774\ub77c \uba85\uba85\ud558\uc600\ub294\ub370, \uc774 \uc0ac\uac74\uc774 \uad6c\uc138\ub300 \uc6d0\ub85c\uc6d0 \uc758\uc6d0\ub4e4\uc744 \ubab0\uc544\ub0b4\uace0 \uc0c8\ub85c\uc6b4 \uc815\uce58 \uc138\ub825\uc744 \uad6c\ucd95\ud558\uc5ec \ub4b7\ub0a0 \uc544\uc6b0\uad6c\uc2a4\ud22c\uc2a4 \uc2dc\ub300\uc758 \uc6d0\ub3d9\ub825\uc774 \ub418\uc5c8\uae30 \ub54c\ubb38\uc774\ub2e4.", "paragraph_answer": "\uae30\uc6d0\uc804 43\ub144 10\uc6d4, \ubcfc\ub85c\ub0d0 \uadfc\ucc98\uc5d0\uc11c \uc5f4\ub9b0 \ud68c\ub2f4\uc5d0\uc11c \uc625\ud0c0\ube44\uc544\ub204\uc2a4, \uc548\ud1a0\ub2c8\uc6b0\uc2a4, \ub808\ud53c\ub450\uc2a4\ub294 \uad70\uc0ac \ub3c5\uc7ac \uccb4\uc81c\uc778 \uc81c2\ucc28 \uc0bc\ub450 \uc815\uce58\ub97c \uacb0\uc131\ud558\uc600\ub2e4. \uc774 5\ub144\uac04\uc758 \uba85\ubc31\ud55c \uc6d4\uad8c \ud589\uc704\ub294 \ube44\uacf5\uc2dd\uc801\uc774\uc5c8\ub358 \uadf8\ub098\uc774\uc6b0\uc2a4 \ud3fc\ud398\uc774\uc6b0\uc2a4 \ub9c8\uadf8\ub204\uc2a4, \uc728\ub9ac\uc6b0\uc2a4 \uce74\uc774\uc0ac\ub974, \ub9c8\ub974\ucfe0\uc2a4 \ub9ac\ud0a4\ub2c8\uc6b0\uc2a4 \ud06c\ub77c\uc218\uc2a4\uc758 \uc81c1\ucc28 \uc0bc\ub450 \uc815\uce58\uc640 \ub2ec\ub9ac \ud3c9\ubbfc\ub4e4\uc5d0\uac8c \ud1b5\uacfc\ub41c \ubc95\uc548\uc5d0 \ub530\ub77c \ud06c\uac8c \uc9c0\uc9c0\ubc1b\uc558\ub2e4. 3\uba85\uc758 \uc9d1\uc815\uad00\uc740 300\uba85\uc758 \uc6d0\ub85c\uc6d0 \uc758\uc6d0\uacfc 2,000\uba85\uc758 \uae30\uc0ac \uacc4\uae09 \ucd9c\uc2e0\uc744 \ubc94\ubc95\uc790\ub85c \uaddc\uc815\ud558\uace0 \ucd94\ubc29 \uba85\ub839\uc744 \ub0b4\ub838\uc73c\uba70 \uadf8\ub4e4\uc758 \uc7ac\uc0b0\uc744 \ubab0\uc218\ud558\uace0 \ub9cc\uc57d \uadf8\ub4e4\uc774 \uad6d\uc678 \ud0c8\ucd9c\uc744 \uae30\ub3c4\ud558\uba74 \uadf8\ub4e4\uc744 \uc7a1\uc544 \ucc98\ud615\ud558\uc600\ub2e4.\uc138 \uc9d1\uc815\uad00\ub4e4\uc740 \uce74\uc774\uc0ac\ub974\uc758 \uc554\uc0b4\uc790\uc778 \ub9c8\ub974\ucfe0\uc2a4 \ube0c\ub8e8\ud22c\uc2a4, \uce74\uc2dc\uc6b0\uc2a4 \ub871\uae30\ub204\uc2a4\uc640 \uace7 \uc2f8\uc6b0\uae30 \uc704\ud574 \uc790\uc2e0\ub4e4\uc758 \ubcd1\uc0ac\ub4e4\uc758 \uc6d4\uae09\uc744 \uc62c\ub824\uc8fc\uc5b4\uc57c \ud588\uae30 \ub54c\ubb38\uc5d0 \uc774\ub4e4\uc740 \uc774\uc5d0 \ud06c\uac8c \uc790\uadf9\ubc1b\uc544 \uc774 \ubc95\uc548\uc744 \ub9cc\ub4e4\uc5c8\ub2e4. 3\uba85\uc758 \uc9d1\uc815\uad00\uc774 \uc554\uc0b4\uc790\ub4e4\uc758 \uc720\uc0b0\uacfc \uc7ac\uc0b0\uc744 \ubab0\uc218\ud558\ub294 \ub3d9\uc548 \uc554\uc0b4\uc790\ub4e4\uc5d0\uac8c \ub0b4\uac74 \ud604\uc0c1\uae08\uc740 \ub85c\ub9c8 \uc2dc\ubbfc\uc744 \ud06c\uac8c \uace0\ubb34\uc2dc\ucf30\ub2e4. \uc138 \uc9d1\uc815\uad00\uc758 \uc774\ub7ec\ud55c \uc218\ub2e8\uc740 \uc554\uc0b4\uc790\ub4e4\uacfc \uce5c\ubd84\uc774 \uc788\ub294 \uc0ac\ub78c\ubfd0\ub9cc \uc544\ub2c8\ub77c \ub354 \ub9ce\uc740 \uc0ac\ub78c\uc744 \uc219\uccad\ud558\uc600\ub2e4. \uc625\ud0c0\ube44\uc544\ub204\uc2a4\ub294 \ucc98\uc74c\uc5d0\ub294 \ucd94\ubc29 \uba85\ub839\ubc95\uc744 \uc81c\uc815\ud558\ub294 \uac83\uc5d0 \ubc18\ub300\ud558\uc600\ub294\ub370 \ucd94\ubc29 \uba85\ub839\uc744 \ubc1b\uc740 \uc790\uc2e0\uc758 \uc0c8 \uc870\ub825\uc790, \ud0a4\ucf00\ub85c \uc758 \ubaa9\uc228\uc744 \uc9c0\ud0a4\uace0 \uc2f6\uc5c8\uae30 \ub54c\ubb38\uc774\uc5c8\ub2e4. \uadf8\ub7ec\ub098 \uc548\ud1a0\ub2c8\uc6b0\uc2a4\uc758 \ud0a4\ucf00\ub85c\uc5d0 \ub300\ud55c \uc99d\uc624\ub294 \ub9e4\uc6b0 \ucee4\uc11c \uacb0\uad6d \ud0a4\ucf00\ub85c\ub3c4 \uc219\uccad\uc758 \ud76c\uc0dd\uc790\uac00 \ub418\uc5c8\ub2e4. \ub9ce\uc740 \uc218\uc758 \uacf5\ud654\ud30c \uc6d0\ub85c\uc6d0 \uc758\uc6d0\uc774 \uc8fd\uc5c8\uace0, 3\uba85\uc758 \uc9d1\uc815\uad00\uc740 \uc790\uc2e0\uc758 \uc9c0\uc9c0\uc790\ub4e4\ub85c \uc6d0\ub85c\uc6d0\uc758 \ube48\uc790\ub9ac\ub97c \ucc44\uc6e0\ub2e4. 20\uc138\uae30\uc758 \uc5ed\uc0ac\ud559\uc790 \ub85c\ub110\ub4dc \uc0ac\uc784\uc740 \uc774\ub97c \ub85c\ub9c8 \ud601\uba85(Roman revolution)\uc774\ub77c \uba85\uba85\ud558\uc600\ub294\ub370, \uc774 \uc0ac\uac74\uc774 \uad6c\uc138\ub300 \uc6d0\ub85c\uc6d0 \uc758\uc6d0\ub4e4\uc744 \ubab0\uc544\ub0b4\uace0 \uc0c8\ub85c\uc6b4 \uc815\uce58 \uc138\ub825\uc744 \uad6c\ucd95\ud558\uc5ec \ub4b7\ub0a0 \uc544\uc6b0\uad6c\uc2a4\ud22c\uc2a4 \uc2dc\ub300\uc758 \uc6d0\ub3d9\ub825\uc774 \ub418\uc5c8\uae30 \ub54c\ubb38\uc774\ub2e4.", "sentence_answer": "\uc625\ud0c0\ube44\uc544\ub204\uc2a4\ub294 \ucc98\uc74c\uc5d0\ub294 \ucd94\ubc29 \uba85\ub839\ubc95\uc744 \uc81c\uc815\ud558\ub294 \uac83\uc5d0 \ubc18\ub300\ud558\uc600\ub294\ub370 \ucd94\ubc29 \uba85\ub839\uc744 \ubc1b\uc740 \uc790\uc2e0\uc758 \uc0c8 \uc870\ub825\uc790, \ud0a4\ucf00\ub85c \uc758 \ubaa9\uc228\uc744 \uc9c0\ud0a4\uace0 \uc2f6\uc5c8\uae30 \ub54c\ubb38\uc774\uc5c8\ub2e4.", "paragraph_id": "6657258-9-1", "paragraph_question": "question: \uc625\ud0c0\ube44\uc544\ub204\uc2a4\uac00 \ucd94\ubc29 \uba85\ub839\ubc95 \uc81c\uc815\uc744 \ubc18\ub300\ud558\uba74\uc11c\uae4c\uc9c0 \uc9c0\ud0a4\uace0 \uc2f6\uc5c8\ub358 \uc0ac\ub78c\uc740 \ub204\uad6c\uc778\uac00?, context: \uae30\uc6d0\uc804 43\ub144 10\uc6d4, \ubcfc\ub85c\ub0d0 \uadfc\ucc98\uc5d0\uc11c \uc5f4\ub9b0 \ud68c\ub2f4\uc5d0\uc11c \uc625\ud0c0\ube44\uc544\ub204\uc2a4, \uc548\ud1a0\ub2c8\uc6b0\uc2a4, \ub808\ud53c\ub450\uc2a4\ub294 \uad70\uc0ac \ub3c5\uc7ac \uccb4\uc81c\uc778 \uc81c2\ucc28 \uc0bc\ub450 \uc815\uce58\ub97c \uacb0\uc131\ud558\uc600\ub2e4. \uc774 5\ub144\uac04\uc758 \uba85\ubc31\ud55c \uc6d4\uad8c \ud589\uc704\ub294 \ube44\uacf5\uc2dd\uc801\uc774\uc5c8\ub358 \uadf8\ub098\uc774\uc6b0\uc2a4 \ud3fc\ud398\uc774\uc6b0\uc2a4 \ub9c8\uadf8\ub204\uc2a4, \uc728\ub9ac\uc6b0\uc2a4 \uce74\uc774\uc0ac\ub974, \ub9c8\ub974\ucfe0\uc2a4 \ub9ac\ud0a4\ub2c8\uc6b0\uc2a4 \ud06c\ub77c\uc218\uc2a4\uc758 \uc81c1\ucc28 \uc0bc\ub450 \uc815\uce58\uc640 \ub2ec\ub9ac \ud3c9\ubbfc\ub4e4\uc5d0\uac8c \ud1b5\uacfc\ub41c \ubc95\uc548\uc5d0 \ub530\ub77c \ud06c\uac8c \uc9c0\uc9c0\ubc1b\uc558\ub2e4. 3\uba85\uc758 \uc9d1\uc815\uad00\uc740 300\uba85\uc758 \uc6d0\ub85c\uc6d0 \uc758\uc6d0\uacfc 2,000\uba85\uc758 \uae30\uc0ac \uacc4\uae09 \ucd9c\uc2e0\uc744 \ubc94\ubc95\uc790\ub85c \uaddc\uc815\ud558\uace0 \ucd94\ubc29 \uba85\ub839\uc744 \ub0b4\ub838\uc73c\uba70 \uadf8\ub4e4\uc758 \uc7ac\uc0b0\uc744 \ubab0\uc218\ud558\uace0 \ub9cc\uc57d \uadf8\ub4e4\uc774 \uad6d\uc678 \ud0c8\ucd9c\uc744 \uae30\ub3c4\ud558\uba74 \uadf8\ub4e4\uc744 \uc7a1\uc544 \ucc98\ud615\ud558\uc600\ub2e4.\uc138 \uc9d1\uc815\uad00\ub4e4\uc740 \uce74\uc774\uc0ac\ub974\uc758 \uc554\uc0b4\uc790\uc778 \ub9c8\ub974\ucfe0\uc2a4 \ube0c\ub8e8\ud22c\uc2a4, \uce74\uc2dc\uc6b0\uc2a4 \ub871\uae30\ub204\uc2a4\uc640 \uace7 \uc2f8\uc6b0\uae30 \uc704\ud574 \uc790\uc2e0\ub4e4\uc758 \ubcd1\uc0ac\ub4e4\uc758 \uc6d4\uae09\uc744 \uc62c\ub824\uc8fc\uc5b4\uc57c \ud588\uae30 \ub54c\ubb38\uc5d0 \uc774\ub4e4\uc740 \uc774\uc5d0 \ud06c\uac8c \uc790\uadf9\ubc1b\uc544 \uc774 \ubc95\uc548\uc744 \ub9cc\ub4e4\uc5c8\ub2e4. 3\uba85\uc758 \uc9d1\uc815\uad00\uc774 \uc554\uc0b4\uc790\ub4e4\uc758 \uc720\uc0b0\uacfc \uc7ac\uc0b0\uc744 \ubab0\uc218\ud558\ub294 \ub3d9\uc548 \uc554\uc0b4\uc790\ub4e4\uc5d0\uac8c \ub0b4\uac74 \ud604\uc0c1\uae08\uc740 \ub85c\ub9c8 \uc2dc\ubbfc\uc744 \ud06c\uac8c \uace0\ubb34\uc2dc\ucf30\ub2e4. \uc138 \uc9d1\uc815\uad00\uc758 \uc774\ub7ec\ud55c \uc218\ub2e8\uc740 \uc554\uc0b4\uc790\ub4e4\uacfc \uce5c\ubd84\uc774 \uc788\ub294 \uc0ac\ub78c\ubfd0\ub9cc \uc544\ub2c8\ub77c \ub354 \ub9ce\uc740 \uc0ac\ub78c\uc744 \uc219\uccad\ud558\uc600\ub2e4. \uc625\ud0c0\ube44\uc544\ub204\uc2a4\ub294 \ucc98\uc74c\uc5d0\ub294 \ucd94\ubc29 \uba85\ub839\ubc95\uc744 \uc81c\uc815\ud558\ub294 \uac83\uc5d0 \ubc18\ub300\ud558\uc600\ub294\ub370 \ucd94\ubc29 \uba85\ub839\uc744 \ubc1b\uc740 \uc790\uc2e0\uc758 \uc0c8 \uc870\ub825\uc790, \ud0a4\ucf00\ub85c\uc758 \ubaa9\uc228\uc744 \uc9c0\ud0a4\uace0 \uc2f6\uc5c8\uae30 \ub54c\ubb38\uc774\uc5c8\ub2e4. \uadf8\ub7ec\ub098 \uc548\ud1a0\ub2c8\uc6b0\uc2a4\uc758 \ud0a4\ucf00\ub85c\uc5d0 \ub300\ud55c \uc99d\uc624\ub294 \ub9e4\uc6b0 \ucee4\uc11c \uacb0\uad6d \ud0a4\ucf00\ub85c\ub3c4 \uc219\uccad\uc758 \ud76c\uc0dd\uc790\uac00 \ub418\uc5c8\ub2e4. \ub9ce\uc740 \uc218\uc758 \uacf5\ud654\ud30c \uc6d0\ub85c\uc6d0 \uc758\uc6d0\uc774 \uc8fd\uc5c8\uace0, 3\uba85\uc758 \uc9d1\uc815\uad00\uc740 \uc790\uc2e0\uc758 \uc9c0\uc9c0\uc790\ub4e4\ub85c \uc6d0\ub85c\uc6d0\uc758 \ube48\uc790\ub9ac\ub97c \ucc44\uc6e0\ub2e4. 20\uc138\uae30\uc758 \uc5ed\uc0ac\ud559\uc790 \ub85c\ub110\ub4dc \uc0ac\uc784\uc740 \uc774\ub97c \ub85c\ub9c8 \ud601\uba85(Roman revolution)\uc774\ub77c \uba85\uba85\ud558\uc600\ub294\ub370, \uc774 \uc0ac\uac74\uc774 \uad6c\uc138\ub300 \uc6d0\ub85c\uc6d0 \uc758\uc6d0\ub4e4\uc744 \ubab0\uc544\ub0b4\uace0 \uc0c8\ub85c\uc6b4 \uc815\uce58 \uc138\ub825\uc744 \uad6c\ucd95\ud558\uc5ec \ub4b7\ub0a0 \uc544\uc6b0\uad6c\uc2a4\ud22c\uc2a4 \uc2dc\ub300\uc758 \uc6d0\ub3d9\ub825\uc774 \ub418\uc5c8\uae30 \ub54c\ubb38\uc774\ub2e4."} {"question": "\ubcc4\uc77c \uc5c6\uc774 \uc0b0\ub2e4\uac00 \uccab \uc815\uaddc \uc74c\ubc18\uc774\ub77c\uc11c \ub300\uc2e0 \uacf5\uc7a5\uc5d0\uc11c \uc0dd\uc0b0\ud558\uae30\ub85c \ud558\uace0, \uc0ac\uc6a9\ud558\uc9c0 \uc54a\uc740 \uae30\uc874 \uc0dd\uc0b0\ubc29\uc2dd\uc740 \ubb34\uc5c7\uc785\ub2c8\uae4c?", "paragraph": "\uc7a5\uae30\ud558\ub294 2008\ub144 12\uc6d4 31\uc77c \uc790 \u300a\ud55c\uaca8\ub808\u300b \uae30\uc0ac\uc5d0\uc11c \"2\uc6d4 \ub9d0\uc5d0 \uc7a5\uae30\ud558\uc640 \uc5bc\uad74\ub4e4 \uc815\uaddc \uc74c\ubc18\uc774 \ub098\uc628\ub2e4.\"\uace0 \uc5b8\uae09\ud588\ub2e4. \uc18c\uc18d\uc0ac \ubd95\uac00\ubd95\uac00\ub808\ucf54\ub4dc \uad00\uacc4\uc790\ub294 2009\ub144 2\uc6d4 3\uc77c\uc5d0 \"\uc7a5\uae30\ud558\uc640 \uc5bc\uad74\ub4e4\uc758 \uc815\uaddc 1\uc9d1\uc774 \uc624\ub294 27\uc77c \ubc1c\ub9e4\ub420 \uc608\uc815\"\uc774\ub77c\uba70 \"\ubc1c\ub9e4\uc77c\uc5d0 \ub9de\ucdb0 \ubc1c\ub9e4 \uae30\ub150\uacf5\uc5f0\ub3c4 \ud568\uaed8 \uc5f0\ub2e4\"\uace0 \ubc1c\ub9e4 \uc77c\uc790\ub97c \ud655\uc815\uc9c0\uc5c8\ub2e4. \uc774 \uad00\uacc4\uc790\ub294 \"\ud604\uc7ac \ub9c8\ubb34\ub9ac \uc74c\ubc18 \uc791\uc5c5\uc744 \uc9c4\ud589 \uc911\"\uc774\ub77c\uba70 \"\uc74c\ubc18 \ud0c0\uc774\ud2c0\uacfc \uc218\ub85d\uace1 \uc804\ubd80\uac00 \uacb0\uc815\ub41c \uc0c1\ud0dc\ub294 \uc544\ub2c8\ub2e4\"\ub77c\uace0 \ub367\ubd99\uc600\ub2e4. \uc7a5\uae30\ud558\ub294 \uc608\uc2a424 \uc778\ud130\ubdf0\uc5d0\uc11c \"\uc0c1\uc0c1\ub9c8\ub2f9\uc5d0\uc11c 2\uc6d4 27\uc77c \uacf5\uc5f0 \uacb8 \uc1fc\ucf00\uc774\uc2a4\uac00 \uc7a1\ud600 \uc788\uc5b4\uc11c \uc790\uc5f0\uc2a4\ub7fd\uac8c 1\uc9d1 \ub9c8\uac10 \uc2dc\ud55c\uc774 \uacb0\uc815\ub418\uc5c8\ub2e4.\"\uace0 \ubc1d\ud614\ub2e4. \uccab \uc815\uaddc \uc74c\ubc18\uc778 \ub9cc\ud07c \u300a\ubcc4\uc77c \uc5c6\uc774 \uc0b0\ub2e4\u300b\ub294 \uac00\ub0b4\uc218\uacf5\uc5c5\uc744 \ud1b5\ud55c \uc18c\ub7c9 \uc0dd\uc0b0\ubc29\uc2dd\uc744 \ud0dd\ud558\uc9c0 \uc54a\uace0 \uacf5\uc7a5\uc5d0\uc11c \ucc0d\uc5b4\ub0c8\ub2e4.", "answer": "\uac00\ub0b4\uc218\uacf5\uc5c5\uc744 \ud1b5\ud55c \uc18c\ub7c9 \uc0dd\uc0b0\ubc29\uc2dd", "sentence": "\uccab \uc815\uaddc \uc74c\ubc18\uc778 \ub9cc\ud07c \u300a\ubcc4\uc77c \uc5c6\uc774 \uc0b0\ub2e4\u300b\ub294 \uac00\ub0b4\uc218\uacf5\uc5c5\uc744 \ud1b5\ud55c \uc18c\ub7c9 \uc0dd\uc0b0\ubc29\uc2dd \uc744 \ud0dd\ud558\uc9c0 \uc54a\uace0 \uacf5\uc7a5\uc5d0\uc11c \ucc0d\uc5b4\ub0c8\ub2e4.", "paragraph_sentence": "\uc7a5\uae30\ud558\ub294 2008\ub144 12\uc6d4 31\uc77c \uc790 \u300a\ud55c\uaca8\ub808\u300b \uae30\uc0ac\uc5d0\uc11c \"2\uc6d4 \ub9d0\uc5d0 \uc7a5\uae30\ud558\uc640 \uc5bc\uad74\ub4e4 \uc815\uaddc \uc74c\ubc18\uc774 \ub098\uc628\ub2e4. \"\uace0 \uc5b8\uae09\ud588\ub2e4. \uc18c\uc18d\uc0ac \ubd95\uac00\ubd95\uac00\ub808\ucf54\ub4dc \uad00\uacc4\uc790\ub294 2009\ub144 2\uc6d4 3\uc77c\uc5d0 \"\uc7a5\uae30\ud558\uc640 \uc5bc\uad74\ub4e4\uc758 \uc815\uaddc 1\uc9d1\uc774 \uc624\ub294 27\uc77c \ubc1c\ub9e4\ub420 \uc608\uc815\"\uc774\ub77c\uba70 \"\ubc1c\ub9e4\uc77c\uc5d0 \ub9de\ucdb0 \ubc1c\ub9e4 \uae30\ub150\uacf5\uc5f0\ub3c4 \ud568\uaed8 \uc5f0\ub2e4\"\uace0 \ubc1c\ub9e4 \uc77c\uc790\ub97c \ud655\uc815\uc9c0\uc5c8\ub2e4. \uc774 \uad00\uacc4\uc790\ub294 \"\ud604\uc7ac \ub9c8\ubb34\ub9ac \uc74c\ubc18 \uc791\uc5c5\uc744 \uc9c4\ud589 \uc911\"\uc774\ub77c\uba70 \"\uc74c\ubc18 \ud0c0\uc774\ud2c0\uacfc \uc218\ub85d\uace1 \uc804\ubd80\uac00 \uacb0\uc815\ub41c \uc0c1\ud0dc\ub294 \uc544\ub2c8\ub2e4\"\ub77c\uace0 \ub367\ubd99\uc600\ub2e4. \uc7a5\uae30\ud558\ub294 \uc608\uc2a424 \uc778\ud130\ubdf0\uc5d0\uc11c \"\uc0c1\uc0c1\ub9c8\ub2f9\uc5d0\uc11c 2\uc6d4 27\uc77c \uacf5\uc5f0 \uacb8 \uc1fc\ucf00\uc774\uc2a4\uac00 \uc7a1\ud600 \uc788\uc5b4\uc11c \uc790\uc5f0\uc2a4\ub7fd\uac8c 1\uc9d1 \ub9c8\uac10 \uc2dc\ud55c\uc774 \uacb0\uc815\ub418\uc5c8\ub2e4. \"\uace0 \ubc1d\ud614\ub2e4. \uccab \uc815\uaddc \uc74c\ubc18\uc778 \ub9cc\ud07c \u300a\ubcc4\uc77c \uc5c6\uc774 \uc0b0\ub2e4\u300b\ub294 \uac00\ub0b4\uc218\uacf5\uc5c5\uc744 \ud1b5\ud55c \uc18c\ub7c9 \uc0dd\uc0b0\ubc29\uc2dd \uc744 \ud0dd\ud558\uc9c0 \uc54a\uace0 \uacf5\uc7a5\uc5d0\uc11c \ucc0d\uc5b4\ub0c8\ub2e4. ", "paragraph_answer": "\uc7a5\uae30\ud558\ub294 2008\ub144 12\uc6d4 31\uc77c \uc790 \u300a\ud55c\uaca8\ub808\u300b \uae30\uc0ac\uc5d0\uc11c \"2\uc6d4 \ub9d0\uc5d0 \uc7a5\uae30\ud558\uc640 \uc5bc\uad74\ub4e4 \uc815\uaddc \uc74c\ubc18\uc774 \ub098\uc628\ub2e4.\"\uace0 \uc5b8\uae09\ud588\ub2e4. \uc18c\uc18d\uc0ac \ubd95\uac00\ubd95\uac00\ub808\ucf54\ub4dc \uad00\uacc4\uc790\ub294 2009\ub144 2\uc6d4 3\uc77c\uc5d0 \"\uc7a5\uae30\ud558\uc640 \uc5bc\uad74\ub4e4\uc758 \uc815\uaddc 1\uc9d1\uc774 \uc624\ub294 27\uc77c \ubc1c\ub9e4\ub420 \uc608\uc815\"\uc774\ub77c\uba70 \"\ubc1c\ub9e4\uc77c\uc5d0 \ub9de\ucdb0 \ubc1c\ub9e4 \uae30\ub150\uacf5\uc5f0\ub3c4 \ud568\uaed8 \uc5f0\ub2e4\"\uace0 \ubc1c\ub9e4 \uc77c\uc790\ub97c \ud655\uc815\uc9c0\uc5c8\ub2e4. \uc774 \uad00\uacc4\uc790\ub294 \"\ud604\uc7ac \ub9c8\ubb34\ub9ac \uc74c\ubc18 \uc791\uc5c5\uc744 \uc9c4\ud589 \uc911\"\uc774\ub77c\uba70 \"\uc74c\ubc18 \ud0c0\uc774\ud2c0\uacfc \uc218\ub85d\uace1 \uc804\ubd80\uac00 \uacb0\uc815\ub41c \uc0c1\ud0dc\ub294 \uc544\ub2c8\ub2e4\"\ub77c\uace0 \ub367\ubd99\uc600\ub2e4. \uc7a5\uae30\ud558\ub294 \uc608\uc2a424 \uc778\ud130\ubdf0\uc5d0\uc11c \"\uc0c1\uc0c1\ub9c8\ub2f9\uc5d0\uc11c 2\uc6d4 27\uc77c \uacf5\uc5f0 \uacb8 \uc1fc\ucf00\uc774\uc2a4\uac00 \uc7a1\ud600 \uc788\uc5b4\uc11c \uc790\uc5f0\uc2a4\ub7fd\uac8c 1\uc9d1 \ub9c8\uac10 \uc2dc\ud55c\uc774 \uacb0\uc815\ub418\uc5c8\ub2e4.\"\uace0 \ubc1d\ud614\ub2e4. \uccab \uc815\uaddc \uc74c\ubc18\uc778 \ub9cc\ud07c \u300a\ubcc4\uc77c \uc5c6\uc774 \uc0b0\ub2e4\u300b\ub294 \uac00\ub0b4\uc218\uacf5\uc5c5\uc744 \ud1b5\ud55c \uc18c\ub7c9 \uc0dd\uc0b0\ubc29\uc2dd \uc744 \ud0dd\ud558\uc9c0 \uc54a\uace0 \uacf5\uc7a5\uc5d0\uc11c \ucc0d\uc5b4\ub0c8\ub2e4.", "sentence_answer": "\uccab \uc815\uaddc \uc74c\ubc18\uc778 \ub9cc\ud07c \u300a\ubcc4\uc77c \uc5c6\uc774 \uc0b0\ub2e4\u300b\ub294 \uac00\ub0b4\uc218\uacf5\uc5c5\uc744 \ud1b5\ud55c \uc18c\ub7c9 \uc0dd\uc0b0\ubc29\uc2dd \uc744 \ud0dd\ud558\uc9c0 \uc54a\uace0 \uacf5\uc7a5\uc5d0\uc11c \ucc0d\uc5b4\ub0c8\ub2e4.", "paragraph_id": "6508646-1-2", "paragraph_question": "question: \ubcc4\uc77c \uc5c6\uc774 \uc0b0\ub2e4\uac00 \uccab \uc815\uaddc \uc74c\ubc18\uc774\ub77c\uc11c \ub300\uc2e0 \uacf5\uc7a5\uc5d0\uc11c \uc0dd\uc0b0\ud558\uae30\ub85c \ud558\uace0, \uc0ac\uc6a9\ud558\uc9c0 \uc54a\uc740 \uae30\uc874 \uc0dd\uc0b0\ubc29\uc2dd\uc740 \ubb34\uc5c7\uc785\ub2c8\uae4c?, context: \uc7a5\uae30\ud558\ub294 2008\ub144 12\uc6d4 31\uc77c \uc790 \u300a\ud55c\uaca8\ub808\u300b \uae30\uc0ac\uc5d0\uc11c \"2\uc6d4 \ub9d0\uc5d0 \uc7a5\uae30\ud558\uc640 \uc5bc\uad74\ub4e4 \uc815\uaddc \uc74c\ubc18\uc774 \ub098\uc628\ub2e4.\"\uace0 \uc5b8\uae09\ud588\ub2e4. \uc18c\uc18d\uc0ac \ubd95\uac00\ubd95\uac00\ub808\ucf54\ub4dc \uad00\uacc4\uc790\ub294 2009\ub144 2\uc6d4 3\uc77c\uc5d0 \"\uc7a5\uae30\ud558\uc640 \uc5bc\uad74\ub4e4\uc758 \uc815\uaddc 1\uc9d1\uc774 \uc624\ub294 27\uc77c \ubc1c\ub9e4\ub420 \uc608\uc815\"\uc774\ub77c\uba70 \"\ubc1c\ub9e4\uc77c\uc5d0 \ub9de\ucdb0 \ubc1c\ub9e4 \uae30\ub150\uacf5\uc5f0\ub3c4 \ud568\uaed8 \uc5f0\ub2e4\"\uace0 \ubc1c\ub9e4 \uc77c\uc790\ub97c \ud655\uc815\uc9c0\uc5c8\ub2e4. \uc774 \uad00\uacc4\uc790\ub294 \"\ud604\uc7ac \ub9c8\ubb34\ub9ac \uc74c\ubc18 \uc791\uc5c5\uc744 \uc9c4\ud589 \uc911\"\uc774\ub77c\uba70 \"\uc74c\ubc18 \ud0c0\uc774\ud2c0\uacfc \uc218\ub85d\uace1 \uc804\ubd80\uac00 \uacb0\uc815\ub41c \uc0c1\ud0dc\ub294 \uc544\ub2c8\ub2e4\"\ub77c\uace0 \ub367\ubd99\uc600\ub2e4. \uc7a5\uae30\ud558\ub294 \uc608\uc2a424 \uc778\ud130\ubdf0\uc5d0\uc11c \"\uc0c1\uc0c1\ub9c8\ub2f9\uc5d0\uc11c 2\uc6d4 27\uc77c \uacf5\uc5f0 \uacb8 \uc1fc\ucf00\uc774\uc2a4\uac00 \uc7a1\ud600 \uc788\uc5b4\uc11c \uc790\uc5f0\uc2a4\ub7fd\uac8c 1\uc9d1 \ub9c8\uac10 \uc2dc\ud55c\uc774 \uacb0\uc815\ub418\uc5c8\ub2e4.\"\uace0 \ubc1d\ud614\ub2e4. \uccab \uc815\uaddc \uc74c\ubc18\uc778 \ub9cc\ud07c \u300a\ubcc4\uc77c \uc5c6\uc774 \uc0b0\ub2e4\u300b\ub294 \uac00\ub0b4\uc218\uacf5\uc5c5\uc744 \ud1b5\ud55c \uc18c\ub7c9 \uc0dd\uc0b0\ubc29\uc2dd\uc744 \ud0dd\ud558\uc9c0 \uc54a\uace0 \uacf5\uc7a5\uc5d0\uc11c \ucc0d\uc5b4\ub0c8\ub2e4."} {"question": "1730\ub144\ub300\uc640 1740\ub144\ub300\uc5d0 \uc77c\uc5b4\ub09c \uae30\ub3c5\uad50 \ubd80\ud765\uc8fc\uc758 \uc6b4\ub3d9\uc740 \ubb34\uc5c7\uc778\uac00?", "paragraph": "\uc544\uba54\ub9ac\uce74 \uc2dd\ubbfc\uc9c0\ub294 \ucd9c\uc0b0\uc728\uc774 \ub192\uace0 \uc0ac\ub9dd\ub960\uc774 \ub0ae\uc73c\uba70, \uc5ec\uc131\uc744 \ud3ec\ud568\ud55c \uc774\ubbfc\uc790\uac00 \ub04a\uc784\uc5c6\uc774 \uc720\uc785\ud558\uc5ec \uc778\uad6c\uac00 \uae09\uc18d\ud788 \uc99d\uac00\ud588\ub2e4. (\ubbf8\uad6d\uc758 \ud3c9\uade0\uc218\uba85\uc740 \uc601\uad6d\uc758 \ud3c9\uade0\uc218\uba85\ubcf4\ub2e4 10\uc138\uc774\uc0c1 \uc55e\uc11c\uc788\uc5c8\ub2e4.) \ud504\ub80c\uce58 \uc778\ub514\uc5b8 \uc804\uc7c1\uc5d0\uc11c \uc601\uad6d\uad70\uc774 \ud504\ub791\uc2a4\uc5d0\uac8c \uce90\ub098\ub2e4\ub97c \ube7c\uc557\uc558\uc73c\ub098, \uc774 \uacf3\uc758 \ud504\ub791\uc2a4\uc5b4\ub97c \uc4f0\ub294 \uc8fc\ubbfc\ub4e4\uc740 \ub0a8\ucabd \uc2dd\ubbfc\uc9c0\uc640\ub294 \uc815\uce58\uc801\uc73c\ub85c \ub3c5\ub9bd\ub41c \uc0c1\ud0dc\ub97c \uc720\uc9c0\ud558\uc600\ub2e4. \uc544\uba54\ub9ac\uce74 \ud1a0\ucc29\ubbfc\ub4e4\uc744 \uc81c\uc678\ud55c \uc774\ub4e4 13\uac1c \uc2dd\ubbfc\uc9c0\uc758 \uc778\uad6c\ub294 1770\ub144\uc5d0 260\ub9cc\uc5ec \uba85\uc73c\ub85c \ub2f9\uc2dc \uc601\uad6d\uc758 1/3 \uc218\uc900\uc774\uc5c8\uc73c\uba70, \uc544\uba54\ub9ac\uce74 \uc778\uad6c\uc758 1/5\uc740 \uc544\ud504\ub9ac\uce74\uacc4 \ubbf8\uad6d\uc778 \ub178\uc608\ub4e4\uc774\uc5c8\ub2e4. \uc544\uba54\ub9ac\uce74 \uc2dd\ubbfc\uc9c0\ub294 \uc601\uad6d\uc5d0 \uc870\uc138\ub97c \ub0a9\ubd80\ud588\uc73c\ub098, \uc601\uad6d \uc758\ud68c\uc5d0 \ub300\ud45c\uc790\ub97c \ub450\uc9c0 \ubabb\ud588\ub2e4. 1730\ub144\ub300\uc640 1740\ub144\ub300\uc758 \ub300\uac01\uc131 \uc6b4\ub3d9\uc774\ub77c\ub294 \uae30\ub3c5\uad50 \ubd80\ud765\uc8fc\uc758 \uc6b4\ub3d9\uc73c\ub85c \uc885\uad50\uc640 \uc885\uad50 \uc790\uc720\uc5d0 \ub300\ud55c \uad00\uc2ec\uc774 \ucee4\uc84c\ub2e4. \ub610\ud55c \uc774 \uc6b4\ub3d9\uc73c\ub85c \uc778\ud574 \ubbf8\uad6d \ub300\ud559\uc758 \uc5ed\uc0ac\uac00 \uc2dc\uc791\ub418\ub294\ub370, \uc885\uad50\uc758 \uc790\uc720\uc640 \uacfc\ud559\uc758 \ud0d0\uad6c\ub97c \uc704\ud574 \uc801\uc9c0\uc54a\uc740 \ub300\ud559\ub4e4\uc774 \uc124\ub9bd\ub418\uc5c8\ub2e4. 1636\ub144\uc5d0 \ud558\ubc84\ub4dc \ub300\ud559\uad50\uac00, \uc774\ud6c4\uc5d0 \uc70c\ub9ac\uc5c4 \uc564 \uba54\ub9ac \ub300\ud559\uad50, \uc608\uc77c \ub300\ud559\uad50, \ud504\ub9b0\uc2a4\ud134 \ub300\ud559\uad50, \ud39c\uc2e4\ubca0\uc774\ub2c8\uc544 \ub300\ud559\uad50 \ub4f1 \uc18c\uc704 \uc544\uc774\ube44\ub9ac\uadf8\ub85c \uc9c0\uce6d\ub418\ub294 \uba85\ubb38 \uc0ac\ub9bd \ub300\ud559\ub4e4\uc774 \uc124\ub9bd\ub418\uc5c8\ub2e4.", "answer": "\ub300\uac01\uc131 \uc6b4\ub3d9", "sentence": "1730\ub144\ub300\uc640 1740\ub144\ub300\uc758 \ub300\uac01\uc131 \uc6b4\ub3d9 \uc774\ub77c\ub294 \uae30\ub3c5\uad50 \ubd80\ud765\uc8fc\uc758 \uc6b4\ub3d9\uc73c\ub85c \uc885\uad50\uc640 \uc885\uad50 \uc790\uc720\uc5d0 \ub300\ud55c \uad00\uc2ec\uc774 \ucee4\uc84c\ub2e4.", "paragraph_sentence": "\uc544\uba54\ub9ac\uce74 \uc2dd\ubbfc\uc9c0\ub294 \ucd9c\uc0b0\uc728\uc774 \ub192\uace0 \uc0ac\ub9dd\ub960\uc774 \ub0ae\uc73c\uba70, \uc5ec\uc131\uc744 \ud3ec\ud568\ud55c \uc774\ubbfc\uc790\uac00 \ub04a\uc784\uc5c6\uc774 \uc720\uc785\ud558\uc5ec \uc778\uad6c\uac00 \uae09\uc18d\ud788 \uc99d\uac00\ud588\ub2e4. (\ubbf8\uad6d\uc758 \ud3c9\uade0\uc218\uba85\uc740 \uc601\uad6d\uc758 \ud3c9\uade0\uc218\uba85\ubcf4\ub2e4 10\uc138\uc774\uc0c1 \uc55e\uc11c\uc788\uc5c8\ub2e4.) \ud504\ub80c\uce58 \uc778\ub514\uc5b8 \uc804\uc7c1\uc5d0\uc11c \uc601\uad6d\uad70\uc774 \ud504\ub791\uc2a4\uc5d0\uac8c \uce90\ub098\ub2e4\ub97c \ube7c\uc557\uc558\uc73c\ub098, \uc774 \uacf3\uc758 \ud504\ub791\uc2a4\uc5b4\ub97c \uc4f0\ub294 \uc8fc\ubbfc\ub4e4\uc740 \ub0a8\ucabd \uc2dd\ubbfc\uc9c0\uc640\ub294 \uc815\uce58\uc801\uc73c\ub85c \ub3c5\ub9bd\ub41c \uc0c1\ud0dc\ub97c \uc720\uc9c0\ud558\uc600\ub2e4. \uc544\uba54\ub9ac\uce74 \ud1a0\ucc29\ubbfc\ub4e4\uc744 \uc81c\uc678\ud55c \uc774\ub4e4 13\uac1c \uc2dd\ubbfc\uc9c0\uc758 \uc778\uad6c\ub294 1770\ub144\uc5d0 260\ub9cc\uc5ec \uba85\uc73c\ub85c \ub2f9\uc2dc \uc601\uad6d\uc758 1/3 \uc218\uc900\uc774\uc5c8\uc73c\uba70, \uc544\uba54\ub9ac\uce74 \uc778\uad6c\uc758 1/5\uc740 \uc544\ud504\ub9ac\uce74\uacc4 \ubbf8\uad6d\uc778 \ub178\uc608\ub4e4\uc774\uc5c8\ub2e4. \uc544\uba54\ub9ac\uce74 \uc2dd\ubbfc\uc9c0\ub294 \uc601\uad6d\uc5d0 \uc870\uc138\ub97c \ub0a9\ubd80\ud588\uc73c\ub098, \uc601\uad6d \uc758\ud68c\uc5d0 \ub300\ud45c\uc790\ub97c \ub450\uc9c0 \ubabb\ud588\ub2e4. 1730\ub144\ub300\uc640 1740\ub144\ub300\uc758 \ub300\uac01\uc131 \uc6b4\ub3d9 \uc774\ub77c\ub294 \uae30\ub3c5\uad50 \ubd80\ud765\uc8fc\uc758 \uc6b4\ub3d9\uc73c\ub85c \uc885\uad50\uc640 \uc885\uad50 \uc790\uc720\uc5d0 \ub300\ud55c \uad00\uc2ec\uc774 \ucee4\uc84c\ub2e4. \ub610\ud55c \uc774 \uc6b4\ub3d9\uc73c\ub85c \uc778\ud574 \ubbf8\uad6d \ub300\ud559\uc758 \uc5ed\uc0ac\uac00 \uc2dc\uc791\ub418\ub294\ub370, \uc885\uad50\uc758 \uc790\uc720\uc640 \uacfc\ud559\uc758 \ud0d0\uad6c\ub97c \uc704\ud574 \uc801\uc9c0\uc54a\uc740 \ub300\ud559\ub4e4\uc774 \uc124\ub9bd\ub418\uc5c8\ub2e4. 1636\ub144\uc5d0 \ud558\ubc84\ub4dc \ub300\ud559\uad50\uac00, \uc774\ud6c4\uc5d0 \uc70c\ub9ac\uc5c4 \uc564 \uba54\ub9ac \ub300\ud559\uad50, \uc608\uc77c \ub300\ud559\uad50, \ud504\ub9b0\uc2a4\ud134 \ub300\ud559\uad50, \ud39c\uc2e4\ubca0\uc774\ub2c8\uc544 \ub300\ud559\uad50 \ub4f1 \uc18c\uc704 \uc544\uc774\ube44\ub9ac\uadf8\ub85c \uc9c0\uce6d\ub418\ub294 \uba85\ubb38 \uc0ac\ub9bd \ub300\ud559\ub4e4\uc774 \uc124\ub9bd\ub418\uc5c8\ub2e4.", "paragraph_answer": "\uc544\uba54\ub9ac\uce74 \uc2dd\ubbfc\uc9c0\ub294 \ucd9c\uc0b0\uc728\uc774 \ub192\uace0 \uc0ac\ub9dd\ub960\uc774 \ub0ae\uc73c\uba70, \uc5ec\uc131\uc744 \ud3ec\ud568\ud55c \uc774\ubbfc\uc790\uac00 \ub04a\uc784\uc5c6\uc774 \uc720\uc785\ud558\uc5ec \uc778\uad6c\uac00 \uae09\uc18d\ud788 \uc99d\uac00\ud588\ub2e4. (\ubbf8\uad6d\uc758 \ud3c9\uade0\uc218\uba85\uc740 \uc601\uad6d\uc758 \ud3c9\uade0\uc218\uba85\ubcf4\ub2e4 10\uc138\uc774\uc0c1 \uc55e\uc11c\uc788\uc5c8\ub2e4.) \ud504\ub80c\uce58 \uc778\ub514\uc5b8 \uc804\uc7c1\uc5d0\uc11c \uc601\uad6d\uad70\uc774 \ud504\ub791\uc2a4\uc5d0\uac8c \uce90\ub098\ub2e4\ub97c \ube7c\uc557\uc558\uc73c\ub098, \uc774 \uacf3\uc758 \ud504\ub791\uc2a4\uc5b4\ub97c \uc4f0\ub294 \uc8fc\ubbfc\ub4e4\uc740 \ub0a8\ucabd \uc2dd\ubbfc\uc9c0\uc640\ub294 \uc815\uce58\uc801\uc73c\ub85c \ub3c5\ub9bd\ub41c \uc0c1\ud0dc\ub97c \uc720\uc9c0\ud558\uc600\ub2e4. \uc544\uba54\ub9ac\uce74 \ud1a0\ucc29\ubbfc\ub4e4\uc744 \uc81c\uc678\ud55c \uc774\ub4e4 13\uac1c \uc2dd\ubbfc\uc9c0\uc758 \uc778\uad6c\ub294 1770\ub144\uc5d0 260\ub9cc\uc5ec \uba85\uc73c\ub85c \ub2f9\uc2dc \uc601\uad6d\uc758 1/3 \uc218\uc900\uc774\uc5c8\uc73c\uba70, \uc544\uba54\ub9ac\uce74 \uc778\uad6c\uc758 1/5\uc740 \uc544\ud504\ub9ac\uce74\uacc4 \ubbf8\uad6d\uc778 \ub178\uc608\ub4e4\uc774\uc5c8\ub2e4. \uc544\uba54\ub9ac\uce74 \uc2dd\ubbfc\uc9c0\ub294 \uc601\uad6d\uc5d0 \uc870\uc138\ub97c \ub0a9\ubd80\ud588\uc73c\ub098, \uc601\uad6d \uc758\ud68c\uc5d0 \ub300\ud45c\uc790\ub97c \ub450\uc9c0 \ubabb\ud588\ub2e4. 1730\ub144\ub300\uc640 1740\ub144\ub300\uc758 \ub300\uac01\uc131 \uc6b4\ub3d9 \uc774\ub77c\ub294 \uae30\ub3c5\uad50 \ubd80\ud765\uc8fc\uc758 \uc6b4\ub3d9\uc73c\ub85c \uc885\uad50\uc640 \uc885\uad50 \uc790\uc720\uc5d0 \ub300\ud55c \uad00\uc2ec\uc774 \ucee4\uc84c\ub2e4. \ub610\ud55c \uc774 \uc6b4\ub3d9\uc73c\ub85c \uc778\ud574 \ubbf8\uad6d \ub300\ud559\uc758 \uc5ed\uc0ac\uac00 \uc2dc\uc791\ub418\ub294\ub370, \uc885\uad50\uc758 \uc790\uc720\uc640 \uacfc\ud559\uc758 \ud0d0\uad6c\ub97c \uc704\ud574 \uc801\uc9c0\uc54a\uc740 \ub300\ud559\ub4e4\uc774 \uc124\ub9bd\ub418\uc5c8\ub2e4. 1636\ub144\uc5d0 \ud558\ubc84\ub4dc \ub300\ud559\uad50\uac00, \uc774\ud6c4\uc5d0 \uc70c\ub9ac\uc5c4 \uc564 \uba54\ub9ac \ub300\ud559\uad50, \uc608\uc77c \ub300\ud559\uad50, \ud504\ub9b0\uc2a4\ud134 \ub300\ud559\uad50, \ud39c\uc2e4\ubca0\uc774\ub2c8\uc544 \ub300\ud559\uad50 \ub4f1 \uc18c\uc704 \uc544\uc774\ube44\ub9ac\uadf8\ub85c \uc9c0\uce6d\ub418\ub294 \uba85\ubb38 \uc0ac\ub9bd \ub300\ud559\ub4e4\uc774 \uc124\ub9bd\ub418\uc5c8\ub2e4.", "sentence_answer": "1730\ub144\ub300\uc640 1740\ub144\ub300\uc758 \ub300\uac01\uc131 \uc6b4\ub3d9 \uc774\ub77c\ub294 \uae30\ub3c5\uad50 \ubd80\ud765\uc8fc\uc758 \uc6b4\ub3d9\uc73c\ub85c \uc885\uad50\uc640 \uc885\uad50 \uc790\uc720\uc5d0 \ub300\ud55c \uad00\uc2ec\uc774 \ucee4\uc84c\ub2e4.", "paragraph_id": "6412933-6-1", "paragraph_question": "question: 1730\ub144\ub300\uc640 1740\ub144\ub300\uc5d0 \uc77c\uc5b4\ub09c \uae30\ub3c5\uad50 \ubd80\ud765\uc8fc\uc758 \uc6b4\ub3d9\uc740 \ubb34\uc5c7\uc778\uac00?, context: \uc544\uba54\ub9ac\uce74 \uc2dd\ubbfc\uc9c0\ub294 \ucd9c\uc0b0\uc728\uc774 \ub192\uace0 \uc0ac\ub9dd\ub960\uc774 \ub0ae\uc73c\uba70, \uc5ec\uc131\uc744 \ud3ec\ud568\ud55c \uc774\ubbfc\uc790\uac00 \ub04a\uc784\uc5c6\uc774 \uc720\uc785\ud558\uc5ec \uc778\uad6c\uac00 \uae09\uc18d\ud788 \uc99d\uac00\ud588\ub2e4. (\ubbf8\uad6d\uc758 \ud3c9\uade0\uc218\uba85\uc740 \uc601\uad6d\uc758 \ud3c9\uade0\uc218\uba85\ubcf4\ub2e4 10\uc138\uc774\uc0c1 \uc55e\uc11c\uc788\uc5c8\ub2e4.) \ud504\ub80c\uce58 \uc778\ub514\uc5b8 \uc804\uc7c1\uc5d0\uc11c \uc601\uad6d\uad70\uc774 \ud504\ub791\uc2a4\uc5d0\uac8c \uce90\ub098\ub2e4\ub97c \ube7c\uc557\uc558\uc73c\ub098, \uc774 \uacf3\uc758 \ud504\ub791\uc2a4\uc5b4\ub97c \uc4f0\ub294 \uc8fc\ubbfc\ub4e4\uc740 \ub0a8\ucabd \uc2dd\ubbfc\uc9c0\uc640\ub294 \uc815\uce58\uc801\uc73c\ub85c \ub3c5\ub9bd\ub41c \uc0c1\ud0dc\ub97c \uc720\uc9c0\ud558\uc600\ub2e4. \uc544\uba54\ub9ac\uce74 \ud1a0\ucc29\ubbfc\ub4e4\uc744 \uc81c\uc678\ud55c \uc774\ub4e4 13\uac1c \uc2dd\ubbfc\uc9c0\uc758 \uc778\uad6c\ub294 1770\ub144\uc5d0 260\ub9cc\uc5ec \uba85\uc73c\ub85c \ub2f9\uc2dc \uc601\uad6d\uc758 1/3 \uc218\uc900\uc774\uc5c8\uc73c\uba70, \uc544\uba54\ub9ac\uce74 \uc778\uad6c\uc758 1/5\uc740 \uc544\ud504\ub9ac\uce74\uacc4 \ubbf8\uad6d\uc778 \ub178\uc608\ub4e4\uc774\uc5c8\ub2e4. \uc544\uba54\ub9ac\uce74 \uc2dd\ubbfc\uc9c0\ub294 \uc601\uad6d\uc5d0 \uc870\uc138\ub97c \ub0a9\ubd80\ud588\uc73c\ub098, \uc601\uad6d \uc758\ud68c\uc5d0 \ub300\ud45c\uc790\ub97c \ub450\uc9c0 \ubabb\ud588\ub2e4. 1730\ub144\ub300\uc640 1740\ub144\ub300\uc758 \ub300\uac01\uc131 \uc6b4\ub3d9\uc774\ub77c\ub294 \uae30\ub3c5\uad50 \ubd80\ud765\uc8fc\uc758 \uc6b4\ub3d9\uc73c\ub85c \uc885\uad50\uc640 \uc885\uad50 \uc790\uc720\uc5d0 \ub300\ud55c \uad00\uc2ec\uc774 \ucee4\uc84c\ub2e4. \ub610\ud55c \uc774 \uc6b4\ub3d9\uc73c\ub85c \uc778\ud574 \ubbf8\uad6d \ub300\ud559\uc758 \uc5ed\uc0ac\uac00 \uc2dc\uc791\ub418\ub294\ub370, \uc885\uad50\uc758 \uc790\uc720\uc640 \uacfc\ud559\uc758 \ud0d0\uad6c\ub97c \uc704\ud574 \uc801\uc9c0\uc54a\uc740 \ub300\ud559\ub4e4\uc774 \uc124\ub9bd\ub418\uc5c8\ub2e4. 1636\ub144\uc5d0 \ud558\ubc84\ub4dc \ub300\ud559\uad50\uac00, \uc774\ud6c4\uc5d0 \uc70c\ub9ac\uc5c4 \uc564 \uba54\ub9ac \ub300\ud559\uad50, \uc608\uc77c \ub300\ud559\uad50, \ud504\ub9b0\uc2a4\ud134 \ub300\ud559\uad50, \ud39c\uc2e4\ubca0\uc774\ub2c8\uc544 \ub300\ud559\uad50 \ub4f1 \uc18c\uc704 \uc544\uc774\ube44\ub9ac\uadf8\ub85c \uc9c0\uce6d\ub418\ub294 \uba85\ubb38 \uc0ac\ub9bd \ub300\ud559\ub4e4\uc774 \uc124\ub9bd\ub418\uc5c8\ub2e4."} {"question": "\ub3c5\uc77c\uad70\uc758 \uaca8\uc6b8\ud3ed\ud48d \uc791\uc804\uc73c\ub85c \uc18c\ub828\uad70\uc758 \ubcd1\ub825 \uc7ac\ubc30\uce58\uac00 \uc774\ub904\uc9c0\uace0 \uacf5\uc138 \uaddc\ubaa8\uac00 \uc904\uc5b4\ub4e4\uc5b4 \ud1a0\uc131 \uc791\uc804\uc740 \uc5b4\ub5a4 \ubcc4\uba85\uc774 \ubd99\uc5c8\ub098?", "paragraph": "\ud55c\ud3b8, \uc18c\ub828\uad70\uc740 \ub85c\uc2a4\ud1a0\ud504\ub97c \ud0c8\ud658\ud558\uace0 \ub3c5\uc77c A \uc9d1\ub2e8\uad70\uc744 \uace0\ub9bd\uc2dc\ud0a4\uae30 \uc704\ud55c \uc790\uae30\ub124\ub4e4\uc758 \uacf5\uc138\uc791\uc804\uc744 \uac1c\uc2dc\ud588\ub2e4(\ud1a0\uc131 \uc791\uc804). \uadf8\ub7ec\ub098 \uaca8\uc6b8\ud3ed\ud48d \uc791\uc804\uc774 \uac1c\uc2dc\ub418\uc790 \uc18c\ub828\uad70\uc740 \uc2a4\ud0c8\ub9b0\uadf8\ub77c\ub4dc \uad6c\uc6d0\uc744 \ub9c9\uae30 \uc704\ud558\uc5ec \ubcd1\ub825 \uc77c\ubd80\ub97c \uc7ac\ubc30\uce58\ud574\uc57c \ud588\ub2e4. \ub54c\ubb38\uc5d0 \uacf5\uc138\uc791\uc804\uc758 \uaddc\ubaa8\uac00 \uc904\uc5b4\ub4e4\uc5c8\uace0, \u201c\uc18c\ud1a0\uc131 \uc791\uc804\u201d\uc774\ub77c\ub294 \ubcc4\uba85\uc774 \ubd99\uc5c8\ub2e4. \uc544\ubb34\ud2bc \ud1a0\uc131 \uc791\uc804\uc774 \uc2dc\uc791\ub418\uc790 \ub9cc\uc288\ud0c0\uc778\uc740 \uc804\uc120 \uc804\uccb4\uc758 \ubd95\uad34\ub97c \ub9c9\uae30 \uc704\ud574 \ubcd1\ub825\uc758 \ubd84\uc0b0\uc744 \uac15\uc694\ub2f9\ud588\ub2e4. \ub610\ud55c \uc774 \uacf5\uaca9\uc73c\ub85c \uc778\ud574 \uc624\ud1a0 \ud3f0 \ud06c\ub178\ubca8\uc2a4\ub3c4\ub974\ud504 \uae30\uac11\ub300\uc7a5\uc758 \uc81c48\uae30\uac11\uad70\ub2e8(\uc608\ud558 \ubd80\ub300: \uc81c336\ubcf4\ubcd1\uc0ac\ub2e8, \uc81c3\uacf5\uad70\uc57c\uc804\uc0ac\ub2e8, \uc81c11\uae30\uac11\uc0ac\ub2e8)\uc774 \ud30c\uc6b8\ub8e8\uc2a4\uc758 \uc81c27\uae30\uac11\uad70\ub2e8\uacfc \uc811\uc120\ud558\ub824\ub358 \uacc4\ud68d \uc790\uccb4\uac00 \ud2c0\uc5b4\uc838 \ubc84\ub838\ub2e4. \uc81c48\uad70\ub2e8\uc740 \uad6c\ucd9c \uc784\ubb34\uc5d0 \ud22c\uc785\ub418\ub294 \ub300\uc2e0 \uce58\ub974 \uac15\uc744 \ub530\ub77c \ub3c4\uc5f4\ud558\uc5ec \uc18c\ub828\uad70\uc758 \uacf5\uaca9\uc744 \ub9c9\uc544\ub0c8\ub2e4. \ud5e4\ub974\ub9cc \ubc1c\ud06c \uae30\uac11\ub300\uc7a5\uc774 \uc81c11\uc0ac\ub2e8\uc73c\ub85c \uc5ed\uacf5\uc744 \uac00\ud55c \uac83\uc774 \uc8fc\ud6a8\ud588\ub2e4. \ub3c5\uc77c\uad70\uc740 \uc804\uc5f4\ubd95\uad34 \uc9c1\uc804\uc5d0 \uc804\uc5f4\uc744 \uc7ac\uc815\ube44\ud588\uc73c\ub098, \uce21\uba74\uc758 \uc774\ud0c8\ub9ac\uc544 \uc81c8\uad70\uc740 \uc18c\ub828\uad70\uc5d0\uac8c \uc555\ub3c4\ub418\uc5b4 \uada4\uba78\ub2f9\ud588\ub2e4.", "answer": "\uc18c\ud1a0\uc131 \uc791\uc804", "sentence": "\ub54c\ubb38\uc5d0 \uacf5\uc138\uc791\uc804\uc758 \uaddc\ubaa8\uac00 \uc904\uc5b4\ub4e4\uc5c8\uace0, \u201c \uc18c\ud1a0\uc131 \uc791\uc804 \u201d\uc774\ub77c\ub294 \ubcc4\uba85\uc774 \ubd99\uc5c8\ub2e4.", "paragraph_sentence": "\ud55c\ud3b8, \uc18c\ub828\uad70\uc740 \ub85c\uc2a4\ud1a0\ud504\ub97c \ud0c8\ud658\ud558\uace0 \ub3c5\uc77c A \uc9d1\ub2e8\uad70\uc744 \uace0\ub9bd\uc2dc\ud0a4\uae30 \uc704\ud55c \uc790\uae30\ub124\ub4e4\uc758 \uacf5\uc138\uc791\uc804\uc744 \uac1c\uc2dc\ud588\ub2e4(\ud1a0\uc131 \uc791\uc804). \uadf8\ub7ec\ub098 \uaca8\uc6b8\ud3ed\ud48d \uc791\uc804\uc774 \uac1c\uc2dc\ub418\uc790 \uc18c\ub828\uad70\uc740 \uc2a4\ud0c8\ub9b0\uadf8\ub77c\ub4dc \uad6c\uc6d0\uc744 \ub9c9\uae30 \uc704\ud558\uc5ec \ubcd1\ub825 \uc77c\ubd80\ub97c \uc7ac\ubc30\uce58\ud574\uc57c \ud588\ub2e4. \ub54c\ubb38\uc5d0 \uacf5\uc138\uc791\uc804\uc758 \uaddc\ubaa8\uac00 \uc904\uc5b4\ub4e4\uc5c8\uace0, \u201c \uc18c\ud1a0\uc131 \uc791\uc804 \u201d\uc774\ub77c\ub294 \ubcc4\uba85\uc774 \ubd99\uc5c8\ub2e4. \uc544\ubb34\ud2bc \ud1a0\uc131 \uc791\uc804\uc774 \uc2dc\uc791\ub418\uc790 \ub9cc\uc288\ud0c0\uc778\uc740 \uc804\uc120 \uc804\uccb4\uc758 \ubd95\uad34\ub97c \ub9c9\uae30 \uc704\ud574 \ubcd1\ub825\uc758 \ubd84\uc0b0\uc744 \uac15\uc694\ub2f9\ud588\ub2e4. \ub610\ud55c \uc774 \uacf5\uaca9\uc73c\ub85c \uc778\ud574 \uc624\ud1a0 \ud3f0 \ud06c\ub178\ubca8\uc2a4\ub3c4\ub974\ud504 \uae30\uac11\ub300\uc7a5\uc758 \uc81c48\uae30\uac11\uad70\ub2e8(\uc608\ud558 \ubd80\ub300: \uc81c336\ubcf4\ubcd1\uc0ac\ub2e8, \uc81c3\uacf5\uad70\uc57c\uc804\uc0ac\ub2e8, \uc81c11\uae30\uac11\uc0ac\ub2e8)\uc774 \ud30c\uc6b8\ub8e8\uc2a4\uc758 \uc81c27\uae30\uac11\uad70\ub2e8\uacfc \uc811\uc120\ud558\ub824\ub358 \uacc4\ud68d \uc790\uccb4\uac00 \ud2c0\uc5b4\uc838 \ubc84\ub838\ub2e4. \uc81c48\uad70\ub2e8\uc740 \uad6c\ucd9c \uc784\ubb34\uc5d0 \ud22c\uc785\ub418\ub294 \ub300\uc2e0 \uce58\ub974 \uac15\uc744 \ub530\ub77c \ub3c4\uc5f4\ud558\uc5ec \uc18c\ub828\uad70\uc758 \uacf5\uaca9\uc744 \ub9c9\uc544\ub0c8\ub2e4. \ud5e4\ub974\ub9cc \ubc1c\ud06c \uae30\uac11\ub300\uc7a5\uc774 \uc81c11\uc0ac\ub2e8\uc73c\ub85c \uc5ed\uacf5\uc744 \uac00\ud55c \uac83\uc774 \uc8fc\ud6a8\ud588\ub2e4. \ub3c5\uc77c\uad70\uc740 \uc804\uc5f4\ubd95\uad34 \uc9c1\uc804\uc5d0 \uc804\uc5f4\uc744 \uc7ac\uc815\ube44\ud588\uc73c\ub098, \uce21\uba74\uc758 \uc774\ud0c8\ub9ac\uc544 \uc81c8\uad70\uc740 \uc18c\ub828\uad70\uc5d0\uac8c \uc555\ub3c4\ub418\uc5b4 \uada4\uba78\ub2f9\ud588\ub2e4.", "paragraph_answer": "\ud55c\ud3b8, \uc18c\ub828\uad70\uc740 \ub85c\uc2a4\ud1a0\ud504\ub97c \ud0c8\ud658\ud558\uace0 \ub3c5\uc77c A \uc9d1\ub2e8\uad70\uc744 \uace0\ub9bd\uc2dc\ud0a4\uae30 \uc704\ud55c \uc790\uae30\ub124\ub4e4\uc758 \uacf5\uc138\uc791\uc804\uc744 \uac1c\uc2dc\ud588\ub2e4(\ud1a0\uc131 \uc791\uc804). \uadf8\ub7ec\ub098 \uaca8\uc6b8\ud3ed\ud48d \uc791\uc804\uc774 \uac1c\uc2dc\ub418\uc790 \uc18c\ub828\uad70\uc740 \uc2a4\ud0c8\ub9b0\uadf8\ub77c\ub4dc \uad6c\uc6d0\uc744 \ub9c9\uae30 \uc704\ud558\uc5ec \ubcd1\ub825 \uc77c\ubd80\ub97c \uc7ac\ubc30\uce58\ud574\uc57c \ud588\ub2e4. \ub54c\ubb38\uc5d0 \uacf5\uc138\uc791\uc804\uc758 \uaddc\ubaa8\uac00 \uc904\uc5b4\ub4e4\uc5c8\uace0, \u201c \uc18c\ud1a0\uc131 \uc791\uc804 \u201d\uc774\ub77c\ub294 \ubcc4\uba85\uc774 \ubd99\uc5c8\ub2e4. \uc544\ubb34\ud2bc \ud1a0\uc131 \uc791\uc804\uc774 \uc2dc\uc791\ub418\uc790 \ub9cc\uc288\ud0c0\uc778\uc740 \uc804\uc120 \uc804\uccb4\uc758 \ubd95\uad34\ub97c \ub9c9\uae30 \uc704\ud574 \ubcd1\ub825\uc758 \ubd84\uc0b0\uc744 \uac15\uc694\ub2f9\ud588\ub2e4. \ub610\ud55c \uc774 \uacf5\uaca9\uc73c\ub85c \uc778\ud574 \uc624\ud1a0 \ud3f0 \ud06c\ub178\ubca8\uc2a4\ub3c4\ub974\ud504 \uae30\uac11\ub300\uc7a5\uc758 \uc81c48\uae30\uac11\uad70\ub2e8(\uc608\ud558 \ubd80\ub300: \uc81c336\ubcf4\ubcd1\uc0ac\ub2e8, \uc81c3\uacf5\uad70\uc57c\uc804\uc0ac\ub2e8, \uc81c11\uae30\uac11\uc0ac\ub2e8)\uc774 \ud30c\uc6b8\ub8e8\uc2a4\uc758 \uc81c27\uae30\uac11\uad70\ub2e8\uacfc \uc811\uc120\ud558\ub824\ub358 \uacc4\ud68d \uc790\uccb4\uac00 \ud2c0\uc5b4\uc838 \ubc84\ub838\ub2e4. \uc81c48\uad70\ub2e8\uc740 \uad6c\ucd9c \uc784\ubb34\uc5d0 \ud22c\uc785\ub418\ub294 \ub300\uc2e0 \uce58\ub974 \uac15\uc744 \ub530\ub77c \ub3c4\uc5f4\ud558\uc5ec \uc18c\ub828\uad70\uc758 \uacf5\uaca9\uc744 \ub9c9\uc544\ub0c8\ub2e4. \ud5e4\ub974\ub9cc \ubc1c\ud06c \uae30\uac11\ub300\uc7a5\uc774 \uc81c11\uc0ac\ub2e8\uc73c\ub85c \uc5ed\uacf5\uc744 \uac00\ud55c \uac83\uc774 \uc8fc\ud6a8\ud588\ub2e4. \ub3c5\uc77c\uad70\uc740 \uc804\uc5f4\ubd95\uad34 \uc9c1\uc804\uc5d0 \uc804\uc5f4\uc744 \uc7ac\uc815\ube44\ud588\uc73c\ub098, \uce21\uba74\uc758 \uc774\ud0c8\ub9ac\uc544 \uc81c8\uad70\uc740 \uc18c\ub828\uad70\uc5d0\uac8c \uc555\ub3c4\ub418\uc5b4 \uada4\uba78\ub2f9\ud588\ub2e4.", "sentence_answer": "\ub54c\ubb38\uc5d0 \uacf5\uc138\uc791\uc804\uc758 \uaddc\ubaa8\uac00 \uc904\uc5b4\ub4e4\uc5c8\uace0, \u201c \uc18c\ud1a0\uc131 \uc791\uc804 \u201d\uc774\ub77c\ub294 \ubcc4\uba85\uc774 \ubd99\uc5c8\ub2e4.", "paragraph_id": "6540644-22-1", "paragraph_question": "question: \ub3c5\uc77c\uad70\uc758 \uaca8\uc6b8\ud3ed\ud48d \uc791\uc804\uc73c\ub85c \uc18c\ub828\uad70\uc758 \ubcd1\ub825 \uc7ac\ubc30\uce58\uac00 \uc774\ub904\uc9c0\uace0 \uacf5\uc138 \uaddc\ubaa8\uac00 \uc904\uc5b4\ub4e4\uc5b4 \ud1a0\uc131 \uc791\uc804\uc740 \uc5b4\ub5a4 \ubcc4\uba85\uc774 \ubd99\uc5c8\ub098?, context: \ud55c\ud3b8, \uc18c\ub828\uad70\uc740 \ub85c\uc2a4\ud1a0\ud504\ub97c \ud0c8\ud658\ud558\uace0 \ub3c5\uc77c A \uc9d1\ub2e8\uad70\uc744 \uace0\ub9bd\uc2dc\ud0a4\uae30 \uc704\ud55c \uc790\uae30\ub124\ub4e4\uc758 \uacf5\uc138\uc791\uc804\uc744 \uac1c\uc2dc\ud588\ub2e4(\ud1a0\uc131 \uc791\uc804). \uadf8\ub7ec\ub098 \uaca8\uc6b8\ud3ed\ud48d \uc791\uc804\uc774 \uac1c\uc2dc\ub418\uc790 \uc18c\ub828\uad70\uc740 \uc2a4\ud0c8\ub9b0\uadf8\ub77c\ub4dc \uad6c\uc6d0\uc744 \ub9c9\uae30 \uc704\ud558\uc5ec \ubcd1\ub825 \uc77c\ubd80\ub97c \uc7ac\ubc30\uce58\ud574\uc57c \ud588\ub2e4. \ub54c\ubb38\uc5d0 \uacf5\uc138\uc791\uc804\uc758 \uaddc\ubaa8\uac00 \uc904\uc5b4\ub4e4\uc5c8\uace0, \u201c\uc18c\ud1a0\uc131 \uc791\uc804\u201d\uc774\ub77c\ub294 \ubcc4\uba85\uc774 \ubd99\uc5c8\ub2e4. \uc544\ubb34\ud2bc \ud1a0\uc131 \uc791\uc804\uc774 \uc2dc\uc791\ub418\uc790 \ub9cc\uc288\ud0c0\uc778\uc740 \uc804\uc120 \uc804\uccb4\uc758 \ubd95\uad34\ub97c \ub9c9\uae30 \uc704\ud574 \ubcd1\ub825\uc758 \ubd84\uc0b0\uc744 \uac15\uc694\ub2f9\ud588\ub2e4. \ub610\ud55c \uc774 \uacf5\uaca9\uc73c\ub85c \uc778\ud574 \uc624\ud1a0 \ud3f0 \ud06c\ub178\ubca8\uc2a4\ub3c4\ub974\ud504 \uae30\uac11\ub300\uc7a5\uc758 \uc81c48\uae30\uac11\uad70\ub2e8(\uc608\ud558 \ubd80\ub300: \uc81c336\ubcf4\ubcd1\uc0ac\ub2e8, \uc81c3\uacf5\uad70\uc57c\uc804\uc0ac\ub2e8, \uc81c11\uae30\uac11\uc0ac\ub2e8)\uc774 \ud30c\uc6b8\ub8e8\uc2a4\uc758 \uc81c27\uae30\uac11\uad70\ub2e8\uacfc \uc811\uc120\ud558\ub824\ub358 \uacc4\ud68d \uc790\uccb4\uac00 \ud2c0\uc5b4\uc838 \ubc84\ub838\ub2e4. \uc81c48\uad70\ub2e8\uc740 \uad6c\ucd9c \uc784\ubb34\uc5d0 \ud22c\uc785\ub418\ub294 \ub300\uc2e0 \uce58\ub974 \uac15\uc744 \ub530\ub77c \ub3c4\uc5f4\ud558\uc5ec \uc18c\ub828\uad70\uc758 \uacf5\uaca9\uc744 \ub9c9\uc544\ub0c8\ub2e4. \ud5e4\ub974\ub9cc \ubc1c\ud06c \uae30\uac11\ub300\uc7a5\uc774 \uc81c11\uc0ac\ub2e8\uc73c\ub85c \uc5ed\uacf5\uc744 \uac00\ud55c \uac83\uc774 \uc8fc\ud6a8\ud588\ub2e4. \ub3c5\uc77c\uad70\uc740 \uc804\uc5f4\ubd95\uad34 \uc9c1\uc804\uc5d0 \uc804\uc5f4\uc744 \uc7ac\uc815\ube44\ud588\uc73c\ub098, \uce21\uba74\uc758 \uc774\ud0c8\ub9ac\uc544 \uc81c8\uad70\uc740 \uc18c\ub828\uad70\uc5d0\uac8c \uc555\ub3c4\ub418\uc5b4 \uada4\uba78\ub2f9\ud588\ub2e4."} {"question": "\uac11\uc2e0\uc815\ubcc0\uc774 \uc77c\uc5b4\ub09c \ub0a0\uc740?", "paragraph": "\uc790\uae30\ubcf4\ub2e4 \uc5f4 \uc0b4 \uc5b4\ub9b0 \ud559\uc0dd\ub4e4\uacfc \ud568\uaed8 \uc601\uc5b4\u00b7\uc218\ud559\u00b7\uc9c0\ub9ac\u00b7\ub77c\ud2f4\uc5b4 \ub4f1\uc744 \uacf5\ubd80\ud558\ub294 \uac8c \uc27d\uc9c0 \uc54a\uc558\uc9c0\ub9cc \ud559\uc2b5 \ub2a5\ub825\uc740 \ub6f0\uc5b4\ub0ac\ub2e4. \uc11d \ub2ec \ub9cc\uc5d0 \ub3c5\ud559\uc73c\ub85c \uc601\uc5b4\ud68c\ud654\uac00 \uac00\ub2a5\ud588\ub2e4\uace0 \ud55c\ub2e4. \uc720\uae38\uc900\uc758 \ud6c4\uacac\uc778\uc774\ub77c\uace0 \ud560 \ubaa8\uc2a4 \uad50\uc218\uc640\uc758 \ud3b8\uc9c0\uc5d0\uc11c\ub294 \ud559\uad50\uc0dd\ud65c\uc5d0 \uad00\ud55c \ub0b4\uc6a9\uc774 \ub9ce\uc774 \ub098\uc628\ub2e4. \uc720\uae38\uc900\uc740 1884\ub144 11\uc6d4 3\uc77c \ud3b8\uc9c0\uc5d0\uc11c \u201c\ud654\uc0b0\u00b7\uc9c0\uc9c4\u00b7\uac04\ud5d0\ucc9c \ub4f1\uc5d0 \uad00\ud55c \uc9c0\uad6c\uacfc\ud559 \uc2dc\ud5d8\uc5d0\uc11c 94\uc810, \uc218\ud559 \uc2dc\ud5d8\uc5d0\uc11c 100\uc810\uc744 \ub9de\uc544 \ub9e4\uc6b0 \uae30\uc058\ub2e4\u201d\uace0 \ub9d0\ud588\ub2e4. \ub2e4\ub978 \ud3b8\uc9c0\uc5d0\uc11c\ub294 \u201c\uc5b4\uc81c \uc2dc\ud5d8\uc744 \ubcf4\uc558\ub294\ub370 87\uc810\uc744 \ub9de\uc558\ub2e4. \ub2e4\ub978 \ud559\uc0dd\ubcf4\ub2e4 16\uc810 \ub354 \ub192\uc9c0\ub9cc \ub9cc\uc810\uc778 100\uc810\ubcf4\ub2e4 13\uc810 \ub0ae\uc740 \uc810\uc218\u201d\ub77c\uace0 \ub9d0\ud558\uace0 \uc788\ub2e4. \uc720\uae38\uc900\uc774 \uc720\ud559 \uc911\uc774\ub358 1884\ub144 12\uc6d4 4\uc77c \uc870\uc120\uc5d0\uc11c\ub294 \uac11\uc2e0\uc815\ubcc0\uc774 \uc77c\uc5b4\ub09c\ub2e4. \uc774\ud6c4 \uadf8\ub294 \ud559\uc5c5\uc744 \uacc4\uc18d\ud558\uc5ec 1885\ub144 \ub354\uba38 \ud559\uc6d0\uc744 \uc878\uc5c5\ud558\uace0 \ubcf4\uc2a4\ud134 \ub300\ud559\uad50\uc5d0 \uc785\ud559\ud558\uc600\ub2e4.", "answer": "1884\ub144 12\uc6d4 4\uc77c", "sentence": "\uc720\uae38\uc900\uc774 \uc720\ud559 \uc911\uc774\ub358 1884\ub144 12\uc6d4 4\uc77c \uc870\uc120\uc5d0\uc11c\ub294 \uac11\uc2e0\uc815\ubcc0\uc774 \uc77c\uc5b4\ub09c\ub2e4.", "paragraph_sentence": "\uc790\uae30\ubcf4\ub2e4 \uc5f4 \uc0b4 \uc5b4\ub9b0 \ud559\uc0dd\ub4e4\uacfc \ud568\uaed8 \uc601\uc5b4\u00b7\uc218\ud559\u00b7\uc9c0\ub9ac\u00b7\ub77c\ud2f4\uc5b4 \ub4f1\uc744 \uacf5\ubd80\ud558\ub294 \uac8c \uc27d\uc9c0 \uc54a\uc558\uc9c0\ub9cc \ud559\uc2b5 \ub2a5\ub825\uc740 \ub6f0\uc5b4\ub0ac\ub2e4. \uc11d \ub2ec \ub9cc\uc5d0 \ub3c5\ud559\uc73c\ub85c \uc601\uc5b4\ud68c\ud654\uac00 \uac00\ub2a5\ud588\ub2e4\uace0 \ud55c\ub2e4. \uc720\uae38\uc900\uc758 \ud6c4\uacac\uc778\uc774\ub77c\uace0 \ud560 \ubaa8\uc2a4 \uad50\uc218\uc640\uc758 \ud3b8\uc9c0\uc5d0\uc11c\ub294 \ud559\uad50\uc0dd\ud65c\uc5d0 \uad00\ud55c \ub0b4\uc6a9\uc774 \ub9ce\uc774 \ub098\uc628\ub2e4. \uc720\uae38\uc900\uc740 1884\ub144 11\uc6d4 3\uc77c \ud3b8\uc9c0\uc5d0\uc11c \u201c\ud654\uc0b0\u00b7\uc9c0\uc9c4\u00b7\uac04\ud5d0\ucc9c \ub4f1\uc5d0 \uad00\ud55c \uc9c0\uad6c\uacfc\ud559 \uc2dc\ud5d8\uc5d0\uc11c 94\uc810, \uc218\ud559 \uc2dc\ud5d8\uc5d0\uc11c 100\uc810\uc744 \ub9de\uc544 \ub9e4\uc6b0 \uae30\uc058\ub2e4\u201d\uace0 \ub9d0\ud588\ub2e4. \ub2e4\ub978 \ud3b8\uc9c0\uc5d0\uc11c\ub294 \u201c\uc5b4\uc81c \uc2dc\ud5d8\uc744 \ubcf4\uc558\ub294\ub370 87\uc810\uc744 \ub9de\uc558\ub2e4. \ub2e4\ub978 \ud559\uc0dd\ubcf4\ub2e4 16\uc810 \ub354 \ub192\uc9c0\ub9cc \ub9cc\uc810\uc778 100\uc810\ubcf4\ub2e4 13\uc810 \ub0ae\uc740 \uc810\uc218\u201d\ub77c\uace0 \ub9d0\ud558\uace0 \uc788\ub2e4. \uc720\uae38\uc900\uc774 \uc720\ud559 \uc911\uc774\ub358 1884\ub144 12\uc6d4 4\uc77c \uc870\uc120\uc5d0\uc11c\ub294 \uac11\uc2e0\uc815\ubcc0\uc774 \uc77c\uc5b4\ub09c\ub2e4. \uc774\ud6c4 \uadf8\ub294 \ud559\uc5c5\uc744 \uacc4\uc18d\ud558\uc5ec 1885\ub144 \ub354\uba38 \ud559\uc6d0\uc744 \uc878\uc5c5\ud558\uace0 \ubcf4\uc2a4\ud134 \ub300\ud559\uad50\uc5d0 \uc785\ud559\ud558\uc600\ub2e4.", "paragraph_answer": "\uc790\uae30\ubcf4\ub2e4 \uc5f4 \uc0b4 \uc5b4\ub9b0 \ud559\uc0dd\ub4e4\uacfc \ud568\uaed8 \uc601\uc5b4\u00b7\uc218\ud559\u00b7\uc9c0\ub9ac\u00b7\ub77c\ud2f4\uc5b4 \ub4f1\uc744 \uacf5\ubd80\ud558\ub294 \uac8c \uc27d\uc9c0 \uc54a\uc558\uc9c0\ub9cc \ud559\uc2b5 \ub2a5\ub825\uc740 \ub6f0\uc5b4\ub0ac\ub2e4. \uc11d \ub2ec \ub9cc\uc5d0 \ub3c5\ud559\uc73c\ub85c \uc601\uc5b4\ud68c\ud654\uac00 \uac00\ub2a5\ud588\ub2e4\uace0 \ud55c\ub2e4. \uc720\uae38\uc900\uc758 \ud6c4\uacac\uc778\uc774\ub77c\uace0 \ud560 \ubaa8\uc2a4 \uad50\uc218\uc640\uc758 \ud3b8\uc9c0\uc5d0\uc11c\ub294 \ud559\uad50\uc0dd\ud65c\uc5d0 \uad00\ud55c \ub0b4\uc6a9\uc774 \ub9ce\uc774 \ub098\uc628\ub2e4. \uc720\uae38\uc900\uc740 1884\ub144 11\uc6d4 3\uc77c \ud3b8\uc9c0\uc5d0\uc11c \u201c\ud654\uc0b0\u00b7\uc9c0\uc9c4\u00b7\uac04\ud5d0\ucc9c \ub4f1\uc5d0 \uad00\ud55c \uc9c0\uad6c\uacfc\ud559 \uc2dc\ud5d8\uc5d0\uc11c 94\uc810, \uc218\ud559 \uc2dc\ud5d8\uc5d0\uc11c 100\uc810\uc744 \ub9de\uc544 \ub9e4\uc6b0 \uae30\uc058\ub2e4\u201d\uace0 \ub9d0\ud588\ub2e4. \ub2e4\ub978 \ud3b8\uc9c0\uc5d0\uc11c\ub294 \u201c\uc5b4\uc81c \uc2dc\ud5d8\uc744 \ubcf4\uc558\ub294\ub370 87\uc810\uc744 \ub9de\uc558\ub2e4. \ub2e4\ub978 \ud559\uc0dd\ubcf4\ub2e4 16\uc810 \ub354 \ub192\uc9c0\ub9cc \ub9cc\uc810\uc778 100\uc810\ubcf4\ub2e4 13\uc810 \ub0ae\uc740 \uc810\uc218\u201d\ub77c\uace0 \ub9d0\ud558\uace0 \uc788\ub2e4. \uc720\uae38\uc900\uc774 \uc720\ud559 \uc911\uc774\ub358 1884\ub144 12\uc6d4 4\uc77c \uc870\uc120\uc5d0\uc11c\ub294 \uac11\uc2e0\uc815\ubcc0\uc774 \uc77c\uc5b4\ub09c\ub2e4. \uc774\ud6c4 \uadf8\ub294 \ud559\uc5c5\uc744 \uacc4\uc18d\ud558\uc5ec 1885\ub144 \ub354\uba38 \ud559\uc6d0\uc744 \uc878\uc5c5\ud558\uace0 \ubcf4\uc2a4\ud134 \ub300\ud559\uad50\uc5d0 \uc785\ud559\ud558\uc600\ub2e4.", "sentence_answer": "\uc720\uae38\uc900\uc774 \uc720\ud559 \uc911\uc774\ub358 1884\ub144 12\uc6d4 4\uc77c \uc870\uc120\uc5d0\uc11c\ub294 \uac11\uc2e0\uc815\ubcc0\uc774 \uc77c\uc5b4\ub09c\ub2e4.", "paragraph_id": "6542752-11-0", "paragraph_question": "question: \uac11\uc2e0\uc815\ubcc0\uc774 \uc77c\uc5b4\ub09c \ub0a0\uc740?, context: \uc790\uae30\ubcf4\ub2e4 \uc5f4 \uc0b4 \uc5b4\ub9b0 \ud559\uc0dd\ub4e4\uacfc \ud568\uaed8 \uc601\uc5b4\u00b7\uc218\ud559\u00b7\uc9c0\ub9ac\u00b7\ub77c\ud2f4\uc5b4 \ub4f1\uc744 \uacf5\ubd80\ud558\ub294 \uac8c \uc27d\uc9c0 \uc54a\uc558\uc9c0\ub9cc \ud559\uc2b5 \ub2a5\ub825\uc740 \ub6f0\uc5b4\ub0ac\ub2e4. \uc11d \ub2ec \ub9cc\uc5d0 \ub3c5\ud559\uc73c\ub85c \uc601\uc5b4\ud68c\ud654\uac00 \uac00\ub2a5\ud588\ub2e4\uace0 \ud55c\ub2e4. \uc720\uae38\uc900\uc758 \ud6c4\uacac\uc778\uc774\ub77c\uace0 \ud560 \ubaa8\uc2a4 \uad50\uc218\uc640\uc758 \ud3b8\uc9c0\uc5d0\uc11c\ub294 \ud559\uad50\uc0dd\ud65c\uc5d0 \uad00\ud55c \ub0b4\uc6a9\uc774 \ub9ce\uc774 \ub098\uc628\ub2e4. \uc720\uae38\uc900\uc740 1884\ub144 11\uc6d4 3\uc77c \ud3b8\uc9c0\uc5d0\uc11c \u201c\ud654\uc0b0\u00b7\uc9c0\uc9c4\u00b7\uac04\ud5d0\ucc9c \ub4f1\uc5d0 \uad00\ud55c \uc9c0\uad6c\uacfc\ud559 \uc2dc\ud5d8\uc5d0\uc11c 94\uc810, \uc218\ud559 \uc2dc\ud5d8\uc5d0\uc11c 100\uc810\uc744 \ub9de\uc544 \ub9e4\uc6b0 \uae30\uc058\ub2e4\u201d\uace0 \ub9d0\ud588\ub2e4. \ub2e4\ub978 \ud3b8\uc9c0\uc5d0\uc11c\ub294 \u201c\uc5b4\uc81c \uc2dc\ud5d8\uc744 \ubcf4\uc558\ub294\ub370 87\uc810\uc744 \ub9de\uc558\ub2e4. \ub2e4\ub978 \ud559\uc0dd\ubcf4\ub2e4 16\uc810 \ub354 \ub192\uc9c0\ub9cc \ub9cc\uc810\uc778 100\uc810\ubcf4\ub2e4 13\uc810 \ub0ae\uc740 \uc810\uc218\u201d\ub77c\uace0 \ub9d0\ud558\uace0 \uc788\ub2e4. \uc720\uae38\uc900\uc774 \uc720\ud559 \uc911\uc774\ub358 1884\ub144 12\uc6d4 4\uc77c \uc870\uc120\uc5d0\uc11c\ub294 \uac11\uc2e0\uc815\ubcc0\uc774 \uc77c\uc5b4\ub09c\ub2e4. \uc774\ud6c4 \uadf8\ub294 \ud559\uc5c5\uc744 \uacc4\uc18d\ud558\uc5ec 1885\ub144 \ub354\uba38 \ud559\uc6d0\uc744 \uc878\uc5c5\ud558\uace0 \ubcf4\uc2a4\ud134 \ub300\ud559\uad50\uc5d0 \uc785\ud559\ud558\uc600\ub2e4."} {"question": "\ub2f9\uc2dc \uc9c4\uacbd\uc900 \uc804 \uac80\uc0ac\uc7a5\uc5d0\uac8c \ub1cc\ubb3c\uc744 \uc900 \ub125\uc13c \ub300\ud45c\uc758 \uc774\ub984\uc740?", "paragraph": "\uc11c\uc6b8\uc911\uc559\uc9c0\ubc95 \ud615\uc0ac\ud569\uc75827\ubd80(\uc7ac\ud310\uc7a5 \uae40\uc9c4\ub3d9 \ubd80\uc7a5\ud310\uc0ac)\ub294 2016\ub144 12\uc6d4 13\uc77c \u2018\ubc95\uc870 \uac8c\uc774\ud2b8\u2019\uc5d0 \uc5f0\ub8e8\ub41c \uc9c4\uacbd\uc900 \uc804 \uac80\uc0ac\uc7a5(49)\uc5d0 \ub300\ud574 \ub125\uc2a8 \ucc3d\uc5c5\uc8fc\uc778 \uae40\uc815\uc8fc NXC \ub300\ud45c(48)\ub85c\ubd80\ud130 \ub125\uc2a8 \uacf5\uc9dc\uc8fc\uc2dd 1\ub9cc\uc8fc, \uc81c\ub124\uc2dc\uc2a4 \ucc28\ub7c9, \uc5ec\ud589\uacbd\ube44 5000\uc5ec\ub9cc\uc6d0 \ub4f1 \ucd1d 9\uc5b5\uc6d0\uc758 \uae08\ud488\uc744 \ubc1b\uc740 \ud610\uc758(\ud2b9\uc815\ubc94\uc8c4\uac00\uc911\ucc98\ubc8c\ubc95\uc0c1 \ub1cc\ubb3c) \ubd80\ubd84\uc744 \ubb34\uc8c4\ub77c\uace0 \ud558\uba74\uc11c \uc11c\uc6a9\uc6d0 \ud55c\uc9c4 \ub300\ud45c(67)\uac00 \ub300\ud55c\ud56d\uacf5 \ub300\ud45c\uc774\uc0ac\uc774\ub358 2010\ub144 \uc9c4\uacbd\uc900 \ucc98\ub0a8\uc758 \uccad\uc18c\ud68c\uc0ac\uc640 \ub300\ud55c\ud56d\uacf5\uc774 \uc6a9\uc5ed\uacc4\uc57d \ub9fa\ub3c4\ub85d \ud55c \ubd80\ubd84\uc5d0 \ub300\ud574\uc11c\ub9cc \ud610\uc758\ub97c \uc778\uc815\ud574 \uc9d5\uc5ed 4\ub144\uc744 \uc120\uace0\ud558\uba74\uc11c \"\uae40\uc815\uc8fc \ub125\uc13c \ub300\ud45c\uac00 \uae08\ud488\uc744 \uc900 \uc2dc\uc810\uacfc \uae40 \uc804 \ub300\ud45c\u00b7\ub125\uc2a8\uc758 \ud604\uc548, \uace0\ub4f1\ud559\uad50 \ub54c\ubd80\ud130 \uc54c\uace0 \uc9c0\ub0b8 \u2018\uc9c0\uc74c(\u77e5\u97f3)\u2019\uc758 \uad00\uacc4\uc600\uace0, \ud3b8\uc758\ub97c \ubd10\uc900 \uc0ac\uac74\uc774 \ud2b9\uc815\ub418\uc9c0 \uc54a\uc558\ub294\ub370 \ub2e8\uc9c0 \uc9c4\uacbd\uc900\uc774 \uac80\uc0ac\ub77c\ub294 \uc2e0\ubd84\uc744 \uac00\uc84c\ub2e4\ub294 \uc774\uc720\ub85c \uad11\ubc94\uc704\ud558\uac8c \uc9c1\ubb34\uc640\uc758 \uc5f0\uad00\uc131\uc774 \uc788\ub2e4\uace0 \ud310\ub2e8\ub418\uc9c0 \uc54a\ub294\ub2e4\"\uace0 \ud588\ub2e4. \uac80\ucc30\uc774 \"\uae40\uc815\uc8fc \ub300\ud45c\uac00 \uc7a5\ub798 \uc9c4\uacbd\uc900\uc73c\ub85c\ubd80\ud130 \ub3c4\uc6c0\uc744 \ubc1b\uc744 \uc218 \uc788\ub294 \ubd80\ubd84\ub3c4 \uace0\ub824\ud558\uace0 \uc900 \uae08\ud488\uc774\ub77c \ub300\uac00\uc131\uc774 \uc788\uc5c8\ub2e4\"\uace0 \ud55c \uac83\uc5d0 \ub300\ud574\uc11c\ub3c4 \"\uae40\uc815\uc8fc \ub300\ud45c\uac00 \uae08\ud488\uc744 \uc81c\uacf5\ud55c \ub4a4 \uc9c4\uacbd\uc900\uc774 \uc2e4\uc81c \uae40\uc815\uc8fc\uc5d0\uac8c \ub3c4\uc6c0\uc744 \uc900 \uac1c\uc5f0\uc131\uc774 \uc5c6\ub2e4\"\uace0 \ud588\ub2e4.", "answer": "\uae40\uc815\uc8fc", "sentence": "\uc11c\uc6b8\uc911\uc559\uc9c0\ubc95 \ud615\uc0ac\ud569\uc75827\ubd80(\uc7ac\ud310\uc7a5 \uae40\uc9c4\ub3d9 \ubd80\uc7a5\ud310\uc0ac)\ub294 2016\ub144 12\uc6d4 13\uc77c \u2018\ubc95\uc870 \uac8c\uc774\ud2b8\u2019\uc5d0 \uc5f0\ub8e8\ub41c \uc9c4\uacbd\uc900 \uc804 \uac80\uc0ac\uc7a5(49)\uc5d0 \ub300\ud574 \ub125\uc2a8 \ucc3d\uc5c5\uc8fc\uc778 \uae40\uc815\uc8fc NXC \ub300\ud45c(48)\ub85c\ubd80\ud130 \ub125\uc2a8 \uacf5\uc9dc\uc8fc\uc2dd 1\ub9cc\uc8fc, \uc81c\ub124\uc2dc\uc2a4 \ucc28\ub7c9, \uc5ec\ud589\uacbd\ube44 5000\uc5ec\ub9cc\uc6d0 \ub4f1 \ucd1d 9\uc5b5\uc6d0\uc758 \uae08\ud488\uc744 \ubc1b\uc740 \ud610\uc758(\ud2b9\uc815\ubc94\uc8c4\uac00\uc911\ucc98\ubc8c\ubc95\uc0c1 \ub1cc\ubb3c) \ubd80\ubd84\uc744 \ubb34\uc8c4\ub77c\uace0 \ud558\uba74\uc11c \uc11c\uc6a9\uc6d0 \ud55c\uc9c4 \ub300\ud45c(67)\uac00 \ub300\ud55c\ud56d\uacf5 \ub300\ud45c\uc774\uc0ac\uc774\ub358 2010\ub144 \uc9c4\uacbd\uc900 \ucc98\ub0a8\uc758 \uccad\uc18c\ud68c\uc0ac\uc640 \ub300\ud55c\ud56d\uacf5\uc774 \uc6a9\uc5ed\uacc4\uc57d \ub9fa\ub3c4\ub85d \ud55c \ubd80\ubd84\uc5d0 \ub300\ud574\uc11c\ub9cc \ud610\uc758\ub97c \uc778\uc815\ud574 \uc9d5\uc5ed 4\ub144\uc744 \uc120\uace0\ud558\uba74\uc11c \"\uae40\uc815\uc8fc \ub125\uc13c \ub300\ud45c\uac00 \uae08\ud488\uc744 \uc900 \uc2dc\uc810\uacfc \uae40 \uc804 \ub300\ud45c\u00b7\ub125\uc2a8\uc758 \ud604\uc548, \uace0\ub4f1\ud559\uad50 \ub54c\ubd80\ud130 \uc54c\uace0 \uc9c0\ub0b8 \u2018\uc9c0\uc74c(\u77e5\u97f3)\u2019\uc758 \uad00\uacc4\uc600\uace0, \ud3b8\uc758\ub97c \ubd10\uc900 \uc0ac\uac74\uc774 \ud2b9\uc815\ub418\uc9c0 \uc54a\uc558\ub294\ub370 \ub2e8\uc9c0 \uc9c4\uacbd\uc900\uc774 \uac80\uc0ac\ub77c\ub294 \uc2e0\ubd84\uc744 \uac00\uc84c\ub2e4\ub294 \uc774\uc720\ub85c \uad11\ubc94\uc704\ud558\uac8c \uc9c1\ubb34\uc640\uc758 \uc5f0\uad00\uc131\uc774 \uc788\ub2e4\uace0 \ud310\ub2e8\ub418\uc9c0 \uc54a\ub294\ub2e4\"\uace0 \ud588\ub2e4.", "paragraph_sentence": " \uc11c\uc6b8\uc911\uc559\uc9c0\ubc95 \ud615\uc0ac\ud569\uc75827\ubd80(\uc7ac\ud310\uc7a5 \uae40\uc9c4\ub3d9 \ubd80\uc7a5\ud310\uc0ac)\ub294 2016\ub144 12\uc6d4 13\uc77c \u2018\ubc95\uc870 \uac8c\uc774\ud2b8\u2019\uc5d0 \uc5f0\ub8e8\ub41c \uc9c4\uacbd\uc900 \uc804 \uac80\uc0ac\uc7a5(49)\uc5d0 \ub300\ud574 \ub125\uc2a8 \ucc3d\uc5c5\uc8fc\uc778 \uae40\uc815\uc8fc NXC \ub300\ud45c(48)\ub85c\ubd80\ud130 \ub125\uc2a8 \uacf5\uc9dc\uc8fc\uc2dd 1\ub9cc\uc8fc, \uc81c\ub124\uc2dc\uc2a4 \ucc28\ub7c9, \uc5ec\ud589\uacbd\ube44 5000\uc5ec\ub9cc\uc6d0 \ub4f1 \ucd1d 9\uc5b5\uc6d0\uc758 \uae08\ud488\uc744 \ubc1b\uc740 \ud610\uc758(\ud2b9\uc815\ubc94\uc8c4\uac00\uc911\ucc98\ubc8c\ubc95\uc0c1 \ub1cc\ubb3c) \ubd80\ubd84\uc744 \ubb34\uc8c4\ub77c\uace0 \ud558\uba74\uc11c \uc11c\uc6a9\uc6d0 \ud55c\uc9c4 \ub300\ud45c(67)\uac00 \ub300\ud55c\ud56d\uacf5 \ub300\ud45c\uc774\uc0ac\uc774\ub358 2010\ub144 \uc9c4\uacbd\uc900 \ucc98\ub0a8\uc758 \uccad\uc18c\ud68c\uc0ac\uc640 \ub300\ud55c\ud56d\uacf5\uc774 \uc6a9\uc5ed\uacc4\uc57d \ub9fa\ub3c4\ub85d \ud55c \ubd80\ubd84\uc5d0 \ub300\ud574\uc11c\ub9cc \ud610\uc758\ub97c \uc778\uc815\ud574 \uc9d5\uc5ed 4\ub144\uc744 \uc120\uace0\ud558\uba74\uc11c \"\uae40\uc815\uc8fc \ub125\uc13c \ub300\ud45c\uac00 \uae08\ud488\uc744 \uc900 \uc2dc\uc810\uacfc \uae40 \uc804 \ub300\ud45c\u00b7\ub125\uc2a8\uc758 \ud604\uc548, \uace0\ub4f1\ud559\uad50 \ub54c\ubd80\ud130 \uc54c\uace0 \uc9c0\ub0b8 \u2018\uc9c0\uc74c(\u77e5\u97f3)\u2019\uc758 \uad00\uacc4\uc600\uace0, \ud3b8\uc758\ub97c \ubd10\uc900 \uc0ac\uac74\uc774 \ud2b9\uc815\ub418\uc9c0 \uc54a\uc558\ub294\ub370 \ub2e8\uc9c0 \uc9c4\uacbd\uc900\uc774 \uac80\uc0ac\ub77c\ub294 \uc2e0\ubd84\uc744 \uac00\uc84c\ub2e4\ub294 \uc774\uc720\ub85c \uad11\ubc94\uc704\ud558\uac8c \uc9c1\ubb34\uc640\uc758 \uc5f0\uad00\uc131\uc774 \uc788\ub2e4\uace0 \ud310\ub2e8\ub418\uc9c0 \uc54a\ub294\ub2e4\"\uace0 \ud588\ub2e4. \uac80\ucc30\uc774 \"\uae40\uc815\uc8fc \ub300\ud45c\uac00 \uc7a5\ub798 \uc9c4\uacbd\uc900\uc73c\ub85c\ubd80\ud130 \ub3c4\uc6c0\uc744 \ubc1b\uc744 \uc218 \uc788\ub294 \ubd80\ubd84\ub3c4 \uace0\ub824\ud558\uace0 \uc900 \uae08\ud488\uc774\ub77c \ub300\uac00\uc131\uc774 \uc788\uc5c8\ub2e4\"\uace0 \ud55c \uac83\uc5d0 \ub300\ud574\uc11c\ub3c4 \"\uae40\uc815\uc8fc \ub300\ud45c\uac00 \uae08\ud488\uc744 \uc81c\uacf5\ud55c \ub4a4 \uc9c4\uacbd\uc900\uc774 \uc2e4\uc81c \uae40\uc815\uc8fc\uc5d0\uac8c \ub3c4\uc6c0\uc744 \uc900 \uac1c\uc5f0\uc131\uc774 \uc5c6\ub2e4\"\uace0 \ud588\ub2e4.", "paragraph_answer": "\uc11c\uc6b8\uc911\uc559\uc9c0\ubc95 \ud615\uc0ac\ud569\uc75827\ubd80(\uc7ac\ud310\uc7a5 \uae40\uc9c4\ub3d9 \ubd80\uc7a5\ud310\uc0ac)\ub294 2016\ub144 12\uc6d4 13\uc77c \u2018\ubc95\uc870 \uac8c\uc774\ud2b8\u2019\uc5d0 \uc5f0\ub8e8\ub41c \uc9c4\uacbd\uc900 \uc804 \uac80\uc0ac\uc7a5(49)\uc5d0 \ub300\ud574 \ub125\uc2a8 \ucc3d\uc5c5\uc8fc\uc778 \uae40\uc815\uc8fc NXC \ub300\ud45c(48)\ub85c\ubd80\ud130 \ub125\uc2a8 \uacf5\uc9dc\uc8fc\uc2dd 1\ub9cc\uc8fc, \uc81c\ub124\uc2dc\uc2a4 \ucc28\ub7c9, \uc5ec\ud589\uacbd\ube44 5000\uc5ec\ub9cc\uc6d0 \ub4f1 \ucd1d 9\uc5b5\uc6d0\uc758 \uae08\ud488\uc744 \ubc1b\uc740 \ud610\uc758(\ud2b9\uc815\ubc94\uc8c4\uac00\uc911\ucc98\ubc8c\ubc95\uc0c1 \ub1cc\ubb3c) \ubd80\ubd84\uc744 \ubb34\uc8c4\ub77c\uace0 \ud558\uba74\uc11c \uc11c\uc6a9\uc6d0 \ud55c\uc9c4 \ub300\ud45c(67)\uac00 \ub300\ud55c\ud56d\uacf5 \ub300\ud45c\uc774\uc0ac\uc774\ub358 2010\ub144 \uc9c4\uacbd\uc900 \ucc98\ub0a8\uc758 \uccad\uc18c\ud68c\uc0ac\uc640 \ub300\ud55c\ud56d\uacf5\uc774 \uc6a9\uc5ed\uacc4\uc57d \ub9fa\ub3c4\ub85d \ud55c \ubd80\ubd84\uc5d0 \ub300\ud574\uc11c\ub9cc \ud610\uc758\ub97c \uc778\uc815\ud574 \uc9d5\uc5ed 4\ub144\uc744 \uc120\uace0\ud558\uba74\uc11c \"\uae40\uc815\uc8fc \ub125\uc13c \ub300\ud45c\uac00 \uae08\ud488\uc744 \uc900 \uc2dc\uc810\uacfc \uae40 \uc804 \ub300\ud45c\u00b7\ub125\uc2a8\uc758 \ud604\uc548, \uace0\ub4f1\ud559\uad50 \ub54c\ubd80\ud130 \uc54c\uace0 \uc9c0\ub0b8 \u2018\uc9c0\uc74c(\u77e5\u97f3)\u2019\uc758 \uad00\uacc4\uc600\uace0, \ud3b8\uc758\ub97c \ubd10\uc900 \uc0ac\uac74\uc774 \ud2b9\uc815\ub418\uc9c0 \uc54a\uc558\ub294\ub370 \ub2e8\uc9c0 \uc9c4\uacbd\uc900\uc774 \uac80\uc0ac\ub77c\ub294 \uc2e0\ubd84\uc744 \uac00\uc84c\ub2e4\ub294 \uc774\uc720\ub85c \uad11\ubc94\uc704\ud558\uac8c \uc9c1\ubb34\uc640\uc758 \uc5f0\uad00\uc131\uc774 \uc788\ub2e4\uace0 \ud310\ub2e8\ub418\uc9c0 \uc54a\ub294\ub2e4\"\uace0 \ud588\ub2e4. \uac80\ucc30\uc774 \"\uae40\uc815\uc8fc \ub300\ud45c\uac00 \uc7a5\ub798 \uc9c4\uacbd\uc900\uc73c\ub85c\ubd80\ud130 \ub3c4\uc6c0\uc744 \ubc1b\uc744 \uc218 \uc788\ub294 \ubd80\ubd84\ub3c4 \uace0\ub824\ud558\uace0 \uc900 \uae08\ud488\uc774\ub77c \ub300\uac00\uc131\uc774 \uc788\uc5c8\ub2e4\"\uace0 \ud55c \uac83\uc5d0 \ub300\ud574\uc11c\ub3c4 \"\uae40\uc815\uc8fc \ub300\ud45c\uac00 \uae08\ud488\uc744 \uc81c\uacf5\ud55c \ub4a4 \uc9c4\uacbd\uc900\uc774 \uc2e4\uc81c \uae40\uc815\uc8fc\uc5d0\uac8c \ub3c4\uc6c0\uc744 \uc900 \uac1c\uc5f0\uc131\uc774 \uc5c6\ub2e4\"\uace0 \ud588\ub2e4.", "sentence_answer": "\uc11c\uc6b8\uc911\uc559\uc9c0\ubc95 \ud615\uc0ac\ud569\uc75827\ubd80(\uc7ac\ud310\uc7a5 \uae40\uc9c4\ub3d9 \ubd80\uc7a5\ud310\uc0ac)\ub294 2016\ub144 12\uc6d4 13\uc77c \u2018\ubc95\uc870 \uac8c\uc774\ud2b8\u2019\uc5d0 \uc5f0\ub8e8\ub41c \uc9c4\uacbd\uc900 \uc804 \uac80\uc0ac\uc7a5(49)\uc5d0 \ub300\ud574 \ub125\uc2a8 \ucc3d\uc5c5\uc8fc\uc778 \uae40\uc815\uc8fc NXC \ub300\ud45c(48)\ub85c\ubd80\ud130 \ub125\uc2a8 \uacf5\uc9dc\uc8fc\uc2dd 1\ub9cc\uc8fc, \uc81c\ub124\uc2dc\uc2a4 \ucc28\ub7c9, \uc5ec\ud589\uacbd\ube44 5000\uc5ec\ub9cc\uc6d0 \ub4f1 \ucd1d 9\uc5b5\uc6d0\uc758 \uae08\ud488\uc744 \ubc1b\uc740 \ud610\uc758(\ud2b9\uc815\ubc94\uc8c4\uac00\uc911\ucc98\ubc8c\ubc95\uc0c1 \ub1cc\ubb3c) \ubd80\ubd84\uc744 \ubb34\uc8c4\ub77c\uace0 \ud558\uba74\uc11c \uc11c\uc6a9\uc6d0 \ud55c\uc9c4 \ub300\ud45c(67)\uac00 \ub300\ud55c\ud56d\uacf5 \ub300\ud45c\uc774\uc0ac\uc774\ub358 2010\ub144 \uc9c4\uacbd\uc900 \ucc98\ub0a8\uc758 \uccad\uc18c\ud68c\uc0ac\uc640 \ub300\ud55c\ud56d\uacf5\uc774 \uc6a9\uc5ed\uacc4\uc57d \ub9fa\ub3c4\ub85d \ud55c \ubd80\ubd84\uc5d0 \ub300\ud574\uc11c\ub9cc \ud610\uc758\ub97c \uc778\uc815\ud574 \uc9d5\uc5ed 4\ub144\uc744 \uc120\uace0\ud558\uba74\uc11c \"\uae40\uc815\uc8fc \ub125\uc13c \ub300\ud45c\uac00 \uae08\ud488\uc744 \uc900 \uc2dc\uc810\uacfc \uae40 \uc804 \ub300\ud45c\u00b7\ub125\uc2a8\uc758 \ud604\uc548, \uace0\ub4f1\ud559\uad50 \ub54c\ubd80\ud130 \uc54c\uace0 \uc9c0\ub0b8 \u2018\uc9c0\uc74c(\u77e5\u97f3)\u2019\uc758 \uad00\uacc4\uc600\uace0, \ud3b8\uc758\ub97c \ubd10\uc900 \uc0ac\uac74\uc774 \ud2b9\uc815\ub418\uc9c0 \uc54a\uc558\ub294\ub370 \ub2e8\uc9c0 \uc9c4\uacbd\uc900\uc774 \uac80\uc0ac\ub77c\ub294 \uc2e0\ubd84\uc744 \uac00\uc84c\ub2e4\ub294 \uc774\uc720\ub85c \uad11\ubc94\uc704\ud558\uac8c \uc9c1\ubb34\uc640\uc758 \uc5f0\uad00\uc131\uc774 \uc788\ub2e4\uace0 \ud310\ub2e8\ub418\uc9c0 \uc54a\ub294\ub2e4\"\uace0 \ud588\ub2e4.", "paragraph_id": "6557223-1-2", "paragraph_question": "question: \ub2f9\uc2dc \uc9c4\uacbd\uc900 \uc804 \uac80\uc0ac\uc7a5\uc5d0\uac8c \ub1cc\ubb3c\uc744 \uc900 \ub125\uc13c \ub300\ud45c\uc758 \uc774\ub984\uc740?, context: \uc11c\uc6b8\uc911\uc559\uc9c0\ubc95 \ud615\uc0ac\ud569\uc75827\ubd80(\uc7ac\ud310\uc7a5 \uae40\uc9c4\ub3d9 \ubd80\uc7a5\ud310\uc0ac)\ub294 2016\ub144 12\uc6d4 13\uc77c \u2018\ubc95\uc870 \uac8c\uc774\ud2b8\u2019\uc5d0 \uc5f0\ub8e8\ub41c \uc9c4\uacbd\uc900 \uc804 \uac80\uc0ac\uc7a5(49)\uc5d0 \ub300\ud574 \ub125\uc2a8 \ucc3d\uc5c5\uc8fc\uc778 \uae40\uc815\uc8fc NXC \ub300\ud45c(48)\ub85c\ubd80\ud130 \ub125\uc2a8 \uacf5\uc9dc\uc8fc\uc2dd 1\ub9cc\uc8fc, \uc81c\ub124\uc2dc\uc2a4 \ucc28\ub7c9, \uc5ec\ud589\uacbd\ube44 5000\uc5ec\ub9cc\uc6d0 \ub4f1 \ucd1d 9\uc5b5\uc6d0\uc758 \uae08\ud488\uc744 \ubc1b\uc740 \ud610\uc758(\ud2b9\uc815\ubc94\uc8c4\uac00\uc911\ucc98\ubc8c\ubc95\uc0c1 \ub1cc\ubb3c) \ubd80\ubd84\uc744 \ubb34\uc8c4\ub77c\uace0 \ud558\uba74\uc11c \uc11c\uc6a9\uc6d0 \ud55c\uc9c4 \ub300\ud45c(67)\uac00 \ub300\ud55c\ud56d\uacf5 \ub300\ud45c\uc774\uc0ac\uc774\ub358 2010\ub144 \uc9c4\uacbd\uc900 \ucc98\ub0a8\uc758 \uccad\uc18c\ud68c\uc0ac\uc640 \ub300\ud55c\ud56d\uacf5\uc774 \uc6a9\uc5ed\uacc4\uc57d \ub9fa\ub3c4\ub85d \ud55c \ubd80\ubd84\uc5d0 \ub300\ud574\uc11c\ub9cc \ud610\uc758\ub97c \uc778\uc815\ud574 \uc9d5\uc5ed 4\ub144\uc744 \uc120\uace0\ud558\uba74\uc11c \"\uae40\uc815\uc8fc \ub125\uc13c \ub300\ud45c\uac00 \uae08\ud488\uc744 \uc900 \uc2dc\uc810\uacfc \uae40 \uc804 \ub300\ud45c\u00b7\ub125\uc2a8\uc758 \ud604\uc548, \uace0\ub4f1\ud559\uad50 \ub54c\ubd80\ud130 \uc54c\uace0 \uc9c0\ub0b8 \u2018\uc9c0\uc74c(\u77e5\u97f3)\u2019\uc758 \uad00\uacc4\uc600\uace0, \ud3b8\uc758\ub97c \ubd10\uc900 \uc0ac\uac74\uc774 \ud2b9\uc815\ub418\uc9c0 \uc54a\uc558\ub294\ub370 \ub2e8\uc9c0 \uc9c4\uacbd\uc900\uc774 \uac80\uc0ac\ub77c\ub294 \uc2e0\ubd84\uc744 \uac00\uc84c\ub2e4\ub294 \uc774\uc720\ub85c \uad11\ubc94\uc704\ud558\uac8c \uc9c1\ubb34\uc640\uc758 \uc5f0\uad00\uc131\uc774 \uc788\ub2e4\uace0 \ud310\ub2e8\ub418\uc9c0 \uc54a\ub294\ub2e4\"\uace0 \ud588\ub2e4. \uac80\ucc30\uc774 \"\uae40\uc815\uc8fc \ub300\ud45c\uac00 \uc7a5\ub798 \uc9c4\uacbd\uc900\uc73c\ub85c\ubd80\ud130 \ub3c4\uc6c0\uc744 \ubc1b\uc744 \uc218 \uc788\ub294 \ubd80\ubd84\ub3c4 \uace0\ub824\ud558\uace0 \uc900 \uae08\ud488\uc774\ub77c \ub300\uac00\uc131\uc774 \uc788\uc5c8\ub2e4\"\uace0 \ud55c \uac83\uc5d0 \ub300\ud574\uc11c\ub3c4 \"\uae40\uc815\uc8fc \ub300\ud45c\uac00 \uae08\ud488\uc744 \uc81c\uacf5\ud55c \ub4a4 \uc9c4\uacbd\uc900\uc774 \uc2e4\uc81c \uae40\uc815\uc8fc\uc5d0\uac8c \ub3c4\uc6c0\uc744 \uc900 \uac1c\uc5f0\uc131\uc774 \uc5c6\ub2e4\"\uace0 \ud588\ub2e4."} {"question": "\uae40\ub300\uc911\uacfc 40\ub144 \uc9c0\uae30\uc600\ub358 \ub3c5\uc77c\uc758 \uc804\uc9c1 \ub300\ud1b5\ub839\uc740?", "paragraph": "\uc559\uac94\ub77c \uba54\ub974\ucf08 \ub3c5\uc77c \ucd1d\ub9ac\ub294 \uc870\uc804\uc744 \ud1b5\ud574 \uc560\ub3c4\ub97c \ud45c\uc2dc\ud558\uba70 \"\uae40 \uc804 \ub300\ud1b5\ub839\uc740 \ud55c\uad6d\uc758 \uad6d\uacbd\uc744 \ub118\uc5b4 \uad6d\uc81c\uc801\uc73c\ub85c \uc874\uacbd\ubc1b\ub294 \uc815\uce58\uc778\uc774\uc790 \ubbfc\uc8fc\uc8fc\uc758\uc790\uc600\ub2e4.\"\uba74\uc11c \"\ud55c\uad6d\uc758 \ubbfc\uc8fc\ud654\uc640 \uc778\uad8c, \uadf8\ub9ac\uace0 \ub3d9\ubd81\uc544\uc758 \ud3c9\ud654\uc640 \uc548\uc804\uc744 \uc704\ud55c \uae40 \uc804 \ub300\ud1b5\ub839\uc758 \uc815\uce58\uc801 \uc5c5\uc801\uc740 \uc78a\ud788\uc9c0 \uc54a\uc744 \uac83\uc774\ub2e4.\"\uc774\ub77c\uace0 \uac15\uc870\ud588\ub2e4. \uba54\ub974\ucf08 \ucd1d\ub9ac\ub294 \uc774\uc5b4 \"\ub3c5\uc77c\uacfc\uc758 \uc624\ub79c \uc778\uc5f0\uc73c\ub85c \ub3c5\uc77c\uc778\ub4e4\uc758 \ub9c8\uc74c\uc5d0 \uae4a\uc774 \uc0c8\uaca8\uc838 \uc788\ub294 \uae40 \uc804 \ub300\ud1b5\ub839\uc758 \uc11c\uac70\ub85c \ub3c5\uc77c\uc5f0\ubc29\uacf5\ud654\uad6d\uc740 \uc88b\uc740 \uce5c\uad6c\ub97c \uc783\uc5c8\ub2e4.\"\uba74\uc11c \"\uadf8\ub294 \ub300\ud1b5\ub839 \uc7ac\uc784\uae30\uac04 \uc591\uad6d\uad00\uacc4\uc758 \uc2ec\ud654\uc640 \uad50\ub958\uac15\ud654\ub97c \ud1b5\ud574 \ud6cc\ub96d\ud55c \uc5c5\uc801\uc744 \ub0a8\uacbc\ub2e4.\"\ub77c\uace0 \ud3c9\uac00\ud588\ub2e4. \uae40 \uc804 \ub300\ud1b5\ub839\uc758 40\ub144 \uc9c0\uae30\uc778 \ub9ac\ud558\ub974\ud2b8 \ud3f0 \ubc14\uc774\uce20\uccb4\ucee4 \uc804 \ub3c5\uc77c \ub300\ud1b5\ub839\uc740 \uae40 \uc804 \ub300\ud1b5\ub839\uc758 \uc0ac\ub9dd\uc5d0 '\uc4f0\ub77c\ub9b0 \uc544\ud514'\uc744 \ub290\ub080\ub2e4\uace0 \ubc1d\ud614\ub2e4. \ud3f0 \ubc14\uc774\uccb4\ucee4 \uc804 \ub300\ud1b5\ub839\uc740 \uae40 \uc804 \ub300\ud1b5\ub839\uc774 \uc0ac\ub9dd\ud588\ub2e4\ub294 \uc18c\uc2dd\uc744 \ub4e4\uc740 \ub4a4 \uc5f0\ud569\ub274\uc2a4\uc5d0 \"\ub098\uc758 \uc624\ub79c \uce5c\uad6c\uc778 \uae40\ub300\uc911 \uc804 \ub300\ud1b5\ub839\uacfc \uc4f0\ub77c\ub9b0 \uc544\ud514\uc73c\ub85c \uc791\ubcc4\ud558\uac8c \ub410\ub2e4\"\ub77c\uace0 \ub9d0\ud588\ub2e4. \uadf8\ub294 \uc774\uc5b4 \"\uae40 \uc804 \ub300\ud1b5\ub839\uc740 \ubd88\uad74\uc758 \uc6a9\uae30, \uc790\uc720 \ubbfc\uc8fc\uc8fc\uc758\ub97c \uc704\ud55c \uc0ac\ub791, \uadf8\ub9ac\uace0 \ubb34\uc5c7\ubcf4\ub2e4 \ud55c\ubbfc\uc871 \uc804\uccb4\uc758 \ud3c9\ud654\ub97c \uc704\ud55c \ud0c1\uc6d4\ud55c \uae30\uc5ec\ub97c \ud1b5\ud574 \uc6b0\ub9ac \ubaa8\ub450\uc5d0\uac8c \ubaa8\ubc94\uc774 \ub410\ub2e4\"\ub77c\uace0 \uac15\uc870\ud588\ub2e4.", "answer": "\ub9ac\ud558\ub974\ud2b8 \ud3f0 \ubc14\uc774\uce20\uccb4\ucee4", "sentence": "\uae40 \uc804 \ub300\ud1b5\ub839\uc758 40\ub144 \uc9c0\uae30\uc778 \ub9ac\ud558\ub974\ud2b8 \ud3f0 \ubc14\uc774\uce20\uccb4\ucee4 \uc804 \ub3c5\uc77c \ub300\ud1b5\ub839\uc740 \uae40 \uc804 \ub300\ud1b5\ub839\uc758 \uc0ac\ub9dd\uc5d0 '\uc4f0\ub77c\ub9b0 \uc544\ud514'\uc744 \ub290\ub080\ub2e4\uace0 \ubc1d\ud614\ub2e4.", "paragraph_sentence": "\uc559\uac94\ub77c \uba54\ub974\ucf08 \ub3c5\uc77c \ucd1d\ub9ac\ub294 \uc870\uc804\uc744 \ud1b5\ud574 \uc560\ub3c4\ub97c \ud45c\uc2dc\ud558\uba70 \"\uae40 \uc804 \ub300\ud1b5\ub839\uc740 \ud55c\uad6d\uc758 \uad6d\uacbd\uc744 \ub118\uc5b4 \uad6d\uc81c\uc801\uc73c\ub85c \uc874\uacbd\ubc1b\ub294 \uc815\uce58\uc778\uc774\uc790 \ubbfc\uc8fc\uc8fc\uc758\uc790\uc600\ub2e4. \"\uba74\uc11c \"\ud55c\uad6d\uc758 \ubbfc\uc8fc\ud654\uc640 \uc778\uad8c, \uadf8\ub9ac\uace0 \ub3d9\ubd81\uc544\uc758 \ud3c9\ud654\uc640 \uc548\uc804\uc744 \uc704\ud55c \uae40 \uc804 \ub300\ud1b5\ub839\uc758 \uc815\uce58\uc801 \uc5c5\uc801\uc740 \uc78a\ud788\uc9c0 \uc54a\uc744 \uac83\uc774\ub2e4. \"\uc774\ub77c\uace0 \uac15\uc870\ud588\ub2e4. \uba54\ub974\ucf08 \ucd1d\ub9ac\ub294 \uc774\uc5b4 \"\ub3c5\uc77c\uacfc\uc758 \uc624\ub79c \uc778\uc5f0\uc73c\ub85c \ub3c5\uc77c\uc778\ub4e4\uc758 \ub9c8\uc74c\uc5d0 \uae4a\uc774 \uc0c8\uaca8\uc838 \uc788\ub294 \uae40 \uc804 \ub300\ud1b5\ub839\uc758 \uc11c\uac70\ub85c \ub3c5\uc77c\uc5f0\ubc29\uacf5\ud654\uad6d\uc740 \uc88b\uc740 \uce5c\uad6c\ub97c \uc783\uc5c8\ub2e4. \"\uba74\uc11c \"\uadf8\ub294 \ub300\ud1b5\ub839 \uc7ac\uc784\uae30\uac04 \uc591\uad6d\uad00\uacc4\uc758 \uc2ec\ud654\uc640 \uad50\ub958\uac15\ud654\ub97c \ud1b5\ud574 \ud6cc\ub96d\ud55c \uc5c5\uc801\uc744 \ub0a8\uacbc\ub2e4. \"\ub77c\uace0 \ud3c9\uac00\ud588\ub2e4. \uae40 \uc804 \ub300\ud1b5\ub839\uc758 40\ub144 \uc9c0\uae30\uc778 \ub9ac\ud558\ub974\ud2b8 \ud3f0 \ubc14\uc774\uce20\uccb4\ucee4 \uc804 \ub3c5\uc77c \ub300\ud1b5\ub839\uc740 \uae40 \uc804 \ub300\ud1b5\ub839\uc758 \uc0ac\ub9dd\uc5d0 '\uc4f0\ub77c\ub9b0 \uc544\ud514'\uc744 \ub290\ub080\ub2e4\uace0 \ubc1d\ud614\ub2e4. \ud3f0 \ubc14\uc774\uccb4\ucee4 \uc804 \ub300\ud1b5\ub839\uc740 \uae40 \uc804 \ub300\ud1b5\ub839\uc774 \uc0ac\ub9dd\ud588\ub2e4\ub294 \uc18c\uc2dd\uc744 \ub4e4\uc740 \ub4a4 \uc5f0\ud569\ub274\uc2a4\uc5d0 \"\ub098\uc758 \uc624\ub79c \uce5c\uad6c\uc778 \uae40\ub300\uc911 \uc804 \ub300\ud1b5\ub839\uacfc \uc4f0\ub77c\ub9b0 \uc544\ud514\uc73c\ub85c \uc791\ubcc4\ud558\uac8c \ub410\ub2e4\"\ub77c\uace0 \ub9d0\ud588\ub2e4. \uadf8\ub294 \uc774\uc5b4 \"\uae40 \uc804 \ub300\ud1b5\ub839\uc740 \ubd88\uad74\uc758 \uc6a9\uae30, \uc790\uc720 \ubbfc\uc8fc\uc8fc\uc758\ub97c \uc704\ud55c \uc0ac\ub791, \uadf8\ub9ac\uace0 \ubb34\uc5c7\ubcf4\ub2e4 \ud55c\ubbfc\uc871 \uc804\uccb4\uc758 \ud3c9\ud654\ub97c \uc704\ud55c \ud0c1\uc6d4\ud55c \uae30\uc5ec\ub97c \ud1b5\ud574 \uc6b0\ub9ac \ubaa8\ub450\uc5d0\uac8c \ubaa8\ubc94\uc774 \ub410\ub2e4\"\ub77c\uace0 \uac15\uc870\ud588\ub2e4.", "paragraph_answer": "\uc559\uac94\ub77c \uba54\ub974\ucf08 \ub3c5\uc77c \ucd1d\ub9ac\ub294 \uc870\uc804\uc744 \ud1b5\ud574 \uc560\ub3c4\ub97c \ud45c\uc2dc\ud558\uba70 \"\uae40 \uc804 \ub300\ud1b5\ub839\uc740 \ud55c\uad6d\uc758 \uad6d\uacbd\uc744 \ub118\uc5b4 \uad6d\uc81c\uc801\uc73c\ub85c \uc874\uacbd\ubc1b\ub294 \uc815\uce58\uc778\uc774\uc790 \ubbfc\uc8fc\uc8fc\uc758\uc790\uc600\ub2e4.\"\uba74\uc11c \"\ud55c\uad6d\uc758 \ubbfc\uc8fc\ud654\uc640 \uc778\uad8c, \uadf8\ub9ac\uace0 \ub3d9\ubd81\uc544\uc758 \ud3c9\ud654\uc640 \uc548\uc804\uc744 \uc704\ud55c \uae40 \uc804 \ub300\ud1b5\ub839\uc758 \uc815\uce58\uc801 \uc5c5\uc801\uc740 \uc78a\ud788\uc9c0 \uc54a\uc744 \uac83\uc774\ub2e4.\"\uc774\ub77c\uace0 \uac15\uc870\ud588\ub2e4. \uba54\ub974\ucf08 \ucd1d\ub9ac\ub294 \uc774\uc5b4 \"\ub3c5\uc77c\uacfc\uc758 \uc624\ub79c \uc778\uc5f0\uc73c\ub85c \ub3c5\uc77c\uc778\ub4e4\uc758 \ub9c8\uc74c\uc5d0 \uae4a\uc774 \uc0c8\uaca8\uc838 \uc788\ub294 \uae40 \uc804 \ub300\ud1b5\ub839\uc758 \uc11c\uac70\ub85c \ub3c5\uc77c\uc5f0\ubc29\uacf5\ud654\uad6d\uc740 \uc88b\uc740 \uce5c\uad6c\ub97c \uc783\uc5c8\ub2e4.\"\uba74\uc11c \"\uadf8\ub294 \ub300\ud1b5\ub839 \uc7ac\uc784\uae30\uac04 \uc591\uad6d\uad00\uacc4\uc758 \uc2ec\ud654\uc640 \uad50\ub958\uac15\ud654\ub97c \ud1b5\ud574 \ud6cc\ub96d\ud55c \uc5c5\uc801\uc744 \ub0a8\uacbc\ub2e4.\"\ub77c\uace0 \ud3c9\uac00\ud588\ub2e4. \uae40 \uc804 \ub300\ud1b5\ub839\uc758 40\ub144 \uc9c0\uae30\uc778 \ub9ac\ud558\ub974\ud2b8 \ud3f0 \ubc14\uc774\uce20\uccb4\ucee4 \uc804 \ub3c5\uc77c \ub300\ud1b5\ub839\uc740 \uae40 \uc804 \ub300\ud1b5\ub839\uc758 \uc0ac\ub9dd\uc5d0 '\uc4f0\ub77c\ub9b0 \uc544\ud514'\uc744 \ub290\ub080\ub2e4\uace0 \ubc1d\ud614\ub2e4. \ud3f0 \ubc14\uc774\uccb4\ucee4 \uc804 \ub300\ud1b5\ub839\uc740 \uae40 \uc804 \ub300\ud1b5\ub839\uc774 \uc0ac\ub9dd\ud588\ub2e4\ub294 \uc18c\uc2dd\uc744 \ub4e4\uc740 \ub4a4 \uc5f0\ud569\ub274\uc2a4\uc5d0 \"\ub098\uc758 \uc624\ub79c \uce5c\uad6c\uc778 \uae40\ub300\uc911 \uc804 \ub300\ud1b5\ub839\uacfc \uc4f0\ub77c\ub9b0 \uc544\ud514\uc73c\ub85c \uc791\ubcc4\ud558\uac8c \ub410\ub2e4\"\ub77c\uace0 \ub9d0\ud588\ub2e4. \uadf8\ub294 \uc774\uc5b4 \"\uae40 \uc804 \ub300\ud1b5\ub839\uc740 \ubd88\uad74\uc758 \uc6a9\uae30, \uc790\uc720 \ubbfc\uc8fc\uc8fc\uc758\ub97c \uc704\ud55c \uc0ac\ub791, \uadf8\ub9ac\uace0 \ubb34\uc5c7\ubcf4\ub2e4 \ud55c\ubbfc\uc871 \uc804\uccb4\uc758 \ud3c9\ud654\ub97c \uc704\ud55c \ud0c1\uc6d4\ud55c \uae30\uc5ec\ub97c \ud1b5\ud574 \uc6b0\ub9ac \ubaa8\ub450\uc5d0\uac8c \ubaa8\ubc94\uc774 \ub410\ub2e4\"\ub77c\uace0 \uac15\uc870\ud588\ub2e4.", "sentence_answer": "\uae40 \uc804 \ub300\ud1b5\ub839\uc758 40\ub144 \uc9c0\uae30\uc778 \ub9ac\ud558\ub974\ud2b8 \ud3f0 \ubc14\uc774\uce20\uccb4\ucee4 \uc804 \ub3c5\uc77c \ub300\ud1b5\ub839\uc740 \uae40 \uc804 \ub300\ud1b5\ub839\uc758 \uc0ac\ub9dd\uc5d0 '\uc4f0\ub77c\ub9b0 \uc544\ud514'\uc744 \ub290\ub080\ub2e4\uace0 \ubc1d\ud614\ub2e4.", "paragraph_id": "6488203-47-0", "paragraph_question": "question: \uae40\ub300\uc911\uacfc 40\ub144 \uc9c0\uae30\uc600\ub358 \ub3c5\uc77c\uc758 \uc804\uc9c1 \ub300\ud1b5\ub839\uc740?, context: \uc559\uac94\ub77c \uba54\ub974\ucf08 \ub3c5\uc77c \ucd1d\ub9ac\ub294 \uc870\uc804\uc744 \ud1b5\ud574 \uc560\ub3c4\ub97c \ud45c\uc2dc\ud558\uba70 \"\uae40 \uc804 \ub300\ud1b5\ub839\uc740 \ud55c\uad6d\uc758 \uad6d\uacbd\uc744 \ub118\uc5b4 \uad6d\uc81c\uc801\uc73c\ub85c \uc874\uacbd\ubc1b\ub294 \uc815\uce58\uc778\uc774\uc790 \ubbfc\uc8fc\uc8fc\uc758\uc790\uc600\ub2e4.\"\uba74\uc11c \"\ud55c\uad6d\uc758 \ubbfc\uc8fc\ud654\uc640 \uc778\uad8c, \uadf8\ub9ac\uace0 \ub3d9\ubd81\uc544\uc758 \ud3c9\ud654\uc640 \uc548\uc804\uc744 \uc704\ud55c \uae40 \uc804 \ub300\ud1b5\ub839\uc758 \uc815\uce58\uc801 \uc5c5\uc801\uc740 \uc78a\ud788\uc9c0 \uc54a\uc744 \uac83\uc774\ub2e4.\"\uc774\ub77c\uace0 \uac15\uc870\ud588\ub2e4. \uba54\ub974\ucf08 \ucd1d\ub9ac\ub294 \uc774\uc5b4 \"\ub3c5\uc77c\uacfc\uc758 \uc624\ub79c \uc778\uc5f0\uc73c\ub85c \ub3c5\uc77c\uc778\ub4e4\uc758 \ub9c8\uc74c\uc5d0 \uae4a\uc774 \uc0c8\uaca8\uc838 \uc788\ub294 \uae40 \uc804 \ub300\ud1b5\ub839\uc758 \uc11c\uac70\ub85c \ub3c5\uc77c\uc5f0\ubc29\uacf5\ud654\uad6d\uc740 \uc88b\uc740 \uce5c\uad6c\ub97c \uc783\uc5c8\ub2e4.\"\uba74\uc11c \"\uadf8\ub294 \ub300\ud1b5\ub839 \uc7ac\uc784\uae30\uac04 \uc591\uad6d\uad00\uacc4\uc758 \uc2ec\ud654\uc640 \uad50\ub958\uac15\ud654\ub97c \ud1b5\ud574 \ud6cc\ub96d\ud55c \uc5c5\uc801\uc744 \ub0a8\uacbc\ub2e4.\"\ub77c\uace0 \ud3c9\uac00\ud588\ub2e4. \uae40 \uc804 \ub300\ud1b5\ub839\uc758 40\ub144 \uc9c0\uae30\uc778 \ub9ac\ud558\ub974\ud2b8 \ud3f0 \ubc14\uc774\uce20\uccb4\ucee4 \uc804 \ub3c5\uc77c \ub300\ud1b5\ub839\uc740 \uae40 \uc804 \ub300\ud1b5\ub839\uc758 \uc0ac\ub9dd\uc5d0 '\uc4f0\ub77c\ub9b0 \uc544\ud514'\uc744 \ub290\ub080\ub2e4\uace0 \ubc1d\ud614\ub2e4. \ud3f0 \ubc14\uc774\uccb4\ucee4 \uc804 \ub300\ud1b5\ub839\uc740 \uae40 \uc804 \ub300\ud1b5\ub839\uc774 \uc0ac\ub9dd\ud588\ub2e4\ub294 \uc18c\uc2dd\uc744 \ub4e4\uc740 \ub4a4 \uc5f0\ud569\ub274\uc2a4\uc5d0 \"\ub098\uc758 \uc624\ub79c \uce5c\uad6c\uc778 \uae40\ub300\uc911 \uc804 \ub300\ud1b5\ub839\uacfc \uc4f0\ub77c\ub9b0 \uc544\ud514\uc73c\ub85c \uc791\ubcc4\ud558\uac8c \ub410\ub2e4\"\ub77c\uace0 \ub9d0\ud588\ub2e4. \uadf8\ub294 \uc774\uc5b4 \"\uae40 \uc804 \ub300\ud1b5\ub839\uc740 \ubd88\uad74\uc758 \uc6a9\uae30, \uc790\uc720 \ubbfc\uc8fc\uc8fc\uc758\ub97c \uc704\ud55c \uc0ac\ub791, \uadf8\ub9ac\uace0 \ubb34\uc5c7\ubcf4\ub2e4 \ud55c\ubbfc\uc871 \uc804\uccb4\uc758 \ud3c9\ud654\ub97c \uc704\ud55c \ud0c1\uc6d4\ud55c \uae30\uc5ec\ub97c \ud1b5\ud574 \uc6b0\ub9ac \ubaa8\ub450\uc5d0\uac8c \ubaa8\ubc94\uc774 \ub410\ub2e4\"\ub77c\uace0 \uac15\uc870\ud588\ub2e4."} {"question": "\uc601\uad6c \ud3c9\ud654\ub77c\uace0\ub3c4 \uc54c\ub824\uc9c4 \uc870\uc57d\uc740?", "paragraph": "\uae34 \ud611\uc0c1 \ub05d\uc5d0 13\uac1c \uce78\ud1a4\uacfc \uc774\ub4e4\uc758 \ub3d9\ub9f9 (\uc7a5\ud06c\ud2b8\uac08\ub80c \uc218\ub3c4\uc6d0\uacfc \uc2dc, 3\uac1c\uad6d \ub3d9\ub9f9, \ubc1c\ub808, \ubb04\ub8e8\uc988\ub97c \ucd94\uce21\uc73c\ub85c \ud558\ub294 \ucabd\uacfc \ud504\ub791\uc2a4\uc758 \uc655 \ubc0f \ubc00\ub77c\ub178 \uacf5\uc791\uc778 \ud504\ub791\uc218\uc544 \uce21\uc740 1516\ub144 11\uc6d4 29\uc77c \ud504\ub9ac\ubd80\ub974\uc5d0\uc11c \uc870\uc57d\uc744 \uccb4\uacb0\ud588\ub2e4. \"\uc601\uad6c \ud3c9\ud654\" (Ewiger Frieden, Paix perp\u00e9tuelle)\ub77c\uace0\ub3c4 \uc54c\ub824\uc9c4 \ud504\ub9ac\ubd80\ub974 \uc870\uc57d\uc5d0\uc11c \uc2a4\uc704\uc2a4 \uc5f0\ubc29\uc740 \ubc00\ub77c\ub178\uc758 \ubcf4\ud638\uad6d\uc774\ub77c\ub294 \ubaa8\ub4e0 \uad8c\ub9ac\ub4e4\uc744 \ud3ec\uae30\ud588\ub2e4. \ub300\uc2e0\uc5d0 \ud504\ub791\uc2a4 \uce21\uc740 \uc5f0\ubc29 \uce21\uc5d0 \ubcf4\uc0c1\uc73c\ub85c 700,000 \uace8\ub4dc \ud06c\ub77c\uc6b4\uc744 \uc9c0\ubd88\ud588\ub2e4. \uc591 \uce21\uc740 \uc0c1\ub300\ubc29\uc758 \uc801\uc5d0\uac8c \uc5b4\ub5a0\ud55c \uc9c0\uc6d0\ub97c \ud558\uc9c0 \uc54a\uae30\ub85c \ub3d9\uc758\ud588\uace0 (\uc870\uc57d \uc774\uc804\uc5d0 \uccb4\uacb0\ub41c \uac83\uc740 \uc81c\uc678\ud558\uace0) \ubbf8\ub798\uc758 \ubd84\uc7c1\uc758 \uacbd\uc6b0\uc560\ub294 \uc911\uc7ac \uc870\uc815\uc744 \ud558\uae30\ub85c \ub3d9\uc758\ud588\ub2e4. \ud504\ub791\uc2a4\ub294 \uc2a4\uc704\uc2a4\uac00 \uc54c\ud504\uc2a4 \uc774\ub0a8 \uc9c0\uc5ed (\ubca8\ub9b0\ucd08\ub098\ub97c \uc81c\uc678\ud558\uace0)\uc744 \uae30\uaebc\uc774 \uc591\ubcf4\ud558\uaca0\ub2e4\uba74 300,000 \ud06c\ub77c\uc6b4\uc744 \uc9c0\ubd88\ud558\uaca0\ub2e4\uace0 \uc81c\uc548\ud588\uc73c\ub098, \uc774 \uc81c\uc548\uc740 \uac70\ubd80\ub418\uc5c8\ub2e4. \ucd94\uac00\uc801\uc73c\ub85c \uc774 \uc870\uc57d\uc740 \ubc00\ub77c\ub178\uc640 \ub9ac\uc639\uc5d0\uc11c \uc2a4\uc704\uc2a4\uc5d0 \ub300\ud55c \ubb34\uc5ed \ud2b9\uad8c\uc744 \ubd80\uc5ec\ud588\ub2e4. \uc624\uc194\ub77c \uacc4\uace1\ub9cc\uc774 \ubc00\ub77c\ub178\uc5d0 \ubc18\ud658\ub410\uace0, \ub098\uba38\uc9c0 \uc774\ub0a8 \uc9c0\uc5ed\uc740 \uc774 \uc2dc\uae30\ubd80\ud130 \ud3ec\ud06c\ud2b8\ub4e4\uc774 \ud1b5\uce58\ud558\ub294 \uc2a4\uc704\uc2a4\uc758 \uc77c\ubd80\ub85c\uc11c \ub0a8\uac8c\ub418\uc5b4, 1803\ub144\ubd80\ud130\ub294 \ud2f0\uce58\ub178 \uc8fc\uac00 \ub418\uc5c8\ub2e4(\ud55c\ud3b8 3\uac1c\uad6d \ub3d9\ub9f9\uc740 1797\ub144\uc5d0 \ubc1c\ud154\ub9ac\ub098\uc5d0 \ub300\ud55c \uc9c0\ubc30\uad8c\uc744 \uc0c1\uc2e4\ud588\ub2e4).", "answer": "\ud504\ub9ac\ubd80\ub974 \uc870\uc57d", "sentence": "\"\uc601\uad6c \ud3c9\ud654\" (Ewiger Frieden, Paix perp\u00e9tuelle)\ub77c\uace0\ub3c4 \uc54c\ub824\uc9c4 \ud504\ub9ac\ubd80\ub974 \uc870\uc57d \uc5d0\uc11c \uc2a4\uc704\uc2a4 \uc5f0\ubc29\uc740 \ubc00\ub77c\ub178\uc758 \ubcf4\ud638\uad6d\uc774\ub77c\ub294 \ubaa8\ub4e0 \uad8c\ub9ac\ub4e4\uc744 \ud3ec\uae30\ud588\ub2e4.", "paragraph_sentence": "\uae34 \ud611\uc0c1 \ub05d\uc5d0 13\uac1c \uce78\ud1a4\uacfc \uc774\ub4e4\uc758 \ub3d9\ub9f9 (\uc7a5\ud06c\ud2b8\uac08\ub80c \uc218\ub3c4\uc6d0\uacfc \uc2dc, 3\uac1c\uad6d \ub3d9\ub9f9, \ubc1c\ub808, \ubb04\ub8e8\uc988\ub97c \ucd94\uce21\uc73c\ub85c \ud558\ub294 \ucabd\uacfc \ud504\ub791\uc2a4\uc758 \uc655 \ubc0f \ubc00\ub77c\ub178 \uacf5\uc791\uc778 \ud504\ub791\uc218\uc544 \uce21\uc740 1516\ub144 11\uc6d4 29\uc77c \ud504\ub9ac\ubd80\ub974\uc5d0\uc11c \uc870\uc57d\uc744 \uccb4\uacb0\ud588\ub2e4. \"\uc601\uad6c \ud3c9\ud654\" (Ewiger Frieden, Paix perp\u00e9tuelle)\ub77c\uace0\ub3c4 \uc54c\ub824\uc9c4 \ud504\ub9ac\ubd80\ub974 \uc870\uc57d \uc5d0\uc11c \uc2a4\uc704\uc2a4 \uc5f0\ubc29\uc740 \ubc00\ub77c\ub178\uc758 \ubcf4\ud638\uad6d\uc774\ub77c\ub294 \ubaa8\ub4e0 \uad8c\ub9ac\ub4e4\uc744 \ud3ec\uae30\ud588\ub2e4. \ub300\uc2e0\uc5d0 \ud504\ub791\uc2a4 \uce21\uc740 \uc5f0\ubc29 \uce21\uc5d0 \ubcf4\uc0c1\uc73c\ub85c 700,000 \uace8\ub4dc \ud06c\ub77c\uc6b4\uc744 \uc9c0\ubd88\ud588\ub2e4. \uc591 \uce21\uc740 \uc0c1\ub300\ubc29\uc758 \uc801\uc5d0\uac8c \uc5b4\ub5a0\ud55c \uc9c0\uc6d0\ub97c \ud558\uc9c0 \uc54a\uae30\ub85c \ub3d9\uc758\ud588\uace0 (\uc870\uc57d \uc774\uc804\uc5d0 \uccb4\uacb0\ub41c \uac83\uc740 \uc81c\uc678\ud558\uace0) \ubbf8\ub798\uc758 \ubd84\uc7c1\uc758 \uacbd\uc6b0\uc560\ub294 \uc911\uc7ac \uc870\uc815\uc744 \ud558\uae30\ub85c \ub3d9\uc758\ud588\ub2e4. \ud504\ub791\uc2a4\ub294 \uc2a4\uc704\uc2a4\uac00 \uc54c\ud504\uc2a4 \uc774\ub0a8 \uc9c0\uc5ed (\ubca8\ub9b0\ucd08\ub098\ub97c \uc81c\uc678\ud558\uace0)\uc744 \uae30\uaebc\uc774 \uc591\ubcf4\ud558\uaca0\ub2e4\uba74 300,000 \ud06c\ub77c\uc6b4\uc744 \uc9c0\ubd88\ud558\uaca0\ub2e4\uace0 \uc81c\uc548\ud588\uc73c\ub098, \uc774 \uc81c\uc548\uc740 \uac70\ubd80\ub418\uc5c8\ub2e4. \ucd94\uac00\uc801\uc73c\ub85c \uc774 \uc870\uc57d\uc740 \ubc00\ub77c\ub178\uc640 \ub9ac\uc639\uc5d0\uc11c \uc2a4\uc704\uc2a4\uc5d0 \ub300\ud55c \ubb34\uc5ed \ud2b9\uad8c\uc744 \ubd80\uc5ec\ud588\ub2e4. \uc624\uc194\ub77c \uacc4\uace1\ub9cc\uc774 \ubc00\ub77c\ub178\uc5d0 \ubc18\ud658\ub410\uace0, \ub098\uba38\uc9c0 \uc774\ub0a8 \uc9c0\uc5ed\uc740 \uc774 \uc2dc\uae30\ubd80\ud130 \ud3ec\ud06c\ud2b8\ub4e4\uc774 \ud1b5\uce58\ud558\ub294 \uc2a4\uc704\uc2a4\uc758 \uc77c\ubd80\ub85c\uc11c \ub0a8\uac8c\ub418\uc5b4, 1803\ub144\ubd80\ud130\ub294 \ud2f0\uce58\ub178 \uc8fc\uac00 \ub418\uc5c8\ub2e4(\ud55c\ud3b8 3\uac1c\uad6d \ub3d9\ub9f9\uc740 1797\ub144\uc5d0 \ubc1c\ud154\ub9ac\ub098\uc5d0 \ub300\ud55c \uc9c0\ubc30\uad8c\uc744 \uc0c1\uc2e4\ud588\ub2e4).", "paragraph_answer": "\uae34 \ud611\uc0c1 \ub05d\uc5d0 13\uac1c \uce78\ud1a4\uacfc \uc774\ub4e4\uc758 \ub3d9\ub9f9 (\uc7a5\ud06c\ud2b8\uac08\ub80c \uc218\ub3c4\uc6d0\uacfc \uc2dc, 3\uac1c\uad6d \ub3d9\ub9f9, \ubc1c\ub808, \ubb04\ub8e8\uc988\ub97c \ucd94\uce21\uc73c\ub85c \ud558\ub294 \ucabd\uacfc \ud504\ub791\uc2a4\uc758 \uc655 \ubc0f \ubc00\ub77c\ub178 \uacf5\uc791\uc778 \ud504\ub791\uc218\uc544 \uce21\uc740 1516\ub144 11\uc6d4 29\uc77c \ud504\ub9ac\ubd80\ub974\uc5d0\uc11c \uc870\uc57d\uc744 \uccb4\uacb0\ud588\ub2e4. \"\uc601\uad6c \ud3c9\ud654\" (Ewiger Frieden, Paix perp\u00e9tuelle)\ub77c\uace0\ub3c4 \uc54c\ub824\uc9c4 \ud504\ub9ac\ubd80\ub974 \uc870\uc57d \uc5d0\uc11c \uc2a4\uc704\uc2a4 \uc5f0\ubc29\uc740 \ubc00\ub77c\ub178\uc758 \ubcf4\ud638\uad6d\uc774\ub77c\ub294 \ubaa8\ub4e0 \uad8c\ub9ac\ub4e4\uc744 \ud3ec\uae30\ud588\ub2e4. \ub300\uc2e0\uc5d0 \ud504\ub791\uc2a4 \uce21\uc740 \uc5f0\ubc29 \uce21\uc5d0 \ubcf4\uc0c1\uc73c\ub85c 700,000 \uace8\ub4dc \ud06c\ub77c\uc6b4\uc744 \uc9c0\ubd88\ud588\ub2e4. \uc591 \uce21\uc740 \uc0c1\ub300\ubc29\uc758 \uc801\uc5d0\uac8c \uc5b4\ub5a0\ud55c \uc9c0\uc6d0\ub97c \ud558\uc9c0 \uc54a\uae30\ub85c \ub3d9\uc758\ud588\uace0 (\uc870\uc57d \uc774\uc804\uc5d0 \uccb4\uacb0\ub41c \uac83\uc740 \uc81c\uc678\ud558\uace0) \ubbf8\ub798\uc758 \ubd84\uc7c1\uc758 \uacbd\uc6b0\uc560\ub294 \uc911\uc7ac \uc870\uc815\uc744 \ud558\uae30\ub85c \ub3d9\uc758\ud588\ub2e4. \ud504\ub791\uc2a4\ub294 \uc2a4\uc704\uc2a4\uac00 \uc54c\ud504\uc2a4 \uc774\ub0a8 \uc9c0\uc5ed (\ubca8\ub9b0\ucd08\ub098\ub97c \uc81c\uc678\ud558\uace0)\uc744 \uae30\uaebc\uc774 \uc591\ubcf4\ud558\uaca0\ub2e4\uba74 300,000 \ud06c\ub77c\uc6b4\uc744 \uc9c0\ubd88\ud558\uaca0\ub2e4\uace0 \uc81c\uc548\ud588\uc73c\ub098, \uc774 \uc81c\uc548\uc740 \uac70\ubd80\ub418\uc5c8\ub2e4. \ucd94\uac00\uc801\uc73c\ub85c \uc774 \uc870\uc57d\uc740 \ubc00\ub77c\ub178\uc640 \ub9ac\uc639\uc5d0\uc11c \uc2a4\uc704\uc2a4\uc5d0 \ub300\ud55c \ubb34\uc5ed \ud2b9\uad8c\uc744 \ubd80\uc5ec\ud588\ub2e4. \uc624\uc194\ub77c \uacc4\uace1\ub9cc\uc774 \ubc00\ub77c\ub178\uc5d0 \ubc18\ud658\ub410\uace0, \ub098\uba38\uc9c0 \uc774\ub0a8 \uc9c0\uc5ed\uc740 \uc774 \uc2dc\uae30\ubd80\ud130 \ud3ec\ud06c\ud2b8\ub4e4\uc774 \ud1b5\uce58\ud558\ub294 \uc2a4\uc704\uc2a4\uc758 \uc77c\ubd80\ub85c\uc11c \ub0a8\uac8c\ub418\uc5b4, 1803\ub144\ubd80\ud130\ub294 \ud2f0\uce58\ub178 \uc8fc\uac00 \ub418\uc5c8\ub2e4(\ud55c\ud3b8 3\uac1c\uad6d \ub3d9\ub9f9\uc740 1797\ub144\uc5d0 \ubc1c\ud154\ub9ac\ub098\uc5d0 \ub300\ud55c \uc9c0\ubc30\uad8c\uc744 \uc0c1\uc2e4\ud588\ub2e4).", "sentence_answer": "\"\uc601\uad6c \ud3c9\ud654\" (Ewiger Frieden, Paix perp\u00e9tuelle)\ub77c\uace0\ub3c4 \uc54c\ub824\uc9c4 \ud504\ub9ac\ubd80\ub974 \uc870\uc57d \uc5d0\uc11c \uc2a4\uc704\uc2a4 \uc5f0\ubc29\uc740 \ubc00\ub77c\ub178\uc758 \ubcf4\ud638\uad6d\uc774\ub77c\ub294 \ubaa8\ub4e0 \uad8c\ub9ac\ub4e4\uc744 \ud3ec\uae30\ud588\ub2e4.", "paragraph_id": "6514001-5-0", "paragraph_question": "question: \uc601\uad6c \ud3c9\ud654\ub77c\uace0\ub3c4 \uc54c\ub824\uc9c4 \uc870\uc57d\uc740?, context: \uae34 \ud611\uc0c1 \ub05d\uc5d0 13\uac1c \uce78\ud1a4\uacfc \uc774\ub4e4\uc758 \ub3d9\ub9f9 (\uc7a5\ud06c\ud2b8\uac08\ub80c \uc218\ub3c4\uc6d0\uacfc \uc2dc, 3\uac1c\uad6d \ub3d9\ub9f9, \ubc1c\ub808, \ubb04\ub8e8\uc988\ub97c \ucd94\uce21\uc73c\ub85c \ud558\ub294 \ucabd\uacfc \ud504\ub791\uc2a4\uc758 \uc655 \ubc0f \ubc00\ub77c\ub178 \uacf5\uc791\uc778 \ud504\ub791\uc218\uc544 \uce21\uc740 1516\ub144 11\uc6d4 29\uc77c \ud504\ub9ac\ubd80\ub974\uc5d0\uc11c \uc870\uc57d\uc744 \uccb4\uacb0\ud588\ub2e4. \"\uc601\uad6c \ud3c9\ud654\" (Ewiger Frieden, Paix perp\u00e9tuelle)\ub77c\uace0\ub3c4 \uc54c\ub824\uc9c4 \ud504\ub9ac\ubd80\ub974 \uc870\uc57d\uc5d0\uc11c \uc2a4\uc704\uc2a4 \uc5f0\ubc29\uc740 \ubc00\ub77c\ub178\uc758 \ubcf4\ud638\uad6d\uc774\ub77c\ub294 \ubaa8\ub4e0 \uad8c\ub9ac\ub4e4\uc744 \ud3ec\uae30\ud588\ub2e4. \ub300\uc2e0\uc5d0 \ud504\ub791\uc2a4 \uce21\uc740 \uc5f0\ubc29 \uce21\uc5d0 \ubcf4\uc0c1\uc73c\ub85c 700,000 \uace8\ub4dc \ud06c\ub77c\uc6b4\uc744 \uc9c0\ubd88\ud588\ub2e4. \uc591 \uce21\uc740 \uc0c1\ub300\ubc29\uc758 \uc801\uc5d0\uac8c \uc5b4\ub5a0\ud55c \uc9c0\uc6d0\ub97c \ud558\uc9c0 \uc54a\uae30\ub85c \ub3d9\uc758\ud588\uace0 (\uc870\uc57d \uc774\uc804\uc5d0 \uccb4\uacb0\ub41c \uac83\uc740 \uc81c\uc678\ud558\uace0) \ubbf8\ub798\uc758 \ubd84\uc7c1\uc758 \uacbd\uc6b0\uc560\ub294 \uc911\uc7ac \uc870\uc815\uc744 \ud558\uae30\ub85c \ub3d9\uc758\ud588\ub2e4. \ud504\ub791\uc2a4\ub294 \uc2a4\uc704\uc2a4\uac00 \uc54c\ud504\uc2a4 \uc774\ub0a8 \uc9c0\uc5ed (\ubca8\ub9b0\ucd08\ub098\ub97c \uc81c\uc678\ud558\uace0)\uc744 \uae30\uaebc\uc774 \uc591\ubcf4\ud558\uaca0\ub2e4\uba74 300,000 \ud06c\ub77c\uc6b4\uc744 \uc9c0\ubd88\ud558\uaca0\ub2e4\uace0 \uc81c\uc548\ud588\uc73c\ub098, \uc774 \uc81c\uc548\uc740 \uac70\ubd80\ub418\uc5c8\ub2e4. \ucd94\uac00\uc801\uc73c\ub85c \uc774 \uc870\uc57d\uc740 \ubc00\ub77c\ub178\uc640 \ub9ac\uc639\uc5d0\uc11c \uc2a4\uc704\uc2a4\uc5d0 \ub300\ud55c \ubb34\uc5ed \ud2b9\uad8c\uc744 \ubd80\uc5ec\ud588\ub2e4. \uc624\uc194\ub77c \uacc4\uace1\ub9cc\uc774 \ubc00\ub77c\ub178\uc5d0 \ubc18\ud658\ub410\uace0, \ub098\uba38\uc9c0 \uc774\ub0a8 \uc9c0\uc5ed\uc740 \uc774 \uc2dc\uae30\ubd80\ud130 \ud3ec\ud06c\ud2b8\ub4e4\uc774 \ud1b5\uce58\ud558\ub294 \uc2a4\uc704\uc2a4\uc758 \uc77c\ubd80\ub85c\uc11c \ub0a8\uac8c\ub418\uc5b4, 1803\ub144\ubd80\ud130\ub294 \ud2f0\uce58\ub178 \uc8fc\uac00 \ub418\uc5c8\ub2e4(\ud55c\ud3b8 3\uac1c\uad6d \ub3d9\ub9f9\uc740 1797\ub144\uc5d0 \ubc1c\ud154\ub9ac\ub098\uc5d0 \ub300\ud55c \uc9c0\ubc30\uad8c\uc744 \uc0c1\uc2e4\ud588\ub2e4)."} {"question": "\uac74\ubb3c\uc744 \uc138\uc6b8\ub54c \uc774\uc6c3 \ud1a0\uc9c0\uc640\uc758 \uacbd\uacc4\uc120\uc5d0\uc11c \uc5bc\ub9cc\ud07c \uc774\uc0c1\uc758 \uac70\ub9ac\ub97c \ub450\uc5b4\uc57c\ud558\ub294\uac00?", "paragraph": "\uac74\ubb3c\uc744 \uc138\uc6b8 \ub54c\ub294 \uacbd\uacc4\uc120\uc5d0\uc11c 50\uc13c\ud2f0 \uc774\uc0c1\uc758 \uac70\ub9ac\ub97c \ub450\uc9c0 \uc54a\uc73c\uba74 \uc548 \ub41c\ub2e4(242\uc870). \uc774 \uc81c\ud55c\uc744 \ubb34\uc2dc\ud558\uace0 \uac74\ucd95\uc744 \ud558\ub824 \ud560 \ub54c\ub294 \uc774\uc6c3 \ud1a0\uc9c0 \uc18c\uc720\uc790\ub294 \uadf8 \uac74\ubb3c\uc758 \ubcc0\uacbd\uc774\ub098 \ucca0\uac70\ub97c \uccad\uad6c\ud560 \uc218 \uc788\ub2e4(242\uc870 2\ud56d \ubcf8\ubb38). \uadf8\ub7ec\ub098 \uac74\ucd95\uc744 \uc2dc\uc791\ud558\uace0 1\ub144\uc774 \uc9c0\ub0ac\uac70\ub098 \ub610\ub294 \uc774\ubbf8 \uac74\ucd95\uc774 \uc644\uc131\ub418\uc5c8\uc73c\uba74 \ub2e4\ub9cc \uc190\ud574\ubc30\uc0c1\uc744 \uccad\uad6c\ud560 \uc218 \uc788\uc744 \ubfd0\uc774\ub2e4(242\uc870 2\ud56d \ub2e8\uc11c). \uadf8 \ubc16\uc5d0 \ud0c0\uc778\uc758 \uc8fc\ud0dd\ub0b4\ubd80\ub97c \uad00\ub9dd\ud560 \uc218 \uc788\ub294 \uc704\uce58\uc5d0 \ucc3d\uc774\ub098 \ub9c8\ub8e8\uac00 \uc788\uc5b4 \uc0ac\uc0dd\ud65c\uc758 \ube44\ubc00\uc774 \ub204\uc124\ub428\uc744 \uc6b0\ub824\ud558\uc5ec \uc774\uc6c3 \uac04\uc5d0 \ubd88\ud654\ub97c \uc77c\uc73c\ud0a4\ub294 \uc218\uac00 \ub9ce\uc73c\ubbc0\ub85c \uc774\ub97c \uc870\uc808\ud558\uae30 \uc704\ud574 \uacbd\uacc4\ub85c\ubd80\ud130 2\ubbf8\ud130 \uc774\ub0b4\uc758 \uac70\ub9ac\uc5d0\uc11c \uc774\uc6c3\uc8fc\ud0dd\uc758 \ub0b4\ubd80\ub97c \uad00\ub9dd\ud560 \uc218 \uc788\ub294 \ucc3d\uc774\ub098 \ub9c8\ub8e8\ub97c \uc124\uce58\ud558\ub294 \uacbd\uc6b0\uc5d0\ub294 \uc801\ub2f9\ud55c \ucc28\uba74\uc2dc\uc124(\u906e\u9762\u65bd\u8a2d)\uc744 \ud558\ub3c4\ub85d \uaddc\uc815\ud558\uace0 \uc788\ub2e4(243\uc870).", "answer": "50\uc13c\ud2f0 \uc774\uc0c1", "sentence": "\uac74\ubb3c\uc744 \uc138\uc6b8 \ub54c\ub294 \uacbd\uacc4\uc120\uc5d0\uc11c 50\uc13c\ud2f0 \uc774\uc0c1 \uc758 \uac70\ub9ac\ub97c \ub450\uc9c0 \uc54a\uc73c\uba74 \uc548 \ub41c\ub2e4(242\uc870).", "paragraph_sentence": " \uac74\ubb3c\uc744 \uc138\uc6b8 \ub54c\ub294 \uacbd\uacc4\uc120\uc5d0\uc11c 50\uc13c\ud2f0 \uc774\uc0c1 \uc758 \uac70\ub9ac\ub97c \ub450\uc9c0 \uc54a\uc73c\uba74 \uc548 \ub41c\ub2e4(242\uc870). \uc774 \uc81c\ud55c\uc744 \ubb34\uc2dc\ud558\uace0 \uac74\ucd95\uc744 \ud558\ub824 \ud560 \ub54c\ub294 \uc774\uc6c3 \ud1a0\uc9c0 \uc18c\uc720\uc790\ub294 \uadf8 \uac74\ubb3c\uc758 \ubcc0\uacbd\uc774\ub098 \ucca0\uac70\ub97c \uccad\uad6c\ud560 \uc218 \uc788\ub2e4(242\uc870 2\ud56d \ubcf8\ubb38) . \uadf8\ub7ec\ub098 \uac74\ucd95\uc744 \uc2dc\uc791\ud558\uace0 1\ub144\uc774 \uc9c0\ub0ac\uac70\ub098 \ub610\ub294 \uc774\ubbf8 \uac74\ucd95\uc774 \uc644\uc131\ub418\uc5c8\uc73c\uba74 \ub2e4\ub9cc \uc190\ud574\ubc30\uc0c1\uc744 \uccad\uad6c\ud560 \uc218 \uc788\uc744 \ubfd0\uc774\ub2e4(242\uc870 2\ud56d \ub2e8\uc11c). \uadf8 \ubc16\uc5d0 \ud0c0\uc778\uc758 \uc8fc\ud0dd\ub0b4\ubd80\ub97c \uad00\ub9dd\ud560 \uc218 \uc788\ub294 \uc704\uce58\uc5d0 \ucc3d\uc774\ub098 \ub9c8\ub8e8\uac00 \uc788\uc5b4 \uc0ac\uc0dd\ud65c\uc758 \ube44\ubc00\uc774 \ub204\uc124\ub428\uc744 \uc6b0\ub824\ud558\uc5ec \uc774\uc6c3 \uac04\uc5d0 \ubd88\ud654\ub97c \uc77c\uc73c\ud0a4\ub294 \uc218\uac00 \ub9ce\uc73c\ubbc0\ub85c \uc774\ub97c \uc870\uc808\ud558\uae30 \uc704\ud574 \uacbd\uacc4\ub85c\ubd80\ud130 2\ubbf8\ud130 \uc774\ub0b4\uc758 \uac70\ub9ac\uc5d0\uc11c \uc774\uc6c3\uc8fc\ud0dd\uc758 \ub0b4\ubd80\ub97c \uad00\ub9dd\ud560 \uc218 \uc788\ub294 \ucc3d\uc774\ub098 \ub9c8\ub8e8\ub97c \uc124\uce58\ud558\ub294 \uacbd\uc6b0\uc5d0\ub294 \uc801\ub2f9\ud55c \ucc28\uba74\uc2dc\uc124(\u906e\u9762\u65bd\u8a2d)\uc744 \ud558\ub3c4\ub85d \uaddc\uc815\ud558\uace0 \uc788\ub2e4(243\uc870).", "paragraph_answer": "\uac74\ubb3c\uc744 \uc138\uc6b8 \ub54c\ub294 \uacbd\uacc4\uc120\uc5d0\uc11c 50\uc13c\ud2f0 \uc774\uc0c1 \uc758 \uac70\ub9ac\ub97c \ub450\uc9c0 \uc54a\uc73c\uba74 \uc548 \ub41c\ub2e4(242\uc870). \uc774 \uc81c\ud55c\uc744 \ubb34\uc2dc\ud558\uace0 \uac74\ucd95\uc744 \ud558\ub824 \ud560 \ub54c\ub294 \uc774\uc6c3 \ud1a0\uc9c0 \uc18c\uc720\uc790\ub294 \uadf8 \uac74\ubb3c\uc758 \ubcc0\uacbd\uc774\ub098 \ucca0\uac70\ub97c \uccad\uad6c\ud560 \uc218 \uc788\ub2e4(242\uc870 2\ud56d \ubcf8\ubb38). \uadf8\ub7ec\ub098 \uac74\ucd95\uc744 \uc2dc\uc791\ud558\uace0 1\ub144\uc774 \uc9c0\ub0ac\uac70\ub098 \ub610\ub294 \uc774\ubbf8 \uac74\ucd95\uc774 \uc644\uc131\ub418\uc5c8\uc73c\uba74 \ub2e4\ub9cc \uc190\ud574\ubc30\uc0c1\uc744 \uccad\uad6c\ud560 \uc218 \uc788\uc744 \ubfd0\uc774\ub2e4(242\uc870 2\ud56d \ub2e8\uc11c). \uadf8 \ubc16\uc5d0 \ud0c0\uc778\uc758 \uc8fc\ud0dd\ub0b4\ubd80\ub97c \uad00\ub9dd\ud560 \uc218 \uc788\ub294 \uc704\uce58\uc5d0 \ucc3d\uc774\ub098 \ub9c8\ub8e8\uac00 \uc788\uc5b4 \uc0ac\uc0dd\ud65c\uc758 \ube44\ubc00\uc774 \ub204\uc124\ub428\uc744 \uc6b0\ub824\ud558\uc5ec \uc774\uc6c3 \uac04\uc5d0 \ubd88\ud654\ub97c \uc77c\uc73c\ud0a4\ub294 \uc218\uac00 \ub9ce\uc73c\ubbc0\ub85c \uc774\ub97c \uc870\uc808\ud558\uae30 \uc704\ud574 \uacbd\uacc4\ub85c\ubd80\ud130 2\ubbf8\ud130 \uc774\ub0b4\uc758 \uac70\ub9ac\uc5d0\uc11c \uc774\uc6c3\uc8fc\ud0dd\uc758 \ub0b4\ubd80\ub97c \uad00\ub9dd\ud560 \uc218 \uc788\ub294 \ucc3d\uc774\ub098 \ub9c8\ub8e8\ub97c \uc124\uce58\ud558\ub294 \uacbd\uc6b0\uc5d0\ub294 \uc801\ub2f9\ud55c \ucc28\uba74\uc2dc\uc124(\u906e\u9762\u65bd\u8a2d)\uc744 \ud558\ub3c4\ub85d \uaddc\uc815\ud558\uace0 \uc788\ub2e4(243\uc870).", "sentence_answer": "\uac74\ubb3c\uc744 \uc138\uc6b8 \ub54c\ub294 \uacbd\uacc4\uc120\uc5d0\uc11c 50\uc13c\ud2f0 \uc774\uc0c1 \uc758 \uac70\ub9ac\ub97c \ub450\uc9c0 \uc54a\uc73c\uba74 \uc548 \ub41c\ub2e4(242\uc870).", "paragraph_id": "6511298-2-0", "paragraph_question": "question: \uac74\ubb3c\uc744 \uc138\uc6b8\ub54c \uc774\uc6c3 \ud1a0\uc9c0\uc640\uc758 \uacbd\uacc4\uc120\uc5d0\uc11c \uc5bc\ub9cc\ud07c \uc774\uc0c1\uc758 \uac70\ub9ac\ub97c \ub450\uc5b4\uc57c\ud558\ub294\uac00?, context: \uac74\ubb3c\uc744 \uc138\uc6b8 \ub54c\ub294 \uacbd\uacc4\uc120\uc5d0\uc11c 50\uc13c\ud2f0 \uc774\uc0c1\uc758 \uac70\ub9ac\ub97c \ub450\uc9c0 \uc54a\uc73c\uba74 \uc548 \ub41c\ub2e4(242\uc870). \uc774 \uc81c\ud55c\uc744 \ubb34\uc2dc\ud558\uace0 \uac74\ucd95\uc744 \ud558\ub824 \ud560 \ub54c\ub294 \uc774\uc6c3 \ud1a0\uc9c0 \uc18c\uc720\uc790\ub294 \uadf8 \uac74\ubb3c\uc758 \ubcc0\uacbd\uc774\ub098 \ucca0\uac70\ub97c \uccad\uad6c\ud560 \uc218 \uc788\ub2e4(242\uc870 2\ud56d \ubcf8\ubb38). \uadf8\ub7ec\ub098 \uac74\ucd95\uc744 \uc2dc\uc791\ud558\uace0 1\ub144\uc774 \uc9c0\ub0ac\uac70\ub098 \ub610\ub294 \uc774\ubbf8 \uac74\ucd95\uc774 \uc644\uc131\ub418\uc5c8\uc73c\uba74 \ub2e4\ub9cc \uc190\ud574\ubc30\uc0c1\uc744 \uccad\uad6c\ud560 \uc218 \uc788\uc744 \ubfd0\uc774\ub2e4(242\uc870 2\ud56d \ub2e8\uc11c). \uadf8 \ubc16\uc5d0 \ud0c0\uc778\uc758 \uc8fc\ud0dd\ub0b4\ubd80\ub97c \uad00\ub9dd\ud560 \uc218 \uc788\ub294 \uc704\uce58\uc5d0 \ucc3d\uc774\ub098 \ub9c8\ub8e8\uac00 \uc788\uc5b4 \uc0ac\uc0dd\ud65c\uc758 \ube44\ubc00\uc774 \ub204\uc124\ub428\uc744 \uc6b0\ub824\ud558\uc5ec \uc774\uc6c3 \uac04\uc5d0 \ubd88\ud654\ub97c \uc77c\uc73c\ud0a4\ub294 \uc218\uac00 \ub9ce\uc73c\ubbc0\ub85c \uc774\ub97c \uc870\uc808\ud558\uae30 \uc704\ud574 \uacbd\uacc4\ub85c\ubd80\ud130 2\ubbf8\ud130 \uc774\ub0b4\uc758 \uac70\ub9ac\uc5d0\uc11c \uc774\uc6c3\uc8fc\ud0dd\uc758 \ub0b4\ubd80\ub97c \uad00\ub9dd\ud560 \uc218 \uc788\ub294 \ucc3d\uc774\ub098 \ub9c8\ub8e8\ub97c \uc124\uce58\ud558\ub294 \uacbd\uc6b0\uc5d0\ub294 \uc801\ub2f9\ud55c \ucc28\uba74\uc2dc\uc124(\u906e\u9762\u65bd\u8a2d)\uc744 \ud558\ub3c4\ub85d \uaddc\uc815\ud558\uace0 \uc788\ub2e4(243\uc870)."} {"question": "\uc870\uc120\uc778\uc744 \uac15\uc81c \uc9d5\uc6a9\ud558\uc5ec \uc11d\ud0c4 \ub178\ub3d9\uc744 \uc2dc\ud0a8 \ud558\uc2dc\ub9c8 \uc12c\uc758 \uc6b0\ub9ac\ub098\ub77c \uba85\uce6d\uc740?", "paragraph": "\ud558\uc2dc\ub9c8 \uc12c(\uc77c\ubcf8\uc5b4: \u7aef\u5cf6)\uc740 \uc77c\ubcf8 \ub098\uac00\uc0ac\ud0a4\ud604 \ub098\uac00\uc0ac\ud0a4\uc2dc\uc5d0 \uc788\ub294 \ubb34\uc778\ub3c4\uc774\ub2e4. \uc12c\uc758 \ubaa8\uc2b5\uc774 \ub9c8\uce58 \uad70\ud568\uacfc \ube44\uc2b7\ud558\uac8c \uc0dd\uacbc\uae30 \ub54c\ubb38\uc5d0 \uad70\uce78\uc9c0\ub9c8(\u8ecd\u8266\u5cf6 (\u3050\u3093\u304b\u3093\u3058\u307e), \uad70\ud568\ub3c4)\ub77c\uace0\ub3c4 \ubd88\ub9b0\ub2e4. 1940\ub144\ubd80\ud130 1945\ub144\uae4c\uc9c0 \uc870\uc120\uc778\uc744 \uac15\uc81c \uc9d5\uc6a9\ud558\uc5ec \uc11d\ud0c4 \ub178\ub3d9\uc744 \uc2dc\ud0a8 \uacf3\uc774\uae30\ub3c4 \ud558\uba70 \ub610\ud55c 1960\ub144\ub300\uae4c\uc9c0 \ud0c4\uad11 \ub3c4\uc2dc\ub85c\uc11c \ub9ce\uc740 \uc0ac\ub78c\ub4e4\uc774 \uac70\uc8fc\ud558\uace0 \uc788\uc5c8\uc9c0\ub9cc, \ud3d0\uc0b0 \uc774\ud6c4 \uc8fc\ubbfc\ub4e4\uc774 \uc774\uc8fc\ud558\uc600\uc73c\uba70, \uc12c\uc5d0\ub294 \ub2f9\uc2dc\uc758 \uac74\ubb3c\ub4e4\uc774 \uadf8\ub300\ub85c \ub0a8\uc544 \uc788\uc5c8\ub2e4. \uc77c\ubcf8\uc740 2015\ub144 \uc720\ub124\uc2a4\ucf54 \uc138\uacc4\uc720\uc0b0\uc5d0 \ub4f1\uc7ac \uc2e0\uccad\uc744 \ud558\uc600\uc73c\ub098, \ud55c\uad6d\uc758 \ubc18\ub300\ub85c \uc9c0\uc815\uc5d0 \ub09c\ud56d\uc744 \uacaa\uc5c8\ub2e4. \uadf8\ud6c4 \uac15\uc81c \ub178\uc5ed\uc5d0 \ub300\ud574 \uba85\uc2dc\ub97c \ud558\uaca0\ub2e4\ub294 \uc870\uac74\uc744 \ud1b5\ud574 \uac04\uc2e0\ud788 \ub4f1\uc7ac\ud558\uc600\uc73c\ub098, \uc9c1\ud6c4 \ud0dc\ub3c4\ub97c \ubc14\uafb8\uc5b4 \uac15\uc81c \ub178\ub3d9\uc758 \uc0ac\uc2e4\uc744 \uc5b8\uae09\ud558\uc9c0 \uc54a\uace0 \uc788\ub2e4.", "answer": "\uad70\ud568\ub3c4", "sentence": "\uc12c\uc758 \ubaa8\uc2b5\uc774 \ub9c8\uce58 \uad70\ud568\uacfc \ube44\uc2b7\ud558\uac8c \uc0dd\uacbc\uae30 \ub54c\ubb38\uc5d0 \uad70\uce78\uc9c0\ub9c8(\u8ecd\u8266\u5cf6 (\u3050\u3093\u304b\u3093\u3058\u307e), \uad70\ud568\ub3c4 )", "paragraph_sentence": "\ud558\uc2dc\ub9c8 \uc12c(\uc77c\ubcf8\uc5b4: \u7aef\u5cf6)\uc740 \uc77c\ubcf8 \ub098\uac00\uc0ac\ud0a4\ud604 \ub098\uac00\uc0ac\ud0a4\uc2dc\uc5d0 \uc788\ub294 \ubb34\uc778\ub3c4\uc774\ub2e4. \uc12c\uc758 \ubaa8\uc2b5\uc774 \ub9c8\uce58 \uad70\ud568\uacfc \ube44\uc2b7\ud558\uac8c \uc0dd\uacbc\uae30 \ub54c\ubb38\uc5d0 \uad70\uce78\uc9c0\ub9c8(\u8ecd\u8266\u5cf6 (\u3050\u3093\u304b\u3093\u3058\u307e), \uad70\ud568\ub3c4 ) \ub77c\uace0\ub3c4 \ubd88\ub9b0\ub2e4. 1940\ub144\ubd80\ud130 1945\ub144\uae4c\uc9c0 \uc870\uc120\uc778\uc744 \uac15\uc81c \uc9d5\uc6a9\ud558\uc5ec \uc11d\ud0c4 \ub178\ub3d9\uc744 \uc2dc\ud0a8 \uacf3\uc774\uae30\ub3c4 \ud558\uba70 \ub610\ud55c 1960\ub144\ub300\uae4c\uc9c0 \ud0c4\uad11 \ub3c4\uc2dc\ub85c\uc11c \ub9ce\uc740 \uc0ac\ub78c\ub4e4\uc774 \uac70\uc8fc\ud558\uace0 \uc788\uc5c8\uc9c0\ub9cc, \ud3d0\uc0b0 \uc774\ud6c4 \uc8fc\ubbfc\ub4e4\uc774 \uc774\uc8fc\ud558\uc600\uc73c\uba70, \uc12c\uc5d0\ub294 \ub2f9\uc2dc\uc758 \uac74\ubb3c\ub4e4\uc774 \uadf8\ub300\ub85c \ub0a8\uc544 \uc788\uc5c8\ub2e4. \uc77c\ubcf8\uc740 2015\ub144 \uc720\ub124\uc2a4\ucf54 \uc138\uacc4\uc720\uc0b0\uc5d0 \ub4f1\uc7ac \uc2e0\uccad\uc744 \ud558\uc600\uc73c\ub098, \ud55c\uad6d\uc758 \ubc18\ub300\ub85c \uc9c0\uc815\uc5d0 \ub09c\ud56d\uc744 \uacaa\uc5c8\ub2e4. \uadf8\ud6c4 \uac15\uc81c \ub178\uc5ed\uc5d0 \ub300\ud574 \uba85\uc2dc\ub97c \ud558\uaca0\ub2e4\ub294 \uc870\uac74\uc744 \ud1b5\ud574 \uac04\uc2e0\ud788 \ub4f1\uc7ac\ud558\uc600\uc73c\ub098, \uc9c1\ud6c4 \ud0dc\ub3c4\ub97c \ubc14\uafb8\uc5b4 \uac15\uc81c \ub178\ub3d9\uc758 \uc0ac\uc2e4\uc744 \uc5b8\uae09\ud558\uc9c0 \uc54a\uace0 \uc788\ub2e4.", "paragraph_answer": "\ud558\uc2dc\ub9c8 \uc12c(\uc77c\ubcf8\uc5b4: \u7aef\u5cf6)\uc740 \uc77c\ubcf8 \ub098\uac00\uc0ac\ud0a4\ud604 \ub098\uac00\uc0ac\ud0a4\uc2dc\uc5d0 \uc788\ub294 \ubb34\uc778\ub3c4\uc774\ub2e4. \uc12c\uc758 \ubaa8\uc2b5\uc774 \ub9c8\uce58 \uad70\ud568\uacfc \ube44\uc2b7\ud558\uac8c \uc0dd\uacbc\uae30 \ub54c\ubb38\uc5d0 \uad70\uce78\uc9c0\ub9c8(\u8ecd\u8266\u5cf6 (\u3050\u3093\u304b\u3093\u3058\u307e), \uad70\ud568\ub3c4 )\ub77c\uace0\ub3c4 \ubd88\ub9b0\ub2e4. 1940\ub144\ubd80\ud130 1945\ub144\uae4c\uc9c0 \uc870\uc120\uc778\uc744 \uac15\uc81c \uc9d5\uc6a9\ud558\uc5ec \uc11d\ud0c4 \ub178\ub3d9\uc744 \uc2dc\ud0a8 \uacf3\uc774\uae30\ub3c4 \ud558\uba70 \ub610\ud55c 1960\ub144\ub300\uae4c\uc9c0 \ud0c4\uad11 \ub3c4\uc2dc\ub85c\uc11c \ub9ce\uc740 \uc0ac\ub78c\ub4e4\uc774 \uac70\uc8fc\ud558\uace0 \uc788\uc5c8\uc9c0\ub9cc, \ud3d0\uc0b0 \uc774\ud6c4 \uc8fc\ubbfc\ub4e4\uc774 \uc774\uc8fc\ud558\uc600\uc73c\uba70, \uc12c\uc5d0\ub294 \ub2f9\uc2dc\uc758 \uac74\ubb3c\ub4e4\uc774 \uadf8\ub300\ub85c \ub0a8\uc544 \uc788\uc5c8\ub2e4. \uc77c\ubcf8\uc740 2015\ub144 \uc720\ub124\uc2a4\ucf54 \uc138\uacc4\uc720\uc0b0\uc5d0 \ub4f1\uc7ac \uc2e0\uccad\uc744 \ud558\uc600\uc73c\ub098, \ud55c\uad6d\uc758 \ubc18\ub300\ub85c \uc9c0\uc815\uc5d0 \ub09c\ud56d\uc744 \uacaa\uc5c8\ub2e4. \uadf8\ud6c4 \uac15\uc81c \ub178\uc5ed\uc5d0 \ub300\ud574 \uba85\uc2dc\ub97c \ud558\uaca0\ub2e4\ub294 \uc870\uac74\uc744 \ud1b5\ud574 \uac04\uc2e0\ud788 \ub4f1\uc7ac\ud558\uc600\uc73c\ub098, \uc9c1\ud6c4 \ud0dc\ub3c4\ub97c \ubc14\uafb8\uc5b4 \uac15\uc81c \ub178\ub3d9\uc758 \uc0ac\uc2e4\uc744 \uc5b8\uae09\ud558\uc9c0 \uc54a\uace0 \uc788\ub2e4.", "sentence_answer": "\uc12c\uc758 \ubaa8\uc2b5\uc774 \ub9c8\uce58 \uad70\ud568\uacfc \ube44\uc2b7\ud558\uac8c \uc0dd\uacbc\uae30 \ub54c\ubb38\uc5d0 \uad70\uce78\uc9c0\ub9c8(\u8ecd\u8266\u5cf6 (\u3050\u3093\u304b\u3093\u3058\u307e), \uad70\ud568\ub3c4 )", "paragraph_id": "6569652-0-1", "paragraph_question": "question: \uc870\uc120\uc778\uc744 \uac15\uc81c \uc9d5\uc6a9\ud558\uc5ec \uc11d\ud0c4 \ub178\ub3d9\uc744 \uc2dc\ud0a8 \ud558\uc2dc\ub9c8 \uc12c\uc758 \uc6b0\ub9ac\ub098\ub77c \uba85\uce6d\uc740?, context: \ud558\uc2dc\ub9c8 \uc12c(\uc77c\ubcf8\uc5b4: \u7aef\u5cf6)\uc740 \uc77c\ubcf8 \ub098\uac00\uc0ac\ud0a4\ud604 \ub098\uac00\uc0ac\ud0a4\uc2dc\uc5d0 \uc788\ub294 \ubb34\uc778\ub3c4\uc774\ub2e4. \uc12c\uc758 \ubaa8\uc2b5\uc774 \ub9c8\uce58 \uad70\ud568\uacfc \ube44\uc2b7\ud558\uac8c \uc0dd\uacbc\uae30 \ub54c\ubb38\uc5d0 \uad70\uce78\uc9c0\ub9c8(\u8ecd\u8266\u5cf6 (\u3050\u3093\u304b\u3093\u3058\u307e), \uad70\ud568\ub3c4)\ub77c\uace0\ub3c4 \ubd88\ub9b0\ub2e4. 1940\ub144\ubd80\ud130 1945\ub144\uae4c\uc9c0 \uc870\uc120\uc778\uc744 \uac15\uc81c \uc9d5\uc6a9\ud558\uc5ec \uc11d\ud0c4 \ub178\ub3d9\uc744 \uc2dc\ud0a8 \uacf3\uc774\uae30\ub3c4 \ud558\uba70 \ub610\ud55c 1960\ub144\ub300\uae4c\uc9c0 \ud0c4\uad11 \ub3c4\uc2dc\ub85c\uc11c \ub9ce\uc740 \uc0ac\ub78c\ub4e4\uc774 \uac70\uc8fc\ud558\uace0 \uc788\uc5c8\uc9c0\ub9cc, \ud3d0\uc0b0 \uc774\ud6c4 \uc8fc\ubbfc\ub4e4\uc774 \uc774\uc8fc\ud558\uc600\uc73c\uba70, \uc12c\uc5d0\ub294 \ub2f9\uc2dc\uc758 \uac74\ubb3c\ub4e4\uc774 \uadf8\ub300\ub85c \ub0a8\uc544 \uc788\uc5c8\ub2e4. \uc77c\ubcf8\uc740 2015\ub144 \uc720\ub124\uc2a4\ucf54 \uc138\uacc4\uc720\uc0b0\uc5d0 \ub4f1\uc7ac \uc2e0\uccad\uc744 \ud558\uc600\uc73c\ub098, \ud55c\uad6d\uc758 \ubc18\ub300\ub85c \uc9c0\uc815\uc5d0 \ub09c\ud56d\uc744 \uacaa\uc5c8\ub2e4. \uadf8\ud6c4 \uac15\uc81c \ub178\uc5ed\uc5d0 \ub300\ud574 \uba85\uc2dc\ub97c \ud558\uaca0\ub2e4\ub294 \uc870\uac74\uc744 \ud1b5\ud574 \uac04\uc2e0\ud788 \ub4f1\uc7ac\ud558\uc600\uc73c\ub098, \uc9c1\ud6c4 \ud0dc\ub3c4\ub97c \ubc14\uafb8\uc5b4 \uac15\uc81c \ub178\ub3d9\uc758 \uc0ac\uc2e4\uc744 \uc5b8\uae09\ud558\uc9c0 \uc54a\uace0 \uc788\ub2e4."} {"question": "2012\ub144 KBO \ub9ac\uadf8\uc5d0\uc11c \uc2b9\ubd80\uc870\uc791 \uc0ac\uac74\uc73c\ub85c \uc870\uc0ac \ubc1b\uc740 \uc120\uc218\ub294 \ubc15\ud604\uc900, \uae40\uc131\ud604\uacfc \ub610 \ub204\uad6c\uc778\uac00?", "paragraph": "2012\ub144 \ud55c\uad6d\ud504\ub85c\uc57c\uad6c \uc2b9\ubd80\uc870\uc791 \uc0ac\uac74\uc740 KBO \ub9ac\uadf8\uc5d0\uc11c \ubc8c\uc5b4\uc9c4 \uc2b9\ubd80\uc870\uc791 \uc0ac\uac74\uc774\ub2e4. \uc77c\ubd80 \uc120\uc218\uac00 \ub300\uac00\ub97c \uc9c0\ubd88\ubc1b\uace0 \uacbd\uae30 \ub0b4\uc6a9\uacfc \uc2b9\ubd80\ub97c \uc870\uc791\ud558\uc600\ub2e4. 2012\ub144 \ub300\ud55c\ubbfc\uad6d \ud504\ub85c \ucd95\uad6c\uc758 K\ub9ac\uadf8 \uc2b9\ubd80\uc870\uc791 \uc0ac\uac74\uacfc \ud504\ub85c \ubc30\uad6c\uc758 V-\ub9ac\uadf8 \uc2b9\ubd80\uc870\uc791 \uc0ac\uac74\uc758 \uc804\ubaa8\uac00 \ubc1d\ud600\uc9c0\uae30 \uc2dc\uc791\ud558\uba74\uc11c \uc0ac\uac74\uc5d0 \uc5f0\ub8e8\ub41c \ud55c \ube0c\ub85c\ucee4\uac00 \ud55c\uad6d\ud504\ub85c\uc57c\uad6c\uc5d0\uc11c\ub3c4 \uc2b9\ubd80\uc870\uc791\uc774 \uc774\ub8e8\uc5b4\uc84c\ub2e4\uace0 \ubc1d\ud788\uba74\uc11c \ubcf8\uaca9\uc801\uc73c\ub85c \ud504\ub85c \uc57c\uad6c\uc5d0\uc11c\ub3c4 \uc2b9\ubd80\uc870\uc791 \uc0ac\uac74\uc774 \uc218\uba74 \uc704\ub85c \ub5a0\uc624\ub974\uac8c \ub418\uc5c8\ub2e4. \uc774 \uacfc\uc815\uc5d0\uc11c \ub9ce\uc740 \uc120\uc218\ub4e4\uc774 \uc2b9\ubd80\uc870\uc791\uc744 \ud588\ub2e4\ub294 \uc758\uc2ec\uc744 \ubc1b\uc558\uace0, \ubb38\uc131\ud604, \ubc15\ud604\uc900, \uae40\uc131\ud604\uc774 \uac80\ucc30\uc5d0 \uc18c\ud658\ub418\uc5b4 \uc870\uc0ac\ub97c \ubc1b\uc558\ub2e4. \uc774\ub4e4 \uc911 \ubc15\ud604\uc900\uacfc \uae40\uc131\ud604\uc740 \uc2b9\ubd80\uc870\uc791 \ud610\uc758\uac00 \uc778\uc815\ub418\uc5b4 \uac01\uac01 \ubd88\uad6c\uc18d, \uad6c\uc18d \uae30\uc18c\ub418\uc5c8\uace0, KBO\ub294 \uc774 \ub450 \uc120\uc218\ub97c \uc601\uad6c\uc81c\uba85\ud588\ub2e4.", "answer": "\ubb38\uc131\ud604", "sentence": "\uc774 \uacfc\uc815\uc5d0\uc11c \ub9ce\uc740 \uc120\uc218\ub4e4\uc774 \uc2b9\ubd80\uc870\uc791\uc744 \ud588\ub2e4\ub294 \uc758\uc2ec\uc744 \ubc1b\uc558\uace0, \ubb38\uc131\ud604 ,", "paragraph_sentence": "2012\ub144 \ud55c\uad6d\ud504\ub85c\uc57c\uad6c \uc2b9\ubd80\uc870\uc791 \uc0ac\uac74\uc740 KBO \ub9ac\uadf8\uc5d0\uc11c \ubc8c\uc5b4\uc9c4 \uc2b9\ubd80\uc870\uc791 \uc0ac\uac74\uc774\ub2e4. \uc77c\ubd80 \uc120\uc218\uac00 \ub300\uac00\ub97c \uc9c0\ubd88\ubc1b\uace0 \uacbd\uae30 \ub0b4\uc6a9\uacfc \uc2b9\ubd80\ub97c \uc870\uc791\ud558\uc600\ub2e4. 2012\ub144 \ub300\ud55c\ubbfc\uad6d \ud504\ub85c \ucd95\uad6c\uc758 K\ub9ac\uadf8 \uc2b9\ubd80\uc870\uc791 \uc0ac\uac74\uacfc \ud504\ub85c \ubc30\uad6c\uc758 V-\ub9ac\uadf8 \uc2b9\ubd80\uc870\uc791 \uc0ac\uac74\uc758 \uc804\ubaa8\uac00 \ubc1d\ud600\uc9c0\uae30 \uc2dc\uc791\ud558\uba74\uc11c \uc0ac\uac74\uc5d0 \uc5f0\ub8e8\ub41c \ud55c \ube0c\ub85c\ucee4\uac00 \ud55c\uad6d\ud504\ub85c\uc57c\uad6c\uc5d0\uc11c\ub3c4 \uc2b9\ubd80\uc870\uc791\uc774 \uc774\ub8e8\uc5b4\uc84c\ub2e4\uace0 \ubc1d\ud788\uba74\uc11c \ubcf8\uaca9\uc801\uc73c\ub85c \ud504\ub85c \uc57c\uad6c\uc5d0\uc11c\ub3c4 \uc2b9\ubd80\uc870\uc791 \uc0ac\uac74\uc774 \uc218\uba74 \uc704\ub85c \ub5a0\uc624\ub974\uac8c \ub418\uc5c8\ub2e4. \uc774 \uacfc\uc815\uc5d0\uc11c \ub9ce\uc740 \uc120\uc218\ub4e4\uc774 \uc2b9\ubd80\uc870\uc791\uc744 \ud588\ub2e4\ub294 \uc758\uc2ec\uc744 \ubc1b\uc558\uace0, \ubb38\uc131\ud604 , \ubc15\ud604\uc900, \uae40\uc131\ud604\uc774 \uac80\ucc30\uc5d0 \uc18c\ud658\ub418\uc5b4 \uc870\uc0ac\ub97c \ubc1b\uc558\ub2e4. \uc774\ub4e4 \uc911 \ubc15\ud604\uc900\uacfc \uae40\uc131\ud604\uc740 \uc2b9\ubd80\uc870\uc791 \ud610\uc758\uac00 \uc778\uc815\ub418\uc5b4 \uac01\uac01 \ubd88\uad6c\uc18d, \uad6c\uc18d \uae30\uc18c\ub418\uc5c8\uace0, KBO\ub294 \uc774 \ub450 \uc120\uc218\ub97c \uc601\uad6c\uc81c\uba85\ud588\ub2e4.", "paragraph_answer": "2012\ub144 \ud55c\uad6d\ud504\ub85c\uc57c\uad6c \uc2b9\ubd80\uc870\uc791 \uc0ac\uac74\uc740 KBO \ub9ac\uadf8\uc5d0\uc11c \ubc8c\uc5b4\uc9c4 \uc2b9\ubd80\uc870\uc791 \uc0ac\uac74\uc774\ub2e4. \uc77c\ubd80 \uc120\uc218\uac00 \ub300\uac00\ub97c \uc9c0\ubd88\ubc1b\uace0 \uacbd\uae30 \ub0b4\uc6a9\uacfc \uc2b9\ubd80\ub97c \uc870\uc791\ud558\uc600\ub2e4. 2012\ub144 \ub300\ud55c\ubbfc\uad6d \ud504\ub85c \ucd95\uad6c\uc758 K\ub9ac\uadf8 \uc2b9\ubd80\uc870\uc791 \uc0ac\uac74\uacfc \ud504\ub85c \ubc30\uad6c\uc758 V-\ub9ac\uadf8 \uc2b9\ubd80\uc870\uc791 \uc0ac\uac74\uc758 \uc804\ubaa8\uac00 \ubc1d\ud600\uc9c0\uae30 \uc2dc\uc791\ud558\uba74\uc11c \uc0ac\uac74\uc5d0 \uc5f0\ub8e8\ub41c \ud55c \ube0c\ub85c\ucee4\uac00 \ud55c\uad6d\ud504\ub85c\uc57c\uad6c\uc5d0\uc11c\ub3c4 \uc2b9\ubd80\uc870\uc791\uc774 \uc774\ub8e8\uc5b4\uc84c\ub2e4\uace0 \ubc1d\ud788\uba74\uc11c \ubcf8\uaca9\uc801\uc73c\ub85c \ud504\ub85c \uc57c\uad6c\uc5d0\uc11c\ub3c4 \uc2b9\ubd80\uc870\uc791 \uc0ac\uac74\uc774 \uc218\uba74 \uc704\ub85c \ub5a0\uc624\ub974\uac8c \ub418\uc5c8\ub2e4. \uc774 \uacfc\uc815\uc5d0\uc11c \ub9ce\uc740 \uc120\uc218\ub4e4\uc774 \uc2b9\ubd80\uc870\uc791\uc744 \ud588\ub2e4\ub294 \uc758\uc2ec\uc744 \ubc1b\uc558\uace0, \ubb38\uc131\ud604 , \ubc15\ud604\uc900, \uae40\uc131\ud604\uc774 \uac80\ucc30\uc5d0 \uc18c\ud658\ub418\uc5b4 \uc870\uc0ac\ub97c \ubc1b\uc558\ub2e4. \uc774\ub4e4 \uc911 \ubc15\ud604\uc900\uacfc \uae40\uc131\ud604\uc740 \uc2b9\ubd80\uc870\uc791 \ud610\uc758\uac00 \uc778\uc815\ub418\uc5b4 \uac01\uac01 \ubd88\uad6c\uc18d, \uad6c\uc18d \uae30\uc18c\ub418\uc5c8\uace0, KBO\ub294 \uc774 \ub450 \uc120\uc218\ub97c \uc601\uad6c\uc81c\uba85\ud588\ub2e4.", "sentence_answer": "\uc774 \uacfc\uc815\uc5d0\uc11c \ub9ce\uc740 \uc120\uc218\ub4e4\uc774 \uc2b9\ubd80\uc870\uc791\uc744 \ud588\ub2e4\ub294 \uc758\uc2ec\uc744 \ubc1b\uc558\uace0, \ubb38\uc131\ud604 ,", "paragraph_id": "6527241-0-1", "paragraph_question": "question: 2012\ub144 KBO \ub9ac\uadf8\uc5d0\uc11c \uc2b9\ubd80\uc870\uc791 \uc0ac\uac74\uc73c\ub85c \uc870\uc0ac \ubc1b\uc740 \uc120\uc218\ub294 \ubc15\ud604\uc900, \uae40\uc131\ud604\uacfc \ub610 \ub204\uad6c\uc778\uac00?, context: 2012\ub144 \ud55c\uad6d\ud504\ub85c\uc57c\uad6c \uc2b9\ubd80\uc870\uc791 \uc0ac\uac74\uc740 KBO \ub9ac\uadf8\uc5d0\uc11c \ubc8c\uc5b4\uc9c4 \uc2b9\ubd80\uc870\uc791 \uc0ac\uac74\uc774\ub2e4. \uc77c\ubd80 \uc120\uc218\uac00 \ub300\uac00\ub97c \uc9c0\ubd88\ubc1b\uace0 \uacbd\uae30 \ub0b4\uc6a9\uacfc \uc2b9\ubd80\ub97c \uc870\uc791\ud558\uc600\ub2e4. 2012\ub144 \ub300\ud55c\ubbfc\uad6d \ud504\ub85c \ucd95\uad6c\uc758 K\ub9ac\uadf8 \uc2b9\ubd80\uc870\uc791 \uc0ac\uac74\uacfc \ud504\ub85c \ubc30\uad6c\uc758 V-\ub9ac\uadf8 \uc2b9\ubd80\uc870\uc791 \uc0ac\uac74\uc758 \uc804\ubaa8\uac00 \ubc1d\ud600\uc9c0\uae30 \uc2dc\uc791\ud558\uba74\uc11c \uc0ac\uac74\uc5d0 \uc5f0\ub8e8\ub41c \ud55c \ube0c\ub85c\ucee4\uac00 \ud55c\uad6d\ud504\ub85c\uc57c\uad6c\uc5d0\uc11c\ub3c4 \uc2b9\ubd80\uc870\uc791\uc774 \uc774\ub8e8\uc5b4\uc84c\ub2e4\uace0 \ubc1d\ud788\uba74\uc11c \ubcf8\uaca9\uc801\uc73c\ub85c \ud504\ub85c \uc57c\uad6c\uc5d0\uc11c\ub3c4 \uc2b9\ubd80\uc870\uc791 \uc0ac\uac74\uc774 \uc218\uba74 \uc704\ub85c \ub5a0\uc624\ub974\uac8c \ub418\uc5c8\ub2e4. \uc774 \uacfc\uc815\uc5d0\uc11c \ub9ce\uc740 \uc120\uc218\ub4e4\uc774 \uc2b9\ubd80\uc870\uc791\uc744 \ud588\ub2e4\ub294 \uc758\uc2ec\uc744 \ubc1b\uc558\uace0, \ubb38\uc131\ud604, \ubc15\ud604\uc900, \uae40\uc131\ud604\uc774 \uac80\ucc30\uc5d0 \uc18c\ud658\ub418\uc5b4 \uc870\uc0ac\ub97c \ubc1b\uc558\ub2e4. \uc774\ub4e4 \uc911 \ubc15\ud604\uc900\uacfc \uae40\uc131\ud604\uc740 \uc2b9\ubd80\uc870\uc791 \ud610\uc758\uac00 \uc778\uc815\ub418\uc5b4 \uac01\uac01 \ubd88\uad6c\uc18d, \uad6c\uc18d \uae30\uc18c\ub418\uc5c8\uace0, KBO\ub294 \uc774 \ub450 \uc120\uc218\ub97c \uc601\uad6c\uc81c\uba85\ud588\ub2e4."} {"question": "\uc54c\ubaac\ub4dc \uc18c\uc7a5\uc758 \ud3ec\uc704\uc791\uc804\uc5d0 \uc5b4\uadf8\ub7ec\uc9c4 \uccab\ub2e8\ucd94\ub97c \ub07c\uc6b4 \ubd80\ub300\ub294?", "paragraph": "10\uad70\ub2e8\uc7a5 \uc54c\ubaac\ub4dc \uc18c\uc7a5\uc758 \uba85\ub839\uc73c\ub85c \ud574\ubcd1 1\uc0ac\ub2e8\uc740 \ub2f9\uc77c\ubd80\ud130 5\uc5f0\ub300\ub97c \uc8fc\uacf5\uc73c\ub85c \ud3ec\uc704\uae30\ub3d9\uc758 \ubd81\ucabd \ub0a0\uac1c\ub85c\uc368 \ubbf8 8\uad70\uc744 \ud3ec\uc704\ud558\uace0 \uc788\ub358 \uc911\uacf5\uad70\uc744 \uaca9\ud1f4\ud558\uace0 \ubbf8 8\uad70\uacfc \ud568\uaed8 \ub0ad\ub9bc\uc0b0\ub9e5 \uc11c\ucabd\uc73c\ub85c \uacf5\uaca9\ud560 \uc608\uc815\uc774\uc5c8\uc73c\ub098 \uacc4\ud68d\uacfc \ub2ec\ub9ac \ud3ec\uc704\uc791\uc804\uc758 \ubc29\uc5b4\ubd80\ub300\uc778 \ubbf8 8\uad70\uc774 \uc911\uacf5\uad70\uc758 \uacf5\uaca9\uc73c\ub85c \ud6c4\ud1f4\ud558\uae30 \uc2dc\uc791\ud558\uc600\ub2e4. \uc911\uacf5\uad70\uc740 9\ubcd1\ub2e8 8\uac1c \uc0ac\ub2e8 \uc57d 6\ub9cc\uc5ec \uba85\uc758 \ubcd1\ub825\uc744 \uc7a5\uc9c4\ud638\uc5d0 \uc9d1\uacb0\uc2dc\ucf30\ub2e4. \uc774\uc911 3\uac1c \uc0ac\ub2e8\uc740 \uc720\ub2f4\ub9ac\uc5d0 \ub300\ud55c \uacf5\uaca9\uc744 \uc900\ube44 \uc911\uc774\uc5c8\uace0 1\uac1c \uc0ac\ub2e8(80\uc0ac\ub2e8)\uc740 \ud558\uac08\uc6b0\ub9ac \ud3ec\uc704\ub97c \uc704\ud574 \ud574\ubcd1 1\uc0ac\ub2e8\uc758 \uc6b0\uce21 \ubc29\uc5b4\ub97c \ub2f4\ub2f9\ud558\uace0 \uc788\ub358 \ubbf8 7\ubcf4\ubcd1\uc0ac\ub2e8 31\uc5f0\ub300\uc804\ud22c\ub2e8\uc5d0 \ub300\ud55c \uacf5\uaca9\uc744 \uc900\ube44 \uc911\uc774\uc5c8\ub2e4. \ub2f9\uc77c \uc5f0\ud569\uad70 \ubcd1\ub825\uc740 \ud574\ubcd1\uacfc \ud574\uad70\uc704\uc0dd\ubcd1 13,500\uba85\uacfc \uc721\uad70 4,500\uba85\uc5d0 \ubd88\uacfc\ud558\uc600\ub2e4.", "answer": "\ubbf8 8\uad70", "sentence": "10\uad70\ub2e8\uc7a5 \uc54c\ubaac\ub4dc \uc18c\uc7a5\uc758 \uba85\ub839\uc73c\ub85c \ud574\ubcd1 1\uc0ac\ub2e8\uc740 \ub2f9\uc77c\ubd80\ud130 5\uc5f0\ub300\ub97c \uc8fc\uacf5\uc73c\ub85c \ud3ec\uc704\uae30\ub3d9\uc758 \ubd81\ucabd \ub0a0\uac1c\ub85c\uc368 \ubbf8 8\uad70 \uc744 \ud3ec\uc704\ud558\uace0 \uc788\ub358 \uc911\uacf5\uad70\uc744 \uaca9\ud1f4\ud558\uace0 \ubbf8 8\uad70\uacfc \ud568\uaed8 \ub0ad\ub9bc\uc0b0\ub9e5 \uc11c\ucabd\uc73c\ub85c \uacf5\uaca9\ud560 \uc608\uc815\uc774\uc5c8\uc73c\ub098 \uacc4\ud68d\uacfc \ub2ec\ub9ac \ud3ec\uc704\uc791\uc804\uc758 \ubc29\uc5b4\ubd80\ub300\uc778 \ubbf8 8\uad70\uc774 \uc911\uacf5\uad70\uc758 \uacf5\uaca9\uc73c\ub85c \ud6c4\ud1f4\ud558\uae30 \uc2dc\uc791\ud558\uc600\ub2e4.", "paragraph_sentence": " 10\uad70\ub2e8\uc7a5 \uc54c\ubaac\ub4dc \uc18c\uc7a5\uc758 \uba85\ub839\uc73c\ub85c \ud574\ubcd1 1\uc0ac\ub2e8\uc740 \ub2f9\uc77c\ubd80\ud130 5\uc5f0\ub300\ub97c \uc8fc\uacf5\uc73c\ub85c \ud3ec\uc704\uae30\ub3d9\uc758 \ubd81\ucabd \ub0a0\uac1c\ub85c\uc368 \ubbf8 8\uad70 \uc744 \ud3ec\uc704\ud558\uace0 \uc788\ub358 \uc911\uacf5\uad70\uc744 \uaca9\ud1f4\ud558\uace0 \ubbf8 8\uad70\uacfc \ud568\uaed8 \ub0ad\ub9bc\uc0b0\ub9e5 \uc11c\ucabd\uc73c\ub85c \uacf5\uaca9\ud560 \uc608\uc815\uc774\uc5c8\uc73c\ub098 \uacc4\ud68d\uacfc \ub2ec\ub9ac \ud3ec\uc704\uc791\uc804\uc758 \ubc29\uc5b4\ubd80\ub300\uc778 \ubbf8 8\uad70\uc774 \uc911\uacf5\uad70\uc758 \uacf5\uaca9\uc73c\ub85c \ud6c4\ud1f4\ud558\uae30 \uc2dc\uc791\ud558\uc600\ub2e4. \uc911\uacf5\uad70\uc740 9\ubcd1\ub2e8 8\uac1c \uc0ac\ub2e8 \uc57d 6\ub9cc\uc5ec \uba85\uc758 \ubcd1\ub825\uc744 \uc7a5\uc9c4\ud638\uc5d0 \uc9d1\uacb0\uc2dc\ucf30\ub2e4. \uc774\uc911 3\uac1c \uc0ac\ub2e8\uc740 \uc720\ub2f4\ub9ac\uc5d0 \ub300\ud55c \uacf5\uaca9\uc744 \uc900\ube44 \uc911\uc774\uc5c8\uace0 1\uac1c \uc0ac\ub2e8(80\uc0ac\ub2e8)\uc740 \ud558\uac08\uc6b0\ub9ac \ud3ec\uc704\ub97c \uc704\ud574 \ud574\ubcd1 1\uc0ac\ub2e8\uc758 \uc6b0\uce21 \ubc29\uc5b4\ub97c \ub2f4\ub2f9\ud558\uace0 \uc788\ub358 \ubbf8 7\ubcf4\ubcd1\uc0ac\ub2e8 31\uc5f0\ub300\uc804\ud22c\ub2e8\uc5d0 \ub300\ud55c \uacf5\uaca9\uc744 \uc900\ube44 \uc911\uc774\uc5c8\ub2e4. \ub2f9\uc77c \uc5f0\ud569\uad70 \ubcd1\ub825\uc740 \ud574\ubcd1\uacfc \ud574\uad70\uc704\uc0dd\ubcd1 13,500\uba85\uacfc \uc721\uad70 4,500\uba85\uc5d0 \ubd88\uacfc\ud558\uc600\ub2e4.", "paragraph_answer": "10\uad70\ub2e8\uc7a5 \uc54c\ubaac\ub4dc \uc18c\uc7a5\uc758 \uba85\ub839\uc73c\ub85c \ud574\ubcd1 1\uc0ac\ub2e8\uc740 \ub2f9\uc77c\ubd80\ud130 5\uc5f0\ub300\ub97c \uc8fc\uacf5\uc73c\ub85c \ud3ec\uc704\uae30\ub3d9\uc758 \ubd81\ucabd \ub0a0\uac1c\ub85c\uc368 \ubbf8 8\uad70 \uc744 \ud3ec\uc704\ud558\uace0 \uc788\ub358 \uc911\uacf5\uad70\uc744 \uaca9\ud1f4\ud558\uace0 \ubbf8 8\uad70\uacfc \ud568\uaed8 \ub0ad\ub9bc\uc0b0\ub9e5 \uc11c\ucabd\uc73c\ub85c \uacf5\uaca9\ud560 \uc608\uc815\uc774\uc5c8\uc73c\ub098 \uacc4\ud68d\uacfc \ub2ec\ub9ac \ud3ec\uc704\uc791\uc804\uc758 \ubc29\uc5b4\ubd80\ub300\uc778 \ubbf8 8\uad70\uc774 \uc911\uacf5\uad70\uc758 \uacf5\uaca9\uc73c\ub85c \ud6c4\ud1f4\ud558\uae30 \uc2dc\uc791\ud558\uc600\ub2e4. \uc911\uacf5\uad70\uc740 9\ubcd1\ub2e8 8\uac1c \uc0ac\ub2e8 \uc57d 6\ub9cc\uc5ec \uba85\uc758 \ubcd1\ub825\uc744 \uc7a5\uc9c4\ud638\uc5d0 \uc9d1\uacb0\uc2dc\ucf30\ub2e4. \uc774\uc911 3\uac1c \uc0ac\ub2e8\uc740 \uc720\ub2f4\ub9ac\uc5d0 \ub300\ud55c \uacf5\uaca9\uc744 \uc900\ube44 \uc911\uc774\uc5c8\uace0 1\uac1c \uc0ac\ub2e8(80\uc0ac\ub2e8)\uc740 \ud558\uac08\uc6b0\ub9ac \ud3ec\uc704\ub97c \uc704\ud574 \ud574\ubcd1 1\uc0ac\ub2e8\uc758 \uc6b0\uce21 \ubc29\uc5b4\ub97c \ub2f4\ub2f9\ud558\uace0 \uc788\ub358 \ubbf8 7\ubcf4\ubcd1\uc0ac\ub2e8 31\uc5f0\ub300\uc804\ud22c\ub2e8\uc5d0 \ub300\ud55c \uacf5\uaca9\uc744 \uc900\ube44 \uc911\uc774\uc5c8\ub2e4. \ub2f9\uc77c \uc5f0\ud569\uad70 \ubcd1\ub825\uc740 \ud574\ubcd1\uacfc \ud574\uad70\uc704\uc0dd\ubcd1 13,500\uba85\uacfc \uc721\uad70 4,500\uba85\uc5d0 \ubd88\uacfc\ud558\uc600\ub2e4.", "sentence_answer": "10\uad70\ub2e8\uc7a5 \uc54c\ubaac\ub4dc \uc18c\uc7a5\uc758 \uba85\ub839\uc73c\ub85c \ud574\ubcd1 1\uc0ac\ub2e8\uc740 \ub2f9\uc77c\ubd80\ud130 5\uc5f0\ub300\ub97c \uc8fc\uacf5\uc73c\ub85c \ud3ec\uc704\uae30\ub3d9\uc758 \ubd81\ucabd \ub0a0\uac1c\ub85c\uc368 \ubbf8 8\uad70 \uc744 \ud3ec\uc704\ud558\uace0 \uc788\ub358 \uc911\uacf5\uad70\uc744 \uaca9\ud1f4\ud558\uace0 \ubbf8 8\uad70\uacfc \ud568\uaed8 \ub0ad\ub9bc\uc0b0\ub9e5 \uc11c\ucabd\uc73c\ub85c \uacf5\uaca9\ud560 \uc608\uc815\uc774\uc5c8\uc73c\ub098 \uacc4\ud68d\uacfc \ub2ec\ub9ac \ud3ec\uc704\uc791\uc804\uc758 \ubc29\uc5b4\ubd80\ub300\uc778 \ubbf8 8\uad70\uc774 \uc911\uacf5\uad70\uc758 \uacf5\uaca9\uc73c\ub85c \ud6c4\ud1f4\ud558\uae30 \uc2dc\uc791\ud558\uc600\ub2e4.", "paragraph_id": "6549188-5-1", "paragraph_question": "question: \uc54c\ubaac\ub4dc \uc18c\uc7a5\uc758 \ud3ec\uc704\uc791\uc804\uc5d0 \uc5b4\uadf8\ub7ec\uc9c4 \uccab\ub2e8\ucd94\ub97c \ub07c\uc6b4 \ubd80\ub300\ub294?, context: 10\uad70\ub2e8\uc7a5 \uc54c\ubaac\ub4dc \uc18c\uc7a5\uc758 \uba85\ub839\uc73c\ub85c \ud574\ubcd1 1\uc0ac\ub2e8\uc740 \ub2f9\uc77c\ubd80\ud130 5\uc5f0\ub300\ub97c \uc8fc\uacf5\uc73c\ub85c \ud3ec\uc704\uae30\ub3d9\uc758 \ubd81\ucabd \ub0a0\uac1c\ub85c\uc368 \ubbf8 8\uad70\uc744 \ud3ec\uc704\ud558\uace0 \uc788\ub358 \uc911\uacf5\uad70\uc744 \uaca9\ud1f4\ud558\uace0 \ubbf8 8\uad70\uacfc \ud568\uaed8 \ub0ad\ub9bc\uc0b0\ub9e5 \uc11c\ucabd\uc73c\ub85c \uacf5\uaca9\ud560 \uc608\uc815\uc774\uc5c8\uc73c\ub098 \uacc4\ud68d\uacfc \ub2ec\ub9ac \ud3ec\uc704\uc791\uc804\uc758 \ubc29\uc5b4\ubd80\ub300\uc778 \ubbf8 8\uad70\uc774 \uc911\uacf5\uad70\uc758 \uacf5\uaca9\uc73c\ub85c \ud6c4\ud1f4\ud558\uae30 \uc2dc\uc791\ud558\uc600\ub2e4. \uc911\uacf5\uad70\uc740 9\ubcd1\ub2e8 8\uac1c \uc0ac\ub2e8 \uc57d 6\ub9cc\uc5ec \uba85\uc758 \ubcd1\ub825\uc744 \uc7a5\uc9c4\ud638\uc5d0 \uc9d1\uacb0\uc2dc\ucf30\ub2e4. \uc774\uc911 3\uac1c \uc0ac\ub2e8\uc740 \uc720\ub2f4\ub9ac\uc5d0 \ub300\ud55c \uacf5\uaca9\uc744 \uc900\ube44 \uc911\uc774\uc5c8\uace0 1\uac1c \uc0ac\ub2e8(80\uc0ac\ub2e8)\uc740 \ud558\uac08\uc6b0\ub9ac \ud3ec\uc704\ub97c \uc704\ud574 \ud574\ubcd1 1\uc0ac\ub2e8\uc758 \uc6b0\uce21 \ubc29\uc5b4\ub97c \ub2f4\ub2f9\ud558\uace0 \uc788\ub358 \ubbf8 7\ubcf4\ubcd1\uc0ac\ub2e8 31\uc5f0\ub300\uc804\ud22c\ub2e8\uc5d0 \ub300\ud55c \uacf5\uaca9\uc744 \uc900\ube44 \uc911\uc774\uc5c8\ub2e4. \ub2f9\uc77c \uc5f0\ud569\uad70 \ubcd1\ub825\uc740 \ud574\ubcd1\uacfc \ud574\uad70\uc704\uc0dd\ubcd1 13,500\uba85\uacfc \uc721\uad70 4,500\uba85\uc5d0 \ubd88\uacfc\ud558\uc600\ub2e4."} {"question": "\uc194\ub85c\ub300\ucca9 \ucc38\uac00\uc790\uc758 \ub300\ubd80\ubd84\uc744 \ucc28\uc9c0\ud55c \uac83\uc740?", "paragraph": "\ub300\ud55c\ubbfc\uad6d \ucd5c\ucd08 \ud2b9\ubcc4\ud55c \ub0a0 \"\uc194\ub85c\ub300\ucca9\" \ud589\uc0ac\ub294 \uc11c\uc6b8\ud2b9\ubcc4\uc2dc\uc758 \uacbd\uc6b0 \uc2dc\uc98c\uc81c \ud06c\ub9ac\uc2a4\ub9c8\uc2a4 \uae30\uac04 \uc911 2012\ub144 12\uc6d4 24\uc77c\uc6d4\uc694\uc77c \ud3c9\uc77c/\uc784\uc2dc \uc790\uc728 1\uc77c \ud734\uc77c\uc81c \ud06c\ub9ac\uc2a4\ub9c8\uc2a4 \uc774\ube0c*\uc131\ud0c4\uc804\uc57c\uc81c \ub370\uc774* \uc624\ud6c43~5\uc2dc(\uc57d2\uc2dc\uac04 \ub3d9\uc548)\uc5d0 \ub0a8\uc790\ub4e4\uc740 \ud558\uc580\uc0c9, \uc5ec\uc790\ub4e4\uc740 \ubd89\uc740\uc0c9, \ucee4\ud50c\uc740 \ucd08\ub85d\uc0c9 \uacc4\uc5f4\uc758 \uc637\uc744 \uc785\uace0 \uc11c\uc6b8\uc5ec\uc758\ub3c4\uacf5\uc6d0\uc73c\ub85c \ubaa8\uc774\ub294 \uac83\uc73c\ub85c \uc2dc\uc791\ud558\uc600\ub2e4. \uacf5\uc6d0\uc5d0 \ubaa8\uc778 \ub0a8\ub140\ub294 \uc2e0\ud638 \uc591\ud3b8\uc5d0\uc11c \ub300\uae30\ud558\ub2e4\uac00 \uc9c4\ud589\uc790\uac00 \uc2e0\ud638\ub97c \ubcf4\ub0b4\uba74 \ub9c8\uc74c\uc5d0 \ub4dc\ub294 \uc774\uc131(\uc5ec\ub294 \ub0a8\uc73c\ub85c, \ub0a8\uc740 \uc5ec\ub85c)\uc744 \ud5a5\ud574 \ub2ec\ub824\uac00 \uc190\uc744 \uc7a1\ub294 \ud615\uc2dd\uc73c\ub85c \uc9c4\ud589\ub418\uc5c8\ub2e4. \uc2e4\uc81c \uc774 \ud589\uc0ac\ub97c \ud1b5\ud574 \uba87 \uc30d\uc758 \ucee4\ud50c\uc774 \ud0c4\uc0dd\ud558\uae30\ub3c4 \ud558\uc600\ub2e4. \ud558\uc9c0\ub9cc \uc548\uc804\uc0c1\uc758 \uc774\uc720 \ub4f1\uc73c\ub85c \uc0c1\ub2f9\uc218\uc758 \uc5ec\uc131 \ucc38\uac00 \uc2e0\uccad\uc790\ub4e4\uc774 \uc2e4\uc81c \ud589\uc0ac\uc5d0 \ucc38\uac00\ud558\uc9c0 \uc54a\uc74c\uc73c\ub85c \uc778\ud574, \ud589\uc0ac \ucc38\uac00\uc790 \uc57d 3,000\uba85 \uc911 \ub300\ubd80\ubd84\uc774 \ub0a8\uc131\uc73c\ub85c \ucc44\uc6cc\uc9c0\ub294 \ud604\uc0c1\uc774 \ubc1c\uc0dd\ud588\ub2e4. \ud55c\ud3b8 \uacbd\ucc30 \uce21 \ucd94\uc0b0\uc5d0 \ub530\ub974\uba74 \uc774\ub0a0 \uc11c\uc6b8 \uc5ec\uc758\ub3c4 \uacf5\uc6d0\uc5d0\uc11c \uc9c4\ud589\ub41c \ud589\uc0ac\uc5d0\ub294 \ub0a8\uc131 700\uba85\uacfc \uc5ec\uc131 300\uba85 \ub4f1 \uc57d 1\ucc9c\uc5ec \uba85\uc774 \ucc38\uac00\ud55c \uac83\uc73c\ub85c \uc804\ud574\uc84c\ub2e4. \u201c\ub0a8\uc790\uc640 \uc5ec\uc790\uac00 \ub300\uce58\ud55c \uac83\uc774 \uc544\ub2c8\ub77c \uacbd\ucc30\uacfc \ube44\ub458\uae30\uac00 \ub300\uce58\ud558\ub294 \ubd84\uc704\uae30\u201d\ub77c\ub294 \uae00\uc774 \uc62c\ub77c\uc624\uba70 \ub2f9\uc2dc \uadf9\uba85\ud55c \ub0a8\ub140 \ube44\uc728\uc744 \uc124\uba85\ud558\uae30\ub3c4 \ud588\ub2e4. \uc9dd\uc744 \ucc3e\uc9c0 \ubabb\ud55c \ub300\ubd80\ubd84\uc758 \ucc38\uac00\uc790\ub4e4\uc774 \uadfc\ucc98 \uc74c\uc2dd\uc810, \uc220\uc9d1 \ub4f1\uc73c\ub85c \ud769\uc5b4\uc84c\ub2e4. \uc774 \ud589\uc0ac\uc5d0 \ucc38\uac00\ud588\ub358 \uac1c\uadf8\ub9e8 \ubc15\ud718\uc21c\uc740 \"\uc194\ub85c\ub300\ucca9\uc774 \uc220\ub85c\ub300\ucca9\uc73c\ub85c \uc774\uc5b4\uc9c0\uaca0\ub124\uc694\"\ub77c\ub294 \uae00\uc744 \ub0a8\uae30\uae30\ub3c4 \ud588\ub2e4. \uc218\ub3c4\uad8c\uacfc \uc9c0\ubc29\uc758 \uacbd\uc6b0 \uc5ec\uc131 \ucc38\uac00\uc790\uac00 \ub354 \uc801\uc5b4 \ud589\uc0ac \uc790\uccb4\uac00 \uc81c\ub300\ub85c \uc9c4\ud589\ub418\uc9c0 \ubabb\ud55c \uacbd\uc6b0\ub3c4 \uc788\uc5c8\ub2e4. \ucc9c\uc548 \uc194\ub85c\ub300\ucca9\uc758 \uacbd\uc6b0 \uc5ec\uc131 \ucc38\uac00\uc790\uac00 10\uc5ec \uba85 \uc815\ub3c4\uc5d0 \ubd88\uacfc\ud574 \ud589\uc0ac\ub97c \uce58\ub8e8\uc9c0 \ubabb\ud588\uc9c0\ub9cc, \uac1c\uc778\uc801\uc73c\ub85c \ubc88\ud638\ub97c \uc8fc\uace0\ubc1b\uc544 1\ud638 \ucee4\ud50c\uc774 \ud798\uacb9\uac8c \ud0c4\uc0dd\ub418\uae30\ub3c4 \ud588\ub2e4. \ud55c\ud3b8 \ub300\ubd80\ubd84\uc758 \uc5ec\uc131 \uc5f0\uc608\uc778\ub4e4\uc774 \uc548\uc804\uc0c1\uc758 \uc774\uc720\ub85c \ud589\uc0ac \ubd88\ucc38\uc744 \uc120\uc5b8\ud55c \uac00\uc6b4\ub370, \uc2e0\uc778 \uac78\uadf8\ub8f9 \ube44\ud0a4\ub2c8\uc758 \uba64\ubc84\uc778 \ud574\uc774\uac00 \uc194\ub85c\ub300\ucca9\uc5d0 \ucc38\uac00\ud558\uc5ec \uc2e4\uc2dc\uac04 \uac80\uc0c9 1\uc704\ub97c \ucc28\uc9c0\ud558\uae30\ub3c4 \ud558\uc600\ub2e4.", "answer": "\ub0a8\uc131", "sentence": "\ud558\uc9c0\ub9cc \uc548\uc804\uc0c1\uc758 \uc774\uc720 \ub4f1\uc73c\ub85c \uc0c1\ub2f9\uc218\uc758 \uc5ec\uc131 \ucc38\uac00 \uc2e0\uccad\uc790\ub4e4\uc774 \uc2e4\uc81c \ud589\uc0ac\uc5d0 \ucc38\uac00\ud558\uc9c0 \uc54a\uc74c\uc73c\ub85c \uc778\ud574, \ud589\uc0ac \ucc38\uac00\uc790 \uc57d 3,000\uba85 \uc911 \ub300\ubd80\ubd84\uc774 \ub0a8\uc131 \uc73c\ub85c \ucc44\uc6cc\uc9c0\ub294 \ud604\uc0c1\uc774 \ubc1c\uc0dd\ud588\ub2e4.", "paragraph_sentence": "\ub300\ud55c\ubbfc\uad6d \ucd5c\ucd08 \ud2b9\ubcc4\ud55c \ub0a0 \"\uc194\ub85c\ub300\ucca9\" \ud589\uc0ac\ub294 \uc11c\uc6b8\ud2b9\ubcc4\uc2dc\uc758 \uacbd\uc6b0 \uc2dc\uc98c\uc81c \ud06c\ub9ac\uc2a4\ub9c8\uc2a4 \uae30\uac04 \uc911 2012\ub144 12\uc6d4 24\uc77c\uc6d4\uc694\uc77c \ud3c9\uc77c/\uc784\uc2dc \uc790\uc728 1\uc77c \ud734\uc77c\uc81c \ud06c\ub9ac\uc2a4\ub9c8\uc2a4 \uc774\ube0c*\uc131\ud0c4\uc804\uc57c\uc81c \ub370\uc774* \uc624\ud6c43~5\uc2dc(\uc57d2\uc2dc\uac04 \ub3d9\uc548)\uc5d0 \ub0a8\uc790\ub4e4\uc740 \ud558\uc580\uc0c9, \uc5ec\uc790\ub4e4\uc740 \ubd89\uc740\uc0c9, \ucee4\ud50c\uc740 \ucd08\ub85d\uc0c9 \uacc4\uc5f4\uc758 \uc637\uc744 \uc785\uace0 \uc11c\uc6b8\uc5ec\uc758\ub3c4\uacf5\uc6d0\uc73c\ub85c \ubaa8\uc774\ub294 \uac83\uc73c\ub85c \uc2dc\uc791\ud558\uc600\ub2e4. \uacf5\uc6d0\uc5d0 \ubaa8\uc778 \ub0a8\ub140\ub294 \uc2e0\ud638 \uc591\ud3b8\uc5d0\uc11c \ub300\uae30\ud558\ub2e4\uac00 \uc9c4\ud589\uc790\uac00 \uc2e0\ud638\ub97c \ubcf4\ub0b4\uba74 \ub9c8\uc74c\uc5d0 \ub4dc\ub294 \uc774\uc131(\uc5ec\ub294 \ub0a8\uc73c\ub85c, \ub0a8\uc740 \uc5ec\ub85c)\uc744 \ud5a5\ud574 \ub2ec\ub824\uac00 \uc190\uc744 \uc7a1\ub294 \ud615\uc2dd\uc73c\ub85c \uc9c4\ud589\ub418\uc5c8\ub2e4. \uc2e4\uc81c \uc774 \ud589\uc0ac\ub97c \ud1b5\ud574 \uba87 \uc30d\uc758 \ucee4\ud50c\uc774 \ud0c4\uc0dd\ud558\uae30\ub3c4 \ud558\uc600\ub2e4. \ud558\uc9c0\ub9cc \uc548\uc804\uc0c1\uc758 \uc774\uc720 \ub4f1\uc73c\ub85c \uc0c1\ub2f9\uc218\uc758 \uc5ec\uc131 \ucc38\uac00 \uc2e0\uccad\uc790\ub4e4\uc774 \uc2e4\uc81c \ud589\uc0ac\uc5d0 \ucc38\uac00\ud558\uc9c0 \uc54a\uc74c\uc73c\ub85c \uc778\ud574, \ud589\uc0ac \ucc38\uac00\uc790 \uc57d 3,000\uba85 \uc911 \ub300\ubd80\ubd84\uc774 \ub0a8\uc131 \uc73c\ub85c \ucc44\uc6cc\uc9c0\ub294 \ud604\uc0c1\uc774 \ubc1c\uc0dd\ud588\ub2e4. \ud55c\ud3b8 \uacbd\ucc30 \uce21 \ucd94\uc0b0\uc5d0 \ub530\ub974\uba74 \uc774\ub0a0 \uc11c\uc6b8 \uc5ec\uc758\ub3c4 \uacf5\uc6d0\uc5d0\uc11c \uc9c4\ud589\ub41c \ud589\uc0ac\uc5d0\ub294 \ub0a8\uc131 700\uba85\uacfc \uc5ec\uc131 300\uba85 \ub4f1 \uc57d 1\ucc9c\uc5ec \uba85\uc774 \ucc38\uac00\ud55c \uac83\uc73c\ub85c \uc804\ud574\uc84c\ub2e4. \u201c\ub0a8\uc790\uc640 \uc5ec\uc790\uac00 \ub300\uce58\ud55c \uac83\uc774 \uc544\ub2c8\ub77c \uacbd\ucc30\uacfc \ube44\ub458\uae30\uac00 \ub300\uce58\ud558\ub294 \ubd84\uc704\uae30\u201d\ub77c\ub294 \uae00\uc774 \uc62c\ub77c\uc624\uba70 \ub2f9\uc2dc \uadf9\uba85\ud55c \ub0a8\ub140 \ube44\uc728\uc744 \uc124\uba85\ud558\uae30\ub3c4 \ud588\ub2e4. \uc9dd\uc744 \ucc3e\uc9c0 \ubabb\ud55c \ub300\ubd80\ubd84\uc758 \ucc38\uac00\uc790\ub4e4\uc774 \uadfc\ucc98 \uc74c\uc2dd\uc810, \uc220\uc9d1 \ub4f1\uc73c\ub85c \ud769\uc5b4\uc84c\ub2e4. \uc774 \ud589\uc0ac\uc5d0 \ucc38\uac00\ud588\ub358 \uac1c\uadf8\ub9e8 \ubc15\ud718\uc21c\uc740 \"\uc194\ub85c\ub300\ucca9\uc774 \uc220\ub85c\ub300\ucca9\uc73c\ub85c \uc774\uc5b4\uc9c0\uaca0\ub124\uc694\"\ub77c\ub294 \uae00\uc744 \ub0a8\uae30\uae30\ub3c4 \ud588\ub2e4. \uc218\ub3c4\uad8c\uacfc \uc9c0\ubc29\uc758 \uacbd\uc6b0 \uc5ec\uc131 \ucc38\uac00\uc790\uac00 \ub354 \uc801\uc5b4 \ud589\uc0ac \uc790\uccb4\uac00 \uc81c\ub300\ub85c \uc9c4\ud589\ub418\uc9c0 \ubabb\ud55c \uacbd\uc6b0\ub3c4 \uc788\uc5c8\ub2e4. \ucc9c\uc548 \uc194\ub85c\ub300\ucca9\uc758 \uacbd\uc6b0 \uc5ec\uc131 \ucc38\uac00\uc790\uac00 10\uc5ec \uba85 \uc815\ub3c4\uc5d0 \ubd88\uacfc\ud574 \ud589\uc0ac\ub97c \uce58\ub8e8\uc9c0 \ubabb\ud588\uc9c0\ub9cc, \uac1c\uc778\uc801\uc73c\ub85c \ubc88\ud638\ub97c \uc8fc\uace0\ubc1b\uc544 1\ud638 \ucee4\ud50c\uc774 \ud798\uacb9\uac8c \ud0c4\uc0dd\ub418\uae30\ub3c4 \ud588\ub2e4. \ud55c\ud3b8 \ub300\ubd80\ubd84\uc758 \uc5ec\uc131 \uc5f0\uc608\uc778\ub4e4\uc774 \uc548\uc804\uc0c1\uc758 \uc774\uc720\ub85c \ud589\uc0ac \ubd88\ucc38\uc744 \uc120\uc5b8\ud55c \uac00\uc6b4\ub370, \uc2e0\uc778 \uac78\uadf8\ub8f9 \ube44\ud0a4\ub2c8\uc758 \uba64\ubc84\uc778 \ud574\uc774\uac00 \uc194\ub85c\ub300\ucca9\uc5d0 \ucc38\uac00\ud558\uc5ec \uc2e4\uc2dc\uac04 \uac80\uc0c9 1\uc704\ub97c \ucc28\uc9c0\ud558\uae30\ub3c4 \ud558\uc600\ub2e4.", "paragraph_answer": "\ub300\ud55c\ubbfc\uad6d \ucd5c\ucd08 \ud2b9\ubcc4\ud55c \ub0a0 \"\uc194\ub85c\ub300\ucca9\" \ud589\uc0ac\ub294 \uc11c\uc6b8\ud2b9\ubcc4\uc2dc\uc758 \uacbd\uc6b0 \uc2dc\uc98c\uc81c \ud06c\ub9ac\uc2a4\ub9c8\uc2a4 \uae30\uac04 \uc911 2012\ub144 12\uc6d4 24\uc77c\uc6d4\uc694\uc77c \ud3c9\uc77c/\uc784\uc2dc \uc790\uc728 1\uc77c \ud734\uc77c\uc81c \ud06c\ub9ac\uc2a4\ub9c8\uc2a4 \uc774\ube0c*\uc131\ud0c4\uc804\uc57c\uc81c \ub370\uc774* \uc624\ud6c43~5\uc2dc(\uc57d2\uc2dc\uac04 \ub3d9\uc548)\uc5d0 \ub0a8\uc790\ub4e4\uc740 \ud558\uc580\uc0c9, \uc5ec\uc790\ub4e4\uc740 \ubd89\uc740\uc0c9, \ucee4\ud50c\uc740 \ucd08\ub85d\uc0c9 \uacc4\uc5f4\uc758 \uc637\uc744 \uc785\uace0 \uc11c\uc6b8\uc5ec\uc758\ub3c4\uacf5\uc6d0\uc73c\ub85c \ubaa8\uc774\ub294 \uac83\uc73c\ub85c \uc2dc\uc791\ud558\uc600\ub2e4. \uacf5\uc6d0\uc5d0 \ubaa8\uc778 \ub0a8\ub140\ub294 \uc2e0\ud638 \uc591\ud3b8\uc5d0\uc11c \ub300\uae30\ud558\ub2e4\uac00 \uc9c4\ud589\uc790\uac00 \uc2e0\ud638\ub97c \ubcf4\ub0b4\uba74 \ub9c8\uc74c\uc5d0 \ub4dc\ub294 \uc774\uc131(\uc5ec\ub294 \ub0a8\uc73c\ub85c, \ub0a8\uc740 \uc5ec\ub85c)\uc744 \ud5a5\ud574 \ub2ec\ub824\uac00 \uc190\uc744 \uc7a1\ub294 \ud615\uc2dd\uc73c\ub85c \uc9c4\ud589\ub418\uc5c8\ub2e4. \uc2e4\uc81c \uc774 \ud589\uc0ac\ub97c \ud1b5\ud574 \uba87 \uc30d\uc758 \ucee4\ud50c\uc774 \ud0c4\uc0dd\ud558\uae30\ub3c4 \ud558\uc600\ub2e4. \ud558\uc9c0\ub9cc \uc548\uc804\uc0c1\uc758 \uc774\uc720 \ub4f1\uc73c\ub85c \uc0c1\ub2f9\uc218\uc758 \uc5ec\uc131 \ucc38\uac00 \uc2e0\uccad\uc790\ub4e4\uc774 \uc2e4\uc81c \ud589\uc0ac\uc5d0 \ucc38\uac00\ud558\uc9c0 \uc54a\uc74c\uc73c\ub85c \uc778\ud574, \ud589\uc0ac \ucc38\uac00\uc790 \uc57d 3,000\uba85 \uc911 \ub300\ubd80\ubd84\uc774 \ub0a8\uc131 \uc73c\ub85c \ucc44\uc6cc\uc9c0\ub294 \ud604\uc0c1\uc774 \ubc1c\uc0dd\ud588\ub2e4. \ud55c\ud3b8 \uacbd\ucc30 \uce21 \ucd94\uc0b0\uc5d0 \ub530\ub974\uba74 \uc774\ub0a0 \uc11c\uc6b8 \uc5ec\uc758\ub3c4 \uacf5\uc6d0\uc5d0\uc11c \uc9c4\ud589\ub41c \ud589\uc0ac\uc5d0\ub294 \ub0a8\uc131 700\uba85\uacfc \uc5ec\uc131 300\uba85 \ub4f1 \uc57d 1\ucc9c\uc5ec \uba85\uc774 \ucc38\uac00\ud55c \uac83\uc73c\ub85c \uc804\ud574\uc84c\ub2e4. \u201c\ub0a8\uc790\uc640 \uc5ec\uc790\uac00 \ub300\uce58\ud55c \uac83\uc774 \uc544\ub2c8\ub77c \uacbd\ucc30\uacfc \ube44\ub458\uae30\uac00 \ub300\uce58\ud558\ub294 \ubd84\uc704\uae30\u201d\ub77c\ub294 \uae00\uc774 \uc62c\ub77c\uc624\uba70 \ub2f9\uc2dc \uadf9\uba85\ud55c \ub0a8\ub140 \ube44\uc728\uc744 \uc124\uba85\ud558\uae30\ub3c4 \ud588\ub2e4. \uc9dd\uc744 \ucc3e\uc9c0 \ubabb\ud55c \ub300\ubd80\ubd84\uc758 \ucc38\uac00\uc790\ub4e4\uc774 \uadfc\ucc98 \uc74c\uc2dd\uc810, \uc220\uc9d1 \ub4f1\uc73c\ub85c \ud769\uc5b4\uc84c\ub2e4. \uc774 \ud589\uc0ac\uc5d0 \ucc38\uac00\ud588\ub358 \uac1c\uadf8\ub9e8 \ubc15\ud718\uc21c\uc740 \"\uc194\ub85c\ub300\ucca9\uc774 \uc220\ub85c\ub300\ucca9\uc73c\ub85c \uc774\uc5b4\uc9c0\uaca0\ub124\uc694\"\ub77c\ub294 \uae00\uc744 \ub0a8\uae30\uae30\ub3c4 \ud588\ub2e4. \uc218\ub3c4\uad8c\uacfc \uc9c0\ubc29\uc758 \uacbd\uc6b0 \uc5ec\uc131 \ucc38\uac00\uc790\uac00 \ub354 \uc801\uc5b4 \ud589\uc0ac \uc790\uccb4\uac00 \uc81c\ub300\ub85c \uc9c4\ud589\ub418\uc9c0 \ubabb\ud55c \uacbd\uc6b0\ub3c4 \uc788\uc5c8\ub2e4. \ucc9c\uc548 \uc194\ub85c\ub300\ucca9\uc758 \uacbd\uc6b0 \uc5ec\uc131 \ucc38\uac00\uc790\uac00 10\uc5ec \uba85 \uc815\ub3c4\uc5d0 \ubd88\uacfc\ud574 \ud589\uc0ac\ub97c \uce58\ub8e8\uc9c0 \ubabb\ud588\uc9c0\ub9cc, \uac1c\uc778\uc801\uc73c\ub85c \ubc88\ud638\ub97c \uc8fc\uace0\ubc1b\uc544 1\ud638 \ucee4\ud50c\uc774 \ud798\uacb9\uac8c \ud0c4\uc0dd\ub418\uae30\ub3c4 \ud588\ub2e4. \ud55c\ud3b8 \ub300\ubd80\ubd84\uc758 \uc5ec\uc131 \uc5f0\uc608\uc778\ub4e4\uc774 \uc548\uc804\uc0c1\uc758 \uc774\uc720\ub85c \ud589\uc0ac \ubd88\ucc38\uc744 \uc120\uc5b8\ud55c \uac00\uc6b4\ub370, \uc2e0\uc778 \uac78\uadf8\ub8f9 \ube44\ud0a4\ub2c8\uc758 \uba64\ubc84\uc778 \ud574\uc774\uac00 \uc194\ub85c\ub300\ucca9\uc5d0 \ucc38\uac00\ud558\uc5ec \uc2e4\uc2dc\uac04 \uac80\uc0c9 1\uc704\ub97c \ucc28\uc9c0\ud558\uae30\ub3c4 \ud558\uc600\ub2e4.", "sentence_answer": "\ud558\uc9c0\ub9cc \uc548\uc804\uc0c1\uc758 \uc774\uc720 \ub4f1\uc73c\ub85c \uc0c1\ub2f9\uc218\uc758 \uc5ec\uc131 \ucc38\uac00 \uc2e0\uccad\uc790\ub4e4\uc774 \uc2e4\uc81c \ud589\uc0ac\uc5d0 \ucc38\uac00\ud558\uc9c0 \uc54a\uc74c\uc73c\ub85c \uc778\ud574, \ud589\uc0ac \ucc38\uac00\uc790 \uc57d 3,000\uba85 \uc911 \ub300\ubd80\ubd84\uc774 \ub0a8\uc131 \uc73c\ub85c \ucc44\uc6cc\uc9c0\ub294 \ud604\uc0c1\uc774 \ubc1c\uc0dd\ud588\ub2e4.", "paragraph_id": "6518373-2-2", "paragraph_question": "question: \uc194\ub85c\ub300\ucca9 \ucc38\uac00\uc790\uc758 \ub300\ubd80\ubd84\uc744 \ucc28\uc9c0\ud55c \uac83\uc740?, context: \ub300\ud55c\ubbfc\uad6d \ucd5c\ucd08 \ud2b9\ubcc4\ud55c \ub0a0 \"\uc194\ub85c\ub300\ucca9\" \ud589\uc0ac\ub294 \uc11c\uc6b8\ud2b9\ubcc4\uc2dc\uc758 \uacbd\uc6b0 \uc2dc\uc98c\uc81c \ud06c\ub9ac\uc2a4\ub9c8\uc2a4 \uae30\uac04 \uc911 2012\ub144 12\uc6d4 24\uc77c\uc6d4\uc694\uc77c \ud3c9\uc77c/\uc784\uc2dc \uc790\uc728 1\uc77c \ud734\uc77c\uc81c \ud06c\ub9ac\uc2a4\ub9c8\uc2a4 \uc774\ube0c*\uc131\ud0c4\uc804\uc57c\uc81c \ub370\uc774* \uc624\ud6c43~5\uc2dc(\uc57d2\uc2dc\uac04 \ub3d9\uc548)\uc5d0 \ub0a8\uc790\ub4e4\uc740 \ud558\uc580\uc0c9, \uc5ec\uc790\ub4e4\uc740 \ubd89\uc740\uc0c9, \ucee4\ud50c\uc740 \ucd08\ub85d\uc0c9 \uacc4\uc5f4\uc758 \uc637\uc744 \uc785\uace0 \uc11c\uc6b8\uc5ec\uc758\ub3c4\uacf5\uc6d0\uc73c\ub85c \ubaa8\uc774\ub294 \uac83\uc73c\ub85c \uc2dc\uc791\ud558\uc600\ub2e4. \uacf5\uc6d0\uc5d0 \ubaa8\uc778 \ub0a8\ub140\ub294 \uc2e0\ud638 \uc591\ud3b8\uc5d0\uc11c \ub300\uae30\ud558\ub2e4\uac00 \uc9c4\ud589\uc790\uac00 \uc2e0\ud638\ub97c \ubcf4\ub0b4\uba74 \ub9c8\uc74c\uc5d0 \ub4dc\ub294 \uc774\uc131(\uc5ec\ub294 \ub0a8\uc73c\ub85c, \ub0a8\uc740 \uc5ec\ub85c)\uc744 \ud5a5\ud574 \ub2ec\ub824\uac00 \uc190\uc744 \uc7a1\ub294 \ud615\uc2dd\uc73c\ub85c \uc9c4\ud589\ub418\uc5c8\ub2e4. \uc2e4\uc81c \uc774 \ud589\uc0ac\ub97c \ud1b5\ud574 \uba87 \uc30d\uc758 \ucee4\ud50c\uc774 \ud0c4\uc0dd\ud558\uae30\ub3c4 \ud558\uc600\ub2e4. \ud558\uc9c0\ub9cc \uc548\uc804\uc0c1\uc758 \uc774\uc720 \ub4f1\uc73c\ub85c \uc0c1\ub2f9\uc218\uc758 \uc5ec\uc131 \ucc38\uac00 \uc2e0\uccad\uc790\ub4e4\uc774 \uc2e4\uc81c \ud589\uc0ac\uc5d0 \ucc38\uac00\ud558\uc9c0 \uc54a\uc74c\uc73c\ub85c \uc778\ud574, \ud589\uc0ac \ucc38\uac00\uc790 \uc57d 3,000\uba85 \uc911 \ub300\ubd80\ubd84\uc774 \ub0a8\uc131\uc73c\ub85c \ucc44\uc6cc\uc9c0\ub294 \ud604\uc0c1\uc774 \ubc1c\uc0dd\ud588\ub2e4. \ud55c\ud3b8 \uacbd\ucc30 \uce21 \ucd94\uc0b0\uc5d0 \ub530\ub974\uba74 \uc774\ub0a0 \uc11c\uc6b8 \uc5ec\uc758\ub3c4 \uacf5\uc6d0\uc5d0\uc11c \uc9c4\ud589\ub41c \ud589\uc0ac\uc5d0\ub294 \ub0a8\uc131 700\uba85\uacfc \uc5ec\uc131 300\uba85 \ub4f1 \uc57d 1\ucc9c\uc5ec \uba85\uc774 \ucc38\uac00\ud55c \uac83\uc73c\ub85c \uc804\ud574\uc84c\ub2e4. \u201c\ub0a8\uc790\uc640 \uc5ec\uc790\uac00 \ub300\uce58\ud55c \uac83\uc774 \uc544\ub2c8\ub77c \uacbd\ucc30\uacfc \ube44\ub458\uae30\uac00 \ub300\uce58\ud558\ub294 \ubd84\uc704\uae30\u201d\ub77c\ub294 \uae00\uc774 \uc62c\ub77c\uc624\uba70 \ub2f9\uc2dc \uadf9\uba85\ud55c \ub0a8\ub140 \ube44\uc728\uc744 \uc124\uba85\ud558\uae30\ub3c4 \ud588\ub2e4. \uc9dd\uc744 \ucc3e\uc9c0 \ubabb\ud55c \ub300\ubd80\ubd84\uc758 \ucc38\uac00\uc790\ub4e4\uc774 \uadfc\ucc98 \uc74c\uc2dd\uc810, \uc220\uc9d1 \ub4f1\uc73c\ub85c \ud769\uc5b4\uc84c\ub2e4. \uc774 \ud589\uc0ac\uc5d0 \ucc38\uac00\ud588\ub358 \uac1c\uadf8\ub9e8 \ubc15\ud718\uc21c\uc740 \"\uc194\ub85c\ub300\ucca9\uc774 \uc220\ub85c\ub300\ucca9\uc73c\ub85c \uc774\uc5b4\uc9c0\uaca0\ub124\uc694\"\ub77c\ub294 \uae00\uc744 \ub0a8\uae30\uae30\ub3c4 \ud588\ub2e4. \uc218\ub3c4\uad8c\uacfc \uc9c0\ubc29\uc758 \uacbd\uc6b0 \uc5ec\uc131 \ucc38\uac00\uc790\uac00 \ub354 \uc801\uc5b4 \ud589\uc0ac \uc790\uccb4\uac00 \uc81c\ub300\ub85c \uc9c4\ud589\ub418\uc9c0 \ubabb\ud55c \uacbd\uc6b0\ub3c4 \uc788\uc5c8\ub2e4. \ucc9c\uc548 \uc194\ub85c\ub300\ucca9\uc758 \uacbd\uc6b0 \uc5ec\uc131 \ucc38\uac00\uc790\uac00 10\uc5ec \uba85 \uc815\ub3c4\uc5d0 \ubd88\uacfc\ud574 \ud589\uc0ac\ub97c \uce58\ub8e8\uc9c0 \ubabb\ud588\uc9c0\ub9cc, \uac1c\uc778\uc801\uc73c\ub85c \ubc88\ud638\ub97c \uc8fc\uace0\ubc1b\uc544 1\ud638 \ucee4\ud50c\uc774 \ud798\uacb9\uac8c \ud0c4\uc0dd\ub418\uae30\ub3c4 \ud588\ub2e4. \ud55c\ud3b8 \ub300\ubd80\ubd84\uc758 \uc5ec\uc131 \uc5f0\uc608\uc778\ub4e4\uc774 \uc548\uc804\uc0c1\uc758 \uc774\uc720\ub85c \ud589\uc0ac \ubd88\ucc38\uc744 \uc120\uc5b8\ud55c \uac00\uc6b4\ub370, \uc2e0\uc778 \uac78\uadf8\ub8f9 \ube44\ud0a4\ub2c8\uc758 \uba64\ubc84\uc778 \ud574\uc774\uac00 \uc194\ub85c\ub300\ucca9\uc5d0 \ucc38\uac00\ud558\uc5ec \uc2e4\uc2dc\uac04 \uac80\uc0c9 1\uc704\ub97c \ucc28\uc9c0\ud558\uae30\ub3c4 \ud558\uc600\ub2e4."} {"question": "\ud5e4\uc774\uc8e0\ucfc4\ub85c \uc218\ub3c4\ub97c \ucc9c\ub3c4\ud55c \ub54c\ub294 \uba87 \ub144\ub3c4\uc778\uac00?", "paragraph": "710\ub144\uc5d0 \uc218\ub3c4\ub97c \ub098\ub77c\uc758 \ud5e4\uc774\uc870\ucfc4\ub85c \ucc9c\ub3c4\ud558\uc600\ub2e4. \ud5e4\uc774\uc870\ucfc4\ub294 \uc911\uad6d\uc758 \uc218\ub3c4 \uc7a5\uc548\uc744 \ubaa8\ubc29\ud558\uc5ec \ub9cc\ub4e4\uc5b4\uc9c4 \ub3c4\uc2dc\ub85c, \uacf5\ubb34\uc6d0\uc774 \uc8fc\ubbfc\uc758 \ub300\ubd80\ubd84\uc744 \ucc28\uc9c0\ud558\ub294 \uc815\uce58\ub3c4\uc2dc\uc600\ub2e4. \ud5e4\uc774\uc870\ucfc4\ub85c \ucc9c\ub3c4\ud558\ub294 \ub370\ub294 \ud6c4\uc9c0\uc640\ub77c\ub178 \ud6c4\ud788\ud1a0\uac00 \uc911\uc694\ud55c \uc5ed\ud560\uc744 \uc218\ud589\ud558\uc600\ub2e4. \ud5e4\uc774\uc8e0\ucf54 \ucc9c\ub3c4\uc5d0 \uc55e\uc11c \uc120\uc815\ub418\uace0 \uc2dc\ud589\ub41c \ub2e4\uc774\ud638 \uc728\ub839(\u5927\u5b9d\u5f8b\u4ee4)\uc774 \uc77c\ubcf8\uad6d\ub0b4\uc758 \uc2e4\uc815\uc5d0 \ub9de\uac8c \uc5ec\ub7ec \ubc29\uba74\uc5d0\uc11c \uac80\ud1a0\ud558\uace0 \uc218\uc815\ud558\uc5ec \uc2dc\ud589\ucc29\uc624\ub97c \uac70\uce58\uba74\uc11c \uc728\ub839\uad6d\uac00, \ucc9c\ud669\uc911\uc2ec\uc758 \uc804\uc81c\uad6d\uac00, \uc911\uc559\uc9d1\uad8c\uad6d\uac00\ub97c \uc9c0\ud5a5\ud558\ub358 \uc2dc\ub300\uc600\ub2e4. \uc728\ub839\uad6d\uac00\ub77c\ub294 \uc131\uaca9\uc740 \ud638\uc801\uacfc \uacc4\uc7a5(\u8a08\u5e33)\uc73c\ub85c \uc778\ubbfc\uc744 \ud30c\uc545\ud558\uace0, \uc870\uc6a9\uc870\uc640 \uad70\uc5ed\uc744 \ubd80\uacfc\ud588\ub2e4\ub294 \uc810\uc5d0\uc11c \ub4dc\ub7ec\ub09c\ub2e4. \ub2e4\uc774\ud638 \uc728\ub839\uc740 \uc774\ud6c4\uc5d0 \ud6c4\uc9c0\uc640\ub77c\ub178 \ud6c4\ud788\ud1a0 \ub4f1\uc5d0 \uc758\ud574 \ub204\ucc28 \uc190\uc9c8\ub418\uc5b4\uc624\uba70 10\uc138\uae30\uae4c\uc9c0 \uc77c\ubcf8\uc5d0\uc11c \ucd5c\uace0 \ubc95\uc804\uc758 \uc9c0\uc704\ub97c \uc720\uc9c0\ud55c\ub2e4.", "answer": "710\ub144", "sentence": "710\ub144 \uc5d0 \uc218\ub3c4\ub97c \ub098\ub77c\uc758 \ud5e4\uc774\uc870\ucfc4\ub85c \ucc9c\ub3c4\ud558\uc600\ub2e4.", "paragraph_sentence": " 710\ub144 \uc5d0 \uc218\ub3c4\ub97c \ub098\ub77c\uc758 \ud5e4\uc774\uc870\ucfc4\ub85c \ucc9c\ub3c4\ud558\uc600\ub2e4. \ud5e4\uc774\uc870\ucfc4\ub294 \uc911\uad6d\uc758 \uc218\ub3c4 \uc7a5\uc548\uc744 \ubaa8\ubc29\ud558\uc5ec \ub9cc\ub4e4\uc5b4\uc9c4 \ub3c4\uc2dc\ub85c, \uacf5\ubb34\uc6d0\uc774 \uc8fc\ubbfc\uc758 \ub300\ubd80\ubd84\uc744 \ucc28\uc9c0\ud558\ub294 \uc815\uce58\ub3c4\uc2dc\uc600\ub2e4. \ud5e4\uc774\uc870\ucfc4\ub85c \ucc9c\ub3c4\ud558\ub294 \ub370\ub294 \ud6c4\uc9c0\uc640\ub77c\ub178 \ud6c4\ud788\ud1a0\uac00 \uc911\uc694\ud55c \uc5ed\ud560\uc744 \uc218\ud589\ud558\uc600\ub2e4. \ud5e4\uc774\uc8e0\ucf54 \ucc9c\ub3c4\uc5d0 \uc55e\uc11c \uc120\uc815\ub418\uace0 \uc2dc\ud589\ub41c \ub2e4\uc774\ud638 \uc728\ub839(\u5927\u5b9d\u5f8b\u4ee4)\uc774 \uc77c\ubcf8\uad6d\ub0b4\uc758 \uc2e4\uc815\uc5d0 \ub9de\uac8c \uc5ec\ub7ec \ubc29\uba74\uc5d0\uc11c \uac80\ud1a0\ud558\uace0 \uc218\uc815\ud558\uc5ec \uc2dc\ud589\ucc29\uc624\ub97c \uac70\uce58\uba74\uc11c \uc728\ub839\uad6d\uac00, \ucc9c\ud669\uc911\uc2ec\uc758 \uc804\uc81c\uad6d\uac00, \uc911\uc559\uc9d1\uad8c\uad6d\uac00\ub97c \uc9c0\ud5a5\ud558\ub358 \uc2dc\ub300\uc600\ub2e4. \uc728\ub839\uad6d\uac00\ub77c\ub294 \uc131\uaca9\uc740 \ud638\uc801\uacfc \uacc4\uc7a5(\u8a08\u5e33)\uc73c\ub85c \uc778\ubbfc\uc744 \ud30c\uc545\ud558\uace0, \uc870\uc6a9\uc870\uc640 \uad70\uc5ed\uc744 \ubd80\uacfc\ud588\ub2e4\ub294 \uc810\uc5d0\uc11c \ub4dc\ub7ec\ub09c\ub2e4. \ub2e4\uc774\ud638 \uc728\ub839\uc740 \uc774\ud6c4\uc5d0 \ud6c4\uc9c0\uc640\ub77c\ub178 \ud6c4\ud788\ud1a0 \ub4f1\uc5d0 \uc758\ud574 \ub204\ucc28 \uc190\uc9c8\ub418\uc5b4\uc624\uba70 10\uc138\uae30\uae4c\uc9c0 \uc77c\ubcf8\uc5d0\uc11c \ucd5c\uace0 \ubc95\uc804\uc758 \uc9c0\uc704\ub97c \uc720\uc9c0\ud55c\ub2e4.", "paragraph_answer": " 710\ub144 \uc5d0 \uc218\ub3c4\ub97c \ub098\ub77c\uc758 \ud5e4\uc774\uc870\ucfc4\ub85c \ucc9c\ub3c4\ud558\uc600\ub2e4. \ud5e4\uc774\uc870\ucfc4\ub294 \uc911\uad6d\uc758 \uc218\ub3c4 \uc7a5\uc548\uc744 \ubaa8\ubc29\ud558\uc5ec \ub9cc\ub4e4\uc5b4\uc9c4 \ub3c4\uc2dc\ub85c, \uacf5\ubb34\uc6d0\uc774 \uc8fc\ubbfc\uc758 \ub300\ubd80\ubd84\uc744 \ucc28\uc9c0\ud558\ub294 \uc815\uce58\ub3c4\uc2dc\uc600\ub2e4. \ud5e4\uc774\uc870\ucfc4\ub85c \ucc9c\ub3c4\ud558\ub294 \ub370\ub294 \ud6c4\uc9c0\uc640\ub77c\ub178 \ud6c4\ud788\ud1a0\uac00 \uc911\uc694\ud55c \uc5ed\ud560\uc744 \uc218\ud589\ud558\uc600\ub2e4. \ud5e4\uc774\uc8e0\ucf54 \ucc9c\ub3c4\uc5d0 \uc55e\uc11c \uc120\uc815\ub418\uace0 \uc2dc\ud589\ub41c \ub2e4\uc774\ud638 \uc728\ub839(\u5927\u5b9d\u5f8b\u4ee4)\uc774 \uc77c\ubcf8\uad6d\ub0b4\uc758 \uc2e4\uc815\uc5d0 \ub9de\uac8c \uc5ec\ub7ec \ubc29\uba74\uc5d0\uc11c \uac80\ud1a0\ud558\uace0 \uc218\uc815\ud558\uc5ec \uc2dc\ud589\ucc29\uc624\ub97c \uac70\uce58\uba74\uc11c \uc728\ub839\uad6d\uac00, \ucc9c\ud669\uc911\uc2ec\uc758 \uc804\uc81c\uad6d\uac00, \uc911\uc559\uc9d1\uad8c\uad6d\uac00\ub97c \uc9c0\ud5a5\ud558\ub358 \uc2dc\ub300\uc600\ub2e4. \uc728\ub839\uad6d\uac00\ub77c\ub294 \uc131\uaca9\uc740 \ud638\uc801\uacfc \uacc4\uc7a5(\u8a08\u5e33)\uc73c\ub85c \uc778\ubbfc\uc744 \ud30c\uc545\ud558\uace0, \uc870\uc6a9\uc870\uc640 \uad70\uc5ed\uc744 \ubd80\uacfc\ud588\ub2e4\ub294 \uc810\uc5d0\uc11c \ub4dc\ub7ec\ub09c\ub2e4. \ub2e4\uc774\ud638 \uc728\ub839\uc740 \uc774\ud6c4\uc5d0 \ud6c4\uc9c0\uc640\ub77c\ub178 \ud6c4\ud788\ud1a0 \ub4f1\uc5d0 \uc758\ud574 \ub204\ucc28 \uc190\uc9c8\ub418\uc5b4\uc624\uba70 10\uc138\uae30\uae4c\uc9c0 \uc77c\ubcf8\uc5d0\uc11c \ucd5c\uace0 \ubc95\uc804\uc758 \uc9c0\uc704\ub97c \uc720\uc9c0\ud55c\ub2e4.", "sentence_answer": " 710\ub144 \uc5d0 \uc218\ub3c4\ub97c \ub098\ub77c\uc758 \ud5e4\uc774\uc870\ucfc4\ub85c \ucc9c\ub3c4\ud558\uc600\ub2e4.", "paragraph_id": "6461620-0-1", "paragraph_question": "question: \ud5e4\uc774\uc8e0\ucfc4\ub85c \uc218\ub3c4\ub97c \ucc9c\ub3c4\ud55c \ub54c\ub294 \uba87 \ub144\ub3c4\uc778\uac00?, context: 710\ub144\uc5d0 \uc218\ub3c4\ub97c \ub098\ub77c\uc758 \ud5e4\uc774\uc870\ucfc4\ub85c \ucc9c\ub3c4\ud558\uc600\ub2e4. \ud5e4\uc774\uc870\ucfc4\ub294 \uc911\uad6d\uc758 \uc218\ub3c4 \uc7a5\uc548\uc744 \ubaa8\ubc29\ud558\uc5ec \ub9cc\ub4e4\uc5b4\uc9c4 \ub3c4\uc2dc\ub85c, \uacf5\ubb34\uc6d0\uc774 \uc8fc\ubbfc\uc758 \ub300\ubd80\ubd84\uc744 \ucc28\uc9c0\ud558\ub294 \uc815\uce58\ub3c4\uc2dc\uc600\ub2e4. \ud5e4\uc774\uc870\ucfc4\ub85c \ucc9c\ub3c4\ud558\ub294 \ub370\ub294 \ud6c4\uc9c0\uc640\ub77c\ub178 \ud6c4\ud788\ud1a0\uac00 \uc911\uc694\ud55c \uc5ed\ud560\uc744 \uc218\ud589\ud558\uc600\ub2e4. \ud5e4\uc774\uc8e0\ucf54 \ucc9c\ub3c4\uc5d0 \uc55e\uc11c \uc120\uc815\ub418\uace0 \uc2dc\ud589\ub41c \ub2e4\uc774\ud638 \uc728\ub839(\u5927\u5b9d\u5f8b\u4ee4)\uc774 \uc77c\ubcf8\uad6d\ub0b4\uc758 \uc2e4\uc815\uc5d0 \ub9de\uac8c \uc5ec\ub7ec \ubc29\uba74\uc5d0\uc11c \uac80\ud1a0\ud558\uace0 \uc218\uc815\ud558\uc5ec \uc2dc\ud589\ucc29\uc624\ub97c \uac70\uce58\uba74\uc11c \uc728\ub839\uad6d\uac00, \ucc9c\ud669\uc911\uc2ec\uc758 \uc804\uc81c\uad6d\uac00, \uc911\uc559\uc9d1\uad8c\uad6d\uac00\ub97c \uc9c0\ud5a5\ud558\ub358 \uc2dc\ub300\uc600\ub2e4. \uc728\ub839\uad6d\uac00\ub77c\ub294 \uc131\uaca9\uc740 \ud638\uc801\uacfc \uacc4\uc7a5(\u8a08\u5e33)\uc73c\ub85c \uc778\ubbfc\uc744 \ud30c\uc545\ud558\uace0, \uc870\uc6a9\uc870\uc640 \uad70\uc5ed\uc744 \ubd80\uacfc\ud588\ub2e4\ub294 \uc810\uc5d0\uc11c \ub4dc\ub7ec\ub09c\ub2e4. \ub2e4\uc774\ud638 \uc728\ub839\uc740 \uc774\ud6c4\uc5d0 \ud6c4\uc9c0\uc640\ub77c\ub178 \ud6c4\ud788\ud1a0 \ub4f1\uc5d0 \uc758\ud574 \ub204\ucc28 \uc190\uc9c8\ub418\uc5b4\uc624\uba70 10\uc138\uae30\uae4c\uc9c0 \uc77c\ubcf8\uc5d0\uc11c \ucd5c\uace0 \ubc95\uc804\uc758 \uc9c0\uc704\ub97c \uc720\uc9c0\ud55c\ub2e4."} {"question": "ARU\uac00 \ud480\uba3c\uc0ac \ud30c\uc5c5\uc5d0 \uc5f0\ub300\ud55c \ub144\ub3c4\ub294?", "paragraph": "\uc720\uc9c4 \ub370\ube0c\uc2a4\ub294 \ub2f9\uc2dc \uae30\uad00\ucc28 \ud654\ubd80\ub4e4\uc758 \ub178\uc870 \uc9c0\ub3c4\ubd80\uc600\ub294\ub370, \uc0ac\uc784\ud558\uace0 \uc0c8\ub85c\uc774 \ucca0\ub3c4\uc0b0\uc5c5\uc5d0 \uc885\uc0ac\ud558\ub294 \ubaa8\ub4e0 \ub178\ub3d9\uc790\ub97c \ub300\uc0c1\uc73c\ub85c \ud558\ub294 \uc804\ubbf8\ucca0\ub3c4\ub178\uc870(ARU)\ub97c \uc124\ub9bd\ud55c\ub2e4. 1894\ub144 6\uc6d4 ARU\uac00 \ud480\uba3c\uc0ac \ud30c\uc5c5\uc5d0 \uc5f0\ub300\ud558\uc790 \ubbf8\uad6d \uc804\uc5ed\uc758 \ucca0\ub3c4\uad50\ud1b5\uc774 \uac70\uc758 \ub9c8\ube44\ub418\uc5c8\ub2e4. \uc0b0\uc5c5\ubcc4 \ub178\ub3d9\uc870\ud569\uc758 \ud6a8\uacfc\ub294 \uadf8 \uc2dc\uc791\ubd80\ud130 \ub9e4\uc6b0 \uba85\ubc31\ud55c \uc99d\uac70\ub97c \ubcf4\uc5ec\uc8fc\uc5c8\ub2e4. \uc2dc\uce74\uace0 \ud55c \uacf3\uc5d0\ub9cc 14,000 \uba85\uc758 \uad70\ub300\uc640 2600\uba85\uc758 \ubcf4\uc548\uad00\ub4e4\uc774 \ubc30\uce58\ub418\ub294 \ub4f1 \uc815\ubd80\uac00 \uc0ac\uce21 \ud3b8\uc744 \ub4e4\uba70 \ub300\uaddc\ubaa8\ub85c \uac1c\uc785\ud558\uc5ec \ud480\uba3c\uc0ac \ud30c\uc5c5\uc740 \uacb0\uad6d \ubd84\uc1c4\ub41c\ub2e4. \ub370\ube0c\uc2a4\ub294 AFL\uc758 \uc5f4\ucc28\ud615\uc81c\ub2e8\uc5d0 \uc81c\ud734\ub97c \uc694\uccad\ud558\ub294 \ud55c\ud3b8 \uc5f4\ucc28\uad00\ub9ac\uc790\ud611\ud68c\uc5d0 \u201c\ubaa8\ub4e0 \ub178\ub3d9\uc790\ub4e4\uc758 \uc704\uce58\uac00 \uc804\uacfc \uac19\uc774 \ubcf5\uad6c\ub418\uba74 \uadf8\ub4e4\uc740 \ud55c\uaebc\ubc88\uc5d0 \uc9c1\uc7a5\uc73c\ub85c \ub3cc\uc544\uac08 \uac83\uc774\ub098, \uadf8\ub807\uc9c0 \uc544\ub2c8\ud560 \uacbd\uc6b0 \ucd1d\ud30c\uc5c5\uc774 \uc788\uc744 \uac83\u201d\uc774\ub77c\ub294 \uc804\uc5b8\uc744 \ubcf4\ub0c8\ub2e4.", "answer": "1894\ub144", "sentence": "1894\ub144 6\uc6d4 ARU\uac00 \ud480\uba3c\uc0ac \ud30c\uc5c5\uc5d0 \uc5f0\ub300\ud558\uc790 \ubbf8\uad6d \uc804\uc5ed\uc758 \ucca0\ub3c4\uad50\ud1b5\uc774 \uac70\uc758 \ub9c8\ube44\ub418\uc5c8\ub2e4.", "paragraph_sentence": "\uc720\uc9c4 \ub370\ube0c\uc2a4\ub294 \ub2f9\uc2dc \uae30\uad00\ucc28 \ud654\ubd80\ub4e4\uc758 \ub178\uc870 \uc9c0\ub3c4\ubd80\uc600\ub294\ub370, \uc0ac\uc784\ud558\uace0 \uc0c8\ub85c\uc774 \ucca0\ub3c4\uc0b0\uc5c5\uc5d0 \uc885\uc0ac\ud558\ub294 \ubaa8\ub4e0 \ub178\ub3d9\uc790\ub97c \ub300\uc0c1\uc73c\ub85c \ud558\ub294 \uc804\ubbf8\ucca0\ub3c4\ub178\uc870(ARU)\ub97c \uc124\ub9bd\ud55c\ub2e4. 1894\ub144 6\uc6d4 ARU\uac00 \ud480\uba3c\uc0ac \ud30c\uc5c5\uc5d0 \uc5f0\ub300\ud558\uc790 \ubbf8\uad6d \uc804\uc5ed\uc758 \ucca0\ub3c4\uad50\ud1b5\uc774 \uac70\uc758 \ub9c8\ube44\ub418\uc5c8\ub2e4. \uc0b0\uc5c5\ubcc4 \ub178\ub3d9\uc870\ud569\uc758 \ud6a8\uacfc\ub294 \uadf8 \uc2dc\uc791\ubd80\ud130 \ub9e4\uc6b0 \uba85\ubc31\ud55c \uc99d\uac70\ub97c \ubcf4\uc5ec\uc8fc\uc5c8\ub2e4. \uc2dc\uce74\uace0 \ud55c \uacf3\uc5d0\ub9cc 14,000 \uba85\uc758 \uad70\ub300\uc640 2600\uba85\uc758 \ubcf4\uc548\uad00\ub4e4\uc774 \ubc30\uce58\ub418\ub294 \ub4f1 \uc815\ubd80\uac00 \uc0ac\uce21 \ud3b8\uc744 \ub4e4\uba70 \ub300\uaddc\ubaa8\ub85c \uac1c\uc785\ud558\uc5ec \ud480\uba3c\uc0ac \ud30c\uc5c5\uc740 \uacb0\uad6d \ubd84\uc1c4\ub41c\ub2e4. \ub370\ube0c\uc2a4\ub294 AFL\uc758 \uc5f4\ucc28\ud615\uc81c\ub2e8\uc5d0 \uc81c\ud734\ub97c \uc694\uccad\ud558\ub294 \ud55c\ud3b8 \uc5f4\ucc28\uad00\ub9ac\uc790\ud611\ud68c\uc5d0 \u201c\ubaa8\ub4e0 \ub178\ub3d9\uc790\ub4e4\uc758 \uc704\uce58\uac00 \uc804\uacfc \uac19\uc774 \ubcf5\uad6c\ub418\uba74 \uadf8\ub4e4\uc740 \ud55c\uaebc\ubc88\uc5d0 \uc9c1\uc7a5\uc73c\ub85c \ub3cc\uc544\uac08 \uac83\uc774\ub098, \uadf8\ub807\uc9c0 \uc544\ub2c8\ud560 \uacbd\uc6b0 \ucd1d\ud30c\uc5c5\uc774 \uc788\uc744 \uac83\u201d\uc774\ub77c\ub294 \uc804\uc5b8\uc744 \ubcf4\ub0c8\ub2e4.", "paragraph_answer": "\uc720\uc9c4 \ub370\ube0c\uc2a4\ub294 \ub2f9\uc2dc \uae30\uad00\ucc28 \ud654\ubd80\ub4e4\uc758 \ub178\uc870 \uc9c0\ub3c4\ubd80\uc600\ub294\ub370, \uc0ac\uc784\ud558\uace0 \uc0c8\ub85c\uc774 \ucca0\ub3c4\uc0b0\uc5c5\uc5d0 \uc885\uc0ac\ud558\ub294 \ubaa8\ub4e0 \ub178\ub3d9\uc790\ub97c \ub300\uc0c1\uc73c\ub85c \ud558\ub294 \uc804\ubbf8\ucca0\ub3c4\ub178\uc870(ARU)\ub97c \uc124\ub9bd\ud55c\ub2e4. 1894\ub144 6\uc6d4 ARU\uac00 \ud480\uba3c\uc0ac \ud30c\uc5c5\uc5d0 \uc5f0\ub300\ud558\uc790 \ubbf8\uad6d \uc804\uc5ed\uc758 \ucca0\ub3c4\uad50\ud1b5\uc774 \uac70\uc758 \ub9c8\ube44\ub418\uc5c8\ub2e4. \uc0b0\uc5c5\ubcc4 \ub178\ub3d9\uc870\ud569\uc758 \ud6a8\uacfc\ub294 \uadf8 \uc2dc\uc791\ubd80\ud130 \ub9e4\uc6b0 \uba85\ubc31\ud55c \uc99d\uac70\ub97c \ubcf4\uc5ec\uc8fc\uc5c8\ub2e4. \uc2dc\uce74\uace0 \ud55c \uacf3\uc5d0\ub9cc 14,000 \uba85\uc758 \uad70\ub300\uc640 2600\uba85\uc758 \ubcf4\uc548\uad00\ub4e4\uc774 \ubc30\uce58\ub418\ub294 \ub4f1 \uc815\ubd80\uac00 \uc0ac\uce21 \ud3b8\uc744 \ub4e4\uba70 \ub300\uaddc\ubaa8\ub85c \uac1c\uc785\ud558\uc5ec \ud480\uba3c\uc0ac \ud30c\uc5c5\uc740 \uacb0\uad6d \ubd84\uc1c4\ub41c\ub2e4. \ub370\ube0c\uc2a4\ub294 AFL\uc758 \uc5f4\ucc28\ud615\uc81c\ub2e8\uc5d0 \uc81c\ud734\ub97c \uc694\uccad\ud558\ub294 \ud55c\ud3b8 \uc5f4\ucc28\uad00\ub9ac\uc790\ud611\ud68c\uc5d0 \u201c\ubaa8\ub4e0 \ub178\ub3d9\uc790\ub4e4\uc758 \uc704\uce58\uac00 \uc804\uacfc \uac19\uc774 \ubcf5\uad6c\ub418\uba74 \uadf8\ub4e4\uc740 \ud55c\uaebc\ubc88\uc5d0 \uc9c1\uc7a5\uc73c\ub85c \ub3cc\uc544\uac08 \uac83\uc774\ub098, \uadf8\ub807\uc9c0 \uc544\ub2c8\ud560 \uacbd\uc6b0 \ucd1d\ud30c\uc5c5\uc774 \uc788\uc744 \uac83\u201d\uc774\ub77c\ub294 \uc804\uc5b8\uc744 \ubcf4\ub0c8\ub2e4.", "sentence_answer": " 1894\ub144 6\uc6d4 ARU\uac00 \ud480\uba3c\uc0ac \ud30c\uc5c5\uc5d0 \uc5f0\ub300\ud558\uc790 \ubbf8\uad6d \uc804\uc5ed\uc758 \ucca0\ub3c4\uad50\ud1b5\uc774 \uac70\uc758 \ub9c8\ube44\ub418\uc5c8\ub2e4.", "paragraph_id": "6657171-10-0", "paragraph_question": "question: ARU\uac00 \ud480\uba3c\uc0ac \ud30c\uc5c5\uc5d0 \uc5f0\ub300\ud55c \ub144\ub3c4\ub294?, context: \uc720\uc9c4 \ub370\ube0c\uc2a4\ub294 \ub2f9\uc2dc \uae30\uad00\ucc28 \ud654\ubd80\ub4e4\uc758 \ub178\uc870 \uc9c0\ub3c4\ubd80\uc600\ub294\ub370, \uc0ac\uc784\ud558\uace0 \uc0c8\ub85c\uc774 \ucca0\ub3c4\uc0b0\uc5c5\uc5d0 \uc885\uc0ac\ud558\ub294 \ubaa8\ub4e0 \ub178\ub3d9\uc790\ub97c \ub300\uc0c1\uc73c\ub85c \ud558\ub294 \uc804\ubbf8\ucca0\ub3c4\ub178\uc870(ARU)\ub97c \uc124\ub9bd\ud55c\ub2e4. 1894\ub144 6\uc6d4 ARU\uac00 \ud480\uba3c\uc0ac \ud30c\uc5c5\uc5d0 \uc5f0\ub300\ud558\uc790 \ubbf8\uad6d \uc804\uc5ed\uc758 \ucca0\ub3c4\uad50\ud1b5\uc774 \uac70\uc758 \ub9c8\ube44\ub418\uc5c8\ub2e4. \uc0b0\uc5c5\ubcc4 \ub178\ub3d9\uc870\ud569\uc758 \ud6a8\uacfc\ub294 \uadf8 \uc2dc\uc791\ubd80\ud130 \ub9e4\uc6b0 \uba85\ubc31\ud55c \uc99d\uac70\ub97c \ubcf4\uc5ec\uc8fc\uc5c8\ub2e4. \uc2dc\uce74\uace0 \ud55c \uacf3\uc5d0\ub9cc 14,000 \uba85\uc758 \uad70\ub300\uc640 2600\uba85\uc758 \ubcf4\uc548\uad00\ub4e4\uc774 \ubc30\uce58\ub418\ub294 \ub4f1 \uc815\ubd80\uac00 \uc0ac\uce21 \ud3b8\uc744 \ub4e4\uba70 \ub300\uaddc\ubaa8\ub85c \uac1c\uc785\ud558\uc5ec \ud480\uba3c\uc0ac \ud30c\uc5c5\uc740 \uacb0\uad6d \ubd84\uc1c4\ub41c\ub2e4. \ub370\ube0c\uc2a4\ub294 AFL\uc758 \uc5f4\ucc28\ud615\uc81c\ub2e8\uc5d0 \uc81c\ud734\ub97c \uc694\uccad\ud558\ub294 \ud55c\ud3b8 \uc5f4\ucc28\uad00\ub9ac\uc790\ud611\ud68c\uc5d0 \u201c\ubaa8\ub4e0 \ub178\ub3d9\uc790\ub4e4\uc758 \uc704\uce58\uac00 \uc804\uacfc \uac19\uc774 \ubcf5\uad6c\ub418\uba74 \uadf8\ub4e4\uc740 \ud55c\uaebc\ubc88\uc5d0 \uc9c1\uc7a5\uc73c\ub85c \ub3cc\uc544\uac08 \uac83\uc774\ub098, \uadf8\ub807\uc9c0 \uc544\ub2c8\ud560 \uacbd\uc6b0 \ucd1d\ud30c\uc5c5\uc774 \uc788\uc744 \uac83\u201d\uc774\ub77c\ub294 \uc804\uc5b8\uc744 \ubcf4\ub0c8\ub2e4."} {"question": "\ub85c\ubca0\ub974\ud2b8\uac00 \uc77c\uacf1 \ubc88\uc9f8 \ubc24\uc5d0 \uc218\ud559 \uadc0\uc2e0\uc5d0\uac8c \ubc30\uc6b4\uac83\uc740?", "paragraph": "\ub2e4\uc12f \ubc88\uc9f8 \ubc24\uc5d0 \uadf8\ub294 \uc0ac\ub9c9\uc744 \ud5e4\ub9e4\ub2e4\uac00 \uc218\ud559 \uadc0\uc2e0\uc744 \ub9cc\ub098 \uc57c\uc790\uc218 \uc5f4\ub9e4\ub85c \uc0bc\uac01\ud615 \uc22b\uc790\ub97c \ubc30\uc6b4\ub2e4. \uc5ec\uc12f \ubc88\uc9f8 \ubc24\uc5d0\ub294 \uac08\uc0c9\uacfc \ud770\uc0c9 \ud1a0\ub07c\uac00 \ubc88\uc2dd\ud558\uba74\uc11c \ub298\uc5b4\ub098\ub294 \uacfc\uc815\uc744 \ud1b5\ud574 \uc790\uc5f0 \uc18d\uc5d0\uc11c \ud53c\ubcf4\ub098\uce58 \uc218\ub97c \ubc30\uc6b4\ub2e4. \uc774\ub54c\ubd80\ud130 \ub85c\ubca0\ub974\ud2b8\uc758 \uc5c4\ub9c8\ub294 \ub85c\ubca0\ub974\ud2b8\uac00 \uc218\ud559\uc5d0 \ub300\ud55c \ud765\ubbf8\ub97c \uac00\uc9c0\uac8c \ub41c \uac83\uc744 \ubcf4\uace0 \ub180\ub77c\uc6cc \ud588\uace0, \uc774\ud6c4\ub85c \ub85c\ubca0\ub974\ud2b8\ub294 \uc218\ud559 \uadc0\uc2e0\uc744 \ub9cc\ub098\uae30 \uc704\ud574 \ud3c9\uc18c\ubcf4\ub2e4 \ub354 \uc77c\ucc0d \uc7a0\uc774 \ub4e0\ub2e4. \uc77c\uacf1 \ubc88\uc9f8 \ubc24\uc5d0 \ub85c\ubca0\ub974\ud2b8\ub294 \ud145 \ube48 \ud770 \ubc29\uc5d0\uc11c \uc218\ud559 \uadc0\uc2e0\uacfc \ud568\uaed8 \ud30c\uc2a4\uce7c\uc758 \uc0bc\uac01\ud615\uc744 \ubc30\uc6b4\ub2e4. \uc5ec\ub35f \ubc88\uc9f8 \ubc24\uc5d0 \ub85c\ubca0\ub974\ud2b8\ub294 \uadf8\uac00 \ub2e4\ub2c8\ub358 \ud559\uad50 \uad50\uc2e4\ub85c \uc624\uac8c \ub41c\ub2e4. \uc218\ud559 \uadc0\uc2e0\uc740 \uad50\uc2e4\uc5d0\uc11c \uc544\uc774\ub4e4\uc758 \uc790\ub9ac\ub97c \uc5ec\ub7ec\uac00\uc9c0 \ubc29\ubc95\uc73c\ub85c \ubc30\uc815\ud574\uc8fc\uba74\uc11c \ub85c\ubca0\ub974\ud2b8\uc5d0\uac8c \uc21c\uc5f4\uacfc \ucf85(\ud329\ud1a0\ub9ac\uc5bc)\uc744 \uac00\ub974\uccd0\uc900\ub2e4.", "answer": "\ud30c\uc2a4\uce7c\uc758 \uc0bc\uac01\ud615", "sentence": "\uc77c\uacf1 \ubc88\uc9f8 \ubc24\uc5d0 \ub85c\ubca0\ub974\ud2b8\ub294 \ud145 \ube48 \ud770 \ubc29\uc5d0\uc11c \uc218\ud559 \uadc0\uc2e0\uacfc \ud568\uaed8 \ud30c\uc2a4\uce7c\uc758 \uc0bc\uac01\ud615 \uc744 \ubc30\uc6b4\ub2e4.", "paragraph_sentence": "\ub2e4\uc12f \ubc88\uc9f8 \ubc24\uc5d0 \uadf8\ub294 \uc0ac\ub9c9\uc744 \ud5e4\ub9e4\ub2e4\uac00 \uc218\ud559 \uadc0\uc2e0\uc744 \ub9cc\ub098 \uc57c\uc790\uc218 \uc5f4\ub9e4\ub85c \uc0bc\uac01\ud615 \uc22b\uc790\ub97c \ubc30\uc6b4\ub2e4. \uc5ec\uc12f \ubc88\uc9f8 \ubc24\uc5d0\ub294 \uac08\uc0c9\uacfc \ud770\uc0c9 \ud1a0\ub07c\uac00 \ubc88\uc2dd\ud558\uba74\uc11c \ub298\uc5b4\ub098\ub294 \uacfc\uc815\uc744 \ud1b5\ud574 \uc790\uc5f0 \uc18d\uc5d0\uc11c \ud53c\ubcf4\ub098\uce58 \uc218\ub97c \ubc30\uc6b4\ub2e4. \uc774\ub54c\ubd80\ud130 \ub85c\ubca0\ub974\ud2b8\uc758 \uc5c4\ub9c8\ub294 \ub85c\ubca0\ub974\ud2b8\uac00 \uc218\ud559\uc5d0 \ub300\ud55c \ud765\ubbf8\ub97c \uac00\uc9c0\uac8c \ub41c \uac83\uc744 \ubcf4\uace0 \ub180\ub77c\uc6cc \ud588\uace0, \uc774\ud6c4\ub85c \ub85c\ubca0\ub974\ud2b8\ub294 \uc218\ud559 \uadc0\uc2e0\uc744 \ub9cc\ub098\uae30 \uc704\ud574 \ud3c9\uc18c\ubcf4\ub2e4 \ub354 \uc77c\ucc0d \uc7a0\uc774 \ub4e0\ub2e4. \uc77c\uacf1 \ubc88\uc9f8 \ubc24\uc5d0 \ub85c\ubca0\ub974\ud2b8\ub294 \ud145 \ube48 \ud770 \ubc29\uc5d0\uc11c \uc218\ud559 \uadc0\uc2e0\uacfc \ud568\uaed8 \ud30c\uc2a4\uce7c\uc758 \uc0bc\uac01\ud615 \uc744 \ubc30\uc6b4\ub2e4. \uc5ec\ub35f \ubc88\uc9f8 \ubc24\uc5d0 \ub85c\ubca0\ub974\ud2b8\ub294 \uadf8\uac00 \ub2e4\ub2c8\ub358 \ud559\uad50 \uad50\uc2e4\ub85c \uc624\uac8c \ub41c\ub2e4. \uc218\ud559 \uadc0\uc2e0\uc740 \uad50\uc2e4\uc5d0\uc11c \uc544\uc774\ub4e4\uc758 \uc790\ub9ac\ub97c \uc5ec\ub7ec\uac00\uc9c0 \ubc29\ubc95\uc73c\ub85c \ubc30\uc815\ud574\uc8fc\uba74\uc11c \ub85c\ubca0\ub974\ud2b8\uc5d0\uac8c \uc21c\uc5f4\uacfc \ucf85(\ud329\ud1a0\ub9ac\uc5bc)\uc744 \uac00\ub974\uccd0\uc900\ub2e4.", "paragraph_answer": "\ub2e4\uc12f \ubc88\uc9f8 \ubc24\uc5d0 \uadf8\ub294 \uc0ac\ub9c9\uc744 \ud5e4\ub9e4\ub2e4\uac00 \uc218\ud559 \uadc0\uc2e0\uc744 \ub9cc\ub098 \uc57c\uc790\uc218 \uc5f4\ub9e4\ub85c \uc0bc\uac01\ud615 \uc22b\uc790\ub97c \ubc30\uc6b4\ub2e4. \uc5ec\uc12f \ubc88\uc9f8 \ubc24\uc5d0\ub294 \uac08\uc0c9\uacfc \ud770\uc0c9 \ud1a0\ub07c\uac00 \ubc88\uc2dd\ud558\uba74\uc11c \ub298\uc5b4\ub098\ub294 \uacfc\uc815\uc744 \ud1b5\ud574 \uc790\uc5f0 \uc18d\uc5d0\uc11c \ud53c\ubcf4\ub098\uce58 \uc218\ub97c \ubc30\uc6b4\ub2e4. \uc774\ub54c\ubd80\ud130 \ub85c\ubca0\ub974\ud2b8\uc758 \uc5c4\ub9c8\ub294 \ub85c\ubca0\ub974\ud2b8\uac00 \uc218\ud559\uc5d0 \ub300\ud55c \ud765\ubbf8\ub97c \uac00\uc9c0\uac8c \ub41c \uac83\uc744 \ubcf4\uace0 \ub180\ub77c\uc6cc \ud588\uace0, \uc774\ud6c4\ub85c \ub85c\ubca0\ub974\ud2b8\ub294 \uc218\ud559 \uadc0\uc2e0\uc744 \ub9cc\ub098\uae30 \uc704\ud574 \ud3c9\uc18c\ubcf4\ub2e4 \ub354 \uc77c\ucc0d \uc7a0\uc774 \ub4e0\ub2e4. \uc77c\uacf1 \ubc88\uc9f8 \ubc24\uc5d0 \ub85c\ubca0\ub974\ud2b8\ub294 \ud145 \ube48 \ud770 \ubc29\uc5d0\uc11c \uc218\ud559 \uadc0\uc2e0\uacfc \ud568\uaed8 \ud30c\uc2a4\uce7c\uc758 \uc0bc\uac01\ud615 \uc744 \ubc30\uc6b4\ub2e4. \uc5ec\ub35f \ubc88\uc9f8 \ubc24\uc5d0 \ub85c\ubca0\ub974\ud2b8\ub294 \uadf8\uac00 \ub2e4\ub2c8\ub358 \ud559\uad50 \uad50\uc2e4\ub85c \uc624\uac8c \ub41c\ub2e4. \uc218\ud559 \uadc0\uc2e0\uc740 \uad50\uc2e4\uc5d0\uc11c \uc544\uc774\ub4e4\uc758 \uc790\ub9ac\ub97c \uc5ec\ub7ec\uac00\uc9c0 \ubc29\ubc95\uc73c\ub85c \ubc30\uc815\ud574\uc8fc\uba74\uc11c \ub85c\ubca0\ub974\ud2b8\uc5d0\uac8c \uc21c\uc5f4\uacfc \ucf85(\ud329\ud1a0\ub9ac\uc5bc)\uc744 \uac00\ub974\uccd0\uc900\ub2e4.", "sentence_answer": "\uc77c\uacf1 \ubc88\uc9f8 \ubc24\uc5d0 \ub85c\ubca0\ub974\ud2b8\ub294 \ud145 \ube48 \ud770 \ubc29\uc5d0\uc11c \uc218\ud559 \uadc0\uc2e0\uacfc \ud568\uaed8 \ud30c\uc2a4\uce7c\uc758 \uc0bc\uac01\ud615 \uc744 \ubc30\uc6b4\ub2e4.", "paragraph_id": "6518864-0-1", "paragraph_question": "question: \ub85c\ubca0\ub974\ud2b8\uac00 \uc77c\uacf1 \ubc88\uc9f8 \ubc24\uc5d0 \uc218\ud559 \uadc0\uc2e0\uc5d0\uac8c \ubc30\uc6b4\uac83\uc740?, context: \ub2e4\uc12f \ubc88\uc9f8 \ubc24\uc5d0 \uadf8\ub294 \uc0ac\ub9c9\uc744 \ud5e4\ub9e4\ub2e4\uac00 \uc218\ud559 \uadc0\uc2e0\uc744 \ub9cc\ub098 \uc57c\uc790\uc218 \uc5f4\ub9e4\ub85c \uc0bc\uac01\ud615 \uc22b\uc790\ub97c \ubc30\uc6b4\ub2e4. \uc5ec\uc12f \ubc88\uc9f8 \ubc24\uc5d0\ub294 \uac08\uc0c9\uacfc \ud770\uc0c9 \ud1a0\ub07c\uac00 \ubc88\uc2dd\ud558\uba74\uc11c \ub298\uc5b4\ub098\ub294 \uacfc\uc815\uc744 \ud1b5\ud574 \uc790\uc5f0 \uc18d\uc5d0\uc11c \ud53c\ubcf4\ub098\uce58 \uc218\ub97c \ubc30\uc6b4\ub2e4. \uc774\ub54c\ubd80\ud130 \ub85c\ubca0\ub974\ud2b8\uc758 \uc5c4\ub9c8\ub294 \ub85c\ubca0\ub974\ud2b8\uac00 \uc218\ud559\uc5d0 \ub300\ud55c \ud765\ubbf8\ub97c \uac00\uc9c0\uac8c \ub41c \uac83\uc744 \ubcf4\uace0 \ub180\ub77c\uc6cc \ud588\uace0, \uc774\ud6c4\ub85c \ub85c\ubca0\ub974\ud2b8\ub294 \uc218\ud559 \uadc0\uc2e0\uc744 \ub9cc\ub098\uae30 \uc704\ud574 \ud3c9\uc18c\ubcf4\ub2e4 \ub354 \uc77c\ucc0d \uc7a0\uc774 \ub4e0\ub2e4. \uc77c\uacf1 \ubc88\uc9f8 \ubc24\uc5d0 \ub85c\ubca0\ub974\ud2b8\ub294 \ud145 \ube48 \ud770 \ubc29\uc5d0\uc11c \uc218\ud559 \uadc0\uc2e0\uacfc \ud568\uaed8 \ud30c\uc2a4\uce7c\uc758 \uc0bc\uac01\ud615\uc744 \ubc30\uc6b4\ub2e4. \uc5ec\ub35f \ubc88\uc9f8 \ubc24\uc5d0 \ub85c\ubca0\ub974\ud2b8\ub294 \uadf8\uac00 \ub2e4\ub2c8\ub358 \ud559\uad50 \uad50\uc2e4\ub85c \uc624\uac8c \ub41c\ub2e4. \uc218\ud559 \uadc0\uc2e0\uc740 \uad50\uc2e4\uc5d0\uc11c \uc544\uc774\ub4e4\uc758 \uc790\ub9ac\ub97c \uc5ec\ub7ec\uac00\uc9c0 \ubc29\ubc95\uc73c\ub85c \ubc30\uc815\ud574\uc8fc\uba74\uc11c \ub85c\ubca0\ub974\ud2b8\uc5d0\uac8c \uc21c\uc5f4\uacfc \ucf85(\ud329\ud1a0\ub9ac\uc5bc)\uc744 \uac00\ub974\uccd0\uc900\ub2e4."} {"question": "\ud074\ub85c\ubc84\ud544\ub4dc \ud328\ub7ec\ub3c5\uc2a4\uac00 \uac01\uc0c9\ud55c \uac83\uc740?", "paragraph": "\uc774 \uc601\ud654\ub294 \uc624\ub80c \uc6b0\uc9c0\uc5d8\uc774 \uc6b0\uc8fc \uc815\uac70\uc7a5\uc758 \uc120\uc6d0\ub4e4\uc744 \uc8fc\uc694 \uc904\uac70\ub9ac\ub85c \ub2e4\ub8e8\ub294 \u300a\uac13 \ud30c\ud2f0\ud074\u300b(God Particle)\uc744 \uac01\uc0c9\ud55c \uac83\uc774\uba70, \uc6d0\ub798\ub294 \u300a\ud074\ub85c\ubc84\ud544\ub4dc\u300b\uc640 \uad00\ub828\uc774 \uc5c6\uc5c8\ub2e4. \uc774 \uac01\ubcf8\uc740 2012\ub144\uc5d0 \ud30c\ub77c\ub9c8\uc6b4\ud2b8 \ud53d\uccd0\uc2a4\uc640 \ubc30\ub4dc \ub85c\ubd07 \ud504\ub85c\ub355\uc158\uc2a4\uac00 \ud68d\ub4dd\ud588\ub2e4. \ucd08\uae30\uc5d0\ub294 \ud30c\ub77c\ub9c8\uc6b4\ud2b8\uc758 \uc800\uc608\uc0b0 \ubc30\uae09 \uc81c\uc791\uc0ac Insurge\uc758 \uc791\ud488\uc73c\ub85c \uacc4\ud68d\ud588\uc5c8\uc73c\ub098, \ud574\ub2f9 \uc81c\uc791\uc0ac\uac00 \uc815\ub9ac\ub418\uba74\uc11c, \ud30c\ub77c\ub9c8\uc6b4\ud2b8 \ubc30\uae09 \uc601\ud654\ub85c \ud655\uc7a5\ub418\uc5c8\ub2e4. \uc601\ud654\uac00 \uc81c\uc791\ub418\uace0 \uc788\uc744 \ub3d9\uc548\uc5d0 \uc5d0\uc774\ube0c\ub7fc\uc2a4\ub294 \ud074\ub85c\ub7ec\ud544\ub4dc\uc640 \uc774 \uc601\ud654\ub97c \uc5f0\uacb0\uc2dc\ud0a4\uae30\ub85c \uacb0\uc815\ud558\uba74\uc11c, \u300a\ud074\ub85c\ubc84\ud544\ub4dc 10\ubc88\uc9c0\u300b\uc758 \uc6d0\uc791 \uac01\ubcf8 \uc140\ub7ec \u300aThe Cellar\u300b\uc5d0\uc11c \uac19\uc740 \ubc29\ubc95\uc744 \ud588\ub358\uac70\ucc98\ub7fc \uc6b0\uc9c0\uc5d8\uc758 \uac01\ubcf8\uc744 \uac01\uc0c9\ud558\uace0 \ud074\ub85c\ubc84\ud544\ub4dc\uc640 \uc5f0\uacb0\ub418\ub294 \uc7a5\uba74\uc744 \ucd94\uac00\ud588\ub2e4. \uc5d0\uc774\ube0c\ub7fc\uc2a4\ub294 \uc785\uc790 \uac00\uc18d\uae30 \uc0ac\uace0\uac00 \ubbf8\ub798\uc758 \uc0ac\uac74\ub4e4\uc774 \uacfc\uac70\uc5d0 \ubcc0\ud654\ub97c \uc77c\uc73c\ud0ac \uc218\uc788\ub294 \uc218\ub2e8\uc73c\ub85c \uc0ac\uc6a9\ub420 \uc218 \uc788\uc74c\uc744 \uc788\ub2e4\uace0 \ubcf4\uc558\uace0, \ud568\uaed8 \ud074\ub85c\ubc84\ud544\ub4dc \ud504\ub79c\ucc28\uc774\uc988\uc5d0 \uc11c\uc220\uc801\uc73c\ub85c \uc5f0\uacb0\ub418\ub294 \ubc29\ubc95\uc744 \uad6c\ucd95\ud588\ub2e4.", "answer": "\uac13 \ud30c\ud2f0\ud074", "sentence": "\uc774 \uc601\ud654\ub294 \uc624\ub80c \uc6b0\uc9c0\uc5d8\uc774 \uc6b0\uc8fc \uc815\uac70\uc7a5\uc758 \uc120\uc6d0\ub4e4\uc744 \uc8fc\uc694 \uc904\uac70\ub9ac\ub85c \ub2e4\ub8e8\ub294 \u300a \uac13 \ud30c\ud2f0\ud074 \u300b(God Particle)\uc744 \uac01\uc0c9\ud55c \uac83\uc774\uba70, \uc6d0\ub798\ub294 \u300a\ud074\ub85c\ubc84\ud544\ub4dc\u300b\uc640 \uad00\ub828\uc774 \uc5c6\uc5c8\ub2e4.", "paragraph_sentence": " \uc774 \uc601\ud654\ub294 \uc624\ub80c \uc6b0\uc9c0\uc5d8\uc774 \uc6b0\uc8fc \uc815\uac70\uc7a5\uc758 \uc120\uc6d0\ub4e4\uc744 \uc8fc\uc694 \uc904\uac70\ub9ac\ub85c \ub2e4\ub8e8\ub294 \u300a \uac13 \ud30c\ud2f0\ud074 \u300b(God Particle)\uc744 \uac01\uc0c9\ud55c \uac83\uc774\uba70, \uc6d0\ub798\ub294 \u300a\ud074\ub85c\ubc84\ud544\ub4dc\u300b\uc640 \uad00\ub828\uc774 \uc5c6\uc5c8\ub2e4. \uc774 \uac01\ubcf8\uc740 2012\ub144\uc5d0 \ud30c\ub77c\ub9c8\uc6b4\ud2b8 \ud53d\uccd0\uc2a4\uc640 \ubc30\ub4dc \ub85c\ubd07 \ud504\ub85c\ub355\uc158\uc2a4\uac00 \ud68d\ub4dd\ud588\ub2e4. \ucd08\uae30\uc5d0\ub294 \ud30c\ub77c\ub9c8\uc6b4\ud2b8\uc758 \uc800\uc608\uc0b0 \ubc30\uae09 \uc81c\uc791\uc0ac Insurge\uc758 \uc791\ud488\uc73c\ub85c \uacc4\ud68d\ud588\uc5c8\uc73c\ub098, \ud574\ub2f9 \uc81c\uc791\uc0ac\uac00 \uc815\ub9ac\ub418\uba74\uc11c, \ud30c\ub77c\ub9c8\uc6b4\ud2b8 \ubc30\uae09 \uc601\ud654\ub85c \ud655\uc7a5\ub418\uc5c8\ub2e4. \uc601\ud654\uac00 \uc81c\uc791\ub418\uace0 \uc788\uc744 \ub3d9\uc548\uc5d0 \uc5d0\uc774\ube0c\ub7fc\uc2a4\ub294 \ud074\ub85c\ub7ec\ud544\ub4dc\uc640 \uc774 \uc601\ud654\ub97c \uc5f0\uacb0\uc2dc\ud0a4\uae30\ub85c \uacb0\uc815\ud558\uba74\uc11c, \u300a\ud074\ub85c\ubc84\ud544\ub4dc 10\ubc88\uc9c0\u300b\uc758 \uc6d0\uc791 \uac01\ubcf8 \uc140\ub7ec \u300aThe Cellar\u300b\uc5d0\uc11c \uac19\uc740 \ubc29\ubc95\uc744 \ud588\ub358\uac70\ucc98\ub7fc \uc6b0\uc9c0\uc5d8\uc758 \uac01\ubcf8\uc744 \uac01\uc0c9\ud558\uace0 \ud074\ub85c\ubc84\ud544\ub4dc\uc640 \uc5f0\uacb0\ub418\ub294 \uc7a5\uba74\uc744 \ucd94\uac00\ud588\ub2e4. \uc5d0\uc774\ube0c\ub7fc\uc2a4\ub294 \uc785\uc790 \uac00\uc18d\uae30 \uc0ac\uace0\uac00 \ubbf8\ub798\uc758 \uc0ac\uac74\ub4e4\uc774 \uacfc\uac70\uc5d0 \ubcc0\ud654\ub97c \uc77c\uc73c\ud0ac \uc218\uc788\ub294 \uc218\ub2e8\uc73c\ub85c \uc0ac\uc6a9\ub420 \uc218 \uc788\uc74c\uc744 \uc788\ub2e4\uace0 \ubcf4\uc558\uace0, \ud568\uaed8 \ud074\ub85c\ubc84\ud544\ub4dc \ud504\ub79c\ucc28\uc774\uc988\uc5d0 \uc11c\uc220\uc801\uc73c\ub85c \uc5f0\uacb0\ub418\ub294 \ubc29\ubc95\uc744 \uad6c\ucd95\ud588\ub2e4.", "paragraph_answer": "\uc774 \uc601\ud654\ub294 \uc624\ub80c \uc6b0\uc9c0\uc5d8\uc774 \uc6b0\uc8fc \uc815\uac70\uc7a5\uc758 \uc120\uc6d0\ub4e4\uc744 \uc8fc\uc694 \uc904\uac70\ub9ac\ub85c \ub2e4\ub8e8\ub294 \u300a \uac13 \ud30c\ud2f0\ud074 \u300b(God Particle)\uc744 \uac01\uc0c9\ud55c \uac83\uc774\uba70, \uc6d0\ub798\ub294 \u300a\ud074\ub85c\ubc84\ud544\ub4dc\u300b\uc640 \uad00\ub828\uc774 \uc5c6\uc5c8\ub2e4. \uc774 \uac01\ubcf8\uc740 2012\ub144\uc5d0 \ud30c\ub77c\ub9c8\uc6b4\ud2b8 \ud53d\uccd0\uc2a4\uc640 \ubc30\ub4dc \ub85c\ubd07 \ud504\ub85c\ub355\uc158\uc2a4\uac00 \ud68d\ub4dd\ud588\ub2e4. \ucd08\uae30\uc5d0\ub294 \ud30c\ub77c\ub9c8\uc6b4\ud2b8\uc758 \uc800\uc608\uc0b0 \ubc30\uae09 \uc81c\uc791\uc0ac Insurge\uc758 \uc791\ud488\uc73c\ub85c \uacc4\ud68d\ud588\uc5c8\uc73c\ub098, \ud574\ub2f9 \uc81c\uc791\uc0ac\uac00 \uc815\ub9ac\ub418\uba74\uc11c, \ud30c\ub77c\ub9c8\uc6b4\ud2b8 \ubc30\uae09 \uc601\ud654\ub85c \ud655\uc7a5\ub418\uc5c8\ub2e4. \uc601\ud654\uac00 \uc81c\uc791\ub418\uace0 \uc788\uc744 \ub3d9\uc548\uc5d0 \uc5d0\uc774\ube0c\ub7fc\uc2a4\ub294 \ud074\ub85c\ub7ec\ud544\ub4dc\uc640 \uc774 \uc601\ud654\ub97c \uc5f0\uacb0\uc2dc\ud0a4\uae30\ub85c \uacb0\uc815\ud558\uba74\uc11c, \u300a\ud074\ub85c\ubc84\ud544\ub4dc 10\ubc88\uc9c0\u300b\uc758 \uc6d0\uc791 \uac01\ubcf8 \uc140\ub7ec \u300aThe Cellar\u300b\uc5d0\uc11c \uac19\uc740 \ubc29\ubc95\uc744 \ud588\ub358\uac70\ucc98\ub7fc \uc6b0\uc9c0\uc5d8\uc758 \uac01\ubcf8\uc744 \uac01\uc0c9\ud558\uace0 \ud074\ub85c\ubc84\ud544\ub4dc\uc640 \uc5f0\uacb0\ub418\ub294 \uc7a5\uba74\uc744 \ucd94\uac00\ud588\ub2e4. \uc5d0\uc774\ube0c\ub7fc\uc2a4\ub294 \uc785\uc790 \uac00\uc18d\uae30 \uc0ac\uace0\uac00 \ubbf8\ub798\uc758 \uc0ac\uac74\ub4e4\uc774 \uacfc\uac70\uc5d0 \ubcc0\ud654\ub97c \uc77c\uc73c\ud0ac \uc218\uc788\ub294 \uc218\ub2e8\uc73c\ub85c \uc0ac\uc6a9\ub420 \uc218 \uc788\uc74c\uc744 \uc788\ub2e4\uace0 \ubcf4\uc558\uace0, \ud568\uaed8 \ud074\ub85c\ubc84\ud544\ub4dc \ud504\ub79c\ucc28\uc774\uc988\uc5d0 \uc11c\uc220\uc801\uc73c\ub85c \uc5f0\uacb0\ub418\ub294 \ubc29\ubc95\uc744 \uad6c\ucd95\ud588\ub2e4.", "sentence_answer": "\uc774 \uc601\ud654\ub294 \uc624\ub80c \uc6b0\uc9c0\uc5d8\uc774 \uc6b0\uc8fc \uc815\uac70\uc7a5\uc758 \uc120\uc6d0\ub4e4\uc744 \uc8fc\uc694 \uc904\uac70\ub9ac\ub85c \ub2e4\ub8e8\ub294 \u300a \uac13 \ud30c\ud2f0\ud074 \u300b(God Particle)\uc744 \uac01\uc0c9\ud55c \uac83\uc774\uba70, \uc6d0\ub798\ub294 \u300a\ud074\ub85c\ubc84\ud544\ub4dc\u300b\uc640 \uad00\ub828\uc774 \uc5c6\uc5c8\ub2e4.", "paragraph_id": "5910360-1-0", "paragraph_question": "question: \ud074\ub85c\ubc84\ud544\ub4dc \ud328\ub7ec\ub3c5\uc2a4\uac00 \uac01\uc0c9\ud55c \uac83\uc740?, context: \uc774 \uc601\ud654\ub294 \uc624\ub80c \uc6b0\uc9c0\uc5d8\uc774 \uc6b0\uc8fc \uc815\uac70\uc7a5\uc758 \uc120\uc6d0\ub4e4\uc744 \uc8fc\uc694 \uc904\uac70\ub9ac\ub85c \ub2e4\ub8e8\ub294 \u300a\uac13 \ud30c\ud2f0\ud074\u300b(God Particle)\uc744 \uac01\uc0c9\ud55c \uac83\uc774\uba70, \uc6d0\ub798\ub294 \u300a\ud074\ub85c\ubc84\ud544\ub4dc\u300b\uc640 \uad00\ub828\uc774 \uc5c6\uc5c8\ub2e4. \uc774 \uac01\ubcf8\uc740 2012\ub144\uc5d0 \ud30c\ub77c\ub9c8\uc6b4\ud2b8 \ud53d\uccd0\uc2a4\uc640 \ubc30\ub4dc \ub85c\ubd07 \ud504\ub85c\ub355\uc158\uc2a4\uac00 \ud68d\ub4dd\ud588\ub2e4. \ucd08\uae30\uc5d0\ub294 \ud30c\ub77c\ub9c8\uc6b4\ud2b8\uc758 \uc800\uc608\uc0b0 \ubc30\uae09 \uc81c\uc791\uc0ac Insurge\uc758 \uc791\ud488\uc73c\ub85c \uacc4\ud68d\ud588\uc5c8\uc73c\ub098, \ud574\ub2f9 \uc81c\uc791\uc0ac\uac00 \uc815\ub9ac\ub418\uba74\uc11c, \ud30c\ub77c\ub9c8\uc6b4\ud2b8 \ubc30\uae09 \uc601\ud654\ub85c \ud655\uc7a5\ub418\uc5c8\ub2e4. \uc601\ud654\uac00 \uc81c\uc791\ub418\uace0 \uc788\uc744 \ub3d9\uc548\uc5d0 \uc5d0\uc774\ube0c\ub7fc\uc2a4\ub294 \ud074\ub85c\ub7ec\ud544\ub4dc\uc640 \uc774 \uc601\ud654\ub97c \uc5f0\uacb0\uc2dc\ud0a4\uae30\ub85c \uacb0\uc815\ud558\uba74\uc11c, \u300a\ud074\ub85c\ubc84\ud544\ub4dc 10\ubc88\uc9c0\u300b\uc758 \uc6d0\uc791 \uac01\ubcf8 \uc140\ub7ec \u300aThe Cellar\u300b\uc5d0\uc11c \uac19\uc740 \ubc29\ubc95\uc744 \ud588\ub358\uac70\ucc98\ub7fc \uc6b0\uc9c0\uc5d8\uc758 \uac01\ubcf8\uc744 \uac01\uc0c9\ud558\uace0 \ud074\ub85c\ubc84\ud544\ub4dc\uc640 \uc5f0\uacb0\ub418\ub294 \uc7a5\uba74\uc744 \ucd94\uac00\ud588\ub2e4. \uc5d0\uc774\ube0c\ub7fc\uc2a4\ub294 \uc785\uc790 \uac00\uc18d\uae30 \uc0ac\uace0\uac00 \ubbf8\ub798\uc758 \uc0ac\uac74\ub4e4\uc774 \uacfc\uac70\uc5d0 \ubcc0\ud654\ub97c \uc77c\uc73c\ud0ac \uc218\uc788\ub294 \uc218\ub2e8\uc73c\ub85c \uc0ac\uc6a9\ub420 \uc218 \uc788\uc74c\uc744 \uc788\ub2e4\uace0 \ubcf4\uc558\uace0, \ud568\uaed8 \ud074\ub85c\ubc84\ud544\ub4dc \ud504\ub79c\ucc28\uc774\uc988\uc5d0 \uc11c\uc220\uc801\uc73c\ub85c \uc5f0\uacb0\ub418\ub294 \ubc29\ubc95\uc744 \uad6c\ucd95\ud588\ub2e4."} {"question": "\ubd81\ud55c\uc5d0 1\ub144 5\uac1c\uc6d4 \uac00\ub7c9 \uc5b5\ub958\ub418\uc5b4 \uc788\ub2e4\uac00 \ud63c\uc218\uc0c1\ud0dc\ub85c \ubbf8\uad6d\uc73c\ub85c \ub3cc\uc544\uc640 \uc77c\uc8fc\uc77c\ub9cc\uc5d0 \uc0ac\ub9dd\ud55c \uc0ac\ub78c\uc740 \ub204\uad6c\uc778\uac00?", "paragraph": "\uc774\ub97c \uc704\ud574 \ud55c\ubbf8 \uc815\uc0c1\ud68c\ub2f4\uc758 \uc870\uae30 \uc131\uc0ac\uac00 \ucd94\uc9c4\ub418\uc5c8\ub2e4. \uac00\uc7a5 \uc911\uc694\ud55c \uc7c1\uc810\uc740 \uc0ac\ub4dc \ubc30\uce58\uc5d0 \uad00\ud55c \uac83\uc774\uc5c8\ub294\ub370, 6\uc6d4 9\uc77c \uc815\uc758\uc6a9 \uad6d\uac00\uc548\ubcf4\uc2e4\uc7a5\uc740 \"\uc0ac\ub4dc\ub294 \ubd81\ud55c\uc758 \uc810\uc99d\ud558\ub294 \uc704\ud611\uc73c\ub85c\ubd80\ud130 \ud55c\uad6d\uacfc \uc8fc\ud55c\ubbf8\uad70\uc744 \ubcf4\ud638\ud558\uae30 \uc704\ud574 \uacb0\uc815\ud55c \uac83\"\uc774\ub77c\uba70 \"\uc815\uad8c\uc774 \uad50\uccb4\ub418\uc5c8\ub2e4\uace0 \ud574\uc11c \uc774 \uacb0\uc815\uc744 \uacb0\ucf54 \uac00\ubccd\uac8c \uc5ec\uae30\uc9c0 \uc54a\uc744 \uac83\uc774\uba70 \ubbf8\uad6d\uacfc \uacc4\uc18d \uae34\ubc00\ud788 \ud611\uc758\ud560 \uac83\"\uc774\ub77c\uace0 \ubc1d\ud788\uba74\uc11c \"\uc815\ubd80\ub294 \ud55c\ubbf8\ub3d9\ub9f9 \ucc28\uc6d0\uc5d0\uc11c \uc57d\uc18d\ud55c \ub0b4\uc6a9\uc744 \uadfc\ubcf8\uc801\uc73c\ub85c \ubc14\uafb8\ub824\ub294 \uc758\ub3c4\ub294 \uc5c6\ub2e4\"\uace0 \uac15\uc870\ud588\ub2e4. \ub2e4\ub9cc, \ubbfc\uc8fc\uc801\u00b7\uc808\ucc28\uc801 \uc815\ub2f9\uc131\uc744 \uc704\ud574 \ud658\uacbd\uc601\ud5a5\ud3c9\uac00\uc758 \ud544\uc694\uc131\ub3c4 \uc5b8\uae09\ud588\ub2e4. \uc774\uc5d0 \ub300\ud574 \ubbf8\uad6d \ub0b4\uc5d0\uc11c\ub294 \"\uc0ac\ub4dc\uc758 \uc644\uc804\ud55c \ubc30\uce58\uc640 \uad00\ub828\ud55c \uc5b4\ub5a4 \ud658\uacbd\uc801 \uc6b0\ub824\ub3c4 \uc2e0\uc18d\ud558\uace0 \ucca0\uc800\ud55c \uac80\ud1a0\ub97c \ud1b5\ud574 \ud574\uc18c\ub418\uae38 \ubc14\ub780\ub2e4\"\uba74\uc11c\ub3c4 \ud658\uacbd\uc601\ud5a5\ud3c9\uac00\ub97c \uc774\ud574\ud558\uc9c0 \ubabb\ud558\uaca0\ub2e4\ub294 \uc758\uacac\uc774 \ub098\uc624\uae30\ub3c4 \ud588\ub2e4. 16\uc77c\uc5d0\ub294 \ubbf8\uad6d\uc744 \ubc29\ubb38 \uc911\uc778 \ubb38\uc815\uc778 \ud1b5\uc77c\uc678\uad50\uc548\ubcf4\ud2b9\ubcc4\ubcf4\uc88c\uad00\uc774 \"\ubd81\ud55c\uc774 \ud575 \ubbf8\uc0ac\uc77c \ud65c\ub3d9\uc744 \uc911\ub2e8\ud558\uba74 \ubbf8\uad6d\uc758 \ud55c\ubc18\ub3c4 \uc804\ub7b5\uc790\uc0b0\uacfc \ud55c\ubbf8\ud569\ub3d9\uad70\uc0ac\ud6c8\ub828\uc744 \ucd95\uc18c\ud560 \uc218 \uc788\ub2e4\"\uace0 \ubc1d\ud788\uace0 \ud559\uc790\ub85c\uc11c\uc758 \uacac\ud574\uc784\uc744 \uc804\uc81c\ud55c \ub4a4 \"\uc0ac\ub4dc \ubb38\uc81c\uac00 \ud574\uacb0\ub418\uc9c0 \uc54a\uc73c\uba74 \ud55c\ubbf8\ub3d9\ub9f9\uc774 \uae68\uc9c4\ub2e4\ub294 \uc778\uc2dd\uc774 \uc788\ub294\ub370 \uadf8\ub807\ub2e4\uba74 \uadf8\uac8c \ubb34\uc2a8 \ub3d9\ub9f9\uc774\ub0d0\"\ub294 \ubc1c\uc5b8\uc744 \ud558\uc600\ub2e4. \uc774\ub294 '\uc120 \ubd81\ud575\ub3d9\uacb0, \ud6c4 \uc644\uc804\ud55c \ube44\ud575\ud654'\ub97c \uc8fc\ucc3d\ud55c \ubb38\uc7ac\uc778 \uc815\ubd80\uc758 \uacf5\uc57d\uc5d0\uc11c \ud06c\uac8c \ubc97\uc5b4\ub098\uc9c0 \uc54a\uc9c0\ub9cc \ud55c\ubbf8\uc815\uc0c1\ud68c\ub2f4\uc744 \uc55e\ub450\ub85c \ud55c\ubbf8 \uac04 \uc774\uacac\uc73c\ub85c \ubc88\uc9c0\uba74\uc11c \ub17c\ub780\uc744 \uc77c\uc73c\ucf30\ub2e4. \uc774\uc5d0 \uad6d\ubc29\ubd80\ub294 \"\ud559\uc790 \uac1c\uc778\uc801 \uacac\ud574\uc784\uc744 \uc804\uc81c\ub85c \uc774\ub904\uc9c4 \uac83\uc774\uae30 \ub54c\ubb38\uc5d0 \uc815\ubd80\uc640 \uc870\uc728\ub41c \uc785\uc7a5\uc740 \uc544\ub2cc \uac83\uc73c\ub85c \uc54c\uace0 \uc788\ub2e4\"\uace0 \ud588\uc73c\uba70, \uccad\uc640\ub300\ub3c4 \"\ubb38 \ud2b9\ubcf4 \ubc1c\uc5b8\uc740 \uac1c\uc778 \uc544\uc774\ub514\uc5b4\"\ub77c\uba70 \uc0ac\uc804 \uc870\uc728\uc774 \uc5c6\uc5c8\uc74c\uc744 \uac15\uc870\ud588\ub2e4. \ubbf8\uad6d \uad6d\ubb34\ubd80\ub3c4 \"\uc6b0\ub9ac\ub294 \uc774\ub7f0 \uc2dc\uac01\uc774 \ubb38 \ud2b9\ubcf4\uc758 \uac1c\uc778\uc801 \uacac\ud574\ub85c \ud55c\uad6d \uc815\ubd80\uc758 \uacf5\uc2dd\uc801\uc778 \uc815\ucc45\uc744 \ubc18\uc601\ud55c \uac8c \uc544\ub2d0 \uc218 \uc788\ub294 \uac83\uc73c\ub85c \uc774\ud574\ud558\uace0 \uc788\ub2e4\"\ub294 \ubc18\uc751\uc744 \ubcf4\uc600\ub2e4. \uc774\ud6c4 20\uc77c\uc5d0\ub294 \ubd81\ud55c\uc5d0 1\ub144 5\uac1c\uc6d4\uac00\ub7c9 \uc5b5\ub958\ub418\uc5b4 \uc788\ub2e4\uac00 \ud574\ubc29\ub41c \uc624\ud1a0 \uc6dc\ube44\uc5b4\uac00 \ud63c\uc218\uc0c1\ud0dc\ub85c \ub3cc\uc544\uc628 \uc9c0 \uc77c\uc8fc\uc77c \ub9cc\uc5d0 \uc0ac\ub9dd\ud558\uba74\uc11c \ub610 \ub2e4\ub978 \ubb38\uc81c\ub97c \ub9cc\ub4e4\uc5c8\ub2e4. \ud558\uc9c0\ub9cc \uc815\uc0c1\ud68c\ub2f4\uc744 \uc55e\ub450\uace0 \uc5ec\ub7ec \uc554\ucd08\ub85c \ud55c\ubbf8 \uac08\ub4f1\uc774 \ud45c\uba74\ud654\ub418\ub294 \uc6b0\ub824\uc5d0 \ub300\ud574 \uccad\uc640\ub300\ub294 '\uc678\uad50\uc548\ubcf4\ub77c\uc778\uc774 \uacf5\uace0\ud558\uae30 \ub54c\ubb38\uc5d0 \uc0ac\ub4dc \ub17c\ub780\uc740 \uc624\ud574 \uc5c6\uc774 \uc798 \ud480\uc5c8\uace0, \ubb38 \ub300\ud1b5\ub839\uc774 \ubd81\ud55c \uc778\uad8c\ubb38\uc81c\uc5d0 \ub300\ud574 \ub2e8\ud638\ud55c \ud0dc\ub3c4\ub97c \ubc1d\ud78c \ub9cc\ud07c \ubbf8\uad6d\uacfc \ubd88\ud611\ud654\uc74c\uc774 \uc0dd\uae38 \uc5ec\uc9c0\uac00 \uc5c6\ub2e4'\ub294 \uc785\uc7a5\uc744 \ubcf4\uc600\ub2e4. \ub2e4\ub9cc, \uc0c8 \uc815\ubd80\uc758 \ub300\ubd81\uc811\uadfc\ubc95\uc774 \ud070 \ud2c0\uc5d0\uc11c \uc774\uc804 \uc815\uad8c\uc5d0 \ub2e4\ub97c \uc218 \uc788\ub2e4\ub294 \uc810\uc740 \ubbf8\uad6d\ub3c4 \uc778\uc815\ud574\uc57c \ud55c\ub2e4\uace0 \ub367\ubd99\uc600\ub2e4.", "answer": "\uc624\ud1a0 \uc6dc\ube44\uc5b4", "sentence": "\uc774\ud6c4 20\uc77c\uc5d0\ub294 \ubd81\ud55c\uc5d0 1\ub144 5\uac1c\uc6d4\uac00\ub7c9 \uc5b5\ub958\ub418\uc5b4 \uc788\ub2e4\uac00 \ud574\ubc29\ub41c \uc624\ud1a0 \uc6dc\ube44\uc5b4 \uac00 \ud63c\uc218\uc0c1\ud0dc\ub85c \ub3cc\uc544\uc628 \uc9c0 \uc77c\uc8fc\uc77c \ub9cc\uc5d0 \uc0ac\ub9dd\ud558\uba74\uc11c \ub610 \ub2e4\ub978 \ubb38\uc81c\ub97c \ub9cc\ub4e4\uc5c8\ub2e4.", "paragraph_sentence": "\uc774\ub97c \uc704\ud574 \ud55c\ubbf8 \uc815\uc0c1\ud68c\ub2f4\uc758 \uc870\uae30 \uc131\uc0ac\uac00 \ucd94\uc9c4\ub418\uc5c8\ub2e4. \uac00\uc7a5 \uc911\uc694\ud55c \uc7c1\uc810\uc740 \uc0ac\ub4dc \ubc30\uce58\uc5d0 \uad00\ud55c \uac83\uc774\uc5c8\ub294\ub370, 6\uc6d4 9\uc77c \uc815\uc758\uc6a9 \uad6d\uac00\uc548\ubcf4\uc2e4\uc7a5\uc740 \"\uc0ac\ub4dc\ub294 \ubd81\ud55c\uc758 \uc810\uc99d\ud558\ub294 \uc704\ud611\uc73c\ub85c\ubd80\ud130 \ud55c\uad6d\uacfc \uc8fc\ud55c\ubbf8\uad70\uc744 \ubcf4\ud638\ud558\uae30 \uc704\ud574 \uacb0\uc815\ud55c \uac83\"\uc774\ub77c\uba70 \"\uc815\uad8c\uc774 \uad50\uccb4\ub418\uc5c8\ub2e4\uace0 \ud574\uc11c \uc774 \uacb0\uc815\uc744 \uacb0\ucf54 \uac00\ubccd\uac8c \uc5ec\uae30\uc9c0 \uc54a\uc744 \uac83\uc774\uba70 \ubbf8\uad6d\uacfc \uacc4\uc18d \uae34\ubc00\ud788 \ud611\uc758\ud560 \uac83\"\uc774\ub77c\uace0 \ubc1d\ud788\uba74\uc11c \"\uc815\ubd80\ub294 \ud55c\ubbf8\ub3d9\ub9f9 \ucc28\uc6d0\uc5d0\uc11c \uc57d\uc18d\ud55c \ub0b4\uc6a9\uc744 \uadfc\ubcf8\uc801\uc73c\ub85c \ubc14\uafb8\ub824\ub294 \uc758\ub3c4\ub294 \uc5c6\ub2e4\"\uace0 \uac15\uc870\ud588\ub2e4. \ub2e4\ub9cc, \ubbfc\uc8fc\uc801\u00b7\uc808\ucc28\uc801 \uc815\ub2f9\uc131\uc744 \uc704\ud574 \ud658\uacbd\uc601\ud5a5\ud3c9\uac00\uc758 \ud544\uc694\uc131\ub3c4 \uc5b8\uae09\ud588\ub2e4. \uc774\uc5d0 \ub300\ud574 \ubbf8\uad6d \ub0b4\uc5d0\uc11c\ub294 \"\uc0ac\ub4dc\uc758 \uc644\uc804\ud55c \ubc30\uce58\uc640 \uad00\ub828\ud55c \uc5b4\ub5a4 \ud658\uacbd\uc801 \uc6b0\ub824\ub3c4 \uc2e0\uc18d\ud558\uace0 \ucca0\uc800\ud55c \uac80\ud1a0\ub97c \ud1b5\ud574 \ud574\uc18c\ub418\uae38 \ubc14\ub780\ub2e4\"\uba74\uc11c\ub3c4 \ud658\uacbd\uc601\ud5a5\ud3c9\uac00\ub97c \uc774\ud574\ud558\uc9c0 \ubabb\ud558\uaca0\ub2e4\ub294 \uc758\uacac\uc774 \ub098\uc624\uae30\ub3c4 \ud588\ub2e4. 16\uc77c\uc5d0\ub294 \ubbf8\uad6d\uc744 \ubc29\ubb38 \uc911\uc778 \ubb38\uc815\uc778 \ud1b5\uc77c\uc678\uad50\uc548\ubcf4\ud2b9\ubcc4\ubcf4\uc88c\uad00\uc774 \"\ubd81\ud55c\uc774 \ud575 \ubbf8\uc0ac\uc77c \ud65c\ub3d9\uc744 \uc911\ub2e8\ud558\uba74 \ubbf8\uad6d\uc758 \ud55c\ubc18\ub3c4 \uc804\ub7b5\uc790\uc0b0\uacfc \ud55c\ubbf8\ud569\ub3d9\uad70\uc0ac\ud6c8\ub828\uc744 \ucd95\uc18c\ud560 \uc218 \uc788\ub2e4\"\uace0 \ubc1d\ud788\uace0 \ud559\uc790\ub85c\uc11c\uc758 \uacac\ud574\uc784\uc744 \uc804\uc81c\ud55c \ub4a4 \"\uc0ac\ub4dc \ubb38\uc81c\uac00 \ud574\uacb0\ub418\uc9c0 \uc54a\uc73c\uba74 \ud55c\ubbf8\ub3d9\ub9f9\uc774 \uae68\uc9c4\ub2e4\ub294 \uc778\uc2dd\uc774 \uc788\ub294\ub370 \uadf8\ub807\ub2e4\uba74 \uadf8\uac8c \ubb34\uc2a8 \ub3d9\ub9f9\uc774\ub0d0\"\ub294 \ubc1c\uc5b8\uc744 \ud558\uc600\ub2e4. \uc774\ub294 '\uc120 \ubd81\ud575\ub3d9\uacb0, \ud6c4 \uc644\uc804\ud55c \ube44\ud575\ud654'\ub97c \uc8fc\ucc3d\ud55c \ubb38\uc7ac\uc778 \uc815\ubd80\uc758 \uacf5\uc57d\uc5d0\uc11c \ud06c\uac8c \ubc97\uc5b4\ub098\uc9c0 \uc54a\uc9c0\ub9cc \ud55c\ubbf8\uc815\uc0c1\ud68c\ub2f4\uc744 \uc55e\ub450\ub85c \ud55c\ubbf8 \uac04 \uc774\uacac\uc73c\ub85c \ubc88\uc9c0\uba74\uc11c \ub17c\ub780\uc744 \uc77c\uc73c\ucf30\ub2e4. \uc774\uc5d0 \uad6d\ubc29\ubd80\ub294 \"\ud559\uc790 \uac1c\uc778\uc801 \uacac\ud574\uc784\uc744 \uc804\uc81c\ub85c \uc774\ub904\uc9c4 \uac83\uc774\uae30 \ub54c\ubb38\uc5d0 \uc815\ubd80\uc640 \uc870\uc728\ub41c \uc785\uc7a5\uc740 \uc544\ub2cc \uac83\uc73c\ub85c \uc54c\uace0 \uc788\ub2e4\"\uace0 \ud588\uc73c\uba70, \uccad\uc640\ub300\ub3c4 \"\ubb38 \ud2b9\ubcf4 \ubc1c\uc5b8\uc740 \uac1c\uc778 \uc544\uc774\ub514\uc5b4\"\ub77c\uba70 \uc0ac\uc804 \uc870\uc728\uc774 \uc5c6\uc5c8\uc74c\uc744 \uac15\uc870\ud588\ub2e4. \ubbf8\uad6d \uad6d\ubb34\ubd80\ub3c4 \"\uc6b0\ub9ac\ub294 \uc774\ub7f0 \uc2dc\uac01\uc774 \ubb38 \ud2b9\ubcf4\uc758 \uac1c\uc778\uc801 \uacac\ud574\ub85c \ud55c\uad6d \uc815\ubd80\uc758 \uacf5\uc2dd\uc801\uc778 \uc815\ucc45\uc744 \ubc18\uc601\ud55c \uac8c \uc544\ub2d0 \uc218 \uc788\ub294 \uac83\uc73c\ub85c \uc774\ud574\ud558\uace0 \uc788\ub2e4\"\ub294 \ubc18\uc751\uc744 \ubcf4\uc600\ub2e4. \uc774\ud6c4 20\uc77c\uc5d0\ub294 \ubd81\ud55c\uc5d0 1\ub144 5\uac1c\uc6d4\uac00\ub7c9 \uc5b5\ub958\ub418\uc5b4 \uc788\ub2e4\uac00 \ud574\ubc29\ub41c \uc624\ud1a0 \uc6dc\ube44\uc5b4 \uac00 \ud63c\uc218\uc0c1\ud0dc\ub85c \ub3cc\uc544\uc628 \uc9c0 \uc77c\uc8fc\uc77c \ub9cc\uc5d0 \uc0ac\ub9dd\ud558\uba74\uc11c \ub610 \ub2e4\ub978 \ubb38\uc81c\ub97c \ub9cc\ub4e4\uc5c8\ub2e4. \ud558\uc9c0\ub9cc \uc815\uc0c1\ud68c\ub2f4\uc744 \uc55e\ub450\uace0 \uc5ec\ub7ec \uc554\ucd08\ub85c \ud55c\ubbf8 \uac08\ub4f1\uc774 \ud45c\uba74\ud654\ub418\ub294 \uc6b0\ub824\uc5d0 \ub300\ud574 \uccad\uc640\ub300\ub294 '\uc678\uad50\uc548\ubcf4\ub77c\uc778\uc774 \uacf5\uace0\ud558\uae30 \ub54c\ubb38\uc5d0 \uc0ac\ub4dc \ub17c\ub780\uc740 \uc624\ud574 \uc5c6\uc774 \uc798 \ud480\uc5c8\uace0, \ubb38 \ub300\ud1b5\ub839\uc774 \ubd81\ud55c \uc778\uad8c\ubb38\uc81c\uc5d0 \ub300\ud574 \ub2e8\ud638\ud55c \ud0dc\ub3c4\ub97c \ubc1d\ud78c \ub9cc\ud07c \ubbf8\uad6d\uacfc \ubd88\ud611\ud654\uc74c\uc774 \uc0dd\uae38 \uc5ec\uc9c0\uac00 \uc5c6\ub2e4'\ub294 \uc785\uc7a5\uc744 \ubcf4\uc600\ub2e4. \ub2e4\ub9cc, \uc0c8 \uc815\ubd80\uc758 \ub300\ubd81\uc811\uadfc\ubc95\uc774 \ud070 \ud2c0\uc5d0\uc11c \uc774\uc804 \uc815\uad8c\uc5d0 \ub2e4\ub97c \uc218 \uc788\ub2e4\ub294 \uc810\uc740 \ubbf8\uad6d\ub3c4 \uc778\uc815\ud574\uc57c \ud55c\ub2e4\uace0 \ub367\ubd99\uc600\ub2e4.", "paragraph_answer": "\uc774\ub97c \uc704\ud574 \ud55c\ubbf8 \uc815\uc0c1\ud68c\ub2f4\uc758 \uc870\uae30 \uc131\uc0ac\uac00 \ucd94\uc9c4\ub418\uc5c8\ub2e4. \uac00\uc7a5 \uc911\uc694\ud55c \uc7c1\uc810\uc740 \uc0ac\ub4dc \ubc30\uce58\uc5d0 \uad00\ud55c \uac83\uc774\uc5c8\ub294\ub370, 6\uc6d4 9\uc77c \uc815\uc758\uc6a9 \uad6d\uac00\uc548\ubcf4\uc2e4\uc7a5\uc740 \"\uc0ac\ub4dc\ub294 \ubd81\ud55c\uc758 \uc810\uc99d\ud558\ub294 \uc704\ud611\uc73c\ub85c\ubd80\ud130 \ud55c\uad6d\uacfc \uc8fc\ud55c\ubbf8\uad70\uc744 \ubcf4\ud638\ud558\uae30 \uc704\ud574 \uacb0\uc815\ud55c \uac83\"\uc774\ub77c\uba70 \"\uc815\uad8c\uc774 \uad50\uccb4\ub418\uc5c8\ub2e4\uace0 \ud574\uc11c \uc774 \uacb0\uc815\uc744 \uacb0\ucf54 \uac00\ubccd\uac8c \uc5ec\uae30\uc9c0 \uc54a\uc744 \uac83\uc774\uba70 \ubbf8\uad6d\uacfc \uacc4\uc18d \uae34\ubc00\ud788 \ud611\uc758\ud560 \uac83\"\uc774\ub77c\uace0 \ubc1d\ud788\uba74\uc11c \"\uc815\ubd80\ub294 \ud55c\ubbf8\ub3d9\ub9f9 \ucc28\uc6d0\uc5d0\uc11c \uc57d\uc18d\ud55c \ub0b4\uc6a9\uc744 \uadfc\ubcf8\uc801\uc73c\ub85c \ubc14\uafb8\ub824\ub294 \uc758\ub3c4\ub294 \uc5c6\ub2e4\"\uace0 \uac15\uc870\ud588\ub2e4. \ub2e4\ub9cc, \ubbfc\uc8fc\uc801\u00b7\uc808\ucc28\uc801 \uc815\ub2f9\uc131\uc744 \uc704\ud574 \ud658\uacbd\uc601\ud5a5\ud3c9\uac00\uc758 \ud544\uc694\uc131\ub3c4 \uc5b8\uae09\ud588\ub2e4. \uc774\uc5d0 \ub300\ud574 \ubbf8\uad6d \ub0b4\uc5d0\uc11c\ub294 \"\uc0ac\ub4dc\uc758 \uc644\uc804\ud55c \ubc30\uce58\uc640 \uad00\ub828\ud55c \uc5b4\ub5a4 \ud658\uacbd\uc801 \uc6b0\ub824\ub3c4 \uc2e0\uc18d\ud558\uace0 \ucca0\uc800\ud55c \uac80\ud1a0\ub97c \ud1b5\ud574 \ud574\uc18c\ub418\uae38 \ubc14\ub780\ub2e4\"\uba74\uc11c\ub3c4 \ud658\uacbd\uc601\ud5a5\ud3c9\uac00\ub97c \uc774\ud574\ud558\uc9c0 \ubabb\ud558\uaca0\ub2e4\ub294 \uc758\uacac\uc774 \ub098\uc624\uae30\ub3c4 \ud588\ub2e4. 16\uc77c\uc5d0\ub294 \ubbf8\uad6d\uc744 \ubc29\ubb38 \uc911\uc778 \ubb38\uc815\uc778 \ud1b5\uc77c\uc678\uad50\uc548\ubcf4\ud2b9\ubcc4\ubcf4\uc88c\uad00\uc774 \"\ubd81\ud55c\uc774 \ud575 \ubbf8\uc0ac\uc77c \ud65c\ub3d9\uc744 \uc911\ub2e8\ud558\uba74 \ubbf8\uad6d\uc758 \ud55c\ubc18\ub3c4 \uc804\ub7b5\uc790\uc0b0\uacfc \ud55c\ubbf8\ud569\ub3d9\uad70\uc0ac\ud6c8\ub828\uc744 \ucd95\uc18c\ud560 \uc218 \uc788\ub2e4\"\uace0 \ubc1d\ud788\uace0 \ud559\uc790\ub85c\uc11c\uc758 \uacac\ud574\uc784\uc744 \uc804\uc81c\ud55c \ub4a4 \"\uc0ac\ub4dc \ubb38\uc81c\uac00 \ud574\uacb0\ub418\uc9c0 \uc54a\uc73c\uba74 \ud55c\ubbf8\ub3d9\ub9f9\uc774 \uae68\uc9c4\ub2e4\ub294 \uc778\uc2dd\uc774 \uc788\ub294\ub370 \uadf8\ub807\ub2e4\uba74 \uadf8\uac8c \ubb34\uc2a8 \ub3d9\ub9f9\uc774\ub0d0\"\ub294 \ubc1c\uc5b8\uc744 \ud558\uc600\ub2e4. \uc774\ub294 '\uc120 \ubd81\ud575\ub3d9\uacb0, \ud6c4 \uc644\uc804\ud55c \ube44\ud575\ud654'\ub97c \uc8fc\ucc3d\ud55c \ubb38\uc7ac\uc778 \uc815\ubd80\uc758 \uacf5\uc57d\uc5d0\uc11c \ud06c\uac8c \ubc97\uc5b4\ub098\uc9c0 \uc54a\uc9c0\ub9cc \ud55c\ubbf8\uc815\uc0c1\ud68c\ub2f4\uc744 \uc55e\ub450\ub85c \ud55c\ubbf8 \uac04 \uc774\uacac\uc73c\ub85c \ubc88\uc9c0\uba74\uc11c \ub17c\ub780\uc744 \uc77c\uc73c\ucf30\ub2e4. \uc774\uc5d0 \uad6d\ubc29\ubd80\ub294 \"\ud559\uc790 \uac1c\uc778\uc801 \uacac\ud574\uc784\uc744 \uc804\uc81c\ub85c \uc774\ub904\uc9c4 \uac83\uc774\uae30 \ub54c\ubb38\uc5d0 \uc815\ubd80\uc640 \uc870\uc728\ub41c \uc785\uc7a5\uc740 \uc544\ub2cc \uac83\uc73c\ub85c \uc54c\uace0 \uc788\ub2e4\"\uace0 \ud588\uc73c\uba70, \uccad\uc640\ub300\ub3c4 \"\ubb38 \ud2b9\ubcf4 \ubc1c\uc5b8\uc740 \uac1c\uc778 \uc544\uc774\ub514\uc5b4\"\ub77c\uba70 \uc0ac\uc804 \uc870\uc728\uc774 \uc5c6\uc5c8\uc74c\uc744 \uac15\uc870\ud588\ub2e4. \ubbf8\uad6d \uad6d\ubb34\ubd80\ub3c4 \"\uc6b0\ub9ac\ub294 \uc774\ub7f0 \uc2dc\uac01\uc774 \ubb38 \ud2b9\ubcf4\uc758 \uac1c\uc778\uc801 \uacac\ud574\ub85c \ud55c\uad6d \uc815\ubd80\uc758 \uacf5\uc2dd\uc801\uc778 \uc815\ucc45\uc744 \ubc18\uc601\ud55c \uac8c \uc544\ub2d0 \uc218 \uc788\ub294 \uac83\uc73c\ub85c \uc774\ud574\ud558\uace0 \uc788\ub2e4\"\ub294 \ubc18\uc751\uc744 \ubcf4\uc600\ub2e4. \uc774\ud6c4 20\uc77c\uc5d0\ub294 \ubd81\ud55c\uc5d0 1\ub144 5\uac1c\uc6d4\uac00\ub7c9 \uc5b5\ub958\ub418\uc5b4 \uc788\ub2e4\uac00 \ud574\ubc29\ub41c \uc624\ud1a0 \uc6dc\ube44\uc5b4 \uac00 \ud63c\uc218\uc0c1\ud0dc\ub85c \ub3cc\uc544\uc628 \uc9c0 \uc77c\uc8fc\uc77c \ub9cc\uc5d0 \uc0ac\ub9dd\ud558\uba74\uc11c \ub610 \ub2e4\ub978 \ubb38\uc81c\ub97c \ub9cc\ub4e4\uc5c8\ub2e4. \ud558\uc9c0\ub9cc \uc815\uc0c1\ud68c\ub2f4\uc744 \uc55e\ub450\uace0 \uc5ec\ub7ec \uc554\ucd08\ub85c \ud55c\ubbf8 \uac08\ub4f1\uc774 \ud45c\uba74\ud654\ub418\ub294 \uc6b0\ub824\uc5d0 \ub300\ud574 \uccad\uc640\ub300\ub294 '\uc678\uad50\uc548\ubcf4\ub77c\uc778\uc774 \uacf5\uace0\ud558\uae30 \ub54c\ubb38\uc5d0 \uc0ac\ub4dc \ub17c\ub780\uc740 \uc624\ud574 \uc5c6\uc774 \uc798 \ud480\uc5c8\uace0, \ubb38 \ub300\ud1b5\ub839\uc774 \ubd81\ud55c \uc778\uad8c\ubb38\uc81c\uc5d0 \ub300\ud574 \ub2e8\ud638\ud55c \ud0dc\ub3c4\ub97c \ubc1d\ud78c \ub9cc\ud07c \ubbf8\uad6d\uacfc \ubd88\ud611\ud654\uc74c\uc774 \uc0dd\uae38 \uc5ec\uc9c0\uac00 \uc5c6\ub2e4'\ub294 \uc785\uc7a5\uc744 \ubcf4\uc600\ub2e4. \ub2e4\ub9cc, \uc0c8 \uc815\ubd80\uc758 \ub300\ubd81\uc811\uadfc\ubc95\uc774 \ud070 \ud2c0\uc5d0\uc11c \uc774\uc804 \uc815\uad8c\uc5d0 \ub2e4\ub97c \uc218 \uc788\ub2e4\ub294 \uc810\uc740 \ubbf8\uad6d\ub3c4 \uc778\uc815\ud574\uc57c \ud55c\ub2e4\uace0 \ub367\ubd99\uc600\ub2e4.", "sentence_answer": "\uc774\ud6c4 20\uc77c\uc5d0\ub294 \ubd81\ud55c\uc5d0 1\ub144 5\uac1c\uc6d4\uac00\ub7c9 \uc5b5\ub958\ub418\uc5b4 \uc788\ub2e4\uac00 \ud574\ubc29\ub41c \uc624\ud1a0 \uc6dc\ube44\uc5b4 \uac00 \ud63c\uc218\uc0c1\ud0dc\ub85c \ub3cc\uc544\uc628 \uc9c0 \uc77c\uc8fc\uc77c \ub9cc\uc5d0 \uc0ac\ub9dd\ud558\uba74\uc11c \ub610 \ub2e4\ub978 \ubb38\uc81c\ub97c \ub9cc\ub4e4\uc5c8\ub2e4.", "paragraph_id": "6580768-4-2", "paragraph_question": "question: \ubd81\ud55c\uc5d0 1\ub144 5\uac1c\uc6d4 \uac00\ub7c9 \uc5b5\ub958\ub418\uc5b4 \uc788\ub2e4\uac00 \ud63c\uc218\uc0c1\ud0dc\ub85c \ubbf8\uad6d\uc73c\ub85c \ub3cc\uc544\uc640 \uc77c\uc8fc\uc77c\ub9cc\uc5d0 \uc0ac\ub9dd\ud55c \uc0ac\ub78c\uc740 \ub204\uad6c\uc778\uac00?, context: \uc774\ub97c \uc704\ud574 \ud55c\ubbf8 \uc815\uc0c1\ud68c\ub2f4\uc758 \uc870\uae30 \uc131\uc0ac\uac00 \ucd94\uc9c4\ub418\uc5c8\ub2e4. \uac00\uc7a5 \uc911\uc694\ud55c \uc7c1\uc810\uc740 \uc0ac\ub4dc \ubc30\uce58\uc5d0 \uad00\ud55c \uac83\uc774\uc5c8\ub294\ub370, 6\uc6d4 9\uc77c \uc815\uc758\uc6a9 \uad6d\uac00\uc548\ubcf4\uc2e4\uc7a5\uc740 \"\uc0ac\ub4dc\ub294 \ubd81\ud55c\uc758 \uc810\uc99d\ud558\ub294 \uc704\ud611\uc73c\ub85c\ubd80\ud130 \ud55c\uad6d\uacfc \uc8fc\ud55c\ubbf8\uad70\uc744 \ubcf4\ud638\ud558\uae30 \uc704\ud574 \uacb0\uc815\ud55c \uac83\"\uc774\ub77c\uba70 \"\uc815\uad8c\uc774 \uad50\uccb4\ub418\uc5c8\ub2e4\uace0 \ud574\uc11c \uc774 \uacb0\uc815\uc744 \uacb0\ucf54 \uac00\ubccd\uac8c \uc5ec\uae30\uc9c0 \uc54a\uc744 \uac83\uc774\uba70 \ubbf8\uad6d\uacfc \uacc4\uc18d \uae34\ubc00\ud788 \ud611\uc758\ud560 \uac83\"\uc774\ub77c\uace0 \ubc1d\ud788\uba74\uc11c \"\uc815\ubd80\ub294 \ud55c\ubbf8\ub3d9\ub9f9 \ucc28\uc6d0\uc5d0\uc11c \uc57d\uc18d\ud55c \ub0b4\uc6a9\uc744 \uadfc\ubcf8\uc801\uc73c\ub85c \ubc14\uafb8\ub824\ub294 \uc758\ub3c4\ub294 \uc5c6\ub2e4\"\uace0 \uac15\uc870\ud588\ub2e4. \ub2e4\ub9cc, \ubbfc\uc8fc\uc801\u00b7\uc808\ucc28\uc801 \uc815\ub2f9\uc131\uc744 \uc704\ud574 \ud658\uacbd\uc601\ud5a5\ud3c9\uac00\uc758 \ud544\uc694\uc131\ub3c4 \uc5b8\uae09\ud588\ub2e4. \uc774\uc5d0 \ub300\ud574 \ubbf8\uad6d \ub0b4\uc5d0\uc11c\ub294 \"\uc0ac\ub4dc\uc758 \uc644\uc804\ud55c \ubc30\uce58\uc640 \uad00\ub828\ud55c \uc5b4\ub5a4 \ud658\uacbd\uc801 \uc6b0\ub824\ub3c4 \uc2e0\uc18d\ud558\uace0 \ucca0\uc800\ud55c \uac80\ud1a0\ub97c \ud1b5\ud574 \ud574\uc18c\ub418\uae38 \ubc14\ub780\ub2e4\"\uba74\uc11c\ub3c4 \ud658\uacbd\uc601\ud5a5\ud3c9\uac00\ub97c \uc774\ud574\ud558\uc9c0 \ubabb\ud558\uaca0\ub2e4\ub294 \uc758\uacac\uc774 \ub098\uc624\uae30\ub3c4 \ud588\ub2e4. 16\uc77c\uc5d0\ub294 \ubbf8\uad6d\uc744 \ubc29\ubb38 \uc911\uc778 \ubb38\uc815\uc778 \ud1b5\uc77c\uc678\uad50\uc548\ubcf4\ud2b9\ubcc4\ubcf4\uc88c\uad00\uc774 \"\ubd81\ud55c\uc774 \ud575 \ubbf8\uc0ac\uc77c \ud65c\ub3d9\uc744 \uc911\ub2e8\ud558\uba74 \ubbf8\uad6d\uc758 \ud55c\ubc18\ub3c4 \uc804\ub7b5\uc790\uc0b0\uacfc \ud55c\ubbf8\ud569\ub3d9\uad70\uc0ac\ud6c8\ub828\uc744 \ucd95\uc18c\ud560 \uc218 \uc788\ub2e4\"\uace0 \ubc1d\ud788\uace0 \ud559\uc790\ub85c\uc11c\uc758 \uacac\ud574\uc784\uc744 \uc804\uc81c\ud55c \ub4a4 \"\uc0ac\ub4dc \ubb38\uc81c\uac00 \ud574\uacb0\ub418\uc9c0 \uc54a\uc73c\uba74 \ud55c\ubbf8\ub3d9\ub9f9\uc774 \uae68\uc9c4\ub2e4\ub294 \uc778\uc2dd\uc774 \uc788\ub294\ub370 \uadf8\ub807\ub2e4\uba74 \uadf8\uac8c \ubb34\uc2a8 \ub3d9\ub9f9\uc774\ub0d0\"\ub294 \ubc1c\uc5b8\uc744 \ud558\uc600\ub2e4. \uc774\ub294 '\uc120 \ubd81\ud575\ub3d9\uacb0, \ud6c4 \uc644\uc804\ud55c \ube44\ud575\ud654'\ub97c \uc8fc\ucc3d\ud55c \ubb38\uc7ac\uc778 \uc815\ubd80\uc758 \uacf5\uc57d\uc5d0\uc11c \ud06c\uac8c \ubc97\uc5b4\ub098\uc9c0 \uc54a\uc9c0\ub9cc \ud55c\ubbf8\uc815\uc0c1\ud68c\ub2f4\uc744 \uc55e\ub450\ub85c \ud55c\ubbf8 \uac04 \uc774\uacac\uc73c\ub85c \ubc88\uc9c0\uba74\uc11c \ub17c\ub780\uc744 \uc77c\uc73c\ucf30\ub2e4. \uc774\uc5d0 \uad6d\ubc29\ubd80\ub294 \"\ud559\uc790 \uac1c\uc778\uc801 \uacac\ud574\uc784\uc744 \uc804\uc81c\ub85c \uc774\ub904\uc9c4 \uac83\uc774\uae30 \ub54c\ubb38\uc5d0 \uc815\ubd80\uc640 \uc870\uc728\ub41c \uc785\uc7a5\uc740 \uc544\ub2cc \uac83\uc73c\ub85c \uc54c\uace0 \uc788\ub2e4\"\uace0 \ud588\uc73c\uba70, \uccad\uc640\ub300\ub3c4 \"\ubb38 \ud2b9\ubcf4 \ubc1c\uc5b8\uc740 \uac1c\uc778 \uc544\uc774\ub514\uc5b4\"\ub77c\uba70 \uc0ac\uc804 \uc870\uc728\uc774 \uc5c6\uc5c8\uc74c\uc744 \uac15\uc870\ud588\ub2e4. \ubbf8\uad6d \uad6d\ubb34\ubd80\ub3c4 \"\uc6b0\ub9ac\ub294 \uc774\ub7f0 \uc2dc\uac01\uc774 \ubb38 \ud2b9\ubcf4\uc758 \uac1c\uc778\uc801 \uacac\ud574\ub85c \ud55c\uad6d \uc815\ubd80\uc758 \uacf5\uc2dd\uc801\uc778 \uc815\ucc45\uc744 \ubc18\uc601\ud55c \uac8c \uc544\ub2d0 \uc218 \uc788\ub294 \uac83\uc73c\ub85c \uc774\ud574\ud558\uace0 \uc788\ub2e4\"\ub294 \ubc18\uc751\uc744 \ubcf4\uc600\ub2e4. \uc774\ud6c4 20\uc77c\uc5d0\ub294 \ubd81\ud55c\uc5d0 1\ub144 5\uac1c\uc6d4\uac00\ub7c9 \uc5b5\ub958\ub418\uc5b4 \uc788\ub2e4\uac00 \ud574\ubc29\ub41c \uc624\ud1a0 \uc6dc\ube44\uc5b4\uac00 \ud63c\uc218\uc0c1\ud0dc\ub85c \ub3cc\uc544\uc628 \uc9c0 \uc77c\uc8fc\uc77c \ub9cc\uc5d0 \uc0ac\ub9dd\ud558\uba74\uc11c \ub610 \ub2e4\ub978 \ubb38\uc81c\ub97c \ub9cc\ub4e4\uc5c8\ub2e4. \ud558\uc9c0\ub9cc \uc815\uc0c1\ud68c\ub2f4\uc744 \uc55e\ub450\uace0 \uc5ec\ub7ec \uc554\ucd08\ub85c \ud55c\ubbf8 \uac08\ub4f1\uc774 \ud45c\uba74\ud654\ub418\ub294 \uc6b0\ub824\uc5d0 \ub300\ud574 \uccad\uc640\ub300\ub294 '\uc678\uad50\uc548\ubcf4\ub77c\uc778\uc774 \uacf5\uace0\ud558\uae30 \ub54c\ubb38\uc5d0 \uc0ac\ub4dc \ub17c\ub780\uc740 \uc624\ud574 \uc5c6\uc774 \uc798 \ud480\uc5c8\uace0, \ubb38 \ub300\ud1b5\ub839\uc774 \ubd81\ud55c \uc778\uad8c\ubb38\uc81c\uc5d0 \ub300\ud574 \ub2e8\ud638\ud55c \ud0dc\ub3c4\ub97c \ubc1d\ud78c \ub9cc\ud07c \ubbf8\uad6d\uacfc \ubd88\ud611\ud654\uc74c\uc774 \uc0dd\uae38 \uc5ec\uc9c0\uac00 \uc5c6\ub2e4'\ub294 \uc785\uc7a5\uc744 \ubcf4\uc600\ub2e4. \ub2e4\ub9cc, \uc0c8 \uc815\ubd80\uc758 \ub300\ubd81\uc811\uadfc\ubc95\uc774 \ud070 \ud2c0\uc5d0\uc11c \uc774\uc804 \uc815\uad8c\uc5d0 \ub2e4\ub97c \uc218 \uc788\ub2e4\ub294 \uc810\uc740 \ubbf8\uad6d\ub3c4 \uc778\uc815\ud574\uc57c \ud55c\ub2e4\uace0 \ub367\ubd99\uc600\ub2e4."} {"question": "\uc655\ub140 \ud06c\ub9ac\uc2a4\ud2f0\ub098\uc640 \uad00\uacc4\uac00 \uc88b\uc9c0 \uc54a\uc558\ub358 \ubaa8\uce5c\uc774\ub984\uc740?", "paragraph": "1626\ub144 \uc2a4\uc6e8\ub374 \uc2a4\ud1a1\ud640\ub984\uc5d0\uc11c \ucd9c\uc0dd\ud558\uc600\ub2e4. \uc810\uc131\uc220\uc758 \uc0ad\uc758 \uc2dc\uae30\uc5d0 \ud0dc\uc5b4\ub098, \uc55e\uc73c\ub85c \ud0dc\uc5b4\ub0a0 \uc544\uae30\uac00 \uc0ac\ub0b4\uc544\uc774\uae30\ub97c \uc5f4\ub9dd\ud558\uba74\uc11c \uc774 \uc0ad\uc774 \uc544\uc774\uc5d0 \uc5b4\ub5a4 \uc601\ud5a5\uc774 \uc788\uc744\uae4c \ud558\ub294 \uc5b5\uce21\ub4e4\uc774 \uc77c\uc5b4\ub0ac\ub2e4. \uc655\ube44\uc640\uc758 \uc0ac\uc774\uc5d0\uc11c \uc5bb\uc740 \ub450 \uc544\ub4e4\uc744 \uac01\uac01 \uc0ac\uc0b0\uacfc \uc870\uc878\ub85c \uc783\uc740 \uad6c\uc2a4\ud0c0\ube0c 2\uc138\ub294 \ub0a8\uc790 \uc0c1\uc18d\uc790\uc5d0 \ub300\ud55c \uc555\ubc15\uc774 \ub9e4\uc6b0 \uc808\uc2e4\ud55c \uc0c1\ud669\uc774\uc5c8\ub2e4. \uae30\ub300\uc640 \ub2ec\ub9ac \uc655\ub140\uc778 \ud06c\ub9ac\uc2a4\ud2f0\ub098\uac00 \ud0dc\uc5b4\ub0ac\uc73c\ub098 \uad6c\uc2a4\ud0c0\ube0c 2\uc138\ub294 \uc2e4\ub9dd\ud558\uc9c0 \uc54a\uace0 \ud06c\ub9ac\uc2a4\ud2f0\ub098\uc5d0\uac8c \ub0a8\uc790 \uc637\uc744 \uc785\ud788\uace0 \ubcf4\ud1b5 \uc18c\ub144\ub4e4\uacfc \ub2e4\ub97c \ubc14 \uc5c6\ub294 \ubc29\uc2dd\uc73c\ub85c \uad50\uc721\uc2dc\ucf30\ub2e4. \ud06c\ub9ac\uc2a4\ud2f0\ub098 \ubcf8\uc778 \ub610\ud55c \uc774\uc640 \uac19\uc740 \ubc29\uc2dd\uc744 \ub098\uc058\uc9c0 \uc54a\uac8c \uc5ec\uaca8 \ubd80\uc655\uc774 \uc0ac\ub9dd\ud558\uc5ec \ub354 \uc774\uc0c1 \uadf8\ub7ec\ud55c \ubc29\uc2dd\uc744 \ub530\ub97c \ud544\uc694\uac00 \uc5c6\uc5b4\uc9c4 \ub4a4\uc5d0\ub3c4 \uc5ec\uc804\ud788 \ub0a8\uc790 \uac19\uc740 \uc758\uc0c1\uacfc \ud0dc\ub3c4\ub97c \uace0\uc218\ud558\uc600\ub2e4. \uadf8\ub7ec\ub098 \ud56d\uc0c1 \uad00\uacc4\uac00 \uc88b\uc558\ub358 \ubd80\uce5c\uacfc \ub2ec\ub9ac \ubaa8\uce5c\uacfc \ud06c\ub9ac\uc2a4\ud2f0\ub098\uc758 \uad00\uacc4\ub294 \uc88b\uc9c0 \uc54a\uc558\ub2e4. \ud638\uc5d4\ucd10\ub808\ub978 \uac00\ubb38 \ucd9c\uc2e0\uc774\uc5c8\ub358 \ub9c8\ub9ac\uc544 \uc5d8\ub808\uc624\ub178\ub77c\ub294 \uc815\uc2e0 \ud63c\ub780\uc758 \uae30\uc9c8\uc774 \uc788\uc5c8\uc73c\uba70, \uadf8\ub140\uc758 \ub09c\uc0b0\uc73c\ub85c \uc778\ud558\uc5ec \ud06c\ub9ac\uc2a4\ud2f0\ub098\uc5d0\uac8c \uc8c4\ucc45\uac10\uc744 \ub290\ub07c\uac8c \ud558\ub824\uace0 \uc2dc\ub3c4\ud588\ub2e4. \ub610 \ub2e4\ub978 \ubb34\uc11c\uc6b4 \uc774\uc57c\uae30\ub294 \uc655\uc704\ub97c \uc774\uc744 \uc0c1\uc18d\uc790\ub97c \ucd9c\uc0b0\ud558\ub824\ub294 \uc804\ub9dd\uc5d0 \ubc18\ud558\uc5ec \ud06c\ub9ac\uc2a4\ud2f0\ub098\uc5d0\uac8c \ud3b8\uacac\uc744 \uac00\uc9c0\uace0 \uc788\uc5c8\ub2e4.", "answer": "\ub9c8\ub9ac\uc544 \uc5d8\ub808\uc624\ub178\ub77c", "sentence": "\ud638\uc5d4\ucd10\ub808\ub978 \uac00\ubb38 \ucd9c\uc2e0\uc774\uc5c8\ub358 \ub9c8\ub9ac\uc544 \uc5d8\ub808\uc624\ub178\ub77c \ub294 \uc815\uc2e0 \ud63c\ub780\uc758 \uae30\uc9c8\uc774 \uc788\uc5c8\uc73c\uba70, \uadf8\ub140\uc758 \ub09c\uc0b0\uc73c\ub85c \uc778\ud558\uc5ec \ud06c\ub9ac\uc2a4\ud2f0\ub098\uc5d0\uac8c \uc8c4\ucc45\uac10\uc744 \ub290\ub07c\uac8c \ud558\ub824\uace0 \uc2dc\ub3c4\ud588\ub2e4.", "paragraph_sentence": "1626\ub144 \uc2a4\uc6e8\ub374 \uc2a4\ud1a1\ud640\ub984\uc5d0\uc11c \ucd9c\uc0dd\ud558\uc600\ub2e4. \uc810\uc131\uc220\uc758 \uc0ad\uc758 \uc2dc\uae30\uc5d0 \ud0dc\uc5b4\ub098, \uc55e\uc73c\ub85c \ud0dc\uc5b4\ub0a0 \uc544\uae30\uac00 \uc0ac\ub0b4\uc544\uc774\uae30\ub97c \uc5f4\ub9dd\ud558\uba74\uc11c \uc774 \uc0ad\uc774 \uc544\uc774\uc5d0 \uc5b4\ub5a4 \uc601\ud5a5\uc774 \uc788\uc744\uae4c \ud558\ub294 \uc5b5\uce21\ub4e4\uc774 \uc77c\uc5b4\ub0ac\ub2e4. \uc655\ube44\uc640\uc758 \uc0ac\uc774\uc5d0\uc11c \uc5bb\uc740 \ub450 \uc544\ub4e4\uc744 \uac01\uac01 \uc0ac\uc0b0\uacfc \uc870\uc878\ub85c \uc783\uc740 \uad6c\uc2a4\ud0c0\ube0c 2\uc138\ub294 \ub0a8\uc790 \uc0c1\uc18d\uc790\uc5d0 \ub300\ud55c \uc555\ubc15\uc774 \ub9e4\uc6b0 \uc808\uc2e4\ud55c \uc0c1\ud669\uc774\uc5c8\ub2e4. \uae30\ub300\uc640 \ub2ec\ub9ac \uc655\ub140\uc778 \ud06c\ub9ac\uc2a4\ud2f0\ub098\uac00 \ud0dc\uc5b4\ub0ac\uc73c\ub098 \uad6c\uc2a4\ud0c0\ube0c 2\uc138\ub294 \uc2e4\ub9dd\ud558\uc9c0 \uc54a\uace0 \ud06c\ub9ac\uc2a4\ud2f0\ub098\uc5d0\uac8c \ub0a8\uc790 \uc637\uc744 \uc785\ud788\uace0 \ubcf4\ud1b5 \uc18c\ub144\ub4e4\uacfc \ub2e4\ub97c \ubc14 \uc5c6\ub294 \ubc29\uc2dd\uc73c\ub85c \uad50\uc721\uc2dc\ucf30\ub2e4. \ud06c\ub9ac\uc2a4\ud2f0\ub098 \ubcf8\uc778 \ub610\ud55c \uc774\uc640 \uac19\uc740 \ubc29\uc2dd\uc744 \ub098\uc058\uc9c0 \uc54a\uac8c \uc5ec\uaca8 \ubd80\uc655\uc774 \uc0ac\ub9dd\ud558\uc5ec \ub354 \uc774\uc0c1 \uadf8\ub7ec\ud55c \ubc29\uc2dd\uc744 \ub530\ub97c \ud544\uc694\uac00 \uc5c6\uc5b4\uc9c4 \ub4a4\uc5d0\ub3c4 \uc5ec\uc804\ud788 \ub0a8\uc790 \uac19\uc740 \uc758\uc0c1\uacfc \ud0dc\ub3c4\ub97c \uace0\uc218\ud558\uc600\ub2e4. \uadf8\ub7ec\ub098 \ud56d\uc0c1 \uad00\uacc4\uac00 \uc88b\uc558\ub358 \ubd80\uce5c\uacfc \ub2ec\ub9ac \ubaa8\uce5c\uacfc \ud06c\ub9ac\uc2a4\ud2f0\ub098\uc758 \uad00\uacc4\ub294 \uc88b\uc9c0 \uc54a\uc558\ub2e4. \ud638\uc5d4\ucd10\ub808\ub978 \uac00\ubb38 \ucd9c\uc2e0\uc774\uc5c8\ub358 \ub9c8\ub9ac\uc544 \uc5d8\ub808\uc624\ub178\ub77c \ub294 \uc815\uc2e0 \ud63c\ub780\uc758 \uae30\uc9c8\uc774 \uc788\uc5c8\uc73c\uba70, \uadf8\ub140\uc758 \ub09c\uc0b0\uc73c\ub85c \uc778\ud558\uc5ec \ud06c\ub9ac\uc2a4\ud2f0\ub098\uc5d0\uac8c \uc8c4\ucc45\uac10\uc744 \ub290\ub07c\uac8c \ud558\ub824\uace0 \uc2dc\ub3c4\ud588\ub2e4. \ub610 \ub2e4\ub978 \ubb34\uc11c\uc6b4 \uc774\uc57c\uae30\ub294 \uc655\uc704\ub97c \uc774\uc744 \uc0c1\uc18d\uc790\ub97c \ucd9c\uc0b0\ud558\ub824\ub294 \uc804\ub9dd\uc5d0 \ubc18\ud558\uc5ec \ud06c\ub9ac\uc2a4\ud2f0\ub098\uc5d0\uac8c \ud3b8\uacac\uc744 \uac00\uc9c0\uace0 \uc788\uc5c8\ub2e4.", "paragraph_answer": "1626\ub144 \uc2a4\uc6e8\ub374 \uc2a4\ud1a1\ud640\ub984\uc5d0\uc11c \ucd9c\uc0dd\ud558\uc600\ub2e4. \uc810\uc131\uc220\uc758 \uc0ad\uc758 \uc2dc\uae30\uc5d0 \ud0dc\uc5b4\ub098, \uc55e\uc73c\ub85c \ud0dc\uc5b4\ub0a0 \uc544\uae30\uac00 \uc0ac\ub0b4\uc544\uc774\uae30\ub97c \uc5f4\ub9dd\ud558\uba74\uc11c \uc774 \uc0ad\uc774 \uc544\uc774\uc5d0 \uc5b4\ub5a4 \uc601\ud5a5\uc774 \uc788\uc744\uae4c \ud558\ub294 \uc5b5\uce21\ub4e4\uc774 \uc77c\uc5b4\ub0ac\ub2e4. \uc655\ube44\uc640\uc758 \uc0ac\uc774\uc5d0\uc11c \uc5bb\uc740 \ub450 \uc544\ub4e4\uc744 \uac01\uac01 \uc0ac\uc0b0\uacfc \uc870\uc878\ub85c \uc783\uc740 \uad6c\uc2a4\ud0c0\ube0c 2\uc138\ub294 \ub0a8\uc790 \uc0c1\uc18d\uc790\uc5d0 \ub300\ud55c \uc555\ubc15\uc774 \ub9e4\uc6b0 \uc808\uc2e4\ud55c \uc0c1\ud669\uc774\uc5c8\ub2e4. \uae30\ub300\uc640 \ub2ec\ub9ac \uc655\ub140\uc778 \ud06c\ub9ac\uc2a4\ud2f0\ub098\uac00 \ud0dc\uc5b4\ub0ac\uc73c\ub098 \uad6c\uc2a4\ud0c0\ube0c 2\uc138\ub294 \uc2e4\ub9dd\ud558\uc9c0 \uc54a\uace0 \ud06c\ub9ac\uc2a4\ud2f0\ub098\uc5d0\uac8c \ub0a8\uc790 \uc637\uc744 \uc785\ud788\uace0 \ubcf4\ud1b5 \uc18c\ub144\ub4e4\uacfc \ub2e4\ub97c \ubc14 \uc5c6\ub294 \ubc29\uc2dd\uc73c\ub85c \uad50\uc721\uc2dc\ucf30\ub2e4. \ud06c\ub9ac\uc2a4\ud2f0\ub098 \ubcf8\uc778 \ub610\ud55c \uc774\uc640 \uac19\uc740 \ubc29\uc2dd\uc744 \ub098\uc058\uc9c0 \uc54a\uac8c \uc5ec\uaca8 \ubd80\uc655\uc774 \uc0ac\ub9dd\ud558\uc5ec \ub354 \uc774\uc0c1 \uadf8\ub7ec\ud55c \ubc29\uc2dd\uc744 \ub530\ub97c \ud544\uc694\uac00 \uc5c6\uc5b4\uc9c4 \ub4a4\uc5d0\ub3c4 \uc5ec\uc804\ud788 \ub0a8\uc790 \uac19\uc740 \uc758\uc0c1\uacfc \ud0dc\ub3c4\ub97c \uace0\uc218\ud558\uc600\ub2e4. \uadf8\ub7ec\ub098 \ud56d\uc0c1 \uad00\uacc4\uac00 \uc88b\uc558\ub358 \ubd80\uce5c\uacfc \ub2ec\ub9ac \ubaa8\uce5c\uacfc \ud06c\ub9ac\uc2a4\ud2f0\ub098\uc758 \uad00\uacc4\ub294 \uc88b\uc9c0 \uc54a\uc558\ub2e4. \ud638\uc5d4\ucd10\ub808\ub978 \uac00\ubb38 \ucd9c\uc2e0\uc774\uc5c8\ub358 \ub9c8\ub9ac\uc544 \uc5d8\ub808\uc624\ub178\ub77c \ub294 \uc815\uc2e0 \ud63c\ub780\uc758 \uae30\uc9c8\uc774 \uc788\uc5c8\uc73c\uba70, \uadf8\ub140\uc758 \ub09c\uc0b0\uc73c\ub85c \uc778\ud558\uc5ec \ud06c\ub9ac\uc2a4\ud2f0\ub098\uc5d0\uac8c \uc8c4\ucc45\uac10\uc744 \ub290\ub07c\uac8c \ud558\ub824\uace0 \uc2dc\ub3c4\ud588\ub2e4. \ub610 \ub2e4\ub978 \ubb34\uc11c\uc6b4 \uc774\uc57c\uae30\ub294 \uc655\uc704\ub97c \uc774\uc744 \uc0c1\uc18d\uc790\ub97c \ucd9c\uc0b0\ud558\ub824\ub294 \uc804\ub9dd\uc5d0 \ubc18\ud558\uc5ec \ud06c\ub9ac\uc2a4\ud2f0\ub098\uc5d0\uac8c \ud3b8\uacac\uc744 \uac00\uc9c0\uace0 \uc788\uc5c8\ub2e4.", "sentence_answer": "\ud638\uc5d4\ucd10\ub808\ub978 \uac00\ubb38 \ucd9c\uc2e0\uc774\uc5c8\ub358 \ub9c8\ub9ac\uc544 \uc5d8\ub808\uc624\ub178\ub77c \ub294 \uc815\uc2e0 \ud63c\ub780\uc758 \uae30\uc9c8\uc774 \uc788\uc5c8\uc73c\uba70, \uadf8\ub140\uc758 \ub09c\uc0b0\uc73c\ub85c \uc778\ud558\uc5ec \ud06c\ub9ac\uc2a4\ud2f0\ub098\uc5d0\uac8c \uc8c4\ucc45\uac10\uc744 \ub290\ub07c\uac8c \ud558\ub824\uace0 \uc2dc\ub3c4\ud588\ub2e4.", "paragraph_id": "6564001-0-1", "paragraph_question": "question: \uc655\ub140 \ud06c\ub9ac\uc2a4\ud2f0\ub098\uc640 \uad00\uacc4\uac00 \uc88b\uc9c0 \uc54a\uc558\ub358 \ubaa8\uce5c\uc774\ub984\uc740?, context: 1626\ub144 \uc2a4\uc6e8\ub374 \uc2a4\ud1a1\ud640\ub984\uc5d0\uc11c \ucd9c\uc0dd\ud558\uc600\ub2e4. \uc810\uc131\uc220\uc758 \uc0ad\uc758 \uc2dc\uae30\uc5d0 \ud0dc\uc5b4\ub098, \uc55e\uc73c\ub85c \ud0dc\uc5b4\ub0a0 \uc544\uae30\uac00 \uc0ac\ub0b4\uc544\uc774\uae30\ub97c \uc5f4\ub9dd\ud558\uba74\uc11c \uc774 \uc0ad\uc774 \uc544\uc774\uc5d0 \uc5b4\ub5a4 \uc601\ud5a5\uc774 \uc788\uc744\uae4c \ud558\ub294 \uc5b5\uce21\ub4e4\uc774 \uc77c\uc5b4\ub0ac\ub2e4. \uc655\ube44\uc640\uc758 \uc0ac\uc774\uc5d0\uc11c \uc5bb\uc740 \ub450 \uc544\ub4e4\uc744 \uac01\uac01 \uc0ac\uc0b0\uacfc \uc870\uc878\ub85c \uc783\uc740 \uad6c\uc2a4\ud0c0\ube0c 2\uc138\ub294 \ub0a8\uc790 \uc0c1\uc18d\uc790\uc5d0 \ub300\ud55c \uc555\ubc15\uc774 \ub9e4\uc6b0 \uc808\uc2e4\ud55c \uc0c1\ud669\uc774\uc5c8\ub2e4. \uae30\ub300\uc640 \ub2ec\ub9ac \uc655\ub140\uc778 \ud06c\ub9ac\uc2a4\ud2f0\ub098\uac00 \ud0dc\uc5b4\ub0ac\uc73c\ub098 \uad6c\uc2a4\ud0c0\ube0c 2\uc138\ub294 \uc2e4\ub9dd\ud558\uc9c0 \uc54a\uace0 \ud06c\ub9ac\uc2a4\ud2f0\ub098\uc5d0\uac8c \ub0a8\uc790 \uc637\uc744 \uc785\ud788\uace0 \ubcf4\ud1b5 \uc18c\ub144\ub4e4\uacfc \ub2e4\ub97c \ubc14 \uc5c6\ub294 \ubc29\uc2dd\uc73c\ub85c \uad50\uc721\uc2dc\ucf30\ub2e4. \ud06c\ub9ac\uc2a4\ud2f0\ub098 \ubcf8\uc778 \ub610\ud55c \uc774\uc640 \uac19\uc740 \ubc29\uc2dd\uc744 \ub098\uc058\uc9c0 \uc54a\uac8c \uc5ec\uaca8 \ubd80\uc655\uc774 \uc0ac\ub9dd\ud558\uc5ec \ub354 \uc774\uc0c1 \uadf8\ub7ec\ud55c \ubc29\uc2dd\uc744 \ub530\ub97c \ud544\uc694\uac00 \uc5c6\uc5b4\uc9c4 \ub4a4\uc5d0\ub3c4 \uc5ec\uc804\ud788 \ub0a8\uc790 \uac19\uc740 \uc758\uc0c1\uacfc \ud0dc\ub3c4\ub97c \uace0\uc218\ud558\uc600\ub2e4. \uadf8\ub7ec\ub098 \ud56d\uc0c1 \uad00\uacc4\uac00 \uc88b\uc558\ub358 \ubd80\uce5c\uacfc \ub2ec\ub9ac \ubaa8\uce5c\uacfc \ud06c\ub9ac\uc2a4\ud2f0\ub098\uc758 \uad00\uacc4\ub294 \uc88b\uc9c0 \uc54a\uc558\ub2e4. \ud638\uc5d4\ucd10\ub808\ub978 \uac00\ubb38 \ucd9c\uc2e0\uc774\uc5c8\ub358 \ub9c8\ub9ac\uc544 \uc5d8\ub808\uc624\ub178\ub77c\ub294 \uc815\uc2e0 \ud63c\ub780\uc758 \uae30\uc9c8\uc774 \uc788\uc5c8\uc73c\uba70, \uadf8\ub140\uc758 \ub09c\uc0b0\uc73c\ub85c \uc778\ud558\uc5ec \ud06c\ub9ac\uc2a4\ud2f0\ub098\uc5d0\uac8c \uc8c4\ucc45\uac10\uc744 \ub290\ub07c\uac8c \ud558\ub824\uace0 \uc2dc\ub3c4\ud588\ub2e4. \ub610 \ub2e4\ub978 \ubb34\uc11c\uc6b4 \uc774\uc57c\uae30\ub294 \uc655\uc704\ub97c \uc774\uc744 \uc0c1\uc18d\uc790\ub97c \ucd9c\uc0b0\ud558\ub824\ub294 \uc804\ub9dd\uc5d0 \ubc18\ud558\uc5ec \ud06c\ub9ac\uc2a4\ud2f0\ub098\uc5d0\uac8c \ud3b8\uacac\uc744 \uac00\uc9c0\uace0 \uc788\uc5c8\ub2e4."} {"question": "\ubc18\uc790\ubcf8\uc8fc\uc758\uc801 \uc131\ud5a5\uc774 \uac15\ud55c \uc5ec\uc131\uc8fc\uc758 \uad00\ub828 \ubd84\ud30c\ub294 \ubb34\uc5c7\uc778\uac00?", "paragraph": "\uc5ec\uc131\uc8fc\uc758 \uad00\ub828 \ubd84\ud30c\uc911\uc5d0\uc11c\ub294 \uc0ac\ud68c\uc8fc\uc758\uc801 \uc5ec\uc131\uc8fc\uc758\uc790, \uc0dd\ud0dc\uc5ec\uc131\uc8fc\uc758\uc790\ub4e4\uc740 \ubc18\uc790\ubcf8\uc8fc\uc758\uc801 \uc131\ud5a5\uc774 \uac15\ud588\ub2e4. \uc790\ubcf8\uc8fc\uc758\ub294 \uc778\uac04\uc758 \uc9d0\uc2b9\uacfc \uac19\uc740 \uc695\ub9dd\uc744 \ucda9\uc871\uc2dc\ud0a4\uae30\uc704\ud574 \uc5ec\uc131\uc758 \uc2e0\uccb4\ub97c \uc0c1\ud488\ud654\ud588\ub2e4\ub294 \uc810\uc5d0\uc11c \ud070 \ube44\ud310\uc744 \ud588\uc73c\uba70, \ub0a8\uc131\uc758 \uc695\uc2ec\uacfc \uc790\ubcf8\uc8fc\uc758\uc758 \uc870\ud654\ub85c \uc804\uc7c1\uc774 \ud0c4\uc0dd\ud588\ub2e4\uace0 \uc8fc\uc7a5\ud588\ub2e4. \ub300\ud45c\uc801\uc73c\ub85c \uc0ac\ud68c\uc8fc\uc758\uc801 \uc5ec\uc131\uc8fc\uc758\uc5d0\uc11c \ubc18\uc790\ubcf8\uc8fc\uc758\ub294 \uc9c4\uc815\ud55c \uc758\ubbf8\uc758 \uc5ec\uc131 \ud574\ubc29\uc774 \uc77c\uc5b4\ub098\uae30 \uc704\ud574\uc11c\ub294 \uc5ec\uc131\ub3c4 \ub0a8\uc131\uacfc \uac19\uc740 \uacbd\uc81c\uc801 \ub2a5\ub825\uc744 \uac16\ucd94\ub294\ub370\uc11c \ube44\ub86f\ub41c\ub2e4\uace0 \ud558\uc600\ub2e4. \uc989, \uc790\ubcf8\uc8fc\uc758 \uccb4\uc81c \uc548\uc5d0\uc11c \uc5ec\uc131\uc758 \uacbd\uc81c\uc801 \ub2a5\ub825\uc774 \ud55c\uc815\ub418\uc5b4\uc788\uae30 \ub54c\ubb38\uc5d0 \uc790\ubcf8\uc8fc\uc758 \ub0b4\uc5d0\uc11c \uc5ec\uc131\uc740 \ub0a8\uc131\uc5d0 \ube44\ud574 \ub4a4\ub5a8\uc5b4\uc9c8 \uc218 \ubc16\uc5d0 \uc5c6\ub294 \uc874\uc7ac\ub85c \uc5ec\uaca8\uc9c8 \uc218 \ubc16\uc5d0 \uc5c6\ub294 \uccb4\uc81c\ub85c \uc778\uc2dd\ud558\uc600\ub2e4. \uadf8 \ub54c\ubb38\uc5d0 \uc0ac\ud68c\uc8fc\uc758\uc801 \uc5ec\uc131\uc8fc\uc758\uc5d0\uc11c \uc5ec\uc131\uc740 \ubaa8\ub450\uac00 \ud3c9\ub4f1\ud558\uac8c \uc77c\ud558\uace0 \ubc8c \uc218 \uc788\ub294 \uc0ac\ud68c\uc8fc\uc758\uc801 \uacbd\uc81c \uad6c\uc870\ub97c \uc9c0\uc9c0\ud574\uc57c\ud55c\ub2e4\uace0 \ud638\uc18c\ud588\ub2e4. \ub300\ud55c\ubbfc\uad6d\uc5d0\uc11c \ub300\ud45c\uc801\uc778 \uc0ac\ud68c\uc8fc\uc758\uc801 \uc5ec\uc131\uc8fc\uc758\uc790\ub294 \ud5c8\uc815\uc219\uc774\ub2e4. \uc0ac\ud68c\uc8fc\uc758\uc801 \uc5ec\uc131\uc8fc\uc758\uc790\ub4e4\uc740 \uc774\ub7ec\ud55c \uc790\ubcf8\uc8fc\uc758\uc801 \uad6c\uc870\ub3c4 \ube44\ud310\ud558\uc9c0\ub9cc, \ub0a8\uc131\uc5d0\uac8c \uc758\uc9c0\ud558\uc5ec \uc548\uc77c\ud558\uac8c \uc0ac\ub824\ub294 \uc790\ubcf8\uc8fc\uc758\uc801 \uc5ec\uc131\ub4e4 \ub610\ud55c \ube44\ud310\ud588\ub2e4.", "answer": "\uc0ac\ud68c\uc8fc\uc758\uc801 \uc5ec\uc131\uc8fc\uc758\uc790, \uc0dd\ud0dc\uc5ec\uc131\uc8fc\uc758\uc790", "sentence": "\uc5ec\uc131\uc8fc\uc758 \uad00\ub828 \ubd84\ud30c\uc911\uc5d0\uc11c\ub294 \uc0ac\ud68c\uc8fc\uc758\uc801 \uc5ec\uc131\uc8fc\uc758\uc790, \uc0dd\ud0dc\uc5ec\uc131\uc8fc\uc758\uc790 \ub4e4\uc740 \ubc18\uc790\ubcf8\uc8fc\uc758\uc801 \uc131\ud5a5\uc774 \uac15\ud588\ub2e4.", "paragraph_sentence": " \uc5ec\uc131\uc8fc\uc758 \uad00\ub828 \ubd84\ud30c\uc911\uc5d0\uc11c\ub294 \uc0ac\ud68c\uc8fc\uc758\uc801 \uc5ec\uc131\uc8fc\uc758\uc790, \uc0dd\ud0dc\uc5ec\uc131\uc8fc\uc758\uc790 \ub4e4\uc740 \ubc18\uc790\ubcf8\uc8fc\uc758\uc801 \uc131\ud5a5\uc774 \uac15\ud588\ub2e4. \uc790\ubcf8\uc8fc\uc758\ub294 \uc778\uac04\uc758 \uc9d0\uc2b9\uacfc \uac19\uc740 \uc695\ub9dd\uc744 \ucda9\uc871\uc2dc\ud0a4\uae30\uc704\ud574 \uc5ec\uc131\uc758 \uc2e0\uccb4\ub97c \uc0c1\ud488\ud654\ud588\ub2e4\ub294 \uc810\uc5d0\uc11c \ud070 \ube44\ud310\uc744 \ud588\uc73c\uba70, \ub0a8\uc131\uc758 \uc695\uc2ec\uacfc \uc790\ubcf8\uc8fc\uc758\uc758 \uc870\ud654\ub85c \uc804\uc7c1\uc774 \ud0c4\uc0dd\ud588\ub2e4\uace0 \uc8fc\uc7a5\ud588\ub2e4. \ub300\ud45c\uc801\uc73c\ub85c \uc0ac\ud68c\uc8fc\uc758\uc801 \uc5ec\uc131\uc8fc\uc758\uc5d0\uc11c \ubc18\uc790\ubcf8\uc8fc\uc758\ub294 \uc9c4\uc815\ud55c \uc758\ubbf8\uc758 \uc5ec\uc131 \ud574\ubc29\uc774 \uc77c\uc5b4\ub098\uae30 \uc704\ud574\uc11c\ub294 \uc5ec\uc131\ub3c4 \ub0a8\uc131\uacfc \uac19\uc740 \uacbd\uc81c\uc801 \ub2a5\ub825\uc744 \uac16\ucd94\ub294\ub370\uc11c \ube44\ub86f\ub41c\ub2e4\uace0 \ud558\uc600\ub2e4. \uc989, \uc790\ubcf8\uc8fc\uc758 \uccb4\uc81c \uc548\uc5d0\uc11c \uc5ec\uc131\uc758 \uacbd\uc81c\uc801 \ub2a5\ub825\uc774 \ud55c\uc815\ub418\uc5b4\uc788\uae30 \ub54c\ubb38\uc5d0 \uc790\ubcf8\uc8fc\uc758 \ub0b4\uc5d0\uc11c \uc5ec\uc131\uc740 \ub0a8\uc131\uc5d0 \ube44\ud574 \ub4a4\ub5a8\uc5b4\uc9c8 \uc218 \ubc16\uc5d0 \uc5c6\ub294 \uc874\uc7ac\ub85c \uc5ec\uaca8\uc9c8 \uc218 \ubc16\uc5d0 \uc5c6\ub294 \uccb4\uc81c\ub85c \uc778\uc2dd\ud558\uc600\ub2e4. \uadf8 \ub54c\ubb38\uc5d0 \uc0ac\ud68c\uc8fc\uc758\uc801 \uc5ec\uc131\uc8fc\uc758\uc5d0\uc11c \uc5ec\uc131\uc740 \ubaa8\ub450\uac00 \ud3c9\ub4f1\ud558\uac8c \uc77c\ud558\uace0 \ubc8c \uc218 \uc788\ub294 \uc0ac\ud68c\uc8fc\uc758\uc801 \uacbd\uc81c \uad6c\uc870\ub97c \uc9c0\uc9c0\ud574\uc57c\ud55c\ub2e4\uace0 \ud638\uc18c\ud588\ub2e4. \ub300\ud55c\ubbfc\uad6d\uc5d0\uc11c \ub300\ud45c\uc801\uc778 \uc0ac\ud68c\uc8fc\uc758\uc801 \uc5ec\uc131\uc8fc\uc758\uc790\ub294 \ud5c8\uc815\uc219\uc774\ub2e4. \uc0ac\ud68c\uc8fc\uc758\uc801 \uc5ec\uc131\uc8fc\uc758\uc790\ub4e4\uc740 \uc774\ub7ec\ud55c \uc790\ubcf8\uc8fc\uc758\uc801 \uad6c\uc870\ub3c4 \ube44\ud310\ud558\uc9c0\ub9cc, \ub0a8\uc131\uc5d0\uac8c \uc758\uc9c0\ud558\uc5ec \uc548\uc77c\ud558\uac8c \uc0ac\ub824\ub294 \uc790\ubcf8\uc8fc\uc758\uc801 \uc5ec\uc131\ub4e4 \ub610\ud55c \ube44\ud310\ud588\ub2e4.", "paragraph_answer": "\uc5ec\uc131\uc8fc\uc758 \uad00\ub828 \ubd84\ud30c\uc911\uc5d0\uc11c\ub294 \uc0ac\ud68c\uc8fc\uc758\uc801 \uc5ec\uc131\uc8fc\uc758\uc790, \uc0dd\ud0dc\uc5ec\uc131\uc8fc\uc758\uc790 \ub4e4\uc740 \ubc18\uc790\ubcf8\uc8fc\uc758\uc801 \uc131\ud5a5\uc774 \uac15\ud588\ub2e4. \uc790\ubcf8\uc8fc\uc758\ub294 \uc778\uac04\uc758 \uc9d0\uc2b9\uacfc \uac19\uc740 \uc695\ub9dd\uc744 \ucda9\uc871\uc2dc\ud0a4\uae30\uc704\ud574 \uc5ec\uc131\uc758 \uc2e0\uccb4\ub97c \uc0c1\ud488\ud654\ud588\ub2e4\ub294 \uc810\uc5d0\uc11c \ud070 \ube44\ud310\uc744 \ud588\uc73c\uba70, \ub0a8\uc131\uc758 \uc695\uc2ec\uacfc \uc790\ubcf8\uc8fc\uc758\uc758 \uc870\ud654\ub85c \uc804\uc7c1\uc774 \ud0c4\uc0dd\ud588\ub2e4\uace0 \uc8fc\uc7a5\ud588\ub2e4. \ub300\ud45c\uc801\uc73c\ub85c \uc0ac\ud68c\uc8fc\uc758\uc801 \uc5ec\uc131\uc8fc\uc758\uc5d0\uc11c \ubc18\uc790\ubcf8\uc8fc\uc758\ub294 \uc9c4\uc815\ud55c \uc758\ubbf8\uc758 \uc5ec\uc131 \ud574\ubc29\uc774 \uc77c\uc5b4\ub098\uae30 \uc704\ud574\uc11c\ub294 \uc5ec\uc131\ub3c4 \ub0a8\uc131\uacfc \uac19\uc740 \uacbd\uc81c\uc801 \ub2a5\ub825\uc744 \uac16\ucd94\ub294\ub370\uc11c \ube44\ub86f\ub41c\ub2e4\uace0 \ud558\uc600\ub2e4. \uc989, \uc790\ubcf8\uc8fc\uc758 \uccb4\uc81c \uc548\uc5d0\uc11c \uc5ec\uc131\uc758 \uacbd\uc81c\uc801 \ub2a5\ub825\uc774 \ud55c\uc815\ub418\uc5b4\uc788\uae30 \ub54c\ubb38\uc5d0 \uc790\ubcf8\uc8fc\uc758 \ub0b4\uc5d0\uc11c \uc5ec\uc131\uc740 \ub0a8\uc131\uc5d0 \ube44\ud574 \ub4a4\ub5a8\uc5b4\uc9c8 \uc218 \ubc16\uc5d0 \uc5c6\ub294 \uc874\uc7ac\ub85c \uc5ec\uaca8\uc9c8 \uc218 \ubc16\uc5d0 \uc5c6\ub294 \uccb4\uc81c\ub85c \uc778\uc2dd\ud558\uc600\ub2e4. \uadf8 \ub54c\ubb38\uc5d0 \uc0ac\ud68c\uc8fc\uc758\uc801 \uc5ec\uc131\uc8fc\uc758\uc5d0\uc11c \uc5ec\uc131\uc740 \ubaa8\ub450\uac00 \ud3c9\ub4f1\ud558\uac8c \uc77c\ud558\uace0 \ubc8c \uc218 \uc788\ub294 \uc0ac\ud68c\uc8fc\uc758\uc801 \uacbd\uc81c \uad6c\uc870\ub97c \uc9c0\uc9c0\ud574\uc57c\ud55c\ub2e4\uace0 \ud638\uc18c\ud588\ub2e4. \ub300\ud55c\ubbfc\uad6d\uc5d0\uc11c \ub300\ud45c\uc801\uc778 \uc0ac\ud68c\uc8fc\uc758\uc801 \uc5ec\uc131\uc8fc\uc758\uc790\ub294 \ud5c8\uc815\uc219\uc774\ub2e4. \uc0ac\ud68c\uc8fc\uc758\uc801 \uc5ec\uc131\uc8fc\uc758\uc790\ub4e4\uc740 \uc774\ub7ec\ud55c \uc790\ubcf8\uc8fc\uc758\uc801 \uad6c\uc870\ub3c4 \ube44\ud310\ud558\uc9c0\ub9cc, \ub0a8\uc131\uc5d0\uac8c \uc758\uc9c0\ud558\uc5ec \uc548\uc77c\ud558\uac8c \uc0ac\ub824\ub294 \uc790\ubcf8\uc8fc\uc758\uc801 \uc5ec\uc131\ub4e4 \ub610\ud55c \ube44\ud310\ud588\ub2e4.", "sentence_answer": "\uc5ec\uc131\uc8fc\uc758 \uad00\ub828 \ubd84\ud30c\uc911\uc5d0\uc11c\ub294 \uc0ac\ud68c\uc8fc\uc758\uc801 \uc5ec\uc131\uc8fc\uc758\uc790, \uc0dd\ud0dc\uc5ec\uc131\uc8fc\uc758\uc790 \ub4e4\uc740 \ubc18\uc790\ubcf8\uc8fc\uc758\uc801 \uc131\ud5a5\uc774 \uac15\ud588\ub2e4.", "paragraph_id": "6530258-1-0", "paragraph_question": "question: \ubc18\uc790\ubcf8\uc8fc\uc758\uc801 \uc131\ud5a5\uc774 \uac15\ud55c \uc5ec\uc131\uc8fc\uc758 \uad00\ub828 \ubd84\ud30c\ub294 \ubb34\uc5c7\uc778\uac00?, context: \uc5ec\uc131\uc8fc\uc758 \uad00\ub828 \ubd84\ud30c\uc911\uc5d0\uc11c\ub294 \uc0ac\ud68c\uc8fc\uc758\uc801 \uc5ec\uc131\uc8fc\uc758\uc790, \uc0dd\ud0dc\uc5ec\uc131\uc8fc\uc758\uc790\ub4e4\uc740 \ubc18\uc790\ubcf8\uc8fc\uc758\uc801 \uc131\ud5a5\uc774 \uac15\ud588\ub2e4. \uc790\ubcf8\uc8fc\uc758\ub294 \uc778\uac04\uc758 \uc9d0\uc2b9\uacfc \uac19\uc740 \uc695\ub9dd\uc744 \ucda9\uc871\uc2dc\ud0a4\uae30\uc704\ud574 \uc5ec\uc131\uc758 \uc2e0\uccb4\ub97c \uc0c1\ud488\ud654\ud588\ub2e4\ub294 \uc810\uc5d0\uc11c \ud070 \ube44\ud310\uc744 \ud588\uc73c\uba70, \ub0a8\uc131\uc758 \uc695\uc2ec\uacfc \uc790\ubcf8\uc8fc\uc758\uc758 \uc870\ud654\ub85c \uc804\uc7c1\uc774 \ud0c4\uc0dd\ud588\ub2e4\uace0 \uc8fc\uc7a5\ud588\ub2e4. \ub300\ud45c\uc801\uc73c\ub85c \uc0ac\ud68c\uc8fc\uc758\uc801 \uc5ec\uc131\uc8fc\uc758\uc5d0\uc11c \ubc18\uc790\ubcf8\uc8fc\uc758\ub294 \uc9c4\uc815\ud55c \uc758\ubbf8\uc758 \uc5ec\uc131 \ud574\ubc29\uc774 \uc77c\uc5b4\ub098\uae30 \uc704\ud574\uc11c\ub294 \uc5ec\uc131\ub3c4 \ub0a8\uc131\uacfc \uac19\uc740 \uacbd\uc81c\uc801 \ub2a5\ub825\uc744 \uac16\ucd94\ub294\ub370\uc11c \ube44\ub86f\ub41c\ub2e4\uace0 \ud558\uc600\ub2e4. \uc989, \uc790\ubcf8\uc8fc\uc758 \uccb4\uc81c \uc548\uc5d0\uc11c \uc5ec\uc131\uc758 \uacbd\uc81c\uc801 \ub2a5\ub825\uc774 \ud55c\uc815\ub418\uc5b4\uc788\uae30 \ub54c\ubb38\uc5d0 \uc790\ubcf8\uc8fc\uc758 \ub0b4\uc5d0\uc11c \uc5ec\uc131\uc740 \ub0a8\uc131\uc5d0 \ube44\ud574 \ub4a4\ub5a8\uc5b4\uc9c8 \uc218 \ubc16\uc5d0 \uc5c6\ub294 \uc874\uc7ac\ub85c \uc5ec\uaca8\uc9c8 \uc218 \ubc16\uc5d0 \uc5c6\ub294 \uccb4\uc81c\ub85c \uc778\uc2dd\ud558\uc600\ub2e4. \uadf8 \ub54c\ubb38\uc5d0 \uc0ac\ud68c\uc8fc\uc758\uc801 \uc5ec\uc131\uc8fc\uc758\uc5d0\uc11c \uc5ec\uc131\uc740 \ubaa8\ub450\uac00 \ud3c9\ub4f1\ud558\uac8c \uc77c\ud558\uace0 \ubc8c \uc218 \uc788\ub294 \uc0ac\ud68c\uc8fc\uc758\uc801 \uacbd\uc81c \uad6c\uc870\ub97c \uc9c0\uc9c0\ud574\uc57c\ud55c\ub2e4\uace0 \ud638\uc18c\ud588\ub2e4. \ub300\ud55c\ubbfc\uad6d\uc5d0\uc11c \ub300\ud45c\uc801\uc778 \uc0ac\ud68c\uc8fc\uc758\uc801 \uc5ec\uc131\uc8fc\uc758\uc790\ub294 \ud5c8\uc815\uc219\uc774\ub2e4. \uc0ac\ud68c\uc8fc\uc758\uc801 \uc5ec\uc131\uc8fc\uc758\uc790\ub4e4\uc740 \uc774\ub7ec\ud55c \uc790\ubcf8\uc8fc\uc758\uc801 \uad6c\uc870\ub3c4 \ube44\ud310\ud558\uc9c0\ub9cc, \ub0a8\uc131\uc5d0\uac8c \uc758\uc9c0\ud558\uc5ec \uc548\uc77c\ud558\uac8c \uc0ac\ub824\ub294 \uc790\ubcf8\uc8fc\uc758\uc801 \uc5ec\uc131\ub4e4 \ub610\ud55c \ube44\ud310\ud588\ub2e4."} {"question": "\uc870\uc120\uc778\ubbfc\uad70\uc774 \ub300\ud55c\ubbfc\uad6d\uc744 \uae30\uc2b5 \ub0a8\uce68\ud55c \uac83\uc740 \uc5b8\uc81c\uc778\uac00?", "paragraph": "\ubd81\ud55c\uc758 \uae40\uc77c\uc131\uc758 \uba85\ub839 \ud558\uc5d0, \ub300\ud55c\ubbfc\uad6d\uc774 \uacf5\uaca9\ud574 \uc654\uc73c\ubbc0\ub85c \ubd80\ub4dd\uc774 \ubc18\uaca9\ud55c\ub2e4\ub294 \ud5c8\uc704 \uc120\uc804\uc744 \ud558\uba74\uc11c, \uc18c\ube44\uc5d0\ud2b8 \uc5f0\ubc29\uc5d0\uc11c \uc9c0\uc6d0\ud55c \uc804\ucc28\ub97c \uc55e\uc138\uc6b4 \uc870\uc120\uc778\ubbfc\uad70\uc774 1950\ub144 6\uc6d4 25\uc77c \uc0c8\ubcbd 4\uc2dc \uacbd, 38\ub3c4\uc120\uacfc \ub3d9\ud574\uc548 \uc5f0\uc120(\u6cbf\u7dda) \ub4f1 11\uac1c\uc18c\uc5d0\uc11c \uacbd\uacc4\ub97c \ub118\uc5b4 \ub300\ud55c\ubbfc\uad6d\uc744 \uae30\uc2b5 \ub0a8\uce68\ud558\uc600\ub2e4. \ub300\ud55c\ubbfc\uad6d\uc740 \uc218\uc801\uc73c\ub85c \uc5f4\uc138\uc778\ub370\ub2e4 \uc7a5\ube44\ub3c4 \ubd80\uc871\ud588\ub2e4. \uc624\uc804 9\uc2dc\uacbd\uc5d0\ub294 \uac1c\uc131\ubc29\uc5b4\uc120\uc744 \uaca9\ud30c\ud558\uace0 \ub2f9\uc77c \uc624\uc804\uc5d0 \ub3d9\ub450\ucc9c\uacfc \ud3ec\ucc9c\uc744 \ud568\ub77d\uc2dc\ucf30\ub2e4. 26\uc77c \uc624\ud6c4\uc5d0 \uc758\uc815\ubd80\ub97c, 27\uc77c \uc815\uc624\uc5d0\ub294 \uc774\ubbf8 \uc11c\uc6b8 \ub3c4\ubd09\uad6c\uc758 \ucc3d\ub3d9\ubc29\uc5b4\uc120\uc744 \ub118\uc5c8\ub2e4. \ucc3d\ub3d9\ubc29\uc5b4\uc120\uc774 \ub6ab\ub9b0 \ub300\ud55c\ubbfc\uad6d \uad6d\uad70\uc740 \ubbf8\uc544\ub9ac\ubc29\uc5b4\uc120\uc744 \uad6c\ucd95\ud558\uc600\uc73c\ub098 \uc870\uc120\uc778\ubbfc\uad70\uc758 \uc804\ucc28\uc5d0 \uc758\ud574 \ubd95\uad34\ub418\uc5c8\ub2e4. 28\uc77c \uc0c8\ubcbd\uc5d0\ub294 \uc11c\uc6b8 \uc2dc\ub0b4\uac00 \uc810\ub839\ub418\uace0, \uc624\uc804 2\uc2dc 30\ubd84\uc5d0 \ubbf8\uad6d \uacf5\uad70 \ud56d\uacf5\uae30\uac00 \ud55c\uac15\uc5d0 \uc788\ub358 \ud55c\uac15 \ub300\uad50\ub97c \uc2b5\uaca9\ud574 \u201c\ub2f9\uc2dc \ubbf8 \uacf5\uad70\uc774 \uc0ac\uc6a9\ud558\ub358 \ud56d\uacf5\uae30 \ud22c\ud558\uc6a9 \u2018AN-M64\u2019 \ubbf8\uc81c \ud3ed\ud0c4\u201d\uc73c\ub85c \ud55c\uac15 \ub300\uad50\uc640 \ud55c\uac15 \ucca0\uad50\ub97c \ud3ed\ud30c\uc2dc\ucf30 \uace0, \uc774\ud6c4 \uc11c\uc6b8\uc5d0\ub294 \uacf5\uc0b0\uad70\uc774 \uc8fc\ub454\ud558\uac8c \ub418\uc5c8\ub2e4.", "answer": "1950\ub144 6\uc6d4 25\uc77c", "sentence": "\ubd81\ud55c\uc758 \uae40\uc77c\uc131\uc758 \uba85\ub839 \ud558\uc5d0, \ub300\ud55c\ubbfc\uad6d\uc774 \uacf5\uaca9\ud574 \uc654\uc73c\ubbc0\ub85c \ubd80\ub4dd\uc774 \ubc18\uaca9\ud55c\ub2e4\ub294 \ud5c8\uc704 \uc120\uc804\uc744 \ud558\uba74\uc11c, \uc18c\ube44\uc5d0\ud2b8 \uc5f0\ubc29\uc5d0\uc11c \uc9c0\uc6d0\ud55c \uc804\ucc28\ub97c \uc55e\uc138\uc6b4 \uc870\uc120\uc778\ubbfc\uad70\uc774 1950\ub144 6\uc6d4 25\uc77c \uc0c8\ubcbd 4\uc2dc \uacbd, 38\ub3c4\uc120\uacfc \ub3d9\ud574\uc548 \uc5f0\uc120(\u6cbf\u7dda) \ub4f1 11\uac1c\uc18c\uc5d0\uc11c \uacbd\uacc4\ub97c \ub118\uc5b4 \ub300\ud55c\ubbfc\uad6d\uc744 \uae30\uc2b5 \ub0a8\uce68\ud558\uc600\ub2e4.", "paragraph_sentence": " \ubd81\ud55c\uc758 \uae40\uc77c\uc131\uc758 \uba85\ub839 \ud558\uc5d0, \ub300\ud55c\ubbfc\uad6d\uc774 \uacf5\uaca9\ud574 \uc654\uc73c\ubbc0\ub85c \ubd80\ub4dd\uc774 \ubc18\uaca9\ud55c\ub2e4\ub294 \ud5c8\uc704 \uc120\uc804\uc744 \ud558\uba74\uc11c, \uc18c\ube44\uc5d0\ud2b8 \uc5f0\ubc29\uc5d0\uc11c \uc9c0\uc6d0\ud55c \uc804\ucc28\ub97c \uc55e\uc138\uc6b4 \uc870\uc120\uc778\ubbfc\uad70\uc774 1950\ub144 6\uc6d4 25\uc77c \uc0c8\ubcbd 4\uc2dc \uacbd, 38\ub3c4\uc120\uacfc \ub3d9\ud574\uc548 \uc5f0\uc120(\u6cbf\u7dda) \ub4f1 11\uac1c\uc18c\uc5d0\uc11c \uacbd\uacc4\ub97c \ub118\uc5b4 \ub300\ud55c\ubbfc\uad6d\uc744 \uae30\uc2b5 \ub0a8\uce68\ud558\uc600\ub2e4. \ub300\ud55c\ubbfc\uad6d\uc740 \uc218\uc801\uc73c\ub85c \uc5f4\uc138\uc778\ub370\ub2e4 \uc7a5\ube44\ub3c4 \ubd80\uc871\ud588\ub2e4. \uc624\uc804 9\uc2dc\uacbd\uc5d0\ub294 \uac1c\uc131\ubc29\uc5b4\uc120\uc744 \uaca9\ud30c\ud558\uace0 \ub2f9\uc77c \uc624\uc804\uc5d0 \ub3d9\ub450\ucc9c\uacfc \ud3ec\ucc9c\uc744 \ud568\ub77d\uc2dc\ucf30\ub2e4. 26\uc77c \uc624\ud6c4\uc5d0 \uc758\uc815\ubd80\ub97c, 27\uc77c \uc815\uc624\uc5d0\ub294 \uc774\ubbf8 \uc11c\uc6b8 \ub3c4\ubd09\uad6c\uc758 \ucc3d\ub3d9\ubc29\uc5b4\uc120\uc744 \ub118\uc5c8\ub2e4. \ucc3d\ub3d9\ubc29\uc5b4\uc120\uc774 \ub6ab\ub9b0 \ub300\ud55c\ubbfc\uad6d \uad6d\uad70\uc740 \ubbf8\uc544\ub9ac\ubc29\uc5b4\uc120\uc744 \uad6c\ucd95\ud558\uc600\uc73c\ub098 \uc870\uc120\uc778\ubbfc\uad70\uc758 \uc804\ucc28\uc5d0 \uc758\ud574 \ubd95\uad34\ub418\uc5c8\ub2e4. 28\uc77c \uc0c8\ubcbd\uc5d0\ub294 \uc11c\uc6b8 \uc2dc\ub0b4\uac00 \uc810\ub839\ub418\uace0, \uc624\uc804 2\uc2dc 30\ubd84\uc5d0 \ubbf8\uad6d \uacf5\uad70 \ud56d\uacf5\uae30\uac00 \ud55c\uac15\uc5d0 \uc788\ub358 \ud55c\uac15 \ub300\uad50\ub97c \uc2b5\uaca9\ud574 \u201c\ub2f9\uc2dc \ubbf8 \uacf5\uad70\uc774 \uc0ac\uc6a9\ud558\ub358 \ud56d\uacf5\uae30 \ud22c\ud558\uc6a9 \u2018AN-M64\u2019 \ubbf8\uc81c \ud3ed\ud0c4\u201d\uc73c\ub85c \ud55c\uac15 \ub300\uad50\uc640 \ud55c\uac15 \ucca0\uad50\ub97c \ud3ed\ud30c\uc2dc\ucf30 \uace0, \uc774\ud6c4 \uc11c\uc6b8\uc5d0\ub294 \uacf5\uc0b0\uad70\uc774 \uc8fc\ub454\ud558\uac8c \ub418\uc5c8\ub2e4.", "paragraph_answer": "\ubd81\ud55c\uc758 \uae40\uc77c\uc131\uc758 \uba85\ub839 \ud558\uc5d0, \ub300\ud55c\ubbfc\uad6d\uc774 \uacf5\uaca9\ud574 \uc654\uc73c\ubbc0\ub85c \ubd80\ub4dd\uc774 \ubc18\uaca9\ud55c\ub2e4\ub294 \ud5c8\uc704 \uc120\uc804\uc744 \ud558\uba74\uc11c, \uc18c\ube44\uc5d0\ud2b8 \uc5f0\ubc29\uc5d0\uc11c \uc9c0\uc6d0\ud55c \uc804\ucc28\ub97c \uc55e\uc138\uc6b4 \uc870\uc120\uc778\ubbfc\uad70\uc774 1950\ub144 6\uc6d4 25\uc77c \uc0c8\ubcbd 4\uc2dc \uacbd, 38\ub3c4\uc120\uacfc \ub3d9\ud574\uc548 \uc5f0\uc120(\u6cbf\u7dda) \ub4f1 11\uac1c\uc18c\uc5d0\uc11c \uacbd\uacc4\ub97c \ub118\uc5b4 \ub300\ud55c\ubbfc\uad6d\uc744 \uae30\uc2b5 \ub0a8\uce68\ud558\uc600\ub2e4. \ub300\ud55c\ubbfc\uad6d\uc740 \uc218\uc801\uc73c\ub85c \uc5f4\uc138\uc778\ub370\ub2e4 \uc7a5\ube44\ub3c4 \ubd80\uc871\ud588\ub2e4. \uc624\uc804 9\uc2dc\uacbd\uc5d0\ub294 \uac1c\uc131\ubc29\uc5b4\uc120\uc744 \uaca9\ud30c\ud558\uace0 \ub2f9\uc77c \uc624\uc804\uc5d0 \ub3d9\ub450\ucc9c\uacfc \ud3ec\ucc9c\uc744 \ud568\ub77d\uc2dc\ucf30\ub2e4. 26\uc77c \uc624\ud6c4\uc5d0 \uc758\uc815\ubd80\ub97c, 27\uc77c \uc815\uc624\uc5d0\ub294 \uc774\ubbf8 \uc11c\uc6b8 \ub3c4\ubd09\uad6c\uc758 \ucc3d\ub3d9\ubc29\uc5b4\uc120\uc744 \ub118\uc5c8\ub2e4. \ucc3d\ub3d9\ubc29\uc5b4\uc120\uc774 \ub6ab\ub9b0 \ub300\ud55c\ubbfc\uad6d \uad6d\uad70\uc740 \ubbf8\uc544\ub9ac\ubc29\uc5b4\uc120\uc744 \uad6c\ucd95\ud558\uc600\uc73c\ub098 \uc870\uc120\uc778\ubbfc\uad70\uc758 \uc804\ucc28\uc5d0 \uc758\ud574 \ubd95\uad34\ub418\uc5c8\ub2e4. 28\uc77c \uc0c8\ubcbd\uc5d0\ub294 \uc11c\uc6b8 \uc2dc\ub0b4\uac00 \uc810\ub839\ub418\uace0, \uc624\uc804 2\uc2dc 30\ubd84\uc5d0 \ubbf8\uad6d \uacf5\uad70 \ud56d\uacf5\uae30\uac00 \ud55c\uac15\uc5d0 \uc788\ub358 \ud55c\uac15 \ub300\uad50\ub97c \uc2b5\uaca9\ud574 \u201c\ub2f9\uc2dc \ubbf8 \uacf5\uad70\uc774 \uc0ac\uc6a9\ud558\ub358 \ud56d\uacf5\uae30 \ud22c\ud558\uc6a9 \u2018AN-M64\u2019 \ubbf8\uc81c \ud3ed\ud0c4\u201d\uc73c\ub85c \ud55c\uac15 \ub300\uad50\uc640 \ud55c\uac15 \ucca0\uad50\ub97c \ud3ed\ud30c\uc2dc\ucf30 \uace0, \uc774\ud6c4 \uc11c\uc6b8\uc5d0\ub294 \uacf5\uc0b0\uad70\uc774 \uc8fc\ub454\ud558\uac8c \ub418\uc5c8\ub2e4.", "sentence_answer": "\ubd81\ud55c\uc758 \uae40\uc77c\uc131\uc758 \uba85\ub839 \ud558\uc5d0, \ub300\ud55c\ubbfc\uad6d\uc774 \uacf5\uaca9\ud574 \uc654\uc73c\ubbc0\ub85c \ubd80\ub4dd\uc774 \ubc18\uaca9\ud55c\ub2e4\ub294 \ud5c8\uc704 \uc120\uc804\uc744 \ud558\uba74\uc11c, \uc18c\ube44\uc5d0\ud2b8 \uc5f0\ubc29\uc5d0\uc11c \uc9c0\uc6d0\ud55c \uc804\ucc28\ub97c \uc55e\uc138\uc6b4 \uc870\uc120\uc778\ubbfc\uad70\uc774 1950\ub144 6\uc6d4 25\uc77c \uc0c8\ubcbd 4\uc2dc \uacbd, 38\ub3c4\uc120\uacfc \ub3d9\ud574\uc548 \uc5f0\uc120(\u6cbf\u7dda) \ub4f1 11\uac1c\uc18c\uc5d0\uc11c \uacbd\uacc4\ub97c \ub118\uc5b4 \ub300\ud55c\ubbfc\uad6d\uc744 \uae30\uc2b5 \ub0a8\uce68\ud558\uc600\ub2e4.", "paragraph_id": "6569867-24-0", "paragraph_question": "question: \uc870\uc120\uc778\ubbfc\uad70\uc774 \ub300\ud55c\ubbfc\uad6d\uc744 \uae30\uc2b5 \ub0a8\uce68\ud55c \uac83\uc740 \uc5b8\uc81c\uc778\uac00?, context: \ubd81\ud55c\uc758 \uae40\uc77c\uc131\uc758 \uba85\ub839 \ud558\uc5d0, \ub300\ud55c\ubbfc\uad6d\uc774 \uacf5\uaca9\ud574 \uc654\uc73c\ubbc0\ub85c \ubd80\ub4dd\uc774 \ubc18\uaca9\ud55c\ub2e4\ub294 \ud5c8\uc704 \uc120\uc804\uc744 \ud558\uba74\uc11c, \uc18c\ube44\uc5d0\ud2b8 \uc5f0\ubc29\uc5d0\uc11c \uc9c0\uc6d0\ud55c \uc804\ucc28\ub97c \uc55e\uc138\uc6b4 \uc870\uc120\uc778\ubbfc\uad70\uc774 1950\ub144 6\uc6d4 25\uc77c \uc0c8\ubcbd 4\uc2dc \uacbd, 38\ub3c4\uc120\uacfc \ub3d9\ud574\uc548 \uc5f0\uc120(\u6cbf\u7dda) \ub4f1 11\uac1c\uc18c\uc5d0\uc11c \uacbd\uacc4\ub97c \ub118\uc5b4 \ub300\ud55c\ubbfc\uad6d\uc744 \uae30\uc2b5 \ub0a8\uce68\ud558\uc600\ub2e4. \ub300\ud55c\ubbfc\uad6d\uc740 \uc218\uc801\uc73c\ub85c \uc5f4\uc138\uc778\ub370\ub2e4 \uc7a5\ube44\ub3c4 \ubd80\uc871\ud588\ub2e4. \uc624\uc804 9\uc2dc\uacbd\uc5d0\ub294 \uac1c\uc131\ubc29\uc5b4\uc120\uc744 \uaca9\ud30c\ud558\uace0 \ub2f9\uc77c \uc624\uc804\uc5d0 \ub3d9\ub450\ucc9c\uacfc \ud3ec\ucc9c\uc744 \ud568\ub77d\uc2dc\ucf30\ub2e4. 26\uc77c \uc624\ud6c4\uc5d0 \uc758\uc815\ubd80\ub97c, 27\uc77c \uc815\uc624\uc5d0\ub294 \uc774\ubbf8 \uc11c\uc6b8 \ub3c4\ubd09\uad6c\uc758 \ucc3d\ub3d9\ubc29\uc5b4\uc120\uc744 \ub118\uc5c8\ub2e4. \ucc3d\ub3d9\ubc29\uc5b4\uc120\uc774 \ub6ab\ub9b0 \ub300\ud55c\ubbfc\uad6d \uad6d\uad70\uc740 \ubbf8\uc544\ub9ac\ubc29\uc5b4\uc120\uc744 \uad6c\ucd95\ud558\uc600\uc73c\ub098 \uc870\uc120\uc778\ubbfc\uad70\uc758 \uc804\ucc28\uc5d0 \uc758\ud574 \ubd95\uad34\ub418\uc5c8\ub2e4. 28\uc77c \uc0c8\ubcbd\uc5d0\ub294 \uc11c\uc6b8 \uc2dc\ub0b4\uac00 \uc810\ub839\ub418\uace0, \uc624\uc804 2\uc2dc 30\ubd84\uc5d0 \ubbf8\uad6d \uacf5\uad70 \ud56d\uacf5\uae30\uac00 \ud55c\uac15\uc5d0 \uc788\ub358 \ud55c\uac15 \ub300\uad50\ub97c \uc2b5\uaca9\ud574 \u201c\ub2f9\uc2dc \ubbf8 \uacf5\uad70\uc774 \uc0ac\uc6a9\ud558\ub358 \ud56d\uacf5\uae30 \ud22c\ud558\uc6a9 \u2018AN-M64\u2019 \ubbf8\uc81c \ud3ed\ud0c4\u201d\uc73c\ub85c \ud55c\uac15 \ub300\uad50\uc640 \ud55c\uac15 \ucca0\uad50\ub97c \ud3ed\ud30c\uc2dc\ucf30 \uace0, \uc774\ud6c4 \uc11c\uc6b8\uc5d0\ub294 \uacf5\uc0b0\uad70\uc774 \uc8fc\ub454\ud558\uac8c \ub418\uc5c8\ub2e4."} {"question": "\ube0c\uc774 \ud3ec \ubca4\ub370\ud0c0\uc5d0\uc11c \uad6d\ud68c\uc758\uc0ac\ub2f9\uc744 \ud30c\uad34\ud558\ub824\uace0 \ud55c \uacc4\ud68d\uc758 \ubaa8\ud2f0\ube0c\uac00 \ub41c \uc0ac\uac74\uc740 \ubb34\uc5c7\uc778\uac00?", "paragraph": "\uc774 \uc601\ud654\ub294 \uc601\uad6d\uc5d0\uc11c \ubc18\ub780\uc744 \uc720\ubc1c\uc2dc\ud0a4\uae30 \uc704\ud574 \uad6d\ud68c\uc758\uc0ac\ub2f9\uc744 \ud30c\uad34\ud558\ub824\ub294 \uacc4\ud68d\uc744 \uc138\uc6e0\ub358 \uac00\ud1a8\ub9ad\uad50\ub3c4\ub4e4\uc774 1605\ub144\uc5d0 \uacf5\ubaa8\ud588\ub358 \ud654\uc57d \uc74c\ubaa8 \uc0ac\uac74\uc758 \uc774\ubbf8\uc9c0\ub97c \ub2e4\uc18c \ucc44\uc6a9\ud588\ub294\ub370, \uc601\ud654\uc758 \ud0dc\uadf8\ub77c\uc778\uc740 \ud654\uc57d \uc74c\ubaa8 \uc0ac\uac74\uc5d0 \ub300\ud55c \uc601\uad6d\uc758 \uc804\ud1b5\uc801\uc778 \uae30\ub150\uc0ac \u201c\uae30\uc5b5\ud558\ub77c, \uae30\uc5b5\ud558\ub77c, 11\uc6d4\uc758 5\uc77c\uc744 \uae30\uc5b5\ud558\ub77c\u201d \uc774\uace0, \uc601\ud654 \uac1c\ubd09\uc77c\ub3c4 \uc6d0\ub798 2005\ub144 11\uc6d4 5\uc77c, \ud654\uc57d \uc74c\ubaa8 \uc0ac\uac74 400\uc8fc\ub144 \uae30\ub150\uc77c\ub85c \uacc4\ud68d\ub418\uc5c8\uc5c8\ub2e4. \uadf8\ub7ec\ub098 \uac1c\ubd09\uc77c\uc774 \uc5f0\uae30\ub418\uba74\uc11c \uc774\ub7f0 \ub9c8\ucf00\ud305\uc740 \uadf8 \uac00\uce58\ub97c \ub2e4\uc18c \uc783\uc5b4\ubc84\ub838\ub2e4. \ub9ce\uc740 \uc0ac\ub78c\ub4e4\uc740 \uac1c\ubd09 \uc5f0\uae30\uac00 \ub7f0\ub358 \uc9c0\ud558\ucca0 \ud3ed\ud0c4 \ud22c\ucc99 \uc0ac\uac74 \ub54c\ubb38\uc774\ub77c\uace0 \ucd94\uce21\ud588\ub2e4. \ud558\uc9c0\ub9cc \uc601\ud654 \uc81c\uc791\uc790\ub4e4\uc740 \uc774 \uc18c\ubb38\uc744 \ubd80\uc778\ud588\uace0, \uac1c\ubd09 \uc5f0\uae30\ub294 \uc2dc\uac01 \ud6a8\uacfc\uc5d0 \uc2dc\uac04\uc774 \ub354 \ud544\uc694\ud588\uae30 \ub54c\ubb38\uc774\ub77c\uace0 \ud588\ub2e4.. \u300a\ube0c\uc774 \ud3ec \ubca4\ub370\ud0c0\u300b\ub294 2\uc6d4 13\uc77c, \ubca0\ub97c\ub9b0 \uc601\ud654\uc81c\uc5d0\uc11c \ucc98\uc74c\uc73c\ub85c \ud2b9\ubcc4 \uc0c1\uc601\uc744 \ud588\ub2e4 \uc77c\ubc18 \uac1c\ubd09\uc740 2006\ub144 3\uc6d4 17\uc77c\uc774\uc5c8\uace0, \ubbf8\uad6d\uacfc \uc601\uad6d, \uadf8\ub9ac\uace0 \uadf8 \uc678 6\uac1c\uad6d\uc758 3,365\uac1c \uadf9\uc7a5\uc5d0\uc11c \uc0c1\uc601\ud588\ub2e4.", "answer": "1605\ub144\uc5d0 \uacf5\ubaa8\ud588\ub358 \ud654\uc57d \uc74c\ubaa8 \uc0ac\uac74", "sentence": "\uc774 \uc601\ud654\ub294 \uc601\uad6d\uc5d0\uc11c \ubc18\ub780\uc744 \uc720\ubc1c\uc2dc\ud0a4\uae30 \uc704\ud574 \uad6d\ud68c\uc758\uc0ac\ub2f9\uc744 \ud30c\uad34\ud558\ub824\ub294 \uacc4\ud68d\uc744 \uc138\uc6e0\ub358 \uac00\ud1a8\ub9ad\uad50\ub3c4\ub4e4\uc774 1605\ub144\uc5d0 \uacf5\ubaa8\ud588\ub358 \ud654\uc57d \uc74c\ubaa8 \uc0ac\uac74 \uc758 \uc774\ubbf8\uc9c0\ub97c \ub2e4\uc18c \ucc44\uc6a9\ud588\ub294\ub370, \uc601\ud654\uc758 \ud0dc\uadf8\ub77c\uc778\uc740 \ud654\uc57d \uc74c\ubaa8 \uc0ac\uac74\uc5d0 \ub300\ud55c \uc601\uad6d\uc758 \uc804\ud1b5\uc801\uc778 \uae30\ub150\uc0ac \u201c\uae30\uc5b5\ud558\ub77c, \uae30\uc5b5\ud558\ub77c, 11\uc6d4\uc758 5\uc77c\uc744 \uae30\uc5b5\ud558\ub77c\u201d \uc774\uace0, \uc601\ud654 \uac1c\ubd09\uc77c\ub3c4 \uc6d0\ub798 2005\ub144 11\uc6d4 5\uc77c, \ud654\uc57d \uc74c\ubaa8 \uc0ac\uac74 400\uc8fc\ub144 \uae30\ub150\uc77c\ub85c \uacc4\ud68d\ub418\uc5c8\uc5c8\ub2e4.", "paragraph_sentence": " \uc774 \uc601\ud654\ub294 \uc601\uad6d\uc5d0\uc11c \ubc18\ub780\uc744 \uc720\ubc1c\uc2dc\ud0a4\uae30 \uc704\ud574 \uad6d\ud68c\uc758\uc0ac\ub2f9\uc744 \ud30c\uad34\ud558\ub824\ub294 \uacc4\ud68d\uc744 \uc138\uc6e0\ub358 \uac00\ud1a8\ub9ad\uad50\ub3c4\ub4e4\uc774 1605\ub144\uc5d0 \uacf5\ubaa8\ud588\ub358 \ud654\uc57d \uc74c\ubaa8 \uc0ac\uac74 \uc758 \uc774\ubbf8\uc9c0\ub97c \ub2e4\uc18c \ucc44\uc6a9\ud588\ub294\ub370, \uc601\ud654\uc758 \ud0dc\uadf8\ub77c\uc778\uc740 \ud654\uc57d \uc74c\ubaa8 \uc0ac\uac74\uc5d0 \ub300\ud55c \uc601\uad6d\uc758 \uc804\ud1b5\uc801\uc778 \uae30\ub150\uc0ac \u201c\uae30\uc5b5\ud558\ub77c, \uae30\uc5b5\ud558\ub77c, 11\uc6d4\uc758 5\uc77c\uc744 \uae30\uc5b5\ud558\ub77c\u201d \uc774\uace0, \uc601\ud654 \uac1c\ubd09\uc77c\ub3c4 \uc6d0\ub798 2005\ub144 11\uc6d4 5\uc77c, \ud654\uc57d \uc74c\ubaa8 \uc0ac\uac74 400\uc8fc\ub144 \uae30\ub150\uc77c\ub85c \uacc4\ud68d\ub418\uc5c8\uc5c8\ub2e4. \uadf8\ub7ec\ub098 \uac1c\ubd09\uc77c\uc774 \uc5f0\uae30\ub418\uba74\uc11c \uc774\ub7f0 \ub9c8\ucf00\ud305\uc740 \uadf8 \uac00\uce58\ub97c \ub2e4\uc18c \uc783\uc5b4\ubc84\ub838\ub2e4. \ub9ce\uc740 \uc0ac\ub78c\ub4e4\uc740 \uac1c\ubd09 \uc5f0\uae30\uac00 \ub7f0\ub358 \uc9c0\ud558\ucca0 \ud3ed\ud0c4 \ud22c\ucc99 \uc0ac\uac74 \ub54c\ubb38\uc774\ub77c\uace0 \ucd94\uce21\ud588\ub2e4. \ud558\uc9c0\ub9cc \uc601\ud654 \uc81c\uc791\uc790\ub4e4\uc740 \uc774 \uc18c\ubb38\uc744 \ubd80\uc778\ud588\uace0, \uac1c\ubd09 \uc5f0\uae30\ub294 \uc2dc\uac01 \ud6a8\uacfc\uc5d0 \uc2dc\uac04\uc774 \ub354 \ud544\uc694\ud588\uae30 \ub54c\ubb38\uc774\ub77c\uace0 \ud588\ub2e4.. \u300a\ube0c\uc774 \ud3ec \ubca4\ub370\ud0c0\u300b\ub294 2\uc6d4 13\uc77c, \ubca0\ub97c\ub9b0 \uc601\ud654\uc81c\uc5d0\uc11c \ucc98\uc74c\uc73c\ub85c \ud2b9\ubcc4 \uc0c1\uc601\uc744 \ud588\ub2e4 \uc77c\ubc18 \uac1c\ubd09\uc740 2006\ub144 3\uc6d4 17\uc77c\uc774\uc5c8\uace0, \ubbf8\uad6d\uacfc \uc601\uad6d, \uadf8\ub9ac\uace0 \uadf8 \uc678 6\uac1c\uad6d\uc758 3,365\uac1c \uadf9\uc7a5\uc5d0\uc11c \uc0c1\uc601\ud588\ub2e4.", "paragraph_answer": "\uc774 \uc601\ud654\ub294 \uc601\uad6d\uc5d0\uc11c \ubc18\ub780\uc744 \uc720\ubc1c\uc2dc\ud0a4\uae30 \uc704\ud574 \uad6d\ud68c\uc758\uc0ac\ub2f9\uc744 \ud30c\uad34\ud558\ub824\ub294 \uacc4\ud68d\uc744 \uc138\uc6e0\ub358 \uac00\ud1a8\ub9ad\uad50\ub3c4\ub4e4\uc774 1605\ub144\uc5d0 \uacf5\ubaa8\ud588\ub358 \ud654\uc57d \uc74c\ubaa8 \uc0ac\uac74 \uc758 \uc774\ubbf8\uc9c0\ub97c \ub2e4\uc18c \ucc44\uc6a9\ud588\ub294\ub370, \uc601\ud654\uc758 \ud0dc\uadf8\ub77c\uc778\uc740 \ud654\uc57d \uc74c\ubaa8 \uc0ac\uac74\uc5d0 \ub300\ud55c \uc601\uad6d\uc758 \uc804\ud1b5\uc801\uc778 \uae30\ub150\uc0ac \u201c\uae30\uc5b5\ud558\ub77c, \uae30\uc5b5\ud558\ub77c, 11\uc6d4\uc758 5\uc77c\uc744 \uae30\uc5b5\ud558\ub77c\u201d \uc774\uace0, \uc601\ud654 \uac1c\ubd09\uc77c\ub3c4 \uc6d0\ub798 2005\ub144 11\uc6d4 5\uc77c, \ud654\uc57d \uc74c\ubaa8 \uc0ac\uac74 400\uc8fc\ub144 \uae30\ub150\uc77c\ub85c \uacc4\ud68d\ub418\uc5c8\uc5c8\ub2e4. \uadf8\ub7ec\ub098 \uac1c\ubd09\uc77c\uc774 \uc5f0\uae30\ub418\uba74\uc11c \uc774\ub7f0 \ub9c8\ucf00\ud305\uc740 \uadf8 \uac00\uce58\ub97c \ub2e4\uc18c \uc783\uc5b4\ubc84\ub838\ub2e4. \ub9ce\uc740 \uc0ac\ub78c\ub4e4\uc740 \uac1c\ubd09 \uc5f0\uae30\uac00 \ub7f0\ub358 \uc9c0\ud558\ucca0 \ud3ed\ud0c4 \ud22c\ucc99 \uc0ac\uac74 \ub54c\ubb38\uc774\ub77c\uace0 \ucd94\uce21\ud588\ub2e4. \ud558\uc9c0\ub9cc \uc601\ud654 \uc81c\uc791\uc790\ub4e4\uc740 \uc774 \uc18c\ubb38\uc744 \ubd80\uc778\ud588\uace0, \uac1c\ubd09 \uc5f0\uae30\ub294 \uc2dc\uac01 \ud6a8\uacfc\uc5d0 \uc2dc\uac04\uc774 \ub354 \ud544\uc694\ud588\uae30 \ub54c\ubb38\uc774\ub77c\uace0 \ud588\ub2e4.. \u300a\ube0c\uc774 \ud3ec \ubca4\ub370\ud0c0\u300b\ub294 2\uc6d4 13\uc77c, \ubca0\ub97c\ub9b0 \uc601\ud654\uc81c\uc5d0\uc11c \ucc98\uc74c\uc73c\ub85c \ud2b9\ubcc4 \uc0c1\uc601\uc744 \ud588\ub2e4 \uc77c\ubc18 \uac1c\ubd09\uc740 2006\ub144 3\uc6d4 17\uc77c\uc774\uc5c8\uace0, \ubbf8\uad6d\uacfc \uc601\uad6d, \uadf8\ub9ac\uace0 \uadf8 \uc678 6\uac1c\uad6d\uc758 3,365\uac1c \uadf9\uc7a5\uc5d0\uc11c \uc0c1\uc601\ud588\ub2e4.", "sentence_answer": "\uc774 \uc601\ud654\ub294 \uc601\uad6d\uc5d0\uc11c \ubc18\ub780\uc744 \uc720\ubc1c\uc2dc\ud0a4\uae30 \uc704\ud574 \uad6d\ud68c\uc758\uc0ac\ub2f9\uc744 \ud30c\uad34\ud558\ub824\ub294 \uacc4\ud68d\uc744 \uc138\uc6e0\ub358 \uac00\ud1a8\ub9ad\uad50\ub3c4\ub4e4\uc774 1605\ub144\uc5d0 \uacf5\ubaa8\ud588\ub358 \ud654\uc57d \uc74c\ubaa8 \uc0ac\uac74 \uc758 \uc774\ubbf8\uc9c0\ub97c \ub2e4\uc18c \ucc44\uc6a9\ud588\ub294\ub370, \uc601\ud654\uc758 \ud0dc\uadf8\ub77c\uc778\uc740 \ud654\uc57d \uc74c\ubaa8 \uc0ac\uac74\uc5d0 \ub300\ud55c \uc601\uad6d\uc758 \uc804\ud1b5\uc801\uc778 \uae30\ub150\uc0ac \u201c\uae30\uc5b5\ud558\ub77c, \uae30\uc5b5\ud558\ub77c, 11\uc6d4\uc758 5\uc77c\uc744 \uae30\uc5b5\ud558\ub77c\u201d \uc774\uace0, \uc601\ud654 \uac1c\ubd09\uc77c\ub3c4 \uc6d0\ub798 2005\ub144 11\uc6d4 5\uc77c, \ud654\uc57d \uc74c\ubaa8 \uc0ac\uac74 400\uc8fc\ub144 \uae30\ub150\uc77c\ub85c \uacc4\ud68d\ub418\uc5c8\uc5c8\ub2e4.", "paragraph_id": "6462898-9-0", "paragraph_question": "question: \ube0c\uc774 \ud3ec \ubca4\ub370\ud0c0\uc5d0\uc11c \uad6d\ud68c\uc758\uc0ac\ub2f9\uc744 \ud30c\uad34\ud558\ub824\uace0 \ud55c \uacc4\ud68d\uc758 \ubaa8\ud2f0\ube0c\uac00 \ub41c \uc0ac\uac74\uc740 \ubb34\uc5c7\uc778\uac00?, context: \uc774 \uc601\ud654\ub294 \uc601\uad6d\uc5d0\uc11c \ubc18\ub780\uc744 \uc720\ubc1c\uc2dc\ud0a4\uae30 \uc704\ud574 \uad6d\ud68c\uc758\uc0ac\ub2f9\uc744 \ud30c\uad34\ud558\ub824\ub294 \uacc4\ud68d\uc744 \uc138\uc6e0\ub358 \uac00\ud1a8\ub9ad\uad50\ub3c4\ub4e4\uc774 1605\ub144\uc5d0 \uacf5\ubaa8\ud588\ub358 \ud654\uc57d \uc74c\ubaa8 \uc0ac\uac74\uc758 \uc774\ubbf8\uc9c0\ub97c \ub2e4\uc18c \ucc44\uc6a9\ud588\ub294\ub370, \uc601\ud654\uc758 \ud0dc\uadf8\ub77c\uc778\uc740 \ud654\uc57d \uc74c\ubaa8 \uc0ac\uac74\uc5d0 \ub300\ud55c \uc601\uad6d\uc758 \uc804\ud1b5\uc801\uc778 \uae30\ub150\uc0ac \u201c\uae30\uc5b5\ud558\ub77c, \uae30\uc5b5\ud558\ub77c, 11\uc6d4\uc758 5\uc77c\uc744 \uae30\uc5b5\ud558\ub77c\u201d \uc774\uace0, \uc601\ud654 \uac1c\ubd09\uc77c\ub3c4 \uc6d0\ub798 2005\ub144 11\uc6d4 5\uc77c, \ud654\uc57d \uc74c\ubaa8 \uc0ac\uac74 400\uc8fc\ub144 \uae30\ub150\uc77c\ub85c \uacc4\ud68d\ub418\uc5c8\uc5c8\ub2e4. \uadf8\ub7ec\ub098 \uac1c\ubd09\uc77c\uc774 \uc5f0\uae30\ub418\uba74\uc11c \uc774\ub7f0 \ub9c8\ucf00\ud305\uc740 \uadf8 \uac00\uce58\ub97c \ub2e4\uc18c \uc783\uc5b4\ubc84\ub838\ub2e4. \ub9ce\uc740 \uc0ac\ub78c\ub4e4\uc740 \uac1c\ubd09 \uc5f0\uae30\uac00 \ub7f0\ub358 \uc9c0\ud558\ucca0 \ud3ed\ud0c4 \ud22c\ucc99 \uc0ac\uac74 \ub54c\ubb38\uc774\ub77c\uace0 \ucd94\uce21\ud588\ub2e4. \ud558\uc9c0\ub9cc \uc601\ud654 \uc81c\uc791\uc790\ub4e4\uc740 \uc774 \uc18c\ubb38\uc744 \ubd80\uc778\ud588\uace0, \uac1c\ubd09 \uc5f0\uae30\ub294 \uc2dc\uac01 \ud6a8\uacfc\uc5d0 \uc2dc\uac04\uc774 \ub354 \ud544\uc694\ud588\uae30 \ub54c\ubb38\uc774\ub77c\uace0 \ud588\ub2e4.. \u300a\ube0c\uc774 \ud3ec \ubca4\ub370\ud0c0\u300b\ub294 2\uc6d4 13\uc77c, \ubca0\ub97c\ub9b0 \uc601\ud654\uc81c\uc5d0\uc11c \ucc98\uc74c\uc73c\ub85c \ud2b9\ubcc4 \uc0c1\uc601\uc744 \ud588\ub2e4 \uc77c\ubc18 \uac1c\ubd09\uc740 2006\ub144 3\uc6d4 17\uc77c\uc774\uc5c8\uace0, \ubbf8\uad6d\uacfc \uc601\uad6d, \uadf8\ub9ac\uace0 \uadf8 \uc678 6\uac1c\uad6d\uc758 3,365\uac1c \uadf9\uc7a5\uc5d0\uc11c \uc0c1\uc601\ud588\ub2e4."} {"question": "\ub780\uc2ac\ub86f\uc744 \uc544\ub4e4\ucc98\ub7fc \uad50\uc721\uc2dc\ud0a8 \uc694\uc815\uc740?", "paragraph": "\uc774 \ud638\uc218\ub294 \ud638\uc218\uc758 \ubd80\uc778\uc758 \uac70\ucc98\uc600\uace0, \ub9c8\uc220\uc0ac, \ub9c8\ubc95\uc0ac\uc758 \ub098\ub77c, \ub9c8\ubc95\uc774 \uac78\ub9b0 \uc544\ubc1c\ub871(Avalon) \uc12c\uc73c\ub85c \ud5a5\ud558\ub294 \uac00\uad50\uc600\ub2e4. \ube44\ube44\uc548 \uc694\uc815\uc740 \ub780\uc2ac\ub86f\uc744 \ubc14\ub2f7 \uc18d \uadf8\uc758 \uad81\uc804\uc73c\ub85c \ub370\ub824\uc628 \uc7a5\ubcf8\uc774\uae30 \ub54c\ubb38\uc5d0, \uadf8\ub97c \uc544\ub4e4\ucc98\ub7fc \uad50\uc721\uc2dc\ucf30\ub2e4. \uc774 \ub54c\ubd80\ud130 \ub780\uc2ac\ub86f \ub4c0 \ub77d((Lancelot du lac, \uc8fc : \ud638\uc218\uc758 \ub780\uc2ac\ub86f)\uc774\ub77c\ub294 \uadf8\uc758 \ubcc4\uba85\uc774 \ubd99\uac8c \ub418\uc5c8\ub2e4. \uc694\uc815\uc73c\ub85c\ubd80\ud130 '\uc655\uc758 \uc544\ub4e4' \ub610\ub294 '\uc544\ub984\ub2e4\uc6b4 \ubc1c\uacac'\uc73c\ub85c \ubd88\ub9b0 \uadf8\ub294 \uadf8\uc758 \ubaa8\ub4e0 \ubfcc\ub9ac\ub97c \uc78a\uac8c \ub418\uc5c8\ub2e4. \uadf8\uc758 \uc80a\uc740 \uc2dc\uc808 \ub3d9\uc548 \uadf8\ub140\ub294 \uadf8\ub97c \uc644\ubcbd\ud55c \uae30\uc0ac\ub85c \uad50\uc721\uc2dc\ucf30\uace0, \uc0ac\ub0e5, \uc74c\uc545, \uacb0\ud22c, \uad81\uc815 \uc608\uc808, \uadc0\uc871\uc758 \uace0\uacb0\ud55c \uc815\uc2e0 \ub4f1\uc744 \uac00\ub974\ucce4\ub2e4. \uadf8\ub294 \uc544\ub354\uc655, \uadc0\ub124\ube44\uc5b4(Gueni\u00e8vre)\uc5ec\uc655\uc744 \ud5a5\ud55c \ucda9\uc131\uc2ec\uc73c\ub85c \ub9e4\uc6b0 \ub9dd\uba85\uc774 \ub192\uc558\uc73c\ub098, \ubd88\ud0c0\uc624\ub974\ub294 \uc0ac\ub791\uc73c\ub85c \uc778\ud558\uc5ec \uc5ec\uc655\uc744 \ubc94\ud558\ub294 \uc2e4\uc218\ub97c \ud558\uc600\uace0, \uc774\ub294 \ucc28\ud6c4 \uc544\ub354\uc655\uc73c\ub85c \ubd80\ud130 \uc6a9\uc11c \ubc1b\uc558\ub2e4. \uadf8\ub7ec\ub098 \uadf8\ub4e4\uc758 \ubd80\uc801\uc808\ud55c \uad00\uacc4\uac00 \uc544\ub354\uc758 \uc655\uad6d\uacfc \uce74\uba5c\ub86f\uc758 \ubd95\uad34\ub97c \ucd08\ub798\ud55c \uc6d0\uc778 \uc911 \ud558\ub098\uac00 \ub418\uc5c8\ub2e4.", "answer": "\ube44\ube44\uc548", "sentence": "\ube44\ube44\uc548 \uc694\uc815\uc740 \ub780\uc2ac\ub86f\uc744 \ubc14\ub2f7 \uc18d \uadf8\uc758 \uad81\uc804\uc73c\ub85c \ub370\ub824\uc628 \uc7a5\ubcf8\uc774\uae30 \ub54c\ubb38\uc5d0, \uadf8\ub97c \uc544\ub4e4\ucc98\ub7fc \uad50\uc721\uc2dc\ucf30\ub2e4.", "paragraph_sentence": "\uc774 \ud638\uc218\ub294 \ud638\uc218\uc758 \ubd80\uc778\uc758 \uac70\ucc98\uc600\uace0, \ub9c8\uc220\uc0ac, \ub9c8\ubc95\uc0ac\uc758 \ub098\ub77c, \ub9c8\ubc95\uc774 \uac78\ub9b0 \uc544\ubc1c\ub871(Avalon) \uc12c\uc73c\ub85c \ud5a5\ud558\ub294 \uac00\uad50\uc600\ub2e4. \ube44\ube44\uc548 \uc694\uc815\uc740 \ub780\uc2ac\ub86f\uc744 \ubc14\ub2f7 \uc18d \uadf8\uc758 \uad81\uc804\uc73c\ub85c \ub370\ub824\uc628 \uc7a5\ubcf8\uc774\uae30 \ub54c\ubb38\uc5d0, \uadf8\ub97c \uc544\ub4e4\ucc98\ub7fc \uad50\uc721\uc2dc\ucf30\ub2e4. \uc774 \ub54c\ubd80\ud130 \ub780\uc2ac\ub86f \ub4c0 \ub77d((Lancelot du lac, \uc8fc : \ud638\uc218\uc758 \ub780\uc2ac\ub86f)\uc774\ub77c\ub294 \uadf8\uc758 \ubcc4\uba85\uc774 \ubd99\uac8c \ub418\uc5c8\ub2e4. \uc694\uc815\uc73c\ub85c\ubd80\ud130 '\uc655\uc758 \uc544\ub4e4' \ub610\ub294 '\uc544\ub984\ub2e4\uc6b4 \ubc1c\uacac'\uc73c\ub85c \ubd88\ub9b0 \uadf8\ub294 \uadf8\uc758 \ubaa8\ub4e0 \ubfcc\ub9ac\ub97c \uc78a\uac8c \ub418\uc5c8\ub2e4. \uadf8\uc758 \uc80a\uc740 \uc2dc\uc808 \ub3d9\uc548 \uadf8\ub140\ub294 \uadf8\ub97c \uc644\ubcbd\ud55c \uae30\uc0ac\ub85c \uad50\uc721\uc2dc\ucf30\uace0, \uc0ac\ub0e5, \uc74c\uc545, \uacb0\ud22c, \uad81\uc815 \uc608\uc808, \uadc0\uc871\uc758 \uace0\uacb0\ud55c \uc815\uc2e0 \ub4f1\uc744 \uac00\ub974\ucce4\ub2e4. \uadf8\ub294 \uc544\ub354\uc655, \uadc0\ub124\ube44\uc5b4(Gueni\u00e8vre)\uc5ec\uc655\uc744 \ud5a5\ud55c \ucda9\uc131\uc2ec\uc73c\ub85c \ub9e4\uc6b0 \ub9dd\uba85\uc774 \ub192\uc558\uc73c\ub098, \ubd88\ud0c0\uc624\ub974\ub294 \uc0ac\ub791\uc73c\ub85c \uc778\ud558\uc5ec \uc5ec\uc655\uc744 \ubc94\ud558\ub294 \uc2e4\uc218\ub97c \ud558\uc600\uace0, \uc774\ub294 \ucc28\ud6c4 \uc544\ub354\uc655\uc73c\ub85c \ubd80\ud130 \uc6a9\uc11c \ubc1b\uc558\ub2e4. \uadf8\ub7ec\ub098 \uadf8\ub4e4\uc758 \ubd80\uc801\uc808\ud55c \uad00\uacc4\uac00 \uc544\ub354\uc758 \uc655\uad6d\uacfc \uce74\uba5c\ub86f\uc758 \ubd95\uad34\ub97c \ucd08\ub798\ud55c \uc6d0\uc778 \uc911 \ud558\ub098\uac00 \ub418\uc5c8\ub2e4.", "paragraph_answer": "\uc774 \ud638\uc218\ub294 \ud638\uc218\uc758 \ubd80\uc778\uc758 \uac70\ucc98\uc600\uace0, \ub9c8\uc220\uc0ac, \ub9c8\ubc95\uc0ac\uc758 \ub098\ub77c, \ub9c8\ubc95\uc774 \uac78\ub9b0 \uc544\ubc1c\ub871(Avalon) \uc12c\uc73c\ub85c \ud5a5\ud558\ub294 \uac00\uad50\uc600\ub2e4. \ube44\ube44\uc548 \uc694\uc815\uc740 \ub780\uc2ac\ub86f\uc744 \ubc14\ub2f7 \uc18d \uadf8\uc758 \uad81\uc804\uc73c\ub85c \ub370\ub824\uc628 \uc7a5\ubcf8\uc774\uae30 \ub54c\ubb38\uc5d0, \uadf8\ub97c \uc544\ub4e4\ucc98\ub7fc \uad50\uc721\uc2dc\ucf30\ub2e4. \uc774 \ub54c\ubd80\ud130 \ub780\uc2ac\ub86f \ub4c0 \ub77d((Lancelot du lac, \uc8fc : \ud638\uc218\uc758 \ub780\uc2ac\ub86f)\uc774\ub77c\ub294 \uadf8\uc758 \ubcc4\uba85\uc774 \ubd99\uac8c \ub418\uc5c8\ub2e4. \uc694\uc815\uc73c\ub85c\ubd80\ud130 '\uc655\uc758 \uc544\ub4e4' \ub610\ub294 '\uc544\ub984\ub2e4\uc6b4 \ubc1c\uacac'\uc73c\ub85c \ubd88\ub9b0 \uadf8\ub294 \uadf8\uc758 \ubaa8\ub4e0 \ubfcc\ub9ac\ub97c \uc78a\uac8c \ub418\uc5c8\ub2e4. \uadf8\uc758 \uc80a\uc740 \uc2dc\uc808 \ub3d9\uc548 \uadf8\ub140\ub294 \uadf8\ub97c \uc644\ubcbd\ud55c \uae30\uc0ac\ub85c \uad50\uc721\uc2dc\ucf30\uace0, \uc0ac\ub0e5, \uc74c\uc545, \uacb0\ud22c, \uad81\uc815 \uc608\uc808, \uadc0\uc871\uc758 \uace0\uacb0\ud55c \uc815\uc2e0 \ub4f1\uc744 \uac00\ub974\ucce4\ub2e4. \uadf8\ub294 \uc544\ub354\uc655, \uadc0\ub124\ube44\uc5b4(Gueni\u00e8vre)\uc5ec\uc655\uc744 \ud5a5\ud55c \ucda9\uc131\uc2ec\uc73c\ub85c \ub9e4\uc6b0 \ub9dd\uba85\uc774 \ub192\uc558\uc73c\ub098, \ubd88\ud0c0\uc624\ub974\ub294 \uc0ac\ub791\uc73c\ub85c \uc778\ud558\uc5ec \uc5ec\uc655\uc744 \ubc94\ud558\ub294 \uc2e4\uc218\ub97c \ud558\uc600\uace0, \uc774\ub294 \ucc28\ud6c4 \uc544\ub354\uc655\uc73c\ub85c \ubd80\ud130 \uc6a9\uc11c \ubc1b\uc558\ub2e4. \uadf8\ub7ec\ub098 \uadf8\ub4e4\uc758 \ubd80\uc801\uc808\ud55c \uad00\uacc4\uac00 \uc544\ub354\uc758 \uc655\uad6d\uacfc \uce74\uba5c\ub86f\uc758 \ubd95\uad34\ub97c \ucd08\ub798\ud55c \uc6d0\uc778 \uc911 \ud558\ub098\uac00 \ub418\uc5c8\ub2e4.", "sentence_answer": " \ube44\ube44\uc548 \uc694\uc815\uc740 \ub780\uc2ac\ub86f\uc744 \ubc14\ub2f7 \uc18d \uadf8\uc758 \uad81\uc804\uc73c\ub85c \ub370\ub824\uc628 \uc7a5\ubcf8\uc774\uae30 \ub54c\ubb38\uc5d0, \uadf8\ub97c \uc544\ub4e4\ucc98\ub7fc \uad50\uc721\uc2dc\ucf30\ub2e4.", "paragraph_id": "6052892-3-0", "paragraph_question": "question: \ub780\uc2ac\ub86f\uc744 \uc544\ub4e4\ucc98\ub7fc \uad50\uc721\uc2dc\ud0a8 \uc694\uc815\uc740?, context: \uc774 \ud638\uc218\ub294 \ud638\uc218\uc758 \ubd80\uc778\uc758 \uac70\ucc98\uc600\uace0, \ub9c8\uc220\uc0ac, \ub9c8\ubc95\uc0ac\uc758 \ub098\ub77c, \ub9c8\ubc95\uc774 \uac78\ub9b0 \uc544\ubc1c\ub871(Avalon) \uc12c\uc73c\ub85c \ud5a5\ud558\ub294 \uac00\uad50\uc600\ub2e4. \ube44\ube44\uc548 \uc694\uc815\uc740 \ub780\uc2ac\ub86f\uc744 \ubc14\ub2f7 \uc18d \uadf8\uc758 \uad81\uc804\uc73c\ub85c \ub370\ub824\uc628 \uc7a5\ubcf8\uc774\uae30 \ub54c\ubb38\uc5d0, \uadf8\ub97c \uc544\ub4e4\ucc98\ub7fc \uad50\uc721\uc2dc\ucf30\ub2e4. \uc774 \ub54c\ubd80\ud130 \ub780\uc2ac\ub86f \ub4c0 \ub77d((Lancelot du lac, \uc8fc : \ud638\uc218\uc758 \ub780\uc2ac\ub86f)\uc774\ub77c\ub294 \uadf8\uc758 \ubcc4\uba85\uc774 \ubd99\uac8c \ub418\uc5c8\ub2e4. \uc694\uc815\uc73c\ub85c\ubd80\ud130 '\uc655\uc758 \uc544\ub4e4' \ub610\ub294 '\uc544\ub984\ub2e4\uc6b4 \ubc1c\uacac'\uc73c\ub85c \ubd88\ub9b0 \uadf8\ub294 \uadf8\uc758 \ubaa8\ub4e0 \ubfcc\ub9ac\ub97c \uc78a\uac8c \ub418\uc5c8\ub2e4. \uadf8\uc758 \uc80a\uc740 \uc2dc\uc808 \ub3d9\uc548 \uadf8\ub140\ub294 \uadf8\ub97c \uc644\ubcbd\ud55c \uae30\uc0ac\ub85c \uad50\uc721\uc2dc\ucf30\uace0, \uc0ac\ub0e5, \uc74c\uc545, \uacb0\ud22c, \uad81\uc815 \uc608\uc808, \uadc0\uc871\uc758 \uace0\uacb0\ud55c \uc815\uc2e0 \ub4f1\uc744 \uac00\ub974\ucce4\ub2e4. \uadf8\ub294 \uc544\ub354\uc655, \uadc0\ub124\ube44\uc5b4(Gueni\u00e8vre)\uc5ec\uc655\uc744 \ud5a5\ud55c \ucda9\uc131\uc2ec\uc73c\ub85c \ub9e4\uc6b0 \ub9dd\uba85\uc774 \ub192\uc558\uc73c\ub098, \ubd88\ud0c0\uc624\ub974\ub294 \uc0ac\ub791\uc73c\ub85c \uc778\ud558\uc5ec \uc5ec\uc655\uc744 \ubc94\ud558\ub294 \uc2e4\uc218\ub97c \ud558\uc600\uace0, \uc774\ub294 \ucc28\ud6c4 \uc544\ub354\uc655\uc73c\ub85c \ubd80\ud130 \uc6a9\uc11c \ubc1b\uc558\ub2e4. \uadf8\ub7ec\ub098 \uadf8\ub4e4\uc758 \ubd80\uc801\uc808\ud55c \uad00\uacc4\uac00 \uc544\ub354\uc758 \uc655\uad6d\uacfc \uce74\uba5c\ub86f\uc758 \ubd95\uad34\ub97c \ucd08\ub798\ud55c \uc6d0\uc778 \uc911 \ud558\ub098\uac00 \ub418\uc5c8\ub2e4."} {"question": "\ubb34\uc5c7\uc73c\ub85c\uc778\ud574 AF\uc18d\ub3c4\uac00 \ube68\ub77c\uc84c\ub294\uac00", "paragraph": "\ud6c4\uba74 \uce74\uba54\ub77c\ub294 OIS \uae30\uc220\uc774 \uc801\uc6a9\ub41c \uce74\uba54\ub77c \ubaa8\ub4c8\uc5d0 \uc0bc\uc131\uc804\uc790 \uc2dc\uc2a4\ud15c LSI \uc0ac\uc5c5\ubd80 \uc544\uc774\uc18c\uc140 S5K2L1 \uc13c\uc11c\uc640 \uc18c\ub2c8 \uc5d1\uc2a4\ubaa8\uc5b4 IMX260 \uc13c\uc11c\ub97c \ud63c\uc6a9\ud558\uc5ec 1,200\ub9cc \ud654\uc18c \uce74\uba54\ub77c\ub97c \ud0d1\uc7ac\ud588\ub2e4. \uadf8\ub9ac\uace0 AF \ud2b8\ub798\ud0b9\uc744 \uc9c0\uc6d0\ud558\uace0 \"Dual Pixel\" \uae30\uc220\uc744 \ud65c\uc6a9\ud55c \uc704\uc0c1\ucc28 \uac80\ucd9c AF\ub97c \uc9c0\uc6d0\ud55c\ub2e4. \uc774\ub85c \uc778\ud574 AF \uc18d\ub3c4\uac00 \ube68\ub77c\uc84c\ub2e4\uace0 \ud55c\ub2e4. \ub610\ud55c, \uc13c\uc11c \ud06c\uae30\ub294 1/2.5\uc778\uce58\uc774\uba70 \uc870\ub9ac\uac1c \ubc1d\uae30\ub294 F/1.7\uc774\ub2e4. \ucd5c\ub300 \ud654\uc18c\uc218\ub294 \uc804\uc791\uc778 \uac24\ub7ed\uc2dc \ub178\ud2b85\ubcf4\ub2e4 \uc904\uc5b4\ub4e4\uc5c8\uc73c\ub098 \uc13c\uc11c \ud06c\uae30\uac00 1/2.6\uc778\uce58\uc5d0\uc11c \ucee4\uc84c\uace0 \uc870\ub9ac\uac1c \ubc1d\uae30\ub3c4 F/1.9\uc5d0\uc11c \ubc1d\uc544\uc84c\uc73c\uba70 \uc13c\uc11c \ube44\uc728 \uc5ed\uc2dc 16:9 \ube44\uc728\uc5d0\uc11c 4:3 \ube44\uc728\ub85c \ubc14\ub00c\uc5b4, \uce74\uba54\ub77c \ubaa8\ub4c8\uc774 \ub3cc\ucd9c\ub418\ub294 \uae30\ub2a5\ub3c4 \ub300\ud3ed \uac1c\uc120\ub418\uace0 \uc13c\uc11c \uba74\uc801\ub3c4 \uc804\uc791\uc778 \uac24\ub7ed\uc2dc \ub178\ud2b85\ubcf4\ub2e4 \uc57d 25% \uac00\ub7c9 \ucee4\uc84c\ub2e4. \uc804\uba74 \uce74\uba54\ub77c\ub294 \uc0bc\uc131\uc804\uc790 \uc2dc\uc2a4\ud15c LSI \uc0ac\uc5c5\ubd80 \uc544\uc774\uc18c\uc140 S5K4E6 \uc13c\uc11c\uc758 500\ub9cc \ud654\uc18c \uce74\uba54\ub77c\ub97c \ud0d1\uc7ac\ud588\ub2e4. \uc5ec\uae30\uc5d0 \ub514\uc2a4\ud50c\ub808\uc774\uac00 \ud50c\ub798\uc2dc \uc5ed\ud560\uc744 \ud558\ub294 '\uc140\ud53c \ud50c\ub798\uc2dc' \uae30\ub2a5\uc774 \ud0d1\uc7ac\ub418\uc5c8\ub2e4. \ub610\ud55c, \uc870\ub9ac\uac1c \ubc1d\uae30\ub294 \uc804\uc791\uc778 \uac24\ub7ed\uc2dc \ub178\ud2b85\uc758 \uc870\ub9ac\uac1c \ubc1d\uae30\uc778 F/1.9\ubcf4\ub2e4 \ubc1d\uc544\uc9c4 F/1.7\uc774\ub2e4.", "answer": "Dual Pixel", "sentence": "\uadf8\ub9ac\uace0 AF \ud2b8\ub798\ud0b9\uc744 \uc9c0\uc6d0\ud558\uace0 \" Dual Pixel \"", "paragraph_sentence": "\ud6c4\uba74 \uce74\uba54\ub77c\ub294 OIS \uae30\uc220\uc774 \uc801\uc6a9\ub41c \uce74\uba54\ub77c \ubaa8\ub4c8\uc5d0 \uc0bc\uc131\uc804\uc790 \uc2dc\uc2a4\ud15c LSI \uc0ac\uc5c5\ubd80 \uc544\uc774\uc18c\uc140 S5K2L1 \uc13c\uc11c\uc640 \uc18c\ub2c8 \uc5d1\uc2a4\ubaa8\uc5b4 IMX260 \uc13c\uc11c\ub97c \ud63c\uc6a9\ud558\uc5ec 1,200\ub9cc \ud654\uc18c \uce74\uba54\ub77c\ub97c \ud0d1\uc7ac\ud588\ub2e4. \uadf8\ub9ac\uace0 AF \ud2b8\ub798\ud0b9\uc744 \uc9c0\uc6d0\ud558\uace0 \" Dual Pixel \" \uae30\uc220\uc744 \ud65c\uc6a9\ud55c \uc704\uc0c1\ucc28 \uac80\ucd9c AF\ub97c \uc9c0\uc6d0\ud55c\ub2e4. \uc774\ub85c \uc778\ud574 AF \uc18d\ub3c4\uac00 \ube68\ub77c\uc84c\ub2e4\uace0 \ud55c\ub2e4. \ub610\ud55c, \uc13c\uc11c \ud06c\uae30\ub294 1/2.5\uc778\uce58\uc774\uba70 \uc870\ub9ac\uac1c \ubc1d\uae30\ub294 F/1.7\uc774\ub2e4. \ucd5c\ub300 \ud654\uc18c\uc218\ub294 \uc804\uc791\uc778 \uac24\ub7ed\uc2dc \ub178\ud2b85\ubcf4\ub2e4 \uc904\uc5b4\ub4e4\uc5c8\uc73c\ub098 \uc13c\uc11c \ud06c\uae30\uac00 1/2.6\uc778\uce58\uc5d0\uc11c \ucee4\uc84c\uace0 \uc870\ub9ac\uac1c \ubc1d\uae30\ub3c4 F/1.9\uc5d0\uc11c \ubc1d\uc544\uc84c\uc73c\uba70 \uc13c\uc11c \ube44\uc728 \uc5ed\uc2dc 16:9 \ube44\uc728\uc5d0\uc11c 4:3 \ube44\uc728\ub85c \ubc14\ub00c\uc5b4, \uce74\uba54\ub77c \ubaa8\ub4c8\uc774 \ub3cc\ucd9c\ub418\ub294 \uae30\ub2a5\ub3c4 \ub300\ud3ed \uac1c\uc120\ub418\uace0 \uc13c\uc11c \uba74\uc801\ub3c4 \uc804\uc791\uc778 \uac24\ub7ed\uc2dc \ub178\ud2b85\ubcf4\ub2e4 \uc57d 25% \uac00\ub7c9 \ucee4\uc84c\ub2e4. \uc804\uba74 \uce74\uba54\ub77c\ub294 \uc0bc\uc131\uc804\uc790 \uc2dc\uc2a4\ud15c LSI \uc0ac\uc5c5\ubd80 \uc544\uc774\uc18c\uc140 S5K4E6 \uc13c\uc11c\uc758 500\ub9cc \ud654\uc18c \uce74\uba54\ub77c\ub97c \ud0d1\uc7ac\ud588\ub2e4. \uc5ec\uae30\uc5d0 \ub514\uc2a4\ud50c\ub808\uc774\uac00 \ud50c\ub798\uc2dc \uc5ed\ud560\uc744 \ud558\ub294 '\uc140\ud53c \ud50c\ub798\uc2dc' \uae30\ub2a5\uc774 \ud0d1\uc7ac\ub418\uc5c8\ub2e4. \ub610\ud55c, \uc870\ub9ac\uac1c \ubc1d\uae30\ub294 \uc804\uc791\uc778 \uac24\ub7ed\uc2dc \ub178\ud2b85\uc758 \uc870\ub9ac\uac1c \ubc1d\uae30\uc778 F/1.9\ubcf4\ub2e4 \ubc1d\uc544\uc9c4 F/1.7\uc774\ub2e4.", "paragraph_answer": "\ud6c4\uba74 \uce74\uba54\ub77c\ub294 OIS \uae30\uc220\uc774 \uc801\uc6a9\ub41c \uce74\uba54\ub77c \ubaa8\ub4c8\uc5d0 \uc0bc\uc131\uc804\uc790 \uc2dc\uc2a4\ud15c LSI \uc0ac\uc5c5\ubd80 \uc544\uc774\uc18c\uc140 S5K2L1 \uc13c\uc11c\uc640 \uc18c\ub2c8 \uc5d1\uc2a4\ubaa8\uc5b4 IMX260 \uc13c\uc11c\ub97c \ud63c\uc6a9\ud558\uc5ec 1,200\ub9cc \ud654\uc18c \uce74\uba54\ub77c\ub97c \ud0d1\uc7ac\ud588\ub2e4. \uadf8\ub9ac\uace0 AF \ud2b8\ub798\ud0b9\uc744 \uc9c0\uc6d0\ud558\uace0 \" Dual Pixel \" \uae30\uc220\uc744 \ud65c\uc6a9\ud55c \uc704\uc0c1\ucc28 \uac80\ucd9c AF\ub97c \uc9c0\uc6d0\ud55c\ub2e4. \uc774\ub85c \uc778\ud574 AF \uc18d\ub3c4\uac00 \ube68\ub77c\uc84c\ub2e4\uace0 \ud55c\ub2e4. \ub610\ud55c, \uc13c\uc11c \ud06c\uae30\ub294 1/2.5\uc778\uce58\uc774\uba70 \uc870\ub9ac\uac1c \ubc1d\uae30\ub294 F/1.7\uc774\ub2e4. \ucd5c\ub300 \ud654\uc18c\uc218\ub294 \uc804\uc791\uc778 \uac24\ub7ed\uc2dc \ub178\ud2b85\ubcf4\ub2e4 \uc904\uc5b4\ub4e4\uc5c8\uc73c\ub098 \uc13c\uc11c \ud06c\uae30\uac00 1/2.6\uc778\uce58\uc5d0\uc11c \ucee4\uc84c\uace0 \uc870\ub9ac\uac1c \ubc1d\uae30\ub3c4 F/1.9\uc5d0\uc11c \ubc1d\uc544\uc84c\uc73c\uba70 \uc13c\uc11c \ube44\uc728 \uc5ed\uc2dc 16:9 \ube44\uc728\uc5d0\uc11c 4:3 \ube44\uc728\ub85c \ubc14\ub00c\uc5b4, \uce74\uba54\ub77c \ubaa8\ub4c8\uc774 \ub3cc\ucd9c\ub418\ub294 \uae30\ub2a5\ub3c4 \ub300\ud3ed \uac1c\uc120\ub418\uace0 \uc13c\uc11c \uba74\uc801\ub3c4 \uc804\uc791\uc778 \uac24\ub7ed\uc2dc \ub178\ud2b85\ubcf4\ub2e4 \uc57d 25% \uac00\ub7c9 \ucee4\uc84c\ub2e4. \uc804\uba74 \uce74\uba54\ub77c\ub294 \uc0bc\uc131\uc804\uc790 \uc2dc\uc2a4\ud15c LSI \uc0ac\uc5c5\ubd80 \uc544\uc774\uc18c\uc140 S5K4E6 \uc13c\uc11c\uc758 500\ub9cc \ud654\uc18c \uce74\uba54\ub77c\ub97c \ud0d1\uc7ac\ud588\ub2e4. \uc5ec\uae30\uc5d0 \ub514\uc2a4\ud50c\ub808\uc774\uac00 \ud50c\ub798\uc2dc \uc5ed\ud560\uc744 \ud558\ub294 '\uc140\ud53c \ud50c\ub798\uc2dc' \uae30\ub2a5\uc774 \ud0d1\uc7ac\ub418\uc5c8\ub2e4. \ub610\ud55c, \uc870\ub9ac\uac1c \ubc1d\uae30\ub294 \uc804\uc791\uc778 \uac24\ub7ed\uc2dc \ub178\ud2b85\uc758 \uc870\ub9ac\uac1c \ubc1d\uae30\uc778 F/1.9\ubcf4\ub2e4 \ubc1d\uc544\uc9c4 F/1.7\uc774\ub2e4.", "sentence_answer": "\uadf8\ub9ac\uace0 AF \ud2b8\ub798\ud0b9\uc744 \uc9c0\uc6d0\ud558\uace0 \" Dual Pixel \"", "paragraph_id": "6135925-4-0", "paragraph_question": "question: \ubb34\uc5c7\uc73c\ub85c\uc778\ud574 AF\uc18d\ub3c4\uac00 \ube68\ub77c\uc84c\ub294\uac00, context: \ud6c4\uba74 \uce74\uba54\ub77c\ub294 OIS \uae30\uc220\uc774 \uc801\uc6a9\ub41c \uce74\uba54\ub77c \ubaa8\ub4c8\uc5d0 \uc0bc\uc131\uc804\uc790 \uc2dc\uc2a4\ud15c LSI \uc0ac\uc5c5\ubd80 \uc544\uc774\uc18c\uc140 S5K2L1 \uc13c\uc11c\uc640 \uc18c\ub2c8 \uc5d1\uc2a4\ubaa8\uc5b4 IMX260 \uc13c\uc11c\ub97c \ud63c\uc6a9\ud558\uc5ec 1,200\ub9cc \ud654\uc18c \uce74\uba54\ub77c\ub97c \ud0d1\uc7ac\ud588\ub2e4. \uadf8\ub9ac\uace0 AF \ud2b8\ub798\ud0b9\uc744 \uc9c0\uc6d0\ud558\uace0 \"Dual Pixel\" \uae30\uc220\uc744 \ud65c\uc6a9\ud55c \uc704\uc0c1\ucc28 \uac80\ucd9c AF\ub97c \uc9c0\uc6d0\ud55c\ub2e4. \uc774\ub85c \uc778\ud574 AF \uc18d\ub3c4\uac00 \ube68\ub77c\uc84c\ub2e4\uace0 \ud55c\ub2e4. \ub610\ud55c, \uc13c\uc11c \ud06c\uae30\ub294 1/2.5\uc778\uce58\uc774\uba70 \uc870\ub9ac\uac1c \ubc1d\uae30\ub294 F/1.7\uc774\ub2e4. \ucd5c\ub300 \ud654\uc18c\uc218\ub294 \uc804\uc791\uc778 \uac24\ub7ed\uc2dc \ub178\ud2b85\ubcf4\ub2e4 \uc904\uc5b4\ub4e4\uc5c8\uc73c\ub098 \uc13c\uc11c \ud06c\uae30\uac00 1/2.6\uc778\uce58\uc5d0\uc11c \ucee4\uc84c\uace0 \uc870\ub9ac\uac1c \ubc1d\uae30\ub3c4 F/1.9\uc5d0\uc11c \ubc1d\uc544\uc84c\uc73c\uba70 \uc13c\uc11c \ube44\uc728 \uc5ed\uc2dc 16:9 \ube44\uc728\uc5d0\uc11c 4:3 \ube44\uc728\ub85c \ubc14\ub00c\uc5b4, \uce74\uba54\ub77c \ubaa8\ub4c8\uc774 \ub3cc\ucd9c\ub418\ub294 \uae30\ub2a5\ub3c4 \ub300\ud3ed \uac1c\uc120\ub418\uace0 \uc13c\uc11c \uba74\uc801\ub3c4 \uc804\uc791\uc778 \uac24\ub7ed\uc2dc \ub178\ud2b85\ubcf4\ub2e4 \uc57d 25% \uac00\ub7c9 \ucee4\uc84c\ub2e4. \uc804\uba74 \uce74\uba54\ub77c\ub294 \uc0bc\uc131\uc804\uc790 \uc2dc\uc2a4\ud15c LSI \uc0ac\uc5c5\ubd80 \uc544\uc774\uc18c\uc140 S5K4E6 \uc13c\uc11c\uc758 500\ub9cc \ud654\uc18c \uce74\uba54\ub77c\ub97c \ud0d1\uc7ac\ud588\ub2e4. \uc5ec\uae30\uc5d0 \ub514\uc2a4\ud50c\ub808\uc774\uac00 \ud50c\ub798\uc2dc \uc5ed\ud560\uc744 \ud558\ub294 '\uc140\ud53c \ud50c\ub798\uc2dc' \uae30\ub2a5\uc774 \ud0d1\uc7ac\ub418\uc5c8\ub2e4. \ub610\ud55c, \uc870\ub9ac\uac1c \ubc1d\uae30\ub294 \uc804\uc791\uc778 \uac24\ub7ed\uc2dc \ub178\ud2b85\uc758 \uc870\ub9ac\uac1c \ubc1d\uae30\uc778 F/1.9\ubcf4\ub2e4 \ubc1d\uc544\uc9c4 F/1.7\uc774\ub2e4."} {"question": "\ub2c8\uccb4\uac00 \uc0ac\ub9dd\ud55c \uacf3\uc740?", "paragraph": "35\uc138\uc5d0 \ubc14\uc824 \ub300\ud559\uad50\uc5d0\uc11c \ud1f4\uc9c1\ud55c \uc774\ud6c4 \uadf8\ub294 \uac15\uc5f0\ub3c4 \uadf8\ub9cc\ub450\uace0, \ubcd1\ub4e0 \ubab8\uc774 \uc801\uc751\ud560 \uc218 \uc788\ub294 \uacf3\uc744 \ucc3e\uc544 \uc720\ub7fd \uac01\uc9c0 \ub97c \ub3cc\uc544\ub2e4\ub2c8\uba74\uc11c \uc9d1\ud544\uc0dd\ud65c\uc5d0 \ubab0\ub450\ud558\uc600\ub2e4. 1881\ub144, \ud504\ub791\uc2a4\uac00 \ud280\ub2c8\uc9c0\ub97c \uc810\ub839\ud588\uc744 \ub54c, \ud280\ub2c8\uc9c0\ub85c \uc5ec\ud589\ud558\uae30\ub85c \ub9c8\uc74c\uba39\ub294\ub2e4. \ud558\uc9c0\ub9cc \uc774\ub0b4 \uadf8 \uacc4\ud68d\uc744 \uc811\uace0 \ub9cc\ub2e4.(\uac74\uac15\uc5d0 \ub300\ud55c \ubb38\uc81c\ub85c \uc5ec\ud589\uc744 \ucde8\uc18c\ud55c \uac83\uc73c\ub85c \ubcf4\uc778\ub2e4.) 1889\ub144 1\uc6d4 \uc774\ud0c8\ub9ac\uc544 \ud1a0\ub9ac\ub178\uc5d0\uc11c \uc878\ub3c4\ud55c \ud6c4 \uc815\uc2e0\ubcd1\uc6d0\uc5d0 \uc785\uc6d0\ud558\uc5ec \uc0dd\uc560\uc758 \ub9c8\uc9c0\ub9c9 10\ub144\uc744 \ubcf4\ub0c8\ub2e4. \ub2c8\uccb4\ub294 \uc815\uc2e0\ubcd1 \ubc1c\uc791\uc744 \uc77c\uc73c\ud0a8 \ud6c4 \uc644\uc804\ud788 \uc815\uc2e0 \uc0c1\uc2e4\uc790\uac00 \ub418\uc5c8\uace0, \uc774\ub54c\ubd80\ud130 \uc5b4\uba38\ub2c8\uc640 \ud568\uaed8 \uc608\ub098\uc5d0\uc11c \uac70\uc8fc\ud588\ub2e4. \uc5b4\uba38\ub2c8\uac00 \uc8fd\uc790 \ub204\uc774\ub3d9\uc0dd \uc5d8\ub9ac\uc790\ubca0\ud2b8\uac00 \ub2c8\uccb4\ub97c \ubc14\uc774\ub9c8\ub974\ub85c \uc62e\uacbc\uace0, \ub2c8\uccb4\ub294 1900\ub144 8\uc6d4 25\uc77c \ubc14\uc774\ub9c8\ub974\uc5d0\uc11c \uc8fd\uc5c8\ub2e4. \ub2c8\uccb4\uac00 \uc8fd\uc790 \uc5d8\ub9ac\uc790\ubca0\ud2b8\ub294 \uace0\ud5a5 \ub8b0\ucf04\uc758 \uc544\ubc84\uc9c0 \ubb18 \uc606\uc5d0 \ub2c8\uccb4\ub97c \uc548\uc7a5\ud588\ub2e4.", "answer": "\ubc14\uc774\ub9c8\ub974", "sentence": "\ub2c8\uccb4\ub97c \ubc14\uc774\ub9c8\ub974 \ub85c \uc62e\uacbc\uace0, \ub2c8\uccb4\ub294 1900\ub144 8\uc6d4 25\uc77c \ubc14\uc774\ub9c8\ub974\uc5d0\uc11c \uc8fd\uc5c8\ub2e4.", "paragraph_sentence": "35\uc138\uc5d0 \ubc14\uc824 \ub300\ud559\uad50\uc5d0\uc11c \ud1f4\uc9c1\ud55c \uc774\ud6c4 \uadf8\ub294 \uac15\uc5f0\ub3c4 \uadf8\ub9cc\ub450\uace0, \ubcd1\ub4e0 \ubab8\uc774 \uc801\uc751\ud560 \uc218 \uc788\ub294 \uacf3\uc744 \ucc3e\uc544 \uc720\ub7fd \uac01\uc9c0 \ub97c \ub3cc\uc544\ub2e4\ub2c8\uba74\uc11c \uc9d1\ud544\uc0dd\ud65c\uc5d0 \ubab0\ub450\ud558\uc600\ub2e4. 1881\ub144, \ud504\ub791\uc2a4\uac00 \ud280\ub2c8\uc9c0\ub97c \uc810\ub839\ud588\uc744 \ub54c, \ud280\ub2c8\uc9c0\ub85c \uc5ec\ud589\ud558\uae30\ub85c \ub9c8\uc74c\uba39\ub294\ub2e4. \ud558\uc9c0\ub9cc \uc774\ub0b4 \uadf8 \uacc4\ud68d\uc744 \uc811\uace0 \ub9cc\ub2e4.(\uac74\uac15\uc5d0 \ub300\ud55c \ubb38\uc81c\ub85c \uc5ec\ud589\uc744 \ucde8\uc18c\ud55c \uac83\uc73c\ub85c \ubcf4\uc778\ub2e4. ) 1889\ub144 1\uc6d4 \uc774\ud0c8\ub9ac\uc544 \ud1a0\ub9ac\ub178\uc5d0\uc11c \uc878\ub3c4\ud55c \ud6c4 \uc815\uc2e0\ubcd1\uc6d0\uc5d0 \uc785\uc6d0\ud558\uc5ec \uc0dd\uc560\uc758 \ub9c8\uc9c0\ub9c9 10\ub144\uc744 \ubcf4\ub0c8\ub2e4. \ub2c8\uccb4\ub294 \uc815\uc2e0\ubcd1 \ubc1c\uc791\uc744 \uc77c\uc73c\ud0a8 \ud6c4 \uc644\uc804\ud788 \uc815\uc2e0 \uc0c1\uc2e4\uc790\uac00 \ub418\uc5c8\uace0, \uc774\ub54c\ubd80\ud130 \uc5b4\uba38\ub2c8\uc640 \ud568\uaed8 \uc608\ub098\uc5d0\uc11c \uac70\uc8fc\ud588\ub2e4. \uc5b4\uba38\ub2c8\uac00 \uc8fd\uc790 \ub204\uc774\ub3d9\uc0dd \uc5d8\ub9ac\uc790\ubca0\ud2b8\uac00 \ub2c8\uccb4\ub97c \ubc14\uc774\ub9c8\ub974 \ub85c \uc62e\uacbc\uace0, \ub2c8\uccb4\ub294 1900\ub144 8\uc6d4 25\uc77c \ubc14\uc774\ub9c8\ub974\uc5d0\uc11c \uc8fd\uc5c8\ub2e4. \ub2c8\uccb4\uac00 \uc8fd\uc790 \uc5d8\ub9ac\uc790\ubca0\ud2b8\ub294 \uace0\ud5a5 \ub8b0\ucf04\uc758 \uc544\ubc84\uc9c0 \ubb18 \uc606\uc5d0 \ub2c8\uccb4\ub97c \uc548\uc7a5\ud588\ub2e4.", "paragraph_answer": "35\uc138\uc5d0 \ubc14\uc824 \ub300\ud559\uad50\uc5d0\uc11c \ud1f4\uc9c1\ud55c \uc774\ud6c4 \uadf8\ub294 \uac15\uc5f0\ub3c4 \uadf8\ub9cc\ub450\uace0, \ubcd1\ub4e0 \ubab8\uc774 \uc801\uc751\ud560 \uc218 \uc788\ub294 \uacf3\uc744 \ucc3e\uc544 \uc720\ub7fd \uac01\uc9c0 \ub97c \ub3cc\uc544\ub2e4\ub2c8\uba74\uc11c \uc9d1\ud544\uc0dd\ud65c\uc5d0 \ubab0\ub450\ud558\uc600\ub2e4. 1881\ub144, \ud504\ub791\uc2a4\uac00 \ud280\ub2c8\uc9c0\ub97c \uc810\ub839\ud588\uc744 \ub54c, \ud280\ub2c8\uc9c0\ub85c \uc5ec\ud589\ud558\uae30\ub85c \ub9c8\uc74c\uba39\ub294\ub2e4. \ud558\uc9c0\ub9cc \uc774\ub0b4 \uadf8 \uacc4\ud68d\uc744 \uc811\uace0 \ub9cc\ub2e4.(\uac74\uac15\uc5d0 \ub300\ud55c \ubb38\uc81c\ub85c \uc5ec\ud589\uc744 \ucde8\uc18c\ud55c \uac83\uc73c\ub85c \ubcf4\uc778\ub2e4.) 1889\ub144 1\uc6d4 \uc774\ud0c8\ub9ac\uc544 \ud1a0\ub9ac\ub178\uc5d0\uc11c \uc878\ub3c4\ud55c \ud6c4 \uc815\uc2e0\ubcd1\uc6d0\uc5d0 \uc785\uc6d0\ud558\uc5ec \uc0dd\uc560\uc758 \ub9c8\uc9c0\ub9c9 10\ub144\uc744 \ubcf4\ub0c8\ub2e4. \ub2c8\uccb4\ub294 \uc815\uc2e0\ubcd1 \ubc1c\uc791\uc744 \uc77c\uc73c\ud0a8 \ud6c4 \uc644\uc804\ud788 \uc815\uc2e0 \uc0c1\uc2e4\uc790\uac00 \ub418\uc5c8\uace0, \uc774\ub54c\ubd80\ud130 \uc5b4\uba38\ub2c8\uc640 \ud568\uaed8 \uc608\ub098\uc5d0\uc11c \uac70\uc8fc\ud588\ub2e4. \uc5b4\uba38\ub2c8\uac00 \uc8fd\uc790 \ub204\uc774\ub3d9\uc0dd \uc5d8\ub9ac\uc790\ubca0\ud2b8\uac00 \ub2c8\uccb4\ub97c \ubc14\uc774\ub9c8\ub974 \ub85c \uc62e\uacbc\uace0, \ub2c8\uccb4\ub294 1900\ub144 8\uc6d4 25\uc77c \ubc14\uc774\ub9c8\ub974\uc5d0\uc11c \uc8fd\uc5c8\ub2e4. \ub2c8\uccb4\uac00 \uc8fd\uc790 \uc5d8\ub9ac\uc790\ubca0\ud2b8\ub294 \uace0\ud5a5 \ub8b0\ucf04\uc758 \uc544\ubc84\uc9c0 \ubb18 \uc606\uc5d0 \ub2c8\uccb4\ub97c \uc548\uc7a5\ud588\ub2e4.", "sentence_answer": "\ub2c8\uccb4\ub97c \ubc14\uc774\ub9c8\ub974 \ub85c \uc62e\uacbc\uace0, \ub2c8\uccb4\ub294 1900\ub144 8\uc6d4 25\uc77c \ubc14\uc774\ub9c8\ub974\uc5d0\uc11c \uc8fd\uc5c8\ub2e4.", "paragraph_id": "6568883-4-1", "paragraph_question": "question: \ub2c8\uccb4\uac00 \uc0ac\ub9dd\ud55c \uacf3\uc740?, context: 35\uc138\uc5d0 \ubc14\uc824 \ub300\ud559\uad50\uc5d0\uc11c \ud1f4\uc9c1\ud55c \uc774\ud6c4 \uadf8\ub294 \uac15\uc5f0\ub3c4 \uadf8\ub9cc\ub450\uace0, \ubcd1\ub4e0 \ubab8\uc774 \uc801\uc751\ud560 \uc218 \uc788\ub294 \uacf3\uc744 \ucc3e\uc544 \uc720\ub7fd \uac01\uc9c0 \ub97c \ub3cc\uc544\ub2e4\ub2c8\uba74\uc11c \uc9d1\ud544\uc0dd\ud65c\uc5d0 \ubab0\ub450\ud558\uc600\ub2e4. 1881\ub144, \ud504\ub791\uc2a4\uac00 \ud280\ub2c8\uc9c0\ub97c \uc810\ub839\ud588\uc744 \ub54c, \ud280\ub2c8\uc9c0\ub85c \uc5ec\ud589\ud558\uae30\ub85c \ub9c8\uc74c\uba39\ub294\ub2e4. \ud558\uc9c0\ub9cc \uc774\ub0b4 \uadf8 \uacc4\ud68d\uc744 \uc811\uace0 \ub9cc\ub2e4.(\uac74\uac15\uc5d0 \ub300\ud55c \ubb38\uc81c\ub85c \uc5ec\ud589\uc744 \ucde8\uc18c\ud55c \uac83\uc73c\ub85c \ubcf4\uc778\ub2e4.) 1889\ub144 1\uc6d4 \uc774\ud0c8\ub9ac\uc544 \ud1a0\ub9ac\ub178\uc5d0\uc11c \uc878\ub3c4\ud55c \ud6c4 \uc815\uc2e0\ubcd1\uc6d0\uc5d0 \uc785\uc6d0\ud558\uc5ec \uc0dd\uc560\uc758 \ub9c8\uc9c0\ub9c9 10\ub144\uc744 \ubcf4\ub0c8\ub2e4. \ub2c8\uccb4\ub294 \uc815\uc2e0\ubcd1 \ubc1c\uc791\uc744 \uc77c\uc73c\ud0a8 \ud6c4 \uc644\uc804\ud788 \uc815\uc2e0 \uc0c1\uc2e4\uc790\uac00 \ub418\uc5c8\uace0, \uc774\ub54c\ubd80\ud130 \uc5b4\uba38\ub2c8\uc640 \ud568\uaed8 \uc608\ub098\uc5d0\uc11c \uac70\uc8fc\ud588\ub2e4. \uc5b4\uba38\ub2c8\uac00 \uc8fd\uc790 \ub204\uc774\ub3d9\uc0dd \uc5d8\ub9ac\uc790\ubca0\ud2b8\uac00 \ub2c8\uccb4\ub97c \ubc14\uc774\ub9c8\ub974\ub85c \uc62e\uacbc\uace0, \ub2c8\uccb4\ub294 1900\ub144 8\uc6d4 25\uc77c \ubc14\uc774\ub9c8\ub974\uc5d0\uc11c \uc8fd\uc5c8\ub2e4. \ub2c8\uccb4\uac00 \uc8fd\uc790 \uc5d8\ub9ac\uc790\ubca0\ud2b8\ub294 \uace0\ud5a5 \ub8b0\ucf04\uc758 \uc544\ubc84\uc9c0 \ubb18 \uc606\uc5d0 \ub2c8\uccb4\ub97c \uc548\uc7a5\ud588\ub2e4."} {"question": "\ud55c\uc77c\uc5b4\uc5fd\ud611\uc815\uc744 \uc218\uc815\ud574\uc57c \ud55c\ub2e4\ub294 \uc8fc\uc7a5\uc5d0 \uc2e4\ud6a8\uc131\uc774 \uc5c6\ub2e4\uace0 \ud55c \uc0ac\ub78c\uc740?", "paragraph": "\ub3c5\ub3c4\ub97c \ub300\ud55c\ubbfc\uad6d\uc758 \uc601\ud1a0\uc778\ub370\ub3c4 \uc774\ub97c \uc911\uac04\uc218\uc5ed\uc73c\ub85c \uc124\uc815\ud558\uba74\uc11c \uc77c\ubcf8\uc758 \ub3c5\ub3c4\uac15\ud0c8 \uc2dc\ub3c4\ub97c \uadf8\ub300\ub85c \ubc1b\uc544 \ub4e4\uc600\ub2e4\ub294 \uc8fc\uc7a5\uc774 \uc788\ub2e4. \uc81c\uc131\ud638 \uad50\uc218\ub294 \uc774\ub7f0 \uc911\uac04\uc218\uc5ed\uc740 \uc77c\uc885\uc758 '\ubd84\uc7c1\uc218\uc5ed'\uc774\ub77c\uace0 \ud3c9\ud588\ub2e4. \ub3c5\ub3c4\ub97c \ud55c\uc77c\uc911\uac04\uc218\uc5ed\uc5d0 \ub193\uc774\uac8c \ud55c \uc2e0 \ud55c\uc77c\uc5b4\uc5c5\ud611\uc815\uc744 \uc218\uc815\ud574\uc57c \ud55c\ub2e4\ub294 \uc77c\uac01\uc758 \uc8fc\uc7a5\uc5d0 \ub300\ud574 \ucd5c\ub099\uc815 \uc804 \ud574\uc591\uc218\uc0b0\ubd80 \uc7a5\uad00\uc740 \"\uc2e4\ud6a8\uc131\uc774 \uc5c6\ub2e4\"\uace0 \uc77c\ucd95\ud588\ub2e4. \ud611\uc0c1\uc744 \ub2e4\uc2dc \ud55c\ub2e4\uace0 \ud574\ub3c4 \uc77c\ubcf8\uc774 \ub3c5\ub3c4\ub97c \uc6b0\ub9ac \ub545\uc73c\ub85c \uc778\uc815\ud574\uc8fc\uc9c0\ub3c4 \uc54a\uc744 \uac83\uc774\uace0, \ub610 \ub2e4\ub978 \ub9c8\ucc30\uc744 \uc77c\uc73c\ud0ac \uc218 \uc788\uae30 \ub54c\ubb38\uc5d0 \uc2e4\ud6a8\uc131\uc774 \uc5c6\ub2e4\"\uace0 \ubc1d\ud614\ub2e4. \uadf8\ub294 \uc774\uc5b4 \"\uc2e0 \ud55c\uc77c\uc5b4\uc5c5\ud611\uc815\uc740 \uae40\uc601\uc0bc \uc815\ubd80 \ub54c\ubd80\ud130 5\ub144 \uac04 \uc9c4\ud589\ub418\ub2e4 \uae40\ub300\uc911 \uc815\ubd80 \ub4e4\uc5b4 \ub9c8\ubb34\ub9ac \ub41c \uac83\"\uc73c\ub85c \"\ub3c5\ub3c4 \uc601\uc720\uad8c\uc744 \ud6fc\uc190\ud588\ub2e4\uace0 \ubcf4\uc9c0\ub3c4 \uc54a\ub294\ub2e4\"\uace0 \ub9d0\ud588\ub2e4. \ucd5c\ub099\uc815 \uc804 \uc7a5\uad00\uc740 \ub610, \ub3c5\ub3c4\uac00 \ubc30\ud0c0\uc801 \uacbd\uc81c\uc218\uc5ed\uc774 \uc544\ub2cc \ud55c\uc77c\uc911\uac04\uc218\uc5ed\uc5d0 \ub193\uc778 \uac83\uc774 \uc77c\ubcf8\uc758 \ub3c5\ub3c4 \uc601\uc720\uad8c \uc8fc\uc7a5\uc5d0 \ube4c\ubbf8\ub97c \uc900 \uac83 \uc544\ub2c8\ub0d0\ub294 \uc9c8\uc758\uc5d0 \"\uc5b4\uc5c5\ud611\uc815\uc740 \uc601\ud1a0 \uc601\uc720\uad8c \ubb38\uc81c\uc640\ub294 \ubb34\uad00\ud558\ub2e4\"\uace0 \uc124\uba85\ud588\ub2e4. \ucd5c \uc804 \uc7a5\uad00\uc740 \"\uc2e0 \ud55c\uc77c\uc5b4\uc5c5\ud611\uc815 \uc870\ubb38\uc5d0\uc11c \uc774 \ud611\uc815\uc740 \uc601\uc720\uad8c\uc5d0\ub294 \uc804\ud600 \uc601\ud5a5\uc774 \uc5c6\ub2e4\uace0 \ubc1d\ud788\uace0 \uc788\uace0, \ud5cc\ubc95\uc7ac\ud310\uc18c\ub098 \uad6d\uc81c\uc0ac\ubc95\uc7ac\ud310\uc18c \uc5ed\uc2dc \uc5b4\uc5c5\ud611\uc815\uc740 \uc5b4\uc5c5 \ubb38\uc81c\uc5d0\ub9cc \uad6d\ud55c\ub428\uc744 \ud310\uc2dc\ud588\ub2e4\"\uace0 \ub367\ubd99\uc600\ub2e4.", "answer": "\ucd5c\ub099\uc815", "sentence": "\ub3c5\ub3c4\ub97c \ud55c\uc77c\uc911\uac04\uc218\uc5ed\uc5d0 \ub193\uc774\uac8c \ud55c \uc2e0 \ud55c\uc77c\uc5b4\uc5c5\ud611\uc815\uc744 \uc218\uc815\ud574\uc57c \ud55c\ub2e4\ub294 \uc77c\uac01\uc758 \uc8fc\uc7a5\uc5d0 \ub300\ud574 \ucd5c\ub099\uc815 \uc804 \ud574\uc591\uc218\uc0b0\ubd80 \uc7a5\uad00\uc740 \"\uc2e4\ud6a8\uc131\uc774 \uc5c6\ub2e4\"\uace0 \uc77c\ucd95\ud588\ub2e4.", "paragraph_sentence": "\ub3c5\ub3c4\ub97c \ub300\ud55c\ubbfc\uad6d\uc758 \uc601\ud1a0\uc778\ub370\ub3c4 \uc774\ub97c \uc911\uac04\uc218\uc5ed\uc73c\ub85c \uc124\uc815\ud558\uba74\uc11c \uc77c\ubcf8\uc758 \ub3c5\ub3c4\uac15\ud0c8 \uc2dc\ub3c4\ub97c \uadf8\ub300\ub85c \ubc1b\uc544 \ub4e4\uc600\ub2e4\ub294 \uc8fc\uc7a5\uc774 \uc788\ub2e4. \uc81c\uc131\ud638 \uad50\uc218\ub294 \uc774\ub7f0 \uc911\uac04\uc218\uc5ed\uc740 \uc77c\uc885\uc758 '\ubd84\uc7c1\uc218\uc5ed'\uc774\ub77c\uace0 \ud3c9\ud588\ub2e4. \ub3c5\ub3c4\ub97c \ud55c\uc77c\uc911\uac04\uc218\uc5ed\uc5d0 \ub193\uc774\uac8c \ud55c \uc2e0 \ud55c\uc77c\uc5b4\uc5c5\ud611\uc815\uc744 \uc218\uc815\ud574\uc57c \ud55c\ub2e4\ub294 \uc77c\uac01\uc758 \uc8fc\uc7a5\uc5d0 \ub300\ud574 \ucd5c\ub099\uc815 \uc804 \ud574\uc591\uc218\uc0b0\ubd80 \uc7a5\uad00\uc740 \"\uc2e4\ud6a8\uc131\uc774 \uc5c6\ub2e4\"\uace0 \uc77c\ucd95\ud588\ub2e4. \ud611\uc0c1\uc744 \ub2e4\uc2dc \ud55c\ub2e4\uace0 \ud574\ub3c4 \uc77c\ubcf8\uc774 \ub3c5\ub3c4\ub97c \uc6b0\ub9ac \ub545\uc73c\ub85c \uc778\uc815\ud574\uc8fc\uc9c0\ub3c4 \uc54a\uc744 \uac83\uc774\uace0, \ub610 \ub2e4\ub978 \ub9c8\ucc30\uc744 \uc77c\uc73c\ud0ac \uc218 \uc788\uae30 \ub54c\ubb38\uc5d0 \uc2e4\ud6a8\uc131\uc774 \uc5c6\ub2e4\"\uace0 \ubc1d\ud614\ub2e4. \uadf8\ub294 \uc774\uc5b4 \"\uc2e0 \ud55c\uc77c\uc5b4\uc5c5\ud611\uc815\uc740 \uae40\uc601\uc0bc \uc815\ubd80 \ub54c\ubd80\ud130 5\ub144 \uac04 \uc9c4\ud589\ub418\ub2e4 \uae40\ub300\uc911 \uc815\ubd80 \ub4e4\uc5b4 \ub9c8\ubb34\ub9ac \ub41c \uac83\"\uc73c\ub85c \"\ub3c5\ub3c4 \uc601\uc720\uad8c\uc744 \ud6fc\uc190\ud588\ub2e4\uace0 \ubcf4\uc9c0\ub3c4 \uc54a\ub294\ub2e4\"\uace0 \ub9d0\ud588\ub2e4. \ucd5c\ub099\uc815 \uc804 \uc7a5\uad00\uc740 \ub610, \ub3c5\ub3c4\uac00 \ubc30\ud0c0\uc801 \uacbd\uc81c\uc218\uc5ed\uc774 \uc544\ub2cc \ud55c\uc77c\uc911\uac04\uc218\uc5ed\uc5d0 \ub193\uc778 \uac83\uc774 \uc77c\ubcf8\uc758 \ub3c5\ub3c4 \uc601\uc720\uad8c \uc8fc\uc7a5\uc5d0 \ube4c\ubbf8\ub97c \uc900 \uac83 \uc544\ub2c8\ub0d0\ub294 \uc9c8\uc758\uc5d0 \"\uc5b4\uc5c5\ud611\uc815\uc740 \uc601\ud1a0 \uc601\uc720\uad8c \ubb38\uc81c\uc640\ub294 \ubb34\uad00\ud558\ub2e4\"\uace0 \uc124\uba85\ud588\ub2e4. \ucd5c \uc804 \uc7a5\uad00\uc740 \"\uc2e0 \ud55c\uc77c\uc5b4\uc5c5\ud611\uc815 \uc870\ubb38\uc5d0\uc11c \uc774 \ud611\uc815\uc740 \uc601\uc720\uad8c\uc5d0\ub294 \uc804\ud600 \uc601\ud5a5\uc774 \uc5c6\ub2e4\uace0 \ubc1d\ud788\uace0 \uc788\uace0, \ud5cc\ubc95\uc7ac\ud310\uc18c\ub098 \uad6d\uc81c\uc0ac\ubc95\uc7ac\ud310\uc18c \uc5ed\uc2dc \uc5b4\uc5c5\ud611\uc815\uc740 \uc5b4\uc5c5 \ubb38\uc81c\uc5d0\ub9cc \uad6d\ud55c\ub428\uc744 \ud310\uc2dc\ud588\ub2e4\"\uace0 \ub367\ubd99\uc600\ub2e4.", "paragraph_answer": "\ub3c5\ub3c4\ub97c \ub300\ud55c\ubbfc\uad6d\uc758 \uc601\ud1a0\uc778\ub370\ub3c4 \uc774\ub97c \uc911\uac04\uc218\uc5ed\uc73c\ub85c \uc124\uc815\ud558\uba74\uc11c \uc77c\ubcf8\uc758 \ub3c5\ub3c4\uac15\ud0c8 \uc2dc\ub3c4\ub97c \uadf8\ub300\ub85c \ubc1b\uc544 \ub4e4\uc600\ub2e4\ub294 \uc8fc\uc7a5\uc774 \uc788\ub2e4. \uc81c\uc131\ud638 \uad50\uc218\ub294 \uc774\ub7f0 \uc911\uac04\uc218\uc5ed\uc740 \uc77c\uc885\uc758 '\ubd84\uc7c1\uc218\uc5ed'\uc774\ub77c\uace0 \ud3c9\ud588\ub2e4. \ub3c5\ub3c4\ub97c \ud55c\uc77c\uc911\uac04\uc218\uc5ed\uc5d0 \ub193\uc774\uac8c \ud55c \uc2e0 \ud55c\uc77c\uc5b4\uc5c5\ud611\uc815\uc744 \uc218\uc815\ud574\uc57c \ud55c\ub2e4\ub294 \uc77c\uac01\uc758 \uc8fc\uc7a5\uc5d0 \ub300\ud574 \ucd5c\ub099\uc815 \uc804 \ud574\uc591\uc218\uc0b0\ubd80 \uc7a5\uad00\uc740 \"\uc2e4\ud6a8\uc131\uc774 \uc5c6\ub2e4\"\uace0 \uc77c\ucd95\ud588\ub2e4. \ud611\uc0c1\uc744 \ub2e4\uc2dc \ud55c\ub2e4\uace0 \ud574\ub3c4 \uc77c\ubcf8\uc774 \ub3c5\ub3c4\ub97c \uc6b0\ub9ac \ub545\uc73c\ub85c \uc778\uc815\ud574\uc8fc\uc9c0\ub3c4 \uc54a\uc744 \uac83\uc774\uace0, \ub610 \ub2e4\ub978 \ub9c8\ucc30\uc744 \uc77c\uc73c\ud0ac \uc218 \uc788\uae30 \ub54c\ubb38\uc5d0 \uc2e4\ud6a8\uc131\uc774 \uc5c6\ub2e4\"\uace0 \ubc1d\ud614\ub2e4. \uadf8\ub294 \uc774\uc5b4 \"\uc2e0 \ud55c\uc77c\uc5b4\uc5c5\ud611\uc815\uc740 \uae40\uc601\uc0bc \uc815\ubd80 \ub54c\ubd80\ud130 5\ub144 \uac04 \uc9c4\ud589\ub418\ub2e4 \uae40\ub300\uc911 \uc815\ubd80 \ub4e4\uc5b4 \ub9c8\ubb34\ub9ac \ub41c \uac83\"\uc73c\ub85c \"\ub3c5\ub3c4 \uc601\uc720\uad8c\uc744 \ud6fc\uc190\ud588\ub2e4\uace0 \ubcf4\uc9c0\ub3c4 \uc54a\ub294\ub2e4\"\uace0 \ub9d0\ud588\ub2e4. \ucd5c\ub099\uc815 \uc804 \uc7a5\uad00\uc740 \ub610, \ub3c5\ub3c4\uac00 \ubc30\ud0c0\uc801 \uacbd\uc81c\uc218\uc5ed\uc774 \uc544\ub2cc \ud55c\uc77c\uc911\uac04\uc218\uc5ed\uc5d0 \ub193\uc778 \uac83\uc774 \uc77c\ubcf8\uc758 \ub3c5\ub3c4 \uc601\uc720\uad8c \uc8fc\uc7a5\uc5d0 \ube4c\ubbf8\ub97c \uc900 \uac83 \uc544\ub2c8\ub0d0\ub294 \uc9c8\uc758\uc5d0 \"\uc5b4\uc5c5\ud611\uc815\uc740 \uc601\ud1a0 \uc601\uc720\uad8c \ubb38\uc81c\uc640\ub294 \ubb34\uad00\ud558\ub2e4\"\uace0 \uc124\uba85\ud588\ub2e4. \ucd5c \uc804 \uc7a5\uad00\uc740 \"\uc2e0 \ud55c\uc77c\uc5b4\uc5c5\ud611\uc815 \uc870\ubb38\uc5d0\uc11c \uc774 \ud611\uc815\uc740 \uc601\uc720\uad8c\uc5d0\ub294 \uc804\ud600 \uc601\ud5a5\uc774 \uc5c6\ub2e4\uace0 \ubc1d\ud788\uace0 \uc788\uace0, \ud5cc\ubc95\uc7ac\ud310\uc18c\ub098 \uad6d\uc81c\uc0ac\ubc95\uc7ac\ud310\uc18c \uc5ed\uc2dc \uc5b4\uc5c5\ud611\uc815\uc740 \uc5b4\uc5c5 \ubb38\uc81c\uc5d0\ub9cc \uad6d\ud55c\ub428\uc744 \ud310\uc2dc\ud588\ub2e4\"\uace0 \ub367\ubd99\uc600\ub2e4.", "sentence_answer": "\ub3c5\ub3c4\ub97c \ud55c\uc77c\uc911\uac04\uc218\uc5ed\uc5d0 \ub193\uc774\uac8c \ud55c \uc2e0 \ud55c\uc77c\uc5b4\uc5c5\ud611\uc815\uc744 \uc218\uc815\ud574\uc57c \ud55c\ub2e4\ub294 \uc77c\uac01\uc758 \uc8fc\uc7a5\uc5d0 \ub300\ud574 \ucd5c\ub099\uc815 \uc804 \ud574\uc591\uc218\uc0b0\ubd80 \uc7a5\uad00\uc740 \"\uc2e4\ud6a8\uc131\uc774 \uc5c6\ub2e4\"\uace0 \uc77c\ucd95\ud588\ub2e4.", "paragraph_id": "6576830-29-0", "paragraph_question": "question: \ud55c\uc77c\uc5b4\uc5fd\ud611\uc815\uc744 \uc218\uc815\ud574\uc57c \ud55c\ub2e4\ub294 \uc8fc\uc7a5\uc5d0 \uc2e4\ud6a8\uc131\uc774 \uc5c6\ub2e4\uace0 \ud55c \uc0ac\ub78c\uc740?, context: \ub3c5\ub3c4\ub97c \ub300\ud55c\ubbfc\uad6d\uc758 \uc601\ud1a0\uc778\ub370\ub3c4 \uc774\ub97c \uc911\uac04\uc218\uc5ed\uc73c\ub85c \uc124\uc815\ud558\uba74\uc11c \uc77c\ubcf8\uc758 \ub3c5\ub3c4\uac15\ud0c8 \uc2dc\ub3c4\ub97c \uadf8\ub300\ub85c \ubc1b\uc544 \ub4e4\uc600\ub2e4\ub294 \uc8fc\uc7a5\uc774 \uc788\ub2e4. \uc81c\uc131\ud638 \uad50\uc218\ub294 \uc774\ub7f0 \uc911\uac04\uc218\uc5ed\uc740 \uc77c\uc885\uc758 '\ubd84\uc7c1\uc218\uc5ed'\uc774\ub77c\uace0 \ud3c9\ud588\ub2e4. \ub3c5\ub3c4\ub97c \ud55c\uc77c\uc911\uac04\uc218\uc5ed\uc5d0 \ub193\uc774\uac8c \ud55c \uc2e0 \ud55c\uc77c\uc5b4\uc5c5\ud611\uc815\uc744 \uc218\uc815\ud574\uc57c \ud55c\ub2e4\ub294 \uc77c\uac01\uc758 \uc8fc\uc7a5\uc5d0 \ub300\ud574 \ucd5c\ub099\uc815 \uc804 \ud574\uc591\uc218\uc0b0\ubd80 \uc7a5\uad00\uc740 \"\uc2e4\ud6a8\uc131\uc774 \uc5c6\ub2e4\"\uace0 \uc77c\ucd95\ud588\ub2e4. \ud611\uc0c1\uc744 \ub2e4\uc2dc \ud55c\ub2e4\uace0 \ud574\ub3c4 \uc77c\ubcf8\uc774 \ub3c5\ub3c4\ub97c \uc6b0\ub9ac \ub545\uc73c\ub85c \uc778\uc815\ud574\uc8fc\uc9c0\ub3c4 \uc54a\uc744 \uac83\uc774\uace0, \ub610 \ub2e4\ub978 \ub9c8\ucc30\uc744 \uc77c\uc73c\ud0ac \uc218 \uc788\uae30 \ub54c\ubb38\uc5d0 \uc2e4\ud6a8\uc131\uc774 \uc5c6\ub2e4\"\uace0 \ubc1d\ud614\ub2e4. \uadf8\ub294 \uc774\uc5b4 \"\uc2e0 \ud55c\uc77c\uc5b4\uc5c5\ud611\uc815\uc740 \uae40\uc601\uc0bc \uc815\ubd80 \ub54c\ubd80\ud130 5\ub144 \uac04 \uc9c4\ud589\ub418\ub2e4 \uae40\ub300\uc911 \uc815\ubd80 \ub4e4\uc5b4 \ub9c8\ubb34\ub9ac \ub41c \uac83\"\uc73c\ub85c \"\ub3c5\ub3c4 \uc601\uc720\uad8c\uc744 \ud6fc\uc190\ud588\ub2e4\uace0 \ubcf4\uc9c0\ub3c4 \uc54a\ub294\ub2e4\"\uace0 \ub9d0\ud588\ub2e4. \ucd5c\ub099\uc815 \uc804 \uc7a5\uad00\uc740 \ub610, \ub3c5\ub3c4\uac00 \ubc30\ud0c0\uc801 \uacbd\uc81c\uc218\uc5ed\uc774 \uc544\ub2cc \ud55c\uc77c\uc911\uac04\uc218\uc5ed\uc5d0 \ub193\uc778 \uac83\uc774 \uc77c\ubcf8\uc758 \ub3c5\ub3c4 \uc601\uc720\uad8c \uc8fc\uc7a5\uc5d0 \ube4c\ubbf8\ub97c \uc900 \uac83 \uc544\ub2c8\ub0d0\ub294 \uc9c8\uc758\uc5d0 \"\uc5b4\uc5c5\ud611\uc815\uc740 \uc601\ud1a0 \uc601\uc720\uad8c \ubb38\uc81c\uc640\ub294 \ubb34\uad00\ud558\ub2e4\"\uace0 \uc124\uba85\ud588\ub2e4. \ucd5c \uc804 \uc7a5\uad00\uc740 \"\uc2e0 \ud55c\uc77c\uc5b4\uc5c5\ud611\uc815 \uc870\ubb38\uc5d0\uc11c \uc774 \ud611\uc815\uc740 \uc601\uc720\uad8c\uc5d0\ub294 \uc804\ud600 \uc601\ud5a5\uc774 \uc5c6\ub2e4\uace0 \ubc1d\ud788\uace0 \uc788\uace0, \ud5cc\ubc95\uc7ac\ud310\uc18c\ub098 \uad6d\uc81c\uc0ac\ubc95\uc7ac\ud310\uc18c \uc5ed\uc2dc \uc5b4\uc5c5\ud611\uc815\uc740 \uc5b4\uc5c5 \ubb38\uc81c\uc5d0\ub9cc \uad6d\ud55c\ub428\uc744 \ud310\uc2dc\ud588\ub2e4\"\uace0 \ub367\ubd99\uc600\ub2e4."} {"question": "\uc601\ud654 \ubca4\ud5c8 \ub9ac\uba54\uc774\ud06c\ud310\uc758 \uac10\ub3c5 \uc640\uc77c\ub7ec\uac00 \ubc30\uc5ed\uc744 \uc124\uc815\ud560 \ub54c \ubbf8\uad6d\ucd9c\uc2e0 \ubc30\uc6b0\ub97c \uae30\uc6a9\ud55c \uc5ed\ud560\uc740?", "paragraph": "1957\ub144 \uc911\uc21c MGM\uc740 \ub85c\ub9c8\uc5d0 \uce90\uc2a4\ud305 \uc0ac\ubb34\uc18c\ub97c \uc5f4\uc5b4 \uc870\uc5f0 \ubc0f \uc5d1\uc2a4\ud2b8\ub77c\ub85c \ud65c\uc57d\ud560 5\ub9cc \uba85\uc744 \uc120\uc815\ud588\ub294\ub370 \uc774 \uc911 \ub300\uc0ac\uac00 \uc788\ub294 \ubc30\uc6b0\ub294 365\uba85, \uadf8 \uc911\uc5d0\uc11c \ube44\uc911 \uc788\ub294 \uc5ed\ud560\uc744 \ub9e1\uc740 \ubc30\uc6b0\uac00 45\uba85\uc774\uc5c8\ub2e4. \uc640\uc77c\ub7ec \uac10\ub3c5\uc740 \ubc30\uc5ed\uc744 \uc124\uc815\ud560 \ub54c \uc678\ubaa8\ub098 \uacfc\uac70 \uc5f0\uae30 \uacbd\ub825\ubcf4\ub2e4\ub294 \uc778\ubb3c\uc758 \uc131\uaca9\uc744 \uc798 \ud45c\ud604\ud560 \uc218 \uc788\ub294 \ub2a5\ub825\uc5d0 \ub354 \ud070 \ube44\uc911\uc744 \ub480\uc73c\uba70 \uc791\uc911 \ub85c\ub9c8\uc778 \uc5ed\ud560\uc5d0\ub294 \uc601\uad6d\ucd9c\uc2e0 \ubc30\uc6b0\ub97c, \uc720\ub300\uc778 \uc5ed\ud560\uc5d0\ub294 \ubbf8\uad6d\ucd9c\uc2e0 \ubc30\uc6b0\ub97c \uc758\ub3c4\uc801\uc73c\ub85c \uad6c\ubcc4 \ubc30\uce58\ud558\uc5ec \uace0\ub300 \ub450 \ubbfc\uc871 \uc0ac\uc774\uc758 \ucc28\uc774\uc810\uc744 \uac15\uc870\ud558\uace0\uc790 \ud588\ub2e4. \uac10\ub3c5\uc774 \uc601\uad6d\uc778\uc744 \uace0\ub300 \ub85c\ub9c8\uc778 \uc5f0\uae30\uc790\ub85c \uc815\ud55c \uc774\uc720\ub294 \ubbf8\uad6d \uad00\uc911\ub4e4\uc774 \uc601\uad6d\uc2dd \ub9d0\ud22c\uac00 \uc0c1\ub958\uce35\uc774 \uc4f0\ub294 \ub2e8\uc5b4\ub77c\uace0 \uc0dd\uac01\ud558\ubbc0\ub85c \uc791\uc911 \ub85c\ub9c8\uc778\ub4e4\uc758 \uc2e0\ubd84\uc778 \uadc0\uc871 \ubc0f \uad8c\ub825\uc790 \uc124\uc815\uc5d0 \uc5b4\uc6b8\ub9b0\ub2e4\uace0 \uc0dd\uac01\ud588\uae30 \ub54c\ubb38\uc774\ub2e4.", "answer": "\uc720\ub300\uc778", "sentence": "\uc640\uc77c\ub7ec \uac10\ub3c5\uc740 \ubc30\uc5ed\uc744 \uc124\uc815\ud560 \ub54c \uc678\ubaa8\ub098 \uacfc\uac70 \uc5f0\uae30 \uacbd\ub825\ubcf4\ub2e4\ub294 \uc778\ubb3c\uc758 \uc131\uaca9\uc744 \uc798 \ud45c\ud604\ud560 \uc218 \uc788\ub294 \ub2a5\ub825\uc5d0 \ub354 \ud070 \ube44\uc911\uc744 \ub480\uc73c\uba70 \uc791\uc911 \ub85c\ub9c8\uc778 \uc5ed\ud560\uc5d0\ub294 \uc601\uad6d\ucd9c\uc2e0 \ubc30\uc6b0\ub97c, \uc720\ub300\uc778 \uc5ed\ud560\uc5d0\ub294 \ubbf8\uad6d\ucd9c\uc2e0 \ubc30\uc6b0\ub97c \uc758\ub3c4\uc801\uc73c\ub85c \uad6c\ubcc4 \ubc30\uce58\ud558\uc5ec \uace0\ub300 \ub450 \ubbfc\uc871 \uc0ac\uc774\uc758 \ucc28\uc774\uc810\uc744 \uac15\uc870\ud558\uace0\uc790 \ud588\ub2e4.", "paragraph_sentence": "1957\ub144 \uc911\uc21c MGM\uc740 \ub85c\ub9c8\uc5d0 \uce90\uc2a4\ud305 \uc0ac\ubb34\uc18c\ub97c \uc5f4\uc5b4 \uc870\uc5f0 \ubc0f \uc5d1\uc2a4\ud2b8\ub77c\ub85c \ud65c\uc57d\ud560 5\ub9cc \uba85\uc744 \uc120\uc815\ud588\ub294\ub370 \uc774 \uc911 \ub300\uc0ac\uac00 \uc788\ub294 \ubc30\uc6b0\ub294 365\uba85, \uadf8 \uc911\uc5d0\uc11c \ube44\uc911 \uc788\ub294 \uc5ed\ud560\uc744 \ub9e1\uc740 \ubc30\uc6b0\uac00 45\uba85\uc774\uc5c8\ub2e4. \uc640\uc77c\ub7ec \uac10\ub3c5\uc740 \ubc30\uc5ed\uc744 \uc124\uc815\ud560 \ub54c \uc678\ubaa8\ub098 \uacfc\uac70 \uc5f0\uae30 \uacbd\ub825\ubcf4\ub2e4\ub294 \uc778\ubb3c\uc758 \uc131\uaca9\uc744 \uc798 \ud45c\ud604\ud560 \uc218 \uc788\ub294 \ub2a5\ub825\uc5d0 \ub354 \ud070 \ube44\uc911\uc744 \ub480\uc73c\uba70 \uc791\uc911 \ub85c\ub9c8\uc778 \uc5ed\ud560\uc5d0\ub294 \uc601\uad6d\ucd9c\uc2e0 \ubc30\uc6b0\ub97c, \uc720\ub300\uc778 \uc5ed\ud560\uc5d0\ub294 \ubbf8\uad6d\ucd9c\uc2e0 \ubc30\uc6b0\ub97c \uc758\ub3c4\uc801\uc73c\ub85c \uad6c\ubcc4 \ubc30\uce58\ud558\uc5ec \uace0\ub300 \ub450 \ubbfc\uc871 \uc0ac\uc774\uc758 \ucc28\uc774\uc810\uc744 \uac15\uc870\ud558\uace0\uc790 \ud588\ub2e4. \uac10\ub3c5\uc774 \uc601\uad6d\uc778\uc744 \uace0\ub300 \ub85c\ub9c8\uc778 \uc5f0\uae30\uc790\ub85c \uc815\ud55c \uc774\uc720\ub294 \ubbf8\uad6d \uad00\uc911\ub4e4\uc774 \uc601\uad6d\uc2dd \ub9d0\ud22c\uac00 \uc0c1\ub958\uce35\uc774 \uc4f0\ub294 \ub2e8\uc5b4\ub77c\uace0 \uc0dd\uac01\ud558\ubbc0\ub85c \uc791\uc911 \ub85c\ub9c8\uc778\ub4e4\uc758 \uc2e0\ubd84\uc778 \uadc0\uc871 \ubc0f \uad8c\ub825\uc790 \uc124\uc815\uc5d0 \uc5b4\uc6b8\ub9b0\ub2e4\uace0 \uc0dd\uac01\ud588\uae30 \ub54c\ubb38\uc774\ub2e4.", "paragraph_answer": "1957\ub144 \uc911\uc21c MGM\uc740 \ub85c\ub9c8\uc5d0 \uce90\uc2a4\ud305 \uc0ac\ubb34\uc18c\ub97c \uc5f4\uc5b4 \uc870\uc5f0 \ubc0f \uc5d1\uc2a4\ud2b8\ub77c\ub85c \ud65c\uc57d\ud560 5\ub9cc \uba85\uc744 \uc120\uc815\ud588\ub294\ub370 \uc774 \uc911 \ub300\uc0ac\uac00 \uc788\ub294 \ubc30\uc6b0\ub294 365\uba85, \uadf8 \uc911\uc5d0\uc11c \ube44\uc911 \uc788\ub294 \uc5ed\ud560\uc744 \ub9e1\uc740 \ubc30\uc6b0\uac00 45\uba85\uc774\uc5c8\ub2e4. \uc640\uc77c\ub7ec \uac10\ub3c5\uc740 \ubc30\uc5ed\uc744 \uc124\uc815\ud560 \ub54c \uc678\ubaa8\ub098 \uacfc\uac70 \uc5f0\uae30 \uacbd\ub825\ubcf4\ub2e4\ub294 \uc778\ubb3c\uc758 \uc131\uaca9\uc744 \uc798 \ud45c\ud604\ud560 \uc218 \uc788\ub294 \ub2a5\ub825\uc5d0 \ub354 \ud070 \ube44\uc911\uc744 \ub480\uc73c\uba70 \uc791\uc911 \ub85c\ub9c8\uc778 \uc5ed\ud560\uc5d0\ub294 \uc601\uad6d\ucd9c\uc2e0 \ubc30\uc6b0\ub97c, \uc720\ub300\uc778 \uc5ed\ud560\uc5d0\ub294 \ubbf8\uad6d\ucd9c\uc2e0 \ubc30\uc6b0\ub97c \uc758\ub3c4\uc801\uc73c\ub85c \uad6c\ubcc4 \ubc30\uce58\ud558\uc5ec \uace0\ub300 \ub450 \ubbfc\uc871 \uc0ac\uc774\uc758 \ucc28\uc774\uc810\uc744 \uac15\uc870\ud558\uace0\uc790 \ud588\ub2e4. \uac10\ub3c5\uc774 \uc601\uad6d\uc778\uc744 \uace0\ub300 \ub85c\ub9c8\uc778 \uc5f0\uae30\uc790\ub85c \uc815\ud55c \uc774\uc720\ub294 \ubbf8\uad6d \uad00\uc911\ub4e4\uc774 \uc601\uad6d\uc2dd \ub9d0\ud22c\uac00 \uc0c1\ub958\uce35\uc774 \uc4f0\ub294 \ub2e8\uc5b4\ub77c\uace0 \uc0dd\uac01\ud558\ubbc0\ub85c \uc791\uc911 \ub85c\ub9c8\uc778\ub4e4\uc758 \uc2e0\ubd84\uc778 \uadc0\uc871 \ubc0f \uad8c\ub825\uc790 \uc124\uc815\uc5d0 \uc5b4\uc6b8\ub9b0\ub2e4\uace0 \uc0dd\uac01\ud588\uae30 \ub54c\ubb38\uc774\ub2e4.", "sentence_answer": "\uc640\uc77c\ub7ec \uac10\ub3c5\uc740 \ubc30\uc5ed\uc744 \uc124\uc815\ud560 \ub54c \uc678\ubaa8\ub098 \uacfc\uac70 \uc5f0\uae30 \uacbd\ub825\ubcf4\ub2e4\ub294 \uc778\ubb3c\uc758 \uc131\uaca9\uc744 \uc798 \ud45c\ud604\ud560 \uc218 \uc788\ub294 \ub2a5\ub825\uc5d0 \ub354 \ud070 \ube44\uc911\uc744 \ub480\uc73c\uba70 \uc791\uc911 \ub85c\ub9c8\uc778 \uc5ed\ud560\uc5d0\ub294 \uc601\uad6d\ucd9c\uc2e0 \ubc30\uc6b0\ub97c, \uc720\ub300\uc778 \uc5ed\ud560\uc5d0\ub294 \ubbf8\uad6d\ucd9c\uc2e0 \ubc30\uc6b0\ub97c \uc758\ub3c4\uc801\uc73c\ub85c \uad6c\ubcc4 \ubc30\uce58\ud558\uc5ec \uace0\ub300 \ub450 \ubbfc\uc871 \uc0ac\uc774\uc758 \ucc28\uc774\uc810\uc744 \uac15\uc870\ud558\uace0\uc790 \ud588\ub2e4.", "paragraph_id": "6516549-18-2", "paragraph_question": "question: \uc601\ud654 \ubca4\ud5c8 \ub9ac\uba54\uc774\ud06c\ud310\uc758 \uac10\ub3c5 \uc640\uc77c\ub7ec\uac00 \ubc30\uc5ed\uc744 \uc124\uc815\ud560 \ub54c \ubbf8\uad6d\ucd9c\uc2e0 \ubc30\uc6b0\ub97c \uae30\uc6a9\ud55c \uc5ed\ud560\uc740?, context: 1957\ub144 \uc911\uc21c MGM\uc740 \ub85c\ub9c8\uc5d0 \uce90\uc2a4\ud305 \uc0ac\ubb34\uc18c\ub97c \uc5f4\uc5b4 \uc870\uc5f0 \ubc0f \uc5d1\uc2a4\ud2b8\ub77c\ub85c \ud65c\uc57d\ud560 5\ub9cc \uba85\uc744 \uc120\uc815\ud588\ub294\ub370 \uc774 \uc911 \ub300\uc0ac\uac00 \uc788\ub294 \ubc30\uc6b0\ub294 365\uba85, \uadf8 \uc911\uc5d0\uc11c \ube44\uc911 \uc788\ub294 \uc5ed\ud560\uc744 \ub9e1\uc740 \ubc30\uc6b0\uac00 45\uba85\uc774\uc5c8\ub2e4. \uc640\uc77c\ub7ec \uac10\ub3c5\uc740 \ubc30\uc5ed\uc744 \uc124\uc815\ud560 \ub54c \uc678\ubaa8\ub098 \uacfc\uac70 \uc5f0\uae30 \uacbd\ub825\ubcf4\ub2e4\ub294 \uc778\ubb3c\uc758 \uc131\uaca9\uc744 \uc798 \ud45c\ud604\ud560 \uc218 \uc788\ub294 \ub2a5\ub825\uc5d0 \ub354 \ud070 \ube44\uc911\uc744 \ub480\uc73c\uba70 \uc791\uc911 \ub85c\ub9c8\uc778 \uc5ed\ud560\uc5d0\ub294 \uc601\uad6d\ucd9c\uc2e0 \ubc30\uc6b0\ub97c, \uc720\ub300\uc778 \uc5ed\ud560\uc5d0\ub294 \ubbf8\uad6d\ucd9c\uc2e0 \ubc30\uc6b0\ub97c \uc758\ub3c4\uc801\uc73c\ub85c \uad6c\ubcc4 \ubc30\uce58\ud558\uc5ec \uace0\ub300 \ub450 \ubbfc\uc871 \uc0ac\uc774\uc758 \ucc28\uc774\uc810\uc744 \uac15\uc870\ud558\uace0\uc790 \ud588\ub2e4. \uac10\ub3c5\uc774 \uc601\uad6d\uc778\uc744 \uace0\ub300 \ub85c\ub9c8\uc778 \uc5f0\uae30\uc790\ub85c \uc815\ud55c \uc774\uc720\ub294 \ubbf8\uad6d \uad00\uc911\ub4e4\uc774 \uc601\uad6d\uc2dd \ub9d0\ud22c\uac00 \uc0c1\ub958\uce35\uc774 \uc4f0\ub294 \ub2e8\uc5b4\ub77c\uace0 \uc0dd\uac01\ud558\ubbc0\ub85c \uc791\uc911 \ub85c\ub9c8\uc778\ub4e4\uc758 \uc2e0\ubd84\uc778 \uadc0\uc871 \ubc0f \uad8c\ub825\uc790 \uc124\uc815\uc5d0 \uc5b4\uc6b8\ub9b0\ub2e4\uace0 \uc0dd\uac01\ud588\uae30 \ub54c\ubb38\uc774\ub2e4."} {"question": "\uc468\uc6d0\uc774 \uc77c\ubcf8 \ub9dd\uba85 \ud6c4 \uc704\uc548\uc2a4\uce74\uc774 \uad70\ubc8c \uc815\ubd80\uc5d0 \uc800\ud56d\ud558\uace0\uc790 \uacb0\uc131\ud55c \ub2e8\uccb4\ub294?", "paragraph": "\uad6d\ubbfc\ub2f9\uc740 1913\ub144 1\uc6d4\uacfc 2\uc6d4\uc5d0 \uac78\uccd0 \uc2dc\ud589\ub41c \uad6d\ud68c\uc120\uac70\uc5d0\uc11c \ubbfc\uc911\ub4e4\uc758 \uc9c0\uc9c0\ub97c \ubc1b\uc544 \uacfc\ubc18\uc218\uc758 \uc758\uc11d\uc744 \ucc28\uc9c0\ud558\uc600\ub2e4. \uadf8\ub7ec\ub098 \uad6d\ubbfc\ub2f9\uc758 \ub0b4\uac01\uc744 \uaebc\ub9b0 \uc704\uc548\uc2a4\uce74\uc774\ub294 \uc554\uc0b4\ubc94\uc744 \ubcf4\ub0b4 3\uc6d4 20\uc77c \uad6d\ubb34\ucd1d\ub9ac\ub85c \uc608\uc815\ub41c \uc479\uc790\uc624\ub7f0\uc744 \uc800\uaca9, 3\uc6d4 22\uc77c \uc479\uc790\uc624\ub7f0\uc740 \uc228\uc744 \uac70\ub450\uc5c8\ub2e4. \uc704\uc548\uc2a4\uce74\uc774\ub294 \uc774\uc5b4 \uad6d\ud68c\uc758 \ubc18\ub300\ub97c \ubb34\ub985\uc4f0\uace0 1913\ub144 4\uc6d4 25\uc77c 5\uac1c\uad6d(\uc601\uad6d, \ub3c5\uc77c, \ud504\ub791\uc2a4, \uc77c\ubcf8, \ub7ec\uc2dc\uc544) \uc740\ud589\ub2e8\uc73c\ub85c\ubd80\ud130 2500\ub9cc \ud30c\uc6b4\ub4dc\uc5d0 \ub2ec\ud558\ub294 \uac70\uc561\uc758 \ucc28\uad00\uc744 \ub3c4\uc785\ud558\ub294 \ud611\uc815\uc744 \uccb4\uacb0\ud558\uc5ec \ub3c5\uc7ac\ub97c \uc704\ud55c \uc7ac\uc815\uae30\ubc18\uc744 \ud655\ubcf4\ud588\ub2e4. 6\uc6d4\uc5d0 \uc704\uc548\uc2a4\uce74\uc774\uac00 \uad6d\ubbfc\ub2f9 \uacc4\uc5f4\uc758 \ub3c4\ub3c5 3\uc778\uc744 \ud30c\uba74\ud558\uc790 \uc468\uc6d0, \ucc9c\uce58\uba54\uc774\ub4f1\uc744 \ube44\ub86f\ud55c \uad6d\ubbfc\ub2f9 \uc138\ub825\uc774 \ubd09\uae30(\uc81c2\ucc28 \ud601\uba85)\ub97c \ud638\uc18c\ud588\uc73c\uba70, 1913\ub144 6\uc6d4 9\uc77c \ud55c \ubd09\uae30\ub294 6\uc6d4 9\uc77c\uc5d0\uc11c 7\uc6d4 12\uc77c\uae4c\uc9c0 \uc591\uc790 \uac15 \uc911\ub958\uc640 \ud558\ub958 \uc9c0\uc5ed \uc77c\ub300\uc5d0\uc11c \uc804\ud22c\uac00 \uc77c\uc5b4\ub0ac\uc73c\ub098\ub098 \ub2f9\uc2dc \uc804\ub780 \uc7ac\ubc1c\uc744 \ub2ec\uac00\uc6cc\ud558\uc9c0 \uc54a\ub294 \ubd84\uc704\uae30\uc5d0\uc11c \ud06c\uac8c \ud638\uc751\ubc1b\uc9c0 \ubabb\ud588\uace0 1913\ub144 9\uc6d4 \uc704\uc548\uc2a4\uce74\uc774\uc758 \uc555\ub3c4\uc801\uc778 \ubb34\ub825\uc5d0 \uc644\ubcbd\ud788 \uacb0\uad6d \uc9c4\uc555\ub410\uace0, \uc468\uc6d0\uc740 7\uc6d4 \uc77c\ubcf8 \ub3c4\ucfc4\ub85c \ub9dd\uba85\ud588\ub2e4. \uac70\uae30\uc11c \uc468\uc6d0\uc740 1914\ub144 7\uc6d4\uc5d0 \u2018\uc911\ud654 \ud601\uba85\ub2f9\u2019\uc744 \uacb0\uc131\ud574 \uad6d\uc678\uc5d0\uc11c \uc704\uc548\uc2a4\uce74\uc774 \uad70\ubc8c \uc815\ubd80\uc5d0 \uc800\ud56d\ud588\ub2e4. 10\uc6d4\uc5d0 \uad6d\ud68c\ub294 \u2018\uacf5\ubbfc\ub2e8\u2019\uc774\ub77c \uc790\uce6d\ud558\ub294 \ud3ed\ub3c4\uc5d0\uac8c \ud611\ubc15\ubc1b\ub294 \uac00\uc6b4\ub370 \uc704\uc548\uc2a4\uce74\uc774\ub97c \uc815\uc2dd \ub300\ucd1d\ud1b5\uc73c\ub85c \uc120\ucd9c\ud588\ub2e4. \uace7 \uc774\uc5b4 \uc704\uc548\uc2a4\uce74\uc774\ub294 \uad6d\ubbfc\ub2f9 \ud574\uc0b0\uc744 \uba85\ud558\uace0 \ub300\ucd1d\ud1b5 \uad8c\ud55c\uc744 \ub300\ud3ed \uac15\ud654\uc2dc\ud0a8 \u2018\uc2e0\uc57d\ubc95\u2019\uc744 \uc81c\uc815\ud588\ub2e4. \uacb0\uad6d \u2018\uc704\uc548\uc2a4\uce74\uc774\ub97c \ubc30\ub824\ud55c \ud601\uba85\u2019\uc774 \ub418\uc5b4\ubc84\ub838\ub2e4. \uc774\ud6c4 \uc77c\ubcf8\uc774 \uc704\uc548\uc2a4\uce74\uc774\uc5d0\uac8c \u201c\ud669\uc81c \ub4f1\uadf9\uc744 \uc778\uc815\ud574 \uc8fc\ub294 \ub300\uac00\ub85c \u201821\uac1c\uc870 \uc694\uad6c\u2019\ub97c \ub4e4\uc5b4\ub2ec\ub77c\u201d\uace0 \uc694\uad6c\ud558\uc790 \u2018\ud669\uc81c \ub4f1\uadf9\u2019\uc744 \uc6d0\ud588\ub358 \uc704\uc548\uc2a4\uce74\uc774\ub294 \uc774\ub97c \uc218\uc6a9 \ud558\uc5ec 1915\ub144 12\uc6d4 \u2018\ud669\uc81c \uc81c\ub3c4\u2019\uc758 \ubd80\ud65c\uc744 \uc2dc\ub3c4\ud588\ub2e4\uac00 \uad70\uc911\uc5d0\uac8c \uc9c0\ud0c4\ubc1b\uc544 \uc2e4\ud328\ud558\uace0(\uc81c3\ucc28 \ud601\uba85), 1916\ub144 6\uc6d4 \uc704\uc548\uc2a4\uce74\uc774\ub294 \uc0ac\ub9dd\ud588\ub2e4.", "answer": "\uc911\ud654 \ud601\uba85\ub2f9", "sentence": "\uac70\uae30\uc11c \uc468\uc6d0\uc740 1914\ub144 7\uc6d4\uc5d0 \u2018 \uc911\ud654 \ud601\uba85\ub2f9 \u2019\uc744 \uacb0\uc131\ud574 \uad6d\uc678\uc5d0\uc11c \uc704\uc548\uc2a4\uce74\uc774 \uad70\ubc8c \uc815\ubd80\uc5d0 \uc800\ud56d\ud588\ub2e4.", "paragraph_sentence": "\uad6d\ubbfc\ub2f9\uc740 1913\ub144 1\uc6d4\uacfc 2\uc6d4\uc5d0 \uac78\uccd0 \uc2dc\ud589\ub41c \uad6d\ud68c\uc120\uac70\uc5d0\uc11c \ubbfc\uc911\ub4e4\uc758 \uc9c0\uc9c0\ub97c \ubc1b\uc544 \uacfc\ubc18\uc218\uc758 \uc758\uc11d\uc744 \ucc28\uc9c0\ud558\uc600\ub2e4. \uadf8\ub7ec\ub098 \uad6d\ubbfc\ub2f9\uc758 \ub0b4\uac01\uc744 \uaebc\ub9b0 \uc704\uc548\uc2a4\uce74\uc774\ub294 \uc554\uc0b4\ubc94\uc744 \ubcf4\ub0b4 3\uc6d4 20\uc77c \uad6d\ubb34\ucd1d\ub9ac\ub85c \uc608\uc815\ub41c \uc479\uc790\uc624\ub7f0\uc744 \uc800\uaca9, 3\uc6d4 22\uc77c \uc479\uc790\uc624\ub7f0\uc740 \uc228\uc744 \uac70\ub450\uc5c8\ub2e4. \uc704\uc548\uc2a4\uce74\uc774\ub294 \uc774\uc5b4 \uad6d\ud68c\uc758 \ubc18\ub300\ub97c \ubb34\ub985\uc4f0\uace0 1913\ub144 4\uc6d4 25\uc77c 5\uac1c\uad6d(\uc601\uad6d, \ub3c5\uc77c, \ud504\ub791\uc2a4, \uc77c\ubcf8, \ub7ec\uc2dc\uc544) \uc740\ud589\ub2e8\uc73c\ub85c\ubd80\ud130 2500\ub9cc \ud30c\uc6b4\ub4dc\uc5d0 \ub2ec\ud558\ub294 \uac70\uc561\uc758 \ucc28\uad00\uc744 \ub3c4\uc785\ud558\ub294 \ud611\uc815\uc744 \uccb4\uacb0\ud558\uc5ec \ub3c5\uc7ac\ub97c \uc704\ud55c \uc7ac\uc815\uae30\ubc18\uc744 \ud655\ubcf4\ud588\ub2e4. 6\uc6d4\uc5d0 \uc704\uc548\uc2a4\uce74\uc774\uac00 \uad6d\ubbfc\ub2f9 \uacc4\uc5f4\uc758 \ub3c4\ub3c5 3\uc778\uc744 \ud30c\uba74\ud558\uc790 \uc468\uc6d0, \ucc9c\uce58\uba54\uc774\ub4f1\uc744 \ube44\ub86f\ud55c \uad6d\ubbfc\ub2f9 \uc138\ub825\uc774 \ubd09\uae30(\uc81c2\ucc28 \ud601\uba85)\ub97c \ud638\uc18c\ud588\uc73c\uba70, 1913\ub144 6\uc6d4 9\uc77c \ud55c \ubd09\uae30\ub294 6\uc6d4 9\uc77c\uc5d0\uc11c 7\uc6d4 12\uc77c\uae4c\uc9c0 \uc591\uc790 \uac15 \uc911\ub958\uc640 \ud558\ub958 \uc9c0\uc5ed \uc77c\ub300\uc5d0\uc11c \uc804\ud22c\uac00 \uc77c\uc5b4\ub0ac\uc73c\ub098\ub098 \ub2f9\uc2dc \uc804\ub780 \uc7ac\ubc1c\uc744 \ub2ec\uac00\uc6cc\ud558\uc9c0 \uc54a\ub294 \ubd84\uc704\uae30\uc5d0\uc11c \ud06c\uac8c \ud638\uc751\ubc1b\uc9c0 \ubabb\ud588\uace0 1913\ub144 9\uc6d4 \uc704\uc548\uc2a4\uce74\uc774\uc758 \uc555\ub3c4\uc801\uc778 \ubb34\ub825\uc5d0 \uc644\ubcbd\ud788 \uacb0\uad6d \uc9c4\uc555\ub410\uace0, \uc468\uc6d0\uc740 7\uc6d4 \uc77c\ubcf8 \ub3c4\ucfc4\ub85c \ub9dd\uba85\ud588\ub2e4. \uac70\uae30\uc11c \uc468\uc6d0\uc740 1914\ub144 7\uc6d4\uc5d0 \u2018 \uc911\ud654 \ud601\uba85\ub2f9 \u2019\uc744 \uacb0\uc131\ud574 \uad6d\uc678\uc5d0\uc11c \uc704\uc548\uc2a4\uce74\uc774 \uad70\ubc8c \uc815\ubd80\uc5d0 \uc800\ud56d\ud588\ub2e4. 10\uc6d4\uc5d0 \uad6d\ud68c\ub294 \u2018\uacf5\ubbfc\ub2e8\u2019\uc774\ub77c \uc790\uce6d\ud558\ub294 \ud3ed\ub3c4\uc5d0\uac8c \ud611\ubc15\ubc1b\ub294 \uac00\uc6b4\ub370 \uc704\uc548\uc2a4\uce74\uc774\ub97c \uc815\uc2dd \ub300\ucd1d\ud1b5\uc73c\ub85c \uc120\ucd9c\ud588\ub2e4. \uace7 \uc774\uc5b4 \uc704\uc548\uc2a4\uce74\uc774\ub294 \uad6d\ubbfc\ub2f9 \ud574\uc0b0\uc744 \uba85\ud558\uace0 \ub300\ucd1d\ud1b5 \uad8c\ud55c\uc744 \ub300\ud3ed \uac15\ud654\uc2dc\ud0a8 \u2018\uc2e0\uc57d\ubc95\u2019\uc744 \uc81c\uc815\ud588\ub2e4. \uacb0\uad6d \u2018\uc704\uc548\uc2a4\uce74\uc774\ub97c \ubc30\ub824\ud55c \ud601\uba85\u2019\uc774 \ub418\uc5b4\ubc84\ub838\ub2e4. \uc774\ud6c4 \uc77c\ubcf8\uc774 \uc704\uc548\uc2a4\uce74\uc774\uc5d0\uac8c \u201c\ud669\uc81c \ub4f1\uadf9\uc744 \uc778\uc815\ud574 \uc8fc\ub294 \ub300\uac00\ub85c \u201821\uac1c\uc870 \uc694\uad6c\u2019\ub97c \ub4e4\uc5b4\ub2ec\ub77c\u201d\uace0 \uc694\uad6c\ud558\uc790 \u2018\ud669\uc81c \ub4f1\uadf9\u2019\uc744 \uc6d0\ud588\ub358 \uc704\uc548\uc2a4\uce74\uc774\ub294 \uc774\ub97c \uc218\uc6a9 \ud558\uc5ec 1915\ub144 12\uc6d4 \u2018\ud669\uc81c \uc81c\ub3c4\u2019\uc758 \ubd80\ud65c\uc744 \uc2dc\ub3c4\ud588\ub2e4\uac00 \uad70\uc911\uc5d0\uac8c \uc9c0\ud0c4\ubc1b\uc544 \uc2e4\ud328\ud558\uace0(\uc81c3\ucc28 \ud601\uba85), 1916\ub144 6\uc6d4 \uc704\uc548\uc2a4\uce74\uc774\ub294 \uc0ac\ub9dd\ud588\ub2e4.", "paragraph_answer": "\uad6d\ubbfc\ub2f9\uc740 1913\ub144 1\uc6d4\uacfc 2\uc6d4\uc5d0 \uac78\uccd0 \uc2dc\ud589\ub41c \uad6d\ud68c\uc120\uac70\uc5d0\uc11c \ubbfc\uc911\ub4e4\uc758 \uc9c0\uc9c0\ub97c \ubc1b\uc544 \uacfc\ubc18\uc218\uc758 \uc758\uc11d\uc744 \ucc28\uc9c0\ud558\uc600\ub2e4. \uadf8\ub7ec\ub098 \uad6d\ubbfc\ub2f9\uc758 \ub0b4\uac01\uc744 \uaebc\ub9b0 \uc704\uc548\uc2a4\uce74\uc774\ub294 \uc554\uc0b4\ubc94\uc744 \ubcf4\ub0b4 3\uc6d4 20\uc77c \uad6d\ubb34\ucd1d\ub9ac\ub85c \uc608\uc815\ub41c \uc479\uc790\uc624\ub7f0\uc744 \uc800\uaca9, 3\uc6d4 22\uc77c \uc479\uc790\uc624\ub7f0\uc740 \uc228\uc744 \uac70\ub450\uc5c8\ub2e4. \uc704\uc548\uc2a4\uce74\uc774\ub294 \uc774\uc5b4 \uad6d\ud68c\uc758 \ubc18\ub300\ub97c \ubb34\ub985\uc4f0\uace0 1913\ub144 4\uc6d4 25\uc77c 5\uac1c\uad6d(\uc601\uad6d, \ub3c5\uc77c, \ud504\ub791\uc2a4, \uc77c\ubcf8, \ub7ec\uc2dc\uc544) \uc740\ud589\ub2e8\uc73c\ub85c\ubd80\ud130 2500\ub9cc \ud30c\uc6b4\ub4dc\uc5d0 \ub2ec\ud558\ub294 \uac70\uc561\uc758 \ucc28\uad00\uc744 \ub3c4\uc785\ud558\ub294 \ud611\uc815\uc744 \uccb4\uacb0\ud558\uc5ec \ub3c5\uc7ac\ub97c \uc704\ud55c \uc7ac\uc815\uae30\ubc18\uc744 \ud655\ubcf4\ud588\ub2e4. 6\uc6d4\uc5d0 \uc704\uc548\uc2a4\uce74\uc774\uac00 \uad6d\ubbfc\ub2f9 \uacc4\uc5f4\uc758 \ub3c4\ub3c5 3\uc778\uc744 \ud30c\uba74\ud558\uc790 \uc468\uc6d0, \ucc9c\uce58\uba54\uc774\ub4f1\uc744 \ube44\ub86f\ud55c \uad6d\ubbfc\ub2f9 \uc138\ub825\uc774 \ubd09\uae30(\uc81c2\ucc28 \ud601\uba85)\ub97c \ud638\uc18c\ud588\uc73c\uba70, 1913\ub144 6\uc6d4 9\uc77c \ud55c \ubd09\uae30\ub294 6\uc6d4 9\uc77c\uc5d0\uc11c 7\uc6d4 12\uc77c\uae4c\uc9c0 \uc591\uc790 \uac15 \uc911\ub958\uc640 \ud558\ub958 \uc9c0\uc5ed \uc77c\ub300\uc5d0\uc11c \uc804\ud22c\uac00 \uc77c\uc5b4\ub0ac\uc73c\ub098\ub098 \ub2f9\uc2dc \uc804\ub780 \uc7ac\ubc1c\uc744 \ub2ec\uac00\uc6cc\ud558\uc9c0 \uc54a\ub294 \ubd84\uc704\uae30\uc5d0\uc11c \ud06c\uac8c \ud638\uc751\ubc1b\uc9c0 \ubabb\ud588\uace0 1913\ub144 9\uc6d4 \uc704\uc548\uc2a4\uce74\uc774\uc758 \uc555\ub3c4\uc801\uc778 \ubb34\ub825\uc5d0 \uc644\ubcbd\ud788 \uacb0\uad6d \uc9c4\uc555\ub410\uace0, \uc468\uc6d0\uc740 7\uc6d4 \uc77c\ubcf8 \ub3c4\ucfc4\ub85c \ub9dd\uba85\ud588\ub2e4. \uac70\uae30\uc11c \uc468\uc6d0\uc740 1914\ub144 7\uc6d4\uc5d0 \u2018 \uc911\ud654 \ud601\uba85\ub2f9 \u2019\uc744 \uacb0\uc131\ud574 \uad6d\uc678\uc5d0\uc11c \uc704\uc548\uc2a4\uce74\uc774 \uad70\ubc8c \uc815\ubd80\uc5d0 \uc800\ud56d\ud588\ub2e4. 10\uc6d4\uc5d0 \uad6d\ud68c\ub294 \u2018\uacf5\ubbfc\ub2e8\u2019\uc774\ub77c \uc790\uce6d\ud558\ub294 \ud3ed\ub3c4\uc5d0\uac8c \ud611\ubc15\ubc1b\ub294 \uac00\uc6b4\ub370 \uc704\uc548\uc2a4\uce74\uc774\ub97c \uc815\uc2dd \ub300\ucd1d\ud1b5\uc73c\ub85c \uc120\ucd9c\ud588\ub2e4. \uace7 \uc774\uc5b4 \uc704\uc548\uc2a4\uce74\uc774\ub294 \uad6d\ubbfc\ub2f9 \ud574\uc0b0\uc744 \uba85\ud558\uace0 \ub300\ucd1d\ud1b5 \uad8c\ud55c\uc744 \ub300\ud3ed \uac15\ud654\uc2dc\ud0a8 \u2018\uc2e0\uc57d\ubc95\u2019\uc744 \uc81c\uc815\ud588\ub2e4. \uacb0\uad6d \u2018\uc704\uc548\uc2a4\uce74\uc774\ub97c \ubc30\ub824\ud55c \ud601\uba85\u2019\uc774 \ub418\uc5b4\ubc84\ub838\ub2e4. \uc774\ud6c4 \uc77c\ubcf8\uc774 \uc704\uc548\uc2a4\uce74\uc774\uc5d0\uac8c \u201c\ud669\uc81c \ub4f1\uadf9\uc744 \uc778\uc815\ud574 \uc8fc\ub294 \ub300\uac00\ub85c \u201821\uac1c\uc870 \uc694\uad6c\u2019\ub97c \ub4e4\uc5b4\ub2ec\ub77c\u201d\uace0 \uc694\uad6c\ud558\uc790 \u2018\ud669\uc81c \ub4f1\uadf9\u2019\uc744 \uc6d0\ud588\ub358 \uc704\uc548\uc2a4\uce74\uc774\ub294 \uc774\ub97c \uc218\uc6a9 \ud558\uc5ec 1915\ub144 12\uc6d4 \u2018\ud669\uc81c \uc81c\ub3c4\u2019\uc758 \ubd80\ud65c\uc744 \uc2dc\ub3c4\ud588\ub2e4\uac00 \uad70\uc911\uc5d0\uac8c \uc9c0\ud0c4\ubc1b\uc544 \uc2e4\ud328\ud558\uace0(\uc81c3\ucc28 \ud601\uba85), 1916\ub144 6\uc6d4 \uc704\uc548\uc2a4\uce74\uc774\ub294 \uc0ac\ub9dd\ud588\ub2e4.", "sentence_answer": "\uac70\uae30\uc11c \uc468\uc6d0\uc740 1914\ub144 7\uc6d4\uc5d0 \u2018 \uc911\ud654 \ud601\uba85\ub2f9 \u2019\uc744 \uacb0\uc131\ud574 \uad6d\uc678\uc5d0\uc11c \uc704\uc548\uc2a4\uce74\uc774 \uad70\ubc8c \uc815\ubd80\uc5d0 \uc800\ud56d\ud588\ub2e4.", "paragraph_id": "6533378-21-2", "paragraph_question": "question: \uc468\uc6d0\uc774 \uc77c\ubcf8 \ub9dd\uba85 \ud6c4 \uc704\uc548\uc2a4\uce74\uc774 \uad70\ubc8c \uc815\ubd80\uc5d0 \uc800\ud56d\ud558\uace0\uc790 \uacb0\uc131\ud55c \ub2e8\uccb4\ub294?, context: \uad6d\ubbfc\ub2f9\uc740 1913\ub144 1\uc6d4\uacfc 2\uc6d4\uc5d0 \uac78\uccd0 \uc2dc\ud589\ub41c \uad6d\ud68c\uc120\uac70\uc5d0\uc11c \ubbfc\uc911\ub4e4\uc758 \uc9c0\uc9c0\ub97c \ubc1b\uc544 \uacfc\ubc18\uc218\uc758 \uc758\uc11d\uc744 \ucc28\uc9c0\ud558\uc600\ub2e4. \uadf8\ub7ec\ub098 \uad6d\ubbfc\ub2f9\uc758 \ub0b4\uac01\uc744 \uaebc\ub9b0 \uc704\uc548\uc2a4\uce74\uc774\ub294 \uc554\uc0b4\ubc94\uc744 \ubcf4\ub0b4 3\uc6d4 20\uc77c \uad6d\ubb34\ucd1d\ub9ac\ub85c \uc608\uc815\ub41c \uc479\uc790\uc624\ub7f0\uc744 \uc800\uaca9, 3\uc6d4 22\uc77c \uc479\uc790\uc624\ub7f0\uc740 \uc228\uc744 \uac70\ub450\uc5c8\ub2e4. \uc704\uc548\uc2a4\uce74\uc774\ub294 \uc774\uc5b4 \uad6d\ud68c\uc758 \ubc18\ub300\ub97c \ubb34\ub985\uc4f0\uace0 1913\ub144 4\uc6d4 25\uc77c 5\uac1c\uad6d(\uc601\uad6d, \ub3c5\uc77c, \ud504\ub791\uc2a4, \uc77c\ubcf8, \ub7ec\uc2dc\uc544) \uc740\ud589\ub2e8\uc73c\ub85c\ubd80\ud130 2500\ub9cc \ud30c\uc6b4\ub4dc\uc5d0 \ub2ec\ud558\ub294 \uac70\uc561\uc758 \ucc28\uad00\uc744 \ub3c4\uc785\ud558\ub294 \ud611\uc815\uc744 \uccb4\uacb0\ud558\uc5ec \ub3c5\uc7ac\ub97c \uc704\ud55c \uc7ac\uc815\uae30\ubc18\uc744 \ud655\ubcf4\ud588\ub2e4. 6\uc6d4\uc5d0 \uc704\uc548\uc2a4\uce74\uc774\uac00 \uad6d\ubbfc\ub2f9 \uacc4\uc5f4\uc758 \ub3c4\ub3c5 3\uc778\uc744 \ud30c\uba74\ud558\uc790 \uc468\uc6d0, \ucc9c\uce58\uba54\uc774\ub4f1\uc744 \ube44\ub86f\ud55c \uad6d\ubbfc\ub2f9 \uc138\ub825\uc774 \ubd09\uae30(\uc81c2\ucc28 \ud601\uba85)\ub97c \ud638\uc18c\ud588\uc73c\uba70, 1913\ub144 6\uc6d4 9\uc77c \ud55c \ubd09\uae30\ub294 6\uc6d4 9\uc77c\uc5d0\uc11c 7\uc6d4 12\uc77c\uae4c\uc9c0 \uc591\uc790 \uac15 \uc911\ub958\uc640 \ud558\ub958 \uc9c0\uc5ed \uc77c\ub300\uc5d0\uc11c \uc804\ud22c\uac00 \uc77c\uc5b4\ub0ac\uc73c\ub098\ub098 \ub2f9\uc2dc \uc804\ub780 \uc7ac\ubc1c\uc744 \ub2ec\uac00\uc6cc\ud558\uc9c0 \uc54a\ub294 \ubd84\uc704\uae30\uc5d0\uc11c \ud06c\uac8c \ud638\uc751\ubc1b\uc9c0 \ubabb\ud588\uace0 1913\ub144 9\uc6d4 \uc704\uc548\uc2a4\uce74\uc774\uc758 \uc555\ub3c4\uc801\uc778 \ubb34\ub825\uc5d0 \uc644\ubcbd\ud788 \uacb0\uad6d \uc9c4\uc555\ub410\uace0, \uc468\uc6d0\uc740 7\uc6d4 \uc77c\ubcf8 \ub3c4\ucfc4\ub85c \ub9dd\uba85\ud588\ub2e4. \uac70\uae30\uc11c \uc468\uc6d0\uc740 1914\ub144 7\uc6d4\uc5d0 \u2018\uc911\ud654 \ud601\uba85\ub2f9\u2019\uc744 \uacb0\uc131\ud574 \uad6d\uc678\uc5d0\uc11c \uc704\uc548\uc2a4\uce74\uc774 \uad70\ubc8c \uc815\ubd80\uc5d0 \uc800\ud56d\ud588\ub2e4. 10\uc6d4\uc5d0 \uad6d\ud68c\ub294 \u2018\uacf5\ubbfc\ub2e8\u2019\uc774\ub77c \uc790\uce6d\ud558\ub294 \ud3ed\ub3c4\uc5d0\uac8c \ud611\ubc15\ubc1b\ub294 \uac00\uc6b4\ub370 \uc704\uc548\uc2a4\uce74\uc774\ub97c \uc815\uc2dd \ub300\ucd1d\ud1b5\uc73c\ub85c \uc120\ucd9c\ud588\ub2e4. \uace7 \uc774\uc5b4 \uc704\uc548\uc2a4\uce74\uc774\ub294 \uad6d\ubbfc\ub2f9 \ud574\uc0b0\uc744 \uba85\ud558\uace0 \ub300\ucd1d\ud1b5 \uad8c\ud55c\uc744 \ub300\ud3ed \uac15\ud654\uc2dc\ud0a8 \u2018\uc2e0\uc57d\ubc95\u2019\uc744 \uc81c\uc815\ud588\ub2e4. \uacb0\uad6d \u2018\uc704\uc548\uc2a4\uce74\uc774\ub97c \ubc30\ub824\ud55c \ud601\uba85\u2019\uc774 \ub418\uc5b4\ubc84\ub838\ub2e4. \uc774\ud6c4 \uc77c\ubcf8\uc774 \uc704\uc548\uc2a4\uce74\uc774\uc5d0\uac8c \u201c\ud669\uc81c \ub4f1\uadf9\uc744 \uc778\uc815\ud574 \uc8fc\ub294 \ub300\uac00\ub85c \u201821\uac1c\uc870 \uc694\uad6c\u2019\ub97c \ub4e4\uc5b4\ub2ec\ub77c\u201d\uace0 \uc694\uad6c\ud558\uc790 \u2018\ud669\uc81c \ub4f1\uadf9\u2019\uc744 \uc6d0\ud588\ub358 \uc704\uc548\uc2a4\uce74\uc774\ub294 \uc774\ub97c \uc218\uc6a9 \ud558\uc5ec 1915\ub144 12\uc6d4 \u2018\ud669\uc81c \uc81c\ub3c4\u2019\uc758 \ubd80\ud65c\uc744 \uc2dc\ub3c4\ud588\ub2e4\uac00 \uad70\uc911\uc5d0\uac8c \uc9c0\ud0c4\ubc1b\uc544 \uc2e4\ud328\ud558\uace0(\uc81c3\ucc28 \ud601\uba85), 1916\ub144 6\uc6d4 \uc704\uc548\uc2a4\uce74\uc774\ub294 \uc0ac\ub9dd\ud588\ub2e4."} {"question": "\ud504\ub791\uc2a4\uac00 \ubbf8\uad6d\uacfc \ub3d9\ub9f9\uc744 \ub9fa\uace0 \ubbf8\uad6d \ub3c5\ub9bd \uc804\uc7c1\uc5d0 \ucc38\uc804\ud55c \ud574\ub294?", "paragraph": "\ubc38\ub9ac \ud3ec\uc9c0(Valley Forge)\ub294 \ubbf8\uad6d \ub3c5\ub9bd \uc804\uc7c1 \uc911\uc778 1777\ub144\uc5d0\uc11c 1778\ub144 \uc0ac\uc774\uc5d0 \ub300\ub959\uad70\uc774 \ub3d9\uacc4\uc5d0 \uc8fc\ub454\ud55c \uacf3\uc73c\ub85c \ud544\ub77c\ub378\ud53c\uc544\uc5d0\uc11c \ubd81\uc11c\ucabd\uc73c\ub85c \uc57d 20\ub9c8\uc77c \ub5a8\uc5b4\uc9c4 \ud39c\uc2e4\ubca0\uc774\ub2c8\uc544\uc5d0 \uc704\uce58\ud558\uace0 \uc788\ub2e4. \uadf8\ub54c\uae4c\uc9c0 \ub300\ub959\ud68c\uc758\uc758 \uac1c\ucd5c \uc7a5\uc18c\uc774\uc790, \uc2e4\uc9c8\uc801\uc778 \uc218\ub3c4\uc758 \uc5ed\ud560\uc744 \ud588\ub358 \ud544\ub77c\ub378\ud53c\uc544\uac00 \uc601\uad6d\uad70\uc5d0 \uc810\ub839\ub2f9\ud558\uace0, \uadf8 \uaca8\uc6b8 \uc870\uc9c0 \uc6cc\uc2f1\ud134 \uc7a5\uad70\uc740 \uac00\uc7a5 \uc5b4\ub824\uc6b4 \uc0c1\ud669\uc5d0 \ub193\uc5ec \uc788\uc5c8\uc9c0\ub9cc, \uad70\ub300 \uc7ac\uad50\uc721\uc744 \ud65c\uc131\ud654\uc2dc\ud0a4\ub294 \uc2dc\uae30\uac00 \ub418\uae30\ub3c4 \ud558\uc600\ub2e4. 1778\ub144 \ud504\ub791\uc2a4\uac00 \ubbf8\uad6d\uacfc \ub3d9\ub9f9\uc744 \ub9fa\uace0 \ucc38\uc804\ud55c \ud574\uc9c0\ub9cc, \uc774\ud6c4 \uba87 \ub144 \ub3d9\uc548 \ubbf8\uad6d \ub3c5\ub9bd \uc804\uc7c1\uc740 \uc9c0\ub9ac\ud55c \uc18c\ubaa8\uc804 \uc591\uc0c1\uc744 \ubcf4\uc600\ub2e4. \uad76\uc8fc\ub9bc\uacfc \uc9c8\ubcd1 \uadf8\ub9ac\uace0 \ub3d9\uc0c1\uc73c\ub85c 1778\ub144 2\uc6d4\ub9d0\uacbd\uc5d0\ub294 \uac70\uc758 2,500\uba85\uc758 \ubbf8\ud569\uc911\uad6d \uad70\uc778\ub4e4\uc774 \uc0ac\ub9dd\ud588\ub2e4.", "answer": "1778\ub144", "sentence": "\ubc38\ub9ac \ud3ec\uc9c0(Valley Forge)\ub294 \ubbf8\uad6d \ub3c5\ub9bd \uc804\uc7c1 \uc911\uc778 1777\ub144\uc5d0\uc11c 1778\ub144 \uc0ac\uc774\uc5d0 \ub300\ub959\uad70\uc774 \ub3d9\uacc4\uc5d0 \uc8fc\ub454\ud55c \uacf3\uc73c\ub85c \ud544\ub77c\ub378\ud53c\uc544\uc5d0\uc11c \ubd81\uc11c\ucabd\uc73c\ub85c \uc57d 20\ub9c8\uc77c \ub5a8\uc5b4\uc9c4 \ud39c\uc2e4\ubca0\uc774\ub2c8\uc544\uc5d0 \uc704\uce58\ud558\uace0 \uc788\ub2e4.", "paragraph_sentence": " \ubc38\ub9ac \ud3ec\uc9c0(Valley Forge)\ub294 \ubbf8\uad6d \ub3c5\ub9bd \uc804\uc7c1 \uc911\uc778 1777\ub144\uc5d0\uc11c 1778\ub144 \uc0ac\uc774\uc5d0 \ub300\ub959\uad70\uc774 \ub3d9\uacc4\uc5d0 \uc8fc\ub454\ud55c \uacf3\uc73c\ub85c \ud544\ub77c\ub378\ud53c\uc544\uc5d0\uc11c \ubd81\uc11c\ucabd\uc73c\ub85c \uc57d 20\ub9c8\uc77c \ub5a8\uc5b4\uc9c4 \ud39c\uc2e4\ubca0\uc774\ub2c8\uc544\uc5d0 \uc704\uce58\ud558\uace0 \uc788\ub2e4. \uadf8\ub54c\uae4c\uc9c0 \ub300\ub959\ud68c\uc758\uc758 \uac1c\ucd5c \uc7a5\uc18c\uc774\uc790, \uc2e4\uc9c8\uc801\uc778 \uc218\ub3c4\uc758 \uc5ed\ud560\uc744 \ud588\ub358 \ud544\ub77c\ub378\ud53c\uc544\uac00 \uc601\uad6d\uad70\uc5d0 \uc810\ub839\ub2f9\ud558\uace0, \uadf8 \uaca8\uc6b8 \uc870\uc9c0 \uc6cc\uc2f1\ud134 \uc7a5\uad70\uc740 \uac00\uc7a5 \uc5b4\ub824\uc6b4 \uc0c1\ud669\uc5d0 \ub193\uc5ec \uc788\uc5c8\uc9c0\ub9cc, \uad70\ub300 \uc7ac\uad50\uc721\uc744 \ud65c\uc131\ud654\uc2dc\ud0a4\ub294 \uc2dc\uae30\uac00 \ub418\uae30\ub3c4 \ud558\uc600\ub2e4. 1778\ub144 \ud504\ub791\uc2a4\uac00 \ubbf8\uad6d\uacfc \ub3d9\ub9f9\uc744 \ub9fa\uace0 \ucc38\uc804\ud55c \ud574\uc9c0\ub9cc, \uc774\ud6c4 \uba87 \ub144 \ub3d9\uc548 \ubbf8\uad6d \ub3c5\ub9bd \uc804\uc7c1\uc740 \uc9c0\ub9ac\ud55c \uc18c\ubaa8\uc804 \uc591\uc0c1\uc744 \ubcf4\uc600\ub2e4. \uad76\uc8fc\ub9bc\uacfc \uc9c8\ubcd1 \uadf8\ub9ac\uace0 \ub3d9\uc0c1\uc73c\ub85c 1778\ub144 2\uc6d4\ub9d0\uacbd\uc5d0\ub294 \uac70\uc758 2,500\uba85\uc758 \ubbf8\ud569\uc911\uad6d \uad70\uc778\ub4e4\uc774 \uc0ac\ub9dd\ud588\ub2e4.", "paragraph_answer": "\ubc38\ub9ac \ud3ec\uc9c0(Valley Forge)\ub294 \ubbf8\uad6d \ub3c5\ub9bd \uc804\uc7c1 \uc911\uc778 1777\ub144\uc5d0\uc11c 1778\ub144 \uc0ac\uc774\uc5d0 \ub300\ub959\uad70\uc774 \ub3d9\uacc4\uc5d0 \uc8fc\ub454\ud55c \uacf3\uc73c\ub85c \ud544\ub77c\ub378\ud53c\uc544\uc5d0\uc11c \ubd81\uc11c\ucabd\uc73c\ub85c \uc57d 20\ub9c8\uc77c \ub5a8\uc5b4\uc9c4 \ud39c\uc2e4\ubca0\uc774\ub2c8\uc544\uc5d0 \uc704\uce58\ud558\uace0 \uc788\ub2e4. \uadf8\ub54c\uae4c\uc9c0 \ub300\ub959\ud68c\uc758\uc758 \uac1c\ucd5c \uc7a5\uc18c\uc774\uc790, \uc2e4\uc9c8\uc801\uc778 \uc218\ub3c4\uc758 \uc5ed\ud560\uc744 \ud588\ub358 \ud544\ub77c\ub378\ud53c\uc544\uac00 \uc601\uad6d\uad70\uc5d0 \uc810\ub839\ub2f9\ud558\uace0, \uadf8 \uaca8\uc6b8 \uc870\uc9c0 \uc6cc\uc2f1\ud134 \uc7a5\uad70\uc740 \uac00\uc7a5 \uc5b4\ub824\uc6b4 \uc0c1\ud669\uc5d0 \ub193\uc5ec \uc788\uc5c8\uc9c0\ub9cc, \uad70\ub300 \uc7ac\uad50\uc721\uc744 \ud65c\uc131\ud654\uc2dc\ud0a4\ub294 \uc2dc\uae30\uac00 \ub418\uae30\ub3c4 \ud558\uc600\ub2e4. 1778\ub144 \ud504\ub791\uc2a4\uac00 \ubbf8\uad6d\uacfc \ub3d9\ub9f9\uc744 \ub9fa\uace0 \ucc38\uc804\ud55c \ud574\uc9c0\ub9cc, \uc774\ud6c4 \uba87 \ub144 \ub3d9\uc548 \ubbf8\uad6d \ub3c5\ub9bd \uc804\uc7c1\uc740 \uc9c0\ub9ac\ud55c \uc18c\ubaa8\uc804 \uc591\uc0c1\uc744 \ubcf4\uc600\ub2e4. \uad76\uc8fc\ub9bc\uacfc \uc9c8\ubcd1 \uadf8\ub9ac\uace0 \ub3d9\uc0c1\uc73c\ub85c 1778\ub144 2\uc6d4\ub9d0\uacbd\uc5d0\ub294 \uac70\uc758 2,500\uba85\uc758 \ubbf8\ud569\uc911\uad6d \uad70\uc778\ub4e4\uc774 \uc0ac\ub9dd\ud588\ub2e4.", "sentence_answer": "\ubc38\ub9ac \ud3ec\uc9c0(Valley Forge)\ub294 \ubbf8\uad6d \ub3c5\ub9bd \uc804\uc7c1 \uc911\uc778 1777\ub144\uc5d0\uc11c 1778\ub144 \uc0ac\uc774\uc5d0 \ub300\ub959\uad70\uc774 \ub3d9\uacc4\uc5d0 \uc8fc\ub454\ud55c \uacf3\uc73c\ub85c \ud544\ub77c\ub378\ud53c\uc544\uc5d0\uc11c \ubd81\uc11c\ucabd\uc73c\ub85c \uc57d 20\ub9c8\uc77c \ub5a8\uc5b4\uc9c4 \ud39c\uc2e4\ubca0\uc774\ub2c8\uc544\uc5d0 \uc704\uce58\ud558\uace0 \uc788\ub2e4.", "paragraph_id": "6508062-0-1", "paragraph_question": "question: \ud504\ub791\uc2a4\uac00 \ubbf8\uad6d\uacfc \ub3d9\ub9f9\uc744 \ub9fa\uace0 \ubbf8\uad6d \ub3c5\ub9bd \uc804\uc7c1\uc5d0 \ucc38\uc804\ud55c \ud574\ub294?, context: \ubc38\ub9ac \ud3ec\uc9c0(Valley Forge)\ub294 \ubbf8\uad6d \ub3c5\ub9bd \uc804\uc7c1 \uc911\uc778 1777\ub144\uc5d0\uc11c 1778\ub144 \uc0ac\uc774\uc5d0 \ub300\ub959\uad70\uc774 \ub3d9\uacc4\uc5d0 \uc8fc\ub454\ud55c \uacf3\uc73c\ub85c \ud544\ub77c\ub378\ud53c\uc544\uc5d0\uc11c \ubd81\uc11c\ucabd\uc73c\ub85c \uc57d 20\ub9c8\uc77c \ub5a8\uc5b4\uc9c4 \ud39c\uc2e4\ubca0\uc774\ub2c8\uc544\uc5d0 \uc704\uce58\ud558\uace0 \uc788\ub2e4. \uadf8\ub54c\uae4c\uc9c0 \ub300\ub959\ud68c\uc758\uc758 \uac1c\ucd5c \uc7a5\uc18c\uc774\uc790, \uc2e4\uc9c8\uc801\uc778 \uc218\ub3c4\uc758 \uc5ed\ud560\uc744 \ud588\ub358 \ud544\ub77c\ub378\ud53c\uc544\uac00 \uc601\uad6d\uad70\uc5d0 \uc810\ub839\ub2f9\ud558\uace0, \uadf8 \uaca8\uc6b8 \uc870\uc9c0 \uc6cc\uc2f1\ud134 \uc7a5\uad70\uc740 \uac00\uc7a5 \uc5b4\ub824\uc6b4 \uc0c1\ud669\uc5d0 \ub193\uc5ec \uc788\uc5c8\uc9c0\ub9cc, \uad70\ub300 \uc7ac\uad50\uc721\uc744 \ud65c\uc131\ud654\uc2dc\ud0a4\ub294 \uc2dc\uae30\uac00 \ub418\uae30\ub3c4 \ud558\uc600\ub2e4. 1778\ub144 \ud504\ub791\uc2a4\uac00 \ubbf8\uad6d\uacfc \ub3d9\ub9f9\uc744 \ub9fa\uace0 \ucc38\uc804\ud55c \ud574\uc9c0\ub9cc, \uc774\ud6c4 \uba87 \ub144 \ub3d9\uc548 \ubbf8\uad6d \ub3c5\ub9bd \uc804\uc7c1\uc740 \uc9c0\ub9ac\ud55c \uc18c\ubaa8\uc804 \uc591\uc0c1\uc744 \ubcf4\uc600\ub2e4. \uad76\uc8fc\ub9bc\uacfc \uc9c8\ubcd1 \uadf8\ub9ac\uace0 \ub3d9\uc0c1\uc73c\ub85c 1778\ub144 2\uc6d4\ub9d0\uacbd\uc5d0\ub294 \uac70\uc758 2,500\uba85\uc758 \ubbf8\ud569\uc911\uad6d \uad70\uc778\ub4e4\uc774 \uc0ac\ub9dd\ud588\ub2e4."} {"question": "<\uac00\ub514\uc5b8>\uc758 \uce5c\ud658\uacbd \ub3c4\uc2dc \uc2dc\ubbac\ub808\uc774\uc158 \uc2e4\ud5d8\uc5d0\uc11c \ube44\uad50\ub300\uc0c1\uc73c\ub85c \uc0bc\uc740 \ub374\ub9c8\ud06c\uc758 \ub3c4\uc2dc\ub294?", "paragraph": "\uac00\ub514\uc5b8\uc5d0\uc11c\ub294 \uc2e4\uc81c \ub3c4\uc2dc \uacc4\ud68d\uc0c1\uc758 \uc774\uc0c1\uc774 \ucef4\ud4e8\ud130 \uc2dc\ubbac\ub808\uc774\uc158\uc5d0\uc11c \uc77c\uc5b4\ub098\ub294\uc9c0 \ubcf4\uae30 \uc704\ud574\uc11c \uc2dc\ud2f0\uc988: \uc2a4\uce74\uc774\ub77c\uc778\uc744 \uc774\uc6a9\ud558\uc5ec \ud658\uacbd \uc804\ubb38 \uae30\uc790\uc778 Karl Mathiesen\uc5d0\uac8c \uce5c\ud658\uacbd\uc801\uc778 \ub3c4\uc2dc\ub97c \ub9cc\ub4e4\uc5b4\ubcf4\ub77c\uace0 \uc2dc\ud0a4\uae30\ub3c4 \ud558\uc600\ub2e4. \uae30\uc790\ub294 \uc2dc\ud2f0\uc988: \uc2a4\uce74\uc774\ub77c\uc778\uc73c\ub85c 7\ub9cc 8\ucc9c\uba85\ub418\ub294 \ub3c4\uc2dc\uc758 \uc5d0\ub108\uc9c0\ub97c \ubaa8\ub450 \uc7ac\uc0dd\uac00\ub2a5\ud55c \uc5d0\ub108\uc9c0\ub97c \uc0ac\uc6a9\ud558\uac8c \ud558\uc600\uc73c\uba70, \ud6a8\uacfc\uc801\uc778 \uc218\uc1a1\uacfc \ud558\uc774\ud14c\ud06c\uc640 \uacf5\ud574\ub97c \ubc1c\uc0dd\uc2dc\ud0a4\uc9c0 \uc54a\ub294 \uacf5\uc5c5\uc5d0 \ub300\ud55c \uc88b\uc740 \uc77c\uc790\ub9ac\ub97c \uc870\uac74\uc73c\ub85c \ub3c4\uc2dc\ub97c \ub9cc\ub4e4\uc5c8\ub2e4. \uadf8\ub9ac\uace0 \uc2dc\ud2f0\uc988: \uc2a4\uce74\uc774\ub77c\uc778\uc744 \ub3cc\ub9b0 Karl Mathiesen\uc740 \uc7ac\uc0dd \uac00\ub2a5 \uc5d0\ub108\uc9c0\uac00 \uac8c\uc784\uc0c1\uc758 \ube44\ud604\uc2e4\uc801\uc778 \uc88b\uc740 \ub0a0\uc528\ub85c \uc778\ud574\uc11c \uc800\ube44\uc6a9\uc73c\ub85c \ub098\uc654\ub2e4\uace0 \ub9d0\ud588\uc9c0\ub9cc, \ub3c4\uc2dc\uc758 \ub300\uc911\uad50\ud1b5 \uc2dc\uc2a4\ud15c\uc740 \uac8c\uc784\uc0c1\uc758 \ub3c4\uc2dc\uc640 \ube44\uad50 \ub300\uc0c1\uc73c\ub85c \uc0bc\uc740 \ub374\ub9c8\ud06c \ucf54\ud39c\ud558\uac90\ubcf4\ub2e4 \ub354 \ub5a8\uc5b4\uc84c\uc73c\uba70, \ucf54\ud39c\ud558\uac90\uacfc \ub354\ubd88\uc5b4 \uac8c\uc784\uc5d0\uc11c\ub3c4 \uc4f0\ub808\uae30 \ucc98\ub9ac\uc5d0 \ub300\ud55c \uc7a5\uae30\uc801\uc778 \ud574\uacb0\ucc45\uc744 \uac1c\ubc1c \ubabb\ud55c \uac83\uc774 \ud070 \uc7a5\uc560\ubb3c\uc774\ub77c\uace0 \ud3c9\uac00 \ud588\ub2e4.", "answer": "\ucf54\ud39c\ud558\uac90", "sentence": "\ub3c4\uc2dc\uc758 \ub300\uc911\uad50\ud1b5 \uc2dc\uc2a4\ud15c\uc740 \uac8c\uc784\uc0c1\uc758 \ub3c4\uc2dc\uc640 \ube44\uad50 \ub300\uc0c1\uc73c\ub85c \uc0bc\uc740 \ub374\ub9c8\ud06c \ucf54\ud39c\ud558\uac90 \ubcf4\ub2e4 \ub354 \ub5a8\uc5b4\uc84c\uc73c\uba70, \ucf54\ud39c\ud558\uac90\uacfc \ub354\ubd88\uc5b4 \uac8c\uc784\uc5d0\uc11c\ub3c4 \uc4f0\ub808\uae30 \ucc98\ub9ac\uc5d0 \ub300\ud55c \uc7a5\uae30\uc801\uc778 \ud574\uacb0\ucc45\uc744 \uac1c\ubc1c \ubabb\ud55c \uac83\uc774 \ud070 \uc7a5\uc560\ubb3c\uc774\ub77c\uace0 \ud3c9\uac00 \ud588\ub2e4.", "paragraph_sentence": "\uac00\ub514\uc5b8\uc5d0\uc11c\ub294 \uc2e4\uc81c \ub3c4\uc2dc \uacc4\ud68d\uc0c1\uc758 \uc774\uc0c1\uc774 \ucef4\ud4e8\ud130 \uc2dc\ubbac\ub808\uc774\uc158\uc5d0\uc11c \uc77c\uc5b4\ub098\ub294\uc9c0 \ubcf4\uae30 \uc704\ud574\uc11c \uc2dc\ud2f0\uc988: \uc2a4\uce74\uc774\ub77c\uc778\uc744 \uc774\uc6a9\ud558\uc5ec \ud658\uacbd \uc804\ubb38 \uae30\uc790\uc778 Karl Mathiesen\uc5d0\uac8c \uce5c\ud658\uacbd\uc801\uc778 \ub3c4\uc2dc\ub97c \ub9cc\ub4e4\uc5b4\ubcf4\ub77c\uace0 \uc2dc\ud0a4\uae30\ub3c4 \ud558\uc600\ub2e4. \uae30\uc790\ub294 \uc2dc\ud2f0\uc988: \uc2a4\uce74\uc774\ub77c\uc778\uc73c\ub85c 7\ub9cc 8\ucc9c\uba85\ub418\ub294 \ub3c4\uc2dc\uc758 \uc5d0\ub108\uc9c0\ub97c \ubaa8\ub450 \uc7ac\uc0dd\uac00\ub2a5\ud55c \uc5d0\ub108\uc9c0\ub97c \uc0ac\uc6a9\ud558\uac8c \ud558\uc600\uc73c\uba70, \ud6a8\uacfc\uc801\uc778 \uc218\uc1a1\uacfc \ud558\uc774\ud14c\ud06c\uc640 \uacf5\ud574\ub97c \ubc1c\uc0dd\uc2dc\ud0a4\uc9c0 \uc54a\ub294 \uacf5\uc5c5\uc5d0 \ub300\ud55c \uc88b\uc740 \uc77c\uc790\ub9ac\ub97c \uc870\uac74\uc73c\ub85c \ub3c4\uc2dc\ub97c \ub9cc\ub4e4\uc5c8\ub2e4. \uadf8\ub9ac\uace0 \uc2dc\ud2f0\uc988: \uc2a4\uce74\uc774\ub77c\uc778\uc744 \ub3cc\ub9b0 Karl Mathiesen\uc740 \uc7ac\uc0dd \uac00\ub2a5 \uc5d0\ub108\uc9c0\uac00 \uac8c\uc784\uc0c1\uc758 \ube44\ud604\uc2e4\uc801\uc778 \uc88b\uc740 \ub0a0\uc528\ub85c \uc778\ud574\uc11c \uc800\ube44\uc6a9\uc73c\ub85c \ub098\uc654\ub2e4\uace0 \ub9d0\ud588\uc9c0\ub9cc, \ub3c4\uc2dc\uc758 \ub300\uc911\uad50\ud1b5 \uc2dc\uc2a4\ud15c\uc740 \uac8c\uc784\uc0c1\uc758 \ub3c4\uc2dc\uc640 \ube44\uad50 \ub300\uc0c1\uc73c\ub85c \uc0bc\uc740 \ub374\ub9c8\ud06c \ucf54\ud39c\ud558\uac90 \ubcf4\ub2e4 \ub354 \ub5a8\uc5b4\uc84c\uc73c\uba70, \ucf54\ud39c\ud558\uac90\uacfc \ub354\ubd88\uc5b4 \uac8c\uc784\uc5d0\uc11c\ub3c4 \uc4f0\ub808\uae30 \ucc98\ub9ac\uc5d0 \ub300\ud55c \uc7a5\uae30\uc801\uc778 \ud574\uacb0\ucc45\uc744 \uac1c\ubc1c \ubabb\ud55c \uac83\uc774 \ud070 \uc7a5\uc560\ubb3c\uc774\ub77c\uace0 \ud3c9\uac00 \ud588\ub2e4. ", "paragraph_answer": "\uac00\ub514\uc5b8\uc5d0\uc11c\ub294 \uc2e4\uc81c \ub3c4\uc2dc \uacc4\ud68d\uc0c1\uc758 \uc774\uc0c1\uc774 \ucef4\ud4e8\ud130 \uc2dc\ubbac\ub808\uc774\uc158\uc5d0\uc11c \uc77c\uc5b4\ub098\ub294\uc9c0 \ubcf4\uae30 \uc704\ud574\uc11c \uc2dc\ud2f0\uc988: \uc2a4\uce74\uc774\ub77c\uc778\uc744 \uc774\uc6a9\ud558\uc5ec \ud658\uacbd \uc804\ubb38 \uae30\uc790\uc778 Karl Mathiesen\uc5d0\uac8c \uce5c\ud658\uacbd\uc801\uc778 \ub3c4\uc2dc\ub97c \ub9cc\ub4e4\uc5b4\ubcf4\ub77c\uace0 \uc2dc\ud0a4\uae30\ub3c4 \ud558\uc600\ub2e4. \uae30\uc790\ub294 \uc2dc\ud2f0\uc988: \uc2a4\uce74\uc774\ub77c\uc778\uc73c\ub85c 7\ub9cc 8\ucc9c\uba85\ub418\ub294 \ub3c4\uc2dc\uc758 \uc5d0\ub108\uc9c0\ub97c \ubaa8\ub450 \uc7ac\uc0dd\uac00\ub2a5\ud55c \uc5d0\ub108\uc9c0\ub97c \uc0ac\uc6a9\ud558\uac8c \ud558\uc600\uc73c\uba70, \ud6a8\uacfc\uc801\uc778 \uc218\uc1a1\uacfc \ud558\uc774\ud14c\ud06c\uc640 \uacf5\ud574\ub97c \ubc1c\uc0dd\uc2dc\ud0a4\uc9c0 \uc54a\ub294 \uacf5\uc5c5\uc5d0 \ub300\ud55c \uc88b\uc740 \uc77c\uc790\ub9ac\ub97c \uc870\uac74\uc73c\ub85c \ub3c4\uc2dc\ub97c \ub9cc\ub4e4\uc5c8\ub2e4. \uadf8\ub9ac\uace0 \uc2dc\ud2f0\uc988: \uc2a4\uce74\uc774\ub77c\uc778\uc744 \ub3cc\ub9b0 Karl Mathiesen\uc740 \uc7ac\uc0dd \uac00\ub2a5 \uc5d0\ub108\uc9c0\uac00 \uac8c\uc784\uc0c1\uc758 \ube44\ud604\uc2e4\uc801\uc778 \uc88b\uc740 \ub0a0\uc528\ub85c \uc778\ud574\uc11c \uc800\ube44\uc6a9\uc73c\ub85c \ub098\uc654\ub2e4\uace0 \ub9d0\ud588\uc9c0\ub9cc, \ub3c4\uc2dc\uc758 \ub300\uc911\uad50\ud1b5 \uc2dc\uc2a4\ud15c\uc740 \uac8c\uc784\uc0c1\uc758 \ub3c4\uc2dc\uc640 \ube44\uad50 \ub300\uc0c1\uc73c\ub85c \uc0bc\uc740 \ub374\ub9c8\ud06c \ucf54\ud39c\ud558\uac90 \ubcf4\ub2e4 \ub354 \ub5a8\uc5b4\uc84c\uc73c\uba70, \ucf54\ud39c\ud558\uac90\uacfc \ub354\ubd88\uc5b4 \uac8c\uc784\uc5d0\uc11c\ub3c4 \uc4f0\ub808\uae30 \ucc98\ub9ac\uc5d0 \ub300\ud55c \uc7a5\uae30\uc801\uc778 \ud574\uacb0\ucc45\uc744 \uac1c\ubc1c \ubabb\ud55c \uac83\uc774 \ud070 \uc7a5\uc560\ubb3c\uc774\ub77c\uace0 \ud3c9\uac00 \ud588\ub2e4.", "sentence_answer": "\ub3c4\uc2dc\uc758 \ub300\uc911\uad50\ud1b5 \uc2dc\uc2a4\ud15c\uc740 \uac8c\uc784\uc0c1\uc758 \ub3c4\uc2dc\uc640 \ube44\uad50 \ub300\uc0c1\uc73c\ub85c \uc0bc\uc740 \ub374\ub9c8\ud06c \ucf54\ud39c\ud558\uac90 \ubcf4\ub2e4 \ub354 \ub5a8\uc5b4\uc84c\uc73c\uba70, \ucf54\ud39c\ud558\uac90\uacfc \ub354\ubd88\uc5b4 \uac8c\uc784\uc5d0\uc11c\ub3c4 \uc4f0\ub808\uae30 \ucc98\ub9ac\uc5d0 \ub300\ud55c \uc7a5\uae30\uc801\uc778 \ud574\uacb0\ucc45\uc744 \uac1c\ubc1c \ubabb\ud55c \uac83\uc774 \ud070 \uc7a5\uc560\ubb3c\uc774\ub77c\uace0 \ud3c9\uac00 \ud588\ub2e4.", "paragraph_id": "6519573-11-1", "paragraph_question": "question: <\uac00\ub514\uc5b8>\uc758 \uce5c\ud658\uacbd \ub3c4\uc2dc \uc2dc\ubbac\ub808\uc774\uc158 \uc2e4\ud5d8\uc5d0\uc11c \ube44\uad50\ub300\uc0c1\uc73c\ub85c \uc0bc\uc740 \ub374\ub9c8\ud06c\uc758 \ub3c4\uc2dc\ub294?, context: \uac00\ub514\uc5b8\uc5d0\uc11c\ub294 \uc2e4\uc81c \ub3c4\uc2dc \uacc4\ud68d\uc0c1\uc758 \uc774\uc0c1\uc774 \ucef4\ud4e8\ud130 \uc2dc\ubbac\ub808\uc774\uc158\uc5d0\uc11c \uc77c\uc5b4\ub098\ub294\uc9c0 \ubcf4\uae30 \uc704\ud574\uc11c \uc2dc\ud2f0\uc988: \uc2a4\uce74\uc774\ub77c\uc778\uc744 \uc774\uc6a9\ud558\uc5ec \ud658\uacbd \uc804\ubb38 \uae30\uc790\uc778 Karl Mathiesen\uc5d0\uac8c \uce5c\ud658\uacbd\uc801\uc778 \ub3c4\uc2dc\ub97c \ub9cc\ub4e4\uc5b4\ubcf4\ub77c\uace0 \uc2dc\ud0a4\uae30\ub3c4 \ud558\uc600\ub2e4. \uae30\uc790\ub294 \uc2dc\ud2f0\uc988: \uc2a4\uce74\uc774\ub77c\uc778\uc73c\ub85c 7\ub9cc 8\ucc9c\uba85\ub418\ub294 \ub3c4\uc2dc\uc758 \uc5d0\ub108\uc9c0\ub97c \ubaa8\ub450 \uc7ac\uc0dd\uac00\ub2a5\ud55c \uc5d0\ub108\uc9c0\ub97c \uc0ac\uc6a9\ud558\uac8c \ud558\uc600\uc73c\uba70, \ud6a8\uacfc\uc801\uc778 \uc218\uc1a1\uacfc \ud558\uc774\ud14c\ud06c\uc640 \uacf5\ud574\ub97c \ubc1c\uc0dd\uc2dc\ud0a4\uc9c0 \uc54a\ub294 \uacf5\uc5c5\uc5d0 \ub300\ud55c \uc88b\uc740 \uc77c\uc790\ub9ac\ub97c \uc870\uac74\uc73c\ub85c \ub3c4\uc2dc\ub97c \ub9cc\ub4e4\uc5c8\ub2e4. \uadf8\ub9ac\uace0 \uc2dc\ud2f0\uc988: \uc2a4\uce74\uc774\ub77c\uc778\uc744 \ub3cc\ub9b0 Karl Mathiesen\uc740 \uc7ac\uc0dd \uac00\ub2a5 \uc5d0\ub108\uc9c0\uac00 \uac8c\uc784\uc0c1\uc758 \ube44\ud604\uc2e4\uc801\uc778 \uc88b\uc740 \ub0a0\uc528\ub85c \uc778\ud574\uc11c \uc800\ube44\uc6a9\uc73c\ub85c \ub098\uc654\ub2e4\uace0 \ub9d0\ud588\uc9c0\ub9cc, \ub3c4\uc2dc\uc758 \ub300\uc911\uad50\ud1b5 \uc2dc\uc2a4\ud15c\uc740 \uac8c\uc784\uc0c1\uc758 \ub3c4\uc2dc\uc640 \ube44\uad50 \ub300\uc0c1\uc73c\ub85c \uc0bc\uc740 \ub374\ub9c8\ud06c \ucf54\ud39c\ud558\uac90\ubcf4\ub2e4 \ub354 \ub5a8\uc5b4\uc84c\uc73c\uba70, \ucf54\ud39c\ud558\uac90\uacfc \ub354\ubd88\uc5b4 \uac8c\uc784\uc5d0\uc11c\ub3c4 \uc4f0\ub808\uae30 \ucc98\ub9ac\uc5d0 \ub300\ud55c \uc7a5\uae30\uc801\uc778 \ud574\uacb0\ucc45\uc744 \uac1c\ubc1c \ubabb\ud55c \uac83\uc774 \ud070 \uc7a5\uc560\ubb3c\uc774\ub77c\uace0 \ud3c9\uac00 \ud588\ub2e4."} {"question": "\uc2e0\ub77c\uad70\uc774 \ub0a8\ucabd\uc740 \uc774\ubbf8 \ud3c9\uc815\ud558\uc600\uace0 \ubd81\ucabd\uc744 \ubc8c\ud55c\ub2e4\uace0 \uae30\uc220\ud55c \ucc45\uc758 \uc774\ub984\uc740 \ubb34\uc5c7\uc778\uac00?", "paragraph": "\u300a\uc0bc\uad6d\uc0ac\uae30\u300b \ubc0f \u300a\ub2f9\uc11c\u300b\uc5d0\ub294 \ubcf5\uc2e0\uc774 \ud48d\uc655\uc744 \uc2dc\ud574\ud558\uace0\uc790 \ubcd1\uc744 \ud551\uacc4\ub85c \ub3d9\uad74\uc5d0 \uc228\uc5c8\ub2e4\ub294 \uae30\uc220\uc774 \uc788\ub294\ub370, \uc704\uae08\uc554\uc0b0\uc131\uc758 \uc6b8\uae08\ubc14\uc704\uc5d0\ub294 \uc138 \uac1c\uc758 \uad74\uc774 \uc788\uc5b4 \ub3d9\uad74\ub9c8\ub2e4 \uc2b9\ub824\ub4e4\uc774 \uac70\uc8fc\ud588\ub2e4\ub294 \u300a\uc2e0\uc99d\ub3d9\uad6d\uc5ec\uc9c0\uc2b9\ub78c\u300b\uacfc \ud568\uaed8, \uc2e0\ub77c\uc758 \uace0\uc2b9 \uc6d0\ud6a8\uac00 \uc218\ud589\ud588\ub2e4 \ud574\uc11c '\uc6d0\ud6a8\uad74(\uc6d0\ud6a8\ubc29)'\ub85c\ub3c4 \ubd88\ub9ac\ub294 \ub3d9\uad74\uc774 \uc6b8\uae08\ubc14\uc704 \uc911\uac04\uc5d0 \uc788\uc5b4 \uc0ac\ub78c\uc758 \uc811\uadfc\uc774 \ub9e4\uc6b0 \uc5b4\ub835\uace0, \uc6b8\uae08\ubc14\uc704 \ubc14\ub2e5\uc5d0\ub294 \ubcf5\uc2e0\uc774 \uc228\uc5c8\ub2e4\ub294 \ubcf5\uc2e0\uad74(\uc678\uad74:\uae38\uc774 17m, \ud3ed 21m, \ub192\uc774 17m. \ub0b4\uad74: \uae38\uc774 7m, \ud3ed 6.6m, \ub192\uc774 3~5m)\uc77c \uac00\ub2a5\uc131\uc774 \uc9c0\uc801\ub41c\ub2e4. \ub610\ud55c \u300a\uc0bc\uad6d\uc0ac\uae30\u300b\uc5d0 \ubc31\uc81c \ubd80\ud765\uad70\uc758 \uc9c0\ud718\ubd80\uc600\ub358 \ub450\uc194\uc131(\uc8fc\ub958\uc131)\uc744 \ud568\ub77d\uc2dc\ud0a8 \uc2e0\ub77c\uad70\uc774, \ub9c8\uc9c0\ub9c9 \ub0a8\uc740 \ubd80\ud765\uad70 \uac70\uc810\uc774\uc5c8\ub358 \uc784\uc874\uc131\uc744 \uce58\ub294 \uac83\uc5d0 \ub300\ud574 \"\ub0a8\ucabd\uc740 \uc774\ubbf8 \ud3c9\uc815\ub418\uc5c8\uc73c\ubbc0\ub85c \uad70\uc0ac\ub97c \ub3cc\ub824 \ubd81\ucabd\uc744 \ubc8c\ud55c\ub2e4(\u5357\u65b9\u65e3\u5b9a, \u5efb\u8ecd\u5317\u4f10)\"\uace0 \ud55c \uae30\uc220\uc774 \uc788\ub294\ub370 \uc774 \uae30\ub85d\uacfc\ub3c4 \ud569\uce58\ud558\ub294 \uac83\uc73c\ub85c \uc5ec\uaca8\uc9c0\uace0 \uc788\ub2e4.", "answer": "\uc0bc\uad6d\uc0ac\uae30", "sentence": "\u300a \uc0bc\uad6d\uc0ac\uae30 \u300b \ubc0f \u300a\ub2f9\uc11c\u300b\uc5d0\ub294 \ubcf5\uc2e0\uc774 \ud48d\uc655\uc744 \uc2dc\ud574\ud558\uace0\uc790 \ubcd1\uc744 \ud551\uacc4\ub85c \ub3d9\uad74\uc5d0 \uc228\uc5c8\ub2e4\ub294 \uae30\uc220\uc774 \uc788\ub294\ub370, \uc704\uae08\uc554\uc0b0\uc131\uc758 \uc6b8\uae08\ubc14\uc704\uc5d0\ub294 \uc138 \uac1c\uc758 \uad74\uc774 \uc788\uc5b4 \ub3d9\uad74\ub9c8\ub2e4 \uc2b9\ub824\ub4e4\uc774 \uac70\uc8fc\ud588\ub2e4\ub294 \u300a\uc2e0\uc99d\ub3d9\uad6d\uc5ec\uc9c0\uc2b9\ub78c\u300b\uacfc \ud568\uaed8, \uc2e0\ub77c\uc758 \uace0\uc2b9 \uc6d0\ud6a8\uac00 \uc218\ud589\ud588\ub2e4 \ud574\uc11c '\uc6d0\ud6a8\uad74(\uc6d0\ud6a8\ubc29)'\ub85c\ub3c4 \ubd88\ub9ac\ub294 \ub3d9\uad74\uc774 \uc6b8\uae08\ubc14\uc704 \uc911\uac04\uc5d0 \uc788\uc5b4 \uc0ac\ub78c\uc758 \uc811\uadfc\uc774 \ub9e4\uc6b0 \uc5b4\ub835\uace0, \uc6b8\uae08\ubc14\uc704 \ubc14\ub2e5\uc5d0\ub294 \ubcf5\uc2e0\uc774 \uc228\uc5c8\ub2e4\ub294 \ubcf5\uc2e0\uad74(\uc678\uad74:\uae38\uc774 17m, \ud3ed 21m, \ub192\uc774 17m. \ub0b4\uad74: \uae38\uc774 7m, \ud3ed 6.6m, \ub192\uc774 3~5m)\uc77c \uac00\ub2a5\uc131\uc774 \uc9c0\uc801\ub41c\ub2e4.", "paragraph_sentence": " \u300a \uc0bc\uad6d\uc0ac\uae30 \u300b \ubc0f \u300a\ub2f9\uc11c\u300b\uc5d0\ub294 \ubcf5\uc2e0\uc774 \ud48d\uc655\uc744 \uc2dc\ud574\ud558\uace0\uc790 \ubcd1\uc744 \ud551\uacc4\ub85c \ub3d9\uad74\uc5d0 \uc228\uc5c8\ub2e4\ub294 \uae30\uc220\uc774 \uc788\ub294\ub370, \uc704\uae08\uc554\uc0b0\uc131\uc758 \uc6b8\uae08\ubc14\uc704\uc5d0\ub294 \uc138 \uac1c\uc758 \uad74\uc774 \uc788\uc5b4 \ub3d9\uad74\ub9c8\ub2e4 \uc2b9\ub824\ub4e4\uc774 \uac70\uc8fc\ud588\ub2e4\ub294 \u300a\uc2e0\uc99d\ub3d9\uad6d\uc5ec\uc9c0\uc2b9\ub78c\u300b\uacfc \ud568\uaed8, \uc2e0\ub77c\uc758 \uace0\uc2b9 \uc6d0\ud6a8\uac00 \uc218\ud589\ud588\ub2e4 \ud574\uc11c '\uc6d0\ud6a8\uad74(\uc6d0\ud6a8\ubc29)'\ub85c\ub3c4 \ubd88\ub9ac\ub294 \ub3d9\uad74\uc774 \uc6b8\uae08\ubc14\uc704 \uc911\uac04\uc5d0 \uc788\uc5b4 \uc0ac\ub78c\uc758 \uc811\uadfc\uc774 \ub9e4\uc6b0 \uc5b4\ub835\uace0, \uc6b8\uae08\ubc14\uc704 \ubc14\ub2e5\uc5d0\ub294 \ubcf5\uc2e0\uc774 \uc228\uc5c8\ub2e4\ub294 \ubcf5\uc2e0\uad74(\uc678\uad74:\uae38\uc774 17m, \ud3ed 21m, \ub192\uc774 17m. \ub0b4\uad74: \uae38\uc774 7m, \ud3ed 6.6m, \ub192\uc774 3~5m)\uc77c \uac00\ub2a5\uc131\uc774 \uc9c0\uc801\ub41c\ub2e4. \ub610\ud55c \u300a\uc0bc\uad6d\uc0ac\uae30\u300b\uc5d0 \ubc31\uc81c \ubd80\ud765\uad70\uc758 \uc9c0\ud718\ubd80\uc600\ub358 \ub450\uc194\uc131(\uc8fc\ub958\uc131)\uc744 \ud568\ub77d\uc2dc\ud0a8 \uc2e0\ub77c\uad70\uc774, \ub9c8\uc9c0\ub9c9 \ub0a8\uc740 \ubd80\ud765\uad70 \uac70\uc810\uc774\uc5c8\ub358 \uc784\uc874\uc131\uc744 \uce58\ub294 \uac83\uc5d0 \ub300\ud574 \"\ub0a8\ucabd\uc740 \uc774\ubbf8 \ud3c9\uc815\ub418\uc5c8\uc73c\ubbc0\ub85c \uad70\uc0ac\ub97c \ub3cc\ub824 \ubd81\ucabd\uc744 \ubc8c\ud55c\ub2e4(\u5357\u65b9\u65e3\u5b9a, \u5efb\u8ecd\u5317\u4f10)\"\uace0 \ud55c \uae30\uc220\uc774 \uc788\ub294\ub370 \uc774 \uae30\ub85d\uacfc\ub3c4 \ud569\uce58\ud558\ub294 \uac83\uc73c\ub85c \uc5ec\uaca8\uc9c0\uace0 \uc788\ub2e4.", "paragraph_answer": "\u300a \uc0bc\uad6d\uc0ac\uae30 \u300b \ubc0f \u300a\ub2f9\uc11c\u300b\uc5d0\ub294 \ubcf5\uc2e0\uc774 \ud48d\uc655\uc744 \uc2dc\ud574\ud558\uace0\uc790 \ubcd1\uc744 \ud551\uacc4\ub85c \ub3d9\uad74\uc5d0 \uc228\uc5c8\ub2e4\ub294 \uae30\uc220\uc774 \uc788\ub294\ub370, \uc704\uae08\uc554\uc0b0\uc131\uc758 \uc6b8\uae08\ubc14\uc704\uc5d0\ub294 \uc138 \uac1c\uc758 \uad74\uc774 \uc788\uc5b4 \ub3d9\uad74\ub9c8\ub2e4 \uc2b9\ub824\ub4e4\uc774 \uac70\uc8fc\ud588\ub2e4\ub294 \u300a\uc2e0\uc99d\ub3d9\uad6d\uc5ec\uc9c0\uc2b9\ub78c\u300b\uacfc \ud568\uaed8, \uc2e0\ub77c\uc758 \uace0\uc2b9 \uc6d0\ud6a8\uac00 \uc218\ud589\ud588\ub2e4 \ud574\uc11c '\uc6d0\ud6a8\uad74(\uc6d0\ud6a8\ubc29)'\ub85c\ub3c4 \ubd88\ub9ac\ub294 \ub3d9\uad74\uc774 \uc6b8\uae08\ubc14\uc704 \uc911\uac04\uc5d0 \uc788\uc5b4 \uc0ac\ub78c\uc758 \uc811\uadfc\uc774 \ub9e4\uc6b0 \uc5b4\ub835\uace0, \uc6b8\uae08\ubc14\uc704 \ubc14\ub2e5\uc5d0\ub294 \ubcf5\uc2e0\uc774 \uc228\uc5c8\ub2e4\ub294 \ubcf5\uc2e0\uad74(\uc678\uad74:\uae38\uc774 17m, \ud3ed 21m, \ub192\uc774 17m. \ub0b4\uad74: \uae38\uc774 7m, \ud3ed 6.6m, \ub192\uc774 3~5m)\uc77c \uac00\ub2a5\uc131\uc774 \uc9c0\uc801\ub41c\ub2e4. \ub610\ud55c \u300a\uc0bc\uad6d\uc0ac\uae30\u300b\uc5d0 \ubc31\uc81c \ubd80\ud765\uad70\uc758 \uc9c0\ud718\ubd80\uc600\ub358 \ub450\uc194\uc131(\uc8fc\ub958\uc131)\uc744 \ud568\ub77d\uc2dc\ud0a8 \uc2e0\ub77c\uad70\uc774, \ub9c8\uc9c0\ub9c9 \ub0a8\uc740 \ubd80\ud765\uad70 \uac70\uc810\uc774\uc5c8\ub358 \uc784\uc874\uc131\uc744 \uce58\ub294 \uac83\uc5d0 \ub300\ud574 \"\ub0a8\ucabd\uc740 \uc774\ubbf8 \ud3c9\uc815\ub418\uc5c8\uc73c\ubbc0\ub85c \uad70\uc0ac\ub97c \ub3cc\ub824 \ubd81\ucabd\uc744 \ubc8c\ud55c\ub2e4(\u5357\u65b9\u65e3\u5b9a, \u5efb\u8ecd\u5317\u4f10)\"\uace0 \ud55c \uae30\uc220\uc774 \uc788\ub294\ub370 \uc774 \uae30\ub85d\uacfc\ub3c4 \ud569\uce58\ud558\ub294 \uac83\uc73c\ub85c \uc5ec\uaca8\uc9c0\uace0 \uc788\ub2e4.", "sentence_answer": "\u300a \uc0bc\uad6d\uc0ac\uae30 \u300b \ubc0f \u300a\ub2f9\uc11c\u300b\uc5d0\ub294 \ubcf5\uc2e0\uc774 \ud48d\uc655\uc744 \uc2dc\ud574\ud558\uace0\uc790 \ubcd1\uc744 \ud551\uacc4\ub85c \ub3d9\uad74\uc5d0 \uc228\uc5c8\ub2e4\ub294 \uae30\uc220\uc774 \uc788\ub294\ub370, \uc704\uae08\uc554\uc0b0\uc131\uc758 \uc6b8\uae08\ubc14\uc704\uc5d0\ub294 \uc138 \uac1c\uc758 \uad74\uc774 \uc788\uc5b4 \ub3d9\uad74\ub9c8\ub2e4 \uc2b9\ub824\ub4e4\uc774 \uac70\uc8fc\ud588\ub2e4\ub294 \u300a\uc2e0\uc99d\ub3d9\uad6d\uc5ec\uc9c0\uc2b9\ub78c\u300b\uacfc \ud568\uaed8, \uc2e0\ub77c\uc758 \uace0\uc2b9 \uc6d0\ud6a8\uac00 \uc218\ud589\ud588\ub2e4 \ud574\uc11c '\uc6d0\ud6a8\uad74(\uc6d0\ud6a8\ubc29)'\ub85c\ub3c4 \ubd88\ub9ac\ub294 \ub3d9\uad74\uc774 \uc6b8\uae08\ubc14\uc704 \uc911\uac04\uc5d0 \uc788\uc5b4 \uc0ac\ub78c\uc758 \uc811\uadfc\uc774 \ub9e4\uc6b0 \uc5b4\ub835\uace0, \uc6b8\uae08\ubc14\uc704 \ubc14\ub2e5\uc5d0\ub294 \ubcf5\uc2e0\uc774 \uc228\uc5c8\ub2e4\ub294 \ubcf5\uc2e0\uad74(\uc678\uad74:\uae38\uc774 17m, \ud3ed 21m, \ub192\uc774 17m. \ub0b4\uad74: \uae38\uc774 7m, \ud3ed 6.6m, \ub192\uc774 3~5m)\uc77c \uac00\ub2a5\uc131\uc774 \uc9c0\uc801\ub41c\ub2e4.", "paragraph_id": "6555572-12-1", "paragraph_question": "question: \uc2e0\ub77c\uad70\uc774 \ub0a8\ucabd\uc740 \uc774\ubbf8 \ud3c9\uc815\ud558\uc600\uace0 \ubd81\ucabd\uc744 \ubc8c\ud55c\ub2e4\uace0 \uae30\uc220\ud55c \ucc45\uc758 \uc774\ub984\uc740 \ubb34\uc5c7\uc778\uac00?, context: \u300a\uc0bc\uad6d\uc0ac\uae30\u300b \ubc0f \u300a\ub2f9\uc11c\u300b\uc5d0\ub294 \ubcf5\uc2e0\uc774 \ud48d\uc655\uc744 \uc2dc\ud574\ud558\uace0\uc790 \ubcd1\uc744 \ud551\uacc4\ub85c \ub3d9\uad74\uc5d0 \uc228\uc5c8\ub2e4\ub294 \uae30\uc220\uc774 \uc788\ub294\ub370, \uc704\uae08\uc554\uc0b0\uc131\uc758 \uc6b8\uae08\ubc14\uc704\uc5d0\ub294 \uc138 \uac1c\uc758 \uad74\uc774 \uc788\uc5b4 \ub3d9\uad74\ub9c8\ub2e4 \uc2b9\ub824\ub4e4\uc774 \uac70\uc8fc\ud588\ub2e4\ub294 \u300a\uc2e0\uc99d\ub3d9\uad6d\uc5ec\uc9c0\uc2b9\ub78c\u300b\uacfc \ud568\uaed8, \uc2e0\ub77c\uc758 \uace0\uc2b9 \uc6d0\ud6a8\uac00 \uc218\ud589\ud588\ub2e4 \ud574\uc11c '\uc6d0\ud6a8\uad74(\uc6d0\ud6a8\ubc29)'\ub85c\ub3c4 \ubd88\ub9ac\ub294 \ub3d9\uad74\uc774 \uc6b8\uae08\ubc14\uc704 \uc911\uac04\uc5d0 \uc788\uc5b4 \uc0ac\ub78c\uc758 \uc811\uadfc\uc774 \ub9e4\uc6b0 \uc5b4\ub835\uace0, \uc6b8\uae08\ubc14\uc704 \ubc14\ub2e5\uc5d0\ub294 \ubcf5\uc2e0\uc774 \uc228\uc5c8\ub2e4\ub294 \ubcf5\uc2e0\uad74(\uc678\uad74:\uae38\uc774 17m, \ud3ed 21m, \ub192\uc774 17m. \ub0b4\uad74: \uae38\uc774 7m, \ud3ed 6.6m, \ub192\uc774 3~5m)\uc77c \uac00\ub2a5\uc131\uc774 \uc9c0\uc801\ub41c\ub2e4. \ub610\ud55c \u300a\uc0bc\uad6d\uc0ac\uae30\u300b\uc5d0 \ubc31\uc81c \ubd80\ud765\uad70\uc758 \uc9c0\ud718\ubd80\uc600\ub358 \ub450\uc194\uc131(\uc8fc\ub958\uc131)\uc744 \ud568\ub77d\uc2dc\ud0a8 \uc2e0\ub77c\uad70\uc774, \ub9c8\uc9c0\ub9c9 \ub0a8\uc740 \ubd80\ud765\uad70 \uac70\uc810\uc774\uc5c8\ub358 \uc784\uc874\uc131\uc744 \uce58\ub294 \uac83\uc5d0 \ub300\ud574 \"\ub0a8\ucabd\uc740 \uc774\ubbf8 \ud3c9\uc815\ub418\uc5c8\uc73c\ubbc0\ub85c \uad70\uc0ac\ub97c \ub3cc\ub824 \ubd81\ucabd\uc744 \ubc8c\ud55c\ub2e4(\u5357\u65b9\u65e3\u5b9a, \u5efb\u8ecd\u5317\u4f10)\"\uace0 \ud55c \uae30\uc220\uc774 \uc788\ub294\ub370 \uc774 \uae30\ub85d\uacfc\ub3c4 \ud569\uce58\ud558\ub294 \uac83\uc73c\ub85c \uc5ec\uaca8\uc9c0\uace0 \uc788\ub2e4."} {"question": "1492\ub144 \uc544\uba54\ub9ac\uce74 \ud574\ub85c\ub97c \ud3ec\ub974\ud22c\uac08\uc778\ub4e4\uacfc \ubc1c\uacac\ud588\ub358 \uc0ac\ub78c\uc740?", "paragraph": "130\ub144\uac04 \uc9c0\uc18d\ub418\ub358 \ub974\ub124\uc0c1\uc2a4\ub294 1530\ub144\uacbd \ub05d\uc774 \ub0ac\ub2e4. \uadf8 \uc774\uc720\ub294 \uc6b0\uc120 1492\ub144 \ud06c\ub9ac\uc2a4\ud1a0\ubc1c \ucf54\ub860\uc774 \ud3ec\ub974\ud22c\uac08\uc778\ub4e4\uacfc \ud568\uaed8 \uc778\ub3c4\ub85c \uac00\ub294 \ud574\ub85c\ub97c \ubc1c\uacac\ud588\uae30 \ub54c\ubb38\uc774\ub2e4.(\uc0ac\uc2e4\uc740 \uc544\uba54\ub9ac\uce74\ub85c \uac00\ub294 \ud574\ub85c\uc600\ub2e4.) \uadf8 \ud6c4\uc5d0\ub294 \ubd81\uc11c\ubd80 \uc720\ub7fd\uc758 \uc0c1\uc778\ub4e4\uc774 \ubb34\uc5ed\uc0c1\ud488\uc744 \ub9ac\uc2a4\ubcf8\uacfc \uc548\ud2b8\uc6e8\ub974\ud39c\uc744 \ud1b5\ud574 \uac70\ub798\ud558\ub294 \uac83\uc744 \uc120\ud638\ud558\uac8c \ub418\uc5c8\ub2e4. \ub610\ud55c 1517\ub144 \uc544\uc6b0\uad6c\uc2a4\ud2f0\ub178\ud68c\uc758 \uc218\ub3c4\uc0ac\uc778 \ub9c8\ub974\ud2f4 \ub8e8\ud130\uac00 \uc885\uad50 \uac1c\ud601\uc744 \ub2e8\ud589\ud588\uae30 \ub54c\ubb38\uc774\ub2e4. \uc774\ub85c \uc778\ud574 \uc11c\uc720\ub7fd \uad50\ud68c\ub294 \ub85c\ub9c8 \uac00\ud1a8\ub9ad\uad50\ud68c\uc640 \uac1c\uc2e0\uad50 \uad50\ud68c\ub85c \ubd84\uc5f4\ub418\uc5c8\ub2e4. \uad50\ud68c\uc758 \ubd84\uc5f4\uc740 \uadf8\ub3d9\uc548 \ub0a9\ubd80\uae08\uacfc \uc138\uae08\uc758 \ud615\ud0dc\ub85c \uc774\ud0c8\ub9ac\uc544\ub97c \ud48d\uc694\ub86d\uac8c \ud588\ub358 \ub3c8\uc904\uc758 \uace0\uac08\uc744 \uc758\ubbf8\ud588\ub2e4. \uc720\ub7fd\uc778\uc758 \uc544\uba54\ub9ac\uce74 \ub300\ub959 \uc0c1\ub959\uacfc \uc885\uad50\uac1c\ud601\uc73c\ub85c \uacb0\uad6d \uc774\ud0c8\ub9ac\uc544\ub294 \uc0c1\uc5c5\uc801 \ubb34\uc5ed\uc790\ubcf8\uacfc \uc790\ubcf8, \ub450\uac00\uc9c0\ub97c \ub3d9\uc2dc\uc5d0 \uc783\uc5c8\ub2e4 \ub610 \uc774\ud0c8\ub9ac\uc544\ub294 \uadf8 \ub2f9\uc2dc \uc815\uce58\uc801\uc73c\ub85c \uad49\uc7a5\ud788 \ud63c\ub780\uc2a4\ub7ec\uc6b4 \uc0c1\ud0dc\uc774\uae30\ub3c4 \ud558\uc600\ub2e4. \uc774\ub85c \uc778\ud574 \ub974\ub124\uc0c1\uc2a4\ub294 \uc1e0\ud1f4\ud560 \uc218\ubc16\uc5d0 \uc5c6\uc5c8\ub2e4.", "answer": "\ud06c\ub9ac\uc2a4\ud1a0\ubc1c \ucf54\ub860", "sentence": "\uadf8 \uc774\uc720\ub294 \uc6b0\uc120 1492\ub144 \ud06c\ub9ac\uc2a4\ud1a0\ubc1c \ucf54\ub860 \uc774 \ud3ec\ub974\ud22c\uac08\uc778\ub4e4\uacfc \ud568\uaed8 \uc778\ub3c4\ub85c \uac00\ub294 \ud574\ub85c\ub97c \ubc1c\uacac\ud588\uae30 \ub54c\ubb38\uc774\ub2e4.", "paragraph_sentence": "130\ub144\uac04 \uc9c0\uc18d\ub418\ub358 \ub974\ub124\uc0c1\uc2a4\ub294 1530\ub144\uacbd \ub05d\uc774 \ub0ac\ub2e4. \uadf8 \uc774\uc720\ub294 \uc6b0\uc120 1492\ub144 \ud06c\ub9ac\uc2a4\ud1a0\ubc1c \ucf54\ub860 \uc774 \ud3ec\ub974\ud22c\uac08\uc778\ub4e4\uacfc \ud568\uaed8 \uc778\ub3c4\ub85c \uac00\ub294 \ud574\ub85c\ub97c \ubc1c\uacac\ud588\uae30 \ub54c\ubb38\uc774\ub2e4. (\uc0ac\uc2e4\uc740 \uc544\uba54\ub9ac\uce74\ub85c \uac00\ub294 \ud574\ub85c\uc600\ub2e4.) \uadf8 \ud6c4\uc5d0\ub294 \ubd81\uc11c\ubd80 \uc720\ub7fd\uc758 \uc0c1\uc778\ub4e4\uc774 \ubb34\uc5ed\uc0c1\ud488\uc744 \ub9ac\uc2a4\ubcf8\uacfc \uc548\ud2b8\uc6e8\ub974\ud39c\uc744 \ud1b5\ud574 \uac70\ub798\ud558\ub294 \uac83\uc744 \uc120\ud638\ud558\uac8c \ub418\uc5c8\ub2e4. \ub610\ud55c 1517\ub144 \uc544\uc6b0\uad6c\uc2a4\ud2f0\ub178\ud68c\uc758 \uc218\ub3c4\uc0ac\uc778 \ub9c8\ub974\ud2f4 \ub8e8\ud130\uac00 \uc885\uad50 \uac1c\ud601\uc744 \ub2e8\ud589\ud588\uae30 \ub54c\ubb38\uc774\ub2e4. \uc774\ub85c \uc778\ud574 \uc11c\uc720\ub7fd \uad50\ud68c\ub294 \ub85c\ub9c8 \uac00\ud1a8\ub9ad\uad50\ud68c\uc640 \uac1c\uc2e0\uad50 \uad50\ud68c\ub85c \ubd84\uc5f4\ub418\uc5c8\ub2e4. \uad50\ud68c\uc758 \ubd84\uc5f4\uc740 \uadf8\ub3d9\uc548 \ub0a9\ubd80\uae08\uacfc \uc138\uae08\uc758 \ud615\ud0dc\ub85c \uc774\ud0c8\ub9ac\uc544\ub97c \ud48d\uc694\ub86d\uac8c \ud588\ub358 \ub3c8\uc904\uc758 \uace0\uac08\uc744 \uc758\ubbf8\ud588\ub2e4. \uc720\ub7fd\uc778\uc758 \uc544\uba54\ub9ac\uce74 \ub300\ub959 \uc0c1\ub959\uacfc \uc885\uad50\uac1c\ud601\uc73c\ub85c \uacb0\uad6d \uc774\ud0c8\ub9ac\uc544\ub294 \uc0c1\uc5c5\uc801 \ubb34\uc5ed\uc790\ubcf8\uacfc \uc790\ubcf8, \ub450\uac00\uc9c0\ub97c \ub3d9\uc2dc\uc5d0 \uc783\uc5c8\ub2e4 \ub610 \uc774\ud0c8\ub9ac\uc544\ub294 \uadf8 \ub2f9\uc2dc \uc815\uce58\uc801\uc73c\ub85c \uad49\uc7a5\ud788 \ud63c\ub780\uc2a4\ub7ec\uc6b4 \uc0c1\ud0dc\uc774\uae30\ub3c4 \ud558\uc600\ub2e4. \uc774\ub85c \uc778\ud574 \ub974\ub124\uc0c1\uc2a4\ub294 \uc1e0\ud1f4\ud560 \uc218\ubc16\uc5d0 \uc5c6\uc5c8\ub2e4.", "paragraph_answer": "130\ub144\uac04 \uc9c0\uc18d\ub418\ub358 \ub974\ub124\uc0c1\uc2a4\ub294 1530\ub144\uacbd \ub05d\uc774 \ub0ac\ub2e4. \uadf8 \uc774\uc720\ub294 \uc6b0\uc120 1492\ub144 \ud06c\ub9ac\uc2a4\ud1a0\ubc1c \ucf54\ub860 \uc774 \ud3ec\ub974\ud22c\uac08\uc778\ub4e4\uacfc \ud568\uaed8 \uc778\ub3c4\ub85c \uac00\ub294 \ud574\ub85c\ub97c \ubc1c\uacac\ud588\uae30 \ub54c\ubb38\uc774\ub2e4.(\uc0ac\uc2e4\uc740 \uc544\uba54\ub9ac\uce74\ub85c \uac00\ub294 \ud574\ub85c\uc600\ub2e4.) \uadf8 \ud6c4\uc5d0\ub294 \ubd81\uc11c\ubd80 \uc720\ub7fd\uc758 \uc0c1\uc778\ub4e4\uc774 \ubb34\uc5ed\uc0c1\ud488\uc744 \ub9ac\uc2a4\ubcf8\uacfc \uc548\ud2b8\uc6e8\ub974\ud39c\uc744 \ud1b5\ud574 \uac70\ub798\ud558\ub294 \uac83\uc744 \uc120\ud638\ud558\uac8c \ub418\uc5c8\ub2e4. \ub610\ud55c 1517\ub144 \uc544\uc6b0\uad6c\uc2a4\ud2f0\ub178\ud68c\uc758 \uc218\ub3c4\uc0ac\uc778 \ub9c8\ub974\ud2f4 \ub8e8\ud130\uac00 \uc885\uad50 \uac1c\ud601\uc744 \ub2e8\ud589\ud588\uae30 \ub54c\ubb38\uc774\ub2e4. \uc774\ub85c \uc778\ud574 \uc11c\uc720\ub7fd \uad50\ud68c\ub294 \ub85c\ub9c8 \uac00\ud1a8\ub9ad\uad50\ud68c\uc640 \uac1c\uc2e0\uad50 \uad50\ud68c\ub85c \ubd84\uc5f4\ub418\uc5c8\ub2e4. \uad50\ud68c\uc758 \ubd84\uc5f4\uc740 \uadf8\ub3d9\uc548 \ub0a9\ubd80\uae08\uacfc \uc138\uae08\uc758 \ud615\ud0dc\ub85c \uc774\ud0c8\ub9ac\uc544\ub97c \ud48d\uc694\ub86d\uac8c \ud588\ub358 \ub3c8\uc904\uc758 \uace0\uac08\uc744 \uc758\ubbf8\ud588\ub2e4. \uc720\ub7fd\uc778\uc758 \uc544\uba54\ub9ac\uce74 \ub300\ub959 \uc0c1\ub959\uacfc \uc885\uad50\uac1c\ud601\uc73c\ub85c \uacb0\uad6d \uc774\ud0c8\ub9ac\uc544\ub294 \uc0c1\uc5c5\uc801 \ubb34\uc5ed\uc790\ubcf8\uacfc \uc790\ubcf8, \ub450\uac00\uc9c0\ub97c \ub3d9\uc2dc\uc5d0 \uc783\uc5c8\ub2e4 \ub610 \uc774\ud0c8\ub9ac\uc544\ub294 \uadf8 \ub2f9\uc2dc \uc815\uce58\uc801\uc73c\ub85c \uad49\uc7a5\ud788 \ud63c\ub780\uc2a4\ub7ec\uc6b4 \uc0c1\ud0dc\uc774\uae30\ub3c4 \ud558\uc600\ub2e4. \uc774\ub85c \uc778\ud574 \ub974\ub124\uc0c1\uc2a4\ub294 \uc1e0\ud1f4\ud560 \uc218\ubc16\uc5d0 \uc5c6\uc5c8\ub2e4.", "sentence_answer": "\uadf8 \uc774\uc720\ub294 \uc6b0\uc120 1492\ub144 \ud06c\ub9ac\uc2a4\ud1a0\ubc1c \ucf54\ub860 \uc774 \ud3ec\ub974\ud22c\uac08\uc778\ub4e4\uacfc \ud568\uaed8 \uc778\ub3c4\ub85c \uac00\ub294 \ud574\ub85c\ub97c \ubc1c\uacac\ud588\uae30 \ub54c\ubb38\uc774\ub2e4.", "paragraph_id": "6498134-5-0", "paragraph_question": "question: 1492\ub144 \uc544\uba54\ub9ac\uce74 \ud574\ub85c\ub97c \ud3ec\ub974\ud22c\uac08\uc778\ub4e4\uacfc \ubc1c\uacac\ud588\ub358 \uc0ac\ub78c\uc740?, context: 130\ub144\uac04 \uc9c0\uc18d\ub418\ub358 \ub974\ub124\uc0c1\uc2a4\ub294 1530\ub144\uacbd \ub05d\uc774 \ub0ac\ub2e4. \uadf8 \uc774\uc720\ub294 \uc6b0\uc120 1492\ub144 \ud06c\ub9ac\uc2a4\ud1a0\ubc1c \ucf54\ub860\uc774 \ud3ec\ub974\ud22c\uac08\uc778\ub4e4\uacfc \ud568\uaed8 \uc778\ub3c4\ub85c \uac00\ub294 \ud574\ub85c\ub97c \ubc1c\uacac\ud588\uae30 \ub54c\ubb38\uc774\ub2e4.(\uc0ac\uc2e4\uc740 \uc544\uba54\ub9ac\uce74\ub85c \uac00\ub294 \ud574\ub85c\uc600\ub2e4.) \uadf8 \ud6c4\uc5d0\ub294 \ubd81\uc11c\ubd80 \uc720\ub7fd\uc758 \uc0c1\uc778\ub4e4\uc774 \ubb34\uc5ed\uc0c1\ud488\uc744 \ub9ac\uc2a4\ubcf8\uacfc \uc548\ud2b8\uc6e8\ub974\ud39c\uc744 \ud1b5\ud574 \uac70\ub798\ud558\ub294 \uac83\uc744 \uc120\ud638\ud558\uac8c \ub418\uc5c8\ub2e4. \ub610\ud55c 1517\ub144 \uc544\uc6b0\uad6c\uc2a4\ud2f0\ub178\ud68c\uc758 \uc218\ub3c4\uc0ac\uc778 \ub9c8\ub974\ud2f4 \ub8e8\ud130\uac00 \uc885\uad50 \uac1c\ud601\uc744 \ub2e8\ud589\ud588\uae30 \ub54c\ubb38\uc774\ub2e4. \uc774\ub85c \uc778\ud574 \uc11c\uc720\ub7fd \uad50\ud68c\ub294 \ub85c\ub9c8 \uac00\ud1a8\ub9ad\uad50\ud68c\uc640 \uac1c\uc2e0\uad50 \uad50\ud68c\ub85c \ubd84\uc5f4\ub418\uc5c8\ub2e4. \uad50\ud68c\uc758 \ubd84\uc5f4\uc740 \uadf8\ub3d9\uc548 \ub0a9\ubd80\uae08\uacfc \uc138\uae08\uc758 \ud615\ud0dc\ub85c \uc774\ud0c8\ub9ac\uc544\ub97c \ud48d\uc694\ub86d\uac8c \ud588\ub358 \ub3c8\uc904\uc758 \uace0\uac08\uc744 \uc758\ubbf8\ud588\ub2e4. \uc720\ub7fd\uc778\uc758 \uc544\uba54\ub9ac\uce74 \ub300\ub959 \uc0c1\ub959\uacfc \uc885\uad50\uac1c\ud601\uc73c\ub85c \uacb0\uad6d \uc774\ud0c8\ub9ac\uc544\ub294 \uc0c1\uc5c5\uc801 \ubb34\uc5ed\uc790\ubcf8\uacfc \uc790\ubcf8, \ub450\uac00\uc9c0\ub97c \ub3d9\uc2dc\uc5d0 \uc783\uc5c8\ub2e4 \ub610 \uc774\ud0c8\ub9ac\uc544\ub294 \uadf8 \ub2f9\uc2dc \uc815\uce58\uc801\uc73c\ub85c \uad49\uc7a5\ud788 \ud63c\ub780\uc2a4\ub7ec\uc6b4 \uc0c1\ud0dc\uc774\uae30\ub3c4 \ud558\uc600\ub2e4. \uc774\ub85c \uc778\ud574 \ub974\ub124\uc0c1\uc2a4\ub294 \uc1e0\ud1f4\ud560 \uc218\ubc16\uc5d0 \uc5c6\uc5c8\ub2e4."} {"question": "\uc548\ucca0\uc218\uac00 \uc874\ub313\ub9d0\uc744 \uc4f0\uac8c \ub41c \uc774\uc720\ub294 \ub204\uad6c\uc758 \uc601\ud5a5\uc778\uac00?", "paragraph": "\uc548\ucca0\uc218\ub294 \ubaa8\ub4e0 \uc0ac\ub78c\ub4e4\uc5d0\uac8c \ub098\uc774\uc640 \uc9c0\uc704\uace0\ud558\ub97c \ub9c9\ub860\ud558\uace0 \uc874\ub313\ub9d0\uc744 \uc4f4\ub2e4. \ub2e4\ub978 \uc0ac\ub78c\ub4e4\uc5d0\uac8c \ubc18\ub9d0\uc744 \ubabb\ud558\ub294 \uc548\ucca0\uc218\ub294 \uc2ec\uc9c0\uc5b4 \uad70\uc758\uad00\uc73c\ub85c \ubcf5\ubb34\ud558\ub358 \uc2dc\uc808\uc5d0\ub3c4 \ubcd1\uc0ac\ub4e4\uc5d0\uac8c \ubc18\ub9d0\uc744 \ud558\uc9c0 \ubabb\ud574 \uc560\ub97c \uba39\uae30\ub3c4 \ud588\ub2e4. \uac04\ud638\uc0ac\uc640 \uc548\ucca0\uc218\uc5f0\uad6c\uc18c \uc9c1\uc6d0\uacfc \uc790\uc2e0\uc774 \uac00\ub974\uce58\ub294 \ud559\uc0dd\ub4e4\ucc98\ub7fc \uc0ac\ud68c\uc801\uc778 \uc9c0\uc704\uac00 \uc790\uc2e0\ubcf4\ub2e4 \ub0ae\uc740 \uc0ac\ub78c\ub4e4\uacfc \uac00\uc871\uc5d0\uac8c\ub3c4 \uc874\ub313\ub9d0\uc744 \uc4f4\ub2e4\ub294 \uc810\uc774 \ud3c9\ubc94\ud55c \uc0ac\ub78c\ub4e4\uacfc \ub2e4\ub978 \uc548\ucca0\uc218\ub294 \ubd80\ubd80\uc2f8\uc6c0\uc744 \ud558\ub290\ub0d0\ub294 \uc9c8\ubb38\uc5d0 \u201c\uc874\ub313\ub9d0\ub85c \ud55c\ub2e4\u201d\uba70 \u201c\uc5b4\ub9b0 \uc2dc\uc808 \uc790\uc2e0\uc5d0\uac8c \ub298 \uc874\ub313\ub9d0\uc744 \uc4f0\uc2dc\ub358 \uc5b4\uba38\ub2c8\uc758 \uc601\ud5a5\uc774 \ucef8\ub2e4\u201d\ub77c\uace0 \ub9d0\ud588\ub2e4. \uc5b4\uba38\ub2c8\ub294 \uc548\ucca0\uc218\uc5d0\uac8c \ub9e4\ubc88 \uc874\ub313\ub9d0\ub85c \ub300\ud574\uc8fc\uc5c8\uc73c\uba70 \ud63c\ub0bc \ub54c\ub3c4 \ub9c8\ucc2c\uac00\uc9c0\uc600\ub294\ub370 \uadf8\ub3d9\uc548 \uc5b4\uba38\ub2c8\uc758 \uc874\ub313\ub9d0\uc744 \ub2f9\uc5f0\ud55c \uac83\uc73c\ub85c \uc0dd\uac01\ud558\uc600\uc73c\ub098 \uace0\ub4f1\ud559\uad50 1\ud559\ub144 \ub54c \ub2a6\uc7a0\uc744 \uc790\uc11c \ud0dd\uc2dc\ud0c0\uace0 \ub4f1\uad50\ud560 \ub54c \uc5b4\uba38\ub2c8\uac00 \u201c\ud559\uad50 \uc798 \ub2e4\ub140\uc624\uc138\uc694\u201d\ub77c\uace0 \ud558\uc790 \ud0dd\uc2dc\uae30\uc0ac\uac00 \ucc98\uc74c\uc5d0\ub294 \uc0ac\ucd0c\uc774\ub098 \uce5c\ub204\ub098\ub85c \ucc29\uac01\ud558\uace0 \u201c\ub204\ub098\uac00 \ucc38 \ucc29\ud558\ub124\uc694\u201d \ub77c\uace0 \ud558\uc600\uc73c\ub098 \ud6c4\uc5d0 \uc5b4\uba38\ub2c8\ub77c\uace0 \ubc1d\ud788\uba74\uc11c \u201c\uc5b4\ub5bb\uac8c \uc5b4\uba38\ub2c8\uac00 \uc874\ub313\ub9d0\uc744 \ud560 \uc218 \uc788\ub290\ub0d0\u201d\ub77c\ub294 \ub9d0\uc744 \ub4e3\uace0 \uc5b4\uba38\ub2c8\uc758 \uc874\ub313\ub9d0\uc774 \ud3c9\ubc94\ud558\uc9c0 \uc54a\uc740 \uac83\uc784\uc744 \uae68\ub2eb\uac8c \ub410\ub2e4\uace0 \ud55c\ub2e4.", "answer": "\uc5b4\uba38\ub2c8", "sentence": "\uac04\ud638\uc0ac\uc640 \uc548\ucca0\uc218\uc5f0\uad6c\uc18c \uc9c1\uc6d0\uacfc \uc790\uc2e0\uc774 \uac00\ub974\uce58\ub294 \ud559\uc0dd\ub4e4\ucc98\ub7fc \uc0ac\ud68c\uc801\uc778 \uc9c0\uc704\uac00 \uc790\uc2e0\ubcf4\ub2e4 \ub0ae\uc740 \uc0ac\ub78c\ub4e4\uacfc \uac00\uc871\uc5d0\uac8c\ub3c4 \uc874\ub313\ub9d0\uc744 \uc4f4\ub2e4\ub294 \uc810\uc774 \ud3c9\ubc94\ud55c \uc0ac\ub78c\ub4e4\uacfc \ub2e4\ub978 \uc548\ucca0\uc218\ub294 \ubd80\ubd80\uc2f8\uc6c0\uc744 \ud558\ub290\ub0d0\ub294 \uc9c8\ubb38\uc5d0 \u201c\uc874\ub313\ub9d0\ub85c \ud55c\ub2e4\u201d\uba70 \u201c\uc5b4\ub9b0 \uc2dc\uc808 \uc790\uc2e0\uc5d0\uac8c \ub298 \uc874\ub313\ub9d0\uc744 \uc4f0\uc2dc\ub358 \uc5b4\uba38\ub2c8 \uc758 \uc601\ud5a5\uc774 \ucef8\ub2e4\u201d\ub77c\uace0 \ub9d0\ud588\ub2e4.", "paragraph_sentence": "\uc548\ucca0\uc218\ub294 \ubaa8\ub4e0 \uc0ac\ub78c\ub4e4\uc5d0\uac8c \ub098\uc774\uc640 \uc9c0\uc704\uace0\ud558\ub97c \ub9c9\ub860\ud558\uace0 \uc874\ub313\ub9d0\uc744 \uc4f4\ub2e4. \ub2e4\ub978 \uc0ac\ub78c\ub4e4\uc5d0\uac8c \ubc18\ub9d0\uc744 \ubabb\ud558\ub294 \uc548\ucca0\uc218\ub294 \uc2ec\uc9c0\uc5b4 \uad70\uc758\uad00\uc73c\ub85c \ubcf5\ubb34\ud558\ub358 \uc2dc\uc808\uc5d0\ub3c4 \ubcd1\uc0ac\ub4e4\uc5d0\uac8c \ubc18\ub9d0\uc744 \ud558\uc9c0 \ubabb\ud574 \uc560\ub97c \uba39\uae30\ub3c4 \ud588\ub2e4. \uac04\ud638\uc0ac\uc640 \uc548\ucca0\uc218\uc5f0\uad6c\uc18c \uc9c1\uc6d0\uacfc \uc790\uc2e0\uc774 \uac00\ub974\uce58\ub294 \ud559\uc0dd\ub4e4\ucc98\ub7fc \uc0ac\ud68c\uc801\uc778 \uc9c0\uc704\uac00 \uc790\uc2e0\ubcf4\ub2e4 \ub0ae\uc740 \uc0ac\ub78c\ub4e4\uacfc \uac00\uc871\uc5d0\uac8c\ub3c4 \uc874\ub313\ub9d0\uc744 \uc4f4\ub2e4\ub294 \uc810\uc774 \ud3c9\ubc94\ud55c \uc0ac\ub78c\ub4e4\uacfc \ub2e4\ub978 \uc548\ucca0\uc218\ub294 \ubd80\ubd80\uc2f8\uc6c0\uc744 \ud558\ub290\ub0d0\ub294 \uc9c8\ubb38\uc5d0 \u201c\uc874\ub313\ub9d0\ub85c \ud55c\ub2e4\u201d\uba70 \u201c\uc5b4\ub9b0 \uc2dc\uc808 \uc790\uc2e0\uc5d0\uac8c \ub298 \uc874\ub313\ub9d0\uc744 \uc4f0\uc2dc\ub358 \uc5b4\uba38\ub2c8 \uc758 \uc601\ud5a5\uc774 \ucef8\ub2e4\u201d\ub77c\uace0 \ub9d0\ud588\ub2e4. \uc5b4\uba38\ub2c8\ub294 \uc548\ucca0\uc218\uc5d0\uac8c \ub9e4\ubc88 \uc874\ub313\ub9d0\ub85c \ub300\ud574\uc8fc\uc5c8\uc73c\uba70 \ud63c\ub0bc \ub54c\ub3c4 \ub9c8\ucc2c\uac00\uc9c0\uc600\ub294\ub370 \uadf8\ub3d9\uc548 \uc5b4\uba38\ub2c8\uc758 \uc874\ub313\ub9d0\uc744 \ub2f9\uc5f0\ud55c \uac83\uc73c\ub85c \uc0dd\uac01\ud558\uc600\uc73c\ub098 \uace0\ub4f1\ud559\uad50 1\ud559\ub144 \ub54c \ub2a6\uc7a0\uc744 \uc790\uc11c \ud0dd\uc2dc\ud0c0\uace0 \ub4f1\uad50\ud560 \ub54c \uc5b4\uba38\ub2c8\uac00 \u201c\ud559\uad50 \uc798 \ub2e4\ub140\uc624\uc138\uc694\u201d\ub77c\uace0 \ud558\uc790 \ud0dd\uc2dc\uae30\uc0ac\uac00 \ucc98\uc74c\uc5d0\ub294 \uc0ac\ucd0c\uc774\ub098 \uce5c\ub204\ub098\ub85c \ucc29\uac01\ud558\uace0 \u201c\ub204\ub098\uac00 \ucc38 \ucc29\ud558\ub124\uc694\u201d \ub77c\uace0 \ud558\uc600\uc73c\ub098 \ud6c4\uc5d0 \uc5b4\uba38\ub2c8\ub77c\uace0 \ubc1d\ud788\uba74\uc11c \u201c\uc5b4\ub5bb\uac8c \uc5b4\uba38\ub2c8\uac00 \uc874\ub313\ub9d0\uc744 \ud560 \uc218 \uc788\ub290\ub0d0\u201d\ub77c\ub294 \ub9d0\uc744 \ub4e3\uace0 \uc5b4\uba38\ub2c8\uc758 \uc874\ub313\ub9d0\uc774 \ud3c9\ubc94\ud558\uc9c0 \uc54a\uc740 \uac83\uc784\uc744 \uae68\ub2eb\uac8c \ub410\ub2e4\uace0 \ud55c\ub2e4.", "paragraph_answer": "\uc548\ucca0\uc218\ub294 \ubaa8\ub4e0 \uc0ac\ub78c\ub4e4\uc5d0\uac8c \ub098\uc774\uc640 \uc9c0\uc704\uace0\ud558\ub97c \ub9c9\ub860\ud558\uace0 \uc874\ub313\ub9d0\uc744 \uc4f4\ub2e4. \ub2e4\ub978 \uc0ac\ub78c\ub4e4\uc5d0\uac8c \ubc18\ub9d0\uc744 \ubabb\ud558\ub294 \uc548\ucca0\uc218\ub294 \uc2ec\uc9c0\uc5b4 \uad70\uc758\uad00\uc73c\ub85c \ubcf5\ubb34\ud558\ub358 \uc2dc\uc808\uc5d0\ub3c4 \ubcd1\uc0ac\ub4e4\uc5d0\uac8c \ubc18\ub9d0\uc744 \ud558\uc9c0 \ubabb\ud574 \uc560\ub97c \uba39\uae30\ub3c4 \ud588\ub2e4. \uac04\ud638\uc0ac\uc640 \uc548\ucca0\uc218\uc5f0\uad6c\uc18c \uc9c1\uc6d0\uacfc \uc790\uc2e0\uc774 \uac00\ub974\uce58\ub294 \ud559\uc0dd\ub4e4\ucc98\ub7fc \uc0ac\ud68c\uc801\uc778 \uc9c0\uc704\uac00 \uc790\uc2e0\ubcf4\ub2e4 \ub0ae\uc740 \uc0ac\ub78c\ub4e4\uacfc \uac00\uc871\uc5d0\uac8c\ub3c4 \uc874\ub313\ub9d0\uc744 \uc4f4\ub2e4\ub294 \uc810\uc774 \ud3c9\ubc94\ud55c \uc0ac\ub78c\ub4e4\uacfc \ub2e4\ub978 \uc548\ucca0\uc218\ub294 \ubd80\ubd80\uc2f8\uc6c0\uc744 \ud558\ub290\ub0d0\ub294 \uc9c8\ubb38\uc5d0 \u201c\uc874\ub313\ub9d0\ub85c \ud55c\ub2e4\u201d\uba70 \u201c\uc5b4\ub9b0 \uc2dc\uc808 \uc790\uc2e0\uc5d0\uac8c \ub298 \uc874\ub313\ub9d0\uc744 \uc4f0\uc2dc\ub358 \uc5b4\uba38\ub2c8 \uc758 \uc601\ud5a5\uc774 \ucef8\ub2e4\u201d\ub77c\uace0 \ub9d0\ud588\ub2e4. \uc5b4\uba38\ub2c8\ub294 \uc548\ucca0\uc218\uc5d0\uac8c \ub9e4\ubc88 \uc874\ub313\ub9d0\ub85c \ub300\ud574\uc8fc\uc5c8\uc73c\uba70 \ud63c\ub0bc \ub54c\ub3c4 \ub9c8\ucc2c\uac00\uc9c0\uc600\ub294\ub370 \uadf8\ub3d9\uc548 \uc5b4\uba38\ub2c8\uc758 \uc874\ub313\ub9d0\uc744 \ub2f9\uc5f0\ud55c \uac83\uc73c\ub85c \uc0dd\uac01\ud558\uc600\uc73c\ub098 \uace0\ub4f1\ud559\uad50 1\ud559\ub144 \ub54c \ub2a6\uc7a0\uc744 \uc790\uc11c \ud0dd\uc2dc\ud0c0\uace0 \ub4f1\uad50\ud560 \ub54c \uc5b4\uba38\ub2c8\uac00 \u201c\ud559\uad50 \uc798 \ub2e4\ub140\uc624\uc138\uc694\u201d\ub77c\uace0 \ud558\uc790 \ud0dd\uc2dc\uae30\uc0ac\uac00 \ucc98\uc74c\uc5d0\ub294 \uc0ac\ucd0c\uc774\ub098 \uce5c\ub204\ub098\ub85c \ucc29\uac01\ud558\uace0 \u201c\ub204\ub098\uac00 \ucc38 \ucc29\ud558\ub124\uc694\u201d \ub77c\uace0 \ud558\uc600\uc73c\ub098 \ud6c4\uc5d0 \uc5b4\uba38\ub2c8\ub77c\uace0 \ubc1d\ud788\uba74\uc11c \u201c\uc5b4\ub5bb\uac8c \uc5b4\uba38\ub2c8\uac00 \uc874\ub313\ub9d0\uc744 \ud560 \uc218 \uc788\ub290\ub0d0\u201d\ub77c\ub294 \ub9d0\uc744 \ub4e3\uace0 \uc5b4\uba38\ub2c8\uc758 \uc874\ub313\ub9d0\uc774 \ud3c9\ubc94\ud558\uc9c0 \uc54a\uc740 \uac83\uc784\uc744 \uae68\ub2eb\uac8c \ub410\ub2e4\uace0 \ud55c\ub2e4.", "sentence_answer": "\uac04\ud638\uc0ac\uc640 \uc548\ucca0\uc218\uc5f0\uad6c\uc18c \uc9c1\uc6d0\uacfc \uc790\uc2e0\uc774 \uac00\ub974\uce58\ub294 \ud559\uc0dd\ub4e4\ucc98\ub7fc \uc0ac\ud68c\uc801\uc778 \uc9c0\uc704\uac00 \uc790\uc2e0\ubcf4\ub2e4 \ub0ae\uc740 \uc0ac\ub78c\ub4e4\uacfc \uac00\uc871\uc5d0\uac8c\ub3c4 \uc874\ub313\ub9d0\uc744 \uc4f4\ub2e4\ub294 \uc810\uc774 \ud3c9\ubc94\ud55c \uc0ac\ub78c\ub4e4\uacfc \ub2e4\ub978 \uc548\ucca0\uc218\ub294 \ubd80\ubd80\uc2f8\uc6c0\uc744 \ud558\ub290\ub0d0\ub294 \uc9c8\ubb38\uc5d0 \u201c\uc874\ub313\ub9d0\ub85c \ud55c\ub2e4\u201d\uba70 \u201c\uc5b4\ub9b0 \uc2dc\uc808 \uc790\uc2e0\uc5d0\uac8c \ub298 \uc874\ub313\ub9d0\uc744 \uc4f0\uc2dc\ub358 \uc5b4\uba38\ub2c8 \uc758 \uc601\ud5a5\uc774 \ucef8\ub2e4\u201d\ub77c\uace0 \ub9d0\ud588\ub2e4.", "paragraph_id": "6547507-19-1", "paragraph_question": "question: \uc548\ucca0\uc218\uac00 \uc874\ub313\ub9d0\uc744 \uc4f0\uac8c \ub41c \uc774\uc720\ub294 \ub204\uad6c\uc758 \uc601\ud5a5\uc778\uac00?, context: \uc548\ucca0\uc218\ub294 \ubaa8\ub4e0 \uc0ac\ub78c\ub4e4\uc5d0\uac8c \ub098\uc774\uc640 \uc9c0\uc704\uace0\ud558\ub97c \ub9c9\ub860\ud558\uace0 \uc874\ub313\ub9d0\uc744 \uc4f4\ub2e4. \ub2e4\ub978 \uc0ac\ub78c\ub4e4\uc5d0\uac8c \ubc18\ub9d0\uc744 \ubabb\ud558\ub294 \uc548\ucca0\uc218\ub294 \uc2ec\uc9c0\uc5b4 \uad70\uc758\uad00\uc73c\ub85c \ubcf5\ubb34\ud558\ub358 \uc2dc\uc808\uc5d0\ub3c4 \ubcd1\uc0ac\ub4e4\uc5d0\uac8c \ubc18\ub9d0\uc744 \ud558\uc9c0 \ubabb\ud574 \uc560\ub97c \uba39\uae30\ub3c4 \ud588\ub2e4. \uac04\ud638\uc0ac\uc640 \uc548\ucca0\uc218\uc5f0\uad6c\uc18c \uc9c1\uc6d0\uacfc \uc790\uc2e0\uc774 \uac00\ub974\uce58\ub294 \ud559\uc0dd\ub4e4\ucc98\ub7fc \uc0ac\ud68c\uc801\uc778 \uc9c0\uc704\uac00 \uc790\uc2e0\ubcf4\ub2e4 \ub0ae\uc740 \uc0ac\ub78c\ub4e4\uacfc \uac00\uc871\uc5d0\uac8c\ub3c4 \uc874\ub313\ub9d0\uc744 \uc4f4\ub2e4\ub294 \uc810\uc774 \ud3c9\ubc94\ud55c \uc0ac\ub78c\ub4e4\uacfc \ub2e4\ub978 \uc548\ucca0\uc218\ub294 \ubd80\ubd80\uc2f8\uc6c0\uc744 \ud558\ub290\ub0d0\ub294 \uc9c8\ubb38\uc5d0 \u201c\uc874\ub313\ub9d0\ub85c \ud55c\ub2e4\u201d\uba70 \u201c\uc5b4\ub9b0 \uc2dc\uc808 \uc790\uc2e0\uc5d0\uac8c \ub298 \uc874\ub313\ub9d0\uc744 \uc4f0\uc2dc\ub358 \uc5b4\uba38\ub2c8\uc758 \uc601\ud5a5\uc774 \ucef8\ub2e4\u201d\ub77c\uace0 \ub9d0\ud588\ub2e4. \uc5b4\uba38\ub2c8\ub294 \uc548\ucca0\uc218\uc5d0\uac8c \ub9e4\ubc88 \uc874\ub313\ub9d0\ub85c \ub300\ud574\uc8fc\uc5c8\uc73c\uba70 \ud63c\ub0bc \ub54c\ub3c4 \ub9c8\ucc2c\uac00\uc9c0\uc600\ub294\ub370 \uadf8\ub3d9\uc548 \uc5b4\uba38\ub2c8\uc758 \uc874\ub313\ub9d0\uc744 \ub2f9\uc5f0\ud55c \uac83\uc73c\ub85c \uc0dd\uac01\ud558\uc600\uc73c\ub098 \uace0\ub4f1\ud559\uad50 1\ud559\ub144 \ub54c \ub2a6\uc7a0\uc744 \uc790\uc11c \ud0dd\uc2dc\ud0c0\uace0 \ub4f1\uad50\ud560 \ub54c \uc5b4\uba38\ub2c8\uac00 \u201c\ud559\uad50 \uc798 \ub2e4\ub140\uc624\uc138\uc694\u201d\ub77c\uace0 \ud558\uc790 \ud0dd\uc2dc\uae30\uc0ac\uac00 \ucc98\uc74c\uc5d0\ub294 \uc0ac\ucd0c\uc774\ub098 \uce5c\ub204\ub098\ub85c \ucc29\uac01\ud558\uace0 \u201c\ub204\ub098\uac00 \ucc38 \ucc29\ud558\ub124\uc694\u201d \ub77c\uace0 \ud558\uc600\uc73c\ub098 \ud6c4\uc5d0 \uc5b4\uba38\ub2c8\ub77c\uace0 \ubc1d\ud788\uba74\uc11c \u201c\uc5b4\ub5bb\uac8c \uc5b4\uba38\ub2c8\uac00 \uc874\ub313\ub9d0\uc744 \ud560 \uc218 \uc788\ub290\ub0d0\u201d\ub77c\ub294 \ub9d0\uc744 \ub4e3\uace0 \uc5b4\uba38\ub2c8\uc758 \uc874\ub313\ub9d0\uc774 \ud3c9\ubc94\ud558\uc9c0 \uc54a\uc740 \uac83\uc784\uc744 \uae68\ub2eb\uac8c \ub410\ub2e4\uace0 \ud55c\ub2e4."} {"question": "\uc720\ud608\uc0ac\ud0dc\uac00 \ubc1c\uc0dd\ud558\uc790 \ub85c\ub9c8 \uc2dc \uc7a5\uad00\ub4e4\uc774 \uace8 \uc9c0\uc5ed\uc73c\ub85c \ucd94\ubc29\ud55c \uc778\ubb3c\uc740 \ub204\uad6c\uc778\uac00?", "paragraph": "366\ub144 9\uc6d4 24\uc77c \ub9ac\ubca0\ub9ac\uc624\uac00 \uc120\uc885\ud558\uc790 \ub85c\ub9c8 \uad50\ud68c\ub294 \ub450 \ud30c\ubc8c\ub85c \ubd84\uc5f4\ub418\uc5c8\ub2e4. \ud558\ub098\ub294 \ub9ac\ubca0\ub9ac\uc624\uc758 \ubd80\uc81c\ub85c \ubd09\uc0ac\ud558\uc600\ub358 \uc6b0\ub974\uc2dc\ub178\ub97c \ucd94\uc885\ud558\ub294 \uc138\ub825\uc774\uc5c8\uc73c\uba70, \ub098\uba38\uc9c0 \ud558\ub098\ub294 \ud55c\ub54c \ub300\ub9bd \uad50\ud669 \ud3a0\ub9ad\uc2a4 2\uc138\uc5d0\uac8c \ucda9\uc131\ud558\uc600\ub358 \ub2e4\ub9c8\uc18c\ub97c \ucd94\uc885\ud558\ub294 \uc138\ub825\uc774\uc5c8\ub2e4. \ub300\ub9bd \uad50\ud669 \ud3a0\ub9ad\uc2a4 2\uc138\ub97c \ub530\ub790\ub358 \uadc0\uc871\uce35\uc740 \ub2e4\ub9c8\uc18c\ub97c \uc9c0\uc9c0\ud55c \ubc18\uba74, \ub9ac\ubca0\ub9ac\uc624\ub97c \ub530\ub790\ub358 \ubd80\uc81c\ub4e4\uacfc \ud3c9\uc2e0\ub3c4\ub4e4\uc740 \uc6b0\ub974\uc2dc\ub178\ub97c \uc9c0\uc9c0\ud558\uc600\ub2e4. \ub2f9\uc7a5 \ud3ed\ub3d9\uc774\ub77c\ub3c4 \uc77c\uc5b4\ub0a0 \uac83\ub9cc \uac19\uc740 \uc0bc\uc5c4\ud55c \ubd84\uc704\uae30 \uc18d\uc5d0\uc11c \ub2e4\ub9c8\uc18c\uc640 \uc6b0\ub974\uc2dc\ub178 \ub450 \uc0ac\ub78c\uc740 \uac01\uac01 \uc591 \uc9c4\uc601\uc5d0\uc11c \ub85c\ub9c8\uc758 \uc8fc\uad50\ub85c \uc120\ucd9c\ub418\uc5c8\ub2e4. \ub2e4\ub9c8\uc18c\ub294 \uc0b0 \ub85c\ub80c\ucd08 \uc778 \ub8e8\uce58\ub098 \uc131\ub2f9\uc5d0\uc11c \ub85c\ub9c8 \uc8fc\uad50\ub85c \uc120\ucd9c\ub418\uc5c8\ub2e4. \uadf8\ub9ac\ud558\uc5ec \uacb0\uad6d \ub450 \uc0ac\ub78c\uc758 \uc9c0\uc9c0\uc790\ub4e4\uc740 \uadf8\ud574 10\uc6d4\uc5d0 \ucda9\ub3cc\ud558\uac8c \ub418\uc5c8\ub2e4. \uc591 \uc9c4\uc601\uc740 \ub85c\ub9c8 \uad50\uc678\uc5d0\uc11c \ucda9\ub3cc\ud558\uc5ec \uacb0\uad6d \uc2dc\uc2e0\ub2c8\uc624 \uc131\ub2f9(\ud6d7\ub0a0\uc758 \uc0b0\ud0c0 \ub9c8\ub9ac\uc544 \ub9c8\uc870\ub808 \ub300\uc131\uc804)\uc5d0\uc11c 137\uba85\uc774 \ub300\ub7c9 \ud559\uc0b4\ub418\ub294 \uc0ac\uac74\uc774 \uc77c\uc5b4\ub098\uae30\uc5d0 \uc774\ub974\ub800\ub2e4. \uc774\ub7ec\ud55c \ud3ed\ub825\uc801\uc778 \uc720\ud608 \uc0ac\ud0dc\uac00 \ubc1c\ubc1c\ud558\uc790 \ub85c\ub9c8 \uc2dc \uc7a5\uad00\ub4e4\uc774 \uac1c\uc785\ud558\uc5ec \uac00\uae4c\uc2a4\ub85c \uc9c4\uc555\ud558\ub294\ub370 \uc131\uacf5\ud558\uc600\ub2e4. \uadf8\ub9ac\uace0 \uc7a5\uad00\ub4e4\uc740 \uc6b0\ub974\uc2dc\ub178\ub97c \uace8 \uc9c0\uc5ed\uc73c\ub85c \ucd94\ubc29\ud558\uc600\ub2e4. \uadf8\ub7ec\ub098 \uc6b0\ub974\uc2dc\ub178\uac00 \ucd94\ubc29\ub41c \ud6c4\uc5d0\ub3c4 \uc591\uce21 \uc9c0\uc9c0\uc790\ub4e4 \uac04\uc758 \ucda9\ub3cc\uc740 \ub04a\uc774\uc9c0 \uc54a\uace0 \uc624\ud788\ub824 \ub354 \uc2ec\ud654\ub418\uc5c8\ub2e4.", "answer": "\uc6b0\ub974\uc2dc\ub178\ub97c", "sentence": "\ud558\ub098\ub294 \ub9ac\ubca0\ub9ac\uc624\uc758 \ubd80\uc81c\ub85c \ubd09\uc0ac\ud558\uc600\ub358 \uc6b0\ub974\uc2dc\ub178\ub97c \ucd94\uc885\ud558\ub294 \uc138\ub825\uc774\uc5c8\uc73c\uba70, \ub098\uba38\uc9c0 \ud558\ub098\ub294 \ud55c\ub54c \ub300\ub9bd \uad50\ud669 \ud3a0\ub9ad\uc2a4 2\uc138\uc5d0\uac8c \ucda9\uc131\ud558\uc600\ub358 \ub2e4\ub9c8\uc18c\ub97c \ucd94\uc885\ud558\ub294 \uc138\ub825\uc774\uc5c8\ub2e4.", "paragraph_sentence": "366\ub144 9\uc6d4 24\uc77c \ub9ac\ubca0\ub9ac\uc624\uac00 \uc120\uc885\ud558\uc790 \ub85c\ub9c8 \uad50\ud68c\ub294 \ub450 \ud30c\ubc8c\ub85c \ubd84\uc5f4\ub418\uc5c8\ub2e4. \ud558\ub098\ub294 \ub9ac\ubca0\ub9ac\uc624\uc758 \ubd80\uc81c\ub85c \ubd09\uc0ac\ud558\uc600\ub358 \uc6b0\ub974\uc2dc\ub178\ub97c \ucd94\uc885\ud558\ub294 \uc138\ub825\uc774\uc5c8\uc73c\uba70, \ub098\uba38\uc9c0 \ud558\ub098\ub294 \ud55c\ub54c \ub300\ub9bd \uad50\ud669 \ud3a0\ub9ad\uc2a4 2\uc138\uc5d0\uac8c \ucda9\uc131\ud558\uc600\ub358 \ub2e4\ub9c8\uc18c\ub97c \ucd94\uc885\ud558\ub294 \uc138\ub825\uc774\uc5c8\ub2e4. \ub300\ub9bd \uad50\ud669 \ud3a0\ub9ad\uc2a4 2\uc138\ub97c \ub530\ub790\ub358 \uadc0\uc871\uce35\uc740 \ub2e4\ub9c8\uc18c\ub97c \uc9c0\uc9c0\ud55c \ubc18\uba74, \ub9ac\ubca0\ub9ac\uc624\ub97c \ub530\ub790\ub358 \ubd80\uc81c\ub4e4\uacfc \ud3c9\uc2e0\ub3c4\ub4e4\uc740 \uc6b0\ub974\uc2dc\ub178\ub97c \uc9c0\uc9c0\ud558\uc600\ub2e4. \ub2f9\uc7a5 \ud3ed\ub3d9\uc774\ub77c\ub3c4 \uc77c\uc5b4\ub0a0 \uac83\ub9cc \uac19\uc740 \uc0bc\uc5c4\ud55c \ubd84\uc704\uae30 \uc18d\uc5d0\uc11c \ub2e4\ub9c8\uc18c\uc640 \uc6b0\ub974\uc2dc\ub178 \ub450 \uc0ac\ub78c\uc740 \uac01\uac01 \uc591 \uc9c4\uc601\uc5d0\uc11c \ub85c\ub9c8\uc758 \uc8fc\uad50\ub85c \uc120\ucd9c\ub418\uc5c8\ub2e4. \ub2e4\ub9c8\uc18c\ub294 \uc0b0 \ub85c\ub80c\ucd08 \uc778 \ub8e8\uce58\ub098 \uc131\ub2f9\uc5d0\uc11c \ub85c\ub9c8 \uc8fc\uad50\ub85c \uc120\ucd9c\ub418\uc5c8\ub2e4. \uadf8\ub9ac\ud558\uc5ec \uacb0\uad6d \ub450 \uc0ac\ub78c\uc758 \uc9c0\uc9c0\uc790\ub4e4\uc740 \uadf8\ud574 10\uc6d4\uc5d0 \ucda9\ub3cc\ud558\uac8c \ub418\uc5c8\ub2e4. \uc591 \uc9c4\uc601\uc740 \ub85c\ub9c8 \uad50\uc678\uc5d0\uc11c \ucda9\ub3cc\ud558\uc5ec \uacb0\uad6d \uc2dc\uc2e0\ub2c8\uc624 \uc131\ub2f9(\ud6d7\ub0a0\uc758 \uc0b0\ud0c0 \ub9c8\ub9ac\uc544 \ub9c8\uc870\ub808 \ub300\uc131\uc804)\uc5d0\uc11c 137\uba85\uc774 \ub300\ub7c9 \ud559\uc0b4\ub418\ub294 \uc0ac\uac74\uc774 \uc77c\uc5b4\ub098\uae30\uc5d0 \uc774\ub974\ub800\ub2e4. \uc774\ub7ec\ud55c \ud3ed\ub825\uc801\uc778 \uc720\ud608 \uc0ac\ud0dc\uac00 \ubc1c\ubc1c\ud558\uc790 \ub85c\ub9c8 \uc2dc \uc7a5\uad00\ub4e4\uc774 \uac1c\uc785\ud558\uc5ec \uac00\uae4c\uc2a4\ub85c \uc9c4\uc555\ud558\ub294\ub370 \uc131\uacf5\ud558\uc600\ub2e4. \uadf8\ub9ac\uace0 \uc7a5\uad00\ub4e4\uc740 \uc6b0\ub974\uc2dc\ub178\ub97c \uace8 \uc9c0\uc5ed\uc73c\ub85c \ucd94\ubc29\ud558\uc600\ub2e4. \uadf8\ub7ec\ub098 \uc6b0\ub974\uc2dc\ub178\uac00 \ucd94\ubc29\ub41c \ud6c4\uc5d0\ub3c4 \uc591\uce21 \uc9c0\uc9c0\uc790\ub4e4 \uac04\uc758 \ucda9\ub3cc\uc740 \ub04a\uc774\uc9c0 \uc54a\uace0 \uc624\ud788\ub824 \ub354 \uc2ec\ud654\ub418\uc5c8\ub2e4.", "paragraph_answer": "366\ub144 9\uc6d4 24\uc77c \ub9ac\ubca0\ub9ac\uc624\uac00 \uc120\uc885\ud558\uc790 \ub85c\ub9c8 \uad50\ud68c\ub294 \ub450 \ud30c\ubc8c\ub85c \ubd84\uc5f4\ub418\uc5c8\ub2e4. \ud558\ub098\ub294 \ub9ac\ubca0\ub9ac\uc624\uc758 \ubd80\uc81c\ub85c \ubd09\uc0ac\ud558\uc600\ub358 \uc6b0\ub974\uc2dc\ub178\ub97c \ucd94\uc885\ud558\ub294 \uc138\ub825\uc774\uc5c8\uc73c\uba70, \ub098\uba38\uc9c0 \ud558\ub098\ub294 \ud55c\ub54c \ub300\ub9bd \uad50\ud669 \ud3a0\ub9ad\uc2a4 2\uc138\uc5d0\uac8c \ucda9\uc131\ud558\uc600\ub358 \ub2e4\ub9c8\uc18c\ub97c \ucd94\uc885\ud558\ub294 \uc138\ub825\uc774\uc5c8\ub2e4. \ub300\ub9bd \uad50\ud669 \ud3a0\ub9ad\uc2a4 2\uc138\ub97c \ub530\ub790\ub358 \uadc0\uc871\uce35\uc740 \ub2e4\ub9c8\uc18c\ub97c \uc9c0\uc9c0\ud55c \ubc18\uba74, \ub9ac\ubca0\ub9ac\uc624\ub97c \ub530\ub790\ub358 \ubd80\uc81c\ub4e4\uacfc \ud3c9\uc2e0\ub3c4\ub4e4\uc740 \uc6b0\ub974\uc2dc\ub178\ub97c \uc9c0\uc9c0\ud558\uc600\ub2e4. \ub2f9\uc7a5 \ud3ed\ub3d9\uc774\ub77c\ub3c4 \uc77c\uc5b4\ub0a0 \uac83\ub9cc \uac19\uc740 \uc0bc\uc5c4\ud55c \ubd84\uc704\uae30 \uc18d\uc5d0\uc11c \ub2e4\ub9c8\uc18c\uc640 \uc6b0\ub974\uc2dc\ub178 \ub450 \uc0ac\ub78c\uc740 \uac01\uac01 \uc591 \uc9c4\uc601\uc5d0\uc11c \ub85c\ub9c8\uc758 \uc8fc\uad50\ub85c \uc120\ucd9c\ub418\uc5c8\ub2e4. \ub2e4\ub9c8\uc18c\ub294 \uc0b0 \ub85c\ub80c\ucd08 \uc778 \ub8e8\uce58\ub098 \uc131\ub2f9\uc5d0\uc11c \ub85c\ub9c8 \uc8fc\uad50\ub85c \uc120\ucd9c\ub418\uc5c8\ub2e4. \uadf8\ub9ac\ud558\uc5ec \uacb0\uad6d \ub450 \uc0ac\ub78c\uc758 \uc9c0\uc9c0\uc790\ub4e4\uc740 \uadf8\ud574 10\uc6d4\uc5d0 \ucda9\ub3cc\ud558\uac8c \ub418\uc5c8\ub2e4. \uc591 \uc9c4\uc601\uc740 \ub85c\ub9c8 \uad50\uc678\uc5d0\uc11c \ucda9\ub3cc\ud558\uc5ec \uacb0\uad6d \uc2dc\uc2e0\ub2c8\uc624 \uc131\ub2f9(\ud6d7\ub0a0\uc758 \uc0b0\ud0c0 \ub9c8\ub9ac\uc544 \ub9c8\uc870\ub808 \ub300\uc131\uc804)\uc5d0\uc11c 137\uba85\uc774 \ub300\ub7c9 \ud559\uc0b4\ub418\ub294 \uc0ac\uac74\uc774 \uc77c\uc5b4\ub098\uae30\uc5d0 \uc774\ub974\ub800\ub2e4. \uc774\ub7ec\ud55c \ud3ed\ub825\uc801\uc778 \uc720\ud608 \uc0ac\ud0dc\uac00 \ubc1c\ubc1c\ud558\uc790 \ub85c\ub9c8 \uc2dc \uc7a5\uad00\ub4e4\uc774 \uac1c\uc785\ud558\uc5ec \uac00\uae4c\uc2a4\ub85c \uc9c4\uc555\ud558\ub294\ub370 \uc131\uacf5\ud558\uc600\ub2e4. \uadf8\ub9ac\uace0 \uc7a5\uad00\ub4e4\uc740 \uc6b0\ub974\uc2dc\ub178\ub97c \uace8 \uc9c0\uc5ed\uc73c\ub85c \ucd94\ubc29\ud558\uc600\ub2e4. \uadf8\ub7ec\ub098 \uc6b0\ub974\uc2dc\ub178\uac00 \ucd94\ubc29\ub41c \ud6c4\uc5d0\ub3c4 \uc591\uce21 \uc9c0\uc9c0\uc790\ub4e4 \uac04\uc758 \ucda9\ub3cc\uc740 \ub04a\uc774\uc9c0 \uc54a\uace0 \uc624\ud788\ub824 \ub354 \uc2ec\ud654\ub418\uc5c8\ub2e4.", "sentence_answer": "\ud558\ub098\ub294 \ub9ac\ubca0\ub9ac\uc624\uc758 \ubd80\uc81c\ub85c \ubd09\uc0ac\ud558\uc600\ub358 \uc6b0\ub974\uc2dc\ub178\ub97c \ucd94\uc885\ud558\ub294 \uc138\ub825\uc774\uc5c8\uc73c\uba70, \ub098\uba38\uc9c0 \ud558\ub098\ub294 \ud55c\ub54c \ub300\ub9bd \uad50\ud669 \ud3a0\ub9ad\uc2a4 2\uc138\uc5d0\uac8c \ucda9\uc131\ud558\uc600\ub358 \ub2e4\ub9c8\uc18c\ub97c \ucd94\uc885\ud558\ub294 \uc138\ub825\uc774\uc5c8\ub2e4.", "paragraph_id": "6656710-2-2", "paragraph_question": "question: \uc720\ud608\uc0ac\ud0dc\uac00 \ubc1c\uc0dd\ud558\uc790 \ub85c\ub9c8 \uc2dc \uc7a5\uad00\ub4e4\uc774 \uace8 \uc9c0\uc5ed\uc73c\ub85c \ucd94\ubc29\ud55c \uc778\ubb3c\uc740 \ub204\uad6c\uc778\uac00?, context: 366\ub144 9\uc6d4 24\uc77c \ub9ac\ubca0\ub9ac\uc624\uac00 \uc120\uc885\ud558\uc790 \ub85c\ub9c8 \uad50\ud68c\ub294 \ub450 \ud30c\ubc8c\ub85c \ubd84\uc5f4\ub418\uc5c8\ub2e4. \ud558\ub098\ub294 \ub9ac\ubca0\ub9ac\uc624\uc758 \ubd80\uc81c\ub85c \ubd09\uc0ac\ud558\uc600\ub358 \uc6b0\ub974\uc2dc\ub178\ub97c \ucd94\uc885\ud558\ub294 \uc138\ub825\uc774\uc5c8\uc73c\uba70, \ub098\uba38\uc9c0 \ud558\ub098\ub294 \ud55c\ub54c \ub300\ub9bd \uad50\ud669 \ud3a0\ub9ad\uc2a4 2\uc138\uc5d0\uac8c \ucda9\uc131\ud558\uc600\ub358 \ub2e4\ub9c8\uc18c\ub97c \ucd94\uc885\ud558\ub294 \uc138\ub825\uc774\uc5c8\ub2e4. \ub300\ub9bd \uad50\ud669 \ud3a0\ub9ad\uc2a4 2\uc138\ub97c \ub530\ub790\ub358 \uadc0\uc871\uce35\uc740 \ub2e4\ub9c8\uc18c\ub97c \uc9c0\uc9c0\ud55c \ubc18\uba74, \ub9ac\ubca0\ub9ac\uc624\ub97c \ub530\ub790\ub358 \ubd80\uc81c\ub4e4\uacfc \ud3c9\uc2e0\ub3c4\ub4e4\uc740 \uc6b0\ub974\uc2dc\ub178\ub97c \uc9c0\uc9c0\ud558\uc600\ub2e4. \ub2f9\uc7a5 \ud3ed\ub3d9\uc774\ub77c\ub3c4 \uc77c\uc5b4\ub0a0 \uac83\ub9cc \uac19\uc740 \uc0bc\uc5c4\ud55c \ubd84\uc704\uae30 \uc18d\uc5d0\uc11c \ub2e4\ub9c8\uc18c\uc640 \uc6b0\ub974\uc2dc\ub178 \ub450 \uc0ac\ub78c\uc740 \uac01\uac01 \uc591 \uc9c4\uc601\uc5d0\uc11c \ub85c\ub9c8\uc758 \uc8fc\uad50\ub85c \uc120\ucd9c\ub418\uc5c8\ub2e4. \ub2e4\ub9c8\uc18c\ub294 \uc0b0 \ub85c\ub80c\ucd08 \uc778 \ub8e8\uce58\ub098 \uc131\ub2f9\uc5d0\uc11c \ub85c\ub9c8 \uc8fc\uad50\ub85c \uc120\ucd9c\ub418\uc5c8\ub2e4. \uadf8\ub9ac\ud558\uc5ec \uacb0\uad6d \ub450 \uc0ac\ub78c\uc758 \uc9c0\uc9c0\uc790\ub4e4\uc740 \uadf8\ud574 10\uc6d4\uc5d0 \ucda9\ub3cc\ud558\uac8c \ub418\uc5c8\ub2e4. \uc591 \uc9c4\uc601\uc740 \ub85c\ub9c8 \uad50\uc678\uc5d0\uc11c \ucda9\ub3cc\ud558\uc5ec \uacb0\uad6d \uc2dc\uc2e0\ub2c8\uc624 \uc131\ub2f9(\ud6d7\ub0a0\uc758 \uc0b0\ud0c0 \ub9c8\ub9ac\uc544 \ub9c8\uc870\ub808 \ub300\uc131\uc804)\uc5d0\uc11c 137\uba85\uc774 \ub300\ub7c9 \ud559\uc0b4\ub418\ub294 \uc0ac\uac74\uc774 \uc77c\uc5b4\ub098\uae30\uc5d0 \uc774\ub974\ub800\ub2e4. \uc774\ub7ec\ud55c \ud3ed\ub825\uc801\uc778 \uc720\ud608 \uc0ac\ud0dc\uac00 \ubc1c\ubc1c\ud558\uc790 \ub85c\ub9c8 \uc2dc \uc7a5\uad00\ub4e4\uc774 \uac1c\uc785\ud558\uc5ec \uac00\uae4c\uc2a4\ub85c \uc9c4\uc555\ud558\ub294\ub370 \uc131\uacf5\ud558\uc600\ub2e4. \uadf8\ub9ac\uace0 \uc7a5\uad00\ub4e4\uc740 \uc6b0\ub974\uc2dc\ub178\ub97c \uace8 \uc9c0\uc5ed\uc73c\ub85c \ucd94\ubc29\ud558\uc600\ub2e4. \uadf8\ub7ec\ub098 \uc6b0\ub974\uc2dc\ub178\uac00 \ucd94\ubc29\ub41c \ud6c4\uc5d0\ub3c4 \uc591\uce21 \uc9c0\uc9c0\uc790\ub4e4 \uac04\uc758 \ucda9\ub3cc\uc740 \ub04a\uc774\uc9c0 \uc54a\uace0 \uc624\ud788\ub824 \ub354 \uc2ec\ud654\ub418\uc5c8\ub2e4."} {"question": "\uc138 \uac1c\uc758 \uc5f0\uc0b0 \uc774\ubca4\ud2b8 \uc8fc\uc5d0 \uccab \ubc88\uc9f8 \uc694\uc18c\ub294?", "paragraph": "\ubbf8\uc2f1\ub178\ub294 \uc138 \uac1c\uc758 \uc5f0\uc0b0 \uc774\ubca4\ud2b8\uc758 \uacb0\uacfc\ub85c\uc11c \ub098\ud0c0\ub098\ub294 \ud604\uc0c1\uc774\ub2e4. \uccab \ubc88\uc9f8 \uc694\uc18c\ub294 \uac8c\uc784\uc758 \ub79c\ub364 \ub300\uc804 \uc2dc\uc2a4\ud15c\uc774\ub2e4. \ud50c\ub808\uc774\uc5b4\uac00 \uc57c\uc0dd \ud3ec\ucf13\ubaac\uacfc \uc870\uc6b0\ud560 \ub54c \uc811\uadfc\ud558\ub294 \ub370\uc774\ud130 \ubc84\ud37c \ub0b4\uc5d0\uc11c \uac01 \uc9c0\uc5ed\ubcc4 \ud3ec\ucf13\ubaac\uc5d0 \ud2b9\uc815 \uac12\uc744 \uc9c0\uc815\ud55c\ub2e4. \uadf8\ub7ec\ub098 \ud64d\ub828\uc12c \ub3d9\ucabd \ud574\uc548\uc5d0\uc11c\ub294 \uc774 \ubc84\ud37c\uc5d0 \uc9c0\uc815\ub41c \uac12\uc774 \uc5c6\uae30 \ub54c\ubb38\uc5d0, \uc774\uc804\uc5d0 \ubc29\ubb38\ud588\ub358 \uc9c0\uc5ed\uc758 \uc815\ubcf4\ub97c \ub300\uc2e0 \uc0ac\uc6a9\ud558\uac8c \ub41c\ub2e4. \ub450 \ubc88\uc9f8 \uc694\uc18c\ub294 \ud29c\ud1a0\ub9ac\uc5bc\uc774\ub2e4. \ud29c\ud1a0\ub9ac\uc5bc\uc5d0\uc11c\ub294 \ud50c\ub808\uc774\uc5b4\uc758 \uc774\ub984\uc774 \ub370\uc774\ud130 \ubc84\ud37c\uc5d0 \uc784\uc2dc\ub85c \uc800\uc7a5\ub41c\ub2e4. \uc774 \ub54c\ubb38\uc5d0 \ud64d\ub828\uc12c\uc5d0\uc11c \ud3ec\ucf13\ubaac\uc744 \uc870\uc6b0\ud560 \ub54c 16\uc9c4\uc218 \uac12\uc778 \ud50c\ub808\uc774\uc5b4 \uc774\ub984\uc5d0 \uc561\uc138\uc2a4\ud558\uac8c \ub41c\ub2e4. \uc138 \ubc88\uc9f8 \uc694\uc18c\ub294 \uac8c\uc784\uc758 \uc5d0\ub7ec \ucc98\ub9ac \uc2dc\uc2a4\ud15c\uc5d0 \uc788\ub2e4. \uc2e4\uc874\ud558\ub294 \ud3ec\ucf13\ubaac\uc774 \uc544\ub2cc \ub370\uc774\ud130 \ubc84\ud37c \uac12\uc774 \uc120\ud0dd\ub418\ub294 \uacbd\uc6b0, \ub204\ub77d\ub41c \ubc88\ud638\ub97c \uc758\ubbf8\ud558\ub294 \u201c\ubbf8\uc2f1\ub178\u201d\ub97c \uc774\ub984\uc73c\ub85c \ud55c \ud3ec\ucf13\ubaac\uc774 \ub098\ud0c0\ub098\uac8c \ub418\ub294 \uc11c\ube0c\ub8e8\ud2f4\uc774 \uad6c\ub3d9\ub41c\ub2e4.", "answer": "\uac8c\uc784\uc758 \ub79c\ub364 \ub300\uc804 \uc2dc\uc2a4\ud15c\uc774\ub2e4.", "sentence": "\uccab \ubc88\uc9f8 \uc694\uc18c\ub294 \uac8c\uc784\uc758 \ub79c\ub364 \ub300\uc804 \uc2dc\uc2a4\ud15c\uc774\ub2e4. \ud50c\ub808\uc774\uc5b4\uac00 \uc57c\uc0dd \ud3ec\ucf13\ubaac\uacfc \uc870\uc6b0\ud560 \ub54c \uc811\uadfc\ud558\ub294 \ub370\uc774\ud130 \ubc84\ud37c \ub0b4\uc5d0\uc11c \uac01 \uc9c0\uc5ed\ubcc4 \ud3ec\ucf13\ubaac\uc5d0 \ud2b9\uc815 \uac12\uc744 \uc9c0\uc815\ud55c\ub2e4.", "paragraph_sentence": "\ubbf8\uc2f1\ub178\ub294 \uc138 \uac1c\uc758 \uc5f0\uc0b0 \uc774\ubca4\ud2b8\uc758 \uacb0\uacfc\ub85c\uc11c \ub098\ud0c0\ub098\ub294 \ud604\uc0c1\uc774\ub2e4. \uccab \ubc88\uc9f8 \uc694\uc18c\ub294 \uac8c\uc784\uc758 \ub79c\ub364 \ub300\uc804 \uc2dc\uc2a4\ud15c\uc774\ub2e4. \ud50c\ub808\uc774\uc5b4\uac00 \uc57c\uc0dd \ud3ec\ucf13\ubaac\uacfc \uc870\uc6b0\ud560 \ub54c \uc811\uadfc\ud558\ub294 \ub370\uc774\ud130 \ubc84\ud37c \ub0b4\uc5d0\uc11c \uac01 \uc9c0\uc5ed\ubcc4 \ud3ec\ucf13\ubaac\uc5d0 \ud2b9\uc815 \uac12\uc744 \uc9c0\uc815\ud55c\ub2e4. \uadf8\ub7ec\ub098 \ud64d\ub828\uc12c \ub3d9\ucabd \ud574\uc548\uc5d0\uc11c\ub294 \uc774 \ubc84\ud37c\uc5d0 \uc9c0\uc815\ub41c \uac12\uc774 \uc5c6\uae30 \ub54c\ubb38\uc5d0, \uc774\uc804\uc5d0 \ubc29\ubb38\ud588\ub358 \uc9c0\uc5ed\uc758 \uc815\ubcf4\ub97c \ub300\uc2e0 \uc0ac\uc6a9\ud558\uac8c \ub41c\ub2e4. \ub450 \ubc88\uc9f8 \uc694\uc18c\ub294 \ud29c\ud1a0\ub9ac\uc5bc\uc774\ub2e4. \ud29c\ud1a0\ub9ac\uc5bc\uc5d0\uc11c\ub294 \ud50c\ub808\uc774\uc5b4\uc758 \uc774\ub984\uc774 \ub370\uc774\ud130 \ubc84\ud37c\uc5d0 \uc784\uc2dc\ub85c \uc800\uc7a5\ub41c\ub2e4. \uc774 \ub54c\ubb38\uc5d0 \ud64d\ub828\uc12c\uc5d0\uc11c \ud3ec\ucf13\ubaac\uc744 \uc870\uc6b0\ud560 \ub54c 16\uc9c4\uc218 \uac12\uc778 \ud50c\ub808\uc774\uc5b4 \uc774\ub984\uc5d0 \uc561\uc138\uc2a4\ud558\uac8c \ub41c\ub2e4. \uc138 \ubc88\uc9f8 \uc694\uc18c\ub294 \uac8c\uc784\uc758 \uc5d0\ub7ec \ucc98\ub9ac \uc2dc\uc2a4\ud15c\uc5d0 \uc788\ub2e4. \uc2e4\uc874\ud558\ub294 \ud3ec\ucf13\ubaac\uc774 \uc544\ub2cc \ub370\uc774\ud130 \ubc84\ud37c \uac12\uc774 \uc120\ud0dd\ub418\ub294 \uacbd\uc6b0, \ub204\ub77d\ub41c \ubc88\ud638\ub97c \uc758\ubbf8\ud558\ub294 \u201c\ubbf8\uc2f1\ub178\u201d\ub97c \uc774\ub984\uc73c\ub85c \ud55c \ud3ec\ucf13\ubaac\uc774 \ub098\ud0c0\ub098\uac8c \ub418\ub294 \uc11c\ube0c\ub8e8\ud2f4\uc774 \uad6c\ub3d9\ub41c\ub2e4.", "paragraph_answer": "\ubbf8\uc2f1\ub178\ub294 \uc138 \uac1c\uc758 \uc5f0\uc0b0 \uc774\ubca4\ud2b8\uc758 \uacb0\uacfc\ub85c\uc11c \ub098\ud0c0\ub098\ub294 \ud604\uc0c1\uc774\ub2e4. \uccab \ubc88\uc9f8 \uc694\uc18c\ub294 \uac8c\uc784\uc758 \ub79c\ub364 \ub300\uc804 \uc2dc\uc2a4\ud15c\uc774\ub2e4. \ud50c\ub808\uc774\uc5b4\uac00 \uc57c\uc0dd \ud3ec\ucf13\ubaac\uacfc \uc870\uc6b0\ud560 \ub54c \uc811\uadfc\ud558\ub294 \ub370\uc774\ud130 \ubc84\ud37c \ub0b4\uc5d0\uc11c \uac01 \uc9c0\uc5ed\ubcc4 \ud3ec\ucf13\ubaac\uc5d0 \ud2b9\uc815 \uac12\uc744 \uc9c0\uc815\ud55c\ub2e4. \uadf8\ub7ec\ub098 \ud64d\ub828\uc12c \ub3d9\ucabd \ud574\uc548\uc5d0\uc11c\ub294 \uc774 \ubc84\ud37c\uc5d0 \uc9c0\uc815\ub41c \uac12\uc774 \uc5c6\uae30 \ub54c\ubb38\uc5d0, \uc774\uc804\uc5d0 \ubc29\ubb38\ud588\ub358 \uc9c0\uc5ed\uc758 \uc815\ubcf4\ub97c \ub300\uc2e0 \uc0ac\uc6a9\ud558\uac8c \ub41c\ub2e4. \ub450 \ubc88\uc9f8 \uc694\uc18c\ub294 \ud29c\ud1a0\ub9ac\uc5bc\uc774\ub2e4. \ud29c\ud1a0\ub9ac\uc5bc\uc5d0\uc11c\ub294 \ud50c\ub808\uc774\uc5b4\uc758 \uc774\ub984\uc774 \ub370\uc774\ud130 \ubc84\ud37c\uc5d0 \uc784\uc2dc\ub85c \uc800\uc7a5\ub41c\ub2e4. \uc774 \ub54c\ubb38\uc5d0 \ud64d\ub828\uc12c\uc5d0\uc11c \ud3ec\ucf13\ubaac\uc744 \uc870\uc6b0\ud560 \ub54c 16\uc9c4\uc218 \uac12\uc778 \ud50c\ub808\uc774\uc5b4 \uc774\ub984\uc5d0 \uc561\uc138\uc2a4\ud558\uac8c \ub41c\ub2e4. \uc138 \ubc88\uc9f8 \uc694\uc18c\ub294 \uac8c\uc784\uc758 \uc5d0\ub7ec \ucc98\ub9ac \uc2dc\uc2a4\ud15c\uc5d0 \uc788\ub2e4. \uc2e4\uc874\ud558\ub294 \ud3ec\ucf13\ubaac\uc774 \uc544\ub2cc \ub370\uc774\ud130 \ubc84\ud37c \uac12\uc774 \uc120\ud0dd\ub418\ub294 \uacbd\uc6b0, \ub204\ub77d\ub41c \ubc88\ud638\ub97c \uc758\ubbf8\ud558\ub294 \u201c\ubbf8\uc2f1\ub178\u201d\ub97c \uc774\ub984\uc73c\ub85c \ud55c \ud3ec\ucf13\ubaac\uc774 \ub098\ud0c0\ub098\uac8c \ub418\ub294 \uc11c\ube0c\ub8e8\ud2f4\uc774 \uad6c\ub3d9\ub41c\ub2e4.", "sentence_answer": "\uccab \ubc88\uc9f8 \uc694\uc18c\ub294 \uac8c\uc784\uc758 \ub79c\ub364 \ub300\uc804 \uc2dc\uc2a4\ud15c\uc774\ub2e4. \ud50c\ub808\uc774\uc5b4\uac00 \uc57c\uc0dd \ud3ec\ucf13\ubaac\uacfc \uc870\uc6b0\ud560 \ub54c \uc811\uadfc\ud558\ub294 \ub370\uc774\ud130 \ubc84\ud37c \ub0b4\uc5d0\uc11c \uac01 \uc9c0\uc5ed\ubcc4 \ud3ec\ucf13\ubaac\uc5d0 \ud2b9\uc815 \uac12\uc744 \uc9c0\uc815\ud55c\ub2e4.", "paragraph_id": "6094346-1-2", "paragraph_question": "question: \uc138 \uac1c\uc758 \uc5f0\uc0b0 \uc774\ubca4\ud2b8 \uc8fc\uc5d0 \uccab \ubc88\uc9f8 \uc694\uc18c\ub294?, context: \ubbf8\uc2f1\ub178\ub294 \uc138 \uac1c\uc758 \uc5f0\uc0b0 \uc774\ubca4\ud2b8\uc758 \uacb0\uacfc\ub85c\uc11c \ub098\ud0c0\ub098\ub294 \ud604\uc0c1\uc774\ub2e4. \uccab \ubc88\uc9f8 \uc694\uc18c\ub294 \uac8c\uc784\uc758 \ub79c\ub364 \ub300\uc804 \uc2dc\uc2a4\ud15c\uc774\ub2e4. \ud50c\ub808\uc774\uc5b4\uac00 \uc57c\uc0dd \ud3ec\ucf13\ubaac\uacfc \uc870\uc6b0\ud560 \ub54c \uc811\uadfc\ud558\ub294 \ub370\uc774\ud130 \ubc84\ud37c \ub0b4\uc5d0\uc11c \uac01 \uc9c0\uc5ed\ubcc4 \ud3ec\ucf13\ubaac\uc5d0 \ud2b9\uc815 \uac12\uc744 \uc9c0\uc815\ud55c\ub2e4. \uadf8\ub7ec\ub098 \ud64d\ub828\uc12c \ub3d9\ucabd \ud574\uc548\uc5d0\uc11c\ub294 \uc774 \ubc84\ud37c\uc5d0 \uc9c0\uc815\ub41c \uac12\uc774 \uc5c6\uae30 \ub54c\ubb38\uc5d0, \uc774\uc804\uc5d0 \ubc29\ubb38\ud588\ub358 \uc9c0\uc5ed\uc758 \uc815\ubcf4\ub97c \ub300\uc2e0 \uc0ac\uc6a9\ud558\uac8c \ub41c\ub2e4. \ub450 \ubc88\uc9f8 \uc694\uc18c\ub294 \ud29c\ud1a0\ub9ac\uc5bc\uc774\ub2e4. \ud29c\ud1a0\ub9ac\uc5bc\uc5d0\uc11c\ub294 \ud50c\ub808\uc774\uc5b4\uc758 \uc774\ub984\uc774 \ub370\uc774\ud130 \ubc84\ud37c\uc5d0 \uc784\uc2dc\ub85c \uc800\uc7a5\ub41c\ub2e4. \uc774 \ub54c\ubb38\uc5d0 \ud64d\ub828\uc12c\uc5d0\uc11c \ud3ec\ucf13\ubaac\uc744 \uc870\uc6b0\ud560 \ub54c 16\uc9c4\uc218 \uac12\uc778 \ud50c\ub808\uc774\uc5b4 \uc774\ub984\uc5d0 \uc561\uc138\uc2a4\ud558\uac8c \ub41c\ub2e4. \uc138 \ubc88\uc9f8 \uc694\uc18c\ub294 \uac8c\uc784\uc758 \uc5d0\ub7ec \ucc98\ub9ac \uc2dc\uc2a4\ud15c\uc5d0 \uc788\ub2e4. \uc2e4\uc874\ud558\ub294 \ud3ec\ucf13\ubaac\uc774 \uc544\ub2cc \ub370\uc774\ud130 \ubc84\ud37c \uac12\uc774 \uc120\ud0dd\ub418\ub294 \uacbd\uc6b0, \ub204\ub77d\ub41c \ubc88\ud638\ub97c \uc758\ubbf8\ud558\ub294 \u201c\ubbf8\uc2f1\ub178\u201d\ub97c \uc774\ub984\uc73c\ub85c \ud55c \ud3ec\ucf13\ubaac\uc774 \ub098\ud0c0\ub098\uac8c \ub418\ub294 \uc11c\ube0c\ub8e8\ud2f4\uc774 \uad6c\ub3d9\ub41c\ub2e4."} {"question": "\ub124\ubc84\uc708\ud130 \ub098\uc774\uce20\uc5d0\uc11c \uc11c\ubc84\uc758 \uc644\uc804\ud55c \uc81c\uc5b4\uad8c\uc744 \uc81c\uacf5\ud558\ub294 \uac83\uc740?", "paragraph": "\u300a\ub124\ubc84\uc708\ud130 \ub098\uc774\uce20\u300b\uc758 \uac00\uc7a5 \uc911\uc694\ud55c \ud2b9\uc9d5 \uc911 \ud558\ub098\ub294 DM \ub610\ub294 \ub358\uc804 \ub9c8\uc2a4\ud130 \ud074\ub77c\uc774\uc5b8\ud2b8\uc774\ub2e4. \uc774 \ub3c4\uad6c\ub294 \uac1c\uc778\uc774 \uc804\ud1b5\uc801\uc778 \ub358\uc804 \ub9c8\uc2a4\ud130\uc758 \uc5ed\ud560\uc744 \ub9e1\uc544 \ud50c\ub808\uc774\uc5b4\ub4e4\uc744 \uc774\uc57c\uae30 \uc18d\uc73c\ub85c \uc774\ub04c\uc5b4 \uac08 \uc218 \uc788\ub3c4\ub85d \ud574\uc8fc\uba70, \uc11c\ubc84\uc758 \uc644\uc804\ud55c \uc81c\uc5b4\uad8c\uc744 \uc81c\uacf5\ud55c\ub2e4. \ube44\ub85d \u300a\ub124\ubc84\uc708\ud130 \ub098\uc774\uce20\u300b\uac00 \uc774 \uae30\ub2a5\uc744 \uc0ac\uc6a9\ud55c \uccab \ubc88\uc9f8 \uac8c\uc784\uc740 \uc544\ub2c8\uc9c0\ub9cc(\ud654\uc774\ud2b8 \uc6b8\ud504\uc5d0\uc11c \ucd9c\ud310\ud55c \uac8c\uc784\ubd81\uc5d0 \uae30\ubc18\ud55c \uac8c\uc784 \u300a\ubc40\ud30c\uc774\uc5b4: \ub354 \uba38\uc2a4\ucee4\ub808\uc774\ub4dc\u300b\uac00 \ubcf4\ub2e4 \uae30\ucd08\uc801\uc778 \ubc84\uc804\uc758 \uc0ac\ub840 \uc911 \ud558\ub098\uc774\ub2e4), \u300a\ub124\ubc84\uc708\ud130 \ub098\uc774\uce20\u300b\ub294 \uc774 \uae30\ub2a5\uc758 \uac00\uc7a5 \ubc1c\uc804\ub41c \ud615\ud0dc\ub97c \uac00\uc9c0\uace0 \uc788\uc73c\uba70, \uadf8\ub807\uae30 \ub54c\ubb38\uc5d0 \ud604\uc7ac \ubc1c\ub9e4\ub41c \ucef4\ud4e8\ud130 \ub864\ud50c\ub808\uc789 \uac8c\uc784 \uc911\uc5d0\uc11c \uac00\uc7a5 \ubab0\uc785\ub3c4\uac00 \ub192\uc744 \ub864\ud50c\ub808\uc789 \uacbd\ud5d8\uc744 \uc81c\uacf5\ud574 \uc900\ub2e4. DM \ud074\ub77c\uc774\uc5b8\ud2b8\ub294 \ud50c\ub808\uc774\uc5b4\ub4e4\uc774 \ud68d\uc77c\uc801\uc778 \ucea0\ud328\uc778\uc758 \uc9c4\ud589\uc5d0 \uad00\uc5ec\ud560 \uc218 \uc788\ub3c4\ub85d \ud588\uc73c\uba70, \ud37c\uc2dc\uc2a4\ud134\ud2b8 \uc6d4\ub4dc \uc11c\ubc84\ub294 \uc0ac\uc2e4\uc131\uc744 \ub192\uc774\uae30 \uc704\ud574 \uc11c\ubc84\uc758 \ub358\uc804 \ub9c8\uc2a4\ud130\uac00 NPC\ub97c \uc2e4\uc2dc\uac04\uc73c\ub85c \uc9c0\ubc30\ud560 \uc218 \uc788\ub3c4\ub85d \ud568\uc73c\ub85c\uc368 \ub354 \ub2e4\uc591\ud55c \ud50c\ub808\uc774 \uacbd\ud5d8\uc744 \uc81c\uacf5\ud55c\ub2e4.", "answer": "\ub358\uc804 \ub9c8\uc2a4\ud130 \ud074\ub77c\uc774\uc5b8\ud2b8", "sentence": "\u300a\ub124\ubc84\uc708\ud130 \ub098\uc774\uce20\u300b\uc758 \uac00\uc7a5 \uc911\uc694\ud55c \ud2b9\uc9d5 \uc911 \ud558\ub098\ub294 DM \ub610\ub294 \ub358\uc804 \ub9c8\uc2a4\ud130 \ud074\ub77c\uc774\uc5b8\ud2b8 \uc774\ub2e4.", "paragraph_sentence": " \u300a\ub124\ubc84\uc708\ud130 \ub098\uc774\uce20\u300b\uc758 \uac00\uc7a5 \uc911\uc694\ud55c \ud2b9\uc9d5 \uc911 \ud558\ub098\ub294 DM \ub610\ub294 \ub358\uc804 \ub9c8\uc2a4\ud130 \ud074\ub77c\uc774\uc5b8\ud2b8 \uc774\ub2e4. \uc774 \ub3c4\uad6c\ub294 \uac1c\uc778\uc774 \uc804\ud1b5\uc801\uc778 \ub358\uc804 \ub9c8\uc2a4\ud130\uc758 \uc5ed\ud560\uc744 \ub9e1\uc544 \ud50c\ub808\uc774\uc5b4\ub4e4\uc744 \uc774\uc57c\uae30 \uc18d\uc73c\ub85c \uc774\ub04c\uc5b4 \uac08 \uc218 \uc788\ub3c4\ub85d \ud574\uc8fc\uba70, \uc11c\ubc84\uc758 \uc644\uc804\ud55c \uc81c\uc5b4\uad8c\uc744 \uc81c\uacf5\ud55c\ub2e4. \ube44\ub85d \u300a\ub124\ubc84\uc708\ud130 \ub098\uc774\uce20\u300b\uac00 \uc774 \uae30\ub2a5\uc744 \uc0ac\uc6a9\ud55c \uccab \ubc88\uc9f8 \uac8c\uc784\uc740 \uc544\ub2c8\uc9c0\ub9cc(\ud654\uc774\ud2b8 \uc6b8\ud504\uc5d0\uc11c \ucd9c\ud310\ud55c \uac8c\uc784\ubd81\uc5d0 \uae30\ubc18\ud55c \uac8c\uc784 \u300a\ubc40\ud30c\uc774\uc5b4: \ub354 \uba38\uc2a4\ucee4\ub808\uc774\ub4dc\u300b\uac00 \ubcf4\ub2e4 \uae30\ucd08\uc801\uc778 \ubc84\uc804\uc758 \uc0ac\ub840 \uc911 \ud558\ub098\uc774\ub2e4), \u300a\ub124\ubc84\uc708\ud130 \ub098\uc774\uce20\u300b\ub294 \uc774 \uae30\ub2a5\uc758 \uac00\uc7a5 \ubc1c\uc804\ub41c \ud615\ud0dc\ub97c \uac00\uc9c0\uace0 \uc788\uc73c\uba70, \uadf8\ub807\uae30 \ub54c\ubb38\uc5d0 \ud604\uc7ac \ubc1c\ub9e4\ub41c \ucef4\ud4e8\ud130 \ub864\ud50c\ub808\uc789 \uac8c\uc784 \uc911\uc5d0\uc11c \uac00\uc7a5 \ubab0\uc785\ub3c4\uac00 \ub192\uc744 \ub864\ud50c\ub808\uc789 \uacbd\ud5d8\uc744 \uc81c\uacf5\ud574 \uc900\ub2e4. DM \ud074\ub77c\uc774\uc5b8\ud2b8\ub294 \ud50c\ub808\uc774\uc5b4\ub4e4\uc774 \ud68d\uc77c\uc801\uc778 \ucea0\ud328\uc778\uc758 \uc9c4\ud589\uc5d0 \uad00\uc5ec\ud560 \uc218 \uc788\ub3c4\ub85d \ud588\uc73c\uba70, \ud37c\uc2dc\uc2a4\ud134\ud2b8 \uc6d4\ub4dc \uc11c\ubc84\ub294 \uc0ac\uc2e4\uc131\uc744 \ub192\uc774\uae30 \uc704\ud574 \uc11c\ubc84\uc758 \ub358\uc804 \ub9c8\uc2a4\ud130\uac00 NPC\ub97c \uc2e4\uc2dc\uac04\uc73c\ub85c \uc9c0\ubc30\ud560 \uc218 \uc788\ub3c4\ub85d \ud568\uc73c\ub85c\uc368 \ub354 \ub2e4\uc591\ud55c \ud50c\ub808\uc774 \uacbd\ud5d8\uc744 \uc81c\uacf5\ud55c\ub2e4.", "paragraph_answer": "\u300a\ub124\ubc84\uc708\ud130 \ub098\uc774\uce20\u300b\uc758 \uac00\uc7a5 \uc911\uc694\ud55c \ud2b9\uc9d5 \uc911 \ud558\ub098\ub294 DM \ub610\ub294 \ub358\uc804 \ub9c8\uc2a4\ud130 \ud074\ub77c\uc774\uc5b8\ud2b8 \uc774\ub2e4. \uc774 \ub3c4\uad6c\ub294 \uac1c\uc778\uc774 \uc804\ud1b5\uc801\uc778 \ub358\uc804 \ub9c8\uc2a4\ud130\uc758 \uc5ed\ud560\uc744 \ub9e1\uc544 \ud50c\ub808\uc774\uc5b4\ub4e4\uc744 \uc774\uc57c\uae30 \uc18d\uc73c\ub85c \uc774\ub04c\uc5b4 \uac08 \uc218 \uc788\ub3c4\ub85d \ud574\uc8fc\uba70, \uc11c\ubc84\uc758 \uc644\uc804\ud55c \uc81c\uc5b4\uad8c\uc744 \uc81c\uacf5\ud55c\ub2e4. \ube44\ub85d \u300a\ub124\ubc84\uc708\ud130 \ub098\uc774\uce20\u300b\uac00 \uc774 \uae30\ub2a5\uc744 \uc0ac\uc6a9\ud55c \uccab \ubc88\uc9f8 \uac8c\uc784\uc740 \uc544\ub2c8\uc9c0\ub9cc(\ud654\uc774\ud2b8 \uc6b8\ud504\uc5d0\uc11c \ucd9c\ud310\ud55c \uac8c\uc784\ubd81\uc5d0 \uae30\ubc18\ud55c \uac8c\uc784 \u300a\ubc40\ud30c\uc774\uc5b4: \ub354 \uba38\uc2a4\ucee4\ub808\uc774\ub4dc\u300b\uac00 \ubcf4\ub2e4 \uae30\ucd08\uc801\uc778 \ubc84\uc804\uc758 \uc0ac\ub840 \uc911 \ud558\ub098\uc774\ub2e4), \u300a\ub124\ubc84\uc708\ud130 \ub098\uc774\uce20\u300b\ub294 \uc774 \uae30\ub2a5\uc758 \uac00\uc7a5 \ubc1c\uc804\ub41c \ud615\ud0dc\ub97c \uac00\uc9c0\uace0 \uc788\uc73c\uba70, \uadf8\ub807\uae30 \ub54c\ubb38\uc5d0 \ud604\uc7ac \ubc1c\ub9e4\ub41c \ucef4\ud4e8\ud130 \ub864\ud50c\ub808\uc789 \uac8c\uc784 \uc911\uc5d0\uc11c \uac00\uc7a5 \ubab0\uc785\ub3c4\uac00 \ub192\uc744 \ub864\ud50c\ub808\uc789 \uacbd\ud5d8\uc744 \uc81c\uacf5\ud574 \uc900\ub2e4. DM \ud074\ub77c\uc774\uc5b8\ud2b8\ub294 \ud50c\ub808\uc774\uc5b4\ub4e4\uc774 \ud68d\uc77c\uc801\uc778 \ucea0\ud328\uc778\uc758 \uc9c4\ud589\uc5d0 \uad00\uc5ec\ud560 \uc218 \uc788\ub3c4\ub85d \ud588\uc73c\uba70, \ud37c\uc2dc\uc2a4\ud134\ud2b8 \uc6d4\ub4dc \uc11c\ubc84\ub294 \uc0ac\uc2e4\uc131\uc744 \ub192\uc774\uae30 \uc704\ud574 \uc11c\ubc84\uc758 \ub358\uc804 \ub9c8\uc2a4\ud130\uac00 NPC\ub97c \uc2e4\uc2dc\uac04\uc73c\ub85c \uc9c0\ubc30\ud560 \uc218 \uc788\ub3c4\ub85d \ud568\uc73c\ub85c\uc368 \ub354 \ub2e4\uc591\ud55c \ud50c\ub808\uc774 \uacbd\ud5d8\uc744 \uc81c\uacf5\ud55c\ub2e4.", "sentence_answer": "\u300a\ub124\ubc84\uc708\ud130 \ub098\uc774\uce20\u300b\uc758 \uac00\uc7a5 \uc911\uc694\ud55c \ud2b9\uc9d5 \uc911 \ud558\ub098\ub294 DM \ub610\ub294 \ub358\uc804 \ub9c8\uc2a4\ud130 \ud074\ub77c\uc774\uc5b8\ud2b8 \uc774\ub2e4.", "paragraph_id": "6483773-4-0", "paragraph_question": "question: \ub124\ubc84\uc708\ud130 \ub098\uc774\uce20\uc5d0\uc11c \uc11c\ubc84\uc758 \uc644\uc804\ud55c \uc81c\uc5b4\uad8c\uc744 \uc81c\uacf5\ud558\ub294 \uac83\uc740?, context: \u300a\ub124\ubc84\uc708\ud130 \ub098\uc774\uce20\u300b\uc758 \uac00\uc7a5 \uc911\uc694\ud55c \ud2b9\uc9d5 \uc911 \ud558\ub098\ub294 DM \ub610\ub294 \ub358\uc804 \ub9c8\uc2a4\ud130 \ud074\ub77c\uc774\uc5b8\ud2b8\uc774\ub2e4. \uc774 \ub3c4\uad6c\ub294 \uac1c\uc778\uc774 \uc804\ud1b5\uc801\uc778 \ub358\uc804 \ub9c8\uc2a4\ud130\uc758 \uc5ed\ud560\uc744 \ub9e1\uc544 \ud50c\ub808\uc774\uc5b4\ub4e4\uc744 \uc774\uc57c\uae30 \uc18d\uc73c\ub85c \uc774\ub04c\uc5b4 \uac08 \uc218 \uc788\ub3c4\ub85d \ud574\uc8fc\uba70, \uc11c\ubc84\uc758 \uc644\uc804\ud55c \uc81c\uc5b4\uad8c\uc744 \uc81c\uacf5\ud55c\ub2e4. \ube44\ub85d \u300a\ub124\ubc84\uc708\ud130 \ub098\uc774\uce20\u300b\uac00 \uc774 \uae30\ub2a5\uc744 \uc0ac\uc6a9\ud55c \uccab \ubc88\uc9f8 \uac8c\uc784\uc740 \uc544\ub2c8\uc9c0\ub9cc(\ud654\uc774\ud2b8 \uc6b8\ud504\uc5d0\uc11c \ucd9c\ud310\ud55c \uac8c\uc784\ubd81\uc5d0 \uae30\ubc18\ud55c \uac8c\uc784 \u300a\ubc40\ud30c\uc774\uc5b4: \ub354 \uba38\uc2a4\ucee4\ub808\uc774\ub4dc\u300b\uac00 \ubcf4\ub2e4 \uae30\ucd08\uc801\uc778 \ubc84\uc804\uc758 \uc0ac\ub840 \uc911 \ud558\ub098\uc774\ub2e4), \u300a\ub124\ubc84\uc708\ud130 \ub098\uc774\uce20\u300b\ub294 \uc774 \uae30\ub2a5\uc758 \uac00\uc7a5 \ubc1c\uc804\ub41c \ud615\ud0dc\ub97c \uac00\uc9c0\uace0 \uc788\uc73c\uba70, \uadf8\ub807\uae30 \ub54c\ubb38\uc5d0 \ud604\uc7ac \ubc1c\ub9e4\ub41c \ucef4\ud4e8\ud130 \ub864\ud50c\ub808\uc789 \uac8c\uc784 \uc911\uc5d0\uc11c \uac00\uc7a5 \ubab0\uc785\ub3c4\uac00 \ub192\uc744 \ub864\ud50c\ub808\uc789 \uacbd\ud5d8\uc744 \uc81c\uacf5\ud574 \uc900\ub2e4. DM \ud074\ub77c\uc774\uc5b8\ud2b8\ub294 \ud50c\ub808\uc774\uc5b4\ub4e4\uc774 \ud68d\uc77c\uc801\uc778 \ucea0\ud328\uc778\uc758 \uc9c4\ud589\uc5d0 \uad00\uc5ec\ud560 \uc218 \uc788\ub3c4\ub85d \ud588\uc73c\uba70, \ud37c\uc2dc\uc2a4\ud134\ud2b8 \uc6d4\ub4dc \uc11c\ubc84\ub294 \uc0ac\uc2e4\uc131\uc744 \ub192\uc774\uae30 \uc704\ud574 \uc11c\ubc84\uc758 \ub358\uc804 \ub9c8\uc2a4\ud130\uac00 NPC\ub97c \uc2e4\uc2dc\uac04\uc73c\ub85c \uc9c0\ubc30\ud560 \uc218 \uc788\ub3c4\ub85d \ud568\uc73c\ub85c\uc368 \ub354 \ub2e4\uc591\ud55c \ud50c\ub808\uc774 \uacbd\ud5d8\uc744 \uc81c\uacf5\ud55c\ub2e4."} {"question": "4\ub300\uac15 \uacf5\uc0ac\uc7a5\uc744 \ub458\ub7ec\ubcf8 \uacb0\uacfc \uc6b4\ud558\uac74\uc124 \uc0ac\uc5c5\uc774\ub77c\ub294 \uc778\uc0c1\uc744 \uc9c0\uc6b8 \uc218 \uc5c6\uc5c8\ub2e4\uba70 \uc911\ub2e8\ud574\uc57c \ud55c\ub2e4\uace0 \ucd09\uad6c\ud55c \uc0ac\ub78c\uc740?", "paragraph": "2010\ub144 7\uc6d4 11\uc77c YWCA \uac15\ub2f9\uc5d0\uc11c \uc5f4\ub9b0 4\ub300\uac15 \uc0ac\uc5c5 \ud55c\u00b7\uc77c \uc870\uc0ac\ub2e8 \ud1a0\ub860\ud68c\uc7a5\uc5d0\uc11c '\ud638\ub9ac \ub8cc\uc774\uce58' \ub78c\uc0ac\ub974 \ub124\ud2b8\uc6cc\ud06c \uc77c\ubcf8\ub300\ud45c\ub294 \"\ub78c\uc0ac\ub974 \ucd1d\ud68c\ub97c \uac1c\ucd5c\ud55c \ub098\ub77c\uac00 \ub9de\ub294\uac00. \ucda9\uaca9\uc801\uc774\ub2e4. 4\ub300\uac15 \uc0ac\uc5c5\uc740 \ubaa9\uc801\uc774 \ubd88\ubd84\uba85\ud558\uace0 \ub78c\uc0ac\ub974\ud611\uc57d\uacfc \uc0dd\ubb3c\ub2e4\uc591\uc131\ud611\uc57d\uc5d0 \uc704\ubc30\ub418\ub294 \uba85\ubc31\ud55c \ud658\uacbd\ud30c\uad34 \uc0ac\uc5c5\uc774\ub2e4\"\ub77c\uba70 \uc0ac\uc5c5\uc744 \ub9c9\uc544\uc57c \ud55c\ub2e4\uace0 \ube44\ud310\ud588\ub2e4. \ub8cc\uc774\uce58\ub294 \"2008\ub144 \ub78c\uc0ac\ub974\ud611\uc57d \ub2f9\uc0ac\uad6d \ucd1d\ud68c\ub97c \uac1c\ucd5c, \ud55c\uad6d\uc774 \uc804 \uc138\uacc4\uc5d0 \uc2b5\uc9c0\ub97c \ubcf4\ud638\ud558\uaca0\ub2e4\uace0 \uc57d\uc18d\ud574 \ub193\uace0\ub3c4, \uc774\ub807\uac8c \uc2b5\uc9c0\ub97c \ud30c\uad34\ud558\ub294 \uac83\uc740 \uc788\uc744 \uc218 \uc5c6\ub294 \uc77c\"\uc774\ub77c\uace0 \uc9c0\uc801\ud588\ub2e4. \uc774\ub9c8\ubaa8\ud1a0 \ud788\ub85c\ub2e4\ucf00 \uad50\ud1a0\ub300 \uba85\uc608\uad50\uc218\ub3c4 \u201c4\ub300\uac15 \uacf5\uc0ac\uc7a5\uc744 \ub458\ub7ec\ubcf8 \uacb0\uacfc \u2018\uc6b4\ud558\uac74\uc124 \uc0ac\uc5c5\u2019\uc774\ub77c\ub294 \uc778\uc0c1\uc744 \uc9c0\uc6b8 \uc218 \uc5c6\uc5c8\ub2e4\"\uba70 \uc911\ub2e8\ud574\uc57c \ud55c\ub2e4\uace0 \ucd09\uad6c\ud588\ub2e4. \uc77c\ubcf8\ub78c\uc0ac\ub974\ub124\ud2b8\uc6cc\ud06c\uc640 \ud55c\uad6d\uc2b5\uc9c0NGO\ub124\ud2b8\uc6cc\ud06c\ub294 \"4\ub300\uac15 \uc0ac\uc5c5 \uc911\ub2e8\"\uacfc \ud568\uaed8 \"\uce58\uc218\u00b7\uc774\uc218\ub97c \ud3ec\ud568\ud55c \ubaa8\ub4e0 \uc0ac\uc5c5\ub4e4\uc758 \uc6d0\uc810\uc5d0\uc11c \uc7ac\uac80\ud1a0\ud574\uc57c \ud55c\ub2e4\"\ub77c\uace0 \uc9c0\uc801\ud588\ub2e4. \ub9f7 \ucf58\ub3cc\ud504 (Matt Kondolf) \ubbf8\uad6d \uce98\ub9ac\ud3ec\ub2c8\uc544 \ub300\ud559\uad50 \ubc84\ud074\ub9ac \uad50\uc218\ub294 4\ub300\uac15 \uc0ac\uc5c5\uc744 \ube44\ud310\ud558\uc600\ub2e4. 2010\ub144 11\uc6d4 \uc11c\uc6b8\uc5d0\uc11c \uc5f4\ub9ac\ub294 G20 \uc815\uc0c1\ud68c\ub2f4\uc5d0\uc11c \uc815\ubd80\uac00 \uc77c\ubc29\uc801\uc73c\ub85c 4\ub300\uac15 \ud64d\ubcf4\ub97c \ud558\uc600\ub294\ub370 \uc774\uc5d0 \ub300\ud574 \uad6d\ub0b4 \uc77c\ubd80 \uae30\uc790\ub4e4\uc774 \ub0c9\ub2f4\ud55c \ubc18\uc751\uc744 \ubcf4\uc774\uae30\ub3c4 \ud588\ub2e4. 2017\ub144 11\uc6d4\uc5d0 \uc601\uad6d\uc758 \uc2e0\ubb38 \uac00\ub514\uc5b8\uc740 \uc774 \uc0ac\uc5c5\uc744 \uc870\uc120\ubbfc\uc8fc\uc8fc\uc758\uc778\ubbfc\uacf5\ud654\uad6d\uc758 \ub958\uacbd\ud638\ud154 \ub4f1\uacfc \ud568\uaed8 \uc138\uacc4 10\ub300 '\ub3c8\ub9cc \uba39\ub294 \uc4f0\ub808\uae30' \uac74\ucd95\ubb3c\u00b7\uc2dc\uc124\ub85c \uaf3d\uc558\ub2e4.", "answer": "\uc774\ub9c8\ubaa8\ud1a0 \ud788\ub85c\ub2e4\ucf00 \uad50\ud1a0\ub300 \uba85\uc608\uad50\uc218", "sentence": "\uc774\ub9c8\ubaa8\ud1a0 \ud788\ub85c\ub2e4\ucf00 \uad50\ud1a0\ub300 \uba85\uc608\uad50\uc218 \ub3c4 \u201c4\ub300\uac15 \uacf5\uc0ac\uc7a5\uc744 \ub458\ub7ec\ubcf8 \uacb0\uacfc \u2018\uc6b4\ud558\uac74\uc124 \uc0ac\uc5c5\u2019\uc774\ub77c\ub294 \uc778\uc0c1\uc744 \uc9c0\uc6b8 \uc218 \uc5c6\uc5c8\ub2e4\"\uba70 \uc911\ub2e8\ud574\uc57c \ud55c\ub2e4\uace0 \ucd09\uad6c\ud588\ub2e4.", "paragraph_sentence": "2010\ub144 7\uc6d4 11\uc77c YWCA \uac15\ub2f9\uc5d0\uc11c \uc5f4\ub9b0 4\ub300\uac15 \uc0ac\uc5c5 \ud55c\u00b7\uc77c \uc870\uc0ac\ub2e8 \ud1a0\ub860\ud68c\uc7a5\uc5d0\uc11c '\ud638\ub9ac \ub8cc\uc774\uce58' \ub78c\uc0ac\ub974 \ub124\ud2b8\uc6cc\ud06c \uc77c\ubcf8\ub300\ud45c\ub294 \"\ub78c\uc0ac\ub974 \ucd1d\ud68c\ub97c \uac1c\ucd5c\ud55c \ub098\ub77c\uac00 \ub9de\ub294\uac00. \ucda9\uaca9\uc801\uc774\ub2e4. 4\ub300\uac15 \uc0ac\uc5c5\uc740 \ubaa9\uc801\uc774 \ubd88\ubd84\uba85\ud558\uace0 \ub78c\uc0ac\ub974\ud611\uc57d\uacfc \uc0dd\ubb3c\ub2e4\uc591\uc131\ud611\uc57d\uc5d0 \uc704\ubc30\ub418\ub294 \uba85\ubc31\ud55c \ud658\uacbd\ud30c\uad34 \uc0ac\uc5c5\uc774\ub2e4\"\ub77c\uba70 \uc0ac\uc5c5\uc744 \ub9c9\uc544\uc57c \ud55c\ub2e4\uace0 \ube44\ud310\ud588\ub2e4. \ub8cc\uc774\uce58\ub294 \"2008\ub144 \ub78c\uc0ac\ub974\ud611\uc57d \ub2f9\uc0ac\uad6d \ucd1d\ud68c\ub97c \uac1c\ucd5c, \ud55c\uad6d\uc774 \uc804 \uc138\uacc4\uc5d0 \uc2b5\uc9c0\ub97c \ubcf4\ud638\ud558\uaca0\ub2e4\uace0 \uc57d\uc18d\ud574 \ub193\uace0\ub3c4, \uc774\ub807\uac8c \uc2b5\uc9c0\ub97c \ud30c\uad34\ud558\ub294 \uac83\uc740 \uc788\uc744 \uc218 \uc5c6\ub294 \uc77c\"\uc774\ub77c\uace0 \uc9c0\uc801\ud588\ub2e4. \uc774\ub9c8\ubaa8\ud1a0 \ud788\ub85c\ub2e4\ucf00 \uad50\ud1a0\ub300 \uba85\uc608\uad50\uc218 \ub3c4 \u201c4\ub300\uac15 \uacf5\uc0ac\uc7a5\uc744 \ub458\ub7ec\ubcf8 \uacb0\uacfc \u2018\uc6b4\ud558\uac74\uc124 \uc0ac\uc5c5\u2019\uc774\ub77c\ub294 \uc778\uc0c1\uc744 \uc9c0\uc6b8 \uc218 \uc5c6\uc5c8\ub2e4\"\uba70 \uc911\ub2e8\ud574\uc57c \ud55c\ub2e4\uace0 \ucd09\uad6c\ud588\ub2e4. \uc77c\ubcf8\ub78c\uc0ac\ub974\ub124\ud2b8\uc6cc\ud06c\uc640 \ud55c\uad6d\uc2b5\uc9c0NGO\ub124\ud2b8\uc6cc\ud06c\ub294 \"4\ub300\uac15 \uc0ac\uc5c5 \uc911\ub2e8\"\uacfc \ud568\uaed8 \"\uce58\uc218\u00b7\uc774\uc218\ub97c \ud3ec\ud568\ud55c \ubaa8\ub4e0 \uc0ac\uc5c5\ub4e4\uc758 \uc6d0\uc810\uc5d0\uc11c \uc7ac\uac80\ud1a0\ud574\uc57c \ud55c\ub2e4\"\ub77c\uace0 \uc9c0\uc801\ud588\ub2e4. \ub9f7 \ucf58\ub3cc\ud504 (Matt Kondolf) \ubbf8\uad6d \uce98\ub9ac\ud3ec\ub2c8\uc544 \ub300\ud559\uad50 \ubc84\ud074\ub9ac \uad50\uc218\ub294 4\ub300\uac15 \uc0ac\uc5c5\uc744 \ube44\ud310\ud558\uc600\ub2e4. 2010\ub144 11\uc6d4 \uc11c\uc6b8\uc5d0\uc11c \uc5f4\ub9ac\ub294 G20 \uc815\uc0c1\ud68c\ub2f4\uc5d0\uc11c \uc815\ubd80\uac00 \uc77c\ubc29\uc801\uc73c\ub85c 4\ub300\uac15 \ud64d\ubcf4\ub97c \ud558\uc600\ub294\ub370 \uc774\uc5d0 \ub300\ud574 \uad6d\ub0b4 \uc77c\ubd80 \uae30\uc790\ub4e4\uc774 \ub0c9\ub2f4\ud55c \ubc18\uc751\uc744 \ubcf4\uc774\uae30\ub3c4 \ud588\ub2e4. 2017\ub144 11\uc6d4\uc5d0 \uc601\uad6d\uc758 \uc2e0\ubb38 \uac00\ub514\uc5b8\uc740 \uc774 \uc0ac\uc5c5\uc744 \uc870\uc120\ubbfc\uc8fc\uc8fc\uc758\uc778\ubbfc\uacf5\ud654\uad6d\uc758 \ub958\uacbd\ud638\ud154 \ub4f1\uacfc \ud568\uaed8 \uc138\uacc4 10\ub300 '\ub3c8\ub9cc \uba39\ub294 \uc4f0\ub808\uae30' \uac74\ucd95\ubb3c\u00b7\uc2dc\uc124\ub85c \uaf3d\uc558\ub2e4.", "paragraph_answer": "2010\ub144 7\uc6d4 11\uc77c YWCA \uac15\ub2f9\uc5d0\uc11c \uc5f4\ub9b0 4\ub300\uac15 \uc0ac\uc5c5 \ud55c\u00b7\uc77c \uc870\uc0ac\ub2e8 \ud1a0\ub860\ud68c\uc7a5\uc5d0\uc11c '\ud638\ub9ac \ub8cc\uc774\uce58' \ub78c\uc0ac\ub974 \ub124\ud2b8\uc6cc\ud06c \uc77c\ubcf8\ub300\ud45c\ub294 \"\ub78c\uc0ac\ub974 \ucd1d\ud68c\ub97c \uac1c\ucd5c\ud55c \ub098\ub77c\uac00 \ub9de\ub294\uac00. \ucda9\uaca9\uc801\uc774\ub2e4. 4\ub300\uac15 \uc0ac\uc5c5\uc740 \ubaa9\uc801\uc774 \ubd88\ubd84\uba85\ud558\uace0 \ub78c\uc0ac\ub974\ud611\uc57d\uacfc \uc0dd\ubb3c\ub2e4\uc591\uc131\ud611\uc57d\uc5d0 \uc704\ubc30\ub418\ub294 \uba85\ubc31\ud55c \ud658\uacbd\ud30c\uad34 \uc0ac\uc5c5\uc774\ub2e4\"\ub77c\uba70 \uc0ac\uc5c5\uc744 \ub9c9\uc544\uc57c \ud55c\ub2e4\uace0 \ube44\ud310\ud588\ub2e4. \ub8cc\uc774\uce58\ub294 \"2008\ub144 \ub78c\uc0ac\ub974\ud611\uc57d \ub2f9\uc0ac\uad6d \ucd1d\ud68c\ub97c \uac1c\ucd5c, \ud55c\uad6d\uc774 \uc804 \uc138\uacc4\uc5d0 \uc2b5\uc9c0\ub97c \ubcf4\ud638\ud558\uaca0\ub2e4\uace0 \uc57d\uc18d\ud574 \ub193\uace0\ub3c4, \uc774\ub807\uac8c \uc2b5\uc9c0\ub97c \ud30c\uad34\ud558\ub294 \uac83\uc740 \uc788\uc744 \uc218 \uc5c6\ub294 \uc77c\"\uc774\ub77c\uace0 \uc9c0\uc801\ud588\ub2e4. \uc774\ub9c8\ubaa8\ud1a0 \ud788\ub85c\ub2e4\ucf00 \uad50\ud1a0\ub300 \uba85\uc608\uad50\uc218 \ub3c4 \u201c4\ub300\uac15 \uacf5\uc0ac\uc7a5\uc744 \ub458\ub7ec\ubcf8 \uacb0\uacfc \u2018\uc6b4\ud558\uac74\uc124 \uc0ac\uc5c5\u2019\uc774\ub77c\ub294 \uc778\uc0c1\uc744 \uc9c0\uc6b8 \uc218 \uc5c6\uc5c8\ub2e4\"\uba70 \uc911\ub2e8\ud574\uc57c \ud55c\ub2e4\uace0 \ucd09\uad6c\ud588\ub2e4. \uc77c\ubcf8\ub78c\uc0ac\ub974\ub124\ud2b8\uc6cc\ud06c\uc640 \ud55c\uad6d\uc2b5\uc9c0NGO\ub124\ud2b8\uc6cc\ud06c\ub294 \"4\ub300\uac15 \uc0ac\uc5c5 \uc911\ub2e8\"\uacfc \ud568\uaed8 \"\uce58\uc218\u00b7\uc774\uc218\ub97c \ud3ec\ud568\ud55c \ubaa8\ub4e0 \uc0ac\uc5c5\ub4e4\uc758 \uc6d0\uc810\uc5d0\uc11c \uc7ac\uac80\ud1a0\ud574\uc57c \ud55c\ub2e4\"\ub77c\uace0 \uc9c0\uc801\ud588\ub2e4. \ub9f7 \ucf58\ub3cc\ud504 (Matt Kondolf) \ubbf8\uad6d \uce98\ub9ac\ud3ec\ub2c8\uc544 \ub300\ud559\uad50 \ubc84\ud074\ub9ac \uad50\uc218\ub294 4\ub300\uac15 \uc0ac\uc5c5\uc744 \ube44\ud310\ud558\uc600\ub2e4. 2010\ub144 11\uc6d4 \uc11c\uc6b8\uc5d0\uc11c \uc5f4\ub9ac\ub294 G20 \uc815\uc0c1\ud68c\ub2f4\uc5d0\uc11c \uc815\ubd80\uac00 \uc77c\ubc29\uc801\uc73c\ub85c 4\ub300\uac15 \ud64d\ubcf4\ub97c \ud558\uc600\ub294\ub370 \uc774\uc5d0 \ub300\ud574 \uad6d\ub0b4 \uc77c\ubd80 \uae30\uc790\ub4e4\uc774 \ub0c9\ub2f4\ud55c \ubc18\uc751\uc744 \ubcf4\uc774\uae30\ub3c4 \ud588\ub2e4. 2017\ub144 11\uc6d4\uc5d0 \uc601\uad6d\uc758 \uc2e0\ubb38 \uac00\ub514\uc5b8\uc740 \uc774 \uc0ac\uc5c5\uc744 \uc870\uc120\ubbfc\uc8fc\uc8fc\uc758\uc778\ubbfc\uacf5\ud654\uad6d\uc758 \ub958\uacbd\ud638\ud154 \ub4f1\uacfc \ud568\uaed8 \uc138\uacc4 10\ub300 '\ub3c8\ub9cc \uba39\ub294 \uc4f0\ub808\uae30' \uac74\ucd95\ubb3c\u00b7\uc2dc\uc124\ub85c \uaf3d\uc558\ub2e4.", "sentence_answer": " \uc774\ub9c8\ubaa8\ud1a0 \ud788\ub85c\ub2e4\ucf00 \uad50\ud1a0\ub300 \uba85\uc608\uad50\uc218 \ub3c4 \u201c4\ub300\uac15 \uacf5\uc0ac\uc7a5\uc744 \ub458\ub7ec\ubcf8 \uacb0\uacfc \u2018\uc6b4\ud558\uac74\uc124 \uc0ac\uc5c5\u2019\uc774\ub77c\ub294 \uc778\uc0c1\uc744 \uc9c0\uc6b8 \uc218 \uc5c6\uc5c8\ub2e4\"\uba70 \uc911\ub2e8\ud574\uc57c \ud55c\ub2e4\uace0 \ucd09\uad6c\ud588\ub2e4.", "paragraph_id": "5780412-2-0", "paragraph_question": "question: 4\ub300\uac15 \uacf5\uc0ac\uc7a5\uc744 \ub458\ub7ec\ubcf8 \uacb0\uacfc \uc6b4\ud558\uac74\uc124 \uc0ac\uc5c5\uc774\ub77c\ub294 \uc778\uc0c1\uc744 \uc9c0\uc6b8 \uc218 \uc5c6\uc5c8\ub2e4\uba70 \uc911\ub2e8\ud574\uc57c \ud55c\ub2e4\uace0 \ucd09\uad6c\ud55c \uc0ac\ub78c\uc740?, context: 2010\ub144 7\uc6d4 11\uc77c YWCA \uac15\ub2f9\uc5d0\uc11c \uc5f4\ub9b0 4\ub300\uac15 \uc0ac\uc5c5 \ud55c\u00b7\uc77c \uc870\uc0ac\ub2e8 \ud1a0\ub860\ud68c\uc7a5\uc5d0\uc11c '\ud638\ub9ac \ub8cc\uc774\uce58' \ub78c\uc0ac\ub974 \ub124\ud2b8\uc6cc\ud06c \uc77c\ubcf8\ub300\ud45c\ub294 \"\ub78c\uc0ac\ub974 \ucd1d\ud68c\ub97c \uac1c\ucd5c\ud55c \ub098\ub77c\uac00 \ub9de\ub294\uac00. \ucda9\uaca9\uc801\uc774\ub2e4. 4\ub300\uac15 \uc0ac\uc5c5\uc740 \ubaa9\uc801\uc774 \ubd88\ubd84\uba85\ud558\uace0 \ub78c\uc0ac\ub974\ud611\uc57d\uacfc \uc0dd\ubb3c\ub2e4\uc591\uc131\ud611\uc57d\uc5d0 \uc704\ubc30\ub418\ub294 \uba85\ubc31\ud55c \ud658\uacbd\ud30c\uad34 \uc0ac\uc5c5\uc774\ub2e4\"\ub77c\uba70 \uc0ac\uc5c5\uc744 \ub9c9\uc544\uc57c \ud55c\ub2e4\uace0 \ube44\ud310\ud588\ub2e4. \ub8cc\uc774\uce58\ub294 \"2008\ub144 \ub78c\uc0ac\ub974\ud611\uc57d \ub2f9\uc0ac\uad6d \ucd1d\ud68c\ub97c \uac1c\ucd5c, \ud55c\uad6d\uc774 \uc804 \uc138\uacc4\uc5d0 \uc2b5\uc9c0\ub97c \ubcf4\ud638\ud558\uaca0\ub2e4\uace0 \uc57d\uc18d\ud574 \ub193\uace0\ub3c4, \uc774\ub807\uac8c \uc2b5\uc9c0\ub97c \ud30c\uad34\ud558\ub294 \uac83\uc740 \uc788\uc744 \uc218 \uc5c6\ub294 \uc77c\"\uc774\ub77c\uace0 \uc9c0\uc801\ud588\ub2e4. \uc774\ub9c8\ubaa8\ud1a0 \ud788\ub85c\ub2e4\ucf00 \uad50\ud1a0\ub300 \uba85\uc608\uad50\uc218\ub3c4 \u201c4\ub300\uac15 \uacf5\uc0ac\uc7a5\uc744 \ub458\ub7ec\ubcf8 \uacb0\uacfc \u2018\uc6b4\ud558\uac74\uc124 \uc0ac\uc5c5\u2019\uc774\ub77c\ub294 \uc778\uc0c1\uc744 \uc9c0\uc6b8 \uc218 \uc5c6\uc5c8\ub2e4\"\uba70 \uc911\ub2e8\ud574\uc57c \ud55c\ub2e4\uace0 \ucd09\uad6c\ud588\ub2e4. \uc77c\ubcf8\ub78c\uc0ac\ub974\ub124\ud2b8\uc6cc\ud06c\uc640 \ud55c\uad6d\uc2b5\uc9c0NGO\ub124\ud2b8\uc6cc\ud06c\ub294 \"4\ub300\uac15 \uc0ac\uc5c5 \uc911\ub2e8\"\uacfc \ud568\uaed8 \"\uce58\uc218\u00b7\uc774\uc218\ub97c \ud3ec\ud568\ud55c \ubaa8\ub4e0 \uc0ac\uc5c5\ub4e4\uc758 \uc6d0\uc810\uc5d0\uc11c \uc7ac\uac80\ud1a0\ud574\uc57c \ud55c\ub2e4\"\ub77c\uace0 \uc9c0\uc801\ud588\ub2e4. \ub9f7 \ucf58\ub3cc\ud504 (Matt Kondolf) \ubbf8\uad6d \uce98\ub9ac\ud3ec\ub2c8\uc544 \ub300\ud559\uad50 \ubc84\ud074\ub9ac \uad50\uc218\ub294 4\ub300\uac15 \uc0ac\uc5c5\uc744 \ube44\ud310\ud558\uc600\ub2e4. 2010\ub144 11\uc6d4 \uc11c\uc6b8\uc5d0\uc11c \uc5f4\ub9ac\ub294 G20 \uc815\uc0c1\ud68c\ub2f4\uc5d0\uc11c \uc815\ubd80\uac00 \uc77c\ubc29\uc801\uc73c\ub85c 4\ub300\uac15 \ud64d\ubcf4\ub97c \ud558\uc600\ub294\ub370 \uc774\uc5d0 \ub300\ud574 \uad6d\ub0b4 \uc77c\ubd80 \uae30\uc790\ub4e4\uc774 \ub0c9\ub2f4\ud55c \ubc18\uc751\uc744 \ubcf4\uc774\uae30\ub3c4 \ud588\ub2e4. 2017\ub144 11\uc6d4\uc5d0 \uc601\uad6d\uc758 \uc2e0\ubb38 \uac00\ub514\uc5b8\uc740 \uc774 \uc0ac\uc5c5\uc744 \uc870\uc120\ubbfc\uc8fc\uc8fc\uc758\uc778\ubbfc\uacf5\ud654\uad6d\uc758 \ub958\uacbd\ud638\ud154 \ub4f1\uacfc \ud568\uaed8 \uc138\uacc4 10\ub300 '\ub3c8\ub9cc \uba39\ub294 \uc4f0\ub808\uae30' \uac74\ucd95\ubb3c\u00b7\uc2dc\uc124\ub85c \uaf3d\uc558\ub2e4."} {"question": "\uce58\uce58\uce74\uc2a4\ud14c\ub0ad\uace0\uc758 \uc2dc\uc7a5\uc774 \uc5f4\ub9ac\ub294 \uc694\uc77c\uc740?", "paragraph": "\uacfc\ud14c\ub9d0\ub77c\ub294 300\u223c900\ub144\uacbd\uc5d0 \uc6d0\uc8fc\ubbfc \ub9c8\uc57c\ubbfc\uc871\uc774 \ucc2c\ub780\ud55c \ub9c8\uc57c\ubb38\uba85\uc744 \uaf43\ud53c\uc6e0\ub358 \uacf3\uc774\ub098 \ud604\uc7ac\ub294 \uad6d\ubbfc\uc758 \uc808\ubc18 \uc774\uc0c1\uc774 \ubb38\ub9f9\uc774\uba70, \uc758\ubb34\uad50\uc721\uc81c\ub3c4\uc758 \ud6a8\uacfc\ub97c \uac70\ub450\uc9c0 \ubabb\ud558\uace0 \uc788\ub2e4. \uacfc\ud14c\ub9d0\ub77c\uc758 \ubbfc\uc18d\uc545\uae30 \ub9c8\ub9bc\ubc14\ub294 \uc138\uacc4\uc801\uc73c\ub85c \uc720\uba85\ud558\ub2e4. \uacf3\uacf3\uc5d0 \ub9c8\uc57c\uc758 \ud48d\uc18d\uc774 \uc9d9\uac8c \ub0a8\uc544 \uc788\uc5b4, \ub3c4\uc2dc \ud2b9\uad8c\uce35\uc758 \uc11c\uad6c \uc0dd\ud65c\uc591\uc2dd\uacfc \ub300\uc870\ub97c \ubcf4\uc774\uace0 \uc788\ub2e4. \uacfc\ud14c\ub9d0\ub77c\ub294 \ub9c8\uc744\uc5d0 \ub530\ub77c \uc637\uc0c9\uae54\uc774\ub098 \uba38\ub9ac \uc7a5\uc2dd\uc774 \ub2e4\ub978\ub370, \ud558\ub098\uac19\uc774 \uc6d0\uc0c9\uc758 \uae30\ud558\ud559\uc801 \ubb34\ub2ac\ub97c \uc990\uaca8 \uc0ac\uc6a9\ud55c\ub2e4. \uadf8\ub798\uc11c \uacfc\ud14c\ub9d0\ub77c\uc758 \uc2dc\uc7a5\uc740 \ud56d\uc0c1 \uc6d0\uc0c9\uc758 \ubb3c\uacb0\uc744 \uc774\ub8ec\ub2e4. \ud574\ubc1c 2071m\uc5d0 \uc790\ub9ac\uc7a1\uc740 \uce58\uce58\uce74\uc2a4\ud14c\ub0ad\uace0\ub294 \uc778\ub514\uc624\ub9d0\ub85c \uac00\uc2dc\uac00 \uc788\ub294 \uc9c0\uce58\uc790\ub77c\ub294 \ub545\uc744 \ub73b\ud558\ub294\ub370 \ubaa9\uc694\uc77c\uacfc \uc77c\uc694\uc77c\uc5d0 \uc11c\ub294 \uc2dc\uc7a5\uc73c\ub85c \uc720\uba85\ud55c \uacf3\uc774\ub2e4. \uacfc\ud14c\ub9d0\ub77c\uc758 \ucd95\uc81c\ub294 \uadf8\ub9ac\uc2a4\ub3c4\uad50\uc640 \ub9c8\uc57c \ud1a0\ucc29\uc885\uad50\uc758 \uc81c\uc0ac\uac00 \ud63c\ud569\ub41c \uac83\uc73c\ub85c \uadf8 \uc218\uac00 \ub9ce\ub2e4. \ub9c8\uc744\ub9c8\ub2e4 \uc218\ud638 \uc131\uc778\uc758 \ucd95\uc81c\uc77c\uc774 \uc788\ub2e4. \ucd95\uc81c\uc77c\uc5d0\ub294 \ud070\ubd81, \ud53c\ub9ac \ub4f1\uc758 \ubbfc\uc18d\uc545\uae30 \uc5f0\uc8fc\uc640 \uc815\ubcf5\uc790\uc758 \ucda4, \ubb34\uc5b4\uc778\uacfc \uadf8\ub9ac\uc2a4\ub3c4\uad50\ub3c4\uc758 \ucda4 \ub4f1 \uc804\ud1b5\ubb34\uc6a9\uc744 \ubcfc \uc218 \uc788\ub2e4.", "answer": "\ubaa9\uc694\uc77c\uacfc \uc77c\uc694\uc77c", "sentence": "\ud574\ubc1c 2071m\uc5d0 \uc790\ub9ac\uc7a1\uc740 \uce58\uce58\uce74\uc2a4\ud14c\ub0ad\uace0\ub294 \uc778\ub514\uc624\ub9d0\ub85c \uac00\uc2dc\uac00 \uc788\ub294 \uc9c0\uce58\uc790\ub77c\ub294 \ub545\uc744 \ub73b\ud558\ub294\ub370 \ubaa9\uc694\uc77c\uacfc \uc77c\uc694\uc77c \uc5d0 \uc11c\ub294 \uc2dc\uc7a5\uc73c\ub85c \uc720\uba85\ud55c \uacf3\uc774\ub2e4.", "paragraph_sentence": "\uacfc\ud14c\ub9d0\ub77c\ub294 300\u223c900\ub144\uacbd\uc5d0 \uc6d0\uc8fc\ubbfc \ub9c8\uc57c\ubbfc\uc871\uc774 \ucc2c\ub780\ud55c \ub9c8\uc57c\ubb38\uba85\uc744 \uaf43\ud53c\uc6e0\ub358 \uacf3\uc774\ub098 \ud604\uc7ac\ub294 \uad6d\ubbfc\uc758 \uc808\ubc18 \uc774\uc0c1\uc774 \ubb38\ub9f9\uc774\uba70, \uc758\ubb34\uad50\uc721\uc81c\ub3c4\uc758 \ud6a8\uacfc\ub97c \uac70\ub450\uc9c0 \ubabb\ud558\uace0 \uc788\ub2e4. \uacfc\ud14c\ub9d0\ub77c\uc758 \ubbfc\uc18d\uc545\uae30 \ub9c8\ub9bc\ubc14\ub294 \uc138\uacc4\uc801\uc73c\ub85c \uc720\uba85\ud558\ub2e4. \uacf3\uacf3\uc5d0 \ub9c8\uc57c\uc758 \ud48d\uc18d\uc774 \uc9d9\uac8c \ub0a8\uc544 \uc788\uc5b4, \ub3c4\uc2dc \ud2b9\uad8c\uce35\uc758 \uc11c\uad6c \uc0dd\ud65c\uc591\uc2dd\uacfc \ub300\uc870\ub97c \ubcf4\uc774\uace0 \uc788\ub2e4. \uacfc\ud14c\ub9d0\ub77c\ub294 \ub9c8\uc744\uc5d0 \ub530\ub77c \uc637\uc0c9\uae54\uc774\ub098 \uba38\ub9ac \uc7a5\uc2dd\uc774 \ub2e4\ub978\ub370, \ud558\ub098\uac19\uc774 \uc6d0\uc0c9\uc758 \uae30\ud558\ud559\uc801 \ubb34\ub2ac\ub97c \uc990\uaca8 \uc0ac\uc6a9\ud55c\ub2e4. \uadf8\ub798\uc11c \uacfc\ud14c\ub9d0\ub77c\uc758 \uc2dc\uc7a5\uc740 \ud56d\uc0c1 \uc6d0\uc0c9\uc758 \ubb3c\uacb0\uc744 \uc774\ub8ec\ub2e4. \ud574\ubc1c 2071m\uc5d0 \uc790\ub9ac\uc7a1\uc740 \uce58\uce58\uce74\uc2a4\ud14c\ub0ad\uace0\ub294 \uc778\ub514\uc624\ub9d0\ub85c \uac00\uc2dc\uac00 \uc788\ub294 \uc9c0\uce58\uc790\ub77c\ub294 \ub545\uc744 \ub73b\ud558\ub294\ub370 \ubaa9\uc694\uc77c\uacfc \uc77c\uc694\uc77c \uc5d0 \uc11c\ub294 \uc2dc\uc7a5\uc73c\ub85c \uc720\uba85\ud55c \uacf3\uc774\ub2e4. \uacfc\ud14c\ub9d0\ub77c\uc758 \ucd95\uc81c\ub294 \uadf8\ub9ac\uc2a4\ub3c4\uad50\uc640 \ub9c8\uc57c \ud1a0\ucc29\uc885\uad50\uc758 \uc81c\uc0ac\uac00 \ud63c\ud569\ub41c \uac83\uc73c\ub85c \uadf8 \uc218\uac00 \ub9ce\ub2e4. \ub9c8\uc744\ub9c8\ub2e4 \uc218\ud638 \uc131\uc778\uc758 \ucd95\uc81c\uc77c\uc774 \uc788\ub2e4. \ucd95\uc81c\uc77c\uc5d0\ub294 \ud070\ubd81, \ud53c\ub9ac \ub4f1\uc758 \ubbfc\uc18d\uc545\uae30 \uc5f0\uc8fc\uc640 \uc815\ubcf5\uc790\uc758 \ucda4, \ubb34\uc5b4\uc778\uacfc \uadf8\ub9ac\uc2a4\ub3c4\uad50\ub3c4\uc758 \ucda4 \ub4f1 \uc804\ud1b5\ubb34\uc6a9\uc744 \ubcfc \uc218 \uc788\ub2e4.", "paragraph_answer": "\uacfc\ud14c\ub9d0\ub77c\ub294 300\u223c900\ub144\uacbd\uc5d0 \uc6d0\uc8fc\ubbfc \ub9c8\uc57c\ubbfc\uc871\uc774 \ucc2c\ub780\ud55c \ub9c8\uc57c\ubb38\uba85\uc744 \uaf43\ud53c\uc6e0\ub358 \uacf3\uc774\ub098 \ud604\uc7ac\ub294 \uad6d\ubbfc\uc758 \uc808\ubc18 \uc774\uc0c1\uc774 \ubb38\ub9f9\uc774\uba70, \uc758\ubb34\uad50\uc721\uc81c\ub3c4\uc758 \ud6a8\uacfc\ub97c \uac70\ub450\uc9c0 \ubabb\ud558\uace0 \uc788\ub2e4. \uacfc\ud14c\ub9d0\ub77c\uc758 \ubbfc\uc18d\uc545\uae30 \ub9c8\ub9bc\ubc14\ub294 \uc138\uacc4\uc801\uc73c\ub85c \uc720\uba85\ud558\ub2e4. \uacf3\uacf3\uc5d0 \ub9c8\uc57c\uc758 \ud48d\uc18d\uc774 \uc9d9\uac8c \ub0a8\uc544 \uc788\uc5b4, \ub3c4\uc2dc \ud2b9\uad8c\uce35\uc758 \uc11c\uad6c \uc0dd\ud65c\uc591\uc2dd\uacfc \ub300\uc870\ub97c \ubcf4\uc774\uace0 \uc788\ub2e4. \uacfc\ud14c\ub9d0\ub77c\ub294 \ub9c8\uc744\uc5d0 \ub530\ub77c \uc637\uc0c9\uae54\uc774\ub098 \uba38\ub9ac \uc7a5\uc2dd\uc774 \ub2e4\ub978\ub370, \ud558\ub098\uac19\uc774 \uc6d0\uc0c9\uc758 \uae30\ud558\ud559\uc801 \ubb34\ub2ac\ub97c \uc990\uaca8 \uc0ac\uc6a9\ud55c\ub2e4. \uadf8\ub798\uc11c \uacfc\ud14c\ub9d0\ub77c\uc758 \uc2dc\uc7a5\uc740 \ud56d\uc0c1 \uc6d0\uc0c9\uc758 \ubb3c\uacb0\uc744 \uc774\ub8ec\ub2e4. \ud574\ubc1c 2071m\uc5d0 \uc790\ub9ac\uc7a1\uc740 \uce58\uce58\uce74\uc2a4\ud14c\ub0ad\uace0\ub294 \uc778\ub514\uc624\ub9d0\ub85c \uac00\uc2dc\uac00 \uc788\ub294 \uc9c0\uce58\uc790\ub77c\ub294 \ub545\uc744 \ub73b\ud558\ub294\ub370 \ubaa9\uc694\uc77c\uacfc \uc77c\uc694\uc77c \uc5d0 \uc11c\ub294 \uc2dc\uc7a5\uc73c\ub85c \uc720\uba85\ud55c \uacf3\uc774\ub2e4. \uacfc\ud14c\ub9d0\ub77c\uc758 \ucd95\uc81c\ub294 \uadf8\ub9ac\uc2a4\ub3c4\uad50\uc640 \ub9c8\uc57c \ud1a0\ucc29\uc885\uad50\uc758 \uc81c\uc0ac\uac00 \ud63c\ud569\ub41c \uac83\uc73c\ub85c \uadf8 \uc218\uac00 \ub9ce\ub2e4. \ub9c8\uc744\ub9c8\ub2e4 \uc218\ud638 \uc131\uc778\uc758 \ucd95\uc81c\uc77c\uc774 \uc788\ub2e4. \ucd95\uc81c\uc77c\uc5d0\ub294 \ud070\ubd81, \ud53c\ub9ac \ub4f1\uc758 \ubbfc\uc18d\uc545\uae30 \uc5f0\uc8fc\uc640 \uc815\ubcf5\uc790\uc758 \ucda4, \ubb34\uc5b4\uc778\uacfc \uadf8\ub9ac\uc2a4\ub3c4\uad50\ub3c4\uc758 \ucda4 \ub4f1 \uc804\ud1b5\ubb34\uc6a9\uc744 \ubcfc \uc218 \uc788\ub2e4.", "sentence_answer": "\ud574\ubc1c 2071m\uc5d0 \uc790\ub9ac\uc7a1\uc740 \uce58\uce58\uce74\uc2a4\ud14c\ub0ad\uace0\ub294 \uc778\ub514\uc624\ub9d0\ub85c \uac00\uc2dc\uac00 \uc788\ub294 \uc9c0\uce58\uc790\ub77c\ub294 \ub545\uc744 \ub73b\ud558\ub294\ub370 \ubaa9\uc694\uc77c\uacfc \uc77c\uc694\uc77c \uc5d0 \uc11c\ub294 \uc2dc\uc7a5\uc73c\ub85c \uc720\uba85\ud55c \uacf3\uc774\ub2e4.", "paragraph_id": "5956261-3-1", "paragraph_question": "question: \uce58\uce58\uce74\uc2a4\ud14c\ub0ad\uace0\uc758 \uc2dc\uc7a5\uc774 \uc5f4\ub9ac\ub294 \uc694\uc77c\uc740?, context: \uacfc\ud14c\ub9d0\ub77c\ub294 300\u223c900\ub144\uacbd\uc5d0 \uc6d0\uc8fc\ubbfc \ub9c8\uc57c\ubbfc\uc871\uc774 \ucc2c\ub780\ud55c \ub9c8\uc57c\ubb38\uba85\uc744 \uaf43\ud53c\uc6e0\ub358 \uacf3\uc774\ub098 \ud604\uc7ac\ub294 \uad6d\ubbfc\uc758 \uc808\ubc18 \uc774\uc0c1\uc774 \ubb38\ub9f9\uc774\uba70, \uc758\ubb34\uad50\uc721\uc81c\ub3c4\uc758 \ud6a8\uacfc\ub97c \uac70\ub450\uc9c0 \ubabb\ud558\uace0 \uc788\ub2e4. \uacfc\ud14c\ub9d0\ub77c\uc758 \ubbfc\uc18d\uc545\uae30 \ub9c8\ub9bc\ubc14\ub294 \uc138\uacc4\uc801\uc73c\ub85c \uc720\uba85\ud558\ub2e4. \uacf3\uacf3\uc5d0 \ub9c8\uc57c\uc758 \ud48d\uc18d\uc774 \uc9d9\uac8c \ub0a8\uc544 \uc788\uc5b4, \ub3c4\uc2dc \ud2b9\uad8c\uce35\uc758 \uc11c\uad6c \uc0dd\ud65c\uc591\uc2dd\uacfc \ub300\uc870\ub97c \ubcf4\uc774\uace0 \uc788\ub2e4. \uacfc\ud14c\ub9d0\ub77c\ub294 \ub9c8\uc744\uc5d0 \ub530\ub77c \uc637\uc0c9\uae54\uc774\ub098 \uba38\ub9ac \uc7a5\uc2dd\uc774 \ub2e4\ub978\ub370, \ud558\ub098\uac19\uc774 \uc6d0\uc0c9\uc758 \uae30\ud558\ud559\uc801 \ubb34\ub2ac\ub97c \uc990\uaca8 \uc0ac\uc6a9\ud55c\ub2e4. \uadf8\ub798\uc11c \uacfc\ud14c\ub9d0\ub77c\uc758 \uc2dc\uc7a5\uc740 \ud56d\uc0c1 \uc6d0\uc0c9\uc758 \ubb3c\uacb0\uc744 \uc774\ub8ec\ub2e4. \ud574\ubc1c 2071m\uc5d0 \uc790\ub9ac\uc7a1\uc740 \uce58\uce58\uce74\uc2a4\ud14c\ub0ad\uace0\ub294 \uc778\ub514\uc624\ub9d0\ub85c \uac00\uc2dc\uac00 \uc788\ub294 \uc9c0\uce58\uc790\ub77c\ub294 \ub545\uc744 \ub73b\ud558\ub294\ub370 \ubaa9\uc694\uc77c\uacfc \uc77c\uc694\uc77c\uc5d0 \uc11c\ub294 \uc2dc\uc7a5\uc73c\ub85c \uc720\uba85\ud55c \uacf3\uc774\ub2e4. \uacfc\ud14c\ub9d0\ub77c\uc758 \ucd95\uc81c\ub294 \uadf8\ub9ac\uc2a4\ub3c4\uad50\uc640 \ub9c8\uc57c \ud1a0\ucc29\uc885\uad50\uc758 \uc81c\uc0ac\uac00 \ud63c\ud569\ub41c \uac83\uc73c\ub85c \uadf8 \uc218\uac00 \ub9ce\ub2e4. \ub9c8\uc744\ub9c8\ub2e4 \uc218\ud638 \uc131\uc778\uc758 \ucd95\uc81c\uc77c\uc774 \uc788\ub2e4. \ucd95\uc81c\uc77c\uc5d0\ub294 \ud070\ubd81, \ud53c\ub9ac \ub4f1\uc758 \ubbfc\uc18d\uc545\uae30 \uc5f0\uc8fc\uc640 \uc815\ubcf5\uc790\uc758 \ucda4, \ubb34\uc5b4\uc778\uacfc \uadf8\ub9ac\uc2a4\ub3c4\uad50\ub3c4\uc758 \ucda4 \ub4f1 \uc804\ud1b5\ubb34\uc6a9\uc744 \ubcfc \uc218 \uc788\ub2e4."} {"question": "\uc218\ud544 \uc694\uc815 \uc774\uc57c\uae30\uc5d0 \ub300\ud574\uc11c \uc81c 2\uc758 \uc138\uacc4\ub77c\uace0 \ud55c \uc778\ubb3c\uc740?", "paragraph": "\ubb38\ud559\uc5d0\uc11c \uc138\uacc4\uad00\uc744 \ub9cc\ub4dc\ub294 \uac83\uc740 \uc870\uc9c0 \ub9e5\ub3c4\ub110\ub4dc(George MacDonald), J. R. R. \ud1a8\ud0a8, \ub3c4\ub85c\uc2dc L. \uc138\uc774\uc5b4\uc988(Dorothy L. Sayers), C. S. \ub8e8\uc774\uc2a4 \ub4f1\uc758 \ud310\ud0c0\uc9c0 \uc791\uac00\ub4e4\uc5d0 \uc758\ud574 \ucd5c\ucd08\ub85c \uc2e4\ud5d8\ub418\uc5c8\ub2e4. \ud1a8\ud0a8\uc740 \uc774 \uc8fc\uc81c\ub97c \uadf8\uc758 \uc218\ud544 \uc694\uc815 \uc774\uc57c\uae30\uc5d0 \ub300\ud574\uc11c(On Fairy Stories)\uc5d0\uc11c, \"\uc81c2\uc758 \uc138\uacc4\" \ub610\ub294 \"\ud53c\uc870\ubb3c\"(\uac00\uacf5 \uc138\uacc4)\ub294 \uadf9\ubcf8\uc744 \uc4f0\ub294 \uac83\uacfc\ub294 \ub9ce\uc774 \ub2e4\ub974\ub2e4\uace0 \uc5b8\uae09\ud588\ub2e4: \"\ub098\ubb34\ub4e4\ub85c\uc11c\uc758 \ub098\ubb34\ub4e4\uc5d0 \ub300\ud574\uc11c\ub294 \uc544\uc8fc \uc870\uae08\ub9cc\uc774 \uc5f0\uadf9\uc5d0 \ub4e4\uc5b4\uac08 \uc218 \uc788\ub2e4.\" \uac00\uacf5 \uc138\uacc4\ub4e4\uc740 \uc2a4\ud1a0\ub9ac\ud154\ub9c1, \uc774\uc57c\uae30, \ub4f1\uc7a5\uc778\ubb3c\uacfc \ubd84\ub9ac\ub418\uc5b4, \"\ub098\ubb34\ub4e4\ub85c\uc11c\uc758 \ub098\ubb34\ub4e4\" \ub610\ub294 \uac00\uacf5 \uc138\uacc4 \uc790\uccb4\uc5d0 \ub300\ud55c \uba74\ub4e4\uc744 \ud0d0\uc0c9\ud558\uac8c \ud560 \uc218 \uc788\ub2e4. 1960\ub144\ub300\uc758 \ud310\ud0c0\uc9c0 \uc138\uacc4\uc5d0 \ub300\ud55c \uc774\ud6c4\uc758 \ubd84\uc11d\uc740, \uadf8 \uc138\uacc4 \uc18d\uc758 \uc774\uc57c\uae30\ub4e4\uc5d0 \ub300\ud55c \ubd84\uc11d\uc744 \uc81c\uacf5\ud558\uba74\uc11c, \uadf8\ub4e4\uc744 \uc138\uacc4 \uc790\uccb4\ub85c\uc11c\uac00 \uc544\ub2c8\ub77c \ub9e4\uccb4\uc640 \uc11c\uc0ac \uce21\uba74\uc5d0\uc11c \ub9e5\ub77d\ud654\ud588\ub2e4. \ubbf8\ub514\uc5b4 \ud504\ub79c\ucc28\uc774\uc988\ub4e4\ub3c4 \uac00\uacf5 \uc138\uacc4\ub4e4\uc5d0 \ub300\ud574\uc11c\ub294 \uc774\uc640 \ube44\uc2b7\ud588\uc9c0\ub9cc \uc774\uc5d0 \uc644\uc804\ud788 \uc758\uc874\ud558\uc9c0\ub294 \uc54a\uc558\ub2e4; \uc608\ub97c \ub4e4\uc5b4 \uc2a4\ud0c0\uc6cc\uc988\uc5d0\ub294 \uac00\uacf5 \uc138\uacc4\uac00 \uc788\uc9c0\ub9cc, \ud504\ub79c\ucc28\uc774\uc988\ub85c\uc11c\uc758 \ube44\ud2c0\uc988\ub294 \uadf8\ub807\uc9c0 \uc54a\ub2e4. 2000\ub144\ub300\uc5d0\ub294, \uc601\ud654\uc5d0\uc11c\uc758 \uc138\uacc4 \uc124\uc815\uc774 \uc778\uae30\ub97c \uc5bb\uc5c8\ub2e4. \uadf8 \uc774\uc804\uc5d0\ub294, \uc791\uac00\ub4e4\uc774 \uc5ec\ub7ec \uc774\uc57c\uae30\ub4e4\uc744 \uac10\ub2f9\ud560 \uc218 \uc788\ub294 \ub4f1\uc7a5\uc778\ubb3c\uc744 \ub9cc\ub4e4\ub824 \ud588\uc9c0\ub9cc, \uc9c0\uae08\uc740 \uc5ec\ub7ec \ub4f1\uc7a5\uc778\ubb3c\ub4e4\uacfc \uc774\uc57c\uae30\ub4e4\uc744 \uac10\ub2f9\ud560 \uc218 \uc788\ub294 \uc138\uacc4\ub97c \ub9cc\ub4e4\uace0 \uc788\ub2e4.", "answer": "\ud1a8\ud0a8", "sentence": "\ud1a8\ud0a8 ,", "paragraph_sentence": "\ubb38\ud559\uc5d0\uc11c \uc138\uacc4\uad00\uc744 \ub9cc\ub4dc\ub294 \uac83\uc740 \uc870\uc9c0 \ub9e5\ub3c4\ub110\ub4dc(George MacDonald), J. R. R. \ud1a8\ud0a8 , \ub3c4\ub85c\uc2dc L. \uc138\uc774\uc5b4\uc988(Dorothy L. Sayers), C. S. \ub8e8\uc774\uc2a4 \ub4f1\uc758 \ud310\ud0c0\uc9c0 \uc791\uac00\ub4e4\uc5d0 \uc758\ud574 \ucd5c\ucd08\ub85c \uc2e4\ud5d8\ub418\uc5c8\ub2e4. \ud1a8\ud0a8\uc740 \uc774 \uc8fc\uc81c\ub97c \uadf8\uc758 \uc218\ud544 \uc694\uc815 \uc774\uc57c\uae30\uc5d0 \ub300\ud574\uc11c(On Fairy Stories)\uc5d0\uc11c, \"\uc81c2\uc758 \uc138\uacc4\" \ub610\ub294 \"\ud53c\uc870\ubb3c\"(\uac00\uacf5 \uc138\uacc4)\ub294 \uadf9\ubcf8\uc744 \uc4f0\ub294 \uac83\uacfc\ub294 \ub9ce\uc774 \ub2e4\ub974\ub2e4\uace0 \uc5b8\uae09\ud588\ub2e4: \"\ub098\ubb34\ub4e4\ub85c\uc11c\uc758 \ub098\ubb34\ub4e4\uc5d0 \ub300\ud574\uc11c\ub294 \uc544\uc8fc \uc870\uae08\ub9cc\uc774 \uc5f0\uadf9\uc5d0 \ub4e4\uc5b4\uac08 \uc218 \uc788\ub2e4.\" \uac00\uacf5 \uc138\uacc4\ub4e4\uc740 \uc2a4\ud1a0\ub9ac\ud154\ub9c1, \uc774\uc57c\uae30, \ub4f1\uc7a5\uc778\ubb3c\uacfc \ubd84\ub9ac\ub418\uc5b4, \"\ub098\ubb34\ub4e4\ub85c\uc11c\uc758 \ub098\ubb34\ub4e4\" \ub610\ub294 \uac00\uacf5 \uc138\uacc4 \uc790\uccb4\uc5d0 \ub300\ud55c \uba74\ub4e4\uc744 \ud0d0\uc0c9\ud558\uac8c \ud560 \uc218 \uc788\ub2e4. 1960\ub144\ub300\uc758 \ud310\ud0c0\uc9c0 \uc138\uacc4\uc5d0 \ub300\ud55c \uc774\ud6c4\uc758 \ubd84\uc11d\uc740, \uadf8 \uc138\uacc4 \uc18d\uc758 \uc774\uc57c\uae30\ub4e4\uc5d0 \ub300\ud55c \ubd84\uc11d\uc744 \uc81c\uacf5\ud558\uba74\uc11c, \uadf8\ub4e4\uc744 \uc138\uacc4 \uc790\uccb4\ub85c\uc11c\uac00 \uc544\ub2c8\ub77c \ub9e4\uccb4\uc640 \uc11c\uc0ac \uce21\uba74\uc5d0\uc11c \ub9e5\ub77d\ud654\ud588\ub2e4. \ubbf8\ub514\uc5b4 \ud504\ub79c\ucc28\uc774\uc988\ub4e4\ub3c4 \uac00\uacf5 \uc138\uacc4\ub4e4\uc5d0 \ub300\ud574\uc11c\ub294 \uc774\uc640 \ube44\uc2b7\ud588\uc9c0\ub9cc \uc774\uc5d0 \uc644\uc804\ud788 \uc758\uc874\ud558\uc9c0\ub294 \uc54a\uc558\ub2e4; \uc608\ub97c \ub4e4\uc5b4 \uc2a4\ud0c0\uc6cc\uc988\uc5d0\ub294 \uac00\uacf5 \uc138\uacc4\uac00 \uc788\uc9c0\ub9cc, \ud504\ub79c\ucc28\uc774\uc988\ub85c\uc11c\uc758 \ube44\ud2c0\uc988\ub294 \uadf8\ub807\uc9c0 \uc54a\ub2e4. 2000\ub144\ub300\uc5d0\ub294, \uc601\ud654\uc5d0\uc11c\uc758 \uc138\uacc4 \uc124\uc815\uc774 \uc778\uae30\ub97c \uc5bb\uc5c8\ub2e4. \uadf8 \uc774\uc804\uc5d0\ub294, \uc791\uac00\ub4e4\uc774 \uc5ec\ub7ec \uc774\uc57c\uae30\ub4e4\uc744 \uac10\ub2f9\ud560 \uc218 \uc788\ub294 \ub4f1\uc7a5\uc778\ubb3c\uc744 \ub9cc\ub4e4\ub824 \ud588\uc9c0\ub9cc, \uc9c0\uae08\uc740 \uc5ec\ub7ec \ub4f1\uc7a5\uc778\ubb3c\ub4e4\uacfc \uc774\uc57c\uae30\ub4e4\uc744 \uac10\ub2f9\ud560 \uc218 \uc788\ub294 \uc138\uacc4\ub97c \ub9cc\ub4e4\uace0 \uc788\ub2e4.", "paragraph_answer": "\ubb38\ud559\uc5d0\uc11c \uc138\uacc4\uad00\uc744 \ub9cc\ub4dc\ub294 \uac83\uc740 \uc870\uc9c0 \ub9e5\ub3c4\ub110\ub4dc(George MacDonald), J. R. R. \ud1a8\ud0a8 , \ub3c4\ub85c\uc2dc L. \uc138\uc774\uc5b4\uc988(Dorothy L. Sayers), C. S. \ub8e8\uc774\uc2a4 \ub4f1\uc758 \ud310\ud0c0\uc9c0 \uc791\uac00\ub4e4\uc5d0 \uc758\ud574 \ucd5c\ucd08\ub85c \uc2e4\ud5d8\ub418\uc5c8\ub2e4. \ud1a8\ud0a8\uc740 \uc774 \uc8fc\uc81c\ub97c \uadf8\uc758 \uc218\ud544 \uc694\uc815 \uc774\uc57c\uae30\uc5d0 \ub300\ud574\uc11c(On Fairy Stories)\uc5d0\uc11c, \"\uc81c2\uc758 \uc138\uacc4\" \ub610\ub294 \"\ud53c\uc870\ubb3c\"(\uac00\uacf5 \uc138\uacc4)\ub294 \uadf9\ubcf8\uc744 \uc4f0\ub294 \uac83\uacfc\ub294 \ub9ce\uc774 \ub2e4\ub974\ub2e4\uace0 \uc5b8\uae09\ud588\ub2e4: \"\ub098\ubb34\ub4e4\ub85c\uc11c\uc758 \ub098\ubb34\ub4e4\uc5d0 \ub300\ud574\uc11c\ub294 \uc544\uc8fc \uc870\uae08\ub9cc\uc774 \uc5f0\uadf9\uc5d0 \ub4e4\uc5b4\uac08 \uc218 \uc788\ub2e4.\" \uac00\uacf5 \uc138\uacc4\ub4e4\uc740 \uc2a4\ud1a0\ub9ac\ud154\ub9c1, \uc774\uc57c\uae30, \ub4f1\uc7a5\uc778\ubb3c\uacfc \ubd84\ub9ac\ub418\uc5b4, \"\ub098\ubb34\ub4e4\ub85c\uc11c\uc758 \ub098\ubb34\ub4e4\" \ub610\ub294 \uac00\uacf5 \uc138\uacc4 \uc790\uccb4\uc5d0 \ub300\ud55c \uba74\ub4e4\uc744 \ud0d0\uc0c9\ud558\uac8c \ud560 \uc218 \uc788\ub2e4. 1960\ub144\ub300\uc758 \ud310\ud0c0\uc9c0 \uc138\uacc4\uc5d0 \ub300\ud55c \uc774\ud6c4\uc758 \ubd84\uc11d\uc740, \uadf8 \uc138\uacc4 \uc18d\uc758 \uc774\uc57c\uae30\ub4e4\uc5d0 \ub300\ud55c \ubd84\uc11d\uc744 \uc81c\uacf5\ud558\uba74\uc11c, \uadf8\ub4e4\uc744 \uc138\uacc4 \uc790\uccb4\ub85c\uc11c\uac00 \uc544\ub2c8\ub77c \ub9e4\uccb4\uc640 \uc11c\uc0ac \uce21\uba74\uc5d0\uc11c \ub9e5\ub77d\ud654\ud588\ub2e4. \ubbf8\ub514\uc5b4 \ud504\ub79c\ucc28\uc774\uc988\ub4e4\ub3c4 \uac00\uacf5 \uc138\uacc4\ub4e4\uc5d0 \ub300\ud574\uc11c\ub294 \uc774\uc640 \ube44\uc2b7\ud588\uc9c0\ub9cc \uc774\uc5d0 \uc644\uc804\ud788 \uc758\uc874\ud558\uc9c0\ub294 \uc54a\uc558\ub2e4; \uc608\ub97c \ub4e4\uc5b4 \uc2a4\ud0c0\uc6cc\uc988\uc5d0\ub294 \uac00\uacf5 \uc138\uacc4\uac00 \uc788\uc9c0\ub9cc, \ud504\ub79c\ucc28\uc774\uc988\ub85c\uc11c\uc758 \ube44\ud2c0\uc988\ub294 \uadf8\ub807\uc9c0 \uc54a\ub2e4. 2000\ub144\ub300\uc5d0\ub294, \uc601\ud654\uc5d0\uc11c\uc758 \uc138\uacc4 \uc124\uc815\uc774 \uc778\uae30\ub97c \uc5bb\uc5c8\ub2e4. \uadf8 \uc774\uc804\uc5d0\ub294, \uc791\uac00\ub4e4\uc774 \uc5ec\ub7ec \uc774\uc57c\uae30\ub4e4\uc744 \uac10\ub2f9\ud560 \uc218 \uc788\ub294 \ub4f1\uc7a5\uc778\ubb3c\uc744 \ub9cc\ub4e4\ub824 \ud588\uc9c0\ub9cc, \uc9c0\uae08\uc740 \uc5ec\ub7ec \ub4f1\uc7a5\uc778\ubb3c\ub4e4\uacfc \uc774\uc57c\uae30\ub4e4\uc744 \uac10\ub2f9\ud560 \uc218 \uc788\ub294 \uc138\uacc4\ub97c \ub9cc\ub4e4\uace0 \uc788\ub2e4.", "sentence_answer": " \ud1a8\ud0a8 ,", "paragraph_id": "6536338-1-0", "paragraph_question": "question: \uc218\ud544 \uc694\uc815 \uc774\uc57c\uae30\uc5d0 \ub300\ud574\uc11c \uc81c 2\uc758 \uc138\uacc4\ub77c\uace0 \ud55c \uc778\ubb3c\uc740?, context: \ubb38\ud559\uc5d0\uc11c \uc138\uacc4\uad00\uc744 \ub9cc\ub4dc\ub294 \uac83\uc740 \uc870\uc9c0 \ub9e5\ub3c4\ub110\ub4dc(George MacDonald), J. R. R. \ud1a8\ud0a8, \ub3c4\ub85c\uc2dc L. \uc138\uc774\uc5b4\uc988(Dorothy L. Sayers), C. S. \ub8e8\uc774\uc2a4 \ub4f1\uc758 \ud310\ud0c0\uc9c0 \uc791\uac00\ub4e4\uc5d0 \uc758\ud574 \ucd5c\ucd08\ub85c \uc2e4\ud5d8\ub418\uc5c8\ub2e4. \ud1a8\ud0a8\uc740 \uc774 \uc8fc\uc81c\ub97c \uadf8\uc758 \uc218\ud544 \uc694\uc815 \uc774\uc57c\uae30\uc5d0 \ub300\ud574\uc11c(On Fairy Stories)\uc5d0\uc11c, \"\uc81c2\uc758 \uc138\uacc4\" \ub610\ub294 \"\ud53c\uc870\ubb3c\"(\uac00\uacf5 \uc138\uacc4)\ub294 \uadf9\ubcf8\uc744 \uc4f0\ub294 \uac83\uacfc\ub294 \ub9ce\uc774 \ub2e4\ub974\ub2e4\uace0 \uc5b8\uae09\ud588\ub2e4: \"\ub098\ubb34\ub4e4\ub85c\uc11c\uc758 \ub098\ubb34\ub4e4\uc5d0 \ub300\ud574\uc11c\ub294 \uc544\uc8fc \uc870\uae08\ub9cc\uc774 \uc5f0\uadf9\uc5d0 \ub4e4\uc5b4\uac08 \uc218 \uc788\ub2e4.\" \uac00\uacf5 \uc138\uacc4\ub4e4\uc740 \uc2a4\ud1a0\ub9ac\ud154\ub9c1, \uc774\uc57c\uae30, \ub4f1\uc7a5\uc778\ubb3c\uacfc \ubd84\ub9ac\ub418\uc5b4, \"\ub098\ubb34\ub4e4\ub85c\uc11c\uc758 \ub098\ubb34\ub4e4\" \ub610\ub294 \uac00\uacf5 \uc138\uacc4 \uc790\uccb4\uc5d0 \ub300\ud55c \uba74\ub4e4\uc744 \ud0d0\uc0c9\ud558\uac8c \ud560 \uc218 \uc788\ub2e4. 1960\ub144\ub300\uc758 \ud310\ud0c0\uc9c0 \uc138\uacc4\uc5d0 \ub300\ud55c \uc774\ud6c4\uc758 \ubd84\uc11d\uc740, \uadf8 \uc138\uacc4 \uc18d\uc758 \uc774\uc57c\uae30\ub4e4\uc5d0 \ub300\ud55c \ubd84\uc11d\uc744 \uc81c\uacf5\ud558\uba74\uc11c, \uadf8\ub4e4\uc744 \uc138\uacc4 \uc790\uccb4\ub85c\uc11c\uac00 \uc544\ub2c8\ub77c \ub9e4\uccb4\uc640 \uc11c\uc0ac \uce21\uba74\uc5d0\uc11c \ub9e5\ub77d\ud654\ud588\ub2e4. \ubbf8\ub514\uc5b4 \ud504\ub79c\ucc28\uc774\uc988\ub4e4\ub3c4 \uac00\uacf5 \uc138\uacc4\ub4e4\uc5d0 \ub300\ud574\uc11c\ub294 \uc774\uc640 \ube44\uc2b7\ud588\uc9c0\ub9cc \uc774\uc5d0 \uc644\uc804\ud788 \uc758\uc874\ud558\uc9c0\ub294 \uc54a\uc558\ub2e4; \uc608\ub97c \ub4e4\uc5b4 \uc2a4\ud0c0\uc6cc\uc988\uc5d0\ub294 \uac00\uacf5 \uc138\uacc4\uac00 \uc788\uc9c0\ub9cc, \ud504\ub79c\ucc28\uc774\uc988\ub85c\uc11c\uc758 \ube44\ud2c0\uc988\ub294 \uadf8\ub807\uc9c0 \uc54a\ub2e4. 2000\ub144\ub300\uc5d0\ub294, \uc601\ud654\uc5d0\uc11c\uc758 \uc138\uacc4 \uc124\uc815\uc774 \uc778\uae30\ub97c \uc5bb\uc5c8\ub2e4. \uadf8 \uc774\uc804\uc5d0\ub294, \uc791\uac00\ub4e4\uc774 \uc5ec\ub7ec \uc774\uc57c\uae30\ub4e4\uc744 \uac10\ub2f9\ud560 \uc218 \uc788\ub294 \ub4f1\uc7a5\uc778\ubb3c\uc744 \ub9cc\ub4e4\ub824 \ud588\uc9c0\ub9cc, \uc9c0\uae08\uc740 \uc5ec\ub7ec \ub4f1\uc7a5\uc778\ubb3c\ub4e4\uacfc \uc774\uc57c\uae30\ub4e4\uc744 \uac10\ub2f9\ud560 \uc218 \uc788\ub294 \uc138\uacc4\ub97c \ub9cc\ub4e4\uace0 \uc788\ub2e4."} {"question": "1979\ub144 \uba55\uc2dc\ucf54 \uc720\uc815\uc5d0\uc11c \ubc1c\uc0dd\ud55c \uc0ac\uace0\uc758 \uc774\ub984\uc740?", "paragraph": "1979\ub144 \uba55\uc2dc\ucf54 \uc815\ubd80 \uc18c\uc720\uc758 \uc720\uc815\uc5d0\uc11c \ubc1c\uc0dd\ud55c \uc775\uc2a4\ud1a1 \uc720\ucd9c \uc0ac\uace0\ub294 \uc5ed\uc0ac\uc0c1 \ucd5c\uc545\uc758 \uc0ac\uace0\ub85c \uae30\ub85d\ub418\uc5c8\ub2e4. \ub2f9\uc2dc \uc720\uc815 \uc804\ubb38\uac00 \ub808\ub4dc \uc544\ub4dc\ub808\ub294 \uc774 \uc0ac\uac74\uc5d0 \ud22c\uc785\ub418\uc5c8\uace0 \ud3ab \ucf10\ubca8\ub3c4 \ud568\uaed8 \uc77c\ud588\ub2e4. \uc774\ub4e4\uc740 \ubc18\uad6c\ub97c \ub9cc\ub4e4\uc5b4 \uc720\uc815\uc5d0 \uc50c\uc6b0\ub294 \uc791\uc5c5\uc73c\ub85c \uc0ac\uace0\ub97c \uc218\uc2b5\ud588\uc5c8\ub2e4. \uc775\uc2a4\ud1a1 \uc720\ucd9c\uc740 \ubc14\ub2e4 \ubc11 48\ubbf8\ud130\uc5d0 \ubc1c\uc0dd\ud55c \uac83\uc774\uc5c8\ub2e4. \uacfc\uc5f0 1,500\ubbf8\ud130 \uc544\ub798\uc5d0\uc11c\ub3c4 \uc774 \ubc18\uad6c\uac00 \ud6a8\uacfc\uc801\uc77c\uc9c0\ub294 \uc54c \uc218 \uc5c6\uc5c8\ub2e4. \ud3ab \ucf10\ubca8\uc740 \ud478\uc26c\uc639 \ud56d\uad6c\uc5d0\uc11c \ubc18\uad6c\ub97c \ub9cc\ub4e4\uc5c8\ub2e4. \uac00\ubb3c\ub9c9\uc774 \ub310\uc774\ub2e4. \ud574\uc800 \uc190\uc0c1\ub41c \uc720\uc815\uc5d0\uc11c \uc791\uc5c5\uc744 \ud574\uc57c \ud558\ub294 \ub2e4\uc774\ubc84\ub4e4\uc774 \uc274 \uc218 \uc788\ub294 \uc6a9\ub3c4\ub85c \ub9cc\ub4e4\uc5b4\uc84c\ub2e4. \ub192\uc774 4.5\ubbf8\ud130\uc5d0 \ud3ed\uc740 3\ubbf8\ud130\uc600\ub2e4. \uc2dc\ucd94 \ud30c\uc774\ud504 \ub05d\ubd80\ubd84\uc5d0 \uac00\ubb3c\ub9c9\uc774\ub97c \ub36e\uc744 \uacc4\ud68d\uc774\uc5c8\ub2e4. \uc11d\uc720 \ub300\ubd80\ubd84\uc774 \uac00\ubb3c\ub9c9\uc774 \uc548\uc5d0 \uac07\ud788\uac8c \ub418\uba74 \uad6c\uba4d\uc5d0 \uae54\ub54c\uae30\ub97c \uaf42\uc544 \ubc30\uc704\ub85c \uc11d\uc720\ub97c \ub04c\uc5b4 \uc62c\ub9ac\ub294 \uac83\uc774\uc5c8\ub2e4. \uac00\ubb3c\ub9c9\uc774\ub294 \ud574\uc800 \ubc14\ub2e5 \ud658\uacbd\uc5d0 \uc801\ud569\ud558\ub3c4\ub85d \uc0ac\ubc29 \uc5b4\ub290 \uacf3\uc774\ub4e0 \ud1b5\ub85c\ub97c \uc124\uce58 \ud560 \uc218 \uc788\ub3c4\ub85d \uc218\uc815\ud588\ub2e4. \uc774 \uac00\ubb3c\ub9c9\uc774\ub294 \uc720\uc815\uc5d0\uc11c \ub098\uc624\ub294 \uc11d\uc720\uc758 80%\uc815\ub3c4\ub294 \ubaa8\uc744 \uc218 \uc788\uc744\uac70\ub77c\ub294 \uae30\ub300\uac10\uc5d0 \ud729\uc2f8\uc600\ub2e4. 1,500\ubbf8\ud130 \uc544\ub798 \uc12d\uc528 148\ub3c4\uc5d0 \uc774\ub974\ub294 \ub728\uac70\uc6b4 \uc11d\uc720\uc640, \uba54\ud0c4 \uac00\uc2a4\ub97c \ubfdc\uc5b4\ub0b4\uace0 \uc788\uc5c8\ub2e4. \uc5c4\uccad\ub09c \uc555\ub825 \uc18d\uc5d0 \uac00\uc2a4\uc640 \ubd80\ub52a\ud788\ub294 \ubc14\ub2f7\ubb3c\uc740 \uc218\ud654\ub418\uc5b4 \uc5bc\uc74c\uacfc \ube44\uc2b7\ud55c \ubb3c\uc9c8\uc774 \ub418\uc5b4 \uac00\ubb3c\ub9c9\uc774\ub97c \uc190\uc0c1\uc2dc\ud0ac \uc218 \uc788\uc5c8\ub2e4. \uc774\ub97c \ubc29\uc9c0\ud558\uae30 \uc704\ud574 \ubc18\uad6c \uc548\uc5d0 \ubd84\uc0ac\ud3ec\ud2b8 \uc124\uce58\ub97c \uacc4\ud68d\ud588\ub2e4. \uc717\ubd80\ubd84\uc5d0 \ub179\uc0c9 \ud30c\uc774\ud504\uac00 \uc788\ub294\ub370 \uc774 \ubd84\uc0ac\ud3ec\ud2b8\uac00 \uba54\ud0c4\uc62c\uc758 \uc218\ud654\ub97c \ub9c9\ub294 \uc5ed\ud560\uc744 \ud558\ub294 \uac83\uc774\ub2e4. \uc774 \uc791\uc5c5\uae4c\uc9c0 18\uc77c\uc774 \uac78\ub838\ub2e4. 87\ud1a4\uc758 \uac00\ubb3c\ub9c9\uc774\uac00 \ubc14\ub2e5\uae4c\uc9c0 \uac00\ub77c\uc549\ub294\ub370 6\uc2dc\uac04\uc774 \uac78\ub838\ub2e4. \ubc14\ub2e5\uc5d0 \uac00\ub77c\uc549\uc790 \uc6d0\uc720\uac00 \uac00\ubb3c\ub9c9\uc774 \ubc14\ub2e5\uc5d0\uc11c \ubfdc\uc5b4\uc838 \ub098\uc624\ub294 \uac83\ucc98\ub7fc \ubcf4\uc600\ub2e4. \ud0c4\ud654\uc218\uc18c\uac00 \uac00\ubb3c\ub9c9\uc774 \uc717\ubd80\ubd84\uc73c\ub85c \ub098\uc624\uc9c0 \uc54a\ub294 \uac83\uc774\uc5c8\ub2e4. \uc544\ubb34\ub3c4 \uc608\uc0c1\ud558\uc9c0 \ubabb\ud55c \uac83\uc774\uc5c8\ub2e4. \uac00\ubb3c\ub9c9\uc774\uac00 \ud558\uac15\ud558\ub294 \ub3d9\uc548 \uac00\uc2a4 \uae30\ub465\uc744 \uc774\ub3d9\uc2dc\ud0a4\uace0 \uc788\ub358 \uac83\uc774\uc5c8\ub2e4. \uac00\ubb3c\ub9c9\uc774\ub294 \ud574\uc218\uba74 \uadfc\ucc98\uc5d0\uc11c \ud0c4\ud654\uc218\uc18c\ub97c \ubaa8\uc558\uc73c\ub098 1,500\ubbf8\ud130 \uc544\ub798\ub85c \ub0b4\ub824\uac00\ub294\ub3d9\uc548 700\ubbf8\ud130 \uc9c0\uc810\ubd80\ud130 \uc5bc\uc74c\uc73c\ub85c \uac00\ub4dd \ucc28\uac8c \ub41c \uac83\uc774\uc5c8\ub2e4. \uad6c\uc870\ubb3c \ub0b4\ubd80\uc5d0\uc11c \uc774\ub8e8\uc5b4\uc9c4 \uc218\ud654\uc791\uc6a9\uc740 \uac00\uc2a4\uac00 \ube60\uc838\ub098\uac08 \uad6c\uba4d\uc744 \ub9c9\uc544\ubc84\ub838\ub2e4. \uacb0\uad6d \uc774 \uacc4\ud68d\uc740 \uc2e4\ud328\ub85c \ub3cc\uc544\uac14\ub2e4.", "answer": "\uc775\uc2a4\ud1a1 \uc720\ucd9c \uc0ac\uace0", "sentence": "1979\ub144 \uba55\uc2dc\ucf54 \uc815\ubd80 \uc18c\uc720\uc758 \uc720\uc815\uc5d0\uc11c \ubc1c\uc0dd\ud55c \uc775\uc2a4\ud1a1 \uc720\ucd9c \uc0ac\uace0 \ub294 \uc5ed\uc0ac\uc0c1 \ucd5c\uc545\uc758 \uc0ac\uace0\ub85c \uae30\ub85d\ub418\uc5c8\ub2e4.", "paragraph_sentence": " 1979\ub144 \uba55\uc2dc\ucf54 \uc815\ubd80 \uc18c\uc720\uc758 \uc720\uc815\uc5d0\uc11c \ubc1c\uc0dd\ud55c \uc775\uc2a4\ud1a1 \uc720\ucd9c \uc0ac\uace0 \ub294 \uc5ed\uc0ac\uc0c1 \ucd5c\uc545\uc758 \uc0ac\uace0\ub85c \uae30\ub85d\ub418\uc5c8\ub2e4. \ub2f9\uc2dc \uc720\uc815 \uc804\ubb38\uac00 \ub808\ub4dc \uc544\ub4dc\ub808\ub294 \uc774 \uc0ac\uac74\uc5d0 \ud22c\uc785\ub418\uc5c8\uace0 \ud3ab \ucf10\ubca8\ub3c4 \ud568\uaed8 \uc77c\ud588\ub2e4. \uc774\ub4e4\uc740 \ubc18\uad6c\ub97c \ub9cc\ub4e4\uc5b4 \uc720\uc815\uc5d0 \uc50c\uc6b0\ub294 \uc791\uc5c5\uc73c\ub85c \uc0ac\uace0\ub97c \uc218\uc2b5\ud588\uc5c8\ub2e4. \uc775\uc2a4\ud1a1 \uc720\ucd9c\uc740 \ubc14\ub2e4 \ubc11 48\ubbf8\ud130\uc5d0 \ubc1c\uc0dd\ud55c \uac83\uc774\uc5c8\ub2e4. \uacfc\uc5f0 1,500\ubbf8\ud130 \uc544\ub798\uc5d0\uc11c\ub3c4 \uc774 \ubc18\uad6c\uac00 \ud6a8\uacfc\uc801\uc77c\uc9c0\ub294 \uc54c \uc218 \uc5c6\uc5c8\ub2e4. \ud3ab \ucf10\ubca8\uc740 \ud478\uc26c\uc639 \ud56d\uad6c\uc5d0\uc11c \ubc18\uad6c\ub97c \ub9cc\ub4e4\uc5c8\ub2e4. \uac00\ubb3c\ub9c9\uc774 \ub310\uc774\ub2e4. \ud574\uc800 \uc190\uc0c1\ub41c \uc720\uc815\uc5d0\uc11c \uc791\uc5c5\uc744 \ud574\uc57c \ud558\ub294 \ub2e4\uc774\ubc84\ub4e4\uc774 \uc274 \uc218 \uc788\ub294 \uc6a9\ub3c4\ub85c \ub9cc\ub4e4\uc5b4\uc84c\ub2e4. \ub192\uc774 4.5\ubbf8\ud130\uc5d0 \ud3ed\uc740 3\ubbf8\ud130\uc600\ub2e4. \uc2dc\ucd94 \ud30c\uc774\ud504 \ub05d\ubd80\ubd84\uc5d0 \uac00\ubb3c\ub9c9\uc774\ub97c \ub36e\uc744 \uacc4\ud68d\uc774\uc5c8\ub2e4. \uc11d\uc720 \ub300\ubd80\ubd84\uc774 \uac00\ubb3c\ub9c9\uc774 \uc548\uc5d0 \uac07\ud788\uac8c \ub418\uba74 \uad6c\uba4d\uc5d0 \uae54\ub54c\uae30\ub97c \uaf42\uc544 \ubc30\uc704\ub85c \uc11d\uc720\ub97c \ub04c\uc5b4 \uc62c\ub9ac\ub294 \uac83\uc774\uc5c8\ub2e4. \uac00\ubb3c\ub9c9\uc774\ub294 \ud574\uc800 \ubc14\ub2e5 \ud658\uacbd\uc5d0 \uc801\ud569\ud558\ub3c4\ub85d \uc0ac\ubc29 \uc5b4\ub290 \uacf3\uc774\ub4e0 \ud1b5\ub85c\ub97c \uc124\uce58 \ud560 \uc218 \uc788\ub3c4\ub85d \uc218\uc815\ud588\ub2e4. \uc774 \uac00\ubb3c\ub9c9\uc774\ub294 \uc720\uc815\uc5d0\uc11c \ub098\uc624\ub294 \uc11d\uc720\uc758 80%\uc815\ub3c4\ub294 \ubaa8\uc744 \uc218 \uc788\uc744\uac70\ub77c\ub294 \uae30\ub300\uac10\uc5d0 \ud729\uc2f8\uc600\ub2e4. 1,500\ubbf8\ud130 \uc544\ub798 \uc12d\uc528 148\ub3c4\uc5d0 \uc774\ub974\ub294 \ub728\uac70\uc6b4 \uc11d\uc720\uc640, \uba54\ud0c4 \uac00\uc2a4\ub97c \ubfdc\uc5b4\ub0b4\uace0 \uc788\uc5c8\ub2e4. \uc5c4\uccad\ub09c \uc555\ub825 \uc18d\uc5d0 \uac00\uc2a4\uc640 \ubd80\ub52a\ud788\ub294 \ubc14\ub2f7\ubb3c\uc740 \uc218\ud654\ub418\uc5b4 \uc5bc\uc74c\uacfc \ube44\uc2b7\ud55c \ubb3c\uc9c8\uc774 \ub418\uc5b4 \uac00\ubb3c\ub9c9\uc774\ub97c \uc190\uc0c1\uc2dc\ud0ac \uc218 \uc788\uc5c8\ub2e4. \uc774\ub97c \ubc29\uc9c0\ud558\uae30 \uc704\ud574 \ubc18\uad6c \uc548\uc5d0 \ubd84\uc0ac\ud3ec\ud2b8 \uc124\uce58\ub97c \uacc4\ud68d\ud588\ub2e4. \uc717\ubd80\ubd84\uc5d0 \ub179\uc0c9 \ud30c\uc774\ud504\uac00 \uc788\ub294\ub370 \uc774 \ubd84\uc0ac\ud3ec\ud2b8\uac00 \uba54\ud0c4\uc62c\uc758 \uc218\ud654\ub97c \ub9c9\ub294 \uc5ed\ud560\uc744 \ud558\ub294 \uac83\uc774\ub2e4. \uc774 \uc791\uc5c5\uae4c\uc9c0 18\uc77c\uc774 \uac78\ub838\ub2e4. 87\ud1a4\uc758 \uac00\ubb3c\ub9c9\uc774\uac00 \ubc14\ub2e5\uae4c\uc9c0 \uac00\ub77c\uc549\ub294\ub370 6\uc2dc\uac04\uc774 \uac78\ub838\ub2e4. \ubc14\ub2e5\uc5d0 \uac00\ub77c\uc549\uc790 \uc6d0\uc720\uac00 \uac00\ubb3c\ub9c9\uc774 \ubc14\ub2e5\uc5d0\uc11c \ubfdc\uc5b4\uc838 \ub098\uc624\ub294 \uac83\ucc98\ub7fc \ubcf4\uc600\ub2e4. \ud0c4\ud654\uc218\uc18c\uac00 \uac00\ubb3c\ub9c9\uc774 \uc717\ubd80\ubd84\uc73c\ub85c \ub098\uc624\uc9c0 \uc54a\ub294 \uac83\uc774\uc5c8\ub2e4. \uc544\ubb34\ub3c4 \uc608\uc0c1\ud558\uc9c0 \ubabb\ud55c \uac83\uc774\uc5c8\ub2e4. \uac00\ubb3c\ub9c9\uc774\uac00 \ud558\uac15\ud558\ub294 \ub3d9\uc548 \uac00\uc2a4 \uae30\ub465\uc744 \uc774\ub3d9\uc2dc\ud0a4\uace0 \uc788\ub358 \uac83\uc774\uc5c8\ub2e4. \uac00\ubb3c\ub9c9\uc774\ub294 \ud574\uc218\uba74 \uadfc\ucc98\uc5d0\uc11c \ud0c4\ud654\uc218\uc18c\ub97c \ubaa8\uc558\uc73c\ub098 1,500\ubbf8\ud130 \uc544\ub798\ub85c \ub0b4\ub824\uac00\ub294\ub3d9\uc548 700\ubbf8\ud130 \uc9c0\uc810\ubd80\ud130 \uc5bc\uc74c\uc73c\ub85c \uac00\ub4dd \ucc28\uac8c \ub41c \uac83\uc774\uc5c8\ub2e4. \uad6c\uc870\ubb3c \ub0b4\ubd80\uc5d0\uc11c \uc774\ub8e8\uc5b4\uc9c4 \uc218\ud654\uc791\uc6a9\uc740 \uac00\uc2a4\uac00 \ube60\uc838\ub098\uac08 \uad6c\uba4d\uc744 \ub9c9\uc544\ubc84\ub838\ub2e4. \uacb0\uad6d \uc774 \uacc4\ud68d\uc740 \uc2e4\ud328\ub85c \ub3cc\uc544\uac14\ub2e4.", "paragraph_answer": "1979\ub144 \uba55\uc2dc\ucf54 \uc815\ubd80 \uc18c\uc720\uc758 \uc720\uc815\uc5d0\uc11c \ubc1c\uc0dd\ud55c \uc775\uc2a4\ud1a1 \uc720\ucd9c \uc0ac\uace0 \ub294 \uc5ed\uc0ac\uc0c1 \ucd5c\uc545\uc758 \uc0ac\uace0\ub85c \uae30\ub85d\ub418\uc5c8\ub2e4. \ub2f9\uc2dc \uc720\uc815 \uc804\ubb38\uac00 \ub808\ub4dc \uc544\ub4dc\ub808\ub294 \uc774 \uc0ac\uac74\uc5d0 \ud22c\uc785\ub418\uc5c8\uace0 \ud3ab \ucf10\ubca8\ub3c4 \ud568\uaed8 \uc77c\ud588\ub2e4. \uc774\ub4e4\uc740 \ubc18\uad6c\ub97c \ub9cc\ub4e4\uc5b4 \uc720\uc815\uc5d0 \uc50c\uc6b0\ub294 \uc791\uc5c5\uc73c\ub85c \uc0ac\uace0\ub97c \uc218\uc2b5\ud588\uc5c8\ub2e4. \uc775\uc2a4\ud1a1 \uc720\ucd9c\uc740 \ubc14\ub2e4 \ubc11 48\ubbf8\ud130\uc5d0 \ubc1c\uc0dd\ud55c \uac83\uc774\uc5c8\ub2e4. \uacfc\uc5f0 1,500\ubbf8\ud130 \uc544\ub798\uc5d0\uc11c\ub3c4 \uc774 \ubc18\uad6c\uac00 \ud6a8\uacfc\uc801\uc77c\uc9c0\ub294 \uc54c \uc218 \uc5c6\uc5c8\ub2e4. \ud3ab \ucf10\ubca8\uc740 \ud478\uc26c\uc639 \ud56d\uad6c\uc5d0\uc11c \ubc18\uad6c\ub97c \ub9cc\ub4e4\uc5c8\ub2e4. \uac00\ubb3c\ub9c9\uc774 \ub310\uc774\ub2e4. \ud574\uc800 \uc190\uc0c1\ub41c \uc720\uc815\uc5d0\uc11c \uc791\uc5c5\uc744 \ud574\uc57c \ud558\ub294 \ub2e4\uc774\ubc84\ub4e4\uc774 \uc274 \uc218 \uc788\ub294 \uc6a9\ub3c4\ub85c \ub9cc\ub4e4\uc5b4\uc84c\ub2e4. \ub192\uc774 4.5\ubbf8\ud130\uc5d0 \ud3ed\uc740 3\ubbf8\ud130\uc600\ub2e4. \uc2dc\ucd94 \ud30c\uc774\ud504 \ub05d\ubd80\ubd84\uc5d0 \uac00\ubb3c\ub9c9\uc774\ub97c \ub36e\uc744 \uacc4\ud68d\uc774\uc5c8\ub2e4. \uc11d\uc720 \ub300\ubd80\ubd84\uc774 \uac00\ubb3c\ub9c9\uc774 \uc548\uc5d0 \uac07\ud788\uac8c \ub418\uba74 \uad6c\uba4d\uc5d0 \uae54\ub54c\uae30\ub97c \uaf42\uc544 \ubc30\uc704\ub85c \uc11d\uc720\ub97c \ub04c\uc5b4 \uc62c\ub9ac\ub294 \uac83\uc774\uc5c8\ub2e4. \uac00\ubb3c\ub9c9\uc774\ub294 \ud574\uc800 \ubc14\ub2e5 \ud658\uacbd\uc5d0 \uc801\ud569\ud558\ub3c4\ub85d \uc0ac\ubc29 \uc5b4\ub290 \uacf3\uc774\ub4e0 \ud1b5\ub85c\ub97c \uc124\uce58 \ud560 \uc218 \uc788\ub3c4\ub85d \uc218\uc815\ud588\ub2e4. \uc774 \uac00\ubb3c\ub9c9\uc774\ub294 \uc720\uc815\uc5d0\uc11c \ub098\uc624\ub294 \uc11d\uc720\uc758 80%\uc815\ub3c4\ub294 \ubaa8\uc744 \uc218 \uc788\uc744\uac70\ub77c\ub294 \uae30\ub300\uac10\uc5d0 \ud729\uc2f8\uc600\ub2e4. 1,500\ubbf8\ud130 \uc544\ub798 \uc12d\uc528 148\ub3c4\uc5d0 \uc774\ub974\ub294 \ub728\uac70\uc6b4 \uc11d\uc720\uc640, \uba54\ud0c4 \uac00\uc2a4\ub97c \ubfdc\uc5b4\ub0b4\uace0 \uc788\uc5c8\ub2e4. \uc5c4\uccad\ub09c \uc555\ub825 \uc18d\uc5d0 \uac00\uc2a4\uc640 \ubd80\ub52a\ud788\ub294 \ubc14\ub2f7\ubb3c\uc740 \uc218\ud654\ub418\uc5b4 \uc5bc\uc74c\uacfc \ube44\uc2b7\ud55c \ubb3c\uc9c8\uc774 \ub418\uc5b4 \uac00\ubb3c\ub9c9\uc774\ub97c \uc190\uc0c1\uc2dc\ud0ac \uc218 \uc788\uc5c8\ub2e4. \uc774\ub97c \ubc29\uc9c0\ud558\uae30 \uc704\ud574 \ubc18\uad6c \uc548\uc5d0 \ubd84\uc0ac\ud3ec\ud2b8 \uc124\uce58\ub97c \uacc4\ud68d\ud588\ub2e4. \uc717\ubd80\ubd84\uc5d0 \ub179\uc0c9 \ud30c\uc774\ud504\uac00 \uc788\ub294\ub370 \uc774 \ubd84\uc0ac\ud3ec\ud2b8\uac00 \uba54\ud0c4\uc62c\uc758 \uc218\ud654\ub97c \ub9c9\ub294 \uc5ed\ud560\uc744 \ud558\ub294 \uac83\uc774\ub2e4. \uc774 \uc791\uc5c5\uae4c\uc9c0 18\uc77c\uc774 \uac78\ub838\ub2e4. 87\ud1a4\uc758 \uac00\ubb3c\ub9c9\uc774\uac00 \ubc14\ub2e5\uae4c\uc9c0 \uac00\ub77c\uc549\ub294\ub370 6\uc2dc\uac04\uc774 \uac78\ub838\ub2e4. \ubc14\ub2e5\uc5d0 \uac00\ub77c\uc549\uc790 \uc6d0\uc720\uac00 \uac00\ubb3c\ub9c9\uc774 \ubc14\ub2e5\uc5d0\uc11c \ubfdc\uc5b4\uc838 \ub098\uc624\ub294 \uac83\ucc98\ub7fc \ubcf4\uc600\ub2e4. \ud0c4\ud654\uc218\uc18c\uac00 \uac00\ubb3c\ub9c9\uc774 \uc717\ubd80\ubd84\uc73c\ub85c \ub098\uc624\uc9c0 \uc54a\ub294 \uac83\uc774\uc5c8\ub2e4. \uc544\ubb34\ub3c4 \uc608\uc0c1\ud558\uc9c0 \ubabb\ud55c \uac83\uc774\uc5c8\ub2e4. \uac00\ubb3c\ub9c9\uc774\uac00 \ud558\uac15\ud558\ub294 \ub3d9\uc548 \uac00\uc2a4 \uae30\ub465\uc744 \uc774\ub3d9\uc2dc\ud0a4\uace0 \uc788\ub358 \uac83\uc774\uc5c8\ub2e4. \uac00\ubb3c\ub9c9\uc774\ub294 \ud574\uc218\uba74 \uadfc\ucc98\uc5d0\uc11c \ud0c4\ud654\uc218\uc18c\ub97c \ubaa8\uc558\uc73c\ub098 1,500\ubbf8\ud130 \uc544\ub798\ub85c \ub0b4\ub824\uac00\ub294\ub3d9\uc548 700\ubbf8\ud130 \uc9c0\uc810\ubd80\ud130 \uc5bc\uc74c\uc73c\ub85c \uac00\ub4dd \ucc28\uac8c \ub41c \uac83\uc774\uc5c8\ub2e4. \uad6c\uc870\ubb3c \ub0b4\ubd80\uc5d0\uc11c \uc774\ub8e8\uc5b4\uc9c4 \uc218\ud654\uc791\uc6a9\uc740 \uac00\uc2a4\uac00 \ube60\uc838\ub098\uac08 \uad6c\uba4d\uc744 \ub9c9\uc544\ubc84\ub838\ub2e4. \uacb0\uad6d \uc774 \uacc4\ud68d\uc740 \uc2e4\ud328\ub85c \ub3cc\uc544\uac14\ub2e4.", "sentence_answer": "1979\ub144 \uba55\uc2dc\ucf54 \uc815\ubd80 \uc18c\uc720\uc758 \uc720\uc815\uc5d0\uc11c \ubc1c\uc0dd\ud55c \uc775\uc2a4\ud1a1 \uc720\ucd9c \uc0ac\uace0 \ub294 \uc5ed\uc0ac\uc0c1 \ucd5c\uc545\uc758 \uc0ac\uace0\ub85c \uae30\ub85d\ub418\uc5c8\ub2e4.", "paragraph_id": "6513252-5-0", "paragraph_question": "question: 1979\ub144 \uba55\uc2dc\ucf54 \uc720\uc815\uc5d0\uc11c \ubc1c\uc0dd\ud55c \uc0ac\uace0\uc758 \uc774\ub984\uc740?, context: 1979\ub144 \uba55\uc2dc\ucf54 \uc815\ubd80 \uc18c\uc720\uc758 \uc720\uc815\uc5d0\uc11c \ubc1c\uc0dd\ud55c \uc775\uc2a4\ud1a1 \uc720\ucd9c \uc0ac\uace0\ub294 \uc5ed\uc0ac\uc0c1 \ucd5c\uc545\uc758 \uc0ac\uace0\ub85c \uae30\ub85d\ub418\uc5c8\ub2e4. \ub2f9\uc2dc \uc720\uc815 \uc804\ubb38\uac00 \ub808\ub4dc \uc544\ub4dc\ub808\ub294 \uc774 \uc0ac\uac74\uc5d0 \ud22c\uc785\ub418\uc5c8\uace0 \ud3ab \ucf10\ubca8\ub3c4 \ud568\uaed8 \uc77c\ud588\ub2e4. \uc774\ub4e4\uc740 \ubc18\uad6c\ub97c \ub9cc\ub4e4\uc5b4 \uc720\uc815\uc5d0 \uc50c\uc6b0\ub294 \uc791\uc5c5\uc73c\ub85c \uc0ac\uace0\ub97c \uc218\uc2b5\ud588\uc5c8\ub2e4. \uc775\uc2a4\ud1a1 \uc720\ucd9c\uc740 \ubc14\ub2e4 \ubc11 48\ubbf8\ud130\uc5d0 \ubc1c\uc0dd\ud55c \uac83\uc774\uc5c8\ub2e4. \uacfc\uc5f0 1,500\ubbf8\ud130 \uc544\ub798\uc5d0\uc11c\ub3c4 \uc774 \ubc18\uad6c\uac00 \ud6a8\uacfc\uc801\uc77c\uc9c0\ub294 \uc54c \uc218 \uc5c6\uc5c8\ub2e4. \ud3ab \ucf10\ubca8\uc740 \ud478\uc26c\uc639 \ud56d\uad6c\uc5d0\uc11c \ubc18\uad6c\ub97c \ub9cc\ub4e4\uc5c8\ub2e4. \uac00\ubb3c\ub9c9\uc774 \ub310\uc774\ub2e4. \ud574\uc800 \uc190\uc0c1\ub41c \uc720\uc815\uc5d0\uc11c \uc791\uc5c5\uc744 \ud574\uc57c \ud558\ub294 \ub2e4\uc774\ubc84\ub4e4\uc774 \uc274 \uc218 \uc788\ub294 \uc6a9\ub3c4\ub85c \ub9cc\ub4e4\uc5b4\uc84c\ub2e4. \ub192\uc774 4.5\ubbf8\ud130\uc5d0 \ud3ed\uc740 3\ubbf8\ud130\uc600\ub2e4. \uc2dc\ucd94 \ud30c\uc774\ud504 \ub05d\ubd80\ubd84\uc5d0 \uac00\ubb3c\ub9c9\uc774\ub97c \ub36e\uc744 \uacc4\ud68d\uc774\uc5c8\ub2e4. \uc11d\uc720 \ub300\ubd80\ubd84\uc774 \uac00\ubb3c\ub9c9\uc774 \uc548\uc5d0 \uac07\ud788\uac8c \ub418\uba74 \uad6c\uba4d\uc5d0 \uae54\ub54c\uae30\ub97c \uaf42\uc544 \ubc30\uc704\ub85c \uc11d\uc720\ub97c \ub04c\uc5b4 \uc62c\ub9ac\ub294 \uac83\uc774\uc5c8\ub2e4. \uac00\ubb3c\ub9c9\uc774\ub294 \ud574\uc800 \ubc14\ub2e5 \ud658\uacbd\uc5d0 \uc801\ud569\ud558\ub3c4\ub85d \uc0ac\ubc29 \uc5b4\ub290 \uacf3\uc774\ub4e0 \ud1b5\ub85c\ub97c \uc124\uce58 \ud560 \uc218 \uc788\ub3c4\ub85d \uc218\uc815\ud588\ub2e4. \uc774 \uac00\ubb3c\ub9c9\uc774\ub294 \uc720\uc815\uc5d0\uc11c \ub098\uc624\ub294 \uc11d\uc720\uc758 80%\uc815\ub3c4\ub294 \ubaa8\uc744 \uc218 \uc788\uc744\uac70\ub77c\ub294 \uae30\ub300\uac10\uc5d0 \ud729\uc2f8\uc600\ub2e4. 1,500\ubbf8\ud130 \uc544\ub798 \uc12d\uc528 148\ub3c4\uc5d0 \uc774\ub974\ub294 \ub728\uac70\uc6b4 \uc11d\uc720\uc640, \uba54\ud0c4 \uac00\uc2a4\ub97c \ubfdc\uc5b4\ub0b4\uace0 \uc788\uc5c8\ub2e4. \uc5c4\uccad\ub09c \uc555\ub825 \uc18d\uc5d0 \uac00\uc2a4\uc640 \ubd80\ub52a\ud788\ub294 \ubc14\ub2f7\ubb3c\uc740 \uc218\ud654\ub418\uc5b4 \uc5bc\uc74c\uacfc \ube44\uc2b7\ud55c \ubb3c\uc9c8\uc774 \ub418\uc5b4 \uac00\ubb3c\ub9c9\uc774\ub97c \uc190\uc0c1\uc2dc\ud0ac \uc218 \uc788\uc5c8\ub2e4. \uc774\ub97c \ubc29\uc9c0\ud558\uae30 \uc704\ud574 \ubc18\uad6c \uc548\uc5d0 \ubd84\uc0ac\ud3ec\ud2b8 \uc124\uce58\ub97c \uacc4\ud68d\ud588\ub2e4. \uc717\ubd80\ubd84\uc5d0 \ub179\uc0c9 \ud30c\uc774\ud504\uac00 \uc788\ub294\ub370 \uc774 \ubd84\uc0ac\ud3ec\ud2b8\uac00 \uba54\ud0c4\uc62c\uc758 \uc218\ud654\ub97c \ub9c9\ub294 \uc5ed\ud560\uc744 \ud558\ub294 \uac83\uc774\ub2e4. \uc774 \uc791\uc5c5\uae4c\uc9c0 18\uc77c\uc774 \uac78\ub838\ub2e4. 87\ud1a4\uc758 \uac00\ubb3c\ub9c9\uc774\uac00 \ubc14\ub2e5\uae4c\uc9c0 \uac00\ub77c\uc549\ub294\ub370 6\uc2dc\uac04\uc774 \uac78\ub838\ub2e4. \ubc14\ub2e5\uc5d0 \uac00\ub77c\uc549\uc790 \uc6d0\uc720\uac00 \uac00\ubb3c\ub9c9\uc774 \ubc14\ub2e5\uc5d0\uc11c \ubfdc\uc5b4\uc838 \ub098\uc624\ub294 \uac83\ucc98\ub7fc \ubcf4\uc600\ub2e4. \ud0c4\ud654\uc218\uc18c\uac00 \uac00\ubb3c\ub9c9\uc774 \uc717\ubd80\ubd84\uc73c\ub85c \ub098\uc624\uc9c0 \uc54a\ub294 \uac83\uc774\uc5c8\ub2e4. \uc544\ubb34\ub3c4 \uc608\uc0c1\ud558\uc9c0 \ubabb\ud55c \uac83\uc774\uc5c8\ub2e4. \uac00\ubb3c\ub9c9\uc774\uac00 \ud558\uac15\ud558\ub294 \ub3d9\uc548 \uac00\uc2a4 \uae30\ub465\uc744 \uc774\ub3d9\uc2dc\ud0a4\uace0 \uc788\ub358 \uac83\uc774\uc5c8\ub2e4. \uac00\ubb3c\ub9c9\uc774\ub294 \ud574\uc218\uba74 \uadfc\ucc98\uc5d0\uc11c \ud0c4\ud654\uc218\uc18c\ub97c \ubaa8\uc558\uc73c\ub098 1,500\ubbf8\ud130 \uc544\ub798\ub85c \ub0b4\ub824\uac00\ub294\ub3d9\uc548 700\ubbf8\ud130 \uc9c0\uc810\ubd80\ud130 \uc5bc\uc74c\uc73c\ub85c \uac00\ub4dd \ucc28\uac8c \ub41c \uac83\uc774\uc5c8\ub2e4. \uad6c\uc870\ubb3c \ub0b4\ubd80\uc5d0\uc11c \uc774\ub8e8\uc5b4\uc9c4 \uc218\ud654\uc791\uc6a9\uc740 \uac00\uc2a4\uac00 \ube60\uc838\ub098\uac08 \uad6c\uba4d\uc744 \ub9c9\uc544\ubc84\ub838\ub2e4. \uacb0\uad6d \uc774 \uacc4\ud68d\uc740 \uc2e4\ud328\ub85c \ub3cc\uc544\uac14\ub2e4."} {"question": "\ubbf8\uc2a4 \uc720\ub2c8\ubc84\uc2a4\ub97c \ucc98\uc74c\uc73c\ub85c \uac1c\ucd5c\ud55c \uacf3\uc740?", "paragraph": "\ub3c4\ub110\ub4dc \ud2b8\ub7fc\ud504\ub294 1996\ub144 \ubbf8\uc2a4 \uc720\ub2c8\ubc84\uc2a4 \uc870\uc9c1\ud68c\ub97c \uc778\uc218\ud574, \ud574\ub9c8\ub2e4 \ubbf8\uc2a4 \uc720\ub2c8\ubc84\uc2a4, \ubbf8\uc2a4 USA, \ubbf8\uc2a4 \ud2f4 USA\ub97c 2015\ub144\uae4c\uc9c0 \uc5f4\uc5c8\ub2e4. \ubbf8\uc2a4 \uc720\ub2c8\ubc84\uc2a4\ub294 \uc138\uacc4\uc5d0\uc11c \uac00\uc7a5 \uc720\uba85\ud55c \ubbf8\uc778 \ub300\ud68c\ub85c 1952\ub144 \uce98\ub9ac\ud3ec\ub2c8\uc544 \uc8fc\uc5d0 \uc704\uce58\ud55c \ud37c\uc2dc\ud53d \ubc00\uc2a4\ub77c\ub294 \ud68c\uc0ac\uac00 \ucc98\uc74c\uc73c\ub85c \uc5f4\uc5c8\ub2e4. \uadf8\ub294 \ubc29\uc1a1 \uc2a4\ucf00\uc974\uc5d0 \ub300\ud55c \ubd88\ub9cc\uc73c\ub85c \uae30\uc874\uc5d0 \ubbf8\uad6d\uc758 \ubc29\uc1a1\uc0ac\uc778 CBS\uc5d0\uc11c \uc5f4\ub9ac\ub358 \ubbf8\uc2a4 \uc720\ub2c8\ubc84\uc2a4, \ubbf8\uc2a4 USA, \ubbf8\uc2a4 \ud2f4 USA\ub97c 2002\ub144\uc5d0 NBC\ub85c \uc62e\uacbc\ub2e4. \uadf8\ub7ec\ub098 \ud2b8\ub7fc\ud504\uac00 2015\ub144 6\uc6d4 \uba55\uc2dc\ucf54 \uc774\ubbfc\uc790\ub4e4\uc5d0 \ub300\ud574 \ub9c9\ub9d0\uc744 \ud574 \ud30c\ubb38\uc744 \uc77c\uc73c\ud0a4\uba74\uc11c, \uacf5\ub3d9 \uc8fc\ucd5c\ud55c NBC \ubc29\uc1a1\uacfc \uc720\ub2c8\ubc84\uc124\uc740 \ud2b8\ub7fc\ud504\uc640 \ud568\uaed8 \uc0ac\uc5c5\ud558\uc9c0 \uc54a\uaca0\ub2e4 \uc120\uc5b8\ud588\ub2e4. \ud2b8\ub7fc\ud504\ub294 \ubc29\uc1a1\uc0ac \uc720\ub2c8\ube44\uc804\uc774 \uacc4\uc57d\uc744 \uc704\ubc18\ud588\ub2e4\uba70 5\uc5b5 \ub2ec\ub7ec\uc758 \uc190\ud574\ubc30\uc0c1 \uccad\uad6c\uc18c\uc1a1\uc744 \uc81c\uae30\ud588\ub2e4. 9\uc6d4, \uadf8\ub294 \ubbf8\uc2a4\uc720\ub2c8\ubc84\uc2a4 \uc6b4\uc601\uad8c\uc744 WME-IMG\uc5d0 \ub9e4\uac01\ud588\ub2e4.", "answer": "\ud37c\uc2dc\ud53d \ubc00\uc2a4", "sentence": "\ubbf8\uc2a4 \uc720\ub2c8\ubc84\uc2a4\ub294 \uc138\uacc4\uc5d0\uc11c \uac00\uc7a5 \uc720\uba85\ud55c \ubbf8\uc778 \ub300\ud68c\ub85c 1952\ub144 \uce98\ub9ac\ud3ec\ub2c8\uc544 \uc8fc\uc5d0 \uc704\uce58\ud55c \ud37c\uc2dc\ud53d \ubc00\uc2a4 \ub77c\ub294 \ud68c\uc0ac\uac00 \ucc98\uc74c\uc73c\ub85c \uc5f4\uc5c8\ub2e4.", "paragraph_sentence": "\ub3c4\ub110\ub4dc \ud2b8\ub7fc\ud504\ub294 1996\ub144 \ubbf8\uc2a4 \uc720\ub2c8\ubc84\uc2a4 \uc870\uc9c1\ud68c\ub97c \uc778\uc218\ud574, \ud574\ub9c8\ub2e4 \ubbf8\uc2a4 \uc720\ub2c8\ubc84\uc2a4, \ubbf8\uc2a4 USA, \ubbf8\uc2a4 \ud2f4 USA\ub97c 2015\ub144\uae4c\uc9c0 \uc5f4\uc5c8\ub2e4. \ubbf8\uc2a4 \uc720\ub2c8\ubc84\uc2a4\ub294 \uc138\uacc4\uc5d0\uc11c \uac00\uc7a5 \uc720\uba85\ud55c \ubbf8\uc778 \ub300\ud68c\ub85c 1952\ub144 \uce98\ub9ac\ud3ec\ub2c8\uc544 \uc8fc\uc5d0 \uc704\uce58\ud55c \ud37c\uc2dc\ud53d \ubc00\uc2a4 \ub77c\ub294 \ud68c\uc0ac\uac00 \ucc98\uc74c\uc73c\ub85c \uc5f4\uc5c8\ub2e4. \uadf8\ub294 \ubc29\uc1a1 \uc2a4\ucf00\uc974\uc5d0 \ub300\ud55c \ubd88\ub9cc\uc73c\ub85c \uae30\uc874\uc5d0 \ubbf8\uad6d\uc758 \ubc29\uc1a1\uc0ac\uc778 CBS\uc5d0\uc11c \uc5f4\ub9ac\ub358 \ubbf8\uc2a4 \uc720\ub2c8\ubc84\uc2a4, \ubbf8\uc2a4 USA, \ubbf8\uc2a4 \ud2f4 USA\ub97c 2002\ub144\uc5d0 NBC\ub85c \uc62e\uacbc\ub2e4. \uadf8\ub7ec\ub098 \ud2b8\ub7fc\ud504\uac00 2015\ub144 6\uc6d4 \uba55\uc2dc\ucf54 \uc774\ubbfc\uc790\ub4e4\uc5d0 \ub300\ud574 \ub9c9\ub9d0\uc744 \ud574 \ud30c\ubb38\uc744 \uc77c\uc73c\ud0a4\uba74\uc11c, \uacf5\ub3d9 \uc8fc\ucd5c\ud55c NBC \ubc29\uc1a1\uacfc \uc720\ub2c8\ubc84\uc124\uc740 \ud2b8\ub7fc\ud504\uc640 \ud568\uaed8 \uc0ac\uc5c5\ud558\uc9c0 \uc54a\uaca0\ub2e4 \uc120\uc5b8\ud588\ub2e4. \ud2b8\ub7fc\ud504\ub294 \ubc29\uc1a1\uc0ac \uc720\ub2c8\ube44\uc804\uc774 \uacc4\uc57d\uc744 \uc704\ubc18\ud588\ub2e4\uba70 5\uc5b5 \ub2ec\ub7ec\uc758 \uc190\ud574\ubc30\uc0c1 \uccad\uad6c\uc18c\uc1a1\uc744 \uc81c\uae30\ud588\ub2e4. 9\uc6d4, \uadf8\ub294 \ubbf8\uc2a4\uc720\ub2c8\ubc84\uc2a4 \uc6b4\uc601\uad8c\uc744 WME-IMG\uc5d0 \ub9e4\uac01\ud588\ub2e4.", "paragraph_answer": "\ub3c4\ub110\ub4dc \ud2b8\ub7fc\ud504\ub294 1996\ub144 \ubbf8\uc2a4 \uc720\ub2c8\ubc84\uc2a4 \uc870\uc9c1\ud68c\ub97c \uc778\uc218\ud574, \ud574\ub9c8\ub2e4 \ubbf8\uc2a4 \uc720\ub2c8\ubc84\uc2a4, \ubbf8\uc2a4 USA, \ubbf8\uc2a4 \ud2f4 USA\ub97c 2015\ub144\uae4c\uc9c0 \uc5f4\uc5c8\ub2e4. \ubbf8\uc2a4 \uc720\ub2c8\ubc84\uc2a4\ub294 \uc138\uacc4\uc5d0\uc11c \uac00\uc7a5 \uc720\uba85\ud55c \ubbf8\uc778 \ub300\ud68c\ub85c 1952\ub144 \uce98\ub9ac\ud3ec\ub2c8\uc544 \uc8fc\uc5d0 \uc704\uce58\ud55c \ud37c\uc2dc\ud53d \ubc00\uc2a4 \ub77c\ub294 \ud68c\uc0ac\uac00 \ucc98\uc74c\uc73c\ub85c \uc5f4\uc5c8\ub2e4. \uadf8\ub294 \ubc29\uc1a1 \uc2a4\ucf00\uc974\uc5d0 \ub300\ud55c \ubd88\ub9cc\uc73c\ub85c \uae30\uc874\uc5d0 \ubbf8\uad6d\uc758 \ubc29\uc1a1\uc0ac\uc778 CBS\uc5d0\uc11c \uc5f4\ub9ac\ub358 \ubbf8\uc2a4 \uc720\ub2c8\ubc84\uc2a4, \ubbf8\uc2a4 USA, \ubbf8\uc2a4 \ud2f4 USA\ub97c 2002\ub144\uc5d0 NBC\ub85c \uc62e\uacbc\ub2e4. \uadf8\ub7ec\ub098 \ud2b8\ub7fc\ud504\uac00 2015\ub144 6\uc6d4 \uba55\uc2dc\ucf54 \uc774\ubbfc\uc790\ub4e4\uc5d0 \ub300\ud574 \ub9c9\ub9d0\uc744 \ud574 \ud30c\ubb38\uc744 \uc77c\uc73c\ud0a4\uba74\uc11c, \uacf5\ub3d9 \uc8fc\ucd5c\ud55c NBC \ubc29\uc1a1\uacfc \uc720\ub2c8\ubc84\uc124\uc740 \ud2b8\ub7fc\ud504\uc640 \ud568\uaed8 \uc0ac\uc5c5\ud558\uc9c0 \uc54a\uaca0\ub2e4 \uc120\uc5b8\ud588\ub2e4. \ud2b8\ub7fc\ud504\ub294 \ubc29\uc1a1\uc0ac \uc720\ub2c8\ube44\uc804\uc774 \uacc4\uc57d\uc744 \uc704\ubc18\ud588\ub2e4\uba70 5\uc5b5 \ub2ec\ub7ec\uc758 \uc190\ud574\ubc30\uc0c1 \uccad\uad6c\uc18c\uc1a1\uc744 \uc81c\uae30\ud588\ub2e4. 9\uc6d4, \uadf8\ub294 \ubbf8\uc2a4\uc720\ub2c8\ubc84\uc2a4 \uc6b4\uc601\uad8c\uc744 WME-IMG\uc5d0 \ub9e4\uac01\ud588\ub2e4.", "sentence_answer": "\ubbf8\uc2a4 \uc720\ub2c8\ubc84\uc2a4\ub294 \uc138\uacc4\uc5d0\uc11c \uac00\uc7a5 \uc720\uba85\ud55c \ubbf8\uc778 \ub300\ud68c\ub85c 1952\ub144 \uce98\ub9ac\ud3ec\ub2c8\uc544 \uc8fc\uc5d0 \uc704\uce58\ud55c \ud37c\uc2dc\ud53d \ubc00\uc2a4 \ub77c\ub294 \ud68c\uc0ac\uac00 \ucc98\uc74c\uc73c\ub85c \uc5f4\uc5c8\ub2e4.", "paragraph_id": "6491753-1-1", "paragraph_question": "question: \ubbf8\uc2a4 \uc720\ub2c8\ubc84\uc2a4\ub97c \ucc98\uc74c\uc73c\ub85c \uac1c\ucd5c\ud55c \uacf3\uc740?, context: \ub3c4\ub110\ub4dc \ud2b8\ub7fc\ud504\ub294 1996\ub144 \ubbf8\uc2a4 \uc720\ub2c8\ubc84\uc2a4 \uc870\uc9c1\ud68c\ub97c \uc778\uc218\ud574, \ud574\ub9c8\ub2e4 \ubbf8\uc2a4 \uc720\ub2c8\ubc84\uc2a4, \ubbf8\uc2a4 USA, \ubbf8\uc2a4 \ud2f4 USA\ub97c 2015\ub144\uae4c\uc9c0 \uc5f4\uc5c8\ub2e4. \ubbf8\uc2a4 \uc720\ub2c8\ubc84\uc2a4\ub294 \uc138\uacc4\uc5d0\uc11c \uac00\uc7a5 \uc720\uba85\ud55c \ubbf8\uc778 \ub300\ud68c\ub85c 1952\ub144 \uce98\ub9ac\ud3ec\ub2c8\uc544 \uc8fc\uc5d0 \uc704\uce58\ud55c \ud37c\uc2dc\ud53d \ubc00\uc2a4\ub77c\ub294 \ud68c\uc0ac\uac00 \ucc98\uc74c\uc73c\ub85c \uc5f4\uc5c8\ub2e4. \uadf8\ub294 \ubc29\uc1a1 \uc2a4\ucf00\uc974\uc5d0 \ub300\ud55c \ubd88\ub9cc\uc73c\ub85c \uae30\uc874\uc5d0 \ubbf8\uad6d\uc758 \ubc29\uc1a1\uc0ac\uc778 CBS\uc5d0\uc11c \uc5f4\ub9ac\ub358 \ubbf8\uc2a4 \uc720\ub2c8\ubc84\uc2a4, \ubbf8\uc2a4 USA, \ubbf8\uc2a4 \ud2f4 USA\ub97c 2002\ub144\uc5d0 NBC\ub85c \uc62e\uacbc\ub2e4. \uadf8\ub7ec\ub098 \ud2b8\ub7fc\ud504\uac00 2015\ub144 6\uc6d4 \uba55\uc2dc\ucf54 \uc774\ubbfc\uc790\ub4e4\uc5d0 \ub300\ud574 \ub9c9\ub9d0\uc744 \ud574 \ud30c\ubb38\uc744 \uc77c\uc73c\ud0a4\uba74\uc11c, \uacf5\ub3d9 \uc8fc\ucd5c\ud55c NBC \ubc29\uc1a1\uacfc \uc720\ub2c8\ubc84\uc124\uc740 \ud2b8\ub7fc\ud504\uc640 \ud568\uaed8 \uc0ac\uc5c5\ud558\uc9c0 \uc54a\uaca0\ub2e4 \uc120\uc5b8\ud588\ub2e4. \ud2b8\ub7fc\ud504\ub294 \ubc29\uc1a1\uc0ac \uc720\ub2c8\ube44\uc804\uc774 \uacc4\uc57d\uc744 \uc704\ubc18\ud588\ub2e4\uba70 5\uc5b5 \ub2ec\ub7ec\uc758 \uc190\ud574\ubc30\uc0c1 \uccad\uad6c\uc18c\uc1a1\uc744 \uc81c\uae30\ud588\ub2e4. 9\uc6d4, \uadf8\ub294 \ubbf8\uc2a4\uc720\ub2c8\ubc84\uc2a4 \uc6b4\uc601\uad8c\uc744 WME-IMG\uc5d0 \ub9e4\uac01\ud588\ub2e4."} {"question": "\ubb34\uc5c7\uc744 \ub05d\uc5c6\uc774 \ud655\uc7a5\ud558\uace0 \uacfc\uc7a5\ud558\ub2e4 \ubcf4\ub2c8 \uc2e4\uc81c \uc2e4\ud589\ub418\uc5c8\ub358 \uc800\ud56d\uc6b4\ub3d9\ub4e4\uc774 \uc8fc\ubaa9\ubc1b\uc9c0 \ubabb\ud558\ub294 \uacbd\uc6b0\uac00 \ubc1c\uc0dd\ud558\ub294\uac00?", "paragraph": "\ub9c8\ucc2c\uac00\uc9c0\ub85c \ud754\uce58 \uc54a\uac70\ub098 \uc18c\uaddc\ubaa8\uc758 \uac1c\ubcc4\uc801\uc778 \uc0ac\ub840\ub4e4\uc744 \ub2f9\uc2dc\uc758 \uc2dc\ub300\uc0c1\uc778 \uc591 \ud3ec\uc7a5\ud558\uba74 \uc2e4\uc81c\ub85c \ubc8c\uc5b4\uc84c\ub358 \uc0ac\uac74\ub4e4\uc758 \uc758\uc758\uc640 \uae30\ubcf8\uc131\uaca9\uae4c\uc9c0\ub3c4 \uc65c\uace1\uc2dc\ucf1c\ubc84\ub9b4 \uc218\uac00 \uc788\ub2e4. \uc2e4\uc81c\ub85c \uc800\ud56d\uc758 \uc815\uc758\ub97c \ub05d\uc5c6\uc774 \ud655\uc7a5\ud558\uace0 \uc0ac\ub840\ub4e4\uc740 \uacfc\uc7a5\ud558\ub2e4\ubcf4\ub2c8, \uc624\ud788\ub824 \uc2e4\uc81c\ub85c \uc2e4\ud589\ub418\uc5c8\ub358 \uc800\ud56d\ub2e4\uc6b4 \uc800\ud56d\uc6b4\ub3d9\ub4e4\uc740 \uc81c\ub300\ub85c \uc8fc\ubaa9\ubc1b\uc9c0 \ubabb\ud558\ub294 \uacbd\uc6b0\uac00 \ubc1c\uc0dd\ud558\uace0 \uc788\ub2e4. \uc774\ub7ec\ud55c \ud750\ub984\ub300\ub85c \uac04\ub2e4\uba74 \uc601\uc6c5\uc801 \uc815\uc2e0\uc740 \uc720\ub7fd\uc758 \ubaa8\ub4e0 \uc720\ub300\uc778\ub4e4\uacfc \uadf8 \uacf5\ub3d9\uccb4\ub4e4\uc758 \uae30\ubcf8 \uc18c\uc591\uc73c\ub85c \uce58\ubd80\ub418\uc5b4\ubc84\ub9ac\uace0, \uc815\uc791 \uc800\ud56d\uc744 \ud589\ub3d9\uc5d0 \uc62e\uacbc\ub358 \uc18c\uc218 \uc720\ub300\uc778\ub4e4\uc758 \uc9c4\uc815\ud55c \uc6a9\uae30\ub294 \ud1f4\uc0c9\uc2dc\ud0a4\ub294 \uacb0\uacfc\ub97c \ub0b3\uac8c \ub420 \uac83\uc774\ub2e4. \uc5ec\uae30\uc11c \uc6b0\ub9ac\ub294 \uc720\ub300\uc0ac\ud68c\uc758 \uc774\ub7ec\ud55c \u2018\uc21c\uc751\ud558\ub294 \ub300\ub2e4\uc218\uc640 \ud589\ub3d9\ud558\ub294 \uadf9\uc18c\uc218\uc758 \ud63c\uc7ac\uc640 \ubd88\ud1b5\u2019\uc744 \ub2e8\uc21c\ud788 \ubb38\uc81c\ud574\uacb0 \uacfc\uc815\uc744 \ud750\ub9ac\uba4d\ud145\ud558\uac8c \ub9cc\ub4dc\ub294 \ub9f9\ud0d5 \uacf5\ub3d9\uccb4\ub85c \uce58\ubd80\ud558\ub294\ub370 \uadf8\uccd0\uc11c\ub294 \uc548\ub41c\ub2e4. \uc774 \uad6c\uc870\ub294 \uc0ac\uc2e4\uc0c1 \uc790\uc2e0\ub4e4\uc774 \uc18d\ud55c \uacf5\ub3d9\uccb4\uc5d0 \ub300\ud55c \uc0dd\uc0b0\uc801\uc778 \uc9c8\ubb38\uacfc \ub300\uc548\uc744 \uc8fc\uace0\ubc1b\ub294 \uac83\uc774 \ubd88\uac00\ub2a5\ud558\uba70, \uc774\ub294 \uace7 \uacf5\ub3d9\uccb4 \uc2a4\uc2a4\ub85c \ucd94\ub860\ub2a5\ub825\uacfc \uc0dd\uc874\uc804\ub7b5\uc744 \ud3ec\uae30\ud558\ub294 \uac83\uacfc \ub9c8\ucc2c\uac00\uc9c0\uc774\ub2e4. \uc774\ub7ec\ud55c \ubb38\uc81c\ub97c \uc678\uba74\ud55c\ub2e4\uba74, \uc720\ub300\uc778\uc758 \uc5ed\uc0ac\ub294 \ub354 \uc774\uc0c1 \uc9c0\uc18d\ub420 \uc218 \uc5c6\uc744 \uac83\uc774\ub2e4.\u201d", "answer": "\uc800\ud56d\uc758 \uc815\uc758", "sentence": "\uc2e4\uc81c\ub85c \uc800\ud56d\uc758 \uc815\uc758 \ub97c \ub05d\uc5c6\uc774 \ud655\uc7a5\ud558\uace0 \uc0ac\ub840\ub4e4\uc740 \uacfc\uc7a5\ud558\ub2e4\ubcf4\ub2c8, \uc624\ud788\ub824 \uc2e4\uc81c\ub85c \uc2e4\ud589\ub418\uc5c8\ub358 \uc800\ud56d\ub2e4\uc6b4 \uc800\ud56d\uc6b4\ub3d9\ub4e4\uc740 \uc81c\ub300\ub85c \uc8fc\ubaa9\ubc1b\uc9c0 \ubabb\ud558\ub294 \uacbd\uc6b0\uac00 \ubc1c\uc0dd\ud558\uace0 \uc788\ub2e4.", "paragraph_sentence": "\ub9c8\ucc2c\uac00\uc9c0\ub85c \ud754\uce58 \uc54a\uac70\ub098 \uc18c\uaddc\ubaa8\uc758 \uac1c\ubcc4\uc801\uc778 \uc0ac\ub840\ub4e4\uc744 \ub2f9\uc2dc\uc758 \uc2dc\ub300\uc0c1\uc778 \uc591 \ud3ec\uc7a5\ud558\uba74 \uc2e4\uc81c\ub85c \ubc8c\uc5b4\uc84c\ub358 \uc0ac\uac74\ub4e4\uc758 \uc758\uc758\uc640 \uae30\ubcf8\uc131\uaca9\uae4c\uc9c0\ub3c4 \uc65c\uace1\uc2dc\ucf1c\ubc84\ub9b4 \uc218\uac00 \uc788\ub2e4. \uc2e4\uc81c\ub85c \uc800\ud56d\uc758 \uc815\uc758 \ub97c \ub05d\uc5c6\uc774 \ud655\uc7a5\ud558\uace0 \uc0ac\ub840\ub4e4\uc740 \uacfc\uc7a5\ud558\ub2e4\ubcf4\ub2c8, \uc624\ud788\ub824 \uc2e4\uc81c\ub85c \uc2e4\ud589\ub418\uc5c8\ub358 \uc800\ud56d\ub2e4\uc6b4 \uc800\ud56d\uc6b4\ub3d9\ub4e4\uc740 \uc81c\ub300\ub85c \uc8fc\ubaa9\ubc1b\uc9c0 \ubabb\ud558\ub294 \uacbd\uc6b0\uac00 \ubc1c\uc0dd\ud558\uace0 \uc788\ub2e4. \uc774\ub7ec\ud55c \ud750\ub984\ub300\ub85c \uac04\ub2e4\uba74 \uc601\uc6c5\uc801 \uc815\uc2e0\uc740 \uc720\ub7fd\uc758 \ubaa8\ub4e0 \uc720\ub300\uc778\ub4e4\uacfc \uadf8 \uacf5\ub3d9\uccb4\ub4e4\uc758 \uae30\ubcf8 \uc18c\uc591\uc73c\ub85c \uce58\ubd80\ub418\uc5b4\ubc84\ub9ac\uace0, \uc815\uc791 \uc800\ud56d\uc744 \ud589\ub3d9\uc5d0 \uc62e\uacbc\ub358 \uc18c\uc218 \uc720\ub300\uc778\ub4e4\uc758 \uc9c4\uc815\ud55c \uc6a9\uae30\ub294 \ud1f4\uc0c9\uc2dc\ud0a4\ub294 \uacb0\uacfc\ub97c \ub0b3\uac8c \ub420 \uac83\uc774\ub2e4. \uc5ec\uae30\uc11c \uc6b0\ub9ac\ub294 \uc720\ub300\uc0ac\ud68c\uc758 \uc774\ub7ec\ud55c \u2018\uc21c\uc751\ud558\ub294 \ub300\ub2e4\uc218\uc640 \ud589\ub3d9\ud558\ub294 \uadf9\uc18c\uc218\uc758 \ud63c\uc7ac\uc640 \ubd88\ud1b5\u2019\uc744 \ub2e8\uc21c\ud788 \ubb38\uc81c\ud574\uacb0 \uacfc\uc815\uc744 \ud750\ub9ac\uba4d\ud145\ud558\uac8c \ub9cc\ub4dc\ub294 \ub9f9\ud0d5 \uacf5\ub3d9\uccb4\ub85c \uce58\ubd80\ud558\ub294\ub370 \uadf8\uccd0\uc11c\ub294 \uc548\ub41c\ub2e4. \uc774 \uad6c\uc870\ub294 \uc0ac\uc2e4\uc0c1 \uc790\uc2e0\ub4e4\uc774 \uc18d\ud55c \uacf5\ub3d9\uccb4\uc5d0 \ub300\ud55c \uc0dd\uc0b0\uc801\uc778 \uc9c8\ubb38\uacfc \ub300\uc548\uc744 \uc8fc\uace0\ubc1b\ub294 \uac83\uc774 \ubd88\uac00\ub2a5\ud558\uba70, \uc774\ub294 \uace7 \uacf5\ub3d9\uccb4 \uc2a4\uc2a4\ub85c \ucd94\ub860\ub2a5\ub825\uacfc \uc0dd\uc874\uc804\ub7b5\uc744 \ud3ec\uae30\ud558\ub294 \uac83\uacfc \ub9c8\ucc2c\uac00\uc9c0\uc774\ub2e4. \uc774\ub7ec\ud55c \ubb38\uc81c\ub97c \uc678\uba74\ud55c\ub2e4\uba74, \uc720\ub300\uc778\uc758 \uc5ed\uc0ac\ub294 \ub354 \uc774\uc0c1 \uc9c0\uc18d\ub420 \uc218 \uc5c6\uc744 \uac83\uc774\ub2e4. \u201d", "paragraph_answer": "\ub9c8\ucc2c\uac00\uc9c0\ub85c \ud754\uce58 \uc54a\uac70\ub098 \uc18c\uaddc\ubaa8\uc758 \uac1c\ubcc4\uc801\uc778 \uc0ac\ub840\ub4e4\uc744 \ub2f9\uc2dc\uc758 \uc2dc\ub300\uc0c1\uc778 \uc591 \ud3ec\uc7a5\ud558\uba74 \uc2e4\uc81c\ub85c \ubc8c\uc5b4\uc84c\ub358 \uc0ac\uac74\ub4e4\uc758 \uc758\uc758\uc640 \uae30\ubcf8\uc131\uaca9\uae4c\uc9c0\ub3c4 \uc65c\uace1\uc2dc\ucf1c\ubc84\ub9b4 \uc218\uac00 \uc788\ub2e4. \uc2e4\uc81c\ub85c \uc800\ud56d\uc758 \uc815\uc758 \ub97c \ub05d\uc5c6\uc774 \ud655\uc7a5\ud558\uace0 \uc0ac\ub840\ub4e4\uc740 \uacfc\uc7a5\ud558\ub2e4\ubcf4\ub2c8, \uc624\ud788\ub824 \uc2e4\uc81c\ub85c \uc2e4\ud589\ub418\uc5c8\ub358 \uc800\ud56d\ub2e4\uc6b4 \uc800\ud56d\uc6b4\ub3d9\ub4e4\uc740 \uc81c\ub300\ub85c \uc8fc\ubaa9\ubc1b\uc9c0 \ubabb\ud558\ub294 \uacbd\uc6b0\uac00 \ubc1c\uc0dd\ud558\uace0 \uc788\ub2e4. \uc774\ub7ec\ud55c \ud750\ub984\ub300\ub85c \uac04\ub2e4\uba74 \uc601\uc6c5\uc801 \uc815\uc2e0\uc740 \uc720\ub7fd\uc758 \ubaa8\ub4e0 \uc720\ub300\uc778\ub4e4\uacfc \uadf8 \uacf5\ub3d9\uccb4\ub4e4\uc758 \uae30\ubcf8 \uc18c\uc591\uc73c\ub85c \uce58\ubd80\ub418\uc5b4\ubc84\ub9ac\uace0, \uc815\uc791 \uc800\ud56d\uc744 \ud589\ub3d9\uc5d0 \uc62e\uacbc\ub358 \uc18c\uc218 \uc720\ub300\uc778\ub4e4\uc758 \uc9c4\uc815\ud55c \uc6a9\uae30\ub294 \ud1f4\uc0c9\uc2dc\ud0a4\ub294 \uacb0\uacfc\ub97c \ub0b3\uac8c \ub420 \uac83\uc774\ub2e4. \uc5ec\uae30\uc11c \uc6b0\ub9ac\ub294 \uc720\ub300\uc0ac\ud68c\uc758 \uc774\ub7ec\ud55c \u2018\uc21c\uc751\ud558\ub294 \ub300\ub2e4\uc218\uc640 \ud589\ub3d9\ud558\ub294 \uadf9\uc18c\uc218\uc758 \ud63c\uc7ac\uc640 \ubd88\ud1b5\u2019\uc744 \ub2e8\uc21c\ud788 \ubb38\uc81c\ud574\uacb0 \uacfc\uc815\uc744 \ud750\ub9ac\uba4d\ud145\ud558\uac8c \ub9cc\ub4dc\ub294 \ub9f9\ud0d5 \uacf5\ub3d9\uccb4\ub85c \uce58\ubd80\ud558\ub294\ub370 \uadf8\uccd0\uc11c\ub294 \uc548\ub41c\ub2e4. \uc774 \uad6c\uc870\ub294 \uc0ac\uc2e4\uc0c1 \uc790\uc2e0\ub4e4\uc774 \uc18d\ud55c \uacf5\ub3d9\uccb4\uc5d0 \ub300\ud55c \uc0dd\uc0b0\uc801\uc778 \uc9c8\ubb38\uacfc \ub300\uc548\uc744 \uc8fc\uace0\ubc1b\ub294 \uac83\uc774 \ubd88\uac00\ub2a5\ud558\uba70, \uc774\ub294 \uace7 \uacf5\ub3d9\uccb4 \uc2a4\uc2a4\ub85c \ucd94\ub860\ub2a5\ub825\uacfc \uc0dd\uc874\uc804\ub7b5\uc744 \ud3ec\uae30\ud558\ub294 \uac83\uacfc \ub9c8\ucc2c\uac00\uc9c0\uc774\ub2e4. \uc774\ub7ec\ud55c \ubb38\uc81c\ub97c \uc678\uba74\ud55c\ub2e4\uba74, \uc720\ub300\uc778\uc758 \uc5ed\uc0ac\ub294 \ub354 \uc774\uc0c1 \uc9c0\uc18d\ub420 \uc218 \uc5c6\uc744 \uac83\uc774\ub2e4.\u201d", "sentence_answer": "\uc2e4\uc81c\ub85c \uc800\ud56d\uc758 \uc815\uc758 \ub97c \ub05d\uc5c6\uc774 \ud655\uc7a5\ud558\uace0 \uc0ac\ub840\ub4e4\uc740 \uacfc\uc7a5\ud558\ub2e4\ubcf4\ub2c8, \uc624\ud788\ub824 \uc2e4\uc81c\ub85c \uc2e4\ud589\ub418\uc5c8\ub358 \uc800\ud56d\ub2e4\uc6b4 \uc800\ud56d\uc6b4\ub3d9\ub4e4\uc740 \uc81c\ub300\ub85c \uc8fc\ubaa9\ubc1b\uc9c0 \ubabb\ud558\ub294 \uacbd\uc6b0\uac00 \ubc1c\uc0dd\ud558\uace0 \uc788\ub2e4.", "paragraph_id": "6571961-24-1", "paragraph_question": "question: \ubb34\uc5c7\uc744 \ub05d\uc5c6\uc774 \ud655\uc7a5\ud558\uace0 \uacfc\uc7a5\ud558\ub2e4 \ubcf4\ub2c8 \uc2e4\uc81c \uc2e4\ud589\ub418\uc5c8\ub358 \uc800\ud56d\uc6b4\ub3d9\ub4e4\uc774 \uc8fc\ubaa9\ubc1b\uc9c0 \ubabb\ud558\ub294 \uacbd\uc6b0\uac00 \ubc1c\uc0dd\ud558\ub294\uac00?, context: \ub9c8\ucc2c\uac00\uc9c0\ub85c \ud754\uce58 \uc54a\uac70\ub098 \uc18c\uaddc\ubaa8\uc758 \uac1c\ubcc4\uc801\uc778 \uc0ac\ub840\ub4e4\uc744 \ub2f9\uc2dc\uc758 \uc2dc\ub300\uc0c1\uc778 \uc591 \ud3ec\uc7a5\ud558\uba74 \uc2e4\uc81c\ub85c \ubc8c\uc5b4\uc84c\ub358 \uc0ac\uac74\ub4e4\uc758 \uc758\uc758\uc640 \uae30\ubcf8\uc131\uaca9\uae4c\uc9c0\ub3c4 \uc65c\uace1\uc2dc\ucf1c\ubc84\ub9b4 \uc218\uac00 \uc788\ub2e4. \uc2e4\uc81c\ub85c \uc800\ud56d\uc758 \uc815\uc758\ub97c \ub05d\uc5c6\uc774 \ud655\uc7a5\ud558\uace0 \uc0ac\ub840\ub4e4\uc740 \uacfc\uc7a5\ud558\ub2e4\ubcf4\ub2c8, \uc624\ud788\ub824 \uc2e4\uc81c\ub85c \uc2e4\ud589\ub418\uc5c8\ub358 \uc800\ud56d\ub2e4\uc6b4 \uc800\ud56d\uc6b4\ub3d9\ub4e4\uc740 \uc81c\ub300\ub85c \uc8fc\ubaa9\ubc1b\uc9c0 \ubabb\ud558\ub294 \uacbd\uc6b0\uac00 \ubc1c\uc0dd\ud558\uace0 \uc788\ub2e4. \uc774\ub7ec\ud55c \ud750\ub984\ub300\ub85c \uac04\ub2e4\uba74 \uc601\uc6c5\uc801 \uc815\uc2e0\uc740 \uc720\ub7fd\uc758 \ubaa8\ub4e0 \uc720\ub300\uc778\ub4e4\uacfc \uadf8 \uacf5\ub3d9\uccb4\ub4e4\uc758 \uae30\ubcf8 \uc18c\uc591\uc73c\ub85c \uce58\ubd80\ub418\uc5b4\ubc84\ub9ac\uace0, \uc815\uc791 \uc800\ud56d\uc744 \ud589\ub3d9\uc5d0 \uc62e\uacbc\ub358 \uc18c\uc218 \uc720\ub300\uc778\ub4e4\uc758 \uc9c4\uc815\ud55c \uc6a9\uae30\ub294 \ud1f4\uc0c9\uc2dc\ud0a4\ub294 \uacb0\uacfc\ub97c \ub0b3\uac8c \ub420 \uac83\uc774\ub2e4. \uc5ec\uae30\uc11c \uc6b0\ub9ac\ub294 \uc720\ub300\uc0ac\ud68c\uc758 \uc774\ub7ec\ud55c \u2018\uc21c\uc751\ud558\ub294 \ub300\ub2e4\uc218\uc640 \ud589\ub3d9\ud558\ub294 \uadf9\uc18c\uc218\uc758 \ud63c\uc7ac\uc640 \ubd88\ud1b5\u2019\uc744 \ub2e8\uc21c\ud788 \ubb38\uc81c\ud574\uacb0 \uacfc\uc815\uc744 \ud750\ub9ac\uba4d\ud145\ud558\uac8c \ub9cc\ub4dc\ub294 \ub9f9\ud0d5 \uacf5\ub3d9\uccb4\ub85c \uce58\ubd80\ud558\ub294\ub370 \uadf8\uccd0\uc11c\ub294 \uc548\ub41c\ub2e4. \uc774 \uad6c\uc870\ub294 \uc0ac\uc2e4\uc0c1 \uc790\uc2e0\ub4e4\uc774 \uc18d\ud55c \uacf5\ub3d9\uccb4\uc5d0 \ub300\ud55c \uc0dd\uc0b0\uc801\uc778 \uc9c8\ubb38\uacfc \ub300\uc548\uc744 \uc8fc\uace0\ubc1b\ub294 \uac83\uc774 \ubd88\uac00\ub2a5\ud558\uba70, \uc774\ub294 \uace7 \uacf5\ub3d9\uccb4 \uc2a4\uc2a4\ub85c \ucd94\ub860\ub2a5\ub825\uacfc \uc0dd\uc874\uc804\ub7b5\uc744 \ud3ec\uae30\ud558\ub294 \uac83\uacfc \ub9c8\ucc2c\uac00\uc9c0\uc774\ub2e4. \uc774\ub7ec\ud55c \ubb38\uc81c\ub97c \uc678\uba74\ud55c\ub2e4\uba74, \uc720\ub300\uc778\uc758 \uc5ed\uc0ac\ub294 \ub354 \uc774\uc0c1 \uc9c0\uc18d\ub420 \uc218 \uc5c6\uc744 \uac83\uc774\ub2e4.\u201d"} {"question": "\ubbf8\uad6d \ub3c4\uc2dc \uc911 \ucd5c\ucd08\uc758 \ub3c4\uc2dc \uacf5\uc6d0\uc740?", "paragraph": "19\uc138\uae30, \ub274\uc695\uc740 \uc774\ubbfc\uc790\uc758 \uc720\uc785 \ubc0f \ub3c4\uc2dc \uac1c\ubc1c\uc5d0 \uc758\ud574 \ud06c\uac8c \ubc14\ub00c\uc5c8\ub2e4. 1811\ub144 \uc704\uc6d0\ud68c \uacc4\ud68d\uc5d0 \uc758\ud574 \ub9e8\ud574\ud2bc \uc804\uc5ed\uc740 \uaca9\uc790 \uac70\ub9ac\ub85c \ubc14\ub00c\uc5c8\ub2e4. 1819\ub144\uc5d0 \uc774\ub9ac \uc6b4\ud558\uac00 \uac1c\ud1b5\ub418\uc5b4, \ub300\uc11c\uc591\uc758 \ud56d\uad6c\uc640 \ubd81\uc544\uba54\ub9ac\uce74 \ub0b4\ub959\uc758 \uad11\ub300\ud55c \ub18d\uc5c5 \uc2dc\uc7a5\uc774 \uc5f0\uacb0\ub418\uc5c8\ub2e4. \uc774 \uc2dc\uae30\uc5d0\ub294 \uc544\uc77c\ub79c\ub4dc\uacc4 \uc815\ub2f9 \uc870\uc9c1\uc778 \ud0dc\uba38\ub2c8\ud640\uc774 \ub274\uc695\uc758 \uc815\uce58\ub97c \uc88c\uc9c0\uc6b0\uc9c0\ud558\uace0 \uc788\uc5c8\ub2e4. \uc13c\ud2b8\ub7f4 \ud30c\ud06c\ub3c4 \uac74\uc124\uc774 \uc2dc\uc791\ub418\uc5c8\uc73c\uba70, 1857\ub144 \ubbf8\uad6d\uc758 \ub3c4\uc2dc \uc911 \ucd5c\ucd08\uc758 \ub3c4\uc2dc \uacf5\uc6d0\uc774 \ub418\uc5c8\ub2e4. \ub9e8\ud574\ud2bc\uc774\ub098 \ube0c\ub8e8\ud074\ub9b0\uc5d0\ub294 \uc544\ud504\ub9ac\uce74\uacc4 \uc0ac\ub78c (\ud751\uc778)\ub3c4 \ub9ce\uc558\ub2e4. \ub274\uc695\uc740 1827\ub144\uae4c\uc9c0 \ub178\uc608 \uc81c\ub3c4\uac00 \uc720\uc9c0\ub418\uace0 \uc788\uc5c8\uace0, 1830\ub144\ub300\uc5d0 \ub274\uc695 \ubd81\ubd80\uac00 \ub178\uc608\uc81c \ud3d0\uc9c0 \uc6b4\ub3d9\uc758 \uc911\uc2ec\uc9c0\uac00 \ub418\uc5c8\ub2e4. 1840\ub144 \uc2dc\uc810\uc5d0\uc11c \ub274\uc695\uc758 \uc544\ud504\ub9ac\uce74\uacc4 \uc778\uad6c\ub294 1\ub9cc 6000\uba85\uc744 \ub118\uc5c8\ub2e4. 1847\ub144 \ubc1c\uc0dd\ud55c \uc544\uc77c\ub79c\ub4dc \ub300\uae30\uadfc\uc73c\ub85c \uc778\ud574 \uc544\uc77c\ub79c\ub4dc \uc774\uc8fc\ubbfc\ub4e4\uc758 \ub300\uaddc\ubaa8 \uc720\uc785\uc774 \uc77c\uc5b4\ub0ac\uace0, 1860\ub144\uc5d0\ub294 \uc544\uc77c\ub79c\ub4dc\uc778\ub4e4\uc774 20\ub9cc \uba85\uc744 \ub118\uc5b4 \ub274\uc695 \uc778\uad6c \ub124 \uba85 \uc911 \ud55c \uba85\uc740 \uc544\uc77c\ub79c\ub4dc\uc778\uc774\uc5c8\ub2e4. \ub3c5\uc77c\uc5d0\uc11c\ub3c4 \ub9ce\uc740 \uc774\ubbfc\uc790\ub4e4\uc774 \uc654\uc73c\uba70, 1860\ub144 \ub274\uc695 \uc778\uad6c \uc911 \uc544\uc77c\ub79c\ub4dc\uc778\ub4e4\uc744 \uc81c\uc678\ud558\uace0 25%\ub97c \ub3c5\uc77c\uc778\ub4e4\uc774 \ucc28\uc9c0\ud588\ub2e4.", "answer": "\uc13c\ud2b8\ub7f4 \ud30c\ud06c", "sentence": "\uc13c\ud2b8\ub7f4 \ud30c\ud06c \ub3c4 \uac74\uc124\uc774 \uc2dc\uc791\ub418\uc5c8\uc73c\uba70, 1857\ub144 \ubbf8\uad6d\uc758 \ub3c4\uc2dc \uc911 \ucd5c\ucd08\uc758 \ub3c4\uc2dc \uacf5\uc6d0\uc774 \ub418\uc5c8\ub2e4.", "paragraph_sentence": "19\uc138\uae30, \ub274\uc695\uc740 \uc774\ubbfc\uc790\uc758 \uc720\uc785 \ubc0f \ub3c4\uc2dc \uac1c\ubc1c\uc5d0 \uc758\ud574 \ud06c\uac8c \ubc14\ub00c\uc5c8\ub2e4. 1811\ub144 \uc704\uc6d0\ud68c \uacc4\ud68d\uc5d0 \uc758\ud574 \ub9e8\ud574\ud2bc \uc804\uc5ed\uc740 \uaca9\uc790 \uac70\ub9ac\ub85c \ubc14\ub00c\uc5c8\ub2e4. 1819\ub144\uc5d0 \uc774\ub9ac \uc6b4\ud558\uac00 \uac1c\ud1b5\ub418\uc5b4, \ub300\uc11c\uc591\uc758 \ud56d\uad6c\uc640 \ubd81\uc544\uba54\ub9ac\uce74 \ub0b4\ub959\uc758 \uad11\ub300\ud55c \ub18d\uc5c5 \uc2dc\uc7a5\uc774 \uc5f0\uacb0\ub418\uc5c8\ub2e4. \uc774 \uc2dc\uae30\uc5d0\ub294 \uc544\uc77c\ub79c\ub4dc\uacc4 \uc815\ub2f9 \uc870\uc9c1\uc778 \ud0dc\uba38\ub2c8\ud640\uc774 \ub274\uc695\uc758 \uc815\uce58\ub97c \uc88c\uc9c0\uc6b0\uc9c0\ud558\uace0 \uc788\uc5c8\ub2e4. \uc13c\ud2b8\ub7f4 \ud30c\ud06c \ub3c4 \uac74\uc124\uc774 \uc2dc\uc791\ub418\uc5c8\uc73c\uba70, 1857\ub144 \ubbf8\uad6d\uc758 \ub3c4\uc2dc \uc911 \ucd5c\ucd08\uc758 \ub3c4\uc2dc \uacf5\uc6d0\uc774 \ub418\uc5c8\ub2e4. \ub9e8\ud574\ud2bc\uc774\ub098 \ube0c\ub8e8\ud074\ub9b0\uc5d0\ub294 \uc544\ud504\ub9ac\uce74\uacc4 \uc0ac\ub78c (\ud751\uc778)\ub3c4 \ub9ce\uc558\ub2e4. \ub274\uc695\uc740 1827\ub144\uae4c\uc9c0 \ub178\uc608 \uc81c\ub3c4\uac00 \uc720\uc9c0\ub418\uace0 \uc788\uc5c8\uace0, 1830\ub144\ub300\uc5d0 \ub274\uc695 \ubd81\ubd80\uac00 \ub178\uc608\uc81c \ud3d0\uc9c0 \uc6b4\ub3d9\uc758 \uc911\uc2ec\uc9c0\uac00 \ub418\uc5c8\ub2e4. 1840\ub144 \uc2dc\uc810\uc5d0\uc11c \ub274\uc695\uc758 \uc544\ud504\ub9ac\uce74\uacc4 \uc778\uad6c\ub294 1\ub9cc 6000\uba85\uc744 \ub118\uc5c8\ub2e4. 1847\ub144 \ubc1c\uc0dd\ud55c \uc544\uc77c\ub79c\ub4dc \ub300\uae30\uadfc\uc73c\ub85c \uc778\ud574 \uc544\uc77c\ub79c\ub4dc \uc774\uc8fc\ubbfc\ub4e4\uc758 \ub300\uaddc\ubaa8 \uc720\uc785\uc774 \uc77c\uc5b4\ub0ac\uace0, 1860\ub144\uc5d0\ub294 \uc544\uc77c\ub79c\ub4dc\uc778\ub4e4\uc774 20\ub9cc \uba85\uc744 \ub118\uc5b4 \ub274\uc695 \uc778\uad6c \ub124 \uba85 \uc911 \ud55c \uba85\uc740 \uc544\uc77c\ub79c\ub4dc\uc778\uc774\uc5c8\ub2e4. \ub3c5\uc77c\uc5d0\uc11c\ub3c4 \ub9ce\uc740 \uc774\ubbfc\uc790\ub4e4\uc774 \uc654\uc73c\uba70, 1860\ub144 \ub274\uc695 \uc778\uad6c \uc911 \uc544\uc77c\ub79c\ub4dc\uc778\ub4e4\uc744 \uc81c\uc678\ud558\uace0 25%\ub97c \ub3c5\uc77c\uc778\ub4e4\uc774 \ucc28\uc9c0\ud588\ub2e4.", "paragraph_answer": "19\uc138\uae30, \ub274\uc695\uc740 \uc774\ubbfc\uc790\uc758 \uc720\uc785 \ubc0f \ub3c4\uc2dc \uac1c\ubc1c\uc5d0 \uc758\ud574 \ud06c\uac8c \ubc14\ub00c\uc5c8\ub2e4. 1811\ub144 \uc704\uc6d0\ud68c \uacc4\ud68d\uc5d0 \uc758\ud574 \ub9e8\ud574\ud2bc \uc804\uc5ed\uc740 \uaca9\uc790 \uac70\ub9ac\ub85c \ubc14\ub00c\uc5c8\ub2e4. 1819\ub144\uc5d0 \uc774\ub9ac \uc6b4\ud558\uac00 \uac1c\ud1b5\ub418\uc5b4, \ub300\uc11c\uc591\uc758 \ud56d\uad6c\uc640 \ubd81\uc544\uba54\ub9ac\uce74 \ub0b4\ub959\uc758 \uad11\ub300\ud55c \ub18d\uc5c5 \uc2dc\uc7a5\uc774 \uc5f0\uacb0\ub418\uc5c8\ub2e4. \uc774 \uc2dc\uae30\uc5d0\ub294 \uc544\uc77c\ub79c\ub4dc\uacc4 \uc815\ub2f9 \uc870\uc9c1\uc778 \ud0dc\uba38\ub2c8\ud640\uc774 \ub274\uc695\uc758 \uc815\uce58\ub97c \uc88c\uc9c0\uc6b0\uc9c0\ud558\uace0 \uc788\uc5c8\ub2e4. \uc13c\ud2b8\ub7f4 \ud30c\ud06c \ub3c4 \uac74\uc124\uc774 \uc2dc\uc791\ub418\uc5c8\uc73c\uba70, 1857\ub144 \ubbf8\uad6d\uc758 \ub3c4\uc2dc \uc911 \ucd5c\ucd08\uc758 \ub3c4\uc2dc \uacf5\uc6d0\uc774 \ub418\uc5c8\ub2e4. \ub9e8\ud574\ud2bc\uc774\ub098 \ube0c\ub8e8\ud074\ub9b0\uc5d0\ub294 \uc544\ud504\ub9ac\uce74\uacc4 \uc0ac\ub78c (\ud751\uc778)\ub3c4 \ub9ce\uc558\ub2e4. \ub274\uc695\uc740 1827\ub144\uae4c\uc9c0 \ub178\uc608 \uc81c\ub3c4\uac00 \uc720\uc9c0\ub418\uace0 \uc788\uc5c8\uace0, 1830\ub144\ub300\uc5d0 \ub274\uc695 \ubd81\ubd80\uac00 \ub178\uc608\uc81c \ud3d0\uc9c0 \uc6b4\ub3d9\uc758 \uc911\uc2ec\uc9c0\uac00 \ub418\uc5c8\ub2e4. 1840\ub144 \uc2dc\uc810\uc5d0\uc11c \ub274\uc695\uc758 \uc544\ud504\ub9ac\uce74\uacc4 \uc778\uad6c\ub294 1\ub9cc 6000\uba85\uc744 \ub118\uc5c8\ub2e4. 1847\ub144 \ubc1c\uc0dd\ud55c \uc544\uc77c\ub79c\ub4dc \ub300\uae30\uadfc\uc73c\ub85c \uc778\ud574 \uc544\uc77c\ub79c\ub4dc \uc774\uc8fc\ubbfc\ub4e4\uc758 \ub300\uaddc\ubaa8 \uc720\uc785\uc774 \uc77c\uc5b4\ub0ac\uace0, 1860\ub144\uc5d0\ub294 \uc544\uc77c\ub79c\ub4dc\uc778\ub4e4\uc774 20\ub9cc \uba85\uc744 \ub118\uc5b4 \ub274\uc695 \uc778\uad6c \ub124 \uba85 \uc911 \ud55c \uba85\uc740 \uc544\uc77c\ub79c\ub4dc\uc778\uc774\uc5c8\ub2e4. \ub3c5\uc77c\uc5d0\uc11c\ub3c4 \ub9ce\uc740 \uc774\ubbfc\uc790\ub4e4\uc774 \uc654\uc73c\uba70, 1860\ub144 \ub274\uc695 \uc778\uad6c \uc911 \uc544\uc77c\ub79c\ub4dc\uc778\ub4e4\uc744 \uc81c\uc678\ud558\uace0 25%\ub97c \ub3c5\uc77c\uc778\ub4e4\uc774 \ucc28\uc9c0\ud588\ub2e4.", "sentence_answer": " \uc13c\ud2b8\ub7f4 \ud30c\ud06c \ub3c4 \uac74\uc124\uc774 \uc2dc\uc791\ub418\uc5c8\uc73c\uba70, 1857\ub144 \ubbf8\uad6d\uc758 \ub3c4\uc2dc \uc911 \ucd5c\ucd08\uc758 \ub3c4\uc2dc \uacf5\uc6d0\uc774 \ub418\uc5c8\ub2e4.", "paragraph_id": "6383054-1-1", "paragraph_question": "question: \ubbf8\uad6d \ub3c4\uc2dc \uc911 \ucd5c\ucd08\uc758 \ub3c4\uc2dc \uacf5\uc6d0\uc740?, context: 19\uc138\uae30, \ub274\uc695\uc740 \uc774\ubbfc\uc790\uc758 \uc720\uc785 \ubc0f \ub3c4\uc2dc \uac1c\ubc1c\uc5d0 \uc758\ud574 \ud06c\uac8c \ubc14\ub00c\uc5c8\ub2e4. 1811\ub144 \uc704\uc6d0\ud68c \uacc4\ud68d\uc5d0 \uc758\ud574 \ub9e8\ud574\ud2bc \uc804\uc5ed\uc740 \uaca9\uc790 \uac70\ub9ac\ub85c \ubc14\ub00c\uc5c8\ub2e4. 1819\ub144\uc5d0 \uc774\ub9ac \uc6b4\ud558\uac00 \uac1c\ud1b5\ub418\uc5b4, \ub300\uc11c\uc591\uc758 \ud56d\uad6c\uc640 \ubd81\uc544\uba54\ub9ac\uce74 \ub0b4\ub959\uc758 \uad11\ub300\ud55c \ub18d\uc5c5 \uc2dc\uc7a5\uc774 \uc5f0\uacb0\ub418\uc5c8\ub2e4. \uc774 \uc2dc\uae30\uc5d0\ub294 \uc544\uc77c\ub79c\ub4dc\uacc4 \uc815\ub2f9 \uc870\uc9c1\uc778 \ud0dc\uba38\ub2c8\ud640\uc774 \ub274\uc695\uc758 \uc815\uce58\ub97c \uc88c\uc9c0\uc6b0\uc9c0\ud558\uace0 \uc788\uc5c8\ub2e4. \uc13c\ud2b8\ub7f4 \ud30c\ud06c\ub3c4 \uac74\uc124\uc774 \uc2dc\uc791\ub418\uc5c8\uc73c\uba70, 1857\ub144 \ubbf8\uad6d\uc758 \ub3c4\uc2dc \uc911 \ucd5c\ucd08\uc758 \ub3c4\uc2dc \uacf5\uc6d0\uc774 \ub418\uc5c8\ub2e4. \ub9e8\ud574\ud2bc\uc774\ub098 \ube0c\ub8e8\ud074\ub9b0\uc5d0\ub294 \uc544\ud504\ub9ac\uce74\uacc4 \uc0ac\ub78c (\ud751\uc778)\ub3c4 \ub9ce\uc558\ub2e4. \ub274\uc695\uc740 1827\ub144\uae4c\uc9c0 \ub178\uc608 \uc81c\ub3c4\uac00 \uc720\uc9c0\ub418\uace0 \uc788\uc5c8\uace0, 1830\ub144\ub300\uc5d0 \ub274\uc695 \ubd81\ubd80\uac00 \ub178\uc608\uc81c \ud3d0\uc9c0 \uc6b4\ub3d9\uc758 \uc911\uc2ec\uc9c0\uac00 \ub418\uc5c8\ub2e4. 1840\ub144 \uc2dc\uc810\uc5d0\uc11c \ub274\uc695\uc758 \uc544\ud504\ub9ac\uce74\uacc4 \uc778\uad6c\ub294 1\ub9cc 6000\uba85\uc744 \ub118\uc5c8\ub2e4. 1847\ub144 \ubc1c\uc0dd\ud55c \uc544\uc77c\ub79c\ub4dc \ub300\uae30\uadfc\uc73c\ub85c \uc778\ud574 \uc544\uc77c\ub79c\ub4dc \uc774\uc8fc\ubbfc\ub4e4\uc758 \ub300\uaddc\ubaa8 \uc720\uc785\uc774 \uc77c\uc5b4\ub0ac\uace0, 1860\ub144\uc5d0\ub294 \uc544\uc77c\ub79c\ub4dc\uc778\ub4e4\uc774 20\ub9cc \uba85\uc744 \ub118\uc5b4 \ub274\uc695 \uc778\uad6c \ub124 \uba85 \uc911 \ud55c \uba85\uc740 \uc544\uc77c\ub79c\ub4dc\uc778\uc774\uc5c8\ub2e4. \ub3c5\uc77c\uc5d0\uc11c\ub3c4 \ub9ce\uc740 \uc774\ubbfc\uc790\ub4e4\uc774 \uc654\uc73c\uba70, 1860\ub144 \ub274\uc695 \uc778\uad6c \uc911 \uc544\uc77c\ub79c\ub4dc\uc778\ub4e4\uc744 \uc81c\uc678\ud558\uace0 25%\ub97c \ub3c5\uc77c\uc778\ub4e4\uc774 \ucc28\uc9c0\ud588\ub2e4."} {"question": "2011\ub144 10\uc6d4 28\uc77c, \ubca0\uc77c\uc740 \uc5b4\ub5a4 \uc720\ud2f0\ud3fc\uc744 \uc785\uace0 \ucd2c\uc601\uc5d0 \uc784\ud588\ub294\uac00?", "paragraph": "\ubca0\uc77c\uc740 2012\ub144 \ud558\uacc4 \uc62c\ub9bc\ud53d\uc5d0 \ucc38\uac00\ud560 \uc218 \uc788\ub294 \uc5f0\ub839 \uc870\uac74\uc744 \ub9cc\uc871\ud588\ub294\ub370, \uadf8\ub294 \uc6e8\uc77c\uc2a4 \ucd95\uad6c \ud611\ud68c\uc758 \ubc18\ub300\uc5d0\ub3c4 \ubd88\uad6c\ud558\uace0 \uc601\uad6d \uc62c\ub9bc\ud53d \ucd95\uad6c \ub300\ud45c\ud300\uc744 \ub300\ud45c\ud558\uc5ec \"\uc62c\ub9bc\ud53d\uc5d0 \ucd9c\uc804\ud558\uace0 \uc2f6\ub2e4\" \ub77c\uace0 \ub9d0\ud55c \uc801\uc774 \uc788\uc5c8\ub2e4. 2011\ub144 10\uc6d4 28\uc77c, \ubca0\uc77c\uc740 \uc601\uad6d\uc758 2012\ub144 \ub7f0\ub358 \uc62c\ub9bc\ud53d \ucd95\uad6c \ud32c \uc720\ub2c8\ud3fc\uc744 \uc785\uace0 \ucd2c\uc601\uc5d0 \uc784\ud55c \uccab \uc120\uc218\uc600\ub2e4. \uc774\uc5d0 \ub300\uc751\ud574 \uc6e8\uc77c\uc2a4 \ucd95\uad6c \ud611\ud68c\uc758 \ub300\ud45c \uc774\uc0ac \uc870\ub108\uc120 \ud3ec\ub4dc\ub294: \"\uc6b0\ub9ac\uc758 \uacb0\uc815\uc740 \ubc88\ubcf5\ub418\uc9c0 \uc54a\uc558\uc2b5\ub2c8\ub2e4. \uc6b0\ub9ac\ub294 \uc601\uad6d \ub300\ud45c\ud300\uc744 \uc9c0\uc9c0\ud558\uc9c0 \uc54a\uc2b5\ub2c8\ub2e4. \uac1c\ub7ec\uc2a4\ub294 \uc790\uc2e0\ub9cc\uc758 \uc120\ud0dd\uc744 \ud560 \uad8c\ud55c\uc774 \uc788\uc2b5\ub2c8\ub2e4. \uadf8\ub7ec\ub098, \uc6b0\ub9ac\ub294 \ub3d9\ucc38\ud558\ub294 \uac83\uc744 \ub9c9\uc9c0\ub294 \uc54a\uaca0\uc2b5\ub2c8\ub2e4.\" \ub77c\uace0 \ub9d0\ud588\ub2e4. \ubca0\uc77c\uc758 \ub300\ubcc0\uc778\uc740 \"\uadf8\ub294 \uc21c\uc218 \uc6e8\uc77c\uc2a4\uc778\uc774\uc9c0\ub9cc, \uc601\uad6d\uc778\uc774\uae30\ub3c4 \ud569\ub2c8\ub2e4.\" \ub77c\uace0 \uc804\ub2ec\ud588\ub2e4.", "answer": "\uc601\uad6d\uc758 2012\ub144 \ub7f0\ub358 \uc62c\ub9bc\ud53d \ucd95\uad6c \ud32c \uc720\ub2c8\ud3fc", "sentence": "2011\ub144 10\uc6d4 28\uc77c, \ubca0\uc77c\uc740 \uc601\uad6d\uc758 2012\ub144 \ub7f0\ub358 \uc62c\ub9bc\ud53d \ucd95\uad6c \ud32c \uc720\ub2c8\ud3fc \uc744 \uc785\uace0 \ucd2c\uc601\uc5d0 \uc784\ud55c \uccab \uc120\uc218\uc600\ub2e4.", "paragraph_sentence": "\ubca0\uc77c\uc740 2012\ub144 \ud558\uacc4 \uc62c\ub9bc\ud53d\uc5d0 \ucc38\uac00\ud560 \uc218 \uc788\ub294 \uc5f0\ub839 \uc870\uac74\uc744 \ub9cc\uc871\ud588\ub294\ub370, \uadf8\ub294 \uc6e8\uc77c\uc2a4 \ucd95\uad6c \ud611\ud68c\uc758 \ubc18\ub300\uc5d0\ub3c4 \ubd88\uad6c\ud558\uace0 \uc601\uad6d \uc62c\ub9bc\ud53d \ucd95\uad6c \ub300\ud45c\ud300\uc744 \ub300\ud45c\ud558\uc5ec \"\uc62c\ub9bc\ud53d\uc5d0 \ucd9c\uc804\ud558\uace0 \uc2f6\ub2e4\" \ub77c\uace0 \ub9d0\ud55c \uc801\uc774 \uc788\uc5c8\ub2e4. 2011\ub144 10\uc6d4 28\uc77c, \ubca0\uc77c\uc740 \uc601\uad6d\uc758 2012\ub144 \ub7f0\ub358 \uc62c\ub9bc\ud53d \ucd95\uad6c \ud32c \uc720\ub2c8\ud3fc \uc744 \uc785\uace0 \ucd2c\uc601\uc5d0 \uc784\ud55c \uccab \uc120\uc218\uc600\ub2e4. \uc774\uc5d0 \ub300\uc751\ud574 \uc6e8\uc77c\uc2a4 \ucd95\uad6c \ud611\ud68c\uc758 \ub300\ud45c \uc774\uc0ac \uc870\ub108\uc120 \ud3ec\ub4dc\ub294: \"\uc6b0\ub9ac\uc758 \uacb0\uc815\uc740 \ubc88\ubcf5\ub418\uc9c0 \uc54a\uc558\uc2b5\ub2c8\ub2e4. \uc6b0\ub9ac\ub294 \uc601\uad6d \ub300\ud45c\ud300\uc744 \uc9c0\uc9c0\ud558\uc9c0 \uc54a\uc2b5\ub2c8\ub2e4. \uac1c\ub7ec\uc2a4\ub294 \uc790\uc2e0\ub9cc\uc758 \uc120\ud0dd\uc744 \ud560 \uad8c\ud55c\uc774 \uc788\uc2b5\ub2c8\ub2e4. \uadf8\ub7ec\ub098, \uc6b0\ub9ac\ub294 \ub3d9\ucc38\ud558\ub294 \uac83\uc744 \ub9c9\uc9c0\ub294 \uc54a\uaca0\uc2b5\ub2c8\ub2e4.\" \ub77c\uace0 \ub9d0\ud588\ub2e4. \ubca0\uc77c\uc758 \ub300\ubcc0\uc778\uc740 \"\uadf8\ub294 \uc21c\uc218 \uc6e8\uc77c\uc2a4\uc778\uc774\uc9c0\ub9cc, \uc601\uad6d\uc778\uc774\uae30\ub3c4 \ud569\ub2c8\ub2e4.\" \ub77c\uace0 \uc804\ub2ec\ud588\ub2e4.", "paragraph_answer": "\ubca0\uc77c\uc740 2012\ub144 \ud558\uacc4 \uc62c\ub9bc\ud53d\uc5d0 \ucc38\uac00\ud560 \uc218 \uc788\ub294 \uc5f0\ub839 \uc870\uac74\uc744 \ub9cc\uc871\ud588\ub294\ub370, \uadf8\ub294 \uc6e8\uc77c\uc2a4 \ucd95\uad6c \ud611\ud68c\uc758 \ubc18\ub300\uc5d0\ub3c4 \ubd88\uad6c\ud558\uace0 \uc601\uad6d \uc62c\ub9bc\ud53d \ucd95\uad6c \ub300\ud45c\ud300\uc744 \ub300\ud45c\ud558\uc5ec \"\uc62c\ub9bc\ud53d\uc5d0 \ucd9c\uc804\ud558\uace0 \uc2f6\ub2e4\" \ub77c\uace0 \ub9d0\ud55c \uc801\uc774 \uc788\uc5c8\ub2e4. 2011\ub144 10\uc6d4 28\uc77c, \ubca0\uc77c\uc740 \uc601\uad6d\uc758 2012\ub144 \ub7f0\ub358 \uc62c\ub9bc\ud53d \ucd95\uad6c \ud32c \uc720\ub2c8\ud3fc \uc744 \uc785\uace0 \ucd2c\uc601\uc5d0 \uc784\ud55c \uccab \uc120\uc218\uc600\ub2e4. \uc774\uc5d0 \ub300\uc751\ud574 \uc6e8\uc77c\uc2a4 \ucd95\uad6c \ud611\ud68c\uc758 \ub300\ud45c \uc774\uc0ac \uc870\ub108\uc120 \ud3ec\ub4dc\ub294: \"\uc6b0\ub9ac\uc758 \uacb0\uc815\uc740 \ubc88\ubcf5\ub418\uc9c0 \uc54a\uc558\uc2b5\ub2c8\ub2e4. \uc6b0\ub9ac\ub294 \uc601\uad6d \ub300\ud45c\ud300\uc744 \uc9c0\uc9c0\ud558\uc9c0 \uc54a\uc2b5\ub2c8\ub2e4. \uac1c\ub7ec\uc2a4\ub294 \uc790\uc2e0\ub9cc\uc758 \uc120\ud0dd\uc744 \ud560 \uad8c\ud55c\uc774 \uc788\uc2b5\ub2c8\ub2e4. \uadf8\ub7ec\ub098, \uc6b0\ub9ac\ub294 \ub3d9\ucc38\ud558\ub294 \uac83\uc744 \ub9c9\uc9c0\ub294 \uc54a\uaca0\uc2b5\ub2c8\ub2e4.\" \ub77c\uace0 \ub9d0\ud588\ub2e4. \ubca0\uc77c\uc758 \ub300\ubcc0\uc778\uc740 \"\uadf8\ub294 \uc21c\uc218 \uc6e8\uc77c\uc2a4\uc778\uc774\uc9c0\ub9cc, \uc601\uad6d\uc778\uc774\uae30\ub3c4 \ud569\ub2c8\ub2e4.\" \ub77c\uace0 \uc804\ub2ec\ud588\ub2e4.", "sentence_answer": "2011\ub144 10\uc6d4 28\uc77c, \ubca0\uc77c\uc740 \uc601\uad6d\uc758 2012\ub144 \ub7f0\ub358 \uc62c\ub9bc\ud53d \ucd95\uad6c \ud32c \uc720\ub2c8\ud3fc \uc744 \uc785\uace0 \ucd2c\uc601\uc5d0 \uc784\ud55c \uccab \uc120\uc218\uc600\ub2e4.", "paragraph_id": "6473548-16-0", "paragraph_question": "question: 2011\ub144 10\uc6d4 28\uc77c, \ubca0\uc77c\uc740 \uc5b4\ub5a4 \uc720\ud2f0\ud3fc\uc744 \uc785\uace0 \ucd2c\uc601\uc5d0 \uc784\ud588\ub294\uac00?, context: \ubca0\uc77c\uc740 2012\ub144 \ud558\uacc4 \uc62c\ub9bc\ud53d\uc5d0 \ucc38\uac00\ud560 \uc218 \uc788\ub294 \uc5f0\ub839 \uc870\uac74\uc744 \ub9cc\uc871\ud588\ub294\ub370, \uadf8\ub294 \uc6e8\uc77c\uc2a4 \ucd95\uad6c \ud611\ud68c\uc758 \ubc18\ub300\uc5d0\ub3c4 \ubd88\uad6c\ud558\uace0 \uc601\uad6d \uc62c\ub9bc\ud53d \ucd95\uad6c \ub300\ud45c\ud300\uc744 \ub300\ud45c\ud558\uc5ec \"\uc62c\ub9bc\ud53d\uc5d0 \ucd9c\uc804\ud558\uace0 \uc2f6\ub2e4\" \ub77c\uace0 \ub9d0\ud55c \uc801\uc774 \uc788\uc5c8\ub2e4. 2011\ub144 10\uc6d4 28\uc77c, \ubca0\uc77c\uc740 \uc601\uad6d\uc758 2012\ub144 \ub7f0\ub358 \uc62c\ub9bc\ud53d \ucd95\uad6c \ud32c \uc720\ub2c8\ud3fc\uc744 \uc785\uace0 \ucd2c\uc601\uc5d0 \uc784\ud55c \uccab \uc120\uc218\uc600\ub2e4. \uc774\uc5d0 \ub300\uc751\ud574 \uc6e8\uc77c\uc2a4 \ucd95\uad6c \ud611\ud68c\uc758 \ub300\ud45c \uc774\uc0ac \uc870\ub108\uc120 \ud3ec\ub4dc\ub294: \"\uc6b0\ub9ac\uc758 \uacb0\uc815\uc740 \ubc88\ubcf5\ub418\uc9c0 \uc54a\uc558\uc2b5\ub2c8\ub2e4. \uc6b0\ub9ac\ub294 \uc601\uad6d \ub300\ud45c\ud300\uc744 \uc9c0\uc9c0\ud558\uc9c0 \uc54a\uc2b5\ub2c8\ub2e4. \uac1c\ub7ec\uc2a4\ub294 \uc790\uc2e0\ub9cc\uc758 \uc120\ud0dd\uc744 \ud560 \uad8c\ud55c\uc774 \uc788\uc2b5\ub2c8\ub2e4. \uadf8\ub7ec\ub098, \uc6b0\ub9ac\ub294 \ub3d9\ucc38\ud558\ub294 \uac83\uc744 \ub9c9\uc9c0\ub294 \uc54a\uaca0\uc2b5\ub2c8\ub2e4.\" \ub77c\uace0 \ub9d0\ud588\ub2e4. \ubca0\uc77c\uc758 \ub300\ubcc0\uc778\uc740 \"\uadf8\ub294 \uc21c\uc218 \uc6e8\uc77c\uc2a4\uc778\uc774\uc9c0\ub9cc, \uc601\uad6d\uc778\uc774\uae30\ub3c4 \ud569\ub2c8\ub2e4.\" \ub77c\uace0 \uc804\ub2ec\ud588\ub2e4."} {"question": "\ud0dc\ud48d \ud30c\ub9c8\ub85c \uc778\ud574 \ucd5c\ucd08\ub85c \ud0dc\ud48d\uacbd\ubcf4 \uc81c3\ub2e8\uacc4\uac00 \ubc1c\ub839\ub41c \ud544\ub9ac\ud540 \uc9c0\uc5ed\uc740 \uc5b4\ub514\uc778\uac00?", "paragraph": "\ud0dc\ud48d \ud30c\ub9c8\uac00 \ud544\ub9ac\ud540 \ubd81\ubd80\uc5d0 \uc0c1\ub959\ud558\uae30 \ud55c\ucc38 \uc804\ubd80\ud130 \uc5ec\ub7ec \uad50\ud1b5\uc218\ub2e8\ub4e4\uc758 \uc6b4\ud589\uc774 \uc911\uc9c0\ub418\uc5c8\ub2e4. \uce74\ud0c4\ub450\uc544\ub124\uc2a4 \uc8fc\uc5d0\uc11c\ub294 \uc804\uad6d\uc5d0\uc11c \ucd5c\ucd08\ub85c \ud0dc\ud48d\uacbd\ubcf4 \uc81c3\ub2e8\uacc4\uac00 \ubc1c\ub839\ub418\uc5c8\uace0 \uc8fc \uc548\uc5d0\uc11c\uc758 \uc804\uae30 \uacf5\uae09\uc774 \uc911\ub2e8\ub418\uc5c8\uace0 \ud1b5\uc2e0\uc774 \ub450\uc808\ub418\uc5c8\ub2e4. \uc8fc\uc694 \uac04\uc120\ub3c4\ub85c\uc5d0 \ubf51\ud78c \ub098\ubb34\ub4e4\uc774 \uc313\uc5ec\uc788\uc5c8\ub2e4. \ub2f9\uc2dc \uc57d 3\ub9cc \uac00\uad6c\uac00 \ub300\ud53c\ub97c \uc644\ub8cc\ud588\ub2e4. \ube44\ucf5c \uc9c0\ubc29\uc5d0\uc11c\ub294 \uc5ec\uac1d\uc120 \uc2b9\uac1d 2,000\uba85\uc774 \ud56d\uad6c\uc5d0\uc11c \uace0\ub9bd\ub418\uc5c8\uace0 \ub9c8\ub2d0\ub77c \uc8fc\ubcc0 39\uac1c \uc8fc\uc5d0 \ud0dc\ud48d\uacbd\ubcf4\uac00 \ub0b4\ub824\uc84c\uace0 \uc774 \uc9c0\uc5ed\uc5d0\uc11c \uadf9\uc2ec\ud55c \ud3ed\uc6b0\uacfc \uac15\ud48d\uc744 \uacaa\uc5c8\ub2e4. \ud30c\ub9c8\ub294 \uce74\uac00\uc580 \uc8fc \ubd81\ub3d9\ubd80 \ud574\uc548\uc5d0 \uc0c1\ub959\ud588\uace0 \uc774 \uc8fc\uc5d0\uc11c \ub18d\uc5c5\ubd84\uc57c 2033\ub9cc \ud544\ub9ac\ud540 \ud398\uc18c\uc758 \uc7ac\uc0b0\ud53c\ud574\uac00 \ub0ac\ub2e4. \uce7c\ub9c1\uac00 \uc8fc\uc5d0\uc11c\ub294 \uc8fc\uc758 \uccad\uc0ac\uc640 \uc5f0\uacb0\ub418\ub294 \uae38\uc774 \uc0b0\uc0ac\ud0dc\ub85c \uc778\ud574 \ucc28\ub2e8\ub418\uc5c8\ub2e4. \uc0bc\ubc1c\ub808\uc2a4 \uc8fc\uc5d0\uc11c\ub294 \uc57d 2,100\uac00\uad6c\uac00 \ub300\ud53c\ud588\uace0 \ud321\uac00\uc2dc\ub09c \uc8fc\uc758 \ud55c \ub310\uacfc \ub204\uc5d0\ubc14\uc5d0\uc2dc\ud558 \uc8fc\uc758 \ud55c \ub310\uc774 \ud3ed\uc6b0\ub85c \uc778\ud574 \ub310 \uc548\uc758 \ubb3c\uc774 \ubc94\ub78c\uc704\uae30\uc5d0 \ucc98\ud558\uc790 \uc5c4\uccad\ub09c \uc591\uc758 \ubb3c\uc744 \ubc29\ub958\ud558\uae30 \uc2dc\uc791\ud588\ub2e4. \uc5c4\uccad\ub09c \uc591\uc758 \ubb3c\uc774 \ubc29\ub958\ud568\uc5d0 \ub530\ub77c \ud321\uac00\uc2dc\ub09c \uc8fc \ub3d9\ubd80\uc640 \uc911\ubd80, \ub204\uc5d0\ubcf4\uc5d0\uc2dc\ud558 \uc8fc\uc5d0 \ud64d\uc218\ub97c \uc77c\uc73c\ucf30\ub2e4. \ub9c8\ub2d0\ub77c \uc8fc\ubcc0 \uc9c0\uc5ed\uc5d0\uc11c\ub294 \uc5c4\uccad\ub09c \ud3ed\uc6b0\uc640 \uac15\ud48d\uc774 \ubd88\uc5b4\ub2e5\ucce4\uace0 \ud64d\uc218\ub85c \uc778\ud574 \uc218\uba74\uc774 \uc810\ucc28 \uc62c\ub77c\uac00\uae30 \uc2dc\uc791\ud588\ub2e4. \ubca0\uc774 \ud638\uc5d0\uc11c\ub294 90\ub144 \ub9cc\uc5d0 \ud638\uc218\uc758 \ubb3c\uc774 \uc5c4\uccad\ub09c \uc591\uc73c\ub85c \uc99d\uac00\ud588\uace0 \uc774\ub85c \uc778\ud574 \ub9c8\ub2d0\ub77c \ub3c4\uc2ec\uacfc \uadfc\uad50\uc9c0\uc5ed\uc774 \ubb3c\uc5d0 \uc7a0\uae38 \uc704\ud5d8\uc5d0 \ucc98\ud588\ub2e4. \ubcb5\uac8c\ud2b8 \uc8fc\uc5d0\uc11c\ub294 \uc0b0\uc0ac\ud0dc\uac00 \uc57d 200\uba85\uc744 \uc228\uc9c0\uac8c \ud588\uace0 \ub819\ud1a0\uc2a4\ud53c\ub77c\ubcd1\uc774 \ubc1c\uc0dd\ud574 \ub9ce\uc740 \uc0ac\ub78c\ub4e4\uc5d0\uac8c \uc601\ud5a5\uc744 \ub07c\ucce4\ub2e4. \ud0dc\ud48d \ud30c\ub9c8\ub85c \uc778\ud574 \ud544\ub9ac\ud540\uc5d0\uc11c \uc9d1\uacc4\ub41c \uc804\uccb4 \ud53c\ud574\uc561\uc740 273\uc5b5 \ud544\ub9ac\ud540 \ud398\uc18c, 6\uc5b5 8\ubc31\ub9cc\ub2ec\ub7ec\ub85c \ub098\ud0c0\ub098 1\ub144\uc804\uc5d0 \ud544\ub9ac\ud540\uc744 \uac15\ud0c0\ud558\uace0 \uc9c0\ub09c \ud0dc\ud48d \ud391\uc120\uc73c\ub85c \uc778\ud55c \ud53c\ud574\uc561 3\uc5b5 1\ubc31\ub9cc\ub2ec\ub7ec\uc758 \uc218\uce58\ub97c 2\ubc30 \uc774\uc0c1 \ub6f0\uc5b4\ub118\uc5b4 \uc7ac\uc0b0\ud53c\ud574\uc561 \ubd84\uc57c\uc758 1\uc704\uc5d0 \ub7ad\ud06c\ub418\uc5c8\ub2e4.", "answer": "\uce74\ud0c4\ub450\uc544\ub124\uc2a4 \uc8fc", "sentence": "\uce74\ud0c4\ub450\uc544\ub124\uc2a4 \uc8fc \uc5d0\uc11c\ub294 \uc804\uad6d\uc5d0\uc11c \ucd5c\ucd08\ub85c \ud0dc\ud48d\uacbd\ubcf4 \uc81c3\ub2e8\uacc4\uac00 \ubc1c\ub839\ub418\uc5c8\uace0 \uc8fc \uc548\uc5d0\uc11c\uc758 \uc804\uae30 \uacf5\uae09\uc774 \uc911\ub2e8\ub418\uc5c8\uace0 \ud1b5\uc2e0\uc774 \ub450\uc808\ub418\uc5c8\ub2e4.", "paragraph_sentence": "\ud0dc\ud48d \ud30c\ub9c8\uac00 \ud544\ub9ac\ud540 \ubd81\ubd80\uc5d0 \uc0c1\ub959\ud558\uae30 \ud55c\ucc38 \uc804\ubd80\ud130 \uc5ec\ub7ec \uad50\ud1b5\uc218\ub2e8\ub4e4\uc758 \uc6b4\ud589\uc774 \uc911\uc9c0\ub418\uc5c8\ub2e4. \uce74\ud0c4\ub450\uc544\ub124\uc2a4 \uc8fc \uc5d0\uc11c\ub294 \uc804\uad6d\uc5d0\uc11c \ucd5c\ucd08\ub85c \ud0dc\ud48d\uacbd\ubcf4 \uc81c3\ub2e8\uacc4\uac00 \ubc1c\ub839\ub418\uc5c8\uace0 \uc8fc \uc548\uc5d0\uc11c\uc758 \uc804\uae30 \uacf5\uae09\uc774 \uc911\ub2e8\ub418\uc5c8\uace0 \ud1b5\uc2e0\uc774 \ub450\uc808\ub418\uc5c8\ub2e4. \uc8fc\uc694 \uac04\uc120\ub3c4\ub85c\uc5d0 \ubf51\ud78c \ub098\ubb34\ub4e4\uc774 \uc313\uc5ec\uc788\uc5c8\ub2e4. \ub2f9\uc2dc \uc57d 3\ub9cc \uac00\uad6c\uac00 \ub300\ud53c\ub97c \uc644\ub8cc\ud588\ub2e4. \ube44\ucf5c \uc9c0\ubc29\uc5d0\uc11c\ub294 \uc5ec\uac1d\uc120 \uc2b9\uac1d 2,000\uba85\uc774 \ud56d\uad6c\uc5d0\uc11c \uace0\ub9bd\ub418\uc5c8\uace0 \ub9c8\ub2d0\ub77c \uc8fc\ubcc0 39\uac1c \uc8fc\uc5d0 \ud0dc\ud48d\uacbd\ubcf4\uac00 \ub0b4\ub824\uc84c\uace0 \uc774 \uc9c0\uc5ed\uc5d0\uc11c \uadf9\uc2ec\ud55c \ud3ed\uc6b0\uacfc \uac15\ud48d\uc744 \uacaa\uc5c8\ub2e4. \ud30c\ub9c8\ub294 \uce74\uac00\uc580 \uc8fc \ubd81\ub3d9\ubd80 \ud574\uc548\uc5d0 \uc0c1\ub959\ud588\uace0 \uc774 \uc8fc\uc5d0\uc11c \ub18d\uc5c5\ubd84\uc57c 2033\ub9cc \ud544\ub9ac\ud540 \ud398\uc18c\uc758 \uc7ac\uc0b0\ud53c\ud574\uac00 \ub0ac\ub2e4. \uce7c\ub9c1\uac00 \uc8fc\uc5d0\uc11c\ub294 \uc8fc\uc758 \uccad\uc0ac\uc640 \uc5f0\uacb0\ub418\ub294 \uae38\uc774 \uc0b0\uc0ac\ud0dc\ub85c \uc778\ud574 \ucc28\ub2e8\ub418\uc5c8\ub2e4. \uc0bc\ubc1c\ub808\uc2a4 \uc8fc\uc5d0\uc11c\ub294 \uc57d 2,100\uac00\uad6c\uac00 \ub300\ud53c\ud588\uace0 \ud321\uac00\uc2dc\ub09c \uc8fc\uc758 \ud55c \ub310\uacfc \ub204\uc5d0\ubc14\uc5d0\uc2dc\ud558 \uc8fc\uc758 \ud55c \ub310\uc774 \ud3ed\uc6b0\ub85c \uc778\ud574 \ub310 \uc548\uc758 \ubb3c\uc774 \ubc94\ub78c\uc704\uae30\uc5d0 \ucc98\ud558\uc790 \uc5c4\uccad\ub09c \uc591\uc758 \ubb3c\uc744 \ubc29\ub958\ud558\uae30 \uc2dc\uc791\ud588\ub2e4. \uc5c4\uccad\ub09c \uc591\uc758 \ubb3c\uc774 \ubc29\ub958\ud568\uc5d0 \ub530\ub77c \ud321\uac00\uc2dc\ub09c \uc8fc \ub3d9\ubd80\uc640 \uc911\ubd80, \ub204\uc5d0\ubcf4\uc5d0\uc2dc\ud558 \uc8fc\uc5d0 \ud64d\uc218\ub97c \uc77c\uc73c\ucf30\ub2e4. \ub9c8\ub2d0\ub77c \uc8fc\ubcc0 \uc9c0\uc5ed\uc5d0\uc11c\ub294 \uc5c4\uccad\ub09c \ud3ed\uc6b0\uc640 \uac15\ud48d\uc774 \ubd88\uc5b4\ub2e5\ucce4\uace0 \ud64d\uc218\ub85c \uc778\ud574 \uc218\uba74\uc774 \uc810\ucc28 \uc62c\ub77c\uac00\uae30 \uc2dc\uc791\ud588\ub2e4. \ubca0\uc774 \ud638\uc5d0\uc11c\ub294 90\ub144 \ub9cc\uc5d0 \ud638\uc218\uc758 \ubb3c\uc774 \uc5c4\uccad\ub09c \uc591\uc73c\ub85c \uc99d\uac00\ud588\uace0 \uc774\ub85c \uc778\ud574 \ub9c8\ub2d0\ub77c \ub3c4\uc2ec\uacfc \uadfc\uad50\uc9c0\uc5ed\uc774 \ubb3c\uc5d0 \uc7a0\uae38 \uc704\ud5d8\uc5d0 \ucc98\ud588\ub2e4. \ubcb5\uac8c\ud2b8 \uc8fc\uc5d0\uc11c\ub294 \uc0b0\uc0ac\ud0dc\uac00 \uc57d 200\uba85\uc744 \uc228\uc9c0\uac8c \ud588\uace0 \ub819\ud1a0\uc2a4\ud53c\ub77c\ubcd1\uc774 \ubc1c\uc0dd\ud574 \ub9ce\uc740 \uc0ac\ub78c\ub4e4\uc5d0\uac8c \uc601\ud5a5\uc744 \ub07c\ucce4\ub2e4. \ud0dc\ud48d \ud30c\ub9c8\ub85c \uc778\ud574 \ud544\ub9ac\ud540\uc5d0\uc11c \uc9d1\uacc4\ub41c \uc804\uccb4 \ud53c\ud574\uc561\uc740 273\uc5b5 \ud544\ub9ac\ud540 \ud398\uc18c, 6\uc5b5 8\ubc31\ub9cc\ub2ec\ub7ec\ub85c \ub098\ud0c0\ub098 1\ub144\uc804\uc5d0 \ud544\ub9ac\ud540\uc744 \uac15\ud0c0\ud558\uace0 \uc9c0\ub09c \ud0dc\ud48d \ud391\uc120\uc73c\ub85c \uc778\ud55c \ud53c\ud574\uc561 3\uc5b5 1\ubc31\ub9cc\ub2ec\ub7ec\uc758 \uc218\uce58\ub97c 2\ubc30 \uc774\uc0c1 \ub6f0\uc5b4\ub118\uc5b4 \uc7ac\uc0b0\ud53c\ud574\uc561 \ubd84\uc57c\uc758 1\uc704\uc5d0 \ub7ad\ud06c\ub418\uc5c8\ub2e4.", "paragraph_answer": "\ud0dc\ud48d \ud30c\ub9c8\uac00 \ud544\ub9ac\ud540 \ubd81\ubd80\uc5d0 \uc0c1\ub959\ud558\uae30 \ud55c\ucc38 \uc804\ubd80\ud130 \uc5ec\ub7ec \uad50\ud1b5\uc218\ub2e8\ub4e4\uc758 \uc6b4\ud589\uc774 \uc911\uc9c0\ub418\uc5c8\ub2e4. \uce74\ud0c4\ub450\uc544\ub124\uc2a4 \uc8fc \uc5d0\uc11c\ub294 \uc804\uad6d\uc5d0\uc11c \ucd5c\ucd08\ub85c \ud0dc\ud48d\uacbd\ubcf4 \uc81c3\ub2e8\uacc4\uac00 \ubc1c\ub839\ub418\uc5c8\uace0 \uc8fc \uc548\uc5d0\uc11c\uc758 \uc804\uae30 \uacf5\uae09\uc774 \uc911\ub2e8\ub418\uc5c8\uace0 \ud1b5\uc2e0\uc774 \ub450\uc808\ub418\uc5c8\ub2e4. \uc8fc\uc694 \uac04\uc120\ub3c4\ub85c\uc5d0 \ubf51\ud78c \ub098\ubb34\ub4e4\uc774 \uc313\uc5ec\uc788\uc5c8\ub2e4. \ub2f9\uc2dc \uc57d 3\ub9cc \uac00\uad6c\uac00 \ub300\ud53c\ub97c \uc644\ub8cc\ud588\ub2e4. \ube44\ucf5c \uc9c0\ubc29\uc5d0\uc11c\ub294 \uc5ec\uac1d\uc120 \uc2b9\uac1d 2,000\uba85\uc774 \ud56d\uad6c\uc5d0\uc11c \uace0\ub9bd\ub418\uc5c8\uace0 \ub9c8\ub2d0\ub77c \uc8fc\ubcc0 39\uac1c \uc8fc\uc5d0 \ud0dc\ud48d\uacbd\ubcf4\uac00 \ub0b4\ub824\uc84c\uace0 \uc774 \uc9c0\uc5ed\uc5d0\uc11c \uadf9\uc2ec\ud55c \ud3ed\uc6b0\uacfc \uac15\ud48d\uc744 \uacaa\uc5c8\ub2e4. \ud30c\ub9c8\ub294 \uce74\uac00\uc580 \uc8fc \ubd81\ub3d9\ubd80 \ud574\uc548\uc5d0 \uc0c1\ub959\ud588\uace0 \uc774 \uc8fc\uc5d0\uc11c \ub18d\uc5c5\ubd84\uc57c 2033\ub9cc \ud544\ub9ac\ud540 \ud398\uc18c\uc758 \uc7ac\uc0b0\ud53c\ud574\uac00 \ub0ac\ub2e4. \uce7c\ub9c1\uac00 \uc8fc\uc5d0\uc11c\ub294 \uc8fc\uc758 \uccad\uc0ac\uc640 \uc5f0\uacb0\ub418\ub294 \uae38\uc774 \uc0b0\uc0ac\ud0dc\ub85c \uc778\ud574 \ucc28\ub2e8\ub418\uc5c8\ub2e4. \uc0bc\ubc1c\ub808\uc2a4 \uc8fc\uc5d0\uc11c\ub294 \uc57d 2,100\uac00\uad6c\uac00 \ub300\ud53c\ud588\uace0 \ud321\uac00\uc2dc\ub09c \uc8fc\uc758 \ud55c \ub310\uacfc \ub204\uc5d0\ubc14\uc5d0\uc2dc\ud558 \uc8fc\uc758 \ud55c \ub310\uc774 \ud3ed\uc6b0\ub85c \uc778\ud574 \ub310 \uc548\uc758 \ubb3c\uc774 \ubc94\ub78c\uc704\uae30\uc5d0 \ucc98\ud558\uc790 \uc5c4\uccad\ub09c \uc591\uc758 \ubb3c\uc744 \ubc29\ub958\ud558\uae30 \uc2dc\uc791\ud588\ub2e4. \uc5c4\uccad\ub09c \uc591\uc758 \ubb3c\uc774 \ubc29\ub958\ud568\uc5d0 \ub530\ub77c \ud321\uac00\uc2dc\ub09c \uc8fc \ub3d9\ubd80\uc640 \uc911\ubd80, \ub204\uc5d0\ubcf4\uc5d0\uc2dc\ud558 \uc8fc\uc5d0 \ud64d\uc218\ub97c \uc77c\uc73c\ucf30\ub2e4. \ub9c8\ub2d0\ub77c \uc8fc\ubcc0 \uc9c0\uc5ed\uc5d0\uc11c\ub294 \uc5c4\uccad\ub09c \ud3ed\uc6b0\uc640 \uac15\ud48d\uc774 \ubd88\uc5b4\ub2e5\ucce4\uace0 \ud64d\uc218\ub85c \uc778\ud574 \uc218\uba74\uc774 \uc810\ucc28 \uc62c\ub77c\uac00\uae30 \uc2dc\uc791\ud588\ub2e4. \ubca0\uc774 \ud638\uc5d0\uc11c\ub294 90\ub144 \ub9cc\uc5d0 \ud638\uc218\uc758 \ubb3c\uc774 \uc5c4\uccad\ub09c \uc591\uc73c\ub85c \uc99d\uac00\ud588\uace0 \uc774\ub85c \uc778\ud574 \ub9c8\ub2d0\ub77c \ub3c4\uc2ec\uacfc \uadfc\uad50\uc9c0\uc5ed\uc774 \ubb3c\uc5d0 \uc7a0\uae38 \uc704\ud5d8\uc5d0 \ucc98\ud588\ub2e4. \ubcb5\uac8c\ud2b8 \uc8fc\uc5d0\uc11c\ub294 \uc0b0\uc0ac\ud0dc\uac00 \uc57d 200\uba85\uc744 \uc228\uc9c0\uac8c \ud588\uace0 \ub819\ud1a0\uc2a4\ud53c\ub77c\ubcd1\uc774 \ubc1c\uc0dd\ud574 \ub9ce\uc740 \uc0ac\ub78c\ub4e4\uc5d0\uac8c \uc601\ud5a5\uc744 \ub07c\ucce4\ub2e4. \ud0dc\ud48d \ud30c\ub9c8\ub85c \uc778\ud574 \ud544\ub9ac\ud540\uc5d0\uc11c \uc9d1\uacc4\ub41c \uc804\uccb4 \ud53c\ud574\uc561\uc740 273\uc5b5 \ud544\ub9ac\ud540 \ud398\uc18c, 6\uc5b5 8\ubc31\ub9cc\ub2ec\ub7ec\ub85c \ub098\ud0c0\ub098 1\ub144\uc804\uc5d0 \ud544\ub9ac\ud540\uc744 \uac15\ud0c0\ud558\uace0 \uc9c0\ub09c \ud0dc\ud48d \ud391\uc120\uc73c\ub85c \uc778\ud55c \ud53c\ud574\uc561 3\uc5b5 1\ubc31\ub9cc\ub2ec\ub7ec\uc758 \uc218\uce58\ub97c 2\ubc30 \uc774\uc0c1 \ub6f0\uc5b4\ub118\uc5b4 \uc7ac\uc0b0\ud53c\ud574\uc561 \ubd84\uc57c\uc758 1\uc704\uc5d0 \ub7ad\ud06c\ub418\uc5c8\ub2e4.", "sentence_answer": " \uce74\ud0c4\ub450\uc544\ub124\uc2a4 \uc8fc \uc5d0\uc11c\ub294 \uc804\uad6d\uc5d0\uc11c \ucd5c\ucd08\ub85c \ud0dc\ud48d\uacbd\ubcf4 \uc81c3\ub2e8\uacc4\uac00 \ubc1c\ub839\ub418\uc5c8\uace0 \uc8fc \uc548\uc5d0\uc11c\uc758 \uc804\uae30 \uacf5\uae09\uc774 \uc911\ub2e8\ub418\uc5c8\uace0 \ud1b5\uc2e0\uc774 \ub450\uc808\ub418\uc5c8\ub2e4.", "paragraph_id": "6565447-2-0", "paragraph_question": "question: \ud0dc\ud48d \ud30c\ub9c8\ub85c \uc778\ud574 \ucd5c\ucd08\ub85c \ud0dc\ud48d\uacbd\ubcf4 \uc81c3\ub2e8\uacc4\uac00 \ubc1c\ub839\ub41c \ud544\ub9ac\ud540 \uc9c0\uc5ed\uc740 \uc5b4\ub514\uc778\uac00?, context: \ud0dc\ud48d \ud30c\ub9c8\uac00 \ud544\ub9ac\ud540 \ubd81\ubd80\uc5d0 \uc0c1\ub959\ud558\uae30 \ud55c\ucc38 \uc804\ubd80\ud130 \uc5ec\ub7ec \uad50\ud1b5\uc218\ub2e8\ub4e4\uc758 \uc6b4\ud589\uc774 \uc911\uc9c0\ub418\uc5c8\ub2e4. \uce74\ud0c4\ub450\uc544\ub124\uc2a4 \uc8fc\uc5d0\uc11c\ub294 \uc804\uad6d\uc5d0\uc11c \ucd5c\ucd08\ub85c \ud0dc\ud48d\uacbd\ubcf4 \uc81c3\ub2e8\uacc4\uac00 \ubc1c\ub839\ub418\uc5c8\uace0 \uc8fc \uc548\uc5d0\uc11c\uc758 \uc804\uae30 \uacf5\uae09\uc774 \uc911\ub2e8\ub418\uc5c8\uace0 \ud1b5\uc2e0\uc774 \ub450\uc808\ub418\uc5c8\ub2e4. \uc8fc\uc694 \uac04\uc120\ub3c4\ub85c\uc5d0 \ubf51\ud78c \ub098\ubb34\ub4e4\uc774 \uc313\uc5ec\uc788\uc5c8\ub2e4. \ub2f9\uc2dc \uc57d 3\ub9cc \uac00\uad6c\uac00 \ub300\ud53c\ub97c \uc644\ub8cc\ud588\ub2e4. \ube44\ucf5c \uc9c0\ubc29\uc5d0\uc11c\ub294 \uc5ec\uac1d\uc120 \uc2b9\uac1d 2,000\uba85\uc774 \ud56d\uad6c\uc5d0\uc11c \uace0\ub9bd\ub418\uc5c8\uace0 \ub9c8\ub2d0\ub77c \uc8fc\ubcc0 39\uac1c \uc8fc\uc5d0 \ud0dc\ud48d\uacbd\ubcf4\uac00 \ub0b4\ub824\uc84c\uace0 \uc774 \uc9c0\uc5ed\uc5d0\uc11c \uadf9\uc2ec\ud55c \ud3ed\uc6b0\uacfc \uac15\ud48d\uc744 \uacaa\uc5c8\ub2e4. \ud30c\ub9c8\ub294 \uce74\uac00\uc580 \uc8fc \ubd81\ub3d9\ubd80 \ud574\uc548\uc5d0 \uc0c1\ub959\ud588\uace0 \uc774 \uc8fc\uc5d0\uc11c \ub18d\uc5c5\ubd84\uc57c 2033\ub9cc \ud544\ub9ac\ud540 \ud398\uc18c\uc758 \uc7ac\uc0b0\ud53c\ud574\uac00 \ub0ac\ub2e4. \uce7c\ub9c1\uac00 \uc8fc\uc5d0\uc11c\ub294 \uc8fc\uc758 \uccad\uc0ac\uc640 \uc5f0\uacb0\ub418\ub294 \uae38\uc774 \uc0b0\uc0ac\ud0dc\ub85c \uc778\ud574 \ucc28\ub2e8\ub418\uc5c8\ub2e4. \uc0bc\ubc1c\ub808\uc2a4 \uc8fc\uc5d0\uc11c\ub294 \uc57d 2,100\uac00\uad6c\uac00 \ub300\ud53c\ud588\uace0 \ud321\uac00\uc2dc\ub09c \uc8fc\uc758 \ud55c \ub310\uacfc \ub204\uc5d0\ubc14\uc5d0\uc2dc\ud558 \uc8fc\uc758 \ud55c \ub310\uc774 \ud3ed\uc6b0\ub85c \uc778\ud574 \ub310 \uc548\uc758 \ubb3c\uc774 \ubc94\ub78c\uc704\uae30\uc5d0 \ucc98\ud558\uc790 \uc5c4\uccad\ub09c \uc591\uc758 \ubb3c\uc744 \ubc29\ub958\ud558\uae30 \uc2dc\uc791\ud588\ub2e4. \uc5c4\uccad\ub09c \uc591\uc758 \ubb3c\uc774 \ubc29\ub958\ud568\uc5d0 \ub530\ub77c \ud321\uac00\uc2dc\ub09c \uc8fc \ub3d9\ubd80\uc640 \uc911\ubd80, \ub204\uc5d0\ubcf4\uc5d0\uc2dc\ud558 \uc8fc\uc5d0 \ud64d\uc218\ub97c \uc77c\uc73c\ucf30\ub2e4. \ub9c8\ub2d0\ub77c \uc8fc\ubcc0 \uc9c0\uc5ed\uc5d0\uc11c\ub294 \uc5c4\uccad\ub09c \ud3ed\uc6b0\uc640 \uac15\ud48d\uc774 \ubd88\uc5b4\ub2e5\ucce4\uace0 \ud64d\uc218\ub85c \uc778\ud574 \uc218\uba74\uc774 \uc810\ucc28 \uc62c\ub77c\uac00\uae30 \uc2dc\uc791\ud588\ub2e4. \ubca0\uc774 \ud638\uc5d0\uc11c\ub294 90\ub144 \ub9cc\uc5d0 \ud638\uc218\uc758 \ubb3c\uc774 \uc5c4\uccad\ub09c \uc591\uc73c\ub85c \uc99d\uac00\ud588\uace0 \uc774\ub85c \uc778\ud574 \ub9c8\ub2d0\ub77c \ub3c4\uc2ec\uacfc \uadfc\uad50\uc9c0\uc5ed\uc774 \ubb3c\uc5d0 \uc7a0\uae38 \uc704\ud5d8\uc5d0 \ucc98\ud588\ub2e4. \ubcb5\uac8c\ud2b8 \uc8fc\uc5d0\uc11c\ub294 \uc0b0\uc0ac\ud0dc\uac00 \uc57d 200\uba85\uc744 \uc228\uc9c0\uac8c \ud588\uace0 \ub819\ud1a0\uc2a4\ud53c\ub77c\ubcd1\uc774 \ubc1c\uc0dd\ud574 \ub9ce\uc740 \uc0ac\ub78c\ub4e4\uc5d0\uac8c \uc601\ud5a5\uc744 \ub07c\ucce4\ub2e4. \ud0dc\ud48d \ud30c\ub9c8\ub85c \uc778\ud574 \ud544\ub9ac\ud540\uc5d0\uc11c \uc9d1\uacc4\ub41c \uc804\uccb4 \ud53c\ud574\uc561\uc740 273\uc5b5 \ud544\ub9ac\ud540 \ud398\uc18c, 6\uc5b5 8\ubc31\ub9cc\ub2ec\ub7ec\ub85c \ub098\ud0c0\ub098 1\ub144\uc804\uc5d0 \ud544\ub9ac\ud540\uc744 \uac15\ud0c0\ud558\uace0 \uc9c0\ub09c \ud0dc\ud48d \ud391\uc120\uc73c\ub85c \uc778\ud55c \ud53c\ud574\uc561 3\uc5b5 1\ubc31\ub9cc\ub2ec\ub7ec\uc758 \uc218\uce58\ub97c 2\ubc30 \uc774\uc0c1 \ub6f0\uc5b4\ub118\uc5b4 \uc7ac\uc0b0\ud53c\ud574\uc561 \ubd84\uc57c\uc758 1\uc704\uc5d0 \ub7ad\ud06c\ub418\uc5c8\ub2e4."} {"question": "\ubb38\uc885\uc774 \uacbd\ud61c\uacf5\uc8fc\uc758 \uc0b4\ub9bc\uc9d1\uc744 \ucc28\ub824\uc900\uacf3\uc740 \uc5b4\ub514\uc778\uac00?", "paragraph": "\uc774\ud6c4 \ubb38\uc885\uc758 \uc989\uc704\ub85c \uacbd\ud61c\uacf5\uc8fc\ub294 \uacf5\uc8fc\ub85c \ucc45\ubd09\ub418\uc5c8\uace0, \ubb38\uc885\uc740 \ub538\uc758 \uc0b4\ub9bc\uc9d1\uc744 \ub2f9\uc2dc\uc758 \ub300\ud45c\uc801 \ubd80\uc790\ub3d9\ub124\uc600\ub358 \uc591\ub355\ubc29(\u967d\u5fb7\u574a) \ud5a5\uad50\ub3d9\uc5d0 \ucc28\ub824\uc8fc\uc5c8\ub2e4. \ub2f9\uc2dc \uc0ac\ud5cc\ubd80 \uc9c0\ud3c9(\u53f8\u61b2\u5e9c\u6301\u5e73) \uc724\uba74(\u5c39\u6c94), \ubb38\uc5ec\ub7c9(\u6587\u6c5d\u826f) \ub4f1\uc774 \uacf5\uc8fc\uc640 \ubd80\ub9c8\ub97c \uc704\ud574 \uc0c8\ub85c\uc774 \uc9d3\ub294 \uc9d1 \ub54c\ubb38\uc5d0 30\uc5ec\ucc44\uc758 \ubbfc\uac00\uac00 \ucca0\uac70\ub41c\ub2e4\uace0 \uc544\ub8b0\uc5c8\ub358 \uae30\ub85d\uc774 \uc788\ub2e4. \uc774\ud6c4 \uc5bc\ub9c8 \uc9c0\ub098\uc9c0 \uc54a\uc544 \ubb38\uc885\uc774 \uc2b9\ud558\ud558\uace0 \uacbd\ud61c\uacf5\uc8fc\uc758 \ub3d9\uc0dd \ub2e8\uc885\uc774 \uc989\uc704\ud588\ub2e4. \ub2e8\uc885\uc740 \uc790\uc8fc \ud5a5\uad50\ub3d9 \uacbd\ud61c\uacf5\uc8fc\uc758 \uc9d1\uc744 \ucc3e\uc558\uace0 \uacc4\uc720\uc815\ub09c\uc774 \uc788\ub358 \ub0a0 \ubc24\uc5d0\ub3c4 \uacbd\ud61c\uacf5\uc8fc\uc758 \uc9d1\uc5d0\uc11c \uc790\uace0 \uc788\uc5c8\ub2e4. \uacbd\ud61c\uacf5\uc8fc\uc758 \ub0a8\ud3b8 \uc815\uc885\uc740 \uae08\uc131\ub300\uad70 \uc0ac\uac74\uc5d0 \uc5f0\ub8e8\ub418\uc5b4 \uac15\uc6d0\ub3c4 \uc601\uc6d4\ub85c \uc720\ubc30\ub418\uc5c8\ub2e4\uac00 \uacbd\uae30\ub3c4 \uc591\uadfc\uc73c\ub85c \uc62e\uaca8\uc84c\ub2e4. \uacf5\uc8fc\uac00 \ubcd1\uc774 \ub098\uc790 \ub2e8\uc885\uc740 \uc138\uc870\uc5d0\uac8c \uc0ac\ub78c\uc744 \ubcf4\ub0b4\uc5b4 \uc815\uc885\uc744 \ud55c\uc591\uc73c\ub85c \ubd88\ub7ec\ub4e4\uc774\ub77c\uace0 \uc77c\ub800\uace0 \uc138\uc870\ub3c4 \ubb38\uc885\uc758 \uc720\uc77c\ud55c \uc0ac\uc704\ub77c \ud558\uba70 \uc815\uc885\uc744 \uc7a0\uc2dc \ub3c4\uc74d\uc73c\ub85c \ub3cc\uc544\uc624\uac8c \ud588\ub2e4. \uadf8\ub7ec\ub098 \uc0ac\uac04\uc6d0\uc758 \uc0c1\uc18c\ub85c \uc815\uc885\uc740 \uacf5\uc8fc\uc758 \ubcd1\uc774 \ub0ab\uc790 \ub2e4\uc2dc \uc720\ubc30\ub418\uc5c8\ub2e4. \uacf5\uc8fc\ub294 \uc774\ub54c \uc815\uc885\uc758 \uc720\ubc30\uc9c0\uc778 \uc218\uc6d0\uc73c\ub85c \ud5a5\ud588\uace0 \ub098\uc911\uc5d0 \ud1b5\uc9c4(\u901a\u6d25)\uc744 \uac70\uccd0 \uc804\ub77c\ub3c4 \uad11\uc8fc(\u5149\u5dde)\uae4c\uc9c0 \ub530\ub77c\uac14\ub2e4. \uc774\ud6c4 \uc815\uc885\uc740 \uc2b9\ub824 \uc131\ud0c4 \ub4f1\uacfc \uacb0\ud0c1\ud574 \ubaa8\ubc18\uc744 \uaf80\ud588\ub2e4\ub294 \ud610\uc758\ub85c \uac70\uc5f4\ud615\uc744 \ub2f9\ud558\uc600\ub2e4. \ub2f9\uc2dc \uacbd\ud61c\uacf5\uc8fc\ub294 \uc784\uc2e0 \uc911\uc774\uc5c8\ub2e4. \u300a\uc5f0\ub824\uc2e4\uae30\uc220\u300b\uc5d0\uc11c\ub294 \uacbd\ud61c\uacf5\uc8fc \ub610\ud55c \ub0a8\ud3b8\uc758 \uc8c4\uc5d0 \uc5f0\uc88c\ub418\uc5b4 \uac00\uc0b0\uc774 \uc801\ubab0\ub418\uace0 \uc720\ubc30\ub418\uc5b4 \uc21c\ucc9c\uc758 \uad00\ube44\uac00 \ub418\uc5c8\ub2e4\uace0 \uae30\ub85d\ud558\uace0 \uc788\ub2e4.", "answer": "\uc591\ub355\ubc29(\u967d\u5fb7\u574a) \ud5a5\uad50\ub3d9", "sentence": "\ub2f9\uc2dc\uc758 \ub300\ud45c\uc801 \ubd80\uc790\ub3d9\ub124\uc600\ub358 \uc591\ub355\ubc29(\u967d\u5fb7\u574a) \ud5a5\uad50\ub3d9 \uc5d0 \ucc28\ub824\uc8fc\uc5c8\ub2e4.", "paragraph_sentence": "\uc774\ud6c4 \ubb38\uc885\uc758 \uc989\uc704\ub85c \uacbd\ud61c\uacf5\uc8fc\ub294 \uacf5\uc8fc\ub85c \ucc45\ubd09\ub418\uc5c8\uace0, \ubb38\uc885\uc740 \ub538\uc758 \uc0b4\ub9bc\uc9d1\uc744 \ub2f9\uc2dc\uc758 \ub300\ud45c\uc801 \ubd80\uc790\ub3d9\ub124\uc600\ub358 \uc591\ub355\ubc29(\u967d\u5fb7\u574a) \ud5a5\uad50\ub3d9 \uc5d0 \ucc28\ub824\uc8fc\uc5c8\ub2e4. \ub2f9\uc2dc \uc0ac\ud5cc\ubd80 \uc9c0\ud3c9(\u53f8\u61b2\u5e9c\u6301\u5e73) \uc724\uba74(\u5c39\u6c94), \ubb38\uc5ec\ub7c9(\u6587\u6c5d\u826f) \ub4f1\uc774 \uacf5\uc8fc\uc640 \ubd80\ub9c8\ub97c \uc704\ud574 \uc0c8\ub85c\uc774 \uc9d3\ub294 \uc9d1 \ub54c\ubb38\uc5d0 30\uc5ec\ucc44\uc758 \ubbfc\uac00\uac00 \ucca0\uac70\ub41c\ub2e4\uace0 \uc544\ub8b0\uc5c8\ub358 \uae30\ub85d\uc774 \uc788\ub2e4. \uc774\ud6c4 \uc5bc\ub9c8 \uc9c0\ub098\uc9c0 \uc54a\uc544 \ubb38\uc885\uc774 \uc2b9\ud558\ud558\uace0 \uacbd\ud61c\uacf5\uc8fc\uc758 \ub3d9\uc0dd \ub2e8\uc885\uc774 \uc989\uc704\ud588\ub2e4. \ub2e8\uc885\uc740 \uc790\uc8fc \ud5a5\uad50\ub3d9 \uacbd\ud61c\uacf5\uc8fc\uc758 \uc9d1\uc744 \ucc3e\uc558\uace0 \uacc4\uc720\uc815\ub09c\uc774 \uc788\ub358 \ub0a0 \ubc24\uc5d0\ub3c4 \uacbd\ud61c\uacf5\uc8fc\uc758 \uc9d1\uc5d0\uc11c \uc790\uace0 \uc788\uc5c8\ub2e4. \uacbd\ud61c\uacf5\uc8fc\uc758 \ub0a8\ud3b8 \uc815\uc885\uc740 \uae08\uc131\ub300\uad70 \uc0ac\uac74\uc5d0 \uc5f0\ub8e8\ub418\uc5b4 \uac15\uc6d0\ub3c4 \uc601\uc6d4\ub85c \uc720\ubc30\ub418\uc5c8\ub2e4\uac00 \uacbd\uae30\ub3c4 \uc591\uadfc\uc73c\ub85c \uc62e\uaca8\uc84c\ub2e4. \uacf5\uc8fc\uac00 \ubcd1\uc774 \ub098\uc790 \ub2e8\uc885\uc740 \uc138\uc870\uc5d0\uac8c \uc0ac\ub78c\uc744 \ubcf4\ub0b4\uc5b4 \uc815\uc885\uc744 \ud55c\uc591\uc73c\ub85c \ubd88\ub7ec\ub4e4\uc774\ub77c\uace0 \uc77c\ub800\uace0 \uc138\uc870\ub3c4 \ubb38\uc885\uc758 \uc720\uc77c\ud55c \uc0ac\uc704\ub77c \ud558\uba70 \uc815\uc885\uc744 \uc7a0\uc2dc \ub3c4\uc74d\uc73c\ub85c \ub3cc\uc544\uc624\uac8c \ud588\ub2e4. \uadf8\ub7ec\ub098 \uc0ac\uac04\uc6d0\uc758 \uc0c1\uc18c\ub85c \uc815\uc885\uc740 \uacf5\uc8fc\uc758 \ubcd1\uc774 \ub0ab\uc790 \ub2e4\uc2dc \uc720\ubc30\ub418\uc5c8\ub2e4. \uacf5\uc8fc\ub294 \uc774\ub54c \uc815\uc885\uc758 \uc720\ubc30\uc9c0\uc778 \uc218\uc6d0\uc73c\ub85c \ud5a5\ud588\uace0 \ub098\uc911\uc5d0 \ud1b5\uc9c4(\u901a\u6d25)\uc744 \uac70\uccd0 \uc804\ub77c\ub3c4 \uad11\uc8fc(\u5149\u5dde)\uae4c\uc9c0 \ub530\ub77c\uac14\ub2e4. \uc774\ud6c4 \uc815\uc885\uc740 \uc2b9\ub824 \uc131\ud0c4 \ub4f1\uacfc \uacb0\ud0c1\ud574 \ubaa8\ubc18\uc744 \uaf80\ud588\ub2e4\ub294 \ud610\uc758\ub85c \uac70\uc5f4\ud615\uc744 \ub2f9\ud558\uc600\ub2e4. \ub2f9\uc2dc \uacbd\ud61c\uacf5\uc8fc\ub294 \uc784\uc2e0 \uc911\uc774\uc5c8\ub2e4. \u300a\uc5f0\ub824\uc2e4\uae30\uc220\u300b\uc5d0\uc11c\ub294 \uacbd\ud61c\uacf5\uc8fc \ub610\ud55c \ub0a8\ud3b8\uc758 \uc8c4\uc5d0 \uc5f0\uc88c\ub418\uc5b4 \uac00\uc0b0\uc774 \uc801\ubab0\ub418\uace0 \uc720\ubc30\ub418\uc5b4 \uc21c\ucc9c\uc758 \uad00\ube44\uac00 \ub418\uc5c8\ub2e4\uace0 \uae30\ub85d\ud558\uace0 \uc788\ub2e4.", "paragraph_answer": "\uc774\ud6c4 \ubb38\uc885\uc758 \uc989\uc704\ub85c \uacbd\ud61c\uacf5\uc8fc\ub294 \uacf5\uc8fc\ub85c \ucc45\ubd09\ub418\uc5c8\uace0, \ubb38\uc885\uc740 \ub538\uc758 \uc0b4\ub9bc\uc9d1\uc744 \ub2f9\uc2dc\uc758 \ub300\ud45c\uc801 \ubd80\uc790\ub3d9\ub124\uc600\ub358 \uc591\ub355\ubc29(\u967d\u5fb7\u574a) \ud5a5\uad50\ub3d9 \uc5d0 \ucc28\ub824\uc8fc\uc5c8\ub2e4. \ub2f9\uc2dc \uc0ac\ud5cc\ubd80 \uc9c0\ud3c9(\u53f8\u61b2\u5e9c\u6301\u5e73) \uc724\uba74(\u5c39\u6c94), \ubb38\uc5ec\ub7c9(\u6587\u6c5d\u826f) \ub4f1\uc774 \uacf5\uc8fc\uc640 \ubd80\ub9c8\ub97c \uc704\ud574 \uc0c8\ub85c\uc774 \uc9d3\ub294 \uc9d1 \ub54c\ubb38\uc5d0 30\uc5ec\ucc44\uc758 \ubbfc\uac00\uac00 \ucca0\uac70\ub41c\ub2e4\uace0 \uc544\ub8b0\uc5c8\ub358 \uae30\ub85d\uc774 \uc788\ub2e4. \uc774\ud6c4 \uc5bc\ub9c8 \uc9c0\ub098\uc9c0 \uc54a\uc544 \ubb38\uc885\uc774 \uc2b9\ud558\ud558\uace0 \uacbd\ud61c\uacf5\uc8fc\uc758 \ub3d9\uc0dd \ub2e8\uc885\uc774 \uc989\uc704\ud588\ub2e4. \ub2e8\uc885\uc740 \uc790\uc8fc \ud5a5\uad50\ub3d9 \uacbd\ud61c\uacf5\uc8fc\uc758 \uc9d1\uc744 \ucc3e\uc558\uace0 \uacc4\uc720\uc815\ub09c\uc774 \uc788\ub358 \ub0a0 \ubc24\uc5d0\ub3c4 \uacbd\ud61c\uacf5\uc8fc\uc758 \uc9d1\uc5d0\uc11c \uc790\uace0 \uc788\uc5c8\ub2e4. \uacbd\ud61c\uacf5\uc8fc\uc758 \ub0a8\ud3b8 \uc815\uc885\uc740 \uae08\uc131\ub300\uad70 \uc0ac\uac74\uc5d0 \uc5f0\ub8e8\ub418\uc5b4 \uac15\uc6d0\ub3c4 \uc601\uc6d4\ub85c \uc720\ubc30\ub418\uc5c8\ub2e4\uac00 \uacbd\uae30\ub3c4 \uc591\uadfc\uc73c\ub85c \uc62e\uaca8\uc84c\ub2e4. \uacf5\uc8fc\uac00 \ubcd1\uc774 \ub098\uc790 \ub2e8\uc885\uc740 \uc138\uc870\uc5d0\uac8c \uc0ac\ub78c\uc744 \ubcf4\ub0b4\uc5b4 \uc815\uc885\uc744 \ud55c\uc591\uc73c\ub85c \ubd88\ub7ec\ub4e4\uc774\ub77c\uace0 \uc77c\ub800\uace0 \uc138\uc870\ub3c4 \ubb38\uc885\uc758 \uc720\uc77c\ud55c \uc0ac\uc704\ub77c \ud558\uba70 \uc815\uc885\uc744 \uc7a0\uc2dc \ub3c4\uc74d\uc73c\ub85c \ub3cc\uc544\uc624\uac8c \ud588\ub2e4. \uadf8\ub7ec\ub098 \uc0ac\uac04\uc6d0\uc758 \uc0c1\uc18c\ub85c \uc815\uc885\uc740 \uacf5\uc8fc\uc758 \ubcd1\uc774 \ub0ab\uc790 \ub2e4\uc2dc \uc720\ubc30\ub418\uc5c8\ub2e4. \uacf5\uc8fc\ub294 \uc774\ub54c \uc815\uc885\uc758 \uc720\ubc30\uc9c0\uc778 \uc218\uc6d0\uc73c\ub85c \ud5a5\ud588\uace0 \ub098\uc911\uc5d0 \ud1b5\uc9c4(\u901a\u6d25)\uc744 \uac70\uccd0 \uc804\ub77c\ub3c4 \uad11\uc8fc(\u5149\u5dde)\uae4c\uc9c0 \ub530\ub77c\uac14\ub2e4. \uc774\ud6c4 \uc815\uc885\uc740 \uc2b9\ub824 \uc131\ud0c4 \ub4f1\uacfc \uacb0\ud0c1\ud574 \ubaa8\ubc18\uc744 \uaf80\ud588\ub2e4\ub294 \ud610\uc758\ub85c \uac70\uc5f4\ud615\uc744 \ub2f9\ud558\uc600\ub2e4. \ub2f9\uc2dc \uacbd\ud61c\uacf5\uc8fc\ub294 \uc784\uc2e0 \uc911\uc774\uc5c8\ub2e4. \u300a\uc5f0\ub824\uc2e4\uae30\uc220\u300b\uc5d0\uc11c\ub294 \uacbd\ud61c\uacf5\uc8fc \ub610\ud55c \ub0a8\ud3b8\uc758 \uc8c4\uc5d0 \uc5f0\uc88c\ub418\uc5b4 \uac00\uc0b0\uc774 \uc801\ubab0\ub418\uace0 \uc720\ubc30\ub418\uc5b4 \uc21c\ucc9c\uc758 \uad00\ube44\uac00 \ub418\uc5c8\ub2e4\uace0 \uae30\ub85d\ud558\uace0 \uc788\ub2e4.", "sentence_answer": "\ub2f9\uc2dc\uc758 \ub300\ud45c\uc801 \ubd80\uc790\ub3d9\ub124\uc600\ub358 \uc591\ub355\ubc29(\u967d\u5fb7\u574a) \ud5a5\uad50\ub3d9 \uc5d0 \ucc28\ub824\uc8fc\uc5c8\ub2e4.", "paragraph_id": "6479067-1-0", "paragraph_question": "question: \ubb38\uc885\uc774 \uacbd\ud61c\uacf5\uc8fc\uc758 \uc0b4\ub9bc\uc9d1\uc744 \ucc28\ub824\uc900\uacf3\uc740 \uc5b4\ub514\uc778\uac00?, context: \uc774\ud6c4 \ubb38\uc885\uc758 \uc989\uc704\ub85c \uacbd\ud61c\uacf5\uc8fc\ub294 \uacf5\uc8fc\ub85c \ucc45\ubd09\ub418\uc5c8\uace0, \ubb38\uc885\uc740 \ub538\uc758 \uc0b4\ub9bc\uc9d1\uc744 \ub2f9\uc2dc\uc758 \ub300\ud45c\uc801 \ubd80\uc790\ub3d9\ub124\uc600\ub358 \uc591\ub355\ubc29(\u967d\u5fb7\u574a) \ud5a5\uad50\ub3d9\uc5d0 \ucc28\ub824\uc8fc\uc5c8\ub2e4. \ub2f9\uc2dc \uc0ac\ud5cc\ubd80 \uc9c0\ud3c9(\u53f8\u61b2\u5e9c\u6301\u5e73) \uc724\uba74(\u5c39\u6c94), \ubb38\uc5ec\ub7c9(\u6587\u6c5d\u826f) \ub4f1\uc774 \uacf5\uc8fc\uc640 \ubd80\ub9c8\ub97c \uc704\ud574 \uc0c8\ub85c\uc774 \uc9d3\ub294 \uc9d1 \ub54c\ubb38\uc5d0 30\uc5ec\ucc44\uc758 \ubbfc\uac00\uac00 \ucca0\uac70\ub41c\ub2e4\uace0 \uc544\ub8b0\uc5c8\ub358 \uae30\ub85d\uc774 \uc788\ub2e4. \uc774\ud6c4 \uc5bc\ub9c8 \uc9c0\ub098\uc9c0 \uc54a\uc544 \ubb38\uc885\uc774 \uc2b9\ud558\ud558\uace0 \uacbd\ud61c\uacf5\uc8fc\uc758 \ub3d9\uc0dd \ub2e8\uc885\uc774 \uc989\uc704\ud588\ub2e4. \ub2e8\uc885\uc740 \uc790\uc8fc \ud5a5\uad50\ub3d9 \uacbd\ud61c\uacf5\uc8fc\uc758 \uc9d1\uc744 \ucc3e\uc558\uace0 \uacc4\uc720\uc815\ub09c\uc774 \uc788\ub358 \ub0a0 \ubc24\uc5d0\ub3c4 \uacbd\ud61c\uacf5\uc8fc\uc758 \uc9d1\uc5d0\uc11c \uc790\uace0 \uc788\uc5c8\ub2e4. \uacbd\ud61c\uacf5\uc8fc\uc758 \ub0a8\ud3b8 \uc815\uc885\uc740 \uae08\uc131\ub300\uad70 \uc0ac\uac74\uc5d0 \uc5f0\ub8e8\ub418\uc5b4 \uac15\uc6d0\ub3c4 \uc601\uc6d4\ub85c \uc720\ubc30\ub418\uc5c8\ub2e4\uac00 \uacbd\uae30\ub3c4 \uc591\uadfc\uc73c\ub85c \uc62e\uaca8\uc84c\ub2e4. \uacf5\uc8fc\uac00 \ubcd1\uc774 \ub098\uc790 \ub2e8\uc885\uc740 \uc138\uc870\uc5d0\uac8c \uc0ac\ub78c\uc744 \ubcf4\ub0b4\uc5b4 \uc815\uc885\uc744 \ud55c\uc591\uc73c\ub85c \ubd88\ub7ec\ub4e4\uc774\ub77c\uace0 \uc77c\ub800\uace0 \uc138\uc870\ub3c4 \ubb38\uc885\uc758 \uc720\uc77c\ud55c \uc0ac\uc704\ub77c \ud558\uba70 \uc815\uc885\uc744 \uc7a0\uc2dc \ub3c4\uc74d\uc73c\ub85c \ub3cc\uc544\uc624\uac8c \ud588\ub2e4. \uadf8\ub7ec\ub098 \uc0ac\uac04\uc6d0\uc758 \uc0c1\uc18c\ub85c \uc815\uc885\uc740 \uacf5\uc8fc\uc758 \ubcd1\uc774 \ub0ab\uc790 \ub2e4\uc2dc \uc720\ubc30\ub418\uc5c8\ub2e4. \uacf5\uc8fc\ub294 \uc774\ub54c \uc815\uc885\uc758 \uc720\ubc30\uc9c0\uc778 \uc218\uc6d0\uc73c\ub85c \ud5a5\ud588\uace0 \ub098\uc911\uc5d0 \ud1b5\uc9c4(\u901a\u6d25)\uc744 \uac70\uccd0 \uc804\ub77c\ub3c4 \uad11\uc8fc(\u5149\u5dde)\uae4c\uc9c0 \ub530\ub77c\uac14\ub2e4. \uc774\ud6c4 \uc815\uc885\uc740 \uc2b9\ub824 \uc131\ud0c4 \ub4f1\uacfc \uacb0\ud0c1\ud574 \ubaa8\ubc18\uc744 \uaf80\ud588\ub2e4\ub294 \ud610\uc758\ub85c \uac70\uc5f4\ud615\uc744 \ub2f9\ud558\uc600\ub2e4. \ub2f9\uc2dc \uacbd\ud61c\uacf5\uc8fc\ub294 \uc784\uc2e0 \uc911\uc774\uc5c8\ub2e4. \u300a\uc5f0\ub824\uc2e4\uae30\uc220\u300b\uc5d0\uc11c\ub294 \uacbd\ud61c\uacf5\uc8fc \ub610\ud55c \ub0a8\ud3b8\uc758 \uc8c4\uc5d0 \uc5f0\uc88c\ub418\uc5b4 \uac00\uc0b0\uc774 \uc801\ubab0\ub418\uace0 \uc720\ubc30\ub418\uc5b4 \uc21c\ucc9c\uc758 \uad00\ube44\uac00 \ub418\uc5c8\ub2e4\uace0 \uae30\ub85d\ud558\uace0 \uc788\ub2e4."} {"question": "\ucc44\uc2a4\ud14c\uc778\uc774 \uac00\uc7a5 \ud070 \uc0c1\uc5c5\uc801\uc778 \uc131\uacf5\uc744 \uac70\ub454 \uacf5\uc0c1\uacfc\ud559 \uc601\ud654\ub294 \ubb34\uc5c7\uc778\uac00?", "paragraph": "\ub610\ud55c 2012\ub144\uc5d0 \ucc44\uc2a4\ud14c\uc778\uc740 \ube0c\ub85c\ub4dc\uc6e8\uc774\uc5d0\uc11c \uc7ac\uacf5\uc5f0 \ub41c \u300a\uc0ac\ub791\uc544 \ub098\ub294 \ud1b5\uace1\ud55c\ub2e4\u300b\uc5d0\uc11c \uac15\ub825\ud55c \uc5ec\uc790\ub85c \ubcc0\ubaa8\ud558\ub294 \uc21c\uc9c4 \uc5b4\ub9b0 \uc18c\ub140\uc758 \uc5ed\ud560\uc744 \uc5f0\uae30\ud558\uba70 \uadf8\ub140\uc758 \ube0c\ub85c\ub4dc\uc6e8\uc774 \ub370\ubdd4\ub97c \uc131\uacf5\uc801\uc73c\ub85c \ub9c8\ucce4\ub2e4. 2013\ub144\uc5d0 \uadf8\ub140\ub294 \uacf5\ud3ec \uc601\ud654 \u300a\ub9c8\ub9c8\u300b\uc5d0\uc11c \uc8fc\uc5f0\uc744 \ub9e1\uc558\uc73c\uba70, \ub124\ub4dc \ubca4\uc2a8 \uac10\ub3c5\uc758 \uc601\ud654 \u300a\uc5d8\ub9ac\ub178\uc5b4 \ub9ad\ube44: \uadf8 \ub0a8\uc790 \uadf8 \uc5ec\uc790\u300b\uc5d0\uc11c \ubd88\ud589\ud55c \uacb0\ud63c\uc744 \ud55c \uc5ec\uc790\ub97c \uc5f0\uae30\ud588\ub2e4. \ucc44\uc2a4\ud14c\uc778\uc758 \uac00\uc7a5 \ud070 \uc0c1\uc5c5\uc801\uc778 \uc131\uacf5\uc744 \uac70\ub454 \uacf5\uc0c1 \uacfc\ud559 \uc601\ud654 \u300a\uc778\ud130\uc2a4\ud154\ub77c\u300b(2014)\uc640 \u300a\ub9c8\uc158\u300b(2015)\uc73c\ub85c \ud5a5\ud6c4 2\ub144\uac04\uc5d0 \uac1c\ubd09 \ub418\uc5c8\uc73c\uba70, \uc774\ub4e4 \ubaa8\ub450\ub294 \uc804 \uc138\uacc4\uc801\uc73c\ub85c 6\uc5b5 \ub2ec\ub7ec \uc774\uc0c1\uc744 \ubc8c\uc5b4 \ub4e4\uc600\ub2e4. \uc804\uc791\uc758 \ud06c\ub9ac\uc2a4\ud1a0\ud37c \ub180\ub780\uc774 \uac10\ub3c5\ud55c \u300a\uc778\ud130\uc2a4\ud154\ub77c\u300b\uc5d0\uc11c\ub294 \uacfc\ud559\uc790\uc600\uace0, \ud6c4\uc790\uc758 \ub9ac\ub4e4\ub9ac \uc2a4\ucf67\uc774 \uac10\ub3c5\ud55c \u300a\ub9c8\uc158\u300b\uc5d0\uc11c\ub294 \ub9f7 \ub370\uc774\uba3c\uacfc \ud568\uaed8 \uc6b0\uc8fc \ube44\ud589\uc0ac\ub97c \uc5f0\uae30\ud588\ub2e4. \ucc44\uc2a4\ud14c\uc778\uc740 \uc815\uce58 \uc2a4\ub9b4\ub7ec \u300a\ubbf8\uc2a4 \uc904\ub9ac\u300b(2016), \uc5ed\uc0ac \ub4dc\ub77c\ub9c8 \u300a\uc8fc\ud0a4\ud37c\uc2a4 \uc640\uc774\ud504\u300b(2017)\uacfc \ubc94\uc8c4 \uc601\ud654 \u300a\ubab0\ub9ac\uc758 \uac8c\uc784\u300b(2017)\uc5d0\uc11c \uac15\ud55c \uc758\uc9c0\ub97c \uac00\uc9c4 \uc8fc\uc778\uacf5\uc744 \uc5f0\uae30\ud588\ub2e4.", "answer": "\u300a\uc778\ud130\uc2a4\ud154\ub77c\u300b(2014)\uc640 \u300a\ub9c8\uc158\u300b(2015)", "sentence": "\ucc44\uc2a4\ud14c\uc778\uc758 \uac00\uc7a5 \ud070 \uc0c1\uc5c5\uc801\uc778 \uc131\uacf5\uc744 \uac70\ub454 \uacf5\uc0c1 \uacfc\ud559 \uc601\ud654 \u300a\uc778\ud130\uc2a4\ud154\ub77c\u300b(2014)\uc640 \u300a\ub9c8\uc158\u300b(2015) \uc73c\ub85c \ud5a5\ud6c4 2\ub144\uac04\uc5d0 \uac1c\ubd09 \ub418\uc5c8\uc73c\uba70, \uc774\ub4e4 \ubaa8\ub450\ub294 \uc804 \uc138\uacc4\uc801\uc73c\ub85c 6\uc5b5 \ub2ec\ub7ec \uc774\uc0c1\uc744 \ubc8c\uc5b4 \ub4e4\uc600\ub2e4.", "paragraph_sentence": "\ub610\ud55c 2012\ub144\uc5d0 \ucc44\uc2a4\ud14c\uc778\uc740 \ube0c\ub85c\ub4dc\uc6e8\uc774\uc5d0\uc11c \uc7ac\uacf5\uc5f0 \ub41c \u300a\uc0ac\ub791\uc544 \ub098\ub294 \ud1b5\uace1\ud55c\ub2e4\u300b\uc5d0\uc11c \uac15\ub825\ud55c \uc5ec\uc790\ub85c \ubcc0\ubaa8\ud558\ub294 \uc21c\uc9c4 \uc5b4\ub9b0 \uc18c\ub140\uc758 \uc5ed\ud560\uc744 \uc5f0\uae30\ud558\uba70 \uadf8\ub140\uc758 \ube0c\ub85c\ub4dc\uc6e8\uc774 \ub370\ubdd4\ub97c \uc131\uacf5\uc801\uc73c\ub85c \ub9c8\ucce4\ub2e4. 2013\ub144\uc5d0 \uadf8\ub140\ub294 \uacf5\ud3ec \uc601\ud654 \u300a\ub9c8\ub9c8\u300b\uc5d0\uc11c \uc8fc\uc5f0\uc744 \ub9e1\uc558\uc73c\uba70, \ub124\ub4dc \ubca4\uc2a8 \uac10\ub3c5\uc758 \uc601\ud654 \u300a\uc5d8\ub9ac\ub178\uc5b4 \ub9ad\ube44: \uadf8 \ub0a8\uc790 \uadf8 \uc5ec\uc790\u300b\uc5d0\uc11c \ubd88\ud589\ud55c \uacb0\ud63c\uc744 \ud55c \uc5ec\uc790\ub97c \uc5f0\uae30\ud588\ub2e4. \ucc44\uc2a4\ud14c\uc778\uc758 \uac00\uc7a5 \ud070 \uc0c1\uc5c5\uc801\uc778 \uc131\uacf5\uc744 \uac70\ub454 \uacf5\uc0c1 \uacfc\ud559 \uc601\ud654 \u300a\uc778\ud130\uc2a4\ud154\ub77c\u300b(2014)\uc640 \u300a\ub9c8\uc158\u300b(2015) \uc73c\ub85c \ud5a5\ud6c4 2\ub144\uac04\uc5d0 \uac1c\ubd09 \ub418\uc5c8\uc73c\uba70, \uc774\ub4e4 \ubaa8\ub450\ub294 \uc804 \uc138\uacc4\uc801\uc73c\ub85c 6\uc5b5 \ub2ec\ub7ec \uc774\uc0c1\uc744 \ubc8c\uc5b4 \ub4e4\uc600\ub2e4. \uc804\uc791\uc758 \ud06c\ub9ac\uc2a4\ud1a0\ud37c \ub180\ub780\uc774 \uac10\ub3c5\ud55c \u300a\uc778\ud130\uc2a4\ud154\ub77c\u300b\uc5d0\uc11c\ub294 \uacfc\ud559\uc790\uc600\uace0, \ud6c4\uc790\uc758 \ub9ac\ub4e4\ub9ac \uc2a4\ucf67\uc774 \uac10\ub3c5\ud55c \u300a\ub9c8\uc158\u300b\uc5d0\uc11c\ub294 \ub9f7 \ub370\uc774\uba3c\uacfc \ud568\uaed8 \uc6b0\uc8fc \ube44\ud589\uc0ac\ub97c \uc5f0\uae30\ud588\ub2e4. \ucc44\uc2a4\ud14c\uc778\uc740 \uc815\uce58 \uc2a4\ub9b4\ub7ec \u300a\ubbf8\uc2a4 \uc904\ub9ac\u300b(2016), \uc5ed\uc0ac \ub4dc\ub77c\ub9c8 \u300a\uc8fc\ud0a4\ud37c\uc2a4 \uc640\uc774\ud504\u300b(2017)\uacfc \ubc94\uc8c4 \uc601\ud654 \u300a\ubab0\ub9ac\uc758 \uac8c\uc784\u300b(2017)\uc5d0\uc11c \uac15\ud55c \uc758\uc9c0\ub97c \uac00\uc9c4 \uc8fc\uc778\uacf5\uc744 \uc5f0\uae30\ud588\ub2e4.", "paragraph_answer": "\ub610\ud55c 2012\ub144\uc5d0 \ucc44\uc2a4\ud14c\uc778\uc740 \ube0c\ub85c\ub4dc\uc6e8\uc774\uc5d0\uc11c \uc7ac\uacf5\uc5f0 \ub41c \u300a\uc0ac\ub791\uc544 \ub098\ub294 \ud1b5\uace1\ud55c\ub2e4\u300b\uc5d0\uc11c \uac15\ub825\ud55c \uc5ec\uc790\ub85c \ubcc0\ubaa8\ud558\ub294 \uc21c\uc9c4 \uc5b4\ub9b0 \uc18c\ub140\uc758 \uc5ed\ud560\uc744 \uc5f0\uae30\ud558\uba70 \uadf8\ub140\uc758 \ube0c\ub85c\ub4dc\uc6e8\uc774 \ub370\ubdd4\ub97c \uc131\uacf5\uc801\uc73c\ub85c \ub9c8\ucce4\ub2e4. 2013\ub144\uc5d0 \uadf8\ub140\ub294 \uacf5\ud3ec \uc601\ud654 \u300a\ub9c8\ub9c8\u300b\uc5d0\uc11c \uc8fc\uc5f0\uc744 \ub9e1\uc558\uc73c\uba70, \ub124\ub4dc \ubca4\uc2a8 \uac10\ub3c5\uc758 \uc601\ud654 \u300a\uc5d8\ub9ac\ub178\uc5b4 \ub9ad\ube44: \uadf8 \ub0a8\uc790 \uadf8 \uc5ec\uc790\u300b\uc5d0\uc11c \ubd88\ud589\ud55c \uacb0\ud63c\uc744 \ud55c \uc5ec\uc790\ub97c \uc5f0\uae30\ud588\ub2e4. \ucc44\uc2a4\ud14c\uc778\uc758 \uac00\uc7a5 \ud070 \uc0c1\uc5c5\uc801\uc778 \uc131\uacf5\uc744 \uac70\ub454 \uacf5\uc0c1 \uacfc\ud559 \uc601\ud654 \u300a\uc778\ud130\uc2a4\ud154\ub77c\u300b(2014)\uc640 \u300a\ub9c8\uc158\u300b(2015) \uc73c\ub85c \ud5a5\ud6c4 2\ub144\uac04\uc5d0 \uac1c\ubd09 \ub418\uc5c8\uc73c\uba70, \uc774\ub4e4 \ubaa8\ub450\ub294 \uc804 \uc138\uacc4\uc801\uc73c\ub85c 6\uc5b5 \ub2ec\ub7ec \uc774\uc0c1\uc744 \ubc8c\uc5b4 \ub4e4\uc600\ub2e4. \uc804\uc791\uc758 \ud06c\ub9ac\uc2a4\ud1a0\ud37c \ub180\ub780\uc774 \uac10\ub3c5\ud55c \u300a\uc778\ud130\uc2a4\ud154\ub77c\u300b\uc5d0\uc11c\ub294 \uacfc\ud559\uc790\uc600\uace0, \ud6c4\uc790\uc758 \ub9ac\ub4e4\ub9ac \uc2a4\ucf67\uc774 \uac10\ub3c5\ud55c \u300a\ub9c8\uc158\u300b\uc5d0\uc11c\ub294 \ub9f7 \ub370\uc774\uba3c\uacfc \ud568\uaed8 \uc6b0\uc8fc \ube44\ud589\uc0ac\ub97c \uc5f0\uae30\ud588\ub2e4. \ucc44\uc2a4\ud14c\uc778\uc740 \uc815\uce58 \uc2a4\ub9b4\ub7ec \u300a\ubbf8\uc2a4 \uc904\ub9ac\u300b(2016), \uc5ed\uc0ac \ub4dc\ub77c\ub9c8 \u300a\uc8fc\ud0a4\ud37c\uc2a4 \uc640\uc774\ud504\u300b(2017)\uacfc \ubc94\uc8c4 \uc601\ud654 \u300a\ubab0\ub9ac\uc758 \uac8c\uc784\u300b(2017)\uc5d0\uc11c \uac15\ud55c \uc758\uc9c0\ub97c \uac00\uc9c4 \uc8fc\uc778\uacf5\uc744 \uc5f0\uae30\ud588\ub2e4.", "sentence_answer": "\ucc44\uc2a4\ud14c\uc778\uc758 \uac00\uc7a5 \ud070 \uc0c1\uc5c5\uc801\uc778 \uc131\uacf5\uc744 \uac70\ub454 \uacf5\uc0c1 \uacfc\ud559 \uc601\ud654 \u300a\uc778\ud130\uc2a4\ud154\ub77c\u300b(2014)\uc640 \u300a\ub9c8\uc158\u300b(2015) \uc73c\ub85c \ud5a5\ud6c4 2\ub144\uac04\uc5d0 \uac1c\ubd09 \ub418\uc5c8\uc73c\uba70, \uc774\ub4e4 \ubaa8\ub450\ub294 \uc804 \uc138\uacc4\uc801\uc73c\ub85c 6\uc5b5 \ub2ec\ub7ec \uc774\uc0c1\uc744 \ubc8c\uc5b4 \ub4e4\uc600\ub2e4.", "paragraph_id": "6550953-2-1", "paragraph_question": "question: \ucc44\uc2a4\ud14c\uc778\uc774 \uac00\uc7a5 \ud070 \uc0c1\uc5c5\uc801\uc778 \uc131\uacf5\uc744 \uac70\ub454 \uacf5\uc0c1\uacfc\ud559 \uc601\ud654\ub294 \ubb34\uc5c7\uc778\uac00?, context: \ub610\ud55c 2012\ub144\uc5d0 \ucc44\uc2a4\ud14c\uc778\uc740 \ube0c\ub85c\ub4dc\uc6e8\uc774\uc5d0\uc11c \uc7ac\uacf5\uc5f0 \ub41c \u300a\uc0ac\ub791\uc544 \ub098\ub294 \ud1b5\uace1\ud55c\ub2e4\u300b\uc5d0\uc11c \uac15\ub825\ud55c \uc5ec\uc790\ub85c \ubcc0\ubaa8\ud558\ub294 \uc21c\uc9c4 \uc5b4\ub9b0 \uc18c\ub140\uc758 \uc5ed\ud560\uc744 \uc5f0\uae30\ud558\uba70 \uadf8\ub140\uc758 \ube0c\ub85c\ub4dc\uc6e8\uc774 \ub370\ubdd4\ub97c \uc131\uacf5\uc801\uc73c\ub85c \ub9c8\ucce4\ub2e4. 2013\ub144\uc5d0 \uadf8\ub140\ub294 \uacf5\ud3ec \uc601\ud654 \u300a\ub9c8\ub9c8\u300b\uc5d0\uc11c \uc8fc\uc5f0\uc744 \ub9e1\uc558\uc73c\uba70, \ub124\ub4dc \ubca4\uc2a8 \uac10\ub3c5\uc758 \uc601\ud654 \u300a\uc5d8\ub9ac\ub178\uc5b4 \ub9ad\ube44: \uadf8 \ub0a8\uc790 \uadf8 \uc5ec\uc790\u300b\uc5d0\uc11c \ubd88\ud589\ud55c \uacb0\ud63c\uc744 \ud55c \uc5ec\uc790\ub97c \uc5f0\uae30\ud588\ub2e4. \ucc44\uc2a4\ud14c\uc778\uc758 \uac00\uc7a5 \ud070 \uc0c1\uc5c5\uc801\uc778 \uc131\uacf5\uc744 \uac70\ub454 \uacf5\uc0c1 \uacfc\ud559 \uc601\ud654 \u300a\uc778\ud130\uc2a4\ud154\ub77c\u300b(2014)\uc640 \u300a\ub9c8\uc158\u300b(2015)\uc73c\ub85c \ud5a5\ud6c4 2\ub144\uac04\uc5d0 \uac1c\ubd09 \ub418\uc5c8\uc73c\uba70, \uc774\ub4e4 \ubaa8\ub450\ub294 \uc804 \uc138\uacc4\uc801\uc73c\ub85c 6\uc5b5 \ub2ec\ub7ec \uc774\uc0c1\uc744 \ubc8c\uc5b4 \ub4e4\uc600\ub2e4. \uc804\uc791\uc758 \ud06c\ub9ac\uc2a4\ud1a0\ud37c \ub180\ub780\uc774 \uac10\ub3c5\ud55c \u300a\uc778\ud130\uc2a4\ud154\ub77c\u300b\uc5d0\uc11c\ub294 \uacfc\ud559\uc790\uc600\uace0, \ud6c4\uc790\uc758 \ub9ac\ub4e4\ub9ac \uc2a4\ucf67\uc774 \uac10\ub3c5\ud55c \u300a\ub9c8\uc158\u300b\uc5d0\uc11c\ub294 \ub9f7 \ub370\uc774\uba3c\uacfc \ud568\uaed8 \uc6b0\uc8fc \ube44\ud589\uc0ac\ub97c \uc5f0\uae30\ud588\ub2e4. \ucc44\uc2a4\ud14c\uc778\uc740 \uc815\uce58 \uc2a4\ub9b4\ub7ec \u300a\ubbf8\uc2a4 \uc904\ub9ac\u300b(2016), \uc5ed\uc0ac \ub4dc\ub77c\ub9c8 \u300a\uc8fc\ud0a4\ud37c\uc2a4 \uc640\uc774\ud504\u300b(2017)\uacfc \ubc94\uc8c4 \uc601\ud654 \u300a\ubab0\ub9ac\uc758 \uac8c\uc784\u300b(2017)\uc5d0\uc11c \uac15\ud55c \uc758\uc9c0\ub97c \uac00\uc9c4 \uc8fc\uc778\uacf5\uc744 \uc5f0\uae30\ud588\ub2e4."} {"question": "\uac10\uc625\uc744 \ub098\uc628 \ub85c\ube48\uc774 \ub3c4\uc801\ub5bc\uc640 \ub9cc\ub098\ub294 \uc7a5\uc18c\ub294?", "paragraph": "\ub85c\ube48 \ub85d\uc2ac\ub9ac\ub294 \uc5f0\uc778 \ub9c8\ub9ac\uc548\uc5d0\uac8c \uaf2d \uc2b9\ub9ac\ud574\uc11c \ub3cc\uc544\uc624\uaca0\ub2e4\ub294 \uc57d\uc18d\uc744 \ud55c \ub4a4 \uce5c\uad6c \uae38\ubc84\ud2b8\uc640 \ud568\uaed8 \uc804\uc7c1\ud130\ub85c \ub5a0\ub09c\ub2e4. \uc804\ud22c\ub97c \ud558\ub358 \uc911 \uae38\ubc84\ud2b8\ub294 \uc0ac\uc790\uc655 \ub9ac\ucc98\ub4dc\ub97c \uc0b4\ud574\ud558\uace0 \ub85c\ube48\uc5d0\uac8c \ubaa8\ub4e0 \uac83\uc744 \ub4a4\uc9d1\uc5b4 \uc50c\uc6cc \uc655\uc744 \uc0b4\ud574\ud55c \ubc18\uc5ed\uc8c4\uc778\uc73c\ub85c \ubab0\ub9ac\uac8c \ud55c\ub2e4. \ub85c\ube48\uc740 \ub9ac\ucc98\ub4dc\uc655\uc758 \uc655\uad00\uc744 \ub4e4\uace0 \uac00\uc871\uc774 \uc788\ub294 \uacf3\uc73c\ub85c \ub3c4\ub9dd\uc744 \uc654\uc73c\ub098 \uc774\ubbf8 \ubd80\ubaa8\ub2d8\uacfc \ub3d9\uc0dd\uc740 \uad50\uc218\ud615\uc5d0 \ucc98\ud574\uc9c4 \ud6c4\uc600\ub2e4. \uae38\ubc84\ud2b8\ub294 \ub178\ud305\uc5c4\uc758 \uc601\uc8fc\uac00 \ub418\uc5b4 \ud638\ud654\ub85c\uc6b4 \uc0b6\uc744 \uc0b4\uac8c \ub418\uace0 \ub85c\ube48\uc740 \uc9c0\ud558 \uac10\uc625\uc5d0 \uac07\ud788\uc9c0\ub9cc \ub9c8\ub9ac\uc548\uc774 \ub4f1\uc7a5\ud558\uc5ec \ub85c\ube48\uc744 \uac10\uc625\uc5d0\uc11c \uaebc\ub0b4\uc900\ub2e4. \uadf8\ub7ec\ub098 \ub9c8\ub9ac\uc548\uc740 \uc774\ubbf8 \uc131\uacf5\uc744 \uc704\ud574 \uae38\ubc84\ud2b8\uc640 \uacb0\ud63c\ud588\uc73c\uba70 \uc774\uac83\uc774 \uc790\uc2e0\uc774 \ud574\uc904 \uc218 \uc788\ub294 \ub9c8\uc9c0\ub9c9\uc774\ub77c\uba70 \ub85c\ube48\uc744 \ub5a0\ub09c\ub2e4. \ub85c\ube48\uc740 \uac10\uc625\uc744 \ubc97\uc5b4\ub098 \uc154\uc6b0\ub4dc \uc232\uc73c\ub85c \ub4e4\uc5b4\uac00 \ub3c4\uc801\ub5bc\uc640 \ub9cc\ub098 \ub85c\ube48\ud6d7\uc774\ub77c\ub294 \uc774\ub984\uc744 \uc5bb\uac8c \ub41c\ub2e4. \ub3c4\uc801\ub5bc\ub4e4\uacfc \ud568\uaed8 \ub85c\ube48\ud6d7\uc740 \uc154\uc6b0\ub4dc \uc232\uc744 \ud1b5\uacfc\ud558\ub294 \uc138\uae08\ub9c8\ucc28\ub97c \ud138\uc5b4 \ub9c8\uc744\uc0ac\ub78c\ub4e4\uc5d0\uac8c \ub098\ub204\uc5b4\uc8fc\ub294 \ub4f1\uc758 \uc77c\uc744 \ud558\uba70 \uc9c0\ub0b8\ub2e4. \uadf8\ub7ec\ub358 \uc911 \ud504\ub791\uc2a4\uc5d0 \uc788\ub358 \ud544\ub9bd \uc655\uc138\uc790\ub294 \uc655\uc704 \uacc4\uc2b9\uc744 \uc704\ud574 \uc2e0\ud558 \uadf8\ub808\uace0\ub9ac\uc640 \ud568\uaed8 \ub7f0\ub358\uc73c\ub85c \uc640\uc57c\ud558\ub294 \uc0c1\ud669\uc5d0\uc11c \ud639\uc2dc \ubaa8\ub97c \uc704\ud5d8\uc744 \ud53c\ud558\uae30 \uc704\ud574 \uc154\uc6b0\ub4dc \uc232\uc744 \ud1b5\ud55c \uc9c0\ub984\uae38\uc744 \ud0dd\ud558\uac8c \ub41c\ub2e4. \uc655\uc774 \ub418\uae30 \uc2eb\ub2e4\ub294 \ud544\ub9bd\uc740 \uc232\uc5d0\uc11c \ub85c\ube48\ud6d7\uc744 \ub9cc\ub098 \uc9c4\uc815\ud55c \uc655\uc774 \ub418\uae30 \uc704\ud574 \ubcc0\ud654\ud558\uae30 \uc2dc\uc791\ud55c\ub2e4.", "answer": "\uc154\uc6b0\ub4dc \uc232", "sentence": "\ub85c\ube48\uc740 \uac10\uc625\uc744 \ubc97\uc5b4\ub098 \uc154\uc6b0\ub4dc \uc232 \uc73c\ub85c \ub4e4\uc5b4\uac00 \ub3c4\uc801\ub5bc\uc640 \ub9cc\ub098 \ub85c\ube48\ud6d7\uc774\ub77c\ub294 \uc774\ub984\uc744 \uc5bb\uac8c \ub41c\ub2e4.", "paragraph_sentence": "\ub85c\ube48 \ub85d\uc2ac\ub9ac\ub294 \uc5f0\uc778 \ub9c8\ub9ac\uc548\uc5d0\uac8c \uaf2d \uc2b9\ub9ac\ud574\uc11c \ub3cc\uc544\uc624\uaca0\ub2e4\ub294 \uc57d\uc18d\uc744 \ud55c \ub4a4 \uce5c\uad6c \uae38\ubc84\ud2b8\uc640 \ud568\uaed8 \uc804\uc7c1\ud130\ub85c \ub5a0\ub09c\ub2e4. \uc804\ud22c\ub97c \ud558\ub358 \uc911 \uae38\ubc84\ud2b8\ub294 \uc0ac\uc790\uc655 \ub9ac\ucc98\ub4dc\ub97c \uc0b4\ud574\ud558\uace0 \ub85c\ube48\uc5d0\uac8c \ubaa8\ub4e0 \uac83\uc744 \ub4a4\uc9d1\uc5b4 \uc50c\uc6cc \uc655\uc744 \uc0b4\ud574\ud55c \ubc18\uc5ed\uc8c4\uc778\uc73c\ub85c \ubab0\ub9ac\uac8c \ud55c\ub2e4. \ub85c\ube48\uc740 \ub9ac\ucc98\ub4dc\uc655\uc758 \uc655\uad00\uc744 \ub4e4\uace0 \uac00\uc871\uc774 \uc788\ub294 \uacf3\uc73c\ub85c \ub3c4\ub9dd\uc744 \uc654\uc73c\ub098 \uc774\ubbf8 \ubd80\ubaa8\ub2d8\uacfc \ub3d9\uc0dd\uc740 \uad50\uc218\ud615\uc5d0 \ucc98\ud574\uc9c4 \ud6c4\uc600\ub2e4. \uae38\ubc84\ud2b8\ub294 \ub178\ud305\uc5c4\uc758 \uc601\uc8fc\uac00 \ub418\uc5b4 \ud638\ud654\ub85c\uc6b4 \uc0b6\uc744 \uc0b4\uac8c \ub418\uace0 \ub85c\ube48\uc740 \uc9c0\ud558 \uac10\uc625\uc5d0 \uac07\ud788\uc9c0\ub9cc \ub9c8\ub9ac\uc548\uc774 \ub4f1\uc7a5\ud558\uc5ec \ub85c\ube48\uc744 \uac10\uc625\uc5d0\uc11c \uaebc\ub0b4\uc900\ub2e4. \uadf8\ub7ec\ub098 \ub9c8\ub9ac\uc548\uc740 \uc774\ubbf8 \uc131\uacf5\uc744 \uc704\ud574 \uae38\ubc84\ud2b8\uc640 \uacb0\ud63c\ud588\uc73c\uba70 \uc774\uac83\uc774 \uc790\uc2e0\uc774 \ud574\uc904 \uc218 \uc788\ub294 \ub9c8\uc9c0\ub9c9\uc774\ub77c\uba70 \ub85c\ube48\uc744 \ub5a0\ub09c\ub2e4. \ub85c\ube48\uc740 \uac10\uc625\uc744 \ubc97\uc5b4\ub098 \uc154\uc6b0\ub4dc \uc232 \uc73c\ub85c \ub4e4\uc5b4\uac00 \ub3c4\uc801\ub5bc\uc640 \ub9cc\ub098 \ub85c\ube48\ud6d7\uc774\ub77c\ub294 \uc774\ub984\uc744 \uc5bb\uac8c \ub41c\ub2e4. \ub3c4\uc801\ub5bc\ub4e4\uacfc \ud568\uaed8 \ub85c\ube48\ud6d7\uc740 \uc154\uc6b0\ub4dc \uc232\uc744 \ud1b5\uacfc\ud558\ub294 \uc138\uae08\ub9c8\ucc28\ub97c \ud138\uc5b4 \ub9c8\uc744\uc0ac\ub78c\ub4e4\uc5d0\uac8c \ub098\ub204\uc5b4\uc8fc\ub294 \ub4f1\uc758 \uc77c\uc744 \ud558\uba70 \uc9c0\ub0b8\ub2e4. \uadf8\ub7ec\ub358 \uc911 \ud504\ub791\uc2a4\uc5d0 \uc788\ub358 \ud544\ub9bd \uc655\uc138\uc790\ub294 \uc655\uc704 \uacc4\uc2b9\uc744 \uc704\ud574 \uc2e0\ud558 \uadf8\ub808\uace0\ub9ac\uc640 \ud568\uaed8 \ub7f0\ub358\uc73c\ub85c \uc640\uc57c\ud558\ub294 \uc0c1\ud669\uc5d0\uc11c \ud639\uc2dc \ubaa8\ub97c \uc704\ud5d8\uc744 \ud53c\ud558\uae30 \uc704\ud574 \uc154\uc6b0\ub4dc \uc232\uc744 \ud1b5\ud55c \uc9c0\ub984\uae38\uc744 \ud0dd\ud558\uac8c \ub41c\ub2e4. \uc655\uc774 \ub418\uae30 \uc2eb\ub2e4\ub294 \ud544\ub9bd\uc740 \uc232\uc5d0\uc11c \ub85c\ube48\ud6d7\uc744 \ub9cc\ub098 \uc9c4\uc815\ud55c \uc655\uc774 \ub418\uae30 \uc704\ud574 \ubcc0\ud654\ud558\uae30 \uc2dc\uc791\ud55c\ub2e4.", "paragraph_answer": "\ub85c\ube48 \ub85d\uc2ac\ub9ac\ub294 \uc5f0\uc778 \ub9c8\ub9ac\uc548\uc5d0\uac8c \uaf2d \uc2b9\ub9ac\ud574\uc11c \ub3cc\uc544\uc624\uaca0\ub2e4\ub294 \uc57d\uc18d\uc744 \ud55c \ub4a4 \uce5c\uad6c \uae38\ubc84\ud2b8\uc640 \ud568\uaed8 \uc804\uc7c1\ud130\ub85c \ub5a0\ub09c\ub2e4. \uc804\ud22c\ub97c \ud558\ub358 \uc911 \uae38\ubc84\ud2b8\ub294 \uc0ac\uc790\uc655 \ub9ac\ucc98\ub4dc\ub97c \uc0b4\ud574\ud558\uace0 \ub85c\ube48\uc5d0\uac8c \ubaa8\ub4e0 \uac83\uc744 \ub4a4\uc9d1\uc5b4 \uc50c\uc6cc \uc655\uc744 \uc0b4\ud574\ud55c \ubc18\uc5ed\uc8c4\uc778\uc73c\ub85c \ubab0\ub9ac\uac8c \ud55c\ub2e4. \ub85c\ube48\uc740 \ub9ac\ucc98\ub4dc\uc655\uc758 \uc655\uad00\uc744 \ub4e4\uace0 \uac00\uc871\uc774 \uc788\ub294 \uacf3\uc73c\ub85c \ub3c4\ub9dd\uc744 \uc654\uc73c\ub098 \uc774\ubbf8 \ubd80\ubaa8\ub2d8\uacfc \ub3d9\uc0dd\uc740 \uad50\uc218\ud615\uc5d0 \ucc98\ud574\uc9c4 \ud6c4\uc600\ub2e4. \uae38\ubc84\ud2b8\ub294 \ub178\ud305\uc5c4\uc758 \uc601\uc8fc\uac00 \ub418\uc5b4 \ud638\ud654\ub85c\uc6b4 \uc0b6\uc744 \uc0b4\uac8c \ub418\uace0 \ub85c\ube48\uc740 \uc9c0\ud558 \uac10\uc625\uc5d0 \uac07\ud788\uc9c0\ub9cc \ub9c8\ub9ac\uc548\uc774 \ub4f1\uc7a5\ud558\uc5ec \ub85c\ube48\uc744 \uac10\uc625\uc5d0\uc11c \uaebc\ub0b4\uc900\ub2e4. \uadf8\ub7ec\ub098 \ub9c8\ub9ac\uc548\uc740 \uc774\ubbf8 \uc131\uacf5\uc744 \uc704\ud574 \uae38\ubc84\ud2b8\uc640 \uacb0\ud63c\ud588\uc73c\uba70 \uc774\uac83\uc774 \uc790\uc2e0\uc774 \ud574\uc904 \uc218 \uc788\ub294 \ub9c8\uc9c0\ub9c9\uc774\ub77c\uba70 \ub85c\ube48\uc744 \ub5a0\ub09c\ub2e4. \ub85c\ube48\uc740 \uac10\uc625\uc744 \ubc97\uc5b4\ub098 \uc154\uc6b0\ub4dc \uc232 \uc73c\ub85c \ub4e4\uc5b4\uac00 \ub3c4\uc801\ub5bc\uc640 \ub9cc\ub098 \ub85c\ube48\ud6d7\uc774\ub77c\ub294 \uc774\ub984\uc744 \uc5bb\uac8c \ub41c\ub2e4. \ub3c4\uc801\ub5bc\ub4e4\uacfc \ud568\uaed8 \ub85c\ube48\ud6d7\uc740 \uc154\uc6b0\ub4dc \uc232\uc744 \ud1b5\uacfc\ud558\ub294 \uc138\uae08\ub9c8\ucc28\ub97c \ud138\uc5b4 \ub9c8\uc744\uc0ac\ub78c\ub4e4\uc5d0\uac8c \ub098\ub204\uc5b4\uc8fc\ub294 \ub4f1\uc758 \uc77c\uc744 \ud558\uba70 \uc9c0\ub0b8\ub2e4. \uadf8\ub7ec\ub358 \uc911 \ud504\ub791\uc2a4\uc5d0 \uc788\ub358 \ud544\ub9bd \uc655\uc138\uc790\ub294 \uc655\uc704 \uacc4\uc2b9\uc744 \uc704\ud574 \uc2e0\ud558 \uadf8\ub808\uace0\ub9ac\uc640 \ud568\uaed8 \ub7f0\ub358\uc73c\ub85c \uc640\uc57c\ud558\ub294 \uc0c1\ud669\uc5d0\uc11c \ud639\uc2dc \ubaa8\ub97c \uc704\ud5d8\uc744 \ud53c\ud558\uae30 \uc704\ud574 \uc154\uc6b0\ub4dc \uc232\uc744 \ud1b5\ud55c \uc9c0\ub984\uae38\uc744 \ud0dd\ud558\uac8c \ub41c\ub2e4. \uc655\uc774 \ub418\uae30 \uc2eb\ub2e4\ub294 \ud544\ub9bd\uc740 \uc232\uc5d0\uc11c \ub85c\ube48\ud6d7\uc744 \ub9cc\ub098 \uc9c4\uc815\ud55c \uc655\uc774 \ub418\uae30 \uc704\ud574 \ubcc0\ud654\ud558\uae30 \uc2dc\uc791\ud55c\ub2e4.", "sentence_answer": "\ub85c\ube48\uc740 \uac10\uc625\uc744 \ubc97\uc5b4\ub098 \uc154\uc6b0\ub4dc \uc232 \uc73c\ub85c \ub4e4\uc5b4\uac00 \ub3c4\uc801\ub5bc\uc640 \ub9cc\ub098 \ub85c\ube48\ud6d7\uc774\ub77c\ub294 \uc774\ub984\uc744 \uc5bb\uac8c \ub41c\ub2e4.", "paragraph_id": "6513736-1-1", "paragraph_question": "question: \uac10\uc625\uc744 \ub098\uc628 \ub85c\ube48\uc774 \ub3c4\uc801\ub5bc\uc640 \ub9cc\ub098\ub294 \uc7a5\uc18c\ub294?, context: \ub85c\ube48 \ub85d\uc2ac\ub9ac\ub294 \uc5f0\uc778 \ub9c8\ub9ac\uc548\uc5d0\uac8c \uaf2d \uc2b9\ub9ac\ud574\uc11c \ub3cc\uc544\uc624\uaca0\ub2e4\ub294 \uc57d\uc18d\uc744 \ud55c \ub4a4 \uce5c\uad6c \uae38\ubc84\ud2b8\uc640 \ud568\uaed8 \uc804\uc7c1\ud130\ub85c \ub5a0\ub09c\ub2e4. \uc804\ud22c\ub97c \ud558\ub358 \uc911 \uae38\ubc84\ud2b8\ub294 \uc0ac\uc790\uc655 \ub9ac\ucc98\ub4dc\ub97c \uc0b4\ud574\ud558\uace0 \ub85c\ube48\uc5d0\uac8c \ubaa8\ub4e0 \uac83\uc744 \ub4a4\uc9d1\uc5b4 \uc50c\uc6cc \uc655\uc744 \uc0b4\ud574\ud55c \ubc18\uc5ed\uc8c4\uc778\uc73c\ub85c \ubab0\ub9ac\uac8c \ud55c\ub2e4. \ub85c\ube48\uc740 \ub9ac\ucc98\ub4dc\uc655\uc758 \uc655\uad00\uc744 \ub4e4\uace0 \uac00\uc871\uc774 \uc788\ub294 \uacf3\uc73c\ub85c \ub3c4\ub9dd\uc744 \uc654\uc73c\ub098 \uc774\ubbf8 \ubd80\ubaa8\ub2d8\uacfc \ub3d9\uc0dd\uc740 \uad50\uc218\ud615\uc5d0 \ucc98\ud574\uc9c4 \ud6c4\uc600\ub2e4. \uae38\ubc84\ud2b8\ub294 \ub178\ud305\uc5c4\uc758 \uc601\uc8fc\uac00 \ub418\uc5b4 \ud638\ud654\ub85c\uc6b4 \uc0b6\uc744 \uc0b4\uac8c \ub418\uace0 \ub85c\ube48\uc740 \uc9c0\ud558 \uac10\uc625\uc5d0 \uac07\ud788\uc9c0\ub9cc \ub9c8\ub9ac\uc548\uc774 \ub4f1\uc7a5\ud558\uc5ec \ub85c\ube48\uc744 \uac10\uc625\uc5d0\uc11c \uaebc\ub0b4\uc900\ub2e4. \uadf8\ub7ec\ub098 \ub9c8\ub9ac\uc548\uc740 \uc774\ubbf8 \uc131\uacf5\uc744 \uc704\ud574 \uae38\ubc84\ud2b8\uc640 \uacb0\ud63c\ud588\uc73c\uba70 \uc774\uac83\uc774 \uc790\uc2e0\uc774 \ud574\uc904 \uc218 \uc788\ub294 \ub9c8\uc9c0\ub9c9\uc774\ub77c\uba70 \ub85c\ube48\uc744 \ub5a0\ub09c\ub2e4. \ub85c\ube48\uc740 \uac10\uc625\uc744 \ubc97\uc5b4\ub098 \uc154\uc6b0\ub4dc \uc232\uc73c\ub85c \ub4e4\uc5b4\uac00 \ub3c4\uc801\ub5bc\uc640 \ub9cc\ub098 \ub85c\ube48\ud6d7\uc774\ub77c\ub294 \uc774\ub984\uc744 \uc5bb\uac8c \ub41c\ub2e4. \ub3c4\uc801\ub5bc\ub4e4\uacfc \ud568\uaed8 \ub85c\ube48\ud6d7\uc740 \uc154\uc6b0\ub4dc \uc232\uc744 \ud1b5\uacfc\ud558\ub294 \uc138\uae08\ub9c8\ucc28\ub97c \ud138\uc5b4 \ub9c8\uc744\uc0ac\ub78c\ub4e4\uc5d0\uac8c \ub098\ub204\uc5b4\uc8fc\ub294 \ub4f1\uc758 \uc77c\uc744 \ud558\uba70 \uc9c0\ub0b8\ub2e4. \uadf8\ub7ec\ub358 \uc911 \ud504\ub791\uc2a4\uc5d0 \uc788\ub358 \ud544\ub9bd \uc655\uc138\uc790\ub294 \uc655\uc704 \uacc4\uc2b9\uc744 \uc704\ud574 \uc2e0\ud558 \uadf8\ub808\uace0\ub9ac\uc640 \ud568\uaed8 \ub7f0\ub358\uc73c\ub85c \uc640\uc57c\ud558\ub294 \uc0c1\ud669\uc5d0\uc11c \ud639\uc2dc \ubaa8\ub97c \uc704\ud5d8\uc744 \ud53c\ud558\uae30 \uc704\ud574 \uc154\uc6b0\ub4dc \uc232\uc744 \ud1b5\ud55c \uc9c0\ub984\uae38\uc744 \ud0dd\ud558\uac8c \ub41c\ub2e4. \uc655\uc774 \ub418\uae30 \uc2eb\ub2e4\ub294 \ud544\ub9bd\uc740 \uc232\uc5d0\uc11c \ub85c\ube48\ud6d7\uc744 \ub9cc\ub098 \uc9c4\uc815\ud55c \uc655\uc774 \ub418\uae30 \uc704\ud574 \ubcc0\ud654\ud558\uae30 \uc2dc\uc791\ud55c\ub2e4."} {"question": "\uadf8\ub9ac\uc2a4\uc758 \uc5d8\ub9ac\uc2dc\uc628\uc758 \ubc1c\uc0c1\uc740 \uc774\uc9d1\ud2b8 \uc885\uad50\uc5d0 \uc5b4\ub514\uc5d0\uc11c \ube44\ub86f\ub418\uc5b4 \uc654\ub2e4\uace0 \ucd94\uc815\ud558\ub294\uac00?", "paragraph": "\uc774\uc9d1\ud2b8 \uc885\uad50\ub294 \uace0\ub300 \uc774\uc9d1\ud2b8 \uc0ac\ud68c\ub97c \uac00\uc7a5 \uc798 \ubcf4\uc5ec\uc8fc\ub294 \uae30\ub150\ubb3c\uc778 \uc218 \ub9ce\uc740 \uc2e0\uc804\ub4e4\uacfc \ubb34\ub364\ub4e4\uc744 \uc0dd\uc0b0\ud558\uc600\uc73c\uba70 \uc218 \ub9ce\uc740 \ubb38\ud654\ub4e4\uc5d0 \uac70\ub300\ud55c \uc601\ud5a5\uc744 \ub0a8\uacbc\ub2e4. \ud30c\ub77c\uc624 \uc2dc\ub300\uc758 \uc2a4\ud551\ud06c\uc2a4\uc640 \ud0dc\uc591 \uc6d0\ubc18\ub4f1\uc744 \ube44\ub86f\ud55c \ub2e4\uc591\ud55c \uc0c1\uc9d5\uacfc \uae30\ud638\ub4e4\uc740 \uc9c0\uc911\ud574\uc640 \uadfc\ub3d9\uc744 \ub118\uc5b4 \uad11\ubc94\uc704\ud558\uac8c \ud37c\uc838\ub098\uac14\ub2e4. \uadf8\ub9ac\uc2a4\uc778\ub4e4\uc758 \uc5d8\ub9ac\uc2dc\uc628\uc758 \ubc1c\uc0c1\uc740 \uc774\uc9d1\ud2b8\uc758 \uc0ac\ud6c4 \uc138\uacc4\uc5d0\uc11c \ube44\ub86f\ub41c \uac83\uc73c\ub85c \ucd94\uc815\ub418\uace0 \uc788\uc73c\uba70 \uc9c0\uadf8\ubb38\ud2b8 \ud504\ub85c\uc774\ud2b8\ub97c \uc2dc\uc791\uc73c\ub85c \ub9ce\uc740 \ud559\uc790\ub4e4\uc740 \uc720\ub300\uad50 \ub610\ud55c \uc774\uc9d1\ud2b8\uc758 \uc544\ud150\uc8fc\uc758\uc5d0\uc11c \uc2dc\uc791\ub418\uc5c8\uc744 \uac83\uc774\ub77c\uace0 \uc0dd\uac01\ud55c\ub2e4. \uace0\ub300 \ud6c4\uae30\uc5d0 \uc2dc\uc791\ub41c \uae30\ub3c5\uad50\uc758 \uc9c0\uc625\uc5d0 \ub300\ud55c \uac1c\ub150 \uc5ed\uc2dc \uc774\uc9d1\ud2b8 \uc2e0\ud654\uc5d0 \ub4f1\uc7a5\ud558\ub294 \ub450\uc544\ud2b8\uc640 \ub2ee\uc740 \uc810\uc774 \ub9ce\uc73c\uba70 \uc131\ubaa8\ub9c8\ub9ac\uc544 \ub610\ud55c \uc774\uc2dc\uc2a4\uc640 \ud638\ub8e8\uc2a4 \uc2e0\ud654\uc5d0\uc11c \ube44\ub86f\ub418\uc5c8\ub2e4. \uc774\uc9d1\ud2b8 \uc885\uad50\ub294 \uc601\uc9c0\uc8fc\uc758\uc758 \ud0c4\uc0dd\uacfc \uac1c\ub150\ud654\uc5d0 \uc601\ud5a5\uc744 \ub07c\ucce4\ub2e4. \uc608\ub97c \ub4e4\uc5b4 \uc2e0\ube44\uc8fc\uc758\ub294 \uc774\uc9d1\ud2b8\uc778\ub4e4\uc774 \ubbff\uc5c8\ub358 \ube44\ubc00\uc2a4\ub7ec\uc6b4 \ub9c8\ubc95 \uc9c0\uc2dd\uacfc \ud63c\ud569\ub41c \ud1a0\ud2b8 \uc2e0\uc559\uc758 \uc804\ud1b5\uc5d0\uc11c \ube44\ub86f\ub418\uc5c8\ub2e4.", "answer": "\uc0ac\ud6c4 \uc138\uacc4", "sentence": "\uadf8\ub9ac\uc2a4\uc778\ub4e4\uc758 \uc5d8\ub9ac\uc2dc\uc628\uc758 \ubc1c\uc0c1\uc740 \uc774\uc9d1\ud2b8\uc758 \uc0ac\ud6c4 \uc138\uacc4 \uc5d0\uc11c \ube44\ub86f\ub41c \uac83\uc73c\ub85c \ucd94\uc815\ub418\uace0 \uc788\uc73c\uba70 \uc9c0\uadf8\ubb38\ud2b8 \ud504\ub85c\uc774\ud2b8\ub97c \uc2dc\uc791\uc73c\ub85c \ub9ce\uc740 \ud559\uc790\ub4e4\uc740 \uc720\ub300\uad50 \ub610\ud55c \uc774\uc9d1\ud2b8\uc758 \uc544\ud150\uc8fc\uc758\uc5d0\uc11c \uc2dc\uc791\ub418\uc5c8\uc744 \uac83\uc774\ub77c\uace0 \uc0dd\uac01\ud55c\ub2e4.", "paragraph_sentence": "\uc774\uc9d1\ud2b8 \uc885\uad50\ub294 \uace0\ub300 \uc774\uc9d1\ud2b8 \uc0ac\ud68c\ub97c \uac00\uc7a5 \uc798 \ubcf4\uc5ec\uc8fc\ub294 \uae30\ub150\ubb3c\uc778 \uc218 \ub9ce\uc740 \uc2e0\uc804\ub4e4\uacfc \ubb34\ub364\ub4e4\uc744 \uc0dd\uc0b0\ud558\uc600\uc73c\uba70 \uc218 \ub9ce\uc740 \ubb38\ud654\ub4e4\uc5d0 \uac70\ub300\ud55c \uc601\ud5a5\uc744 \ub0a8\uacbc\ub2e4. \ud30c\ub77c\uc624 \uc2dc\ub300\uc758 \uc2a4\ud551\ud06c\uc2a4\uc640 \ud0dc\uc591 \uc6d0\ubc18\ub4f1\uc744 \ube44\ub86f\ud55c \ub2e4\uc591\ud55c \uc0c1\uc9d5\uacfc \uae30\ud638\ub4e4\uc740 \uc9c0\uc911\ud574\uc640 \uadfc\ub3d9\uc744 \ub118\uc5b4 \uad11\ubc94\uc704\ud558\uac8c \ud37c\uc838\ub098\uac14\ub2e4. \uadf8\ub9ac\uc2a4\uc778\ub4e4\uc758 \uc5d8\ub9ac\uc2dc\uc628\uc758 \ubc1c\uc0c1\uc740 \uc774\uc9d1\ud2b8\uc758 \uc0ac\ud6c4 \uc138\uacc4 \uc5d0\uc11c \ube44\ub86f\ub41c \uac83\uc73c\ub85c \ucd94\uc815\ub418\uace0 \uc788\uc73c\uba70 \uc9c0\uadf8\ubb38\ud2b8 \ud504\ub85c\uc774\ud2b8\ub97c \uc2dc\uc791\uc73c\ub85c \ub9ce\uc740 \ud559\uc790\ub4e4\uc740 \uc720\ub300\uad50 \ub610\ud55c \uc774\uc9d1\ud2b8\uc758 \uc544\ud150\uc8fc\uc758\uc5d0\uc11c \uc2dc\uc791\ub418\uc5c8\uc744 \uac83\uc774\ub77c\uace0 \uc0dd\uac01\ud55c\ub2e4. \uace0\ub300 \ud6c4\uae30\uc5d0 \uc2dc\uc791\ub41c \uae30\ub3c5\uad50\uc758 \uc9c0\uc625\uc5d0 \ub300\ud55c \uac1c\ub150 \uc5ed\uc2dc \uc774\uc9d1\ud2b8 \uc2e0\ud654\uc5d0 \ub4f1\uc7a5\ud558\ub294 \ub450\uc544\ud2b8\uc640 \ub2ee\uc740 \uc810\uc774 \ub9ce\uc73c\uba70 \uc131\ubaa8\ub9c8\ub9ac\uc544 \ub610\ud55c \uc774\uc2dc\uc2a4\uc640 \ud638\ub8e8\uc2a4 \uc2e0\ud654\uc5d0\uc11c \ube44\ub86f\ub418\uc5c8\ub2e4. \uc774\uc9d1\ud2b8 \uc885\uad50\ub294 \uc601\uc9c0\uc8fc\uc758\uc758 \ud0c4\uc0dd\uacfc \uac1c\ub150\ud654\uc5d0 \uc601\ud5a5\uc744 \ub07c\ucce4\ub2e4. \uc608\ub97c \ub4e4\uc5b4 \uc2e0\ube44\uc8fc\uc758\ub294 \uc774\uc9d1\ud2b8\uc778\ub4e4\uc774 \ubbff\uc5c8\ub358 \ube44\ubc00\uc2a4\ub7ec\uc6b4 \ub9c8\ubc95 \uc9c0\uc2dd\uacfc \ud63c\ud569\ub41c \ud1a0\ud2b8 \uc2e0\uc559\uc758 \uc804\ud1b5\uc5d0\uc11c \ube44\ub86f\ub418\uc5c8\ub2e4.", "paragraph_answer": "\uc774\uc9d1\ud2b8 \uc885\uad50\ub294 \uace0\ub300 \uc774\uc9d1\ud2b8 \uc0ac\ud68c\ub97c \uac00\uc7a5 \uc798 \ubcf4\uc5ec\uc8fc\ub294 \uae30\ub150\ubb3c\uc778 \uc218 \ub9ce\uc740 \uc2e0\uc804\ub4e4\uacfc \ubb34\ub364\ub4e4\uc744 \uc0dd\uc0b0\ud558\uc600\uc73c\uba70 \uc218 \ub9ce\uc740 \ubb38\ud654\ub4e4\uc5d0 \uac70\ub300\ud55c \uc601\ud5a5\uc744 \ub0a8\uacbc\ub2e4. \ud30c\ub77c\uc624 \uc2dc\ub300\uc758 \uc2a4\ud551\ud06c\uc2a4\uc640 \ud0dc\uc591 \uc6d0\ubc18\ub4f1\uc744 \ube44\ub86f\ud55c \ub2e4\uc591\ud55c \uc0c1\uc9d5\uacfc \uae30\ud638\ub4e4\uc740 \uc9c0\uc911\ud574\uc640 \uadfc\ub3d9\uc744 \ub118\uc5b4 \uad11\ubc94\uc704\ud558\uac8c \ud37c\uc838\ub098\uac14\ub2e4. \uadf8\ub9ac\uc2a4\uc778\ub4e4\uc758 \uc5d8\ub9ac\uc2dc\uc628\uc758 \ubc1c\uc0c1\uc740 \uc774\uc9d1\ud2b8\uc758 \uc0ac\ud6c4 \uc138\uacc4 \uc5d0\uc11c \ube44\ub86f\ub41c \uac83\uc73c\ub85c \ucd94\uc815\ub418\uace0 \uc788\uc73c\uba70 \uc9c0\uadf8\ubb38\ud2b8 \ud504\ub85c\uc774\ud2b8\ub97c \uc2dc\uc791\uc73c\ub85c \ub9ce\uc740 \ud559\uc790\ub4e4\uc740 \uc720\ub300\uad50 \ub610\ud55c \uc774\uc9d1\ud2b8\uc758 \uc544\ud150\uc8fc\uc758\uc5d0\uc11c \uc2dc\uc791\ub418\uc5c8\uc744 \uac83\uc774\ub77c\uace0 \uc0dd\uac01\ud55c\ub2e4. \uace0\ub300 \ud6c4\uae30\uc5d0 \uc2dc\uc791\ub41c \uae30\ub3c5\uad50\uc758 \uc9c0\uc625\uc5d0 \ub300\ud55c \uac1c\ub150 \uc5ed\uc2dc \uc774\uc9d1\ud2b8 \uc2e0\ud654\uc5d0 \ub4f1\uc7a5\ud558\ub294 \ub450\uc544\ud2b8\uc640 \ub2ee\uc740 \uc810\uc774 \ub9ce\uc73c\uba70 \uc131\ubaa8\ub9c8\ub9ac\uc544 \ub610\ud55c \uc774\uc2dc\uc2a4\uc640 \ud638\ub8e8\uc2a4 \uc2e0\ud654\uc5d0\uc11c \ube44\ub86f\ub418\uc5c8\ub2e4. \uc774\uc9d1\ud2b8 \uc885\uad50\ub294 \uc601\uc9c0\uc8fc\uc758\uc758 \ud0c4\uc0dd\uacfc \uac1c\ub150\ud654\uc5d0 \uc601\ud5a5\uc744 \ub07c\ucce4\ub2e4. \uc608\ub97c \ub4e4\uc5b4 \uc2e0\ube44\uc8fc\uc758\ub294 \uc774\uc9d1\ud2b8\uc778\ub4e4\uc774 \ubbff\uc5c8\ub358 \ube44\ubc00\uc2a4\ub7ec\uc6b4 \ub9c8\ubc95 \uc9c0\uc2dd\uacfc \ud63c\ud569\ub41c \ud1a0\ud2b8 \uc2e0\uc559\uc758 \uc804\ud1b5\uc5d0\uc11c \ube44\ub86f\ub418\uc5c8\ub2e4.", "sentence_answer": "\uadf8\ub9ac\uc2a4\uc778\ub4e4\uc758 \uc5d8\ub9ac\uc2dc\uc628\uc758 \ubc1c\uc0c1\uc740 \uc774\uc9d1\ud2b8\uc758 \uc0ac\ud6c4 \uc138\uacc4 \uc5d0\uc11c \ube44\ub86f\ub41c \uac83\uc73c\ub85c \ucd94\uc815\ub418\uace0 \uc788\uc73c\uba70 \uc9c0\uadf8\ubb38\ud2b8 \ud504\ub85c\uc774\ud2b8\ub97c \uc2dc\uc791\uc73c\ub85c \ub9ce\uc740 \ud559\uc790\ub4e4\uc740 \uc720\ub300\uad50 \ub610\ud55c \uc774\uc9d1\ud2b8\uc758 \uc544\ud150\uc8fc\uc758\uc5d0\uc11c \uc2dc\uc791\ub418\uc5c8\uc744 \uac83\uc774\ub77c\uace0 \uc0dd\uac01\ud55c\ub2e4.", "paragraph_id": "6471098-21-2", "paragraph_question": "question: \uadf8\ub9ac\uc2a4\uc758 \uc5d8\ub9ac\uc2dc\uc628\uc758 \ubc1c\uc0c1\uc740 \uc774\uc9d1\ud2b8 \uc885\uad50\uc5d0 \uc5b4\ub514\uc5d0\uc11c \ube44\ub86f\ub418\uc5b4 \uc654\ub2e4\uace0 \ucd94\uc815\ud558\ub294\uac00?, context: \uc774\uc9d1\ud2b8 \uc885\uad50\ub294 \uace0\ub300 \uc774\uc9d1\ud2b8 \uc0ac\ud68c\ub97c \uac00\uc7a5 \uc798 \ubcf4\uc5ec\uc8fc\ub294 \uae30\ub150\ubb3c\uc778 \uc218 \ub9ce\uc740 \uc2e0\uc804\ub4e4\uacfc \ubb34\ub364\ub4e4\uc744 \uc0dd\uc0b0\ud558\uc600\uc73c\uba70 \uc218 \ub9ce\uc740 \ubb38\ud654\ub4e4\uc5d0 \uac70\ub300\ud55c \uc601\ud5a5\uc744 \ub0a8\uacbc\ub2e4. \ud30c\ub77c\uc624 \uc2dc\ub300\uc758 \uc2a4\ud551\ud06c\uc2a4\uc640 \ud0dc\uc591 \uc6d0\ubc18\ub4f1\uc744 \ube44\ub86f\ud55c \ub2e4\uc591\ud55c \uc0c1\uc9d5\uacfc \uae30\ud638\ub4e4\uc740 \uc9c0\uc911\ud574\uc640 \uadfc\ub3d9\uc744 \ub118\uc5b4 \uad11\ubc94\uc704\ud558\uac8c \ud37c\uc838\ub098\uac14\ub2e4. \uadf8\ub9ac\uc2a4\uc778\ub4e4\uc758 \uc5d8\ub9ac\uc2dc\uc628\uc758 \ubc1c\uc0c1\uc740 \uc774\uc9d1\ud2b8\uc758 \uc0ac\ud6c4 \uc138\uacc4\uc5d0\uc11c \ube44\ub86f\ub41c \uac83\uc73c\ub85c \ucd94\uc815\ub418\uace0 \uc788\uc73c\uba70 \uc9c0\uadf8\ubb38\ud2b8 \ud504\ub85c\uc774\ud2b8\ub97c \uc2dc\uc791\uc73c\ub85c \ub9ce\uc740 \ud559\uc790\ub4e4\uc740 \uc720\ub300\uad50 \ub610\ud55c \uc774\uc9d1\ud2b8\uc758 \uc544\ud150\uc8fc\uc758\uc5d0\uc11c \uc2dc\uc791\ub418\uc5c8\uc744 \uac83\uc774\ub77c\uace0 \uc0dd\uac01\ud55c\ub2e4. \uace0\ub300 \ud6c4\uae30\uc5d0 \uc2dc\uc791\ub41c \uae30\ub3c5\uad50\uc758 \uc9c0\uc625\uc5d0 \ub300\ud55c \uac1c\ub150 \uc5ed\uc2dc \uc774\uc9d1\ud2b8 \uc2e0\ud654\uc5d0 \ub4f1\uc7a5\ud558\ub294 \ub450\uc544\ud2b8\uc640 \ub2ee\uc740 \uc810\uc774 \ub9ce\uc73c\uba70 \uc131\ubaa8\ub9c8\ub9ac\uc544 \ub610\ud55c \uc774\uc2dc\uc2a4\uc640 \ud638\ub8e8\uc2a4 \uc2e0\ud654\uc5d0\uc11c \ube44\ub86f\ub418\uc5c8\ub2e4. \uc774\uc9d1\ud2b8 \uc885\uad50\ub294 \uc601\uc9c0\uc8fc\uc758\uc758 \ud0c4\uc0dd\uacfc \uac1c\ub150\ud654\uc5d0 \uc601\ud5a5\uc744 \ub07c\ucce4\ub2e4. \uc608\ub97c \ub4e4\uc5b4 \uc2e0\ube44\uc8fc\uc758\ub294 \uc774\uc9d1\ud2b8\uc778\ub4e4\uc774 \ubbff\uc5c8\ub358 \ube44\ubc00\uc2a4\ub7ec\uc6b4 \ub9c8\ubc95 \uc9c0\uc2dd\uacfc \ud63c\ud569\ub41c \ud1a0\ud2b8 \uc2e0\uc559\uc758 \uc804\ud1b5\uc5d0\uc11c \ube44\ub86f\ub418\uc5c8\ub2e4."} {"question": "\uadfc\ub300 \uc815\ub2f9 \uc81c\ub3c4\ub294 \ubb34\uc5c7\uc744 \ud1b5\ud574\uc11c\uc57c \ube44\ub85c\uc18c \uac00\ub2a5\ud55c\uac00?", "paragraph": "\uc81c\ud5cc, \uc785\ubc95\uacfc\uc815\uc5d0\uc11c \uad6d\ubbfc\uc758\ud68c\uc758 \uc8fc\ubaa9\ud560 \ub9cc\ud55c \uc5c5\uc801\uc5d0\ub3c4 \ubd88\uad6c\ud558\uace0 \ud504\ub791\ud06c \ub85c\ub80c\uce20 \ubb90\ub7ec(Frank Lorenz M\u00fcller)\ub294 \uad6d\ubbfc\uc758\ud68c\uac00 \"\uc801\uadf9\uc801\uc774\uace0 \uc2e4\uc6a9\uc801\uc73c\ub85c \ud589\ub3d9\ud558\ub294 \ub2e8\uccb4\"\ub85c\uae4c\uc9c0 \uc131\uc7a5\ud558\uc9c0\ub294 \ubabb\ud588\uc73c\uba70, \uadf8\uc800 \ub108\ubb34 \uc804\ubb38\uc801\uc774\uace0 \ucca0\ud559\uc801\uc73c\ub85c \ub0a8\uc558\ub2e4\uace0 \uc0dd\uac01\ud55c\ub2e4. \ubca0\ub978\ud558\ub974\ud2b8 \ub9cc(Bernhard Mann)\uc740 \uad6d\ubbfc\uc758\ud68c\uac00 1848\ub144 5\uc6d4\uacfc 6\uc6d4\uc5d0 \uadf8\ub4e4\uc758 \uc720\ub9ac\ud55c \uc0c1\ud669\uc744 \uc9c0\ub098\uce58\uac8c \uc790\uadf9\ud588\uc73c\uba70 \uc815\ubd80 \uc5c6\uc774 \ud5cc\ubc95\uc744 \ub9cc\ub4e4\uc5b4 \uadf8\ub4e4\uc758 \uc9c0\uc704\uac00 \ubd88\uc548\ud588\ub2e4\uace0 \ube44\ub09c\ud55c\ub2e4. \ubfd0\ub9cc \uc544\ub2c8\ub77c \uad6d\ubbfc\uc758\ud68c\ub294 \uc758\ud68c\uc8fc\uc758\uc640 \uc815\ub2f9\ud615\uc131\uc774 \ub3c5\uc77c \uc804\uccb4\uc5d0\uc11c \uc774\ub8e8\uc5b4\uc9c0\uac8c \ud558\uae30 \uc704\ud574\uc11c \uadf8\ub4e4\uc5d0\uac8c \uc644\uc804\ud788 \uc885\uc18d\ub41c \uc911\uc559 \uad8c\ub825\uc744 \ub9cc\ub4e4\uae30 \uc704\ud574 \ub178\ub825\ud588\uc5b4\uc57c \ud588\ub2e4. \ub2e4\ub9cc \uadfc\ub300 \uc815\ub2f9 \uc81c\ub3c4\uac00 \ubb34\uc5c7\ubcf4\ub2e4\ub3c4 \uc5b8\ub860\u00b7\uc9d1\ud68c\u00b7\uacb0\uc0ac\uc758 \uc790\uc720\ub97c \ud1b5\ud574\uc11c\uc57c \ube44\ub85c\uc18c \uac00\ub2a5\ud574\uc84c\ub2e4\ub294 \uc810\uacfc \uc758\ud68c\uc8fc\uc758\uc5d0 \ub300\ud55c \uacbd\ud5d8\uc774 \uc624\uc2a4\ud2b8\ub9ac\uc544 \uc815\uce58\uac00\ub4e4\uc740 \ub9e4\uc6b0 \uc801\uc5c8\uace0 \ud504\ub85c\uc774\uc13c \uc815\uce58\uac00\ub4e4\uc740 \uc5b4\uca0c\ub4e0 \uc804\ud600 \uc5c6\uc5c8\ub2e4\ub294 \uc810\uc740 \ub367\ubd99\uc5ec\uc57c \uc815\ub2f9\ud55c \ud3c9\uac00\uac00 \ub420 \uac83\uc774\ub2e4.", "answer": "\uc5b8\ub860\u00b7\uc9d1\ud68c\u00b7\uacb0\uc0ac\uc758 \uc790\uc720", "sentence": "\ub2e4\ub9cc \uadfc\ub300 \uc815\ub2f9 \uc81c\ub3c4\uac00 \ubb34\uc5c7\ubcf4\ub2e4\ub3c4 \uc5b8\ub860\u00b7\uc9d1\ud68c\u00b7\uacb0\uc0ac\uc758 \uc790\uc720 \ub97c \ud1b5\ud574\uc11c\uc57c \ube44\ub85c\uc18c \uac00\ub2a5\ud574\uc84c\ub2e4\ub294 \uc810\uacfc \uc758\ud68c\uc8fc\uc758\uc5d0 \ub300\ud55c \uacbd\ud5d8\uc774 \uc624\uc2a4\ud2b8\ub9ac\uc544 \uc815\uce58\uac00\ub4e4\uc740 \ub9e4\uc6b0 \uc801\uc5c8\uace0 \ud504\ub85c\uc774\uc13c \uc815\uce58\uac00\ub4e4\uc740 \uc5b4\uca0c\ub4e0 \uc804\ud600 \uc5c6\uc5c8\ub2e4\ub294 \uc810\uc740 \ub367\ubd99\uc5ec\uc57c \uc815\ub2f9\ud55c \ud3c9\uac00\uac00 \ub420 \uac83\uc774\ub2e4.", "paragraph_sentence": "\uc81c\ud5cc, \uc785\ubc95\uacfc\uc815\uc5d0\uc11c \uad6d\ubbfc\uc758\ud68c\uc758 \uc8fc\ubaa9\ud560 \ub9cc\ud55c \uc5c5\uc801\uc5d0\ub3c4 \ubd88\uad6c\ud558\uace0 \ud504\ub791\ud06c \ub85c\ub80c\uce20 \ubb90\ub7ec(Frank Lorenz M\u00fcller)\ub294 \uad6d\ubbfc\uc758\ud68c\uac00 \"\uc801\uadf9\uc801\uc774\uace0 \uc2e4\uc6a9\uc801\uc73c\ub85c \ud589\ub3d9\ud558\ub294 \ub2e8\uccb4\"\ub85c\uae4c\uc9c0 \uc131\uc7a5\ud558\uc9c0\ub294 \ubabb\ud588\uc73c\uba70, \uadf8\uc800 \ub108\ubb34 \uc804\ubb38\uc801\uc774\uace0 \ucca0\ud559\uc801\uc73c\ub85c \ub0a8\uc558\ub2e4\uace0 \uc0dd\uac01\ud55c\ub2e4. \ubca0\ub978\ud558\ub974\ud2b8 \ub9cc(Bernhard Mann)\uc740 \uad6d\ubbfc\uc758\ud68c\uac00 1848\ub144 5\uc6d4\uacfc 6\uc6d4\uc5d0 \uadf8\ub4e4\uc758 \uc720\ub9ac\ud55c \uc0c1\ud669\uc744 \uc9c0\ub098\uce58\uac8c \uc790\uadf9\ud588\uc73c\uba70 \uc815\ubd80 \uc5c6\uc774 \ud5cc\ubc95\uc744 \ub9cc\ub4e4\uc5b4 \uadf8\ub4e4\uc758 \uc9c0\uc704\uac00 \ubd88\uc548\ud588\ub2e4\uace0 \ube44\ub09c\ud55c\ub2e4. \ubfd0\ub9cc \uc544\ub2c8\ub77c \uad6d\ubbfc\uc758\ud68c\ub294 \uc758\ud68c\uc8fc\uc758\uc640 \uc815\ub2f9\ud615\uc131\uc774 \ub3c5\uc77c \uc804\uccb4\uc5d0\uc11c \uc774\ub8e8\uc5b4\uc9c0\uac8c \ud558\uae30 \uc704\ud574\uc11c \uadf8\ub4e4\uc5d0\uac8c \uc644\uc804\ud788 \uc885\uc18d\ub41c \uc911\uc559 \uad8c\ub825\uc744 \ub9cc\ub4e4\uae30 \uc704\ud574 \ub178\ub825\ud588\uc5b4\uc57c \ud588\ub2e4. \ub2e4\ub9cc \uadfc\ub300 \uc815\ub2f9 \uc81c\ub3c4\uac00 \ubb34\uc5c7\ubcf4\ub2e4\ub3c4 \uc5b8\ub860\u00b7\uc9d1\ud68c\u00b7\uacb0\uc0ac\uc758 \uc790\uc720 \ub97c \ud1b5\ud574\uc11c\uc57c \ube44\ub85c\uc18c \uac00\ub2a5\ud574\uc84c\ub2e4\ub294 \uc810\uacfc \uc758\ud68c\uc8fc\uc758\uc5d0 \ub300\ud55c \uacbd\ud5d8\uc774 \uc624\uc2a4\ud2b8\ub9ac\uc544 \uc815\uce58\uac00\ub4e4\uc740 \ub9e4\uc6b0 \uc801\uc5c8\uace0 \ud504\ub85c\uc774\uc13c \uc815\uce58\uac00\ub4e4\uc740 \uc5b4\uca0c\ub4e0 \uc804\ud600 \uc5c6\uc5c8\ub2e4\ub294 \uc810\uc740 \ub367\ubd99\uc5ec\uc57c \uc815\ub2f9\ud55c \ud3c9\uac00\uac00 \ub420 \uac83\uc774\ub2e4. ", "paragraph_answer": "\uc81c\ud5cc, \uc785\ubc95\uacfc\uc815\uc5d0\uc11c \uad6d\ubbfc\uc758\ud68c\uc758 \uc8fc\ubaa9\ud560 \ub9cc\ud55c \uc5c5\uc801\uc5d0\ub3c4 \ubd88\uad6c\ud558\uace0 \ud504\ub791\ud06c \ub85c\ub80c\uce20 \ubb90\ub7ec(Frank Lorenz M\u00fcller)\ub294 \uad6d\ubbfc\uc758\ud68c\uac00 \"\uc801\uadf9\uc801\uc774\uace0 \uc2e4\uc6a9\uc801\uc73c\ub85c \ud589\ub3d9\ud558\ub294 \ub2e8\uccb4\"\ub85c\uae4c\uc9c0 \uc131\uc7a5\ud558\uc9c0\ub294 \ubabb\ud588\uc73c\uba70, \uadf8\uc800 \ub108\ubb34 \uc804\ubb38\uc801\uc774\uace0 \ucca0\ud559\uc801\uc73c\ub85c \ub0a8\uc558\ub2e4\uace0 \uc0dd\uac01\ud55c\ub2e4. \ubca0\ub978\ud558\ub974\ud2b8 \ub9cc(Bernhard Mann)\uc740 \uad6d\ubbfc\uc758\ud68c\uac00 1848\ub144 5\uc6d4\uacfc 6\uc6d4\uc5d0 \uadf8\ub4e4\uc758 \uc720\ub9ac\ud55c \uc0c1\ud669\uc744 \uc9c0\ub098\uce58\uac8c \uc790\uadf9\ud588\uc73c\uba70 \uc815\ubd80 \uc5c6\uc774 \ud5cc\ubc95\uc744 \ub9cc\ub4e4\uc5b4 \uadf8\ub4e4\uc758 \uc9c0\uc704\uac00 \ubd88\uc548\ud588\ub2e4\uace0 \ube44\ub09c\ud55c\ub2e4. \ubfd0\ub9cc \uc544\ub2c8\ub77c \uad6d\ubbfc\uc758\ud68c\ub294 \uc758\ud68c\uc8fc\uc758\uc640 \uc815\ub2f9\ud615\uc131\uc774 \ub3c5\uc77c \uc804\uccb4\uc5d0\uc11c \uc774\ub8e8\uc5b4\uc9c0\uac8c \ud558\uae30 \uc704\ud574\uc11c \uadf8\ub4e4\uc5d0\uac8c \uc644\uc804\ud788 \uc885\uc18d\ub41c \uc911\uc559 \uad8c\ub825\uc744 \ub9cc\ub4e4\uae30 \uc704\ud574 \ub178\ub825\ud588\uc5b4\uc57c \ud588\ub2e4. \ub2e4\ub9cc \uadfc\ub300 \uc815\ub2f9 \uc81c\ub3c4\uac00 \ubb34\uc5c7\ubcf4\ub2e4\ub3c4 \uc5b8\ub860\u00b7\uc9d1\ud68c\u00b7\uacb0\uc0ac\uc758 \uc790\uc720 \ub97c \ud1b5\ud574\uc11c\uc57c \ube44\ub85c\uc18c \uac00\ub2a5\ud574\uc84c\ub2e4\ub294 \uc810\uacfc \uc758\ud68c\uc8fc\uc758\uc5d0 \ub300\ud55c \uacbd\ud5d8\uc774 \uc624\uc2a4\ud2b8\ub9ac\uc544 \uc815\uce58\uac00\ub4e4\uc740 \ub9e4\uc6b0 \uc801\uc5c8\uace0 \ud504\ub85c\uc774\uc13c \uc815\uce58\uac00\ub4e4\uc740 \uc5b4\uca0c\ub4e0 \uc804\ud600 \uc5c6\uc5c8\ub2e4\ub294 \uc810\uc740 \ub367\ubd99\uc5ec\uc57c \uc815\ub2f9\ud55c \ud3c9\uac00\uac00 \ub420 \uac83\uc774\ub2e4.", "sentence_answer": "\ub2e4\ub9cc \uadfc\ub300 \uc815\ub2f9 \uc81c\ub3c4\uac00 \ubb34\uc5c7\ubcf4\ub2e4\ub3c4 \uc5b8\ub860\u00b7\uc9d1\ud68c\u00b7\uacb0\uc0ac\uc758 \uc790\uc720 \ub97c \ud1b5\ud574\uc11c\uc57c \ube44\ub85c\uc18c \uac00\ub2a5\ud574\uc84c\ub2e4\ub294 \uc810\uacfc \uc758\ud68c\uc8fc\uc758\uc5d0 \ub300\ud55c \uacbd\ud5d8\uc774 \uc624\uc2a4\ud2b8\ub9ac\uc544 \uc815\uce58\uac00\ub4e4\uc740 \ub9e4\uc6b0 \uc801\uc5c8\uace0 \ud504\ub85c\uc774\uc13c \uc815\uce58\uac00\ub4e4\uc740 \uc5b4\uca0c\ub4e0 \uc804\ud600 \uc5c6\uc5c8\ub2e4\ub294 \uc810\uc740 \ub367\ubd99\uc5ec\uc57c \uc815\ub2f9\ud55c \ud3c9\uac00\uac00 \ub420 \uac83\uc774\ub2e4.", "paragraph_id": "6465718-33-1", "paragraph_question": "question: \uadfc\ub300 \uc815\ub2f9 \uc81c\ub3c4\ub294 \ubb34\uc5c7\uc744 \ud1b5\ud574\uc11c\uc57c \ube44\ub85c\uc18c \uac00\ub2a5\ud55c\uac00?, context: \uc81c\ud5cc, \uc785\ubc95\uacfc\uc815\uc5d0\uc11c \uad6d\ubbfc\uc758\ud68c\uc758 \uc8fc\ubaa9\ud560 \ub9cc\ud55c \uc5c5\uc801\uc5d0\ub3c4 \ubd88\uad6c\ud558\uace0 \ud504\ub791\ud06c \ub85c\ub80c\uce20 \ubb90\ub7ec(Frank Lorenz M\u00fcller)\ub294 \uad6d\ubbfc\uc758\ud68c\uac00 \"\uc801\uadf9\uc801\uc774\uace0 \uc2e4\uc6a9\uc801\uc73c\ub85c \ud589\ub3d9\ud558\ub294 \ub2e8\uccb4\"\ub85c\uae4c\uc9c0 \uc131\uc7a5\ud558\uc9c0\ub294 \ubabb\ud588\uc73c\uba70, \uadf8\uc800 \ub108\ubb34 \uc804\ubb38\uc801\uc774\uace0 \ucca0\ud559\uc801\uc73c\ub85c \ub0a8\uc558\ub2e4\uace0 \uc0dd\uac01\ud55c\ub2e4. \ubca0\ub978\ud558\ub974\ud2b8 \ub9cc(Bernhard Mann)\uc740 \uad6d\ubbfc\uc758\ud68c\uac00 1848\ub144 5\uc6d4\uacfc 6\uc6d4\uc5d0 \uadf8\ub4e4\uc758 \uc720\ub9ac\ud55c \uc0c1\ud669\uc744 \uc9c0\ub098\uce58\uac8c \uc790\uadf9\ud588\uc73c\uba70 \uc815\ubd80 \uc5c6\uc774 \ud5cc\ubc95\uc744 \ub9cc\ub4e4\uc5b4 \uadf8\ub4e4\uc758 \uc9c0\uc704\uac00 \ubd88\uc548\ud588\ub2e4\uace0 \ube44\ub09c\ud55c\ub2e4. \ubfd0\ub9cc \uc544\ub2c8\ub77c \uad6d\ubbfc\uc758\ud68c\ub294 \uc758\ud68c\uc8fc\uc758\uc640 \uc815\ub2f9\ud615\uc131\uc774 \ub3c5\uc77c \uc804\uccb4\uc5d0\uc11c \uc774\ub8e8\uc5b4\uc9c0\uac8c \ud558\uae30 \uc704\ud574\uc11c \uadf8\ub4e4\uc5d0\uac8c \uc644\uc804\ud788 \uc885\uc18d\ub41c \uc911\uc559 \uad8c\ub825\uc744 \ub9cc\ub4e4\uae30 \uc704\ud574 \ub178\ub825\ud588\uc5b4\uc57c \ud588\ub2e4. \ub2e4\ub9cc \uadfc\ub300 \uc815\ub2f9 \uc81c\ub3c4\uac00 \ubb34\uc5c7\ubcf4\ub2e4\ub3c4 \uc5b8\ub860\u00b7\uc9d1\ud68c\u00b7\uacb0\uc0ac\uc758 \uc790\uc720\ub97c \ud1b5\ud574\uc11c\uc57c \ube44\ub85c\uc18c \uac00\ub2a5\ud574\uc84c\ub2e4\ub294 \uc810\uacfc \uc758\ud68c\uc8fc\uc758\uc5d0 \ub300\ud55c \uacbd\ud5d8\uc774 \uc624\uc2a4\ud2b8\ub9ac\uc544 \uc815\uce58\uac00\ub4e4\uc740 \ub9e4\uc6b0 \uc801\uc5c8\uace0 \ud504\ub85c\uc774\uc13c \uc815\uce58\uac00\ub4e4\uc740 \uc5b4\uca0c\ub4e0 \uc804\ud600 \uc5c6\uc5c8\ub2e4\ub294 \uc810\uc740 \ub367\ubd99\uc5ec\uc57c \uc815\ub2f9\ud55c \ud3c9\uac00\uac00 \ub420 \uac83\uc774\ub2e4."} {"question": "\uae40\ud0dc\ub9ac\uac00 \ub300\ud559\ub54c \ucd9c\uc5f0\ud55c \ub3c5\ub9bd\uc601\ud654 \uc81c\ubaa9\uc740?", "paragraph": "\uae40\ud0dc\ub9ac\ub294 1990\ub144 4\uc6d4 24\uc77c \uc11c\uc6b8\uc5d0\uc11c 1\ub0a8 1\ub140 \uc911 \ub9c9\ub0b4\ub85c \ud0dc\uc5b4\ub0ac\uc73c\uba70, 2\uc0b4 \uc704\uc758 \uc624\ube60\uac00 \uc788\ub2e4. \uc5b4\ub9b0 \uc2dc\uc808\ubd80\ud130 \ub370\ubdd4 \ucd08\uae4c\uc9c0\ub294 \ud560\uba38\ub2c8\uc640 \uc0dd\ud65c\ud588\uace0 \ud604\uc7ac\ub294 \ub3c5\ub9bd\ud574 \ud63c\uc790 \uc0b4\uace0 \uc788\ub2e4. \uae40\ud0dc\ub9ac\ub77c\ub294 \uc774\ub984\uc740 \ubcf8\uba85\uc73c\ub85c '\ud0dc'(\u6cf0)\ub294 \uc9d1\uc548\uc758 \ub3cc\ub9bc\uc790\uace0 '\ub9ac'(\u68a8)\ub294 \ubc30\ub098\ubb34\ub97c \ub73b\ud55c\ub2e4. \ud0dc\uc5b4\ub0a0 \ub54c \ub3d9\ub124\uc5d0 \ubc30\uaf43\uc774 \ub9cc\uac1c\ud55c \uac83\uc5d0\uc11c \ub530\uc628 \uc774\ub984\uc774\ub2e4. \uc5b4\uba38\ub2c8\ub294 \uc815\uce58\ub97c \ud558\uae38 \ubc14\ub77c\ub294 \ub9c8\uc74c\uc5d0\uc11c '\ud0dc\uc815'\uc744 \uc6d0\ud558\uc168\uc9c0\ub9cc \uc544\ubc84\uc9c0\uac00 \ucd9c\uc0dd\uc2e0\uace0 \ud558\ub7ec \uac00\ub294 \uae38\uc5d0 '\ud0dc\ub9ac'\ub85c \ubc14\uafb8\uc168\ub2e4\uace0 \ud55c\ub2e4.\uc601\uc2e0\uc5ec\uc790\uc2e4\uc5c5\uace0\ub4f1\ud559\uad50 \ub514\uc790\uc778\uacfc\uc5d0 \uc9c4\ud559\ud55c \ud6c4, \ub9c9\uc5f0\ud788 \uc544\ub098\uc6b4\uc11c\uac00 \ub418\uace0 \uc2f6\uc740 \uc0dd\uac01\uc5d0 \uacbd\ud76c\ub300\ud559\uad50 \uc2e0\ubb38\ubc29\uc1a1\ud559\uacfc\uc5d0 \uc785\ud559\ud588\ub2e4. \uc774\ub54c\uae4c\uc9c0\ub9cc \ud574\ub3c4 \uc5f0\uae30, \uc5f0\uadf9, \uc601\ud654\uc5d0 \ub300\ud55c \uad00\uc2ec\uc740 \ud558\ub098\ub3c4 \uc5c6\uc5c8\ub2e4. \uae40\ud0dc\ub9ac\uac00 \ucc98\uc74c \uc5f0\uae30\ub97c \uc811\ud55c \uacc4\uae30\ub294 \ub300\ud559 \uc5f0\uadf9 \ub3d9\uc544\ub9ac. 1\ud559\ub144 \ub54c \uc2e0\uc785\ub2e8\uc6d0\uc744 \ubaa8\uc9d1\ud558\ub294 \uc5f0\uadf9\uc744 \ubcf4\uace0 \uc5f0\uadf9 \ub3d9\uc544\ub9ac\uc5d0 \uac00\uc785\ud588\uace0, 2\ud559\ub144 \ub54c \uc9c4\ub85c\ub97c \uc5f0\uadf9\ubc30\uc6b0\ub85c \uacb0\uc815\ud588\ub2e4\uace0 \ud55c\ub2e4. \ub3d9\uc544\ub9ac \ud65c\ub3d9\uc744 \ud558\uba70 \ub3c5\ub9bd \uc601\ud654 \u300a\uc2dc\ubbfc\uc880\ube44\u300b\uc5d0 \ucd9c\uc5f0\ud558\uae30\ub3c4 \ud588\uc73c\uba70, \ub300\ud559\uc744 \uc878\uc5c5\ud560 \uc988\uc74c\uc5d4 \ub300\ud559\ub85c\ub85c \ub098\uac00 \uadf9\ub2e8\uc5d0 \ub4e4\uc5b4\uac14\ub2e4.", "answer": "\uc2dc\ubbfc\uc880\ube44", "sentence": "\ub3c5\ub9bd \uc601\ud654 \u300a \uc2dc\ubbfc\uc880\ube44 \u300b\uc5d0 \ucd9c\uc5f0\ud558\uae30\ub3c4 \ud588\uc73c\uba70, \ub300\ud559\uc744 \uc878\uc5c5\ud560 \uc988\uc74c\uc5d4 \ub300\ud559\ub85c\ub85c \ub098\uac00 \uadf9\ub2e8\uc5d0 \ub4e4\uc5b4\uac14\ub2e4.", "paragraph_sentence": "\uae40\ud0dc\ub9ac\ub294 1990\ub144 4\uc6d4 24\uc77c \uc11c\uc6b8\uc5d0\uc11c 1\ub0a8 1\ub140 \uc911 \ub9c9\ub0b4\ub85c \ud0dc\uc5b4\ub0ac\uc73c\uba70, 2\uc0b4 \uc704\uc758 \uc624\ube60\uac00 \uc788\ub2e4. \uc5b4\ub9b0 \uc2dc\uc808\ubd80\ud130 \ub370\ubdd4 \ucd08\uae4c\uc9c0\ub294 \ud560\uba38\ub2c8\uc640 \uc0dd\ud65c\ud588\uace0 \ud604\uc7ac\ub294 \ub3c5\ub9bd\ud574 \ud63c\uc790 \uc0b4\uace0 \uc788\ub2e4. \uae40\ud0dc\ub9ac\ub77c\ub294 \uc774\ub984\uc740 \ubcf8\uba85\uc73c\ub85c '\ud0dc'(\u6cf0)\ub294 \uc9d1\uc548\uc758 \ub3cc\ub9bc\uc790\uace0 '\ub9ac'(\u68a8)\ub294 \ubc30\ub098\ubb34\ub97c \ub73b\ud55c\ub2e4. \ud0dc\uc5b4\ub0a0 \ub54c \ub3d9\ub124\uc5d0 \ubc30\uaf43\uc774 \ub9cc\uac1c\ud55c \uac83\uc5d0\uc11c \ub530\uc628 \uc774\ub984\uc774\ub2e4. \uc5b4\uba38\ub2c8\ub294 \uc815\uce58\ub97c \ud558\uae38 \ubc14\ub77c\ub294 \ub9c8\uc74c\uc5d0\uc11c '\ud0dc\uc815'\uc744 \uc6d0\ud558\uc168\uc9c0\ub9cc \uc544\ubc84\uc9c0\uac00 \ucd9c\uc0dd\uc2e0\uace0 \ud558\ub7ec \uac00\ub294 \uae38\uc5d0 '\ud0dc\ub9ac'\ub85c \ubc14\uafb8\uc168\ub2e4\uace0 \ud55c\ub2e4.\uc601\uc2e0\uc5ec\uc790\uc2e4\uc5c5\uace0\ub4f1\ud559\uad50 \ub514\uc790\uc778\uacfc\uc5d0 \uc9c4\ud559\ud55c \ud6c4, \ub9c9\uc5f0\ud788 \uc544\ub098\uc6b4\uc11c\uac00 \ub418\uace0 \uc2f6\uc740 \uc0dd\uac01\uc5d0 \uacbd\ud76c\ub300\ud559\uad50 \uc2e0\ubb38\ubc29\uc1a1\ud559\uacfc\uc5d0 \uc785\ud559\ud588\ub2e4. \uc774\ub54c\uae4c\uc9c0\ub9cc \ud574\ub3c4 \uc5f0\uae30, \uc5f0\uadf9, \uc601\ud654\uc5d0 \ub300\ud55c \uad00\uc2ec\uc740 \ud558\ub098\ub3c4 \uc5c6\uc5c8\ub2e4. \uae40\ud0dc\ub9ac\uac00 \ucc98\uc74c \uc5f0\uae30\ub97c \uc811\ud55c \uacc4\uae30\ub294 \ub300\ud559 \uc5f0\uadf9 \ub3d9\uc544\ub9ac. 1\ud559\ub144 \ub54c \uc2e0\uc785\ub2e8\uc6d0\uc744 \ubaa8\uc9d1\ud558\ub294 \uc5f0\uadf9\uc744 \ubcf4\uace0 \uc5f0\uadf9 \ub3d9\uc544\ub9ac\uc5d0 \uac00\uc785\ud588\uace0, 2\ud559\ub144 \ub54c \uc9c4\ub85c\ub97c \uc5f0\uadf9\ubc30\uc6b0\ub85c \uacb0\uc815\ud588\ub2e4\uace0 \ud55c\ub2e4. \ub3d9\uc544\ub9ac \ud65c\ub3d9\uc744 \ud558\uba70 \ub3c5\ub9bd \uc601\ud654 \u300a \uc2dc\ubbfc\uc880\ube44 \u300b\uc5d0 \ucd9c\uc5f0\ud558\uae30\ub3c4 \ud588\uc73c\uba70, \ub300\ud559\uc744 \uc878\uc5c5\ud560 \uc988\uc74c\uc5d4 \ub300\ud559\ub85c\ub85c \ub098\uac00 \uadf9\ub2e8\uc5d0 \ub4e4\uc5b4\uac14\ub2e4. ", "paragraph_answer": "\uae40\ud0dc\ub9ac\ub294 1990\ub144 4\uc6d4 24\uc77c \uc11c\uc6b8\uc5d0\uc11c 1\ub0a8 1\ub140 \uc911 \ub9c9\ub0b4\ub85c \ud0dc\uc5b4\ub0ac\uc73c\uba70, 2\uc0b4 \uc704\uc758 \uc624\ube60\uac00 \uc788\ub2e4. \uc5b4\ub9b0 \uc2dc\uc808\ubd80\ud130 \ub370\ubdd4 \ucd08\uae4c\uc9c0\ub294 \ud560\uba38\ub2c8\uc640 \uc0dd\ud65c\ud588\uace0 \ud604\uc7ac\ub294 \ub3c5\ub9bd\ud574 \ud63c\uc790 \uc0b4\uace0 \uc788\ub2e4. \uae40\ud0dc\ub9ac\ub77c\ub294 \uc774\ub984\uc740 \ubcf8\uba85\uc73c\ub85c '\ud0dc'(\u6cf0)\ub294 \uc9d1\uc548\uc758 \ub3cc\ub9bc\uc790\uace0 '\ub9ac'(\u68a8)\ub294 \ubc30\ub098\ubb34\ub97c \ub73b\ud55c\ub2e4. \ud0dc\uc5b4\ub0a0 \ub54c \ub3d9\ub124\uc5d0 \ubc30\uaf43\uc774 \ub9cc\uac1c\ud55c \uac83\uc5d0\uc11c \ub530\uc628 \uc774\ub984\uc774\ub2e4. \uc5b4\uba38\ub2c8\ub294 \uc815\uce58\ub97c \ud558\uae38 \ubc14\ub77c\ub294 \ub9c8\uc74c\uc5d0\uc11c '\ud0dc\uc815'\uc744 \uc6d0\ud558\uc168\uc9c0\ub9cc \uc544\ubc84\uc9c0\uac00 \ucd9c\uc0dd\uc2e0\uace0 \ud558\ub7ec \uac00\ub294 \uae38\uc5d0 '\ud0dc\ub9ac'\ub85c \ubc14\uafb8\uc168\ub2e4\uace0 \ud55c\ub2e4.\uc601\uc2e0\uc5ec\uc790\uc2e4\uc5c5\uace0\ub4f1\ud559\uad50 \ub514\uc790\uc778\uacfc\uc5d0 \uc9c4\ud559\ud55c \ud6c4, \ub9c9\uc5f0\ud788 \uc544\ub098\uc6b4\uc11c\uac00 \ub418\uace0 \uc2f6\uc740 \uc0dd\uac01\uc5d0 \uacbd\ud76c\ub300\ud559\uad50 \uc2e0\ubb38\ubc29\uc1a1\ud559\uacfc\uc5d0 \uc785\ud559\ud588\ub2e4. \uc774\ub54c\uae4c\uc9c0\ub9cc \ud574\ub3c4 \uc5f0\uae30, \uc5f0\uadf9, \uc601\ud654\uc5d0 \ub300\ud55c \uad00\uc2ec\uc740 \ud558\ub098\ub3c4 \uc5c6\uc5c8\ub2e4. \uae40\ud0dc\ub9ac\uac00 \ucc98\uc74c \uc5f0\uae30\ub97c \uc811\ud55c \uacc4\uae30\ub294 \ub300\ud559 \uc5f0\uadf9 \ub3d9\uc544\ub9ac. 1\ud559\ub144 \ub54c \uc2e0\uc785\ub2e8\uc6d0\uc744 \ubaa8\uc9d1\ud558\ub294 \uc5f0\uadf9\uc744 \ubcf4\uace0 \uc5f0\uadf9 \ub3d9\uc544\ub9ac\uc5d0 \uac00\uc785\ud588\uace0, 2\ud559\ub144 \ub54c \uc9c4\ub85c\ub97c \uc5f0\uadf9\ubc30\uc6b0\ub85c \uacb0\uc815\ud588\ub2e4\uace0 \ud55c\ub2e4. \ub3d9\uc544\ub9ac \ud65c\ub3d9\uc744 \ud558\uba70 \ub3c5\ub9bd \uc601\ud654 \u300a \uc2dc\ubbfc\uc880\ube44 \u300b\uc5d0 \ucd9c\uc5f0\ud558\uae30\ub3c4 \ud588\uc73c\uba70, \ub300\ud559\uc744 \uc878\uc5c5\ud560 \uc988\uc74c\uc5d4 \ub300\ud559\ub85c\ub85c \ub098\uac00 \uadf9\ub2e8\uc5d0 \ub4e4\uc5b4\uac14\ub2e4.", "sentence_answer": "\ub3c5\ub9bd \uc601\ud654 \u300a \uc2dc\ubbfc\uc880\ube44 \u300b\uc5d0 \ucd9c\uc5f0\ud558\uae30\ub3c4 \ud588\uc73c\uba70, \ub300\ud559\uc744 \uc878\uc5c5\ud560 \uc988\uc74c\uc5d4 \ub300\ud559\ub85c\ub85c \ub098\uac00 \uadf9\ub2e8\uc5d0 \ub4e4\uc5b4\uac14\ub2e4.", "paragraph_id": "6601912-0-0", "paragraph_question": "question: \uae40\ud0dc\ub9ac\uac00 \ub300\ud559\ub54c \ucd9c\uc5f0\ud55c \ub3c5\ub9bd\uc601\ud654 \uc81c\ubaa9\uc740?, context: \uae40\ud0dc\ub9ac\ub294 1990\ub144 4\uc6d4 24\uc77c \uc11c\uc6b8\uc5d0\uc11c 1\ub0a8 1\ub140 \uc911 \ub9c9\ub0b4\ub85c \ud0dc\uc5b4\ub0ac\uc73c\uba70, 2\uc0b4 \uc704\uc758 \uc624\ube60\uac00 \uc788\ub2e4. \uc5b4\ub9b0 \uc2dc\uc808\ubd80\ud130 \ub370\ubdd4 \ucd08\uae4c\uc9c0\ub294 \ud560\uba38\ub2c8\uc640 \uc0dd\ud65c\ud588\uace0 \ud604\uc7ac\ub294 \ub3c5\ub9bd\ud574 \ud63c\uc790 \uc0b4\uace0 \uc788\ub2e4. \uae40\ud0dc\ub9ac\ub77c\ub294 \uc774\ub984\uc740 \ubcf8\uba85\uc73c\ub85c '\ud0dc'(\u6cf0)\ub294 \uc9d1\uc548\uc758 \ub3cc\ub9bc\uc790\uace0 '\ub9ac'(\u68a8)\ub294 \ubc30\ub098\ubb34\ub97c \ub73b\ud55c\ub2e4. \ud0dc\uc5b4\ub0a0 \ub54c \ub3d9\ub124\uc5d0 \ubc30\uaf43\uc774 \ub9cc\uac1c\ud55c \uac83\uc5d0\uc11c \ub530\uc628 \uc774\ub984\uc774\ub2e4. \uc5b4\uba38\ub2c8\ub294 \uc815\uce58\ub97c \ud558\uae38 \ubc14\ub77c\ub294 \ub9c8\uc74c\uc5d0\uc11c '\ud0dc\uc815'\uc744 \uc6d0\ud558\uc168\uc9c0\ub9cc \uc544\ubc84\uc9c0\uac00 \ucd9c\uc0dd\uc2e0\uace0 \ud558\ub7ec \uac00\ub294 \uae38\uc5d0 '\ud0dc\ub9ac'\ub85c \ubc14\uafb8\uc168\ub2e4\uace0 \ud55c\ub2e4.\uc601\uc2e0\uc5ec\uc790\uc2e4\uc5c5\uace0\ub4f1\ud559\uad50 \ub514\uc790\uc778\uacfc\uc5d0 \uc9c4\ud559\ud55c \ud6c4, \ub9c9\uc5f0\ud788 \uc544\ub098\uc6b4\uc11c\uac00 \ub418\uace0 \uc2f6\uc740 \uc0dd\uac01\uc5d0 \uacbd\ud76c\ub300\ud559\uad50 \uc2e0\ubb38\ubc29\uc1a1\ud559\uacfc\uc5d0 \uc785\ud559\ud588\ub2e4. \uc774\ub54c\uae4c\uc9c0\ub9cc \ud574\ub3c4 \uc5f0\uae30, \uc5f0\uadf9, \uc601\ud654\uc5d0 \ub300\ud55c \uad00\uc2ec\uc740 \ud558\ub098\ub3c4 \uc5c6\uc5c8\ub2e4. \uae40\ud0dc\ub9ac\uac00 \ucc98\uc74c \uc5f0\uae30\ub97c \uc811\ud55c \uacc4\uae30\ub294 \ub300\ud559 \uc5f0\uadf9 \ub3d9\uc544\ub9ac. 1\ud559\ub144 \ub54c \uc2e0\uc785\ub2e8\uc6d0\uc744 \ubaa8\uc9d1\ud558\ub294 \uc5f0\uadf9\uc744 \ubcf4\uace0 \uc5f0\uadf9 \ub3d9\uc544\ub9ac\uc5d0 \uac00\uc785\ud588\uace0, 2\ud559\ub144 \ub54c \uc9c4\ub85c\ub97c \uc5f0\uadf9\ubc30\uc6b0\ub85c \uacb0\uc815\ud588\ub2e4\uace0 \ud55c\ub2e4. \ub3d9\uc544\ub9ac \ud65c\ub3d9\uc744 \ud558\uba70 \ub3c5\ub9bd \uc601\ud654 \u300a\uc2dc\ubbfc\uc880\ube44\u300b\uc5d0 \ucd9c\uc5f0\ud558\uae30\ub3c4 \ud588\uc73c\uba70, \ub300\ud559\uc744 \uc878\uc5c5\ud560 \uc988\uc74c\uc5d4 \ub300\ud559\ub85c\ub85c \ub098\uac00 \uadf9\ub2e8\uc5d0 \ub4e4\uc5b4\uac14\ub2e4."} {"question": "\ubaa8\ub85c\uc758 \ub3d9\ud5a5\uc778\ub4e4\uc774 \ubaa8\ub85c\ub97c \ub4a4\ubb34\ub9ac\uc5d0\uc640 \ud53c\uc288\uadf8\ub93c\uc640 \ube44\uc720\ud558\uba70 \uc9c0\uce6d\ud558\ub294 \ub9d0\uc740?", "paragraph": "\ubaa8\ub85c\ub294 \uc544\ub0b4\uc758 \uc120\ub3d9\uc73c\ub85c \uc720\ub7fd\uc5d0 \ub3cc\uc544\uc634\uacfc \ub3d9\uc2dc\uc5d0 \uacf5\ud654\uc8fc\uc758\uc790 \uc74c\ubaa8\uc790\ub4e4\uc758 \ubaa8\uc784\uc758 \uc61b \uce5c\uad6c\ub4e4\uacfc \ud611\uc0c1\ud558\uae30 \uc2dc\uc791\ud588\ub2e4:\uc774\uc804 \uc7a5-\ubc25\ud2f0\uc2a4\ud2b8 \ubca0\ub974\ub098\ub3c4\ud2b8, \ud604\uc7ac \uc2a4\uc6e8\ub374\uc758 \uce74\ub97c \uc694\ud55c \uacf5(\ud6d7\ub0a0 \uad6d\uc655 \uc2a4\uc6e8\ub374\uc758 \uce74\ub97c 14\uc138 \uc694\ud55c). \uce74\ub97c \uc694\ud55c\uacfc \ucc28\ub974 \ub7ec\uc2dc\uc544\uc758 \uc54c\ub809\uc0b0\ub4dc\ub974 1\uc138\ub294 \ub098\ud3f4\ub808\uc639\uc744 \uc0c1\ub300\ub85c \ud568\uaed8 \uad70\ub300\ub97c \uc77c\uc73c\ucf30\uace0, \ud504\ub85c\uc774\uc13c\uacfc \uc624\uc2a4\ud2b8\ub9ac\uc544\ub97c \uc774\ub04c\uc5c8\ub2e4. \ubaa8\ub85c\ub294 \ub098\ud3f4\ub808\uc639\uc758 \ud328\ubc30\uc640 \uacf5\ud654\uad6d \uc815\ubd80\uac00 \uc138\uc6cc\uc9c0\ub294 \uac83\uc744 \ubcf4\uae30\ub97c \ud76c\ub9dd\ud588\ub2e4. \ubaa8\ub85c\ub294 \uc2a4\uc6e8\ub374\uacfc \ub7ec\uc2dc\uc544\uc758 \uc9c0\ub3c4\uc790\ub4e4\uc5d0\uac8c \ud504\ub791\uc2a4\ub97c \ud328\ubc30\uc2dc\ud0a4\ub294 \uac00\uc7a5 \ucd5c\uc120\uc758 \ubc29\ubc95 \ub4f1\uc744 \uc870\uc5b8\ud558\ub294 \uc5ed\ud560\uc744 \ub9e1\uc558\ub2e4. \ubaa8\ub85c\ub294 1813\ub144 8\uc6d4 27\uc77c \ub4dc\ub808\uc2a4\ub374 \uc804\ud22c\uc5d0\uc11c \uc911\uc0c1\uc744 \uc785\uace0 \ucc28\ub974\uc640 \ub300\ud654\ub97c \ub098\ub208 \ub4a4 \ub77c\uc6b0\ub2c8(Louny)\uc5d0\uc11c 9\uc6d4 2\uc77c \uc0ac\ub9dd\ud588\ub2e4. \ubaa8\ub85c\ub294 \uc0c1\ud2b8\ud398\ud14c\ub974\ubd80\ub974\ud06c\uc758 \uc131 \uce74\ud0c0\ub9ac\ub098 \uac00\ud1a8\ub9ad \uad50\ud68c(Catholic Church of St. Catherine)\uc5d0 \ub9e4\uc7a5\ub418\uc5c8\ub2e4. \ubaa8\ub85c\uc758 \uc544\ub0b4\ub294 \ucc28\ub974\ub85c\ubd80\ud130 \uc5f0\uae08\uc744 \ubc1b\uc558\uace0, \ub8e8\uc774 18\uc138\ub85c\ubd80\ud130 \uc6d0\uc218(mar\u00e9chal)\uc758 \uc9c0\uc704\ub97c \ud558\uc0ac\ubc1b\uc558\uc73c\ub098 \ubaa8\ub85c\uc758 \ub3d9\ud5a5\uc778\ub4e4\uc740 \ubaa8\ub85c\uc5d0 \ub300\ud574 \uc774\uc57c\uae30\ud560 \ub54c \"\ubcc0\uc808\uc790\" \ub77c\uba70 \ub4a4\ubb34\ub9ac\uc5d0\uc640 \ud53c\uc288\uadf8\ub93c\uc5d0 \ube44\uc720\ud588\ub2e4.", "answer": "\ubcc0\uc808\uc790", "sentence": "\ubaa8\ub85c\uc758 \uc544\ub0b4\ub294 \ucc28\ub974\ub85c\ubd80\ud130 \uc5f0\uae08\uc744 \ubc1b\uc558\uace0, \ub8e8\uc774 18\uc138\ub85c\ubd80\ud130 \uc6d0\uc218(mar\u00e9chal)\uc758 \uc9c0\uc704\ub97c \ud558\uc0ac\ubc1b\uc558\uc73c\ub098 \ubaa8\ub85c\uc758 \ub3d9\ud5a5\uc778\ub4e4\uc740 \ubaa8\ub85c\uc5d0 \ub300\ud574 \uc774\uc57c\uae30\ud560 \ub54c \" \ubcc0\uc808\uc790 \"", "paragraph_sentence": "\ubaa8\ub85c\ub294 \uc544\ub0b4\uc758 \uc120\ub3d9\uc73c\ub85c \uc720\ub7fd\uc5d0 \ub3cc\uc544\uc634\uacfc \ub3d9\uc2dc\uc5d0 \uacf5\ud654\uc8fc\uc758\uc790 \uc74c\ubaa8\uc790\ub4e4\uc758 \ubaa8\uc784\uc758 \uc61b \uce5c\uad6c\ub4e4\uacfc \ud611\uc0c1\ud558\uae30 \uc2dc\uc791\ud588\ub2e4:\uc774\uc804 \uc7a5-\ubc25\ud2f0\uc2a4\ud2b8 \ubca0\ub974\ub098\ub3c4\ud2b8, \ud604\uc7ac \uc2a4\uc6e8\ub374\uc758 \uce74\ub97c \uc694\ud55c \uacf5(\ud6d7\ub0a0 \uad6d\uc655 \uc2a4\uc6e8\ub374\uc758 \uce74\ub97c 14\uc138 \uc694\ud55c). \uce74\ub97c \uc694\ud55c\uacfc \ucc28\ub974 \ub7ec\uc2dc\uc544\uc758 \uc54c\ub809\uc0b0\ub4dc\ub974 1\uc138\ub294 \ub098\ud3f4\ub808\uc639\uc744 \uc0c1\ub300\ub85c \ud568\uaed8 \uad70\ub300\ub97c \uc77c\uc73c\ucf30\uace0, \ud504\ub85c\uc774\uc13c\uacfc \uc624\uc2a4\ud2b8\ub9ac\uc544\ub97c \uc774\ub04c\uc5c8\ub2e4. \ubaa8\ub85c\ub294 \ub098\ud3f4\ub808\uc639\uc758 \ud328\ubc30\uc640 \uacf5\ud654\uad6d \uc815\ubd80\uac00 \uc138\uc6cc\uc9c0\ub294 \uac83\uc744 \ubcf4\uae30\ub97c \ud76c\ub9dd\ud588\ub2e4. \ubaa8\ub85c\ub294 \uc2a4\uc6e8\ub374\uacfc \ub7ec\uc2dc\uc544\uc758 \uc9c0\ub3c4\uc790\ub4e4\uc5d0\uac8c \ud504\ub791\uc2a4\ub97c \ud328\ubc30\uc2dc\ud0a4\ub294 \uac00\uc7a5 \ucd5c\uc120\uc758 \ubc29\ubc95 \ub4f1\uc744 \uc870\uc5b8\ud558\ub294 \uc5ed\ud560\uc744 \ub9e1\uc558\ub2e4. \ubaa8\ub85c\ub294 1813\ub144 8\uc6d4 27\uc77c \ub4dc\ub808\uc2a4\ub374 \uc804\ud22c\uc5d0\uc11c \uc911\uc0c1\uc744 \uc785\uace0 \ucc28\ub974\uc640 \ub300\ud654\ub97c \ub098\ub208 \ub4a4 \ub77c\uc6b0\ub2c8(Louny)\uc5d0\uc11c 9\uc6d4 2\uc77c \uc0ac\ub9dd\ud588\ub2e4. \ubaa8\ub85c\ub294 \uc0c1\ud2b8\ud398\ud14c\ub974\ubd80\ub974\ud06c\uc758 \uc131 \uce74\ud0c0\ub9ac\ub098 \uac00\ud1a8\ub9ad \uad50\ud68c(Catholic Church of St. Catherine)\uc5d0 \ub9e4\uc7a5\ub418\uc5c8\ub2e4. \ubaa8\ub85c\uc758 \uc544\ub0b4\ub294 \ucc28\ub974\ub85c\ubd80\ud130 \uc5f0\uae08\uc744 \ubc1b\uc558\uace0, \ub8e8\uc774 18\uc138\ub85c\ubd80\ud130 \uc6d0\uc218(mar\u00e9chal)\uc758 \uc9c0\uc704\ub97c \ud558\uc0ac\ubc1b\uc558\uc73c\ub098 \ubaa8\ub85c\uc758 \ub3d9\ud5a5\uc778\ub4e4\uc740 \ubaa8\ub85c\uc5d0 \ub300\ud574 \uc774\uc57c\uae30\ud560 \ub54c \" \ubcc0\uc808\uc790 \" \ub77c\uba70 \ub4a4\ubb34\ub9ac\uc5d0\uc640 \ud53c\uc288\uadf8\ub93c\uc5d0 \ube44\uc720\ud588\ub2e4.", "paragraph_answer": "\ubaa8\ub85c\ub294 \uc544\ub0b4\uc758 \uc120\ub3d9\uc73c\ub85c \uc720\ub7fd\uc5d0 \ub3cc\uc544\uc634\uacfc \ub3d9\uc2dc\uc5d0 \uacf5\ud654\uc8fc\uc758\uc790 \uc74c\ubaa8\uc790\ub4e4\uc758 \ubaa8\uc784\uc758 \uc61b \uce5c\uad6c\ub4e4\uacfc \ud611\uc0c1\ud558\uae30 \uc2dc\uc791\ud588\ub2e4:\uc774\uc804 \uc7a5-\ubc25\ud2f0\uc2a4\ud2b8 \ubca0\ub974\ub098\ub3c4\ud2b8, \ud604\uc7ac \uc2a4\uc6e8\ub374\uc758 \uce74\ub97c \uc694\ud55c \uacf5(\ud6d7\ub0a0 \uad6d\uc655 \uc2a4\uc6e8\ub374\uc758 \uce74\ub97c 14\uc138 \uc694\ud55c). \uce74\ub97c \uc694\ud55c\uacfc \ucc28\ub974 \ub7ec\uc2dc\uc544\uc758 \uc54c\ub809\uc0b0\ub4dc\ub974 1\uc138\ub294 \ub098\ud3f4\ub808\uc639\uc744 \uc0c1\ub300\ub85c \ud568\uaed8 \uad70\ub300\ub97c \uc77c\uc73c\ucf30\uace0, \ud504\ub85c\uc774\uc13c\uacfc \uc624\uc2a4\ud2b8\ub9ac\uc544\ub97c \uc774\ub04c\uc5c8\ub2e4. \ubaa8\ub85c\ub294 \ub098\ud3f4\ub808\uc639\uc758 \ud328\ubc30\uc640 \uacf5\ud654\uad6d \uc815\ubd80\uac00 \uc138\uc6cc\uc9c0\ub294 \uac83\uc744 \ubcf4\uae30\ub97c \ud76c\ub9dd\ud588\ub2e4. \ubaa8\ub85c\ub294 \uc2a4\uc6e8\ub374\uacfc \ub7ec\uc2dc\uc544\uc758 \uc9c0\ub3c4\uc790\ub4e4\uc5d0\uac8c \ud504\ub791\uc2a4\ub97c \ud328\ubc30\uc2dc\ud0a4\ub294 \uac00\uc7a5 \ucd5c\uc120\uc758 \ubc29\ubc95 \ub4f1\uc744 \uc870\uc5b8\ud558\ub294 \uc5ed\ud560\uc744 \ub9e1\uc558\ub2e4. \ubaa8\ub85c\ub294 1813\ub144 8\uc6d4 27\uc77c \ub4dc\ub808\uc2a4\ub374 \uc804\ud22c\uc5d0\uc11c \uc911\uc0c1\uc744 \uc785\uace0 \ucc28\ub974\uc640 \ub300\ud654\ub97c \ub098\ub208 \ub4a4 \ub77c\uc6b0\ub2c8(Louny)\uc5d0\uc11c 9\uc6d4 2\uc77c \uc0ac\ub9dd\ud588\ub2e4. \ubaa8\ub85c\ub294 \uc0c1\ud2b8\ud398\ud14c\ub974\ubd80\ub974\ud06c\uc758 \uc131 \uce74\ud0c0\ub9ac\ub098 \uac00\ud1a8\ub9ad \uad50\ud68c(Catholic Church of St. Catherine)\uc5d0 \ub9e4\uc7a5\ub418\uc5c8\ub2e4. \ubaa8\ub85c\uc758 \uc544\ub0b4\ub294 \ucc28\ub974\ub85c\ubd80\ud130 \uc5f0\uae08\uc744 \ubc1b\uc558\uace0, \ub8e8\uc774 18\uc138\ub85c\ubd80\ud130 \uc6d0\uc218(mar\u00e9chal)\uc758 \uc9c0\uc704\ub97c \ud558\uc0ac\ubc1b\uc558\uc73c\ub098 \ubaa8\ub85c\uc758 \ub3d9\ud5a5\uc778\ub4e4\uc740 \ubaa8\ub85c\uc5d0 \ub300\ud574 \uc774\uc57c\uae30\ud560 \ub54c \" \ubcc0\uc808\uc790 \" \ub77c\uba70 \ub4a4\ubb34\ub9ac\uc5d0\uc640 \ud53c\uc288\uadf8\ub93c\uc5d0 \ube44\uc720\ud588\ub2e4.", "sentence_answer": "\ubaa8\ub85c\uc758 \uc544\ub0b4\ub294 \ucc28\ub974\ub85c\ubd80\ud130 \uc5f0\uae08\uc744 \ubc1b\uc558\uace0, \ub8e8\uc774 18\uc138\ub85c\ubd80\ud130 \uc6d0\uc218(mar\u00e9chal)\uc758 \uc9c0\uc704\ub97c \ud558\uc0ac\ubc1b\uc558\uc73c\ub098 \ubaa8\ub85c\uc758 \ub3d9\ud5a5\uc778\ub4e4\uc740 \ubaa8\ub85c\uc5d0 \ub300\ud574 \uc774\uc57c\uae30\ud560 \ub54c \" \ubcc0\uc808\uc790 \"", "paragraph_id": "6548933-4-2", "paragraph_question": "question: \ubaa8\ub85c\uc758 \ub3d9\ud5a5\uc778\ub4e4\uc774 \ubaa8\ub85c\ub97c \ub4a4\ubb34\ub9ac\uc5d0\uc640 \ud53c\uc288\uadf8\ub93c\uc640 \ube44\uc720\ud558\uba70 \uc9c0\uce6d\ud558\ub294 \ub9d0\uc740?, context: \ubaa8\ub85c\ub294 \uc544\ub0b4\uc758 \uc120\ub3d9\uc73c\ub85c \uc720\ub7fd\uc5d0 \ub3cc\uc544\uc634\uacfc \ub3d9\uc2dc\uc5d0 \uacf5\ud654\uc8fc\uc758\uc790 \uc74c\ubaa8\uc790\ub4e4\uc758 \ubaa8\uc784\uc758 \uc61b \uce5c\uad6c\ub4e4\uacfc \ud611\uc0c1\ud558\uae30 \uc2dc\uc791\ud588\ub2e4:\uc774\uc804 \uc7a5-\ubc25\ud2f0\uc2a4\ud2b8 \ubca0\ub974\ub098\ub3c4\ud2b8, \ud604\uc7ac \uc2a4\uc6e8\ub374\uc758 \uce74\ub97c \uc694\ud55c \uacf5(\ud6d7\ub0a0 \uad6d\uc655 \uc2a4\uc6e8\ub374\uc758 \uce74\ub97c 14\uc138 \uc694\ud55c). \uce74\ub97c \uc694\ud55c\uacfc \ucc28\ub974 \ub7ec\uc2dc\uc544\uc758 \uc54c\ub809\uc0b0\ub4dc\ub974 1\uc138\ub294 \ub098\ud3f4\ub808\uc639\uc744 \uc0c1\ub300\ub85c \ud568\uaed8 \uad70\ub300\ub97c \uc77c\uc73c\ucf30\uace0, \ud504\ub85c\uc774\uc13c\uacfc \uc624\uc2a4\ud2b8\ub9ac\uc544\ub97c \uc774\ub04c\uc5c8\ub2e4. \ubaa8\ub85c\ub294 \ub098\ud3f4\ub808\uc639\uc758 \ud328\ubc30\uc640 \uacf5\ud654\uad6d \uc815\ubd80\uac00 \uc138\uc6cc\uc9c0\ub294 \uac83\uc744 \ubcf4\uae30\ub97c \ud76c\ub9dd\ud588\ub2e4. \ubaa8\ub85c\ub294 \uc2a4\uc6e8\ub374\uacfc \ub7ec\uc2dc\uc544\uc758 \uc9c0\ub3c4\uc790\ub4e4\uc5d0\uac8c \ud504\ub791\uc2a4\ub97c \ud328\ubc30\uc2dc\ud0a4\ub294 \uac00\uc7a5 \ucd5c\uc120\uc758 \ubc29\ubc95 \ub4f1\uc744 \uc870\uc5b8\ud558\ub294 \uc5ed\ud560\uc744 \ub9e1\uc558\ub2e4. \ubaa8\ub85c\ub294 1813\ub144 8\uc6d4 27\uc77c \ub4dc\ub808\uc2a4\ub374 \uc804\ud22c\uc5d0\uc11c \uc911\uc0c1\uc744 \uc785\uace0 \ucc28\ub974\uc640 \ub300\ud654\ub97c \ub098\ub208 \ub4a4 \ub77c\uc6b0\ub2c8(Louny)\uc5d0\uc11c 9\uc6d4 2\uc77c \uc0ac\ub9dd\ud588\ub2e4. \ubaa8\ub85c\ub294 \uc0c1\ud2b8\ud398\ud14c\ub974\ubd80\ub974\ud06c\uc758 \uc131 \uce74\ud0c0\ub9ac\ub098 \uac00\ud1a8\ub9ad \uad50\ud68c(Catholic Church of St. Catherine)\uc5d0 \ub9e4\uc7a5\ub418\uc5c8\ub2e4. \ubaa8\ub85c\uc758 \uc544\ub0b4\ub294 \ucc28\ub974\ub85c\ubd80\ud130 \uc5f0\uae08\uc744 \ubc1b\uc558\uace0, \ub8e8\uc774 18\uc138\ub85c\ubd80\ud130 \uc6d0\uc218(mar\u00e9chal)\uc758 \uc9c0\uc704\ub97c \ud558\uc0ac\ubc1b\uc558\uc73c\ub098 \ubaa8\ub85c\uc758 \ub3d9\ud5a5\uc778\ub4e4\uc740 \ubaa8\ub85c\uc5d0 \ub300\ud574 \uc774\uc57c\uae30\ud560 \ub54c \"\ubcc0\uc808\uc790\" \ub77c\uba70 \ub4a4\ubb34\ub9ac\uc5d0\uc640 \ud53c\uc288\uadf8\ub93c\uc5d0 \ube44\uc720\ud588\ub2e4."} {"question": "\uba54\uac74 \ud3ed\uc2a4\uac00 \uc720\uba85\ud574\uc9c4 \uc774\ud6c4 \ud560\ub9ac\uc6b0\ub4dc\uacc4 \ub300\ud45c \uc139\uc2dc \ubc30\uc6b0 \ub204\uad6c\uc640 \ube44\uad50\ub2f9\ud558\uc600\ub294\uac00?", "paragraph": "\uba54\uac74 \ud3ed\uc2a4\ub294 \uc720\uba85\ud574\uc9d0\uacfc \ub3d9\uc2dc\uc5d0 \uc885\uc885 \ud560\ub9ac\uc6b0\ub4dc\uacc4\uc758 \ub300\ud45c \uc139\uc2dc \ubc30\uc6b0 \uc548\uc824\ub9ac\ub098 \uc878\ub9ac\uc640 \ube44\uad50\ub97c \ub2f9\ud558\uba70 '\ud3ec\uc2a4\ud2b8 \uc548\uc824\ub9ac\ub098 \uc878\ub9ac'\ub77c\uace0\ub3c4 \ubd88\ub9b4 \uc815\ub3c4\uc600\ub2e4. \ub300\uc911 \ub9e4\uccb4\ub4e4 \ub610\ud55c \uba54\uac74 \ud3ed\uc2a4\ub97c \uc548\uc824\ub9ac\ub098 \uc878\ub9ac\uc640 \uc5f0\uad00\uc131\uc744 \uc9c0\uc5c8\ub2e4. \ud55c\ud3b8, \uc548\uc824\ub9ac\ub098 \uc878\ub9ac\ub294 \ub098\uc774\uac00 \ub4ec\uacfc \ub3d9\uc2dc\uc5d0 \uc139\uc2dc \uc774\ubbf8\uc9c0\ub97c \uc2ec\uc5b4\uc900 \uc601\ud654 \u300a\ud23c \ub808\uc774\ub354\u300b\uc5d0\uc11c \ud558\ucc28 \ud558\uc600\ub2e4. \uc774\ub85c\uc368 \uc601\ud654 \uad00\uacc4\uc790\ub4e4\uc740 \uc774 \ud6c4\uc758 \ubc30\uc6b0\ub85c \uba54\uac74 \ud3ed\uc2a4\ub97c \uc810\ucc0d\uc5c8\uace0 \uba54\uac74 \ud3ed\uc2a4\ub610\ud55c '\ub77c\ub77c \ud06c\ub85c\ud504\ud2b8'\uc5ed\uc744 \ub9e4\uc6b0 \uc695\uc2ec\ub0c8\uc9c0\ub9cc, \uc548\uc824\ub9ac\ub098 \uc878\ub9ac\uc640 \ube44\uad50\ub2f9\ud558\ub294\uac83\uc744 \ub450\ub824\uc6cc\ud588\ub2e4\uba70 \uce21\uadfc\ub4e4\uc740 \ubc1c\uc5b8\ud588\uace0 \ud3ed\uc2a4\ub294 \uacb0\uad6d \uce90\uc2a4\ud305\uc744 \uac70\uc808\ud558\uc600\ub2e4. \ub610\ud55c \uc878\ub9ac\uc758 \uc544\ubc84\uc9c0\uc778 \uc874 \ubcf4\uc774\ud2b8\ub294 \uc678\uc2e0\uacfc\uc758 \uc778\ud130\ubdf0\uc911 \u201c\uba54\uac74\ud3ed\uc2a4\uc640 \ub538\uc774 \ube44\uad50 \ub420 \ub9cc\ud558\ub2e4\uace0 \uc0dd\uac01\ud558\ub0d0?\u201d\ub294 \uc9c8\ubb38\uc5d0 \"\uadf8\ub807\uc9c0 \uc54a\ub2e4\"\ub77c\uace0 \ub2f5\ud558\uba70 \ubd80\uc815\uc801\uc778 \ubc18\uc751\uc744 \ubcf4\uc774\uae30\ub3c4 \ud558\uc600\uc73c\uba70, \uc774\ubbf8 \uc591\uc131\uc560\uc790\uc784\uc744 \ubc1d\ud78c \ud3ed\uc2a4\ub294 \uc5b8\ub860\uacfc\uc758 \uc778\ud130\ubdf0\uc911 \"\uc878\ub9ac\uc758 \uc5ec\uc790\uce5c\uad6c\uac00 \ub418\uace0 \uc2f6\ub2e4\"\ub77c\uace0 \ub9d0\ud558\uba70 \ud070 \ud654\uc81c\ub97c \ubaa8\uc558\ub2e4.", "answer": "\uc548\uc824\ub9ac\ub098 \uc878\ub9ac", "sentence": "\uba54\uac74 \ud3ed\uc2a4\ub294 \uc720\uba85\ud574\uc9d0\uacfc \ub3d9\uc2dc\uc5d0 \uc885\uc885 \ud560\ub9ac\uc6b0\ub4dc\uacc4\uc758 \ub300\ud45c \uc139\uc2dc \ubc30\uc6b0 \uc548\uc824\ub9ac\ub098 \uc878\ub9ac \uc640 \ube44\uad50\ub97c \ub2f9\ud558\uba70 '\ud3ec\uc2a4\ud2b8 \uc548\uc824\ub9ac\ub098 \uc878\ub9ac'\ub77c\uace0\ub3c4 \ubd88\ub9b4 \uc815\ub3c4\uc600\ub2e4.", "paragraph_sentence": " \uba54\uac74 \ud3ed\uc2a4\ub294 \uc720\uba85\ud574\uc9d0\uacfc \ub3d9\uc2dc\uc5d0 \uc885\uc885 \ud560\ub9ac\uc6b0\ub4dc\uacc4\uc758 \ub300\ud45c \uc139\uc2dc \ubc30\uc6b0 \uc548\uc824\ub9ac\ub098 \uc878\ub9ac \uc640 \ube44\uad50\ub97c \ub2f9\ud558\uba70 '\ud3ec\uc2a4\ud2b8 \uc548\uc824\ub9ac\ub098 \uc878\ub9ac'\ub77c\uace0\ub3c4 \ubd88\ub9b4 \uc815\ub3c4\uc600\ub2e4. \ub300\uc911 \ub9e4\uccb4\ub4e4 \ub610\ud55c \uba54\uac74 \ud3ed\uc2a4\ub97c \uc548\uc824\ub9ac\ub098 \uc878\ub9ac\uc640 \uc5f0\uad00\uc131\uc744 \uc9c0\uc5c8\ub2e4. \ud55c\ud3b8, \uc548\uc824\ub9ac\ub098 \uc878\ub9ac\ub294 \ub098\uc774\uac00 \ub4ec\uacfc \ub3d9\uc2dc\uc5d0 \uc139\uc2dc \uc774\ubbf8\uc9c0\ub97c \uc2ec\uc5b4\uc900 \uc601\ud654 \u300a\ud23c \ub808\uc774\ub354\u300b\uc5d0\uc11c \ud558\ucc28 \ud558\uc600\ub2e4. \uc774\ub85c\uc368 \uc601\ud654 \uad00\uacc4\uc790\ub4e4\uc740 \uc774 \ud6c4\uc758 \ubc30\uc6b0\ub85c \uba54\uac74 \ud3ed\uc2a4\ub97c \uc810\ucc0d\uc5c8\uace0 \uba54\uac74 \ud3ed\uc2a4\ub610\ud55c '\ub77c\ub77c \ud06c\ub85c\ud504\ud2b8'\uc5ed\uc744 \ub9e4\uc6b0 \uc695\uc2ec\ub0c8\uc9c0\ub9cc, \uc548\uc824\ub9ac\ub098 \uc878\ub9ac\uc640 \ube44\uad50\ub2f9\ud558\ub294\uac83\uc744 \ub450\ub824\uc6cc\ud588\ub2e4\uba70 \uce21\uadfc\ub4e4\uc740 \ubc1c\uc5b8\ud588\uace0 \ud3ed\uc2a4\ub294 \uacb0\uad6d \uce90\uc2a4\ud305\uc744 \uac70\uc808\ud558\uc600\ub2e4. \ub610\ud55c \uc878\ub9ac\uc758 \uc544\ubc84\uc9c0\uc778 \uc874 \ubcf4\uc774\ud2b8\ub294 \uc678\uc2e0\uacfc\uc758 \uc778\ud130\ubdf0\uc911 \u201c\uba54\uac74\ud3ed\uc2a4\uc640 \ub538\uc774 \ube44\uad50 \ub420 \ub9cc\ud558\ub2e4\uace0 \uc0dd\uac01\ud558\ub0d0?\u201d\ub294 \uc9c8\ubb38\uc5d0 \"\uadf8\ub807\uc9c0 \uc54a\ub2e4\"\ub77c\uace0 \ub2f5\ud558\uba70 \ubd80\uc815\uc801\uc778 \ubc18\uc751\uc744 \ubcf4\uc774\uae30\ub3c4 \ud558\uc600\uc73c\uba70, \uc774\ubbf8 \uc591\uc131\uc560\uc790\uc784\uc744 \ubc1d\ud78c \ud3ed\uc2a4\ub294 \uc5b8\ub860\uacfc\uc758 \uc778\ud130\ubdf0\uc911 \"\uc878\ub9ac\uc758 \uc5ec\uc790\uce5c\uad6c\uac00 \ub418\uace0 \uc2f6\ub2e4\"\ub77c\uace0 \ub9d0\ud558\uba70 \ud070 \ud654\uc81c\ub97c \ubaa8\uc558\ub2e4.", "paragraph_answer": "\uba54\uac74 \ud3ed\uc2a4\ub294 \uc720\uba85\ud574\uc9d0\uacfc \ub3d9\uc2dc\uc5d0 \uc885\uc885 \ud560\ub9ac\uc6b0\ub4dc\uacc4\uc758 \ub300\ud45c \uc139\uc2dc \ubc30\uc6b0 \uc548\uc824\ub9ac\ub098 \uc878\ub9ac \uc640 \ube44\uad50\ub97c \ub2f9\ud558\uba70 '\ud3ec\uc2a4\ud2b8 \uc548\uc824\ub9ac\ub098 \uc878\ub9ac'\ub77c\uace0\ub3c4 \ubd88\ub9b4 \uc815\ub3c4\uc600\ub2e4. \ub300\uc911 \ub9e4\uccb4\ub4e4 \ub610\ud55c \uba54\uac74 \ud3ed\uc2a4\ub97c \uc548\uc824\ub9ac\ub098 \uc878\ub9ac\uc640 \uc5f0\uad00\uc131\uc744 \uc9c0\uc5c8\ub2e4. \ud55c\ud3b8, \uc548\uc824\ub9ac\ub098 \uc878\ub9ac\ub294 \ub098\uc774\uac00 \ub4ec\uacfc \ub3d9\uc2dc\uc5d0 \uc139\uc2dc \uc774\ubbf8\uc9c0\ub97c \uc2ec\uc5b4\uc900 \uc601\ud654 \u300a\ud23c \ub808\uc774\ub354\u300b\uc5d0\uc11c \ud558\ucc28 \ud558\uc600\ub2e4. \uc774\ub85c\uc368 \uc601\ud654 \uad00\uacc4\uc790\ub4e4\uc740 \uc774 \ud6c4\uc758 \ubc30\uc6b0\ub85c \uba54\uac74 \ud3ed\uc2a4\ub97c \uc810\ucc0d\uc5c8\uace0 \uba54\uac74 \ud3ed\uc2a4\ub610\ud55c '\ub77c\ub77c \ud06c\ub85c\ud504\ud2b8'\uc5ed\uc744 \ub9e4\uc6b0 \uc695\uc2ec\ub0c8\uc9c0\ub9cc, \uc548\uc824\ub9ac\ub098 \uc878\ub9ac\uc640 \ube44\uad50\ub2f9\ud558\ub294\uac83\uc744 \ub450\ub824\uc6cc\ud588\ub2e4\uba70 \uce21\uadfc\ub4e4\uc740 \ubc1c\uc5b8\ud588\uace0 \ud3ed\uc2a4\ub294 \uacb0\uad6d \uce90\uc2a4\ud305\uc744 \uac70\uc808\ud558\uc600\ub2e4. \ub610\ud55c \uc878\ub9ac\uc758 \uc544\ubc84\uc9c0\uc778 \uc874 \ubcf4\uc774\ud2b8\ub294 \uc678\uc2e0\uacfc\uc758 \uc778\ud130\ubdf0\uc911 \u201c\uba54\uac74\ud3ed\uc2a4\uc640 \ub538\uc774 \ube44\uad50 \ub420 \ub9cc\ud558\ub2e4\uace0 \uc0dd\uac01\ud558\ub0d0?\u201d\ub294 \uc9c8\ubb38\uc5d0 \"\uadf8\ub807\uc9c0 \uc54a\ub2e4\"\ub77c\uace0 \ub2f5\ud558\uba70 \ubd80\uc815\uc801\uc778 \ubc18\uc751\uc744 \ubcf4\uc774\uae30\ub3c4 \ud558\uc600\uc73c\uba70, \uc774\ubbf8 \uc591\uc131\uc560\uc790\uc784\uc744 \ubc1d\ud78c \ud3ed\uc2a4\ub294 \uc5b8\ub860\uacfc\uc758 \uc778\ud130\ubdf0\uc911 \"\uc878\ub9ac\uc758 \uc5ec\uc790\uce5c\uad6c\uac00 \ub418\uace0 \uc2f6\ub2e4\"\ub77c\uace0 \ub9d0\ud558\uba70 \ud070 \ud654\uc81c\ub97c \ubaa8\uc558\ub2e4.", "sentence_answer": "\uba54\uac74 \ud3ed\uc2a4\ub294 \uc720\uba85\ud574\uc9d0\uacfc \ub3d9\uc2dc\uc5d0 \uc885\uc885 \ud560\ub9ac\uc6b0\ub4dc\uacc4\uc758 \ub300\ud45c \uc139\uc2dc \ubc30\uc6b0 \uc548\uc824\ub9ac\ub098 \uc878\ub9ac \uc640 \ube44\uad50\ub97c \ub2f9\ud558\uba70 '\ud3ec\uc2a4\ud2b8 \uc548\uc824\ub9ac\ub098 \uc878\ub9ac'\ub77c\uace0\ub3c4 \ubd88\ub9b4 \uc815\ub3c4\uc600\ub2e4.", "paragraph_id": "6502694-5-0", "paragraph_question": "question: \uba54\uac74 \ud3ed\uc2a4\uac00 \uc720\uba85\ud574\uc9c4 \uc774\ud6c4 \ud560\ub9ac\uc6b0\ub4dc\uacc4 \ub300\ud45c \uc139\uc2dc \ubc30\uc6b0 \ub204\uad6c\uc640 \ube44\uad50\ub2f9\ud558\uc600\ub294\uac00?, context: \uba54\uac74 \ud3ed\uc2a4\ub294 \uc720\uba85\ud574\uc9d0\uacfc \ub3d9\uc2dc\uc5d0 \uc885\uc885 \ud560\ub9ac\uc6b0\ub4dc\uacc4\uc758 \ub300\ud45c \uc139\uc2dc \ubc30\uc6b0 \uc548\uc824\ub9ac\ub098 \uc878\ub9ac\uc640 \ube44\uad50\ub97c \ub2f9\ud558\uba70 '\ud3ec\uc2a4\ud2b8 \uc548\uc824\ub9ac\ub098 \uc878\ub9ac'\ub77c\uace0\ub3c4 \ubd88\ub9b4 \uc815\ub3c4\uc600\ub2e4. \ub300\uc911 \ub9e4\uccb4\ub4e4 \ub610\ud55c \uba54\uac74 \ud3ed\uc2a4\ub97c \uc548\uc824\ub9ac\ub098 \uc878\ub9ac\uc640 \uc5f0\uad00\uc131\uc744 \uc9c0\uc5c8\ub2e4. \ud55c\ud3b8, \uc548\uc824\ub9ac\ub098 \uc878\ub9ac\ub294 \ub098\uc774\uac00 \ub4ec\uacfc \ub3d9\uc2dc\uc5d0 \uc139\uc2dc \uc774\ubbf8\uc9c0\ub97c \uc2ec\uc5b4\uc900 \uc601\ud654 \u300a\ud23c \ub808\uc774\ub354\u300b\uc5d0\uc11c \ud558\ucc28 \ud558\uc600\ub2e4. \uc774\ub85c\uc368 \uc601\ud654 \uad00\uacc4\uc790\ub4e4\uc740 \uc774 \ud6c4\uc758 \ubc30\uc6b0\ub85c \uba54\uac74 \ud3ed\uc2a4\ub97c \uc810\ucc0d\uc5c8\uace0 \uba54\uac74 \ud3ed\uc2a4\ub610\ud55c '\ub77c\ub77c \ud06c\ub85c\ud504\ud2b8'\uc5ed\uc744 \ub9e4\uc6b0 \uc695\uc2ec\ub0c8\uc9c0\ub9cc, \uc548\uc824\ub9ac\ub098 \uc878\ub9ac\uc640 \ube44\uad50\ub2f9\ud558\ub294\uac83\uc744 \ub450\ub824\uc6cc\ud588\ub2e4\uba70 \uce21\uadfc\ub4e4\uc740 \ubc1c\uc5b8\ud588\uace0 \ud3ed\uc2a4\ub294 \uacb0\uad6d \uce90\uc2a4\ud305\uc744 \uac70\uc808\ud558\uc600\ub2e4. \ub610\ud55c \uc878\ub9ac\uc758 \uc544\ubc84\uc9c0\uc778 \uc874 \ubcf4\uc774\ud2b8\ub294 \uc678\uc2e0\uacfc\uc758 \uc778\ud130\ubdf0\uc911 \u201c\uba54\uac74\ud3ed\uc2a4\uc640 \ub538\uc774 \ube44\uad50 \ub420 \ub9cc\ud558\ub2e4\uace0 \uc0dd\uac01\ud558\ub0d0?\u201d\ub294 \uc9c8\ubb38\uc5d0 \"\uadf8\ub807\uc9c0 \uc54a\ub2e4\"\ub77c\uace0 \ub2f5\ud558\uba70 \ubd80\uc815\uc801\uc778 \ubc18\uc751\uc744 \ubcf4\uc774\uae30\ub3c4 \ud558\uc600\uc73c\uba70, \uc774\ubbf8 \uc591\uc131\uc560\uc790\uc784\uc744 \ubc1d\ud78c \ud3ed\uc2a4\ub294 \uc5b8\ub860\uacfc\uc758 \uc778\ud130\ubdf0\uc911 \"\uc878\ub9ac\uc758 \uc5ec\uc790\uce5c\uad6c\uac00 \ub418\uace0 \uc2f6\ub2e4\"\ub77c\uace0 \ub9d0\ud558\uba70 \ud070 \ud654\uc81c\ub97c \ubaa8\uc558\ub2e4."} {"question": "1941\ub144 \ub3c5\uc77c\uc758 60\uac1c \ub3c4\uc2dc\uc5d0\uc11c \ud761\uc5f0\uc744 \uae08\uc9c0\ud558\uc600\ub358 \uc2e4\ub0b4 \uc7a5\uc18c\ub294 \uc5b4\ub514\uc600\ub098?", "paragraph": "1941\ub144\uc5d0\ub294 \ub3c5\uc77c\uc758 60\uac1c \ub3c4\uc2dc\uac00 \ub178\uba74 \uc804\ucc28 \uc548\uc5d0\uc11c \ud761\uc5f0\ud558\ub294 \uac83\uc744 \uae08\uc9c0\ud588\ub2e4. \ubc29\uacf5\ud638\uc5d0\uc11c\ub3c4 \ud761\uc5f0\uc774 \uae08\uc9c0\ub418\uc5c8\uc9c0\ub9cc \uc77c\ubd80 \ubc29\uacf5\ud638\uc5d0\ub294 \ubcc4\ub3c4\uc758 \ud761\uc5f0 \uad6c\uc5ed\uc774 \ub9c8\ub828\ub418\uc5b4 \uc788\uc5c8\ub2e4. \ud55c\ud3b8 \uc5ec\uc131\uc758 \ud761\uc5f0\uc744 \ub9c9\uae30 \uc704\ud574 \ud2b9\ubcc4\ud55c \uc8fc\uc758\ub97c \ub0b4\ub9ac\uae30\ub3c4 \ud588\ub294\ub370 \ub3c5\uc77c \uc758\ub8cc \ud611\ud68c \ud68c\uc7a5\uc740 \"\ub3c5\uc77c \uc5ec\uc131\uc740 \ub2f4\ubc30\ub97c \ud53c\uc6b0\uc9c0 \uc54a\ub294\ub2e4\"\uace0 \uc120\uc5b8\ud588\uc73c\uba70 \uc81c2\ucc28 \uc138\uacc4 \ub300\uc804 \uc911\uc5d0\ub294 \uc784\uc0b0\ubd80\uc640 25\uc138 \uc774\ud558\uc758 \uc5ec\uc131, 55\uc138 \uc774\uc0c1\uc758 \uc5ec\uc131\uc5d0\uac8c \ub2f4\ubc30 \ubc30\uae09 \uce74\ub4dc\ub97c \uc9c0\uae09\ud558\uc9c0 \uc54a\uc558\uc73c\uba70 \uc11c\ube44\uc2a4\uc5c5\uc790\uc640 \uc2dd\ud488 \uc18c\ub9e4\uc5c5\uc790\uc5d0\uac8c\ub294 \uc5ec\uc131\uc758 \ub2f4\ubc30 \uad6c\ub9e4\ub97c \uc81c\ud55c\ud558\ub294 \uc870\uce58\ub97c \ub0b4\ub838\ub2e4. \uadf9\uc7a5\uc5d0\uc11c\ub294 \uc5ec\uc131\uc744 \ud0c0\uac9f\uc73c\ub85c \ud55c \ub2f4\ubc30 \ubc18\ub300 \uc601\ud654\uac00 \uacf5\uac1c\uc801\uc73c\ub85c \uc0c1\uc601\ub418\uc5c8\uc73c\uba70 \uc2e0\ubb38\uc5d0\ub294 \ud761\uc5f0\uacfc \uc774\uc5d0 \ub300\ud55c \uc601\ud5a5\uc744 \ub2e4\ub8ec \uc0ac\uc124\uc774 \uac8c\uc7ac\ub418\uc5c8\ub2e4. \uad6d\uac00\uc0ac\ud68c\uc8fc\uc758 \ub178\ub3d9 \uc870\ud569\uc758 \uac01 \uc9c0\ubd80\ub294 \ub2f4\ubc30\ub97c \ud53c\uc6b0\ub294 \uc5ec\uc131 \ud68c\uc6d0\uc744 \uacf5\uac1c\uc801\uc73c\ub85c \ucd94\ubc29\uc2dc\ud0a4\ub294 \uc5c4\uaca9\ud55c \uc870\uce58\ub97c \ub0b4\ub838\ub2e4. 1943\ub144 7\uc6d4\uc5d0\ub294 18\uc138 \ubbf8\ub9cc\uc758 \uad6d\ubbfc\uc774 \uacf5\uacf5 \uc7a5\uc18c\uc5d0\uc11c \ud761\uc5f0\ud558\ub294 \uac83\uc774 \uae08\uc9c0\ub418\uc5c8\uace0 1944\ub144\uc5d0\ub294 \ubc84\uc2a4\uc640 \uc5f4\ucc28 \uc548\uc5d0\uc11c \ub2f4\ubc30\ub97c \ud53c\uc6b0\ub294 \uac83\uc870\ucc28 \uae08\uc9c0\ub418\uc5c8\ub2e4. \ud788\ud2c0\ub7ec\ub294 \uc5ec\uc131\uc774 \uac04\uc811 \ud761\uc5f0\uc758 \ud76c\uc0dd\uc790\uac00 \ub420 \uac83\uc744 \uc6b0\ub824\ud588\ub2e4.", "answer": "\ub178\uba74 \uc804\ucc28", "sentence": "1941\ub144\uc5d0\ub294 \ub3c5\uc77c\uc758 60\uac1c \ub3c4\uc2dc\uac00 \ub178\uba74 \uc804\ucc28 \uc548\uc5d0\uc11c \ud761\uc5f0\ud558\ub294 \uac83\uc744 \uae08\uc9c0\ud588\ub2e4.", "paragraph_sentence": " 1941\ub144\uc5d0\ub294 \ub3c5\uc77c\uc758 60\uac1c \ub3c4\uc2dc\uac00 \ub178\uba74 \uc804\ucc28 \uc548\uc5d0\uc11c \ud761\uc5f0\ud558\ub294 \uac83\uc744 \uae08\uc9c0\ud588\ub2e4. \ubc29\uacf5\ud638\uc5d0\uc11c\ub3c4 \ud761\uc5f0\uc774 \uae08\uc9c0\ub418\uc5c8\uc9c0\ub9cc \uc77c\ubd80 \ubc29\uacf5\ud638\uc5d0\ub294 \ubcc4\ub3c4\uc758 \ud761\uc5f0 \uad6c\uc5ed\uc774 \ub9c8\ub828\ub418\uc5b4 \uc788\uc5c8\ub2e4. \ud55c\ud3b8 \uc5ec\uc131\uc758 \ud761\uc5f0\uc744 \ub9c9\uae30 \uc704\ud574 \ud2b9\ubcc4\ud55c \uc8fc\uc758\ub97c \ub0b4\ub9ac\uae30\ub3c4 \ud588\ub294\ub370 \ub3c5\uc77c \uc758\ub8cc \ud611\ud68c \ud68c\uc7a5\uc740 \"\ub3c5\uc77c \uc5ec\uc131\uc740 \ub2f4\ubc30\ub97c \ud53c\uc6b0\uc9c0 \uc54a\ub294\ub2e4\"\uace0 \uc120\uc5b8\ud588\uc73c\uba70 \uc81c2\ucc28 \uc138\uacc4 \ub300\uc804 \uc911\uc5d0\ub294 \uc784\uc0b0\ubd80\uc640 25\uc138 \uc774\ud558\uc758 \uc5ec\uc131, 55\uc138 \uc774\uc0c1\uc758 \uc5ec\uc131\uc5d0\uac8c \ub2f4\ubc30 \ubc30\uae09 \uce74\ub4dc\ub97c \uc9c0\uae09\ud558\uc9c0 \uc54a\uc558\uc73c\uba70 \uc11c\ube44\uc2a4\uc5c5\uc790\uc640 \uc2dd\ud488 \uc18c\ub9e4\uc5c5\uc790\uc5d0\uac8c\ub294 \uc5ec\uc131\uc758 \ub2f4\ubc30 \uad6c\ub9e4\ub97c \uc81c\ud55c\ud558\ub294 \uc870\uce58\ub97c \ub0b4\ub838\ub2e4. \uadf9\uc7a5\uc5d0\uc11c\ub294 \uc5ec\uc131\uc744 \ud0c0\uac9f\uc73c\ub85c \ud55c \ub2f4\ubc30 \ubc18\ub300 \uc601\ud654\uac00 \uacf5\uac1c\uc801\uc73c\ub85c \uc0c1\uc601\ub418\uc5c8\uc73c\uba70 \uc2e0\ubb38\uc5d0\ub294 \ud761\uc5f0\uacfc \uc774\uc5d0 \ub300\ud55c \uc601\ud5a5\uc744 \ub2e4\ub8ec \uc0ac\uc124\uc774 \uac8c\uc7ac\ub418\uc5c8\ub2e4. \uad6d\uac00\uc0ac\ud68c\uc8fc\uc758 \ub178\ub3d9 \uc870\ud569\uc758 \uac01 \uc9c0\ubd80\ub294 \ub2f4\ubc30\ub97c \ud53c\uc6b0\ub294 \uc5ec\uc131 \ud68c\uc6d0\uc744 \uacf5\uac1c\uc801\uc73c\ub85c \ucd94\ubc29\uc2dc\ud0a4\ub294 \uc5c4\uaca9\ud55c \uc870\uce58\ub97c \ub0b4\ub838\ub2e4. 1943\ub144 7\uc6d4\uc5d0\ub294 18\uc138 \ubbf8\ub9cc\uc758 \uad6d\ubbfc\uc774 \uacf5\uacf5 \uc7a5\uc18c\uc5d0\uc11c \ud761\uc5f0\ud558\ub294 \uac83\uc774 \uae08\uc9c0\ub418\uc5c8\uace0 1944\ub144\uc5d0\ub294 \ubc84\uc2a4\uc640 \uc5f4\ucc28 \uc548\uc5d0\uc11c \ub2f4\ubc30\ub97c \ud53c\uc6b0\ub294 \uac83\uc870\ucc28 \uae08\uc9c0\ub418\uc5c8\ub2e4. \ud788\ud2c0\ub7ec\ub294 \uc5ec\uc131\uc774 \uac04\uc811 \ud761\uc5f0\uc758 \ud76c\uc0dd\uc790\uac00 \ub420 \uac83\uc744 \uc6b0\ub824\ud588\ub2e4.", "paragraph_answer": "1941\ub144\uc5d0\ub294 \ub3c5\uc77c\uc758 60\uac1c \ub3c4\uc2dc\uac00 \ub178\uba74 \uc804\ucc28 \uc548\uc5d0\uc11c \ud761\uc5f0\ud558\ub294 \uac83\uc744 \uae08\uc9c0\ud588\ub2e4. \ubc29\uacf5\ud638\uc5d0\uc11c\ub3c4 \ud761\uc5f0\uc774 \uae08\uc9c0\ub418\uc5c8\uc9c0\ub9cc \uc77c\ubd80 \ubc29\uacf5\ud638\uc5d0\ub294 \ubcc4\ub3c4\uc758 \ud761\uc5f0 \uad6c\uc5ed\uc774 \ub9c8\ub828\ub418\uc5b4 \uc788\uc5c8\ub2e4. \ud55c\ud3b8 \uc5ec\uc131\uc758 \ud761\uc5f0\uc744 \ub9c9\uae30 \uc704\ud574 \ud2b9\ubcc4\ud55c \uc8fc\uc758\ub97c \ub0b4\ub9ac\uae30\ub3c4 \ud588\ub294\ub370 \ub3c5\uc77c \uc758\ub8cc \ud611\ud68c \ud68c\uc7a5\uc740 \"\ub3c5\uc77c \uc5ec\uc131\uc740 \ub2f4\ubc30\ub97c \ud53c\uc6b0\uc9c0 \uc54a\ub294\ub2e4\"\uace0 \uc120\uc5b8\ud588\uc73c\uba70 \uc81c2\ucc28 \uc138\uacc4 \ub300\uc804 \uc911\uc5d0\ub294 \uc784\uc0b0\ubd80\uc640 25\uc138 \uc774\ud558\uc758 \uc5ec\uc131, 55\uc138 \uc774\uc0c1\uc758 \uc5ec\uc131\uc5d0\uac8c \ub2f4\ubc30 \ubc30\uae09 \uce74\ub4dc\ub97c \uc9c0\uae09\ud558\uc9c0 \uc54a\uc558\uc73c\uba70 \uc11c\ube44\uc2a4\uc5c5\uc790\uc640 \uc2dd\ud488 \uc18c\ub9e4\uc5c5\uc790\uc5d0\uac8c\ub294 \uc5ec\uc131\uc758 \ub2f4\ubc30 \uad6c\ub9e4\ub97c \uc81c\ud55c\ud558\ub294 \uc870\uce58\ub97c \ub0b4\ub838\ub2e4. \uadf9\uc7a5\uc5d0\uc11c\ub294 \uc5ec\uc131\uc744 \ud0c0\uac9f\uc73c\ub85c \ud55c \ub2f4\ubc30 \ubc18\ub300 \uc601\ud654\uac00 \uacf5\uac1c\uc801\uc73c\ub85c \uc0c1\uc601\ub418\uc5c8\uc73c\uba70 \uc2e0\ubb38\uc5d0\ub294 \ud761\uc5f0\uacfc \uc774\uc5d0 \ub300\ud55c \uc601\ud5a5\uc744 \ub2e4\ub8ec \uc0ac\uc124\uc774 \uac8c\uc7ac\ub418\uc5c8\ub2e4. \uad6d\uac00\uc0ac\ud68c\uc8fc\uc758 \ub178\ub3d9 \uc870\ud569\uc758 \uac01 \uc9c0\ubd80\ub294 \ub2f4\ubc30\ub97c \ud53c\uc6b0\ub294 \uc5ec\uc131 \ud68c\uc6d0\uc744 \uacf5\uac1c\uc801\uc73c\ub85c \ucd94\ubc29\uc2dc\ud0a4\ub294 \uc5c4\uaca9\ud55c \uc870\uce58\ub97c \ub0b4\ub838\ub2e4. 1943\ub144 7\uc6d4\uc5d0\ub294 18\uc138 \ubbf8\ub9cc\uc758 \uad6d\ubbfc\uc774 \uacf5\uacf5 \uc7a5\uc18c\uc5d0\uc11c \ud761\uc5f0\ud558\ub294 \uac83\uc774 \uae08\uc9c0\ub418\uc5c8\uace0 1944\ub144\uc5d0\ub294 \ubc84\uc2a4\uc640 \uc5f4\ucc28 \uc548\uc5d0\uc11c \ub2f4\ubc30\ub97c \ud53c\uc6b0\ub294 \uac83\uc870\ucc28 \uae08\uc9c0\ub418\uc5c8\ub2e4. \ud788\ud2c0\ub7ec\ub294 \uc5ec\uc131\uc774 \uac04\uc811 \ud761\uc5f0\uc758 \ud76c\uc0dd\uc790\uac00 \ub420 \uac83\uc744 \uc6b0\ub824\ud588\ub2e4.", "sentence_answer": "1941\ub144\uc5d0\ub294 \ub3c5\uc77c\uc758 60\uac1c \ub3c4\uc2dc\uac00 \ub178\uba74 \uc804\ucc28 \uc548\uc5d0\uc11c \ud761\uc5f0\ud558\ub294 \uac83\uc744 \uae08\uc9c0\ud588\ub2e4.", "paragraph_id": "6488722-6-0", "paragraph_question": "question: 1941\ub144 \ub3c5\uc77c\uc758 60\uac1c \ub3c4\uc2dc\uc5d0\uc11c \ud761\uc5f0\uc744 \uae08\uc9c0\ud558\uc600\ub358 \uc2e4\ub0b4 \uc7a5\uc18c\ub294 \uc5b4\ub514\uc600\ub098?, context: 1941\ub144\uc5d0\ub294 \ub3c5\uc77c\uc758 60\uac1c \ub3c4\uc2dc\uac00 \ub178\uba74 \uc804\ucc28 \uc548\uc5d0\uc11c \ud761\uc5f0\ud558\ub294 \uac83\uc744 \uae08\uc9c0\ud588\ub2e4. \ubc29\uacf5\ud638\uc5d0\uc11c\ub3c4 \ud761\uc5f0\uc774 \uae08\uc9c0\ub418\uc5c8\uc9c0\ub9cc \uc77c\ubd80 \ubc29\uacf5\ud638\uc5d0\ub294 \ubcc4\ub3c4\uc758 \ud761\uc5f0 \uad6c\uc5ed\uc774 \ub9c8\ub828\ub418\uc5b4 \uc788\uc5c8\ub2e4. \ud55c\ud3b8 \uc5ec\uc131\uc758 \ud761\uc5f0\uc744 \ub9c9\uae30 \uc704\ud574 \ud2b9\ubcc4\ud55c \uc8fc\uc758\ub97c \ub0b4\ub9ac\uae30\ub3c4 \ud588\ub294\ub370 \ub3c5\uc77c \uc758\ub8cc \ud611\ud68c \ud68c\uc7a5\uc740 \"\ub3c5\uc77c \uc5ec\uc131\uc740 \ub2f4\ubc30\ub97c \ud53c\uc6b0\uc9c0 \uc54a\ub294\ub2e4\"\uace0 \uc120\uc5b8\ud588\uc73c\uba70 \uc81c2\ucc28 \uc138\uacc4 \ub300\uc804 \uc911\uc5d0\ub294 \uc784\uc0b0\ubd80\uc640 25\uc138 \uc774\ud558\uc758 \uc5ec\uc131, 55\uc138 \uc774\uc0c1\uc758 \uc5ec\uc131\uc5d0\uac8c \ub2f4\ubc30 \ubc30\uae09 \uce74\ub4dc\ub97c \uc9c0\uae09\ud558\uc9c0 \uc54a\uc558\uc73c\uba70 \uc11c\ube44\uc2a4\uc5c5\uc790\uc640 \uc2dd\ud488 \uc18c\ub9e4\uc5c5\uc790\uc5d0\uac8c\ub294 \uc5ec\uc131\uc758 \ub2f4\ubc30 \uad6c\ub9e4\ub97c \uc81c\ud55c\ud558\ub294 \uc870\uce58\ub97c \ub0b4\ub838\ub2e4. \uadf9\uc7a5\uc5d0\uc11c\ub294 \uc5ec\uc131\uc744 \ud0c0\uac9f\uc73c\ub85c \ud55c \ub2f4\ubc30 \ubc18\ub300 \uc601\ud654\uac00 \uacf5\uac1c\uc801\uc73c\ub85c \uc0c1\uc601\ub418\uc5c8\uc73c\uba70 \uc2e0\ubb38\uc5d0\ub294 \ud761\uc5f0\uacfc \uc774\uc5d0 \ub300\ud55c \uc601\ud5a5\uc744 \ub2e4\ub8ec \uc0ac\uc124\uc774 \uac8c\uc7ac\ub418\uc5c8\ub2e4. \uad6d\uac00\uc0ac\ud68c\uc8fc\uc758 \ub178\ub3d9 \uc870\ud569\uc758 \uac01 \uc9c0\ubd80\ub294 \ub2f4\ubc30\ub97c \ud53c\uc6b0\ub294 \uc5ec\uc131 \ud68c\uc6d0\uc744 \uacf5\uac1c\uc801\uc73c\ub85c \ucd94\ubc29\uc2dc\ud0a4\ub294 \uc5c4\uaca9\ud55c \uc870\uce58\ub97c \ub0b4\ub838\ub2e4. 1943\ub144 7\uc6d4\uc5d0\ub294 18\uc138 \ubbf8\ub9cc\uc758 \uad6d\ubbfc\uc774 \uacf5\uacf5 \uc7a5\uc18c\uc5d0\uc11c \ud761\uc5f0\ud558\ub294 \uac83\uc774 \uae08\uc9c0\ub418\uc5c8\uace0 1944\ub144\uc5d0\ub294 \ubc84\uc2a4\uc640 \uc5f4\ucc28 \uc548\uc5d0\uc11c \ub2f4\ubc30\ub97c \ud53c\uc6b0\ub294 \uac83\uc870\ucc28 \uae08\uc9c0\ub418\uc5c8\ub2e4. \ud788\ud2c0\ub7ec\ub294 \uc5ec\uc131\uc774 \uac04\uc811 \ud761\uc5f0\uc758 \ud76c\uc0dd\uc790\uac00 \ub420 \uac83\uc744 \uc6b0\ub824\ud588\ub2e4."} {"question": "\uc784\uc9c4\uc65c\ub780\ub54c \uc784\uc9c4\uac15 \ubc29\uc5b4\uc5d0 \uc2e4\ud328\ud558\uc600\ub358 \ub3c4\uc6d0\uc218\ub294 \ub204\uad6c\uc778\uac00?", "paragraph": "\uc625\ud3ec\ud574\uc804\uc740 \uc774\uc21c\uc2e0\uc774 \uccab \uc2b9\uc804\uc744 \uc54c\ub9ac\uac8c \ub41c \ud574\uc804\uc774\ub2e4. 6\uc6d4 16\uc77c(\uc74c\ub825 5\uc6d4 7\uc77c) \uc815\uc624 \uc625\ud3ec\ud56d\uc5d0\uc11c \uc815\ubc15\ud558\uc5ec \uc625\ud3ec\uc5d0 \uce68\ub7b5\ud588\ub358 \uc65c\uad70\uc744 \uacf5\uaca9\ud558\uc5ec \ubaa8\ub450 26\ucc99\uc744 \uce68\ubab0\uc2dc\ucf30\uc73c\uba70, \uc7a1\ud600\uc788\ub358 \ud3ec\ub85c\ub4e4\uc744 \uad6c\ud574\ub0b4\uc5c8\ub2e4. \uac19\uc740 \ub0a0 \uc624\ud6c4\uc5d0\ub294 \uc6c5\ucc9c\ud604\uc758 \ud569\ud3ec \uc55e\ubc14\ub2e4\uc5d0\uc11c \ud070 \ubc30 \ud55c \ucc99\uc744 \ub9cc\ub098 \uc774 \ub610\ud55c \uaca9\ud30c\ud558\uc600\ub2e4. \uc804\ud22c\ub294 17\uc77c(\uc74c\ub825 5\uc6d4 8\uc77c)\uc5d0\ub3c4 \uacc4\uc18d\ub418\uc5c8\uc73c\uba70, \uc801\uc9c4\ud3ec\uc5d0\uc11c \uc65c\uc120 13\ucc99\uc744 \uce68\ubab0\uc2dc\ucf30\uc73c\ub098, \uc804\uc138\uac00 \ubd88\ub9ac\ud558\uc5ec \uc721\uc9c0\ub85c \ub3c4\ub9dd\uac04 \uc801\ub4e4\uc744 \ucad3\uc9c0\ub294 \ubabb\ud558\uc600\ub2e4. \uc774\ub54c \uc870\uc120 \uc870\uc815\uc740 \uc774\ubbf8 \ud55c\uc591\uc5d0\uc11c \ucca0\uc218\ud558\uc5ec \ud3c9\uc548\ub3c4\ub85c \ud6c4\ud1f4\ud558\uace0 \uc788\uc5c8\ub2e4. 27\uc77c(\uc74c\ub825 5\uc6d4 18\uc77c)\uc5d0\ub294 \ub3c4\uc6d0\uc218 \uae40\uba85\uc6d0(\u91d1\u547d\u5143)\uc774 \uc784\uc9c4\uac15 \ubc29\uc5b4\uc5d0 \uc2e4\ud328\ud558\uc5ec \uc870\uc120\ubc18\ub3c4\uc758 \ub300\ubd80\ubd84\uc740 \uc65c\uad70\uc5d0 \uc758\ud558\uc5ec \uc720\ub9b0\ub418\uace0 \uc788\uc5c8\ub2e4.", "answer": "\uae40\uba85\uc6d0", "sentence": "27\uc77c(\uc74c\ub825 5\uc6d4 18\uc77c)\uc5d0\ub294 \ub3c4\uc6d0\uc218 \uae40\uba85\uc6d0 (\u91d1\u547d\u5143)", "paragraph_sentence": "\uc625\ud3ec\ud574\uc804\uc740 \uc774\uc21c\uc2e0\uc774 \uccab \uc2b9\uc804\uc744 \uc54c\ub9ac\uac8c \ub41c \ud574\uc804\uc774\ub2e4. 6\uc6d4 16\uc77c(\uc74c\ub825 5\uc6d4 7\uc77c) \uc815\uc624 \uc625\ud3ec\ud56d\uc5d0\uc11c \uc815\ubc15\ud558\uc5ec \uc625\ud3ec\uc5d0 \uce68\ub7b5\ud588\ub358 \uc65c\uad70\uc744 \uacf5\uaca9\ud558\uc5ec \ubaa8\ub450 26\ucc99\uc744 \uce68\ubab0\uc2dc\ucf30\uc73c\uba70, \uc7a1\ud600\uc788\ub358 \ud3ec\ub85c\ub4e4\uc744 \uad6c\ud574\ub0b4\uc5c8\ub2e4. \uac19\uc740 \ub0a0 \uc624\ud6c4\uc5d0\ub294 \uc6c5\ucc9c\ud604\uc758 \ud569\ud3ec \uc55e\ubc14\ub2e4\uc5d0\uc11c \ud070 \ubc30 \ud55c \ucc99\uc744 \ub9cc\ub098 \uc774 \ub610\ud55c \uaca9\ud30c\ud558\uc600\ub2e4. \uc804\ud22c\ub294 17\uc77c(\uc74c\ub825 5\uc6d4 8\uc77c)\uc5d0\ub3c4 \uacc4\uc18d\ub418\uc5c8\uc73c\uba70, \uc801\uc9c4\ud3ec\uc5d0\uc11c \uc65c\uc120 13\ucc99\uc744 \uce68\ubab0\uc2dc\ucf30\uc73c\ub098, \uc804\uc138\uac00 \ubd88\ub9ac\ud558\uc5ec \uc721\uc9c0\ub85c \ub3c4\ub9dd\uac04 \uc801\ub4e4\uc744 \ucad3\uc9c0\ub294 \ubabb\ud558\uc600\ub2e4. \uc774\ub54c \uc870\uc120 \uc870\uc815\uc740 \uc774\ubbf8 \ud55c\uc591\uc5d0\uc11c \ucca0\uc218\ud558\uc5ec \ud3c9\uc548\ub3c4\ub85c \ud6c4\ud1f4\ud558\uace0 \uc788\uc5c8\ub2e4. 27\uc77c(\uc74c\ub825 5\uc6d4 18\uc77c)\uc5d0\ub294 \ub3c4\uc6d0\uc218 \uae40\uba85\uc6d0 (\u91d1\u547d\u5143) \uc774 \uc784\uc9c4\uac15 \ubc29\uc5b4\uc5d0 \uc2e4\ud328\ud558\uc5ec \uc870\uc120\ubc18\ub3c4\uc758 \ub300\ubd80\ubd84\uc740 \uc65c\uad70\uc5d0 \uc758\ud558\uc5ec \uc720\ub9b0\ub418\uace0 \uc788\uc5c8\ub2e4.", "paragraph_answer": "\uc625\ud3ec\ud574\uc804\uc740 \uc774\uc21c\uc2e0\uc774 \uccab \uc2b9\uc804\uc744 \uc54c\ub9ac\uac8c \ub41c \ud574\uc804\uc774\ub2e4. 6\uc6d4 16\uc77c(\uc74c\ub825 5\uc6d4 7\uc77c) \uc815\uc624 \uc625\ud3ec\ud56d\uc5d0\uc11c \uc815\ubc15\ud558\uc5ec \uc625\ud3ec\uc5d0 \uce68\ub7b5\ud588\ub358 \uc65c\uad70\uc744 \uacf5\uaca9\ud558\uc5ec \ubaa8\ub450 26\ucc99\uc744 \uce68\ubab0\uc2dc\ucf30\uc73c\uba70, \uc7a1\ud600\uc788\ub358 \ud3ec\ub85c\ub4e4\uc744 \uad6c\ud574\ub0b4\uc5c8\ub2e4. \uac19\uc740 \ub0a0 \uc624\ud6c4\uc5d0\ub294 \uc6c5\ucc9c\ud604\uc758 \ud569\ud3ec \uc55e\ubc14\ub2e4\uc5d0\uc11c \ud070 \ubc30 \ud55c \ucc99\uc744 \ub9cc\ub098 \uc774 \ub610\ud55c \uaca9\ud30c\ud558\uc600\ub2e4. \uc804\ud22c\ub294 17\uc77c(\uc74c\ub825 5\uc6d4 8\uc77c)\uc5d0\ub3c4 \uacc4\uc18d\ub418\uc5c8\uc73c\uba70, \uc801\uc9c4\ud3ec\uc5d0\uc11c \uc65c\uc120 13\ucc99\uc744 \uce68\ubab0\uc2dc\ucf30\uc73c\ub098, \uc804\uc138\uac00 \ubd88\ub9ac\ud558\uc5ec \uc721\uc9c0\ub85c \ub3c4\ub9dd\uac04 \uc801\ub4e4\uc744 \ucad3\uc9c0\ub294 \ubabb\ud558\uc600\ub2e4. \uc774\ub54c \uc870\uc120 \uc870\uc815\uc740 \uc774\ubbf8 \ud55c\uc591\uc5d0\uc11c \ucca0\uc218\ud558\uc5ec \ud3c9\uc548\ub3c4\ub85c \ud6c4\ud1f4\ud558\uace0 \uc788\uc5c8\ub2e4. 27\uc77c(\uc74c\ub825 5\uc6d4 18\uc77c)\uc5d0\ub294 \ub3c4\uc6d0\uc218 \uae40\uba85\uc6d0 (\u91d1\u547d\u5143)\uc774 \uc784\uc9c4\uac15 \ubc29\uc5b4\uc5d0 \uc2e4\ud328\ud558\uc5ec \uc870\uc120\ubc18\ub3c4\uc758 \ub300\ubd80\ubd84\uc740 \uc65c\uad70\uc5d0 \uc758\ud558\uc5ec \uc720\ub9b0\ub418\uace0 \uc788\uc5c8\ub2e4.", "sentence_answer": "27\uc77c(\uc74c\ub825 5\uc6d4 18\uc77c)\uc5d0\ub294 \ub3c4\uc6d0\uc218 \uae40\uba85\uc6d0 (\u91d1\u547d\u5143)", "paragraph_id": "6161989-4-1", "paragraph_question": "question: \uc784\uc9c4\uc65c\ub780\ub54c \uc784\uc9c4\uac15 \ubc29\uc5b4\uc5d0 \uc2e4\ud328\ud558\uc600\ub358 \ub3c4\uc6d0\uc218\ub294 \ub204\uad6c\uc778\uac00?, context: \uc625\ud3ec\ud574\uc804\uc740 \uc774\uc21c\uc2e0\uc774 \uccab \uc2b9\uc804\uc744 \uc54c\ub9ac\uac8c \ub41c \ud574\uc804\uc774\ub2e4. 6\uc6d4 16\uc77c(\uc74c\ub825 5\uc6d4 7\uc77c) \uc815\uc624 \uc625\ud3ec\ud56d\uc5d0\uc11c \uc815\ubc15\ud558\uc5ec \uc625\ud3ec\uc5d0 \uce68\ub7b5\ud588\ub358 \uc65c\uad70\uc744 \uacf5\uaca9\ud558\uc5ec \ubaa8\ub450 26\ucc99\uc744 \uce68\ubab0\uc2dc\ucf30\uc73c\uba70, \uc7a1\ud600\uc788\ub358 \ud3ec\ub85c\ub4e4\uc744 \uad6c\ud574\ub0b4\uc5c8\ub2e4. \uac19\uc740 \ub0a0 \uc624\ud6c4\uc5d0\ub294 \uc6c5\ucc9c\ud604\uc758 \ud569\ud3ec \uc55e\ubc14\ub2e4\uc5d0\uc11c \ud070 \ubc30 \ud55c \ucc99\uc744 \ub9cc\ub098 \uc774 \ub610\ud55c \uaca9\ud30c\ud558\uc600\ub2e4. \uc804\ud22c\ub294 17\uc77c(\uc74c\ub825 5\uc6d4 8\uc77c)\uc5d0\ub3c4 \uacc4\uc18d\ub418\uc5c8\uc73c\uba70, \uc801\uc9c4\ud3ec\uc5d0\uc11c \uc65c\uc120 13\ucc99\uc744 \uce68\ubab0\uc2dc\ucf30\uc73c\ub098, \uc804\uc138\uac00 \ubd88\ub9ac\ud558\uc5ec \uc721\uc9c0\ub85c \ub3c4\ub9dd\uac04 \uc801\ub4e4\uc744 \ucad3\uc9c0\ub294 \ubabb\ud558\uc600\ub2e4. \uc774\ub54c \uc870\uc120 \uc870\uc815\uc740 \uc774\ubbf8 \ud55c\uc591\uc5d0\uc11c \ucca0\uc218\ud558\uc5ec \ud3c9\uc548\ub3c4\ub85c \ud6c4\ud1f4\ud558\uace0 \uc788\uc5c8\ub2e4. 27\uc77c(\uc74c\ub825 5\uc6d4 18\uc77c)\uc5d0\ub294 \ub3c4\uc6d0\uc218 \uae40\uba85\uc6d0(\u91d1\u547d\u5143)\uc774 \uc784\uc9c4\uac15 \ubc29\uc5b4\uc5d0 \uc2e4\ud328\ud558\uc5ec \uc870\uc120\ubc18\ub3c4\uc758 \ub300\ubd80\ubd84\uc740 \uc65c\uad70\uc5d0 \uc758\ud558\uc5ec \uc720\ub9b0\ub418\uace0 \uc788\uc5c8\ub2e4."} {"question": "\ud574\ub9ac \ud30c\ud06c\uc2a4\uc758 \uc774\ub984\uc744 \ub534 \ud64d\ucf69 \uc720\uc820\uc655 \uad6c\uc758 \uae38\uac70\ub9ac \uc774\ub984\uc740?", "paragraph": "\ud574\ub9ac \uc2a4\ubbf8\uc2a4 \ud30c\ud06c\uc2a4(Sir Harry Smith Parkes, CGMG, KCB, 1828\ub144 2\uc6d4 24\uc77c \u2013 1885\ub144 3\uc6d4 22\uc77c) \uacbd\uc740 \uc601\uad6d\uc758 \ub300\uc911\uad6d \uc678\uad50\uad00\uc73c\ub85c \uc560\ub85c\ud638 \uc0ac\uac74\uc744 \ud06c\uac8c \ubd88\ub824 \uc81c2\ucc28 \uc544\ud3b8\uc804\uc7c1\uc744 \uc77c\uc73c\ud0a8 \uc7a5\ubcf8\uc778\uc774\ub2e4. \uc6d0\ub798 \ud3c9\ubbfc \ucd9c\uc2e0\uc774\ub098 \uc774 \ub54c\uc758 \uacf5\uc73c\ub85c \uae30\uc0ac\uc791\uc704\uc5d0 \uc62c\ub790\ub2e4. \uc774\ud6c4 \uc77c\ubcf8\uc5b4\uc640 \uc5ed\uc0ac\uae4c\uc9c0 \uc775\ud600 \ub3d9\uc544\uc2dc\uc544 \uc804\ubb38\uac00\uac00 \ub41c \uadf8\ub294 1865\ub144 \uc790\uc2e0\uc758 \uc61b \uc0c1\uad00\uc774\uc790 \uc8fc\uc77c\uc601\uad6d\uacf5\uc0ac \ub7ec\ub354\ud37c\ub4dc \uc62c\ucf55\uc758 \ud6c4\uc784\uc73c\ub85c 18\ub144\uac04 \uc8fc\uc77c\uc601\uad6d\uacf5\uc0ac\ub97c \ub9e1\uc558\ub2e4. 1884\ub144\uc5d0\ub294 \uc8fc\uccad\uc601\uad6d\uacf5\uc0ac \uacb8 \uc8fc\uc870\uc120\uc601\uad6d\uacf5\uc0ac\uc5d0 \uc784\uba85\ub3fc \uc911\uad6d \ubd81\uacbd\uc73c\ub85c \uac74\ub108\uac14\uace0, \uc774\ub4ec\ud574 \uc784\uc9c0\uc5d0\uc11c \uc21c\uc9c1\ud588\ub2e4. \uadf8\ub294 \ud55c\ubc18\ub3c4\ub97c \ub2f4\ub2f9\ud55c \ucd5c\ucd08\uc758 \uc601\uad6d \uc678\uad50\uad00\uc774\ub2e4. \ud604\uc7ac \ud64d\ucf69 \uc720\uc820\uc655 \uad6c\uc758 \ud30c\ud06c\uc2a4 \uc2a4\ud2b8\ub9ac\ud2b8(Parkes Street.)\ub294 \uadf8\uc758 \uc774\ub984\uc744 \ub534 \uac83\uc774\ub2e4.", "answer": "\ud30c\ud06c\uc2a4 \uc2a4\ud2b8\ub9ac\ud2b8", "sentence": "\ud604\uc7ac \ud64d\ucf69 \uc720\uc820\uc655 \uad6c\uc758 \ud30c\ud06c\uc2a4 \uc2a4\ud2b8\ub9ac\ud2b8 (Parkes Street.)", "paragraph_sentence": "\ud574\ub9ac \uc2a4\ubbf8\uc2a4 \ud30c\ud06c\uc2a4(Sir Harry Smith Parkes, CGMG, KCB, 1828\ub144 2\uc6d4 24\uc77c \u2013 1885\ub144 3\uc6d4 22\uc77c) \uacbd\uc740 \uc601\uad6d\uc758 \ub300\uc911\uad6d \uc678\uad50\uad00\uc73c\ub85c \uc560\ub85c\ud638 \uc0ac\uac74\uc744 \ud06c\uac8c \ubd88\ub824 \uc81c2\ucc28 \uc544\ud3b8\uc804\uc7c1\uc744 \uc77c\uc73c\ud0a8 \uc7a5\ubcf8\uc778\uc774\ub2e4. \uc6d0\ub798 \ud3c9\ubbfc \ucd9c\uc2e0\uc774\ub098 \uc774 \ub54c\uc758 \uacf5\uc73c\ub85c \uae30\uc0ac\uc791\uc704\uc5d0 \uc62c\ub790\ub2e4. \uc774\ud6c4 \uc77c\ubcf8\uc5b4\uc640 \uc5ed\uc0ac\uae4c\uc9c0 \uc775\ud600 \ub3d9\uc544\uc2dc\uc544 \uc804\ubb38\uac00\uac00 \ub41c \uadf8\ub294 1865\ub144 \uc790\uc2e0\uc758 \uc61b \uc0c1\uad00\uc774\uc790 \uc8fc\uc77c\uc601\uad6d\uacf5\uc0ac \ub7ec\ub354\ud37c\ub4dc \uc62c\ucf55\uc758 \ud6c4\uc784\uc73c\ub85c 18\ub144\uac04 \uc8fc\uc77c\uc601\uad6d\uacf5\uc0ac\ub97c \ub9e1\uc558\ub2e4. 1884\ub144\uc5d0\ub294 \uc8fc\uccad\uc601\uad6d\uacf5\uc0ac \uacb8 \uc8fc\uc870\uc120\uc601\uad6d\uacf5\uc0ac\uc5d0 \uc784\uba85\ub3fc \uc911\uad6d \ubd81\uacbd\uc73c\ub85c \uac74\ub108\uac14\uace0, \uc774\ub4ec\ud574 \uc784\uc9c0\uc5d0\uc11c \uc21c\uc9c1\ud588\ub2e4. \uadf8\ub294 \ud55c\ubc18\ub3c4\ub97c \ub2f4\ub2f9\ud55c \ucd5c\ucd08\uc758 \uc601\uad6d \uc678\uad50\uad00\uc774\ub2e4. \ud604\uc7ac \ud64d\ucf69 \uc720\uc820\uc655 \uad6c\uc758 \ud30c\ud06c\uc2a4 \uc2a4\ud2b8\ub9ac\ud2b8 (Parkes Street.) \ub294 \uadf8\uc758 \uc774\ub984\uc744 \ub534 \uac83\uc774\ub2e4.", "paragraph_answer": "\ud574\ub9ac \uc2a4\ubbf8\uc2a4 \ud30c\ud06c\uc2a4(Sir Harry Smith Parkes, CGMG, KCB, 1828\ub144 2\uc6d4 24\uc77c \u2013 1885\ub144 3\uc6d4 22\uc77c) \uacbd\uc740 \uc601\uad6d\uc758 \ub300\uc911\uad6d \uc678\uad50\uad00\uc73c\ub85c \uc560\ub85c\ud638 \uc0ac\uac74\uc744 \ud06c\uac8c \ubd88\ub824 \uc81c2\ucc28 \uc544\ud3b8\uc804\uc7c1\uc744 \uc77c\uc73c\ud0a8 \uc7a5\ubcf8\uc778\uc774\ub2e4. \uc6d0\ub798 \ud3c9\ubbfc \ucd9c\uc2e0\uc774\ub098 \uc774 \ub54c\uc758 \uacf5\uc73c\ub85c \uae30\uc0ac\uc791\uc704\uc5d0 \uc62c\ub790\ub2e4. \uc774\ud6c4 \uc77c\ubcf8\uc5b4\uc640 \uc5ed\uc0ac\uae4c\uc9c0 \uc775\ud600 \ub3d9\uc544\uc2dc\uc544 \uc804\ubb38\uac00\uac00 \ub41c \uadf8\ub294 1865\ub144 \uc790\uc2e0\uc758 \uc61b \uc0c1\uad00\uc774\uc790 \uc8fc\uc77c\uc601\uad6d\uacf5\uc0ac \ub7ec\ub354\ud37c\ub4dc \uc62c\ucf55\uc758 \ud6c4\uc784\uc73c\ub85c 18\ub144\uac04 \uc8fc\uc77c\uc601\uad6d\uacf5\uc0ac\ub97c \ub9e1\uc558\ub2e4. 1884\ub144\uc5d0\ub294 \uc8fc\uccad\uc601\uad6d\uacf5\uc0ac \uacb8 \uc8fc\uc870\uc120\uc601\uad6d\uacf5\uc0ac\uc5d0 \uc784\uba85\ub3fc \uc911\uad6d \ubd81\uacbd\uc73c\ub85c \uac74\ub108\uac14\uace0, \uc774\ub4ec\ud574 \uc784\uc9c0\uc5d0\uc11c \uc21c\uc9c1\ud588\ub2e4. \uadf8\ub294 \ud55c\ubc18\ub3c4\ub97c \ub2f4\ub2f9\ud55c \ucd5c\ucd08\uc758 \uc601\uad6d \uc678\uad50\uad00\uc774\ub2e4. \ud604\uc7ac \ud64d\ucf69 \uc720\uc820\uc655 \uad6c\uc758 \ud30c\ud06c\uc2a4 \uc2a4\ud2b8\ub9ac\ud2b8 (Parkes Street.)\ub294 \uadf8\uc758 \uc774\ub984\uc744 \ub534 \uac83\uc774\ub2e4.", "sentence_answer": "\ud604\uc7ac \ud64d\ucf69 \uc720\uc820\uc655 \uad6c\uc758 \ud30c\ud06c\uc2a4 \uc2a4\ud2b8\ub9ac\ud2b8 (Parkes Street.)", "paragraph_id": "6570476-0-2", "paragraph_question": "question: \ud574\ub9ac \ud30c\ud06c\uc2a4\uc758 \uc774\ub984\uc744 \ub534 \ud64d\ucf69 \uc720\uc820\uc655 \uad6c\uc758 \uae38\uac70\ub9ac \uc774\ub984\uc740?, context: \ud574\ub9ac \uc2a4\ubbf8\uc2a4 \ud30c\ud06c\uc2a4(Sir Harry Smith Parkes, CGMG, KCB, 1828\ub144 2\uc6d4 24\uc77c \u2013 1885\ub144 3\uc6d4 22\uc77c) \uacbd\uc740 \uc601\uad6d\uc758 \ub300\uc911\uad6d \uc678\uad50\uad00\uc73c\ub85c \uc560\ub85c\ud638 \uc0ac\uac74\uc744 \ud06c\uac8c \ubd88\ub824 \uc81c2\ucc28 \uc544\ud3b8\uc804\uc7c1\uc744 \uc77c\uc73c\ud0a8 \uc7a5\ubcf8\uc778\uc774\ub2e4. \uc6d0\ub798 \ud3c9\ubbfc \ucd9c\uc2e0\uc774\ub098 \uc774 \ub54c\uc758 \uacf5\uc73c\ub85c \uae30\uc0ac\uc791\uc704\uc5d0 \uc62c\ub790\ub2e4. \uc774\ud6c4 \uc77c\ubcf8\uc5b4\uc640 \uc5ed\uc0ac\uae4c\uc9c0 \uc775\ud600 \ub3d9\uc544\uc2dc\uc544 \uc804\ubb38\uac00\uac00 \ub41c \uadf8\ub294 1865\ub144 \uc790\uc2e0\uc758 \uc61b \uc0c1\uad00\uc774\uc790 \uc8fc\uc77c\uc601\uad6d\uacf5\uc0ac \ub7ec\ub354\ud37c\ub4dc \uc62c\ucf55\uc758 \ud6c4\uc784\uc73c\ub85c 18\ub144\uac04 \uc8fc\uc77c\uc601\uad6d\uacf5\uc0ac\ub97c \ub9e1\uc558\ub2e4. 1884\ub144\uc5d0\ub294 \uc8fc\uccad\uc601\uad6d\uacf5\uc0ac \uacb8 \uc8fc\uc870\uc120\uc601\uad6d\uacf5\uc0ac\uc5d0 \uc784\uba85\ub3fc \uc911\uad6d \ubd81\uacbd\uc73c\ub85c \uac74\ub108\uac14\uace0, \uc774\ub4ec\ud574 \uc784\uc9c0\uc5d0\uc11c \uc21c\uc9c1\ud588\ub2e4. \uadf8\ub294 \ud55c\ubc18\ub3c4\ub97c \ub2f4\ub2f9\ud55c \ucd5c\ucd08\uc758 \uc601\uad6d \uc678\uad50\uad00\uc774\ub2e4. \ud604\uc7ac \ud64d\ucf69 \uc720\uc820\uc655 \uad6c\uc758 \ud30c\ud06c\uc2a4 \uc2a4\ud2b8\ub9ac\ud2b8(Parkes Street.)\ub294 \uadf8\uc758 \uc774\ub984\uc744 \ub534 \uac83\uc774\ub2e4."} {"question": "\ub808\uc774\ub514 \uac00\uac00\uac00 \uc608\uba85\uc744 \uc9d3\ub294\ub370 \uc601\uac10\uc744 \ubc1b\uc740 \ud038\uc758 \ub178\ub798\ub294?", "paragraph": "\uc74c\uc545\uc801\uc73c\ub85c \uac00\uac00\ub294 \ub9c8\ub3c8\ub098, \ub9c8\uc774\ud074 \uc7ad\uc2a8\uacfc \uac19\uc740 \ub304\uc2a4 \ud31d \uac00\uc218\uc640 \ub370\uc774\ube44\ub4dc \ubcf4\uc704, \ud038\uacfc \uac19\uc740 \uae00\ub7a8 \ub85d \uac00\uc218 \ub4f1 \uc218 \ub9ce\uc740 \uac00\uc218\ub4e4\uc5d0\uac8c \uc601\ud5a5\uc744 \ubc1b\uc558\uace0, \uc564\ub514 \uc6cc\ud640\uacfc \uac19\uc740 \uc544\ud2f0\uc2a4\ud2b8\uc758 \uc5f0\uadf9\uc5d0\uc11c \uc74c\uc545 \ud37c\ud3ec\uba3c\uc2a4 \uae30\uc220\uc5d0 \uc601\ud5a5\uc744 \uc92c\ub2e4. \ud038\uc758 \ub178\ub798 \"Radio Ga Ga\"\ub294 \uc608\uba85\uc744 \uc9d3\ub294\ub370 \uc601\uac10\uc744 \uc92c\ub294\ub370, \"\ub098\ub294 \ud504\ub808\ub514 \uba38\ud050\ub9ac\uc640 \ud038\uc744 \uc88b\uc544\ud558\uace0, \"Radio Ga Ga\"\uac00 \ud788\ud2b8\ud588\ub2e4\uace0 \ubcf8\ub2e4. \ub0b4\uac00 \ub808\uc774\ub514 \uac00\uac00\ub77c\ub294 \uc774\ub984\uc744 \uc88b\uc544\ud558\ub294 \uc774\uc720\uc774\ub2e4. \ud504\ub808\ub514\ub294 \ud31d \uc74c\uc545\uacc4\uc5d0\uc11c \uc720\uc77c\ud558\uba74\uc11c\ub3c4 \uac00\uc7a5 \uc874\uc7ac\uac10\uc774 \ud070 \uc720\uba85\uc778 \uc911 \ud55c \uba85\uc774\ub2e4\"\ub77c\uace0 \ub9d0\ud588\ub2e4. \uac00\uac00\ub294 \ub9c8\ub3c8\ub098\uc640 \uc790\uc8fc \ube44\uad50\ub2f9\ud588\ub294\ub370, \uc790\uc2e0\uc758 \ubaa8\uc2b5\uc744 \uac70\uc6b8\ub85c \ubcf4\ub294 \uac83 \uac19\ub2e4\uace0 \uac00\uac00\ub294 \uc2a4\uc2a4\ub85c\ub3c4 \uc778\uc815\ud588\ub2e4. \uac00\uac00\ub294 \ube44\uad50\ub2f9\ud558\ub294 \uac83\uc5d0 \ub300\ud574\uc11c \"\ub098\ub294 \uc8fc\uc81c\ub118\uc740 \uc18c\ub9ac\ub97c \ud558\uace0 \uc2f6\uc9c0 \uc54a\uc9c0\ub9cc, \ud31d \uc74c\uc545\uc5d0 \ub300\ubcc0\ud601\uc744 \uc77c\uc73c\ud0a4\ub294\uac8c \ub0b4 \ubaa9\ud45c\uc774\ub2e4. \ub9c8\ub3c8\ub098\ub294 \uc774\ubbf8 25\ub144 \uc804 \ud601\uba85\uc744 \uc77c\uc73c\ucf30\ub2e4\"\uace0 \uc815\ud558\uba70 \"\ub098\ubcf4\ub2e4 \ub354 \ub9c8\ub3c8\ub098\ub97c \uc0ac\ub791\ud558\uace0 \uc88b\uc544\ud558\ub294 \ud32c\uc740 \uc5c6\ub2e4. \ub098\ub294 \uc9c1\uc5c5\uc801\uc73c\ub85c\ub3c4 \uac1c\uc778\uc801\uc73c\ub85c\ub3c4 \uac00\uc7a5 \ud070 \ud32c\uc774\ub2e4\"\ub77c\uace0 \ub9d0\uc744 \ub354\ud588\ub2e4.", "answer": "Radio Ga Ga", "sentence": "\ud038\uc758 \ub178\ub798 \" Radio Ga Ga \"\ub294 \uc608\uba85\uc744 \uc9d3\ub294\ub370 \uc601\uac10\uc744 \uc92c\ub294\ub370, \"\ub098\ub294 \ud504\ub808\ub514 \uba38\ud050\ub9ac\uc640 \ud038\uc744 \uc88b\uc544\ud558\uace0, \"Radio Ga Ga\"\uac00 \ud788\ud2b8\ud588\ub2e4\uace0 \ubcf8\ub2e4.", "paragraph_sentence": "\uc74c\uc545\uc801\uc73c\ub85c \uac00\uac00\ub294 \ub9c8\ub3c8\ub098, \ub9c8\uc774\ud074 \uc7ad\uc2a8\uacfc \uac19\uc740 \ub304\uc2a4 \ud31d \uac00\uc218\uc640 \ub370\uc774\ube44\ub4dc \ubcf4\uc704, \ud038\uacfc \uac19\uc740 \uae00\ub7a8 \ub85d \uac00\uc218 \ub4f1 \uc218 \ub9ce\uc740 \uac00\uc218\ub4e4\uc5d0\uac8c \uc601\ud5a5\uc744 \ubc1b\uc558\uace0, \uc564\ub514 \uc6cc\ud640\uacfc \uac19\uc740 \uc544\ud2f0\uc2a4\ud2b8\uc758 \uc5f0\uadf9\uc5d0\uc11c \uc74c\uc545 \ud37c\ud3ec\uba3c\uc2a4 \uae30\uc220\uc5d0 \uc601\ud5a5\uc744 \uc92c\ub2e4. \ud038\uc758 \ub178\ub798 \" Radio Ga Ga \"\ub294 \uc608\uba85\uc744 \uc9d3\ub294\ub370 \uc601\uac10\uc744 \uc92c\ub294\ub370, \"\ub098\ub294 \ud504\ub808\ub514 \uba38\ud050\ub9ac\uc640 \ud038\uc744 \uc88b\uc544\ud558\uace0, \"Radio Ga Ga\"\uac00 \ud788\ud2b8\ud588\ub2e4\uace0 \ubcf8\ub2e4. \ub0b4\uac00 \ub808\uc774\ub514 \uac00\uac00\ub77c\ub294 \uc774\ub984\uc744 \uc88b\uc544\ud558\ub294 \uc774\uc720\uc774\ub2e4. \ud504\ub808\ub514\ub294 \ud31d \uc74c\uc545\uacc4\uc5d0\uc11c \uc720\uc77c\ud558\uba74\uc11c\ub3c4 \uac00\uc7a5 \uc874\uc7ac\uac10\uc774 \ud070 \uc720\uba85\uc778 \uc911 \ud55c \uba85\uc774\ub2e4\"\ub77c\uace0 \ub9d0\ud588\ub2e4. \uac00\uac00\ub294 \ub9c8\ub3c8\ub098\uc640 \uc790\uc8fc \ube44\uad50\ub2f9\ud588\ub294\ub370, \uc790\uc2e0\uc758 \ubaa8\uc2b5\uc744 \uac70\uc6b8\ub85c \ubcf4\ub294 \uac83 \uac19\ub2e4\uace0 \uac00\uac00\ub294 \uc2a4\uc2a4\ub85c\ub3c4 \uc778\uc815\ud588\ub2e4. \uac00\uac00\ub294 \ube44\uad50\ub2f9\ud558\ub294 \uac83\uc5d0 \ub300\ud574\uc11c \"\ub098\ub294 \uc8fc\uc81c\ub118\uc740 \uc18c\ub9ac\ub97c \ud558\uace0 \uc2f6\uc9c0 \uc54a\uc9c0\ub9cc, \ud31d \uc74c\uc545\uc5d0 \ub300\ubcc0\ud601\uc744 \uc77c\uc73c\ud0a4\ub294\uac8c \ub0b4 \ubaa9\ud45c\uc774\ub2e4. \ub9c8\ub3c8\ub098\ub294 \uc774\ubbf8 25\ub144 \uc804 \ud601\uba85\uc744 \uc77c\uc73c\ucf30\ub2e4\"\uace0 \uc815\ud558\uba70 \"\ub098\ubcf4\ub2e4 \ub354 \ub9c8\ub3c8\ub098\ub97c \uc0ac\ub791\ud558\uace0 \uc88b\uc544\ud558\ub294 \ud32c\uc740 \uc5c6\ub2e4. \ub098\ub294 \uc9c1\uc5c5\uc801\uc73c\ub85c\ub3c4 \uac1c\uc778\uc801\uc73c\ub85c\ub3c4 \uac00\uc7a5 \ud070 \ud32c\uc774\ub2e4\"\ub77c\uace0 \ub9d0\uc744 \ub354\ud588\ub2e4.", "paragraph_answer": "\uc74c\uc545\uc801\uc73c\ub85c \uac00\uac00\ub294 \ub9c8\ub3c8\ub098, \ub9c8\uc774\ud074 \uc7ad\uc2a8\uacfc \uac19\uc740 \ub304\uc2a4 \ud31d \uac00\uc218\uc640 \ub370\uc774\ube44\ub4dc \ubcf4\uc704, \ud038\uacfc \uac19\uc740 \uae00\ub7a8 \ub85d \uac00\uc218 \ub4f1 \uc218 \ub9ce\uc740 \uac00\uc218\ub4e4\uc5d0\uac8c \uc601\ud5a5\uc744 \ubc1b\uc558\uace0, \uc564\ub514 \uc6cc\ud640\uacfc \uac19\uc740 \uc544\ud2f0\uc2a4\ud2b8\uc758 \uc5f0\uadf9\uc5d0\uc11c \uc74c\uc545 \ud37c\ud3ec\uba3c\uc2a4 \uae30\uc220\uc5d0 \uc601\ud5a5\uc744 \uc92c\ub2e4. \ud038\uc758 \ub178\ub798 \" Radio Ga Ga \"\ub294 \uc608\uba85\uc744 \uc9d3\ub294\ub370 \uc601\uac10\uc744 \uc92c\ub294\ub370, \"\ub098\ub294 \ud504\ub808\ub514 \uba38\ud050\ub9ac\uc640 \ud038\uc744 \uc88b\uc544\ud558\uace0, \"Radio Ga Ga\"\uac00 \ud788\ud2b8\ud588\ub2e4\uace0 \ubcf8\ub2e4. \ub0b4\uac00 \ub808\uc774\ub514 \uac00\uac00\ub77c\ub294 \uc774\ub984\uc744 \uc88b\uc544\ud558\ub294 \uc774\uc720\uc774\ub2e4. \ud504\ub808\ub514\ub294 \ud31d \uc74c\uc545\uacc4\uc5d0\uc11c \uc720\uc77c\ud558\uba74\uc11c\ub3c4 \uac00\uc7a5 \uc874\uc7ac\uac10\uc774 \ud070 \uc720\uba85\uc778 \uc911 \ud55c \uba85\uc774\ub2e4\"\ub77c\uace0 \ub9d0\ud588\ub2e4. \uac00\uac00\ub294 \ub9c8\ub3c8\ub098\uc640 \uc790\uc8fc \ube44\uad50\ub2f9\ud588\ub294\ub370, \uc790\uc2e0\uc758 \ubaa8\uc2b5\uc744 \uac70\uc6b8\ub85c \ubcf4\ub294 \uac83 \uac19\ub2e4\uace0 \uac00\uac00\ub294 \uc2a4\uc2a4\ub85c\ub3c4 \uc778\uc815\ud588\ub2e4. \uac00\uac00\ub294 \ube44\uad50\ub2f9\ud558\ub294 \uac83\uc5d0 \ub300\ud574\uc11c \"\ub098\ub294 \uc8fc\uc81c\ub118\uc740 \uc18c\ub9ac\ub97c \ud558\uace0 \uc2f6\uc9c0 \uc54a\uc9c0\ub9cc, \ud31d \uc74c\uc545\uc5d0 \ub300\ubcc0\ud601\uc744 \uc77c\uc73c\ud0a4\ub294\uac8c \ub0b4 \ubaa9\ud45c\uc774\ub2e4. \ub9c8\ub3c8\ub098\ub294 \uc774\ubbf8 25\ub144 \uc804 \ud601\uba85\uc744 \uc77c\uc73c\ucf30\ub2e4\"\uace0 \uc815\ud558\uba70 \"\ub098\ubcf4\ub2e4 \ub354 \ub9c8\ub3c8\ub098\ub97c \uc0ac\ub791\ud558\uace0 \uc88b\uc544\ud558\ub294 \ud32c\uc740 \uc5c6\ub2e4. \ub098\ub294 \uc9c1\uc5c5\uc801\uc73c\ub85c\ub3c4 \uac1c\uc778\uc801\uc73c\ub85c\ub3c4 \uac00\uc7a5 \ud070 \ud32c\uc774\ub2e4\"\ub77c\uace0 \ub9d0\uc744 \ub354\ud588\ub2e4.", "sentence_answer": "\ud038\uc758 \ub178\ub798 \" Radio Ga Ga \"\ub294 \uc608\uba85\uc744 \uc9d3\ub294\ub370 \uc601\uac10\uc744 \uc92c\ub294\ub370, \"\ub098\ub294 \ud504\ub808\ub514 \uba38\ud050\ub9ac\uc640 \ud038\uc744 \uc88b\uc544\ud558\uace0, \"Radio Ga Ga\"\uac00 \ud788\ud2b8\ud588\ub2e4\uace0 \ubcf8\ub2e4.", "paragraph_id": "6579539-18-0", "paragraph_question": "question: \ub808\uc774\ub514 \uac00\uac00\uac00 \uc608\uba85\uc744 \uc9d3\ub294\ub370 \uc601\uac10\uc744 \ubc1b\uc740 \ud038\uc758 \ub178\ub798\ub294?, context: \uc74c\uc545\uc801\uc73c\ub85c \uac00\uac00\ub294 \ub9c8\ub3c8\ub098, \ub9c8\uc774\ud074 \uc7ad\uc2a8\uacfc \uac19\uc740 \ub304\uc2a4 \ud31d \uac00\uc218\uc640 \ub370\uc774\ube44\ub4dc \ubcf4\uc704, \ud038\uacfc \uac19\uc740 \uae00\ub7a8 \ub85d \uac00\uc218 \ub4f1 \uc218 \ub9ce\uc740 \uac00\uc218\ub4e4\uc5d0\uac8c \uc601\ud5a5\uc744 \ubc1b\uc558\uace0, \uc564\ub514 \uc6cc\ud640\uacfc \uac19\uc740 \uc544\ud2f0\uc2a4\ud2b8\uc758 \uc5f0\uadf9\uc5d0\uc11c \uc74c\uc545 \ud37c\ud3ec\uba3c\uc2a4 \uae30\uc220\uc5d0 \uc601\ud5a5\uc744 \uc92c\ub2e4. \ud038\uc758 \ub178\ub798 \"Radio Ga Ga\"\ub294 \uc608\uba85\uc744 \uc9d3\ub294\ub370 \uc601\uac10\uc744 \uc92c\ub294\ub370, \"\ub098\ub294 \ud504\ub808\ub514 \uba38\ud050\ub9ac\uc640 \ud038\uc744 \uc88b\uc544\ud558\uace0, \"Radio Ga Ga\"\uac00 \ud788\ud2b8\ud588\ub2e4\uace0 \ubcf8\ub2e4. \ub0b4\uac00 \ub808\uc774\ub514 \uac00\uac00\ub77c\ub294 \uc774\ub984\uc744 \uc88b\uc544\ud558\ub294 \uc774\uc720\uc774\ub2e4. \ud504\ub808\ub514\ub294 \ud31d \uc74c\uc545\uacc4\uc5d0\uc11c \uc720\uc77c\ud558\uba74\uc11c\ub3c4 \uac00\uc7a5 \uc874\uc7ac\uac10\uc774 \ud070 \uc720\uba85\uc778 \uc911 \ud55c \uba85\uc774\ub2e4\"\ub77c\uace0 \ub9d0\ud588\ub2e4. \uac00\uac00\ub294 \ub9c8\ub3c8\ub098\uc640 \uc790\uc8fc \ube44\uad50\ub2f9\ud588\ub294\ub370, \uc790\uc2e0\uc758 \ubaa8\uc2b5\uc744 \uac70\uc6b8\ub85c \ubcf4\ub294 \uac83 \uac19\ub2e4\uace0 \uac00\uac00\ub294 \uc2a4\uc2a4\ub85c\ub3c4 \uc778\uc815\ud588\ub2e4. \uac00\uac00\ub294 \ube44\uad50\ub2f9\ud558\ub294 \uac83\uc5d0 \ub300\ud574\uc11c \"\ub098\ub294 \uc8fc\uc81c\ub118\uc740 \uc18c\ub9ac\ub97c \ud558\uace0 \uc2f6\uc9c0 \uc54a\uc9c0\ub9cc, \ud31d \uc74c\uc545\uc5d0 \ub300\ubcc0\ud601\uc744 \uc77c\uc73c\ud0a4\ub294\uac8c \ub0b4 \ubaa9\ud45c\uc774\ub2e4. \ub9c8\ub3c8\ub098\ub294 \uc774\ubbf8 25\ub144 \uc804 \ud601\uba85\uc744 \uc77c\uc73c\ucf30\ub2e4\"\uace0 \uc815\ud558\uba70 \"\ub098\ubcf4\ub2e4 \ub354 \ub9c8\ub3c8\ub098\ub97c \uc0ac\ub791\ud558\uace0 \uc88b\uc544\ud558\ub294 \ud32c\uc740 \uc5c6\ub2e4. \ub098\ub294 \uc9c1\uc5c5\uc801\uc73c\ub85c\ub3c4 \uac1c\uc778\uc801\uc73c\ub85c\ub3c4 \uac00\uc7a5 \ud070 \ud32c\uc774\ub2e4\"\ub77c\uace0 \ub9d0\uc744 \ub354\ud588\ub2e4."} {"question": "\uae00\ub85c\ubc8c \uad50\uc6d0\uc591\uc131 \uac70\uc810\ub300\ud559\uc73c\ub85c \uc120\uc815\ub41c \ud574\ub294 \uba87 \ub144\ub3c4\uc778\uac00?", "paragraph": "2013\ub144 7\uc6d4 \u2018\uae00\ub85c\ubc8c \uad50\uc6d0\uc591\uc131 \uac70\uc810\ub300\ud559\u2019(GTU, Global Teacher's University)\uc73c\ub85c \uc120\uc815\ub418\uc5b4 \uad50\uc721\ubd80\ub85c\ubd80\ud130 \ub9e4\ub144 \uc57d 2\uc5b5 \uc6d0\uc529, 12\ub144\uac04 \uc57d 23\uc5b5 \uc6d0\uc758 \uc0ac\uc5c5\ube44\ub97c \ubc1b\ub294\ub2e4. \ud559\ubd80\uc640 \ub300\ud559\uc6d0 \uacfc\uc815\uc5d0 \uae00\ub85c\ubc8c \uad50\uc6d0\uc591\uc131 \ud504\ub85c\uadf8\ub7a8\uc744 \uc124\uce58\ud558\uc5ec \ud574\uc678\ub300\ud559\uacfc\uc758 \uacf5\ub3d9 \uad50\uc721\uacfc\uc815\uc744 \uc6b4\uc601\ud558\uace0, \ubcf5\uc218\ud559\uc704 \ubc0f \ud574\uc678 \uad50\uc6d0\uc790\uaca9\uc99d \ucde8\ub4dd\uc744 \uc9c0\uc6d0\ud55c\ub2e4. 2015\ub144 9\uc6d4\uc5d0\ub294 \uc911\uad6d \uae38\ub9bc\uc0ac\ubc94\ub300\ud559\uacfc \uacf5\ub3d9 \uc6b4\uc601\ud558\ub294 \u2018\ud55c\u2027\uc911 \uc720\uc544\uad50\uc721\uc804\uacf5 \ubcf8\uacfc \uad50\uc721 \ud504\ub85c\uadf8\ub7a8\u2019\uc758 \uccab \uc2e0\uc785\uc0dd \uc785\ud559\uc2dd\uc744 \uac70\ud589\ud558\uc600\ub2e4. \uc774 \ud504\ub85c\uadf8\ub7a8\uc740 \ub450 \ud559\uad50\uac00 \uacf5\ub3d9\uc73c\ub85c \uad6c\uc131\ud558\ub294 \uc720\uc544\uad50\uc721\uacfc\uc815\uc744 \uc774\uc218\ud558\ub294 \ud559\uc0dd\uc5d0\uac8c \uae38\ub9bc\uc0ac\ubc94\ub300\ud559 \ud559\uc704 \ubc0f \uc911\uad6d \uc720\uc544\uad50\uc0ac \uc790\uaca9\uc99d\uc744 \uc218\uc5ec\ud558\ub294 \ud504\ub85c\uadf8\ub7a8\uc73c\ub85c\uc11c \uc911\uad6d \uad50\uc721\ubd80\uc758 \uc815\uc2dd \uc778\uac00\ub97c \ubc1b\uc544 \uc815\uaddc \uc720\uc544\uad50\uc0ac \uc591\uc131\uacfc\uc815\uc73c\ub85c \uc6b4\uc601\ub41c\ub2e4.", "answer": "2013\ub144", "sentence": "2013\ub144 7\uc6d4 \u2018\uae00\ub85c\ubc8c \uad50\uc6d0\uc591\uc131 \uac70\uc810\ub300\ud559\u2019(GTU, Global Teacher's University)\uc73c\ub85c \uc120\uc815\ub418\uc5b4 \uad50\uc721\ubd80\ub85c\ubd80\ud130 \ub9e4\ub144 \uc57d 2\uc5b5 \uc6d0\uc529, 12\ub144\uac04 \uc57d 23\uc5b5 \uc6d0\uc758 \uc0ac\uc5c5\ube44\ub97c \ubc1b\ub294\ub2e4.", "paragraph_sentence": " 2013\ub144 7\uc6d4 \u2018\uae00\ub85c\ubc8c \uad50\uc6d0\uc591\uc131 \uac70\uc810\ub300\ud559\u2019(GTU, Global Teacher's University)\uc73c\ub85c \uc120\uc815\ub418\uc5b4 \uad50\uc721\ubd80\ub85c\ubd80\ud130 \ub9e4\ub144 \uc57d 2\uc5b5 \uc6d0\uc529, 12\ub144\uac04 \uc57d 23\uc5b5 \uc6d0\uc758 \uc0ac\uc5c5\ube44\ub97c \ubc1b\ub294\ub2e4. \ud559\ubd80\uc640 \ub300\ud559\uc6d0 \uacfc\uc815\uc5d0 \uae00\ub85c\ubc8c \uad50\uc6d0\uc591\uc131 \ud504\ub85c\uadf8\ub7a8\uc744 \uc124\uce58\ud558\uc5ec \ud574\uc678\ub300\ud559\uacfc\uc758 \uacf5\ub3d9 \uad50\uc721\uacfc\uc815\uc744 \uc6b4\uc601\ud558\uace0, \ubcf5\uc218\ud559\uc704 \ubc0f \ud574\uc678 \uad50\uc6d0\uc790\uaca9\uc99d \ucde8\ub4dd\uc744 \uc9c0\uc6d0\ud55c\ub2e4. 2015\ub144 9\uc6d4\uc5d0\ub294 \uc911\uad6d \uae38\ub9bc\uc0ac\ubc94\ub300\ud559\uacfc \uacf5\ub3d9 \uc6b4\uc601\ud558\ub294 \u2018\ud55c\u2027\uc911 \uc720\uc544\uad50\uc721\uc804\uacf5 \ubcf8\uacfc \uad50\uc721 \ud504\ub85c\uadf8\ub7a8\u2019\uc758 \uccab \uc2e0\uc785\uc0dd \uc785\ud559\uc2dd\uc744 \uac70\ud589\ud558\uc600\ub2e4. \uc774 \ud504\ub85c\uadf8\ub7a8\uc740 \ub450 \ud559\uad50\uac00 \uacf5\ub3d9\uc73c\ub85c \uad6c\uc131\ud558\ub294 \uc720\uc544\uad50\uc721\uacfc\uc815\uc744 \uc774\uc218\ud558\ub294 \ud559\uc0dd\uc5d0\uac8c \uae38\ub9bc\uc0ac\ubc94\ub300\ud559 \ud559\uc704 \ubc0f \uc911\uad6d \uc720\uc544\uad50\uc0ac \uc790\uaca9\uc99d\uc744 \uc218\uc5ec\ud558\ub294 \ud504\ub85c\uadf8\ub7a8\uc73c\ub85c\uc11c \uc911\uad6d \uad50\uc721\ubd80\uc758 \uc815\uc2dd \uc778\uac00\ub97c \ubc1b\uc544 \uc815\uaddc \uc720\uc544\uad50\uc0ac \uc591\uc131\uacfc\uc815\uc73c\ub85c \uc6b4\uc601\ub41c\ub2e4.", "paragraph_answer": " 2013\ub144 7\uc6d4 \u2018\uae00\ub85c\ubc8c \uad50\uc6d0\uc591\uc131 \uac70\uc810\ub300\ud559\u2019(GTU, Global Teacher's University)\uc73c\ub85c \uc120\uc815\ub418\uc5b4 \uad50\uc721\ubd80\ub85c\ubd80\ud130 \ub9e4\ub144 \uc57d 2\uc5b5 \uc6d0\uc529, 12\ub144\uac04 \uc57d 23\uc5b5 \uc6d0\uc758 \uc0ac\uc5c5\ube44\ub97c \ubc1b\ub294\ub2e4. \ud559\ubd80\uc640 \ub300\ud559\uc6d0 \uacfc\uc815\uc5d0 \uae00\ub85c\ubc8c \uad50\uc6d0\uc591\uc131 \ud504\ub85c\uadf8\ub7a8\uc744 \uc124\uce58\ud558\uc5ec \ud574\uc678\ub300\ud559\uacfc\uc758 \uacf5\ub3d9 \uad50\uc721\uacfc\uc815\uc744 \uc6b4\uc601\ud558\uace0, \ubcf5\uc218\ud559\uc704 \ubc0f \ud574\uc678 \uad50\uc6d0\uc790\uaca9\uc99d \ucde8\ub4dd\uc744 \uc9c0\uc6d0\ud55c\ub2e4. 2015\ub144 9\uc6d4\uc5d0\ub294 \uc911\uad6d \uae38\ub9bc\uc0ac\ubc94\ub300\ud559\uacfc \uacf5\ub3d9 \uc6b4\uc601\ud558\ub294 \u2018\ud55c\u2027\uc911 \uc720\uc544\uad50\uc721\uc804\uacf5 \ubcf8\uacfc \uad50\uc721 \ud504\ub85c\uadf8\ub7a8\u2019\uc758 \uccab \uc2e0\uc785\uc0dd \uc785\ud559\uc2dd\uc744 \uac70\ud589\ud558\uc600\ub2e4. \uc774 \ud504\ub85c\uadf8\ub7a8\uc740 \ub450 \ud559\uad50\uac00 \uacf5\ub3d9\uc73c\ub85c \uad6c\uc131\ud558\ub294 \uc720\uc544\uad50\uc721\uacfc\uc815\uc744 \uc774\uc218\ud558\ub294 \ud559\uc0dd\uc5d0\uac8c \uae38\ub9bc\uc0ac\ubc94\ub300\ud559 \ud559\uc704 \ubc0f \uc911\uad6d \uc720\uc544\uad50\uc0ac \uc790\uaca9\uc99d\uc744 \uc218\uc5ec\ud558\ub294 \ud504\ub85c\uadf8\ub7a8\uc73c\ub85c\uc11c \uc911\uad6d \uad50\uc721\ubd80\uc758 \uc815\uc2dd \uc778\uac00\ub97c \ubc1b\uc544 \uc815\uaddc \uc720\uc544\uad50\uc0ac \uc591\uc131\uacfc\uc815\uc73c\ub85c \uc6b4\uc601\ub41c\ub2e4.", "sentence_answer": " 2013\ub144 7\uc6d4 \u2018\uae00\ub85c\ubc8c \uad50\uc6d0\uc591\uc131 \uac70\uc810\ub300\ud559\u2019(GTU, Global Teacher's University)\uc73c\ub85c \uc120\uc815\ub418\uc5b4 \uad50\uc721\ubd80\ub85c\ubd80\ud130 \ub9e4\ub144 \uc57d 2\uc5b5 \uc6d0\uc529, 12\ub144\uac04 \uc57d 23\uc5b5 \uc6d0\uc758 \uc0ac\uc5c5\ube44\ub97c \ubc1b\ub294\ub2e4.", "paragraph_id": "6569875-10-0", "paragraph_question": "question: \uae00\ub85c\ubc8c \uad50\uc6d0\uc591\uc131 \uac70\uc810\ub300\ud559\uc73c\ub85c \uc120\uc815\ub41c \ud574\ub294 \uba87 \ub144\ub3c4\uc778\uac00?, context: 2013\ub144 7\uc6d4 \u2018\uae00\ub85c\ubc8c \uad50\uc6d0\uc591\uc131 \uac70\uc810\ub300\ud559\u2019(GTU, Global Teacher's University)\uc73c\ub85c \uc120\uc815\ub418\uc5b4 \uad50\uc721\ubd80\ub85c\ubd80\ud130 \ub9e4\ub144 \uc57d 2\uc5b5 \uc6d0\uc529, 12\ub144\uac04 \uc57d 23\uc5b5 \uc6d0\uc758 \uc0ac\uc5c5\ube44\ub97c \ubc1b\ub294\ub2e4. \ud559\ubd80\uc640 \ub300\ud559\uc6d0 \uacfc\uc815\uc5d0 \uae00\ub85c\ubc8c \uad50\uc6d0\uc591\uc131 \ud504\ub85c\uadf8\ub7a8\uc744 \uc124\uce58\ud558\uc5ec \ud574\uc678\ub300\ud559\uacfc\uc758 \uacf5\ub3d9 \uad50\uc721\uacfc\uc815\uc744 \uc6b4\uc601\ud558\uace0, \ubcf5\uc218\ud559\uc704 \ubc0f \ud574\uc678 \uad50\uc6d0\uc790\uaca9\uc99d \ucde8\ub4dd\uc744 \uc9c0\uc6d0\ud55c\ub2e4. 2015\ub144 9\uc6d4\uc5d0\ub294 \uc911\uad6d \uae38\ub9bc\uc0ac\ubc94\ub300\ud559\uacfc \uacf5\ub3d9 \uc6b4\uc601\ud558\ub294 \u2018\ud55c\u2027\uc911 \uc720\uc544\uad50\uc721\uc804\uacf5 \ubcf8\uacfc \uad50\uc721 \ud504\ub85c\uadf8\ub7a8\u2019\uc758 \uccab \uc2e0\uc785\uc0dd \uc785\ud559\uc2dd\uc744 \uac70\ud589\ud558\uc600\ub2e4. \uc774 \ud504\ub85c\uadf8\ub7a8\uc740 \ub450 \ud559\uad50\uac00 \uacf5\ub3d9\uc73c\ub85c \uad6c\uc131\ud558\ub294 \uc720\uc544\uad50\uc721\uacfc\uc815\uc744 \uc774\uc218\ud558\ub294 \ud559\uc0dd\uc5d0\uac8c \uae38\ub9bc\uc0ac\ubc94\ub300\ud559 \ud559\uc704 \ubc0f \uc911\uad6d \uc720\uc544\uad50\uc0ac \uc790\uaca9\uc99d\uc744 \uc218\uc5ec\ud558\ub294 \ud504\ub85c\uadf8\ub7a8\uc73c\ub85c\uc11c \uc911\uad6d \uad50\uc721\ubd80\uc758 \uc815\uc2dd \uc778\uac00\ub97c \ubc1b\uc544 \uc815\uaddc \uc720\uc544\uad50\uc0ac \uc591\uc131\uacfc\uc815\uc73c\ub85c \uc6b4\uc601\ub41c\ub2e4."} {"question": "2013\ub144 \ud2b8\ub80c\ud2f0\ub178 \ub3d9\uacc4\uc720\ub2c8\ubc84\uc2dc\uc544\ub4dc\ub300\ud68c \ud300\ucd94\uc6d4\uc5d0\uc11c 2\uc704\ub97c \uae30\ub85d\ud55c \ub098\ub77c\ub294?", "paragraph": "2013\ub144 12\uc6d4 13\uc77c \uc774\ud0c8\ub9ac\uc544 \ud2b8\ub80c\ud2f0\ub178\uc758 \ubc14\uc140\uac00\ub514\ud53c\ub124\uc544\uc774\uc2a4\ub9c1\ud06c\uacbd\uae30\uc7a5\uc5d0\uc11c \uc5f4\ub9b0 \uc81c26\ud68c \ud2b8\ub80c\ud2f0\ub178 \ub3d9\uacc4\uc720\ub2c8\ubc84\uc2dc\uc544\ub4dc\ub300\ud68c 1500m \uacbd\uae30\uc5d0\uc11c \uae40\ubcf4\ub984\uc740 2\ubd841\ucd0819\uc758 \uae30\ub85d\uc73c\ub85c \uae08\uba54\ub2ec\uc744 \ud68d\ub4dd\ud588\ub2e4. \uac19\uc740 \uacbd\uae30\uc5d0 \ucd9c\uc804\ud55c \uc591\uc2e0\uc601, \ubc15\ub3c4\uc601\uc740 \uac01\uac01 5\uc704, 6\uc704\ub97c \uae30\ub85d\ud588\ub2e4. 15\uc77c \uc5f4\ub9b0 3000m\uc5d0\uc11c\ub294 \uae40\ubcf4\ub984\uc774 \uc740\uba54\ub2ec\uc744 \ub544\ub2e4. 18\uc77c \uc5f4\ub9b0 5000m\uc5d0\uc11c\ub3c4 \uae40\ubcf4\ub984\uc774 7\ubd8417\ucd0882\ub9cc\uc5d0 \uacb0\uc2b9\uc120\uc744 \ud1b5\uacfc, \ub9c8\ub974\ud2f0\ub098 \uc0ac\ube0c\ub9ac\ucf54\ubc14(\uccb4\ucf54, 7\ubd8405\ucd0817)\uc5d0 \uc774\uc5b4 \uc740\uba54\ub2ec\uc744 \ud68d\ub4dd\ud588\ub2e4. \ub3d9\ubc18 \ucd9c\uc804\ud55c \ubc15\ub3c4\uc601(\ud55c\uad6d\uccb4\ub300)\uc740 7\ubd8426\ucd0858\uc758 \uae30\ub85d\uc73c\ub85c 3\uc704\uc5d0 \uc62c\ub790\ub2e4. 20\uc77c \uc5f4\ub9b0 \ud300\ucd94\uc6d4\uc5d0\uc11c\ub294 \uae40\ubcf4\ub984\uacfc \ubc15\ub3c4\uc601(20-\ud55c\uad6d\uccb4\ub300), \uc591\uc2e0\uc601(23-\uc804\ubd81\ub3c4\uccad)\uc758 \uc5ec\uc790\ub300\ud45c\ud300\uc774 3\ubd8406\ucd0853\uc73c\ub85c \uacb0\uc2b9\uc120\uc744 \ud1b5\uacfc, \uc77c\ubcf8(3\ubd8411\ucd0839)\uc744 \uc81c\uce58\uace0 \uc815\uc0c1\uc5d0 \uc62c\ub790\ub2e4.", "answer": "\uc77c\ubcf8", "sentence": "20\uc77c \uc5f4\ub9b0 \ud300\ucd94\uc6d4\uc5d0\uc11c\ub294 \uae40\ubcf4\ub984\uacfc \ubc15\ub3c4\uc601(20-\ud55c\uad6d\uccb4\ub300), \uc591\uc2e0\uc601(23-\uc804\ubd81\ub3c4\uccad)\uc758 \uc5ec\uc790\ub300\ud45c\ud300\uc774 3\ubd8406\ucd0853\uc73c\ub85c \uacb0\uc2b9\uc120\uc744 \ud1b5\uacfc, \uc77c\ubcf8 (3\ubd8411\ucd0839)\uc744 \uc81c\uce58\uace0 \uc815\uc0c1\uc5d0 \uc62c\ub790\ub2e4.", "paragraph_sentence": "2013\ub144 12\uc6d4 13\uc77c \uc774\ud0c8\ub9ac\uc544 \ud2b8\ub80c\ud2f0\ub178\uc758 \ubc14\uc140\uac00\ub514\ud53c\ub124\uc544\uc774\uc2a4\ub9c1\ud06c\uacbd\uae30\uc7a5\uc5d0\uc11c \uc5f4\ub9b0 \uc81c26\ud68c \ud2b8\ub80c\ud2f0\ub178 \ub3d9\uacc4\uc720\ub2c8\ubc84\uc2dc\uc544\ub4dc\ub300\ud68c 1500m \uacbd\uae30\uc5d0\uc11c \uae40\ubcf4\ub984\uc740 2\ubd841\ucd0819\uc758 \uae30\ub85d\uc73c\ub85c \uae08\uba54\ub2ec\uc744 \ud68d\ub4dd\ud588\ub2e4. \uac19\uc740 \uacbd\uae30\uc5d0 \ucd9c\uc804\ud55c \uc591\uc2e0\uc601, \ubc15\ub3c4\uc601\uc740 \uac01\uac01 5\uc704, 6\uc704\ub97c \uae30\ub85d\ud588\ub2e4. 15\uc77c \uc5f4\ub9b0 3000m\uc5d0\uc11c\ub294 \uae40\ubcf4\ub984\uc774 \uc740\uba54\ub2ec\uc744 \ub544\ub2e4. 18\uc77c \uc5f4\ub9b0 5000m\uc5d0\uc11c\ub3c4 \uae40\ubcf4\ub984\uc774 7\ubd8417\ucd0882\ub9cc\uc5d0 \uacb0\uc2b9\uc120\uc744 \ud1b5\uacfc, \ub9c8\ub974\ud2f0\ub098 \uc0ac\ube0c\ub9ac\ucf54\ubc14(\uccb4\ucf54, 7\ubd8405\ucd0817)\uc5d0 \uc774\uc5b4 \uc740\uba54\ub2ec\uc744 \ud68d\ub4dd\ud588\ub2e4. \ub3d9\ubc18 \ucd9c\uc804\ud55c \ubc15\ub3c4\uc601(\ud55c\uad6d\uccb4\ub300)\uc740 7\ubd8426\ucd0858\uc758 \uae30\ub85d\uc73c\ub85c 3\uc704\uc5d0 \uc62c\ub790\ub2e4. 20\uc77c \uc5f4\ub9b0 \ud300\ucd94\uc6d4\uc5d0\uc11c\ub294 \uae40\ubcf4\ub984\uacfc \ubc15\ub3c4\uc601(20-\ud55c\uad6d\uccb4\ub300), \uc591\uc2e0\uc601(23-\uc804\ubd81\ub3c4\uccad)\uc758 \uc5ec\uc790\ub300\ud45c\ud300\uc774 3\ubd8406\ucd0853\uc73c\ub85c \uacb0\uc2b9\uc120\uc744 \ud1b5\uacfc, \uc77c\ubcf8 (3\ubd8411\ucd0839)\uc744 \uc81c\uce58\uace0 \uc815\uc0c1\uc5d0 \uc62c\ub790\ub2e4. ", "paragraph_answer": "2013\ub144 12\uc6d4 13\uc77c \uc774\ud0c8\ub9ac\uc544 \ud2b8\ub80c\ud2f0\ub178\uc758 \ubc14\uc140\uac00\ub514\ud53c\ub124\uc544\uc774\uc2a4\ub9c1\ud06c\uacbd\uae30\uc7a5\uc5d0\uc11c \uc5f4\ub9b0 \uc81c26\ud68c \ud2b8\ub80c\ud2f0\ub178 \ub3d9\uacc4\uc720\ub2c8\ubc84\uc2dc\uc544\ub4dc\ub300\ud68c 1500m \uacbd\uae30\uc5d0\uc11c \uae40\ubcf4\ub984\uc740 2\ubd841\ucd0819\uc758 \uae30\ub85d\uc73c\ub85c \uae08\uba54\ub2ec\uc744 \ud68d\ub4dd\ud588\ub2e4. \uac19\uc740 \uacbd\uae30\uc5d0 \ucd9c\uc804\ud55c \uc591\uc2e0\uc601, \ubc15\ub3c4\uc601\uc740 \uac01\uac01 5\uc704, 6\uc704\ub97c \uae30\ub85d\ud588\ub2e4. 15\uc77c \uc5f4\ub9b0 3000m\uc5d0\uc11c\ub294 \uae40\ubcf4\ub984\uc774 \uc740\uba54\ub2ec\uc744 \ub544\ub2e4. 18\uc77c \uc5f4\ub9b0 5000m\uc5d0\uc11c\ub3c4 \uae40\ubcf4\ub984\uc774 7\ubd8417\ucd0882\ub9cc\uc5d0 \uacb0\uc2b9\uc120\uc744 \ud1b5\uacfc, \ub9c8\ub974\ud2f0\ub098 \uc0ac\ube0c\ub9ac\ucf54\ubc14(\uccb4\ucf54, 7\ubd8405\ucd0817)\uc5d0 \uc774\uc5b4 \uc740\uba54\ub2ec\uc744 \ud68d\ub4dd\ud588\ub2e4. \ub3d9\ubc18 \ucd9c\uc804\ud55c \ubc15\ub3c4\uc601(\ud55c\uad6d\uccb4\ub300)\uc740 7\ubd8426\ucd0858\uc758 \uae30\ub85d\uc73c\ub85c 3\uc704\uc5d0 \uc62c\ub790\ub2e4. 20\uc77c \uc5f4\ub9b0 \ud300\ucd94\uc6d4\uc5d0\uc11c\ub294 \uae40\ubcf4\ub984\uacfc \ubc15\ub3c4\uc601(20-\ud55c\uad6d\uccb4\ub300), \uc591\uc2e0\uc601(23-\uc804\ubd81\ub3c4\uccad)\uc758 \uc5ec\uc790\ub300\ud45c\ud300\uc774 3\ubd8406\ucd0853\uc73c\ub85c \uacb0\uc2b9\uc120\uc744 \ud1b5\uacfc, \uc77c\ubcf8 (3\ubd8411\ucd0839)\uc744 \uc81c\uce58\uace0 \uc815\uc0c1\uc5d0 \uc62c\ub790\ub2e4.", "sentence_answer": "20\uc77c \uc5f4\ub9b0 \ud300\ucd94\uc6d4\uc5d0\uc11c\ub294 \uae40\ubcf4\ub984\uacfc \ubc15\ub3c4\uc601(20-\ud55c\uad6d\uccb4\ub300), \uc591\uc2e0\uc601(23-\uc804\ubd81\ub3c4\uccad)\uc758 \uc5ec\uc790\ub300\ud45c\ud300\uc774 3\ubd8406\ucd0853\uc73c\ub85c \uacb0\uc2b9\uc120\uc744 \ud1b5\uacfc, \uc77c\ubcf8 (3\ubd8411\ucd0839)\uc744 \uc81c\uce58\uace0 \uc815\uc0c1\uc5d0 \uc62c\ub790\ub2e4.", "paragraph_id": "6488275-7-1", "paragraph_question": "question: 2013\ub144 \ud2b8\ub80c\ud2f0\ub178 \ub3d9\uacc4\uc720\ub2c8\ubc84\uc2dc\uc544\ub4dc\ub300\ud68c \ud300\ucd94\uc6d4\uc5d0\uc11c 2\uc704\ub97c \uae30\ub85d\ud55c \ub098\ub77c\ub294?, context: 2013\ub144 12\uc6d4 13\uc77c \uc774\ud0c8\ub9ac\uc544 \ud2b8\ub80c\ud2f0\ub178\uc758 \ubc14\uc140\uac00\ub514\ud53c\ub124\uc544\uc774\uc2a4\ub9c1\ud06c\uacbd\uae30\uc7a5\uc5d0\uc11c \uc5f4\ub9b0 \uc81c26\ud68c \ud2b8\ub80c\ud2f0\ub178 \ub3d9\uacc4\uc720\ub2c8\ubc84\uc2dc\uc544\ub4dc\ub300\ud68c 1500m \uacbd\uae30\uc5d0\uc11c \uae40\ubcf4\ub984\uc740 2\ubd841\ucd0819\uc758 \uae30\ub85d\uc73c\ub85c \uae08\uba54\ub2ec\uc744 \ud68d\ub4dd\ud588\ub2e4. \uac19\uc740 \uacbd\uae30\uc5d0 \ucd9c\uc804\ud55c \uc591\uc2e0\uc601, \ubc15\ub3c4\uc601\uc740 \uac01\uac01 5\uc704, 6\uc704\ub97c \uae30\ub85d\ud588\ub2e4. 15\uc77c \uc5f4\ub9b0 3000m\uc5d0\uc11c\ub294 \uae40\ubcf4\ub984\uc774 \uc740\uba54\ub2ec\uc744 \ub544\ub2e4. 18\uc77c \uc5f4\ub9b0 5000m\uc5d0\uc11c\ub3c4 \uae40\ubcf4\ub984\uc774 7\ubd8417\ucd0882\ub9cc\uc5d0 \uacb0\uc2b9\uc120\uc744 \ud1b5\uacfc, \ub9c8\ub974\ud2f0\ub098 \uc0ac\ube0c\ub9ac\ucf54\ubc14(\uccb4\ucf54, 7\ubd8405\ucd0817)\uc5d0 \uc774\uc5b4 \uc740\uba54\ub2ec\uc744 \ud68d\ub4dd\ud588\ub2e4. \ub3d9\ubc18 \ucd9c\uc804\ud55c \ubc15\ub3c4\uc601(\ud55c\uad6d\uccb4\ub300)\uc740 7\ubd8426\ucd0858\uc758 \uae30\ub85d\uc73c\ub85c 3\uc704\uc5d0 \uc62c\ub790\ub2e4. 20\uc77c \uc5f4\ub9b0 \ud300\ucd94\uc6d4\uc5d0\uc11c\ub294 \uae40\ubcf4\ub984\uacfc \ubc15\ub3c4\uc601(20-\ud55c\uad6d\uccb4\ub300), \uc591\uc2e0\uc601(23-\uc804\ubd81\ub3c4\uccad)\uc758 \uc5ec\uc790\ub300\ud45c\ud300\uc774 3\ubd8406\ucd0853\uc73c\ub85c \uacb0\uc2b9\uc120\uc744 \ud1b5\uacfc, \uc77c\ubcf8(3\ubd8411\ucd0839)\uc744 \uc81c\uce58\uace0 \uc815\uc0c1\uc5d0 \uc62c\ub790\ub2e4."} {"question": "2012\ub144 \ub300\uc120\ub54c \uad6d\uac00\uc815\ubcf4\uc6d0\uc740 \uc5b4\ub514\ub97c \ub3d9\uc6d0\ud558\uc5ec \uc5ec\ub860 \uc870\uc791\uc744 \uc2e4\uc2dc\ud558\uc600\ub294\uac00?", "paragraph": "2012\ub144 \ub300\uc120 \ub2f9\uc2dc \uad6d\uac00\uc815\ubcf4\uc6d0\uacfc \uad6d\uad70\uc0ac\uc774\ubc84\uc0ac\ub839\ubd80\uac00 \uc9c1\uc6d0\ub4e4\uc744 \ub3d9\uc6d0\ud574 \ubb38\uc7ac\uc778\uc744 \ube44\ubc29\ud558\ub294 \ub313\uae00\uc744 \uc4f0\ub294 \ub4f1 \ubc15\uadfc\ud61c\uc5d0\uac8c \uc720\ub9ac\ud558\ub3c4\ub85d \uc5ec\ub860\uc744 \uc870\uc791\ud55c \uac83\uc774 \uc544\ub2c8\ub0d0\ub294 \uc8fc\uc7a5\uc774 \uc81c\uae30\ub418\uc5c8\ub2e4. '\uad6d\uac00\uc815\ubcf4\uc6d0 \uc0ac\uac74'\uc774 \ubd88\uac70\uc838 \uc218\uc0ac\ub97c \uc9c4\ud589 \uc911\uc774\ub358 2012\ub144 12\uc6d4 16\uc77c, \ubc15\uadfc\ud61c\ub294 \ubbfc\uc8fc\ud1b5\ud569\ub2f9 \ubb38\uc7ac\uc778 \ud6c4\ubcf4\uc640\uc758 \ud1a0\ub860 \uc911 \ub2f9\ud574 \uc0ac\uac74\uc758 \uc870\uc0ac \uacb0\uacfc\uac00 \ubc1c\ud45c\ub418\uae30 \uc804\uc774\uc5c8\uc74c\uc5d0\ub3c4, \ud574\ub2f9 \uc0ac\uac74\uc758 \uc99d\uac70\uac00 \uc5c6\ub294 \uac83\uc73c\ub85c \ub098\uc654\ub2e4\uace0 \ubc1c\uc5b8\ud558\uc600\ub2e4. \ud1a0\ub860\ud68c \uc9c1\ud6c4 \ub300\uc120 3\uc77c \uc804\uc778 12\uc6d4 16\uc77c \uc77c\uc694\uc77c, \uc11c\uc6b8\uc9c0\ubc29\uacbd\ucc30\uccad\uc740 \uad6d\uc815\uc6d0 \uc5ec\uc9c1\uc6d0\uc774 \ub313\uae00\uc744 \ub2e8 \ud754\uc801\uc774 \uc5c6\ub2e4\uace0 \uc0ac\uc804 \ubc1c\ud45c\ud588\ub2e4. \uadf8\ub7ec\ub098 \ub300\uc120 \uc774\ud6c4\uc778 2013\ub144 6\uc6d4, \uac80\ucc30\uc740 \uacbd\ucc30\uc758 \ubc1c\ud45c\uc640 \ub2ec\ub9ac \uad6d\uc815\uc6d0 \uc9c1\uc6d0\ub4e4\uc774 \ub300\ud1b5\ub839 \uc120\uac70\uc640 \uad00\ub828\ub41c \ub313\uae00\uc744 \ub2e8 \uc0ac\uc2e4\uc774 \uc788\ub2e4\uace0 \uc0c1\ubc18\ub41c \ubc1c\ud45c\ub97c \ud588\ub2e4. \ub354\uad6c\ub098 \uc11c\uc6b8\uc9c0\ubc29\uacbd\ucc30\uccad\uc774 \uac70\uc9d3 \ubc1c\ud45c\ub97c \ud560 \ub2f9\uc2dc \uad6d\uc815\uc6d0 \uc9c1\uc6d0\ub4e4\uc774 \ub300\ud1b5\ub839 \uc120\uac70\uc640 \uad00\ub828\ub41c \ub313\uae00\uc744 \ub2e8 \uc0ac\uc2e4\uc774 \uc788\ub294 \uac83\uc744 \uc774\ubbf8 \ud655\uc778\ud588\ub358 \uac83\uc73c\ub85c \ub4dc\ub7ec\ub0ac\ub2e4.", "answer": "\uad6d\uad70\uc0ac\uc774\ubc84\uc0ac\ub839\ubd80", "sentence": "2012\ub144 \ub300\uc120 \ub2f9\uc2dc \uad6d\uac00\uc815\ubcf4\uc6d0\uacfc \uad6d\uad70\uc0ac\uc774\ubc84\uc0ac\ub839\ubd80 \uac00 \uc9c1\uc6d0\ub4e4\uc744 \ub3d9\uc6d0\ud574 \ubb38\uc7ac\uc778\uc744 \ube44\ubc29\ud558\ub294 \ub313\uae00\uc744 \uc4f0\ub294 \ub4f1 \ubc15\uadfc\ud61c\uc5d0\uac8c \uc720\ub9ac\ud558\ub3c4\ub85d \uc5ec\ub860\uc744 \uc870\uc791\ud55c \uac83\uc774 \uc544\ub2c8\ub0d0\ub294 \uc8fc\uc7a5\uc774 \uc81c\uae30\ub418\uc5c8\ub2e4.", "paragraph_sentence": " 2012\ub144 \ub300\uc120 \ub2f9\uc2dc \uad6d\uac00\uc815\ubcf4\uc6d0\uacfc \uad6d\uad70\uc0ac\uc774\ubc84\uc0ac\ub839\ubd80 \uac00 \uc9c1\uc6d0\ub4e4\uc744 \ub3d9\uc6d0\ud574 \ubb38\uc7ac\uc778\uc744 \ube44\ubc29\ud558\ub294 \ub313\uae00\uc744 \uc4f0\ub294 \ub4f1 \ubc15\uadfc\ud61c\uc5d0\uac8c \uc720\ub9ac\ud558\ub3c4\ub85d \uc5ec\ub860\uc744 \uc870\uc791\ud55c \uac83\uc774 \uc544\ub2c8\ub0d0\ub294 \uc8fc\uc7a5\uc774 \uc81c\uae30\ub418\uc5c8\ub2e4. '\uad6d\uac00\uc815\ubcf4\uc6d0 \uc0ac\uac74'\uc774 \ubd88\uac70\uc838 \uc218\uc0ac\ub97c \uc9c4\ud589 \uc911\uc774\ub358 2012\ub144 12\uc6d4 16\uc77c, \ubc15\uadfc\ud61c\ub294 \ubbfc\uc8fc\ud1b5\ud569\ub2f9 \ubb38\uc7ac\uc778 \ud6c4\ubcf4\uc640\uc758 \ud1a0\ub860 \uc911 \ub2f9\ud574 \uc0ac\uac74\uc758 \uc870\uc0ac \uacb0\uacfc\uac00 \ubc1c\ud45c\ub418\uae30 \uc804\uc774\uc5c8\uc74c\uc5d0\ub3c4, \ud574\ub2f9 \uc0ac\uac74\uc758 \uc99d\uac70\uac00 \uc5c6\ub294 \uac83\uc73c\ub85c \ub098\uc654\ub2e4\uace0 \ubc1c\uc5b8\ud558\uc600\ub2e4. \ud1a0\ub860\ud68c \uc9c1\ud6c4 \ub300\uc120 3\uc77c \uc804\uc778 12\uc6d4 16\uc77c \uc77c\uc694\uc77c, \uc11c\uc6b8\uc9c0\ubc29\uacbd\ucc30\uccad\uc740 \uad6d\uc815\uc6d0 \uc5ec\uc9c1\uc6d0\uc774 \ub313\uae00\uc744 \ub2e8 \ud754\uc801\uc774 \uc5c6\ub2e4\uace0 \uc0ac\uc804 \ubc1c\ud45c\ud588\ub2e4. \uadf8\ub7ec\ub098 \ub300\uc120 \uc774\ud6c4\uc778 2013\ub144 6\uc6d4, \uac80\ucc30\uc740 \uacbd\ucc30\uc758 \ubc1c\ud45c\uc640 \ub2ec\ub9ac \uad6d\uc815\uc6d0 \uc9c1\uc6d0\ub4e4\uc774 \ub300\ud1b5\ub839 \uc120\uac70\uc640 \uad00\ub828\ub41c \ub313\uae00\uc744 \ub2e8 \uc0ac\uc2e4\uc774 \uc788\ub2e4\uace0 \uc0c1\ubc18\ub41c \ubc1c\ud45c\ub97c \ud588\ub2e4. \ub354\uad6c\ub098 \uc11c\uc6b8\uc9c0\ubc29\uacbd\ucc30\uccad\uc774 \uac70\uc9d3 \ubc1c\ud45c\ub97c \ud560 \ub2f9\uc2dc \uad6d\uc815\uc6d0 \uc9c1\uc6d0\ub4e4\uc774 \ub300\ud1b5\ub839 \uc120\uac70\uc640 \uad00\ub828\ub41c \ub313\uae00\uc744 \ub2e8 \uc0ac\uc2e4\uc774 \uc788\ub294 \uac83\uc744 \uc774\ubbf8 \ud655\uc778\ud588\ub358 \uac83\uc73c\ub85c \ub4dc\ub7ec\ub0ac\ub2e4.", "paragraph_answer": "2012\ub144 \ub300\uc120 \ub2f9\uc2dc \uad6d\uac00\uc815\ubcf4\uc6d0\uacfc \uad6d\uad70\uc0ac\uc774\ubc84\uc0ac\ub839\ubd80 \uac00 \uc9c1\uc6d0\ub4e4\uc744 \ub3d9\uc6d0\ud574 \ubb38\uc7ac\uc778\uc744 \ube44\ubc29\ud558\ub294 \ub313\uae00\uc744 \uc4f0\ub294 \ub4f1 \ubc15\uadfc\ud61c\uc5d0\uac8c \uc720\ub9ac\ud558\ub3c4\ub85d \uc5ec\ub860\uc744 \uc870\uc791\ud55c \uac83\uc774 \uc544\ub2c8\ub0d0\ub294 \uc8fc\uc7a5\uc774 \uc81c\uae30\ub418\uc5c8\ub2e4. '\uad6d\uac00\uc815\ubcf4\uc6d0 \uc0ac\uac74'\uc774 \ubd88\uac70\uc838 \uc218\uc0ac\ub97c \uc9c4\ud589 \uc911\uc774\ub358 2012\ub144 12\uc6d4 16\uc77c, \ubc15\uadfc\ud61c\ub294 \ubbfc\uc8fc\ud1b5\ud569\ub2f9 \ubb38\uc7ac\uc778 \ud6c4\ubcf4\uc640\uc758 \ud1a0\ub860 \uc911 \ub2f9\ud574 \uc0ac\uac74\uc758 \uc870\uc0ac \uacb0\uacfc\uac00 \ubc1c\ud45c\ub418\uae30 \uc804\uc774\uc5c8\uc74c\uc5d0\ub3c4, \ud574\ub2f9 \uc0ac\uac74\uc758 \uc99d\uac70\uac00 \uc5c6\ub294 \uac83\uc73c\ub85c \ub098\uc654\ub2e4\uace0 \ubc1c\uc5b8\ud558\uc600\ub2e4. \ud1a0\ub860\ud68c \uc9c1\ud6c4 \ub300\uc120 3\uc77c \uc804\uc778 12\uc6d4 16\uc77c \uc77c\uc694\uc77c, \uc11c\uc6b8\uc9c0\ubc29\uacbd\ucc30\uccad\uc740 \uad6d\uc815\uc6d0 \uc5ec\uc9c1\uc6d0\uc774 \ub313\uae00\uc744 \ub2e8 \ud754\uc801\uc774 \uc5c6\ub2e4\uace0 \uc0ac\uc804 \ubc1c\ud45c\ud588\ub2e4. \uadf8\ub7ec\ub098 \ub300\uc120 \uc774\ud6c4\uc778 2013\ub144 6\uc6d4, \uac80\ucc30\uc740 \uacbd\ucc30\uc758 \ubc1c\ud45c\uc640 \ub2ec\ub9ac \uad6d\uc815\uc6d0 \uc9c1\uc6d0\ub4e4\uc774 \ub300\ud1b5\ub839 \uc120\uac70\uc640 \uad00\ub828\ub41c \ub313\uae00\uc744 \ub2e8 \uc0ac\uc2e4\uc774 \uc788\ub2e4\uace0 \uc0c1\ubc18\ub41c \ubc1c\ud45c\ub97c \ud588\ub2e4. \ub354\uad6c\ub098 \uc11c\uc6b8\uc9c0\ubc29\uacbd\ucc30\uccad\uc774 \uac70\uc9d3 \ubc1c\ud45c\ub97c \ud560 \ub2f9\uc2dc \uad6d\uc815\uc6d0 \uc9c1\uc6d0\ub4e4\uc774 \ub300\ud1b5\ub839 \uc120\uac70\uc640 \uad00\ub828\ub41c \ub313\uae00\uc744 \ub2e8 \uc0ac\uc2e4\uc774 \uc788\ub294 \uac83\uc744 \uc774\ubbf8 \ud655\uc778\ud588\ub358 \uac83\uc73c\ub85c \ub4dc\ub7ec\ub0ac\ub2e4.", "sentence_answer": "2012\ub144 \ub300\uc120 \ub2f9\uc2dc \uad6d\uac00\uc815\ubcf4\uc6d0\uacfc \uad6d\uad70\uc0ac\uc774\ubc84\uc0ac\ub839\ubd80 \uac00 \uc9c1\uc6d0\ub4e4\uc744 \ub3d9\uc6d0\ud574 \ubb38\uc7ac\uc778\uc744 \ube44\ubc29\ud558\ub294 \ub313\uae00\uc744 \uc4f0\ub294 \ub4f1 \ubc15\uadfc\ud61c\uc5d0\uac8c \uc720\ub9ac\ud558\ub3c4\ub85d \uc5ec\ub860\uc744 \uc870\uc791\ud55c \uac83\uc774 \uc544\ub2c8\ub0d0\ub294 \uc8fc\uc7a5\uc774 \uc81c\uae30\ub418\uc5c8\ub2e4.", "paragraph_id": "6506540-20-0", "paragraph_question": "question: 2012\ub144 \ub300\uc120\ub54c \uad6d\uac00\uc815\ubcf4\uc6d0\uc740 \uc5b4\ub514\ub97c \ub3d9\uc6d0\ud558\uc5ec \uc5ec\ub860 \uc870\uc791\uc744 \uc2e4\uc2dc\ud558\uc600\ub294\uac00?, context: 2012\ub144 \ub300\uc120 \ub2f9\uc2dc \uad6d\uac00\uc815\ubcf4\uc6d0\uacfc \uad6d\uad70\uc0ac\uc774\ubc84\uc0ac\ub839\ubd80\uac00 \uc9c1\uc6d0\ub4e4\uc744 \ub3d9\uc6d0\ud574 \ubb38\uc7ac\uc778\uc744 \ube44\ubc29\ud558\ub294 \ub313\uae00\uc744 \uc4f0\ub294 \ub4f1 \ubc15\uadfc\ud61c\uc5d0\uac8c \uc720\ub9ac\ud558\ub3c4\ub85d \uc5ec\ub860\uc744 \uc870\uc791\ud55c \uac83\uc774 \uc544\ub2c8\ub0d0\ub294 \uc8fc\uc7a5\uc774 \uc81c\uae30\ub418\uc5c8\ub2e4. '\uad6d\uac00\uc815\ubcf4\uc6d0 \uc0ac\uac74'\uc774 \ubd88\uac70\uc838 \uc218\uc0ac\ub97c \uc9c4\ud589 \uc911\uc774\ub358 2012\ub144 12\uc6d4 16\uc77c, \ubc15\uadfc\ud61c\ub294 \ubbfc\uc8fc\ud1b5\ud569\ub2f9 \ubb38\uc7ac\uc778 \ud6c4\ubcf4\uc640\uc758 \ud1a0\ub860 \uc911 \ub2f9\ud574 \uc0ac\uac74\uc758 \uc870\uc0ac \uacb0\uacfc\uac00 \ubc1c\ud45c\ub418\uae30 \uc804\uc774\uc5c8\uc74c\uc5d0\ub3c4, \ud574\ub2f9 \uc0ac\uac74\uc758 \uc99d\uac70\uac00 \uc5c6\ub294 \uac83\uc73c\ub85c \ub098\uc654\ub2e4\uace0 \ubc1c\uc5b8\ud558\uc600\ub2e4. \ud1a0\ub860\ud68c \uc9c1\ud6c4 \ub300\uc120 3\uc77c \uc804\uc778 12\uc6d4 16\uc77c \uc77c\uc694\uc77c, \uc11c\uc6b8\uc9c0\ubc29\uacbd\ucc30\uccad\uc740 \uad6d\uc815\uc6d0 \uc5ec\uc9c1\uc6d0\uc774 \ub313\uae00\uc744 \ub2e8 \ud754\uc801\uc774 \uc5c6\ub2e4\uace0 \uc0ac\uc804 \ubc1c\ud45c\ud588\ub2e4. \uadf8\ub7ec\ub098 \ub300\uc120 \uc774\ud6c4\uc778 2013\ub144 6\uc6d4, \uac80\ucc30\uc740 \uacbd\ucc30\uc758 \ubc1c\ud45c\uc640 \ub2ec\ub9ac \uad6d\uc815\uc6d0 \uc9c1\uc6d0\ub4e4\uc774 \ub300\ud1b5\ub839 \uc120\uac70\uc640 \uad00\ub828\ub41c \ub313\uae00\uc744 \ub2e8 \uc0ac\uc2e4\uc774 \uc788\ub2e4\uace0 \uc0c1\ubc18\ub41c \ubc1c\ud45c\ub97c \ud588\ub2e4. \ub354\uad6c\ub098 \uc11c\uc6b8\uc9c0\ubc29\uacbd\ucc30\uccad\uc774 \uac70\uc9d3 \ubc1c\ud45c\ub97c \ud560 \ub2f9\uc2dc \uad6d\uc815\uc6d0 \uc9c1\uc6d0\ub4e4\uc774 \ub300\ud1b5\ub839 \uc120\uac70\uc640 \uad00\ub828\ub41c \ub313\uae00\uc744 \ub2e8 \uc0ac\uc2e4\uc774 \uc788\ub294 \uac83\uc744 \uc774\ubbf8 \ud655\uc778\ud588\ub358 \uac83\uc73c\ub85c \ub4dc\ub7ec\ub0ac\ub2e4."} {"question": "\uad6d\uc81c \uc0ac\ud68c\uc5d0 \uc5d0\ubcfc\ub77c \ubc14\uc774\ub7ec\uc2a4 \uc804\uc5fc\ubcd1 \ud1f4\uce58\ub97c \uc704\ud55c \uc9c0\uc6d0 \ucd09\uad6c \uc131\uba85\uc744 \ubc1c\ud45c\ud55c \ub2e8\uccb4\ub294?", "paragraph": "\uae30\ub2c8, \uc2dc\uc5d0\ub77c\ub9ac\uc628, \ub77c\uc774\ubca0\ub9ac\uc544\uc758 \uc815\ubd80 \uae30\uad6c\ub294 \uad6d\uac00 \ube44\uc0c1\uc0ac\ud0dc\ub97c \ubc1c\ub839\ud558\uace0 \uc5d0\ubcfc\ub77c \ubc14\uc774\ub7ec\uc2a4 \ub300\uc751 \uacc4\ud68d \ubc0f \uc2e4\uc2dc\uc5d0 \ub300\ud55c \uc900\ube44\uc5d0 \ub4e4\uc5b4\uac14\ub2e4. \uc11c\uc544\ud504\ub9ac\uce74 \uacbd\uc81c \uacf5\ub3d9\uccb4 (ECOWAS)\ub294 \uc804\uc5fc\ubcd1 \ud1f4\uce58\ub97c \uc704\ud55c \uad6d\uc81c \uc0ac\ud68c\uc758 \uc9c0\uc6d0\uc744 \ucd09\uad6c\ud558\ub294 \uc131\uba85\uc744 \ubc1c\ud45c\ud588\ub2e4. \"ECOWAS\uc758 \ubcf4\uc548 \ubc0f \uc911\uc7ac\uc704\uc6d0\ud68c\ub294 \uc5d0\ubcfc\ub77c \uc804\uc5fc\ubcd1 \uad00\ub9ac \uc9c0\uc6d0\uc744 \uc704\ud574 \uad6d\uc81c \uc0ac\ud68c\uc5d0\uac8c \ud638\uc18c\ud55c\ub2e4\"\ub77c\uace0 \ub9d0\ud588\ub2e4. 2014\ub144 3\uc6d4 30\uc77c, \uc11c\uc544\ud504\ub9ac\uc544 \uad6d\uac00\uc758 \uc815\ubd80\ub4e4\uc774 \ubaa8\uc778 44\ud68c \uc815\uc0c1\ud68c\ub2f4\uc5d0\uc11c, \uc11c\uc544\ud504\ub9ac\uce74 \uacbd\uc81c \uacf5\ub3d9\uccb4\ub294 \uc804\uc5fc\ubcd1 \ubc1c\uc0dd\uc5d0 \ub300\ucc98\ud558\uae30 \uc704\ud574 25\ub9cc \ub2ec\ub7ec\ub97c \uc9c0\uc6d0\ud588\ub2e4. \uc2dc\uc5d0\ub77c\ub9ac\uc628\uc740 \uae30\ub2c8 \ub610\ub294 \ub77c\uc774\ubca0\ub9ac\uc544\uc5d0\uc11c \uc624\ub294 \ubaa8\ub4e0 \uc5ec\ud589\uac1d\ub4e4\uc5d0 \ub300\ud574 \uac74\uac15 \uc0c1\ud0dc\ub97c \ud655\uc778\ud558\uae30 \uc704\ud55c \uc5c4\uaca9\ud55c \uc2ec\uc0ac\ub97c \ud3ec\ud568\ud558\ub294 \"\uc801\uadf9\uc801 \uac10\uc2dc \uccb4\uacc4\"\uc758 \ud65c\uc131\ud654\ub97c \ud3ec\ud568\ud558\ub294 \uc784\uc2dc \uc870\uce58\ub97c \uc81c\uc815\ud588\ub2e4.", "answer": "\uc11c\uc544\ud504\ub9ac\uce74 \uacbd\uc81c \uacf5\ub3d9\uccb4", "sentence": "\uc11c\uc544\ud504\ub9ac\uce74 \uacbd\uc81c \uacf5\ub3d9\uccb4 (ECOWAS)\ub294 \uc804\uc5fc\ubcd1 \ud1f4\uce58\ub97c \uc704\ud55c \uad6d\uc81c \uc0ac\ud68c\uc758 \uc9c0\uc6d0\uc744 \ucd09\uad6c\ud558\ub294 \uc131\uba85\uc744 \ubc1c\ud45c\ud588\ub2e4.", "paragraph_sentence": "\uae30\ub2c8, \uc2dc\uc5d0\ub77c\ub9ac\uc628, \ub77c\uc774\ubca0\ub9ac\uc544\uc758 \uc815\ubd80 \uae30\uad6c\ub294 \uad6d\uac00 \ube44\uc0c1\uc0ac\ud0dc\ub97c \ubc1c\ub839\ud558\uace0 \uc5d0\ubcfc\ub77c \ubc14\uc774\ub7ec\uc2a4 \ub300\uc751 \uacc4\ud68d \ubc0f \uc2e4\uc2dc\uc5d0 \ub300\ud55c \uc900\ube44\uc5d0 \ub4e4\uc5b4\uac14\ub2e4. \uc11c\uc544\ud504\ub9ac\uce74 \uacbd\uc81c \uacf5\ub3d9\uccb4 (ECOWAS)\ub294 \uc804\uc5fc\ubcd1 \ud1f4\uce58\ub97c \uc704\ud55c \uad6d\uc81c \uc0ac\ud68c\uc758 \uc9c0\uc6d0\uc744 \ucd09\uad6c\ud558\ub294 \uc131\uba85\uc744 \ubc1c\ud45c\ud588\ub2e4. \"ECOWAS\uc758 \ubcf4\uc548 \ubc0f \uc911\uc7ac\uc704\uc6d0\ud68c\ub294 \uc5d0\ubcfc\ub77c \uc804\uc5fc\ubcd1 \uad00\ub9ac \uc9c0\uc6d0\uc744 \uc704\ud574 \uad6d\uc81c \uc0ac\ud68c\uc5d0\uac8c \ud638\uc18c\ud55c\ub2e4\"\ub77c\uace0 \ub9d0\ud588\ub2e4. 2014\ub144 3\uc6d4 30\uc77c, \uc11c\uc544\ud504\ub9ac\uc544 \uad6d\uac00\uc758 \uc815\ubd80\ub4e4\uc774 \ubaa8\uc778 44\ud68c \uc815\uc0c1\ud68c\ub2f4\uc5d0\uc11c, \uc11c\uc544\ud504\ub9ac\uce74 \uacbd\uc81c \uacf5\ub3d9\uccb4\ub294 \uc804\uc5fc\ubcd1 \ubc1c\uc0dd\uc5d0 \ub300\ucc98\ud558\uae30 \uc704\ud574 25\ub9cc \ub2ec\ub7ec\ub97c \uc9c0\uc6d0\ud588\ub2e4. \uc2dc\uc5d0\ub77c\ub9ac\uc628\uc740 \uae30\ub2c8 \ub610\ub294 \ub77c\uc774\ubca0\ub9ac\uc544\uc5d0\uc11c \uc624\ub294 \ubaa8\ub4e0 \uc5ec\ud589\uac1d\ub4e4\uc5d0 \ub300\ud574 \uac74\uac15 \uc0c1\ud0dc\ub97c \ud655\uc778\ud558\uae30 \uc704\ud55c \uc5c4\uaca9\ud55c \uc2ec\uc0ac\ub97c \ud3ec\ud568\ud558\ub294 \"\uc801\uadf9\uc801 \uac10\uc2dc \uccb4\uacc4\"\uc758 \ud65c\uc131\ud654\ub97c \ud3ec\ud568\ud558\ub294 \uc784\uc2dc \uc870\uce58\ub97c \uc81c\uc815\ud588\ub2e4.", "paragraph_answer": "\uae30\ub2c8, \uc2dc\uc5d0\ub77c\ub9ac\uc628, \ub77c\uc774\ubca0\ub9ac\uc544\uc758 \uc815\ubd80 \uae30\uad6c\ub294 \uad6d\uac00 \ube44\uc0c1\uc0ac\ud0dc\ub97c \ubc1c\ub839\ud558\uace0 \uc5d0\ubcfc\ub77c \ubc14\uc774\ub7ec\uc2a4 \ub300\uc751 \uacc4\ud68d \ubc0f \uc2e4\uc2dc\uc5d0 \ub300\ud55c \uc900\ube44\uc5d0 \ub4e4\uc5b4\uac14\ub2e4. \uc11c\uc544\ud504\ub9ac\uce74 \uacbd\uc81c \uacf5\ub3d9\uccb4 (ECOWAS)\ub294 \uc804\uc5fc\ubcd1 \ud1f4\uce58\ub97c \uc704\ud55c \uad6d\uc81c \uc0ac\ud68c\uc758 \uc9c0\uc6d0\uc744 \ucd09\uad6c\ud558\ub294 \uc131\uba85\uc744 \ubc1c\ud45c\ud588\ub2e4. \"ECOWAS\uc758 \ubcf4\uc548 \ubc0f \uc911\uc7ac\uc704\uc6d0\ud68c\ub294 \uc5d0\ubcfc\ub77c \uc804\uc5fc\ubcd1 \uad00\ub9ac \uc9c0\uc6d0\uc744 \uc704\ud574 \uad6d\uc81c \uc0ac\ud68c\uc5d0\uac8c \ud638\uc18c\ud55c\ub2e4\"\ub77c\uace0 \ub9d0\ud588\ub2e4. 2014\ub144 3\uc6d4 30\uc77c, \uc11c\uc544\ud504\ub9ac\uc544 \uad6d\uac00\uc758 \uc815\ubd80\ub4e4\uc774 \ubaa8\uc778 44\ud68c \uc815\uc0c1\ud68c\ub2f4\uc5d0\uc11c, \uc11c\uc544\ud504\ub9ac\uce74 \uacbd\uc81c \uacf5\ub3d9\uccb4\ub294 \uc804\uc5fc\ubcd1 \ubc1c\uc0dd\uc5d0 \ub300\ucc98\ud558\uae30 \uc704\ud574 25\ub9cc \ub2ec\ub7ec\ub97c \uc9c0\uc6d0\ud588\ub2e4. \uc2dc\uc5d0\ub77c\ub9ac\uc628\uc740 \uae30\ub2c8 \ub610\ub294 \ub77c\uc774\ubca0\ub9ac\uc544\uc5d0\uc11c \uc624\ub294 \ubaa8\ub4e0 \uc5ec\ud589\uac1d\ub4e4\uc5d0 \ub300\ud574 \uac74\uac15 \uc0c1\ud0dc\ub97c \ud655\uc778\ud558\uae30 \uc704\ud55c \uc5c4\uaca9\ud55c \uc2ec\uc0ac\ub97c \ud3ec\ud568\ud558\ub294 \"\uc801\uadf9\uc801 \uac10\uc2dc \uccb4\uacc4\"\uc758 \ud65c\uc131\ud654\ub97c \ud3ec\ud568\ud558\ub294 \uc784\uc2dc \uc870\uce58\ub97c \uc81c\uc815\ud588\ub2e4.", "sentence_answer": " \uc11c\uc544\ud504\ub9ac\uce74 \uacbd\uc81c \uacf5\ub3d9\uccb4 (ECOWAS)\ub294 \uc804\uc5fc\ubcd1 \ud1f4\uce58\ub97c \uc704\ud55c \uad6d\uc81c \uc0ac\ud68c\uc758 \uc9c0\uc6d0\uc744 \ucd09\uad6c\ud558\ub294 \uc131\uba85\uc744 \ubc1c\ud45c\ud588\ub2e4.", "paragraph_id": "6531765-3-0", "paragraph_question": "question: \uad6d\uc81c \uc0ac\ud68c\uc5d0 \uc5d0\ubcfc\ub77c \ubc14\uc774\ub7ec\uc2a4 \uc804\uc5fc\ubcd1 \ud1f4\uce58\ub97c \uc704\ud55c \uc9c0\uc6d0 \ucd09\uad6c \uc131\uba85\uc744 \ubc1c\ud45c\ud55c \ub2e8\uccb4\ub294?, context: \uae30\ub2c8, \uc2dc\uc5d0\ub77c\ub9ac\uc628, \ub77c\uc774\ubca0\ub9ac\uc544\uc758 \uc815\ubd80 \uae30\uad6c\ub294 \uad6d\uac00 \ube44\uc0c1\uc0ac\ud0dc\ub97c \ubc1c\ub839\ud558\uace0 \uc5d0\ubcfc\ub77c \ubc14\uc774\ub7ec\uc2a4 \ub300\uc751 \uacc4\ud68d \ubc0f \uc2e4\uc2dc\uc5d0 \ub300\ud55c \uc900\ube44\uc5d0 \ub4e4\uc5b4\uac14\ub2e4. \uc11c\uc544\ud504\ub9ac\uce74 \uacbd\uc81c \uacf5\ub3d9\uccb4 (ECOWAS)\ub294 \uc804\uc5fc\ubcd1 \ud1f4\uce58\ub97c \uc704\ud55c \uad6d\uc81c \uc0ac\ud68c\uc758 \uc9c0\uc6d0\uc744 \ucd09\uad6c\ud558\ub294 \uc131\uba85\uc744 \ubc1c\ud45c\ud588\ub2e4. \"ECOWAS\uc758 \ubcf4\uc548 \ubc0f \uc911\uc7ac\uc704\uc6d0\ud68c\ub294 \uc5d0\ubcfc\ub77c \uc804\uc5fc\ubcd1 \uad00\ub9ac \uc9c0\uc6d0\uc744 \uc704\ud574 \uad6d\uc81c \uc0ac\ud68c\uc5d0\uac8c \ud638\uc18c\ud55c\ub2e4\"\ub77c\uace0 \ub9d0\ud588\ub2e4. 2014\ub144 3\uc6d4 30\uc77c, \uc11c\uc544\ud504\ub9ac\uc544 \uad6d\uac00\uc758 \uc815\ubd80\ub4e4\uc774 \ubaa8\uc778 44\ud68c \uc815\uc0c1\ud68c\ub2f4\uc5d0\uc11c, \uc11c\uc544\ud504\ub9ac\uce74 \uacbd\uc81c \uacf5\ub3d9\uccb4\ub294 \uc804\uc5fc\ubcd1 \ubc1c\uc0dd\uc5d0 \ub300\ucc98\ud558\uae30 \uc704\ud574 25\ub9cc \ub2ec\ub7ec\ub97c \uc9c0\uc6d0\ud588\ub2e4. \uc2dc\uc5d0\ub77c\ub9ac\uc628\uc740 \uae30\ub2c8 \ub610\ub294 \ub77c\uc774\ubca0\ub9ac\uc544\uc5d0\uc11c \uc624\ub294 \ubaa8\ub4e0 \uc5ec\ud589\uac1d\ub4e4\uc5d0 \ub300\ud574 \uac74\uac15 \uc0c1\ud0dc\ub97c \ud655\uc778\ud558\uae30 \uc704\ud55c \uc5c4\uaca9\ud55c \uc2ec\uc0ac\ub97c \ud3ec\ud568\ud558\ub294 \"\uc801\uadf9\uc801 \uac10\uc2dc \uccb4\uacc4\"\uc758 \ud65c\uc131\ud654\ub97c \ud3ec\ud568\ud558\ub294 \uc784\uc2dc \uc870\uce58\ub97c \uc81c\uc815\ud588\ub2e4."} {"question": "\ub9ac\uc988 \uc704\ub354\uc2a4\ud47c\uacfc \uc9d0 \ud1a0\uc2a4\ub294 \uc5b4\ub5a4 \uc2dc\uc5d0\uc11c \uacb0\ud63c\ud588\ub294\uac00?", "paragraph": "2010\ub144 2\uc6d4 \ucd08 \uc704\ub354\uc2a4\ud47c\uc774 \uc9d0 \ud1a0\uc2a4\uc640 \ub370\uc774\ud2b8\ud558\uae30 \uc2dc\uc791\ud588\ub2e4\ub294 \ubcf4\ub3c4\uac00 \ub098\uc624\uae30 \uc2dc\uc791\ud588\ub2e4. \ub2f9\uc2dc \ud1a0\uc2a4\ub294 \ud06c\ub9ac\uc5d0\uc774\ud2f0\ube0c \uc544\ud2f0\uc2a4\ud2b8 \uc5d0\uc774\uc804\uc2dc\uc5d0\uc11c \uc5d0\uc774\uc804\ud2b8\ub85c \uc77c\ud558\uace0 \uc788\uc5c8\uc73c\uba70, \uc704\ub354\uc2a4\ud47c\uacfc \ud568\uaed8 \uc77c\ud558\uace0 \uc788\uc5c8\ub2e4. 2010\ub144 9\uc6d4 \uadf8\ub294 \uc601\ud654 \ud0e4\ub7f0\ud2b8 \ub2f4\ub2f9 \ub300\ud45c\uc9c1\uc744 \ub9e1\uac8c \ub41c\ub2e4. \uae00\ub798\uba38 \uc9c0 2011\ub144 1\uc6d4\ud310\uc5d0 \uac8c\uc7ac\ub41c \uc778\ud130\ubdf0\uc5d0\uc11c \uc704\ub354\uc2a4\ud47c\uc740 \ud1a0\uc2a4\uc640\uc758 \uad00\uacc4\uc5d0 \ub300\ud574 \uc5b8\uae09\ud55c\ub2e4. \"\uc81c\uac00 \ubc1c\uacac\ud55c \uac83 \uc911\uc5d0\uc11c \ud3b8\uc548\ud558\uace0 \uc88b\uc740 \uc810\uc73c\ub85c \ubc1b\uc544\ub4e4\uc774\uac8c \ub41c \uc810\uc740 \ub204\uad70\uac00\uac00 \ub098\uc774 \uacbd\ub825\uacfc \uc77c\uc744 \uc774\ud574\ud558\uc9c0\ub9cc \ud55c\ud3b8\uc73c\ub85c \ub0b4 \uc77c \uc790\uccb4\ub97c \ud558\uc9c0\ub294 \uc54a\ub294\ub2e4\ub294 \uc0ac\uc2e4\uc744 \uc54c\uac8c \ub41c\ub2e4\ub294 \ub370 \uc788\uc5b4\uc694. \uadf8\ub294 \uc815\ub9d0 \uba4b\uc788\ub294 \uc0ac\ub78c\uc774\uace0 \uba4b\uc9c0\ub2f5\ub2c8\ub2e4.\" \uc704\ub354\uc2a4\ud47c\uacfc \ud1a0\uc2a4\ub294 2010\ub144 12\uc6d4 28\uc77c \uc57d\ud63c\ud558\uc600\uace0 2011\ub144 3\uc6d4 26\uc77c \uadf8\ub140\uc758 \uac1c\uc778 \ubd80\ub3d9\uc0b0\uc774 \uc788\ub294 \uce98\ub9ac\ud3ec\ub2c8\uc544 \uc8fc, \uc624\ud558\uc774 \uc2dc\uc5d0\uc11c \uacb0\ud63c\ud558\uc600\ub2e4.", "answer": "\uc624\ud558\uc774 \uc2dc", "sentence": "\uc704\ub354\uc2a4\ud47c\uacfc \ud1a0\uc2a4\ub294 2010\ub144 12\uc6d4 28\uc77c \uc57d\ud63c\ud558\uc600\uace0 2011\ub144 3\uc6d4 26\uc77c \uadf8\ub140\uc758 \uac1c\uc778 \ubd80\ub3d9\uc0b0\uc774 \uc788\ub294 \uce98\ub9ac\ud3ec\ub2c8\uc544 \uc8fc, \uc624\ud558\uc774 \uc2dc \uc5d0\uc11c \uacb0\ud63c\ud558\uc600\ub2e4.", "paragraph_sentence": "2010\ub144 2\uc6d4 \ucd08 \uc704\ub354\uc2a4\ud47c\uc774 \uc9d0 \ud1a0\uc2a4\uc640 \ub370\uc774\ud2b8\ud558\uae30 \uc2dc\uc791\ud588\ub2e4\ub294 \ubcf4\ub3c4\uac00 \ub098\uc624\uae30 \uc2dc\uc791\ud588\ub2e4. \ub2f9\uc2dc \ud1a0\uc2a4\ub294 \ud06c\ub9ac\uc5d0\uc774\ud2f0\ube0c \uc544\ud2f0\uc2a4\ud2b8 \uc5d0\uc774\uc804\uc2dc\uc5d0\uc11c \uc5d0\uc774\uc804\ud2b8\ub85c \uc77c\ud558\uace0 \uc788\uc5c8\uc73c\uba70, \uc704\ub354\uc2a4\ud47c\uacfc \ud568\uaed8 \uc77c\ud558\uace0 \uc788\uc5c8\ub2e4. 2010\ub144 9\uc6d4 \uadf8\ub294 \uc601\ud654 \ud0e4\ub7f0\ud2b8 \ub2f4\ub2f9 \ub300\ud45c\uc9c1\uc744 \ub9e1\uac8c \ub41c\ub2e4. \uae00\ub798\uba38 \uc9c0 2011\ub144 1\uc6d4\ud310\uc5d0 \uac8c\uc7ac\ub41c \uc778\ud130\ubdf0\uc5d0\uc11c \uc704\ub354\uc2a4\ud47c\uc740 \ud1a0\uc2a4\uc640\uc758 \uad00\uacc4\uc5d0 \ub300\ud574 \uc5b8\uae09\ud55c\ub2e4. \"\uc81c\uac00 \ubc1c\uacac\ud55c \uac83 \uc911\uc5d0\uc11c \ud3b8\uc548\ud558\uace0 \uc88b\uc740 \uc810\uc73c\ub85c \ubc1b\uc544\ub4e4\uc774\uac8c \ub41c \uc810\uc740 \ub204\uad70\uac00\uac00 \ub098\uc774 \uacbd\ub825\uacfc \uc77c\uc744 \uc774\ud574\ud558\uc9c0\ub9cc \ud55c\ud3b8\uc73c\ub85c \ub0b4 \uc77c \uc790\uccb4\ub97c \ud558\uc9c0\ub294 \uc54a\ub294\ub2e4\ub294 \uc0ac\uc2e4\uc744 \uc54c\uac8c \ub41c\ub2e4\ub294 \ub370 \uc788\uc5b4\uc694. \uadf8\ub294 \uc815\ub9d0 \uba4b\uc788\ub294 \uc0ac\ub78c\uc774\uace0 \uba4b\uc9c0\ub2f5\ub2c8\ub2e4. \" \uc704\ub354\uc2a4\ud47c\uacfc \ud1a0\uc2a4\ub294 2010\ub144 12\uc6d4 28\uc77c \uc57d\ud63c\ud558\uc600\uace0 2011\ub144 3\uc6d4 26\uc77c \uadf8\ub140\uc758 \uac1c\uc778 \ubd80\ub3d9\uc0b0\uc774 \uc788\ub294 \uce98\ub9ac\ud3ec\ub2c8\uc544 \uc8fc, \uc624\ud558\uc774 \uc2dc \uc5d0\uc11c \uacb0\ud63c\ud558\uc600\ub2e4. ", "paragraph_answer": "2010\ub144 2\uc6d4 \ucd08 \uc704\ub354\uc2a4\ud47c\uc774 \uc9d0 \ud1a0\uc2a4\uc640 \ub370\uc774\ud2b8\ud558\uae30 \uc2dc\uc791\ud588\ub2e4\ub294 \ubcf4\ub3c4\uac00 \ub098\uc624\uae30 \uc2dc\uc791\ud588\ub2e4. \ub2f9\uc2dc \ud1a0\uc2a4\ub294 \ud06c\ub9ac\uc5d0\uc774\ud2f0\ube0c \uc544\ud2f0\uc2a4\ud2b8 \uc5d0\uc774\uc804\uc2dc\uc5d0\uc11c \uc5d0\uc774\uc804\ud2b8\ub85c \uc77c\ud558\uace0 \uc788\uc5c8\uc73c\uba70, \uc704\ub354\uc2a4\ud47c\uacfc \ud568\uaed8 \uc77c\ud558\uace0 \uc788\uc5c8\ub2e4. 2010\ub144 9\uc6d4 \uadf8\ub294 \uc601\ud654 \ud0e4\ub7f0\ud2b8 \ub2f4\ub2f9 \ub300\ud45c\uc9c1\uc744 \ub9e1\uac8c \ub41c\ub2e4. \uae00\ub798\uba38 \uc9c0 2011\ub144 1\uc6d4\ud310\uc5d0 \uac8c\uc7ac\ub41c \uc778\ud130\ubdf0\uc5d0\uc11c \uc704\ub354\uc2a4\ud47c\uc740 \ud1a0\uc2a4\uc640\uc758 \uad00\uacc4\uc5d0 \ub300\ud574 \uc5b8\uae09\ud55c\ub2e4. \"\uc81c\uac00 \ubc1c\uacac\ud55c \uac83 \uc911\uc5d0\uc11c \ud3b8\uc548\ud558\uace0 \uc88b\uc740 \uc810\uc73c\ub85c \ubc1b\uc544\ub4e4\uc774\uac8c \ub41c \uc810\uc740 \ub204\uad70\uac00\uac00 \ub098\uc774 \uacbd\ub825\uacfc \uc77c\uc744 \uc774\ud574\ud558\uc9c0\ub9cc \ud55c\ud3b8\uc73c\ub85c \ub0b4 \uc77c \uc790\uccb4\ub97c \ud558\uc9c0\ub294 \uc54a\ub294\ub2e4\ub294 \uc0ac\uc2e4\uc744 \uc54c\uac8c \ub41c\ub2e4\ub294 \ub370 \uc788\uc5b4\uc694. \uadf8\ub294 \uc815\ub9d0 \uba4b\uc788\ub294 \uc0ac\ub78c\uc774\uace0 \uba4b\uc9c0\ub2f5\ub2c8\ub2e4.\" \uc704\ub354\uc2a4\ud47c\uacfc \ud1a0\uc2a4\ub294 2010\ub144 12\uc6d4 28\uc77c \uc57d\ud63c\ud558\uc600\uace0 2011\ub144 3\uc6d4 26\uc77c \uadf8\ub140\uc758 \uac1c\uc778 \ubd80\ub3d9\uc0b0\uc774 \uc788\ub294 \uce98\ub9ac\ud3ec\ub2c8\uc544 \uc8fc, \uc624\ud558\uc774 \uc2dc \uc5d0\uc11c \uacb0\ud63c\ud558\uc600\ub2e4.", "sentence_answer": "\uc704\ub354\uc2a4\ud47c\uacfc \ud1a0\uc2a4\ub294 2010\ub144 12\uc6d4 28\uc77c \uc57d\ud63c\ud558\uc600\uace0 2011\ub144 3\uc6d4 26\uc77c \uadf8\ub140\uc758 \uac1c\uc778 \ubd80\ub3d9\uc0b0\uc774 \uc788\ub294 \uce98\ub9ac\ud3ec\ub2c8\uc544 \uc8fc, \uc624\ud558\uc774 \uc2dc \uc5d0\uc11c \uacb0\ud63c\ud558\uc600\ub2e4.", "paragraph_id": "6529086-13-0", "paragraph_question": "question: \ub9ac\uc988 \uc704\ub354\uc2a4\ud47c\uacfc \uc9d0 \ud1a0\uc2a4\ub294 \uc5b4\ub5a4 \uc2dc\uc5d0\uc11c \uacb0\ud63c\ud588\ub294\uac00?, context: 2010\ub144 2\uc6d4 \ucd08 \uc704\ub354\uc2a4\ud47c\uc774 \uc9d0 \ud1a0\uc2a4\uc640 \ub370\uc774\ud2b8\ud558\uae30 \uc2dc\uc791\ud588\ub2e4\ub294 \ubcf4\ub3c4\uac00 \ub098\uc624\uae30 \uc2dc\uc791\ud588\ub2e4. \ub2f9\uc2dc \ud1a0\uc2a4\ub294 \ud06c\ub9ac\uc5d0\uc774\ud2f0\ube0c \uc544\ud2f0\uc2a4\ud2b8 \uc5d0\uc774\uc804\uc2dc\uc5d0\uc11c \uc5d0\uc774\uc804\ud2b8\ub85c \uc77c\ud558\uace0 \uc788\uc5c8\uc73c\uba70, \uc704\ub354\uc2a4\ud47c\uacfc \ud568\uaed8 \uc77c\ud558\uace0 \uc788\uc5c8\ub2e4. 2010\ub144 9\uc6d4 \uadf8\ub294 \uc601\ud654 \ud0e4\ub7f0\ud2b8 \ub2f4\ub2f9 \ub300\ud45c\uc9c1\uc744 \ub9e1\uac8c \ub41c\ub2e4. \uae00\ub798\uba38 \uc9c0 2011\ub144 1\uc6d4\ud310\uc5d0 \uac8c\uc7ac\ub41c \uc778\ud130\ubdf0\uc5d0\uc11c \uc704\ub354\uc2a4\ud47c\uc740 \ud1a0\uc2a4\uc640\uc758 \uad00\uacc4\uc5d0 \ub300\ud574 \uc5b8\uae09\ud55c\ub2e4. \"\uc81c\uac00 \ubc1c\uacac\ud55c \uac83 \uc911\uc5d0\uc11c \ud3b8\uc548\ud558\uace0 \uc88b\uc740 \uc810\uc73c\ub85c \ubc1b\uc544\ub4e4\uc774\uac8c \ub41c \uc810\uc740 \ub204\uad70\uac00\uac00 \ub098\uc774 \uacbd\ub825\uacfc \uc77c\uc744 \uc774\ud574\ud558\uc9c0\ub9cc \ud55c\ud3b8\uc73c\ub85c \ub0b4 \uc77c \uc790\uccb4\ub97c \ud558\uc9c0\ub294 \uc54a\ub294\ub2e4\ub294 \uc0ac\uc2e4\uc744 \uc54c\uac8c \ub41c\ub2e4\ub294 \ub370 \uc788\uc5b4\uc694. \uadf8\ub294 \uc815\ub9d0 \uba4b\uc788\ub294 \uc0ac\ub78c\uc774\uace0 \uba4b\uc9c0\ub2f5\ub2c8\ub2e4.\" \uc704\ub354\uc2a4\ud47c\uacfc \ud1a0\uc2a4\ub294 2010\ub144 12\uc6d4 28\uc77c \uc57d\ud63c\ud558\uc600\uace0 2011\ub144 3\uc6d4 26\uc77c \uadf8\ub140\uc758 \uac1c\uc778 \ubd80\ub3d9\uc0b0\uc774 \uc788\ub294 \uce98\ub9ac\ud3ec\ub2c8\uc544 \uc8fc, \uc624\ud558\uc774 \uc2dc\uc5d0\uc11c \uacb0\ud63c\ud558\uc600\ub2e4."} {"question": "\uc218\ub2e8\uc758 \ucd1d \ub178\ub3d9\uc778\uad6c \uc911 \ub18d\uc5c5 \ub610\ub294 \ubaa9\ucd95\uc5c5\uc5d0 \uc885\uc0ac\ud558\uace0 \uc788\ub294 \ube44\uc728\uc740 \uc5bc\ub9c8\uc778\uac00?", "paragraph": "1996-2005\ub144 \uae30\uac04 \ub3d9\uc548 \uc5f0\ud3c9\uade0 6.3%\uc758 \uacbd\uc81c \uc131\uc7a5\uc744 \uae30\ub85d\ud558\uc600\ub2e4. \ucd1d\ub178\ub3d9\uc778\uad6c\uc758 80%\uac00 \ub18d\uc5c5 \ub610\ub294 \ubaa9\ucd95\uc5c5\uc5d0 \uc885\uc0ac\ud558\uace0 \uc788\ub2e4. \uacbd\uc9c0 \uba74\uc801\uc740 800\ub9cc\u33ca\ub85c \uc8fc\ub85c \ub098\uc77c\uac15 \uc720\uc5ed\uc5d0 \ubd84\ud3ec\ud55c\ub2e4. \uc8fc\uc694 \ub18d\uc0b0\ubb3c\uc740 \uba74\ud654\uc778\ub370, \uc8fc\ub85c \uc601\uad6d\uc778\uc774 \uac74\uc124\ud55c \uc13c\ub108\ub310\uc5d0 \uc758\ud574 \uad00\uac1c(\u704c\u6f11)\ub41c \ucf00\ub514\ub77c \uc9c0\uad6c\uc5d0\uc11c \uc7ac\ubc30\ub41c\ub2e4. \uba74\ud654 \uc774\uc678\uc758 \uacbd\uc791\ubb3c\ub85c\ub294 \uc544\ub77c\ube44\uc544\uace0\ubb34, \uae68, \ub099\ud654\uc0dd, \uc0ac\ud0d5\uc218\uc218, \uc870, \ubc00 \ub4f1\uc774 \uc788\ub2e4. \uacf5\uc5c5\uc5d0 \uc788\uc5b4\uc11c\ub294 \ub3c5\ub9bd \ud6c4 \uc12c\uc720, \uc2dd\ud488, \uc81c\ub2f9, \uc81c\uc9c0, \uc815\uc720, \uc2dc\uba58\ud2b8 \ub4f1\uc758 \uacf5\uc7a5\uc774 \uac74\uc124\ub418\uc5c8\ub2e4. \ud604\uc7ac \uc218\ub2e8\uc5d0\uc11c \uacf5\uc5c5\ud654\uc758 \uc7a5\uc560\uac00 \ub418\uace0 \uc788\ub294 \uac83\uc740 \uc790\ubcf8\uc758 \ubd80\uc871, \uad50\ud1b5\uc2dc\uc124\uc758 \ubbf8\ube44, \uae30\uc220\uc758 \ud6c4\uc9c4\uc131 \ub4f1\uc774\ub2e4. \uc9c0\ud558\uc790\uc6d0\uc73c\ub85c\ub294 \ucca0\uad11\uc11d\u00b7\ub9dd\uac04\u00b7\ub3d9 \ub4f1\uc774 \uc788\ub2e4. 2002\ub144 \uc218\ucd9c 17\uc5b5 1,770\ub9cc \ub2ec\ub7ec, \uc218\uc785 19\uc5b5 1,480\ub9cc \ub2ec\ub7ec\ub97c \uae30\ub85d\ud588\ub2e4. \uc8fc\uc694 \uc218\ucd9c\ud488\uc740 \uba74\ud654\uc640 \ub099\ud654\uc0dd, \uc544\ub77c\ube44\uc544\uace0\ubb34 \ub4f1\uc774\ub2e4. \ub300\ud3ed\uc801\uc778 \ubb34\uc5ed\uc801\uc790\ub294 \uc678\uad6d\uc758 \uac01\uc885 \uc6d0\uc870\uc640 \uc544\ub78d\uc0b0\uc720\uad6d\uc5d0 \ucde8\uc5c5\ud55c \ub178\ub3d9\uc790\uc758 \ubcf8\uad6d \uc1a1\uae08\uc73c\ub85c \uba54\uc6b4\ub2e4.", "answer": "80%", "sentence": "\ucd1d\ub178\ub3d9\uc778\uad6c\uc758 80% \uac00 \ub18d\uc5c5 \ub610\ub294 \ubaa9\ucd95\uc5c5\uc5d0 \uc885\uc0ac\ud558\uace0 \uc788\ub2e4.", "paragraph_sentence": "1996-2005\ub144 \uae30\uac04 \ub3d9\uc548 \uc5f0\ud3c9\uade0 6.3%\uc758 \uacbd\uc81c \uc131\uc7a5\uc744 \uae30\ub85d\ud558\uc600\ub2e4. \ucd1d\ub178\ub3d9\uc778\uad6c\uc758 80% \uac00 \ub18d\uc5c5 \ub610\ub294 \ubaa9\ucd95\uc5c5\uc5d0 \uc885\uc0ac\ud558\uace0 \uc788\ub2e4. \uacbd\uc9c0 \uba74\uc801\uc740 800\ub9cc\u33ca\ub85c \uc8fc\ub85c \ub098\uc77c\uac15 \uc720\uc5ed\uc5d0 \ubd84\ud3ec\ud55c\ub2e4. \uc8fc\uc694 \ub18d\uc0b0\ubb3c\uc740 \uba74\ud654\uc778\ub370, \uc8fc\ub85c \uc601\uad6d\uc778\uc774 \uac74\uc124\ud55c \uc13c\ub108\ub310\uc5d0 \uc758\ud574 \uad00\uac1c(\u704c\u6f11)\ub41c \ucf00\ub514\ub77c \uc9c0\uad6c\uc5d0\uc11c \uc7ac\ubc30\ub41c\ub2e4. \uba74\ud654 \uc774\uc678\uc758 \uacbd\uc791\ubb3c\ub85c\ub294 \uc544\ub77c\ube44\uc544\uace0\ubb34, \uae68, \ub099\ud654\uc0dd, \uc0ac\ud0d5\uc218\uc218, \uc870, \ubc00 \ub4f1\uc774 \uc788\ub2e4. \uacf5\uc5c5\uc5d0 \uc788\uc5b4\uc11c\ub294 \ub3c5\ub9bd \ud6c4 \uc12c\uc720, \uc2dd\ud488, \uc81c\ub2f9, \uc81c\uc9c0, \uc815\uc720, \uc2dc\uba58\ud2b8 \ub4f1\uc758 \uacf5\uc7a5\uc774 \uac74\uc124\ub418\uc5c8\ub2e4. \ud604\uc7ac \uc218\ub2e8\uc5d0\uc11c \uacf5\uc5c5\ud654\uc758 \uc7a5\uc560\uac00 \ub418\uace0 \uc788\ub294 \uac83\uc740 \uc790\ubcf8\uc758 \ubd80\uc871, \uad50\ud1b5\uc2dc\uc124\uc758 \ubbf8\ube44, \uae30\uc220\uc758 \ud6c4\uc9c4\uc131 \ub4f1\uc774\ub2e4. \uc9c0\ud558\uc790\uc6d0\uc73c\ub85c\ub294 \ucca0\uad11\uc11d\u00b7\ub9dd\uac04\u00b7\ub3d9 \ub4f1\uc774 \uc788\ub2e4. 2002\ub144 \uc218\ucd9c 17\uc5b5 1,770\ub9cc \ub2ec\ub7ec, \uc218\uc785 19\uc5b5 1,480\ub9cc \ub2ec\ub7ec\ub97c \uae30\ub85d\ud588\ub2e4. \uc8fc\uc694 \uc218\ucd9c\ud488\uc740 \uba74\ud654\uc640 \ub099\ud654\uc0dd, \uc544\ub77c\ube44\uc544\uace0\ubb34 \ub4f1\uc774\ub2e4. \ub300\ud3ed\uc801\uc778 \ubb34\uc5ed\uc801\uc790\ub294 \uc678\uad6d\uc758 \uac01\uc885 \uc6d0\uc870\uc640 \uc544\ub78d\uc0b0\uc720\uad6d\uc5d0 \ucde8\uc5c5\ud55c \ub178\ub3d9\uc790\uc758 \ubcf8\uad6d \uc1a1\uae08\uc73c\ub85c \uba54\uc6b4\ub2e4.", "paragraph_answer": "1996-2005\ub144 \uae30\uac04 \ub3d9\uc548 \uc5f0\ud3c9\uade0 6.3%\uc758 \uacbd\uc81c \uc131\uc7a5\uc744 \uae30\ub85d\ud558\uc600\ub2e4. \ucd1d\ub178\ub3d9\uc778\uad6c\uc758 80% \uac00 \ub18d\uc5c5 \ub610\ub294 \ubaa9\ucd95\uc5c5\uc5d0 \uc885\uc0ac\ud558\uace0 \uc788\ub2e4. \uacbd\uc9c0 \uba74\uc801\uc740 800\ub9cc\u33ca\ub85c \uc8fc\ub85c \ub098\uc77c\uac15 \uc720\uc5ed\uc5d0 \ubd84\ud3ec\ud55c\ub2e4. \uc8fc\uc694 \ub18d\uc0b0\ubb3c\uc740 \uba74\ud654\uc778\ub370, \uc8fc\ub85c \uc601\uad6d\uc778\uc774 \uac74\uc124\ud55c \uc13c\ub108\ub310\uc5d0 \uc758\ud574 \uad00\uac1c(\u704c\u6f11)\ub41c \ucf00\ub514\ub77c \uc9c0\uad6c\uc5d0\uc11c \uc7ac\ubc30\ub41c\ub2e4. \uba74\ud654 \uc774\uc678\uc758 \uacbd\uc791\ubb3c\ub85c\ub294 \uc544\ub77c\ube44\uc544\uace0\ubb34, \uae68, \ub099\ud654\uc0dd, \uc0ac\ud0d5\uc218\uc218, \uc870, \ubc00 \ub4f1\uc774 \uc788\ub2e4. \uacf5\uc5c5\uc5d0 \uc788\uc5b4\uc11c\ub294 \ub3c5\ub9bd \ud6c4 \uc12c\uc720, \uc2dd\ud488, \uc81c\ub2f9, \uc81c\uc9c0, \uc815\uc720, \uc2dc\uba58\ud2b8 \ub4f1\uc758 \uacf5\uc7a5\uc774 \uac74\uc124\ub418\uc5c8\ub2e4. \ud604\uc7ac \uc218\ub2e8\uc5d0\uc11c \uacf5\uc5c5\ud654\uc758 \uc7a5\uc560\uac00 \ub418\uace0 \uc788\ub294 \uac83\uc740 \uc790\ubcf8\uc758 \ubd80\uc871, \uad50\ud1b5\uc2dc\uc124\uc758 \ubbf8\ube44, \uae30\uc220\uc758 \ud6c4\uc9c4\uc131 \ub4f1\uc774\ub2e4. \uc9c0\ud558\uc790\uc6d0\uc73c\ub85c\ub294 \ucca0\uad11\uc11d\u00b7\ub9dd\uac04\u00b7\ub3d9 \ub4f1\uc774 \uc788\ub2e4. 2002\ub144 \uc218\ucd9c 17\uc5b5 1,770\ub9cc \ub2ec\ub7ec, \uc218\uc785 19\uc5b5 1,480\ub9cc \ub2ec\ub7ec\ub97c \uae30\ub85d\ud588\ub2e4. \uc8fc\uc694 \uc218\ucd9c\ud488\uc740 \uba74\ud654\uc640 \ub099\ud654\uc0dd, \uc544\ub77c\ube44\uc544\uace0\ubb34 \ub4f1\uc774\ub2e4. \ub300\ud3ed\uc801\uc778 \ubb34\uc5ed\uc801\uc790\ub294 \uc678\uad6d\uc758 \uac01\uc885 \uc6d0\uc870\uc640 \uc544\ub78d\uc0b0\uc720\uad6d\uc5d0 \ucde8\uc5c5\ud55c \ub178\ub3d9\uc790\uc758 \ubcf8\uad6d \uc1a1\uae08\uc73c\ub85c \uba54\uc6b4\ub2e4.", "sentence_answer": "\ucd1d\ub178\ub3d9\uc778\uad6c\uc758 80% \uac00 \ub18d\uc5c5 \ub610\ub294 \ubaa9\ucd95\uc5c5\uc5d0 \uc885\uc0ac\ud558\uace0 \uc788\ub2e4.", "paragraph_id": "6532482-1-2", "paragraph_question": "question: \uc218\ub2e8\uc758 \ucd1d \ub178\ub3d9\uc778\uad6c \uc911 \ub18d\uc5c5 \ub610\ub294 \ubaa9\ucd95\uc5c5\uc5d0 \uc885\uc0ac\ud558\uace0 \uc788\ub294 \ube44\uc728\uc740 \uc5bc\ub9c8\uc778\uac00?, context: 1996-2005\ub144 \uae30\uac04 \ub3d9\uc548 \uc5f0\ud3c9\uade0 6.3%\uc758 \uacbd\uc81c \uc131\uc7a5\uc744 \uae30\ub85d\ud558\uc600\ub2e4. \ucd1d\ub178\ub3d9\uc778\uad6c\uc758 80%\uac00 \ub18d\uc5c5 \ub610\ub294 \ubaa9\ucd95\uc5c5\uc5d0 \uc885\uc0ac\ud558\uace0 \uc788\ub2e4. \uacbd\uc9c0 \uba74\uc801\uc740 800\ub9cc\u33ca\ub85c \uc8fc\ub85c \ub098\uc77c\uac15 \uc720\uc5ed\uc5d0 \ubd84\ud3ec\ud55c\ub2e4. \uc8fc\uc694 \ub18d\uc0b0\ubb3c\uc740 \uba74\ud654\uc778\ub370, \uc8fc\ub85c \uc601\uad6d\uc778\uc774 \uac74\uc124\ud55c \uc13c\ub108\ub310\uc5d0 \uc758\ud574 \uad00\uac1c(\u704c\u6f11)\ub41c \ucf00\ub514\ub77c \uc9c0\uad6c\uc5d0\uc11c \uc7ac\ubc30\ub41c\ub2e4. \uba74\ud654 \uc774\uc678\uc758 \uacbd\uc791\ubb3c\ub85c\ub294 \uc544\ub77c\ube44\uc544\uace0\ubb34, \uae68, \ub099\ud654\uc0dd, \uc0ac\ud0d5\uc218\uc218, \uc870, \ubc00 \ub4f1\uc774 \uc788\ub2e4. \uacf5\uc5c5\uc5d0 \uc788\uc5b4\uc11c\ub294 \ub3c5\ub9bd \ud6c4 \uc12c\uc720, \uc2dd\ud488, \uc81c\ub2f9, \uc81c\uc9c0, \uc815\uc720, \uc2dc\uba58\ud2b8 \ub4f1\uc758 \uacf5\uc7a5\uc774 \uac74\uc124\ub418\uc5c8\ub2e4. \ud604\uc7ac \uc218\ub2e8\uc5d0\uc11c \uacf5\uc5c5\ud654\uc758 \uc7a5\uc560\uac00 \ub418\uace0 \uc788\ub294 \uac83\uc740 \uc790\ubcf8\uc758 \ubd80\uc871, \uad50\ud1b5\uc2dc\uc124\uc758 \ubbf8\ube44, \uae30\uc220\uc758 \ud6c4\uc9c4\uc131 \ub4f1\uc774\ub2e4. \uc9c0\ud558\uc790\uc6d0\uc73c\ub85c\ub294 \ucca0\uad11\uc11d\u00b7\ub9dd\uac04\u00b7\ub3d9 \ub4f1\uc774 \uc788\ub2e4. 2002\ub144 \uc218\ucd9c 17\uc5b5 1,770\ub9cc \ub2ec\ub7ec, \uc218\uc785 19\uc5b5 1,480\ub9cc \ub2ec\ub7ec\ub97c \uae30\ub85d\ud588\ub2e4. \uc8fc\uc694 \uc218\ucd9c\ud488\uc740 \uba74\ud654\uc640 \ub099\ud654\uc0dd, \uc544\ub77c\ube44\uc544\uace0\ubb34 \ub4f1\uc774\ub2e4. \ub300\ud3ed\uc801\uc778 \ubb34\uc5ed\uc801\uc790\ub294 \uc678\uad6d\uc758 \uac01\uc885 \uc6d0\uc870\uc640 \uc544\ub78d\uc0b0\uc720\uad6d\uc5d0 \ucde8\uc5c5\ud55c \ub178\ub3d9\uc790\uc758 \ubcf8\uad6d \uc1a1\uae08\uc73c\ub85c \uba54\uc6b4\ub2e4."} {"question": "\uc5ed\uc0ac \uc758\uc2dd\uc5d0 \uc758\ubb38\uc774 \uc81c\uae30\ub418\uc790 2007\ub144 5.16\uc744 \ubb34\uc5c7\uc774\ub77c\uace0 \ud558\uc600\ub294\uac00?", "paragraph": "1990\ub144 \uc7a1\uc9c0 \uc778\ud130\ubdf0\uc5d0\uc11c \ubc15\uadfc\ud61c\ub294 5\u00b716\uacfc 4\u00b719 \uc815\uc2e0\uc744 \uacc4\uc2b9\ud558\ub294 \uac83\uc774 \ubc15 \uc804 \ub300\ud1b5\ub839\uc758 \ub73b\uc744 \uacc4\uc2b9\ud558\ub294 \uac83\uc774\ub77c\uace0 \ud558\uc600\ub2e4. \uc5ed\uc0ac \uc758\uc2dd\uc5d0 \uc758\ubb38\uc774 \uc81c\uae30\ub418\uc790 2007\ub144 5.16\uc744 \uad6c\uad6d\uc758 \ud601\uba85\uc774\ub77c\uace0 \ubcf8\ub2e4\ub294 \uc790\uc2e0\uc758 \uc0dd\uac01\uc744 \ubc1d\ud614\ub2e4. 2007\ub144 7\uc6d4\uc758 \ud55c\ub098\ub77c\ub2f9 \ub300\uc120\ud6c4\ubcf4 \uac80\uc99d \uccad\ubb38\ud68c\uc5d0\uc11c \uadf8\ub294 '\ubc15 \uc804 \ub300\ud1b5\ub839 \uc720\uc9c0 \uacc4\uc2b9' \uc5ed\uc0ac\uc758\uc2dd\uc5d0 \ub300\ud55c \uc9c8\ubb38\uc744 \ubc1b\uc558\ub2e4. \uadf8\ub294 \"5\u00b716\uc740 \uad6c\uad6d(\u6551\u570b)\uc758 \ud601\uba85\uc774\uc5c8\ub2e4\uace0 \uc0dd\uac01\ud558\uace0 \uc788\ub2e4. \uadf8\ub54c \uc0c1\ud669\uc774 \ub108\ubb34\ub098 \ud63c\ub780\uc2a4\ub7ec\uc6e0\uace0 \ub0a8\ubd81\ub300\uce58 \uc0c1\ud669\uc5d0\uc11c \uc798\ubabb\ud558\uba74 \ubd81\ud55c\uc5d0 \ud761\uc218\ub420 \uc218\ub3c4 \uc788\ub294 \uadf8\ub7f0 \uc0c1\ud669\uc774\uc5c8\ub2e4. \ud601\uba85 \uacf5\uc57d\uc5d0\ub3c4 \uae30\uc544 \uc120\uc0c1\uc5d0 \ud5e4\ub9e4\ub294 \uad6d\ubbfc\uc744 \uad6c\uc81c\ud55c\ub2e4\ub294 \uc598\uae30\uac00 \ub098\uc628\ub2e4.\"\ub294 \uac83\uc774\ub2e4. \ub2e4\ub9cc 10\uc6d4 \uc720\uc2e0\uc5d0 \ub300\ud574\uc11c\ub294 \ud310\ub2e8\uc720\ubcf4\ub97c \ub0b4\ub838\ub2e4. 5.16 \ub4a4\uc758 \uc720\uc2e0\uc2dc\ub300\uc5d0 \ub300\ud574\uc11c\ub294 \uc5b4\ub5bb\uac8c \ud3c9\uac00\ud558\ub098 \ub294 \uc9c8\ubb38\uc5d0 \"\uc5ed\uc0ac\uc5d0 \ud310\ub2e8\uc744 \ub9e1\uaca8\uc57c \ud55c\ub2e4\ub294 \uc785\uc7a5\uc744 \uac16\uace0 \uc788\ub2e4. \ub2e4\ub9cc \uc720\uc2e0\uc2dc\ub300\uc5d0 \ubbfc\uc8fc\ud654 \uc6b4\ub3d9\uc744 \ud558\uba74\uc11c \ud5cc\uc2e0\ud558\uace0 \uace0\ud1b5\ubc1b\uc740 \ubd84\ub4e4\uc5d0 \ub300\ud574\uc11c\ub294 \uc9c4\uc2ec\uc73c\ub85c \ud56d\uc0c1 \uc8c4\uc1a1\ud558\uac8c \uc0dd\uac01\ud55c\ub2e4.\"\uace0 \ub2f5\ud558\uc600\ub2e4. \uc774 \ub54c\ubb38\uc5d0 \uc57c\ub2f9 \uc778\uc0ac\ub4e4\ub85c\ubd80\ud130 \ube44\ub09c\uc744 \ubc1b\uae30\ub3c4 \ud558\uc600\ub2e4.", "answer": "\uad6c\uad6d\uc758 \ud601\uba85", "sentence": "\uc5ed\uc0ac \uc758\uc2dd\uc5d0 \uc758\ubb38\uc774 \uc81c\uae30\ub418\uc790 2007\ub144 5.16\uc744 \uad6c\uad6d\uc758 \ud601\uba85 \uc774\ub77c\uace0 \ubcf8\ub2e4\ub294 \uc790\uc2e0\uc758 \uc0dd\uac01\uc744 \ubc1d\ud614\ub2e4.", "paragraph_sentence": "1990\ub144 \uc7a1\uc9c0 \uc778\ud130\ubdf0\uc5d0\uc11c \ubc15\uadfc\ud61c\ub294 5\u00b716\uacfc 4\u00b719 \uc815\uc2e0\uc744 \uacc4\uc2b9\ud558\ub294 \uac83\uc774 \ubc15 \uc804 \ub300\ud1b5\ub839\uc758 \ub73b\uc744 \uacc4\uc2b9\ud558\ub294 \uac83\uc774\ub77c\uace0 \ud558\uc600\ub2e4. \uc5ed\uc0ac \uc758\uc2dd\uc5d0 \uc758\ubb38\uc774 \uc81c\uae30\ub418\uc790 2007\ub144 5.16\uc744 \uad6c\uad6d\uc758 \ud601\uba85 \uc774\ub77c\uace0 \ubcf8\ub2e4\ub294 \uc790\uc2e0\uc758 \uc0dd\uac01\uc744 \ubc1d\ud614\ub2e4. 2007\ub144 7\uc6d4\uc758 \ud55c\ub098\ub77c\ub2f9 \ub300\uc120\ud6c4\ubcf4 \uac80\uc99d \uccad\ubb38\ud68c\uc5d0\uc11c \uadf8\ub294 '\ubc15 \uc804 \ub300\ud1b5\ub839 \uc720\uc9c0 \uacc4\uc2b9' \uc5ed\uc0ac\uc758\uc2dd\uc5d0 \ub300\ud55c \uc9c8\ubb38\uc744 \ubc1b\uc558\ub2e4. \uadf8\ub294 \"5\u00b716\uc740 \uad6c\uad6d(\u6551\u570b)\uc758 \ud601\uba85\uc774\uc5c8\ub2e4\uace0 \uc0dd\uac01\ud558\uace0 \uc788\ub2e4. \uadf8\ub54c \uc0c1\ud669\uc774 \ub108\ubb34\ub098 \ud63c\ub780\uc2a4\ub7ec\uc6e0\uace0 \ub0a8\ubd81\ub300\uce58 \uc0c1\ud669\uc5d0\uc11c \uc798\ubabb\ud558\uba74 \ubd81\ud55c\uc5d0 \ud761\uc218\ub420 \uc218\ub3c4 \uc788\ub294 \uadf8\ub7f0 \uc0c1\ud669\uc774\uc5c8\ub2e4. \ud601\uba85 \uacf5\uc57d\uc5d0\ub3c4 \uae30\uc544 \uc120\uc0c1\uc5d0 \ud5e4\ub9e4\ub294 \uad6d\ubbfc\uc744 \uad6c\uc81c\ud55c\ub2e4\ub294 \uc598\uae30\uac00 \ub098\uc628\ub2e4. \"\ub294 \uac83\uc774\ub2e4. \ub2e4\ub9cc 10\uc6d4 \uc720\uc2e0\uc5d0 \ub300\ud574\uc11c\ub294 \ud310\ub2e8\uc720\ubcf4\ub97c \ub0b4\ub838\ub2e4. 5.16 \ub4a4\uc758 \uc720\uc2e0\uc2dc\ub300\uc5d0 \ub300\ud574\uc11c\ub294 \uc5b4\ub5bb\uac8c \ud3c9\uac00\ud558\ub098 \ub294 \uc9c8\ubb38\uc5d0 \"\uc5ed\uc0ac\uc5d0 \ud310\ub2e8\uc744 \ub9e1\uaca8\uc57c \ud55c\ub2e4\ub294 \uc785\uc7a5\uc744 \uac16\uace0 \uc788\ub2e4. \ub2e4\ub9cc \uc720\uc2e0\uc2dc\ub300\uc5d0 \ubbfc\uc8fc\ud654 \uc6b4\ub3d9\uc744 \ud558\uba74\uc11c \ud5cc\uc2e0\ud558\uace0 \uace0\ud1b5\ubc1b\uc740 \ubd84\ub4e4\uc5d0 \ub300\ud574\uc11c\ub294 \uc9c4\uc2ec\uc73c\ub85c \ud56d\uc0c1 \uc8c4\uc1a1\ud558\uac8c \uc0dd\uac01\ud55c\ub2e4. \"\uace0 \ub2f5\ud558\uc600\ub2e4. \uc774 \ub54c\ubb38\uc5d0 \uc57c\ub2f9 \uc778\uc0ac\ub4e4\ub85c\ubd80\ud130 \ube44\ub09c\uc744 \ubc1b\uae30\ub3c4 \ud558\uc600\ub2e4.", "paragraph_answer": "1990\ub144 \uc7a1\uc9c0 \uc778\ud130\ubdf0\uc5d0\uc11c \ubc15\uadfc\ud61c\ub294 5\u00b716\uacfc 4\u00b719 \uc815\uc2e0\uc744 \uacc4\uc2b9\ud558\ub294 \uac83\uc774 \ubc15 \uc804 \ub300\ud1b5\ub839\uc758 \ub73b\uc744 \uacc4\uc2b9\ud558\ub294 \uac83\uc774\ub77c\uace0 \ud558\uc600\ub2e4. \uc5ed\uc0ac \uc758\uc2dd\uc5d0 \uc758\ubb38\uc774 \uc81c\uae30\ub418\uc790 2007\ub144 5.16\uc744 \uad6c\uad6d\uc758 \ud601\uba85 \uc774\ub77c\uace0 \ubcf8\ub2e4\ub294 \uc790\uc2e0\uc758 \uc0dd\uac01\uc744 \ubc1d\ud614\ub2e4. 2007\ub144 7\uc6d4\uc758 \ud55c\ub098\ub77c\ub2f9 \ub300\uc120\ud6c4\ubcf4 \uac80\uc99d \uccad\ubb38\ud68c\uc5d0\uc11c \uadf8\ub294 '\ubc15 \uc804 \ub300\ud1b5\ub839 \uc720\uc9c0 \uacc4\uc2b9' \uc5ed\uc0ac\uc758\uc2dd\uc5d0 \ub300\ud55c \uc9c8\ubb38\uc744 \ubc1b\uc558\ub2e4. \uadf8\ub294 \"5\u00b716\uc740 \uad6c\uad6d(\u6551\u570b)\uc758 \ud601\uba85\uc774\uc5c8\ub2e4\uace0 \uc0dd\uac01\ud558\uace0 \uc788\ub2e4. \uadf8\ub54c \uc0c1\ud669\uc774 \ub108\ubb34\ub098 \ud63c\ub780\uc2a4\ub7ec\uc6e0\uace0 \ub0a8\ubd81\ub300\uce58 \uc0c1\ud669\uc5d0\uc11c \uc798\ubabb\ud558\uba74 \ubd81\ud55c\uc5d0 \ud761\uc218\ub420 \uc218\ub3c4 \uc788\ub294 \uadf8\ub7f0 \uc0c1\ud669\uc774\uc5c8\ub2e4. \ud601\uba85 \uacf5\uc57d\uc5d0\ub3c4 \uae30\uc544 \uc120\uc0c1\uc5d0 \ud5e4\ub9e4\ub294 \uad6d\ubbfc\uc744 \uad6c\uc81c\ud55c\ub2e4\ub294 \uc598\uae30\uac00 \ub098\uc628\ub2e4.\"\ub294 \uac83\uc774\ub2e4. \ub2e4\ub9cc 10\uc6d4 \uc720\uc2e0\uc5d0 \ub300\ud574\uc11c\ub294 \ud310\ub2e8\uc720\ubcf4\ub97c \ub0b4\ub838\ub2e4. 5.16 \ub4a4\uc758 \uc720\uc2e0\uc2dc\ub300\uc5d0 \ub300\ud574\uc11c\ub294 \uc5b4\ub5bb\uac8c \ud3c9\uac00\ud558\ub098 \ub294 \uc9c8\ubb38\uc5d0 \"\uc5ed\uc0ac\uc5d0 \ud310\ub2e8\uc744 \ub9e1\uaca8\uc57c \ud55c\ub2e4\ub294 \uc785\uc7a5\uc744 \uac16\uace0 \uc788\ub2e4. \ub2e4\ub9cc \uc720\uc2e0\uc2dc\ub300\uc5d0 \ubbfc\uc8fc\ud654 \uc6b4\ub3d9\uc744 \ud558\uba74\uc11c \ud5cc\uc2e0\ud558\uace0 \uace0\ud1b5\ubc1b\uc740 \ubd84\ub4e4\uc5d0 \ub300\ud574\uc11c\ub294 \uc9c4\uc2ec\uc73c\ub85c \ud56d\uc0c1 \uc8c4\uc1a1\ud558\uac8c \uc0dd\uac01\ud55c\ub2e4.\"\uace0 \ub2f5\ud558\uc600\ub2e4. \uc774 \ub54c\ubb38\uc5d0 \uc57c\ub2f9 \uc778\uc0ac\ub4e4\ub85c\ubd80\ud130 \ube44\ub09c\uc744 \ubc1b\uae30\ub3c4 \ud558\uc600\ub2e4.", "sentence_answer": "\uc5ed\uc0ac \uc758\uc2dd\uc5d0 \uc758\ubb38\uc774 \uc81c\uae30\ub418\uc790 2007\ub144 5.16\uc744 \uad6c\uad6d\uc758 \ud601\uba85 \uc774\ub77c\uace0 \ubcf8\ub2e4\ub294 \uc790\uc2e0\uc758 \uc0dd\uac01\uc744 \ubc1d\ud614\ub2e4.", "paragraph_id": "6581158-50-1", "paragraph_question": "question: \uc5ed\uc0ac \uc758\uc2dd\uc5d0 \uc758\ubb38\uc774 \uc81c\uae30\ub418\uc790 2007\ub144 5.16\uc744 \ubb34\uc5c7\uc774\ub77c\uace0 \ud558\uc600\ub294\uac00?, context: 1990\ub144 \uc7a1\uc9c0 \uc778\ud130\ubdf0\uc5d0\uc11c \ubc15\uadfc\ud61c\ub294 5\u00b716\uacfc 4\u00b719 \uc815\uc2e0\uc744 \uacc4\uc2b9\ud558\ub294 \uac83\uc774 \ubc15 \uc804 \ub300\ud1b5\ub839\uc758 \ub73b\uc744 \uacc4\uc2b9\ud558\ub294 \uac83\uc774\ub77c\uace0 \ud558\uc600\ub2e4. \uc5ed\uc0ac \uc758\uc2dd\uc5d0 \uc758\ubb38\uc774 \uc81c\uae30\ub418\uc790 2007\ub144 5.16\uc744 \uad6c\uad6d\uc758 \ud601\uba85\uc774\ub77c\uace0 \ubcf8\ub2e4\ub294 \uc790\uc2e0\uc758 \uc0dd\uac01\uc744 \ubc1d\ud614\ub2e4. 2007\ub144 7\uc6d4\uc758 \ud55c\ub098\ub77c\ub2f9 \ub300\uc120\ud6c4\ubcf4 \uac80\uc99d \uccad\ubb38\ud68c\uc5d0\uc11c \uadf8\ub294 '\ubc15 \uc804 \ub300\ud1b5\ub839 \uc720\uc9c0 \uacc4\uc2b9' \uc5ed\uc0ac\uc758\uc2dd\uc5d0 \ub300\ud55c \uc9c8\ubb38\uc744 \ubc1b\uc558\ub2e4. \uadf8\ub294 \"5\u00b716\uc740 \uad6c\uad6d(\u6551\u570b)\uc758 \ud601\uba85\uc774\uc5c8\ub2e4\uace0 \uc0dd\uac01\ud558\uace0 \uc788\ub2e4. \uadf8\ub54c \uc0c1\ud669\uc774 \ub108\ubb34\ub098 \ud63c\ub780\uc2a4\ub7ec\uc6e0\uace0 \ub0a8\ubd81\ub300\uce58 \uc0c1\ud669\uc5d0\uc11c \uc798\ubabb\ud558\uba74 \ubd81\ud55c\uc5d0 \ud761\uc218\ub420 \uc218\ub3c4 \uc788\ub294 \uadf8\ub7f0 \uc0c1\ud669\uc774\uc5c8\ub2e4. \ud601\uba85 \uacf5\uc57d\uc5d0\ub3c4 \uae30\uc544 \uc120\uc0c1\uc5d0 \ud5e4\ub9e4\ub294 \uad6d\ubbfc\uc744 \uad6c\uc81c\ud55c\ub2e4\ub294 \uc598\uae30\uac00 \ub098\uc628\ub2e4.\"\ub294 \uac83\uc774\ub2e4. \ub2e4\ub9cc 10\uc6d4 \uc720\uc2e0\uc5d0 \ub300\ud574\uc11c\ub294 \ud310\ub2e8\uc720\ubcf4\ub97c \ub0b4\ub838\ub2e4. 5.16 \ub4a4\uc758 \uc720\uc2e0\uc2dc\ub300\uc5d0 \ub300\ud574\uc11c\ub294 \uc5b4\ub5bb\uac8c \ud3c9\uac00\ud558\ub098 \ub294 \uc9c8\ubb38\uc5d0 \"\uc5ed\uc0ac\uc5d0 \ud310\ub2e8\uc744 \ub9e1\uaca8\uc57c \ud55c\ub2e4\ub294 \uc785\uc7a5\uc744 \uac16\uace0 \uc788\ub2e4. \ub2e4\ub9cc \uc720\uc2e0\uc2dc\ub300\uc5d0 \ubbfc\uc8fc\ud654 \uc6b4\ub3d9\uc744 \ud558\uba74\uc11c \ud5cc\uc2e0\ud558\uace0 \uace0\ud1b5\ubc1b\uc740 \ubd84\ub4e4\uc5d0 \ub300\ud574\uc11c\ub294 \uc9c4\uc2ec\uc73c\ub85c \ud56d\uc0c1 \uc8c4\uc1a1\ud558\uac8c \uc0dd\uac01\ud55c\ub2e4.\"\uace0 \ub2f5\ud558\uc600\ub2e4. \uc774 \ub54c\ubb38\uc5d0 \uc57c\ub2f9 \uc778\uc0ac\ub4e4\ub85c\ubd80\ud130 \ube44\ub09c\uc744 \ubc1b\uae30\ub3c4 \ud558\uc600\ub2e4."} {"question": "5\uc6d4 10\uc77c \uacbd\uae30 \uc804 \ub204\uac00 \ub358\uc9c4 \uacf5\uc5d0 \ub9de\uc544 \ubd80\uc0c1\uc744 \ub2f9\ud588\ub294\uac00?", "paragraph": "2000\ub144 \uc2dc\uc98c \uac1c\ub9c9 \ub2f9\uc2dc\uc5d0\ub294 \ubc34\ucfe0\ubc84 \uce90\ub098\ub514\uc548\uc2a4\uc5d0\uc11c \uba38\ubb34\ub974\uba74\uc11c 15\uacbd\uae30 3\ud648\ub7f0 15\ud0c0\uc810 .400\uc758 \ud0c0\uc728\uc744 \uae30\ub85d\ud588\uace0, 4\uc6d4 24\uc77c \uba54\uc774\uc800 \ub9ac\uadf8\uc5d0 \uc2b9\uaca9\ub418\uc5c8\ub2e4. \uadf8\ub0a0 \ubc14\ub85c \ud1a0\ub860\ud1a0 \ube14\ub8e8\uc81c\uc774\uc2a4\uc640\uc758 \uacbd\uae30\uc5d0 \uc120\ubc1c \ucd9c\uc7a5\ud574 \ub370\uc774\ube44\ub4dc \uc6f0\uc2a4\ub97c \uc0c1\ub300\ub85c \uac1c\uc778 \ud1b5\uc0b0 \uccab \uc548\ud0c0\ub97c \uc2e0\uace0\ud588\ub2e4. \uc774\uc5b4 \ub2e4\uc74c\ub0a0\uc5d0\ub294 \ube14\ub8e8\uc81c\uc774\uc2a4\uc758 \ub85c\uc774 \ud560\ub7ec\ub370\uc774\ub97c \uc0c1\ub300\ub85c \uba54\uc774\uc800 \ub9ac\uadf8 \uccab \ud648\ub7f0\uc744 \ub54c\ub824\ub0c8\ub2e4. \uc774\ud6c4 \uc5d0\uc774\uc2a4\uc758 \uc8fc\uc804 \ub9ac\ub4dc \uc624\ud504 \uc911\uacac\uc218\ub85c \uc790\ub9ac\uc7a1\uc558\ub2e4. 5\uc6d4 10\uc77c \uacbd\uae30 \uc804 \ubab8\uc744 \ud478\ub294 \ub3c4\uc911 \uc5bc\uad74\uc5d0 \ud300 \ub3d9\ub8cc \ud504\ub7ad\ud06c \uba54\ub124\uce58\ub178\uac00 \uc6b0\uc5f0\ud788 \ub358\uc9c4 \uacf5\uc744 \ub9de\uc544 \uc774\uac00 \ud718\uc5b4\uc9c0\ub294 \ubd80\uc0c1\uc744 \ub2f9\ud574 \uba70\uce60\uac04 \uacb0\uc7a5\ud588\ub2e4. \ubd80\uc0c1 \uc774\ud6c4\uc5d0\ub294 \ub0b4\ub0b4 \ud131 \ubcf4\ud638\ub300\uac00 \uc788\ub294 \ud5ec\uba67\uc744 \ucc29\uc6a9\ud588\ub2e4. 6\uc6d4 \uc911\ud6c4\ubc18\uc5d0\ub294 17\uacbd\uae30 \uc5f0\uc18d \uc548\ud0c0\ub97c \uccd0\ub0b4\uba70 \uc624\ud074\ub79c\ub4dc \uc2e0\uc778 \uc5f0\uc18d \uacbd\uae30 \uc548\ud0c0 \ud0c0\uc774 \uae30\ub85d\uc5d0 \uc774\ub984\uc744 \uc62c\ub838\ub2e4. 8\uc6d4 24\uc77c \ud074\ub9ac\ube14\ub79c\ub4dc \uc778\ub514\uc5b8\uc2a4\uc640\uc758 \uacbd\uae30\uc5d0\uc11c\ub294 \ud1b5\uc0b0 \uccab \ub9cc\ub8e8 \ud648\ub7f0 \ubc0f \uac1c\uc778 \ud55c \uacbd\uae30 \ucd5c\ub2e4 6\ud0c0\uc810\uc744 \uc4f8\uc5b4\ub2f4\uc73c\uba70 \ud300\uc758 11-7 \uc2b9\ub9ac\ub97c \uc774\ub04c\uc5c8\ub2e4. \uc2e0\uc778 \uc2dc\uc98c \ucd5c\uc885 \uae30\ub85d\uc740 138\uacbd\uae30\uc5d0 \ucd9c\uc804\ud574 168\uc548\ud0c0, 18\ud648\ub7f0, 34\uac1c 2\ub8e8\ud0c0, 104\ub4dd\uc810, 80\ud0c0\uc810, \ud0c0\uc728 .288\uc774\ub2e4. \ud2b9\ud788 \uc548\ud0c0\uc640 \ub4dd\uc810 \ubd80\ubb38\uc740 \uc624\ud074\ub79c\ub4dc \uc2e0\uc778 \uae30\ub85d(\uc548\ud0c0\ub294 \ud0c0\uc774)\uc774\uc5c8\ub2e4.", "answer": "\ud504\ub7ad\ud06c \uba54\ub124\uce58\ub178", "sentence": "5\uc6d4 10\uc77c \uacbd\uae30 \uc804 \ubab8\uc744 \ud478\ub294 \ub3c4\uc911 \uc5bc\uad74\uc5d0 \ud300 \ub3d9\ub8cc \ud504\ub7ad\ud06c \uba54\ub124\uce58\ub178 \uac00 \uc6b0\uc5f0\ud788 \ub358\uc9c4 \uacf5\uc744 \ub9de\uc544 \uc774\uac00 \ud718\uc5b4\uc9c0\ub294 \ubd80\uc0c1\uc744 \ub2f9\ud574 \uba70\uce60\uac04 \uacb0\uc7a5\ud588\ub2e4.", "paragraph_sentence": "2000\ub144 \uc2dc\uc98c \uac1c\ub9c9 \ub2f9\uc2dc\uc5d0\ub294 \ubc34\ucfe0\ubc84 \uce90\ub098\ub514\uc548\uc2a4\uc5d0\uc11c \uba38\ubb34\ub974\uba74\uc11c 15\uacbd\uae30 3\ud648\ub7f0 15\ud0c0\uc810 .400\uc758 \ud0c0\uc728\uc744 \uae30\ub85d\ud588\uace0, 4\uc6d4 24\uc77c \uba54\uc774\uc800 \ub9ac\uadf8\uc5d0 \uc2b9\uaca9\ub418\uc5c8\ub2e4. \uadf8\ub0a0 \ubc14\ub85c \ud1a0\ub860\ud1a0 \ube14\ub8e8\uc81c\uc774\uc2a4\uc640\uc758 \uacbd\uae30\uc5d0 \uc120\ubc1c \ucd9c\uc7a5\ud574 \ub370\uc774\ube44\ub4dc \uc6f0\uc2a4\ub97c \uc0c1\ub300\ub85c \uac1c\uc778 \ud1b5\uc0b0 \uccab \uc548\ud0c0\ub97c \uc2e0\uace0\ud588\ub2e4. \uc774\uc5b4 \ub2e4\uc74c\ub0a0\uc5d0\ub294 \ube14\ub8e8\uc81c\uc774\uc2a4\uc758 \ub85c\uc774 \ud560\ub7ec\ub370\uc774\ub97c \uc0c1\ub300\ub85c \uba54\uc774\uc800 \ub9ac\uadf8 \uccab \ud648\ub7f0\uc744 \ub54c\ub824\ub0c8\ub2e4. \uc774\ud6c4 \uc5d0\uc774\uc2a4\uc758 \uc8fc\uc804 \ub9ac\ub4dc \uc624\ud504 \uc911\uacac\uc218\ub85c \uc790\ub9ac\uc7a1\uc558\ub2e4. 5\uc6d4 10\uc77c \uacbd\uae30 \uc804 \ubab8\uc744 \ud478\ub294 \ub3c4\uc911 \uc5bc\uad74\uc5d0 \ud300 \ub3d9\ub8cc \ud504\ub7ad\ud06c \uba54\ub124\uce58\ub178 \uac00 \uc6b0\uc5f0\ud788 \ub358\uc9c4 \uacf5\uc744 \ub9de\uc544 \uc774\uac00 \ud718\uc5b4\uc9c0\ub294 \ubd80\uc0c1\uc744 \ub2f9\ud574 \uba70\uce60\uac04 \uacb0\uc7a5\ud588\ub2e4. \ubd80\uc0c1 \uc774\ud6c4\uc5d0\ub294 \ub0b4\ub0b4 \ud131 \ubcf4\ud638\ub300\uac00 \uc788\ub294 \ud5ec\uba67\uc744 \ucc29\uc6a9\ud588\ub2e4. 6\uc6d4 \uc911\ud6c4\ubc18\uc5d0\ub294 17\uacbd\uae30 \uc5f0\uc18d \uc548\ud0c0\ub97c \uccd0\ub0b4\uba70 \uc624\ud074\ub79c\ub4dc \uc2e0\uc778 \uc5f0\uc18d \uacbd\uae30 \uc548\ud0c0 \ud0c0\uc774 \uae30\ub85d\uc5d0 \uc774\ub984\uc744 \uc62c\ub838\ub2e4. 8\uc6d4 24\uc77c \ud074\ub9ac\ube14\ub79c\ub4dc \uc778\ub514\uc5b8\uc2a4\uc640\uc758 \uacbd\uae30\uc5d0\uc11c\ub294 \ud1b5\uc0b0 \uccab \ub9cc\ub8e8 \ud648\ub7f0 \ubc0f \uac1c\uc778 \ud55c \uacbd\uae30 \ucd5c\ub2e4 6\ud0c0\uc810\uc744 \uc4f8\uc5b4\ub2f4\uc73c\uba70 \ud300\uc758 11-7 \uc2b9\ub9ac\ub97c \uc774\ub04c\uc5c8\ub2e4. \uc2e0\uc778 \uc2dc\uc98c \ucd5c\uc885 \uae30\ub85d\uc740 138\uacbd\uae30\uc5d0 \ucd9c\uc804\ud574 168\uc548\ud0c0, 18\ud648\ub7f0, 34\uac1c 2\ub8e8\ud0c0, 104\ub4dd\uc810, 80\ud0c0\uc810, \ud0c0\uc728 .288\uc774\ub2e4. \ud2b9\ud788 \uc548\ud0c0\uc640 \ub4dd\uc810 \ubd80\ubb38\uc740 \uc624\ud074\ub79c\ub4dc \uc2e0\uc778 \uae30\ub85d(\uc548\ud0c0\ub294 \ud0c0\uc774)\uc774\uc5c8\ub2e4.", "paragraph_answer": "2000\ub144 \uc2dc\uc98c \uac1c\ub9c9 \ub2f9\uc2dc\uc5d0\ub294 \ubc34\ucfe0\ubc84 \uce90\ub098\ub514\uc548\uc2a4\uc5d0\uc11c \uba38\ubb34\ub974\uba74\uc11c 15\uacbd\uae30 3\ud648\ub7f0 15\ud0c0\uc810 .400\uc758 \ud0c0\uc728\uc744 \uae30\ub85d\ud588\uace0, 4\uc6d4 24\uc77c \uba54\uc774\uc800 \ub9ac\uadf8\uc5d0 \uc2b9\uaca9\ub418\uc5c8\ub2e4. \uadf8\ub0a0 \ubc14\ub85c \ud1a0\ub860\ud1a0 \ube14\ub8e8\uc81c\uc774\uc2a4\uc640\uc758 \uacbd\uae30\uc5d0 \uc120\ubc1c \ucd9c\uc7a5\ud574 \ub370\uc774\ube44\ub4dc \uc6f0\uc2a4\ub97c \uc0c1\ub300\ub85c \uac1c\uc778 \ud1b5\uc0b0 \uccab \uc548\ud0c0\ub97c \uc2e0\uace0\ud588\ub2e4. \uc774\uc5b4 \ub2e4\uc74c\ub0a0\uc5d0\ub294 \ube14\ub8e8\uc81c\uc774\uc2a4\uc758 \ub85c\uc774 \ud560\ub7ec\ub370\uc774\ub97c \uc0c1\ub300\ub85c \uba54\uc774\uc800 \ub9ac\uadf8 \uccab \ud648\ub7f0\uc744 \ub54c\ub824\ub0c8\ub2e4. \uc774\ud6c4 \uc5d0\uc774\uc2a4\uc758 \uc8fc\uc804 \ub9ac\ub4dc \uc624\ud504 \uc911\uacac\uc218\ub85c \uc790\ub9ac\uc7a1\uc558\ub2e4. 5\uc6d4 10\uc77c \uacbd\uae30 \uc804 \ubab8\uc744 \ud478\ub294 \ub3c4\uc911 \uc5bc\uad74\uc5d0 \ud300 \ub3d9\ub8cc \ud504\ub7ad\ud06c \uba54\ub124\uce58\ub178 \uac00 \uc6b0\uc5f0\ud788 \ub358\uc9c4 \uacf5\uc744 \ub9de\uc544 \uc774\uac00 \ud718\uc5b4\uc9c0\ub294 \ubd80\uc0c1\uc744 \ub2f9\ud574 \uba70\uce60\uac04 \uacb0\uc7a5\ud588\ub2e4. \ubd80\uc0c1 \uc774\ud6c4\uc5d0\ub294 \ub0b4\ub0b4 \ud131 \ubcf4\ud638\ub300\uac00 \uc788\ub294 \ud5ec\uba67\uc744 \ucc29\uc6a9\ud588\ub2e4. 6\uc6d4 \uc911\ud6c4\ubc18\uc5d0\ub294 17\uacbd\uae30 \uc5f0\uc18d \uc548\ud0c0\ub97c \uccd0\ub0b4\uba70 \uc624\ud074\ub79c\ub4dc \uc2e0\uc778 \uc5f0\uc18d \uacbd\uae30 \uc548\ud0c0 \ud0c0\uc774 \uae30\ub85d\uc5d0 \uc774\ub984\uc744 \uc62c\ub838\ub2e4. 8\uc6d4 24\uc77c \ud074\ub9ac\ube14\ub79c\ub4dc \uc778\ub514\uc5b8\uc2a4\uc640\uc758 \uacbd\uae30\uc5d0\uc11c\ub294 \ud1b5\uc0b0 \uccab \ub9cc\ub8e8 \ud648\ub7f0 \ubc0f \uac1c\uc778 \ud55c \uacbd\uae30 \ucd5c\ub2e4 6\ud0c0\uc810\uc744 \uc4f8\uc5b4\ub2f4\uc73c\uba70 \ud300\uc758 11-7 \uc2b9\ub9ac\ub97c \uc774\ub04c\uc5c8\ub2e4. \uc2e0\uc778 \uc2dc\uc98c \ucd5c\uc885 \uae30\ub85d\uc740 138\uacbd\uae30\uc5d0 \ucd9c\uc804\ud574 168\uc548\ud0c0, 18\ud648\ub7f0, 34\uac1c 2\ub8e8\ud0c0, 104\ub4dd\uc810, 80\ud0c0\uc810, \ud0c0\uc728 .288\uc774\ub2e4. \ud2b9\ud788 \uc548\ud0c0\uc640 \ub4dd\uc810 \ubd80\ubb38\uc740 \uc624\ud074\ub79c\ub4dc \uc2e0\uc778 \uae30\ub85d(\uc548\ud0c0\ub294 \ud0c0\uc774)\uc774\uc5c8\ub2e4.", "sentence_answer": "5\uc6d4 10\uc77c \uacbd\uae30 \uc804 \ubab8\uc744 \ud478\ub294 \ub3c4\uc911 \uc5bc\uad74\uc5d0 \ud300 \ub3d9\ub8cc \ud504\ub7ad\ud06c \uba54\ub124\uce58\ub178 \uac00 \uc6b0\uc5f0\ud788 \ub358\uc9c4 \uacf5\uc744 \ub9de\uc544 \uc774\uac00 \ud718\uc5b4\uc9c0\ub294 \ubd80\uc0c1\uc744 \ub2f9\ud574 \uba70\uce60\uac04 \uacb0\uc7a5\ud588\ub2e4.", "paragraph_id": "6565684-5-1", "paragraph_question": "question: 5\uc6d4 10\uc77c \uacbd\uae30 \uc804 \ub204\uac00 \ub358\uc9c4 \uacf5\uc5d0 \ub9de\uc544 \ubd80\uc0c1\uc744 \ub2f9\ud588\ub294\uac00?, context: 2000\ub144 \uc2dc\uc98c \uac1c\ub9c9 \ub2f9\uc2dc\uc5d0\ub294 \ubc34\ucfe0\ubc84 \uce90\ub098\ub514\uc548\uc2a4\uc5d0\uc11c \uba38\ubb34\ub974\uba74\uc11c 15\uacbd\uae30 3\ud648\ub7f0 15\ud0c0\uc810 .400\uc758 \ud0c0\uc728\uc744 \uae30\ub85d\ud588\uace0, 4\uc6d4 24\uc77c \uba54\uc774\uc800 \ub9ac\uadf8\uc5d0 \uc2b9\uaca9\ub418\uc5c8\ub2e4. \uadf8\ub0a0 \ubc14\ub85c \ud1a0\ub860\ud1a0 \ube14\ub8e8\uc81c\uc774\uc2a4\uc640\uc758 \uacbd\uae30\uc5d0 \uc120\ubc1c \ucd9c\uc7a5\ud574 \ub370\uc774\ube44\ub4dc \uc6f0\uc2a4\ub97c \uc0c1\ub300\ub85c \uac1c\uc778 \ud1b5\uc0b0 \uccab \uc548\ud0c0\ub97c \uc2e0\uace0\ud588\ub2e4. \uc774\uc5b4 \ub2e4\uc74c\ub0a0\uc5d0\ub294 \ube14\ub8e8\uc81c\uc774\uc2a4\uc758 \ub85c\uc774 \ud560\ub7ec\ub370\uc774\ub97c \uc0c1\ub300\ub85c \uba54\uc774\uc800 \ub9ac\uadf8 \uccab \ud648\ub7f0\uc744 \ub54c\ub824\ub0c8\ub2e4. \uc774\ud6c4 \uc5d0\uc774\uc2a4\uc758 \uc8fc\uc804 \ub9ac\ub4dc \uc624\ud504 \uc911\uacac\uc218\ub85c \uc790\ub9ac\uc7a1\uc558\ub2e4. 5\uc6d4 10\uc77c \uacbd\uae30 \uc804 \ubab8\uc744 \ud478\ub294 \ub3c4\uc911 \uc5bc\uad74\uc5d0 \ud300 \ub3d9\ub8cc \ud504\ub7ad\ud06c \uba54\ub124\uce58\ub178\uac00 \uc6b0\uc5f0\ud788 \ub358\uc9c4 \uacf5\uc744 \ub9de\uc544 \uc774\uac00 \ud718\uc5b4\uc9c0\ub294 \ubd80\uc0c1\uc744 \ub2f9\ud574 \uba70\uce60\uac04 \uacb0\uc7a5\ud588\ub2e4. \ubd80\uc0c1 \uc774\ud6c4\uc5d0\ub294 \ub0b4\ub0b4 \ud131 \ubcf4\ud638\ub300\uac00 \uc788\ub294 \ud5ec\uba67\uc744 \ucc29\uc6a9\ud588\ub2e4. 6\uc6d4 \uc911\ud6c4\ubc18\uc5d0\ub294 17\uacbd\uae30 \uc5f0\uc18d \uc548\ud0c0\ub97c \uccd0\ub0b4\uba70 \uc624\ud074\ub79c\ub4dc \uc2e0\uc778 \uc5f0\uc18d \uacbd\uae30 \uc548\ud0c0 \ud0c0\uc774 \uae30\ub85d\uc5d0 \uc774\ub984\uc744 \uc62c\ub838\ub2e4. 8\uc6d4 24\uc77c \ud074\ub9ac\ube14\ub79c\ub4dc \uc778\ub514\uc5b8\uc2a4\uc640\uc758 \uacbd\uae30\uc5d0\uc11c\ub294 \ud1b5\uc0b0 \uccab \ub9cc\ub8e8 \ud648\ub7f0 \ubc0f \uac1c\uc778 \ud55c \uacbd\uae30 \ucd5c\ub2e4 6\ud0c0\uc810\uc744 \uc4f8\uc5b4\ub2f4\uc73c\uba70 \ud300\uc758 11-7 \uc2b9\ub9ac\ub97c \uc774\ub04c\uc5c8\ub2e4. \uc2e0\uc778 \uc2dc\uc98c \ucd5c\uc885 \uae30\ub85d\uc740 138\uacbd\uae30\uc5d0 \ucd9c\uc804\ud574 168\uc548\ud0c0, 18\ud648\ub7f0, 34\uac1c 2\ub8e8\ud0c0, 104\ub4dd\uc810, 80\ud0c0\uc810, \ud0c0\uc728 .288\uc774\ub2e4. \ud2b9\ud788 \uc548\ud0c0\uc640 \ub4dd\uc810 \ubd80\ubb38\uc740 \uc624\ud074\ub79c\ub4dc \uc2e0\uc778 \uae30\ub85d(\uc548\ud0c0\ub294 \ud0c0\uc774)\uc774\uc5c8\ub2e4."} {"question": "1860\ub144\ub300 \uc5d8\ub77c\ud2f0\uc544\ud0b9\uc544\uc774\uac00 \ud48d\ubd80\ud55c \ud1f4\uc801\uce35\uc744 \ubc1c\uacac\ud55c \uc0ac\ub78c\ub4e4\uc740?", "paragraph": "\uc2e0\ub300\ub959 \ucabd\uc5d0\uc11c\ub294 \ubbf8\uad6d\uc758 \ud654\uc11d \uc0ac\ub0e5\uafbc\ub4e4\uc774 1860 \ub144\ub300\uc5d0 \uc720\ud0c0\uc8fc \uc11c\ubd80\uc5d0\uc11c \uc5d8\ub77c\ud2f0\uc544 \ud0b9\uc544\uc774\uac00 \ud48d\ubd80\ud558\uac8c \ubc1c\uacac\ub418\ub294 \ud1f4\uc801\uce35\uc744 \ucc3e\uc544\ub0c8\ub2e4. 1900\ub144\ub300 \ucd08\uae30\uae4c\uc9c0 \uc720\ud0c0 \uc8fc\uc758 \uc6b0\ud14c \uc544\uba54\ub9ac\uce74 \uc6d0\uc8fc\ubbfc\ub4e4\uc740 \uc774 \uc0bc\uc5fd\ucda9\uc744 \ud30c\ucc28\ube44(pachavee, \uc791\uc740 \ubb3c\ubc8c\ub808)\ub77c\uace0 \ubd80\ub974\uba70 \ubd80\uc801\uc0bc\uc544\uc11c \ubab8\uc5d0 \uc9c0\ub2c8\uace0 \ub2e4\ub154\ub2e4. \uc0bc\uc5fd\ucda9\uc758 \uba38\ub9ac\uc5d0 \uad6c\uba4d\uc744 \ub6ab\uace0 \ub048\uc744 \uc5f0\uacb0\ud574 \uac78\uace0 \ub2e4\ub154\ub2e4. \uc6b0\ud14c\uc871\ub4e4\uc5d0 \uc758\ud558\uba74 \uc0bc\uc5fd\ucda9 \ubaa9\uac78\uc774\ub294 \ubcd1\uacfc \ucd1d\uc54c\ub85c\ubd80\ud130 \uc790\uc2e0\uc744 \uc9c0\ucf1c\uc900\ub2e4\uace0 \ud55c\ub2e4. 1931 \ub144\uc5d0 \ud504\ub7ad\ud06c \ubca1\uc704\ub4dc\uac00 \uc6b0\ud14c\uc871\uc774 \uc0bc\uc5fd\ucda9\uc744 \uc0ac\uc6a9\ud588\ub2e4\ub294 \uc99d\uac70\ub97c \ucc3e\uc544\ub0c8\ub2e4. \ubc30\ub4dc\ub79c\ub4dc\ub97c \uc5ec\ud589\ud558\ub2e4\uac00 \uc0bc\uc5fd\ucda9\uc744 \uc758\ubbf8\ud558\ub294 \uac83\uc774 \uac70\uc758 \ud655\uc2e4\ud574 \ubcf4\uc774\ub294 \ub450 \uac1c\uc758 \uc554\uac01\ubb38\uc744 \uc0ac\uc9c4\ucc0d\uc5c8\ub2e4. \uc774 \uc5ec\ud589\uc5d0\uc11c \uadf8\ub294 \ub610 \uc2dc\ub300\uac00 \uc54c\ub824\uc9c0\uc9c0 \uc54a\uc740 \ubb34\ub364\uc5d0\uc11c \uad6c\uba4d \ub6ab\ub9b0 \uc0bc\uc5fd\ucda9 \ud654\uc11d\uc774 \ub9e4\uc7a5\ub41c \uc0ac\ub78c\uc758 \ud749\uacfd \uc548\uc5d0 \ub193\uc5ec \uc788\ub294 \uac83\uc744 \ubc1c\uacac\ud588\ub2e4. \uadf8 \uc774\ud6c4 \uc0bc\uc5fd\ucda9 \ubd80\uc801\uc740 \uadf8\ub808\uc774\ud2b8\ubca0\uc774\uc2a8 \uc804\ubc18\uc5d0 \uac78\uccd0 \ubc1c\uacac\ub418\uc5c8\uc73c\uba70 \ube0c\ub9ac\ud2f0\uc2dc \ucf5c\ub7fc\ube44\uc544\uc640 \ud638\uc8fc\uc5d0\uc11c\ub3c4 \ubc1c\uacac\ub418\uc5c8\ub2e4. 1880 \ub144\ub300\uc5d0 \uace0\uace0\ud559\uc790\ub4e4\uc740 \ud504\ub791\uc2a4\uc758 \uc698\ub290 \ubd80\uadfc, \uc0bc\uc5fd\ucda9 \ub3d9\uad74 (Grotte du Trilobite)\uc5d0\uc11c \ud39c\ub358\ud2b8\ub85c \uc0ac\uc6a9\ud558\uae30 \uc704\ud574 \uad6c\uba4d\uc744 \ub6ab\uc740 \ub4ef\ud55c, \uc190\uc744 \ub9ce\uc774 \ud0c4 \uc0bc\uc5fd\ucda9 \ud654\uc11d\uc744 \ubc1c\uacac\ud588\ub2e4. \uc774\uac83\uc774 \ubc1c\uacac\ub41c \uc9c0\uce35\uc740 1\ub9cc 5000\ub144 \ub41c \uac83\uc73c\ub85c \ud655\uc778\ub418\uc5c8\ub2e4. \ud39c\ub358\ud2b8\uc5d0 \uc190\ub54c\uac00 \ub9ce\uc774 \ubb3b\uc5c8\uae30 \ub54c\ubb38\uc5d0 \uc0bc\uc5fd\ucda9\uc758 \uc885\uc740 \ud655\uc778\ud560 \uc218 \uc5c6\uc5c8\ub2e4. \ud558\uc9c0\ub9cc \uadf8\ub7f0 \ud615\ud0dc\uc758 \uc0bc\uc5fd\ucda9\uc740 \uc698\ub290 \ubd80\uadfc\uc5d0\uc11c\ub294 \ubc1c\uacac\ub418\uc9c0 \uc54a\uae30 \ub54c\ubb38\uc5d0 \uc544\ub9c8\ub3c4 \uad50\uc5ed\uc744 \ud1b5\ud574 \ub2e4\ub978 \uacf3\uc5d0\uc11c \uc654\uc73c\uba70 \ub192\uc740 \uac00\uce58\ub97c \uac00\uc9c0\uace0 \uc788\uc5c8\uc744 \uac83\uc73c\ub85c \ubcf4\uc778\ub2e4.", "answer": "\ubbf8\uad6d\uc758 \ud654\uc11d \uc0ac\ub0e5\uafbc\ub4e4", "sentence": "\uc2e0\ub300\ub959 \ucabd\uc5d0\uc11c\ub294 \ubbf8\uad6d\uc758 \ud654\uc11d \uc0ac\ub0e5\uafbc\ub4e4 \uc774 1860 \ub144\ub300\uc5d0 \uc720\ud0c0\uc8fc \uc11c\ubd80\uc5d0\uc11c \uc5d8\ub77c\ud2f0\uc544 \ud0b9\uc544\uc774\uac00 \ud48d\ubd80\ud558\uac8c \ubc1c\uacac\ub418\ub294 \ud1f4\uc801\uce35\uc744 \ucc3e\uc544\ub0c8\ub2e4.", "paragraph_sentence": " \uc2e0\ub300\ub959 \ucabd\uc5d0\uc11c\ub294 \ubbf8\uad6d\uc758 \ud654\uc11d \uc0ac\ub0e5\uafbc\ub4e4 \uc774 1860 \ub144\ub300\uc5d0 \uc720\ud0c0\uc8fc \uc11c\ubd80\uc5d0\uc11c \uc5d8\ub77c\ud2f0\uc544 \ud0b9\uc544\uc774\uac00 \ud48d\ubd80\ud558\uac8c \ubc1c\uacac\ub418\ub294 \ud1f4\uc801\uce35\uc744 \ucc3e\uc544\ub0c8\ub2e4. 1900\ub144\ub300 \ucd08\uae30\uae4c\uc9c0 \uc720\ud0c0 \uc8fc\uc758 \uc6b0\ud14c \uc544\uba54\ub9ac\uce74 \uc6d0\uc8fc\ubbfc\ub4e4\uc740 \uc774 \uc0bc\uc5fd\ucda9\uc744 \ud30c\ucc28\ube44(pachavee, \uc791\uc740 \ubb3c\ubc8c\ub808)\ub77c\uace0 \ubd80\ub974\uba70 \ubd80\uc801\uc0bc\uc544\uc11c \ubab8\uc5d0 \uc9c0\ub2c8\uace0 \ub2e4\ub154\ub2e4. \uc0bc\uc5fd\ucda9\uc758 \uba38\ub9ac\uc5d0 \uad6c\uba4d\uc744 \ub6ab\uace0 \ub048\uc744 \uc5f0\uacb0\ud574 \uac78\uace0 \ub2e4\ub154\ub2e4. \uc6b0\ud14c\uc871\ub4e4\uc5d0 \uc758\ud558\uba74 \uc0bc\uc5fd\ucda9 \ubaa9\uac78\uc774\ub294 \ubcd1\uacfc \ucd1d\uc54c\ub85c\ubd80\ud130 \uc790\uc2e0\uc744 \uc9c0\ucf1c\uc900\ub2e4\uace0 \ud55c\ub2e4. 1931 \ub144\uc5d0 \ud504\ub7ad\ud06c \ubca1\uc704\ub4dc\uac00 \uc6b0\ud14c\uc871\uc774 \uc0bc\uc5fd\ucda9\uc744 \uc0ac\uc6a9\ud588\ub2e4\ub294 \uc99d\uac70\ub97c \ucc3e\uc544\ub0c8\ub2e4. \ubc30\ub4dc\ub79c\ub4dc\ub97c \uc5ec\ud589\ud558\ub2e4\uac00 \uc0bc\uc5fd\ucda9\uc744 \uc758\ubbf8\ud558\ub294 \uac83\uc774 \uac70\uc758 \ud655\uc2e4\ud574 \ubcf4\uc774\ub294 \ub450 \uac1c\uc758 \uc554\uac01\ubb38\uc744 \uc0ac\uc9c4\ucc0d\uc5c8\ub2e4. \uc774 \uc5ec\ud589\uc5d0\uc11c \uadf8\ub294 \ub610 \uc2dc\ub300\uac00 \uc54c\ub824\uc9c0\uc9c0 \uc54a\uc740 \ubb34\ub364\uc5d0\uc11c \uad6c\uba4d \ub6ab\ub9b0 \uc0bc\uc5fd\ucda9 \ud654\uc11d\uc774 \ub9e4\uc7a5\ub41c \uc0ac\ub78c\uc758 \ud749\uacfd \uc548\uc5d0 \ub193\uc5ec \uc788\ub294 \uac83\uc744 \ubc1c\uacac\ud588\ub2e4. \uadf8 \uc774\ud6c4 \uc0bc\uc5fd\ucda9 \ubd80\uc801\uc740 \uadf8\ub808\uc774\ud2b8\ubca0\uc774\uc2a8 \uc804\ubc18\uc5d0 \uac78\uccd0 \ubc1c\uacac\ub418\uc5c8\uc73c\uba70 \ube0c\ub9ac\ud2f0\uc2dc \ucf5c\ub7fc\ube44\uc544\uc640 \ud638\uc8fc\uc5d0\uc11c\ub3c4 \ubc1c\uacac\ub418\uc5c8\ub2e4. 1880 \ub144\ub300\uc5d0 \uace0\uace0\ud559\uc790\ub4e4\uc740 \ud504\ub791\uc2a4\uc758 \uc698\ub290 \ubd80\uadfc, \uc0bc\uc5fd\ucda9 \ub3d9\uad74 (Grotte du Trilobite)\uc5d0\uc11c \ud39c\ub358\ud2b8\ub85c \uc0ac\uc6a9\ud558\uae30 \uc704\ud574 \uad6c\uba4d\uc744 \ub6ab\uc740 \ub4ef\ud55c, \uc190\uc744 \ub9ce\uc774 \ud0c4 \uc0bc\uc5fd\ucda9 \ud654\uc11d\uc744 \ubc1c\uacac\ud588\ub2e4. \uc774\uac83\uc774 \ubc1c\uacac\ub41c \uc9c0\uce35\uc740 1\ub9cc 5000\ub144 \ub41c \uac83\uc73c\ub85c \ud655\uc778\ub418\uc5c8\ub2e4. \ud39c\ub358\ud2b8\uc5d0 \uc190\ub54c\uac00 \ub9ce\uc774 \ubb3b\uc5c8\uae30 \ub54c\ubb38\uc5d0 \uc0bc\uc5fd\ucda9\uc758 \uc885\uc740 \ud655\uc778\ud560 \uc218 \uc5c6\uc5c8\ub2e4. \ud558\uc9c0\ub9cc \uadf8\ub7f0 \ud615\ud0dc\uc758 \uc0bc\uc5fd\ucda9\uc740 \uc698\ub290 \ubd80\uadfc\uc5d0\uc11c\ub294 \ubc1c\uacac\ub418\uc9c0 \uc54a\uae30 \ub54c\ubb38\uc5d0 \uc544\ub9c8\ub3c4 \uad50\uc5ed\uc744 \ud1b5\ud574 \ub2e4\ub978 \uacf3\uc5d0\uc11c \uc654\uc73c\uba70 \ub192\uc740 \uac00\uce58\ub97c \uac00\uc9c0\uace0 \uc788\uc5c8\uc744 \uac83\uc73c\ub85c \ubcf4\uc778\ub2e4.", "paragraph_answer": "\uc2e0\ub300\ub959 \ucabd\uc5d0\uc11c\ub294 \ubbf8\uad6d\uc758 \ud654\uc11d \uc0ac\ub0e5\uafbc\ub4e4 \uc774 1860 \ub144\ub300\uc5d0 \uc720\ud0c0\uc8fc \uc11c\ubd80\uc5d0\uc11c \uc5d8\ub77c\ud2f0\uc544 \ud0b9\uc544\uc774\uac00 \ud48d\ubd80\ud558\uac8c \ubc1c\uacac\ub418\ub294 \ud1f4\uc801\uce35\uc744 \ucc3e\uc544\ub0c8\ub2e4. 1900\ub144\ub300 \ucd08\uae30\uae4c\uc9c0 \uc720\ud0c0 \uc8fc\uc758 \uc6b0\ud14c \uc544\uba54\ub9ac\uce74 \uc6d0\uc8fc\ubbfc\ub4e4\uc740 \uc774 \uc0bc\uc5fd\ucda9\uc744 \ud30c\ucc28\ube44(pachavee, \uc791\uc740 \ubb3c\ubc8c\ub808)\ub77c\uace0 \ubd80\ub974\uba70 \ubd80\uc801\uc0bc\uc544\uc11c \ubab8\uc5d0 \uc9c0\ub2c8\uace0 \ub2e4\ub154\ub2e4. \uc0bc\uc5fd\ucda9\uc758 \uba38\ub9ac\uc5d0 \uad6c\uba4d\uc744 \ub6ab\uace0 \ub048\uc744 \uc5f0\uacb0\ud574 \uac78\uace0 \ub2e4\ub154\ub2e4. \uc6b0\ud14c\uc871\ub4e4\uc5d0 \uc758\ud558\uba74 \uc0bc\uc5fd\ucda9 \ubaa9\uac78\uc774\ub294 \ubcd1\uacfc \ucd1d\uc54c\ub85c\ubd80\ud130 \uc790\uc2e0\uc744 \uc9c0\ucf1c\uc900\ub2e4\uace0 \ud55c\ub2e4. 1931 \ub144\uc5d0 \ud504\ub7ad\ud06c \ubca1\uc704\ub4dc\uac00 \uc6b0\ud14c\uc871\uc774 \uc0bc\uc5fd\ucda9\uc744 \uc0ac\uc6a9\ud588\ub2e4\ub294 \uc99d\uac70\ub97c \ucc3e\uc544\ub0c8\ub2e4. \ubc30\ub4dc\ub79c\ub4dc\ub97c \uc5ec\ud589\ud558\ub2e4\uac00 \uc0bc\uc5fd\ucda9\uc744 \uc758\ubbf8\ud558\ub294 \uac83\uc774 \uac70\uc758 \ud655\uc2e4\ud574 \ubcf4\uc774\ub294 \ub450 \uac1c\uc758 \uc554\uac01\ubb38\uc744 \uc0ac\uc9c4\ucc0d\uc5c8\ub2e4. \uc774 \uc5ec\ud589\uc5d0\uc11c \uadf8\ub294 \ub610 \uc2dc\ub300\uac00 \uc54c\ub824\uc9c0\uc9c0 \uc54a\uc740 \ubb34\ub364\uc5d0\uc11c \uad6c\uba4d \ub6ab\ub9b0 \uc0bc\uc5fd\ucda9 \ud654\uc11d\uc774 \ub9e4\uc7a5\ub41c \uc0ac\ub78c\uc758 \ud749\uacfd \uc548\uc5d0 \ub193\uc5ec \uc788\ub294 \uac83\uc744 \ubc1c\uacac\ud588\ub2e4. \uadf8 \uc774\ud6c4 \uc0bc\uc5fd\ucda9 \ubd80\uc801\uc740 \uadf8\ub808\uc774\ud2b8\ubca0\uc774\uc2a8 \uc804\ubc18\uc5d0 \uac78\uccd0 \ubc1c\uacac\ub418\uc5c8\uc73c\uba70 \ube0c\ub9ac\ud2f0\uc2dc \ucf5c\ub7fc\ube44\uc544\uc640 \ud638\uc8fc\uc5d0\uc11c\ub3c4 \ubc1c\uacac\ub418\uc5c8\ub2e4. 1880 \ub144\ub300\uc5d0 \uace0\uace0\ud559\uc790\ub4e4\uc740 \ud504\ub791\uc2a4\uc758 \uc698\ub290 \ubd80\uadfc, \uc0bc\uc5fd\ucda9 \ub3d9\uad74 (Grotte du Trilobite)\uc5d0\uc11c \ud39c\ub358\ud2b8\ub85c \uc0ac\uc6a9\ud558\uae30 \uc704\ud574 \uad6c\uba4d\uc744 \ub6ab\uc740 \ub4ef\ud55c, \uc190\uc744 \ub9ce\uc774 \ud0c4 \uc0bc\uc5fd\ucda9 \ud654\uc11d\uc744 \ubc1c\uacac\ud588\ub2e4. \uc774\uac83\uc774 \ubc1c\uacac\ub41c \uc9c0\uce35\uc740 1\ub9cc 5000\ub144 \ub41c \uac83\uc73c\ub85c \ud655\uc778\ub418\uc5c8\ub2e4. \ud39c\ub358\ud2b8\uc5d0 \uc190\ub54c\uac00 \ub9ce\uc774 \ubb3b\uc5c8\uae30 \ub54c\ubb38\uc5d0 \uc0bc\uc5fd\ucda9\uc758 \uc885\uc740 \ud655\uc778\ud560 \uc218 \uc5c6\uc5c8\ub2e4. \ud558\uc9c0\ub9cc \uadf8\ub7f0 \ud615\ud0dc\uc758 \uc0bc\uc5fd\ucda9\uc740 \uc698\ub290 \ubd80\uadfc\uc5d0\uc11c\ub294 \ubc1c\uacac\ub418\uc9c0 \uc54a\uae30 \ub54c\ubb38\uc5d0 \uc544\ub9c8\ub3c4 \uad50\uc5ed\uc744 \ud1b5\ud574 \ub2e4\ub978 \uacf3\uc5d0\uc11c \uc654\uc73c\uba70 \ub192\uc740 \uac00\uce58\ub97c \uac00\uc9c0\uace0 \uc788\uc5c8\uc744 \uac83\uc73c\ub85c \ubcf4\uc778\ub2e4.", "sentence_answer": "\uc2e0\ub300\ub959 \ucabd\uc5d0\uc11c\ub294 \ubbf8\uad6d\uc758 \ud654\uc11d \uc0ac\ub0e5\uafbc\ub4e4 \uc774 1860 \ub144\ub300\uc5d0 \uc720\ud0c0\uc8fc \uc11c\ubd80\uc5d0\uc11c \uc5d8\ub77c\ud2f0\uc544 \ud0b9\uc544\uc774\uac00 \ud48d\ubd80\ud558\uac8c \ubc1c\uacac\ub418\ub294 \ud1f4\uc801\uce35\uc744 \ucc3e\uc544\ub0c8\ub2e4.", "paragraph_id": "6583774-17-1", "paragraph_question": "question: 1860\ub144\ub300 \uc5d8\ub77c\ud2f0\uc544\ud0b9\uc544\uc774\uac00 \ud48d\ubd80\ud55c \ud1f4\uc801\uce35\uc744 \ubc1c\uacac\ud55c \uc0ac\ub78c\ub4e4\uc740?, context: \uc2e0\ub300\ub959 \ucabd\uc5d0\uc11c\ub294 \ubbf8\uad6d\uc758 \ud654\uc11d \uc0ac\ub0e5\uafbc\ub4e4\uc774 1860 \ub144\ub300\uc5d0 \uc720\ud0c0\uc8fc \uc11c\ubd80\uc5d0\uc11c \uc5d8\ub77c\ud2f0\uc544 \ud0b9\uc544\uc774\uac00 \ud48d\ubd80\ud558\uac8c \ubc1c\uacac\ub418\ub294 \ud1f4\uc801\uce35\uc744 \ucc3e\uc544\ub0c8\ub2e4. 1900\ub144\ub300 \ucd08\uae30\uae4c\uc9c0 \uc720\ud0c0 \uc8fc\uc758 \uc6b0\ud14c \uc544\uba54\ub9ac\uce74 \uc6d0\uc8fc\ubbfc\ub4e4\uc740 \uc774 \uc0bc\uc5fd\ucda9\uc744 \ud30c\ucc28\ube44(pachavee, \uc791\uc740 \ubb3c\ubc8c\ub808)\ub77c\uace0 \ubd80\ub974\uba70 \ubd80\uc801\uc0bc\uc544\uc11c \ubab8\uc5d0 \uc9c0\ub2c8\uace0 \ub2e4\ub154\ub2e4. \uc0bc\uc5fd\ucda9\uc758 \uba38\ub9ac\uc5d0 \uad6c\uba4d\uc744 \ub6ab\uace0 \ub048\uc744 \uc5f0\uacb0\ud574 \uac78\uace0 \ub2e4\ub154\ub2e4. \uc6b0\ud14c\uc871\ub4e4\uc5d0 \uc758\ud558\uba74 \uc0bc\uc5fd\ucda9 \ubaa9\uac78\uc774\ub294 \ubcd1\uacfc \ucd1d\uc54c\ub85c\ubd80\ud130 \uc790\uc2e0\uc744 \uc9c0\ucf1c\uc900\ub2e4\uace0 \ud55c\ub2e4. 1931 \ub144\uc5d0 \ud504\ub7ad\ud06c \ubca1\uc704\ub4dc\uac00 \uc6b0\ud14c\uc871\uc774 \uc0bc\uc5fd\ucda9\uc744 \uc0ac\uc6a9\ud588\ub2e4\ub294 \uc99d\uac70\ub97c \ucc3e\uc544\ub0c8\ub2e4. \ubc30\ub4dc\ub79c\ub4dc\ub97c \uc5ec\ud589\ud558\ub2e4\uac00 \uc0bc\uc5fd\ucda9\uc744 \uc758\ubbf8\ud558\ub294 \uac83\uc774 \uac70\uc758 \ud655\uc2e4\ud574 \ubcf4\uc774\ub294 \ub450 \uac1c\uc758 \uc554\uac01\ubb38\uc744 \uc0ac\uc9c4\ucc0d\uc5c8\ub2e4. \uc774 \uc5ec\ud589\uc5d0\uc11c \uadf8\ub294 \ub610 \uc2dc\ub300\uac00 \uc54c\ub824\uc9c0\uc9c0 \uc54a\uc740 \ubb34\ub364\uc5d0\uc11c \uad6c\uba4d \ub6ab\ub9b0 \uc0bc\uc5fd\ucda9 \ud654\uc11d\uc774 \ub9e4\uc7a5\ub41c \uc0ac\ub78c\uc758 \ud749\uacfd \uc548\uc5d0 \ub193\uc5ec \uc788\ub294 \uac83\uc744 \ubc1c\uacac\ud588\ub2e4. \uadf8 \uc774\ud6c4 \uc0bc\uc5fd\ucda9 \ubd80\uc801\uc740 \uadf8\ub808\uc774\ud2b8\ubca0\uc774\uc2a8 \uc804\ubc18\uc5d0 \uac78\uccd0 \ubc1c\uacac\ub418\uc5c8\uc73c\uba70 \ube0c\ub9ac\ud2f0\uc2dc \ucf5c\ub7fc\ube44\uc544\uc640 \ud638\uc8fc\uc5d0\uc11c\ub3c4 \ubc1c\uacac\ub418\uc5c8\ub2e4. 1880 \ub144\ub300\uc5d0 \uace0\uace0\ud559\uc790\ub4e4\uc740 \ud504\ub791\uc2a4\uc758 \uc698\ub290 \ubd80\uadfc, \uc0bc\uc5fd\ucda9 \ub3d9\uad74 (Grotte du Trilobite)\uc5d0\uc11c \ud39c\ub358\ud2b8\ub85c \uc0ac\uc6a9\ud558\uae30 \uc704\ud574 \uad6c\uba4d\uc744 \ub6ab\uc740 \ub4ef\ud55c, \uc190\uc744 \ub9ce\uc774 \ud0c4 \uc0bc\uc5fd\ucda9 \ud654\uc11d\uc744 \ubc1c\uacac\ud588\ub2e4. \uc774\uac83\uc774 \ubc1c\uacac\ub41c \uc9c0\uce35\uc740 1\ub9cc 5000\ub144 \ub41c \uac83\uc73c\ub85c \ud655\uc778\ub418\uc5c8\ub2e4. \ud39c\ub358\ud2b8\uc5d0 \uc190\ub54c\uac00 \ub9ce\uc774 \ubb3b\uc5c8\uae30 \ub54c\ubb38\uc5d0 \uc0bc\uc5fd\ucda9\uc758 \uc885\uc740 \ud655\uc778\ud560 \uc218 \uc5c6\uc5c8\ub2e4. \ud558\uc9c0\ub9cc \uadf8\ub7f0 \ud615\ud0dc\uc758 \uc0bc\uc5fd\ucda9\uc740 \uc698\ub290 \ubd80\uadfc\uc5d0\uc11c\ub294 \ubc1c\uacac\ub418\uc9c0 \uc54a\uae30 \ub54c\ubb38\uc5d0 \uc544\ub9c8\ub3c4 \uad50\uc5ed\uc744 \ud1b5\ud574 \ub2e4\ub978 \uacf3\uc5d0\uc11c \uc654\uc73c\uba70 \ub192\uc740 \uac00\uce58\ub97c \uac00\uc9c0\uace0 \uc788\uc5c8\uc744 \uac83\uc73c\ub85c \ubcf4\uc778\ub2e4."} {"question": "\uc785\uc6d0\ud55c \ubcd1\uc2e4\uacfc \uc9d1\uc548\uc758 \ud53c\ud574 \uc0c1\ud669\uc744 \uacf5\uac1c\ud588\ub358 \uac83\uc740, \uc870\uc131\ubbfc\uc758 \uc5b4\ub5a4 \uc8fc\uc7a5\uc774 \uac70\uc9d3\uc774\ub77c\ub294 \uac83\uc744 \uc785\uc99d\ud558\uae30 \uc704\ud55c \uc870\uce58\uc600\ub098?", "paragraph": "\ucd5c\uc9c4\uc2e4\uc774 2\uc2ec\uc5d0\uc11c \uc2b9\uc18c\ud558\uba74\uc11c \uc720\uba85 \uc5f0\uc608\uc778\uc758 \uc778\uad8c\uc5d0 \ub300\ud574 \uc0c8\ub86d\uac8c \ub3cc\uc544\ubcf4\ub294 \uacc4\uae30\uac00 \ub9c8\ub828\ub418\uc5c8\ub2e4. 2\uc2ec \ud310\uacb0\uc740 \ucd5c\uc9c4\uc2e4\uc740 \uac00\uc815\ud3ed\ub825\uc758 \ud53c\ud574\uc790\uc77c \ubfd0 \ud3ed\ub825 \ubd80\ubd84\uc774\ub098 \uc5b8\ub860 \ub300\uc751 \ubd80\ubd84\uc5d0 \uc788\uc5b4 \uacfc\uc2e4\uc774 \uc5c6\uc74c\uc744 \ubc1d\ud78c \ud310\ub2e8\uc774\uc5c8\uace0 \uba85\uc608\ud6fc\uc190 \ubd80\ubd84\uc5d0 \uc788\uc5b4\uc11c\ub3c4 \uc778\uacfc\uad00\uacc4\uac00 \uc778\uc815\ub418\uc9c0 \uc54a\uc74c\uc744 \uba85\ud655\ud788 \ud55c \ud310\uacb0\uc774\uc5c8\ub2e4. 2006\ub144 5\uc6d4 3\uc77c, 2\uc2ec \ud310\uacb0\uacfc \uad00\ub828\ud558\uc5ec \uac15\uc9c0\uc6d0\uc740 \"\ub9ce\uc740 \ubd84\ub4e4\uc774 \uc774 \uc0ac\uac74\uc758 \uc2e4\uccb4\ub97c \uc815\ud655\ud788 \ubaa8\ub974\uc2dc\ub294 \uac83 \uac19\ub2e4. \uc774\ubc88 \uc0ac\uac74\uc740 \uc5ec\uc131 \uc778\uad8c \ucc28\uc6d0\uc758 \ubb38\uc81c\uc600\uc73c\uba70 \uc720\uba85 \uc5f0\uc608\uc778\uc758 \uc778\uad8c \uc5ed\uc2dc \ubcf4\ud638\ubc1b\uc544\uc57c \ud55c\ub2e4\ub294 \uac83\uc744 \ub4dc\ub7ec\ub0b8 \uac83\uc774\ub2e4. \uc774\ubc88 \uc0ac\uac74\uc758 \ubcf8\uc9c8\uc740 \uc5ec\uc131 \uc778\uad8c \ubb38\uc81c\uc640 \uc5f0\uc608\uc778 \uc778\uad8c \ubb38\uc81c\uac00 \uacb0\ud569\ub41c \uac83\uc784\uc744 \uc9c1\uc2dc\ud574\uc57c\ud560 \uac83\uc774\ub2e4. \uc774\ubc88 \ud310\uacb0\uc744 \ud1b5\ud574 \uc5f0\uc608\uc778 \uc778\uad8c \ubb38\uc81c\ub3c4 \ud2b9\uc815 \uc5f0\uc608\uc778\uc5d0 \ub300\ud55c \uc120\ud638\ub3c4\uc640 \uc0c1\uad00\uc5c6\uc774 \uc778\uad8c \ucc28\uc6d0\uc5d0\uc11c \uc811\uadfc\ud560 \uc218 \uc788\ub294 \uc218\uc900 \ub192\uc740 \uc778\uc2dd\uc774 \ud655\uc0b0\ub410\uc73c\uba74 \uc88b\uaca0\ub2e4. \ub9cc\uc57d \ucd5c\uc9c4\uc2e4 \uc528\uac00 \ub9c8\uc57d\uc774\ub098 \uc74c\uc8fc\uc6b4\uc804, \ubcd1\uc5ed \uae30\ud53c \uac19\uc740 \ubcf8\uc778\uc758 \uc798\ubabb\uc73c\ub85c \uc778\ud574 \uae30\uc5c5 \uc774\ubbf8\uc9c0\ub97c \uc2e4\ucd94\ud588\ub2e4\uba74 \uae30\uc5c5 \uce21\uc774 \uc18c\uc1a1\uc744 \uc81c\uae30\ud560 \uc218 \uc788\uc5c8\uaca0\uc73c\ub098 \ucd5c\uc9c4\uc2e4 \uc528\ub294 \ud55c\ubc24\uc911\uc5d0 \uc220 \ub9c8\uc2e0 \ub0a8\ud3b8\uc5d0\uac8c \uc77c\ubc29\uc801 \ud3ed\ud589\uc744 \ub2f9\ud55c \uc5c4\uc5f0\ud55c \uac00\uc815 \ud3ed\ub825\uc758 \ud53c\ud574\uc790\uc600\ub2e4. \uc7ac\ud310\ubd80\uac00 \uc5ec\uc131 \uc778\uad8c \ubb38\uc81c\ub85c \uc811\uadfc\ud55c \ubcc0\ud638\uc778\ub2e8\uc758 \uc8fc\uc7a5\uc744 \ubc1b\uc544\ub4e4\uc5ec\uc900 \uac83\uc73c\ub85c \ubcf4\uc778\ub2e4. \uc778\uc801 \ud53c\ud574 \uc0c1\ud669\uc744 \uc54c\ub9ac\uae30 \uc704\ud574 \uc785\uc6d0\ud574 \uc788\ub294 \ubcd1\uc2e4\uc744, \ubb3c\uc801 \ud53c\ud574 \uc0c1\ud669\uc744 \uc54c\ub9ac\uae30 \uc704\ud574 \uc9d1\uc548\uc744 \uacf5\uac1c\ud588\ub358 \uac83\uc774\ub2e4. \uc7ac\ud310\ubd80\uc5d0 \uacb0\ucf54 \uc30d\ubc29 \ud3ed\ud589\uc774 \uc544\ub2c8\ub77c\ub294 \uc0ac\uc2e4\uc744 \uc785\uc99d\ud558\uae30 \uc704\ud574 \ucde8\ud588\ub358 \uc870\uce58\ub77c\uace0 \uc8fc\uc7a5\ud588\ub2e4. \uc774\ubc88 \uc0ac\uac74\uc744 \ud1b5\ud574 \uc720\uba85 \uc5f0\uc608\uc778\uc758 \uc778\uad8c\uc5d0 \ub300\ud574\uc11c\ub3c4 \uc2ec\uac01\ud55c \uace0\ub824\ub97c \ud574\uc57c \ud55c\ub2e4\uace0 \uc0dd\uac01\ud588\ub2e4. \ubc29\uc1a1\uad6d\uc5d0\uc11c \uc5f0\uc608\uc778\ub4e4\uc744 \ub9cc\ub098\uba74 '\ub0b4 \ubaa9\uc228\uc774 \uacb0\ucf54 \ub0b4 \ubaa9\uc228\uc774 \uc544\ub2c8\ub2e4'\ub77c\uace0 \ub9d0\ud558\ub294 \uacbd\uc6b0\ub97c \uc885\uc885 \ubd10\uc654\ub2e4. \ube44\ub85d \ub300\uc911\uc5d0\uac8c \uc0ac\uc0dd\ud65c\uc774 \uacf5\uac1c\ub420 \uc218\ubc16\uc5d0 \uc5c6\ub294 \uc5f0\uc608\uc778\uc758 \uc704\uce58\ub77c\ub3c4 \ud2b8\uc704\uc2a4\ud2b8 \uae40\uacfc \uac19\uc774 \uc74c\ub780 \uc0ac\uc774\ud2b8\uc5d0 \uc774\ub984\uc774 \ub3c4\uc6a9\ub418\uac70\ub098 \uc548\ud2f0 \uc138\ub825\uc73c\ub85c\ubd80\ud130 \ubb34\uc870\uac74\uc801\uc778 \uc778\uaca9\uc801 \ubaa8\uba78\uc744 \ubc1b\ub294 \uacbd\uc6b0 \uc774\ub97c \ubc29\uc5b4\ud560 \uc218 \uc788\ub294 \ucd5c\uc18c\ud55c\uc758 \uad8c\ub9ac\uac00 \uc788\uc5b4\uc57c \ud55c\ub2e4. \uac74\uc124\uc0ac\uac00 \uc774\ubbf8\uc9c0\uac00 \uc0dd\uba85\uc778 \uc720\uba85 \uc5f0\uc608\uc778\uc758 \uc57d\uc810\uc744 \uc774\uc6a9\ud574 30\uc5b5 \uc6d0\uc774\ub77c\ub294 \uc5b4\ub9c8\uc5b4\ub9c8\ud55c \uc18c\uc1a1\uc744 \uc81c\uae30\ud558\ub294 \uac83\uc740 \uc18c\uc1a1\uc758 \ub0a8\uc6a9\uc774\ub77c\ub294 \uc810\uc744 \uc7ac\ud310\ubd80\uc5d0 \uc801\uadf9 \ubc1d\ud614\ub2e4. \uc774 \uc0ac\uac74\uc774 \ub204\uad6c\ub4e0\uc9c0 \ub2f9\ud560 \uc218 \uc788\ub294 \uac00\uc815 \ud3ed\ub825\uc758 \ud53c\ud574\uc790, \uc989 \uc18c\uc1a1\uc758 \ud575\uc2ec\uc774\uc5c8\ub358 \uc778\uad8c \ubb38\uc81c\ub85c \uc778\uc9c0\ub418\uc9c0 \ubabb\ud55c \ucc44 \ub2e8\uc21c\ud788 \uc2a4\ud0c0\uc758 \uc0ac\uc0dd\ud65c\ub85c\ub9cc \ube44\ucdb0\uc9c0\uc9c0 \uc54a\uc744\uae4c \uac71\uc815\ub41c\ub2e4\"\ub77c\uace0 \ub9d0\ud588\ub2e4.", "answer": "\uc30d\ubc29 \ud3ed\ud589", "sentence": "\uc7ac\ud310\ubd80\uc5d0 \uacb0\ucf54 \uc30d\ubc29 \ud3ed\ud589 \uc774 \uc544\ub2c8\ub77c\ub294 \uc0ac\uc2e4\uc744 \uc785\uc99d\ud558\uae30 \uc704\ud574 \ucde8\ud588\ub358 \uc870\uce58\ub77c\uace0 \uc8fc\uc7a5\ud588\ub2e4.", "paragraph_sentence": "\ucd5c\uc9c4\uc2e4\uc774 2\uc2ec\uc5d0\uc11c \uc2b9\uc18c\ud558\uba74\uc11c \uc720\uba85 \uc5f0\uc608\uc778\uc758 \uc778\uad8c\uc5d0 \ub300\ud574 \uc0c8\ub86d\uac8c \ub3cc\uc544\ubcf4\ub294 \uacc4\uae30\uac00 \ub9c8\ub828\ub418\uc5c8\ub2e4. 2\uc2ec \ud310\uacb0\uc740 \ucd5c\uc9c4\uc2e4\uc740 \uac00\uc815\ud3ed\ub825\uc758 \ud53c\ud574\uc790\uc77c \ubfd0 \ud3ed\ub825 \ubd80\ubd84\uc774\ub098 \uc5b8\ub860 \ub300\uc751 \ubd80\ubd84\uc5d0 \uc788\uc5b4 \uacfc\uc2e4\uc774 \uc5c6\uc74c\uc744 \ubc1d\ud78c \ud310\ub2e8\uc774\uc5c8\uace0 \uba85\uc608\ud6fc\uc190 \ubd80\ubd84\uc5d0 \uc788\uc5b4\uc11c\ub3c4 \uc778\uacfc\uad00\uacc4\uac00 \uc778\uc815\ub418\uc9c0 \uc54a\uc74c\uc744 \uba85\ud655\ud788 \ud55c \ud310\uacb0\uc774\uc5c8\ub2e4. 2006\ub144 5\uc6d4 3\uc77c, 2\uc2ec \ud310\uacb0\uacfc \uad00\ub828\ud558\uc5ec \uac15\uc9c0\uc6d0\uc740 \"\ub9ce\uc740 \ubd84\ub4e4\uc774 \uc774 \uc0ac\uac74\uc758 \uc2e4\uccb4\ub97c \uc815\ud655\ud788 \ubaa8\ub974\uc2dc\ub294 \uac83 \uac19\ub2e4. \uc774\ubc88 \uc0ac\uac74\uc740 \uc5ec\uc131 \uc778\uad8c \ucc28\uc6d0\uc758 \ubb38\uc81c\uc600\uc73c\uba70 \uc720\uba85 \uc5f0\uc608\uc778\uc758 \uc778\uad8c \uc5ed\uc2dc \ubcf4\ud638\ubc1b\uc544\uc57c \ud55c\ub2e4\ub294 \uac83\uc744 \ub4dc\ub7ec\ub0b8 \uac83\uc774\ub2e4. \uc774\ubc88 \uc0ac\uac74\uc758 \ubcf8\uc9c8\uc740 \uc5ec\uc131 \uc778\uad8c \ubb38\uc81c\uc640 \uc5f0\uc608\uc778 \uc778\uad8c \ubb38\uc81c\uac00 \uacb0\ud569\ub41c \uac83\uc784\uc744 \uc9c1\uc2dc\ud574\uc57c\ud560 \uac83\uc774\ub2e4. \uc774\ubc88 \ud310\uacb0\uc744 \ud1b5\ud574 \uc5f0\uc608\uc778 \uc778\uad8c \ubb38\uc81c\ub3c4 \ud2b9\uc815 \uc5f0\uc608\uc778\uc5d0 \ub300\ud55c \uc120\ud638\ub3c4\uc640 \uc0c1\uad00\uc5c6\uc774 \uc778\uad8c \ucc28\uc6d0\uc5d0\uc11c \uc811\uadfc\ud560 \uc218 \uc788\ub294 \uc218\uc900 \ub192\uc740 \uc778\uc2dd\uc774 \ud655\uc0b0\ub410\uc73c\uba74 \uc88b\uaca0\ub2e4. \ub9cc\uc57d \ucd5c\uc9c4\uc2e4 \uc528\uac00 \ub9c8\uc57d\uc774\ub098 \uc74c\uc8fc\uc6b4\uc804, \ubcd1\uc5ed \uae30\ud53c \uac19\uc740 \ubcf8\uc778\uc758 \uc798\ubabb\uc73c\ub85c \uc778\ud574 \uae30\uc5c5 \uc774\ubbf8\uc9c0\ub97c \uc2e4\ucd94\ud588\ub2e4\uba74 \uae30\uc5c5 \uce21\uc774 \uc18c\uc1a1\uc744 \uc81c\uae30\ud560 \uc218 \uc788\uc5c8\uaca0\uc73c\ub098 \ucd5c\uc9c4\uc2e4 \uc528\ub294 \ud55c\ubc24\uc911\uc5d0 \uc220 \ub9c8\uc2e0 \ub0a8\ud3b8\uc5d0\uac8c \uc77c\ubc29\uc801 \ud3ed\ud589\uc744 \ub2f9\ud55c \uc5c4\uc5f0\ud55c \uac00\uc815 \ud3ed\ub825\uc758 \ud53c\ud574\uc790\uc600\ub2e4. \uc7ac\ud310\ubd80\uac00 \uc5ec\uc131 \uc778\uad8c \ubb38\uc81c\ub85c \uc811\uadfc\ud55c \ubcc0\ud638\uc778\ub2e8\uc758 \uc8fc\uc7a5\uc744 \ubc1b\uc544\ub4e4\uc5ec\uc900 \uac83\uc73c\ub85c \ubcf4\uc778\ub2e4. \uc778\uc801 \ud53c\ud574 \uc0c1\ud669\uc744 \uc54c\ub9ac\uae30 \uc704\ud574 \uc785\uc6d0\ud574 \uc788\ub294 \ubcd1\uc2e4\uc744, \ubb3c\uc801 \ud53c\ud574 \uc0c1\ud669\uc744 \uc54c\ub9ac\uae30 \uc704\ud574 \uc9d1\uc548\uc744 \uacf5\uac1c\ud588\ub358 \uac83\uc774\ub2e4. \uc7ac\ud310\ubd80\uc5d0 \uacb0\ucf54 \uc30d\ubc29 \ud3ed\ud589 \uc774 \uc544\ub2c8\ub77c\ub294 \uc0ac\uc2e4\uc744 \uc785\uc99d\ud558\uae30 \uc704\ud574 \ucde8\ud588\ub358 \uc870\uce58\ub77c\uace0 \uc8fc\uc7a5\ud588\ub2e4. \uc774\ubc88 \uc0ac\uac74\uc744 \ud1b5\ud574 \uc720\uba85 \uc5f0\uc608\uc778\uc758 \uc778\uad8c\uc5d0 \ub300\ud574\uc11c\ub3c4 \uc2ec\uac01\ud55c \uace0\ub824\ub97c \ud574\uc57c \ud55c\ub2e4\uace0 \uc0dd\uac01\ud588\ub2e4. \ubc29\uc1a1\uad6d\uc5d0\uc11c \uc5f0\uc608\uc778\ub4e4\uc744 \ub9cc\ub098\uba74 '\ub0b4 \ubaa9\uc228\uc774 \uacb0\ucf54 \ub0b4 \ubaa9\uc228\uc774 \uc544\ub2c8\ub2e4'\ub77c\uace0 \ub9d0\ud558\ub294 \uacbd\uc6b0\ub97c \uc885\uc885 \ubd10\uc654\ub2e4. \ube44\ub85d \ub300\uc911\uc5d0\uac8c \uc0ac\uc0dd\ud65c\uc774 \uacf5\uac1c\ub420 \uc218\ubc16\uc5d0 \uc5c6\ub294 \uc5f0\uc608\uc778\uc758 \uc704\uce58\ub77c\ub3c4 \ud2b8\uc704\uc2a4\ud2b8 \uae40\uacfc \uac19\uc774 \uc74c\ub780 \uc0ac\uc774\ud2b8\uc5d0 \uc774\ub984\uc774 \ub3c4\uc6a9\ub418\uac70\ub098 \uc548\ud2f0 \uc138\ub825\uc73c\ub85c\ubd80\ud130 \ubb34\uc870\uac74\uc801\uc778 \uc778\uaca9\uc801 \ubaa8\uba78\uc744 \ubc1b\ub294 \uacbd\uc6b0 \uc774\ub97c \ubc29\uc5b4\ud560 \uc218 \uc788\ub294 \ucd5c\uc18c\ud55c\uc758 \uad8c\ub9ac\uac00 \uc788\uc5b4\uc57c \ud55c\ub2e4. \uac74\uc124\uc0ac\uac00 \uc774\ubbf8\uc9c0\uac00 \uc0dd\uba85\uc778 \uc720\uba85 \uc5f0\uc608\uc778\uc758 \uc57d\uc810\uc744 \uc774\uc6a9\ud574 30\uc5b5 \uc6d0\uc774\ub77c\ub294 \uc5b4\ub9c8\uc5b4\ub9c8\ud55c \uc18c\uc1a1\uc744 \uc81c\uae30\ud558\ub294 \uac83\uc740 \uc18c\uc1a1\uc758 \ub0a8\uc6a9\uc774\ub77c\ub294 \uc810\uc744 \uc7ac\ud310\ubd80\uc5d0 \uc801\uadf9 \ubc1d\ud614\ub2e4. \uc774 \uc0ac\uac74\uc774 \ub204\uad6c\ub4e0\uc9c0 \ub2f9\ud560 \uc218 \uc788\ub294 \uac00\uc815 \ud3ed\ub825\uc758 \ud53c\ud574\uc790, \uc989 \uc18c\uc1a1\uc758 \ud575\uc2ec\uc774\uc5c8\ub358 \uc778\uad8c \ubb38\uc81c\ub85c \uc778\uc9c0\ub418\uc9c0 \ubabb\ud55c \ucc44 \ub2e8\uc21c\ud788 \uc2a4\ud0c0\uc758 \uc0ac\uc0dd\ud65c\ub85c\ub9cc \ube44\ucdb0\uc9c0\uc9c0 \uc54a\uc744\uae4c \uac71\uc815\ub41c\ub2e4\"\ub77c\uace0 \ub9d0\ud588\ub2e4.", "paragraph_answer": "\ucd5c\uc9c4\uc2e4\uc774 2\uc2ec\uc5d0\uc11c \uc2b9\uc18c\ud558\uba74\uc11c \uc720\uba85 \uc5f0\uc608\uc778\uc758 \uc778\uad8c\uc5d0 \ub300\ud574 \uc0c8\ub86d\uac8c \ub3cc\uc544\ubcf4\ub294 \uacc4\uae30\uac00 \ub9c8\ub828\ub418\uc5c8\ub2e4. 2\uc2ec \ud310\uacb0\uc740 \ucd5c\uc9c4\uc2e4\uc740 \uac00\uc815\ud3ed\ub825\uc758 \ud53c\ud574\uc790\uc77c \ubfd0 \ud3ed\ub825 \ubd80\ubd84\uc774\ub098 \uc5b8\ub860 \ub300\uc751 \ubd80\ubd84\uc5d0 \uc788\uc5b4 \uacfc\uc2e4\uc774 \uc5c6\uc74c\uc744 \ubc1d\ud78c \ud310\ub2e8\uc774\uc5c8\uace0 \uba85\uc608\ud6fc\uc190 \ubd80\ubd84\uc5d0 \uc788\uc5b4\uc11c\ub3c4 \uc778\uacfc\uad00\uacc4\uac00 \uc778\uc815\ub418\uc9c0 \uc54a\uc74c\uc744 \uba85\ud655\ud788 \ud55c \ud310\uacb0\uc774\uc5c8\ub2e4. 2006\ub144 5\uc6d4 3\uc77c, 2\uc2ec \ud310\uacb0\uacfc \uad00\ub828\ud558\uc5ec \uac15\uc9c0\uc6d0\uc740 \"\ub9ce\uc740 \ubd84\ub4e4\uc774 \uc774 \uc0ac\uac74\uc758 \uc2e4\uccb4\ub97c \uc815\ud655\ud788 \ubaa8\ub974\uc2dc\ub294 \uac83 \uac19\ub2e4. \uc774\ubc88 \uc0ac\uac74\uc740 \uc5ec\uc131 \uc778\uad8c \ucc28\uc6d0\uc758 \ubb38\uc81c\uc600\uc73c\uba70 \uc720\uba85 \uc5f0\uc608\uc778\uc758 \uc778\uad8c \uc5ed\uc2dc \ubcf4\ud638\ubc1b\uc544\uc57c \ud55c\ub2e4\ub294 \uac83\uc744 \ub4dc\ub7ec\ub0b8 \uac83\uc774\ub2e4. \uc774\ubc88 \uc0ac\uac74\uc758 \ubcf8\uc9c8\uc740 \uc5ec\uc131 \uc778\uad8c \ubb38\uc81c\uc640 \uc5f0\uc608\uc778 \uc778\uad8c \ubb38\uc81c\uac00 \uacb0\ud569\ub41c \uac83\uc784\uc744 \uc9c1\uc2dc\ud574\uc57c\ud560 \uac83\uc774\ub2e4. \uc774\ubc88 \ud310\uacb0\uc744 \ud1b5\ud574 \uc5f0\uc608\uc778 \uc778\uad8c \ubb38\uc81c\ub3c4 \ud2b9\uc815 \uc5f0\uc608\uc778\uc5d0 \ub300\ud55c \uc120\ud638\ub3c4\uc640 \uc0c1\uad00\uc5c6\uc774 \uc778\uad8c \ucc28\uc6d0\uc5d0\uc11c \uc811\uadfc\ud560 \uc218 \uc788\ub294 \uc218\uc900 \ub192\uc740 \uc778\uc2dd\uc774 \ud655\uc0b0\ub410\uc73c\uba74 \uc88b\uaca0\ub2e4. \ub9cc\uc57d \ucd5c\uc9c4\uc2e4 \uc528\uac00 \ub9c8\uc57d\uc774\ub098 \uc74c\uc8fc\uc6b4\uc804, \ubcd1\uc5ed \uae30\ud53c \uac19\uc740 \ubcf8\uc778\uc758 \uc798\ubabb\uc73c\ub85c \uc778\ud574 \uae30\uc5c5 \uc774\ubbf8\uc9c0\ub97c \uc2e4\ucd94\ud588\ub2e4\uba74 \uae30\uc5c5 \uce21\uc774 \uc18c\uc1a1\uc744 \uc81c\uae30\ud560 \uc218 \uc788\uc5c8\uaca0\uc73c\ub098 \ucd5c\uc9c4\uc2e4 \uc528\ub294 \ud55c\ubc24\uc911\uc5d0 \uc220 \ub9c8\uc2e0 \ub0a8\ud3b8\uc5d0\uac8c \uc77c\ubc29\uc801 \ud3ed\ud589\uc744 \ub2f9\ud55c \uc5c4\uc5f0\ud55c \uac00\uc815 \ud3ed\ub825\uc758 \ud53c\ud574\uc790\uc600\ub2e4. \uc7ac\ud310\ubd80\uac00 \uc5ec\uc131 \uc778\uad8c \ubb38\uc81c\ub85c \uc811\uadfc\ud55c \ubcc0\ud638\uc778\ub2e8\uc758 \uc8fc\uc7a5\uc744 \ubc1b\uc544\ub4e4\uc5ec\uc900 \uac83\uc73c\ub85c \ubcf4\uc778\ub2e4. \uc778\uc801 \ud53c\ud574 \uc0c1\ud669\uc744 \uc54c\ub9ac\uae30 \uc704\ud574 \uc785\uc6d0\ud574 \uc788\ub294 \ubcd1\uc2e4\uc744, \ubb3c\uc801 \ud53c\ud574 \uc0c1\ud669\uc744 \uc54c\ub9ac\uae30 \uc704\ud574 \uc9d1\uc548\uc744 \uacf5\uac1c\ud588\ub358 \uac83\uc774\ub2e4. \uc7ac\ud310\ubd80\uc5d0 \uacb0\ucf54 \uc30d\ubc29 \ud3ed\ud589 \uc774 \uc544\ub2c8\ub77c\ub294 \uc0ac\uc2e4\uc744 \uc785\uc99d\ud558\uae30 \uc704\ud574 \ucde8\ud588\ub358 \uc870\uce58\ub77c\uace0 \uc8fc\uc7a5\ud588\ub2e4. \uc774\ubc88 \uc0ac\uac74\uc744 \ud1b5\ud574 \uc720\uba85 \uc5f0\uc608\uc778\uc758 \uc778\uad8c\uc5d0 \ub300\ud574\uc11c\ub3c4 \uc2ec\uac01\ud55c \uace0\ub824\ub97c \ud574\uc57c \ud55c\ub2e4\uace0 \uc0dd\uac01\ud588\ub2e4. \ubc29\uc1a1\uad6d\uc5d0\uc11c \uc5f0\uc608\uc778\ub4e4\uc744 \ub9cc\ub098\uba74 '\ub0b4 \ubaa9\uc228\uc774 \uacb0\ucf54 \ub0b4 \ubaa9\uc228\uc774 \uc544\ub2c8\ub2e4'\ub77c\uace0 \ub9d0\ud558\ub294 \uacbd\uc6b0\ub97c \uc885\uc885 \ubd10\uc654\ub2e4. \ube44\ub85d \ub300\uc911\uc5d0\uac8c \uc0ac\uc0dd\ud65c\uc774 \uacf5\uac1c\ub420 \uc218\ubc16\uc5d0 \uc5c6\ub294 \uc5f0\uc608\uc778\uc758 \uc704\uce58\ub77c\ub3c4 \ud2b8\uc704\uc2a4\ud2b8 \uae40\uacfc \uac19\uc774 \uc74c\ub780 \uc0ac\uc774\ud2b8\uc5d0 \uc774\ub984\uc774 \ub3c4\uc6a9\ub418\uac70\ub098 \uc548\ud2f0 \uc138\ub825\uc73c\ub85c\ubd80\ud130 \ubb34\uc870\uac74\uc801\uc778 \uc778\uaca9\uc801 \ubaa8\uba78\uc744 \ubc1b\ub294 \uacbd\uc6b0 \uc774\ub97c \ubc29\uc5b4\ud560 \uc218 \uc788\ub294 \ucd5c\uc18c\ud55c\uc758 \uad8c\ub9ac\uac00 \uc788\uc5b4\uc57c \ud55c\ub2e4. \uac74\uc124\uc0ac\uac00 \uc774\ubbf8\uc9c0\uac00 \uc0dd\uba85\uc778 \uc720\uba85 \uc5f0\uc608\uc778\uc758 \uc57d\uc810\uc744 \uc774\uc6a9\ud574 30\uc5b5 \uc6d0\uc774\ub77c\ub294 \uc5b4\ub9c8\uc5b4\ub9c8\ud55c \uc18c\uc1a1\uc744 \uc81c\uae30\ud558\ub294 \uac83\uc740 \uc18c\uc1a1\uc758 \ub0a8\uc6a9\uc774\ub77c\ub294 \uc810\uc744 \uc7ac\ud310\ubd80\uc5d0 \uc801\uadf9 \ubc1d\ud614\ub2e4. \uc774 \uc0ac\uac74\uc774 \ub204\uad6c\ub4e0\uc9c0 \ub2f9\ud560 \uc218 \uc788\ub294 \uac00\uc815 \ud3ed\ub825\uc758 \ud53c\ud574\uc790, \uc989 \uc18c\uc1a1\uc758 \ud575\uc2ec\uc774\uc5c8\ub358 \uc778\uad8c \ubb38\uc81c\ub85c \uc778\uc9c0\ub418\uc9c0 \ubabb\ud55c \ucc44 \ub2e8\uc21c\ud788 \uc2a4\ud0c0\uc758 \uc0ac\uc0dd\ud65c\ub85c\ub9cc \ube44\ucdb0\uc9c0\uc9c0 \uc54a\uc744\uae4c \uac71\uc815\ub41c\ub2e4\"\ub77c\uace0 \ub9d0\ud588\ub2e4.", "sentence_answer": "\uc7ac\ud310\ubd80\uc5d0 \uacb0\ucf54 \uc30d\ubc29 \ud3ed\ud589 \uc774 \uc544\ub2c8\ub77c\ub294 \uc0ac\uc2e4\uc744 \uc785\uc99d\ud558\uae30 \uc704\ud574 \ucde8\ud588\ub358 \uc870\uce58\ub77c\uace0 \uc8fc\uc7a5\ud588\ub2e4.", "paragraph_id": "6344596-17-1", "paragraph_question": "question: \uc785\uc6d0\ud55c \ubcd1\uc2e4\uacfc \uc9d1\uc548\uc758 \ud53c\ud574 \uc0c1\ud669\uc744 \uacf5\uac1c\ud588\ub358 \uac83\uc740, \uc870\uc131\ubbfc\uc758 \uc5b4\ub5a4 \uc8fc\uc7a5\uc774 \uac70\uc9d3\uc774\ub77c\ub294 \uac83\uc744 \uc785\uc99d\ud558\uae30 \uc704\ud55c \uc870\uce58\uc600\ub098?, context: \ucd5c\uc9c4\uc2e4\uc774 2\uc2ec\uc5d0\uc11c \uc2b9\uc18c\ud558\uba74\uc11c \uc720\uba85 \uc5f0\uc608\uc778\uc758 \uc778\uad8c\uc5d0 \ub300\ud574 \uc0c8\ub86d\uac8c \ub3cc\uc544\ubcf4\ub294 \uacc4\uae30\uac00 \ub9c8\ub828\ub418\uc5c8\ub2e4. 2\uc2ec \ud310\uacb0\uc740 \ucd5c\uc9c4\uc2e4\uc740 \uac00\uc815\ud3ed\ub825\uc758 \ud53c\ud574\uc790\uc77c \ubfd0 \ud3ed\ub825 \ubd80\ubd84\uc774\ub098 \uc5b8\ub860 \ub300\uc751 \ubd80\ubd84\uc5d0 \uc788\uc5b4 \uacfc\uc2e4\uc774 \uc5c6\uc74c\uc744 \ubc1d\ud78c \ud310\ub2e8\uc774\uc5c8\uace0 \uba85\uc608\ud6fc\uc190 \ubd80\ubd84\uc5d0 \uc788\uc5b4\uc11c\ub3c4 \uc778\uacfc\uad00\uacc4\uac00 \uc778\uc815\ub418\uc9c0 \uc54a\uc74c\uc744 \uba85\ud655\ud788 \ud55c \ud310\uacb0\uc774\uc5c8\ub2e4. 2006\ub144 5\uc6d4 3\uc77c, 2\uc2ec \ud310\uacb0\uacfc \uad00\ub828\ud558\uc5ec \uac15\uc9c0\uc6d0\uc740 \"\ub9ce\uc740 \ubd84\ub4e4\uc774 \uc774 \uc0ac\uac74\uc758 \uc2e4\uccb4\ub97c \uc815\ud655\ud788 \ubaa8\ub974\uc2dc\ub294 \uac83 \uac19\ub2e4. \uc774\ubc88 \uc0ac\uac74\uc740 \uc5ec\uc131 \uc778\uad8c \ucc28\uc6d0\uc758 \ubb38\uc81c\uc600\uc73c\uba70 \uc720\uba85 \uc5f0\uc608\uc778\uc758 \uc778\uad8c \uc5ed\uc2dc \ubcf4\ud638\ubc1b\uc544\uc57c \ud55c\ub2e4\ub294 \uac83\uc744 \ub4dc\ub7ec\ub0b8 \uac83\uc774\ub2e4. \uc774\ubc88 \uc0ac\uac74\uc758 \ubcf8\uc9c8\uc740 \uc5ec\uc131 \uc778\uad8c \ubb38\uc81c\uc640 \uc5f0\uc608\uc778 \uc778\uad8c \ubb38\uc81c\uac00 \uacb0\ud569\ub41c \uac83\uc784\uc744 \uc9c1\uc2dc\ud574\uc57c\ud560 \uac83\uc774\ub2e4. \uc774\ubc88 \ud310\uacb0\uc744 \ud1b5\ud574 \uc5f0\uc608\uc778 \uc778\uad8c \ubb38\uc81c\ub3c4 \ud2b9\uc815 \uc5f0\uc608\uc778\uc5d0 \ub300\ud55c \uc120\ud638\ub3c4\uc640 \uc0c1\uad00\uc5c6\uc774 \uc778\uad8c \ucc28\uc6d0\uc5d0\uc11c \uc811\uadfc\ud560 \uc218 \uc788\ub294 \uc218\uc900 \ub192\uc740 \uc778\uc2dd\uc774 \ud655\uc0b0\ub410\uc73c\uba74 \uc88b\uaca0\ub2e4. \ub9cc\uc57d \ucd5c\uc9c4\uc2e4 \uc528\uac00 \ub9c8\uc57d\uc774\ub098 \uc74c\uc8fc\uc6b4\uc804, \ubcd1\uc5ed \uae30\ud53c \uac19\uc740 \ubcf8\uc778\uc758 \uc798\ubabb\uc73c\ub85c \uc778\ud574 \uae30\uc5c5 \uc774\ubbf8\uc9c0\ub97c \uc2e4\ucd94\ud588\ub2e4\uba74 \uae30\uc5c5 \uce21\uc774 \uc18c\uc1a1\uc744 \uc81c\uae30\ud560 \uc218 \uc788\uc5c8\uaca0\uc73c\ub098 \ucd5c\uc9c4\uc2e4 \uc528\ub294 \ud55c\ubc24\uc911\uc5d0 \uc220 \ub9c8\uc2e0 \ub0a8\ud3b8\uc5d0\uac8c \uc77c\ubc29\uc801 \ud3ed\ud589\uc744 \ub2f9\ud55c \uc5c4\uc5f0\ud55c \uac00\uc815 \ud3ed\ub825\uc758 \ud53c\ud574\uc790\uc600\ub2e4. \uc7ac\ud310\ubd80\uac00 \uc5ec\uc131 \uc778\uad8c \ubb38\uc81c\ub85c \uc811\uadfc\ud55c \ubcc0\ud638\uc778\ub2e8\uc758 \uc8fc\uc7a5\uc744 \ubc1b\uc544\ub4e4\uc5ec\uc900 \uac83\uc73c\ub85c \ubcf4\uc778\ub2e4. \uc778\uc801 \ud53c\ud574 \uc0c1\ud669\uc744 \uc54c\ub9ac\uae30 \uc704\ud574 \uc785\uc6d0\ud574 \uc788\ub294 \ubcd1\uc2e4\uc744, \ubb3c\uc801 \ud53c\ud574 \uc0c1\ud669\uc744 \uc54c\ub9ac\uae30 \uc704\ud574 \uc9d1\uc548\uc744 \uacf5\uac1c\ud588\ub358 \uac83\uc774\ub2e4. \uc7ac\ud310\ubd80\uc5d0 \uacb0\ucf54 \uc30d\ubc29 \ud3ed\ud589\uc774 \uc544\ub2c8\ub77c\ub294 \uc0ac\uc2e4\uc744 \uc785\uc99d\ud558\uae30 \uc704\ud574 \ucde8\ud588\ub358 \uc870\uce58\ub77c\uace0 \uc8fc\uc7a5\ud588\ub2e4. \uc774\ubc88 \uc0ac\uac74\uc744 \ud1b5\ud574 \uc720\uba85 \uc5f0\uc608\uc778\uc758 \uc778\uad8c\uc5d0 \ub300\ud574\uc11c\ub3c4 \uc2ec\uac01\ud55c \uace0\ub824\ub97c \ud574\uc57c \ud55c\ub2e4\uace0 \uc0dd\uac01\ud588\ub2e4. \ubc29\uc1a1\uad6d\uc5d0\uc11c \uc5f0\uc608\uc778\ub4e4\uc744 \ub9cc\ub098\uba74 '\ub0b4 \ubaa9\uc228\uc774 \uacb0\ucf54 \ub0b4 \ubaa9\uc228\uc774 \uc544\ub2c8\ub2e4'\ub77c\uace0 \ub9d0\ud558\ub294 \uacbd\uc6b0\ub97c \uc885\uc885 \ubd10\uc654\ub2e4. \ube44\ub85d \ub300\uc911\uc5d0\uac8c \uc0ac\uc0dd\ud65c\uc774 \uacf5\uac1c\ub420 \uc218\ubc16\uc5d0 \uc5c6\ub294 \uc5f0\uc608\uc778\uc758 \uc704\uce58\ub77c\ub3c4 \ud2b8\uc704\uc2a4\ud2b8 \uae40\uacfc \uac19\uc774 \uc74c\ub780 \uc0ac\uc774\ud2b8\uc5d0 \uc774\ub984\uc774 \ub3c4\uc6a9\ub418\uac70\ub098 \uc548\ud2f0 \uc138\ub825\uc73c\ub85c\ubd80\ud130 \ubb34\uc870\uac74\uc801\uc778 \uc778\uaca9\uc801 \ubaa8\uba78\uc744 \ubc1b\ub294 \uacbd\uc6b0 \uc774\ub97c \ubc29\uc5b4\ud560 \uc218 \uc788\ub294 \ucd5c\uc18c\ud55c\uc758 \uad8c\ub9ac\uac00 \uc788\uc5b4\uc57c \ud55c\ub2e4. \uac74\uc124\uc0ac\uac00 \uc774\ubbf8\uc9c0\uac00 \uc0dd\uba85\uc778 \uc720\uba85 \uc5f0\uc608\uc778\uc758 \uc57d\uc810\uc744 \uc774\uc6a9\ud574 30\uc5b5 \uc6d0\uc774\ub77c\ub294 \uc5b4\ub9c8\uc5b4\ub9c8\ud55c \uc18c\uc1a1\uc744 \uc81c\uae30\ud558\ub294 \uac83\uc740 \uc18c\uc1a1\uc758 \ub0a8\uc6a9\uc774\ub77c\ub294 \uc810\uc744 \uc7ac\ud310\ubd80\uc5d0 \uc801\uadf9 \ubc1d\ud614\ub2e4. \uc774 \uc0ac\uac74\uc774 \ub204\uad6c\ub4e0\uc9c0 \ub2f9\ud560 \uc218 \uc788\ub294 \uac00\uc815 \ud3ed\ub825\uc758 \ud53c\ud574\uc790, \uc989 \uc18c\uc1a1\uc758 \ud575\uc2ec\uc774\uc5c8\ub358 \uc778\uad8c \ubb38\uc81c\ub85c \uc778\uc9c0\ub418\uc9c0 \ubabb\ud55c \ucc44 \ub2e8\uc21c\ud788 \uc2a4\ud0c0\uc758 \uc0ac\uc0dd\ud65c\ub85c\ub9cc \ube44\ucdb0\uc9c0\uc9c0 \uc54a\uc744\uae4c \uac71\uc815\ub41c\ub2e4\"\ub77c\uace0 \ub9d0\ud588\ub2e4."} {"question": "\ubaa9\uc131\uc758 \uc704\uc131\uc744 \ubc1c\uacac\ud55c \uc0ac\ub78c\uc740?", "paragraph": "\uce7c\ub9ac\uc2a4\ud1a0(\uc601\uc5b4: Callisto, /k\u0259\u02c8l\u026asto\u028a/, \uadf8\ub9ac\uc2a4\uc5b4: \u039a\u03b1\u03bb\u03bb\u03b9\u03c3\u03c4\u03ce) \ub610\ub294 \ubaa9\uc131 IV\ub294 \ubaa9\uc131\uc758 \uc704\uc131\uc73c\ub85c, 1610\ub144 \uac08\ub9b4\ub808\uc624 \uac08\ub9b4\ub808\uc774\uac00 \ubc1c\uacac\ud588\ub2e4. \uce7c\ub9ac\uc2a4\ud1a0\ub294 \ud0dc\uc591\uacc4\uc5d0\uc11c \uc138 \ubc88\uc9f8\ub85c \ud070 \uc704\uc131\uc774\uace0, \ubaa9\uc131\uc758 \uc704\uc131 \uc911\uc5d0\uc11c\ub294 \uac00\ub2c8\uba54\ub370 \ub2e4\uc74c\uc73c\ub85c \ud06c\uba70, \ud589\uc131 \ubd84\ud654\uc728\uc740 \ud0dc\uc591\uacc4\uc5d0\uc11c \uc81c\uc77c \ub0ae\ub2e4. \uce7c\ub9ac\uc2a4\ud1a0\uc758 \uc9c0\ub984\uc740 4,821 km\uc774\uba70 \uc218\uc131\uc758 99%\uc5d0 \ub2ec\ud558\ub294 \ud06c\uae30\ub97c \uac00\uc84c\uc9c0\ub9cc \uc9c8\ub7c9\uc740 3\ubd84\uc758 1\ubc16\uc5d0 \ub418\uc9c0 \uc54a\ub294\ub2e4. \uada4\ub3c4 \ubc18\uacbd\uc740 1,880,000 km\uc774\uace0 \uac08\ub9b4\ub808\uc774 \uc704\uc131 \uc911\uc5d0\uc11c\ub294 \ubaa9\uc131\uc73c\ub85c\ubd80\ud130 \uc81c\uc77c \uba40\ub9ac \uc788\ub2e4. \uce7c\ub9ac\uc2a4\ud1a0\ub294 \ub0b4\ubd80 \uac08\ub9b4\ub808\uc774 \uc704\uc131\ub4e4(\uc774\uc624, \uc720\ub85c\ud30c, \uac00\ub2c8\uba54\ub370)\uacfc \uada4\ub3c4 \uacf5\uba85\uc744 \uc77c\uc73c\ud0a4\uc9c0 \uc54a\uace0, \ub530\ub77c\uc11c \uc870\uc11d \uac00\uc5f4\uc758 \uc815\ub3c4\uac00 \uc57d\ud558\ub2e4. \uce7c\ub9ac\uc2a4\ud1a0\ub294 \ubaa9\uc131\uc5d0 \uc870\uc11d \uace0\uc815\ub418\uc5b4 \uc788\uae30 \ub54c\ubb38\uc5d0, \ud56d\uc0c1 \uac19\uc740 \uba74\ub9cc \ubaa9\uc131\uc744 \ubc14\ub77c\ubcf4\uac8c \ub41c\ub2e4. \uce7c\ub9ac\uc2a4\ud1a0\ub294 \ub2e4\ub978 \ubaa9\uc131\uc758 \uc704\uc131\ub4e4\uc5d0 \ube44\ud574 \uc790\uae30\uad8c\uc774 \uc57d\ud55c\ub370, \uc774\ub294 \ubaa9\uc131\uc758 \ubc29\uc0ac\uc120\ub300\uc5d0\uc11c \uba40\ub9ac \ub5a8\uc5b4\uc838 \uc788\uae30 \ub54c\ubb38\uc774\ub2e4.", "answer": "\uac08\ub9b4\ub808\uc624 \uac08\ub9b4\ub808\uc774", "sentence": "1610\ub144 \uac08\ub9b4\ub808\uc624 \uac08\ub9b4\ub808\uc774 \uac00 \ubc1c\uacac\ud588\ub2e4.", "paragraph_sentence": "\uce7c\ub9ac\uc2a4\ud1a0(\uc601\uc5b4: Callisto, /k\u0259\u02c8l\u026asto\u028a/, \uadf8\ub9ac\uc2a4\uc5b4: \u039a\u03b1\u03bb\u03bb\u03b9\u03c3\u03c4\u03ce) \ub610\ub294 \ubaa9\uc131 IV\ub294 \ubaa9\uc131\uc758 \uc704\uc131\uc73c\ub85c, 1610\ub144 \uac08\ub9b4\ub808\uc624 \uac08\ub9b4\ub808\uc774 \uac00 \ubc1c\uacac\ud588\ub2e4. \uce7c\ub9ac\uc2a4\ud1a0\ub294 \ud0dc\uc591\uacc4\uc5d0\uc11c \uc138 \ubc88\uc9f8\ub85c \ud070 \uc704\uc131\uc774\uace0, \ubaa9\uc131\uc758 \uc704\uc131 \uc911\uc5d0\uc11c\ub294 \uac00\ub2c8\uba54\ub370 \ub2e4\uc74c\uc73c\ub85c \ud06c\uba70, \ud589\uc131 \ubd84\ud654\uc728\uc740 \ud0dc\uc591\uacc4\uc5d0\uc11c \uc81c\uc77c \ub0ae\ub2e4. \uce7c\ub9ac\uc2a4\ud1a0\uc758 \uc9c0\ub984\uc740 4,821 km\uc774\uba70 \uc218\uc131\uc758 99%\uc5d0 \ub2ec\ud558\ub294 \ud06c\uae30\ub97c \uac00\uc84c\uc9c0\ub9cc \uc9c8\ub7c9\uc740 3\ubd84\uc758 1\ubc16\uc5d0 \ub418\uc9c0 \uc54a\ub294\ub2e4. \uada4\ub3c4 \ubc18\uacbd\uc740 1,880,000 km\uc774\uace0 \uac08\ub9b4\ub808\uc774 \uc704\uc131 \uc911\uc5d0\uc11c\ub294 \ubaa9\uc131\uc73c\ub85c\ubd80\ud130 \uc81c\uc77c \uba40\ub9ac \uc788\ub2e4. \uce7c\ub9ac\uc2a4\ud1a0\ub294 \ub0b4\ubd80 \uac08\ub9b4\ub808\uc774 \uc704\uc131\ub4e4(\uc774\uc624, \uc720\ub85c\ud30c, \uac00\ub2c8\uba54\ub370)\uacfc \uada4\ub3c4 \uacf5\uba85\uc744 \uc77c\uc73c\ud0a4\uc9c0 \uc54a\uace0, \ub530\ub77c\uc11c \uc870\uc11d \uac00\uc5f4\uc758 \uc815\ub3c4\uac00 \uc57d\ud558\ub2e4. \uce7c\ub9ac\uc2a4\ud1a0\ub294 \ubaa9\uc131\uc5d0 \uc870\uc11d \uace0\uc815\ub418\uc5b4 \uc788\uae30 \ub54c\ubb38\uc5d0, \ud56d\uc0c1 \uac19\uc740 \uba74\ub9cc \ubaa9\uc131\uc744 \ubc14\ub77c\ubcf4\uac8c \ub41c\ub2e4. \uce7c\ub9ac\uc2a4\ud1a0\ub294 \ub2e4\ub978 \ubaa9\uc131\uc758 \uc704\uc131\ub4e4\uc5d0 \ube44\ud574 \uc790\uae30\uad8c\uc774 \uc57d\ud55c\ub370, \uc774\ub294 \ubaa9\uc131\uc758 \ubc29\uc0ac\uc120\ub300\uc5d0\uc11c \uba40\ub9ac \ub5a8\uc5b4\uc838 \uc788\uae30 \ub54c\ubb38\uc774\ub2e4.", "paragraph_answer": "\uce7c\ub9ac\uc2a4\ud1a0(\uc601\uc5b4: Callisto, /k\u0259\u02c8l\u026asto\u028a/, \uadf8\ub9ac\uc2a4\uc5b4: \u039a\u03b1\u03bb\u03bb\u03b9\u03c3\u03c4\u03ce) \ub610\ub294 \ubaa9\uc131 IV\ub294 \ubaa9\uc131\uc758 \uc704\uc131\uc73c\ub85c, 1610\ub144 \uac08\ub9b4\ub808\uc624 \uac08\ub9b4\ub808\uc774 \uac00 \ubc1c\uacac\ud588\ub2e4. \uce7c\ub9ac\uc2a4\ud1a0\ub294 \ud0dc\uc591\uacc4\uc5d0\uc11c \uc138 \ubc88\uc9f8\ub85c \ud070 \uc704\uc131\uc774\uace0, \ubaa9\uc131\uc758 \uc704\uc131 \uc911\uc5d0\uc11c\ub294 \uac00\ub2c8\uba54\ub370 \ub2e4\uc74c\uc73c\ub85c \ud06c\uba70, \ud589\uc131 \ubd84\ud654\uc728\uc740 \ud0dc\uc591\uacc4\uc5d0\uc11c \uc81c\uc77c \ub0ae\ub2e4. \uce7c\ub9ac\uc2a4\ud1a0\uc758 \uc9c0\ub984\uc740 4,821 km\uc774\uba70 \uc218\uc131\uc758 99%\uc5d0 \ub2ec\ud558\ub294 \ud06c\uae30\ub97c \uac00\uc84c\uc9c0\ub9cc \uc9c8\ub7c9\uc740 3\ubd84\uc758 1\ubc16\uc5d0 \ub418\uc9c0 \uc54a\ub294\ub2e4. \uada4\ub3c4 \ubc18\uacbd\uc740 1,880,000 km\uc774\uace0 \uac08\ub9b4\ub808\uc774 \uc704\uc131 \uc911\uc5d0\uc11c\ub294 \ubaa9\uc131\uc73c\ub85c\ubd80\ud130 \uc81c\uc77c \uba40\ub9ac \uc788\ub2e4. \uce7c\ub9ac\uc2a4\ud1a0\ub294 \ub0b4\ubd80 \uac08\ub9b4\ub808\uc774 \uc704\uc131\ub4e4(\uc774\uc624, \uc720\ub85c\ud30c, \uac00\ub2c8\uba54\ub370)\uacfc \uada4\ub3c4 \uacf5\uba85\uc744 \uc77c\uc73c\ud0a4\uc9c0 \uc54a\uace0, \ub530\ub77c\uc11c \uc870\uc11d \uac00\uc5f4\uc758 \uc815\ub3c4\uac00 \uc57d\ud558\ub2e4. \uce7c\ub9ac\uc2a4\ud1a0\ub294 \ubaa9\uc131\uc5d0 \uc870\uc11d \uace0\uc815\ub418\uc5b4 \uc788\uae30 \ub54c\ubb38\uc5d0, \ud56d\uc0c1 \uac19\uc740 \uba74\ub9cc \ubaa9\uc131\uc744 \ubc14\ub77c\ubcf4\uac8c \ub41c\ub2e4. \uce7c\ub9ac\uc2a4\ud1a0\ub294 \ub2e4\ub978 \ubaa9\uc131\uc758 \uc704\uc131\ub4e4\uc5d0 \ube44\ud574 \uc790\uae30\uad8c\uc774 \uc57d\ud55c\ub370, \uc774\ub294 \ubaa9\uc131\uc758 \ubc29\uc0ac\uc120\ub300\uc5d0\uc11c \uba40\ub9ac \ub5a8\uc5b4\uc838 \uc788\uae30 \ub54c\ubb38\uc774\ub2e4.", "sentence_answer": "1610\ub144 \uac08\ub9b4\ub808\uc624 \uac08\ub9b4\ub808\uc774 \uac00 \ubc1c\uacac\ud588\ub2e4.", "paragraph_id": "6561307-0-0", "paragraph_question": "question: \ubaa9\uc131\uc758 \uc704\uc131\uc744 \ubc1c\uacac\ud55c \uc0ac\ub78c\uc740?, context: \uce7c\ub9ac\uc2a4\ud1a0(\uc601\uc5b4: Callisto, /k\u0259\u02c8l\u026asto\u028a/, \uadf8\ub9ac\uc2a4\uc5b4: \u039a\u03b1\u03bb\u03bb\u03b9\u03c3\u03c4\u03ce) \ub610\ub294 \ubaa9\uc131 IV\ub294 \ubaa9\uc131\uc758 \uc704\uc131\uc73c\ub85c, 1610\ub144 \uac08\ub9b4\ub808\uc624 \uac08\ub9b4\ub808\uc774\uac00 \ubc1c\uacac\ud588\ub2e4. \uce7c\ub9ac\uc2a4\ud1a0\ub294 \ud0dc\uc591\uacc4\uc5d0\uc11c \uc138 \ubc88\uc9f8\ub85c \ud070 \uc704\uc131\uc774\uace0, \ubaa9\uc131\uc758 \uc704\uc131 \uc911\uc5d0\uc11c\ub294 \uac00\ub2c8\uba54\ub370 \ub2e4\uc74c\uc73c\ub85c \ud06c\uba70, \ud589\uc131 \ubd84\ud654\uc728\uc740 \ud0dc\uc591\uacc4\uc5d0\uc11c \uc81c\uc77c \ub0ae\ub2e4. \uce7c\ub9ac\uc2a4\ud1a0\uc758 \uc9c0\ub984\uc740 4,821 km\uc774\uba70 \uc218\uc131\uc758 99%\uc5d0 \ub2ec\ud558\ub294 \ud06c\uae30\ub97c \uac00\uc84c\uc9c0\ub9cc \uc9c8\ub7c9\uc740 3\ubd84\uc758 1\ubc16\uc5d0 \ub418\uc9c0 \uc54a\ub294\ub2e4. \uada4\ub3c4 \ubc18\uacbd\uc740 1,880,000 km\uc774\uace0 \uac08\ub9b4\ub808\uc774 \uc704\uc131 \uc911\uc5d0\uc11c\ub294 \ubaa9\uc131\uc73c\ub85c\ubd80\ud130 \uc81c\uc77c \uba40\ub9ac \uc788\ub2e4. \uce7c\ub9ac\uc2a4\ud1a0\ub294 \ub0b4\ubd80 \uac08\ub9b4\ub808\uc774 \uc704\uc131\ub4e4(\uc774\uc624, \uc720\ub85c\ud30c, \uac00\ub2c8\uba54\ub370)\uacfc \uada4\ub3c4 \uacf5\uba85\uc744 \uc77c\uc73c\ud0a4\uc9c0 \uc54a\uace0, \ub530\ub77c\uc11c \uc870\uc11d \uac00\uc5f4\uc758 \uc815\ub3c4\uac00 \uc57d\ud558\ub2e4. \uce7c\ub9ac\uc2a4\ud1a0\ub294 \ubaa9\uc131\uc5d0 \uc870\uc11d \uace0\uc815\ub418\uc5b4 \uc788\uae30 \ub54c\ubb38\uc5d0, \ud56d\uc0c1 \uac19\uc740 \uba74\ub9cc \ubaa9\uc131\uc744 \ubc14\ub77c\ubcf4\uac8c \ub41c\ub2e4. \uce7c\ub9ac\uc2a4\ud1a0\ub294 \ub2e4\ub978 \ubaa9\uc131\uc758 \uc704\uc131\ub4e4\uc5d0 \ube44\ud574 \uc790\uae30\uad8c\uc774 \uc57d\ud55c\ub370, \uc774\ub294 \ubaa9\uc131\uc758 \ubc29\uc0ac\uc120\ub300\uc5d0\uc11c \uba40\ub9ac \ub5a8\uc5b4\uc838 \uc788\uae30 \ub54c\ubb38\uc774\ub2e4."} {"question": "\ub178\uba3c \ubca0\uc21c\uacfc \uacb0\ud63c \ud55c \uc0ac\ub78c\uc740?", "paragraph": "1923\ub144 \uac00\uc744 \uc5d0\ub518\ubc84\ub7ec \uba85\ubb38\uac00\uc758 \ub538\uc778 \ud504\ub780\uc2dc\uc2a4 \ucea0\ubca8 \ud328\ub2c8\ub97c \ub9cc\ub0ac\uace0 \uc0ac\ub791\uc5d0 \ube60\uc838 \uc774\ub4ec\ud574 \ub7f0\ub358\uc5d0\uc11c \uacb0\ud63c\ud588\ub2e4. \ub450 \uc0ac\ub78c\uc740 \ubbf8\uad6d\uacfc \uce90\ub098\ub2e4\ub97c \uc804\uc804\ud558\ub2e4\uac00 \ubbf8\uc2dc\uac04 \uc8fc \ub514\ud2b8\ub85c\uc774\ud2b8\uc5d0 \uc815\ucc29\ud588\uace0 \uadf8\uacf3\uc5d0\uc11c \ucd5c\ucd08\uc758 \uac1c\uc778\ubcd1\uc6d0\uc744 \uac1c\uc5c5\ud588\ub2e4. \ub514\ud2b8\ub85c\uc774\ud2b8\uc5d0\uc11c \uadf8\ub294 \ub6f0\uc5b4\ub09c \uc218\uc220\uc2e4\ub825\uc73c\ub85c \ud070\ub3c8\uc744 \ubc8c\uc5c8\uace0 \uadf8 \ub3c8\uc744 \uac00\uc9c0\uace0 \uac00\ub09c\ud55c \ud658\uc790\ub4e4\uc744 \ub3cc\ubcf4\ub294 \ub370 \uc37c\ub2e4. \uadf8\ub294 \uc5f4\uc815\uc801\uc73c\ub85c \uac00\ub09c\ud55c \ud658\uc790\ub4e4\uc744 \ub3cc\ubcf4\uc558\ub294\ub370 \uadf8\ub9cc \ud3d0\uacb0\ud575\uc5d0 \uac78\ub9ac\uace0 \ub9d0\uc558\ub2e4. \ub2f9\uc2dc \ud3d0\uacb0\ud575 \uce58\ub8cc\uc218\uc900\uc740 \ud615\ud3b8\uc5c6\uc5c8\ub2e4. \ubca0\uc21c\ub3c4 \uadf8 \uc790\uc2e0\uc774 \uc758\uc0ac\uc774\uba74\uc11c \uadf8\uc800 \uc694\uc591\uc18c\uc5d0\uc11c \uc8fd\uc74c\uc744 \uae30\ub2e4\ub9ac\ub294 \uc2e0\uc138\uac00 \ub418\uc5c8\ub2e4. \uc6b0\uc5f0\ud788 \uc694\uc591\uc18c\uc5d0\uc11c \uc0c8\ub85c \uace0\uc548\ub41c \uae09\uc9c4\uc801\uc778 \ud3d0\uacb0\ud575 \uce58\ub8cc\ubc95\uc778 '\uae30\ud749\uce58\ub8cc\ubc95'\uc744 \uc54c\uac8c \ub418\uc5c8\uace0 \uadf8 \uce58\ub8cc\ubc95\uc5d0 \uc758\ud55c \uc2dc\uc220\uc744 \uc694\uad6c\ud558\uc5ec \ub2e4\uc2dc \uac74\uac15\uc774 \ud68c\ubcf5\ub418\uc5c8\ub2e4.", "answer": "\ud504\ub780\uc2dc\uc2a4 \ucea0\ubca8 \ud328\ub2c8", "sentence": "1923\ub144 \uac00\uc744 \uc5d0\ub518\ubc84\ub7ec \uba85\ubb38\uac00\uc758 \ub538\uc778 \ud504\ub780\uc2dc\uc2a4 \ucea0\ubca8 \ud328\ub2c8 \ub97c \ub9cc\ub0ac\uace0 \uc0ac\ub791\uc5d0 \ube60\uc838 \uc774\ub4ec\ud574 \ub7f0\ub358\uc5d0\uc11c \uacb0\ud63c\ud588\ub2e4.", "paragraph_sentence": " 1923\ub144 \uac00\uc744 \uc5d0\ub518\ubc84\ub7ec \uba85\ubb38\uac00\uc758 \ub538\uc778 \ud504\ub780\uc2dc\uc2a4 \ucea0\ubca8 \ud328\ub2c8 \ub97c \ub9cc\ub0ac\uace0 \uc0ac\ub791\uc5d0 \ube60\uc838 \uc774\ub4ec\ud574 \ub7f0\ub358\uc5d0\uc11c \uacb0\ud63c\ud588\ub2e4. \ub450 \uc0ac\ub78c\uc740 \ubbf8\uad6d\uacfc \uce90\ub098\ub2e4\ub97c \uc804\uc804\ud558\ub2e4\uac00 \ubbf8\uc2dc\uac04 \uc8fc \ub514\ud2b8\ub85c\uc774\ud2b8\uc5d0 \uc815\ucc29\ud588\uace0 \uadf8\uacf3\uc5d0\uc11c \ucd5c\ucd08\uc758 \uac1c\uc778\ubcd1\uc6d0\uc744 \uac1c\uc5c5\ud588\ub2e4. \ub514\ud2b8\ub85c\uc774\ud2b8\uc5d0\uc11c \uadf8\ub294 \ub6f0\uc5b4\ub09c \uc218\uc220\uc2e4\ub825\uc73c\ub85c \ud070\ub3c8\uc744 \ubc8c\uc5c8\uace0 \uadf8 \ub3c8\uc744 \uac00\uc9c0\uace0 \uac00\ub09c\ud55c \ud658\uc790\ub4e4\uc744 \ub3cc\ubcf4\ub294 \ub370 \uc37c\ub2e4. \uadf8\ub294 \uc5f4\uc815\uc801\uc73c\ub85c \uac00\ub09c\ud55c \ud658\uc790\ub4e4\uc744 \ub3cc\ubcf4\uc558\ub294\ub370 \uadf8\ub9cc \ud3d0\uacb0\ud575\uc5d0 \uac78\ub9ac\uace0 \ub9d0\uc558\ub2e4. \ub2f9\uc2dc \ud3d0\uacb0\ud575 \uce58\ub8cc\uc218\uc900\uc740 \ud615\ud3b8\uc5c6\uc5c8\ub2e4. \ubca0\uc21c\ub3c4 \uadf8 \uc790\uc2e0\uc774 \uc758\uc0ac\uc774\uba74\uc11c \uadf8\uc800 \uc694\uc591\uc18c\uc5d0\uc11c \uc8fd\uc74c\uc744 \uae30\ub2e4\ub9ac\ub294 \uc2e0\uc138\uac00 \ub418\uc5c8\ub2e4. \uc6b0\uc5f0\ud788 \uc694\uc591\uc18c\uc5d0\uc11c \uc0c8\ub85c \uace0\uc548\ub41c \uae09\uc9c4\uc801\uc778 \ud3d0\uacb0\ud575 \uce58\ub8cc\ubc95\uc778 '\uae30\ud749\uce58\ub8cc\ubc95'\uc744 \uc54c\uac8c \ub418\uc5c8\uace0 \uadf8 \uce58\ub8cc\ubc95\uc5d0 \uc758\ud55c \uc2dc\uc220\uc744 \uc694\uad6c\ud558\uc5ec \ub2e4\uc2dc \uac74\uac15\uc774 \ud68c\ubcf5\ub418\uc5c8\ub2e4.", "paragraph_answer": "1923\ub144 \uac00\uc744 \uc5d0\ub518\ubc84\ub7ec \uba85\ubb38\uac00\uc758 \ub538\uc778 \ud504\ub780\uc2dc\uc2a4 \ucea0\ubca8 \ud328\ub2c8 \ub97c \ub9cc\ub0ac\uace0 \uc0ac\ub791\uc5d0 \ube60\uc838 \uc774\ub4ec\ud574 \ub7f0\ub358\uc5d0\uc11c \uacb0\ud63c\ud588\ub2e4. \ub450 \uc0ac\ub78c\uc740 \ubbf8\uad6d\uacfc \uce90\ub098\ub2e4\ub97c \uc804\uc804\ud558\ub2e4\uac00 \ubbf8\uc2dc\uac04 \uc8fc \ub514\ud2b8\ub85c\uc774\ud2b8\uc5d0 \uc815\ucc29\ud588\uace0 \uadf8\uacf3\uc5d0\uc11c \ucd5c\ucd08\uc758 \uac1c\uc778\ubcd1\uc6d0\uc744 \uac1c\uc5c5\ud588\ub2e4. \ub514\ud2b8\ub85c\uc774\ud2b8\uc5d0\uc11c \uadf8\ub294 \ub6f0\uc5b4\ub09c \uc218\uc220\uc2e4\ub825\uc73c\ub85c \ud070\ub3c8\uc744 \ubc8c\uc5c8\uace0 \uadf8 \ub3c8\uc744 \uac00\uc9c0\uace0 \uac00\ub09c\ud55c \ud658\uc790\ub4e4\uc744 \ub3cc\ubcf4\ub294 \ub370 \uc37c\ub2e4. \uadf8\ub294 \uc5f4\uc815\uc801\uc73c\ub85c \uac00\ub09c\ud55c \ud658\uc790\ub4e4\uc744 \ub3cc\ubcf4\uc558\ub294\ub370 \uadf8\ub9cc \ud3d0\uacb0\ud575\uc5d0 \uac78\ub9ac\uace0 \ub9d0\uc558\ub2e4. \ub2f9\uc2dc \ud3d0\uacb0\ud575 \uce58\ub8cc\uc218\uc900\uc740 \ud615\ud3b8\uc5c6\uc5c8\ub2e4. \ubca0\uc21c\ub3c4 \uadf8 \uc790\uc2e0\uc774 \uc758\uc0ac\uc774\uba74\uc11c \uadf8\uc800 \uc694\uc591\uc18c\uc5d0\uc11c \uc8fd\uc74c\uc744 \uae30\ub2e4\ub9ac\ub294 \uc2e0\uc138\uac00 \ub418\uc5c8\ub2e4. \uc6b0\uc5f0\ud788 \uc694\uc591\uc18c\uc5d0\uc11c \uc0c8\ub85c \uace0\uc548\ub41c \uae09\uc9c4\uc801\uc778 \ud3d0\uacb0\ud575 \uce58\ub8cc\ubc95\uc778 '\uae30\ud749\uce58\ub8cc\ubc95'\uc744 \uc54c\uac8c \ub418\uc5c8\uace0 \uadf8 \uce58\ub8cc\ubc95\uc5d0 \uc758\ud55c \uc2dc\uc220\uc744 \uc694\uad6c\ud558\uc5ec \ub2e4\uc2dc \uac74\uac15\uc774 \ud68c\ubcf5\ub418\uc5c8\ub2e4.", "sentence_answer": "1923\ub144 \uac00\uc744 \uc5d0\ub518\ubc84\ub7ec \uba85\ubb38\uac00\uc758 \ub538\uc778 \ud504\ub780\uc2dc\uc2a4 \ucea0\ubca8 \ud328\ub2c8 \ub97c \ub9cc\ub0ac\uace0 \uc0ac\ub791\uc5d0 \ube60\uc838 \uc774\ub4ec\ud574 \ub7f0\ub358\uc5d0\uc11c \uacb0\ud63c\ud588\ub2e4.", "paragraph_id": "6488953-1-0", "paragraph_question": "question: \ub178\uba3c \ubca0\uc21c\uacfc \uacb0\ud63c \ud55c \uc0ac\ub78c\uc740?, context: 1923\ub144 \uac00\uc744 \uc5d0\ub518\ubc84\ub7ec \uba85\ubb38\uac00\uc758 \ub538\uc778 \ud504\ub780\uc2dc\uc2a4 \ucea0\ubca8 \ud328\ub2c8\ub97c \ub9cc\ub0ac\uace0 \uc0ac\ub791\uc5d0 \ube60\uc838 \uc774\ub4ec\ud574 \ub7f0\ub358\uc5d0\uc11c \uacb0\ud63c\ud588\ub2e4. \ub450 \uc0ac\ub78c\uc740 \ubbf8\uad6d\uacfc \uce90\ub098\ub2e4\ub97c \uc804\uc804\ud558\ub2e4\uac00 \ubbf8\uc2dc\uac04 \uc8fc \ub514\ud2b8\ub85c\uc774\ud2b8\uc5d0 \uc815\ucc29\ud588\uace0 \uadf8\uacf3\uc5d0\uc11c \ucd5c\ucd08\uc758 \uac1c\uc778\ubcd1\uc6d0\uc744 \uac1c\uc5c5\ud588\ub2e4. \ub514\ud2b8\ub85c\uc774\ud2b8\uc5d0\uc11c \uadf8\ub294 \ub6f0\uc5b4\ub09c \uc218\uc220\uc2e4\ub825\uc73c\ub85c \ud070\ub3c8\uc744 \ubc8c\uc5c8\uace0 \uadf8 \ub3c8\uc744 \uac00\uc9c0\uace0 \uac00\ub09c\ud55c \ud658\uc790\ub4e4\uc744 \ub3cc\ubcf4\ub294 \ub370 \uc37c\ub2e4. \uadf8\ub294 \uc5f4\uc815\uc801\uc73c\ub85c \uac00\ub09c\ud55c \ud658\uc790\ub4e4\uc744 \ub3cc\ubcf4\uc558\ub294\ub370 \uadf8\ub9cc \ud3d0\uacb0\ud575\uc5d0 \uac78\ub9ac\uace0 \ub9d0\uc558\ub2e4. \ub2f9\uc2dc \ud3d0\uacb0\ud575 \uce58\ub8cc\uc218\uc900\uc740 \ud615\ud3b8\uc5c6\uc5c8\ub2e4. \ubca0\uc21c\ub3c4 \uadf8 \uc790\uc2e0\uc774 \uc758\uc0ac\uc774\uba74\uc11c \uadf8\uc800 \uc694\uc591\uc18c\uc5d0\uc11c \uc8fd\uc74c\uc744 \uae30\ub2e4\ub9ac\ub294 \uc2e0\uc138\uac00 \ub418\uc5c8\ub2e4. \uc6b0\uc5f0\ud788 \uc694\uc591\uc18c\uc5d0\uc11c \uc0c8\ub85c \uace0\uc548\ub41c \uae09\uc9c4\uc801\uc778 \ud3d0\uacb0\ud575 \uce58\ub8cc\ubc95\uc778 '\uae30\ud749\uce58\ub8cc\ubc95'\uc744 \uc54c\uac8c \ub418\uc5c8\uace0 \uadf8 \uce58\ub8cc\ubc95\uc5d0 \uc758\ud55c \uc2dc\uc220\uc744 \uc694\uad6c\ud558\uc5ec \ub2e4\uc2dc \uac74\uac15\uc774 \ud68c\ubcf5\ub418\uc5c8\ub2e4."} {"question": "\uc9c4\uc6d0\uc5ed \uc11c\ucabd \uc9c0\uac01 \uae4a\uc740 \uacf3\uc5d0\uc11c \ubc1c\uc0dd\ud558\uace0 \uc788\ub294 \uc5ec\uc9c4\uc758 \uc885\ub958\ub294?", "paragraph": "\ubcf8\uc9c4 \uc774\ud6c4 \uc77c\uc5b4\ub098\uace0 \uc788\ub294 \uc5ec\uc9c4\uc740 \uc774\uc640\ud14c \ud604 \uc55e\ubc14\ub2e4\ubd80\ud130 \uc774\ubc14\ub77c\ud0a4 \ud604 \uc55e\ubc14\ub2e4\uae4c\uc9c0 \ub3d9\uc11c 200km, \ub0a8\ubd81 500km\uc758 \uc77c\ubcf8 \ud574\uad6c \ub3d9\ucabd\uc758 \uc601\uc5ed\uc5d0\uc11c \uc77c\uc5b4\ub098\uace0 \uc788\ub2e4. \uc5ec\uc9c4\uc758 \ub300\ubd80\ubd84\uc740 \ubcf8\uc9c4\uacfc \ube44\uc2b7\ud55c \ud574\uad6c\ud615 \uc9c0\uc9c4(\ud310 \uacbd\uacc4\uc5d0\uc11c\uc758 \uc9c0\uc9c4)\uc774\uc9c0\ub9cc, \uc9c4\uc6d0\uc5ed \uc11c\ucabd \uc9c0\uac01\uc758 \uc589\uc740 \uacf3\uc5d0\uc11c\ub294 \uc721\uc9c0\uc9c0\uac01 \ub0b4 \uc9c0\uc9c4(\ub0b4\ub959 \uc9c1\ud558\ud615 \uc9c0\uc9c4, \ub300\ub959\ud310 \ub0b4 \uc9c0\uc9c4)\uc774 \uc77c\uc5b4\ub098\uace0, \uc9c4\uc6d0\uc5ed \uc11c\ucabd \uc9c0\uac01 \uae4a\uc740 \uacf3\uc5d0\uc11c\ub294 \uc2ec\ubc1c\uc9c0\uc9c4\uc774 \uc77c\uc5b4\ub098\uace0 \uc788\uc73c\uba70, \ud574\uad6c \ub3d9\ucabd \uc9c0\uc5ed\uc5d0\uc11c\ub294 \ud574\uc591\uc9c0\uac01 \ub0b4 \uc9c0\uc9c4(\uc544\uc6c3\ud130\ub77c\uc774\uc988 \uc9c0\uc9c4)\ub3c4 \uc77c\uc5b4\ub098\uace0 \uc788\ub2e4. \ud2b9\ud788, \uc77c\ubcf8 \ud574\uad6c \ub3d9\ucabd \uc735\uae30 \uc9c0\ub300\uc5d0\uc11c\ub294 \ubcf8\uc9c4\uc73c\ub85c \uc778\ud55c \uc9c0\uac01\ubcc0\ub3d9\uc758 \uc601\ud5a5\uc73c\ub85c \uc5ec\uc9c4\uc774 \uc77c\uc5b4\ub0a0 \uacbd\uc6b0 \ud6c4\uc220\ud560 \ub0b4\uc6a9\ucc98\ub7fc \uc9c0\uc9c4\uc758 \uaddc\ubaa8\uac00 \ub354 \ucee4\uc9c8 \uc218\ub3c4 \uc788\ub2e4\ub294 \uc8fc\uc7a5\ub3c4 \ub098\uc624\uace0 \uc788\ub2e4. \uc774 \ubc16\uc5d0\ub3c4, \uc9c4\uc6d0\uc5ed\uc5d0\uc11c \uc218\ubc31 km \ub5a8\uc5b4\uc9c4 \uacf3\uc5d0\uc11c\ub3c4 \uaddc\ubaa8 M6 \uc774\uc0c1\uc758 \ube44\uad50\uc801 \ud070 \uc9c0\uc9c4\uc774 \uc77c\uc5b4\ub098\uace0 \uc788\uc73c\uba70, \uac70\ub300\uc9c0\uc9c4\uc73c\ub85c \uc778\ud55c \uc9c0\uac01\ubcc0\ub3d9\uc73c\ub85c \uc5ec\ub7ec \uacf3\uc5d0\uc11c \uc9c0\uc9c4\uc774 \uc77c\uc5b4\ub0a0 \uc218\ub3c4 \uc788\ub2e4\ub294 \uc8fc\uc7a5\ub3c4 \uc788\ub2e4.", "answer": "\uc2ec\ubc1c\uc9c0\uc9c4", "sentence": "\uc5ec\uc9c4\uc758 \ub300\ubd80\ubd84\uc740 \ubcf8\uc9c4\uacfc \ube44\uc2b7\ud55c \ud574\uad6c\ud615 \uc9c0\uc9c4(\ud310 \uacbd\uacc4\uc5d0\uc11c\uc758 \uc9c0\uc9c4)\uc774\uc9c0\ub9cc, \uc9c4\uc6d0\uc5ed \uc11c\ucabd \uc9c0\uac01\uc758 \uc589\uc740 \uacf3\uc5d0\uc11c\ub294 \uc721\uc9c0\uc9c0\uac01 \ub0b4 \uc9c0\uc9c4(\ub0b4\ub959 \uc9c1\ud558\ud615 \uc9c0\uc9c4, \ub300\ub959\ud310 \ub0b4 \uc9c0\uc9c4)\uc774 \uc77c\uc5b4\ub098\uace0, \uc9c4\uc6d0\uc5ed \uc11c\ucabd \uc9c0\uac01 \uae4a\uc740 \uacf3\uc5d0\uc11c\ub294 \uc2ec\ubc1c\uc9c0\uc9c4 \uc774 \uc77c\uc5b4\ub098\uace0 \uc788\uc73c\uba70, \ud574\uad6c \ub3d9\ucabd \uc9c0\uc5ed\uc5d0\uc11c\ub294 \ud574\uc591\uc9c0\uac01 \ub0b4 \uc9c0\uc9c4(\uc544\uc6c3\ud130\ub77c\uc774\uc988 \uc9c0\uc9c4)\ub3c4 \uc77c\uc5b4\ub098\uace0 \uc788\ub2e4.", "paragraph_sentence": "\ubcf8\uc9c4 \uc774\ud6c4 \uc77c\uc5b4\ub098\uace0 \uc788\ub294 \uc5ec\uc9c4\uc740 \uc774\uc640\ud14c \ud604 \uc55e\ubc14\ub2e4\ubd80\ud130 \uc774\ubc14\ub77c\ud0a4 \ud604 \uc55e\ubc14\ub2e4\uae4c\uc9c0 \ub3d9\uc11c 200km, \ub0a8\ubd81 500km\uc758 \uc77c\ubcf8 \ud574\uad6c \ub3d9\ucabd\uc758 \uc601\uc5ed\uc5d0\uc11c \uc77c\uc5b4\ub098\uace0 \uc788\ub2e4. \uc5ec\uc9c4\uc758 \ub300\ubd80\ubd84\uc740 \ubcf8\uc9c4\uacfc \ube44\uc2b7\ud55c \ud574\uad6c\ud615 \uc9c0\uc9c4(\ud310 \uacbd\uacc4\uc5d0\uc11c\uc758 \uc9c0\uc9c4)\uc774\uc9c0\ub9cc, \uc9c4\uc6d0\uc5ed \uc11c\ucabd \uc9c0\uac01\uc758 \uc589\uc740 \uacf3\uc5d0\uc11c\ub294 \uc721\uc9c0\uc9c0\uac01 \ub0b4 \uc9c0\uc9c4(\ub0b4\ub959 \uc9c1\ud558\ud615 \uc9c0\uc9c4, \ub300\ub959\ud310 \ub0b4 \uc9c0\uc9c4)\uc774 \uc77c\uc5b4\ub098\uace0, \uc9c4\uc6d0\uc5ed \uc11c\ucabd \uc9c0\uac01 \uae4a\uc740 \uacf3\uc5d0\uc11c\ub294 \uc2ec\ubc1c\uc9c0\uc9c4 \uc774 \uc77c\uc5b4\ub098\uace0 \uc788\uc73c\uba70, \ud574\uad6c \ub3d9\ucabd \uc9c0\uc5ed\uc5d0\uc11c\ub294 \ud574\uc591\uc9c0\uac01 \ub0b4 \uc9c0\uc9c4(\uc544\uc6c3\ud130\ub77c\uc774\uc988 \uc9c0\uc9c4)\ub3c4 \uc77c\uc5b4\ub098\uace0 \uc788\ub2e4. \ud2b9\ud788, \uc77c\ubcf8 \ud574\uad6c \ub3d9\ucabd \uc735\uae30 \uc9c0\ub300\uc5d0\uc11c\ub294 \ubcf8\uc9c4\uc73c\ub85c \uc778\ud55c \uc9c0\uac01\ubcc0\ub3d9\uc758 \uc601\ud5a5\uc73c\ub85c \uc5ec\uc9c4\uc774 \uc77c\uc5b4\ub0a0 \uacbd\uc6b0 \ud6c4\uc220\ud560 \ub0b4\uc6a9\ucc98\ub7fc \uc9c0\uc9c4\uc758 \uaddc\ubaa8\uac00 \ub354 \ucee4\uc9c8 \uc218\ub3c4 \uc788\ub2e4\ub294 \uc8fc\uc7a5\ub3c4 \ub098\uc624\uace0 \uc788\ub2e4. \uc774 \ubc16\uc5d0\ub3c4, \uc9c4\uc6d0\uc5ed\uc5d0\uc11c \uc218\ubc31 km \ub5a8\uc5b4\uc9c4 \uacf3\uc5d0\uc11c\ub3c4 \uaddc\ubaa8 M6 \uc774\uc0c1\uc758 \ube44\uad50\uc801 \ud070 \uc9c0\uc9c4\uc774 \uc77c\uc5b4\ub098\uace0 \uc788\uc73c\uba70, \uac70\ub300\uc9c0\uc9c4\uc73c\ub85c \uc778\ud55c \uc9c0\uac01\ubcc0\ub3d9\uc73c\ub85c \uc5ec\ub7ec \uacf3\uc5d0\uc11c \uc9c0\uc9c4\uc774 \uc77c\uc5b4\ub0a0 \uc218\ub3c4 \uc788\ub2e4\ub294 \uc8fc\uc7a5\ub3c4 \uc788\ub2e4.", "paragraph_answer": "\ubcf8\uc9c4 \uc774\ud6c4 \uc77c\uc5b4\ub098\uace0 \uc788\ub294 \uc5ec\uc9c4\uc740 \uc774\uc640\ud14c \ud604 \uc55e\ubc14\ub2e4\ubd80\ud130 \uc774\ubc14\ub77c\ud0a4 \ud604 \uc55e\ubc14\ub2e4\uae4c\uc9c0 \ub3d9\uc11c 200km, \ub0a8\ubd81 500km\uc758 \uc77c\ubcf8 \ud574\uad6c \ub3d9\ucabd\uc758 \uc601\uc5ed\uc5d0\uc11c \uc77c\uc5b4\ub098\uace0 \uc788\ub2e4. \uc5ec\uc9c4\uc758 \ub300\ubd80\ubd84\uc740 \ubcf8\uc9c4\uacfc \ube44\uc2b7\ud55c \ud574\uad6c\ud615 \uc9c0\uc9c4(\ud310 \uacbd\uacc4\uc5d0\uc11c\uc758 \uc9c0\uc9c4)\uc774\uc9c0\ub9cc, \uc9c4\uc6d0\uc5ed \uc11c\ucabd \uc9c0\uac01\uc758 \uc589\uc740 \uacf3\uc5d0\uc11c\ub294 \uc721\uc9c0\uc9c0\uac01 \ub0b4 \uc9c0\uc9c4(\ub0b4\ub959 \uc9c1\ud558\ud615 \uc9c0\uc9c4, \ub300\ub959\ud310 \ub0b4 \uc9c0\uc9c4)\uc774 \uc77c\uc5b4\ub098\uace0, \uc9c4\uc6d0\uc5ed \uc11c\ucabd \uc9c0\uac01 \uae4a\uc740 \uacf3\uc5d0\uc11c\ub294 \uc2ec\ubc1c\uc9c0\uc9c4 \uc774 \uc77c\uc5b4\ub098\uace0 \uc788\uc73c\uba70, \ud574\uad6c \ub3d9\ucabd \uc9c0\uc5ed\uc5d0\uc11c\ub294 \ud574\uc591\uc9c0\uac01 \ub0b4 \uc9c0\uc9c4(\uc544\uc6c3\ud130\ub77c\uc774\uc988 \uc9c0\uc9c4)\ub3c4 \uc77c\uc5b4\ub098\uace0 \uc788\ub2e4. \ud2b9\ud788, \uc77c\ubcf8 \ud574\uad6c \ub3d9\ucabd \uc735\uae30 \uc9c0\ub300\uc5d0\uc11c\ub294 \ubcf8\uc9c4\uc73c\ub85c \uc778\ud55c \uc9c0\uac01\ubcc0\ub3d9\uc758 \uc601\ud5a5\uc73c\ub85c \uc5ec\uc9c4\uc774 \uc77c\uc5b4\ub0a0 \uacbd\uc6b0 \ud6c4\uc220\ud560 \ub0b4\uc6a9\ucc98\ub7fc \uc9c0\uc9c4\uc758 \uaddc\ubaa8\uac00 \ub354 \ucee4\uc9c8 \uc218\ub3c4 \uc788\ub2e4\ub294 \uc8fc\uc7a5\ub3c4 \ub098\uc624\uace0 \uc788\ub2e4. \uc774 \ubc16\uc5d0\ub3c4, \uc9c4\uc6d0\uc5ed\uc5d0\uc11c \uc218\ubc31 km \ub5a8\uc5b4\uc9c4 \uacf3\uc5d0\uc11c\ub3c4 \uaddc\ubaa8 M6 \uc774\uc0c1\uc758 \ube44\uad50\uc801 \ud070 \uc9c0\uc9c4\uc774 \uc77c\uc5b4\ub098\uace0 \uc788\uc73c\uba70, \uac70\ub300\uc9c0\uc9c4\uc73c\ub85c \uc778\ud55c \uc9c0\uac01\ubcc0\ub3d9\uc73c\ub85c \uc5ec\ub7ec \uacf3\uc5d0\uc11c \uc9c0\uc9c4\uc774 \uc77c\uc5b4\ub0a0 \uc218\ub3c4 \uc788\ub2e4\ub294 \uc8fc\uc7a5\ub3c4 \uc788\ub2e4.", "sentence_answer": "\uc5ec\uc9c4\uc758 \ub300\ubd80\ubd84\uc740 \ubcf8\uc9c4\uacfc \ube44\uc2b7\ud55c \ud574\uad6c\ud615 \uc9c0\uc9c4(\ud310 \uacbd\uacc4\uc5d0\uc11c\uc758 \uc9c0\uc9c4)\uc774\uc9c0\ub9cc, \uc9c4\uc6d0\uc5ed \uc11c\ucabd \uc9c0\uac01\uc758 \uc589\uc740 \uacf3\uc5d0\uc11c\ub294 \uc721\uc9c0\uc9c0\uac01 \ub0b4 \uc9c0\uc9c4(\ub0b4\ub959 \uc9c1\ud558\ud615 \uc9c0\uc9c4, \ub300\ub959\ud310 \ub0b4 \uc9c0\uc9c4)\uc774 \uc77c\uc5b4\ub098\uace0, \uc9c4\uc6d0\uc5ed \uc11c\ucabd \uc9c0\uac01 \uae4a\uc740 \uacf3\uc5d0\uc11c\ub294 \uc2ec\ubc1c\uc9c0\uc9c4 \uc774 \uc77c\uc5b4\ub098\uace0 \uc788\uc73c\uba70, \ud574\uad6c \ub3d9\ucabd \uc9c0\uc5ed\uc5d0\uc11c\ub294 \ud574\uc591\uc9c0\uac01 \ub0b4 \uc9c0\uc9c4(\uc544\uc6c3\ud130\ub77c\uc774\uc988 \uc9c0\uc9c4)\ub3c4 \uc77c\uc5b4\ub098\uace0 \uc788\ub2e4.", "paragraph_id": "6492086-21-1", "paragraph_question": "question: \uc9c4\uc6d0\uc5ed \uc11c\ucabd \uc9c0\uac01 \uae4a\uc740 \uacf3\uc5d0\uc11c \ubc1c\uc0dd\ud558\uace0 \uc788\ub294 \uc5ec\uc9c4\uc758 \uc885\ub958\ub294?, context: \ubcf8\uc9c4 \uc774\ud6c4 \uc77c\uc5b4\ub098\uace0 \uc788\ub294 \uc5ec\uc9c4\uc740 \uc774\uc640\ud14c \ud604 \uc55e\ubc14\ub2e4\ubd80\ud130 \uc774\ubc14\ub77c\ud0a4 \ud604 \uc55e\ubc14\ub2e4\uae4c\uc9c0 \ub3d9\uc11c 200km, \ub0a8\ubd81 500km\uc758 \uc77c\ubcf8 \ud574\uad6c \ub3d9\ucabd\uc758 \uc601\uc5ed\uc5d0\uc11c \uc77c\uc5b4\ub098\uace0 \uc788\ub2e4. \uc5ec\uc9c4\uc758 \ub300\ubd80\ubd84\uc740 \ubcf8\uc9c4\uacfc \ube44\uc2b7\ud55c \ud574\uad6c\ud615 \uc9c0\uc9c4(\ud310 \uacbd\uacc4\uc5d0\uc11c\uc758 \uc9c0\uc9c4)\uc774\uc9c0\ub9cc, \uc9c4\uc6d0\uc5ed \uc11c\ucabd \uc9c0\uac01\uc758 \uc589\uc740 \uacf3\uc5d0\uc11c\ub294 \uc721\uc9c0\uc9c0\uac01 \ub0b4 \uc9c0\uc9c4(\ub0b4\ub959 \uc9c1\ud558\ud615 \uc9c0\uc9c4, \ub300\ub959\ud310 \ub0b4 \uc9c0\uc9c4)\uc774 \uc77c\uc5b4\ub098\uace0, \uc9c4\uc6d0\uc5ed \uc11c\ucabd \uc9c0\uac01 \uae4a\uc740 \uacf3\uc5d0\uc11c\ub294 \uc2ec\ubc1c\uc9c0\uc9c4\uc774 \uc77c\uc5b4\ub098\uace0 \uc788\uc73c\uba70, \ud574\uad6c \ub3d9\ucabd \uc9c0\uc5ed\uc5d0\uc11c\ub294 \ud574\uc591\uc9c0\uac01 \ub0b4 \uc9c0\uc9c4(\uc544\uc6c3\ud130\ub77c\uc774\uc988 \uc9c0\uc9c4)\ub3c4 \uc77c\uc5b4\ub098\uace0 \uc788\ub2e4. \ud2b9\ud788, \uc77c\ubcf8 \ud574\uad6c \ub3d9\ucabd \uc735\uae30 \uc9c0\ub300\uc5d0\uc11c\ub294 \ubcf8\uc9c4\uc73c\ub85c \uc778\ud55c \uc9c0\uac01\ubcc0\ub3d9\uc758 \uc601\ud5a5\uc73c\ub85c \uc5ec\uc9c4\uc774 \uc77c\uc5b4\ub0a0 \uacbd\uc6b0 \ud6c4\uc220\ud560 \ub0b4\uc6a9\ucc98\ub7fc \uc9c0\uc9c4\uc758 \uaddc\ubaa8\uac00 \ub354 \ucee4\uc9c8 \uc218\ub3c4 \uc788\ub2e4\ub294 \uc8fc\uc7a5\ub3c4 \ub098\uc624\uace0 \uc788\ub2e4. \uc774 \ubc16\uc5d0\ub3c4, \uc9c4\uc6d0\uc5ed\uc5d0\uc11c \uc218\ubc31 km \ub5a8\uc5b4\uc9c4 \uacf3\uc5d0\uc11c\ub3c4 \uaddc\ubaa8 M6 \uc774\uc0c1\uc758 \ube44\uad50\uc801 \ud070 \uc9c0\uc9c4\uc774 \uc77c\uc5b4\ub098\uace0 \uc788\uc73c\uba70, \uac70\ub300\uc9c0\uc9c4\uc73c\ub85c \uc778\ud55c \uc9c0\uac01\ubcc0\ub3d9\uc73c\ub85c \uc5ec\ub7ec \uacf3\uc5d0\uc11c \uc9c0\uc9c4\uc774 \uc77c\uc5b4\ub0a0 \uc218\ub3c4 \uc788\ub2e4\ub294 \uc8fc\uc7a5\ub3c4 \uc788\ub2e4."} {"question": "KBO\uc5d0\uc11c \ubc15\ud604\uc900\uacfc \uae40\uc131\ud604\uc758 \uc2b9\ubd80\uc870\uc791\uc5d0 \ub300\ud574 \uadf8\ub4e4\uc5d0\uac8c \uac00\ud55c \uc870\uce58\ub294?", "paragraph": "2012\ub144 \ud55c\uad6d\ud504\ub85c\uc57c\uad6c \uc2b9\ubd80\uc870\uc791 \uc0ac\uac74\uc740 KBO \ub9ac\uadf8\uc5d0\uc11c \ubc8c\uc5b4\uc9c4 \uc2b9\ubd80\uc870\uc791 \uc0ac\uac74\uc774\ub2e4. \uc77c\ubd80 \uc120\uc218\uac00 \ub300\uac00\ub97c \uc9c0\ubd88\ubc1b\uace0 \uacbd\uae30 \ub0b4\uc6a9\uacfc \uc2b9\ubd80\ub97c \uc870\uc791\ud558\uc600\ub2e4. 2012\ub144 \ub300\ud55c\ubbfc\uad6d \ud504\ub85c \ucd95\uad6c\uc758 K\ub9ac\uadf8 \uc2b9\ubd80\uc870\uc791 \uc0ac\uac74\uacfc \ud504\ub85c \ubc30\uad6c\uc758 V-\ub9ac\uadf8 \uc2b9\ubd80\uc870\uc791 \uc0ac\uac74\uc758 \uc804\ubaa8\uac00 \ubc1d\ud600\uc9c0\uae30 \uc2dc\uc791\ud558\uba74\uc11c \uc0ac\uac74\uc5d0 \uc5f0\ub8e8\ub41c \ud55c \ube0c\ub85c\ucee4\uac00 \ud55c\uad6d\ud504\ub85c\uc57c\uad6c\uc5d0\uc11c\ub3c4 \uc2b9\ubd80\uc870\uc791\uc774 \uc774\ub8e8\uc5b4\uc84c\ub2e4\uace0 \ubc1d\ud788\uba74\uc11c \ubcf8\uaca9\uc801\uc73c\ub85c \ud504\ub85c \uc57c\uad6c\uc5d0\uc11c\ub3c4 \uc2b9\ubd80\uc870\uc791 \uc0ac\uac74\uc774 \uc218\uba74 \uc704\ub85c \ub5a0\uc624\ub974\uac8c \ub418\uc5c8\ub2e4. \uc774 \uacfc\uc815\uc5d0\uc11c \ub9ce\uc740 \uc120\uc218\ub4e4\uc774 \uc2b9\ubd80\uc870\uc791\uc744 \ud588\ub2e4\ub294 \uc758\uc2ec\uc744 \ubc1b\uc558\uace0, \ubb38\uc131\ud604, \ubc15\ud604\uc900, \uae40\uc131\ud604\uc774 \uac80\ucc30\uc5d0 \uc18c\ud658\ub418\uc5b4 \uc870\uc0ac\ub97c \ubc1b\uc558\ub2e4. \uc774\ub4e4 \uc911 \ubc15\ud604\uc900\uacfc \uae40\uc131\ud604\uc740 \uc2b9\ubd80\uc870\uc791 \ud610\uc758\uac00 \uc778\uc815\ub418\uc5b4 \uac01\uac01 \ubd88\uad6c\uc18d, \uad6c\uc18d \uae30\uc18c\ub418\uc5c8\uace0, KBO\ub294 \uc774 \ub450 \uc120\uc218\ub97c \uc601\uad6c\uc81c\uba85\ud588\ub2e4.", "answer": "\uc601\uad6c\uc81c\uba85", "sentence": "KBO\ub294 \uc774 \ub450 \uc120\uc218\ub97c \uc601\uad6c\uc81c\uba85 \ud588\ub2e4.", "paragraph_sentence": "2012\ub144 \ud55c\uad6d\ud504\ub85c\uc57c\uad6c \uc2b9\ubd80\uc870\uc791 \uc0ac\uac74\uc740 KBO \ub9ac\uadf8\uc5d0\uc11c \ubc8c\uc5b4\uc9c4 \uc2b9\ubd80\uc870\uc791 \uc0ac\uac74\uc774\ub2e4. \uc77c\ubd80 \uc120\uc218\uac00 \ub300\uac00\ub97c \uc9c0\ubd88\ubc1b\uace0 \uacbd\uae30 \ub0b4\uc6a9\uacfc \uc2b9\ubd80\ub97c \uc870\uc791\ud558\uc600\ub2e4. 2012\ub144 \ub300\ud55c\ubbfc\uad6d \ud504\ub85c \ucd95\uad6c\uc758 K\ub9ac\uadf8 \uc2b9\ubd80\uc870\uc791 \uc0ac\uac74\uacfc \ud504\ub85c \ubc30\uad6c\uc758 V-\ub9ac\uadf8 \uc2b9\ubd80\uc870\uc791 \uc0ac\uac74\uc758 \uc804\ubaa8\uac00 \ubc1d\ud600\uc9c0\uae30 \uc2dc\uc791\ud558\uba74\uc11c \uc0ac\uac74\uc5d0 \uc5f0\ub8e8\ub41c \ud55c \ube0c\ub85c\ucee4\uac00 \ud55c\uad6d\ud504\ub85c\uc57c\uad6c\uc5d0\uc11c\ub3c4 \uc2b9\ubd80\uc870\uc791\uc774 \uc774\ub8e8\uc5b4\uc84c\ub2e4\uace0 \ubc1d\ud788\uba74\uc11c \ubcf8\uaca9\uc801\uc73c\ub85c \ud504\ub85c \uc57c\uad6c\uc5d0\uc11c\ub3c4 \uc2b9\ubd80\uc870\uc791 \uc0ac\uac74\uc774 \uc218\uba74 \uc704\ub85c \ub5a0\uc624\ub974\uac8c \ub418\uc5c8\ub2e4. \uc774 \uacfc\uc815\uc5d0\uc11c \ub9ce\uc740 \uc120\uc218\ub4e4\uc774 \uc2b9\ubd80\uc870\uc791\uc744 \ud588\ub2e4\ub294 \uc758\uc2ec\uc744 \ubc1b\uc558\uace0, \ubb38\uc131\ud604, \ubc15\ud604\uc900, \uae40\uc131\ud604\uc774 \uac80\ucc30\uc5d0 \uc18c\ud658\ub418\uc5b4 \uc870\uc0ac\ub97c \ubc1b\uc558\ub2e4. \uc774\ub4e4 \uc911 \ubc15\ud604\uc900\uacfc \uae40\uc131\ud604\uc740 \uc2b9\ubd80\uc870\uc791 \ud610\uc758\uac00 \uc778\uc815\ub418\uc5b4 \uac01\uac01 \ubd88\uad6c\uc18d, \uad6c\uc18d \uae30\uc18c\ub418\uc5c8\uace0, KBO\ub294 \uc774 \ub450 \uc120\uc218\ub97c \uc601\uad6c\uc81c\uba85 \ud588\ub2e4. ", "paragraph_answer": "2012\ub144 \ud55c\uad6d\ud504\ub85c\uc57c\uad6c \uc2b9\ubd80\uc870\uc791 \uc0ac\uac74\uc740 KBO \ub9ac\uadf8\uc5d0\uc11c \ubc8c\uc5b4\uc9c4 \uc2b9\ubd80\uc870\uc791 \uc0ac\uac74\uc774\ub2e4. \uc77c\ubd80 \uc120\uc218\uac00 \ub300\uac00\ub97c \uc9c0\ubd88\ubc1b\uace0 \uacbd\uae30 \ub0b4\uc6a9\uacfc \uc2b9\ubd80\ub97c \uc870\uc791\ud558\uc600\ub2e4. 2012\ub144 \ub300\ud55c\ubbfc\uad6d \ud504\ub85c \ucd95\uad6c\uc758 K\ub9ac\uadf8 \uc2b9\ubd80\uc870\uc791 \uc0ac\uac74\uacfc \ud504\ub85c \ubc30\uad6c\uc758 V-\ub9ac\uadf8 \uc2b9\ubd80\uc870\uc791 \uc0ac\uac74\uc758 \uc804\ubaa8\uac00 \ubc1d\ud600\uc9c0\uae30 \uc2dc\uc791\ud558\uba74\uc11c \uc0ac\uac74\uc5d0 \uc5f0\ub8e8\ub41c \ud55c \ube0c\ub85c\ucee4\uac00 \ud55c\uad6d\ud504\ub85c\uc57c\uad6c\uc5d0\uc11c\ub3c4 \uc2b9\ubd80\uc870\uc791\uc774 \uc774\ub8e8\uc5b4\uc84c\ub2e4\uace0 \ubc1d\ud788\uba74\uc11c \ubcf8\uaca9\uc801\uc73c\ub85c \ud504\ub85c \uc57c\uad6c\uc5d0\uc11c\ub3c4 \uc2b9\ubd80\uc870\uc791 \uc0ac\uac74\uc774 \uc218\uba74 \uc704\ub85c \ub5a0\uc624\ub974\uac8c \ub418\uc5c8\ub2e4. \uc774 \uacfc\uc815\uc5d0\uc11c \ub9ce\uc740 \uc120\uc218\ub4e4\uc774 \uc2b9\ubd80\uc870\uc791\uc744 \ud588\ub2e4\ub294 \uc758\uc2ec\uc744 \ubc1b\uc558\uace0, \ubb38\uc131\ud604, \ubc15\ud604\uc900, \uae40\uc131\ud604\uc774 \uac80\ucc30\uc5d0 \uc18c\ud658\ub418\uc5b4 \uc870\uc0ac\ub97c \ubc1b\uc558\ub2e4. \uc774\ub4e4 \uc911 \ubc15\ud604\uc900\uacfc \uae40\uc131\ud604\uc740 \uc2b9\ubd80\uc870\uc791 \ud610\uc758\uac00 \uc778\uc815\ub418\uc5b4 \uac01\uac01 \ubd88\uad6c\uc18d, \uad6c\uc18d \uae30\uc18c\ub418\uc5c8\uace0, KBO\ub294 \uc774 \ub450 \uc120\uc218\ub97c \uc601\uad6c\uc81c\uba85 \ud588\ub2e4.", "sentence_answer": "KBO\ub294 \uc774 \ub450 \uc120\uc218\ub97c \uc601\uad6c\uc81c\uba85 \ud588\ub2e4.", "paragraph_id": "6515806-0-0", "paragraph_question": "question: KBO\uc5d0\uc11c \ubc15\ud604\uc900\uacfc \uae40\uc131\ud604\uc758 \uc2b9\ubd80\uc870\uc791\uc5d0 \ub300\ud574 \uadf8\ub4e4\uc5d0\uac8c \uac00\ud55c \uc870\uce58\ub294?, context: 2012\ub144 \ud55c\uad6d\ud504\ub85c\uc57c\uad6c \uc2b9\ubd80\uc870\uc791 \uc0ac\uac74\uc740 KBO \ub9ac\uadf8\uc5d0\uc11c \ubc8c\uc5b4\uc9c4 \uc2b9\ubd80\uc870\uc791 \uc0ac\uac74\uc774\ub2e4. \uc77c\ubd80 \uc120\uc218\uac00 \ub300\uac00\ub97c \uc9c0\ubd88\ubc1b\uace0 \uacbd\uae30 \ub0b4\uc6a9\uacfc \uc2b9\ubd80\ub97c \uc870\uc791\ud558\uc600\ub2e4. 2012\ub144 \ub300\ud55c\ubbfc\uad6d \ud504\ub85c \ucd95\uad6c\uc758 K\ub9ac\uadf8 \uc2b9\ubd80\uc870\uc791 \uc0ac\uac74\uacfc \ud504\ub85c \ubc30\uad6c\uc758 V-\ub9ac\uadf8 \uc2b9\ubd80\uc870\uc791 \uc0ac\uac74\uc758 \uc804\ubaa8\uac00 \ubc1d\ud600\uc9c0\uae30 \uc2dc\uc791\ud558\uba74\uc11c \uc0ac\uac74\uc5d0 \uc5f0\ub8e8\ub41c \ud55c \ube0c\ub85c\ucee4\uac00 \ud55c\uad6d\ud504\ub85c\uc57c\uad6c\uc5d0\uc11c\ub3c4 \uc2b9\ubd80\uc870\uc791\uc774 \uc774\ub8e8\uc5b4\uc84c\ub2e4\uace0 \ubc1d\ud788\uba74\uc11c \ubcf8\uaca9\uc801\uc73c\ub85c \ud504\ub85c \uc57c\uad6c\uc5d0\uc11c\ub3c4 \uc2b9\ubd80\uc870\uc791 \uc0ac\uac74\uc774 \uc218\uba74 \uc704\ub85c \ub5a0\uc624\ub974\uac8c \ub418\uc5c8\ub2e4. \uc774 \uacfc\uc815\uc5d0\uc11c \ub9ce\uc740 \uc120\uc218\ub4e4\uc774 \uc2b9\ubd80\uc870\uc791\uc744 \ud588\ub2e4\ub294 \uc758\uc2ec\uc744 \ubc1b\uc558\uace0, \ubb38\uc131\ud604, \ubc15\ud604\uc900, \uae40\uc131\ud604\uc774 \uac80\ucc30\uc5d0 \uc18c\ud658\ub418\uc5b4 \uc870\uc0ac\ub97c \ubc1b\uc558\ub2e4. \uc774\ub4e4 \uc911 \ubc15\ud604\uc900\uacfc \uae40\uc131\ud604\uc740 \uc2b9\ubd80\uc870\uc791 \ud610\uc758\uac00 \uc778\uc815\ub418\uc5b4 \uac01\uac01 \ubd88\uad6c\uc18d, \uad6c\uc18d \uae30\uc18c\ub418\uc5c8\uace0, KBO\ub294 \uc774 \ub450 \uc120\uc218\ub97c \uc601\uad6c\uc81c\uba85\ud588\ub2e4."} {"question": "\uc720\ub85c\ud30c\uc758 \ub300\uae30\uad8c\uc740 \ubd84\uc790\ub4e4\uc758 \ubb34\uc5c7\uc73c\ub85c \uc0dd\uc131\ub418\uc5c8\ub098?", "paragraph": "\uc9c0\uad6c\uc758 \ub300\uae30\uad8c\uacfc\ub294 \ub2e4\ub974\uac8c, \uc720\ub85c\ud30c\uc758 \ub300\uae30\uad8c\uc740 \uc0dd\ubb3c\ud559\uc801\uc73c\ub85c \uc0dd\uaca8\ub09c \uac83\uc774 \uc544\ub2c8\ub77c \ubd84\uc790\ub4e4\uc774 \ubc29\uc0ac\uc120 \ubd84\ud574\uac00 \ub418\uc5b4 \uc0dd\uc131\ub418\uc5c8\ub2e4. \ud0dc\uc591\uc5d0\uc11c \uc624\ub294 \uc790\uc678\uc120\uacfc \ubaa9\uc131\uc758 \uc790\uae30\uad8c\uc5d0\uc11c \uc624\ub294 \ub300\uc804 \uc785\uc790(\uc774\uc628\uc774\ub098 \uc804\uc790)\uac00 \uc720\ub85c\ud30c\uc758 \ubb3c \ubd84\uc790\ub97c \ub54c\ub824, \uc218\uc18c\uc640 \uc0b0\uc18c\ub85c \ub098\ub220 \ub300\uae30 \uc911\uc73c\ub85c \uc2a4\ud37c\ud130\ub9c1\ub418\uac70\ub098 \ud761\uc218\ub418\uac8c \ub9cc\ub4e0\ub2e4. \uac19\uc740 \ubc29\uc0ac\uc120\uc774 \uc774 \ud761\uc218\ub41c \ubb3c\uc9c8\ub4e4\uc744 \ub54c\uc5b4\ub0b4\uace0, \ub450 \uc791\uc6a9\uc774 \uc54c\ub9de\uac8c \uade0\ud615\uc744 \uc774\ub8e8\ub294 \ub300\uae30\uad8c\uc744 \ub9cc\ub4e4\uc5b4\ub0b8\ub2e4. \uc0b0\uc18c \ubd84\uc790\ub294 \uc218\uba85\uc774 \uae38\uae30 \ub54c\ubb38\uc5d0 \ub300\uae30\uc758 \uc8fc\uc694 \uad6c\uc131 \uc131\ubd84\uc774 \ub41c\ub2e4. \uc65c\ub0d0\ud558\uba74 \uc0b0\uc18c \ubd84\uc790\uc640 \uac19\uc740 \ubb3c\uc9c8\ub4e4\uc774 \ud45c\uba74\uc73c\ub85c \ub3cc\uc544\uac00\uba74 \ubb3c\uc774\ub098 \uacfc\uc0b0\ud654\uc218\uc18c\uc640 \uac19\uc740 \ubd84\uc790\ub85c \ub9cc\ub4e4\uc5b4\uc9c0\ub294 \uac83\uc774 \uc544\ub2c8\ub77c \ube60\uc838\ub098\uc640 \ub2e4\ub978 \uc791\uc6a9\uc744 \uc2dc\uc791\ud558\uae30 \ub54c\ubb38\uc774\ub2e4. \uc218\uc18c \ubd84\uc790\ub294 \uac00\ubccd\uae30 \ub54c\ubb38\uc5d0 \uc720\ub85c\ud30c\uc758 \uc911\ub825\uc5d0\uc11c \ubc97\uc5b4\ub098\uae30 \uc27d\uace0, \ub2e4\uc2dc\ub294 \ud45c\uba74\uc73c\ub85c \ub3cc\uc544\uac00\uc9c0 \ubabb\ud55c\ub2e4.", "answer": "\ubc29\uc0ac\uc120 \ubd84\ud574", "sentence": "\uc9c0\uad6c\uc758 \ub300\uae30\uad8c\uacfc\ub294 \ub2e4\ub974\uac8c, \uc720\ub85c\ud30c\uc758 \ub300\uae30\uad8c\uc740 \uc0dd\ubb3c\ud559\uc801\uc73c\ub85c \uc0dd\uaca8\ub09c \uac83\uc774 \uc544\ub2c8\ub77c \ubd84\uc790\ub4e4\uc774 \ubc29\uc0ac\uc120 \ubd84\ud574 \uac00 \ub418\uc5b4 \uc0dd\uc131\ub418\uc5c8\ub2e4.", "paragraph_sentence": " \uc9c0\uad6c\uc758 \ub300\uae30\uad8c\uacfc\ub294 \ub2e4\ub974\uac8c, \uc720\ub85c\ud30c\uc758 \ub300\uae30\uad8c\uc740 \uc0dd\ubb3c\ud559\uc801\uc73c\ub85c \uc0dd\uaca8\ub09c \uac83\uc774 \uc544\ub2c8\ub77c \ubd84\uc790\ub4e4\uc774 \ubc29\uc0ac\uc120 \ubd84\ud574 \uac00 \ub418\uc5b4 \uc0dd\uc131\ub418\uc5c8\ub2e4. \ud0dc\uc591\uc5d0\uc11c \uc624\ub294 \uc790\uc678\uc120\uacfc \ubaa9\uc131\uc758 \uc790\uae30\uad8c\uc5d0\uc11c \uc624\ub294 \ub300\uc804 \uc785\uc790(\uc774\uc628\uc774\ub098 \uc804\uc790)\uac00 \uc720\ub85c\ud30c\uc758 \ubb3c \ubd84\uc790\ub97c \ub54c\ub824, \uc218\uc18c\uc640 \uc0b0\uc18c\ub85c \ub098\ub220 \ub300\uae30 \uc911\uc73c\ub85c \uc2a4\ud37c\ud130\ub9c1\ub418\uac70\ub098 \ud761\uc218\ub418\uac8c \ub9cc\ub4e0\ub2e4. \uac19\uc740 \ubc29\uc0ac\uc120\uc774 \uc774 \ud761\uc218\ub41c \ubb3c\uc9c8\ub4e4\uc744 \ub54c\uc5b4\ub0b4\uace0, \ub450 \uc791\uc6a9\uc774 \uc54c\ub9de\uac8c \uade0\ud615\uc744 \uc774\ub8e8\ub294 \ub300\uae30\uad8c\uc744 \ub9cc\ub4e4\uc5b4\ub0b8\ub2e4. \uc0b0\uc18c \ubd84\uc790\ub294 \uc218\uba85\uc774 \uae38\uae30 \ub54c\ubb38\uc5d0 \ub300\uae30\uc758 \uc8fc\uc694 \uad6c\uc131 \uc131\ubd84\uc774 \ub41c\ub2e4. \uc65c\ub0d0\ud558\uba74 \uc0b0\uc18c \ubd84\uc790\uc640 \uac19\uc740 \ubb3c\uc9c8\ub4e4\uc774 \ud45c\uba74\uc73c\ub85c \ub3cc\uc544\uac00\uba74 \ubb3c\uc774\ub098 \uacfc\uc0b0\ud654\uc218\uc18c\uc640 \uac19\uc740 \ubd84\uc790\ub85c \ub9cc\ub4e4\uc5b4\uc9c0\ub294 \uac83\uc774 \uc544\ub2c8\ub77c \ube60\uc838\ub098\uc640 \ub2e4\ub978 \uc791\uc6a9\uc744 \uc2dc\uc791\ud558\uae30 \ub54c\ubb38\uc774\ub2e4. \uc218\uc18c \ubd84\uc790\ub294 \uac00\ubccd\uae30 \ub54c\ubb38\uc5d0 \uc720\ub85c\ud30c\uc758 \uc911\ub825\uc5d0\uc11c \ubc97\uc5b4\ub098\uae30 \uc27d\uace0, \ub2e4\uc2dc\ub294 \ud45c\uba74\uc73c\ub85c \ub3cc\uc544\uac00\uc9c0 \ubabb\ud55c\ub2e4.", "paragraph_answer": "\uc9c0\uad6c\uc758 \ub300\uae30\uad8c\uacfc\ub294 \ub2e4\ub974\uac8c, \uc720\ub85c\ud30c\uc758 \ub300\uae30\uad8c\uc740 \uc0dd\ubb3c\ud559\uc801\uc73c\ub85c \uc0dd\uaca8\ub09c \uac83\uc774 \uc544\ub2c8\ub77c \ubd84\uc790\ub4e4\uc774 \ubc29\uc0ac\uc120 \ubd84\ud574 \uac00 \ub418\uc5b4 \uc0dd\uc131\ub418\uc5c8\ub2e4. \ud0dc\uc591\uc5d0\uc11c \uc624\ub294 \uc790\uc678\uc120\uacfc \ubaa9\uc131\uc758 \uc790\uae30\uad8c\uc5d0\uc11c \uc624\ub294 \ub300\uc804 \uc785\uc790(\uc774\uc628\uc774\ub098 \uc804\uc790)\uac00 \uc720\ub85c\ud30c\uc758 \ubb3c \ubd84\uc790\ub97c \ub54c\ub824, \uc218\uc18c\uc640 \uc0b0\uc18c\ub85c \ub098\ub220 \ub300\uae30 \uc911\uc73c\ub85c \uc2a4\ud37c\ud130\ub9c1\ub418\uac70\ub098 \ud761\uc218\ub418\uac8c \ub9cc\ub4e0\ub2e4. \uac19\uc740 \ubc29\uc0ac\uc120\uc774 \uc774 \ud761\uc218\ub41c \ubb3c\uc9c8\ub4e4\uc744 \ub54c\uc5b4\ub0b4\uace0, \ub450 \uc791\uc6a9\uc774 \uc54c\ub9de\uac8c \uade0\ud615\uc744 \uc774\ub8e8\ub294 \ub300\uae30\uad8c\uc744 \ub9cc\ub4e4\uc5b4\ub0b8\ub2e4. \uc0b0\uc18c \ubd84\uc790\ub294 \uc218\uba85\uc774 \uae38\uae30 \ub54c\ubb38\uc5d0 \ub300\uae30\uc758 \uc8fc\uc694 \uad6c\uc131 \uc131\ubd84\uc774 \ub41c\ub2e4. \uc65c\ub0d0\ud558\uba74 \uc0b0\uc18c \ubd84\uc790\uc640 \uac19\uc740 \ubb3c\uc9c8\ub4e4\uc774 \ud45c\uba74\uc73c\ub85c \ub3cc\uc544\uac00\uba74 \ubb3c\uc774\ub098 \uacfc\uc0b0\ud654\uc218\uc18c\uc640 \uac19\uc740 \ubd84\uc790\ub85c \ub9cc\ub4e4\uc5b4\uc9c0\ub294 \uac83\uc774 \uc544\ub2c8\ub77c \ube60\uc838\ub098\uc640 \ub2e4\ub978 \uc791\uc6a9\uc744 \uc2dc\uc791\ud558\uae30 \ub54c\ubb38\uc774\ub2e4. \uc218\uc18c \ubd84\uc790\ub294 \uac00\ubccd\uae30 \ub54c\ubb38\uc5d0 \uc720\ub85c\ud30c\uc758 \uc911\ub825\uc5d0\uc11c \ubc97\uc5b4\ub098\uae30 \uc27d\uace0, \ub2e4\uc2dc\ub294 \ud45c\uba74\uc73c\ub85c \ub3cc\uc544\uac00\uc9c0 \ubabb\ud55c\ub2e4.", "sentence_answer": "\uc9c0\uad6c\uc758 \ub300\uae30\uad8c\uacfc\ub294 \ub2e4\ub974\uac8c, \uc720\ub85c\ud30c\uc758 \ub300\uae30\uad8c\uc740 \uc0dd\ubb3c\ud559\uc801\uc73c\ub85c \uc0dd\uaca8\ub09c \uac83\uc774 \uc544\ub2c8\ub77c \ubd84\uc790\ub4e4\uc774 \ubc29\uc0ac\uc120 \ubd84\ud574 \uac00 \ub418\uc5b4 \uc0dd\uc131\ub418\uc5c8\ub2e4.", "paragraph_id": "6657470-9-0", "paragraph_question": "question: \uc720\ub85c\ud30c\uc758 \ub300\uae30\uad8c\uc740 \ubd84\uc790\ub4e4\uc758 \ubb34\uc5c7\uc73c\ub85c \uc0dd\uc131\ub418\uc5c8\ub098?, context: \uc9c0\uad6c\uc758 \ub300\uae30\uad8c\uacfc\ub294 \ub2e4\ub974\uac8c, \uc720\ub85c\ud30c\uc758 \ub300\uae30\uad8c\uc740 \uc0dd\ubb3c\ud559\uc801\uc73c\ub85c \uc0dd\uaca8\ub09c \uac83\uc774 \uc544\ub2c8\ub77c \ubd84\uc790\ub4e4\uc774 \ubc29\uc0ac\uc120 \ubd84\ud574\uac00 \ub418\uc5b4 \uc0dd\uc131\ub418\uc5c8\ub2e4. \ud0dc\uc591\uc5d0\uc11c \uc624\ub294 \uc790\uc678\uc120\uacfc \ubaa9\uc131\uc758 \uc790\uae30\uad8c\uc5d0\uc11c \uc624\ub294 \ub300\uc804 \uc785\uc790(\uc774\uc628\uc774\ub098 \uc804\uc790)\uac00 \uc720\ub85c\ud30c\uc758 \ubb3c \ubd84\uc790\ub97c \ub54c\ub824, \uc218\uc18c\uc640 \uc0b0\uc18c\ub85c \ub098\ub220 \ub300\uae30 \uc911\uc73c\ub85c \uc2a4\ud37c\ud130\ub9c1\ub418\uac70\ub098 \ud761\uc218\ub418\uac8c \ub9cc\ub4e0\ub2e4. \uac19\uc740 \ubc29\uc0ac\uc120\uc774 \uc774 \ud761\uc218\ub41c \ubb3c\uc9c8\ub4e4\uc744 \ub54c\uc5b4\ub0b4\uace0, \ub450 \uc791\uc6a9\uc774 \uc54c\ub9de\uac8c \uade0\ud615\uc744 \uc774\ub8e8\ub294 \ub300\uae30\uad8c\uc744 \ub9cc\ub4e4\uc5b4\ub0b8\ub2e4. \uc0b0\uc18c \ubd84\uc790\ub294 \uc218\uba85\uc774 \uae38\uae30 \ub54c\ubb38\uc5d0 \ub300\uae30\uc758 \uc8fc\uc694 \uad6c\uc131 \uc131\ubd84\uc774 \ub41c\ub2e4. \uc65c\ub0d0\ud558\uba74 \uc0b0\uc18c \ubd84\uc790\uc640 \uac19\uc740 \ubb3c\uc9c8\ub4e4\uc774 \ud45c\uba74\uc73c\ub85c \ub3cc\uc544\uac00\uba74 \ubb3c\uc774\ub098 \uacfc\uc0b0\ud654\uc218\uc18c\uc640 \uac19\uc740 \ubd84\uc790\ub85c \ub9cc\ub4e4\uc5b4\uc9c0\ub294 \uac83\uc774 \uc544\ub2c8\ub77c \ube60\uc838\ub098\uc640 \ub2e4\ub978 \uc791\uc6a9\uc744 \uc2dc\uc791\ud558\uae30 \ub54c\ubb38\uc774\ub2e4. \uc218\uc18c \ubd84\uc790\ub294 \uac00\ubccd\uae30 \ub54c\ubb38\uc5d0 \uc720\ub85c\ud30c\uc758 \uc911\ub825\uc5d0\uc11c \ubc97\uc5b4\ub098\uae30 \uc27d\uace0, \ub2e4\uc2dc\ub294 \ud45c\uba74\uc73c\ub85c \ub3cc\uc544\uac00\uc9c0 \ubabb\ud55c\ub2e4."} {"question": "\uae40\uc544\ub791\uc740 1000\ubbf8\ud130 \uc804\uccb4\uc21c\uc704\uc5d0 \uba87\uc704\ub97c \ud558\uc600\ub294\uac00?", "paragraph": "2018 \ud3c9\ucc3d\uc62c\ub9bc\ud53d \uc1fc\ud2b8\ud2b8\ub799 \uc2a4\ud53c\ub4dc\uc2a4\ucf00\uc774\ud305 500\ubbf8\ud130\uc5d0\uc11c \uae40\uc544\ub791\uc740 \uc2ec\uc11d\ud76c\uc640 \ub354\ubd88\uc5b4 \uc608\uc120 \ud0c8\ub77d\uc744 \ud558\uc600\ub2e4. 1500\ubbf8\ud130\uc5d0\uc11c \uacf5\uad50\ub86d\uac8c\ub3c4 \uae40\uc544\ub791\uc740 \uce90\ub098\ub2e4\uc758 \ud0b4 \ubd80\ud0f1\uacfc \uc608\uc120, \uc900\uacb0\uc2b9 \ubaa8\ub450 \ud568\uaed8 \ud3b8\uc131\ub418\uc5b4 \ud0b4 \ubd80\ud0f1\uc744 \ub530\ub3cc\ub9ac\uace0 \uac01\uae30 \uc870 1\uc704\ub97c \ucc28\uc9c0\ud558\uc600\uc73c\ub098 \uacb0\uc2b9\uc5d0\uc11c\ub294 \uc544\uae5d\uac8c\ub3c4 \ud0b4 \ubd80\ud0f1 \ub2e4\uc74c\uc73c\ub85c 4\uc704\uc5d0 \uadf8\uce58\uace0 \ub9d0\uc5c8\ub2e4. \ub2f9\uc2dc 1500\ubbf8\ud130\uc5d0\uc11c \uae08\uba54\ub2ec\uc744 \ub530\uace0 \uc6b8\uba39\uc774\ub294 \ucd5c\ubbfc\uc815\uc744 \ub2e4\ub3c5\uc5ec\uc8fc\ub294 \uae40\uc544\ub791\uc758 \ubaa8\uc2b5\uc740 \ub9ce\uc740 \uc0ac\ub78c\ub4e4\uc5d0\uac8c \uac10\ub3d9\uc744 \uc8fc\uc5c8\ub2e4. 1000\ubbf8\ud130\uc5d0\uc11c \uae40\uc544\ub791\uc740 \ud30c\uc774\ub110 B\uc5d0 \uc9c4\ucd9c\ud558\uc5ec \uc544\uc27d\uac8c\ub3c4 1000\ubbf8\ud130 \uc804\uccb4 \uc21c\uc704 5\uc704\uc5d0 \uadf8\ucce4\ub2e4. 3000\ubbf8\ud130 \uacc4\uc8fc\uc5d0\uc11c \uc5ec\uc790\ub300\ud45c\ud300\uc740 \uac00\uc7a5 \uba3c\uc800 \uacb0\uc2b9\uc120\uc744 \ud1b5\uacfc\ud558\uc5ec \uc18c\uce58\uc5d0 \uc774\uc5b4\uc11c 3000\ubbf8\ud130 \uacc4\uc8fc \uc62c\ub9bc\ud53d 2\uc5f0\ud328\ub97c \ub2ec\uc131\ud558\uc600\ub2e4. \uae40\uc544\ub791\uc740 \uacbd\uae30 \uc9c1\ud6c4 \uc5b4\ub290 \uc778\ud130\ubdf0\uc5d0\uc11c, \uadf8\ub3d9\uc548 \ud798\ub4e0 \uc77c\uc774 \ub9ce\uc558\ub2e4\uace0 \ud558\uc600\ub294\ub370 \uadf8 \ud798\ub4e0 \uc77c\uc774 \ubb34\uc5c7\uc774\ub0d0\ub294 \uae30\uc790\uc758 \uc9c8\ubb38\uc5d0, \uc18c\uce58 \ub300\ud68c \uc774\ud6c4 \ud06c\uace0 \uc791\uc740 \ubd80\uc0c1\uc774 \ub9ce\uc558\ub294\ub370 \uae30\ub7c9\uc774 \ub5a8\uc5b4\uc84c\ub358 \uac78 \uc778\uc815\ud55c\ub2e4\uba70 \uadf8 \ud6c4 \ubc14\ub2e5\ubd80\ud130 \uc2dc\uc791\ud55c\ub2e4 \uc0dd\uac01\ud558\uace0 \uc5f4\uc2ec\ud788 \uc900\ube44\ud588\ub294\ub370 \uc6d4\ub4dc\ucef5\uc5d0\uc11c \uae30\ub300 \uc774\ud558\uc758 \uc131\uc801\uc744 \ubcf4\uc5ec\uc8fc\uc5b4\uc11c \uc790\uc2e0\uc5d0\uac8c \uc57d\uc774 \ub410\ub358 \uc2dc\uac04\uc774\uc5c8\ub2e4\uace0 \ub2f5\ubcc0\ud558\uc600\ub2e4. 2017-2018 \uc2dc\uc98c\uc758 \ub9c8\uc9c0\ub9c9 \ub300\ud68c\uc778 2018 \ubaac\ud2b8\ub9ac\uc62c \uc138\uacc4 \uc120\uc218\uad8c \ub300\ud68c\uc5d0\uc11c \uae40\uc544\ub791\uc740 \uac1c\uc778 \uc885\ud569 \uc21c\uc704 5\uc704\ub97c \uae30\ub85d\ud558\uc600\ub2e4.", "answer": "5\uc704", "sentence": "\uc544\uc27d\uac8c\ub3c4 1000\ubbf8\ud130 \uc804\uccb4 \uc21c\uc704 5\uc704 \uc5d0 \uadf8\ucce4\ub2e4.", "paragraph_sentence": "2018 \ud3c9\ucc3d\uc62c\ub9bc\ud53d \uc1fc\ud2b8\ud2b8\ub799 \uc2a4\ud53c\ub4dc\uc2a4\ucf00\uc774\ud305 500\ubbf8\ud130\uc5d0\uc11c \uae40\uc544\ub791\uc740 \uc2ec\uc11d\ud76c\uc640 \ub354\ubd88\uc5b4 \uc608\uc120 \ud0c8\ub77d\uc744 \ud558\uc600\ub2e4. 1500\ubbf8\ud130\uc5d0\uc11c \uacf5\uad50\ub86d\uac8c\ub3c4 \uae40\uc544\ub791\uc740 \uce90\ub098\ub2e4\uc758 \ud0b4 \ubd80\ud0f1\uacfc \uc608\uc120, \uc900\uacb0\uc2b9 \ubaa8\ub450 \ud568\uaed8 \ud3b8\uc131\ub418\uc5b4 \ud0b4 \ubd80\ud0f1\uc744 \ub530\ub3cc\ub9ac\uace0 \uac01\uae30 \uc870 1\uc704\ub97c \ucc28\uc9c0\ud558\uc600\uc73c\ub098 \uacb0\uc2b9\uc5d0\uc11c\ub294 \uc544\uae5d\uac8c\ub3c4 \ud0b4 \ubd80\ud0f1 \ub2e4\uc74c\uc73c\ub85c 4\uc704\uc5d0 \uadf8\uce58\uace0 \ub9d0\uc5c8\ub2e4. \ub2f9\uc2dc 1500\ubbf8\ud130\uc5d0\uc11c \uae08\uba54\ub2ec\uc744 \ub530\uace0 \uc6b8\uba39\uc774\ub294 \ucd5c\ubbfc\uc815\uc744 \ub2e4\ub3c5\uc5ec\uc8fc\ub294 \uae40\uc544\ub791\uc758 \ubaa8\uc2b5\uc740 \ub9ce\uc740 \uc0ac\ub78c\ub4e4\uc5d0\uac8c \uac10\ub3d9\uc744 \uc8fc\uc5c8\ub2e4. 1000\ubbf8\ud130\uc5d0\uc11c \uae40\uc544\ub791\uc740 \ud30c\uc774\ub110 B\uc5d0 \uc9c4\ucd9c\ud558\uc5ec \uc544\uc27d\uac8c\ub3c4 1000\ubbf8\ud130 \uc804\uccb4 \uc21c\uc704 5\uc704 \uc5d0 \uadf8\ucce4\ub2e4. 3000\ubbf8\ud130 \uacc4\uc8fc\uc5d0\uc11c \uc5ec\uc790\ub300\ud45c\ud300\uc740 \uac00\uc7a5 \uba3c\uc800 \uacb0\uc2b9\uc120\uc744 \ud1b5\uacfc\ud558\uc5ec \uc18c\uce58\uc5d0 \uc774\uc5b4\uc11c 3000\ubbf8\ud130 \uacc4\uc8fc \uc62c\ub9bc\ud53d 2\uc5f0\ud328\ub97c \ub2ec\uc131\ud558\uc600\ub2e4. \uae40\uc544\ub791\uc740 \uacbd\uae30 \uc9c1\ud6c4 \uc5b4\ub290 \uc778\ud130\ubdf0\uc5d0\uc11c, \uadf8\ub3d9\uc548 \ud798\ub4e0 \uc77c\uc774 \ub9ce\uc558\ub2e4\uace0 \ud558\uc600\ub294\ub370 \uadf8 \ud798\ub4e0 \uc77c\uc774 \ubb34\uc5c7\uc774\ub0d0\ub294 \uae30\uc790\uc758 \uc9c8\ubb38\uc5d0, \uc18c\uce58 \ub300\ud68c \uc774\ud6c4 \ud06c\uace0 \uc791\uc740 \ubd80\uc0c1\uc774 \ub9ce\uc558\ub294\ub370 \uae30\ub7c9\uc774 \ub5a8\uc5b4\uc84c\ub358 \uac78 \uc778\uc815\ud55c\ub2e4\uba70 \uadf8 \ud6c4 \ubc14\ub2e5\ubd80\ud130 \uc2dc\uc791\ud55c\ub2e4 \uc0dd\uac01\ud558\uace0 \uc5f4\uc2ec\ud788 \uc900\ube44\ud588\ub294\ub370 \uc6d4\ub4dc\ucef5\uc5d0\uc11c \uae30\ub300 \uc774\ud558\uc758 \uc131\uc801\uc744 \ubcf4\uc5ec\uc8fc\uc5b4\uc11c \uc790\uc2e0\uc5d0\uac8c \uc57d\uc774 \ub410\ub358 \uc2dc\uac04\uc774\uc5c8\ub2e4\uace0 \ub2f5\ubcc0\ud558\uc600\ub2e4. 2017-2018 \uc2dc\uc98c\uc758 \ub9c8\uc9c0\ub9c9 \ub300\ud68c\uc778 2018 \ubaac\ud2b8\ub9ac\uc62c \uc138\uacc4 \uc120\uc218\uad8c \ub300\ud68c\uc5d0\uc11c \uae40\uc544\ub791\uc740 \uac1c\uc778 \uc885\ud569 \uc21c\uc704 5\uc704\ub97c \uae30\ub85d\ud558\uc600\ub2e4.", "paragraph_answer": "2018 \ud3c9\ucc3d\uc62c\ub9bc\ud53d \uc1fc\ud2b8\ud2b8\ub799 \uc2a4\ud53c\ub4dc\uc2a4\ucf00\uc774\ud305 500\ubbf8\ud130\uc5d0\uc11c \uae40\uc544\ub791\uc740 \uc2ec\uc11d\ud76c\uc640 \ub354\ubd88\uc5b4 \uc608\uc120 \ud0c8\ub77d\uc744 \ud558\uc600\ub2e4. 1500\ubbf8\ud130\uc5d0\uc11c \uacf5\uad50\ub86d\uac8c\ub3c4 \uae40\uc544\ub791\uc740 \uce90\ub098\ub2e4\uc758 \ud0b4 \ubd80\ud0f1\uacfc \uc608\uc120, \uc900\uacb0\uc2b9 \ubaa8\ub450 \ud568\uaed8 \ud3b8\uc131\ub418\uc5b4 \ud0b4 \ubd80\ud0f1\uc744 \ub530\ub3cc\ub9ac\uace0 \uac01\uae30 \uc870 1\uc704\ub97c \ucc28\uc9c0\ud558\uc600\uc73c\ub098 \uacb0\uc2b9\uc5d0\uc11c\ub294 \uc544\uae5d\uac8c\ub3c4 \ud0b4 \ubd80\ud0f1 \ub2e4\uc74c\uc73c\ub85c 4\uc704\uc5d0 \uadf8\uce58\uace0 \ub9d0\uc5c8\ub2e4. \ub2f9\uc2dc 1500\ubbf8\ud130\uc5d0\uc11c \uae08\uba54\ub2ec\uc744 \ub530\uace0 \uc6b8\uba39\uc774\ub294 \ucd5c\ubbfc\uc815\uc744 \ub2e4\ub3c5\uc5ec\uc8fc\ub294 \uae40\uc544\ub791\uc758 \ubaa8\uc2b5\uc740 \ub9ce\uc740 \uc0ac\ub78c\ub4e4\uc5d0\uac8c \uac10\ub3d9\uc744 \uc8fc\uc5c8\ub2e4. 1000\ubbf8\ud130\uc5d0\uc11c \uae40\uc544\ub791\uc740 \ud30c\uc774\ub110 B\uc5d0 \uc9c4\ucd9c\ud558\uc5ec \uc544\uc27d\uac8c\ub3c4 1000\ubbf8\ud130 \uc804\uccb4 \uc21c\uc704 5\uc704 \uc5d0 \uadf8\ucce4\ub2e4. 3000\ubbf8\ud130 \uacc4\uc8fc\uc5d0\uc11c \uc5ec\uc790\ub300\ud45c\ud300\uc740 \uac00\uc7a5 \uba3c\uc800 \uacb0\uc2b9\uc120\uc744 \ud1b5\uacfc\ud558\uc5ec \uc18c\uce58\uc5d0 \uc774\uc5b4\uc11c 3000\ubbf8\ud130 \uacc4\uc8fc \uc62c\ub9bc\ud53d 2\uc5f0\ud328\ub97c \ub2ec\uc131\ud558\uc600\ub2e4. \uae40\uc544\ub791\uc740 \uacbd\uae30 \uc9c1\ud6c4 \uc5b4\ub290 \uc778\ud130\ubdf0\uc5d0\uc11c, \uadf8\ub3d9\uc548 \ud798\ub4e0 \uc77c\uc774 \ub9ce\uc558\ub2e4\uace0 \ud558\uc600\ub294\ub370 \uadf8 \ud798\ub4e0 \uc77c\uc774 \ubb34\uc5c7\uc774\ub0d0\ub294 \uae30\uc790\uc758 \uc9c8\ubb38\uc5d0, \uc18c\uce58 \ub300\ud68c \uc774\ud6c4 \ud06c\uace0 \uc791\uc740 \ubd80\uc0c1\uc774 \ub9ce\uc558\ub294\ub370 \uae30\ub7c9\uc774 \ub5a8\uc5b4\uc84c\ub358 \uac78 \uc778\uc815\ud55c\ub2e4\uba70 \uadf8 \ud6c4 \ubc14\ub2e5\ubd80\ud130 \uc2dc\uc791\ud55c\ub2e4 \uc0dd\uac01\ud558\uace0 \uc5f4\uc2ec\ud788 \uc900\ube44\ud588\ub294\ub370 \uc6d4\ub4dc\ucef5\uc5d0\uc11c \uae30\ub300 \uc774\ud558\uc758 \uc131\uc801\uc744 \ubcf4\uc5ec\uc8fc\uc5b4\uc11c \uc790\uc2e0\uc5d0\uac8c \uc57d\uc774 \ub410\ub358 \uc2dc\uac04\uc774\uc5c8\ub2e4\uace0 \ub2f5\ubcc0\ud558\uc600\ub2e4. 2017-2018 \uc2dc\uc98c\uc758 \ub9c8\uc9c0\ub9c9 \ub300\ud68c\uc778 2018 \ubaac\ud2b8\ub9ac\uc62c \uc138\uacc4 \uc120\uc218\uad8c \ub300\ud68c\uc5d0\uc11c \uae40\uc544\ub791\uc740 \uac1c\uc778 \uc885\ud569 \uc21c\uc704 5\uc704\ub97c \uae30\ub85d\ud558\uc600\ub2e4.", "sentence_answer": "\uc544\uc27d\uac8c\ub3c4 1000\ubbf8\ud130 \uc804\uccb4 \uc21c\uc704 5\uc704 \uc5d0 \uadf8\ucce4\ub2e4.", "paragraph_id": "6481874-6-1", "paragraph_question": "question: \uae40\uc544\ub791\uc740 1000\ubbf8\ud130 \uc804\uccb4\uc21c\uc704\uc5d0 \uba87\uc704\ub97c \ud558\uc600\ub294\uac00?, context: 2018 \ud3c9\ucc3d\uc62c\ub9bc\ud53d \uc1fc\ud2b8\ud2b8\ub799 \uc2a4\ud53c\ub4dc\uc2a4\ucf00\uc774\ud305 500\ubbf8\ud130\uc5d0\uc11c \uae40\uc544\ub791\uc740 \uc2ec\uc11d\ud76c\uc640 \ub354\ubd88\uc5b4 \uc608\uc120 \ud0c8\ub77d\uc744 \ud558\uc600\ub2e4. 1500\ubbf8\ud130\uc5d0\uc11c \uacf5\uad50\ub86d\uac8c\ub3c4 \uae40\uc544\ub791\uc740 \uce90\ub098\ub2e4\uc758 \ud0b4 \ubd80\ud0f1\uacfc \uc608\uc120, \uc900\uacb0\uc2b9 \ubaa8\ub450 \ud568\uaed8 \ud3b8\uc131\ub418\uc5b4 \ud0b4 \ubd80\ud0f1\uc744 \ub530\ub3cc\ub9ac\uace0 \uac01\uae30 \uc870 1\uc704\ub97c \ucc28\uc9c0\ud558\uc600\uc73c\ub098 \uacb0\uc2b9\uc5d0\uc11c\ub294 \uc544\uae5d\uac8c\ub3c4 \ud0b4 \ubd80\ud0f1 \ub2e4\uc74c\uc73c\ub85c 4\uc704\uc5d0 \uadf8\uce58\uace0 \ub9d0\uc5c8\ub2e4. \ub2f9\uc2dc 1500\ubbf8\ud130\uc5d0\uc11c \uae08\uba54\ub2ec\uc744 \ub530\uace0 \uc6b8\uba39\uc774\ub294 \ucd5c\ubbfc\uc815\uc744 \ub2e4\ub3c5\uc5ec\uc8fc\ub294 \uae40\uc544\ub791\uc758 \ubaa8\uc2b5\uc740 \ub9ce\uc740 \uc0ac\ub78c\ub4e4\uc5d0\uac8c \uac10\ub3d9\uc744 \uc8fc\uc5c8\ub2e4. 1000\ubbf8\ud130\uc5d0\uc11c \uae40\uc544\ub791\uc740 \ud30c\uc774\ub110 B\uc5d0 \uc9c4\ucd9c\ud558\uc5ec \uc544\uc27d\uac8c\ub3c4 1000\ubbf8\ud130 \uc804\uccb4 \uc21c\uc704 5\uc704\uc5d0 \uadf8\ucce4\ub2e4. 3000\ubbf8\ud130 \uacc4\uc8fc\uc5d0\uc11c \uc5ec\uc790\ub300\ud45c\ud300\uc740 \uac00\uc7a5 \uba3c\uc800 \uacb0\uc2b9\uc120\uc744 \ud1b5\uacfc\ud558\uc5ec \uc18c\uce58\uc5d0 \uc774\uc5b4\uc11c 3000\ubbf8\ud130 \uacc4\uc8fc \uc62c\ub9bc\ud53d 2\uc5f0\ud328\ub97c \ub2ec\uc131\ud558\uc600\ub2e4. \uae40\uc544\ub791\uc740 \uacbd\uae30 \uc9c1\ud6c4 \uc5b4\ub290 \uc778\ud130\ubdf0\uc5d0\uc11c, \uadf8\ub3d9\uc548 \ud798\ub4e0 \uc77c\uc774 \ub9ce\uc558\ub2e4\uace0 \ud558\uc600\ub294\ub370 \uadf8 \ud798\ub4e0 \uc77c\uc774 \ubb34\uc5c7\uc774\ub0d0\ub294 \uae30\uc790\uc758 \uc9c8\ubb38\uc5d0, \uc18c\uce58 \ub300\ud68c \uc774\ud6c4 \ud06c\uace0 \uc791\uc740 \ubd80\uc0c1\uc774 \ub9ce\uc558\ub294\ub370 \uae30\ub7c9\uc774 \ub5a8\uc5b4\uc84c\ub358 \uac78 \uc778\uc815\ud55c\ub2e4\uba70 \uadf8 \ud6c4 \ubc14\ub2e5\ubd80\ud130 \uc2dc\uc791\ud55c\ub2e4 \uc0dd\uac01\ud558\uace0 \uc5f4\uc2ec\ud788 \uc900\ube44\ud588\ub294\ub370 \uc6d4\ub4dc\ucef5\uc5d0\uc11c \uae30\ub300 \uc774\ud558\uc758 \uc131\uc801\uc744 \ubcf4\uc5ec\uc8fc\uc5b4\uc11c \uc790\uc2e0\uc5d0\uac8c \uc57d\uc774 \ub410\ub358 \uc2dc\uac04\uc774\uc5c8\ub2e4\uace0 \ub2f5\ubcc0\ud558\uc600\ub2e4. 2017-2018 \uc2dc\uc98c\uc758 \ub9c8\uc9c0\ub9c9 \ub300\ud68c\uc778 2018 \ubaac\ud2b8\ub9ac\uc62c \uc138\uacc4 \uc120\uc218\uad8c \ub300\ud68c\uc5d0\uc11c \uae40\uc544\ub791\uc740 \uac1c\uc778 \uc885\ud569 \uc21c\uc704 5\uc704\ub97c \uae30\ub85d\ud558\uc600\ub2e4."} {"question": "\ubd84\uc790 \uad6c\ub984\uc758 \uc77c\ubd80\ubd84\uc774 \uc911\ub825 \ubd95\uad34\ub97c \uc77c\uc73c\ucf1c \uc0dd\uaca8\ub09c\uac83\uc740?", "paragraph": "\uc131\uc6b4 \uc774\ub860\uc5d0 \ub530\ub974\uba74 \ud0dc\uc591\uacc4\ub294 \ud3ed\uc774 \uc218 \uad11\ub144\uc5d0 \uc774\ub974\ub294 \ubd84\uc790 \uad6c\ub984\uc758 \uc77c\ubd80\ubd84\uc774 \uc911\ub825 \ubd95\uad34\ub97c \uc77c\uc73c\ucf1c \uc0dd\uaca8\ub0ac\ub2e4. 2000\ub144\ub300 \ucd08\uae4c\uc9c0\ub9cc \ud574\ub3c4 \ud0dc\uc591\uc740 \ud640\ub85c \ud0dc\uc5b4\ub09c\ub2e4\uace0 \uc54c\ub824\uc838 \uc654\ub2e4. \uadf8\ub7ec\ub098 \ucd5c\uadfc \ud0dc\uc591\uacc4 \uc0dd\uc131 \ucd08\uae30\ubd80\ud130 \uc874\uc7ac\ud588\ub358 \uc6b4\uc11d\uc758 \uc131\ubd84\uc744 \uc870\uc0ac\ud55c \uacb0\uacfc \ucca0-60\uacfc \uac19\uc774 \ucd08\uc2e0\uc131 \ud3ed\ubc1c\uc744 \ud1b5\ud574\uc11c\ub9cc \uc0dd\uae38 \uc218 \uc788\ub294 \ub3d9\uc704 \uc6d0\uc18c\uc758 \ud754\uc801\uc774 \ubc1c\uacac\ub418\uc5c8\ub2e4. \uc774\ub294 \ud0dc\uc591\uc774 \uc0dd\uaca8\ub09c \uacf3 \uadfc\ucc98\uc5d0 \ucd08\uc2e0\uc131\uc774 \uc5ec\ub7ff \uc788\uc5c8\uc74c\uc744 \uc758\ubbf8\ud55c\ub2e4. \uc774\ub4e4 \ucd08\uc2e0\uc131\uc5d0\uc11c \ub098\uc628 \ucda9\uaca9\ud30c\ub294 \ud0dc\uc591 \uc131\uc6b4\uc758 \ubc00\ub3c4\ub97c \uc99d\uac00\uc2dc\ucf30\uace0, \uc911\ub825 \ubd95\uad34\ub97c \uc77c\uc73c\ucf30\ub2e4. \uc624\uc9c1 \ubb34\uac81\uace0 \uae08\ubc29 \uc8fd\ub294 \ubcc4\ub9cc\uc774 \ucd08\uc2e0\uc131\uc73c\ub85c \ucd5c\ud6c4\ub97c \ub9de\uae30 \ub54c\ubb38\uc5d0, \ud0dc\uc591\uc740 \uc624\ub9ac\uc628\uc131\uc6b4\ucc98\ub7fc \ubb34\uac70\uc6b4 \ubcc4\ub4e4\uc744 \uc0dd\uc0b0\ud574 \ub0b4\ub294 \ud56d\uc131 \uc0dd\uc131 \uc7a5\uc18c\uc5d0\uc11c \ud615\uc131\ub418\uc5c8\uc74c\uc774 \ud2c0\ub9bc\uc5c6\ub098.", "answer": "\ud0dc\uc591\uacc4", "sentence": "\uc131\uc6b4 \uc774\ub860\uc5d0 \ub530\ub974\uba74 \ud0dc\uc591\uacc4 \ub294 \ud3ed\uc774 \uc218 \uad11\ub144\uc5d0 \uc774\ub974\ub294 \ubd84\uc790 \uad6c\ub984\uc758 \uc77c\ubd80\ubd84\uc774 \uc911\ub825 \ubd95\uad34\ub97c \uc77c\uc73c\ucf1c \uc0dd\uaca8\ub0ac\ub2e4.", "paragraph_sentence": " \uc131\uc6b4 \uc774\ub860\uc5d0 \ub530\ub974\uba74 \ud0dc\uc591\uacc4 \ub294 \ud3ed\uc774 \uc218 \uad11\ub144\uc5d0 \uc774\ub974\ub294 \ubd84\uc790 \uad6c\ub984\uc758 \uc77c\ubd80\ubd84\uc774 \uc911\ub825 \ubd95\uad34\ub97c \uc77c\uc73c\ucf1c \uc0dd\uaca8\ub0ac\ub2e4. 2000\ub144\ub300 \ucd08\uae4c\uc9c0\ub9cc \ud574\ub3c4 \ud0dc\uc591\uc740 \ud640\ub85c \ud0dc\uc5b4\ub09c\ub2e4\uace0 \uc54c\ub824\uc838 \uc654\ub2e4. \uadf8\ub7ec\ub098 \ucd5c\uadfc \ud0dc\uc591\uacc4 \uc0dd\uc131 \ucd08\uae30\ubd80\ud130 \uc874\uc7ac\ud588\ub358 \uc6b4\uc11d\uc758 \uc131\ubd84\uc744 \uc870\uc0ac\ud55c \uacb0\uacfc \ucca0-60\uacfc \uac19\uc774 \ucd08\uc2e0\uc131 \ud3ed\ubc1c\uc744 \ud1b5\ud574\uc11c\ub9cc \uc0dd\uae38 \uc218 \uc788\ub294 \ub3d9\uc704 \uc6d0\uc18c\uc758 \ud754\uc801\uc774 \ubc1c\uacac\ub418\uc5c8\ub2e4. \uc774\ub294 \ud0dc\uc591\uc774 \uc0dd\uaca8\ub09c \uacf3 \uadfc\ucc98\uc5d0 \ucd08\uc2e0\uc131\uc774 \uc5ec\ub7ff \uc788\uc5c8\uc74c\uc744 \uc758\ubbf8\ud55c\ub2e4. \uc774\ub4e4 \ucd08\uc2e0\uc131\uc5d0\uc11c \ub098\uc628 \ucda9\uaca9\ud30c\ub294 \ud0dc\uc591 \uc131\uc6b4\uc758 \ubc00\ub3c4\ub97c \uc99d\uac00\uc2dc\ucf30\uace0, \uc911\ub825 \ubd95\uad34\ub97c \uc77c\uc73c\ucf30\ub2e4. \uc624\uc9c1 \ubb34\uac81\uace0 \uae08\ubc29 \uc8fd\ub294 \ubcc4\ub9cc\uc774 \ucd08\uc2e0\uc131\uc73c\ub85c \ucd5c\ud6c4\ub97c \ub9de\uae30 \ub54c\ubb38\uc5d0, \ud0dc\uc591\uc740 \uc624\ub9ac\uc628\uc131\uc6b4\ucc98\ub7fc \ubb34\uac70\uc6b4 \ubcc4\ub4e4\uc744 \uc0dd\uc0b0\ud574 \ub0b4\ub294 \ud56d\uc131 \uc0dd\uc131 \uc7a5\uc18c\uc5d0\uc11c \ud615\uc131\ub418\uc5c8\uc74c\uc774 \ud2c0\ub9bc\uc5c6\ub098.", "paragraph_answer": "\uc131\uc6b4 \uc774\ub860\uc5d0 \ub530\ub974\uba74 \ud0dc\uc591\uacc4 \ub294 \ud3ed\uc774 \uc218 \uad11\ub144\uc5d0 \uc774\ub974\ub294 \ubd84\uc790 \uad6c\ub984\uc758 \uc77c\ubd80\ubd84\uc774 \uc911\ub825 \ubd95\uad34\ub97c \uc77c\uc73c\ucf1c \uc0dd\uaca8\ub0ac\ub2e4. 2000\ub144\ub300 \ucd08\uae4c\uc9c0\ub9cc \ud574\ub3c4 \ud0dc\uc591\uc740 \ud640\ub85c \ud0dc\uc5b4\ub09c\ub2e4\uace0 \uc54c\ub824\uc838 \uc654\ub2e4. \uadf8\ub7ec\ub098 \ucd5c\uadfc \ud0dc\uc591\uacc4 \uc0dd\uc131 \ucd08\uae30\ubd80\ud130 \uc874\uc7ac\ud588\ub358 \uc6b4\uc11d\uc758 \uc131\ubd84\uc744 \uc870\uc0ac\ud55c \uacb0\uacfc \ucca0-60\uacfc \uac19\uc774 \ucd08\uc2e0\uc131 \ud3ed\ubc1c\uc744 \ud1b5\ud574\uc11c\ub9cc \uc0dd\uae38 \uc218 \uc788\ub294 \ub3d9\uc704 \uc6d0\uc18c\uc758 \ud754\uc801\uc774 \ubc1c\uacac\ub418\uc5c8\ub2e4. \uc774\ub294 \ud0dc\uc591\uc774 \uc0dd\uaca8\ub09c \uacf3 \uadfc\ucc98\uc5d0 \ucd08\uc2e0\uc131\uc774 \uc5ec\ub7ff \uc788\uc5c8\uc74c\uc744 \uc758\ubbf8\ud55c\ub2e4. \uc774\ub4e4 \ucd08\uc2e0\uc131\uc5d0\uc11c \ub098\uc628 \ucda9\uaca9\ud30c\ub294 \ud0dc\uc591 \uc131\uc6b4\uc758 \ubc00\ub3c4\ub97c \uc99d\uac00\uc2dc\ucf30\uace0, \uc911\ub825 \ubd95\uad34\ub97c \uc77c\uc73c\ucf30\ub2e4. \uc624\uc9c1 \ubb34\uac81\uace0 \uae08\ubc29 \uc8fd\ub294 \ubcc4\ub9cc\uc774 \ucd08\uc2e0\uc131\uc73c\ub85c \ucd5c\ud6c4\ub97c \ub9de\uae30 \ub54c\ubb38\uc5d0, \ud0dc\uc591\uc740 \uc624\ub9ac\uc628\uc131\uc6b4\ucc98\ub7fc \ubb34\uac70\uc6b4 \ubcc4\ub4e4\uc744 \uc0dd\uc0b0\ud574 \ub0b4\ub294 \ud56d\uc131 \uc0dd\uc131 \uc7a5\uc18c\uc5d0\uc11c \ud615\uc131\ub418\uc5c8\uc74c\uc774 \ud2c0\ub9bc\uc5c6\ub098.", "sentence_answer": "\uc131\uc6b4 \uc774\ub860\uc5d0 \ub530\ub974\uba74 \ud0dc\uc591\uacc4 \ub294 \ud3ed\uc774 \uc218 \uad11\ub144\uc5d0 \uc774\ub974\ub294 \ubd84\uc790 \uad6c\ub984\uc758 \uc77c\ubd80\ubd84\uc774 \uc911\ub825 \ubd95\uad34\ub97c \uc77c\uc73c\ucf1c \uc0dd\uaca8\ub0ac\ub2e4.", "paragraph_id": "5911343-2-0", "paragraph_question": "question: \ubd84\uc790 \uad6c\ub984\uc758 \uc77c\ubd80\ubd84\uc774 \uc911\ub825 \ubd95\uad34\ub97c \uc77c\uc73c\ucf1c \uc0dd\uaca8\ub09c\uac83\uc740?, context: \uc131\uc6b4 \uc774\ub860\uc5d0 \ub530\ub974\uba74 \ud0dc\uc591\uacc4\ub294 \ud3ed\uc774 \uc218 \uad11\ub144\uc5d0 \uc774\ub974\ub294 \ubd84\uc790 \uad6c\ub984\uc758 \uc77c\ubd80\ubd84\uc774 \uc911\ub825 \ubd95\uad34\ub97c \uc77c\uc73c\ucf1c \uc0dd\uaca8\ub0ac\ub2e4. 2000\ub144\ub300 \ucd08\uae4c\uc9c0\ub9cc \ud574\ub3c4 \ud0dc\uc591\uc740 \ud640\ub85c \ud0dc\uc5b4\ub09c\ub2e4\uace0 \uc54c\ub824\uc838 \uc654\ub2e4. \uadf8\ub7ec\ub098 \ucd5c\uadfc \ud0dc\uc591\uacc4 \uc0dd\uc131 \ucd08\uae30\ubd80\ud130 \uc874\uc7ac\ud588\ub358 \uc6b4\uc11d\uc758 \uc131\ubd84\uc744 \uc870\uc0ac\ud55c \uacb0\uacfc \ucca0-60\uacfc \uac19\uc774 \ucd08\uc2e0\uc131 \ud3ed\ubc1c\uc744 \ud1b5\ud574\uc11c\ub9cc \uc0dd\uae38 \uc218 \uc788\ub294 \ub3d9\uc704 \uc6d0\uc18c\uc758 \ud754\uc801\uc774 \ubc1c\uacac\ub418\uc5c8\ub2e4. \uc774\ub294 \ud0dc\uc591\uc774 \uc0dd\uaca8\ub09c \uacf3 \uadfc\ucc98\uc5d0 \ucd08\uc2e0\uc131\uc774 \uc5ec\ub7ff \uc788\uc5c8\uc74c\uc744 \uc758\ubbf8\ud55c\ub2e4. \uc774\ub4e4 \ucd08\uc2e0\uc131\uc5d0\uc11c \ub098\uc628 \ucda9\uaca9\ud30c\ub294 \ud0dc\uc591 \uc131\uc6b4\uc758 \ubc00\ub3c4\ub97c \uc99d\uac00\uc2dc\ucf30\uace0, \uc911\ub825 \ubd95\uad34\ub97c \uc77c\uc73c\ucf30\ub2e4. \uc624\uc9c1 \ubb34\uac81\uace0 \uae08\ubc29 \uc8fd\ub294 \ubcc4\ub9cc\uc774 \ucd08\uc2e0\uc131\uc73c\ub85c \ucd5c\ud6c4\ub97c \ub9de\uae30 \ub54c\ubb38\uc5d0, \ud0dc\uc591\uc740 \uc624\ub9ac\uc628\uc131\uc6b4\ucc98\ub7fc \ubb34\uac70\uc6b4 \ubcc4\ub4e4\uc744 \uc0dd\uc0b0\ud574 \ub0b4\ub294 \ud56d\uc131 \uc0dd\uc131 \uc7a5\uc18c\uc5d0\uc11c \ud615\uc131\ub418\uc5c8\uc74c\uc774 \ud2c0\ub9bc\uc5c6\ub098."} {"question": "\ud504\ub9ac\ub4dc\ub9ac\ud788\uac00 \uc81c\uad6d\uc758\ud68c \uc120\uac70\uc5d0 \uad00\uc5ec\ud55c \uc774\uc720\ub85c \uacbd\uc9c8\ud588\ub358\n\ud504\ub85c\uc774\uc13c \ub0b4\ubb34\uc7a5\uad00 \uc774\ub984\uc740?", "paragraph": "\ube44\ub85d \ubcd1\uc138\ub294 \uc810\uc810 \uc545\ud654\ub428\uc5d0\ub3c4 \ubd88\uad6c\ud558\uace0, \ud504\ub9ac\ub4dc\ub9ac\ud788 3\uc138\ub294 \ud669\uc81c\ub85c\uc11c\uc758 \ucc45\ubb34\ub97c \ub2e4\ud558\ub824 \ub178\ub825\ud558\uc600\ub2e4. \uc989\uc704 \ubc1c\ud45c \uc9c1\ud6c4, \ud504\ub9ac\ub4dc\ub9ac\ud788\ub294 \uc790\uc2e0\uc758 \uc815\uc7a5\uc5d0 \ubc15\ud600\uc788\ub294 \uac80\uc740 \ub3c5\uc218\ub9ac \ud6c8\uc7a5\uc744 \ube7c\uc11c \uc544\ub0b4 \ube45\ud1a0\ub9ac\uc544\uc758 \ub4dc\ub808\uc2a4\uc5d0 \ub2ec\uc544\uc92c\ub294\ub370, \uc774\ub294 \ube45\ud1a0\ub9ac\uc544\ub97c \ud669\ud6c4\ub85c \ub192\uc774\uaca0\ub2e4\ub294 \ub73b\uc744 \ucc9c\uba85\ud55c \uac83\uc774\uc5c8\ub2e4. \ud504\ub9ac\ub4dc\ub9ac\ud788\ub294 \uc7ac\uc704 \uae30\uac04 \ub3d9\uc548 \uc7a5\ubaa8\uc778 \ube45\ud1a0\ub9ac\uc544 \uc5ec\uc655\uacfc \uc2a4\uc6e8\ub374\uacfc \ub178\ub974\uc6e8\uc774\uc758 \uad6d\uc655 \uc624\uc2a4\uce74\ub974 2\uc138\uc758 \uacf5\uc2dd \uc608\ubc29\uc744 \ubc1b\uc558\uace0, \ucc28\ub0a8\uc778 \ud558\uc778\ub9ac\ud788\uc640 \uc870\uce74\ub538\uc778 \uc774\ub808\ub124 \uacf5\uc8fc\uc758 \uacb0\ud63c\uc2dd\uc5d0\ub3c4 \ucc38\uc11d\ud558\uc600\ub2e4. \uadf8\ub7ec\ub098 \ud504\ub9ac\ub4dc\ub9ac\ud788\uc758 \uc7ac\uc704 \uae30\uac04\uc740 \uace0\uc791 99\uc77c\uc774\uc5c8\uace0, \uc774\ub7ec\ud55c \uc9e7\uc740 \uce58\uc138\ub85c \uc778\ud574 \uad04\ubaa9\ud560 \ub9cc\ud55c \ubcc0\ud654\ub97c \ub610\ud55c \uac00\uc838\uc624\uc9c0 \ubabb\ud588\ub2e4. \uc989\uc704 \uc9c1\uc804 \uadf8\uac00 \ubc1c\ud45c\ud55c \uce59\ub839\uc5d0\uc11c\ub294 \ud5cc\ubc95\uc5d0 \ub530\ub77c \ud669\uc81c\uc640 \uc81c\uad6d\uc218\uc0c1\uc758 \uad8c\ud55c\uc744 \uc81c\ud55c\ud560 \uac83\uc744 \uc57d\uc18d\ud558\uc600\uc73c\ub098 \uc774 \uce59\ub839\uc740 \ub05d\ub0b4 \uc2dc\ud589\ub418\uc9c0 \ubabb\ud588\ub2e4. 6\uc6d4 8\uc77c\uc5d0\ub294 \ud504\ub85c\uc774\uc13c \ub0b4\ubb34\uc7a5\uad00 \ub85c\ubca0\ub974\ud2b8 \ud3f0 \ud478\ud2b8\uce74\uba38\uac00 \uc81c\uad6d\uc758\ud68c \uc120\uac70\uc5d0 \uad00\uc5ec\ud574 \uc628 \uc0ac\uc2e4\uc774 \ub4dc\ub7ec\ub098\uc790, \uadf8\ub97c \uacbd\uc9c8\ud558\uc600\ub2e4. \ub9e4\ucf04\uc9c0 \ubc15\uc0ac\ub294 \ud504\ub9ac\ub4dc\ub9ac\ud788\uac00 \ud669\uc81c\ub85c\uc11c\uc758 \ucc45\ubb34\uc5d0 \ub300\ud574 \uc555\ub3c4\uc801\uc778 \uac10\uac01\uc744 \uc9c0\ub2c8\uace0 \uc788\uc5c8\ub2e4\uace0 \ud68c\uace0\ud588\ub2e4. \ube45\ud1a0\ub9ac\uc544 \ud669\ud6c4\ub294 \ud504\ub79c\uc2dc\uc2a4 \ub124\uc774\ud53c\uc5b4\uc5d0\uac8c \ubcf4\ub0b4\ub294 \ud3b8\uc9c0\uc5d0\uc11c \u201c\ud669\uc81c\uaed8\uc120 \ud68c\uc758\uc5d0\ub3c4 \ucc38\uc11d\ud558\uc2dc\uace0 \ub9e4\uc6b0 \uc798\ud558\uace0 \uacc4\uc2dc\uba70 \uc5ec\uc804\ud788 \uc2dc\ub3c4\ub294 \ud558\uc2dc\uc9c0\ub9cc \uc544\uc9c1\ub3c4 \ub9d0\uc744 \ud558\uc2e4 \uc218 \uc5c6\ub2e4\u201d\ub77c\uace0 \uc37c\ub2e4. \uc5f4\uc815\uc774 \ub118\ucce4\uc73c\ub098 \uc790\uc2e0\uc774 \ub73b\ud558\ub358 \ubc14\ub97c \uc131\ucde8\uc2dc\ud0ac \uc2dc\uac04\uc740 \ub9ce\uc9c0 \uc54a\uc558\ub358 \ud504\ub9ac\ub4dc\ub9ac\ud788\ub294 1888\ub144 5\uc6d4\uc5d0 \u201c\ub09c \uc544\uc9c1 \uc8fd\uc744 \uc218 \uc5c6\ub2e4... \uc774\uc81c \ub3c5\uc77c\uc740 \uc5b4\ub5bb\uac8c \ub420 \uac83\uc778\uac00?\u201d\ub77c\uace0 \uc808\uaddc\ud558\uc600\ub2e4.", "answer": "\ub85c\ubca0\ub974\ud2b8 \ud3f0 \ud478\ud2b8\uce74\uba38", "sentence": "6\uc6d4 8\uc77c\uc5d0\ub294 \ud504\ub85c\uc774\uc13c \ub0b4\ubb34\uc7a5\uad00 \ub85c\ubca0\ub974\ud2b8 \ud3f0 \ud478\ud2b8\uce74\uba38 \uac00 \uc81c\uad6d\uc758\ud68c \uc120\uac70\uc5d0 \uad00\uc5ec\ud574 \uc628 \uc0ac\uc2e4\uc774 \ub4dc\ub7ec\ub098\uc790, \uadf8\ub97c \uacbd\uc9c8\ud558\uc600\ub2e4.", "paragraph_sentence": "\ube44\ub85d \ubcd1\uc138\ub294 \uc810\uc810 \uc545\ud654\ub428\uc5d0\ub3c4 \ubd88\uad6c\ud558\uace0, \ud504\ub9ac\ub4dc\ub9ac\ud788 3\uc138\ub294 \ud669\uc81c\ub85c\uc11c\uc758 \ucc45\ubb34\ub97c \ub2e4\ud558\ub824 \ub178\ub825\ud558\uc600\ub2e4. \uc989\uc704 \ubc1c\ud45c \uc9c1\ud6c4, \ud504\ub9ac\ub4dc\ub9ac\ud788\ub294 \uc790\uc2e0\uc758 \uc815\uc7a5\uc5d0 \ubc15\ud600\uc788\ub294 \uac80\uc740 \ub3c5\uc218\ub9ac \ud6c8\uc7a5\uc744 \ube7c\uc11c \uc544\ub0b4 \ube45\ud1a0\ub9ac\uc544\uc758 \ub4dc\ub808\uc2a4\uc5d0 \ub2ec\uc544\uc92c\ub294\ub370, \uc774\ub294 \ube45\ud1a0\ub9ac\uc544\ub97c \ud669\ud6c4\ub85c \ub192\uc774\uaca0\ub2e4\ub294 \ub73b\uc744 \ucc9c\uba85\ud55c \uac83\uc774\uc5c8\ub2e4. \ud504\ub9ac\ub4dc\ub9ac\ud788\ub294 \uc7ac\uc704 \uae30\uac04 \ub3d9\uc548 \uc7a5\ubaa8\uc778 \ube45\ud1a0\ub9ac\uc544 \uc5ec\uc655\uacfc \uc2a4\uc6e8\ub374\uacfc \ub178\ub974\uc6e8\uc774\uc758 \uad6d\uc655 \uc624\uc2a4\uce74\ub974 2\uc138\uc758 \uacf5\uc2dd \uc608\ubc29\uc744 \ubc1b\uc558\uace0, \ucc28\ub0a8\uc778 \ud558\uc778\ub9ac\ud788\uc640 \uc870\uce74\ub538\uc778 \uc774\ub808\ub124 \uacf5\uc8fc\uc758 \uacb0\ud63c\uc2dd\uc5d0\ub3c4 \ucc38\uc11d\ud558\uc600\ub2e4. \uadf8\ub7ec\ub098 \ud504\ub9ac\ub4dc\ub9ac\ud788\uc758 \uc7ac\uc704 \uae30\uac04\uc740 \uace0\uc791 99\uc77c\uc774\uc5c8\uace0, \uc774\ub7ec\ud55c \uc9e7\uc740 \uce58\uc138\ub85c \uc778\ud574 \uad04\ubaa9\ud560 \ub9cc\ud55c \ubcc0\ud654\ub97c \ub610\ud55c \uac00\uc838\uc624\uc9c0 \ubabb\ud588\ub2e4. \uc989\uc704 \uc9c1\uc804 \uadf8\uac00 \ubc1c\ud45c\ud55c \uce59\ub839\uc5d0\uc11c\ub294 \ud5cc\ubc95\uc5d0 \ub530\ub77c \ud669\uc81c\uc640 \uc81c\uad6d\uc218\uc0c1\uc758 \uad8c\ud55c\uc744 \uc81c\ud55c\ud560 \uac83\uc744 \uc57d\uc18d\ud558\uc600\uc73c\ub098 \uc774 \uce59\ub839\uc740 \ub05d\ub0b4 \uc2dc\ud589\ub418\uc9c0 \ubabb\ud588\ub2e4. 6\uc6d4 8\uc77c\uc5d0\ub294 \ud504\ub85c\uc774\uc13c \ub0b4\ubb34\uc7a5\uad00 \ub85c\ubca0\ub974\ud2b8 \ud3f0 \ud478\ud2b8\uce74\uba38 \uac00 \uc81c\uad6d\uc758\ud68c \uc120\uac70\uc5d0 \uad00\uc5ec\ud574 \uc628 \uc0ac\uc2e4\uc774 \ub4dc\ub7ec\ub098\uc790, \uadf8\ub97c \uacbd\uc9c8\ud558\uc600\ub2e4. \ub9e4\ucf04\uc9c0 \ubc15\uc0ac\ub294 \ud504\ub9ac\ub4dc\ub9ac\ud788\uac00 \ud669\uc81c\ub85c\uc11c\uc758 \ucc45\ubb34\uc5d0 \ub300\ud574 \uc555\ub3c4\uc801\uc778 \uac10\uac01\uc744 \uc9c0\ub2c8\uace0 \uc788\uc5c8\ub2e4\uace0 \ud68c\uace0\ud588\ub2e4. \ube45\ud1a0\ub9ac\uc544 \ud669\ud6c4\ub294 \ud504\ub79c\uc2dc\uc2a4 \ub124\uc774\ud53c\uc5b4\uc5d0\uac8c \ubcf4\ub0b4\ub294 \ud3b8\uc9c0\uc5d0\uc11c \u201c\ud669\uc81c\uaed8\uc120 \ud68c\uc758\uc5d0\ub3c4 \ucc38\uc11d\ud558\uc2dc\uace0 \ub9e4\uc6b0 \uc798\ud558\uace0 \uacc4\uc2dc\uba70 \uc5ec\uc804\ud788 \uc2dc\ub3c4\ub294 \ud558\uc2dc\uc9c0\ub9cc \uc544\uc9c1\ub3c4 \ub9d0\uc744 \ud558\uc2e4 \uc218 \uc5c6\ub2e4\u201d\ub77c\uace0 \uc37c\ub2e4. \uc5f4\uc815\uc774 \ub118\ucce4\uc73c\ub098 \uc790\uc2e0\uc774 \ub73b\ud558\ub358 \ubc14\ub97c \uc131\ucde8\uc2dc\ud0ac \uc2dc\uac04\uc740 \ub9ce\uc9c0 \uc54a\uc558\ub358 \ud504\ub9ac\ub4dc\ub9ac\ud788\ub294 1888\ub144 5\uc6d4\uc5d0 \u201c\ub09c \uc544\uc9c1 \uc8fd\uc744 \uc218 \uc5c6\ub2e4... \uc774\uc81c \ub3c5\uc77c\uc740 \uc5b4\ub5bb\uac8c \ub420 \uac83\uc778\uac00?\u201d\ub77c\uace0 \uc808\uaddc\ud558\uc600\ub2e4.", "paragraph_answer": "\ube44\ub85d \ubcd1\uc138\ub294 \uc810\uc810 \uc545\ud654\ub428\uc5d0\ub3c4 \ubd88\uad6c\ud558\uace0, \ud504\ub9ac\ub4dc\ub9ac\ud788 3\uc138\ub294 \ud669\uc81c\ub85c\uc11c\uc758 \ucc45\ubb34\ub97c \ub2e4\ud558\ub824 \ub178\ub825\ud558\uc600\ub2e4. \uc989\uc704 \ubc1c\ud45c \uc9c1\ud6c4, \ud504\ub9ac\ub4dc\ub9ac\ud788\ub294 \uc790\uc2e0\uc758 \uc815\uc7a5\uc5d0 \ubc15\ud600\uc788\ub294 \uac80\uc740 \ub3c5\uc218\ub9ac \ud6c8\uc7a5\uc744 \ube7c\uc11c \uc544\ub0b4 \ube45\ud1a0\ub9ac\uc544\uc758 \ub4dc\ub808\uc2a4\uc5d0 \ub2ec\uc544\uc92c\ub294\ub370, \uc774\ub294 \ube45\ud1a0\ub9ac\uc544\ub97c \ud669\ud6c4\ub85c \ub192\uc774\uaca0\ub2e4\ub294 \ub73b\uc744 \ucc9c\uba85\ud55c \uac83\uc774\uc5c8\ub2e4. \ud504\ub9ac\ub4dc\ub9ac\ud788\ub294 \uc7ac\uc704 \uae30\uac04 \ub3d9\uc548 \uc7a5\ubaa8\uc778 \ube45\ud1a0\ub9ac\uc544 \uc5ec\uc655\uacfc \uc2a4\uc6e8\ub374\uacfc \ub178\ub974\uc6e8\uc774\uc758 \uad6d\uc655 \uc624\uc2a4\uce74\ub974 2\uc138\uc758 \uacf5\uc2dd \uc608\ubc29\uc744 \ubc1b\uc558\uace0, \ucc28\ub0a8\uc778 \ud558\uc778\ub9ac\ud788\uc640 \uc870\uce74\ub538\uc778 \uc774\ub808\ub124 \uacf5\uc8fc\uc758 \uacb0\ud63c\uc2dd\uc5d0\ub3c4 \ucc38\uc11d\ud558\uc600\ub2e4. \uadf8\ub7ec\ub098 \ud504\ub9ac\ub4dc\ub9ac\ud788\uc758 \uc7ac\uc704 \uae30\uac04\uc740 \uace0\uc791 99\uc77c\uc774\uc5c8\uace0, \uc774\ub7ec\ud55c \uc9e7\uc740 \uce58\uc138\ub85c \uc778\ud574 \uad04\ubaa9\ud560 \ub9cc\ud55c \ubcc0\ud654\ub97c \ub610\ud55c \uac00\uc838\uc624\uc9c0 \ubabb\ud588\ub2e4. \uc989\uc704 \uc9c1\uc804 \uadf8\uac00 \ubc1c\ud45c\ud55c \uce59\ub839\uc5d0\uc11c\ub294 \ud5cc\ubc95\uc5d0 \ub530\ub77c \ud669\uc81c\uc640 \uc81c\uad6d\uc218\uc0c1\uc758 \uad8c\ud55c\uc744 \uc81c\ud55c\ud560 \uac83\uc744 \uc57d\uc18d\ud558\uc600\uc73c\ub098 \uc774 \uce59\ub839\uc740 \ub05d\ub0b4 \uc2dc\ud589\ub418\uc9c0 \ubabb\ud588\ub2e4. 6\uc6d4 8\uc77c\uc5d0\ub294 \ud504\ub85c\uc774\uc13c \ub0b4\ubb34\uc7a5\uad00 \ub85c\ubca0\ub974\ud2b8 \ud3f0 \ud478\ud2b8\uce74\uba38 \uac00 \uc81c\uad6d\uc758\ud68c \uc120\uac70\uc5d0 \uad00\uc5ec\ud574 \uc628 \uc0ac\uc2e4\uc774 \ub4dc\ub7ec\ub098\uc790, \uadf8\ub97c \uacbd\uc9c8\ud558\uc600\ub2e4. \ub9e4\ucf04\uc9c0 \ubc15\uc0ac\ub294 \ud504\ub9ac\ub4dc\ub9ac\ud788\uac00 \ud669\uc81c\ub85c\uc11c\uc758 \ucc45\ubb34\uc5d0 \ub300\ud574 \uc555\ub3c4\uc801\uc778 \uac10\uac01\uc744 \uc9c0\ub2c8\uace0 \uc788\uc5c8\ub2e4\uace0 \ud68c\uace0\ud588\ub2e4. \ube45\ud1a0\ub9ac\uc544 \ud669\ud6c4\ub294 \ud504\ub79c\uc2dc\uc2a4 \ub124\uc774\ud53c\uc5b4\uc5d0\uac8c \ubcf4\ub0b4\ub294 \ud3b8\uc9c0\uc5d0\uc11c \u201c\ud669\uc81c\uaed8\uc120 \ud68c\uc758\uc5d0\ub3c4 \ucc38\uc11d\ud558\uc2dc\uace0 \ub9e4\uc6b0 \uc798\ud558\uace0 \uacc4\uc2dc\uba70 \uc5ec\uc804\ud788 \uc2dc\ub3c4\ub294 \ud558\uc2dc\uc9c0\ub9cc \uc544\uc9c1\ub3c4 \ub9d0\uc744 \ud558\uc2e4 \uc218 \uc5c6\ub2e4\u201d\ub77c\uace0 \uc37c\ub2e4. \uc5f4\uc815\uc774 \ub118\ucce4\uc73c\ub098 \uc790\uc2e0\uc774 \ub73b\ud558\ub358 \ubc14\ub97c \uc131\ucde8\uc2dc\ud0ac \uc2dc\uac04\uc740 \ub9ce\uc9c0 \uc54a\uc558\ub358 \ud504\ub9ac\ub4dc\ub9ac\ud788\ub294 1888\ub144 5\uc6d4\uc5d0 \u201c\ub09c \uc544\uc9c1 \uc8fd\uc744 \uc218 \uc5c6\ub2e4... \uc774\uc81c \ub3c5\uc77c\uc740 \uc5b4\ub5bb\uac8c \ub420 \uac83\uc778\uac00?\u201d\ub77c\uace0 \uc808\uaddc\ud558\uc600\ub2e4.", "sentence_answer": "6\uc6d4 8\uc77c\uc5d0\ub294 \ud504\ub85c\uc774\uc13c \ub0b4\ubb34\uc7a5\uad00 \ub85c\ubca0\ub974\ud2b8 \ud3f0 \ud478\ud2b8\uce74\uba38 \uac00 \uc81c\uad6d\uc758\ud68c \uc120\uac70\uc5d0 \uad00\uc5ec\ud574 \uc628 \uc0ac\uc2e4\uc774 \ub4dc\ub7ec\ub098\uc790, \uadf8\ub97c \uacbd\uc9c8\ud558\uc600\ub2e4.", "paragraph_id": "6223777-12-1", "paragraph_question": "question: \ud504\ub9ac\ub4dc\ub9ac\ud788\uac00 \uc81c\uad6d\uc758\ud68c \uc120\uac70\uc5d0 \uad00\uc5ec\ud55c \uc774\uc720\ub85c \uacbd\uc9c8\ud588\ub358\n\ud504\ub85c\uc774\uc13c \ub0b4\ubb34\uc7a5\uad00 \uc774\ub984\uc740?, context: \ube44\ub85d \ubcd1\uc138\ub294 \uc810\uc810 \uc545\ud654\ub428\uc5d0\ub3c4 \ubd88\uad6c\ud558\uace0, \ud504\ub9ac\ub4dc\ub9ac\ud788 3\uc138\ub294 \ud669\uc81c\ub85c\uc11c\uc758 \ucc45\ubb34\ub97c \ub2e4\ud558\ub824 \ub178\ub825\ud558\uc600\ub2e4. \uc989\uc704 \ubc1c\ud45c \uc9c1\ud6c4, \ud504\ub9ac\ub4dc\ub9ac\ud788\ub294 \uc790\uc2e0\uc758 \uc815\uc7a5\uc5d0 \ubc15\ud600\uc788\ub294 \uac80\uc740 \ub3c5\uc218\ub9ac \ud6c8\uc7a5\uc744 \ube7c\uc11c \uc544\ub0b4 \ube45\ud1a0\ub9ac\uc544\uc758 \ub4dc\ub808\uc2a4\uc5d0 \ub2ec\uc544\uc92c\ub294\ub370, \uc774\ub294 \ube45\ud1a0\ub9ac\uc544\ub97c \ud669\ud6c4\ub85c \ub192\uc774\uaca0\ub2e4\ub294 \ub73b\uc744 \ucc9c\uba85\ud55c \uac83\uc774\uc5c8\ub2e4. \ud504\ub9ac\ub4dc\ub9ac\ud788\ub294 \uc7ac\uc704 \uae30\uac04 \ub3d9\uc548 \uc7a5\ubaa8\uc778 \ube45\ud1a0\ub9ac\uc544 \uc5ec\uc655\uacfc \uc2a4\uc6e8\ub374\uacfc \ub178\ub974\uc6e8\uc774\uc758 \uad6d\uc655 \uc624\uc2a4\uce74\ub974 2\uc138\uc758 \uacf5\uc2dd \uc608\ubc29\uc744 \ubc1b\uc558\uace0, \ucc28\ub0a8\uc778 \ud558\uc778\ub9ac\ud788\uc640 \uc870\uce74\ub538\uc778 \uc774\ub808\ub124 \uacf5\uc8fc\uc758 \uacb0\ud63c\uc2dd\uc5d0\ub3c4 \ucc38\uc11d\ud558\uc600\ub2e4. \uadf8\ub7ec\ub098 \ud504\ub9ac\ub4dc\ub9ac\ud788\uc758 \uc7ac\uc704 \uae30\uac04\uc740 \uace0\uc791 99\uc77c\uc774\uc5c8\uace0, \uc774\ub7ec\ud55c \uc9e7\uc740 \uce58\uc138\ub85c \uc778\ud574 \uad04\ubaa9\ud560 \ub9cc\ud55c \ubcc0\ud654\ub97c \ub610\ud55c \uac00\uc838\uc624\uc9c0 \ubabb\ud588\ub2e4. \uc989\uc704 \uc9c1\uc804 \uadf8\uac00 \ubc1c\ud45c\ud55c \uce59\ub839\uc5d0\uc11c\ub294 \ud5cc\ubc95\uc5d0 \ub530\ub77c \ud669\uc81c\uc640 \uc81c\uad6d\uc218\uc0c1\uc758 \uad8c\ud55c\uc744 \uc81c\ud55c\ud560 \uac83\uc744 \uc57d\uc18d\ud558\uc600\uc73c\ub098 \uc774 \uce59\ub839\uc740 \ub05d\ub0b4 \uc2dc\ud589\ub418\uc9c0 \ubabb\ud588\ub2e4. 6\uc6d4 8\uc77c\uc5d0\ub294 \ud504\ub85c\uc774\uc13c \ub0b4\ubb34\uc7a5\uad00 \ub85c\ubca0\ub974\ud2b8 \ud3f0 \ud478\ud2b8\uce74\uba38\uac00 \uc81c\uad6d\uc758\ud68c \uc120\uac70\uc5d0 \uad00\uc5ec\ud574 \uc628 \uc0ac\uc2e4\uc774 \ub4dc\ub7ec\ub098\uc790, \uadf8\ub97c \uacbd\uc9c8\ud558\uc600\ub2e4. \ub9e4\ucf04\uc9c0 \ubc15\uc0ac\ub294 \ud504\ub9ac\ub4dc\ub9ac\ud788\uac00 \ud669\uc81c\ub85c\uc11c\uc758 \ucc45\ubb34\uc5d0 \ub300\ud574 \uc555\ub3c4\uc801\uc778 \uac10\uac01\uc744 \uc9c0\ub2c8\uace0 \uc788\uc5c8\ub2e4\uace0 \ud68c\uace0\ud588\ub2e4. \ube45\ud1a0\ub9ac\uc544 \ud669\ud6c4\ub294 \ud504\ub79c\uc2dc\uc2a4 \ub124\uc774\ud53c\uc5b4\uc5d0\uac8c \ubcf4\ub0b4\ub294 \ud3b8\uc9c0\uc5d0\uc11c \u201c\ud669\uc81c\uaed8\uc120 \ud68c\uc758\uc5d0\ub3c4 \ucc38\uc11d\ud558\uc2dc\uace0 \ub9e4\uc6b0 \uc798\ud558\uace0 \uacc4\uc2dc\uba70 \uc5ec\uc804\ud788 \uc2dc\ub3c4\ub294 \ud558\uc2dc\uc9c0\ub9cc \uc544\uc9c1\ub3c4 \ub9d0\uc744 \ud558\uc2e4 \uc218 \uc5c6\ub2e4\u201d\ub77c\uace0 \uc37c\ub2e4. \uc5f4\uc815\uc774 \ub118\ucce4\uc73c\ub098 \uc790\uc2e0\uc774 \ub73b\ud558\ub358 \ubc14\ub97c \uc131\ucde8\uc2dc\ud0ac \uc2dc\uac04\uc740 \ub9ce\uc9c0 \uc54a\uc558\ub358 \ud504\ub9ac\ub4dc\ub9ac\ud788\ub294 1888\ub144 5\uc6d4\uc5d0 \u201c\ub09c \uc544\uc9c1 \uc8fd\uc744 \uc218 \uc5c6\ub2e4... \uc774\uc81c \ub3c5\uc77c\uc740 \uc5b4\ub5bb\uac8c \ub420 \uac83\uc778\uac00?\u201d\ub77c\uace0 \uc808\uaddc\ud558\uc600\ub2e4."} {"question": "\ud3f4\ub780\ub4dc-\ubaa8\uc2a4\ud06c\ubc14 \uc804\uc7c1\uc5d0\uc11c \ub7ec\uc2dc\uc544\uad70\uc774 5\ubc30\ub098 \ub354 \ub9ce\uc740 \ubcd1\ub825\uc744 \uac00\uc9c0\uace0 \uc788\uc5c8\uc74c\uc5d0\ub3c4 \ud328\ud558\uc600\ub294\ub370 \uc774 \uc804\ud22c\uc758 \uba85\uce6d\uc740 \ubb34\uc5c7\uc778\uac00?", "paragraph": "1577\ub144 \ub8e8\ube44\uc170\ud504 \uc804\ud22c(Battle of Lubiszew)\uc5d0\uc11c \ud6c4\uc0ac\ub974 \ubd80\ub300\uc758 '\ud669\uae08 \uc2dc\ub300'(Golden Age)\uac00 \uc2dc\uc791\ub418\uc5c8\ub2e4. 1683\ub144 \ube48 \uc804\ud22c(Battle of Vienna)\uc5d0 \uc774\ub974\uae30\uae4c\uc9c0 \ud3f4\ub780\ub4dc-\ub9ac\ud22c\uc544\ub2c8\uc544 \ud6c4\uc0ac\ub974 \ubd80\ub300\ub294 \uc218\ub9ce\uc740 \uc801\ub4e4\uc5d0 \ub300\ud56d\ud558\uc5ec \ubb34\uc218\ud55c \uad70\uc0ac\uc801 \ud589\ub3d9\uc744 \ubc8c\uc600\uace0, \uac70\uc758 \ud328\ud55c \uc801\uc774 \uc5c6\uc5c8\ub2e4. 1577\ub144 \ub8e8\ube44\uc170\ud504 \uc804\ud22c(Battle of Lubiszew), \ube44\uce58\ub098(Byczyna 1588), \ucf54\ucf04\ud558\uc6b0\uc820(Kokenhausen 1601), \ud0a4\ub974\ud640\ub984(Kircholm 1605), \ud06c\uc6b0\uc2e0(K\u0142uszyn 1610), \ud2b8\uc288\uce58\uc544\ub098(Trzciana 1629), \ud638\uce68(Chocim 1673), \ub974\ubd80\ud504(Lw\u00f3w 1675)\uc640 \uac19\uc740 \uc77c\ub828\uc758 \uc804\ud22c\uc5d0\uc11c \ud3f4\ub780\ub4dc-\ub9ac\ud22c\uc544\ub2c8\uc544 \ud6c4\uc0ac\ub974 \ubd80\ub300\ub294 \ubb34\uc218\ud788 \ub9ce\uc740 \uc801\uad70\uc744 \uaca9\ud30c\ud558\ub294 \ub370 \uacb0\uc815\uc801\uc778 \uc5ed\ud560\uc744 \ud558\uc600\ub2e4. \uc608\ub97c \ub4e4\uc5b4 \ud3f4\ub780\ub4dc-\ubaa8\uc2a4\ud06c\ubc14 \uc804\uc7c1(Polish-Muscovite War) \ub3d9\uc548\uc5d0 \ubc8c\uc5b4\uc9c4 \ud06c\uc6b0\uc2e0 \uc804\ud22c(Battle of Kluszyn)\uc5d0\uc11c \ub7ec\uc2dc\uc544\uad70\uc740 \ud3f4\ub780\ub4dc-\ub9ac\ud22c\uc544\ub2c8\uc544 \uad70\uc5d0 \ube44\ud574 5\ubc30\ub098 \ub9ce\uc558\uc73c\ub098 \ud6c4\uc0ac\ub974\uc758 \ud65c\uc57d\uc5d0 \uc758\ud574 \ub300\ud328\ud558\uace0 \ub9d0\uc558\ub2e4.", "answer": "\ud06c\uc6b0\uc2e0 \uc804\ud22c", "sentence": "\uc608\ub97c \ub4e4\uc5b4 \ud3f4\ub780\ub4dc-\ubaa8\uc2a4\ud06c\ubc14 \uc804\uc7c1(Polish-Muscovite War) \ub3d9\uc548\uc5d0 \ubc8c\uc5b4\uc9c4 \ud06c\uc6b0\uc2e0 \uc804\ud22c (Battle of Kluszyn)\uc5d0\uc11c \ub7ec\uc2dc\uc544\uad70\uc740 \ud3f4\ub780\ub4dc-\ub9ac\ud22c\uc544\ub2c8\uc544 \uad70\uc5d0 \ube44\ud574 5\ubc30\ub098 \ub9ce\uc558\uc73c\ub098 \ud6c4\uc0ac\ub974\uc758 \ud65c\uc57d\uc5d0 \uc758\ud574 \ub300\ud328\ud558\uace0 \ub9d0\uc558\ub2e4.", "paragraph_sentence": "1577\ub144 \ub8e8\ube44\uc170\ud504 \uc804\ud22c(Battle of Lubiszew)\uc5d0\uc11c \ud6c4\uc0ac\ub974 \ubd80\ub300\uc758 '\ud669\uae08 \uc2dc\ub300'(Golden Age)\uac00 \uc2dc\uc791\ub418\uc5c8\ub2e4. 1683\ub144 \ube48 \uc804\ud22c(Battle of Vienna)\uc5d0 \uc774\ub974\uae30\uae4c\uc9c0 \ud3f4\ub780\ub4dc-\ub9ac\ud22c\uc544\ub2c8\uc544 \ud6c4\uc0ac\ub974 \ubd80\ub300\ub294 \uc218\ub9ce\uc740 \uc801\ub4e4\uc5d0 \ub300\ud56d\ud558\uc5ec \ubb34\uc218\ud55c \uad70\uc0ac\uc801 \ud589\ub3d9\uc744 \ubc8c\uc600\uace0, \uac70\uc758 \ud328\ud55c \uc801\uc774 \uc5c6\uc5c8\ub2e4. 1577\ub144 \ub8e8\ube44\uc170\ud504 \uc804\ud22c(Battle of Lubiszew), \ube44\uce58\ub098(Byczyna 1588), \ucf54\ucf04\ud558\uc6b0\uc820(Kokenhausen 1601), \ud0a4\ub974\ud640\ub984(Kircholm 1605), \ud06c\uc6b0\uc2e0(K\u0142uszyn 1610), \ud2b8\uc288\uce58\uc544\ub098(Trzciana 1629), \ud638\uce68(Chocim 1673), \ub974\ubd80\ud504(Lw\u00f3w 1675)\uc640 \uac19\uc740 \uc77c\ub828\uc758 \uc804\ud22c\uc5d0\uc11c \ud3f4\ub780\ub4dc-\ub9ac\ud22c\uc544\ub2c8\uc544 \ud6c4\uc0ac\ub974 \ubd80\ub300\ub294 \ubb34\uc218\ud788 \ub9ce\uc740 \uc801\uad70\uc744 \uaca9\ud30c\ud558\ub294 \ub370 \uacb0\uc815\uc801\uc778 \uc5ed\ud560\uc744 \ud558\uc600\ub2e4. \uc608\ub97c \ub4e4\uc5b4 \ud3f4\ub780\ub4dc-\ubaa8\uc2a4\ud06c\ubc14 \uc804\uc7c1(Polish-Muscovite War) \ub3d9\uc548\uc5d0 \ubc8c\uc5b4\uc9c4 \ud06c\uc6b0\uc2e0 \uc804\ud22c (Battle of Kluszyn)\uc5d0\uc11c \ub7ec\uc2dc\uc544\uad70\uc740 \ud3f4\ub780\ub4dc-\ub9ac\ud22c\uc544\ub2c8\uc544 \uad70\uc5d0 \ube44\ud574 5\ubc30\ub098 \ub9ce\uc558\uc73c\ub098 \ud6c4\uc0ac\ub974\uc758 \ud65c\uc57d\uc5d0 \uc758\ud574 \ub300\ud328\ud558\uace0 \ub9d0\uc558\ub2e4. ", "paragraph_answer": "1577\ub144 \ub8e8\ube44\uc170\ud504 \uc804\ud22c(Battle of Lubiszew)\uc5d0\uc11c \ud6c4\uc0ac\ub974 \ubd80\ub300\uc758 '\ud669\uae08 \uc2dc\ub300'(Golden Age)\uac00 \uc2dc\uc791\ub418\uc5c8\ub2e4. 1683\ub144 \ube48 \uc804\ud22c(Battle of Vienna)\uc5d0 \uc774\ub974\uae30\uae4c\uc9c0 \ud3f4\ub780\ub4dc-\ub9ac\ud22c\uc544\ub2c8\uc544 \ud6c4\uc0ac\ub974 \ubd80\ub300\ub294 \uc218\ub9ce\uc740 \uc801\ub4e4\uc5d0 \ub300\ud56d\ud558\uc5ec \ubb34\uc218\ud55c \uad70\uc0ac\uc801 \ud589\ub3d9\uc744 \ubc8c\uc600\uace0, \uac70\uc758 \ud328\ud55c \uc801\uc774 \uc5c6\uc5c8\ub2e4. 1577\ub144 \ub8e8\ube44\uc170\ud504 \uc804\ud22c(Battle of Lubiszew), \ube44\uce58\ub098(Byczyna 1588), \ucf54\ucf04\ud558\uc6b0\uc820(Kokenhausen 1601), \ud0a4\ub974\ud640\ub984(Kircholm 1605), \ud06c\uc6b0\uc2e0(K\u0142uszyn 1610), \ud2b8\uc288\uce58\uc544\ub098(Trzciana 1629), \ud638\uce68(Chocim 1673), \ub974\ubd80\ud504(Lw\u00f3w 1675)\uc640 \uac19\uc740 \uc77c\ub828\uc758 \uc804\ud22c\uc5d0\uc11c \ud3f4\ub780\ub4dc-\ub9ac\ud22c\uc544\ub2c8\uc544 \ud6c4\uc0ac\ub974 \ubd80\ub300\ub294 \ubb34\uc218\ud788 \ub9ce\uc740 \uc801\uad70\uc744 \uaca9\ud30c\ud558\ub294 \ub370 \uacb0\uc815\uc801\uc778 \uc5ed\ud560\uc744 \ud558\uc600\ub2e4. \uc608\ub97c \ub4e4\uc5b4 \ud3f4\ub780\ub4dc-\ubaa8\uc2a4\ud06c\ubc14 \uc804\uc7c1(Polish-Muscovite War) \ub3d9\uc548\uc5d0 \ubc8c\uc5b4\uc9c4 \ud06c\uc6b0\uc2e0 \uc804\ud22c (Battle of Kluszyn)\uc5d0\uc11c \ub7ec\uc2dc\uc544\uad70\uc740 \ud3f4\ub780\ub4dc-\ub9ac\ud22c\uc544\ub2c8\uc544 \uad70\uc5d0 \ube44\ud574 5\ubc30\ub098 \ub9ce\uc558\uc73c\ub098 \ud6c4\uc0ac\ub974\uc758 \ud65c\uc57d\uc5d0 \uc758\ud574 \ub300\ud328\ud558\uace0 \ub9d0\uc558\ub2e4.", "sentence_answer": "\uc608\ub97c \ub4e4\uc5b4 \ud3f4\ub780\ub4dc-\ubaa8\uc2a4\ud06c\ubc14 \uc804\uc7c1(Polish-Muscovite War) \ub3d9\uc548\uc5d0 \ubc8c\uc5b4\uc9c4 \ud06c\uc6b0\uc2e0 \uc804\ud22c (Battle of Kluszyn)\uc5d0\uc11c \ub7ec\uc2dc\uc544\uad70\uc740 \ud3f4\ub780\ub4dc-\ub9ac\ud22c\uc544\ub2c8\uc544 \uad70\uc5d0 \ube44\ud574 5\ubc30\ub098 \ub9ce\uc558\uc73c\ub098 \ud6c4\uc0ac\ub974\uc758 \ud65c\uc57d\uc5d0 \uc758\ud574 \ub300\ud328\ud558\uace0 \ub9d0\uc558\ub2e4.", "paragraph_id": "6568140-2-1", "paragraph_question": "question: \ud3f4\ub780\ub4dc-\ubaa8\uc2a4\ud06c\ubc14 \uc804\uc7c1\uc5d0\uc11c \ub7ec\uc2dc\uc544\uad70\uc774 5\ubc30\ub098 \ub354 \ub9ce\uc740 \ubcd1\ub825\uc744 \uac00\uc9c0\uace0 \uc788\uc5c8\uc74c\uc5d0\ub3c4 \ud328\ud558\uc600\ub294\ub370 \uc774 \uc804\ud22c\uc758 \uba85\uce6d\uc740 \ubb34\uc5c7\uc778\uac00?, context: 1577\ub144 \ub8e8\ube44\uc170\ud504 \uc804\ud22c(Battle of Lubiszew)\uc5d0\uc11c \ud6c4\uc0ac\ub974 \ubd80\ub300\uc758 '\ud669\uae08 \uc2dc\ub300'(Golden Age)\uac00 \uc2dc\uc791\ub418\uc5c8\ub2e4. 1683\ub144 \ube48 \uc804\ud22c(Battle of Vienna)\uc5d0 \uc774\ub974\uae30\uae4c\uc9c0 \ud3f4\ub780\ub4dc-\ub9ac\ud22c\uc544\ub2c8\uc544 \ud6c4\uc0ac\ub974 \ubd80\ub300\ub294 \uc218\ub9ce\uc740 \uc801\ub4e4\uc5d0 \ub300\ud56d\ud558\uc5ec \ubb34\uc218\ud55c \uad70\uc0ac\uc801 \ud589\ub3d9\uc744 \ubc8c\uc600\uace0, \uac70\uc758 \ud328\ud55c \uc801\uc774 \uc5c6\uc5c8\ub2e4. 1577\ub144 \ub8e8\ube44\uc170\ud504 \uc804\ud22c(Battle of Lubiszew), \ube44\uce58\ub098(Byczyna 1588), \ucf54\ucf04\ud558\uc6b0\uc820(Kokenhausen 1601), \ud0a4\ub974\ud640\ub984(Kircholm 1605), \ud06c\uc6b0\uc2e0(K\u0142uszyn 1610), \ud2b8\uc288\uce58\uc544\ub098(Trzciana 1629), \ud638\uce68(Chocim 1673), \ub974\ubd80\ud504(Lw\u00f3w 1675)\uc640 \uac19\uc740 \uc77c\ub828\uc758 \uc804\ud22c\uc5d0\uc11c \ud3f4\ub780\ub4dc-\ub9ac\ud22c\uc544\ub2c8\uc544 \ud6c4\uc0ac\ub974 \ubd80\ub300\ub294 \ubb34\uc218\ud788 \ub9ce\uc740 \uc801\uad70\uc744 \uaca9\ud30c\ud558\ub294 \ub370 \uacb0\uc815\uc801\uc778 \uc5ed\ud560\uc744 \ud558\uc600\ub2e4. \uc608\ub97c \ub4e4\uc5b4 \ud3f4\ub780\ub4dc-\ubaa8\uc2a4\ud06c\ubc14 \uc804\uc7c1(Polish-Muscovite War) \ub3d9\uc548\uc5d0 \ubc8c\uc5b4\uc9c4 \ud06c\uc6b0\uc2e0 \uc804\ud22c(Battle of Kluszyn)\uc5d0\uc11c \ub7ec\uc2dc\uc544\uad70\uc740 \ud3f4\ub780\ub4dc-\ub9ac\ud22c\uc544\ub2c8\uc544 \uad70\uc5d0 \ube44\ud574 5\ubc30\ub098 \ub9ce\uc558\uc73c\ub098 \ud6c4\uc0ac\ub974\uc758 \ud65c\uc57d\uc5d0 \uc758\ud574 \ub300\ud328\ud558\uace0 \ub9d0\uc558\ub2e4."} {"question": "\ud328\uc158\uacfc \ub304\uc2a4\uc5d0\uc11c\ub9cc \uc55e\uc11c\uac04\ub2e4\uace0 \uc0dd\uac01\ud558\uc9c0\ub9cc \uc0ac\uc2e4 \uc54c\uace0\ubcf4\uba74 \uc11c\uc778\uc601\uc758 \uac00\ucc3d\ub825\uc740 \uc0c1\uc0c1\uc774\uc0c1\uc774\ub77c\uace0 \ud3c9\uac00\ud55c \uacf3\uc740?", "paragraph": "\uc11c\uc778\uc601\uc740 \uc194\ub85c \ub370\ubdd4 \uc774\ud6c4 \ub304\uc2a4, \uac00\ucc3d\ub825 \ubaa8\ub450 \uc778\uc815 \ubc1b\uace0 \uc788\ub294 \uac00\uc218\uc774\ub2e4. \u300a\ub274\uc2a4\uc5d4\u300b\uc5d0\uc11c\ub294 \"\ub9ce\uc740 \uc774\ub4e4\uc774 \uc11c\uc778\uc601\uc774 \ud328\uc158\uacfc \ub304\uc2a4\uc5d0\uc11c\ub9cc \uc55e\uc11c\uac04\ub2e4\uace0 \uc0dd\uac01\ud558\uc9c0\ub9cc \uc0ac\uc2e4 \uc54c\uace0\ubcf4\uba74 \uc11c\uc778\uc601\uc758 \uac00\ucc3d\ub825\uc740 \uc0c1\uc0c1 \uc774\uc0c1\uc774\ub2e4\"\ub77c\uace0 \ub9d0\ud588\ub2e4. \u300a\uc77c\uac04\uc2a4\ud3ec\uce20\u300b\ub294 \"\uc548\uc815\ub41c \uac00\ucc3d\ub825\uc744 \ubc14\ud0d5\uc73c\ub85c, \uc644\uc131\ud615\uc5d0 \uc774\ub978 \ubcf4\uceec\ub9ac\uc2a4\ud2b8\uc9c0\ub9cc \uac00\ucc3d\ub825\uc744 \uc778\uc815\ubc1b\uc740 \uacbd\uc6b0\ub294 \ub9ce\uc9c0 \uc54a\ub2e4. \ub304\uc2a4 \uc2e4\ub825\uae4c\uc9c0 \uacb8\ube44\ud574 \ud654\ub824\ud55c \ud37c\ud3ec\uba3c\uc2a4\ub97c \uc120\ubcf4\uc774\ub2e4 \ubcf4\ub2c8 \uc624\ud788\ub824 \uac00\ucc3d\ub825\uc774 \ubb3b\ud600\ubc84\ub9b0 \uacbd\uc6b0\ub2e4\"\ub77c\uba70 \uc2e4\ub825\uc5d0 \ube44\ud574 \uc74c\uc545\uc801\uc73c\ub85c \uc800\ud3c9\uac00\ub41c \ub300\ud45c\uc801 \uac00\uc218\ub77c\uace0 \uc37c\ub2e4. \u300a\ubd88\ud6c4\uc758 \uba85\uace1 2 - \uc804\uc124\uc744 \ub178\ub798\ud558\ub2e4\u300b\uc758 \ud2b9\uc9d1\uc73c\ub85c \ucd9c\uc5f0 \ub2f9\uc2dc \uc7c1\uc7c1\ud55c \uc5ec\uac00\uc218\ub4e4\uc744 \uc774\uae30\uace0 \uc6b0\uc2b9\ud558\uba70 \uc2e0\uc2b9\ud6c8\uc740 \uc11c\uc778\uc601\uc5d0\uac8c \"\ubaa9\uc18c\ub9ac\ub97c \uac16\uace0 \ub178\ub294 \ub2a5\ub825\uc774 \uc788\ub2e4\"\uace0 \ub9d0\ud588\ub2e4.", "answer": "\ub274\uc2a4\uc5d4", "sentence": "\u300a \ub274\uc2a4\uc5d4 \u300b\uc5d0\uc11c\ub294 \"\ub9ce\uc740 \uc774\ub4e4\uc774 \uc11c\uc778\uc601\uc774 \ud328\uc158\uacfc \ub304\uc2a4\uc5d0\uc11c\ub9cc \uc55e\uc11c\uac04\ub2e4\uace0 \uc0dd\uac01\ud558\uc9c0\ub9cc \uc0ac\uc2e4 \uc54c\uace0\ubcf4\uba74 \uc11c\uc778\uc601\uc758 \uac00\ucc3d\ub825\uc740 \uc0c1\uc0c1 \uc774\uc0c1\uc774\ub2e4\"\ub77c\uace0 \ub9d0\ud588\ub2e4.", "paragraph_sentence": "\uc11c\uc778\uc601\uc740 \uc194\ub85c \ub370\ubdd4 \uc774\ud6c4 \ub304\uc2a4, \uac00\ucc3d\ub825 \ubaa8\ub450 \uc778\uc815 \ubc1b\uace0 \uc788\ub294 \uac00\uc218\uc774\ub2e4. \u300a \ub274\uc2a4\uc5d4 \u300b\uc5d0\uc11c\ub294 \"\ub9ce\uc740 \uc774\ub4e4\uc774 \uc11c\uc778\uc601\uc774 \ud328\uc158\uacfc \ub304\uc2a4\uc5d0\uc11c\ub9cc \uc55e\uc11c\uac04\ub2e4\uace0 \uc0dd\uac01\ud558\uc9c0\ub9cc \uc0ac\uc2e4 \uc54c\uace0\ubcf4\uba74 \uc11c\uc778\uc601\uc758 \uac00\ucc3d\ub825\uc740 \uc0c1\uc0c1 \uc774\uc0c1\uc774\ub2e4\"\ub77c\uace0 \ub9d0\ud588\ub2e4. \u300a\uc77c\uac04\uc2a4\ud3ec\uce20\u300b\ub294 \"\uc548\uc815\ub41c \uac00\ucc3d\ub825\uc744 \ubc14\ud0d5\uc73c\ub85c, \uc644\uc131\ud615\uc5d0 \uc774\ub978 \ubcf4\uceec\ub9ac\uc2a4\ud2b8\uc9c0\ub9cc \uac00\ucc3d\ub825\uc744 \uc778\uc815\ubc1b\uc740 \uacbd\uc6b0\ub294 \ub9ce\uc9c0 \uc54a\ub2e4. \ub304\uc2a4 \uc2e4\ub825\uae4c\uc9c0 \uacb8\ube44\ud574 \ud654\ub824\ud55c \ud37c\ud3ec\uba3c\uc2a4\ub97c \uc120\ubcf4\uc774\ub2e4 \ubcf4\ub2c8 \uc624\ud788\ub824 \uac00\ucc3d\ub825\uc774 \ubb3b\ud600\ubc84\ub9b0 \uacbd\uc6b0\ub2e4\"\ub77c\uba70 \uc2e4\ub825\uc5d0 \ube44\ud574 \uc74c\uc545\uc801\uc73c\ub85c \uc800\ud3c9\uac00\ub41c \ub300\ud45c\uc801 \uac00\uc218\ub77c\uace0 \uc37c\ub2e4. \u300a\ubd88\ud6c4\uc758 \uba85\uace1 2 - \uc804\uc124\uc744 \ub178\ub798\ud558\ub2e4\u300b\uc758 \ud2b9\uc9d1\uc73c\ub85c \ucd9c\uc5f0 \ub2f9\uc2dc \uc7c1\uc7c1\ud55c \uc5ec\uac00\uc218\ub4e4\uc744 \uc774\uae30\uace0 \uc6b0\uc2b9\ud558\uba70 \uc2e0\uc2b9\ud6c8\uc740 \uc11c\uc778\uc601\uc5d0\uac8c \"\ubaa9\uc18c\ub9ac\ub97c \uac16\uace0 \ub178\ub294 \ub2a5\ub825\uc774 \uc788\ub2e4\"\uace0 \ub9d0\ud588\ub2e4.", "paragraph_answer": "\uc11c\uc778\uc601\uc740 \uc194\ub85c \ub370\ubdd4 \uc774\ud6c4 \ub304\uc2a4, \uac00\ucc3d\ub825 \ubaa8\ub450 \uc778\uc815 \ubc1b\uace0 \uc788\ub294 \uac00\uc218\uc774\ub2e4. \u300a \ub274\uc2a4\uc5d4 \u300b\uc5d0\uc11c\ub294 \"\ub9ce\uc740 \uc774\ub4e4\uc774 \uc11c\uc778\uc601\uc774 \ud328\uc158\uacfc \ub304\uc2a4\uc5d0\uc11c\ub9cc \uc55e\uc11c\uac04\ub2e4\uace0 \uc0dd\uac01\ud558\uc9c0\ub9cc \uc0ac\uc2e4 \uc54c\uace0\ubcf4\uba74 \uc11c\uc778\uc601\uc758 \uac00\ucc3d\ub825\uc740 \uc0c1\uc0c1 \uc774\uc0c1\uc774\ub2e4\"\ub77c\uace0 \ub9d0\ud588\ub2e4. \u300a\uc77c\uac04\uc2a4\ud3ec\uce20\u300b\ub294 \"\uc548\uc815\ub41c \uac00\ucc3d\ub825\uc744 \ubc14\ud0d5\uc73c\ub85c, \uc644\uc131\ud615\uc5d0 \uc774\ub978 \ubcf4\uceec\ub9ac\uc2a4\ud2b8\uc9c0\ub9cc \uac00\ucc3d\ub825\uc744 \uc778\uc815\ubc1b\uc740 \uacbd\uc6b0\ub294 \ub9ce\uc9c0 \uc54a\ub2e4. \ub304\uc2a4 \uc2e4\ub825\uae4c\uc9c0 \uacb8\ube44\ud574 \ud654\ub824\ud55c \ud37c\ud3ec\uba3c\uc2a4\ub97c \uc120\ubcf4\uc774\ub2e4 \ubcf4\ub2c8 \uc624\ud788\ub824 \uac00\ucc3d\ub825\uc774 \ubb3b\ud600\ubc84\ub9b0 \uacbd\uc6b0\ub2e4\"\ub77c\uba70 \uc2e4\ub825\uc5d0 \ube44\ud574 \uc74c\uc545\uc801\uc73c\ub85c \uc800\ud3c9\uac00\ub41c \ub300\ud45c\uc801 \uac00\uc218\ub77c\uace0 \uc37c\ub2e4. \u300a\ubd88\ud6c4\uc758 \uba85\uace1 2 - \uc804\uc124\uc744 \ub178\ub798\ud558\ub2e4\u300b\uc758 \ud2b9\uc9d1\uc73c\ub85c \ucd9c\uc5f0 \ub2f9\uc2dc \uc7c1\uc7c1\ud55c \uc5ec\uac00\uc218\ub4e4\uc744 \uc774\uae30\uace0 \uc6b0\uc2b9\ud558\uba70 \uc2e0\uc2b9\ud6c8\uc740 \uc11c\uc778\uc601\uc5d0\uac8c \"\ubaa9\uc18c\ub9ac\ub97c \uac16\uace0 \ub178\ub294 \ub2a5\ub825\uc774 \uc788\ub2e4\"\uace0 \ub9d0\ud588\ub2e4.", "sentence_answer": "\u300a \ub274\uc2a4\uc5d4 \u300b\uc5d0\uc11c\ub294 \"\ub9ce\uc740 \uc774\ub4e4\uc774 \uc11c\uc778\uc601\uc774 \ud328\uc158\uacfc \ub304\uc2a4\uc5d0\uc11c\ub9cc \uc55e\uc11c\uac04\ub2e4\uace0 \uc0dd\uac01\ud558\uc9c0\ub9cc \uc0ac\uc2e4 \uc54c\uace0\ubcf4\uba74 \uc11c\uc778\uc601\uc758 \uac00\ucc3d\ub825\uc740 \uc0c1\uc0c1 \uc774\uc0c1\uc774\ub2e4\"\ub77c\uace0 \ub9d0\ud588\ub2e4.", "paragraph_id": "6517609-9-1", "paragraph_question": "question: \ud328\uc158\uacfc \ub304\uc2a4\uc5d0\uc11c\ub9cc \uc55e\uc11c\uac04\ub2e4\uace0 \uc0dd\uac01\ud558\uc9c0\ub9cc \uc0ac\uc2e4 \uc54c\uace0\ubcf4\uba74 \uc11c\uc778\uc601\uc758 \uac00\ucc3d\ub825\uc740 \uc0c1\uc0c1\uc774\uc0c1\uc774\ub77c\uace0 \ud3c9\uac00\ud55c \uacf3\uc740?, context: \uc11c\uc778\uc601\uc740 \uc194\ub85c \ub370\ubdd4 \uc774\ud6c4 \ub304\uc2a4, \uac00\ucc3d\ub825 \ubaa8\ub450 \uc778\uc815 \ubc1b\uace0 \uc788\ub294 \uac00\uc218\uc774\ub2e4. \u300a\ub274\uc2a4\uc5d4\u300b\uc5d0\uc11c\ub294 \"\ub9ce\uc740 \uc774\ub4e4\uc774 \uc11c\uc778\uc601\uc774 \ud328\uc158\uacfc \ub304\uc2a4\uc5d0\uc11c\ub9cc \uc55e\uc11c\uac04\ub2e4\uace0 \uc0dd\uac01\ud558\uc9c0\ub9cc \uc0ac\uc2e4 \uc54c\uace0\ubcf4\uba74 \uc11c\uc778\uc601\uc758 \uac00\ucc3d\ub825\uc740 \uc0c1\uc0c1 \uc774\uc0c1\uc774\ub2e4\"\ub77c\uace0 \ub9d0\ud588\ub2e4. \u300a\uc77c\uac04\uc2a4\ud3ec\uce20\u300b\ub294 \"\uc548\uc815\ub41c \uac00\ucc3d\ub825\uc744 \ubc14\ud0d5\uc73c\ub85c, \uc644\uc131\ud615\uc5d0 \uc774\ub978 \ubcf4\uceec\ub9ac\uc2a4\ud2b8\uc9c0\ub9cc \uac00\ucc3d\ub825\uc744 \uc778\uc815\ubc1b\uc740 \uacbd\uc6b0\ub294 \ub9ce\uc9c0 \uc54a\ub2e4. \ub304\uc2a4 \uc2e4\ub825\uae4c\uc9c0 \uacb8\ube44\ud574 \ud654\ub824\ud55c \ud37c\ud3ec\uba3c\uc2a4\ub97c \uc120\ubcf4\uc774\ub2e4 \ubcf4\ub2c8 \uc624\ud788\ub824 \uac00\ucc3d\ub825\uc774 \ubb3b\ud600\ubc84\ub9b0 \uacbd\uc6b0\ub2e4\"\ub77c\uba70 \uc2e4\ub825\uc5d0 \ube44\ud574 \uc74c\uc545\uc801\uc73c\ub85c \uc800\ud3c9\uac00\ub41c \ub300\ud45c\uc801 \uac00\uc218\ub77c\uace0 \uc37c\ub2e4. \u300a\ubd88\ud6c4\uc758 \uba85\uace1 2 - \uc804\uc124\uc744 \ub178\ub798\ud558\ub2e4\u300b\uc758 \ud2b9\uc9d1\uc73c\ub85c \ucd9c\uc5f0 \ub2f9\uc2dc \uc7c1\uc7c1\ud55c \uc5ec\uac00\uc218\ub4e4\uc744 \uc774\uae30\uace0 \uc6b0\uc2b9\ud558\uba70 \uc2e0\uc2b9\ud6c8\uc740 \uc11c\uc778\uc601\uc5d0\uac8c \"\ubaa9\uc18c\ub9ac\ub97c \uac16\uace0 \ub178\ub294 \ub2a5\ub825\uc774 \uc788\ub2e4\"\uace0 \ub9d0\ud588\ub2e4."} {"question": "\ub18d\uc2ec\uc744 \uc2b9\uacc4\ud560\uac83\uc73c\ub85c \ucd94\uce21\ub418\ub294 \uc544\ub4e4\uc758 \uc774\ub984\uc740?", "paragraph": "\ub18d\uc2ec\uadf8\ub8f9\uc740 \uc0ac\uc5c5\uc8fc \uc77c\uac00\uac00 \ub18d\uc2ec\ud640\ub529\uc2a4\ub97c \ud1b5\ud574 \uc2dd\ud488\uc0ac\uc5c5\uc778 '\ub18d\uc2ec'\uacfc \ud654\ud559\uc0ac\uc5c5\uc778 '\uc728\ucd0c\ud654\ud559'\uc744 \uacbd\uc601\ud558\uace0 \uc788\ub2e4. \ub18d\uc2ec\uc740 \ucc3d\uc5c5\uc8fc\uc778 \uc2e0\ucd98\ud638 \ud68c\uc7a5 \uccb4\uc81c\uc5d0\uc11c 2\uc138 \uacbd\uc601\uc774 \ud55c\ucc3d \uc9c4\ud589 \uc911\uc774\ub2e4. \uadf8 \uc8fc\uc778\uacf5\uc740 \uc2e0 \ud68c\uc7a5\uc758 \uc138 \uc544\ub4e4\uc778 \uc2e0\ub3d9\uc6d0(\uc7a5\ub0a8) \ub18d\uc2ec \ubd80\ud68c\uc7a5\uacfc \uc2e0\ub3d9\uc724(\ucc28\ub0a8) \uc728\ucd0c\ud654\ud559 \ubd80\ud68c\uc7a5, \uc2e0\ub3d9\uc775(\uc0bc\ub0a8) \uba54\uac00\ub9c8\ud2b8 \ubd80\ud68c\uc7a5\uc774\ub2e4. \uc7a5\ub0a8\uc5d0\uac8c\ub294 \uc2dd\ud488\uc0ac\uc5c5\uc744, \ucc28\ub0a8\uc5d0\uac8c\ub294 \ud654\ud559\uc0ac\uc5c5\uc744 \ub9e1\uae30\uace0, \uc0bc\ub0a8\uc5d0\uac8c\ub294 \uc720\ud1b5\ud68c\uc0ac\uc778 \uba54\uac00\ub9c8\ud2b8 \uacbd\uc601\uc744 \ub118\uae30\uba70 \uacc4\uc5f4 \ubd84\ub9ac\ub97c \uc900\ube44\ud574\uc654\ub2e4. \uc2e0\ucd98\ud638 \ud68c\uc7a5\uc740 2\uc138 \uc911 \ub204\uad6c\uc5d0\uac8c \ud68c\uc0ac \ub97c \ub9e1\uae30\uaca0\ub2e4\uace0 \uacf5\uc2dd\uc801\uc73c\ub85c \ubc1d\ud78c \uc801\uc740 \uc5c6\ub2e4. \ub2e4\ub9cc 20 \uc5ec \ub144 \uc804\ubd80\ud130 \uc5c5\ubb34\ub97c \ucc28\ubcc4\ud654\ud558\uace0, \uc9c0\uc8fc\uc0ac\uc778 \ub18d\uc2ec\ud640\ub529\uc2a4 \uc9c0\ubd84\uc744 \ucc28\ub4f1 \ubc30\ubd84\ud558\ub294 \ubc29\uc2dd\uc73c\ub85c \ud6c4\uacc4\uad6c\ub3c4\ub97c \uc7a4\ub2e4. 2017\ub144 5\uc6d4\uacfc 6\uc6d4\uc5d0 \uc9c4\ud589\ub41c \ub18d\uc2ec\uc758 \uc9c0\ubd84 \ubcc0\ud654\uac00 \ub208\uc5d0 \ub748\ub2e4. 5\uc6d4 4\uc77c \uc2e0\ub3d9\uc6d0 \ub18d\uc2ec \ubd80\ud68c\uc7a5\uacfc \uc2e0\ub3d9\uc724 \uc728\ucd0c\ud654\ud559 \ubd80\ud68c\uc7a5\uc740 \uc790\uc2e0\ub4e4\uc774 \ubcf4\uc720\ud558\ub358 \uc0c1\ub300\ubc29 \ud68c\uc0ac\uc758 \uc8fc\uc2dd\uc744 \uc11c\ub85c \uc8fc\uace0\ubc1b\uc544 \uac01\uc790\uc758 \ud68c\uc0ac \uc9c0\ubd84\uc744 \ub298\ub838\ub2e4. \uc774 \uc8fc\uc2dd \uad50\ud658\uc73c\ub85c \uc7a5\ub0a8\uc740 \ub18d\uc2ec\ud640\ub529\uc2a4 \uc9c0\ubd84\uc744 36.93%\uc5d0\uc11c 42.92%\ub85c \ub298\ub838\uace0, \ucc28\ub0a8\uc740 \uc728\ucd0c\ud654\ud559 \uc9c0\ubd84\uc744 5.10%\uc5d0\uc11c 13.93%\ub85c \ud655\ub300\ud588\ub2e4. \ub18d\uc2ec\uc758 \uc9c0\uc8fc\ud68c\uc0ac\uc778 \ub18d\uc2ec\ud640\ub529\uc2a4\uc758 \ucd5c\ub300\uc8fc\uc8fc\uc778 \uc2e0\ub3d9\uc6d0 \ubd80\ud68c\uc7a5\uc740 \uc9c0\ubc30\ub825\uc774 \ud55c\uce35 \uac15\ud654\ub410\ub2e4. 6\uc6d4 1\uc77c\uc5d0\ub294 \uc2e0\ucd98\ud638 \ud68c\uc7a5\uc774 \uc790\uc2e0\uc758 \ub18d\uc2ec \uc8fc\uc2dd 10\ub9cc\uc8fc\ub97c \uc2e0\ub3d9\uc775 \uba54\uac00\ub9c8\ud2b8 \ubd80\ud68c\uc7a5\uc5d0\uac8c \uc99d\uc5ec\ud588\ub2e4. \uc774 \uc99d\uc5ec\ub85c \uc14b\uc9f8 \uc544\ub4e4\uc740 1.64%\uc758 \uc9c0\ubd84\uc728\uc774\uc9c0\ub9cc \ucc98\uc74c\uc73c\ub85c \ub18d\uc2ec \uc8fc\uc2dd\uc744 \ubcf4\uc720\ud558\uac8c \ub410\ub2e4. \ub610 \ub18d\uc2ec\uadf8\ub8f9\uc758 \ub9e4\ucd9c \uc911 \uc808\ubc18 \uac00\ub7c9\uc744 \ucc28\uc9c0 \ud558\uace0 \uc788\ub294 '\ub18d\uc2ec'\uc744 \uc7a5\ub0a8\uc778 \uc2e0\ub3d9\uc6d0 \ubd80\ud68c\uc7a5\uc774 \uc774\ub04c\uace0 \uc788\ub2e4. \uc774 \ub54c\ubb38\uc5d0 \ub18d\uc2ec\uc740 \uc7a5\uc790\uc778 \uc2e0\ub3d9\uc6d0 \ubd80\ud68c\uc7a5\uc5d0\uac8c \uc2b9\uacc4\ub420 \uac83\uc73c\ub85c \ucd94\uce21\ub41c\ub2e4. \ub610 \uc0bc\ub0a8\uc5d0\uac8c \ub18d\uc2ec \uc9c0\ubd84 \uc77c\ubd80\ub97c \uc99d\uc5ec\ud55c \uac83\uc740 \uc7a5\ub0a8\uacfc \ucc28\ub0a8\uc5d0 \ube44\ud574 \ubaab\uc774 \uc801\uc740 \uc0bc\ub0a8\uc5d0\uac8c \uc77c\ubd80 \uc9c0\uc6d0\ud55c \uac83\uc774 \uc544\ub2c8\ub0d0\ub294 \ucd94\uce21\uc774\ub2e4.", "answer": "\uc2e0\ub3d9\uc6d0", "sentence": "\uadf8 \uc8fc\uc778\uacf5\uc740 \uc2e0 \ud68c\uc7a5\uc758 \uc138 \uc544\ub4e4\uc778 \uc2e0\ub3d9\uc6d0 (\uc7a5\ub0a8)", "paragraph_sentence": "\ub18d\uc2ec\uadf8\ub8f9\uc740 \uc0ac\uc5c5\uc8fc \uc77c\uac00\uac00 \ub18d\uc2ec\ud640\ub529\uc2a4\ub97c \ud1b5\ud574 \uc2dd\ud488\uc0ac\uc5c5\uc778 '\ub18d\uc2ec'\uacfc \ud654\ud559\uc0ac\uc5c5\uc778 '\uc728\ucd0c\ud654\ud559'\uc744 \uacbd\uc601\ud558\uace0 \uc788\ub2e4. \ub18d\uc2ec\uc740 \ucc3d\uc5c5\uc8fc\uc778 \uc2e0\ucd98\ud638 \ud68c\uc7a5 \uccb4\uc81c\uc5d0\uc11c 2\uc138 \uacbd\uc601\uc774 \ud55c\ucc3d \uc9c4\ud589 \uc911\uc774\ub2e4. \uadf8 \uc8fc\uc778\uacf5\uc740 \uc2e0 \ud68c\uc7a5\uc758 \uc138 \uc544\ub4e4\uc778 \uc2e0\ub3d9\uc6d0 (\uc7a5\ub0a8) \ub18d\uc2ec \ubd80\ud68c\uc7a5\uacfc \uc2e0\ub3d9\uc724(\ucc28\ub0a8) \uc728\ucd0c\ud654\ud559 \ubd80\ud68c\uc7a5, \uc2e0\ub3d9\uc775(\uc0bc\ub0a8) \uba54\uac00\ub9c8\ud2b8 \ubd80\ud68c\uc7a5\uc774\ub2e4. \uc7a5\ub0a8\uc5d0\uac8c\ub294 \uc2dd\ud488\uc0ac\uc5c5\uc744, \ucc28\ub0a8\uc5d0\uac8c\ub294 \ud654\ud559\uc0ac\uc5c5\uc744 \ub9e1\uae30\uace0, \uc0bc\ub0a8\uc5d0\uac8c\ub294 \uc720\ud1b5\ud68c\uc0ac\uc778 \uba54\uac00\ub9c8\ud2b8 \uacbd\uc601\uc744 \ub118\uae30\uba70 \uacc4\uc5f4 \ubd84\ub9ac\ub97c \uc900\ube44\ud574\uc654\ub2e4. \uc2e0\ucd98\ud638 \ud68c\uc7a5\uc740 2\uc138 \uc911 \ub204\uad6c\uc5d0\uac8c \ud68c\uc0ac \ub97c \ub9e1\uae30\uaca0\ub2e4\uace0 \uacf5\uc2dd\uc801\uc73c\ub85c \ubc1d\ud78c \uc801\uc740 \uc5c6\ub2e4. \ub2e4\ub9cc 20 \uc5ec \ub144 \uc804\ubd80\ud130 \uc5c5\ubb34\ub97c \ucc28\ubcc4\ud654\ud558\uace0, \uc9c0\uc8fc\uc0ac\uc778 \ub18d\uc2ec\ud640\ub529\uc2a4 \uc9c0\ubd84\uc744 \ucc28\ub4f1 \ubc30\ubd84\ud558\ub294 \ubc29\uc2dd\uc73c\ub85c \ud6c4\uacc4\uad6c\ub3c4\ub97c \uc7a4\ub2e4. 2017\ub144 5\uc6d4\uacfc 6\uc6d4\uc5d0 \uc9c4\ud589\ub41c \ub18d\uc2ec\uc758 \uc9c0\ubd84 \ubcc0\ud654\uac00 \ub208\uc5d0 \ub748\ub2e4. 5\uc6d4 4\uc77c \uc2e0\ub3d9\uc6d0 \ub18d\uc2ec \ubd80\ud68c\uc7a5\uacfc \uc2e0\ub3d9\uc724 \uc728\ucd0c\ud654\ud559 \ubd80\ud68c\uc7a5\uc740 \uc790\uc2e0\ub4e4\uc774 \ubcf4\uc720\ud558\ub358 \uc0c1\ub300\ubc29 \ud68c\uc0ac\uc758 \uc8fc\uc2dd\uc744 \uc11c\ub85c \uc8fc\uace0\ubc1b\uc544 \uac01\uc790\uc758 \ud68c\uc0ac \uc9c0\ubd84\uc744 \ub298\ub838\ub2e4. \uc774 \uc8fc\uc2dd \uad50\ud658\uc73c\ub85c \uc7a5\ub0a8\uc740 \ub18d\uc2ec\ud640\ub529\uc2a4 \uc9c0\ubd84\uc744 36.93%\uc5d0\uc11c 42.92%\ub85c \ub298\ub838\uace0, \ucc28\ub0a8\uc740 \uc728\ucd0c\ud654\ud559 \uc9c0\ubd84\uc744 5.10%\uc5d0\uc11c 13.93%\ub85c \ud655\ub300\ud588\ub2e4. \ub18d\uc2ec\uc758 \uc9c0\uc8fc\ud68c\uc0ac\uc778 \ub18d\uc2ec\ud640\ub529\uc2a4\uc758 \ucd5c\ub300\uc8fc\uc8fc\uc778 \uc2e0\ub3d9\uc6d0 \ubd80\ud68c\uc7a5\uc740 \uc9c0\ubc30\ub825\uc774 \ud55c\uce35 \uac15\ud654\ub410\ub2e4. 6\uc6d4 1\uc77c\uc5d0\ub294 \uc2e0\ucd98\ud638 \ud68c\uc7a5\uc774 \uc790\uc2e0\uc758 \ub18d\uc2ec \uc8fc\uc2dd 10\ub9cc\uc8fc\ub97c \uc2e0\ub3d9\uc775 \uba54\uac00\ub9c8\ud2b8 \ubd80\ud68c\uc7a5\uc5d0\uac8c \uc99d\uc5ec\ud588\ub2e4. \uc774 \uc99d\uc5ec\ub85c \uc14b\uc9f8 \uc544\ub4e4\uc740 1.64%\uc758 \uc9c0\ubd84\uc728\uc774\uc9c0\ub9cc \ucc98\uc74c\uc73c\ub85c \ub18d\uc2ec \uc8fc\uc2dd\uc744 \ubcf4\uc720\ud558\uac8c \ub410\ub2e4. \ub610 \ub18d\uc2ec\uadf8\ub8f9\uc758 \ub9e4\ucd9c \uc911 \uc808\ubc18 \uac00\ub7c9\uc744 \ucc28\uc9c0 \ud558\uace0 \uc788\ub294 '\ub18d\uc2ec'\uc744 \uc7a5\ub0a8\uc778 \uc2e0\ub3d9\uc6d0 \ubd80\ud68c\uc7a5\uc774 \uc774\ub04c\uace0 \uc788\ub2e4. \uc774 \ub54c\ubb38\uc5d0 \ub18d\uc2ec\uc740 \uc7a5\uc790\uc778 \uc2e0\ub3d9\uc6d0 \ubd80\ud68c\uc7a5\uc5d0\uac8c \uc2b9\uacc4\ub420 \uac83\uc73c\ub85c \ucd94\uce21\ub41c\ub2e4. \ub610 \uc0bc\ub0a8\uc5d0\uac8c \ub18d\uc2ec \uc9c0\ubd84 \uc77c\ubd80\ub97c \uc99d\uc5ec\ud55c \uac83\uc740 \uc7a5\ub0a8\uacfc \ucc28\ub0a8\uc5d0 \ube44\ud574 \ubaab\uc774 \uc801\uc740 \uc0bc\ub0a8\uc5d0\uac8c \uc77c\ubd80 \uc9c0\uc6d0\ud55c \uac83\uc774 \uc544\ub2c8\ub0d0\ub294 \ucd94\uce21\uc774\ub2e4.", "paragraph_answer": "\ub18d\uc2ec\uadf8\ub8f9\uc740 \uc0ac\uc5c5\uc8fc \uc77c\uac00\uac00 \ub18d\uc2ec\ud640\ub529\uc2a4\ub97c \ud1b5\ud574 \uc2dd\ud488\uc0ac\uc5c5\uc778 '\ub18d\uc2ec'\uacfc \ud654\ud559\uc0ac\uc5c5\uc778 '\uc728\ucd0c\ud654\ud559'\uc744 \uacbd\uc601\ud558\uace0 \uc788\ub2e4. \ub18d\uc2ec\uc740 \ucc3d\uc5c5\uc8fc\uc778 \uc2e0\ucd98\ud638 \ud68c\uc7a5 \uccb4\uc81c\uc5d0\uc11c 2\uc138 \uacbd\uc601\uc774 \ud55c\ucc3d \uc9c4\ud589 \uc911\uc774\ub2e4. \uadf8 \uc8fc\uc778\uacf5\uc740 \uc2e0 \ud68c\uc7a5\uc758 \uc138 \uc544\ub4e4\uc778 \uc2e0\ub3d9\uc6d0 (\uc7a5\ub0a8) \ub18d\uc2ec \ubd80\ud68c\uc7a5\uacfc \uc2e0\ub3d9\uc724(\ucc28\ub0a8) \uc728\ucd0c\ud654\ud559 \ubd80\ud68c\uc7a5, \uc2e0\ub3d9\uc775(\uc0bc\ub0a8) \uba54\uac00\ub9c8\ud2b8 \ubd80\ud68c\uc7a5\uc774\ub2e4. \uc7a5\ub0a8\uc5d0\uac8c\ub294 \uc2dd\ud488\uc0ac\uc5c5\uc744, \ucc28\ub0a8\uc5d0\uac8c\ub294 \ud654\ud559\uc0ac\uc5c5\uc744 \ub9e1\uae30\uace0, \uc0bc\ub0a8\uc5d0\uac8c\ub294 \uc720\ud1b5\ud68c\uc0ac\uc778 \uba54\uac00\ub9c8\ud2b8 \uacbd\uc601\uc744 \ub118\uae30\uba70 \uacc4\uc5f4 \ubd84\ub9ac\ub97c \uc900\ube44\ud574\uc654\ub2e4. \uc2e0\ucd98\ud638 \ud68c\uc7a5\uc740 2\uc138 \uc911 \ub204\uad6c\uc5d0\uac8c \ud68c\uc0ac \ub97c \ub9e1\uae30\uaca0\ub2e4\uace0 \uacf5\uc2dd\uc801\uc73c\ub85c \ubc1d\ud78c \uc801\uc740 \uc5c6\ub2e4. \ub2e4\ub9cc 20 \uc5ec \ub144 \uc804\ubd80\ud130 \uc5c5\ubb34\ub97c \ucc28\ubcc4\ud654\ud558\uace0, \uc9c0\uc8fc\uc0ac\uc778 \ub18d\uc2ec\ud640\ub529\uc2a4 \uc9c0\ubd84\uc744 \ucc28\ub4f1 \ubc30\ubd84\ud558\ub294 \ubc29\uc2dd\uc73c\ub85c \ud6c4\uacc4\uad6c\ub3c4\ub97c \uc7a4\ub2e4. 2017\ub144 5\uc6d4\uacfc 6\uc6d4\uc5d0 \uc9c4\ud589\ub41c \ub18d\uc2ec\uc758 \uc9c0\ubd84 \ubcc0\ud654\uac00 \ub208\uc5d0 \ub748\ub2e4. 5\uc6d4 4\uc77c \uc2e0\ub3d9\uc6d0 \ub18d\uc2ec \ubd80\ud68c\uc7a5\uacfc \uc2e0\ub3d9\uc724 \uc728\ucd0c\ud654\ud559 \ubd80\ud68c\uc7a5\uc740 \uc790\uc2e0\ub4e4\uc774 \ubcf4\uc720\ud558\ub358 \uc0c1\ub300\ubc29 \ud68c\uc0ac\uc758 \uc8fc\uc2dd\uc744 \uc11c\ub85c \uc8fc\uace0\ubc1b\uc544 \uac01\uc790\uc758 \ud68c\uc0ac \uc9c0\ubd84\uc744 \ub298\ub838\ub2e4. \uc774 \uc8fc\uc2dd \uad50\ud658\uc73c\ub85c \uc7a5\ub0a8\uc740 \ub18d\uc2ec\ud640\ub529\uc2a4 \uc9c0\ubd84\uc744 36.93%\uc5d0\uc11c 42.92%\ub85c \ub298\ub838\uace0, \ucc28\ub0a8\uc740 \uc728\ucd0c\ud654\ud559 \uc9c0\ubd84\uc744 5.10%\uc5d0\uc11c 13.93%\ub85c \ud655\ub300\ud588\ub2e4. \ub18d\uc2ec\uc758 \uc9c0\uc8fc\ud68c\uc0ac\uc778 \ub18d\uc2ec\ud640\ub529\uc2a4\uc758 \ucd5c\ub300\uc8fc\uc8fc\uc778 \uc2e0\ub3d9\uc6d0 \ubd80\ud68c\uc7a5\uc740 \uc9c0\ubc30\ub825\uc774 \ud55c\uce35 \uac15\ud654\ub410\ub2e4. 6\uc6d4 1\uc77c\uc5d0\ub294 \uc2e0\ucd98\ud638 \ud68c\uc7a5\uc774 \uc790\uc2e0\uc758 \ub18d\uc2ec \uc8fc\uc2dd 10\ub9cc\uc8fc\ub97c \uc2e0\ub3d9\uc775 \uba54\uac00\ub9c8\ud2b8 \ubd80\ud68c\uc7a5\uc5d0\uac8c \uc99d\uc5ec\ud588\ub2e4. \uc774 \uc99d\uc5ec\ub85c \uc14b\uc9f8 \uc544\ub4e4\uc740 1.64%\uc758 \uc9c0\ubd84\uc728\uc774\uc9c0\ub9cc \ucc98\uc74c\uc73c\ub85c \ub18d\uc2ec \uc8fc\uc2dd\uc744 \ubcf4\uc720\ud558\uac8c \ub410\ub2e4. \ub610 \ub18d\uc2ec\uadf8\ub8f9\uc758 \ub9e4\ucd9c \uc911 \uc808\ubc18 \uac00\ub7c9\uc744 \ucc28\uc9c0 \ud558\uace0 \uc788\ub294 '\ub18d\uc2ec'\uc744 \uc7a5\ub0a8\uc778 \uc2e0\ub3d9\uc6d0 \ubd80\ud68c\uc7a5\uc774 \uc774\ub04c\uace0 \uc788\ub2e4. \uc774 \ub54c\ubb38\uc5d0 \ub18d\uc2ec\uc740 \uc7a5\uc790\uc778 \uc2e0\ub3d9\uc6d0 \ubd80\ud68c\uc7a5\uc5d0\uac8c \uc2b9\uacc4\ub420 \uac83\uc73c\ub85c \ucd94\uce21\ub41c\ub2e4. \ub610 \uc0bc\ub0a8\uc5d0\uac8c \ub18d\uc2ec \uc9c0\ubd84 \uc77c\ubd80\ub97c \uc99d\uc5ec\ud55c \uac83\uc740 \uc7a5\ub0a8\uacfc \ucc28\ub0a8\uc5d0 \ube44\ud574 \ubaab\uc774 \uc801\uc740 \uc0bc\ub0a8\uc5d0\uac8c \uc77c\ubd80 \uc9c0\uc6d0\ud55c \uac83\uc774 \uc544\ub2c8\ub0d0\ub294 \ucd94\uce21\uc774\ub2e4.", "sentence_answer": "\uadf8 \uc8fc\uc778\uacf5\uc740 \uc2e0 \ud68c\uc7a5\uc758 \uc138 \uc544\ub4e4\uc778 \uc2e0\ub3d9\uc6d0 (\uc7a5\ub0a8)", "paragraph_id": "6467462-1-1", "paragraph_question": "question: \ub18d\uc2ec\uc744 \uc2b9\uacc4\ud560\uac83\uc73c\ub85c \ucd94\uce21\ub418\ub294 \uc544\ub4e4\uc758 \uc774\ub984\uc740?, context: \ub18d\uc2ec\uadf8\ub8f9\uc740 \uc0ac\uc5c5\uc8fc \uc77c\uac00\uac00 \ub18d\uc2ec\ud640\ub529\uc2a4\ub97c \ud1b5\ud574 \uc2dd\ud488\uc0ac\uc5c5\uc778 '\ub18d\uc2ec'\uacfc \ud654\ud559\uc0ac\uc5c5\uc778 '\uc728\ucd0c\ud654\ud559'\uc744 \uacbd\uc601\ud558\uace0 \uc788\ub2e4. \ub18d\uc2ec\uc740 \ucc3d\uc5c5\uc8fc\uc778 \uc2e0\ucd98\ud638 \ud68c\uc7a5 \uccb4\uc81c\uc5d0\uc11c 2\uc138 \uacbd\uc601\uc774 \ud55c\ucc3d \uc9c4\ud589 \uc911\uc774\ub2e4. \uadf8 \uc8fc\uc778\uacf5\uc740 \uc2e0 \ud68c\uc7a5\uc758 \uc138 \uc544\ub4e4\uc778 \uc2e0\ub3d9\uc6d0(\uc7a5\ub0a8) \ub18d\uc2ec \ubd80\ud68c\uc7a5\uacfc \uc2e0\ub3d9\uc724(\ucc28\ub0a8) \uc728\ucd0c\ud654\ud559 \ubd80\ud68c\uc7a5, \uc2e0\ub3d9\uc775(\uc0bc\ub0a8) \uba54\uac00\ub9c8\ud2b8 \ubd80\ud68c\uc7a5\uc774\ub2e4. \uc7a5\ub0a8\uc5d0\uac8c\ub294 \uc2dd\ud488\uc0ac\uc5c5\uc744, \ucc28\ub0a8\uc5d0\uac8c\ub294 \ud654\ud559\uc0ac\uc5c5\uc744 \ub9e1\uae30\uace0, \uc0bc\ub0a8\uc5d0\uac8c\ub294 \uc720\ud1b5\ud68c\uc0ac\uc778 \uba54\uac00\ub9c8\ud2b8 \uacbd\uc601\uc744 \ub118\uae30\uba70 \uacc4\uc5f4 \ubd84\ub9ac\ub97c \uc900\ube44\ud574\uc654\ub2e4. \uc2e0\ucd98\ud638 \ud68c\uc7a5\uc740 2\uc138 \uc911 \ub204\uad6c\uc5d0\uac8c \ud68c\uc0ac \ub97c \ub9e1\uae30\uaca0\ub2e4\uace0 \uacf5\uc2dd\uc801\uc73c\ub85c \ubc1d\ud78c \uc801\uc740 \uc5c6\ub2e4. \ub2e4\ub9cc 20 \uc5ec \ub144 \uc804\ubd80\ud130 \uc5c5\ubb34\ub97c \ucc28\ubcc4\ud654\ud558\uace0, \uc9c0\uc8fc\uc0ac\uc778 \ub18d\uc2ec\ud640\ub529\uc2a4 \uc9c0\ubd84\uc744 \ucc28\ub4f1 \ubc30\ubd84\ud558\ub294 \ubc29\uc2dd\uc73c\ub85c \ud6c4\uacc4\uad6c\ub3c4\ub97c \uc7a4\ub2e4. 2017\ub144 5\uc6d4\uacfc 6\uc6d4\uc5d0 \uc9c4\ud589\ub41c \ub18d\uc2ec\uc758 \uc9c0\ubd84 \ubcc0\ud654\uac00 \ub208\uc5d0 \ub748\ub2e4. 5\uc6d4 4\uc77c \uc2e0\ub3d9\uc6d0 \ub18d\uc2ec \ubd80\ud68c\uc7a5\uacfc \uc2e0\ub3d9\uc724 \uc728\ucd0c\ud654\ud559 \ubd80\ud68c\uc7a5\uc740 \uc790\uc2e0\ub4e4\uc774 \ubcf4\uc720\ud558\ub358 \uc0c1\ub300\ubc29 \ud68c\uc0ac\uc758 \uc8fc\uc2dd\uc744 \uc11c\ub85c \uc8fc\uace0\ubc1b\uc544 \uac01\uc790\uc758 \ud68c\uc0ac \uc9c0\ubd84\uc744 \ub298\ub838\ub2e4. \uc774 \uc8fc\uc2dd \uad50\ud658\uc73c\ub85c \uc7a5\ub0a8\uc740 \ub18d\uc2ec\ud640\ub529\uc2a4 \uc9c0\ubd84\uc744 36.93%\uc5d0\uc11c 42.92%\ub85c \ub298\ub838\uace0, \ucc28\ub0a8\uc740 \uc728\ucd0c\ud654\ud559 \uc9c0\ubd84\uc744 5.10%\uc5d0\uc11c 13.93%\ub85c \ud655\ub300\ud588\ub2e4. \ub18d\uc2ec\uc758 \uc9c0\uc8fc\ud68c\uc0ac\uc778 \ub18d\uc2ec\ud640\ub529\uc2a4\uc758 \ucd5c\ub300\uc8fc\uc8fc\uc778 \uc2e0\ub3d9\uc6d0 \ubd80\ud68c\uc7a5\uc740 \uc9c0\ubc30\ub825\uc774 \ud55c\uce35 \uac15\ud654\ub410\ub2e4. 6\uc6d4 1\uc77c\uc5d0\ub294 \uc2e0\ucd98\ud638 \ud68c\uc7a5\uc774 \uc790\uc2e0\uc758 \ub18d\uc2ec \uc8fc\uc2dd 10\ub9cc\uc8fc\ub97c \uc2e0\ub3d9\uc775 \uba54\uac00\ub9c8\ud2b8 \ubd80\ud68c\uc7a5\uc5d0\uac8c \uc99d\uc5ec\ud588\ub2e4. \uc774 \uc99d\uc5ec\ub85c \uc14b\uc9f8 \uc544\ub4e4\uc740 1.64%\uc758 \uc9c0\ubd84\uc728\uc774\uc9c0\ub9cc \ucc98\uc74c\uc73c\ub85c \ub18d\uc2ec \uc8fc\uc2dd\uc744 \ubcf4\uc720\ud558\uac8c \ub410\ub2e4. \ub610 \ub18d\uc2ec\uadf8\ub8f9\uc758 \ub9e4\ucd9c \uc911 \uc808\ubc18 \uac00\ub7c9\uc744 \ucc28\uc9c0 \ud558\uace0 \uc788\ub294 '\ub18d\uc2ec'\uc744 \uc7a5\ub0a8\uc778 \uc2e0\ub3d9\uc6d0 \ubd80\ud68c\uc7a5\uc774 \uc774\ub04c\uace0 \uc788\ub2e4. \uc774 \ub54c\ubb38\uc5d0 \ub18d\uc2ec\uc740 \uc7a5\uc790\uc778 \uc2e0\ub3d9\uc6d0 \ubd80\ud68c\uc7a5\uc5d0\uac8c \uc2b9\uacc4\ub420 \uac83\uc73c\ub85c \ucd94\uce21\ub41c\ub2e4. \ub610 \uc0bc\ub0a8\uc5d0\uac8c \ub18d\uc2ec \uc9c0\ubd84 \uc77c\ubd80\ub97c \uc99d\uc5ec\ud55c \uac83\uc740 \uc7a5\ub0a8\uacfc \ucc28\ub0a8\uc5d0 \ube44\ud574 \ubaab\uc774 \uc801\uc740 \uc0bc\ub0a8\uc5d0\uac8c \uc77c\ubd80 \uc9c0\uc6d0\ud55c \uac83\uc774 \uc544\ub2c8\ub0d0\ub294 \ucd94\uce21\uc774\ub2e4."} {"question": "\uacfc\ud559 \uae30\uc220\uc758 \ubc1c\uc804\uc744 \ub3c5\ub824\ud558\uba70 \uc11c\uc720\uacac\ubb38\uc744 \uc4f4 \uc800\uc790\ub294?", "paragraph": "\uadf8\ub294 \uc0c1\uacf5\uc5c5 \ubc0f \ubb34\uc5ed\uc758 \uc9c4\ud765, \uadfc\ub300\uc801\uc778 \ud654\ud3d0 \ubc0f \uc870\uc138\uc81c\ub3c4\uc758 \uc218\ub9bd, \uadfc\ub300\uc801\uc778 \uad50\uc721\uc81c\ub3c4\uc758 \uc2e4\uc2dc\ub97c \ub2f9\uba74 \uacfc\uc81c\ub85c \uc81c\uc2dc\ud558\uc600\ub2e4. \uc720\uae38\uc900\uc740 \uacc4\ubabd \uac15\uc5f0\uc5d0\uc11c\ub3c4 \uc2e0\ub77c\uc758 \uc0bc\uad6d\ud1b5\uc77c, \uc2e0\ub77c\uc758 \uc774\ub450, \uace0\ub824\uc2dc\ub300\uc758 \uae08\uc18d\ud65c\uc790, \uc870\uc120\uc758 \uac70\ubd81\uc120, \uace0\ub824\uccad\uc790, \uc870\uc120\ubc31\uc790, \ucd5c\ubb34\uc120\uc758 \ud654\ud3ec\uc640 \ucd1d\ud1b5, \uc870\uc120 \ucd08\uae30\uc758 \uc2e0\uae30\uc804 \ub4f1 \uae30\uc220\uc801 \ud601\uc2e0\uc744 \uc774\ub8e9\ud574 \uc654\ub2e4\uace0 \uc8fc\uc7a5\ud588\ub2e4. \uadf8\ub294 \uc790\uc2e0\uc758 \uc800\uc11c \u3008\uc11c\uc720\uacac\ubb38 \u897f\u904a\u898b\u805e\u3009\uc5d0\uc11c\ub3c4 \uc870\uc120\uc758 \uacfc\ud559\uae30\uc220\uc774 \ub0b3\uc740 \uc704\ub300\ud55c \uc131\uacfc\ub85c \uc774\ub450, \uace0\ub824\uc790\uae30, \uac70\ubd81\uc120, \uae08\uc18d\ud65c\uc790\ub97c \ub4e4\uace0 \uc788\ub2e4. \uadf8\ub7f0\ub370 \uace0\ub824 \ub9d0\ubd80\ud130 \uc870\uc120\uc655\uc870 5\ubc31\ub144\uac04 \ubb38\uc778\ub4e4\ub9cc\uc744 \uc6b0\ub300\ud558\uace0, \uae30\uc220\uc790\uc640 \uad70\uc778\ub4e4\uc740 \ucc9c\ub300\ud558\uba74\uc11c \uc774\ub7ec\ud55c \ud6cc\ub96d\ud55c \uae30\uc220\uacfc \uc131\uacfc\ub4e4\uc774 \uc0ac\uc7a5\ub418\uc5c8\uc74c\uc744 \uc5ed\uc124\ud588\ub2e4. \uadf8\ub294 \ub9cc\uc57d \ud6c4\uc190\ub4e4\uc774 \ube44\uc2e4\uc6a9\uc801\uc778 \ud559\ubb38\uc5d0 \uc9d1\ucc29\ud558\uc9c0 \uc54a\uace0, \uc774\ub7f0 \uacfc\ud559 \uae30\uc220 \uc804\ud1b5\uc744 \uc5f0\uad6c, \ubc1c\uc804\uc2dc\ucf30\ub354\ub77c\uba74 \uc9c0\uae08 \uc138\uacc4\uc758 \uc601\uad11\uc774 \uc870\uc120\uc5d0 \ub3cc\ub824\uc84c\uc744 \uac83\uc774\uc9c0\ub9cc, \ud6c4\uc190\ub4e4\uc774 \uadf8\ub807\uac8c \ud558\uc9c0 \ubabb\ud588\uc74c\uc744 \uc548\ud0c0\uae4c\uc6cc\ud558\uc600\ub2e4. \uadf8\ub294 \uacfc\ud559, \uae30\uc220\uc758 \ucc9c\ub300\uac00 \uc870\uc120\uc744 \uba78\ub9dd\uc758 \ub098\ub77d\uc73c\ub85c \uc774\ub04c\uc5c8\ub2e4\uace0 \ud655\uc2e0\ud588\ub2e4.", "answer": "\uc720\uae38\uc900", "sentence": "\uc720\uae38\uc900 \uc740 \uacc4\ubabd \uac15\uc5f0\uc5d0\uc11c\ub3c4 \uc2e0\ub77c\uc758 \uc0bc\uad6d\ud1b5\uc77c, \uc2e0\ub77c\uc758 \uc774\ub450, \uace0\ub824\uc2dc\ub300\uc758 \uae08\uc18d\ud65c\uc790, \uc870\uc120\uc758 \uac70\ubd81\uc120, \uace0\ub824\uccad\uc790, \uc870\uc120\ubc31\uc790, \ucd5c\ubb34\uc120\uc758 \ud654\ud3ec\uc640 \ucd1d\ud1b5, \uc870\uc120 \ucd08\uae30\uc758 \uc2e0\uae30\uc804 \ub4f1 \uae30\uc220\uc801 \ud601\uc2e0\uc744 \uc774\ub8e9\ud574 \uc654\ub2e4\uace0 \uc8fc\uc7a5\ud588\ub2e4.", "paragraph_sentence": "\uadf8\ub294 \uc0c1\uacf5\uc5c5 \ubc0f \ubb34\uc5ed\uc758 \uc9c4\ud765, \uadfc\ub300\uc801\uc778 \ud654\ud3d0 \ubc0f \uc870\uc138\uc81c\ub3c4\uc758 \uc218\ub9bd, \uadfc\ub300\uc801\uc778 \uad50\uc721\uc81c\ub3c4\uc758 \uc2e4\uc2dc\ub97c \ub2f9\uba74 \uacfc\uc81c\ub85c \uc81c\uc2dc\ud558\uc600\ub2e4. \uc720\uae38\uc900 \uc740 \uacc4\ubabd \uac15\uc5f0\uc5d0\uc11c\ub3c4 \uc2e0\ub77c\uc758 \uc0bc\uad6d\ud1b5\uc77c, \uc2e0\ub77c\uc758 \uc774\ub450, \uace0\ub824\uc2dc\ub300\uc758 \uae08\uc18d\ud65c\uc790, \uc870\uc120\uc758 \uac70\ubd81\uc120, \uace0\ub824\uccad\uc790, \uc870\uc120\ubc31\uc790, \ucd5c\ubb34\uc120\uc758 \ud654\ud3ec\uc640 \ucd1d\ud1b5, \uc870\uc120 \ucd08\uae30\uc758 \uc2e0\uae30\uc804 \ub4f1 \uae30\uc220\uc801 \ud601\uc2e0\uc744 \uc774\ub8e9\ud574 \uc654\ub2e4\uace0 \uc8fc\uc7a5\ud588\ub2e4. \uadf8\ub294 \uc790\uc2e0\uc758 \uc800\uc11c \u3008\uc11c\uc720\uacac\ubb38 \u897f\u904a\u898b\u805e\u3009\uc5d0\uc11c\ub3c4 \uc870\uc120\uc758 \uacfc\ud559\uae30\uc220\uc774 \ub0b3\uc740 \uc704\ub300\ud55c \uc131\uacfc\ub85c \uc774\ub450, \uace0\ub824\uc790\uae30, \uac70\ubd81\uc120, \uae08\uc18d\ud65c\uc790\ub97c \ub4e4\uace0 \uc788\ub2e4. \uadf8\ub7f0\ub370 \uace0\ub824 \ub9d0\ubd80\ud130 \uc870\uc120\uc655\uc870 5\ubc31\ub144\uac04 \ubb38\uc778\ub4e4\ub9cc\uc744 \uc6b0\ub300\ud558\uace0, \uae30\uc220\uc790\uc640 \uad70\uc778\ub4e4\uc740 \ucc9c\ub300\ud558\uba74\uc11c \uc774\ub7ec\ud55c \ud6cc\ub96d\ud55c \uae30\uc220\uacfc \uc131\uacfc\ub4e4\uc774 \uc0ac\uc7a5\ub418\uc5c8\uc74c\uc744 \uc5ed\uc124\ud588\ub2e4. \uadf8\ub294 \ub9cc\uc57d \ud6c4\uc190\ub4e4\uc774 \ube44\uc2e4\uc6a9\uc801\uc778 \ud559\ubb38\uc5d0 \uc9d1\ucc29\ud558\uc9c0 \uc54a\uace0, \uc774\ub7f0 \uacfc\ud559 \uae30\uc220 \uc804\ud1b5\uc744 \uc5f0\uad6c, \ubc1c\uc804\uc2dc\ucf30\ub354\ub77c\uba74 \uc9c0\uae08 \uc138\uacc4\uc758 \uc601\uad11\uc774 \uc870\uc120\uc5d0 \ub3cc\ub824\uc84c\uc744 \uac83\uc774\uc9c0\ub9cc, \ud6c4\uc190\ub4e4\uc774 \uadf8\ub807\uac8c \ud558\uc9c0 \ubabb\ud588\uc74c\uc744 \uc548\ud0c0\uae4c\uc6cc\ud558\uc600\ub2e4. \uadf8\ub294 \uacfc\ud559, \uae30\uc220\uc758 \ucc9c\ub300\uac00 \uc870\uc120\uc744 \uba78\ub9dd\uc758 \ub098\ub77d\uc73c\ub85c \uc774\ub04c\uc5c8\ub2e4\uace0 \ud655\uc2e0\ud588\ub2e4.", "paragraph_answer": "\uadf8\ub294 \uc0c1\uacf5\uc5c5 \ubc0f \ubb34\uc5ed\uc758 \uc9c4\ud765, \uadfc\ub300\uc801\uc778 \ud654\ud3d0 \ubc0f \uc870\uc138\uc81c\ub3c4\uc758 \uc218\ub9bd, \uadfc\ub300\uc801\uc778 \uad50\uc721\uc81c\ub3c4\uc758 \uc2e4\uc2dc\ub97c \ub2f9\uba74 \uacfc\uc81c\ub85c \uc81c\uc2dc\ud558\uc600\ub2e4. \uc720\uae38\uc900 \uc740 \uacc4\ubabd \uac15\uc5f0\uc5d0\uc11c\ub3c4 \uc2e0\ub77c\uc758 \uc0bc\uad6d\ud1b5\uc77c, \uc2e0\ub77c\uc758 \uc774\ub450, \uace0\ub824\uc2dc\ub300\uc758 \uae08\uc18d\ud65c\uc790, \uc870\uc120\uc758 \uac70\ubd81\uc120, \uace0\ub824\uccad\uc790, \uc870\uc120\ubc31\uc790, \ucd5c\ubb34\uc120\uc758 \ud654\ud3ec\uc640 \ucd1d\ud1b5, \uc870\uc120 \ucd08\uae30\uc758 \uc2e0\uae30\uc804 \ub4f1 \uae30\uc220\uc801 \ud601\uc2e0\uc744 \uc774\ub8e9\ud574 \uc654\ub2e4\uace0 \uc8fc\uc7a5\ud588\ub2e4. \uadf8\ub294 \uc790\uc2e0\uc758 \uc800\uc11c \u3008\uc11c\uc720\uacac\ubb38 \u897f\u904a\u898b\u805e\u3009\uc5d0\uc11c\ub3c4 \uc870\uc120\uc758 \uacfc\ud559\uae30\uc220\uc774 \ub0b3\uc740 \uc704\ub300\ud55c \uc131\uacfc\ub85c \uc774\ub450, \uace0\ub824\uc790\uae30, \uac70\ubd81\uc120, \uae08\uc18d\ud65c\uc790\ub97c \ub4e4\uace0 \uc788\ub2e4. \uadf8\ub7f0\ub370 \uace0\ub824 \ub9d0\ubd80\ud130 \uc870\uc120\uc655\uc870 5\ubc31\ub144\uac04 \ubb38\uc778\ub4e4\ub9cc\uc744 \uc6b0\ub300\ud558\uace0, \uae30\uc220\uc790\uc640 \uad70\uc778\ub4e4\uc740 \ucc9c\ub300\ud558\uba74\uc11c \uc774\ub7ec\ud55c \ud6cc\ub96d\ud55c \uae30\uc220\uacfc \uc131\uacfc\ub4e4\uc774 \uc0ac\uc7a5\ub418\uc5c8\uc74c\uc744 \uc5ed\uc124\ud588\ub2e4. \uadf8\ub294 \ub9cc\uc57d \ud6c4\uc190\ub4e4\uc774 \ube44\uc2e4\uc6a9\uc801\uc778 \ud559\ubb38\uc5d0 \uc9d1\ucc29\ud558\uc9c0 \uc54a\uace0, \uc774\ub7f0 \uacfc\ud559 \uae30\uc220 \uc804\ud1b5\uc744 \uc5f0\uad6c, \ubc1c\uc804\uc2dc\ucf30\ub354\ub77c\uba74 \uc9c0\uae08 \uc138\uacc4\uc758 \uc601\uad11\uc774 \uc870\uc120\uc5d0 \ub3cc\ub824\uc84c\uc744 \uac83\uc774\uc9c0\ub9cc, \ud6c4\uc190\ub4e4\uc774 \uadf8\ub807\uac8c \ud558\uc9c0 \ubabb\ud588\uc74c\uc744 \uc548\ud0c0\uae4c\uc6cc\ud558\uc600\ub2e4. \uadf8\ub294 \uacfc\ud559, \uae30\uc220\uc758 \ucc9c\ub300\uac00 \uc870\uc120\uc744 \uba78\ub9dd\uc758 \ub098\ub77d\uc73c\ub85c \uc774\ub04c\uc5c8\ub2e4\uace0 \ud655\uc2e0\ud588\ub2e4.", "sentence_answer": " \uc720\uae38\uc900 \uc740 \uacc4\ubabd \uac15\uc5f0\uc5d0\uc11c\ub3c4 \uc2e0\ub77c\uc758 \uc0bc\uad6d\ud1b5\uc77c, \uc2e0\ub77c\uc758 \uc774\ub450, \uace0\ub824\uc2dc\ub300\uc758 \uae08\uc18d\ud65c\uc790, \uc870\uc120\uc758 \uac70\ubd81\uc120, \uace0\ub824\uccad\uc790, \uc870\uc120\ubc31\uc790, \ucd5c\ubb34\uc120\uc758 \ud654\ud3ec\uc640 \ucd1d\ud1b5, \uc870\uc120 \ucd08\uae30\uc758 \uc2e0\uae30\uc804 \ub4f1 \uae30\uc220\uc801 \ud601\uc2e0\uc744 \uc774\ub8e9\ud574 \uc654\ub2e4\uace0 \uc8fc\uc7a5\ud588\ub2e4.", "paragraph_id": "6542749-28-1", "paragraph_question": "question: \uacfc\ud559 \uae30\uc220\uc758 \ubc1c\uc804\uc744 \ub3c5\ub824\ud558\uba70 \uc11c\uc720\uacac\ubb38\uc744 \uc4f4 \uc800\uc790\ub294?, context: \uadf8\ub294 \uc0c1\uacf5\uc5c5 \ubc0f \ubb34\uc5ed\uc758 \uc9c4\ud765, \uadfc\ub300\uc801\uc778 \ud654\ud3d0 \ubc0f \uc870\uc138\uc81c\ub3c4\uc758 \uc218\ub9bd, \uadfc\ub300\uc801\uc778 \uad50\uc721\uc81c\ub3c4\uc758 \uc2e4\uc2dc\ub97c \ub2f9\uba74 \uacfc\uc81c\ub85c \uc81c\uc2dc\ud558\uc600\ub2e4. \uc720\uae38\uc900\uc740 \uacc4\ubabd \uac15\uc5f0\uc5d0\uc11c\ub3c4 \uc2e0\ub77c\uc758 \uc0bc\uad6d\ud1b5\uc77c, \uc2e0\ub77c\uc758 \uc774\ub450, \uace0\ub824\uc2dc\ub300\uc758 \uae08\uc18d\ud65c\uc790, \uc870\uc120\uc758 \uac70\ubd81\uc120, \uace0\ub824\uccad\uc790, \uc870\uc120\ubc31\uc790, \ucd5c\ubb34\uc120\uc758 \ud654\ud3ec\uc640 \ucd1d\ud1b5, \uc870\uc120 \ucd08\uae30\uc758 \uc2e0\uae30\uc804 \ub4f1 \uae30\uc220\uc801 \ud601\uc2e0\uc744 \uc774\ub8e9\ud574 \uc654\ub2e4\uace0 \uc8fc\uc7a5\ud588\ub2e4. \uadf8\ub294 \uc790\uc2e0\uc758 \uc800\uc11c \u3008\uc11c\uc720\uacac\ubb38 \u897f\u904a\u898b\u805e\u3009\uc5d0\uc11c\ub3c4 \uc870\uc120\uc758 \uacfc\ud559\uae30\uc220\uc774 \ub0b3\uc740 \uc704\ub300\ud55c \uc131\uacfc\ub85c \uc774\ub450, \uace0\ub824\uc790\uae30, \uac70\ubd81\uc120, \uae08\uc18d\ud65c\uc790\ub97c \ub4e4\uace0 \uc788\ub2e4. \uadf8\ub7f0\ub370 \uace0\ub824 \ub9d0\ubd80\ud130 \uc870\uc120\uc655\uc870 5\ubc31\ub144\uac04 \ubb38\uc778\ub4e4\ub9cc\uc744 \uc6b0\ub300\ud558\uace0, \uae30\uc220\uc790\uc640 \uad70\uc778\ub4e4\uc740 \ucc9c\ub300\ud558\uba74\uc11c \uc774\ub7ec\ud55c \ud6cc\ub96d\ud55c \uae30\uc220\uacfc \uc131\uacfc\ub4e4\uc774 \uc0ac\uc7a5\ub418\uc5c8\uc74c\uc744 \uc5ed\uc124\ud588\ub2e4. \uadf8\ub294 \ub9cc\uc57d \ud6c4\uc190\ub4e4\uc774 \ube44\uc2e4\uc6a9\uc801\uc778 \ud559\ubb38\uc5d0 \uc9d1\ucc29\ud558\uc9c0 \uc54a\uace0, \uc774\ub7f0 \uacfc\ud559 \uae30\uc220 \uc804\ud1b5\uc744 \uc5f0\uad6c, \ubc1c\uc804\uc2dc\ucf30\ub354\ub77c\uba74 \uc9c0\uae08 \uc138\uacc4\uc758 \uc601\uad11\uc774 \uc870\uc120\uc5d0 \ub3cc\ub824\uc84c\uc744 \uac83\uc774\uc9c0\ub9cc, \ud6c4\uc190\ub4e4\uc774 \uadf8\ub807\uac8c \ud558\uc9c0 \ubabb\ud588\uc74c\uc744 \uc548\ud0c0\uae4c\uc6cc\ud558\uc600\ub2e4. \uadf8\ub294 \uacfc\ud559, \uae30\uc220\uc758 \ucc9c\ub300\uac00 \uc870\uc120\uc744 \uba78\ub9dd\uc758 \ub098\ub77d\uc73c\ub85c \uc774\ub04c\uc5c8\ub2e4\uace0 \ud655\uc2e0\ud588\ub2e4."} {"question": "\uc548\uc0b0\uc2dc\uc758 \uc2dc\ud2f0\ud22c\uc5b4 \ubc84\uc2a4\ub294 \ucd1d \uba87\uac1c\uc758 \uacbd\ub85c\uac00 \uc788\ub294\uac00?", "paragraph": "\uc548\uc0b0\uc2dc\ub294 \uac01\uc9c0\uc5d0 \uc0b0\uc7ac\ud55c \ubb38\ud654\uc720\uc0b0\ub4e4\uc744 \ud6a8\uc728\uc801\uc73c\ub85c \ud65c\uc6a9\ud558\uae30 \uc704\ud558\uc5ec 2008\ub144 6\uc6d4 7\uc77c\ubd80\ud130 \uc2dc\ud2f0\ud22c\uc5b4 \ubc84\uc2a4\ub97c \ub9e4\ub144 \uc77c\uc815 \uae30\uac04 \ub3d9\uc548 \uc6b4\ud589\ud558\uace0 \uc788\ub2e4. \uc774 \uc2dc\ud2f0\ud22c\uc5b4 \ubc84\uc2a4\uc5d0\ub294 \uc911\uc559\uc5ed \uad11\uc7a5\uc744 \uc2dc\uc885\uc810\uc73c\ub85c \ud558\ub294 4\uac1c\uc758 \uacbd\ub85c\uac00 \uc788\ub294\ub370, \uc131\ud638\uae30\ub150\uad00\uacfc \uc2dc\ud654\ubc29\uc870\uc81c\ub97c \uc8fc\uc694 \uacbd\uc720\uc9c0\ub85c \ud558\ub294 \uc81c1\ucf54\uc2a4, \uac08\ub300\uc2b5\uc9c0\uacf5\uc6d0\uacfc \uc120\uac10\ub3c4\ub97c \uc8fc\uc694 \uacbd\uc720\uc9c0\ub85c \ud558\ub294 \uc81c2\ucf54\uc2a4, \uc131\ud638\uae30\ub150\uad00\uacfc \uac08\ub300\uc2b5\uc9c0\uacf5\uc6d0, \ucd5c\uc6a9\uc2e0\uae30\ub150\uad00\uc744 \uc8fc\uc694 \uacbd\uc720\uc9c0\ub85c \ud558\ub294 \uc81c3\ucf54\uc2a4, \uc131\ud638\uae30\ub150\uad00, \uac08\ub300\uc2b5\uc9c0\uacf5\uc6d0, \uc120\uac10\ub3c4\ub97c \uacbd\uc720\ud55c \ub4a4 \ud589\ubcf5\uc608\uc808\uad00 \uccb4\ud5d8 \ud65c\ub3d9\uc744 \ud3ec\ud568\ud558\ub294 \uc81c4\ucf54\uc2a4\uac00 \uc788\ub2e4. \uc774\ubc16\uc5d0 \uc2dc\ud2f0\ud22c\uc5b4 \uad00\uad11\uac1d\ub4e4\uc774 \uc8fc\ub85c \ucc3e\ub294 \uad00\uad11\uc9c0\ub4e4\uc5d0\uc11c \uccb4\ud5d8 \ud589\uc0ac\ub97c \uc5f4\uc218 \uc788\ub3c4\ub85d \uc9c0\uc6d0\ud558\uace0 \uc788\uc73c\uba70, \ub300\ubd80\ub3c4\uc758 \uac2f\ubc8c\u00b7\uc5fc\uc804 \uccb4\ud5d8 \ud589\uc0ac \ub4f1\uc774 \ub300\ud45c\uc801\uc774\ub2e4. \ud558\uc9c0\ub9cc \uc774\uc640 \uac19\uc740 \ud589\uc0ac\ub4e4\uc758 \ub300\ubd80\ubd84\uc740 \uc218\ud655 \uc2dc\uae30\uac00 \uc815\ud574\uc9c4 \uacbd\uc6b0\uac00 \ub9ce\uc544 \ud2b9\uc815 \uae30\uac04\uc5d0\ub9cc \uc6b4\uc601\ub41c\ub2e4\ub294 \ud55c\uacc4\uac00 \uc788\ub2e4.", "answer": "4\uac1c", "sentence": "\uc774 \uc2dc\ud2f0\ud22c\uc5b4 \ubc84\uc2a4\uc5d0\ub294 \uc911\uc559\uc5ed \uad11\uc7a5\uc744 \uc2dc\uc885\uc810\uc73c\ub85c \ud558\ub294 4\uac1c \uc758 \uacbd\ub85c\uac00 \uc788\ub294\ub370, \uc131\ud638\uae30\ub150\uad00\uacfc \uc2dc\ud654\ubc29\uc870\uc81c\ub97c \uc8fc\uc694 \uacbd\uc720\uc9c0\ub85c \ud558\ub294 \uc81c1\ucf54\uc2a4, \uac08\ub300\uc2b5\uc9c0\uacf5\uc6d0\uacfc \uc120\uac10\ub3c4\ub97c \uc8fc\uc694 \uacbd\uc720\uc9c0\ub85c \ud558\ub294 \uc81c2\ucf54\uc2a4, \uc131\ud638\uae30\ub150\uad00\uacfc \uac08\ub300\uc2b5\uc9c0\uacf5\uc6d0, \ucd5c\uc6a9\uc2e0\uae30\ub150\uad00\uc744 \uc8fc\uc694 \uacbd\uc720\uc9c0\ub85c \ud558\ub294 \uc81c3\ucf54\uc2a4, \uc131\ud638\uae30\ub150\uad00, \uac08\ub300\uc2b5\uc9c0\uacf5\uc6d0, \uc120\uac10\ub3c4\ub97c \uacbd\uc720\ud55c \ub4a4 \ud589\ubcf5\uc608\uc808\uad00 \uccb4\ud5d8 \ud65c\ub3d9\uc744 \ud3ec\ud568\ud558\ub294 \uc81c4\ucf54\uc2a4\uac00 \uc788\ub2e4.", "paragraph_sentence": "\uc548\uc0b0\uc2dc\ub294 \uac01\uc9c0\uc5d0 \uc0b0\uc7ac\ud55c \ubb38\ud654\uc720\uc0b0\ub4e4\uc744 \ud6a8\uc728\uc801\uc73c\ub85c \ud65c\uc6a9\ud558\uae30 \uc704\ud558\uc5ec 2008\ub144 6\uc6d4 7\uc77c\ubd80\ud130 \uc2dc\ud2f0\ud22c\uc5b4 \ubc84\uc2a4\ub97c \ub9e4\ub144 \uc77c\uc815 \uae30\uac04 \ub3d9\uc548 \uc6b4\ud589\ud558\uace0 \uc788\ub2e4. \uc774 \uc2dc\ud2f0\ud22c\uc5b4 \ubc84\uc2a4\uc5d0\ub294 \uc911\uc559\uc5ed \uad11\uc7a5\uc744 \uc2dc\uc885\uc810\uc73c\ub85c \ud558\ub294 4\uac1c \uc758 \uacbd\ub85c\uac00 \uc788\ub294\ub370, \uc131\ud638\uae30\ub150\uad00\uacfc \uc2dc\ud654\ubc29\uc870\uc81c\ub97c \uc8fc\uc694 \uacbd\uc720\uc9c0\ub85c \ud558\ub294 \uc81c1\ucf54\uc2a4, \uac08\ub300\uc2b5\uc9c0\uacf5\uc6d0\uacfc \uc120\uac10\ub3c4\ub97c \uc8fc\uc694 \uacbd\uc720\uc9c0\ub85c \ud558\ub294 \uc81c2\ucf54\uc2a4, \uc131\ud638\uae30\ub150\uad00\uacfc \uac08\ub300\uc2b5\uc9c0\uacf5\uc6d0, \ucd5c\uc6a9\uc2e0\uae30\ub150\uad00\uc744 \uc8fc\uc694 \uacbd\uc720\uc9c0\ub85c \ud558\ub294 \uc81c3\ucf54\uc2a4, \uc131\ud638\uae30\ub150\uad00, \uac08\ub300\uc2b5\uc9c0\uacf5\uc6d0, \uc120\uac10\ub3c4\ub97c \uacbd\uc720\ud55c \ub4a4 \ud589\ubcf5\uc608\uc808\uad00 \uccb4\ud5d8 \ud65c\ub3d9\uc744 \ud3ec\ud568\ud558\ub294 \uc81c4\ucf54\uc2a4\uac00 \uc788\ub2e4. \uc774\ubc16\uc5d0 \uc2dc\ud2f0\ud22c\uc5b4 \uad00\uad11\uac1d\ub4e4\uc774 \uc8fc\ub85c \ucc3e\ub294 \uad00\uad11\uc9c0\ub4e4\uc5d0\uc11c \uccb4\ud5d8 \ud589\uc0ac\ub97c \uc5f4\uc218 \uc788\ub3c4\ub85d \uc9c0\uc6d0\ud558\uace0 \uc788\uc73c\uba70, \ub300\ubd80\ub3c4\uc758 \uac2f\ubc8c\u00b7\uc5fc\uc804 \uccb4\ud5d8 \ud589\uc0ac \ub4f1\uc774 \ub300\ud45c\uc801\uc774\ub2e4. \ud558\uc9c0\ub9cc \uc774\uc640 \uac19\uc740 \ud589\uc0ac\ub4e4\uc758 \ub300\ubd80\ubd84\uc740 \uc218\ud655 \uc2dc\uae30\uac00 \uc815\ud574\uc9c4 \uacbd\uc6b0\uac00 \ub9ce\uc544 \ud2b9\uc815 \uae30\uac04\uc5d0\ub9cc \uc6b4\uc601\ub41c\ub2e4\ub294 \ud55c\uacc4\uac00 \uc788\ub2e4.", "paragraph_answer": "\uc548\uc0b0\uc2dc\ub294 \uac01\uc9c0\uc5d0 \uc0b0\uc7ac\ud55c \ubb38\ud654\uc720\uc0b0\ub4e4\uc744 \ud6a8\uc728\uc801\uc73c\ub85c \ud65c\uc6a9\ud558\uae30 \uc704\ud558\uc5ec 2008\ub144 6\uc6d4 7\uc77c\ubd80\ud130 \uc2dc\ud2f0\ud22c\uc5b4 \ubc84\uc2a4\ub97c \ub9e4\ub144 \uc77c\uc815 \uae30\uac04 \ub3d9\uc548 \uc6b4\ud589\ud558\uace0 \uc788\ub2e4. \uc774 \uc2dc\ud2f0\ud22c\uc5b4 \ubc84\uc2a4\uc5d0\ub294 \uc911\uc559\uc5ed \uad11\uc7a5\uc744 \uc2dc\uc885\uc810\uc73c\ub85c \ud558\ub294 4\uac1c \uc758 \uacbd\ub85c\uac00 \uc788\ub294\ub370, \uc131\ud638\uae30\ub150\uad00\uacfc \uc2dc\ud654\ubc29\uc870\uc81c\ub97c \uc8fc\uc694 \uacbd\uc720\uc9c0\ub85c \ud558\ub294 \uc81c1\ucf54\uc2a4, \uac08\ub300\uc2b5\uc9c0\uacf5\uc6d0\uacfc \uc120\uac10\ub3c4\ub97c \uc8fc\uc694 \uacbd\uc720\uc9c0\ub85c \ud558\ub294 \uc81c2\ucf54\uc2a4, \uc131\ud638\uae30\ub150\uad00\uacfc \uac08\ub300\uc2b5\uc9c0\uacf5\uc6d0, \ucd5c\uc6a9\uc2e0\uae30\ub150\uad00\uc744 \uc8fc\uc694 \uacbd\uc720\uc9c0\ub85c \ud558\ub294 \uc81c3\ucf54\uc2a4, \uc131\ud638\uae30\ub150\uad00, \uac08\ub300\uc2b5\uc9c0\uacf5\uc6d0, \uc120\uac10\ub3c4\ub97c \uacbd\uc720\ud55c \ub4a4 \ud589\ubcf5\uc608\uc808\uad00 \uccb4\ud5d8 \ud65c\ub3d9\uc744 \ud3ec\ud568\ud558\ub294 \uc81c4\ucf54\uc2a4\uac00 \uc788\ub2e4. \uc774\ubc16\uc5d0 \uc2dc\ud2f0\ud22c\uc5b4 \uad00\uad11\uac1d\ub4e4\uc774 \uc8fc\ub85c \ucc3e\ub294 \uad00\uad11\uc9c0\ub4e4\uc5d0\uc11c \uccb4\ud5d8 \ud589\uc0ac\ub97c \uc5f4\uc218 \uc788\ub3c4\ub85d \uc9c0\uc6d0\ud558\uace0 \uc788\uc73c\uba70, \ub300\ubd80\ub3c4\uc758 \uac2f\ubc8c\u00b7\uc5fc\uc804 \uccb4\ud5d8 \ud589\uc0ac \ub4f1\uc774 \ub300\ud45c\uc801\uc774\ub2e4. \ud558\uc9c0\ub9cc \uc774\uc640 \uac19\uc740 \ud589\uc0ac\ub4e4\uc758 \ub300\ubd80\ubd84\uc740 \uc218\ud655 \uc2dc\uae30\uac00 \uc815\ud574\uc9c4 \uacbd\uc6b0\uac00 \ub9ce\uc544 \ud2b9\uc815 \uae30\uac04\uc5d0\ub9cc \uc6b4\uc601\ub41c\ub2e4\ub294 \ud55c\uacc4\uac00 \uc788\ub2e4.", "sentence_answer": "\uc774 \uc2dc\ud2f0\ud22c\uc5b4 \ubc84\uc2a4\uc5d0\ub294 \uc911\uc559\uc5ed \uad11\uc7a5\uc744 \uc2dc\uc885\uc810\uc73c\ub85c \ud558\ub294 4\uac1c \uc758 \uacbd\ub85c\uac00 \uc788\ub294\ub370, \uc131\ud638\uae30\ub150\uad00\uacfc \uc2dc\ud654\ubc29\uc870\uc81c\ub97c \uc8fc\uc694 \uacbd\uc720\uc9c0\ub85c \ud558\ub294 \uc81c1\ucf54\uc2a4, \uac08\ub300\uc2b5\uc9c0\uacf5\uc6d0\uacfc \uc120\uac10\ub3c4\ub97c \uc8fc\uc694 \uacbd\uc720\uc9c0\ub85c \ud558\ub294 \uc81c2\ucf54\uc2a4, \uc131\ud638\uae30\ub150\uad00\uacfc \uac08\ub300\uc2b5\uc9c0\uacf5\uc6d0, \ucd5c\uc6a9\uc2e0\uae30\ub150\uad00\uc744 \uc8fc\uc694 \uacbd\uc720\uc9c0\ub85c \ud558\ub294 \uc81c3\ucf54\uc2a4, \uc131\ud638\uae30\ub150\uad00, \uac08\ub300\uc2b5\uc9c0\uacf5\uc6d0, \uc120\uac10\ub3c4\ub97c \uacbd\uc720\ud55c \ub4a4 \ud589\ubcf5\uc608\uc808\uad00 \uccb4\ud5d8 \ud65c\ub3d9\uc744 \ud3ec\ud568\ud558\ub294 \uc81c4\ucf54\uc2a4\uac00 \uc788\ub2e4.", "paragraph_id": "6538930-17-1", "paragraph_question": "question: \uc548\uc0b0\uc2dc\uc758 \uc2dc\ud2f0\ud22c\uc5b4 \ubc84\uc2a4\ub294 \ucd1d \uba87\uac1c\uc758 \uacbd\ub85c\uac00 \uc788\ub294\uac00?, context: \uc548\uc0b0\uc2dc\ub294 \uac01\uc9c0\uc5d0 \uc0b0\uc7ac\ud55c \ubb38\ud654\uc720\uc0b0\ub4e4\uc744 \ud6a8\uc728\uc801\uc73c\ub85c \ud65c\uc6a9\ud558\uae30 \uc704\ud558\uc5ec 2008\ub144 6\uc6d4 7\uc77c\ubd80\ud130 \uc2dc\ud2f0\ud22c\uc5b4 \ubc84\uc2a4\ub97c \ub9e4\ub144 \uc77c\uc815 \uae30\uac04 \ub3d9\uc548 \uc6b4\ud589\ud558\uace0 \uc788\ub2e4. \uc774 \uc2dc\ud2f0\ud22c\uc5b4 \ubc84\uc2a4\uc5d0\ub294 \uc911\uc559\uc5ed \uad11\uc7a5\uc744 \uc2dc\uc885\uc810\uc73c\ub85c \ud558\ub294 4\uac1c\uc758 \uacbd\ub85c\uac00 \uc788\ub294\ub370, \uc131\ud638\uae30\ub150\uad00\uacfc \uc2dc\ud654\ubc29\uc870\uc81c\ub97c \uc8fc\uc694 \uacbd\uc720\uc9c0\ub85c \ud558\ub294 \uc81c1\ucf54\uc2a4, \uac08\ub300\uc2b5\uc9c0\uacf5\uc6d0\uacfc \uc120\uac10\ub3c4\ub97c \uc8fc\uc694 \uacbd\uc720\uc9c0\ub85c \ud558\ub294 \uc81c2\ucf54\uc2a4, \uc131\ud638\uae30\ub150\uad00\uacfc \uac08\ub300\uc2b5\uc9c0\uacf5\uc6d0, \ucd5c\uc6a9\uc2e0\uae30\ub150\uad00\uc744 \uc8fc\uc694 \uacbd\uc720\uc9c0\ub85c \ud558\ub294 \uc81c3\ucf54\uc2a4, \uc131\ud638\uae30\ub150\uad00, \uac08\ub300\uc2b5\uc9c0\uacf5\uc6d0, \uc120\uac10\ub3c4\ub97c \uacbd\uc720\ud55c \ub4a4 \ud589\ubcf5\uc608\uc808\uad00 \uccb4\ud5d8 \ud65c\ub3d9\uc744 \ud3ec\ud568\ud558\ub294 \uc81c4\ucf54\uc2a4\uac00 \uc788\ub2e4. \uc774\ubc16\uc5d0 \uc2dc\ud2f0\ud22c\uc5b4 \uad00\uad11\uac1d\ub4e4\uc774 \uc8fc\ub85c \ucc3e\ub294 \uad00\uad11\uc9c0\ub4e4\uc5d0\uc11c \uccb4\ud5d8 \ud589\uc0ac\ub97c \uc5f4\uc218 \uc788\ub3c4\ub85d \uc9c0\uc6d0\ud558\uace0 \uc788\uc73c\uba70, \ub300\ubd80\ub3c4\uc758 \uac2f\ubc8c\u00b7\uc5fc\uc804 \uccb4\ud5d8 \ud589\uc0ac \ub4f1\uc774 \ub300\ud45c\uc801\uc774\ub2e4. \ud558\uc9c0\ub9cc \uc774\uc640 \uac19\uc740 \ud589\uc0ac\ub4e4\uc758 \ub300\ubd80\ubd84\uc740 \uc218\ud655 \uc2dc\uae30\uac00 \uc815\ud574\uc9c4 \uacbd\uc6b0\uac00 \ub9ce\uc544 \ud2b9\uc815 \uae30\uac04\uc5d0\ub9cc \uc6b4\uc601\ub41c\ub2e4\ub294 \ud55c\uacc4\uac00 \uc788\ub2e4."} {"question": "\uc6a9\uae30\uc788\ub294 \ubbfc\uc8fc\ud654\uc640 \uc778\uad8c \ud22c\uc0ac\uc778 \uae40 \uc804 \ub300\ud1b5\ub839\uc758 \uc11c\uac70\ub85c \uc2ac\ud514\uc5d0 \ube60\uc84c\ub2e4\ub294 \uc560\ub3c4 \uc131\uba85\uc744 \ubc1c\ud45c\ud55c \uc0ac\ub78c\uc740?", "paragraph": "\ubc84\ub77d \uc624\ubc14\ub9c8 \ubbf8\uad6d \ub300\ud1b5\ub839\uc740 \uc560\ub3c4 \uc131\uba85\uc744 \ud1b5\ud574 \"\uc6a9\uae30\uc788\ub294 \ubbfc\uc8fc\ud654\uc640 \uc778\uad8c \ud22c\uc0ac\uc778 \uae40 \uc804 \ub300\ud1b5\ub839\uc758 \uc11c\uac70\ub85c \uc2ac\ud514\uc5d0 \ube60\uc84c\ub2e4.\"\uba74\uc11c \uadf8\ub294 \"\uae40 \uc804 \ub300\ud1b5\ub839\uc740 \ud55c\uad6d\uc758 \uc5ed\ub3d9\uc801\uc778 \ubbfc\uc8fc\ud654\uc5d0 \uc911\uc694\ud55c \uc5ed\ud560\uc744 \ud55c \uc815\uce58\uc6b4\ub3d9\uc744 \uc77c\uc73c\ud0a4\uace0 \uc774\ub04c\uc5b4 \uc624\ub294\ub370 \ubaa9\uc228\uc744 \ubc14\ucce4\ub2e4.\"\uba70 \"\uadf8\uc758 \uc870\uad6d\uc5d0 \ub300\ud55c \ud5cc\uc2e0\uacfc \ud55c\ubc18\ub3c4 \ud3c9\ud654\ub97c \uc704\ud55c \uc9c0\uce60 \uc904 \ubaa8\ub974\ub294 \ub178\ub825, \uc790\uc720\ub97c \uc704\ud55c \uac1c\uc778\uc801\uc778 \ud76c\uc0dd\uc740 \uace0\ubb34\uc801\uc774\uba70 \uacb0\ucf54 \uc78a\ud600\uc9c0\uc9c0 \uc54a\uc744 \uac83.\"\uc774\ub77c\uace0 \ub9d0\ud588\ub2e4. \ubbf8\uad6d \uad6d\ubb34\ubd80\ub3c4 \uae40\ub300\uc911\uc758 \uc11c\uac70\uc5d0 \uc560\ub3c4\uc758 \ub73b\uc744 \ud45c\uba85\ud588\ub2e4. \uc774\uc5b8 \ucf08\ub9ac \ub300\ubcc0\uc778\uc740 \uc815\ub840\ube0c\ub9ac\ud551\uc5d0\uc11c \"\uae40 \uc804 \ub300\ud1b5\ub839\uc758 \uc11c\uac70\ub97c \ud55c\uad6d \uad6d\ubbfc\ub4e4\uacfc \ud568\uaed8 \uc560\ub3c4\ud55c\ub2e4 \"\uba74\uc11c \"\uc720\uac00\uc871\uacfc \ud55c\uad6d \uad6d\ubbfc\uc5d0\uac8c \uc870\uc758\ub97c \uc804\ud55c\ub2e4.\"\ub77c\uace0 \ubc1d\ud614\ub2e4. \ucf08\ub9ac \ub300\ubcc0\uc778\uc740 \uc774\uc5b4 \"\uae40 \uc804 \ub300\ud1b5\ub839\uc740 \ud55c\uad6d \uad6d\ubbfc\uc758 \ubbfc\uc8fc\uc8fc\uc758 \uc5f4\ub9dd\uc5d0 \uc601\uac10\uc744 \uc77c\uc73c\ud0a8 \uc9c0\ub3c4\uc790\uc774\uc790 \uc0c1\uc9d5\uc774\uc5c8\ub2e4.\"\uba74\uc11c \"\ud55c\ubc18\ub3c4 \ud3c9\ud654\uc5d0 \ub300\ud55c \uae30\uc5ec\ub85c 2000\ub144\uc5d0 \ub178\ubca8\uc0c1\uc744 \uc218\uc0c1\ud558\uae30\ub3c4 \ud588\ub2e4.\"\ub77c\uace0 \ub9d0\ud588\ub2e4. \ube4c \ud074\ub9b0\ud134 \uc804 \ubbf8\uad6d \ub300\ud1b5\ub839\uc740 \ubd80\uc778 \ud790\ub7ec\ub9ac \ud074\ub9b0\ud134 \uad6d\ubb34\uc7a5\uad00\uacfc \ud568\uaed8 \uae40\ub300\uc911 \uc804 \ub300\ud1b5\ub839\uc758 \uc0ac\ub9dd\uc5d0 \uc560\ub3c4\ub97c \ud45c\ud588\ub2e4. \ud074\ub9b0\ud134 \ub300\ud1b5\ub839\uc740 \ube4c \ud074\ub9b0\ud134\uc7ac\ub2e8 \ud648\ud398\uc774\uc9c0\uc5d0 \uc62c\ub9b0 \uc131\uba85\uc5d0\uc11c \"\uae40\ub300\uc911 \uc804 \ub300\ud1b5\ub839\uc740 \uc6a9\uac10\ud558\uace0 \uac15\ub825\ud55c \uc774\uc0c1\uc744 \uac00\uc9c4 \uc9c0\ub3c4\uc790\uc600\ub2e4 \"\uba70 \"\ud55c\uad6d\uc774 \uc2ec\uac01\ud55c \uacbd\uc81c\uc704\uae30\ub97c \ub118\uc5b4\uc11c\uac8c \ud558\uace0 \ud55c\ubc18\ub3c4 \ud3c9\ud654\ub97c \uc704\ud55c \uae38\uc744 \ub2e6\uace0 \uad6d\uc81c\uc801\uc73c\ub85c \uc778\uad8c\uc744 \ubcf4\ud638\ud558\ub294\ub370 \uc55e\uc7a5\uc130\ub2e4.\"\uace0 \uae40 \uc804 \ub300\ud1b5\ub839\uc758 \uc5c5\uc801\uc744 \uae30\ub838\ub2e4. \uadf8\ub294 \"\uae40 \uc804 \ub300\ud1b5\ub839\uc774 \ub300\ud1b5\ub839\uc774 \ub41c \ud6c4 \ub098\ub294 \ub0a8\ubd81\ud55c\uc758 \ud654\ud574\ub97c \uc704\ud574 \uadf8\uc640 \ud568\uaed8 \uc77c\ud558\ub294 \uc601\uad11\uc744 \ub204\ub838\ub2e4.\"\uba70 \"\ud587\ubcd5\uc815\ucc45\uc774 \ud55c\uad6d \uc804\uc7c1 \uc774\ud6c4 \uc5b4\ub290 \ub54c\ubcf4\ub2e4\ub3c4 \uc601\uc18d\uc801\uc778 \ud3c9\ud654\uc5d0 \ub300\ud55c \ud76c\ub9dd\uc744 \uc81c\uacf5\ud588\ub2e4.\"\uace0 \ud3c9\uac00\ud588\ub2e4. \uc774\uc5b4 \uadf8\ub294 \"\ubd80\uc778\uacfc \ub098\ub294 \uc6b0\ub9ac\uc758 \uc88b\uc740 \uce5c\uad6c\uc600\ub358 \uae40 \uc804 \ub300\ud1b5\ub839\uc744 \uadf8\ub9ac\uc6cc\ud560 \uac83\uc774\ub2e4.\"\uc774\ub77c\uace0 \uc560\ub3c4\uc758 \ub73b\uc744 \uc804\ud588\ub2e4.", "answer": "\ubc84\ub77d \uc624\ubc14\ub9c8", "sentence": "\ubc84\ub77d \uc624\ubc14\ub9c8 \ubbf8\uad6d \ub300\ud1b5\ub839\uc740 \uc560\ub3c4 \uc131\uba85\uc744 \ud1b5\ud574 \"\uc6a9\uae30\uc788\ub294 \ubbfc\uc8fc\ud654\uc640 \uc778\uad8c \ud22c\uc0ac\uc778 \uae40 \uc804 \ub300\ud1b5\ub839\uc758 \uc11c\uac70\ub85c \uc2ac\ud514\uc5d0 \ube60\uc84c\ub2e4.", "paragraph_sentence": " \ubc84\ub77d \uc624\ubc14\ub9c8 \ubbf8\uad6d \ub300\ud1b5\ub839\uc740 \uc560\ub3c4 \uc131\uba85\uc744 \ud1b5\ud574 \"\uc6a9\uae30\uc788\ub294 \ubbfc\uc8fc\ud654\uc640 \uc778\uad8c \ud22c\uc0ac\uc778 \uae40 \uc804 \ub300\ud1b5\ub839\uc758 \uc11c\uac70\ub85c \uc2ac\ud514\uc5d0 \ube60\uc84c\ub2e4. \"\uba74\uc11c \uadf8\ub294 \"\uae40 \uc804 \ub300\ud1b5\ub839\uc740 \ud55c\uad6d\uc758 \uc5ed\ub3d9\uc801\uc778 \ubbfc\uc8fc\ud654\uc5d0 \uc911\uc694\ud55c \uc5ed\ud560\uc744 \ud55c \uc815\uce58\uc6b4\ub3d9\uc744 \uc77c\uc73c\ud0a4\uace0 \uc774\ub04c\uc5b4 \uc624\ub294\ub370 \ubaa9\uc228\uc744 \ubc14\ucce4\ub2e4. \"\uba70 \"\uadf8\uc758 \uc870\uad6d\uc5d0 \ub300\ud55c \ud5cc\uc2e0\uacfc \ud55c\ubc18\ub3c4 \ud3c9\ud654\ub97c \uc704\ud55c \uc9c0\uce60 \uc904 \ubaa8\ub974\ub294 \ub178\ub825, \uc790\uc720\ub97c \uc704\ud55c \uac1c\uc778\uc801\uc778 \ud76c\uc0dd\uc740 \uace0\ubb34\uc801\uc774\uba70 \uacb0\ucf54 \uc78a\ud600\uc9c0\uc9c0 \uc54a\uc744 \uac83. \" \uc774\ub77c\uace0 \ub9d0\ud588\ub2e4. \ubbf8\uad6d \uad6d\ubb34\ubd80\ub3c4 \uae40\ub300\uc911\uc758 \uc11c\uac70\uc5d0 \uc560\ub3c4\uc758 \ub73b\uc744 \ud45c\uba85\ud588\ub2e4. \uc774\uc5b8 \ucf08\ub9ac \ub300\ubcc0\uc778\uc740 \uc815\ub840\ube0c\ub9ac\ud551\uc5d0\uc11c \"\uae40 \uc804 \ub300\ud1b5\ub839\uc758 \uc11c\uac70\ub97c \ud55c\uad6d \uad6d\ubbfc\ub4e4\uacfc \ud568\uaed8 \uc560\ub3c4\ud55c\ub2e4 \"\uba74\uc11c \"\uc720\uac00\uc871\uacfc \ud55c\uad6d \uad6d\ubbfc\uc5d0\uac8c \uc870\uc758\ub97c \uc804\ud55c\ub2e4. \"\ub77c\uace0 \ubc1d\ud614\ub2e4. \ucf08\ub9ac \ub300\ubcc0\uc778\uc740 \uc774\uc5b4 \"\uae40 \uc804 \ub300\ud1b5\ub839\uc740 \ud55c\uad6d \uad6d\ubbfc\uc758 \ubbfc\uc8fc\uc8fc\uc758 \uc5f4\ub9dd\uc5d0 \uc601\uac10\uc744 \uc77c\uc73c\ud0a8 \uc9c0\ub3c4\uc790\uc774\uc790 \uc0c1\uc9d5\uc774\uc5c8\ub2e4. \"\uba74\uc11c \"\ud55c\ubc18\ub3c4 \ud3c9\ud654\uc5d0 \ub300\ud55c \uae30\uc5ec\ub85c 2000\ub144\uc5d0 \ub178\ubca8\uc0c1\uc744 \uc218\uc0c1\ud558\uae30\ub3c4 \ud588\ub2e4. \"\ub77c\uace0 \ub9d0\ud588\ub2e4. \ube4c \ud074\ub9b0\ud134 \uc804 \ubbf8\uad6d \ub300\ud1b5\ub839\uc740 \ubd80\uc778 \ud790\ub7ec\ub9ac \ud074\ub9b0\ud134 \uad6d\ubb34\uc7a5\uad00\uacfc \ud568\uaed8 \uae40\ub300\uc911 \uc804 \ub300\ud1b5\ub839\uc758 \uc0ac\ub9dd\uc5d0 \uc560\ub3c4\ub97c \ud45c\ud588\ub2e4. \ud074\ub9b0\ud134 \ub300\ud1b5\ub839\uc740 \ube4c \ud074\ub9b0\ud134\uc7ac\ub2e8 \ud648\ud398\uc774\uc9c0\uc5d0 \uc62c\ub9b0 \uc131\uba85\uc5d0\uc11c \"\uae40\ub300\uc911 \uc804 \ub300\ud1b5\ub839\uc740 \uc6a9\uac10\ud558\uace0 \uac15\ub825\ud55c \uc774\uc0c1\uc744 \uac00\uc9c4 \uc9c0\ub3c4\uc790\uc600\ub2e4 \"\uba70 \"\ud55c\uad6d\uc774 \uc2ec\uac01\ud55c \uacbd\uc81c\uc704\uae30\ub97c \ub118\uc5b4\uc11c\uac8c \ud558\uace0 \ud55c\ubc18\ub3c4 \ud3c9\ud654\ub97c \uc704\ud55c \uae38\uc744 \ub2e6\uace0 \uad6d\uc81c\uc801\uc73c\ub85c \uc778\uad8c\uc744 \ubcf4\ud638\ud558\ub294\ub370 \uc55e\uc7a5\uc130\ub2e4. \"\uace0 \uae40 \uc804 \ub300\ud1b5\ub839\uc758 \uc5c5\uc801\uc744 \uae30\ub838\ub2e4. \uadf8\ub294 \"\uae40 \uc804 \ub300\ud1b5\ub839\uc774 \ub300\ud1b5\ub839\uc774 \ub41c \ud6c4 \ub098\ub294 \ub0a8\ubd81\ud55c\uc758 \ud654\ud574\ub97c \uc704\ud574 \uadf8\uc640 \ud568\uaed8 \uc77c\ud558\ub294 \uc601\uad11\uc744 \ub204\ub838\ub2e4. \"\uba70 \"\ud587\ubcd5\uc815\ucc45\uc774 \ud55c\uad6d \uc804\uc7c1 \uc774\ud6c4 \uc5b4\ub290 \ub54c\ubcf4\ub2e4\ub3c4 \uc601\uc18d\uc801\uc778 \ud3c9\ud654\uc5d0 \ub300\ud55c \ud76c\ub9dd\uc744 \uc81c\uacf5\ud588\ub2e4. \"\uace0 \ud3c9\uac00\ud588\ub2e4. \uc774\uc5b4 \uadf8\ub294 \"\ubd80\uc778\uacfc \ub098\ub294 \uc6b0\ub9ac\uc758 \uc88b\uc740 \uce5c\uad6c\uc600\ub358 \uae40 \uc804 \ub300\ud1b5\ub839\uc744 \uadf8\ub9ac\uc6cc\ud560 \uac83\uc774\ub2e4. \"\uc774\ub77c\uace0 \uc560\ub3c4\uc758 \ub73b\uc744 \uc804\ud588\ub2e4.", "paragraph_answer": " \ubc84\ub77d \uc624\ubc14\ub9c8 \ubbf8\uad6d \ub300\ud1b5\ub839\uc740 \uc560\ub3c4 \uc131\uba85\uc744 \ud1b5\ud574 \"\uc6a9\uae30\uc788\ub294 \ubbfc\uc8fc\ud654\uc640 \uc778\uad8c \ud22c\uc0ac\uc778 \uae40 \uc804 \ub300\ud1b5\ub839\uc758 \uc11c\uac70\ub85c \uc2ac\ud514\uc5d0 \ube60\uc84c\ub2e4.\"\uba74\uc11c \uadf8\ub294 \"\uae40 \uc804 \ub300\ud1b5\ub839\uc740 \ud55c\uad6d\uc758 \uc5ed\ub3d9\uc801\uc778 \ubbfc\uc8fc\ud654\uc5d0 \uc911\uc694\ud55c \uc5ed\ud560\uc744 \ud55c \uc815\uce58\uc6b4\ub3d9\uc744 \uc77c\uc73c\ud0a4\uace0 \uc774\ub04c\uc5b4 \uc624\ub294\ub370 \ubaa9\uc228\uc744 \ubc14\ucce4\ub2e4.\"\uba70 \"\uadf8\uc758 \uc870\uad6d\uc5d0 \ub300\ud55c \ud5cc\uc2e0\uacfc \ud55c\ubc18\ub3c4 \ud3c9\ud654\ub97c \uc704\ud55c \uc9c0\uce60 \uc904 \ubaa8\ub974\ub294 \ub178\ub825, \uc790\uc720\ub97c \uc704\ud55c \uac1c\uc778\uc801\uc778 \ud76c\uc0dd\uc740 \uace0\ubb34\uc801\uc774\uba70 \uacb0\ucf54 \uc78a\ud600\uc9c0\uc9c0 \uc54a\uc744 \uac83.\"\uc774\ub77c\uace0 \ub9d0\ud588\ub2e4. \ubbf8\uad6d \uad6d\ubb34\ubd80\ub3c4 \uae40\ub300\uc911\uc758 \uc11c\uac70\uc5d0 \uc560\ub3c4\uc758 \ub73b\uc744 \ud45c\uba85\ud588\ub2e4. \uc774\uc5b8 \ucf08\ub9ac \ub300\ubcc0\uc778\uc740 \uc815\ub840\ube0c\ub9ac\ud551\uc5d0\uc11c \"\uae40 \uc804 \ub300\ud1b5\ub839\uc758 \uc11c\uac70\ub97c \ud55c\uad6d \uad6d\ubbfc\ub4e4\uacfc \ud568\uaed8 \uc560\ub3c4\ud55c\ub2e4 \"\uba74\uc11c \"\uc720\uac00\uc871\uacfc \ud55c\uad6d \uad6d\ubbfc\uc5d0\uac8c \uc870\uc758\ub97c \uc804\ud55c\ub2e4.\"\ub77c\uace0 \ubc1d\ud614\ub2e4. \ucf08\ub9ac \ub300\ubcc0\uc778\uc740 \uc774\uc5b4 \"\uae40 \uc804 \ub300\ud1b5\ub839\uc740 \ud55c\uad6d \uad6d\ubbfc\uc758 \ubbfc\uc8fc\uc8fc\uc758 \uc5f4\ub9dd\uc5d0 \uc601\uac10\uc744 \uc77c\uc73c\ud0a8 \uc9c0\ub3c4\uc790\uc774\uc790 \uc0c1\uc9d5\uc774\uc5c8\ub2e4.\"\uba74\uc11c \"\ud55c\ubc18\ub3c4 \ud3c9\ud654\uc5d0 \ub300\ud55c \uae30\uc5ec\ub85c 2000\ub144\uc5d0 \ub178\ubca8\uc0c1\uc744 \uc218\uc0c1\ud558\uae30\ub3c4 \ud588\ub2e4.\"\ub77c\uace0 \ub9d0\ud588\ub2e4. \ube4c \ud074\ub9b0\ud134 \uc804 \ubbf8\uad6d \ub300\ud1b5\ub839\uc740 \ubd80\uc778 \ud790\ub7ec\ub9ac \ud074\ub9b0\ud134 \uad6d\ubb34\uc7a5\uad00\uacfc \ud568\uaed8 \uae40\ub300\uc911 \uc804 \ub300\ud1b5\ub839\uc758 \uc0ac\ub9dd\uc5d0 \uc560\ub3c4\ub97c \ud45c\ud588\ub2e4. \ud074\ub9b0\ud134 \ub300\ud1b5\ub839\uc740 \ube4c \ud074\ub9b0\ud134\uc7ac\ub2e8 \ud648\ud398\uc774\uc9c0\uc5d0 \uc62c\ub9b0 \uc131\uba85\uc5d0\uc11c \"\uae40\ub300\uc911 \uc804 \ub300\ud1b5\ub839\uc740 \uc6a9\uac10\ud558\uace0 \uac15\ub825\ud55c \uc774\uc0c1\uc744 \uac00\uc9c4 \uc9c0\ub3c4\uc790\uc600\ub2e4 \"\uba70 \"\ud55c\uad6d\uc774 \uc2ec\uac01\ud55c \uacbd\uc81c\uc704\uae30\ub97c \ub118\uc5b4\uc11c\uac8c \ud558\uace0 \ud55c\ubc18\ub3c4 \ud3c9\ud654\ub97c \uc704\ud55c \uae38\uc744 \ub2e6\uace0 \uad6d\uc81c\uc801\uc73c\ub85c \uc778\uad8c\uc744 \ubcf4\ud638\ud558\ub294\ub370 \uc55e\uc7a5\uc130\ub2e4.\"\uace0 \uae40 \uc804 \ub300\ud1b5\ub839\uc758 \uc5c5\uc801\uc744 \uae30\ub838\ub2e4. \uadf8\ub294 \"\uae40 \uc804 \ub300\ud1b5\ub839\uc774 \ub300\ud1b5\ub839\uc774 \ub41c \ud6c4 \ub098\ub294 \ub0a8\ubd81\ud55c\uc758 \ud654\ud574\ub97c \uc704\ud574 \uadf8\uc640 \ud568\uaed8 \uc77c\ud558\ub294 \uc601\uad11\uc744 \ub204\ub838\ub2e4.\"\uba70 \"\ud587\ubcd5\uc815\ucc45\uc774 \ud55c\uad6d \uc804\uc7c1 \uc774\ud6c4 \uc5b4\ub290 \ub54c\ubcf4\ub2e4\ub3c4 \uc601\uc18d\uc801\uc778 \ud3c9\ud654\uc5d0 \ub300\ud55c \ud76c\ub9dd\uc744 \uc81c\uacf5\ud588\ub2e4.\"\uace0 \ud3c9\uac00\ud588\ub2e4. \uc774\uc5b4 \uadf8\ub294 \"\ubd80\uc778\uacfc \ub098\ub294 \uc6b0\ub9ac\uc758 \uc88b\uc740 \uce5c\uad6c\uc600\ub358 \uae40 \uc804 \ub300\ud1b5\ub839\uc744 \uadf8\ub9ac\uc6cc\ud560 \uac83\uc774\ub2e4.\"\uc774\ub77c\uace0 \uc560\ub3c4\uc758 \ub73b\uc744 \uc804\ud588\ub2e4.", "sentence_answer": " \ubc84\ub77d \uc624\ubc14\ub9c8 \ubbf8\uad6d \ub300\ud1b5\ub839\uc740 \uc560\ub3c4 \uc131\uba85\uc744 \ud1b5\ud574 \"\uc6a9\uae30\uc788\ub294 \ubbfc\uc8fc\ud654\uc640 \uc778\uad8c \ud22c\uc0ac\uc778 \uae40 \uc804 \ub300\ud1b5\ub839\uc758 \uc11c\uac70\ub85c \uc2ac\ud514\uc5d0 \ube60\uc84c\ub2e4.", "paragraph_id": "6576830-43-0", "paragraph_question": "question: \uc6a9\uae30\uc788\ub294 \ubbfc\uc8fc\ud654\uc640 \uc778\uad8c \ud22c\uc0ac\uc778 \uae40 \uc804 \ub300\ud1b5\ub839\uc758 \uc11c\uac70\ub85c \uc2ac\ud514\uc5d0 \ube60\uc84c\ub2e4\ub294 \uc560\ub3c4 \uc131\uba85\uc744 \ubc1c\ud45c\ud55c \uc0ac\ub78c\uc740?, context: \ubc84\ub77d \uc624\ubc14\ub9c8 \ubbf8\uad6d \ub300\ud1b5\ub839\uc740 \uc560\ub3c4 \uc131\uba85\uc744 \ud1b5\ud574 \"\uc6a9\uae30\uc788\ub294 \ubbfc\uc8fc\ud654\uc640 \uc778\uad8c \ud22c\uc0ac\uc778 \uae40 \uc804 \ub300\ud1b5\ub839\uc758 \uc11c\uac70\ub85c \uc2ac\ud514\uc5d0 \ube60\uc84c\ub2e4.\"\uba74\uc11c \uadf8\ub294 \"\uae40 \uc804 \ub300\ud1b5\ub839\uc740 \ud55c\uad6d\uc758 \uc5ed\ub3d9\uc801\uc778 \ubbfc\uc8fc\ud654\uc5d0 \uc911\uc694\ud55c \uc5ed\ud560\uc744 \ud55c \uc815\uce58\uc6b4\ub3d9\uc744 \uc77c\uc73c\ud0a4\uace0 \uc774\ub04c\uc5b4 \uc624\ub294\ub370 \ubaa9\uc228\uc744 \ubc14\ucce4\ub2e4.\"\uba70 \"\uadf8\uc758 \uc870\uad6d\uc5d0 \ub300\ud55c \ud5cc\uc2e0\uacfc \ud55c\ubc18\ub3c4 \ud3c9\ud654\ub97c \uc704\ud55c \uc9c0\uce60 \uc904 \ubaa8\ub974\ub294 \ub178\ub825, \uc790\uc720\ub97c \uc704\ud55c \uac1c\uc778\uc801\uc778 \ud76c\uc0dd\uc740 \uace0\ubb34\uc801\uc774\uba70 \uacb0\ucf54 \uc78a\ud600\uc9c0\uc9c0 \uc54a\uc744 \uac83.\"\uc774\ub77c\uace0 \ub9d0\ud588\ub2e4. \ubbf8\uad6d \uad6d\ubb34\ubd80\ub3c4 \uae40\ub300\uc911\uc758 \uc11c\uac70\uc5d0 \uc560\ub3c4\uc758 \ub73b\uc744 \ud45c\uba85\ud588\ub2e4. \uc774\uc5b8 \ucf08\ub9ac \ub300\ubcc0\uc778\uc740 \uc815\ub840\ube0c\ub9ac\ud551\uc5d0\uc11c \"\uae40 \uc804 \ub300\ud1b5\ub839\uc758 \uc11c\uac70\ub97c \ud55c\uad6d \uad6d\ubbfc\ub4e4\uacfc \ud568\uaed8 \uc560\ub3c4\ud55c\ub2e4 \"\uba74\uc11c \"\uc720\uac00\uc871\uacfc \ud55c\uad6d \uad6d\ubbfc\uc5d0\uac8c \uc870\uc758\ub97c \uc804\ud55c\ub2e4.\"\ub77c\uace0 \ubc1d\ud614\ub2e4. \ucf08\ub9ac \ub300\ubcc0\uc778\uc740 \uc774\uc5b4 \"\uae40 \uc804 \ub300\ud1b5\ub839\uc740 \ud55c\uad6d \uad6d\ubbfc\uc758 \ubbfc\uc8fc\uc8fc\uc758 \uc5f4\ub9dd\uc5d0 \uc601\uac10\uc744 \uc77c\uc73c\ud0a8 \uc9c0\ub3c4\uc790\uc774\uc790 \uc0c1\uc9d5\uc774\uc5c8\ub2e4.\"\uba74\uc11c \"\ud55c\ubc18\ub3c4 \ud3c9\ud654\uc5d0 \ub300\ud55c \uae30\uc5ec\ub85c 2000\ub144\uc5d0 \ub178\ubca8\uc0c1\uc744 \uc218\uc0c1\ud558\uae30\ub3c4 \ud588\ub2e4.\"\ub77c\uace0 \ub9d0\ud588\ub2e4. \ube4c \ud074\ub9b0\ud134 \uc804 \ubbf8\uad6d \ub300\ud1b5\ub839\uc740 \ubd80\uc778 \ud790\ub7ec\ub9ac \ud074\ub9b0\ud134 \uad6d\ubb34\uc7a5\uad00\uacfc \ud568\uaed8 \uae40\ub300\uc911 \uc804 \ub300\ud1b5\ub839\uc758 \uc0ac\ub9dd\uc5d0 \uc560\ub3c4\ub97c \ud45c\ud588\ub2e4. \ud074\ub9b0\ud134 \ub300\ud1b5\ub839\uc740 \ube4c \ud074\ub9b0\ud134\uc7ac\ub2e8 \ud648\ud398\uc774\uc9c0\uc5d0 \uc62c\ub9b0 \uc131\uba85\uc5d0\uc11c \"\uae40\ub300\uc911 \uc804 \ub300\ud1b5\ub839\uc740 \uc6a9\uac10\ud558\uace0 \uac15\ub825\ud55c \uc774\uc0c1\uc744 \uac00\uc9c4 \uc9c0\ub3c4\uc790\uc600\ub2e4 \"\uba70 \"\ud55c\uad6d\uc774 \uc2ec\uac01\ud55c \uacbd\uc81c\uc704\uae30\ub97c \ub118\uc5b4\uc11c\uac8c \ud558\uace0 \ud55c\ubc18\ub3c4 \ud3c9\ud654\ub97c \uc704\ud55c \uae38\uc744 \ub2e6\uace0 \uad6d\uc81c\uc801\uc73c\ub85c \uc778\uad8c\uc744 \ubcf4\ud638\ud558\ub294\ub370 \uc55e\uc7a5\uc130\ub2e4.\"\uace0 \uae40 \uc804 \ub300\ud1b5\ub839\uc758 \uc5c5\uc801\uc744 \uae30\ub838\ub2e4. \uadf8\ub294 \"\uae40 \uc804 \ub300\ud1b5\ub839\uc774 \ub300\ud1b5\ub839\uc774 \ub41c \ud6c4 \ub098\ub294 \ub0a8\ubd81\ud55c\uc758 \ud654\ud574\ub97c \uc704\ud574 \uadf8\uc640 \ud568\uaed8 \uc77c\ud558\ub294 \uc601\uad11\uc744 \ub204\ub838\ub2e4.\"\uba70 \"\ud587\ubcd5\uc815\ucc45\uc774 \ud55c\uad6d \uc804\uc7c1 \uc774\ud6c4 \uc5b4\ub290 \ub54c\ubcf4\ub2e4\ub3c4 \uc601\uc18d\uc801\uc778 \ud3c9\ud654\uc5d0 \ub300\ud55c \ud76c\ub9dd\uc744 \uc81c\uacf5\ud588\ub2e4.\"\uace0 \ud3c9\uac00\ud588\ub2e4. \uc774\uc5b4 \uadf8\ub294 \"\ubd80\uc778\uacfc \ub098\ub294 \uc6b0\ub9ac\uc758 \uc88b\uc740 \uce5c\uad6c\uc600\ub358 \uae40 \uc804 \ub300\ud1b5\ub839\uc744 \uadf8\ub9ac\uc6cc\ud560 \uac83\uc774\ub2e4.\"\uc774\ub77c\uace0 \uc560\ub3c4\uc758 \ub73b\uc744 \uc804\ud588\ub2e4."} {"question": "<\uc548\ub155, \ud5e4\uc774\uc990>\uc5d0\uc11c \ud5e4\uc774\uc990\uc758 \uc544\ubc84\uc9c0\uc778 \ub9c8\uc774\ud074 \ub7ad\uac70\uc2a4\ud130 \uc5ed\uc5d0 \uce90\uc2a4\ud305\ub41c \ubc30\uc6b0\ub294?", "paragraph": "2013\ub144 3\uc6d4 19\uc77c, \uc5d4\ud130\ud14c\uc778\uba3c\ud2b8 \uc704\ud074\ub9ac\ub294 \uc170\uc77c\ub9b0 \uc6b0\ub4e4\ub9ac\uac00 \uc8fc\uc778\uacf5 \ud5e4\uc774\uc990 \uadf8\ub808\uc774\uc2a4 \ub7ad\ucee4\uc2a4\ud130 \uc5ed\uc744 \ub9e1\uac8c \ub418\uc5c8\ub2e4\uace0 \ubcf4\ub3c4\ud558\uc600\ub2e4. \uac10\ub3c5 \uc870\uc2dc \ubd84\uc740 \"\uc6b0\ub9ac\ub294 \ud5e4\uc774\uc990 \uc5ed\uc744 \uce90\uc2a4\ud305\ud558\uae30 \uc704\ud574 150\uc5ec\uba85\uc758 \uc5ec\ubc30\uc6b0\uc640 \uc624\ub514\uc158\uc744 \ubcf4\uc558\uc5b4\uc694. \ub0b4\uac00 \ubcf8 \uac83\uc740 \uadf8 \uc911 50\uba85 \uc815\ub3c4\uc600\uc5b4\uc694. \uc6b0\ub4e4\ub9ac\uc758 \uc624\ub514\uc158\uc5d0 \uac78\ub9b0 \uc2dc\uac04\uc740 10\ucd08 \ub610\ub294 15\ucd08\uc600\ub2e4\uace0 \uc0dd\uac01\ud558\uc9c0\ub9cc, \ub098\ub294 \uadf8\ub140\uc57c\ub9d0\ub85c \ud5e4\uc774\uc990\uacfc \uc801\uc5ed\uc774\uc600\ub2e4\uace0 \uc0dd\uac01\ud574\uc694. \uadf8\ub140\uac00 \ub300\ubcf8\uc744 \ub4e4\uace0\uc788\uc744 \ub54c, \uadf8\ub140\uc758 \ub208\uc740 \ud758\ub054\ud758\ub054 \uccd0\ub2e4\ubcf4\ub354\ub77c\uad6c\uc694.\"\ub77c\uace0 \ub9d0\ud558\uc600\ub2e4. 2013\ub144 5\uc6d4 10\uc77c, \uc564\uc124 \uc5d8\uace0\ud2b8\uac00 \uc5b4\uac70\uc2a4\ud130\uc2a4 \uc6cc\ud130\uc2a4\ub97c \uc5f0\uae30\ud558\ub294 \uac83\uc774 \uacb0\uc815\ub418\uc5c8\ub2e4. \ubcf4\uadf8\uac78 2014\ub144 8\uc6d4\ud638\uc5d0 \ub530\ub974\uba74, \uc5d8\uace0\ud2b8\ub294 \uce90\uc2a4\ud305 \uacfc\uc815\uc5d0 \ub300\ud574 \"(\uc911\ub7b5) \uc9d1 \uacc4\ub2e8\uc5d0\uc11c \ud734\ub300\ud3f0\uc73c\ub85c \uc624\ub514\uc158 \uc601\uc0c1\uc744 \ucc0d\uc5b4 \ubcf4\ub0c8\ub2e4. \uc601\ud654\uc0ac \ucabd\uc5d0\uc11c \uc880 \ub2e4\ub974\uac8c \ucc0d\uc5b4\ubcf4\ub77c\uba70 \uc601\uc0c1\uc744 \ub3cc\ub824\ubcf4\ub0c8\uae30\uc5d0, \ub2e4\uc2dc \ud55c \ubc88 \ubcf4\ub0c8\ub2e4. \uadf8\ub9ac\uace0 \ub098\uc11c LA\ub85c \uc640\uc11c \uc170\uc77c\ub9b0 \uc6b0\ub4e4\ub9ac\uc640 \ud568\uaed8 \uc624\ub514\uc158\uc744 \ubcf4\ub77c\ub294 \uc5f0\ub77d\uc744 \ubc1b\uac8c \ub410\ub2e4. (\uc911\ub7b5) \ud798\ub4e0 \uacfc\uc815\uc774\uc600\uc9c0\ub9cc, \uacb0\uad6d \ud569\uaca9\ud588\ub2e4.\"\uace0 \ub9d0\ud588\ub2e4. \uac19\uc740 \ud574 7\uc6d4 23\uc77c, \ub85c\ub77c \ub358\uc774 \ud5e4\uc774\uc990\uc758 \uc5b4\uba38\ub2c8 \ud504\ub798\ub2c8 \ub7ad\ucee4\uc2a4\ud130\ub97c, \ub0c7 \uc6b8\ud504\uac00 \uc6cc\ud130\uc2a4\uc758 \ub9f9\uc778 \uce5c\uad6c \uc544\uc774\uc791\uc744 \uc5f0\uae30\ud558\ub294 \uac83\uc774 \ubcf4\ub3c4\ub418\uc5c8\ub2e4. \uc6b8\ud504\ub294 \ud5c8\ud504\ud3ec\uc2a4\ud2b8 \ub77c\uc774\ube0c\uc5d0\uc11c \"\ub098\ub294 \ud765\ubd84\ub418\uba70 \uc6b4\uc774 \uc88b\uc544\uc694. \ub0b4\uac00 \ub6f0\uc5b4\ub09c \uc7ac\ub2a5\uc744 \uac00\uc9c4 \uc0ac\ub78c\ub4e4\uacfc \ud611\ub825\ud560 \uc218 \uc788\uc5b4\uc11c\uc694. \uba4b\uc9c4 \uc0ac\ub78c\ub4e4\uacfc \uac19\uc774 \uc77c\uc744 \ud558\uae30 \ub54c\ubb38\uc5d0, \ub098\ub294 \uc601\ud654\uc5d0\uc11c \uc88b\uac8c \ubcf4\uc5ec\uc9c0\ub294 \uac70\uc5d0\uc694.\"\ub77c\uace0 \ub9d0\ud588\ub2e4. 8\uc6d4 14\uc77c, \uc0d8 \ud2b8\ub798\uba40\uc774 \ud5e4\uc774\uc990\uc758 \uc544\ubc84\uc9c0\uc778 \ub9c8\uc774\ud074 \ub7ad\ucee4\uc2a4\ud130 \uc5ed\uc5d0 \uc815\ud574\uc84c\ub2e4. 8\uc6d4 28\uc77c, \uc6d0\uc791\uc790 \uc778 \uc874 \uadf8\ub9b0\uc774 \uc790\uc2e0\uc758 \ud2b8\uc704\ud130\uc5d0\uc11c \ub9c8\uc774\ud06c \ubc84\ube44\uae00\ub9ac\uc544\uac00 \ud328\ud2b8\ub9ad\uc744 \uc5f0\uae30\ud560 \uac83\uc774\ub77c\uace0 \ubc1d\ud614\ub2e4. \ub610\ud55c 9\uc6d4 6\uc77c\uc5d0 \uadf8\ub9b0\uc740 \uc70c\ub7fc \ub354\ud3ec\uac00 \ubc34 \ud558\uc6b0\ud2bc\uc744 \uc5f0\uae30\ud55c\ub2e4\uace0 \ud2b8\uc717\ud558\uc600\ub2e4.", "answer": "\uc0d8 \ud2b8\ub798\uba40", "sentence": "8\uc6d4 14\uc77c, \uc0d8 \ud2b8\ub798\uba40 \uc774 \ud5e4\uc774\uc990\uc758 \uc544\ubc84\uc9c0\uc778 \ub9c8\uc774\ud074 \ub7ad\ucee4\uc2a4\ud130 \uc5ed\uc5d0 \uc815\ud574\uc84c\ub2e4.", "paragraph_sentence": "2013\ub144 3\uc6d4 19\uc77c, \uc5d4\ud130\ud14c\uc778\uba3c\ud2b8 \uc704\ud074\ub9ac\ub294 \uc170\uc77c\ub9b0 \uc6b0\ub4e4\ub9ac\uac00 \uc8fc\uc778\uacf5 \ud5e4\uc774\uc990 \uadf8\ub808\uc774\uc2a4 \ub7ad\ucee4\uc2a4\ud130 \uc5ed\uc744 \ub9e1\uac8c \ub418\uc5c8\ub2e4\uace0 \ubcf4\ub3c4\ud558\uc600\ub2e4. \uac10\ub3c5 \uc870\uc2dc \ubd84\uc740 \"\uc6b0\ub9ac\ub294 \ud5e4\uc774\uc990 \uc5ed\uc744 \uce90\uc2a4\ud305\ud558\uae30 \uc704\ud574 150\uc5ec\uba85\uc758 \uc5ec\ubc30\uc6b0\uc640 \uc624\ub514\uc158\uc744 \ubcf4\uc558\uc5b4\uc694. \ub0b4\uac00 \ubcf8 \uac83\uc740 \uadf8 \uc911 50\uba85 \uc815\ub3c4\uc600\uc5b4\uc694. \uc6b0\ub4e4\ub9ac\uc758 \uc624\ub514\uc158\uc5d0 \uac78\ub9b0 \uc2dc\uac04\uc740 10\ucd08 \ub610\ub294 15\ucd08\uc600\ub2e4\uace0 \uc0dd\uac01\ud558\uc9c0\ub9cc, \ub098\ub294 \uadf8\ub140\uc57c\ub9d0\ub85c \ud5e4\uc774\uc990\uacfc \uc801\uc5ed\uc774\uc600\ub2e4\uace0 \uc0dd\uac01\ud574\uc694. \uadf8\ub140\uac00 \ub300\ubcf8\uc744 \ub4e4\uace0\uc788\uc744 \ub54c, \uadf8\ub140\uc758 \ub208\uc740 \ud758\ub054\ud758\ub054 \uccd0\ub2e4\ubcf4\ub354\ub77c\uad6c\uc694. \"\ub77c\uace0 \ub9d0\ud558\uc600\ub2e4. 2013\ub144 5\uc6d4 10\uc77c, \uc564\uc124 \uc5d8\uace0\ud2b8\uac00 \uc5b4\uac70\uc2a4\ud130\uc2a4 \uc6cc\ud130\uc2a4\ub97c \uc5f0\uae30\ud558\ub294 \uac83\uc774 \uacb0\uc815\ub418\uc5c8\ub2e4. \ubcf4\uadf8\uac78 2014\ub144 8\uc6d4\ud638\uc5d0 \ub530\ub974\uba74, \uc5d8\uace0\ud2b8\ub294 \uce90\uc2a4\ud305 \uacfc\uc815\uc5d0 \ub300\ud574 \"(\uc911\ub7b5) \uc9d1 \uacc4\ub2e8\uc5d0\uc11c \ud734\ub300\ud3f0\uc73c\ub85c \uc624\ub514\uc158 \uc601\uc0c1\uc744 \ucc0d\uc5b4 \ubcf4\ub0c8\ub2e4. \uc601\ud654\uc0ac \ucabd\uc5d0\uc11c \uc880 \ub2e4\ub974\uac8c \ucc0d\uc5b4\ubcf4\ub77c\uba70 \uc601\uc0c1\uc744 \ub3cc\ub824\ubcf4\ub0c8\uae30\uc5d0, \ub2e4\uc2dc \ud55c \ubc88 \ubcf4\ub0c8\ub2e4. \uadf8\ub9ac\uace0 \ub098\uc11c LA\ub85c \uc640\uc11c \uc170\uc77c\ub9b0 \uc6b0\ub4e4\ub9ac\uc640 \ud568\uaed8 \uc624\ub514\uc158\uc744 \ubcf4\ub77c\ub294 \uc5f0\ub77d\uc744 \ubc1b\uac8c \ub410\ub2e4. (\uc911\ub7b5) \ud798\ub4e0 \uacfc\uc815\uc774\uc600\uc9c0\ub9cc, \uacb0\uad6d \ud569\uaca9\ud588\ub2e4. \"\uace0 \ub9d0\ud588\ub2e4. \uac19\uc740 \ud574 7\uc6d4 23\uc77c, \ub85c\ub77c \ub358\uc774 \ud5e4\uc774\uc990\uc758 \uc5b4\uba38\ub2c8 \ud504\ub798\ub2c8 \ub7ad\ucee4\uc2a4\ud130\ub97c, \ub0c7 \uc6b8\ud504\uac00 \uc6cc\ud130\uc2a4\uc758 \ub9f9\uc778 \uce5c\uad6c \uc544\uc774\uc791\uc744 \uc5f0\uae30\ud558\ub294 \uac83\uc774 \ubcf4\ub3c4\ub418\uc5c8\ub2e4. \uc6b8\ud504\ub294 \ud5c8\ud504\ud3ec\uc2a4\ud2b8 \ub77c\uc774\ube0c\uc5d0\uc11c \"\ub098\ub294 \ud765\ubd84\ub418\uba70 \uc6b4\uc774 \uc88b\uc544\uc694. \ub0b4\uac00 \ub6f0\uc5b4\ub09c \uc7ac\ub2a5\uc744 \uac00\uc9c4 \uc0ac\ub78c\ub4e4\uacfc \ud611\ub825\ud560 \uc218 \uc788\uc5b4\uc11c\uc694. \uba4b\uc9c4 \uc0ac\ub78c\ub4e4\uacfc \uac19\uc774 \uc77c\uc744 \ud558\uae30 \ub54c\ubb38\uc5d0, \ub098\ub294 \uc601\ud654\uc5d0\uc11c \uc88b\uac8c \ubcf4\uc5ec\uc9c0\ub294 \uac70\uc5d0\uc694. \"\ub77c\uace0 \ub9d0\ud588\ub2e4. 8\uc6d4 14\uc77c, \uc0d8 \ud2b8\ub798\uba40 \uc774 \ud5e4\uc774\uc990\uc758 \uc544\ubc84\uc9c0\uc778 \ub9c8\uc774\ud074 \ub7ad\ucee4\uc2a4\ud130 \uc5ed\uc5d0 \uc815\ud574\uc84c\ub2e4. 8\uc6d4 28\uc77c, \uc6d0\uc791\uc790 \uc778 \uc874 \uadf8\ub9b0\uc774 \uc790\uc2e0\uc758 \ud2b8\uc704\ud130\uc5d0\uc11c \ub9c8\uc774\ud06c \ubc84\ube44\uae00\ub9ac\uc544\uac00 \ud328\ud2b8\ub9ad\uc744 \uc5f0\uae30\ud560 \uac83\uc774\ub77c\uace0 \ubc1d\ud614\ub2e4. \ub610\ud55c 9\uc6d4 6\uc77c\uc5d0 \uadf8\ub9b0\uc740 \uc70c\ub7fc \ub354\ud3ec\uac00 \ubc34 \ud558\uc6b0\ud2bc\uc744 \uc5f0\uae30\ud55c\ub2e4\uace0 \ud2b8\uc717\ud558\uc600\ub2e4.", "paragraph_answer": "2013\ub144 3\uc6d4 19\uc77c, \uc5d4\ud130\ud14c\uc778\uba3c\ud2b8 \uc704\ud074\ub9ac\ub294 \uc170\uc77c\ub9b0 \uc6b0\ub4e4\ub9ac\uac00 \uc8fc\uc778\uacf5 \ud5e4\uc774\uc990 \uadf8\ub808\uc774\uc2a4 \ub7ad\ucee4\uc2a4\ud130 \uc5ed\uc744 \ub9e1\uac8c \ub418\uc5c8\ub2e4\uace0 \ubcf4\ub3c4\ud558\uc600\ub2e4. \uac10\ub3c5 \uc870\uc2dc \ubd84\uc740 \"\uc6b0\ub9ac\ub294 \ud5e4\uc774\uc990 \uc5ed\uc744 \uce90\uc2a4\ud305\ud558\uae30 \uc704\ud574 150\uc5ec\uba85\uc758 \uc5ec\ubc30\uc6b0\uc640 \uc624\ub514\uc158\uc744 \ubcf4\uc558\uc5b4\uc694. \ub0b4\uac00 \ubcf8 \uac83\uc740 \uadf8 \uc911 50\uba85 \uc815\ub3c4\uc600\uc5b4\uc694. \uc6b0\ub4e4\ub9ac\uc758 \uc624\ub514\uc158\uc5d0 \uac78\ub9b0 \uc2dc\uac04\uc740 10\ucd08 \ub610\ub294 15\ucd08\uc600\ub2e4\uace0 \uc0dd\uac01\ud558\uc9c0\ub9cc, \ub098\ub294 \uadf8\ub140\uc57c\ub9d0\ub85c \ud5e4\uc774\uc990\uacfc \uc801\uc5ed\uc774\uc600\ub2e4\uace0 \uc0dd\uac01\ud574\uc694. \uadf8\ub140\uac00 \ub300\ubcf8\uc744 \ub4e4\uace0\uc788\uc744 \ub54c, \uadf8\ub140\uc758 \ub208\uc740 \ud758\ub054\ud758\ub054 \uccd0\ub2e4\ubcf4\ub354\ub77c\uad6c\uc694.\"\ub77c\uace0 \ub9d0\ud558\uc600\ub2e4. 2013\ub144 5\uc6d4 10\uc77c, \uc564\uc124 \uc5d8\uace0\ud2b8\uac00 \uc5b4\uac70\uc2a4\ud130\uc2a4 \uc6cc\ud130\uc2a4\ub97c \uc5f0\uae30\ud558\ub294 \uac83\uc774 \uacb0\uc815\ub418\uc5c8\ub2e4. \ubcf4\uadf8\uac78 2014\ub144 8\uc6d4\ud638\uc5d0 \ub530\ub974\uba74, \uc5d8\uace0\ud2b8\ub294 \uce90\uc2a4\ud305 \uacfc\uc815\uc5d0 \ub300\ud574 \"(\uc911\ub7b5) \uc9d1 \uacc4\ub2e8\uc5d0\uc11c \ud734\ub300\ud3f0\uc73c\ub85c \uc624\ub514\uc158 \uc601\uc0c1\uc744 \ucc0d\uc5b4 \ubcf4\ub0c8\ub2e4. \uc601\ud654\uc0ac \ucabd\uc5d0\uc11c \uc880 \ub2e4\ub974\uac8c \ucc0d\uc5b4\ubcf4\ub77c\uba70 \uc601\uc0c1\uc744 \ub3cc\ub824\ubcf4\ub0c8\uae30\uc5d0, \ub2e4\uc2dc \ud55c \ubc88 \ubcf4\ub0c8\ub2e4. \uadf8\ub9ac\uace0 \ub098\uc11c LA\ub85c \uc640\uc11c \uc170\uc77c\ub9b0 \uc6b0\ub4e4\ub9ac\uc640 \ud568\uaed8 \uc624\ub514\uc158\uc744 \ubcf4\ub77c\ub294 \uc5f0\ub77d\uc744 \ubc1b\uac8c \ub410\ub2e4. (\uc911\ub7b5) \ud798\ub4e0 \uacfc\uc815\uc774\uc600\uc9c0\ub9cc, \uacb0\uad6d \ud569\uaca9\ud588\ub2e4.\"\uace0 \ub9d0\ud588\ub2e4. \uac19\uc740 \ud574 7\uc6d4 23\uc77c, \ub85c\ub77c \ub358\uc774 \ud5e4\uc774\uc990\uc758 \uc5b4\uba38\ub2c8 \ud504\ub798\ub2c8 \ub7ad\ucee4\uc2a4\ud130\ub97c, \ub0c7 \uc6b8\ud504\uac00 \uc6cc\ud130\uc2a4\uc758 \ub9f9\uc778 \uce5c\uad6c \uc544\uc774\uc791\uc744 \uc5f0\uae30\ud558\ub294 \uac83\uc774 \ubcf4\ub3c4\ub418\uc5c8\ub2e4. \uc6b8\ud504\ub294 \ud5c8\ud504\ud3ec\uc2a4\ud2b8 \ub77c\uc774\ube0c\uc5d0\uc11c \"\ub098\ub294 \ud765\ubd84\ub418\uba70 \uc6b4\uc774 \uc88b\uc544\uc694. \ub0b4\uac00 \ub6f0\uc5b4\ub09c \uc7ac\ub2a5\uc744 \uac00\uc9c4 \uc0ac\ub78c\ub4e4\uacfc \ud611\ub825\ud560 \uc218 \uc788\uc5b4\uc11c\uc694. \uba4b\uc9c4 \uc0ac\ub78c\ub4e4\uacfc \uac19\uc774 \uc77c\uc744 \ud558\uae30 \ub54c\ubb38\uc5d0, \ub098\ub294 \uc601\ud654\uc5d0\uc11c \uc88b\uac8c \ubcf4\uc5ec\uc9c0\ub294 \uac70\uc5d0\uc694.\"\ub77c\uace0 \ub9d0\ud588\ub2e4. 8\uc6d4 14\uc77c, \uc0d8 \ud2b8\ub798\uba40 \uc774 \ud5e4\uc774\uc990\uc758 \uc544\ubc84\uc9c0\uc778 \ub9c8\uc774\ud074 \ub7ad\ucee4\uc2a4\ud130 \uc5ed\uc5d0 \uc815\ud574\uc84c\ub2e4. 8\uc6d4 28\uc77c, \uc6d0\uc791\uc790 \uc778 \uc874 \uadf8\ub9b0\uc774 \uc790\uc2e0\uc758 \ud2b8\uc704\ud130\uc5d0\uc11c \ub9c8\uc774\ud06c \ubc84\ube44\uae00\ub9ac\uc544\uac00 \ud328\ud2b8\ub9ad\uc744 \uc5f0\uae30\ud560 \uac83\uc774\ub77c\uace0 \ubc1d\ud614\ub2e4. \ub610\ud55c 9\uc6d4 6\uc77c\uc5d0 \uadf8\ub9b0\uc740 \uc70c\ub7fc \ub354\ud3ec\uac00 \ubc34 \ud558\uc6b0\ud2bc\uc744 \uc5f0\uae30\ud55c\ub2e4\uace0 \ud2b8\uc717\ud558\uc600\ub2e4.", "sentence_answer": "8\uc6d4 14\uc77c, \uc0d8 \ud2b8\ub798\uba40 \uc774 \ud5e4\uc774\uc990\uc758 \uc544\ubc84\uc9c0\uc778 \ub9c8\uc774\ud074 \ub7ad\ucee4\uc2a4\ud130 \uc5ed\uc5d0 \uc815\ud574\uc84c\ub2e4.", "paragraph_id": "6521399-3-2", "paragraph_question": "question: <\uc548\ub155, \ud5e4\uc774\uc990>\uc5d0\uc11c \ud5e4\uc774\uc990\uc758 \uc544\ubc84\uc9c0\uc778 \ub9c8\uc774\ud074 \ub7ad\uac70\uc2a4\ud130 \uc5ed\uc5d0 \uce90\uc2a4\ud305\ub41c \ubc30\uc6b0\ub294?, context: 2013\ub144 3\uc6d4 19\uc77c, \uc5d4\ud130\ud14c\uc778\uba3c\ud2b8 \uc704\ud074\ub9ac\ub294 \uc170\uc77c\ub9b0 \uc6b0\ub4e4\ub9ac\uac00 \uc8fc\uc778\uacf5 \ud5e4\uc774\uc990 \uadf8\ub808\uc774\uc2a4 \ub7ad\ucee4\uc2a4\ud130 \uc5ed\uc744 \ub9e1\uac8c \ub418\uc5c8\ub2e4\uace0 \ubcf4\ub3c4\ud558\uc600\ub2e4. \uac10\ub3c5 \uc870\uc2dc \ubd84\uc740 \"\uc6b0\ub9ac\ub294 \ud5e4\uc774\uc990 \uc5ed\uc744 \uce90\uc2a4\ud305\ud558\uae30 \uc704\ud574 150\uc5ec\uba85\uc758 \uc5ec\ubc30\uc6b0\uc640 \uc624\ub514\uc158\uc744 \ubcf4\uc558\uc5b4\uc694. \ub0b4\uac00 \ubcf8 \uac83\uc740 \uadf8 \uc911 50\uba85 \uc815\ub3c4\uc600\uc5b4\uc694. \uc6b0\ub4e4\ub9ac\uc758 \uc624\ub514\uc158\uc5d0 \uac78\ub9b0 \uc2dc\uac04\uc740 10\ucd08 \ub610\ub294 15\ucd08\uc600\ub2e4\uace0 \uc0dd\uac01\ud558\uc9c0\ub9cc, \ub098\ub294 \uadf8\ub140\uc57c\ub9d0\ub85c \ud5e4\uc774\uc990\uacfc \uc801\uc5ed\uc774\uc600\ub2e4\uace0 \uc0dd\uac01\ud574\uc694. \uadf8\ub140\uac00 \ub300\ubcf8\uc744 \ub4e4\uace0\uc788\uc744 \ub54c, \uadf8\ub140\uc758 \ub208\uc740 \ud758\ub054\ud758\ub054 \uccd0\ub2e4\ubcf4\ub354\ub77c\uad6c\uc694.\"\ub77c\uace0 \ub9d0\ud558\uc600\ub2e4. 2013\ub144 5\uc6d4 10\uc77c, \uc564\uc124 \uc5d8\uace0\ud2b8\uac00 \uc5b4\uac70\uc2a4\ud130\uc2a4 \uc6cc\ud130\uc2a4\ub97c \uc5f0\uae30\ud558\ub294 \uac83\uc774 \uacb0\uc815\ub418\uc5c8\ub2e4. \ubcf4\uadf8\uac78 2014\ub144 8\uc6d4\ud638\uc5d0 \ub530\ub974\uba74, \uc5d8\uace0\ud2b8\ub294 \uce90\uc2a4\ud305 \uacfc\uc815\uc5d0 \ub300\ud574 \"(\uc911\ub7b5) \uc9d1 \uacc4\ub2e8\uc5d0\uc11c \ud734\ub300\ud3f0\uc73c\ub85c \uc624\ub514\uc158 \uc601\uc0c1\uc744 \ucc0d\uc5b4 \ubcf4\ub0c8\ub2e4. \uc601\ud654\uc0ac \ucabd\uc5d0\uc11c \uc880 \ub2e4\ub974\uac8c \ucc0d\uc5b4\ubcf4\ub77c\uba70 \uc601\uc0c1\uc744 \ub3cc\ub824\ubcf4\ub0c8\uae30\uc5d0, \ub2e4\uc2dc \ud55c \ubc88 \ubcf4\ub0c8\ub2e4. \uadf8\ub9ac\uace0 \ub098\uc11c LA\ub85c \uc640\uc11c \uc170\uc77c\ub9b0 \uc6b0\ub4e4\ub9ac\uc640 \ud568\uaed8 \uc624\ub514\uc158\uc744 \ubcf4\ub77c\ub294 \uc5f0\ub77d\uc744 \ubc1b\uac8c \ub410\ub2e4. (\uc911\ub7b5) \ud798\ub4e0 \uacfc\uc815\uc774\uc600\uc9c0\ub9cc, \uacb0\uad6d \ud569\uaca9\ud588\ub2e4.\"\uace0 \ub9d0\ud588\ub2e4. \uac19\uc740 \ud574 7\uc6d4 23\uc77c, \ub85c\ub77c \ub358\uc774 \ud5e4\uc774\uc990\uc758 \uc5b4\uba38\ub2c8 \ud504\ub798\ub2c8 \ub7ad\ucee4\uc2a4\ud130\ub97c, \ub0c7 \uc6b8\ud504\uac00 \uc6cc\ud130\uc2a4\uc758 \ub9f9\uc778 \uce5c\uad6c \uc544\uc774\uc791\uc744 \uc5f0\uae30\ud558\ub294 \uac83\uc774 \ubcf4\ub3c4\ub418\uc5c8\ub2e4. \uc6b8\ud504\ub294 \ud5c8\ud504\ud3ec\uc2a4\ud2b8 \ub77c\uc774\ube0c\uc5d0\uc11c \"\ub098\ub294 \ud765\ubd84\ub418\uba70 \uc6b4\uc774 \uc88b\uc544\uc694. \ub0b4\uac00 \ub6f0\uc5b4\ub09c \uc7ac\ub2a5\uc744 \uac00\uc9c4 \uc0ac\ub78c\ub4e4\uacfc \ud611\ub825\ud560 \uc218 \uc788\uc5b4\uc11c\uc694. \uba4b\uc9c4 \uc0ac\ub78c\ub4e4\uacfc \uac19\uc774 \uc77c\uc744 \ud558\uae30 \ub54c\ubb38\uc5d0, \ub098\ub294 \uc601\ud654\uc5d0\uc11c \uc88b\uac8c \ubcf4\uc5ec\uc9c0\ub294 \uac70\uc5d0\uc694.\"\ub77c\uace0 \ub9d0\ud588\ub2e4. 8\uc6d4 14\uc77c, \uc0d8 \ud2b8\ub798\uba40\uc774 \ud5e4\uc774\uc990\uc758 \uc544\ubc84\uc9c0\uc778 \ub9c8\uc774\ud074 \ub7ad\ucee4\uc2a4\ud130 \uc5ed\uc5d0 \uc815\ud574\uc84c\ub2e4. 8\uc6d4 28\uc77c, \uc6d0\uc791\uc790 \uc778 \uc874 \uadf8\ub9b0\uc774 \uc790\uc2e0\uc758 \ud2b8\uc704\ud130\uc5d0\uc11c \ub9c8\uc774\ud06c \ubc84\ube44\uae00\ub9ac\uc544\uac00 \ud328\ud2b8\ub9ad\uc744 \uc5f0\uae30\ud560 \uac83\uc774\ub77c\uace0 \ubc1d\ud614\ub2e4. \ub610\ud55c 9\uc6d4 6\uc77c\uc5d0 \uadf8\ub9b0\uc740 \uc70c\ub7fc \ub354\ud3ec\uac00 \ubc34 \ud558\uc6b0\ud2bc\uc744 \uc5f0\uae30\ud55c\ub2e4\uace0 \ud2b8\uc717\ud558\uc600\ub2e4."} {"question": "\uae40\ud0dd\uc6a9\uc758 \uc804\uc5ed\uc77c\uc740 \uc5b8\uc81c\uc778\uac00?", "paragraph": "11\uc6d4 20\uc77c \uc6d4\uc694\uc77c, \uae40\ud0dd\uc6a9\uc740 \ubcf8\uc778\uc758 \uac1c\uc778\ubc29\uc1a1\uc5d0\uc11c \uc785\ub300 \uacf5\uc9c0\ub97c \ub744\uc6e0\ub2e4. ASL \uc2dc\uc98c4 \ubc30\uacbd\uc74c\uc545\uc73c\ub85c '\uae40\uad11\uc11d-\uc774\ub4f1\ubcd1\uc758 \ud3b8\uc9c0'\ub97c \uc0ac\uc6a9\ud558\uae30\ub3c4 \ud588\uace0, 8\uac15 \uc2b9\uc790 \uc778\ud130\ubdf0\uc5d0\uc11c \uc9c1\uc811 \ub9c8\uc9c0\ub9c9\uc77c \uc218 \uc788\ub294 \ub300\ud68c\ub77c\uace0 \uc5b8\uae09\ud558\ub294 \ub4f1 \uae40\ud0dd\uc6a9\uc740 \uad70 \uc785\ub300\uc5d0 \ub300\ud55c \uc608\uace0\ub97c \uc9c0\uc18d\uc801\uc73c\ub85c \ud574\uc654\ub2e4. 11\uc6d4 20\uc77c 10\uc2dc 39\ubd84 \uae40\ud0dd\uc6a9\uc740 \uac1c\uc778 \ubc29\uc1a1\uc744 \ud1b5\ud574 \uc785\ub300\uc0ac\uc2e4\uc744 \uc804\ud588\ub2e4. \uae40\ud0dd\uc6a9\uc740 \uc785\ub300 \uc804 \uc544\ud504\ub9ac\uce74 \ud2f0\ube44 \ubc29\uc1a1\uc744 \ud1b5\ud574 \uc774\uc601\ud638, \uc5fc\ubcf4\uc131, \ucca0\uad6c\uc640 \ud568\uaed8 \uc0ad\ubc1c\uc2dd\uc744 \uc9c1\uc811 \uc0dd\uc911\uacc4\ud558\uace0 \uc2dc\uccad\uc790\ub4e4\uc5d0 \ub300\ud55c \uacbd\ub840\ub97c \ub9c8\uc9c0\ub9c9\uc73c\ub85c \ubc29\uc1a1\uc744 \uc885\ub8cc\ud588\ub2e4. \ub9c8\uc9c0\ub9c9 \ubc29\uc1a1\uc5d0\uc11c \uae40\ud0dd\uc6a9\uc740 \u201c\ubd80\ub044\ub7fd\ub124\uc694. \uc544 \ubd80\ub044\ub7fd\ub2e4\u201d\ub77c\uba70 \u201c\uadf8\ub3d9\uc548 \uc0ac\ub791\ud574\uc8fc\uc154\uc11c \uac10\uc0ac\ub4dc\ub9ac\uace0 SK \uc774\uc801\ud560 \ub54c \ub208\ubb3c \ub0ac\uc5c8\ub294\ub370 \uadf8 \uc774\ud6c4\ub85c \ub610 \ub208\ubb3c\uc774 \ub0a0 \uac83 \uac19\ub2e4. \ub9c8\uc9c0\ub9c9 \uac00\ub294 \ubaa8\uc2b5 \ub2e4\ub4e4 \ucc59\uaca8\uc918\uc11c \uac10\uc0ac\ub4dc\ub9b0\ub2e4\u201d\ub77c\uba70 \ud32c\ub4e4\uc5d0\uac8c \uac10\uc0ac \uc778\uc0ac\ub97c \uc804\ud588\ub2e4. \uc774\uc5b4 \u201c\uc804\uc5ed\uc77c\uc740 2019\ub144 9\uc6d4 4\uc77c\uc774\ub2e4. \ub3cc\uc544\uc640\uc11c \uc0c8\ub85c\uc6b4 \ubaa8\uc2b5\uc73c\ub85c \uc5ec\ub7ec\ubd84\uc744 \ub9cc\ub098\uaca0\ub2e4\u201d\ub77c\uba70 \uad70 \uc785\ub300 \uac01\uc624\ub97c \uc804\ud588\ub2e4.\uae40\ud0dd\uc6a9\uc740 2017\ub144 12\uc6d4 5\uc77c \uc138\uc885\uc2dc\uc5d0 \uc704\uce58\ud55c \ubc31\ub8e1\ubd80\ub300\ub85c \uc785\ub300\ud558\uc5ec 2\ub144\uc5ec\uac04\uc758 \uad70\uc0dd\ud65c\uc744 \uc2dc\uc791\ud588\uace0 \uc804\uc5ed\uc77c\uc740 2019\ub144 9\uc6d4 4\uc77c\uc774\ub2e4.", "answer": "2019\ub144 9\uc6d4 4\uc77c", "sentence": "\uc774\uc5b4 \u201c\uc804\uc5ed\uc77c\uc740 2019\ub144 9\uc6d4 4\uc77c \uc774\ub2e4.", "paragraph_sentence": "11\uc6d4 20\uc77c \uc6d4\uc694\uc77c, \uae40\ud0dd\uc6a9\uc740 \ubcf8\uc778\uc758 \uac1c\uc778\ubc29\uc1a1\uc5d0\uc11c \uc785\ub300 \uacf5\uc9c0\ub97c \ub744\uc6e0\ub2e4. ASL \uc2dc\uc98c4 \ubc30\uacbd\uc74c\uc545\uc73c\ub85c '\uae40\uad11\uc11d-\uc774\ub4f1\ubcd1\uc758 \ud3b8\uc9c0'\ub97c \uc0ac\uc6a9\ud558\uae30\ub3c4 \ud588\uace0, 8\uac15 \uc2b9\uc790 \uc778\ud130\ubdf0\uc5d0\uc11c \uc9c1\uc811 \ub9c8\uc9c0\ub9c9\uc77c \uc218 \uc788\ub294 \ub300\ud68c\ub77c\uace0 \uc5b8\uae09\ud558\ub294 \ub4f1 \uae40\ud0dd\uc6a9\uc740 \uad70 \uc785\ub300\uc5d0 \ub300\ud55c \uc608\uace0\ub97c \uc9c0\uc18d\uc801\uc73c\ub85c \ud574\uc654\ub2e4. 11\uc6d4 20\uc77c 10\uc2dc 39\ubd84 \uae40\ud0dd\uc6a9\uc740 \uac1c\uc778 \ubc29\uc1a1\uc744 \ud1b5\ud574 \uc785\ub300\uc0ac\uc2e4\uc744 \uc804\ud588\ub2e4. \uae40\ud0dd\uc6a9\uc740 \uc785\ub300 \uc804 \uc544\ud504\ub9ac\uce74 \ud2f0\ube44 \ubc29\uc1a1\uc744 \ud1b5\ud574 \uc774\uc601\ud638, \uc5fc\ubcf4\uc131, \ucca0\uad6c\uc640 \ud568\uaed8 \uc0ad\ubc1c\uc2dd\uc744 \uc9c1\uc811 \uc0dd\uc911\uacc4\ud558\uace0 \uc2dc\uccad\uc790\ub4e4\uc5d0 \ub300\ud55c \uacbd\ub840\ub97c \ub9c8\uc9c0\ub9c9\uc73c\ub85c \ubc29\uc1a1\uc744 \uc885\ub8cc\ud588\ub2e4. \ub9c8\uc9c0\ub9c9 \ubc29\uc1a1\uc5d0\uc11c \uae40\ud0dd\uc6a9\uc740 \u201c\ubd80\ub044\ub7fd\ub124\uc694. \uc544 \ubd80\ub044\ub7fd\ub2e4\u201d\ub77c\uba70 \u201c\uadf8\ub3d9\uc548 \uc0ac\ub791\ud574\uc8fc\uc154\uc11c \uac10\uc0ac\ub4dc\ub9ac\uace0 SK \uc774\uc801\ud560 \ub54c \ub208\ubb3c \ub0ac\uc5c8\ub294\ub370 \uadf8 \uc774\ud6c4\ub85c \ub610 \ub208\ubb3c\uc774 \ub0a0 \uac83 \uac19\ub2e4. \ub9c8\uc9c0\ub9c9 \uac00\ub294 \ubaa8\uc2b5 \ub2e4\ub4e4 \ucc59\uaca8\uc918\uc11c \uac10\uc0ac\ub4dc\ub9b0\ub2e4\u201d\ub77c\uba70 \ud32c\ub4e4\uc5d0\uac8c \uac10\uc0ac \uc778\uc0ac\ub97c \uc804\ud588\ub2e4. \uc774\uc5b4 \u201c\uc804\uc5ed\uc77c\uc740 2019\ub144 9\uc6d4 4\uc77c \uc774\ub2e4. \ub3cc\uc544\uc640\uc11c \uc0c8\ub85c\uc6b4 \ubaa8\uc2b5\uc73c\ub85c \uc5ec\ub7ec\ubd84\uc744 \ub9cc\ub098\uaca0\ub2e4\u201d\ub77c\uba70 \uad70 \uc785\ub300 \uac01\uc624\ub97c \uc804\ud588\ub2e4.\uae40\ud0dd\uc6a9\uc740 2017\ub144 12\uc6d4 5\uc77c \uc138\uc885\uc2dc\uc5d0 \uc704\uce58\ud55c \ubc31\ub8e1\ubd80\ub300\ub85c \uc785\ub300\ud558\uc5ec 2\ub144\uc5ec\uac04\uc758 \uad70\uc0dd\ud65c\uc744 \uc2dc\uc791\ud588\uace0 \uc804\uc5ed\uc77c\uc740 2019\ub144 9\uc6d4 4\uc77c\uc774\ub2e4.", "paragraph_answer": "11\uc6d4 20\uc77c \uc6d4\uc694\uc77c, \uae40\ud0dd\uc6a9\uc740 \ubcf8\uc778\uc758 \uac1c\uc778\ubc29\uc1a1\uc5d0\uc11c \uc785\ub300 \uacf5\uc9c0\ub97c \ub744\uc6e0\ub2e4. ASL \uc2dc\uc98c4 \ubc30\uacbd\uc74c\uc545\uc73c\ub85c '\uae40\uad11\uc11d-\uc774\ub4f1\ubcd1\uc758 \ud3b8\uc9c0'\ub97c \uc0ac\uc6a9\ud558\uae30\ub3c4 \ud588\uace0, 8\uac15 \uc2b9\uc790 \uc778\ud130\ubdf0\uc5d0\uc11c \uc9c1\uc811 \ub9c8\uc9c0\ub9c9\uc77c \uc218 \uc788\ub294 \ub300\ud68c\ub77c\uace0 \uc5b8\uae09\ud558\ub294 \ub4f1 \uae40\ud0dd\uc6a9\uc740 \uad70 \uc785\ub300\uc5d0 \ub300\ud55c \uc608\uace0\ub97c \uc9c0\uc18d\uc801\uc73c\ub85c \ud574\uc654\ub2e4. 11\uc6d4 20\uc77c 10\uc2dc 39\ubd84 \uae40\ud0dd\uc6a9\uc740 \uac1c\uc778 \ubc29\uc1a1\uc744 \ud1b5\ud574 \uc785\ub300\uc0ac\uc2e4\uc744 \uc804\ud588\ub2e4. \uae40\ud0dd\uc6a9\uc740 \uc785\ub300 \uc804 \uc544\ud504\ub9ac\uce74 \ud2f0\ube44 \ubc29\uc1a1\uc744 \ud1b5\ud574 \uc774\uc601\ud638, \uc5fc\ubcf4\uc131, \ucca0\uad6c\uc640 \ud568\uaed8 \uc0ad\ubc1c\uc2dd\uc744 \uc9c1\uc811 \uc0dd\uc911\uacc4\ud558\uace0 \uc2dc\uccad\uc790\ub4e4\uc5d0 \ub300\ud55c \uacbd\ub840\ub97c \ub9c8\uc9c0\ub9c9\uc73c\ub85c \ubc29\uc1a1\uc744 \uc885\ub8cc\ud588\ub2e4. \ub9c8\uc9c0\ub9c9 \ubc29\uc1a1\uc5d0\uc11c \uae40\ud0dd\uc6a9\uc740 \u201c\ubd80\ub044\ub7fd\ub124\uc694. \uc544 \ubd80\ub044\ub7fd\ub2e4\u201d\ub77c\uba70 \u201c\uadf8\ub3d9\uc548 \uc0ac\ub791\ud574\uc8fc\uc154\uc11c \uac10\uc0ac\ub4dc\ub9ac\uace0 SK \uc774\uc801\ud560 \ub54c \ub208\ubb3c \ub0ac\uc5c8\ub294\ub370 \uadf8 \uc774\ud6c4\ub85c \ub610 \ub208\ubb3c\uc774 \ub0a0 \uac83 \uac19\ub2e4. \ub9c8\uc9c0\ub9c9 \uac00\ub294 \ubaa8\uc2b5 \ub2e4\ub4e4 \ucc59\uaca8\uc918\uc11c \uac10\uc0ac\ub4dc\ub9b0\ub2e4\u201d\ub77c\uba70 \ud32c\ub4e4\uc5d0\uac8c \uac10\uc0ac \uc778\uc0ac\ub97c \uc804\ud588\ub2e4. \uc774\uc5b4 \u201c\uc804\uc5ed\uc77c\uc740 2019\ub144 9\uc6d4 4\uc77c \uc774\ub2e4. \ub3cc\uc544\uc640\uc11c \uc0c8\ub85c\uc6b4 \ubaa8\uc2b5\uc73c\ub85c \uc5ec\ub7ec\ubd84\uc744 \ub9cc\ub098\uaca0\ub2e4\u201d\ub77c\uba70 \uad70 \uc785\ub300 \uac01\uc624\ub97c \uc804\ud588\ub2e4.\uae40\ud0dd\uc6a9\uc740 2017\ub144 12\uc6d4 5\uc77c \uc138\uc885\uc2dc\uc5d0 \uc704\uce58\ud55c \ubc31\ub8e1\ubd80\ub300\ub85c \uc785\ub300\ud558\uc5ec 2\ub144\uc5ec\uac04\uc758 \uad70\uc0dd\ud65c\uc744 \uc2dc\uc791\ud588\uace0 \uc804\uc5ed\uc77c\uc740 2019\ub144 9\uc6d4 4\uc77c\uc774\ub2e4.", "sentence_answer": "\uc774\uc5b4 \u201c\uc804\uc5ed\uc77c\uc740 2019\ub144 9\uc6d4 4\uc77c \uc774\ub2e4.", "paragraph_id": "6656802-5-1", "paragraph_question": "question: \uae40\ud0dd\uc6a9\uc758 \uc804\uc5ed\uc77c\uc740 \uc5b8\uc81c\uc778\uac00?, context: 11\uc6d4 20\uc77c \uc6d4\uc694\uc77c, \uae40\ud0dd\uc6a9\uc740 \ubcf8\uc778\uc758 \uac1c\uc778\ubc29\uc1a1\uc5d0\uc11c \uc785\ub300 \uacf5\uc9c0\ub97c \ub744\uc6e0\ub2e4. ASL \uc2dc\uc98c4 \ubc30\uacbd\uc74c\uc545\uc73c\ub85c '\uae40\uad11\uc11d-\uc774\ub4f1\ubcd1\uc758 \ud3b8\uc9c0'\ub97c \uc0ac\uc6a9\ud558\uae30\ub3c4 \ud588\uace0, 8\uac15 \uc2b9\uc790 \uc778\ud130\ubdf0\uc5d0\uc11c \uc9c1\uc811 \ub9c8\uc9c0\ub9c9\uc77c \uc218 \uc788\ub294 \ub300\ud68c\ub77c\uace0 \uc5b8\uae09\ud558\ub294 \ub4f1 \uae40\ud0dd\uc6a9\uc740 \uad70 \uc785\ub300\uc5d0 \ub300\ud55c \uc608\uace0\ub97c \uc9c0\uc18d\uc801\uc73c\ub85c \ud574\uc654\ub2e4. 11\uc6d4 20\uc77c 10\uc2dc 39\ubd84 \uae40\ud0dd\uc6a9\uc740 \uac1c\uc778 \ubc29\uc1a1\uc744 \ud1b5\ud574 \uc785\ub300\uc0ac\uc2e4\uc744 \uc804\ud588\ub2e4. \uae40\ud0dd\uc6a9\uc740 \uc785\ub300 \uc804 \uc544\ud504\ub9ac\uce74 \ud2f0\ube44 \ubc29\uc1a1\uc744 \ud1b5\ud574 \uc774\uc601\ud638, \uc5fc\ubcf4\uc131, \ucca0\uad6c\uc640 \ud568\uaed8 \uc0ad\ubc1c\uc2dd\uc744 \uc9c1\uc811 \uc0dd\uc911\uacc4\ud558\uace0 \uc2dc\uccad\uc790\ub4e4\uc5d0 \ub300\ud55c \uacbd\ub840\ub97c \ub9c8\uc9c0\ub9c9\uc73c\ub85c \ubc29\uc1a1\uc744 \uc885\ub8cc\ud588\ub2e4. \ub9c8\uc9c0\ub9c9 \ubc29\uc1a1\uc5d0\uc11c \uae40\ud0dd\uc6a9\uc740 \u201c\ubd80\ub044\ub7fd\ub124\uc694. \uc544 \ubd80\ub044\ub7fd\ub2e4\u201d\ub77c\uba70 \u201c\uadf8\ub3d9\uc548 \uc0ac\ub791\ud574\uc8fc\uc154\uc11c \uac10\uc0ac\ub4dc\ub9ac\uace0 SK \uc774\uc801\ud560 \ub54c \ub208\ubb3c \ub0ac\uc5c8\ub294\ub370 \uadf8 \uc774\ud6c4\ub85c \ub610 \ub208\ubb3c\uc774 \ub0a0 \uac83 \uac19\ub2e4. \ub9c8\uc9c0\ub9c9 \uac00\ub294 \ubaa8\uc2b5 \ub2e4\ub4e4 \ucc59\uaca8\uc918\uc11c \uac10\uc0ac\ub4dc\ub9b0\ub2e4\u201d\ub77c\uba70 \ud32c\ub4e4\uc5d0\uac8c \uac10\uc0ac \uc778\uc0ac\ub97c \uc804\ud588\ub2e4. \uc774\uc5b4 \u201c\uc804\uc5ed\uc77c\uc740 2019\ub144 9\uc6d4 4\uc77c\uc774\ub2e4. \ub3cc\uc544\uc640\uc11c \uc0c8\ub85c\uc6b4 \ubaa8\uc2b5\uc73c\ub85c \uc5ec\ub7ec\ubd84\uc744 \ub9cc\ub098\uaca0\ub2e4\u201d\ub77c\uba70 \uad70 \uc785\ub300 \uac01\uc624\ub97c \uc804\ud588\ub2e4.\uae40\ud0dd\uc6a9\uc740 2017\ub144 12\uc6d4 5\uc77c \uc138\uc885\uc2dc\uc5d0 \uc704\uce58\ud55c \ubc31\ub8e1\ubd80\ub300\ub85c \uc785\ub300\ud558\uc5ec 2\ub144\uc5ec\uac04\uc758 \uad70\uc0dd\ud65c\uc744 \uc2dc\uc791\ud588\uace0 \uc804\uc5ed\uc77c\uc740 2019\ub144 9\uc6d4 4\uc77c\uc774\ub2e4."} {"question": "\ubc18\uacf5\uc8fc\uc758\uc801\uc774\uace0 \ubc18\uc790\ubcf8\uc8fc\uc758\uc801\uc774\uba74\uc11c \uc778\uc885 \ucc28\ubcc4\uc744 \ud558\uc9c0 \uc54a\uc740 \uc544\ub974\ud5e8\ud2f0\ub098\uc758 \ub300\ud1b5\ub839\uc758 \uc774\ub984\uc740?", "paragraph": "\uc544\ub974\ud5e8\ud2f0\ub098\uc5d0\uc11c\uc758 \uc81c3\uc758 \uc704\uce58 \uc774\ub860\uc740 \uc88c\uc775\uc801 \uc81c3\uc758 \uc704\uce58 \uc774\ub860\uacfc \uac70\uc758 \ub3d9\uc77c\uc2dc \ub418\uc5c8\ub2e4. \ub0c9\uc804 \uc2dc\uc808 \uc804 \uc544\ub974\ud5e8\ud2f0\ub098\uc758 \ub300\ud1b5\ub839\uc778 \ud6c4\uc548 \ud398\ub860(1946\ub144 ~ 1955\ub144, 1973\ub144 ~ 1974\ub144)\uc740 \ucd08\uae30 \ud30c\uc2dc\uc998\uacfc \ub9e4\uc6b0 \uac19\uc740 \uc591\uc0c1\uc73c\ub85c \uad6d\uac00 \uc6b4\uc601\uc744 \ubcf4\uc5ec\uc8fc\uc5c8\ub2e4. \uc778\uc885\uc8fc\uc758\uc640 \uc81c\uad6d\uc8fc\uc758\uac00 \uc5c6\ub294 \ucd08\uae30 \ud30c\uc2dc\uc998\uacfc \uac19\uc774 \ud6c4\uc548 \ud398\ub860\uc740 \ubc18\uacf5\uc8fc\uc758\uc801\uc774\uace0 \ubc18\uc790\ubcf8\uc8fc\uc758\uc801\uc774\uba74\uc11c \uc778\uc885 \ucc28\ubcc4\uc744 \ud558\uc9c0 \uc54a\uc558\uc73c\uba70, \uc720\ub300\uc778\uc744 \ud0c4\uc555\ud558\uc9c0 \uc54a\uc558\ub2e4. \ub300\uc2e0 \uad6d\uac00\ud1b5\uc81c\uc8fc\uc758\uc801\uc774\uace0 \ube44-\uc778\uc885\ucc28\ubcc4\uc801 \ubbfc\uc871\uc8fc\uc758 \uc815\uce58 \ud0dc\uc138\ub97c \ubcf4\uc5ec\uc654\ub2e4. \uadf8\ub294 \uc790\uc2e0\uc758 \uc815\uce58\ub97c \uba85\ud655\ud558\uac8c \ud30c\uc2dc\uc998\uc774\ub77c\uace0 \ud558\uc9c0\ub294 \uc54a\uc558\uc9c0\ub9cc '\uad6d\uc81c\uc8fc\uc758\uc801 \uc81c3\uc758 \uc218\ub2e8' \ub610\ub294 '\ud398\ub860\uc8fc\uc758'\ub77c\uace0 \uc77c\uceec\uc5c8\ub2e4. \uadf8\ub294 \ub300\ud1b5\ub839\uc774 \ub418\uae30 \uc804 \ub2f9\uc2dc \uc774\ud0c8\ub9ac\uc544\uc758 \ubca0\ub2c8\ud1a0 \ubb34\uc194\ub9ac\ub2c8 \uc815\uce58\ub97c \uc639\ud638\ud588\uc73c\uba70, \uc774\ub7ec\ud55c \ud589\uc704\ub294 \uadf8\uac00 \ucd95\ucd9c\ub2f9\ud558\ub294 \uacb0\uc815\uc801 \uacc4\uae30\uac00 \ub41c\ub2e4.", "answer": "\ud6c4\uc548 \ud398\ub860", "sentence": "\ub0c9\uc804 \uc2dc\uc808 \uc804 \uc544\ub974\ud5e8\ud2f0\ub098\uc758 \ub300\ud1b5\ub839\uc778 \ud6c4\uc548 \ud398\ub860 (1946\ub144 ~ 1955\ub144, 1973\ub144 ~ 1974\ub144)\uc740 \ucd08\uae30 \ud30c\uc2dc\uc998\uacfc \ub9e4\uc6b0 \uac19\uc740 \uc591\uc0c1\uc73c\ub85c \uad6d\uac00 \uc6b4\uc601\uc744 \ubcf4\uc5ec\uc8fc\uc5c8\ub2e4.", "paragraph_sentence": "\uc544\ub974\ud5e8\ud2f0\ub098\uc5d0\uc11c\uc758 \uc81c3\uc758 \uc704\uce58 \uc774\ub860\uc740 \uc88c\uc775\uc801 \uc81c3\uc758 \uc704\uce58 \uc774\ub860\uacfc \uac70\uc758 \ub3d9\uc77c\uc2dc \ub418\uc5c8\ub2e4. \ub0c9\uc804 \uc2dc\uc808 \uc804 \uc544\ub974\ud5e8\ud2f0\ub098\uc758 \ub300\ud1b5\ub839\uc778 \ud6c4\uc548 \ud398\ub860 (1946\ub144 ~ 1955\ub144, 1973\ub144 ~ 1974\ub144)\uc740 \ucd08\uae30 \ud30c\uc2dc\uc998\uacfc \ub9e4\uc6b0 \uac19\uc740 \uc591\uc0c1\uc73c\ub85c \uad6d\uac00 \uc6b4\uc601\uc744 \ubcf4\uc5ec\uc8fc\uc5c8\ub2e4. \uc778\uc885\uc8fc\uc758\uc640 \uc81c\uad6d\uc8fc\uc758\uac00 \uc5c6\ub294 \ucd08\uae30 \ud30c\uc2dc\uc998\uacfc \uac19\uc774 \ud6c4\uc548 \ud398\ub860\uc740 \ubc18\uacf5\uc8fc\uc758\uc801\uc774\uace0 \ubc18\uc790\ubcf8\uc8fc\uc758\uc801\uc774\uba74\uc11c \uc778\uc885 \ucc28\ubcc4\uc744 \ud558\uc9c0 \uc54a\uc558\uc73c\uba70, \uc720\ub300\uc778\uc744 \ud0c4\uc555\ud558\uc9c0 \uc54a\uc558\ub2e4. \ub300\uc2e0 \uad6d\uac00\ud1b5\uc81c\uc8fc\uc758\uc801\uc774\uace0 \ube44-\uc778\uc885\ucc28\ubcc4\uc801 \ubbfc\uc871\uc8fc\uc758 \uc815\uce58 \ud0dc\uc138\ub97c \ubcf4\uc5ec\uc654\ub2e4. \uadf8\ub294 \uc790\uc2e0\uc758 \uc815\uce58\ub97c \uba85\ud655\ud558\uac8c \ud30c\uc2dc\uc998\uc774\ub77c\uace0 \ud558\uc9c0\ub294 \uc54a\uc558\uc9c0\ub9cc '\uad6d\uc81c\uc8fc\uc758\uc801 \uc81c3\uc758 \uc218\ub2e8' \ub610\ub294 '\ud398\ub860\uc8fc\uc758'\ub77c\uace0 \uc77c\uceec\uc5c8\ub2e4. \uadf8\ub294 \ub300\ud1b5\ub839\uc774 \ub418\uae30 \uc804 \ub2f9\uc2dc \uc774\ud0c8\ub9ac\uc544\uc758 \ubca0\ub2c8\ud1a0 \ubb34\uc194\ub9ac\ub2c8 \uc815\uce58\ub97c \uc639\ud638\ud588\uc73c\uba70, \uc774\ub7ec\ud55c \ud589\uc704\ub294 \uadf8\uac00 \ucd95\ucd9c\ub2f9\ud558\ub294 \uacb0\uc815\uc801 \uacc4\uae30\uac00 \ub41c\ub2e4.", "paragraph_answer": "\uc544\ub974\ud5e8\ud2f0\ub098\uc5d0\uc11c\uc758 \uc81c3\uc758 \uc704\uce58 \uc774\ub860\uc740 \uc88c\uc775\uc801 \uc81c3\uc758 \uc704\uce58 \uc774\ub860\uacfc \uac70\uc758 \ub3d9\uc77c\uc2dc \ub418\uc5c8\ub2e4. \ub0c9\uc804 \uc2dc\uc808 \uc804 \uc544\ub974\ud5e8\ud2f0\ub098\uc758 \ub300\ud1b5\ub839\uc778 \ud6c4\uc548 \ud398\ub860 (1946\ub144 ~ 1955\ub144, 1973\ub144 ~ 1974\ub144)\uc740 \ucd08\uae30 \ud30c\uc2dc\uc998\uacfc \ub9e4\uc6b0 \uac19\uc740 \uc591\uc0c1\uc73c\ub85c \uad6d\uac00 \uc6b4\uc601\uc744 \ubcf4\uc5ec\uc8fc\uc5c8\ub2e4. \uc778\uc885\uc8fc\uc758\uc640 \uc81c\uad6d\uc8fc\uc758\uac00 \uc5c6\ub294 \ucd08\uae30 \ud30c\uc2dc\uc998\uacfc \uac19\uc774 \ud6c4\uc548 \ud398\ub860\uc740 \ubc18\uacf5\uc8fc\uc758\uc801\uc774\uace0 \ubc18\uc790\ubcf8\uc8fc\uc758\uc801\uc774\uba74\uc11c \uc778\uc885 \ucc28\ubcc4\uc744 \ud558\uc9c0 \uc54a\uc558\uc73c\uba70, \uc720\ub300\uc778\uc744 \ud0c4\uc555\ud558\uc9c0 \uc54a\uc558\ub2e4. \ub300\uc2e0 \uad6d\uac00\ud1b5\uc81c\uc8fc\uc758\uc801\uc774\uace0 \ube44-\uc778\uc885\ucc28\ubcc4\uc801 \ubbfc\uc871\uc8fc\uc758 \uc815\uce58 \ud0dc\uc138\ub97c \ubcf4\uc5ec\uc654\ub2e4. \uadf8\ub294 \uc790\uc2e0\uc758 \uc815\uce58\ub97c \uba85\ud655\ud558\uac8c \ud30c\uc2dc\uc998\uc774\ub77c\uace0 \ud558\uc9c0\ub294 \uc54a\uc558\uc9c0\ub9cc '\uad6d\uc81c\uc8fc\uc758\uc801 \uc81c3\uc758 \uc218\ub2e8' \ub610\ub294 '\ud398\ub860\uc8fc\uc758'\ub77c\uace0 \uc77c\uceec\uc5c8\ub2e4. \uadf8\ub294 \ub300\ud1b5\ub839\uc774 \ub418\uae30 \uc804 \ub2f9\uc2dc \uc774\ud0c8\ub9ac\uc544\uc758 \ubca0\ub2c8\ud1a0 \ubb34\uc194\ub9ac\ub2c8 \uc815\uce58\ub97c \uc639\ud638\ud588\uc73c\uba70, \uc774\ub7ec\ud55c \ud589\uc704\ub294 \uadf8\uac00 \ucd95\ucd9c\ub2f9\ud558\ub294 \uacb0\uc815\uc801 \uacc4\uae30\uac00 \ub41c\ub2e4.", "sentence_answer": "\ub0c9\uc804 \uc2dc\uc808 \uc804 \uc544\ub974\ud5e8\ud2f0\ub098\uc758 \ub300\ud1b5\ub839\uc778 \ud6c4\uc548 \ud398\ub860 (1946\ub144 ~ 1955\ub144, 1973\ub144 ~ 1974\ub144)\uc740 \ucd08\uae30 \ud30c\uc2dc\uc998\uacfc \ub9e4\uc6b0 \uac19\uc740 \uc591\uc0c1\uc73c\ub85c \uad6d\uac00 \uc6b4\uc601\uc744 \ubcf4\uc5ec\uc8fc\uc5c8\ub2e4.", "paragraph_id": "6459770-3-0", "paragraph_question": "question: \ubc18\uacf5\uc8fc\uc758\uc801\uc774\uace0 \ubc18\uc790\ubcf8\uc8fc\uc758\uc801\uc774\uba74\uc11c \uc778\uc885 \ucc28\ubcc4\uc744 \ud558\uc9c0 \uc54a\uc740 \uc544\ub974\ud5e8\ud2f0\ub098\uc758 \ub300\ud1b5\ub839\uc758 \uc774\ub984\uc740?, context: \uc544\ub974\ud5e8\ud2f0\ub098\uc5d0\uc11c\uc758 \uc81c3\uc758 \uc704\uce58 \uc774\ub860\uc740 \uc88c\uc775\uc801 \uc81c3\uc758 \uc704\uce58 \uc774\ub860\uacfc \uac70\uc758 \ub3d9\uc77c\uc2dc \ub418\uc5c8\ub2e4. \ub0c9\uc804 \uc2dc\uc808 \uc804 \uc544\ub974\ud5e8\ud2f0\ub098\uc758 \ub300\ud1b5\ub839\uc778 \ud6c4\uc548 \ud398\ub860(1946\ub144 ~ 1955\ub144, 1973\ub144 ~ 1974\ub144)\uc740 \ucd08\uae30 \ud30c\uc2dc\uc998\uacfc \ub9e4\uc6b0 \uac19\uc740 \uc591\uc0c1\uc73c\ub85c \uad6d\uac00 \uc6b4\uc601\uc744 \ubcf4\uc5ec\uc8fc\uc5c8\ub2e4. \uc778\uc885\uc8fc\uc758\uc640 \uc81c\uad6d\uc8fc\uc758\uac00 \uc5c6\ub294 \ucd08\uae30 \ud30c\uc2dc\uc998\uacfc \uac19\uc774 \ud6c4\uc548 \ud398\ub860\uc740 \ubc18\uacf5\uc8fc\uc758\uc801\uc774\uace0 \ubc18\uc790\ubcf8\uc8fc\uc758\uc801\uc774\uba74\uc11c \uc778\uc885 \ucc28\ubcc4\uc744 \ud558\uc9c0 \uc54a\uc558\uc73c\uba70, \uc720\ub300\uc778\uc744 \ud0c4\uc555\ud558\uc9c0 \uc54a\uc558\ub2e4. \ub300\uc2e0 \uad6d\uac00\ud1b5\uc81c\uc8fc\uc758\uc801\uc774\uace0 \ube44-\uc778\uc885\ucc28\ubcc4\uc801 \ubbfc\uc871\uc8fc\uc758 \uc815\uce58 \ud0dc\uc138\ub97c \ubcf4\uc5ec\uc654\ub2e4. \uadf8\ub294 \uc790\uc2e0\uc758 \uc815\uce58\ub97c \uba85\ud655\ud558\uac8c \ud30c\uc2dc\uc998\uc774\ub77c\uace0 \ud558\uc9c0\ub294 \uc54a\uc558\uc9c0\ub9cc '\uad6d\uc81c\uc8fc\uc758\uc801 \uc81c3\uc758 \uc218\ub2e8' \ub610\ub294 '\ud398\ub860\uc8fc\uc758'\ub77c\uace0 \uc77c\uceec\uc5c8\ub2e4. \uadf8\ub294 \ub300\ud1b5\ub839\uc774 \ub418\uae30 \uc804 \ub2f9\uc2dc \uc774\ud0c8\ub9ac\uc544\uc758 \ubca0\ub2c8\ud1a0 \ubb34\uc194\ub9ac\ub2c8 \uc815\uce58\ub97c \uc639\ud638\ud588\uc73c\uba70, \uc774\ub7ec\ud55c \ud589\uc704\ub294 \uadf8\uac00 \ucd95\ucd9c\ub2f9\ud558\ub294 \uacb0\uc815\uc801 \uacc4\uae30\uac00 \ub41c\ub2e4."} {"question": "2015\ub144 \uc138\uadf8\uc6e8\uc774\ub97c \uac1c\ub7c9\ud55c \uc911\uad6d \ub098\uc778\ubd07\uc0ac\uc758 \uc81c\ud488\uba85\uc740 \ubb34\uc5c7\uc778\uac00?", "paragraph": "\ucd08\uae30 \uc138\uadf8\uc6e8\uc774\uc758 \uac00\uaca9\uc740 1000\ub9cc \uc6d0 \ub300\ub97c \ud6cc\uca4d \ub6f0\uc5b4\ub118\uc5c8\ub2e4. \uc77c\ubc18 \ub300\uc911\ub4e4\uc774 \ub808\uc838\uc6a9 \ud639\uc740 \uc77c\uc0c1\uc5d0\uc11c \uc0dd\ud65c\ud558\ub824\uace0 \uad6c\ub9e4\ud558\uae30\uc5d0\ub294 \ubd80\ub2f4\uc2a4\ub7f0\uc6b4 \uae08\uc561\uc774\uc5c8\ub2e4. 2015\ub144 \ud604\uc7ac\uc5d0 \uc774\ub974\uae30\uae4c\uc9c0 \uac00\uaca9\uc774 \uc810\uc810 \ud558\ub77d\ud558\uae30\ub294 \ud588\uc9c0\ub9cc \uc544\uc9c1\uae4c\uc9c0\ub294 \ub300\uc911\ub4e4\uc774 \ubc1b\uc544\ub4e4\uc774\uae30\uc5d0\ub294 \ube44\uc2fc \uac00\uaca9\uc744 \uc720\uc9c0\ud558\uace0 \uc788\ub294 \ud604\uc2e4\uc774\ub2e4. \uc774\uc5d0 \ub300\ud55c \ud0c0\uac1c\ucc45\uc73c\ub85c \uc138\uadf8\uc6e8\uc774\uc0ac\ub97c \uc778\uc218\ud55c \uc911\uad6d\uc758 \ub098\uc778\ubd07\uc0ac\uac00 2015\ub144 10\uc6d4 19\uc77c '\ub098\uc778\ubd07 \ubbf8\ub2c8'\uc774\ub77c\ub294 \uc138\uadf8\uc6e8\uc774\ub97c \uac1c\ub7c9\ud55c \uae30\uae30\ub97c \ubc1c\ub9e4\ud558\uc5ec \uc911\uad6d\ub0b4\uc218\uc2dc\uc7a5\uc5d0\uc11c \ud55c\ud654\ub85c \uc57d 35\ub9cc\uc6d0\uc758 \uac00\uaca9\uc5d0 \ud310\ub9e4\ud558\uace0 \uc788\ub2e4. \uae30\uae30\uc758 \uae30\ub2a5\uc801 \uce21\uba74\uc5d0\uc11c \ud55c\uacc4\uc810\uc73c\ub85c\ub294 \uac00\ub3d9\uac70\ub9ac\ub97c \uaf3d\uc744 \uc218 \uc788\ub2e4. \ud55c \ubc88 \ucda9\uc804\uc744 \ud558\uba74 \ucd5c\ub300 25~30km\uae4c\uc9c0\ubc16\uc5d0 \uc6b4\ud589\uc744 \ud560 \uc218 \uc5c6\uae30 \ub54c\ubb38\uc5d0 \uc774 \ubd80\ubd84\uc5d0\uc11c\ub3c4 \ud55c\uacc4\uc810\uc744 \uc9c0\ub2c8\uace0 \uc788\ub2e4.", "answer": "\ub098\uc778\ubd07 \ubbf8\ub2c8", "sentence": "\uc774\uc5d0 \ub300\ud55c \ud0c0\uac1c\ucc45\uc73c\ub85c \uc138\uadf8\uc6e8\uc774\uc0ac\ub97c \uc778\uc218\ud55c \uc911\uad6d\uc758 \ub098\uc778\ubd07\uc0ac\uac00 2015\ub144 10\uc6d4 19\uc77c ' \ub098\uc778\ubd07 \ubbf8\ub2c8 '\uc774\ub77c\ub294 \uc138\uadf8\uc6e8\uc774\ub97c \uac1c\ub7c9\ud55c \uae30\uae30\ub97c \ubc1c\ub9e4\ud558\uc5ec \uc911\uad6d\ub0b4\uc218\uc2dc\uc7a5\uc5d0\uc11c \ud55c\ud654\ub85c \uc57d 35\ub9cc\uc6d0\uc758 \uac00\uaca9\uc5d0 \ud310\ub9e4\ud558\uace0 \uc788\ub2e4.", "paragraph_sentence": "\ucd08\uae30 \uc138\uadf8\uc6e8\uc774\uc758 \uac00\uaca9\uc740 1000\ub9cc \uc6d0 \ub300\ub97c \ud6cc\uca4d \ub6f0\uc5b4\ub118\uc5c8\ub2e4. \uc77c\ubc18 \ub300\uc911\ub4e4\uc774 \ub808\uc838\uc6a9 \ud639\uc740 \uc77c\uc0c1\uc5d0\uc11c \uc0dd\ud65c\ud558\ub824\uace0 \uad6c\ub9e4\ud558\uae30\uc5d0\ub294 \ubd80\ub2f4\uc2a4\ub7f0\uc6b4 \uae08\uc561\uc774\uc5c8\ub2e4. 2015\ub144 \ud604\uc7ac\uc5d0 \uc774\ub974\uae30\uae4c\uc9c0 \uac00\uaca9\uc774 \uc810\uc810 \ud558\ub77d\ud558\uae30\ub294 \ud588\uc9c0\ub9cc \uc544\uc9c1\uae4c\uc9c0\ub294 \ub300\uc911\ub4e4\uc774 \ubc1b\uc544\ub4e4\uc774\uae30\uc5d0\ub294 \ube44\uc2fc \uac00\uaca9\uc744 \uc720\uc9c0\ud558\uace0 \uc788\ub294 \ud604\uc2e4\uc774\ub2e4. \uc774\uc5d0 \ub300\ud55c \ud0c0\uac1c\ucc45\uc73c\ub85c \uc138\uadf8\uc6e8\uc774\uc0ac\ub97c \uc778\uc218\ud55c \uc911\uad6d\uc758 \ub098\uc778\ubd07\uc0ac\uac00 2015\ub144 10\uc6d4 19\uc77c ' \ub098\uc778\ubd07 \ubbf8\ub2c8 '\uc774\ub77c\ub294 \uc138\uadf8\uc6e8\uc774\ub97c \uac1c\ub7c9\ud55c \uae30\uae30\ub97c \ubc1c\ub9e4\ud558\uc5ec \uc911\uad6d\ub0b4\uc218\uc2dc\uc7a5\uc5d0\uc11c \ud55c\ud654\ub85c \uc57d 35\ub9cc\uc6d0\uc758 \uac00\uaca9\uc5d0 \ud310\ub9e4\ud558\uace0 \uc788\ub2e4. \uae30\uae30\uc758 \uae30\ub2a5\uc801 \uce21\uba74\uc5d0\uc11c \ud55c\uacc4\uc810\uc73c\ub85c\ub294 \uac00\ub3d9\uac70\ub9ac\ub97c \uaf3d\uc744 \uc218 \uc788\ub2e4. \ud55c \ubc88 \ucda9\uc804\uc744 \ud558\uba74 \ucd5c\ub300 25~30km\uae4c\uc9c0\ubc16\uc5d0 \uc6b4\ud589\uc744 \ud560 \uc218 \uc5c6\uae30 \ub54c\ubb38\uc5d0 \uc774 \ubd80\ubd84\uc5d0\uc11c\ub3c4 \ud55c\uacc4\uc810\uc744 \uc9c0\ub2c8\uace0 \uc788\ub2e4.", "paragraph_answer": "\ucd08\uae30 \uc138\uadf8\uc6e8\uc774\uc758 \uac00\uaca9\uc740 1000\ub9cc \uc6d0 \ub300\ub97c \ud6cc\uca4d \ub6f0\uc5b4\ub118\uc5c8\ub2e4. \uc77c\ubc18 \ub300\uc911\ub4e4\uc774 \ub808\uc838\uc6a9 \ud639\uc740 \uc77c\uc0c1\uc5d0\uc11c \uc0dd\ud65c\ud558\ub824\uace0 \uad6c\ub9e4\ud558\uae30\uc5d0\ub294 \ubd80\ub2f4\uc2a4\ub7f0\uc6b4 \uae08\uc561\uc774\uc5c8\ub2e4. 2015\ub144 \ud604\uc7ac\uc5d0 \uc774\ub974\uae30\uae4c\uc9c0 \uac00\uaca9\uc774 \uc810\uc810 \ud558\ub77d\ud558\uae30\ub294 \ud588\uc9c0\ub9cc \uc544\uc9c1\uae4c\uc9c0\ub294 \ub300\uc911\ub4e4\uc774 \ubc1b\uc544\ub4e4\uc774\uae30\uc5d0\ub294 \ube44\uc2fc \uac00\uaca9\uc744 \uc720\uc9c0\ud558\uace0 \uc788\ub294 \ud604\uc2e4\uc774\ub2e4. \uc774\uc5d0 \ub300\ud55c \ud0c0\uac1c\ucc45\uc73c\ub85c \uc138\uadf8\uc6e8\uc774\uc0ac\ub97c \uc778\uc218\ud55c \uc911\uad6d\uc758 \ub098\uc778\ubd07\uc0ac\uac00 2015\ub144 10\uc6d4 19\uc77c ' \ub098\uc778\ubd07 \ubbf8\ub2c8 '\uc774\ub77c\ub294 \uc138\uadf8\uc6e8\uc774\ub97c \uac1c\ub7c9\ud55c \uae30\uae30\ub97c \ubc1c\ub9e4\ud558\uc5ec \uc911\uad6d\ub0b4\uc218\uc2dc\uc7a5\uc5d0\uc11c \ud55c\ud654\ub85c \uc57d 35\ub9cc\uc6d0\uc758 \uac00\uaca9\uc5d0 \ud310\ub9e4\ud558\uace0 \uc788\ub2e4. \uae30\uae30\uc758 \uae30\ub2a5\uc801 \uce21\uba74\uc5d0\uc11c \ud55c\uacc4\uc810\uc73c\ub85c\ub294 \uac00\ub3d9\uac70\ub9ac\ub97c \uaf3d\uc744 \uc218 \uc788\ub2e4. \ud55c \ubc88 \ucda9\uc804\uc744 \ud558\uba74 \ucd5c\ub300 25~30km\uae4c\uc9c0\ubc16\uc5d0 \uc6b4\ud589\uc744 \ud560 \uc218 \uc5c6\uae30 \ub54c\ubb38\uc5d0 \uc774 \ubd80\ubd84\uc5d0\uc11c\ub3c4 \ud55c\uacc4\uc810\uc744 \uc9c0\ub2c8\uace0 \uc788\ub2e4.", "sentence_answer": "\uc774\uc5d0 \ub300\ud55c \ud0c0\uac1c\ucc45\uc73c\ub85c \uc138\uadf8\uc6e8\uc774\uc0ac\ub97c \uc778\uc218\ud55c \uc911\uad6d\uc758 \ub098\uc778\ubd07\uc0ac\uac00 2015\ub144 10\uc6d4 19\uc77c ' \ub098\uc778\ubd07 \ubbf8\ub2c8 '\uc774\ub77c\ub294 \uc138\uadf8\uc6e8\uc774\ub97c \uac1c\ub7c9\ud55c \uae30\uae30\ub97c \ubc1c\ub9e4\ud558\uc5ec \uc911\uad6d\ub0b4\uc218\uc2dc\uc7a5\uc5d0\uc11c \ud55c\ud654\ub85c \uc57d 35\ub9cc\uc6d0\uc758 \uac00\uaca9\uc5d0 \ud310\ub9e4\ud558\uace0 \uc788\ub2e4.", "paragraph_id": "6517967-4-0", "paragraph_question": "question: 2015\ub144 \uc138\uadf8\uc6e8\uc774\ub97c \uac1c\ub7c9\ud55c \uc911\uad6d \ub098\uc778\ubd07\uc0ac\uc758 \uc81c\ud488\uba85\uc740 \ubb34\uc5c7\uc778\uac00?, context: \ucd08\uae30 \uc138\uadf8\uc6e8\uc774\uc758 \uac00\uaca9\uc740 1000\ub9cc \uc6d0 \ub300\ub97c \ud6cc\uca4d \ub6f0\uc5b4\ub118\uc5c8\ub2e4. \uc77c\ubc18 \ub300\uc911\ub4e4\uc774 \ub808\uc838\uc6a9 \ud639\uc740 \uc77c\uc0c1\uc5d0\uc11c \uc0dd\ud65c\ud558\ub824\uace0 \uad6c\ub9e4\ud558\uae30\uc5d0\ub294 \ubd80\ub2f4\uc2a4\ub7f0\uc6b4 \uae08\uc561\uc774\uc5c8\ub2e4. 2015\ub144 \ud604\uc7ac\uc5d0 \uc774\ub974\uae30\uae4c\uc9c0 \uac00\uaca9\uc774 \uc810\uc810 \ud558\ub77d\ud558\uae30\ub294 \ud588\uc9c0\ub9cc \uc544\uc9c1\uae4c\uc9c0\ub294 \ub300\uc911\ub4e4\uc774 \ubc1b\uc544\ub4e4\uc774\uae30\uc5d0\ub294 \ube44\uc2fc \uac00\uaca9\uc744 \uc720\uc9c0\ud558\uace0 \uc788\ub294 \ud604\uc2e4\uc774\ub2e4. \uc774\uc5d0 \ub300\ud55c \ud0c0\uac1c\ucc45\uc73c\ub85c \uc138\uadf8\uc6e8\uc774\uc0ac\ub97c \uc778\uc218\ud55c \uc911\uad6d\uc758 \ub098\uc778\ubd07\uc0ac\uac00 2015\ub144 10\uc6d4 19\uc77c '\ub098\uc778\ubd07 \ubbf8\ub2c8'\uc774\ub77c\ub294 \uc138\uadf8\uc6e8\uc774\ub97c \uac1c\ub7c9\ud55c \uae30\uae30\ub97c \ubc1c\ub9e4\ud558\uc5ec \uc911\uad6d\ub0b4\uc218\uc2dc\uc7a5\uc5d0\uc11c \ud55c\ud654\ub85c \uc57d 35\ub9cc\uc6d0\uc758 \uac00\uaca9\uc5d0 \ud310\ub9e4\ud558\uace0 \uc788\ub2e4. \uae30\uae30\uc758 \uae30\ub2a5\uc801 \uce21\uba74\uc5d0\uc11c \ud55c\uacc4\uc810\uc73c\ub85c\ub294 \uac00\ub3d9\uac70\ub9ac\ub97c \uaf3d\uc744 \uc218 \uc788\ub2e4. \ud55c \ubc88 \ucda9\uc804\uc744 \ud558\uba74 \ucd5c\ub300 25~30km\uae4c\uc9c0\ubc16\uc5d0 \uc6b4\ud589\uc744 \ud560 \uc218 \uc5c6\uae30 \ub54c\ubb38\uc5d0 \uc774 \ubd80\ubd84\uc5d0\uc11c\ub3c4 \ud55c\uacc4\uc810\uc744 \uc9c0\ub2c8\uace0 \uc788\ub2e4."} {"question": "\ud544\ub85c\ud3ec\uc774\uba58\uc740 \uc544\uce74\uc774\uc544 \ub3d9\ub9f9\uc758 \ubb34\uc5c7\uc73c\ub85c \uc784\uba85\ub418\uc5c8\ub098?", "paragraph": "\uae30\uc6d0\uc804 221\ub144, \ud544\ub85c\ud3ec\uc774\uba58\uc740 \uc804\uc7c1 \uc911\uc778 \ud06c\ub808\ud0c0\uc5d0\uc11c \uc6a9\ubcd1 \ub300\uc7a5\uc73c\ub85c \uc2f8\uc6e0\ub2e4. 10\ub144 \ud6c4 \uadc0\uad6d\uc5d0 \uc988\uc74c\ud558\uc5ec \uadf8\ub294 \uc544\uce74\uc774\uc544 \ub3d9\ub9f9\uc758 \uae30\ubcd1\ub300 \uc9c0\ud718\uad00\uc73c\ub85c \uc784\uba85\ub418\uc5c8\ub2e4. \ub2f9\uc2dc \uc544\uce74\uc774\uc544\uc758 \uae30\ubcd1\ub4e4\uc740 \uc57d\ud588\uae30 \ub54c\ubb38\uc5d0 \uadf8\ub294 \uae30\ubcd1\uc744 \uc6a9\ub9f9\ud558\uac8c \ub2e8\ub828\ud588\ub2e4. \uac19\uc740 \ud574\uc5d0 \uc81c1\ucc28 \ub9c8\ucf00\ub3c4\ub2c8\uc544 \uc804\uc7c1\uacfc \uc5f0\uad00\ub41c \uc804\ud22c\uc5d0\uc11c \ud544\ub85c\ud3ec\uc774\uba58\uc774 \uc774\ub044\ub294 \uc544\uce74\uc774\uc544 \uad70\uc740 \uc5d8\ub9ac\uc2a4\uc758 \uacbd\uacc4\uc5d0 \uc788\ub294 \ub77c\ub9ac\uc0ac \uac15\uc5d0\uc11c \uc544\uc774\ud1a8\ub9ac\uc544, \uc5d8\ub9ac\uc2a4 \uc5f0\ud569\uad70\uacfc \uc2f8\uc6cc \ubb3c\ub9ac\uce58\uace0, \uc801\uc7a5 \ub2e4\ubaa8\ud310\ud22c\uc2a4\uc640 \uc9c1\uc811 \ub9de\uc11c\uc11c \ubb3c\ub9ac\uce58\uace0 \uc2b9\ub9ac\ub97c \uac70\ub450\uc5c8\ub2e4. \uc774 \uc2b9\ub9ac\ub85c \ud544\ub85c\ud3ec\uc774\uba58\uc758 \uba85\uc131\uc740 \uadf8\ub9ac\uc2a4\uc5d0 \ub110\ub9ac \ud37c\uc84c\uc73c\uba70, \ub610\ud55c \uadf8\ub294 \uc544\uce74\uc774\uc544 \ub3d9\ub9f9\uc758 \ubcf4\ubcd1 \uc7a5\ube44\ub97c \uac1c\ud601\ud588\ub2e4. \uadf8\ub54c\uae4c\uc9c0 \uc544\uce74\uc774\uc544 \uad70\uc758 \uc7a5\ube44\ub294 \uc9e7\uc740 \ucc3d\uacfc \uac00\ubccd\uac8c \uc587\uc740 \ud0c0\uc6d0\ud615 \ubc29\ud328\uc600\uc73c\uba70, \uc774\uac83\uc740 \uc6d0\uac70\ub9ac \uc804\ud22c\uc5d0\uc11c\ub294 \uc720\ub9ac\ud558\uc9c0\ub9cc, \ubc31\ubcd1\uc804\uc5d0\uc11c\ub294 \ubd88\ub9ac\ud558\uc5ec \uc27d\uac8c \uc804\uc5f4\uc774 \ub3cc\ud30c\ub418\uace0 \uc788\uc5c8\ub2e4. \uadf8\ub294 \ud314\ub791\ud06c\uc2a4\uc758 \ubc00\uc9d1\ub300\ud615\uc744 \ud615\uc131\ud560 \ud615\ud3b8\uc774 \uc88b\uc740 \uc7a5\ucc3d\uacfc \ub465\uadfc\ubc29\ud328\ub85c \ubc14\uafb8\uc5c8\ub2e4.", "answer": "\uae30\ubcd1\ub300 \uc9c0\ud718\uad00", "sentence": "10\ub144 \ud6c4 \uadc0\uad6d\uc5d0 \uc988\uc74c\ud558\uc5ec \uadf8\ub294 \uc544\uce74\uc774\uc544 \ub3d9\ub9f9\uc758 \uae30\ubcd1\ub300 \uc9c0\ud718\uad00 \uc73c\ub85c \uc784\uba85\ub418\uc5c8\ub2e4.", "paragraph_sentence": "\uae30\uc6d0\uc804 221\ub144, \ud544\ub85c\ud3ec\uc774\uba58\uc740 \uc804\uc7c1 \uc911\uc778 \ud06c\ub808\ud0c0\uc5d0\uc11c \uc6a9\ubcd1 \ub300\uc7a5\uc73c\ub85c \uc2f8\uc6e0\ub2e4. 10\ub144 \ud6c4 \uadc0\uad6d\uc5d0 \uc988\uc74c\ud558\uc5ec \uadf8\ub294 \uc544\uce74\uc774\uc544 \ub3d9\ub9f9\uc758 \uae30\ubcd1\ub300 \uc9c0\ud718\uad00 \uc73c\ub85c \uc784\uba85\ub418\uc5c8\ub2e4. \ub2f9\uc2dc \uc544\uce74\uc774\uc544\uc758 \uae30\ubcd1\ub4e4\uc740 \uc57d\ud588\uae30 \ub54c\ubb38\uc5d0 \uadf8\ub294 \uae30\ubcd1\uc744 \uc6a9\ub9f9\ud558\uac8c \ub2e8\ub828\ud588\ub2e4. \uac19\uc740 \ud574\uc5d0 \uc81c1\ucc28 \ub9c8\ucf00\ub3c4\ub2c8\uc544 \uc804\uc7c1\uacfc \uc5f0\uad00\ub41c \uc804\ud22c\uc5d0\uc11c \ud544\ub85c\ud3ec\uc774\uba58\uc774 \uc774\ub044\ub294 \uc544\uce74\uc774\uc544 \uad70\uc740 \uc5d8\ub9ac\uc2a4\uc758 \uacbd\uacc4\uc5d0 \uc788\ub294 \ub77c\ub9ac\uc0ac \uac15\uc5d0\uc11c \uc544\uc774\ud1a8\ub9ac\uc544, \uc5d8\ub9ac\uc2a4 \uc5f0\ud569\uad70\uacfc \uc2f8\uc6cc \ubb3c\ub9ac\uce58\uace0, \uc801\uc7a5 \ub2e4\ubaa8\ud310\ud22c\uc2a4\uc640 \uc9c1\uc811 \ub9de\uc11c\uc11c \ubb3c\ub9ac\uce58\uace0 \uc2b9\ub9ac\ub97c \uac70\ub450\uc5c8\ub2e4. \uc774 \uc2b9\ub9ac\ub85c \ud544\ub85c\ud3ec\uc774\uba58\uc758 \uba85\uc131\uc740 \uadf8\ub9ac\uc2a4\uc5d0 \ub110\ub9ac \ud37c\uc84c\uc73c\uba70, \ub610\ud55c \uadf8\ub294 \uc544\uce74\uc774\uc544 \ub3d9\ub9f9\uc758 \ubcf4\ubcd1 \uc7a5\ube44\ub97c \uac1c\ud601\ud588\ub2e4. \uadf8\ub54c\uae4c\uc9c0 \uc544\uce74\uc774\uc544 \uad70\uc758 \uc7a5\ube44\ub294 \uc9e7\uc740 \ucc3d\uacfc \uac00\ubccd\uac8c \uc587\uc740 \ud0c0\uc6d0\ud615 \ubc29\ud328\uc600\uc73c\uba70, \uc774\uac83\uc740 \uc6d0\uac70\ub9ac \uc804\ud22c\uc5d0\uc11c\ub294 \uc720\ub9ac\ud558\uc9c0\ub9cc, \ubc31\ubcd1\uc804\uc5d0\uc11c\ub294 \ubd88\ub9ac\ud558\uc5ec \uc27d\uac8c \uc804\uc5f4\uc774 \ub3cc\ud30c\ub418\uace0 \uc788\uc5c8\ub2e4. \uadf8\ub294 \ud314\ub791\ud06c\uc2a4\uc758 \ubc00\uc9d1\ub300\ud615\uc744 \ud615\uc131\ud560 \ud615\ud3b8\uc774 \uc88b\uc740 \uc7a5\ucc3d\uacfc \ub465\uadfc\ubc29\ud328\ub85c \ubc14\uafb8\uc5c8\ub2e4.", "paragraph_answer": "\uae30\uc6d0\uc804 221\ub144, \ud544\ub85c\ud3ec\uc774\uba58\uc740 \uc804\uc7c1 \uc911\uc778 \ud06c\ub808\ud0c0\uc5d0\uc11c \uc6a9\ubcd1 \ub300\uc7a5\uc73c\ub85c \uc2f8\uc6e0\ub2e4. 10\ub144 \ud6c4 \uadc0\uad6d\uc5d0 \uc988\uc74c\ud558\uc5ec \uadf8\ub294 \uc544\uce74\uc774\uc544 \ub3d9\ub9f9\uc758 \uae30\ubcd1\ub300 \uc9c0\ud718\uad00 \uc73c\ub85c \uc784\uba85\ub418\uc5c8\ub2e4. \ub2f9\uc2dc \uc544\uce74\uc774\uc544\uc758 \uae30\ubcd1\ub4e4\uc740 \uc57d\ud588\uae30 \ub54c\ubb38\uc5d0 \uadf8\ub294 \uae30\ubcd1\uc744 \uc6a9\ub9f9\ud558\uac8c \ub2e8\ub828\ud588\ub2e4. \uac19\uc740 \ud574\uc5d0 \uc81c1\ucc28 \ub9c8\ucf00\ub3c4\ub2c8\uc544 \uc804\uc7c1\uacfc \uc5f0\uad00\ub41c \uc804\ud22c\uc5d0\uc11c \ud544\ub85c\ud3ec\uc774\uba58\uc774 \uc774\ub044\ub294 \uc544\uce74\uc774\uc544 \uad70\uc740 \uc5d8\ub9ac\uc2a4\uc758 \uacbd\uacc4\uc5d0 \uc788\ub294 \ub77c\ub9ac\uc0ac \uac15\uc5d0\uc11c \uc544\uc774\ud1a8\ub9ac\uc544, \uc5d8\ub9ac\uc2a4 \uc5f0\ud569\uad70\uacfc \uc2f8\uc6cc \ubb3c\ub9ac\uce58\uace0, \uc801\uc7a5 \ub2e4\ubaa8\ud310\ud22c\uc2a4\uc640 \uc9c1\uc811 \ub9de\uc11c\uc11c \ubb3c\ub9ac\uce58\uace0 \uc2b9\ub9ac\ub97c \uac70\ub450\uc5c8\ub2e4. \uc774 \uc2b9\ub9ac\ub85c \ud544\ub85c\ud3ec\uc774\uba58\uc758 \uba85\uc131\uc740 \uadf8\ub9ac\uc2a4\uc5d0 \ub110\ub9ac \ud37c\uc84c\uc73c\uba70, \ub610\ud55c \uadf8\ub294 \uc544\uce74\uc774\uc544 \ub3d9\ub9f9\uc758 \ubcf4\ubcd1 \uc7a5\ube44\ub97c \uac1c\ud601\ud588\ub2e4. \uadf8\ub54c\uae4c\uc9c0 \uc544\uce74\uc774\uc544 \uad70\uc758 \uc7a5\ube44\ub294 \uc9e7\uc740 \ucc3d\uacfc \uac00\ubccd\uac8c \uc587\uc740 \ud0c0\uc6d0\ud615 \ubc29\ud328\uc600\uc73c\uba70, \uc774\uac83\uc740 \uc6d0\uac70\ub9ac \uc804\ud22c\uc5d0\uc11c\ub294 \uc720\ub9ac\ud558\uc9c0\ub9cc, \ubc31\ubcd1\uc804\uc5d0\uc11c\ub294 \ubd88\ub9ac\ud558\uc5ec \uc27d\uac8c \uc804\uc5f4\uc774 \ub3cc\ud30c\ub418\uace0 \uc788\uc5c8\ub2e4. \uadf8\ub294 \ud314\ub791\ud06c\uc2a4\uc758 \ubc00\uc9d1\ub300\ud615\uc744 \ud615\uc131\ud560 \ud615\ud3b8\uc774 \uc88b\uc740 \uc7a5\ucc3d\uacfc \ub465\uadfc\ubc29\ud328\ub85c \ubc14\uafb8\uc5c8\ub2e4.", "sentence_answer": "10\ub144 \ud6c4 \uadc0\uad6d\uc5d0 \uc988\uc74c\ud558\uc5ec \uadf8\ub294 \uc544\uce74\uc774\uc544 \ub3d9\ub9f9\uc758 \uae30\ubcd1\ub300 \uc9c0\ud718\uad00 \uc73c\ub85c \uc784\uba85\ub418\uc5c8\ub2e4.", "paragraph_id": "6569448-3-0", "paragraph_question": "question: \ud544\ub85c\ud3ec\uc774\uba58\uc740 \uc544\uce74\uc774\uc544 \ub3d9\ub9f9\uc758 \ubb34\uc5c7\uc73c\ub85c \uc784\uba85\ub418\uc5c8\ub098?, context: \uae30\uc6d0\uc804 221\ub144, \ud544\ub85c\ud3ec\uc774\uba58\uc740 \uc804\uc7c1 \uc911\uc778 \ud06c\ub808\ud0c0\uc5d0\uc11c \uc6a9\ubcd1 \ub300\uc7a5\uc73c\ub85c \uc2f8\uc6e0\ub2e4. 10\ub144 \ud6c4 \uadc0\uad6d\uc5d0 \uc988\uc74c\ud558\uc5ec \uadf8\ub294 \uc544\uce74\uc774\uc544 \ub3d9\ub9f9\uc758 \uae30\ubcd1\ub300 \uc9c0\ud718\uad00\uc73c\ub85c \uc784\uba85\ub418\uc5c8\ub2e4. \ub2f9\uc2dc \uc544\uce74\uc774\uc544\uc758 \uae30\ubcd1\ub4e4\uc740 \uc57d\ud588\uae30 \ub54c\ubb38\uc5d0 \uadf8\ub294 \uae30\ubcd1\uc744 \uc6a9\ub9f9\ud558\uac8c \ub2e8\ub828\ud588\ub2e4. \uac19\uc740 \ud574\uc5d0 \uc81c1\ucc28 \ub9c8\ucf00\ub3c4\ub2c8\uc544 \uc804\uc7c1\uacfc \uc5f0\uad00\ub41c \uc804\ud22c\uc5d0\uc11c \ud544\ub85c\ud3ec\uc774\uba58\uc774 \uc774\ub044\ub294 \uc544\uce74\uc774\uc544 \uad70\uc740 \uc5d8\ub9ac\uc2a4\uc758 \uacbd\uacc4\uc5d0 \uc788\ub294 \ub77c\ub9ac\uc0ac \uac15\uc5d0\uc11c \uc544\uc774\ud1a8\ub9ac\uc544, \uc5d8\ub9ac\uc2a4 \uc5f0\ud569\uad70\uacfc \uc2f8\uc6cc \ubb3c\ub9ac\uce58\uace0, \uc801\uc7a5 \ub2e4\ubaa8\ud310\ud22c\uc2a4\uc640 \uc9c1\uc811 \ub9de\uc11c\uc11c \ubb3c\ub9ac\uce58\uace0 \uc2b9\ub9ac\ub97c \uac70\ub450\uc5c8\ub2e4. \uc774 \uc2b9\ub9ac\ub85c \ud544\ub85c\ud3ec\uc774\uba58\uc758 \uba85\uc131\uc740 \uadf8\ub9ac\uc2a4\uc5d0 \ub110\ub9ac \ud37c\uc84c\uc73c\uba70, \ub610\ud55c \uadf8\ub294 \uc544\uce74\uc774\uc544 \ub3d9\ub9f9\uc758 \ubcf4\ubcd1 \uc7a5\ube44\ub97c \uac1c\ud601\ud588\ub2e4. \uadf8\ub54c\uae4c\uc9c0 \uc544\uce74\uc774\uc544 \uad70\uc758 \uc7a5\ube44\ub294 \uc9e7\uc740 \ucc3d\uacfc \uac00\ubccd\uac8c \uc587\uc740 \ud0c0\uc6d0\ud615 \ubc29\ud328\uc600\uc73c\uba70, \uc774\uac83\uc740 \uc6d0\uac70\ub9ac \uc804\ud22c\uc5d0\uc11c\ub294 \uc720\ub9ac\ud558\uc9c0\ub9cc, \ubc31\ubcd1\uc804\uc5d0\uc11c\ub294 \ubd88\ub9ac\ud558\uc5ec \uc27d\uac8c \uc804\uc5f4\uc774 \ub3cc\ud30c\ub418\uace0 \uc788\uc5c8\ub2e4. \uadf8\ub294 \ud314\ub791\ud06c\uc2a4\uc758 \ubc00\uc9d1\ub300\ud615\uc744 \ud615\uc131\ud560 \ud615\ud3b8\uc774 \uc88b\uc740 \uc7a5\ucc3d\uacfc \ub465\uadfc\ubc29\ud328\ub85c \ubc14\uafb8\uc5c8\ub2e4."} {"question": "\uc774\ub780\uc740 \uc774\ub77c\ud06c\uc5d0\uc11c \ubbf8\uad6d, \uc774\ub77c\ud06c \uc815\ubd80\uc640 \uc5f0\ud569\ud574 \ub204\uad6c\uc640 \ub9df\uc11c \uc2f8\uc6b0\ub294\uac00", "paragraph": "\uc774 \ub3d9\ub9f9\uc758 \ub610\ub2e4\ub978 \uc8fc\uc694 \ub3d9\ub9f9 \ub2e8\uccb4\ub294 \ub808\ubc14\ub17c\uacfc \uadf8 \uc9c0\uc5ed\uc758 \ubb34\uc7a5 \ub2e8\uccb4\uc774\uc790 \uc815\ub2f9\uc778 \ud5e4\uc988\ubcfc\ub77c\uc774\uc9c0\ub9cc, 1979\ub144 \uc774\ub780 \ud601\uba85 \uc774\ud6c4 \uc774\ub780\uc758 \uc9c0\uc18d\uc801\uc778 \ub3d9\ub9f9\uad6d\uc740 \uc2dc\ub9ac\uc544\uac00 \uc720\uc77c\ud588\ub2e4. \uc774\ub780\uc740 \uc218\uc1a1, \uae30\uc220, \uae08\uc735 \uc9c0\uc6d0\uc744 \ud3ec\ud568\ud574 \uc2dc\ub9ac\uc544 \ub0b4\uc804\uc5d0\uc11c \uc2dc\ub9ac\uc544 \uc815\ubd80\ub97c \uc801\uadf9\uc801\uc73c\ub85c \uc9c0\uc6d0\ud588\ub2e4. 2014\ub144 4\uc6d4, \uc774\ub780 \uc678\uad50\ubd80 \ucc28\uad00\uc778 \ud638\uc138\uc778 \uc544\ubbf8\ub974\uc555\ub3cc\ub77c\ud78c\uc740 \"\uc6b0\ub9ac\ub294 \ubc14\uc0ac\ub974 \uc544\uc0ac\ub4dc \ub300\ud1b5\ub839\uc774 \ubaa9\uc228\uc744 \ubd80\uc9c0\ud558\ub294 \uac83\uc744 \uc6d0\ud558\uc9c0 \uc54a\ub294\ub2e4. \ud558\uc9c0\ub9cc \uc6b0\ub9ac\ub294 \ud14c\ub7ec\ub098 \uadf9\ub2e8\uc8fc\uc758\uc790\ub4e4\uc774 \uc544\uc0ac\ub4dc\uc640 \uc2dc\ub9ac\uc544 \uc815\ubd80\ub97c \ub9dd\uce58\ub294 \uac83\uc744 \ubcf4\uace0 \uc788\uc744 \uc218\ub9cc\uc740 \uc5c6\ub2e4\"\ub77c\uace0 \ub9d0\ud588\ub2e4. 7\uc6d4 24\uc77c, \ucfe0\ub4dc\uc2a4\uad70 \uc0ac\ub839\uad00 \uce74\uc82c \uc194\ub77c\uc774\ub9c8\ub2c8\uac00 \ubaa8\uc2a4\ud06c\ubc14\ub97c \ubc29\ubb38\ud588\uace0, \uc2dc\ub9ac\uc544\uc5d0\uc11c\uc758 \uc591\uad6d \uac04 \ud611\uc758 \uacc4\ud68d\uc774 \ub9c8\ub828\ub418\uc5c8\ub2e4. 2\uac1c\uc758 \ub808\ubc14\ub17c \uc790\ub8cc\uc5d0 \ub530\ub974\uba74, \ub9ac\uc6b0\ud130\ub294 2015\ub144 10\uc6d4 1\uc77c\uc5d0 \uc218\ubc31 \uba85\uc758 \uc774\ub780 \uad70\ub300\uac00 10\uc77c \uc804\ubd80\ud130 \uc2dc\ub9ac\uc544\uc5d0 \ub3c4\ucc29\ud558\uc5ec \uc2dc\ub9ac\uc544 \uc815\ubd80\uad70\uacfc \uace7 \uc5f0\ud569\ud560 \uc900\ube44\ub97c \ub9c8\ucce4\ub2e4\uace0 \ubcf4\uace0\ud588\ub2e4. \uadf8\ub4e4\uc758 \ub808\ubc14\ub17c \ub3d9\ub9f9 \ub2e8\uccb4\uc778 \ud5e4\uc988\ubcfc\ub77c \ub610\ud55c \ub7ec\uc2dc\uc544 \uacf5\uc2b5\uc758 \uc9c0\uc6d0 \ud558\uc5d0 \uc8fc\uc694 \uc721\uc0c1 \uacf5\uc138\ub97c \ud3bc\uce60 \uac83\uc774\ub77c\uace0 \ubcf4\uc558\ub2e4. \uc774\ub780\uc740 \ud604\uc7ac \uc774\ub77c\ud06c\uc5d0\uc11c\ub3c4 \ubbf8\uad6d \ubc0f \uc774\ub77c\ud06c \uc815\ubd80\uc640 \uc5f0\ud569\ud574 ISIS\uc5d0 \ub9de\uc11c \uc2f8\uc6b0\uace0 \uc788\ub2e4.", "answer": "ISIS", "sentence": "\uc774\ub780\uc740 \ud604\uc7ac \uc774\ub77c\ud06c\uc5d0\uc11c\ub3c4 \ubbf8\uad6d \ubc0f \uc774\ub77c\ud06c \uc815\ubd80\uc640 \uc5f0\ud569\ud574 ISIS \uc5d0", "paragraph_sentence": "\uc774 \ub3d9\ub9f9\uc758 \ub610\ub2e4\ub978 \uc8fc\uc694 \ub3d9\ub9f9 \ub2e8\uccb4\ub294 \ub808\ubc14\ub17c\uacfc \uadf8 \uc9c0\uc5ed\uc758 \ubb34\uc7a5 \ub2e8\uccb4\uc774\uc790 \uc815\ub2f9\uc778 \ud5e4\uc988\ubcfc\ub77c\uc774\uc9c0\ub9cc, 1979\ub144 \uc774\ub780 \ud601\uba85 \uc774\ud6c4 \uc774\ub780\uc758 \uc9c0\uc18d\uc801\uc778 \ub3d9\ub9f9\uad6d\uc740 \uc2dc\ub9ac\uc544\uac00 \uc720\uc77c\ud588\ub2e4. \uc774\ub780\uc740 \uc218\uc1a1, \uae30\uc220, \uae08\uc735 \uc9c0\uc6d0\uc744 \ud3ec\ud568\ud574 \uc2dc\ub9ac\uc544 \ub0b4\uc804\uc5d0\uc11c \uc2dc\ub9ac\uc544 \uc815\ubd80\ub97c \uc801\uadf9\uc801\uc73c\ub85c \uc9c0\uc6d0\ud588\ub2e4. 2014\ub144 4\uc6d4, \uc774\ub780 \uc678\uad50\ubd80 \ucc28\uad00\uc778 \ud638\uc138\uc778 \uc544\ubbf8\ub974\uc555\ub3cc\ub77c\ud78c\uc740 \"\uc6b0\ub9ac\ub294 \ubc14\uc0ac\ub974 \uc544\uc0ac\ub4dc \ub300\ud1b5\ub839\uc774 \ubaa9\uc228\uc744 \ubd80\uc9c0\ud558\ub294 \uac83\uc744 \uc6d0\ud558\uc9c0 \uc54a\ub294\ub2e4. \ud558\uc9c0\ub9cc \uc6b0\ub9ac\ub294 \ud14c\ub7ec\ub098 \uadf9\ub2e8\uc8fc\uc758\uc790\ub4e4\uc774 \uc544\uc0ac\ub4dc\uc640 \uc2dc\ub9ac\uc544 \uc815\ubd80\ub97c \ub9dd\uce58\ub294 \uac83\uc744 \ubcf4\uace0 \uc788\uc744 \uc218\ub9cc\uc740 \uc5c6\ub2e4\"\ub77c\uace0 \ub9d0\ud588\ub2e4. 7\uc6d4 24\uc77c, \ucfe0\ub4dc\uc2a4\uad70 \uc0ac\ub839\uad00 \uce74\uc82c \uc194\ub77c\uc774\ub9c8\ub2c8\uac00 \ubaa8\uc2a4\ud06c\ubc14\ub97c \ubc29\ubb38\ud588\uace0, \uc2dc\ub9ac\uc544\uc5d0\uc11c\uc758 \uc591\uad6d \uac04 \ud611\uc758 \uacc4\ud68d\uc774 \ub9c8\ub828\ub418\uc5c8\ub2e4. 2\uac1c\uc758 \ub808\ubc14\ub17c \uc790\ub8cc\uc5d0 \ub530\ub974\uba74, \ub9ac\uc6b0\ud130\ub294 2015\ub144 10\uc6d4 1\uc77c\uc5d0 \uc218\ubc31 \uba85\uc758 \uc774\ub780 \uad70\ub300\uac00 10\uc77c \uc804\ubd80\ud130 \uc2dc\ub9ac\uc544\uc5d0 \ub3c4\ucc29\ud558\uc5ec \uc2dc\ub9ac\uc544 \uc815\ubd80\uad70\uacfc \uace7 \uc5f0\ud569\ud560 \uc900\ube44\ub97c \ub9c8\ucce4\ub2e4\uace0 \ubcf4\uace0\ud588\ub2e4. \uadf8\ub4e4\uc758 \ub808\ubc14\ub17c \ub3d9\ub9f9 \ub2e8\uccb4\uc778 \ud5e4\uc988\ubcfc\ub77c \ub610\ud55c \ub7ec\uc2dc\uc544 \uacf5\uc2b5\uc758 \uc9c0\uc6d0 \ud558\uc5d0 \uc8fc\uc694 \uc721\uc0c1 \uacf5\uc138\ub97c \ud3bc\uce60 \uac83\uc774\ub77c\uace0 \ubcf4\uc558\ub2e4. \uc774\ub780\uc740 \ud604\uc7ac \uc774\ub77c\ud06c\uc5d0\uc11c\ub3c4 \ubbf8\uad6d \ubc0f \uc774\ub77c\ud06c \uc815\ubd80\uc640 \uc5f0\ud569\ud574 ISIS \uc5d0 \ub9de\uc11c \uc2f8\uc6b0\uace0 \uc788\ub2e4.", "paragraph_answer": "\uc774 \ub3d9\ub9f9\uc758 \ub610\ub2e4\ub978 \uc8fc\uc694 \ub3d9\ub9f9 \ub2e8\uccb4\ub294 \ub808\ubc14\ub17c\uacfc \uadf8 \uc9c0\uc5ed\uc758 \ubb34\uc7a5 \ub2e8\uccb4\uc774\uc790 \uc815\ub2f9\uc778 \ud5e4\uc988\ubcfc\ub77c\uc774\uc9c0\ub9cc, 1979\ub144 \uc774\ub780 \ud601\uba85 \uc774\ud6c4 \uc774\ub780\uc758 \uc9c0\uc18d\uc801\uc778 \ub3d9\ub9f9\uad6d\uc740 \uc2dc\ub9ac\uc544\uac00 \uc720\uc77c\ud588\ub2e4. \uc774\ub780\uc740 \uc218\uc1a1, \uae30\uc220, \uae08\uc735 \uc9c0\uc6d0\uc744 \ud3ec\ud568\ud574 \uc2dc\ub9ac\uc544 \ub0b4\uc804\uc5d0\uc11c \uc2dc\ub9ac\uc544 \uc815\ubd80\ub97c \uc801\uadf9\uc801\uc73c\ub85c \uc9c0\uc6d0\ud588\ub2e4. 2014\ub144 4\uc6d4, \uc774\ub780 \uc678\uad50\ubd80 \ucc28\uad00\uc778 \ud638\uc138\uc778 \uc544\ubbf8\ub974\uc555\ub3cc\ub77c\ud78c\uc740 \"\uc6b0\ub9ac\ub294 \ubc14\uc0ac\ub974 \uc544\uc0ac\ub4dc \ub300\ud1b5\ub839\uc774 \ubaa9\uc228\uc744 \ubd80\uc9c0\ud558\ub294 \uac83\uc744 \uc6d0\ud558\uc9c0 \uc54a\ub294\ub2e4. \ud558\uc9c0\ub9cc \uc6b0\ub9ac\ub294 \ud14c\ub7ec\ub098 \uadf9\ub2e8\uc8fc\uc758\uc790\ub4e4\uc774 \uc544\uc0ac\ub4dc\uc640 \uc2dc\ub9ac\uc544 \uc815\ubd80\ub97c \ub9dd\uce58\ub294 \uac83\uc744 \ubcf4\uace0 \uc788\uc744 \uc218\ub9cc\uc740 \uc5c6\ub2e4\"\ub77c\uace0 \ub9d0\ud588\ub2e4. 7\uc6d4 24\uc77c, \ucfe0\ub4dc\uc2a4\uad70 \uc0ac\ub839\uad00 \uce74\uc82c \uc194\ub77c\uc774\ub9c8\ub2c8\uac00 \ubaa8\uc2a4\ud06c\ubc14\ub97c \ubc29\ubb38\ud588\uace0, \uc2dc\ub9ac\uc544\uc5d0\uc11c\uc758 \uc591\uad6d \uac04 \ud611\uc758 \uacc4\ud68d\uc774 \ub9c8\ub828\ub418\uc5c8\ub2e4. 2\uac1c\uc758 \ub808\ubc14\ub17c \uc790\ub8cc\uc5d0 \ub530\ub974\uba74, \ub9ac\uc6b0\ud130\ub294 2015\ub144 10\uc6d4 1\uc77c\uc5d0 \uc218\ubc31 \uba85\uc758 \uc774\ub780 \uad70\ub300\uac00 10\uc77c \uc804\ubd80\ud130 \uc2dc\ub9ac\uc544\uc5d0 \ub3c4\ucc29\ud558\uc5ec \uc2dc\ub9ac\uc544 \uc815\ubd80\uad70\uacfc \uace7 \uc5f0\ud569\ud560 \uc900\ube44\ub97c \ub9c8\ucce4\ub2e4\uace0 \ubcf4\uace0\ud588\ub2e4. \uadf8\ub4e4\uc758 \ub808\ubc14\ub17c \ub3d9\ub9f9 \ub2e8\uccb4\uc778 \ud5e4\uc988\ubcfc\ub77c \ub610\ud55c \ub7ec\uc2dc\uc544 \uacf5\uc2b5\uc758 \uc9c0\uc6d0 \ud558\uc5d0 \uc8fc\uc694 \uc721\uc0c1 \uacf5\uc138\ub97c \ud3bc\uce60 \uac83\uc774\ub77c\uace0 \ubcf4\uc558\ub2e4. \uc774\ub780\uc740 \ud604\uc7ac \uc774\ub77c\ud06c\uc5d0\uc11c\ub3c4 \ubbf8\uad6d \ubc0f \uc774\ub77c\ud06c \uc815\ubd80\uc640 \uc5f0\ud569\ud574 ISIS \uc5d0 \ub9de\uc11c \uc2f8\uc6b0\uace0 \uc788\ub2e4.", "sentence_answer": "\uc774\ub780\uc740 \ud604\uc7ac \uc774\ub77c\ud06c\uc5d0\uc11c\ub3c4 \ubbf8\uad6d \ubc0f \uc774\ub77c\ud06c \uc815\ubd80\uc640 \uc5f0\ud569\ud574 ISIS \uc5d0", "paragraph_id": "6656963-4-1", "paragraph_question": "question: \uc774\ub780\uc740 \uc774\ub77c\ud06c\uc5d0\uc11c \ubbf8\uad6d, \uc774\ub77c\ud06c \uc815\ubd80\uc640 \uc5f0\ud569\ud574 \ub204\uad6c\uc640 \ub9df\uc11c \uc2f8\uc6b0\ub294\uac00, context: \uc774 \ub3d9\ub9f9\uc758 \ub610\ub2e4\ub978 \uc8fc\uc694 \ub3d9\ub9f9 \ub2e8\uccb4\ub294 \ub808\ubc14\ub17c\uacfc \uadf8 \uc9c0\uc5ed\uc758 \ubb34\uc7a5 \ub2e8\uccb4\uc774\uc790 \uc815\ub2f9\uc778 \ud5e4\uc988\ubcfc\ub77c\uc774\uc9c0\ub9cc, 1979\ub144 \uc774\ub780 \ud601\uba85 \uc774\ud6c4 \uc774\ub780\uc758 \uc9c0\uc18d\uc801\uc778 \ub3d9\ub9f9\uad6d\uc740 \uc2dc\ub9ac\uc544\uac00 \uc720\uc77c\ud588\ub2e4. \uc774\ub780\uc740 \uc218\uc1a1, \uae30\uc220, \uae08\uc735 \uc9c0\uc6d0\uc744 \ud3ec\ud568\ud574 \uc2dc\ub9ac\uc544 \ub0b4\uc804\uc5d0\uc11c \uc2dc\ub9ac\uc544 \uc815\ubd80\ub97c \uc801\uadf9\uc801\uc73c\ub85c \uc9c0\uc6d0\ud588\ub2e4. 2014\ub144 4\uc6d4, \uc774\ub780 \uc678\uad50\ubd80 \ucc28\uad00\uc778 \ud638\uc138\uc778 \uc544\ubbf8\ub974\uc555\ub3cc\ub77c\ud78c\uc740 \"\uc6b0\ub9ac\ub294 \ubc14\uc0ac\ub974 \uc544\uc0ac\ub4dc \ub300\ud1b5\ub839\uc774 \ubaa9\uc228\uc744 \ubd80\uc9c0\ud558\ub294 \uac83\uc744 \uc6d0\ud558\uc9c0 \uc54a\ub294\ub2e4. \ud558\uc9c0\ub9cc \uc6b0\ub9ac\ub294 \ud14c\ub7ec\ub098 \uadf9\ub2e8\uc8fc\uc758\uc790\ub4e4\uc774 \uc544\uc0ac\ub4dc\uc640 \uc2dc\ub9ac\uc544 \uc815\ubd80\ub97c \ub9dd\uce58\ub294 \uac83\uc744 \ubcf4\uace0 \uc788\uc744 \uc218\ub9cc\uc740 \uc5c6\ub2e4\"\ub77c\uace0 \ub9d0\ud588\ub2e4. 7\uc6d4 24\uc77c, \ucfe0\ub4dc\uc2a4\uad70 \uc0ac\ub839\uad00 \uce74\uc82c \uc194\ub77c\uc774\ub9c8\ub2c8\uac00 \ubaa8\uc2a4\ud06c\ubc14\ub97c \ubc29\ubb38\ud588\uace0, \uc2dc\ub9ac\uc544\uc5d0\uc11c\uc758 \uc591\uad6d \uac04 \ud611\uc758 \uacc4\ud68d\uc774 \ub9c8\ub828\ub418\uc5c8\ub2e4. 2\uac1c\uc758 \ub808\ubc14\ub17c \uc790\ub8cc\uc5d0 \ub530\ub974\uba74, \ub9ac\uc6b0\ud130\ub294 2015\ub144 10\uc6d4 1\uc77c\uc5d0 \uc218\ubc31 \uba85\uc758 \uc774\ub780 \uad70\ub300\uac00 10\uc77c \uc804\ubd80\ud130 \uc2dc\ub9ac\uc544\uc5d0 \ub3c4\ucc29\ud558\uc5ec \uc2dc\ub9ac\uc544 \uc815\ubd80\uad70\uacfc \uace7 \uc5f0\ud569\ud560 \uc900\ube44\ub97c \ub9c8\ucce4\ub2e4\uace0 \ubcf4\uace0\ud588\ub2e4. \uadf8\ub4e4\uc758 \ub808\ubc14\ub17c \ub3d9\ub9f9 \ub2e8\uccb4\uc778 \ud5e4\uc988\ubcfc\ub77c \ub610\ud55c \ub7ec\uc2dc\uc544 \uacf5\uc2b5\uc758 \uc9c0\uc6d0 \ud558\uc5d0 \uc8fc\uc694 \uc721\uc0c1 \uacf5\uc138\ub97c \ud3bc\uce60 \uac83\uc774\ub77c\uace0 \ubcf4\uc558\ub2e4. \uc774\ub780\uc740 \ud604\uc7ac \uc774\ub77c\ud06c\uc5d0\uc11c\ub3c4 \ubbf8\uad6d \ubc0f \uc774\ub77c\ud06c \uc815\ubd80\uc640 \uc5f0\ud569\ud574 ISIS\uc5d0 \ub9de\uc11c \uc2f8\uc6b0\uace0 \uc788\ub2e4."} {"question": "\uc774\uc21c\uc2e0\uc774 \uc0bc\ub3c4\uc218\uad70\ud1b5\uc81c\uc0ac\ub85c \uc784\uba85\ub418\uba74\uc11c \ub2e4\uc2dc \uc870\uc120 \uc218\uad70\uc744 \ubaa8\uc558\uc744 \ub54c, \ud568\uc120\uc740 \uba87 \ucc99 \ub0a8\uc544\uc788\uc5c8\uc2b5\ub2c8\uae4c?", "paragraph": "1597\ub144 8\uc6d4 28\uc77c(\uc74c\ub825 7\uc6d4 16\uc77c)\uc5d0 \uc0bc\ub3c4\uc218\uad70\ud1b5\uc81c\uc0ac\uc5d0 \uc624\ub978 \uc6d0\uade0\uc774 \uc774\ub044\ub294 \uc870\uc120 \ud568\ub300\uac00 \uce60\ucc9c\ub7c9 \ud574\uc804\uc5d0\uc11c \uc77c\ubcf8\uad70\uc758 \uae30\uc2b5\uc744 \ubc1b\uc544 \ucd98\uc6d0\ud3ec\ub85c \ud6c4\ud1f4, \uc218\uad70\ub4e4\uc740 \uc0c1\ub959\ud558\uc5ec \ub3c4\uc8fc\ud558\uace0 \ud310\uc625\uc120 \ub300\ubd80\ubd84\uc774 \ubd88\ud0c0\uac70\ub098 \uc65c\uad70\uc5d0\uac8c \ub178\ud68d\ub2f9\ud574 \uc624\uc0ac\uce74\ub85c \ub04c\ub824\uac04\ub2e4. \uc774\ub97c \uc218\uc2b5\ud558\uae30 \uc704\ud558\uc5ec \uc870\uc120 \uc870\uc815\uc5d0\uc11c\ub294 \uacbd\ub9bc\uad70(\u6176\u6797\u541b) \uae40\uba85\uc6d0(\u91d1\u547d\u5143), \ubcd1\uc870 \ud310\uc11c \uc774\ud56d\ubcf5(\u674e\u6052\u798f)\uc758 \uac74\uc758 \ub85c \uc774\uc21c\uc2e0\uc744 \ub2e4\uc2dc \uc0bc\ub3c4\uc218\uad70\ud1b5\uc81c\uc0ac\ub85c \uc784\uba85\ud558\uc600\ub2e4. \ud558\uc9c0\ub9cc \uc774\uc21c\uc2e0\uc774 \ub2e4\uc2dc \uc870\uc120 \uc218\uad70\uc744 \ubaa8\uc544 \uc815\ube44\ud588\uc744 \ub54c \ud568\uc120\uc740 12\ucc99\ubc16\uc5d0 \ub0a8\uc544 \uc788\uc9c0 \uc54a\uc558\ub2e4. \uc870\uc120 \uc815\ubd80\uc5d0\uc11c\ub294 \uc774 \ubcd1\ub825\uc73c\ub85c \uc801\uc744 \ub300\ud56d\ud0a4 \uc5b4\ub835\ub2e4 \ud558\uc5ec \uc218\uad70\uc744 \ud3d0\uc9c0\ud558\ub77c\ub294 \uba85\ub839\uc744 \ub0b4\ub838\uc73c\ub098, \uc774\uc21c\uc2e0\uc740 \uc544\uc9c1\ub3c4 12\ucc99\uc758 \ubc30\uac00 \ub0a8\uc544 \uc788\uc73c\uba70 \ub0b4\uac00 \uc8fd\uc9c0 \uc54a\ub294 \ud55c \uc801\uc774 \uac10\ud788 \uc6b0\ub9ac\uc758 \uc218\uad70\uc744 \uc5c5\uc2e0\uc5ec\uae30\uc9c0 \ubabb\ud560 \uac83\uc774\ub77c\ub294 \ube44\uc7a5\ud55c \uacb0\uc758\ub97c \ud45c\ud558\uc600\ub2e4\uace0 \ud55c\ub2e4. \uadf8 \ub4a4 \uc804\uc5f4\uc744 \uc7ac\uc815\ube44\ud558\uae30 \uc704\ud574 10\uc6d4 9\uc77c(\uc74c\ub825 8\uc6d4 29\uc77c)\uc5d0 \uc9c4\ub3c4 \ubcbd\ud30c\uc9c4\uc73c\ub85c \uc9c4\uc744 \uc62e\uacbc\ub2e4.", "answer": "12\ucc99", "sentence": "\uc815\ube44\ud588\uc744 \ub54c \ud568\uc120\uc740 12\ucc99 \ubc16\uc5d0 \ub0a8\uc544 \uc788\uc9c0 \uc54a\uc558\ub2e4.", "paragraph_sentence": "1597\ub144 8\uc6d4 28\uc77c(\uc74c\ub825 7\uc6d4 16\uc77c)\uc5d0 \uc0bc\ub3c4\uc218\uad70\ud1b5\uc81c\uc0ac\uc5d0 \uc624\ub978 \uc6d0\uade0\uc774 \uc774\ub044\ub294 \uc870\uc120 \ud568\ub300\uac00 \uce60\ucc9c\ub7c9 \ud574\uc804\uc5d0\uc11c \uc77c\ubcf8\uad70\uc758 \uae30\uc2b5\uc744 \ubc1b\uc544 \ucd98\uc6d0\ud3ec\ub85c \ud6c4\ud1f4, \uc218\uad70\ub4e4\uc740 \uc0c1\ub959\ud558\uc5ec \ub3c4\uc8fc\ud558\uace0 \ud310\uc625\uc120 \ub300\ubd80\ubd84\uc774 \ubd88\ud0c0\uac70\ub098 \uc65c\uad70\uc5d0\uac8c \ub178\ud68d\ub2f9\ud574 \uc624\uc0ac\uce74\ub85c \ub04c\ub824\uac04\ub2e4. \uc774\ub97c \uc218\uc2b5\ud558\uae30 \uc704\ud558\uc5ec \uc870\uc120 \uc870\uc815\uc5d0\uc11c\ub294 \uacbd\ub9bc\uad70(\u6176\u6797\u541b) \uae40\uba85\uc6d0(\u91d1\u547d\u5143), \ubcd1\uc870 \ud310\uc11c \uc774\ud56d\ubcf5(\u674e\u6052\u798f)\uc758 \uac74\uc758 \ub85c \uc774\uc21c\uc2e0\uc744 \ub2e4\uc2dc \uc0bc\ub3c4\uc218\uad70\ud1b5\uc81c\uc0ac\ub85c \uc784\uba85\ud558\uc600\ub2e4. \ud558\uc9c0\ub9cc \uc774\uc21c\uc2e0\uc774 \ub2e4\uc2dc \uc870\uc120 \uc218\uad70\uc744 \ubaa8\uc544 \uc815\ube44\ud588\uc744 \ub54c \ud568\uc120\uc740 12\ucc99 \ubc16\uc5d0 \ub0a8\uc544 \uc788\uc9c0 \uc54a\uc558\ub2e4. \uc870\uc120 \uc815\ubd80\uc5d0\uc11c\ub294 \uc774 \ubcd1\ub825\uc73c\ub85c \uc801\uc744 \ub300\ud56d\ud0a4 \uc5b4\ub835\ub2e4 \ud558\uc5ec \uc218\uad70\uc744 \ud3d0\uc9c0\ud558\ub77c\ub294 \uba85\ub839\uc744 \ub0b4\ub838\uc73c\ub098, \uc774\uc21c\uc2e0\uc740 \uc544\uc9c1\ub3c4 12\ucc99\uc758 \ubc30\uac00 \ub0a8\uc544 \uc788\uc73c\uba70 \ub0b4\uac00 \uc8fd\uc9c0 \uc54a\ub294 \ud55c \uc801\uc774 \uac10\ud788 \uc6b0\ub9ac\uc758 \uc218\uad70\uc744 \uc5c5\uc2e0\uc5ec\uae30\uc9c0 \ubabb\ud560 \uac83\uc774\ub77c\ub294 \ube44\uc7a5\ud55c \uacb0\uc758\ub97c \ud45c\ud558\uc600\ub2e4\uace0 \ud55c\ub2e4. \uadf8 \ub4a4 \uc804\uc5f4\uc744 \uc7ac\uc815\ube44\ud558\uae30 \uc704\ud574 10\uc6d4 9\uc77c(\uc74c\ub825 8\uc6d4 29\uc77c)\uc5d0 \uc9c4\ub3c4 \ubcbd\ud30c\uc9c4\uc73c\ub85c \uc9c4\uc744 \uc62e\uacbc\ub2e4.", "paragraph_answer": "1597\ub144 8\uc6d4 28\uc77c(\uc74c\ub825 7\uc6d4 16\uc77c)\uc5d0 \uc0bc\ub3c4\uc218\uad70\ud1b5\uc81c\uc0ac\uc5d0 \uc624\ub978 \uc6d0\uade0\uc774 \uc774\ub044\ub294 \uc870\uc120 \ud568\ub300\uac00 \uce60\ucc9c\ub7c9 \ud574\uc804\uc5d0\uc11c \uc77c\ubcf8\uad70\uc758 \uae30\uc2b5\uc744 \ubc1b\uc544 \ucd98\uc6d0\ud3ec\ub85c \ud6c4\ud1f4, \uc218\uad70\ub4e4\uc740 \uc0c1\ub959\ud558\uc5ec \ub3c4\uc8fc\ud558\uace0 \ud310\uc625\uc120 \ub300\ubd80\ubd84\uc774 \ubd88\ud0c0\uac70\ub098 \uc65c\uad70\uc5d0\uac8c \ub178\ud68d\ub2f9\ud574 \uc624\uc0ac\uce74\ub85c \ub04c\ub824\uac04\ub2e4. \uc774\ub97c \uc218\uc2b5\ud558\uae30 \uc704\ud558\uc5ec \uc870\uc120 \uc870\uc815\uc5d0\uc11c\ub294 \uacbd\ub9bc\uad70(\u6176\u6797\u541b) \uae40\uba85\uc6d0(\u91d1\u547d\u5143), \ubcd1\uc870 \ud310\uc11c \uc774\ud56d\ubcf5(\u674e\u6052\u798f)\uc758 \uac74\uc758 \ub85c \uc774\uc21c\uc2e0\uc744 \ub2e4\uc2dc \uc0bc\ub3c4\uc218\uad70\ud1b5\uc81c\uc0ac\ub85c \uc784\uba85\ud558\uc600\ub2e4. \ud558\uc9c0\ub9cc \uc774\uc21c\uc2e0\uc774 \ub2e4\uc2dc \uc870\uc120 \uc218\uad70\uc744 \ubaa8\uc544 \uc815\ube44\ud588\uc744 \ub54c \ud568\uc120\uc740 12\ucc99 \ubc16\uc5d0 \ub0a8\uc544 \uc788\uc9c0 \uc54a\uc558\ub2e4. \uc870\uc120 \uc815\ubd80\uc5d0\uc11c\ub294 \uc774 \ubcd1\ub825\uc73c\ub85c \uc801\uc744 \ub300\ud56d\ud0a4 \uc5b4\ub835\ub2e4 \ud558\uc5ec \uc218\uad70\uc744 \ud3d0\uc9c0\ud558\ub77c\ub294 \uba85\ub839\uc744 \ub0b4\ub838\uc73c\ub098, \uc774\uc21c\uc2e0\uc740 \uc544\uc9c1\ub3c4 12\ucc99\uc758 \ubc30\uac00 \ub0a8\uc544 \uc788\uc73c\uba70 \ub0b4\uac00 \uc8fd\uc9c0 \uc54a\ub294 \ud55c \uc801\uc774 \uac10\ud788 \uc6b0\ub9ac\uc758 \uc218\uad70\uc744 \uc5c5\uc2e0\uc5ec\uae30\uc9c0 \ubabb\ud560 \uac83\uc774\ub77c\ub294 \ube44\uc7a5\ud55c \uacb0\uc758\ub97c \ud45c\ud558\uc600\ub2e4\uace0 \ud55c\ub2e4. \uadf8 \ub4a4 \uc804\uc5f4\uc744 \uc7ac\uc815\ube44\ud558\uae30 \uc704\ud574 10\uc6d4 9\uc77c(\uc74c\ub825 8\uc6d4 29\uc77c)\uc5d0 \uc9c4\ub3c4 \ubcbd\ud30c\uc9c4\uc73c\ub85c \uc9c4\uc744 \uc62e\uacbc\ub2e4.", "sentence_answer": "\uc815\ube44\ud588\uc744 \ub54c \ud568\uc120\uc740 12\ucc99 \ubc16\uc5d0 \ub0a8\uc544 \uc788\uc9c0 \uc54a\uc558\ub2e4.", "paragraph_id": "6545635-10-0", "paragraph_question": "question: \uc774\uc21c\uc2e0\uc774 \uc0bc\ub3c4\uc218\uad70\ud1b5\uc81c\uc0ac\ub85c \uc784\uba85\ub418\uba74\uc11c \ub2e4\uc2dc \uc870\uc120 \uc218\uad70\uc744 \ubaa8\uc558\uc744 \ub54c, \ud568\uc120\uc740 \uba87 \ucc99 \ub0a8\uc544\uc788\uc5c8\uc2b5\ub2c8\uae4c?, context: 1597\ub144 8\uc6d4 28\uc77c(\uc74c\ub825 7\uc6d4 16\uc77c)\uc5d0 \uc0bc\ub3c4\uc218\uad70\ud1b5\uc81c\uc0ac\uc5d0 \uc624\ub978 \uc6d0\uade0\uc774 \uc774\ub044\ub294 \uc870\uc120 \ud568\ub300\uac00 \uce60\ucc9c\ub7c9 \ud574\uc804\uc5d0\uc11c \uc77c\ubcf8\uad70\uc758 \uae30\uc2b5\uc744 \ubc1b\uc544 \ucd98\uc6d0\ud3ec\ub85c \ud6c4\ud1f4, \uc218\uad70\ub4e4\uc740 \uc0c1\ub959\ud558\uc5ec \ub3c4\uc8fc\ud558\uace0 \ud310\uc625\uc120 \ub300\ubd80\ubd84\uc774 \ubd88\ud0c0\uac70\ub098 \uc65c\uad70\uc5d0\uac8c \ub178\ud68d\ub2f9\ud574 \uc624\uc0ac\uce74\ub85c \ub04c\ub824\uac04\ub2e4. \uc774\ub97c \uc218\uc2b5\ud558\uae30 \uc704\ud558\uc5ec \uc870\uc120 \uc870\uc815\uc5d0\uc11c\ub294 \uacbd\ub9bc\uad70(\u6176\u6797\u541b) \uae40\uba85\uc6d0(\u91d1\u547d\u5143), \ubcd1\uc870 \ud310\uc11c \uc774\ud56d\ubcf5(\u674e\u6052\u798f)\uc758 \uac74\uc758 \ub85c \uc774\uc21c\uc2e0\uc744 \ub2e4\uc2dc \uc0bc\ub3c4\uc218\uad70\ud1b5\uc81c\uc0ac\ub85c \uc784\uba85\ud558\uc600\ub2e4. \ud558\uc9c0\ub9cc \uc774\uc21c\uc2e0\uc774 \ub2e4\uc2dc \uc870\uc120 \uc218\uad70\uc744 \ubaa8\uc544 \uc815\ube44\ud588\uc744 \ub54c \ud568\uc120\uc740 12\ucc99\ubc16\uc5d0 \ub0a8\uc544 \uc788\uc9c0 \uc54a\uc558\ub2e4. \uc870\uc120 \uc815\ubd80\uc5d0\uc11c\ub294 \uc774 \ubcd1\ub825\uc73c\ub85c \uc801\uc744 \ub300\ud56d\ud0a4 \uc5b4\ub835\ub2e4 \ud558\uc5ec \uc218\uad70\uc744 \ud3d0\uc9c0\ud558\ub77c\ub294 \uba85\ub839\uc744 \ub0b4\ub838\uc73c\ub098, \uc774\uc21c\uc2e0\uc740 \uc544\uc9c1\ub3c4 12\ucc99\uc758 \ubc30\uac00 \ub0a8\uc544 \uc788\uc73c\uba70 \ub0b4\uac00 \uc8fd\uc9c0 \uc54a\ub294 \ud55c \uc801\uc774 \uac10\ud788 \uc6b0\ub9ac\uc758 \uc218\uad70\uc744 \uc5c5\uc2e0\uc5ec\uae30\uc9c0 \ubabb\ud560 \uac83\uc774\ub77c\ub294 \ube44\uc7a5\ud55c \uacb0\uc758\ub97c \ud45c\ud558\uc600\ub2e4\uace0 \ud55c\ub2e4. \uadf8 \ub4a4 \uc804\uc5f4\uc744 \uc7ac\uc815\ube44\ud558\uae30 \uc704\ud574 10\uc6d4 9\uc77c(\uc74c\ub825 8\uc6d4 29\uc77c)\uc5d0 \uc9c4\ub3c4 \ubcbd\ud30c\uc9c4\uc73c\ub85c \uc9c4\uc744 \uc62e\uacbc\ub2e4."} {"question": "Born This Way \ub294 \ube4c\ubcf4\ub4dc\uc5d0\uc11c \uba87 \uc704\ub85c \ub370\ubdd4\ud558\uc600\ub294\uac00?", "paragraph": "2011\ub144 5\uc6d4 23\uc77c \uac00\uac00\ub294 \ub450 \ubc88\uc9f8 \uc815\uaddc \uc568\ubc94 Born This Way\uc744 \ubc1c\ub9e4\ud588\ub2e4. \uac00\uac00\ub294 \uc77c\ub809\ud2b8\ub85c\ub2c9 \uc74c\uc545\uacfc \uba54\ud0c8, \ub77d\uc564\ub864, \ud31d, \uc131\uac00\uc640 \uac19\uc740 \uba5c\ub85c\ub514\ub97c \uac00\ubbf8\ud558\uace0 \ud070\ub9dd\uce58\ub85c \ub0b4\ub9ac\uce58\ub294 \uac83\uacfc \uac19\uc740 \ub304\uc2a4 \ube44\ud2b8\ub97c \uac00\uc84c\ub2e4\uace0 \uc124\uba85\ud588\ub2e4. \uc774\uc5b4 \"\ud654\uc81c\uac00 \ub418\uc5c8\ub358 \uac00\ubc1c, \ub9bd\uc2a4\ud2f1, \uc0dd\uace0\uae30 \ub4dc\ub808\uc2a4\ubcf4\ub2e4 \ud6e8\uc52c \ub354 \uc2ec\uc624\uc2a4\ub7fd\ub2e4\"\uace0 \ud2b9\uc9d5\uc744 \ub9d0\ud588\ub294\ub370, \uc5d0\uc774\ucf58\uc740 \uc568\ubc94\uc744 \ub4e3\uace0 \uc74c\uc545\uc774 \"\ud55c \ub2e8\uacc4 \ub354 \uc9c4\ud654\"\ud588\ub2e4\uace0 \ub9d0\ud588\ub2e4. \ubc1c\ub9e4\ub418\uace0\ub098\uc11c \uc568\ubc94\uc740 \ud3c9\ub860\uac00\ub4e4\uc5d0\uac8c \uc7a5\ub974\uc758 \ub2e4\uc601\uc131\uacfc \uac00\uac00\uc758 \ubcf4\uceec\uc774 \ud638\ud3c9\uc744 \ub4e4\uc73c\uba74\uc11c \ub300\uccb4\ub85c \uae0d\uc815\uc801\uc778 \ud3c9\uac00\ub97c \ubc1b\uc558\ub2e4. Born This Way\ub294 \ubbf8\uad6d \ubc1c\ub9e4 \uccab \uc8fc 1,108,000\ub9cc \uc7a5\uc744 \ud314\uc544\uce58\uc6b0\uba74\uc11c \ube4c\ubcf4\ub4dc 200 1\uc704\ub85c \ub370\ubdd4\ud588\uace0, \uc774 \uc678\uc5d0 20\uac1c \uad6d\uac00\uc5d0\uc11c\ub3c4 1\uc704\ub97c \ud588\ub2e4. \uc774\ud6c4 \uc138\uacc4\uc801\uc73c\ub85c 800\ub9cc \uc7a5\uc5d0 \uc774\ub974\ub294 \ud310\ub9e4\ub7c9\uc744 \uae30\ub85d\ud588\uace0, Born This Way\ub294 \uadf8\ub798\ubbf8 \uc0c1 3\uac1c \ubd80\ubb38 \ud6c4\ubcf4\ub85c \uc62c\ub790\ub294\ub370, \ud2b9\ud788 3\ub144 \uc5f0\uc18d \uc62c\ud574\uc758 \uc568\ubc94 \ud6c4\ubcf4\ub85c \uc9c0\uba85\ub418\uc5c8\ub2e4. 2012\ub144 3\uc6d4 \u300a\ube4c\ubcf4\ub4dc\u300b\uac00 \ubc1c\ud45c\ud55c 2011\ub144 \uac00\uc7a5 \ub9ce\uc740 \ub3c8\uc744 \ubc8c\uc5b4\ub4dc\ub9b0 \uac00\uc218 \uc21c\uc704 \uc911 \uac00\uac00\ub294 Born This Way \ud310\ub9e4\uc218\uc775\uacfc Monster Ball Tour \uc218\uc775\uc73c\ub85c 25,353,039\ub9cc \ub2ec\ub7ec\ub97c \ubc8c\uc5b4\ub4e4\uc5ec 4\uc704\uc5d0 \uc62c\ub790\ub2e4.", "answer": "1\uc704", "sentence": "\ube4c\ubcf4\ub4dc 200 1\uc704 \ub85c \ub370\ubdd4\ud588\uace0, \uc774 \uc678\uc5d0 20\uac1c \uad6d\uac00\uc5d0\uc11c\ub3c4 1\uc704\ub97c \ud588\ub2e4.", "paragraph_sentence": "2011\ub144 5\uc6d4 23\uc77c \uac00\uac00\ub294 \ub450 \ubc88\uc9f8 \uc815\uaddc \uc568\ubc94 Born This Way\uc744 \ubc1c\ub9e4\ud588\ub2e4. \uac00\uac00\ub294 \uc77c\ub809\ud2b8\ub85c\ub2c9 \uc74c\uc545\uacfc \uba54\ud0c8, \ub77d\uc564\ub864, \ud31d, \uc131\uac00\uc640 \uac19\uc740 \uba5c\ub85c\ub514\ub97c \uac00\ubbf8\ud558\uace0 \ud070\ub9dd\uce58\ub85c \ub0b4\ub9ac\uce58\ub294 \uac83\uacfc \uac19\uc740 \ub304\uc2a4 \ube44\ud2b8\ub97c \uac00\uc84c\ub2e4\uace0 \uc124\uba85\ud588\ub2e4. \uc774\uc5b4 \"\ud654\uc81c\uac00 \ub418\uc5c8\ub358 \uac00\ubc1c, \ub9bd\uc2a4\ud2f1, \uc0dd\uace0\uae30 \ub4dc\ub808\uc2a4\ubcf4\ub2e4 \ud6e8\uc52c \ub354 \uc2ec\uc624\uc2a4\ub7fd\ub2e4\"\uace0 \ud2b9\uc9d5\uc744 \ub9d0\ud588\ub294\ub370, \uc5d0\uc774\ucf58\uc740 \uc568\ubc94\uc744 \ub4e3\uace0 \uc74c\uc545\uc774 \"\ud55c \ub2e8\uacc4 \ub354 \uc9c4\ud654\"\ud588\ub2e4\uace0 \ub9d0\ud588\ub2e4. \ubc1c\ub9e4\ub418\uace0\ub098\uc11c \uc568\ubc94\uc740 \ud3c9\ub860\uac00\ub4e4\uc5d0\uac8c \uc7a5\ub974\uc758 \ub2e4\uc601\uc131\uacfc \uac00\uac00\uc758 \ubcf4\uceec\uc774 \ud638\ud3c9\uc744 \ub4e4\uc73c\uba74\uc11c \ub300\uccb4\ub85c \uae0d\uc815\uc801\uc778 \ud3c9\uac00\ub97c \ubc1b\uc558\ub2e4. Born This Way\ub294 \ubbf8\uad6d \ubc1c\ub9e4 \uccab \uc8fc 1,108,000\ub9cc \uc7a5\uc744 \ud314\uc544\uce58\uc6b0\uba74\uc11c \ube4c\ubcf4\ub4dc 200 1\uc704 \ub85c \ub370\ubdd4\ud588\uace0, \uc774 \uc678\uc5d0 20\uac1c \uad6d\uac00\uc5d0\uc11c\ub3c4 1\uc704\ub97c \ud588\ub2e4. \uc774\ud6c4 \uc138\uacc4\uc801\uc73c\ub85c 800\ub9cc \uc7a5\uc5d0 \uc774\ub974\ub294 \ud310\ub9e4\ub7c9\uc744 \uae30\ub85d\ud588\uace0, Born This Way\ub294 \uadf8\ub798\ubbf8 \uc0c1 3\uac1c \ubd80\ubb38 \ud6c4\ubcf4\ub85c \uc62c\ub790\ub294\ub370, \ud2b9\ud788 3\ub144 \uc5f0\uc18d \uc62c\ud574\uc758 \uc568\ubc94 \ud6c4\ubcf4\ub85c \uc9c0\uba85\ub418\uc5c8\ub2e4. 2012\ub144 3\uc6d4 \u300a\ube4c\ubcf4\ub4dc\u300b\uac00 \ubc1c\ud45c\ud55c 2011\ub144 \uac00\uc7a5 \ub9ce\uc740 \ub3c8\uc744 \ubc8c\uc5b4\ub4dc\ub9b0 \uac00\uc218 \uc21c\uc704 \uc911 \uac00\uac00\ub294 Born This Way \ud310\ub9e4\uc218\uc775\uacfc Monster Ball Tour \uc218\uc775\uc73c\ub85c 25,353,039\ub9cc \ub2ec\ub7ec\ub97c \ubc8c\uc5b4\ub4e4\uc5ec 4\uc704\uc5d0 \uc62c\ub790\ub2e4.", "paragraph_answer": "2011\ub144 5\uc6d4 23\uc77c \uac00\uac00\ub294 \ub450 \ubc88\uc9f8 \uc815\uaddc \uc568\ubc94 Born This Way\uc744 \ubc1c\ub9e4\ud588\ub2e4. \uac00\uac00\ub294 \uc77c\ub809\ud2b8\ub85c\ub2c9 \uc74c\uc545\uacfc \uba54\ud0c8, \ub77d\uc564\ub864, \ud31d, \uc131\uac00\uc640 \uac19\uc740 \uba5c\ub85c\ub514\ub97c \uac00\ubbf8\ud558\uace0 \ud070\ub9dd\uce58\ub85c \ub0b4\ub9ac\uce58\ub294 \uac83\uacfc \uac19\uc740 \ub304\uc2a4 \ube44\ud2b8\ub97c \uac00\uc84c\ub2e4\uace0 \uc124\uba85\ud588\ub2e4. \uc774\uc5b4 \"\ud654\uc81c\uac00 \ub418\uc5c8\ub358 \uac00\ubc1c, \ub9bd\uc2a4\ud2f1, \uc0dd\uace0\uae30 \ub4dc\ub808\uc2a4\ubcf4\ub2e4 \ud6e8\uc52c \ub354 \uc2ec\uc624\uc2a4\ub7fd\ub2e4\"\uace0 \ud2b9\uc9d5\uc744 \ub9d0\ud588\ub294\ub370, \uc5d0\uc774\ucf58\uc740 \uc568\ubc94\uc744 \ub4e3\uace0 \uc74c\uc545\uc774 \"\ud55c \ub2e8\uacc4 \ub354 \uc9c4\ud654\"\ud588\ub2e4\uace0 \ub9d0\ud588\ub2e4. \ubc1c\ub9e4\ub418\uace0\ub098\uc11c \uc568\ubc94\uc740 \ud3c9\ub860\uac00\ub4e4\uc5d0\uac8c \uc7a5\ub974\uc758 \ub2e4\uc601\uc131\uacfc \uac00\uac00\uc758 \ubcf4\uceec\uc774 \ud638\ud3c9\uc744 \ub4e4\uc73c\uba74\uc11c \ub300\uccb4\ub85c \uae0d\uc815\uc801\uc778 \ud3c9\uac00\ub97c \ubc1b\uc558\ub2e4. Born This Way\ub294 \ubbf8\uad6d \ubc1c\ub9e4 \uccab \uc8fc 1,108,000\ub9cc \uc7a5\uc744 \ud314\uc544\uce58\uc6b0\uba74\uc11c \ube4c\ubcf4\ub4dc 200 1\uc704 \ub85c \ub370\ubdd4\ud588\uace0, \uc774 \uc678\uc5d0 20\uac1c \uad6d\uac00\uc5d0\uc11c\ub3c4 1\uc704\ub97c \ud588\ub2e4. \uc774\ud6c4 \uc138\uacc4\uc801\uc73c\ub85c 800\ub9cc \uc7a5\uc5d0 \uc774\ub974\ub294 \ud310\ub9e4\ub7c9\uc744 \uae30\ub85d\ud588\uace0, Born This Way\ub294 \uadf8\ub798\ubbf8 \uc0c1 3\uac1c \ubd80\ubb38 \ud6c4\ubcf4\ub85c \uc62c\ub790\ub294\ub370, \ud2b9\ud788 3\ub144 \uc5f0\uc18d \uc62c\ud574\uc758 \uc568\ubc94 \ud6c4\ubcf4\ub85c \uc9c0\uba85\ub418\uc5c8\ub2e4. 2012\ub144 3\uc6d4 \u300a\ube4c\ubcf4\ub4dc\u300b\uac00 \ubc1c\ud45c\ud55c 2011\ub144 \uac00\uc7a5 \ub9ce\uc740 \ub3c8\uc744 \ubc8c\uc5b4\ub4dc\ub9b0 \uac00\uc218 \uc21c\uc704 \uc911 \uac00\uac00\ub294 Born This Way \ud310\ub9e4\uc218\uc775\uacfc Monster Ball Tour \uc218\uc775\uc73c\ub85c 25,353,039\ub9cc \ub2ec\ub7ec\ub97c \ubc8c\uc5b4\ub4e4\uc5ec 4\uc704\uc5d0 \uc62c\ub790\ub2e4.", "sentence_answer": "\ube4c\ubcf4\ub4dc 200 1\uc704 \ub85c \ub370\ubdd4\ud588\uace0, \uc774 \uc678\uc5d0 20\uac1c \uad6d\uac00\uc5d0\uc11c\ub3c4 1\uc704\ub97c \ud588\ub2e4.", "paragraph_id": "6513681-9-2", "paragraph_question": "question: Born This Way \ub294 \ube4c\ubcf4\ub4dc\uc5d0\uc11c \uba87 \uc704\ub85c \ub370\ubdd4\ud558\uc600\ub294\uac00?, context: 2011\ub144 5\uc6d4 23\uc77c \uac00\uac00\ub294 \ub450 \ubc88\uc9f8 \uc815\uaddc \uc568\ubc94 Born This Way\uc744 \ubc1c\ub9e4\ud588\ub2e4. \uac00\uac00\ub294 \uc77c\ub809\ud2b8\ub85c\ub2c9 \uc74c\uc545\uacfc \uba54\ud0c8, \ub77d\uc564\ub864, \ud31d, \uc131\uac00\uc640 \uac19\uc740 \uba5c\ub85c\ub514\ub97c \uac00\ubbf8\ud558\uace0 \ud070\ub9dd\uce58\ub85c \ub0b4\ub9ac\uce58\ub294 \uac83\uacfc \uac19\uc740 \ub304\uc2a4 \ube44\ud2b8\ub97c \uac00\uc84c\ub2e4\uace0 \uc124\uba85\ud588\ub2e4. \uc774\uc5b4 \"\ud654\uc81c\uac00 \ub418\uc5c8\ub358 \uac00\ubc1c, \ub9bd\uc2a4\ud2f1, \uc0dd\uace0\uae30 \ub4dc\ub808\uc2a4\ubcf4\ub2e4 \ud6e8\uc52c \ub354 \uc2ec\uc624\uc2a4\ub7fd\ub2e4\"\uace0 \ud2b9\uc9d5\uc744 \ub9d0\ud588\ub294\ub370, \uc5d0\uc774\ucf58\uc740 \uc568\ubc94\uc744 \ub4e3\uace0 \uc74c\uc545\uc774 \"\ud55c \ub2e8\uacc4 \ub354 \uc9c4\ud654\"\ud588\ub2e4\uace0 \ub9d0\ud588\ub2e4. \ubc1c\ub9e4\ub418\uace0\ub098\uc11c \uc568\ubc94\uc740 \ud3c9\ub860\uac00\ub4e4\uc5d0\uac8c \uc7a5\ub974\uc758 \ub2e4\uc601\uc131\uacfc \uac00\uac00\uc758 \ubcf4\uceec\uc774 \ud638\ud3c9\uc744 \ub4e4\uc73c\uba74\uc11c \ub300\uccb4\ub85c \uae0d\uc815\uc801\uc778 \ud3c9\uac00\ub97c \ubc1b\uc558\ub2e4. Born This Way\ub294 \ubbf8\uad6d \ubc1c\ub9e4 \uccab \uc8fc 1,108,000\ub9cc \uc7a5\uc744 \ud314\uc544\uce58\uc6b0\uba74\uc11c \ube4c\ubcf4\ub4dc 200 1\uc704\ub85c \ub370\ubdd4\ud588\uace0, \uc774 \uc678\uc5d0 20\uac1c \uad6d\uac00\uc5d0\uc11c\ub3c4 1\uc704\ub97c \ud588\ub2e4. \uc774\ud6c4 \uc138\uacc4\uc801\uc73c\ub85c 800\ub9cc \uc7a5\uc5d0 \uc774\ub974\ub294 \ud310\ub9e4\ub7c9\uc744 \uae30\ub85d\ud588\uace0, Born This Way\ub294 \uadf8\ub798\ubbf8 \uc0c1 3\uac1c \ubd80\ubb38 \ud6c4\ubcf4\ub85c \uc62c\ub790\ub294\ub370, \ud2b9\ud788 3\ub144 \uc5f0\uc18d \uc62c\ud574\uc758 \uc568\ubc94 \ud6c4\ubcf4\ub85c \uc9c0\uba85\ub418\uc5c8\ub2e4. 2012\ub144 3\uc6d4 \u300a\ube4c\ubcf4\ub4dc\u300b\uac00 \ubc1c\ud45c\ud55c 2011\ub144 \uac00\uc7a5 \ub9ce\uc740 \ub3c8\uc744 \ubc8c\uc5b4\ub4dc\ub9b0 \uac00\uc218 \uc21c\uc704 \uc911 \uac00\uac00\ub294 Born This Way \ud310\ub9e4\uc218\uc775\uacfc Monster Ball Tour \uc218\uc775\uc73c\ub85c 25,353,039\ub9cc \ub2ec\ub7ec\ub97c \ubc8c\uc5b4\ub4e4\uc5ec 4\uc704\uc5d0 \uc62c\ub790\ub2e4."} {"question": "1950\ub144 12\uc6d4 24\uc77c \ubbf8\uad6d 10\uad70\ub2e8\uc774 \ucca0\uc218\uc791\uc804\uc744 \ud3bc\uce5c \uacf3\uc740?", "paragraph": "1950\ub144 6\uc6d4 \ub300\ud55c\ubbfc\uad6d\uc5d0 \ub300\ud55c \uae30\uc2b5 \uacf5\uaca9 \uc774\ud6c4 \uc870\uc120\uc778\ubbfc\uad70\uc740 1950\ub144 9\uc6d4 \ub099\ub3d9\uac15 \uc804\ud22c\uc640 \uc778\ucc9c \uc0c1\ub959 \uc791\uc804\uc73c\ub85c \uad6d\uc81c \uc5f0\ud569\uad70\uc5d0 \uc758\ud574 \ubd84\uc1c4\ub418\uc5c8\uace0, \uc870\uc120\uc778\ubbfc\uad70\uc758 \uc794\uc874 \ubcd1\ub825\uc740 \ubd81\ucabd\uc73c\ub85c \ub3c4\ub9dd\uce58\uba70 \ud55c\uc911 \uad6d\uacbd \uc9c0\ub300\uc758 \uc0b0\uc545 \uc9c0\uc5ed\uc5d0\uc11c \ud53c\ub09c\ucc98\ub97c \ucc3e\uace0 \uc788\uc5c8\ub2e4. \uc870\uc120\uc778\ubbfc\uad70\uc758 \uada4\uba78\uc740 \uc911\ud654\uc778\ubbfc\uacf5\ud654\uad6d\uc774 \ud55c\uad6d \uc804\uc7c1\uc5d0 \uc989\uac01\uc801\uc73c\ub85c \uac1c\uc785\ud558\uac1c \ub9cc\ub4e4\uc5c8\uace0 \uc911\uacf5\uad70\uc740 1950\ub144 11\uc6d4\ubd80\ud130 \uad6d\uacbd \uc9c0\uc5ed\uc758 \uc720\uc5d4\uad70\uc5d0 \ub9de\uc11c \ub300\uaddc\ubaa8 \uae30\uc2b5 \uacf5\uaca9\uc744 \uac1c\uc2dc\ud588\ub2e4. \uc81c2\ucc28 \uccad\ucc9c\uac15 \uc804\ud22c\uc640 \uc7a5\uc9c4\ud638 \uc804\ud22c\ub85c \uc720\uc5d4\uad70\uc740 1950\ub144 12\uc6d4 38\ub3c4\uc120 \uc77c\ub300\ub85c \ucca0\uc218\ud560 \uc218 \ubc16\uc5d0 \uc5c6\uc5c8\ub2e4. \ub3d9\ubd80 \uc804\uc120\uc5d0\uc11c \ubbf8\uad6d \uc81c10\uad70\ub2e8\uc740 \uc911\uacf5\uad70\uc758 \uae30\uc2b5 \uacf5\uc138 \ub3d9\uc548 \uc7a5\uc9c4\ud638 \uc778\uadfc\uc758 \ud765\ub0a8 \ud56d\uad6c\uc5d0 \ud3ec\uc704\ub418\uc5c8\uace0, 1950\ub144 12\uc6d4 24\uc77c \ud574\uc0c1\uc73c\ub85c \ubd81\ud55c \uc9c0\uc5ed\uc5d0\uc11c \ucca0\uc218\ud588\ub2e4. \ub300\ud55c\ubbfc\uad6d \uc721\uad70\uc740 \ubbf8\uad70\uc758 \ubd80\uc7ac\ub85c 38\uc120 \uc77c\ub300\uc758 \uc911\ubd80 \ubc0f \ub3d9\ubd80 \uc804\uc120\uc5d0 \uc704\uce58\ud55c \ubc29\uc5b4\uc9c4\uc9c0\ub4e4\uc744 \uc778\uacc4\ud560 \uc218 \ubc16\uc5d0 \uc5c6\uc5c8\uace0, \uc911\ubd80 \uc804\uc120 \uc778\uadfc\uc5d0 \uc704\uce58\ud55c \uc6d0\uc8fc\uc2dc\uc758 \uc911\uc694 \uad50\ucc28\ub85c\ub3c4 \ud3ec\ud568\ub418\uc5b4 \uc788\uc5c8\ub2e4. \uc720\uc5d4\uad70\uc758 \uac11\uc791\uc2a4\ub7ec\uc6b4 \ud328\ubc30\ub85c \uc778\ud574 \ubd81\ud55c\uad70\uc740 \uc9e7\uac8c\ub098\ub9c8 \ud734\uc2dd\uae30\uac04\uc744 \uac00\uc9c8 \uc218 \uc788\uc5c8\uace0 1950\ub144 \ub9d0 \ubd84\uc0b0\ub41c \ubd81\ud55c\uad70\uc740 \ubcd1\ub825\uc744 \ub2e4\uc2dc \ud68c\ubcf5\uc2dc\ucf30\ub2e4.", "answer": "\ud765\ub0a8 \ud56d\uad6c", "sentence": "\ub3d9\ubd80 \uc804\uc120\uc5d0\uc11c \ubbf8\uad6d \uc81c10\uad70\ub2e8\uc740 \uc911\uacf5\uad70\uc758 \uae30\uc2b5 \uacf5\uc138 \ub3d9\uc548 \uc7a5\uc9c4\ud638 \uc778\uadfc\uc758 \ud765\ub0a8 \ud56d\uad6c \uc5d0 \ud3ec\uc704\ub418\uc5c8\uace0, 1950\ub144 12\uc6d4 24\uc77c \ud574\uc0c1\uc73c\ub85c \ubd81\ud55c \uc9c0\uc5ed\uc5d0\uc11c \ucca0\uc218\ud588\ub2e4.", "paragraph_sentence": "1950\ub144 6\uc6d4 \ub300\ud55c\ubbfc\uad6d\uc5d0 \ub300\ud55c \uae30\uc2b5 \uacf5\uaca9 \uc774\ud6c4 \uc870\uc120\uc778\ubbfc\uad70\uc740 1950\ub144 9\uc6d4 \ub099\ub3d9\uac15 \uc804\ud22c\uc640 \uc778\ucc9c \uc0c1\ub959 \uc791\uc804\uc73c\ub85c \uad6d\uc81c \uc5f0\ud569\uad70\uc5d0 \uc758\ud574 \ubd84\uc1c4\ub418\uc5c8\uace0, \uc870\uc120\uc778\ubbfc\uad70\uc758 \uc794\uc874 \ubcd1\ub825\uc740 \ubd81\ucabd\uc73c\ub85c \ub3c4\ub9dd\uce58\uba70 \ud55c\uc911 \uad6d\uacbd \uc9c0\ub300\uc758 \uc0b0\uc545 \uc9c0\uc5ed\uc5d0\uc11c \ud53c\ub09c\ucc98\ub97c \ucc3e\uace0 \uc788\uc5c8\ub2e4. \uc870\uc120\uc778\ubbfc\uad70\uc758 \uada4\uba78\uc740 \uc911\ud654\uc778\ubbfc\uacf5\ud654\uad6d\uc774 \ud55c\uad6d \uc804\uc7c1\uc5d0 \uc989\uac01\uc801\uc73c\ub85c \uac1c\uc785\ud558\uac1c \ub9cc\ub4e4\uc5c8\uace0 \uc911\uacf5\uad70\uc740 1950\ub144 11\uc6d4\ubd80\ud130 \uad6d\uacbd \uc9c0\uc5ed\uc758 \uc720\uc5d4\uad70\uc5d0 \ub9de\uc11c \ub300\uaddc\ubaa8 \uae30\uc2b5 \uacf5\uaca9\uc744 \uac1c\uc2dc\ud588\ub2e4. \uc81c2\ucc28 \uccad\ucc9c\uac15 \uc804\ud22c\uc640 \uc7a5\uc9c4\ud638 \uc804\ud22c\ub85c \uc720\uc5d4\uad70\uc740 1950\ub144 12\uc6d4 38\ub3c4\uc120 \uc77c\ub300\ub85c \ucca0\uc218\ud560 \uc218 \ubc16\uc5d0 \uc5c6\uc5c8\ub2e4. \ub3d9\ubd80 \uc804\uc120\uc5d0\uc11c \ubbf8\uad6d \uc81c10\uad70\ub2e8\uc740 \uc911\uacf5\uad70\uc758 \uae30\uc2b5 \uacf5\uc138 \ub3d9\uc548 \uc7a5\uc9c4\ud638 \uc778\uadfc\uc758 \ud765\ub0a8 \ud56d\uad6c \uc5d0 \ud3ec\uc704\ub418\uc5c8\uace0, 1950\ub144 12\uc6d4 24\uc77c \ud574\uc0c1\uc73c\ub85c \ubd81\ud55c \uc9c0\uc5ed\uc5d0\uc11c \ucca0\uc218\ud588\ub2e4. \ub300\ud55c\ubbfc\uad6d \uc721\uad70\uc740 \ubbf8\uad70\uc758 \ubd80\uc7ac\ub85c 38\uc120 \uc77c\ub300\uc758 \uc911\ubd80 \ubc0f \ub3d9\ubd80 \uc804\uc120\uc5d0 \uc704\uce58\ud55c \ubc29\uc5b4\uc9c4\uc9c0\ub4e4\uc744 \uc778\uacc4\ud560 \uc218 \ubc16\uc5d0 \uc5c6\uc5c8\uace0, \uc911\ubd80 \uc804\uc120 \uc778\uadfc\uc5d0 \uc704\uce58\ud55c \uc6d0\uc8fc\uc2dc\uc758 \uc911\uc694 \uad50\ucc28\ub85c\ub3c4 \ud3ec\ud568\ub418\uc5b4 \uc788\uc5c8\ub2e4. \uc720\uc5d4\uad70\uc758 \uac11\uc791\uc2a4\ub7ec\uc6b4 \ud328\ubc30\ub85c \uc778\ud574 \ubd81\ud55c\uad70\uc740 \uc9e7\uac8c\ub098\ub9c8 \ud734\uc2dd\uae30\uac04\uc744 \uac00\uc9c8 \uc218 \uc788\uc5c8\uace0 1950\ub144 \ub9d0 \ubd84\uc0b0\ub41c \ubd81\ud55c\uad70\uc740 \ubcd1\ub825\uc744 \ub2e4\uc2dc \ud68c\ubcf5\uc2dc\ucf30\ub2e4.", "paragraph_answer": "1950\ub144 6\uc6d4 \ub300\ud55c\ubbfc\uad6d\uc5d0 \ub300\ud55c \uae30\uc2b5 \uacf5\uaca9 \uc774\ud6c4 \uc870\uc120\uc778\ubbfc\uad70\uc740 1950\ub144 9\uc6d4 \ub099\ub3d9\uac15 \uc804\ud22c\uc640 \uc778\ucc9c \uc0c1\ub959 \uc791\uc804\uc73c\ub85c \uad6d\uc81c \uc5f0\ud569\uad70\uc5d0 \uc758\ud574 \ubd84\uc1c4\ub418\uc5c8\uace0, \uc870\uc120\uc778\ubbfc\uad70\uc758 \uc794\uc874 \ubcd1\ub825\uc740 \ubd81\ucabd\uc73c\ub85c \ub3c4\ub9dd\uce58\uba70 \ud55c\uc911 \uad6d\uacbd \uc9c0\ub300\uc758 \uc0b0\uc545 \uc9c0\uc5ed\uc5d0\uc11c \ud53c\ub09c\ucc98\ub97c \ucc3e\uace0 \uc788\uc5c8\ub2e4. \uc870\uc120\uc778\ubbfc\uad70\uc758 \uada4\uba78\uc740 \uc911\ud654\uc778\ubbfc\uacf5\ud654\uad6d\uc774 \ud55c\uad6d \uc804\uc7c1\uc5d0 \uc989\uac01\uc801\uc73c\ub85c \uac1c\uc785\ud558\uac1c \ub9cc\ub4e4\uc5c8\uace0 \uc911\uacf5\uad70\uc740 1950\ub144 11\uc6d4\ubd80\ud130 \uad6d\uacbd \uc9c0\uc5ed\uc758 \uc720\uc5d4\uad70\uc5d0 \ub9de\uc11c \ub300\uaddc\ubaa8 \uae30\uc2b5 \uacf5\uaca9\uc744 \uac1c\uc2dc\ud588\ub2e4. \uc81c2\ucc28 \uccad\ucc9c\uac15 \uc804\ud22c\uc640 \uc7a5\uc9c4\ud638 \uc804\ud22c\ub85c \uc720\uc5d4\uad70\uc740 1950\ub144 12\uc6d4 38\ub3c4\uc120 \uc77c\ub300\ub85c \ucca0\uc218\ud560 \uc218 \ubc16\uc5d0 \uc5c6\uc5c8\ub2e4. \ub3d9\ubd80 \uc804\uc120\uc5d0\uc11c \ubbf8\uad6d \uc81c10\uad70\ub2e8\uc740 \uc911\uacf5\uad70\uc758 \uae30\uc2b5 \uacf5\uc138 \ub3d9\uc548 \uc7a5\uc9c4\ud638 \uc778\uadfc\uc758 \ud765\ub0a8 \ud56d\uad6c \uc5d0 \ud3ec\uc704\ub418\uc5c8\uace0, 1950\ub144 12\uc6d4 24\uc77c \ud574\uc0c1\uc73c\ub85c \ubd81\ud55c \uc9c0\uc5ed\uc5d0\uc11c \ucca0\uc218\ud588\ub2e4. \ub300\ud55c\ubbfc\uad6d \uc721\uad70\uc740 \ubbf8\uad70\uc758 \ubd80\uc7ac\ub85c 38\uc120 \uc77c\ub300\uc758 \uc911\ubd80 \ubc0f \ub3d9\ubd80 \uc804\uc120\uc5d0 \uc704\uce58\ud55c \ubc29\uc5b4\uc9c4\uc9c0\ub4e4\uc744 \uc778\uacc4\ud560 \uc218 \ubc16\uc5d0 \uc5c6\uc5c8\uace0, \uc911\ubd80 \uc804\uc120 \uc778\uadfc\uc5d0 \uc704\uce58\ud55c \uc6d0\uc8fc\uc2dc\uc758 \uc911\uc694 \uad50\ucc28\ub85c\ub3c4 \ud3ec\ud568\ub418\uc5b4 \uc788\uc5c8\ub2e4. \uc720\uc5d4\uad70\uc758 \uac11\uc791\uc2a4\ub7ec\uc6b4 \ud328\ubc30\ub85c \uc778\ud574 \ubd81\ud55c\uad70\uc740 \uc9e7\uac8c\ub098\ub9c8 \ud734\uc2dd\uae30\uac04\uc744 \uac00\uc9c8 \uc218 \uc788\uc5c8\uace0 1950\ub144 \ub9d0 \ubd84\uc0b0\ub41c \ubd81\ud55c\uad70\uc740 \ubcd1\ub825\uc744 \ub2e4\uc2dc \ud68c\ubcf5\uc2dc\ucf30\ub2e4.", "sentence_answer": "\ub3d9\ubd80 \uc804\uc120\uc5d0\uc11c \ubbf8\uad6d \uc81c10\uad70\ub2e8\uc740 \uc911\uacf5\uad70\uc758 \uae30\uc2b5 \uacf5\uc138 \ub3d9\uc548 \uc7a5\uc9c4\ud638 \uc778\uadfc\uc758 \ud765\ub0a8 \ud56d\uad6c \uc5d0 \ud3ec\uc704\ub418\uc5c8\uace0, 1950\ub144 12\uc6d4 24\uc77c \ud574\uc0c1\uc73c\ub85c \ubd81\ud55c \uc9c0\uc5ed\uc5d0\uc11c \ucca0\uc218\ud588\ub2e4.", "paragraph_id": "6459762-0-0", "paragraph_question": "question: 1950\ub144 12\uc6d4 24\uc77c \ubbf8\uad6d 10\uad70\ub2e8\uc774 \ucca0\uc218\uc791\uc804\uc744 \ud3bc\uce5c \uacf3\uc740?, context: 1950\ub144 6\uc6d4 \ub300\ud55c\ubbfc\uad6d\uc5d0 \ub300\ud55c \uae30\uc2b5 \uacf5\uaca9 \uc774\ud6c4 \uc870\uc120\uc778\ubbfc\uad70\uc740 1950\ub144 9\uc6d4 \ub099\ub3d9\uac15 \uc804\ud22c\uc640 \uc778\ucc9c \uc0c1\ub959 \uc791\uc804\uc73c\ub85c \uad6d\uc81c \uc5f0\ud569\uad70\uc5d0 \uc758\ud574 \ubd84\uc1c4\ub418\uc5c8\uace0, \uc870\uc120\uc778\ubbfc\uad70\uc758 \uc794\uc874 \ubcd1\ub825\uc740 \ubd81\ucabd\uc73c\ub85c \ub3c4\ub9dd\uce58\uba70 \ud55c\uc911 \uad6d\uacbd \uc9c0\ub300\uc758 \uc0b0\uc545 \uc9c0\uc5ed\uc5d0\uc11c \ud53c\ub09c\ucc98\ub97c \ucc3e\uace0 \uc788\uc5c8\ub2e4. \uc870\uc120\uc778\ubbfc\uad70\uc758 \uada4\uba78\uc740 \uc911\ud654\uc778\ubbfc\uacf5\ud654\uad6d\uc774 \ud55c\uad6d \uc804\uc7c1\uc5d0 \uc989\uac01\uc801\uc73c\ub85c \uac1c\uc785\ud558\uac1c \ub9cc\ub4e4\uc5c8\uace0 \uc911\uacf5\uad70\uc740 1950\ub144 11\uc6d4\ubd80\ud130 \uad6d\uacbd \uc9c0\uc5ed\uc758 \uc720\uc5d4\uad70\uc5d0 \ub9de\uc11c \ub300\uaddc\ubaa8 \uae30\uc2b5 \uacf5\uaca9\uc744 \uac1c\uc2dc\ud588\ub2e4. \uc81c2\ucc28 \uccad\ucc9c\uac15 \uc804\ud22c\uc640 \uc7a5\uc9c4\ud638 \uc804\ud22c\ub85c \uc720\uc5d4\uad70\uc740 1950\ub144 12\uc6d4 38\ub3c4\uc120 \uc77c\ub300\ub85c \ucca0\uc218\ud560 \uc218 \ubc16\uc5d0 \uc5c6\uc5c8\ub2e4. \ub3d9\ubd80 \uc804\uc120\uc5d0\uc11c \ubbf8\uad6d \uc81c10\uad70\ub2e8\uc740 \uc911\uacf5\uad70\uc758 \uae30\uc2b5 \uacf5\uc138 \ub3d9\uc548 \uc7a5\uc9c4\ud638 \uc778\uadfc\uc758 \ud765\ub0a8 \ud56d\uad6c\uc5d0 \ud3ec\uc704\ub418\uc5c8\uace0, 1950\ub144 12\uc6d4 24\uc77c \ud574\uc0c1\uc73c\ub85c \ubd81\ud55c \uc9c0\uc5ed\uc5d0\uc11c \ucca0\uc218\ud588\ub2e4. \ub300\ud55c\ubbfc\uad6d \uc721\uad70\uc740 \ubbf8\uad70\uc758 \ubd80\uc7ac\ub85c 38\uc120 \uc77c\ub300\uc758 \uc911\ubd80 \ubc0f \ub3d9\ubd80 \uc804\uc120\uc5d0 \uc704\uce58\ud55c \ubc29\uc5b4\uc9c4\uc9c0\ub4e4\uc744 \uc778\uacc4\ud560 \uc218 \ubc16\uc5d0 \uc5c6\uc5c8\uace0, \uc911\ubd80 \uc804\uc120 \uc778\uadfc\uc5d0 \uc704\uce58\ud55c \uc6d0\uc8fc\uc2dc\uc758 \uc911\uc694 \uad50\ucc28\ub85c\ub3c4 \ud3ec\ud568\ub418\uc5b4 \uc788\uc5c8\ub2e4. \uc720\uc5d4\uad70\uc758 \uac11\uc791\uc2a4\ub7ec\uc6b4 \ud328\ubc30\ub85c \uc778\ud574 \ubd81\ud55c\uad70\uc740 \uc9e7\uac8c\ub098\ub9c8 \ud734\uc2dd\uae30\uac04\uc744 \uac00\uc9c8 \uc218 \uc788\uc5c8\uace0 1950\ub144 \ub9d0 \ubd84\uc0b0\ub41c \ubd81\ud55c\uad70\uc740 \ubcd1\ub825\uc744 \ub2e4\uc2dc \ud68c\ubcf5\uc2dc\ucf30\ub2e4."} {"question": "\uc720\ub7fd\uc758 \uad50\uc721 \ubc29\uc2dd\uc774 \uc911\uc2ec\uc73c\ub85c \ud558\ub294 \uac83\uc740?", "paragraph": "\uc2e0\ubb38\ubc29\uc1a1\ud559\uc740 \uc2e0\ubb38\uacfc \ubc29\uc1a1\uc744 \ub300\ud45c\ub85c \ud558\ub294 \ub300\uc911 \ub9e4\uccb4\uc5d0\uc11c \uc77c\uc5b4\ub098\ub294 \ub9e4\uc2a4 \ucee4\ubba4\ub2c8\ucf00\uc774\uc158\uc744 \uc5f0\uad6c\ud558\ub294 \ud559\ubb38\uc774\ub2e4. \ud604\uc7ac\ub294 \uc2e0\ubb38\uacfc \ubc29\uc1a1 \ubd84\uc57c\uc5d0 \uad6d\ud55c\ub418\uc9c0 \uc54a\uace0 \uc804\ubc18\uc801\uc778 \uc0ac\ud68c\uacfc\ud559\uacfc \ubbf8\ub514\uc5b4 \uad00\ub828 \uc2e4\ubb34 \uad50\uc721\uc744 \ud568\uaed8 \ub2f4\ub2f9\ud558\uace0 \uc788\ub2e4. \ub300\ud55c\ubbfc\uad6d\uc758 \ub2f9\ucd08 \uc2e0\ubb38\ubc29\uc1a1\ud559\uc758 \ucd9c\ubc1c\uc740 \ubbf8\uad6d \uad50\uc721\ubc29\uc2dd\uc758 \uc778\uc2dd\uc5d0\uc11c \ube44\ub86f\ub410\ub2e4. \uc720\ub7fd\uc758 \uacbd\uc6b0 \uc8fc\ub85c \uc9c1\uc5c5 \uad50\uc721 \uc911\uc2ec\uc778\ub370 \ubc18\ud574\uc11c \ubbf8\uad6d\uc758 \uacbd\uc6b0 \uc9c1\uc5c5 \uad50\uc721 + \uad50\uc591\ud559\uc2b5\uc774\ub77c\ub294 \ud1a1\ud2b9\ud55c \uad50\uc721\uccb4\uc81c\ub97c \ubc1c\ub2ec\uc2dc\ucf30\ub2e4. \ud64d\uc775\ub300\ud559\uad50\ub97c \uc2dc\uc791\uc73c\ub85c \uc774\ud654\uc5ec\uc790\ub300\ud559\uad50, \uc911\uc559\ub300\ud559\uad50, \ud55c\uc591\ub300\ud559\uad50, \uc131\uade0\uad00\ub300\ud559\uad50, \uace0\ub824\ub300\ud559\uad50, \uc11c\uc6b8\ub300\ud559\uad50 \ub4f1\uc5d0\uc11c \uc2e0\ubb38\ud559\uacfc\ub97c \ub9cc\ub4e4\uace0 \uac00\ub974\uce58\uae30 \uc2dc\uc791\ud588\ub2e4. \ub2f9\uc2dc \ucd08\uae30\uc758 \uad50\uc218\uc9c4\ub4e4\uc740 \uae30\uc790 \uc0dd\ud65c\uc744 \ud558\ub2e4\uac00 \ubbf8\uad6d \uc720\ud559 \uc911\uc5d0 \uc800\ub110\ub9ac\uc998 \uad50\uc721\uc744 \ubc1b\uace0 \uc628 \uacbd\uc6b0\uac00 \ub9ce\uc558\ub2e4.", "answer": "\uc9c1\uc5c5 \uad50\uc721", "sentence": "\uc720\ub7fd\uc758 \uacbd\uc6b0 \uc8fc\ub85c \uc9c1\uc5c5 \uad50\uc721 \uc911\uc2ec\uc778\ub370 \ubc18\ud574\uc11c \ubbf8\uad6d\uc758 \uacbd\uc6b0 \uc9c1\uc5c5 \uad50\uc721 + \uad50\uc591\ud559\uc2b5\uc774\ub77c\ub294 \ud1a1\ud2b9\ud55c \uad50\uc721\uccb4\uc81c\ub97c \ubc1c\ub2ec\uc2dc\ucf30\ub2e4.", "paragraph_sentence": "\uc2e0\ubb38\ubc29\uc1a1\ud559\uc740 \uc2e0\ubb38\uacfc \ubc29\uc1a1\uc744 \ub300\ud45c\ub85c \ud558\ub294 \ub300\uc911 \ub9e4\uccb4\uc5d0\uc11c \uc77c\uc5b4\ub098\ub294 \ub9e4\uc2a4 \ucee4\ubba4\ub2c8\ucf00\uc774\uc158\uc744 \uc5f0\uad6c\ud558\ub294 \ud559\ubb38\uc774\ub2e4. \ud604\uc7ac\ub294 \uc2e0\ubb38\uacfc \ubc29\uc1a1 \ubd84\uc57c\uc5d0 \uad6d\ud55c\ub418\uc9c0 \uc54a\uace0 \uc804\ubc18\uc801\uc778 \uc0ac\ud68c\uacfc\ud559\uacfc \ubbf8\ub514\uc5b4 \uad00\ub828 \uc2e4\ubb34 \uad50\uc721\uc744 \ud568\uaed8 \ub2f4\ub2f9\ud558\uace0 \uc788\ub2e4. \ub300\ud55c\ubbfc\uad6d\uc758 \ub2f9\ucd08 \uc2e0\ubb38\ubc29\uc1a1\ud559\uc758 \ucd9c\ubc1c\uc740 \ubbf8\uad6d \uad50\uc721\ubc29\uc2dd\uc758 \uc778\uc2dd\uc5d0\uc11c \ube44\ub86f\ub410\ub2e4. \uc720\ub7fd\uc758 \uacbd\uc6b0 \uc8fc\ub85c \uc9c1\uc5c5 \uad50\uc721 \uc911\uc2ec\uc778\ub370 \ubc18\ud574\uc11c \ubbf8\uad6d\uc758 \uacbd\uc6b0 \uc9c1\uc5c5 \uad50\uc721 + \uad50\uc591\ud559\uc2b5\uc774\ub77c\ub294 \ud1a1\ud2b9\ud55c \uad50\uc721\uccb4\uc81c\ub97c \ubc1c\ub2ec\uc2dc\ucf30\ub2e4. \ud64d\uc775\ub300\ud559\uad50\ub97c \uc2dc\uc791\uc73c\ub85c \uc774\ud654\uc5ec\uc790\ub300\ud559\uad50, \uc911\uc559\ub300\ud559\uad50, \ud55c\uc591\ub300\ud559\uad50, \uc131\uade0\uad00\ub300\ud559\uad50, \uace0\ub824\ub300\ud559\uad50, \uc11c\uc6b8\ub300\ud559\uad50 \ub4f1\uc5d0\uc11c \uc2e0\ubb38\ud559\uacfc\ub97c \ub9cc\ub4e4\uace0 \uac00\ub974\uce58\uae30 \uc2dc\uc791\ud588\ub2e4. \ub2f9\uc2dc \ucd08\uae30\uc758 \uad50\uc218\uc9c4\ub4e4\uc740 \uae30\uc790 \uc0dd\ud65c\uc744 \ud558\ub2e4\uac00 \ubbf8\uad6d \uc720\ud559 \uc911\uc5d0 \uc800\ub110\ub9ac\uc998 \uad50\uc721\uc744 \ubc1b\uace0 \uc628 \uacbd\uc6b0\uac00 \ub9ce\uc558\ub2e4.", "paragraph_answer": "\uc2e0\ubb38\ubc29\uc1a1\ud559\uc740 \uc2e0\ubb38\uacfc \ubc29\uc1a1\uc744 \ub300\ud45c\ub85c \ud558\ub294 \ub300\uc911 \ub9e4\uccb4\uc5d0\uc11c \uc77c\uc5b4\ub098\ub294 \ub9e4\uc2a4 \ucee4\ubba4\ub2c8\ucf00\uc774\uc158\uc744 \uc5f0\uad6c\ud558\ub294 \ud559\ubb38\uc774\ub2e4. \ud604\uc7ac\ub294 \uc2e0\ubb38\uacfc \ubc29\uc1a1 \ubd84\uc57c\uc5d0 \uad6d\ud55c\ub418\uc9c0 \uc54a\uace0 \uc804\ubc18\uc801\uc778 \uc0ac\ud68c\uacfc\ud559\uacfc \ubbf8\ub514\uc5b4 \uad00\ub828 \uc2e4\ubb34 \uad50\uc721\uc744 \ud568\uaed8 \ub2f4\ub2f9\ud558\uace0 \uc788\ub2e4. \ub300\ud55c\ubbfc\uad6d\uc758 \ub2f9\ucd08 \uc2e0\ubb38\ubc29\uc1a1\ud559\uc758 \ucd9c\ubc1c\uc740 \ubbf8\uad6d \uad50\uc721\ubc29\uc2dd\uc758 \uc778\uc2dd\uc5d0\uc11c \ube44\ub86f\ub410\ub2e4. \uc720\ub7fd\uc758 \uacbd\uc6b0 \uc8fc\ub85c \uc9c1\uc5c5 \uad50\uc721 \uc911\uc2ec\uc778\ub370 \ubc18\ud574\uc11c \ubbf8\uad6d\uc758 \uacbd\uc6b0 \uc9c1\uc5c5 \uad50\uc721 + \uad50\uc591\ud559\uc2b5\uc774\ub77c\ub294 \ud1a1\ud2b9\ud55c \uad50\uc721\uccb4\uc81c\ub97c \ubc1c\ub2ec\uc2dc\ucf30\ub2e4. \ud64d\uc775\ub300\ud559\uad50\ub97c \uc2dc\uc791\uc73c\ub85c \uc774\ud654\uc5ec\uc790\ub300\ud559\uad50, \uc911\uc559\ub300\ud559\uad50, \ud55c\uc591\ub300\ud559\uad50, \uc131\uade0\uad00\ub300\ud559\uad50, \uace0\ub824\ub300\ud559\uad50, \uc11c\uc6b8\ub300\ud559\uad50 \ub4f1\uc5d0\uc11c \uc2e0\ubb38\ud559\uacfc\ub97c \ub9cc\ub4e4\uace0 \uac00\ub974\uce58\uae30 \uc2dc\uc791\ud588\ub2e4. \ub2f9\uc2dc \ucd08\uae30\uc758 \uad50\uc218\uc9c4\ub4e4\uc740 \uae30\uc790 \uc0dd\ud65c\uc744 \ud558\ub2e4\uac00 \ubbf8\uad6d \uc720\ud559 \uc911\uc5d0 \uc800\ub110\ub9ac\uc998 \uad50\uc721\uc744 \ubc1b\uace0 \uc628 \uacbd\uc6b0\uac00 \ub9ce\uc558\ub2e4.", "sentence_answer": "\uc720\ub7fd\uc758 \uacbd\uc6b0 \uc8fc\ub85c \uc9c1\uc5c5 \uad50\uc721 \uc911\uc2ec\uc778\ub370 \ubc18\ud574\uc11c \ubbf8\uad6d\uc758 \uacbd\uc6b0 \uc9c1\uc5c5 \uad50\uc721 + \uad50\uc591\ud559\uc2b5\uc774\ub77c\ub294 \ud1a1\ud2b9\ud55c \uad50\uc721\uccb4\uc81c\ub97c \ubc1c\ub2ec\uc2dc\ucf30\ub2e4.", "paragraph_id": "6604284-0-1", "paragraph_question": "question: \uc720\ub7fd\uc758 \uad50\uc721 \ubc29\uc2dd\uc774 \uc911\uc2ec\uc73c\ub85c \ud558\ub294 \uac83\uc740?, context: \uc2e0\ubb38\ubc29\uc1a1\ud559\uc740 \uc2e0\ubb38\uacfc \ubc29\uc1a1\uc744 \ub300\ud45c\ub85c \ud558\ub294 \ub300\uc911 \ub9e4\uccb4\uc5d0\uc11c \uc77c\uc5b4\ub098\ub294 \ub9e4\uc2a4 \ucee4\ubba4\ub2c8\ucf00\uc774\uc158\uc744 \uc5f0\uad6c\ud558\ub294 \ud559\ubb38\uc774\ub2e4. \ud604\uc7ac\ub294 \uc2e0\ubb38\uacfc \ubc29\uc1a1 \ubd84\uc57c\uc5d0 \uad6d\ud55c\ub418\uc9c0 \uc54a\uace0 \uc804\ubc18\uc801\uc778 \uc0ac\ud68c\uacfc\ud559\uacfc \ubbf8\ub514\uc5b4 \uad00\ub828 \uc2e4\ubb34 \uad50\uc721\uc744 \ud568\uaed8 \ub2f4\ub2f9\ud558\uace0 \uc788\ub2e4. \ub300\ud55c\ubbfc\uad6d\uc758 \ub2f9\ucd08 \uc2e0\ubb38\ubc29\uc1a1\ud559\uc758 \ucd9c\ubc1c\uc740 \ubbf8\uad6d \uad50\uc721\ubc29\uc2dd\uc758 \uc778\uc2dd\uc5d0\uc11c \ube44\ub86f\ub410\ub2e4. \uc720\ub7fd\uc758 \uacbd\uc6b0 \uc8fc\ub85c \uc9c1\uc5c5 \uad50\uc721 \uc911\uc2ec\uc778\ub370 \ubc18\ud574\uc11c \ubbf8\uad6d\uc758 \uacbd\uc6b0 \uc9c1\uc5c5 \uad50\uc721 + \uad50\uc591\ud559\uc2b5\uc774\ub77c\ub294 \ud1a1\ud2b9\ud55c \uad50\uc721\uccb4\uc81c\ub97c \ubc1c\ub2ec\uc2dc\ucf30\ub2e4. \ud64d\uc775\ub300\ud559\uad50\ub97c \uc2dc\uc791\uc73c\ub85c \uc774\ud654\uc5ec\uc790\ub300\ud559\uad50, \uc911\uc559\ub300\ud559\uad50, \ud55c\uc591\ub300\ud559\uad50, \uc131\uade0\uad00\ub300\ud559\uad50, \uace0\ub824\ub300\ud559\uad50, \uc11c\uc6b8\ub300\ud559\uad50 \ub4f1\uc5d0\uc11c \uc2e0\ubb38\ud559\uacfc\ub97c \ub9cc\ub4e4\uace0 \uac00\ub974\uce58\uae30 \uc2dc\uc791\ud588\ub2e4. \ub2f9\uc2dc \ucd08\uae30\uc758 \uad50\uc218\uc9c4\ub4e4\uc740 \uae30\uc790 \uc0dd\ud65c\uc744 \ud558\ub2e4\uac00 \ubbf8\uad6d \uc720\ud559 \uc911\uc5d0 \uc800\ub110\ub9ac\uc998 \uad50\uc721\uc744 \ubc1b\uace0 \uc628 \uacbd\uc6b0\uac00 \ub9ce\uc558\ub2e4."} {"question": "\uad6d\ubc29\ubd80\uc640 \ud658\uacbd\ubd80\uac00 \uc131\uc8fc\uad70\uacfc \uae40\ucc9c\uc2dc \uad00\uacc4\uc790\uc640 \uae30\uc790\ub4e4\uc744 \ucc38\uad00 \uc18d\uc5d0 \ud658\uacbd\uc601\ud5a5\ud3c9\uac00 \uacb0\uacfc\ub97c \ubc1c\ud45c\ud55c \uac83\uc740 \uc5b8\uc81c\uc778\uac00?", "paragraph": "\ud558\uc9c0\ub9cc 6\uc6d4 9\uc77c \uc815\uc758\uc6a9 \uad6d\uac00\uc548\ubcf4\uc2e4\uc7a5\uc774 \"\uc815\uad8c\uc774 \uad50\uccb4\ub418\uc5c8\ub2e4\uace0 \ud574\uc11c \uc774 \uacb0\uc815\uc744 \uacb0\ucf54 \uac00\ubccd\uac8c \uc5ec\uae30\uc9c0 \uc54a\uc744 \uac83\uc774\ub2e4. \ub610 \ubbf8\uad6d\uacfc \uacc4\uc18d \uae34\ubc00\ud788 \ud611\uc758\ud574 \ub098\uac08 \uac83\uc774\ub2e4\"\ub294 \uacf5\uc2dd \uc785\uc7a5\uc744 \ub0b4\ub193\uc558\ub294\ub370 \uc774\ub294 \uc774\ub2ec \ub9d0\uc5d0 \uc608\uc815\ub41c \ud55c\ubbf8\uc815\uc0c1\ud68c\ub2f4 \uc900\ube44\ub97c \uc6d0\ud65c\ud788 \uc870\uc728\ud558\uace0 \ud55c\ubbf8 \ub3d9\ub9f9\uc744 \ud30c\uae30\ud560 \uc758\ub3c4\uac00 \uc5c6\ub2e4\ub294 \uc810\uc744 \ubd84\uba85\ud788 \ud558\uae30 \uc704\ud55c \uac83\uc73c\ub85c \ubcf4\uc778\ub2e4. \ud558\uc9c0\ub9cc \uad6d\uc775\uacfc \uc548\ubcf4\ub97c \ucd5c\uc6b0\uc120\uc73c\ub85c \uace0\ub824\ud558\uaca0\ub2e4\uba74\uc11c \"\ud569\ub9ac\uc801\uc774\uace0 \ub610 \ud569\ubc95\uc801\uc778 \ubc29\ubc95\uc73c\ub85c \ud22c\uba85\ud558\uac8c \uc9c4\ud589\ud574 \ub098\uac08 \uac83\uc774\"\ub77c\uace0 \ud558\uba74\uc11c \ubbfc\uc8fc\uc801\u00b7\uc808\ucc28\uc801 \uc815\ub2f9\uc131\uacfc \ud22c\uba85\uc131 \ud655\ubcf4\ub97c \uc704\ud574 \ud544\uc694\ud55c \uc808\ucc28\ub3c4 \ubc1f\uaca0\ub2e4\ub294 \uc758\uc0ac\ub97c \ub0b4\ube44\ucce4\ub2e4. 29\uc77c \ubbf8\uad6d\uc744 \ubc29\ubb38\ud574\uc11c\ub294 \ubb38\uc7ac\uc778\uc774 \uc9c1\uc811 \"\uc0ac\ub4dc\ub294 \ud55c\ubbf8\ub3d9\ub9f9\uc5d0 \uae30\ucd08\ud55c \ud569\uc758\uc774\uace0 \ud55c\uad6d\ubbfc\uacfc \uc8fc\ud55c\ubbf8\uad70\uc758 \uc0dd\uba85\uc744 \ubcf4\ud638\ud558\uae30 \uc704\ud55c \uac83\"\uc774\ub77c\uba70 \uc0ac\ub4dc \ubc30\uce58\ub97c \ucca0\ud68c\ud558\uc9c0 \uc54a\uaca0\ub2e4\ub294 \uc785\uc7a5\uc744 \ubd84\uba85\ud788 \ubc1d\ud614\ub2e4. \ub9e5 \uc190\ubca0\ub9ac \ud558\uc6d0 \uad70\uc0ac\uc704\uc6d0\uc7a5\uc740 \"\ubd81\ud55c\uc5d0\ub294 \ud55c\u00b7\ubbf8 \uac04 \uc774\uacac\uc774 \uc5c6\ub2e4\ub294 \uac83\uacfc \uad70\uc0ac\uc801\uc73c\ub85c \uacac\uace0\ud558\ub2e4\ub294 \uac83\uc744 \ubcf4\uc5ec\uc8fc\ub294 \uac83\uc774 \uc911\uc694\ud558\ub2e4\"\uba70 \uc0ac\ub4dc \uad00\ub828 \ud655\uc778\uc5d0 \uac10\uc0ac\uc758 \uc758\ubbf8\ub97c \uc804\ud588\ub2e4. 8\uc6d4 12\uc77c\uc5d0\ub294 \uad6d\ubc29\ubd80\uc640 \ud658\uacbd\ubd80\uac00 \uc131\uc8fc\uad70\u00b7\uae40\ucc9c\uc2dc \uad00\uacc4\uc790\uc640 \uae30\uc790\ub4e4\uc758 \ucc38\uad00 \uc18d\uc5d0 \uc0ac\ub4dc \ub808\uc774\ub354\ub85c\ubd80\ud130 100m, 500m, 600m, 700m \uac70\ub9ac\uc5d0\uc11c \uc804\uc790\ud30c\uc640 \uc18c\uc74c\uc744 \uce21\uc815\ud55c \uacb0\uacfc \"\ud734\ub300\uc804\ud654\uac00 \uae30\uc9c0\uad6d\uc744 \ucc3e\uc744 \ub54c \ub098\uc624\ub294 \uc804\uc790\ud30c\ubcf4\ub2e4 \uc791\uc740 \uc218\uc900\uc774\uace0, \uc18c\uc74c\ub3c4 \ub300\ud654\ud560 \ub54c \ub098\uc624\ub294 \uc18c\ub9ac \uc815\ub3c4\"\uac00 \ub098\uc654\ub2e4\uace0 \ud658\uacbd\uc601\ud5a5\ud3c9\uac00\uc758 \uacb0\uacfc\ub97c \ubc1c\ud45c\ud588\ub2e4. \uc804\uc790\ud30c \uc218\uce58\uac00 \ucd5c\uace0 0.01947W/m\uac00 \ub098\uc640 \uae30\uc900\uc774 \uc774\ud558\uac00 \ub098\uc628 \uac83\uc774\ub2e4. \uc774\ud6c4 \uad6d\ubc29\ubd80\ub294 \uc774 \uacb0\uacfc\ub97c \ubc14\ud0d5\uc73c\ub85c \uc8fc\ubbfc\ub4e4\uc744 \uacc4\uc18d \uc124\ub4dd\ud558\uaca0\ub2e4\uace0 \ud588\uc9c0\ub9cc \uc0ac\ub4dc \ubc30\uce58 \ucca0\ud68c\ub97c \uc8fc\uc7a5\ud558\ub294 \uc8fc\ubbfc\ub4e4\uc740 \"\uad6c\uccb4\uc801\uc778 \uce21\uc815 \ubc29\uc2dd\uc744 \uacf5\uac1c\ud558\uc9c0 \uc54a\uc740 \ucc44 \ub098\uc628 \uacb0\uacfc\ub97c \uc5b4\ub5bb\uac8c \ubbff\uc744 \uc218 \uc788\ub290\ub0d0\", \"\ud3c9\uac00\ub2e8\uc5d0 \uc8fc\ubbfc\uc774 \ucd94\ucc9c\ud558\ub294 \uc804\ubb38\uac00\ub97c \ud3ec\ud568\uc2dc\ucf1c\uc904 \uac83\uc744 \uc694\uad6c\ud588\uc9c0\ub9cc \ubc1b\uc544\ub4e4\uc5ec\uc9c0\uc9c0 \uc54a\uc558\ub2e4\"\uba70 \uc5ec\uc804\ud788 \ubc18\ubc1c\uc744 \uc774\uc5b4\uac14\ub2e4. \ud55c\ud3b8, \uc774\ub7ec\ud55c \uacb0\uacfc\uc5d0 \uc57c\ub2f9\ub4e4\uc740 \ud55c\ubaa9\uc18c\ub9ac\ub85c \ucd94\uac00 \uc0ac\ub4dc \ubc1c\uc0ac\ub300 \ub4f1\uc758 \uc870\uc18d\ud55c \ubc30\uce58 \uc644\ub8cc\ub97c \ucd09\uad6c\ud558\uba74\uc11c \ubd88\ud544\uc694\ud55c \uc0ac\ud68c \uac08\ub4f1\uacfc \uad6d\ub860 \ubd84\uc5f4\uc744 \uc77c\uc73c\ud0a8 \uac83\uc5d0 \ub300\ud55c \uc0ac\uacfc\ub97c \uc694\uad6c\ud588\ub2e4. \ubc18\uba74, \ubbfc\uc8fc\ub2f9\uc740 \uc8fc\ubbfc \uc758\uacac\uc774 \uc218\ub834\ub41c \uc77c\ubc18 \ud658\uacbd\uc601\ud5a5\ud3c9\uac00\ub97c \uac70\uccd0\uc57c \ud55c\ub2e4\ub294 \uc8fc\uc7a5\uc744 \ub0c8\ub2e4.", "answer": "8\uc6d4 12\uc77c", "sentence": "8\uc6d4 12\uc77c \uc5d0\ub294 \uad6d\ubc29\ubd80\uc640 \ud658\uacbd\ubd80\uac00 \uc131\uc8fc\uad70\u00b7\uae40\ucc9c\uc2dc \uad00\uacc4\uc790\uc640 \uae30\uc790\ub4e4\uc758 \ucc38\uad00 \uc18d\uc5d0 \uc0ac\ub4dc \ub808\uc774\ub354\ub85c\ubd80\ud130 100m, 500m, 600m, 700m \uac70\ub9ac\uc5d0\uc11c \uc804\uc790\ud30c\uc640 \uc18c\uc74c\uc744 \uce21\uc815\ud55c \uacb0\uacfc \"\ud734\ub300\uc804\ud654\uac00 \uae30\uc9c0\uad6d\uc744 \ucc3e\uc744 \ub54c \ub098\uc624\ub294 \uc804\uc790\ud30c\ubcf4\ub2e4 \uc791\uc740 \uc218\uc900\uc774\uace0, \uc18c\uc74c\ub3c4 \ub300\ud654\ud560 \ub54c \ub098\uc624\ub294 \uc18c\ub9ac \uc815\ub3c4\"\uac00 \ub098\uc654\ub2e4\uace0 \ud658\uacbd\uc601\ud5a5\ud3c9\uac00\uc758 \uacb0\uacfc\ub97c \ubc1c\ud45c\ud588\ub2e4.", "paragraph_sentence": "\ud558\uc9c0\ub9cc 6\uc6d4 9\uc77c \uc815\uc758\uc6a9 \uad6d\uac00\uc548\ubcf4\uc2e4\uc7a5\uc774 \"\uc815\uad8c\uc774 \uad50\uccb4\ub418\uc5c8\ub2e4\uace0 \ud574\uc11c \uc774 \uacb0\uc815\uc744 \uacb0\ucf54 \uac00\ubccd\uac8c \uc5ec\uae30\uc9c0 \uc54a\uc744 \uac83\uc774\ub2e4. \ub610 \ubbf8\uad6d\uacfc \uacc4\uc18d \uae34\ubc00\ud788 \ud611\uc758\ud574 \ub098\uac08 \uac83\uc774\ub2e4\"\ub294 \uacf5\uc2dd \uc785\uc7a5\uc744 \ub0b4\ub193\uc558\ub294\ub370 \uc774\ub294 \uc774\ub2ec \ub9d0\uc5d0 \uc608\uc815\ub41c \ud55c\ubbf8\uc815\uc0c1\ud68c\ub2f4 \uc900\ube44\ub97c \uc6d0\ud65c\ud788 \uc870\uc728\ud558\uace0 \ud55c\ubbf8 \ub3d9\ub9f9\uc744 \ud30c\uae30\ud560 \uc758\ub3c4\uac00 \uc5c6\ub2e4\ub294 \uc810\uc744 \ubd84\uba85\ud788 \ud558\uae30 \uc704\ud55c \uac83\uc73c\ub85c \ubcf4\uc778\ub2e4. \ud558\uc9c0\ub9cc \uad6d\uc775\uacfc \uc548\ubcf4\ub97c \ucd5c\uc6b0\uc120\uc73c\ub85c \uace0\ub824\ud558\uaca0\ub2e4\uba74\uc11c \"\ud569\ub9ac\uc801\uc774\uace0 \ub610 \ud569\ubc95\uc801\uc778 \ubc29\ubc95\uc73c\ub85c \ud22c\uba85\ud558\uac8c \uc9c4\ud589\ud574 \ub098\uac08 \uac83\uc774\"\ub77c\uace0 \ud558\uba74\uc11c \ubbfc\uc8fc\uc801\u00b7\uc808\ucc28\uc801 \uc815\ub2f9\uc131\uacfc \ud22c\uba85\uc131 \ud655\ubcf4\ub97c \uc704\ud574 \ud544\uc694\ud55c \uc808\ucc28\ub3c4 \ubc1f\uaca0\ub2e4\ub294 \uc758\uc0ac\ub97c \ub0b4\ube44\ucce4\ub2e4. 29\uc77c \ubbf8\uad6d\uc744 \ubc29\ubb38\ud574\uc11c\ub294 \ubb38\uc7ac\uc778\uc774 \uc9c1\uc811 \"\uc0ac\ub4dc\ub294 \ud55c\ubbf8\ub3d9\ub9f9\uc5d0 \uae30\ucd08\ud55c \ud569\uc758\uc774\uace0 \ud55c\uad6d\ubbfc\uacfc \uc8fc\ud55c\ubbf8\uad70\uc758 \uc0dd\uba85\uc744 \ubcf4\ud638\ud558\uae30 \uc704\ud55c \uac83\"\uc774\ub77c\uba70 \uc0ac\ub4dc \ubc30\uce58\ub97c \ucca0\ud68c\ud558\uc9c0 \uc54a\uaca0\ub2e4\ub294 \uc785\uc7a5\uc744 \ubd84\uba85\ud788 \ubc1d\ud614\ub2e4. \ub9e5 \uc190\ubca0\ub9ac \ud558\uc6d0 \uad70\uc0ac\uc704\uc6d0\uc7a5\uc740 \"\ubd81\ud55c\uc5d0\ub294 \ud55c\u00b7\ubbf8 \uac04 \uc774\uacac\uc774 \uc5c6\ub2e4\ub294 \uac83\uacfc \uad70\uc0ac\uc801\uc73c\ub85c \uacac\uace0\ud558\ub2e4\ub294 \uac83\uc744 \ubcf4\uc5ec\uc8fc\ub294 \uac83\uc774 \uc911\uc694\ud558\ub2e4\"\uba70 \uc0ac\ub4dc \uad00\ub828 \ud655\uc778\uc5d0 \uac10\uc0ac\uc758 \uc758\ubbf8\ub97c \uc804\ud588\ub2e4. 8\uc6d4 12\uc77c \uc5d0\ub294 \uad6d\ubc29\ubd80\uc640 \ud658\uacbd\ubd80\uac00 \uc131\uc8fc\uad70\u00b7\uae40\ucc9c\uc2dc \uad00\uacc4\uc790\uc640 \uae30\uc790\ub4e4\uc758 \ucc38\uad00 \uc18d\uc5d0 \uc0ac\ub4dc \ub808\uc774\ub354\ub85c\ubd80\ud130 100m, 500m, 600m, 700m \uac70\ub9ac\uc5d0\uc11c \uc804\uc790\ud30c\uc640 \uc18c\uc74c\uc744 \uce21\uc815\ud55c \uacb0\uacfc \"\ud734\ub300\uc804\ud654\uac00 \uae30\uc9c0\uad6d\uc744 \ucc3e\uc744 \ub54c \ub098\uc624\ub294 \uc804\uc790\ud30c\ubcf4\ub2e4 \uc791\uc740 \uc218\uc900\uc774\uace0, \uc18c\uc74c\ub3c4 \ub300\ud654\ud560 \ub54c \ub098\uc624\ub294 \uc18c\ub9ac \uc815\ub3c4\"\uac00 \ub098\uc654\ub2e4\uace0 \ud658\uacbd\uc601\ud5a5\ud3c9\uac00\uc758 \uacb0\uacfc\ub97c \ubc1c\ud45c\ud588\ub2e4. \uc804\uc790\ud30c \uc218\uce58\uac00 \ucd5c\uace0 0.01947W/m\uac00 \ub098\uc640 \uae30\uc900\uc774 \uc774\ud558\uac00 \ub098\uc628 \uac83\uc774\ub2e4. \uc774\ud6c4 \uad6d\ubc29\ubd80\ub294 \uc774 \uacb0\uacfc\ub97c \ubc14\ud0d5\uc73c\ub85c \uc8fc\ubbfc\ub4e4\uc744 \uacc4\uc18d \uc124\ub4dd\ud558\uaca0\ub2e4\uace0 \ud588\uc9c0\ub9cc \uc0ac\ub4dc \ubc30\uce58 \ucca0\ud68c\ub97c \uc8fc\uc7a5\ud558\ub294 \uc8fc\ubbfc\ub4e4\uc740 \"\uad6c\uccb4\uc801\uc778 \uce21\uc815 \ubc29\uc2dd\uc744 \uacf5\uac1c\ud558\uc9c0 \uc54a\uc740 \ucc44 \ub098\uc628 \uacb0\uacfc\ub97c \uc5b4\ub5bb\uac8c \ubbff\uc744 \uc218 \uc788\ub290\ub0d0\", \"\ud3c9\uac00\ub2e8\uc5d0 \uc8fc\ubbfc\uc774 \ucd94\ucc9c\ud558\ub294 \uc804\ubb38\uac00\ub97c \ud3ec\ud568\uc2dc\ucf1c\uc904 \uac83\uc744 \uc694\uad6c\ud588\uc9c0\ub9cc \ubc1b\uc544\ub4e4\uc5ec\uc9c0\uc9c0 \uc54a\uc558\ub2e4\"\uba70 \uc5ec\uc804\ud788 \ubc18\ubc1c\uc744 \uc774\uc5b4\uac14\ub2e4. \ud55c\ud3b8, \uc774\ub7ec\ud55c \uacb0\uacfc\uc5d0 \uc57c\ub2f9\ub4e4\uc740 \ud55c\ubaa9\uc18c\ub9ac\ub85c \ucd94\uac00 \uc0ac\ub4dc \ubc1c\uc0ac\ub300 \ub4f1\uc758 \uc870\uc18d\ud55c \ubc30\uce58 \uc644\ub8cc\ub97c \ucd09\uad6c\ud558\uba74\uc11c \ubd88\ud544\uc694\ud55c \uc0ac\ud68c \uac08\ub4f1\uacfc \uad6d\ub860 \ubd84\uc5f4\uc744 \uc77c\uc73c\ud0a8 \uac83\uc5d0 \ub300\ud55c \uc0ac\uacfc\ub97c \uc694\uad6c\ud588\ub2e4. \ubc18\uba74, \ubbfc\uc8fc\ub2f9\uc740 \uc8fc\ubbfc \uc758\uacac\uc774 \uc218\ub834\ub41c \uc77c\ubc18 \ud658\uacbd\uc601\ud5a5\ud3c9\uac00\ub97c \uac70\uccd0\uc57c \ud55c\ub2e4\ub294 \uc8fc\uc7a5\uc744 \ub0c8\ub2e4.", "paragraph_answer": "\ud558\uc9c0\ub9cc 6\uc6d4 9\uc77c \uc815\uc758\uc6a9 \uad6d\uac00\uc548\ubcf4\uc2e4\uc7a5\uc774 \"\uc815\uad8c\uc774 \uad50\uccb4\ub418\uc5c8\ub2e4\uace0 \ud574\uc11c \uc774 \uacb0\uc815\uc744 \uacb0\ucf54 \uac00\ubccd\uac8c \uc5ec\uae30\uc9c0 \uc54a\uc744 \uac83\uc774\ub2e4. \ub610 \ubbf8\uad6d\uacfc \uacc4\uc18d \uae34\ubc00\ud788 \ud611\uc758\ud574 \ub098\uac08 \uac83\uc774\ub2e4\"\ub294 \uacf5\uc2dd \uc785\uc7a5\uc744 \ub0b4\ub193\uc558\ub294\ub370 \uc774\ub294 \uc774\ub2ec \ub9d0\uc5d0 \uc608\uc815\ub41c \ud55c\ubbf8\uc815\uc0c1\ud68c\ub2f4 \uc900\ube44\ub97c \uc6d0\ud65c\ud788 \uc870\uc728\ud558\uace0 \ud55c\ubbf8 \ub3d9\ub9f9\uc744 \ud30c\uae30\ud560 \uc758\ub3c4\uac00 \uc5c6\ub2e4\ub294 \uc810\uc744 \ubd84\uba85\ud788 \ud558\uae30 \uc704\ud55c \uac83\uc73c\ub85c \ubcf4\uc778\ub2e4. \ud558\uc9c0\ub9cc \uad6d\uc775\uacfc \uc548\ubcf4\ub97c \ucd5c\uc6b0\uc120\uc73c\ub85c \uace0\ub824\ud558\uaca0\ub2e4\uba74\uc11c \"\ud569\ub9ac\uc801\uc774\uace0 \ub610 \ud569\ubc95\uc801\uc778 \ubc29\ubc95\uc73c\ub85c \ud22c\uba85\ud558\uac8c \uc9c4\ud589\ud574 \ub098\uac08 \uac83\uc774\"\ub77c\uace0 \ud558\uba74\uc11c \ubbfc\uc8fc\uc801\u00b7\uc808\ucc28\uc801 \uc815\ub2f9\uc131\uacfc \ud22c\uba85\uc131 \ud655\ubcf4\ub97c \uc704\ud574 \ud544\uc694\ud55c \uc808\ucc28\ub3c4 \ubc1f\uaca0\ub2e4\ub294 \uc758\uc0ac\ub97c \ub0b4\ube44\ucce4\ub2e4. 29\uc77c \ubbf8\uad6d\uc744 \ubc29\ubb38\ud574\uc11c\ub294 \ubb38\uc7ac\uc778\uc774 \uc9c1\uc811 \"\uc0ac\ub4dc\ub294 \ud55c\ubbf8\ub3d9\ub9f9\uc5d0 \uae30\ucd08\ud55c \ud569\uc758\uc774\uace0 \ud55c\uad6d\ubbfc\uacfc \uc8fc\ud55c\ubbf8\uad70\uc758 \uc0dd\uba85\uc744 \ubcf4\ud638\ud558\uae30 \uc704\ud55c \uac83\"\uc774\ub77c\uba70 \uc0ac\ub4dc \ubc30\uce58\ub97c \ucca0\ud68c\ud558\uc9c0 \uc54a\uaca0\ub2e4\ub294 \uc785\uc7a5\uc744 \ubd84\uba85\ud788 \ubc1d\ud614\ub2e4. \ub9e5 \uc190\ubca0\ub9ac \ud558\uc6d0 \uad70\uc0ac\uc704\uc6d0\uc7a5\uc740 \"\ubd81\ud55c\uc5d0\ub294 \ud55c\u00b7\ubbf8 \uac04 \uc774\uacac\uc774 \uc5c6\ub2e4\ub294 \uac83\uacfc \uad70\uc0ac\uc801\uc73c\ub85c \uacac\uace0\ud558\ub2e4\ub294 \uac83\uc744 \ubcf4\uc5ec\uc8fc\ub294 \uac83\uc774 \uc911\uc694\ud558\ub2e4\"\uba70 \uc0ac\ub4dc \uad00\ub828 \ud655\uc778\uc5d0 \uac10\uc0ac\uc758 \uc758\ubbf8\ub97c \uc804\ud588\ub2e4. 8\uc6d4 12\uc77c \uc5d0\ub294 \uad6d\ubc29\ubd80\uc640 \ud658\uacbd\ubd80\uac00 \uc131\uc8fc\uad70\u00b7\uae40\ucc9c\uc2dc \uad00\uacc4\uc790\uc640 \uae30\uc790\ub4e4\uc758 \ucc38\uad00 \uc18d\uc5d0 \uc0ac\ub4dc \ub808\uc774\ub354\ub85c\ubd80\ud130 100m, 500m, 600m, 700m \uac70\ub9ac\uc5d0\uc11c \uc804\uc790\ud30c\uc640 \uc18c\uc74c\uc744 \uce21\uc815\ud55c \uacb0\uacfc \"\ud734\ub300\uc804\ud654\uac00 \uae30\uc9c0\uad6d\uc744 \ucc3e\uc744 \ub54c \ub098\uc624\ub294 \uc804\uc790\ud30c\ubcf4\ub2e4 \uc791\uc740 \uc218\uc900\uc774\uace0, \uc18c\uc74c\ub3c4 \ub300\ud654\ud560 \ub54c \ub098\uc624\ub294 \uc18c\ub9ac \uc815\ub3c4\"\uac00 \ub098\uc654\ub2e4\uace0 \ud658\uacbd\uc601\ud5a5\ud3c9\uac00\uc758 \uacb0\uacfc\ub97c \ubc1c\ud45c\ud588\ub2e4. \uc804\uc790\ud30c \uc218\uce58\uac00 \ucd5c\uace0 0.01947W/m\uac00 \ub098\uc640 \uae30\uc900\uc774 \uc774\ud558\uac00 \ub098\uc628 \uac83\uc774\ub2e4. \uc774\ud6c4 \uad6d\ubc29\ubd80\ub294 \uc774 \uacb0\uacfc\ub97c \ubc14\ud0d5\uc73c\ub85c \uc8fc\ubbfc\ub4e4\uc744 \uacc4\uc18d \uc124\ub4dd\ud558\uaca0\ub2e4\uace0 \ud588\uc9c0\ub9cc \uc0ac\ub4dc \ubc30\uce58 \ucca0\ud68c\ub97c \uc8fc\uc7a5\ud558\ub294 \uc8fc\ubbfc\ub4e4\uc740 \"\uad6c\uccb4\uc801\uc778 \uce21\uc815 \ubc29\uc2dd\uc744 \uacf5\uac1c\ud558\uc9c0 \uc54a\uc740 \ucc44 \ub098\uc628 \uacb0\uacfc\ub97c \uc5b4\ub5bb\uac8c \ubbff\uc744 \uc218 \uc788\ub290\ub0d0\", \"\ud3c9\uac00\ub2e8\uc5d0 \uc8fc\ubbfc\uc774 \ucd94\ucc9c\ud558\ub294 \uc804\ubb38\uac00\ub97c \ud3ec\ud568\uc2dc\ucf1c\uc904 \uac83\uc744 \uc694\uad6c\ud588\uc9c0\ub9cc \ubc1b\uc544\ub4e4\uc5ec\uc9c0\uc9c0 \uc54a\uc558\ub2e4\"\uba70 \uc5ec\uc804\ud788 \ubc18\ubc1c\uc744 \uc774\uc5b4\uac14\ub2e4. \ud55c\ud3b8, \uc774\ub7ec\ud55c \uacb0\uacfc\uc5d0 \uc57c\ub2f9\ub4e4\uc740 \ud55c\ubaa9\uc18c\ub9ac\ub85c \ucd94\uac00 \uc0ac\ub4dc \ubc1c\uc0ac\ub300 \ub4f1\uc758 \uc870\uc18d\ud55c \ubc30\uce58 \uc644\ub8cc\ub97c \ucd09\uad6c\ud558\uba74\uc11c \ubd88\ud544\uc694\ud55c \uc0ac\ud68c \uac08\ub4f1\uacfc \uad6d\ub860 \ubd84\uc5f4\uc744 \uc77c\uc73c\ud0a8 \uac83\uc5d0 \ub300\ud55c \uc0ac\uacfc\ub97c \uc694\uad6c\ud588\ub2e4. \ubc18\uba74, \ubbfc\uc8fc\ub2f9\uc740 \uc8fc\ubbfc \uc758\uacac\uc774 \uc218\ub834\ub41c \uc77c\ubc18 \ud658\uacbd\uc601\ud5a5\ud3c9\uac00\ub97c \uac70\uccd0\uc57c \ud55c\ub2e4\ub294 \uc8fc\uc7a5\uc744 \ub0c8\ub2e4.", "sentence_answer": " 8\uc6d4 12\uc77c \uc5d0\ub294 \uad6d\ubc29\ubd80\uc640 \ud658\uacbd\ubd80\uac00 \uc131\uc8fc\uad70\u00b7\uae40\ucc9c\uc2dc \uad00\uacc4\uc790\uc640 \uae30\uc790\ub4e4\uc758 \ucc38\uad00 \uc18d\uc5d0 \uc0ac\ub4dc \ub808\uc774\ub354\ub85c\ubd80\ud130 100m, 500m, 600m, 700m \uac70\ub9ac\uc5d0\uc11c \uc804\uc790\ud30c\uc640 \uc18c\uc74c\uc744 \uce21\uc815\ud55c \uacb0\uacfc \"\ud734\ub300\uc804\ud654\uac00 \uae30\uc9c0\uad6d\uc744 \ucc3e\uc744 \ub54c \ub098\uc624\ub294 \uc804\uc790\ud30c\ubcf4\ub2e4 \uc791\uc740 \uc218\uc900\uc774\uace0, \uc18c\uc74c\ub3c4 \ub300\ud654\ud560 \ub54c \ub098\uc624\ub294 \uc18c\ub9ac \uc815\ub3c4\"\uac00 \ub098\uc654\ub2e4\uace0 \ud658\uacbd\uc601\ud5a5\ud3c9\uac00\uc758 \uacb0\uacfc\ub97c \ubc1c\ud45c\ud588\ub2e4.", "paragraph_id": "6580768-56-1", "paragraph_question": "question: \uad6d\ubc29\ubd80\uc640 \ud658\uacbd\ubd80\uac00 \uc131\uc8fc\uad70\uacfc \uae40\ucc9c\uc2dc \uad00\uacc4\uc790\uc640 \uae30\uc790\ub4e4\uc744 \ucc38\uad00 \uc18d\uc5d0 \ud658\uacbd\uc601\ud5a5\ud3c9\uac00 \uacb0\uacfc\ub97c \ubc1c\ud45c\ud55c \uac83\uc740 \uc5b8\uc81c\uc778\uac00?, context: \ud558\uc9c0\ub9cc 6\uc6d4 9\uc77c \uc815\uc758\uc6a9 \uad6d\uac00\uc548\ubcf4\uc2e4\uc7a5\uc774 \"\uc815\uad8c\uc774 \uad50\uccb4\ub418\uc5c8\ub2e4\uace0 \ud574\uc11c \uc774 \uacb0\uc815\uc744 \uacb0\ucf54 \uac00\ubccd\uac8c \uc5ec\uae30\uc9c0 \uc54a\uc744 \uac83\uc774\ub2e4. \ub610 \ubbf8\uad6d\uacfc \uacc4\uc18d \uae34\ubc00\ud788 \ud611\uc758\ud574 \ub098\uac08 \uac83\uc774\ub2e4\"\ub294 \uacf5\uc2dd \uc785\uc7a5\uc744 \ub0b4\ub193\uc558\ub294\ub370 \uc774\ub294 \uc774\ub2ec \ub9d0\uc5d0 \uc608\uc815\ub41c \ud55c\ubbf8\uc815\uc0c1\ud68c\ub2f4 \uc900\ube44\ub97c \uc6d0\ud65c\ud788 \uc870\uc728\ud558\uace0 \ud55c\ubbf8 \ub3d9\ub9f9\uc744 \ud30c\uae30\ud560 \uc758\ub3c4\uac00 \uc5c6\ub2e4\ub294 \uc810\uc744 \ubd84\uba85\ud788 \ud558\uae30 \uc704\ud55c \uac83\uc73c\ub85c \ubcf4\uc778\ub2e4. \ud558\uc9c0\ub9cc \uad6d\uc775\uacfc \uc548\ubcf4\ub97c \ucd5c\uc6b0\uc120\uc73c\ub85c \uace0\ub824\ud558\uaca0\ub2e4\uba74\uc11c \"\ud569\ub9ac\uc801\uc774\uace0 \ub610 \ud569\ubc95\uc801\uc778 \ubc29\ubc95\uc73c\ub85c \ud22c\uba85\ud558\uac8c \uc9c4\ud589\ud574 \ub098\uac08 \uac83\uc774\"\ub77c\uace0 \ud558\uba74\uc11c \ubbfc\uc8fc\uc801\u00b7\uc808\ucc28\uc801 \uc815\ub2f9\uc131\uacfc \ud22c\uba85\uc131 \ud655\ubcf4\ub97c \uc704\ud574 \ud544\uc694\ud55c \uc808\ucc28\ub3c4 \ubc1f\uaca0\ub2e4\ub294 \uc758\uc0ac\ub97c \ub0b4\ube44\ucce4\ub2e4. 29\uc77c \ubbf8\uad6d\uc744 \ubc29\ubb38\ud574\uc11c\ub294 \ubb38\uc7ac\uc778\uc774 \uc9c1\uc811 \"\uc0ac\ub4dc\ub294 \ud55c\ubbf8\ub3d9\ub9f9\uc5d0 \uae30\ucd08\ud55c \ud569\uc758\uc774\uace0 \ud55c\uad6d\ubbfc\uacfc \uc8fc\ud55c\ubbf8\uad70\uc758 \uc0dd\uba85\uc744 \ubcf4\ud638\ud558\uae30 \uc704\ud55c \uac83\"\uc774\ub77c\uba70 \uc0ac\ub4dc \ubc30\uce58\ub97c \ucca0\ud68c\ud558\uc9c0 \uc54a\uaca0\ub2e4\ub294 \uc785\uc7a5\uc744 \ubd84\uba85\ud788 \ubc1d\ud614\ub2e4. \ub9e5 \uc190\ubca0\ub9ac \ud558\uc6d0 \uad70\uc0ac\uc704\uc6d0\uc7a5\uc740 \"\ubd81\ud55c\uc5d0\ub294 \ud55c\u00b7\ubbf8 \uac04 \uc774\uacac\uc774 \uc5c6\ub2e4\ub294 \uac83\uacfc \uad70\uc0ac\uc801\uc73c\ub85c \uacac\uace0\ud558\ub2e4\ub294 \uac83\uc744 \ubcf4\uc5ec\uc8fc\ub294 \uac83\uc774 \uc911\uc694\ud558\ub2e4\"\uba70 \uc0ac\ub4dc \uad00\ub828 \ud655\uc778\uc5d0 \uac10\uc0ac\uc758 \uc758\ubbf8\ub97c \uc804\ud588\ub2e4. 8\uc6d4 12\uc77c\uc5d0\ub294 \uad6d\ubc29\ubd80\uc640 \ud658\uacbd\ubd80\uac00 \uc131\uc8fc\uad70\u00b7\uae40\ucc9c\uc2dc \uad00\uacc4\uc790\uc640 \uae30\uc790\ub4e4\uc758 \ucc38\uad00 \uc18d\uc5d0 \uc0ac\ub4dc \ub808\uc774\ub354\ub85c\ubd80\ud130 100m, 500m, 600m, 700m \uac70\ub9ac\uc5d0\uc11c \uc804\uc790\ud30c\uc640 \uc18c\uc74c\uc744 \uce21\uc815\ud55c \uacb0\uacfc \"\ud734\ub300\uc804\ud654\uac00 \uae30\uc9c0\uad6d\uc744 \ucc3e\uc744 \ub54c \ub098\uc624\ub294 \uc804\uc790\ud30c\ubcf4\ub2e4 \uc791\uc740 \uc218\uc900\uc774\uace0, \uc18c\uc74c\ub3c4 \ub300\ud654\ud560 \ub54c \ub098\uc624\ub294 \uc18c\ub9ac \uc815\ub3c4\"\uac00 \ub098\uc654\ub2e4\uace0 \ud658\uacbd\uc601\ud5a5\ud3c9\uac00\uc758 \uacb0\uacfc\ub97c \ubc1c\ud45c\ud588\ub2e4. \uc804\uc790\ud30c \uc218\uce58\uac00 \ucd5c\uace0 0.01947W/m\uac00 \ub098\uc640 \uae30\uc900\uc774 \uc774\ud558\uac00 \ub098\uc628 \uac83\uc774\ub2e4. \uc774\ud6c4 \uad6d\ubc29\ubd80\ub294 \uc774 \uacb0\uacfc\ub97c \ubc14\ud0d5\uc73c\ub85c \uc8fc\ubbfc\ub4e4\uc744 \uacc4\uc18d \uc124\ub4dd\ud558\uaca0\ub2e4\uace0 \ud588\uc9c0\ub9cc \uc0ac\ub4dc \ubc30\uce58 \ucca0\ud68c\ub97c \uc8fc\uc7a5\ud558\ub294 \uc8fc\ubbfc\ub4e4\uc740 \"\uad6c\uccb4\uc801\uc778 \uce21\uc815 \ubc29\uc2dd\uc744 \uacf5\uac1c\ud558\uc9c0 \uc54a\uc740 \ucc44 \ub098\uc628 \uacb0\uacfc\ub97c \uc5b4\ub5bb\uac8c \ubbff\uc744 \uc218 \uc788\ub290\ub0d0\", \"\ud3c9\uac00\ub2e8\uc5d0 \uc8fc\ubbfc\uc774 \ucd94\ucc9c\ud558\ub294 \uc804\ubb38\uac00\ub97c \ud3ec\ud568\uc2dc\ucf1c\uc904 \uac83\uc744 \uc694\uad6c\ud588\uc9c0\ub9cc \ubc1b\uc544\ub4e4\uc5ec\uc9c0\uc9c0 \uc54a\uc558\ub2e4\"\uba70 \uc5ec\uc804\ud788 \ubc18\ubc1c\uc744 \uc774\uc5b4\uac14\ub2e4. \ud55c\ud3b8, \uc774\ub7ec\ud55c \uacb0\uacfc\uc5d0 \uc57c\ub2f9\ub4e4\uc740 \ud55c\ubaa9\uc18c\ub9ac\ub85c \ucd94\uac00 \uc0ac\ub4dc \ubc1c\uc0ac\ub300 \ub4f1\uc758 \uc870\uc18d\ud55c \ubc30\uce58 \uc644\ub8cc\ub97c \ucd09\uad6c\ud558\uba74\uc11c \ubd88\ud544\uc694\ud55c \uc0ac\ud68c \uac08\ub4f1\uacfc \uad6d\ub860 \ubd84\uc5f4\uc744 \uc77c\uc73c\ud0a8 \uac83\uc5d0 \ub300\ud55c \uc0ac\uacfc\ub97c \uc694\uad6c\ud588\ub2e4. \ubc18\uba74, \ubbfc\uc8fc\ub2f9\uc740 \uc8fc\ubbfc \uc758\uacac\uc774 \uc218\ub834\ub41c \uc77c\ubc18 \ud658\uacbd\uc601\ud5a5\ud3c9\uac00\ub97c \uac70\uccd0\uc57c \ud55c\ub2e4\ub294 \uc8fc\uc7a5\uc744 \ub0c8\ub2e4."} {"question": "\uc138\uc885\ubcd1\uc6d0 \uc0ac\uace0 \ub2f9\uc2dc \ud6a8\uc131\uc758\ub8cc\uc7ac\ub2e8\uc758 \uc774\uc0ac\uc7a5\uc740 \ub204\uad6c\uc778\uac00?", "paragraph": "\ub610\ud55c \ud6a8\uc131\uc758\ub8cc\uc7ac\ub2e8 \uc774\uc0ac\uc7a5 \uc190\uacbd\ucca0\uc740 2012\ub144 4\uc6d4 \uacbd \uc138\uc885\ubcd1\uc6d0 \ubc0f \uc138\uc885\uc694\uc591\ubcd1\uc6d0\uc5d0 10kw \ubc1c\uc804\uae30 1\ub300\ub9cc \uc124\uce58\ud558\uace0\ub3c4, 20kw\uc758 \ubc1c\uc804\uae30 2\ub300\ub97c \uc124\uce58\ud55c \uac83\ucc98\ub7fc \ud5c8\uc704\uc758 \uc2dc\ud5d8\uc131\uc801\uc11c\ub97c \ubcf4\uac74\uc18c\uc5d0 \uc81c\ucd9c\ud558\uc5ec \ub2f4\ub2f9 \uacf5\ubb34\uc6d0\uc758 \uc758\ub8cc\uc2dc\uc124 \uc810\uac80 \uc5c5\ubb34\ub97c \ubc29\ud574(\uc704\uacc4\uacf5\ubb34\uc9d1\ud589\ubc29\ud574)\ud558\uc600\uc73c\uba70, 2014\ub144 ~ 2017\ub144 \uacbd \ubaa9\uc870 \ud734\uac8c\uc2e4, \ucee8\ud14c\uc774\ub108 \ucc3d\uace0, \uc0cc\ub4dc\uc704\uce58\ud310\ub12c \ucc3d\uace0, \ubaa9\uc870 \ube44\uac00\ub9bc \ucc9c\uc7a5 \ub4f1 4\uac74\uc758 \ubd88\ubc95 \uc99d\uac1c\ucd95(\uac74\ucd95\ubc95\uc704\ubc18)\uc744 \ud558\uc600\ub2e4. \uc138\uc885\ubcd1\uc6d0\uacfc \uc138\uc885\uc694\uc591\ubcd1\uc6d0\uc744 \uc787\ub294 2\uce35 \uc5f0\uacb0\ud1b5\ub85c \ubc14\ub2e5\uc5d0\ub294 1\uce35\uacfc \ud1b5\ud558\ub294 \uacf5\uac04\uc774 \uc788\ub294\ub370, \uc774\ub97c \ud1b5\ud558\uc5ec \uc720\uc785\ub41c 1\uce35\uc758 \uc720\ub3c5\uac00\uc2a4\uac00 \ubd88\ubc95 \uac74\ucd95\ub41c \ud3d0\uc1c4\ud615 \ube44\uac00\ub9bc\ub9c9\uc73c\ub85c \uc778\ud558\uc5ec \uc678\ubd80\ub85c \ubc30\ucd9c\ub418\uc9c0 \ubabb\ud558\uc5ec \ud53c\ud574\uac00 \ud655\ub300\ub418\uc5c8\ub2e4. \uc138\uc885\ubcd1\uc6d0\uc740 \uad00\ud560\ub2f9\uad6d\uc73c\ub85c\ubd80\ud130 \uc704 \ud3d0\uc1c4\ud615 \ube44\uac00\ub9bc\ub9c9\uc5d0 \ub300\ud558\uc5ec \uc218\ucc28\ub840 \ucca0\uac70 \uba85\ub839\uc744 \ubc1b\uc558\uc73c\ub098 \uc774\ud589\ud558\uc9c0 \uc544\ub2c8\ud558\uace0, \uc57d 3,000\ub9cc\uc6d0\uc758 \uc774\ud589\uac15\uc81c\uae08\ub9cc \ub0a9\ubd80\ud558\uba70 \uacc4\uc18d \uc0ac\uc6a9\ud558\uace0 \uc788\uc5c8\ub2e4. \uc190\uacbd\ucca0 \uc774\uc0ac\uc7a5\uacfc \uc138\uc885\ubcd1\uc6d0 \ud589\uc815\uc774\uc0ac\ub294 2017\ub144 2\uc6d4 28\uc77c ~ 2018\ub144 1\uc6d4 26\uc77c \uc758\ub8cc\uc778 \uc218 \ubcc0\uacbd \ud5c8\uac00 \uc5c6\uc774 \ub2f9\uc9c1\uc758\uc0ac 4\uba85\uc744 \uace0\uc6a9\ud558\uc600\uc73c\uba70(\uc758\ub8cc\ubc95\uc704\ubc18), 2018\ub144 1\uc6d4 8\uc77c ~ 26\uc77c \ub2f9\uc9c1\uc758\ub8cc\uc778\uc73c\ub85c \uc758\uc0ac 1\uba85, \uac04\ud638\uc0ac 2\uba85\uc744 \ub450\uc5b4\uc57c \ud568\uc5d0\ub3c4 \uac04\ud638\uc0ac\ub97c \ub450\uc9c0 \uc54a\uc740(\uc758\ub8cc\ubc95\uc704\ubc18) \uc0ac\uc2e4\ub3c4 \ud655\uc778\ub418\uc5c8\ub2e4. \uc138\uc885\ubcd1\uc6d0 \ubcd1\uc6d0\uc7a5\uc740 2018\ub144 1\uc6d4 15\uc77c ~ 26\uc77c H \ub4f1 4\uba85\uc758 \uc758\uc0ac\ub85c \ud558\uc5ec\uae08 \ubcd1\uc6d0\uc7a5\uc778 \uc790\uc2e0\uc758 \uba85\uc758\ub85c \ucc98\ubc29\uc804\uc744 \uc791\uc131\ud558\uac8c \ud55c \uc758\ub8cc\ubc95 \uc704\ubc18 \uc0ac\uc2e4\uc774 \ud655\uc778\ub418\uc5c8\ub2e4(\ub300\uc9c4\uc758\uc0ac 4\uba85\ub3c4 \uc758\ub8cc\ubc95\uc704\ubc18).", "answer": "\uc190\uacbd\ucca0", "sentence": "\ub610\ud55c \ud6a8\uc131\uc758\ub8cc\uc7ac\ub2e8 \uc774\uc0ac\uc7a5 \uc190\uacbd\ucca0 \uc740 2012\ub144 4\uc6d4 \uacbd \uc138\uc885\ubcd1\uc6d0 \ubc0f \uc138\uc885\uc694\uc591\ubcd1\uc6d0\uc5d0 10kw \ubc1c\uc804\uae30 1\ub300\ub9cc \uc124\uce58\ud558\uace0\ub3c4, 20kw\uc758 \ubc1c\uc804\uae30 2\ub300\ub97c \uc124\uce58\ud55c \uac83\ucc98\ub7fc \ud5c8\uc704\uc758 \uc2dc\ud5d8\uc131\uc801\uc11c\ub97c \ubcf4\uac74\uc18c\uc5d0 \uc81c\ucd9c\ud558\uc5ec \ub2f4\ub2f9 \uacf5\ubb34\uc6d0\uc758 \uc758\ub8cc\uc2dc\uc124 \uc810\uac80 \uc5c5\ubb34\ub97c \ubc29\ud574(\uc704\uacc4\uacf5\ubb34\uc9d1\ud589\ubc29\ud574)\ud558\uc600\uc73c\uba70, 2014\ub144 ~ 2017\ub144 \uacbd \ubaa9\uc870 \ud734\uac8c\uc2e4, \ucee8\ud14c\uc774\ub108 \ucc3d\uace0, \uc0cc\ub4dc\uc704\uce58\ud310\ub12c \ucc3d\uace0, \ubaa9\uc870 \ube44\uac00\ub9bc \ucc9c\uc7a5 \ub4f1 4\uac74\uc758 \ubd88\ubc95 \uc99d\uac1c\ucd95(\uac74\ucd95\ubc95\uc704\ubc18)\uc744 \ud558\uc600\ub2e4.", "paragraph_sentence": " \ub610\ud55c \ud6a8\uc131\uc758\ub8cc\uc7ac\ub2e8 \uc774\uc0ac\uc7a5 \uc190\uacbd\ucca0 \uc740 2012\ub144 4\uc6d4 \uacbd \uc138\uc885\ubcd1\uc6d0 \ubc0f \uc138\uc885\uc694\uc591\ubcd1\uc6d0\uc5d0 10kw \ubc1c\uc804\uae30 1\ub300\ub9cc \uc124\uce58\ud558\uace0\ub3c4, 20kw\uc758 \ubc1c\uc804\uae30 2\ub300\ub97c \uc124\uce58\ud55c \uac83\ucc98\ub7fc \ud5c8\uc704\uc758 \uc2dc\ud5d8\uc131\uc801\uc11c\ub97c \ubcf4\uac74\uc18c\uc5d0 \uc81c\ucd9c\ud558\uc5ec \ub2f4\ub2f9 \uacf5\ubb34\uc6d0\uc758 \uc758\ub8cc\uc2dc\uc124 \uc810\uac80 \uc5c5\ubb34\ub97c \ubc29\ud574(\uc704\uacc4\uacf5\ubb34\uc9d1\ud589\ubc29\ud574)\ud558\uc600\uc73c\uba70, 2014\ub144 ~ 2017\ub144 \uacbd \ubaa9\uc870 \ud734\uac8c\uc2e4, \ucee8\ud14c\uc774\ub108 \ucc3d\uace0, \uc0cc\ub4dc\uc704\uce58\ud310\ub12c \ucc3d\uace0, \ubaa9\uc870 \ube44\uac00\ub9bc \ucc9c\uc7a5 \ub4f1 4\uac74\uc758 \ubd88\ubc95 \uc99d\uac1c\ucd95(\uac74\ucd95\ubc95\uc704\ubc18)\uc744 \ud558\uc600\ub2e4. \uc138\uc885\ubcd1\uc6d0\uacfc \uc138\uc885\uc694\uc591\ubcd1\uc6d0\uc744 \uc787\ub294 2\uce35 \uc5f0\uacb0\ud1b5\ub85c \ubc14\ub2e5\uc5d0\ub294 1\uce35\uacfc \ud1b5\ud558\ub294 \uacf5\uac04\uc774 \uc788\ub294\ub370, \uc774\ub97c \ud1b5\ud558\uc5ec \uc720\uc785\ub41c 1\uce35\uc758 \uc720\ub3c5\uac00\uc2a4\uac00 \ubd88\ubc95 \uac74\ucd95\ub41c \ud3d0\uc1c4\ud615 \ube44\uac00\ub9bc\ub9c9\uc73c\ub85c \uc778\ud558\uc5ec \uc678\ubd80\ub85c \ubc30\ucd9c\ub418\uc9c0 \ubabb\ud558\uc5ec \ud53c\ud574\uac00 \ud655\ub300\ub418\uc5c8\ub2e4. \uc138\uc885\ubcd1\uc6d0\uc740 \uad00\ud560\ub2f9\uad6d\uc73c\ub85c\ubd80\ud130 \uc704 \ud3d0\uc1c4\ud615 \ube44\uac00\ub9bc\ub9c9\uc5d0 \ub300\ud558\uc5ec \uc218\ucc28\ub840 \ucca0\uac70 \uba85\ub839\uc744 \ubc1b\uc558\uc73c\ub098 \uc774\ud589\ud558\uc9c0 \uc544\ub2c8\ud558\uace0, \uc57d 3,000\ub9cc\uc6d0\uc758 \uc774\ud589\uac15\uc81c\uae08\ub9cc \ub0a9\ubd80\ud558\uba70 \uacc4\uc18d \uc0ac\uc6a9\ud558\uace0 \uc788\uc5c8\ub2e4. \uc190\uacbd\ucca0 \uc774\uc0ac\uc7a5\uacfc \uc138\uc885\ubcd1\uc6d0 \ud589\uc815\uc774\uc0ac\ub294 2017\ub144 2\uc6d4 28\uc77c ~ 2018\ub144 1\uc6d4 26\uc77c \uc758\ub8cc\uc778 \uc218 \ubcc0\uacbd \ud5c8\uac00 \uc5c6\uc774 \ub2f9\uc9c1\uc758\uc0ac 4\uba85\uc744 \uace0\uc6a9\ud558\uc600\uc73c\uba70(\uc758\ub8cc\ubc95\uc704\ubc18), 2018\ub144 1\uc6d4 8\uc77c ~ 26\uc77c \ub2f9\uc9c1\uc758\ub8cc\uc778\uc73c\ub85c \uc758\uc0ac 1\uba85, \uac04\ud638\uc0ac 2\uba85\uc744 \ub450\uc5b4\uc57c \ud568\uc5d0\ub3c4 \uac04\ud638\uc0ac\ub97c \ub450\uc9c0 \uc54a\uc740(\uc758\ub8cc\ubc95\uc704\ubc18) \uc0ac\uc2e4\ub3c4 \ud655\uc778\ub418\uc5c8\ub2e4. \uc138\uc885\ubcd1\uc6d0 \ubcd1\uc6d0\uc7a5\uc740 2018\ub144 1\uc6d4 15\uc77c ~ 26\uc77c H \ub4f1 4\uba85\uc758 \uc758\uc0ac\ub85c \ud558\uc5ec\uae08 \ubcd1\uc6d0\uc7a5\uc778 \uc790\uc2e0\uc758 \uba85\uc758\ub85c \ucc98\ubc29\uc804\uc744 \uc791\uc131\ud558\uac8c \ud55c \uc758\ub8cc\ubc95 \uc704\ubc18 \uc0ac\uc2e4\uc774 \ud655\uc778\ub418\uc5c8\ub2e4(\ub300\uc9c4\uc758\uc0ac 4\uba85\ub3c4 \uc758\ub8cc\ubc95\uc704\ubc18).", "paragraph_answer": "\ub610\ud55c \ud6a8\uc131\uc758\ub8cc\uc7ac\ub2e8 \uc774\uc0ac\uc7a5 \uc190\uacbd\ucca0 \uc740 2012\ub144 4\uc6d4 \uacbd \uc138\uc885\ubcd1\uc6d0 \ubc0f \uc138\uc885\uc694\uc591\ubcd1\uc6d0\uc5d0 10kw \ubc1c\uc804\uae30 1\ub300\ub9cc \uc124\uce58\ud558\uace0\ub3c4, 20kw\uc758 \ubc1c\uc804\uae30 2\ub300\ub97c \uc124\uce58\ud55c \uac83\ucc98\ub7fc \ud5c8\uc704\uc758 \uc2dc\ud5d8\uc131\uc801\uc11c\ub97c \ubcf4\uac74\uc18c\uc5d0 \uc81c\ucd9c\ud558\uc5ec \ub2f4\ub2f9 \uacf5\ubb34\uc6d0\uc758 \uc758\ub8cc\uc2dc\uc124 \uc810\uac80 \uc5c5\ubb34\ub97c \ubc29\ud574(\uc704\uacc4\uacf5\ubb34\uc9d1\ud589\ubc29\ud574)\ud558\uc600\uc73c\uba70, 2014\ub144 ~ 2017\ub144 \uacbd \ubaa9\uc870 \ud734\uac8c\uc2e4, \ucee8\ud14c\uc774\ub108 \ucc3d\uace0, \uc0cc\ub4dc\uc704\uce58\ud310\ub12c \ucc3d\uace0, \ubaa9\uc870 \ube44\uac00\ub9bc \ucc9c\uc7a5 \ub4f1 4\uac74\uc758 \ubd88\ubc95 \uc99d\uac1c\ucd95(\uac74\ucd95\ubc95\uc704\ubc18)\uc744 \ud558\uc600\ub2e4. \uc138\uc885\ubcd1\uc6d0\uacfc \uc138\uc885\uc694\uc591\ubcd1\uc6d0\uc744 \uc787\ub294 2\uce35 \uc5f0\uacb0\ud1b5\ub85c \ubc14\ub2e5\uc5d0\ub294 1\uce35\uacfc \ud1b5\ud558\ub294 \uacf5\uac04\uc774 \uc788\ub294\ub370, \uc774\ub97c \ud1b5\ud558\uc5ec \uc720\uc785\ub41c 1\uce35\uc758 \uc720\ub3c5\uac00\uc2a4\uac00 \ubd88\ubc95 \uac74\ucd95\ub41c \ud3d0\uc1c4\ud615 \ube44\uac00\ub9bc\ub9c9\uc73c\ub85c \uc778\ud558\uc5ec \uc678\ubd80\ub85c \ubc30\ucd9c\ub418\uc9c0 \ubabb\ud558\uc5ec \ud53c\ud574\uac00 \ud655\ub300\ub418\uc5c8\ub2e4. \uc138\uc885\ubcd1\uc6d0\uc740 \uad00\ud560\ub2f9\uad6d\uc73c\ub85c\ubd80\ud130 \uc704 \ud3d0\uc1c4\ud615 \ube44\uac00\ub9bc\ub9c9\uc5d0 \ub300\ud558\uc5ec \uc218\ucc28\ub840 \ucca0\uac70 \uba85\ub839\uc744 \ubc1b\uc558\uc73c\ub098 \uc774\ud589\ud558\uc9c0 \uc544\ub2c8\ud558\uace0, \uc57d 3,000\ub9cc\uc6d0\uc758 \uc774\ud589\uac15\uc81c\uae08\ub9cc \ub0a9\ubd80\ud558\uba70 \uacc4\uc18d \uc0ac\uc6a9\ud558\uace0 \uc788\uc5c8\ub2e4. \uc190\uacbd\ucca0 \uc774\uc0ac\uc7a5\uacfc \uc138\uc885\ubcd1\uc6d0 \ud589\uc815\uc774\uc0ac\ub294 2017\ub144 2\uc6d4 28\uc77c ~ 2018\ub144 1\uc6d4 26\uc77c \uc758\ub8cc\uc778 \uc218 \ubcc0\uacbd \ud5c8\uac00 \uc5c6\uc774 \ub2f9\uc9c1\uc758\uc0ac 4\uba85\uc744 \uace0\uc6a9\ud558\uc600\uc73c\uba70(\uc758\ub8cc\ubc95\uc704\ubc18), 2018\ub144 1\uc6d4 8\uc77c ~ 26\uc77c \ub2f9\uc9c1\uc758\ub8cc\uc778\uc73c\ub85c \uc758\uc0ac 1\uba85, \uac04\ud638\uc0ac 2\uba85\uc744 \ub450\uc5b4\uc57c \ud568\uc5d0\ub3c4 \uac04\ud638\uc0ac\ub97c \ub450\uc9c0 \uc54a\uc740(\uc758\ub8cc\ubc95\uc704\ubc18) \uc0ac\uc2e4\ub3c4 \ud655\uc778\ub418\uc5c8\ub2e4. \uc138\uc885\ubcd1\uc6d0 \ubcd1\uc6d0\uc7a5\uc740 2018\ub144 1\uc6d4 15\uc77c ~ 26\uc77c H \ub4f1 4\uba85\uc758 \uc758\uc0ac\ub85c \ud558\uc5ec\uae08 \ubcd1\uc6d0\uc7a5\uc778 \uc790\uc2e0\uc758 \uba85\uc758\ub85c \ucc98\ubc29\uc804\uc744 \uc791\uc131\ud558\uac8c \ud55c \uc758\ub8cc\ubc95 \uc704\ubc18 \uc0ac\uc2e4\uc774 \ud655\uc778\ub418\uc5c8\ub2e4(\ub300\uc9c4\uc758\uc0ac 4\uba85\ub3c4 \uc758\ub8cc\ubc95\uc704\ubc18).", "sentence_answer": "\ub610\ud55c \ud6a8\uc131\uc758\ub8cc\uc7ac\ub2e8 \uc774\uc0ac\uc7a5 \uc190\uacbd\ucca0 \uc740 2012\ub144 4\uc6d4 \uacbd \uc138\uc885\ubcd1\uc6d0 \ubc0f \uc138\uc885\uc694\uc591\ubcd1\uc6d0\uc5d0 10kw \ubc1c\uc804\uae30 1\ub300\ub9cc \uc124\uce58\ud558\uace0\ub3c4, 20kw\uc758 \ubc1c\uc804\uae30 2\ub300\ub97c \uc124\uce58\ud55c \uac83\ucc98\ub7fc \ud5c8\uc704\uc758 \uc2dc\ud5d8\uc131\uc801\uc11c\ub97c \ubcf4\uac74\uc18c\uc5d0 \uc81c\ucd9c\ud558\uc5ec \ub2f4\ub2f9 \uacf5\ubb34\uc6d0\uc758 \uc758\ub8cc\uc2dc\uc124 \uc810\uac80 \uc5c5\ubb34\ub97c \ubc29\ud574(\uc704\uacc4\uacf5\ubb34\uc9d1\ud589\ubc29\ud574)\ud558\uc600\uc73c\uba70, 2014\ub144 ~ 2017\ub144 \uacbd \ubaa9\uc870 \ud734\uac8c\uc2e4, \ucee8\ud14c\uc774\ub108 \ucc3d\uace0, \uc0cc\ub4dc\uc704\uce58\ud310\ub12c \ucc3d\uace0, \ubaa9\uc870 \ube44\uac00\ub9bc \ucc9c\uc7a5 \ub4f1 4\uac74\uc758 \ubd88\ubc95 \uc99d\uac1c\ucd95(\uac74\ucd95\ubc95\uc704\ubc18)\uc744 \ud558\uc600\ub2e4.", "paragraph_id": "6505708-3-1", "paragraph_question": "question: \uc138\uc885\ubcd1\uc6d0 \uc0ac\uace0 \ub2f9\uc2dc \ud6a8\uc131\uc758\ub8cc\uc7ac\ub2e8\uc758 \uc774\uc0ac\uc7a5\uc740 \ub204\uad6c\uc778\uac00?, context: \ub610\ud55c \ud6a8\uc131\uc758\ub8cc\uc7ac\ub2e8 \uc774\uc0ac\uc7a5 \uc190\uacbd\ucca0\uc740 2012\ub144 4\uc6d4 \uacbd \uc138\uc885\ubcd1\uc6d0 \ubc0f \uc138\uc885\uc694\uc591\ubcd1\uc6d0\uc5d0 10kw \ubc1c\uc804\uae30 1\ub300\ub9cc \uc124\uce58\ud558\uace0\ub3c4, 20kw\uc758 \ubc1c\uc804\uae30 2\ub300\ub97c \uc124\uce58\ud55c \uac83\ucc98\ub7fc \ud5c8\uc704\uc758 \uc2dc\ud5d8\uc131\uc801\uc11c\ub97c \ubcf4\uac74\uc18c\uc5d0 \uc81c\ucd9c\ud558\uc5ec \ub2f4\ub2f9 \uacf5\ubb34\uc6d0\uc758 \uc758\ub8cc\uc2dc\uc124 \uc810\uac80 \uc5c5\ubb34\ub97c \ubc29\ud574(\uc704\uacc4\uacf5\ubb34\uc9d1\ud589\ubc29\ud574)\ud558\uc600\uc73c\uba70, 2014\ub144 ~ 2017\ub144 \uacbd \ubaa9\uc870 \ud734\uac8c\uc2e4, \ucee8\ud14c\uc774\ub108 \ucc3d\uace0, \uc0cc\ub4dc\uc704\uce58\ud310\ub12c \ucc3d\uace0, \ubaa9\uc870 \ube44\uac00\ub9bc \ucc9c\uc7a5 \ub4f1 4\uac74\uc758 \ubd88\ubc95 \uc99d\uac1c\ucd95(\uac74\ucd95\ubc95\uc704\ubc18)\uc744 \ud558\uc600\ub2e4. \uc138\uc885\ubcd1\uc6d0\uacfc \uc138\uc885\uc694\uc591\ubcd1\uc6d0\uc744 \uc787\ub294 2\uce35 \uc5f0\uacb0\ud1b5\ub85c \ubc14\ub2e5\uc5d0\ub294 1\uce35\uacfc \ud1b5\ud558\ub294 \uacf5\uac04\uc774 \uc788\ub294\ub370, \uc774\ub97c \ud1b5\ud558\uc5ec \uc720\uc785\ub41c 1\uce35\uc758 \uc720\ub3c5\uac00\uc2a4\uac00 \ubd88\ubc95 \uac74\ucd95\ub41c \ud3d0\uc1c4\ud615 \ube44\uac00\ub9bc\ub9c9\uc73c\ub85c \uc778\ud558\uc5ec \uc678\ubd80\ub85c \ubc30\ucd9c\ub418\uc9c0 \ubabb\ud558\uc5ec \ud53c\ud574\uac00 \ud655\ub300\ub418\uc5c8\ub2e4. \uc138\uc885\ubcd1\uc6d0\uc740 \uad00\ud560\ub2f9\uad6d\uc73c\ub85c\ubd80\ud130 \uc704 \ud3d0\uc1c4\ud615 \ube44\uac00\ub9bc\ub9c9\uc5d0 \ub300\ud558\uc5ec \uc218\ucc28\ub840 \ucca0\uac70 \uba85\ub839\uc744 \ubc1b\uc558\uc73c\ub098 \uc774\ud589\ud558\uc9c0 \uc544\ub2c8\ud558\uace0, \uc57d 3,000\ub9cc\uc6d0\uc758 \uc774\ud589\uac15\uc81c\uae08\ub9cc \ub0a9\ubd80\ud558\uba70 \uacc4\uc18d \uc0ac\uc6a9\ud558\uace0 \uc788\uc5c8\ub2e4. \uc190\uacbd\ucca0 \uc774\uc0ac\uc7a5\uacfc \uc138\uc885\ubcd1\uc6d0 \ud589\uc815\uc774\uc0ac\ub294 2017\ub144 2\uc6d4 28\uc77c ~ 2018\ub144 1\uc6d4 26\uc77c \uc758\ub8cc\uc778 \uc218 \ubcc0\uacbd \ud5c8\uac00 \uc5c6\uc774 \ub2f9\uc9c1\uc758\uc0ac 4\uba85\uc744 \uace0\uc6a9\ud558\uc600\uc73c\uba70(\uc758\ub8cc\ubc95\uc704\ubc18), 2018\ub144 1\uc6d4 8\uc77c ~ 26\uc77c \ub2f9\uc9c1\uc758\ub8cc\uc778\uc73c\ub85c \uc758\uc0ac 1\uba85, \uac04\ud638\uc0ac 2\uba85\uc744 \ub450\uc5b4\uc57c \ud568\uc5d0\ub3c4 \uac04\ud638\uc0ac\ub97c \ub450\uc9c0 \uc54a\uc740(\uc758\ub8cc\ubc95\uc704\ubc18) \uc0ac\uc2e4\ub3c4 \ud655\uc778\ub418\uc5c8\ub2e4. \uc138\uc885\ubcd1\uc6d0 \ubcd1\uc6d0\uc7a5\uc740 2018\ub144 1\uc6d4 15\uc77c ~ 26\uc77c H \ub4f1 4\uba85\uc758 \uc758\uc0ac\ub85c \ud558\uc5ec\uae08 \ubcd1\uc6d0\uc7a5\uc778 \uc790\uc2e0\uc758 \uba85\uc758\ub85c \ucc98\ubc29\uc804\uc744 \uc791\uc131\ud558\uac8c \ud55c \uc758\ub8cc\ubc95 \uc704\ubc18 \uc0ac\uc2e4\uc774 \ud655\uc778\ub418\uc5c8\ub2e4(\ub300\uc9c4\uc758\uc0ac 4\uba85\ub3c4 \uc758\ub8cc\ubc95\uc704\ubc18)."} {"question": "\uc5d0\ub7f0 \ub7a8\uc9c0\uac00 \uc544\ubd80 \ub514\uc544\ube44 \ub300\uc2e0 \ubbf8\ub4dc\ud544\ub354\ub85c \ucd9c\uc804\ud55c \ub144\ub3c4\ub294?", "paragraph": "\uccab \ub9ac\uadf8 4\uacbd\uae30\uc5d0\uc11c\ub294 \ud6c4\ubcf4\ub85c \ub6f0\uc5c8\ub2e4. 2012\ub144 9\uc6d4 15\uc77c 6:1\ub85c \ub300\uc2b9\uc744 \uac70\ub454 \uc0ac\uc6b0\uc2a4\ud584\ud2bc FC\uc804\uc5d0\uc11c \ud504\ub79c\uc2dc\uc2a4 \ucf54\ud074\ub7ad \ub300\uc2e0 \uad50\uccb4\ud22c\uc785\ub3fc \uace8\ub300\ub97c \ub9de\ucdb0 5\ubc88\uc9f8 \uace8\uc744 \uc774\ub04c\uc5b4\ub0c8\ub2e4. 9\uc6d4 23\uc77c \ub514\ud39c\ub529 \ucc54\ud53c\uc5b8 \ub9e8\uccb4\uc2a4\ud130 \uc2dc\ud2f0\uc804\uc5d0\uc11c \uccab \uc120\ubc1c \ucd9c\uc804\uc744 \ud574 \uc624\ub978\ucabd \ubbf8\ub4dc\ud544\ub354 \uc5ed\ud560\uc744 \ub9e1\uc544 \uad1c\ucc2e\uc740 \uacbd\uae30\ub97c \ud3bc\ucce4\ub2e4. 10\uc6d4 3\uc77c \uc62c\ub9bc\ud53c\uc544\ucf54\uc2a4 FC\uc804\uc5d0\uc11c \uc624\ub978\ubc1c \uce69\uc0f7\uc73c\ub85c \ub4dd\uc810\ud574 \ud300\uc758 3:1 \uc2b9\ub9ac\uc5d0 \uae30\uc5ec\ud588\ub2e4. 12\uc6d4 19\uc77c \ud300 \ub3d9\ub8cc \uc568\ub9ad\uc2a4 \uc625\uc2ac\ub808\uc774\ub4dc\uccb4\uc784\ubc8c\ub9b0, \ud0a4\uc5b4\ub7f0 \uae41\uc2a4, \uc7ad \uc70c\uc154, \uce7c \uc831\ud0a8\uc2a8\uacfc \ud568\uaed8 \uc7a5\uae30\uacc4\uc57d\uc744 \uccb4\uacb0\ud588\ub2e4. 2013\ub144 1\uc6d4 24\uc77c, 5:1 \ub300\uc2b9\uc744 \uac70\ub454 \uc6e8\uc2a4\ud2b8\ud584\uc804\uc5d0\uc11c \ubd80\uc0c1\ub2f9\ud55c \ubbf8\ucf08 \uc544\ub974\ud14c\ud0c0\uc640 \ud734\uc2dd\uc744 \ubd80\uc5ec\ubc1b\uc740 \uc544\ubd80 \ub514\uc544\ube44 \ub300\uc2e0 \ubbf8\ub4dc\ud544\ub354\ub85c \ucd9c\uc804\ud574 \uc88b\uc740 \ud328\uc2a4\ub97c \ucc14\ub7ec\uc8fc\uba70 \ud65c\uc57d\ud574 \ud32c\ub4e4\uc758 \uae30\ub300\uac10\uc744 \ub298\ub838\ub2e4. 1\uc6d4 27\uc77c FA\ucef5 \ube0c\ub77c\uc774\ud2bc\uc804\uc5d0\uc11c \ub2e4\uc2dc \ub525-\ub77c\uc789 \ud50c\ub808\uc774\uba54\uc774\ucee4\ub85c \ucd9c\uc804\ud574 \uad1c\ucc2e\uc740 \ubaa8\uc2b5\uc744 \uc774\uc5b4\uac14\ub2e4.", "answer": "2013\ub144", "sentence": "2013\ub144 1\uc6d4 24\uc77c, 5:1 \ub300\uc2b9\uc744 \uac70\ub454 \uc6e8\uc2a4\ud2b8\ud584\uc804\uc5d0\uc11c \ubd80\uc0c1\ub2f9\ud55c \ubbf8\ucf08 \uc544\ub974\ud14c\ud0c0\uc640 \ud734\uc2dd\uc744 \ubd80\uc5ec\ubc1b\uc740 \uc544\ubd80 \ub514\uc544\ube44 \ub300\uc2e0 \ubbf8\ub4dc\ud544\ub354\ub85c \ucd9c\uc804\ud574 \uc88b\uc740 \ud328\uc2a4\ub97c \ucc14\ub7ec\uc8fc\uba70 \ud65c\uc57d\ud574 \ud32c\ub4e4\uc758 \uae30\ub300\uac10\uc744 \ub298\ub838\ub2e4.", "paragraph_sentence": "\uccab \ub9ac\uadf8 4\uacbd\uae30\uc5d0\uc11c\ub294 \ud6c4\ubcf4\ub85c \ub6f0\uc5c8\ub2e4. 2012\ub144 9\uc6d4 15\uc77c 6:1\ub85c \ub300\uc2b9\uc744 \uac70\ub454 \uc0ac\uc6b0\uc2a4\ud584\ud2bc FC\uc804\uc5d0\uc11c \ud504\ub79c\uc2dc\uc2a4 \ucf54\ud074\ub7ad \ub300\uc2e0 \uad50\uccb4\ud22c\uc785\ub3fc \uace8\ub300\ub97c \ub9de\ucdb0 5\ubc88\uc9f8 \uace8\uc744 \uc774\ub04c\uc5b4\ub0c8\ub2e4. 9\uc6d4 23\uc77c \ub514\ud39c\ub529 \ucc54\ud53c\uc5b8 \ub9e8\uccb4\uc2a4\ud130 \uc2dc\ud2f0\uc804\uc5d0\uc11c \uccab \uc120\ubc1c \ucd9c\uc804\uc744 \ud574 \uc624\ub978\ucabd \ubbf8\ub4dc\ud544\ub354 \uc5ed\ud560\uc744 \ub9e1\uc544 \uad1c\ucc2e\uc740 \uacbd\uae30\ub97c \ud3bc\ucce4\ub2e4. 10\uc6d4 3\uc77c \uc62c\ub9bc\ud53c\uc544\ucf54\uc2a4 FC\uc804\uc5d0\uc11c \uc624\ub978\ubc1c \uce69\uc0f7\uc73c\ub85c \ub4dd\uc810\ud574 \ud300\uc758 3:1 \uc2b9\ub9ac\uc5d0 \uae30\uc5ec\ud588\ub2e4. 12\uc6d4 19\uc77c \ud300 \ub3d9\ub8cc \uc568\ub9ad\uc2a4 \uc625\uc2ac\ub808\uc774\ub4dc\uccb4\uc784\ubc8c\ub9b0, \ud0a4\uc5b4\ub7f0 \uae41\uc2a4, \uc7ad \uc70c\uc154, \uce7c \uc831\ud0a8\uc2a8\uacfc \ud568\uaed8 \uc7a5\uae30\uacc4\uc57d\uc744 \uccb4\uacb0\ud588\ub2e4. 2013\ub144 1\uc6d4 24\uc77c, 5:1 \ub300\uc2b9\uc744 \uac70\ub454 \uc6e8\uc2a4\ud2b8\ud584\uc804\uc5d0\uc11c \ubd80\uc0c1\ub2f9\ud55c \ubbf8\ucf08 \uc544\ub974\ud14c\ud0c0\uc640 \ud734\uc2dd\uc744 \ubd80\uc5ec\ubc1b\uc740 \uc544\ubd80 \ub514\uc544\ube44 \ub300\uc2e0 \ubbf8\ub4dc\ud544\ub354\ub85c \ucd9c\uc804\ud574 \uc88b\uc740 \ud328\uc2a4\ub97c \ucc14\ub7ec\uc8fc\uba70 \ud65c\uc57d\ud574 \ud32c\ub4e4\uc758 \uae30\ub300\uac10\uc744 \ub298\ub838\ub2e4. 1\uc6d4 27\uc77c FA\ucef5 \ube0c\ub77c\uc774\ud2bc\uc804\uc5d0\uc11c \ub2e4\uc2dc \ub525-\ub77c\uc789 \ud50c\ub808\uc774\uba54\uc774\ucee4\ub85c \ucd9c\uc804\ud574 \uad1c\ucc2e\uc740 \ubaa8\uc2b5\uc744 \uc774\uc5b4\uac14\ub2e4.", "paragraph_answer": "\uccab \ub9ac\uadf8 4\uacbd\uae30\uc5d0\uc11c\ub294 \ud6c4\ubcf4\ub85c \ub6f0\uc5c8\ub2e4. 2012\ub144 9\uc6d4 15\uc77c 6:1\ub85c \ub300\uc2b9\uc744 \uac70\ub454 \uc0ac\uc6b0\uc2a4\ud584\ud2bc FC\uc804\uc5d0\uc11c \ud504\ub79c\uc2dc\uc2a4 \ucf54\ud074\ub7ad \ub300\uc2e0 \uad50\uccb4\ud22c\uc785\ub3fc \uace8\ub300\ub97c \ub9de\ucdb0 5\ubc88\uc9f8 \uace8\uc744 \uc774\ub04c\uc5b4\ub0c8\ub2e4. 9\uc6d4 23\uc77c \ub514\ud39c\ub529 \ucc54\ud53c\uc5b8 \ub9e8\uccb4\uc2a4\ud130 \uc2dc\ud2f0\uc804\uc5d0\uc11c \uccab \uc120\ubc1c \ucd9c\uc804\uc744 \ud574 \uc624\ub978\ucabd \ubbf8\ub4dc\ud544\ub354 \uc5ed\ud560\uc744 \ub9e1\uc544 \uad1c\ucc2e\uc740 \uacbd\uae30\ub97c \ud3bc\ucce4\ub2e4. 10\uc6d4 3\uc77c \uc62c\ub9bc\ud53c\uc544\ucf54\uc2a4 FC\uc804\uc5d0\uc11c \uc624\ub978\ubc1c \uce69\uc0f7\uc73c\ub85c \ub4dd\uc810\ud574 \ud300\uc758 3:1 \uc2b9\ub9ac\uc5d0 \uae30\uc5ec\ud588\ub2e4. 12\uc6d4 19\uc77c \ud300 \ub3d9\ub8cc \uc568\ub9ad\uc2a4 \uc625\uc2ac\ub808\uc774\ub4dc\uccb4\uc784\ubc8c\ub9b0, \ud0a4\uc5b4\ub7f0 \uae41\uc2a4, \uc7ad \uc70c\uc154, \uce7c \uc831\ud0a8\uc2a8\uacfc \ud568\uaed8 \uc7a5\uae30\uacc4\uc57d\uc744 \uccb4\uacb0\ud588\ub2e4. 2013\ub144 1\uc6d4 24\uc77c, 5:1 \ub300\uc2b9\uc744 \uac70\ub454 \uc6e8\uc2a4\ud2b8\ud584\uc804\uc5d0\uc11c \ubd80\uc0c1\ub2f9\ud55c \ubbf8\ucf08 \uc544\ub974\ud14c\ud0c0\uc640 \ud734\uc2dd\uc744 \ubd80\uc5ec\ubc1b\uc740 \uc544\ubd80 \ub514\uc544\ube44 \ub300\uc2e0 \ubbf8\ub4dc\ud544\ub354\ub85c \ucd9c\uc804\ud574 \uc88b\uc740 \ud328\uc2a4\ub97c \ucc14\ub7ec\uc8fc\uba70 \ud65c\uc57d\ud574 \ud32c\ub4e4\uc758 \uae30\ub300\uac10\uc744 \ub298\ub838\ub2e4. 1\uc6d4 27\uc77c FA\ucef5 \ube0c\ub77c\uc774\ud2bc\uc804\uc5d0\uc11c \ub2e4\uc2dc \ub525-\ub77c\uc789 \ud50c\ub808\uc774\uba54\uc774\ucee4\ub85c \ucd9c\uc804\ud574 \uad1c\ucc2e\uc740 \ubaa8\uc2b5\uc744 \uc774\uc5b4\uac14\ub2e4.", "sentence_answer": " 2013\ub144 1\uc6d4 24\uc77c, 5:1 \ub300\uc2b9\uc744 \uac70\ub454 \uc6e8\uc2a4\ud2b8\ud584\uc804\uc5d0\uc11c \ubd80\uc0c1\ub2f9\ud55c \ubbf8\ucf08 \uc544\ub974\ud14c\ud0c0\uc640 \ud734\uc2dd\uc744 \ubd80\uc5ec\ubc1b\uc740 \uc544\ubd80 \ub514\uc544\ube44 \ub300\uc2e0 \ubbf8\ub4dc\ud544\ub354\ub85c \ucd9c\uc804\ud574 \uc88b\uc740 \ud328\uc2a4\ub97c \ucc14\ub7ec\uc8fc\uba70 \ud65c\uc57d\ud574 \ud32c\ub4e4\uc758 \uae30\ub300\uac10\uc744 \ub298\ub838\ub2e4.", "paragraph_id": "6540618-6-1", "paragraph_question": "question: \uc5d0\ub7f0 \ub7a8\uc9c0\uac00 \uc544\ubd80 \ub514\uc544\ube44 \ub300\uc2e0 \ubbf8\ub4dc\ud544\ub354\ub85c \ucd9c\uc804\ud55c \ub144\ub3c4\ub294?, context: \uccab \ub9ac\uadf8 4\uacbd\uae30\uc5d0\uc11c\ub294 \ud6c4\ubcf4\ub85c \ub6f0\uc5c8\ub2e4. 2012\ub144 9\uc6d4 15\uc77c 6:1\ub85c \ub300\uc2b9\uc744 \uac70\ub454 \uc0ac\uc6b0\uc2a4\ud584\ud2bc FC\uc804\uc5d0\uc11c \ud504\ub79c\uc2dc\uc2a4 \ucf54\ud074\ub7ad \ub300\uc2e0 \uad50\uccb4\ud22c\uc785\ub3fc \uace8\ub300\ub97c \ub9de\ucdb0 5\ubc88\uc9f8 \uace8\uc744 \uc774\ub04c\uc5b4\ub0c8\ub2e4. 9\uc6d4 23\uc77c \ub514\ud39c\ub529 \ucc54\ud53c\uc5b8 \ub9e8\uccb4\uc2a4\ud130 \uc2dc\ud2f0\uc804\uc5d0\uc11c \uccab \uc120\ubc1c \ucd9c\uc804\uc744 \ud574 \uc624\ub978\ucabd \ubbf8\ub4dc\ud544\ub354 \uc5ed\ud560\uc744 \ub9e1\uc544 \uad1c\ucc2e\uc740 \uacbd\uae30\ub97c \ud3bc\ucce4\ub2e4. 10\uc6d4 3\uc77c \uc62c\ub9bc\ud53c\uc544\ucf54\uc2a4 FC\uc804\uc5d0\uc11c \uc624\ub978\ubc1c \uce69\uc0f7\uc73c\ub85c \ub4dd\uc810\ud574 \ud300\uc758 3:1 \uc2b9\ub9ac\uc5d0 \uae30\uc5ec\ud588\ub2e4. 12\uc6d4 19\uc77c \ud300 \ub3d9\ub8cc \uc568\ub9ad\uc2a4 \uc625\uc2ac\ub808\uc774\ub4dc\uccb4\uc784\ubc8c\ub9b0, \ud0a4\uc5b4\ub7f0 \uae41\uc2a4, \uc7ad \uc70c\uc154, \uce7c \uc831\ud0a8\uc2a8\uacfc \ud568\uaed8 \uc7a5\uae30\uacc4\uc57d\uc744 \uccb4\uacb0\ud588\ub2e4. 2013\ub144 1\uc6d4 24\uc77c, 5:1 \ub300\uc2b9\uc744 \uac70\ub454 \uc6e8\uc2a4\ud2b8\ud584\uc804\uc5d0\uc11c \ubd80\uc0c1\ub2f9\ud55c \ubbf8\ucf08 \uc544\ub974\ud14c\ud0c0\uc640 \ud734\uc2dd\uc744 \ubd80\uc5ec\ubc1b\uc740 \uc544\ubd80 \ub514\uc544\ube44 \ub300\uc2e0 \ubbf8\ub4dc\ud544\ub354\ub85c \ucd9c\uc804\ud574 \uc88b\uc740 \ud328\uc2a4\ub97c \ucc14\ub7ec\uc8fc\uba70 \ud65c\uc57d\ud574 \ud32c\ub4e4\uc758 \uae30\ub300\uac10\uc744 \ub298\ub838\ub2e4. 1\uc6d4 27\uc77c FA\ucef5 \ube0c\ub77c\uc774\ud2bc\uc804\uc5d0\uc11c \ub2e4\uc2dc \ub525-\ub77c\uc789 \ud50c\ub808\uc774\uba54\uc774\ucee4\ub85c \ucd9c\uc804\ud574 \uad1c\ucc2e\uc740 \ubaa8\uc2b5\uc744 \uc774\uc5b4\uac14\ub2e4."} {"question": "\uc784\uc6d0 \uc911\uc5d0 \ub300\ud55c\ubbfc\uad6d \uad6d\ubbfc\uc774 \uc544\ub2cc \uc0ac\ub78c\uc740 \uad6d\ub0b4\uad6d\uc81c\ud56d\uacf5\uc6b4\uc1a1\uc0ac\uc5c5 \uba74\ud5c8\uc758 \uacb0\uaca9\uc0ac\uc720\ub85c \ubc95\uc73c\ub85c \uc815\ud574\uc838 \uc788\ub294 \ubc95\uc774\ub984\uc740?", "paragraph": "\uc870\ud604\ubbfc \uc9c4\uc5d0\uc5b4 \ub9c8\ucf00\ud305\ubcf8\ubd80 \ubcf8\ubd80\uc7a5 (\ubd80\uc0ac\uc7a5)\uc774 2010\ub144\ubd80\ud130 2016\ub144 \uae4c\uc9c0 6\ub144\uac04 \ubd88\ubc95\uc73c\ub85c \uc9c4\uc5d0\uc5b4 \ub4f1\uae30\uc784\uc6d0\uc5d0 \uc62c\ub790\ub358 \uac83\uc73c\ub85c \ud655\uc778\ub418\uc5b4 \ub17c\ub780\uc774 \uc77c\uc5c8\ub2e4. 2018\ub144 '\ubb3c\ubcbc\ub77d \uac11\uc9c8 \ub17c\ub780'\uc73c\ub85c \uc0ac\ud1f4 \uc555\ub825\uc774 \uac70\uc138\uc9c0\ub294 \uac00\uc6b4\ub370 \uc870\ud604\ubbfc \ubd80\uc0ac\uc7a5\uc774 \uc624\ub7ab\ub3d9\uc548 \ubd88\ubc95\uc801\uc778 \uacbd\uc601\uc9c0\uc704\ub97c \ub204\ub9b0 \uc0ac\uc2e4\uc774 \ucd94\uac00\ub85c \ub4dc\ub7ec\ub098\uba74\uc11c \ub610 \ub2e4\ub978 \ub17c\ub780\uc774 \ub418\uace0 \uc788\ub2e4. \uc5f0\ud569\ub274\uc2a4\uac00 \uae08\uc735\uac10\ub3c5\uc6d0 \uc804\uc790\uacf5\uc2dc\uc2dc\uc2a4\ud15c \ub4f1\uc744 \ud1b5\ud574 \ud655\uc778\ud55c \uc9c4\uc5d0\uc5b4 \uad00\ub828 \uacf5\uc2dc\ub97c \uc885\ud569\ud558\uba74 '\uc870 \uc5d0\ubc00\ub9ac \ub9ac'(CHO EMILY LEE)\ub77c\ub294 \uc778\ubb3c\uc774 2010\ub144 3\uc6d4\ubd80\ud130 2016\ub144 3\uc6d4\uae4c\uc9c0 \uc9c4\uc5d0\uc5b4 \uc0ac\ub0b4\uc774\uc0ac\ub85c \ub4f1\uc7ac\ub410\ub2e4. \uc678\uad6d\uc778\uc774 \uad6d\uc801\ud56d\uacf5\uc0ac \ub4f1\uae30\uc784\uc6d0\uc73c\ub85c \uc624\ub978 \uac83\uc740 \uba85\ubc31\ud55c \ubd88\ubc95\uc774\ub2e4. \ud56d\uacf5\uc0ac\uc5c5\ubc95 \uc81c9\uc870\ub294 '\uad6d\ub0b4\u00b7\uad6d\uc81c\ud56d\uacf5\uc6b4\uc1a1\uc0ac\uc5c5 \uba74\ud5c8\uc758 \uacb0\uaca9\uc0ac\uc720' \uc911 \ud558\ub098\ub85c \uc784\uc6d0 \uc911\uc5d0 '\ub300\ud55c\ubbfc\uad6d \uad6d\ubbfc\uc774 \uc544\ub2cc \uc0ac\ub78c'\uc774 \uc788\ub294 \uacbd\uc6b0\ub97c \uaf3d\uace0 \uc788\ub2e4. \uc9c4\uc5d0\uc5b4 \uad00\uacc4\uc790\uc5d0 \ub530\ub974\uba74 \uc870\ud604\ubbfc \ubd80\uc0ac\uc7a5\uc740 \ub2f9\uc2dc \ub17c\ub780\uc758 \uc18c\uc9c0\uac00 \uc788\uc5b4 2016\ub144\uc5d0 \ub4f1\uae30 \uc784\uc6d0\uc744 \uc0ac\uc784\ud55c \uac83\uc73c\ub85c \uc548\ub2e4\uace0 \ubc1d\ud614\ub2e4.", "answer": "\ud56d\uacf5\uc0ac\uc5c5\ubc95", "sentence": "\ud56d\uacf5\uc0ac\uc5c5\ubc95 \uc81c9\uc870\ub294 '\uad6d\ub0b4\u00b7\uad6d\uc81c\ud56d\uacf5\uc6b4\uc1a1\uc0ac\uc5c5 \uba74\ud5c8\uc758 \uacb0\uaca9\uc0ac\uc720' \uc911 \ud558\ub098\ub85c \uc784\uc6d0 \uc911\uc5d0 '\ub300\ud55c\ubbfc\uad6d \uad6d\ubbfc\uc774 \uc544\ub2cc \uc0ac\ub78c'\uc774 \uc788\ub294 \uacbd\uc6b0\ub97c \uaf3d\uace0 \uc788\ub2e4.", "paragraph_sentence": "\uc870\ud604\ubbfc \uc9c4\uc5d0\uc5b4 \ub9c8\ucf00\ud305\ubcf8\ubd80 \ubcf8\ubd80\uc7a5 (\ubd80\uc0ac\uc7a5)\uc774 2010\ub144\ubd80\ud130 2016\ub144 \uae4c\uc9c0 6\ub144\uac04 \ubd88\ubc95\uc73c\ub85c \uc9c4\uc5d0\uc5b4 \ub4f1\uae30\uc784\uc6d0\uc5d0 \uc62c\ub790\ub358 \uac83\uc73c\ub85c \ud655\uc778\ub418\uc5b4 \ub17c\ub780\uc774 \uc77c\uc5c8\ub2e4. 2018\ub144 '\ubb3c\ubcbc\ub77d \uac11\uc9c8 \ub17c\ub780'\uc73c\ub85c \uc0ac\ud1f4 \uc555\ub825\uc774 \uac70\uc138\uc9c0\ub294 \uac00\uc6b4\ub370 \uc870\ud604\ubbfc \ubd80\uc0ac\uc7a5\uc774 \uc624\ub7ab\ub3d9\uc548 \ubd88\ubc95\uc801\uc778 \uacbd\uc601\uc9c0\uc704\ub97c \ub204\ub9b0 \uc0ac\uc2e4\uc774 \ucd94\uac00\ub85c \ub4dc\ub7ec\ub098\uba74\uc11c \ub610 \ub2e4\ub978 \ub17c\ub780\uc774 \ub418\uace0 \uc788\ub2e4. \uc5f0\ud569\ub274\uc2a4\uac00 \uae08\uc735\uac10\ub3c5\uc6d0 \uc804\uc790\uacf5\uc2dc\uc2dc\uc2a4\ud15c \ub4f1\uc744 \ud1b5\ud574 \ud655\uc778\ud55c \uc9c4\uc5d0\uc5b4 \uad00\ub828 \uacf5\uc2dc\ub97c \uc885\ud569\ud558\uba74 '\uc870 \uc5d0\ubc00\ub9ac \ub9ac'(CHO EMILY LEE)\ub77c\ub294 \uc778\ubb3c\uc774 2010\ub144 3\uc6d4\ubd80\ud130 2016\ub144 3\uc6d4\uae4c\uc9c0 \uc9c4\uc5d0\uc5b4 \uc0ac\ub0b4\uc774\uc0ac\ub85c \ub4f1\uc7ac\ub410\ub2e4. \uc678\uad6d\uc778\uc774 \uad6d\uc801\ud56d\uacf5\uc0ac \ub4f1\uae30\uc784\uc6d0\uc73c\ub85c \uc624\ub978 \uac83\uc740 \uba85\ubc31\ud55c \ubd88\ubc95\uc774\ub2e4. \ud56d\uacf5\uc0ac\uc5c5\ubc95 \uc81c9\uc870\ub294 '\uad6d\ub0b4\u00b7\uad6d\uc81c\ud56d\uacf5\uc6b4\uc1a1\uc0ac\uc5c5 \uba74\ud5c8\uc758 \uacb0\uaca9\uc0ac\uc720' \uc911 \ud558\ub098\ub85c \uc784\uc6d0 \uc911\uc5d0 '\ub300\ud55c\ubbfc\uad6d \uad6d\ubbfc\uc774 \uc544\ub2cc \uc0ac\ub78c'\uc774 \uc788\ub294 \uacbd\uc6b0\ub97c \uaf3d\uace0 \uc788\ub2e4. \uc9c4\uc5d0\uc5b4 \uad00\uacc4\uc790\uc5d0 \ub530\ub974\uba74 \uc870\ud604\ubbfc \ubd80\uc0ac\uc7a5\uc740 \ub2f9\uc2dc \ub17c\ub780\uc758 \uc18c\uc9c0\uac00 \uc788\uc5b4 2016\ub144\uc5d0 \ub4f1\uae30 \uc784\uc6d0\uc744 \uc0ac\uc784\ud55c \uac83\uc73c\ub85c \uc548\ub2e4\uace0 \ubc1d\ud614\ub2e4.", "paragraph_answer": "\uc870\ud604\ubbfc \uc9c4\uc5d0\uc5b4 \ub9c8\ucf00\ud305\ubcf8\ubd80 \ubcf8\ubd80\uc7a5 (\ubd80\uc0ac\uc7a5)\uc774 2010\ub144\ubd80\ud130 2016\ub144 \uae4c\uc9c0 6\ub144\uac04 \ubd88\ubc95\uc73c\ub85c \uc9c4\uc5d0\uc5b4 \ub4f1\uae30\uc784\uc6d0\uc5d0 \uc62c\ub790\ub358 \uac83\uc73c\ub85c \ud655\uc778\ub418\uc5b4 \ub17c\ub780\uc774 \uc77c\uc5c8\ub2e4. 2018\ub144 '\ubb3c\ubcbc\ub77d \uac11\uc9c8 \ub17c\ub780'\uc73c\ub85c \uc0ac\ud1f4 \uc555\ub825\uc774 \uac70\uc138\uc9c0\ub294 \uac00\uc6b4\ub370 \uc870\ud604\ubbfc \ubd80\uc0ac\uc7a5\uc774 \uc624\ub7ab\ub3d9\uc548 \ubd88\ubc95\uc801\uc778 \uacbd\uc601\uc9c0\uc704\ub97c \ub204\ub9b0 \uc0ac\uc2e4\uc774 \ucd94\uac00\ub85c \ub4dc\ub7ec\ub098\uba74\uc11c \ub610 \ub2e4\ub978 \ub17c\ub780\uc774 \ub418\uace0 \uc788\ub2e4. \uc5f0\ud569\ub274\uc2a4\uac00 \uae08\uc735\uac10\ub3c5\uc6d0 \uc804\uc790\uacf5\uc2dc\uc2dc\uc2a4\ud15c \ub4f1\uc744 \ud1b5\ud574 \ud655\uc778\ud55c \uc9c4\uc5d0\uc5b4 \uad00\ub828 \uacf5\uc2dc\ub97c \uc885\ud569\ud558\uba74 '\uc870 \uc5d0\ubc00\ub9ac \ub9ac'(CHO EMILY LEE)\ub77c\ub294 \uc778\ubb3c\uc774 2010\ub144 3\uc6d4\ubd80\ud130 2016\ub144 3\uc6d4\uae4c\uc9c0 \uc9c4\uc5d0\uc5b4 \uc0ac\ub0b4\uc774\uc0ac\ub85c \ub4f1\uc7ac\ub410\ub2e4. \uc678\uad6d\uc778\uc774 \uad6d\uc801\ud56d\uacf5\uc0ac \ub4f1\uae30\uc784\uc6d0\uc73c\ub85c \uc624\ub978 \uac83\uc740 \uba85\ubc31\ud55c \ubd88\ubc95\uc774\ub2e4. \ud56d\uacf5\uc0ac\uc5c5\ubc95 \uc81c9\uc870\ub294 '\uad6d\ub0b4\u00b7\uad6d\uc81c\ud56d\uacf5\uc6b4\uc1a1\uc0ac\uc5c5 \uba74\ud5c8\uc758 \uacb0\uaca9\uc0ac\uc720' \uc911 \ud558\ub098\ub85c \uc784\uc6d0 \uc911\uc5d0 '\ub300\ud55c\ubbfc\uad6d \uad6d\ubbfc\uc774 \uc544\ub2cc \uc0ac\ub78c'\uc774 \uc788\ub294 \uacbd\uc6b0\ub97c \uaf3d\uace0 \uc788\ub2e4. \uc9c4\uc5d0\uc5b4 \uad00\uacc4\uc790\uc5d0 \ub530\ub974\uba74 \uc870\ud604\ubbfc \ubd80\uc0ac\uc7a5\uc740 \ub2f9\uc2dc \ub17c\ub780\uc758 \uc18c\uc9c0\uac00 \uc788\uc5b4 2016\ub144\uc5d0 \ub4f1\uae30 \uc784\uc6d0\uc744 \uc0ac\uc784\ud55c \uac83\uc73c\ub85c \uc548\ub2e4\uace0 \ubc1d\ud614\ub2e4.", "sentence_answer": " \ud56d\uacf5\uc0ac\uc5c5\ubc95 \uc81c9\uc870\ub294 '\uad6d\ub0b4\u00b7\uad6d\uc81c\ud56d\uacf5\uc6b4\uc1a1\uc0ac\uc5c5 \uba74\ud5c8\uc758 \uacb0\uaca9\uc0ac\uc720' \uc911 \ud558\ub098\ub85c \uc784\uc6d0 \uc911\uc5d0 '\ub300\ud55c\ubbfc\uad6d \uad6d\ubbfc\uc774 \uc544\ub2cc \uc0ac\ub78c'\uc774 \uc788\ub294 \uacbd\uc6b0\ub97c \uaf3d\uace0 \uc788\ub2e4.", "paragraph_id": "5886467-0-2", "paragraph_question": "question: \uc784\uc6d0 \uc911\uc5d0 \ub300\ud55c\ubbfc\uad6d \uad6d\ubbfc\uc774 \uc544\ub2cc \uc0ac\ub78c\uc740 \uad6d\ub0b4\uad6d\uc81c\ud56d\uacf5\uc6b4\uc1a1\uc0ac\uc5c5 \uba74\ud5c8\uc758 \uacb0\uaca9\uc0ac\uc720\ub85c \ubc95\uc73c\ub85c \uc815\ud574\uc838 \uc788\ub294 \ubc95\uc774\ub984\uc740?, context: \uc870\ud604\ubbfc \uc9c4\uc5d0\uc5b4 \ub9c8\ucf00\ud305\ubcf8\ubd80 \ubcf8\ubd80\uc7a5 (\ubd80\uc0ac\uc7a5)\uc774 2010\ub144\ubd80\ud130 2016\ub144 \uae4c\uc9c0 6\ub144\uac04 \ubd88\ubc95\uc73c\ub85c \uc9c4\uc5d0\uc5b4 \ub4f1\uae30\uc784\uc6d0\uc5d0 \uc62c\ub790\ub358 \uac83\uc73c\ub85c \ud655\uc778\ub418\uc5b4 \ub17c\ub780\uc774 \uc77c\uc5c8\ub2e4. 2018\ub144 '\ubb3c\ubcbc\ub77d \uac11\uc9c8 \ub17c\ub780'\uc73c\ub85c \uc0ac\ud1f4 \uc555\ub825\uc774 \uac70\uc138\uc9c0\ub294 \uac00\uc6b4\ub370 \uc870\ud604\ubbfc \ubd80\uc0ac\uc7a5\uc774 \uc624\ub7ab\ub3d9\uc548 \ubd88\ubc95\uc801\uc778 \uacbd\uc601\uc9c0\uc704\ub97c \ub204\ub9b0 \uc0ac\uc2e4\uc774 \ucd94\uac00\ub85c \ub4dc\ub7ec\ub098\uba74\uc11c \ub610 \ub2e4\ub978 \ub17c\ub780\uc774 \ub418\uace0 \uc788\ub2e4. \uc5f0\ud569\ub274\uc2a4\uac00 \uae08\uc735\uac10\ub3c5\uc6d0 \uc804\uc790\uacf5\uc2dc\uc2dc\uc2a4\ud15c \ub4f1\uc744 \ud1b5\ud574 \ud655\uc778\ud55c \uc9c4\uc5d0\uc5b4 \uad00\ub828 \uacf5\uc2dc\ub97c \uc885\ud569\ud558\uba74 '\uc870 \uc5d0\ubc00\ub9ac \ub9ac'(CHO EMILY LEE)\ub77c\ub294 \uc778\ubb3c\uc774 2010\ub144 3\uc6d4\ubd80\ud130 2016\ub144 3\uc6d4\uae4c\uc9c0 \uc9c4\uc5d0\uc5b4 \uc0ac\ub0b4\uc774\uc0ac\ub85c \ub4f1\uc7ac\ub410\ub2e4. \uc678\uad6d\uc778\uc774 \uad6d\uc801\ud56d\uacf5\uc0ac \ub4f1\uae30\uc784\uc6d0\uc73c\ub85c \uc624\ub978 \uac83\uc740 \uba85\ubc31\ud55c \ubd88\ubc95\uc774\ub2e4. \ud56d\uacf5\uc0ac\uc5c5\ubc95 \uc81c9\uc870\ub294 '\uad6d\ub0b4\u00b7\uad6d\uc81c\ud56d\uacf5\uc6b4\uc1a1\uc0ac\uc5c5 \uba74\ud5c8\uc758 \uacb0\uaca9\uc0ac\uc720' \uc911 \ud558\ub098\ub85c \uc784\uc6d0 \uc911\uc5d0 '\ub300\ud55c\ubbfc\uad6d \uad6d\ubbfc\uc774 \uc544\ub2cc \uc0ac\ub78c'\uc774 \uc788\ub294 \uacbd\uc6b0\ub97c \uaf3d\uace0 \uc788\ub2e4. \uc9c4\uc5d0\uc5b4 \uad00\uacc4\uc790\uc5d0 \ub530\ub974\uba74 \uc870\ud604\ubbfc \ubd80\uc0ac\uc7a5\uc740 \ub2f9\uc2dc \ub17c\ub780\uc758 \uc18c\uc9c0\uac00 \uc788\uc5b4 2016\ub144\uc5d0 \ub4f1\uae30 \uc784\uc6d0\uc744 \uc0ac\uc784\ud55c \uac83\uc73c\ub85c \uc548\ub2e4\uace0 \ubc1d\ud614\ub2e4."} {"question": "\uc758\uc0ac \ub9d0\ucf64\uc774 \uc18c\ub144 \ucf5c\uc774 \uac04\uc9c1\ud55c \ub450\ub824\uc6c0\uc758 \uc6d0\uc778\uc73c\ub85c \ubcf8\uac83\uc740?", "paragraph": "\ubd80\ubaa8\uc758 \uc774\ud63c\uc73c\ub85c \ucda9\uaca9\uc744 \ubc1b\uc740 \uac83\uc73c\ub85c \ub9d0\ucf64\uc740 \uc774\ud574\ud558\ub824 \ud558\uc9c0\ub9cc, \uc18c\ub144\uc740 \uae4a\ud788 \uac04\uc9c1\ub41c \uc9c4\uc2e4\ud55c \ub450\ub824\uc6c0\uc758 \uc6d0\uc778\uc744 \ub9d0\ucf64\uc740 \ubb3c\ub860 \uc5c4\ub9c8(\ud1a0\ub2c8 \ucf5c\ub81b)\uc5d0\uac8c\ub3c4 \ubc1d\ud788\uae30\ub97c \uaebc\ub824\ud55c\ub2e4. \uce58\ub8cc\uacfc\uc815\uc774 \uc2ec\ub3c4\ub97c \ub354\ud574 \uac00\uba74\uc11c \uc18c\ub144\uc740 \ub9d0\ucf64\uc744 \uc2e0\ub8b0\ud558\uac8c \ub418\uace0, \uc790\uc2e0\uc774 \ub610 \ub2e4\ub978 \uac10\uac01\uc744 \uc9c0\ub154\uc74c\uc744 \uadf8\uc5d0\uac8c \uc54c\ub9b0\ub2e4. \uc18c\ub144\uc740 \uc8fd\uc740 \uc0ac\ub78c\ub4e4\uc744 \ubcfc \uc218 \uc788\uc744 \ubfd0\ub9cc \uc544\ub2c8\ub77c \uadf8\ub4e4\uacfc \uc774\uc57c\uae30\ub97c \ud560 \uc218 \uc788\ub294 \uc9c0\uac01\uc744 \uac00\uc9c4 \uac83\uc774\ub2e4. \uc8fd\uc740 \uc18c\ub140\uac00 \ub098\ud0c0\ub098 \ud1a0\ud558\uba74\uc11c \uba54\uc2dc\uc9c0\ub97c \uc804\ud558\uace0, \uc8fd\uc740 \uc5ec\uc778\uc740 \ub0a8\ud3b8\uc744 \ud5a5\ud574 \uc18c\ub9ac\uc9c0\ub974\ub294 \ub4ef\uc774 \ucf5c\uc5d0\uac8c \uc790\uc2e0\uc758 \uc5b5\uc6b8\ud568\uc744 \ud138\uc5b4\ub193\ub294\uac00 \ud558\uba74, 100\ub144\uc804 \uad50\uc218\ud615\uc5d0 \ucc98\ud574\uc84c\ub358 \uac00\uc871\uc758 \ucc98\ucc38\ud55c \ubaa8\uc2b5\uc774 \uc18c\ub144\uc5d0\uac8c \ubcf4\uc774\ub294 \uac83\uc774\ub2e4. \ub9d0\ucf64\uc740 \ubbff\uc744 \uc218 \uc5c6\ub294 \uc0ac\uc2e4\uc5d0 \uc758\ud639\uc744 \ud488\uc73c\uba74\uc11c\ub3c4 \ubaa8\ub4e0 \uc218\ub2e8\uc744 \uc774\uc6a9\ud558\uc5ec \uc18c\ub144\uc758 \uace0\ub09c\uc2a4\ub7ec\uc6b4 \uc138\uacc4\ub85c \ud568\uaed8 \ud30c\uace0\ub4e0\ub2e4. \ud55c\ud3b8 \ub9d0\ucf64\uc740 \uac1c\uc778\uc801\uc778 \uc0ac\uc0dd\ud65c\uc758 \uc704\uae30\ub3c4 \ub9de\ub294\ub2e4. \uadf8\uc758 \uc544\ub0b4\ub294 \uacb0\ud63c\uc2dd\ub54c \ucc0d\uc740 \ube44\ub514\uc624\ub9cc\uc744 \uacc4\uc18d \ubcf4\uba74\uc11c \uacfc\uac70 \ub0a8\ud3b8\uacfc\uc758 \uc990\uac70\uc6e0\ub358 \uc2dc\uc808\ub9cc\uc744 \uadf8\ub9ac\uba70 \uc0dd\ud65c\ud560 \ubfd0, \ub9d0\ucf64\uacfc\ub294 \ub354 \uc774\uc0c1 \ub300\ud654\ub97c \ud558\uc9c0 \uc54a\uace0, \uc80a\uc740 \ub0a8\uc790\ub97c \ub9cc\ub098\uba74\uc11c \uc678\ub3c4\ub97c \ubc94\ud558\ub294\ub370\u2026", "answer": "\ubd80\ubaa8\uc758 \uc774\ud63c", "sentence": "\ubd80\ubaa8\uc758 \uc774\ud63c \uc73c\ub85c \ucda9\uaca9\uc744 \ubc1b\uc740 \uac83\uc73c\ub85c \ub9d0\ucf64\uc740 \uc774\ud574\ud558\ub824 \ud558\uc9c0\ub9cc, \uc18c\ub144\uc740 \uae4a\ud788 \uac04\uc9c1\ub41c \uc9c4\uc2e4\ud55c \ub450\ub824\uc6c0\uc758 \uc6d0\uc778\uc744 \ub9d0\ucf64\uc740 \ubb3c\ub860 \uc5c4\ub9c8(\ud1a0\ub2c8 \ucf5c\ub81b)\uc5d0\uac8c\ub3c4 \ubc1d\ud788\uae30\ub97c \uaebc\ub824\ud55c\ub2e4.", "paragraph_sentence": " \ubd80\ubaa8\uc758 \uc774\ud63c \uc73c\ub85c \ucda9\uaca9\uc744 \ubc1b\uc740 \uac83\uc73c\ub85c \ub9d0\ucf64\uc740 \uc774\ud574\ud558\ub824 \ud558\uc9c0\ub9cc, \uc18c\ub144\uc740 \uae4a\ud788 \uac04\uc9c1\ub41c \uc9c4\uc2e4\ud55c \ub450\ub824\uc6c0\uc758 \uc6d0\uc778\uc744 \ub9d0\ucf64\uc740 \ubb3c\ub860 \uc5c4\ub9c8(\ud1a0\ub2c8 \ucf5c\ub81b)\uc5d0\uac8c\ub3c4 \ubc1d\ud788\uae30\ub97c \uaebc\ub824\ud55c\ub2e4. \uce58\ub8cc\uacfc\uc815\uc774 \uc2ec\ub3c4\ub97c \ub354\ud574 \uac00\uba74\uc11c \uc18c\ub144\uc740 \ub9d0\ucf64\uc744 \uc2e0\ub8b0\ud558\uac8c \ub418\uace0, \uc790\uc2e0\uc774 \ub610 \ub2e4\ub978 \uac10\uac01\uc744 \uc9c0\ub154\uc74c\uc744 \uadf8\uc5d0\uac8c \uc54c\ub9b0\ub2e4. \uc18c\ub144\uc740 \uc8fd\uc740 \uc0ac\ub78c\ub4e4\uc744 \ubcfc \uc218 \uc788\uc744 \ubfd0\ub9cc \uc544\ub2c8\ub77c \uadf8\ub4e4\uacfc \uc774\uc57c\uae30\ub97c \ud560 \uc218 \uc788\ub294 \uc9c0\uac01\uc744 \uac00\uc9c4 \uac83\uc774\ub2e4. \uc8fd\uc740 \uc18c\ub140\uac00 \ub098\ud0c0\ub098 \ud1a0\ud558\uba74\uc11c \uba54\uc2dc\uc9c0\ub97c \uc804\ud558\uace0, \uc8fd\uc740 \uc5ec\uc778\uc740 \ub0a8\ud3b8\uc744 \ud5a5\ud574 \uc18c\ub9ac\uc9c0\ub974\ub294 \ub4ef\uc774 \ucf5c\uc5d0\uac8c \uc790\uc2e0\uc758 \uc5b5\uc6b8\ud568\uc744 \ud138\uc5b4\ub193\ub294\uac00 \ud558\uba74, 100\ub144\uc804 \uad50\uc218\ud615\uc5d0 \ucc98\ud574\uc84c\ub358 \uac00\uc871\uc758 \ucc98\ucc38\ud55c \ubaa8\uc2b5\uc774 \uc18c\ub144\uc5d0\uac8c \ubcf4\uc774\ub294 \uac83\uc774\ub2e4. \ub9d0\ucf64\uc740 \ubbff\uc744 \uc218 \uc5c6\ub294 \uc0ac\uc2e4\uc5d0 \uc758\ud639\uc744 \ud488\uc73c\uba74\uc11c\ub3c4 \ubaa8\ub4e0 \uc218\ub2e8\uc744 \uc774\uc6a9\ud558\uc5ec \uc18c\ub144\uc758 \uace0\ub09c\uc2a4\ub7ec\uc6b4 \uc138\uacc4\ub85c \ud568\uaed8 \ud30c\uace0\ub4e0\ub2e4. \ud55c\ud3b8 \ub9d0\ucf64\uc740 \uac1c\uc778\uc801\uc778 \uc0ac\uc0dd\ud65c\uc758 \uc704\uae30\ub3c4 \ub9de\ub294\ub2e4. \uadf8\uc758 \uc544\ub0b4\ub294 \uacb0\ud63c\uc2dd\ub54c \ucc0d\uc740 \ube44\ub514\uc624\ub9cc\uc744 \uacc4\uc18d \ubcf4\uba74\uc11c \uacfc\uac70 \ub0a8\ud3b8\uacfc\uc758 \uc990\uac70\uc6e0\ub358 \uc2dc\uc808\ub9cc\uc744 \uadf8\ub9ac\uba70 \uc0dd\ud65c\ud560 \ubfd0, \ub9d0\ucf64\uacfc\ub294 \ub354 \uc774\uc0c1 \ub300\ud654\ub97c \ud558\uc9c0 \uc54a\uace0, \uc80a\uc740 \ub0a8\uc790\ub97c \ub9cc\ub098\uba74\uc11c \uc678\ub3c4\ub97c \ubc94\ud558\ub294\ub370 \u2026", "paragraph_answer": " \ubd80\ubaa8\uc758 \uc774\ud63c \uc73c\ub85c \ucda9\uaca9\uc744 \ubc1b\uc740 \uac83\uc73c\ub85c \ub9d0\ucf64\uc740 \uc774\ud574\ud558\ub824 \ud558\uc9c0\ub9cc, \uc18c\ub144\uc740 \uae4a\ud788 \uac04\uc9c1\ub41c \uc9c4\uc2e4\ud55c \ub450\ub824\uc6c0\uc758 \uc6d0\uc778\uc744 \ub9d0\ucf64\uc740 \ubb3c\ub860 \uc5c4\ub9c8(\ud1a0\ub2c8 \ucf5c\ub81b)\uc5d0\uac8c\ub3c4 \ubc1d\ud788\uae30\ub97c \uaebc\ub824\ud55c\ub2e4. \uce58\ub8cc\uacfc\uc815\uc774 \uc2ec\ub3c4\ub97c \ub354\ud574 \uac00\uba74\uc11c \uc18c\ub144\uc740 \ub9d0\ucf64\uc744 \uc2e0\ub8b0\ud558\uac8c \ub418\uace0, \uc790\uc2e0\uc774 \ub610 \ub2e4\ub978 \uac10\uac01\uc744 \uc9c0\ub154\uc74c\uc744 \uadf8\uc5d0\uac8c \uc54c\ub9b0\ub2e4. \uc18c\ub144\uc740 \uc8fd\uc740 \uc0ac\ub78c\ub4e4\uc744 \ubcfc \uc218 \uc788\uc744 \ubfd0\ub9cc \uc544\ub2c8\ub77c \uadf8\ub4e4\uacfc \uc774\uc57c\uae30\ub97c \ud560 \uc218 \uc788\ub294 \uc9c0\uac01\uc744 \uac00\uc9c4 \uac83\uc774\ub2e4. \uc8fd\uc740 \uc18c\ub140\uac00 \ub098\ud0c0\ub098 \ud1a0\ud558\uba74\uc11c \uba54\uc2dc\uc9c0\ub97c \uc804\ud558\uace0, \uc8fd\uc740 \uc5ec\uc778\uc740 \ub0a8\ud3b8\uc744 \ud5a5\ud574 \uc18c\ub9ac\uc9c0\ub974\ub294 \ub4ef\uc774 \ucf5c\uc5d0\uac8c \uc790\uc2e0\uc758 \uc5b5\uc6b8\ud568\uc744 \ud138\uc5b4\ub193\ub294\uac00 \ud558\uba74, 100\ub144\uc804 \uad50\uc218\ud615\uc5d0 \ucc98\ud574\uc84c\ub358 \uac00\uc871\uc758 \ucc98\ucc38\ud55c \ubaa8\uc2b5\uc774 \uc18c\ub144\uc5d0\uac8c \ubcf4\uc774\ub294 \uac83\uc774\ub2e4. \ub9d0\ucf64\uc740 \ubbff\uc744 \uc218 \uc5c6\ub294 \uc0ac\uc2e4\uc5d0 \uc758\ud639\uc744 \ud488\uc73c\uba74\uc11c\ub3c4 \ubaa8\ub4e0 \uc218\ub2e8\uc744 \uc774\uc6a9\ud558\uc5ec \uc18c\ub144\uc758 \uace0\ub09c\uc2a4\ub7ec\uc6b4 \uc138\uacc4\ub85c \ud568\uaed8 \ud30c\uace0\ub4e0\ub2e4. \ud55c\ud3b8 \ub9d0\ucf64\uc740 \uac1c\uc778\uc801\uc778 \uc0ac\uc0dd\ud65c\uc758 \uc704\uae30\ub3c4 \ub9de\ub294\ub2e4. \uadf8\uc758 \uc544\ub0b4\ub294 \uacb0\ud63c\uc2dd\ub54c \ucc0d\uc740 \ube44\ub514\uc624\ub9cc\uc744 \uacc4\uc18d \ubcf4\uba74\uc11c \uacfc\uac70 \ub0a8\ud3b8\uacfc\uc758 \uc990\uac70\uc6e0\ub358 \uc2dc\uc808\ub9cc\uc744 \uadf8\ub9ac\uba70 \uc0dd\ud65c\ud560 \ubfd0, \ub9d0\ucf64\uacfc\ub294 \ub354 \uc774\uc0c1 \ub300\ud654\ub97c \ud558\uc9c0 \uc54a\uace0, \uc80a\uc740 \ub0a8\uc790\ub97c \ub9cc\ub098\uba74\uc11c \uc678\ub3c4\ub97c \ubc94\ud558\ub294\ub370\u2026", "sentence_answer": " \ubd80\ubaa8\uc758 \uc774\ud63c \uc73c\ub85c \ucda9\uaca9\uc744 \ubc1b\uc740 \uac83\uc73c\ub85c \ub9d0\ucf64\uc740 \uc774\ud574\ud558\ub824 \ud558\uc9c0\ub9cc, \uc18c\ub144\uc740 \uae4a\ud788 \uac04\uc9c1\ub41c \uc9c4\uc2e4\ud55c \ub450\ub824\uc6c0\uc758 \uc6d0\uc778\uc744 \ub9d0\ucf64\uc740 \ubb3c\ub860 \uc5c4\ub9c8(\ud1a0\ub2c8 \ucf5c\ub81b)\uc5d0\uac8c\ub3c4 \ubc1d\ud788\uae30\ub97c \uaebc\ub824\ud55c\ub2e4.", "paragraph_id": "6519608-2-1", "paragraph_question": "question: \uc758\uc0ac \ub9d0\ucf64\uc774 \uc18c\ub144 \ucf5c\uc774 \uac04\uc9c1\ud55c \ub450\ub824\uc6c0\uc758 \uc6d0\uc778\uc73c\ub85c \ubcf8\uac83\uc740?, context: \ubd80\ubaa8\uc758 \uc774\ud63c\uc73c\ub85c \ucda9\uaca9\uc744 \ubc1b\uc740 \uac83\uc73c\ub85c \ub9d0\ucf64\uc740 \uc774\ud574\ud558\ub824 \ud558\uc9c0\ub9cc, \uc18c\ub144\uc740 \uae4a\ud788 \uac04\uc9c1\ub41c \uc9c4\uc2e4\ud55c \ub450\ub824\uc6c0\uc758 \uc6d0\uc778\uc744 \ub9d0\ucf64\uc740 \ubb3c\ub860 \uc5c4\ub9c8(\ud1a0\ub2c8 \ucf5c\ub81b)\uc5d0\uac8c\ub3c4 \ubc1d\ud788\uae30\ub97c \uaebc\ub824\ud55c\ub2e4. \uce58\ub8cc\uacfc\uc815\uc774 \uc2ec\ub3c4\ub97c \ub354\ud574 \uac00\uba74\uc11c \uc18c\ub144\uc740 \ub9d0\ucf64\uc744 \uc2e0\ub8b0\ud558\uac8c \ub418\uace0, \uc790\uc2e0\uc774 \ub610 \ub2e4\ub978 \uac10\uac01\uc744 \uc9c0\ub154\uc74c\uc744 \uadf8\uc5d0\uac8c \uc54c\ub9b0\ub2e4. \uc18c\ub144\uc740 \uc8fd\uc740 \uc0ac\ub78c\ub4e4\uc744 \ubcfc \uc218 \uc788\uc744 \ubfd0\ub9cc \uc544\ub2c8\ub77c \uadf8\ub4e4\uacfc \uc774\uc57c\uae30\ub97c \ud560 \uc218 \uc788\ub294 \uc9c0\uac01\uc744 \uac00\uc9c4 \uac83\uc774\ub2e4. \uc8fd\uc740 \uc18c\ub140\uac00 \ub098\ud0c0\ub098 \ud1a0\ud558\uba74\uc11c \uba54\uc2dc\uc9c0\ub97c \uc804\ud558\uace0, \uc8fd\uc740 \uc5ec\uc778\uc740 \ub0a8\ud3b8\uc744 \ud5a5\ud574 \uc18c\ub9ac\uc9c0\ub974\ub294 \ub4ef\uc774 \ucf5c\uc5d0\uac8c \uc790\uc2e0\uc758 \uc5b5\uc6b8\ud568\uc744 \ud138\uc5b4\ub193\ub294\uac00 \ud558\uba74, 100\ub144\uc804 \uad50\uc218\ud615\uc5d0 \ucc98\ud574\uc84c\ub358 \uac00\uc871\uc758 \ucc98\ucc38\ud55c \ubaa8\uc2b5\uc774 \uc18c\ub144\uc5d0\uac8c \ubcf4\uc774\ub294 \uac83\uc774\ub2e4. \ub9d0\ucf64\uc740 \ubbff\uc744 \uc218 \uc5c6\ub294 \uc0ac\uc2e4\uc5d0 \uc758\ud639\uc744 \ud488\uc73c\uba74\uc11c\ub3c4 \ubaa8\ub4e0 \uc218\ub2e8\uc744 \uc774\uc6a9\ud558\uc5ec \uc18c\ub144\uc758 \uace0\ub09c\uc2a4\ub7ec\uc6b4 \uc138\uacc4\ub85c \ud568\uaed8 \ud30c\uace0\ub4e0\ub2e4. \ud55c\ud3b8 \ub9d0\ucf64\uc740 \uac1c\uc778\uc801\uc778 \uc0ac\uc0dd\ud65c\uc758 \uc704\uae30\ub3c4 \ub9de\ub294\ub2e4. \uadf8\uc758 \uc544\ub0b4\ub294 \uacb0\ud63c\uc2dd\ub54c \ucc0d\uc740 \ube44\ub514\uc624\ub9cc\uc744 \uacc4\uc18d \ubcf4\uba74\uc11c \uacfc\uac70 \ub0a8\ud3b8\uacfc\uc758 \uc990\uac70\uc6e0\ub358 \uc2dc\uc808\ub9cc\uc744 \uadf8\ub9ac\uba70 \uc0dd\ud65c\ud560 \ubfd0, \ub9d0\ucf64\uacfc\ub294 \ub354 \uc774\uc0c1 \ub300\ud654\ub97c \ud558\uc9c0 \uc54a\uace0, \uc80a\uc740 \ub0a8\uc790\ub97c \ub9cc\ub098\uba74\uc11c \uc678\ub3c4\ub97c \ubc94\ud558\ub294\ub370\u2026"} {"question": "\uc2f8\uc774\uac00 \ubcd1\uc5ed\uc744 \ubaa8\ub450 \ub9c8\uce58\uace0 \uc81c\ub300\ud55c \ub0a0\uc9dc\ub294?", "paragraph": "\uc2f8\uc774\ub294 2007\ub144 7\uc6d4 14\uc77c \ubcd1\ubb34\uccad\uc73c\ub85c\ubd80\ud130 \uacb0\uad6d \ud604\uc5ed \uc7ac\ubcf5\ubb34\ub97c \ud1b5\ubcf4\ubc1b\uc558\ub2e4. \uc2f8\uc774 \uce21 \ubcc0\ud638\uc0ac\ub294 \ub300\ud559 \uad50\uc218\uac00 \uc4f4 \uc18c\ud504\ud2b8\uc6e8\uc5b4 \uacf5\ud559\ub860\uc5d0 \uc0ac\uc6b4\ub4dc\u00b7\uae30\ud68d\u00b7\ud14c\uc2a4\ud305 \ub4f1\ub3c4 \ubd84\uba85\ud55c \uc18c\ud504\ud2b8\uc6e8\uc5b4 \uac1c\ubc1c \uc5c5\ubb34\ub77c\uace0 \uba85\uc2dc\ub418\uc5b4 \uc788\ub294\ub370 \uc5b4\uc9f8\uc11c \uc9c0\uc815 \uc5c5\ubb34\uac00 \uc544\ub2c8\ub0d0\uace0 \ubc18\ubc15\ud558\uba70 \ud56d\uace0\ud588\uc9c0\ub9cc 2007\ub144 12\uc6d4 12\uc77c\uc5d0 \uc5f4\ub9b0 \ud56d\uace0\uc2ec \uc120\uace0 \uacf5\ud310\uc5d0\uc11c \ubc95\uc6d0\uc740 \uc2f8\uc774\uc758 \uccad\uad6c\ub97c \uae30\uac01\ud558\uc5ec \uc6d0\uace0 \ud328\uc18c \ud310\uacb0\uc744 \ub0b4\ub838\ub2e4. \uadf8\ub9ac\uace0 \ubcd1\ubb34\uccad\uc740 \uc2f8\uc774\uc5d0\uac8c \uc785\ub300\uc77c\uc744 \uac19\uc740 \ud574 12\uc6d4 17\uc77c\ub85c \uc815\ud558\uc5ec \ub17c\uc0b0 \uc721\uad70\ud6c8\ub828\uc18c\ub85c \uc785\uc601 \ud1b5\uc9c0\ub97c \ub2e4\uc2dc \ud588\ub2e4. \uc774\uc5d0 \ub530\ub77c \uc2f8\uc774\ub294 20\uac1c\uc6d4\uc758 \ubcf5\ubb34 \uae30\uac04, \ud604\uc5ed\uc73c\ub85c \uc7ac\uc785\ub300\ud588\ub2e4. \ub610, \uc7ac\ubcf5\ubb34 \uc911 1\uc2ec \uc120\uace0 \uacf5\ud310\uc5d0 \ub300\ud574 \ud56d\uc18c\ud588\uc73c\ub098 \ud328\uc18c\ud558\uc600\ub2e4. 2008\ub144 8\uc6d4 21\uc77c, \ub300\ubc95\uc6d0\uc5d0\uc11c\ub294 \uc6d0\uc2ec\uc744 \ud655\uc815\ud588\ub2e4. \uc2a4\uc2a4\ub85c \ubc1d\ud78c \ubc14\uc5d0 \ub530\ub974\uba74 \uc7ac\uc785\ub300 \uc77c\uc790\ub97c \ud1b5\uace0\ubc1b\uc558\uc744 \ub2f9\uc2dc, \uadf8 \ub0a0\uc9dc\uc5d0\uc11c 2\uc8fc\ub9cc \ubc84\ud2f0\uba74 \ub9cc 30\uc138\uac00 \ub418\uc5b4 \ud604\uc5ed\uc785\ub300\uac00 \ubd88\uac00\ub2a5\ud574\uc9c0\uace0 \ub300\uccb4\ubcf5\ubb34\uc778 \uacf5\uc775\uadfc\ubb34\uc694\uc6d0(\uc0ac\ud68c\ubcf5\ubb34\uc694\uc6d0) \uc18c\uc9d1\uc73c\ub85c\uc758 \uc804\ud658\uc774 \uac00\ub2a5\ud558\ub2e4\ub294 \uc0ac\uc2e4\ub85c \uc778\ud574 \uace0\ubbfc\uc774 \ub9ce\uc558\ub2e4\uace0 \ud55c\ub2e4. \ub204\uad70\uac00\uc5d0\uac8c \uc2dc\ube44\ub97c \uac78\uc5b4 \uc804\uce58 3\uc8fc \uc774\uc0c1\uc758 \ubd80\uc0c1\uc744 \uc785\uc5b4 \ubcfc\uae4c \ud558\ub294 \uad6c\uccb4\uc801\uc778 \uacc4\ud68d\uc744 \uc138\uc6b0\uae30\ub3c4 \ud588\uc5c8\ub2e4. \uc7ac\uc785\ub300 \ucd08\uae30\uc5d0\ub294 \ud1b5\uc2e0\ubcd1\uc73c\ub85c \ubcf5\ubb34\ud558\ub2e4\uac00, 2009\ub144 1\uc6d4 \ud1a0\ub2c8\uc548\uacfc \ud568\uaed8 \uc5f0\uc608\ubcd1\uc0ac\ub85c \uc790\ub9ac\ub97c \uc62e\uaca8 \uad6d\uad70\ubc29\uc1a1\uc758 \ud504\ub85c\uadf8\ub7a8\uc744 \uc9c4\ud589\ud558\uc600\uc73c\uba70 \uc721\uad70\ucc38\ubaa8\ucd1d\uc7a5\uc0c1 \uc744 \uc218\uc0c1\ud558\ub294 \ub4f1 \uc131\uc2e4\ud55c \ubaa8\uc2b5\uc744 \ubcf4\uc5ec \uc8fc\uc5c8\ub2e4. 2009\ub144 7\uc6d4 11\uc77c \ubcd1\uc5ed\uc744 \ubaa8\ub450 \ub9c8\uce58\uace0 \uc81c\ub300\ud55c \uc774\ud6c4 \uadf8\ub294 \u201c\uad70 \ubcf5\ubb34 \ub2f9\uc2dc \ub098\ub77c\uc640 \uc0c1\uad00, \ub9c8\uc9c0\ub9c9\uc73c\ub85c\ub294 \uc18c\ub140\uc2dc\ub300\uc5d0 \ucda9\uc131\ud588\ub2e4\u201d\ub77c\ub294 \ud68c\uace0\ub85c \ud3ed\uc18c\ub97c \uc790\uc544 \ub0b4\uae30\ub3c4 \ud588\ub2e4. \uc81c\ub300\ud55c \uc774\ud6c4 \uba87 \ucc28\ub840 \uad70 \uc704\ubb38 \uacf5\uc5f0\uc744 \uac00\uc9c0\uae30\ub3c4 \ud588\uc73c\uba70, \ucd9c\uc5f0\ub8cc \uc804\uc561\uc744 \uad70\uc5d0 \uae30\ubd80\ud558\uc5ec \uc7ac\uc785\ub300\uc640 \uc81c\ub300 \uc774\ud6c4 \uc624\ud788\ub824 \ub0a8\ub2e4\ub978 \uad70 \uc0ac\ub791\uc744 \ubcf4\uc5ec \uc8fc\uc5c8\ub2e4.", "answer": "2009\ub144 7\uc6d4 11\uc77c", "sentence": "2009\ub144 7\uc6d4 11\uc77c \ubcd1\uc5ed\uc744 \ubaa8\ub450 \ub9c8\uce58\uace0 \uc81c\ub300\ud55c \uc774\ud6c4 \uadf8\ub294 \u201c\uad70 \ubcf5\ubb34 \ub2f9\uc2dc \ub098\ub77c\uc640 \uc0c1\uad00, \ub9c8\uc9c0\ub9c9\uc73c\ub85c\ub294 \uc18c\ub140\uc2dc\ub300\uc5d0 \ucda9\uc131\ud588\ub2e4\u201d\ub77c\ub294 \ud68c\uace0\ub85c \ud3ed\uc18c\ub97c \uc790\uc544 \ub0b4\uae30\ub3c4 \ud588\ub2e4.", "paragraph_sentence": "\uc2f8\uc774\ub294 2007\ub144 7\uc6d4 14\uc77c \ubcd1\ubb34\uccad\uc73c\ub85c\ubd80\ud130 \uacb0\uad6d \ud604\uc5ed \uc7ac\ubcf5\ubb34\ub97c \ud1b5\ubcf4\ubc1b\uc558\ub2e4. \uc2f8\uc774 \uce21 \ubcc0\ud638\uc0ac\ub294 \ub300\ud559 \uad50\uc218\uac00 \uc4f4 \uc18c\ud504\ud2b8\uc6e8\uc5b4 \uacf5\ud559\ub860\uc5d0 \uc0ac\uc6b4\ub4dc\u00b7\uae30\ud68d\u00b7\ud14c\uc2a4\ud305 \ub4f1\ub3c4 \ubd84\uba85\ud55c \uc18c\ud504\ud2b8\uc6e8\uc5b4 \uac1c\ubc1c \uc5c5\ubb34\ub77c\uace0 \uba85\uc2dc\ub418\uc5b4 \uc788\ub294\ub370 \uc5b4\uc9f8\uc11c \uc9c0\uc815 \uc5c5\ubb34\uac00 \uc544\ub2c8\ub0d0\uace0 \ubc18\ubc15\ud558\uba70 \ud56d\uace0\ud588\uc9c0\ub9cc 2007\ub144 12\uc6d4 12\uc77c\uc5d0 \uc5f4\ub9b0 \ud56d\uace0\uc2ec \uc120\uace0 \uacf5\ud310\uc5d0\uc11c \ubc95\uc6d0\uc740 \uc2f8\uc774\uc758 \uccad\uad6c\ub97c \uae30\uac01\ud558\uc5ec \uc6d0\uace0 \ud328\uc18c \ud310\uacb0\uc744 \ub0b4\ub838\ub2e4. \uadf8\ub9ac\uace0 \ubcd1\ubb34\uccad\uc740 \uc2f8\uc774\uc5d0\uac8c \uc785\ub300\uc77c\uc744 \uac19\uc740 \ud574 12\uc6d4 17\uc77c\ub85c \uc815\ud558\uc5ec \ub17c\uc0b0 \uc721\uad70\ud6c8\ub828\uc18c\ub85c \uc785\uc601 \ud1b5\uc9c0\ub97c \ub2e4\uc2dc \ud588\ub2e4. \uc774\uc5d0 \ub530\ub77c \uc2f8\uc774\ub294 20\uac1c\uc6d4\uc758 \ubcf5\ubb34 \uae30\uac04, \ud604\uc5ed\uc73c\ub85c \uc7ac\uc785\ub300\ud588\ub2e4. \ub610, \uc7ac\ubcf5\ubb34 \uc911 1\uc2ec \uc120\uace0 \uacf5\ud310\uc5d0 \ub300\ud574 \ud56d\uc18c\ud588\uc73c\ub098 \ud328\uc18c\ud558\uc600\ub2e4. 2008\ub144 8\uc6d4 21\uc77c, \ub300\ubc95\uc6d0\uc5d0\uc11c\ub294 \uc6d0\uc2ec\uc744 \ud655\uc815\ud588\ub2e4. \uc2a4\uc2a4\ub85c \ubc1d\ud78c \ubc14\uc5d0 \ub530\ub974\uba74 \uc7ac\uc785\ub300 \uc77c\uc790\ub97c \ud1b5\uace0\ubc1b\uc558\uc744 \ub2f9\uc2dc, \uadf8 \ub0a0\uc9dc\uc5d0\uc11c 2\uc8fc\ub9cc \ubc84\ud2f0\uba74 \ub9cc 30\uc138\uac00 \ub418\uc5b4 \ud604\uc5ed\uc785\ub300\uac00 \ubd88\uac00\ub2a5\ud574\uc9c0\uace0 \ub300\uccb4\ubcf5\ubb34\uc778 \uacf5\uc775\uadfc\ubb34\uc694\uc6d0(\uc0ac\ud68c\ubcf5\ubb34\uc694\uc6d0) \uc18c\uc9d1\uc73c\ub85c\uc758 \uc804\ud658\uc774 \uac00\ub2a5\ud558\ub2e4\ub294 \uc0ac\uc2e4\ub85c \uc778\ud574 \uace0\ubbfc\uc774 \ub9ce\uc558\ub2e4\uace0 \ud55c\ub2e4. \ub204\uad70\uac00\uc5d0\uac8c \uc2dc\ube44\ub97c \uac78\uc5b4 \uc804\uce58 3\uc8fc \uc774\uc0c1\uc758 \ubd80\uc0c1\uc744 \uc785\uc5b4 \ubcfc\uae4c \ud558\ub294 \uad6c\uccb4\uc801\uc778 \uacc4\ud68d\uc744 \uc138\uc6b0\uae30\ub3c4 \ud588\uc5c8\ub2e4. \uc7ac\uc785\ub300 \ucd08\uae30\uc5d0\ub294 \ud1b5\uc2e0\ubcd1\uc73c\ub85c \ubcf5\ubb34\ud558\ub2e4\uac00, 2009\ub144 1\uc6d4 \ud1a0\ub2c8\uc548\uacfc \ud568\uaed8 \uc5f0\uc608\ubcd1\uc0ac\ub85c \uc790\ub9ac\ub97c \uc62e\uaca8 \uad6d\uad70\ubc29\uc1a1\uc758 \ud504\ub85c\uadf8\ub7a8\uc744 \uc9c4\ud589\ud558\uc600\uc73c\uba70 \uc721\uad70\ucc38\ubaa8\ucd1d\uc7a5\uc0c1 \uc744 \uc218\uc0c1\ud558\ub294 \ub4f1 \uc131\uc2e4\ud55c \ubaa8\uc2b5\uc744 \ubcf4\uc5ec \uc8fc\uc5c8\ub2e4. 2009\ub144 7\uc6d4 11\uc77c \ubcd1\uc5ed\uc744 \ubaa8\ub450 \ub9c8\uce58\uace0 \uc81c\ub300\ud55c \uc774\ud6c4 \uadf8\ub294 \u201c\uad70 \ubcf5\ubb34 \ub2f9\uc2dc \ub098\ub77c\uc640 \uc0c1\uad00, \ub9c8\uc9c0\ub9c9\uc73c\ub85c\ub294 \uc18c\ub140\uc2dc\ub300\uc5d0 \ucda9\uc131\ud588\ub2e4\u201d\ub77c\ub294 \ud68c\uace0\ub85c \ud3ed\uc18c\ub97c \uc790\uc544 \ub0b4\uae30\ub3c4 \ud588\ub2e4. \uc81c\ub300\ud55c \uc774\ud6c4 \uba87 \ucc28\ub840 \uad70 \uc704\ubb38 \uacf5\uc5f0\uc744 \uac00\uc9c0\uae30\ub3c4 \ud588\uc73c\uba70, \ucd9c\uc5f0\ub8cc \uc804\uc561\uc744 \uad70\uc5d0 \uae30\ubd80\ud558\uc5ec \uc7ac\uc785\ub300\uc640 \uc81c\ub300 \uc774\ud6c4 \uc624\ud788\ub824 \ub0a8\ub2e4\ub978 \uad70 \uc0ac\ub791\uc744 \ubcf4\uc5ec \uc8fc\uc5c8\ub2e4.", "paragraph_answer": "\uc2f8\uc774\ub294 2007\ub144 7\uc6d4 14\uc77c \ubcd1\ubb34\uccad\uc73c\ub85c\ubd80\ud130 \uacb0\uad6d \ud604\uc5ed \uc7ac\ubcf5\ubb34\ub97c \ud1b5\ubcf4\ubc1b\uc558\ub2e4. \uc2f8\uc774 \uce21 \ubcc0\ud638\uc0ac\ub294 \ub300\ud559 \uad50\uc218\uac00 \uc4f4 \uc18c\ud504\ud2b8\uc6e8\uc5b4 \uacf5\ud559\ub860\uc5d0 \uc0ac\uc6b4\ub4dc\u00b7\uae30\ud68d\u00b7\ud14c\uc2a4\ud305 \ub4f1\ub3c4 \ubd84\uba85\ud55c \uc18c\ud504\ud2b8\uc6e8\uc5b4 \uac1c\ubc1c \uc5c5\ubb34\ub77c\uace0 \uba85\uc2dc\ub418\uc5b4 \uc788\ub294\ub370 \uc5b4\uc9f8\uc11c \uc9c0\uc815 \uc5c5\ubb34\uac00 \uc544\ub2c8\ub0d0\uace0 \ubc18\ubc15\ud558\uba70 \ud56d\uace0\ud588\uc9c0\ub9cc 2007\ub144 12\uc6d4 12\uc77c\uc5d0 \uc5f4\ub9b0 \ud56d\uace0\uc2ec \uc120\uace0 \uacf5\ud310\uc5d0\uc11c \ubc95\uc6d0\uc740 \uc2f8\uc774\uc758 \uccad\uad6c\ub97c \uae30\uac01\ud558\uc5ec \uc6d0\uace0 \ud328\uc18c \ud310\uacb0\uc744 \ub0b4\ub838\ub2e4. \uadf8\ub9ac\uace0 \ubcd1\ubb34\uccad\uc740 \uc2f8\uc774\uc5d0\uac8c \uc785\ub300\uc77c\uc744 \uac19\uc740 \ud574 12\uc6d4 17\uc77c\ub85c \uc815\ud558\uc5ec \ub17c\uc0b0 \uc721\uad70\ud6c8\ub828\uc18c\ub85c \uc785\uc601 \ud1b5\uc9c0\ub97c \ub2e4\uc2dc \ud588\ub2e4. \uc774\uc5d0 \ub530\ub77c \uc2f8\uc774\ub294 20\uac1c\uc6d4\uc758 \ubcf5\ubb34 \uae30\uac04, \ud604\uc5ed\uc73c\ub85c \uc7ac\uc785\ub300\ud588\ub2e4. \ub610, \uc7ac\ubcf5\ubb34 \uc911 1\uc2ec \uc120\uace0 \uacf5\ud310\uc5d0 \ub300\ud574 \ud56d\uc18c\ud588\uc73c\ub098 \ud328\uc18c\ud558\uc600\ub2e4. 2008\ub144 8\uc6d4 21\uc77c, \ub300\ubc95\uc6d0\uc5d0\uc11c\ub294 \uc6d0\uc2ec\uc744 \ud655\uc815\ud588\ub2e4. \uc2a4\uc2a4\ub85c \ubc1d\ud78c \ubc14\uc5d0 \ub530\ub974\uba74 \uc7ac\uc785\ub300 \uc77c\uc790\ub97c \ud1b5\uace0\ubc1b\uc558\uc744 \ub2f9\uc2dc, \uadf8 \ub0a0\uc9dc\uc5d0\uc11c 2\uc8fc\ub9cc \ubc84\ud2f0\uba74 \ub9cc 30\uc138\uac00 \ub418\uc5b4 \ud604\uc5ed\uc785\ub300\uac00 \ubd88\uac00\ub2a5\ud574\uc9c0\uace0 \ub300\uccb4\ubcf5\ubb34\uc778 \uacf5\uc775\uadfc\ubb34\uc694\uc6d0(\uc0ac\ud68c\ubcf5\ubb34\uc694\uc6d0) \uc18c\uc9d1\uc73c\ub85c\uc758 \uc804\ud658\uc774 \uac00\ub2a5\ud558\ub2e4\ub294 \uc0ac\uc2e4\ub85c \uc778\ud574 \uace0\ubbfc\uc774 \ub9ce\uc558\ub2e4\uace0 \ud55c\ub2e4. \ub204\uad70\uac00\uc5d0\uac8c \uc2dc\ube44\ub97c \uac78\uc5b4 \uc804\uce58 3\uc8fc \uc774\uc0c1\uc758 \ubd80\uc0c1\uc744 \uc785\uc5b4 \ubcfc\uae4c \ud558\ub294 \uad6c\uccb4\uc801\uc778 \uacc4\ud68d\uc744 \uc138\uc6b0\uae30\ub3c4 \ud588\uc5c8\ub2e4. \uc7ac\uc785\ub300 \ucd08\uae30\uc5d0\ub294 \ud1b5\uc2e0\ubcd1\uc73c\ub85c \ubcf5\ubb34\ud558\ub2e4\uac00, 2009\ub144 1\uc6d4 \ud1a0\ub2c8\uc548\uacfc \ud568\uaed8 \uc5f0\uc608\ubcd1\uc0ac\ub85c \uc790\ub9ac\ub97c \uc62e\uaca8 \uad6d\uad70\ubc29\uc1a1\uc758 \ud504\ub85c\uadf8\ub7a8\uc744 \uc9c4\ud589\ud558\uc600\uc73c\uba70 \uc721\uad70\ucc38\ubaa8\ucd1d\uc7a5\uc0c1 \uc744 \uc218\uc0c1\ud558\ub294 \ub4f1 \uc131\uc2e4\ud55c \ubaa8\uc2b5\uc744 \ubcf4\uc5ec \uc8fc\uc5c8\ub2e4. 2009\ub144 7\uc6d4 11\uc77c \ubcd1\uc5ed\uc744 \ubaa8\ub450 \ub9c8\uce58\uace0 \uc81c\ub300\ud55c \uc774\ud6c4 \uadf8\ub294 \u201c\uad70 \ubcf5\ubb34 \ub2f9\uc2dc \ub098\ub77c\uc640 \uc0c1\uad00, \ub9c8\uc9c0\ub9c9\uc73c\ub85c\ub294 \uc18c\ub140\uc2dc\ub300\uc5d0 \ucda9\uc131\ud588\ub2e4\u201d\ub77c\ub294 \ud68c\uace0\ub85c \ud3ed\uc18c\ub97c \uc790\uc544 \ub0b4\uae30\ub3c4 \ud588\ub2e4. \uc81c\ub300\ud55c \uc774\ud6c4 \uba87 \ucc28\ub840 \uad70 \uc704\ubb38 \uacf5\uc5f0\uc744 \uac00\uc9c0\uae30\ub3c4 \ud588\uc73c\uba70, \ucd9c\uc5f0\ub8cc \uc804\uc561\uc744 \uad70\uc5d0 \uae30\ubd80\ud558\uc5ec \uc7ac\uc785\ub300\uc640 \uc81c\ub300 \uc774\ud6c4 \uc624\ud788\ub824 \ub0a8\ub2e4\ub978 \uad70 \uc0ac\ub791\uc744 \ubcf4\uc5ec \uc8fc\uc5c8\ub2e4.", "sentence_answer": " 2009\ub144 7\uc6d4 11\uc77c \ubcd1\uc5ed\uc744 \ubaa8\ub450 \ub9c8\uce58\uace0 \uc81c\ub300\ud55c \uc774\ud6c4 \uadf8\ub294 \u201c\uad70 \ubcf5\ubb34 \ub2f9\uc2dc \ub098\ub77c\uc640 \uc0c1\uad00, \ub9c8\uc9c0\ub9c9\uc73c\ub85c\ub294 \uc18c\ub140\uc2dc\ub300\uc5d0 \ucda9\uc131\ud588\ub2e4\u201d\ub77c\ub294 \ud68c\uace0\ub85c \ud3ed\uc18c\ub97c \uc790\uc544 \ub0b4\uae30\ub3c4 \ud588\ub2e4.", "paragraph_id": "6547275-9-2", "paragraph_question": "question: \uc2f8\uc774\uac00 \ubcd1\uc5ed\uc744 \ubaa8\ub450 \ub9c8\uce58\uace0 \uc81c\ub300\ud55c \ub0a0\uc9dc\ub294?, context: \uc2f8\uc774\ub294 2007\ub144 7\uc6d4 14\uc77c \ubcd1\ubb34\uccad\uc73c\ub85c\ubd80\ud130 \uacb0\uad6d \ud604\uc5ed \uc7ac\ubcf5\ubb34\ub97c \ud1b5\ubcf4\ubc1b\uc558\ub2e4. \uc2f8\uc774 \uce21 \ubcc0\ud638\uc0ac\ub294 \ub300\ud559 \uad50\uc218\uac00 \uc4f4 \uc18c\ud504\ud2b8\uc6e8\uc5b4 \uacf5\ud559\ub860\uc5d0 \uc0ac\uc6b4\ub4dc\u00b7\uae30\ud68d\u00b7\ud14c\uc2a4\ud305 \ub4f1\ub3c4 \ubd84\uba85\ud55c \uc18c\ud504\ud2b8\uc6e8\uc5b4 \uac1c\ubc1c \uc5c5\ubb34\ub77c\uace0 \uba85\uc2dc\ub418\uc5b4 \uc788\ub294\ub370 \uc5b4\uc9f8\uc11c \uc9c0\uc815 \uc5c5\ubb34\uac00 \uc544\ub2c8\ub0d0\uace0 \ubc18\ubc15\ud558\uba70 \ud56d\uace0\ud588\uc9c0\ub9cc 2007\ub144 12\uc6d4 12\uc77c\uc5d0 \uc5f4\ub9b0 \ud56d\uace0\uc2ec \uc120\uace0 \uacf5\ud310\uc5d0\uc11c \ubc95\uc6d0\uc740 \uc2f8\uc774\uc758 \uccad\uad6c\ub97c \uae30\uac01\ud558\uc5ec \uc6d0\uace0 \ud328\uc18c \ud310\uacb0\uc744 \ub0b4\ub838\ub2e4. \uadf8\ub9ac\uace0 \ubcd1\ubb34\uccad\uc740 \uc2f8\uc774\uc5d0\uac8c \uc785\ub300\uc77c\uc744 \uac19\uc740 \ud574 12\uc6d4 17\uc77c\ub85c \uc815\ud558\uc5ec \ub17c\uc0b0 \uc721\uad70\ud6c8\ub828\uc18c\ub85c \uc785\uc601 \ud1b5\uc9c0\ub97c \ub2e4\uc2dc \ud588\ub2e4. \uc774\uc5d0 \ub530\ub77c \uc2f8\uc774\ub294 20\uac1c\uc6d4\uc758 \ubcf5\ubb34 \uae30\uac04, \ud604\uc5ed\uc73c\ub85c \uc7ac\uc785\ub300\ud588\ub2e4. \ub610, \uc7ac\ubcf5\ubb34 \uc911 1\uc2ec \uc120\uace0 \uacf5\ud310\uc5d0 \ub300\ud574 \ud56d\uc18c\ud588\uc73c\ub098 \ud328\uc18c\ud558\uc600\ub2e4. 2008\ub144 8\uc6d4 21\uc77c, \ub300\ubc95\uc6d0\uc5d0\uc11c\ub294 \uc6d0\uc2ec\uc744 \ud655\uc815\ud588\ub2e4. \uc2a4\uc2a4\ub85c \ubc1d\ud78c \ubc14\uc5d0 \ub530\ub974\uba74 \uc7ac\uc785\ub300 \uc77c\uc790\ub97c \ud1b5\uace0\ubc1b\uc558\uc744 \ub2f9\uc2dc, \uadf8 \ub0a0\uc9dc\uc5d0\uc11c 2\uc8fc\ub9cc \ubc84\ud2f0\uba74 \ub9cc 30\uc138\uac00 \ub418\uc5b4 \ud604\uc5ed\uc785\ub300\uac00 \ubd88\uac00\ub2a5\ud574\uc9c0\uace0 \ub300\uccb4\ubcf5\ubb34\uc778 \uacf5\uc775\uadfc\ubb34\uc694\uc6d0(\uc0ac\ud68c\ubcf5\ubb34\uc694\uc6d0) \uc18c\uc9d1\uc73c\ub85c\uc758 \uc804\ud658\uc774 \uac00\ub2a5\ud558\ub2e4\ub294 \uc0ac\uc2e4\ub85c \uc778\ud574 \uace0\ubbfc\uc774 \ub9ce\uc558\ub2e4\uace0 \ud55c\ub2e4. \ub204\uad70\uac00\uc5d0\uac8c \uc2dc\ube44\ub97c \uac78\uc5b4 \uc804\uce58 3\uc8fc \uc774\uc0c1\uc758 \ubd80\uc0c1\uc744 \uc785\uc5b4 \ubcfc\uae4c \ud558\ub294 \uad6c\uccb4\uc801\uc778 \uacc4\ud68d\uc744 \uc138\uc6b0\uae30\ub3c4 \ud588\uc5c8\ub2e4. \uc7ac\uc785\ub300 \ucd08\uae30\uc5d0\ub294 \ud1b5\uc2e0\ubcd1\uc73c\ub85c \ubcf5\ubb34\ud558\ub2e4\uac00, 2009\ub144 1\uc6d4 \ud1a0\ub2c8\uc548\uacfc \ud568\uaed8 \uc5f0\uc608\ubcd1\uc0ac\ub85c \uc790\ub9ac\ub97c \uc62e\uaca8 \uad6d\uad70\ubc29\uc1a1\uc758 \ud504\ub85c\uadf8\ub7a8\uc744 \uc9c4\ud589\ud558\uc600\uc73c\uba70 \uc721\uad70\ucc38\ubaa8\ucd1d\uc7a5\uc0c1 \uc744 \uc218\uc0c1\ud558\ub294 \ub4f1 \uc131\uc2e4\ud55c \ubaa8\uc2b5\uc744 \ubcf4\uc5ec \uc8fc\uc5c8\ub2e4. 2009\ub144 7\uc6d4 11\uc77c \ubcd1\uc5ed\uc744 \ubaa8\ub450 \ub9c8\uce58\uace0 \uc81c\ub300\ud55c \uc774\ud6c4 \uadf8\ub294 \u201c\uad70 \ubcf5\ubb34 \ub2f9\uc2dc \ub098\ub77c\uc640 \uc0c1\uad00, \ub9c8\uc9c0\ub9c9\uc73c\ub85c\ub294 \uc18c\ub140\uc2dc\ub300\uc5d0 \ucda9\uc131\ud588\ub2e4\u201d\ub77c\ub294 \ud68c\uace0\ub85c \ud3ed\uc18c\ub97c \uc790\uc544 \ub0b4\uae30\ub3c4 \ud588\ub2e4. \uc81c\ub300\ud55c \uc774\ud6c4 \uba87 \ucc28\ub840 \uad70 \uc704\ubb38 \uacf5\uc5f0\uc744 \uac00\uc9c0\uae30\ub3c4 \ud588\uc73c\uba70, \ucd9c\uc5f0\ub8cc \uc804\uc561\uc744 \uad70\uc5d0 \uae30\ubd80\ud558\uc5ec \uc7ac\uc785\ub300\uc640 \uc81c\ub300 \uc774\ud6c4 \uc624\ud788\ub824 \ub0a8\ub2e4\ub978 \uad70 \uc0ac\ub791\uc744 \ubcf4\uc5ec \uc8fc\uc5c8\ub2e4."} {"question": "\ucc98\uc74c\uc73c\ub85c \ud558\uacc4 \uc62c\ub9bc\ud53d\uc5d0 2\ub144 \uc55e\uc11c \uce58\ub904\uc9c4 \ub3d9\uacc4\uc62c\ub9bc\ud53d\uc774 \uac1c\ucd5c\ub41c \uad6d\uac00\ub294?", "paragraph": "1986\ub144 \uad6d\uc81c\uc62c\ub9bc\ud53d\uc704\uc6d0\ud68c\uac00 \uac19\uc740 \ud574\uc5d0 \uce58\ub7ec\uc9c0\ub358 \ub3d9\ud558\uacc4 \uc62c\ub9bc\ud53d \ub300\ud68c\ub97c \ubd84\ub9ac\uc2dc\ucf1c 2\ub144 \uaca9\ucc28\ub85c \uc5f0\ub2e4\ub294 \uacb0\uc815\uc744 \ud22c\ud45c\ub97c \ud1b5\ud574 \ucd5c\uc885 \ud655\uc815\ud558\uc600\uace0, 1994\ub144 \ub3d9\uacc4 \uc62c\ub9bc\ud53d\ubd80\ud130 \uc774 \uacb0\uc815\uc744 \ub530\ub974\uac8c \ub418\uc5c8\ub2e4. \ub178\ub974\uc6e8\uc774 \ub9b4\ub808\ud568\uba54\ub974\uc5d0\uc11c \uc5f4\ub9b0 \uc774 \ub300\ud68c\ub294 \ucc98\uc74c\uc73c\ub85c \ud558\uacc4 \uc62c\ub9bc\ud53d\uc5d0 2\ub144 \uc55e\uc11c \uce58\ub7ec\uc9c4 \ub300\ud68c\uac00 \ub418\uc5c8\ub2e4. 1993\ub144 \uccb4\ucf54\uc2ac\ub85c\ubc14\ud0a4\uc544 \ud574\uccb4\ub97c \uacaa\uc740 \uccb4\ucf54\uc640 \uc2ac\ub85c\ubc14\ud0a4\uc544\uac00 \ubcc4\uac1c\uc758 \uc120\uc218\ub2e8\uc73c\ub85c\uc11c \ub3d9\uacc4\uc62c\ub9bc\ud53d\uc5d0 \ucc98\uc74c \ucd9c\uc804\ud588\ub2e4. \ud55c\ud3b8 1994\ub144 1\uc6d4 6\uc77c \ubbf8\uad6d\uc758 \ud53c\uaca8\uc2a4\ucf00\uc774\ud305 \uc120\uc218 \ub0b8\uc2dc \ucf00\ub9ac\uac74\uc774 \uacbd\uc7c1\uc790 \ud1a0\ub0d0 \ud558\ub529\uc758 \uc804 \ub0a8\ud3b8\uc758 \uc0ac\uc8fc\ub85c \ud53c\uaca9\uc744 \ubc1b\ub294 \uc0ac\uc0c1\ucd08\uc720\uc758 \uc0ac\ud0dc\uac00 \ubc8c\uc5b4\uc9c0\uba74\uc11c, \uc5ec\uc790 \ud53c\uaca8\uc2a4\ucf00\uc774\ud305 \uacbd\uae30\uac00 \uc0c8\uc0bc \uc5b8\ub860\uc758 \uc774\ubaa9\uc744 \ub04c\uac8c \ub418\uc5c8\ub2e4. \ucf00\ub9ac\uac74\uacfc \ud558\ub529 \ubaa8\ub450 \ub300\ud68c\uc5d0\ub294 \uc815\uc0c1 \ucd9c\uc804\ud558\uc600\uc73c\ub098 \uce90\ub9ac\uac74\ub9cc \uc740\uba54\ub2ec\uc744 \ud68d\ub4dd\ud558\uc600\uace0, \uc6b0\ud06c\ub77c\uc774\ub098\uc758 \uc624\ud06c\uc0ac\ub098 \ubc14\uc774\uc6b8\uc774 \uc6b0\uc2b9\ud558\uba74\uc11c \uace0\uad6d\uc5d0 \uccab \uae08\uba54\ub2ec\uc744 \uc548\uacbc\ub2e4. \ud55c\ud3b8\uc73c\ub85c \uac1c\ucd5c\uad6d \ub178\ub974\uc6e8\uc774\uac00 \ucc98\uc74c\uc73c\ub85c \uc2a4\ud53c\ub4dc\uc2a4\ucf00\uc774\ud305 \uc804 \uc885\ubaa9\uc5d0 \ucd9c\uc804\ud574 3\uad00\uc655\uc744 \ub2ec\uc131\ud558\uc600\uace0, \ub300\ud55c\ubbfc\uad6d\uc758 \uae40\uc724\ubbf8 \uc120\uc218\uac00 \ub9cc 13\uc138\uc758 \ub098\uc774\ub85c \uc1fc\ud2b8\ud2b8\ub799 \ub2e8\uccb4\uacc4\uc8fc \uae08\uba54\ub2ec\uc744 \ucc28\uc9c0\ud558\uba74\uc11c, \ub3d9\uacc4\uc62c\ub9bc\ud53d \uc5ed\ub300 \ucd5c\uc5f0\uc18c \uae08\uba54\ub2ec\ub9ac\uc2a4\ud2b8\ub85c \ub0a8\uac8c \ub418\uc5c8\ub2e4. \ud6c4\uc548 \uc548\ud1a0\ub2c8\uc624 \uc0ac\ub9c8\ub780\uce58 IOC \uc704\uc6d0\uc7a5\uc740 \ub300\ud68c \ud3d0\ub9c9\uc2dd\uc5d0\uc11c \ub9b4\ub808\ud568\uba54\ub974 \ub3d9\uacc4\uc62c\ub9bc\ud53d\uc774 \uc0ac\uc0c1 \ucd5c\uace0\uc758 \ub3d9\uacc4\uc62c\ub9bc\ud53d \ub300\ud68c\uc600\ub2e4\uace0 \uce6d\ucc2c\ud558\uae30\ub3c4 \ud558\uc600\ub2e4.", "answer": "\ub178\ub974\uc6e8\uc774", "sentence": "\ub178\ub974\uc6e8\uc774 \ub9b4\ub808\ud568\uba54\ub974\uc5d0\uc11c \uc5f4\ub9b0 \uc774 \ub300\ud68c\ub294 \ucc98\uc74c\uc73c\ub85c \ud558\uacc4 \uc62c\ub9bc\ud53d\uc5d0 2\ub144 \uc55e\uc11c \uce58\ub7ec\uc9c4 \ub300\ud68c\uac00 \ub418\uc5c8\ub2e4.", "paragraph_sentence": "1986\ub144 \uad6d\uc81c\uc62c\ub9bc\ud53d\uc704\uc6d0\ud68c\uac00 \uac19\uc740 \ud574\uc5d0 \uce58\ub7ec\uc9c0\ub358 \ub3d9\ud558\uacc4 \uc62c\ub9bc\ud53d \ub300\ud68c\ub97c \ubd84\ub9ac\uc2dc\ucf1c 2\ub144 \uaca9\ucc28\ub85c \uc5f0\ub2e4\ub294 \uacb0\uc815\uc744 \ud22c\ud45c\ub97c \ud1b5\ud574 \ucd5c\uc885 \ud655\uc815\ud558\uc600\uace0, 1994\ub144 \ub3d9\uacc4 \uc62c\ub9bc\ud53d\ubd80\ud130 \uc774 \uacb0\uc815\uc744 \ub530\ub974\uac8c \ub418\uc5c8\ub2e4. \ub178\ub974\uc6e8\uc774 \ub9b4\ub808\ud568\uba54\ub974\uc5d0\uc11c \uc5f4\ub9b0 \uc774 \ub300\ud68c\ub294 \ucc98\uc74c\uc73c\ub85c \ud558\uacc4 \uc62c\ub9bc\ud53d\uc5d0 2\ub144 \uc55e\uc11c \uce58\ub7ec\uc9c4 \ub300\ud68c\uac00 \ub418\uc5c8\ub2e4. 1993\ub144 \uccb4\ucf54\uc2ac\ub85c\ubc14\ud0a4\uc544 \ud574\uccb4\ub97c \uacaa\uc740 \uccb4\ucf54\uc640 \uc2ac\ub85c\ubc14\ud0a4\uc544\uac00 \ubcc4\uac1c\uc758 \uc120\uc218\ub2e8\uc73c\ub85c\uc11c \ub3d9\uacc4\uc62c\ub9bc\ud53d\uc5d0 \ucc98\uc74c \ucd9c\uc804\ud588\ub2e4. \ud55c\ud3b8 1994\ub144 1\uc6d4 6\uc77c \ubbf8\uad6d\uc758 \ud53c\uaca8\uc2a4\ucf00\uc774\ud305 \uc120\uc218 \ub0b8\uc2dc \ucf00\ub9ac\uac74\uc774 \uacbd\uc7c1\uc790 \ud1a0\ub0d0 \ud558\ub529\uc758 \uc804 \ub0a8\ud3b8\uc758 \uc0ac\uc8fc\ub85c \ud53c\uaca9\uc744 \ubc1b\ub294 \uc0ac\uc0c1\ucd08\uc720\uc758 \uc0ac\ud0dc\uac00 \ubc8c\uc5b4\uc9c0\uba74\uc11c, \uc5ec\uc790 \ud53c\uaca8\uc2a4\ucf00\uc774\ud305 \uacbd\uae30\uac00 \uc0c8\uc0bc \uc5b8\ub860\uc758 \uc774\ubaa9\uc744 \ub04c\uac8c \ub418\uc5c8\ub2e4. \ucf00\ub9ac\uac74\uacfc \ud558\ub529 \ubaa8\ub450 \ub300\ud68c\uc5d0\ub294 \uc815\uc0c1 \ucd9c\uc804\ud558\uc600\uc73c\ub098 \uce90\ub9ac\uac74\ub9cc \uc740\uba54\ub2ec\uc744 \ud68d\ub4dd\ud558\uc600\uace0, \uc6b0\ud06c\ub77c\uc774\ub098\uc758 \uc624\ud06c\uc0ac\ub098 \ubc14\uc774\uc6b8\uc774 \uc6b0\uc2b9\ud558\uba74\uc11c \uace0\uad6d\uc5d0 \uccab \uae08\uba54\ub2ec\uc744 \uc548\uacbc\ub2e4. \ud55c\ud3b8\uc73c\ub85c \uac1c\ucd5c\uad6d \ub178\ub974\uc6e8\uc774\uac00 \ucc98\uc74c\uc73c\ub85c \uc2a4\ud53c\ub4dc\uc2a4\ucf00\uc774\ud305 \uc804 \uc885\ubaa9\uc5d0 \ucd9c\uc804\ud574 3\uad00\uc655\uc744 \ub2ec\uc131\ud558\uc600\uace0, \ub300\ud55c\ubbfc\uad6d\uc758 \uae40\uc724\ubbf8 \uc120\uc218\uac00 \ub9cc 13\uc138\uc758 \ub098\uc774\ub85c \uc1fc\ud2b8\ud2b8\ub799 \ub2e8\uccb4\uacc4\uc8fc \uae08\uba54\ub2ec\uc744 \ucc28\uc9c0\ud558\uba74\uc11c, \ub3d9\uacc4\uc62c\ub9bc\ud53d \uc5ed\ub300 \ucd5c\uc5f0\uc18c \uae08\uba54\ub2ec\ub9ac\uc2a4\ud2b8\ub85c \ub0a8\uac8c \ub418\uc5c8\ub2e4. \ud6c4\uc548 \uc548\ud1a0\ub2c8\uc624 \uc0ac\ub9c8\ub780\uce58 IOC \uc704\uc6d0\uc7a5\uc740 \ub300\ud68c \ud3d0\ub9c9\uc2dd\uc5d0\uc11c \ub9b4\ub808\ud568\uba54\ub974 \ub3d9\uacc4\uc62c\ub9bc\ud53d\uc774 \uc0ac\uc0c1 \ucd5c\uace0\uc758 \ub3d9\uacc4\uc62c\ub9bc\ud53d \ub300\ud68c\uc600\ub2e4\uace0 \uce6d\ucc2c\ud558\uae30\ub3c4 \ud558\uc600\ub2e4.", "paragraph_answer": "1986\ub144 \uad6d\uc81c\uc62c\ub9bc\ud53d\uc704\uc6d0\ud68c\uac00 \uac19\uc740 \ud574\uc5d0 \uce58\ub7ec\uc9c0\ub358 \ub3d9\ud558\uacc4 \uc62c\ub9bc\ud53d \ub300\ud68c\ub97c \ubd84\ub9ac\uc2dc\ucf1c 2\ub144 \uaca9\ucc28\ub85c \uc5f0\ub2e4\ub294 \uacb0\uc815\uc744 \ud22c\ud45c\ub97c \ud1b5\ud574 \ucd5c\uc885 \ud655\uc815\ud558\uc600\uace0, 1994\ub144 \ub3d9\uacc4 \uc62c\ub9bc\ud53d\ubd80\ud130 \uc774 \uacb0\uc815\uc744 \ub530\ub974\uac8c \ub418\uc5c8\ub2e4. \ub178\ub974\uc6e8\uc774 \ub9b4\ub808\ud568\uba54\ub974\uc5d0\uc11c \uc5f4\ub9b0 \uc774 \ub300\ud68c\ub294 \ucc98\uc74c\uc73c\ub85c \ud558\uacc4 \uc62c\ub9bc\ud53d\uc5d0 2\ub144 \uc55e\uc11c \uce58\ub7ec\uc9c4 \ub300\ud68c\uac00 \ub418\uc5c8\ub2e4. 1993\ub144 \uccb4\ucf54\uc2ac\ub85c\ubc14\ud0a4\uc544 \ud574\uccb4\ub97c \uacaa\uc740 \uccb4\ucf54\uc640 \uc2ac\ub85c\ubc14\ud0a4\uc544\uac00 \ubcc4\uac1c\uc758 \uc120\uc218\ub2e8\uc73c\ub85c\uc11c \ub3d9\uacc4\uc62c\ub9bc\ud53d\uc5d0 \ucc98\uc74c \ucd9c\uc804\ud588\ub2e4. \ud55c\ud3b8 1994\ub144 1\uc6d4 6\uc77c \ubbf8\uad6d\uc758 \ud53c\uaca8\uc2a4\ucf00\uc774\ud305 \uc120\uc218 \ub0b8\uc2dc \ucf00\ub9ac\uac74\uc774 \uacbd\uc7c1\uc790 \ud1a0\ub0d0 \ud558\ub529\uc758 \uc804 \ub0a8\ud3b8\uc758 \uc0ac\uc8fc\ub85c \ud53c\uaca9\uc744 \ubc1b\ub294 \uc0ac\uc0c1\ucd08\uc720\uc758 \uc0ac\ud0dc\uac00 \ubc8c\uc5b4\uc9c0\uba74\uc11c, \uc5ec\uc790 \ud53c\uaca8\uc2a4\ucf00\uc774\ud305 \uacbd\uae30\uac00 \uc0c8\uc0bc \uc5b8\ub860\uc758 \uc774\ubaa9\uc744 \ub04c\uac8c \ub418\uc5c8\ub2e4. \ucf00\ub9ac\uac74\uacfc \ud558\ub529 \ubaa8\ub450 \ub300\ud68c\uc5d0\ub294 \uc815\uc0c1 \ucd9c\uc804\ud558\uc600\uc73c\ub098 \uce90\ub9ac\uac74\ub9cc \uc740\uba54\ub2ec\uc744 \ud68d\ub4dd\ud558\uc600\uace0, \uc6b0\ud06c\ub77c\uc774\ub098\uc758 \uc624\ud06c\uc0ac\ub098 \ubc14\uc774\uc6b8\uc774 \uc6b0\uc2b9\ud558\uba74\uc11c \uace0\uad6d\uc5d0 \uccab \uae08\uba54\ub2ec\uc744 \uc548\uacbc\ub2e4. \ud55c\ud3b8\uc73c\ub85c \uac1c\ucd5c\uad6d \ub178\ub974\uc6e8\uc774\uac00 \ucc98\uc74c\uc73c\ub85c \uc2a4\ud53c\ub4dc\uc2a4\ucf00\uc774\ud305 \uc804 \uc885\ubaa9\uc5d0 \ucd9c\uc804\ud574 3\uad00\uc655\uc744 \ub2ec\uc131\ud558\uc600\uace0, \ub300\ud55c\ubbfc\uad6d\uc758 \uae40\uc724\ubbf8 \uc120\uc218\uac00 \ub9cc 13\uc138\uc758 \ub098\uc774\ub85c \uc1fc\ud2b8\ud2b8\ub799 \ub2e8\uccb4\uacc4\uc8fc \uae08\uba54\ub2ec\uc744 \ucc28\uc9c0\ud558\uba74\uc11c, \ub3d9\uacc4\uc62c\ub9bc\ud53d \uc5ed\ub300 \ucd5c\uc5f0\uc18c \uae08\uba54\ub2ec\ub9ac\uc2a4\ud2b8\ub85c \ub0a8\uac8c \ub418\uc5c8\ub2e4. \ud6c4\uc548 \uc548\ud1a0\ub2c8\uc624 \uc0ac\ub9c8\ub780\uce58 IOC \uc704\uc6d0\uc7a5\uc740 \ub300\ud68c \ud3d0\ub9c9\uc2dd\uc5d0\uc11c \ub9b4\ub808\ud568\uba54\ub974 \ub3d9\uacc4\uc62c\ub9bc\ud53d\uc774 \uc0ac\uc0c1 \ucd5c\uace0\uc758 \ub3d9\uacc4\uc62c\ub9bc\ud53d \ub300\ud68c\uc600\ub2e4\uace0 \uce6d\ucc2c\ud558\uae30\ub3c4 \ud558\uc600\ub2e4.", "sentence_answer": " \ub178\ub974\uc6e8\uc774 \ub9b4\ub808\ud568\uba54\ub974\uc5d0\uc11c \uc5f4\ub9b0 \uc774 \ub300\ud68c\ub294 \ucc98\uc74c\uc73c\ub85c \ud558\uacc4 \uc62c\ub9bc\ud53d\uc5d0 2\ub144 \uc55e\uc11c \uce58\ub7ec\uc9c4 \ub300\ud68c\uac00 \ub418\uc5c8\ub2e4.", "paragraph_id": "6047160-12-0", "paragraph_question": "question: \ucc98\uc74c\uc73c\ub85c \ud558\uacc4 \uc62c\ub9bc\ud53d\uc5d0 2\ub144 \uc55e\uc11c \uce58\ub904\uc9c4 \ub3d9\uacc4\uc62c\ub9bc\ud53d\uc774 \uac1c\ucd5c\ub41c \uad6d\uac00\ub294?, context: 1986\ub144 \uad6d\uc81c\uc62c\ub9bc\ud53d\uc704\uc6d0\ud68c\uac00 \uac19\uc740 \ud574\uc5d0 \uce58\ub7ec\uc9c0\ub358 \ub3d9\ud558\uacc4 \uc62c\ub9bc\ud53d \ub300\ud68c\ub97c \ubd84\ub9ac\uc2dc\ucf1c 2\ub144 \uaca9\ucc28\ub85c \uc5f0\ub2e4\ub294 \uacb0\uc815\uc744 \ud22c\ud45c\ub97c \ud1b5\ud574 \ucd5c\uc885 \ud655\uc815\ud558\uc600\uace0, 1994\ub144 \ub3d9\uacc4 \uc62c\ub9bc\ud53d\ubd80\ud130 \uc774 \uacb0\uc815\uc744 \ub530\ub974\uac8c \ub418\uc5c8\ub2e4. \ub178\ub974\uc6e8\uc774 \ub9b4\ub808\ud568\uba54\ub974\uc5d0\uc11c \uc5f4\ub9b0 \uc774 \ub300\ud68c\ub294 \ucc98\uc74c\uc73c\ub85c \ud558\uacc4 \uc62c\ub9bc\ud53d\uc5d0 2\ub144 \uc55e\uc11c \uce58\ub7ec\uc9c4 \ub300\ud68c\uac00 \ub418\uc5c8\ub2e4. 1993\ub144 \uccb4\ucf54\uc2ac\ub85c\ubc14\ud0a4\uc544 \ud574\uccb4\ub97c \uacaa\uc740 \uccb4\ucf54\uc640 \uc2ac\ub85c\ubc14\ud0a4\uc544\uac00 \ubcc4\uac1c\uc758 \uc120\uc218\ub2e8\uc73c\ub85c\uc11c \ub3d9\uacc4\uc62c\ub9bc\ud53d\uc5d0 \ucc98\uc74c \ucd9c\uc804\ud588\ub2e4. \ud55c\ud3b8 1994\ub144 1\uc6d4 6\uc77c \ubbf8\uad6d\uc758 \ud53c\uaca8\uc2a4\ucf00\uc774\ud305 \uc120\uc218 \ub0b8\uc2dc \ucf00\ub9ac\uac74\uc774 \uacbd\uc7c1\uc790 \ud1a0\ub0d0 \ud558\ub529\uc758 \uc804 \ub0a8\ud3b8\uc758 \uc0ac\uc8fc\ub85c \ud53c\uaca9\uc744 \ubc1b\ub294 \uc0ac\uc0c1\ucd08\uc720\uc758 \uc0ac\ud0dc\uac00 \ubc8c\uc5b4\uc9c0\uba74\uc11c, \uc5ec\uc790 \ud53c\uaca8\uc2a4\ucf00\uc774\ud305 \uacbd\uae30\uac00 \uc0c8\uc0bc \uc5b8\ub860\uc758 \uc774\ubaa9\uc744 \ub04c\uac8c \ub418\uc5c8\ub2e4. \ucf00\ub9ac\uac74\uacfc \ud558\ub529 \ubaa8\ub450 \ub300\ud68c\uc5d0\ub294 \uc815\uc0c1 \ucd9c\uc804\ud558\uc600\uc73c\ub098 \uce90\ub9ac\uac74\ub9cc \uc740\uba54\ub2ec\uc744 \ud68d\ub4dd\ud558\uc600\uace0, \uc6b0\ud06c\ub77c\uc774\ub098\uc758 \uc624\ud06c\uc0ac\ub098 \ubc14\uc774\uc6b8\uc774 \uc6b0\uc2b9\ud558\uba74\uc11c \uace0\uad6d\uc5d0 \uccab \uae08\uba54\ub2ec\uc744 \uc548\uacbc\ub2e4. \ud55c\ud3b8\uc73c\ub85c \uac1c\ucd5c\uad6d \ub178\ub974\uc6e8\uc774\uac00 \ucc98\uc74c\uc73c\ub85c \uc2a4\ud53c\ub4dc\uc2a4\ucf00\uc774\ud305 \uc804 \uc885\ubaa9\uc5d0 \ucd9c\uc804\ud574 3\uad00\uc655\uc744 \ub2ec\uc131\ud558\uc600\uace0, \ub300\ud55c\ubbfc\uad6d\uc758 \uae40\uc724\ubbf8 \uc120\uc218\uac00 \ub9cc 13\uc138\uc758 \ub098\uc774\ub85c \uc1fc\ud2b8\ud2b8\ub799 \ub2e8\uccb4\uacc4\uc8fc \uae08\uba54\ub2ec\uc744 \ucc28\uc9c0\ud558\uba74\uc11c, \ub3d9\uacc4\uc62c\ub9bc\ud53d \uc5ed\ub300 \ucd5c\uc5f0\uc18c \uae08\uba54\ub2ec\ub9ac\uc2a4\ud2b8\ub85c \ub0a8\uac8c \ub418\uc5c8\ub2e4. \ud6c4\uc548 \uc548\ud1a0\ub2c8\uc624 \uc0ac\ub9c8\ub780\uce58 IOC \uc704\uc6d0\uc7a5\uc740 \ub300\ud68c \ud3d0\ub9c9\uc2dd\uc5d0\uc11c \ub9b4\ub808\ud568\uba54\ub974 \ub3d9\uacc4\uc62c\ub9bc\ud53d\uc774 \uc0ac\uc0c1 \ucd5c\uace0\uc758 \ub3d9\uacc4\uc62c\ub9bc\ud53d \ub300\ud68c\uc600\ub2e4\uace0 \uce6d\ucc2c\ud558\uae30\ub3c4 \ud558\uc600\ub2e4."} {"question": "\uc911\uc559\uc77c\ubcf4\uc0ac \uc804 \uc0ac\uc7a5\uc758 \uc774\ub984\uc740?", "paragraph": "\uae40\uad6c\ub77c\uc5d0 \uc758\ud558\uba74 \uc870\uc7ac\ud604\uc774 \ubd80\ub3d9\uc0b0\ub9cc 600\uc5b5\ub300\uc758 \ubd80\uc790\ub77c\uace0 \ud558\uba70 \uc870\uc7ac\ud604\uc758 \ubd80\uce5c\uc774 \uc11d\uc720\uad00\ub828\ubd80\ud638\ub77c\uace0 \ud55c\ub2e4. \uc989 \uc774\uac74\ud76c\ub098 \uae40\ud559\uc758\ucc98\ub7fc \uc7ac\ub825\uacfc \uc778\ub9e5\uc5d0 \uc758\ud558\uc5ec \ucc98\ubc8c\uc744 \uc548 \ubc1b\ub294 \uac83\uc77c \uac00\ub2a5\uc131\uc774 \ub192\ub2e4. \uc21c\uc9c4\ud55c \uc5b4\ub9b0\uc774\uac00 \uc544\ub2c8\ub77c\uba74 \ud55c\uad6d\uc5d0\uc11c \uc5b4\ub5bb\uac8c \uc778\ub9e5\uacfc \ub3c8\uc774 \uc791\uc6a9\ud558\ub294\uc9c0 \uc54c \uac83\uc774\ub2e4. \ud2b9\ud788 \uc11d\uc720\uad00\ub828\ubd80\ud638\ub77c\uba74 \uad6d\uc81c\uc801 \uc778\ub9e5\uc758 \uc77c\ubd80\ub77c\uace0 \ubcf4\uc5ec\uc9c0\uba70 \uc911\uc559\uc77c\ubcf4\uc0ac \uc804 \uc0ac\uc7a5\uc778 \ud64d\uc11d\ud604\ub3c4 \uad6d\uc81c\uc801 \uc778\ub9e5\uc758 \ubcf4\uc720\uc790\ub85c \uc774\uac74\ud76c \uc0bc\uc131\uacfc \uc5f0\uad00\uc774 \uc788\ub2e4\ub294 \uc810\uc5d0\uc11c \uc870\uc7ac\ud604\uc774 \ubbf8\ud22c\ub85c \uc774\ub807\uac8c \uc5b8\ub860\uc5d0\uc11c \ub9de\ub294\ub300\ub3c4 \ubc84\ud2f0\ub294 \uac83\uc740 \uadf8 \ub4a4\uc758 \uac70\ub300\ud55c \uc7ac\ub825\ub54c\ubb38\uc784\uc774\ud655\uc2e4\ud574\ubcf4\uc778\ub2e4. \ubb38\uc7ac\uc778\ub300\ud1b5\ub839\ub3c4 \uc801\ud3d0\uccad\uc0b0\uc744 \ud55c\ub2e4\uace0 \ud558\uc9c0\ub9cc \uc624\ud788\ub824 \uc131\ubc94\uc8c4\uc5d0 \ub300\ud558\uc5ec \uac1c\uc785\uc744 \uc548\ud558\ub294\ub370 \uc774\uac74\ud76c, \uae40\ud559\uc758 \uc0ac\ub840\ub098 \uc5d8\uc2dc\ud2f0 \ub8f8\uc0b4\ub871\uc811\ub300 \ubcf4\ub3c4\ucc98\ub7fc \uae30\ub4dd\uad8c\uc774 \uc131\ub9e4\ub9e4 \ubc0f \uc131\ubc94\uc8c4\uc758 \uc628\uc0c1\uc73c\ub85c\uc11c \uac70\ub300\ud55c \ubcf4\ud638\ub97c \ubc1b\uae30 \ub54c\ubb38\uc73c\ub85c \ubcf4\uc778\ub2e4. \uadf8\ub7f0\ub370 \uc544\uc774\ub7ec\ub2c8\ud55c \uc810\uc740 \uc815\ubcf4\uc5d0 \ube60\ub978 \uadf8\ub4e4\uc774 \uc624\ud788\ub824 \uc870\uc7ac\ud604\uc758 \uac00\uc871\uc774 \ub178\ucd9c\ub418\ub3c4\ub85d \ud568\uc73c\ub85c\uc368 \uac00\uc871\ub9c8\uc800 \uce58\uba85\ud0c0\ub97c \uc5bb\uac8c \ub9cc\ub4e4\uc5c8\ub2e4\ub294 \uc0ac\uc2e4\uc774\ub2e4. \uadf8\ub9ac\uace0 \uae40\uae30\ub355\uc774 \uadf8\ub807\uac8c \uc601\ud654\uacc4\uc5d0\uc11c \uc798 \ub098\uac04 \uac83 \uc5ed\uc2dc \uc870\uc7ac\ud604\uc774\ub77c\ub294 \uad6d\uc81c\uc801 \uc778\ub9e5\uc758 \ubc30\uc6b0\uc640 \uadf8\uc790\uc5d0\uac8c \uc798 \uc5b4\uc6b8\ub9ac\ub294 \ubc94\uc8c4\ucee4\ub125\uc158\uc758 \uae40\uae30\ub355\uc744 \ub52e\uc2a4\ud14c\uc774\ud2b8\uc5d0\uc11c \ub3c4\uc640\uc904 \uac00\ub2a5\uc131\uc774 \ub192\ub2e4. \ubc29\ud0c4\uc18c\ub144\ub2e8\uc744 \ubcf4\ub354\ub77c\ub3c4 \ubba4\ube44\uc5d0\uc11c \uc77c\ub8e8\ubbf8\ub098\ud2f0\ub97c \uc804\ud558\ub294 \uc601\uc0c1\uc744 \ubcf4\uc5ec\uc8fc\ub294\ub370 \uacf5\uad50\ub86d\uac8c\ub3c4 \uadf8 \uc774\ud6c4 \ud06c\uac8c \ub5b4\uc73c\uba70 \uc774\ud6c4 \ucd94\ubbf8\uc560\uac00 \ubc29\ud0c4\uc18c\ub144\ub2e8\uc744 \ub744\uc6cc \uc8fc\uba74\uc11c \uc815\uce58\uc778\ub4e4\ub354\ub7ec \uc798\ud558\uc790\uace0 \ud588\ub294\ub370 \uad6d\uc81c\uc815\ubcf4\uc5d0 \ube60\ub978 \ucd94\ubbf8\uc560\uac00 \uadf8\ub7ac\ub2e4\ub294 \uac83\uc740 \ub354\ubd88\uc5b4\ubbfc\uc8fc\ub2f9 \uc5ed\uc2dc \ub52e\uc2a4\ud14c\uc774\ud2b8\uc758 \uc885\uc73c\ub85c\uc11c \uc790\ud55c\ub2f9\uacfc \uac19\uc740 \ucc98\uc9c0\uc784\uc744 \ub4dc\ub7ec\ub0b8\ub2e4. \uc131\ub839\ubaa8\ub3c5\ubc94\uc8c4\ub97c \ub36e\uc73c\ub824\uace0 \ubc14\ud2f0\uce78\uacfc \ubb38\uc815\ubd80\uac00 \uc801\ud3d0\uccad\uc0b0 \uc1fc\ub97c \ub300\ub300\uc801\uc73c\ub85c \ubc8c\uc774\uba74\uc11c \uadf8 \uc1fc\uac00 \uc131\uacf5\ud558\uba74 \uadf8\ud6c4 \ub52e\uc2a4\ud14c\uc774\ud2b8\ub294 \ud55c\ubbfc\uc871\uc744 \uc57d\uc18c\ubbfc\uc871\uc73c\ub85c\uc11c \uc804\uc7c1\uacfc \uacbd\uc81c\ub780\uc73c\ub85c \uc9d3\ubc1f\uc744 \uac83\uc774\uace0 \uacc4\ud68d\ub300\ub85c \uc2e4\ud328\ud558\uba74 \uc624\ud788\ub824 \uadf8 \uccad\uc0b0\uc1fc\uac00 \uae30\ud68c\ub97c \uc7a1\uc73c\ub824 \ud588\ub358 \uac83\uc774\ub77c\uace0 \ub2e4\uc2dc \uac70\uc9d3\ub9d0\ud560 \uac83\uc774\ub2e4.", "answer": "\ud64d\uc11d\ud604", "sentence": "\ud2b9\ud788 \uc11d\uc720\uad00\ub828\ubd80\ud638\ub77c\uba74 \uad6d\uc81c\uc801 \uc778\ub9e5\uc758 \uc77c\ubd80\ub77c\uace0 \ubcf4\uc5ec\uc9c0\uba70 \uc911\uc559\uc77c\ubcf4\uc0ac \uc804 \uc0ac\uc7a5\uc778 \ud64d\uc11d\ud604 \ub3c4 \uad6d\uc81c\uc801 \uc778\ub9e5\uc758 \ubcf4\uc720\uc790\ub85c \uc774\uac74\ud76c \uc0bc\uc131\uacfc \uc5f0\uad00\uc774 \uc788\ub2e4\ub294 \uc810\uc5d0\uc11c \uc870\uc7ac\ud604\uc774 \ubbf8\ud22c\ub85c \uc774\ub807\uac8c \uc5b8\ub860\uc5d0\uc11c \ub9de\ub294\ub300\ub3c4 \ubc84\ud2f0\ub294 \uac83\uc740 \uadf8 \ub4a4\uc758 \uac70\ub300\ud55c \uc7ac\ub825\ub54c\ubb38\uc784\uc774\ud655\uc2e4\ud574\ubcf4\uc778\ub2e4.", "paragraph_sentence": "\uae40\uad6c\ub77c\uc5d0 \uc758\ud558\uba74 \uc870\uc7ac\ud604\uc774 \ubd80\ub3d9\uc0b0\ub9cc 600\uc5b5\ub300\uc758 \ubd80\uc790\ub77c\uace0 \ud558\uba70 \uc870\uc7ac\ud604\uc758 \ubd80\uce5c\uc774 \uc11d\uc720\uad00\ub828\ubd80\ud638\ub77c\uace0 \ud55c\ub2e4. \uc989 \uc774\uac74\ud76c\ub098 \uae40\ud559\uc758\ucc98\ub7fc \uc7ac\ub825\uacfc \uc778\ub9e5\uc5d0 \uc758\ud558\uc5ec \ucc98\ubc8c\uc744 \uc548 \ubc1b\ub294 \uac83\uc77c \uac00\ub2a5\uc131\uc774 \ub192\ub2e4. \uc21c\uc9c4\ud55c \uc5b4\ub9b0\uc774\uac00 \uc544\ub2c8\ub77c\uba74 \ud55c\uad6d\uc5d0\uc11c \uc5b4\ub5bb\uac8c \uc778\ub9e5\uacfc \ub3c8\uc774 \uc791\uc6a9\ud558\ub294\uc9c0 \uc54c \uac83\uc774\ub2e4. \ud2b9\ud788 \uc11d\uc720\uad00\ub828\ubd80\ud638\ub77c\uba74 \uad6d\uc81c\uc801 \uc778\ub9e5\uc758 \uc77c\ubd80\ub77c\uace0 \ubcf4\uc5ec\uc9c0\uba70 \uc911\uc559\uc77c\ubcf4\uc0ac \uc804 \uc0ac\uc7a5\uc778 \ud64d\uc11d\ud604 \ub3c4 \uad6d\uc81c\uc801 \uc778\ub9e5\uc758 \ubcf4\uc720\uc790\ub85c \uc774\uac74\ud76c \uc0bc\uc131\uacfc \uc5f0\uad00\uc774 \uc788\ub2e4\ub294 \uc810\uc5d0\uc11c \uc870\uc7ac\ud604\uc774 \ubbf8\ud22c\ub85c \uc774\ub807\uac8c \uc5b8\ub860\uc5d0\uc11c \ub9de\ub294\ub300\ub3c4 \ubc84\ud2f0\ub294 \uac83\uc740 \uadf8 \ub4a4\uc758 \uac70\ub300\ud55c \uc7ac\ub825\ub54c\ubb38\uc784\uc774\ud655\uc2e4\ud574\ubcf4\uc778\ub2e4. \ubb38\uc7ac\uc778\ub300\ud1b5\ub839\ub3c4 \uc801\ud3d0\uccad\uc0b0\uc744 \ud55c\ub2e4\uace0 \ud558\uc9c0\ub9cc \uc624\ud788\ub824 \uc131\ubc94\uc8c4\uc5d0 \ub300\ud558\uc5ec \uac1c\uc785\uc744 \uc548\ud558\ub294\ub370 \uc774\uac74\ud76c, \uae40\ud559\uc758 \uc0ac\ub840\ub098 \uc5d8\uc2dc\ud2f0 \ub8f8\uc0b4\ub871\uc811\ub300 \ubcf4\ub3c4\ucc98\ub7fc \uae30\ub4dd\uad8c\uc774 \uc131\ub9e4\ub9e4 \ubc0f \uc131\ubc94\uc8c4\uc758 \uc628\uc0c1\uc73c\ub85c\uc11c \uac70\ub300\ud55c \ubcf4\ud638\ub97c \ubc1b\uae30 \ub54c\ubb38\uc73c\ub85c \ubcf4\uc778\ub2e4. \uadf8\ub7f0\ub370 \uc544\uc774\ub7ec\ub2c8\ud55c \uc810\uc740 \uc815\ubcf4\uc5d0 \ube60\ub978 \uadf8\ub4e4\uc774 \uc624\ud788\ub824 \uc870\uc7ac\ud604\uc758 \uac00\uc871\uc774 \ub178\ucd9c\ub418\ub3c4\ub85d \ud568\uc73c\ub85c\uc368 \uac00\uc871\ub9c8\uc800 \uce58\uba85\ud0c0\ub97c \uc5bb\uac8c \ub9cc\ub4e4\uc5c8\ub2e4\ub294 \uc0ac\uc2e4\uc774\ub2e4. \uadf8\ub9ac\uace0 \uae40\uae30\ub355\uc774 \uadf8\ub807\uac8c \uc601\ud654\uacc4\uc5d0\uc11c \uc798 \ub098\uac04 \uac83 \uc5ed\uc2dc \uc870\uc7ac\ud604\uc774\ub77c\ub294 \uad6d\uc81c\uc801 \uc778\ub9e5\uc758 \ubc30\uc6b0\uc640 \uadf8\uc790\uc5d0\uac8c \uc798 \uc5b4\uc6b8\ub9ac\ub294 \ubc94\uc8c4\ucee4\ub125\uc158\uc758 \uae40\uae30\ub355\uc744 \ub52e\uc2a4\ud14c\uc774\ud2b8\uc5d0\uc11c \ub3c4\uc640\uc904 \uac00\ub2a5\uc131\uc774 \ub192\ub2e4. \ubc29\ud0c4\uc18c\ub144\ub2e8\uc744 \ubcf4\ub354\ub77c\ub3c4 \ubba4\ube44\uc5d0\uc11c \uc77c\ub8e8\ubbf8\ub098\ud2f0\ub97c \uc804\ud558\ub294 \uc601\uc0c1\uc744 \ubcf4\uc5ec\uc8fc\ub294\ub370 \uacf5\uad50\ub86d\uac8c\ub3c4 \uadf8 \uc774\ud6c4 \ud06c\uac8c \ub5b4\uc73c\uba70 \uc774\ud6c4 \ucd94\ubbf8\uc560\uac00 \ubc29\ud0c4\uc18c\ub144\ub2e8\uc744 \ub744\uc6cc \uc8fc\uba74\uc11c \uc815\uce58\uc778\ub4e4\ub354\ub7ec \uc798\ud558\uc790\uace0 \ud588\ub294\ub370 \uad6d\uc81c\uc815\ubcf4\uc5d0 \ube60\ub978 \ucd94\ubbf8\uc560\uac00 \uadf8\ub7ac\ub2e4\ub294 \uac83\uc740 \ub354\ubd88\uc5b4\ubbfc\uc8fc\ub2f9 \uc5ed\uc2dc \ub52e\uc2a4\ud14c\uc774\ud2b8\uc758 \uc885\uc73c\ub85c\uc11c \uc790\ud55c\ub2f9\uacfc \uac19\uc740 \ucc98\uc9c0\uc784\uc744 \ub4dc\ub7ec\ub0b8\ub2e4. \uc131\ub839\ubaa8\ub3c5\ubc94\uc8c4\ub97c \ub36e\uc73c\ub824\uace0 \ubc14\ud2f0\uce78\uacfc \ubb38\uc815\ubd80\uac00 \uc801\ud3d0\uccad\uc0b0 \uc1fc\ub97c \ub300\ub300\uc801\uc73c\ub85c \ubc8c\uc774\uba74\uc11c \uadf8 \uc1fc\uac00 \uc131\uacf5\ud558\uba74 \uadf8\ud6c4 \ub52e\uc2a4\ud14c\uc774\ud2b8\ub294 \ud55c\ubbfc\uc871\uc744 \uc57d\uc18c\ubbfc\uc871\uc73c\ub85c\uc11c \uc804\uc7c1\uacfc \uacbd\uc81c\ub780\uc73c\ub85c \uc9d3\ubc1f\uc744 \uac83\uc774\uace0 \uacc4\ud68d\ub300\ub85c \uc2e4\ud328\ud558\uba74 \uc624\ud788\ub824 \uadf8 \uccad\uc0b0\uc1fc\uac00 \uae30\ud68c\ub97c \uc7a1\uc73c\ub824 \ud588\ub358 \uac83\uc774\ub77c\uace0 \ub2e4\uc2dc \uac70\uc9d3\ub9d0\ud560 \uac83\uc774\ub2e4.", "paragraph_answer": "\uae40\uad6c\ub77c\uc5d0 \uc758\ud558\uba74 \uc870\uc7ac\ud604\uc774 \ubd80\ub3d9\uc0b0\ub9cc 600\uc5b5\ub300\uc758 \ubd80\uc790\ub77c\uace0 \ud558\uba70 \uc870\uc7ac\ud604\uc758 \ubd80\uce5c\uc774 \uc11d\uc720\uad00\ub828\ubd80\ud638\ub77c\uace0 \ud55c\ub2e4. \uc989 \uc774\uac74\ud76c\ub098 \uae40\ud559\uc758\ucc98\ub7fc \uc7ac\ub825\uacfc \uc778\ub9e5\uc5d0 \uc758\ud558\uc5ec \ucc98\ubc8c\uc744 \uc548 \ubc1b\ub294 \uac83\uc77c \uac00\ub2a5\uc131\uc774 \ub192\ub2e4. \uc21c\uc9c4\ud55c \uc5b4\ub9b0\uc774\uac00 \uc544\ub2c8\ub77c\uba74 \ud55c\uad6d\uc5d0\uc11c \uc5b4\ub5bb\uac8c \uc778\ub9e5\uacfc \ub3c8\uc774 \uc791\uc6a9\ud558\ub294\uc9c0 \uc54c \uac83\uc774\ub2e4. \ud2b9\ud788 \uc11d\uc720\uad00\ub828\ubd80\ud638\ub77c\uba74 \uad6d\uc81c\uc801 \uc778\ub9e5\uc758 \uc77c\ubd80\ub77c\uace0 \ubcf4\uc5ec\uc9c0\uba70 \uc911\uc559\uc77c\ubcf4\uc0ac \uc804 \uc0ac\uc7a5\uc778 \ud64d\uc11d\ud604 \ub3c4 \uad6d\uc81c\uc801 \uc778\ub9e5\uc758 \ubcf4\uc720\uc790\ub85c \uc774\uac74\ud76c \uc0bc\uc131\uacfc \uc5f0\uad00\uc774 \uc788\ub2e4\ub294 \uc810\uc5d0\uc11c \uc870\uc7ac\ud604\uc774 \ubbf8\ud22c\ub85c \uc774\ub807\uac8c \uc5b8\ub860\uc5d0\uc11c \ub9de\ub294\ub300\ub3c4 \ubc84\ud2f0\ub294 \uac83\uc740 \uadf8 \ub4a4\uc758 \uac70\ub300\ud55c \uc7ac\ub825\ub54c\ubb38\uc784\uc774\ud655\uc2e4\ud574\ubcf4\uc778\ub2e4. \ubb38\uc7ac\uc778\ub300\ud1b5\ub839\ub3c4 \uc801\ud3d0\uccad\uc0b0\uc744 \ud55c\ub2e4\uace0 \ud558\uc9c0\ub9cc \uc624\ud788\ub824 \uc131\ubc94\uc8c4\uc5d0 \ub300\ud558\uc5ec \uac1c\uc785\uc744 \uc548\ud558\ub294\ub370 \uc774\uac74\ud76c, \uae40\ud559\uc758 \uc0ac\ub840\ub098 \uc5d8\uc2dc\ud2f0 \ub8f8\uc0b4\ub871\uc811\ub300 \ubcf4\ub3c4\ucc98\ub7fc \uae30\ub4dd\uad8c\uc774 \uc131\ub9e4\ub9e4 \ubc0f \uc131\ubc94\uc8c4\uc758 \uc628\uc0c1\uc73c\ub85c\uc11c \uac70\ub300\ud55c \ubcf4\ud638\ub97c \ubc1b\uae30 \ub54c\ubb38\uc73c\ub85c \ubcf4\uc778\ub2e4. \uadf8\ub7f0\ub370 \uc544\uc774\ub7ec\ub2c8\ud55c \uc810\uc740 \uc815\ubcf4\uc5d0 \ube60\ub978 \uadf8\ub4e4\uc774 \uc624\ud788\ub824 \uc870\uc7ac\ud604\uc758 \uac00\uc871\uc774 \ub178\ucd9c\ub418\ub3c4\ub85d \ud568\uc73c\ub85c\uc368 \uac00\uc871\ub9c8\uc800 \uce58\uba85\ud0c0\ub97c \uc5bb\uac8c \ub9cc\ub4e4\uc5c8\ub2e4\ub294 \uc0ac\uc2e4\uc774\ub2e4. \uadf8\ub9ac\uace0 \uae40\uae30\ub355\uc774 \uadf8\ub807\uac8c \uc601\ud654\uacc4\uc5d0\uc11c \uc798 \ub098\uac04 \uac83 \uc5ed\uc2dc \uc870\uc7ac\ud604\uc774\ub77c\ub294 \uad6d\uc81c\uc801 \uc778\ub9e5\uc758 \ubc30\uc6b0\uc640 \uadf8\uc790\uc5d0\uac8c \uc798 \uc5b4\uc6b8\ub9ac\ub294 \ubc94\uc8c4\ucee4\ub125\uc158\uc758 \uae40\uae30\ub355\uc744 \ub52e\uc2a4\ud14c\uc774\ud2b8\uc5d0\uc11c \ub3c4\uc640\uc904 \uac00\ub2a5\uc131\uc774 \ub192\ub2e4. \ubc29\ud0c4\uc18c\ub144\ub2e8\uc744 \ubcf4\ub354\ub77c\ub3c4 \ubba4\ube44\uc5d0\uc11c \uc77c\ub8e8\ubbf8\ub098\ud2f0\ub97c \uc804\ud558\ub294 \uc601\uc0c1\uc744 \ubcf4\uc5ec\uc8fc\ub294\ub370 \uacf5\uad50\ub86d\uac8c\ub3c4 \uadf8 \uc774\ud6c4 \ud06c\uac8c \ub5b4\uc73c\uba70 \uc774\ud6c4 \ucd94\ubbf8\uc560\uac00 \ubc29\ud0c4\uc18c\ub144\ub2e8\uc744 \ub744\uc6cc \uc8fc\uba74\uc11c \uc815\uce58\uc778\ub4e4\ub354\ub7ec \uc798\ud558\uc790\uace0 \ud588\ub294\ub370 \uad6d\uc81c\uc815\ubcf4\uc5d0 \ube60\ub978 \ucd94\ubbf8\uc560\uac00 \uadf8\ub7ac\ub2e4\ub294 \uac83\uc740 \ub354\ubd88\uc5b4\ubbfc\uc8fc\ub2f9 \uc5ed\uc2dc \ub52e\uc2a4\ud14c\uc774\ud2b8\uc758 \uc885\uc73c\ub85c\uc11c \uc790\ud55c\ub2f9\uacfc \uac19\uc740 \ucc98\uc9c0\uc784\uc744 \ub4dc\ub7ec\ub0b8\ub2e4. \uc131\ub839\ubaa8\ub3c5\ubc94\uc8c4\ub97c \ub36e\uc73c\ub824\uace0 \ubc14\ud2f0\uce78\uacfc \ubb38\uc815\ubd80\uac00 \uc801\ud3d0\uccad\uc0b0 \uc1fc\ub97c \ub300\ub300\uc801\uc73c\ub85c \ubc8c\uc774\uba74\uc11c \uadf8 \uc1fc\uac00 \uc131\uacf5\ud558\uba74 \uadf8\ud6c4 \ub52e\uc2a4\ud14c\uc774\ud2b8\ub294 \ud55c\ubbfc\uc871\uc744 \uc57d\uc18c\ubbfc\uc871\uc73c\ub85c\uc11c \uc804\uc7c1\uacfc \uacbd\uc81c\ub780\uc73c\ub85c \uc9d3\ubc1f\uc744 \uac83\uc774\uace0 \uacc4\ud68d\ub300\ub85c \uc2e4\ud328\ud558\uba74 \uc624\ud788\ub824 \uadf8 \uccad\uc0b0\uc1fc\uac00 \uae30\ud68c\ub97c \uc7a1\uc73c\ub824 \ud588\ub358 \uac83\uc774\ub77c\uace0 \ub2e4\uc2dc \uac70\uc9d3\ub9d0\ud560 \uac83\uc774\ub2e4.", "sentence_answer": "\ud2b9\ud788 \uc11d\uc720\uad00\ub828\ubd80\ud638\ub77c\uba74 \uad6d\uc81c\uc801 \uc778\ub9e5\uc758 \uc77c\ubd80\ub77c\uace0 \ubcf4\uc5ec\uc9c0\uba70 \uc911\uc559\uc77c\ubcf4\uc0ac \uc804 \uc0ac\uc7a5\uc778 \ud64d\uc11d\ud604 \ub3c4 \uad6d\uc81c\uc801 \uc778\ub9e5\uc758 \ubcf4\uc720\uc790\ub85c \uc774\uac74\ud76c \uc0bc\uc131\uacfc \uc5f0\uad00\uc774 \uc788\ub2e4\ub294 \uc810\uc5d0\uc11c \uc870\uc7ac\ud604\uc774 \ubbf8\ud22c\ub85c \uc774\ub807\uac8c \uc5b8\ub860\uc5d0\uc11c \ub9de\ub294\ub300\ub3c4 \ubc84\ud2f0\ub294 \uac83\uc740 \uadf8 \ub4a4\uc758 \uac70\ub300\ud55c \uc7ac\ub825\ub54c\ubb38\uc784\uc774\ud655\uc2e4\ud574\ubcf4\uc778\ub2e4.", "paragraph_id": "6554140-2-0", "paragraph_question": "question: \uc911\uc559\uc77c\ubcf4\uc0ac \uc804 \uc0ac\uc7a5\uc758 \uc774\ub984\uc740?, context: \uae40\uad6c\ub77c\uc5d0 \uc758\ud558\uba74 \uc870\uc7ac\ud604\uc774 \ubd80\ub3d9\uc0b0\ub9cc 600\uc5b5\ub300\uc758 \ubd80\uc790\ub77c\uace0 \ud558\uba70 \uc870\uc7ac\ud604\uc758 \ubd80\uce5c\uc774 \uc11d\uc720\uad00\ub828\ubd80\ud638\ub77c\uace0 \ud55c\ub2e4. \uc989 \uc774\uac74\ud76c\ub098 \uae40\ud559\uc758\ucc98\ub7fc \uc7ac\ub825\uacfc \uc778\ub9e5\uc5d0 \uc758\ud558\uc5ec \ucc98\ubc8c\uc744 \uc548 \ubc1b\ub294 \uac83\uc77c \uac00\ub2a5\uc131\uc774 \ub192\ub2e4. \uc21c\uc9c4\ud55c \uc5b4\ub9b0\uc774\uac00 \uc544\ub2c8\ub77c\uba74 \ud55c\uad6d\uc5d0\uc11c \uc5b4\ub5bb\uac8c \uc778\ub9e5\uacfc \ub3c8\uc774 \uc791\uc6a9\ud558\ub294\uc9c0 \uc54c \uac83\uc774\ub2e4. \ud2b9\ud788 \uc11d\uc720\uad00\ub828\ubd80\ud638\ub77c\uba74 \uad6d\uc81c\uc801 \uc778\ub9e5\uc758 \uc77c\ubd80\ub77c\uace0 \ubcf4\uc5ec\uc9c0\uba70 \uc911\uc559\uc77c\ubcf4\uc0ac \uc804 \uc0ac\uc7a5\uc778 \ud64d\uc11d\ud604\ub3c4 \uad6d\uc81c\uc801 \uc778\ub9e5\uc758 \ubcf4\uc720\uc790\ub85c \uc774\uac74\ud76c \uc0bc\uc131\uacfc \uc5f0\uad00\uc774 \uc788\ub2e4\ub294 \uc810\uc5d0\uc11c \uc870\uc7ac\ud604\uc774 \ubbf8\ud22c\ub85c \uc774\ub807\uac8c \uc5b8\ub860\uc5d0\uc11c \ub9de\ub294\ub300\ub3c4 \ubc84\ud2f0\ub294 \uac83\uc740 \uadf8 \ub4a4\uc758 \uac70\ub300\ud55c \uc7ac\ub825\ub54c\ubb38\uc784\uc774\ud655\uc2e4\ud574\ubcf4\uc778\ub2e4. \ubb38\uc7ac\uc778\ub300\ud1b5\ub839\ub3c4 \uc801\ud3d0\uccad\uc0b0\uc744 \ud55c\ub2e4\uace0 \ud558\uc9c0\ub9cc \uc624\ud788\ub824 \uc131\ubc94\uc8c4\uc5d0 \ub300\ud558\uc5ec \uac1c\uc785\uc744 \uc548\ud558\ub294\ub370 \uc774\uac74\ud76c, \uae40\ud559\uc758 \uc0ac\ub840\ub098 \uc5d8\uc2dc\ud2f0 \ub8f8\uc0b4\ub871\uc811\ub300 \ubcf4\ub3c4\ucc98\ub7fc \uae30\ub4dd\uad8c\uc774 \uc131\ub9e4\ub9e4 \ubc0f \uc131\ubc94\uc8c4\uc758 \uc628\uc0c1\uc73c\ub85c\uc11c \uac70\ub300\ud55c \ubcf4\ud638\ub97c \ubc1b\uae30 \ub54c\ubb38\uc73c\ub85c \ubcf4\uc778\ub2e4. \uadf8\ub7f0\ub370 \uc544\uc774\ub7ec\ub2c8\ud55c \uc810\uc740 \uc815\ubcf4\uc5d0 \ube60\ub978 \uadf8\ub4e4\uc774 \uc624\ud788\ub824 \uc870\uc7ac\ud604\uc758 \uac00\uc871\uc774 \ub178\ucd9c\ub418\ub3c4\ub85d \ud568\uc73c\ub85c\uc368 \uac00\uc871\ub9c8\uc800 \uce58\uba85\ud0c0\ub97c \uc5bb\uac8c \ub9cc\ub4e4\uc5c8\ub2e4\ub294 \uc0ac\uc2e4\uc774\ub2e4. \uadf8\ub9ac\uace0 \uae40\uae30\ub355\uc774 \uadf8\ub807\uac8c \uc601\ud654\uacc4\uc5d0\uc11c \uc798 \ub098\uac04 \uac83 \uc5ed\uc2dc \uc870\uc7ac\ud604\uc774\ub77c\ub294 \uad6d\uc81c\uc801 \uc778\ub9e5\uc758 \ubc30\uc6b0\uc640 \uadf8\uc790\uc5d0\uac8c \uc798 \uc5b4\uc6b8\ub9ac\ub294 \ubc94\uc8c4\ucee4\ub125\uc158\uc758 \uae40\uae30\ub355\uc744 \ub52e\uc2a4\ud14c\uc774\ud2b8\uc5d0\uc11c \ub3c4\uc640\uc904 \uac00\ub2a5\uc131\uc774 \ub192\ub2e4. \ubc29\ud0c4\uc18c\ub144\ub2e8\uc744 \ubcf4\ub354\ub77c\ub3c4 \ubba4\ube44\uc5d0\uc11c \uc77c\ub8e8\ubbf8\ub098\ud2f0\ub97c \uc804\ud558\ub294 \uc601\uc0c1\uc744 \ubcf4\uc5ec\uc8fc\ub294\ub370 \uacf5\uad50\ub86d\uac8c\ub3c4 \uadf8 \uc774\ud6c4 \ud06c\uac8c \ub5b4\uc73c\uba70 \uc774\ud6c4 \ucd94\ubbf8\uc560\uac00 \ubc29\ud0c4\uc18c\ub144\ub2e8\uc744 \ub744\uc6cc \uc8fc\uba74\uc11c \uc815\uce58\uc778\ub4e4\ub354\ub7ec \uc798\ud558\uc790\uace0 \ud588\ub294\ub370 \uad6d\uc81c\uc815\ubcf4\uc5d0 \ube60\ub978 \ucd94\ubbf8\uc560\uac00 \uadf8\ub7ac\ub2e4\ub294 \uac83\uc740 \ub354\ubd88\uc5b4\ubbfc\uc8fc\ub2f9 \uc5ed\uc2dc \ub52e\uc2a4\ud14c\uc774\ud2b8\uc758 \uc885\uc73c\ub85c\uc11c \uc790\ud55c\ub2f9\uacfc \uac19\uc740 \ucc98\uc9c0\uc784\uc744 \ub4dc\ub7ec\ub0b8\ub2e4. \uc131\ub839\ubaa8\ub3c5\ubc94\uc8c4\ub97c \ub36e\uc73c\ub824\uace0 \ubc14\ud2f0\uce78\uacfc \ubb38\uc815\ubd80\uac00 \uc801\ud3d0\uccad\uc0b0 \uc1fc\ub97c \ub300\ub300\uc801\uc73c\ub85c \ubc8c\uc774\uba74\uc11c \uadf8 \uc1fc\uac00 \uc131\uacf5\ud558\uba74 \uadf8\ud6c4 \ub52e\uc2a4\ud14c\uc774\ud2b8\ub294 \ud55c\ubbfc\uc871\uc744 \uc57d\uc18c\ubbfc\uc871\uc73c\ub85c\uc11c \uc804\uc7c1\uacfc \uacbd\uc81c\ub780\uc73c\ub85c \uc9d3\ubc1f\uc744 \uac83\uc774\uace0 \uacc4\ud68d\ub300\ub85c \uc2e4\ud328\ud558\uba74 \uc624\ud788\ub824 \uadf8 \uccad\uc0b0\uc1fc\uac00 \uae30\ud68c\ub97c \uc7a1\uc73c\ub824 \ud588\ub358 \uac83\uc774\ub77c\uace0 \ub2e4\uc2dc \uac70\uc9d3\ub9d0\ud560 \uac83\uc774\ub2e4."} {"question": "\ube0c\uc774 \ud3ec \ubca4\ub370\ud0c0\uc758 \uc81c\uc791\uc790\ub294 \ub204\uad6c\uc778\uac00?", "paragraph": "\uc774 \uc601\ud654\uc758 \uc81c\uc791\uc790\ub4e4 \uc911 \ub2e4\uc218\ub294 \u300a\ub9e4\ud2b8\ub9ad\uc2a4 \uc2dc\ub9ac\uc988\u300b\uc640 \uad00\ub828\uc774 \uc788\ub294 \uc0ac\ub78c\ub4e4\uc774\ub2e4. 1988\ub144, \uc81c\uc791\uc790 \uc870\uc5d8 \uc2e4\ubc84\ub294 \uc568\ub7f0 \ubb34\uc5b4\uc758 \ub450 \uc791\ud488 \u300a\ube0c\uc774 \ud3ec \ubca4\ub370\ud0c0\u300b\uc640 \u300a\uc653\uce58\ub9e8\u300b\uc758 \uad8c\ub9ac\ub97c \ud68d\ub4dd\ud588\ub2e4. \u300a\ube0c\uc774 \ud3ec \ubca4\ub370\ud0c0\u300b\uc758 \ud32c\uc774\uc5c8\ub358 \uc6cc\uc1fc\uc2a4\ud0a4 \ud615\uc81c\ub294 \u300a\ub9e4\ud2b8\ub9ad\uc2a4\u300b \uc791\uc5c5\uc744 \ud558\uae30 \uc804\uc778 1990\ub144\ub300 \uc911\ubc18\uc5d0 \uc6d0\uc791\uc744 \ucda9\uc2e4\ud558\uac8c \ub530\ub978 \uac01\ubcf8\uc758 \ucd08\uc548\uc744 \uc37c\ub2e4. \u300a\ub9e4\ud2b8\ub9ad\uc2a4\u300b\uc758 2\ud3b8\uacfc 3\ud3b8\uc744 \ud3b8\uc9d1\ud558\ub358 \ub3d9\uc548, \uadf8\ub4e4\uc740 \uac01\ubcf8\uc744 \uac80\ud1a0\ud55c \ub4a4 \uc81c\uc784\uc2a4 \ub9e5\ud14c\uc774\uadf8\uc5d0\uac8c \uac10\ub3c5\uc9c1\uc744 \uc81c\uc548\ud588\ub2e4. \uc138 \uc0ac\ub78c \ubaa8\ub450 \uc6d0\uc791\uc758 \ud14c\ub9c8\ub4e4\uc774 \ud604\uc7ac\uc758 \uc815\uce58 \uad6c\ub3c4\uc5d0 \uc0c1\uc751\ud55c\ub2e4\ub294 \uac83\uc744 \ubc1c\uacac\ud588\ub2e4. \uac01\ubcf8\uc744 \uac80\ud1a0\ud558\uba74\uc11c, \uc6cc\uc1fc\uc2a4\ud0a4 \ud615\uc81c\ub294 \uc774\uc57c\uae30\ub97c \uc751\ucd95\ud558\uace0 \ud604\ub300\ud654\ud558\uba74\uc11c\ub3c4 \uadf8 \uadfc\uac04\uacfc \ud14c\ub9c8\ub97c \uc720\uc9c0\ud558\uae30 \uc704\ud574 \uc218\uc815\uc5d0 \ucc29\uc218\ud588\ub2e4. \ub9e5\ud14c\uc774\uadf8\ub294 \u300a\ube0c\uc774 \ud3ec \ubca4\ub370\ud0c0\u300b\ub97c \uc900\ube44\ud558\uba74\uc11c \uc601\ud654 \u300a\uc54c\uc81c\ub9ac \uc804\ud22c\u300b\uc5d0\uc11c \uc8fc\uc694\ud55c \uc601\ud5a5\uc744 \ubc1b\uc558\ub2e4.", "answer": "\uc870\uc5d8 \uc2e4\ubc84", "sentence": "1988\ub144, \uc81c\uc791\uc790 \uc870\uc5d8 \uc2e4\ubc84 \ub294 \uc568\ub7f0 \ubb34\uc5b4\uc758 \ub450 \uc791\ud488 \u300a\ube0c\uc774 \ud3ec \ubca4\ub370\ud0c0\u300b\uc640 \u300a\uc653\uce58\ub9e8\u300b\uc758 \uad8c\ub9ac\ub97c \ud68d\ub4dd\ud588\ub2e4.", "paragraph_sentence": "\uc774 \uc601\ud654\uc758 \uc81c\uc791\uc790\ub4e4 \uc911 \ub2e4\uc218\ub294 \u300a\ub9e4\ud2b8\ub9ad\uc2a4 \uc2dc\ub9ac\uc988\u300b\uc640 \uad00\ub828\uc774 \uc788\ub294 \uc0ac\ub78c\ub4e4\uc774\ub2e4. 1988\ub144, \uc81c\uc791\uc790 \uc870\uc5d8 \uc2e4\ubc84 \ub294 \uc568\ub7f0 \ubb34\uc5b4\uc758 \ub450 \uc791\ud488 \u300a\ube0c\uc774 \ud3ec \ubca4\ub370\ud0c0\u300b\uc640 \u300a\uc653\uce58\ub9e8\u300b\uc758 \uad8c\ub9ac\ub97c \ud68d\ub4dd\ud588\ub2e4. \u300a\ube0c\uc774 \ud3ec \ubca4\ub370\ud0c0\u300b\uc758 \ud32c\uc774\uc5c8\ub358 \uc6cc\uc1fc\uc2a4\ud0a4 \ud615\uc81c\ub294 \u300a\ub9e4\ud2b8\ub9ad\uc2a4\u300b \uc791\uc5c5\uc744 \ud558\uae30 \uc804\uc778 1990\ub144\ub300 \uc911\ubc18\uc5d0 \uc6d0\uc791\uc744 \ucda9\uc2e4\ud558\uac8c \ub530\ub978 \uac01\ubcf8\uc758 \ucd08\uc548\uc744 \uc37c\ub2e4. \u300a\ub9e4\ud2b8\ub9ad\uc2a4\u300b\uc758 2\ud3b8\uacfc 3\ud3b8\uc744 \ud3b8\uc9d1\ud558\ub358 \ub3d9\uc548, \uadf8\ub4e4\uc740 \uac01\ubcf8\uc744 \uac80\ud1a0\ud55c \ub4a4 \uc81c\uc784\uc2a4 \ub9e5\ud14c\uc774\uadf8\uc5d0\uac8c \uac10\ub3c5\uc9c1\uc744 \uc81c\uc548\ud588\ub2e4. \uc138 \uc0ac\ub78c \ubaa8\ub450 \uc6d0\uc791\uc758 \ud14c\ub9c8\ub4e4\uc774 \ud604\uc7ac\uc758 \uc815\uce58 \uad6c\ub3c4\uc5d0 \uc0c1\uc751\ud55c\ub2e4\ub294 \uac83\uc744 \ubc1c\uacac\ud588\ub2e4. \uac01\ubcf8\uc744 \uac80\ud1a0\ud558\uba74\uc11c, \uc6cc\uc1fc\uc2a4\ud0a4 \ud615\uc81c\ub294 \uc774\uc57c\uae30\ub97c \uc751\ucd95\ud558\uace0 \ud604\ub300\ud654\ud558\uba74\uc11c\ub3c4 \uadf8 \uadfc\uac04\uacfc \ud14c\ub9c8\ub97c \uc720\uc9c0\ud558\uae30 \uc704\ud574 \uc218\uc815\uc5d0 \ucc29\uc218\ud588\ub2e4. \ub9e5\ud14c\uc774\uadf8\ub294 \u300a\ube0c\uc774 \ud3ec \ubca4\ub370\ud0c0\u300b\ub97c \uc900\ube44\ud558\uba74\uc11c \uc601\ud654 \u300a\uc54c\uc81c\ub9ac \uc804\ud22c\u300b\uc5d0\uc11c \uc8fc\uc694\ud55c \uc601\ud5a5\uc744 \ubc1b\uc558\ub2e4.", "paragraph_answer": "\uc774 \uc601\ud654\uc758 \uc81c\uc791\uc790\ub4e4 \uc911 \ub2e4\uc218\ub294 \u300a\ub9e4\ud2b8\ub9ad\uc2a4 \uc2dc\ub9ac\uc988\u300b\uc640 \uad00\ub828\uc774 \uc788\ub294 \uc0ac\ub78c\ub4e4\uc774\ub2e4. 1988\ub144, \uc81c\uc791\uc790 \uc870\uc5d8 \uc2e4\ubc84 \ub294 \uc568\ub7f0 \ubb34\uc5b4\uc758 \ub450 \uc791\ud488 \u300a\ube0c\uc774 \ud3ec \ubca4\ub370\ud0c0\u300b\uc640 \u300a\uc653\uce58\ub9e8\u300b\uc758 \uad8c\ub9ac\ub97c \ud68d\ub4dd\ud588\ub2e4. \u300a\ube0c\uc774 \ud3ec \ubca4\ub370\ud0c0\u300b\uc758 \ud32c\uc774\uc5c8\ub358 \uc6cc\uc1fc\uc2a4\ud0a4 \ud615\uc81c\ub294 \u300a\ub9e4\ud2b8\ub9ad\uc2a4\u300b \uc791\uc5c5\uc744 \ud558\uae30 \uc804\uc778 1990\ub144\ub300 \uc911\ubc18\uc5d0 \uc6d0\uc791\uc744 \ucda9\uc2e4\ud558\uac8c \ub530\ub978 \uac01\ubcf8\uc758 \ucd08\uc548\uc744 \uc37c\ub2e4. \u300a\ub9e4\ud2b8\ub9ad\uc2a4\u300b\uc758 2\ud3b8\uacfc 3\ud3b8\uc744 \ud3b8\uc9d1\ud558\ub358 \ub3d9\uc548, \uadf8\ub4e4\uc740 \uac01\ubcf8\uc744 \uac80\ud1a0\ud55c \ub4a4 \uc81c\uc784\uc2a4 \ub9e5\ud14c\uc774\uadf8\uc5d0\uac8c \uac10\ub3c5\uc9c1\uc744 \uc81c\uc548\ud588\ub2e4. \uc138 \uc0ac\ub78c \ubaa8\ub450 \uc6d0\uc791\uc758 \ud14c\ub9c8\ub4e4\uc774 \ud604\uc7ac\uc758 \uc815\uce58 \uad6c\ub3c4\uc5d0 \uc0c1\uc751\ud55c\ub2e4\ub294 \uac83\uc744 \ubc1c\uacac\ud588\ub2e4. \uac01\ubcf8\uc744 \uac80\ud1a0\ud558\uba74\uc11c, \uc6cc\uc1fc\uc2a4\ud0a4 \ud615\uc81c\ub294 \uc774\uc57c\uae30\ub97c \uc751\ucd95\ud558\uace0 \ud604\ub300\ud654\ud558\uba74\uc11c\ub3c4 \uadf8 \uadfc\uac04\uacfc \ud14c\ub9c8\ub97c \uc720\uc9c0\ud558\uae30 \uc704\ud574 \uc218\uc815\uc5d0 \ucc29\uc218\ud588\ub2e4. \ub9e5\ud14c\uc774\uadf8\ub294 \u300a\ube0c\uc774 \ud3ec \ubca4\ub370\ud0c0\u300b\ub97c \uc900\ube44\ud558\uba74\uc11c \uc601\ud654 \u300a\uc54c\uc81c\ub9ac \uc804\ud22c\u300b\uc5d0\uc11c \uc8fc\uc694\ud55c \uc601\ud5a5\uc744 \ubc1b\uc558\ub2e4.", "sentence_answer": "1988\ub144, \uc81c\uc791\uc790 \uc870\uc5d8 \uc2e4\ubc84 \ub294 \uc568\ub7f0 \ubb34\uc5b4\uc758 \ub450 \uc791\ud488 \u300a\ube0c\uc774 \ud3ec \ubca4\ub370\ud0c0\u300b\uc640 \u300a\uc653\uce58\ub9e8\u300b\uc758 \uad8c\ub9ac\ub97c \ud68d\ub4dd\ud588\ub2e4.", "paragraph_id": "6516998-5-0", "paragraph_question": "question: \ube0c\uc774 \ud3ec \ubca4\ub370\ud0c0\uc758 \uc81c\uc791\uc790\ub294 \ub204\uad6c\uc778\uac00?, context: \uc774 \uc601\ud654\uc758 \uc81c\uc791\uc790\ub4e4 \uc911 \ub2e4\uc218\ub294 \u300a\ub9e4\ud2b8\ub9ad\uc2a4 \uc2dc\ub9ac\uc988\u300b\uc640 \uad00\ub828\uc774 \uc788\ub294 \uc0ac\ub78c\ub4e4\uc774\ub2e4. 1988\ub144, \uc81c\uc791\uc790 \uc870\uc5d8 \uc2e4\ubc84\ub294 \uc568\ub7f0 \ubb34\uc5b4\uc758 \ub450 \uc791\ud488 \u300a\ube0c\uc774 \ud3ec \ubca4\ub370\ud0c0\u300b\uc640 \u300a\uc653\uce58\ub9e8\u300b\uc758 \uad8c\ub9ac\ub97c \ud68d\ub4dd\ud588\ub2e4. \u300a\ube0c\uc774 \ud3ec \ubca4\ub370\ud0c0\u300b\uc758 \ud32c\uc774\uc5c8\ub358 \uc6cc\uc1fc\uc2a4\ud0a4 \ud615\uc81c\ub294 \u300a\ub9e4\ud2b8\ub9ad\uc2a4\u300b \uc791\uc5c5\uc744 \ud558\uae30 \uc804\uc778 1990\ub144\ub300 \uc911\ubc18\uc5d0 \uc6d0\uc791\uc744 \ucda9\uc2e4\ud558\uac8c \ub530\ub978 \uac01\ubcf8\uc758 \ucd08\uc548\uc744 \uc37c\ub2e4. \u300a\ub9e4\ud2b8\ub9ad\uc2a4\u300b\uc758 2\ud3b8\uacfc 3\ud3b8\uc744 \ud3b8\uc9d1\ud558\ub358 \ub3d9\uc548, \uadf8\ub4e4\uc740 \uac01\ubcf8\uc744 \uac80\ud1a0\ud55c \ub4a4 \uc81c\uc784\uc2a4 \ub9e5\ud14c\uc774\uadf8\uc5d0\uac8c \uac10\ub3c5\uc9c1\uc744 \uc81c\uc548\ud588\ub2e4. \uc138 \uc0ac\ub78c \ubaa8\ub450 \uc6d0\uc791\uc758 \ud14c\ub9c8\ub4e4\uc774 \ud604\uc7ac\uc758 \uc815\uce58 \uad6c\ub3c4\uc5d0 \uc0c1\uc751\ud55c\ub2e4\ub294 \uac83\uc744 \ubc1c\uacac\ud588\ub2e4. \uac01\ubcf8\uc744 \uac80\ud1a0\ud558\uba74\uc11c, \uc6cc\uc1fc\uc2a4\ud0a4 \ud615\uc81c\ub294 \uc774\uc57c\uae30\ub97c \uc751\ucd95\ud558\uace0 \ud604\ub300\ud654\ud558\uba74\uc11c\ub3c4 \uadf8 \uadfc\uac04\uacfc \ud14c\ub9c8\ub97c \uc720\uc9c0\ud558\uae30 \uc704\ud574 \uc218\uc815\uc5d0 \ucc29\uc218\ud588\ub2e4. \ub9e5\ud14c\uc774\uadf8\ub294 \u300a\ube0c\uc774 \ud3ec \ubca4\ub370\ud0c0\u300b\ub97c \uc900\ube44\ud558\uba74\uc11c \uc601\ud654 \u300a\uc54c\uc81c\ub9ac \uc804\ud22c\u300b\uc5d0\uc11c \uc8fc\uc694\ud55c \uc601\ud5a5\uc744 \ubc1b\uc558\ub2e4."} {"question": "\uc624\uc2a4\ud2b8\ub9ac\uc544 \uc81c\uad6d \uc804\uccb4\uac00 \ud1b5\uc77c \ub3c5\uc77c\uc5d0 \ucc38\uc5ec\ud558\ub294 \ubc29\uc2dd\uc744 \uc774\ub974\ub294 \ub9d0\uc740?", "paragraph": "\uc624\uc2a4\ud2b8\ub9ac\uc544\uc758 \ub3c5\uc77c\uc778 \uc815\uce58\uac00\ub4e4\uc740 \uc624\uc2a4\ud2b8\ub9ac\uc544 \uc81c\uad6d\uc744 \uc77c\ubd80\ub9cc\uc744 \ucc38\uc5ec\uc2dc\ud0a4\ub294 \ud504\ub791\ud06c\ud478\ub974\ud2b8\uc2dd \ub300\ub3c5\uc77c\uc8fc\uc758(Frankfurterisches Gro\u00dfdetschtum)\uc5d0 \ubc18\ub300\ud558\uc5ec, \uc624\uc2a4\ud2b8\ub9ac\uc544 \uc81c\uad6d \uc804\uccb4\uac00 \ud1b5\uc77c \ub3c5\uc77c\uc5d0 \ucc38\uc5ec\ud574\uc57c \ud55c\ub2e4\ub294 \uc624\uc2a4\ud2b8\ub9ac\uc544\uc2dd \ub300\ub3c5\uc77c\uc8fc\uc758(\u00d6sterreichisches Gro\u00dfdeutschtum)\ub97c \uc8fc\uc7a5\ud588\ub2e4. \ud2b9\ud788 \uc624\uc2a4\ud2b8\ub9ac\uc544\uc758 \ud669\uc81c \ud398\ub974\ub514\ub09c\ud2b8 1\uc138\ub294 \uc790\uc2e0\uc758 \uad6d\uac00\ub97c \ucabc\uac24 \ub9c8\uc74c\uc774 \uc804\ud600 \uc5c6\uc5c8\uc73c\uba70, \ud504\ub780\uce20 \uc694\uc81c\ud504 1\uc138\uc5d0\uac8c \uc81c\uc704\ub97c \ub118\uaca8\uc8fc\uae30 \uaca8\uc6b0 \uba70\uce60 \uc804\uc778 11\uc6d4 27\uc77c\uc5d0\ub3c4 \uc790\uad6d\uc758 \ucd1d\ub9ac\uc778 \uc288\ubc14\ub974\uccb8\ubca0\ub974\ud06c(Felix zu Schwarzenberg)\uc5d0\uac8c \uc624\uc2a4\ud2b8\ub9ac\uc544\uc758 \ubd88\uac00\ubd84\uc131\uc744 \ucc9c\uba85\ud558\ub294 \uad50\uc11c\ub97c \ubc1c\ud45c\ud558\uac8c \ud558\uc600\ub2e4. \uc774\ub85c\uc368 \uad6d\ubbfc\uc758\ud68c\uac00 \ub3c4\ub2ec\ud560 \uc218 \uc788\ub294 \uad6d\uac00\uc801 \ud1b5\uc77c\uc758 \ucd5c\ub300\uce58\ub294 \ud504\ub85c\uc774\uc13c\uc774 \ucd5c\uac15\ub300\uad6d\uc778 \uc18c\ub3c5\uc77c\uc8fc\uc758 \ud574\ubc95\uc758 \ubc94\uc704 \uc548\uc5d0\uc11c\uc784\uc774 \uba85\ubc31\ud574\uc84c\ub2e4. \uad6d\ubbfc\uc758\ud68c\ub294 \uae30\uc874\uc758 \ub2f9\ud30c\uc640 \ubcc4\uac1c\ub85c \ub300\ub3c5\uc77c\uc8fc\uc758\uc640 \uc18c\ub3c5\uc77c\uc8fc\uc758\ub85c \ubd84\uc5f4\ub418\uc5c8\ub2e4. \uc288\ubc14\ub974\uccb8\ubca0\ub974\ud06c\uac00 1849\ub144 3\uc6d4\uc5d0 \ub2e4\uc2dc \ud55c \ubc88 \uc624\uc2a4\ud2b8\ub9ac\uc544 \uc804\uccb4\ub97c \uc0c8\ub85c \uac74\uc124\ub418\ub294 \uad6d\uac00\uc5d0 \ud3ec\ud568\ud558\ub294 \uac83\uacfc \ud569\uc2a4\ubd80\ub974\ud06c \uc655\uc870\uc758 \uc601\ud5a5\ub825\uc744 \ud604\uc800\ud788 \uc99d\ub300\uc2dc\ud0ac \uac83\uc744 \uc694\uad6c\ud558\uae30\ub294 \ud588\uc9c0\ub9cc, \uc624\uc2a4\ud2b8\ub9ac\uc544\uc778\uc778 \uc288\uba54\uc5b4\ub9c1\uc774 \uc624\uc2a4\ud2b8\ub9ac\uc544\uc758 \uc785\uc7a5\uacfc \uad6d\ubbfc\uc758\ud68c\uc758 \uc790\uc138 \uc0ac\uc774\uc758 \ub354 \uc774\uc0c1 \ud654\ud574\uac00\ub2a5\ud558\uc9c0 \uc54a\uc740 \ub300\ub9bd \ub54c\ubb38\uc5d0 \ucd1d\ub9ac \uc790\ub9ac\uc5d0\uc11c \uc0ac\ud1f4\ud55c \ud6c4\uc778 1848\ub144 12\uc6d4\uc5d0 \uc8fc\uc0ac\uc704\ub294 \uc774\ubbf8 \uc18c\ub3c5\uc77c\uc801 \uc81c\uad6d\uc73c\ub85c \uae30\uc6b8\uc5b4\uc84c\ub2e4. \uc288\uba54\uc5b4\ub9c1\uc758 \ub4a4\ub294 \ud558\uc778\ub9ac\ud788 \ud3f0 \uac00\uac8c\ub978\uc774 \uc774\uc5c8\ub2e4. \uac00\uac8c\ub978\uc740 \ud504\ub85c\uc774\uc13c \uc655\uc744 \ud1b5\uc77c \uc81c\uad6d\uc758 \uc138\uc2b5\ud669\uc81c\ub85c \ucd94\ub300\ud558\ub294 \uacc4\ud68d\uc744 \ucd94\uc9c4\ud558\uc600\ub2e4.", "answer": "\uc624\uc2a4\ud2b8\ub9ac\uc544\uc2dd \ub300\ub3c5\uc77c\uc8fc\uc758", "sentence": "\uc624\uc2a4\ud2b8\ub9ac\uc544\uc758 \ub3c5\uc77c\uc778 \uc815\uce58\uac00\ub4e4\uc740 \uc624\uc2a4\ud2b8\ub9ac\uc544 \uc81c\uad6d\uc744 \uc77c\ubd80\ub9cc\uc744 \ucc38\uc5ec\uc2dc\ud0a4\ub294 \ud504\ub791\ud06c\ud478\ub974\ud2b8\uc2dd \ub300\ub3c5\uc77c\uc8fc\uc758(Frankfurterisches Gro\u00dfdetschtum)\uc5d0 \ubc18\ub300\ud558\uc5ec, \uc624\uc2a4\ud2b8\ub9ac\uc544 \uc81c\uad6d \uc804\uccb4\uac00 \ud1b5\uc77c \ub3c5\uc77c\uc5d0 \ucc38\uc5ec\ud574\uc57c \ud55c\ub2e4\ub294 \uc624\uc2a4\ud2b8\ub9ac\uc544\uc2dd \ub300\ub3c5\uc77c\uc8fc\uc758 (\u00d6sterreichisches Gro\u00dfdeutschtum)", "paragraph_sentence": " \uc624\uc2a4\ud2b8\ub9ac\uc544\uc758 \ub3c5\uc77c\uc778 \uc815\uce58\uac00\ub4e4\uc740 \uc624\uc2a4\ud2b8\ub9ac\uc544 \uc81c\uad6d\uc744 \uc77c\ubd80\ub9cc\uc744 \ucc38\uc5ec\uc2dc\ud0a4\ub294 \ud504\ub791\ud06c\ud478\ub974\ud2b8\uc2dd \ub300\ub3c5\uc77c\uc8fc\uc758(Frankfurterisches Gro\u00dfdetschtum)\uc5d0 \ubc18\ub300\ud558\uc5ec, \uc624\uc2a4\ud2b8\ub9ac\uc544 \uc81c\uad6d \uc804\uccb4\uac00 \ud1b5\uc77c \ub3c5\uc77c\uc5d0 \ucc38\uc5ec\ud574\uc57c \ud55c\ub2e4\ub294 \uc624\uc2a4\ud2b8\ub9ac\uc544\uc2dd \ub300\ub3c5\uc77c\uc8fc\uc758 (\u00d6sterreichisches Gro\u00dfdeutschtum) \ub97c \uc8fc\uc7a5\ud588\ub2e4. \ud2b9\ud788 \uc624\uc2a4\ud2b8\ub9ac\uc544\uc758 \ud669\uc81c \ud398\ub974\ub514\ub09c\ud2b8 1\uc138\ub294 \uc790\uc2e0\uc758 \uad6d\uac00\ub97c \ucabc\uac24 \ub9c8\uc74c\uc774 \uc804\ud600 \uc5c6\uc5c8\uc73c\uba70, \ud504\ub780\uce20 \uc694\uc81c\ud504 1\uc138\uc5d0\uac8c \uc81c\uc704\ub97c \ub118\uaca8\uc8fc\uae30 \uaca8\uc6b0 \uba70\uce60 \uc804\uc778 11\uc6d4 27\uc77c\uc5d0\ub3c4 \uc790\uad6d\uc758 \ucd1d\ub9ac\uc778 \uc288\ubc14\ub974\uccb8\ubca0\ub974\ud06c(Felix zu Schwarzenberg)\uc5d0\uac8c \uc624\uc2a4\ud2b8\ub9ac\uc544\uc758 \ubd88\uac00\ubd84\uc131\uc744 \ucc9c\uba85\ud558\ub294 \uad50\uc11c\ub97c \ubc1c\ud45c\ud558\uac8c \ud558\uc600\ub2e4. \uc774\ub85c\uc368 \uad6d\ubbfc\uc758\ud68c\uac00 \ub3c4\ub2ec\ud560 \uc218 \uc788\ub294 \uad6d\uac00\uc801 \ud1b5\uc77c\uc758 \ucd5c\ub300\uce58\ub294 \ud504\ub85c\uc774\uc13c\uc774 \ucd5c\uac15\ub300\uad6d\uc778 \uc18c\ub3c5\uc77c\uc8fc\uc758 \ud574\ubc95\uc758 \ubc94\uc704 \uc548\uc5d0\uc11c\uc784\uc774 \uba85\ubc31\ud574\uc84c\ub2e4. \uad6d\ubbfc\uc758\ud68c\ub294 \uae30\uc874\uc758 \ub2f9\ud30c\uc640 \ubcc4\uac1c\ub85c \ub300\ub3c5\uc77c\uc8fc\uc758\uc640 \uc18c\ub3c5\uc77c\uc8fc\uc758\ub85c \ubd84\uc5f4\ub418\uc5c8\ub2e4. \uc288\ubc14\ub974\uccb8\ubca0\ub974\ud06c\uac00 1849\ub144 3\uc6d4\uc5d0 \ub2e4\uc2dc \ud55c \ubc88 \uc624\uc2a4\ud2b8\ub9ac\uc544 \uc804\uccb4\ub97c \uc0c8\ub85c \uac74\uc124\ub418\ub294 \uad6d\uac00\uc5d0 \ud3ec\ud568\ud558\ub294 \uac83\uacfc \ud569\uc2a4\ubd80\ub974\ud06c \uc655\uc870\uc758 \uc601\ud5a5\ub825\uc744 \ud604\uc800\ud788 \uc99d\ub300\uc2dc\ud0ac \uac83\uc744 \uc694\uad6c\ud558\uae30\ub294 \ud588\uc9c0\ub9cc, \uc624\uc2a4\ud2b8\ub9ac\uc544\uc778\uc778 \uc288\uba54\uc5b4\ub9c1\uc774 \uc624\uc2a4\ud2b8\ub9ac\uc544\uc758 \uc785\uc7a5\uacfc \uad6d\ubbfc\uc758\ud68c\uc758 \uc790\uc138 \uc0ac\uc774\uc758 \ub354 \uc774\uc0c1 \ud654\ud574\uac00\ub2a5\ud558\uc9c0 \uc54a\uc740 \ub300\ub9bd \ub54c\ubb38\uc5d0 \ucd1d\ub9ac \uc790\ub9ac\uc5d0\uc11c \uc0ac\ud1f4\ud55c \ud6c4\uc778 1848\ub144 12\uc6d4\uc5d0 \uc8fc\uc0ac\uc704\ub294 \uc774\ubbf8 \uc18c\ub3c5\uc77c\uc801 \uc81c\uad6d\uc73c\ub85c \uae30\uc6b8\uc5b4\uc84c\ub2e4. \uc288\uba54\uc5b4\ub9c1\uc758 \ub4a4\ub294 \ud558\uc778\ub9ac\ud788 \ud3f0 \uac00\uac8c\ub978\uc774 \uc774\uc5c8\ub2e4. \uac00\uac8c\ub978\uc740 \ud504\ub85c\uc774\uc13c \uc655\uc744 \ud1b5\uc77c \uc81c\uad6d\uc758 \uc138\uc2b5\ud669\uc81c\ub85c \ucd94\ub300\ud558\ub294 \uacc4\ud68d\uc744 \ucd94\uc9c4\ud558\uc600\ub2e4.", "paragraph_answer": "\uc624\uc2a4\ud2b8\ub9ac\uc544\uc758 \ub3c5\uc77c\uc778 \uc815\uce58\uac00\ub4e4\uc740 \uc624\uc2a4\ud2b8\ub9ac\uc544 \uc81c\uad6d\uc744 \uc77c\ubd80\ub9cc\uc744 \ucc38\uc5ec\uc2dc\ud0a4\ub294 \ud504\ub791\ud06c\ud478\ub974\ud2b8\uc2dd \ub300\ub3c5\uc77c\uc8fc\uc758(Frankfurterisches Gro\u00dfdetschtum)\uc5d0 \ubc18\ub300\ud558\uc5ec, \uc624\uc2a4\ud2b8\ub9ac\uc544 \uc81c\uad6d \uc804\uccb4\uac00 \ud1b5\uc77c \ub3c5\uc77c\uc5d0 \ucc38\uc5ec\ud574\uc57c \ud55c\ub2e4\ub294 \uc624\uc2a4\ud2b8\ub9ac\uc544\uc2dd \ub300\ub3c5\uc77c\uc8fc\uc758 (\u00d6sterreichisches Gro\u00dfdeutschtum)\ub97c \uc8fc\uc7a5\ud588\ub2e4. \ud2b9\ud788 \uc624\uc2a4\ud2b8\ub9ac\uc544\uc758 \ud669\uc81c \ud398\ub974\ub514\ub09c\ud2b8 1\uc138\ub294 \uc790\uc2e0\uc758 \uad6d\uac00\ub97c \ucabc\uac24 \ub9c8\uc74c\uc774 \uc804\ud600 \uc5c6\uc5c8\uc73c\uba70, \ud504\ub780\uce20 \uc694\uc81c\ud504 1\uc138\uc5d0\uac8c \uc81c\uc704\ub97c \ub118\uaca8\uc8fc\uae30 \uaca8\uc6b0 \uba70\uce60 \uc804\uc778 11\uc6d4 27\uc77c\uc5d0\ub3c4 \uc790\uad6d\uc758 \ucd1d\ub9ac\uc778 \uc288\ubc14\ub974\uccb8\ubca0\ub974\ud06c(Felix zu Schwarzenberg)\uc5d0\uac8c \uc624\uc2a4\ud2b8\ub9ac\uc544\uc758 \ubd88\uac00\ubd84\uc131\uc744 \ucc9c\uba85\ud558\ub294 \uad50\uc11c\ub97c \ubc1c\ud45c\ud558\uac8c \ud558\uc600\ub2e4. \uc774\ub85c\uc368 \uad6d\ubbfc\uc758\ud68c\uac00 \ub3c4\ub2ec\ud560 \uc218 \uc788\ub294 \uad6d\uac00\uc801 \ud1b5\uc77c\uc758 \ucd5c\ub300\uce58\ub294 \ud504\ub85c\uc774\uc13c\uc774 \ucd5c\uac15\ub300\uad6d\uc778 \uc18c\ub3c5\uc77c\uc8fc\uc758 \ud574\ubc95\uc758 \ubc94\uc704 \uc548\uc5d0\uc11c\uc784\uc774 \uba85\ubc31\ud574\uc84c\ub2e4. \uad6d\ubbfc\uc758\ud68c\ub294 \uae30\uc874\uc758 \ub2f9\ud30c\uc640 \ubcc4\uac1c\ub85c \ub300\ub3c5\uc77c\uc8fc\uc758\uc640 \uc18c\ub3c5\uc77c\uc8fc\uc758\ub85c \ubd84\uc5f4\ub418\uc5c8\ub2e4. \uc288\ubc14\ub974\uccb8\ubca0\ub974\ud06c\uac00 1849\ub144 3\uc6d4\uc5d0 \ub2e4\uc2dc \ud55c \ubc88 \uc624\uc2a4\ud2b8\ub9ac\uc544 \uc804\uccb4\ub97c \uc0c8\ub85c \uac74\uc124\ub418\ub294 \uad6d\uac00\uc5d0 \ud3ec\ud568\ud558\ub294 \uac83\uacfc \ud569\uc2a4\ubd80\ub974\ud06c \uc655\uc870\uc758 \uc601\ud5a5\ub825\uc744 \ud604\uc800\ud788 \uc99d\ub300\uc2dc\ud0ac \uac83\uc744 \uc694\uad6c\ud558\uae30\ub294 \ud588\uc9c0\ub9cc, \uc624\uc2a4\ud2b8\ub9ac\uc544\uc778\uc778 \uc288\uba54\uc5b4\ub9c1\uc774 \uc624\uc2a4\ud2b8\ub9ac\uc544\uc758 \uc785\uc7a5\uacfc \uad6d\ubbfc\uc758\ud68c\uc758 \uc790\uc138 \uc0ac\uc774\uc758 \ub354 \uc774\uc0c1 \ud654\ud574\uac00\ub2a5\ud558\uc9c0 \uc54a\uc740 \ub300\ub9bd \ub54c\ubb38\uc5d0 \ucd1d\ub9ac \uc790\ub9ac\uc5d0\uc11c \uc0ac\ud1f4\ud55c \ud6c4\uc778 1848\ub144 12\uc6d4\uc5d0 \uc8fc\uc0ac\uc704\ub294 \uc774\ubbf8 \uc18c\ub3c5\uc77c\uc801 \uc81c\uad6d\uc73c\ub85c \uae30\uc6b8\uc5b4\uc84c\ub2e4. \uc288\uba54\uc5b4\ub9c1\uc758 \ub4a4\ub294 \ud558\uc778\ub9ac\ud788 \ud3f0 \uac00\uac8c\ub978\uc774 \uc774\uc5c8\ub2e4. \uac00\uac8c\ub978\uc740 \ud504\ub85c\uc774\uc13c \uc655\uc744 \ud1b5\uc77c \uc81c\uad6d\uc758 \uc138\uc2b5\ud669\uc81c\ub85c \ucd94\ub300\ud558\ub294 \uacc4\ud68d\uc744 \ucd94\uc9c4\ud558\uc600\ub2e4.", "sentence_answer": "\uc624\uc2a4\ud2b8\ub9ac\uc544\uc758 \ub3c5\uc77c\uc778 \uc815\uce58\uac00\ub4e4\uc740 \uc624\uc2a4\ud2b8\ub9ac\uc544 \uc81c\uad6d\uc744 \uc77c\ubd80\ub9cc\uc744 \ucc38\uc5ec\uc2dc\ud0a4\ub294 \ud504\ub791\ud06c\ud478\ub974\ud2b8\uc2dd \ub300\ub3c5\uc77c\uc8fc\uc758(Frankfurterisches Gro\u00dfdetschtum)\uc5d0 \ubc18\ub300\ud558\uc5ec, \uc624\uc2a4\ud2b8\ub9ac\uc544 \uc81c\uad6d \uc804\uccb4\uac00 \ud1b5\uc77c \ub3c5\uc77c\uc5d0 \ucc38\uc5ec\ud574\uc57c \ud55c\ub2e4\ub294 \uc624\uc2a4\ud2b8\ub9ac\uc544\uc2dd \ub300\ub3c5\uc77c\uc8fc\uc758 (\u00d6sterreichisches Gro\u00dfdeutschtum)", "paragraph_id": "6465718-23-1", "paragraph_question": "question: \uc624\uc2a4\ud2b8\ub9ac\uc544 \uc81c\uad6d \uc804\uccb4\uac00 \ud1b5\uc77c \ub3c5\uc77c\uc5d0 \ucc38\uc5ec\ud558\ub294 \ubc29\uc2dd\uc744 \uc774\ub974\ub294 \ub9d0\uc740?, context: \uc624\uc2a4\ud2b8\ub9ac\uc544\uc758 \ub3c5\uc77c\uc778 \uc815\uce58\uac00\ub4e4\uc740 \uc624\uc2a4\ud2b8\ub9ac\uc544 \uc81c\uad6d\uc744 \uc77c\ubd80\ub9cc\uc744 \ucc38\uc5ec\uc2dc\ud0a4\ub294 \ud504\ub791\ud06c\ud478\ub974\ud2b8\uc2dd \ub300\ub3c5\uc77c\uc8fc\uc758(Frankfurterisches Gro\u00dfdetschtum)\uc5d0 \ubc18\ub300\ud558\uc5ec, \uc624\uc2a4\ud2b8\ub9ac\uc544 \uc81c\uad6d \uc804\uccb4\uac00 \ud1b5\uc77c \ub3c5\uc77c\uc5d0 \ucc38\uc5ec\ud574\uc57c \ud55c\ub2e4\ub294 \uc624\uc2a4\ud2b8\ub9ac\uc544\uc2dd \ub300\ub3c5\uc77c\uc8fc\uc758(\u00d6sterreichisches Gro\u00dfdeutschtum)\ub97c \uc8fc\uc7a5\ud588\ub2e4. \ud2b9\ud788 \uc624\uc2a4\ud2b8\ub9ac\uc544\uc758 \ud669\uc81c \ud398\ub974\ub514\ub09c\ud2b8 1\uc138\ub294 \uc790\uc2e0\uc758 \uad6d\uac00\ub97c \ucabc\uac24 \ub9c8\uc74c\uc774 \uc804\ud600 \uc5c6\uc5c8\uc73c\uba70, \ud504\ub780\uce20 \uc694\uc81c\ud504 1\uc138\uc5d0\uac8c \uc81c\uc704\ub97c \ub118\uaca8\uc8fc\uae30 \uaca8\uc6b0 \uba70\uce60 \uc804\uc778 11\uc6d4 27\uc77c\uc5d0\ub3c4 \uc790\uad6d\uc758 \ucd1d\ub9ac\uc778 \uc288\ubc14\ub974\uccb8\ubca0\ub974\ud06c(Felix zu Schwarzenberg)\uc5d0\uac8c \uc624\uc2a4\ud2b8\ub9ac\uc544\uc758 \ubd88\uac00\ubd84\uc131\uc744 \ucc9c\uba85\ud558\ub294 \uad50\uc11c\ub97c \ubc1c\ud45c\ud558\uac8c \ud558\uc600\ub2e4. \uc774\ub85c\uc368 \uad6d\ubbfc\uc758\ud68c\uac00 \ub3c4\ub2ec\ud560 \uc218 \uc788\ub294 \uad6d\uac00\uc801 \ud1b5\uc77c\uc758 \ucd5c\ub300\uce58\ub294 \ud504\ub85c\uc774\uc13c\uc774 \ucd5c\uac15\ub300\uad6d\uc778 \uc18c\ub3c5\uc77c\uc8fc\uc758 \ud574\ubc95\uc758 \ubc94\uc704 \uc548\uc5d0\uc11c\uc784\uc774 \uba85\ubc31\ud574\uc84c\ub2e4. \uad6d\ubbfc\uc758\ud68c\ub294 \uae30\uc874\uc758 \ub2f9\ud30c\uc640 \ubcc4\uac1c\ub85c \ub300\ub3c5\uc77c\uc8fc\uc758\uc640 \uc18c\ub3c5\uc77c\uc8fc\uc758\ub85c \ubd84\uc5f4\ub418\uc5c8\ub2e4. \uc288\ubc14\ub974\uccb8\ubca0\ub974\ud06c\uac00 1849\ub144 3\uc6d4\uc5d0 \ub2e4\uc2dc \ud55c \ubc88 \uc624\uc2a4\ud2b8\ub9ac\uc544 \uc804\uccb4\ub97c \uc0c8\ub85c \uac74\uc124\ub418\ub294 \uad6d\uac00\uc5d0 \ud3ec\ud568\ud558\ub294 \uac83\uacfc \ud569\uc2a4\ubd80\ub974\ud06c \uc655\uc870\uc758 \uc601\ud5a5\ub825\uc744 \ud604\uc800\ud788 \uc99d\ub300\uc2dc\ud0ac \uac83\uc744 \uc694\uad6c\ud558\uae30\ub294 \ud588\uc9c0\ub9cc, \uc624\uc2a4\ud2b8\ub9ac\uc544\uc778\uc778 \uc288\uba54\uc5b4\ub9c1\uc774 \uc624\uc2a4\ud2b8\ub9ac\uc544\uc758 \uc785\uc7a5\uacfc \uad6d\ubbfc\uc758\ud68c\uc758 \uc790\uc138 \uc0ac\uc774\uc758 \ub354 \uc774\uc0c1 \ud654\ud574\uac00\ub2a5\ud558\uc9c0 \uc54a\uc740 \ub300\ub9bd \ub54c\ubb38\uc5d0 \ucd1d\ub9ac \uc790\ub9ac\uc5d0\uc11c \uc0ac\ud1f4\ud55c \ud6c4\uc778 1848\ub144 12\uc6d4\uc5d0 \uc8fc\uc0ac\uc704\ub294 \uc774\ubbf8 \uc18c\ub3c5\uc77c\uc801 \uc81c\uad6d\uc73c\ub85c \uae30\uc6b8\uc5b4\uc84c\ub2e4. \uc288\uba54\uc5b4\ub9c1\uc758 \ub4a4\ub294 \ud558\uc778\ub9ac\ud788 \ud3f0 \uac00\uac8c\ub978\uc774 \uc774\uc5c8\ub2e4. \uac00\uac8c\ub978\uc740 \ud504\ub85c\uc774\uc13c \uc655\uc744 \ud1b5\uc77c \uc81c\uad6d\uc758 \uc138\uc2b5\ud669\uc81c\ub85c \ucd94\ub300\ud558\ub294 \uacc4\ud68d\uc744 \ucd94\uc9c4\ud558\uc600\ub2e4."} {"question": "\ub098\uce74\ubaa8\ub9ac\uac00 \ub514\uc2a4\ud1a0\uc158 \ubbf9\uc2f1 \uc791\uc5c5\uc744 \ud1b5\ud574 \ud504\ub85c\ub4c0\uc2f1\ud55c \uc74c\ubc18\uc740?", "paragraph": "\u300a\ubd88\uac00\uc0ac\uc758\u300b(\uc77c\ubcf8\uc5b4: \u4e0d\u601d\u8b70, \u3075\u3057\u304e \ud6c4\uc2dc\uae30)\ub294 1986\ub144 8\uc6d4 11\uc77c\uc5d0 \ubc1c\ub9e4\ub41c \uc77c\ubcf8\uc758 \uc5ec\uac00\uc218 \ub098\uce74\ubaa8\ub9ac \uc544\ud0a4\ub098(\u4e2d\u68ee\u660e\u83dc)\uc758 \uc544\ud649 \ubc88\uc9f8 \uc2a4\ud29c\ub514\uc624 \uc74c\ubc18\uc774\ub2e4. (LP: L-12595, CT: LKF-8095, CD: 32XL-155) \uac00\uc218\uc778 \ub098\uce74\ubaa8\ub9ac\uac00 \uc9c1\uc811 \ud504\ub85c\ub4c0\uc11c\ub85c\uc11c \ucc38\uc5ec\ud55c \ucd5c\ucd08\uc758 \uc74c\ubc18\uc774\uba70, \uace1\ub4e4\uc758 \ucee8\uc149\ud2b8\uc5d0\uc11c\ubd80\ud130 \ud6c4\ubc18 \uc791\uc5c5\uc5d0\uae4c\uc9c0 \uad00\uc5ec\ud55c \uc758\uc695\uc801\uc778 \uc791\ud488\uc73c\ub85c \ucd1d 10\uac1c\uc758 \uace1\uc774 \uc218\ub85d\ub418\uc5c8\ub2e4. \ud754\ud788 \u201c\ub098\uce74\ubaa8\ub9ac \uc544\ud0a4\ub098 \ucd5c\ub300\uc758 \ubb38\uc81c\uc791\u201d\uc774\ub77c \ubd80\ub974\uae30\ub3c4 \ud558\ub294\ub370, \uae30\ud68d \ub2e8\uacc4\uc5d0\uc11c\ubd80\ud130 \uc74c\ubc18\uc758 \ucee8\uc149\ud2b8\ub97c \u201c\ubaa9\uc18c\ub9ac\ub3c4 \uc545\uae30\uc758 \uc77c\ubd80\u201d\ub85c \uc815\ud558\uace0, \uadf8\uc5d0 \ub9de\ucdb0\uc9c4 \uc774\uc9c8\uc801\uc778 \uace1 \ubd84\uc704\uae30\uc640 \ub2f9\uc2dc\uc5d0 \ud754\uce58 \uc54a\uc558\ub358 \ub514\uc2a4\ud1a0\uc158 \ubbf9\uc2f1 \uc791\uc5c5\uc744 \uac70\uccd0\uc11c \ubcf4\uceec\uc774 \ub9e4\uc6b0 \ud76c\ubbf8\ud558\uac8c \ub4e4\ub838\uae30 \ub54c\ubb38\uc774\ub2e4. \uc774 \ub54c\ubb38\uc5d0 \ubd88\ub7c9\ud488\uc774 \uc544\ub2c8\ub0d0\ub294 \ud56d\uc758\ub97c \ubc1b\uc740 \ud574\ud504\ub2dd\ub3c4 \uc788\uc5c8\uc73c\ub098, \uace7 \ud3c9\ub860\uac00\ub4e4\ub85c\ubd80\ud130 \ub192\uc740 \ud3c9\uac00\ub97c \ubc1b\uc73c\uba70 \ub098\uce74\ubaa8\ub9ac\uc758 \uc74c\uc545\uc131\uacfc \uc81c\uc791 \uc5ed\ub7c9\uc744 \uc778\uc815\ubc1b\uc740 \uc74c\ubc18\uc774\ub2e4.", "answer": "\ubd88\uac00\uc0ac\uc758", "sentence": "\u300a \ubd88\uac00\uc0ac\uc758 \u300b(\uc77c\ubcf8\uc5b4: \u4e0d\u601d\u8b70, \u3075\u3057\u304e \ud6c4\uc2dc\uae30)\ub294 1986\ub144 8\uc6d4 11\uc77c\uc5d0 \ubc1c\ub9e4\ub41c \uc77c\ubcf8\uc758 \uc5ec\uac00\uc218 \ub098\uce74\ubaa8\ub9ac \uc544\ud0a4\ub098(\u4e2d\u68ee\u660e\u83dc)\uc758 \uc544\ud649 \ubc88\uc9f8 \uc2a4\ud29c\ub514\uc624 \uc74c\ubc18\uc774\ub2e4.", "paragraph_sentence": " \u300a \ubd88\uac00\uc0ac\uc758 \u300b(\uc77c\ubcf8\uc5b4: \u4e0d\u601d\u8b70, \u3075\u3057\u304e \ud6c4\uc2dc\uae30)\ub294 1986\ub144 8\uc6d4 11\uc77c\uc5d0 \ubc1c\ub9e4\ub41c \uc77c\ubcf8\uc758 \uc5ec\uac00\uc218 \ub098\uce74\ubaa8\ub9ac \uc544\ud0a4\ub098(\u4e2d\u68ee\u660e\u83dc)\uc758 \uc544\ud649 \ubc88\uc9f8 \uc2a4\ud29c\ub514\uc624 \uc74c\ubc18\uc774\ub2e4. (LP: L-12595, CT: LKF-8095, CD: 32XL-155) \uac00\uc218\uc778 \ub098\uce74\ubaa8\ub9ac\uac00 \uc9c1\uc811 \ud504\ub85c\ub4c0\uc11c\ub85c\uc11c \ucc38\uc5ec\ud55c \ucd5c\ucd08\uc758 \uc74c\ubc18\uc774\uba70, \uace1\ub4e4\uc758 \ucee8\uc149\ud2b8\uc5d0\uc11c\ubd80\ud130 \ud6c4\ubc18 \uc791\uc5c5\uc5d0\uae4c\uc9c0 \uad00\uc5ec\ud55c \uc758\uc695\uc801\uc778 \uc791\ud488\uc73c\ub85c \ucd1d 10\uac1c\uc758 \uace1\uc774 \uc218\ub85d\ub418\uc5c8\ub2e4. \ud754\ud788 \u201c\ub098\uce74\ubaa8\ub9ac \uc544\ud0a4\ub098 \ucd5c\ub300\uc758 \ubb38\uc81c\uc791\u201d\uc774\ub77c \ubd80\ub974\uae30\ub3c4 \ud558\ub294\ub370, \uae30\ud68d \ub2e8\uacc4\uc5d0\uc11c\ubd80\ud130 \uc74c\ubc18\uc758 \ucee8\uc149\ud2b8\ub97c \u201c\ubaa9\uc18c\ub9ac\ub3c4 \uc545\uae30\uc758 \uc77c\ubd80\u201d\ub85c \uc815\ud558\uace0, \uadf8\uc5d0 \ub9de\ucdb0\uc9c4 \uc774\uc9c8\uc801\uc778 \uace1 \ubd84\uc704\uae30\uc640 \ub2f9\uc2dc\uc5d0 \ud754\uce58 \uc54a\uc558\ub358 \ub514\uc2a4\ud1a0\uc158 \ubbf9\uc2f1 \uc791\uc5c5\uc744 \uac70\uccd0\uc11c \ubcf4\uceec\uc774 \ub9e4\uc6b0 \ud76c\ubbf8\ud558\uac8c \ub4e4\ub838\uae30 \ub54c\ubb38\uc774\ub2e4. \uc774 \ub54c\ubb38\uc5d0 \ubd88\ub7c9\ud488\uc774 \uc544\ub2c8\ub0d0\ub294 \ud56d\uc758\ub97c \ubc1b\uc740 \ud574\ud504\ub2dd\ub3c4 \uc788\uc5c8\uc73c\ub098, \uace7 \ud3c9\ub860\uac00\ub4e4\ub85c\ubd80\ud130 \ub192\uc740 \ud3c9\uac00\ub97c \ubc1b\uc73c\uba70 \ub098\uce74\ubaa8\ub9ac\uc758 \uc74c\uc545\uc131\uacfc \uc81c\uc791 \uc5ed\ub7c9\uc744 \uc778\uc815\ubc1b\uc740 \uc74c\ubc18\uc774\ub2e4.", "paragraph_answer": "\u300a \ubd88\uac00\uc0ac\uc758 \u300b(\uc77c\ubcf8\uc5b4: \u4e0d\u601d\u8b70, \u3075\u3057\u304e \ud6c4\uc2dc\uae30)\ub294 1986\ub144 8\uc6d4 11\uc77c\uc5d0 \ubc1c\ub9e4\ub41c \uc77c\ubcf8\uc758 \uc5ec\uac00\uc218 \ub098\uce74\ubaa8\ub9ac \uc544\ud0a4\ub098(\u4e2d\u68ee\u660e\u83dc)\uc758 \uc544\ud649 \ubc88\uc9f8 \uc2a4\ud29c\ub514\uc624 \uc74c\ubc18\uc774\ub2e4. (LP: L-12595, CT: LKF-8095, CD: 32XL-155) \uac00\uc218\uc778 \ub098\uce74\ubaa8\ub9ac\uac00 \uc9c1\uc811 \ud504\ub85c\ub4c0\uc11c\ub85c\uc11c \ucc38\uc5ec\ud55c \ucd5c\ucd08\uc758 \uc74c\ubc18\uc774\uba70, \uace1\ub4e4\uc758 \ucee8\uc149\ud2b8\uc5d0\uc11c\ubd80\ud130 \ud6c4\ubc18 \uc791\uc5c5\uc5d0\uae4c\uc9c0 \uad00\uc5ec\ud55c \uc758\uc695\uc801\uc778 \uc791\ud488\uc73c\ub85c \ucd1d 10\uac1c\uc758 \uace1\uc774 \uc218\ub85d\ub418\uc5c8\ub2e4. \ud754\ud788 \u201c\ub098\uce74\ubaa8\ub9ac \uc544\ud0a4\ub098 \ucd5c\ub300\uc758 \ubb38\uc81c\uc791\u201d\uc774\ub77c \ubd80\ub974\uae30\ub3c4 \ud558\ub294\ub370, \uae30\ud68d \ub2e8\uacc4\uc5d0\uc11c\ubd80\ud130 \uc74c\ubc18\uc758 \ucee8\uc149\ud2b8\ub97c \u201c\ubaa9\uc18c\ub9ac\ub3c4 \uc545\uae30\uc758 \uc77c\ubd80\u201d\ub85c \uc815\ud558\uace0, \uadf8\uc5d0 \ub9de\ucdb0\uc9c4 \uc774\uc9c8\uc801\uc778 \uace1 \ubd84\uc704\uae30\uc640 \ub2f9\uc2dc\uc5d0 \ud754\uce58 \uc54a\uc558\ub358 \ub514\uc2a4\ud1a0\uc158 \ubbf9\uc2f1 \uc791\uc5c5\uc744 \uac70\uccd0\uc11c \ubcf4\uceec\uc774 \ub9e4\uc6b0 \ud76c\ubbf8\ud558\uac8c \ub4e4\ub838\uae30 \ub54c\ubb38\uc774\ub2e4. \uc774 \ub54c\ubb38\uc5d0 \ubd88\ub7c9\ud488\uc774 \uc544\ub2c8\ub0d0\ub294 \ud56d\uc758\ub97c \ubc1b\uc740 \ud574\ud504\ub2dd\ub3c4 \uc788\uc5c8\uc73c\ub098, \uace7 \ud3c9\ub860\uac00\ub4e4\ub85c\ubd80\ud130 \ub192\uc740 \ud3c9\uac00\ub97c \ubc1b\uc73c\uba70 \ub098\uce74\ubaa8\ub9ac\uc758 \uc74c\uc545\uc131\uacfc \uc81c\uc791 \uc5ed\ub7c9\uc744 \uc778\uc815\ubc1b\uc740 \uc74c\ubc18\uc774\ub2e4.", "sentence_answer": "\u300a \ubd88\uac00\uc0ac\uc758 \u300b(\uc77c\ubcf8\uc5b4: \u4e0d\u601d\u8b70, \u3075\u3057\u304e \ud6c4\uc2dc\uae30)\ub294 1986\ub144 8\uc6d4 11\uc77c\uc5d0 \ubc1c\ub9e4\ub41c \uc77c\ubcf8\uc758 \uc5ec\uac00\uc218 \ub098\uce74\ubaa8\ub9ac \uc544\ud0a4\ub098(\u4e2d\u68ee\u660e\u83dc)\uc758 \uc544\ud649 \ubc88\uc9f8 \uc2a4\ud29c\ub514\uc624 \uc74c\ubc18\uc774\ub2e4.", "paragraph_id": "6425449-0-0", "paragraph_question": "question: \ub098\uce74\ubaa8\ub9ac\uac00 \ub514\uc2a4\ud1a0\uc158 \ubbf9\uc2f1 \uc791\uc5c5\uc744 \ud1b5\ud574 \ud504\ub85c\ub4c0\uc2f1\ud55c \uc74c\ubc18\uc740?, context: \u300a\ubd88\uac00\uc0ac\uc758\u300b(\uc77c\ubcf8\uc5b4: \u4e0d\u601d\u8b70, \u3075\u3057\u304e \ud6c4\uc2dc\uae30)\ub294 1986\ub144 8\uc6d4 11\uc77c\uc5d0 \ubc1c\ub9e4\ub41c \uc77c\ubcf8\uc758 \uc5ec\uac00\uc218 \ub098\uce74\ubaa8\ub9ac \uc544\ud0a4\ub098(\u4e2d\u68ee\u660e\u83dc)\uc758 \uc544\ud649 \ubc88\uc9f8 \uc2a4\ud29c\ub514\uc624 \uc74c\ubc18\uc774\ub2e4. (LP: L-12595, CT: LKF-8095, CD: 32XL-155) \uac00\uc218\uc778 \ub098\uce74\ubaa8\ub9ac\uac00 \uc9c1\uc811 \ud504\ub85c\ub4c0\uc11c\ub85c\uc11c \ucc38\uc5ec\ud55c \ucd5c\ucd08\uc758 \uc74c\ubc18\uc774\uba70, \uace1\ub4e4\uc758 \ucee8\uc149\ud2b8\uc5d0\uc11c\ubd80\ud130 \ud6c4\ubc18 \uc791\uc5c5\uc5d0\uae4c\uc9c0 \uad00\uc5ec\ud55c \uc758\uc695\uc801\uc778 \uc791\ud488\uc73c\ub85c \ucd1d 10\uac1c\uc758 \uace1\uc774 \uc218\ub85d\ub418\uc5c8\ub2e4. \ud754\ud788 \u201c\ub098\uce74\ubaa8\ub9ac \uc544\ud0a4\ub098 \ucd5c\ub300\uc758 \ubb38\uc81c\uc791\u201d\uc774\ub77c \ubd80\ub974\uae30\ub3c4 \ud558\ub294\ub370, \uae30\ud68d \ub2e8\uacc4\uc5d0\uc11c\ubd80\ud130 \uc74c\ubc18\uc758 \ucee8\uc149\ud2b8\ub97c \u201c\ubaa9\uc18c\ub9ac\ub3c4 \uc545\uae30\uc758 \uc77c\ubd80\u201d\ub85c \uc815\ud558\uace0, \uadf8\uc5d0 \ub9de\ucdb0\uc9c4 \uc774\uc9c8\uc801\uc778 \uace1 \ubd84\uc704\uae30\uc640 \ub2f9\uc2dc\uc5d0 \ud754\uce58 \uc54a\uc558\ub358 \ub514\uc2a4\ud1a0\uc158 \ubbf9\uc2f1 \uc791\uc5c5\uc744 \uac70\uccd0\uc11c \ubcf4\uceec\uc774 \ub9e4\uc6b0 \ud76c\ubbf8\ud558\uac8c \ub4e4\ub838\uae30 \ub54c\ubb38\uc774\ub2e4. \uc774 \ub54c\ubb38\uc5d0 \ubd88\ub7c9\ud488\uc774 \uc544\ub2c8\ub0d0\ub294 \ud56d\uc758\ub97c \ubc1b\uc740 \ud574\ud504\ub2dd\ub3c4 \uc788\uc5c8\uc73c\ub098, \uace7 \ud3c9\ub860\uac00\ub4e4\ub85c\ubd80\ud130 \ub192\uc740 \ud3c9\uac00\ub97c \ubc1b\uc73c\uba70 \ub098\uce74\ubaa8\ub9ac\uc758 \uc74c\uc545\uc131\uacfc \uc81c\uc791 \uc5ed\ub7c9\uc744 \uc778\uc815\ubc1b\uc740 \uc74c\ubc18\uc774\ub2e4."} {"question": "\uc5d4\ud2b8\ubd80\uc778\uc758 \ud589\ubc29\uc774 \ubb18\uc5f0\ud574\uc9c4 \uac83\uc740 \uc5b4\ub290 \uc804\uc7c1 \uc774\ud6c4\ub85c \ucd94\uc815\ub418\ub294\uac00?", "paragraph": "\uc5d4\ud2b8\uc5d0\ub294 \ubc18\ub824\uac00 \ub418\ub294 \uc5d4\ud2b8 \ubd80\uc778(Entwife)\uc774 \uc788\uc5c8\ub2e4. \ud0dc\uace0\uc5d0\ub294 \ubaa8\ub450 \uc0dd\ud65c \uc5d4\ud2b8\uc640\uc758 \uc0ac\uc774\uc5d0 \uc544\uc774\ub97c \uc774\ub8e8\uc5c8\uc9c0\ub9cc, \uc810\ucc28 \ub9c8\uc74c\uc5d0 \uac78\uce58\ub294 \uac83\uc774 \ucc28\uc774\uac00 \ub098 \ub098\ubb34 \ub530\ub85c \ub530\ub85c \uc0b4\uac8c \ub418\uc5c8\ub2e4. \uc5d4\ud2b8\ub4e4\uc774 \ub300\ubaa9\uc744 \uc0ac\ub791\ud574, \ub098\ubb34\ub4e4\uc5d0 \ub9d0\uc744 \uac78\uc5b4 \uc232\uc744 \ubc29\ud669 \uc788\uc5b4 \uac77\ub294 \uac83\uc744 \uc88b\uc544\ud55c \uac83\uc5d0 \ube44\ud574, \uc5d4\ud2b8 \ubd80\uc778\ub4e4\uc740 \uc232\uc758 \ubc16\uc758 \uc138\uacc4\uc758 \uac83\ubcf4\ub2e4 \uc791\uc740 \ud480\uaf43\uc774\ub098 \uc18c\ub4dd\uc744 \uac00\uc838\uc624\ub294 \uc2dd\ubb3c\uc5d0 \uad00\uc2ec\uc744 \uac00\uc838 \uadf8\uac83\uc744 \uc9c8\uc11c \uc138\uc6cc \uae30\ub974\ub294 \uac83\uc744 \ubc14\ub7ac\uae30 \ub54c\ubb38\uc774\ub2e4. \ubd81\ucabd\uc5d0\uc11c\uc758 \uc704\ud611\uc774 \ub2e4\uac00\uc624\uba74 \uc5d4\ud2b8 \ubd80\uc778\ub4e4\uc740 \ud310\uace0\ub8ec\uc758 \uc232\uc744 \ub098\uc640 \ub3d9\ucabd\uc744 \ud5a5\ud574, \ub2e4\uc0c9\uc758 \uac15\uc744 \uac74\ub108 \ub300\uc548\uc758 \uc81c3\uc2dc\ub300\uc5d0\uc11c\ub294 \uac08\uc0c9\uc758 \ub098\ub77c\ub85c \ubd88\ub9ac\ub294 \uadfc\ucc98\uc5d0 \uc0c8\ub86d\uac8c \ub730\uc744 \ub9cc\ub4e4\uc5b4 \ubc2d\uc744 \uacbd\uc791\ud574 \uace1\ubb3c\uc774\ub098 \uacfc\uc77c\uc744 \uc7ac\ubc30\ud574, \ud3c9\ud654\ub85c\uc6b4 \uc2dc\ub300\uc5d0\ub294 \uc778\uac04\ub4e4\uc5d0\uac8c\ub3c4 \uadf8 \ubc29\ubc95\uc744 \uac00\ub974\uce5c \ub4ef\ud558\ub2e4. \uadf8\ub7ec\ub098, \ud6c4\uc5d0 \uc5ec\uc790\ub4e4\uc774 \uc0dd\ud65c\uc744 \uc601\uc704\ud558\ub294 \uc9c0\uc5ed\uc774 \uc804\uc7c1(\uc9c0\ub9ac\uc801\uc778 \uc870\uac74\uc73c\ub85c\ubd80\ud130 \uc81c2\uc2dc\ub300\uc758 \ub2e4\uace0\ub974\ub77c\ub4dc\uc758 \uc2f8\uc6c0\uc774\ub77c\uace0 \uc0dd\uac01\ud560 \uc218 \uc788\ub2e4)\uc5d0 \ub9d0\ub824 \ub4e4\uc5b4\uac00 \ub730\uc740 \ud669\ud3d0\ud574 \uc5d4\ud2b8 \ubd80\uc778\uc758 \ud589\ubc29\ub3c4 \ubaa8\ub974\uac8c \ub418\uc5b4 \ubc84\ub838\ub2e4. \uadf8 \ub54c\ubb38\uc5d0 \uc5d4\ud2b8\ub294 \uc99d\uac00\ud558\uc9c0 \uc54a\uac8c \ub418\uc5c8\ub2e4.", "answer": "\ub2e4\uace0\ub974\ub77c\ub4dc\uc758 \uc2f8\uc6c0", "sentence": "\uadf8\ub7ec\ub098, \ud6c4\uc5d0 \uc5ec\uc790\ub4e4\uc774 \uc0dd\ud65c\uc744 \uc601\uc704\ud558\ub294 \uc9c0\uc5ed\uc774 \uc804\uc7c1(\uc9c0\ub9ac\uc801\uc778 \uc870\uac74\uc73c\ub85c\ubd80\ud130 \uc81c2\uc2dc\ub300\uc758 \ub2e4\uace0\ub974\ub77c\ub4dc\uc758 \uc2f8\uc6c0 \uc774\ub77c\uace0 \uc0dd\uac01\ud560 \uc218 \uc788\ub2e4)", "paragraph_sentence": "\uc5d4\ud2b8\uc5d0\ub294 \ubc18\ub824\uac00 \ub418\ub294 \uc5d4\ud2b8 \ubd80\uc778(Entwife)\uc774 \uc788\uc5c8\ub2e4. \ud0dc\uace0\uc5d0\ub294 \ubaa8\ub450 \uc0dd\ud65c \uc5d4\ud2b8\uc640\uc758 \uc0ac\uc774\uc5d0 \uc544\uc774\ub97c \uc774\ub8e8\uc5c8\uc9c0\ub9cc, \uc810\ucc28 \ub9c8\uc74c\uc5d0 \uac78\uce58\ub294 \uac83\uc774 \ucc28\uc774\uac00 \ub098 \ub098\ubb34 \ub530\ub85c \ub530\ub85c \uc0b4\uac8c \ub418\uc5c8\ub2e4. \uc5d4\ud2b8\ub4e4\uc774 \ub300\ubaa9\uc744 \uc0ac\ub791\ud574, \ub098\ubb34\ub4e4\uc5d0 \ub9d0\uc744 \uac78\uc5b4 \uc232\uc744 \ubc29\ud669 \uc788\uc5b4 \uac77\ub294 \uac83\uc744 \uc88b\uc544\ud55c \uac83\uc5d0 \ube44\ud574, \uc5d4\ud2b8 \ubd80\uc778\ub4e4\uc740 \uc232\uc758 \ubc16\uc758 \uc138\uacc4\uc758 \uac83\ubcf4\ub2e4 \uc791\uc740 \ud480\uaf43\uc774\ub098 \uc18c\ub4dd\uc744 \uac00\uc838\uc624\ub294 \uc2dd\ubb3c\uc5d0 \uad00\uc2ec\uc744 \uac00\uc838 \uadf8\uac83\uc744 \uc9c8\uc11c \uc138\uc6cc \uae30\ub974\ub294 \uac83\uc744 \ubc14\ub7ac\uae30 \ub54c\ubb38\uc774\ub2e4. \ubd81\ucabd\uc5d0\uc11c\uc758 \uc704\ud611\uc774 \ub2e4\uac00\uc624\uba74 \uc5d4\ud2b8 \ubd80\uc778\ub4e4\uc740 \ud310\uace0\ub8ec\uc758 \uc232\uc744 \ub098\uc640 \ub3d9\ucabd\uc744 \ud5a5\ud574, \ub2e4\uc0c9\uc758 \uac15\uc744 \uac74\ub108 \ub300\uc548\uc758 \uc81c3\uc2dc\ub300\uc5d0\uc11c\ub294 \uac08\uc0c9\uc758 \ub098\ub77c\ub85c \ubd88\ub9ac\ub294 \uadfc\ucc98\uc5d0 \uc0c8\ub86d\uac8c \ub730\uc744 \ub9cc\ub4e4\uc5b4 \ubc2d\uc744 \uacbd\uc791\ud574 \uace1\ubb3c\uc774\ub098 \uacfc\uc77c\uc744 \uc7ac\ubc30\ud574, \ud3c9\ud654\ub85c\uc6b4 \uc2dc\ub300\uc5d0\ub294 \uc778\uac04\ub4e4\uc5d0\uac8c\ub3c4 \uadf8 \ubc29\ubc95\uc744 \uac00\ub974\uce5c \ub4ef\ud558\ub2e4. \uadf8\ub7ec\ub098, \ud6c4\uc5d0 \uc5ec\uc790\ub4e4\uc774 \uc0dd\ud65c\uc744 \uc601\uc704\ud558\ub294 \uc9c0\uc5ed\uc774 \uc804\uc7c1(\uc9c0\ub9ac\uc801\uc778 \uc870\uac74\uc73c\ub85c\ubd80\ud130 \uc81c2\uc2dc\ub300\uc758 \ub2e4\uace0\ub974\ub77c\ub4dc\uc758 \uc2f8\uc6c0 \uc774\ub77c\uace0 \uc0dd\uac01\ud560 \uc218 \uc788\ub2e4) \uc5d0 \ub9d0\ub824 \ub4e4\uc5b4\uac00 \ub730\uc740 \ud669\ud3d0\ud574 \uc5d4\ud2b8 \ubd80\uc778\uc758 \ud589\ubc29\ub3c4 \ubaa8\ub974\uac8c \ub418\uc5b4 \ubc84\ub838\ub2e4. \uadf8 \ub54c\ubb38\uc5d0 \uc5d4\ud2b8\ub294 \uc99d\uac00\ud558\uc9c0 \uc54a\uac8c \ub418\uc5c8\ub2e4.", "paragraph_answer": "\uc5d4\ud2b8\uc5d0\ub294 \ubc18\ub824\uac00 \ub418\ub294 \uc5d4\ud2b8 \ubd80\uc778(Entwife)\uc774 \uc788\uc5c8\ub2e4. \ud0dc\uace0\uc5d0\ub294 \ubaa8\ub450 \uc0dd\ud65c \uc5d4\ud2b8\uc640\uc758 \uc0ac\uc774\uc5d0 \uc544\uc774\ub97c \uc774\ub8e8\uc5c8\uc9c0\ub9cc, \uc810\ucc28 \ub9c8\uc74c\uc5d0 \uac78\uce58\ub294 \uac83\uc774 \ucc28\uc774\uac00 \ub098 \ub098\ubb34 \ub530\ub85c \ub530\ub85c \uc0b4\uac8c \ub418\uc5c8\ub2e4. \uc5d4\ud2b8\ub4e4\uc774 \ub300\ubaa9\uc744 \uc0ac\ub791\ud574, \ub098\ubb34\ub4e4\uc5d0 \ub9d0\uc744 \uac78\uc5b4 \uc232\uc744 \ubc29\ud669 \uc788\uc5b4 \uac77\ub294 \uac83\uc744 \uc88b\uc544\ud55c \uac83\uc5d0 \ube44\ud574, \uc5d4\ud2b8 \ubd80\uc778\ub4e4\uc740 \uc232\uc758 \ubc16\uc758 \uc138\uacc4\uc758 \uac83\ubcf4\ub2e4 \uc791\uc740 \ud480\uaf43\uc774\ub098 \uc18c\ub4dd\uc744 \uac00\uc838\uc624\ub294 \uc2dd\ubb3c\uc5d0 \uad00\uc2ec\uc744 \uac00\uc838 \uadf8\uac83\uc744 \uc9c8\uc11c \uc138\uc6cc \uae30\ub974\ub294 \uac83\uc744 \ubc14\ub7ac\uae30 \ub54c\ubb38\uc774\ub2e4. \ubd81\ucabd\uc5d0\uc11c\uc758 \uc704\ud611\uc774 \ub2e4\uac00\uc624\uba74 \uc5d4\ud2b8 \ubd80\uc778\ub4e4\uc740 \ud310\uace0\ub8ec\uc758 \uc232\uc744 \ub098\uc640 \ub3d9\ucabd\uc744 \ud5a5\ud574, \ub2e4\uc0c9\uc758 \uac15\uc744 \uac74\ub108 \ub300\uc548\uc758 \uc81c3\uc2dc\ub300\uc5d0\uc11c\ub294 \uac08\uc0c9\uc758 \ub098\ub77c\ub85c \ubd88\ub9ac\ub294 \uadfc\ucc98\uc5d0 \uc0c8\ub86d\uac8c \ub730\uc744 \ub9cc\ub4e4\uc5b4 \ubc2d\uc744 \uacbd\uc791\ud574 \uace1\ubb3c\uc774\ub098 \uacfc\uc77c\uc744 \uc7ac\ubc30\ud574, \ud3c9\ud654\ub85c\uc6b4 \uc2dc\ub300\uc5d0\ub294 \uc778\uac04\ub4e4\uc5d0\uac8c\ub3c4 \uadf8 \ubc29\ubc95\uc744 \uac00\ub974\uce5c \ub4ef\ud558\ub2e4. \uadf8\ub7ec\ub098, \ud6c4\uc5d0 \uc5ec\uc790\ub4e4\uc774 \uc0dd\ud65c\uc744 \uc601\uc704\ud558\ub294 \uc9c0\uc5ed\uc774 \uc804\uc7c1(\uc9c0\ub9ac\uc801\uc778 \uc870\uac74\uc73c\ub85c\ubd80\ud130 \uc81c2\uc2dc\ub300\uc758 \ub2e4\uace0\ub974\ub77c\ub4dc\uc758 \uc2f8\uc6c0 \uc774\ub77c\uace0 \uc0dd\uac01\ud560 \uc218 \uc788\ub2e4)\uc5d0 \ub9d0\ub824 \ub4e4\uc5b4\uac00 \ub730\uc740 \ud669\ud3d0\ud574 \uc5d4\ud2b8 \ubd80\uc778\uc758 \ud589\ubc29\ub3c4 \ubaa8\ub974\uac8c \ub418\uc5b4 \ubc84\ub838\ub2e4. \uadf8 \ub54c\ubb38\uc5d0 \uc5d4\ud2b8\ub294 \uc99d\uac00\ud558\uc9c0 \uc54a\uac8c \ub418\uc5c8\ub2e4.", "sentence_answer": "\uadf8\ub7ec\ub098, \ud6c4\uc5d0 \uc5ec\uc790\ub4e4\uc774 \uc0dd\ud65c\uc744 \uc601\uc704\ud558\ub294 \uc9c0\uc5ed\uc774 \uc804\uc7c1(\uc9c0\ub9ac\uc801\uc778 \uc870\uac74\uc73c\ub85c\ubd80\ud130 \uc81c2\uc2dc\ub300\uc758 \ub2e4\uace0\ub974\ub77c\ub4dc\uc758 \uc2f8\uc6c0 \uc774\ub77c\uace0 \uc0dd\uac01\ud560 \uc218 \uc788\ub2e4)", "paragraph_id": "6466068-1-1", "paragraph_question": "question: \uc5d4\ud2b8\ubd80\uc778\uc758 \ud589\ubc29\uc774 \ubb18\uc5f0\ud574\uc9c4 \uac83\uc740 \uc5b4\ub290 \uc804\uc7c1 \uc774\ud6c4\ub85c \ucd94\uc815\ub418\ub294\uac00?, context: \uc5d4\ud2b8\uc5d0\ub294 \ubc18\ub824\uac00 \ub418\ub294 \uc5d4\ud2b8 \ubd80\uc778(Entwife)\uc774 \uc788\uc5c8\ub2e4. \ud0dc\uace0\uc5d0\ub294 \ubaa8\ub450 \uc0dd\ud65c \uc5d4\ud2b8\uc640\uc758 \uc0ac\uc774\uc5d0 \uc544\uc774\ub97c \uc774\ub8e8\uc5c8\uc9c0\ub9cc, \uc810\ucc28 \ub9c8\uc74c\uc5d0 \uac78\uce58\ub294 \uac83\uc774 \ucc28\uc774\uac00 \ub098 \ub098\ubb34 \ub530\ub85c \ub530\ub85c \uc0b4\uac8c \ub418\uc5c8\ub2e4. \uc5d4\ud2b8\ub4e4\uc774 \ub300\ubaa9\uc744 \uc0ac\ub791\ud574, \ub098\ubb34\ub4e4\uc5d0 \ub9d0\uc744 \uac78\uc5b4 \uc232\uc744 \ubc29\ud669 \uc788\uc5b4 \uac77\ub294 \uac83\uc744 \uc88b\uc544\ud55c \uac83\uc5d0 \ube44\ud574, \uc5d4\ud2b8 \ubd80\uc778\ub4e4\uc740 \uc232\uc758 \ubc16\uc758 \uc138\uacc4\uc758 \uac83\ubcf4\ub2e4 \uc791\uc740 \ud480\uaf43\uc774\ub098 \uc18c\ub4dd\uc744 \uac00\uc838\uc624\ub294 \uc2dd\ubb3c\uc5d0 \uad00\uc2ec\uc744 \uac00\uc838 \uadf8\uac83\uc744 \uc9c8\uc11c \uc138\uc6cc \uae30\ub974\ub294 \uac83\uc744 \ubc14\ub7ac\uae30 \ub54c\ubb38\uc774\ub2e4. \ubd81\ucabd\uc5d0\uc11c\uc758 \uc704\ud611\uc774 \ub2e4\uac00\uc624\uba74 \uc5d4\ud2b8 \ubd80\uc778\ub4e4\uc740 \ud310\uace0\ub8ec\uc758 \uc232\uc744 \ub098\uc640 \ub3d9\ucabd\uc744 \ud5a5\ud574, \ub2e4\uc0c9\uc758 \uac15\uc744 \uac74\ub108 \ub300\uc548\uc758 \uc81c3\uc2dc\ub300\uc5d0\uc11c\ub294 \uac08\uc0c9\uc758 \ub098\ub77c\ub85c \ubd88\ub9ac\ub294 \uadfc\ucc98\uc5d0 \uc0c8\ub86d\uac8c \ub730\uc744 \ub9cc\ub4e4\uc5b4 \ubc2d\uc744 \uacbd\uc791\ud574 \uace1\ubb3c\uc774\ub098 \uacfc\uc77c\uc744 \uc7ac\ubc30\ud574, \ud3c9\ud654\ub85c\uc6b4 \uc2dc\ub300\uc5d0\ub294 \uc778\uac04\ub4e4\uc5d0\uac8c\ub3c4 \uadf8 \ubc29\ubc95\uc744 \uac00\ub974\uce5c \ub4ef\ud558\ub2e4. \uadf8\ub7ec\ub098, \ud6c4\uc5d0 \uc5ec\uc790\ub4e4\uc774 \uc0dd\ud65c\uc744 \uc601\uc704\ud558\ub294 \uc9c0\uc5ed\uc774 \uc804\uc7c1(\uc9c0\ub9ac\uc801\uc778 \uc870\uac74\uc73c\ub85c\ubd80\ud130 \uc81c2\uc2dc\ub300\uc758 \ub2e4\uace0\ub974\ub77c\ub4dc\uc758 \uc2f8\uc6c0\uc774\ub77c\uace0 \uc0dd\uac01\ud560 \uc218 \uc788\ub2e4)\uc5d0 \ub9d0\ub824 \ub4e4\uc5b4\uac00 \ub730\uc740 \ud669\ud3d0\ud574 \uc5d4\ud2b8 \ubd80\uc778\uc758 \ud589\ubc29\ub3c4 \ubaa8\ub974\uac8c \ub418\uc5b4 \ubc84\ub838\ub2e4. \uadf8 \ub54c\ubb38\uc5d0 \uc5d4\ud2b8\ub294 \uc99d\uac00\ud558\uc9c0 \uc54a\uac8c \ub418\uc5c8\ub2e4."} {"question": "\ub51c\ub7f0\uc774 \uc0c8\ub86d\uac8c \ucc3d\uc870\ud558\uace0 \ubc1c\uc804\uc2dc\ud0a8 \uc74c\uc545\uc801 \uc601\uc5ed\uc740?", "paragraph": "1965\ub144 \ub274\ud3ec\ud2b8 \ud3ec\ud06c \ud398\uc2a4\ud2f0\ubc8c(The Newport Folk Festival)\uc5d0\uc11c \ub85d \ubc34\ub4dc \ud3f4 \ubc84\ud130\ud544\ub4dc \ube14\ub8e8\uc2a4 \ubc34\ub4dc\uc640 \ud0a4\ubcf4\ub514\uc2a4\ud2b8 \uc54c \ucfe0\ud37c\ub97c \ub300\ub3d9\ud558\uace0 \uc77c\ub809\ud2b8\ub9ad \uc0ac\uc6b4\ub4dc\ub97c \uc120\ubcf4\uc778 \uc0ac\uac74\uc740 \uc218\ub9ce\uc740 \ub300\uc911\uacfc \ud3ec\ud06c \ud32c\ub4e4\uc758 \uc57c\uc720\uc640 \ubc18\ubc1c\uc744 \ubd88\ub7ec \uc77c\uc73c\ud0a4\uc9c0\ub9cc, \ub51c\ub7f0\uc740 \uc790\uc2e0\uc758 \uc74c\uc545\uc801 \uc804\ud658\uc744 \ud655\uace0\ud558\uac8c \uc774\uc5b4\uac14\uc73c\uba70 \uc774\ub97c \ud1b5\ud574 \ud3ec\ud06c \ub85d\uc774\ub77c\ub294 \uc0c8\ub85c\uc6b4 \uc74c\uc545\uc801 \uc601\uc5ed\uc744 \ucc3d\uc870\ud558\uace0 \ubc1c\uc804\uc2dc\ucf30\ub2e4. \uc774 \uc2dc\uae30\uc758 \uc74c\ubc18\uc73c\ub85c\ub294 \u300aBringing It All Back Home\u300b(1965), \u300aHighway 61 Revisited\u300b(1965), \u300aBlonde On Blonde\u300b(1966)\uac00 \uc788\ub2e4. 1965\ub144 11\uc6d4 \ub51c\ub7f0\uc740 25\uc138\uc758 \uc804\uc9c1 \ubaa8\ub378 \uc0ac\ub77c \ub77c\uc6b4\uc988\uc640 \uadf9\ube44\ub9ac\uc5d0 \ud63c\uc778\uc2dd\uc744 \uce58\ub974\uace0 \uc774\ub4ec\ud574 1966\ub144 \ubd04 \ud638\uc8fc\uc640 \uc720\ub7fd\uc744 \uc911\uc2ec\uc73c\ub85c \ud22c\uc5b4\ub97c \uc2dc\uc791\ud55c\ub2e4. \ub51c\ub7f0\uc758 \uc5f0\uc8fc \uc5ec\ud589\uc5d0 \ub3d9\ucc38\ud558\uac8c \ub41c \uc0c8\ub85c\uc6b4 \ubc31\uc5c5 \ubc34\ub4dc\uc758 \uc774\ub984\uc740 \ub354 \ud639\uc2a4(\ub354 \ubc34\ub4dc\uc758 \uc804\uc2e0). \uc804\ud6c4\ubc18\uc73c\ub85c \ub098\ub25c \uacf5\uc5f0 \ud504\ub85c\uadf8\ub7a8\uc5d0\uc11c 1\ubd80\ub294 \ub51c\ub7f0 \ud640\ub85c \ud50c\ub808\uc774\ud558\ub294 \uc5b4\ucfe0\uc2a4\ud2f1 \uc2a4\ud14c\uc774\uc9c0, 2\ubd80\ub294 \ub354 \ud639\uc2a4\ub97c \ub300\ub3d9\ud55c \ub85c\ud070\ub864 \uc2a4\ud14c\uc774\uc9c0\ub85c \uc774\ub8e8\uc5b4\uc838 \uc788\uc5c8\ub2e4.", "answer": "\ud3ec\ud06c \ub85d", "sentence": "1965\ub144 \ub274\ud3ec\ud2b8 \ud3ec\ud06c \ud398\uc2a4\ud2f0\ubc8c(The Newport Folk Festival)\uc5d0\uc11c \ub85d \ubc34\ub4dc \ud3f4 \ubc84\ud130\ud544\ub4dc \ube14\ub8e8\uc2a4 \ubc34\ub4dc\uc640 \ud0a4\ubcf4\ub514\uc2a4\ud2b8 \uc54c \ucfe0\ud37c\ub97c \ub300\ub3d9\ud558\uace0 \uc77c\ub809\ud2b8\ub9ad \uc0ac\uc6b4\ub4dc\ub97c \uc120\ubcf4\uc778 \uc0ac\uac74\uc740 \uc218\ub9ce\uc740 \ub300\uc911\uacfc \ud3ec\ud06c \ud32c\ub4e4\uc758 \uc57c\uc720\uc640 \ubc18\ubc1c\uc744 \ubd88\ub7ec \uc77c\uc73c\ud0a4\uc9c0\ub9cc, \ub51c\ub7f0\uc740 \uc790\uc2e0\uc758 \uc74c\uc545\uc801 \uc804\ud658\uc744 \ud655\uace0\ud558\uac8c \uc774\uc5b4\uac14\uc73c\uba70 \uc774\ub97c \ud1b5\ud574 \ud3ec\ud06c \ub85d \uc774\ub77c\ub294 \uc0c8\ub85c\uc6b4 \uc74c\uc545\uc801 \uc601\uc5ed\uc744 \ucc3d\uc870\ud558\uace0 \ubc1c\uc804\uc2dc\ucf30\ub2e4.", "paragraph_sentence": " 1965\ub144 \ub274\ud3ec\ud2b8 \ud3ec\ud06c \ud398\uc2a4\ud2f0\ubc8c(The Newport Folk Festival)\uc5d0\uc11c \ub85d \ubc34\ub4dc \ud3f4 \ubc84\ud130\ud544\ub4dc \ube14\ub8e8\uc2a4 \ubc34\ub4dc\uc640 \ud0a4\ubcf4\ub514\uc2a4\ud2b8 \uc54c \ucfe0\ud37c\ub97c \ub300\ub3d9\ud558\uace0 \uc77c\ub809\ud2b8\ub9ad \uc0ac\uc6b4\ub4dc\ub97c \uc120\ubcf4\uc778 \uc0ac\uac74\uc740 \uc218\ub9ce\uc740 \ub300\uc911\uacfc \ud3ec\ud06c \ud32c\ub4e4\uc758 \uc57c\uc720\uc640 \ubc18\ubc1c\uc744 \ubd88\ub7ec \uc77c\uc73c\ud0a4\uc9c0\ub9cc, \ub51c\ub7f0\uc740 \uc790\uc2e0\uc758 \uc74c\uc545\uc801 \uc804\ud658\uc744 \ud655\uace0\ud558\uac8c \uc774\uc5b4\uac14\uc73c\uba70 \uc774\ub97c \ud1b5\ud574 \ud3ec\ud06c \ub85d \uc774\ub77c\ub294 \uc0c8\ub85c\uc6b4 \uc74c\uc545\uc801 \uc601\uc5ed\uc744 \ucc3d\uc870\ud558\uace0 \ubc1c\uc804\uc2dc\ucf30\ub2e4. \uc774 \uc2dc\uae30\uc758 \uc74c\ubc18\uc73c\ub85c\ub294 \u300aBringing It All Back Home\u300b(1965), \u300aHighway 61 Revisited\u300b(1965), \u300aBlonde On Blonde\u300b(1966)\uac00 \uc788\ub2e4. 1965\ub144 11\uc6d4 \ub51c\ub7f0\uc740 25\uc138\uc758 \uc804\uc9c1 \ubaa8\ub378 \uc0ac\ub77c \ub77c\uc6b4\uc988\uc640 \uadf9\ube44\ub9ac\uc5d0 \ud63c\uc778\uc2dd\uc744 \uce58\ub974\uace0 \uc774\ub4ec\ud574 1966\ub144 \ubd04 \ud638\uc8fc\uc640 \uc720\ub7fd\uc744 \uc911\uc2ec\uc73c\ub85c \ud22c\uc5b4\ub97c \uc2dc\uc791\ud55c\ub2e4. \ub51c\ub7f0\uc758 \uc5f0\uc8fc \uc5ec\ud589\uc5d0 \ub3d9\ucc38\ud558\uac8c \ub41c \uc0c8\ub85c\uc6b4 \ubc31\uc5c5 \ubc34\ub4dc\uc758 \uc774\ub984\uc740 \ub354 \ud639\uc2a4(\ub354 \ubc34\ub4dc\uc758 \uc804\uc2e0). \uc804\ud6c4\ubc18\uc73c\ub85c \ub098\ub25c \uacf5\uc5f0 \ud504\ub85c\uadf8\ub7a8\uc5d0\uc11c 1\ubd80\ub294 \ub51c\ub7f0 \ud640\ub85c \ud50c\ub808\uc774\ud558\ub294 \uc5b4\ucfe0\uc2a4\ud2f1 \uc2a4\ud14c\uc774\uc9c0, 2\ubd80\ub294 \ub354 \ud639\uc2a4\ub97c \ub300\ub3d9\ud55c \ub85c\ud070\ub864 \uc2a4\ud14c\uc774\uc9c0\ub85c \uc774\ub8e8\uc5b4\uc838 \uc788\uc5c8\ub2e4.", "paragraph_answer": "1965\ub144 \ub274\ud3ec\ud2b8 \ud3ec\ud06c \ud398\uc2a4\ud2f0\ubc8c(The Newport Folk Festival)\uc5d0\uc11c \ub85d \ubc34\ub4dc \ud3f4 \ubc84\ud130\ud544\ub4dc \ube14\ub8e8\uc2a4 \ubc34\ub4dc\uc640 \ud0a4\ubcf4\ub514\uc2a4\ud2b8 \uc54c \ucfe0\ud37c\ub97c \ub300\ub3d9\ud558\uace0 \uc77c\ub809\ud2b8\ub9ad \uc0ac\uc6b4\ub4dc\ub97c \uc120\ubcf4\uc778 \uc0ac\uac74\uc740 \uc218\ub9ce\uc740 \ub300\uc911\uacfc \ud3ec\ud06c \ud32c\ub4e4\uc758 \uc57c\uc720\uc640 \ubc18\ubc1c\uc744 \ubd88\ub7ec \uc77c\uc73c\ud0a4\uc9c0\ub9cc, \ub51c\ub7f0\uc740 \uc790\uc2e0\uc758 \uc74c\uc545\uc801 \uc804\ud658\uc744 \ud655\uace0\ud558\uac8c \uc774\uc5b4\uac14\uc73c\uba70 \uc774\ub97c \ud1b5\ud574 \ud3ec\ud06c \ub85d \uc774\ub77c\ub294 \uc0c8\ub85c\uc6b4 \uc74c\uc545\uc801 \uc601\uc5ed\uc744 \ucc3d\uc870\ud558\uace0 \ubc1c\uc804\uc2dc\ucf30\ub2e4. \uc774 \uc2dc\uae30\uc758 \uc74c\ubc18\uc73c\ub85c\ub294 \u300aBringing It All Back Home\u300b(1965), \u300aHighway 61 Revisited\u300b(1965), \u300aBlonde On Blonde\u300b(1966)\uac00 \uc788\ub2e4. 1965\ub144 11\uc6d4 \ub51c\ub7f0\uc740 25\uc138\uc758 \uc804\uc9c1 \ubaa8\ub378 \uc0ac\ub77c \ub77c\uc6b4\uc988\uc640 \uadf9\ube44\ub9ac\uc5d0 \ud63c\uc778\uc2dd\uc744 \uce58\ub974\uace0 \uc774\ub4ec\ud574 1966\ub144 \ubd04 \ud638\uc8fc\uc640 \uc720\ub7fd\uc744 \uc911\uc2ec\uc73c\ub85c \ud22c\uc5b4\ub97c \uc2dc\uc791\ud55c\ub2e4. \ub51c\ub7f0\uc758 \uc5f0\uc8fc \uc5ec\ud589\uc5d0 \ub3d9\ucc38\ud558\uac8c \ub41c \uc0c8\ub85c\uc6b4 \ubc31\uc5c5 \ubc34\ub4dc\uc758 \uc774\ub984\uc740 \ub354 \ud639\uc2a4(\ub354 \ubc34\ub4dc\uc758 \uc804\uc2e0). \uc804\ud6c4\ubc18\uc73c\ub85c \ub098\ub25c \uacf5\uc5f0 \ud504\ub85c\uadf8\ub7a8\uc5d0\uc11c 1\ubd80\ub294 \ub51c\ub7f0 \ud640\ub85c \ud50c\ub808\uc774\ud558\ub294 \uc5b4\ucfe0\uc2a4\ud2f1 \uc2a4\ud14c\uc774\uc9c0, 2\ubd80\ub294 \ub354 \ud639\uc2a4\ub97c \ub300\ub3d9\ud55c \ub85c\ud070\ub864 \uc2a4\ud14c\uc774\uc9c0\ub85c \uc774\ub8e8\uc5b4\uc838 \uc788\uc5c8\ub2e4.", "sentence_answer": "1965\ub144 \ub274\ud3ec\ud2b8 \ud3ec\ud06c \ud398\uc2a4\ud2f0\ubc8c(The Newport Folk Festival)\uc5d0\uc11c \ub85d \ubc34\ub4dc \ud3f4 \ubc84\ud130\ud544\ub4dc \ube14\ub8e8\uc2a4 \ubc34\ub4dc\uc640 \ud0a4\ubcf4\ub514\uc2a4\ud2b8 \uc54c \ucfe0\ud37c\ub97c \ub300\ub3d9\ud558\uace0 \uc77c\ub809\ud2b8\ub9ad \uc0ac\uc6b4\ub4dc\ub97c \uc120\ubcf4\uc778 \uc0ac\uac74\uc740 \uc218\ub9ce\uc740 \ub300\uc911\uacfc \ud3ec\ud06c \ud32c\ub4e4\uc758 \uc57c\uc720\uc640 \ubc18\ubc1c\uc744 \ubd88\ub7ec \uc77c\uc73c\ud0a4\uc9c0\ub9cc, \ub51c\ub7f0\uc740 \uc790\uc2e0\uc758 \uc74c\uc545\uc801 \uc804\ud658\uc744 \ud655\uace0\ud558\uac8c \uc774\uc5b4\uac14\uc73c\uba70 \uc774\ub97c \ud1b5\ud574 \ud3ec\ud06c \ub85d \uc774\ub77c\ub294 \uc0c8\ub85c\uc6b4 \uc74c\uc545\uc801 \uc601\uc5ed\uc744 \ucc3d\uc870\ud558\uace0 \ubc1c\uc804\uc2dc\ucf30\ub2e4.", "paragraph_id": "6657114-6-0", "paragraph_question": "question: \ub51c\ub7f0\uc774 \uc0c8\ub86d\uac8c \ucc3d\uc870\ud558\uace0 \ubc1c\uc804\uc2dc\ud0a8 \uc74c\uc545\uc801 \uc601\uc5ed\uc740?, context: 1965\ub144 \ub274\ud3ec\ud2b8 \ud3ec\ud06c \ud398\uc2a4\ud2f0\ubc8c(The Newport Folk Festival)\uc5d0\uc11c \ub85d \ubc34\ub4dc \ud3f4 \ubc84\ud130\ud544\ub4dc \ube14\ub8e8\uc2a4 \ubc34\ub4dc\uc640 \ud0a4\ubcf4\ub514\uc2a4\ud2b8 \uc54c \ucfe0\ud37c\ub97c \ub300\ub3d9\ud558\uace0 \uc77c\ub809\ud2b8\ub9ad \uc0ac\uc6b4\ub4dc\ub97c \uc120\ubcf4\uc778 \uc0ac\uac74\uc740 \uc218\ub9ce\uc740 \ub300\uc911\uacfc \ud3ec\ud06c \ud32c\ub4e4\uc758 \uc57c\uc720\uc640 \ubc18\ubc1c\uc744 \ubd88\ub7ec \uc77c\uc73c\ud0a4\uc9c0\ub9cc, \ub51c\ub7f0\uc740 \uc790\uc2e0\uc758 \uc74c\uc545\uc801 \uc804\ud658\uc744 \ud655\uace0\ud558\uac8c \uc774\uc5b4\uac14\uc73c\uba70 \uc774\ub97c \ud1b5\ud574 \ud3ec\ud06c \ub85d\uc774\ub77c\ub294 \uc0c8\ub85c\uc6b4 \uc74c\uc545\uc801 \uc601\uc5ed\uc744 \ucc3d\uc870\ud558\uace0 \ubc1c\uc804\uc2dc\ucf30\ub2e4. \uc774 \uc2dc\uae30\uc758 \uc74c\ubc18\uc73c\ub85c\ub294 \u300aBringing It All Back Home\u300b(1965), \u300aHighway 61 Revisited\u300b(1965), \u300aBlonde On Blonde\u300b(1966)\uac00 \uc788\ub2e4. 1965\ub144 11\uc6d4 \ub51c\ub7f0\uc740 25\uc138\uc758 \uc804\uc9c1 \ubaa8\ub378 \uc0ac\ub77c \ub77c\uc6b4\uc988\uc640 \uadf9\ube44\ub9ac\uc5d0 \ud63c\uc778\uc2dd\uc744 \uce58\ub974\uace0 \uc774\ub4ec\ud574 1966\ub144 \ubd04 \ud638\uc8fc\uc640 \uc720\ub7fd\uc744 \uc911\uc2ec\uc73c\ub85c \ud22c\uc5b4\ub97c \uc2dc\uc791\ud55c\ub2e4. \ub51c\ub7f0\uc758 \uc5f0\uc8fc \uc5ec\ud589\uc5d0 \ub3d9\ucc38\ud558\uac8c \ub41c \uc0c8\ub85c\uc6b4 \ubc31\uc5c5 \ubc34\ub4dc\uc758 \uc774\ub984\uc740 \ub354 \ud639\uc2a4(\ub354 \ubc34\ub4dc\uc758 \uc804\uc2e0). \uc804\ud6c4\ubc18\uc73c\ub85c \ub098\ub25c \uacf5\uc5f0 \ud504\ub85c\uadf8\ub7a8\uc5d0\uc11c 1\ubd80\ub294 \ub51c\ub7f0 \ud640\ub85c \ud50c\ub808\uc774\ud558\ub294 \uc5b4\ucfe0\uc2a4\ud2f1 \uc2a4\ud14c\uc774\uc9c0, 2\ubd80\ub294 \ub354 \ud639\uc2a4\ub97c \ub300\ub3d9\ud55c \ub85c\ud070\ub864 \uc2a4\ud14c\uc774\uc9c0\ub85c \uc774\ub8e8\uc5b4\uc838 \uc788\uc5c8\ub2e4."} {"question": "\uc77c\ubc18\uc0c1\ub300\ub860\uc758 \uac1c\ub860\uc5d0\uc11c \uc2e4\ud5d8\uc744 \ud1b5\ud574 \uc0c1\ub300\uc131\uc774\ub860\uc774 \ub9de\uc74c\uc744 \ud655\uc778\ud55c \ub0a0\uc9dc\ub294 \uc5b8\uc81c\uc778\uac00?", "paragraph": "\uadf8 \ub4a4\ub85c \uace0\uc548\ub41c \uc77c\ubc18\uc0c1\ub300\ub860 \uc2dc\ud5d8 \uc2e4\ud5d8\uc73c\ub85c\ub294 \uc0e4\ud53c\ub85c \ud6a8\uacfc \ub610\ub294 \ube5b\uc758 \uc911\ub825\uc2dc\uac04\uc9c0\uc5f0\uc744 \uc815\ubc00\ud788 \uce21\uc815\ud558\ub294 \uac83\uc774 \uc788\ub2e4. \uac00\uc7a5 \ucd5c\uadfc\uc5d0 \uc774\ub8e8\uc5b4\uc9c4 \ud574\ub2f9 \uc2e4\ud5d8\uc740 2002\ub144\uc5d0 \uce74\uc2dc\ub2c8 \uc6b0\uc8fc\ud0d0\uc0ac\uc120\uc774 \uc218\ud589\ud55c \uc2e4\ud5d8\uc774\ub2e4. \ub610\ub2e4\ub978 \uc2e4\ud5d8\uc73c\ub85c\ub294 \uacf5\uac04 \uc18d\uc5d0\uc11c \uc790\uc774\ub85c\uc2a4\ucf54\ud504\uc758 \ud589\ub3d9\uc758 \uc77c\ubc18\uc0c1\ub300\ub860\uc801 \uc608\uce21\uc744 \uac80\uc99d\ud558\ub294 \uac83\uc774 \uc788\ub2e4. \ub2ec\uc5d0 \uc124\uce58\ub41c \uc778\uacf5\uc704\uc131 \uc6d4\uba74 \ub808\uc774\uc800 \uc7a5\uce58\uac00 \uadf8 \uc608\uce21 \uc911 \ud558\ub098\uc778 \uce21\uc9c0 \uc138\ucc28\ub97c \uc2e4\ud5d8\ud558\uc600\ub2e4. \ub610\ub2e4\ub978 \uc2e4\ud5d8\uc73c\ub85c\ub294 \ud68c\uc804\ud558\ub294 \uc9c8\ub7c9\uacfc \uad00\ub828\ub41c \ud2c0 \ub04c\ub9bc \ud604\uc0c1\uc5d0 \uad00\ud55c \uac83\uc774 \uc788\ub2e4. 2004\ub144 \ubc1c\uc0ac\ub41c \uc911\ub825\ud0d0\uc0ac\uc120 B\ud638 \uc778\uacf5\uc704\uc131\uc740 \ud2b9\uc9c0\ud6a8\uacfc\uc640 \ud2c0 \ub04c\ub9bc \ud6a8\uacfc\ub97c \ubaa8\ub450 \uc2e4\ud5d8\ud574 \ubcf4\uc558\uc73c\uba70, 2008\ub144 12\uc6d4 \uae30\uc900\uc73c\ub85c \uac01\uac01 0.5%\uc640 15% \uc624\ucc28 \ub0b4\uc5d0\uc11c \uc0c1\ub300\uc131\uc774\ub860\uc774 \ub9de\uc74c\uc744 \ud655\uc778\ud558\uc600\ub2e4.", "answer": "2008\ub144 12\uc6d4", "sentence": "2004\ub144 \ubc1c\uc0ac\ub41c \uc911\ub825\ud0d0\uc0ac\uc120 B\ud638 \uc778\uacf5\uc704\uc131\uc740 \ud2b9\uc9c0\ud6a8\uacfc\uc640 \ud2c0 \ub04c\ub9bc \ud6a8\uacfc\ub97c \ubaa8\ub450 \uc2e4\ud5d8\ud574 \ubcf4\uc558\uc73c\uba70, 2008\ub144 12\uc6d4 \uae30\uc900\uc73c\ub85c \uac01\uac01 0.5%\uc640 15% \uc624\ucc28 \ub0b4\uc5d0\uc11c \uc0c1\ub300\uc131\uc774\ub860\uc774 \ub9de\uc74c\uc744 \ud655\uc778\ud558\uc600\ub2e4.", "paragraph_sentence": "\uadf8 \ub4a4\ub85c \uace0\uc548\ub41c \uc77c\ubc18\uc0c1\ub300\ub860 \uc2dc\ud5d8 \uc2e4\ud5d8\uc73c\ub85c\ub294 \uc0e4\ud53c\ub85c \ud6a8\uacfc \ub610\ub294 \ube5b\uc758 \uc911\ub825\uc2dc\uac04\uc9c0\uc5f0\uc744 \uc815\ubc00\ud788 \uce21\uc815\ud558\ub294 \uac83\uc774 \uc788\ub2e4. \uac00\uc7a5 \ucd5c\uadfc\uc5d0 \uc774\ub8e8\uc5b4\uc9c4 \ud574\ub2f9 \uc2e4\ud5d8\uc740 2002\ub144\uc5d0 \uce74\uc2dc\ub2c8 \uc6b0\uc8fc\ud0d0\uc0ac\uc120\uc774 \uc218\ud589\ud55c \uc2e4\ud5d8\uc774\ub2e4. \ub610\ub2e4\ub978 \uc2e4\ud5d8\uc73c\ub85c\ub294 \uacf5\uac04 \uc18d\uc5d0\uc11c \uc790\uc774\ub85c\uc2a4\ucf54\ud504\uc758 \ud589\ub3d9\uc758 \uc77c\ubc18\uc0c1\ub300\ub860\uc801 \uc608\uce21\uc744 \uac80\uc99d\ud558\ub294 \uac83\uc774 \uc788\ub2e4. \ub2ec\uc5d0 \uc124\uce58\ub41c \uc778\uacf5\uc704\uc131 \uc6d4\uba74 \ub808\uc774\uc800 \uc7a5\uce58\uac00 \uadf8 \uc608\uce21 \uc911 \ud558\ub098\uc778 \uce21\uc9c0 \uc138\ucc28\ub97c \uc2e4\ud5d8\ud558\uc600\ub2e4. \ub610\ub2e4\ub978 \uc2e4\ud5d8\uc73c\ub85c\ub294 \ud68c\uc804\ud558\ub294 \uc9c8\ub7c9\uacfc \uad00\ub828\ub41c \ud2c0 \ub04c\ub9bc \ud604\uc0c1\uc5d0 \uad00\ud55c \uac83\uc774 \uc788\ub2e4. 2004\ub144 \ubc1c\uc0ac\ub41c \uc911\ub825\ud0d0\uc0ac\uc120 B\ud638 \uc778\uacf5\uc704\uc131\uc740 \ud2b9\uc9c0\ud6a8\uacfc\uc640 \ud2c0 \ub04c\ub9bc \ud6a8\uacfc\ub97c \ubaa8\ub450 \uc2e4\ud5d8\ud574 \ubcf4\uc558\uc73c\uba70, 2008\ub144 12\uc6d4 \uae30\uc900\uc73c\ub85c \uac01\uac01 0.5%\uc640 15% \uc624\ucc28 \ub0b4\uc5d0\uc11c \uc0c1\ub300\uc131\uc774\ub860\uc774 \ub9de\uc74c\uc744 \ud655\uc778\ud558\uc600\ub2e4. ", "paragraph_answer": "\uadf8 \ub4a4\ub85c \uace0\uc548\ub41c \uc77c\ubc18\uc0c1\ub300\ub860 \uc2dc\ud5d8 \uc2e4\ud5d8\uc73c\ub85c\ub294 \uc0e4\ud53c\ub85c \ud6a8\uacfc \ub610\ub294 \ube5b\uc758 \uc911\ub825\uc2dc\uac04\uc9c0\uc5f0\uc744 \uc815\ubc00\ud788 \uce21\uc815\ud558\ub294 \uac83\uc774 \uc788\ub2e4. \uac00\uc7a5 \ucd5c\uadfc\uc5d0 \uc774\ub8e8\uc5b4\uc9c4 \ud574\ub2f9 \uc2e4\ud5d8\uc740 2002\ub144\uc5d0 \uce74\uc2dc\ub2c8 \uc6b0\uc8fc\ud0d0\uc0ac\uc120\uc774 \uc218\ud589\ud55c \uc2e4\ud5d8\uc774\ub2e4. \ub610\ub2e4\ub978 \uc2e4\ud5d8\uc73c\ub85c\ub294 \uacf5\uac04 \uc18d\uc5d0\uc11c \uc790\uc774\ub85c\uc2a4\ucf54\ud504\uc758 \ud589\ub3d9\uc758 \uc77c\ubc18\uc0c1\ub300\ub860\uc801 \uc608\uce21\uc744 \uac80\uc99d\ud558\ub294 \uac83\uc774 \uc788\ub2e4. \ub2ec\uc5d0 \uc124\uce58\ub41c \uc778\uacf5\uc704\uc131 \uc6d4\uba74 \ub808\uc774\uc800 \uc7a5\uce58\uac00 \uadf8 \uc608\uce21 \uc911 \ud558\ub098\uc778 \uce21\uc9c0 \uc138\ucc28\ub97c \uc2e4\ud5d8\ud558\uc600\ub2e4. \ub610\ub2e4\ub978 \uc2e4\ud5d8\uc73c\ub85c\ub294 \ud68c\uc804\ud558\ub294 \uc9c8\ub7c9\uacfc \uad00\ub828\ub41c \ud2c0 \ub04c\ub9bc \ud604\uc0c1\uc5d0 \uad00\ud55c \uac83\uc774 \uc788\ub2e4. 2004\ub144 \ubc1c\uc0ac\ub41c \uc911\ub825\ud0d0\uc0ac\uc120 B\ud638 \uc778\uacf5\uc704\uc131\uc740 \ud2b9\uc9c0\ud6a8\uacfc\uc640 \ud2c0 \ub04c\ub9bc \ud6a8\uacfc\ub97c \ubaa8\ub450 \uc2e4\ud5d8\ud574 \ubcf4\uc558\uc73c\uba70, 2008\ub144 12\uc6d4 \uae30\uc900\uc73c\ub85c \uac01\uac01 0.5%\uc640 15% \uc624\ucc28 \ub0b4\uc5d0\uc11c \uc0c1\ub300\uc131\uc774\ub860\uc774 \ub9de\uc74c\uc744 \ud655\uc778\ud558\uc600\ub2e4.", "sentence_answer": "2004\ub144 \ubc1c\uc0ac\ub41c \uc911\ub825\ud0d0\uc0ac\uc120 B\ud638 \uc778\uacf5\uc704\uc131\uc740 \ud2b9\uc9c0\ud6a8\uacfc\uc640 \ud2c0 \ub04c\ub9bc \ud6a8\uacfc\ub97c \ubaa8\ub450 \uc2e4\ud5d8\ud574 \ubcf4\uc558\uc73c\uba70, 2008\ub144 12\uc6d4 \uae30\uc900\uc73c\ub85c \uac01\uac01 0.5%\uc640 15% \uc624\ucc28 \ub0b4\uc5d0\uc11c \uc0c1\ub300\uc131\uc774\ub860\uc774 \ub9de\uc74c\uc744 \ud655\uc778\ud558\uc600\ub2e4.", "paragraph_id": "6657578-9-1", "paragraph_question": "question: \uc77c\ubc18\uc0c1\ub300\ub860\uc758 \uac1c\ub860\uc5d0\uc11c \uc2e4\ud5d8\uc744 \ud1b5\ud574 \uc0c1\ub300\uc131\uc774\ub860\uc774 \ub9de\uc74c\uc744 \ud655\uc778\ud55c \ub0a0\uc9dc\ub294 \uc5b8\uc81c\uc778\uac00?, context: \uadf8 \ub4a4\ub85c \uace0\uc548\ub41c \uc77c\ubc18\uc0c1\ub300\ub860 \uc2dc\ud5d8 \uc2e4\ud5d8\uc73c\ub85c\ub294 \uc0e4\ud53c\ub85c \ud6a8\uacfc \ub610\ub294 \ube5b\uc758 \uc911\ub825\uc2dc\uac04\uc9c0\uc5f0\uc744 \uc815\ubc00\ud788 \uce21\uc815\ud558\ub294 \uac83\uc774 \uc788\ub2e4. \uac00\uc7a5 \ucd5c\uadfc\uc5d0 \uc774\ub8e8\uc5b4\uc9c4 \ud574\ub2f9 \uc2e4\ud5d8\uc740 2002\ub144\uc5d0 \uce74\uc2dc\ub2c8 \uc6b0\uc8fc\ud0d0\uc0ac\uc120\uc774 \uc218\ud589\ud55c \uc2e4\ud5d8\uc774\ub2e4. \ub610\ub2e4\ub978 \uc2e4\ud5d8\uc73c\ub85c\ub294 \uacf5\uac04 \uc18d\uc5d0\uc11c \uc790\uc774\ub85c\uc2a4\ucf54\ud504\uc758 \ud589\ub3d9\uc758 \uc77c\ubc18\uc0c1\ub300\ub860\uc801 \uc608\uce21\uc744 \uac80\uc99d\ud558\ub294 \uac83\uc774 \uc788\ub2e4. \ub2ec\uc5d0 \uc124\uce58\ub41c \uc778\uacf5\uc704\uc131 \uc6d4\uba74 \ub808\uc774\uc800 \uc7a5\uce58\uac00 \uadf8 \uc608\uce21 \uc911 \ud558\ub098\uc778 \uce21\uc9c0 \uc138\ucc28\ub97c \uc2e4\ud5d8\ud558\uc600\ub2e4. \ub610\ub2e4\ub978 \uc2e4\ud5d8\uc73c\ub85c\ub294 \ud68c\uc804\ud558\ub294 \uc9c8\ub7c9\uacfc \uad00\ub828\ub41c \ud2c0 \ub04c\ub9bc \ud604\uc0c1\uc5d0 \uad00\ud55c \uac83\uc774 \uc788\ub2e4. 2004\ub144 \ubc1c\uc0ac\ub41c \uc911\ub825\ud0d0\uc0ac\uc120 B\ud638 \uc778\uacf5\uc704\uc131\uc740 \ud2b9\uc9c0\ud6a8\uacfc\uc640 \ud2c0 \ub04c\ub9bc \ud6a8\uacfc\ub97c \ubaa8\ub450 \uc2e4\ud5d8\ud574 \ubcf4\uc558\uc73c\uba70, 2008\ub144 12\uc6d4 \uae30\uc900\uc73c\ub85c \uac01\uac01 0.5%\uc640 15% \uc624\ucc28 \ub0b4\uc5d0\uc11c \uc0c1\ub300\uc131\uc774\ub860\uc774 \ub9de\uc74c\uc744 \ud655\uc778\ud558\uc600\ub2e4."} {"question": "1997\ub144\uc5d0\uc11c 98\ub144 \uc2dc\uc98c \ub9ac\uadf8 6\uc704\uc5d0 \uc624\ub978 \uc140\ud0c0 \ube44\uace0\uac00 \uc9c4\ucd9c\uad8c\uc744 \ud68d\ub4dd\ud55c \ub300\ud68c\ub294?", "paragraph": "1997/98 \uc2dc\uc98c \ub9ac\uadf8 6\uc704\uc5d0 \uc624\ub974\uba70 UEFA\ucef5 \uc9c4\ucd9c\uad8c\uc744 \ud68d\ub4dd\ud55c \uc140\ud0c0\ub294 \uc774\ub4ec \ud574 UEFA\ucef5 8\uac15\uc5d0 \uc62c\ub790\ub2e4. 98\ub144, \uc140\ud0c0\ub294 \ub9ac\ubc84\ud480\uacfc \uc544\uc2a4\ud1a4 \ube4c\ub77c\ub97c \ucc28\ub840\ub85c \ubb34\ub108\ub728\ub9ac\uba70 UEFA\ucef5 8\uac15\uc5d0 \uc9c4\ucd9c\ud558\ub294 \uae30\uc5fc\uc744 \ud1a0\ud588\ub2e4. \ub2f9\uc2dc \uc140\ud0c0\ub294 UEFA\ucef5\uc5d0\uc11c\uc758 \ud65c\uc57d\uc740 \ubb3c\ub860 \ud504\ub9ac\uba54\ub77c \ub9ac\uadf8\uc5d0\uc11c\ub3c4 \ubc1c\uad70\uc758 \ud65c\uc57d\uc744 \ubcf4\uc600\ub294\ub370, \uadf8\ub4e4\uc740 \uc5d0\uc2a4\ud0c0\ub514\uc624 \ubc1c\ub77c\uc774\ub3c4\uc2a4(\uc140\ud0c0\uc758 \ud648 \uad6c\uc7a5)\uc5d0\uc11c 1\ub144\uc774 \ub118\uac8c \ub2e8 \ud55c\ubc88\ub3c4 \ud328\ud558\uc9c0 \uc54a\uc744 \uc815\ub3c4\ub85c \ub9c9\uac15\ud55c \ud300\uc774\uc5c8\ub2e4. \uc140\ud0c0\ub294 \ub2f9\uc2dc \ube7c\uc5b4\ub09c \ud65c\uc57d\uc744 \ubcf4\uc778 \ud300\uc758 \uac04\ud310 \ubaa8\uc2a4\ud1a0\ubcf4\uc774, \ub9c8\ucf08\ub810\ub808, \uc0b4\uac00\ub3c4\ub97c \ube44\ub86f\ud574 1994 \ubbf8\uad6d \uc6d4\ub4dc\ucef5 \uc6b0\uc2b9\uc744 \uacbd\ud5d8\ud55c \ube0c\ub77c\uc9c8 \ucd9c\uc2e0 \ub9c8\uc9d5\uc694\uae4c\uc9c0 \ubcf4\uc720\ud558\uace0 \uc788\uc744 \uc815\ub3c4\ub85c \uac15\ub825\ud55c \uc120\uc218\ub2e8\uc744 \ubcf4\uc720\ud588\uc5c8\ub2e4. 98\ub144 \uc2dc\uc98c\uc774 \uc9c4\ud589\ub418\ub294 \ub3d9\ud55c 4\uc704\uad8c \ubc16\uc744 \uac70\uc758 \ubc97\uc5b4\ub098\uc9c0 \uc54a\uc744 \uc815\ub3c4\ub85c \uc548\uc815\ub41c \ud300\uc774\uc5c8\uc9c0\ub9cc, \uc2dc\uc98c \ub9c8\uc9c0\ub9c9 \uacbd\uae30\uc5d0\uc11c \ud1b5\ud55c\uc758 \ud328\ubc30\ub97c \ub2f9\ud558\uba70 UEFA \ucc54\ud53c\uc5b8\uc2a4\ub9ac\uadf8 \ud2f0\ucf13\uc744 \ub193\uce58\uace0 \ub9cc\ub2e4. \uadf8 \uc774\ud6c4 \uc140\ud0c0\ub294 \uba87\uba87\uc758 \uc8fc\uc804\uae09 \uc120\uc218\ub4e4\uc774 \uc740\ud1f4, \ud639\uc740 \ube45\ud074\ub7fd\uc73c\ub85c \uc774\uc801\ud558\uba70 \uc804\ub825 \uc190\uc2e4\uc774 \uc608\uc0c1 \ub410\uc9c0\ub9cc 3,000\ub9cc \uc720\ub85c\uac00 \ub118\ub294 \ub2f9\uc2dc\ub85c\uc11c\ub294 \uc5b4\ub9c8\uc5b4\ub9c8\ud55c \uc561\uc218\ub97c \ud22c\uc790\ud558\uba70 \ub8e8\uc774\uc2a4 \uc5d0\ub450, \ud5e4\uc220\ub9ac, \uce74\ud0c0\ub0d0, \uce74\ub974\ud540, \ubca0\ub2c8 \ub9e5\uce74\uc2dc \ub4f1\uc744 \uc601\uc785\ud558\uba70 \uc120\uc218\ub2e8 \ubcf4\uac15\uc744 \uc131\uacf5\uc801\uc73c\ub85c \ub9c8\uce5c\ub2e4.", "answer": "UEFA\ucef5", "sentence": "1997/98 \uc2dc\uc98c \ub9ac\uadf8 6\uc704\uc5d0 \uc624\ub974\uba70 UEFA\ucef5 \uc9c4\ucd9c\uad8c\uc744 \ud68d\ub4dd\ud55c \uc140\ud0c0\ub294 \uc774\ub4ec \ud574 UEFA\ucef5 8\uac15\uc5d0 \uc62c\ub790\ub2e4.", "paragraph_sentence": " 1997/98 \uc2dc\uc98c \ub9ac\uadf8 6\uc704\uc5d0 \uc624\ub974\uba70 UEFA\ucef5 \uc9c4\ucd9c\uad8c\uc744 \ud68d\ub4dd\ud55c \uc140\ud0c0\ub294 \uc774\ub4ec \ud574 UEFA\ucef5 8\uac15\uc5d0 \uc62c\ub790\ub2e4. 98\ub144, \uc140\ud0c0\ub294 \ub9ac\ubc84\ud480\uacfc \uc544\uc2a4\ud1a4 \ube4c\ub77c\ub97c \ucc28\ub840\ub85c \ubb34\ub108\ub728\ub9ac\uba70 UEFA\ucef5 8\uac15\uc5d0 \uc9c4\ucd9c\ud558\ub294 \uae30\uc5fc\uc744 \ud1a0\ud588\ub2e4. \ub2f9\uc2dc \uc140\ud0c0\ub294 UEFA\ucef5\uc5d0\uc11c\uc758 \ud65c\uc57d\uc740 \ubb3c\ub860 \ud504\ub9ac\uba54\ub77c \ub9ac\uadf8\uc5d0\uc11c\ub3c4 \ubc1c\uad70\uc758 \ud65c\uc57d\uc744 \ubcf4\uc600\ub294\ub370, \uadf8\ub4e4\uc740 \uc5d0\uc2a4\ud0c0\ub514\uc624 \ubc1c\ub77c\uc774\ub3c4\uc2a4(\uc140\ud0c0\uc758 \ud648 \uad6c\uc7a5)\uc5d0\uc11c 1\ub144\uc774 \ub118\uac8c \ub2e8 \ud55c\ubc88\ub3c4 \ud328\ud558\uc9c0 \uc54a\uc744 \uc815\ub3c4\ub85c \ub9c9\uac15\ud55c \ud300\uc774\uc5c8\ub2e4. \uc140\ud0c0\ub294 \ub2f9\uc2dc \ube7c\uc5b4\ub09c \ud65c\uc57d\uc744 \ubcf4\uc778 \ud300\uc758 \uac04\ud310 \ubaa8\uc2a4\ud1a0\ubcf4\uc774, \ub9c8\ucf08\ub810\ub808, \uc0b4\uac00\ub3c4\ub97c \ube44\ub86f\ud574 1994 \ubbf8\uad6d \uc6d4\ub4dc\ucef5 \uc6b0\uc2b9\uc744 \uacbd\ud5d8\ud55c \ube0c\ub77c\uc9c8 \ucd9c\uc2e0 \ub9c8\uc9d5\uc694\uae4c\uc9c0 \ubcf4\uc720\ud558\uace0 \uc788\uc744 \uc815\ub3c4\ub85c \uac15\ub825\ud55c \uc120\uc218\ub2e8\uc744 \ubcf4\uc720\ud588\uc5c8\ub2e4. 98\ub144 \uc2dc\uc98c\uc774 \uc9c4\ud589\ub418\ub294 \ub3d9\ud55c 4\uc704\uad8c \ubc16\uc744 \uac70\uc758 \ubc97\uc5b4\ub098\uc9c0 \uc54a\uc744 \uc815\ub3c4\ub85c \uc548\uc815\ub41c \ud300\uc774\uc5c8\uc9c0\ub9cc, \uc2dc\uc98c \ub9c8\uc9c0\ub9c9 \uacbd\uae30\uc5d0\uc11c \ud1b5\ud55c\uc758 \ud328\ubc30\ub97c \ub2f9\ud558\uba70 UEFA \ucc54\ud53c\uc5b8\uc2a4\ub9ac\uadf8 \ud2f0\ucf13\uc744 \ub193\uce58\uace0 \ub9cc\ub2e4. \uadf8 \uc774\ud6c4 \uc140\ud0c0\ub294 \uba87\uba87\uc758 \uc8fc\uc804\uae09 \uc120\uc218\ub4e4\uc774 \uc740\ud1f4, \ud639\uc740 \ube45\ud074\ub7fd\uc73c\ub85c \uc774\uc801\ud558\uba70 \uc804\ub825 \uc190\uc2e4\uc774 \uc608\uc0c1 \ub410\uc9c0\ub9cc 3,000\ub9cc \uc720\ub85c\uac00 \ub118\ub294 \ub2f9\uc2dc\ub85c\uc11c\ub294 \uc5b4\ub9c8\uc5b4\ub9c8\ud55c \uc561\uc218\ub97c \ud22c\uc790\ud558\uba70 \ub8e8\uc774\uc2a4 \uc5d0\ub450, \ud5e4\uc220\ub9ac, \uce74\ud0c0\ub0d0, \uce74\ub974\ud540, \ubca0\ub2c8 \ub9e5\uce74\uc2dc \ub4f1\uc744 \uc601\uc785\ud558\uba70 \uc120\uc218\ub2e8 \ubcf4\uac15\uc744 \uc131\uacf5\uc801\uc73c\ub85c \ub9c8\uce5c\ub2e4.", "paragraph_answer": "1997/98 \uc2dc\uc98c \ub9ac\uadf8 6\uc704\uc5d0 \uc624\ub974\uba70 UEFA\ucef5 \uc9c4\ucd9c\uad8c\uc744 \ud68d\ub4dd\ud55c \uc140\ud0c0\ub294 \uc774\ub4ec \ud574 UEFA\ucef5 8\uac15\uc5d0 \uc62c\ub790\ub2e4. 98\ub144, \uc140\ud0c0\ub294 \ub9ac\ubc84\ud480\uacfc \uc544\uc2a4\ud1a4 \ube4c\ub77c\ub97c \ucc28\ub840\ub85c \ubb34\ub108\ub728\ub9ac\uba70 UEFA\ucef5 8\uac15\uc5d0 \uc9c4\ucd9c\ud558\ub294 \uae30\uc5fc\uc744 \ud1a0\ud588\ub2e4. \ub2f9\uc2dc \uc140\ud0c0\ub294 UEFA\ucef5\uc5d0\uc11c\uc758 \ud65c\uc57d\uc740 \ubb3c\ub860 \ud504\ub9ac\uba54\ub77c \ub9ac\uadf8\uc5d0\uc11c\ub3c4 \ubc1c\uad70\uc758 \ud65c\uc57d\uc744 \ubcf4\uc600\ub294\ub370, \uadf8\ub4e4\uc740 \uc5d0\uc2a4\ud0c0\ub514\uc624 \ubc1c\ub77c\uc774\ub3c4\uc2a4(\uc140\ud0c0\uc758 \ud648 \uad6c\uc7a5)\uc5d0\uc11c 1\ub144\uc774 \ub118\uac8c \ub2e8 \ud55c\ubc88\ub3c4 \ud328\ud558\uc9c0 \uc54a\uc744 \uc815\ub3c4\ub85c \ub9c9\uac15\ud55c \ud300\uc774\uc5c8\ub2e4. \uc140\ud0c0\ub294 \ub2f9\uc2dc \ube7c\uc5b4\ub09c \ud65c\uc57d\uc744 \ubcf4\uc778 \ud300\uc758 \uac04\ud310 \ubaa8\uc2a4\ud1a0\ubcf4\uc774, \ub9c8\ucf08\ub810\ub808, \uc0b4\uac00\ub3c4\ub97c \ube44\ub86f\ud574 1994 \ubbf8\uad6d \uc6d4\ub4dc\ucef5 \uc6b0\uc2b9\uc744 \uacbd\ud5d8\ud55c \ube0c\ub77c\uc9c8 \ucd9c\uc2e0 \ub9c8\uc9d5\uc694\uae4c\uc9c0 \ubcf4\uc720\ud558\uace0 \uc788\uc744 \uc815\ub3c4\ub85c \uac15\ub825\ud55c \uc120\uc218\ub2e8\uc744 \ubcf4\uc720\ud588\uc5c8\ub2e4. 98\ub144 \uc2dc\uc98c\uc774 \uc9c4\ud589\ub418\ub294 \ub3d9\ud55c 4\uc704\uad8c \ubc16\uc744 \uac70\uc758 \ubc97\uc5b4\ub098\uc9c0 \uc54a\uc744 \uc815\ub3c4\ub85c \uc548\uc815\ub41c \ud300\uc774\uc5c8\uc9c0\ub9cc, \uc2dc\uc98c \ub9c8\uc9c0\ub9c9 \uacbd\uae30\uc5d0\uc11c \ud1b5\ud55c\uc758 \ud328\ubc30\ub97c \ub2f9\ud558\uba70 UEFA \ucc54\ud53c\uc5b8\uc2a4\ub9ac\uadf8 \ud2f0\ucf13\uc744 \ub193\uce58\uace0 \ub9cc\ub2e4. \uadf8 \uc774\ud6c4 \uc140\ud0c0\ub294 \uba87\uba87\uc758 \uc8fc\uc804\uae09 \uc120\uc218\ub4e4\uc774 \uc740\ud1f4, \ud639\uc740 \ube45\ud074\ub7fd\uc73c\ub85c \uc774\uc801\ud558\uba70 \uc804\ub825 \uc190\uc2e4\uc774 \uc608\uc0c1 \ub410\uc9c0\ub9cc 3,000\ub9cc \uc720\ub85c\uac00 \ub118\ub294 \ub2f9\uc2dc\ub85c\uc11c\ub294 \uc5b4\ub9c8\uc5b4\ub9c8\ud55c \uc561\uc218\ub97c \ud22c\uc790\ud558\uba70 \ub8e8\uc774\uc2a4 \uc5d0\ub450, \ud5e4\uc220\ub9ac, \uce74\ud0c0\ub0d0, \uce74\ub974\ud540, \ubca0\ub2c8 \ub9e5\uce74\uc2dc \ub4f1\uc744 \uc601\uc785\ud558\uba70 \uc120\uc218\ub2e8 \ubcf4\uac15\uc744 \uc131\uacf5\uc801\uc73c\ub85c \ub9c8\uce5c\ub2e4.", "sentence_answer": "1997/98 \uc2dc\uc98c \ub9ac\uadf8 6\uc704\uc5d0 \uc624\ub974\uba70 UEFA\ucef5 \uc9c4\ucd9c\uad8c\uc744 \ud68d\ub4dd\ud55c \uc140\ud0c0\ub294 \uc774\ub4ec \ud574 UEFA\ucef5 8\uac15\uc5d0 \uc62c\ub790\ub2e4.", "paragraph_id": "6518223-3-0", "paragraph_question": "question: 1997\ub144\uc5d0\uc11c 98\ub144 \uc2dc\uc98c \ub9ac\uadf8 6\uc704\uc5d0 \uc624\ub978 \uc140\ud0c0 \ube44\uace0\uac00 \uc9c4\ucd9c\uad8c\uc744 \ud68d\ub4dd\ud55c \ub300\ud68c\ub294?, context: 1997/98 \uc2dc\uc98c \ub9ac\uadf8 6\uc704\uc5d0 \uc624\ub974\uba70 UEFA\ucef5 \uc9c4\ucd9c\uad8c\uc744 \ud68d\ub4dd\ud55c \uc140\ud0c0\ub294 \uc774\ub4ec \ud574 UEFA\ucef5 8\uac15\uc5d0 \uc62c\ub790\ub2e4. 98\ub144, \uc140\ud0c0\ub294 \ub9ac\ubc84\ud480\uacfc \uc544\uc2a4\ud1a4 \ube4c\ub77c\ub97c \ucc28\ub840\ub85c \ubb34\ub108\ub728\ub9ac\uba70 UEFA\ucef5 8\uac15\uc5d0 \uc9c4\ucd9c\ud558\ub294 \uae30\uc5fc\uc744 \ud1a0\ud588\ub2e4. \ub2f9\uc2dc \uc140\ud0c0\ub294 UEFA\ucef5\uc5d0\uc11c\uc758 \ud65c\uc57d\uc740 \ubb3c\ub860 \ud504\ub9ac\uba54\ub77c \ub9ac\uadf8\uc5d0\uc11c\ub3c4 \ubc1c\uad70\uc758 \ud65c\uc57d\uc744 \ubcf4\uc600\ub294\ub370, \uadf8\ub4e4\uc740 \uc5d0\uc2a4\ud0c0\ub514\uc624 \ubc1c\ub77c\uc774\ub3c4\uc2a4(\uc140\ud0c0\uc758 \ud648 \uad6c\uc7a5)\uc5d0\uc11c 1\ub144\uc774 \ub118\uac8c \ub2e8 \ud55c\ubc88\ub3c4 \ud328\ud558\uc9c0 \uc54a\uc744 \uc815\ub3c4\ub85c \ub9c9\uac15\ud55c \ud300\uc774\uc5c8\ub2e4. \uc140\ud0c0\ub294 \ub2f9\uc2dc \ube7c\uc5b4\ub09c \ud65c\uc57d\uc744 \ubcf4\uc778 \ud300\uc758 \uac04\ud310 \ubaa8\uc2a4\ud1a0\ubcf4\uc774, \ub9c8\ucf08\ub810\ub808, \uc0b4\uac00\ub3c4\ub97c \ube44\ub86f\ud574 1994 \ubbf8\uad6d \uc6d4\ub4dc\ucef5 \uc6b0\uc2b9\uc744 \uacbd\ud5d8\ud55c \ube0c\ub77c\uc9c8 \ucd9c\uc2e0 \ub9c8\uc9d5\uc694\uae4c\uc9c0 \ubcf4\uc720\ud558\uace0 \uc788\uc744 \uc815\ub3c4\ub85c \uac15\ub825\ud55c \uc120\uc218\ub2e8\uc744 \ubcf4\uc720\ud588\uc5c8\ub2e4. 98\ub144 \uc2dc\uc98c\uc774 \uc9c4\ud589\ub418\ub294 \ub3d9\ud55c 4\uc704\uad8c \ubc16\uc744 \uac70\uc758 \ubc97\uc5b4\ub098\uc9c0 \uc54a\uc744 \uc815\ub3c4\ub85c \uc548\uc815\ub41c \ud300\uc774\uc5c8\uc9c0\ub9cc, \uc2dc\uc98c \ub9c8\uc9c0\ub9c9 \uacbd\uae30\uc5d0\uc11c \ud1b5\ud55c\uc758 \ud328\ubc30\ub97c \ub2f9\ud558\uba70 UEFA \ucc54\ud53c\uc5b8\uc2a4\ub9ac\uadf8 \ud2f0\ucf13\uc744 \ub193\uce58\uace0 \ub9cc\ub2e4. \uadf8 \uc774\ud6c4 \uc140\ud0c0\ub294 \uba87\uba87\uc758 \uc8fc\uc804\uae09 \uc120\uc218\ub4e4\uc774 \uc740\ud1f4, \ud639\uc740 \ube45\ud074\ub7fd\uc73c\ub85c \uc774\uc801\ud558\uba70 \uc804\ub825 \uc190\uc2e4\uc774 \uc608\uc0c1 \ub410\uc9c0\ub9cc 3,000\ub9cc \uc720\ub85c\uac00 \ub118\ub294 \ub2f9\uc2dc\ub85c\uc11c\ub294 \uc5b4\ub9c8\uc5b4\ub9c8\ud55c \uc561\uc218\ub97c \ud22c\uc790\ud558\uba70 \ub8e8\uc774\uc2a4 \uc5d0\ub450, \ud5e4\uc220\ub9ac, \uce74\ud0c0\ub0d0, \uce74\ub974\ud540, \ubca0\ub2c8 \ub9e5\uce74\uc2dc \ub4f1\uc744 \uc601\uc785\ud558\uba70 \uc120\uc218\ub2e8 \ubcf4\uac15\uc744 \uc131\uacf5\uc801\uc73c\ub85c \ub9c8\uce5c\ub2e4."} {"question": "\ub178\uc2e0\uc601\uc740 \ub204\uad6c\uc758 \uc0ac\uc784\uc744 \uc608\uc2dc\ub85c \ub4e4\uba74\uc11c \uace0\ubb38\uce58\uc0ac \uc0ac\uac74 \ubc1c\ud45c\uc758 \ub2f9\uc704\uc131\uc744 \uc8fc\uc7a5\ud588\ub294\uac00?", "paragraph": "1987\ub144 1\uc6d4 14\uc77c 6\uc6d4 \ud56d\uc7c1\uc73c\ub85c \ucd09\ubc1c\ub418\ub294 \ubc15\uc885\ucca0 \uace0\ubb38\uce58\uc0ac \uc0ac\uac74\uc774 \ubc1c\uc0dd\ud558\uc600\ub2e4. 1\uc6d4 15\uc77c \uc800\ub141, \uc77c\uac04\uc9c0\ub97c \uc77d\ub358 \ub3c4\uc911 \uad6c\uc11d\uc5d0 \uc870\uadf8\ub9e3\uac8c \uc50c\uc5b4\uc9c4 \uae30\uc0ac\ub97c \uc774\uc0c1\ud558\uac8c \uc5ec\uae34 \ub178\uc2e0\uc601\uc740 \ub2e4\uc74c\ub0a0 \uc544\uce68 \uae40\uc885\ud638 \ub0b4\ubb34\ubd80\uc7a5\uad00 \uacfc \uc815\uc7ac\ucca0 \uc815\ubb34\uc7a5\uad00\uc744 \ucd94\uad81\ud55c\ub2e4. \ucc98\uc74c\uc5d0 \uae40\uc885\ud638\ub294 \uacbd\ucc30\uc774 \ubc1c\ud45c\ud55c \uae30\uc0ac \uadf8\ub300\ub85c\ub77c\uba70 \uac70\uc9d3\uc744 \uace0\ud558\uc600\uc73c\ub098 \uc815\uc7ac\ucca0\uc774 \uc790\ub9ac\ub97c \ub728\uc790 \uace7 \uace0\ubb38\uce58\uc0ac \uc0ac\uac74\uc774\ub77c\uba70 \uc0ac\uc2e4\uc744 \ub9d0\ud55c\ub2e4. \uc774\uc5d0 \uace7 \ub300\ucc45\ud68c\uc758\ub97c \uc18c\uc9d1\ud558\uc5ec \ub2f9\uc2dc \ub178\ud0dc\uc6b0 \ub300\ud45c\uc704\uc6d0\uacfc \ubc15\uc601\uc218 \uccad\uc640\ub300\ube44\uc11c\uc2e4\uc7a5, \uadf8\ub9ac\uace0 \uc7a5\uc138\ub3d9 \uc548\uae30\ubd80\uc7a5, \ub178\uc2e0\uc601 4\uc0ac\ub78c\uc774 \uc774 \ubb38\uc81c\uc5d0 \ub300\ud574 \ud611\uc758\ud558\uc600\uace0 \uc774 \ud68c\uc758\uc5d0\uc11c \ub178\uc2e0\uc601\uc740 \"\uc804\ub0a0\uc758 \ub2c9\uc2a8 \uc804 \ubbf8\uad6d\ub300\ud1b5\ub839\uc758 \uc0ac\uc784\uc774 \ub3c4\uccad \uc0ac\uc2e4\uc744 \uc740\ud3d0\ud558\uc600\uae30\uc5d0 \ubc1c\uc0dd\ud55c \uc77c\uc774\ub77c\uba70, \ub2a6\uc5c8\uc9c0\ub9cc \uc774 \uc0ac\uc2e4\uc744 \ubc29\uc1a1\ub9e4\uccb4\ub97c \ud1b5\ud558\uc5ec \uc804\uad6d\ubbfc\uc5d0\uac8c \uc54c\ub824\uc57c \ud55c\ub2e4\"\uace0 \ub9d0\ud558\uc600\ub2e4. \ud558\uc9c0\ub9cc \ud6c4\uc5d0 \ub610 \ubc94\uc778\ucd95\uc18c \uc870\uc791\uc0ac\uac74\uc774 \ubc1c\uc0dd\ud558\uc600\uace0 \ucd1d\ub9ac\uc778 \uc790\uc2e0\uc774 \ubaa8\ub4e0 \ucc45\uc784\uc744 \uc9c0\uace0 \ud1f4\uc784\uc744 \ud558\uc9c0 \uc54a\uc744\uc2dc\uc5d4 \uadf8 \uc5ec\ud30c\uac00 \uccad\uc640\ub300\uae4c\uc9c0 \ubbf8\uce60 \uc218\ub3c4 \uc788\uc74c\uc744 \uc6b0\ub824\ud558\uc5ec \uc804\ub450\ud658\uc5d0\uac8c \uc0ac\uc758\ub97c \ud45c\uba85\ud55c\ub2e4. \ucc98\uc74c\uc5d4 \uc804\ub450\ud658 \uc5ed\uc2dc \uc784\uae30\uac00 9\uac1c\uc6d4\ubc16\uc5d0 \ub0a8\uc9c0 \uc54a\uc558\uc73c\ub2c8 \uadf8\ub0e5 \uc720\uc784\ud560 \uac83\uc744 \uad8c\uace0\ud558\uc600\uc73c\ub098 \uc5ec\ub860\uc774 \uae09\uc18d\ub3c4\ub85c \uc545\ud654\ub418\uc5b4 \uacb0\uad6d \uadf8\uc758 \uc0ac\uc784\uc744 \uacb0\uc815\ud558\uac8c \ub418\uc5c8\ub2e4.", "answer": "\ub2c9\uc2a8 \uc804 \ubbf8\uad6d\ub300\ud1b5\ub839", "sentence": "\uc774\uc5d0 \uace7 \ub300\ucc45\ud68c\uc758\ub97c \uc18c\uc9d1\ud558\uc5ec \ub2f9\uc2dc \ub178\ud0dc\uc6b0 \ub300\ud45c\uc704\uc6d0\uacfc \ubc15\uc601\uc218 \uccad\uc640\ub300\ube44\uc11c\uc2e4\uc7a5, \uadf8\ub9ac\uace0 \uc7a5\uc138\ub3d9 \uc548\uae30\ubd80\uc7a5, \ub178\uc2e0\uc601 4\uc0ac\ub78c\uc774 \uc774 \ubb38\uc81c\uc5d0 \ub300\ud574 \ud611\uc758\ud558\uc600\uace0 \uc774 \ud68c\uc758\uc5d0\uc11c \ub178\uc2e0\uc601\uc740 \"\uc804\ub0a0\uc758 \ub2c9\uc2a8 \uc804 \ubbf8\uad6d\ub300\ud1b5\ub839 \uc758 \uc0ac\uc784\uc774 \ub3c4\uccad \uc0ac\uc2e4\uc744 \uc740\ud3d0\ud558\uc600\uae30\uc5d0 \ubc1c\uc0dd\ud55c \uc77c\uc774\ub77c\uba70, \ub2a6\uc5c8\uc9c0\ub9cc \uc774 \uc0ac\uc2e4\uc744 \ubc29\uc1a1\ub9e4\uccb4\ub97c \ud1b5\ud558\uc5ec \uc804\uad6d\ubbfc\uc5d0\uac8c \uc54c\ub824\uc57c \ud55c\ub2e4\"\uace0 \ub9d0\ud558\uc600\ub2e4.", "paragraph_sentence": "1987\ub144 1\uc6d4 14\uc77c 6\uc6d4 \ud56d\uc7c1\uc73c\ub85c \ucd09\ubc1c\ub418\ub294 \ubc15\uc885\ucca0 \uace0\ubb38\uce58\uc0ac \uc0ac\uac74\uc774 \ubc1c\uc0dd\ud558\uc600\ub2e4. 1\uc6d4 15\uc77c \uc800\ub141, \uc77c\uac04\uc9c0\ub97c \uc77d\ub358 \ub3c4\uc911 \uad6c\uc11d\uc5d0 \uc870\uadf8\ub9e3\uac8c \uc50c\uc5b4\uc9c4 \uae30\uc0ac\ub97c \uc774\uc0c1\ud558\uac8c \uc5ec\uae34 \ub178\uc2e0\uc601\uc740 \ub2e4\uc74c\ub0a0 \uc544\uce68 \uae40\uc885\ud638 \ub0b4\ubb34\ubd80\uc7a5\uad00 \uacfc \uc815\uc7ac\ucca0 \uc815\ubb34\uc7a5\uad00\uc744 \ucd94\uad81\ud55c\ub2e4. \ucc98\uc74c\uc5d0 \uae40\uc885\ud638\ub294 \uacbd\ucc30\uc774 \ubc1c\ud45c\ud55c \uae30\uc0ac \uadf8\ub300\ub85c\ub77c\uba70 \uac70\uc9d3\uc744 \uace0\ud558\uc600\uc73c\ub098 \uc815\uc7ac\ucca0\uc774 \uc790\ub9ac\ub97c \ub728\uc790 \uace7 \uace0\ubb38\uce58\uc0ac \uc0ac\uac74\uc774\ub77c\uba70 \uc0ac\uc2e4\uc744 \ub9d0\ud55c\ub2e4. \uc774\uc5d0 \uace7 \ub300\ucc45\ud68c\uc758\ub97c \uc18c\uc9d1\ud558\uc5ec \ub2f9\uc2dc \ub178\ud0dc\uc6b0 \ub300\ud45c\uc704\uc6d0\uacfc \ubc15\uc601\uc218 \uccad\uc640\ub300\ube44\uc11c\uc2e4\uc7a5, \uadf8\ub9ac\uace0 \uc7a5\uc138\ub3d9 \uc548\uae30\ubd80\uc7a5, \ub178\uc2e0\uc601 4\uc0ac\ub78c\uc774 \uc774 \ubb38\uc81c\uc5d0 \ub300\ud574 \ud611\uc758\ud558\uc600\uace0 \uc774 \ud68c\uc758\uc5d0\uc11c \ub178\uc2e0\uc601\uc740 \"\uc804\ub0a0\uc758 \ub2c9\uc2a8 \uc804 \ubbf8\uad6d\ub300\ud1b5\ub839 \uc758 \uc0ac\uc784\uc774 \ub3c4\uccad \uc0ac\uc2e4\uc744 \uc740\ud3d0\ud558\uc600\uae30\uc5d0 \ubc1c\uc0dd\ud55c \uc77c\uc774\ub77c\uba70, \ub2a6\uc5c8\uc9c0\ub9cc \uc774 \uc0ac\uc2e4\uc744 \ubc29\uc1a1\ub9e4\uccb4\ub97c \ud1b5\ud558\uc5ec \uc804\uad6d\ubbfc\uc5d0\uac8c \uc54c\ub824\uc57c \ud55c\ub2e4\"\uace0 \ub9d0\ud558\uc600\ub2e4. \ud558\uc9c0\ub9cc \ud6c4\uc5d0 \ub610 \ubc94\uc778\ucd95\uc18c \uc870\uc791\uc0ac\uac74\uc774 \ubc1c\uc0dd\ud558\uc600\uace0 \ucd1d\ub9ac\uc778 \uc790\uc2e0\uc774 \ubaa8\ub4e0 \ucc45\uc784\uc744 \uc9c0\uace0 \ud1f4\uc784\uc744 \ud558\uc9c0 \uc54a\uc744\uc2dc\uc5d4 \uadf8 \uc5ec\ud30c\uac00 \uccad\uc640\ub300\uae4c\uc9c0 \ubbf8\uce60 \uc218\ub3c4 \uc788\uc74c\uc744 \uc6b0\ub824\ud558\uc5ec \uc804\ub450\ud658\uc5d0\uac8c \uc0ac\uc758\ub97c \ud45c\uba85\ud55c\ub2e4. \ucc98\uc74c\uc5d4 \uc804\ub450\ud658 \uc5ed\uc2dc \uc784\uae30\uac00 9\uac1c\uc6d4\ubc16\uc5d0 \ub0a8\uc9c0 \uc54a\uc558\uc73c\ub2c8 \uadf8\ub0e5 \uc720\uc784\ud560 \uac83\uc744 \uad8c\uace0\ud558\uc600\uc73c\ub098 \uc5ec\ub860\uc774 \uae09\uc18d\ub3c4\ub85c \uc545\ud654\ub418\uc5b4 \uacb0\uad6d \uadf8\uc758 \uc0ac\uc784\uc744 \uacb0\uc815\ud558\uac8c \ub418\uc5c8\ub2e4.", "paragraph_answer": "1987\ub144 1\uc6d4 14\uc77c 6\uc6d4 \ud56d\uc7c1\uc73c\ub85c \ucd09\ubc1c\ub418\ub294 \ubc15\uc885\ucca0 \uace0\ubb38\uce58\uc0ac \uc0ac\uac74\uc774 \ubc1c\uc0dd\ud558\uc600\ub2e4. 1\uc6d4 15\uc77c \uc800\ub141, \uc77c\uac04\uc9c0\ub97c \uc77d\ub358 \ub3c4\uc911 \uad6c\uc11d\uc5d0 \uc870\uadf8\ub9e3\uac8c \uc50c\uc5b4\uc9c4 \uae30\uc0ac\ub97c \uc774\uc0c1\ud558\uac8c \uc5ec\uae34 \ub178\uc2e0\uc601\uc740 \ub2e4\uc74c\ub0a0 \uc544\uce68 \uae40\uc885\ud638 \ub0b4\ubb34\ubd80\uc7a5\uad00 \uacfc \uc815\uc7ac\ucca0 \uc815\ubb34\uc7a5\uad00\uc744 \ucd94\uad81\ud55c\ub2e4. \ucc98\uc74c\uc5d0 \uae40\uc885\ud638\ub294 \uacbd\ucc30\uc774 \ubc1c\ud45c\ud55c \uae30\uc0ac \uadf8\ub300\ub85c\ub77c\uba70 \uac70\uc9d3\uc744 \uace0\ud558\uc600\uc73c\ub098 \uc815\uc7ac\ucca0\uc774 \uc790\ub9ac\ub97c \ub728\uc790 \uace7 \uace0\ubb38\uce58\uc0ac \uc0ac\uac74\uc774\ub77c\uba70 \uc0ac\uc2e4\uc744 \ub9d0\ud55c\ub2e4. \uc774\uc5d0 \uace7 \ub300\ucc45\ud68c\uc758\ub97c \uc18c\uc9d1\ud558\uc5ec \ub2f9\uc2dc \ub178\ud0dc\uc6b0 \ub300\ud45c\uc704\uc6d0\uacfc \ubc15\uc601\uc218 \uccad\uc640\ub300\ube44\uc11c\uc2e4\uc7a5, \uadf8\ub9ac\uace0 \uc7a5\uc138\ub3d9 \uc548\uae30\ubd80\uc7a5, \ub178\uc2e0\uc601 4\uc0ac\ub78c\uc774 \uc774 \ubb38\uc81c\uc5d0 \ub300\ud574 \ud611\uc758\ud558\uc600\uace0 \uc774 \ud68c\uc758\uc5d0\uc11c \ub178\uc2e0\uc601\uc740 \"\uc804\ub0a0\uc758 \ub2c9\uc2a8 \uc804 \ubbf8\uad6d\ub300\ud1b5\ub839 \uc758 \uc0ac\uc784\uc774 \ub3c4\uccad \uc0ac\uc2e4\uc744 \uc740\ud3d0\ud558\uc600\uae30\uc5d0 \ubc1c\uc0dd\ud55c \uc77c\uc774\ub77c\uba70, \ub2a6\uc5c8\uc9c0\ub9cc \uc774 \uc0ac\uc2e4\uc744 \ubc29\uc1a1\ub9e4\uccb4\ub97c \ud1b5\ud558\uc5ec \uc804\uad6d\ubbfc\uc5d0\uac8c \uc54c\ub824\uc57c \ud55c\ub2e4\"\uace0 \ub9d0\ud558\uc600\ub2e4. \ud558\uc9c0\ub9cc \ud6c4\uc5d0 \ub610 \ubc94\uc778\ucd95\uc18c \uc870\uc791\uc0ac\uac74\uc774 \ubc1c\uc0dd\ud558\uc600\uace0 \ucd1d\ub9ac\uc778 \uc790\uc2e0\uc774 \ubaa8\ub4e0 \ucc45\uc784\uc744 \uc9c0\uace0 \ud1f4\uc784\uc744 \ud558\uc9c0 \uc54a\uc744\uc2dc\uc5d4 \uadf8 \uc5ec\ud30c\uac00 \uccad\uc640\ub300\uae4c\uc9c0 \ubbf8\uce60 \uc218\ub3c4 \uc788\uc74c\uc744 \uc6b0\ub824\ud558\uc5ec \uc804\ub450\ud658\uc5d0\uac8c \uc0ac\uc758\ub97c \ud45c\uba85\ud55c\ub2e4. \ucc98\uc74c\uc5d4 \uc804\ub450\ud658 \uc5ed\uc2dc \uc784\uae30\uac00 9\uac1c\uc6d4\ubc16\uc5d0 \ub0a8\uc9c0 \uc54a\uc558\uc73c\ub2c8 \uadf8\ub0e5 \uc720\uc784\ud560 \uac83\uc744 \uad8c\uace0\ud558\uc600\uc73c\ub098 \uc5ec\ub860\uc774 \uae09\uc18d\ub3c4\ub85c \uc545\ud654\ub418\uc5b4 \uacb0\uad6d \uadf8\uc758 \uc0ac\uc784\uc744 \uacb0\uc815\ud558\uac8c \ub418\uc5c8\ub2e4.", "sentence_answer": "\uc774\uc5d0 \uace7 \ub300\ucc45\ud68c\uc758\ub97c \uc18c\uc9d1\ud558\uc5ec \ub2f9\uc2dc \ub178\ud0dc\uc6b0 \ub300\ud45c\uc704\uc6d0\uacfc \ubc15\uc601\uc218 \uccad\uc640\ub300\ube44\uc11c\uc2e4\uc7a5, \uadf8\ub9ac\uace0 \uc7a5\uc138\ub3d9 \uc548\uae30\ubd80\uc7a5, \ub178\uc2e0\uc601 4\uc0ac\ub78c\uc774 \uc774 \ubb38\uc81c\uc5d0 \ub300\ud574 \ud611\uc758\ud558\uc600\uace0 \uc774 \ud68c\uc758\uc5d0\uc11c \ub178\uc2e0\uc601\uc740 \"\uc804\ub0a0\uc758 \ub2c9\uc2a8 \uc804 \ubbf8\uad6d\ub300\ud1b5\ub839 \uc758 \uc0ac\uc784\uc774 \ub3c4\uccad \uc0ac\uc2e4\uc744 \uc740\ud3d0\ud558\uc600\uae30\uc5d0 \ubc1c\uc0dd\ud55c \uc77c\uc774\ub77c\uba70, \ub2a6\uc5c8\uc9c0\ub9cc \uc774 \uc0ac\uc2e4\uc744 \ubc29\uc1a1\ub9e4\uccb4\ub97c \ud1b5\ud558\uc5ec \uc804\uad6d\ubbfc\uc5d0\uac8c \uc54c\ub824\uc57c \ud55c\ub2e4\"\uace0 \ub9d0\ud558\uc600\ub2e4.", "paragraph_id": "6026475-5-0", "paragraph_question": "question: \ub178\uc2e0\uc601\uc740 \ub204\uad6c\uc758 \uc0ac\uc784\uc744 \uc608\uc2dc\ub85c \ub4e4\uba74\uc11c \uace0\ubb38\uce58\uc0ac \uc0ac\uac74 \ubc1c\ud45c\uc758 \ub2f9\uc704\uc131\uc744 \uc8fc\uc7a5\ud588\ub294\uac00?, context: 1987\ub144 1\uc6d4 14\uc77c 6\uc6d4 \ud56d\uc7c1\uc73c\ub85c \ucd09\ubc1c\ub418\ub294 \ubc15\uc885\ucca0 \uace0\ubb38\uce58\uc0ac \uc0ac\uac74\uc774 \ubc1c\uc0dd\ud558\uc600\ub2e4. 1\uc6d4 15\uc77c \uc800\ub141, \uc77c\uac04\uc9c0\ub97c \uc77d\ub358 \ub3c4\uc911 \uad6c\uc11d\uc5d0 \uc870\uadf8\ub9e3\uac8c \uc50c\uc5b4\uc9c4 \uae30\uc0ac\ub97c \uc774\uc0c1\ud558\uac8c \uc5ec\uae34 \ub178\uc2e0\uc601\uc740 \ub2e4\uc74c\ub0a0 \uc544\uce68 \uae40\uc885\ud638 \ub0b4\ubb34\ubd80\uc7a5\uad00 \uacfc \uc815\uc7ac\ucca0 \uc815\ubb34\uc7a5\uad00\uc744 \ucd94\uad81\ud55c\ub2e4. \ucc98\uc74c\uc5d0 \uae40\uc885\ud638\ub294 \uacbd\ucc30\uc774 \ubc1c\ud45c\ud55c \uae30\uc0ac \uadf8\ub300\ub85c\ub77c\uba70 \uac70\uc9d3\uc744 \uace0\ud558\uc600\uc73c\ub098 \uc815\uc7ac\ucca0\uc774 \uc790\ub9ac\ub97c \ub728\uc790 \uace7 \uace0\ubb38\uce58\uc0ac \uc0ac\uac74\uc774\ub77c\uba70 \uc0ac\uc2e4\uc744 \ub9d0\ud55c\ub2e4. \uc774\uc5d0 \uace7 \ub300\ucc45\ud68c\uc758\ub97c \uc18c\uc9d1\ud558\uc5ec \ub2f9\uc2dc \ub178\ud0dc\uc6b0 \ub300\ud45c\uc704\uc6d0\uacfc \ubc15\uc601\uc218 \uccad\uc640\ub300\ube44\uc11c\uc2e4\uc7a5, \uadf8\ub9ac\uace0 \uc7a5\uc138\ub3d9 \uc548\uae30\ubd80\uc7a5, \ub178\uc2e0\uc601 4\uc0ac\ub78c\uc774 \uc774 \ubb38\uc81c\uc5d0 \ub300\ud574 \ud611\uc758\ud558\uc600\uace0 \uc774 \ud68c\uc758\uc5d0\uc11c \ub178\uc2e0\uc601\uc740 \"\uc804\ub0a0\uc758 \ub2c9\uc2a8 \uc804 \ubbf8\uad6d\ub300\ud1b5\ub839\uc758 \uc0ac\uc784\uc774 \ub3c4\uccad \uc0ac\uc2e4\uc744 \uc740\ud3d0\ud558\uc600\uae30\uc5d0 \ubc1c\uc0dd\ud55c \uc77c\uc774\ub77c\uba70, \ub2a6\uc5c8\uc9c0\ub9cc \uc774 \uc0ac\uc2e4\uc744 \ubc29\uc1a1\ub9e4\uccb4\ub97c \ud1b5\ud558\uc5ec \uc804\uad6d\ubbfc\uc5d0\uac8c \uc54c\ub824\uc57c \ud55c\ub2e4\"\uace0 \ub9d0\ud558\uc600\ub2e4. \ud558\uc9c0\ub9cc \ud6c4\uc5d0 \ub610 \ubc94\uc778\ucd95\uc18c \uc870\uc791\uc0ac\uac74\uc774 \ubc1c\uc0dd\ud558\uc600\uace0 \ucd1d\ub9ac\uc778 \uc790\uc2e0\uc774 \ubaa8\ub4e0 \ucc45\uc784\uc744 \uc9c0\uace0 \ud1f4\uc784\uc744 \ud558\uc9c0 \uc54a\uc744\uc2dc\uc5d4 \uadf8 \uc5ec\ud30c\uac00 \uccad\uc640\ub300\uae4c\uc9c0 \ubbf8\uce60 \uc218\ub3c4 \uc788\uc74c\uc744 \uc6b0\ub824\ud558\uc5ec \uc804\ub450\ud658\uc5d0\uac8c \uc0ac\uc758\ub97c \ud45c\uba85\ud55c\ub2e4. \ucc98\uc74c\uc5d4 \uc804\ub450\ud658 \uc5ed\uc2dc \uc784\uae30\uac00 9\uac1c\uc6d4\ubc16\uc5d0 \ub0a8\uc9c0 \uc54a\uc558\uc73c\ub2c8 \uadf8\ub0e5 \uc720\uc784\ud560 \uac83\uc744 \uad8c\uace0\ud558\uc600\uc73c\ub098 \uc5ec\ub860\uc774 \uae09\uc18d\ub3c4\ub85c \uc545\ud654\ub418\uc5b4 \uacb0\uad6d \uadf8\uc758 \uc0ac\uc784\uc744 \uacb0\uc815\ud558\uac8c \ub418\uc5c8\ub2e4."} {"question": "\uc778\uc870\ubc18\uc815 \ud6c4 \uc11c\uc778\uc774 \uc815\uad8c\uc744 \uc7a5\uc545\ud558\uba70 \ucde8\ud55c \uc815\ucc45\uc740?", "paragraph": "\uadf8\ub7ec\ub098 \uad11\ud574\uad70\uc740 \uc644\uc131\uc744 \ubcf4\uc9c0 \ubabb\ud558\uace0 \ud3d0\uc704\ub418\uc5c8\uc73c\uba70, \ub0a8\ud55c\uc0b0\uc131\uc774 \ubcf8\uaca9\uc801\uc73c\ub85c \uc9c0\uae08\uacfc \uac19\uc740 \ud615\ud0dc\ub85c \uc99d\ucd95\ub41c \uac83\uc740 \uc778\uc870 \ub300\uc5d0 \uc774\ub974\ub7ec\uc11c\ub2e4. \uc778\uc870\ubc18\uc815\uc73c\ub85c \uad11\ud574\uad70\uc774 \ubb3c\ub7ec\ub098\uace0 \uc9d1\uad8c\uc138\ub825\uc774 \uad50\uccb4\ub418\uba74\uc11c \uc870\uc120\uc758 \ub300\uc678\uc815\ucc45\uc5d0 \ud070 \ubcc0\ud654\uac00 \ucd08\ub798\ub418\uc5c8\ub2e4. \uc11c\uc778\uc774 \uc815\uad8c\uc744 \uc7a5\uc545\ud558\uba74\uc11c \uce5c\uba85\ubc30\uae08 \uc815\ucc45\uc744 \ucde8\ud55c \uac83\uc774\ub2e4. \ud6c4\uae08\uc774 \uce68\uc785\ud560 \uc218 \uc788\ub2e4\ub294 \uc704\ud5d8\uc131\uc774 \uace0\uc870\ub418\ub294 \uac00\uc6b4\ub370, \ub354\uc6b1\uc774 \uc778\uc870 2\ub144(1624\ub144)\uc5d0 \uc77c\uc5b4\ub09c \uc774\uad04\uc758 \ub09c\uc744 \uacc4\uae30\ub85c \ub3c4\uc131\uc744 \ubc29\uc5b4\ud558\uace0 \uc720\uc0ac\uc2dc \uc655\uc774 \ud53c\uc2e0\ud560 \uc218 \uc788\ub294 \uac70\ucc98\ub85c\uc11c \ub0a8\ud55c\uc0b0\uc131\uc758 \uc911\uc694\uc131\uc774 \ud06c\uac8c \uc99d\ub300\ub418\uc5c8\ub2e4. \uc774\uc5d0 \uc778\uc870\ub294 1624\ub144\ubd80\ud130 1626\ub144\uae4c\uc9c0 2\ub144\uac04 \uacf5\uc0ac\ub97c \ucd94\uc9c4\ud558\uc5ec, \uacf5\uc0ac \uc644\ub8cc \ud6c4 \uad11\uc8fc\ubaa9\uc774 \ub0a8\ud55c\uc0b0\uc131\uc73c\ub85c \uc774\uc804\ud588\uace0, \uc218\uc5b4\uccad\ub3c4 \uac19\uc740 \ud574\uc5d0 \ub0a8\ud55c\uc0b0\uc131\uc5d0 \uc124\uce58\ub418\uc5c8\ub2e4. \uc131\uc758 \ub458\ub808\ub3c4 6,927\ubcf4\uac00 \ub418\uc5c8\ub2e4. \uc655\uc774 \uac70\ucc98\ud560 \ud589\uad81\ub3c4 \uc778\uc870 4\ub144 \uc131\uacfd\uc774 \uc644\uc131\ub420 \ub54c \uac74\ucd95\ub418\uc5b4 \u300a\ub0a8\ud55c\uc9c0\u300b, \u300a\uad11\uc8fc\ubd80\uc74d\uc9c0\u300b\uc5d0 \uc758\ud558\uba74 \uc0c1\uad90\uc774 72\uce78 \ubc18, \ud558\uad90 154\uce78\uc73c\ub85c \ubaa8\ub450 227\uce78\uc774 \uac74\ucd95\ub418\uc5c8\ub2e4.", "answer": "\uce5c\uba85\ubc30\uae08 \uc815\ucc45", "sentence": "\uc11c\uc778\uc774 \uc815\uad8c\uc744 \uc7a5\uc545\ud558\uba74\uc11c \uce5c\uba85\ubc30\uae08 \uc815\ucc45 \uc744 \ucde8\ud55c \uac83\uc774\ub2e4.", "paragraph_sentence": "\uadf8\ub7ec\ub098 \uad11\ud574\uad70\uc740 \uc644\uc131\uc744 \ubcf4\uc9c0 \ubabb\ud558\uace0 \ud3d0\uc704\ub418\uc5c8\uc73c\uba70, \ub0a8\ud55c\uc0b0\uc131\uc774 \ubcf8\uaca9\uc801\uc73c\ub85c \uc9c0\uae08\uacfc \uac19\uc740 \ud615\ud0dc\ub85c \uc99d\ucd95\ub41c \uac83\uc740 \uc778\uc870 \ub300\uc5d0 \uc774\ub974\ub7ec\uc11c\ub2e4. \uc778\uc870\ubc18\uc815\uc73c\ub85c \uad11\ud574\uad70\uc774 \ubb3c\ub7ec\ub098\uace0 \uc9d1\uad8c\uc138\ub825\uc774 \uad50\uccb4\ub418\uba74\uc11c \uc870\uc120\uc758 \ub300\uc678\uc815\ucc45\uc5d0 \ud070 \ubcc0\ud654\uac00 \ucd08\ub798\ub418\uc5c8\ub2e4. \uc11c\uc778\uc774 \uc815\uad8c\uc744 \uc7a5\uc545\ud558\uba74\uc11c \uce5c\uba85\ubc30\uae08 \uc815\ucc45 \uc744 \ucde8\ud55c \uac83\uc774\ub2e4. \ud6c4\uae08\uc774 \uce68\uc785\ud560 \uc218 \uc788\ub2e4\ub294 \uc704\ud5d8\uc131\uc774 \uace0\uc870\ub418\ub294 \uac00\uc6b4\ub370, \ub354\uc6b1\uc774 \uc778\uc870 2\ub144(1624\ub144)\uc5d0 \uc77c\uc5b4\ub09c \uc774\uad04\uc758 \ub09c\uc744 \uacc4\uae30\ub85c \ub3c4\uc131\uc744 \ubc29\uc5b4\ud558\uace0 \uc720\uc0ac\uc2dc \uc655\uc774 \ud53c\uc2e0\ud560 \uc218 \uc788\ub294 \uac70\ucc98\ub85c\uc11c \ub0a8\ud55c\uc0b0\uc131\uc758 \uc911\uc694\uc131\uc774 \ud06c\uac8c \uc99d\ub300\ub418\uc5c8\ub2e4. \uc774\uc5d0 \uc778\uc870\ub294 1624\ub144\ubd80\ud130 1626\ub144\uae4c\uc9c0 2\ub144\uac04 \uacf5\uc0ac\ub97c \ucd94\uc9c4\ud558\uc5ec, \uacf5\uc0ac \uc644\ub8cc \ud6c4 \uad11\uc8fc\ubaa9\uc774 \ub0a8\ud55c\uc0b0\uc131\uc73c\ub85c \uc774\uc804\ud588\uace0, \uc218\uc5b4\uccad\ub3c4 \uac19\uc740 \ud574\uc5d0 \ub0a8\ud55c\uc0b0\uc131\uc5d0 \uc124\uce58\ub418\uc5c8\ub2e4. \uc131\uc758 \ub458\ub808\ub3c4 6,927\ubcf4\uac00 \ub418\uc5c8\ub2e4. \uc655\uc774 \uac70\ucc98\ud560 \ud589\uad81\ub3c4 \uc778\uc870 4\ub144 \uc131\uacfd\uc774 \uc644\uc131\ub420 \ub54c \uac74\ucd95\ub418\uc5b4 \u300a\ub0a8\ud55c\uc9c0\u300b, \u300a\uad11\uc8fc\ubd80\uc74d\uc9c0\u300b\uc5d0 \uc758\ud558\uba74 \uc0c1\uad90\uc774 72\uce78 \ubc18, \ud558\uad90 154\uce78\uc73c\ub85c \ubaa8\ub450 227\uce78\uc774 \uac74\ucd95\ub418\uc5c8\ub2e4.", "paragraph_answer": "\uadf8\ub7ec\ub098 \uad11\ud574\uad70\uc740 \uc644\uc131\uc744 \ubcf4\uc9c0 \ubabb\ud558\uace0 \ud3d0\uc704\ub418\uc5c8\uc73c\uba70, \ub0a8\ud55c\uc0b0\uc131\uc774 \ubcf8\uaca9\uc801\uc73c\ub85c \uc9c0\uae08\uacfc \uac19\uc740 \ud615\ud0dc\ub85c \uc99d\ucd95\ub41c \uac83\uc740 \uc778\uc870 \ub300\uc5d0 \uc774\ub974\ub7ec\uc11c\ub2e4. \uc778\uc870\ubc18\uc815\uc73c\ub85c \uad11\ud574\uad70\uc774 \ubb3c\ub7ec\ub098\uace0 \uc9d1\uad8c\uc138\ub825\uc774 \uad50\uccb4\ub418\uba74\uc11c \uc870\uc120\uc758 \ub300\uc678\uc815\ucc45\uc5d0 \ud070 \ubcc0\ud654\uac00 \ucd08\ub798\ub418\uc5c8\ub2e4. \uc11c\uc778\uc774 \uc815\uad8c\uc744 \uc7a5\uc545\ud558\uba74\uc11c \uce5c\uba85\ubc30\uae08 \uc815\ucc45 \uc744 \ucde8\ud55c \uac83\uc774\ub2e4. \ud6c4\uae08\uc774 \uce68\uc785\ud560 \uc218 \uc788\ub2e4\ub294 \uc704\ud5d8\uc131\uc774 \uace0\uc870\ub418\ub294 \uac00\uc6b4\ub370, \ub354\uc6b1\uc774 \uc778\uc870 2\ub144(1624\ub144)\uc5d0 \uc77c\uc5b4\ub09c \uc774\uad04\uc758 \ub09c\uc744 \uacc4\uae30\ub85c \ub3c4\uc131\uc744 \ubc29\uc5b4\ud558\uace0 \uc720\uc0ac\uc2dc \uc655\uc774 \ud53c\uc2e0\ud560 \uc218 \uc788\ub294 \uac70\ucc98\ub85c\uc11c \ub0a8\ud55c\uc0b0\uc131\uc758 \uc911\uc694\uc131\uc774 \ud06c\uac8c \uc99d\ub300\ub418\uc5c8\ub2e4. \uc774\uc5d0 \uc778\uc870\ub294 1624\ub144\ubd80\ud130 1626\ub144\uae4c\uc9c0 2\ub144\uac04 \uacf5\uc0ac\ub97c \ucd94\uc9c4\ud558\uc5ec, \uacf5\uc0ac \uc644\ub8cc \ud6c4 \uad11\uc8fc\ubaa9\uc774 \ub0a8\ud55c\uc0b0\uc131\uc73c\ub85c \uc774\uc804\ud588\uace0, \uc218\uc5b4\uccad\ub3c4 \uac19\uc740 \ud574\uc5d0 \ub0a8\ud55c\uc0b0\uc131\uc5d0 \uc124\uce58\ub418\uc5c8\ub2e4. \uc131\uc758 \ub458\ub808\ub3c4 6,927\ubcf4\uac00 \ub418\uc5c8\ub2e4. \uc655\uc774 \uac70\ucc98\ud560 \ud589\uad81\ub3c4 \uc778\uc870 4\ub144 \uc131\uacfd\uc774 \uc644\uc131\ub420 \ub54c \uac74\ucd95\ub418\uc5b4 \u300a\ub0a8\ud55c\uc9c0\u300b, \u300a\uad11\uc8fc\ubd80\uc74d\uc9c0\u300b\uc5d0 \uc758\ud558\uba74 \uc0c1\uad90\uc774 72\uce78 \ubc18, \ud558\uad90 154\uce78\uc73c\ub85c \ubaa8\ub450 227\uce78\uc774 \uac74\ucd95\ub418\uc5c8\ub2e4.", "sentence_answer": "\uc11c\uc778\uc774 \uc815\uad8c\uc744 \uc7a5\uc545\ud558\uba74\uc11c \uce5c\uba85\ubc30\uae08 \uc815\ucc45 \uc744 \ucde8\ud55c \uac83\uc774\ub2e4.", "paragraph_id": "6022976-10-2", "paragraph_question": "question: \uc778\uc870\ubc18\uc815 \ud6c4 \uc11c\uc778\uc774 \uc815\uad8c\uc744 \uc7a5\uc545\ud558\uba70 \ucde8\ud55c \uc815\ucc45\uc740?, context: \uadf8\ub7ec\ub098 \uad11\ud574\uad70\uc740 \uc644\uc131\uc744 \ubcf4\uc9c0 \ubabb\ud558\uace0 \ud3d0\uc704\ub418\uc5c8\uc73c\uba70, \ub0a8\ud55c\uc0b0\uc131\uc774 \ubcf8\uaca9\uc801\uc73c\ub85c \uc9c0\uae08\uacfc \uac19\uc740 \ud615\ud0dc\ub85c \uc99d\ucd95\ub41c \uac83\uc740 \uc778\uc870 \ub300\uc5d0 \uc774\ub974\ub7ec\uc11c\ub2e4. \uc778\uc870\ubc18\uc815\uc73c\ub85c \uad11\ud574\uad70\uc774 \ubb3c\ub7ec\ub098\uace0 \uc9d1\uad8c\uc138\ub825\uc774 \uad50\uccb4\ub418\uba74\uc11c \uc870\uc120\uc758 \ub300\uc678\uc815\ucc45\uc5d0 \ud070 \ubcc0\ud654\uac00 \ucd08\ub798\ub418\uc5c8\ub2e4. \uc11c\uc778\uc774 \uc815\uad8c\uc744 \uc7a5\uc545\ud558\uba74\uc11c \uce5c\uba85\ubc30\uae08 \uc815\ucc45\uc744 \ucde8\ud55c \uac83\uc774\ub2e4. \ud6c4\uae08\uc774 \uce68\uc785\ud560 \uc218 \uc788\ub2e4\ub294 \uc704\ud5d8\uc131\uc774 \uace0\uc870\ub418\ub294 \uac00\uc6b4\ub370, \ub354\uc6b1\uc774 \uc778\uc870 2\ub144(1624\ub144)\uc5d0 \uc77c\uc5b4\ub09c \uc774\uad04\uc758 \ub09c\uc744 \uacc4\uae30\ub85c \ub3c4\uc131\uc744 \ubc29\uc5b4\ud558\uace0 \uc720\uc0ac\uc2dc \uc655\uc774 \ud53c\uc2e0\ud560 \uc218 \uc788\ub294 \uac70\ucc98\ub85c\uc11c \ub0a8\ud55c\uc0b0\uc131\uc758 \uc911\uc694\uc131\uc774 \ud06c\uac8c \uc99d\ub300\ub418\uc5c8\ub2e4. \uc774\uc5d0 \uc778\uc870\ub294 1624\ub144\ubd80\ud130 1626\ub144\uae4c\uc9c0 2\ub144\uac04 \uacf5\uc0ac\ub97c \ucd94\uc9c4\ud558\uc5ec, \uacf5\uc0ac \uc644\ub8cc \ud6c4 \uad11\uc8fc\ubaa9\uc774 \ub0a8\ud55c\uc0b0\uc131\uc73c\ub85c \uc774\uc804\ud588\uace0, \uc218\uc5b4\uccad\ub3c4 \uac19\uc740 \ud574\uc5d0 \ub0a8\ud55c\uc0b0\uc131\uc5d0 \uc124\uce58\ub418\uc5c8\ub2e4. \uc131\uc758 \ub458\ub808\ub3c4 6,927\ubcf4\uac00 \ub418\uc5c8\ub2e4. \uc655\uc774 \uac70\ucc98\ud560 \ud589\uad81\ub3c4 \uc778\uc870 4\ub144 \uc131\uacfd\uc774 \uc644\uc131\ub420 \ub54c \uac74\ucd95\ub418\uc5b4 \u300a\ub0a8\ud55c\uc9c0\u300b, \u300a\uad11\uc8fc\ubd80\uc74d\uc9c0\u300b\uc5d0 \uc758\ud558\uba74 \uc0c1\uad90\uc774 72\uce78 \ubc18, \ud558\uad90 154\uce78\uc73c\ub85c \ubaa8\ub450 227\uce78\uc774 \uac74\ucd95\ub418\uc5c8\ub2e4."} {"question": "BMP-2\uc5d0\uc11c \uc4f0\uc774\ub294 \uae30\uad00\ud3ec\uc758 \uc0ac\uc774\uc988\ub294?", "paragraph": "\ucd08\ucc3d\uae30 \ub4f1\uc7a5\ud55c IFV\ub294 \uae30\uacc4\ud654\ubcf4\ubcd1\uc774 \ucc28\ub7c9 \ub0b4\ubd80\uc5d0\uc11c \uc0ac\uaca9\ud560 \uc218 \uc788\ub3c4\ub85d \ucd1d\uc548\uad6c(Gun Ports)\ub97c \ubcf4\ubcd1 \ud0d1\uc2b9\uad6c\uc5ed\uc5d0 \uc124\uce58\ud588\ub2e4. IFV\ub294 \uacf5\uaca9 \uc911\uc5d0 \uc9c1\uc811 \ud654\ub825 \uc9c0\uc6d0\uc744 \uc81c\uacf5\ud560 \uc218 \uc788\ub294 \uc911\ubb34\uc7a5\uc744 \uac16\ucdc4\uace0, \ubcf4\ubcd1\uc774 \uac1c\uc778 \ud654\uae30\ub97c \uac70\uce58\ud558\uace0 \uc0ac\uaca9\ud560 \uc218 \uc788\ub294 \ucd1d\uc548\uad6c(Gun Ports; Pistol Ports)\ub97c \uc124\uce58\ud558\uace0 \uc7a5\uac11\uc774 \uac15\ud654\ub418\uc5c8\ub2e4\ub294 \uc810\uc5d0\uc11c \ucd08\uae30 APC\ub4e4\uacfc \ub2ec\ub790\ub2e4. \ubbf8\uad6d, \uc601\uad6d, \ub3c5\uc77c, \ud504\ub791\uc2a4 \ub4f1\uc758 IFV\ub4e4\uc740 \ub300\uac1c 20mm \uc774\uc0c1\uc758 \uae30\uad00\ud3ec\uc640 \ub300\uc804\ucc28\ubbf8\uc0ac\uc77c\uc744 \uc7a5\ucc29\ud588\uc73c\uba70, \uc18c\ub828\uc740 BMP \uc2dc\ub9ac\uc988\uc758 \uac01 \ubb34\uc7a5\uc774 \ubaa8\ub450 \ub2ec\ub77c \uacbd\uc804\ucc28 \uc218\uc900\uc758 \ubb34\uc7a5\uc744 \uc7a5\ucc29\ud588\ub2e4. BMP-1\uc740 73mm \uc800\uc555\ud3ec\uc640 7.62mm PKT \uae30\uad00\ucd1d, AT-3 \uc0c8\uac70/AT-4 \uc2a4\ud53c\uac13/AT-5 \uc2a4\ud310\ub4dc\ub810 \ub300\uc804\ucc28\ubbf8\uc0ac\uc77c\uc744 \uc7a5\ucc29\ud588\uc73c\uba70, BMP-2\ub294 30mm \uae30\uad00\ud3ec\uc640 \ub3d9\uc77c\ud55c \ub300\uc804\ucc28\ubbf8\uc0ac\uc77c\uc744 \uc7a5\ucc29\ud588\ub2e4. BMP-3\uc740 \uac00\uc7a5 \uc911\ubb34\uc7a5\uc744 \uc790\ub791\ud558\ub294\ub370 100mm \uc800\uc555\ud3ec\uc640 30mm \uae30\uad00\ud3ec \ubc0f \ub3d9\ucd95 \uae30\uad00\ucd1d\uc744 \uc7a5\ucc29\ud588\ub2e4. \ubcf4\ubcd1\uc804\ud22c\ucc28\ub294 \ub300\uac1c \uada4\ub3c4\uc2dd\uc774\uc9c0\ub9cc, \ubc14\ud034\uc2dd \uc77c\ubd80\ub3c4 \uc774 \ubc94\uc8fc\uc5d0 \uc18d\ud558\ub294 \uacbd\uc6b0\uac00 \uc788\ub2e4.", "answer": "30mm", "sentence": "BMP-2\ub294 30mm \uae30\uad00\ud3ec\uc640 \ub3d9\uc77c\ud55c \ub300\uc804\ucc28\ubbf8\uc0ac\uc77c\uc744 \uc7a5\ucc29\ud588\ub2e4.", "paragraph_sentence": "\ucd08\ucc3d\uae30 \ub4f1\uc7a5\ud55c IFV\ub294 \uae30\uacc4\ud654\ubcf4\ubcd1\uc774 \ucc28\ub7c9 \ub0b4\ubd80\uc5d0\uc11c \uc0ac\uaca9\ud560 \uc218 \uc788\ub3c4\ub85d \ucd1d\uc548\uad6c(Gun Ports)\ub97c \ubcf4\ubcd1 \ud0d1\uc2b9\uad6c\uc5ed\uc5d0 \uc124\uce58\ud588\ub2e4. IFV\ub294 \uacf5\uaca9 \uc911\uc5d0 \uc9c1\uc811 \ud654\ub825 \uc9c0\uc6d0\uc744 \uc81c\uacf5\ud560 \uc218 \uc788\ub294 \uc911\ubb34\uc7a5\uc744 \uac16\ucdc4\uace0, \ubcf4\ubcd1\uc774 \uac1c\uc778 \ud654\uae30\ub97c \uac70\uce58\ud558\uace0 \uc0ac\uaca9\ud560 \uc218 \uc788\ub294 \ucd1d\uc548\uad6c(Gun Ports; Pistol Ports)\ub97c \uc124\uce58\ud558\uace0 \uc7a5\uac11\uc774 \uac15\ud654\ub418\uc5c8\ub2e4\ub294 \uc810\uc5d0\uc11c \ucd08\uae30 APC\ub4e4\uacfc \ub2ec\ub790\ub2e4. \ubbf8\uad6d, \uc601\uad6d, \ub3c5\uc77c, \ud504\ub791\uc2a4 \ub4f1\uc758 IFV\ub4e4\uc740 \ub300\uac1c 20mm \uc774\uc0c1\uc758 \uae30\uad00\ud3ec\uc640 \ub300\uc804\ucc28\ubbf8\uc0ac\uc77c\uc744 \uc7a5\ucc29\ud588\uc73c\uba70, \uc18c\ub828\uc740 BMP \uc2dc\ub9ac\uc988\uc758 \uac01 \ubb34\uc7a5\uc774 \ubaa8\ub450 \ub2ec\ub77c \uacbd\uc804\ucc28 \uc218\uc900\uc758 \ubb34\uc7a5\uc744 \uc7a5\ucc29\ud588\ub2e4. BMP-1\uc740 73mm \uc800\uc555\ud3ec\uc640 7.62mm PKT \uae30\uad00\ucd1d, AT-3 \uc0c8\uac70/AT-4 \uc2a4\ud53c\uac13/AT-5 \uc2a4\ud310\ub4dc\ub810 \ub300\uc804\ucc28\ubbf8\uc0ac\uc77c\uc744 \uc7a5\ucc29\ud588\uc73c\uba70, BMP-2\ub294 30mm \uae30\uad00\ud3ec\uc640 \ub3d9\uc77c\ud55c \ub300\uc804\ucc28\ubbf8\uc0ac\uc77c\uc744 \uc7a5\ucc29\ud588\ub2e4. BMP-3\uc740 \uac00\uc7a5 \uc911\ubb34\uc7a5\uc744 \uc790\ub791\ud558\ub294\ub370 100mm \uc800\uc555\ud3ec\uc640 30mm \uae30\uad00\ud3ec \ubc0f \ub3d9\ucd95 \uae30\uad00\ucd1d\uc744 \uc7a5\ucc29\ud588\ub2e4. \ubcf4\ubcd1\uc804\ud22c\ucc28\ub294 \ub300\uac1c \uada4\ub3c4\uc2dd\uc774\uc9c0\ub9cc, \ubc14\ud034\uc2dd \uc77c\ubd80\ub3c4 \uc774 \ubc94\uc8fc\uc5d0 \uc18d\ud558\ub294 \uacbd\uc6b0\uac00 \uc788\ub2e4.", "paragraph_answer": "\ucd08\ucc3d\uae30 \ub4f1\uc7a5\ud55c IFV\ub294 \uae30\uacc4\ud654\ubcf4\ubcd1\uc774 \ucc28\ub7c9 \ub0b4\ubd80\uc5d0\uc11c \uc0ac\uaca9\ud560 \uc218 \uc788\ub3c4\ub85d \ucd1d\uc548\uad6c(Gun Ports)\ub97c \ubcf4\ubcd1 \ud0d1\uc2b9\uad6c\uc5ed\uc5d0 \uc124\uce58\ud588\ub2e4. IFV\ub294 \uacf5\uaca9 \uc911\uc5d0 \uc9c1\uc811 \ud654\ub825 \uc9c0\uc6d0\uc744 \uc81c\uacf5\ud560 \uc218 \uc788\ub294 \uc911\ubb34\uc7a5\uc744 \uac16\ucdc4\uace0, \ubcf4\ubcd1\uc774 \uac1c\uc778 \ud654\uae30\ub97c \uac70\uce58\ud558\uace0 \uc0ac\uaca9\ud560 \uc218 \uc788\ub294 \ucd1d\uc548\uad6c(Gun Ports; Pistol Ports)\ub97c \uc124\uce58\ud558\uace0 \uc7a5\uac11\uc774 \uac15\ud654\ub418\uc5c8\ub2e4\ub294 \uc810\uc5d0\uc11c \ucd08\uae30 APC\ub4e4\uacfc \ub2ec\ub790\ub2e4. \ubbf8\uad6d, \uc601\uad6d, \ub3c5\uc77c, \ud504\ub791\uc2a4 \ub4f1\uc758 IFV\ub4e4\uc740 \ub300\uac1c 20mm \uc774\uc0c1\uc758 \uae30\uad00\ud3ec\uc640 \ub300\uc804\ucc28\ubbf8\uc0ac\uc77c\uc744 \uc7a5\ucc29\ud588\uc73c\uba70, \uc18c\ub828\uc740 BMP \uc2dc\ub9ac\uc988\uc758 \uac01 \ubb34\uc7a5\uc774 \ubaa8\ub450 \ub2ec\ub77c \uacbd\uc804\ucc28 \uc218\uc900\uc758 \ubb34\uc7a5\uc744 \uc7a5\ucc29\ud588\ub2e4. BMP-1\uc740 73mm \uc800\uc555\ud3ec\uc640 7.62mm PKT \uae30\uad00\ucd1d, AT-3 \uc0c8\uac70/AT-4 \uc2a4\ud53c\uac13/AT-5 \uc2a4\ud310\ub4dc\ub810 \ub300\uc804\ucc28\ubbf8\uc0ac\uc77c\uc744 \uc7a5\ucc29\ud588\uc73c\uba70, BMP-2\ub294 30mm \uae30\uad00\ud3ec\uc640 \ub3d9\uc77c\ud55c \ub300\uc804\ucc28\ubbf8\uc0ac\uc77c\uc744 \uc7a5\ucc29\ud588\ub2e4. BMP-3\uc740 \uac00\uc7a5 \uc911\ubb34\uc7a5\uc744 \uc790\ub791\ud558\ub294\ub370 100mm \uc800\uc555\ud3ec\uc640 30mm \uae30\uad00\ud3ec \ubc0f \ub3d9\ucd95 \uae30\uad00\ucd1d\uc744 \uc7a5\ucc29\ud588\ub2e4. \ubcf4\ubcd1\uc804\ud22c\ucc28\ub294 \ub300\uac1c \uada4\ub3c4\uc2dd\uc774\uc9c0\ub9cc, \ubc14\ud034\uc2dd \uc77c\ubd80\ub3c4 \uc774 \ubc94\uc8fc\uc5d0 \uc18d\ud558\ub294 \uacbd\uc6b0\uac00 \uc788\ub2e4.", "sentence_answer": "BMP-2\ub294 30mm \uae30\uad00\ud3ec\uc640 \ub3d9\uc77c\ud55c \ub300\uc804\ucc28\ubbf8\uc0ac\uc77c\uc744 \uc7a5\ucc29\ud588\ub2e4.", "paragraph_id": "6480804-3-2", "paragraph_question": "question: BMP-2\uc5d0\uc11c \uc4f0\uc774\ub294 \uae30\uad00\ud3ec\uc758 \uc0ac\uc774\uc988\ub294?, context: \ucd08\ucc3d\uae30 \ub4f1\uc7a5\ud55c IFV\ub294 \uae30\uacc4\ud654\ubcf4\ubcd1\uc774 \ucc28\ub7c9 \ub0b4\ubd80\uc5d0\uc11c \uc0ac\uaca9\ud560 \uc218 \uc788\ub3c4\ub85d \ucd1d\uc548\uad6c(Gun Ports)\ub97c \ubcf4\ubcd1 \ud0d1\uc2b9\uad6c\uc5ed\uc5d0 \uc124\uce58\ud588\ub2e4. IFV\ub294 \uacf5\uaca9 \uc911\uc5d0 \uc9c1\uc811 \ud654\ub825 \uc9c0\uc6d0\uc744 \uc81c\uacf5\ud560 \uc218 \uc788\ub294 \uc911\ubb34\uc7a5\uc744 \uac16\ucdc4\uace0, \ubcf4\ubcd1\uc774 \uac1c\uc778 \ud654\uae30\ub97c \uac70\uce58\ud558\uace0 \uc0ac\uaca9\ud560 \uc218 \uc788\ub294 \ucd1d\uc548\uad6c(Gun Ports; Pistol Ports)\ub97c \uc124\uce58\ud558\uace0 \uc7a5\uac11\uc774 \uac15\ud654\ub418\uc5c8\ub2e4\ub294 \uc810\uc5d0\uc11c \ucd08\uae30 APC\ub4e4\uacfc \ub2ec\ub790\ub2e4. \ubbf8\uad6d, \uc601\uad6d, \ub3c5\uc77c, \ud504\ub791\uc2a4 \ub4f1\uc758 IFV\ub4e4\uc740 \ub300\uac1c 20mm \uc774\uc0c1\uc758 \uae30\uad00\ud3ec\uc640 \ub300\uc804\ucc28\ubbf8\uc0ac\uc77c\uc744 \uc7a5\ucc29\ud588\uc73c\uba70, \uc18c\ub828\uc740 BMP \uc2dc\ub9ac\uc988\uc758 \uac01 \ubb34\uc7a5\uc774 \ubaa8\ub450 \ub2ec\ub77c \uacbd\uc804\ucc28 \uc218\uc900\uc758 \ubb34\uc7a5\uc744 \uc7a5\ucc29\ud588\ub2e4. BMP-1\uc740 73mm \uc800\uc555\ud3ec\uc640 7.62mm PKT \uae30\uad00\ucd1d, AT-3 \uc0c8\uac70/AT-4 \uc2a4\ud53c\uac13/AT-5 \uc2a4\ud310\ub4dc\ub810 \ub300\uc804\ucc28\ubbf8\uc0ac\uc77c\uc744 \uc7a5\ucc29\ud588\uc73c\uba70, BMP-2\ub294 30mm \uae30\uad00\ud3ec\uc640 \ub3d9\uc77c\ud55c \ub300\uc804\ucc28\ubbf8\uc0ac\uc77c\uc744 \uc7a5\ucc29\ud588\ub2e4. BMP-3\uc740 \uac00\uc7a5 \uc911\ubb34\uc7a5\uc744 \uc790\ub791\ud558\ub294\ub370 100mm \uc800\uc555\ud3ec\uc640 30mm \uae30\uad00\ud3ec \ubc0f \ub3d9\ucd95 \uae30\uad00\ucd1d\uc744 \uc7a5\ucc29\ud588\ub2e4. \ubcf4\ubcd1\uc804\ud22c\ucc28\ub294 \ub300\uac1c \uada4\ub3c4\uc2dd\uc774\uc9c0\ub9cc, \ubc14\ud034\uc2dd \uc77c\ubd80\ub3c4 \uc774 \ubc94\uc8fc\uc5d0 \uc18d\ud558\ub294 \uacbd\uc6b0\uac00 \uc788\ub2e4."} {"question": "\uc601\uad6d\uc774 \ub2e4\ub978 \ub098\ub77c\ub4e4\ubcf4\ub2e4 \uc55e\uc11c \uacbd\uc81c \ubc1c\uc804\uc744 \uc774\ub8f0 \uc218 \uc788\uc5c8\ub358 \uc774\uc720\ub294 \ubb34\uc5c7\uc774\uc5c8\ub294\uac00?", "paragraph": "\uc774 \ub098\ub77c\ub294 \ub2e4\ub978 \uc5ec\ub7ec \ub098\ub77c\uc5d0 \uc55e\uc11c\uc11c \uc0b0\uc5c5\ud601\uba85\uc744 \uc131\ucde8\ud558\uc5ec \uc624\ub7ab\ub3d9\uc548 \uc138\uacc4 \uacbd\uc81c\uc5d0 \uad70\ub9bc\ud574 \uc654\ub2e4. \uadf8\ub7ec\ub098 20\uc138\uae30\uc5d0 \ub4e4\uc5b4\uc640\uc11c\ub294 \uc720\ub7fd \ub300\ub959\uacfc \ubd81\uc544\uba54\ub9ac\uce74\uc758 \uacf5\uc5c5\uc774 \uae09\uc18d\ub3c4\ub85c \ubc1c\ub2ec\ud558\uc5ec 1\ucc28\ub300\uc804 \uc774\ud6c4 \uc138\uacc4 \uacbd\uc81c\uc5d0 \uc788\uc5b4\uc11c\uc758 \uc9c0\uc704\ub294 \ucc28\ucc28 \uc800\ud558\ud588\ub2e4. \ud574\uc678 \ud22c\uc790\ub85c\ubd80\ud130\uc758 \uc218\uc785\uc73c\ub85c \uacbd\uc81c\uc758 \uc548\uc815\uc744 \uaf80\ud588\uc73c\ub098 2\ucc28\ub300\uc804\uc5d0 \uc758\ud574 \ud070 \ud0c0\uaca9\uc744 \ubc1b\uace0 \uc7ac\ud3b8\uc131\ud558\uae30\uc5d0 \uc774\ub974\ub800\ub2e4. \uadf8\uac83\uc740 \uae30\uac04\uc0b0\uc5c5\uc758 \uad6d\uc720\ud654\ub97c \ube44\ub86f\ud55c \uacc4\ud68d\uacbd\uc81c\uc758 \ub3c4\uc785\uc73c\ub85c \uad6d\uc81c\uc218\uc9c0\ub97c \uac1c\uc120\ud558\uace0 \uc0ac\ud68c\ubcf4\uc7a5\uc81c\ub3c4\ub97c \ud655\ucda9\ud558\uc5ec \ubcf5\uc9c0 \uad6d\uac00\uc758 \uc2e4\ud604\uc744 \uaf80\ud558\ub294 \uac83\uc744 \uc8fc\uc694 \ubaa9\uc801\uc73c\ub85c \ud558\ub294 \uacbd\uc81c \uccb4\uc81c\uc774\ub2e4. \uadf8\ub7ec\ub098 \ub192\uc740 \uace0\uc6a9\uc218\uc900\uc744 \uc720\uc9c0\ud558\uba74\uc11c\ub3c4 \ub300\uc678\uc801\uc73c\ub85c\ub294 \uad6d\uc81c \uc218\uc9c0\uc758 \uc545\ud654, \ub300\ub0b4\uc801\uc73c\ub85c\ub294 \ucf54\uc2a4\ud2b8 \uc778\ud50c\ub808\uc774\uc158\uc5d0 \uc758\ud55c \uad34\ub85c\uc6c0\uc744 \ub2f9\ud558\uace0 \uc788\ub2e4. \uc774\ub7ec\ud55c \uc0c1\ud669\uc744 \ud0c0\uac1c\ud558\uae30 \uc704\ud574 \uc601\uad6d\uc740 1961\ub144\ubd80\ud130 \uc720\ub7fd\uacbd\uc81c\uacf5\ub3d9\uccb4(EEC) \uac00\uc785\uc744 \uc2e0\uccad\ud588\uc73c\ub098 \ud504\ub791\uc2a4\uc758 \uac70\ubd80\ub85c \uc2e4\ud604\uc744 \ubcf4\uc9c0 \ubabb\ud558\ub2e4\uac00 1971\ub144 6\uc6d4 \ub9c8\uce68\ub0b4 \uc218\ucd9c\uc785\uc815\ucc45 \ubc0f \ud30c\uc6b4\ub4dc\ud654(\u8ca8) \ubb38\uc81c\uc5d0 \ub9e4\ub4ed\uc744 \uc9d3\uace0 \uc601\u00b7\ubd88 \uc591\uad6d\uac04\uc5d0 \ucd5c\uc885 \ud569\uc758\uc5d0 \ub3c4\ub2ec\ud568\uc73c\ub85c\uc368 1973\ub144 1\uc6d4\ubd80\ud130 \uc815\uc2dd \uac00\uc785(\uacbd\uacfc\uae30\uac04 5\ub144\uc744 \uacc4\uc0b0\ud558\uba74 \uc2e4\uc9c8\uc801\uc778 \uac00\uc785\uc740 1978\ub144\ubd80\ud130\uac00 \ub428)\ud558\uac8c \ub418\uc5c8\ub2e4.", "answer": "\uc0b0\uc5c5\ud601\uba85", "sentence": "\uc774 \ub098\ub77c\ub294 \ub2e4\ub978 \uc5ec\ub7ec \ub098\ub77c\uc5d0 \uc55e\uc11c\uc11c \uc0b0\uc5c5\ud601\uba85 \uc744 \uc131\ucde8\ud558\uc5ec \uc624\ub7ab\ub3d9\uc548 \uc138\uacc4 \uacbd\uc81c\uc5d0 \uad70\ub9bc\ud574 \uc654\ub2e4.", "paragraph_sentence": " \uc774 \ub098\ub77c\ub294 \ub2e4\ub978 \uc5ec\ub7ec \ub098\ub77c\uc5d0 \uc55e\uc11c\uc11c \uc0b0\uc5c5\ud601\uba85 \uc744 \uc131\ucde8\ud558\uc5ec \uc624\ub7ab\ub3d9\uc548 \uc138\uacc4 \uacbd\uc81c\uc5d0 \uad70\ub9bc\ud574 \uc654\ub2e4. \uadf8\ub7ec\ub098 20\uc138\uae30\uc5d0 \ub4e4\uc5b4\uc640\uc11c\ub294 \uc720\ub7fd \ub300\ub959\uacfc \ubd81\uc544\uba54\ub9ac\uce74\uc758 \uacf5\uc5c5\uc774 \uae09\uc18d\ub3c4\ub85c \ubc1c\ub2ec\ud558\uc5ec 1\ucc28\ub300\uc804 \uc774\ud6c4 \uc138\uacc4 \uacbd\uc81c\uc5d0 \uc788\uc5b4\uc11c\uc758 \uc9c0\uc704\ub294 \ucc28\ucc28 \uc800\ud558\ud588\ub2e4. \ud574\uc678 \ud22c\uc790\ub85c\ubd80\ud130\uc758 \uc218\uc785\uc73c\ub85c \uacbd\uc81c\uc758 \uc548\uc815\uc744 \uaf80\ud588\uc73c\ub098 2\ucc28\ub300\uc804\uc5d0 \uc758\ud574 \ud070 \ud0c0\uaca9\uc744 \ubc1b\uace0 \uc7ac\ud3b8\uc131\ud558\uae30\uc5d0 \uc774\ub974\ub800\ub2e4. \uadf8\uac83\uc740 \uae30\uac04\uc0b0\uc5c5\uc758 \uad6d\uc720\ud654\ub97c \ube44\ub86f\ud55c \uacc4\ud68d\uacbd\uc81c\uc758 \ub3c4\uc785\uc73c\ub85c \uad6d\uc81c\uc218\uc9c0\ub97c \uac1c\uc120\ud558\uace0 \uc0ac\ud68c\ubcf4\uc7a5\uc81c\ub3c4\ub97c \ud655\ucda9\ud558\uc5ec \ubcf5\uc9c0 \uad6d\uac00\uc758 \uc2e4\ud604\uc744 \uaf80\ud558\ub294 \uac83\uc744 \uc8fc\uc694 \ubaa9\uc801\uc73c\ub85c \ud558\ub294 \uacbd\uc81c \uccb4\uc81c\uc774\ub2e4. \uadf8\ub7ec\ub098 \ub192\uc740 \uace0\uc6a9\uc218\uc900\uc744 \uc720\uc9c0\ud558\uba74\uc11c\ub3c4 \ub300\uc678\uc801\uc73c\ub85c\ub294 \uad6d\uc81c \uc218\uc9c0\uc758 \uc545\ud654, \ub300\ub0b4\uc801\uc73c\ub85c\ub294 \ucf54\uc2a4\ud2b8 \uc778\ud50c\ub808\uc774\uc158\uc5d0 \uc758\ud55c \uad34\ub85c\uc6c0\uc744 \ub2f9\ud558\uace0 \uc788\ub2e4. \uc774\ub7ec\ud55c \uc0c1\ud669\uc744 \ud0c0\uac1c\ud558\uae30 \uc704\ud574 \uc601\uad6d\uc740 1961\ub144\ubd80\ud130 \uc720\ub7fd\uacbd\uc81c\uacf5\ub3d9\uccb4(EEC) \uac00\uc785\uc744 \uc2e0\uccad\ud588\uc73c\ub098 \ud504\ub791\uc2a4\uc758 \uac70\ubd80\ub85c \uc2e4\ud604\uc744 \ubcf4\uc9c0 \ubabb\ud558\ub2e4\uac00 1971\ub144 6\uc6d4 \ub9c8\uce68\ub0b4 \uc218\ucd9c\uc785\uc815\ucc45 \ubc0f \ud30c\uc6b4\ub4dc\ud654(\u8ca8) \ubb38\uc81c\uc5d0 \ub9e4\ub4ed\uc744 \uc9d3\uace0 \uc601\u00b7\ubd88 \uc591\uad6d\uac04\uc5d0 \ucd5c\uc885 \ud569\uc758\uc5d0 \ub3c4\ub2ec\ud568\uc73c\ub85c\uc368 1973\ub144 1\uc6d4\ubd80\ud130 \uc815\uc2dd \uac00\uc785(\uacbd\uacfc\uae30\uac04 5\ub144\uc744 \uacc4\uc0b0\ud558\uba74 \uc2e4\uc9c8\uc801\uc778 \uac00\uc785\uc740 1978\ub144\ubd80\ud130\uac00 \ub428)\ud558\uac8c \ub418\uc5c8\ub2e4.", "paragraph_answer": "\uc774 \ub098\ub77c\ub294 \ub2e4\ub978 \uc5ec\ub7ec \ub098\ub77c\uc5d0 \uc55e\uc11c\uc11c \uc0b0\uc5c5\ud601\uba85 \uc744 \uc131\ucde8\ud558\uc5ec \uc624\ub7ab\ub3d9\uc548 \uc138\uacc4 \uacbd\uc81c\uc5d0 \uad70\ub9bc\ud574 \uc654\ub2e4. \uadf8\ub7ec\ub098 20\uc138\uae30\uc5d0 \ub4e4\uc5b4\uc640\uc11c\ub294 \uc720\ub7fd \ub300\ub959\uacfc \ubd81\uc544\uba54\ub9ac\uce74\uc758 \uacf5\uc5c5\uc774 \uae09\uc18d\ub3c4\ub85c \ubc1c\ub2ec\ud558\uc5ec 1\ucc28\ub300\uc804 \uc774\ud6c4 \uc138\uacc4 \uacbd\uc81c\uc5d0 \uc788\uc5b4\uc11c\uc758 \uc9c0\uc704\ub294 \ucc28\ucc28 \uc800\ud558\ud588\ub2e4. \ud574\uc678 \ud22c\uc790\ub85c\ubd80\ud130\uc758 \uc218\uc785\uc73c\ub85c \uacbd\uc81c\uc758 \uc548\uc815\uc744 \uaf80\ud588\uc73c\ub098 2\ucc28\ub300\uc804\uc5d0 \uc758\ud574 \ud070 \ud0c0\uaca9\uc744 \ubc1b\uace0 \uc7ac\ud3b8\uc131\ud558\uae30\uc5d0 \uc774\ub974\ub800\ub2e4. \uadf8\uac83\uc740 \uae30\uac04\uc0b0\uc5c5\uc758 \uad6d\uc720\ud654\ub97c \ube44\ub86f\ud55c \uacc4\ud68d\uacbd\uc81c\uc758 \ub3c4\uc785\uc73c\ub85c \uad6d\uc81c\uc218\uc9c0\ub97c \uac1c\uc120\ud558\uace0 \uc0ac\ud68c\ubcf4\uc7a5\uc81c\ub3c4\ub97c \ud655\ucda9\ud558\uc5ec \ubcf5\uc9c0 \uad6d\uac00\uc758 \uc2e4\ud604\uc744 \uaf80\ud558\ub294 \uac83\uc744 \uc8fc\uc694 \ubaa9\uc801\uc73c\ub85c \ud558\ub294 \uacbd\uc81c \uccb4\uc81c\uc774\ub2e4. \uadf8\ub7ec\ub098 \ub192\uc740 \uace0\uc6a9\uc218\uc900\uc744 \uc720\uc9c0\ud558\uba74\uc11c\ub3c4 \ub300\uc678\uc801\uc73c\ub85c\ub294 \uad6d\uc81c \uc218\uc9c0\uc758 \uc545\ud654, \ub300\ub0b4\uc801\uc73c\ub85c\ub294 \ucf54\uc2a4\ud2b8 \uc778\ud50c\ub808\uc774\uc158\uc5d0 \uc758\ud55c \uad34\ub85c\uc6c0\uc744 \ub2f9\ud558\uace0 \uc788\ub2e4. \uc774\ub7ec\ud55c \uc0c1\ud669\uc744 \ud0c0\uac1c\ud558\uae30 \uc704\ud574 \uc601\uad6d\uc740 1961\ub144\ubd80\ud130 \uc720\ub7fd\uacbd\uc81c\uacf5\ub3d9\uccb4(EEC) \uac00\uc785\uc744 \uc2e0\uccad\ud588\uc73c\ub098 \ud504\ub791\uc2a4\uc758 \uac70\ubd80\ub85c \uc2e4\ud604\uc744 \ubcf4\uc9c0 \ubabb\ud558\ub2e4\uac00 1971\ub144 6\uc6d4 \ub9c8\uce68\ub0b4 \uc218\ucd9c\uc785\uc815\ucc45 \ubc0f \ud30c\uc6b4\ub4dc\ud654(\u8ca8) \ubb38\uc81c\uc5d0 \ub9e4\ub4ed\uc744 \uc9d3\uace0 \uc601\u00b7\ubd88 \uc591\uad6d\uac04\uc5d0 \ucd5c\uc885 \ud569\uc758\uc5d0 \ub3c4\ub2ec\ud568\uc73c\ub85c\uc368 1973\ub144 1\uc6d4\ubd80\ud130 \uc815\uc2dd \uac00\uc785(\uacbd\uacfc\uae30\uac04 5\ub144\uc744 \uacc4\uc0b0\ud558\uba74 \uc2e4\uc9c8\uc801\uc778 \uac00\uc785\uc740 1978\ub144\ubd80\ud130\uac00 \ub428)\ud558\uac8c \ub418\uc5c8\ub2e4.", "sentence_answer": "\uc774 \ub098\ub77c\ub294 \ub2e4\ub978 \uc5ec\ub7ec \ub098\ub77c\uc5d0 \uc55e\uc11c\uc11c \uc0b0\uc5c5\ud601\uba85 \uc744 \uc131\ucde8\ud558\uc5ec \uc624\ub7ab\ub3d9\uc548 \uc138\uacc4 \uacbd\uc81c\uc5d0 \uad70\ub9bc\ud574 \uc654\ub2e4.", "paragraph_id": "6587522-15-0", "paragraph_question": "question: \uc601\uad6d\uc774 \ub2e4\ub978 \ub098\ub77c\ub4e4\ubcf4\ub2e4 \uc55e\uc11c \uacbd\uc81c \ubc1c\uc804\uc744 \uc774\ub8f0 \uc218 \uc788\uc5c8\ub358 \uc774\uc720\ub294 \ubb34\uc5c7\uc774\uc5c8\ub294\uac00?, context: \uc774 \ub098\ub77c\ub294 \ub2e4\ub978 \uc5ec\ub7ec \ub098\ub77c\uc5d0 \uc55e\uc11c\uc11c \uc0b0\uc5c5\ud601\uba85\uc744 \uc131\ucde8\ud558\uc5ec \uc624\ub7ab\ub3d9\uc548 \uc138\uacc4 \uacbd\uc81c\uc5d0 \uad70\ub9bc\ud574 \uc654\ub2e4. \uadf8\ub7ec\ub098 20\uc138\uae30\uc5d0 \ub4e4\uc5b4\uc640\uc11c\ub294 \uc720\ub7fd \ub300\ub959\uacfc \ubd81\uc544\uba54\ub9ac\uce74\uc758 \uacf5\uc5c5\uc774 \uae09\uc18d\ub3c4\ub85c \ubc1c\ub2ec\ud558\uc5ec 1\ucc28\ub300\uc804 \uc774\ud6c4 \uc138\uacc4 \uacbd\uc81c\uc5d0 \uc788\uc5b4\uc11c\uc758 \uc9c0\uc704\ub294 \ucc28\ucc28 \uc800\ud558\ud588\ub2e4. \ud574\uc678 \ud22c\uc790\ub85c\ubd80\ud130\uc758 \uc218\uc785\uc73c\ub85c \uacbd\uc81c\uc758 \uc548\uc815\uc744 \uaf80\ud588\uc73c\ub098 2\ucc28\ub300\uc804\uc5d0 \uc758\ud574 \ud070 \ud0c0\uaca9\uc744 \ubc1b\uace0 \uc7ac\ud3b8\uc131\ud558\uae30\uc5d0 \uc774\ub974\ub800\ub2e4. \uadf8\uac83\uc740 \uae30\uac04\uc0b0\uc5c5\uc758 \uad6d\uc720\ud654\ub97c \ube44\ub86f\ud55c \uacc4\ud68d\uacbd\uc81c\uc758 \ub3c4\uc785\uc73c\ub85c \uad6d\uc81c\uc218\uc9c0\ub97c \uac1c\uc120\ud558\uace0 \uc0ac\ud68c\ubcf4\uc7a5\uc81c\ub3c4\ub97c \ud655\ucda9\ud558\uc5ec \ubcf5\uc9c0 \uad6d\uac00\uc758 \uc2e4\ud604\uc744 \uaf80\ud558\ub294 \uac83\uc744 \uc8fc\uc694 \ubaa9\uc801\uc73c\ub85c \ud558\ub294 \uacbd\uc81c \uccb4\uc81c\uc774\ub2e4. \uadf8\ub7ec\ub098 \ub192\uc740 \uace0\uc6a9\uc218\uc900\uc744 \uc720\uc9c0\ud558\uba74\uc11c\ub3c4 \ub300\uc678\uc801\uc73c\ub85c\ub294 \uad6d\uc81c \uc218\uc9c0\uc758 \uc545\ud654, \ub300\ub0b4\uc801\uc73c\ub85c\ub294 \ucf54\uc2a4\ud2b8 \uc778\ud50c\ub808\uc774\uc158\uc5d0 \uc758\ud55c \uad34\ub85c\uc6c0\uc744 \ub2f9\ud558\uace0 \uc788\ub2e4. \uc774\ub7ec\ud55c \uc0c1\ud669\uc744 \ud0c0\uac1c\ud558\uae30 \uc704\ud574 \uc601\uad6d\uc740 1961\ub144\ubd80\ud130 \uc720\ub7fd\uacbd\uc81c\uacf5\ub3d9\uccb4(EEC) \uac00\uc785\uc744 \uc2e0\uccad\ud588\uc73c\ub098 \ud504\ub791\uc2a4\uc758 \uac70\ubd80\ub85c \uc2e4\ud604\uc744 \ubcf4\uc9c0 \ubabb\ud558\ub2e4\uac00 1971\ub144 6\uc6d4 \ub9c8\uce68\ub0b4 \uc218\ucd9c\uc785\uc815\ucc45 \ubc0f \ud30c\uc6b4\ub4dc\ud654(\u8ca8) \ubb38\uc81c\uc5d0 \ub9e4\ub4ed\uc744 \uc9d3\uace0 \uc601\u00b7\ubd88 \uc591\uad6d\uac04\uc5d0 \ucd5c\uc885 \ud569\uc758\uc5d0 \ub3c4\ub2ec\ud568\uc73c\ub85c\uc368 1973\ub144 1\uc6d4\ubd80\ud130 \uc815\uc2dd \uac00\uc785(\uacbd\uacfc\uae30\uac04 5\ub144\uc744 \uacc4\uc0b0\ud558\uba74 \uc2e4\uc9c8\uc801\uc778 \uac00\uc785\uc740 1978\ub144\ubd80\ud130\uac00 \ub428)\ud558\uac8c \ub418\uc5c8\ub2e4."} {"question": "\ucd5c\uc9c4\uc2e4\uc740 \ub0a8\ud3b8 \uc870\uc131\ubbfc\uc758 \uc9c1\uc5c5\uc740 \ubb34\uc5c7\uc778\uac00?", "paragraph": "\ucd5c\uc9c4\uc2e4\uc740 '\ub0a8\ud3b8 \uc870\uc131\ubbfc\uc5d0\uac8c \ubcf4\ub0b4\ub294 \ud3b8\uc9c0'\ub85c \"\ub098 \ucd5c\uc9c4\uc2e4\uc740 \uc2e0\ub791 \uc870\uc131\ubbfc\uc744 \ubcf4\ud638\ud558\uace0 \uc774 \uc138\uc0c1 \ub05d\ub0a0 \ub54c\uae4c\uc9c0 \ub2f9\uc2e0\ub9cc\uc744 \uc0ac\ub791\ud558\uba70 \uc544\ub07c\uaca0\uc2b5\ub2c8\ub2e4. \uc57c\uad6c\uc120\uc218\uc778 \ub0a8\ud3b8\uc744 \uc704\ud574 \uc544\ub08c\uc5c6\uc774 \ub0b4\uc870\ud560 \uac83\uc744 \uc774 \uc790\ub9ac\uc5d0 \ucc38\uc11d\ud55c \ubaa8\ub4e0 \uc57c\uad6c \uad00\uacc4\uc790\ub4e4\uc5d0\uac8c \uc57d\uc18d\ud569\ub2c8\ub2e4.\"\ub77c\uace0 \ud654\ub2f5\ud574 \ud558\uac1d\uc73c\ub85c \ucc38\uc11d\ud55c \uc57c\uad6c\uad00\uacc4\uc790\ub4e4\ub85c\ubd80\ud130 \ubc15\uc218\ub97c \ubc1b\uc558\ub2e4. \ub610\ud55c, \ucd5c\uc9c4\uc2e4\uc740 \uc5b4\uba38\ub2c8 \uc815\uc625\uc21c\uaed8 \ub4dc\ub9ac\ub294 \uac10\uc0ac\uc758 \ud3b8\uc9c0\uc5d0\uc11c \"\uc5b4\uc81c \ub9ce\uc774 \uc6b8\uc5c8\ub2e4. \ub9ce\uc774 \uc11c\ud234\uace0 \uc5c4\ub9c8\uac00 \ubcf4\uae30\uc5d4 \uc5b4\ub824 \ubcf4\uc77c\uc9c0 \ubaa8\ub974\uc9c0\ub9cc \uc798\ud560 \uc790\uc2e0\uc788\ub2e4. \uacb0\ud63c\uc744 \ub9de\uc774\ud558\uace0\uc11c\uc57c \uc5b4\uba38\ub2c8\uc640 \ud568\uaed8 \uc0b0\ub2e4\ub294 \uac83\uc774 \ubb34\uc5c7\uc778\uc9c0 \uc54c\uc558\ub2e4. \ub0b4\uac00 \uc5c6\ub294 \uc9d1\uc5d0\uc11c \uc678\ub86d\uac8c \uc9c0\ub0bc \uc5c4\ub9c8\ub97c \uc0dd\uac01\ud558\ub2c8 \ub9c8\uc74c\uc774 \uc800\ub824\uc628\ub2e4. \ub0b3\uc544\uc8fc\uc2dc\uace0 \ud0a4\uc6cc\uc8fc\uc2e0 \uc740\ud61c\uc5d0 \uac10\uc0ac\ud55c\ub2e4.\"\uba70 \ub3d9\uc0dd \ucd5c\uc9c4\uc601\uc5d0\uac8c\ub294 \uc5c4\ub9c8\ub97c \uc798\ubaa8\uc2dc\ub77c\uace0 \ub2f9\ubd80\ud558\uc600\ub2e4.", "answer": "\uc57c\uad6c\uc120\uc218", "sentence": "\uc57c\uad6c\uc120\uc218 \uc778 \ub0a8\ud3b8\uc744 \uc704\ud574 \uc544\ub08c\uc5c6\uc774 \ub0b4\uc870\ud560 \uac83\uc744 \uc774 \uc790\ub9ac\uc5d0 \ucc38\uc11d\ud55c \ubaa8\ub4e0 \uc57c\uad6c \uad00\uacc4\uc790\ub4e4\uc5d0\uac8c \uc57d\uc18d\ud569\ub2c8\ub2e4.", "paragraph_sentence": "\ucd5c\uc9c4\uc2e4\uc740 '\ub0a8\ud3b8 \uc870\uc131\ubbfc\uc5d0\uac8c \ubcf4\ub0b4\ub294 \ud3b8\uc9c0'\ub85c \"\ub098 \ucd5c\uc9c4\uc2e4\uc740 \uc2e0\ub791 \uc870\uc131\ubbfc\uc744 \ubcf4\ud638\ud558\uace0 \uc774 \uc138\uc0c1 \ub05d\ub0a0 \ub54c\uae4c\uc9c0 \ub2f9\uc2e0\ub9cc\uc744 \uc0ac\ub791\ud558\uba70 \uc544\ub07c\uaca0\uc2b5\ub2c8\ub2e4. \uc57c\uad6c\uc120\uc218 \uc778 \ub0a8\ud3b8\uc744 \uc704\ud574 \uc544\ub08c\uc5c6\uc774 \ub0b4\uc870\ud560 \uac83\uc744 \uc774 \uc790\ub9ac\uc5d0 \ucc38\uc11d\ud55c \ubaa8\ub4e0 \uc57c\uad6c \uad00\uacc4\uc790\ub4e4\uc5d0\uac8c \uc57d\uc18d\ud569\ub2c8\ub2e4. \"\ub77c\uace0 \ud654\ub2f5\ud574 \ud558\uac1d\uc73c\ub85c \ucc38\uc11d\ud55c \uc57c\uad6c\uad00\uacc4\uc790\ub4e4\ub85c\ubd80\ud130 \ubc15\uc218\ub97c \ubc1b\uc558\ub2e4. \ub610\ud55c, \ucd5c\uc9c4\uc2e4\uc740 \uc5b4\uba38\ub2c8 \uc815\uc625\uc21c\uaed8 \ub4dc\ub9ac\ub294 \uac10\uc0ac\uc758 \ud3b8\uc9c0\uc5d0\uc11c \"\uc5b4\uc81c \ub9ce\uc774 \uc6b8\uc5c8\ub2e4. \ub9ce\uc774 \uc11c\ud234\uace0 \uc5c4\ub9c8\uac00 \ubcf4\uae30\uc5d4 \uc5b4\ub824 \ubcf4\uc77c\uc9c0 \ubaa8\ub974\uc9c0\ub9cc \uc798\ud560 \uc790\uc2e0\uc788\ub2e4. \uacb0\ud63c\uc744 \ub9de\uc774\ud558\uace0\uc11c\uc57c \uc5b4\uba38\ub2c8\uc640 \ud568\uaed8 \uc0b0\ub2e4\ub294 \uac83\uc774 \ubb34\uc5c7\uc778\uc9c0 \uc54c\uc558\ub2e4. \ub0b4\uac00 \uc5c6\ub294 \uc9d1\uc5d0\uc11c \uc678\ub86d\uac8c \uc9c0\ub0bc \uc5c4\ub9c8\ub97c \uc0dd\uac01\ud558\ub2c8 \ub9c8\uc74c\uc774 \uc800\ub824\uc628\ub2e4. \ub0b3\uc544\uc8fc\uc2dc\uace0 \ud0a4\uc6cc\uc8fc\uc2e0 \uc740\ud61c\uc5d0 \uac10\uc0ac\ud55c\ub2e4. \"\uba70 \ub3d9\uc0dd \ucd5c\uc9c4\uc601\uc5d0\uac8c\ub294 \uc5c4\ub9c8\ub97c \uc798\ubaa8\uc2dc\ub77c\uace0 \ub2f9\ubd80\ud558\uc600\ub2e4.", "paragraph_answer": "\ucd5c\uc9c4\uc2e4\uc740 '\ub0a8\ud3b8 \uc870\uc131\ubbfc\uc5d0\uac8c \ubcf4\ub0b4\ub294 \ud3b8\uc9c0'\ub85c \"\ub098 \ucd5c\uc9c4\uc2e4\uc740 \uc2e0\ub791 \uc870\uc131\ubbfc\uc744 \ubcf4\ud638\ud558\uace0 \uc774 \uc138\uc0c1 \ub05d\ub0a0 \ub54c\uae4c\uc9c0 \ub2f9\uc2e0\ub9cc\uc744 \uc0ac\ub791\ud558\uba70 \uc544\ub07c\uaca0\uc2b5\ub2c8\ub2e4. \uc57c\uad6c\uc120\uc218 \uc778 \ub0a8\ud3b8\uc744 \uc704\ud574 \uc544\ub08c\uc5c6\uc774 \ub0b4\uc870\ud560 \uac83\uc744 \uc774 \uc790\ub9ac\uc5d0 \ucc38\uc11d\ud55c \ubaa8\ub4e0 \uc57c\uad6c \uad00\uacc4\uc790\ub4e4\uc5d0\uac8c \uc57d\uc18d\ud569\ub2c8\ub2e4.\"\ub77c\uace0 \ud654\ub2f5\ud574 \ud558\uac1d\uc73c\ub85c \ucc38\uc11d\ud55c \uc57c\uad6c\uad00\uacc4\uc790\ub4e4\ub85c\ubd80\ud130 \ubc15\uc218\ub97c \ubc1b\uc558\ub2e4. \ub610\ud55c, \ucd5c\uc9c4\uc2e4\uc740 \uc5b4\uba38\ub2c8 \uc815\uc625\uc21c\uaed8 \ub4dc\ub9ac\ub294 \uac10\uc0ac\uc758 \ud3b8\uc9c0\uc5d0\uc11c \"\uc5b4\uc81c \ub9ce\uc774 \uc6b8\uc5c8\ub2e4. \ub9ce\uc774 \uc11c\ud234\uace0 \uc5c4\ub9c8\uac00 \ubcf4\uae30\uc5d4 \uc5b4\ub824 \ubcf4\uc77c\uc9c0 \ubaa8\ub974\uc9c0\ub9cc \uc798\ud560 \uc790\uc2e0\uc788\ub2e4. \uacb0\ud63c\uc744 \ub9de\uc774\ud558\uace0\uc11c\uc57c \uc5b4\uba38\ub2c8\uc640 \ud568\uaed8 \uc0b0\ub2e4\ub294 \uac83\uc774 \ubb34\uc5c7\uc778\uc9c0 \uc54c\uc558\ub2e4. \ub0b4\uac00 \uc5c6\ub294 \uc9d1\uc5d0\uc11c \uc678\ub86d\uac8c \uc9c0\ub0bc \uc5c4\ub9c8\ub97c \uc0dd\uac01\ud558\ub2c8 \ub9c8\uc74c\uc774 \uc800\ub824\uc628\ub2e4. \ub0b3\uc544\uc8fc\uc2dc\uace0 \ud0a4\uc6cc\uc8fc\uc2e0 \uc740\ud61c\uc5d0 \uac10\uc0ac\ud55c\ub2e4.\"\uba70 \ub3d9\uc0dd \ucd5c\uc9c4\uc601\uc5d0\uac8c\ub294 \uc5c4\ub9c8\ub97c \uc798\ubaa8\uc2dc\ub77c\uace0 \ub2f9\ubd80\ud558\uc600\ub2e4.", "sentence_answer": " \uc57c\uad6c\uc120\uc218 \uc778 \ub0a8\ud3b8\uc744 \uc704\ud574 \uc544\ub08c\uc5c6\uc774 \ub0b4\uc870\ud560 \uac83\uc744 \uc774 \uc790\ub9ac\uc5d0 \ucc38\uc11d\ud55c \ubaa8\ub4e0 \uc57c\uad6c \uad00\uacc4\uc790\ub4e4\uc5d0\uac8c \uc57d\uc18d\ud569\ub2c8\ub2e4.", "paragraph_id": "6552612-4-2", "paragraph_question": "question: \ucd5c\uc9c4\uc2e4\uc740 \ub0a8\ud3b8 \uc870\uc131\ubbfc\uc758 \uc9c1\uc5c5\uc740 \ubb34\uc5c7\uc778\uac00?, context: \ucd5c\uc9c4\uc2e4\uc740 '\ub0a8\ud3b8 \uc870\uc131\ubbfc\uc5d0\uac8c \ubcf4\ub0b4\ub294 \ud3b8\uc9c0'\ub85c \"\ub098 \ucd5c\uc9c4\uc2e4\uc740 \uc2e0\ub791 \uc870\uc131\ubbfc\uc744 \ubcf4\ud638\ud558\uace0 \uc774 \uc138\uc0c1 \ub05d\ub0a0 \ub54c\uae4c\uc9c0 \ub2f9\uc2e0\ub9cc\uc744 \uc0ac\ub791\ud558\uba70 \uc544\ub07c\uaca0\uc2b5\ub2c8\ub2e4. \uc57c\uad6c\uc120\uc218\uc778 \ub0a8\ud3b8\uc744 \uc704\ud574 \uc544\ub08c\uc5c6\uc774 \ub0b4\uc870\ud560 \uac83\uc744 \uc774 \uc790\ub9ac\uc5d0 \ucc38\uc11d\ud55c \ubaa8\ub4e0 \uc57c\uad6c \uad00\uacc4\uc790\ub4e4\uc5d0\uac8c \uc57d\uc18d\ud569\ub2c8\ub2e4.\"\ub77c\uace0 \ud654\ub2f5\ud574 \ud558\uac1d\uc73c\ub85c \ucc38\uc11d\ud55c \uc57c\uad6c\uad00\uacc4\uc790\ub4e4\ub85c\ubd80\ud130 \ubc15\uc218\ub97c \ubc1b\uc558\ub2e4. \ub610\ud55c, \ucd5c\uc9c4\uc2e4\uc740 \uc5b4\uba38\ub2c8 \uc815\uc625\uc21c\uaed8 \ub4dc\ub9ac\ub294 \uac10\uc0ac\uc758 \ud3b8\uc9c0\uc5d0\uc11c \"\uc5b4\uc81c \ub9ce\uc774 \uc6b8\uc5c8\ub2e4. \ub9ce\uc774 \uc11c\ud234\uace0 \uc5c4\ub9c8\uac00 \ubcf4\uae30\uc5d4 \uc5b4\ub824 \ubcf4\uc77c\uc9c0 \ubaa8\ub974\uc9c0\ub9cc \uc798\ud560 \uc790\uc2e0\uc788\ub2e4. \uacb0\ud63c\uc744 \ub9de\uc774\ud558\uace0\uc11c\uc57c \uc5b4\uba38\ub2c8\uc640 \ud568\uaed8 \uc0b0\ub2e4\ub294 \uac83\uc774 \ubb34\uc5c7\uc778\uc9c0 \uc54c\uc558\ub2e4. \ub0b4\uac00 \uc5c6\ub294 \uc9d1\uc5d0\uc11c \uc678\ub86d\uac8c \uc9c0\ub0bc \uc5c4\ub9c8\ub97c \uc0dd\uac01\ud558\ub2c8 \ub9c8\uc74c\uc774 \uc800\ub824\uc628\ub2e4. \ub0b3\uc544\uc8fc\uc2dc\uace0 \ud0a4\uc6cc\uc8fc\uc2e0 \uc740\ud61c\uc5d0 \uac10\uc0ac\ud55c\ub2e4.\"\uba70 \ub3d9\uc0dd \ucd5c\uc9c4\uc601\uc5d0\uac8c\ub294 \uc5c4\ub9c8\ub97c \uc798\ubaa8\uc2dc\ub77c\uace0 \ub2f9\ubd80\ud558\uc600\ub2e4."} {"question": "<\ub85c\ud0a4\uc758 \ub9d0\ub2e4\ud23c> 22\uc7a5\uc5d0 \ub098\uc624\ub294 \uc2dc <\uae30\ub098\uae34 \uac00\uc744>\uc774 \uc4f0\uc778 \uc2dc\uae30\ub294?", "paragraph": "\u3008\uc2dc\uc5b4\ubc95\u3009 \ub4b7\ubd80\ubd84\uc5d0\uc11c \ud1a0\ub974\ub4dc \uc0e4\ub808\ud06c\uc190\uc774 \uc9c0\uc2a8 \uc2a4\uce7c\ub4dc \uc2dc\uac00\uac00 \uc778\uc6a9\ub41c\ub2e4. \uc774 \uc2dc\uc5d0\uc11c \uc2a4\uce74\ub514\ub294 \"\ud604\uba85\ud55c \uc5ec\uc2e0\"(the wise god-bride)\uc774\ub77c\uace0 \uce6d\ud574\uc9c0\uba70 \"\ubc18\uc744 \uc0ac\ub791\ud560 \uc218 \uc5c6\uc5c8\ub2e4\"\uace0 \ud55c\ub2e4. \uc2dc \uc544\ub798\uc5d0 \ub538\ub9b0 \uc0b0\ubb38 \ud574\uc124\uc5d0\uc11c \uc774\uac83\uc774 \uc2a4\uce74\ub514\uac00 \ub1e8\ub974\ub4dc\ub97c \ub5a0\ub098\ub294 \uac83\uc744 \uac00\ub9ac\ud0a8\ub2e4\uace0 \uba85\uc2dc\ud558\uace0 \uc788\ub2e4. \uc81c16\uc7a5\uc5d0\uc11c \ub85c\ud0a4\uc758 \uc774\uba85(\u7570\u540d)\ub4e4\uc774 \ub098\uc5f4\ub418\ub294\ub370 \uadf8 \uc911\uc5d0\ub294 \"\ud5e4\uc784\ub2ec\uacfc \uc2a4\uce74\ub514\uc758 \uc6d0\uc218\"\ub77c\ub294 \uac83\ub3c4 \uc788\ub2e4. \uc81c22\uc7a5\uc5d0\uc11c 10\uc138\uae30\uc758 \uc2dc \u3008\uae30\ub098\uae34 \uac00\uc744\u3009\uc774 \ub098\uc624\ub294\ub370, \uc5ec\uae30\uc11c \ud750\ube44\ub2c8\ub974\uc758 \uc1fc\ub3cc\ud504\ub294 \uc218\uc18c\ub97c \u201c\ud65c\uc2dc\uc704 \ubc14\ub974(\uc2a4\uce74\ub514)\uc758 \uace0\ub798\u201d\ub77c\uace0 \ud55c\ub2e4. \uc81c23\uc7a5\uc5d0\uc11c \ube0c\ub77c\uae30 \ubcf4\ub2e4\uc190\uc740 \uc0e4\uce58\ub97c \u201c\uc2a4\ud0a4\uc758 \ub514\uc2a4\uc758 \uc544\ubc84\uc9c0\u201d\ub77c\uace0 \ud55c\ub2e4. \uc81c32\uc7a5\uc5d0\uc11c \uc2a4\uce74\ub514\ub294 \uc5d0\uc2dc\ub974\uc758 \uc5f0\ud68c\uc5d0 \ucc38\uc11d\ud55c 6\uba85\uc758 \uc5ec\uc2e0\ub4e4 \uc911 \ud558\ub098\ub85c \uc774\ub984\uc774 \uc62c\ub77c\uc640 \uc788\ub2e4. \uc81c75\uc7a5\uc5d0\uc11c \uc544\uc26c\ub274\ub974 27\uc5ec\uc2e0\uc758 \uba85\ub2e8\uc5d0\ub3c4 \uc2a4\uce74\ub514\uac00 \uc62c\ub77c\uc640 \uc788\ub2e4.", "answer": "10\uc138\uae30", "sentence": "\uc81c22\uc7a5\uc5d0\uc11c 10\uc138\uae30 \uc758 \uc2dc \u3008\uae30\ub098\uae34 \uac00\uc744\u3009\uc774 \ub098\uc624\ub294\ub370, \uc5ec\uae30\uc11c \ud750\ube44\ub2c8\ub974\uc758 \uc1fc\ub3cc\ud504\ub294 \uc218\uc18c\ub97c \u201c\ud65c\uc2dc\uc704 \ubc14\ub974(\uc2a4\uce74\ub514)\uc758 \uace0\ub798\u201d\ub77c\uace0 \ud55c\ub2e4.", "paragraph_sentence": "\u3008\uc2dc\uc5b4\ubc95\u3009 \ub4b7\ubd80\ubd84\uc5d0\uc11c \ud1a0\ub974\ub4dc \uc0e4\ub808\ud06c\uc190\uc774 \uc9c0\uc2a8 \uc2a4\uce7c\ub4dc \uc2dc\uac00\uac00 \uc778\uc6a9\ub41c\ub2e4. \uc774 \uc2dc\uc5d0\uc11c \uc2a4\uce74\ub514\ub294 \"\ud604\uba85\ud55c \uc5ec\uc2e0\"(the wise god-bride)\uc774\ub77c\uace0 \uce6d\ud574\uc9c0\uba70 \"\ubc18\uc744 \uc0ac\ub791\ud560 \uc218 \uc5c6\uc5c8\ub2e4\"\uace0 \ud55c\ub2e4. \uc2dc \uc544\ub798\uc5d0 \ub538\ub9b0 \uc0b0\ubb38 \ud574\uc124\uc5d0\uc11c \uc774\uac83\uc774 \uc2a4\uce74\ub514\uac00 \ub1e8\ub974\ub4dc\ub97c \ub5a0\ub098\ub294 \uac83\uc744 \uac00\ub9ac\ud0a8\ub2e4\uace0 \uba85\uc2dc\ud558\uace0 \uc788\ub2e4. \uc81c16\uc7a5\uc5d0\uc11c \ub85c\ud0a4\uc758 \uc774\uba85(\u7570\u540d)\ub4e4\uc774 \ub098\uc5f4\ub418\ub294\ub370 \uadf8 \uc911\uc5d0\ub294 \"\ud5e4\uc784\ub2ec\uacfc \uc2a4\uce74\ub514\uc758 \uc6d0\uc218\"\ub77c\ub294 \uac83\ub3c4 \uc788\ub2e4. \uc81c22\uc7a5\uc5d0\uc11c 10\uc138\uae30 \uc758 \uc2dc \u3008\uae30\ub098\uae34 \uac00\uc744\u3009\uc774 \ub098\uc624\ub294\ub370, \uc5ec\uae30\uc11c \ud750\ube44\ub2c8\ub974\uc758 \uc1fc\ub3cc\ud504\ub294 \uc218\uc18c\ub97c \u201c\ud65c\uc2dc\uc704 \ubc14\ub974(\uc2a4\uce74\ub514)\uc758 \uace0\ub798\u201d\ub77c\uace0 \ud55c\ub2e4. \uc81c23\uc7a5\uc5d0\uc11c \ube0c\ub77c\uae30 \ubcf4\ub2e4\uc190\uc740 \uc0e4\uce58\ub97c \u201c\uc2a4\ud0a4\uc758 \ub514\uc2a4\uc758 \uc544\ubc84\uc9c0\u201d\ub77c\uace0 \ud55c\ub2e4. \uc81c32\uc7a5\uc5d0\uc11c \uc2a4\uce74\ub514\ub294 \uc5d0\uc2dc\ub974\uc758 \uc5f0\ud68c\uc5d0 \ucc38\uc11d\ud55c 6\uba85\uc758 \uc5ec\uc2e0\ub4e4 \uc911 \ud558\ub098\ub85c \uc774\ub984\uc774 \uc62c\ub77c\uc640 \uc788\ub2e4. \uc81c75\uc7a5\uc5d0\uc11c \uc544\uc26c\ub274\ub974 27\uc5ec\uc2e0\uc758 \uba85\ub2e8\uc5d0\ub3c4 \uc2a4\uce74\ub514\uac00 \uc62c\ub77c\uc640 \uc788\ub2e4.", "paragraph_answer": "\u3008\uc2dc\uc5b4\ubc95\u3009 \ub4b7\ubd80\ubd84\uc5d0\uc11c \ud1a0\ub974\ub4dc \uc0e4\ub808\ud06c\uc190\uc774 \uc9c0\uc2a8 \uc2a4\uce7c\ub4dc \uc2dc\uac00\uac00 \uc778\uc6a9\ub41c\ub2e4. \uc774 \uc2dc\uc5d0\uc11c \uc2a4\uce74\ub514\ub294 \"\ud604\uba85\ud55c \uc5ec\uc2e0\"(the wise god-bride)\uc774\ub77c\uace0 \uce6d\ud574\uc9c0\uba70 \"\ubc18\uc744 \uc0ac\ub791\ud560 \uc218 \uc5c6\uc5c8\ub2e4\"\uace0 \ud55c\ub2e4. \uc2dc \uc544\ub798\uc5d0 \ub538\ub9b0 \uc0b0\ubb38 \ud574\uc124\uc5d0\uc11c \uc774\uac83\uc774 \uc2a4\uce74\ub514\uac00 \ub1e8\ub974\ub4dc\ub97c \ub5a0\ub098\ub294 \uac83\uc744 \uac00\ub9ac\ud0a8\ub2e4\uace0 \uba85\uc2dc\ud558\uace0 \uc788\ub2e4. \uc81c16\uc7a5\uc5d0\uc11c \ub85c\ud0a4\uc758 \uc774\uba85(\u7570\u540d)\ub4e4\uc774 \ub098\uc5f4\ub418\ub294\ub370 \uadf8 \uc911\uc5d0\ub294 \"\ud5e4\uc784\ub2ec\uacfc \uc2a4\uce74\ub514\uc758 \uc6d0\uc218\"\ub77c\ub294 \uac83\ub3c4 \uc788\ub2e4. \uc81c22\uc7a5\uc5d0\uc11c 10\uc138\uae30 \uc758 \uc2dc \u3008\uae30\ub098\uae34 \uac00\uc744\u3009\uc774 \ub098\uc624\ub294\ub370, \uc5ec\uae30\uc11c \ud750\ube44\ub2c8\ub974\uc758 \uc1fc\ub3cc\ud504\ub294 \uc218\uc18c\ub97c \u201c\ud65c\uc2dc\uc704 \ubc14\ub974(\uc2a4\uce74\ub514)\uc758 \uace0\ub798\u201d\ub77c\uace0 \ud55c\ub2e4. \uc81c23\uc7a5\uc5d0\uc11c \ube0c\ub77c\uae30 \ubcf4\ub2e4\uc190\uc740 \uc0e4\uce58\ub97c \u201c\uc2a4\ud0a4\uc758 \ub514\uc2a4\uc758 \uc544\ubc84\uc9c0\u201d\ub77c\uace0 \ud55c\ub2e4. \uc81c32\uc7a5\uc5d0\uc11c \uc2a4\uce74\ub514\ub294 \uc5d0\uc2dc\ub974\uc758 \uc5f0\ud68c\uc5d0 \ucc38\uc11d\ud55c 6\uba85\uc758 \uc5ec\uc2e0\ub4e4 \uc911 \ud558\ub098\ub85c \uc774\ub984\uc774 \uc62c\ub77c\uc640 \uc788\ub2e4. \uc81c75\uc7a5\uc5d0\uc11c \uc544\uc26c\ub274\ub974 27\uc5ec\uc2e0\uc758 \uba85\ub2e8\uc5d0\ub3c4 \uc2a4\uce74\ub514\uac00 \uc62c\ub77c\uc640 \uc788\ub2e4.", "sentence_answer": "\uc81c22\uc7a5\uc5d0\uc11c 10\uc138\uae30 \uc758 \uc2dc \u3008\uae30\ub098\uae34 \uac00\uc744\u3009\uc774 \ub098\uc624\ub294\ub370, \uc5ec\uae30\uc11c \ud750\ube44\ub2c8\ub974\uc758 \uc1fc\ub3cc\ud504\ub294 \uc218\uc18c\ub97c \u201c\ud65c\uc2dc\uc704 \ubc14\ub974(\uc2a4\uce74\ub514)\uc758 \uace0\ub798\u201d\ub77c\uace0 \ud55c\ub2e4.", "paragraph_id": "6519067-1-1", "paragraph_question": "question: <\ub85c\ud0a4\uc758 \ub9d0\ub2e4\ud23c> 22\uc7a5\uc5d0 \ub098\uc624\ub294 \uc2dc <\uae30\ub098\uae34 \uac00\uc744>\uc774 \uc4f0\uc778 \uc2dc\uae30\ub294?, context: \u3008\uc2dc\uc5b4\ubc95\u3009 \ub4b7\ubd80\ubd84\uc5d0\uc11c \ud1a0\ub974\ub4dc \uc0e4\ub808\ud06c\uc190\uc774 \uc9c0\uc2a8 \uc2a4\uce7c\ub4dc \uc2dc\uac00\uac00 \uc778\uc6a9\ub41c\ub2e4. \uc774 \uc2dc\uc5d0\uc11c \uc2a4\uce74\ub514\ub294 \"\ud604\uba85\ud55c \uc5ec\uc2e0\"(the wise god-bride)\uc774\ub77c\uace0 \uce6d\ud574\uc9c0\uba70 \"\ubc18\uc744 \uc0ac\ub791\ud560 \uc218 \uc5c6\uc5c8\ub2e4\"\uace0 \ud55c\ub2e4. \uc2dc \uc544\ub798\uc5d0 \ub538\ub9b0 \uc0b0\ubb38 \ud574\uc124\uc5d0\uc11c \uc774\uac83\uc774 \uc2a4\uce74\ub514\uac00 \ub1e8\ub974\ub4dc\ub97c \ub5a0\ub098\ub294 \uac83\uc744 \uac00\ub9ac\ud0a8\ub2e4\uace0 \uba85\uc2dc\ud558\uace0 \uc788\ub2e4. \uc81c16\uc7a5\uc5d0\uc11c \ub85c\ud0a4\uc758 \uc774\uba85(\u7570\u540d)\ub4e4\uc774 \ub098\uc5f4\ub418\ub294\ub370 \uadf8 \uc911\uc5d0\ub294 \"\ud5e4\uc784\ub2ec\uacfc \uc2a4\uce74\ub514\uc758 \uc6d0\uc218\"\ub77c\ub294 \uac83\ub3c4 \uc788\ub2e4. \uc81c22\uc7a5\uc5d0\uc11c 10\uc138\uae30\uc758 \uc2dc \u3008\uae30\ub098\uae34 \uac00\uc744\u3009\uc774 \ub098\uc624\ub294\ub370, \uc5ec\uae30\uc11c \ud750\ube44\ub2c8\ub974\uc758 \uc1fc\ub3cc\ud504\ub294 \uc218\uc18c\ub97c \u201c\ud65c\uc2dc\uc704 \ubc14\ub974(\uc2a4\uce74\ub514)\uc758 \uace0\ub798\u201d\ub77c\uace0 \ud55c\ub2e4. \uc81c23\uc7a5\uc5d0\uc11c \ube0c\ub77c\uae30 \ubcf4\ub2e4\uc190\uc740 \uc0e4\uce58\ub97c \u201c\uc2a4\ud0a4\uc758 \ub514\uc2a4\uc758 \uc544\ubc84\uc9c0\u201d\ub77c\uace0 \ud55c\ub2e4. \uc81c32\uc7a5\uc5d0\uc11c \uc2a4\uce74\ub514\ub294 \uc5d0\uc2dc\ub974\uc758 \uc5f0\ud68c\uc5d0 \ucc38\uc11d\ud55c 6\uba85\uc758 \uc5ec\uc2e0\ub4e4 \uc911 \ud558\ub098\ub85c \uc774\ub984\uc774 \uc62c\ub77c\uc640 \uc788\ub2e4. \uc81c75\uc7a5\uc5d0\uc11c \uc544\uc26c\ub274\ub974 27\uc5ec\uc2e0\uc758 \uba85\ub2e8\uc5d0\ub3c4 \uc2a4\uce74\ub514\uac00 \uc62c\ub77c\uc640 \uc788\ub2e4."} {"question": "\ubaa8\uce58\ud788\ud1a0 \uc655\uc774 \uc2e4\ud328\ud55c \ud0c0\ub3c4 \uacc4\ud68d\uc758 \ub300\uc0c1\uc740?", "paragraph": "\ubaa8\uce58\ud788\ud1a0 \uc655 \uc790\uc2e0\uc758 \ud5e4\uc774\ucf00 \ud0c0\ub3c4 \uacc4\ud68d\uc740 \uc2e4\ud328\ub85c \ub05d\ub0ac\uc9c0\ub9cc, \uadf8\uc758 \uc601\uc9c0\ub97c \ubc1b\uc740 \ubbf8\ub098\ubaa8\ud1a0\ub178 \uc694\ub9ac\ud1a0\ubaa8\ub098 \uae30\uc18c \uc694\uc2dc\ub098\uce74(\u6728\u66fe\u7fa9\u4ef2) \ub4f1 \uac01\uc9c0 \uac90\uc9c0\ub4e4\uc774 \uc787\ub530\ub77c \ubcd1\uc0ac\ub97c \uc77c\uc73c\ucf1c, \ud5e4\uc774\ucf00 \uba78\ub9dd\uc758 \uc2dc\ubc1c\uc810\uc774 \ub418\uc5c8\ub2e4. \ud55c\ud3b8 \uc870\uc815\uc740 \ub2f9\ucd08\uc5d0 \ubaa8\uce58\ud788\ud1a0 \uc655\uc774 \ub0b4\ub838\ub2e4\ub294 \uc601\uc9c0 \uc790\uccb4\ub97c \uac00\uc9dc\ub77c\uace0 \uc0dd\uac01\ud588\uc9c0\ub9cc, \ub098\uc911\uc5d0 \uac00\uc11c\uc57c \uc601\uc9c0 \uc790\uccb4\ub294 \uc9c4\uc9dc\ub77c\ub294 \uac83\uc774 \ubc1d\ud600\uc84c\uace0, \ub098\uc544\uac00 \ubaa8\uce58\ud788\ud1a0 \uc655\uc774 \uc790\uc2e0\uc758 \ub3d9\uc0dd \ub2e4\uce74\ucfe0\ub77c \ucc9c\ud669(\u9ad8\u5009\u5929\u7687)\uc774\ub098 \uc870\uce74 \uc548\ud1a0\ucfe0 \ucc9c\ud669(\u5b89\u5fb3\u5929\u7687)\uc744 \uc81c\uce58\uace0 \uc989\uc704\ud560 \uac83\uc774\ub77c\ub294 \ubb38\uc7a5\uc774 \ub2f4\uae34 \uac83\uc744 \ubcf4\uace0 \ubc18\ubc1c\ud588\ub2e4\uace0 \ud55c\ub2e4. \ubc18\ub780\uc774 \uc77c\uc5b4\ub098\uace0 16\ub144\uc774 \uc9c0\ub09c \uac90\ud050(\u5efa\u4e45) 7\ub144(1196\ub144)\uae4c\uc9c0\ub3c4 \ubaa8\uce58\ud788\ud1a0 \uc655\uc740 \u300c\ud615\uc778(\u5211\u4eba)\u300d\uc774\ub77c \ubd88\ub9ac\uba70 \ubaa8\ubc18\uc790 \ucde8\uae09\uc744 \ubc1b\uc558\ub2e4(\u300a\uad50\ucfe0\uc694\u300b \uac90\ud050 7\ub144 \uc815\uc6d4 15\uc77c\uc870).", "answer": "\ud5e4\uc774\ucf00", "sentence": "\ubaa8\uce58\ud788\ud1a0 \uc655 \uc790\uc2e0\uc758 \ud5e4\uc774\ucf00 \ud0c0\ub3c4 \uacc4\ud68d\uc740 \uc2e4\ud328\ub85c \ub05d\ub0ac\uc9c0\ub9cc, \uadf8\uc758 \uc601\uc9c0\ub97c \ubc1b\uc740 \ubbf8\ub098\ubaa8\ud1a0\ub178 \uc694\ub9ac\ud1a0\ubaa8\ub098 \uae30\uc18c \uc694\uc2dc\ub098\uce74(\u6728\u66fe\u7fa9\u4ef2) \ub4f1 \uac01\uc9c0 \uac90\uc9c0\ub4e4\uc774 \uc787\ub530\ub77c \ubcd1\uc0ac\ub97c \uc77c\uc73c\ucf1c, \ud5e4\uc774\ucf00 \uba78\ub9dd\uc758 \uc2dc\ubc1c\uc810\uc774 \ub418\uc5c8\ub2e4.", "paragraph_sentence": " \ubaa8\uce58\ud788\ud1a0 \uc655 \uc790\uc2e0\uc758 \ud5e4\uc774\ucf00 \ud0c0\ub3c4 \uacc4\ud68d\uc740 \uc2e4\ud328\ub85c \ub05d\ub0ac\uc9c0\ub9cc, \uadf8\uc758 \uc601\uc9c0\ub97c \ubc1b\uc740 \ubbf8\ub098\ubaa8\ud1a0\ub178 \uc694\ub9ac\ud1a0\ubaa8\ub098 \uae30\uc18c \uc694\uc2dc\ub098\uce74(\u6728\u66fe\u7fa9\u4ef2) \ub4f1 \uac01\uc9c0 \uac90\uc9c0\ub4e4\uc774 \uc787\ub530\ub77c \ubcd1\uc0ac\ub97c \uc77c\uc73c\ucf1c, \ud5e4\uc774\ucf00 \uba78\ub9dd\uc758 \uc2dc\ubc1c\uc810\uc774 \ub418\uc5c8\ub2e4. \ud55c\ud3b8 \uc870\uc815\uc740 \ub2f9\ucd08\uc5d0 \ubaa8\uce58\ud788\ud1a0 \uc655\uc774 \ub0b4\ub838\ub2e4\ub294 \uc601\uc9c0 \uc790\uccb4\ub97c \uac00\uc9dc\ub77c\uace0 \uc0dd\uac01\ud588\uc9c0\ub9cc, \ub098\uc911\uc5d0 \uac00\uc11c\uc57c \uc601\uc9c0 \uc790\uccb4\ub294 \uc9c4\uc9dc\ub77c\ub294 \uac83\uc774 \ubc1d\ud600\uc84c\uace0, \ub098\uc544\uac00 \ubaa8\uce58\ud788\ud1a0 \uc655\uc774 \uc790\uc2e0\uc758 \ub3d9\uc0dd \ub2e4\uce74\ucfe0\ub77c \ucc9c\ud669(\u9ad8\u5009\u5929\u7687)\uc774\ub098 \uc870\uce74 \uc548\ud1a0\ucfe0 \ucc9c\ud669(\u5b89\u5fb3\u5929\u7687)\uc744 \uc81c\uce58\uace0 \uc989\uc704\ud560 \uac83\uc774\ub77c\ub294 \ubb38\uc7a5\uc774 \ub2f4\uae34 \uac83\uc744 \ubcf4\uace0 \ubc18\ubc1c\ud588\ub2e4\uace0 \ud55c\ub2e4. \ubc18\ub780\uc774 \uc77c\uc5b4\ub098\uace0 16\ub144\uc774 \uc9c0\ub09c \uac90\ud050(\u5efa\u4e45) 7\ub144(1196\ub144)\uae4c\uc9c0\ub3c4 \ubaa8\uce58\ud788\ud1a0 \uc655\uc740 \u300c\ud615\uc778(\u5211\u4eba)\u300d\uc774\ub77c \ubd88\ub9ac\uba70 \ubaa8\ubc18\uc790 \ucde8\uae09\uc744 \ubc1b\uc558\ub2e4(\u300a\uad50\ucfe0\uc694\u300b \uac90\ud050 7\ub144 \uc815\uc6d4 15\uc77c\uc870).", "paragraph_answer": "\ubaa8\uce58\ud788\ud1a0 \uc655 \uc790\uc2e0\uc758 \ud5e4\uc774\ucf00 \ud0c0\ub3c4 \uacc4\ud68d\uc740 \uc2e4\ud328\ub85c \ub05d\ub0ac\uc9c0\ub9cc, \uadf8\uc758 \uc601\uc9c0\ub97c \ubc1b\uc740 \ubbf8\ub098\ubaa8\ud1a0\ub178 \uc694\ub9ac\ud1a0\ubaa8\ub098 \uae30\uc18c \uc694\uc2dc\ub098\uce74(\u6728\u66fe\u7fa9\u4ef2) \ub4f1 \uac01\uc9c0 \uac90\uc9c0\ub4e4\uc774 \uc787\ub530\ub77c \ubcd1\uc0ac\ub97c \uc77c\uc73c\ucf1c, \ud5e4\uc774\ucf00 \uba78\ub9dd\uc758 \uc2dc\ubc1c\uc810\uc774 \ub418\uc5c8\ub2e4. \ud55c\ud3b8 \uc870\uc815\uc740 \ub2f9\ucd08\uc5d0 \ubaa8\uce58\ud788\ud1a0 \uc655\uc774 \ub0b4\ub838\ub2e4\ub294 \uc601\uc9c0 \uc790\uccb4\ub97c \uac00\uc9dc\ub77c\uace0 \uc0dd\uac01\ud588\uc9c0\ub9cc, \ub098\uc911\uc5d0 \uac00\uc11c\uc57c \uc601\uc9c0 \uc790\uccb4\ub294 \uc9c4\uc9dc\ub77c\ub294 \uac83\uc774 \ubc1d\ud600\uc84c\uace0, \ub098\uc544\uac00 \ubaa8\uce58\ud788\ud1a0 \uc655\uc774 \uc790\uc2e0\uc758 \ub3d9\uc0dd \ub2e4\uce74\ucfe0\ub77c \ucc9c\ud669(\u9ad8\u5009\u5929\u7687)\uc774\ub098 \uc870\uce74 \uc548\ud1a0\ucfe0 \ucc9c\ud669(\u5b89\u5fb3\u5929\u7687)\uc744 \uc81c\uce58\uace0 \uc989\uc704\ud560 \uac83\uc774\ub77c\ub294 \ubb38\uc7a5\uc774 \ub2f4\uae34 \uac83\uc744 \ubcf4\uace0 \ubc18\ubc1c\ud588\ub2e4\uace0 \ud55c\ub2e4. \ubc18\ub780\uc774 \uc77c\uc5b4\ub098\uace0 16\ub144\uc774 \uc9c0\ub09c \uac90\ud050(\u5efa\u4e45) 7\ub144(1196\ub144)\uae4c\uc9c0\ub3c4 \ubaa8\uce58\ud788\ud1a0 \uc655\uc740 \u300c\ud615\uc778(\u5211\u4eba)\u300d\uc774\ub77c \ubd88\ub9ac\uba70 \ubaa8\ubc18\uc790 \ucde8\uae09\uc744 \ubc1b\uc558\ub2e4(\u300a\uad50\ucfe0\uc694\u300b \uac90\ud050 7\ub144 \uc815\uc6d4 15\uc77c\uc870).", "sentence_answer": "\ubaa8\uce58\ud788\ud1a0 \uc655 \uc790\uc2e0\uc758 \ud5e4\uc774\ucf00 \ud0c0\ub3c4 \uacc4\ud68d\uc740 \uc2e4\ud328\ub85c \ub05d\ub0ac\uc9c0\ub9cc, \uadf8\uc758 \uc601\uc9c0\ub97c \ubc1b\uc740 \ubbf8\ub098\ubaa8\ud1a0\ub178 \uc694\ub9ac\ud1a0\ubaa8\ub098 \uae30\uc18c \uc694\uc2dc\ub098\uce74(\u6728\u66fe\u7fa9\u4ef2) \ub4f1 \uac01\uc9c0 \uac90\uc9c0\ub4e4\uc774 \uc787\ub530\ub77c \ubcd1\uc0ac\ub97c \uc77c\uc73c\ucf1c, \ud5e4\uc774\ucf00 \uba78\ub9dd\uc758 \uc2dc\ubc1c\uc810\uc774 \ub418\uc5c8\ub2e4.", "paragraph_id": "6514557-2-0", "paragraph_question": "question: \ubaa8\uce58\ud788\ud1a0 \uc655\uc774 \uc2e4\ud328\ud55c \ud0c0\ub3c4 \uacc4\ud68d\uc758 \ub300\uc0c1\uc740?, context: \ubaa8\uce58\ud788\ud1a0 \uc655 \uc790\uc2e0\uc758 \ud5e4\uc774\ucf00 \ud0c0\ub3c4 \uacc4\ud68d\uc740 \uc2e4\ud328\ub85c \ub05d\ub0ac\uc9c0\ub9cc, \uadf8\uc758 \uc601\uc9c0\ub97c \ubc1b\uc740 \ubbf8\ub098\ubaa8\ud1a0\ub178 \uc694\ub9ac\ud1a0\ubaa8\ub098 \uae30\uc18c \uc694\uc2dc\ub098\uce74(\u6728\u66fe\u7fa9\u4ef2) \ub4f1 \uac01\uc9c0 \uac90\uc9c0\ub4e4\uc774 \uc787\ub530\ub77c \ubcd1\uc0ac\ub97c \uc77c\uc73c\ucf1c, \ud5e4\uc774\ucf00 \uba78\ub9dd\uc758 \uc2dc\ubc1c\uc810\uc774 \ub418\uc5c8\ub2e4. \ud55c\ud3b8 \uc870\uc815\uc740 \ub2f9\ucd08\uc5d0 \ubaa8\uce58\ud788\ud1a0 \uc655\uc774 \ub0b4\ub838\ub2e4\ub294 \uc601\uc9c0 \uc790\uccb4\ub97c \uac00\uc9dc\ub77c\uace0 \uc0dd\uac01\ud588\uc9c0\ub9cc, \ub098\uc911\uc5d0 \uac00\uc11c\uc57c \uc601\uc9c0 \uc790\uccb4\ub294 \uc9c4\uc9dc\ub77c\ub294 \uac83\uc774 \ubc1d\ud600\uc84c\uace0, \ub098\uc544\uac00 \ubaa8\uce58\ud788\ud1a0 \uc655\uc774 \uc790\uc2e0\uc758 \ub3d9\uc0dd \ub2e4\uce74\ucfe0\ub77c \ucc9c\ud669(\u9ad8\u5009\u5929\u7687)\uc774\ub098 \uc870\uce74 \uc548\ud1a0\ucfe0 \ucc9c\ud669(\u5b89\u5fb3\u5929\u7687)\uc744 \uc81c\uce58\uace0 \uc989\uc704\ud560 \uac83\uc774\ub77c\ub294 \ubb38\uc7a5\uc774 \ub2f4\uae34 \uac83\uc744 \ubcf4\uace0 \ubc18\ubc1c\ud588\ub2e4\uace0 \ud55c\ub2e4. \ubc18\ub780\uc774 \uc77c\uc5b4\ub098\uace0 16\ub144\uc774 \uc9c0\ub09c \uac90\ud050(\u5efa\u4e45) 7\ub144(1196\ub144)\uae4c\uc9c0\ub3c4 \ubaa8\uce58\ud788\ud1a0 \uc655\uc740 \u300c\ud615\uc778(\u5211\u4eba)\u300d\uc774\ub77c \ubd88\ub9ac\uba70 \ubaa8\ubc18\uc790 \ucde8\uae09\uc744 \ubc1b\uc558\ub2e4(\u300a\uad50\ucfe0\uc694\u300b \uac90\ud050 7\ub144 \uc815\uc6d4 15\uc77c\uc870)."} {"question": "\uc5e0\ub9c8\uc2a4\ud1a4\uc774 7\ud68c\ub9cc\uc5d0 \ud558\ucc28\ud574\uc57c \ud588\ub358 \ub4dc\ub77c\ub9c8\ub294?", "paragraph": "2004\ub144, VH1\uc758 \ud0e4\ub7f0\ud2b8 \ubc1c\uad74 \ud504\ub85c\uadf8\ub7a8 \u300aIn Search of the New Partridge Family\u300b\uc5d0\uc11c \ub85c\ub9ac \ud328\ud2b8\ub9ac\uc9c0 \uc5ed\ud560\uc744 \ub530\ub0b8 \ud6c4 \ud154\ub808\ube44\uc804\uc5d0\uc11c \uacbd\ub825\uc744 \uc2dc\uc791\ud588\ub2e4. \ud0e4\ub7f0\ud2b8 \ubc1c\uad74 \ud504\ub85c\uadf8\ub7a8\uc758 \uacb0\uacfc\ub85c, \uc774\ub4ec\ud574 2005\ub144 TV \ub4dc\ub77c\ub9c8 \u300a\uc0c8\ub85c\uc6b4 \ud30c\ud2b8\ub9ac\uc9c0 \ud328\ubc00\ub9ac\u300b\uac00 \uc81c\uc791\ub418\uc9c0\ub9cc, \ud30c\uc77c\ub7ff \uc5d0\ud53c\uc18c\ub4dc\ub9cc\uc774 \ubc29\uc601\ub418\uc5c8\ub2e4. \uc774\ud6c4 \u300a\uace0\uc2a4\ud2b8 & \ud06c\ub77c\uc784\u300b, \u300a\ub9d0\ucf64\ub124 \uc880 \ub9d0\ub824\uc918\u300b, \u300a\ub7ed\ud0a4 \ub8e8\uc774\u300b \ub4f1 \uc5f0\uc774\uc5b4 \uc815\uae30\uc801\uc73c\ub85c \ucd9c\uc5f0\ud558\uc600\ub2e4. NBC\uc758 SF \ub4dc\ub77c\ub9c8 \u300a\ud788\uc5b4\ub85c\uc988\u300b\uc758 \ud074\ub808\uc5b4 \ubca0\ub137 \uc5ed\ud560\uc5d0 \uc624\ub514\uc158\uc744 \ubcf4\uc558\uc9c0\ub9cc, \uacb0\uad6d \ud5e4\uc774\ub4e0 \ud328\ub124\ud2f0\uc5b4\uac00 \uce90\uc2a4\ud305\ub418\uc5c8\ub2e4. \uc2a4\ud1a4\uc740 \uc774 \uacbd\ud5d8\uc744 \"\ucd5c\uc545\"\uc774\ub77c\uace0 \ud558\uc600\ub2e4. 2007\ub144 4\uc6d4, \uc2a4\ud1a4\uc740 \ud3ed\uc2a4\uc758 \uc561\uc158 \ub4dc\ub77c\ub9c8 \u300a\ub4dc\ub77c\uc774\ube0c\u300b\uc5d0 \ubc14\uc774\uc5bc\ub9bf \ud2b8\ub9bc\ube14 \uc5ed\uc73c\ub85c \uace0\uc815 \ucd9c\uc5f0\ud558\uc600\uc73c\ub098, \ubc29\uc601 \ub3c4\uc911 \ucde8\uc18c\ub418\uc5b4 7\ud68c \ub9cc\uc774 \ubc29\uc601\ub418\uc5c8\ub2e4.", "answer": "\ub4dc\ub77c\uc774\ube0c", "sentence": "2007\ub144 4\uc6d4, \uc2a4\ud1a4\uc740 \ud3ed\uc2a4\uc758 \uc561\uc158 \ub4dc\ub77c\ub9c8 \u300a \ub4dc\ub77c\uc774\ube0c \u300b\uc5d0 \ubc14\uc774\uc5bc\ub9bf \ud2b8\ub9bc\ube14 \uc5ed\uc73c\ub85c \uace0\uc815 \ucd9c\uc5f0\ud558\uc600\uc73c\ub098, \ubc29\uc601 \ub3c4\uc911 \ucde8\uc18c\ub418\uc5b4 7\ud68c \ub9cc\uc774 \ubc29\uc601\ub418\uc5c8\ub2e4.", "paragraph_sentence": "2004\ub144, VH1\uc758 \ud0e4\ub7f0\ud2b8 \ubc1c\uad74 \ud504\ub85c\uadf8\ub7a8 \u300aIn Search of the New Partridge Family\u300b\uc5d0\uc11c \ub85c\ub9ac \ud328\ud2b8\ub9ac\uc9c0 \uc5ed\ud560\uc744 \ub530\ub0b8 \ud6c4 \ud154\ub808\ube44\uc804\uc5d0\uc11c \uacbd\ub825\uc744 \uc2dc\uc791\ud588\ub2e4. \ud0e4\ub7f0\ud2b8 \ubc1c\uad74 \ud504\ub85c\uadf8\ub7a8\uc758 \uacb0\uacfc\ub85c, \uc774\ub4ec\ud574 2005\ub144 TV \ub4dc\ub77c\ub9c8 \u300a\uc0c8\ub85c\uc6b4 \ud30c\ud2b8\ub9ac\uc9c0 \ud328\ubc00\ub9ac\u300b\uac00 \uc81c\uc791\ub418\uc9c0\ub9cc, \ud30c\uc77c\ub7ff \uc5d0\ud53c\uc18c\ub4dc\ub9cc\uc774 \ubc29\uc601\ub418\uc5c8\ub2e4. \uc774\ud6c4 \u300a\uace0\uc2a4\ud2b8 & \ud06c\ub77c\uc784\u300b, \u300a\ub9d0\ucf64\ub124 \uc880 \ub9d0\ub824\uc918\u300b, \u300a\ub7ed\ud0a4 \ub8e8\uc774\u300b \ub4f1 \uc5f0\uc774\uc5b4 \uc815\uae30\uc801\uc73c\ub85c \ucd9c\uc5f0\ud558\uc600\ub2e4. NBC\uc758 SF \ub4dc\ub77c\ub9c8 \u300a\ud788\uc5b4\ub85c\uc988\u300b\uc758 \ud074\ub808\uc5b4 \ubca0\ub137 \uc5ed\ud560\uc5d0 \uc624\ub514\uc158\uc744 \ubcf4\uc558\uc9c0\ub9cc, \uacb0\uad6d \ud5e4\uc774\ub4e0 \ud328\ub124\ud2f0\uc5b4\uac00 \uce90\uc2a4\ud305\ub418\uc5c8\ub2e4. \uc2a4\ud1a4\uc740 \uc774 \uacbd\ud5d8\uc744 \"\ucd5c\uc545\"\uc774\ub77c\uace0 \ud558\uc600\ub2e4. 2007\ub144 4\uc6d4, \uc2a4\ud1a4\uc740 \ud3ed\uc2a4\uc758 \uc561\uc158 \ub4dc\ub77c\ub9c8 \u300a \ub4dc\ub77c\uc774\ube0c \u300b\uc5d0 \ubc14\uc774\uc5bc\ub9bf \ud2b8\ub9bc\ube14 \uc5ed\uc73c\ub85c \uace0\uc815 \ucd9c\uc5f0\ud558\uc600\uc73c\ub098, \ubc29\uc601 \ub3c4\uc911 \ucde8\uc18c\ub418\uc5b4 7\ud68c \ub9cc\uc774 \ubc29\uc601\ub418\uc5c8\ub2e4. ", "paragraph_answer": "2004\ub144, VH1\uc758 \ud0e4\ub7f0\ud2b8 \ubc1c\uad74 \ud504\ub85c\uadf8\ub7a8 \u300aIn Search of the New Partridge Family\u300b\uc5d0\uc11c \ub85c\ub9ac \ud328\ud2b8\ub9ac\uc9c0 \uc5ed\ud560\uc744 \ub530\ub0b8 \ud6c4 \ud154\ub808\ube44\uc804\uc5d0\uc11c \uacbd\ub825\uc744 \uc2dc\uc791\ud588\ub2e4. \ud0e4\ub7f0\ud2b8 \ubc1c\uad74 \ud504\ub85c\uadf8\ub7a8\uc758 \uacb0\uacfc\ub85c, \uc774\ub4ec\ud574 2005\ub144 TV \ub4dc\ub77c\ub9c8 \u300a\uc0c8\ub85c\uc6b4 \ud30c\ud2b8\ub9ac\uc9c0 \ud328\ubc00\ub9ac\u300b\uac00 \uc81c\uc791\ub418\uc9c0\ub9cc, \ud30c\uc77c\ub7ff \uc5d0\ud53c\uc18c\ub4dc\ub9cc\uc774 \ubc29\uc601\ub418\uc5c8\ub2e4. \uc774\ud6c4 \u300a\uace0\uc2a4\ud2b8 & \ud06c\ub77c\uc784\u300b, \u300a\ub9d0\ucf64\ub124 \uc880 \ub9d0\ub824\uc918\u300b, \u300a\ub7ed\ud0a4 \ub8e8\uc774\u300b \ub4f1 \uc5f0\uc774\uc5b4 \uc815\uae30\uc801\uc73c\ub85c \ucd9c\uc5f0\ud558\uc600\ub2e4. NBC\uc758 SF \ub4dc\ub77c\ub9c8 \u300a\ud788\uc5b4\ub85c\uc988\u300b\uc758 \ud074\ub808\uc5b4 \ubca0\ub137 \uc5ed\ud560\uc5d0 \uc624\ub514\uc158\uc744 \ubcf4\uc558\uc9c0\ub9cc, \uacb0\uad6d \ud5e4\uc774\ub4e0 \ud328\ub124\ud2f0\uc5b4\uac00 \uce90\uc2a4\ud305\ub418\uc5c8\ub2e4. \uc2a4\ud1a4\uc740 \uc774 \uacbd\ud5d8\uc744 \"\ucd5c\uc545\"\uc774\ub77c\uace0 \ud558\uc600\ub2e4. 2007\ub144 4\uc6d4, \uc2a4\ud1a4\uc740 \ud3ed\uc2a4\uc758 \uc561\uc158 \ub4dc\ub77c\ub9c8 \u300a \ub4dc\ub77c\uc774\ube0c \u300b\uc5d0 \ubc14\uc774\uc5bc\ub9bf \ud2b8\ub9bc\ube14 \uc5ed\uc73c\ub85c \uace0\uc815 \ucd9c\uc5f0\ud558\uc600\uc73c\ub098, \ubc29\uc601 \ub3c4\uc911 \ucde8\uc18c\ub418\uc5b4 7\ud68c \ub9cc\uc774 \ubc29\uc601\ub418\uc5c8\ub2e4.", "sentence_answer": "2007\ub144 4\uc6d4, \uc2a4\ud1a4\uc740 \ud3ed\uc2a4\uc758 \uc561\uc158 \ub4dc\ub77c\ub9c8 \u300a \ub4dc\ub77c\uc774\ube0c \u300b\uc5d0 \ubc14\uc774\uc5bc\ub9bf \ud2b8\ub9bc\ube14 \uc5ed\uc73c\ub85c \uace0\uc815 \ucd9c\uc5f0\ud558\uc600\uc73c\ub098, \ubc29\uc601 \ub3c4\uc911 \ucde8\uc18c\ub418\uc5b4 7\ud68c \ub9cc\uc774 \ubc29\uc601\ub418\uc5c8\ub2e4.", "paragraph_id": "6523707-2-2", "paragraph_question": "question: \uc5e0\ub9c8\uc2a4\ud1a4\uc774 7\ud68c\ub9cc\uc5d0 \ud558\ucc28\ud574\uc57c \ud588\ub358 \ub4dc\ub77c\ub9c8\ub294?, context: 2004\ub144, VH1\uc758 \ud0e4\ub7f0\ud2b8 \ubc1c\uad74 \ud504\ub85c\uadf8\ub7a8 \u300aIn Search of the New Partridge Family\u300b\uc5d0\uc11c \ub85c\ub9ac \ud328\ud2b8\ub9ac\uc9c0 \uc5ed\ud560\uc744 \ub530\ub0b8 \ud6c4 \ud154\ub808\ube44\uc804\uc5d0\uc11c \uacbd\ub825\uc744 \uc2dc\uc791\ud588\ub2e4. \ud0e4\ub7f0\ud2b8 \ubc1c\uad74 \ud504\ub85c\uadf8\ub7a8\uc758 \uacb0\uacfc\ub85c, \uc774\ub4ec\ud574 2005\ub144 TV \ub4dc\ub77c\ub9c8 \u300a\uc0c8\ub85c\uc6b4 \ud30c\ud2b8\ub9ac\uc9c0 \ud328\ubc00\ub9ac\u300b\uac00 \uc81c\uc791\ub418\uc9c0\ub9cc, \ud30c\uc77c\ub7ff \uc5d0\ud53c\uc18c\ub4dc\ub9cc\uc774 \ubc29\uc601\ub418\uc5c8\ub2e4. \uc774\ud6c4 \u300a\uace0\uc2a4\ud2b8 & \ud06c\ub77c\uc784\u300b, \u300a\ub9d0\ucf64\ub124 \uc880 \ub9d0\ub824\uc918\u300b, \u300a\ub7ed\ud0a4 \ub8e8\uc774\u300b \ub4f1 \uc5f0\uc774\uc5b4 \uc815\uae30\uc801\uc73c\ub85c \ucd9c\uc5f0\ud558\uc600\ub2e4. NBC\uc758 SF \ub4dc\ub77c\ub9c8 \u300a\ud788\uc5b4\ub85c\uc988\u300b\uc758 \ud074\ub808\uc5b4 \ubca0\ub137 \uc5ed\ud560\uc5d0 \uc624\ub514\uc158\uc744 \ubcf4\uc558\uc9c0\ub9cc, \uacb0\uad6d \ud5e4\uc774\ub4e0 \ud328\ub124\ud2f0\uc5b4\uac00 \uce90\uc2a4\ud305\ub418\uc5c8\ub2e4. \uc2a4\ud1a4\uc740 \uc774 \uacbd\ud5d8\uc744 \"\ucd5c\uc545\"\uc774\ub77c\uace0 \ud558\uc600\ub2e4. 2007\ub144 4\uc6d4, \uc2a4\ud1a4\uc740 \ud3ed\uc2a4\uc758 \uc561\uc158 \ub4dc\ub77c\ub9c8 \u300a\ub4dc\ub77c\uc774\ube0c\u300b\uc5d0 \ubc14\uc774\uc5bc\ub9bf \ud2b8\ub9bc\ube14 \uc5ed\uc73c\ub85c \uace0\uc815 \ucd9c\uc5f0\ud558\uc600\uc73c\ub098, \ubc29\uc601 \ub3c4\uc911 \ucde8\uc18c\ub418\uc5b4 7\ud68c \ub9cc\uc774 \ubc29\uc601\ub418\uc5c8\ub2e4."} {"question": "\uc784\uc2dc\uc815\ubd80 \uc678\ubb34\ucd1d\uc7a5\uacfc \ud30c\ub9ac\uac15\ud654\ud68c\uc758 \uc784\uc2dc\uc815\ubd80 \ud55c\uad6d\ub300\ud45c\ub97c \uacb8\ud588\ub358 \uc0ac\ub78c\uc740?", "paragraph": "\uadf8\ub7ec\ub098 \uc2e0\ud55c\uccad\ub144\ub2f9\uc740 1922\ub144 12\uc6d4 \uc911\uc21c \uc0c1\ud558\uc774\uc5d0\uc11c \uc790\uc9c4\ud558\uc5ec \ud574\uc0b0\ud558\uae30\ub85c \uacb0\uc815\ud558\uc600\ub2e4. \uadf8 \uc774\uc720\ub294 \uc678\uad6d\uc5d0\uc11c \uc2e0\ud55c\uccad\ub144\ub2f9\uc758 \uc678\uad50\ud65c\ub3d9\uacfc \uc784\uc2dc\uc815\ubd80\uc758 \uc678\uad50\ud65c\ub3d9\uc744 \ud63c\ub3d9\ud55c \uacb0\uacfc \uc2e0\ud55c\uccad\ub144\ub2f9\uacfc \uc784\uc2dc\uc815\ubd80 \uadf8 \uc790\uccb4\ub97c \ud63c\ub3d9 \ud558\ub294 \uc77c\uc774 \uc790\uc8fc \uc788\uc5c8\uace0 \uadf8\ub85c \ub9d0\ubbf8\uc554\uc544 \uc784\uc2dc\uc815\ubd80\uce21\uc5d0\uc11c \uc6d0\ub85c\uc778 \uc2e0\ud55c\uccad\ub144\ub2f9\uc758 \ud574\uc0b0\uc744 \uc694\ub9dd \ud588\uae30 \ub54c\ubb38\uc774\uc5c8\ub2e4. \ud2b9\ud788 \uc2e0\ud55c\uccad\ub144\ub2f9 \ub300\ud45c\ub85c \ud30c\ub9ac\uac15\ud654\ud68c\uc758\uc5d0 \ud30c\uacac\ub41c \uae40\uaddc\uc2dd\uc774 \uc784\uc2dc\uc815\ubd80\uc5d0 \uc758\ud558\uc5ec \ud55c\uad6d\ub300\ud45c\ub97c \uacb8\ud558\uac8c \ub418\uc5c8\uace0 \ub610 \uc784\uc2dc\uc815\ubd80 \uc678\ubb34\ucd1d\uc7a5\uc744 \uacb8\ud558\uac8c \ub418\uc5c8\uc73c\ubbc0\ub85c \ud55c\uad6d\uc758 \uc2e4\uc815\uc744 \uc790\uc138\ud788 \ubaa8\ub974\ub294 \uc678\uad6d\uc778\ub4e4\uc740 \uc2e0\ud55c\uccad\ub144\ub2f9\uacfc \uc784\uc2dc\uc815\ubd80\ub97c \uc790\uc8fc \ud63c\ub3d9\ud558\uc600\ub2e4. \ub610\ud55c \uc2e0\ud55c\uccad\ub144\ub2f9\uacfc \uc784\uc2dc\uc815\ubd80\uac00 \ubaa8\ub450 \uc0c1\ud558\uc774\uc5d0 \uc788\uc5c8\ub2e4\ub294 \uc0ac\uc2e4\ub3c4 \ud63c\ub3d9\uc758 \ud55c \uc694\uc778\uc73c\ub85c \uc791\ub3d9\ud558\uc600\ub2e4. \uc2e0\ud55c\uccad\ub144\ub2f9 \ub2f9\uc6d0\ub4e4\uc740 1922\ub144 12\uc6d4 \uc911\uc21c \uc11c\ubcd1\ud638\uc758 \uc9d1\uc5d0\uc11c \ud68c\uc758\ub97c \uc5f4\uace0 \uc77c\ubd80\uc758 \ubc18\ub300\uc758\uacac\ub3c4 \uc788\uc5c8\uc9c0\ub9cc \ub300\ud55c\ubbfc\uad6d \uc784\uc2dc\uc815\ubd80\uc758 \ud574\uc0b0 \uc694\ub9dd\uc744 \ubc1b\uc544\ub4e4\uc774\uae30\ub85c \uacb0\uc758 \ud558\uc5ec 1922\ub144 12\uc6d4 \ud574\uc0b0\ud558\uc600\ub2e4.", "answer": "\uae40\uaddc\uc2dd", "sentence": "\ud2b9\ud788 \uc2e0\ud55c\uccad\ub144\ub2f9 \ub300\ud45c\ub85c \ud30c\ub9ac\uac15\ud654\ud68c\uc758\uc5d0 \ud30c\uacac\ub41c \uae40\uaddc\uc2dd \uc774 \uc784\uc2dc\uc815\ubd80\uc5d0 \uc758\ud558\uc5ec \ud55c\uad6d\ub300\ud45c\ub97c \uacb8\ud558\uac8c \ub418\uc5c8\uace0 \ub610 \uc784\uc2dc\uc815\ubd80 \uc678\ubb34\ucd1d\uc7a5\uc744 \uacb8\ud558\uac8c \ub418\uc5c8\uc73c\ubbc0\ub85c \ud55c\uad6d\uc758 \uc2e4\uc815\uc744 \uc790\uc138\ud788 \ubaa8\ub974\ub294 \uc678\uad6d\uc778\ub4e4\uc740 \uc2e0\ud55c\uccad\ub144\ub2f9\uacfc \uc784\uc2dc\uc815\ubd80\ub97c \uc790\uc8fc \ud63c\ub3d9\ud558\uc600\ub2e4.", "paragraph_sentence": "\uadf8\ub7ec\ub098 \uc2e0\ud55c\uccad\ub144\ub2f9\uc740 1922\ub144 12\uc6d4 \uc911\uc21c \uc0c1\ud558\uc774\uc5d0\uc11c \uc790\uc9c4\ud558\uc5ec \ud574\uc0b0\ud558\uae30\ub85c \uacb0\uc815\ud558\uc600\ub2e4. \uadf8 \uc774\uc720\ub294 \uc678\uad6d\uc5d0\uc11c \uc2e0\ud55c\uccad\ub144\ub2f9\uc758 \uc678\uad50\ud65c\ub3d9\uacfc \uc784\uc2dc\uc815\ubd80\uc758 \uc678\uad50\ud65c\ub3d9\uc744 \ud63c\ub3d9\ud55c \uacb0\uacfc \uc2e0\ud55c\uccad\ub144\ub2f9\uacfc \uc784\uc2dc\uc815\ubd80 \uadf8 \uc790\uccb4\ub97c \ud63c\ub3d9 \ud558\ub294 \uc77c\uc774 \uc790\uc8fc \uc788\uc5c8\uace0 \uadf8\ub85c \ub9d0\ubbf8\uc554\uc544 \uc784\uc2dc\uc815\ubd80\uce21\uc5d0\uc11c \uc6d0\ub85c\uc778 \uc2e0\ud55c\uccad\ub144\ub2f9\uc758 \ud574\uc0b0\uc744 \uc694\ub9dd \ud588\uae30 \ub54c\ubb38\uc774\uc5c8\ub2e4. \ud2b9\ud788 \uc2e0\ud55c\uccad\ub144\ub2f9 \ub300\ud45c\ub85c \ud30c\ub9ac\uac15\ud654\ud68c\uc758\uc5d0 \ud30c\uacac\ub41c \uae40\uaddc\uc2dd \uc774 \uc784\uc2dc\uc815\ubd80\uc5d0 \uc758\ud558\uc5ec \ud55c\uad6d\ub300\ud45c\ub97c \uacb8\ud558\uac8c \ub418\uc5c8\uace0 \ub610 \uc784\uc2dc\uc815\ubd80 \uc678\ubb34\ucd1d\uc7a5\uc744 \uacb8\ud558\uac8c \ub418\uc5c8\uc73c\ubbc0\ub85c \ud55c\uad6d\uc758 \uc2e4\uc815\uc744 \uc790\uc138\ud788 \ubaa8\ub974\ub294 \uc678\uad6d\uc778\ub4e4\uc740 \uc2e0\ud55c\uccad\ub144\ub2f9\uacfc \uc784\uc2dc\uc815\ubd80\ub97c \uc790\uc8fc \ud63c\ub3d9\ud558\uc600\ub2e4. \ub610\ud55c \uc2e0\ud55c\uccad\ub144\ub2f9\uacfc \uc784\uc2dc\uc815\ubd80\uac00 \ubaa8\ub450 \uc0c1\ud558\uc774\uc5d0 \uc788\uc5c8\ub2e4\ub294 \uc0ac\uc2e4\ub3c4 \ud63c\ub3d9\uc758 \ud55c \uc694\uc778\uc73c\ub85c \uc791\ub3d9\ud558\uc600\ub2e4. \uc2e0\ud55c\uccad\ub144\ub2f9 \ub2f9\uc6d0\ub4e4\uc740 1922\ub144 12\uc6d4 \uc911\uc21c \uc11c\ubcd1\ud638\uc758 \uc9d1\uc5d0\uc11c \ud68c\uc758\ub97c \uc5f4\uace0 \uc77c\ubd80\uc758 \ubc18\ub300\uc758\uacac\ub3c4 \uc788\uc5c8\uc9c0\ub9cc \ub300\ud55c\ubbfc\uad6d \uc784\uc2dc\uc815\ubd80\uc758 \ud574\uc0b0 \uc694\ub9dd\uc744 \ubc1b\uc544\ub4e4\uc774\uae30\ub85c \uacb0\uc758 \ud558\uc5ec 1922\ub144 12\uc6d4 \ud574\uc0b0\ud558\uc600\ub2e4.", "paragraph_answer": "\uadf8\ub7ec\ub098 \uc2e0\ud55c\uccad\ub144\ub2f9\uc740 1922\ub144 12\uc6d4 \uc911\uc21c \uc0c1\ud558\uc774\uc5d0\uc11c \uc790\uc9c4\ud558\uc5ec \ud574\uc0b0\ud558\uae30\ub85c \uacb0\uc815\ud558\uc600\ub2e4. \uadf8 \uc774\uc720\ub294 \uc678\uad6d\uc5d0\uc11c \uc2e0\ud55c\uccad\ub144\ub2f9\uc758 \uc678\uad50\ud65c\ub3d9\uacfc \uc784\uc2dc\uc815\ubd80\uc758 \uc678\uad50\ud65c\ub3d9\uc744 \ud63c\ub3d9\ud55c \uacb0\uacfc \uc2e0\ud55c\uccad\ub144\ub2f9\uacfc \uc784\uc2dc\uc815\ubd80 \uadf8 \uc790\uccb4\ub97c \ud63c\ub3d9 \ud558\ub294 \uc77c\uc774 \uc790\uc8fc \uc788\uc5c8\uace0 \uadf8\ub85c \ub9d0\ubbf8\uc554\uc544 \uc784\uc2dc\uc815\ubd80\uce21\uc5d0\uc11c \uc6d0\ub85c\uc778 \uc2e0\ud55c\uccad\ub144\ub2f9\uc758 \ud574\uc0b0\uc744 \uc694\ub9dd \ud588\uae30 \ub54c\ubb38\uc774\uc5c8\ub2e4. \ud2b9\ud788 \uc2e0\ud55c\uccad\ub144\ub2f9 \ub300\ud45c\ub85c \ud30c\ub9ac\uac15\ud654\ud68c\uc758\uc5d0 \ud30c\uacac\ub41c \uae40\uaddc\uc2dd \uc774 \uc784\uc2dc\uc815\ubd80\uc5d0 \uc758\ud558\uc5ec \ud55c\uad6d\ub300\ud45c\ub97c \uacb8\ud558\uac8c \ub418\uc5c8\uace0 \ub610 \uc784\uc2dc\uc815\ubd80 \uc678\ubb34\ucd1d\uc7a5\uc744 \uacb8\ud558\uac8c \ub418\uc5c8\uc73c\ubbc0\ub85c \ud55c\uad6d\uc758 \uc2e4\uc815\uc744 \uc790\uc138\ud788 \ubaa8\ub974\ub294 \uc678\uad6d\uc778\ub4e4\uc740 \uc2e0\ud55c\uccad\ub144\ub2f9\uacfc \uc784\uc2dc\uc815\ubd80\ub97c \uc790\uc8fc \ud63c\ub3d9\ud558\uc600\ub2e4. \ub610\ud55c \uc2e0\ud55c\uccad\ub144\ub2f9\uacfc \uc784\uc2dc\uc815\ubd80\uac00 \ubaa8\ub450 \uc0c1\ud558\uc774\uc5d0 \uc788\uc5c8\ub2e4\ub294 \uc0ac\uc2e4\ub3c4 \ud63c\ub3d9\uc758 \ud55c \uc694\uc778\uc73c\ub85c \uc791\ub3d9\ud558\uc600\ub2e4. \uc2e0\ud55c\uccad\ub144\ub2f9 \ub2f9\uc6d0\ub4e4\uc740 1922\ub144 12\uc6d4 \uc911\uc21c \uc11c\ubcd1\ud638\uc758 \uc9d1\uc5d0\uc11c \ud68c\uc758\ub97c \uc5f4\uace0 \uc77c\ubd80\uc758 \ubc18\ub300\uc758\uacac\ub3c4 \uc788\uc5c8\uc9c0\ub9cc \ub300\ud55c\ubbfc\uad6d \uc784\uc2dc\uc815\ubd80\uc758 \ud574\uc0b0 \uc694\ub9dd\uc744 \ubc1b\uc544\ub4e4\uc774\uae30\ub85c \uacb0\uc758 \ud558\uc5ec 1922\ub144 12\uc6d4 \ud574\uc0b0\ud558\uc600\ub2e4.", "sentence_answer": "\ud2b9\ud788 \uc2e0\ud55c\uccad\ub144\ub2f9 \ub300\ud45c\ub85c \ud30c\ub9ac\uac15\ud654\ud68c\uc758\uc5d0 \ud30c\uacac\ub41c \uae40\uaddc\uc2dd \uc774 \uc784\uc2dc\uc815\ubd80\uc5d0 \uc758\ud558\uc5ec \ud55c\uad6d\ub300\ud45c\ub97c \uacb8\ud558\uac8c \ub418\uc5c8\uace0 \ub610 \uc784\uc2dc\uc815\ubd80 \uc678\ubb34\ucd1d\uc7a5\uc744 \uacb8\ud558\uac8c \ub418\uc5c8\uc73c\ubbc0\ub85c \ud55c\uad6d\uc758 \uc2e4\uc815\uc744 \uc790\uc138\ud788 \ubaa8\ub974\ub294 \uc678\uad6d\uc778\ub4e4\uc740 \uc2e0\ud55c\uccad\ub144\ub2f9\uacfc \uc784\uc2dc\uc815\ubd80\ub97c \uc790\uc8fc \ud63c\ub3d9\ud558\uc600\ub2e4.", "paragraph_id": "6519837-9-1", "paragraph_question": "question: \uc784\uc2dc\uc815\ubd80 \uc678\ubb34\ucd1d\uc7a5\uacfc \ud30c\ub9ac\uac15\ud654\ud68c\uc758 \uc784\uc2dc\uc815\ubd80 \ud55c\uad6d\ub300\ud45c\ub97c \uacb8\ud588\ub358 \uc0ac\ub78c\uc740?, context: \uadf8\ub7ec\ub098 \uc2e0\ud55c\uccad\ub144\ub2f9\uc740 1922\ub144 12\uc6d4 \uc911\uc21c \uc0c1\ud558\uc774\uc5d0\uc11c \uc790\uc9c4\ud558\uc5ec \ud574\uc0b0\ud558\uae30\ub85c \uacb0\uc815\ud558\uc600\ub2e4. \uadf8 \uc774\uc720\ub294 \uc678\uad6d\uc5d0\uc11c \uc2e0\ud55c\uccad\ub144\ub2f9\uc758 \uc678\uad50\ud65c\ub3d9\uacfc \uc784\uc2dc\uc815\ubd80\uc758 \uc678\uad50\ud65c\ub3d9\uc744 \ud63c\ub3d9\ud55c \uacb0\uacfc \uc2e0\ud55c\uccad\ub144\ub2f9\uacfc \uc784\uc2dc\uc815\ubd80 \uadf8 \uc790\uccb4\ub97c \ud63c\ub3d9 \ud558\ub294 \uc77c\uc774 \uc790\uc8fc \uc788\uc5c8\uace0 \uadf8\ub85c \ub9d0\ubbf8\uc554\uc544 \uc784\uc2dc\uc815\ubd80\uce21\uc5d0\uc11c \uc6d0\ub85c\uc778 \uc2e0\ud55c\uccad\ub144\ub2f9\uc758 \ud574\uc0b0\uc744 \uc694\ub9dd \ud588\uae30 \ub54c\ubb38\uc774\uc5c8\ub2e4. \ud2b9\ud788 \uc2e0\ud55c\uccad\ub144\ub2f9 \ub300\ud45c\ub85c \ud30c\ub9ac\uac15\ud654\ud68c\uc758\uc5d0 \ud30c\uacac\ub41c \uae40\uaddc\uc2dd\uc774 \uc784\uc2dc\uc815\ubd80\uc5d0 \uc758\ud558\uc5ec \ud55c\uad6d\ub300\ud45c\ub97c \uacb8\ud558\uac8c \ub418\uc5c8\uace0 \ub610 \uc784\uc2dc\uc815\ubd80 \uc678\ubb34\ucd1d\uc7a5\uc744 \uacb8\ud558\uac8c \ub418\uc5c8\uc73c\ubbc0\ub85c \ud55c\uad6d\uc758 \uc2e4\uc815\uc744 \uc790\uc138\ud788 \ubaa8\ub974\ub294 \uc678\uad6d\uc778\ub4e4\uc740 \uc2e0\ud55c\uccad\ub144\ub2f9\uacfc \uc784\uc2dc\uc815\ubd80\ub97c \uc790\uc8fc \ud63c\ub3d9\ud558\uc600\ub2e4. \ub610\ud55c \uc2e0\ud55c\uccad\ub144\ub2f9\uacfc \uc784\uc2dc\uc815\ubd80\uac00 \ubaa8\ub450 \uc0c1\ud558\uc774\uc5d0 \uc788\uc5c8\ub2e4\ub294 \uc0ac\uc2e4\ub3c4 \ud63c\ub3d9\uc758 \ud55c \uc694\uc778\uc73c\ub85c \uc791\ub3d9\ud558\uc600\ub2e4. \uc2e0\ud55c\uccad\ub144\ub2f9 \ub2f9\uc6d0\ub4e4\uc740 1922\ub144 12\uc6d4 \uc911\uc21c \uc11c\ubcd1\ud638\uc758 \uc9d1\uc5d0\uc11c \ud68c\uc758\ub97c \uc5f4\uace0 \uc77c\ubd80\uc758 \ubc18\ub300\uc758\uacac\ub3c4 \uc788\uc5c8\uc9c0\ub9cc \ub300\ud55c\ubbfc\uad6d \uc784\uc2dc\uc815\ubd80\uc758 \ud574\uc0b0 \uc694\ub9dd\uc744 \ubc1b\uc544\ub4e4\uc774\uae30\ub85c \uacb0\uc758 \ud558\uc5ec 1922\ub144 12\uc6d4 \ud574\uc0b0\ud558\uc600\ub2e4."} {"question": "\ubc15\ud0dc\ud658\uc774 \ud0dc\uc5b4\ub09c \ub144\ub3c4\ub294?", "paragraph": "\ubc15\ud0dc\ud658(\u6734\u6cf0\u6853, 1989\ub144 9\uc6d4 27\uc77c ~ )\uc740 \ub300\ud55c\ubbfc\uad6d\uc758 \uc218\uc601 \uc120\uc218\uc774\ub2e4. \ub300\ud55c\ubbfc\uad6d\uc758 \uc218\uc601 \uc5ed\uc0ac\uc0c1 \ucd5c\ucd08\ub85c 2008\ub144 \ubca0\uc774\uc9d5 \uc62c\ub9bc\ud53d \uc790\uc720\ud615 400m\uc5d0\uc11c\uc758 \uae08\uba54\ub2ec\uacfc \uc790\uc720\ud615 200m\uc5d0\uc11c\uc758 \uc740\uba54\ub2ec\uc744 \ud68d\ub4dd\ud568\uc73c\ub85c\uc368 \ud55c\uad6d\uc778 \ucd5c\ucd08\ub85c \uc218\uc601 \uc62c\ub9bc\ud53d \uba54\ub2ec\uc744 \ud68d\ub4dd\ud558\uc600\ub2e4. \ub610\ud55c 2012\ub144 \ud558\uacc4 \uc62c\ub9bc\ud53d \uc790\uc720\ud615 400\ubbf8\ud130\uc11c \uc2e4\uaca9 \ud310\uc815 \ubc88\ubcf5 \uc18d\uc5d0\uc11c\ub3c4 \ucd5c\uc120\uc744 \ub2e4\ud574 \uc740\uba54\ub2ec\uc744 \ud68d\ub4dd\ud55c \ub370 \uc774\uc5b4, \uc218\uc601\uacc4\uc5d0\uc11c \uac00\uc7a5 \uacbd\uc7c1\uc774 \uce58\uc5f4\ud558\ub2e4\ub294 \uc790\uc720\ud615 200\ubbf8\ud130\uc5d0\uc11c\ub3c4 \uac12\uc9c4 \uc740\uba54\ub2ec\uc744 \ucc28\uc9c0\ud558\uc5ec \ud55c\uad6d \uc218\uc601 \ucd5c\ucd08\ub85c 2\ud68c \uc5f0\uc18d \uc62c\ub9bc\ud53d\uc5d0\uc11c \uba54\ub2ec\uc744 \ud68d\ub4dd\ud55c \uc120\uc218\uac00 \ub410\ub2e4. \uad6d\ub0b4\uc5d0\uc11c \uc131\uc801\uc740 \uc804\uad6d\uccb4\uc721\ub300\ud68c\uc5d0\uc11c 2005\ub144 \uc6b8\uc0b0, 2007\ub144 \uad11\uc8fc, 2008\ub144 \uc804\ub0a8 \ub300\ud68c\uae4c\uc9c0 3\ubc88\uc758 \ub300\ud68c MVP\ub97c \uc218\uc0c1\ud558\uc600\uc73c\uba70 \uadf8\ub3d9\uc548 2005\ub144 \uc804\uad6d\uccb4\uc804 4\uad00\uc655\uc744 \ube44\ub86f 2006\ub144 ~ 2008\ub144\uae4c\uc9c0 3\ub144 \uc5f0\uc18d 5\uad00\uc655\uc5d0 \uc62c\ub790\ub2e4. \ub610\ud55c 2004\ub144\u300a\uc81c28\ud68c \uc544\ud14c\ub124 \uc62c\ub9bc\ud53d\u300b\uc218\uc601 \uad6d\uac00\ub300\ud45c, 2006\ub144\u300a\uc81c15\ud68c \ub3c4\ud558 \uc544\uc2dc\uc548\uac8c\uc784\u300b\uc218\uc601 \uad6d\uac00\ub300\ud45c, 2008\ub144\u300a\uc81c29\ud68c \ubca0\uc774\uc9d5 \uc62c\ub9bc\ud53d\u300b\uc218\uc601 \uad6d\uac00\ub300\ud45c, 2010\ub144\u300a\uc81c16\ud68c \uad11\uc800\uc6b0 \uc544\uc2dc\uc548\uac8c\uc784\u300b \uc218\uc601 \uad6d\uac00\ub300\ud45c, 2012\ub144\u300a\uc81c30\ud68c \ub7f0\ub358 \uc62c\ub9bc\ud53d\u300b \uc218\uc601 \uad6d\uac00\ub300\ud45c, 2014\ub144\u300a\uc81c17\ud68c \uc778\ucc9c \uc544\uc2dc\uc548\uac8c\uc784\u300b\uc218\uc601 \uad6d\uac00\ub300\ud45c, 2016\ub144\u300a\uc81c31\ud68c \ub9ac\uc6b0\ub370\uc790\ub124\uc774\ub8e8 \uc62c\ub9bc\ud53d\u300b\uc218\uc601 \uad6d\uac00\ub300\ud45c\ub85c \ubf51\ud614\ub2e4.", "answer": "1989\ub144", "sentence": "\ubc15\ud0dc\ud658(\u6734\u6cf0\u6853, 1989\ub144 9\uc6d4 27\uc77c ~ )\uc740 \ub300\ud55c\ubbfc\uad6d\uc758 \uc218\uc601 \uc120\uc218\uc774\ub2e4.", "paragraph_sentence": " \ubc15\ud0dc\ud658(\u6734\u6cf0\u6853, 1989\ub144 9\uc6d4 27\uc77c ~ )\uc740 \ub300\ud55c\ubbfc\uad6d\uc758 \uc218\uc601 \uc120\uc218\uc774\ub2e4. \ub300\ud55c\ubbfc\uad6d\uc758 \uc218\uc601 \uc5ed\uc0ac\uc0c1 \ucd5c\ucd08\ub85c 2008\ub144 \ubca0\uc774\uc9d5 \uc62c\ub9bc\ud53d \uc790\uc720\ud615 400m\uc5d0\uc11c\uc758 \uae08\uba54\ub2ec\uacfc \uc790\uc720\ud615 200m\uc5d0\uc11c\uc758 \uc740\uba54\ub2ec\uc744 \ud68d\ub4dd\ud568\uc73c\ub85c\uc368 \ud55c\uad6d\uc778 \ucd5c\ucd08\ub85c \uc218\uc601 \uc62c\ub9bc\ud53d \uba54\ub2ec\uc744 \ud68d\ub4dd\ud558\uc600\ub2e4. \ub610\ud55c 2012\ub144 \ud558\uacc4 \uc62c\ub9bc\ud53d \uc790\uc720\ud615 400\ubbf8\ud130\uc11c \uc2e4\uaca9 \ud310\uc815 \ubc88\ubcf5 \uc18d\uc5d0\uc11c\ub3c4 \ucd5c\uc120\uc744 \ub2e4\ud574 \uc740\uba54\ub2ec\uc744 \ud68d\ub4dd\ud55c \ub370 \uc774\uc5b4, \uc218\uc601\uacc4\uc5d0\uc11c \uac00\uc7a5 \uacbd\uc7c1\uc774 \uce58\uc5f4\ud558\ub2e4\ub294 \uc790\uc720\ud615 200\ubbf8\ud130\uc5d0\uc11c\ub3c4 \uac12\uc9c4 \uc740\uba54\ub2ec\uc744 \ucc28\uc9c0\ud558\uc5ec \ud55c\uad6d \uc218\uc601 \ucd5c\ucd08\ub85c 2\ud68c \uc5f0\uc18d \uc62c\ub9bc\ud53d\uc5d0\uc11c \uba54\ub2ec\uc744 \ud68d\ub4dd\ud55c \uc120\uc218\uac00 \ub410\ub2e4. \uad6d\ub0b4\uc5d0\uc11c \uc131\uc801\uc740 \uc804\uad6d\uccb4\uc721\ub300\ud68c\uc5d0\uc11c 2005\ub144 \uc6b8\uc0b0, 2007\ub144 \uad11\uc8fc, 2008\ub144 \uc804\ub0a8 \ub300\ud68c\uae4c\uc9c0 3\ubc88\uc758 \ub300\ud68c MVP\ub97c \uc218\uc0c1\ud558\uc600\uc73c\uba70 \uadf8\ub3d9\uc548 2005\ub144 \uc804\uad6d\uccb4\uc804 4\uad00\uc655\uc744 \ube44\ub86f 2006\ub144 ~ 2008\ub144\uae4c\uc9c0 3\ub144 \uc5f0\uc18d 5\uad00\uc655\uc5d0 \uc62c\ub790\ub2e4. \ub610\ud55c 2004\ub144\u300a\uc81c28\ud68c \uc544\ud14c\ub124 \uc62c\ub9bc\ud53d\u300b\uc218\uc601 \uad6d\uac00\ub300\ud45c, 2006\ub144\u300a\uc81c15\ud68c \ub3c4\ud558 \uc544\uc2dc\uc548\uac8c\uc784\u300b\uc218\uc601 \uad6d\uac00\ub300\ud45c, 2008\ub144\u300a\uc81c29\ud68c \ubca0\uc774\uc9d5 \uc62c\ub9bc\ud53d\u300b\uc218\uc601 \uad6d\uac00\ub300\ud45c, 2010\ub144\u300a\uc81c16\ud68c \uad11\uc800\uc6b0 \uc544\uc2dc\uc548\uac8c\uc784\u300b \uc218\uc601 \uad6d\uac00\ub300\ud45c, 2012\ub144\u300a\uc81c30\ud68c \ub7f0\ub358 \uc62c\ub9bc\ud53d\u300b \uc218\uc601 \uad6d\uac00\ub300\ud45c, 2014\ub144\u300a\uc81c17\ud68c \uc778\ucc9c \uc544\uc2dc\uc548\uac8c\uc784\u300b\uc218\uc601 \uad6d\uac00\ub300\ud45c, 2016\ub144\u300a\uc81c31\ud68c \ub9ac\uc6b0\ub370\uc790\ub124\uc774\ub8e8 \uc62c\ub9bc\ud53d\u300b\uc218\uc601 \uad6d\uac00\ub300\ud45c\ub85c \ubf51\ud614\ub2e4.", "paragraph_answer": "\ubc15\ud0dc\ud658(\u6734\u6cf0\u6853, 1989\ub144 9\uc6d4 27\uc77c ~ )\uc740 \ub300\ud55c\ubbfc\uad6d\uc758 \uc218\uc601 \uc120\uc218\uc774\ub2e4. \ub300\ud55c\ubbfc\uad6d\uc758 \uc218\uc601 \uc5ed\uc0ac\uc0c1 \ucd5c\ucd08\ub85c 2008\ub144 \ubca0\uc774\uc9d5 \uc62c\ub9bc\ud53d \uc790\uc720\ud615 400m\uc5d0\uc11c\uc758 \uae08\uba54\ub2ec\uacfc \uc790\uc720\ud615 200m\uc5d0\uc11c\uc758 \uc740\uba54\ub2ec\uc744 \ud68d\ub4dd\ud568\uc73c\ub85c\uc368 \ud55c\uad6d\uc778 \ucd5c\ucd08\ub85c \uc218\uc601 \uc62c\ub9bc\ud53d \uba54\ub2ec\uc744 \ud68d\ub4dd\ud558\uc600\ub2e4. \ub610\ud55c 2012\ub144 \ud558\uacc4 \uc62c\ub9bc\ud53d \uc790\uc720\ud615 400\ubbf8\ud130\uc11c \uc2e4\uaca9 \ud310\uc815 \ubc88\ubcf5 \uc18d\uc5d0\uc11c\ub3c4 \ucd5c\uc120\uc744 \ub2e4\ud574 \uc740\uba54\ub2ec\uc744 \ud68d\ub4dd\ud55c \ub370 \uc774\uc5b4, \uc218\uc601\uacc4\uc5d0\uc11c \uac00\uc7a5 \uacbd\uc7c1\uc774 \uce58\uc5f4\ud558\ub2e4\ub294 \uc790\uc720\ud615 200\ubbf8\ud130\uc5d0\uc11c\ub3c4 \uac12\uc9c4 \uc740\uba54\ub2ec\uc744 \ucc28\uc9c0\ud558\uc5ec \ud55c\uad6d \uc218\uc601 \ucd5c\ucd08\ub85c 2\ud68c \uc5f0\uc18d \uc62c\ub9bc\ud53d\uc5d0\uc11c \uba54\ub2ec\uc744 \ud68d\ub4dd\ud55c \uc120\uc218\uac00 \ub410\ub2e4. \uad6d\ub0b4\uc5d0\uc11c \uc131\uc801\uc740 \uc804\uad6d\uccb4\uc721\ub300\ud68c\uc5d0\uc11c 2005\ub144 \uc6b8\uc0b0, 2007\ub144 \uad11\uc8fc, 2008\ub144 \uc804\ub0a8 \ub300\ud68c\uae4c\uc9c0 3\ubc88\uc758 \ub300\ud68c MVP\ub97c \uc218\uc0c1\ud558\uc600\uc73c\uba70 \uadf8\ub3d9\uc548 2005\ub144 \uc804\uad6d\uccb4\uc804 4\uad00\uc655\uc744 \ube44\ub86f 2006\ub144 ~ 2008\ub144\uae4c\uc9c0 3\ub144 \uc5f0\uc18d 5\uad00\uc655\uc5d0 \uc62c\ub790\ub2e4. \ub610\ud55c 2004\ub144\u300a\uc81c28\ud68c \uc544\ud14c\ub124 \uc62c\ub9bc\ud53d\u300b\uc218\uc601 \uad6d\uac00\ub300\ud45c, 2006\ub144\u300a\uc81c15\ud68c \ub3c4\ud558 \uc544\uc2dc\uc548\uac8c\uc784\u300b\uc218\uc601 \uad6d\uac00\ub300\ud45c, 2008\ub144\u300a\uc81c29\ud68c \ubca0\uc774\uc9d5 \uc62c\ub9bc\ud53d\u300b\uc218\uc601 \uad6d\uac00\ub300\ud45c, 2010\ub144\u300a\uc81c16\ud68c \uad11\uc800\uc6b0 \uc544\uc2dc\uc548\uac8c\uc784\u300b \uc218\uc601 \uad6d\uac00\ub300\ud45c, 2012\ub144\u300a\uc81c30\ud68c \ub7f0\ub358 \uc62c\ub9bc\ud53d\u300b \uc218\uc601 \uad6d\uac00\ub300\ud45c, 2014\ub144\u300a\uc81c17\ud68c \uc778\ucc9c \uc544\uc2dc\uc548\uac8c\uc784\u300b\uc218\uc601 \uad6d\uac00\ub300\ud45c, 2016\ub144\u300a\uc81c31\ud68c \ub9ac\uc6b0\ub370\uc790\ub124\uc774\ub8e8 \uc62c\ub9bc\ud53d\u300b\uc218\uc601 \uad6d\uac00\ub300\ud45c\ub85c \ubf51\ud614\ub2e4.", "sentence_answer": "\ubc15\ud0dc\ud658(\u6734\u6cf0\u6853, 1989\ub144 9\uc6d4 27\uc77c ~ )\uc740 \ub300\ud55c\ubbfc\uad6d\uc758 \uc218\uc601 \uc120\uc218\uc774\ub2e4.", "paragraph_id": "6530244-0-0", "paragraph_question": "question: \ubc15\ud0dc\ud658\uc774 \ud0dc\uc5b4\ub09c \ub144\ub3c4\ub294?, context: \ubc15\ud0dc\ud658(\u6734\u6cf0\u6853, 1989\ub144 9\uc6d4 27\uc77c ~ )\uc740 \ub300\ud55c\ubbfc\uad6d\uc758 \uc218\uc601 \uc120\uc218\uc774\ub2e4. \ub300\ud55c\ubbfc\uad6d\uc758 \uc218\uc601 \uc5ed\uc0ac\uc0c1 \ucd5c\ucd08\ub85c 2008\ub144 \ubca0\uc774\uc9d5 \uc62c\ub9bc\ud53d \uc790\uc720\ud615 400m\uc5d0\uc11c\uc758 \uae08\uba54\ub2ec\uacfc \uc790\uc720\ud615 200m\uc5d0\uc11c\uc758 \uc740\uba54\ub2ec\uc744 \ud68d\ub4dd\ud568\uc73c\ub85c\uc368 \ud55c\uad6d\uc778 \ucd5c\ucd08\ub85c \uc218\uc601 \uc62c\ub9bc\ud53d \uba54\ub2ec\uc744 \ud68d\ub4dd\ud558\uc600\ub2e4. \ub610\ud55c 2012\ub144 \ud558\uacc4 \uc62c\ub9bc\ud53d \uc790\uc720\ud615 400\ubbf8\ud130\uc11c \uc2e4\uaca9 \ud310\uc815 \ubc88\ubcf5 \uc18d\uc5d0\uc11c\ub3c4 \ucd5c\uc120\uc744 \ub2e4\ud574 \uc740\uba54\ub2ec\uc744 \ud68d\ub4dd\ud55c \ub370 \uc774\uc5b4, \uc218\uc601\uacc4\uc5d0\uc11c \uac00\uc7a5 \uacbd\uc7c1\uc774 \uce58\uc5f4\ud558\ub2e4\ub294 \uc790\uc720\ud615 200\ubbf8\ud130\uc5d0\uc11c\ub3c4 \uac12\uc9c4 \uc740\uba54\ub2ec\uc744 \ucc28\uc9c0\ud558\uc5ec \ud55c\uad6d \uc218\uc601 \ucd5c\ucd08\ub85c 2\ud68c \uc5f0\uc18d \uc62c\ub9bc\ud53d\uc5d0\uc11c \uba54\ub2ec\uc744 \ud68d\ub4dd\ud55c \uc120\uc218\uac00 \ub410\ub2e4. \uad6d\ub0b4\uc5d0\uc11c \uc131\uc801\uc740 \uc804\uad6d\uccb4\uc721\ub300\ud68c\uc5d0\uc11c 2005\ub144 \uc6b8\uc0b0, 2007\ub144 \uad11\uc8fc, 2008\ub144 \uc804\ub0a8 \ub300\ud68c\uae4c\uc9c0 3\ubc88\uc758 \ub300\ud68c MVP\ub97c \uc218\uc0c1\ud558\uc600\uc73c\uba70 \uadf8\ub3d9\uc548 2005\ub144 \uc804\uad6d\uccb4\uc804 4\uad00\uc655\uc744 \ube44\ub86f 2006\ub144 ~ 2008\ub144\uae4c\uc9c0 3\ub144 \uc5f0\uc18d 5\uad00\uc655\uc5d0 \uc62c\ub790\ub2e4. \ub610\ud55c 2004\ub144\u300a\uc81c28\ud68c \uc544\ud14c\ub124 \uc62c\ub9bc\ud53d\u300b\uc218\uc601 \uad6d\uac00\ub300\ud45c, 2006\ub144\u300a\uc81c15\ud68c \ub3c4\ud558 \uc544\uc2dc\uc548\uac8c\uc784\u300b\uc218\uc601 \uad6d\uac00\ub300\ud45c, 2008\ub144\u300a\uc81c29\ud68c \ubca0\uc774\uc9d5 \uc62c\ub9bc\ud53d\u300b\uc218\uc601 \uad6d\uac00\ub300\ud45c, 2010\ub144\u300a\uc81c16\ud68c \uad11\uc800\uc6b0 \uc544\uc2dc\uc548\uac8c\uc784\u300b \uc218\uc601 \uad6d\uac00\ub300\ud45c, 2012\ub144\u300a\uc81c30\ud68c \ub7f0\ub358 \uc62c\ub9bc\ud53d\u300b \uc218\uc601 \uad6d\uac00\ub300\ud45c, 2014\ub144\u300a\uc81c17\ud68c \uc778\ucc9c \uc544\uc2dc\uc548\uac8c\uc784\u300b\uc218\uc601 \uad6d\uac00\ub300\ud45c, 2016\ub144\u300a\uc81c31\ud68c \ub9ac\uc6b0\ub370\uc790\ub124\uc774\ub8e8 \uc62c\ub9bc\ud53d\u300b\uc218\uc601 \uad6d\uac00\ub300\ud45c\ub85c \ubf51\ud614\ub2e4."} {"question": "\uc870\ubcc4 \uacbd\uae30\uc5d0\uc11c \uac01 \ud300\ub4e4\uc740 \ucd5c\uc18c\ud55c \uba87 \uacbd\uae30\ub97c \uce58\ub974\ub098?", "paragraph": "\uac01 \uc870\ub294 \ub9ac\uadf8\uc804 \ud615\uc2dd\uc73c\ub85c \uce58\ub7ec\uc9c0\uba70 \uc774\uc5d0 \ub530\ub77c \ubaa8\ub4e0 \ud300\uc774 \uc801\uc5b4\ub3c4 3\uacbd\uae30\ub97c \uce58\ub97c \uc218 \uc788\ub2e4. \uac01 \uc870\uc758 \ub9c8\uc9c0\ub9c9 \ub77c\uc6b4\ub4dc \uacbd\uae30\ub294 \uac01 \ud300 \uac04\uc758 \uacf5\uc815\uc131 \uc720\uc9c0 \ubc0f \uc2b9\ubd80\uc870\uc791\uc744 \ubc29\uc9c0\ud558\uae30 \uc704\ud574 \uac19\uc740 \uc2dc\uac04\uc5d0 \uacbd\uae30\ub97c \uce58\ub974\ub3c4\ub85d \ud558\uace0 \uc788\ub2e4. \uac01 \uc870\uc5d0\uc11c \uc0c1\uc704 \ub450 \uac1c \ud300\uc774 \ubcf8\uc120 \ud1a0\ub108\uba3c\ud2b8\ub85c \uc9c4\ucd9c\ud55c\ub2e4. \uc2b9\uc810\uc740 \uc870 \uc548\uc5d0\uc11c\uc758 \uc21c\uc704\ub97c \uac00\ub974\uae30 \uc704\ud574 \uc0ac\uc6a9\ub41c\ub2e4. 1994\ub144\ubd80\ud130, \uacbd\uae30\uc5d0 \uc2b9\ub9ac\ud588\uc744 \ub54c\ub294 3\uc810\uc774 \uc8fc\uc5b4\uc84c\uace0 \ube44\uacbc\uc744 \ub54c\ub294 1\uc810\uc774 \uc8fc\uc5b4\uc84c\uc73c\uba70 \ud328\ud588\uc744 \ub54c\ub294 0\uc810\uc774 \uc8fc\uc5b4\uc84c\ub2e4. (\uc774 \uc774\uc804\uae4c\uc9c0 \uc2b9\ub9ac \uc2dc \uc2b9\uc810\uc740 2\uc810\uc774\uc5c8\ub2e4.) \ub9cc\uc57d \ub450 \uac1c \ud639\uc740 \uadf8 \uc774\uc0c1\uc758 \ud300\uc774 \uc2b9\uc810\uc774 \uac19\ub2e4\uba74, \ub3d9\uc810 \uc2dc \ucc98\ub9ac \ubc29\ubc95\uc774 \uc0ac\uc6a9\ub41c\ub2e4. \ucc98\uc74c\uc740 \uace8 \ub4dd\uc2e4 \ucc28\ub85c, \uadf8 \ub2e4\uc74c\uc5d0\ub294 \ucd1d \ub4dd\uc810\uc73c\ub85c, \uadf8 \ub2e4\uc74c\uc5d0\ub294 \uc2b9\uc790 \uc2b9 \uacb0\uacfc\ub85c, \ub9c8\uc9c0\ub9c9\uc73c\ub85c\ub294 \ucd94\ucca8\uc774 \ub3d9\uc6d0\ub41c\ub2e4. (\uc989, \ud300\uc758 \uc21c\uc704\uac00 \ubb34\uc791\uc704\ub85c \uacb0\uc815\ub41c\ub2e4.) \uc77c\ub840\ub85c, 1990\ub144 \ub300\ud68c\uc5d0\uc11c F\uc870\ub294 \uc789\uae00\ub79c\ub4dc\uac00 \uc774\uc9d1\ud2b8\uc5d0 1-0\uc73c\ub85c \uc2b9\ub9ac\ub97c \uac70\ub454 \uac83\uc744 \uc81c\uc678\ud558\uace0 \ubaa8\ub450 \ubb34\uc2b9\ubd80\ub97c \uac70\ub450\ub294 \ubc14\ub78c\uc5d0 \ub124\ub35c\ub780\ub4dc\uc640 \uc544\uc77c\ub79c\ub4dc\uac00 3\uc804 \uc804\ubb34\ub85c \uc11c\ub85c \ub3d9\ub960\uc744 \uc774\ub8e8\uace0 \uc788\uc5c8\ub294\ub370, \ub4dd\uc810\ub9c8\uc800\ub3c4 \uc591\ud300\uc774 2\uc810\uc73c\ub85c \uac19\uace0 \uc2b9\uc790\uc2b9 \uc6d0\uce59\uc73c\ub85c\ub3c4 1-1 \ubb34\uc2b9\ubd80\uc774\uae30 \ub54c\ubb38\uc5d0 \uc6b0\uc5f4\uc744 \uac00\ub9b4 \uc218 \uc5c6\uac8c \ub418\uc790 \ucd94\ucca8\uc744 \ud1b5\ud574 \uc544\uc77c\ub79c\ub4dc\uac00 2\uc704\ub85c 16\uac15\uc5d0 \uc9c1\ud589\ud558\ub294 \ud589\uc6b4\uc744 \uac70\uba38\uc950\uc5c8\ub2e4. \uadf8\ub7ec\ub098 3\uc704\ub85c \ub5a8\uc5b4\uc9c4 \ub124\ub35c\ub780\ub4dc\ub3c4 \uac01\uc870 3\uc704\ud300 \uc911 3\uc704\ub97c \uae30\ub85d\ud558\uc5ec 16\uac15\uc5d0 \uc9c4\ucd9c\ud558\uc600\ub2e4. \ubcf8\ub798 \ub9ac\uadf8\uc804\uc740 \uac01 \uacbd\uae30\ub9c8\ub2e4 \ub0a0\uc9dc\ub97c \ub2e4\ub974\uac8c \uc815\ud574\uc11c \uc2dc\ud569\ud558\uac8c \ub418\uc5b4 \uc788\uc5c8\uc73c\ub098 1982\ub144 FIFA \uc6d4\ub4dc\ucef5 \ub2f9\uc2dc \uc54c\uc81c\ub9ac\uc5d0\uac8c \ud328\ud558\uace0 \uc54c\uc81c\ub9ac\uc5d0 \ubc00\ub824 \ud0c8\ub77d\ud560 \uc704\uae30\uc5d0 \ub193\uc778 \uc11c\ub3c5\uc774 \uc870\ubcc4\ub9ac\uadf8 \ub9c8\uc9c0\ub9c9 \uacbd\uae30 \uc911\uc5d0\uc11c \uc54c\uc81c\ub9ac\uc640 \uce60\ub808\uc758 \uacbd\uae30\uac00 \uc11c\ub3c5\uacfc \uc624\uc2a4\ud2b8\ub9ac\uc544\uc758 \uacbd\uae30\ubcf4\ub2e4 \ud558\ub8e8 \uba3c\uc800 \uc2e4\uc2dc\ub41c\ub2e4\ub294 \uc810\uc744 \uc545\uc6a9\ud558\uc5ec \uc54c\uc81c\ub9ac\uc640 \uce60\ub808\uc804\uc744 \uad00\ub78c\ud55c \uc774\ud6c4 \uadf8 \uacbd\uae30 \uacb0\uacfc\uc5d0 \ub9de\ucdb0 \uc624\uc2a4\ud2b8\ub9ac\uc544\uc640 \uc9dc\uace0 \uc2b9\ubd80\uc870\uc791\uc744 \ud574\uc11c \uadf8 \uc870\ub294 \uc11c\ub3c5\uacfc \uc624\uc2a4\ud2b8\ub9ac\uc544\uac00 2\ub77c\uc6b4\ub4dc\uc5d0 \uc9c4\ucd9c\ud588\ub2e4. \uc774\ub7ec\ud55c \uc2b9\ubd80\uc870\uc791\uc744 \ub9c9\uae30 \uc704\ud574 FIFA\uce21\uc740 \uadf8 \ub2e4\uc74c \uc6d4\ub4dc\ucef5\uc778 1986\ub144 FIFA \uc6d4\ub4dc\ucef5\ubd80\ud130\ub294 \uc870\ubcc4\ub9ac\uadf8 \ub9c8\uc9c0\ub9c9 \uacbd\uae30\uc5d0 \ud55c\ud574\uc11c\ub9cc \ub450 \uacbd\uae30\ub97c \ub3d9\uc2dc\uc5d0 \uc9c4\ud589\ud558\ub3c4\ub85d \uaddc\uc815\uc744 \ubc14\uafe8\ub2e4.", "answer": "3\uacbd\uae30", "sentence": "\uc801\uc5b4\ub3c4 3\uacbd\uae30 \ub97c \uce58\ub97c \uc218 \uc788\ub2e4.", "paragraph_sentence": "\uac01 \uc870\ub294 \ub9ac\uadf8\uc804 \ud615\uc2dd\uc73c\ub85c \uce58\ub7ec\uc9c0\uba70 \uc774\uc5d0 \ub530\ub77c \ubaa8\ub4e0 \ud300\uc774 \uc801\uc5b4\ub3c4 3\uacbd\uae30 \ub97c \uce58\ub97c \uc218 \uc788\ub2e4. \uac01 \uc870\uc758 \ub9c8\uc9c0\ub9c9 \ub77c\uc6b4\ub4dc \uacbd\uae30\ub294 \uac01 \ud300 \uac04\uc758 \uacf5\uc815\uc131 \uc720\uc9c0 \ubc0f \uc2b9\ubd80\uc870\uc791\uc744 \ubc29\uc9c0\ud558\uae30 \uc704\ud574 \uac19\uc740 \uc2dc\uac04\uc5d0 \uacbd\uae30\ub97c \uce58\ub974\ub3c4\ub85d \ud558\uace0 \uc788\ub2e4. \uac01 \uc870\uc5d0\uc11c \uc0c1\uc704 \ub450 \uac1c \ud300\uc774 \ubcf8\uc120 \ud1a0\ub108\uba3c\ud2b8\ub85c \uc9c4\ucd9c\ud55c\ub2e4. \uc2b9\uc810\uc740 \uc870 \uc548\uc5d0\uc11c\uc758 \uc21c\uc704\ub97c \uac00\ub974\uae30 \uc704\ud574 \uc0ac\uc6a9\ub41c\ub2e4. 1994\ub144\ubd80\ud130, \uacbd\uae30\uc5d0 \uc2b9\ub9ac\ud588\uc744 \ub54c\ub294 3\uc810\uc774 \uc8fc\uc5b4\uc84c\uace0 \ube44\uacbc\uc744 \ub54c\ub294 1\uc810\uc774 \uc8fc\uc5b4\uc84c\uc73c\uba70 \ud328\ud588\uc744 \ub54c\ub294 0\uc810\uc774 \uc8fc\uc5b4\uc84c\ub2e4. (\uc774 \uc774\uc804\uae4c\uc9c0 \uc2b9\ub9ac \uc2dc \uc2b9\uc810\uc740 2\uc810\uc774\uc5c8\ub2e4. ) \ub9cc\uc57d \ub450 \uac1c \ud639\uc740 \uadf8 \uc774\uc0c1\uc758 \ud300\uc774 \uc2b9\uc810\uc774 \uac19\ub2e4\uba74, \ub3d9\uc810 \uc2dc \ucc98\ub9ac \ubc29\ubc95\uc774 \uc0ac\uc6a9\ub41c\ub2e4. \ucc98\uc74c\uc740 \uace8 \ub4dd\uc2e4 \ucc28\ub85c, \uadf8 \ub2e4\uc74c\uc5d0\ub294 \ucd1d \ub4dd\uc810\uc73c\ub85c, \uadf8 \ub2e4\uc74c\uc5d0\ub294 \uc2b9\uc790 \uc2b9 \uacb0\uacfc\ub85c, \ub9c8\uc9c0\ub9c9\uc73c\ub85c\ub294 \ucd94\ucca8\uc774 \ub3d9\uc6d0\ub41c\ub2e4. (\uc989, \ud300\uc758 \uc21c\uc704\uac00 \ubb34\uc791\uc704\ub85c \uacb0\uc815\ub41c\ub2e4.) \uc77c\ub840\ub85c, 1990\ub144 \ub300\ud68c\uc5d0\uc11c F\uc870\ub294 \uc789\uae00\ub79c\ub4dc\uac00 \uc774\uc9d1\ud2b8\uc5d0 1-0\uc73c\ub85c \uc2b9\ub9ac\ub97c \uac70\ub454 \uac83\uc744 \uc81c\uc678\ud558\uace0 \ubaa8\ub450 \ubb34\uc2b9\ubd80\ub97c \uac70\ub450\ub294 \ubc14\ub78c\uc5d0 \ub124\ub35c\ub780\ub4dc\uc640 \uc544\uc77c\ub79c\ub4dc\uac00 3\uc804 \uc804\ubb34\ub85c \uc11c\ub85c \ub3d9\ub960\uc744 \uc774\ub8e8\uace0 \uc788\uc5c8\ub294\ub370, \ub4dd\uc810\ub9c8\uc800\ub3c4 \uc591\ud300\uc774 2\uc810\uc73c\ub85c \uac19\uace0 \uc2b9\uc790\uc2b9 \uc6d0\uce59\uc73c\ub85c\ub3c4 1-1 \ubb34\uc2b9\ubd80\uc774\uae30 \ub54c\ubb38\uc5d0 \uc6b0\uc5f4\uc744 \uac00\ub9b4 \uc218 \uc5c6\uac8c \ub418\uc790 \ucd94\ucca8\uc744 \ud1b5\ud574 \uc544\uc77c\ub79c\ub4dc\uac00 2\uc704\ub85c 16\uac15\uc5d0 \uc9c1\ud589\ud558\ub294 \ud589\uc6b4\uc744 \uac70\uba38\uc950\uc5c8\ub2e4. \uadf8\ub7ec\ub098 3\uc704\ub85c \ub5a8\uc5b4\uc9c4 \ub124\ub35c\ub780\ub4dc\ub3c4 \uac01\uc870 3\uc704\ud300 \uc911 3\uc704\ub97c \uae30\ub85d\ud558\uc5ec 16\uac15\uc5d0 \uc9c4\ucd9c\ud558\uc600\ub2e4. \ubcf8\ub798 \ub9ac\uadf8\uc804\uc740 \uac01 \uacbd\uae30\ub9c8\ub2e4 \ub0a0\uc9dc\ub97c \ub2e4\ub974\uac8c \uc815\ud574\uc11c \uc2dc\ud569\ud558\uac8c \ub418\uc5b4 \uc788\uc5c8\uc73c\ub098 1982\ub144 FIFA \uc6d4\ub4dc\ucef5 \ub2f9\uc2dc \uc54c\uc81c\ub9ac\uc5d0\uac8c \ud328\ud558\uace0 \uc54c\uc81c\ub9ac\uc5d0 \ubc00\ub824 \ud0c8\ub77d\ud560 \uc704\uae30\uc5d0 \ub193\uc778 \uc11c\ub3c5\uc774 \uc870\ubcc4\ub9ac\uadf8 \ub9c8\uc9c0\ub9c9 \uacbd\uae30 \uc911\uc5d0\uc11c \uc54c\uc81c\ub9ac\uc640 \uce60\ub808\uc758 \uacbd\uae30\uac00 \uc11c\ub3c5\uacfc \uc624\uc2a4\ud2b8\ub9ac\uc544\uc758 \uacbd\uae30\ubcf4\ub2e4 \ud558\ub8e8 \uba3c\uc800 \uc2e4\uc2dc\ub41c\ub2e4\ub294 \uc810\uc744 \uc545\uc6a9\ud558\uc5ec \uc54c\uc81c\ub9ac\uc640 \uce60\ub808\uc804\uc744 \uad00\ub78c\ud55c \uc774\ud6c4 \uadf8 \uacbd\uae30 \uacb0\uacfc\uc5d0 \ub9de\ucdb0 \uc624\uc2a4\ud2b8\ub9ac\uc544\uc640 \uc9dc\uace0 \uc2b9\ubd80\uc870\uc791\uc744 \ud574\uc11c \uadf8 \uc870\ub294 \uc11c\ub3c5\uacfc \uc624\uc2a4\ud2b8\ub9ac\uc544\uac00 2\ub77c\uc6b4\ub4dc\uc5d0 \uc9c4\ucd9c\ud588\ub2e4. \uc774\ub7ec\ud55c \uc2b9\ubd80\uc870\uc791\uc744 \ub9c9\uae30 \uc704\ud574 FIFA\uce21\uc740 \uadf8 \ub2e4\uc74c \uc6d4\ub4dc\ucef5\uc778 1986\ub144 FIFA \uc6d4\ub4dc\ucef5\ubd80\ud130\ub294 \uc870\ubcc4\ub9ac\uadf8 \ub9c8\uc9c0\ub9c9 \uacbd\uae30\uc5d0 \ud55c\ud574\uc11c\ub9cc \ub450 \uacbd\uae30\ub97c \ub3d9\uc2dc\uc5d0 \uc9c4\ud589\ud558\ub3c4\ub85d \uaddc\uc815\uc744 \ubc14\uafe8\ub2e4.", "paragraph_answer": "\uac01 \uc870\ub294 \ub9ac\uadf8\uc804 \ud615\uc2dd\uc73c\ub85c \uce58\ub7ec\uc9c0\uba70 \uc774\uc5d0 \ub530\ub77c \ubaa8\ub4e0 \ud300\uc774 \uc801\uc5b4\ub3c4 3\uacbd\uae30 \ub97c \uce58\ub97c \uc218 \uc788\ub2e4. \uac01 \uc870\uc758 \ub9c8\uc9c0\ub9c9 \ub77c\uc6b4\ub4dc \uacbd\uae30\ub294 \uac01 \ud300 \uac04\uc758 \uacf5\uc815\uc131 \uc720\uc9c0 \ubc0f \uc2b9\ubd80\uc870\uc791\uc744 \ubc29\uc9c0\ud558\uae30 \uc704\ud574 \uac19\uc740 \uc2dc\uac04\uc5d0 \uacbd\uae30\ub97c \uce58\ub974\ub3c4\ub85d \ud558\uace0 \uc788\ub2e4. \uac01 \uc870\uc5d0\uc11c \uc0c1\uc704 \ub450 \uac1c \ud300\uc774 \ubcf8\uc120 \ud1a0\ub108\uba3c\ud2b8\ub85c \uc9c4\ucd9c\ud55c\ub2e4. \uc2b9\uc810\uc740 \uc870 \uc548\uc5d0\uc11c\uc758 \uc21c\uc704\ub97c \uac00\ub974\uae30 \uc704\ud574 \uc0ac\uc6a9\ub41c\ub2e4. 1994\ub144\ubd80\ud130, \uacbd\uae30\uc5d0 \uc2b9\ub9ac\ud588\uc744 \ub54c\ub294 3\uc810\uc774 \uc8fc\uc5b4\uc84c\uace0 \ube44\uacbc\uc744 \ub54c\ub294 1\uc810\uc774 \uc8fc\uc5b4\uc84c\uc73c\uba70 \ud328\ud588\uc744 \ub54c\ub294 0\uc810\uc774 \uc8fc\uc5b4\uc84c\ub2e4. (\uc774 \uc774\uc804\uae4c\uc9c0 \uc2b9\ub9ac \uc2dc \uc2b9\uc810\uc740 2\uc810\uc774\uc5c8\ub2e4.) \ub9cc\uc57d \ub450 \uac1c \ud639\uc740 \uadf8 \uc774\uc0c1\uc758 \ud300\uc774 \uc2b9\uc810\uc774 \uac19\ub2e4\uba74, \ub3d9\uc810 \uc2dc \ucc98\ub9ac \ubc29\ubc95\uc774 \uc0ac\uc6a9\ub41c\ub2e4. \ucc98\uc74c\uc740 \uace8 \ub4dd\uc2e4 \ucc28\ub85c, \uadf8 \ub2e4\uc74c\uc5d0\ub294 \ucd1d \ub4dd\uc810\uc73c\ub85c, \uadf8 \ub2e4\uc74c\uc5d0\ub294 \uc2b9\uc790 \uc2b9 \uacb0\uacfc\ub85c, \ub9c8\uc9c0\ub9c9\uc73c\ub85c\ub294 \ucd94\ucca8\uc774 \ub3d9\uc6d0\ub41c\ub2e4. (\uc989, \ud300\uc758 \uc21c\uc704\uac00 \ubb34\uc791\uc704\ub85c \uacb0\uc815\ub41c\ub2e4.) \uc77c\ub840\ub85c, 1990\ub144 \ub300\ud68c\uc5d0\uc11c F\uc870\ub294 \uc789\uae00\ub79c\ub4dc\uac00 \uc774\uc9d1\ud2b8\uc5d0 1-0\uc73c\ub85c \uc2b9\ub9ac\ub97c \uac70\ub454 \uac83\uc744 \uc81c\uc678\ud558\uace0 \ubaa8\ub450 \ubb34\uc2b9\ubd80\ub97c \uac70\ub450\ub294 \ubc14\ub78c\uc5d0 \ub124\ub35c\ub780\ub4dc\uc640 \uc544\uc77c\ub79c\ub4dc\uac00 3\uc804 \uc804\ubb34\ub85c \uc11c\ub85c \ub3d9\ub960\uc744 \uc774\ub8e8\uace0 \uc788\uc5c8\ub294\ub370, \ub4dd\uc810\ub9c8\uc800\ub3c4 \uc591\ud300\uc774 2\uc810\uc73c\ub85c \uac19\uace0 \uc2b9\uc790\uc2b9 \uc6d0\uce59\uc73c\ub85c\ub3c4 1-1 \ubb34\uc2b9\ubd80\uc774\uae30 \ub54c\ubb38\uc5d0 \uc6b0\uc5f4\uc744 \uac00\ub9b4 \uc218 \uc5c6\uac8c \ub418\uc790 \ucd94\ucca8\uc744 \ud1b5\ud574 \uc544\uc77c\ub79c\ub4dc\uac00 2\uc704\ub85c 16\uac15\uc5d0 \uc9c1\ud589\ud558\ub294 \ud589\uc6b4\uc744 \uac70\uba38\uc950\uc5c8\ub2e4. \uadf8\ub7ec\ub098 3\uc704\ub85c \ub5a8\uc5b4\uc9c4 \ub124\ub35c\ub780\ub4dc\ub3c4 \uac01\uc870 3\uc704\ud300 \uc911 3\uc704\ub97c \uae30\ub85d\ud558\uc5ec 16\uac15\uc5d0 \uc9c4\ucd9c\ud558\uc600\ub2e4. \ubcf8\ub798 \ub9ac\uadf8\uc804\uc740 \uac01 \uacbd\uae30\ub9c8\ub2e4 \ub0a0\uc9dc\ub97c \ub2e4\ub974\uac8c \uc815\ud574\uc11c \uc2dc\ud569\ud558\uac8c \ub418\uc5b4 \uc788\uc5c8\uc73c\ub098 1982\ub144 FIFA \uc6d4\ub4dc\ucef5 \ub2f9\uc2dc \uc54c\uc81c\ub9ac\uc5d0\uac8c \ud328\ud558\uace0 \uc54c\uc81c\ub9ac\uc5d0 \ubc00\ub824 \ud0c8\ub77d\ud560 \uc704\uae30\uc5d0 \ub193\uc778 \uc11c\ub3c5\uc774 \uc870\ubcc4\ub9ac\uadf8 \ub9c8\uc9c0\ub9c9 \uacbd\uae30 \uc911\uc5d0\uc11c \uc54c\uc81c\ub9ac\uc640 \uce60\ub808\uc758 \uacbd\uae30\uac00 \uc11c\ub3c5\uacfc \uc624\uc2a4\ud2b8\ub9ac\uc544\uc758 \uacbd\uae30\ubcf4\ub2e4 \ud558\ub8e8 \uba3c\uc800 \uc2e4\uc2dc\ub41c\ub2e4\ub294 \uc810\uc744 \uc545\uc6a9\ud558\uc5ec \uc54c\uc81c\ub9ac\uc640 \uce60\ub808\uc804\uc744 \uad00\ub78c\ud55c \uc774\ud6c4 \uadf8 \uacbd\uae30 \uacb0\uacfc\uc5d0 \ub9de\ucdb0 \uc624\uc2a4\ud2b8\ub9ac\uc544\uc640 \uc9dc\uace0 \uc2b9\ubd80\uc870\uc791\uc744 \ud574\uc11c \uadf8 \uc870\ub294 \uc11c\ub3c5\uacfc \uc624\uc2a4\ud2b8\ub9ac\uc544\uac00 2\ub77c\uc6b4\ub4dc\uc5d0 \uc9c4\ucd9c\ud588\ub2e4. \uc774\ub7ec\ud55c \uc2b9\ubd80\uc870\uc791\uc744 \ub9c9\uae30 \uc704\ud574 FIFA\uce21\uc740 \uadf8 \ub2e4\uc74c \uc6d4\ub4dc\ucef5\uc778 1986\ub144 FIFA \uc6d4\ub4dc\ucef5\ubd80\ud130\ub294 \uc870\ubcc4\ub9ac\uadf8 \ub9c8\uc9c0\ub9c9 \uacbd\uae30\uc5d0 \ud55c\ud574\uc11c\ub9cc \ub450 \uacbd\uae30\ub97c \ub3d9\uc2dc\uc5d0 \uc9c4\ud589\ud558\ub3c4\ub85d \uaddc\uc815\uc744 \ubc14\uafe8\ub2e4.", "sentence_answer": "\uc801\uc5b4\ub3c4 3\uacbd\uae30 \ub97c \uce58\ub97c \uc218 \uc788\ub2e4.", "paragraph_id": "6475712-7-0", "paragraph_question": "question: \uc870\ubcc4 \uacbd\uae30\uc5d0\uc11c \uac01 \ud300\ub4e4\uc740 \ucd5c\uc18c\ud55c \uba87 \uacbd\uae30\ub97c \uce58\ub974\ub098?, context: \uac01 \uc870\ub294 \ub9ac\uadf8\uc804 \ud615\uc2dd\uc73c\ub85c \uce58\ub7ec\uc9c0\uba70 \uc774\uc5d0 \ub530\ub77c \ubaa8\ub4e0 \ud300\uc774 \uc801\uc5b4\ub3c4 3\uacbd\uae30\ub97c \uce58\ub97c \uc218 \uc788\ub2e4. \uac01 \uc870\uc758 \ub9c8\uc9c0\ub9c9 \ub77c\uc6b4\ub4dc \uacbd\uae30\ub294 \uac01 \ud300 \uac04\uc758 \uacf5\uc815\uc131 \uc720\uc9c0 \ubc0f \uc2b9\ubd80\uc870\uc791\uc744 \ubc29\uc9c0\ud558\uae30 \uc704\ud574 \uac19\uc740 \uc2dc\uac04\uc5d0 \uacbd\uae30\ub97c \uce58\ub974\ub3c4\ub85d \ud558\uace0 \uc788\ub2e4. \uac01 \uc870\uc5d0\uc11c \uc0c1\uc704 \ub450 \uac1c \ud300\uc774 \ubcf8\uc120 \ud1a0\ub108\uba3c\ud2b8\ub85c \uc9c4\ucd9c\ud55c\ub2e4. \uc2b9\uc810\uc740 \uc870 \uc548\uc5d0\uc11c\uc758 \uc21c\uc704\ub97c \uac00\ub974\uae30 \uc704\ud574 \uc0ac\uc6a9\ub41c\ub2e4. 1994\ub144\ubd80\ud130, \uacbd\uae30\uc5d0 \uc2b9\ub9ac\ud588\uc744 \ub54c\ub294 3\uc810\uc774 \uc8fc\uc5b4\uc84c\uace0 \ube44\uacbc\uc744 \ub54c\ub294 1\uc810\uc774 \uc8fc\uc5b4\uc84c\uc73c\uba70 \ud328\ud588\uc744 \ub54c\ub294 0\uc810\uc774 \uc8fc\uc5b4\uc84c\ub2e4. (\uc774 \uc774\uc804\uae4c\uc9c0 \uc2b9\ub9ac \uc2dc \uc2b9\uc810\uc740 2\uc810\uc774\uc5c8\ub2e4.) \ub9cc\uc57d \ub450 \uac1c \ud639\uc740 \uadf8 \uc774\uc0c1\uc758 \ud300\uc774 \uc2b9\uc810\uc774 \uac19\ub2e4\uba74, \ub3d9\uc810 \uc2dc \ucc98\ub9ac \ubc29\ubc95\uc774 \uc0ac\uc6a9\ub41c\ub2e4. \ucc98\uc74c\uc740 \uace8 \ub4dd\uc2e4 \ucc28\ub85c, \uadf8 \ub2e4\uc74c\uc5d0\ub294 \ucd1d \ub4dd\uc810\uc73c\ub85c, \uadf8 \ub2e4\uc74c\uc5d0\ub294 \uc2b9\uc790 \uc2b9 \uacb0\uacfc\ub85c, \ub9c8\uc9c0\ub9c9\uc73c\ub85c\ub294 \ucd94\ucca8\uc774 \ub3d9\uc6d0\ub41c\ub2e4. (\uc989, \ud300\uc758 \uc21c\uc704\uac00 \ubb34\uc791\uc704\ub85c \uacb0\uc815\ub41c\ub2e4.) \uc77c\ub840\ub85c, 1990\ub144 \ub300\ud68c\uc5d0\uc11c F\uc870\ub294 \uc789\uae00\ub79c\ub4dc\uac00 \uc774\uc9d1\ud2b8\uc5d0 1-0\uc73c\ub85c \uc2b9\ub9ac\ub97c \uac70\ub454 \uac83\uc744 \uc81c\uc678\ud558\uace0 \ubaa8\ub450 \ubb34\uc2b9\ubd80\ub97c \uac70\ub450\ub294 \ubc14\ub78c\uc5d0 \ub124\ub35c\ub780\ub4dc\uc640 \uc544\uc77c\ub79c\ub4dc\uac00 3\uc804 \uc804\ubb34\ub85c \uc11c\ub85c \ub3d9\ub960\uc744 \uc774\ub8e8\uace0 \uc788\uc5c8\ub294\ub370, \ub4dd\uc810\ub9c8\uc800\ub3c4 \uc591\ud300\uc774 2\uc810\uc73c\ub85c \uac19\uace0 \uc2b9\uc790\uc2b9 \uc6d0\uce59\uc73c\ub85c\ub3c4 1-1 \ubb34\uc2b9\ubd80\uc774\uae30 \ub54c\ubb38\uc5d0 \uc6b0\uc5f4\uc744 \uac00\ub9b4 \uc218 \uc5c6\uac8c \ub418\uc790 \ucd94\ucca8\uc744 \ud1b5\ud574 \uc544\uc77c\ub79c\ub4dc\uac00 2\uc704\ub85c 16\uac15\uc5d0 \uc9c1\ud589\ud558\ub294 \ud589\uc6b4\uc744 \uac70\uba38\uc950\uc5c8\ub2e4. \uadf8\ub7ec\ub098 3\uc704\ub85c \ub5a8\uc5b4\uc9c4 \ub124\ub35c\ub780\ub4dc\ub3c4 \uac01\uc870 3\uc704\ud300 \uc911 3\uc704\ub97c \uae30\ub85d\ud558\uc5ec 16\uac15\uc5d0 \uc9c4\ucd9c\ud558\uc600\ub2e4. \ubcf8\ub798 \ub9ac\uadf8\uc804\uc740 \uac01 \uacbd\uae30\ub9c8\ub2e4 \ub0a0\uc9dc\ub97c \ub2e4\ub974\uac8c \uc815\ud574\uc11c \uc2dc\ud569\ud558\uac8c \ub418\uc5b4 \uc788\uc5c8\uc73c\ub098 1982\ub144 FIFA \uc6d4\ub4dc\ucef5 \ub2f9\uc2dc \uc54c\uc81c\ub9ac\uc5d0\uac8c \ud328\ud558\uace0 \uc54c\uc81c\ub9ac\uc5d0 \ubc00\ub824 \ud0c8\ub77d\ud560 \uc704\uae30\uc5d0 \ub193\uc778 \uc11c\ub3c5\uc774 \uc870\ubcc4\ub9ac\uadf8 \ub9c8\uc9c0\ub9c9 \uacbd\uae30 \uc911\uc5d0\uc11c \uc54c\uc81c\ub9ac\uc640 \uce60\ub808\uc758 \uacbd\uae30\uac00 \uc11c\ub3c5\uacfc \uc624\uc2a4\ud2b8\ub9ac\uc544\uc758 \uacbd\uae30\ubcf4\ub2e4 \ud558\ub8e8 \uba3c\uc800 \uc2e4\uc2dc\ub41c\ub2e4\ub294 \uc810\uc744 \uc545\uc6a9\ud558\uc5ec \uc54c\uc81c\ub9ac\uc640 \uce60\ub808\uc804\uc744 \uad00\ub78c\ud55c \uc774\ud6c4 \uadf8 \uacbd\uae30 \uacb0\uacfc\uc5d0 \ub9de\ucdb0 \uc624\uc2a4\ud2b8\ub9ac\uc544\uc640 \uc9dc\uace0 \uc2b9\ubd80\uc870\uc791\uc744 \ud574\uc11c \uadf8 \uc870\ub294 \uc11c\ub3c5\uacfc \uc624\uc2a4\ud2b8\ub9ac\uc544\uac00 2\ub77c\uc6b4\ub4dc\uc5d0 \uc9c4\ucd9c\ud588\ub2e4. \uc774\ub7ec\ud55c \uc2b9\ubd80\uc870\uc791\uc744 \ub9c9\uae30 \uc704\ud574 FIFA\uce21\uc740 \uadf8 \ub2e4\uc74c \uc6d4\ub4dc\ucef5\uc778 1986\ub144 FIFA \uc6d4\ub4dc\ucef5\ubd80\ud130\ub294 \uc870\ubcc4\ub9ac\uadf8 \ub9c8\uc9c0\ub9c9 \uacbd\uae30\uc5d0 \ud55c\ud574\uc11c\ub9cc \ub450 \uacbd\uae30\ub97c \ub3d9\uc2dc\uc5d0 \uc9c4\ud589\ud558\ub3c4\ub85d \uaddc\uc815\uc744 \ubc14\uafe8\ub2e4."} {"question": "\ud0e4\ucef7 \ud30c\uc2a8\uc2a4\uac00 \uc0ac\ud68c\uc801 \uc9c8\uc11c\uc758 \uae30\ubcf8 \uc694\uac74\uc73c\ub85c \uc81c\uc2dc\ud55c \uac83 \uac00\uc6b4\ub370 \uc0ac\ud68c\uac00 \ubb38\ud654\uc640 \uad00\uc2b5\uc744 \ubcf4\uc874\ud558\uc5ec \uc790\uae30 \uc815\uccb4\uc131\uc744 \uc720\uc9c0\ud558\ub824\ub294 \uc695\uad6c\ub97c \ub098\ud0c0\ub0b4\ub294 \uac1c\ub150\uc740?", "paragraph": "\ubbf8\uad6d\uc758 \ud0e4\ucef7 \ud30c\uc2a8\uc2a4\ub294 \uc0ac\ud68c\uac00 \uc9c8\uc11c\ub97c \uc720\uc9c0\ud558\uae30 \uc704\ud558\uc5ec \uac16\uace0 \uc788\uc5b4\uc57c \ud560 \ud2b9\uc9d5\uc744 \ud615\ud0dc\uc720\uc9c0\u00b7 \ub2e8\uc704\ud1b5\ud569\u00b7 \ubaa9\ud45c\ub2ec\uc131\u00b7 \uc801\uc751\uc758 4\uac00\uc9c0\ub85c \ub098\ub234\ub2e4. \ud615\ud0dc\uc720\uc9c0\ub294 \uc0ac\ud68c\uac00 \ubb38\ud654\uc640 \uad00\uc2b5\uc744 \ubcf4\uc874\ud568\uc73c\ub85c\uc368 \uc790\uc2e0\uc758 \uc815\uccb4\uc131\uc744 \uc720\uc9c0\ud558\ub824\ub294 \uc695\uad6c\ub85c, \uc0dd\uba85\uccb4\uac00 \uc790\uc2e0\uc758 \uc678\ud615\uacfc \ub0b4\ubd80 \uae30\uad00\uc758 \uae30\ub2a5\uc744 \uc720\uc9c0\ud558\ub294 \uac83\uacfc \ube44\uc2b7\ud558\ub2e4. \uc774 \uc695\uad6c\ub97c \ucda9\uc871\ud558\uae30 \uc704\ud574 \ud559\uad50\ub098 \uc885\uad50\u00b7 \ubc95 \ub4f1\uc774 \uc874\uc7ac\ud55c\ub2e4. \ub2e8\uc704\ud1b5\ud569\uc740 \uc0ac\ud68c\uac00 \uad6c\uc131\uc6d0\uc744 \uc720\uae30\uc801\uc73c\ub85c \uacb0\ud569\ud568\uc73c\ub85c\uc368 \uc640\ud574\ub97c \ub9c9\uace0\uc790\ud558\ub294 \uc695\uad6c\ub85c, \uce5c\uc871\uad00\uacc4\ub098 \uc798 \uc9dc\uc5ec\uc9c4 \uc870\uc9c1\uc774 \uc774\ub7ec\ud55c \uc695\uad6c\ub97c \ucda9\uc871\ud55c\ub2e4. \uc0dd\ubb3c\uccb4\uac00 \ub0b4\ubd80 \uae30\uad00\uc744 \uc720\uae30\uc801\uc73c\ub85c \uc5f0\uacb0\ud558\uc5ec \ud56d\uc0c1\uc131\uc744 \uc720\uc9c0\ud558\ub294 \uac83\uacfc \ube44\uc2b7\ud55c \uc148\uc774\ub2e4. \ud55c\ud3b8, \ubaa9\ud45c\ub2ec\uc131\uc740 \uc0ac\ud68c\uac00 \uacf5\uacf5\uc758 \ubaa9\ud45c\ub97c \uc124\uc815\ud568\uc73c\ub85c\uc368 \uad6c\uc131\uc6d0\uc758 \ud611\ub825\uc744 \ub3c4\ubaa8\ud558\uace0 \uc6b0\ud638\uad00\uacc4\ub97c \uc99d\uc9c4\ud558\uace0\uc790 \ud558\ub294 \uc695\uad6c\ub97c \ub9d0\ud55c\ub2e4. \uc815\uce58\ud65c\ub3d9\uc774 \uc874\uc7ac\ud558\ub294 \uc774\uc720\uac00 \ubc14\ub85c \uc774 \ub54c\ubb38\uc774\ub2e4. \ub9c8\uc9c0\ub9c9\uc73c\ub85c \uc801\uc751\uc740 \uc0ac\ud68c\uac00 \uc678\ubd80\uc758 \ucda9\uaca9\uc5d0\uc73c\ub85c\ubd80\ud130 \uc790\uc2e0\uc744 \ubcf4\ud638\ud558\ub824\ub294 \uc695\uad6c\ub85c, \uc0dd\uba85\uccb4\uac00 \uc678\ubd80 \ubcc0\ud654\uc5d0 \uc801\uc751\ud558\uace0 \uc9c4\ud654\ud558\ub294 \uac83\uacfc \ube44\uc2b7\ud558\ub2e4. \uae30\ub2a5\uc8fc\uc758\ub294 20\uc138\uae30 \uc911\ubc18\uae4c\uc9c0 \uc0ac\ud68c\ud559\uc744 \uc9c0\ubc30\ud558\ub294 \uc0ac\uc0c1\uc774\uc5c8\uc73c\uba70 \uba38\ud134\uc774 \uacfc\ud559\uc0ac\ud68c\ud559\uc744 \uc815\ub9bd\ud558\ub294\ub370 \ud070 \uc601\ud5a5\uc744 \uc8fc\uc5c8\ub2e4.", "answer": "\ud615\ud0dc\uc720\uc9c0", "sentence": "\ubbf8\uad6d\uc758 \ud0e4\ucef7 \ud30c\uc2a8\uc2a4\ub294 \uc0ac\ud68c\uac00 \uc9c8\uc11c\ub97c \uc720\uc9c0\ud558\uae30 \uc704\ud558\uc5ec \uac16\uace0 \uc788\uc5b4\uc57c \ud560 \ud2b9\uc9d5\uc744 \ud615\ud0dc\uc720\uc9c0 \u00b7 \ub2e8\uc704\ud1b5\ud569\u00b7 \ubaa9\ud45c\ub2ec\uc131\u00b7 \uc801\uc751\uc758 4\uac00\uc9c0\ub85c \ub098\ub234\ub2e4.", "paragraph_sentence": " \ubbf8\uad6d\uc758 \ud0e4\ucef7 \ud30c\uc2a8\uc2a4\ub294 \uc0ac\ud68c\uac00 \uc9c8\uc11c\ub97c \uc720\uc9c0\ud558\uae30 \uc704\ud558\uc5ec \uac16\uace0 \uc788\uc5b4\uc57c \ud560 \ud2b9\uc9d5\uc744 \ud615\ud0dc\uc720\uc9c0 \u00b7 \ub2e8\uc704\ud1b5\ud569\u00b7 \ubaa9\ud45c\ub2ec\uc131\u00b7 \uc801\uc751\uc758 4\uac00\uc9c0\ub85c \ub098\ub234\ub2e4. \ud615\ud0dc\uc720\uc9c0\ub294 \uc0ac\ud68c\uac00 \ubb38\ud654\uc640 \uad00\uc2b5\uc744 \ubcf4\uc874\ud568\uc73c\ub85c\uc368 \uc790\uc2e0\uc758 \uc815\uccb4\uc131\uc744 \uc720\uc9c0\ud558\ub824\ub294 \uc695\uad6c\ub85c, \uc0dd\uba85\uccb4\uac00 \uc790\uc2e0\uc758 \uc678\ud615\uacfc \ub0b4\ubd80 \uae30\uad00\uc758 \uae30\ub2a5\uc744 \uc720\uc9c0\ud558\ub294 \uac83\uacfc \ube44\uc2b7\ud558\ub2e4. \uc774 \uc695\uad6c\ub97c \ucda9\uc871\ud558\uae30 \uc704\ud574 \ud559\uad50\ub098 \uc885\uad50\u00b7 \ubc95 \ub4f1\uc774 \uc874\uc7ac\ud55c\ub2e4. \ub2e8\uc704\ud1b5\ud569\uc740 \uc0ac\ud68c\uac00 \uad6c\uc131\uc6d0\uc744 \uc720\uae30\uc801\uc73c\ub85c \uacb0\ud569\ud568\uc73c\ub85c\uc368 \uc640\ud574\ub97c \ub9c9\uace0\uc790\ud558\ub294 \uc695\uad6c\ub85c, \uce5c\uc871\uad00\uacc4\ub098 \uc798 \uc9dc\uc5ec\uc9c4 \uc870\uc9c1\uc774 \uc774\ub7ec\ud55c \uc695\uad6c\ub97c \ucda9\uc871\ud55c\ub2e4. \uc0dd\ubb3c\uccb4\uac00 \ub0b4\ubd80 \uae30\uad00\uc744 \uc720\uae30\uc801\uc73c\ub85c \uc5f0\uacb0\ud558\uc5ec \ud56d\uc0c1\uc131\uc744 \uc720\uc9c0\ud558\ub294 \uac83\uacfc \ube44\uc2b7\ud55c \uc148\uc774\ub2e4. \ud55c\ud3b8, \ubaa9\ud45c\ub2ec\uc131\uc740 \uc0ac\ud68c\uac00 \uacf5\uacf5\uc758 \ubaa9\ud45c\ub97c \uc124\uc815\ud568\uc73c\ub85c\uc368 \uad6c\uc131\uc6d0\uc758 \ud611\ub825\uc744 \ub3c4\ubaa8\ud558\uace0 \uc6b0\ud638\uad00\uacc4\ub97c \uc99d\uc9c4\ud558\uace0\uc790 \ud558\ub294 \uc695\uad6c\ub97c \ub9d0\ud55c\ub2e4. \uc815\uce58\ud65c\ub3d9\uc774 \uc874\uc7ac\ud558\ub294 \uc774\uc720\uac00 \ubc14\ub85c \uc774 \ub54c\ubb38\uc774\ub2e4. \ub9c8\uc9c0\ub9c9\uc73c\ub85c \uc801\uc751\uc740 \uc0ac\ud68c\uac00 \uc678\ubd80\uc758 \ucda9\uaca9\uc5d0\uc73c\ub85c\ubd80\ud130 \uc790\uc2e0\uc744 \ubcf4\ud638\ud558\ub824\ub294 \uc695\uad6c\ub85c, \uc0dd\uba85\uccb4\uac00 \uc678\ubd80 \ubcc0\ud654\uc5d0 \uc801\uc751\ud558\uace0 \uc9c4\ud654\ud558\ub294 \uac83\uacfc \ube44\uc2b7\ud558\ub2e4. \uae30\ub2a5\uc8fc\uc758\ub294 20\uc138\uae30 \uc911\ubc18\uae4c\uc9c0 \uc0ac\ud68c\ud559\uc744 \uc9c0\ubc30\ud558\ub294 \uc0ac\uc0c1\uc774\uc5c8\uc73c\uba70 \uba38\ud134\uc774 \uacfc\ud559\uc0ac\ud68c\ud559\uc744 \uc815\ub9bd\ud558\ub294\ub370 \ud070 \uc601\ud5a5\uc744 \uc8fc\uc5c8\ub2e4.", "paragraph_answer": "\ubbf8\uad6d\uc758 \ud0e4\ucef7 \ud30c\uc2a8\uc2a4\ub294 \uc0ac\ud68c\uac00 \uc9c8\uc11c\ub97c \uc720\uc9c0\ud558\uae30 \uc704\ud558\uc5ec \uac16\uace0 \uc788\uc5b4\uc57c \ud560 \ud2b9\uc9d5\uc744 \ud615\ud0dc\uc720\uc9c0 \u00b7 \ub2e8\uc704\ud1b5\ud569\u00b7 \ubaa9\ud45c\ub2ec\uc131\u00b7 \uc801\uc751\uc758 4\uac00\uc9c0\ub85c \ub098\ub234\ub2e4. \ud615\ud0dc\uc720\uc9c0\ub294 \uc0ac\ud68c\uac00 \ubb38\ud654\uc640 \uad00\uc2b5\uc744 \ubcf4\uc874\ud568\uc73c\ub85c\uc368 \uc790\uc2e0\uc758 \uc815\uccb4\uc131\uc744 \uc720\uc9c0\ud558\ub824\ub294 \uc695\uad6c\ub85c, \uc0dd\uba85\uccb4\uac00 \uc790\uc2e0\uc758 \uc678\ud615\uacfc \ub0b4\ubd80 \uae30\uad00\uc758 \uae30\ub2a5\uc744 \uc720\uc9c0\ud558\ub294 \uac83\uacfc \ube44\uc2b7\ud558\ub2e4. \uc774 \uc695\uad6c\ub97c \ucda9\uc871\ud558\uae30 \uc704\ud574 \ud559\uad50\ub098 \uc885\uad50\u00b7 \ubc95 \ub4f1\uc774 \uc874\uc7ac\ud55c\ub2e4. \ub2e8\uc704\ud1b5\ud569\uc740 \uc0ac\ud68c\uac00 \uad6c\uc131\uc6d0\uc744 \uc720\uae30\uc801\uc73c\ub85c \uacb0\ud569\ud568\uc73c\ub85c\uc368 \uc640\ud574\ub97c \ub9c9\uace0\uc790\ud558\ub294 \uc695\uad6c\ub85c, \uce5c\uc871\uad00\uacc4\ub098 \uc798 \uc9dc\uc5ec\uc9c4 \uc870\uc9c1\uc774 \uc774\ub7ec\ud55c \uc695\uad6c\ub97c \ucda9\uc871\ud55c\ub2e4. \uc0dd\ubb3c\uccb4\uac00 \ub0b4\ubd80 \uae30\uad00\uc744 \uc720\uae30\uc801\uc73c\ub85c \uc5f0\uacb0\ud558\uc5ec \ud56d\uc0c1\uc131\uc744 \uc720\uc9c0\ud558\ub294 \uac83\uacfc \ube44\uc2b7\ud55c \uc148\uc774\ub2e4. \ud55c\ud3b8, \ubaa9\ud45c\ub2ec\uc131\uc740 \uc0ac\ud68c\uac00 \uacf5\uacf5\uc758 \ubaa9\ud45c\ub97c \uc124\uc815\ud568\uc73c\ub85c\uc368 \uad6c\uc131\uc6d0\uc758 \ud611\ub825\uc744 \ub3c4\ubaa8\ud558\uace0 \uc6b0\ud638\uad00\uacc4\ub97c \uc99d\uc9c4\ud558\uace0\uc790 \ud558\ub294 \uc695\uad6c\ub97c \ub9d0\ud55c\ub2e4. \uc815\uce58\ud65c\ub3d9\uc774 \uc874\uc7ac\ud558\ub294 \uc774\uc720\uac00 \ubc14\ub85c \uc774 \ub54c\ubb38\uc774\ub2e4. \ub9c8\uc9c0\ub9c9\uc73c\ub85c \uc801\uc751\uc740 \uc0ac\ud68c\uac00 \uc678\ubd80\uc758 \ucda9\uaca9\uc5d0\uc73c\ub85c\ubd80\ud130 \uc790\uc2e0\uc744 \ubcf4\ud638\ud558\ub824\ub294 \uc695\uad6c\ub85c, \uc0dd\uba85\uccb4\uac00 \uc678\ubd80 \ubcc0\ud654\uc5d0 \uc801\uc751\ud558\uace0 \uc9c4\ud654\ud558\ub294 \uac83\uacfc \ube44\uc2b7\ud558\ub2e4. \uae30\ub2a5\uc8fc\uc758\ub294 20\uc138\uae30 \uc911\ubc18\uae4c\uc9c0 \uc0ac\ud68c\ud559\uc744 \uc9c0\ubc30\ud558\ub294 \uc0ac\uc0c1\uc774\uc5c8\uc73c\uba70 \uba38\ud134\uc774 \uacfc\ud559\uc0ac\ud68c\ud559\uc744 \uc815\ub9bd\ud558\ub294\ub370 \ud070 \uc601\ud5a5\uc744 \uc8fc\uc5c8\ub2e4.", "sentence_answer": "\ubbf8\uad6d\uc758 \ud0e4\ucef7 \ud30c\uc2a8\uc2a4\ub294 \uc0ac\ud68c\uac00 \uc9c8\uc11c\ub97c \uc720\uc9c0\ud558\uae30 \uc704\ud558\uc5ec \uac16\uace0 \uc788\uc5b4\uc57c \ud560 \ud2b9\uc9d5\uc744 \ud615\ud0dc\uc720\uc9c0 \u00b7 \ub2e8\uc704\ud1b5\ud569\u00b7 \ubaa9\ud45c\ub2ec\uc131\u00b7 \uc801\uc751\uc758 4\uac00\uc9c0\ub85c \ub098\ub234\ub2e4.", "paragraph_id": "6489931-2-0", "paragraph_question": "question: \ud0e4\ucef7 \ud30c\uc2a8\uc2a4\uac00 \uc0ac\ud68c\uc801 \uc9c8\uc11c\uc758 \uae30\ubcf8 \uc694\uac74\uc73c\ub85c \uc81c\uc2dc\ud55c \uac83 \uac00\uc6b4\ub370 \uc0ac\ud68c\uac00 \ubb38\ud654\uc640 \uad00\uc2b5\uc744 \ubcf4\uc874\ud558\uc5ec \uc790\uae30 \uc815\uccb4\uc131\uc744 \uc720\uc9c0\ud558\ub824\ub294 \uc695\uad6c\ub97c \ub098\ud0c0\ub0b4\ub294 \uac1c\ub150\uc740?, context: \ubbf8\uad6d\uc758 \ud0e4\ucef7 \ud30c\uc2a8\uc2a4\ub294 \uc0ac\ud68c\uac00 \uc9c8\uc11c\ub97c \uc720\uc9c0\ud558\uae30 \uc704\ud558\uc5ec \uac16\uace0 \uc788\uc5b4\uc57c \ud560 \ud2b9\uc9d5\uc744 \ud615\ud0dc\uc720\uc9c0\u00b7 \ub2e8\uc704\ud1b5\ud569\u00b7 \ubaa9\ud45c\ub2ec\uc131\u00b7 \uc801\uc751\uc758 4\uac00\uc9c0\ub85c \ub098\ub234\ub2e4. \ud615\ud0dc\uc720\uc9c0\ub294 \uc0ac\ud68c\uac00 \ubb38\ud654\uc640 \uad00\uc2b5\uc744 \ubcf4\uc874\ud568\uc73c\ub85c\uc368 \uc790\uc2e0\uc758 \uc815\uccb4\uc131\uc744 \uc720\uc9c0\ud558\ub824\ub294 \uc695\uad6c\ub85c, \uc0dd\uba85\uccb4\uac00 \uc790\uc2e0\uc758 \uc678\ud615\uacfc \ub0b4\ubd80 \uae30\uad00\uc758 \uae30\ub2a5\uc744 \uc720\uc9c0\ud558\ub294 \uac83\uacfc \ube44\uc2b7\ud558\ub2e4. \uc774 \uc695\uad6c\ub97c \ucda9\uc871\ud558\uae30 \uc704\ud574 \ud559\uad50\ub098 \uc885\uad50\u00b7 \ubc95 \ub4f1\uc774 \uc874\uc7ac\ud55c\ub2e4. \ub2e8\uc704\ud1b5\ud569\uc740 \uc0ac\ud68c\uac00 \uad6c\uc131\uc6d0\uc744 \uc720\uae30\uc801\uc73c\ub85c \uacb0\ud569\ud568\uc73c\ub85c\uc368 \uc640\ud574\ub97c \ub9c9\uace0\uc790\ud558\ub294 \uc695\uad6c\ub85c, \uce5c\uc871\uad00\uacc4\ub098 \uc798 \uc9dc\uc5ec\uc9c4 \uc870\uc9c1\uc774 \uc774\ub7ec\ud55c \uc695\uad6c\ub97c \ucda9\uc871\ud55c\ub2e4. \uc0dd\ubb3c\uccb4\uac00 \ub0b4\ubd80 \uae30\uad00\uc744 \uc720\uae30\uc801\uc73c\ub85c \uc5f0\uacb0\ud558\uc5ec \ud56d\uc0c1\uc131\uc744 \uc720\uc9c0\ud558\ub294 \uac83\uacfc \ube44\uc2b7\ud55c \uc148\uc774\ub2e4. \ud55c\ud3b8, \ubaa9\ud45c\ub2ec\uc131\uc740 \uc0ac\ud68c\uac00 \uacf5\uacf5\uc758 \ubaa9\ud45c\ub97c \uc124\uc815\ud568\uc73c\ub85c\uc368 \uad6c\uc131\uc6d0\uc758 \ud611\ub825\uc744 \ub3c4\ubaa8\ud558\uace0 \uc6b0\ud638\uad00\uacc4\ub97c \uc99d\uc9c4\ud558\uace0\uc790 \ud558\ub294 \uc695\uad6c\ub97c \ub9d0\ud55c\ub2e4. \uc815\uce58\ud65c\ub3d9\uc774 \uc874\uc7ac\ud558\ub294 \uc774\uc720\uac00 \ubc14\ub85c \uc774 \ub54c\ubb38\uc774\ub2e4. \ub9c8\uc9c0\ub9c9\uc73c\ub85c \uc801\uc751\uc740 \uc0ac\ud68c\uac00 \uc678\ubd80\uc758 \ucda9\uaca9\uc5d0\uc73c\ub85c\ubd80\ud130 \uc790\uc2e0\uc744 \ubcf4\ud638\ud558\ub824\ub294 \uc695\uad6c\ub85c, \uc0dd\uba85\uccb4\uac00 \uc678\ubd80 \ubcc0\ud654\uc5d0 \uc801\uc751\ud558\uace0 \uc9c4\ud654\ud558\ub294 \uac83\uacfc \ube44\uc2b7\ud558\ub2e4. \uae30\ub2a5\uc8fc\uc758\ub294 20\uc138\uae30 \uc911\ubc18\uae4c\uc9c0 \uc0ac\ud68c\ud559\uc744 \uc9c0\ubc30\ud558\ub294 \uc0ac\uc0c1\uc774\uc5c8\uc73c\uba70 \uba38\ud134\uc774 \uacfc\ud559\uc0ac\ud68c\ud559\uc744 \uc815\ub9bd\ud558\ub294\ub370 \ud070 \uc601\ud5a5\uc744 \uc8fc\uc5c8\ub2e4."} {"question": "\ub300\ud55c\ubbfc\uad6d \uc8fc\uba39\uc774 \uc6b4\ub2e4\uc640 \ud0dc\uadf8 \ub9e4\uce58\ub97c \ubc8c\uc778 \uac83\uc740 \ub204\uad6c\uc778\uac00?", "paragraph": "\ub300\ud55c\ubbfc\uad6d \uc8fc\uba39\uc774 \uc6b4\ub2e4\uc640 \uc77c\ubcf8 \ub514 \uc544\uc6c3\uc0ac\uc774\ub354\uc758 \ub300\uacb0\uc740 \ubb34\uc81c\ud55c\uae09 \ubd80\ubb38\uacfc \ubc34\ud140\uae09 \ubd80\ubb38\uc73c\ub85c \ub098\ub258\uc5b4\uc838 \ud0dc\uadf8 \ub9e4\uce58\ub85c \ubc8c\uc5ec\uc84c\ub2e4. \ubb34\uc81c\ud55c\uae09 \ud0dc\uadf8 \ub9e4\uce58\uc5d0\uc11c\ub294 \ubc15\ud604\uc6b0\uac00 \uc77c\ubcf8 \ub514 \uc544\uc6c3\uc0ac\uc774\ub354\uc758 \uc81c\uc774\uc2a8 \ucf54\uc9c0, \ub2e4\ub098\ubca0 \ub2e4\ucf00\uc774\ud1a0\ub97c \uc0c1\ub300\ub85c \ubaa8\ub450 1\ub77c\uc6b4\ub4dc\uc5d0 \uac01\uac01 \uc11c\ube0c\ubbf8\uc158(\ud0a4 \ub85d), TKO\ub85c \uc2b9\ub9ac, \uae40\uc7ac\ud6c8\uacfc\uc758 \ud0dc\uadf8 \uc5c6\uc774 \ub300\ud55c\ubbfc\uad6d \uc8fc\uba39\uc774 \uc6b4\ub2e4\uc5d0 \uc2b9\ub9ac\ub97c \uc548\uaca8\ub2e4 \uc8fc\uc5c8\ub2e4. \uc774\uc5b4 \ud3bc\uccd0\uc9c4 \ubc34\ud140\uae09 \ud0dc\uadf8 \ub9e4\uce58\uc5d0\uc11c\ub294 \ubc15\ud615\uadfc\uc774 \uba3c\uc800 \ub098\uc640 \uc774\ud1a0 \uac00\uc988\ud0a4\ub97c \uc0c1\ub300\ub85c 1\ub77c\uc6b4\ub4dc \uc11c\ube0c\ubbf8\uc158(\ub9ac\uc5b4\ub124\uc774\ud0a4\ub4dc \ucd08\ud06c\ub85c) \uc2b9\uc744 \uac70\ub450\uc5c8\uace0, \ubc15\ud615\uadfc\uacfc \ud0dc\uadf8\ud55c \uae40\ud615\uc218 \uc5ed\uc2dc \ub2e4\ucf00\uc6b0\uce58 \ub2e4\ub85c\ub97c \uc0c1\ub300\ub85c 1\ub77c\uc6b4\ub4dc TKO \uc2b9\uc744 \uac70\ub450\uba70 \ub300\ud55c\ubbfc\uad6d \uc8fc\uba39\uc774 \uc6b4\ub2e4\uc5d0 \uc2b9\ub9ac\ub97c \uc548\uaca8\ub2e4 \uc8fc\uc5c8\ub2e4. \uacb0\uad6d \ub300\ud55c\ubbfc\uad6d \uc8fc\uba39\uc774 \uc6b4\ub2e4\uc640 \uc77c\ubcf8 \ub514 \uc544\uc6c3\uc0ac\uc774\ub354\uc758 \ub300\uacb0\uc740 \ub300\ud55c\ubbfc\uad6d \uc8fc\uba39\uc774 \uc6b4\ub2e4\uc758 2-0 \uc2b9\ub9ac\ub85c \ub05d\uc774 \ub0ac\ub2e4.", "answer": "\ub514 \uc544\uc6c3\uc0ac\uc774\ub354", "sentence": "\ub300\ud55c\ubbfc\uad6d \uc8fc\uba39\uc774 \uc6b4\ub2e4\uc640 \uc77c\ubcf8 \ub514 \uc544\uc6c3\uc0ac\uc774\ub354 \uc758 \ub300\uacb0\uc740 \ubb34\uc81c\ud55c\uae09 \ubd80\ubb38\uacfc \ubc34\ud140\uae09 \ubd80\ubb38\uc73c\ub85c \ub098\ub258\uc5b4\uc838 \ud0dc\uadf8 \ub9e4\uce58\ub85c \ubc8c\uc5ec\uc84c\ub2e4.", "paragraph_sentence": " \ub300\ud55c\ubbfc\uad6d \uc8fc\uba39\uc774 \uc6b4\ub2e4\uc640 \uc77c\ubcf8 \ub514 \uc544\uc6c3\uc0ac\uc774\ub354 \uc758 \ub300\uacb0\uc740 \ubb34\uc81c\ud55c\uae09 \ubd80\ubb38\uacfc \ubc34\ud140\uae09 \ubd80\ubb38\uc73c\ub85c \ub098\ub258\uc5b4\uc838 \ud0dc\uadf8 \ub9e4\uce58\ub85c \ubc8c\uc5ec\uc84c\ub2e4. \ubb34\uc81c\ud55c\uae09 \ud0dc\uadf8 \ub9e4\uce58\uc5d0\uc11c\ub294 \ubc15\ud604\uc6b0\uac00 \uc77c\ubcf8 \ub514 \uc544\uc6c3\uc0ac\uc774\ub354\uc758 \uc81c\uc774\uc2a8 \ucf54\uc9c0, \ub2e4\ub098\ubca0 \ub2e4\ucf00\uc774\ud1a0\ub97c \uc0c1\ub300\ub85c \ubaa8\ub450 1\ub77c\uc6b4\ub4dc\uc5d0 \uac01\uac01 \uc11c\ube0c\ubbf8\uc158(\ud0a4 \ub85d), TKO\ub85c \uc2b9\ub9ac, \uae40\uc7ac\ud6c8\uacfc\uc758 \ud0dc\uadf8 \uc5c6\uc774 \ub300\ud55c\ubbfc\uad6d \uc8fc\uba39\uc774 \uc6b4\ub2e4\uc5d0 \uc2b9\ub9ac\ub97c \uc548\uaca8\ub2e4 \uc8fc\uc5c8\ub2e4. \uc774\uc5b4 \ud3bc\uccd0\uc9c4 \ubc34\ud140\uae09 \ud0dc\uadf8 \ub9e4\uce58\uc5d0\uc11c\ub294 \ubc15\ud615\uadfc\uc774 \uba3c\uc800 \ub098\uc640 \uc774\ud1a0 \uac00\uc988\ud0a4\ub97c \uc0c1\ub300\ub85c 1\ub77c\uc6b4\ub4dc \uc11c\ube0c\ubbf8\uc158(\ub9ac\uc5b4\ub124\uc774\ud0a4\ub4dc \ucd08\ud06c\ub85c) \uc2b9\uc744 \uac70\ub450\uc5c8\uace0, \ubc15\ud615\uadfc\uacfc \ud0dc\uadf8\ud55c \uae40\ud615\uc218 \uc5ed\uc2dc \ub2e4\ucf00\uc6b0\uce58 \ub2e4\ub85c\ub97c \uc0c1\ub300\ub85c 1\ub77c\uc6b4\ub4dc TKO \uc2b9\uc744 \uac70\ub450\uba70 \ub300\ud55c\ubbfc\uad6d \uc8fc\uba39\uc774 \uc6b4\ub2e4\uc5d0 \uc2b9\ub9ac\ub97c \uc548\uaca8\ub2e4 \uc8fc\uc5c8\ub2e4. \uacb0\uad6d \ub300\ud55c\ubbfc\uad6d \uc8fc\uba39\uc774 \uc6b4\ub2e4\uc640 \uc77c\ubcf8 \ub514 \uc544\uc6c3\uc0ac\uc774\ub354\uc758 \ub300\uacb0\uc740 \ub300\ud55c\ubbfc\uad6d \uc8fc\uba39\uc774 \uc6b4\ub2e4\uc758 2-0 \uc2b9\ub9ac\ub85c \ub05d\uc774 \ub0ac\ub2e4.", "paragraph_answer": "\ub300\ud55c\ubbfc\uad6d \uc8fc\uba39\uc774 \uc6b4\ub2e4\uc640 \uc77c\ubcf8 \ub514 \uc544\uc6c3\uc0ac\uc774\ub354 \uc758 \ub300\uacb0\uc740 \ubb34\uc81c\ud55c\uae09 \ubd80\ubb38\uacfc \ubc34\ud140\uae09 \ubd80\ubb38\uc73c\ub85c \ub098\ub258\uc5b4\uc838 \ud0dc\uadf8 \ub9e4\uce58\ub85c \ubc8c\uc5ec\uc84c\ub2e4. \ubb34\uc81c\ud55c\uae09 \ud0dc\uadf8 \ub9e4\uce58\uc5d0\uc11c\ub294 \ubc15\ud604\uc6b0\uac00 \uc77c\ubcf8 \ub514 \uc544\uc6c3\uc0ac\uc774\ub354\uc758 \uc81c\uc774\uc2a8 \ucf54\uc9c0, \ub2e4\ub098\ubca0 \ub2e4\ucf00\uc774\ud1a0\ub97c \uc0c1\ub300\ub85c \ubaa8\ub450 1\ub77c\uc6b4\ub4dc\uc5d0 \uac01\uac01 \uc11c\ube0c\ubbf8\uc158(\ud0a4 \ub85d), TKO\ub85c \uc2b9\ub9ac, \uae40\uc7ac\ud6c8\uacfc\uc758 \ud0dc\uadf8 \uc5c6\uc774 \ub300\ud55c\ubbfc\uad6d \uc8fc\uba39\uc774 \uc6b4\ub2e4\uc5d0 \uc2b9\ub9ac\ub97c \uc548\uaca8\ub2e4 \uc8fc\uc5c8\ub2e4. \uc774\uc5b4 \ud3bc\uccd0\uc9c4 \ubc34\ud140\uae09 \ud0dc\uadf8 \ub9e4\uce58\uc5d0\uc11c\ub294 \ubc15\ud615\uadfc\uc774 \uba3c\uc800 \ub098\uc640 \uc774\ud1a0 \uac00\uc988\ud0a4\ub97c \uc0c1\ub300\ub85c 1\ub77c\uc6b4\ub4dc \uc11c\ube0c\ubbf8\uc158(\ub9ac\uc5b4\ub124\uc774\ud0a4\ub4dc \ucd08\ud06c\ub85c) \uc2b9\uc744 \uac70\ub450\uc5c8\uace0, \ubc15\ud615\uadfc\uacfc \ud0dc\uadf8\ud55c \uae40\ud615\uc218 \uc5ed\uc2dc \ub2e4\ucf00\uc6b0\uce58 \ub2e4\ub85c\ub97c \uc0c1\ub300\ub85c 1\ub77c\uc6b4\ub4dc TKO \uc2b9\uc744 \uac70\ub450\uba70 \ub300\ud55c\ubbfc\uad6d \uc8fc\uba39\uc774 \uc6b4\ub2e4\uc5d0 \uc2b9\ub9ac\ub97c \uc548\uaca8\ub2e4 \uc8fc\uc5c8\ub2e4. \uacb0\uad6d \ub300\ud55c\ubbfc\uad6d \uc8fc\uba39\uc774 \uc6b4\ub2e4\uc640 \uc77c\ubcf8 \ub514 \uc544\uc6c3\uc0ac\uc774\ub354\uc758 \ub300\uacb0\uc740 \ub300\ud55c\ubbfc\uad6d \uc8fc\uba39\uc774 \uc6b4\ub2e4\uc758 2-0 \uc2b9\ub9ac\ub85c \ub05d\uc774 \ub0ac\ub2e4.", "sentence_answer": "\ub300\ud55c\ubbfc\uad6d \uc8fc\uba39\uc774 \uc6b4\ub2e4\uc640 \uc77c\ubcf8 \ub514 \uc544\uc6c3\uc0ac\uc774\ub354 \uc758 \ub300\uacb0\uc740 \ubb34\uc81c\ud55c\uae09 \ubd80\ubb38\uacfc \ubc34\ud140\uae09 \ubd80\ubb38\uc73c\ub85c \ub098\ub258\uc5b4\uc838 \ud0dc\uadf8 \ub9e4\uce58\ub85c \ubc8c\uc5ec\uc84c\ub2e4.", "paragraph_id": "6595984-5-0", "paragraph_question": "question: \ub300\ud55c\ubbfc\uad6d \uc8fc\uba39\uc774 \uc6b4\ub2e4\uc640 \ud0dc\uadf8 \ub9e4\uce58\ub97c \ubc8c\uc778 \uac83\uc740 \ub204\uad6c\uc778\uac00?, context: \ub300\ud55c\ubbfc\uad6d \uc8fc\uba39\uc774 \uc6b4\ub2e4\uc640 \uc77c\ubcf8 \ub514 \uc544\uc6c3\uc0ac\uc774\ub354\uc758 \ub300\uacb0\uc740 \ubb34\uc81c\ud55c\uae09 \ubd80\ubb38\uacfc \ubc34\ud140\uae09 \ubd80\ubb38\uc73c\ub85c \ub098\ub258\uc5b4\uc838 \ud0dc\uadf8 \ub9e4\uce58\ub85c \ubc8c\uc5ec\uc84c\ub2e4. \ubb34\uc81c\ud55c\uae09 \ud0dc\uadf8 \ub9e4\uce58\uc5d0\uc11c\ub294 \ubc15\ud604\uc6b0\uac00 \uc77c\ubcf8 \ub514 \uc544\uc6c3\uc0ac\uc774\ub354\uc758 \uc81c\uc774\uc2a8 \ucf54\uc9c0, \ub2e4\ub098\ubca0 \ub2e4\ucf00\uc774\ud1a0\ub97c \uc0c1\ub300\ub85c \ubaa8\ub450 1\ub77c\uc6b4\ub4dc\uc5d0 \uac01\uac01 \uc11c\ube0c\ubbf8\uc158(\ud0a4 \ub85d), TKO\ub85c \uc2b9\ub9ac, \uae40\uc7ac\ud6c8\uacfc\uc758 \ud0dc\uadf8 \uc5c6\uc774 \ub300\ud55c\ubbfc\uad6d \uc8fc\uba39\uc774 \uc6b4\ub2e4\uc5d0 \uc2b9\ub9ac\ub97c \uc548\uaca8\ub2e4 \uc8fc\uc5c8\ub2e4. \uc774\uc5b4 \ud3bc\uccd0\uc9c4 \ubc34\ud140\uae09 \ud0dc\uadf8 \ub9e4\uce58\uc5d0\uc11c\ub294 \ubc15\ud615\uadfc\uc774 \uba3c\uc800 \ub098\uc640 \uc774\ud1a0 \uac00\uc988\ud0a4\ub97c \uc0c1\ub300\ub85c 1\ub77c\uc6b4\ub4dc \uc11c\ube0c\ubbf8\uc158(\ub9ac\uc5b4\ub124\uc774\ud0a4\ub4dc \ucd08\ud06c\ub85c) \uc2b9\uc744 \uac70\ub450\uc5c8\uace0, \ubc15\ud615\uadfc\uacfc \ud0dc\uadf8\ud55c \uae40\ud615\uc218 \uc5ed\uc2dc \ub2e4\ucf00\uc6b0\uce58 \ub2e4\ub85c\ub97c \uc0c1\ub300\ub85c 1\ub77c\uc6b4\ub4dc TKO \uc2b9\uc744 \uac70\ub450\uba70 \ub300\ud55c\ubbfc\uad6d \uc8fc\uba39\uc774 \uc6b4\ub2e4\uc5d0 \uc2b9\ub9ac\ub97c \uc548\uaca8\ub2e4 \uc8fc\uc5c8\ub2e4. \uacb0\uad6d \ub300\ud55c\ubbfc\uad6d \uc8fc\uba39\uc774 \uc6b4\ub2e4\uc640 \uc77c\ubcf8 \ub514 \uc544\uc6c3\uc0ac\uc774\ub354\uc758 \ub300\uacb0\uc740 \ub300\ud55c\ubbfc\uad6d \uc8fc\uba39\uc774 \uc6b4\ub2e4\uc758 2-0 \uc2b9\ub9ac\ub85c \ub05d\uc774 \ub0ac\ub2e4."} {"question": "\ubb38\uad50\ubd80\uac00 \ubd80\uc0b0\uc218\uc0b0\uc804\ubb38\ud559\uad50\ub97c \ub300\ud559\uc73c\ub85c \uc2b9\uaca9\uc2dc\ud0a8 \ud574\ub294?", "paragraph": "1946\ub144 5\uc6d4 15\uc77c\uc5d0 \ubb38\uad50\ubd80\ub294 \ubd80\uc0b0\uc218\uc0b0\uc804\ubb38\ud559\uad50\ub97c \ub300\ud559\uc73c\ub85c \uc2b9\uaca9\uc2dc\ucf1c \uc885\ud569\ub300\ud559\uc758 1\uac1c \ub2e8\uacfc\ub300\ud559(\uc218\uc0b0\ud559\ubd80)\uc73c\ub85c, \uadf8\uc640 \ubcc4\ub3c4\ub85c \uc778\ubb38\uacc4\ub300\ud559\uc744 \uc2e0\uc124\ud558\uc5ec \uc778\ubb38\uacc4 1\uac1c \ub2e8\uacfc\ub300\ud559(\uc778\ubb38\ud559\ubd80)\uacfc \uc790\uc5f0\uacc4 1\uac1c \ub2e8\uacfc\ub300\ud559(\uc218\uc0b0\ud559\ubd80) \uccb4\uc81c\uc758 \uad6d\ub9bd \ubd80\uc0b0\ub300\ud559\uad50(\uc885\ud569\ub300\ud559) \uc124\ub9bd\uc744 \uc778\uac00\ud558\uc600\ub294\ub370, \ud559\uad50\uc758 \uba85\uce6d\uc740 \u2018\ubd80\uc0b0\ub300\ud559\u2019\uc73c\ub85c, \ud559\ubd80(\ub2e8\uacfc\ub300\ud559)\ub294 \uc778\ubb38\ud559\ubd80\uc640 \uc218\uc0b0\ud559\ubd80\ub85c \uac1c\uad50\ud558\uc600\uc73c\uba70(\uc774\ub54c\uc758 \ud559\ubd80\ub294 \uc624\ub298\ub0a0\uc758 \ub2e8\uacfc\ub300\ud559(Faculty)\uc5d0 \ud574\ub2f9\ud558\ub294 \uac83\uc73c\ub85c, 1946\ub144 3\uc6d4 7\uc77c\uc5d0 \uacf5\ud3ec\ub41c \ub300\ud559\ub839 \uc81c2\uc870 \ud6c4\ub2e8\uc5d0\ub294 \uc778\ubb38\uacc4 \ubc0f \uc790\uc5f0\uacc4\uc758 \ud559\ubd80\uac00 \ubcd1\uc124\ub420 \ub54c\ub294 2\uac1c \uc774\uc0c1\uc758 \ud559\ubd80\ub85c\uc368 '\uc885\ud569\ub300\ud559\uad50'\ub97c \uad6c\uc131\ud560 \uc218 \uc788\ub3c4\ub85d \uaddc\uc815\ub418\uc5b4 \uc788\uc5c8\ub2e4.) \uadf8\ud6c4 1946\ub144 7\uc6d4 \uc608\uacfc \uc2dc\ud5d8\uc744 \uac70\uce58\uace0, 1946\ub144 8\uc6d4\uc5d0\ub294 \ub300\ud559 \ucd1d\uc7a5\uc73c\ub85c \uc544\uc11c \ub9b0 \ubca0\ucee4(Arthur Lynn Becker 1879~1979, \ud55c\uad6d\uba85 \ubc31\uc544\ub355\u767d\u96c5\u60b3) \ubc15\uc0ac\ub97c \uc784\uba85\ud558\uc5ec 1946\ub144 9\uc6d4 \uc785\ud559\uc2dd\uc744 \uac70\ud589\ud558\uace0 1\ub144 \uc815\ub3c4 \uc885\ud569\ub300\ud559\uc73c\ub85c \uc6b4\uc601\ub418\ub2e4\uac00 1947\ub144 7\uc6d4 \uc218\uc0b0\uacfc\ub300\ud559\uc774 '\uad6d\ub9bd \ubd80\uc0b0\uc218\uc0b0\ub300\ud559'\uc73c\ub85c \ubd84\ub9ac\ub428\uc73c\ub85c\uc368 \uc885\ud569\ub300\ud559\uc73c\ub85c\uc368 '\uad6d\ub9bd \ubd80\uc0b0\ub300\ud559\uad50'\uc740 \uc0ac\uc2e4\uc0c1 \uc640\ud574\ub418\uc5c8\ub2e4. \uadf8 \ud6c4 1953\ub144 4\uc6d4 1\uc77c\uc5d0 \uad6d\ub9bd \ubd80\uc0b0\ub300\ud559\uad50 \uc124\ub9bd\uc548\uc774 \uad6d\ubb34\ud68c\uc758\ub97c \ud1b5\uacfc\ud558\uace0, 4\uc6d4 3\uc77c \uad6d\ubb34\ucd1d\ub9ac\uc758 \uc7ac\uac00\ub97c \ubc1b\uc544 \uac19\uc740 \ud574 9\uc6d415\uc77c \uad6d\ub9bd\ud559\uad50\uc124\uce58\ub839\uc774 \uacf5\ud3ec\ub428\uc5d0 \ub530\ub77c \ub2e4\uc2dc \uc885\ud569\ub300\ud559\uc73c\ub85c \uc2b9\uaca9\ub418\uc5c8\ub2e4.", "answer": "1946\ub144", "sentence": "1946\ub144 5\uc6d4 15\uc77c\uc5d0 \ubb38\uad50\ubd80\ub294 \ubd80\uc0b0\uc218\uc0b0\uc804\ubb38\ud559\uad50\ub97c \ub300\ud559\uc73c\ub85c \uc2b9\uaca9\uc2dc\ucf1c \uc885\ud569\ub300\ud559\uc758 1\uac1c \ub2e8\uacfc\ub300\ud559(\uc218\uc0b0\ud559\ubd80)\uc73c\ub85c, \uadf8\uc640 \ubcc4\ub3c4\ub85c \uc778\ubb38\uacc4\ub300\ud559\uc744 \uc2e0\uc124\ud558\uc5ec \uc778\ubb38\uacc4 1\uac1c \ub2e8\uacfc\ub300\ud559(\uc778\ubb38\ud559\ubd80)\uacfc \uc790\uc5f0\uacc4 1\uac1c \ub2e8\uacfc\ub300\ud559(\uc218\uc0b0\ud559\ubd80) \uccb4\uc81c\uc758 \uad6d\ub9bd \ubd80\uc0b0\ub300\ud559\uad50(\uc885\ud569\ub300\ud559) \uc124\ub9bd\uc744 \uc778\uac00\ud558\uc600\ub294\ub370, \ud559\uad50\uc758 \uba85\uce6d\uc740 \u2018\ubd80\uc0b0\ub300\ud559\u2019\uc73c\ub85c, \ud559\ubd80(\ub2e8\uacfc\ub300\ud559)\ub294 \uc778\ubb38\ud559\ubd80\uc640 \uc218\uc0b0\ud559\ubd80\ub85c \uac1c\uad50\ud558\uc600\uc73c\uba70(\uc774\ub54c\uc758 \ud559\ubd80\ub294 \uc624\ub298\ub0a0\uc758 \ub2e8\uacfc\ub300\ud559(Faculty)\uc5d0 \ud574\ub2f9\ud558\ub294 \uac83\uc73c\ub85c, 1946\ub144 3\uc6d4 7\uc77c\uc5d0 \uacf5\ud3ec\ub41c \ub300\ud559\ub839 \uc81c2\uc870 \ud6c4\ub2e8\uc5d0\ub294 \uc778\ubb38\uacc4 \ubc0f \uc790\uc5f0\uacc4\uc758 \ud559\ubd80\uac00 \ubcd1\uc124\ub420 \ub54c\ub294 2\uac1c \uc774\uc0c1\uc758 \ud559\ubd80\ub85c\uc368 '\uc885\ud569\ub300\ud559\uad50'\ub97c \uad6c\uc131\ud560 \uc218 \uc788\ub3c4\ub85d \uaddc\uc815\ub418\uc5b4 \uc788\uc5c8\ub2e4.", "paragraph_sentence": " 1946\ub144 5\uc6d4 15\uc77c\uc5d0 \ubb38\uad50\ubd80\ub294 \ubd80\uc0b0\uc218\uc0b0\uc804\ubb38\ud559\uad50\ub97c \ub300\ud559\uc73c\ub85c \uc2b9\uaca9\uc2dc\ucf1c \uc885\ud569\ub300\ud559\uc758 1\uac1c \ub2e8\uacfc\ub300\ud559(\uc218\uc0b0\ud559\ubd80)\uc73c\ub85c, \uadf8\uc640 \ubcc4\ub3c4\ub85c \uc778\ubb38\uacc4\ub300\ud559\uc744 \uc2e0\uc124\ud558\uc5ec \uc778\ubb38\uacc4 1\uac1c \ub2e8\uacfc\ub300\ud559(\uc778\ubb38\ud559\ubd80)\uacfc \uc790\uc5f0\uacc4 1\uac1c \ub2e8\uacfc\ub300\ud559(\uc218\uc0b0\ud559\ubd80) \uccb4\uc81c\uc758 \uad6d\ub9bd \ubd80\uc0b0\ub300\ud559\uad50(\uc885\ud569\ub300\ud559) \uc124\ub9bd\uc744 \uc778\uac00\ud558\uc600\ub294\ub370, \ud559\uad50\uc758 \uba85\uce6d\uc740 \u2018\ubd80\uc0b0\ub300\ud559\u2019\uc73c\ub85c, \ud559\ubd80(\ub2e8\uacfc\ub300\ud559)\ub294 \uc778\ubb38\ud559\ubd80\uc640 \uc218\uc0b0\ud559\ubd80\ub85c \uac1c\uad50\ud558\uc600\uc73c\uba70(\uc774\ub54c\uc758 \ud559\ubd80\ub294 \uc624\ub298\ub0a0\uc758 \ub2e8\uacfc\ub300\ud559(Faculty)\uc5d0 \ud574\ub2f9\ud558\ub294 \uac83\uc73c\ub85c, 1946\ub144 3\uc6d4 7\uc77c\uc5d0 \uacf5\ud3ec\ub41c \ub300\ud559\ub839 \uc81c2\uc870 \ud6c4\ub2e8\uc5d0\ub294 \uc778\ubb38\uacc4 \ubc0f \uc790\uc5f0\uacc4\uc758 \ud559\ubd80\uac00 \ubcd1\uc124\ub420 \ub54c\ub294 2\uac1c \uc774\uc0c1\uc758 \ud559\ubd80\ub85c\uc368 '\uc885\ud569\ub300\ud559\uad50'\ub97c \uad6c\uc131\ud560 \uc218 \uc788\ub3c4\ub85d \uaddc\uc815\ub418\uc5b4 \uc788\uc5c8\ub2e4. ) \uadf8\ud6c4 1946\ub144 7\uc6d4 \uc608\uacfc \uc2dc\ud5d8\uc744 \uac70\uce58\uace0, 1946\ub144 8\uc6d4\uc5d0\ub294 \ub300\ud559 \ucd1d\uc7a5\uc73c\ub85c \uc544\uc11c \ub9b0 \ubca0\ucee4(Arthur Lynn Becker 1879~1979, \ud55c\uad6d\uba85 \ubc31\uc544\ub355\u767d\u96c5\u60b3) \ubc15\uc0ac\ub97c \uc784\uba85\ud558\uc5ec 1946\ub144 9\uc6d4 \uc785\ud559\uc2dd\uc744 \uac70\ud589\ud558\uace0 1\ub144 \uc815\ub3c4 \uc885\ud569\ub300\ud559\uc73c\ub85c \uc6b4\uc601\ub418\ub2e4\uac00 1947\ub144 7\uc6d4 \uc218\uc0b0\uacfc\ub300\ud559\uc774 '\uad6d\ub9bd \ubd80\uc0b0\uc218\uc0b0\ub300\ud559'\uc73c\ub85c \ubd84\ub9ac\ub428\uc73c\ub85c\uc368 \uc885\ud569\ub300\ud559\uc73c\ub85c\uc368 '\uad6d\ub9bd \ubd80\uc0b0\ub300\ud559\uad50'\uc740 \uc0ac\uc2e4\uc0c1 \uc640\ud574\ub418\uc5c8\ub2e4. \uadf8 \ud6c4 1953\ub144 4\uc6d4 1\uc77c\uc5d0 \uad6d\ub9bd \ubd80\uc0b0\ub300\ud559\uad50 \uc124\ub9bd\uc548\uc774 \uad6d\ubb34\ud68c\uc758\ub97c \ud1b5\uacfc\ud558\uace0, 4\uc6d4 3\uc77c \uad6d\ubb34\ucd1d\ub9ac\uc758 \uc7ac\uac00\ub97c \ubc1b\uc544 \uac19\uc740 \ud574 9\uc6d415\uc77c \uad6d\ub9bd\ud559\uad50\uc124\uce58\ub839\uc774 \uacf5\ud3ec\ub428\uc5d0 \ub530\ub77c \ub2e4\uc2dc \uc885\ud569\ub300\ud559\uc73c\ub85c \uc2b9\uaca9\ub418\uc5c8\ub2e4.", "paragraph_answer": " 1946\ub144 5\uc6d4 15\uc77c\uc5d0 \ubb38\uad50\ubd80\ub294 \ubd80\uc0b0\uc218\uc0b0\uc804\ubb38\ud559\uad50\ub97c \ub300\ud559\uc73c\ub85c \uc2b9\uaca9\uc2dc\ucf1c \uc885\ud569\ub300\ud559\uc758 1\uac1c \ub2e8\uacfc\ub300\ud559(\uc218\uc0b0\ud559\ubd80)\uc73c\ub85c, \uadf8\uc640 \ubcc4\ub3c4\ub85c \uc778\ubb38\uacc4\ub300\ud559\uc744 \uc2e0\uc124\ud558\uc5ec \uc778\ubb38\uacc4 1\uac1c \ub2e8\uacfc\ub300\ud559(\uc778\ubb38\ud559\ubd80)\uacfc \uc790\uc5f0\uacc4 1\uac1c \ub2e8\uacfc\ub300\ud559(\uc218\uc0b0\ud559\ubd80) \uccb4\uc81c\uc758 \uad6d\ub9bd \ubd80\uc0b0\ub300\ud559\uad50(\uc885\ud569\ub300\ud559) \uc124\ub9bd\uc744 \uc778\uac00\ud558\uc600\ub294\ub370, \ud559\uad50\uc758 \uba85\uce6d\uc740 \u2018\ubd80\uc0b0\ub300\ud559\u2019\uc73c\ub85c, \ud559\ubd80(\ub2e8\uacfc\ub300\ud559)\ub294 \uc778\ubb38\ud559\ubd80\uc640 \uc218\uc0b0\ud559\ubd80\ub85c \uac1c\uad50\ud558\uc600\uc73c\uba70(\uc774\ub54c\uc758 \ud559\ubd80\ub294 \uc624\ub298\ub0a0\uc758 \ub2e8\uacfc\ub300\ud559(Faculty)\uc5d0 \ud574\ub2f9\ud558\ub294 \uac83\uc73c\ub85c, 1946\ub144 3\uc6d4 7\uc77c\uc5d0 \uacf5\ud3ec\ub41c \ub300\ud559\ub839 \uc81c2\uc870 \ud6c4\ub2e8\uc5d0\ub294 \uc778\ubb38\uacc4 \ubc0f \uc790\uc5f0\uacc4\uc758 \ud559\ubd80\uac00 \ubcd1\uc124\ub420 \ub54c\ub294 2\uac1c \uc774\uc0c1\uc758 \ud559\ubd80\ub85c\uc368 '\uc885\ud569\ub300\ud559\uad50'\ub97c \uad6c\uc131\ud560 \uc218 \uc788\ub3c4\ub85d \uaddc\uc815\ub418\uc5b4 \uc788\uc5c8\ub2e4.) \uadf8\ud6c4 1946\ub144 7\uc6d4 \uc608\uacfc \uc2dc\ud5d8\uc744 \uac70\uce58\uace0, 1946\ub144 8\uc6d4\uc5d0\ub294 \ub300\ud559 \ucd1d\uc7a5\uc73c\ub85c \uc544\uc11c \ub9b0 \ubca0\ucee4(Arthur Lynn Becker 1879~1979, \ud55c\uad6d\uba85 \ubc31\uc544\ub355\u767d\u96c5\u60b3) \ubc15\uc0ac\ub97c \uc784\uba85\ud558\uc5ec 1946\ub144 9\uc6d4 \uc785\ud559\uc2dd\uc744 \uac70\ud589\ud558\uace0 1\ub144 \uc815\ub3c4 \uc885\ud569\ub300\ud559\uc73c\ub85c \uc6b4\uc601\ub418\ub2e4\uac00 1947\ub144 7\uc6d4 \uc218\uc0b0\uacfc\ub300\ud559\uc774 '\uad6d\ub9bd \ubd80\uc0b0\uc218\uc0b0\ub300\ud559'\uc73c\ub85c \ubd84\ub9ac\ub428\uc73c\ub85c\uc368 \uc885\ud569\ub300\ud559\uc73c\ub85c\uc368 '\uad6d\ub9bd \ubd80\uc0b0\ub300\ud559\uad50'\uc740 \uc0ac\uc2e4\uc0c1 \uc640\ud574\ub418\uc5c8\ub2e4. \uadf8 \ud6c4 1953\ub144 4\uc6d4 1\uc77c\uc5d0 \uad6d\ub9bd \ubd80\uc0b0\ub300\ud559\uad50 \uc124\ub9bd\uc548\uc774 \uad6d\ubb34\ud68c\uc758\ub97c \ud1b5\uacfc\ud558\uace0, 4\uc6d4 3\uc77c \uad6d\ubb34\ucd1d\ub9ac\uc758 \uc7ac\uac00\ub97c \ubc1b\uc544 \uac19\uc740 \ud574 9\uc6d415\uc77c \uad6d\ub9bd\ud559\uad50\uc124\uce58\ub839\uc774 \uacf5\ud3ec\ub428\uc5d0 \ub530\ub77c \ub2e4\uc2dc \uc885\ud569\ub300\ud559\uc73c\ub85c \uc2b9\uaca9\ub418\uc5c8\ub2e4.", "sentence_answer": " 1946\ub144 5\uc6d4 15\uc77c\uc5d0 \ubb38\uad50\ubd80\ub294 \ubd80\uc0b0\uc218\uc0b0\uc804\ubb38\ud559\uad50\ub97c \ub300\ud559\uc73c\ub85c \uc2b9\uaca9\uc2dc\ucf1c \uc885\ud569\ub300\ud559\uc758 1\uac1c \ub2e8\uacfc\ub300\ud559(\uc218\uc0b0\ud559\ubd80)\uc73c\ub85c, \uadf8\uc640 \ubcc4\ub3c4\ub85c \uc778\ubb38\uacc4\ub300\ud559\uc744 \uc2e0\uc124\ud558\uc5ec \uc778\ubb38\uacc4 1\uac1c \ub2e8\uacfc\ub300\ud559(\uc778\ubb38\ud559\ubd80)\uacfc \uc790\uc5f0\uacc4 1\uac1c \ub2e8\uacfc\ub300\ud559(\uc218\uc0b0\ud559\ubd80) \uccb4\uc81c\uc758 \uad6d\ub9bd \ubd80\uc0b0\ub300\ud559\uad50(\uc885\ud569\ub300\ud559) \uc124\ub9bd\uc744 \uc778\uac00\ud558\uc600\ub294\ub370, \ud559\uad50\uc758 \uba85\uce6d\uc740 \u2018\ubd80\uc0b0\ub300\ud559\u2019\uc73c\ub85c, \ud559\ubd80(\ub2e8\uacfc\ub300\ud559)\ub294 \uc778\ubb38\ud559\ubd80\uc640 \uc218\uc0b0\ud559\ubd80\ub85c \uac1c\uad50\ud558\uc600\uc73c\uba70(\uc774\ub54c\uc758 \ud559\ubd80\ub294 \uc624\ub298\ub0a0\uc758 \ub2e8\uacfc\ub300\ud559(Faculty)\uc5d0 \ud574\ub2f9\ud558\ub294 \uac83\uc73c\ub85c, 1946\ub144 3\uc6d4 7\uc77c\uc5d0 \uacf5\ud3ec\ub41c \ub300\ud559\ub839 \uc81c2\uc870 \ud6c4\ub2e8\uc5d0\ub294 \uc778\ubb38\uacc4 \ubc0f \uc790\uc5f0\uacc4\uc758 \ud559\ubd80\uac00 \ubcd1\uc124\ub420 \ub54c\ub294 2\uac1c \uc774\uc0c1\uc758 \ud559\ubd80\ub85c\uc368 '\uc885\ud569\ub300\ud559\uad50'\ub97c \uad6c\uc131\ud560 \uc218 \uc788\ub3c4\ub85d \uaddc\uc815\ub418\uc5b4 \uc788\uc5c8\ub2e4.", "paragraph_id": "6582250-0-0", "paragraph_question": "question: \ubb38\uad50\ubd80\uac00 \ubd80\uc0b0\uc218\uc0b0\uc804\ubb38\ud559\uad50\ub97c \ub300\ud559\uc73c\ub85c \uc2b9\uaca9\uc2dc\ud0a8 \ud574\ub294?, context: 1946\ub144 5\uc6d4 15\uc77c\uc5d0 \ubb38\uad50\ubd80\ub294 \ubd80\uc0b0\uc218\uc0b0\uc804\ubb38\ud559\uad50\ub97c \ub300\ud559\uc73c\ub85c \uc2b9\uaca9\uc2dc\ucf1c \uc885\ud569\ub300\ud559\uc758 1\uac1c \ub2e8\uacfc\ub300\ud559(\uc218\uc0b0\ud559\ubd80)\uc73c\ub85c, \uadf8\uc640 \ubcc4\ub3c4\ub85c \uc778\ubb38\uacc4\ub300\ud559\uc744 \uc2e0\uc124\ud558\uc5ec \uc778\ubb38\uacc4 1\uac1c \ub2e8\uacfc\ub300\ud559(\uc778\ubb38\ud559\ubd80)\uacfc \uc790\uc5f0\uacc4 1\uac1c \ub2e8\uacfc\ub300\ud559(\uc218\uc0b0\ud559\ubd80) \uccb4\uc81c\uc758 \uad6d\ub9bd \ubd80\uc0b0\ub300\ud559\uad50(\uc885\ud569\ub300\ud559) \uc124\ub9bd\uc744 \uc778\uac00\ud558\uc600\ub294\ub370, \ud559\uad50\uc758 \uba85\uce6d\uc740 \u2018\ubd80\uc0b0\ub300\ud559\u2019\uc73c\ub85c, \ud559\ubd80(\ub2e8\uacfc\ub300\ud559)\ub294 \uc778\ubb38\ud559\ubd80\uc640 \uc218\uc0b0\ud559\ubd80\ub85c \uac1c\uad50\ud558\uc600\uc73c\uba70(\uc774\ub54c\uc758 \ud559\ubd80\ub294 \uc624\ub298\ub0a0\uc758 \ub2e8\uacfc\ub300\ud559(Faculty)\uc5d0 \ud574\ub2f9\ud558\ub294 \uac83\uc73c\ub85c, 1946\ub144 3\uc6d4 7\uc77c\uc5d0 \uacf5\ud3ec\ub41c \ub300\ud559\ub839 \uc81c2\uc870 \ud6c4\ub2e8\uc5d0\ub294 \uc778\ubb38\uacc4 \ubc0f \uc790\uc5f0\uacc4\uc758 \ud559\ubd80\uac00 \ubcd1\uc124\ub420 \ub54c\ub294 2\uac1c \uc774\uc0c1\uc758 \ud559\ubd80\ub85c\uc368 '\uc885\ud569\ub300\ud559\uad50'\ub97c \uad6c\uc131\ud560 \uc218 \uc788\ub3c4\ub85d \uaddc\uc815\ub418\uc5b4 \uc788\uc5c8\ub2e4.) \uadf8\ud6c4 1946\ub144 7\uc6d4 \uc608\uacfc \uc2dc\ud5d8\uc744 \uac70\uce58\uace0, 1946\ub144 8\uc6d4\uc5d0\ub294 \ub300\ud559 \ucd1d\uc7a5\uc73c\ub85c \uc544\uc11c \ub9b0 \ubca0\ucee4(Arthur Lynn Becker 1879~1979, \ud55c\uad6d\uba85 \ubc31\uc544\ub355\u767d\u96c5\u60b3) \ubc15\uc0ac\ub97c \uc784\uba85\ud558\uc5ec 1946\ub144 9\uc6d4 \uc785\ud559\uc2dd\uc744 \uac70\ud589\ud558\uace0 1\ub144 \uc815\ub3c4 \uc885\ud569\ub300\ud559\uc73c\ub85c \uc6b4\uc601\ub418\ub2e4\uac00 1947\ub144 7\uc6d4 \uc218\uc0b0\uacfc\ub300\ud559\uc774 '\uad6d\ub9bd \ubd80\uc0b0\uc218\uc0b0\ub300\ud559'\uc73c\ub85c \ubd84\ub9ac\ub428\uc73c\ub85c\uc368 \uc885\ud569\ub300\ud559\uc73c\ub85c\uc368 '\uad6d\ub9bd \ubd80\uc0b0\ub300\ud559\uad50'\uc740 \uc0ac\uc2e4\uc0c1 \uc640\ud574\ub418\uc5c8\ub2e4. \uadf8 \ud6c4 1953\ub144 4\uc6d4 1\uc77c\uc5d0 \uad6d\ub9bd \ubd80\uc0b0\ub300\ud559\uad50 \uc124\ub9bd\uc548\uc774 \uad6d\ubb34\ud68c\uc758\ub97c \ud1b5\uacfc\ud558\uace0, 4\uc6d4 3\uc77c \uad6d\ubb34\ucd1d\ub9ac\uc758 \uc7ac\uac00\ub97c \ubc1b\uc544 \uac19\uc740 \ud574 9\uc6d415\uc77c \uad6d\ub9bd\ud559\uad50\uc124\uce58\ub839\uc774 \uacf5\ud3ec\ub428\uc5d0 \ub530\ub77c \ub2e4\uc2dc \uc885\ud569\ub300\ud559\uc73c\ub85c \uc2b9\uaca9\ub418\uc5c8\ub2e4."} {"question": "\ud39c\ub4e4\ud134 \uc6cc\ub4dc\uac00 \uac00\uc7a5 \uc88b\uc544\ud558\ub294 \uc7a5\ub974\ub294?", "paragraph": "\ud39c\ub4e4\ud134 \uc6cc\ub4dc\ub294 \u300a\uc5b4\ub4dc\ubca4\ucc98 \ud0c0\uc784\u300b\uc774 \u2018\ub2e4\ud06c \ucf54\ubbf8\ub514\u2019\uc640 \uac19\ub2e4\uace0 \ud558\uba74\uc11c, \u201c\ub2e4\ud06c \ucf54\ubbf8\ub514\ub294 \ub0b4\uac00 \uac00\uc7a5 \uc88b\uc544\ud558\ub294 \uc7a5\ub974\uc774\ub2e4. \uadf8 \uc774\uc720\ub294 \ud589\ubcf5\uacfc \uacf5\ud3ec\ub97c \ub3d9\uc2dc\uc5d0 \ub290\ub084 \uc218 \uc788\uae30 \ub54c\ubb38\uc774\ub2e4. \uac00\uc2b4\uc774 \uc870\ub9c8\uc870\ub9c8\ud558\ub294 \ub3d9\uc2dc\uc5d0 \ud589\ubcf5\uc744 \ub290\ub07c\ub294 \uac83, \uadf8\ub7ec\ud55c \ubaa8\uc21c\uc801\uc778 \uac10\uc815\uc744 \ub290\ub07c\ub294 \uac83\uc774 \uc88b\ub2e4. \uadf8\ub9ac\uace0 \ub0b4 \uc0dd\uac01\uc5d0\ub294 \uc6b0\ub9ac \uc791\ud488 \uc548\uc5d0 \uadf8\ub7ec\ud55c \uc694\uc18c\uac00 \ub9ce\uc774 \ub4e4\uc5b4\uc788\ub294 \uac83 \uac19\ub2e4.\u201d\ub77c\uace0 \uc5b8\uae09\ud588\ub2e4. \uc774\uadf8\uc81c\ud050\ud2f0\ube0c \ud504\ub85c\ub4c0\uc11c\uc778 \ud504\ub808\ub4dc \uc138\uc774\ubc84\ud2b8\ub294 \uc791\ud488\uc758 \uc2a4\ud0c0\uc77c\uc744 \u300a\uace0\uc591\uc774 \ud3a0\ub9ad\uc2a4\u300b\uc640 \ub9e5\uc2a4 \ud50c\ub77c\uc774\uc154\uc758 \uc791\ud488\ub4e4\uc5d0 \ube44\uc720\ud558\uc600\uace0, \ub610\ud55c \uc791\uc911\uc758 \uc138\uacc4\uac00 \uc5ec\ub7ec \ube44\ub514\uc624 \uac8c\uc784\uc758 \uc601\ud5a5\ub3c4 \ubc1b\uc558\ub2e4\uace0 \ub9d0\ud588\ub2e4. \uc6cc\ub4dc\ub294 \uc791\ud488\uc5d0 \ub300\ud55c \uc601\uac10\uc758 \ub300\ubd80\ubd84\uc744 \uc81c\uc791\uc9c4 \ub2e4\uc218\uac00 \uc88b\uc544\ud558\ub294 \ud310\ud0c0\uc9c0 \ud14c\uc774\ube14 \ub864\ud50c\ub808\uc789 \uac8c\uc784\uc778 \u300a\ub358\uc804 \uc564 \ub4dc\ub798\uace4\u300b\uc5d0\uc11c \uc5bb\uc5c8\ub2e4\uace0 \uc5ec\ub7ec \ucc28\ub840 \uc5b8\uae09\ud588\ub2e4. \uadf8\ub294 \uc791\uc911\uc758 \uc138\uacc4\uac00 \u2018\ub9cc\ud654\uc801\uc778 \uc2ac\ub7a9\uc2a4\ud2f1\u2019 \ub300\uc2e0 \uc77c\uc815\ud55c \ubb3c\ub9ac\uc801\uc778 \ud0c0\ub2f9\uc131\uc744 \uac16\ub3c4\ub85d \uc758\ub3c4\ud588\ub2e4. \uadf8\ub85c \uc778\ud574 \uc774\uc57c\uae30 \uc18d\uc5d0 \ub9c8\ubc95\uc774 \uc874\uc7ac\ub294 \ud558\uc9c0\ub9cc, \uc791\uac00\ub4e4\uc740 \ub4f1\uc7a5\uc778\ubb3c\uc774 \uc791\uc911\uc758 \uc138\uacc4\uacfc \uc0c1\ud638 \uc791\uc6a9\uc774 \uac00\ub2a5\ud558\ub3c4\ub85d \ub0b4\uc801\uc778 \uc77c\uace0\ub098\uc131\uc744 \ub9cc\ub4e4\ub824\uace0 \ub178\ub825\ud55c\ub2e4\uace0 \ub9d0\ud588\ub2e4. \ubbf8\uad6d\uc5d0\uc11c \u300a\uc5b4\ub4dc\ubca4\ucc98 \ud0c0\uc784\u300b\uc740 TV-PG \ub4f1\uae09\uc744 \ubc1b\uc558\ub2e4. \uc774\uc5d0 \ub300\ud574 \uc6cc\ub4dc\ub294 \uc790\uc2e0\uc774 \ud574\ub2f9 \ub4f1\uae09\uc744 \ubc1b\ub294 \uac83\uc744 \uc804\ud600 \uc6d0\ud558\uc9c0 \uc54a\uc558\ub2e4\uace0 \ud588\uc73c\uba70, \u201c\uc2dc\uccad \ub4f1\uae09\uc5d0 \ub300\ud574\uc11c\ub294 \uc0dd\uac01\ud574 \ubcf8 \uc801\uc774 \uc5c6\ub2e4. \uc6b0\ub9ac\ub294 \uc0c1\uc2a4\ub7ec\uc6b4 \uac83\uc744 \uc88b\uc544\ud558\uc9c0 \uc54a\ub294\ub2e4. \uadc0\uc5fd\uac70\ub098 \uba4b\uc9c4 \uac83\ub4e4\uc774 \uc88b\ub2e4.\u201d\ub77c\uace0 \ub9d0\ud588\ub2e4.", "answer": "\ub2e4\ud06c \ucf54\ubbf8\ub514", "sentence": "\ud39c\ub4e4\ud134 \uc6cc\ub4dc\ub294 \u300a\uc5b4\ub4dc\ubca4\ucc98 \ud0c0\uc784\u300b\uc774 \u2018 \ub2e4\ud06c \ucf54\ubbf8\ub514 \u2019\uc640 \uac19\ub2e4\uace0 \ud558\uba74\uc11c, \u201c\ub2e4\ud06c \ucf54\ubbf8\ub514\ub294 \ub0b4\uac00 \uac00\uc7a5 \uc88b\uc544\ud558\ub294 \uc7a5\ub974\uc774\ub2e4.", "paragraph_sentence": " \ud39c\ub4e4\ud134 \uc6cc\ub4dc\ub294 \u300a\uc5b4\ub4dc\ubca4\ucc98 \ud0c0\uc784\u300b\uc774 \u2018 \ub2e4\ud06c \ucf54\ubbf8\ub514 \u2019\uc640 \uac19\ub2e4\uace0 \ud558\uba74\uc11c, \u201c\ub2e4\ud06c \ucf54\ubbf8\ub514\ub294 \ub0b4\uac00 \uac00\uc7a5 \uc88b\uc544\ud558\ub294 \uc7a5\ub974\uc774\ub2e4. \uadf8 \uc774\uc720\ub294 \ud589\ubcf5\uacfc \uacf5\ud3ec\ub97c \ub3d9\uc2dc\uc5d0 \ub290\ub084 \uc218 \uc788\uae30 \ub54c\ubb38\uc774\ub2e4. \uac00\uc2b4\uc774 \uc870\ub9c8\uc870\ub9c8\ud558\ub294 \ub3d9\uc2dc\uc5d0 \ud589\ubcf5\uc744 \ub290\ub07c\ub294 \uac83, \uadf8\ub7ec\ud55c \ubaa8\uc21c\uc801\uc778 \uac10\uc815\uc744 \ub290\ub07c\ub294 \uac83\uc774 \uc88b\ub2e4. \uadf8\ub9ac\uace0 \ub0b4 \uc0dd\uac01\uc5d0\ub294 \uc6b0\ub9ac \uc791\ud488 \uc548\uc5d0 \uadf8\ub7ec\ud55c \uc694\uc18c\uac00 \ub9ce\uc774 \ub4e4\uc5b4\uc788\ub294 \uac83 \uac19\ub2e4. \u201d\ub77c\uace0 \uc5b8\uae09\ud588\ub2e4. \uc774\uadf8\uc81c\ud050\ud2f0\ube0c \ud504\ub85c\ub4c0\uc11c\uc778 \ud504\ub808\ub4dc \uc138\uc774\ubc84\ud2b8\ub294 \uc791\ud488\uc758 \uc2a4\ud0c0\uc77c\uc744 \u300a\uace0\uc591\uc774 \ud3a0\ub9ad\uc2a4\u300b\uc640 \ub9e5\uc2a4 \ud50c\ub77c\uc774\uc154\uc758 \uc791\ud488\ub4e4\uc5d0 \ube44\uc720\ud558\uc600\uace0, \ub610\ud55c \uc791\uc911\uc758 \uc138\uacc4\uac00 \uc5ec\ub7ec \ube44\ub514\uc624 \uac8c\uc784\uc758 \uc601\ud5a5\ub3c4 \ubc1b\uc558\ub2e4\uace0 \ub9d0\ud588\ub2e4. \uc6cc\ub4dc\ub294 \uc791\ud488\uc5d0 \ub300\ud55c \uc601\uac10\uc758 \ub300\ubd80\ubd84\uc744 \uc81c\uc791\uc9c4 \ub2e4\uc218\uac00 \uc88b\uc544\ud558\ub294 \ud310\ud0c0\uc9c0 \ud14c\uc774\ube14 \ub864\ud50c\ub808\uc789 \uac8c\uc784\uc778 \u300a\ub358\uc804 \uc564 \ub4dc\ub798\uace4\u300b\uc5d0\uc11c \uc5bb\uc5c8\ub2e4\uace0 \uc5ec\ub7ec \ucc28\ub840 \uc5b8\uae09\ud588\ub2e4. \uadf8\ub294 \uc791\uc911\uc758 \uc138\uacc4\uac00 \u2018\ub9cc\ud654\uc801\uc778 \uc2ac\ub7a9\uc2a4\ud2f1\u2019 \ub300\uc2e0 \uc77c\uc815\ud55c \ubb3c\ub9ac\uc801\uc778 \ud0c0\ub2f9\uc131\uc744 \uac16\ub3c4\ub85d \uc758\ub3c4\ud588\ub2e4. \uadf8\ub85c \uc778\ud574 \uc774\uc57c\uae30 \uc18d\uc5d0 \ub9c8\ubc95\uc774 \uc874\uc7ac\ub294 \ud558\uc9c0\ub9cc, \uc791\uac00\ub4e4\uc740 \ub4f1\uc7a5\uc778\ubb3c\uc774 \uc791\uc911\uc758 \uc138\uacc4\uacfc \uc0c1\ud638 \uc791\uc6a9\uc774 \uac00\ub2a5\ud558\ub3c4\ub85d \ub0b4\uc801\uc778 \uc77c\uace0\ub098\uc131\uc744 \ub9cc\ub4e4\ub824\uace0 \ub178\ub825\ud55c\ub2e4\uace0 \ub9d0\ud588\ub2e4. \ubbf8\uad6d\uc5d0\uc11c \u300a\uc5b4\ub4dc\ubca4\ucc98 \ud0c0\uc784\u300b\uc740 TV-PG \ub4f1\uae09\uc744 \ubc1b\uc558\ub2e4. \uc774\uc5d0 \ub300\ud574 \uc6cc\ub4dc\ub294 \uc790\uc2e0\uc774 \ud574\ub2f9 \ub4f1\uae09\uc744 \ubc1b\ub294 \uac83\uc744 \uc804\ud600 \uc6d0\ud558\uc9c0 \uc54a\uc558\ub2e4\uace0 \ud588\uc73c\uba70, \u201c\uc2dc\uccad \ub4f1\uae09\uc5d0 \ub300\ud574\uc11c\ub294 \uc0dd\uac01\ud574 \ubcf8 \uc801\uc774 \uc5c6\ub2e4. \uc6b0\ub9ac\ub294 \uc0c1\uc2a4\ub7ec\uc6b4 \uac83\uc744 \uc88b\uc544\ud558\uc9c0 \uc54a\ub294\ub2e4. \uadc0\uc5fd\uac70\ub098 \uba4b\uc9c4 \uac83\ub4e4\uc774 \uc88b\ub2e4. \u201d\ub77c\uace0 \ub9d0\ud588\ub2e4.", "paragraph_answer": "\ud39c\ub4e4\ud134 \uc6cc\ub4dc\ub294 \u300a\uc5b4\ub4dc\ubca4\ucc98 \ud0c0\uc784\u300b\uc774 \u2018 \ub2e4\ud06c \ucf54\ubbf8\ub514 \u2019\uc640 \uac19\ub2e4\uace0 \ud558\uba74\uc11c, \u201c\ub2e4\ud06c \ucf54\ubbf8\ub514\ub294 \ub0b4\uac00 \uac00\uc7a5 \uc88b\uc544\ud558\ub294 \uc7a5\ub974\uc774\ub2e4. \uadf8 \uc774\uc720\ub294 \ud589\ubcf5\uacfc \uacf5\ud3ec\ub97c \ub3d9\uc2dc\uc5d0 \ub290\ub084 \uc218 \uc788\uae30 \ub54c\ubb38\uc774\ub2e4. \uac00\uc2b4\uc774 \uc870\ub9c8\uc870\ub9c8\ud558\ub294 \ub3d9\uc2dc\uc5d0 \ud589\ubcf5\uc744 \ub290\ub07c\ub294 \uac83, \uadf8\ub7ec\ud55c \ubaa8\uc21c\uc801\uc778 \uac10\uc815\uc744 \ub290\ub07c\ub294 \uac83\uc774 \uc88b\ub2e4. \uadf8\ub9ac\uace0 \ub0b4 \uc0dd\uac01\uc5d0\ub294 \uc6b0\ub9ac \uc791\ud488 \uc548\uc5d0 \uadf8\ub7ec\ud55c \uc694\uc18c\uac00 \ub9ce\uc774 \ub4e4\uc5b4\uc788\ub294 \uac83 \uac19\ub2e4.\u201d\ub77c\uace0 \uc5b8\uae09\ud588\ub2e4. \uc774\uadf8\uc81c\ud050\ud2f0\ube0c \ud504\ub85c\ub4c0\uc11c\uc778 \ud504\ub808\ub4dc \uc138\uc774\ubc84\ud2b8\ub294 \uc791\ud488\uc758 \uc2a4\ud0c0\uc77c\uc744 \u300a\uace0\uc591\uc774 \ud3a0\ub9ad\uc2a4\u300b\uc640 \ub9e5\uc2a4 \ud50c\ub77c\uc774\uc154\uc758 \uc791\ud488\ub4e4\uc5d0 \ube44\uc720\ud558\uc600\uace0, \ub610\ud55c \uc791\uc911\uc758 \uc138\uacc4\uac00 \uc5ec\ub7ec \ube44\ub514\uc624 \uac8c\uc784\uc758 \uc601\ud5a5\ub3c4 \ubc1b\uc558\ub2e4\uace0 \ub9d0\ud588\ub2e4. \uc6cc\ub4dc\ub294 \uc791\ud488\uc5d0 \ub300\ud55c \uc601\uac10\uc758 \ub300\ubd80\ubd84\uc744 \uc81c\uc791\uc9c4 \ub2e4\uc218\uac00 \uc88b\uc544\ud558\ub294 \ud310\ud0c0\uc9c0 \ud14c\uc774\ube14 \ub864\ud50c\ub808\uc789 \uac8c\uc784\uc778 \u300a\ub358\uc804 \uc564 \ub4dc\ub798\uace4\u300b\uc5d0\uc11c \uc5bb\uc5c8\ub2e4\uace0 \uc5ec\ub7ec \ucc28\ub840 \uc5b8\uae09\ud588\ub2e4. \uadf8\ub294 \uc791\uc911\uc758 \uc138\uacc4\uac00 \u2018\ub9cc\ud654\uc801\uc778 \uc2ac\ub7a9\uc2a4\ud2f1\u2019 \ub300\uc2e0 \uc77c\uc815\ud55c \ubb3c\ub9ac\uc801\uc778 \ud0c0\ub2f9\uc131\uc744 \uac16\ub3c4\ub85d \uc758\ub3c4\ud588\ub2e4. \uadf8\ub85c \uc778\ud574 \uc774\uc57c\uae30 \uc18d\uc5d0 \ub9c8\ubc95\uc774 \uc874\uc7ac\ub294 \ud558\uc9c0\ub9cc, \uc791\uac00\ub4e4\uc740 \ub4f1\uc7a5\uc778\ubb3c\uc774 \uc791\uc911\uc758 \uc138\uacc4\uacfc \uc0c1\ud638 \uc791\uc6a9\uc774 \uac00\ub2a5\ud558\ub3c4\ub85d \ub0b4\uc801\uc778 \uc77c\uace0\ub098\uc131\uc744 \ub9cc\ub4e4\ub824\uace0 \ub178\ub825\ud55c\ub2e4\uace0 \ub9d0\ud588\ub2e4. \ubbf8\uad6d\uc5d0\uc11c \u300a\uc5b4\ub4dc\ubca4\ucc98 \ud0c0\uc784\u300b\uc740 TV-PG \ub4f1\uae09\uc744 \ubc1b\uc558\ub2e4. \uc774\uc5d0 \ub300\ud574 \uc6cc\ub4dc\ub294 \uc790\uc2e0\uc774 \ud574\ub2f9 \ub4f1\uae09\uc744 \ubc1b\ub294 \uac83\uc744 \uc804\ud600 \uc6d0\ud558\uc9c0 \uc54a\uc558\ub2e4\uace0 \ud588\uc73c\uba70, \u201c\uc2dc\uccad \ub4f1\uae09\uc5d0 \ub300\ud574\uc11c\ub294 \uc0dd\uac01\ud574 \ubcf8 \uc801\uc774 \uc5c6\ub2e4. \uc6b0\ub9ac\ub294 \uc0c1\uc2a4\ub7ec\uc6b4 \uac83\uc744 \uc88b\uc544\ud558\uc9c0 \uc54a\ub294\ub2e4. \uadc0\uc5fd\uac70\ub098 \uba4b\uc9c4 \uac83\ub4e4\uc774 \uc88b\ub2e4.\u201d\ub77c\uace0 \ub9d0\ud588\ub2e4.", "sentence_answer": "\ud39c\ub4e4\ud134 \uc6cc\ub4dc\ub294 \u300a\uc5b4\ub4dc\ubca4\ucc98 \ud0c0\uc784\u300b\uc774 \u2018 \ub2e4\ud06c \ucf54\ubbf8\ub514 \u2019\uc640 \uac19\ub2e4\uace0 \ud558\uba74\uc11c, \u201c\ub2e4\ud06c \ucf54\ubbf8\ub514\ub294 \ub0b4\uac00 \uac00\uc7a5 \uc88b\uc544\ud558\ub294 \uc7a5\ub974\uc774\ub2e4.", "paragraph_id": "6569097-5-0", "paragraph_question": "question: \ud39c\ub4e4\ud134 \uc6cc\ub4dc\uac00 \uac00\uc7a5 \uc88b\uc544\ud558\ub294 \uc7a5\ub974\ub294?, context: \ud39c\ub4e4\ud134 \uc6cc\ub4dc\ub294 \u300a\uc5b4\ub4dc\ubca4\ucc98 \ud0c0\uc784\u300b\uc774 \u2018\ub2e4\ud06c \ucf54\ubbf8\ub514\u2019\uc640 \uac19\ub2e4\uace0 \ud558\uba74\uc11c, \u201c\ub2e4\ud06c \ucf54\ubbf8\ub514\ub294 \ub0b4\uac00 \uac00\uc7a5 \uc88b\uc544\ud558\ub294 \uc7a5\ub974\uc774\ub2e4. \uadf8 \uc774\uc720\ub294 \ud589\ubcf5\uacfc \uacf5\ud3ec\ub97c \ub3d9\uc2dc\uc5d0 \ub290\ub084 \uc218 \uc788\uae30 \ub54c\ubb38\uc774\ub2e4. \uac00\uc2b4\uc774 \uc870\ub9c8\uc870\ub9c8\ud558\ub294 \ub3d9\uc2dc\uc5d0 \ud589\ubcf5\uc744 \ub290\ub07c\ub294 \uac83, \uadf8\ub7ec\ud55c \ubaa8\uc21c\uc801\uc778 \uac10\uc815\uc744 \ub290\ub07c\ub294 \uac83\uc774 \uc88b\ub2e4. \uadf8\ub9ac\uace0 \ub0b4 \uc0dd\uac01\uc5d0\ub294 \uc6b0\ub9ac \uc791\ud488 \uc548\uc5d0 \uadf8\ub7ec\ud55c \uc694\uc18c\uac00 \ub9ce\uc774 \ub4e4\uc5b4\uc788\ub294 \uac83 \uac19\ub2e4.\u201d\ub77c\uace0 \uc5b8\uae09\ud588\ub2e4. \uc774\uadf8\uc81c\ud050\ud2f0\ube0c \ud504\ub85c\ub4c0\uc11c\uc778 \ud504\ub808\ub4dc \uc138\uc774\ubc84\ud2b8\ub294 \uc791\ud488\uc758 \uc2a4\ud0c0\uc77c\uc744 \u300a\uace0\uc591\uc774 \ud3a0\ub9ad\uc2a4\u300b\uc640 \ub9e5\uc2a4 \ud50c\ub77c\uc774\uc154\uc758 \uc791\ud488\ub4e4\uc5d0 \ube44\uc720\ud558\uc600\uace0, \ub610\ud55c \uc791\uc911\uc758 \uc138\uacc4\uac00 \uc5ec\ub7ec \ube44\ub514\uc624 \uac8c\uc784\uc758 \uc601\ud5a5\ub3c4 \ubc1b\uc558\ub2e4\uace0 \ub9d0\ud588\ub2e4. \uc6cc\ub4dc\ub294 \uc791\ud488\uc5d0 \ub300\ud55c \uc601\uac10\uc758 \ub300\ubd80\ubd84\uc744 \uc81c\uc791\uc9c4 \ub2e4\uc218\uac00 \uc88b\uc544\ud558\ub294 \ud310\ud0c0\uc9c0 \ud14c\uc774\ube14 \ub864\ud50c\ub808\uc789 \uac8c\uc784\uc778 \u300a\ub358\uc804 \uc564 \ub4dc\ub798\uace4\u300b\uc5d0\uc11c \uc5bb\uc5c8\ub2e4\uace0 \uc5ec\ub7ec \ucc28\ub840 \uc5b8\uae09\ud588\ub2e4. \uadf8\ub294 \uc791\uc911\uc758 \uc138\uacc4\uac00 \u2018\ub9cc\ud654\uc801\uc778 \uc2ac\ub7a9\uc2a4\ud2f1\u2019 \ub300\uc2e0 \uc77c\uc815\ud55c \ubb3c\ub9ac\uc801\uc778 \ud0c0\ub2f9\uc131\uc744 \uac16\ub3c4\ub85d \uc758\ub3c4\ud588\ub2e4. \uadf8\ub85c \uc778\ud574 \uc774\uc57c\uae30 \uc18d\uc5d0 \ub9c8\ubc95\uc774 \uc874\uc7ac\ub294 \ud558\uc9c0\ub9cc, \uc791\uac00\ub4e4\uc740 \ub4f1\uc7a5\uc778\ubb3c\uc774 \uc791\uc911\uc758 \uc138\uacc4\uacfc \uc0c1\ud638 \uc791\uc6a9\uc774 \uac00\ub2a5\ud558\ub3c4\ub85d \ub0b4\uc801\uc778 \uc77c\uace0\ub098\uc131\uc744 \ub9cc\ub4e4\ub824\uace0 \ub178\ub825\ud55c\ub2e4\uace0 \ub9d0\ud588\ub2e4. \ubbf8\uad6d\uc5d0\uc11c \u300a\uc5b4\ub4dc\ubca4\ucc98 \ud0c0\uc784\u300b\uc740 TV-PG \ub4f1\uae09\uc744 \ubc1b\uc558\ub2e4. \uc774\uc5d0 \ub300\ud574 \uc6cc\ub4dc\ub294 \uc790\uc2e0\uc774 \ud574\ub2f9 \ub4f1\uae09\uc744 \ubc1b\ub294 \uac83\uc744 \uc804\ud600 \uc6d0\ud558\uc9c0 \uc54a\uc558\ub2e4\uace0 \ud588\uc73c\uba70, \u201c\uc2dc\uccad \ub4f1\uae09\uc5d0 \ub300\ud574\uc11c\ub294 \uc0dd\uac01\ud574 \ubcf8 \uc801\uc774 \uc5c6\ub2e4. \uc6b0\ub9ac\ub294 \uc0c1\uc2a4\ub7ec\uc6b4 \uac83\uc744 \uc88b\uc544\ud558\uc9c0 \uc54a\ub294\ub2e4. \uadc0\uc5fd\uac70\ub098 \uba4b\uc9c4 \uac83\ub4e4\uc774 \uc88b\ub2e4.\u201d\ub77c\uace0 \ub9d0\ud588\ub2e4."} {"question": "\uba54\uc774\ud2f0\uc81c\ube45\uc774 \ubcf4\uceec\ub9ac\uc2a4\ud2b8\uc778 \ubc34\ub4dc\uc774\ub984\uc740?", "paragraph": "\uc2a4\ud2f8\ud558\ud2b8(Steelheart)\ub294 1990\ub144\uc5d0 \uacb0\uc131\ub41c \ubbf8\uad6d\uc758 \uc74c\uc545 \uadf8\ub8f9\uc774\ub2e4. \ub9c8\uc774\ud074 \uba54\uc774\ud2f0\uc81c\ube45\uc774 \ubcf4\uceec\ub9ac\uc2a4\ud2b8\ub85c \ubab8\ub2f4\uc558\ub358 \uc774 \ubc34\ub4dc\ub294 \ud638\uc18c\ub825 \uc788\ub294 \uba54\ud0c8 \uc74c\uc545\uc744 \uc120\ubcf4\uc5ec \uc678\uad6d\ubcf4\ub2e4\ub294 \ud55c\uad6d\uc5d0\uc11c \ub9ce\uc740 \uc778\uae30\ub97c \uc5bb\uc5c8\ub2e4. \ub370\ubdd4 \uc568\ubc94\uc73c\ub85c \u3008Steelheart\u3009\ub97c \ubc1c\ud45c\ud558\uc600\ub2e4. \uac15\ucca0 \uc2ec\uc7a5\uc744 \ub514\uc790\uc778\ud55c \ub85c\uace0\ub97c \uc7ac\ud0b7\uc5d0 \ub2f4\uc740 \uc774 \uc568\ubc94\uc740 \ud638\uc18c\ub825 \uc9c4\ud55c \ub178\ub798 \u3008She's Gone\u3009\uc774 \ud070 \uc778\uae30\ub97c \uc5bb\uc5c8\uc73c\uba70, \uae54\ub054\ud55c \uba54\ud0c8 \uc0ac\uc6b4\ub4dc\ub85c \ucc44\uc0c9\ub41c \u3008I'll Never Let You Go\u3009, \u3008Can't Stop Me Lovin You\u3009 \ub4f1\uc774 \ud638\ud3c9\uc744 \ubc1b\uc558\ub2e4. 1992\ub144\uc5d0\ub294 2\uc9d1 \u300aTangled In Reins\u300b\ub97c \ubc1c\ud45c\ud558\uc5ec \ub9c8\uc774\ud06c \uc5b4\uba38\ub2c8\uc758 \uc8fd\uc74c\uc744 \uc9c1\uc811 \ubaa9\uaca9\ud55c \uc2ac\ud508\ub0b4\uc6a9\uc744 \ub2f4\uc740 \u3008Mama Don't You Cry\u3009\uc640 1\uc9d1\uc5d0\uc11c \ub9ce\uc740 \ud788\ud2b8\ub97c \uac70\ub450\uc5c8\ub358 \u3008She's Gone\u3009\uc744 \ub77c\uc774\ube0c\ub85c \uc218\ub85d\ud55c \uc791\ud488\uc774 \uc778\uae30\ub97c \uc5bb\uc5c8\ub2e4.", "answer": "\uc2a4\ud2f8\ud558\ud2b8", "sentence": "\uc2a4\ud2f8\ud558\ud2b8 (Steelheart)", "paragraph_sentence": " \uc2a4\ud2f8\ud558\ud2b8 (Steelheart) \ub294 1990\ub144\uc5d0 \uacb0\uc131\ub41c \ubbf8\uad6d\uc758 \uc74c\uc545 \uadf8\ub8f9\uc774\ub2e4. \ub9c8\uc774\ud074 \uba54\uc774\ud2f0\uc81c\ube45\uc774 \ubcf4\uceec\ub9ac\uc2a4\ud2b8\ub85c \ubab8\ub2f4\uc558\ub358 \uc774 \ubc34\ub4dc\ub294 \ud638\uc18c\ub825 \uc788\ub294 \uba54\ud0c8 \uc74c\uc545\uc744 \uc120\ubcf4\uc5ec \uc678\uad6d\ubcf4\ub2e4\ub294 \ud55c\uad6d\uc5d0\uc11c \ub9ce\uc740 \uc778\uae30\ub97c \uc5bb\uc5c8\ub2e4. \ub370\ubdd4 \uc568\ubc94\uc73c\ub85c \u3008Steelheart\u3009\ub97c \ubc1c\ud45c\ud558\uc600\ub2e4. \uac15\ucca0 \uc2ec\uc7a5\uc744 \ub514\uc790\uc778\ud55c \ub85c\uace0\ub97c \uc7ac\ud0b7\uc5d0 \ub2f4\uc740 \uc774 \uc568\ubc94\uc740 \ud638\uc18c\ub825 \uc9c4\ud55c \ub178\ub798 \u3008She's Gone\u3009\uc774 \ud070 \uc778\uae30\ub97c \uc5bb\uc5c8\uc73c\uba70, \uae54\ub054\ud55c \uba54\ud0c8 \uc0ac\uc6b4\ub4dc\ub85c \ucc44\uc0c9\ub41c \u3008I'll Never Let You Go\u3009, \u3008Can't Stop Me Lovin You\u3009 \ub4f1\uc774 \ud638\ud3c9\uc744 \ubc1b\uc558\ub2e4. 1992\ub144\uc5d0\ub294 2\uc9d1 \u300aTangled In Reins\u300b\ub97c \ubc1c\ud45c\ud558\uc5ec \ub9c8\uc774\ud06c \uc5b4\uba38\ub2c8\uc758 \uc8fd\uc74c\uc744 \uc9c1\uc811 \ubaa9\uaca9\ud55c \uc2ac\ud508\ub0b4\uc6a9\uc744 \ub2f4\uc740 \u3008Mama Don't You Cry\u3009\uc640 1\uc9d1\uc5d0\uc11c \ub9ce\uc740 \ud788\ud2b8\ub97c \uac70\ub450\uc5c8\ub358 \u3008She's Gone\u3009\uc744 \ub77c\uc774\ube0c\ub85c \uc218\ub85d\ud55c \uc791\ud488\uc774 \uc778\uae30\ub97c \uc5bb\uc5c8\ub2e4.", "paragraph_answer": " \uc2a4\ud2f8\ud558\ud2b8 (Steelheart)\ub294 1990\ub144\uc5d0 \uacb0\uc131\ub41c \ubbf8\uad6d\uc758 \uc74c\uc545 \uadf8\ub8f9\uc774\ub2e4. \ub9c8\uc774\ud074 \uba54\uc774\ud2f0\uc81c\ube45\uc774 \ubcf4\uceec\ub9ac\uc2a4\ud2b8\ub85c \ubab8\ub2f4\uc558\ub358 \uc774 \ubc34\ub4dc\ub294 \ud638\uc18c\ub825 \uc788\ub294 \uba54\ud0c8 \uc74c\uc545\uc744 \uc120\ubcf4\uc5ec \uc678\uad6d\ubcf4\ub2e4\ub294 \ud55c\uad6d\uc5d0\uc11c \ub9ce\uc740 \uc778\uae30\ub97c \uc5bb\uc5c8\ub2e4. \ub370\ubdd4 \uc568\ubc94\uc73c\ub85c \u3008Steelheart\u3009\ub97c \ubc1c\ud45c\ud558\uc600\ub2e4. \uac15\ucca0 \uc2ec\uc7a5\uc744 \ub514\uc790\uc778\ud55c \ub85c\uace0\ub97c \uc7ac\ud0b7\uc5d0 \ub2f4\uc740 \uc774 \uc568\ubc94\uc740 \ud638\uc18c\ub825 \uc9c4\ud55c \ub178\ub798 \u3008She's Gone\u3009\uc774 \ud070 \uc778\uae30\ub97c \uc5bb\uc5c8\uc73c\uba70, \uae54\ub054\ud55c \uba54\ud0c8 \uc0ac\uc6b4\ub4dc\ub85c \ucc44\uc0c9\ub41c \u3008I'll Never Let You Go\u3009, \u3008Can't Stop Me Lovin You\u3009 \ub4f1\uc774 \ud638\ud3c9\uc744 \ubc1b\uc558\ub2e4. 1992\ub144\uc5d0\ub294 2\uc9d1 \u300aTangled In Reins\u300b\ub97c \ubc1c\ud45c\ud558\uc5ec \ub9c8\uc774\ud06c \uc5b4\uba38\ub2c8\uc758 \uc8fd\uc74c\uc744 \uc9c1\uc811 \ubaa9\uaca9\ud55c \uc2ac\ud508\ub0b4\uc6a9\uc744 \ub2f4\uc740 \u3008Mama Don't You Cry\u3009\uc640 1\uc9d1\uc5d0\uc11c \ub9ce\uc740 \ud788\ud2b8\ub97c \uac70\ub450\uc5c8\ub358 \u3008She's Gone\u3009\uc744 \ub77c\uc774\ube0c\ub85c \uc218\ub85d\ud55c \uc791\ud488\uc774 \uc778\uae30\ub97c \uc5bb\uc5c8\ub2e4.", "sentence_answer": " \uc2a4\ud2f8\ud558\ud2b8 (Steelheart)", "paragraph_id": "6463744-0-0", "paragraph_question": "question: \uba54\uc774\ud2f0\uc81c\ube45\uc774 \ubcf4\uceec\ub9ac\uc2a4\ud2b8\uc778 \ubc34\ub4dc\uc774\ub984\uc740?, context: \uc2a4\ud2f8\ud558\ud2b8(Steelheart)\ub294 1990\ub144\uc5d0 \uacb0\uc131\ub41c \ubbf8\uad6d\uc758 \uc74c\uc545 \uadf8\ub8f9\uc774\ub2e4. \ub9c8\uc774\ud074 \uba54\uc774\ud2f0\uc81c\ube45\uc774 \ubcf4\uceec\ub9ac\uc2a4\ud2b8\ub85c \ubab8\ub2f4\uc558\ub358 \uc774 \ubc34\ub4dc\ub294 \ud638\uc18c\ub825 \uc788\ub294 \uba54\ud0c8 \uc74c\uc545\uc744 \uc120\ubcf4\uc5ec \uc678\uad6d\ubcf4\ub2e4\ub294 \ud55c\uad6d\uc5d0\uc11c \ub9ce\uc740 \uc778\uae30\ub97c \uc5bb\uc5c8\ub2e4. \ub370\ubdd4 \uc568\ubc94\uc73c\ub85c \u3008Steelheart\u3009\ub97c \ubc1c\ud45c\ud558\uc600\ub2e4. \uac15\ucca0 \uc2ec\uc7a5\uc744 \ub514\uc790\uc778\ud55c \ub85c\uace0\ub97c \uc7ac\ud0b7\uc5d0 \ub2f4\uc740 \uc774 \uc568\ubc94\uc740 \ud638\uc18c\ub825 \uc9c4\ud55c \ub178\ub798 \u3008She's Gone\u3009\uc774 \ud070 \uc778\uae30\ub97c \uc5bb\uc5c8\uc73c\uba70, \uae54\ub054\ud55c \uba54\ud0c8 \uc0ac\uc6b4\ub4dc\ub85c \ucc44\uc0c9\ub41c \u3008I'll Never Let You Go\u3009, \u3008Can't Stop Me Lovin You\u3009 \ub4f1\uc774 \ud638\ud3c9\uc744 \ubc1b\uc558\ub2e4. 1992\ub144\uc5d0\ub294 2\uc9d1 \u300aTangled In Reins\u300b\ub97c \ubc1c\ud45c\ud558\uc5ec \ub9c8\uc774\ud06c \uc5b4\uba38\ub2c8\uc758 \uc8fd\uc74c\uc744 \uc9c1\uc811 \ubaa9\uaca9\ud55c \uc2ac\ud508\ub0b4\uc6a9\uc744 \ub2f4\uc740 \u3008Mama Don't You Cry\u3009\uc640 1\uc9d1\uc5d0\uc11c \ub9ce\uc740 \ud788\ud2b8\ub97c \uac70\ub450\uc5c8\ub358 \u3008She's Gone\u3009\uc744 \ub77c\uc774\ube0c\ub85c \uc218\ub85d\ud55c \uc791\ud488\uc774 \uc778\uae30\ub97c \uc5bb\uc5c8\ub2e4."} {"question": "\uc138\uc6d4\ud638\ub294 \uba87\ub144\ub3c4\uc5d0 \uc77c\ubcf8\uc5d0\uc11c \uc218\uc785\ub418\uc5c8\ub294\uac00?", "paragraph": "\ud55c\uad6d\ud574\uc591\ub300 \ud574\uc0ac\uc218\uc1a1\uacfc\ud559\ubd80 \uae40\uae38\uc218 \uad50\uc218\ub294 \uc0ac\uace0\uc758 \uc6d0\uc778\uc774 \uad6c\uc870\ubcc0\uacbd\uc5d0 \uc788\uc744 \uc218 \uc788\ub2e4\uace0 \uc9c0\uc801\ud558\uc600\ub2e4. \uc2e4\uc81c\ub85c \uc138\uc6d4\ud638\ub294 2012\ub144 10\uc6d4 \uc77c\ubcf8\uc5d0\uc11c \uc218\uc785\ub41c \ub4a4, \uc774\ub4ec\ud574 3\uc6d4\uae4c\uc9c0 \uc804\ub0a8 \ubaa9\ud3ec\uc5d0\uc11c \uac1d\uc2e4\uc744 \uc99d\uc124\ud558\uc600\ub2e4. 3~5\uce35\uc5d0 \uac1d\uc2e4\uc774 \uc99d\uc124\ub418\uc5c8\uace0 \uc2b9\uc120\uc815\uc6d0\uc740 181\uba85 \ucd94\uac00\ub41c 921\uba85\uc73c\ub85c \ub9ce\uc544\uc84c\uc73c\uba70, \uc120\ubc15 \ubb34\uac8c\ub3c4 239t \uc99d\uac00\ud588\ub2e4. \uc774\uc5d0 \ub530\ub77c \ubb34\uac8c\uc911\uc2ec\uc774 \uc6d0\ub798\ubcf4\ub2e4 \ub192\uc544\uc84c\ub2e4\ub294 \uac83\uc774\ub2e4. \uae40\uad50\uc218\ub294 \"\ubc30\ub97c \uae09\uaca9\ud788 \ub3cc\ub9b0\ub2e4\uace0 \uc804\ubcf5\uc774 \ub418\uc9c0\ub294 \uc54a\ub294\ub2e4\" \uba70 \"\uad6c\uc870 \ubcc0\uacbd \uacfc\uc815\uc5d0\uc11c \ubb34\uac8c\uc911\uc2ec\uc774 \uc704\ub85c \uc62c\ub77c\uac14\ub2e4\uba74 \ub0b4\u318d\uc678\ubd80\uc5d0\uc11c \ucda9\uaca9\uc774 \uac00\ud574\uc84c\uc744 \ub54c \ubc30\uac00 \ub4a4\uc9d1\ud78c\ub2e4\" \uace0 \uc124\uba85\ud588\ub2e4. \uc774\uc5d0 \ub300\ud574 \uc138\uc6d4\ud638 \ub4f1\ub85d \uac80\uc0ac\ub97c \ub9e1\uc558\ub358 \ud55c\uad6d\uc120\uae09 \uad00\uacc4\uc790\ub294 \"\uac1d\uc2e4 \uc99d\uc124\uc740 \ud569\ubc95\uc801\uc778 \uacf5\uc0ac, \uc810\uac80 \uacb0\uacfc \uc6b4\ud56d\uc5d0 \ubb38\uc81c\uac00 \uc5c6\ub2e4\ub294 \uacb0\ub860\uc774 \ub098\uc640 \ub4f1\ub85d \uac80\uc0ac\ub97c \uc815\uc0c1 \ud1b5\uacfc\ud588\ub2e4\" \uace0 \ubc1d\ud614\ub2e4. \ud55c\ud3b8, \uac1d\uc2e4\ub9cc \uc99d\ucd95\ud55c \uac8c \uc544\ub2c8\ub77c, \ubc30 \uc55e\ucabd \ub7a8\ud504\uc6e8\uc774\ub3c4 \ucca0\uac70\ud55c \uac83\uc73c\ub85c \ub4dc\ub7ec\ub0ac\ub2e4. \ucd9c\uc785\uad6c\ub97c \uc904\uc774\uba74 \ud654\ubb3c\uc744 \ub354 \uc2e4\uc744 \uc218 \uc788\uace0, \ubc30 \uc804\uccb4 \ubb34\uac8c\uac00 \uac00\ubcbc\uc6cc\uc838 \uadf8\ub9cc\ud07c \uac1d\uc2e4 \uc99d\ucd95\uc5d0 \uc720\ub9ac\ud558\uae30 \ub54c\ubb38\uc774\uc5c8\uc744 \uac83\uc73c\ub85c \ucd94\uc815\ub41c\ub2e4. \ud55c\ucabd\uc5d0\ub9cc \uc788\ub358 \ub7a8\ud504\uc6e8\uc774\ub97c \uc5c6\uc560\uba74\uc11c \uc624\ub978\ucabd\uc774 \ub108\ubb34 \uac00\ubcbc\uc6cc\uc9c4 \uac8c \uc544\ub2c8\ub0d0\ub294 \uc758\ubb38\uc744 \uc81c\uae30\ud558\uae30\ub3c4 \ud55c\ub2e4.", "answer": "2012", "sentence": "\uc2e4\uc81c\ub85c \uc138\uc6d4\ud638\ub294 2012 \ub144 10\uc6d4 \uc77c\ubcf8\uc5d0\uc11c \uc218\uc785\ub41c \ub4a4, \uc774\ub4ec\ud574 3\uc6d4\uae4c\uc9c0 \uc804\ub0a8 \ubaa9\ud3ec\uc5d0\uc11c \uac1d\uc2e4\uc744 \uc99d\uc124\ud558\uc600\ub2e4.", "paragraph_sentence": "\ud55c\uad6d\ud574\uc591\ub300 \ud574\uc0ac\uc218\uc1a1\uacfc\ud559\ubd80 \uae40\uae38\uc218 \uad50\uc218\ub294 \uc0ac\uace0\uc758 \uc6d0\uc778\uc774 \uad6c\uc870\ubcc0\uacbd\uc5d0 \uc788\uc744 \uc218 \uc788\ub2e4\uace0 \uc9c0\uc801\ud558\uc600\ub2e4. \uc2e4\uc81c\ub85c \uc138\uc6d4\ud638\ub294 2012 \ub144 10\uc6d4 \uc77c\ubcf8\uc5d0\uc11c \uc218\uc785\ub41c \ub4a4, \uc774\ub4ec\ud574 3\uc6d4\uae4c\uc9c0 \uc804\ub0a8 \ubaa9\ud3ec\uc5d0\uc11c \uac1d\uc2e4\uc744 \uc99d\uc124\ud558\uc600\ub2e4. 3~5\uce35\uc5d0 \uac1d\uc2e4\uc774 \uc99d\uc124\ub418\uc5c8\uace0 \uc2b9\uc120\uc815\uc6d0\uc740 181\uba85 \ucd94\uac00\ub41c 921\uba85\uc73c\ub85c \ub9ce\uc544\uc84c\uc73c\uba70, \uc120\ubc15 \ubb34\uac8c\ub3c4 239t \uc99d\uac00\ud588\ub2e4. \uc774\uc5d0 \ub530\ub77c \ubb34\uac8c\uc911\uc2ec\uc774 \uc6d0\ub798\ubcf4\ub2e4 \ub192\uc544\uc84c\ub2e4\ub294 \uac83\uc774\ub2e4. \uae40\uad50\uc218\ub294 \"\ubc30\ub97c \uae09\uaca9\ud788 \ub3cc\ub9b0\ub2e4\uace0 \uc804\ubcf5\uc774 \ub418\uc9c0\ub294 \uc54a\ub294\ub2e4\" \uba70 \"\uad6c\uc870 \ubcc0\uacbd \uacfc\uc815\uc5d0\uc11c \ubb34\uac8c\uc911\uc2ec\uc774 \uc704\ub85c \uc62c\ub77c\uac14\ub2e4\uba74 \ub0b4\u318d\uc678\ubd80\uc5d0\uc11c \ucda9\uaca9\uc774 \uac00\ud574\uc84c\uc744 \ub54c \ubc30\uac00 \ub4a4\uc9d1\ud78c\ub2e4\" \uace0 \uc124\uba85\ud588\ub2e4. \uc774\uc5d0 \ub300\ud574 \uc138\uc6d4\ud638 \ub4f1\ub85d \uac80\uc0ac\ub97c \ub9e1\uc558\ub358 \ud55c\uad6d\uc120\uae09 \uad00\uacc4\uc790\ub294 \"\uac1d\uc2e4 \uc99d\uc124\uc740 \ud569\ubc95\uc801\uc778 \uacf5\uc0ac, \uc810\uac80 \uacb0\uacfc \uc6b4\ud56d\uc5d0 \ubb38\uc81c\uac00 \uc5c6\ub2e4\ub294 \uacb0\ub860\uc774 \ub098\uc640 \ub4f1\ub85d \uac80\uc0ac\ub97c \uc815\uc0c1 \ud1b5\uacfc\ud588\ub2e4\" \uace0 \ubc1d\ud614\ub2e4. \ud55c\ud3b8, \uac1d\uc2e4\ub9cc \uc99d\ucd95\ud55c \uac8c \uc544\ub2c8\ub77c, \ubc30 \uc55e\ucabd \ub7a8\ud504\uc6e8\uc774\ub3c4 \ucca0\uac70\ud55c \uac83\uc73c\ub85c \ub4dc\ub7ec\ub0ac\ub2e4. \ucd9c\uc785\uad6c\ub97c \uc904\uc774\uba74 \ud654\ubb3c\uc744 \ub354 \uc2e4\uc744 \uc218 \uc788\uace0, \ubc30 \uc804\uccb4 \ubb34\uac8c\uac00 \uac00\ubcbc\uc6cc\uc838 \uadf8\ub9cc\ud07c \uac1d\uc2e4 \uc99d\ucd95\uc5d0 \uc720\ub9ac\ud558\uae30 \ub54c\ubb38\uc774\uc5c8\uc744 \uac83\uc73c\ub85c \ucd94\uc815\ub41c\ub2e4. \ud55c\ucabd\uc5d0\ub9cc \uc788\ub358 \ub7a8\ud504\uc6e8\uc774\ub97c \uc5c6\uc560\uba74\uc11c \uc624\ub978\ucabd\uc774 \ub108\ubb34 \uac00\ubcbc\uc6cc\uc9c4 \uac8c \uc544\ub2c8\ub0d0\ub294 \uc758\ubb38\uc744 \uc81c\uae30\ud558\uae30\ub3c4 \ud55c\ub2e4.", "paragraph_answer": "\ud55c\uad6d\ud574\uc591\ub300 \ud574\uc0ac\uc218\uc1a1\uacfc\ud559\ubd80 \uae40\uae38\uc218 \uad50\uc218\ub294 \uc0ac\uace0\uc758 \uc6d0\uc778\uc774 \uad6c\uc870\ubcc0\uacbd\uc5d0 \uc788\uc744 \uc218 \uc788\ub2e4\uace0 \uc9c0\uc801\ud558\uc600\ub2e4. \uc2e4\uc81c\ub85c \uc138\uc6d4\ud638\ub294 2012 \ub144 10\uc6d4 \uc77c\ubcf8\uc5d0\uc11c \uc218\uc785\ub41c \ub4a4, \uc774\ub4ec\ud574 3\uc6d4\uae4c\uc9c0 \uc804\ub0a8 \ubaa9\ud3ec\uc5d0\uc11c \uac1d\uc2e4\uc744 \uc99d\uc124\ud558\uc600\ub2e4. 3~5\uce35\uc5d0 \uac1d\uc2e4\uc774 \uc99d\uc124\ub418\uc5c8\uace0 \uc2b9\uc120\uc815\uc6d0\uc740 181\uba85 \ucd94\uac00\ub41c 921\uba85\uc73c\ub85c \ub9ce\uc544\uc84c\uc73c\uba70, \uc120\ubc15 \ubb34\uac8c\ub3c4 239t \uc99d\uac00\ud588\ub2e4. \uc774\uc5d0 \ub530\ub77c \ubb34\uac8c\uc911\uc2ec\uc774 \uc6d0\ub798\ubcf4\ub2e4 \ub192\uc544\uc84c\ub2e4\ub294 \uac83\uc774\ub2e4. \uae40\uad50\uc218\ub294 \"\ubc30\ub97c \uae09\uaca9\ud788 \ub3cc\ub9b0\ub2e4\uace0 \uc804\ubcf5\uc774 \ub418\uc9c0\ub294 \uc54a\ub294\ub2e4\" \uba70 \"\uad6c\uc870 \ubcc0\uacbd \uacfc\uc815\uc5d0\uc11c \ubb34\uac8c\uc911\uc2ec\uc774 \uc704\ub85c \uc62c\ub77c\uac14\ub2e4\uba74 \ub0b4\u318d\uc678\ubd80\uc5d0\uc11c \ucda9\uaca9\uc774 \uac00\ud574\uc84c\uc744 \ub54c \ubc30\uac00 \ub4a4\uc9d1\ud78c\ub2e4\" \uace0 \uc124\uba85\ud588\ub2e4. \uc774\uc5d0 \ub300\ud574 \uc138\uc6d4\ud638 \ub4f1\ub85d \uac80\uc0ac\ub97c \ub9e1\uc558\ub358 \ud55c\uad6d\uc120\uae09 \uad00\uacc4\uc790\ub294 \"\uac1d\uc2e4 \uc99d\uc124\uc740 \ud569\ubc95\uc801\uc778 \uacf5\uc0ac, \uc810\uac80 \uacb0\uacfc \uc6b4\ud56d\uc5d0 \ubb38\uc81c\uac00 \uc5c6\ub2e4\ub294 \uacb0\ub860\uc774 \ub098\uc640 \ub4f1\ub85d \uac80\uc0ac\ub97c \uc815\uc0c1 \ud1b5\uacfc\ud588\ub2e4\" \uace0 \ubc1d\ud614\ub2e4. \ud55c\ud3b8, \uac1d\uc2e4\ub9cc \uc99d\ucd95\ud55c \uac8c \uc544\ub2c8\ub77c, \ubc30 \uc55e\ucabd \ub7a8\ud504\uc6e8\uc774\ub3c4 \ucca0\uac70\ud55c \uac83\uc73c\ub85c \ub4dc\ub7ec\ub0ac\ub2e4. \ucd9c\uc785\uad6c\ub97c \uc904\uc774\uba74 \ud654\ubb3c\uc744 \ub354 \uc2e4\uc744 \uc218 \uc788\uace0, \ubc30 \uc804\uccb4 \ubb34\uac8c\uac00 \uac00\ubcbc\uc6cc\uc838 \uadf8\ub9cc\ud07c \uac1d\uc2e4 \uc99d\ucd95\uc5d0 \uc720\ub9ac\ud558\uae30 \ub54c\ubb38\uc774\uc5c8\uc744 \uac83\uc73c\ub85c \ucd94\uc815\ub41c\ub2e4. \ud55c\ucabd\uc5d0\ub9cc \uc788\ub358 \ub7a8\ud504\uc6e8\uc774\ub97c \uc5c6\uc560\uba74\uc11c \uc624\ub978\ucabd\uc774 \ub108\ubb34 \uac00\ubcbc\uc6cc\uc9c4 \uac8c \uc544\ub2c8\ub0d0\ub294 \uc758\ubb38\uc744 \uc81c\uae30\ud558\uae30\ub3c4 \ud55c\ub2e4.", "sentence_answer": "\uc2e4\uc81c\ub85c \uc138\uc6d4\ud638\ub294 2012 \ub144 10\uc6d4 \uc77c\ubcf8\uc5d0\uc11c \uc218\uc785\ub41c \ub4a4, \uc774\ub4ec\ud574 3\uc6d4\uae4c\uc9c0 \uc804\ub0a8 \ubaa9\ud3ec\uc5d0\uc11c \uac1d\uc2e4\uc744 \uc99d\uc124\ud558\uc600\ub2e4.", "paragraph_id": "6532130-12-0", "paragraph_question": "question: \uc138\uc6d4\ud638\ub294 \uba87\ub144\ub3c4\uc5d0 \uc77c\ubcf8\uc5d0\uc11c \uc218\uc785\ub418\uc5c8\ub294\uac00?, context: \ud55c\uad6d\ud574\uc591\ub300 \ud574\uc0ac\uc218\uc1a1\uacfc\ud559\ubd80 \uae40\uae38\uc218 \uad50\uc218\ub294 \uc0ac\uace0\uc758 \uc6d0\uc778\uc774 \uad6c\uc870\ubcc0\uacbd\uc5d0 \uc788\uc744 \uc218 \uc788\ub2e4\uace0 \uc9c0\uc801\ud558\uc600\ub2e4. \uc2e4\uc81c\ub85c \uc138\uc6d4\ud638\ub294 2012\ub144 10\uc6d4 \uc77c\ubcf8\uc5d0\uc11c \uc218\uc785\ub41c \ub4a4, \uc774\ub4ec\ud574 3\uc6d4\uae4c\uc9c0 \uc804\ub0a8 \ubaa9\ud3ec\uc5d0\uc11c \uac1d\uc2e4\uc744 \uc99d\uc124\ud558\uc600\ub2e4. 3~5\uce35\uc5d0 \uac1d\uc2e4\uc774 \uc99d\uc124\ub418\uc5c8\uace0 \uc2b9\uc120\uc815\uc6d0\uc740 181\uba85 \ucd94\uac00\ub41c 921\uba85\uc73c\ub85c \ub9ce\uc544\uc84c\uc73c\uba70, \uc120\ubc15 \ubb34\uac8c\ub3c4 239t \uc99d\uac00\ud588\ub2e4. \uc774\uc5d0 \ub530\ub77c \ubb34\uac8c\uc911\uc2ec\uc774 \uc6d0\ub798\ubcf4\ub2e4 \ub192\uc544\uc84c\ub2e4\ub294 \uac83\uc774\ub2e4. \uae40\uad50\uc218\ub294 \"\ubc30\ub97c \uae09\uaca9\ud788 \ub3cc\ub9b0\ub2e4\uace0 \uc804\ubcf5\uc774 \ub418\uc9c0\ub294 \uc54a\ub294\ub2e4\" \uba70 \"\uad6c\uc870 \ubcc0\uacbd \uacfc\uc815\uc5d0\uc11c \ubb34\uac8c\uc911\uc2ec\uc774 \uc704\ub85c \uc62c\ub77c\uac14\ub2e4\uba74 \ub0b4\u318d\uc678\ubd80\uc5d0\uc11c \ucda9\uaca9\uc774 \uac00\ud574\uc84c\uc744 \ub54c \ubc30\uac00 \ub4a4\uc9d1\ud78c\ub2e4\" \uace0 \uc124\uba85\ud588\ub2e4. \uc774\uc5d0 \ub300\ud574 \uc138\uc6d4\ud638 \ub4f1\ub85d \uac80\uc0ac\ub97c \ub9e1\uc558\ub358 \ud55c\uad6d\uc120\uae09 \uad00\uacc4\uc790\ub294 \"\uac1d\uc2e4 \uc99d\uc124\uc740 \ud569\ubc95\uc801\uc778 \uacf5\uc0ac, \uc810\uac80 \uacb0\uacfc \uc6b4\ud56d\uc5d0 \ubb38\uc81c\uac00 \uc5c6\ub2e4\ub294 \uacb0\ub860\uc774 \ub098\uc640 \ub4f1\ub85d \uac80\uc0ac\ub97c \uc815\uc0c1 \ud1b5\uacfc\ud588\ub2e4\" \uace0 \ubc1d\ud614\ub2e4. \ud55c\ud3b8, \uac1d\uc2e4\ub9cc \uc99d\ucd95\ud55c \uac8c \uc544\ub2c8\ub77c, \ubc30 \uc55e\ucabd \ub7a8\ud504\uc6e8\uc774\ub3c4 \ucca0\uac70\ud55c \uac83\uc73c\ub85c \ub4dc\ub7ec\ub0ac\ub2e4. \ucd9c\uc785\uad6c\ub97c \uc904\uc774\uba74 \ud654\ubb3c\uc744 \ub354 \uc2e4\uc744 \uc218 \uc788\uace0, \ubc30 \uc804\uccb4 \ubb34\uac8c\uac00 \uac00\ubcbc\uc6cc\uc838 \uadf8\ub9cc\ud07c \uac1d\uc2e4 \uc99d\ucd95\uc5d0 \uc720\ub9ac\ud558\uae30 \ub54c\ubb38\uc774\uc5c8\uc744 \uac83\uc73c\ub85c \ucd94\uc815\ub41c\ub2e4. \ud55c\ucabd\uc5d0\ub9cc \uc788\ub358 \ub7a8\ud504\uc6e8\uc774\ub97c \uc5c6\uc560\uba74\uc11c \uc624\ub978\ucabd\uc774 \ub108\ubb34 \uac00\ubcbc\uc6cc\uc9c4 \uac8c \uc544\ub2c8\ub0d0\ub294 \uc758\ubb38\uc744 \uc81c\uae30\ud558\uae30\ub3c4 \ud55c\ub2e4."} {"question": "\ub808\ubc8c\uc5c5\uacfc \ud568\uaed8 \ubd80\uc5ec\ub418\uba70 \ud55c\ubc88 \ubc30\ubd84\ud558\uba74 \ub418\ub3cc\ub824 \ubc1b\uc744 \uc218 \uc5c6\ub294 \uac83\uc740?", "paragraph": "2. \uc77c\uaca9\uc774\ud0c8 \uc804\uc220: \ub04a\uc784\uc5c6\uc774 \uc774\ub3d9\ud558\uba74\uc11c, \uc801\uacfc\uc758 \uac70\ub9ac\ub97c \uc720\uc9c0\ud558\uba70 \uacf5\uaca9\ud558\ub294 \uc804\ubc95. \uc801\uc5d0\uac8c \uacf5\uaca9\uc744 \uac00\ud55c \ud6c4, \uc801\uacfc\uc758 \uac70\ub9ac\ub97c \ud56d\uc2dc \uc8fc\uc758\ud558\uba74\uc11c \uac70\ub9ac\uac00 \uac00\uae4c\uc6cc\uc9c0\uba74 \uc801\uc5d0\uac8c \uacf5\uaca9\uc744 \ub2f9\ud558\uae30 \uc804\uc5d0 \ubbf8\ub9ac \uc774\ub3d9\ud558\uc5ec \ub2e4\uc74c \ubc88 \uacf5\uaca9 \ud3ec\uc9c0\uc158\uc744 \ud0d0\uc0c9\ud558\ub294 \uc804\ubc95\uc774\ub2e4. \ud56d\uc0c1 \uc218 \ucd08 \ud6c4\uc758 \ubaac\uc2a4\ud130 \uc704\uce58\ub97c \uc608\uc0c1\ud558\uba74\uc11c \uce90\ub9ad\ud130\ub97c \ucee8\ud2b8\ub864\ud558\ub294 \uac83\uc740 \uc0c1\ub2f9\ud55c \uc219\ub828\uc744 \ud544\uc694\ub85c \ud560 \uac83\uc774\ub2e4. \ud558\uc9c0\ub9cc, \uc5ec\uae30\uc5d0 \uc775\uc219\ud574\uc9c0\uae30\ub9cc \ud55c\ub2e4\uba74 \uc774\ub860\uc801\uc73c\ub85c\ub294 \uc5b4\ub5a4 \uc0ac\ub0e5\ud130\uc5d0\uc11c\ub4e0 \ud63c\uc790\uc11c \uc0ac\ub0e5\uc744 \ud560 \uc218 \uc788\ub2e4. \ub530\ub77c\uc11c, \uc774\ub7ec\ud55c \uc804\uc220\uc5d0 \uc775\uc219\ud55c \ub9c8\ub3c4\uc0ac\ub9cc\uc774 \uc9c4\uc9dc \ub9c8\ub3c4\uc0ac\ub77c\uace0 \ud560 \uc218 \uc788\uc744 \uac83\uc774\ub2e4. \ub808\ubca8\uc5c5\uacfc \ud568\uaed8 \ubd80\uc5ec\ub418\ub294 \uc2a4\ud0ac \ud3ec\uc778\ud2b8\ub294 \ud55c\ubc88 \ubc30\ubd84\ud558\uba74 \ub418\ub3cc\ub824 \ubc1b\uc744 \uae38\uc774 \uc5c6\ub2e4. \uadf8\ub807\uae30 \ub54c\ubb38\uc5d0 \uc2a4\ud0ac \ud3ec\uc778\ud2b8\ub97c \uc0ac\uc6a9\ud558\uc9c0 \uc54a\uace0 \uc544\uaef4\ub9cc \ub450\ub294 \uacbd\uc6b0\uac00 \ub9ce\uc740\ub370, \ubaa8\ub4e0 \uacf5\uaca9\uc744 \ub9c8\ubc95 \uc2a4\ud0ac\uc5d0 \uc758\uc874\ud558\ub294 \ub9c8\ub3c4\uc0ac\ub4e4\uc5d0\uac8c \uc788\uc5b4\uc11c \uadf8\ub807\uac8c \uc2a4\ud0ac \ud3ec\uc778\ud2b8\ub97c \uc544\ub07c\uae30\ub9cc \ud558\ub294 \uac83\uc740 \uc790\uc2e0\uc758 \uacf5\uaca9\ub825\uc744 \uc81c\ud55c\ud558\ub294 \ud589\ub3d9\uc774 \ub418\uae30\ub3c4 \ud55c\ub2e4. \uc801\ub2f9\ud55c \uc2dc\uc810\uc5d0, \uc801\ub2f9\ud55c \ub9c8\ubc95\uc5d0, \uc801\ub2f9\ud55c \uc2a4\ud0ac \ud3ec\uc778\ud2b8\ub97c \ubd84\ubc30\ud558\ub294 \uac83\uc774 \uac00\uc7a5 \ud604\uba85\ud558\uac8c \uac8c\uc784\uc744 \uc774\ub04c\uc5b4\ub098\uac00\ub294 \ubc29\ubc95\uc774 \ub420 \uac83\uc774\ub2e4.", "answer": "\uc2a4\ud0ac \ud3ec\uc778\ud2b8", "sentence": "\ub808\ubca8\uc5c5\uacfc \ud568\uaed8 \ubd80\uc5ec\ub418\ub294 \uc2a4\ud0ac \ud3ec\uc778\ud2b8 \ub294 \ud55c\ubc88 \ubc30\ubd84\ud558\uba74 \ub418\ub3cc\ub824 \ubc1b\uc744 \uae38\uc774 \uc5c6\ub2e4.", "paragraph_sentence": "2. \uc77c\uaca9\uc774\ud0c8 \uc804\uc220: \ub04a\uc784\uc5c6\uc774 \uc774\ub3d9\ud558\uba74\uc11c, \uc801\uacfc\uc758 \uac70\ub9ac\ub97c \uc720\uc9c0\ud558\uba70 \uacf5\uaca9\ud558\ub294 \uc804\ubc95. \uc801\uc5d0\uac8c \uacf5\uaca9\uc744 \uac00\ud55c \ud6c4, \uc801\uacfc\uc758 \uac70\ub9ac\ub97c \ud56d\uc2dc \uc8fc\uc758\ud558\uba74\uc11c \uac70\ub9ac\uac00 \uac00\uae4c\uc6cc\uc9c0\uba74 \uc801\uc5d0\uac8c \uacf5\uaca9\uc744 \ub2f9\ud558\uae30 \uc804\uc5d0 \ubbf8\ub9ac \uc774\ub3d9\ud558\uc5ec \ub2e4\uc74c \ubc88 \uacf5\uaca9 \ud3ec\uc9c0\uc158\uc744 \ud0d0\uc0c9\ud558\ub294 \uc804\ubc95\uc774\ub2e4. \ud56d\uc0c1 \uc218 \ucd08 \ud6c4\uc758 \ubaac\uc2a4\ud130 \uc704\uce58\ub97c \uc608\uc0c1\ud558\uba74\uc11c \uce90\ub9ad\ud130\ub97c \ucee8\ud2b8\ub864\ud558\ub294 \uac83\uc740 \uc0c1\ub2f9\ud55c \uc219\ub828\uc744 \ud544\uc694\ub85c \ud560 \uac83\uc774\ub2e4. \ud558\uc9c0\ub9cc, \uc5ec\uae30\uc5d0 \uc775\uc219\ud574\uc9c0\uae30\ub9cc \ud55c\ub2e4\uba74 \uc774\ub860\uc801\uc73c\ub85c\ub294 \uc5b4\ub5a4 \uc0ac\ub0e5\ud130\uc5d0\uc11c\ub4e0 \ud63c\uc790\uc11c \uc0ac\ub0e5\uc744 \ud560 \uc218 \uc788\ub2e4. \ub530\ub77c\uc11c, \uc774\ub7ec\ud55c \uc804\uc220\uc5d0 \uc775\uc219\ud55c \ub9c8\ub3c4\uc0ac\ub9cc\uc774 \uc9c4\uc9dc \ub9c8\ub3c4\uc0ac\ub77c\uace0 \ud560 \uc218 \uc788\uc744 \uac83\uc774\ub2e4. \ub808\ubca8\uc5c5\uacfc \ud568\uaed8 \ubd80\uc5ec\ub418\ub294 \uc2a4\ud0ac \ud3ec\uc778\ud2b8 \ub294 \ud55c\ubc88 \ubc30\ubd84\ud558\uba74 \ub418\ub3cc\ub824 \ubc1b\uc744 \uae38\uc774 \uc5c6\ub2e4. \uadf8\ub807\uae30 \ub54c\ubb38\uc5d0 \uc2a4\ud0ac \ud3ec\uc778\ud2b8\ub97c \uc0ac\uc6a9\ud558\uc9c0 \uc54a\uace0 \uc544\uaef4\ub9cc \ub450\ub294 \uacbd\uc6b0\uac00 \ub9ce\uc740\ub370, \ubaa8\ub4e0 \uacf5\uaca9\uc744 \ub9c8\ubc95 \uc2a4\ud0ac\uc5d0 \uc758\uc874\ud558\ub294 \ub9c8\ub3c4\uc0ac\ub4e4\uc5d0\uac8c \uc788\uc5b4\uc11c \uadf8\ub807\uac8c \uc2a4\ud0ac \ud3ec\uc778\ud2b8\ub97c \uc544\ub07c\uae30\ub9cc \ud558\ub294 \uac83\uc740 \uc790\uc2e0\uc758 \uacf5\uaca9\ub825\uc744 \uc81c\ud55c\ud558\ub294 \ud589\ub3d9\uc774 \ub418\uae30\ub3c4 \ud55c\ub2e4. \uc801\ub2f9\ud55c \uc2dc\uc810\uc5d0, \uc801\ub2f9\ud55c \ub9c8\ubc95\uc5d0, \uc801\ub2f9\ud55c \uc2a4\ud0ac \ud3ec\uc778\ud2b8\ub97c \ubd84\ubc30\ud558\ub294 \uac83\uc774 \uac00\uc7a5 \ud604\uba85\ud558\uac8c \uac8c\uc784\uc744 \uc774\ub04c\uc5b4\ub098\uac00\ub294 \ubc29\ubc95\uc774 \ub420 \uac83\uc774\ub2e4.", "paragraph_answer": "2. \uc77c\uaca9\uc774\ud0c8 \uc804\uc220: \ub04a\uc784\uc5c6\uc774 \uc774\ub3d9\ud558\uba74\uc11c, \uc801\uacfc\uc758 \uac70\ub9ac\ub97c \uc720\uc9c0\ud558\uba70 \uacf5\uaca9\ud558\ub294 \uc804\ubc95. \uc801\uc5d0\uac8c \uacf5\uaca9\uc744 \uac00\ud55c \ud6c4, \uc801\uacfc\uc758 \uac70\ub9ac\ub97c \ud56d\uc2dc \uc8fc\uc758\ud558\uba74\uc11c \uac70\ub9ac\uac00 \uac00\uae4c\uc6cc\uc9c0\uba74 \uc801\uc5d0\uac8c \uacf5\uaca9\uc744 \ub2f9\ud558\uae30 \uc804\uc5d0 \ubbf8\ub9ac \uc774\ub3d9\ud558\uc5ec \ub2e4\uc74c \ubc88 \uacf5\uaca9 \ud3ec\uc9c0\uc158\uc744 \ud0d0\uc0c9\ud558\ub294 \uc804\ubc95\uc774\ub2e4. \ud56d\uc0c1 \uc218 \ucd08 \ud6c4\uc758 \ubaac\uc2a4\ud130 \uc704\uce58\ub97c \uc608\uc0c1\ud558\uba74\uc11c \uce90\ub9ad\ud130\ub97c \ucee8\ud2b8\ub864\ud558\ub294 \uac83\uc740 \uc0c1\ub2f9\ud55c \uc219\ub828\uc744 \ud544\uc694\ub85c \ud560 \uac83\uc774\ub2e4. \ud558\uc9c0\ub9cc, \uc5ec\uae30\uc5d0 \uc775\uc219\ud574\uc9c0\uae30\ub9cc \ud55c\ub2e4\uba74 \uc774\ub860\uc801\uc73c\ub85c\ub294 \uc5b4\ub5a4 \uc0ac\ub0e5\ud130\uc5d0\uc11c\ub4e0 \ud63c\uc790\uc11c \uc0ac\ub0e5\uc744 \ud560 \uc218 \uc788\ub2e4. \ub530\ub77c\uc11c, \uc774\ub7ec\ud55c \uc804\uc220\uc5d0 \uc775\uc219\ud55c \ub9c8\ub3c4\uc0ac\ub9cc\uc774 \uc9c4\uc9dc \ub9c8\ub3c4\uc0ac\ub77c\uace0 \ud560 \uc218 \uc788\uc744 \uac83\uc774\ub2e4. \ub808\ubca8\uc5c5\uacfc \ud568\uaed8 \ubd80\uc5ec\ub418\ub294 \uc2a4\ud0ac \ud3ec\uc778\ud2b8 \ub294 \ud55c\ubc88 \ubc30\ubd84\ud558\uba74 \ub418\ub3cc\ub824 \ubc1b\uc744 \uae38\uc774 \uc5c6\ub2e4. \uadf8\ub807\uae30 \ub54c\ubb38\uc5d0 \uc2a4\ud0ac \ud3ec\uc778\ud2b8\ub97c \uc0ac\uc6a9\ud558\uc9c0 \uc54a\uace0 \uc544\uaef4\ub9cc \ub450\ub294 \uacbd\uc6b0\uac00 \ub9ce\uc740\ub370, \ubaa8\ub4e0 \uacf5\uaca9\uc744 \ub9c8\ubc95 \uc2a4\ud0ac\uc5d0 \uc758\uc874\ud558\ub294 \ub9c8\ub3c4\uc0ac\ub4e4\uc5d0\uac8c \uc788\uc5b4\uc11c \uadf8\ub807\uac8c \uc2a4\ud0ac \ud3ec\uc778\ud2b8\ub97c \uc544\ub07c\uae30\ub9cc \ud558\ub294 \uac83\uc740 \uc790\uc2e0\uc758 \uacf5\uaca9\ub825\uc744 \uc81c\ud55c\ud558\ub294 \ud589\ub3d9\uc774 \ub418\uae30\ub3c4 \ud55c\ub2e4. \uc801\ub2f9\ud55c \uc2dc\uc810\uc5d0, \uc801\ub2f9\ud55c \ub9c8\ubc95\uc5d0, \uc801\ub2f9\ud55c \uc2a4\ud0ac \ud3ec\uc778\ud2b8\ub97c \ubd84\ubc30\ud558\ub294 \uac83\uc774 \uac00\uc7a5 \ud604\uba85\ud558\uac8c \uac8c\uc784\uc744 \uc774\ub04c\uc5b4\ub098\uac00\ub294 \ubc29\ubc95\uc774 \ub420 \uac83\uc774\ub2e4.", "sentence_answer": "\ub808\ubca8\uc5c5\uacfc \ud568\uaed8 \ubd80\uc5ec\ub418\ub294 \uc2a4\ud0ac \ud3ec\uc778\ud2b8 \ub294 \ud55c\ubc88 \ubc30\ubd84\ud558\uba74 \ub418\ub3cc\ub824 \ubc1b\uc744 \uae38\uc774 \uc5c6\ub2e4.", "paragraph_id": "6520313-2-1", "paragraph_question": "question: \ub808\ubc8c\uc5c5\uacfc \ud568\uaed8 \ubd80\uc5ec\ub418\uba70 \ud55c\ubc88 \ubc30\ubd84\ud558\uba74 \ub418\ub3cc\ub824 \ubc1b\uc744 \uc218 \uc5c6\ub294 \uac83\uc740?, context: 2. \uc77c\uaca9\uc774\ud0c8 \uc804\uc220: \ub04a\uc784\uc5c6\uc774 \uc774\ub3d9\ud558\uba74\uc11c, \uc801\uacfc\uc758 \uac70\ub9ac\ub97c \uc720\uc9c0\ud558\uba70 \uacf5\uaca9\ud558\ub294 \uc804\ubc95. \uc801\uc5d0\uac8c \uacf5\uaca9\uc744 \uac00\ud55c \ud6c4, \uc801\uacfc\uc758 \uac70\ub9ac\ub97c \ud56d\uc2dc \uc8fc\uc758\ud558\uba74\uc11c \uac70\ub9ac\uac00 \uac00\uae4c\uc6cc\uc9c0\uba74 \uc801\uc5d0\uac8c \uacf5\uaca9\uc744 \ub2f9\ud558\uae30 \uc804\uc5d0 \ubbf8\ub9ac \uc774\ub3d9\ud558\uc5ec \ub2e4\uc74c \ubc88 \uacf5\uaca9 \ud3ec\uc9c0\uc158\uc744 \ud0d0\uc0c9\ud558\ub294 \uc804\ubc95\uc774\ub2e4. \ud56d\uc0c1 \uc218 \ucd08 \ud6c4\uc758 \ubaac\uc2a4\ud130 \uc704\uce58\ub97c \uc608\uc0c1\ud558\uba74\uc11c \uce90\ub9ad\ud130\ub97c \ucee8\ud2b8\ub864\ud558\ub294 \uac83\uc740 \uc0c1\ub2f9\ud55c \uc219\ub828\uc744 \ud544\uc694\ub85c \ud560 \uac83\uc774\ub2e4. \ud558\uc9c0\ub9cc, \uc5ec\uae30\uc5d0 \uc775\uc219\ud574\uc9c0\uae30\ub9cc \ud55c\ub2e4\uba74 \uc774\ub860\uc801\uc73c\ub85c\ub294 \uc5b4\ub5a4 \uc0ac\ub0e5\ud130\uc5d0\uc11c\ub4e0 \ud63c\uc790\uc11c \uc0ac\ub0e5\uc744 \ud560 \uc218 \uc788\ub2e4. \ub530\ub77c\uc11c, \uc774\ub7ec\ud55c \uc804\uc220\uc5d0 \uc775\uc219\ud55c \ub9c8\ub3c4\uc0ac\ub9cc\uc774 \uc9c4\uc9dc \ub9c8\ub3c4\uc0ac\ub77c\uace0 \ud560 \uc218 \uc788\uc744 \uac83\uc774\ub2e4. \ub808\ubca8\uc5c5\uacfc \ud568\uaed8 \ubd80\uc5ec\ub418\ub294 \uc2a4\ud0ac \ud3ec\uc778\ud2b8\ub294 \ud55c\ubc88 \ubc30\ubd84\ud558\uba74 \ub418\ub3cc\ub824 \ubc1b\uc744 \uae38\uc774 \uc5c6\ub2e4. \uadf8\ub807\uae30 \ub54c\ubb38\uc5d0 \uc2a4\ud0ac \ud3ec\uc778\ud2b8\ub97c \uc0ac\uc6a9\ud558\uc9c0 \uc54a\uace0 \uc544\uaef4\ub9cc \ub450\ub294 \uacbd\uc6b0\uac00 \ub9ce\uc740\ub370, \ubaa8\ub4e0 \uacf5\uaca9\uc744 \ub9c8\ubc95 \uc2a4\ud0ac\uc5d0 \uc758\uc874\ud558\ub294 \ub9c8\ub3c4\uc0ac\ub4e4\uc5d0\uac8c \uc788\uc5b4\uc11c \uadf8\ub807\uac8c \uc2a4\ud0ac \ud3ec\uc778\ud2b8\ub97c \uc544\ub07c\uae30\ub9cc \ud558\ub294 \uac83\uc740 \uc790\uc2e0\uc758 \uacf5\uaca9\ub825\uc744 \uc81c\ud55c\ud558\ub294 \ud589\ub3d9\uc774 \ub418\uae30\ub3c4 \ud55c\ub2e4. \uc801\ub2f9\ud55c \uc2dc\uc810\uc5d0, \uc801\ub2f9\ud55c \ub9c8\ubc95\uc5d0, \uc801\ub2f9\ud55c \uc2a4\ud0ac \ud3ec\uc778\ud2b8\ub97c \ubd84\ubc30\ud558\ub294 \uac83\uc774 \uac00\uc7a5 \ud604\uba85\ud558\uac8c \uac8c\uc784\uc744 \uc774\ub04c\uc5b4\ub098\uac00\ub294 \ubc29\ubc95\uc774 \ub420 \uac83\uc774\ub2e4."} {"question": "\ud55c\ud6a8\uc8fc\ub294 \uc0dd\uc560 \uccab \ub300\uc0c1\uc744 \uc5b4\ub290 \ubc29\uc1a1\uad6d\uc5d0\uc11c \uc218\uc0c1 \ubc1b\uc558\ub294\uac00?", "paragraph": "2010\ub144, \ud55c\ud6a8\uc8fc\ub294 \uc0ac\uadf9\uc758 \uac70\uc7a5 \uc774\ubcd1\ud6c8 PD\uc758 \ub300\ud558 \uc0ac\uadf9 \u300a\ub3d9\uc774\u300b\uc5d0\uc11c \ud0c0\uc774\ud2c0\ub864 \ub3d9\uc774 \uc5ed\uc744 \ub9e1\uc544 \ucc9c\ubbfc\uc758 \ub538\ub85c \ud0dc\uc5b4\ub098 \uc655\uc758 \uc5b4\uba38\ub2c8\ub85c \uc131\uc7a5\ud55c \uc5ec\uc778 \ub3d9\uc774\uc758 \ud30c\ub780\ub9cc\uc7a5\ud55c \uc77c\ub300\uae30\ub97c \uc5f0\uae30\ud588\ub2e4. \uadf9 \ucd08\ubc18\uc5d0 \ub300\uc0ac \ucc98\ub9ac\uc640 \ud638\ud761\uc774 \uc0ac\uadf9\uacfc \uc5b4\uc6b8\ub9ac\uc9c0 \uc54a\ub294\ub2e4\ub294 \uc5f0\uae30\uc5d0 \ub300\ud55c \uc9c0\uc801\uc740 \ud6c4\ubc18\ubd80\ub85c \uac08\uc218\ub85d \ud638\ud3c9\uc73c\ub85c \ubc14\ub00c\uc5c8\uace0, 60\ubd80 \uc0ac\uadf9\uc758 \uc6d0\ud1b1 \uc8fc\uc5f0\uc73c\ub85c\uc11c\uc758 \uc874\uc7ac\uac10\uc744 \ub4dc\ub7ec\ub0b4\uba70, \ub2e4\uc591\ud55c \uce90\ub9ad\ud130 \uc5f0\uae30\uc758 \uac00\ub2a5\uc131\uc744 \uc785\uc99d\ud588\ub2e4. \uc774 \uc791\ud488\uc740 20% \ud6c4\ubc18\ub300\uc758 \ub192\uc740 \uc2dc\uccad\ub960\uc744 \uae30\ub85d\ud558\uba70, \uc81c47\ud68c \ubc31\uc0c1\uc608\uc220\ub300\uc0c1 TV\ubd80\ubb38 \uc5ec\uc790\ucd5c\uc6b0\uc218\uc5f0\uae30\uc0c1\uacfc \uc81c4\ud68c \ucf54\ub9ac\uc544\ub4dc\ub77c\ub9c8\uc5b4\uc6cc\uc988 \uc5ec\uc790\uc8fc\uc5f0\uc0c1, MBC \uc5f0\uae30\ub300\uc0c1\uc5d0\uc11c \uc5ec\uc790\uc778\uae30\uc0c1\uacfc \u2018\uc5ed\uc804\uc758 \uc5ec\uc655\u2019\uc758 \uae40\ub0a8\uc8fc\uc640 \uacf5\ub3d9 \ub300\uc0c1\uc744 \uc218\uc0c1\ud558\uba70 \uc0dd\uc560 \uccab \ub300\uc0c1\uc744 \uc218\uc0c1\ud558\ub294 \ucf8c\uac70\ub97c \uc774\ub918\ub2e4.", "answer": "MBC", "sentence": "\uc774 \uc791\ud488\uc740 20% \ud6c4\ubc18\ub300\uc758 \ub192\uc740 \uc2dc\uccad\ub960\uc744 \uae30\ub85d\ud558\uba70, \uc81c47\ud68c \ubc31\uc0c1\uc608\uc220\ub300\uc0c1 TV\ubd80\ubb38 \uc5ec\uc790\ucd5c\uc6b0\uc218\uc5f0\uae30\uc0c1\uacfc \uc81c4\ud68c \ucf54\ub9ac\uc544\ub4dc\ub77c\ub9c8\uc5b4\uc6cc\uc988 \uc5ec\uc790\uc8fc\uc5f0\uc0c1, MBC \uc5f0\uae30\ub300\uc0c1\uc5d0\uc11c \uc5ec\uc790\uc778\uae30\uc0c1\uacfc \u2018\uc5ed\uc804\uc758 \uc5ec\uc655\u2019\uc758 \uae40\ub0a8\uc8fc\uc640 \uacf5\ub3d9 \ub300\uc0c1\uc744 \uc218\uc0c1\ud558\uba70 \uc0dd\uc560 \uccab \ub300\uc0c1\uc744 \uc218\uc0c1\ud558\ub294 \ucf8c\uac70\ub97c \uc774\ub918\ub2e4.", "paragraph_sentence": "2010\ub144, \ud55c\ud6a8\uc8fc\ub294 \uc0ac\uadf9\uc758 \uac70\uc7a5 \uc774\ubcd1\ud6c8 PD\uc758 \ub300\ud558 \uc0ac\uadf9 \u300a\ub3d9\uc774\u300b\uc5d0\uc11c \ud0c0\uc774\ud2c0\ub864 \ub3d9\uc774 \uc5ed\uc744 \ub9e1\uc544 \ucc9c\ubbfc\uc758 \ub538\ub85c \ud0dc\uc5b4\ub098 \uc655\uc758 \uc5b4\uba38\ub2c8\ub85c \uc131\uc7a5\ud55c \uc5ec\uc778 \ub3d9\uc774\uc758 \ud30c\ub780\ub9cc\uc7a5\ud55c \uc77c\ub300\uae30\ub97c \uc5f0\uae30\ud588\ub2e4. \uadf9 \ucd08\ubc18\uc5d0 \ub300\uc0ac \ucc98\ub9ac\uc640 \ud638\ud761\uc774 \uc0ac\uadf9\uacfc \uc5b4\uc6b8\ub9ac\uc9c0 \uc54a\ub294\ub2e4\ub294 \uc5f0\uae30\uc5d0 \ub300\ud55c \uc9c0\uc801\uc740 \ud6c4\ubc18\ubd80\ub85c \uac08\uc218\ub85d \ud638\ud3c9\uc73c\ub85c \ubc14\ub00c\uc5c8\uace0, 60\ubd80 \uc0ac\uadf9\uc758 \uc6d0\ud1b1 \uc8fc\uc5f0\uc73c\ub85c\uc11c\uc758 \uc874\uc7ac\uac10\uc744 \ub4dc\ub7ec\ub0b4\uba70, \ub2e4\uc591\ud55c \uce90\ub9ad\ud130 \uc5f0\uae30\uc758 \uac00\ub2a5\uc131\uc744 \uc785\uc99d\ud588\ub2e4. \uc774 \uc791\ud488\uc740 20% \ud6c4\ubc18\ub300\uc758 \ub192\uc740 \uc2dc\uccad\ub960\uc744 \uae30\ub85d\ud558\uba70, \uc81c47\ud68c \ubc31\uc0c1\uc608\uc220\ub300\uc0c1 TV\ubd80\ubb38 \uc5ec\uc790\ucd5c\uc6b0\uc218\uc5f0\uae30\uc0c1\uacfc \uc81c4\ud68c \ucf54\ub9ac\uc544\ub4dc\ub77c\ub9c8\uc5b4\uc6cc\uc988 \uc5ec\uc790\uc8fc\uc5f0\uc0c1, MBC \uc5f0\uae30\ub300\uc0c1\uc5d0\uc11c \uc5ec\uc790\uc778\uae30\uc0c1\uacfc \u2018\uc5ed\uc804\uc758 \uc5ec\uc655\u2019\uc758 \uae40\ub0a8\uc8fc\uc640 \uacf5\ub3d9 \ub300\uc0c1\uc744 \uc218\uc0c1\ud558\uba70 \uc0dd\uc560 \uccab \ub300\uc0c1\uc744 \uc218\uc0c1\ud558\ub294 \ucf8c\uac70\ub97c \uc774\ub918\ub2e4. ", "paragraph_answer": "2010\ub144, \ud55c\ud6a8\uc8fc\ub294 \uc0ac\uadf9\uc758 \uac70\uc7a5 \uc774\ubcd1\ud6c8 PD\uc758 \ub300\ud558 \uc0ac\uadf9 \u300a\ub3d9\uc774\u300b\uc5d0\uc11c \ud0c0\uc774\ud2c0\ub864 \ub3d9\uc774 \uc5ed\uc744 \ub9e1\uc544 \ucc9c\ubbfc\uc758 \ub538\ub85c \ud0dc\uc5b4\ub098 \uc655\uc758 \uc5b4\uba38\ub2c8\ub85c \uc131\uc7a5\ud55c \uc5ec\uc778 \ub3d9\uc774\uc758 \ud30c\ub780\ub9cc\uc7a5\ud55c \uc77c\ub300\uae30\ub97c \uc5f0\uae30\ud588\ub2e4. \uadf9 \ucd08\ubc18\uc5d0 \ub300\uc0ac \ucc98\ub9ac\uc640 \ud638\ud761\uc774 \uc0ac\uadf9\uacfc \uc5b4\uc6b8\ub9ac\uc9c0 \uc54a\ub294\ub2e4\ub294 \uc5f0\uae30\uc5d0 \ub300\ud55c \uc9c0\uc801\uc740 \ud6c4\ubc18\ubd80\ub85c \uac08\uc218\ub85d \ud638\ud3c9\uc73c\ub85c \ubc14\ub00c\uc5c8\uace0, 60\ubd80 \uc0ac\uadf9\uc758 \uc6d0\ud1b1 \uc8fc\uc5f0\uc73c\ub85c\uc11c\uc758 \uc874\uc7ac\uac10\uc744 \ub4dc\ub7ec\ub0b4\uba70, \ub2e4\uc591\ud55c \uce90\ub9ad\ud130 \uc5f0\uae30\uc758 \uac00\ub2a5\uc131\uc744 \uc785\uc99d\ud588\ub2e4. \uc774 \uc791\ud488\uc740 20% \ud6c4\ubc18\ub300\uc758 \ub192\uc740 \uc2dc\uccad\ub960\uc744 \uae30\ub85d\ud558\uba70, \uc81c47\ud68c \ubc31\uc0c1\uc608\uc220\ub300\uc0c1 TV\ubd80\ubb38 \uc5ec\uc790\ucd5c\uc6b0\uc218\uc5f0\uae30\uc0c1\uacfc \uc81c4\ud68c \ucf54\ub9ac\uc544\ub4dc\ub77c\ub9c8\uc5b4\uc6cc\uc988 \uc5ec\uc790\uc8fc\uc5f0\uc0c1, MBC \uc5f0\uae30\ub300\uc0c1\uc5d0\uc11c \uc5ec\uc790\uc778\uae30\uc0c1\uacfc \u2018\uc5ed\uc804\uc758 \uc5ec\uc655\u2019\uc758 \uae40\ub0a8\uc8fc\uc640 \uacf5\ub3d9 \ub300\uc0c1\uc744 \uc218\uc0c1\ud558\uba70 \uc0dd\uc560 \uccab \ub300\uc0c1\uc744 \uc218\uc0c1\ud558\ub294 \ucf8c\uac70\ub97c \uc774\ub918\ub2e4.", "sentence_answer": "\uc774 \uc791\ud488\uc740 20% \ud6c4\ubc18\ub300\uc758 \ub192\uc740 \uc2dc\uccad\ub960\uc744 \uae30\ub85d\ud558\uba70, \uc81c47\ud68c \ubc31\uc0c1\uc608\uc220\ub300\uc0c1 TV\ubd80\ubb38 \uc5ec\uc790\ucd5c\uc6b0\uc218\uc5f0\uae30\uc0c1\uacfc \uc81c4\ud68c \ucf54\ub9ac\uc544\ub4dc\ub77c\ub9c8\uc5b4\uc6cc\uc988 \uc5ec\uc790\uc8fc\uc5f0\uc0c1, MBC \uc5f0\uae30\ub300\uc0c1\uc5d0\uc11c \uc5ec\uc790\uc778\uae30\uc0c1\uacfc \u2018\uc5ed\uc804\uc758 \uc5ec\uc655\u2019\uc758 \uae40\ub0a8\uc8fc\uc640 \uacf5\ub3d9 \ub300\uc0c1\uc744 \uc218\uc0c1\ud558\uba70 \uc0dd\uc560 \uccab \ub300\uc0c1\uc744 \uc218\uc0c1\ud558\ub294 \ucf8c\uac70\ub97c \uc774\ub918\ub2e4.", "paragraph_id": "5940051-5-1", "paragraph_question": "question: \ud55c\ud6a8\uc8fc\ub294 \uc0dd\uc560 \uccab \ub300\uc0c1\uc744 \uc5b4\ub290 \ubc29\uc1a1\uad6d\uc5d0\uc11c \uc218\uc0c1 \ubc1b\uc558\ub294\uac00?, context: 2010\ub144, \ud55c\ud6a8\uc8fc\ub294 \uc0ac\uadf9\uc758 \uac70\uc7a5 \uc774\ubcd1\ud6c8 PD\uc758 \ub300\ud558 \uc0ac\uadf9 \u300a\ub3d9\uc774\u300b\uc5d0\uc11c \ud0c0\uc774\ud2c0\ub864 \ub3d9\uc774 \uc5ed\uc744 \ub9e1\uc544 \ucc9c\ubbfc\uc758 \ub538\ub85c \ud0dc\uc5b4\ub098 \uc655\uc758 \uc5b4\uba38\ub2c8\ub85c \uc131\uc7a5\ud55c \uc5ec\uc778 \ub3d9\uc774\uc758 \ud30c\ub780\ub9cc\uc7a5\ud55c \uc77c\ub300\uae30\ub97c \uc5f0\uae30\ud588\ub2e4. \uadf9 \ucd08\ubc18\uc5d0 \ub300\uc0ac \ucc98\ub9ac\uc640 \ud638\ud761\uc774 \uc0ac\uadf9\uacfc \uc5b4\uc6b8\ub9ac\uc9c0 \uc54a\ub294\ub2e4\ub294 \uc5f0\uae30\uc5d0 \ub300\ud55c \uc9c0\uc801\uc740 \ud6c4\ubc18\ubd80\ub85c \uac08\uc218\ub85d \ud638\ud3c9\uc73c\ub85c \ubc14\ub00c\uc5c8\uace0, 60\ubd80 \uc0ac\uadf9\uc758 \uc6d0\ud1b1 \uc8fc\uc5f0\uc73c\ub85c\uc11c\uc758 \uc874\uc7ac\uac10\uc744 \ub4dc\ub7ec\ub0b4\uba70, \ub2e4\uc591\ud55c \uce90\ub9ad\ud130 \uc5f0\uae30\uc758 \uac00\ub2a5\uc131\uc744 \uc785\uc99d\ud588\ub2e4. \uc774 \uc791\ud488\uc740 20% \ud6c4\ubc18\ub300\uc758 \ub192\uc740 \uc2dc\uccad\ub960\uc744 \uae30\ub85d\ud558\uba70, \uc81c47\ud68c \ubc31\uc0c1\uc608\uc220\ub300\uc0c1 TV\ubd80\ubb38 \uc5ec\uc790\ucd5c\uc6b0\uc218\uc5f0\uae30\uc0c1\uacfc \uc81c4\ud68c \ucf54\ub9ac\uc544\ub4dc\ub77c\ub9c8\uc5b4\uc6cc\uc988 \uc5ec\uc790\uc8fc\uc5f0\uc0c1, MBC \uc5f0\uae30\ub300\uc0c1\uc5d0\uc11c \uc5ec\uc790\uc778\uae30\uc0c1\uacfc \u2018\uc5ed\uc804\uc758 \uc5ec\uc655\u2019\uc758 \uae40\ub0a8\uc8fc\uc640 \uacf5\ub3d9 \ub300\uc0c1\uc744 \uc218\uc0c1\ud558\uba70 \uc0dd\uc560 \uccab \ub300\uc0c1\uc744 \uc218\uc0c1\ud558\ub294 \ucf8c\uac70\ub97c \uc774\ub918\ub2e4."} {"question": "\uc815\ubcf4 \uc720\ucd9c\uc5d0 \uad00\ud55c \ubb38\uc11c\uc758 \ubc94\uc778\uc73c\ub85c \ub204\uac00 \uc9c0\uba85\ub418\uc5c8\ub294\uac00?", "paragraph": "19\uc138\uae30, \ud504\ub791\uc2a4\ub294 \ub3c5\uc77c\uc744 \uc0c1\ub300\ub85c \ud55c \ubc29\uc5b4 \uc804\uc7c1\uc5d0\uc11c \ucc98\ucc38\ud55c \ud328\ubc30\ub97c \ub2f9\ud558\uace0, \ub3c5\uc77c \uc790\uc2e0\uc774 \uc6d0\ud558\ub294 \ubc30\uc0c1\uae08\uc744 \uc8fc\uba74 \ud30c\ub9ac\uc5d0\uc11c \ubb3c\ub7ec\ub098\uaca0\ub2e4\uace0 \ud558\uc790, \uc628 \ub098\ub77c \uad6d\ubbfc\uc774 \ub3c8\uc744 \ubaa8\uc544 \uac1a\uc73c\uba74\uc11c \uaca8\uc6b0 \ud480\ub824\ub0a0\uc218 \uc788\uc5c8\ub2e4. \uadf8\ub7ec\ub098 \uc774 \uc0ac\uac74\uc73c\ub85c \uac08\uc218\ub85d \ubc18 \ub3c5\uc77c \uac10\uc815\uc774 \ub192\uc544\uc84c\uace0, \ubc18\uc720\ub300\uc8fc\uc758\ub3c4 \uc628 \uc720\ub7fd\uc744 \ud729\uc4f8\uace0 \uc788\uc5c8\ub2e4. \uadf8\ub7f0 \uc0c1\ud669\uc5d0\uc11c \uc5b4\ub290\ub0a0, \ud55c \uccad\uc18c\ubd80\uac00 \ud55c \ubb38\uc11c\ub97c \ubc1c\uacac \ud558\uac8c \ub418\ub294\ub370, \uadf8 \ubb38\uc11c\uac00 \uc815\ubcf4 \uc720\ucd9c\uc5d0 \uad00\ud55c \ubb38\uc11c\uc600\ub2e4. \uadf8\ub7ec\uc790 \uc0ac\ub78c\ub4e4\uc740 \uc720\ub300\uc778 \uc54c\ud504\ub808\ub4dc \ub4dc\ub808\ud4cc\uc2a4\ub97c \ubc94\uc778\uc73c\ub85c \uc9c0\ubaa9\ud55c\ub2e4. \uadf8\ub294 \uc720\ub300\uc778\uc774\uace0, \ub610\ud55c \uadf8 \ub2f9\uc2dc \ud504\ub791\uc2a4 \uc0ac\ub78c\ub4e4\uc758 \ubc18 \ub3c5\uc77c \uac10\uc815\uc758 \ub300\uc0c1\uc774\uc600\ub358 \ub3c5\uc77c\uacc4\uc758 \uc0ac\ub78c\uc774\uc600\ub358 \uac83\uc774\ub2e4.\uadf8\ub7ec\uc790 \uadf8\ub294 \uc21c\uc2dd\uac04\uc5d0 \uc5b5\uc6b8\ud55c \ub204\uba85\uc744 \uc4f0\uace0 \uc7ac\ud310\uc5d0\ub098\uc130\uace0, \uc2ec\uc9c0\uc5b4\ub294 \uadf8\uc5d0\uac8c\ub294 \ubcc0\ud638\uc778\ub3c4 \ud5c8\uc6a9\ub418\uc9c0 \uc54a\uc558\ub2e4. \ud504\ub791\uc2a4 \uc721\uad70\uc758 \ud3ec\ubcd1 \ub300\uc704\uc600\ub358 \uc720\ub300\uc778 \uc54c\ud504\ub808\ub4dc \ub4dc\ub808\ud4cc\uc2a4\ub294 1894\ub144 \uc18c\ub839\uc778 \ud398\ub974\ub514\ub0ad \uc5d0\uc2a4\ud14c\ub77c\uc9c0(\ud504\ub791\uc2a4\uc5b4: Ferdinand Walsin Esterhazy)\ub77c\ub294 \uac04\ucca9\uc774 \uc4f4 \ubb38\uac74\uc73c\ub85c \uc778\ud558\uc5ec \ubc18\uc5ed\uc8c4\ub85c \uc720\uc8c4 \ud310\uacb0\uc744 \ubc1b\uc558\ub2e4. \uc885\uc2e0\ud615\uacfc \uce58\uc695\uc801\uc778 \uad70\uc801 \ubc15\ud0c8\uc2dd\uc744 \uc120\uace0 \ubc1b\uc558\ub2e4. \uadf8\uc758 \uad70\uc801 \ubc15\ud0c8\uc2dd\uc7a5\uc5d0\uc11c \ub05d\uae4c\uc9c0 \uc790\uc2e0\uc758 \ubb34\uc8c4\ub97c \uc678\ucce4\uc9c0\ub9cc \"\uc720\ud0dc\uc778\uc744 \uc8fd\uc5ec\ub77c\"\ub77c\ub294 \uad70\uc911\ub4e4\uc758 \uc18c\ub9ac\uc5d0 \ubb3b\ud788\uace0 \ub9d0\uc558\ub2e4. \uadf8\ub294 \uac15\uc81c\ub85c \ubd88\uba85\uc608 \uc804\uc5ed\ub41c \ub4a4, \ud504\ub791\uc2a4\ub839 \uae30\uc544\ub098\uc758 \uc545\ub9c8\uc12c\uc73c\ub85c \uc720\ubc30\ub2f9\ud55c\ub2e4.(\uc0ac\uc2e4 \uc7ac\ud310 \uacfc\uc815\uc5d0\uc11c \uc11c\uba85\uacfc \uc790\uc2e0\uc758 \uae00\uc528\uccb4\uac00 \ub2e4\ub974\ub2e4\ub294 \uc774\uc758\uac00 \uc788\uc5c8\uc73c\ub098, \ud53c\uace0\uac00 \uae00\uc528\uccb4\ub97c \ubc14\uafb8\uc5b4 \uc37c\ub2e4\ub294 \ub9d0\ub3c4 \uc548\ub418\ub294 \uadfc\uac70\ub97c \ub0b4\uc138\uc6cc \uadf8\uc758 \uc774\uc758\ub294 \ubb35\uc0b4\ub2f9\ud558\uc600\ub2e4. \uc2e4\uc81c \uc790\uc2e0\uc758 \uae00\uc528\uccb4\ub97c \ubc14\uafb8\uc5b4 \uc4f0\uae30\ub294 \ub9e4\uc6b0 \ud798\ub4e0\uc77c\uc774\ub2e4. \uadf8\ub807\uac8c \ub4dc\ub808\ud4cc\uc2a4\ub294 \uc545\ub9c8\uc12c\uc5d0\uc11c \uc720\ubc30\ub97c \ub2f9\ud558\uc600\uace0, \uadf8\uc0ac\uc774 \ud504\ub791\uc2a4 \ubcf8\ud1a0\uc5d0\uc11c\ub294 \uc9c4\ubc94\uc778 \uc5d0\uc2a4\ud0c0\ub77c\uc9c0\uac00 \ubc1c\uac01\ub418\uc5b4 \ub17c\ub780\uc744 \uacaa\ub2e4\uac00, \uacb0\uad6d \ubb34\uc8c4\ud310\uacb0\uc744 \ubc1b\uace0 \ud480\ub824\ub0ac\ub2e4.", "answer": "\uc54c\ud504\ub808\ub4dc \ub4dc\ub808\ud4cc\uc2a4", "sentence": "\uadf8\ub7ec\uc790 \uc0ac\ub78c\ub4e4\uc740 \uc720\ub300\uc778 \uc54c\ud504\ub808\ub4dc \ub4dc\ub808\ud4cc\uc2a4 \ub97c \ubc94\uc778\uc73c\ub85c \uc9c0\ubaa9\ud55c\ub2e4.", "paragraph_sentence": "19\uc138\uae30, \ud504\ub791\uc2a4\ub294 \ub3c5\uc77c\uc744 \uc0c1\ub300\ub85c \ud55c \ubc29\uc5b4 \uc804\uc7c1\uc5d0\uc11c \ucc98\ucc38\ud55c \ud328\ubc30\ub97c \ub2f9\ud558\uace0, \ub3c5\uc77c \uc790\uc2e0\uc774 \uc6d0\ud558\ub294 \ubc30\uc0c1\uae08\uc744 \uc8fc\uba74 \ud30c\ub9ac\uc5d0\uc11c \ubb3c\ub7ec\ub098\uaca0\ub2e4\uace0 \ud558\uc790, \uc628 \ub098\ub77c \uad6d\ubbfc\uc774 \ub3c8\uc744 \ubaa8\uc544 \uac1a\uc73c\uba74\uc11c \uaca8\uc6b0 \ud480\ub824\ub0a0\uc218 \uc788\uc5c8\ub2e4. \uadf8\ub7ec\ub098 \uc774 \uc0ac\uac74\uc73c\ub85c \uac08\uc218\ub85d \ubc18 \ub3c5\uc77c \uac10\uc815\uc774 \ub192\uc544\uc84c\uace0, \ubc18\uc720\ub300\uc8fc\uc758\ub3c4 \uc628 \uc720\ub7fd\uc744 \ud729\uc4f8\uace0 \uc788\uc5c8\ub2e4. \uadf8\ub7f0 \uc0c1\ud669\uc5d0\uc11c \uc5b4\ub290\ub0a0, \ud55c \uccad\uc18c\ubd80\uac00 \ud55c \ubb38\uc11c\ub97c \ubc1c\uacac \ud558\uac8c \ub418\ub294\ub370, \uadf8 \ubb38\uc11c\uac00 \uc815\ubcf4 \uc720\ucd9c\uc5d0 \uad00\ud55c \ubb38\uc11c\uc600\ub2e4. \uadf8\ub7ec\uc790 \uc0ac\ub78c\ub4e4\uc740 \uc720\ub300\uc778 \uc54c\ud504\ub808\ub4dc \ub4dc\ub808\ud4cc\uc2a4 \ub97c \ubc94\uc778\uc73c\ub85c \uc9c0\ubaa9\ud55c\ub2e4. \uadf8\ub294 \uc720\ub300\uc778\uc774\uace0, \ub610\ud55c \uadf8 \ub2f9\uc2dc \ud504\ub791\uc2a4 \uc0ac\ub78c\ub4e4\uc758 \ubc18 \ub3c5\uc77c \uac10\uc815\uc758 \ub300\uc0c1\uc774\uc600\ub358 \ub3c5\uc77c\uacc4\uc758 \uc0ac\ub78c\uc774\uc600\ub358 \uac83\uc774\ub2e4.\uadf8\ub7ec\uc790 \uadf8\ub294 \uc21c\uc2dd\uac04\uc5d0 \uc5b5\uc6b8\ud55c \ub204\uba85\uc744 \uc4f0\uace0 \uc7ac\ud310\uc5d0\ub098\uc130\uace0, \uc2ec\uc9c0\uc5b4\ub294 \uadf8\uc5d0\uac8c\ub294 \ubcc0\ud638\uc778\ub3c4 \ud5c8\uc6a9\ub418\uc9c0 \uc54a\uc558\ub2e4. \ud504\ub791\uc2a4 \uc721\uad70\uc758 \ud3ec\ubcd1 \ub300\uc704\uc600\ub358 \uc720\ub300\uc778 \uc54c\ud504\ub808\ub4dc \ub4dc\ub808\ud4cc\uc2a4\ub294 1894\ub144 \uc18c\ub839\uc778 \ud398\ub974\ub514\ub0ad \uc5d0\uc2a4\ud14c\ub77c\uc9c0(\ud504\ub791\uc2a4\uc5b4: Ferdinand Walsin Esterhazy)\ub77c\ub294 \uac04\ucca9\uc774 \uc4f4 \ubb38\uac74\uc73c\ub85c \uc778\ud558\uc5ec \ubc18\uc5ed\uc8c4\ub85c \uc720\uc8c4 \ud310\uacb0\uc744 \ubc1b\uc558\ub2e4. \uc885\uc2e0\ud615\uacfc \uce58\uc695\uc801\uc778 \uad70\uc801 \ubc15\ud0c8\uc2dd\uc744 \uc120\uace0 \ubc1b\uc558\ub2e4. \uadf8\uc758 \uad70\uc801 \ubc15\ud0c8\uc2dd\uc7a5\uc5d0\uc11c \ub05d\uae4c\uc9c0 \uc790\uc2e0\uc758 \ubb34\uc8c4\ub97c \uc678\ucce4\uc9c0\ub9cc \"\uc720\ud0dc\uc778\uc744 \uc8fd\uc5ec\ub77c\"\ub77c\ub294 \uad70\uc911\ub4e4\uc758 \uc18c\ub9ac\uc5d0 \ubb3b\ud788\uace0 \ub9d0\uc558\ub2e4. \uadf8\ub294 \uac15\uc81c\ub85c \ubd88\uba85\uc608 \uc804\uc5ed\ub41c \ub4a4, \ud504\ub791\uc2a4\ub839 \uae30\uc544\ub098\uc758 \uc545\ub9c8\uc12c\uc73c\ub85c \uc720\ubc30\ub2f9\ud55c\ub2e4.(\uc0ac\uc2e4 \uc7ac\ud310 \uacfc\uc815\uc5d0\uc11c \uc11c\uba85\uacfc \uc790\uc2e0\uc758 \uae00\uc528\uccb4\uac00 \ub2e4\ub974\ub2e4\ub294 \uc774\uc758\uac00 \uc788\uc5c8\uc73c\ub098, \ud53c\uace0\uac00 \uae00\uc528\uccb4\ub97c \ubc14\uafb8\uc5b4 \uc37c\ub2e4\ub294 \ub9d0\ub3c4 \uc548\ub418\ub294 \uadfc\uac70\ub97c \ub0b4\uc138\uc6cc \uadf8\uc758 \uc774\uc758\ub294 \ubb35\uc0b4\ub2f9\ud558\uc600\ub2e4. \uc2e4\uc81c \uc790\uc2e0\uc758 \uae00\uc528\uccb4\ub97c \ubc14\uafb8\uc5b4 \uc4f0\uae30\ub294 \ub9e4\uc6b0 \ud798\ub4e0\uc77c\uc774\ub2e4. \uadf8\ub807\uac8c \ub4dc\ub808\ud4cc\uc2a4\ub294 \uc545\ub9c8\uc12c\uc5d0\uc11c \uc720\ubc30\ub97c \ub2f9\ud558\uc600\uace0, \uadf8\uc0ac\uc774 \ud504\ub791\uc2a4 \ubcf8\ud1a0\uc5d0\uc11c\ub294 \uc9c4\ubc94\uc778 \uc5d0\uc2a4\ud0c0\ub77c\uc9c0\uac00 \ubc1c\uac01\ub418\uc5b4 \ub17c\ub780\uc744 \uacaa\ub2e4\uac00, \uacb0\uad6d \ubb34\uc8c4\ud310\uacb0\uc744 \ubc1b\uace0 \ud480\ub824\ub0ac\ub2e4.", "paragraph_answer": "19\uc138\uae30, \ud504\ub791\uc2a4\ub294 \ub3c5\uc77c\uc744 \uc0c1\ub300\ub85c \ud55c \ubc29\uc5b4 \uc804\uc7c1\uc5d0\uc11c \ucc98\ucc38\ud55c \ud328\ubc30\ub97c \ub2f9\ud558\uace0, \ub3c5\uc77c \uc790\uc2e0\uc774 \uc6d0\ud558\ub294 \ubc30\uc0c1\uae08\uc744 \uc8fc\uba74 \ud30c\ub9ac\uc5d0\uc11c \ubb3c\ub7ec\ub098\uaca0\ub2e4\uace0 \ud558\uc790, \uc628 \ub098\ub77c \uad6d\ubbfc\uc774 \ub3c8\uc744 \ubaa8\uc544 \uac1a\uc73c\uba74\uc11c \uaca8\uc6b0 \ud480\ub824\ub0a0\uc218 \uc788\uc5c8\ub2e4. \uadf8\ub7ec\ub098 \uc774 \uc0ac\uac74\uc73c\ub85c \uac08\uc218\ub85d \ubc18 \ub3c5\uc77c \uac10\uc815\uc774 \ub192\uc544\uc84c\uace0, \ubc18\uc720\ub300\uc8fc\uc758\ub3c4 \uc628 \uc720\ub7fd\uc744 \ud729\uc4f8\uace0 \uc788\uc5c8\ub2e4. \uadf8\ub7f0 \uc0c1\ud669\uc5d0\uc11c \uc5b4\ub290\ub0a0, \ud55c \uccad\uc18c\ubd80\uac00 \ud55c \ubb38\uc11c\ub97c \ubc1c\uacac \ud558\uac8c \ub418\ub294\ub370, \uadf8 \ubb38\uc11c\uac00 \uc815\ubcf4 \uc720\ucd9c\uc5d0 \uad00\ud55c \ubb38\uc11c\uc600\ub2e4. \uadf8\ub7ec\uc790 \uc0ac\ub78c\ub4e4\uc740 \uc720\ub300\uc778 \uc54c\ud504\ub808\ub4dc \ub4dc\ub808\ud4cc\uc2a4 \ub97c \ubc94\uc778\uc73c\ub85c \uc9c0\ubaa9\ud55c\ub2e4. \uadf8\ub294 \uc720\ub300\uc778\uc774\uace0, \ub610\ud55c \uadf8 \ub2f9\uc2dc \ud504\ub791\uc2a4 \uc0ac\ub78c\ub4e4\uc758 \ubc18 \ub3c5\uc77c \uac10\uc815\uc758 \ub300\uc0c1\uc774\uc600\ub358 \ub3c5\uc77c\uacc4\uc758 \uc0ac\ub78c\uc774\uc600\ub358 \uac83\uc774\ub2e4.\uadf8\ub7ec\uc790 \uadf8\ub294 \uc21c\uc2dd\uac04\uc5d0 \uc5b5\uc6b8\ud55c \ub204\uba85\uc744 \uc4f0\uace0 \uc7ac\ud310\uc5d0\ub098\uc130\uace0, \uc2ec\uc9c0\uc5b4\ub294 \uadf8\uc5d0\uac8c\ub294 \ubcc0\ud638\uc778\ub3c4 \ud5c8\uc6a9\ub418\uc9c0 \uc54a\uc558\ub2e4. \ud504\ub791\uc2a4 \uc721\uad70\uc758 \ud3ec\ubcd1 \ub300\uc704\uc600\ub358 \uc720\ub300\uc778 \uc54c\ud504\ub808\ub4dc \ub4dc\ub808\ud4cc\uc2a4\ub294 1894\ub144 \uc18c\ub839\uc778 \ud398\ub974\ub514\ub0ad \uc5d0\uc2a4\ud14c\ub77c\uc9c0(\ud504\ub791\uc2a4\uc5b4: Ferdinand Walsin Esterhazy)\ub77c\ub294 \uac04\ucca9\uc774 \uc4f4 \ubb38\uac74\uc73c\ub85c \uc778\ud558\uc5ec \ubc18\uc5ed\uc8c4\ub85c \uc720\uc8c4 \ud310\uacb0\uc744 \ubc1b\uc558\ub2e4. \uc885\uc2e0\ud615\uacfc \uce58\uc695\uc801\uc778 \uad70\uc801 \ubc15\ud0c8\uc2dd\uc744 \uc120\uace0 \ubc1b\uc558\ub2e4. \uadf8\uc758 \uad70\uc801 \ubc15\ud0c8\uc2dd\uc7a5\uc5d0\uc11c \ub05d\uae4c\uc9c0 \uc790\uc2e0\uc758 \ubb34\uc8c4\ub97c \uc678\ucce4\uc9c0\ub9cc \"\uc720\ud0dc\uc778\uc744 \uc8fd\uc5ec\ub77c\"\ub77c\ub294 \uad70\uc911\ub4e4\uc758 \uc18c\ub9ac\uc5d0 \ubb3b\ud788\uace0 \ub9d0\uc558\ub2e4. \uadf8\ub294 \uac15\uc81c\ub85c \ubd88\uba85\uc608 \uc804\uc5ed\ub41c \ub4a4, \ud504\ub791\uc2a4\ub839 \uae30\uc544\ub098\uc758 \uc545\ub9c8\uc12c\uc73c\ub85c \uc720\ubc30\ub2f9\ud55c\ub2e4.(\uc0ac\uc2e4 \uc7ac\ud310 \uacfc\uc815\uc5d0\uc11c \uc11c\uba85\uacfc \uc790\uc2e0\uc758 \uae00\uc528\uccb4\uac00 \ub2e4\ub974\ub2e4\ub294 \uc774\uc758\uac00 \uc788\uc5c8\uc73c\ub098, \ud53c\uace0\uac00 \uae00\uc528\uccb4\ub97c \ubc14\uafb8\uc5b4 \uc37c\ub2e4\ub294 \ub9d0\ub3c4 \uc548\ub418\ub294 \uadfc\uac70\ub97c \ub0b4\uc138\uc6cc \uadf8\uc758 \uc774\uc758\ub294 \ubb35\uc0b4\ub2f9\ud558\uc600\ub2e4. \uc2e4\uc81c \uc790\uc2e0\uc758 \uae00\uc528\uccb4\ub97c \ubc14\uafb8\uc5b4 \uc4f0\uae30\ub294 \ub9e4\uc6b0 \ud798\ub4e0\uc77c\uc774\ub2e4. \uadf8\ub807\uac8c \ub4dc\ub808\ud4cc\uc2a4\ub294 \uc545\ub9c8\uc12c\uc5d0\uc11c \uc720\ubc30\ub97c \ub2f9\ud558\uc600\uace0, \uadf8\uc0ac\uc774 \ud504\ub791\uc2a4 \ubcf8\ud1a0\uc5d0\uc11c\ub294 \uc9c4\ubc94\uc778 \uc5d0\uc2a4\ud0c0\ub77c\uc9c0\uac00 \ubc1c\uac01\ub418\uc5b4 \ub17c\ub780\uc744 \uacaa\ub2e4\uac00, \uacb0\uad6d \ubb34\uc8c4\ud310\uacb0\uc744 \ubc1b\uace0 \ud480\ub824\ub0ac\ub2e4.", "sentence_answer": "\uadf8\ub7ec\uc790 \uc0ac\ub78c\ub4e4\uc740 \uc720\ub300\uc778 \uc54c\ud504\ub808\ub4dc \ub4dc\ub808\ud4cc\uc2a4 \ub97c \ubc94\uc778\uc73c\ub85c \uc9c0\ubaa9\ud55c\ub2e4.", "paragraph_id": "6579200-1-0", "paragraph_question": "question: \uc815\ubcf4 \uc720\ucd9c\uc5d0 \uad00\ud55c \ubb38\uc11c\uc758 \ubc94\uc778\uc73c\ub85c \ub204\uac00 \uc9c0\uba85\ub418\uc5c8\ub294\uac00?, context: 19\uc138\uae30, \ud504\ub791\uc2a4\ub294 \ub3c5\uc77c\uc744 \uc0c1\ub300\ub85c \ud55c \ubc29\uc5b4 \uc804\uc7c1\uc5d0\uc11c \ucc98\ucc38\ud55c \ud328\ubc30\ub97c \ub2f9\ud558\uace0, \ub3c5\uc77c \uc790\uc2e0\uc774 \uc6d0\ud558\ub294 \ubc30\uc0c1\uae08\uc744 \uc8fc\uba74 \ud30c\ub9ac\uc5d0\uc11c \ubb3c\ub7ec\ub098\uaca0\ub2e4\uace0 \ud558\uc790, \uc628 \ub098\ub77c \uad6d\ubbfc\uc774 \ub3c8\uc744 \ubaa8\uc544 \uac1a\uc73c\uba74\uc11c \uaca8\uc6b0 \ud480\ub824\ub0a0\uc218 \uc788\uc5c8\ub2e4. \uadf8\ub7ec\ub098 \uc774 \uc0ac\uac74\uc73c\ub85c \uac08\uc218\ub85d \ubc18 \ub3c5\uc77c \uac10\uc815\uc774 \ub192\uc544\uc84c\uace0, \ubc18\uc720\ub300\uc8fc\uc758\ub3c4 \uc628 \uc720\ub7fd\uc744 \ud729\uc4f8\uace0 \uc788\uc5c8\ub2e4. \uadf8\ub7f0 \uc0c1\ud669\uc5d0\uc11c \uc5b4\ub290\ub0a0, \ud55c \uccad\uc18c\ubd80\uac00 \ud55c \ubb38\uc11c\ub97c \ubc1c\uacac \ud558\uac8c \ub418\ub294\ub370, \uadf8 \ubb38\uc11c\uac00 \uc815\ubcf4 \uc720\ucd9c\uc5d0 \uad00\ud55c \ubb38\uc11c\uc600\ub2e4. \uadf8\ub7ec\uc790 \uc0ac\ub78c\ub4e4\uc740 \uc720\ub300\uc778 \uc54c\ud504\ub808\ub4dc \ub4dc\ub808\ud4cc\uc2a4\ub97c \ubc94\uc778\uc73c\ub85c \uc9c0\ubaa9\ud55c\ub2e4. \uadf8\ub294 \uc720\ub300\uc778\uc774\uace0, \ub610\ud55c \uadf8 \ub2f9\uc2dc \ud504\ub791\uc2a4 \uc0ac\ub78c\ub4e4\uc758 \ubc18 \ub3c5\uc77c \uac10\uc815\uc758 \ub300\uc0c1\uc774\uc600\ub358 \ub3c5\uc77c\uacc4\uc758 \uc0ac\ub78c\uc774\uc600\ub358 \uac83\uc774\ub2e4.\uadf8\ub7ec\uc790 \uadf8\ub294 \uc21c\uc2dd\uac04\uc5d0 \uc5b5\uc6b8\ud55c \ub204\uba85\uc744 \uc4f0\uace0 \uc7ac\ud310\uc5d0\ub098\uc130\uace0, \uc2ec\uc9c0\uc5b4\ub294 \uadf8\uc5d0\uac8c\ub294 \ubcc0\ud638\uc778\ub3c4 \ud5c8\uc6a9\ub418\uc9c0 \uc54a\uc558\ub2e4. \ud504\ub791\uc2a4 \uc721\uad70\uc758 \ud3ec\ubcd1 \ub300\uc704\uc600\ub358 \uc720\ub300\uc778 \uc54c\ud504\ub808\ub4dc \ub4dc\ub808\ud4cc\uc2a4\ub294 1894\ub144 \uc18c\ub839\uc778 \ud398\ub974\ub514\ub0ad \uc5d0\uc2a4\ud14c\ub77c\uc9c0(\ud504\ub791\uc2a4\uc5b4: Ferdinand Walsin Esterhazy)\ub77c\ub294 \uac04\ucca9\uc774 \uc4f4 \ubb38\uac74\uc73c\ub85c \uc778\ud558\uc5ec \ubc18\uc5ed\uc8c4\ub85c \uc720\uc8c4 \ud310\uacb0\uc744 \ubc1b\uc558\ub2e4. \uc885\uc2e0\ud615\uacfc \uce58\uc695\uc801\uc778 \uad70\uc801 \ubc15\ud0c8\uc2dd\uc744 \uc120\uace0 \ubc1b\uc558\ub2e4. \uadf8\uc758 \uad70\uc801 \ubc15\ud0c8\uc2dd\uc7a5\uc5d0\uc11c \ub05d\uae4c\uc9c0 \uc790\uc2e0\uc758 \ubb34\uc8c4\ub97c \uc678\ucce4\uc9c0\ub9cc \"\uc720\ud0dc\uc778\uc744 \uc8fd\uc5ec\ub77c\"\ub77c\ub294 \uad70\uc911\ub4e4\uc758 \uc18c\ub9ac\uc5d0 \ubb3b\ud788\uace0 \ub9d0\uc558\ub2e4. \uadf8\ub294 \uac15\uc81c\ub85c \ubd88\uba85\uc608 \uc804\uc5ed\ub41c \ub4a4, \ud504\ub791\uc2a4\ub839 \uae30\uc544\ub098\uc758 \uc545\ub9c8\uc12c\uc73c\ub85c \uc720\ubc30\ub2f9\ud55c\ub2e4.(\uc0ac\uc2e4 \uc7ac\ud310 \uacfc\uc815\uc5d0\uc11c \uc11c\uba85\uacfc \uc790\uc2e0\uc758 \uae00\uc528\uccb4\uac00 \ub2e4\ub974\ub2e4\ub294 \uc774\uc758\uac00 \uc788\uc5c8\uc73c\ub098, \ud53c\uace0\uac00 \uae00\uc528\uccb4\ub97c \ubc14\uafb8\uc5b4 \uc37c\ub2e4\ub294 \ub9d0\ub3c4 \uc548\ub418\ub294 \uadfc\uac70\ub97c \ub0b4\uc138\uc6cc \uadf8\uc758 \uc774\uc758\ub294 \ubb35\uc0b4\ub2f9\ud558\uc600\ub2e4. \uc2e4\uc81c \uc790\uc2e0\uc758 \uae00\uc528\uccb4\ub97c \ubc14\uafb8\uc5b4 \uc4f0\uae30\ub294 \ub9e4\uc6b0 \ud798\ub4e0\uc77c\uc774\ub2e4. \uadf8\ub807\uac8c \ub4dc\ub808\ud4cc\uc2a4\ub294 \uc545\ub9c8\uc12c\uc5d0\uc11c \uc720\ubc30\ub97c \ub2f9\ud558\uc600\uace0, \uadf8\uc0ac\uc774 \ud504\ub791\uc2a4 \ubcf8\ud1a0\uc5d0\uc11c\ub294 \uc9c4\ubc94\uc778 \uc5d0\uc2a4\ud0c0\ub77c\uc9c0\uac00 \ubc1c\uac01\ub418\uc5b4 \ub17c\ub780\uc744 \uacaa\ub2e4\uac00, \uacb0\uad6d \ubb34\uc8c4\ud310\uacb0\uc744 \ubc1b\uace0 \ud480\ub824\ub0ac\ub2e4."} {"question": "\ub9ac\ud788\ud150\uc288\ud0c0\uc778\uc758 \uc694\ud55c \uacf5\uc774 \uc774\ub044\ub294 \uc624\uc2a4\ud2b8\ub9ac\uc544 \uae30\ubcd1\ub300\ub294 \uc5b4\ub514\uc5d0 \uc704\uce58\ud588\ub294\uac00?", "paragraph": "\uc694\ud55c \ud3f0 \ud790\ub7ec(Johann von Hiller)(\u2165), \ud558\uc778\ub9ac\ud788 \uadf8\ub77c\ud504 \ud3f0 \ubca8\ub808\uac00\ub974\ub4dc(Heinrich Graf von Bellegarde)(\u2160), \uadf8\ub9ac\uace0 \ud504\ub780\uce20 \uc790\ube44\uc5d0\ub974 \ud504\ub9b0\uc2a4 \ucd94 \ud638\uc5d4\ucd10\ub808\ub978-\ud5e4\ud79d\uac90(Franz Xavier Prince zu Hohenzollern-Hechingen)(\u2161)\uc774 \uc9c0\ud718\ud558\ub294 \uc624\uc2a4\ud2b8\ub9ac\uc544\uad70\uc740 \uc544\uc2a4\ud398\ub978 \ubc29\uba74\uc5d0 \uc9d1\uacb0\ud558\uc600\ub2e4. \ud55c\ud3b8 \ud504\ub780\uce20 \ud4cc\ub974\uc2a4\ud2b8 \ud3f0 \ub85c\uc820\ubca0\ub974\ud06c-\uc624\ub974\uc2dc\ub2c8(Franz F\u00fcrst von Rosenberg-Orsini)(\u2163)\ub294 \uc5d0\uc2ac\ub9c1\uc744 \uacf5\uaca9\ud588\ub2e4. \uc911\uc559\uc5d0 \uc788\ub358 \ub9ac\ud788\ud150\uc288\ud0c0\uc778\uc758 \uc694\ud55c \uacf5(Prince Johann of Liechtenstein's)\uc758 \uc624\uc2a4\ud2b8\ub9ac\uc544 \uae30\ubcd1\ub300\ub294 \uc624\uc2a4\ud2b8\ub9ac\uc544\uad70 \uc885\ub300\uc758 \uba38\ub9ac\ubd80\ubd84\uc744 \uacf5\uaca9\ud558\ub824 \ud558\ub294 \ud504\ub791\uc2a4\uad70\uc5d0 \ub300\uc751\ud558\uc5ec \uc6c0\uc9c1\uc77c \uc900\ube44\ub97c \ud558\uc600\ub2e4. 21\uc77c \ub3d9\uc548 \ud504\ub791\uc2a4\uad70\uc774 \ub3c4\ud558\ub97c \uc900\ube44\ud558\ub294 \ub2e4\ub9ac\ub294 \ubb3c\uc0b4\uc774 \uac70\uc138\uc9d0\uc5d0 \ub530\ub77c \uac08\uc218\ub85d \uc704\ud5d8\ud55c \uc0c1\ud669\uc5d0 \ucc98\ud588\ub2e4. \uadf8\ub7ec\ub098 \ud504\ub791\uc2a4\uad70\uc740 \ud734\uc2dd\uc2dc\uac04\ub3c4 \uc5c6\uc774 \ud558\ub8e8 \uc885\uc77c \uac15\uc744 \uac74\ub11c\uc73c\uba70 \ubc24\uc5d0\ub3c4 \ub9c8\ucc2c\uac00\uc9c0\uc600\ub2e4.", "answer": "\uc911\uc559", "sentence": "\uc911\uc559 \uc5d0 \uc788\ub358 \ub9ac\ud788\ud150\uc288\ud0c0\uc778\uc758 \uc694\ud55c \uacf5(Prince Johann of Liechtenstein's)\uc758 \uc624\uc2a4\ud2b8\ub9ac\uc544 \uae30\ubcd1\ub300\ub294 \uc624\uc2a4\ud2b8\ub9ac\uc544\uad70 \uc885\ub300\uc758 \uba38\ub9ac\ubd80\ubd84\uc744 \uacf5\uaca9\ud558\ub824 \ud558\ub294 \ud504\ub791\uc2a4\uad70\uc5d0 \ub300\uc751\ud558\uc5ec \uc6c0\uc9c1\uc77c \uc900\ube44\ub97c \ud558\uc600\ub2e4.", "paragraph_sentence": "\uc694\ud55c \ud3f0 \ud790\ub7ec(Johann von Hiller)(\u2165), \ud558\uc778\ub9ac\ud788 \uadf8\ub77c\ud504 \ud3f0 \ubca8\ub808\uac00\ub974\ub4dc(Heinrich Graf von Bellegarde)(\u2160), \uadf8\ub9ac\uace0 \ud504\ub780\uce20 \uc790\ube44\uc5d0\ub974 \ud504\ub9b0\uc2a4 \ucd94 \ud638\uc5d4\ucd10\ub808\ub978-\ud5e4\ud79d\uac90(Franz Xavier Prince zu Hohenzollern-Hechingen)(\u2161)\uc774 \uc9c0\ud718\ud558\ub294 \uc624\uc2a4\ud2b8\ub9ac\uc544\uad70\uc740 \uc544\uc2a4\ud398\ub978 \ubc29\uba74\uc5d0 \uc9d1\uacb0\ud558\uc600\ub2e4. \ud55c\ud3b8 \ud504\ub780\uce20 \ud4cc\ub974\uc2a4\ud2b8 \ud3f0 \ub85c\uc820\ubca0\ub974\ud06c-\uc624\ub974\uc2dc\ub2c8(Franz F\u00fcrst von Rosenberg-Orsini)(\u2163)\ub294 \uc5d0\uc2ac\ub9c1\uc744 \uacf5\uaca9\ud588\ub2e4. \uc911\uc559 \uc5d0 \uc788\ub358 \ub9ac\ud788\ud150\uc288\ud0c0\uc778\uc758 \uc694\ud55c \uacf5(Prince Johann of Liechtenstein's)\uc758 \uc624\uc2a4\ud2b8\ub9ac\uc544 \uae30\ubcd1\ub300\ub294 \uc624\uc2a4\ud2b8\ub9ac\uc544\uad70 \uc885\ub300\uc758 \uba38\ub9ac\ubd80\ubd84\uc744 \uacf5\uaca9\ud558\ub824 \ud558\ub294 \ud504\ub791\uc2a4\uad70\uc5d0 \ub300\uc751\ud558\uc5ec \uc6c0\uc9c1\uc77c \uc900\ube44\ub97c \ud558\uc600\ub2e4. 21\uc77c \ub3d9\uc548 \ud504\ub791\uc2a4\uad70\uc774 \ub3c4\ud558\ub97c \uc900\ube44\ud558\ub294 \ub2e4\ub9ac\ub294 \ubb3c\uc0b4\uc774 \uac70\uc138\uc9d0\uc5d0 \ub530\ub77c \uac08\uc218\ub85d \uc704\ud5d8\ud55c \uc0c1\ud669\uc5d0 \ucc98\ud588\ub2e4. \uadf8\ub7ec\ub098 \ud504\ub791\uc2a4\uad70\uc740 \ud734\uc2dd\uc2dc\uac04\ub3c4 \uc5c6\uc774 \ud558\ub8e8 \uc885\uc77c \uac15\uc744 \uac74\ub11c\uc73c\uba70 \ubc24\uc5d0\ub3c4 \ub9c8\ucc2c\uac00\uc9c0\uc600\ub2e4.", "paragraph_answer": "\uc694\ud55c \ud3f0 \ud790\ub7ec(Johann von Hiller)(\u2165), \ud558\uc778\ub9ac\ud788 \uadf8\ub77c\ud504 \ud3f0 \ubca8\ub808\uac00\ub974\ub4dc(Heinrich Graf von Bellegarde)(\u2160), \uadf8\ub9ac\uace0 \ud504\ub780\uce20 \uc790\ube44\uc5d0\ub974 \ud504\ub9b0\uc2a4 \ucd94 \ud638\uc5d4\ucd10\ub808\ub978-\ud5e4\ud79d\uac90(Franz Xavier Prince zu Hohenzollern-Hechingen)(\u2161)\uc774 \uc9c0\ud718\ud558\ub294 \uc624\uc2a4\ud2b8\ub9ac\uc544\uad70\uc740 \uc544\uc2a4\ud398\ub978 \ubc29\uba74\uc5d0 \uc9d1\uacb0\ud558\uc600\ub2e4. \ud55c\ud3b8 \ud504\ub780\uce20 \ud4cc\ub974\uc2a4\ud2b8 \ud3f0 \ub85c\uc820\ubca0\ub974\ud06c-\uc624\ub974\uc2dc\ub2c8(Franz F\u00fcrst von Rosenberg-Orsini)(\u2163)\ub294 \uc5d0\uc2ac\ub9c1\uc744 \uacf5\uaca9\ud588\ub2e4. \uc911\uc559 \uc5d0 \uc788\ub358 \ub9ac\ud788\ud150\uc288\ud0c0\uc778\uc758 \uc694\ud55c \uacf5(Prince Johann of Liechtenstein's)\uc758 \uc624\uc2a4\ud2b8\ub9ac\uc544 \uae30\ubcd1\ub300\ub294 \uc624\uc2a4\ud2b8\ub9ac\uc544\uad70 \uc885\ub300\uc758 \uba38\ub9ac\ubd80\ubd84\uc744 \uacf5\uaca9\ud558\ub824 \ud558\ub294 \ud504\ub791\uc2a4\uad70\uc5d0 \ub300\uc751\ud558\uc5ec \uc6c0\uc9c1\uc77c \uc900\ube44\ub97c \ud558\uc600\ub2e4. 21\uc77c \ub3d9\uc548 \ud504\ub791\uc2a4\uad70\uc774 \ub3c4\ud558\ub97c \uc900\ube44\ud558\ub294 \ub2e4\ub9ac\ub294 \ubb3c\uc0b4\uc774 \uac70\uc138\uc9d0\uc5d0 \ub530\ub77c \uac08\uc218\ub85d \uc704\ud5d8\ud55c \uc0c1\ud669\uc5d0 \ucc98\ud588\ub2e4. \uadf8\ub7ec\ub098 \ud504\ub791\uc2a4\uad70\uc740 \ud734\uc2dd\uc2dc\uac04\ub3c4 \uc5c6\uc774 \ud558\ub8e8 \uc885\uc77c \uac15\uc744 \uac74\ub11c\uc73c\uba70 \ubc24\uc5d0\ub3c4 \ub9c8\ucc2c\uac00\uc9c0\uc600\ub2e4.", "sentence_answer": " \uc911\uc559 \uc5d0 \uc788\ub358 \ub9ac\ud788\ud150\uc288\ud0c0\uc778\uc758 \uc694\ud55c \uacf5(Prince Johann of Liechtenstein's)\uc758 \uc624\uc2a4\ud2b8\ub9ac\uc544 \uae30\ubcd1\ub300\ub294 \uc624\uc2a4\ud2b8\ub9ac\uc544\uad70 \uc885\ub300\uc758 \uba38\ub9ac\ubd80\ubd84\uc744 \uacf5\uaca9\ud558\ub824 \ud558\ub294 \ud504\ub791\uc2a4\uad70\uc5d0 \ub300\uc751\ud558\uc5ec \uc6c0\uc9c1\uc77c \uc900\ube44\ub97c \ud558\uc600\ub2e4.", "paragraph_id": "6585243-2-1", "paragraph_question": "question: \ub9ac\ud788\ud150\uc288\ud0c0\uc778\uc758 \uc694\ud55c \uacf5\uc774 \uc774\ub044\ub294 \uc624\uc2a4\ud2b8\ub9ac\uc544 \uae30\ubcd1\ub300\ub294 \uc5b4\ub514\uc5d0 \uc704\uce58\ud588\ub294\uac00?, context: \uc694\ud55c \ud3f0 \ud790\ub7ec(Johann von Hiller)(\u2165), \ud558\uc778\ub9ac\ud788 \uadf8\ub77c\ud504 \ud3f0 \ubca8\ub808\uac00\ub974\ub4dc(Heinrich Graf von Bellegarde)(\u2160), \uadf8\ub9ac\uace0 \ud504\ub780\uce20 \uc790\ube44\uc5d0\ub974 \ud504\ub9b0\uc2a4 \ucd94 \ud638\uc5d4\ucd10\ub808\ub978-\ud5e4\ud79d\uac90(Franz Xavier Prince zu Hohenzollern-Hechingen)(\u2161)\uc774 \uc9c0\ud718\ud558\ub294 \uc624\uc2a4\ud2b8\ub9ac\uc544\uad70\uc740 \uc544\uc2a4\ud398\ub978 \ubc29\uba74\uc5d0 \uc9d1\uacb0\ud558\uc600\ub2e4. \ud55c\ud3b8 \ud504\ub780\uce20 \ud4cc\ub974\uc2a4\ud2b8 \ud3f0 \ub85c\uc820\ubca0\ub974\ud06c-\uc624\ub974\uc2dc\ub2c8(Franz F\u00fcrst von Rosenberg-Orsini)(\u2163)\ub294 \uc5d0\uc2ac\ub9c1\uc744 \uacf5\uaca9\ud588\ub2e4. \uc911\uc559\uc5d0 \uc788\ub358 \ub9ac\ud788\ud150\uc288\ud0c0\uc778\uc758 \uc694\ud55c \uacf5(Prince Johann of Liechtenstein's)\uc758 \uc624\uc2a4\ud2b8\ub9ac\uc544 \uae30\ubcd1\ub300\ub294 \uc624\uc2a4\ud2b8\ub9ac\uc544\uad70 \uc885\ub300\uc758 \uba38\ub9ac\ubd80\ubd84\uc744 \uacf5\uaca9\ud558\ub824 \ud558\ub294 \ud504\ub791\uc2a4\uad70\uc5d0 \ub300\uc751\ud558\uc5ec \uc6c0\uc9c1\uc77c \uc900\ube44\ub97c \ud558\uc600\ub2e4. 21\uc77c \ub3d9\uc548 \ud504\ub791\uc2a4\uad70\uc774 \ub3c4\ud558\ub97c \uc900\ube44\ud558\ub294 \ub2e4\ub9ac\ub294 \ubb3c\uc0b4\uc774 \uac70\uc138\uc9d0\uc5d0 \ub530\ub77c \uac08\uc218\ub85d \uc704\ud5d8\ud55c \uc0c1\ud669\uc5d0 \ucc98\ud588\ub2e4. \uadf8\ub7ec\ub098 \ud504\ub791\uc2a4\uad70\uc740 \ud734\uc2dd\uc2dc\uac04\ub3c4 \uc5c6\uc774 \ud558\ub8e8 \uc885\uc77c \uac15\uc744 \uac74\ub11c\uc73c\uba70 \ubc24\uc5d0\ub3c4 \ub9c8\ucc2c\uac00\uc9c0\uc600\ub2e4."} {"question": "\ube14\ub808\uc774\ud06c \ub77c\uc774\ube14\ub9ac\uac00 \ud0c0\uc784\uc988 \ub9e4\uac70\uc9c4\uc758 \uac00\uc7a5 \uc601\ud5a5\ub825 \uc788\ub294 100\uc778\uc5d0 \uc120\uc815\ub41c \ud574\ub294?", "paragraph": "2011\ub144, \uadf8\ub140\ub294 \ud0c0\uc784\uc988 \ub9e4\uac70\uc9c4 \u2018\uac00\uc7a5 \uc601\ud5a5\ub825 \uc788\ub294 100\uc778\u2019\uc5d0 \uc120\uc815 \ub418\uc5c8\ub2e4. \ub610\ud55c AskMen.com\uc740 \uadf8\ub140\ub97c 2011\ub144 \uac00\uc7a5 \uc139\uc2dc\ud55c \uc5ec\uc131\uc73c\ub85c \uc120\uc815\ud588\uace0, \ud53c\ud50c \ub9e4\uac70\uc9c4\uc5d0\uc11c\ub294 2012\ub144 \uac00\uc7a5 \uc544\ub984\ub2e4\uc6b4 \ubaa8\ub4e0 \ub098\uc774 \uc911 \ud558\ub098\ub85c \uc120\uc815\ud588\ub2e4. 2012\ub144, \uadf8\ub140\ub294 \uc62c\ub9ac\ubc84 \uc2a4\ud1a4 \uac10\ub3c5\uc758 \u300a\ud30c\uad34\uc790\ub4e4\u300b(\uc6d0\uc81c: Savages)\uc5d0\uc11c \ud14c\uc77c\ub7ec \ud0a4\ucde8, \uc544\ub860 \uc874\uc2a8, \uc140\ub9c8 \ud5e4\uc774\uc5d1\uacfc \uc874 \ud2b8\ub77c\ubcfc\ud0c0\uc640 \uacf5\uc5f0\ud588\ub2e4. \ub2f9\ucd08 \uc624\ud544\ub9ac\uc544 \"\uc624\" \uc138\uc774\uc9c0' \uc5ed\ud560\uc5d0\ub294 \uc81c\ub2c8\ud37c \ub85c\ub80c\uc2a4\uac00 \uce90\uc2a4\ud305\ub418\uc5c8\uc9c0\ub9cc \u300a\ud5dd\uac70\uac8c\uc784: \ud310\uc5e0\uc758 \ubd88\uaf43\u300b\uc758 \uc77c\uc815 \ubb38\uc81c\ub85c \ubb34\uc0b0 \ub418\uba74\uc11c \ub77c\uc774\ube14\ub9ac\uac00 \ub300\uccb4\ud588\ub2e4. HitFix\uc758 \uc601\ud654 \ud3c9\ub860\uac00 \ub4dc\ub958 \ub9e5\uc704\ub2c8\ub294 \ub77c\uc774\ube14\ub9ac\uc758 \uc5f0\uae30\ub97c \uce6d\ucc2c\ud558\uba70, \uadf8\ub294 \u201c\uadf8\ub140\uac00 \"\uc624\"\ub97c \uac00\uc2b4 \uc544\ud504\uace0 \uc791\uc740 \uc18c\uc6b8\ub85c \uc5f0\uae30\ud588\uae30\uc5d0 \uc601\ub9ac\ud558\uace0 \ud655\uc2e4\ud788 \uc2ac\ud390\ub2e4\u201d\ub77c\uace0 \ud3c9\ud588\ub2e4. \uac19\uc740 \ud574 \uadf8\ub140\ub294 \uc0c8\ub85c\uc6b4 \uad6c\ucc0c \ud5a5\uc218\uc778 \u2018Gucci Premiere\u2019\uc758 \ubba4\uc988\ub85c \uc120\uc815 \ub418\uc5c8\uace0, \uadf8\ub140\ub294 \ud5a5\uc218\uc5d0 \ub300\ud55c \ub2c8\ucf5c\ub77c\uc2a4 \uc708\ub529 \ub808\ud508 \uac10\ub3c5\uc758 \ub2e8\ud3b8 \uc601\ud654 \uad11\uace0\uc5d0 \ucd9c\uc5f0\ud588\ub2e4.", "answer": "2011\ub144", "sentence": "2011\ub144 ,", "paragraph_sentence": " 2011\ub144 , \uadf8\ub140\ub294 \ud0c0\uc784\uc988 \ub9e4\uac70\uc9c4 \u2018\uac00\uc7a5 \uc601\ud5a5\ub825 \uc788\ub294 100\uc778\u2019\uc5d0 \uc120\uc815 \ub418\uc5c8\ub2e4. \ub610\ud55c AskMen.com\uc740 \uadf8\ub140\ub97c 2011\ub144 \uac00\uc7a5 \uc139\uc2dc\ud55c \uc5ec\uc131\uc73c\ub85c \uc120\uc815\ud588\uace0, \ud53c\ud50c \ub9e4\uac70\uc9c4\uc5d0\uc11c\ub294 2012\ub144 \uac00\uc7a5 \uc544\ub984\ub2e4\uc6b4 \ubaa8\ub4e0 \ub098\uc774 \uc911 \ud558\ub098\ub85c \uc120\uc815\ud588\ub2e4. 2012\ub144, \uadf8\ub140\ub294 \uc62c\ub9ac\ubc84 \uc2a4\ud1a4 \uac10\ub3c5\uc758 \u300a\ud30c\uad34\uc790\ub4e4\u300b(\uc6d0\uc81c: Savages)\uc5d0\uc11c \ud14c\uc77c\ub7ec \ud0a4\ucde8, \uc544\ub860 \uc874\uc2a8, \uc140\ub9c8 \ud5e4\uc774\uc5d1\uacfc \uc874 \ud2b8\ub77c\ubcfc\ud0c0\uc640 \uacf5\uc5f0\ud588\ub2e4. \ub2f9\ucd08 \uc624\ud544\ub9ac\uc544 \"\uc624\" \uc138\uc774\uc9c0' \uc5ed\ud560\uc5d0\ub294 \uc81c\ub2c8\ud37c \ub85c\ub80c\uc2a4\uac00 \uce90\uc2a4\ud305\ub418\uc5c8\uc9c0\ub9cc \u300a\ud5dd\uac70\uac8c\uc784: \ud310\uc5e0\uc758 \ubd88\uaf43\u300b\uc758 \uc77c\uc815 \ubb38\uc81c\ub85c \ubb34\uc0b0 \ub418\uba74\uc11c \ub77c\uc774\ube14\ub9ac\uac00 \ub300\uccb4\ud588\ub2e4. HitFix\uc758 \uc601\ud654 \ud3c9\ub860\uac00 \ub4dc\ub958 \ub9e5\uc704\ub2c8\ub294 \ub77c\uc774\ube14\ub9ac\uc758 \uc5f0\uae30\ub97c \uce6d\ucc2c\ud558\uba70, \uadf8\ub294 \u201c\uadf8\ub140\uac00 \"\uc624\"\ub97c \uac00\uc2b4 \uc544\ud504\uace0 \uc791\uc740 \uc18c\uc6b8\ub85c \uc5f0\uae30\ud588\uae30\uc5d0 \uc601\ub9ac\ud558\uace0 \ud655\uc2e4\ud788 \uc2ac\ud390\ub2e4\u201d\ub77c\uace0 \ud3c9\ud588\ub2e4. \uac19\uc740 \ud574 \uadf8\ub140\ub294 \uc0c8\ub85c\uc6b4 \uad6c\ucc0c \ud5a5\uc218\uc778 \u2018Gucci Premiere\u2019\uc758 \ubba4\uc988\ub85c \uc120\uc815 \ub418\uc5c8\uace0, \uadf8\ub140\ub294 \ud5a5\uc218\uc5d0 \ub300\ud55c \ub2c8\ucf5c\ub77c\uc2a4 \uc708\ub529 \ub808\ud508 \uac10\ub3c5\uc758 \ub2e8\ud3b8 \uc601\ud654 \uad11\uace0\uc5d0 \ucd9c\uc5f0\ud588\ub2e4.", "paragraph_answer": " 2011\ub144 , \uadf8\ub140\ub294 \ud0c0\uc784\uc988 \ub9e4\uac70\uc9c4 \u2018\uac00\uc7a5 \uc601\ud5a5\ub825 \uc788\ub294 100\uc778\u2019\uc5d0 \uc120\uc815 \ub418\uc5c8\ub2e4. \ub610\ud55c AskMen.com\uc740 \uadf8\ub140\ub97c 2011\ub144 \uac00\uc7a5 \uc139\uc2dc\ud55c \uc5ec\uc131\uc73c\ub85c \uc120\uc815\ud588\uace0, \ud53c\ud50c \ub9e4\uac70\uc9c4\uc5d0\uc11c\ub294 2012\ub144 \uac00\uc7a5 \uc544\ub984\ub2e4\uc6b4 \ubaa8\ub4e0 \ub098\uc774 \uc911 \ud558\ub098\ub85c \uc120\uc815\ud588\ub2e4. 2012\ub144, \uadf8\ub140\ub294 \uc62c\ub9ac\ubc84 \uc2a4\ud1a4 \uac10\ub3c5\uc758 \u300a\ud30c\uad34\uc790\ub4e4\u300b(\uc6d0\uc81c: Savages)\uc5d0\uc11c \ud14c\uc77c\ub7ec \ud0a4\ucde8, \uc544\ub860 \uc874\uc2a8, \uc140\ub9c8 \ud5e4\uc774\uc5d1\uacfc \uc874 \ud2b8\ub77c\ubcfc\ud0c0\uc640 \uacf5\uc5f0\ud588\ub2e4. \ub2f9\ucd08 \uc624\ud544\ub9ac\uc544 \"\uc624\" \uc138\uc774\uc9c0' \uc5ed\ud560\uc5d0\ub294 \uc81c\ub2c8\ud37c \ub85c\ub80c\uc2a4\uac00 \uce90\uc2a4\ud305\ub418\uc5c8\uc9c0\ub9cc \u300a\ud5dd\uac70\uac8c\uc784: \ud310\uc5e0\uc758 \ubd88\uaf43\u300b\uc758 \uc77c\uc815 \ubb38\uc81c\ub85c \ubb34\uc0b0 \ub418\uba74\uc11c \ub77c\uc774\ube14\ub9ac\uac00 \ub300\uccb4\ud588\ub2e4. HitFix\uc758 \uc601\ud654 \ud3c9\ub860\uac00 \ub4dc\ub958 \ub9e5\uc704\ub2c8\ub294 \ub77c\uc774\ube14\ub9ac\uc758 \uc5f0\uae30\ub97c \uce6d\ucc2c\ud558\uba70, \uadf8\ub294 \u201c\uadf8\ub140\uac00 \"\uc624\"\ub97c \uac00\uc2b4 \uc544\ud504\uace0 \uc791\uc740 \uc18c\uc6b8\ub85c \uc5f0\uae30\ud588\uae30\uc5d0 \uc601\ub9ac\ud558\uace0 \ud655\uc2e4\ud788 \uc2ac\ud390\ub2e4\u201d\ub77c\uace0 \ud3c9\ud588\ub2e4. \uac19\uc740 \ud574 \uadf8\ub140\ub294 \uc0c8\ub85c\uc6b4 \uad6c\ucc0c \ud5a5\uc218\uc778 \u2018Gucci Premiere\u2019\uc758 \ubba4\uc988\ub85c \uc120\uc815 \ub418\uc5c8\uace0, \uadf8\ub140\ub294 \ud5a5\uc218\uc5d0 \ub300\ud55c \ub2c8\ucf5c\ub77c\uc2a4 \uc708\ub529 \ub808\ud508 \uac10\ub3c5\uc758 \ub2e8\ud3b8 \uc601\ud654 \uad11\uace0\uc5d0 \ucd9c\uc5f0\ud588\ub2e4.", "sentence_answer": " 2011\ub144 ,", "paragraph_id": "6509731-3-2", "paragraph_question": "question: \ube14\ub808\uc774\ud06c \ub77c\uc774\ube14\ub9ac\uac00 \ud0c0\uc784\uc988 \ub9e4\uac70\uc9c4\uc758 \uac00\uc7a5 \uc601\ud5a5\ub825 \uc788\ub294 100\uc778\uc5d0 \uc120\uc815\ub41c \ud574\ub294?, context: 2011\ub144, \uadf8\ub140\ub294 \ud0c0\uc784\uc988 \ub9e4\uac70\uc9c4 \u2018\uac00\uc7a5 \uc601\ud5a5\ub825 \uc788\ub294 100\uc778\u2019\uc5d0 \uc120\uc815 \ub418\uc5c8\ub2e4. \ub610\ud55c AskMen.com\uc740 \uadf8\ub140\ub97c 2011\ub144 \uac00\uc7a5 \uc139\uc2dc\ud55c \uc5ec\uc131\uc73c\ub85c \uc120\uc815\ud588\uace0, \ud53c\ud50c \ub9e4\uac70\uc9c4\uc5d0\uc11c\ub294 2012\ub144 \uac00\uc7a5 \uc544\ub984\ub2e4\uc6b4 \ubaa8\ub4e0 \ub098\uc774 \uc911 \ud558\ub098\ub85c \uc120\uc815\ud588\ub2e4. 2012\ub144, \uadf8\ub140\ub294 \uc62c\ub9ac\ubc84 \uc2a4\ud1a4 \uac10\ub3c5\uc758 \u300a\ud30c\uad34\uc790\ub4e4\u300b(\uc6d0\uc81c: Savages)\uc5d0\uc11c \ud14c\uc77c\ub7ec \ud0a4\ucde8, \uc544\ub860 \uc874\uc2a8, \uc140\ub9c8 \ud5e4\uc774\uc5d1\uacfc \uc874 \ud2b8\ub77c\ubcfc\ud0c0\uc640 \uacf5\uc5f0\ud588\ub2e4. \ub2f9\ucd08 \uc624\ud544\ub9ac\uc544 \"\uc624\" \uc138\uc774\uc9c0' \uc5ed\ud560\uc5d0\ub294 \uc81c\ub2c8\ud37c \ub85c\ub80c\uc2a4\uac00 \uce90\uc2a4\ud305\ub418\uc5c8\uc9c0\ub9cc \u300a\ud5dd\uac70\uac8c\uc784: \ud310\uc5e0\uc758 \ubd88\uaf43\u300b\uc758 \uc77c\uc815 \ubb38\uc81c\ub85c \ubb34\uc0b0 \ub418\uba74\uc11c \ub77c\uc774\ube14\ub9ac\uac00 \ub300\uccb4\ud588\ub2e4. HitFix\uc758 \uc601\ud654 \ud3c9\ub860\uac00 \ub4dc\ub958 \ub9e5\uc704\ub2c8\ub294 \ub77c\uc774\ube14\ub9ac\uc758 \uc5f0\uae30\ub97c \uce6d\ucc2c\ud558\uba70, \uadf8\ub294 \u201c\uadf8\ub140\uac00 \"\uc624\"\ub97c \uac00\uc2b4 \uc544\ud504\uace0 \uc791\uc740 \uc18c\uc6b8\ub85c \uc5f0\uae30\ud588\uae30\uc5d0 \uc601\ub9ac\ud558\uace0 \ud655\uc2e4\ud788 \uc2ac\ud390\ub2e4\u201d\ub77c\uace0 \ud3c9\ud588\ub2e4. \uac19\uc740 \ud574 \uadf8\ub140\ub294 \uc0c8\ub85c\uc6b4 \uad6c\ucc0c \ud5a5\uc218\uc778 \u2018Gucci Premiere\u2019\uc758 \ubba4\uc988\ub85c \uc120\uc815 \ub418\uc5c8\uace0, \uadf8\ub140\ub294 \ud5a5\uc218\uc5d0 \ub300\ud55c \ub2c8\ucf5c\ub77c\uc2a4 \uc708\ub529 \ub808\ud508 \uac10\ub3c5\uc758 \ub2e8\ud3b8 \uc601\ud654 \uad11\uace0\uc5d0 \ucd9c\uc5f0\ud588\ub2e4."} {"question": "\uc0e4\ud06c\uc640\uc758 \ub4c0\uc5bc \ub3c4\uc911 \ubd88\uac00\uc0ac\uc758\ud55c \uc874\uc7ac\uc778 \uc544\uc2a4\ud2b8\ub784\ub85c\ubd80\ud130 \ub4c0\uc5bc\ub85c \uc774\uae30\uac8c \ub418\ub294\ub370 \uc5ec\uae30\uc11c \ucc98\uc74c \uc811\ud55c \uce74\ub4dc\uc758 \uc774\ub984\uc740?", "paragraph": "\uac00\uae4c\uc6b4 \ubbf8\ub798, \ud558\ud2b8\ub79c\ub4dc\ub77c \ubd88\ub9ac\ub294 \ub3c4\uc2dc\uc5d0 \uc0ac\ub294 \uc720\ub9c8(\uce20\ucfe0\ubaa8 \uc720\ub9c8)\ub294 \ud5c8\ub2f9 \ub4c0\uc5bc\ub9ac\uc2a4\ud2b8\uc774\uc9c0\ub9cc \ub4c0\uc5bc \ucc54\ud53c\uc5b8\uc758 \uafc8\uc744 \uc774\ub8e8\uae30 \uc704\ud574 \ub178\ub825\ud558\ub358 \uc5b4\ub290 \ub0a0, \uce5c\uad6c\uc778 \ud0dc\ud638\uc758 \ub371\uc744 \ube7c\uc558\uace0 \ubd80\ubaa8\ub2d8\uc774 \ubb3c\ub824\uc8fc\uc2e0 \ud669\uc758 \uc5f4\uc1e0\ub97c \ub9dd\uac00\ub728\ub9b0,\ud559\uad50\uc5d0\uc11c \uae61\ud328\ub85c \ubb3c\ub4e0 \uc0e4\ud06c(\uce74\ubbf8\uc2dc\ub85c \ub8cc\uac00)\uc640\uc758 \ub4c0\uc5bc \ub3c4\uc911 '\uc5d1\uc2dc\uc988'\ub77c\ub294 \uce74\ub4dc\ub97c \uc811\ud558\uac8c \ub418\uace0 \uc544\uc2a4\ud2b8\ub784\uc774\ub77c\ub294 \ubd88\uac00\uc0ac\uc758\ud55c \uc874\uc7ac\uc758 \ub3c4\uc6c0\uc73c\ub85c \ub4c0\uc5bc\uc5d0\uc11c \uc774\uae30\uac8c \ub41c\ub2e4. \uadf8\ub9ac\uace0 \uc544\uc2a4\ud2b8\ub784\uc740 \uc783\uc5b4\ubc84\ub9b0 \uae30\uc5b5\uc758 \uc870\uac01\uc778 \ub118\ubc84\uc988\ub97c \ucc3e\uc73c\ub7ec \uc654\uc73c\uba70, \uc720\ub9c8\uac00 \uc9c0\ub2cc \ud669\uc758 \uc5f4\uc1e0\ub85c\ubd80\ud130 \ub5a8\uc5b4\uc9c8 \uc218 \uc5c6\uae30\uc5d0 \uc138\uacc4 \uac01\uc9c0\uc5d0 \ud769\uc5b4\uc838 \uc788\ub294 \ub118\ubc84\uc988\ub97c \ucc3e\uc544\uc57c \ud568\uc744 \uc54c\uac8c \ub41c \uc720\uc6b0\ub9c8\ub294 \ub118\ubc84\uc988 \uc218\uc9d1\uc5d0 \ub531\ud788 \uad00\uc2ec\uc774 \uc5c6\uc74c\uc5d0\ub3c4 \uc790\uc2e0\uc774 \ub118\ubc84\uc988\ub97c \uac00\uc9c0\uace0 \uc788\uae30 \ub54c\ubb38\uc5d0 \ub118\ubc84\uc988\uc5d0 \uad00\ud574 \uc5bd\ud788\uac8c \ub418\uc5b4 \uc544\uc2a4\ud2b8\ub784\uacfc \ud568\uaed8 \ub118\ubc84\uc988\ub97c \ub418\ucc3e\uc73c\ub824 \ub178\ub825\ud558\uac8c \ub41c\ub2e4. \ub3c4\uc911 \ub118\ubc84\uc988 \ud5cc\ud130 \ud150\uc8e0\uc6b0 \uce74\uc774\ud1a0\uc640 \ub9cc\ub098 \ub4c0\uc5bc\uc744 \ud3bc\uce58\uc9c0\ub9cc \ub05d\uc744 \ub0b4\uc9c0 \ubabb\ud558\uace0, \uc81c\uc54c\uc758 \ud798\uc744 \uc5bb\uac8c \ub41c\ub2e4.", "answer": "\uc5d1\uc2dc\uc988", "sentence": "\uac00\uae4c\uc6b4 \ubbf8\ub798, \ud558\ud2b8\ub79c\ub4dc\ub77c \ubd88\ub9ac\ub294 \ub3c4\uc2dc\uc5d0 \uc0ac\ub294 \uc720\ub9c8(\uce20\ucfe0\ubaa8 \uc720\ub9c8)\ub294 \ud5c8\ub2f9 \ub4c0\uc5bc\ub9ac\uc2a4\ud2b8\uc774\uc9c0\ub9cc \ub4c0\uc5bc \ucc54\ud53c\uc5b8\uc758 \uafc8\uc744 \uc774\ub8e8\uae30 \uc704\ud574 \ub178\ub825\ud558\ub358 \uc5b4\ub290 \ub0a0, \uce5c\uad6c\uc778 \ud0dc\ud638\uc758 \ub371\uc744 \ube7c\uc558\uace0 \ubd80\ubaa8\ub2d8\uc774 \ubb3c\ub824\uc8fc\uc2e0 \ud669\uc758 \uc5f4\uc1e0\ub97c \ub9dd\uac00\ub728\ub9b0,\ud559\uad50\uc5d0\uc11c \uae61\ud328\ub85c \ubb3c\ub4e0 \uc0e4\ud06c(\uce74\ubbf8\uc2dc\ub85c \ub8cc\uac00)\uc640\uc758 \ub4c0\uc5bc \ub3c4\uc911 ' \uc5d1\uc2dc\uc988 '\ub77c\ub294 \uce74\ub4dc\ub97c \uc811\ud558\uac8c \ub418\uace0 \uc544\uc2a4\ud2b8\ub784\uc774\ub77c\ub294 \ubd88\uac00\uc0ac\uc758\ud55c \uc874\uc7ac\uc758 \ub3c4\uc6c0\uc73c\ub85c \ub4c0\uc5bc\uc5d0\uc11c \uc774\uae30\uac8c \ub41c\ub2e4.", "paragraph_sentence": " \uac00\uae4c\uc6b4 \ubbf8\ub798, \ud558\ud2b8\ub79c\ub4dc\ub77c \ubd88\ub9ac\ub294 \ub3c4\uc2dc\uc5d0 \uc0ac\ub294 \uc720\ub9c8(\uce20\ucfe0\ubaa8 \uc720\ub9c8)\ub294 \ud5c8\ub2f9 \ub4c0\uc5bc\ub9ac\uc2a4\ud2b8\uc774\uc9c0\ub9cc \ub4c0\uc5bc \ucc54\ud53c\uc5b8\uc758 \uafc8\uc744 \uc774\ub8e8\uae30 \uc704\ud574 \ub178\ub825\ud558\ub358 \uc5b4\ub290 \ub0a0, \uce5c\uad6c\uc778 \ud0dc\ud638\uc758 \ub371\uc744 \ube7c\uc558\uace0 \ubd80\ubaa8\ub2d8\uc774 \ubb3c\ub824\uc8fc\uc2e0 \ud669\uc758 \uc5f4\uc1e0\ub97c \ub9dd\uac00\ub728\ub9b0,\ud559\uad50\uc5d0\uc11c \uae61\ud328\ub85c \ubb3c\ub4e0 \uc0e4\ud06c(\uce74\ubbf8\uc2dc\ub85c \ub8cc\uac00)\uc640\uc758 \ub4c0\uc5bc \ub3c4\uc911 ' \uc5d1\uc2dc\uc988 '\ub77c\ub294 \uce74\ub4dc\ub97c \uc811\ud558\uac8c \ub418\uace0 \uc544\uc2a4\ud2b8\ub784\uc774\ub77c\ub294 \ubd88\uac00\uc0ac\uc758\ud55c \uc874\uc7ac\uc758 \ub3c4\uc6c0\uc73c\ub85c \ub4c0\uc5bc\uc5d0\uc11c \uc774\uae30\uac8c \ub41c\ub2e4. \uadf8\ub9ac\uace0 \uc544\uc2a4\ud2b8\ub784\uc740 \uc783\uc5b4\ubc84\ub9b0 \uae30\uc5b5\uc758 \uc870\uac01\uc778 \ub118\ubc84\uc988\ub97c \ucc3e\uc73c\ub7ec \uc654\uc73c\uba70, \uc720\ub9c8\uac00 \uc9c0\ub2cc \ud669\uc758 \uc5f4\uc1e0\ub85c\ubd80\ud130 \ub5a8\uc5b4\uc9c8 \uc218 \uc5c6\uae30\uc5d0 \uc138\uacc4 \uac01\uc9c0\uc5d0 \ud769\uc5b4\uc838 \uc788\ub294 \ub118\ubc84\uc988\ub97c \ucc3e\uc544\uc57c \ud568\uc744 \uc54c\uac8c \ub41c \uc720\uc6b0\ub9c8\ub294 \ub118\ubc84\uc988 \uc218\uc9d1\uc5d0 \ub531\ud788 \uad00\uc2ec\uc774 \uc5c6\uc74c\uc5d0\ub3c4 \uc790\uc2e0\uc774 \ub118\ubc84\uc988\ub97c \uac00\uc9c0\uace0 \uc788\uae30 \ub54c\ubb38\uc5d0 \ub118\ubc84\uc988\uc5d0 \uad00\ud574 \uc5bd\ud788\uac8c \ub418\uc5b4 \uc544\uc2a4\ud2b8\ub784\uacfc \ud568\uaed8 \ub118\ubc84\uc988\ub97c \ub418\ucc3e\uc73c\ub824 \ub178\ub825\ud558\uac8c \ub41c\ub2e4. \ub3c4\uc911 \ub118\ubc84\uc988 \ud5cc\ud130 \ud150\uc8e0\uc6b0 \uce74\uc774\ud1a0\uc640 \ub9cc\ub098 \ub4c0\uc5bc\uc744 \ud3bc\uce58\uc9c0\ub9cc \ub05d\uc744 \ub0b4\uc9c0 \ubabb\ud558\uace0, \uc81c\uc54c\uc758 \ud798\uc744 \uc5bb\uac8c \ub41c\ub2e4.", "paragraph_answer": "\uac00\uae4c\uc6b4 \ubbf8\ub798, \ud558\ud2b8\ub79c\ub4dc\ub77c \ubd88\ub9ac\ub294 \ub3c4\uc2dc\uc5d0 \uc0ac\ub294 \uc720\ub9c8(\uce20\ucfe0\ubaa8 \uc720\ub9c8)\ub294 \ud5c8\ub2f9 \ub4c0\uc5bc\ub9ac\uc2a4\ud2b8\uc774\uc9c0\ub9cc \ub4c0\uc5bc \ucc54\ud53c\uc5b8\uc758 \uafc8\uc744 \uc774\ub8e8\uae30 \uc704\ud574 \ub178\ub825\ud558\ub358 \uc5b4\ub290 \ub0a0, \uce5c\uad6c\uc778 \ud0dc\ud638\uc758 \ub371\uc744 \ube7c\uc558\uace0 \ubd80\ubaa8\ub2d8\uc774 \ubb3c\ub824\uc8fc\uc2e0 \ud669\uc758 \uc5f4\uc1e0\ub97c \ub9dd\uac00\ub728\ub9b0,\ud559\uad50\uc5d0\uc11c \uae61\ud328\ub85c \ubb3c\ub4e0 \uc0e4\ud06c(\uce74\ubbf8\uc2dc\ub85c \ub8cc\uac00)\uc640\uc758 \ub4c0\uc5bc \ub3c4\uc911 ' \uc5d1\uc2dc\uc988 '\ub77c\ub294 \uce74\ub4dc\ub97c \uc811\ud558\uac8c \ub418\uace0 \uc544\uc2a4\ud2b8\ub784\uc774\ub77c\ub294 \ubd88\uac00\uc0ac\uc758\ud55c \uc874\uc7ac\uc758 \ub3c4\uc6c0\uc73c\ub85c \ub4c0\uc5bc\uc5d0\uc11c \uc774\uae30\uac8c \ub41c\ub2e4. \uadf8\ub9ac\uace0 \uc544\uc2a4\ud2b8\ub784\uc740 \uc783\uc5b4\ubc84\ub9b0 \uae30\uc5b5\uc758 \uc870\uac01\uc778 \ub118\ubc84\uc988\ub97c \ucc3e\uc73c\ub7ec \uc654\uc73c\uba70, \uc720\ub9c8\uac00 \uc9c0\ub2cc \ud669\uc758 \uc5f4\uc1e0\ub85c\ubd80\ud130 \ub5a8\uc5b4\uc9c8 \uc218 \uc5c6\uae30\uc5d0 \uc138\uacc4 \uac01\uc9c0\uc5d0 \ud769\uc5b4\uc838 \uc788\ub294 \ub118\ubc84\uc988\ub97c \ucc3e\uc544\uc57c \ud568\uc744 \uc54c\uac8c \ub41c \uc720\uc6b0\ub9c8\ub294 \ub118\ubc84\uc988 \uc218\uc9d1\uc5d0 \ub531\ud788 \uad00\uc2ec\uc774 \uc5c6\uc74c\uc5d0\ub3c4 \uc790\uc2e0\uc774 \ub118\ubc84\uc988\ub97c \uac00\uc9c0\uace0 \uc788\uae30 \ub54c\ubb38\uc5d0 \ub118\ubc84\uc988\uc5d0 \uad00\ud574 \uc5bd\ud788\uac8c \ub418\uc5b4 \uc544\uc2a4\ud2b8\ub784\uacfc \ud568\uaed8 \ub118\ubc84\uc988\ub97c \ub418\ucc3e\uc73c\ub824 \ub178\ub825\ud558\uac8c \ub41c\ub2e4. \ub3c4\uc911 \ub118\ubc84\uc988 \ud5cc\ud130 \ud150\uc8e0\uc6b0 \uce74\uc774\ud1a0\uc640 \ub9cc\ub098 \ub4c0\uc5bc\uc744 \ud3bc\uce58\uc9c0\ub9cc \ub05d\uc744 \ub0b4\uc9c0 \ubabb\ud558\uace0, \uc81c\uc54c\uc758 \ud798\uc744 \uc5bb\uac8c \ub41c\ub2e4.", "sentence_answer": "\uac00\uae4c\uc6b4 \ubbf8\ub798, \ud558\ud2b8\ub79c\ub4dc\ub77c \ubd88\ub9ac\ub294 \ub3c4\uc2dc\uc5d0 \uc0ac\ub294 \uc720\ub9c8(\uce20\ucfe0\ubaa8 \uc720\ub9c8)\ub294 \ud5c8\ub2f9 \ub4c0\uc5bc\ub9ac\uc2a4\ud2b8\uc774\uc9c0\ub9cc \ub4c0\uc5bc \ucc54\ud53c\uc5b8\uc758 \uafc8\uc744 \uc774\ub8e8\uae30 \uc704\ud574 \ub178\ub825\ud558\ub358 \uc5b4\ub290 \ub0a0, \uce5c\uad6c\uc778 \ud0dc\ud638\uc758 \ub371\uc744 \ube7c\uc558\uace0 \ubd80\ubaa8\ub2d8\uc774 \ubb3c\ub824\uc8fc\uc2e0 \ud669\uc758 \uc5f4\uc1e0\ub97c \ub9dd\uac00\ub728\ub9b0,\ud559\uad50\uc5d0\uc11c \uae61\ud328\ub85c \ubb3c\ub4e0 \uc0e4\ud06c(\uce74\ubbf8\uc2dc\ub85c \ub8cc\uac00)\uc640\uc758 \ub4c0\uc5bc \ub3c4\uc911 ' \uc5d1\uc2dc\uc988 '\ub77c\ub294 \uce74\ub4dc\ub97c \uc811\ud558\uac8c \ub418\uace0 \uc544\uc2a4\ud2b8\ub784\uc774\ub77c\ub294 \ubd88\uac00\uc0ac\uc758\ud55c \uc874\uc7ac\uc758 \ub3c4\uc6c0\uc73c\ub85c \ub4c0\uc5bc\uc5d0\uc11c \uc774\uae30\uac8c \ub41c\ub2e4.", "paragraph_id": "6543571-1-1", "paragraph_question": "question: \uc0e4\ud06c\uc640\uc758 \ub4c0\uc5bc \ub3c4\uc911 \ubd88\uac00\uc0ac\uc758\ud55c \uc874\uc7ac\uc778 \uc544\uc2a4\ud2b8\ub784\ub85c\ubd80\ud130 \ub4c0\uc5bc\ub85c \uc774\uae30\uac8c \ub418\ub294\ub370 \uc5ec\uae30\uc11c \ucc98\uc74c \uc811\ud55c \uce74\ub4dc\uc758 \uc774\ub984\uc740?, context: \uac00\uae4c\uc6b4 \ubbf8\ub798, \ud558\ud2b8\ub79c\ub4dc\ub77c \ubd88\ub9ac\ub294 \ub3c4\uc2dc\uc5d0 \uc0ac\ub294 \uc720\ub9c8(\uce20\ucfe0\ubaa8 \uc720\ub9c8)\ub294 \ud5c8\ub2f9 \ub4c0\uc5bc\ub9ac\uc2a4\ud2b8\uc774\uc9c0\ub9cc \ub4c0\uc5bc \ucc54\ud53c\uc5b8\uc758 \uafc8\uc744 \uc774\ub8e8\uae30 \uc704\ud574 \ub178\ub825\ud558\ub358 \uc5b4\ub290 \ub0a0, \uce5c\uad6c\uc778 \ud0dc\ud638\uc758 \ub371\uc744 \ube7c\uc558\uace0 \ubd80\ubaa8\ub2d8\uc774 \ubb3c\ub824\uc8fc\uc2e0 \ud669\uc758 \uc5f4\uc1e0\ub97c \ub9dd\uac00\ub728\ub9b0,\ud559\uad50\uc5d0\uc11c \uae61\ud328\ub85c \ubb3c\ub4e0 \uc0e4\ud06c(\uce74\ubbf8\uc2dc\ub85c \ub8cc\uac00)\uc640\uc758 \ub4c0\uc5bc \ub3c4\uc911 '\uc5d1\uc2dc\uc988'\ub77c\ub294 \uce74\ub4dc\ub97c \uc811\ud558\uac8c \ub418\uace0 \uc544\uc2a4\ud2b8\ub784\uc774\ub77c\ub294 \ubd88\uac00\uc0ac\uc758\ud55c \uc874\uc7ac\uc758 \ub3c4\uc6c0\uc73c\ub85c \ub4c0\uc5bc\uc5d0\uc11c \uc774\uae30\uac8c \ub41c\ub2e4. \uadf8\ub9ac\uace0 \uc544\uc2a4\ud2b8\ub784\uc740 \uc783\uc5b4\ubc84\ub9b0 \uae30\uc5b5\uc758 \uc870\uac01\uc778 \ub118\ubc84\uc988\ub97c \ucc3e\uc73c\ub7ec \uc654\uc73c\uba70, \uc720\ub9c8\uac00 \uc9c0\ub2cc \ud669\uc758 \uc5f4\uc1e0\ub85c\ubd80\ud130 \ub5a8\uc5b4\uc9c8 \uc218 \uc5c6\uae30\uc5d0 \uc138\uacc4 \uac01\uc9c0\uc5d0 \ud769\uc5b4\uc838 \uc788\ub294 \ub118\ubc84\uc988\ub97c \ucc3e\uc544\uc57c \ud568\uc744 \uc54c\uac8c \ub41c \uc720\uc6b0\ub9c8\ub294 \ub118\ubc84\uc988 \uc218\uc9d1\uc5d0 \ub531\ud788 \uad00\uc2ec\uc774 \uc5c6\uc74c\uc5d0\ub3c4 \uc790\uc2e0\uc774 \ub118\ubc84\uc988\ub97c \uac00\uc9c0\uace0 \uc788\uae30 \ub54c\ubb38\uc5d0 \ub118\ubc84\uc988\uc5d0 \uad00\ud574 \uc5bd\ud788\uac8c \ub418\uc5b4 \uc544\uc2a4\ud2b8\ub784\uacfc \ud568\uaed8 \ub118\ubc84\uc988\ub97c \ub418\ucc3e\uc73c\ub824 \ub178\ub825\ud558\uac8c \ub41c\ub2e4. \ub3c4\uc911 \ub118\ubc84\uc988 \ud5cc\ud130 \ud150\uc8e0\uc6b0 \uce74\uc774\ud1a0\uc640 \ub9cc\ub098 \ub4c0\uc5bc\uc744 \ud3bc\uce58\uc9c0\ub9cc \ub05d\uc744 \ub0b4\uc9c0 \ubabb\ud558\uace0, \uc81c\uc54c\uc758 \ud798\uc744 \uc5bb\uac8c \ub41c\ub2e4."} {"question": "\uc544\uc774\ub124\uc774\uc544\uc2a4\uac00 \ud2b8\ub85c\uc774\uc544\ub97c \ub5a0\ub098 \uc0c8\ub85c\uc6b4 \ub3c4\uc2dc\ub97c \uc138\uc6b0\ub294 \ub0b4\uc6a9\uc744 \uc790\uc138\ud788 \ub2e4\ub8ec \ubca0\ub974\uae38\ub9ac\uc6b0\uc2a4\uc758 \ucc45\uc740?", "paragraph": "\uadf8\ub9ac\uc2a4 \uc2e0\ud654\ub294 \uadf8\ub9ac\uc2a4\uc640 \ud2b8\ub85c\uc774\uc544 \uac04\uc758 \ud2b8\ub85c\uc774\uc544 \uc804\uc7c1\uacfc \uadf8 \uacb0\uacfc\uc5d0\uc11c \uc808\uc815\uc744 \uc774\ub8ec\ub2e4. \ud638\uba54\ub85c\uc2a4 \uc791\ud488\uc5d0\uc11c\ub294 \uc8fc\ub41c \uc904\uac70\ub9ac\uac00 \uc774\ubbf8 \ucda9\ubd84\ud55c \ud615\ud0dc\uc640 \uc694\uc9c0\ub97c \uac16\ucd94\uc5c8\uc73c\uba70, \uac1c\ubcc4\uc801\uc778 \uc8fc\uc81c\uc758 \uacbd\uc6b0\uc5d0\ub294 \uadf8 \ud6c4\uc5d0 \uadf8\ub9ac\uc2a4 \uc5f0\uadf9\uacfc \uac19\uc740 \ub9e4\uccb4\uc5d0\uc11c \ub354\uc6b1 \uc790\uc138\ud574\uc84c\ub2e4. \ud2b8\ub85c\uc774\uc544 \uc804\uc7c1\uc740 \ub610\ud55c \uc544\uc774\ub124\uc774\uc544\uc2a4\uc758 \uc774\uc57c\uae30\ub85c \ub85c\ub9c8 \ubb38\ud654\uc5d0\uc11c \uad49\uc7a5\ud55c \uad00\uc2ec\uc744 \uc774\ub04c\uc5b4\ub0b4\uc5c8\ub2e4. \uc774 \uc774\uc57c\uae30\uc5d0\uc11c\ub294 \ud2b8\ub85c\uc774\uc544\uc758 \uc601\uc6c5\uc778 \uc544\uc774\ub124\uc774\uc544\uc2a4\uac00 \ud2b8\ub85c\uc774\uc544\ub97c \ub5a0\ub098 \ubc29\ub791\ud558\ub358 \uc911 \ub85c\ub9c8 \uc81c\uad6d\uc758 \uac74\uad6d \uc2dc\uc870\uac00 \ub41c \uc0c8\ub85c\uc6b4 \ub3c4\uc2dc\ub97c \uc138\uc6e0\ub2e4\uace0 \uc804\ud558\ub294\ub370, \ubca0\ub974\uae38\ub9ac\uc6b0\uc2a4\uac00 \uc774\ud6c4\uc5d0 \u300a\uc544\uc774\ub124\uc774\uc2a4\u300b\ub77c\ub294 \ucc45\uc73c\ub85c \uc790\uc138\ud788 \ub2e4\ub8e8\uc5c8\ub2e4.(\ubca0\ub974\uae38\ub9ac\uc6b0\uc2a4\uc758 \u300a\uc544\uc774\ub124\uc774\uc2a4\u300b 2\uad8c\uc5d0\ub294 \ub9e4\uc6b0 \uc798 \uc54c\ub824\uc9c4 \ud2b8\ub85c\uc774\uc544 \ubd80\ub300 \uc774\uc57c\uae30\uac00 \uc788\uc74c.) \ub9c8\uc9c0\ub9c9\uc5d0 \uc640\uc11c\ub294 \ub515\ud2f0\uc2a4 \ud06c\ub808\ud150\uc2dc\uc2a4, \ub2e4\ub808\uc2a4 \ud504\ub9ac\uae30\ub204\uc2a4\ub77c\ub294 \uc774\ub984\uc758 \uc800\uc790\uac00 \uc37c\ub2e4\ub294 \ud5c8\uc704 \uc5f0\ub300\uae30 \ub450 \uad8c\uc774 \ub77c\ud2f4\uc5b4\ub85c \uc4f0\uc5ec\uc838 \uc804\ud574 \ub0b4\ub824\uc628\ub2e4.", "answer": "\uc544\uc774\ub124\uc774\uc2a4", "sentence": "\uc774 \uc774\uc57c\uae30\uc5d0\uc11c\ub294 \ud2b8\ub85c\uc774\uc544\uc758 \uc601\uc6c5\uc778 \uc544\uc774\ub124\uc774\uc544\uc2a4\uac00 \ud2b8\ub85c\uc774\uc544\ub97c \ub5a0\ub098 \ubc29\ub791\ud558\ub358 \uc911 \ub85c\ub9c8 \uc81c\uad6d\uc758 \uac74\uad6d \uc2dc\uc870\uac00 \ub41c \uc0c8\ub85c\uc6b4 \ub3c4\uc2dc\ub97c \uc138\uc6e0\ub2e4\uace0 \uc804\ud558\ub294\ub370, \ubca0\ub974\uae38\ub9ac\uc6b0\uc2a4\uac00 \uc774\ud6c4\uc5d0 \u300a \uc544\uc774\ub124\uc774\uc2a4 \u300b\ub77c\ub294 \ucc45\uc73c\ub85c \uc790\uc138\ud788 \ub2e4\ub8e8\uc5c8\ub2e4.(\ubca0\ub974\uae38\ub9ac\uc6b0\uc2a4\uc758 \u300a\uc544\uc774\ub124\uc774\uc2a4\u300b 2\uad8c\uc5d0\ub294 \ub9e4\uc6b0 \uc798 \uc54c\ub824\uc9c4 \ud2b8\ub85c\uc774\uc544 \ubd80\ub300 \uc774\uc57c\uae30\uac00 \uc788\uc74c.", "paragraph_sentence": "\uadf8\ub9ac\uc2a4 \uc2e0\ud654\ub294 \uadf8\ub9ac\uc2a4\uc640 \ud2b8\ub85c\uc774\uc544 \uac04\uc758 \ud2b8\ub85c\uc774\uc544 \uc804\uc7c1\uacfc \uadf8 \uacb0\uacfc\uc5d0\uc11c \uc808\uc815\uc744 \uc774\ub8ec\ub2e4. \ud638\uba54\ub85c\uc2a4 \uc791\ud488\uc5d0\uc11c\ub294 \uc8fc\ub41c \uc904\uac70\ub9ac\uac00 \uc774\ubbf8 \ucda9\ubd84\ud55c \ud615\ud0dc\uc640 \uc694\uc9c0\ub97c \uac16\ucd94\uc5c8\uc73c\uba70, \uac1c\ubcc4\uc801\uc778 \uc8fc\uc81c\uc758 \uacbd\uc6b0\uc5d0\ub294 \uadf8 \ud6c4\uc5d0 \uadf8\ub9ac\uc2a4 \uc5f0\uadf9\uacfc \uac19\uc740 \ub9e4\uccb4\uc5d0\uc11c \ub354\uc6b1 \uc790\uc138\ud574\uc84c\ub2e4. \ud2b8\ub85c\uc774\uc544 \uc804\uc7c1\uc740 \ub610\ud55c \uc544\uc774\ub124\uc774\uc544\uc2a4\uc758 \uc774\uc57c\uae30\ub85c \ub85c\ub9c8 \ubb38\ud654\uc5d0\uc11c \uad49\uc7a5\ud55c \uad00\uc2ec\uc744 \uc774\ub04c\uc5b4\ub0b4\uc5c8\ub2e4. \uc774 \uc774\uc57c\uae30\uc5d0\uc11c\ub294 \ud2b8\ub85c\uc774\uc544\uc758 \uc601\uc6c5\uc778 \uc544\uc774\ub124\uc774\uc544\uc2a4\uac00 \ud2b8\ub85c\uc774\uc544\ub97c \ub5a0\ub098 \ubc29\ub791\ud558\ub358 \uc911 \ub85c\ub9c8 \uc81c\uad6d\uc758 \uac74\uad6d \uc2dc\uc870\uac00 \ub41c \uc0c8\ub85c\uc6b4 \ub3c4\uc2dc\ub97c \uc138\uc6e0\ub2e4\uace0 \uc804\ud558\ub294\ub370, \ubca0\ub974\uae38\ub9ac\uc6b0\uc2a4\uac00 \uc774\ud6c4\uc5d0 \u300a \uc544\uc774\ub124\uc774\uc2a4 \u300b\ub77c\ub294 \ucc45\uc73c\ub85c \uc790\uc138\ud788 \ub2e4\ub8e8\uc5c8\ub2e4.(\ubca0\ub974\uae38\ub9ac\uc6b0\uc2a4\uc758 \u300a\uc544\uc774\ub124\uc774\uc2a4\u300b 2\uad8c\uc5d0\ub294 \ub9e4\uc6b0 \uc798 \uc54c\ub824\uc9c4 \ud2b8\ub85c\uc774\uc544 \ubd80\ub300 \uc774\uc57c\uae30\uac00 \uc788\uc74c. ) \ub9c8\uc9c0\ub9c9\uc5d0 \uc640\uc11c\ub294 \ub515\ud2f0\uc2a4 \ud06c\ub808\ud150\uc2dc\uc2a4, \ub2e4\ub808\uc2a4 \ud504\ub9ac\uae30\ub204\uc2a4\ub77c\ub294 \uc774\ub984\uc758 \uc800\uc790\uac00 \uc37c\ub2e4\ub294 \ud5c8\uc704 \uc5f0\ub300\uae30 \ub450 \uad8c\uc774 \ub77c\ud2f4\uc5b4\ub85c \uc4f0\uc5ec\uc838 \uc804\ud574 \ub0b4\ub824\uc628\ub2e4.", "paragraph_answer": "\uadf8\ub9ac\uc2a4 \uc2e0\ud654\ub294 \uadf8\ub9ac\uc2a4\uc640 \ud2b8\ub85c\uc774\uc544 \uac04\uc758 \ud2b8\ub85c\uc774\uc544 \uc804\uc7c1\uacfc \uadf8 \uacb0\uacfc\uc5d0\uc11c \uc808\uc815\uc744 \uc774\ub8ec\ub2e4. \ud638\uba54\ub85c\uc2a4 \uc791\ud488\uc5d0\uc11c\ub294 \uc8fc\ub41c \uc904\uac70\ub9ac\uac00 \uc774\ubbf8 \ucda9\ubd84\ud55c \ud615\ud0dc\uc640 \uc694\uc9c0\ub97c \uac16\ucd94\uc5c8\uc73c\uba70, \uac1c\ubcc4\uc801\uc778 \uc8fc\uc81c\uc758 \uacbd\uc6b0\uc5d0\ub294 \uadf8 \ud6c4\uc5d0 \uadf8\ub9ac\uc2a4 \uc5f0\uadf9\uacfc \uac19\uc740 \ub9e4\uccb4\uc5d0\uc11c \ub354\uc6b1 \uc790\uc138\ud574\uc84c\ub2e4. \ud2b8\ub85c\uc774\uc544 \uc804\uc7c1\uc740 \ub610\ud55c \uc544\uc774\ub124\uc774\uc544\uc2a4\uc758 \uc774\uc57c\uae30\ub85c \ub85c\ub9c8 \ubb38\ud654\uc5d0\uc11c \uad49\uc7a5\ud55c \uad00\uc2ec\uc744 \uc774\ub04c\uc5b4\ub0b4\uc5c8\ub2e4. \uc774 \uc774\uc57c\uae30\uc5d0\uc11c\ub294 \ud2b8\ub85c\uc774\uc544\uc758 \uc601\uc6c5\uc778 \uc544\uc774\ub124\uc774\uc544\uc2a4\uac00 \ud2b8\ub85c\uc774\uc544\ub97c \ub5a0\ub098 \ubc29\ub791\ud558\ub358 \uc911 \ub85c\ub9c8 \uc81c\uad6d\uc758 \uac74\uad6d \uc2dc\uc870\uac00 \ub41c \uc0c8\ub85c\uc6b4 \ub3c4\uc2dc\ub97c \uc138\uc6e0\ub2e4\uace0 \uc804\ud558\ub294\ub370, \ubca0\ub974\uae38\ub9ac\uc6b0\uc2a4\uac00 \uc774\ud6c4\uc5d0 \u300a \uc544\uc774\ub124\uc774\uc2a4 \u300b\ub77c\ub294 \ucc45\uc73c\ub85c \uc790\uc138\ud788 \ub2e4\ub8e8\uc5c8\ub2e4.(\ubca0\ub974\uae38\ub9ac\uc6b0\uc2a4\uc758 \u300a\uc544\uc774\ub124\uc774\uc2a4\u300b 2\uad8c\uc5d0\ub294 \ub9e4\uc6b0 \uc798 \uc54c\ub824\uc9c4 \ud2b8\ub85c\uc774\uc544 \ubd80\ub300 \uc774\uc57c\uae30\uac00 \uc788\uc74c.) \ub9c8\uc9c0\ub9c9\uc5d0 \uc640\uc11c\ub294 \ub515\ud2f0\uc2a4 \ud06c\ub808\ud150\uc2dc\uc2a4, \ub2e4\ub808\uc2a4 \ud504\ub9ac\uae30\ub204\uc2a4\ub77c\ub294 \uc774\ub984\uc758 \uc800\uc790\uac00 \uc37c\ub2e4\ub294 \ud5c8\uc704 \uc5f0\ub300\uae30 \ub450 \uad8c\uc774 \ub77c\ud2f4\uc5b4\ub85c \uc4f0\uc5ec\uc838 \uc804\ud574 \ub0b4\ub824\uc628\ub2e4.", "sentence_answer": "\uc774 \uc774\uc57c\uae30\uc5d0\uc11c\ub294 \ud2b8\ub85c\uc774\uc544\uc758 \uc601\uc6c5\uc778 \uc544\uc774\ub124\uc774\uc544\uc2a4\uac00 \ud2b8\ub85c\uc774\uc544\ub97c \ub5a0\ub098 \ubc29\ub791\ud558\ub358 \uc911 \ub85c\ub9c8 \uc81c\uad6d\uc758 \uac74\uad6d \uc2dc\uc870\uac00 \ub41c \uc0c8\ub85c\uc6b4 \ub3c4\uc2dc\ub97c \uc138\uc6e0\ub2e4\uace0 \uc804\ud558\ub294\ub370, \ubca0\ub974\uae38\ub9ac\uc6b0\uc2a4\uac00 \uc774\ud6c4\uc5d0 \u300a \uc544\uc774\ub124\uc774\uc2a4 \u300b\ub77c\ub294 \ucc45\uc73c\ub85c \uc790\uc138\ud788 \ub2e4\ub8e8\uc5c8\ub2e4.(\ubca0\ub974\uae38\ub9ac\uc6b0\uc2a4\uc758 \u300a\uc544\uc774\ub124\uc774\uc2a4\u300b 2\uad8c\uc5d0\ub294 \ub9e4\uc6b0 \uc798 \uc54c\ub824\uc9c4 \ud2b8\ub85c\uc774\uc544 \ubd80\ub300 \uc774\uc57c\uae30\uac00 \uc788\uc74c.", "paragraph_id": "6477962-19-0", "paragraph_question": "question: \uc544\uc774\ub124\uc774\uc544\uc2a4\uac00 \ud2b8\ub85c\uc774\uc544\ub97c \ub5a0\ub098 \uc0c8\ub85c\uc6b4 \ub3c4\uc2dc\ub97c \uc138\uc6b0\ub294 \ub0b4\uc6a9\uc744 \uc790\uc138\ud788 \ub2e4\ub8ec \ubca0\ub974\uae38\ub9ac\uc6b0\uc2a4\uc758 \ucc45\uc740?, context: \uadf8\ub9ac\uc2a4 \uc2e0\ud654\ub294 \uadf8\ub9ac\uc2a4\uc640 \ud2b8\ub85c\uc774\uc544 \uac04\uc758 \ud2b8\ub85c\uc774\uc544 \uc804\uc7c1\uacfc \uadf8 \uacb0\uacfc\uc5d0\uc11c \uc808\uc815\uc744 \uc774\ub8ec\ub2e4. \ud638\uba54\ub85c\uc2a4 \uc791\ud488\uc5d0\uc11c\ub294 \uc8fc\ub41c \uc904\uac70\ub9ac\uac00 \uc774\ubbf8 \ucda9\ubd84\ud55c \ud615\ud0dc\uc640 \uc694\uc9c0\ub97c \uac16\ucd94\uc5c8\uc73c\uba70, \uac1c\ubcc4\uc801\uc778 \uc8fc\uc81c\uc758 \uacbd\uc6b0\uc5d0\ub294 \uadf8 \ud6c4\uc5d0 \uadf8\ub9ac\uc2a4 \uc5f0\uadf9\uacfc \uac19\uc740 \ub9e4\uccb4\uc5d0\uc11c \ub354\uc6b1 \uc790\uc138\ud574\uc84c\ub2e4. \ud2b8\ub85c\uc774\uc544 \uc804\uc7c1\uc740 \ub610\ud55c \uc544\uc774\ub124\uc774\uc544\uc2a4\uc758 \uc774\uc57c\uae30\ub85c \ub85c\ub9c8 \ubb38\ud654\uc5d0\uc11c \uad49\uc7a5\ud55c \uad00\uc2ec\uc744 \uc774\ub04c\uc5b4\ub0b4\uc5c8\ub2e4. \uc774 \uc774\uc57c\uae30\uc5d0\uc11c\ub294 \ud2b8\ub85c\uc774\uc544\uc758 \uc601\uc6c5\uc778 \uc544\uc774\ub124\uc774\uc544\uc2a4\uac00 \ud2b8\ub85c\uc774\uc544\ub97c \ub5a0\ub098 \ubc29\ub791\ud558\ub358 \uc911 \ub85c\ub9c8 \uc81c\uad6d\uc758 \uac74\uad6d \uc2dc\uc870\uac00 \ub41c \uc0c8\ub85c\uc6b4 \ub3c4\uc2dc\ub97c \uc138\uc6e0\ub2e4\uace0 \uc804\ud558\ub294\ub370, \ubca0\ub974\uae38\ub9ac\uc6b0\uc2a4\uac00 \uc774\ud6c4\uc5d0 \u300a\uc544\uc774\ub124\uc774\uc2a4\u300b\ub77c\ub294 \ucc45\uc73c\ub85c \uc790\uc138\ud788 \ub2e4\ub8e8\uc5c8\ub2e4.(\ubca0\ub974\uae38\ub9ac\uc6b0\uc2a4\uc758 \u300a\uc544\uc774\ub124\uc774\uc2a4\u300b 2\uad8c\uc5d0\ub294 \ub9e4\uc6b0 \uc798 \uc54c\ub824\uc9c4 \ud2b8\ub85c\uc774\uc544 \ubd80\ub300 \uc774\uc57c\uae30\uac00 \uc788\uc74c.) \ub9c8\uc9c0\ub9c9\uc5d0 \uc640\uc11c\ub294 \ub515\ud2f0\uc2a4 \ud06c\ub808\ud150\uc2dc\uc2a4, \ub2e4\ub808\uc2a4 \ud504\ub9ac\uae30\ub204\uc2a4\ub77c\ub294 \uc774\ub984\uc758 \uc800\uc790\uac00 \uc37c\ub2e4\ub294 \ud5c8\uc704 \uc5f0\ub300\uae30 \ub450 \uad8c\uc774 \ub77c\ud2f4\uc5b4\ub85c \uc4f0\uc5ec\uc838 \uc804\ud574 \ub0b4\ub824\uc628\ub2e4."} {"question": "\ub300\ud55c\ubbfc\uad6d\uc758 \uba85\ubaa9 \uad6d\ub0b4 \ucd1d\uc0dd\uc0b0\uc740 2016\ub144 \uae30\uc900\uc73c\ub85c \uc5bc\ub9c8\uc778\uac00?", "paragraph": "\ub300\ud55c\ubbfc\uad6d\uc740 \ud55c\uad6d \uc804\uc7c1 \uc774\ub798 \uc77c\uba85 '\ud55c\uac15\uc758 \uae30\uc801'\uc774\ub77c\uace0 \ubd88\ub9ac\ub294 \ub192\uc740 \uacbd\uc81c \ubc1c\uc804\uc744 \uc774\ub8e9\ud558\uba70, 1990\ub144\ub300\uc5d0 \uc774\ub974\ub7ec \uc138\uacc4\uc801\uc778 \uacbd\uc81c \uac15\uad6d\uc73c\ub85c \ubc1c\uc804\ud558\uc600\ub2e4. 2015\ub144 \uad6c\ub9e4\ub825 \uae30\uc900 1\uc778\ub2f9 \uad6d\ubbfc \ucd1d\uc18c\ub4dd(GDP)\uc740 36,601\ub2ec\ub7ec\ub85c \uc138\uacc4\uc740\ud589\uc5d0\uc11c \uace0\uc18c\ub4dd \uad6d\uac00\ub85c \ubd84\ub958\ub418\uc5c8\uace0, 2016\ub144 \uc720\uc5d4\uc758 \uc778\uac04 \uac1c\ubc1c \uc9c0\uc218(HDI) \uc870\uc0ac\uc5d0\uc11c \uc138\uacc4 18\uc704\ub85c '\ub9e4\uc6b0 \ub192\uc74c'\uc73c\ub85c \ubd84\ub958\ub418\uc5c8\ub2e4. \ub610\ud55c, \uad6d\uc81c \ud1b5\ud654 \uae30\uae08(IMF)\uc5d0\uc11c\ub294 \ub300\ud55c\ubbfc\uad6d\uc744 \uc120\uc9c4 \uacbd\uc81c\uad6d\uc73c\ub85c \ubd84\ub958\ud558\uace0 \uc788\ub2e4. \ub300\ud55c\ubbfc\uad6d\uc758 \uba85\ubaa9 \uad6d\ub0b4 \ucd1d\uc0dd\uc0b0(GDP)\uc740 2016\ub144 1\uc870 4112\uc5b5 \ub2ec\ub7ec\uc774\ub2e4. \ub610\ud55c, \ub300\ud55c\ubbfc\uad6d\uc740 \uc8fc\uc694 20\uac1c\uad6d(G20), \uacbd\uc81c \ud611\ub825 \uac1c\ubc1c \uae30\uad6c(OECD), \uac1c\ubc1c \uc6d0\uc870 \uc704\uc6d0\ud68c(DAC), \ud30c\ub9ac \ud074\ub7fd\uacfc \uac19\uc740 \uae30\uad6c\uc5d0\uc11c \ud68c\uc6d0\uad6d\uc73c\ub85c \ud65c\ub3d9\ud558\uace0 \uc788\ub2e4.", "answer": "1\uc870 4112\uc5b5 \ub2ec\ub7ec", "sentence": "\ub300\ud55c\ubbfc\uad6d\uc758 \uba85\ubaa9 \uad6d\ub0b4 \ucd1d\uc0dd\uc0b0(GDP)\uc740 2016\ub144 1\uc870 4112\uc5b5 \ub2ec\ub7ec \uc774\ub2e4.", "paragraph_sentence": "\ub300\ud55c\ubbfc\uad6d\uc740 \ud55c\uad6d \uc804\uc7c1 \uc774\ub798 \uc77c\uba85 '\ud55c\uac15\uc758 \uae30\uc801'\uc774\ub77c\uace0 \ubd88\ub9ac\ub294 \ub192\uc740 \uacbd\uc81c \ubc1c\uc804\uc744 \uc774\ub8e9\ud558\uba70, 1990\ub144\ub300\uc5d0 \uc774\ub974\ub7ec \uc138\uacc4\uc801\uc778 \uacbd\uc81c \uac15\uad6d\uc73c\ub85c \ubc1c\uc804\ud558\uc600\ub2e4. 2015\ub144 \uad6c\ub9e4\ub825 \uae30\uc900 1\uc778\ub2f9 \uad6d\ubbfc \ucd1d\uc18c\ub4dd(GDP)\uc740 36,601\ub2ec\ub7ec\ub85c \uc138\uacc4\uc740\ud589\uc5d0\uc11c \uace0\uc18c\ub4dd \uad6d\uac00\ub85c \ubd84\ub958\ub418\uc5c8\uace0, 2016\ub144 \uc720\uc5d4\uc758 \uc778\uac04 \uac1c\ubc1c \uc9c0\uc218(HDI) \uc870\uc0ac\uc5d0\uc11c \uc138\uacc4 18\uc704\ub85c '\ub9e4\uc6b0 \ub192\uc74c'\uc73c\ub85c \ubd84\ub958\ub418\uc5c8\ub2e4. \ub610\ud55c, \uad6d\uc81c \ud1b5\ud654 \uae30\uae08(IMF)\uc5d0\uc11c\ub294 \ub300\ud55c\ubbfc\uad6d\uc744 \uc120\uc9c4 \uacbd\uc81c\uad6d\uc73c\ub85c \ubd84\ub958\ud558\uace0 \uc788\ub2e4. \ub300\ud55c\ubbfc\uad6d\uc758 \uba85\ubaa9 \uad6d\ub0b4 \ucd1d\uc0dd\uc0b0(GDP)\uc740 2016\ub144 1\uc870 4112\uc5b5 \ub2ec\ub7ec \uc774\ub2e4. \ub610\ud55c, \ub300\ud55c\ubbfc\uad6d\uc740 \uc8fc\uc694 20\uac1c\uad6d(G20), \uacbd\uc81c \ud611\ub825 \uac1c\ubc1c \uae30\uad6c(OECD), \uac1c\ubc1c \uc6d0\uc870 \uc704\uc6d0\ud68c(DAC), \ud30c\ub9ac \ud074\ub7fd\uacfc \uac19\uc740 \uae30\uad6c\uc5d0\uc11c \ud68c\uc6d0\uad6d\uc73c\ub85c \ud65c\ub3d9\ud558\uace0 \uc788\ub2e4.", "paragraph_answer": "\ub300\ud55c\ubbfc\uad6d\uc740 \ud55c\uad6d \uc804\uc7c1 \uc774\ub798 \uc77c\uba85 '\ud55c\uac15\uc758 \uae30\uc801'\uc774\ub77c\uace0 \ubd88\ub9ac\ub294 \ub192\uc740 \uacbd\uc81c \ubc1c\uc804\uc744 \uc774\ub8e9\ud558\uba70, 1990\ub144\ub300\uc5d0 \uc774\ub974\ub7ec \uc138\uacc4\uc801\uc778 \uacbd\uc81c \uac15\uad6d\uc73c\ub85c \ubc1c\uc804\ud558\uc600\ub2e4. 2015\ub144 \uad6c\ub9e4\ub825 \uae30\uc900 1\uc778\ub2f9 \uad6d\ubbfc \ucd1d\uc18c\ub4dd(GDP)\uc740 36,601\ub2ec\ub7ec\ub85c \uc138\uacc4\uc740\ud589\uc5d0\uc11c \uace0\uc18c\ub4dd \uad6d\uac00\ub85c \ubd84\ub958\ub418\uc5c8\uace0, 2016\ub144 \uc720\uc5d4\uc758 \uc778\uac04 \uac1c\ubc1c \uc9c0\uc218(HDI) \uc870\uc0ac\uc5d0\uc11c \uc138\uacc4 18\uc704\ub85c '\ub9e4\uc6b0 \ub192\uc74c'\uc73c\ub85c \ubd84\ub958\ub418\uc5c8\ub2e4. \ub610\ud55c, \uad6d\uc81c \ud1b5\ud654 \uae30\uae08(IMF)\uc5d0\uc11c\ub294 \ub300\ud55c\ubbfc\uad6d\uc744 \uc120\uc9c4 \uacbd\uc81c\uad6d\uc73c\ub85c \ubd84\ub958\ud558\uace0 \uc788\ub2e4. \ub300\ud55c\ubbfc\uad6d\uc758 \uba85\ubaa9 \uad6d\ub0b4 \ucd1d\uc0dd\uc0b0(GDP)\uc740 2016\ub144 1\uc870 4112\uc5b5 \ub2ec\ub7ec \uc774\ub2e4. \ub610\ud55c, \ub300\ud55c\ubbfc\uad6d\uc740 \uc8fc\uc694 20\uac1c\uad6d(G20), \uacbd\uc81c \ud611\ub825 \uac1c\ubc1c \uae30\uad6c(OECD), \uac1c\ubc1c \uc6d0\uc870 \uc704\uc6d0\ud68c(DAC), \ud30c\ub9ac \ud074\ub7fd\uacfc \uac19\uc740 \uae30\uad6c\uc5d0\uc11c \ud68c\uc6d0\uad6d\uc73c\ub85c \ud65c\ub3d9\ud558\uace0 \uc788\ub2e4.", "sentence_answer": "\ub300\ud55c\ubbfc\uad6d\uc758 \uba85\ubaa9 \uad6d\ub0b4 \ucd1d\uc0dd\uc0b0(GDP)\uc740 2016\ub144 1\uc870 4112\uc5b5 \ub2ec\ub7ec \uc774\ub2e4.", "paragraph_id": "6031861-1-2", "paragraph_question": "question: \ub300\ud55c\ubbfc\uad6d\uc758 \uba85\ubaa9 \uad6d\ub0b4 \ucd1d\uc0dd\uc0b0\uc740 2016\ub144 \uae30\uc900\uc73c\ub85c \uc5bc\ub9c8\uc778\uac00?, context: \ub300\ud55c\ubbfc\uad6d\uc740 \ud55c\uad6d \uc804\uc7c1 \uc774\ub798 \uc77c\uba85 '\ud55c\uac15\uc758 \uae30\uc801'\uc774\ub77c\uace0 \ubd88\ub9ac\ub294 \ub192\uc740 \uacbd\uc81c \ubc1c\uc804\uc744 \uc774\ub8e9\ud558\uba70, 1990\ub144\ub300\uc5d0 \uc774\ub974\ub7ec \uc138\uacc4\uc801\uc778 \uacbd\uc81c \uac15\uad6d\uc73c\ub85c \ubc1c\uc804\ud558\uc600\ub2e4. 2015\ub144 \uad6c\ub9e4\ub825 \uae30\uc900 1\uc778\ub2f9 \uad6d\ubbfc \ucd1d\uc18c\ub4dd(GDP)\uc740 36,601\ub2ec\ub7ec\ub85c \uc138\uacc4\uc740\ud589\uc5d0\uc11c \uace0\uc18c\ub4dd \uad6d\uac00\ub85c \ubd84\ub958\ub418\uc5c8\uace0, 2016\ub144 \uc720\uc5d4\uc758 \uc778\uac04 \uac1c\ubc1c \uc9c0\uc218(HDI) \uc870\uc0ac\uc5d0\uc11c \uc138\uacc4 18\uc704\ub85c '\ub9e4\uc6b0 \ub192\uc74c'\uc73c\ub85c \ubd84\ub958\ub418\uc5c8\ub2e4. \ub610\ud55c, \uad6d\uc81c \ud1b5\ud654 \uae30\uae08(IMF)\uc5d0\uc11c\ub294 \ub300\ud55c\ubbfc\uad6d\uc744 \uc120\uc9c4 \uacbd\uc81c\uad6d\uc73c\ub85c \ubd84\ub958\ud558\uace0 \uc788\ub2e4. \ub300\ud55c\ubbfc\uad6d\uc758 \uba85\ubaa9 \uad6d\ub0b4 \ucd1d\uc0dd\uc0b0(GDP)\uc740 2016\ub144 1\uc870 4112\uc5b5 \ub2ec\ub7ec\uc774\ub2e4. \ub610\ud55c, \ub300\ud55c\ubbfc\uad6d\uc740 \uc8fc\uc694 20\uac1c\uad6d(G20), \uacbd\uc81c \ud611\ub825 \uac1c\ubc1c \uae30\uad6c(OECD), \uac1c\ubc1c \uc6d0\uc870 \uc704\uc6d0\ud68c(DAC), \ud30c\ub9ac \ud074\ub7fd\uacfc \uac19\uc740 \uae30\uad6c\uc5d0\uc11c \ud68c\uc6d0\uad6d\uc73c\ub85c \ud65c\ub3d9\ud558\uace0 \uc788\ub2e4."} {"question": "\uce98\ub9ac\ud3ec\ub2c8\uc544 \uac00\ub3c4\ub294 \uc8fc\ub85c \uc5b4\ub290 \uacc4\uc808\uc5d0 \ub9ce\uc774 \uc774\uc6a9\ub418\uc5c8\ub294\uac00?", "paragraph": "\uce98\ub9ac\ud3ec\ub2c8\uc544 \uac00\ub3c4\uc758 \uacbd\ub85c\ub294 1829\ub144\ubd80\ud130 1840\ub144\uc5d0 \uac78\uccd0, \ud0a4\ud2b8 \uce74\uc2a8\uacfc \uc870\uc9c0\ud504 \ub808\ub4dc \ud3ec\ub4dc \uc6cc\ucee4, \uc81c\ub370\ub514\uc544 \uc2a4\ubbf8\uc2a4 \ub4f1 \ubbf8\uad6d\uc778 \ubaa8\ud53c \ubb34\uc5ed\uc0c1\ub4e4, \uc2ec\uc9c0\uc5b4 \ud53c\ud130 \uc2a4\ud0a8 \uc624\uadf8\ub374\uc774 \uc774\ub044\ub294 \ud5c8\ub4dc\uc2a8 \ubca0\uc774 \ud68c\uc0ac\uc758 \ub36b\uc744 \ub193\ub294 \uc0ac\ub78c\ub4e4\uc5d0 \uc758\ud574 \ubd80\ubd84\uc801\uc73c\ub85c \ubc1c\uacac\ub418\uc5b4 \uc788\uc5c8\ub2e4. 1841\ub144\uc5d0\uc11c 1844\ub144 \uacbd\uc5d0\ub294 \ud6d4\ubcfc\ud2b8 \uac15 (\ub2f9\uc2dc\ub294 \uc624\uadf8\ub374\uc774 \uba54\ub9ac \uac15\uc774\ub77c\uace0 \ubd88\ub800\ub2e4)\uc744 \ub530\ub77c \uc2dc\uc5d0\ub77c \ub124\ubc14\ub2e4 \uc0b0\ub9e5 \ub108\uba38 \uc0ac\uc6a9\ud560 \uc218 \uc788\ub294 \uac70\uce5c \ub9c8\ucc3b\uae38\uc774 \uce98\ub9ac\ud3ec\ub2c8\uc544\ub85c \ud5a5\ud558\ub294 \uac1c\ucc99\uc790\ub4e4\uc5d0 \uc758\ud574 \uac1c\ucc99\ub418\uc5c8\ub2e4. 1869\ub144\uc5d0 \ucd5c\ucd08\uc758 \ub300\ub959 \ud6a1\ub2e8 \ucca0\ub3c4\uac00 \uac1c\ud1b5\ub420 \ub54c\uae4c\uc9c0 \ud2b9\ud788 \uc5ec\ub984\ucca0\uc5d0 \uc774 \uae38\uc740 \ub9e4\uc6b0 \uc774\uc6a9\ub418\uc5c8\ub2e4. \ucd08\uae30\uc758 \uae38\uc740 \ub9ce\uc740 \uac00\uc9d3 \uae38\uacfc \ub053\uc5b4\uc9c4 \uae38\uc774 \uc788\uc5c8\uc73c\uba70, \uc774\ub7ec\ud55c \uac83\ub4e4\uc744 \ud569\uce58\uba74 \uc57d 8,800km\uc5d0 \ub2ec\ud558\ub294 \uae38\uc774\uc5c8\ub2e4. \uc774 \ub3c4\ub85c\ub4e4 \uc911 \uc57d 1600km\ub294 \ubc14\ud034\uc790\uad6d\uc774 \ub0a8\uc544 \uc788\uc73c\uba70, \uce94\uc790\uc2a4, \ub124\ube0c\ub798\uc2a4\uce74, \uc640\uc774\uc624\ubc0d, \uc544\uc774\ub2e4\ud638, \uc720\ud0c0, \ub124\ubc14\ub2e4\uc640 \uce98\ub9ac\ud3ec\ub2c8\uc544\uc5d0 \uac78\uccd0 \ub300\uaddc\ubaa8 \uc11c\ubd80 \uc774\ubbfc\uc774 \uc788\uc5c8\ub358 \uc5ed\uc0ac\uc801 \uc99d\uac70\uac00 \uc788\ub2e4. \ud604\uc7ac \uc774 \ub3c4\ub85c\uc758 \uc77c\ubd80\ub294 \"\uce98\ub9ac\ud3ec\ub2c8\uc544 \uad6d\ub9bd \uc5ed\uc0ac \uac00\ub3c4'\ub85c \ud1a0\uc9c0 \uad00\ub9ac\uad6d\uacfc \ubbf8\uad6d\uad6d\ub9bd\uacf5\uc6d0\uad6d\uc5d0 \uc758\ud574 \uc9c0\uc815\ub418\uc5b4 \uc788\ub2e4.", "answer": "\uc5ec\ub984\ucca0", "sentence": "1869\ub144\uc5d0 \ucd5c\ucd08\uc758 \ub300\ub959 \ud6a1\ub2e8 \ucca0\ub3c4\uac00 \uac1c\ud1b5\ub420 \ub54c\uae4c\uc9c0 \ud2b9\ud788 \uc5ec\ub984\ucca0 \uc5d0 \uc774 \uae38\uc740 \ub9e4\uc6b0 \uc774\uc6a9\ub418\uc5c8\ub2e4.", "paragraph_sentence": "\uce98\ub9ac\ud3ec\ub2c8\uc544 \uac00\ub3c4\uc758 \uacbd\ub85c\ub294 1829\ub144\ubd80\ud130 1840\ub144\uc5d0 \uac78\uccd0, \ud0a4\ud2b8 \uce74\uc2a8\uacfc \uc870\uc9c0\ud504 \ub808\ub4dc \ud3ec\ub4dc \uc6cc\ucee4, \uc81c\ub370\ub514\uc544 \uc2a4\ubbf8\uc2a4 \ub4f1 \ubbf8\uad6d\uc778 \ubaa8\ud53c \ubb34\uc5ed\uc0c1\ub4e4, \uc2ec\uc9c0\uc5b4 \ud53c\ud130 \uc2a4\ud0a8 \uc624\uadf8\ub374\uc774 \uc774\ub044\ub294 \ud5c8\ub4dc\uc2a8 \ubca0\uc774 \ud68c\uc0ac\uc758 \ub36b\uc744 \ub193\ub294 \uc0ac\ub78c\ub4e4\uc5d0 \uc758\ud574 \ubd80\ubd84\uc801\uc73c\ub85c \ubc1c\uacac\ub418\uc5b4 \uc788\uc5c8\ub2e4. 1841\ub144\uc5d0\uc11c 1844\ub144 \uacbd\uc5d0\ub294 \ud6d4\ubcfc\ud2b8 \uac15 (\ub2f9\uc2dc\ub294 \uc624\uadf8\ub374\uc774 \uba54\ub9ac \uac15\uc774\ub77c\uace0 \ubd88\ub800\ub2e4)\uc744 \ub530\ub77c \uc2dc\uc5d0\ub77c \ub124\ubc14\ub2e4 \uc0b0\ub9e5 \ub108\uba38 \uc0ac\uc6a9\ud560 \uc218 \uc788\ub294 \uac70\uce5c \ub9c8\ucc3b\uae38\uc774 \uce98\ub9ac\ud3ec\ub2c8\uc544\ub85c \ud5a5\ud558\ub294 \uac1c\ucc99\uc790\ub4e4\uc5d0 \uc758\ud574 \uac1c\ucc99\ub418\uc5c8\ub2e4. 1869\ub144\uc5d0 \ucd5c\ucd08\uc758 \ub300\ub959 \ud6a1\ub2e8 \ucca0\ub3c4\uac00 \uac1c\ud1b5\ub420 \ub54c\uae4c\uc9c0 \ud2b9\ud788 \uc5ec\ub984\ucca0 \uc5d0 \uc774 \uae38\uc740 \ub9e4\uc6b0 \uc774\uc6a9\ub418\uc5c8\ub2e4. \ucd08\uae30\uc758 \uae38\uc740 \ub9ce\uc740 \uac00\uc9d3 \uae38\uacfc \ub053\uc5b4\uc9c4 \uae38\uc774 \uc788\uc5c8\uc73c\uba70, \uc774\ub7ec\ud55c \uac83\ub4e4\uc744 \ud569\uce58\uba74 \uc57d 8,800km\uc5d0 \ub2ec\ud558\ub294 \uae38\uc774\uc5c8\ub2e4. \uc774 \ub3c4\ub85c\ub4e4 \uc911 \uc57d 1600km\ub294 \ubc14\ud034\uc790\uad6d\uc774 \ub0a8\uc544 \uc788\uc73c\uba70, \uce94\uc790\uc2a4, \ub124\ube0c\ub798\uc2a4\uce74, \uc640\uc774\uc624\ubc0d, \uc544\uc774\ub2e4\ud638, \uc720\ud0c0, \ub124\ubc14\ub2e4\uc640 \uce98\ub9ac\ud3ec\ub2c8\uc544\uc5d0 \uac78\uccd0 \ub300\uaddc\ubaa8 \uc11c\ubd80 \uc774\ubbfc\uc774 \uc788\uc5c8\ub358 \uc5ed\uc0ac\uc801 \uc99d\uac70\uac00 \uc788\ub2e4. \ud604\uc7ac \uc774 \ub3c4\ub85c\uc758 \uc77c\ubd80\ub294 \"\uce98\ub9ac\ud3ec\ub2c8\uc544 \uad6d\ub9bd \uc5ed\uc0ac \uac00\ub3c4'\ub85c \ud1a0\uc9c0 \uad00\ub9ac\uad6d\uacfc \ubbf8\uad6d\uad6d\ub9bd\uacf5\uc6d0\uad6d\uc5d0 \uc758\ud574 \uc9c0\uc815\ub418\uc5b4 \uc788\ub2e4.", "paragraph_answer": "\uce98\ub9ac\ud3ec\ub2c8\uc544 \uac00\ub3c4\uc758 \uacbd\ub85c\ub294 1829\ub144\ubd80\ud130 1840\ub144\uc5d0 \uac78\uccd0, \ud0a4\ud2b8 \uce74\uc2a8\uacfc \uc870\uc9c0\ud504 \ub808\ub4dc \ud3ec\ub4dc \uc6cc\ucee4, \uc81c\ub370\ub514\uc544 \uc2a4\ubbf8\uc2a4 \ub4f1 \ubbf8\uad6d\uc778 \ubaa8\ud53c \ubb34\uc5ed\uc0c1\ub4e4, \uc2ec\uc9c0\uc5b4 \ud53c\ud130 \uc2a4\ud0a8 \uc624\uadf8\ub374\uc774 \uc774\ub044\ub294 \ud5c8\ub4dc\uc2a8 \ubca0\uc774 \ud68c\uc0ac\uc758 \ub36b\uc744 \ub193\ub294 \uc0ac\ub78c\ub4e4\uc5d0 \uc758\ud574 \ubd80\ubd84\uc801\uc73c\ub85c \ubc1c\uacac\ub418\uc5b4 \uc788\uc5c8\ub2e4. 1841\ub144\uc5d0\uc11c 1844\ub144 \uacbd\uc5d0\ub294 \ud6d4\ubcfc\ud2b8 \uac15 (\ub2f9\uc2dc\ub294 \uc624\uadf8\ub374\uc774 \uba54\ub9ac \uac15\uc774\ub77c\uace0 \ubd88\ub800\ub2e4)\uc744 \ub530\ub77c \uc2dc\uc5d0\ub77c \ub124\ubc14\ub2e4 \uc0b0\ub9e5 \ub108\uba38 \uc0ac\uc6a9\ud560 \uc218 \uc788\ub294 \uac70\uce5c \ub9c8\ucc3b\uae38\uc774 \uce98\ub9ac\ud3ec\ub2c8\uc544\ub85c \ud5a5\ud558\ub294 \uac1c\ucc99\uc790\ub4e4\uc5d0 \uc758\ud574 \uac1c\ucc99\ub418\uc5c8\ub2e4. 1869\ub144\uc5d0 \ucd5c\ucd08\uc758 \ub300\ub959 \ud6a1\ub2e8 \ucca0\ub3c4\uac00 \uac1c\ud1b5\ub420 \ub54c\uae4c\uc9c0 \ud2b9\ud788 \uc5ec\ub984\ucca0 \uc5d0 \uc774 \uae38\uc740 \ub9e4\uc6b0 \uc774\uc6a9\ub418\uc5c8\ub2e4. \ucd08\uae30\uc758 \uae38\uc740 \ub9ce\uc740 \uac00\uc9d3 \uae38\uacfc \ub053\uc5b4\uc9c4 \uae38\uc774 \uc788\uc5c8\uc73c\uba70, \uc774\ub7ec\ud55c \uac83\ub4e4\uc744 \ud569\uce58\uba74 \uc57d 8,800km\uc5d0 \ub2ec\ud558\ub294 \uae38\uc774\uc5c8\ub2e4. \uc774 \ub3c4\ub85c\ub4e4 \uc911 \uc57d 1600km\ub294 \ubc14\ud034\uc790\uad6d\uc774 \ub0a8\uc544 \uc788\uc73c\uba70, \uce94\uc790\uc2a4, \ub124\ube0c\ub798\uc2a4\uce74, \uc640\uc774\uc624\ubc0d, \uc544\uc774\ub2e4\ud638, \uc720\ud0c0, \ub124\ubc14\ub2e4\uc640 \uce98\ub9ac\ud3ec\ub2c8\uc544\uc5d0 \uac78\uccd0 \ub300\uaddc\ubaa8 \uc11c\ubd80 \uc774\ubbfc\uc774 \uc788\uc5c8\ub358 \uc5ed\uc0ac\uc801 \uc99d\uac70\uac00 \uc788\ub2e4. \ud604\uc7ac \uc774 \ub3c4\ub85c\uc758 \uc77c\ubd80\ub294 \"\uce98\ub9ac\ud3ec\ub2c8\uc544 \uad6d\ub9bd \uc5ed\uc0ac \uac00\ub3c4'\ub85c \ud1a0\uc9c0 \uad00\ub9ac\uad6d\uacfc \ubbf8\uad6d\uad6d\ub9bd\uacf5\uc6d0\uad6d\uc5d0 \uc758\ud574 \uc9c0\uc815\ub418\uc5b4 \uc788\ub2e4.", "sentence_answer": "1869\ub144\uc5d0 \ucd5c\ucd08\uc758 \ub300\ub959 \ud6a1\ub2e8 \ucca0\ub3c4\uac00 \uac1c\ud1b5\ub420 \ub54c\uae4c\uc9c0 \ud2b9\ud788 \uc5ec\ub984\ucca0 \uc5d0 \uc774 \uae38\uc740 \ub9e4\uc6b0 \uc774\uc6a9\ub418\uc5c8\ub2e4.", "paragraph_id": "6209453-2-0", "paragraph_question": "question: \uce98\ub9ac\ud3ec\ub2c8\uc544 \uac00\ub3c4\ub294 \uc8fc\ub85c \uc5b4\ub290 \uacc4\uc808\uc5d0 \ub9ce\uc774 \uc774\uc6a9\ub418\uc5c8\ub294\uac00?, context: \uce98\ub9ac\ud3ec\ub2c8\uc544 \uac00\ub3c4\uc758 \uacbd\ub85c\ub294 1829\ub144\ubd80\ud130 1840\ub144\uc5d0 \uac78\uccd0, \ud0a4\ud2b8 \uce74\uc2a8\uacfc \uc870\uc9c0\ud504 \ub808\ub4dc \ud3ec\ub4dc \uc6cc\ucee4, \uc81c\ub370\ub514\uc544 \uc2a4\ubbf8\uc2a4 \ub4f1 \ubbf8\uad6d\uc778 \ubaa8\ud53c \ubb34\uc5ed\uc0c1\ub4e4, \uc2ec\uc9c0\uc5b4 \ud53c\ud130 \uc2a4\ud0a8 \uc624\uadf8\ub374\uc774 \uc774\ub044\ub294 \ud5c8\ub4dc\uc2a8 \ubca0\uc774 \ud68c\uc0ac\uc758 \ub36b\uc744 \ub193\ub294 \uc0ac\ub78c\ub4e4\uc5d0 \uc758\ud574 \ubd80\ubd84\uc801\uc73c\ub85c \ubc1c\uacac\ub418\uc5b4 \uc788\uc5c8\ub2e4. 1841\ub144\uc5d0\uc11c 1844\ub144 \uacbd\uc5d0\ub294 \ud6d4\ubcfc\ud2b8 \uac15 (\ub2f9\uc2dc\ub294 \uc624\uadf8\ub374\uc774 \uba54\ub9ac \uac15\uc774\ub77c\uace0 \ubd88\ub800\ub2e4)\uc744 \ub530\ub77c \uc2dc\uc5d0\ub77c \ub124\ubc14\ub2e4 \uc0b0\ub9e5 \ub108\uba38 \uc0ac\uc6a9\ud560 \uc218 \uc788\ub294 \uac70\uce5c \ub9c8\ucc3b\uae38\uc774 \uce98\ub9ac\ud3ec\ub2c8\uc544\ub85c \ud5a5\ud558\ub294 \uac1c\ucc99\uc790\ub4e4\uc5d0 \uc758\ud574 \uac1c\ucc99\ub418\uc5c8\ub2e4. 1869\ub144\uc5d0 \ucd5c\ucd08\uc758 \ub300\ub959 \ud6a1\ub2e8 \ucca0\ub3c4\uac00 \uac1c\ud1b5\ub420 \ub54c\uae4c\uc9c0 \ud2b9\ud788 \uc5ec\ub984\ucca0\uc5d0 \uc774 \uae38\uc740 \ub9e4\uc6b0 \uc774\uc6a9\ub418\uc5c8\ub2e4. \ucd08\uae30\uc758 \uae38\uc740 \ub9ce\uc740 \uac00\uc9d3 \uae38\uacfc \ub053\uc5b4\uc9c4 \uae38\uc774 \uc788\uc5c8\uc73c\uba70, \uc774\ub7ec\ud55c \uac83\ub4e4\uc744 \ud569\uce58\uba74 \uc57d 8,800km\uc5d0 \ub2ec\ud558\ub294 \uae38\uc774\uc5c8\ub2e4. \uc774 \ub3c4\ub85c\ub4e4 \uc911 \uc57d 1600km\ub294 \ubc14\ud034\uc790\uad6d\uc774 \ub0a8\uc544 \uc788\uc73c\uba70, \uce94\uc790\uc2a4, \ub124\ube0c\ub798\uc2a4\uce74, \uc640\uc774\uc624\ubc0d, \uc544\uc774\ub2e4\ud638, \uc720\ud0c0, \ub124\ubc14\ub2e4\uc640 \uce98\ub9ac\ud3ec\ub2c8\uc544\uc5d0 \uac78\uccd0 \ub300\uaddc\ubaa8 \uc11c\ubd80 \uc774\ubbfc\uc774 \uc788\uc5c8\ub358 \uc5ed\uc0ac\uc801 \uc99d\uac70\uac00 \uc788\ub2e4. \ud604\uc7ac \uc774 \ub3c4\ub85c\uc758 \uc77c\ubd80\ub294 \"\uce98\ub9ac\ud3ec\ub2c8\uc544 \uad6d\ub9bd \uc5ed\uc0ac \uac00\ub3c4'\ub85c \ud1a0\uc9c0 \uad00\ub9ac\uad6d\uacfc \ubbf8\uad6d\uad6d\ub9bd\uacf5\uc6d0\uad6d\uc5d0 \uc758\ud574 \uc9c0\uc815\ub418\uc5b4 \uc788\ub2e4."} {"question": "\uc5ec\uc790\uce5c\uad6c\uc758 \ubc24\uc774 \ube4c\ubcf4\ub4dc \uc568\ubc94\ucc28\ud2b8\uc5d0 \uc624\ub978 \uc21c\uc704\ub294?", "paragraph": "4\uc6d4 30\uc77c \uc5ec\uc12f \ubc88\uc9f8 \ubbf8\ub2c8 \uc568\ubc94 \u300aTime for the moon night\u300b\uc744 \ubc1c\ub9e4\ud558\uc600\ub2e4. \ud0c0\uc774\ud2c0 \uace1\uc740 \uc3d8\uc2a4\ubba4\uc9c1\uc758 \ud558\uc6b0\uc2a4 \ud504\ub85c\ub4c0\uc11c \ub178\uc8fc\ud658\uc774 \uc774\uc6d0\uc885\uacfc \uc791\uc5c5\ud55c \u201c\ubc24\u201d\uc73c\ub85c, \uc0ac\ub791\ud558\ub294 \uc774\ub97c \uc0dd\uac01\ud558\uba70 \uace0\ubbfc\uc5d0 \uc816\uc5b4\ub4dc\ub294 \ubc24\ubd80\ud130 \uc0c8\ubcbd\uae4c\uc9c0\uc758 \uc2dc\uac04\uc744 \uc11c\uc815\uc801\uc73c\ub85c \ud45c\ud604\ud55c \uace1\uc774\ub2e4. \u201c\ubc24\u201d\uc740 \uac00\uc628 \uc8fc\uac04 \ub2e4\uc6b4\ub85c\ub4dc \ucc28\ud2b8, \uc568\ubc94 \ucc28\ud2b8, \uc18c\uc15c \ucc28\ud2b8\uc5d0\uc11c 1\uc704\ub97c \uae30\ub85d\ud558\uc600\uc744 \ubfd0 \uc544\ub2c8\ub77c \ube4c\ubcf4\ub4dc \uc568\ubc94\ucc28\ud2b8 6\uc704\ub97c \ucc28\uc9c0\ud558\uc600\uc73c\uba70, \ub300\ud55c\ubbfc\uad6d\uc5d0\uc11c\ub294 \uba5c\ub860, \ubc85\uc2a4, \uc18c\ub9ac\ubc14\ub2e4, \uc9c0\ub2c8, \ub124\uc774\ubc84 \ub4f1 5\uac1c\uc758 \ucc28\ud2b8\uc5d0\uc11c \uc2e4\uc2dc\uac04 1\uc704\uc5d0 \uc62c\ub790\uace0, \ubba4\uc9c1\ube44\ub514\uc624\ub294 \uacf5\uac1c 1\uc8fc\uc77c \ub9cc\uc5d0 \uc870\ud68c\uc218 1300\ub9cc \ud68c\ub97c \ub2ec\uc131\ud558\uc600\ub2e4. 5\uc6d4 3\uc77c \uc5e0\uce74\uc6b4\ud2b8\ub2e4\uc6b4\uc5d0\uc11c \uccab \ucef4\ubc31 \ubb34\ub300\ub97c \uc120\ubcf4\uc778 \uc774\ud6c4, 5\uc6d4 8\uc77c \ub354 \uc1fc, 5\uc6d4 9\uc77c \uc1fc \ucc54\ud53c\uc5b8, 5\uc6d4 10\uc77c \uc5e0\uce74\uc6b4\ud2b8\ub2e4\uc6b4, 5\uc6d4 11\uc77c \ubba4\uc9c1\ubc45\ud06c, 5\uc6d4 12\uc77c \uc74c\uc545\uc911\uc2ec\uc5d0\uc11c 1\uc704\uc5d0 \uc62c\ub790\ub2e4.", "answer": "6\uc704", "sentence": "\u201c\ubc24\u201d\uc740 \uac00\uc628 \uc8fc\uac04 \ub2e4\uc6b4\ub85c\ub4dc \ucc28\ud2b8, \uc568\ubc94 \ucc28\ud2b8, \uc18c\uc15c \ucc28\ud2b8\uc5d0\uc11c 1\uc704\ub97c \uae30\ub85d\ud558\uc600\uc744 \ubfd0 \uc544\ub2c8\ub77c \ube4c\ubcf4\ub4dc \uc568\ubc94\ucc28\ud2b8 6\uc704 \ub97c \ucc28\uc9c0\ud558\uc600\uc73c\uba70, \ub300\ud55c\ubbfc\uad6d\uc5d0\uc11c\ub294 \uba5c\ub860, \ubc85\uc2a4, \uc18c\ub9ac\ubc14\ub2e4, \uc9c0\ub2c8, \ub124\uc774\ubc84 \ub4f1 5\uac1c\uc758 \ucc28\ud2b8\uc5d0\uc11c \uc2e4\uc2dc\uac04 1\uc704\uc5d0 \uc62c\ub790\uace0, \ubba4\uc9c1\ube44\ub514\uc624\ub294 \uacf5\uac1c 1\uc8fc\uc77c \ub9cc\uc5d0 \uc870\ud68c\uc218 1300\ub9cc \ud68c\ub97c \ub2ec\uc131\ud558\uc600\ub2e4.", "paragraph_sentence": "4\uc6d4 30\uc77c \uc5ec\uc12f \ubc88\uc9f8 \ubbf8\ub2c8 \uc568\ubc94 \u300aTime for the moon night\u300b\uc744 \ubc1c\ub9e4\ud558\uc600\ub2e4. \ud0c0\uc774\ud2c0 \uace1\uc740 \uc3d8\uc2a4\ubba4\uc9c1\uc758 \ud558\uc6b0\uc2a4 \ud504\ub85c\ub4c0\uc11c \ub178\uc8fc\ud658\uc774 \uc774\uc6d0\uc885\uacfc \uc791\uc5c5\ud55c \u201c\ubc24\u201d\uc73c\ub85c, \uc0ac\ub791\ud558\ub294 \uc774\ub97c \uc0dd\uac01\ud558\uba70 \uace0\ubbfc\uc5d0 \uc816\uc5b4\ub4dc\ub294 \ubc24\ubd80\ud130 \uc0c8\ubcbd\uae4c\uc9c0\uc758 \uc2dc\uac04\uc744 \uc11c\uc815\uc801\uc73c\ub85c \ud45c\ud604\ud55c \uace1\uc774\ub2e4. \u201c\ubc24\u201d\uc740 \uac00\uc628 \uc8fc\uac04 \ub2e4\uc6b4\ub85c\ub4dc \ucc28\ud2b8, \uc568\ubc94 \ucc28\ud2b8, \uc18c\uc15c \ucc28\ud2b8\uc5d0\uc11c 1\uc704\ub97c \uae30\ub85d\ud558\uc600\uc744 \ubfd0 \uc544\ub2c8\ub77c \ube4c\ubcf4\ub4dc \uc568\ubc94\ucc28\ud2b8 6\uc704 \ub97c \ucc28\uc9c0\ud558\uc600\uc73c\uba70, \ub300\ud55c\ubbfc\uad6d\uc5d0\uc11c\ub294 \uba5c\ub860, \ubc85\uc2a4, \uc18c\ub9ac\ubc14\ub2e4, \uc9c0\ub2c8, \ub124\uc774\ubc84 \ub4f1 5\uac1c\uc758 \ucc28\ud2b8\uc5d0\uc11c \uc2e4\uc2dc\uac04 1\uc704\uc5d0 \uc62c\ub790\uace0, \ubba4\uc9c1\ube44\ub514\uc624\ub294 \uacf5\uac1c 1\uc8fc\uc77c \ub9cc\uc5d0 \uc870\ud68c\uc218 1300\ub9cc \ud68c\ub97c \ub2ec\uc131\ud558\uc600\ub2e4. 5\uc6d4 3\uc77c \uc5e0\uce74\uc6b4\ud2b8\ub2e4\uc6b4\uc5d0\uc11c \uccab \ucef4\ubc31 \ubb34\ub300\ub97c \uc120\ubcf4\uc778 \uc774\ud6c4, 5\uc6d4 8\uc77c \ub354 \uc1fc, 5\uc6d4 9\uc77c \uc1fc \ucc54\ud53c\uc5b8, 5\uc6d4 10\uc77c \uc5e0\uce74\uc6b4\ud2b8\ub2e4\uc6b4, 5\uc6d4 11\uc77c \ubba4\uc9c1\ubc45\ud06c, 5\uc6d4 12\uc77c \uc74c\uc545\uc911\uc2ec\uc5d0\uc11c 1\uc704\uc5d0 \uc62c\ub790\ub2e4.", "paragraph_answer": "4\uc6d4 30\uc77c \uc5ec\uc12f \ubc88\uc9f8 \ubbf8\ub2c8 \uc568\ubc94 \u300aTime for the moon night\u300b\uc744 \ubc1c\ub9e4\ud558\uc600\ub2e4. \ud0c0\uc774\ud2c0 \uace1\uc740 \uc3d8\uc2a4\ubba4\uc9c1\uc758 \ud558\uc6b0\uc2a4 \ud504\ub85c\ub4c0\uc11c \ub178\uc8fc\ud658\uc774 \uc774\uc6d0\uc885\uacfc \uc791\uc5c5\ud55c \u201c\ubc24\u201d\uc73c\ub85c, \uc0ac\ub791\ud558\ub294 \uc774\ub97c \uc0dd\uac01\ud558\uba70 \uace0\ubbfc\uc5d0 \uc816\uc5b4\ub4dc\ub294 \ubc24\ubd80\ud130 \uc0c8\ubcbd\uae4c\uc9c0\uc758 \uc2dc\uac04\uc744 \uc11c\uc815\uc801\uc73c\ub85c \ud45c\ud604\ud55c \uace1\uc774\ub2e4. \u201c\ubc24\u201d\uc740 \uac00\uc628 \uc8fc\uac04 \ub2e4\uc6b4\ub85c\ub4dc \ucc28\ud2b8, \uc568\ubc94 \ucc28\ud2b8, \uc18c\uc15c \ucc28\ud2b8\uc5d0\uc11c 1\uc704\ub97c \uae30\ub85d\ud558\uc600\uc744 \ubfd0 \uc544\ub2c8\ub77c \ube4c\ubcf4\ub4dc \uc568\ubc94\ucc28\ud2b8 6\uc704 \ub97c \ucc28\uc9c0\ud558\uc600\uc73c\uba70, \ub300\ud55c\ubbfc\uad6d\uc5d0\uc11c\ub294 \uba5c\ub860, \ubc85\uc2a4, \uc18c\ub9ac\ubc14\ub2e4, \uc9c0\ub2c8, \ub124\uc774\ubc84 \ub4f1 5\uac1c\uc758 \ucc28\ud2b8\uc5d0\uc11c \uc2e4\uc2dc\uac04 1\uc704\uc5d0 \uc62c\ub790\uace0, \ubba4\uc9c1\ube44\ub514\uc624\ub294 \uacf5\uac1c 1\uc8fc\uc77c \ub9cc\uc5d0 \uc870\ud68c\uc218 1300\ub9cc \ud68c\ub97c \ub2ec\uc131\ud558\uc600\ub2e4. 5\uc6d4 3\uc77c \uc5e0\uce74\uc6b4\ud2b8\ub2e4\uc6b4\uc5d0\uc11c \uccab \ucef4\ubc31 \ubb34\ub300\ub97c \uc120\ubcf4\uc778 \uc774\ud6c4, 5\uc6d4 8\uc77c \ub354 \uc1fc, 5\uc6d4 9\uc77c \uc1fc \ucc54\ud53c\uc5b8, 5\uc6d4 10\uc77c \uc5e0\uce74\uc6b4\ud2b8\ub2e4\uc6b4, 5\uc6d4 11\uc77c \ubba4\uc9c1\ubc45\ud06c, 5\uc6d4 12\uc77c \uc74c\uc545\uc911\uc2ec\uc5d0\uc11c 1\uc704\uc5d0 \uc62c\ub790\ub2e4.", "sentence_answer": "\u201c\ubc24\u201d\uc740 \uac00\uc628 \uc8fc\uac04 \ub2e4\uc6b4\ub85c\ub4dc \ucc28\ud2b8, \uc568\ubc94 \ucc28\ud2b8, \uc18c\uc15c \ucc28\ud2b8\uc5d0\uc11c 1\uc704\ub97c \uae30\ub85d\ud558\uc600\uc744 \ubfd0 \uc544\ub2c8\ub77c \ube4c\ubcf4\ub4dc \uc568\ubc94\ucc28\ud2b8 6\uc704 \ub97c \ucc28\uc9c0\ud558\uc600\uc73c\uba70, \ub300\ud55c\ubbfc\uad6d\uc5d0\uc11c\ub294 \uba5c\ub860, \ubc85\uc2a4, \uc18c\ub9ac\ubc14\ub2e4, \uc9c0\ub2c8, \ub124\uc774\ubc84 \ub4f1 5\uac1c\uc758 \ucc28\ud2b8\uc5d0\uc11c \uc2e4\uc2dc\uac04 1\uc704\uc5d0 \uc62c\ub790\uace0, \ubba4\uc9c1\ube44\ub514\uc624\ub294 \uacf5\uac1c 1\uc8fc\uc77c \ub9cc\uc5d0 \uc870\ud68c\uc218 1300\ub9cc \ud68c\ub97c \ub2ec\uc131\ud558\uc600\ub2e4.", "paragraph_id": "6587280-4-1", "paragraph_question": "question: \uc5ec\uc790\uce5c\uad6c\uc758 \ubc24\uc774 \ube4c\ubcf4\ub4dc \uc568\ubc94\ucc28\ud2b8\uc5d0 \uc624\ub978 \uc21c\uc704\ub294?, context: 4\uc6d4 30\uc77c \uc5ec\uc12f \ubc88\uc9f8 \ubbf8\ub2c8 \uc568\ubc94 \u300aTime for the moon night\u300b\uc744 \ubc1c\ub9e4\ud558\uc600\ub2e4. \ud0c0\uc774\ud2c0 \uace1\uc740 \uc3d8\uc2a4\ubba4\uc9c1\uc758 \ud558\uc6b0\uc2a4 \ud504\ub85c\ub4c0\uc11c \ub178\uc8fc\ud658\uc774 \uc774\uc6d0\uc885\uacfc \uc791\uc5c5\ud55c \u201c\ubc24\u201d\uc73c\ub85c, \uc0ac\ub791\ud558\ub294 \uc774\ub97c \uc0dd\uac01\ud558\uba70 \uace0\ubbfc\uc5d0 \uc816\uc5b4\ub4dc\ub294 \ubc24\ubd80\ud130 \uc0c8\ubcbd\uae4c\uc9c0\uc758 \uc2dc\uac04\uc744 \uc11c\uc815\uc801\uc73c\ub85c \ud45c\ud604\ud55c \uace1\uc774\ub2e4. \u201c\ubc24\u201d\uc740 \uac00\uc628 \uc8fc\uac04 \ub2e4\uc6b4\ub85c\ub4dc \ucc28\ud2b8, \uc568\ubc94 \ucc28\ud2b8, \uc18c\uc15c \ucc28\ud2b8\uc5d0\uc11c 1\uc704\ub97c \uae30\ub85d\ud558\uc600\uc744 \ubfd0 \uc544\ub2c8\ub77c \ube4c\ubcf4\ub4dc \uc568\ubc94\ucc28\ud2b8 6\uc704\ub97c \ucc28\uc9c0\ud558\uc600\uc73c\uba70, \ub300\ud55c\ubbfc\uad6d\uc5d0\uc11c\ub294 \uba5c\ub860, \ubc85\uc2a4, \uc18c\ub9ac\ubc14\ub2e4, \uc9c0\ub2c8, \ub124\uc774\ubc84 \ub4f1 5\uac1c\uc758 \ucc28\ud2b8\uc5d0\uc11c \uc2e4\uc2dc\uac04 1\uc704\uc5d0 \uc62c\ub790\uace0, \ubba4\uc9c1\ube44\ub514\uc624\ub294 \uacf5\uac1c 1\uc8fc\uc77c \ub9cc\uc5d0 \uc870\ud68c\uc218 1300\ub9cc \ud68c\ub97c \ub2ec\uc131\ud558\uc600\ub2e4. 5\uc6d4 3\uc77c \uc5e0\uce74\uc6b4\ud2b8\ub2e4\uc6b4\uc5d0\uc11c \uccab \ucef4\ubc31 \ubb34\ub300\ub97c \uc120\ubcf4\uc778 \uc774\ud6c4, 5\uc6d4 8\uc77c \ub354 \uc1fc, 5\uc6d4 9\uc77c \uc1fc \ucc54\ud53c\uc5b8, 5\uc6d4 10\uc77c \uc5e0\uce74\uc6b4\ud2b8\ub2e4\uc6b4, 5\uc6d4 11\uc77c \ubba4\uc9c1\ubc45\ud06c, 5\uc6d4 12\uc77c \uc74c\uc545\uc911\uc2ec\uc5d0\uc11c 1\uc704\uc5d0 \uc62c\ub790\ub2e4."} {"question": "\uae40\ub300\uc911\uc774 \ucde8\uc5c5\ubc18\uc5d0\uc11c \uc9c4\ub85c\ub97c \ubcc0\uacbd\ud558\uc5ec \uc62e\uae34 \uacf3\uc740?", "paragraph": "\uadf8\ub294 \ub3c5\uc11c\uad11\uc774\uba74\uc11c \uc815\uce58\uc640 \uc601\uc5b4\uc5d0 \uad00\uc2ec\uc774 \ub9ce\uc544 \uc791\ubb38\uacfc \uc5ed\uc0ac \uc131\uc801\uc774 \ub6f0\uc5b4\ub0ac\uc73c\uba70 \uc6c5\ubcc0\uc5d0\ub3c4 \uc18c\uc9c8\uc744 \ubcf4\uc600\ub2e4 \ud55c\ub2e4. \uace0\uad50 \uc2dc\uc808 \ud559\uc801\ubd80\uc5d0\ub294 \uae40\ub300\uc911\uc5d0 \ub300\ud558\uc5ec \"\uc5b8\ubcc0\uc740 \ub298 \uc815\ud655, \uba85\ub8cc\ud558\ub2e4.\"\ub77c\uace0 \uae30\uc7ac\ub418\uc5b4 \uc788\ub2e4. \uc774 \ud0d3\uc778\uc9c0 \ud6d7\ub0a0 \uadf8\ub294 \uc11c\uc6b8\uc5d0\uc11c \uc6c5\ubcc0\ud559\uc6d0\uc744 \uc6b4\uc601\ud558\uae30\ub3c4 \ud558\uc600\ub2e4. \uadf8\ub9ac\uace0 \uc774 \ud559\uc6d0\uc5d0\uc11c \uc815\uce58\uc801 \ub3d9\uc9c0\uc778 \uae40\uc0c1\ud604\uacfc \ub9cc\ub098\uac8c \ub41c\ub2e4. \uadf8\uc758 \ud55c \ub3d9\ucc3d\uc0dd\uc758 \ub9d0\uc5d0 \uc758\ud558\uba74, \ud55c\ubc88\uc740 \uc77c\ubcf8\uc778 \uad50\uad00\uc774 \uc804\uad50\uc0dd\uc744 \ubaa8\uc544 \ub193\uace0 \uc2dc\uad6d\uc5d0 \ub300\ud55c \uac15\uc5f0\uc744 \ud55c \uc77c\uc774 \uc788\uc5c8\ub2e4. \uac15\uc5f0\uc774 \ub05d\ub098\uace0 \uad50\uad00\uc774 \uc9c8\ubb38\uc774 \uc788\uc73c\uba74 \ud558\ub77c\uace0 \ud558\uc790 \uae40\ub300\uc911\ub9cc\uc774 \uc785\uc744 \uc5f4\uc5c8\ub2e4. \uc9c8\ubb38\uc740 \uc81c2\ucc28 \uc138\uacc4 \ub300\uc804\uacfc \uad00\ub828\ub41c \uc720\ub7fd \uc815\uc138\uc5d0 \uad00\ud55c \uac83\uc774\uc5c8\ub294\ub370 \uad50\uad00\uc740 \ub2f5\ubcc0\uc744 \ud558\uc9c0 \ubabb\ud558\uace0 \uc5b4\ubb3c\uc5b4\ubb3c\ud558\uc5ec \ucc3d\ud53c\ub97c \ub2f9\ud558\uace0 \ub9d0\uc558\ub2e4. 3\ud559\ub144\ubd80\ud130 \ud559\uad50 \uce21\uc758 \uc815\ucc45\uc73c\ub85c \ucde8\uc9c1\ubc18\uacfc \uc9c4\ud559\ubc18\uc774 \ub098\ub258\uc5c8\ub294\ub370, \uc740\ud589 \ucde8\uc5c5\uc744 \ubaa9\ud45c\ub85c \ucde8\uc9c1\ubc18\uc5d0 \ub4e4\uc5b4\uac14\uc73c\ub098 \uc774\ub0b4 \ub300\ud559 \uc9c4\ud559\uc73c\ub85c \uc9c4\ub85c\ub97c \ubcc0\uacbd\ud558\uc5ec \ucde8\uc9c1\ubc18 \ubc18\uc7a5\uc744 \uadf8\ub9cc\ub450\uace0 \uc9c4\ud559\ubc18\uc73c\ub85c \uc62e\uae30\uac8c \ub41c\ub2e4.", "answer": "\uc9c4\ud559\ubc18", "sentence": "3\ud559\ub144\ubd80\ud130 \ud559\uad50 \uce21\uc758 \uc815\ucc45\uc73c\ub85c \ucde8\uc9c1\ubc18\uacfc \uc9c4\ud559\ubc18 \uc774 \ub098\ub258\uc5c8\ub294\ub370, \uc740\ud589 \ucde8\uc5c5\uc744 \ubaa9\ud45c\ub85c \ucde8\uc9c1\ubc18\uc5d0 \ub4e4\uc5b4\uac14\uc73c\ub098 \uc774\ub0b4 \ub300\ud559 \uc9c4\ud559\uc73c\ub85c \uc9c4\ub85c\ub97c \ubcc0\uacbd\ud558\uc5ec \ucde8\uc9c1\ubc18 \ubc18\uc7a5\uc744 \uadf8\ub9cc\ub450\uace0 \uc9c4\ud559\ubc18\uc73c\ub85c \uc62e\uae30\uac8c \ub41c\ub2e4.", "paragraph_sentence": "\uadf8\ub294 \ub3c5\uc11c\uad11\uc774\uba74\uc11c \uc815\uce58\uc640 \uc601\uc5b4\uc5d0 \uad00\uc2ec\uc774 \ub9ce\uc544 \uc791\ubb38\uacfc \uc5ed\uc0ac \uc131\uc801\uc774 \ub6f0\uc5b4\ub0ac\uc73c\uba70 \uc6c5\ubcc0\uc5d0\ub3c4 \uc18c\uc9c8\uc744 \ubcf4\uc600\ub2e4 \ud55c\ub2e4. \uace0\uad50 \uc2dc\uc808 \ud559\uc801\ubd80\uc5d0\ub294 \uae40\ub300\uc911\uc5d0 \ub300\ud558\uc5ec \"\uc5b8\ubcc0\uc740 \ub298 \uc815\ud655, \uba85\ub8cc\ud558\ub2e4. \"\ub77c\uace0 \uae30\uc7ac\ub418\uc5b4 \uc788\ub2e4. \uc774 \ud0d3\uc778\uc9c0 \ud6d7\ub0a0 \uadf8\ub294 \uc11c\uc6b8\uc5d0\uc11c \uc6c5\ubcc0\ud559\uc6d0\uc744 \uc6b4\uc601\ud558\uae30\ub3c4 \ud558\uc600\ub2e4. \uadf8\ub9ac\uace0 \uc774 \ud559\uc6d0\uc5d0\uc11c \uc815\uce58\uc801 \ub3d9\uc9c0\uc778 \uae40\uc0c1\ud604\uacfc \ub9cc\ub098\uac8c \ub41c\ub2e4. \uadf8\uc758 \ud55c \ub3d9\ucc3d\uc0dd\uc758 \ub9d0\uc5d0 \uc758\ud558\uba74, \ud55c\ubc88\uc740 \uc77c\ubcf8\uc778 \uad50\uad00\uc774 \uc804\uad50\uc0dd\uc744 \ubaa8\uc544 \ub193\uace0 \uc2dc\uad6d\uc5d0 \ub300\ud55c \uac15\uc5f0\uc744 \ud55c \uc77c\uc774 \uc788\uc5c8\ub2e4. \uac15\uc5f0\uc774 \ub05d\ub098\uace0 \uad50\uad00\uc774 \uc9c8\ubb38\uc774 \uc788\uc73c\uba74 \ud558\ub77c\uace0 \ud558\uc790 \uae40\ub300\uc911\ub9cc\uc774 \uc785\uc744 \uc5f4\uc5c8\ub2e4. \uc9c8\ubb38\uc740 \uc81c2\ucc28 \uc138\uacc4 \ub300\uc804\uacfc \uad00\ub828\ub41c \uc720\ub7fd \uc815\uc138\uc5d0 \uad00\ud55c \uac83\uc774\uc5c8\ub294\ub370 \uad50\uad00\uc740 \ub2f5\ubcc0\uc744 \ud558\uc9c0 \ubabb\ud558\uace0 \uc5b4\ubb3c\uc5b4\ubb3c\ud558\uc5ec \ucc3d\ud53c\ub97c \ub2f9\ud558\uace0 \ub9d0\uc558\ub2e4. 3\ud559\ub144\ubd80\ud130 \ud559\uad50 \uce21\uc758 \uc815\ucc45\uc73c\ub85c \ucde8\uc9c1\ubc18\uacfc \uc9c4\ud559\ubc18 \uc774 \ub098\ub258\uc5c8\ub294\ub370, \uc740\ud589 \ucde8\uc5c5\uc744 \ubaa9\ud45c\ub85c \ucde8\uc9c1\ubc18\uc5d0 \ub4e4\uc5b4\uac14\uc73c\ub098 \uc774\ub0b4 \ub300\ud559 \uc9c4\ud559\uc73c\ub85c \uc9c4\ub85c\ub97c \ubcc0\uacbd\ud558\uc5ec \ucde8\uc9c1\ubc18 \ubc18\uc7a5\uc744 \uadf8\ub9cc\ub450\uace0 \uc9c4\ud559\ubc18\uc73c\ub85c \uc62e\uae30\uac8c \ub41c\ub2e4. ", "paragraph_answer": "\uadf8\ub294 \ub3c5\uc11c\uad11\uc774\uba74\uc11c \uc815\uce58\uc640 \uc601\uc5b4\uc5d0 \uad00\uc2ec\uc774 \ub9ce\uc544 \uc791\ubb38\uacfc \uc5ed\uc0ac \uc131\uc801\uc774 \ub6f0\uc5b4\ub0ac\uc73c\uba70 \uc6c5\ubcc0\uc5d0\ub3c4 \uc18c\uc9c8\uc744 \ubcf4\uc600\ub2e4 \ud55c\ub2e4. \uace0\uad50 \uc2dc\uc808 \ud559\uc801\ubd80\uc5d0\ub294 \uae40\ub300\uc911\uc5d0 \ub300\ud558\uc5ec \"\uc5b8\ubcc0\uc740 \ub298 \uc815\ud655, \uba85\ub8cc\ud558\ub2e4.\"\ub77c\uace0 \uae30\uc7ac\ub418\uc5b4 \uc788\ub2e4. \uc774 \ud0d3\uc778\uc9c0 \ud6d7\ub0a0 \uadf8\ub294 \uc11c\uc6b8\uc5d0\uc11c \uc6c5\ubcc0\ud559\uc6d0\uc744 \uc6b4\uc601\ud558\uae30\ub3c4 \ud558\uc600\ub2e4. \uadf8\ub9ac\uace0 \uc774 \ud559\uc6d0\uc5d0\uc11c \uc815\uce58\uc801 \ub3d9\uc9c0\uc778 \uae40\uc0c1\ud604\uacfc \ub9cc\ub098\uac8c \ub41c\ub2e4. \uadf8\uc758 \ud55c \ub3d9\ucc3d\uc0dd\uc758 \ub9d0\uc5d0 \uc758\ud558\uba74, \ud55c\ubc88\uc740 \uc77c\ubcf8\uc778 \uad50\uad00\uc774 \uc804\uad50\uc0dd\uc744 \ubaa8\uc544 \ub193\uace0 \uc2dc\uad6d\uc5d0 \ub300\ud55c \uac15\uc5f0\uc744 \ud55c \uc77c\uc774 \uc788\uc5c8\ub2e4. \uac15\uc5f0\uc774 \ub05d\ub098\uace0 \uad50\uad00\uc774 \uc9c8\ubb38\uc774 \uc788\uc73c\uba74 \ud558\ub77c\uace0 \ud558\uc790 \uae40\ub300\uc911\ub9cc\uc774 \uc785\uc744 \uc5f4\uc5c8\ub2e4. \uc9c8\ubb38\uc740 \uc81c2\ucc28 \uc138\uacc4 \ub300\uc804\uacfc \uad00\ub828\ub41c \uc720\ub7fd \uc815\uc138\uc5d0 \uad00\ud55c \uac83\uc774\uc5c8\ub294\ub370 \uad50\uad00\uc740 \ub2f5\ubcc0\uc744 \ud558\uc9c0 \ubabb\ud558\uace0 \uc5b4\ubb3c\uc5b4\ubb3c\ud558\uc5ec \ucc3d\ud53c\ub97c \ub2f9\ud558\uace0 \ub9d0\uc558\ub2e4. 3\ud559\ub144\ubd80\ud130 \ud559\uad50 \uce21\uc758 \uc815\ucc45\uc73c\ub85c \ucde8\uc9c1\ubc18\uacfc \uc9c4\ud559\ubc18 \uc774 \ub098\ub258\uc5c8\ub294\ub370, \uc740\ud589 \ucde8\uc5c5\uc744 \ubaa9\ud45c\ub85c \ucde8\uc9c1\ubc18\uc5d0 \ub4e4\uc5b4\uac14\uc73c\ub098 \uc774\ub0b4 \ub300\ud559 \uc9c4\ud559\uc73c\ub85c \uc9c4\ub85c\ub97c \ubcc0\uacbd\ud558\uc5ec \ucde8\uc9c1\ubc18 \ubc18\uc7a5\uc744 \uadf8\ub9cc\ub450\uace0 \uc9c4\ud559\ubc18\uc73c\ub85c \uc62e\uae30\uac8c \ub41c\ub2e4.", "sentence_answer": "3\ud559\ub144\ubd80\ud130 \ud559\uad50 \uce21\uc758 \uc815\ucc45\uc73c\ub85c \ucde8\uc9c1\ubc18\uacfc \uc9c4\ud559\ubc18 \uc774 \ub098\ub258\uc5c8\ub294\ub370, \uc740\ud589 \ucde8\uc5c5\uc744 \ubaa9\ud45c\ub85c \ucde8\uc9c1\ubc18\uc5d0 \ub4e4\uc5b4\uac14\uc73c\ub098 \uc774\ub0b4 \ub300\ud559 \uc9c4\ud559\uc73c\ub85c \uc9c4\ub85c\ub97c \ubcc0\uacbd\ud558\uc5ec \ucde8\uc9c1\ubc18 \ubc18\uc7a5\uc744 \uadf8\ub9cc\ub450\uace0 \uc9c4\ud559\ubc18\uc73c\ub85c \uc62e\uae30\uac8c \ub41c\ub2e4.", "paragraph_id": "6576830-2-2", "paragraph_question": "question: \uae40\ub300\uc911\uc774 \ucde8\uc5c5\ubc18\uc5d0\uc11c \uc9c4\ub85c\ub97c \ubcc0\uacbd\ud558\uc5ec \uc62e\uae34 \uacf3\uc740?, context: \uadf8\ub294 \ub3c5\uc11c\uad11\uc774\uba74\uc11c \uc815\uce58\uc640 \uc601\uc5b4\uc5d0 \uad00\uc2ec\uc774 \ub9ce\uc544 \uc791\ubb38\uacfc \uc5ed\uc0ac \uc131\uc801\uc774 \ub6f0\uc5b4\ub0ac\uc73c\uba70 \uc6c5\ubcc0\uc5d0\ub3c4 \uc18c\uc9c8\uc744 \ubcf4\uc600\ub2e4 \ud55c\ub2e4. \uace0\uad50 \uc2dc\uc808 \ud559\uc801\ubd80\uc5d0\ub294 \uae40\ub300\uc911\uc5d0 \ub300\ud558\uc5ec \"\uc5b8\ubcc0\uc740 \ub298 \uc815\ud655, \uba85\ub8cc\ud558\ub2e4.\"\ub77c\uace0 \uae30\uc7ac\ub418\uc5b4 \uc788\ub2e4. \uc774 \ud0d3\uc778\uc9c0 \ud6d7\ub0a0 \uadf8\ub294 \uc11c\uc6b8\uc5d0\uc11c \uc6c5\ubcc0\ud559\uc6d0\uc744 \uc6b4\uc601\ud558\uae30\ub3c4 \ud558\uc600\ub2e4. \uadf8\ub9ac\uace0 \uc774 \ud559\uc6d0\uc5d0\uc11c \uc815\uce58\uc801 \ub3d9\uc9c0\uc778 \uae40\uc0c1\ud604\uacfc \ub9cc\ub098\uac8c \ub41c\ub2e4. \uadf8\uc758 \ud55c \ub3d9\ucc3d\uc0dd\uc758 \ub9d0\uc5d0 \uc758\ud558\uba74, \ud55c\ubc88\uc740 \uc77c\ubcf8\uc778 \uad50\uad00\uc774 \uc804\uad50\uc0dd\uc744 \ubaa8\uc544 \ub193\uace0 \uc2dc\uad6d\uc5d0 \ub300\ud55c \uac15\uc5f0\uc744 \ud55c \uc77c\uc774 \uc788\uc5c8\ub2e4. \uac15\uc5f0\uc774 \ub05d\ub098\uace0 \uad50\uad00\uc774 \uc9c8\ubb38\uc774 \uc788\uc73c\uba74 \ud558\ub77c\uace0 \ud558\uc790 \uae40\ub300\uc911\ub9cc\uc774 \uc785\uc744 \uc5f4\uc5c8\ub2e4. \uc9c8\ubb38\uc740 \uc81c2\ucc28 \uc138\uacc4 \ub300\uc804\uacfc \uad00\ub828\ub41c \uc720\ub7fd \uc815\uc138\uc5d0 \uad00\ud55c \uac83\uc774\uc5c8\ub294\ub370 \uad50\uad00\uc740 \ub2f5\ubcc0\uc744 \ud558\uc9c0 \ubabb\ud558\uace0 \uc5b4\ubb3c\uc5b4\ubb3c\ud558\uc5ec \ucc3d\ud53c\ub97c \ub2f9\ud558\uace0 \ub9d0\uc558\ub2e4. 3\ud559\ub144\ubd80\ud130 \ud559\uad50 \uce21\uc758 \uc815\ucc45\uc73c\ub85c \ucde8\uc9c1\ubc18\uacfc \uc9c4\ud559\ubc18\uc774 \ub098\ub258\uc5c8\ub294\ub370, \uc740\ud589 \ucde8\uc5c5\uc744 \ubaa9\ud45c\ub85c \ucde8\uc9c1\ubc18\uc5d0 \ub4e4\uc5b4\uac14\uc73c\ub098 \uc774\ub0b4 \ub300\ud559 \uc9c4\ud559\uc73c\ub85c \uc9c4\ub85c\ub97c \ubcc0\uacbd\ud558\uc5ec \ucde8\uc9c1\ubc18 \ubc18\uc7a5\uc744 \uadf8\ub9cc\ub450\uace0 \uc9c4\ud559\ubc18\uc73c\ub85c \uc62e\uae30\uac8c \ub41c\ub2e4."} {"question": "\uc548\ud1a0\ub2c8\uc6b0\uc2a4\uac00 \uc625\ud0c0\ube44\uc544\ub204\uc2a4\uc5d0\uac8c 120\ucc99\uc758 \uc804\ud568\uc744 \uc9c0\uc6d0\ud558\uace0 \uadf8 \ubcf4\ub2f5\uc73c\ub85c \uc548\ud1a0\ub2c8\uc6b0\uc2a4\uc5d0\uac8c 2\ub9cc\uba85\uc758 \uad70\uc0ac\ub97c \uc9c0\uc6d0\ud55c\ub2e4\ub294 \uc57d\uc18d\uc744 \ud55c \ud611\uc815\uc740 \uc5b4\ub514\uc5d0\uc11c \uc774\ub8e8\uc5b4\uc84c\ub098?", "paragraph": "\uc139\uc2a4\ud22c\uc2a4 \ud3fc\ud398\uc774\uc6b0\uc2a4\ub294 \uc774\ud0c8\ub9ac\uc544 \ubc18\ub3c4\uc640 \uc9c0\uc911\ud574 \uac04\uc758 \uad70\ub7c9 \uc218\uc1a1 \uac70\ubd80\ub85c \uc774\ud0c8\ub9ac\uc544\uc758 \uc625\ud0c0\ube44\uc544\ub204\uc2a4\ub97c \uc704\ud611\ud558\uc600\ub294\ub370 \ud3fc\ud398\uc774\uc6b0\uc2a4\uc758 \uc544\ub4e4\uc774 \ud574\uad70 \uc0ac\ub839\uad00\uc73c\ub85c \uc784\uba85\ub418\uc5b4\uc11c \uc774\ud0c8\ub9ac\uc544\uc5d0 \ud070 \uae30\uadfc\uc758 \uc6d0\uc778\uc774 \ub418\uc5c8\ub2e4. \ud3fc\ud398\uc774\uc6b0\uc2a4\uac00 \uc81c\ud574\uad8c\uc740 \uac00\uc9c0\uace0 \uc788\uc5c8\uae30 \ub54c\ubb38\uc5d0 \ub9ce\uc740 \uc0ac\ub78c\ub4e4\uc774 \uadf8\ub97c \u2018\ub135\ud230\uc758 \uc544\ub4e4\u2019(Neptuni filius)\uc774\ub77c \ubd88\ub800\ub2e4. \uae30\uc6d0\uc804 39\ub144, \ubbf8\uc138\ub214 \uc870\uc57d\uc73c\ub85c \uc784\uc2dc \ud3c9\ud654 \ud611\uc815\uc774 \ub9fa\uc5b4\uc84c\ub294\ub370 \uc774\ud0c8\ub9ac\uc544\uc758 \ud574\uc0c1 \ubd09\uc1c4\uac00 \ud480\ub9ac\uc790 \uc625\ud0c0\ube44\uc544\ub204\uc2a4\ub294 \ud3fc\ud398\uc774\uc6b0\uc2a4\uc5d0\uac8c \uc0ac\ub974\ub370\ub0d0, \ucf54\ub974\uc2dc\uce74, \uc2dc\uce60\ub9ac\uc544\uc640, \ud3a0\ub85c\ud3f0\ub124\uc18c\uc2a4 \ubc18\ub3c4\ub97c \ub0b4\uc8fc\uc5c8\ub2e4. \uadf8\ub9ac\uace0 \ud3fc\ud398\uc774\uc6b0\uc2a4\uac00 \uae30\uc6d0\uc804 35\ub144\uc5d0 \uc9d1\uc815\uad00\uc73c\ub85c \uc784\uba85\ub418\ub294 \uac83\uc744 \ubcf4\uc7a5\ud574 \uc8fc\uc5c8\ub2e4. \uc138 \uba85\uc758 \uc9d1\uc815\uad00\uacfc \uc139\uc2a4\ud22c\uc2a4 \ud3fc\ud398\uc774\uc6b0\uc2a4 \uac04\uc758 \uc601\ud1a0 \ud611\uc815\uc740 \uae30\uc6d0\uc804 38\ub144 1\uc6d4 17\uc77c \uc625\ud0c0\ube44\uc544\ub204\uc2a4\uac00 \uc2a4\ud06c\ub9ac\ubcf4\ub2c8\uc544\uc640 \uc774\ud63c\ud558\uace0 \ub9ac\ube44\uc544\uc640 \uc7ac\ud63c\ud558\uc790 \uade0\ud615\uc774 \ud754\ub4e4\ub9ac\uae30 \uc2dc\uc791\ud558\uc600\ub2e4. \ud3fc\ud398\uc774\uc6b0\uc2a4\uc758 \ud718\ud558 \uc81c\ub3c5 \uc911 \ud558\ub098\uac00 \ud3fc\ud398\uc774\uc6b0\uc2a4\ub97c \ubc30\uc2e0\ud558\uace0 \uc625\ud0c0\ube44\uc544\ub204\uc2a4\uc5d0\uac8c \ub3c4\ub85c \uc0ac\ub974\ub370\ub0d0\uc640 \ucf54\ub974\uc2dc\uce74\ub97c \ubc18\ud658\ud558\uc600\ub2e4. \uadf8\ub7ec\ub098 \ud3fc\ud398\uc774\uc6b0\uc2a4\ub97c \uacf5\uaca9\ud558\ub824\uba74 \uc548\ud1a0\ub2c8\uc6b0\uc2a4\uc758 \uc9c0\uc9c0\uac00 \uc808\ub300\uc801\uc774\uc5c8\ub358 \uc625\ud0c0\ube44\uc544\ub204\uc2a4\ub294 \ud611\uc815\uc744 \ub9fa\uc5b4 \uc0bc\ub450 \uc815\uce58 \uccb4\uc81c\ub97c \uc5f0\uc7a5\ud558\uc5ec \uae30\uc6d0\uc804 37\ub144\ubd80\ud130 \uc2dc\uc791\ud558\uc5ec 5\ub144 \ub3d9\uc548 \ub85c\ub9c8\ub97c \uacf5\ub3d9\uc73c\ub85c \ud1b5\uce58\ud558\uae30\ub85c \ud569\uc758\ud558\uc600\ub2e4. \uc625\ud0c0\ube44\uc544\ub204\uc2a4\uc758 \uc9c0\uc9c0\ub97c \uc5bb\uc740 \uc548\ud1a0\ub2c8\uc6b0\uc2a4\ub294 \ud30c\ub974\ud2f0\uc544\uc640\uc758 \uc6d0\uc815\uc744 \uc704\ud574 \uadf8\uc758 \uc9c0\uc9c0 \uc138\ub825\uc744 \ud655\ub300\ud558\uc600\ub294\ub370 \uadf8\ub294 \uae30\uc6d0\uc804 53\ub144\uc758 \uce74\ub974\ud558\uc774 \uc804\ud22c\uc5d0\uc11c\uc758 \ud328\ubc30\ub97c \ubcf5\uc218\ud558\uace0 \uc2f6\uc5b4\ud558\uc600\ub2e4. \ud0c0\ub80c\ud23c(Tarentum)\uc758 \ud611\uc815\uc5d0\uc11c \uc548\ud1a0\ub2c8\uc6b0\uc2a4\ub294 \uc625\ud0c0\ube44\uc544\ub204\uc2a4\uc5d0\uac8c 120\ucc99\uc758 \uc804\ud568\uc744 \uc8fc\uace0 \uc625\ud0c0\ube44\uc544\ub204\uc2a4\ub294 2\ub9cc \uba85\uc758 \uad70\uc0ac\ub97c \ud30c\ub974\ud2f0\uc544\uc640 \uc2f8\uc6b0\ub824\ub294 \uc548\ud1a0\ub2c8\uc6b0\uc2a4\uc5d0\uac8c \ubcf4\ub0b4\uc900\ub2e4\uace0 \uc57d\uc18d\ud558\uc600\ub2e4. \uadf8\ub7ec\ub098 \uc625\ud0c0\ube44\uc544\ub204\uc2a4\ub294 \uc57d\uc18d\ud588\ub358 \ubcd1\ub825\uc758 \ubd88\uacfc 1/10\uc778 2\ucc9c \uba85\ubc16\uc5d0 \ubcf4\ub0b4\uc8fc\uc9c0 \uc54a\uc558\uace0 \uc774\ub7ec\ud55c \ub3c4\ubc1c\uc740 \uc548\ud1a0\ub2c8\uc6b0\uc2a4\uac00 6\ub144 \ub4a4 \uc11c\ub85c \uc804\ud22c\uc5d0\uc11c \ub9c8\uc8fc \ub300\ud560 \ub54c\uae4c\uc9c0 \uc774 \uc77c\uc744 \ub450\uace0\ub450\uace0 \uc78a\uc9c0 \uc54a\uc558\ub2e4 \ud55c\ub2e4.", "answer": "\ud0c0\ub80c\ud23c", "sentence": "\ud0c0\ub80c\ud23c (Tarentum)", "paragraph_sentence": "\uc139\uc2a4\ud22c\uc2a4 \ud3fc\ud398\uc774\uc6b0\uc2a4\ub294 \uc774\ud0c8\ub9ac\uc544 \ubc18\ub3c4\uc640 \uc9c0\uc911\ud574 \uac04\uc758 \uad70\ub7c9 \uc218\uc1a1 \uac70\ubd80\ub85c \uc774\ud0c8\ub9ac\uc544\uc758 \uc625\ud0c0\ube44\uc544\ub204\uc2a4\ub97c \uc704\ud611\ud558\uc600\ub294\ub370 \ud3fc\ud398\uc774\uc6b0\uc2a4\uc758 \uc544\ub4e4\uc774 \ud574\uad70 \uc0ac\ub839\uad00\uc73c\ub85c \uc784\uba85\ub418\uc5b4\uc11c \uc774\ud0c8\ub9ac\uc544\uc5d0 \ud070 \uae30\uadfc\uc758 \uc6d0\uc778\uc774 \ub418\uc5c8\ub2e4. \ud3fc\ud398\uc774\uc6b0\uc2a4\uac00 \uc81c\ud574\uad8c\uc740 \uac00\uc9c0\uace0 \uc788\uc5c8\uae30 \ub54c\ubb38\uc5d0 \ub9ce\uc740 \uc0ac\ub78c\ub4e4\uc774 \uadf8\ub97c \u2018\ub135\ud230\uc758 \uc544\ub4e4\u2019(Neptuni filius)\uc774\ub77c \ubd88\ub800\ub2e4. \uae30\uc6d0\uc804 39\ub144, \ubbf8\uc138\ub214 \uc870\uc57d\uc73c\ub85c \uc784\uc2dc \ud3c9\ud654 \ud611\uc815\uc774 \ub9fa\uc5b4\uc84c\ub294\ub370 \uc774\ud0c8\ub9ac\uc544\uc758 \ud574\uc0c1 \ubd09\uc1c4\uac00 \ud480\ub9ac\uc790 \uc625\ud0c0\ube44\uc544\ub204\uc2a4\ub294 \ud3fc\ud398\uc774\uc6b0\uc2a4\uc5d0\uac8c \uc0ac\ub974\ub370\ub0d0, \ucf54\ub974\uc2dc\uce74, \uc2dc\uce60\ub9ac\uc544\uc640, \ud3a0\ub85c\ud3f0\ub124\uc18c\uc2a4 \ubc18\ub3c4\ub97c \ub0b4\uc8fc\uc5c8\ub2e4. \uadf8\ub9ac\uace0 \ud3fc\ud398\uc774\uc6b0\uc2a4\uac00 \uae30\uc6d0\uc804 35\ub144\uc5d0 \uc9d1\uc815\uad00\uc73c\ub85c \uc784\uba85\ub418\ub294 \uac83\uc744 \ubcf4\uc7a5\ud574 \uc8fc\uc5c8\ub2e4. \uc138 \uba85\uc758 \uc9d1\uc815\uad00\uacfc \uc139\uc2a4\ud22c\uc2a4 \ud3fc\ud398\uc774\uc6b0\uc2a4 \uac04\uc758 \uc601\ud1a0 \ud611\uc815\uc740 \uae30\uc6d0\uc804 38\ub144 1\uc6d4 17\uc77c \uc625\ud0c0\ube44\uc544\ub204\uc2a4\uac00 \uc2a4\ud06c\ub9ac\ubcf4\ub2c8\uc544\uc640 \uc774\ud63c\ud558\uace0 \ub9ac\ube44\uc544\uc640 \uc7ac\ud63c\ud558\uc790 \uade0\ud615\uc774 \ud754\ub4e4\ub9ac\uae30 \uc2dc\uc791\ud558\uc600\ub2e4. \ud3fc\ud398\uc774\uc6b0\uc2a4\uc758 \ud718\ud558 \uc81c\ub3c5 \uc911 \ud558\ub098\uac00 \ud3fc\ud398\uc774\uc6b0\uc2a4\ub97c \ubc30\uc2e0\ud558\uace0 \uc625\ud0c0\ube44\uc544\ub204\uc2a4\uc5d0\uac8c \ub3c4\ub85c \uc0ac\ub974\ub370\ub0d0\uc640 \ucf54\ub974\uc2dc\uce74\ub97c \ubc18\ud658\ud558\uc600\ub2e4. \uadf8\ub7ec\ub098 \ud3fc\ud398\uc774\uc6b0\uc2a4\ub97c \uacf5\uaca9\ud558\ub824\uba74 \uc548\ud1a0\ub2c8\uc6b0\uc2a4\uc758 \uc9c0\uc9c0\uac00 \uc808\ub300\uc801\uc774\uc5c8\ub358 \uc625\ud0c0\ube44\uc544\ub204\uc2a4\ub294 \ud611\uc815\uc744 \ub9fa\uc5b4 \uc0bc\ub450 \uc815\uce58 \uccb4\uc81c\ub97c \uc5f0\uc7a5\ud558\uc5ec \uae30\uc6d0\uc804 37\ub144\ubd80\ud130 \uc2dc\uc791\ud558\uc5ec 5\ub144 \ub3d9\uc548 \ub85c\ub9c8\ub97c \uacf5\ub3d9\uc73c\ub85c \ud1b5\uce58\ud558\uae30\ub85c \ud569\uc758\ud558\uc600\ub2e4. \uc625\ud0c0\ube44\uc544\ub204\uc2a4\uc758 \uc9c0\uc9c0\ub97c \uc5bb\uc740 \uc548\ud1a0\ub2c8\uc6b0\uc2a4\ub294 \ud30c\ub974\ud2f0\uc544\uc640\uc758 \uc6d0\uc815\uc744 \uc704\ud574 \uadf8\uc758 \uc9c0\uc9c0 \uc138\ub825\uc744 \ud655\ub300\ud558\uc600\ub294\ub370 \uadf8\ub294 \uae30\uc6d0\uc804 53\ub144\uc758 \uce74\ub974\ud558\uc774 \uc804\ud22c\uc5d0\uc11c\uc758 \ud328\ubc30\ub97c \ubcf5\uc218\ud558\uace0 \uc2f6\uc5b4\ud558\uc600\ub2e4. \ud0c0\ub80c\ud23c (Tarentum) \uc758 \ud611\uc815\uc5d0\uc11c \uc548\ud1a0\ub2c8\uc6b0\uc2a4\ub294 \uc625\ud0c0\ube44\uc544\ub204\uc2a4\uc5d0\uac8c 120\ucc99\uc758 \uc804\ud568\uc744 \uc8fc\uace0 \uc625\ud0c0\ube44\uc544\ub204\uc2a4\ub294 2\ub9cc \uba85\uc758 \uad70\uc0ac\ub97c \ud30c\ub974\ud2f0\uc544\uc640 \uc2f8\uc6b0\ub824\ub294 \uc548\ud1a0\ub2c8\uc6b0\uc2a4\uc5d0\uac8c \ubcf4\ub0b4\uc900\ub2e4\uace0 \uc57d\uc18d\ud558\uc600\ub2e4. \uadf8\ub7ec\ub098 \uc625\ud0c0\ube44\uc544\ub204\uc2a4\ub294 \uc57d\uc18d\ud588\ub358 \ubcd1\ub825\uc758 \ubd88\uacfc 1/10\uc778 2\ucc9c \uba85\ubc16\uc5d0 \ubcf4\ub0b4\uc8fc\uc9c0 \uc54a\uc558\uace0 \uc774\ub7ec\ud55c \ub3c4\ubc1c\uc740 \uc548\ud1a0\ub2c8\uc6b0\uc2a4\uac00 6\ub144 \ub4a4 \uc11c\ub85c \uc804\ud22c\uc5d0\uc11c \ub9c8\uc8fc \ub300\ud560 \ub54c\uae4c\uc9c0 \uc774 \uc77c\uc744 \ub450\uace0\ub450\uace0 \uc78a\uc9c0 \uc54a\uc558\ub2e4 \ud55c\ub2e4.", "paragraph_answer": "\uc139\uc2a4\ud22c\uc2a4 \ud3fc\ud398\uc774\uc6b0\uc2a4\ub294 \uc774\ud0c8\ub9ac\uc544 \ubc18\ub3c4\uc640 \uc9c0\uc911\ud574 \uac04\uc758 \uad70\ub7c9 \uc218\uc1a1 \uac70\ubd80\ub85c \uc774\ud0c8\ub9ac\uc544\uc758 \uc625\ud0c0\ube44\uc544\ub204\uc2a4\ub97c \uc704\ud611\ud558\uc600\ub294\ub370 \ud3fc\ud398\uc774\uc6b0\uc2a4\uc758 \uc544\ub4e4\uc774 \ud574\uad70 \uc0ac\ub839\uad00\uc73c\ub85c \uc784\uba85\ub418\uc5b4\uc11c \uc774\ud0c8\ub9ac\uc544\uc5d0 \ud070 \uae30\uadfc\uc758 \uc6d0\uc778\uc774 \ub418\uc5c8\ub2e4. \ud3fc\ud398\uc774\uc6b0\uc2a4\uac00 \uc81c\ud574\uad8c\uc740 \uac00\uc9c0\uace0 \uc788\uc5c8\uae30 \ub54c\ubb38\uc5d0 \ub9ce\uc740 \uc0ac\ub78c\ub4e4\uc774 \uadf8\ub97c \u2018\ub135\ud230\uc758 \uc544\ub4e4\u2019(Neptuni filius)\uc774\ub77c \ubd88\ub800\ub2e4. \uae30\uc6d0\uc804 39\ub144, \ubbf8\uc138\ub214 \uc870\uc57d\uc73c\ub85c \uc784\uc2dc \ud3c9\ud654 \ud611\uc815\uc774 \ub9fa\uc5b4\uc84c\ub294\ub370 \uc774\ud0c8\ub9ac\uc544\uc758 \ud574\uc0c1 \ubd09\uc1c4\uac00 \ud480\ub9ac\uc790 \uc625\ud0c0\ube44\uc544\ub204\uc2a4\ub294 \ud3fc\ud398\uc774\uc6b0\uc2a4\uc5d0\uac8c \uc0ac\ub974\ub370\ub0d0, \ucf54\ub974\uc2dc\uce74, \uc2dc\uce60\ub9ac\uc544\uc640, \ud3a0\ub85c\ud3f0\ub124\uc18c\uc2a4 \ubc18\ub3c4\ub97c \ub0b4\uc8fc\uc5c8\ub2e4. \uadf8\ub9ac\uace0 \ud3fc\ud398\uc774\uc6b0\uc2a4\uac00 \uae30\uc6d0\uc804 35\ub144\uc5d0 \uc9d1\uc815\uad00\uc73c\ub85c \uc784\uba85\ub418\ub294 \uac83\uc744 \ubcf4\uc7a5\ud574 \uc8fc\uc5c8\ub2e4. \uc138 \uba85\uc758 \uc9d1\uc815\uad00\uacfc \uc139\uc2a4\ud22c\uc2a4 \ud3fc\ud398\uc774\uc6b0\uc2a4 \uac04\uc758 \uc601\ud1a0 \ud611\uc815\uc740 \uae30\uc6d0\uc804 38\ub144 1\uc6d4 17\uc77c \uc625\ud0c0\ube44\uc544\ub204\uc2a4\uac00 \uc2a4\ud06c\ub9ac\ubcf4\ub2c8\uc544\uc640 \uc774\ud63c\ud558\uace0 \ub9ac\ube44\uc544\uc640 \uc7ac\ud63c\ud558\uc790 \uade0\ud615\uc774 \ud754\ub4e4\ub9ac\uae30 \uc2dc\uc791\ud558\uc600\ub2e4. \ud3fc\ud398\uc774\uc6b0\uc2a4\uc758 \ud718\ud558 \uc81c\ub3c5 \uc911 \ud558\ub098\uac00 \ud3fc\ud398\uc774\uc6b0\uc2a4\ub97c \ubc30\uc2e0\ud558\uace0 \uc625\ud0c0\ube44\uc544\ub204\uc2a4\uc5d0\uac8c \ub3c4\ub85c \uc0ac\ub974\ub370\ub0d0\uc640 \ucf54\ub974\uc2dc\uce74\ub97c \ubc18\ud658\ud558\uc600\ub2e4. \uadf8\ub7ec\ub098 \ud3fc\ud398\uc774\uc6b0\uc2a4\ub97c \uacf5\uaca9\ud558\ub824\uba74 \uc548\ud1a0\ub2c8\uc6b0\uc2a4\uc758 \uc9c0\uc9c0\uac00 \uc808\ub300\uc801\uc774\uc5c8\ub358 \uc625\ud0c0\ube44\uc544\ub204\uc2a4\ub294 \ud611\uc815\uc744 \ub9fa\uc5b4 \uc0bc\ub450 \uc815\uce58 \uccb4\uc81c\ub97c \uc5f0\uc7a5\ud558\uc5ec \uae30\uc6d0\uc804 37\ub144\ubd80\ud130 \uc2dc\uc791\ud558\uc5ec 5\ub144 \ub3d9\uc548 \ub85c\ub9c8\ub97c \uacf5\ub3d9\uc73c\ub85c \ud1b5\uce58\ud558\uae30\ub85c \ud569\uc758\ud558\uc600\ub2e4. \uc625\ud0c0\ube44\uc544\ub204\uc2a4\uc758 \uc9c0\uc9c0\ub97c \uc5bb\uc740 \uc548\ud1a0\ub2c8\uc6b0\uc2a4\ub294 \ud30c\ub974\ud2f0\uc544\uc640\uc758 \uc6d0\uc815\uc744 \uc704\ud574 \uadf8\uc758 \uc9c0\uc9c0 \uc138\ub825\uc744 \ud655\ub300\ud558\uc600\ub294\ub370 \uadf8\ub294 \uae30\uc6d0\uc804 53\ub144\uc758 \uce74\ub974\ud558\uc774 \uc804\ud22c\uc5d0\uc11c\uc758 \ud328\ubc30\ub97c \ubcf5\uc218\ud558\uace0 \uc2f6\uc5b4\ud558\uc600\ub2e4. \ud0c0\ub80c\ud23c (Tarentum)\uc758 \ud611\uc815\uc5d0\uc11c \uc548\ud1a0\ub2c8\uc6b0\uc2a4\ub294 \uc625\ud0c0\ube44\uc544\ub204\uc2a4\uc5d0\uac8c 120\ucc99\uc758 \uc804\ud568\uc744 \uc8fc\uace0 \uc625\ud0c0\ube44\uc544\ub204\uc2a4\ub294 2\ub9cc \uba85\uc758 \uad70\uc0ac\ub97c \ud30c\ub974\ud2f0\uc544\uc640 \uc2f8\uc6b0\ub824\ub294 \uc548\ud1a0\ub2c8\uc6b0\uc2a4\uc5d0\uac8c \ubcf4\ub0b4\uc900\ub2e4\uace0 \uc57d\uc18d\ud558\uc600\ub2e4. \uadf8\ub7ec\ub098 \uc625\ud0c0\ube44\uc544\ub204\uc2a4\ub294 \uc57d\uc18d\ud588\ub358 \ubcd1\ub825\uc758 \ubd88\uacfc 1/10\uc778 2\ucc9c \uba85\ubc16\uc5d0 \ubcf4\ub0b4\uc8fc\uc9c0 \uc54a\uc558\uace0 \uc774\ub7ec\ud55c \ub3c4\ubc1c\uc740 \uc548\ud1a0\ub2c8\uc6b0\uc2a4\uac00 6\ub144 \ub4a4 \uc11c\ub85c \uc804\ud22c\uc5d0\uc11c \ub9c8\uc8fc \ub300\ud560 \ub54c\uae4c\uc9c0 \uc774 \uc77c\uc744 \ub450\uace0\ub450\uace0 \uc78a\uc9c0 \uc54a\uc558\ub2e4 \ud55c\ub2e4.", "sentence_answer": " \ud0c0\ub80c\ud23c (Tarentum)", "paragraph_id": "6657258-15-1", "paragraph_question": "question: \uc548\ud1a0\ub2c8\uc6b0\uc2a4\uac00 \uc625\ud0c0\ube44\uc544\ub204\uc2a4\uc5d0\uac8c 120\ucc99\uc758 \uc804\ud568\uc744 \uc9c0\uc6d0\ud558\uace0 \uadf8 \ubcf4\ub2f5\uc73c\ub85c \uc548\ud1a0\ub2c8\uc6b0\uc2a4\uc5d0\uac8c 2\ub9cc\uba85\uc758 \uad70\uc0ac\ub97c \uc9c0\uc6d0\ud55c\ub2e4\ub294 \uc57d\uc18d\uc744 \ud55c \ud611\uc815\uc740 \uc5b4\ub514\uc5d0\uc11c \uc774\ub8e8\uc5b4\uc84c\ub098?, context: \uc139\uc2a4\ud22c\uc2a4 \ud3fc\ud398\uc774\uc6b0\uc2a4\ub294 \uc774\ud0c8\ub9ac\uc544 \ubc18\ub3c4\uc640 \uc9c0\uc911\ud574 \uac04\uc758 \uad70\ub7c9 \uc218\uc1a1 \uac70\ubd80\ub85c \uc774\ud0c8\ub9ac\uc544\uc758 \uc625\ud0c0\ube44\uc544\ub204\uc2a4\ub97c \uc704\ud611\ud558\uc600\ub294\ub370 \ud3fc\ud398\uc774\uc6b0\uc2a4\uc758 \uc544\ub4e4\uc774 \ud574\uad70 \uc0ac\ub839\uad00\uc73c\ub85c \uc784\uba85\ub418\uc5b4\uc11c \uc774\ud0c8\ub9ac\uc544\uc5d0 \ud070 \uae30\uadfc\uc758 \uc6d0\uc778\uc774 \ub418\uc5c8\ub2e4. \ud3fc\ud398\uc774\uc6b0\uc2a4\uac00 \uc81c\ud574\uad8c\uc740 \uac00\uc9c0\uace0 \uc788\uc5c8\uae30 \ub54c\ubb38\uc5d0 \ub9ce\uc740 \uc0ac\ub78c\ub4e4\uc774 \uadf8\ub97c \u2018\ub135\ud230\uc758 \uc544\ub4e4\u2019(Neptuni filius)\uc774\ub77c \ubd88\ub800\ub2e4. \uae30\uc6d0\uc804 39\ub144, \ubbf8\uc138\ub214 \uc870\uc57d\uc73c\ub85c \uc784\uc2dc \ud3c9\ud654 \ud611\uc815\uc774 \ub9fa\uc5b4\uc84c\ub294\ub370 \uc774\ud0c8\ub9ac\uc544\uc758 \ud574\uc0c1 \ubd09\uc1c4\uac00 \ud480\ub9ac\uc790 \uc625\ud0c0\ube44\uc544\ub204\uc2a4\ub294 \ud3fc\ud398\uc774\uc6b0\uc2a4\uc5d0\uac8c \uc0ac\ub974\ub370\ub0d0, \ucf54\ub974\uc2dc\uce74, \uc2dc\uce60\ub9ac\uc544\uc640, \ud3a0\ub85c\ud3f0\ub124\uc18c\uc2a4 \ubc18\ub3c4\ub97c \ub0b4\uc8fc\uc5c8\ub2e4. \uadf8\ub9ac\uace0 \ud3fc\ud398\uc774\uc6b0\uc2a4\uac00 \uae30\uc6d0\uc804 35\ub144\uc5d0 \uc9d1\uc815\uad00\uc73c\ub85c \uc784\uba85\ub418\ub294 \uac83\uc744 \ubcf4\uc7a5\ud574 \uc8fc\uc5c8\ub2e4. \uc138 \uba85\uc758 \uc9d1\uc815\uad00\uacfc \uc139\uc2a4\ud22c\uc2a4 \ud3fc\ud398\uc774\uc6b0\uc2a4 \uac04\uc758 \uc601\ud1a0 \ud611\uc815\uc740 \uae30\uc6d0\uc804 38\ub144 1\uc6d4 17\uc77c \uc625\ud0c0\ube44\uc544\ub204\uc2a4\uac00 \uc2a4\ud06c\ub9ac\ubcf4\ub2c8\uc544\uc640 \uc774\ud63c\ud558\uace0 \ub9ac\ube44\uc544\uc640 \uc7ac\ud63c\ud558\uc790 \uade0\ud615\uc774 \ud754\ub4e4\ub9ac\uae30 \uc2dc\uc791\ud558\uc600\ub2e4. \ud3fc\ud398\uc774\uc6b0\uc2a4\uc758 \ud718\ud558 \uc81c\ub3c5 \uc911 \ud558\ub098\uac00 \ud3fc\ud398\uc774\uc6b0\uc2a4\ub97c \ubc30\uc2e0\ud558\uace0 \uc625\ud0c0\ube44\uc544\ub204\uc2a4\uc5d0\uac8c \ub3c4\ub85c \uc0ac\ub974\ub370\ub0d0\uc640 \ucf54\ub974\uc2dc\uce74\ub97c \ubc18\ud658\ud558\uc600\ub2e4. \uadf8\ub7ec\ub098 \ud3fc\ud398\uc774\uc6b0\uc2a4\ub97c \uacf5\uaca9\ud558\ub824\uba74 \uc548\ud1a0\ub2c8\uc6b0\uc2a4\uc758 \uc9c0\uc9c0\uac00 \uc808\ub300\uc801\uc774\uc5c8\ub358 \uc625\ud0c0\ube44\uc544\ub204\uc2a4\ub294 \ud611\uc815\uc744 \ub9fa\uc5b4 \uc0bc\ub450 \uc815\uce58 \uccb4\uc81c\ub97c \uc5f0\uc7a5\ud558\uc5ec \uae30\uc6d0\uc804 37\ub144\ubd80\ud130 \uc2dc\uc791\ud558\uc5ec 5\ub144 \ub3d9\uc548 \ub85c\ub9c8\ub97c \uacf5\ub3d9\uc73c\ub85c \ud1b5\uce58\ud558\uae30\ub85c \ud569\uc758\ud558\uc600\ub2e4. \uc625\ud0c0\ube44\uc544\ub204\uc2a4\uc758 \uc9c0\uc9c0\ub97c \uc5bb\uc740 \uc548\ud1a0\ub2c8\uc6b0\uc2a4\ub294 \ud30c\ub974\ud2f0\uc544\uc640\uc758 \uc6d0\uc815\uc744 \uc704\ud574 \uadf8\uc758 \uc9c0\uc9c0 \uc138\ub825\uc744 \ud655\ub300\ud558\uc600\ub294\ub370 \uadf8\ub294 \uae30\uc6d0\uc804 53\ub144\uc758 \uce74\ub974\ud558\uc774 \uc804\ud22c\uc5d0\uc11c\uc758 \ud328\ubc30\ub97c \ubcf5\uc218\ud558\uace0 \uc2f6\uc5b4\ud558\uc600\ub2e4. \ud0c0\ub80c\ud23c(Tarentum)\uc758 \ud611\uc815\uc5d0\uc11c \uc548\ud1a0\ub2c8\uc6b0\uc2a4\ub294 \uc625\ud0c0\ube44\uc544\ub204\uc2a4\uc5d0\uac8c 120\ucc99\uc758 \uc804\ud568\uc744 \uc8fc\uace0 \uc625\ud0c0\ube44\uc544\ub204\uc2a4\ub294 2\ub9cc \uba85\uc758 \uad70\uc0ac\ub97c \ud30c\ub974\ud2f0\uc544\uc640 \uc2f8\uc6b0\ub824\ub294 \uc548\ud1a0\ub2c8\uc6b0\uc2a4\uc5d0\uac8c \ubcf4\ub0b4\uc900\ub2e4\uace0 \uc57d\uc18d\ud558\uc600\ub2e4. \uadf8\ub7ec\ub098 \uc625\ud0c0\ube44\uc544\ub204\uc2a4\ub294 \uc57d\uc18d\ud588\ub358 \ubcd1\ub825\uc758 \ubd88\uacfc 1/10\uc778 2\ucc9c \uba85\ubc16\uc5d0 \ubcf4\ub0b4\uc8fc\uc9c0 \uc54a\uc558\uace0 \uc774\ub7ec\ud55c \ub3c4\ubc1c\uc740 \uc548\ud1a0\ub2c8\uc6b0\uc2a4\uac00 6\ub144 \ub4a4 \uc11c\ub85c \uc804\ud22c\uc5d0\uc11c \ub9c8\uc8fc \ub300\ud560 \ub54c\uae4c\uc9c0 \uc774 \uc77c\uc744 \ub450\uace0\ub450\uace0 \uc78a\uc9c0 \uc54a\uc558\ub2e4 \ud55c\ub2e4."} {"question": "\uc138\uacc4 \uac01\uad6d\uc758 \uacfc\ud559\uc790\ub4e4\ub85c\ubd80\ud130 \ud575\uc2e4\ud5d8 \uc911\uc9c0\ub97c \uccad\uc6d0\ud558\ub294 \uc11c\uba85\uc744 \ubc1b\uc544 \uc11c\uba85\ubaa9\ub85d\uc744 UN\uc5d0 \uc81c\ucd9c\ud55c \ud574\ub294?", "paragraph": "\ub0c9\uc804\uccb4\uc81c\uac00 \uac15\ud654\ub418\uace0 \uad6d\uac00\uc801\uc73c\ub85c \ubc18\uacf5\uc8fc\uc758\uac00 \ub300\uc138\uc600\ub358 1950\ub144\ub300\uc5d0\ub3c4 \uadf8\ub294 \ubc18\ud575\uc5f0\uc124\uc744 \uacc4\uc18d\ud558\uc600\ub2e4. \uadf8 \ub54c\ubb38\uc5d0 FBI\uc758 \uc870\uc0ac\ub97c \ubc1b\uae30\ub3c4 \ud558\uace0, \ubbf8 \uc815\ubd80\ub85c\ubd80\ud130 \uc5ec\uad8c\ubc1c\uae09\uc744 \uac70\uc808\ub2f9\ud558\uae30\ub3c4 \uc544\uc600\ub2e4. \uadf8\uc758 \uc5ec\uad8c\uc740 1954\ub144 \uadf8\uc758 \uccab \ub178\ubca8\uc0c1 \uc218\uc0c1\uc5d0 \uc55e\uc11c \uc7a0\uc2dc \uc7ac\ubc1c\uae09\ub418\uc5c8\ub2e4. \uce7c\ud14d\uc758 \ub9ce\uc740 \uce5c\uad6c\ub4e4\ub3c4 \uadf8\ub97c \uc678\uba74\ud558\uae30 \uc2dc\uc791\ud588\uace0 \ucd1d\uc7a5 \ub450\ube0c\ub9ac\uc9c0\ub294 \ud3f4\ub9c1\uc5d0\uac8c \uc815\uce58\uc801 \uc5f0\uc124\uc744 \uadf8\ub9cc\ub458 \uac83\uc744 \uc694\uccad\ud558\uc600\ub2e4. \uadf8\ub294 1951\ub144 \ub17c\ub780\uc774 \ub418\ub294 \ub2e8\uccb4\ub4e4\uc744 \ud0c8\ud1f4\ud558\uace0 \uc5f0\uc124\ub4e4\uc744 \uc911\uc9c0\ud558\uc600\uc73c\uba70 \uc5f0\uad6c\uc5d0\ub9cc \uc804\ub150\ud558\uc600\uc73c\ub098, \uc5b8\ub860\uc758 \ube44\ub09c\uacfc \uad6d\ud68c\uc758 \uc758\uc2ec\uc740 \uacc4\uc18d\ub418\uc5c8\ub2e4. 1954\ub144 \ub178\ubca8 \ud654\ud559\uc0c1\uc744 \uc218\uc0c1\ud55c \ud6c4\uc5d0\ub294 \ub300\uc911\uacfc \uc9c0\uc778\uc758 \uc758\uc2ec\uc774 \ub9ce\uc774 \ud480\ub838\uace0 \ud3f4\ub9c1\uc740 \uc138\uacc4 \uacf3\uacf3\uc744 \ub3cc\uc544\ub2e4\ub2c8\uba70 \uc6d0\uc790\ud3ed\ud0c4 \uc0ac\uc6a9\uc5d0 \ubc18\ub300\ud558\ub294 \uc5f0\uc124\uc744 \ud558\uc600\ub2e4. \ud3f4\ub9c1\uc740 \ub2e4\uc2dc \uc6d0\uc790\ud3ed\ud0c4\uc73c\ub85c \uc778\ud55c \ubc29\uc0ac\ub2a5 \ud53c\ud3ed\uacfc \ub099\uc9c4\uc5d0 \uad00\uc2ec\uc744 \uac00\uc9c0\uae30 \uc2dc\uc791\ud558\uc600\ub2e4. 1958\ub144 \ud3f4\ub9c1\uc740 \ubc30\ub9ac \ucee4\uba38\ub108 \ub4f1\uacfc \ud568\uaed8 \ubc29\uc0ac\uc131 \ub3d9\uc704\uc6d0\uc18c Sr-90\uc758 \uc720\uc544 \uce58\uc544\uc5d0 \uc5b4\ub5a4 \uc601\ud5a5\uc744 \ub07c\uce58\ub294\uc9c0 \uc5f0\uad6c\ud558\uc600\uace0 1962\ub144\uc5d0\ub294 \ub8e8\uc774\uc2a4 \ub77c\uc774\uc2a4 \ub4f1\uacfc \ud568\uaed8 \ub099\uc9c4\uc73c\ub85c \ub5a8\uc5b4\uc9c4 Sr-90\uc744 \uc816\uc18c\uac00 \uba39\uace0 \uc816\uc18c\ub85c\ubd80\ud130 \uc6b0\uc720\ub97c \uba39\ub294 \uc0ac\ub78c\uc758 \ubf08\uc5d0 \ucd95\uc801\ub41c\ub2e4\ub294 \uc5f0\uad6c\uacb0\uacfc\ub97c \ubc1c\ud45c\ud558\uc600\ub2e4. \ub610\ud55c \ubb3c\ub9ac\ud559\uc790\uc774\uc790 \ubc18\uacf5\uc8fc\uc758\uc790\uc778 \ud154\ub7ec\uc640 \ubc29\uc0ac\ub2a5 \ub099\uc9c4\uc758 \uc704\ud5d8\uc5d0 \ub300\ud55c TV\ud1a0\ub860\uc5d0\ub3c4 \ucc38\uc5ec\ud558\uc600\ub2e4. \ub610\ud55c \ud3f4\ub9c1\uc740 1958\ub144 \uc138\uacc4 \uac01\uad6d\uc758 \uacfc\ud559\uc790\ub4e4\ub85c\ubd80\ud130 \ud575\uc2e4\ud5d8 \uc911\uc9c0\ub97c \uccad\uc6d0\ud558\ub294 \uc11c\uba85\uc744 \ubc1b\uc544 \uc11c\uba85\ubaa9\ub85d\uc744 UN\uc5d0 \uc81c\ucd9c\ud558\uc600\ub2e4. \ub300\uc911\uc758 \uc555\ubc15\uacfc \ub099\uc9c4\uc758 \uc704\ud5d8\uc5d0 \uad00\ud55c \uc5f0\uad6c\uacb0\uacfc\ub294 1963\ub144 \ubbf8\uad6d\uacfc \uc18c\ub828\uc774 \uc9c0\ud558\ub97c \uc81c\uc678\ud55c \ubaa8\ub4e0 \uacf3\uc5d0\uc11c \ud575\uc2e4\ud5d8\uc744 \uae08\uc9c0\ud558\ub294 \ubd80\ubd84\uc801 \ud575\uc2e4\ud5d8 \uae08\uc9c0\uc870\uc57d\uc5d0 \uc11c\uba85\ud558\uac8c \ub418\ub294 \uacb0\uacfc\ub97c \uac00\uc838\uc654\ub2e4. \uc774 \uc870\uc57d\uc740 \uadf8\ud574 10\uc6d4 10\uc77c\ubd80\ud130 \ud6a8\ub825\uc774 \ubc1c\uc0dd\ud588\uace0 \ubc14\ub85c \uadf8\ub0a0 \ud3f4\ub9c1\uc740 1962\ub144 \ub178\ubca8 \ud3c9\ud654\uc0c1\uc744 \uc218\uc0c1\ud558\uac8c \ub41c\ub2e4. \ud3f4\ub9c1\uc758 \ud654\ud559\uc5d0 \ub300\ud55c \uacf5\ud5cc\uc744 \uc778\uc815\ud55c \uacfc\ud559\uc790\ub4e4\uc744 \ud3ec\ud568\ud55c \ub9ce\uc740 \ub17c\uac1d\ub4e4\uc740 \ud3f4\ub9c1\uc758 \uc815\uce58\uc801 \ud589\ubcf4\uc5d0 \ube44\ud310\uc801\uc774\uc5c8\uc73c\uba70 \uadf8\ub97c \uacf5\uc0b0\uc8fc\uc758\uc790\ub77c\uace0 \ube44\ub09c\ud558\uc600\ub2e4. 1960\ub144 \ud3f4\ub9c1\uc740 \uadf8\ub97c \uacf5\uc0b0\uc8fc\uc758\uc790\ub77c\uace0 \ub099\uc778 \ucc0d\uc740 \uc0c1\uc6d0\uc758 \ud55c \uc870\uc0ac \uc18c\uc704\uc6d0\ud68c\uc5d0 \ucd9c\ub450\ub97c \uba85\ub839\ubc1b\uc544 \uadf8\ud1a0\ub85d \ub9ce\uc740 \uc11c\uba85\ub4e4\uc744 \ubaa8\uc740 \uacfc\uc815\uc5d0 \ub300\ud574 \ud574\uba85\uc744 \uc694\uad6c\ubc1b\uc558\ub2e4. \uadf8\uac00 \ub178\ubca8 \ud3c9\ud654\uc0c1\uc744 \uc218\uc0c1\ud55c \uc774\ud6c4\uc5d0\ub3c4 \ube44\ud310\uc801\uc778 \uc5ec\ub860\uc740 \uacc4\uc18d\ub418\uc5c8\ub2e4. \uadf8\uac00 1962\ub144 \ub178\ubca8\ud3c9\ud654\uc0c1\uc744 \ubc1b\uac8c \ub418\uc790 \ubbf8\uad6d \ub77c\uc774\ud504 \uc7a1\uc9c0\uc0ac\ub294 \u201c\ub178\ub974\uc6e8\uc774\uc5d0\uc11c \uc628 \uc218\uc0c1\ud55c \ubaa8\uc695\u201d\uc774\ub780 \ud5e4\ub4dc\ub77c\uc778\uc744 \ub0c8\ub2e4. \uce7c\ud14d \ucd1d\uc7a5 \ub450\ube0c\ub9ac\uc9c0 \uc5ed\uc2dc \ubbf8\uc801\uc9c0\uadfc\ud55c \ubc18\uc751\ub9cc\uc744 \ubcf4\uc600\ub2e4. \ud3f4\ub9c1\uc740 \ub178\ubca8\uc0c1 \uc0c1\uae08\uc744 \uac00\uc9c0\uace0 \ud3c9\ud654\uc5d0 \uad00\ud55c \uc77c\uc744 \ud558\uae30\ub85c \uacb0\uc2ec\ud558\uace0 1963\ub144 10\uc6d4 18\uc77c \uae30\uc790\ud68c\uacac\uc744 \uc5f4\uc5b4 \uce7c\ud14d\uc744 \ub5a0\ub09c\ub2e4\uace0 \ubc1c\ud45c\ud588\ub2e4.", "answer": "1958\ub144", "sentence": "1958\ub144 \ud3f4\ub9c1\uc740 \ubc30\ub9ac \ucee4\uba38\ub108 \ub4f1\uacfc \ud568\uaed8 \ubc29\uc0ac\uc131 \ub3d9\uc704\uc6d0\uc18c Sr-90\uc758 \uc720\uc544 \uce58\uc544\uc5d0 \uc5b4\ub5a4 \uc601\ud5a5\uc744 \ub07c\uce58\ub294\uc9c0 \uc5f0\uad6c\ud558\uc600\uace0 1962\ub144\uc5d0\ub294 \ub8e8\uc774\uc2a4 \ub77c\uc774\uc2a4 \ub4f1\uacfc \ud568\uaed8 \ub099\uc9c4\uc73c\ub85c \ub5a8\uc5b4\uc9c4 Sr-90\uc744 \uc816\uc18c\uac00 \uba39\uace0 \uc816\uc18c\ub85c\ubd80\ud130 \uc6b0\uc720\ub97c \uba39\ub294 \uc0ac\ub78c\uc758 \ubf08\uc5d0 \ucd95\uc801\ub41c\ub2e4\ub294 \uc5f0\uad6c\uacb0\uacfc\ub97c \ubc1c\ud45c\ud558\uc600\ub2e4.", "paragraph_sentence": "\ub0c9\uc804\uccb4\uc81c\uac00 \uac15\ud654\ub418\uace0 \uad6d\uac00\uc801\uc73c\ub85c \ubc18\uacf5\uc8fc\uc758\uac00 \ub300\uc138\uc600\ub358 1950\ub144\ub300\uc5d0\ub3c4 \uadf8\ub294 \ubc18\ud575\uc5f0\uc124\uc744 \uacc4\uc18d\ud558\uc600\ub2e4. \uadf8 \ub54c\ubb38\uc5d0 FBI\uc758 \uc870\uc0ac\ub97c \ubc1b\uae30\ub3c4 \ud558\uace0, \ubbf8 \uc815\ubd80\ub85c\ubd80\ud130 \uc5ec\uad8c\ubc1c\uae09\uc744 \uac70\uc808\ub2f9\ud558\uae30\ub3c4 \uc544\uc600\ub2e4. \uadf8\uc758 \uc5ec\uad8c\uc740 1954\ub144 \uadf8\uc758 \uccab \ub178\ubca8\uc0c1 \uc218\uc0c1\uc5d0 \uc55e\uc11c \uc7a0\uc2dc \uc7ac\ubc1c\uae09\ub418\uc5c8\ub2e4. \uce7c\ud14d\uc758 \ub9ce\uc740 \uce5c\uad6c\ub4e4\ub3c4 \uadf8\ub97c \uc678\uba74\ud558\uae30 \uc2dc\uc791\ud588\uace0 \ucd1d\uc7a5 \ub450\ube0c\ub9ac\uc9c0\ub294 \ud3f4\ub9c1\uc5d0\uac8c \uc815\uce58\uc801 \uc5f0\uc124\uc744 \uadf8\ub9cc\ub458 \uac83\uc744 \uc694\uccad\ud558\uc600\ub2e4. \uadf8\ub294 1951\ub144 \ub17c\ub780\uc774 \ub418\ub294 \ub2e8\uccb4\ub4e4\uc744 \ud0c8\ud1f4\ud558\uace0 \uc5f0\uc124\ub4e4\uc744 \uc911\uc9c0\ud558\uc600\uc73c\uba70 \uc5f0\uad6c\uc5d0\ub9cc \uc804\ub150\ud558\uc600\uc73c\ub098, \uc5b8\ub860\uc758 \ube44\ub09c\uacfc \uad6d\ud68c\uc758 \uc758\uc2ec\uc740 \uacc4\uc18d\ub418\uc5c8\ub2e4. 1954\ub144 \ub178\ubca8 \ud654\ud559\uc0c1\uc744 \uc218\uc0c1\ud55c \ud6c4\uc5d0\ub294 \ub300\uc911\uacfc \uc9c0\uc778\uc758 \uc758\uc2ec\uc774 \ub9ce\uc774 \ud480\ub838\uace0 \ud3f4\ub9c1\uc740 \uc138\uacc4 \uacf3\uacf3\uc744 \ub3cc\uc544\ub2e4\ub2c8\uba70 \uc6d0\uc790\ud3ed\ud0c4 \uc0ac\uc6a9\uc5d0 \ubc18\ub300\ud558\ub294 \uc5f0\uc124\uc744 \ud558\uc600\ub2e4. \ud3f4\ub9c1\uc740 \ub2e4\uc2dc \uc6d0\uc790\ud3ed\ud0c4\uc73c\ub85c \uc778\ud55c \ubc29\uc0ac\ub2a5 \ud53c\ud3ed\uacfc \ub099\uc9c4\uc5d0 \uad00\uc2ec\uc744 \uac00\uc9c0\uae30 \uc2dc\uc791\ud558\uc600\ub2e4. 1958\ub144 \ud3f4\ub9c1\uc740 \ubc30\ub9ac \ucee4\uba38\ub108 \ub4f1\uacfc \ud568\uaed8 \ubc29\uc0ac\uc131 \ub3d9\uc704\uc6d0\uc18c Sr-90\uc758 \uc720\uc544 \uce58\uc544\uc5d0 \uc5b4\ub5a4 \uc601\ud5a5\uc744 \ub07c\uce58\ub294\uc9c0 \uc5f0\uad6c\ud558\uc600\uace0 1962\ub144\uc5d0\ub294 \ub8e8\uc774\uc2a4 \ub77c\uc774\uc2a4 \ub4f1\uacfc \ud568\uaed8 \ub099\uc9c4\uc73c\ub85c \ub5a8\uc5b4\uc9c4 Sr-90\uc744 \uc816\uc18c\uac00 \uba39\uace0 \uc816\uc18c\ub85c\ubd80\ud130 \uc6b0\uc720\ub97c \uba39\ub294 \uc0ac\ub78c\uc758 \ubf08\uc5d0 \ucd95\uc801\ub41c\ub2e4\ub294 \uc5f0\uad6c\uacb0\uacfc\ub97c \ubc1c\ud45c\ud558\uc600\ub2e4. \ub610\ud55c \ubb3c\ub9ac\ud559\uc790\uc774\uc790 \ubc18\uacf5\uc8fc\uc758\uc790\uc778 \ud154\ub7ec\uc640 \ubc29\uc0ac\ub2a5 \ub099\uc9c4\uc758 \uc704\ud5d8\uc5d0 \ub300\ud55c TV\ud1a0\ub860\uc5d0\ub3c4 \ucc38\uc5ec\ud558\uc600\ub2e4. \ub610\ud55c \ud3f4\ub9c1\uc740 1958\ub144 \uc138\uacc4 \uac01\uad6d\uc758 \uacfc\ud559\uc790\ub4e4\ub85c\ubd80\ud130 \ud575\uc2e4\ud5d8 \uc911\uc9c0\ub97c \uccad\uc6d0\ud558\ub294 \uc11c\uba85\uc744 \ubc1b\uc544 \uc11c\uba85\ubaa9\ub85d\uc744 UN\uc5d0 \uc81c\ucd9c\ud558\uc600\ub2e4. \ub300\uc911\uc758 \uc555\ubc15\uacfc \ub099\uc9c4\uc758 \uc704\ud5d8\uc5d0 \uad00\ud55c \uc5f0\uad6c\uacb0\uacfc\ub294 1963\ub144 \ubbf8\uad6d\uacfc \uc18c\ub828\uc774 \uc9c0\ud558\ub97c \uc81c\uc678\ud55c \ubaa8\ub4e0 \uacf3\uc5d0\uc11c \ud575\uc2e4\ud5d8\uc744 \uae08\uc9c0\ud558\ub294 \ubd80\ubd84\uc801 \ud575\uc2e4\ud5d8 \uae08\uc9c0\uc870\uc57d\uc5d0 \uc11c\uba85\ud558\uac8c \ub418\ub294 \uacb0\uacfc\ub97c \uac00\uc838\uc654\ub2e4. \uc774 \uc870\uc57d\uc740 \uadf8\ud574 10\uc6d4 10\uc77c\ubd80\ud130 \ud6a8\ub825\uc774 \ubc1c\uc0dd\ud588\uace0 \ubc14\ub85c \uadf8\ub0a0 \ud3f4\ub9c1\uc740 1962\ub144 \ub178\ubca8 \ud3c9\ud654\uc0c1\uc744 \uc218\uc0c1\ud558\uac8c \ub41c\ub2e4. \ud3f4\ub9c1\uc758 \ud654\ud559\uc5d0 \ub300\ud55c \uacf5\ud5cc\uc744 \uc778\uc815\ud55c \uacfc\ud559\uc790\ub4e4\uc744 \ud3ec\ud568\ud55c \ub9ce\uc740 \ub17c\uac1d\ub4e4\uc740 \ud3f4\ub9c1\uc758 \uc815\uce58\uc801 \ud589\ubcf4\uc5d0 \ube44\ud310\uc801\uc774\uc5c8\uc73c\uba70 \uadf8\ub97c \uacf5\uc0b0\uc8fc\uc758\uc790\ub77c\uace0 \ube44\ub09c\ud558\uc600\ub2e4. 1960\ub144 \ud3f4\ub9c1\uc740 \uadf8\ub97c \uacf5\uc0b0\uc8fc\uc758\uc790\ub77c\uace0 \ub099\uc778 \ucc0d\uc740 \uc0c1\uc6d0\uc758 \ud55c \uc870\uc0ac \uc18c\uc704\uc6d0\ud68c\uc5d0 \ucd9c\ub450\ub97c \uba85\ub839\ubc1b\uc544 \uadf8\ud1a0\ub85d \ub9ce\uc740 \uc11c\uba85\ub4e4\uc744 \ubaa8\uc740 \uacfc\uc815\uc5d0 \ub300\ud574 \ud574\uba85\uc744 \uc694\uad6c\ubc1b\uc558\ub2e4. \uadf8\uac00 \ub178\ubca8 \ud3c9\ud654\uc0c1\uc744 \uc218\uc0c1\ud55c \uc774\ud6c4\uc5d0\ub3c4 \ube44\ud310\uc801\uc778 \uc5ec\ub860\uc740 \uacc4\uc18d\ub418\uc5c8\ub2e4. \uadf8\uac00 1962\ub144 \ub178\ubca8\ud3c9\ud654\uc0c1\uc744 \ubc1b\uac8c \ub418\uc790 \ubbf8\uad6d \ub77c\uc774\ud504 \uc7a1\uc9c0\uc0ac\ub294 \u201c\ub178\ub974\uc6e8\uc774\uc5d0\uc11c \uc628 \uc218\uc0c1\ud55c \ubaa8\uc695\u201d\uc774\ub780 \ud5e4\ub4dc\ub77c\uc778\uc744 \ub0c8\ub2e4. \uce7c\ud14d \ucd1d\uc7a5 \ub450\ube0c\ub9ac\uc9c0 \uc5ed\uc2dc \ubbf8\uc801\uc9c0\uadfc\ud55c \ubc18\uc751\ub9cc\uc744 \ubcf4\uc600\ub2e4. \ud3f4\ub9c1\uc740 \ub178\ubca8\uc0c1 \uc0c1\uae08\uc744 \uac00\uc9c0\uace0 \ud3c9\ud654\uc5d0 \uad00\ud55c \uc77c\uc744 \ud558\uae30\ub85c \uacb0\uc2ec\ud558\uace0 1963\ub144 10\uc6d4 18\uc77c \uae30\uc790\ud68c\uacac\uc744 \uc5f4\uc5b4 \uce7c\ud14d\uc744 \ub5a0\ub09c\ub2e4\uace0 \ubc1c\ud45c\ud588\ub2e4.", "paragraph_answer": "\ub0c9\uc804\uccb4\uc81c\uac00 \uac15\ud654\ub418\uace0 \uad6d\uac00\uc801\uc73c\ub85c \ubc18\uacf5\uc8fc\uc758\uac00 \ub300\uc138\uc600\ub358 1950\ub144\ub300\uc5d0\ub3c4 \uadf8\ub294 \ubc18\ud575\uc5f0\uc124\uc744 \uacc4\uc18d\ud558\uc600\ub2e4. \uadf8 \ub54c\ubb38\uc5d0 FBI\uc758 \uc870\uc0ac\ub97c \ubc1b\uae30\ub3c4 \ud558\uace0, \ubbf8 \uc815\ubd80\ub85c\ubd80\ud130 \uc5ec\uad8c\ubc1c\uae09\uc744 \uac70\uc808\ub2f9\ud558\uae30\ub3c4 \uc544\uc600\ub2e4. \uadf8\uc758 \uc5ec\uad8c\uc740 1954\ub144 \uadf8\uc758 \uccab \ub178\ubca8\uc0c1 \uc218\uc0c1\uc5d0 \uc55e\uc11c \uc7a0\uc2dc \uc7ac\ubc1c\uae09\ub418\uc5c8\ub2e4. \uce7c\ud14d\uc758 \ub9ce\uc740 \uce5c\uad6c\ub4e4\ub3c4 \uadf8\ub97c \uc678\uba74\ud558\uae30 \uc2dc\uc791\ud588\uace0 \ucd1d\uc7a5 \ub450\ube0c\ub9ac\uc9c0\ub294 \ud3f4\ub9c1\uc5d0\uac8c \uc815\uce58\uc801 \uc5f0\uc124\uc744 \uadf8\ub9cc\ub458 \uac83\uc744 \uc694\uccad\ud558\uc600\ub2e4. \uadf8\ub294 1951\ub144 \ub17c\ub780\uc774 \ub418\ub294 \ub2e8\uccb4\ub4e4\uc744 \ud0c8\ud1f4\ud558\uace0 \uc5f0\uc124\ub4e4\uc744 \uc911\uc9c0\ud558\uc600\uc73c\uba70 \uc5f0\uad6c\uc5d0\ub9cc \uc804\ub150\ud558\uc600\uc73c\ub098, \uc5b8\ub860\uc758 \ube44\ub09c\uacfc \uad6d\ud68c\uc758 \uc758\uc2ec\uc740 \uacc4\uc18d\ub418\uc5c8\ub2e4. 1954\ub144 \ub178\ubca8 \ud654\ud559\uc0c1\uc744 \uc218\uc0c1\ud55c \ud6c4\uc5d0\ub294 \ub300\uc911\uacfc \uc9c0\uc778\uc758 \uc758\uc2ec\uc774 \ub9ce\uc774 \ud480\ub838\uace0 \ud3f4\ub9c1\uc740 \uc138\uacc4 \uacf3\uacf3\uc744 \ub3cc\uc544\ub2e4\ub2c8\uba70 \uc6d0\uc790\ud3ed\ud0c4 \uc0ac\uc6a9\uc5d0 \ubc18\ub300\ud558\ub294 \uc5f0\uc124\uc744 \ud558\uc600\ub2e4. \ud3f4\ub9c1\uc740 \ub2e4\uc2dc \uc6d0\uc790\ud3ed\ud0c4\uc73c\ub85c \uc778\ud55c \ubc29\uc0ac\ub2a5 \ud53c\ud3ed\uacfc \ub099\uc9c4\uc5d0 \uad00\uc2ec\uc744 \uac00\uc9c0\uae30 \uc2dc\uc791\ud558\uc600\ub2e4. 1958\ub144 \ud3f4\ub9c1\uc740 \ubc30\ub9ac \ucee4\uba38\ub108 \ub4f1\uacfc \ud568\uaed8 \ubc29\uc0ac\uc131 \ub3d9\uc704\uc6d0\uc18c Sr-90\uc758 \uc720\uc544 \uce58\uc544\uc5d0 \uc5b4\ub5a4 \uc601\ud5a5\uc744 \ub07c\uce58\ub294\uc9c0 \uc5f0\uad6c\ud558\uc600\uace0 1962\ub144\uc5d0\ub294 \ub8e8\uc774\uc2a4 \ub77c\uc774\uc2a4 \ub4f1\uacfc \ud568\uaed8 \ub099\uc9c4\uc73c\ub85c \ub5a8\uc5b4\uc9c4 Sr-90\uc744 \uc816\uc18c\uac00 \uba39\uace0 \uc816\uc18c\ub85c\ubd80\ud130 \uc6b0\uc720\ub97c \uba39\ub294 \uc0ac\ub78c\uc758 \ubf08\uc5d0 \ucd95\uc801\ub41c\ub2e4\ub294 \uc5f0\uad6c\uacb0\uacfc\ub97c \ubc1c\ud45c\ud558\uc600\ub2e4. \ub610\ud55c \ubb3c\ub9ac\ud559\uc790\uc774\uc790 \ubc18\uacf5\uc8fc\uc758\uc790\uc778 \ud154\ub7ec\uc640 \ubc29\uc0ac\ub2a5 \ub099\uc9c4\uc758 \uc704\ud5d8\uc5d0 \ub300\ud55c TV\ud1a0\ub860\uc5d0\ub3c4 \ucc38\uc5ec\ud558\uc600\ub2e4. \ub610\ud55c \ud3f4\ub9c1\uc740 1958\ub144 \uc138\uacc4 \uac01\uad6d\uc758 \uacfc\ud559\uc790\ub4e4\ub85c\ubd80\ud130 \ud575\uc2e4\ud5d8 \uc911\uc9c0\ub97c \uccad\uc6d0\ud558\ub294 \uc11c\uba85\uc744 \ubc1b\uc544 \uc11c\uba85\ubaa9\ub85d\uc744 UN\uc5d0 \uc81c\ucd9c\ud558\uc600\ub2e4. \ub300\uc911\uc758 \uc555\ubc15\uacfc \ub099\uc9c4\uc758 \uc704\ud5d8\uc5d0 \uad00\ud55c \uc5f0\uad6c\uacb0\uacfc\ub294 1963\ub144 \ubbf8\uad6d\uacfc \uc18c\ub828\uc774 \uc9c0\ud558\ub97c \uc81c\uc678\ud55c \ubaa8\ub4e0 \uacf3\uc5d0\uc11c \ud575\uc2e4\ud5d8\uc744 \uae08\uc9c0\ud558\ub294 \ubd80\ubd84\uc801 \ud575\uc2e4\ud5d8 \uae08\uc9c0\uc870\uc57d\uc5d0 \uc11c\uba85\ud558\uac8c \ub418\ub294 \uacb0\uacfc\ub97c \uac00\uc838\uc654\ub2e4. \uc774 \uc870\uc57d\uc740 \uadf8\ud574 10\uc6d4 10\uc77c\ubd80\ud130 \ud6a8\ub825\uc774 \ubc1c\uc0dd\ud588\uace0 \ubc14\ub85c \uadf8\ub0a0 \ud3f4\ub9c1\uc740 1962\ub144 \ub178\ubca8 \ud3c9\ud654\uc0c1\uc744 \uc218\uc0c1\ud558\uac8c \ub41c\ub2e4. \ud3f4\ub9c1\uc758 \ud654\ud559\uc5d0 \ub300\ud55c \uacf5\ud5cc\uc744 \uc778\uc815\ud55c \uacfc\ud559\uc790\ub4e4\uc744 \ud3ec\ud568\ud55c \ub9ce\uc740 \ub17c\uac1d\ub4e4\uc740 \ud3f4\ub9c1\uc758 \uc815\uce58\uc801 \ud589\ubcf4\uc5d0 \ube44\ud310\uc801\uc774\uc5c8\uc73c\uba70 \uadf8\ub97c \uacf5\uc0b0\uc8fc\uc758\uc790\ub77c\uace0 \ube44\ub09c\ud558\uc600\ub2e4. 1960\ub144 \ud3f4\ub9c1\uc740 \uadf8\ub97c \uacf5\uc0b0\uc8fc\uc758\uc790\ub77c\uace0 \ub099\uc778 \ucc0d\uc740 \uc0c1\uc6d0\uc758 \ud55c \uc870\uc0ac \uc18c\uc704\uc6d0\ud68c\uc5d0 \ucd9c\ub450\ub97c \uba85\ub839\ubc1b\uc544 \uadf8\ud1a0\ub85d \ub9ce\uc740 \uc11c\uba85\ub4e4\uc744 \ubaa8\uc740 \uacfc\uc815\uc5d0 \ub300\ud574 \ud574\uba85\uc744 \uc694\uad6c\ubc1b\uc558\ub2e4. \uadf8\uac00 \ub178\ubca8 \ud3c9\ud654\uc0c1\uc744 \uc218\uc0c1\ud55c \uc774\ud6c4\uc5d0\ub3c4 \ube44\ud310\uc801\uc778 \uc5ec\ub860\uc740 \uacc4\uc18d\ub418\uc5c8\ub2e4. \uadf8\uac00 1962\ub144 \ub178\ubca8\ud3c9\ud654\uc0c1\uc744 \ubc1b\uac8c \ub418\uc790 \ubbf8\uad6d \ub77c\uc774\ud504 \uc7a1\uc9c0\uc0ac\ub294 \u201c\ub178\ub974\uc6e8\uc774\uc5d0\uc11c \uc628 \uc218\uc0c1\ud55c \ubaa8\uc695\u201d\uc774\ub780 \ud5e4\ub4dc\ub77c\uc778\uc744 \ub0c8\ub2e4. \uce7c\ud14d \ucd1d\uc7a5 \ub450\ube0c\ub9ac\uc9c0 \uc5ed\uc2dc \ubbf8\uc801\uc9c0\uadfc\ud55c \ubc18\uc751\ub9cc\uc744 \ubcf4\uc600\ub2e4. \ud3f4\ub9c1\uc740 \ub178\ubca8\uc0c1 \uc0c1\uae08\uc744 \uac00\uc9c0\uace0 \ud3c9\ud654\uc5d0 \uad00\ud55c \uc77c\uc744 \ud558\uae30\ub85c \uacb0\uc2ec\ud558\uace0 1963\ub144 10\uc6d4 18\uc77c \uae30\uc790\ud68c\uacac\uc744 \uc5f4\uc5b4 \uce7c\ud14d\uc744 \ub5a0\ub09c\ub2e4\uace0 \ubc1c\ud45c\ud588\ub2e4.", "sentence_answer": " 1958\ub144 \ud3f4\ub9c1\uc740 \ubc30\ub9ac \ucee4\uba38\ub108 \ub4f1\uacfc \ud568\uaed8 \ubc29\uc0ac\uc131 \ub3d9\uc704\uc6d0\uc18c Sr-90\uc758 \uc720\uc544 \uce58\uc544\uc5d0 \uc5b4\ub5a4 \uc601\ud5a5\uc744 \ub07c\uce58\ub294\uc9c0 \uc5f0\uad6c\ud558\uc600\uace0 1962\ub144\uc5d0\ub294 \ub8e8\uc774\uc2a4 \ub77c\uc774\uc2a4 \ub4f1\uacfc \ud568\uaed8 \ub099\uc9c4\uc73c\ub85c \ub5a8\uc5b4\uc9c4 Sr-90\uc744 \uc816\uc18c\uac00 \uba39\uace0 \uc816\uc18c\ub85c\ubd80\ud130 \uc6b0\uc720\ub97c \uba39\ub294 \uc0ac\ub78c\uc758 \ubf08\uc5d0 \ucd95\uc801\ub41c\ub2e4\ub294 \uc5f0\uad6c\uacb0\uacfc\ub97c \ubc1c\ud45c\ud558\uc600\ub2e4.", "paragraph_id": "6579230-16-2", "paragraph_question": "question: \uc138\uacc4 \uac01\uad6d\uc758 \uacfc\ud559\uc790\ub4e4\ub85c\ubd80\ud130 \ud575\uc2e4\ud5d8 \uc911\uc9c0\ub97c \uccad\uc6d0\ud558\ub294 \uc11c\uba85\uc744 \ubc1b\uc544 \uc11c\uba85\ubaa9\ub85d\uc744 UN\uc5d0 \uc81c\ucd9c\ud55c \ud574\ub294?, context: \ub0c9\uc804\uccb4\uc81c\uac00 \uac15\ud654\ub418\uace0 \uad6d\uac00\uc801\uc73c\ub85c \ubc18\uacf5\uc8fc\uc758\uac00 \ub300\uc138\uc600\ub358 1950\ub144\ub300\uc5d0\ub3c4 \uadf8\ub294 \ubc18\ud575\uc5f0\uc124\uc744 \uacc4\uc18d\ud558\uc600\ub2e4. \uadf8 \ub54c\ubb38\uc5d0 FBI\uc758 \uc870\uc0ac\ub97c \ubc1b\uae30\ub3c4 \ud558\uace0, \ubbf8 \uc815\ubd80\ub85c\ubd80\ud130 \uc5ec\uad8c\ubc1c\uae09\uc744 \uac70\uc808\ub2f9\ud558\uae30\ub3c4 \uc544\uc600\ub2e4. \uadf8\uc758 \uc5ec\uad8c\uc740 1954\ub144 \uadf8\uc758 \uccab \ub178\ubca8\uc0c1 \uc218\uc0c1\uc5d0 \uc55e\uc11c \uc7a0\uc2dc \uc7ac\ubc1c\uae09\ub418\uc5c8\ub2e4. \uce7c\ud14d\uc758 \ub9ce\uc740 \uce5c\uad6c\ub4e4\ub3c4 \uadf8\ub97c \uc678\uba74\ud558\uae30 \uc2dc\uc791\ud588\uace0 \ucd1d\uc7a5 \ub450\ube0c\ub9ac\uc9c0\ub294 \ud3f4\ub9c1\uc5d0\uac8c \uc815\uce58\uc801 \uc5f0\uc124\uc744 \uadf8\ub9cc\ub458 \uac83\uc744 \uc694\uccad\ud558\uc600\ub2e4. \uadf8\ub294 1951\ub144 \ub17c\ub780\uc774 \ub418\ub294 \ub2e8\uccb4\ub4e4\uc744 \ud0c8\ud1f4\ud558\uace0 \uc5f0\uc124\ub4e4\uc744 \uc911\uc9c0\ud558\uc600\uc73c\uba70 \uc5f0\uad6c\uc5d0\ub9cc \uc804\ub150\ud558\uc600\uc73c\ub098, \uc5b8\ub860\uc758 \ube44\ub09c\uacfc \uad6d\ud68c\uc758 \uc758\uc2ec\uc740 \uacc4\uc18d\ub418\uc5c8\ub2e4. 1954\ub144 \ub178\ubca8 \ud654\ud559\uc0c1\uc744 \uc218\uc0c1\ud55c \ud6c4\uc5d0\ub294 \ub300\uc911\uacfc \uc9c0\uc778\uc758 \uc758\uc2ec\uc774 \ub9ce\uc774 \ud480\ub838\uace0 \ud3f4\ub9c1\uc740 \uc138\uacc4 \uacf3\uacf3\uc744 \ub3cc\uc544\ub2e4\ub2c8\uba70 \uc6d0\uc790\ud3ed\ud0c4 \uc0ac\uc6a9\uc5d0 \ubc18\ub300\ud558\ub294 \uc5f0\uc124\uc744 \ud558\uc600\ub2e4. \ud3f4\ub9c1\uc740 \ub2e4\uc2dc \uc6d0\uc790\ud3ed\ud0c4\uc73c\ub85c \uc778\ud55c \ubc29\uc0ac\ub2a5 \ud53c\ud3ed\uacfc \ub099\uc9c4\uc5d0 \uad00\uc2ec\uc744 \uac00\uc9c0\uae30 \uc2dc\uc791\ud558\uc600\ub2e4. 1958\ub144 \ud3f4\ub9c1\uc740 \ubc30\ub9ac \ucee4\uba38\ub108 \ub4f1\uacfc \ud568\uaed8 \ubc29\uc0ac\uc131 \ub3d9\uc704\uc6d0\uc18c Sr-90\uc758 \uc720\uc544 \uce58\uc544\uc5d0 \uc5b4\ub5a4 \uc601\ud5a5\uc744 \ub07c\uce58\ub294\uc9c0 \uc5f0\uad6c\ud558\uc600\uace0 1962\ub144\uc5d0\ub294 \ub8e8\uc774\uc2a4 \ub77c\uc774\uc2a4 \ub4f1\uacfc \ud568\uaed8 \ub099\uc9c4\uc73c\ub85c \ub5a8\uc5b4\uc9c4 Sr-90\uc744 \uc816\uc18c\uac00 \uba39\uace0 \uc816\uc18c\ub85c\ubd80\ud130 \uc6b0\uc720\ub97c \uba39\ub294 \uc0ac\ub78c\uc758 \ubf08\uc5d0 \ucd95\uc801\ub41c\ub2e4\ub294 \uc5f0\uad6c\uacb0\uacfc\ub97c \ubc1c\ud45c\ud558\uc600\ub2e4. \ub610\ud55c \ubb3c\ub9ac\ud559\uc790\uc774\uc790 \ubc18\uacf5\uc8fc\uc758\uc790\uc778 \ud154\ub7ec\uc640 \ubc29\uc0ac\ub2a5 \ub099\uc9c4\uc758 \uc704\ud5d8\uc5d0 \ub300\ud55c TV\ud1a0\ub860\uc5d0\ub3c4 \ucc38\uc5ec\ud558\uc600\ub2e4. \ub610\ud55c \ud3f4\ub9c1\uc740 1958\ub144 \uc138\uacc4 \uac01\uad6d\uc758 \uacfc\ud559\uc790\ub4e4\ub85c\ubd80\ud130 \ud575\uc2e4\ud5d8 \uc911\uc9c0\ub97c \uccad\uc6d0\ud558\ub294 \uc11c\uba85\uc744 \ubc1b\uc544 \uc11c\uba85\ubaa9\ub85d\uc744 UN\uc5d0 \uc81c\ucd9c\ud558\uc600\ub2e4. \ub300\uc911\uc758 \uc555\ubc15\uacfc \ub099\uc9c4\uc758 \uc704\ud5d8\uc5d0 \uad00\ud55c \uc5f0\uad6c\uacb0\uacfc\ub294 1963\ub144 \ubbf8\uad6d\uacfc \uc18c\ub828\uc774 \uc9c0\ud558\ub97c \uc81c\uc678\ud55c \ubaa8\ub4e0 \uacf3\uc5d0\uc11c \ud575\uc2e4\ud5d8\uc744 \uae08\uc9c0\ud558\ub294 \ubd80\ubd84\uc801 \ud575\uc2e4\ud5d8 \uae08\uc9c0\uc870\uc57d\uc5d0 \uc11c\uba85\ud558\uac8c \ub418\ub294 \uacb0\uacfc\ub97c \uac00\uc838\uc654\ub2e4. \uc774 \uc870\uc57d\uc740 \uadf8\ud574 10\uc6d4 10\uc77c\ubd80\ud130 \ud6a8\ub825\uc774 \ubc1c\uc0dd\ud588\uace0 \ubc14\ub85c \uadf8\ub0a0 \ud3f4\ub9c1\uc740 1962\ub144 \ub178\ubca8 \ud3c9\ud654\uc0c1\uc744 \uc218\uc0c1\ud558\uac8c \ub41c\ub2e4. \ud3f4\ub9c1\uc758 \ud654\ud559\uc5d0 \ub300\ud55c \uacf5\ud5cc\uc744 \uc778\uc815\ud55c \uacfc\ud559\uc790\ub4e4\uc744 \ud3ec\ud568\ud55c \ub9ce\uc740 \ub17c\uac1d\ub4e4\uc740 \ud3f4\ub9c1\uc758 \uc815\uce58\uc801 \ud589\ubcf4\uc5d0 \ube44\ud310\uc801\uc774\uc5c8\uc73c\uba70 \uadf8\ub97c \uacf5\uc0b0\uc8fc\uc758\uc790\ub77c\uace0 \ube44\ub09c\ud558\uc600\ub2e4. 1960\ub144 \ud3f4\ub9c1\uc740 \uadf8\ub97c \uacf5\uc0b0\uc8fc\uc758\uc790\ub77c\uace0 \ub099\uc778 \ucc0d\uc740 \uc0c1\uc6d0\uc758 \ud55c \uc870\uc0ac \uc18c\uc704\uc6d0\ud68c\uc5d0 \ucd9c\ub450\ub97c \uba85\ub839\ubc1b\uc544 \uadf8\ud1a0\ub85d \ub9ce\uc740 \uc11c\uba85\ub4e4\uc744 \ubaa8\uc740 \uacfc\uc815\uc5d0 \ub300\ud574 \ud574\uba85\uc744 \uc694\uad6c\ubc1b\uc558\ub2e4. \uadf8\uac00 \ub178\ubca8 \ud3c9\ud654\uc0c1\uc744 \uc218\uc0c1\ud55c \uc774\ud6c4\uc5d0\ub3c4 \ube44\ud310\uc801\uc778 \uc5ec\ub860\uc740 \uacc4\uc18d\ub418\uc5c8\ub2e4. \uadf8\uac00 1962\ub144 \ub178\ubca8\ud3c9\ud654\uc0c1\uc744 \ubc1b\uac8c \ub418\uc790 \ubbf8\uad6d \ub77c\uc774\ud504 \uc7a1\uc9c0\uc0ac\ub294 \u201c\ub178\ub974\uc6e8\uc774\uc5d0\uc11c \uc628 \uc218\uc0c1\ud55c \ubaa8\uc695\u201d\uc774\ub780 \ud5e4\ub4dc\ub77c\uc778\uc744 \ub0c8\ub2e4. \uce7c\ud14d \ucd1d\uc7a5 \ub450\ube0c\ub9ac\uc9c0 \uc5ed\uc2dc \ubbf8\uc801\uc9c0\uadfc\ud55c \ubc18\uc751\ub9cc\uc744 \ubcf4\uc600\ub2e4. \ud3f4\ub9c1\uc740 \ub178\ubca8\uc0c1 \uc0c1\uae08\uc744 \uac00\uc9c0\uace0 \ud3c9\ud654\uc5d0 \uad00\ud55c \uc77c\uc744 \ud558\uae30\ub85c \uacb0\uc2ec\ud558\uace0 1963\ub144 10\uc6d4 18\uc77c \uae30\uc790\ud68c\uacac\uc744 \uc5f4\uc5b4 \uce7c\ud14d\uc744 \ub5a0\ub09c\ub2e4\uace0 \ubc1c\ud45c\ud588\ub2e4."} {"question": "\ub098\uce74\ubaa8\ub9ac \uc544\ud0a4\ub098\uc758 26\ubc88\uc9f8 \uc2f1\uae00 \ubc1c\ub9e4 \ub144\ub3c4\ub294?", "paragraph": "\ub098\uce74\ubaa8\ub9ac \uc544\ud0a4\ub098\ub294 1984\ub144 \uc790\uc2e0\uc758 5\ubc88\uc9f8 \uc815\uaddc \uc74c\ubc18\uc778 \u300aANNIVERSARY\u300b\uc758 \uc218\ub85d\uace1 \u3008\uafc8\uc744 \uafb8\uac8c \ud574\u3009(\u5922\u3092\u898b\u3055\u305b\u3066)\uc758 \uac00\uc0ac\ub97c \uc9c1\uc811 \uc4f4 \uc774\ud6c4 \uc870\uae08\uc529 \uc791\uc0ac\uc5d0\ub3c4 \uc190\uc744 \ub300\uae30 \uc2dc\uc791\ud588\ub2e4. \uc2f1\uae00\ub85c\ub294 1991\ub144 \uc790\uc2e0\uc758 26\ubc88\uc9f8 \uc2f1\uae00\uc774\uc5c8\ub358 \u3008\ud6c4\ud0c0\ub9ac\uc2dc\uc988\uce74 -\u300c\ucc9c\ud558\uc804\uc124\uc0b4\uc778\uc0ac\uac74\u300d\uc73c\ub85c\ubd80\ud130\u3009\uc758 B\uc0ac\uc774\ub4dc \uace1 \u3008\uc78a\uc5b4\u2026\u3009(\u5fd8\u308c\u3066\u2026) \uc758 \uc791\uc0ac\ub97c \uc2dc\uc791\uc73c\ub85c 1995\ub144 32\ubc88\uc9f8 \uc2f1\uae00 \u3008Tokyo Rose\u3009\uc5d0\uc11c\ub3c4 \uacf5\ub3d9 \uc791\uc0ac\uac00\ub85c \ucc38\uc5ec, 2002\ub144 \uc790\uc2e0\uc758 20\ubc88\uc9f8 \uc74c\ubc18 \u300aResonancia\u300b\uc758 \uc218\ub85d\uace1 \u3008Deseo\u3009\ubd80\ud130\ub294 \uc790\uc2e0\uc758 \ubcf8\uba85\uc744 \ubc1d\ud788\ub294 \ub300\uc2e0 Miran:Miran\uc774\ub77c\ub294 \ud544\uba85\uc744 \uc4f0\uae30\ub3c4 \ud588\ub2e4. 2003\ub144\uc5d0 4\uc6d4\uc5d0 \ubc1c\ub9e4\ud55c 42\ubc88\uc9f8 \uc2f1\uae00 \u3008Days\u3009\ub294 \ubcf8\uba85 \uba85\uc758\ub85c \ub2e8\ub3c5 \uc791\uc0ac, \uc2f1\uae00\uc758 A\uc0ac\uc774\ub4dc \uace1\uc73c\ub85c \uc218\ub85d\ub418\uc5c8\uace0, \ub2e4\uc74c\ub2ec\uc5d0 \ucd9c\uc2dc\ud55c 21\ubc88\uc9f8 \uc815\uaddc \uc74c\ubc18 \u300aI hope so\u300b\uc5d0\uc11c\ub294 \u3008Days\u3009\ub97c \ud3ec\ud568\ud558\uc5ec \ubcf8\uba85 \uba85\uc758\ub85c \uc791\uc0ac\ud55c \uace1 4\uac1c\uac00 \uc218\ub85d\ub418\uc5b4 \uc788\ub294 \ub4f1 \uc791\uc0ac\uc758 \uc601\uc5ed \ub610\ud55c \uc810\ucc28 \ub298\ub824\ub098\uac14\ub2e4.", "answer": "1991\ub144", "sentence": "\uc2f1\uae00\ub85c\ub294 1991\ub144 \uc790\uc2e0\uc758 26\ubc88\uc9f8 \uc2f1\uae00\uc774\uc5c8\ub358 \u3008\ud6c4\ud0c0\ub9ac\uc2dc\uc988\uce74 -\u300c\ucc9c\ud558\uc804\uc124\uc0b4\uc778\uc0ac\uac74\u300d\uc73c\ub85c\ubd80\ud130\u3009\uc758 B\uc0ac\uc774\ub4dc \uace1 \u3008\uc78a\uc5b4\u2026\u3009(\u5fd8\u308c\u3066\u2026) \uc758 \uc791\uc0ac\ub97c \uc2dc\uc791\uc73c\ub85c 1995\ub144 32\ubc88\uc9f8 \uc2f1\uae00 \u3008Tokyo Rose\u3009\uc5d0\uc11c\ub3c4 \uacf5\ub3d9 \uc791\uc0ac\uac00\ub85c \ucc38\uc5ec, 2002\ub144 \uc790\uc2e0\uc758 20\ubc88\uc9f8 \uc74c\ubc18 \u300aResonancia\u300b\uc758 \uc218\ub85d\uace1 \u3008Deseo\u3009\ubd80\ud130\ub294 \uc790\uc2e0\uc758 \ubcf8\uba85\uc744 \ubc1d\ud788\ub294 \ub300\uc2e0 Miran:Miran\uc774\ub77c\ub294 \ud544\uba85\uc744 \uc4f0\uae30\ub3c4 \ud588\ub2e4.", "paragraph_sentence": "\ub098\uce74\ubaa8\ub9ac \uc544\ud0a4\ub098\ub294 1984\ub144 \uc790\uc2e0\uc758 5\ubc88\uc9f8 \uc815\uaddc \uc74c\ubc18\uc778 \u300aANNIVERSARY\u300b\uc758 \uc218\ub85d\uace1 \u3008\uafc8\uc744 \uafb8\uac8c \ud574\u3009(\u5922\u3092\u898b\u3055\u305b\u3066)\uc758 \uac00\uc0ac\ub97c \uc9c1\uc811 \uc4f4 \uc774\ud6c4 \uc870\uae08\uc529 \uc791\uc0ac\uc5d0\ub3c4 \uc190\uc744 \ub300\uae30 \uc2dc\uc791\ud588\ub2e4. \uc2f1\uae00\ub85c\ub294 1991\ub144 \uc790\uc2e0\uc758 26\ubc88\uc9f8 \uc2f1\uae00\uc774\uc5c8\ub358 \u3008\ud6c4\ud0c0\ub9ac\uc2dc\uc988\uce74 -\u300c\ucc9c\ud558\uc804\uc124\uc0b4\uc778\uc0ac\uac74\u300d\uc73c\ub85c\ubd80\ud130\u3009\uc758 B\uc0ac\uc774\ub4dc \uace1 \u3008\uc78a\uc5b4\u2026\u3009(\u5fd8\u308c\u3066\u2026) \uc758 \uc791\uc0ac\ub97c \uc2dc\uc791\uc73c\ub85c 1995\ub144 32\ubc88\uc9f8 \uc2f1\uae00 \u3008Tokyo Rose\u3009\uc5d0\uc11c\ub3c4 \uacf5\ub3d9 \uc791\uc0ac\uac00\ub85c \ucc38\uc5ec, 2002\ub144 \uc790\uc2e0\uc758 20\ubc88\uc9f8 \uc74c\ubc18 \u300aResonancia\u300b\uc758 \uc218\ub85d\uace1 \u3008Deseo\u3009\ubd80\ud130\ub294 \uc790\uc2e0\uc758 \ubcf8\uba85\uc744 \ubc1d\ud788\ub294 \ub300\uc2e0 Miran:Miran\uc774\ub77c\ub294 \ud544\uba85\uc744 \uc4f0\uae30\ub3c4 \ud588\ub2e4. 2003\ub144\uc5d0 4\uc6d4\uc5d0 \ubc1c\ub9e4\ud55c 42\ubc88\uc9f8 \uc2f1\uae00 \u3008Days\u3009\ub294 \ubcf8\uba85 \uba85\uc758\ub85c \ub2e8\ub3c5 \uc791\uc0ac, \uc2f1\uae00\uc758 A\uc0ac\uc774\ub4dc \uace1\uc73c\ub85c \uc218\ub85d\ub418\uc5c8\uace0, \ub2e4\uc74c\ub2ec\uc5d0 \ucd9c\uc2dc\ud55c 21\ubc88\uc9f8 \uc815\uaddc \uc74c\ubc18 \u300aI hope so\u300b\uc5d0\uc11c\ub294 \u3008Days\u3009\ub97c \ud3ec\ud568\ud558\uc5ec \ubcf8\uba85 \uba85\uc758\ub85c \uc791\uc0ac\ud55c \uace1 4\uac1c\uac00 \uc218\ub85d\ub418\uc5b4 \uc788\ub294 \ub4f1 \uc791\uc0ac\uc758 \uc601\uc5ed \ub610\ud55c \uc810\ucc28 \ub298\ub824\ub098\uac14\ub2e4.", "paragraph_answer": "\ub098\uce74\ubaa8\ub9ac \uc544\ud0a4\ub098\ub294 1984\ub144 \uc790\uc2e0\uc758 5\ubc88\uc9f8 \uc815\uaddc \uc74c\ubc18\uc778 \u300aANNIVERSARY\u300b\uc758 \uc218\ub85d\uace1 \u3008\uafc8\uc744 \uafb8\uac8c \ud574\u3009(\u5922\u3092\u898b\u3055\u305b\u3066)\uc758 \uac00\uc0ac\ub97c \uc9c1\uc811 \uc4f4 \uc774\ud6c4 \uc870\uae08\uc529 \uc791\uc0ac\uc5d0\ub3c4 \uc190\uc744 \ub300\uae30 \uc2dc\uc791\ud588\ub2e4. \uc2f1\uae00\ub85c\ub294 1991\ub144 \uc790\uc2e0\uc758 26\ubc88\uc9f8 \uc2f1\uae00\uc774\uc5c8\ub358 \u3008\ud6c4\ud0c0\ub9ac\uc2dc\uc988\uce74 -\u300c\ucc9c\ud558\uc804\uc124\uc0b4\uc778\uc0ac\uac74\u300d\uc73c\ub85c\ubd80\ud130\u3009\uc758 B\uc0ac\uc774\ub4dc \uace1 \u3008\uc78a\uc5b4\u2026\u3009(\u5fd8\u308c\u3066\u2026) \uc758 \uc791\uc0ac\ub97c \uc2dc\uc791\uc73c\ub85c 1995\ub144 32\ubc88\uc9f8 \uc2f1\uae00 \u3008Tokyo Rose\u3009\uc5d0\uc11c\ub3c4 \uacf5\ub3d9 \uc791\uc0ac\uac00\ub85c \ucc38\uc5ec, 2002\ub144 \uc790\uc2e0\uc758 20\ubc88\uc9f8 \uc74c\ubc18 \u300aResonancia\u300b\uc758 \uc218\ub85d\uace1 \u3008Deseo\u3009\ubd80\ud130\ub294 \uc790\uc2e0\uc758 \ubcf8\uba85\uc744 \ubc1d\ud788\ub294 \ub300\uc2e0 Miran:Miran\uc774\ub77c\ub294 \ud544\uba85\uc744 \uc4f0\uae30\ub3c4 \ud588\ub2e4. 2003\ub144\uc5d0 4\uc6d4\uc5d0 \ubc1c\ub9e4\ud55c 42\ubc88\uc9f8 \uc2f1\uae00 \u3008Days\u3009\ub294 \ubcf8\uba85 \uba85\uc758\ub85c \ub2e8\ub3c5 \uc791\uc0ac, \uc2f1\uae00\uc758 A\uc0ac\uc774\ub4dc \uace1\uc73c\ub85c \uc218\ub85d\ub418\uc5c8\uace0, \ub2e4\uc74c\ub2ec\uc5d0 \ucd9c\uc2dc\ud55c 21\ubc88\uc9f8 \uc815\uaddc \uc74c\ubc18 \u300aI hope so\u300b\uc5d0\uc11c\ub294 \u3008Days\u3009\ub97c \ud3ec\ud568\ud558\uc5ec \ubcf8\uba85 \uba85\uc758\ub85c \uc791\uc0ac\ud55c \uace1 4\uac1c\uac00 \uc218\ub85d\ub418\uc5b4 \uc788\ub294 \ub4f1 \uc791\uc0ac\uc758 \uc601\uc5ed \ub610\ud55c \uc810\ucc28 \ub298\ub824\ub098\uac14\ub2e4.", "sentence_answer": "\uc2f1\uae00\ub85c\ub294 1991\ub144 \uc790\uc2e0\uc758 26\ubc88\uc9f8 \uc2f1\uae00\uc774\uc5c8\ub358 \u3008\ud6c4\ud0c0\ub9ac\uc2dc\uc988\uce74 -\u300c\ucc9c\ud558\uc804\uc124\uc0b4\uc778\uc0ac\uac74\u300d\uc73c\ub85c\ubd80\ud130\u3009\uc758 B\uc0ac\uc774\ub4dc \uace1 \u3008\uc78a\uc5b4\u2026\u3009(\u5fd8\u308c\u3066\u2026) \uc758 \uc791\uc0ac\ub97c \uc2dc\uc791\uc73c\ub85c 1995\ub144 32\ubc88\uc9f8 \uc2f1\uae00 \u3008Tokyo Rose\u3009\uc5d0\uc11c\ub3c4 \uacf5\ub3d9 \uc791\uc0ac\uac00\ub85c \ucc38\uc5ec, 2002\ub144 \uc790\uc2e0\uc758 20\ubc88\uc9f8 \uc74c\ubc18 \u300aResonancia\u300b\uc758 \uc218\ub85d\uace1 \u3008Deseo\u3009\ubd80\ud130\ub294 \uc790\uc2e0\uc758 \ubcf8\uba85\uc744 \ubc1d\ud788\ub294 \ub300\uc2e0 Miran:Miran\uc774\ub77c\ub294 \ud544\uba85\uc744 \uc4f0\uae30\ub3c4 \ud588\ub2e4.", "paragraph_id": "6511890-30-1", "paragraph_question": "question: \ub098\uce74\ubaa8\ub9ac \uc544\ud0a4\ub098\uc758 26\ubc88\uc9f8 \uc2f1\uae00 \ubc1c\ub9e4 \ub144\ub3c4\ub294?, context: \ub098\uce74\ubaa8\ub9ac \uc544\ud0a4\ub098\ub294 1984\ub144 \uc790\uc2e0\uc758 5\ubc88\uc9f8 \uc815\uaddc \uc74c\ubc18\uc778 \u300aANNIVERSARY\u300b\uc758 \uc218\ub85d\uace1 \u3008\uafc8\uc744 \uafb8\uac8c \ud574\u3009(\u5922\u3092\u898b\u3055\u305b\u3066)\uc758 \uac00\uc0ac\ub97c \uc9c1\uc811 \uc4f4 \uc774\ud6c4 \uc870\uae08\uc529 \uc791\uc0ac\uc5d0\ub3c4 \uc190\uc744 \ub300\uae30 \uc2dc\uc791\ud588\ub2e4. \uc2f1\uae00\ub85c\ub294 1991\ub144 \uc790\uc2e0\uc758 26\ubc88\uc9f8 \uc2f1\uae00\uc774\uc5c8\ub358 \u3008\ud6c4\ud0c0\ub9ac\uc2dc\uc988\uce74 -\u300c\ucc9c\ud558\uc804\uc124\uc0b4\uc778\uc0ac\uac74\u300d\uc73c\ub85c\ubd80\ud130\u3009\uc758 B\uc0ac\uc774\ub4dc \uace1 \u3008\uc78a\uc5b4\u2026\u3009(\u5fd8\u308c\u3066\u2026) \uc758 \uc791\uc0ac\ub97c \uc2dc\uc791\uc73c\ub85c 1995\ub144 32\ubc88\uc9f8 \uc2f1\uae00 \u3008Tokyo Rose\u3009\uc5d0\uc11c\ub3c4 \uacf5\ub3d9 \uc791\uc0ac\uac00\ub85c \ucc38\uc5ec, 2002\ub144 \uc790\uc2e0\uc758 20\ubc88\uc9f8 \uc74c\ubc18 \u300aResonancia\u300b\uc758 \uc218\ub85d\uace1 \u3008Deseo\u3009\ubd80\ud130\ub294 \uc790\uc2e0\uc758 \ubcf8\uba85\uc744 \ubc1d\ud788\ub294 \ub300\uc2e0 Miran:Miran\uc774\ub77c\ub294 \ud544\uba85\uc744 \uc4f0\uae30\ub3c4 \ud588\ub2e4. 2003\ub144\uc5d0 4\uc6d4\uc5d0 \ubc1c\ub9e4\ud55c 42\ubc88\uc9f8 \uc2f1\uae00 \u3008Days\u3009\ub294 \ubcf8\uba85 \uba85\uc758\ub85c \ub2e8\ub3c5 \uc791\uc0ac, \uc2f1\uae00\uc758 A\uc0ac\uc774\ub4dc \uace1\uc73c\ub85c \uc218\ub85d\ub418\uc5c8\uace0, \ub2e4\uc74c\ub2ec\uc5d0 \ucd9c\uc2dc\ud55c 21\ubc88\uc9f8 \uc815\uaddc \uc74c\ubc18 \u300aI hope so\u300b\uc5d0\uc11c\ub294 \u3008Days\u3009\ub97c \ud3ec\ud568\ud558\uc5ec \ubcf8\uba85 \uba85\uc758\ub85c \uc791\uc0ac\ud55c \uace1 4\uac1c\uac00 \uc218\ub85d\ub418\uc5b4 \uc788\ub294 \ub4f1 \uc791\uc0ac\uc758 \uc601\uc5ed \ub610\ud55c \uc810\ucc28 \ub298\ub824\ub098\uac14\ub2e4."} {"question": "\ub9cc\uc288\ud0c0\uc778\uc758 \uace0\ud5a5\uc740 \uc5b4\ub514\uc778\uac00?", "paragraph": "\ud788\ud2c0\ub7ec\uac00 \uc790\uc2e0\uc5d0\uac8c \ub2e4\ub978 \ubcf4\uc9c1\uc744 \ub9e1\uae30\uc9c0 \uc54a\uc744 \uac83\uc784\uc774 \uba85\ubc31\ud574\uc9c0\uc790 \ub9cc\uc288\ud0c0\uc778\uc740 1944\ub144 10\uc6d4 \ud3ec\uba54\ub77c\ub2c8\uc544\uc5d0 \ubd80\ub3d9\uc0b0\uc744 \ud558\ub098 \uc0c0\ub2e4. \uadf8\ub7ec\ub098 \uc5bc\ub9c8 \ubabb \uc788\uc5b4 \uc18c\ub828\uad70\uc774 \uadf8 \uc9c0\uc5ed\uc744 \uc810\ub839\ud574\uc11c \uc783\uace0 \ub9d0\uc558\ub2e4. \ub9cc\uc288\ud0c0\uc778\uc758 \uace0\ud5a5 \ub9ac\uc5d0\uadf8\ub2c8\uce20\ub294 1945\ub144 1\uc6d4 22\uc77c \uc18c\uac1c\ub839\uc774 \ub0b4\ub824\uc84c\uace0 \ub9cc\uc288\ud0c0\uc778 \uac00\uc871\uc740 \ub09c\ubbfc\uc2e0\uc138\uac00 \ub418\uc5b4 \uce5c\uad6c\ub4e4\uacfc \ud568\uaed8 \uc7a0\uc2dc \ubca0\ub97c\ub9b0\uc5d0 \uc228\uc5b4 \uc788\uc5c8\ub2e4. \ub9cc\uc288\ud0c0\uc778\uc740 \ubca0\ub97c\ub9b0\uc5d0\uc11c \ucd1d\ud1b5\uc5c4\ud3d0\ud638 \uc548\uc5d0 \ub4e4\uc5b4\uac00 \uc788\ub294 \ud788\ud2c0\ub7ec\uc640 \ub300\ud654\ud558\ub824\uace0 \uc2dc\ub3c4\ud588\uc73c\ub098 \ucad3\uaca8\ub0ac\ub2e4. \ub9cc\uc288\ud0c0\uc778\uacfc \uadf8 \uac00\uc871\uc740 1945\ub144 5\uc6d4 \ub3c5\uc77c\uc758 \ud328\ubc30\ub85c \uc804\uc7c1\uc774 \ub05d\ub0a0 \ub54c\uae4c\uc9c0 \uacc4\uc18d \uc11c\ucabd\uc73c\ub85c \ub3c4\ub9dd\uac14\ub2e4. \ub9cc\uc288\ud0c0\uc778\uc740 \uc624\ub978\ucabd \ub208\uc5d0 \ub2e4\uc2dc \uace0\ud1b5\uc744 \ud638\uc18c\ud588\uace0 \ud558\uc77c\ub9ac\uac90\ud558\ud39c\uc758 \ubcd1\uc6d0\uc5d0\uc11c \uce58\ub8cc\ub97c \ubc1b\ub2e4\uac00 \uc601\uad6d\uad70\uc5d0\uac8c \uccb4\ud3ec\ub418\uc5b4 8\uc6d4 26\uc77c \ub258\ub978\ubca0\ub974\ud06c \uadfc\uad50 \ud3ec\ub85c\uc218\uc6a9\uc18c\ub85c \uc774\uc1a1\ub418\uc5c8\ub2e4.", "answer": "\ub9ac\uc5d0\uadf8\ub2c8\uce20", "sentence": "\ub9cc\uc288\ud0c0\uc778\uc758 \uace0\ud5a5 \ub9ac\uc5d0\uadf8\ub2c8\uce20 \ub294 1945\ub144 1\uc6d4 22\uc77c \uc18c\uac1c\ub839\uc774 \ub0b4\ub824\uc84c\uace0 \ub9cc\uc288\ud0c0\uc778 \uac00\uc871\uc740 \ub09c\ubbfc\uc2e0\uc138\uac00 \ub418\uc5b4 \uce5c\uad6c\ub4e4\uacfc \ud568\uaed8 \uc7a0\uc2dc \ubca0\ub97c\ub9b0\uc5d0 \uc228\uc5b4 \uc788\uc5c8\ub2e4.", "paragraph_sentence": "\ud788\ud2c0\ub7ec\uac00 \uc790\uc2e0\uc5d0\uac8c \ub2e4\ub978 \ubcf4\uc9c1\uc744 \ub9e1\uae30\uc9c0 \uc54a\uc744 \uac83\uc784\uc774 \uba85\ubc31\ud574\uc9c0\uc790 \ub9cc\uc288\ud0c0\uc778\uc740 1944\ub144 10\uc6d4 \ud3ec\uba54\ub77c\ub2c8\uc544\uc5d0 \ubd80\ub3d9\uc0b0\uc744 \ud558\ub098 \uc0c0\ub2e4. \uadf8\ub7ec\ub098 \uc5bc\ub9c8 \ubabb \uc788\uc5b4 \uc18c\ub828\uad70\uc774 \uadf8 \uc9c0\uc5ed\uc744 \uc810\ub839\ud574\uc11c \uc783\uace0 \ub9d0\uc558\ub2e4. \ub9cc\uc288\ud0c0\uc778\uc758 \uace0\ud5a5 \ub9ac\uc5d0\uadf8\ub2c8\uce20 \ub294 1945\ub144 1\uc6d4 22\uc77c \uc18c\uac1c\ub839\uc774 \ub0b4\ub824\uc84c\uace0 \ub9cc\uc288\ud0c0\uc778 \uac00\uc871\uc740 \ub09c\ubbfc\uc2e0\uc138\uac00 \ub418\uc5b4 \uce5c\uad6c\ub4e4\uacfc \ud568\uaed8 \uc7a0\uc2dc \ubca0\ub97c\ub9b0\uc5d0 \uc228\uc5b4 \uc788\uc5c8\ub2e4. \ub9cc\uc288\ud0c0\uc778\uc740 \ubca0\ub97c\ub9b0\uc5d0\uc11c \ucd1d\ud1b5\uc5c4\ud3d0\ud638 \uc548\uc5d0 \ub4e4\uc5b4\uac00 \uc788\ub294 \ud788\ud2c0\ub7ec\uc640 \ub300\ud654\ud558\ub824\uace0 \uc2dc\ub3c4\ud588\uc73c\ub098 \ucad3\uaca8\ub0ac\ub2e4. \ub9cc\uc288\ud0c0\uc778\uacfc \uadf8 \uac00\uc871\uc740 1945\ub144 5\uc6d4 \ub3c5\uc77c\uc758 \ud328\ubc30\ub85c \uc804\uc7c1\uc774 \ub05d\ub0a0 \ub54c\uae4c\uc9c0 \uacc4\uc18d \uc11c\ucabd\uc73c\ub85c \ub3c4\ub9dd\uac14\ub2e4. \ub9cc\uc288\ud0c0\uc778\uc740 \uc624\ub978\ucabd \ub208\uc5d0 \ub2e4\uc2dc \uace0\ud1b5\uc744 \ud638\uc18c\ud588\uace0 \ud558\uc77c\ub9ac\uac90\ud558\ud39c\uc758 \ubcd1\uc6d0\uc5d0\uc11c \uce58\ub8cc\ub97c \ubc1b\ub2e4\uac00 \uc601\uad6d\uad70\uc5d0\uac8c \uccb4\ud3ec\ub418\uc5b4 8\uc6d4 26\uc77c \ub258\ub978\ubca0\ub974\ud06c \uadfc\uad50 \ud3ec\ub85c\uc218\uc6a9\uc18c\ub85c \uc774\uc1a1\ub418\uc5c8\ub2e4.", "paragraph_answer": "\ud788\ud2c0\ub7ec\uac00 \uc790\uc2e0\uc5d0\uac8c \ub2e4\ub978 \ubcf4\uc9c1\uc744 \ub9e1\uae30\uc9c0 \uc54a\uc744 \uac83\uc784\uc774 \uba85\ubc31\ud574\uc9c0\uc790 \ub9cc\uc288\ud0c0\uc778\uc740 1944\ub144 10\uc6d4 \ud3ec\uba54\ub77c\ub2c8\uc544\uc5d0 \ubd80\ub3d9\uc0b0\uc744 \ud558\ub098 \uc0c0\ub2e4. \uadf8\ub7ec\ub098 \uc5bc\ub9c8 \ubabb \uc788\uc5b4 \uc18c\ub828\uad70\uc774 \uadf8 \uc9c0\uc5ed\uc744 \uc810\ub839\ud574\uc11c \uc783\uace0 \ub9d0\uc558\ub2e4. \ub9cc\uc288\ud0c0\uc778\uc758 \uace0\ud5a5 \ub9ac\uc5d0\uadf8\ub2c8\uce20 \ub294 1945\ub144 1\uc6d4 22\uc77c \uc18c\uac1c\ub839\uc774 \ub0b4\ub824\uc84c\uace0 \ub9cc\uc288\ud0c0\uc778 \uac00\uc871\uc740 \ub09c\ubbfc\uc2e0\uc138\uac00 \ub418\uc5b4 \uce5c\uad6c\ub4e4\uacfc \ud568\uaed8 \uc7a0\uc2dc \ubca0\ub97c\ub9b0\uc5d0 \uc228\uc5b4 \uc788\uc5c8\ub2e4. \ub9cc\uc288\ud0c0\uc778\uc740 \ubca0\ub97c\ub9b0\uc5d0\uc11c \ucd1d\ud1b5\uc5c4\ud3d0\ud638 \uc548\uc5d0 \ub4e4\uc5b4\uac00 \uc788\ub294 \ud788\ud2c0\ub7ec\uc640 \ub300\ud654\ud558\ub824\uace0 \uc2dc\ub3c4\ud588\uc73c\ub098 \ucad3\uaca8\ub0ac\ub2e4. \ub9cc\uc288\ud0c0\uc778\uacfc \uadf8 \uac00\uc871\uc740 1945\ub144 5\uc6d4 \ub3c5\uc77c\uc758 \ud328\ubc30\ub85c \uc804\uc7c1\uc774 \ub05d\ub0a0 \ub54c\uae4c\uc9c0 \uacc4\uc18d \uc11c\ucabd\uc73c\ub85c \ub3c4\ub9dd\uac14\ub2e4. \ub9cc\uc288\ud0c0\uc778\uc740 \uc624\ub978\ucabd \ub208\uc5d0 \ub2e4\uc2dc \uace0\ud1b5\uc744 \ud638\uc18c\ud588\uace0 \ud558\uc77c\ub9ac\uac90\ud558\ud39c\uc758 \ubcd1\uc6d0\uc5d0\uc11c \uce58\ub8cc\ub97c \ubc1b\ub2e4\uac00 \uc601\uad6d\uad70\uc5d0\uac8c \uccb4\ud3ec\ub418\uc5b4 8\uc6d4 26\uc77c \ub258\ub978\ubca0\ub974\ud06c \uadfc\uad50 \ud3ec\ub85c\uc218\uc6a9\uc18c\ub85c \uc774\uc1a1\ub418\uc5c8\ub2e4.", "sentence_answer": "\ub9cc\uc288\ud0c0\uc778\uc758 \uace0\ud5a5 \ub9ac\uc5d0\uadf8\ub2c8\uce20 \ub294 1945\ub144 1\uc6d4 22\uc77c \uc18c\uac1c\ub839\uc774 \ub0b4\ub824\uc84c\uace0 \ub9cc\uc288\ud0c0\uc778 \uac00\uc871\uc740 \ub09c\ubbfc\uc2e0\uc138\uac00 \ub418\uc5b4 \uce5c\uad6c\ub4e4\uacfc \ud568\uaed8 \uc7a0\uc2dc \ubca0\ub97c\ub9b0\uc5d0 \uc228\uc5b4 \uc788\uc5c8\ub2e4.", "paragraph_id": "6540644-31-1", "paragraph_question": "question: \ub9cc\uc288\ud0c0\uc778\uc758 \uace0\ud5a5\uc740 \uc5b4\ub514\uc778\uac00?, context: \ud788\ud2c0\ub7ec\uac00 \uc790\uc2e0\uc5d0\uac8c \ub2e4\ub978 \ubcf4\uc9c1\uc744 \ub9e1\uae30\uc9c0 \uc54a\uc744 \uac83\uc784\uc774 \uba85\ubc31\ud574\uc9c0\uc790 \ub9cc\uc288\ud0c0\uc778\uc740 1944\ub144 10\uc6d4 \ud3ec\uba54\ub77c\ub2c8\uc544\uc5d0 \ubd80\ub3d9\uc0b0\uc744 \ud558\ub098 \uc0c0\ub2e4. \uadf8\ub7ec\ub098 \uc5bc\ub9c8 \ubabb \uc788\uc5b4 \uc18c\ub828\uad70\uc774 \uadf8 \uc9c0\uc5ed\uc744 \uc810\ub839\ud574\uc11c \uc783\uace0 \ub9d0\uc558\ub2e4. \ub9cc\uc288\ud0c0\uc778\uc758 \uace0\ud5a5 \ub9ac\uc5d0\uadf8\ub2c8\uce20\ub294 1945\ub144 1\uc6d4 22\uc77c \uc18c\uac1c\ub839\uc774 \ub0b4\ub824\uc84c\uace0 \ub9cc\uc288\ud0c0\uc778 \uac00\uc871\uc740 \ub09c\ubbfc\uc2e0\uc138\uac00 \ub418\uc5b4 \uce5c\uad6c\ub4e4\uacfc \ud568\uaed8 \uc7a0\uc2dc \ubca0\ub97c\ub9b0\uc5d0 \uc228\uc5b4 \uc788\uc5c8\ub2e4. \ub9cc\uc288\ud0c0\uc778\uc740 \ubca0\ub97c\ub9b0\uc5d0\uc11c \ucd1d\ud1b5\uc5c4\ud3d0\ud638 \uc548\uc5d0 \ub4e4\uc5b4\uac00 \uc788\ub294 \ud788\ud2c0\ub7ec\uc640 \ub300\ud654\ud558\ub824\uace0 \uc2dc\ub3c4\ud588\uc73c\ub098 \ucad3\uaca8\ub0ac\ub2e4. \ub9cc\uc288\ud0c0\uc778\uacfc \uadf8 \uac00\uc871\uc740 1945\ub144 5\uc6d4 \ub3c5\uc77c\uc758 \ud328\ubc30\ub85c \uc804\uc7c1\uc774 \ub05d\ub0a0 \ub54c\uae4c\uc9c0 \uacc4\uc18d \uc11c\ucabd\uc73c\ub85c \ub3c4\ub9dd\uac14\ub2e4. \ub9cc\uc288\ud0c0\uc778\uc740 \uc624\ub978\ucabd \ub208\uc5d0 \ub2e4\uc2dc \uace0\ud1b5\uc744 \ud638\uc18c\ud588\uace0 \ud558\uc77c\ub9ac\uac90\ud558\ud39c\uc758 \ubcd1\uc6d0\uc5d0\uc11c \uce58\ub8cc\ub97c \ubc1b\ub2e4\uac00 \uc601\uad6d\uad70\uc5d0\uac8c \uccb4\ud3ec\ub418\uc5b4 8\uc6d4 26\uc77c \ub258\ub978\ubca0\ub974\ud06c \uadfc\uad50 \ud3ec\ub85c\uc218\uc6a9\uc18c\ub85c \uc774\uc1a1\ub418\uc5c8\ub2e4."} {"question": "\ub9c8\ub9ac\ub0d0\ub178\uc804\ud22c\uc758 \uc9c0\ud718\uad00\ub4e4\uc740 \uc2a4\uc704\uc2a4\uad70\uc758 \ubc29\uce68\uc744 \ubb34\uc5c7\uc73c\ub85c \uacb0\uc815\ud588\ub098?", "paragraph": "\uc2a4\uc704\uc2a4 \ud328\ubc30\uc758 \uc9c1\uc811\uc801\uc778 \uc6d0\uc778\uc740 \uad50\uc804 \ucd08\uae30\uc5d0 \uc801\ub4e4\uc758 \ub300\ud3ec\ub97c \ucc28\uc9c0\ud558\ub294\ub370 \uc2e4\ud328\ud55c \uac83\uacfc \ubca0\ub124\uce58\uc544 \uc9c0\uc6d0\uad70\uc774\uc600\ub2e4. \uadf8\ub7ec\ub098 \uc2a4\uc704\uc2a4\uc758 \ud328\ubc30\uc5d0 \uae30\uc5ec\ud55c \uac83\uc5d0\ub294 \uaddc\uc728 \ubd80\uc871\uacfc \ud6a8\uacfc\uc801\uc778\uc774\uc9c0 \ubabb\ud55c \uc9c0\ud718 \uccb4\uacc4\ub3c4 \uc788\uc5c8\ub2e4. \uac19\uc740 \ud574\uc778 1515\ub144 \ucd08\uc5d0 \uc2a4\uc704\uc2a4 \uc758\ud68c\ub294 \uac01\uac01\uc758 \uc9c0\ud718\uad00\ub4e4\uc5d0\uac8c \uadf8\ub4e4\uc758 \ubd80\ub300\uc5d0 \ub300\ud55c \ud1f4\uac01\uacfc \uad50\uc804\uc758 \uc9c0\uc18d\uc744 \uacb0\uc815\ud560 \uc218 \uc788\ub294 \uad8c\ud55c\uc744 \ubd80\uc5ec\ud588\ub2e4. \uc774 \uc9c0\uce68\uc774 \uc2a4\uc704\uc2a4\uad70\uc774 \ud558\ub098\ub85c \ud1b5\uc77c\ub41c \ud589\ub3d9\uc744 \ubd88\uac00\ub2a5\ud558\uac8c \ub9cc\ub4e4\uc5c8\ub2e4. \uc9c0\ud718 \uccb4\uacc4\ub294 \ucd1d\uc0ac\ub839\uad00\uc73c\ub85c \uc120\ucd9c\ub41c \uc9c0\ud718\uad00 \ub450 \uba85, \ucd94\uac00\uc801\uc73c\ub85c \uac01\uac01\uc758 \ubd80\ub300\uc758 \uc9c0\ud718\uad00\ub4e4\uacfc \ub354\ubd88\uc5b4 \uac01 \uce78\ud1a4\uc758 \uad70\uc778\ub4e4\uc774 \uc120\ucd9c\ud55c \ub300\ud45c\uc790 1\uc778\ub4e4\ub85c \uad6c\uc131\ub418\uc5b4 \uc788\uc5c8\ub2e4. \uc774 \uc9c0\ud718\uad00\ub4e4\uc740 \uc2a4\uc704\uc2a4\uad70\uc758 \ubc29\uce68\uc744 \ud22c\ud45c\ub85c \uacb0\uc815\ud588\ub2e4. \uc2e4\uc804\uc5d0\uc11c \uc9c0\ud718 \uccb4\uacc4\ub294 \ud558\ub824\uace0 \ud558\uba74 \uae08\uc138 \uc791\ub3d9\ud558\uc9c0 \uc54a\ub2e4: \uc9c0\ud718\uad00\ub4e4\uc740 \ud611\uc815\uc744 \uc874\uc911\ud558\uc5ec \uacf5\uaca9\ud558\ub824\ud558\uc9c0 \uc54a\uc740 \ubc18\uba74\uc5d0 \ubd80\ub300\uc758 \ub300\ud45c\uc790\ub4e4, \ud2b9\ud788 \uc6b0\ub9ac, \uc288\ube44\uce20, \uae00\ub77c\ub8e8\uc2a4 \uc9c0\uc5ed, \uc774\ub4e4\uc740 \ucda9\ubd84\ud55c \uc57d\ud0c8\ud488\uc5d0 \ub300\ud55c \uae30\ub300\uc5d0 \ub3d9\uae30\ub97c \ubd80\uc5ec\ubc1b\uc73c\uba70 \uac70\ub9ac\ub08c\uc5c6\uc774 \uacf5\uaca9\ud558\uc790\uace0 \uacb0\uc815\ud588\ub2e4.", "answer": "\ud22c\ud45c", "sentence": "\uc774 \uc9c0\ud718\uad00\ub4e4\uc740 \uc2a4\uc704\uc2a4\uad70\uc758 \ubc29\uce68\uc744 \ud22c\ud45c \ub85c \uacb0\uc815\ud588\ub2e4.", "paragraph_sentence": "\uc2a4\uc704\uc2a4 \ud328\ubc30\uc758 \uc9c1\uc811\uc801\uc778 \uc6d0\uc778\uc740 \uad50\uc804 \ucd08\uae30\uc5d0 \uc801\ub4e4\uc758 \ub300\ud3ec\ub97c \ucc28\uc9c0\ud558\ub294\ub370 \uc2e4\ud328\ud55c \uac83\uacfc \ubca0\ub124\uce58\uc544 \uc9c0\uc6d0\uad70\uc774\uc600\ub2e4. \uadf8\ub7ec\ub098 \uc2a4\uc704\uc2a4\uc758 \ud328\ubc30\uc5d0 \uae30\uc5ec\ud55c \uac83\uc5d0\ub294 \uaddc\uc728 \ubd80\uc871\uacfc \ud6a8\uacfc\uc801\uc778\uc774\uc9c0 \ubabb\ud55c \uc9c0\ud718 \uccb4\uacc4\ub3c4 \uc788\uc5c8\ub2e4. \uac19\uc740 \ud574\uc778 1515\ub144 \ucd08\uc5d0 \uc2a4\uc704\uc2a4 \uc758\ud68c\ub294 \uac01\uac01\uc758 \uc9c0\ud718\uad00\ub4e4\uc5d0\uac8c \uadf8\ub4e4\uc758 \ubd80\ub300\uc5d0 \ub300\ud55c \ud1f4\uac01\uacfc \uad50\uc804\uc758 \uc9c0\uc18d\uc744 \uacb0\uc815\ud560 \uc218 \uc788\ub294 \uad8c\ud55c\uc744 \ubd80\uc5ec\ud588\ub2e4. \uc774 \uc9c0\uce68\uc774 \uc2a4\uc704\uc2a4\uad70\uc774 \ud558\ub098\ub85c \ud1b5\uc77c\ub41c \ud589\ub3d9\uc744 \ubd88\uac00\ub2a5\ud558\uac8c \ub9cc\ub4e4\uc5c8\ub2e4. \uc9c0\ud718 \uccb4\uacc4\ub294 \ucd1d\uc0ac\ub839\uad00\uc73c\ub85c \uc120\ucd9c\ub41c \uc9c0\ud718\uad00 \ub450 \uba85, \ucd94\uac00\uc801\uc73c\ub85c \uac01\uac01\uc758 \ubd80\ub300\uc758 \uc9c0\ud718\uad00\ub4e4\uacfc \ub354\ubd88\uc5b4 \uac01 \uce78\ud1a4\uc758 \uad70\uc778\ub4e4\uc774 \uc120\ucd9c\ud55c \ub300\ud45c\uc790 1\uc778\ub4e4\ub85c \uad6c\uc131\ub418\uc5b4 \uc788\uc5c8\ub2e4. \uc774 \uc9c0\ud718\uad00\ub4e4\uc740 \uc2a4\uc704\uc2a4\uad70\uc758 \ubc29\uce68\uc744 \ud22c\ud45c \ub85c \uacb0\uc815\ud588\ub2e4. \uc2e4\uc804\uc5d0\uc11c \uc9c0\ud718 \uccb4\uacc4\ub294 \ud558\ub824\uace0 \ud558\uba74 \uae08\uc138 \uc791\ub3d9\ud558\uc9c0 \uc54a\ub2e4: \uc9c0\ud718\uad00\ub4e4\uc740 \ud611\uc815\uc744 \uc874\uc911\ud558\uc5ec \uacf5\uaca9\ud558\ub824\ud558\uc9c0 \uc54a\uc740 \ubc18\uba74\uc5d0 \ubd80\ub300\uc758 \ub300\ud45c\uc790\ub4e4, \ud2b9\ud788 \uc6b0\ub9ac, \uc288\ube44\uce20, \uae00\ub77c\ub8e8\uc2a4 \uc9c0\uc5ed, \uc774\ub4e4\uc740 \ucda9\ubd84\ud55c \uc57d\ud0c8\ud488\uc5d0 \ub300\ud55c \uae30\ub300\uc5d0 \ub3d9\uae30\ub97c \ubd80\uc5ec\ubc1b\uc73c\uba70 \uac70\ub9ac\ub08c\uc5c6\uc774 \uacf5\uaca9\ud558\uc790\uace0 \uacb0\uc815\ud588\ub2e4.", "paragraph_answer": "\uc2a4\uc704\uc2a4 \ud328\ubc30\uc758 \uc9c1\uc811\uc801\uc778 \uc6d0\uc778\uc740 \uad50\uc804 \ucd08\uae30\uc5d0 \uc801\ub4e4\uc758 \ub300\ud3ec\ub97c \ucc28\uc9c0\ud558\ub294\ub370 \uc2e4\ud328\ud55c \uac83\uacfc \ubca0\ub124\uce58\uc544 \uc9c0\uc6d0\uad70\uc774\uc600\ub2e4. \uadf8\ub7ec\ub098 \uc2a4\uc704\uc2a4\uc758 \ud328\ubc30\uc5d0 \uae30\uc5ec\ud55c \uac83\uc5d0\ub294 \uaddc\uc728 \ubd80\uc871\uacfc \ud6a8\uacfc\uc801\uc778\uc774\uc9c0 \ubabb\ud55c \uc9c0\ud718 \uccb4\uacc4\ub3c4 \uc788\uc5c8\ub2e4. \uac19\uc740 \ud574\uc778 1515\ub144 \ucd08\uc5d0 \uc2a4\uc704\uc2a4 \uc758\ud68c\ub294 \uac01\uac01\uc758 \uc9c0\ud718\uad00\ub4e4\uc5d0\uac8c \uadf8\ub4e4\uc758 \ubd80\ub300\uc5d0 \ub300\ud55c \ud1f4\uac01\uacfc \uad50\uc804\uc758 \uc9c0\uc18d\uc744 \uacb0\uc815\ud560 \uc218 \uc788\ub294 \uad8c\ud55c\uc744 \ubd80\uc5ec\ud588\ub2e4. \uc774 \uc9c0\uce68\uc774 \uc2a4\uc704\uc2a4\uad70\uc774 \ud558\ub098\ub85c \ud1b5\uc77c\ub41c \ud589\ub3d9\uc744 \ubd88\uac00\ub2a5\ud558\uac8c \ub9cc\ub4e4\uc5c8\ub2e4. \uc9c0\ud718 \uccb4\uacc4\ub294 \ucd1d\uc0ac\ub839\uad00\uc73c\ub85c \uc120\ucd9c\ub41c \uc9c0\ud718\uad00 \ub450 \uba85, \ucd94\uac00\uc801\uc73c\ub85c \uac01\uac01\uc758 \ubd80\ub300\uc758 \uc9c0\ud718\uad00\ub4e4\uacfc \ub354\ubd88\uc5b4 \uac01 \uce78\ud1a4\uc758 \uad70\uc778\ub4e4\uc774 \uc120\ucd9c\ud55c \ub300\ud45c\uc790 1\uc778\ub4e4\ub85c \uad6c\uc131\ub418\uc5b4 \uc788\uc5c8\ub2e4. \uc774 \uc9c0\ud718\uad00\ub4e4\uc740 \uc2a4\uc704\uc2a4\uad70\uc758 \ubc29\uce68\uc744 \ud22c\ud45c \ub85c \uacb0\uc815\ud588\ub2e4. \uc2e4\uc804\uc5d0\uc11c \uc9c0\ud718 \uccb4\uacc4\ub294 \ud558\ub824\uace0 \ud558\uba74 \uae08\uc138 \uc791\ub3d9\ud558\uc9c0 \uc54a\ub2e4: \uc9c0\ud718\uad00\ub4e4\uc740 \ud611\uc815\uc744 \uc874\uc911\ud558\uc5ec \uacf5\uaca9\ud558\ub824\ud558\uc9c0 \uc54a\uc740 \ubc18\uba74\uc5d0 \ubd80\ub300\uc758 \ub300\ud45c\uc790\ub4e4, \ud2b9\ud788 \uc6b0\ub9ac, \uc288\ube44\uce20, \uae00\ub77c\ub8e8\uc2a4 \uc9c0\uc5ed, \uc774\ub4e4\uc740 \ucda9\ubd84\ud55c \uc57d\ud0c8\ud488\uc5d0 \ub300\ud55c \uae30\ub300\uc5d0 \ub3d9\uae30\ub97c \ubd80\uc5ec\ubc1b\uc73c\uba70 \uac70\ub9ac\ub08c\uc5c6\uc774 \uacf5\uaca9\ud558\uc790\uace0 \uacb0\uc815\ud588\ub2e4.", "sentence_answer": "\uc774 \uc9c0\ud718\uad00\ub4e4\uc740 \uc2a4\uc704\uc2a4\uad70\uc758 \ubc29\uce68\uc744 \ud22c\ud45c \ub85c \uacb0\uc815\ud588\ub2e4.", "paragraph_id": "6514001-3-1", "paragraph_question": "question: \ub9c8\ub9ac\ub0d0\ub178\uc804\ud22c\uc758 \uc9c0\ud718\uad00\ub4e4\uc740 \uc2a4\uc704\uc2a4\uad70\uc758 \ubc29\uce68\uc744 \ubb34\uc5c7\uc73c\ub85c \uacb0\uc815\ud588\ub098?, context: \uc2a4\uc704\uc2a4 \ud328\ubc30\uc758 \uc9c1\uc811\uc801\uc778 \uc6d0\uc778\uc740 \uad50\uc804 \ucd08\uae30\uc5d0 \uc801\ub4e4\uc758 \ub300\ud3ec\ub97c \ucc28\uc9c0\ud558\ub294\ub370 \uc2e4\ud328\ud55c \uac83\uacfc \ubca0\ub124\uce58\uc544 \uc9c0\uc6d0\uad70\uc774\uc600\ub2e4. \uadf8\ub7ec\ub098 \uc2a4\uc704\uc2a4\uc758 \ud328\ubc30\uc5d0 \uae30\uc5ec\ud55c \uac83\uc5d0\ub294 \uaddc\uc728 \ubd80\uc871\uacfc \ud6a8\uacfc\uc801\uc778\uc774\uc9c0 \ubabb\ud55c \uc9c0\ud718 \uccb4\uacc4\ub3c4 \uc788\uc5c8\ub2e4. \uac19\uc740 \ud574\uc778 1515\ub144 \ucd08\uc5d0 \uc2a4\uc704\uc2a4 \uc758\ud68c\ub294 \uac01\uac01\uc758 \uc9c0\ud718\uad00\ub4e4\uc5d0\uac8c \uadf8\ub4e4\uc758 \ubd80\ub300\uc5d0 \ub300\ud55c \ud1f4\uac01\uacfc \uad50\uc804\uc758 \uc9c0\uc18d\uc744 \uacb0\uc815\ud560 \uc218 \uc788\ub294 \uad8c\ud55c\uc744 \ubd80\uc5ec\ud588\ub2e4. \uc774 \uc9c0\uce68\uc774 \uc2a4\uc704\uc2a4\uad70\uc774 \ud558\ub098\ub85c \ud1b5\uc77c\ub41c \ud589\ub3d9\uc744 \ubd88\uac00\ub2a5\ud558\uac8c \ub9cc\ub4e4\uc5c8\ub2e4. \uc9c0\ud718 \uccb4\uacc4\ub294 \ucd1d\uc0ac\ub839\uad00\uc73c\ub85c \uc120\ucd9c\ub41c \uc9c0\ud718\uad00 \ub450 \uba85, \ucd94\uac00\uc801\uc73c\ub85c \uac01\uac01\uc758 \ubd80\ub300\uc758 \uc9c0\ud718\uad00\ub4e4\uacfc \ub354\ubd88\uc5b4 \uac01 \uce78\ud1a4\uc758 \uad70\uc778\ub4e4\uc774 \uc120\ucd9c\ud55c \ub300\ud45c\uc790 1\uc778\ub4e4\ub85c \uad6c\uc131\ub418\uc5b4 \uc788\uc5c8\ub2e4. \uc774 \uc9c0\ud718\uad00\ub4e4\uc740 \uc2a4\uc704\uc2a4\uad70\uc758 \ubc29\uce68\uc744 \ud22c\ud45c\ub85c \uacb0\uc815\ud588\ub2e4. \uc2e4\uc804\uc5d0\uc11c \uc9c0\ud718 \uccb4\uacc4\ub294 \ud558\ub824\uace0 \ud558\uba74 \uae08\uc138 \uc791\ub3d9\ud558\uc9c0 \uc54a\ub2e4: \uc9c0\ud718\uad00\ub4e4\uc740 \ud611\uc815\uc744 \uc874\uc911\ud558\uc5ec \uacf5\uaca9\ud558\ub824\ud558\uc9c0 \uc54a\uc740 \ubc18\uba74\uc5d0 \ubd80\ub300\uc758 \ub300\ud45c\uc790\ub4e4, \ud2b9\ud788 \uc6b0\ub9ac, \uc288\ube44\uce20, \uae00\ub77c\ub8e8\uc2a4 \uc9c0\uc5ed, \uc774\ub4e4\uc740 \ucda9\ubd84\ud55c \uc57d\ud0c8\ud488\uc5d0 \ub300\ud55c \uae30\ub300\uc5d0 \ub3d9\uae30\ub97c \ubd80\uc5ec\ubc1b\uc73c\uba70 \uac70\ub9ac\ub08c\uc5c6\uc774 \uacf5\uaca9\ud558\uc790\uace0 \uacb0\uc815\ud588\ub2e4."} {"question": "7\ud68c \uc544\uc2a4\ud0c0\ub098-\uc54c\ub9c8\ud2f0 \ub3d9\uacc4 \uc544\uc2dc\uc548\uac8c\uc784\uc758 3000m\uc5d0\uc11c \uae40\ubcf4\ub984\uc774 \uc5bb\uc740 \uc131\uc801\uc740?", "paragraph": "\uce74\uc790\ud750\uc2a4\ud0c4 \uc544\uc2a4\ud0c0\ub098\uc2e4\ub0b4\uc2a4\ud53c\ub4dc\uc2a4\ucf00\uc774\ud305\uacbd\uae30\uc7a5\uc11c \ud3bc\uccd0\uc9c4 \uc81c7\ud68c \uc544\uc2a4\ud0c0\ub098-\uc54c\ub9c8\ud2f0 \ub3d9\uacc4\uc544\uc2dc\uc548\uac8c\uc784 3000m\uc5d0\uc11c \uae40\ubcf4\ub984\uc774 4\ubd8410\ucd0854\uc758 \uae30\ub85d\uc73c\ub85c \uc740\uba54\ub2ec\uc744 \ucc28\uc9c0\ud588\ub2e4. \uae40\ubcf4\ub984\uc740 3000m \uacb0\uc2b9\uc804\uc5d0\uc11c 3\uc870\ub85c \ucd9c\ubc1c\ud574\uc11c \uc55e\uc11c \ucd9c\ubc1c\ud55c 1,2\uc870 \uc120\uc218\ub4e4\uc5d0 \ube44\ud574 \uae30\ub85d\ucc28\ub97c \ud06c\uac8c \ubc8c\ub9ac\uba70 1\uc704\uc5d0 \uc130\uc9c0\ub9cc \uace7\ubc14\ub85c 4\uc870 \uc77c\ubcf8\uc758 \ud638\uc870\ubbf8 \ub9c8\uc0ac\ucf54\uc5d0 2.72\ucd08 \ub4a4\uc84c\ub2e4. 3\uc704\uc5d0 \uc624\ub978 \uc911\uad6d\uc758 \uc655\ud398\uc774\ub294 4\ubd8419\ucd0877\uc744 \uae30\ub85d\ud588\ub2e4. \ud568\uaed8 \uacbd\uae30\uc5d0 \ub098\uc120 \ubc15\ub3c4\uc601(17, \ub355\uc815\uace0)\uc740 \ucee8\ub514\uc158 \ub09c\uc870\ub85c \uc544\uc27d\uac8c 5\uc704\uc5d0 \uba38\ubb3c\ub800\ub2e4. 2011\ub144 2\uc6d4 5\uc77c \uc5f4\ub9b0 5000m\uc5d0\uc11c\ub294 \ud55c\uad6d\uc758 \ubc15\ub3c4\uc601(18, \ub355\uc815\uace0)\uc774 3\ubc88\uc9f8 \uc870\ub85c \uc77c\ubcf8\uc758 \ud638\uc988\ubbf8 \ub9c8\uc0ac\ucf54\uc640 \ud568\uaed8 \ub808\uc774\uc2a4\ub97c \ud3bc\uccd0 6\ucd0840 \ub4a4\uc9c4 \uae30\ub85d\uc73c\ub85c \uace8\uc778\ud588\uc73c\ub098, \uc804\uccb4 2\uc704(7\ubd8415\ucd0863)\ub97c \ucc28\uc9c0\ud588\ub2e4. \uae40\ubcf4\ub984\uc740 7\ubd8422\ucd0892\uc758 \uae30\ub85d\uc73c\ub85c 4\uc704\ub97c \ucc28\uc9c0\ud574 \uc544\uc27d\uac8c \uba54\ub2ec\uc744 \ub193\ucce4\ub2e4.", "answer": "\uc740\uba54\ub2ec", "sentence": "\uce74\uc790\ud750\uc2a4\ud0c4 \uc544\uc2a4\ud0c0\ub098\uc2e4\ub0b4\uc2a4\ud53c\ub4dc\uc2a4\ucf00\uc774\ud305\uacbd\uae30\uc7a5\uc11c \ud3bc\uccd0\uc9c4 \uc81c7\ud68c \uc544\uc2a4\ud0c0\ub098-\uc54c\ub9c8\ud2f0 \ub3d9\uacc4\uc544\uc2dc\uc548\uac8c\uc784 3000m\uc5d0\uc11c \uae40\ubcf4\ub984\uc774 4\ubd8410\ucd0854\uc758 \uae30\ub85d\uc73c\ub85c \uc740\uba54\ub2ec \uc744 \ucc28\uc9c0\ud588\ub2e4.", "paragraph_sentence": " \uce74\uc790\ud750\uc2a4\ud0c4 \uc544\uc2a4\ud0c0\ub098\uc2e4\ub0b4\uc2a4\ud53c\ub4dc\uc2a4\ucf00\uc774\ud305\uacbd\uae30\uc7a5\uc11c \ud3bc\uccd0\uc9c4 \uc81c7\ud68c \uc544\uc2a4\ud0c0\ub098-\uc54c\ub9c8\ud2f0 \ub3d9\uacc4\uc544\uc2dc\uc548\uac8c\uc784 3000m\uc5d0\uc11c \uae40\ubcf4\ub984\uc774 4\ubd8410\ucd0854\uc758 \uae30\ub85d\uc73c\ub85c \uc740\uba54\ub2ec \uc744 \ucc28\uc9c0\ud588\ub2e4. \uae40\ubcf4\ub984\uc740 3000m \uacb0\uc2b9\uc804\uc5d0\uc11c 3\uc870\ub85c \ucd9c\ubc1c\ud574\uc11c \uc55e\uc11c \ucd9c\ubc1c\ud55c 1,2\uc870 \uc120\uc218\ub4e4\uc5d0 \ube44\ud574 \uae30\ub85d\ucc28\ub97c \ud06c\uac8c \ubc8c\ub9ac\uba70 1\uc704\uc5d0 \uc130\uc9c0\ub9cc \uace7\ubc14\ub85c 4\uc870 \uc77c\ubcf8\uc758 \ud638\uc870\ubbf8 \ub9c8\uc0ac\ucf54\uc5d0 2.72\ucd08 \ub4a4\uc84c\ub2e4. 3\uc704\uc5d0 \uc624\ub978 \uc911\uad6d\uc758 \uc655\ud398\uc774\ub294 4\ubd8419\ucd0877\uc744 \uae30\ub85d\ud588\ub2e4. \ud568\uaed8 \uacbd\uae30\uc5d0 \ub098\uc120 \ubc15\ub3c4\uc601(17, \ub355\uc815\uace0)\uc740 \ucee8\ub514\uc158 \ub09c\uc870\ub85c \uc544\uc27d\uac8c 5\uc704\uc5d0 \uba38\ubb3c\ub800\ub2e4. 2011\ub144 2\uc6d4 5\uc77c \uc5f4\ub9b0 5000m\uc5d0\uc11c\ub294 \ud55c\uad6d\uc758 \ubc15\ub3c4\uc601(18, \ub355\uc815\uace0)\uc774 3\ubc88\uc9f8 \uc870\ub85c \uc77c\ubcf8\uc758 \ud638\uc988\ubbf8 \ub9c8\uc0ac\ucf54\uc640 \ud568\uaed8 \ub808\uc774\uc2a4\ub97c \ud3bc\uccd0 6\ucd0840 \ub4a4\uc9c4 \uae30\ub85d\uc73c\ub85c \uace8\uc778\ud588\uc73c\ub098, \uc804\uccb4 2\uc704(7\ubd8415\ucd0863)\ub97c \ucc28\uc9c0\ud588\ub2e4. \uae40\ubcf4\ub984\uc740 7\ubd8422\ucd0892\uc758 \uae30\ub85d\uc73c\ub85c 4\uc704\ub97c \ucc28\uc9c0\ud574 \uc544\uc27d\uac8c \uba54\ub2ec\uc744 \ub193\ucce4\ub2e4.", "paragraph_answer": "\uce74\uc790\ud750\uc2a4\ud0c4 \uc544\uc2a4\ud0c0\ub098\uc2e4\ub0b4\uc2a4\ud53c\ub4dc\uc2a4\ucf00\uc774\ud305\uacbd\uae30\uc7a5\uc11c \ud3bc\uccd0\uc9c4 \uc81c7\ud68c \uc544\uc2a4\ud0c0\ub098-\uc54c\ub9c8\ud2f0 \ub3d9\uacc4\uc544\uc2dc\uc548\uac8c\uc784 3000m\uc5d0\uc11c \uae40\ubcf4\ub984\uc774 4\ubd8410\ucd0854\uc758 \uae30\ub85d\uc73c\ub85c \uc740\uba54\ub2ec \uc744 \ucc28\uc9c0\ud588\ub2e4. \uae40\ubcf4\ub984\uc740 3000m \uacb0\uc2b9\uc804\uc5d0\uc11c 3\uc870\ub85c \ucd9c\ubc1c\ud574\uc11c \uc55e\uc11c \ucd9c\ubc1c\ud55c 1,2\uc870 \uc120\uc218\ub4e4\uc5d0 \ube44\ud574 \uae30\ub85d\ucc28\ub97c \ud06c\uac8c \ubc8c\ub9ac\uba70 1\uc704\uc5d0 \uc130\uc9c0\ub9cc \uace7\ubc14\ub85c 4\uc870 \uc77c\ubcf8\uc758 \ud638\uc870\ubbf8 \ub9c8\uc0ac\ucf54\uc5d0 2.72\ucd08 \ub4a4\uc84c\ub2e4. 3\uc704\uc5d0 \uc624\ub978 \uc911\uad6d\uc758 \uc655\ud398\uc774\ub294 4\ubd8419\ucd0877\uc744 \uae30\ub85d\ud588\ub2e4. \ud568\uaed8 \uacbd\uae30\uc5d0 \ub098\uc120 \ubc15\ub3c4\uc601(17, \ub355\uc815\uace0)\uc740 \ucee8\ub514\uc158 \ub09c\uc870\ub85c \uc544\uc27d\uac8c 5\uc704\uc5d0 \uba38\ubb3c\ub800\ub2e4. 2011\ub144 2\uc6d4 5\uc77c \uc5f4\ub9b0 5000m\uc5d0\uc11c\ub294 \ud55c\uad6d\uc758 \ubc15\ub3c4\uc601(18, \ub355\uc815\uace0)\uc774 3\ubc88\uc9f8 \uc870\ub85c \uc77c\ubcf8\uc758 \ud638\uc988\ubbf8 \ub9c8\uc0ac\ucf54\uc640 \ud568\uaed8 \ub808\uc774\uc2a4\ub97c \ud3bc\uccd0 6\ucd0840 \ub4a4\uc9c4 \uae30\ub85d\uc73c\ub85c \uace8\uc778\ud588\uc73c\ub098, \uc804\uccb4 2\uc704(7\ubd8415\ucd0863)\ub97c \ucc28\uc9c0\ud588\ub2e4. \uae40\ubcf4\ub984\uc740 7\ubd8422\ucd0892\uc758 \uae30\ub85d\uc73c\ub85c 4\uc704\ub97c \ucc28\uc9c0\ud574 \uc544\uc27d\uac8c \uba54\ub2ec\uc744 \ub193\ucce4\ub2e4.", "sentence_answer": "\uce74\uc790\ud750\uc2a4\ud0c4 \uc544\uc2a4\ud0c0\ub098\uc2e4\ub0b4\uc2a4\ud53c\ub4dc\uc2a4\ucf00\uc774\ud305\uacbd\uae30\uc7a5\uc11c \ud3bc\uccd0\uc9c4 \uc81c7\ud68c \uc544\uc2a4\ud0c0\ub098-\uc54c\ub9c8\ud2f0 \ub3d9\uacc4\uc544\uc2dc\uc548\uac8c\uc784 3000m\uc5d0\uc11c \uae40\ubcf4\ub984\uc774 4\ubd8410\ucd0854\uc758 \uae30\ub85d\uc73c\ub85c \uc740\uba54\ub2ec \uc744 \ucc28\uc9c0\ud588\ub2e4.", "paragraph_id": "6488275-1-0", "paragraph_question": "question: 7\ud68c \uc544\uc2a4\ud0c0\ub098-\uc54c\ub9c8\ud2f0 \ub3d9\uacc4 \uc544\uc2dc\uc548\uac8c\uc784\uc758 3000m\uc5d0\uc11c \uae40\ubcf4\ub984\uc774 \uc5bb\uc740 \uc131\uc801\uc740?, context: \uce74\uc790\ud750\uc2a4\ud0c4 \uc544\uc2a4\ud0c0\ub098\uc2e4\ub0b4\uc2a4\ud53c\ub4dc\uc2a4\ucf00\uc774\ud305\uacbd\uae30\uc7a5\uc11c \ud3bc\uccd0\uc9c4 \uc81c7\ud68c \uc544\uc2a4\ud0c0\ub098-\uc54c\ub9c8\ud2f0 \ub3d9\uacc4\uc544\uc2dc\uc548\uac8c\uc784 3000m\uc5d0\uc11c \uae40\ubcf4\ub984\uc774 4\ubd8410\ucd0854\uc758 \uae30\ub85d\uc73c\ub85c \uc740\uba54\ub2ec\uc744 \ucc28\uc9c0\ud588\ub2e4. \uae40\ubcf4\ub984\uc740 3000m \uacb0\uc2b9\uc804\uc5d0\uc11c 3\uc870\ub85c \ucd9c\ubc1c\ud574\uc11c \uc55e\uc11c \ucd9c\ubc1c\ud55c 1,2\uc870 \uc120\uc218\ub4e4\uc5d0 \ube44\ud574 \uae30\ub85d\ucc28\ub97c \ud06c\uac8c \ubc8c\ub9ac\uba70 1\uc704\uc5d0 \uc130\uc9c0\ub9cc \uace7\ubc14\ub85c 4\uc870 \uc77c\ubcf8\uc758 \ud638\uc870\ubbf8 \ub9c8\uc0ac\ucf54\uc5d0 2.72\ucd08 \ub4a4\uc84c\ub2e4. 3\uc704\uc5d0 \uc624\ub978 \uc911\uad6d\uc758 \uc655\ud398\uc774\ub294 4\ubd8419\ucd0877\uc744 \uae30\ub85d\ud588\ub2e4. \ud568\uaed8 \uacbd\uae30\uc5d0 \ub098\uc120 \ubc15\ub3c4\uc601(17, \ub355\uc815\uace0)\uc740 \ucee8\ub514\uc158 \ub09c\uc870\ub85c \uc544\uc27d\uac8c 5\uc704\uc5d0 \uba38\ubb3c\ub800\ub2e4. 2011\ub144 2\uc6d4 5\uc77c \uc5f4\ub9b0 5000m\uc5d0\uc11c\ub294 \ud55c\uad6d\uc758 \ubc15\ub3c4\uc601(18, \ub355\uc815\uace0)\uc774 3\ubc88\uc9f8 \uc870\ub85c \uc77c\ubcf8\uc758 \ud638\uc988\ubbf8 \ub9c8\uc0ac\ucf54\uc640 \ud568\uaed8 \ub808\uc774\uc2a4\ub97c \ud3bc\uccd0 6\ucd0840 \ub4a4\uc9c4 \uae30\ub85d\uc73c\ub85c \uace8\uc778\ud588\uc73c\ub098, \uc804\uccb4 2\uc704(7\ubd8415\ucd0863)\ub97c \ucc28\uc9c0\ud588\ub2e4. \uae40\ubcf4\ub984\uc740 7\ubd8422\ucd0892\uc758 \uae30\ub85d\uc73c\ub85c 4\uc704\ub97c \ucc28\uc9c0\ud574 \uc544\uc27d\uac8c \uba54\ub2ec\uc744 \ub193\ucce4\ub2e4."} {"question": "\ub2e4\uc2a4\uc5d0\uc11c 80\uc5b5\uc6d0\uc744 \ud6a1\ub839\ud55c \uc870\ubaa8\uc528\uac00 \ub3c8\uc744 \uc804\ub2ec\ud55c \uc774\ubaa8\uc528\uc758 \uc9c1\uc7a5\uc73c\ub85c \ub2e4\uc2a4 \ud611\ub825\uc5c5\uccb4\uc778 \ud68c\uc0ac\uc758 \uc774\ub984\uc740?", "paragraph": "2018\ub144 2\uc6d4 19\uc77c \uac80\ucc30\uc740 \uc911\uac04 \uc218\uc0ac \uacb0\uacfc\ub97c \ubc1c\ud45c\ud588\ub2e4. \uac80\ucc30 \uc218\uc0ac \uacb0\uacfc \ub2e4\uc2a4 \uacbd\ub9ac\uc9c1\uc6d0 \uc870\ubaa8\uc528\ub294 2002\ub144 6\uc6d4\ubd80\ud130 2007\ub144 10\uc6d4 \ubb34\ub835\uae4c\uc9c0 \ub2e4\uc2a4 \uc678\ud658\uc740\ud589 \ubc95\uc778\uacc4\uc88c\uc5d0\uc11c \ucd9c\uae08 \uc5c5\ubb34\ub97c \ucc98\ub9ac\ud558\uba74\uc11c \ud5c8\uc704\ucd9c\uae08 \uc804\ud45c\ub97c \uc0bd\uc785\ud558\uac70\ub098 \ucd9c\uae08\uc561 \uacfc\ub2e4\uae30\uc7ac \uc218\ubc95\uc73c\ub85c \ub9e4\uc6d4 1\uc5b5\u223c2\uc5b5\uc6d0\uc744 \uc218\ud45c\ub098 \ud604\uae08\uc73c\ub85c \ube7c\ub3cc\ub824 80\uc5b5\uc6d0\uc744 \ud6a1\ub839\ud558\uc600\uace0, \uc774\ub97c \ub2e4\uc2a4 \ud611\ub825\uc5c5\uccb4\uc778 \uc138\uad11\uacf5\uc5c5\uc758 \uacbd\ub9ac \uc774\ubaa8\uc528\uc5d0\uac8c \uc804\ub2ec\ud558\uc600\ub2e4. \uc774\uc528\ub294 \uc2dc\uc911\uc740\ud589 3\uacf3, \ubcf4\ud5d8\ud68c\uc0ac 1\uacf3, \ud22c\uc790\uc2e0\ud0c1\ud68c\uc0ac 1\uacf3 \ub4f1 5\uac1c \uae08\uc735\uae30\uad00\uc5d0 17\uba85 \uba85\uc758\uc758 \ucc28\uba85 \uacc4\uc88c 40\uc5ec\uac1c\ub97c \uac1c\uc124\ud574 5\ub144 \uac04 \ubcf4\uc720\ud558\uba70 120\uc5b5 4300\ub9cc\uc6d0\uc73c\ub85c \ubd88\ub838\ub2e4. \ub2e4\uc2a4 \uacbd\ub9ac \uc870\uc528\ub294 \uc774\uc911 \uc790\uc2e0\uc758 \uc5b4\uba38\ub2c8 \uba85\uc758\uc758 \uacc4\uc88c\uc5d0 \ubcf4\uc720\ud55c \ub3c8\uc744 \uc8fc\ud0dd \uad6c\uc785 \uc790\uae08 \uc6a9\ub3c4\ub85c \uc0ac\uc6a9\ud558\uc600\uc73c\uba70, \uc774\uc640 \ubcc4\ub3c4\ub85c \ud68c\uc0ac \uc790\uae08\uc744 \ubc18\ud658\ud558\uc9c0 \uc54a\uace0 \uc740\ub2c9\ud558\uc600\ub2e4. \ub610\ud55c \uae40\uc131\uc6b0 \uc804 \uc0ac\uc7a5, \uad8c\uc2b9\ud638 \uc804 \uc804\ubb34\uac00 \uacbd\ub9ac \uc870\uc528\uc640 \uacf5\ubaa8\ud558\uc5ec \ub2e4\ub978 \ube44\uc790\uae08\uc744 \uc870\uc131\ud55c \uc0ac\uc2e4, \ud68c\uc0ac \ucc28\uc6d0\uc5d0\uc11c \uc870\uc9c0\uc801\uc73c\ub85c \ube44\uc790\uae08\uc744 \uc870\uc131\ud55c \uc0ac\uc2e4, \ub2e4\uc2a4 \uacbd\uc601\uc9c4\uc774 \ub0a9\ud488 \ub300\uac00\ub97c \uba85\ubaa9\uc73c\ub85c \uae08\ud488\uc744 \uc218\uc218\ud55c \uc0ac\uc2e4\ub3c4 \ud655\uc778\ub418\uc5c8\ub2e4. \ud2b9\uc218\uc9c1\ubb34\uc720\uae30\ub85c \uace0\ubc1c\ub41c \uc815\ud638\uc601 \uc804 \ud2b9\ubcc4\uac80\uc0ac\uc5d0 \ub300\ud574\uc11c\ub294 \ubb34\ud610\uc758 \ucc98\ub9ac\ud558\uc600\ub2e4.", "answer": "\uc138\uad11\uacf5\uc5c5", "sentence": "\uc774\ub97c \ub2e4\uc2a4 \ud611\ub825\uc5c5\uccb4\uc778 \uc138\uad11\uacf5\uc5c5 \uc758 \uacbd\ub9ac \uc774\ubaa8\uc528\uc5d0\uac8c \uc804\ub2ec\ud558\uc600\ub2e4.", "paragraph_sentence": "2018\ub144 2\uc6d4 19\uc77c \uac80\ucc30\uc740 \uc911\uac04 \uc218\uc0ac \uacb0\uacfc\ub97c \ubc1c\ud45c\ud588\ub2e4. \uac80\ucc30 \uc218\uc0ac \uacb0\uacfc \ub2e4\uc2a4 \uacbd\ub9ac\uc9c1\uc6d0 \uc870\ubaa8\uc528\ub294 2002\ub144 6\uc6d4\ubd80\ud130 2007\ub144 10\uc6d4 \ubb34\ub835\uae4c\uc9c0 \ub2e4\uc2a4 \uc678\ud658\uc740\ud589 \ubc95\uc778\uacc4\uc88c\uc5d0\uc11c \ucd9c\uae08 \uc5c5\ubb34\ub97c \ucc98\ub9ac\ud558\uba74\uc11c \ud5c8\uc704\ucd9c\uae08 \uc804\ud45c\ub97c \uc0bd\uc785\ud558\uac70\ub098 \ucd9c\uae08\uc561 \uacfc\ub2e4\uae30\uc7ac \uc218\ubc95\uc73c\ub85c \ub9e4\uc6d4 1\uc5b5\u223c2\uc5b5\uc6d0\uc744 \uc218\ud45c\ub098 \ud604\uae08\uc73c\ub85c \ube7c\ub3cc\ub824 80\uc5b5\uc6d0\uc744 \ud6a1\ub839\ud558\uc600\uace0, \uc774\ub97c \ub2e4\uc2a4 \ud611\ub825\uc5c5\uccb4\uc778 \uc138\uad11\uacf5\uc5c5 \uc758 \uacbd\ub9ac \uc774\ubaa8\uc528\uc5d0\uac8c \uc804\ub2ec\ud558\uc600\ub2e4. \uc774\uc528\ub294 \uc2dc\uc911\uc740\ud589 3\uacf3, \ubcf4\ud5d8\ud68c\uc0ac 1\uacf3, \ud22c\uc790\uc2e0\ud0c1\ud68c\uc0ac 1\uacf3 \ub4f1 5\uac1c \uae08\uc735\uae30\uad00\uc5d0 17\uba85 \uba85\uc758\uc758 \ucc28\uba85 \uacc4\uc88c 40\uc5ec\uac1c\ub97c \uac1c\uc124\ud574 5\ub144 \uac04 \ubcf4\uc720\ud558\uba70 120\uc5b5 4300\ub9cc\uc6d0\uc73c\ub85c \ubd88\ub838\ub2e4. \ub2e4\uc2a4 \uacbd\ub9ac \uc870\uc528\ub294 \uc774\uc911 \uc790\uc2e0\uc758 \uc5b4\uba38\ub2c8 \uba85\uc758\uc758 \uacc4\uc88c\uc5d0 \ubcf4\uc720\ud55c \ub3c8\uc744 \uc8fc\ud0dd \uad6c\uc785 \uc790\uae08 \uc6a9\ub3c4\ub85c \uc0ac\uc6a9\ud558\uc600\uc73c\uba70, \uc774\uc640 \ubcc4\ub3c4\ub85c \ud68c\uc0ac \uc790\uae08\uc744 \ubc18\ud658\ud558\uc9c0 \uc54a\uace0 \uc740\ub2c9\ud558\uc600\ub2e4. \ub610\ud55c \uae40\uc131\uc6b0 \uc804 \uc0ac\uc7a5, \uad8c\uc2b9\ud638 \uc804 \uc804\ubb34\uac00 \uacbd\ub9ac \uc870\uc528\uc640 \uacf5\ubaa8\ud558\uc5ec \ub2e4\ub978 \ube44\uc790\uae08\uc744 \uc870\uc131\ud55c \uc0ac\uc2e4, \ud68c\uc0ac \ucc28\uc6d0\uc5d0\uc11c \uc870\uc9c0\uc801\uc73c\ub85c \ube44\uc790\uae08\uc744 \uc870\uc131\ud55c \uc0ac\uc2e4, \ub2e4\uc2a4 \uacbd\uc601\uc9c4\uc774 \ub0a9\ud488 \ub300\uac00\ub97c \uba85\ubaa9\uc73c\ub85c \uae08\ud488\uc744 \uc218\uc218\ud55c \uc0ac\uc2e4\ub3c4 \ud655\uc778\ub418\uc5c8\ub2e4. \ud2b9\uc218\uc9c1\ubb34\uc720\uae30\ub85c \uace0\ubc1c\ub41c \uc815\ud638\uc601 \uc804 \ud2b9\ubcc4\uac80\uc0ac\uc5d0 \ub300\ud574\uc11c\ub294 \ubb34\ud610\uc758 \ucc98\ub9ac\ud558\uc600\ub2e4.", "paragraph_answer": "2018\ub144 2\uc6d4 19\uc77c \uac80\ucc30\uc740 \uc911\uac04 \uc218\uc0ac \uacb0\uacfc\ub97c \ubc1c\ud45c\ud588\ub2e4. \uac80\ucc30 \uc218\uc0ac \uacb0\uacfc \ub2e4\uc2a4 \uacbd\ub9ac\uc9c1\uc6d0 \uc870\ubaa8\uc528\ub294 2002\ub144 6\uc6d4\ubd80\ud130 2007\ub144 10\uc6d4 \ubb34\ub835\uae4c\uc9c0 \ub2e4\uc2a4 \uc678\ud658\uc740\ud589 \ubc95\uc778\uacc4\uc88c\uc5d0\uc11c \ucd9c\uae08 \uc5c5\ubb34\ub97c \ucc98\ub9ac\ud558\uba74\uc11c \ud5c8\uc704\ucd9c\uae08 \uc804\ud45c\ub97c \uc0bd\uc785\ud558\uac70\ub098 \ucd9c\uae08\uc561 \uacfc\ub2e4\uae30\uc7ac \uc218\ubc95\uc73c\ub85c \ub9e4\uc6d4 1\uc5b5\u223c2\uc5b5\uc6d0\uc744 \uc218\ud45c\ub098 \ud604\uae08\uc73c\ub85c \ube7c\ub3cc\ub824 80\uc5b5\uc6d0\uc744 \ud6a1\ub839\ud558\uc600\uace0, \uc774\ub97c \ub2e4\uc2a4 \ud611\ub825\uc5c5\uccb4\uc778 \uc138\uad11\uacf5\uc5c5 \uc758 \uacbd\ub9ac \uc774\ubaa8\uc528\uc5d0\uac8c \uc804\ub2ec\ud558\uc600\ub2e4. \uc774\uc528\ub294 \uc2dc\uc911\uc740\ud589 3\uacf3, \ubcf4\ud5d8\ud68c\uc0ac 1\uacf3, \ud22c\uc790\uc2e0\ud0c1\ud68c\uc0ac 1\uacf3 \ub4f1 5\uac1c \uae08\uc735\uae30\uad00\uc5d0 17\uba85 \uba85\uc758\uc758 \ucc28\uba85 \uacc4\uc88c 40\uc5ec\uac1c\ub97c \uac1c\uc124\ud574 5\ub144 \uac04 \ubcf4\uc720\ud558\uba70 120\uc5b5 4300\ub9cc\uc6d0\uc73c\ub85c \ubd88\ub838\ub2e4. \ub2e4\uc2a4 \uacbd\ub9ac \uc870\uc528\ub294 \uc774\uc911 \uc790\uc2e0\uc758 \uc5b4\uba38\ub2c8 \uba85\uc758\uc758 \uacc4\uc88c\uc5d0 \ubcf4\uc720\ud55c \ub3c8\uc744 \uc8fc\ud0dd \uad6c\uc785 \uc790\uae08 \uc6a9\ub3c4\ub85c \uc0ac\uc6a9\ud558\uc600\uc73c\uba70, \uc774\uc640 \ubcc4\ub3c4\ub85c \ud68c\uc0ac \uc790\uae08\uc744 \ubc18\ud658\ud558\uc9c0 \uc54a\uace0 \uc740\ub2c9\ud558\uc600\ub2e4. \ub610\ud55c \uae40\uc131\uc6b0 \uc804 \uc0ac\uc7a5, \uad8c\uc2b9\ud638 \uc804 \uc804\ubb34\uac00 \uacbd\ub9ac \uc870\uc528\uc640 \uacf5\ubaa8\ud558\uc5ec \ub2e4\ub978 \ube44\uc790\uae08\uc744 \uc870\uc131\ud55c \uc0ac\uc2e4, \ud68c\uc0ac \ucc28\uc6d0\uc5d0\uc11c \uc870\uc9c0\uc801\uc73c\ub85c \ube44\uc790\uae08\uc744 \uc870\uc131\ud55c \uc0ac\uc2e4, \ub2e4\uc2a4 \uacbd\uc601\uc9c4\uc774 \ub0a9\ud488 \ub300\uac00\ub97c \uba85\ubaa9\uc73c\ub85c \uae08\ud488\uc744 \uc218\uc218\ud55c \uc0ac\uc2e4\ub3c4 \ud655\uc778\ub418\uc5c8\ub2e4. \ud2b9\uc218\uc9c1\ubb34\uc720\uae30\ub85c \uace0\ubc1c\ub41c \uc815\ud638\uc601 \uc804 \ud2b9\ubcc4\uac80\uc0ac\uc5d0 \ub300\ud574\uc11c\ub294 \ubb34\ud610\uc758 \ucc98\ub9ac\ud558\uc600\ub2e4.", "sentence_answer": "\uc774\ub97c \ub2e4\uc2a4 \ud611\ub825\uc5c5\uccb4\uc778 \uc138\uad11\uacf5\uc5c5 \uc758 \uacbd\ub9ac \uc774\ubaa8\uc528\uc5d0\uac8c \uc804\ub2ec\ud558\uc600\ub2e4.", "paragraph_id": "6485118-0-1", "paragraph_question": "question: \ub2e4\uc2a4\uc5d0\uc11c 80\uc5b5\uc6d0\uc744 \ud6a1\ub839\ud55c \uc870\ubaa8\uc528\uac00 \ub3c8\uc744 \uc804\ub2ec\ud55c \uc774\ubaa8\uc528\uc758 \uc9c1\uc7a5\uc73c\ub85c \ub2e4\uc2a4 \ud611\ub825\uc5c5\uccb4\uc778 \ud68c\uc0ac\uc758 \uc774\ub984\uc740?, context: 2018\ub144 2\uc6d4 19\uc77c \uac80\ucc30\uc740 \uc911\uac04 \uc218\uc0ac \uacb0\uacfc\ub97c \ubc1c\ud45c\ud588\ub2e4. \uac80\ucc30 \uc218\uc0ac \uacb0\uacfc \ub2e4\uc2a4 \uacbd\ub9ac\uc9c1\uc6d0 \uc870\ubaa8\uc528\ub294 2002\ub144 6\uc6d4\ubd80\ud130 2007\ub144 10\uc6d4 \ubb34\ub835\uae4c\uc9c0 \ub2e4\uc2a4 \uc678\ud658\uc740\ud589 \ubc95\uc778\uacc4\uc88c\uc5d0\uc11c \ucd9c\uae08 \uc5c5\ubb34\ub97c \ucc98\ub9ac\ud558\uba74\uc11c \ud5c8\uc704\ucd9c\uae08 \uc804\ud45c\ub97c \uc0bd\uc785\ud558\uac70\ub098 \ucd9c\uae08\uc561 \uacfc\ub2e4\uae30\uc7ac \uc218\ubc95\uc73c\ub85c \ub9e4\uc6d4 1\uc5b5\u223c2\uc5b5\uc6d0\uc744 \uc218\ud45c\ub098 \ud604\uae08\uc73c\ub85c \ube7c\ub3cc\ub824 80\uc5b5\uc6d0\uc744 \ud6a1\ub839\ud558\uc600\uace0, \uc774\ub97c \ub2e4\uc2a4 \ud611\ub825\uc5c5\uccb4\uc778 \uc138\uad11\uacf5\uc5c5\uc758 \uacbd\ub9ac \uc774\ubaa8\uc528\uc5d0\uac8c \uc804\ub2ec\ud558\uc600\ub2e4. \uc774\uc528\ub294 \uc2dc\uc911\uc740\ud589 3\uacf3, \ubcf4\ud5d8\ud68c\uc0ac 1\uacf3, \ud22c\uc790\uc2e0\ud0c1\ud68c\uc0ac 1\uacf3 \ub4f1 5\uac1c \uae08\uc735\uae30\uad00\uc5d0 17\uba85 \uba85\uc758\uc758 \ucc28\uba85 \uacc4\uc88c 40\uc5ec\uac1c\ub97c \uac1c\uc124\ud574 5\ub144 \uac04 \ubcf4\uc720\ud558\uba70 120\uc5b5 4300\ub9cc\uc6d0\uc73c\ub85c \ubd88\ub838\ub2e4. \ub2e4\uc2a4 \uacbd\ub9ac \uc870\uc528\ub294 \uc774\uc911 \uc790\uc2e0\uc758 \uc5b4\uba38\ub2c8 \uba85\uc758\uc758 \uacc4\uc88c\uc5d0 \ubcf4\uc720\ud55c \ub3c8\uc744 \uc8fc\ud0dd \uad6c\uc785 \uc790\uae08 \uc6a9\ub3c4\ub85c \uc0ac\uc6a9\ud558\uc600\uc73c\uba70, \uc774\uc640 \ubcc4\ub3c4\ub85c \ud68c\uc0ac \uc790\uae08\uc744 \ubc18\ud658\ud558\uc9c0 \uc54a\uace0 \uc740\ub2c9\ud558\uc600\ub2e4. \ub610\ud55c \uae40\uc131\uc6b0 \uc804 \uc0ac\uc7a5, \uad8c\uc2b9\ud638 \uc804 \uc804\ubb34\uac00 \uacbd\ub9ac \uc870\uc528\uc640 \uacf5\ubaa8\ud558\uc5ec \ub2e4\ub978 \ube44\uc790\uae08\uc744 \uc870\uc131\ud55c \uc0ac\uc2e4, \ud68c\uc0ac \ucc28\uc6d0\uc5d0\uc11c \uc870\uc9c0\uc801\uc73c\ub85c \ube44\uc790\uae08\uc744 \uc870\uc131\ud55c \uc0ac\uc2e4, \ub2e4\uc2a4 \uacbd\uc601\uc9c4\uc774 \ub0a9\ud488 \ub300\uac00\ub97c \uba85\ubaa9\uc73c\ub85c \uae08\ud488\uc744 \uc218\uc218\ud55c \uc0ac\uc2e4\ub3c4 \ud655\uc778\ub418\uc5c8\ub2e4. \ud2b9\uc218\uc9c1\ubb34\uc720\uae30\ub85c \uace0\ubc1c\ub41c \uc815\ud638\uc601 \uc804 \ud2b9\ubcc4\uac80\uc0ac\uc5d0 \ub300\ud574\uc11c\ub294 \ubb34\ud610\uc758 \ucc98\ub9ac\ud558\uc600\ub2e4."} {"question": "\uc0ac\ud68c\uc8fc\uc758\uc5d0\uc11c\ub294 \uc5b4\ub5a4 \uacbd\uc81c\ub97c \uc639\ud638\ud558\ub294\uac00?", "paragraph": "\uc0ac\ud68c\uc8fc\uc758\ub294 \uacf5\uc815\ud558\uace0 \ud3c9\ub4f1\ud55c \uc790\uc6d0\uc758 \ubc30\ubd84\uacfc \uacbd\uc81c\ubbfc\uc8fc\uc8fc\uc758\uc5d0 \uc785\uac01\ud55c \ub178\ub3d9\uc790 \uae30\uc5c5 \uc18c\uc720\uad8c \ubc0f \uc790\uc8fc\uad00\ub9ac\ub97c \uc639\ud638\ud568\uc73c\ub85c\uc368 \uadfc\ubcf8\uc801\uc778 \uacbd\uc81c\uc801 \ubb38\uc81c\uc5d0\uc11c \uc790\ubcf8\uc8fc\uc758 \uc774\ub860\ub4e4\uc744 \ube44\ud310\ud558\uba70 \uadf8\uc640 \ub3d9\uc2dc\uc5d0 \uc0ac\ud68c\uc8fc\uc758\ub294 \ub610\ud55c \uc790\ubcf8\uc8fc\uc758\uc758 \uc0ac\ud68c\uc801, \ucca0\ud559\uc801 \ubb38\uc81c\uc5d0\uc11c\ub3c4 \ube44\ud310\ud558\uba70 \uadf8\uac83\uc740 \uc0ac\ud68c\uc8fc\uc758\uac00 \uadf9\ub2e8\uc801 \uac1c\uc778\uc8fc\uc758(\uc774\uae30\uc8fc\uc758\uc640\ub294 \ub2e4\uc74c)\ub97c \ubc18\ub300\ud568\uc73c\ub85c\uc368 \ub098\ud0c0\ub09c\ub2e4. \uc0ac\ud68c\uc8fc\uc758\uc5d0\uc11c \uacbd\uc81c\ub294 \uc9c0\uc5ed \uc0ac\ud68c \uad6c\uc870\uc5d0 \ub530\ub978 \ud611\ub3d9\uacfc \ub300\uc911\uc774 \uc9c1\uc811\uc801\uc73c\ub85c \ucc38\uc5ec\ud558\ub294 \uacc4\ud68d \uacbd\uc81c\ub97c \uc639\ud638\ud55c\ub2e4. \uc0ac\ud68c\uc8fc\uc758\ub294 \uadf9\ub2e8\uc801 \uac1c\uc778\uc8fc\uc758\uc5d0 \ubc18\ub300\ud558\uace0 \uac1c\uc778\uc758 \uc790\uc720\ubcf4\ub2e4 \uc0ac\ud68c\uc801 \ubc1c\uc804\uc744 \uc911\uc2dc\ud55c\ub2e4\ub294 \ucca0\ud559\uc801 \uac00\uce58\uac00 \uadf9\ub2e8\ud654\ub418\ubbc0\ub85c \ube44\ubbfc\uc8fc\uc801\uc778 \ucca0\ud559\uc73c\ub85c \ub9ce\uc774 \ubc1c\uc804\ub418\uc5b4\uc654\uc9c0\ub9cc(\ub300\ud45c\uc801\uc778 \uc608\ub85c \ud30c\uc2dc\uc998) \uad6d\ubbfc\ub4e4\uc758 \uc9c1\uc811 \ubc0f \uac04\uc811\uc801 \ubbfc\uc8fc\uc8fc\uc758, \uacf5\uc720\uc7ac\uc0b0\uc81c\ub97c \uc639\ud638\ud558\ub294 \uac1c\uc870\ub41c \ubbfc\uc8fc\uc8fc\uc758\uc774\uae30\ub3c4 \ud558\ub2e4.", "answer": "\uacc4\ud68d \uacbd\uc81c", "sentence": "\uc0ac\ud68c\uc8fc\uc758\uc5d0\uc11c \uacbd\uc81c\ub294 \uc9c0\uc5ed \uc0ac\ud68c \uad6c\uc870\uc5d0 \ub530\ub978 \ud611\ub3d9\uacfc \ub300\uc911\uc774 \uc9c1\uc811\uc801\uc73c\ub85c \ucc38\uc5ec\ud558\ub294 \uacc4\ud68d \uacbd\uc81c \ub97c \uc639\ud638\ud55c\ub2e4.", "paragraph_sentence": "\uc0ac\ud68c\uc8fc\uc758\ub294 \uacf5\uc815\ud558\uace0 \ud3c9\ub4f1\ud55c \uc790\uc6d0\uc758 \ubc30\ubd84\uacfc \uacbd\uc81c\ubbfc\uc8fc\uc8fc\uc758\uc5d0 \uc785\uac01\ud55c \ub178\ub3d9\uc790 \uae30\uc5c5 \uc18c\uc720\uad8c \ubc0f \uc790\uc8fc\uad00\ub9ac\ub97c \uc639\ud638\ud568\uc73c\ub85c\uc368 \uadfc\ubcf8\uc801\uc778 \uacbd\uc81c\uc801 \ubb38\uc81c\uc5d0\uc11c \uc790\ubcf8\uc8fc\uc758 \uc774\ub860\ub4e4\uc744 \ube44\ud310\ud558\uba70 \uadf8\uc640 \ub3d9\uc2dc\uc5d0 \uc0ac\ud68c\uc8fc\uc758\ub294 \ub610\ud55c \uc790\ubcf8\uc8fc\uc758\uc758 \uc0ac\ud68c\uc801, \ucca0\ud559\uc801 \ubb38\uc81c\uc5d0\uc11c\ub3c4 \ube44\ud310\ud558\uba70 \uadf8\uac83\uc740 \uc0ac\ud68c\uc8fc\uc758\uac00 \uadf9\ub2e8\uc801 \uac1c\uc778\uc8fc\uc758(\uc774\uae30\uc8fc\uc758\uc640\ub294 \ub2e4\uc74c)\ub97c \ubc18\ub300\ud568\uc73c\ub85c\uc368 \ub098\ud0c0\ub09c\ub2e4. \uc0ac\ud68c\uc8fc\uc758\uc5d0\uc11c \uacbd\uc81c\ub294 \uc9c0\uc5ed \uc0ac\ud68c \uad6c\uc870\uc5d0 \ub530\ub978 \ud611\ub3d9\uacfc \ub300\uc911\uc774 \uc9c1\uc811\uc801\uc73c\ub85c \ucc38\uc5ec\ud558\ub294 \uacc4\ud68d \uacbd\uc81c \ub97c \uc639\ud638\ud55c\ub2e4. \uc0ac\ud68c\uc8fc\uc758\ub294 \uadf9\ub2e8\uc801 \uac1c\uc778\uc8fc\uc758\uc5d0 \ubc18\ub300\ud558\uace0 \uac1c\uc778\uc758 \uc790\uc720\ubcf4\ub2e4 \uc0ac\ud68c\uc801 \ubc1c\uc804\uc744 \uc911\uc2dc\ud55c\ub2e4\ub294 \ucca0\ud559\uc801 \uac00\uce58\uac00 \uadf9\ub2e8\ud654\ub418\ubbc0\ub85c \ube44\ubbfc\uc8fc\uc801\uc778 \ucca0\ud559\uc73c\ub85c \ub9ce\uc774 \ubc1c\uc804\ub418\uc5b4\uc654\uc9c0\ub9cc(\ub300\ud45c\uc801\uc778 \uc608\ub85c \ud30c\uc2dc\uc998) \uad6d\ubbfc\ub4e4\uc758 \uc9c1\uc811 \ubc0f \uac04\uc811\uc801 \ubbfc\uc8fc\uc8fc\uc758, \uacf5\uc720\uc7ac\uc0b0\uc81c\ub97c \uc639\ud638\ud558\ub294 \uac1c\uc870\ub41c \ubbfc\uc8fc\uc8fc\uc758\uc774\uae30\ub3c4 \ud558\ub2e4.", "paragraph_answer": "\uc0ac\ud68c\uc8fc\uc758\ub294 \uacf5\uc815\ud558\uace0 \ud3c9\ub4f1\ud55c \uc790\uc6d0\uc758 \ubc30\ubd84\uacfc \uacbd\uc81c\ubbfc\uc8fc\uc8fc\uc758\uc5d0 \uc785\uac01\ud55c \ub178\ub3d9\uc790 \uae30\uc5c5 \uc18c\uc720\uad8c \ubc0f \uc790\uc8fc\uad00\ub9ac\ub97c \uc639\ud638\ud568\uc73c\ub85c\uc368 \uadfc\ubcf8\uc801\uc778 \uacbd\uc81c\uc801 \ubb38\uc81c\uc5d0\uc11c \uc790\ubcf8\uc8fc\uc758 \uc774\ub860\ub4e4\uc744 \ube44\ud310\ud558\uba70 \uadf8\uc640 \ub3d9\uc2dc\uc5d0 \uc0ac\ud68c\uc8fc\uc758\ub294 \ub610\ud55c \uc790\ubcf8\uc8fc\uc758\uc758 \uc0ac\ud68c\uc801, \ucca0\ud559\uc801 \ubb38\uc81c\uc5d0\uc11c\ub3c4 \ube44\ud310\ud558\uba70 \uadf8\uac83\uc740 \uc0ac\ud68c\uc8fc\uc758\uac00 \uadf9\ub2e8\uc801 \uac1c\uc778\uc8fc\uc758(\uc774\uae30\uc8fc\uc758\uc640\ub294 \ub2e4\uc74c)\ub97c \ubc18\ub300\ud568\uc73c\ub85c\uc368 \ub098\ud0c0\ub09c\ub2e4. \uc0ac\ud68c\uc8fc\uc758\uc5d0\uc11c \uacbd\uc81c\ub294 \uc9c0\uc5ed \uc0ac\ud68c \uad6c\uc870\uc5d0 \ub530\ub978 \ud611\ub3d9\uacfc \ub300\uc911\uc774 \uc9c1\uc811\uc801\uc73c\ub85c \ucc38\uc5ec\ud558\ub294 \uacc4\ud68d \uacbd\uc81c \ub97c \uc639\ud638\ud55c\ub2e4. \uc0ac\ud68c\uc8fc\uc758\ub294 \uadf9\ub2e8\uc801 \uac1c\uc778\uc8fc\uc758\uc5d0 \ubc18\ub300\ud558\uace0 \uac1c\uc778\uc758 \uc790\uc720\ubcf4\ub2e4 \uc0ac\ud68c\uc801 \ubc1c\uc804\uc744 \uc911\uc2dc\ud55c\ub2e4\ub294 \ucca0\ud559\uc801 \uac00\uce58\uac00 \uadf9\ub2e8\ud654\ub418\ubbc0\ub85c \ube44\ubbfc\uc8fc\uc801\uc778 \ucca0\ud559\uc73c\ub85c \ub9ce\uc774 \ubc1c\uc804\ub418\uc5b4\uc654\uc9c0\ub9cc(\ub300\ud45c\uc801\uc778 \uc608\ub85c \ud30c\uc2dc\uc998) \uad6d\ubbfc\ub4e4\uc758 \uc9c1\uc811 \ubc0f \uac04\uc811\uc801 \ubbfc\uc8fc\uc8fc\uc758, \uacf5\uc720\uc7ac\uc0b0\uc81c\ub97c \uc639\ud638\ud558\ub294 \uac1c\uc870\ub41c \ubbfc\uc8fc\uc8fc\uc758\uc774\uae30\ub3c4 \ud558\ub2e4.", "sentence_answer": "\uc0ac\ud68c\uc8fc\uc758\uc5d0\uc11c \uacbd\uc81c\ub294 \uc9c0\uc5ed \uc0ac\ud68c \uad6c\uc870\uc5d0 \ub530\ub978 \ud611\ub3d9\uacfc \ub300\uc911\uc774 \uc9c1\uc811\uc801\uc73c\ub85c \ucc38\uc5ec\ud558\ub294 \uacc4\ud68d \uacbd\uc81c \ub97c \uc639\ud638\ud55c\ub2e4.", "paragraph_id": "6530258-2-1", "paragraph_question": "question: \uc0ac\ud68c\uc8fc\uc758\uc5d0\uc11c\ub294 \uc5b4\ub5a4 \uacbd\uc81c\ub97c \uc639\ud638\ud558\ub294\uac00?, context: \uc0ac\ud68c\uc8fc\uc758\ub294 \uacf5\uc815\ud558\uace0 \ud3c9\ub4f1\ud55c \uc790\uc6d0\uc758 \ubc30\ubd84\uacfc \uacbd\uc81c\ubbfc\uc8fc\uc8fc\uc758\uc5d0 \uc785\uac01\ud55c \ub178\ub3d9\uc790 \uae30\uc5c5 \uc18c\uc720\uad8c \ubc0f \uc790\uc8fc\uad00\ub9ac\ub97c \uc639\ud638\ud568\uc73c\ub85c\uc368 \uadfc\ubcf8\uc801\uc778 \uacbd\uc81c\uc801 \ubb38\uc81c\uc5d0\uc11c \uc790\ubcf8\uc8fc\uc758 \uc774\ub860\ub4e4\uc744 \ube44\ud310\ud558\uba70 \uadf8\uc640 \ub3d9\uc2dc\uc5d0 \uc0ac\ud68c\uc8fc\uc758\ub294 \ub610\ud55c \uc790\ubcf8\uc8fc\uc758\uc758 \uc0ac\ud68c\uc801, \ucca0\ud559\uc801 \ubb38\uc81c\uc5d0\uc11c\ub3c4 \ube44\ud310\ud558\uba70 \uadf8\uac83\uc740 \uc0ac\ud68c\uc8fc\uc758\uac00 \uadf9\ub2e8\uc801 \uac1c\uc778\uc8fc\uc758(\uc774\uae30\uc8fc\uc758\uc640\ub294 \ub2e4\uc74c)\ub97c \ubc18\ub300\ud568\uc73c\ub85c\uc368 \ub098\ud0c0\ub09c\ub2e4. \uc0ac\ud68c\uc8fc\uc758\uc5d0\uc11c \uacbd\uc81c\ub294 \uc9c0\uc5ed \uc0ac\ud68c \uad6c\uc870\uc5d0 \ub530\ub978 \ud611\ub3d9\uacfc \ub300\uc911\uc774 \uc9c1\uc811\uc801\uc73c\ub85c \ucc38\uc5ec\ud558\ub294 \uacc4\ud68d \uacbd\uc81c\ub97c \uc639\ud638\ud55c\ub2e4. \uc0ac\ud68c\uc8fc\uc758\ub294 \uadf9\ub2e8\uc801 \uac1c\uc778\uc8fc\uc758\uc5d0 \ubc18\ub300\ud558\uace0 \uac1c\uc778\uc758 \uc790\uc720\ubcf4\ub2e4 \uc0ac\ud68c\uc801 \ubc1c\uc804\uc744 \uc911\uc2dc\ud55c\ub2e4\ub294 \ucca0\ud559\uc801 \uac00\uce58\uac00 \uadf9\ub2e8\ud654\ub418\ubbc0\ub85c \ube44\ubbfc\uc8fc\uc801\uc778 \ucca0\ud559\uc73c\ub85c \ub9ce\uc774 \ubc1c\uc804\ub418\uc5b4\uc654\uc9c0\ub9cc(\ub300\ud45c\uc801\uc778 \uc608\ub85c \ud30c\uc2dc\uc998) \uad6d\ubbfc\ub4e4\uc758 \uc9c1\uc811 \ubc0f \uac04\uc811\uc801 \ubbfc\uc8fc\uc8fc\uc758, \uacf5\uc720\uc7ac\uc0b0\uc81c\ub97c \uc639\ud638\ud558\ub294 \uac1c\uc870\ub41c \ubbfc\uc8fc\uc8fc\uc758\uc774\uae30\ub3c4 \ud558\ub2e4."} {"question": "\uae00\ub80c \ucf08\ud2b8\ub2c8\uc5d0 \uc788\ub294 \uc774 \uc9d0\uc2b9\uc744 \ucf08\ud53c\ub77c \uc0dd\uac01\ud55c \ubbfc\uc18d\ud559\uc790\ub294\ub204\uad6c\uc778\uac00?", "paragraph": "\ucf08\ud53c\ub294 \ud76c\uc0dd\uc790\ub97c \ubb3c \uc18d\uc73c\ub85c \ub04c\uace0 \uac00\uc11c \uc7a1\uc544\uba39\uace0 \ubb3c\uac00\uc5d0 \ub0b4\uc7a5\ub9cc \ub358\uc838\ub193\ub294\ub2e4. \ub9d0\uc758 \ud615\ud0dc\ub97c \ud558\uace0 \uc788\uc744 \ub54c \ucf08\ud53c\ub294 \ubab8\uc758 \uae38\uc774\ub97c \ub298\ub824\uc11c \uc5ec\ub7ec \uba85\uc758 \uc0ac\ub78c\uc744 \ud0dc\uc6cc \ubb3c \uc18d\uc73c\ub85c \uac00\ub77c\uc549\uc744 \uc218 \uc788\ub2e4. \ucf08\ud53c \uc774\uc57c\uae30\uc5d0\uc11c \ud754\ud788 \ub098\ud0c0\ub098\ub294 \ud14c\ub9c8\ub85c \uc5ec\ub7ec \uba85\uc758 \uc544\uc774\ub4e4\uc774 \ucf08\ud53c\uc758 \ub4f1\uc5d0 \uc62c\ub77c\ud0d4\uace0 \uc544\uc774 \ud55c \uba85\ub9cc \ubb3c\uac00\uc5d0 \ub0a8\uac8c \ub418\ub294 \uac83\uc774 \uc788\ub2e4. \ubcf4\ud1b5 \uadf8 \uc544\uc774\ub294 \uc5b4\ub9b0 \uc0ac\ub0b4\uc544\uc774\uc778\ub370, \ub9d0\uc744 \uc4f0\ub2e4\ub4ec\uc73c\ub824 \ud588\ub2e4\uac00 \uc190\uc774 \ub9d0\uc758 \ubaa9\uc5d0 \ub4e4\ub7ec\ubd99\uc5b4 \ubc84\ub9b0\ub2e4. \uc774\uc57c\uae30\uc5d0 \ub530\ub77c \uc0b4\uc544\ub0a8\uae30 \uc704\ud574 \uc190\uac00\ub77d \ub610\ub294 \uc190\uc744 \uc798\ub77c\ub0b4\uc57c \ud588\ub2e4\ub294 \ubcc0\ud615\uc774 \uc788\ub2e4. \uadf8 \uc544\uc774 \ud55c \uba85\uc740 \uc0b4\uc544\ub0a8\uc9c0\ub9cc \ub2e4\ub978 \uc544\uc774\ub4e4\uc740 \ubb3c \uc18d\uc73c\ub85c \ub04c\ub824\uac00 \uc775\uc0ac\ud588\uace0, \ub098\uc911\uc5d0 \uc544\uc774\ub4e4\uc758 \ub0b4\uc7a5\ub9cc \ubc1c\uacac\ub41c\ub2e4. \ud37c\uc2a4\uc154\uc758 \uae00\ub80c \ucf08\ud2b8\ub2c8(Glen Keltney)\uc5d0 \uc0ac\ub294 \uc774 \uc9d0\uc2b9\uc744 20\uc138\uae30 \ubbfc\uc18d\ud559\uc790 \uce90\uc11c\ub9b0 \uba54\ub9ac \ube0c\ub9ad\uc2a4\ub294 \ucf08\ud53c\ub77c\uace0 \ud310\ub2e8\ud588\uc9c0\ub9cc, \uc5ed\uc2dc \ud37c\uc2a4\uc154\ub97c \ubc30\uacbd\uc73c\ub85c \ud558\ub294 \uc720\uc0ac\ud55c \uc774\uc57c\uae30\uc5d0\ub294 \uc5d0\ud750\uc73c\uc2dc\ucee4\uac00 \uc544\uc774\ub4e4\uc744 \uc7a1\uc544\uba39\uc740 \ubc94\uc778\uc774\uace0, \uc0b4\uc544\ub0a8\uc740 \uc0ac\ub0b4\uc544\uc774\uc5d0 \uad00\ud55c \uc774\uc57c\uae30\uc5d0 \uc724\uc0c9\uc774 \ub354\ud574\uc838 \uc788\ub2e4. \ucf08\ud53c\uac00 \ubc94\uc778\uc73c\ub85c \uc804\ud574\uc9c0\ub294 \uc11c\uc18c\uc758 \uc804\uc2b9\uc5d0\uc11c\ub294 \uc544\uc774\uac00 \uc190\uac00\ub77d\uc744 \uc798\ub77c\ub0c8\ub2e4\uace0 \ud55c\ub2e4. \ud558\uc77c\ub79c\ub4dc\uc758 \uc218\ub098\ub974\ud2b8\uc5d0\uc11c \uc804\uc2b9\ub418\ub294 \uac19\uc740 \uc774\uc57c\uae30\uc5d0\ub294 \uc2e4\uc885\ub41c \uc544\uc774\uac00 \uc544\ud649 \uba85\uc774\ub77c\uace0 \uad6c\uccb4\uc801\uc73c\ub85c \ubb18\uc0ac\ub418\uace0, \uadf8 \uc911 \ud55c \uba85\uc758 \ub0b4\uc7a5\ub9cc \ubc1c\uacac\ub418\uc5c8\ub2e4\uace0 \ud55c\ub2e4. \uc5ec\uae30\uc11c\ub3c4 \uc0b4\uc544\ub0a8\uc740 \uc544\uc774\ub294 \uc190\uac00\ub77d\uc744 \uc798\ub77c\ub0b4\uc11c \uc0b4\uc544\ub0ac\uace0, \uadf8 \uc544\uc774\uc758 \uc8fc\uba38\ub2c8\uc5d0 \uc131\uacbd\ucc45\uc774 \ub4e4\uc5b4 \uc788\uc5c8\ub2e4\ub294 \uad70\ub9d0\uc774 \ub367\ubd99\uc5b4 \uc788\ub2e4. \uadf8\ub808\uace0\ub974\uc2a8 \ucea0\ubca8\uc740 \uc774 \uc774\uc57c\uae30\uc758 \uc9d0\uc2b9\uc740 \ucf08\ud53c\ubcf4\ub2e4\ub294 \uc5d0\ud750\uc73c\uc2dc\ucee4\uc77c \uac83\uc774\ub77c\uace0 \uc0dd\uac01\ud558\uba70, \uc774 \uc774\uc57c\uae30\uac00 \u201c\uc544\uc774\ub4e4\uc774 \uc8fc\uc77c\uc5d0 \uc2f8\ub3cc\uc544\ub2e4\ub2c8\uc9c0 \ubabb\ud558\uac8c \ud558\ub824\ub294 \uacbd\uac74\ud55c \uac70\uc9d3\ub9d0\u201d\uc774\ub77c\uace0 \ud55c\ub2e4.", "answer": "\uce90\uc11c\ub9b0 \uba54\ub9ac \ube0c\ub9ad\uc2a4", "sentence": "\ud37c\uc2a4\uc154\uc758 \uae00\ub80c \ucf08\ud2b8\ub2c8(Glen Keltney)\uc5d0 \uc0ac\ub294 \uc774 \uc9d0\uc2b9\uc744 20\uc138\uae30 \ubbfc\uc18d\ud559\uc790 \uce90\uc11c\ub9b0 \uba54\ub9ac \ube0c\ub9ad\uc2a4 \ub294 \ucf08\ud53c\ub77c\uace0 \ud310\ub2e8\ud588\uc9c0\ub9cc, \uc5ed\uc2dc \ud37c\uc2a4\uc154\ub97c \ubc30\uacbd\uc73c\ub85c \ud558\ub294 \uc720\uc0ac\ud55c \uc774\uc57c\uae30\uc5d0\ub294 \uc5d0\ud750\uc73c\uc2dc\ucee4\uac00 \uc544\uc774\ub4e4\uc744 \uc7a1\uc544\uba39\uc740 \ubc94\uc778\uc774\uace0, \uc0b4\uc544\ub0a8\uc740 \uc0ac\ub0b4\uc544\uc774\uc5d0 \uad00\ud55c \uc774\uc57c\uae30\uc5d0 \uc724\uc0c9\uc774 \ub354\ud574\uc838 \uc788\ub2e4.", "paragraph_sentence": "\ucf08\ud53c\ub294 \ud76c\uc0dd\uc790\ub97c \ubb3c \uc18d\uc73c\ub85c \ub04c\uace0 \uac00\uc11c \uc7a1\uc544\uba39\uace0 \ubb3c\uac00\uc5d0 \ub0b4\uc7a5\ub9cc \ub358\uc838\ub193\ub294\ub2e4. \ub9d0\uc758 \ud615\ud0dc\ub97c \ud558\uace0 \uc788\uc744 \ub54c \ucf08\ud53c\ub294 \ubab8\uc758 \uae38\uc774\ub97c \ub298\ub824\uc11c \uc5ec\ub7ec \uba85\uc758 \uc0ac\ub78c\uc744 \ud0dc\uc6cc \ubb3c \uc18d\uc73c\ub85c \uac00\ub77c\uc549\uc744 \uc218 \uc788\ub2e4. \ucf08\ud53c \uc774\uc57c\uae30\uc5d0\uc11c \ud754\ud788 \ub098\ud0c0\ub098\ub294 \ud14c\ub9c8\ub85c \uc5ec\ub7ec \uba85\uc758 \uc544\uc774\ub4e4\uc774 \ucf08\ud53c\uc758 \ub4f1\uc5d0 \uc62c\ub77c\ud0d4\uace0 \uc544\uc774 \ud55c \uba85\ub9cc \ubb3c\uac00\uc5d0 \ub0a8\uac8c \ub418\ub294 \uac83\uc774 \uc788\ub2e4. \ubcf4\ud1b5 \uadf8 \uc544\uc774\ub294 \uc5b4\ub9b0 \uc0ac\ub0b4\uc544\uc774\uc778\ub370, \ub9d0\uc744 \uc4f0\ub2e4\ub4ec\uc73c\ub824 \ud588\ub2e4\uac00 \uc190\uc774 \ub9d0\uc758 \ubaa9\uc5d0 \ub4e4\ub7ec\ubd99\uc5b4 \ubc84\ub9b0\ub2e4. \uc774\uc57c\uae30\uc5d0 \ub530\ub77c \uc0b4\uc544\ub0a8\uae30 \uc704\ud574 \uc190\uac00\ub77d \ub610\ub294 \uc190\uc744 \uc798\ub77c\ub0b4\uc57c \ud588\ub2e4\ub294 \ubcc0\ud615\uc774 \uc788\ub2e4. \uadf8 \uc544\uc774 \ud55c \uba85\uc740 \uc0b4\uc544\ub0a8\uc9c0\ub9cc \ub2e4\ub978 \uc544\uc774\ub4e4\uc740 \ubb3c \uc18d\uc73c\ub85c \ub04c\ub824\uac00 \uc775\uc0ac\ud588\uace0, \ub098\uc911\uc5d0 \uc544\uc774\ub4e4\uc758 \ub0b4\uc7a5\ub9cc \ubc1c\uacac\ub41c\ub2e4. \ud37c\uc2a4\uc154\uc758 \uae00\ub80c \ucf08\ud2b8\ub2c8(Glen Keltney)\uc5d0 \uc0ac\ub294 \uc774 \uc9d0\uc2b9\uc744 20\uc138\uae30 \ubbfc\uc18d\ud559\uc790 \uce90\uc11c\ub9b0 \uba54\ub9ac \ube0c\ub9ad\uc2a4 \ub294 \ucf08\ud53c\ub77c\uace0 \ud310\ub2e8\ud588\uc9c0\ub9cc, \uc5ed\uc2dc \ud37c\uc2a4\uc154\ub97c \ubc30\uacbd\uc73c\ub85c \ud558\ub294 \uc720\uc0ac\ud55c \uc774\uc57c\uae30\uc5d0\ub294 \uc5d0\ud750\uc73c\uc2dc\ucee4\uac00 \uc544\uc774\ub4e4\uc744 \uc7a1\uc544\uba39\uc740 \ubc94\uc778\uc774\uace0, \uc0b4\uc544\ub0a8\uc740 \uc0ac\ub0b4\uc544\uc774\uc5d0 \uad00\ud55c \uc774\uc57c\uae30\uc5d0 \uc724\uc0c9\uc774 \ub354\ud574\uc838 \uc788\ub2e4. \ucf08\ud53c\uac00 \ubc94\uc778\uc73c\ub85c \uc804\ud574\uc9c0\ub294 \uc11c\uc18c\uc758 \uc804\uc2b9\uc5d0\uc11c\ub294 \uc544\uc774\uac00 \uc190\uac00\ub77d\uc744 \uc798\ub77c\ub0c8\ub2e4\uace0 \ud55c\ub2e4. \ud558\uc77c\ub79c\ub4dc\uc758 \uc218\ub098\ub974\ud2b8\uc5d0\uc11c \uc804\uc2b9\ub418\ub294 \uac19\uc740 \uc774\uc57c\uae30\uc5d0\ub294 \uc2e4\uc885\ub41c \uc544\uc774\uac00 \uc544\ud649 \uba85\uc774\ub77c\uace0 \uad6c\uccb4\uc801\uc73c\ub85c \ubb18\uc0ac\ub418\uace0, \uadf8 \uc911 \ud55c \uba85\uc758 \ub0b4\uc7a5\ub9cc \ubc1c\uacac\ub418\uc5c8\ub2e4\uace0 \ud55c\ub2e4. \uc5ec\uae30\uc11c\ub3c4 \uc0b4\uc544\ub0a8\uc740 \uc544\uc774\ub294 \uc190\uac00\ub77d\uc744 \uc798\ub77c\ub0b4\uc11c \uc0b4\uc544\ub0ac\uace0, \uadf8 \uc544\uc774\uc758 \uc8fc\uba38\ub2c8\uc5d0 \uc131\uacbd\ucc45\uc774 \ub4e4\uc5b4 \uc788\uc5c8\ub2e4\ub294 \uad70\ub9d0\uc774 \ub367\ubd99\uc5b4 \uc788\ub2e4. \uadf8\ub808\uace0\ub974\uc2a8 \ucea0\ubca8\uc740 \uc774 \uc774\uc57c\uae30\uc758 \uc9d0\uc2b9\uc740 \ucf08\ud53c\ubcf4\ub2e4\ub294 \uc5d0\ud750\uc73c\uc2dc\ucee4\uc77c \uac83\uc774\ub77c\uace0 \uc0dd\uac01\ud558\uba70, \uc774 \uc774\uc57c\uae30\uac00 \u201c\uc544\uc774\ub4e4\uc774 \uc8fc\uc77c\uc5d0 \uc2f8\ub3cc\uc544\ub2e4\ub2c8\uc9c0 \ubabb\ud558\uac8c \ud558\ub824\ub294 \uacbd\uac74\ud55c \uac70\uc9d3\ub9d0\u201d\uc774\ub77c\uace0 \ud55c\ub2e4.", "paragraph_answer": "\ucf08\ud53c\ub294 \ud76c\uc0dd\uc790\ub97c \ubb3c \uc18d\uc73c\ub85c \ub04c\uace0 \uac00\uc11c \uc7a1\uc544\uba39\uace0 \ubb3c\uac00\uc5d0 \ub0b4\uc7a5\ub9cc \ub358\uc838\ub193\ub294\ub2e4. \ub9d0\uc758 \ud615\ud0dc\ub97c \ud558\uace0 \uc788\uc744 \ub54c \ucf08\ud53c\ub294 \ubab8\uc758 \uae38\uc774\ub97c \ub298\ub824\uc11c \uc5ec\ub7ec \uba85\uc758 \uc0ac\ub78c\uc744 \ud0dc\uc6cc \ubb3c \uc18d\uc73c\ub85c \uac00\ub77c\uc549\uc744 \uc218 \uc788\ub2e4. \ucf08\ud53c \uc774\uc57c\uae30\uc5d0\uc11c \ud754\ud788 \ub098\ud0c0\ub098\ub294 \ud14c\ub9c8\ub85c \uc5ec\ub7ec \uba85\uc758 \uc544\uc774\ub4e4\uc774 \ucf08\ud53c\uc758 \ub4f1\uc5d0 \uc62c\ub77c\ud0d4\uace0 \uc544\uc774 \ud55c \uba85\ub9cc \ubb3c\uac00\uc5d0 \ub0a8\uac8c \ub418\ub294 \uac83\uc774 \uc788\ub2e4. \ubcf4\ud1b5 \uadf8 \uc544\uc774\ub294 \uc5b4\ub9b0 \uc0ac\ub0b4\uc544\uc774\uc778\ub370, \ub9d0\uc744 \uc4f0\ub2e4\ub4ec\uc73c\ub824 \ud588\ub2e4\uac00 \uc190\uc774 \ub9d0\uc758 \ubaa9\uc5d0 \ub4e4\ub7ec\ubd99\uc5b4 \ubc84\ub9b0\ub2e4. \uc774\uc57c\uae30\uc5d0 \ub530\ub77c \uc0b4\uc544\ub0a8\uae30 \uc704\ud574 \uc190\uac00\ub77d \ub610\ub294 \uc190\uc744 \uc798\ub77c\ub0b4\uc57c \ud588\ub2e4\ub294 \ubcc0\ud615\uc774 \uc788\ub2e4. \uadf8 \uc544\uc774 \ud55c \uba85\uc740 \uc0b4\uc544\ub0a8\uc9c0\ub9cc \ub2e4\ub978 \uc544\uc774\ub4e4\uc740 \ubb3c \uc18d\uc73c\ub85c \ub04c\ub824\uac00 \uc775\uc0ac\ud588\uace0, \ub098\uc911\uc5d0 \uc544\uc774\ub4e4\uc758 \ub0b4\uc7a5\ub9cc \ubc1c\uacac\ub41c\ub2e4. \ud37c\uc2a4\uc154\uc758 \uae00\ub80c \ucf08\ud2b8\ub2c8(Glen Keltney)\uc5d0 \uc0ac\ub294 \uc774 \uc9d0\uc2b9\uc744 20\uc138\uae30 \ubbfc\uc18d\ud559\uc790 \uce90\uc11c\ub9b0 \uba54\ub9ac \ube0c\ub9ad\uc2a4 \ub294 \ucf08\ud53c\ub77c\uace0 \ud310\ub2e8\ud588\uc9c0\ub9cc, \uc5ed\uc2dc \ud37c\uc2a4\uc154\ub97c \ubc30\uacbd\uc73c\ub85c \ud558\ub294 \uc720\uc0ac\ud55c \uc774\uc57c\uae30\uc5d0\ub294 \uc5d0\ud750\uc73c\uc2dc\ucee4\uac00 \uc544\uc774\ub4e4\uc744 \uc7a1\uc544\uba39\uc740 \ubc94\uc778\uc774\uace0, \uc0b4\uc544\ub0a8\uc740 \uc0ac\ub0b4\uc544\uc774\uc5d0 \uad00\ud55c \uc774\uc57c\uae30\uc5d0 \uc724\uc0c9\uc774 \ub354\ud574\uc838 \uc788\ub2e4. \ucf08\ud53c\uac00 \ubc94\uc778\uc73c\ub85c \uc804\ud574\uc9c0\ub294 \uc11c\uc18c\uc758 \uc804\uc2b9\uc5d0\uc11c\ub294 \uc544\uc774\uac00 \uc190\uac00\ub77d\uc744 \uc798\ub77c\ub0c8\ub2e4\uace0 \ud55c\ub2e4. \ud558\uc77c\ub79c\ub4dc\uc758 \uc218\ub098\ub974\ud2b8\uc5d0\uc11c \uc804\uc2b9\ub418\ub294 \uac19\uc740 \uc774\uc57c\uae30\uc5d0\ub294 \uc2e4\uc885\ub41c \uc544\uc774\uac00 \uc544\ud649 \uba85\uc774\ub77c\uace0 \uad6c\uccb4\uc801\uc73c\ub85c \ubb18\uc0ac\ub418\uace0, \uadf8 \uc911 \ud55c \uba85\uc758 \ub0b4\uc7a5\ub9cc \ubc1c\uacac\ub418\uc5c8\ub2e4\uace0 \ud55c\ub2e4. \uc5ec\uae30\uc11c\ub3c4 \uc0b4\uc544\ub0a8\uc740 \uc544\uc774\ub294 \uc190\uac00\ub77d\uc744 \uc798\ub77c\ub0b4\uc11c \uc0b4\uc544\ub0ac\uace0, \uadf8 \uc544\uc774\uc758 \uc8fc\uba38\ub2c8\uc5d0 \uc131\uacbd\ucc45\uc774 \ub4e4\uc5b4 \uc788\uc5c8\ub2e4\ub294 \uad70\ub9d0\uc774 \ub367\ubd99\uc5b4 \uc788\ub2e4. \uadf8\ub808\uace0\ub974\uc2a8 \ucea0\ubca8\uc740 \uc774 \uc774\uc57c\uae30\uc758 \uc9d0\uc2b9\uc740 \ucf08\ud53c\ubcf4\ub2e4\ub294 \uc5d0\ud750\uc73c\uc2dc\ucee4\uc77c \uac83\uc774\ub77c\uace0 \uc0dd\uac01\ud558\uba70, \uc774 \uc774\uc57c\uae30\uac00 \u201c\uc544\uc774\ub4e4\uc774 \uc8fc\uc77c\uc5d0 \uc2f8\ub3cc\uc544\ub2e4\ub2c8\uc9c0 \ubabb\ud558\uac8c \ud558\ub824\ub294 \uacbd\uac74\ud55c \uac70\uc9d3\ub9d0\u201d\uc774\ub77c\uace0 \ud55c\ub2e4.", "sentence_answer": "\ud37c\uc2a4\uc154\uc758 \uae00\ub80c \ucf08\ud2b8\ub2c8(Glen Keltney)\uc5d0 \uc0ac\ub294 \uc774 \uc9d0\uc2b9\uc744 20\uc138\uae30 \ubbfc\uc18d\ud559\uc790 \uce90\uc11c\ub9b0 \uba54\ub9ac \ube0c\ub9ad\uc2a4 \ub294 \ucf08\ud53c\ub77c\uace0 \ud310\ub2e8\ud588\uc9c0\ub9cc, \uc5ed\uc2dc \ud37c\uc2a4\uc154\ub97c \ubc30\uacbd\uc73c\ub85c \ud558\ub294 \uc720\uc0ac\ud55c \uc774\uc57c\uae30\uc5d0\ub294 \uc5d0\ud750\uc73c\uc2dc\ucee4\uac00 \uc544\uc774\ub4e4\uc744 \uc7a1\uc544\uba39\uc740 \ubc94\uc778\uc774\uace0, \uc0b4\uc544\ub0a8\uc740 \uc0ac\ub0b4\uc544\uc774\uc5d0 \uad00\ud55c \uc774\uc57c\uae30\uc5d0 \uc724\uc0c9\uc774 \ub354\ud574\uc838 \uc788\ub2e4.", "paragraph_id": "6597820-1-0", "paragraph_question": "question: \uae00\ub80c \ucf08\ud2b8\ub2c8\uc5d0 \uc788\ub294 \uc774 \uc9d0\uc2b9\uc744 \ucf08\ud53c\ub77c \uc0dd\uac01\ud55c \ubbfc\uc18d\ud559\uc790\ub294\ub204\uad6c\uc778\uac00?, context: \ucf08\ud53c\ub294 \ud76c\uc0dd\uc790\ub97c \ubb3c \uc18d\uc73c\ub85c \ub04c\uace0 \uac00\uc11c \uc7a1\uc544\uba39\uace0 \ubb3c\uac00\uc5d0 \ub0b4\uc7a5\ub9cc \ub358\uc838\ub193\ub294\ub2e4. \ub9d0\uc758 \ud615\ud0dc\ub97c \ud558\uace0 \uc788\uc744 \ub54c \ucf08\ud53c\ub294 \ubab8\uc758 \uae38\uc774\ub97c \ub298\ub824\uc11c \uc5ec\ub7ec \uba85\uc758 \uc0ac\ub78c\uc744 \ud0dc\uc6cc \ubb3c \uc18d\uc73c\ub85c \uac00\ub77c\uc549\uc744 \uc218 \uc788\ub2e4. \ucf08\ud53c \uc774\uc57c\uae30\uc5d0\uc11c \ud754\ud788 \ub098\ud0c0\ub098\ub294 \ud14c\ub9c8\ub85c \uc5ec\ub7ec \uba85\uc758 \uc544\uc774\ub4e4\uc774 \ucf08\ud53c\uc758 \ub4f1\uc5d0 \uc62c\ub77c\ud0d4\uace0 \uc544\uc774 \ud55c \uba85\ub9cc \ubb3c\uac00\uc5d0 \ub0a8\uac8c \ub418\ub294 \uac83\uc774 \uc788\ub2e4. \ubcf4\ud1b5 \uadf8 \uc544\uc774\ub294 \uc5b4\ub9b0 \uc0ac\ub0b4\uc544\uc774\uc778\ub370, \ub9d0\uc744 \uc4f0\ub2e4\ub4ec\uc73c\ub824 \ud588\ub2e4\uac00 \uc190\uc774 \ub9d0\uc758 \ubaa9\uc5d0 \ub4e4\ub7ec\ubd99\uc5b4 \ubc84\ub9b0\ub2e4. \uc774\uc57c\uae30\uc5d0 \ub530\ub77c \uc0b4\uc544\ub0a8\uae30 \uc704\ud574 \uc190\uac00\ub77d \ub610\ub294 \uc190\uc744 \uc798\ub77c\ub0b4\uc57c \ud588\ub2e4\ub294 \ubcc0\ud615\uc774 \uc788\ub2e4. \uadf8 \uc544\uc774 \ud55c \uba85\uc740 \uc0b4\uc544\ub0a8\uc9c0\ub9cc \ub2e4\ub978 \uc544\uc774\ub4e4\uc740 \ubb3c \uc18d\uc73c\ub85c \ub04c\ub824\uac00 \uc775\uc0ac\ud588\uace0, \ub098\uc911\uc5d0 \uc544\uc774\ub4e4\uc758 \ub0b4\uc7a5\ub9cc \ubc1c\uacac\ub41c\ub2e4. \ud37c\uc2a4\uc154\uc758 \uae00\ub80c \ucf08\ud2b8\ub2c8(Glen Keltney)\uc5d0 \uc0ac\ub294 \uc774 \uc9d0\uc2b9\uc744 20\uc138\uae30 \ubbfc\uc18d\ud559\uc790 \uce90\uc11c\ub9b0 \uba54\ub9ac \ube0c\ub9ad\uc2a4\ub294 \ucf08\ud53c\ub77c\uace0 \ud310\ub2e8\ud588\uc9c0\ub9cc, \uc5ed\uc2dc \ud37c\uc2a4\uc154\ub97c \ubc30\uacbd\uc73c\ub85c \ud558\ub294 \uc720\uc0ac\ud55c \uc774\uc57c\uae30\uc5d0\ub294 \uc5d0\ud750\uc73c\uc2dc\ucee4\uac00 \uc544\uc774\ub4e4\uc744 \uc7a1\uc544\uba39\uc740 \ubc94\uc778\uc774\uace0, \uc0b4\uc544\ub0a8\uc740 \uc0ac\ub0b4\uc544\uc774\uc5d0 \uad00\ud55c \uc774\uc57c\uae30\uc5d0 \uc724\uc0c9\uc774 \ub354\ud574\uc838 \uc788\ub2e4. \ucf08\ud53c\uac00 \ubc94\uc778\uc73c\ub85c \uc804\ud574\uc9c0\ub294 \uc11c\uc18c\uc758 \uc804\uc2b9\uc5d0\uc11c\ub294 \uc544\uc774\uac00 \uc190\uac00\ub77d\uc744 \uc798\ub77c\ub0c8\ub2e4\uace0 \ud55c\ub2e4. \ud558\uc77c\ub79c\ub4dc\uc758 \uc218\ub098\ub974\ud2b8\uc5d0\uc11c \uc804\uc2b9\ub418\ub294 \uac19\uc740 \uc774\uc57c\uae30\uc5d0\ub294 \uc2e4\uc885\ub41c \uc544\uc774\uac00 \uc544\ud649 \uba85\uc774\ub77c\uace0 \uad6c\uccb4\uc801\uc73c\ub85c \ubb18\uc0ac\ub418\uace0, \uadf8 \uc911 \ud55c \uba85\uc758 \ub0b4\uc7a5\ub9cc \ubc1c\uacac\ub418\uc5c8\ub2e4\uace0 \ud55c\ub2e4. \uc5ec\uae30\uc11c\ub3c4 \uc0b4\uc544\ub0a8\uc740 \uc544\uc774\ub294 \uc190\uac00\ub77d\uc744 \uc798\ub77c\ub0b4\uc11c \uc0b4\uc544\ub0ac\uace0, \uadf8 \uc544\uc774\uc758 \uc8fc\uba38\ub2c8\uc5d0 \uc131\uacbd\ucc45\uc774 \ub4e4\uc5b4 \uc788\uc5c8\ub2e4\ub294 \uad70\ub9d0\uc774 \ub367\ubd99\uc5b4 \uc788\ub2e4. \uadf8\ub808\uace0\ub974\uc2a8 \ucea0\ubca8\uc740 \uc774 \uc774\uc57c\uae30\uc758 \uc9d0\uc2b9\uc740 \ucf08\ud53c\ubcf4\ub2e4\ub294 \uc5d0\ud750\uc73c\uc2dc\ucee4\uc77c \uac83\uc774\ub77c\uace0 \uc0dd\uac01\ud558\uba70, \uc774 \uc774\uc57c\uae30\uac00 \u201c\uc544\uc774\ub4e4\uc774 \uc8fc\uc77c\uc5d0 \uc2f8\ub3cc\uc544\ub2e4\ub2c8\uc9c0 \ubabb\ud558\uac8c \ud558\ub824\ub294 \uacbd\uac74\ud55c \uac70\uc9d3\ub9d0\u201d\uc774\ub77c\uace0 \ud55c\ub2e4."} {"question": "\ubc15\uadfc\ud61c\ub294 2007\ub144 7\uc6d4 19\uc77c 5.16\uc744 \uaddc\uc815\ud55c \uce6d\ud638\ub294?", "paragraph": "\ud55c\uba85\uc219 \uc804 \ucd1d\ub9ac\ub294 \ubc15\uadfc\ud61c \uc804 \ub300\ud45c\uac00 \ud55c\ub098\ub77c\ub2f9 \ub300\uc120\ud6c4\ubcf4 \uac80\uc99d \uccad\ubb38\ud68c\uc5d0\uc11c 5.16\uc744 '\uad6c\uad6d\uc758 \ud601\uba85'\uc774\uc73c\ub85c \uaddc\uc815\ud55c \ub370 \ub300\ud574 '\ub9dd\uc5b8'\uc774\ub77c\uba70 \uac15\ud558\uac8c \ube44\ud310\ud588\ub2e4. \ud55c\uba85\uc219\uc740 2007\ub144 7\uc6d4 19\uc77c \"\uadf8\uac00 5.16 \ucfe0\ub370\ud0c0\ub97c '\uad6c\uad6d\uc758 \ud601\uba85'\uc774\ub77c\uace0 \ubbf8\ud654\ud55c \uac83\uc740 \uc815\uce58\uc801 \uc911\ub9bd\uc744 \uc9c0\ud0a4\uba70 \uad6d\ud1a0\ubc29\uc704\uc5d0 \uc804\ub150\ud558\uace0 \uc788\ub294 69\ub9cc \uad6d\uad70 \uc7a5\ubcd1\ub4e4\uc5d0 \ub300\ud55c \ubaa8\ub3c5\uc774\uc790 \uad70\uc0ac\ub3c5\uc7ac\uc5d0 \ub9de\uc11c \uc2f8\uc6b0\ub2e4 \ud76c\uc0dd\ub2f9\ud55c \ubbfc\uc8fc \uc601\ub839\ub4e4\uc744 \ub450 \ubc88 \uc8fd\uc774\ub294 \uc77c\"\uc774\ub77c\uace0 \uc8fc\uc7a5\ud558\uc600\ub2e4. \ud55c\uba85\uc219\uc740 \uc774\uc5b4 \"\uc544\uc9c1\ub3c4 \ubc15\uc815\ud76c \ucfe0\ub370\ud0c0 \uc815\uad8c\uc5d0 \uace0\ubb38\ub2f9\ud558\uace0 \uc0b4\ud574\ub2f9\ud55c \ubd84\ub4e4\uc758 \uc720\uac00\uc871\uc774 \uc11c\ub7ec\uc6c0\uacfc \ud55c\uc744 \uc548\uace0 \uc0b4\uc544\uac00\uace0 \uc788\ub2e4\"\uba70 \"\uace0(\u6545) \uc7a5\uc900\ud558 \uc120\uc0dd\uc758 \ubd80\uc778\uc744 \ub9cc\ub098 \uace0\uac1c\ub97c \uc870\uc544\ub838\ub358 \ub54c\uac00 \uc5b8\uc81c\ub77c\uace0 \ub610\ub2e4\uc2dc \ub3c5\uc7ac\uc758 \uc720\uc804\uc790\ub97c \ub4dc\ub7ec\ub0b4\ub0d0\"\uace0 \ube44\ub09c\ud558\uc600\ub2e4. \ud55c \uc804 \ucd1d\ub9ac\ub294 \uc790\uc2e0\uc740 \ucd1d\ub9ac \uc778\uc0ac\uccad\ubb38\ud68c\uc5d0\uc11c \uc720\uc2e0\uc2dc\uc808 \uc790\uc2e0\uc744 \uace0\ubb38\ud558\uace0 \ud22c\uc625\ud588\ub358 \uac00\ud574\uc790\ub4e4\uc744 \uc6a9\uc11c\ud588\ub2e4\uace0 \ub9d0\ud588\uc9c0\ub9cc \"\ubc15 \ud6c4\ubcf4\uc758 \ub9dd\uc5b8\uc740 \uc774\ucc98\ub7fc \uc720\uc2e0\ub3c5\uc7ac\uc758 \ud53c\ud574\uc790\ub4e4\uc774 \ub0b4\ubbf8\ub294 \uc6a9\uc11c\uc640 \ud654\ud574\uc758 \uc190\uc744 \ubfcc\ub9ac\uce58\uace0 \ud53c\ud574\uc790\ub4e4\uc758 \uac00\uc2b4\uc5d0 \ub2e4\uc2dc \ud55c \ubc88 \ubabb\uc744 \ubc15\ub294 \uc794\uc778\ud55c \uc9d3\"\uc774\ub77c\uace0 \uc5b8\ub860\uc5d0 \uadf8\ub97c \uacf5\uac1c\uc801\uc73c\ub85c \ube44\ub09c\ud558\uc600\ub2e4.", "answer": "\uad6c\uad6d\uc758 \ud601\uba85", "sentence": "\ud55c\uba85\uc219 \uc804 \ucd1d\ub9ac\ub294 \ubc15\uadfc\ud61c \uc804 \ub300\ud45c\uac00 \ud55c\ub098\ub77c\ub2f9 \ub300\uc120\ud6c4\ubcf4 \uac80\uc99d \uccad\ubb38\ud68c\uc5d0\uc11c 5.16\uc744 ' \uad6c\uad6d\uc758 \ud601\uba85 '\uc774\uc73c\ub85c \uaddc\uc815\ud55c \ub370 \ub300\ud574 '\ub9dd\uc5b8'\uc774\ub77c\uba70 \uac15\ud558\uac8c \ube44\ud310\ud588\ub2e4.", "paragraph_sentence": " \ud55c\uba85\uc219 \uc804 \ucd1d\ub9ac\ub294 \ubc15\uadfc\ud61c \uc804 \ub300\ud45c\uac00 \ud55c\ub098\ub77c\ub2f9 \ub300\uc120\ud6c4\ubcf4 \uac80\uc99d \uccad\ubb38\ud68c\uc5d0\uc11c 5.16\uc744 ' \uad6c\uad6d\uc758 \ud601\uba85 '\uc774\uc73c\ub85c \uaddc\uc815\ud55c \ub370 \ub300\ud574 '\ub9dd\uc5b8'\uc774\ub77c\uba70 \uac15\ud558\uac8c \ube44\ud310\ud588\ub2e4. \ud55c\uba85\uc219\uc740 2007\ub144 7\uc6d4 19\uc77c \"\uadf8\uac00 5.16 \ucfe0\ub370\ud0c0\ub97c '\uad6c\uad6d\uc758 \ud601\uba85'\uc774\ub77c\uace0 \ubbf8\ud654\ud55c \uac83\uc740 \uc815\uce58\uc801 \uc911\ub9bd\uc744 \uc9c0\ud0a4\uba70 \uad6d\ud1a0\ubc29\uc704\uc5d0 \uc804\ub150\ud558\uace0 \uc788\ub294 69\ub9cc \uad6d\uad70 \uc7a5\ubcd1\ub4e4\uc5d0 \ub300\ud55c \ubaa8\ub3c5\uc774\uc790 \uad70\uc0ac\ub3c5\uc7ac\uc5d0 \ub9de\uc11c \uc2f8\uc6b0\ub2e4 \ud76c\uc0dd\ub2f9\ud55c \ubbfc\uc8fc \uc601\ub839\ub4e4\uc744 \ub450 \ubc88 \uc8fd\uc774\ub294 \uc77c\"\uc774\ub77c\uace0 \uc8fc\uc7a5\ud558\uc600\ub2e4. \ud55c\uba85\uc219\uc740 \uc774\uc5b4 \"\uc544\uc9c1\ub3c4 \ubc15\uc815\ud76c \ucfe0\ub370\ud0c0 \uc815\uad8c\uc5d0 \uace0\ubb38\ub2f9\ud558\uace0 \uc0b4\ud574\ub2f9\ud55c \ubd84\ub4e4\uc758 \uc720\uac00\uc871\uc774 \uc11c\ub7ec\uc6c0\uacfc \ud55c\uc744 \uc548\uace0 \uc0b4\uc544\uac00\uace0 \uc788\ub2e4\"\uba70 \"\uace0(\u6545) \uc7a5\uc900\ud558 \uc120\uc0dd\uc758 \ubd80\uc778\uc744 \ub9cc\ub098 \uace0\uac1c\ub97c \uc870\uc544\ub838\ub358 \ub54c\uac00 \uc5b8\uc81c\ub77c\uace0 \ub610\ub2e4\uc2dc \ub3c5\uc7ac\uc758 \uc720\uc804\uc790\ub97c \ub4dc\ub7ec\ub0b4\ub0d0\"\uace0 \ube44\ub09c\ud558\uc600\ub2e4. \ud55c \uc804 \ucd1d\ub9ac\ub294 \uc790\uc2e0\uc740 \ucd1d\ub9ac \uc778\uc0ac\uccad\ubb38\ud68c\uc5d0\uc11c \uc720\uc2e0\uc2dc\uc808 \uc790\uc2e0\uc744 \uace0\ubb38\ud558\uace0 \ud22c\uc625\ud588\ub358 \uac00\ud574\uc790\ub4e4\uc744 \uc6a9\uc11c\ud588\ub2e4\uace0 \ub9d0\ud588\uc9c0\ub9cc \"\ubc15 \ud6c4\ubcf4\uc758 \ub9dd\uc5b8\uc740 \uc774\ucc98\ub7fc \uc720\uc2e0\ub3c5\uc7ac\uc758 \ud53c\ud574\uc790\ub4e4\uc774 \ub0b4\ubbf8\ub294 \uc6a9\uc11c\uc640 \ud654\ud574\uc758 \uc190\uc744 \ubfcc\ub9ac\uce58\uace0 \ud53c\ud574\uc790\ub4e4\uc758 \uac00\uc2b4\uc5d0 \ub2e4\uc2dc \ud55c \ubc88 \ubabb\uc744 \ubc15\ub294 \uc794\uc778\ud55c \uc9d3\"\uc774\ub77c\uace0 \uc5b8\ub860\uc5d0 \uadf8\ub97c \uacf5\uac1c\uc801\uc73c\ub85c \ube44\ub09c\ud558\uc600\ub2e4.", "paragraph_answer": "\ud55c\uba85\uc219 \uc804 \ucd1d\ub9ac\ub294 \ubc15\uadfc\ud61c \uc804 \ub300\ud45c\uac00 \ud55c\ub098\ub77c\ub2f9 \ub300\uc120\ud6c4\ubcf4 \uac80\uc99d \uccad\ubb38\ud68c\uc5d0\uc11c 5.16\uc744 ' \uad6c\uad6d\uc758 \ud601\uba85 '\uc774\uc73c\ub85c \uaddc\uc815\ud55c \ub370 \ub300\ud574 '\ub9dd\uc5b8'\uc774\ub77c\uba70 \uac15\ud558\uac8c \ube44\ud310\ud588\ub2e4. \ud55c\uba85\uc219\uc740 2007\ub144 7\uc6d4 19\uc77c \"\uadf8\uac00 5.16 \ucfe0\ub370\ud0c0\ub97c '\uad6c\uad6d\uc758 \ud601\uba85'\uc774\ub77c\uace0 \ubbf8\ud654\ud55c \uac83\uc740 \uc815\uce58\uc801 \uc911\ub9bd\uc744 \uc9c0\ud0a4\uba70 \uad6d\ud1a0\ubc29\uc704\uc5d0 \uc804\ub150\ud558\uace0 \uc788\ub294 69\ub9cc \uad6d\uad70 \uc7a5\ubcd1\ub4e4\uc5d0 \ub300\ud55c \ubaa8\ub3c5\uc774\uc790 \uad70\uc0ac\ub3c5\uc7ac\uc5d0 \ub9de\uc11c \uc2f8\uc6b0\ub2e4 \ud76c\uc0dd\ub2f9\ud55c \ubbfc\uc8fc \uc601\ub839\ub4e4\uc744 \ub450 \ubc88 \uc8fd\uc774\ub294 \uc77c\"\uc774\ub77c\uace0 \uc8fc\uc7a5\ud558\uc600\ub2e4. \ud55c\uba85\uc219\uc740 \uc774\uc5b4 \"\uc544\uc9c1\ub3c4 \ubc15\uc815\ud76c \ucfe0\ub370\ud0c0 \uc815\uad8c\uc5d0 \uace0\ubb38\ub2f9\ud558\uace0 \uc0b4\ud574\ub2f9\ud55c \ubd84\ub4e4\uc758 \uc720\uac00\uc871\uc774 \uc11c\ub7ec\uc6c0\uacfc \ud55c\uc744 \uc548\uace0 \uc0b4\uc544\uac00\uace0 \uc788\ub2e4\"\uba70 \"\uace0(\u6545) \uc7a5\uc900\ud558 \uc120\uc0dd\uc758 \ubd80\uc778\uc744 \ub9cc\ub098 \uace0\uac1c\ub97c \uc870\uc544\ub838\ub358 \ub54c\uac00 \uc5b8\uc81c\ub77c\uace0 \ub610\ub2e4\uc2dc \ub3c5\uc7ac\uc758 \uc720\uc804\uc790\ub97c \ub4dc\ub7ec\ub0b4\ub0d0\"\uace0 \ube44\ub09c\ud558\uc600\ub2e4. \ud55c \uc804 \ucd1d\ub9ac\ub294 \uc790\uc2e0\uc740 \ucd1d\ub9ac \uc778\uc0ac\uccad\ubb38\ud68c\uc5d0\uc11c \uc720\uc2e0\uc2dc\uc808 \uc790\uc2e0\uc744 \uace0\ubb38\ud558\uace0 \ud22c\uc625\ud588\ub358 \uac00\ud574\uc790\ub4e4\uc744 \uc6a9\uc11c\ud588\ub2e4\uace0 \ub9d0\ud588\uc9c0\ub9cc \"\ubc15 \ud6c4\ubcf4\uc758 \ub9dd\uc5b8\uc740 \uc774\ucc98\ub7fc \uc720\uc2e0\ub3c5\uc7ac\uc758 \ud53c\ud574\uc790\ub4e4\uc774 \ub0b4\ubbf8\ub294 \uc6a9\uc11c\uc640 \ud654\ud574\uc758 \uc190\uc744 \ubfcc\ub9ac\uce58\uace0 \ud53c\ud574\uc790\ub4e4\uc758 \uac00\uc2b4\uc5d0 \ub2e4\uc2dc \ud55c \ubc88 \ubabb\uc744 \ubc15\ub294 \uc794\uc778\ud55c \uc9d3\"\uc774\ub77c\uace0 \uc5b8\ub860\uc5d0 \uadf8\ub97c \uacf5\uac1c\uc801\uc73c\ub85c \ube44\ub09c\ud558\uc600\ub2e4.", "sentence_answer": "\ud55c\uba85\uc219 \uc804 \ucd1d\ub9ac\ub294 \ubc15\uadfc\ud61c \uc804 \ub300\ud45c\uac00 \ud55c\ub098\ub77c\ub2f9 \ub300\uc120\ud6c4\ubcf4 \uac80\uc99d \uccad\ubb38\ud68c\uc5d0\uc11c 5.16\uc744 ' \uad6c\uad6d\uc758 \ud601\uba85 '\uc774\uc73c\ub85c \uaddc\uc815\ud55c \ub370 \ub300\ud574 '\ub9dd\uc5b8'\uc774\ub77c\uba70 \uac15\ud558\uac8c \ube44\ud310\ud588\ub2e4.", "paragraph_id": "6515888-9-1", "paragraph_question": "question: \ubc15\uadfc\ud61c\ub294 2007\ub144 7\uc6d4 19\uc77c 5.16\uc744 \uaddc\uc815\ud55c \uce6d\ud638\ub294?, context: \ud55c\uba85\uc219 \uc804 \ucd1d\ub9ac\ub294 \ubc15\uadfc\ud61c \uc804 \ub300\ud45c\uac00 \ud55c\ub098\ub77c\ub2f9 \ub300\uc120\ud6c4\ubcf4 \uac80\uc99d \uccad\ubb38\ud68c\uc5d0\uc11c 5.16\uc744 '\uad6c\uad6d\uc758 \ud601\uba85'\uc774\uc73c\ub85c \uaddc\uc815\ud55c \ub370 \ub300\ud574 '\ub9dd\uc5b8'\uc774\ub77c\uba70 \uac15\ud558\uac8c \ube44\ud310\ud588\ub2e4. \ud55c\uba85\uc219\uc740 2007\ub144 7\uc6d4 19\uc77c \"\uadf8\uac00 5.16 \ucfe0\ub370\ud0c0\ub97c '\uad6c\uad6d\uc758 \ud601\uba85'\uc774\ub77c\uace0 \ubbf8\ud654\ud55c \uac83\uc740 \uc815\uce58\uc801 \uc911\ub9bd\uc744 \uc9c0\ud0a4\uba70 \uad6d\ud1a0\ubc29\uc704\uc5d0 \uc804\ub150\ud558\uace0 \uc788\ub294 69\ub9cc \uad6d\uad70 \uc7a5\ubcd1\ub4e4\uc5d0 \ub300\ud55c \ubaa8\ub3c5\uc774\uc790 \uad70\uc0ac\ub3c5\uc7ac\uc5d0 \ub9de\uc11c \uc2f8\uc6b0\ub2e4 \ud76c\uc0dd\ub2f9\ud55c \ubbfc\uc8fc \uc601\ub839\ub4e4\uc744 \ub450 \ubc88 \uc8fd\uc774\ub294 \uc77c\"\uc774\ub77c\uace0 \uc8fc\uc7a5\ud558\uc600\ub2e4. \ud55c\uba85\uc219\uc740 \uc774\uc5b4 \"\uc544\uc9c1\ub3c4 \ubc15\uc815\ud76c \ucfe0\ub370\ud0c0 \uc815\uad8c\uc5d0 \uace0\ubb38\ub2f9\ud558\uace0 \uc0b4\ud574\ub2f9\ud55c \ubd84\ub4e4\uc758 \uc720\uac00\uc871\uc774 \uc11c\ub7ec\uc6c0\uacfc \ud55c\uc744 \uc548\uace0 \uc0b4\uc544\uac00\uace0 \uc788\ub2e4\"\uba70 \"\uace0(\u6545) \uc7a5\uc900\ud558 \uc120\uc0dd\uc758 \ubd80\uc778\uc744 \ub9cc\ub098 \uace0\uac1c\ub97c \uc870\uc544\ub838\ub358 \ub54c\uac00 \uc5b8\uc81c\ub77c\uace0 \ub610\ub2e4\uc2dc \ub3c5\uc7ac\uc758 \uc720\uc804\uc790\ub97c \ub4dc\ub7ec\ub0b4\ub0d0\"\uace0 \ube44\ub09c\ud558\uc600\ub2e4. \ud55c \uc804 \ucd1d\ub9ac\ub294 \uc790\uc2e0\uc740 \ucd1d\ub9ac \uc778\uc0ac\uccad\ubb38\ud68c\uc5d0\uc11c \uc720\uc2e0\uc2dc\uc808 \uc790\uc2e0\uc744 \uace0\ubb38\ud558\uace0 \ud22c\uc625\ud588\ub358 \uac00\ud574\uc790\ub4e4\uc744 \uc6a9\uc11c\ud588\ub2e4\uace0 \ub9d0\ud588\uc9c0\ub9cc \"\ubc15 \ud6c4\ubcf4\uc758 \ub9dd\uc5b8\uc740 \uc774\ucc98\ub7fc \uc720\uc2e0\ub3c5\uc7ac\uc758 \ud53c\ud574\uc790\ub4e4\uc774 \ub0b4\ubbf8\ub294 \uc6a9\uc11c\uc640 \ud654\ud574\uc758 \uc190\uc744 \ubfcc\ub9ac\uce58\uace0 \ud53c\ud574\uc790\ub4e4\uc758 \uac00\uc2b4\uc5d0 \ub2e4\uc2dc \ud55c \ubc88 \ubabb\uc744 \ubc15\ub294 \uc794\uc778\ud55c \uc9d3\"\uc774\ub77c\uace0 \uc5b8\ub860\uc5d0 \uadf8\ub97c \uacf5\uac1c\uc801\uc73c\ub85c \ube44\ub09c\ud558\uc600\ub2e4."} {"question": "\ubc1c\uacac \ub2f9\uc77c, \uc138\ub4dc\ub098\uac00 \uc774\ub3d9\ud588\ub2e4\uace0 \ucd94\uce21\ub418\ub294 \uac70\ub9ac\ub294?", "paragraph": "\uc138\ub4dc\ub098(\uc784\uc2dc \uc9c0\uc815\ubc88\ud638 2003 VB12)\ub294 2003\ub144 11\uc6d4 14\uc77c \ub9c8\uc774\ud074 E. \ube0c\ub77c\uc6b4(\uce7c\ud14d), \ucc44\ub4dc \ud2b8\ub8e8\uc9c8\ub9ac\uc624(\uc81c\ubbf8\ub2c8), \ub370\uc774\ube44\ub4dc L. \ub77c\ube44\ub178\ube44\uce20(\uc608\uc77c)\uac00 \ubc1c\uacac\ud588\ub2e4. \uc138\ub4dc\ub098 \ubc1c\uacac\uc740 2001\ub144\ubd80\ud130 \uce98\ub9ac\ud3ec\ub2c8\uc544 \uc8fc \uc0cc\ub514\uc5d0\uc774\uace0 \uadfc\uad50\uc758 \ud314\ub85c\ub9c8 \ucc9c\ubb38\ub300\uc758 \uc0c8\ubba4\uc5bc \uc624\uc2a4\uce5c \ub9dd\uc6d0\uacbd\uacfc \uc608\uc77c\uc758 160 \uba54\uac00\ud53d\uc140 \ud314\ub85c\ub9c8 \ud018\uc2a4\ud2b8 \uce74\uba54\ub77c\ub97c \uc0ac\uc6a9\ud574 \uc218\ud589\ud558\ub358 \uc804\ucc9c\ud0d0\uc0ac\uc758 \uc77c\ud658\uc774\uc5c8\ub2e4. \ubc1c\uacac \ub2f9\uc77c \uc138\ub4dc\ub098\ub294 \ubc30\uacbd \ud56d\uc131\uc5d0 \ub300\ud558\uc5ec 3.1 \uc2dc\uac04\ub9c8\ub2e4 4.6 \uac01\ubd84\uc529 \uc6c0\uc9c1\uc600\uc73c\uba70, \uc5ec\uae30\uc11c \ucd94\uce21\ud560 \uc218 \uc788\ub294 \uac70\ub9ac\ub294 \uc57d 100 AU\uc600\ub2e4. \uc774\ud6c4 2003\ub144 11\uc6d4\uc5d0\uc11c 12\uc6d4 \uc0ac\uc774\uc5d0 \uce60\ub808\uc758 \ucf00\ub85c \ud1a8\ub864\ub85c \ubbf8\uc8fc\uac04 \ucc9c\ubb38\ub300\uc758 SMARTS \ub9dd\uc6d0\uacbd\uacfc \ud558\uc640\uc774 \ub9c8\uc6b0\ub098\ucf00\uc544 \ud654\uc0b0\uc758 \ucf01 \ucc9c\ubb38\ub300\uc758 \ud14c\ub098\uadf8\ub77c \uc81c4\ub9dd\uc6d0\uacbd\uc744 \uc774\uc6a9\ud55c \ucd94\uac00 \uad00\uce21\uc774 \uc774\ub8e8\uc5b4\uc84c\uace0, \uc774 \ucc9c\uccb4\uac00 \uc774\uc2ec\ub960\uc774 \ub9e4\uc6b0 \ud070 \uada4\ub3c4\ub97c \ub530\ub77c \uc6c0\uc9c1\uc778\ub2e4\ub294 \uac83\uc774 \ubc1d\ud600\uc84c\ub2e4. \uc774\ud6c4 \uc0c8\ubba4\uc5bc \uc624\uc2a4\uce5c \ub9dd\uc6d0\uacbd\uacfc \uadfc\uc9c0\uad6c \uc18c\ud589\uc131 \ucd94\uc801 \ud611\ub825\ub2e8\uc774 \uc608\uc804\uc5d0 \ucd2c\uc601\ud55c \uc0ac\uc9c4\ub4e4\uc5d0\uc11c \uc138\ub4dc\ub098\uac00 \uc7ac\ubc1c\uacac\ub418\uc5c8\ub2e4. \uacfc\uac70\uc758 \uc815\ud655\ud55c \uc704\uce58\uac00 \ubc1d\ud600\uc9c0\uc790 \uc138\ub4dc\ub098\uc758 \uacf5\uc804\uada4\ub3c4\uc0c1\uc758 \ubd80\ucc44\uaf34\uc744 \uadf8\ub9b4 \uc218 \uc788\uc5c8\uace0 \uc774\ub97c \ud1b5\ud574 \uada4\ub3c4\ub97c \ubcf4\ub2e4 \uc815\ubc00\ud558\uac8c \uacc4\uc0b0\ud574\ub0c8\ub2e4.", "answer": "\uc57d 100 AU", "sentence": "\uc5ec\uae30\uc11c \ucd94\uce21\ud560 \uc218 \uc788\ub294 \uac70\ub9ac\ub294 \uc57d 100 AU \uc600\ub2e4.", "paragraph_sentence": "\uc138\ub4dc\ub098(\uc784\uc2dc \uc9c0\uc815\ubc88\ud638 2003 VB12)\ub294 2003\ub144 11\uc6d4 14\uc77c \ub9c8\uc774\ud074 E. \ube0c\ub77c\uc6b4(\uce7c\ud14d), \ucc44\ub4dc \ud2b8\ub8e8\uc9c8\ub9ac\uc624(\uc81c\ubbf8\ub2c8), \ub370\uc774\ube44\ub4dc L. \ub77c\ube44\ub178\ube44\uce20(\uc608\uc77c)\uac00 \ubc1c\uacac\ud588\ub2e4. \uc138\ub4dc\ub098 \ubc1c\uacac\uc740 2001\ub144\ubd80\ud130 \uce98\ub9ac\ud3ec\ub2c8\uc544 \uc8fc \uc0cc\ub514\uc5d0\uc774\uace0 \uadfc\uad50\uc758 \ud314\ub85c\ub9c8 \ucc9c\ubb38\ub300\uc758 \uc0c8\ubba4\uc5bc \uc624\uc2a4\uce5c \ub9dd\uc6d0\uacbd\uacfc \uc608\uc77c\uc758 160 \uba54\uac00\ud53d\uc140 \ud314\ub85c\ub9c8 \ud018\uc2a4\ud2b8 \uce74\uba54\ub77c\ub97c \uc0ac\uc6a9\ud574 \uc218\ud589\ud558\ub358 \uc804\ucc9c\ud0d0\uc0ac\uc758 \uc77c\ud658\uc774\uc5c8\ub2e4. \ubc1c\uacac \ub2f9\uc77c \uc138\ub4dc\ub098\ub294 \ubc30\uacbd \ud56d\uc131\uc5d0 \ub300\ud558\uc5ec 3.1 \uc2dc\uac04\ub9c8\ub2e4 4.6 \uac01\ubd84\uc529 \uc6c0\uc9c1\uc600\uc73c\uba70, \uc5ec\uae30\uc11c \ucd94\uce21\ud560 \uc218 \uc788\ub294 \uac70\ub9ac\ub294 \uc57d 100 AU \uc600\ub2e4. \uc774\ud6c4 2003\ub144 11\uc6d4\uc5d0\uc11c 12\uc6d4 \uc0ac\uc774\uc5d0 \uce60\ub808\uc758 \ucf00\ub85c \ud1a8\ub864\ub85c \ubbf8\uc8fc\uac04 \ucc9c\ubb38\ub300\uc758 SMARTS \ub9dd\uc6d0\uacbd\uacfc \ud558\uc640\uc774 \ub9c8\uc6b0\ub098\ucf00\uc544 \ud654\uc0b0\uc758 \ucf01 \ucc9c\ubb38\ub300\uc758 \ud14c\ub098\uadf8\ub77c \uc81c4\ub9dd\uc6d0\uacbd\uc744 \uc774\uc6a9\ud55c \ucd94\uac00 \uad00\uce21\uc774 \uc774\ub8e8\uc5b4\uc84c\uace0, \uc774 \ucc9c\uccb4\uac00 \uc774\uc2ec\ub960\uc774 \ub9e4\uc6b0 \ud070 \uada4\ub3c4\ub97c \ub530\ub77c \uc6c0\uc9c1\uc778\ub2e4\ub294 \uac83\uc774 \ubc1d\ud600\uc84c\ub2e4. \uc774\ud6c4 \uc0c8\ubba4\uc5bc \uc624\uc2a4\uce5c \ub9dd\uc6d0\uacbd\uacfc \uadfc\uc9c0\uad6c \uc18c\ud589\uc131 \ucd94\uc801 \ud611\ub825\ub2e8\uc774 \uc608\uc804\uc5d0 \ucd2c\uc601\ud55c \uc0ac\uc9c4\ub4e4\uc5d0\uc11c \uc138\ub4dc\ub098\uac00 \uc7ac\ubc1c\uacac\ub418\uc5c8\ub2e4. \uacfc\uac70\uc758 \uc815\ud655\ud55c \uc704\uce58\uac00 \ubc1d\ud600\uc9c0\uc790 \uc138\ub4dc\ub098\uc758 \uacf5\uc804\uada4\ub3c4\uc0c1\uc758 \ubd80\ucc44\uaf34\uc744 \uadf8\ub9b4 \uc218 \uc788\uc5c8\uace0 \uc774\ub97c \ud1b5\ud574 \uada4\ub3c4\ub97c \ubcf4\ub2e4 \uc815\ubc00\ud558\uac8c \uacc4\uc0b0\ud574\ub0c8\ub2e4.", "paragraph_answer": "\uc138\ub4dc\ub098(\uc784\uc2dc \uc9c0\uc815\ubc88\ud638 2003 VB12)\ub294 2003\ub144 11\uc6d4 14\uc77c \ub9c8\uc774\ud074 E. \ube0c\ub77c\uc6b4(\uce7c\ud14d), \ucc44\ub4dc \ud2b8\ub8e8\uc9c8\ub9ac\uc624(\uc81c\ubbf8\ub2c8), \ub370\uc774\ube44\ub4dc L. \ub77c\ube44\ub178\ube44\uce20(\uc608\uc77c)\uac00 \ubc1c\uacac\ud588\ub2e4. \uc138\ub4dc\ub098 \ubc1c\uacac\uc740 2001\ub144\ubd80\ud130 \uce98\ub9ac\ud3ec\ub2c8\uc544 \uc8fc \uc0cc\ub514\uc5d0\uc774\uace0 \uadfc\uad50\uc758 \ud314\ub85c\ub9c8 \ucc9c\ubb38\ub300\uc758 \uc0c8\ubba4\uc5bc \uc624\uc2a4\uce5c \ub9dd\uc6d0\uacbd\uacfc \uc608\uc77c\uc758 160 \uba54\uac00\ud53d\uc140 \ud314\ub85c\ub9c8 \ud018\uc2a4\ud2b8 \uce74\uba54\ub77c\ub97c \uc0ac\uc6a9\ud574 \uc218\ud589\ud558\ub358 \uc804\ucc9c\ud0d0\uc0ac\uc758 \uc77c\ud658\uc774\uc5c8\ub2e4. \ubc1c\uacac \ub2f9\uc77c \uc138\ub4dc\ub098\ub294 \ubc30\uacbd \ud56d\uc131\uc5d0 \ub300\ud558\uc5ec 3.1 \uc2dc\uac04\ub9c8\ub2e4 4.6 \uac01\ubd84\uc529 \uc6c0\uc9c1\uc600\uc73c\uba70, \uc5ec\uae30\uc11c \ucd94\uce21\ud560 \uc218 \uc788\ub294 \uac70\ub9ac\ub294 \uc57d 100 AU \uc600\ub2e4. \uc774\ud6c4 2003\ub144 11\uc6d4\uc5d0\uc11c 12\uc6d4 \uc0ac\uc774\uc5d0 \uce60\ub808\uc758 \ucf00\ub85c \ud1a8\ub864\ub85c \ubbf8\uc8fc\uac04 \ucc9c\ubb38\ub300\uc758 SMARTS \ub9dd\uc6d0\uacbd\uacfc \ud558\uc640\uc774 \ub9c8\uc6b0\ub098\ucf00\uc544 \ud654\uc0b0\uc758 \ucf01 \ucc9c\ubb38\ub300\uc758 \ud14c\ub098\uadf8\ub77c \uc81c4\ub9dd\uc6d0\uacbd\uc744 \uc774\uc6a9\ud55c \ucd94\uac00 \uad00\uce21\uc774 \uc774\ub8e8\uc5b4\uc84c\uace0, \uc774 \ucc9c\uccb4\uac00 \uc774\uc2ec\ub960\uc774 \ub9e4\uc6b0 \ud070 \uada4\ub3c4\ub97c \ub530\ub77c \uc6c0\uc9c1\uc778\ub2e4\ub294 \uac83\uc774 \ubc1d\ud600\uc84c\ub2e4. \uc774\ud6c4 \uc0c8\ubba4\uc5bc \uc624\uc2a4\uce5c \ub9dd\uc6d0\uacbd\uacfc \uadfc\uc9c0\uad6c \uc18c\ud589\uc131 \ucd94\uc801 \ud611\ub825\ub2e8\uc774 \uc608\uc804\uc5d0 \ucd2c\uc601\ud55c \uc0ac\uc9c4\ub4e4\uc5d0\uc11c \uc138\ub4dc\ub098\uac00 \uc7ac\ubc1c\uacac\ub418\uc5c8\ub2e4. \uacfc\uac70\uc758 \uc815\ud655\ud55c \uc704\uce58\uac00 \ubc1d\ud600\uc9c0\uc790 \uc138\ub4dc\ub098\uc758 \uacf5\uc804\uada4\ub3c4\uc0c1\uc758 \ubd80\ucc44\uaf34\uc744 \uadf8\ub9b4 \uc218 \uc788\uc5c8\uace0 \uc774\ub97c \ud1b5\ud574 \uada4\ub3c4\ub97c \ubcf4\ub2e4 \uc815\ubc00\ud558\uac8c \uacc4\uc0b0\ud574\ub0c8\ub2e4.", "sentence_answer": "\uc5ec\uae30\uc11c \ucd94\uce21\ud560 \uc218 \uc788\ub294 \uac70\ub9ac\ub294 \uc57d 100 AU \uc600\ub2e4.", "paragraph_id": "6470281-1-1", "paragraph_question": "question: \ubc1c\uacac \ub2f9\uc77c, \uc138\ub4dc\ub098\uac00 \uc774\ub3d9\ud588\ub2e4\uace0 \ucd94\uce21\ub418\ub294 \uac70\ub9ac\ub294?, context: \uc138\ub4dc\ub098(\uc784\uc2dc \uc9c0\uc815\ubc88\ud638 2003 VB12)\ub294 2003\ub144 11\uc6d4 14\uc77c \ub9c8\uc774\ud074 E. \ube0c\ub77c\uc6b4(\uce7c\ud14d), \ucc44\ub4dc \ud2b8\ub8e8\uc9c8\ub9ac\uc624(\uc81c\ubbf8\ub2c8), \ub370\uc774\ube44\ub4dc L. \ub77c\ube44\ub178\ube44\uce20(\uc608\uc77c)\uac00 \ubc1c\uacac\ud588\ub2e4. \uc138\ub4dc\ub098 \ubc1c\uacac\uc740 2001\ub144\ubd80\ud130 \uce98\ub9ac\ud3ec\ub2c8\uc544 \uc8fc \uc0cc\ub514\uc5d0\uc774\uace0 \uadfc\uad50\uc758 \ud314\ub85c\ub9c8 \ucc9c\ubb38\ub300\uc758 \uc0c8\ubba4\uc5bc \uc624\uc2a4\uce5c \ub9dd\uc6d0\uacbd\uacfc \uc608\uc77c\uc758 160 \uba54\uac00\ud53d\uc140 \ud314\ub85c\ub9c8 \ud018\uc2a4\ud2b8 \uce74\uba54\ub77c\ub97c \uc0ac\uc6a9\ud574 \uc218\ud589\ud558\ub358 \uc804\ucc9c\ud0d0\uc0ac\uc758 \uc77c\ud658\uc774\uc5c8\ub2e4. \ubc1c\uacac \ub2f9\uc77c \uc138\ub4dc\ub098\ub294 \ubc30\uacbd \ud56d\uc131\uc5d0 \ub300\ud558\uc5ec 3.1 \uc2dc\uac04\ub9c8\ub2e4 4.6 \uac01\ubd84\uc529 \uc6c0\uc9c1\uc600\uc73c\uba70, \uc5ec\uae30\uc11c \ucd94\uce21\ud560 \uc218 \uc788\ub294 \uac70\ub9ac\ub294 \uc57d 100 AU\uc600\ub2e4. \uc774\ud6c4 2003\ub144 11\uc6d4\uc5d0\uc11c 12\uc6d4 \uc0ac\uc774\uc5d0 \uce60\ub808\uc758 \ucf00\ub85c \ud1a8\ub864\ub85c \ubbf8\uc8fc\uac04 \ucc9c\ubb38\ub300\uc758 SMARTS \ub9dd\uc6d0\uacbd\uacfc \ud558\uc640\uc774 \ub9c8\uc6b0\ub098\ucf00\uc544 \ud654\uc0b0\uc758 \ucf01 \ucc9c\ubb38\ub300\uc758 \ud14c\ub098\uadf8\ub77c \uc81c4\ub9dd\uc6d0\uacbd\uc744 \uc774\uc6a9\ud55c \ucd94\uac00 \uad00\uce21\uc774 \uc774\ub8e8\uc5b4\uc84c\uace0, \uc774 \ucc9c\uccb4\uac00 \uc774\uc2ec\ub960\uc774 \ub9e4\uc6b0 \ud070 \uada4\ub3c4\ub97c \ub530\ub77c \uc6c0\uc9c1\uc778\ub2e4\ub294 \uac83\uc774 \ubc1d\ud600\uc84c\ub2e4. \uc774\ud6c4 \uc0c8\ubba4\uc5bc \uc624\uc2a4\uce5c \ub9dd\uc6d0\uacbd\uacfc \uadfc\uc9c0\uad6c \uc18c\ud589\uc131 \ucd94\uc801 \ud611\ub825\ub2e8\uc774 \uc608\uc804\uc5d0 \ucd2c\uc601\ud55c \uc0ac\uc9c4\ub4e4\uc5d0\uc11c \uc138\ub4dc\ub098\uac00 \uc7ac\ubc1c\uacac\ub418\uc5c8\ub2e4. \uacfc\uac70\uc758 \uc815\ud655\ud55c \uc704\uce58\uac00 \ubc1d\ud600\uc9c0\uc790 \uc138\ub4dc\ub098\uc758 \uacf5\uc804\uada4\ub3c4\uc0c1\uc758 \ubd80\ucc44\uaf34\uc744 \uadf8\ub9b4 \uc218 \uc788\uc5c8\uace0 \uc774\ub97c \ud1b5\ud574 \uada4\ub3c4\ub97c \ubcf4\ub2e4 \uc815\ubc00\ud558\uac8c \uacc4\uc0b0\ud574\ub0c8\ub2e4."} {"question": "\ud328\ud1f4\ud558\uc5ec \ub378\ub9ac\ub85c \ucca0\uc218\ud55c \uc138\ud3ec\uc774 \ubc18\uad70\uc740 \ub204\uac00 \uc9c0\ud718\ud588\uc5c8\ub294\uac00?", "paragraph": "\uc138\ud3ec\uc774 \ud56d\uc7c1 \ubc1c\uc0dd \ucd08\uae30 \uc601\uad6d\uc758 \ub300\uc751\uc740 \ub290\ub838\ub2e4. \uc601\uad6d\uc740 \ud06c\ub9bc \uc804\uc7c1\uc5d0 \ud22c\uc785\ub418\uc5c8\ub358 \ubcd1\ub825\uc744 \uc774\ub780\uc744 \ud1b5\ud574 \uc778\ub3c4\ub85c \uc774\ub3d9\uc2dc\ucf30\ub2e4. \ub610\ud55c \uc911\uad6d\uc5d0 \uc8fc\ub454\ud558\uc600\ub358 \uad70\ub300\uc758 \uc77c\ubd80\ub3c4 \uc778\ub3c4\ub85c \ud22c\uc785\ub418\uc5c8\ub2e4. \uc804\ube44\ub97c \uac16\ucd98 \uc601\uad6d \uce21 \uad70\ub300\ub294 \ubbf8\ub8e8\ud2b8\uc640 \ud788\ub9c8\ucc30\ud504\ub77c\ub370\uc2dc \uc8fc\uc758 \uc8fc\ub3c4\uc778 \uc2ec\ub77c\ub85c \uc9d1\uacb0\ud55c \ud6c4 \ub378\ub9ac\ub85c \ud5a5\ud588\ub2e4. \ud56d\uc7c1 \ubc1c\uc0dd \uc774\ud6c4 2\uac1c\uc6d4\uc774 \uc9c0\ub09c \ub4a4 \ub450 \uad70\ub300\ub294 \ucfe0\ub974\uce74\ub97c \ud3ec\ud568\ud55c \uc5f0\ud569 \ubd80\ub300\ub97c \ud615\uc131\ud558\uc5ec \uc9c4\uad70\ud558\uc600\uace0, \uce74\ub974\ub0a0\uc5d0\uc11c \uc138\ud3ec\uc774 \ubc18\uad70\uacfc \uad50\uc804\ud558\uc600\ub2e4. \uc774 \uc804\ud22c\uc5d0\uc11c \ubc14\ub4e4\ub9ac \ucf00 \uc138\ub77c\uc774\uac00 \uc9c0\ud718\ud558\ub358 \uc138\ud3ec\uc774 \ubc18\uad70\uc740 \ud328\ud1f4\ud558\uc5ec \ub378\ub9ac\ub85c \ucca0\uc218\ud558\uc600\ub2e4. \ubcf8\uad6d\uc5d0\uc11c\ub3c4, \ucd5c\uc2e0\ubb34\uae30\ub97c \uac16\ucd98 2\uac1c \uc5f0\ub300\uc758 5,000\uba85\uc774 \ud574\ub85c\ub97c \ud1b5\ud574 \uc9c0\uc6d0\ub418\uc5c8\ub2e4. \uc774 \ubc16\uc5d0\ub3c4, \uc2dc\ud06c\uc871, \ud398\ub974\uc2dc\uc544\ub4f1 \uc5ec\ub7ec \uc774\ubbfc\uc871\uc758 \uc9c0\uc6d0\uad70\uc774 \uc18d\uc18d \ub3c4\ucc29\ud558\uc600\ub2e4.", "answer": "\ubc14\ub4e4\ub9ac \ucf00 \uc138\ub77c\uc774", "sentence": "\uc774 \uc804\ud22c\uc5d0\uc11c \ubc14\ub4e4\ub9ac \ucf00 \uc138\ub77c\uc774 \uac00 \uc9c0\ud718\ud558\ub358 \uc138\ud3ec\uc774 \ubc18\uad70\uc740 \ud328\ud1f4\ud558\uc5ec \ub378\ub9ac\ub85c \ucca0\uc218\ud558\uc600\ub2e4.", "paragraph_sentence": "\uc138\ud3ec\uc774 \ud56d\uc7c1 \ubc1c\uc0dd \ucd08\uae30 \uc601\uad6d\uc758 \ub300\uc751\uc740 \ub290\ub838\ub2e4. \uc601\uad6d\uc740 \ud06c\ub9bc \uc804\uc7c1\uc5d0 \ud22c\uc785\ub418\uc5c8\ub358 \ubcd1\ub825\uc744 \uc774\ub780\uc744 \ud1b5\ud574 \uc778\ub3c4\ub85c \uc774\ub3d9\uc2dc\ucf30\ub2e4. \ub610\ud55c \uc911\uad6d\uc5d0 \uc8fc\ub454\ud558\uc600\ub358 \uad70\ub300\uc758 \uc77c\ubd80\ub3c4 \uc778\ub3c4\ub85c \ud22c\uc785\ub418\uc5c8\ub2e4. \uc804\ube44\ub97c \uac16\ucd98 \uc601\uad6d \uce21 \uad70\ub300\ub294 \ubbf8\ub8e8\ud2b8\uc640 \ud788\ub9c8\ucc30\ud504\ub77c\ub370\uc2dc \uc8fc\uc758 \uc8fc\ub3c4\uc778 \uc2ec\ub77c\ub85c \uc9d1\uacb0\ud55c \ud6c4 \ub378\ub9ac\ub85c \ud5a5\ud588\ub2e4. \ud56d\uc7c1 \ubc1c\uc0dd \uc774\ud6c4 2\uac1c\uc6d4\uc774 \uc9c0\ub09c \ub4a4 \ub450 \uad70\ub300\ub294 \ucfe0\ub974\uce74\ub97c \ud3ec\ud568\ud55c \uc5f0\ud569 \ubd80\ub300\ub97c \ud615\uc131\ud558\uc5ec \uc9c4\uad70\ud558\uc600\uace0, \uce74\ub974\ub0a0\uc5d0\uc11c \uc138\ud3ec\uc774 \ubc18\uad70\uacfc \uad50\uc804\ud558\uc600\ub2e4. \uc774 \uc804\ud22c\uc5d0\uc11c \ubc14\ub4e4\ub9ac \ucf00 \uc138\ub77c\uc774 \uac00 \uc9c0\ud718\ud558\ub358 \uc138\ud3ec\uc774 \ubc18\uad70\uc740 \ud328\ud1f4\ud558\uc5ec \ub378\ub9ac\ub85c \ucca0\uc218\ud558\uc600\ub2e4. \ubcf8\uad6d\uc5d0\uc11c\ub3c4, \ucd5c\uc2e0\ubb34\uae30\ub97c \uac16\ucd98 2\uac1c \uc5f0\ub300\uc758 5,000\uba85\uc774 \ud574\ub85c\ub97c \ud1b5\ud574 \uc9c0\uc6d0\ub418\uc5c8\ub2e4. \uc774 \ubc16\uc5d0\ub3c4, \uc2dc\ud06c\uc871, \ud398\ub974\uc2dc\uc544\ub4f1 \uc5ec\ub7ec \uc774\ubbfc\uc871\uc758 \uc9c0\uc6d0\uad70\uc774 \uc18d\uc18d \ub3c4\ucc29\ud558\uc600\ub2e4.", "paragraph_answer": "\uc138\ud3ec\uc774 \ud56d\uc7c1 \ubc1c\uc0dd \ucd08\uae30 \uc601\uad6d\uc758 \ub300\uc751\uc740 \ub290\ub838\ub2e4. \uc601\uad6d\uc740 \ud06c\ub9bc \uc804\uc7c1\uc5d0 \ud22c\uc785\ub418\uc5c8\ub358 \ubcd1\ub825\uc744 \uc774\ub780\uc744 \ud1b5\ud574 \uc778\ub3c4\ub85c \uc774\ub3d9\uc2dc\ucf30\ub2e4. \ub610\ud55c \uc911\uad6d\uc5d0 \uc8fc\ub454\ud558\uc600\ub358 \uad70\ub300\uc758 \uc77c\ubd80\ub3c4 \uc778\ub3c4\ub85c \ud22c\uc785\ub418\uc5c8\ub2e4. \uc804\ube44\ub97c \uac16\ucd98 \uc601\uad6d \uce21 \uad70\ub300\ub294 \ubbf8\ub8e8\ud2b8\uc640 \ud788\ub9c8\ucc30\ud504\ub77c\ub370\uc2dc \uc8fc\uc758 \uc8fc\ub3c4\uc778 \uc2ec\ub77c\ub85c \uc9d1\uacb0\ud55c \ud6c4 \ub378\ub9ac\ub85c \ud5a5\ud588\ub2e4. \ud56d\uc7c1 \ubc1c\uc0dd \uc774\ud6c4 2\uac1c\uc6d4\uc774 \uc9c0\ub09c \ub4a4 \ub450 \uad70\ub300\ub294 \ucfe0\ub974\uce74\ub97c \ud3ec\ud568\ud55c \uc5f0\ud569 \ubd80\ub300\ub97c \ud615\uc131\ud558\uc5ec \uc9c4\uad70\ud558\uc600\uace0, \uce74\ub974\ub0a0\uc5d0\uc11c \uc138\ud3ec\uc774 \ubc18\uad70\uacfc \uad50\uc804\ud558\uc600\ub2e4. \uc774 \uc804\ud22c\uc5d0\uc11c \ubc14\ub4e4\ub9ac \ucf00 \uc138\ub77c\uc774 \uac00 \uc9c0\ud718\ud558\ub358 \uc138\ud3ec\uc774 \ubc18\uad70\uc740 \ud328\ud1f4\ud558\uc5ec \ub378\ub9ac\ub85c \ucca0\uc218\ud558\uc600\ub2e4. \ubcf8\uad6d\uc5d0\uc11c\ub3c4, \ucd5c\uc2e0\ubb34\uae30\ub97c \uac16\ucd98 2\uac1c \uc5f0\ub300\uc758 5,000\uba85\uc774 \ud574\ub85c\ub97c \ud1b5\ud574 \uc9c0\uc6d0\ub418\uc5c8\ub2e4. \uc774 \ubc16\uc5d0\ub3c4, \uc2dc\ud06c\uc871, \ud398\ub974\uc2dc\uc544\ub4f1 \uc5ec\ub7ec \uc774\ubbfc\uc871\uc758 \uc9c0\uc6d0\uad70\uc774 \uc18d\uc18d \ub3c4\ucc29\ud558\uc600\ub2e4.", "sentence_answer": "\uc774 \uc804\ud22c\uc5d0\uc11c \ubc14\ub4e4\ub9ac \ucf00 \uc138\ub77c\uc774 \uac00 \uc9c0\ud718\ud558\ub358 \uc138\ud3ec\uc774 \ubc18\uad70\uc740 \ud328\ud1f4\ud558\uc5ec \ub378\ub9ac\ub85c \ucca0\uc218\ud558\uc600\ub2e4.", "paragraph_id": "6532252-7-2", "paragraph_question": "question: \ud328\ud1f4\ud558\uc5ec \ub378\ub9ac\ub85c \ucca0\uc218\ud55c \uc138\ud3ec\uc774 \ubc18\uad70\uc740 \ub204\uac00 \uc9c0\ud718\ud588\uc5c8\ub294\uac00?, context: \uc138\ud3ec\uc774 \ud56d\uc7c1 \ubc1c\uc0dd \ucd08\uae30 \uc601\uad6d\uc758 \ub300\uc751\uc740 \ub290\ub838\ub2e4. \uc601\uad6d\uc740 \ud06c\ub9bc \uc804\uc7c1\uc5d0 \ud22c\uc785\ub418\uc5c8\ub358 \ubcd1\ub825\uc744 \uc774\ub780\uc744 \ud1b5\ud574 \uc778\ub3c4\ub85c \uc774\ub3d9\uc2dc\ucf30\ub2e4. \ub610\ud55c \uc911\uad6d\uc5d0 \uc8fc\ub454\ud558\uc600\ub358 \uad70\ub300\uc758 \uc77c\ubd80\ub3c4 \uc778\ub3c4\ub85c \ud22c\uc785\ub418\uc5c8\ub2e4. \uc804\ube44\ub97c \uac16\ucd98 \uc601\uad6d \uce21 \uad70\ub300\ub294 \ubbf8\ub8e8\ud2b8\uc640 \ud788\ub9c8\ucc30\ud504\ub77c\ub370\uc2dc \uc8fc\uc758 \uc8fc\ub3c4\uc778 \uc2ec\ub77c\ub85c \uc9d1\uacb0\ud55c \ud6c4 \ub378\ub9ac\ub85c \ud5a5\ud588\ub2e4. \ud56d\uc7c1 \ubc1c\uc0dd \uc774\ud6c4 2\uac1c\uc6d4\uc774 \uc9c0\ub09c \ub4a4 \ub450 \uad70\ub300\ub294 \ucfe0\ub974\uce74\ub97c \ud3ec\ud568\ud55c \uc5f0\ud569 \ubd80\ub300\ub97c \ud615\uc131\ud558\uc5ec \uc9c4\uad70\ud558\uc600\uace0, \uce74\ub974\ub0a0\uc5d0\uc11c \uc138\ud3ec\uc774 \ubc18\uad70\uacfc \uad50\uc804\ud558\uc600\ub2e4. \uc774 \uc804\ud22c\uc5d0\uc11c \ubc14\ub4e4\ub9ac \ucf00 \uc138\ub77c\uc774\uac00 \uc9c0\ud718\ud558\ub358 \uc138\ud3ec\uc774 \ubc18\uad70\uc740 \ud328\ud1f4\ud558\uc5ec \ub378\ub9ac\ub85c \ucca0\uc218\ud558\uc600\ub2e4. \ubcf8\uad6d\uc5d0\uc11c\ub3c4, \ucd5c\uc2e0\ubb34\uae30\ub97c \uac16\ucd98 2\uac1c \uc5f0\ub300\uc758 5,000\uba85\uc774 \ud574\ub85c\ub97c \ud1b5\ud574 \uc9c0\uc6d0\ub418\uc5c8\ub2e4. \uc774 \ubc16\uc5d0\ub3c4, \uc2dc\ud06c\uc871, \ud398\ub974\uc2dc\uc544\ub4f1 \uc5ec\ub7ec \uc774\ubbfc\uc871\uc758 \uc9c0\uc6d0\uad70\uc774 \uc18d\uc18d \ub3c4\ucc29\ud558\uc600\ub2e4."} {"question": "\uc8fc\ub2d8\uaed8\uc11c \uc131\ubaa8\ub2d8\uc744 \ud558\ub298\uacfc \ub545\uc758 \ubaa8\ud6c4\ub85c \uc138\uc6e0\ub2e4\uace0 \ub9d0\ud55c \uc774\ub294?", "paragraph": "\ucc9c\uc0c1\uc758 \ubaa8\ud6c4\ub97c \uc8fc\uc81c\ub85c \ud55c \uac00\ud1a8\ub9ad\uad50\ud68c\uc758 \uac00\ub974\uce68\uc740 \uad50\ud669 \ube44\uc624 12\uc138\uac00 \ubc18\ud3ec\ud55c \ud68c\uce59 \u300a\ucc9c\uc0c1\uc758 \ubaa8\ud6c4\uaed8\u300b(Ad Caeli Reginam)\uc5d0\uc11c \uc9d1\uc911\uc801\uc73c\ub85c \ub2e4\ub8e8\uc5b4\uc9c0\uace0 \uc788\ub2e4. \uc774 \ud68c\uce59\uc5d0 \ub530\ub974\uba74, \ub9c8\ub9ac\uc544\uc758 \uc544\ub4e4\uc778 \uc608\uc218 \uadf8\ub9ac\uc2a4\ub3c4\uac00 \uc774\uc2a4\ub77c\uc5d8\uc758 \uc784\uae08\uc774\uba70 \ub354 \ub098\uc544\uac00 \ud558\ub298\ub098\ub77c\uc640 \uc628 \uc6b0\uc8fc\uc758 \ud1b5\uce58\uc790\uc774\ubbc0\ub85c, \uadf8 \uc5b4\uba38\ub2c8\uc778 \ub9c8\ub9ac\uc544\ub294 \ub2f9\uc5f0\ud788 \ucc9c\uc0c1\uc758 \ubaa8\ud6c4\ub85c \ubd88\ub7ec \ub9c8\ub545\ud558\ub2e4\uace0 \uc5b8\uae09\ud55c\ub2e4. \uc77c\ubc18\uc801\uc73c\ub85c \ubaa8\ud6c4(\u6bcd\u540e)\ub780 \ub9d0 \uadf8\ub300\ub85c \uc784\uae08\uc758 \uc5b4\uba38\ub2c8\ub97c \uc9c0\uce6d\ud558\ub294 \uc6a9\uc5b4\uc774\ub2e4. \uad50\ud669 \ube44\uc624 12\uc138\uc758 \uc0ac\ub3c4\uc801 \uc11c\ud55c \u300a\uc9c0\uadf9\ud788 \uad00\ub300\ud558\uc2e0 \ud558\ub290\ub2d8\u300b(Munificentissimus Deus)\uc5d0 \ub530\ub77c, \uac00\ud1a8\ub9ad\uad50\ud68c\uc5d0\uc11c\ub294 \uc131\ubaa8 \ub9c8\ub9ac\uc544\uac00 \uc9c0\uc0c1 \uc0dd\uc560\ub97c \ub9c8\uce5c \ub2e4\uc74c\uc5d0 \uc721\uc2e0\uacfc \uc601\ud63c\uc774 \ubaa8\ub450 \ud558\ub298\ub098\ub77c\ub85c \ub4e4\uc5b4 \uc62c\ub9bc\uc744 \ubc1b\uc740 \uc601\uad11(\uc131\ubaa8 \uc2b9\ucc9c)\uc744 \ubc1b\uc558\ub2e4\uace0 \ubbff\ub294\ub2e4. \uad50\ud669 \ube44\uc624 9\uc138\ub294 \u201c\uc8fc\ub2d8\uaed8\uc11c\ub294 \uc131\ubaa8\ub2d8\uc744 \ud558\ub298\uacfc \ub545\uc758 \ubaa8\ud6c4\ub85c \uc138\uc6b0\uc168\uc2b5\ub2c8\ub2e4. \uc131\ubaa8\ub2d8\uaed8\uc11c\ub294 \ubaa8\ub4e0 \ucc9c\uc0ac\ub4e4\uc758 \ubb34\ub9ac\ubcf4\ub2e4 \uadf8\ub9ac\uace0 \uc131\uc778\ub4e4\uc758 \uacc4\uae09\ubcf4\ub2e4 \ub354 \ub192\uc774 \uc62c\ub9bc\uc744 \ubc1b\uc73c\uc168\uc2b5\ub2c8\ub2e4. \uc131\ubaa8\ub2d8\uaed8\uc11c\ub294 \ub2f9\uc2e0\uc758 \uc678\uc544\ub4dc\ub2d8\uc774\uc2e0 \uc8fc \uc608\uc218 \uadf8\ub9ac\uc2a4\ub3c4\ub2d8\uc758 \uc624\ub978\ud3b8\uc5d0 \uc11c \uacc4\uc2ed\ub2c8\ub2e4. \uc131\ubaa8\ub2d8\uaed8\uc11c\ub294 \ub2f9\uc2e0\uc758 \ubaa8\uc131\uc758 \uae30\ub3c4\ub85c \uac00\uc7a5 \uac15\ub825\ud558\uac8c \uc6b0\ub9ac\ub97c \uc704\ud574 \uc911\uc7ac\ud558\uc2dc\uba70 \ub2f9\uc2e0\uaed8\uc11c \uc6d0\ud558\uc2dc\ub294 \ubc14\ub97c \uc5bb\uc73c\uc2ed\ub2c8\ub2e4. \uc131\ubaa8\ub2d8\uaed8\uc11c\ub294 \uacb0\ucf54 \uc2e4\ub9dd\ud558\uac8c \ub418\uc2e4 \uc218 \uc5c6\uc2b5\ub2c8\ub2e4.\u201d\ub77c\uace0 \ub9d0\ud558\uc600\ub2e4.", "answer": "\uad50\ud669 \ube44\uc624 9\uc138", "sentence": "\uad50\ud669 \ube44\uc624 9\uc138 \ub294 \u201c\uc8fc\ub2d8\uaed8\uc11c\ub294 \uc131\ubaa8\ub2d8\uc744 \ud558\ub298\uacfc \ub545\uc758 \ubaa8\ud6c4\ub85c \uc138\uc6b0\uc168\uc2b5\ub2c8\ub2e4.", "paragraph_sentence": "\ucc9c\uc0c1\uc758 \ubaa8\ud6c4\ub97c \uc8fc\uc81c\ub85c \ud55c \uac00\ud1a8\ub9ad\uad50\ud68c\uc758 \uac00\ub974\uce68\uc740 \uad50\ud669 \ube44\uc624 12\uc138\uac00 \ubc18\ud3ec\ud55c \ud68c\uce59 \u300a\ucc9c\uc0c1\uc758 \ubaa8\ud6c4\uaed8\u300b(Ad Caeli Reginam)\uc5d0\uc11c \uc9d1\uc911\uc801\uc73c\ub85c \ub2e4\ub8e8\uc5b4\uc9c0\uace0 \uc788\ub2e4. \uc774 \ud68c\uce59\uc5d0 \ub530\ub974\uba74, \ub9c8\ub9ac\uc544\uc758 \uc544\ub4e4\uc778 \uc608\uc218 \uadf8\ub9ac\uc2a4\ub3c4\uac00 \uc774\uc2a4\ub77c\uc5d8\uc758 \uc784\uae08\uc774\uba70 \ub354 \ub098\uc544\uac00 \ud558\ub298\ub098\ub77c\uc640 \uc628 \uc6b0\uc8fc\uc758 \ud1b5\uce58\uc790\uc774\ubbc0\ub85c, \uadf8 \uc5b4\uba38\ub2c8\uc778 \ub9c8\ub9ac\uc544\ub294 \ub2f9\uc5f0\ud788 \ucc9c\uc0c1\uc758 \ubaa8\ud6c4\ub85c \ubd88\ub7ec \ub9c8\ub545\ud558\ub2e4\uace0 \uc5b8\uae09\ud55c\ub2e4. \uc77c\ubc18\uc801\uc73c\ub85c \ubaa8\ud6c4(\u6bcd\u540e)\ub780 \ub9d0 \uadf8\ub300\ub85c \uc784\uae08\uc758 \uc5b4\uba38\ub2c8\ub97c \uc9c0\uce6d\ud558\ub294 \uc6a9\uc5b4\uc774\ub2e4. \uad50\ud669 \ube44\uc624 12\uc138\uc758 \uc0ac\ub3c4\uc801 \uc11c\ud55c \u300a\uc9c0\uadf9\ud788 \uad00\ub300\ud558\uc2e0 \ud558\ub290\ub2d8\u300b(Munificentissimus Deus)\uc5d0 \ub530\ub77c, \uac00\ud1a8\ub9ad\uad50\ud68c\uc5d0\uc11c\ub294 \uc131\ubaa8 \ub9c8\ub9ac\uc544\uac00 \uc9c0\uc0c1 \uc0dd\uc560\ub97c \ub9c8\uce5c \ub2e4\uc74c\uc5d0 \uc721\uc2e0\uacfc \uc601\ud63c\uc774 \ubaa8\ub450 \ud558\ub298\ub098\ub77c\ub85c \ub4e4\uc5b4 \uc62c\ub9bc\uc744 \ubc1b\uc740 \uc601\uad11(\uc131\ubaa8 \uc2b9\ucc9c)\uc744 \ubc1b\uc558\ub2e4\uace0 \ubbff\ub294\ub2e4. \uad50\ud669 \ube44\uc624 9\uc138 \ub294 \u201c\uc8fc\ub2d8\uaed8\uc11c\ub294 \uc131\ubaa8\ub2d8\uc744 \ud558\ub298\uacfc \ub545\uc758 \ubaa8\ud6c4\ub85c \uc138\uc6b0\uc168\uc2b5\ub2c8\ub2e4. \uc131\ubaa8\ub2d8\uaed8\uc11c\ub294 \ubaa8\ub4e0 \ucc9c\uc0ac\ub4e4\uc758 \ubb34\ub9ac\ubcf4\ub2e4 \uadf8\ub9ac\uace0 \uc131\uc778\ub4e4\uc758 \uacc4\uae09\ubcf4\ub2e4 \ub354 \ub192\uc774 \uc62c\ub9bc\uc744 \ubc1b\uc73c\uc168\uc2b5\ub2c8\ub2e4. \uc131\ubaa8\ub2d8\uaed8\uc11c\ub294 \ub2f9\uc2e0\uc758 \uc678\uc544\ub4dc\ub2d8\uc774\uc2e0 \uc8fc \uc608\uc218 \uadf8\ub9ac\uc2a4\ub3c4\ub2d8\uc758 \uc624\ub978\ud3b8\uc5d0 \uc11c \uacc4\uc2ed\ub2c8\ub2e4. \uc131\ubaa8\ub2d8\uaed8\uc11c\ub294 \ub2f9\uc2e0\uc758 \ubaa8\uc131\uc758 \uae30\ub3c4\ub85c \uac00\uc7a5 \uac15\ub825\ud558\uac8c \uc6b0\ub9ac\ub97c \uc704\ud574 \uc911\uc7ac\ud558\uc2dc\uba70 \ub2f9\uc2e0\uaed8\uc11c \uc6d0\ud558\uc2dc\ub294 \ubc14\ub97c \uc5bb\uc73c\uc2ed\ub2c8\ub2e4. \uc131\ubaa8\ub2d8\uaed8\uc11c\ub294 \uacb0\ucf54 \uc2e4\ub9dd\ud558\uac8c \ub418\uc2e4 \uc218 \uc5c6\uc2b5\ub2c8\ub2e4. \u201d\ub77c\uace0 \ub9d0\ud558\uc600\ub2e4.", "paragraph_answer": "\ucc9c\uc0c1\uc758 \ubaa8\ud6c4\ub97c \uc8fc\uc81c\ub85c \ud55c \uac00\ud1a8\ub9ad\uad50\ud68c\uc758 \uac00\ub974\uce68\uc740 \uad50\ud669 \ube44\uc624 12\uc138\uac00 \ubc18\ud3ec\ud55c \ud68c\uce59 \u300a\ucc9c\uc0c1\uc758 \ubaa8\ud6c4\uaed8\u300b(Ad Caeli Reginam)\uc5d0\uc11c \uc9d1\uc911\uc801\uc73c\ub85c \ub2e4\ub8e8\uc5b4\uc9c0\uace0 \uc788\ub2e4. \uc774 \ud68c\uce59\uc5d0 \ub530\ub974\uba74, \ub9c8\ub9ac\uc544\uc758 \uc544\ub4e4\uc778 \uc608\uc218 \uadf8\ub9ac\uc2a4\ub3c4\uac00 \uc774\uc2a4\ub77c\uc5d8\uc758 \uc784\uae08\uc774\uba70 \ub354 \ub098\uc544\uac00 \ud558\ub298\ub098\ub77c\uc640 \uc628 \uc6b0\uc8fc\uc758 \ud1b5\uce58\uc790\uc774\ubbc0\ub85c, \uadf8 \uc5b4\uba38\ub2c8\uc778 \ub9c8\ub9ac\uc544\ub294 \ub2f9\uc5f0\ud788 \ucc9c\uc0c1\uc758 \ubaa8\ud6c4\ub85c \ubd88\ub7ec \ub9c8\ub545\ud558\ub2e4\uace0 \uc5b8\uae09\ud55c\ub2e4. \uc77c\ubc18\uc801\uc73c\ub85c \ubaa8\ud6c4(\u6bcd\u540e)\ub780 \ub9d0 \uadf8\ub300\ub85c \uc784\uae08\uc758 \uc5b4\uba38\ub2c8\ub97c \uc9c0\uce6d\ud558\ub294 \uc6a9\uc5b4\uc774\ub2e4. \uad50\ud669 \ube44\uc624 12\uc138\uc758 \uc0ac\ub3c4\uc801 \uc11c\ud55c \u300a\uc9c0\uadf9\ud788 \uad00\ub300\ud558\uc2e0 \ud558\ub290\ub2d8\u300b(Munificentissimus Deus)\uc5d0 \ub530\ub77c, \uac00\ud1a8\ub9ad\uad50\ud68c\uc5d0\uc11c\ub294 \uc131\ubaa8 \ub9c8\ub9ac\uc544\uac00 \uc9c0\uc0c1 \uc0dd\uc560\ub97c \ub9c8\uce5c \ub2e4\uc74c\uc5d0 \uc721\uc2e0\uacfc \uc601\ud63c\uc774 \ubaa8\ub450 \ud558\ub298\ub098\ub77c\ub85c \ub4e4\uc5b4 \uc62c\ub9bc\uc744 \ubc1b\uc740 \uc601\uad11(\uc131\ubaa8 \uc2b9\ucc9c)\uc744 \ubc1b\uc558\ub2e4\uace0 \ubbff\ub294\ub2e4. \uad50\ud669 \ube44\uc624 9\uc138 \ub294 \u201c\uc8fc\ub2d8\uaed8\uc11c\ub294 \uc131\ubaa8\ub2d8\uc744 \ud558\ub298\uacfc \ub545\uc758 \ubaa8\ud6c4\ub85c \uc138\uc6b0\uc168\uc2b5\ub2c8\ub2e4. \uc131\ubaa8\ub2d8\uaed8\uc11c\ub294 \ubaa8\ub4e0 \ucc9c\uc0ac\ub4e4\uc758 \ubb34\ub9ac\ubcf4\ub2e4 \uadf8\ub9ac\uace0 \uc131\uc778\ub4e4\uc758 \uacc4\uae09\ubcf4\ub2e4 \ub354 \ub192\uc774 \uc62c\ub9bc\uc744 \ubc1b\uc73c\uc168\uc2b5\ub2c8\ub2e4. \uc131\ubaa8\ub2d8\uaed8\uc11c\ub294 \ub2f9\uc2e0\uc758 \uc678\uc544\ub4dc\ub2d8\uc774\uc2e0 \uc8fc \uc608\uc218 \uadf8\ub9ac\uc2a4\ub3c4\ub2d8\uc758 \uc624\ub978\ud3b8\uc5d0 \uc11c \uacc4\uc2ed\ub2c8\ub2e4. \uc131\ubaa8\ub2d8\uaed8\uc11c\ub294 \ub2f9\uc2e0\uc758 \ubaa8\uc131\uc758 \uae30\ub3c4\ub85c \uac00\uc7a5 \uac15\ub825\ud558\uac8c \uc6b0\ub9ac\ub97c \uc704\ud574 \uc911\uc7ac\ud558\uc2dc\uba70 \ub2f9\uc2e0\uaed8\uc11c \uc6d0\ud558\uc2dc\ub294 \ubc14\ub97c \uc5bb\uc73c\uc2ed\ub2c8\ub2e4. \uc131\ubaa8\ub2d8\uaed8\uc11c\ub294 \uacb0\ucf54 \uc2e4\ub9dd\ud558\uac8c \ub418\uc2e4 \uc218 \uc5c6\uc2b5\ub2c8\ub2e4.\u201d\ub77c\uace0 \ub9d0\ud558\uc600\ub2e4.", "sentence_answer": " \uad50\ud669 \ube44\uc624 9\uc138 \ub294 \u201c\uc8fc\ub2d8\uaed8\uc11c\ub294 \uc131\ubaa8\ub2d8\uc744 \ud558\ub298\uacfc \ub545\uc758 \ubaa8\ud6c4\ub85c \uc138\uc6b0\uc168\uc2b5\ub2c8\ub2e4.", "paragraph_id": "6596570-0-0", "paragraph_question": "question: \uc8fc\ub2d8\uaed8\uc11c \uc131\ubaa8\ub2d8\uc744 \ud558\ub298\uacfc \ub545\uc758 \ubaa8\ud6c4\ub85c \uc138\uc6e0\ub2e4\uace0 \ub9d0\ud55c \uc774\ub294?, context: \ucc9c\uc0c1\uc758 \ubaa8\ud6c4\ub97c \uc8fc\uc81c\ub85c \ud55c \uac00\ud1a8\ub9ad\uad50\ud68c\uc758 \uac00\ub974\uce68\uc740 \uad50\ud669 \ube44\uc624 12\uc138\uac00 \ubc18\ud3ec\ud55c \ud68c\uce59 \u300a\ucc9c\uc0c1\uc758 \ubaa8\ud6c4\uaed8\u300b(Ad Caeli Reginam)\uc5d0\uc11c \uc9d1\uc911\uc801\uc73c\ub85c \ub2e4\ub8e8\uc5b4\uc9c0\uace0 \uc788\ub2e4. \uc774 \ud68c\uce59\uc5d0 \ub530\ub974\uba74, \ub9c8\ub9ac\uc544\uc758 \uc544\ub4e4\uc778 \uc608\uc218 \uadf8\ub9ac\uc2a4\ub3c4\uac00 \uc774\uc2a4\ub77c\uc5d8\uc758 \uc784\uae08\uc774\uba70 \ub354 \ub098\uc544\uac00 \ud558\ub298\ub098\ub77c\uc640 \uc628 \uc6b0\uc8fc\uc758 \ud1b5\uce58\uc790\uc774\ubbc0\ub85c, \uadf8 \uc5b4\uba38\ub2c8\uc778 \ub9c8\ub9ac\uc544\ub294 \ub2f9\uc5f0\ud788 \ucc9c\uc0c1\uc758 \ubaa8\ud6c4\ub85c \ubd88\ub7ec \ub9c8\ub545\ud558\ub2e4\uace0 \uc5b8\uae09\ud55c\ub2e4. \uc77c\ubc18\uc801\uc73c\ub85c \ubaa8\ud6c4(\u6bcd\u540e)\ub780 \ub9d0 \uadf8\ub300\ub85c \uc784\uae08\uc758 \uc5b4\uba38\ub2c8\ub97c \uc9c0\uce6d\ud558\ub294 \uc6a9\uc5b4\uc774\ub2e4. \uad50\ud669 \ube44\uc624 12\uc138\uc758 \uc0ac\ub3c4\uc801 \uc11c\ud55c \u300a\uc9c0\uadf9\ud788 \uad00\ub300\ud558\uc2e0 \ud558\ub290\ub2d8\u300b(Munificentissimus Deus)\uc5d0 \ub530\ub77c, \uac00\ud1a8\ub9ad\uad50\ud68c\uc5d0\uc11c\ub294 \uc131\ubaa8 \ub9c8\ub9ac\uc544\uac00 \uc9c0\uc0c1 \uc0dd\uc560\ub97c \ub9c8\uce5c \ub2e4\uc74c\uc5d0 \uc721\uc2e0\uacfc \uc601\ud63c\uc774 \ubaa8\ub450 \ud558\ub298\ub098\ub77c\ub85c \ub4e4\uc5b4 \uc62c\ub9bc\uc744 \ubc1b\uc740 \uc601\uad11(\uc131\ubaa8 \uc2b9\ucc9c)\uc744 \ubc1b\uc558\ub2e4\uace0 \ubbff\ub294\ub2e4. \uad50\ud669 \ube44\uc624 9\uc138\ub294 \u201c\uc8fc\ub2d8\uaed8\uc11c\ub294 \uc131\ubaa8\ub2d8\uc744 \ud558\ub298\uacfc \ub545\uc758 \ubaa8\ud6c4\ub85c \uc138\uc6b0\uc168\uc2b5\ub2c8\ub2e4. \uc131\ubaa8\ub2d8\uaed8\uc11c\ub294 \ubaa8\ub4e0 \ucc9c\uc0ac\ub4e4\uc758 \ubb34\ub9ac\ubcf4\ub2e4 \uadf8\ub9ac\uace0 \uc131\uc778\ub4e4\uc758 \uacc4\uae09\ubcf4\ub2e4 \ub354 \ub192\uc774 \uc62c\ub9bc\uc744 \ubc1b\uc73c\uc168\uc2b5\ub2c8\ub2e4. \uc131\ubaa8\ub2d8\uaed8\uc11c\ub294 \ub2f9\uc2e0\uc758 \uc678\uc544\ub4dc\ub2d8\uc774\uc2e0 \uc8fc \uc608\uc218 \uadf8\ub9ac\uc2a4\ub3c4\ub2d8\uc758 \uc624\ub978\ud3b8\uc5d0 \uc11c \uacc4\uc2ed\ub2c8\ub2e4. \uc131\ubaa8\ub2d8\uaed8\uc11c\ub294 \ub2f9\uc2e0\uc758 \ubaa8\uc131\uc758 \uae30\ub3c4\ub85c \uac00\uc7a5 \uac15\ub825\ud558\uac8c \uc6b0\ub9ac\ub97c \uc704\ud574 \uc911\uc7ac\ud558\uc2dc\uba70 \ub2f9\uc2e0\uaed8\uc11c \uc6d0\ud558\uc2dc\ub294 \ubc14\ub97c \uc5bb\uc73c\uc2ed\ub2c8\ub2e4. \uc131\ubaa8\ub2d8\uaed8\uc11c\ub294 \uacb0\ucf54 \uc2e4\ub9dd\ud558\uac8c \ub418\uc2e4 \uc218 \uc5c6\uc2b5\ub2c8\ub2e4.\u201d\ub77c\uace0 \ub9d0\ud558\uc600\ub2e4."} {"question": "\ubcf4\ud5e4\ubbf8\uc544\uc640 \ubaa8\ub77c\ubc14\ub97c \ub3c5\uc77c \uc5f0\ubc29\uc758 \ud55c \ubd80\ubd84\uc73c\ub85c \ud558\uc9c0 \uc54a\uae30 \uc704\ud574 \ub178\ub825\ud55c \ub098\ub77c\ub294?", "paragraph": "\ub3c5\uc77c\uc758 \ubbfc\uc871\uc801 \ud1b5\uc77c\uc774 \ubb34\uc5c7\uc778\uc9c0 \uc815\uc758\ud558\ub294 \uac83\uc740 \ud504\ub791\ud06c\ud478\ub974\ud2b8 \uad6d\ubbfc\uc758\ud68c\ub97c \uc5c4\uccad\ub09c \uc5b4\ub824\uc6c0\uc5d0 \uc9c1\uba74\ud558\uac8c \ud558\uc600\ub2e4. \ud1b5\uc77c\uc774 \ub3c5\uc77c\uc778\ub9cc\uc744 \ud3ec\ud568\ud574\uc57c \ud560\uc9c0, \ub2e4\ub978 \uc18c\uc218 \ubbfc\uc871\uae4c\uc9c0 \ud3ec\ud568\ud574\uc57c \ud560\uc9c0 \uad6d\ubbfc\uad6d\uac00\uc758 \uacbd\uacc4\uc5d0 \ub300\ud55c \ubb38\uc81c\uac00 \uc81c\uae30\ub418\uc5c8\ub2e4. \uc5ec\uae30\uc5d0 \uc788\uc5b4\uc11c\ub294 \uc5ec\uc804\ud788 \uacb0\uc815\ub418\uc9c0 \uc54a\uc740 \ubb38\uc81c\uc778 \uc290\ub808\uc2a4\ube44\ud788\uc758 \ud3ec\ud568 \uc5ec\ubd80\ub294 \uc791\uc740 \ubb38\uc81c\uc5d0 \ubd88\uacfc\ud588\ub2e4. \ub354 \uc5b4\ub824\uc6b4 \uc77c\ub85c \ub4dc\ub7ec\ub09c \uc0ac\uc2e4\uc740, \ub3c5\uc77c \uc5f0\ubc29\uc758 \ub450 \uac15\ub300\uad6d\uc778 \ud504\ub85c\uc774\uc13c\uacfc \ud2b9\ud788 \uc624\uc2a4\ud2b8\ub9ac\uc544\uc758 \ucee4\ub2e4\ub780 \ubd80\ubd84\uc740 \ub3c5\uc77c\uc5b4 \uc0ac\uc6a9 \uad8c\uc5ed\uc758 \ubc16\uc5d0 \ub193\uc5ec \uc788\uc5c8\uace0, \uc774 \uc9c0\ubc29\ub4e4\uc744 \ub3c5\uc77c \uad6d\uac00\uc5d0 \ud3ec\ud568\ud560\uc9c0 \uc5ec\ubd80\ub294 \ub2e8\uc21c\ud788 \uc8fc\ubbfc\uc758 \ubbfc\uc871\uc801 \uc815\uccb4\uc131 \ubb38\uc81c\uc5d0 \uadf8\uce58\ub294 \uac83\uc774 \uc544\ub2c8\ub77c \ub3c5\uc77c \uc601\ub0b4 \uad6d\uac00\ub4e4\uac04\uc758 \uc138\ub825 \uc2f8\uc6c0\uc774\uae30\ub3c4 \ud55c\ub2e4\ub294 \uac83\uc774\uc5c8\ub2e4. \ub300\ud45c\ub4e4\uc740 \uccb4\ucf54\uc758 \ub178\ub825\uc5d0\ub3c4 \ubd88\uad6c\ud558\uace0, \ubcf4\ud5e4\ubbf8\uc544(B\u00f6hmen)\uc640 \ubaa8\ub77c\ubc14(M\u00e4hren)\ub97c \ub3c5\uc77c \uc5f0\ubc29\uc758 \ud55c \ubd80\ubd84\uc73c\ub85c \ud560 \uac83\uc744 \uba85\ubc31\ud788 \ud558\uc600\ub2e4. \ub9c8\ucc2c\uac00\uc9c0\ub85c \uc758\uc6d0\ub4e4\uc740 1848\ub144 \ub300\ud3f4\ub780\ub4dc \ubd09\uae30\ub97c \uc774\uc720\ub85c \ud3ec\uc820 \uc9c0\ubc29\uc744 \ud3b8\uc785\uc2dc\ud0ac \uac83\uc744 \uacb0\uc758\ud558\uc600\ub2e4. \ud3ec\uc820\uc740 \ubcf8\ub798 \ud3f4\ub780\ub4dc\uc758 \uc77c\ubd80\uc600\uc73c\ub098, \ud3f4\ub780\ub4dc \ubd84\ud560\uc73c\ub85c \ud504\ub85c\uc774\uc13c\uc758 \uc9c0\ubc29\uc774 \ub41c \uacf3\uc774\uc5c8\ub2e4. \ud3ec\uc5b4\uba54\ub974\uce20 \uc2dc\uae30\uc5d0 \ud3f4\ub780\ub4dc\ub294 \ubbfc\uc871 \ud574\ubc29\uc758 \uac00\ub2a5\uc131\uc744 \ubcf4\uace0 \ud658\ud76c\ud558\uc600\uc9c0\ub9cc \ud3ec\uc820 \uc9c0\ubc29\uc758 \ub3c5\uc77c \ud3b8\uc785\uc774\ub77c\ub294 \uacb0\uc815\uc740 \uc9c4\ubcf4 \uc9c4\uc601\ub9c8\uc800\ub3c4 \ud3f4\ub780\ub4dc \ubbfc\uc871 \ud574\ubc29\ubcf4\ub2e4\ub294 \ub3c5\uc77c \uad6d\ubbfc\uad6d\uac00 \uc0ac\uc0c1\uc744 \ub354 \ud3b8\uc560\ud55c\ub2e4\ub294 \uac83\uc744 \ubcf4\uc5ec\uc8fc\uc5c8\ub2e4.", "answer": "\uccb4\ucf54", "sentence": "\ub300\ud45c\ub4e4\uc740 \uccb4\ucf54 \uc758 \ub178\ub825\uc5d0\ub3c4 \ubd88\uad6c\ud558\uace0, \ubcf4\ud5e4\ubbf8\uc544(B\u00f6hmen)\uc640 \ubaa8\ub77c\ubc14(M\u00e4hren)\ub97c \ub3c5\uc77c \uc5f0\ubc29\uc758 \ud55c \ubd80\ubd84\uc73c\ub85c \ud560 \uac83\uc744 \uba85\ubc31\ud788 \ud558\uc600\ub2e4.", "paragraph_sentence": "\ub3c5\uc77c\uc758 \ubbfc\uc871\uc801 \ud1b5\uc77c\uc774 \ubb34\uc5c7\uc778\uc9c0 \uc815\uc758\ud558\ub294 \uac83\uc740 \ud504\ub791\ud06c\ud478\ub974\ud2b8 \uad6d\ubbfc\uc758\ud68c\ub97c \uc5c4\uccad\ub09c \uc5b4\ub824\uc6c0\uc5d0 \uc9c1\uba74\ud558\uac8c \ud558\uc600\ub2e4. \ud1b5\uc77c\uc774 \ub3c5\uc77c\uc778\ub9cc\uc744 \ud3ec\ud568\ud574\uc57c \ud560\uc9c0, \ub2e4\ub978 \uc18c\uc218 \ubbfc\uc871\uae4c\uc9c0 \ud3ec\ud568\ud574\uc57c \ud560\uc9c0 \uad6d\ubbfc\uad6d\uac00\uc758 \uacbd\uacc4\uc5d0 \ub300\ud55c \ubb38\uc81c\uac00 \uc81c\uae30\ub418\uc5c8\ub2e4. \uc5ec\uae30\uc5d0 \uc788\uc5b4\uc11c\ub294 \uc5ec\uc804\ud788 \uacb0\uc815\ub418\uc9c0 \uc54a\uc740 \ubb38\uc81c\uc778 \uc290\ub808\uc2a4\ube44\ud788\uc758 \ud3ec\ud568 \uc5ec\ubd80\ub294 \uc791\uc740 \ubb38\uc81c\uc5d0 \ubd88\uacfc\ud588\ub2e4. \ub354 \uc5b4\ub824\uc6b4 \uc77c\ub85c \ub4dc\ub7ec\ub09c \uc0ac\uc2e4\uc740, \ub3c5\uc77c \uc5f0\ubc29\uc758 \ub450 \uac15\ub300\uad6d\uc778 \ud504\ub85c\uc774\uc13c\uacfc \ud2b9\ud788 \uc624\uc2a4\ud2b8\ub9ac\uc544\uc758 \ucee4\ub2e4\ub780 \ubd80\ubd84\uc740 \ub3c5\uc77c\uc5b4 \uc0ac\uc6a9 \uad8c\uc5ed\uc758 \ubc16\uc5d0 \ub193\uc5ec \uc788\uc5c8\uace0, \uc774 \uc9c0\ubc29\ub4e4\uc744 \ub3c5\uc77c \uad6d\uac00\uc5d0 \ud3ec\ud568\ud560\uc9c0 \uc5ec\ubd80\ub294 \ub2e8\uc21c\ud788 \uc8fc\ubbfc\uc758 \ubbfc\uc871\uc801 \uc815\uccb4\uc131 \ubb38\uc81c\uc5d0 \uadf8\uce58\ub294 \uac83\uc774 \uc544\ub2c8\ub77c \ub3c5\uc77c \uc601\ub0b4 \uad6d\uac00\ub4e4\uac04\uc758 \uc138\ub825 \uc2f8\uc6c0\uc774\uae30\ub3c4 \ud55c\ub2e4\ub294 \uac83\uc774\uc5c8\ub2e4. \ub300\ud45c\ub4e4\uc740 \uccb4\ucf54 \uc758 \ub178\ub825\uc5d0\ub3c4 \ubd88\uad6c\ud558\uace0, \ubcf4\ud5e4\ubbf8\uc544(B\u00f6hmen)\uc640 \ubaa8\ub77c\ubc14(M\u00e4hren)\ub97c \ub3c5\uc77c \uc5f0\ubc29\uc758 \ud55c \ubd80\ubd84\uc73c\ub85c \ud560 \uac83\uc744 \uba85\ubc31\ud788 \ud558\uc600\ub2e4. \ub9c8\ucc2c\uac00\uc9c0\ub85c \uc758\uc6d0\ub4e4\uc740 1848\ub144 \ub300\ud3f4\ub780\ub4dc \ubd09\uae30\ub97c \uc774\uc720\ub85c \ud3ec\uc820 \uc9c0\ubc29\uc744 \ud3b8\uc785\uc2dc\ud0ac \uac83\uc744 \uacb0\uc758\ud558\uc600\ub2e4. \ud3ec\uc820\uc740 \ubcf8\ub798 \ud3f4\ub780\ub4dc\uc758 \uc77c\ubd80\uc600\uc73c\ub098, \ud3f4\ub780\ub4dc \ubd84\ud560\uc73c\ub85c \ud504\ub85c\uc774\uc13c\uc758 \uc9c0\ubc29\uc774 \ub41c \uacf3\uc774\uc5c8\ub2e4. \ud3ec\uc5b4\uba54\ub974\uce20 \uc2dc\uae30\uc5d0 \ud3f4\ub780\ub4dc\ub294 \ubbfc\uc871 \ud574\ubc29\uc758 \uac00\ub2a5\uc131\uc744 \ubcf4\uace0 \ud658\ud76c\ud558\uc600\uc9c0\ub9cc \ud3ec\uc820 \uc9c0\ubc29\uc758 \ub3c5\uc77c \ud3b8\uc785\uc774\ub77c\ub294 \uacb0\uc815\uc740 \uc9c4\ubcf4 \uc9c4\uc601\ub9c8\uc800\ub3c4 \ud3f4\ub780\ub4dc \ubbfc\uc871 \ud574\ubc29\ubcf4\ub2e4\ub294 \ub3c5\uc77c \uad6d\ubbfc\uad6d\uac00 \uc0ac\uc0c1\uc744 \ub354 \ud3b8\uc560\ud55c\ub2e4\ub294 \uac83\uc744 \ubcf4\uc5ec\uc8fc\uc5c8\ub2e4.", "paragraph_answer": "\ub3c5\uc77c\uc758 \ubbfc\uc871\uc801 \ud1b5\uc77c\uc774 \ubb34\uc5c7\uc778\uc9c0 \uc815\uc758\ud558\ub294 \uac83\uc740 \ud504\ub791\ud06c\ud478\ub974\ud2b8 \uad6d\ubbfc\uc758\ud68c\ub97c \uc5c4\uccad\ub09c \uc5b4\ub824\uc6c0\uc5d0 \uc9c1\uba74\ud558\uac8c \ud558\uc600\ub2e4. \ud1b5\uc77c\uc774 \ub3c5\uc77c\uc778\ub9cc\uc744 \ud3ec\ud568\ud574\uc57c \ud560\uc9c0, \ub2e4\ub978 \uc18c\uc218 \ubbfc\uc871\uae4c\uc9c0 \ud3ec\ud568\ud574\uc57c \ud560\uc9c0 \uad6d\ubbfc\uad6d\uac00\uc758 \uacbd\uacc4\uc5d0 \ub300\ud55c \ubb38\uc81c\uac00 \uc81c\uae30\ub418\uc5c8\ub2e4. \uc5ec\uae30\uc5d0 \uc788\uc5b4\uc11c\ub294 \uc5ec\uc804\ud788 \uacb0\uc815\ub418\uc9c0 \uc54a\uc740 \ubb38\uc81c\uc778 \uc290\ub808\uc2a4\ube44\ud788\uc758 \ud3ec\ud568 \uc5ec\ubd80\ub294 \uc791\uc740 \ubb38\uc81c\uc5d0 \ubd88\uacfc\ud588\ub2e4. \ub354 \uc5b4\ub824\uc6b4 \uc77c\ub85c \ub4dc\ub7ec\ub09c \uc0ac\uc2e4\uc740, \ub3c5\uc77c \uc5f0\ubc29\uc758 \ub450 \uac15\ub300\uad6d\uc778 \ud504\ub85c\uc774\uc13c\uacfc \ud2b9\ud788 \uc624\uc2a4\ud2b8\ub9ac\uc544\uc758 \ucee4\ub2e4\ub780 \ubd80\ubd84\uc740 \ub3c5\uc77c\uc5b4 \uc0ac\uc6a9 \uad8c\uc5ed\uc758 \ubc16\uc5d0 \ub193\uc5ec \uc788\uc5c8\uace0, \uc774 \uc9c0\ubc29\ub4e4\uc744 \ub3c5\uc77c \uad6d\uac00\uc5d0 \ud3ec\ud568\ud560\uc9c0 \uc5ec\ubd80\ub294 \ub2e8\uc21c\ud788 \uc8fc\ubbfc\uc758 \ubbfc\uc871\uc801 \uc815\uccb4\uc131 \ubb38\uc81c\uc5d0 \uadf8\uce58\ub294 \uac83\uc774 \uc544\ub2c8\ub77c \ub3c5\uc77c \uc601\ub0b4 \uad6d\uac00\ub4e4\uac04\uc758 \uc138\ub825 \uc2f8\uc6c0\uc774\uae30\ub3c4 \ud55c\ub2e4\ub294 \uac83\uc774\uc5c8\ub2e4. \ub300\ud45c\ub4e4\uc740 \uccb4\ucf54 \uc758 \ub178\ub825\uc5d0\ub3c4 \ubd88\uad6c\ud558\uace0, \ubcf4\ud5e4\ubbf8\uc544(B\u00f6hmen)\uc640 \ubaa8\ub77c\ubc14(M\u00e4hren)\ub97c \ub3c5\uc77c \uc5f0\ubc29\uc758 \ud55c \ubd80\ubd84\uc73c\ub85c \ud560 \uac83\uc744 \uba85\ubc31\ud788 \ud558\uc600\ub2e4. \ub9c8\ucc2c\uac00\uc9c0\ub85c \uc758\uc6d0\ub4e4\uc740 1848\ub144 \ub300\ud3f4\ub780\ub4dc \ubd09\uae30\ub97c \uc774\uc720\ub85c \ud3ec\uc820 \uc9c0\ubc29\uc744 \ud3b8\uc785\uc2dc\ud0ac \uac83\uc744 \uacb0\uc758\ud558\uc600\ub2e4. \ud3ec\uc820\uc740 \ubcf8\ub798 \ud3f4\ub780\ub4dc\uc758 \uc77c\ubd80\uc600\uc73c\ub098, \ud3f4\ub780\ub4dc \ubd84\ud560\uc73c\ub85c \ud504\ub85c\uc774\uc13c\uc758 \uc9c0\ubc29\uc774 \ub41c \uacf3\uc774\uc5c8\ub2e4. \ud3ec\uc5b4\uba54\ub974\uce20 \uc2dc\uae30\uc5d0 \ud3f4\ub780\ub4dc\ub294 \ubbfc\uc871 \ud574\ubc29\uc758 \uac00\ub2a5\uc131\uc744 \ubcf4\uace0 \ud658\ud76c\ud558\uc600\uc9c0\ub9cc \ud3ec\uc820 \uc9c0\ubc29\uc758 \ub3c5\uc77c \ud3b8\uc785\uc774\ub77c\ub294 \uacb0\uc815\uc740 \uc9c4\ubcf4 \uc9c4\uc601\ub9c8\uc800\ub3c4 \ud3f4\ub780\ub4dc \ubbfc\uc871 \ud574\ubc29\ubcf4\ub2e4\ub294 \ub3c5\uc77c \uad6d\ubbfc\uad6d\uac00 \uc0ac\uc0c1\uc744 \ub354 \ud3b8\uc560\ud55c\ub2e4\ub294 \uac83\uc744 \ubcf4\uc5ec\uc8fc\uc5c8\ub2e4.", "sentence_answer": "\ub300\ud45c\ub4e4\uc740 \uccb4\ucf54 \uc758 \ub178\ub825\uc5d0\ub3c4 \ubd88\uad6c\ud558\uace0, \ubcf4\ud5e4\ubbf8\uc544(B\u00f6hmen)\uc640 \ubaa8\ub77c\ubc14(M\u00e4hren)\ub97c \ub3c5\uc77c \uc5f0\ubc29\uc758 \ud55c \ubd80\ubd84\uc73c\ub85c \ud560 \uac83\uc744 \uba85\ubc31\ud788 \ud558\uc600\ub2e4.", "paragraph_id": "6465718-22-0", "paragraph_question": "question: \ubcf4\ud5e4\ubbf8\uc544\uc640 \ubaa8\ub77c\ubc14\ub97c \ub3c5\uc77c \uc5f0\ubc29\uc758 \ud55c \ubd80\ubd84\uc73c\ub85c \ud558\uc9c0 \uc54a\uae30 \uc704\ud574 \ub178\ub825\ud55c \ub098\ub77c\ub294?, context: \ub3c5\uc77c\uc758 \ubbfc\uc871\uc801 \ud1b5\uc77c\uc774 \ubb34\uc5c7\uc778\uc9c0 \uc815\uc758\ud558\ub294 \uac83\uc740 \ud504\ub791\ud06c\ud478\ub974\ud2b8 \uad6d\ubbfc\uc758\ud68c\ub97c \uc5c4\uccad\ub09c \uc5b4\ub824\uc6c0\uc5d0 \uc9c1\uba74\ud558\uac8c \ud558\uc600\ub2e4. \ud1b5\uc77c\uc774 \ub3c5\uc77c\uc778\ub9cc\uc744 \ud3ec\ud568\ud574\uc57c \ud560\uc9c0, \ub2e4\ub978 \uc18c\uc218 \ubbfc\uc871\uae4c\uc9c0 \ud3ec\ud568\ud574\uc57c \ud560\uc9c0 \uad6d\ubbfc\uad6d\uac00\uc758 \uacbd\uacc4\uc5d0 \ub300\ud55c \ubb38\uc81c\uac00 \uc81c\uae30\ub418\uc5c8\ub2e4. \uc5ec\uae30\uc5d0 \uc788\uc5b4\uc11c\ub294 \uc5ec\uc804\ud788 \uacb0\uc815\ub418\uc9c0 \uc54a\uc740 \ubb38\uc81c\uc778 \uc290\ub808\uc2a4\ube44\ud788\uc758 \ud3ec\ud568 \uc5ec\ubd80\ub294 \uc791\uc740 \ubb38\uc81c\uc5d0 \ubd88\uacfc\ud588\ub2e4. \ub354 \uc5b4\ub824\uc6b4 \uc77c\ub85c \ub4dc\ub7ec\ub09c \uc0ac\uc2e4\uc740, \ub3c5\uc77c \uc5f0\ubc29\uc758 \ub450 \uac15\ub300\uad6d\uc778 \ud504\ub85c\uc774\uc13c\uacfc \ud2b9\ud788 \uc624\uc2a4\ud2b8\ub9ac\uc544\uc758 \ucee4\ub2e4\ub780 \ubd80\ubd84\uc740 \ub3c5\uc77c\uc5b4 \uc0ac\uc6a9 \uad8c\uc5ed\uc758 \ubc16\uc5d0 \ub193\uc5ec \uc788\uc5c8\uace0, \uc774 \uc9c0\ubc29\ub4e4\uc744 \ub3c5\uc77c \uad6d\uac00\uc5d0 \ud3ec\ud568\ud560\uc9c0 \uc5ec\ubd80\ub294 \ub2e8\uc21c\ud788 \uc8fc\ubbfc\uc758 \ubbfc\uc871\uc801 \uc815\uccb4\uc131 \ubb38\uc81c\uc5d0 \uadf8\uce58\ub294 \uac83\uc774 \uc544\ub2c8\ub77c \ub3c5\uc77c \uc601\ub0b4 \uad6d\uac00\ub4e4\uac04\uc758 \uc138\ub825 \uc2f8\uc6c0\uc774\uae30\ub3c4 \ud55c\ub2e4\ub294 \uac83\uc774\uc5c8\ub2e4. \ub300\ud45c\ub4e4\uc740 \uccb4\ucf54\uc758 \ub178\ub825\uc5d0\ub3c4 \ubd88\uad6c\ud558\uace0, \ubcf4\ud5e4\ubbf8\uc544(B\u00f6hmen)\uc640 \ubaa8\ub77c\ubc14(M\u00e4hren)\ub97c \ub3c5\uc77c \uc5f0\ubc29\uc758 \ud55c \ubd80\ubd84\uc73c\ub85c \ud560 \uac83\uc744 \uba85\ubc31\ud788 \ud558\uc600\ub2e4. \ub9c8\ucc2c\uac00\uc9c0\ub85c \uc758\uc6d0\ub4e4\uc740 1848\ub144 \ub300\ud3f4\ub780\ub4dc \ubd09\uae30\ub97c \uc774\uc720\ub85c \ud3ec\uc820 \uc9c0\ubc29\uc744 \ud3b8\uc785\uc2dc\ud0ac \uac83\uc744 \uacb0\uc758\ud558\uc600\ub2e4. \ud3ec\uc820\uc740 \ubcf8\ub798 \ud3f4\ub780\ub4dc\uc758 \uc77c\ubd80\uc600\uc73c\ub098, \ud3f4\ub780\ub4dc \ubd84\ud560\uc73c\ub85c \ud504\ub85c\uc774\uc13c\uc758 \uc9c0\ubc29\uc774 \ub41c \uacf3\uc774\uc5c8\ub2e4. \ud3ec\uc5b4\uba54\ub974\uce20 \uc2dc\uae30\uc5d0 \ud3f4\ub780\ub4dc\ub294 \ubbfc\uc871 \ud574\ubc29\uc758 \uac00\ub2a5\uc131\uc744 \ubcf4\uace0 \ud658\ud76c\ud558\uc600\uc9c0\ub9cc \ud3ec\uc820 \uc9c0\ubc29\uc758 \ub3c5\uc77c \ud3b8\uc785\uc774\ub77c\ub294 \uacb0\uc815\uc740 \uc9c4\ubcf4 \uc9c4\uc601\ub9c8\uc800\ub3c4 \ud3f4\ub780\ub4dc \ubbfc\uc871 \ud574\ubc29\ubcf4\ub2e4\ub294 \ub3c5\uc77c \uad6d\ubbfc\uad6d\uac00 \uc0ac\uc0c1\uc744 \ub354 \ud3b8\uc560\ud55c\ub2e4\ub294 \uac83\uc744 \ubcf4\uc5ec\uc8fc\uc5c8\ub2e4."} {"question": "\uc2a4\uc704\uc2a4\uad70\uc774 \uac16\ucd98 \uc9c4\ud615\uc758 \uba85\uce6d\uc740?", "paragraph": "\ud504\ub791\uc2a4\uc758 \uac00\uc6b4\ub370 \ub300\ud615\uc5d0\uc11c \ud3ec\ub300\uac00 \ub2e4\uc2dc \uc870\ub9bd\ub418\uc5c8\ub2e4. \uadf8\ub4e4\uacfc \ubc18\ub300\ub85c \uc2a4\uc704\uc2a4\uad70\uc740 \uac70\ub300\ud55c \ud314\ub791\ud06c\uc2a4 \uc9c4\ud615\uc744 \ub2e4\uc2dc \uac16\ucd94\uc5c8\ub2e4. \uc800\ub141 \uc804\uc5d0 \uc6a9\uae30\ub97c \uc5bb\uc740 \uc2a4\uc704\uc2a4\uad70\uc740 \ub2e4\uc2dc \ud55c\ubc88 \ucc3d\uc744 \ub0b4\ub824 \ud504\ub791\uc2a4 \ud3ec\ubcd1\uc5d0 \ub3cc\uc9c4\uc744 \uac00\ud588\ub2e4. \uc774\ub54c\uc5d0 \ud3ec\ub300\ub294 \uadf8\ub4e4\uc5d0\uac8c \uc900\ube44\uac00 \ub418\uc5b4 \uc788\uc5c8\ub2e4. \uc218 \ub9ce\uc740 \ub300\ud3ec\uac00 \uc3d8\uc544\uc838 \ucc9c\ucc9c\ud788 \uc9c4\uad70\ud558\ub358 \uc2a4\uc704\uc2a4\uad70\uc5d0\uac8c \uae4a\uc774 \ud53c \uc6c5\ub369\uc774\ub97c \ub9cc\ub4e4\uc5c8\ub2e4. \ud558\uc9c0\ub9cc \uae30\uc138\uac00 \uaebe\uc774\uc9c0 \uc54a\uc740 \uc2a4\uc704\uc2a4\uad70\uc740 \uacc4\uc18d\uc5d0\uc11c \ud504\ub791\uc2a4 \ubcd1\ub825 \ucabd\uc5d0 \uc811\uadfc\ud558\uc5ec \uc55e\uc73c\ub85c \ubc00\uc5b4\ub0c8\uc9c0. \ub2e4\uc2dc \uc218\ube44\ud558\ub358 \ub780\uce20\ud06c\ub124\ud750\ud2b8\uac00 \ubc00\ub824\ub098\uac14\uc9c0\ub9cc; \uc218 \ub9ce\uc740 \ub300\ud3ec\uc758 \uc0ac\uaca9\uc73c\ub85c \uc2a4\uc704\uc2a4\uad70\uc774 \ub354 \uc55e\uc73c\ub85c \uc9c4\uaca9\ud574\uc624\ub294\uac78 \ub9c9\uc558\ub2e4. \uc774\ub54c\ub294 \ubc14\uc57c\ub974\uac00 \uc774\ub044\ub294 \ub610\ub2e4\ub978 \ud504\ub791\uc2a4 \uae30\ubcd1\ub300\uac00 \ub3cc\uc9c4\ud558\uc5ec \uacf5\uaca9\ud574\uc624\ub358 \uc2a4\uc704\uc2a4\uad70\uc774 \uc774 \uc9c0\uc810\uc744 \ud3ec\uae30\ud558\uac8c \ud558\uc600\ub2e4.", "answer": "\ud314\ub791\ud06c\uc2a4", "sentence": "\uadf8\ub4e4\uacfc \ubc18\ub300\ub85c \uc2a4\uc704\uc2a4\uad70\uc740 \uac70\ub300\ud55c \ud314\ub791\ud06c\uc2a4 \uc9c4\ud615\uc744 \ub2e4\uc2dc \uac16\ucd94\uc5c8\ub2e4.", "paragraph_sentence": "\ud504\ub791\uc2a4\uc758 \uac00\uc6b4\ub370 \ub300\ud615\uc5d0\uc11c \ud3ec\ub300\uac00 \ub2e4\uc2dc \uc870\ub9bd\ub418\uc5c8\ub2e4. \uadf8\ub4e4\uacfc \ubc18\ub300\ub85c \uc2a4\uc704\uc2a4\uad70\uc740 \uac70\ub300\ud55c \ud314\ub791\ud06c\uc2a4 \uc9c4\ud615\uc744 \ub2e4\uc2dc \uac16\ucd94\uc5c8\ub2e4. \uc800\ub141 \uc804\uc5d0 \uc6a9\uae30\ub97c \uc5bb\uc740 \uc2a4\uc704\uc2a4\uad70\uc740 \ub2e4\uc2dc \ud55c\ubc88 \ucc3d\uc744 \ub0b4\ub824 \ud504\ub791\uc2a4 \ud3ec\ubcd1\uc5d0 \ub3cc\uc9c4\uc744 \uac00\ud588\ub2e4. \uc774\ub54c\uc5d0 \ud3ec\ub300\ub294 \uadf8\ub4e4\uc5d0\uac8c \uc900\ube44\uac00 \ub418\uc5b4 \uc788\uc5c8\ub2e4. \uc218 \ub9ce\uc740 \ub300\ud3ec\uac00 \uc3d8\uc544\uc838 \ucc9c\ucc9c\ud788 \uc9c4\uad70\ud558\ub358 \uc2a4\uc704\uc2a4\uad70\uc5d0\uac8c \uae4a\uc774 \ud53c \uc6c5\ub369\uc774\ub97c \ub9cc\ub4e4\uc5c8\ub2e4. \ud558\uc9c0\ub9cc \uae30\uc138\uac00 \uaebe\uc774\uc9c0 \uc54a\uc740 \uc2a4\uc704\uc2a4\uad70\uc740 \uacc4\uc18d\uc5d0\uc11c \ud504\ub791\uc2a4 \ubcd1\ub825 \ucabd\uc5d0 \uc811\uadfc\ud558\uc5ec \uc55e\uc73c\ub85c \ubc00\uc5b4\ub0c8\uc9c0. \ub2e4\uc2dc \uc218\ube44\ud558\ub358 \ub780\uce20\ud06c\ub124\ud750\ud2b8\uac00 \ubc00\ub824\ub098\uac14\uc9c0\ub9cc; \uc218 \ub9ce\uc740 \ub300\ud3ec\uc758 \uc0ac\uaca9\uc73c\ub85c \uc2a4\uc704\uc2a4\uad70\uc774 \ub354 \uc55e\uc73c\ub85c \uc9c4\uaca9\ud574\uc624\ub294\uac78 \ub9c9\uc558\ub2e4. \uc774\ub54c\ub294 \ubc14\uc57c\ub974\uac00 \uc774\ub044\ub294 \ub610\ub2e4\ub978 \ud504\ub791\uc2a4 \uae30\ubcd1\ub300\uac00 \ub3cc\uc9c4\ud558\uc5ec \uacf5\uaca9\ud574\uc624\ub358 \uc2a4\uc704\uc2a4\uad70\uc774 \uc774 \uc9c0\uc810\uc744 \ud3ec\uae30\ud558\uac8c \ud558\uc600\ub2e4.", "paragraph_answer": "\ud504\ub791\uc2a4\uc758 \uac00\uc6b4\ub370 \ub300\ud615\uc5d0\uc11c \ud3ec\ub300\uac00 \ub2e4\uc2dc \uc870\ub9bd\ub418\uc5c8\ub2e4. \uadf8\ub4e4\uacfc \ubc18\ub300\ub85c \uc2a4\uc704\uc2a4\uad70\uc740 \uac70\ub300\ud55c \ud314\ub791\ud06c\uc2a4 \uc9c4\ud615\uc744 \ub2e4\uc2dc \uac16\ucd94\uc5c8\ub2e4. \uc800\ub141 \uc804\uc5d0 \uc6a9\uae30\ub97c \uc5bb\uc740 \uc2a4\uc704\uc2a4\uad70\uc740 \ub2e4\uc2dc \ud55c\ubc88 \ucc3d\uc744 \ub0b4\ub824 \ud504\ub791\uc2a4 \ud3ec\ubcd1\uc5d0 \ub3cc\uc9c4\uc744 \uac00\ud588\ub2e4. \uc774\ub54c\uc5d0 \ud3ec\ub300\ub294 \uadf8\ub4e4\uc5d0\uac8c \uc900\ube44\uac00 \ub418\uc5b4 \uc788\uc5c8\ub2e4. \uc218 \ub9ce\uc740 \ub300\ud3ec\uac00 \uc3d8\uc544\uc838 \ucc9c\ucc9c\ud788 \uc9c4\uad70\ud558\ub358 \uc2a4\uc704\uc2a4\uad70\uc5d0\uac8c \uae4a\uc774 \ud53c \uc6c5\ub369\uc774\ub97c \ub9cc\ub4e4\uc5c8\ub2e4. \ud558\uc9c0\ub9cc \uae30\uc138\uac00 \uaebe\uc774\uc9c0 \uc54a\uc740 \uc2a4\uc704\uc2a4\uad70\uc740 \uacc4\uc18d\uc5d0\uc11c \ud504\ub791\uc2a4 \ubcd1\ub825 \ucabd\uc5d0 \uc811\uadfc\ud558\uc5ec \uc55e\uc73c\ub85c \ubc00\uc5b4\ub0c8\uc9c0. \ub2e4\uc2dc \uc218\ube44\ud558\ub358 \ub780\uce20\ud06c\ub124\ud750\ud2b8\uac00 \ubc00\ub824\ub098\uac14\uc9c0\ub9cc; \uc218 \ub9ce\uc740 \ub300\ud3ec\uc758 \uc0ac\uaca9\uc73c\ub85c \uc2a4\uc704\uc2a4\uad70\uc774 \ub354 \uc55e\uc73c\ub85c \uc9c4\uaca9\ud574\uc624\ub294\uac78 \ub9c9\uc558\ub2e4. \uc774\ub54c\ub294 \ubc14\uc57c\ub974\uac00 \uc774\ub044\ub294 \ub610\ub2e4\ub978 \ud504\ub791\uc2a4 \uae30\ubcd1\ub300\uac00 \ub3cc\uc9c4\ud558\uc5ec \uacf5\uaca9\ud574\uc624\ub358 \uc2a4\uc704\uc2a4\uad70\uc774 \uc774 \uc9c0\uc810\uc744 \ud3ec\uae30\ud558\uac8c \ud558\uc600\ub2e4.", "sentence_answer": "\uadf8\ub4e4\uacfc \ubc18\ub300\ub85c \uc2a4\uc704\uc2a4\uad70\uc740 \uac70\ub300\ud55c \ud314\ub791\ud06c\uc2a4 \uc9c4\ud615\uc744 \ub2e4\uc2dc \uac16\ucd94\uc5c8\ub2e4.", "paragraph_id": "6316177-2-0", "paragraph_question": "question: \uc2a4\uc704\uc2a4\uad70\uc774 \uac16\ucd98 \uc9c4\ud615\uc758 \uba85\uce6d\uc740?, context: \ud504\ub791\uc2a4\uc758 \uac00\uc6b4\ub370 \ub300\ud615\uc5d0\uc11c \ud3ec\ub300\uac00 \ub2e4\uc2dc \uc870\ub9bd\ub418\uc5c8\ub2e4. \uadf8\ub4e4\uacfc \ubc18\ub300\ub85c \uc2a4\uc704\uc2a4\uad70\uc740 \uac70\ub300\ud55c \ud314\ub791\ud06c\uc2a4 \uc9c4\ud615\uc744 \ub2e4\uc2dc \uac16\ucd94\uc5c8\ub2e4. \uc800\ub141 \uc804\uc5d0 \uc6a9\uae30\ub97c \uc5bb\uc740 \uc2a4\uc704\uc2a4\uad70\uc740 \ub2e4\uc2dc \ud55c\ubc88 \ucc3d\uc744 \ub0b4\ub824 \ud504\ub791\uc2a4 \ud3ec\ubcd1\uc5d0 \ub3cc\uc9c4\uc744 \uac00\ud588\ub2e4. \uc774\ub54c\uc5d0 \ud3ec\ub300\ub294 \uadf8\ub4e4\uc5d0\uac8c \uc900\ube44\uac00 \ub418\uc5b4 \uc788\uc5c8\ub2e4. \uc218 \ub9ce\uc740 \ub300\ud3ec\uac00 \uc3d8\uc544\uc838 \ucc9c\ucc9c\ud788 \uc9c4\uad70\ud558\ub358 \uc2a4\uc704\uc2a4\uad70\uc5d0\uac8c \uae4a\uc774 \ud53c \uc6c5\ub369\uc774\ub97c \ub9cc\ub4e4\uc5c8\ub2e4. \ud558\uc9c0\ub9cc \uae30\uc138\uac00 \uaebe\uc774\uc9c0 \uc54a\uc740 \uc2a4\uc704\uc2a4\uad70\uc740 \uacc4\uc18d\uc5d0\uc11c \ud504\ub791\uc2a4 \ubcd1\ub825 \ucabd\uc5d0 \uc811\uadfc\ud558\uc5ec \uc55e\uc73c\ub85c \ubc00\uc5b4\ub0c8\uc9c0. \ub2e4\uc2dc \uc218\ube44\ud558\ub358 \ub780\uce20\ud06c\ub124\ud750\ud2b8\uac00 \ubc00\ub824\ub098\uac14\uc9c0\ub9cc; \uc218 \ub9ce\uc740 \ub300\ud3ec\uc758 \uc0ac\uaca9\uc73c\ub85c \uc2a4\uc704\uc2a4\uad70\uc774 \ub354 \uc55e\uc73c\ub85c \uc9c4\uaca9\ud574\uc624\ub294\uac78 \ub9c9\uc558\ub2e4. \uc774\ub54c\ub294 \ubc14\uc57c\ub974\uac00 \uc774\ub044\ub294 \ub610\ub2e4\ub978 \ud504\ub791\uc2a4 \uae30\ubcd1\ub300\uac00 \ub3cc\uc9c4\ud558\uc5ec \uacf5\uaca9\ud574\uc624\ub358 \uc2a4\uc704\uc2a4\uad70\uc774 \uc774 \uc9c0\uc810\uc744 \ud3ec\uae30\ud558\uac8c \ud558\uc600\ub2e4."} {"question": "\ud540\uc740 \ucfe0 \ud6cc\ub9b0\uc758 \uc5b4\ub5a4 \uc190\uac00\ub77d\uc744 \ubb3c\uc5b4 \ub72f\uc5c8\ub294\uac00?", "paragraph": "\ud6c4\ub300\uc758 \uc544\uc77c\ub79c\ub4dc \ubbfc\ub2f4\uc5d0\uc11c \ucfe0 \ud6cc\ub9b0\uc740 \ud540 \ub9c9 \ucfe0\uc6d4\uacfc \uc801\ub300\ud558\ub294 \uc0ac\uc545\ud55c \uac70\uc778\uc73c\ub85c \ub4f1\uc7a5\ud558\uae30\ub3c4 \ud55c\ub2e4. \uac00\uc7a5 \uc720\uba85\ud55c \uc774\uc57c\uae30\uc5d0\uc11c \ucfe0 \ud6cc\ub9b0\uc758 \ud798\uc740 \uadf8\uc758 \uac00\uc6b4\ub370\uc190\uac00\ub77d\uc5d0 \ub4e4\uc5b4\uc788\uc5c8\ub2e4. \ud540\uc744 \uc4f0\ub7ec\ub728\ub9ac\uace0 \uc2f6\uc5c8\ub358 \ucfe0 \ud6cc\ub9b0\uc740 \ud540\uc758 \uc9d1\uc73c\ub85c \ucc3e\uc544\uac14\ub2e4. \uc774\ub54c \ud540\uc740 \uc544\uae30\ub85c \ubcc0\uc7a5\ud558\uace0 \uc788\uc5c8\uace0, \ud540\uc758 \uc544\ub0b4 \uc624\uc624\ub098\ub294 \uacfc\uc790\ub97c \uad7d\uace0 \uc788\uc5c8\ub2e4. \uacfc\uc790 \uc911 \uba87 \uac1c\ub294 \ubc88\ucca0\uc774 \uc548\uc5d0 \ub4e4\uc5b4\uc788\uc5c8\uace0 \ub098\uba38\uc9c0\ub294 \uadf8\ub807\uc9c0 \uc54a\uc558\ub2e4. \ucfe0 \ud6cc\ub9b0\uc740 \uacfc\uc790\ub97c \uc539\uc744 \uc218\uac00 \uc5c6\uc5c8\ub294\ub370(\uc548\uc5d0 \ucca0\uc870\uac01\uc774 \ub4e4\uc5b4\uc788\uc73c\ubbc0\ub85c), \uc544\uae30\uac00 \uacfc\uc790\ub97c \uc539\uc5b4\uba39\ub294 \uac83\uc744 \ubcf4\uace0(\ud540\uc758 \uacfc\uc790\ub294 \ucca0\uc870\uac01\uc774 \uc5c6\uc73c\ubbc0\ub85c) \uae5c\uc9dd \ub180\ub790\ub2e4. \uc544\uae30\uc758 \uc774\ube68\uc774 \uc5bc\ub9c8\ub098 \ub0a0\uce74\ub85c\uc6b4\uc9c0 \ubcf4\uaca0\ub2e4\uace0 \ucfe0 \ud6cc\ub9b0\uc740 \ud540\uc758 \uc785\uc744 \uc0b4\ud3c8\ub294\ub370, \uc774 \ub54c\ub97c \ub178\ub824 \ud540\uc740 \ucfe0 \ud6cc\ub9b0\uc758 \uac00\uc6b4\ub370\uc190\uac00\ub77d\uc744 \ubb3c\uc5b4\ub72f\uc5b4 \ub04a\uc5b4\ub0c8\ub2e4. \uadf8\ub7ec\uc790 \ucfe0 \ud6cc\ub9b0\uc740 \ud798\ub3c4 \uc783\uace0 \ubab8\uc9d1\ub3c4 \uc791\uac8c \ucabc\uadf8\ub77c\ub4e4\uc5b4 \ubc84\ub838\ub2e4.", "answer": "\uac00\uc6b4\ub370\uc190\uac00\ub77d", "sentence": "\uac00\uc7a5 \uc720\uba85\ud55c \uc774\uc57c\uae30\uc5d0\uc11c \ucfe0 \ud6cc\ub9b0\uc758 \ud798\uc740 \uadf8\uc758 \uac00\uc6b4\ub370\uc190\uac00\ub77d \uc5d0", "paragraph_sentence": "\ud6c4\ub300\uc758 \uc544\uc77c\ub79c\ub4dc \ubbfc\ub2f4\uc5d0\uc11c \ucfe0 \ud6cc\ub9b0\uc740 \ud540 \ub9c9 \ucfe0\uc6d4\uacfc \uc801\ub300\ud558\ub294 \uc0ac\uc545\ud55c \uac70\uc778\uc73c\ub85c \ub4f1\uc7a5\ud558\uae30\ub3c4 \ud55c\ub2e4. \uac00\uc7a5 \uc720\uba85\ud55c \uc774\uc57c\uae30\uc5d0\uc11c \ucfe0 \ud6cc\ub9b0\uc758 \ud798\uc740 \uadf8\uc758 \uac00\uc6b4\ub370\uc190\uac00\ub77d \uc5d0 \ub4e4\uc5b4\uc788\uc5c8\ub2e4. \ud540\uc744 \uc4f0\ub7ec\ub728\ub9ac\uace0 \uc2f6\uc5c8\ub358 \ucfe0 \ud6cc\ub9b0\uc740 \ud540\uc758 \uc9d1\uc73c\ub85c \ucc3e\uc544\uac14\ub2e4. \uc774\ub54c \ud540\uc740 \uc544\uae30\ub85c \ubcc0\uc7a5\ud558\uace0 \uc788\uc5c8\uace0, \ud540\uc758 \uc544\ub0b4 \uc624\uc624\ub098\ub294 \uacfc\uc790\ub97c \uad7d\uace0 \uc788\uc5c8\ub2e4. \uacfc\uc790 \uc911 \uba87 \uac1c\ub294 \ubc88\ucca0\uc774 \uc548\uc5d0 \ub4e4\uc5b4\uc788\uc5c8\uace0 \ub098\uba38\uc9c0\ub294 \uadf8\ub807\uc9c0 \uc54a\uc558\ub2e4. \ucfe0 \ud6cc\ub9b0\uc740 \uacfc\uc790\ub97c \uc539\uc744 \uc218\uac00 \uc5c6\uc5c8\ub294\ub370(\uc548\uc5d0 \ucca0\uc870\uac01\uc774 \ub4e4\uc5b4\uc788\uc73c\ubbc0\ub85c), \uc544\uae30\uac00 \uacfc\uc790\ub97c \uc539\uc5b4\uba39\ub294 \uac83\uc744 \ubcf4\uace0(\ud540\uc758 \uacfc\uc790\ub294 \ucca0\uc870\uac01\uc774 \uc5c6\uc73c\ubbc0\ub85c) \uae5c\uc9dd \ub180\ub790\ub2e4. \uc544\uae30\uc758 \uc774\ube68\uc774 \uc5bc\ub9c8\ub098 \ub0a0\uce74\ub85c\uc6b4\uc9c0 \ubcf4\uaca0\ub2e4\uace0 \ucfe0 \ud6cc\ub9b0\uc740 \ud540\uc758 \uc785\uc744 \uc0b4\ud3c8\ub294\ub370, \uc774 \ub54c\ub97c \ub178\ub824 \ud540\uc740 \ucfe0 \ud6cc\ub9b0\uc758 \uac00\uc6b4\ub370\uc190\uac00\ub77d\uc744 \ubb3c\uc5b4\ub72f\uc5b4 \ub04a\uc5b4\ub0c8\ub2e4. \uadf8\ub7ec\uc790 \ucfe0 \ud6cc\ub9b0\uc740 \ud798\ub3c4 \uc783\uace0 \ubab8\uc9d1\ub3c4 \uc791\uac8c \ucabc\uadf8\ub77c\ub4e4\uc5b4 \ubc84\ub838\ub2e4.", "paragraph_answer": "\ud6c4\ub300\uc758 \uc544\uc77c\ub79c\ub4dc \ubbfc\ub2f4\uc5d0\uc11c \ucfe0 \ud6cc\ub9b0\uc740 \ud540 \ub9c9 \ucfe0\uc6d4\uacfc \uc801\ub300\ud558\ub294 \uc0ac\uc545\ud55c \uac70\uc778\uc73c\ub85c \ub4f1\uc7a5\ud558\uae30\ub3c4 \ud55c\ub2e4. \uac00\uc7a5 \uc720\uba85\ud55c \uc774\uc57c\uae30\uc5d0\uc11c \ucfe0 \ud6cc\ub9b0\uc758 \ud798\uc740 \uadf8\uc758 \uac00\uc6b4\ub370\uc190\uac00\ub77d \uc5d0 \ub4e4\uc5b4\uc788\uc5c8\ub2e4. \ud540\uc744 \uc4f0\ub7ec\ub728\ub9ac\uace0 \uc2f6\uc5c8\ub358 \ucfe0 \ud6cc\ub9b0\uc740 \ud540\uc758 \uc9d1\uc73c\ub85c \ucc3e\uc544\uac14\ub2e4. \uc774\ub54c \ud540\uc740 \uc544\uae30\ub85c \ubcc0\uc7a5\ud558\uace0 \uc788\uc5c8\uace0, \ud540\uc758 \uc544\ub0b4 \uc624\uc624\ub098\ub294 \uacfc\uc790\ub97c \uad7d\uace0 \uc788\uc5c8\ub2e4. \uacfc\uc790 \uc911 \uba87 \uac1c\ub294 \ubc88\ucca0\uc774 \uc548\uc5d0 \ub4e4\uc5b4\uc788\uc5c8\uace0 \ub098\uba38\uc9c0\ub294 \uadf8\ub807\uc9c0 \uc54a\uc558\ub2e4. \ucfe0 \ud6cc\ub9b0\uc740 \uacfc\uc790\ub97c \uc539\uc744 \uc218\uac00 \uc5c6\uc5c8\ub294\ub370(\uc548\uc5d0 \ucca0\uc870\uac01\uc774 \ub4e4\uc5b4\uc788\uc73c\ubbc0\ub85c), \uc544\uae30\uac00 \uacfc\uc790\ub97c \uc539\uc5b4\uba39\ub294 \uac83\uc744 \ubcf4\uace0(\ud540\uc758 \uacfc\uc790\ub294 \ucca0\uc870\uac01\uc774 \uc5c6\uc73c\ubbc0\ub85c) \uae5c\uc9dd \ub180\ub790\ub2e4. \uc544\uae30\uc758 \uc774\ube68\uc774 \uc5bc\ub9c8\ub098 \ub0a0\uce74\ub85c\uc6b4\uc9c0 \ubcf4\uaca0\ub2e4\uace0 \ucfe0 \ud6cc\ub9b0\uc740 \ud540\uc758 \uc785\uc744 \uc0b4\ud3c8\ub294\ub370, \uc774 \ub54c\ub97c \ub178\ub824 \ud540\uc740 \ucfe0 \ud6cc\ub9b0\uc758 \uac00\uc6b4\ub370\uc190\uac00\ub77d\uc744 \ubb3c\uc5b4\ub72f\uc5b4 \ub04a\uc5b4\ub0c8\ub2e4. \uadf8\ub7ec\uc790 \ucfe0 \ud6cc\ub9b0\uc740 \ud798\ub3c4 \uc783\uace0 \ubab8\uc9d1\ub3c4 \uc791\uac8c \ucabc\uadf8\ub77c\ub4e4\uc5b4 \ubc84\ub838\ub2e4.", "sentence_answer": "\uac00\uc7a5 \uc720\uba85\ud55c \uc774\uc57c\uae30\uc5d0\uc11c \ucfe0 \ud6cc\ub9b0\uc758 \ud798\uc740 \uadf8\uc758 \uac00\uc6b4\ub370\uc190\uac00\ub77d \uc5d0", "paragraph_id": "6562782-16-2", "paragraph_question": "question: \ud540\uc740 \ucfe0 \ud6cc\ub9b0\uc758 \uc5b4\ub5a4 \uc190\uac00\ub77d\uc744 \ubb3c\uc5b4 \ub72f\uc5c8\ub294\uac00?, context: \ud6c4\ub300\uc758 \uc544\uc77c\ub79c\ub4dc \ubbfc\ub2f4\uc5d0\uc11c \ucfe0 \ud6cc\ub9b0\uc740 \ud540 \ub9c9 \ucfe0\uc6d4\uacfc \uc801\ub300\ud558\ub294 \uc0ac\uc545\ud55c \uac70\uc778\uc73c\ub85c \ub4f1\uc7a5\ud558\uae30\ub3c4 \ud55c\ub2e4. \uac00\uc7a5 \uc720\uba85\ud55c \uc774\uc57c\uae30\uc5d0\uc11c \ucfe0 \ud6cc\ub9b0\uc758 \ud798\uc740 \uadf8\uc758 \uac00\uc6b4\ub370\uc190\uac00\ub77d\uc5d0 \ub4e4\uc5b4\uc788\uc5c8\ub2e4. \ud540\uc744 \uc4f0\ub7ec\ub728\ub9ac\uace0 \uc2f6\uc5c8\ub358 \ucfe0 \ud6cc\ub9b0\uc740 \ud540\uc758 \uc9d1\uc73c\ub85c \ucc3e\uc544\uac14\ub2e4. \uc774\ub54c \ud540\uc740 \uc544\uae30\ub85c \ubcc0\uc7a5\ud558\uace0 \uc788\uc5c8\uace0, \ud540\uc758 \uc544\ub0b4 \uc624\uc624\ub098\ub294 \uacfc\uc790\ub97c \uad7d\uace0 \uc788\uc5c8\ub2e4. \uacfc\uc790 \uc911 \uba87 \uac1c\ub294 \ubc88\ucca0\uc774 \uc548\uc5d0 \ub4e4\uc5b4\uc788\uc5c8\uace0 \ub098\uba38\uc9c0\ub294 \uadf8\ub807\uc9c0 \uc54a\uc558\ub2e4. \ucfe0 \ud6cc\ub9b0\uc740 \uacfc\uc790\ub97c \uc539\uc744 \uc218\uac00 \uc5c6\uc5c8\ub294\ub370(\uc548\uc5d0 \ucca0\uc870\uac01\uc774 \ub4e4\uc5b4\uc788\uc73c\ubbc0\ub85c), \uc544\uae30\uac00 \uacfc\uc790\ub97c \uc539\uc5b4\uba39\ub294 \uac83\uc744 \ubcf4\uace0(\ud540\uc758 \uacfc\uc790\ub294 \ucca0\uc870\uac01\uc774 \uc5c6\uc73c\ubbc0\ub85c) \uae5c\uc9dd \ub180\ub790\ub2e4. \uc544\uae30\uc758 \uc774\ube68\uc774 \uc5bc\ub9c8\ub098 \ub0a0\uce74\ub85c\uc6b4\uc9c0 \ubcf4\uaca0\ub2e4\uace0 \ucfe0 \ud6cc\ub9b0\uc740 \ud540\uc758 \uc785\uc744 \uc0b4\ud3c8\ub294\ub370, \uc774 \ub54c\ub97c \ub178\ub824 \ud540\uc740 \ucfe0 \ud6cc\ub9b0\uc758 \uac00\uc6b4\ub370\uc190\uac00\ub77d\uc744 \ubb3c\uc5b4\ub72f\uc5b4 \ub04a\uc5b4\ub0c8\ub2e4. \uadf8\ub7ec\uc790 \ucfe0 \ud6cc\ub9b0\uc740 \ud798\ub3c4 \uc783\uace0 \ubab8\uc9d1\ub3c4 \uc791\uac8c \ucabc\uadf8\ub77c\ub4e4\uc5b4 \ubc84\ub838\ub2e4."} {"question": "\ud50c\ub77c\ud14c\ub808\uc2a4\ucf54 \uac74\ucd95\uc591\uc2dd\uc758 \ub300\ud45c\uc801\uc778 \uc608\ub294?", "paragraph": "\uac74\ucd95\uc740 \ub2e4\uc74c\uacfc \uac19\uc774 16\uc138\uae30 \uae08\uc740 \uc138\uacf5\uc0ac\uc5d0 \uc758\ud574 \uc0ac\uc6a9\ub41c \uc7a5\uc2dd\uc744 \ud65c\uc6a9\ud55c \uac74\ucd95\uc591\uc2dd \ud50c\ub77c\ud14c\ub808\uc2a4\ucf54(15\uc138\uae30\uc5d0\uc11c-16\uc138\uae30\uc758 1\ubd84\uae30), \uc21c\uc218(\uc815\ud1b5)\uc8fc\uc758 \ubc0f, \uc774\ud0c8\ub9ac\uc544\ud48d (16\uc138\uae30 \uc804\ubc18\uae30), \uc5d0\ub808\ub9ac\uc544\ub178; \uc5d8\uc5d0\uc2a4\uaf2c\ub9ac\uc54c\uc758 \uac74\ucd95\uac00 \ud6c4\uc548 \ub370 \uc5d0\ub808\ub77c\uc5d0\uc11c \uc5f0\uc720\ud55c \uac74\ucd95\uc6a9\uc5b4(1559\ub144\ubd80\ud130 \ub2e4\uc74c\uc138\uae30 \uc911\ubc18) \ud06c\uac8c \uc138 \uc2dc\ub300\ub85c \ubd84\ub958\uac00 \uac00\ub2a5\ud558\ub2e4. \uc774\uc911 \uccab \ubc88\uc9f8\ub294 \ud45c\uba74\uc801\uc778 \ud615\ud0dc\uc5d0\uc11c \ub4dc\ub7ec\ub098\ub294\ub370 \uc804\ubc18\uc801\uc778 \uace0\ub515\uc591\uc2dd\uc744 \uc720\uc9c0\ud558\uba74\uc11c \ud30c\uc0ac\ub4dc\uc758 \ud654\ub824\ud55c \uc7a5\uc2dd\uc744 \ud558\ub294 \uac83\uc73c\ub85c \ub4dc\ub7ec\ub09c\ub2e4. \uc774 \uc7a5\uc2dd\uc740 \uc138\ubc00\ud558\uace0 \ud48d\ubd80\ud55c \ubaa8\uc2b5\uc744 \uc9c0\ub2c8\uace0 \uc788\ub294\ub370 \uc0b4\ub77c\ub9dd\uce74 \ub300\ud559 \uac74\ubb3c\uc758 \ud30c\uc0ac\ub4dc\uc758 \uc7a5\uc2dd\uc774 \ub300\ud45c\uc801\uc778 \uc608\ub77c\uace0 \ud560 \uc218 \uc788\ub2e4. \uc21c\uc218(\uc815\ud1b5)\uc8fc\uc758\ub294 \uc774\ud0c8\ub9ac\uc544\uc758 \uac00\uc7a5 \ud6c4\ubc18\uae30\uc758 \uce21\uba74\uc744 \ubc18\uc601\ud558\uace0 \uc788\ub294\ub370, \uc54c\ub78c\ube0c\ub77c\uc758 \uce74\ub97c\ub85c\uc2a4 5\uc138 \uad81\uc804\uc774 \ub300\ud45c\uc801\uc778 \uc608\uc774\ub2e4.", "answer": "\uc0b4\ub77c\ub9dd\uce74 \ub300\ud559 \uac74\ubb3c\uc758 \ud30c\uc0ac\ub4dc\uc758 \uc7a5\uc2dd", "sentence": "\uc774 \uc7a5\uc2dd\uc740 \uc138\ubc00\ud558\uace0 \ud48d\ubd80\ud55c \ubaa8\uc2b5\uc744 \uc9c0\ub2c8\uace0 \uc788\ub294\ub370 \uc0b4\ub77c\ub9dd\uce74 \ub300\ud559 \uac74\ubb3c\uc758 \ud30c\uc0ac\ub4dc\uc758 \uc7a5\uc2dd \uc774 \ub300\ud45c\uc801\uc778 \uc608\ub77c\uace0 \ud560 \uc218 \uc788\ub2e4.", "paragraph_sentence": "\uac74\ucd95\uc740 \ub2e4\uc74c\uacfc \uac19\uc774 16\uc138\uae30 \uae08\uc740 \uc138\uacf5\uc0ac\uc5d0 \uc758\ud574 \uc0ac\uc6a9\ub41c \uc7a5\uc2dd\uc744 \ud65c\uc6a9\ud55c \uac74\ucd95\uc591\uc2dd \ud50c\ub77c\ud14c\ub808\uc2a4\ucf54(15\uc138\uae30\uc5d0\uc11c-16\uc138\uae30\uc758 1\ubd84\uae30), \uc21c\uc218(\uc815\ud1b5)\uc8fc\uc758 \ubc0f, \uc774\ud0c8\ub9ac\uc544\ud48d (16\uc138\uae30 \uc804\ubc18\uae30), \uc5d0\ub808\ub9ac\uc544\ub178; \uc5d8\uc5d0\uc2a4\uaf2c\ub9ac\uc54c\uc758 \uac74\ucd95\uac00 \ud6c4\uc548 \ub370 \uc5d0\ub808\ub77c\uc5d0\uc11c \uc5f0\uc720\ud55c \uac74\ucd95\uc6a9\uc5b4(1559\ub144\ubd80\ud130 \ub2e4\uc74c\uc138\uae30 \uc911\ubc18) \ud06c\uac8c \uc138 \uc2dc\ub300\ub85c \ubd84\ub958\uac00 \uac00\ub2a5\ud558\ub2e4. \uc774\uc911 \uccab \ubc88\uc9f8\ub294 \ud45c\uba74\uc801\uc778 \ud615\ud0dc\uc5d0\uc11c \ub4dc\ub7ec\ub098\ub294\ub370 \uc804\ubc18\uc801\uc778 \uace0\ub515\uc591\uc2dd\uc744 \uc720\uc9c0\ud558\uba74\uc11c \ud30c\uc0ac\ub4dc\uc758 \ud654\ub824\ud55c \uc7a5\uc2dd\uc744 \ud558\ub294 \uac83\uc73c\ub85c \ub4dc\ub7ec\ub09c\ub2e4. \uc774 \uc7a5\uc2dd\uc740 \uc138\ubc00\ud558\uace0 \ud48d\ubd80\ud55c \ubaa8\uc2b5\uc744 \uc9c0\ub2c8\uace0 \uc788\ub294\ub370 \uc0b4\ub77c\ub9dd\uce74 \ub300\ud559 \uac74\ubb3c\uc758 \ud30c\uc0ac\ub4dc\uc758 \uc7a5\uc2dd \uc774 \ub300\ud45c\uc801\uc778 \uc608\ub77c\uace0 \ud560 \uc218 \uc788\ub2e4. \uc21c\uc218(\uc815\ud1b5)\uc8fc\uc758\ub294 \uc774\ud0c8\ub9ac\uc544\uc758 \uac00\uc7a5 \ud6c4\ubc18\uae30\uc758 \uce21\uba74\uc744 \ubc18\uc601\ud558\uace0 \uc788\ub294\ub370, \uc54c\ub78c\ube0c\ub77c\uc758 \uce74\ub97c\ub85c\uc2a4 5\uc138 \uad81\uc804\uc774 \ub300\ud45c\uc801\uc778 \uc608\uc774\ub2e4.", "paragraph_answer": "\uac74\ucd95\uc740 \ub2e4\uc74c\uacfc \uac19\uc774 16\uc138\uae30 \uae08\uc740 \uc138\uacf5\uc0ac\uc5d0 \uc758\ud574 \uc0ac\uc6a9\ub41c \uc7a5\uc2dd\uc744 \ud65c\uc6a9\ud55c \uac74\ucd95\uc591\uc2dd \ud50c\ub77c\ud14c\ub808\uc2a4\ucf54(15\uc138\uae30\uc5d0\uc11c-16\uc138\uae30\uc758 1\ubd84\uae30), \uc21c\uc218(\uc815\ud1b5)\uc8fc\uc758 \ubc0f, \uc774\ud0c8\ub9ac\uc544\ud48d (16\uc138\uae30 \uc804\ubc18\uae30), \uc5d0\ub808\ub9ac\uc544\ub178; \uc5d8\uc5d0\uc2a4\uaf2c\ub9ac\uc54c\uc758 \uac74\ucd95\uac00 \ud6c4\uc548 \ub370 \uc5d0\ub808\ub77c\uc5d0\uc11c \uc5f0\uc720\ud55c \uac74\ucd95\uc6a9\uc5b4(1559\ub144\ubd80\ud130 \ub2e4\uc74c\uc138\uae30 \uc911\ubc18) \ud06c\uac8c \uc138 \uc2dc\ub300\ub85c \ubd84\ub958\uac00 \uac00\ub2a5\ud558\ub2e4. \uc774\uc911 \uccab \ubc88\uc9f8\ub294 \ud45c\uba74\uc801\uc778 \ud615\ud0dc\uc5d0\uc11c \ub4dc\ub7ec\ub098\ub294\ub370 \uc804\ubc18\uc801\uc778 \uace0\ub515\uc591\uc2dd\uc744 \uc720\uc9c0\ud558\uba74\uc11c \ud30c\uc0ac\ub4dc\uc758 \ud654\ub824\ud55c \uc7a5\uc2dd\uc744 \ud558\ub294 \uac83\uc73c\ub85c \ub4dc\ub7ec\ub09c\ub2e4. \uc774 \uc7a5\uc2dd\uc740 \uc138\ubc00\ud558\uace0 \ud48d\ubd80\ud55c \ubaa8\uc2b5\uc744 \uc9c0\ub2c8\uace0 \uc788\ub294\ub370 \uc0b4\ub77c\ub9dd\uce74 \ub300\ud559 \uac74\ubb3c\uc758 \ud30c\uc0ac\ub4dc\uc758 \uc7a5\uc2dd \uc774 \ub300\ud45c\uc801\uc778 \uc608\ub77c\uace0 \ud560 \uc218 \uc788\ub2e4. \uc21c\uc218(\uc815\ud1b5)\uc8fc\uc758\ub294 \uc774\ud0c8\ub9ac\uc544\uc758 \uac00\uc7a5 \ud6c4\ubc18\uae30\uc758 \uce21\uba74\uc744 \ubc18\uc601\ud558\uace0 \uc788\ub294\ub370, \uc54c\ub78c\ube0c\ub77c\uc758 \uce74\ub97c\ub85c\uc2a4 5\uc138 \uad81\uc804\uc774 \ub300\ud45c\uc801\uc778 \uc608\uc774\ub2e4.", "sentence_answer": "\uc774 \uc7a5\uc2dd\uc740 \uc138\ubc00\ud558\uace0 \ud48d\ubd80\ud55c \ubaa8\uc2b5\uc744 \uc9c0\ub2c8\uace0 \uc788\ub294\ub370 \uc0b4\ub77c\ub9dd\uce74 \ub300\ud559 \uac74\ubb3c\uc758 \ud30c\uc0ac\ub4dc\uc758 \uc7a5\uc2dd \uc774 \ub300\ud45c\uc801\uc778 \uc608\ub77c\uace0 \ud560 \uc218 \uc788\ub2e4.", "paragraph_id": "6529061-4-0", "paragraph_question": "question: \ud50c\ub77c\ud14c\ub808\uc2a4\ucf54 \uac74\ucd95\uc591\uc2dd\uc758 \ub300\ud45c\uc801\uc778 \uc608\ub294?, context: \uac74\ucd95\uc740 \ub2e4\uc74c\uacfc \uac19\uc774 16\uc138\uae30 \uae08\uc740 \uc138\uacf5\uc0ac\uc5d0 \uc758\ud574 \uc0ac\uc6a9\ub41c \uc7a5\uc2dd\uc744 \ud65c\uc6a9\ud55c \uac74\ucd95\uc591\uc2dd \ud50c\ub77c\ud14c\ub808\uc2a4\ucf54(15\uc138\uae30\uc5d0\uc11c-16\uc138\uae30\uc758 1\ubd84\uae30), \uc21c\uc218(\uc815\ud1b5)\uc8fc\uc758 \ubc0f, \uc774\ud0c8\ub9ac\uc544\ud48d (16\uc138\uae30 \uc804\ubc18\uae30), \uc5d0\ub808\ub9ac\uc544\ub178; \uc5d8\uc5d0\uc2a4\uaf2c\ub9ac\uc54c\uc758 \uac74\ucd95\uac00 \ud6c4\uc548 \ub370 \uc5d0\ub808\ub77c\uc5d0\uc11c \uc5f0\uc720\ud55c \uac74\ucd95\uc6a9\uc5b4(1559\ub144\ubd80\ud130 \ub2e4\uc74c\uc138\uae30 \uc911\ubc18) \ud06c\uac8c \uc138 \uc2dc\ub300\ub85c \ubd84\ub958\uac00 \uac00\ub2a5\ud558\ub2e4. \uc774\uc911 \uccab \ubc88\uc9f8\ub294 \ud45c\uba74\uc801\uc778 \ud615\ud0dc\uc5d0\uc11c \ub4dc\ub7ec\ub098\ub294\ub370 \uc804\ubc18\uc801\uc778 \uace0\ub515\uc591\uc2dd\uc744 \uc720\uc9c0\ud558\uba74\uc11c \ud30c\uc0ac\ub4dc\uc758 \ud654\ub824\ud55c \uc7a5\uc2dd\uc744 \ud558\ub294 \uac83\uc73c\ub85c \ub4dc\ub7ec\ub09c\ub2e4. \uc774 \uc7a5\uc2dd\uc740 \uc138\ubc00\ud558\uace0 \ud48d\ubd80\ud55c \ubaa8\uc2b5\uc744 \uc9c0\ub2c8\uace0 \uc788\ub294\ub370 \uc0b4\ub77c\ub9dd\uce74 \ub300\ud559 \uac74\ubb3c\uc758 \ud30c\uc0ac\ub4dc\uc758 \uc7a5\uc2dd\uc774 \ub300\ud45c\uc801\uc778 \uc608\ub77c\uace0 \ud560 \uc218 \uc788\ub2e4. \uc21c\uc218(\uc815\ud1b5)\uc8fc\uc758\ub294 \uc774\ud0c8\ub9ac\uc544\uc758 \uac00\uc7a5 \ud6c4\ubc18\uae30\uc758 \uce21\uba74\uc744 \ubc18\uc601\ud558\uace0 \uc788\ub294\ub370, \uc54c\ub78c\ube0c\ub77c\uc758 \uce74\ub97c\ub85c\uc2a4 5\uc138 \uad81\uc804\uc774 \ub300\ud45c\uc801\uc778 \uc608\uc774\ub2e4."} {"question": "\ubba4\uc9c0\uceec \ube4c\ub9ac \uc5d8\ub9ac\uc5b4\ud2b8\uc5d0\uc11c \uc548\ubb34\ub97c \ub2f4\ub2f9\ud55c \uac83\uc740 \ub204\uad6c\uc778\uac00?", "paragraph": "\ubba4\uc9c0\uceec \ube4c\ub9ac \uc5d8\ub9ac\uc5b4\ud2b8\ub294 \uc2dc\ub4dc\ub2c8\uc758 \uce90\ud53c\ud0c8 \uc2dc\uc5b4\ud130\uc5d0\uc11c 2007\ub144 11\uc6d4 13\uc77c \ub9c9\uc744 \uc62c\ub838\ub2e4. \ub2ec\ub4dc\ub9ac\uc640 \uc904\ub9ac\uc548 \uc704\ubc84\uac00 \uc5f0\ucd9c\uc744, \ub2ec\ub9c1\uc774 \uc548\ubb34\ub97c \ub2f4\ub2f9\ud588\ub2e4. \ub9ac\uc2a4 \ucf54\uc0ac\ucf54\uc2a4\ud0a4, \ub85c\ud074\ub780 \ub374\ud648, \ub77c\ubbf8\uc548 \ub274\ud2bc, \uadf8\ub9ac\uace0 \ub2c9 \ud2b8\uc704\ub2c8\uac00 \uc8fc\uc5f0\uc744 \ub9e1\uc558\ub2e4. \uc791\ud488\uc740 \ud070 \uc774\ubaa9\uc744 \ub04c\uc5c8\uc73c\uba70, 2008\ub144 1\uc6d4\uc5d0\ub294 \uc2dc\ub4dc\ub2c8 \uc2dc\uc5b4\ud130 \uc5b4\uc6cc\ub4dc\uc5d0\uc11c \ucd5c\uace0\uc758 \ubba4\uc9c0\uceec\uc0c1\uc744 \uc218\uc0c1\ud588\ub2e4. \ubba4\uc9c0\uceec \ube4c\ub9ac \uc5d8\ub9ac\uc5b4\ud2b8\ub294 \ub610\ud55c \ud5ec\ud504\ub9cc \uc5b4\uc6cc\ub4dc\uc5d0\uc11c \ucd5c\uace0\uc758 \ubba4\uc9c0\uceec\uc0c1, \ucd5c\uace0\uc758 \uc5f0\ucd9c\uc0c1, \ucd5c\uace0\uc758 \uc548\ubb34\uc0c1, \uc5ec\uc6b0\uc8fc\uc5f0\uc0c1 (\uc81c\ub098\ube44\ube0c \ub808\ubaac, \uc70c\ud0a8\uc2a8 \ubd80\uc778 \uc5ed), \uadf8\ub9ac\uace0 \ube4c\ub9ac \uc5d8\ub9ac\uc5b4\ud2b8\ub97c \uc5f0\uae30 \ud55c \ub124 \uba85\uc758 \uc18c\ub144\ub4e4 \ubaa8\ub450 \ub0a8\uc6b0\uc8fc\uc5f0\uc0c1 \ub3c4\ud569 7\uac1c \ubd80\ubb38\uc744 \uc11d\uad8c\ud588\ub2e4. \uacf5\uc5f0\uc740 2008\ub144 11\uc6d4 9\uc77c \uc2dc\ub4dc\ub2c8\uc5d0\uc11c \uc5ec\ub35f\uba85\uc758 \ubaa8\ub4e0 \ube4c\ub9ac\uc758 \ud53c\ub0a0\ub808\uc640 \ud568\uaed8 \ub9c9\uc744 \ub0b4\ub838\ub2e4.", "answer": "\ub2ec\ub9c1", "sentence": "\ub2ec\ub4dc\ub9ac\uc640 \uc904\ub9ac\uc548 \uc704\ubc84\uac00 \uc5f0\ucd9c\uc744, \ub2ec\ub9c1 \uc774 \uc548\ubb34\ub97c \ub2f4\ub2f9\ud588\ub2e4.", "paragraph_sentence": "\ubba4\uc9c0\uceec \ube4c\ub9ac \uc5d8\ub9ac\uc5b4\ud2b8\ub294 \uc2dc\ub4dc\ub2c8\uc758 \uce90\ud53c\ud0c8 \uc2dc\uc5b4\ud130\uc5d0\uc11c 2007\ub144 11\uc6d4 13\uc77c \ub9c9\uc744 \uc62c\ub838\ub2e4. \ub2ec\ub4dc\ub9ac\uc640 \uc904\ub9ac\uc548 \uc704\ubc84\uac00 \uc5f0\ucd9c\uc744, \ub2ec\ub9c1 \uc774 \uc548\ubb34\ub97c \ub2f4\ub2f9\ud588\ub2e4. \ub9ac\uc2a4 \ucf54\uc0ac\ucf54\uc2a4\ud0a4, \ub85c\ud074\ub780 \ub374\ud648, \ub77c\ubbf8\uc548 \ub274\ud2bc, \uadf8\ub9ac\uace0 \ub2c9 \ud2b8\uc704\ub2c8\uac00 \uc8fc\uc5f0\uc744 \ub9e1\uc558\ub2e4. \uc791\ud488\uc740 \ud070 \uc774\ubaa9\uc744 \ub04c\uc5c8\uc73c\uba70, 2008\ub144 1\uc6d4\uc5d0\ub294 \uc2dc\ub4dc\ub2c8 \uc2dc\uc5b4\ud130 \uc5b4\uc6cc\ub4dc\uc5d0\uc11c \ucd5c\uace0\uc758 \ubba4\uc9c0\uceec\uc0c1\uc744 \uc218\uc0c1\ud588\ub2e4. \ubba4\uc9c0\uceec \ube4c\ub9ac \uc5d8\ub9ac\uc5b4\ud2b8\ub294 \ub610\ud55c \ud5ec\ud504\ub9cc \uc5b4\uc6cc\ub4dc\uc5d0\uc11c \ucd5c\uace0\uc758 \ubba4\uc9c0\uceec\uc0c1, \ucd5c\uace0\uc758 \uc5f0\ucd9c\uc0c1, \ucd5c\uace0\uc758 \uc548\ubb34\uc0c1, \uc5ec\uc6b0\uc8fc\uc5f0\uc0c1 (\uc81c\ub098\ube44\ube0c \ub808\ubaac, \uc70c\ud0a8\uc2a8 \ubd80\uc778 \uc5ed), \uadf8\ub9ac\uace0 \ube4c\ub9ac \uc5d8\ub9ac\uc5b4\ud2b8\ub97c \uc5f0\uae30 \ud55c \ub124 \uba85\uc758 \uc18c\ub144\ub4e4 \ubaa8\ub450 \ub0a8\uc6b0\uc8fc\uc5f0\uc0c1 \ub3c4\ud569 7\uac1c \ubd80\ubb38\uc744 \uc11d\uad8c\ud588\ub2e4. \uacf5\uc5f0\uc740 2008\ub144 11\uc6d4 9\uc77c \uc2dc\ub4dc\ub2c8\uc5d0\uc11c \uc5ec\ub35f\uba85\uc758 \ubaa8\ub4e0 \ube4c\ub9ac\uc758 \ud53c\ub0a0\ub808\uc640 \ud568\uaed8 \ub9c9\uc744 \ub0b4\ub838\ub2e4.", "paragraph_answer": "\ubba4\uc9c0\uceec \ube4c\ub9ac \uc5d8\ub9ac\uc5b4\ud2b8\ub294 \uc2dc\ub4dc\ub2c8\uc758 \uce90\ud53c\ud0c8 \uc2dc\uc5b4\ud130\uc5d0\uc11c 2007\ub144 11\uc6d4 13\uc77c \ub9c9\uc744 \uc62c\ub838\ub2e4. \ub2ec\ub4dc\ub9ac\uc640 \uc904\ub9ac\uc548 \uc704\ubc84\uac00 \uc5f0\ucd9c\uc744, \ub2ec\ub9c1 \uc774 \uc548\ubb34\ub97c \ub2f4\ub2f9\ud588\ub2e4. \ub9ac\uc2a4 \ucf54\uc0ac\ucf54\uc2a4\ud0a4, \ub85c\ud074\ub780 \ub374\ud648, \ub77c\ubbf8\uc548 \ub274\ud2bc, \uadf8\ub9ac\uace0 \ub2c9 \ud2b8\uc704\ub2c8\uac00 \uc8fc\uc5f0\uc744 \ub9e1\uc558\ub2e4. \uc791\ud488\uc740 \ud070 \uc774\ubaa9\uc744 \ub04c\uc5c8\uc73c\uba70, 2008\ub144 1\uc6d4\uc5d0\ub294 \uc2dc\ub4dc\ub2c8 \uc2dc\uc5b4\ud130 \uc5b4\uc6cc\ub4dc\uc5d0\uc11c \ucd5c\uace0\uc758 \ubba4\uc9c0\uceec\uc0c1\uc744 \uc218\uc0c1\ud588\ub2e4. \ubba4\uc9c0\uceec \ube4c\ub9ac \uc5d8\ub9ac\uc5b4\ud2b8\ub294 \ub610\ud55c \ud5ec\ud504\ub9cc \uc5b4\uc6cc\ub4dc\uc5d0\uc11c \ucd5c\uace0\uc758 \ubba4\uc9c0\uceec\uc0c1, \ucd5c\uace0\uc758 \uc5f0\ucd9c\uc0c1, \ucd5c\uace0\uc758 \uc548\ubb34\uc0c1, \uc5ec\uc6b0\uc8fc\uc5f0\uc0c1 (\uc81c\ub098\ube44\ube0c \ub808\ubaac, \uc70c\ud0a8\uc2a8 \ubd80\uc778 \uc5ed), \uadf8\ub9ac\uace0 \ube4c\ub9ac \uc5d8\ub9ac\uc5b4\ud2b8\ub97c \uc5f0\uae30 \ud55c \ub124 \uba85\uc758 \uc18c\ub144\ub4e4 \ubaa8\ub450 \ub0a8\uc6b0\uc8fc\uc5f0\uc0c1 \ub3c4\ud569 7\uac1c \ubd80\ubb38\uc744 \uc11d\uad8c\ud588\ub2e4. \uacf5\uc5f0\uc740 2008\ub144 11\uc6d4 9\uc77c \uc2dc\ub4dc\ub2c8\uc5d0\uc11c \uc5ec\ub35f\uba85\uc758 \ubaa8\ub4e0 \ube4c\ub9ac\uc758 \ud53c\ub0a0\ub808\uc640 \ud568\uaed8 \ub9c9\uc744 \ub0b4\ub838\ub2e4.", "sentence_answer": "\ub2ec\ub4dc\ub9ac\uc640 \uc904\ub9ac\uc548 \uc704\ubc84\uac00 \uc5f0\ucd9c\uc744, \ub2ec\ub9c1 \uc774 \uc548\ubb34\ub97c \ub2f4\ub2f9\ud588\ub2e4.", "paragraph_id": "6531194-1-1", "paragraph_question": "question: \ubba4\uc9c0\uceec \ube4c\ub9ac \uc5d8\ub9ac\uc5b4\ud2b8\uc5d0\uc11c \uc548\ubb34\ub97c \ub2f4\ub2f9\ud55c \uac83\uc740 \ub204\uad6c\uc778\uac00?, context: \ubba4\uc9c0\uceec \ube4c\ub9ac \uc5d8\ub9ac\uc5b4\ud2b8\ub294 \uc2dc\ub4dc\ub2c8\uc758 \uce90\ud53c\ud0c8 \uc2dc\uc5b4\ud130\uc5d0\uc11c 2007\ub144 11\uc6d4 13\uc77c \ub9c9\uc744 \uc62c\ub838\ub2e4. \ub2ec\ub4dc\ub9ac\uc640 \uc904\ub9ac\uc548 \uc704\ubc84\uac00 \uc5f0\ucd9c\uc744, \ub2ec\ub9c1\uc774 \uc548\ubb34\ub97c \ub2f4\ub2f9\ud588\ub2e4. \ub9ac\uc2a4 \ucf54\uc0ac\ucf54\uc2a4\ud0a4, \ub85c\ud074\ub780 \ub374\ud648, \ub77c\ubbf8\uc548 \ub274\ud2bc, \uadf8\ub9ac\uace0 \ub2c9 \ud2b8\uc704\ub2c8\uac00 \uc8fc\uc5f0\uc744 \ub9e1\uc558\ub2e4. \uc791\ud488\uc740 \ud070 \uc774\ubaa9\uc744 \ub04c\uc5c8\uc73c\uba70, 2008\ub144 1\uc6d4\uc5d0\ub294 \uc2dc\ub4dc\ub2c8 \uc2dc\uc5b4\ud130 \uc5b4\uc6cc\ub4dc\uc5d0\uc11c \ucd5c\uace0\uc758 \ubba4\uc9c0\uceec\uc0c1\uc744 \uc218\uc0c1\ud588\ub2e4. \ubba4\uc9c0\uceec \ube4c\ub9ac \uc5d8\ub9ac\uc5b4\ud2b8\ub294 \ub610\ud55c \ud5ec\ud504\ub9cc \uc5b4\uc6cc\ub4dc\uc5d0\uc11c \ucd5c\uace0\uc758 \ubba4\uc9c0\uceec\uc0c1, \ucd5c\uace0\uc758 \uc5f0\ucd9c\uc0c1, \ucd5c\uace0\uc758 \uc548\ubb34\uc0c1, \uc5ec\uc6b0\uc8fc\uc5f0\uc0c1 (\uc81c\ub098\ube44\ube0c \ub808\ubaac, \uc70c\ud0a8\uc2a8 \ubd80\uc778 \uc5ed), \uadf8\ub9ac\uace0 \ube4c\ub9ac \uc5d8\ub9ac\uc5b4\ud2b8\ub97c \uc5f0\uae30 \ud55c \ub124 \uba85\uc758 \uc18c\ub144\ub4e4 \ubaa8\ub450 \ub0a8\uc6b0\uc8fc\uc5f0\uc0c1 \ub3c4\ud569 7\uac1c \ubd80\ubb38\uc744 \uc11d\uad8c\ud588\ub2e4. \uacf5\uc5f0\uc740 2008\ub144 11\uc6d4 9\uc77c \uc2dc\ub4dc\ub2c8\uc5d0\uc11c \uc5ec\ub35f\uba85\uc758 \ubaa8\ub4e0 \ube4c\ub9ac\uc758 \ud53c\ub0a0\ub808\uc640 \ud568\uaed8 \ub9c9\uc744 \ub0b4\ub838\ub2e4."} {"question": "\uc601\ud654\uc758 \uc804 \uc138\uacc4\uc801 \uc218\uc775\uc740 \uc5bc\ub9c8 \uc774\uc0c1\uc778\uac00?", "paragraph": "\uc6cc\ub108 \ube0c\ub77c\ub354\uc2a4\ub294 \uc6d0\ub798 \uc774 \uc601\ud654\ub97c \uc81c400\ud68c \uac00\uc774 \ud3ec\ud06c\uc2a4\uc758 \ubc24 \uc804\ub0a0\uc778 2005\ub144 11\uc6d4 4\uc77c\uc5d0 \uac1c\ubd09\ud558\ub824 \ud588\uc73c\ub098 \uc5f0\uae30\ub418\uc5b4 2006\ub144 3\uc6d4 17\uc77c\uc5d0 \uac1c\ubd09\ud588\ub2e4. \uc804 \uc138\uacc4\uc801 \ud765\ud589 \uc218\uc775\uc740 1\uc5b5 3\ucc9c2\ubc31\ub9cc \ub2ec\ub7ec \uc774\uc0c1\uc73c\ub85c \uc0c1\uc5c5\uc801\uc73c\ub85c \uc131\uacf5\ud588\uc73c\uba70, \ud3c9\ub860\ub4e4\ub3c4 \uae0d\uc815\uc801\uc774\uc5c8\ub2e4. \uadf8\ub7ec\ub098 \uc6d0\uc791\uc790 \ubb34\uc5b4\ub294 \uc790\uc2e0\uc758 \ub9cc\ud654\ub97c \uc601\ud654\ud654\ud55c \ub2e4\ub978 \ub450 \uc601\ud654 \u300a\ud504\ub7fc \ud5ec\u300b\uacfc \u300a\uc820\ud2c0\ub9e8 \ub9ac\uadf8\u300b\uc5d0 \uc2e4\ub9dd\ud55c \ub4a4, \uc601\ud654\ub97c \ubcf4\ub294 \uac83\uc744 \uac70\ubd80\ud558\uace0 \uc601\ud654\uc5d0\uc11c \uba40\ucc0d\uc774 \uac70\ub9ac\ub97c \ub450\uc5c8\ub2e4. \uc2e4\uc81c\ub85c \uc601\ud654 \ud06c\ub808\ub527\uc5d0\ub294 \ub370\uc774\ube44\ub4dc \ub85c\uc774\ub4dc\uc758 \uc774\ub984\ub9cc \uc62c\ub77c\uac00 \uc788\ub2e4. \uc601\ud654 \uc81c\uc791\uc790\ub4e4\uc740 \uc6d0\uc791\uc5d0 \ub098\uc624\ub294 \ubb34\uc815\ubd80\uc8fc\uc758\uc801 \ud14c\ub9c8\ub4e4\uacfc \ub9c8\uc57d \uad00\ub828 \ub0b4\uc6a9\ub4e4\uc744 \uc0ad\uc81c\ud588\uace0, \uc790\uae30\ub4e4 \uc0dd\uac01\uc5d0 2006\ub144\uc758 \uad00\uac1d\ub4e4\uc5d0\uac8c \ub354 \uc640\ub2ff\uc744 \uac83 \uac19\uc740 \uac83\uc73c\ub85c \uc815\uce58\uc801 \uba54\uc2dc\uc9c0\ub3c4 \ubcc0\uacbd\ud588\ub2e4.", "answer": "1\uc5b5 3\ucc9c2\ubc31\ub9cc \ub2ec\ub7ec \uc774\uc0c1", "sentence": "\uc804 \uc138\uacc4\uc801 \ud765\ud589 \uc218\uc775\uc740 1\uc5b5 3\ucc9c2\ubc31\ub9cc \ub2ec\ub7ec \uc774\uc0c1 \uc73c\ub85c \uc0c1\uc5c5\uc801\uc73c\ub85c \uc131\uacf5\ud588\uc73c\uba70, \ud3c9\ub860\ub4e4\ub3c4 \uae0d\uc815\uc801\uc774\uc5c8\ub2e4.", "paragraph_sentence": "\uc6cc\ub108 \ube0c\ub77c\ub354\uc2a4\ub294 \uc6d0\ub798 \uc774 \uc601\ud654\ub97c \uc81c400\ud68c \uac00\uc774 \ud3ec\ud06c\uc2a4\uc758 \ubc24 \uc804\ub0a0\uc778 2005\ub144 11\uc6d4 4\uc77c\uc5d0 \uac1c\ubd09\ud558\ub824 \ud588\uc73c\ub098 \uc5f0\uae30\ub418\uc5b4 2006\ub144 3\uc6d4 17\uc77c\uc5d0 \uac1c\ubd09\ud588\ub2e4. \uc804 \uc138\uacc4\uc801 \ud765\ud589 \uc218\uc775\uc740 1\uc5b5 3\ucc9c2\ubc31\ub9cc \ub2ec\ub7ec \uc774\uc0c1 \uc73c\ub85c \uc0c1\uc5c5\uc801\uc73c\ub85c \uc131\uacf5\ud588\uc73c\uba70, \ud3c9\ub860\ub4e4\ub3c4 \uae0d\uc815\uc801\uc774\uc5c8\ub2e4. \uadf8\ub7ec\ub098 \uc6d0\uc791\uc790 \ubb34\uc5b4\ub294 \uc790\uc2e0\uc758 \ub9cc\ud654\ub97c \uc601\ud654\ud654\ud55c \ub2e4\ub978 \ub450 \uc601\ud654 \u300a\ud504\ub7fc \ud5ec\u300b\uacfc \u300a\uc820\ud2c0\ub9e8 \ub9ac\uadf8\u300b\uc5d0 \uc2e4\ub9dd\ud55c \ub4a4, \uc601\ud654\ub97c \ubcf4\ub294 \uac83\uc744 \uac70\ubd80\ud558\uace0 \uc601\ud654\uc5d0\uc11c \uba40\ucc0d\uc774 \uac70\ub9ac\ub97c \ub450\uc5c8\ub2e4. \uc2e4\uc81c\ub85c \uc601\ud654 \ud06c\ub808\ub527\uc5d0\ub294 \ub370\uc774\ube44\ub4dc \ub85c\uc774\ub4dc\uc758 \uc774\ub984\ub9cc \uc62c\ub77c\uac00 \uc788\ub2e4. \uc601\ud654 \uc81c\uc791\uc790\ub4e4\uc740 \uc6d0\uc791\uc5d0 \ub098\uc624\ub294 \ubb34\uc815\ubd80\uc8fc\uc758\uc801 \ud14c\ub9c8\ub4e4\uacfc \ub9c8\uc57d \uad00\ub828 \ub0b4\uc6a9\ub4e4\uc744 \uc0ad\uc81c\ud588\uace0, \uc790\uae30\ub4e4 \uc0dd\uac01\uc5d0 2006\ub144\uc758 \uad00\uac1d\ub4e4\uc5d0\uac8c \ub354 \uc640\ub2ff\uc744 \uac83 \uac19\uc740 \uac83\uc73c\ub85c \uc815\uce58\uc801 \uba54\uc2dc\uc9c0\ub3c4 \ubcc0\uacbd\ud588\ub2e4.", "paragraph_answer": "\uc6cc\ub108 \ube0c\ub77c\ub354\uc2a4\ub294 \uc6d0\ub798 \uc774 \uc601\ud654\ub97c \uc81c400\ud68c \uac00\uc774 \ud3ec\ud06c\uc2a4\uc758 \ubc24 \uc804\ub0a0\uc778 2005\ub144 11\uc6d4 4\uc77c\uc5d0 \uac1c\ubd09\ud558\ub824 \ud588\uc73c\ub098 \uc5f0\uae30\ub418\uc5b4 2006\ub144 3\uc6d4 17\uc77c\uc5d0 \uac1c\ubd09\ud588\ub2e4. \uc804 \uc138\uacc4\uc801 \ud765\ud589 \uc218\uc775\uc740 1\uc5b5 3\ucc9c2\ubc31\ub9cc \ub2ec\ub7ec \uc774\uc0c1 \uc73c\ub85c \uc0c1\uc5c5\uc801\uc73c\ub85c \uc131\uacf5\ud588\uc73c\uba70, \ud3c9\ub860\ub4e4\ub3c4 \uae0d\uc815\uc801\uc774\uc5c8\ub2e4. \uadf8\ub7ec\ub098 \uc6d0\uc791\uc790 \ubb34\uc5b4\ub294 \uc790\uc2e0\uc758 \ub9cc\ud654\ub97c \uc601\ud654\ud654\ud55c \ub2e4\ub978 \ub450 \uc601\ud654 \u300a\ud504\ub7fc \ud5ec\u300b\uacfc \u300a\uc820\ud2c0\ub9e8 \ub9ac\uadf8\u300b\uc5d0 \uc2e4\ub9dd\ud55c \ub4a4, \uc601\ud654\ub97c \ubcf4\ub294 \uac83\uc744 \uac70\ubd80\ud558\uace0 \uc601\ud654\uc5d0\uc11c \uba40\ucc0d\uc774 \uac70\ub9ac\ub97c \ub450\uc5c8\ub2e4. \uc2e4\uc81c\ub85c \uc601\ud654 \ud06c\ub808\ub527\uc5d0\ub294 \ub370\uc774\ube44\ub4dc \ub85c\uc774\ub4dc\uc758 \uc774\ub984\ub9cc \uc62c\ub77c\uac00 \uc788\ub2e4. \uc601\ud654 \uc81c\uc791\uc790\ub4e4\uc740 \uc6d0\uc791\uc5d0 \ub098\uc624\ub294 \ubb34\uc815\ubd80\uc8fc\uc758\uc801 \ud14c\ub9c8\ub4e4\uacfc \ub9c8\uc57d \uad00\ub828 \ub0b4\uc6a9\ub4e4\uc744 \uc0ad\uc81c\ud588\uace0, \uc790\uae30\ub4e4 \uc0dd\uac01\uc5d0 2006\ub144\uc758 \uad00\uac1d\ub4e4\uc5d0\uac8c \ub354 \uc640\ub2ff\uc744 \uac83 \uac19\uc740 \uac83\uc73c\ub85c \uc815\uce58\uc801 \uba54\uc2dc\uc9c0\ub3c4 \ubcc0\uacbd\ud588\ub2e4.", "sentence_answer": "\uc804 \uc138\uacc4\uc801 \ud765\ud589 \uc218\uc775\uc740 1\uc5b5 3\ucc9c2\ubc31\ub9cc \ub2ec\ub7ec \uc774\uc0c1 \uc73c\ub85c \uc0c1\uc5c5\uc801\uc73c\ub85c \uc131\uacf5\ud588\uc73c\uba70, \ud3c9\ub860\ub4e4\ub3c4 \uae0d\uc815\uc801\uc774\uc5c8\ub2e4.", "paragraph_id": "6583521-0-1", "paragraph_question": "question: \uc601\ud654\uc758 \uc804 \uc138\uacc4\uc801 \uc218\uc775\uc740 \uc5bc\ub9c8 \uc774\uc0c1\uc778\uac00?, context: \uc6cc\ub108 \ube0c\ub77c\ub354\uc2a4\ub294 \uc6d0\ub798 \uc774 \uc601\ud654\ub97c \uc81c400\ud68c \uac00\uc774 \ud3ec\ud06c\uc2a4\uc758 \ubc24 \uc804\ub0a0\uc778 2005\ub144 11\uc6d4 4\uc77c\uc5d0 \uac1c\ubd09\ud558\ub824 \ud588\uc73c\ub098 \uc5f0\uae30\ub418\uc5b4 2006\ub144 3\uc6d4 17\uc77c\uc5d0 \uac1c\ubd09\ud588\ub2e4. \uc804 \uc138\uacc4\uc801 \ud765\ud589 \uc218\uc775\uc740 1\uc5b5 3\ucc9c2\ubc31\ub9cc \ub2ec\ub7ec \uc774\uc0c1\uc73c\ub85c \uc0c1\uc5c5\uc801\uc73c\ub85c \uc131\uacf5\ud588\uc73c\uba70, \ud3c9\ub860\ub4e4\ub3c4 \uae0d\uc815\uc801\uc774\uc5c8\ub2e4. \uadf8\ub7ec\ub098 \uc6d0\uc791\uc790 \ubb34\uc5b4\ub294 \uc790\uc2e0\uc758 \ub9cc\ud654\ub97c \uc601\ud654\ud654\ud55c \ub2e4\ub978 \ub450 \uc601\ud654 \u300a\ud504\ub7fc \ud5ec\u300b\uacfc \u300a\uc820\ud2c0\ub9e8 \ub9ac\uadf8\u300b\uc5d0 \uc2e4\ub9dd\ud55c \ub4a4, \uc601\ud654\ub97c \ubcf4\ub294 \uac83\uc744 \uac70\ubd80\ud558\uace0 \uc601\ud654\uc5d0\uc11c \uba40\ucc0d\uc774 \uac70\ub9ac\ub97c \ub450\uc5c8\ub2e4. \uc2e4\uc81c\ub85c \uc601\ud654 \ud06c\ub808\ub527\uc5d0\ub294 \ub370\uc774\ube44\ub4dc \ub85c\uc774\ub4dc\uc758 \uc774\ub984\ub9cc \uc62c\ub77c\uac00 \uc788\ub2e4. \uc601\ud654 \uc81c\uc791\uc790\ub4e4\uc740 \uc6d0\uc791\uc5d0 \ub098\uc624\ub294 \ubb34\uc815\ubd80\uc8fc\uc758\uc801 \ud14c\ub9c8\ub4e4\uacfc \ub9c8\uc57d \uad00\ub828 \ub0b4\uc6a9\ub4e4\uc744 \uc0ad\uc81c\ud588\uace0, \uc790\uae30\ub4e4 \uc0dd\uac01\uc5d0 2006\ub144\uc758 \uad00\uac1d\ub4e4\uc5d0\uac8c \ub354 \uc640\ub2ff\uc744 \uac83 \uac19\uc740 \uac83\uc73c\ub85c \uc815\uce58\uc801 \uba54\uc2dc\uc9c0\ub3c4 \ubcc0\uacbd\ud588\ub2e4."} {"question": "\uccad\uc870\uc640 \uad50\ud669\uccad\uc774 \uc678\uad50\uad00\uacc4\ub97c \uc218\ub9bd\ud558\uae30\ub85c \ud55c \uacc4\uae30\ub294?", "paragraph": "\uccad\uc870\uc758 \ub300\uc2e0 \uc774\ud64d\uc7a5\uc740 \uc804\uc784 \ud64d\ucf69\ucd1d\ub3c5 \ud5e4\ub124\uc2dc\ub97c \ub9cc\ub098 \ub0a0\ub85c \uc2ec\uac01\ud574\uc9c0\ub294 \uc911\uad6d\uc758 \uad50\uc548\uc5d0 \ub300\ud574 \ub300\ud654\ub97c \ub098\ub204\uc5c8\ub2e4. \uc774\ud64d\uc7a5\uc740 \uac01\uc9c0\uc5d0\uc11c \uad50\uc548\uc774 \ub04a\uc784\uc5c6\uc774 \ubc1c\uc0dd\ud558\ub294 \uc6d0\uc778\uc758 \ud558\ub098\uac00 \ud504\ub791\uc2a4\uc758 \ubcf4\ud638 \uc544\ub798 \uac00\ud1a8\ub9ad\uad50\ud68c\uac00 \uc788\uae30 \ub54c\ubb38\uc774\ub77c\uace0 \uc0dd\uac01\ud558\uc600\ub2e4. \uadf8\ub294 \ud504\ub791\uc2a4\uac00 \uac00\ud1a8\ub9ad\uad50\ud68c\ub97c \ubcf4\ud638\ud558\ub824\ub294 \uac83\uc740 \ubcf4\ud638\ub77c\ub294 \uba85\ubaa9\uc0c1\uc758 \uc774\uc720\ub85c \uc911\uad6d\uc744 \uac04\uc12d\ud558\ub824\ub294 \ucc45\ub7b5\uc73c\ub85c \ubcf4\uc558\ub2e4. \uc774 \ub54c\ubb38\uc5d0 \uc774\ud64d\uc7a5\uc740 \uad50\uc548\uc744 \uc885\uc2dd\uc2dc\ud0a4\uae30 \uc704\ud574 \uad50\ud669\uccad\uacfc \uc678\uad50\uad00\uacc4\ub97c \ub9fa\uc5b4\uc57c \ud55c\ub2e4\uace0 \uc0dd\uac01\ud588\ub2e4. \uc774\ud64d\uc7a5\uc740 \ud5e4\ub124\uc2dc\uac00 \ub098\uc11c\uc11c \uad50\ud669\uccad\uacfc \uc5f0\ub77d\uc744 \ud574\uc8fc\uae30\ub97c \uc694\uccad\ud558\uc600\uace0 \ud5e4\ub124\uc2dc\ub294 \uad50\ud669\uccad \uad6d\ubb34\uc6d0\uc7a5 \uc57c\ucf54\ube44\ub2c8\uc5d0\uac8c \uc11c\uc2e0\uc744 \ubcf4\ub0b4 \uc774\ud64d\uc7a5\uc758 \uc758\ud5a5\uc744 \uc804\ub2ec\ud588\ub2e4. \uc774\ud6c4 \uccad\uc870\uc640 \uad50\ud669\uccad\uc740 \uc7a0\uc9c0\uad6c\uad50\ub2f9 \uc774\uc804 \ud611\uc758\ub97c \uacc4\uae30\ub85c \uc678\uad50\uad00\uacc4\ub97c \uc218\ub9bd\ud558\uae30\ub85c \ud558\uc600\ub2e4. \uc774\ud64d\uc7a5\uc740 \uad50\ud669\uccad\uacfc \uc678\uad50\uad00\uacc4\ub97c \ud1b5\ud574 \ud504\ub791\uc2a4\uc758 \ubcf4\uad50\uad8c\uc744 \ubc30\uc81c\ud558\uc5ec \uad6d\ub0b4\uc758 \uad50\uc548\uc744 \ud574\uacb0\ud558\ub824 \ud558\uc600\ub2e4. \uadf8\ub7ec\ub098 \ud504\ub791\uc2a4 \uc815\ubd80\ub294 \uad50\ud669\uccad\uc5d0 \ub9cc\uc57d \ud504\ub791\uc2a4\uc758 \ubcf4\uad50\uad8c\uc744 \ubc30\uc81c\ud55c \ucc44 \uad50\ud669\uccad\uc774 \uc8fc\uc911\uacf5\uc0ac\ub97c \ud30c\uacac\ud558\uba74 \ud504\ub791\uc2a4\uc640 \uad50\ud669\uccad\uc774 \uccb4\uacb0\ud55c \uc870\uc57d\uc744 \ubaa8\ub450 \ud3d0\uae30\ud558\uace0 \ud504\ub791\uc2a4 \uc815\ubd80\uac00 \ub9e4\ub144 \ud504\ub791\uc2a4 \uad50\ud68c\uc5d0 \ubc1c\uae09\ud558\ub294 \ubcf4\uc870\uae08\uacfc \uc131\uc9c1\uc790\uc758 \ubd09\uae09\uc744 \uc815\uc9c0\ud560 \uac83\uc774\ub77c\uace0 \ud611\ubc15\ud558\uc600\ub2e4. \uc774\uc5d0 \uad50\ud669\uccad\uc740 \uad50\ud669 \uc0ac\uc808 \ud30c\uacac\uc744 \uc720\ubcf4\ud558\uae30\ub85c \uacb0\uc815\ud588\ub2e4.", "answer": "\uc7a0\uc9c0\uad6c\uad50\ub2f9 \uc774\uc804 \ud611\uc758", "sentence": "\uc774\ud6c4 \uccad\uc870\uc640 \uad50\ud669\uccad\uc740 \uc7a0\uc9c0\uad6c\uad50\ub2f9 \uc774\uc804 \ud611\uc758 \ub97c \uacc4\uae30\ub85c \uc678\uad50\uad00\uacc4\ub97c \uc218\ub9bd\ud558\uae30\ub85c \ud558\uc600\ub2e4.", "paragraph_sentence": "\uccad\uc870\uc758 \ub300\uc2e0 \uc774\ud64d\uc7a5\uc740 \uc804\uc784 \ud64d\ucf69\ucd1d\ub3c5 \ud5e4\ub124\uc2dc\ub97c \ub9cc\ub098 \ub0a0\ub85c \uc2ec\uac01\ud574\uc9c0\ub294 \uc911\uad6d\uc758 \uad50\uc548\uc5d0 \ub300\ud574 \ub300\ud654\ub97c \ub098\ub204\uc5c8\ub2e4. \uc774\ud64d\uc7a5\uc740 \uac01\uc9c0\uc5d0\uc11c \uad50\uc548\uc774 \ub04a\uc784\uc5c6\uc774 \ubc1c\uc0dd\ud558\ub294 \uc6d0\uc778\uc758 \ud558\ub098\uac00 \ud504\ub791\uc2a4\uc758 \ubcf4\ud638 \uc544\ub798 \uac00\ud1a8\ub9ad\uad50\ud68c\uac00 \uc788\uae30 \ub54c\ubb38\uc774\ub77c\uace0 \uc0dd\uac01\ud558\uc600\ub2e4. \uadf8\ub294 \ud504\ub791\uc2a4\uac00 \uac00\ud1a8\ub9ad\uad50\ud68c\ub97c \ubcf4\ud638\ud558\ub824\ub294 \uac83\uc740 \ubcf4\ud638\ub77c\ub294 \uba85\ubaa9\uc0c1\uc758 \uc774\uc720\ub85c \uc911\uad6d\uc744 \uac04\uc12d\ud558\ub824\ub294 \ucc45\ub7b5\uc73c\ub85c \ubcf4\uc558\ub2e4. \uc774 \ub54c\ubb38\uc5d0 \uc774\ud64d\uc7a5\uc740 \uad50\uc548\uc744 \uc885\uc2dd\uc2dc\ud0a4\uae30 \uc704\ud574 \uad50\ud669\uccad\uacfc \uc678\uad50\uad00\uacc4\ub97c \ub9fa\uc5b4\uc57c \ud55c\ub2e4\uace0 \uc0dd\uac01\ud588\ub2e4. \uc774\ud64d\uc7a5\uc740 \ud5e4\ub124\uc2dc\uac00 \ub098\uc11c\uc11c \uad50\ud669\uccad\uacfc \uc5f0\ub77d\uc744 \ud574\uc8fc\uae30\ub97c \uc694\uccad\ud558\uc600\uace0 \ud5e4\ub124\uc2dc\ub294 \uad50\ud669\uccad \uad6d\ubb34\uc6d0\uc7a5 \uc57c\ucf54\ube44\ub2c8\uc5d0\uac8c \uc11c\uc2e0\uc744 \ubcf4\ub0b4 \uc774\ud64d\uc7a5\uc758 \uc758\ud5a5\uc744 \uc804\ub2ec\ud588\ub2e4. \uc774\ud6c4 \uccad\uc870\uc640 \uad50\ud669\uccad\uc740 \uc7a0\uc9c0\uad6c\uad50\ub2f9 \uc774\uc804 \ud611\uc758 \ub97c \uacc4\uae30\ub85c \uc678\uad50\uad00\uacc4\ub97c \uc218\ub9bd\ud558\uae30\ub85c \ud558\uc600\ub2e4. \uc774\ud64d\uc7a5\uc740 \uad50\ud669\uccad\uacfc \uc678\uad50\uad00\uacc4\ub97c \ud1b5\ud574 \ud504\ub791\uc2a4\uc758 \ubcf4\uad50\uad8c\uc744 \ubc30\uc81c\ud558\uc5ec \uad6d\ub0b4\uc758 \uad50\uc548\uc744 \ud574\uacb0\ud558\ub824 \ud558\uc600\ub2e4. \uadf8\ub7ec\ub098 \ud504\ub791\uc2a4 \uc815\ubd80\ub294 \uad50\ud669\uccad\uc5d0 \ub9cc\uc57d \ud504\ub791\uc2a4\uc758 \ubcf4\uad50\uad8c\uc744 \ubc30\uc81c\ud55c \ucc44 \uad50\ud669\uccad\uc774 \uc8fc\uc911\uacf5\uc0ac\ub97c \ud30c\uacac\ud558\uba74 \ud504\ub791\uc2a4\uc640 \uad50\ud669\uccad\uc774 \uccb4\uacb0\ud55c \uc870\uc57d\uc744 \ubaa8\ub450 \ud3d0\uae30\ud558\uace0 \ud504\ub791\uc2a4 \uc815\ubd80\uac00 \ub9e4\ub144 \ud504\ub791\uc2a4 \uad50\ud68c\uc5d0 \ubc1c\uae09\ud558\ub294 \ubcf4\uc870\uae08\uacfc \uc131\uc9c1\uc790\uc758 \ubd09\uae09\uc744 \uc815\uc9c0\ud560 \uac83\uc774\ub77c\uace0 \ud611\ubc15\ud558\uc600\ub2e4. \uc774\uc5d0 \uad50\ud669\uccad\uc740 \uad50\ud669 \uc0ac\uc808 \ud30c\uacac\uc744 \uc720\ubcf4\ud558\uae30\ub85c \uacb0\uc815\ud588\ub2e4.", "paragraph_answer": "\uccad\uc870\uc758 \ub300\uc2e0 \uc774\ud64d\uc7a5\uc740 \uc804\uc784 \ud64d\ucf69\ucd1d\ub3c5 \ud5e4\ub124\uc2dc\ub97c \ub9cc\ub098 \ub0a0\ub85c \uc2ec\uac01\ud574\uc9c0\ub294 \uc911\uad6d\uc758 \uad50\uc548\uc5d0 \ub300\ud574 \ub300\ud654\ub97c \ub098\ub204\uc5c8\ub2e4. \uc774\ud64d\uc7a5\uc740 \uac01\uc9c0\uc5d0\uc11c \uad50\uc548\uc774 \ub04a\uc784\uc5c6\uc774 \ubc1c\uc0dd\ud558\ub294 \uc6d0\uc778\uc758 \ud558\ub098\uac00 \ud504\ub791\uc2a4\uc758 \ubcf4\ud638 \uc544\ub798 \uac00\ud1a8\ub9ad\uad50\ud68c\uac00 \uc788\uae30 \ub54c\ubb38\uc774\ub77c\uace0 \uc0dd\uac01\ud558\uc600\ub2e4. \uadf8\ub294 \ud504\ub791\uc2a4\uac00 \uac00\ud1a8\ub9ad\uad50\ud68c\ub97c \ubcf4\ud638\ud558\ub824\ub294 \uac83\uc740 \ubcf4\ud638\ub77c\ub294 \uba85\ubaa9\uc0c1\uc758 \uc774\uc720\ub85c \uc911\uad6d\uc744 \uac04\uc12d\ud558\ub824\ub294 \ucc45\ub7b5\uc73c\ub85c \ubcf4\uc558\ub2e4. \uc774 \ub54c\ubb38\uc5d0 \uc774\ud64d\uc7a5\uc740 \uad50\uc548\uc744 \uc885\uc2dd\uc2dc\ud0a4\uae30 \uc704\ud574 \uad50\ud669\uccad\uacfc \uc678\uad50\uad00\uacc4\ub97c \ub9fa\uc5b4\uc57c \ud55c\ub2e4\uace0 \uc0dd\uac01\ud588\ub2e4. \uc774\ud64d\uc7a5\uc740 \ud5e4\ub124\uc2dc\uac00 \ub098\uc11c\uc11c \uad50\ud669\uccad\uacfc \uc5f0\ub77d\uc744 \ud574\uc8fc\uae30\ub97c \uc694\uccad\ud558\uc600\uace0 \ud5e4\ub124\uc2dc\ub294 \uad50\ud669\uccad \uad6d\ubb34\uc6d0\uc7a5 \uc57c\ucf54\ube44\ub2c8\uc5d0\uac8c \uc11c\uc2e0\uc744 \ubcf4\ub0b4 \uc774\ud64d\uc7a5\uc758 \uc758\ud5a5\uc744 \uc804\ub2ec\ud588\ub2e4. \uc774\ud6c4 \uccad\uc870\uc640 \uad50\ud669\uccad\uc740 \uc7a0\uc9c0\uad6c\uad50\ub2f9 \uc774\uc804 \ud611\uc758 \ub97c \uacc4\uae30\ub85c \uc678\uad50\uad00\uacc4\ub97c \uc218\ub9bd\ud558\uae30\ub85c \ud558\uc600\ub2e4. \uc774\ud64d\uc7a5\uc740 \uad50\ud669\uccad\uacfc \uc678\uad50\uad00\uacc4\ub97c \ud1b5\ud574 \ud504\ub791\uc2a4\uc758 \ubcf4\uad50\uad8c\uc744 \ubc30\uc81c\ud558\uc5ec \uad6d\ub0b4\uc758 \uad50\uc548\uc744 \ud574\uacb0\ud558\ub824 \ud558\uc600\ub2e4. \uadf8\ub7ec\ub098 \ud504\ub791\uc2a4 \uc815\ubd80\ub294 \uad50\ud669\uccad\uc5d0 \ub9cc\uc57d \ud504\ub791\uc2a4\uc758 \ubcf4\uad50\uad8c\uc744 \ubc30\uc81c\ud55c \ucc44 \uad50\ud669\uccad\uc774 \uc8fc\uc911\uacf5\uc0ac\ub97c \ud30c\uacac\ud558\uba74 \ud504\ub791\uc2a4\uc640 \uad50\ud669\uccad\uc774 \uccb4\uacb0\ud55c \uc870\uc57d\uc744 \ubaa8\ub450 \ud3d0\uae30\ud558\uace0 \ud504\ub791\uc2a4 \uc815\ubd80\uac00 \ub9e4\ub144 \ud504\ub791\uc2a4 \uad50\ud68c\uc5d0 \ubc1c\uae09\ud558\ub294 \ubcf4\uc870\uae08\uacfc \uc131\uc9c1\uc790\uc758 \ubd09\uae09\uc744 \uc815\uc9c0\ud560 \uac83\uc774\ub77c\uace0 \ud611\ubc15\ud558\uc600\ub2e4. \uc774\uc5d0 \uad50\ud669\uccad\uc740 \uad50\ud669 \uc0ac\uc808 \ud30c\uacac\uc744 \uc720\ubcf4\ud558\uae30\ub85c \uacb0\uc815\ud588\ub2e4.", "sentence_answer": "\uc774\ud6c4 \uccad\uc870\uc640 \uad50\ud669\uccad\uc740 \uc7a0\uc9c0\uad6c\uad50\ub2f9 \uc774\uc804 \ud611\uc758 \ub97c \uacc4\uae30\ub85c \uc678\uad50\uad00\uacc4\ub97c \uc218\ub9bd\ud558\uae30\ub85c \ud558\uc600\ub2e4.", "paragraph_id": "6535116-6-1", "paragraph_question": "question: \uccad\uc870\uc640 \uad50\ud669\uccad\uc774 \uc678\uad50\uad00\uacc4\ub97c \uc218\ub9bd\ud558\uae30\ub85c \ud55c \uacc4\uae30\ub294?, context: \uccad\uc870\uc758 \ub300\uc2e0 \uc774\ud64d\uc7a5\uc740 \uc804\uc784 \ud64d\ucf69\ucd1d\ub3c5 \ud5e4\ub124\uc2dc\ub97c \ub9cc\ub098 \ub0a0\ub85c \uc2ec\uac01\ud574\uc9c0\ub294 \uc911\uad6d\uc758 \uad50\uc548\uc5d0 \ub300\ud574 \ub300\ud654\ub97c \ub098\ub204\uc5c8\ub2e4. \uc774\ud64d\uc7a5\uc740 \uac01\uc9c0\uc5d0\uc11c \uad50\uc548\uc774 \ub04a\uc784\uc5c6\uc774 \ubc1c\uc0dd\ud558\ub294 \uc6d0\uc778\uc758 \ud558\ub098\uac00 \ud504\ub791\uc2a4\uc758 \ubcf4\ud638 \uc544\ub798 \uac00\ud1a8\ub9ad\uad50\ud68c\uac00 \uc788\uae30 \ub54c\ubb38\uc774\ub77c\uace0 \uc0dd\uac01\ud558\uc600\ub2e4. \uadf8\ub294 \ud504\ub791\uc2a4\uac00 \uac00\ud1a8\ub9ad\uad50\ud68c\ub97c \ubcf4\ud638\ud558\ub824\ub294 \uac83\uc740 \ubcf4\ud638\ub77c\ub294 \uba85\ubaa9\uc0c1\uc758 \uc774\uc720\ub85c \uc911\uad6d\uc744 \uac04\uc12d\ud558\ub824\ub294 \ucc45\ub7b5\uc73c\ub85c \ubcf4\uc558\ub2e4. \uc774 \ub54c\ubb38\uc5d0 \uc774\ud64d\uc7a5\uc740 \uad50\uc548\uc744 \uc885\uc2dd\uc2dc\ud0a4\uae30 \uc704\ud574 \uad50\ud669\uccad\uacfc \uc678\uad50\uad00\uacc4\ub97c \ub9fa\uc5b4\uc57c \ud55c\ub2e4\uace0 \uc0dd\uac01\ud588\ub2e4. \uc774\ud64d\uc7a5\uc740 \ud5e4\ub124\uc2dc\uac00 \ub098\uc11c\uc11c \uad50\ud669\uccad\uacfc \uc5f0\ub77d\uc744 \ud574\uc8fc\uae30\ub97c \uc694\uccad\ud558\uc600\uace0 \ud5e4\ub124\uc2dc\ub294 \uad50\ud669\uccad \uad6d\ubb34\uc6d0\uc7a5 \uc57c\ucf54\ube44\ub2c8\uc5d0\uac8c \uc11c\uc2e0\uc744 \ubcf4\ub0b4 \uc774\ud64d\uc7a5\uc758 \uc758\ud5a5\uc744 \uc804\ub2ec\ud588\ub2e4. \uc774\ud6c4 \uccad\uc870\uc640 \uad50\ud669\uccad\uc740 \uc7a0\uc9c0\uad6c\uad50\ub2f9 \uc774\uc804 \ud611\uc758\ub97c \uacc4\uae30\ub85c \uc678\uad50\uad00\uacc4\ub97c \uc218\ub9bd\ud558\uae30\ub85c \ud558\uc600\ub2e4. \uc774\ud64d\uc7a5\uc740 \uad50\ud669\uccad\uacfc \uc678\uad50\uad00\uacc4\ub97c \ud1b5\ud574 \ud504\ub791\uc2a4\uc758 \ubcf4\uad50\uad8c\uc744 \ubc30\uc81c\ud558\uc5ec \uad6d\ub0b4\uc758 \uad50\uc548\uc744 \ud574\uacb0\ud558\ub824 \ud558\uc600\ub2e4. \uadf8\ub7ec\ub098 \ud504\ub791\uc2a4 \uc815\ubd80\ub294 \uad50\ud669\uccad\uc5d0 \ub9cc\uc57d \ud504\ub791\uc2a4\uc758 \ubcf4\uad50\uad8c\uc744 \ubc30\uc81c\ud55c \ucc44 \uad50\ud669\uccad\uc774 \uc8fc\uc911\uacf5\uc0ac\ub97c \ud30c\uacac\ud558\uba74 \ud504\ub791\uc2a4\uc640 \uad50\ud669\uccad\uc774 \uccb4\uacb0\ud55c \uc870\uc57d\uc744 \ubaa8\ub450 \ud3d0\uae30\ud558\uace0 \ud504\ub791\uc2a4 \uc815\ubd80\uac00 \ub9e4\ub144 \ud504\ub791\uc2a4 \uad50\ud68c\uc5d0 \ubc1c\uae09\ud558\ub294 \ubcf4\uc870\uae08\uacfc \uc131\uc9c1\uc790\uc758 \ubd09\uae09\uc744 \uc815\uc9c0\ud560 \uac83\uc774\ub77c\uace0 \ud611\ubc15\ud558\uc600\ub2e4. \uc774\uc5d0 \uad50\ud669\uccad\uc740 \uad50\ud669 \uc0ac\uc808 \ud30c\uacac\uc744 \uc720\ubcf4\ud558\uae30\ub85c \uacb0\uc815\ud588\ub2e4."} {"question": "\ud608\uccad\ud559\uc801 \uc870\uc0ac\uc5d0 \ub530\ub974\uba74 \ud56d\uccb4\ub97c \uc9c0\ub2cc \uc0ac\ub78c\ub4e4\uc740 \uba87 \uc138\uc758 \uc5f0\ub839\ub300 \uc774\uc0c1\uc5d0\uc11c \uba74\uc5ed\uc774 \ub418\ub294 \uac83\uc73c\ub85c \ub098\ud0c0\ub0ac\ub294\uac00?", "paragraph": "\uc9c0\uce74\ubc14\uc774\ub7ec\uc2a4\ub294 \uc6b0\uac04\ub2e4\uc640 \ub098\uc774\uc9c0\ub9ac\uc544\uc758 \ud608\uccad\ud559\uc801 \uc870\uc0ac\uc758 \uacb0\uacfc\ub85c \uc778\uac04\uc744 \uac10\uc5fc\uc2dc\ud0a8\ub2e4\ub294 \uac83\uc774 \uc54c\ub824\uc84c\ub2e4. \uc804\uc5f0\ub839\uce35 84\uba85\uc758 \ud608\uccad\ud559\uc801 \uc870\uc0ac(serosurvey)\uc5d0\uc11c 50\uba85\uc774 \ud56d\uccb4\ub97c \uc9c0\ub2c8\uace0 40\uc138\ub97c \ub118\ub294 \uc804\uc5f0\ub839\uc5d0\uc11c \uba74\uc5ed\uc774 \ub418\ub294 \uac83\uc73c\ub85c \ub098\ud0c0\ub0ac\ub2e4. 1954\ub144\uc774 \ub418\uc5b4\uc11c\uc57c \uc778\uac04\uc758 \uc9c0\uce74\ubc14\uc774\ub7ec\uc2a4\uc5d0 \ub300\ud55c \uc131\uacf5\uc801 \ubd84\ub9ac\uac00 \uacf5\ud45c\ub418\uc5c8\ub2e4. \uc774 \ubd84\ub9ac\ub294 \ud669\uc5f4\ub85c \uc758\uc2ec\ub418\ub294 \ud669\ub2ec\uc5d0 \ub300\ud55c 1952\ub144\uc758 \ubc1c\ubcd1 \uc870\uc0ac\uc758 \uc77c\ud658\uc73c\ub85c \uc774\ub8e8\uc5b4\uc84c\ub294\ub370, \ub2e4\uc2dc \ub9d0\ud574 \ud669\ub2ec\uc774 \uc544\ub2cc \ub9d0\ub77c\ub9ac\uc544\uc758 \uc99d\uac70\uc640 \ubbf8\uc5f4\uacfc \ub450\ud1b5\uc774 \uc788\ub294\ub370\ub3c4 3\uc77c\ub9cc\uc5d0 \ud68c\ubcf5\ub41c 10\uc0b4 \uc5ec\uc131\uc758 \ud608\uc561\uc5d0\uc11c \ubc1c\uacac\ub41c \uac83\uc774\ub2e4. \uc774 \ud608\uc561\uc744 15\ub9c8\ub9ac \uacbd\uacfc\uae4c\uc9c0 \uacc4\uc18d\ub41c \uc2e4\ud5d8\uc6a9 \uc950\uc758 \ub1cc\uc5d0 \uc8fc\uc785\ud588\ub2e4. \uadf8 \ub2e4\uc74c\uc5d0 \ud2b9\ubcc4\ud788 \uc9c0\uce74\ubc14\uc774\ub7ec\uc2a4\uc5d0 \uba74\uc5ed\uc774 \ub3fc \uc788\ub294 \ud788\ub9d0\ub77c\uc57c \uc6d0\uc22d\uc774 \ud608\uccad\uc744 \uc774\uc6a9\ud55c \uc911\ud654 \uac80\uc0ac\uc5d0\uc11c \uc950\uc758 \ub1cc\uc758 \ubc14\uc774\ub7ec\uc2a4\uac00 \uc2dc\ud5d8\ub418\uc5c8\ub2e4. 1\uba85\uc740 \uc5f4\uacfc \ud669\ub2ec\uacfc \uae30\uce68\uacfc \uad11\ubc94\uc704 \uad00\uc808\ud1b5, \ub2e4\ub978 1\uba85\uc740 \uc5f4\uacfc \ub450\ud1b5\uacfc \ub208 \ub4a4\uc640 \uad00\uc808\uc758 \ud1b5\uc99d\uc774 \uc788\ub294 \ub450 \uac10\uc5fc \ud658\uc790\uc758 \ud608\uc561\uc5d0\uc11c\ub294 \ub300\uc870\uc801\uc73c\ub85c \uc5b4\ub5a4 \ubc14\uc774\ub7ec\uc2a4\ub3c4 \ubd84\ub9ac\ub418\uc9c0 \uc54a\uc558\ub2e4. \uac10\uc5fc\uc740 \uc9c0\uce74\ubc14\uc774\ub7ec\uc2a4 \ud2b9\uc774 \ud608\uccad \ud56d\uccb4\uc758 \ubc1c\uc6d0\uc73c\ub85c \uc785\uc99d\ub418\uc5c8\ub2e4. \uc778\ub3c4\uc5d0\uc11c \uc218\ud589\ub41c \ud55c \uc5f0\uad6c\uc5d0\uc11c\ub294 '\uc720\uc758\ubbf8\ud55c \uc218'\uc758 \uc778\ub3c4\uc778\ub4e4\uc774 \uc774 \ubc14\uc774\ub7ec\uc2a4\uc5d0 \uba74\uc5ed \ubc18\uc751\uc744 \ub098\ud0c0\ub0b8\ub2e4\ub294 \uac83\uc744 \ubcf4\uc5ec\uc92c\ub294\ub370, \uc774\ub294 \uc774 \ubc14\uc774\ub7ec\uc2a4\uac00 \uc778\uac04 \uac1c\uccb4\uad70 \ub0b4\uc5d0\uc11c \uc624\ub7ab\ub3d9\uc548 \ub110\ub9ac \ud37c\uc838 \uc788\uc5c8\ub2e4\ub294 \uac83\uc744 \uc554\uc2dc\ud55c\ub2e4.", "answer": "40\uc138", "sentence": "\uc804\uc5f0\ub839\uce35 84\uba85\uc758 \ud608\uccad\ud559\uc801 \uc870\uc0ac(serosurvey)\uc5d0\uc11c 50\uba85\uc774 \ud56d\uccb4\ub97c \uc9c0\ub2c8\uace0 40\uc138 \ub97c \ub118\ub294 \uc804\uc5f0\ub839\uc5d0\uc11c \uba74\uc5ed\uc774 \ub418\ub294 \uac83\uc73c\ub85c \ub098\ud0c0\ub0ac\ub2e4.", "paragraph_sentence": "\uc9c0\uce74\ubc14\uc774\ub7ec\uc2a4\ub294 \uc6b0\uac04\ub2e4\uc640 \ub098\uc774\uc9c0\ub9ac\uc544\uc758 \ud608\uccad\ud559\uc801 \uc870\uc0ac\uc758 \uacb0\uacfc\ub85c \uc778\uac04\uc744 \uac10\uc5fc\uc2dc\ud0a8\ub2e4\ub294 \uac83\uc774 \uc54c\ub824\uc84c\ub2e4. \uc804\uc5f0\ub839\uce35 84\uba85\uc758 \ud608\uccad\ud559\uc801 \uc870\uc0ac(serosurvey)\uc5d0\uc11c 50\uba85\uc774 \ud56d\uccb4\ub97c \uc9c0\ub2c8\uace0 40\uc138 \ub97c \ub118\ub294 \uc804\uc5f0\ub839\uc5d0\uc11c \uba74\uc5ed\uc774 \ub418\ub294 \uac83\uc73c\ub85c \ub098\ud0c0\ub0ac\ub2e4. 1954\ub144\uc774 \ub418\uc5b4\uc11c\uc57c \uc778\uac04\uc758 \uc9c0\uce74\ubc14\uc774\ub7ec\uc2a4\uc5d0 \ub300\ud55c \uc131\uacf5\uc801 \ubd84\ub9ac\uac00 \uacf5\ud45c\ub418\uc5c8\ub2e4. \uc774 \ubd84\ub9ac\ub294 \ud669\uc5f4\ub85c \uc758\uc2ec\ub418\ub294 \ud669\ub2ec\uc5d0 \ub300\ud55c 1952\ub144\uc758 \ubc1c\ubcd1 \uc870\uc0ac\uc758 \uc77c\ud658\uc73c\ub85c \uc774\ub8e8\uc5b4\uc84c\ub294\ub370, \ub2e4\uc2dc \ub9d0\ud574 \ud669\ub2ec\uc774 \uc544\ub2cc \ub9d0\ub77c\ub9ac\uc544\uc758 \uc99d\uac70\uc640 \ubbf8\uc5f4\uacfc \ub450\ud1b5\uc774 \uc788\ub294\ub370\ub3c4 3\uc77c\ub9cc\uc5d0 \ud68c\ubcf5\ub41c 10\uc0b4 \uc5ec\uc131\uc758 \ud608\uc561\uc5d0\uc11c \ubc1c\uacac\ub41c \uac83\uc774\ub2e4. \uc774 \ud608\uc561\uc744 15\ub9c8\ub9ac \uacbd\uacfc\uae4c\uc9c0 \uacc4\uc18d\ub41c \uc2e4\ud5d8\uc6a9 \uc950\uc758 \ub1cc\uc5d0 \uc8fc\uc785\ud588\ub2e4. \uadf8 \ub2e4\uc74c\uc5d0 \ud2b9\ubcc4\ud788 \uc9c0\uce74\ubc14\uc774\ub7ec\uc2a4\uc5d0 \uba74\uc5ed\uc774 \ub3fc \uc788\ub294 \ud788\ub9d0\ub77c\uc57c \uc6d0\uc22d\uc774 \ud608\uccad\uc744 \uc774\uc6a9\ud55c \uc911\ud654 \uac80\uc0ac\uc5d0\uc11c \uc950\uc758 \ub1cc\uc758 \ubc14\uc774\ub7ec\uc2a4\uac00 \uc2dc\ud5d8\ub418\uc5c8\ub2e4. 1\uba85\uc740 \uc5f4\uacfc \ud669\ub2ec\uacfc \uae30\uce68\uacfc \uad11\ubc94\uc704 \uad00\uc808\ud1b5, \ub2e4\ub978 1\uba85\uc740 \uc5f4\uacfc \ub450\ud1b5\uacfc \ub208 \ub4a4\uc640 \uad00\uc808\uc758 \ud1b5\uc99d\uc774 \uc788\ub294 \ub450 \uac10\uc5fc \ud658\uc790\uc758 \ud608\uc561\uc5d0\uc11c\ub294 \ub300\uc870\uc801\uc73c\ub85c \uc5b4\ub5a4 \ubc14\uc774\ub7ec\uc2a4\ub3c4 \ubd84\ub9ac\ub418\uc9c0 \uc54a\uc558\ub2e4. \uac10\uc5fc\uc740 \uc9c0\uce74\ubc14\uc774\ub7ec\uc2a4 \ud2b9\uc774 \ud608\uccad \ud56d\uccb4\uc758 \ubc1c\uc6d0\uc73c\ub85c \uc785\uc99d\ub418\uc5c8\ub2e4. \uc778\ub3c4\uc5d0\uc11c \uc218\ud589\ub41c \ud55c \uc5f0\uad6c\uc5d0\uc11c\ub294 '\uc720\uc758\ubbf8\ud55c \uc218'\uc758 \uc778\ub3c4\uc778\ub4e4\uc774 \uc774 \ubc14\uc774\ub7ec\uc2a4\uc5d0 \uba74\uc5ed \ubc18\uc751\uc744 \ub098\ud0c0\ub0b8\ub2e4\ub294 \uac83\uc744 \ubcf4\uc5ec\uc92c\ub294\ub370, \uc774\ub294 \uc774 \ubc14\uc774\ub7ec\uc2a4\uac00 \uc778\uac04 \uac1c\uccb4\uad70 \ub0b4\uc5d0\uc11c \uc624\ub7ab\ub3d9\uc548 \ub110\ub9ac \ud37c\uc838 \uc788\uc5c8\ub2e4\ub294 \uac83\uc744 \uc554\uc2dc\ud55c\ub2e4.", "paragraph_answer": "\uc9c0\uce74\ubc14\uc774\ub7ec\uc2a4\ub294 \uc6b0\uac04\ub2e4\uc640 \ub098\uc774\uc9c0\ub9ac\uc544\uc758 \ud608\uccad\ud559\uc801 \uc870\uc0ac\uc758 \uacb0\uacfc\ub85c \uc778\uac04\uc744 \uac10\uc5fc\uc2dc\ud0a8\ub2e4\ub294 \uac83\uc774 \uc54c\ub824\uc84c\ub2e4. \uc804\uc5f0\ub839\uce35 84\uba85\uc758 \ud608\uccad\ud559\uc801 \uc870\uc0ac(serosurvey)\uc5d0\uc11c 50\uba85\uc774 \ud56d\uccb4\ub97c \uc9c0\ub2c8\uace0 40\uc138 \ub97c \ub118\ub294 \uc804\uc5f0\ub839\uc5d0\uc11c \uba74\uc5ed\uc774 \ub418\ub294 \uac83\uc73c\ub85c \ub098\ud0c0\ub0ac\ub2e4. 1954\ub144\uc774 \ub418\uc5b4\uc11c\uc57c \uc778\uac04\uc758 \uc9c0\uce74\ubc14\uc774\ub7ec\uc2a4\uc5d0 \ub300\ud55c \uc131\uacf5\uc801 \ubd84\ub9ac\uac00 \uacf5\ud45c\ub418\uc5c8\ub2e4. \uc774 \ubd84\ub9ac\ub294 \ud669\uc5f4\ub85c \uc758\uc2ec\ub418\ub294 \ud669\ub2ec\uc5d0 \ub300\ud55c 1952\ub144\uc758 \ubc1c\ubcd1 \uc870\uc0ac\uc758 \uc77c\ud658\uc73c\ub85c \uc774\ub8e8\uc5b4\uc84c\ub294\ub370, \ub2e4\uc2dc \ub9d0\ud574 \ud669\ub2ec\uc774 \uc544\ub2cc \ub9d0\ub77c\ub9ac\uc544\uc758 \uc99d\uac70\uc640 \ubbf8\uc5f4\uacfc \ub450\ud1b5\uc774 \uc788\ub294\ub370\ub3c4 3\uc77c\ub9cc\uc5d0 \ud68c\ubcf5\ub41c 10\uc0b4 \uc5ec\uc131\uc758 \ud608\uc561\uc5d0\uc11c \ubc1c\uacac\ub41c \uac83\uc774\ub2e4. \uc774 \ud608\uc561\uc744 15\ub9c8\ub9ac \uacbd\uacfc\uae4c\uc9c0 \uacc4\uc18d\ub41c \uc2e4\ud5d8\uc6a9 \uc950\uc758 \ub1cc\uc5d0 \uc8fc\uc785\ud588\ub2e4. \uadf8 \ub2e4\uc74c\uc5d0 \ud2b9\ubcc4\ud788 \uc9c0\uce74\ubc14\uc774\ub7ec\uc2a4\uc5d0 \uba74\uc5ed\uc774 \ub3fc \uc788\ub294 \ud788\ub9d0\ub77c\uc57c \uc6d0\uc22d\uc774 \ud608\uccad\uc744 \uc774\uc6a9\ud55c \uc911\ud654 \uac80\uc0ac\uc5d0\uc11c \uc950\uc758 \ub1cc\uc758 \ubc14\uc774\ub7ec\uc2a4\uac00 \uc2dc\ud5d8\ub418\uc5c8\ub2e4. 1\uba85\uc740 \uc5f4\uacfc \ud669\ub2ec\uacfc \uae30\uce68\uacfc \uad11\ubc94\uc704 \uad00\uc808\ud1b5, \ub2e4\ub978 1\uba85\uc740 \uc5f4\uacfc \ub450\ud1b5\uacfc \ub208 \ub4a4\uc640 \uad00\uc808\uc758 \ud1b5\uc99d\uc774 \uc788\ub294 \ub450 \uac10\uc5fc \ud658\uc790\uc758 \ud608\uc561\uc5d0\uc11c\ub294 \ub300\uc870\uc801\uc73c\ub85c \uc5b4\ub5a4 \ubc14\uc774\ub7ec\uc2a4\ub3c4 \ubd84\ub9ac\ub418\uc9c0 \uc54a\uc558\ub2e4. \uac10\uc5fc\uc740 \uc9c0\uce74\ubc14\uc774\ub7ec\uc2a4 \ud2b9\uc774 \ud608\uccad \ud56d\uccb4\uc758 \ubc1c\uc6d0\uc73c\ub85c \uc785\uc99d\ub418\uc5c8\ub2e4. \uc778\ub3c4\uc5d0\uc11c \uc218\ud589\ub41c \ud55c \uc5f0\uad6c\uc5d0\uc11c\ub294 '\uc720\uc758\ubbf8\ud55c \uc218'\uc758 \uc778\ub3c4\uc778\ub4e4\uc774 \uc774 \ubc14\uc774\ub7ec\uc2a4\uc5d0 \uba74\uc5ed \ubc18\uc751\uc744 \ub098\ud0c0\ub0b8\ub2e4\ub294 \uac83\uc744 \ubcf4\uc5ec\uc92c\ub294\ub370, \uc774\ub294 \uc774 \ubc14\uc774\ub7ec\uc2a4\uac00 \uc778\uac04 \uac1c\uccb4\uad70 \ub0b4\uc5d0\uc11c \uc624\ub7ab\ub3d9\uc548 \ub110\ub9ac \ud37c\uc838 \uc788\uc5c8\ub2e4\ub294 \uac83\uc744 \uc554\uc2dc\ud55c\ub2e4.", "sentence_answer": "\uc804\uc5f0\ub839\uce35 84\uba85\uc758 \ud608\uccad\ud559\uc801 \uc870\uc0ac(serosurvey)\uc5d0\uc11c 50\uba85\uc774 \ud56d\uccb4\ub97c \uc9c0\ub2c8\uace0 40\uc138 \ub97c \ub118\ub294 \uc804\uc5f0\ub839\uc5d0\uc11c \uba74\uc5ed\uc774 \ub418\ub294 \uac83\uc73c\ub85c \ub098\ud0c0\ub0ac\ub2e4.", "paragraph_id": "6557022-3-1", "paragraph_question": "question: \ud608\uccad\ud559\uc801 \uc870\uc0ac\uc5d0 \ub530\ub974\uba74 \ud56d\uccb4\ub97c \uc9c0\ub2cc \uc0ac\ub78c\ub4e4\uc740 \uba87 \uc138\uc758 \uc5f0\ub839\ub300 \uc774\uc0c1\uc5d0\uc11c \uba74\uc5ed\uc774 \ub418\ub294 \uac83\uc73c\ub85c \ub098\ud0c0\ub0ac\ub294\uac00?, context: \uc9c0\uce74\ubc14\uc774\ub7ec\uc2a4\ub294 \uc6b0\uac04\ub2e4\uc640 \ub098\uc774\uc9c0\ub9ac\uc544\uc758 \ud608\uccad\ud559\uc801 \uc870\uc0ac\uc758 \uacb0\uacfc\ub85c \uc778\uac04\uc744 \uac10\uc5fc\uc2dc\ud0a8\ub2e4\ub294 \uac83\uc774 \uc54c\ub824\uc84c\ub2e4. \uc804\uc5f0\ub839\uce35 84\uba85\uc758 \ud608\uccad\ud559\uc801 \uc870\uc0ac(serosurvey)\uc5d0\uc11c 50\uba85\uc774 \ud56d\uccb4\ub97c \uc9c0\ub2c8\uace0 40\uc138\ub97c \ub118\ub294 \uc804\uc5f0\ub839\uc5d0\uc11c \uba74\uc5ed\uc774 \ub418\ub294 \uac83\uc73c\ub85c \ub098\ud0c0\ub0ac\ub2e4. 1954\ub144\uc774 \ub418\uc5b4\uc11c\uc57c \uc778\uac04\uc758 \uc9c0\uce74\ubc14\uc774\ub7ec\uc2a4\uc5d0 \ub300\ud55c \uc131\uacf5\uc801 \ubd84\ub9ac\uac00 \uacf5\ud45c\ub418\uc5c8\ub2e4. \uc774 \ubd84\ub9ac\ub294 \ud669\uc5f4\ub85c \uc758\uc2ec\ub418\ub294 \ud669\ub2ec\uc5d0 \ub300\ud55c 1952\ub144\uc758 \ubc1c\ubcd1 \uc870\uc0ac\uc758 \uc77c\ud658\uc73c\ub85c \uc774\ub8e8\uc5b4\uc84c\ub294\ub370, \ub2e4\uc2dc \ub9d0\ud574 \ud669\ub2ec\uc774 \uc544\ub2cc \ub9d0\ub77c\ub9ac\uc544\uc758 \uc99d\uac70\uc640 \ubbf8\uc5f4\uacfc \ub450\ud1b5\uc774 \uc788\ub294\ub370\ub3c4 3\uc77c\ub9cc\uc5d0 \ud68c\ubcf5\ub41c 10\uc0b4 \uc5ec\uc131\uc758 \ud608\uc561\uc5d0\uc11c \ubc1c\uacac\ub41c \uac83\uc774\ub2e4. \uc774 \ud608\uc561\uc744 15\ub9c8\ub9ac \uacbd\uacfc\uae4c\uc9c0 \uacc4\uc18d\ub41c \uc2e4\ud5d8\uc6a9 \uc950\uc758 \ub1cc\uc5d0 \uc8fc\uc785\ud588\ub2e4. \uadf8 \ub2e4\uc74c\uc5d0 \ud2b9\ubcc4\ud788 \uc9c0\uce74\ubc14\uc774\ub7ec\uc2a4\uc5d0 \uba74\uc5ed\uc774 \ub3fc \uc788\ub294 \ud788\ub9d0\ub77c\uc57c \uc6d0\uc22d\uc774 \ud608\uccad\uc744 \uc774\uc6a9\ud55c \uc911\ud654 \uac80\uc0ac\uc5d0\uc11c \uc950\uc758 \ub1cc\uc758 \ubc14\uc774\ub7ec\uc2a4\uac00 \uc2dc\ud5d8\ub418\uc5c8\ub2e4. 1\uba85\uc740 \uc5f4\uacfc \ud669\ub2ec\uacfc \uae30\uce68\uacfc \uad11\ubc94\uc704 \uad00\uc808\ud1b5, \ub2e4\ub978 1\uba85\uc740 \uc5f4\uacfc \ub450\ud1b5\uacfc \ub208 \ub4a4\uc640 \uad00\uc808\uc758 \ud1b5\uc99d\uc774 \uc788\ub294 \ub450 \uac10\uc5fc \ud658\uc790\uc758 \ud608\uc561\uc5d0\uc11c\ub294 \ub300\uc870\uc801\uc73c\ub85c \uc5b4\ub5a4 \ubc14\uc774\ub7ec\uc2a4\ub3c4 \ubd84\ub9ac\ub418\uc9c0 \uc54a\uc558\ub2e4. \uac10\uc5fc\uc740 \uc9c0\uce74\ubc14\uc774\ub7ec\uc2a4 \ud2b9\uc774 \ud608\uccad \ud56d\uccb4\uc758 \ubc1c\uc6d0\uc73c\ub85c \uc785\uc99d\ub418\uc5c8\ub2e4. \uc778\ub3c4\uc5d0\uc11c \uc218\ud589\ub41c \ud55c \uc5f0\uad6c\uc5d0\uc11c\ub294 '\uc720\uc758\ubbf8\ud55c \uc218'\uc758 \uc778\ub3c4\uc778\ub4e4\uc774 \uc774 \ubc14\uc774\ub7ec\uc2a4\uc5d0 \uba74\uc5ed \ubc18\uc751\uc744 \ub098\ud0c0\ub0b8\ub2e4\ub294 \uac83\uc744 \ubcf4\uc5ec\uc92c\ub294\ub370, \uc774\ub294 \uc774 \ubc14\uc774\ub7ec\uc2a4\uac00 \uc778\uac04 \uac1c\uccb4\uad70 \ub0b4\uc5d0\uc11c \uc624\ub7ab\ub3d9\uc548 \ub110\ub9ac \ud37c\uc838 \uc788\uc5c8\ub2e4\ub294 \uac83\uc744 \uc554\uc2dc\ud55c\ub2e4."} {"question": "\ubcc4\uc77c \uc5c6\uc774 \uc0b0\ub2e4 \uc568\ubc94\uc758 \ud6c4\ubc18\ubd80 \ub179\uc74c\uc740 \uc5bc\ub9c8\ub098 \uac78\ub838\uc2b5\ub2c8\uae4c?", "paragraph": "\uc7a5\uae30\ud558\uc758 \ub9d0\uc5d0 \ub530\ub974\uba74, \ub300\ubd80\ubd84\uc758 \ub179\uc74c \uc791\uc5c5\uc740 \uc11c\uc6b8\ud2b9\ubcc4\uc2dc \uc465\uace0\uac1c\uc5d0 \uc704\uce58\ud55c \uc220\ud0c4 \uc624\ube0c \ub354 \ub514\uc2a4\ucf54 \uba64\ubc84 \ub098\uc7a0 \uc218\uc758 \ubc29\uc5d0\uc11c \uc9c4\ud589\ub418\uc5c8\uace0 \uc608\uc678\uc801\uc73c\ub85c \ud1a4 \uc2a4\ud29c\ub514\uc624\uc5d0\uc11c \ub4dc\ub7fc \uc18c\uc2a4\uc640 \uc77c\ubd80 \ubcf4\uceec \uc18c\uc2a4\ub85c \uba87 \uace1\uc744 \uc791\uc5c5\ud588\ub2e4\uace0 \ud55c\ub2e4. \ub4dc\ubb38\ub4dc\ubb38, \uadf8\ub9ac\uace0 \uc7a5\uae30\ud558\uc758 \ud45c\ud604\ub300\ub85c \ud6c4\ubc18\ubd80\uc5d0\uc11c '\ube61\uc138\uac8c' \uc774\ub904\uc9c4 \ub179\uc74c\uc740 \ub300\ub7b5 \ubc18\ub144 \uc815\ub3c4\uc758 \uc2dc\uac04\uc774 \uac78\ub838\ub2e4. \ud648 \ub808\ucf54\ub529\uc744 \ud558\ub294 \uc774\uc720\uc5d0 \ub300\ud574\uc11c \uc7a5\uae30\ud558\ub294 \"\ud2b9\ubcc4\ud788 \ubc29\uc758 \uc5b4\ub5a4 \ud2b9\uc131 \ub54c\ubb38\uc740 \uc544\ub2c8\"\uba70 \"\uc544\ubb34\ub798\ub3c4 \ub204\uad70\uac00\uc640 \uc788\uac70\ub098 \uc0ac\ub78c\uc774 \ub9ce\uc73c\uba74 \uc2e0\uacbd \uc4f0\uc5ec\uc11c.\"\ub77c\uace0 \uc124\uba85\ud588\ub2e4. \ub610\ud55c \ud504\ub85c\ub4c0\uc11c\ub85c \ud611\uc5c5\ud55c \ub098\uc7a0 \uc218\uc5d0 \ub300\ud574\uc11c\ub294 \"... \ub9d0\uc774 \uad49\uc7a5\ud788 \uc798 \ud1b5\ud588\ub2e4. \ub0b4\uac00 \uc774\ub7f0 \uc810\uc774 \ubc14\ub00c\uc5c8\uc73c\uba74 \uc88b\uaca0\ub2e4\uace0 \ud558\uba74 \uadf8\uac78 \uae08\ubc29 \uce90\uce58\ud55c\ub2e4. \ub098\ub294 \ud504\ub85c\uadf8\ub7a8\uc774\ub098 \uc774\ub7f0 \uac78 \uc870\uc791\ud558\ub294 \uac78 \ubaa8\ub974\ub2c8\uae4c \ucd94\uc0c1\uc801\uc73c\ub85c '\ubb34\uc5b8\uac00 \ube44\uc5b4\uc788\ub294 \ub290\ub08c\uc774\ub2e4'\ub77c\ub294 \uc2dd\uc73c\ub85c \ub9d0\uc744 \ud558\uba74 \ub098\uc7a0\uc218\ub294 \uadf8\uac8c \uc774\ub860\uc801\uc73c\ub85c \uc5b4\ub5a4 \ubd80\ubd84\uc744 \uac74\ub4dc\ub824\uc57c \ud558\ub294\uc9c0 \ucc3e\uc544\uc11c \uc218\uc815\ud588\ub2e4. \ud53c\ub4dc\ubc31\uc774 \uad49\uc7a5\ud788 \ube68\ub790\ub2e4.\"\uace0 \ub9d0\ud588\ub2e4.", "answer": "\ubc18\ub144", "sentence": "\ub4dc\ubb38\ub4dc\ubb38, \uadf8\ub9ac\uace0 \uc7a5\uae30\ud558\uc758 \ud45c\ud604\ub300\ub85c \ud6c4\ubc18\ubd80\uc5d0\uc11c '\ube61\uc138\uac8c' \uc774\ub904\uc9c4 \ub179\uc74c\uc740 \ub300\ub7b5 \ubc18\ub144 \uc815\ub3c4\uc758 \uc2dc\uac04\uc774 \uac78\ub838\ub2e4.", "paragraph_sentence": "\uc7a5\uae30\ud558\uc758 \ub9d0\uc5d0 \ub530\ub974\uba74, \ub300\ubd80\ubd84\uc758 \ub179\uc74c \uc791\uc5c5\uc740 \uc11c\uc6b8\ud2b9\ubcc4\uc2dc \uc465\uace0\uac1c\uc5d0 \uc704\uce58\ud55c \uc220\ud0c4 \uc624\ube0c \ub354 \ub514\uc2a4\ucf54 \uba64\ubc84 \ub098\uc7a0 \uc218\uc758 \ubc29\uc5d0\uc11c \uc9c4\ud589\ub418\uc5c8\uace0 \uc608\uc678\uc801\uc73c\ub85c \ud1a4 \uc2a4\ud29c\ub514\uc624\uc5d0\uc11c \ub4dc\ub7fc \uc18c\uc2a4\uc640 \uc77c\ubd80 \ubcf4\uceec \uc18c\uc2a4\ub85c \uba87 \uace1\uc744 \uc791\uc5c5\ud588\ub2e4\uace0 \ud55c\ub2e4. \ub4dc\ubb38\ub4dc\ubb38, \uadf8\ub9ac\uace0 \uc7a5\uae30\ud558\uc758 \ud45c\ud604\ub300\ub85c \ud6c4\ubc18\ubd80\uc5d0\uc11c '\ube61\uc138\uac8c' \uc774\ub904\uc9c4 \ub179\uc74c\uc740 \ub300\ub7b5 \ubc18\ub144 \uc815\ub3c4\uc758 \uc2dc\uac04\uc774 \uac78\ub838\ub2e4. \ud648 \ub808\ucf54\ub529\uc744 \ud558\ub294 \uc774\uc720\uc5d0 \ub300\ud574\uc11c \uc7a5\uae30\ud558\ub294 \"\ud2b9\ubcc4\ud788 \ubc29\uc758 \uc5b4\ub5a4 \ud2b9\uc131 \ub54c\ubb38\uc740 \uc544\ub2c8\"\uba70 \"\uc544\ubb34\ub798\ub3c4 \ub204\uad70\uac00\uc640 \uc788\uac70\ub098 \uc0ac\ub78c\uc774 \ub9ce\uc73c\uba74 \uc2e0\uacbd \uc4f0\uc5ec\uc11c. \"\ub77c\uace0 \uc124\uba85\ud588\ub2e4. \ub610\ud55c \ud504\ub85c\ub4c0\uc11c\ub85c \ud611\uc5c5\ud55c \ub098\uc7a0 \uc218\uc5d0 \ub300\ud574\uc11c\ub294 \"... \ub9d0\uc774 \uad49\uc7a5\ud788 \uc798 \ud1b5\ud588\ub2e4. \ub0b4\uac00 \uc774\ub7f0 \uc810\uc774 \ubc14\ub00c\uc5c8\uc73c\uba74 \uc88b\uaca0\ub2e4\uace0 \ud558\uba74 \uadf8\uac78 \uae08\ubc29 \uce90\uce58\ud55c\ub2e4. \ub098\ub294 \ud504\ub85c\uadf8\ub7a8\uc774\ub098 \uc774\ub7f0 \uac78 \uc870\uc791\ud558\ub294 \uac78 \ubaa8\ub974\ub2c8\uae4c \ucd94\uc0c1\uc801\uc73c\ub85c '\ubb34\uc5b8\uac00 \ube44\uc5b4\uc788\ub294 \ub290\ub08c\uc774\ub2e4'\ub77c\ub294 \uc2dd\uc73c\ub85c \ub9d0\uc744 \ud558\uba74 \ub098\uc7a0\uc218\ub294 \uadf8\uac8c \uc774\ub860\uc801\uc73c\ub85c \uc5b4\ub5a4 \ubd80\ubd84\uc744 \uac74\ub4dc\ub824\uc57c \ud558\ub294\uc9c0 \ucc3e\uc544\uc11c \uc218\uc815\ud588\ub2e4. \ud53c\ub4dc\ubc31\uc774 \uad49\uc7a5\ud788 \ube68\ub790\ub2e4. \"\uace0 \ub9d0\ud588\ub2e4.", "paragraph_answer": "\uc7a5\uae30\ud558\uc758 \ub9d0\uc5d0 \ub530\ub974\uba74, \ub300\ubd80\ubd84\uc758 \ub179\uc74c \uc791\uc5c5\uc740 \uc11c\uc6b8\ud2b9\ubcc4\uc2dc \uc465\uace0\uac1c\uc5d0 \uc704\uce58\ud55c \uc220\ud0c4 \uc624\ube0c \ub354 \ub514\uc2a4\ucf54 \uba64\ubc84 \ub098\uc7a0 \uc218\uc758 \ubc29\uc5d0\uc11c \uc9c4\ud589\ub418\uc5c8\uace0 \uc608\uc678\uc801\uc73c\ub85c \ud1a4 \uc2a4\ud29c\ub514\uc624\uc5d0\uc11c \ub4dc\ub7fc \uc18c\uc2a4\uc640 \uc77c\ubd80 \ubcf4\uceec \uc18c\uc2a4\ub85c \uba87 \uace1\uc744 \uc791\uc5c5\ud588\ub2e4\uace0 \ud55c\ub2e4. \ub4dc\ubb38\ub4dc\ubb38, \uadf8\ub9ac\uace0 \uc7a5\uae30\ud558\uc758 \ud45c\ud604\ub300\ub85c \ud6c4\ubc18\ubd80\uc5d0\uc11c '\ube61\uc138\uac8c' \uc774\ub904\uc9c4 \ub179\uc74c\uc740 \ub300\ub7b5 \ubc18\ub144 \uc815\ub3c4\uc758 \uc2dc\uac04\uc774 \uac78\ub838\ub2e4. \ud648 \ub808\ucf54\ub529\uc744 \ud558\ub294 \uc774\uc720\uc5d0 \ub300\ud574\uc11c \uc7a5\uae30\ud558\ub294 \"\ud2b9\ubcc4\ud788 \ubc29\uc758 \uc5b4\ub5a4 \ud2b9\uc131 \ub54c\ubb38\uc740 \uc544\ub2c8\"\uba70 \"\uc544\ubb34\ub798\ub3c4 \ub204\uad70\uac00\uc640 \uc788\uac70\ub098 \uc0ac\ub78c\uc774 \ub9ce\uc73c\uba74 \uc2e0\uacbd \uc4f0\uc5ec\uc11c.\"\ub77c\uace0 \uc124\uba85\ud588\ub2e4. \ub610\ud55c \ud504\ub85c\ub4c0\uc11c\ub85c \ud611\uc5c5\ud55c \ub098\uc7a0 \uc218\uc5d0 \ub300\ud574\uc11c\ub294 \"... \ub9d0\uc774 \uad49\uc7a5\ud788 \uc798 \ud1b5\ud588\ub2e4. \ub0b4\uac00 \uc774\ub7f0 \uc810\uc774 \ubc14\ub00c\uc5c8\uc73c\uba74 \uc88b\uaca0\ub2e4\uace0 \ud558\uba74 \uadf8\uac78 \uae08\ubc29 \uce90\uce58\ud55c\ub2e4. \ub098\ub294 \ud504\ub85c\uadf8\ub7a8\uc774\ub098 \uc774\ub7f0 \uac78 \uc870\uc791\ud558\ub294 \uac78 \ubaa8\ub974\ub2c8\uae4c \ucd94\uc0c1\uc801\uc73c\ub85c '\ubb34\uc5b8\uac00 \ube44\uc5b4\uc788\ub294 \ub290\ub08c\uc774\ub2e4'\ub77c\ub294 \uc2dd\uc73c\ub85c \ub9d0\uc744 \ud558\uba74 \ub098\uc7a0\uc218\ub294 \uadf8\uac8c \uc774\ub860\uc801\uc73c\ub85c \uc5b4\ub5a4 \ubd80\ubd84\uc744 \uac74\ub4dc\ub824\uc57c \ud558\ub294\uc9c0 \ucc3e\uc544\uc11c \uc218\uc815\ud588\ub2e4. \ud53c\ub4dc\ubc31\uc774 \uad49\uc7a5\ud788 \ube68\ub790\ub2e4.\"\uace0 \ub9d0\ud588\ub2e4.", "sentence_answer": "\ub4dc\ubb38\ub4dc\ubb38, \uadf8\ub9ac\uace0 \uc7a5\uae30\ud558\uc758 \ud45c\ud604\ub300\ub85c \ud6c4\ubc18\ubd80\uc5d0\uc11c '\ube61\uc138\uac8c' \uc774\ub904\uc9c4 \ub179\uc74c\uc740 \ub300\ub7b5 \ubc18\ub144 \uc815\ub3c4\uc758 \uc2dc\uac04\uc774 \uac78\ub838\ub2e4.", "paragraph_id": "6508646-5-0", "paragraph_question": "question: \ubcc4\uc77c \uc5c6\uc774 \uc0b0\ub2e4 \uc568\ubc94\uc758 \ud6c4\ubc18\ubd80 \ub179\uc74c\uc740 \uc5bc\ub9c8\ub098 \uac78\ub838\uc2b5\ub2c8\uae4c?, context: \uc7a5\uae30\ud558\uc758 \ub9d0\uc5d0 \ub530\ub974\uba74, \ub300\ubd80\ubd84\uc758 \ub179\uc74c \uc791\uc5c5\uc740 \uc11c\uc6b8\ud2b9\ubcc4\uc2dc \uc465\uace0\uac1c\uc5d0 \uc704\uce58\ud55c \uc220\ud0c4 \uc624\ube0c \ub354 \ub514\uc2a4\ucf54 \uba64\ubc84 \ub098\uc7a0 \uc218\uc758 \ubc29\uc5d0\uc11c \uc9c4\ud589\ub418\uc5c8\uace0 \uc608\uc678\uc801\uc73c\ub85c \ud1a4 \uc2a4\ud29c\ub514\uc624\uc5d0\uc11c \ub4dc\ub7fc \uc18c\uc2a4\uc640 \uc77c\ubd80 \ubcf4\uceec \uc18c\uc2a4\ub85c \uba87 \uace1\uc744 \uc791\uc5c5\ud588\ub2e4\uace0 \ud55c\ub2e4. \ub4dc\ubb38\ub4dc\ubb38, \uadf8\ub9ac\uace0 \uc7a5\uae30\ud558\uc758 \ud45c\ud604\ub300\ub85c \ud6c4\ubc18\ubd80\uc5d0\uc11c '\ube61\uc138\uac8c' \uc774\ub904\uc9c4 \ub179\uc74c\uc740 \ub300\ub7b5 \ubc18\ub144 \uc815\ub3c4\uc758 \uc2dc\uac04\uc774 \uac78\ub838\ub2e4. \ud648 \ub808\ucf54\ub529\uc744 \ud558\ub294 \uc774\uc720\uc5d0 \ub300\ud574\uc11c \uc7a5\uae30\ud558\ub294 \"\ud2b9\ubcc4\ud788 \ubc29\uc758 \uc5b4\ub5a4 \ud2b9\uc131 \ub54c\ubb38\uc740 \uc544\ub2c8\"\uba70 \"\uc544\ubb34\ub798\ub3c4 \ub204\uad70\uac00\uc640 \uc788\uac70\ub098 \uc0ac\ub78c\uc774 \ub9ce\uc73c\uba74 \uc2e0\uacbd \uc4f0\uc5ec\uc11c.\"\ub77c\uace0 \uc124\uba85\ud588\ub2e4. \ub610\ud55c \ud504\ub85c\ub4c0\uc11c\ub85c \ud611\uc5c5\ud55c \ub098\uc7a0 \uc218\uc5d0 \ub300\ud574\uc11c\ub294 \"... \ub9d0\uc774 \uad49\uc7a5\ud788 \uc798 \ud1b5\ud588\ub2e4. \ub0b4\uac00 \uc774\ub7f0 \uc810\uc774 \ubc14\ub00c\uc5c8\uc73c\uba74 \uc88b\uaca0\ub2e4\uace0 \ud558\uba74 \uadf8\uac78 \uae08\ubc29 \uce90\uce58\ud55c\ub2e4. \ub098\ub294 \ud504\ub85c\uadf8\ub7a8\uc774\ub098 \uc774\ub7f0 \uac78 \uc870\uc791\ud558\ub294 \uac78 \ubaa8\ub974\ub2c8\uae4c \ucd94\uc0c1\uc801\uc73c\ub85c '\ubb34\uc5b8\uac00 \ube44\uc5b4\uc788\ub294 \ub290\ub08c\uc774\ub2e4'\ub77c\ub294 \uc2dd\uc73c\ub85c \ub9d0\uc744 \ud558\uba74 \ub098\uc7a0\uc218\ub294 \uadf8\uac8c \uc774\ub860\uc801\uc73c\ub85c \uc5b4\ub5a4 \ubd80\ubd84\uc744 \uac74\ub4dc\ub824\uc57c \ud558\ub294\uc9c0 \ucc3e\uc544\uc11c \uc218\uc815\ud588\ub2e4. \ud53c\ub4dc\ubc31\uc774 \uad49\uc7a5\ud788 \ube68\ub790\ub2e4.\"\uace0 \ub9d0\ud588\ub2e4."} {"question": "\ud3a0\ub9ac\uc2a4 \ub3c4\uba54\uc2a4\ud2f0\ucfe0\uc2a4 \uccb4\uc2a4\ud130 \uc70c\ub7ec\ub4dc\uc758 \uc885\uc740?", "paragraph": "\ud3a0\ub9ac\uc2a4 \ub3c4\uba54\uc2a4\ud2f0\ucfe0\uc2a4 \uccb4\uc2a4\ud130 \uc70c\ub7ec\ub4dc(Felis Domesticus Chester Willard, 1968\ub144 ~ 1982\ub144)\ub294 \ubbf8\uad6d \ubbf8\uc2dc\uac74 \uc8fc\ub9bd\ub300\ud559 \uc18c\uc18d \ubb3c\ub9ac\ud559\uc790 \uc7ad \ud5e4\ub354\ub9c1\ud134(Jack H. Hetherington)\uc774 \ud0a4\uc6b0\ub358 \uc0f4\uace0\uc591\uc774\ub2e4. \ubbf8\uad6d \ubbf8\uc2dc\uac74 \uc8fc \uc789\uc5c4 \uad70 \ud640\ud2b8 \ube44\ubc95\uc778\uc9c0\uad6c\uc5d0\uc11c \ud0dc\uc5b4\ub0ac\ub2e4. 1975\ub144 \ud5e4\ub354\ub9c1\ud134\uc740 \ud559\uc220\uc9c0 \ud53c\uc9c0\uceec \ub9ac\ubdf0\uc5d0 \uc800\uc628\ubb3c\ub9ac\ud559\uc5d0 \uad00\ud55c \uc790\uae30 \uc5f0\uad6c \uc131\uacfc\ub97c \ub17c\ubb38\uc73c\ub85c \ud22c\uace0\ud558\ub824\uace0 \ud588\ub294\ub370, \ud22c\uace0 \uc804\uc5d0 \uc6d0\uace0\ub97c \uc810\uac80\ud574\uc900 \ub3d9\ub8cc\uac00 \ud5e4\ub354\ub9c1\ud134\uc774 \"\ub098(I)\"\ub97c \uc0ac\uc6a9\ud574\uc57c \ud560 \uacf3\uc5d0 \"\uc6b0\ub9ac(We)\"\ub77c\ub294 \ub9d0\uc744 \uc0ac\uc6a9\ud588\ub2e4\ub294 \uc810\uc744 \uc9c0\uc801\ud588\ub2e4. \uc218\uc815\uc774 \uadc0\ucc2e\uc558\ub358 \ud5e4\ub354\ub9c1\ud134\uc740 \uc790\uae30 \uace0\uc591\uc774\ub97c \uacf5\uc800\uc790\ub85c \uc62c\ub838\ub294\ub370, \uc774\uac83\uc774 \ubc14\ub85c \uc124\uce58\ub958 \ud3ec\uc2dd \ucee8\uc124\ud134\ud2b8(Rodentia Predation Consultant)\uc774\uc790 \ubb3c\ub9ac\ud559\uc790\uc778 \uc18c\uc704 \uccb4\uc2a4\ud130 \uc70c\ub7ec\ub4dc \ubc15\uc0ac\ub2e4.", "answer": "\uc0f4\uace0\uc591\uc774", "sentence": "\ud3a0\ub9ac\uc2a4 \ub3c4\uba54\uc2a4\ud2f0\ucfe0\uc2a4 \uccb4\uc2a4\ud130 \uc70c\ub7ec\ub4dc(Felis Domesticus Chester Willard, 1968\ub144 ~ 1982\ub144)\ub294 \ubbf8\uad6d \ubbf8\uc2dc\uac74 \uc8fc\ub9bd\ub300\ud559 \uc18c\uc18d \ubb3c\ub9ac\ud559\uc790 \uc7ad \ud5e4\ub354\ub9c1\ud134(Jack H. Hetherington)\uc774 \ud0a4\uc6b0\ub358 \uc0f4\uace0\uc591\uc774 \ub2e4.", "paragraph_sentence": " \ud3a0\ub9ac\uc2a4 \ub3c4\uba54\uc2a4\ud2f0\ucfe0\uc2a4 \uccb4\uc2a4\ud130 \uc70c\ub7ec\ub4dc(Felis Domesticus Chester Willard, 1968\ub144 ~ 1982\ub144)\ub294 \ubbf8\uad6d \ubbf8\uc2dc\uac74 \uc8fc\ub9bd\ub300\ud559 \uc18c\uc18d \ubb3c\ub9ac\ud559\uc790 \uc7ad \ud5e4\ub354\ub9c1\ud134(Jack H. Hetherington)\uc774 \ud0a4\uc6b0\ub358 \uc0f4\uace0\uc591\uc774 \ub2e4. \ubbf8\uad6d \ubbf8\uc2dc\uac74 \uc8fc \uc789\uc5c4 \uad70 \ud640\ud2b8 \ube44\ubc95\uc778\uc9c0\uad6c\uc5d0\uc11c \ud0dc\uc5b4\ub0ac\ub2e4. 1975\ub144 \ud5e4\ub354\ub9c1\ud134\uc740 \ud559\uc220\uc9c0 \ud53c\uc9c0\uceec \ub9ac\ubdf0\uc5d0 \uc800\uc628\ubb3c\ub9ac\ud559\uc5d0 \uad00\ud55c \uc790\uae30 \uc5f0\uad6c \uc131\uacfc\ub97c \ub17c\ubb38\uc73c\ub85c \ud22c\uace0\ud558\ub824\uace0 \ud588\ub294\ub370, \ud22c\uace0 \uc804\uc5d0 \uc6d0\uace0\ub97c \uc810\uac80\ud574\uc900 \ub3d9\ub8cc\uac00 \ud5e4\ub354\ub9c1\ud134\uc774 \"\ub098(I)\"\ub97c \uc0ac\uc6a9\ud574\uc57c \ud560 \uacf3\uc5d0 \"\uc6b0\ub9ac(We)\"\ub77c\ub294 \ub9d0\uc744 \uc0ac\uc6a9\ud588\ub2e4\ub294 \uc810\uc744 \uc9c0\uc801\ud588\ub2e4. \uc218\uc815\uc774 \uadc0\ucc2e\uc558\ub358 \ud5e4\ub354\ub9c1\ud134\uc740 \uc790\uae30 \uace0\uc591\uc774\ub97c \uacf5\uc800\uc790\ub85c \uc62c\ub838\ub294\ub370, \uc774\uac83\uc774 \ubc14\ub85c \uc124\uce58\ub958 \ud3ec\uc2dd \ucee8\uc124\ud134\ud2b8(Rodentia Predation Consultant)\uc774\uc790 \ubb3c\ub9ac\ud559\uc790\uc778 \uc18c\uc704 \uccb4\uc2a4\ud130 \uc70c\ub7ec\ub4dc \ubc15\uc0ac\ub2e4.", "paragraph_answer": "\ud3a0\ub9ac\uc2a4 \ub3c4\uba54\uc2a4\ud2f0\ucfe0\uc2a4 \uccb4\uc2a4\ud130 \uc70c\ub7ec\ub4dc(Felis Domesticus Chester Willard, 1968\ub144 ~ 1982\ub144)\ub294 \ubbf8\uad6d \ubbf8\uc2dc\uac74 \uc8fc\ub9bd\ub300\ud559 \uc18c\uc18d \ubb3c\ub9ac\ud559\uc790 \uc7ad \ud5e4\ub354\ub9c1\ud134(Jack H. Hetherington)\uc774 \ud0a4\uc6b0\ub358 \uc0f4\uace0\uc591\uc774 \ub2e4. \ubbf8\uad6d \ubbf8\uc2dc\uac74 \uc8fc \uc789\uc5c4 \uad70 \ud640\ud2b8 \ube44\ubc95\uc778\uc9c0\uad6c\uc5d0\uc11c \ud0dc\uc5b4\ub0ac\ub2e4. 1975\ub144 \ud5e4\ub354\ub9c1\ud134\uc740 \ud559\uc220\uc9c0 \ud53c\uc9c0\uceec \ub9ac\ubdf0\uc5d0 \uc800\uc628\ubb3c\ub9ac\ud559\uc5d0 \uad00\ud55c \uc790\uae30 \uc5f0\uad6c \uc131\uacfc\ub97c \ub17c\ubb38\uc73c\ub85c \ud22c\uace0\ud558\ub824\uace0 \ud588\ub294\ub370, \ud22c\uace0 \uc804\uc5d0 \uc6d0\uace0\ub97c \uc810\uac80\ud574\uc900 \ub3d9\ub8cc\uac00 \ud5e4\ub354\ub9c1\ud134\uc774 \"\ub098(I)\"\ub97c \uc0ac\uc6a9\ud574\uc57c \ud560 \uacf3\uc5d0 \"\uc6b0\ub9ac(We)\"\ub77c\ub294 \ub9d0\uc744 \uc0ac\uc6a9\ud588\ub2e4\ub294 \uc810\uc744 \uc9c0\uc801\ud588\ub2e4. \uc218\uc815\uc774 \uadc0\ucc2e\uc558\ub358 \ud5e4\ub354\ub9c1\ud134\uc740 \uc790\uae30 \uace0\uc591\uc774\ub97c \uacf5\uc800\uc790\ub85c \uc62c\ub838\ub294\ub370, \uc774\uac83\uc774 \ubc14\ub85c \uc124\uce58\ub958 \ud3ec\uc2dd \ucee8\uc124\ud134\ud2b8(Rodentia Predation Consultant)\uc774\uc790 \ubb3c\ub9ac\ud559\uc790\uc778 \uc18c\uc704 \uccb4\uc2a4\ud130 \uc70c\ub7ec\ub4dc \ubc15\uc0ac\ub2e4.", "sentence_answer": "\ud3a0\ub9ac\uc2a4 \ub3c4\uba54\uc2a4\ud2f0\ucfe0\uc2a4 \uccb4\uc2a4\ud130 \uc70c\ub7ec\ub4dc(Felis Domesticus Chester Willard, 1968\ub144 ~ 1982\ub144)\ub294 \ubbf8\uad6d \ubbf8\uc2dc\uac74 \uc8fc\ub9bd\ub300\ud559 \uc18c\uc18d \ubb3c\ub9ac\ud559\uc790 \uc7ad \ud5e4\ub354\ub9c1\ud134(Jack H. Hetherington)\uc774 \ud0a4\uc6b0\ub358 \uc0f4\uace0\uc591\uc774 \ub2e4.", "paragraph_id": "6559634-0-0", "paragraph_question": "question: \ud3a0\ub9ac\uc2a4 \ub3c4\uba54\uc2a4\ud2f0\ucfe0\uc2a4 \uccb4\uc2a4\ud130 \uc70c\ub7ec\ub4dc\uc758 \uc885\uc740?, context: \ud3a0\ub9ac\uc2a4 \ub3c4\uba54\uc2a4\ud2f0\ucfe0\uc2a4 \uccb4\uc2a4\ud130 \uc70c\ub7ec\ub4dc(Felis Domesticus Chester Willard, 1968\ub144 ~ 1982\ub144)\ub294 \ubbf8\uad6d \ubbf8\uc2dc\uac74 \uc8fc\ub9bd\ub300\ud559 \uc18c\uc18d \ubb3c\ub9ac\ud559\uc790 \uc7ad \ud5e4\ub354\ub9c1\ud134(Jack H. Hetherington)\uc774 \ud0a4\uc6b0\ub358 \uc0f4\uace0\uc591\uc774\ub2e4. \ubbf8\uad6d \ubbf8\uc2dc\uac74 \uc8fc \uc789\uc5c4 \uad70 \ud640\ud2b8 \ube44\ubc95\uc778\uc9c0\uad6c\uc5d0\uc11c \ud0dc\uc5b4\ub0ac\ub2e4. 1975\ub144 \ud5e4\ub354\ub9c1\ud134\uc740 \ud559\uc220\uc9c0 \ud53c\uc9c0\uceec \ub9ac\ubdf0\uc5d0 \uc800\uc628\ubb3c\ub9ac\ud559\uc5d0 \uad00\ud55c \uc790\uae30 \uc5f0\uad6c \uc131\uacfc\ub97c \ub17c\ubb38\uc73c\ub85c \ud22c\uace0\ud558\ub824\uace0 \ud588\ub294\ub370, \ud22c\uace0 \uc804\uc5d0 \uc6d0\uace0\ub97c \uc810\uac80\ud574\uc900 \ub3d9\ub8cc\uac00 \ud5e4\ub354\ub9c1\ud134\uc774 \"\ub098(I)\"\ub97c \uc0ac\uc6a9\ud574\uc57c \ud560 \uacf3\uc5d0 \"\uc6b0\ub9ac(We)\"\ub77c\ub294 \ub9d0\uc744 \uc0ac\uc6a9\ud588\ub2e4\ub294 \uc810\uc744 \uc9c0\uc801\ud588\ub2e4. \uc218\uc815\uc774 \uadc0\ucc2e\uc558\ub358 \ud5e4\ub354\ub9c1\ud134\uc740 \uc790\uae30 \uace0\uc591\uc774\ub97c \uacf5\uc800\uc790\ub85c \uc62c\ub838\ub294\ub370, \uc774\uac83\uc774 \ubc14\ub85c \uc124\uce58\ub958 \ud3ec\uc2dd \ucee8\uc124\ud134\ud2b8(Rodentia Predation Consultant)\uc774\uc790 \ubb3c\ub9ac\ud559\uc790\uc778 \uc18c\uc704 \uccb4\uc2a4\ud130 \uc70c\ub7ec\ub4dc \ubc15\uc0ac\ub2e4."} {"question": "\ub9ac\ube44\ud2b8\uac00 \ud0dc\uc5b4\ub09c \ub144\ub3c4\ub294?", "paragraph": "\ud5e8\ub9ac\uc5d0\ud0c0 \uc2a4\uc644 \ub9ac\ube44\ud2b8(Henrietta Swan Leavitt, 1868\ub144 7\uc6d4 4\uc77c ~ 1921\ub144 12\uc6d4 12\uc77c)\ub294 \ubbf8\uad6d\uc758 \ucc9c\ubb38\ud559\uc790\uc774\ub2e4. \ub798\ub4dc\ud074\ub9ac\ud504 \ub300\ud559\uc744 \uc878\uc5c5\ud558\uace0 1893\ub144\ubd80\ud130 \ud558\ubc84\ub4dc \ub300\ud559\uad50 \ucc9c\ubb38\ub300\uc5d0\uc11c \ud56d\uc131\uc758 \ubc1d\uae30\ub97c \uc815\ub9ac\ud558\uae30 \uc704\ud558\uc5ec \uc0ac\uc9c4\uac74\ud310\uc744 \uac80\uc0ac\ud558\ub294 \uacc4\uc0b0\uc218\ub85c \uadfc\ubb34\ud588\ub2e4. \ub9ac\ube44\ud2b8\ub294 \uc138\ud398\uc774\ub4dc\ud615 \ubcc0\uad11\uc131\uc758 \ubcc0\uad11 \uc8fc\uae30\uc640 \uad11\ub3c4 \uc0ac\uc774\uc5d0 \uc0c1\uad00\uad00\uacc4\uac00 \uc788\ub2e4\ub294 \uac83\uc744 \ubc1c\uacac\ud588\ub2e4. \ub9ac\ube44\ud2b8\ub294 \uc0dd\uc804\uc5d0\ub294 \ud569\ub2f9\ud55c \uc778\uc815\uc744 \ubc1b\uc9c0 \ubabb\ud588\uc9c0\ub9cc, \uadf8\ub140\uc758 \uc774 \ubc1c\uacac\uc740 \uc9c0\uad6c\uc5d0\uc11c \uba40\ub9ac \ub5a8\uc5b4\uc838 \uc788\ub294 \uc740\ud558\ub4e4\uae4c\uc9c0\uc758 \uac70\ub9ac\ub97c \uce21\uc815\ud560 \uc218 \uc788\ub294 \ucd5c\ucd08\uc758 \ubc29\ubc95\uc744 \uc81c\uacf5\ud55c \uac83\uc774\uc5c8\ub2e4. \ub9ac\ube44\ud2b8 \uc0ac\ud6c4, \uc5d0\ub4dc\uc708 \ud5c8\ube14\uc774 \uc774 \uc8fc\uae30-\uad11\ub3c4 \uad00\uacc4\ub97c \uc774\uc6a9\ud558\uc5ec \uc6b0\uc8fc\uac00 \ud33d\ucc3d\ud558\uace0 \uc788\uc74c(\ud5c8\ube14\uc758 \ubc95\uce59)\uc744 \ubc1d\ud600\ub0b4\uac8c \ub41c\ub2e4.", "answer": "1868\ub144", "sentence": "\ud5e8\ub9ac\uc5d0\ud0c0 \uc2a4\uc644 \ub9ac\ube44\ud2b8(Henrietta Swan Leavitt, 1868\ub144 7\uc6d4 4\uc77c ~ 1921\ub144 12\uc6d4 12\uc77c)\ub294 \ubbf8\uad6d\uc758 \ucc9c\ubb38\ud559\uc790\uc774\ub2e4.", "paragraph_sentence": " \ud5e8\ub9ac\uc5d0\ud0c0 \uc2a4\uc644 \ub9ac\ube44\ud2b8(Henrietta Swan Leavitt, 1868\ub144 7\uc6d4 4\uc77c ~ 1921\ub144 12\uc6d4 12\uc77c)\ub294 \ubbf8\uad6d\uc758 \ucc9c\ubb38\ud559\uc790\uc774\ub2e4. \ub798\ub4dc\ud074\ub9ac\ud504 \ub300\ud559\uc744 \uc878\uc5c5\ud558\uace0 1893\ub144\ubd80\ud130 \ud558\ubc84\ub4dc \ub300\ud559\uad50 \ucc9c\ubb38\ub300\uc5d0\uc11c \ud56d\uc131\uc758 \ubc1d\uae30\ub97c \uc815\ub9ac\ud558\uae30 \uc704\ud558\uc5ec \uc0ac\uc9c4\uac74\ud310\uc744 \uac80\uc0ac\ud558\ub294 \uacc4\uc0b0\uc218\ub85c \uadfc\ubb34\ud588\ub2e4. \ub9ac\ube44\ud2b8\ub294 \uc138\ud398\uc774\ub4dc\ud615 \ubcc0\uad11\uc131\uc758 \ubcc0\uad11 \uc8fc\uae30\uc640 \uad11\ub3c4 \uc0ac\uc774\uc5d0 \uc0c1\uad00\uad00\uacc4\uac00 \uc788\ub2e4\ub294 \uac83\uc744 \ubc1c\uacac\ud588\ub2e4. \ub9ac\ube44\ud2b8\ub294 \uc0dd\uc804\uc5d0\ub294 \ud569\ub2f9\ud55c \uc778\uc815\uc744 \ubc1b\uc9c0 \ubabb\ud588\uc9c0\ub9cc, \uadf8\ub140\uc758 \uc774 \ubc1c\uacac\uc740 \uc9c0\uad6c\uc5d0\uc11c \uba40\ub9ac \ub5a8\uc5b4\uc838 \uc788\ub294 \uc740\ud558\ub4e4\uae4c\uc9c0\uc758 \uac70\ub9ac\ub97c \uce21\uc815\ud560 \uc218 \uc788\ub294 \ucd5c\ucd08\uc758 \ubc29\ubc95\uc744 \uc81c\uacf5\ud55c \uac83\uc774\uc5c8\ub2e4. \ub9ac\ube44\ud2b8 \uc0ac\ud6c4, \uc5d0\ub4dc\uc708 \ud5c8\ube14\uc774 \uc774 \uc8fc\uae30-\uad11\ub3c4 \uad00\uacc4\ub97c \uc774\uc6a9\ud558\uc5ec \uc6b0\uc8fc\uac00 \ud33d\ucc3d\ud558\uace0 \uc788\uc74c(\ud5c8\ube14\uc758 \ubc95\uce59)\uc744 \ubc1d\ud600\ub0b4\uac8c \ub41c\ub2e4.", "paragraph_answer": "\ud5e8\ub9ac\uc5d0\ud0c0 \uc2a4\uc644 \ub9ac\ube44\ud2b8(Henrietta Swan Leavitt, 1868\ub144 7\uc6d4 4\uc77c ~ 1921\ub144 12\uc6d4 12\uc77c)\ub294 \ubbf8\uad6d\uc758 \ucc9c\ubb38\ud559\uc790\uc774\ub2e4. \ub798\ub4dc\ud074\ub9ac\ud504 \ub300\ud559\uc744 \uc878\uc5c5\ud558\uace0 1893\ub144\ubd80\ud130 \ud558\ubc84\ub4dc \ub300\ud559\uad50 \ucc9c\ubb38\ub300\uc5d0\uc11c \ud56d\uc131\uc758 \ubc1d\uae30\ub97c \uc815\ub9ac\ud558\uae30 \uc704\ud558\uc5ec \uc0ac\uc9c4\uac74\ud310\uc744 \uac80\uc0ac\ud558\ub294 \uacc4\uc0b0\uc218\ub85c \uadfc\ubb34\ud588\ub2e4. \ub9ac\ube44\ud2b8\ub294 \uc138\ud398\uc774\ub4dc\ud615 \ubcc0\uad11\uc131\uc758 \ubcc0\uad11 \uc8fc\uae30\uc640 \uad11\ub3c4 \uc0ac\uc774\uc5d0 \uc0c1\uad00\uad00\uacc4\uac00 \uc788\ub2e4\ub294 \uac83\uc744 \ubc1c\uacac\ud588\ub2e4. \ub9ac\ube44\ud2b8\ub294 \uc0dd\uc804\uc5d0\ub294 \ud569\ub2f9\ud55c \uc778\uc815\uc744 \ubc1b\uc9c0 \ubabb\ud588\uc9c0\ub9cc, \uadf8\ub140\uc758 \uc774 \ubc1c\uacac\uc740 \uc9c0\uad6c\uc5d0\uc11c \uba40\ub9ac \ub5a8\uc5b4\uc838 \uc788\ub294 \uc740\ud558\ub4e4\uae4c\uc9c0\uc758 \uac70\ub9ac\ub97c \uce21\uc815\ud560 \uc218 \uc788\ub294 \ucd5c\ucd08\uc758 \ubc29\ubc95\uc744 \uc81c\uacf5\ud55c \uac83\uc774\uc5c8\ub2e4. \ub9ac\ube44\ud2b8 \uc0ac\ud6c4, \uc5d0\ub4dc\uc708 \ud5c8\ube14\uc774 \uc774 \uc8fc\uae30-\uad11\ub3c4 \uad00\uacc4\ub97c \uc774\uc6a9\ud558\uc5ec \uc6b0\uc8fc\uac00 \ud33d\ucc3d\ud558\uace0 \uc788\uc74c(\ud5c8\ube14\uc758 \ubc95\uce59)\uc744 \ubc1d\ud600\ub0b4\uac8c \ub41c\ub2e4.", "sentence_answer": "\ud5e8\ub9ac\uc5d0\ud0c0 \uc2a4\uc644 \ub9ac\ube44\ud2b8(Henrietta Swan Leavitt, 1868\ub144 7\uc6d4 4\uc77c ~ 1921\ub144 12\uc6d4 12\uc77c)\ub294 \ubbf8\uad6d\uc758 \ucc9c\ubb38\ud559\uc790\uc774\ub2e4.", "paragraph_id": "6571435-0-1", "paragraph_question": "question: \ub9ac\ube44\ud2b8\uac00 \ud0dc\uc5b4\ub09c \ub144\ub3c4\ub294?, context: \ud5e8\ub9ac\uc5d0\ud0c0 \uc2a4\uc644 \ub9ac\ube44\ud2b8(Henrietta Swan Leavitt, 1868\ub144 7\uc6d4 4\uc77c ~ 1921\ub144 12\uc6d4 12\uc77c)\ub294 \ubbf8\uad6d\uc758 \ucc9c\ubb38\ud559\uc790\uc774\ub2e4. \ub798\ub4dc\ud074\ub9ac\ud504 \ub300\ud559\uc744 \uc878\uc5c5\ud558\uace0 1893\ub144\ubd80\ud130 \ud558\ubc84\ub4dc \ub300\ud559\uad50 \ucc9c\ubb38\ub300\uc5d0\uc11c \ud56d\uc131\uc758 \ubc1d\uae30\ub97c \uc815\ub9ac\ud558\uae30 \uc704\ud558\uc5ec \uc0ac\uc9c4\uac74\ud310\uc744 \uac80\uc0ac\ud558\ub294 \uacc4\uc0b0\uc218\ub85c \uadfc\ubb34\ud588\ub2e4. \ub9ac\ube44\ud2b8\ub294 \uc138\ud398\uc774\ub4dc\ud615 \ubcc0\uad11\uc131\uc758 \ubcc0\uad11 \uc8fc\uae30\uc640 \uad11\ub3c4 \uc0ac\uc774\uc5d0 \uc0c1\uad00\uad00\uacc4\uac00 \uc788\ub2e4\ub294 \uac83\uc744 \ubc1c\uacac\ud588\ub2e4. \ub9ac\ube44\ud2b8\ub294 \uc0dd\uc804\uc5d0\ub294 \ud569\ub2f9\ud55c \uc778\uc815\uc744 \ubc1b\uc9c0 \ubabb\ud588\uc9c0\ub9cc, \uadf8\ub140\uc758 \uc774 \ubc1c\uacac\uc740 \uc9c0\uad6c\uc5d0\uc11c \uba40\ub9ac \ub5a8\uc5b4\uc838 \uc788\ub294 \uc740\ud558\ub4e4\uae4c\uc9c0\uc758 \uac70\ub9ac\ub97c \uce21\uc815\ud560 \uc218 \uc788\ub294 \ucd5c\ucd08\uc758 \ubc29\ubc95\uc744 \uc81c\uacf5\ud55c \uac83\uc774\uc5c8\ub2e4. \ub9ac\ube44\ud2b8 \uc0ac\ud6c4, \uc5d0\ub4dc\uc708 \ud5c8\ube14\uc774 \uc774 \uc8fc\uae30-\uad11\ub3c4 \uad00\uacc4\ub97c \uc774\uc6a9\ud558\uc5ec \uc6b0\uc8fc\uac00 \ud33d\ucc3d\ud558\uace0 \uc788\uc74c(\ud5c8\ube14\uc758 \ubc95\uce59)\uc744 \ubc1d\ud600\ub0b4\uac8c \ub41c\ub2e4."} {"question": "\uc815\uc2dd\uc73c\ub85c \uc11c\ube44\uc2a4\ub97c \uc2dc\uc791\ud558\uba74\uc11c Pokemon GO\uc758 \uc2e4\ud589\uc774 \ucd94\uac00\ub41c \uacf3\uc740?", "paragraph": "Pok\u00e9mon GO\uac00 \uc815\uc2dd\uc73c\ub85c \ucd9c\uc2dc\ub418\uc9c0 \uc54a\uc558\ub358 \ub300\ud55c\ubbfc\uad6d\uc5d0\uc11c\ub294, iOS \ud574\uc678 \uc6b0\ud68c \uacc4\uc815\uc774\ub098 \uc548\ub4dc\ub85c\uc774\ub4dc \uc124\uce58\ud30c\uc77c(APK) \ub2e4\uc6b4\ub85c\ub4dc \ub610\ub294 Google Play \ud574\uc678 \uc6b0\ud68c \uacc4\uc815\uc73c\ub85c \uc2e4\ud589\ud55c Pok\u00e9mon GO\uac00 \uc720\uc77c\ud558\uac8c \uac15\uc6d0\ub3c4 \uc18d\ucd08\uc640 \uc591\uc591, \uace0\uc131\uad70, \uc6b8\ub989\ub3c4, \ub3c5\ub3c4\uc5d0\uc11c \uc0ac\uc6a9\uc774 \uac00\ub2a5\ud558\uc600\ub2e4. \uc774 \uc0ac\uc2e4\uc774 \uc18c\uc15c \ubbf8\ub514\uc5b4\ub97c \ud1b5\ud574 \uc54c\ub824\uc9c0\uc790, Pok\u00e9mon GO\ub97c \ud558\ub824\uace0 \uc18d\ucd08\ub85c \ubab0\ub824\uac00\ub294 \uc0ac\ub78c\ub4e4\ub85c \uc778\ud574 \uc77c\ubd80 \uc9c0\uc5ed\uc5d0\uc11c \ud3c9\uc77c \uc18d\ucd08\ud589 \uace0\uc18d\ubc84\uc2a4 \ud45c\uac00 \ub9e4\uc9c4\ub418\ub294 \uc0ac\ud0dc\ub3c4 \ubc8c\uc5b4\uc84c\ub2e4. Pok\u00e9mon GO\ub97c \ud50c\ub808\uc774 \ud558\uae30 \uc704\ud574 \uc18d\ucd08\ub85c \uc9d1\uacb0\ud55c \uc0ac\ub78c\ub4e4\uc740 \uc7a5\ub9db\ube44 \uc18d\uc5d0\uc11c\ub3c4 \ud3ec\ucf13\ubaac \uc7a1\uc774\uc5d0 \ubab0\ub450\ud558\uc5ec Pok\u00e9mon GO \uc5f4\ud48d\uc744 \ubcf4\uc5ec\uc8fc\uc5c8\ub2e4. \uc774\ud6c4 \uc77c\ubcf8\uc774 \uc815\uc2dd \uc11c\ube44\uc2a4\ub97c \uc2dc\uc791\ud558\uc790 \ucd94\uac00\ub85c \uc6b8\uc0b0 \uac04\uc808\uacf6\uc5d0\uc11c \uc0ac\uc6a9 \uac00\ub2a5\ud558\uac8c \ub418\uc5c8\ub2e4.", "answer": "\uc6b8\uc0b0 \uac04\uc808\uacf6", "sentence": "\uc774\ud6c4 \uc77c\ubcf8\uc774 \uc815\uc2dd \uc11c\ube44\uc2a4\ub97c \uc2dc\uc791\ud558\uc790 \ucd94\uac00\ub85c \uc6b8\uc0b0 \uac04\uc808\uacf6 \uc5d0\uc11c \uc0ac\uc6a9 \uac00\ub2a5\ud558\uac8c \ub418\uc5c8\ub2e4.", "paragraph_sentence": "Pok\u00e9mon GO\uac00 \uc815\uc2dd\uc73c\ub85c \ucd9c\uc2dc\ub418\uc9c0 \uc54a\uc558\ub358 \ub300\ud55c\ubbfc\uad6d\uc5d0\uc11c\ub294, iOS \ud574\uc678 \uc6b0\ud68c \uacc4\uc815\uc774\ub098 \uc548\ub4dc\ub85c\uc774\ub4dc \uc124\uce58\ud30c\uc77c(APK) \ub2e4\uc6b4\ub85c\ub4dc \ub610\ub294 Google Play \ud574\uc678 \uc6b0\ud68c \uacc4\uc815\uc73c\ub85c \uc2e4\ud589\ud55c Pok\u00e9mon GO\uac00 \uc720\uc77c\ud558\uac8c \uac15\uc6d0\ub3c4 \uc18d\ucd08\uc640 \uc591\uc591, \uace0\uc131\uad70, \uc6b8\ub989\ub3c4, \ub3c5\ub3c4\uc5d0\uc11c \uc0ac\uc6a9\uc774 \uac00\ub2a5\ud558\uc600\ub2e4. \uc774 \uc0ac\uc2e4\uc774 \uc18c\uc15c \ubbf8\ub514\uc5b4\ub97c \ud1b5\ud574 \uc54c\ub824\uc9c0\uc790, Pok\u00e9mon GO\ub97c \ud558\ub824\uace0 \uc18d\ucd08\ub85c \ubab0\ub824\uac00\ub294 \uc0ac\ub78c\ub4e4\ub85c \uc778\ud574 \uc77c\ubd80 \uc9c0\uc5ed\uc5d0\uc11c \ud3c9\uc77c \uc18d\ucd08\ud589 \uace0\uc18d\ubc84\uc2a4 \ud45c\uac00 \ub9e4\uc9c4\ub418\ub294 \uc0ac\ud0dc\ub3c4 \ubc8c\uc5b4\uc84c\ub2e4. Pok\u00e9mon GO\ub97c \ud50c\ub808\uc774 \ud558\uae30 \uc704\ud574 \uc18d\ucd08\ub85c \uc9d1\uacb0\ud55c \uc0ac\ub78c\ub4e4\uc740 \uc7a5\ub9db\ube44 \uc18d\uc5d0\uc11c\ub3c4 \ud3ec\ucf13\ubaac \uc7a1\uc774\uc5d0 \ubab0\ub450\ud558\uc5ec Pok\u00e9mon GO \uc5f4\ud48d\uc744 \ubcf4\uc5ec\uc8fc\uc5c8\ub2e4. \uc774\ud6c4 \uc77c\ubcf8\uc774 \uc815\uc2dd \uc11c\ube44\uc2a4\ub97c \uc2dc\uc791\ud558\uc790 \ucd94\uac00\ub85c \uc6b8\uc0b0 \uac04\uc808\uacf6 \uc5d0\uc11c \uc0ac\uc6a9 \uac00\ub2a5\ud558\uac8c \ub418\uc5c8\ub2e4. ", "paragraph_answer": "Pok\u00e9mon GO\uac00 \uc815\uc2dd\uc73c\ub85c \ucd9c\uc2dc\ub418\uc9c0 \uc54a\uc558\ub358 \ub300\ud55c\ubbfc\uad6d\uc5d0\uc11c\ub294, iOS \ud574\uc678 \uc6b0\ud68c \uacc4\uc815\uc774\ub098 \uc548\ub4dc\ub85c\uc774\ub4dc \uc124\uce58\ud30c\uc77c(APK) \ub2e4\uc6b4\ub85c\ub4dc \ub610\ub294 Google Play \ud574\uc678 \uc6b0\ud68c \uacc4\uc815\uc73c\ub85c \uc2e4\ud589\ud55c Pok\u00e9mon GO\uac00 \uc720\uc77c\ud558\uac8c \uac15\uc6d0\ub3c4 \uc18d\ucd08\uc640 \uc591\uc591, \uace0\uc131\uad70, \uc6b8\ub989\ub3c4, \ub3c5\ub3c4\uc5d0\uc11c \uc0ac\uc6a9\uc774 \uac00\ub2a5\ud558\uc600\ub2e4. \uc774 \uc0ac\uc2e4\uc774 \uc18c\uc15c \ubbf8\ub514\uc5b4\ub97c \ud1b5\ud574 \uc54c\ub824\uc9c0\uc790, Pok\u00e9mon GO\ub97c \ud558\ub824\uace0 \uc18d\ucd08\ub85c \ubab0\ub824\uac00\ub294 \uc0ac\ub78c\ub4e4\ub85c \uc778\ud574 \uc77c\ubd80 \uc9c0\uc5ed\uc5d0\uc11c \ud3c9\uc77c \uc18d\ucd08\ud589 \uace0\uc18d\ubc84\uc2a4 \ud45c\uac00 \ub9e4\uc9c4\ub418\ub294 \uc0ac\ud0dc\ub3c4 \ubc8c\uc5b4\uc84c\ub2e4. Pok\u00e9mon GO\ub97c \ud50c\ub808\uc774 \ud558\uae30 \uc704\ud574 \uc18d\ucd08\ub85c \uc9d1\uacb0\ud55c \uc0ac\ub78c\ub4e4\uc740 \uc7a5\ub9db\ube44 \uc18d\uc5d0\uc11c\ub3c4 \ud3ec\ucf13\ubaac \uc7a1\uc774\uc5d0 \ubab0\ub450\ud558\uc5ec Pok\u00e9mon GO \uc5f4\ud48d\uc744 \ubcf4\uc5ec\uc8fc\uc5c8\ub2e4. \uc774\ud6c4 \uc77c\ubcf8\uc774 \uc815\uc2dd \uc11c\ube44\uc2a4\ub97c \uc2dc\uc791\ud558\uc790 \ucd94\uac00\ub85c \uc6b8\uc0b0 \uac04\uc808\uacf6 \uc5d0\uc11c \uc0ac\uc6a9 \uac00\ub2a5\ud558\uac8c \ub418\uc5c8\ub2e4.", "sentence_answer": "\uc774\ud6c4 \uc77c\ubcf8\uc774 \uc815\uc2dd \uc11c\ube44\uc2a4\ub97c \uc2dc\uc791\ud558\uc790 \ucd94\uac00\ub85c \uc6b8\uc0b0 \uac04\uc808\uacf6 \uc5d0\uc11c \uc0ac\uc6a9 \uac00\ub2a5\ud558\uac8c \ub418\uc5c8\ub2e4.", "paragraph_id": "6470777-1-1", "paragraph_question": "question: \uc815\uc2dd\uc73c\ub85c \uc11c\ube44\uc2a4\ub97c \uc2dc\uc791\ud558\uba74\uc11c Pokemon GO\uc758 \uc2e4\ud589\uc774 \ucd94\uac00\ub41c \uacf3\uc740?, context: Pok\u00e9mon GO\uac00 \uc815\uc2dd\uc73c\ub85c \ucd9c\uc2dc\ub418\uc9c0 \uc54a\uc558\ub358 \ub300\ud55c\ubbfc\uad6d\uc5d0\uc11c\ub294, iOS \ud574\uc678 \uc6b0\ud68c \uacc4\uc815\uc774\ub098 \uc548\ub4dc\ub85c\uc774\ub4dc \uc124\uce58\ud30c\uc77c(APK) \ub2e4\uc6b4\ub85c\ub4dc \ub610\ub294 Google Play \ud574\uc678 \uc6b0\ud68c \uacc4\uc815\uc73c\ub85c \uc2e4\ud589\ud55c Pok\u00e9mon GO\uac00 \uc720\uc77c\ud558\uac8c \uac15\uc6d0\ub3c4 \uc18d\ucd08\uc640 \uc591\uc591, \uace0\uc131\uad70, \uc6b8\ub989\ub3c4, \ub3c5\ub3c4\uc5d0\uc11c \uc0ac\uc6a9\uc774 \uac00\ub2a5\ud558\uc600\ub2e4. \uc774 \uc0ac\uc2e4\uc774 \uc18c\uc15c \ubbf8\ub514\uc5b4\ub97c \ud1b5\ud574 \uc54c\ub824\uc9c0\uc790, Pok\u00e9mon GO\ub97c \ud558\ub824\uace0 \uc18d\ucd08\ub85c \ubab0\ub824\uac00\ub294 \uc0ac\ub78c\ub4e4\ub85c \uc778\ud574 \uc77c\ubd80 \uc9c0\uc5ed\uc5d0\uc11c \ud3c9\uc77c \uc18d\ucd08\ud589 \uace0\uc18d\ubc84\uc2a4 \ud45c\uac00 \ub9e4\uc9c4\ub418\ub294 \uc0ac\ud0dc\ub3c4 \ubc8c\uc5b4\uc84c\ub2e4. Pok\u00e9mon GO\ub97c \ud50c\ub808\uc774 \ud558\uae30 \uc704\ud574 \uc18d\ucd08\ub85c \uc9d1\uacb0\ud55c \uc0ac\ub78c\ub4e4\uc740 \uc7a5\ub9db\ube44 \uc18d\uc5d0\uc11c\ub3c4 \ud3ec\ucf13\ubaac \uc7a1\uc774\uc5d0 \ubab0\ub450\ud558\uc5ec Pok\u00e9mon GO \uc5f4\ud48d\uc744 \ubcf4\uc5ec\uc8fc\uc5c8\ub2e4. \uc774\ud6c4 \uc77c\ubcf8\uc774 \uc815\uc2dd \uc11c\ube44\uc2a4\ub97c \uc2dc\uc791\ud558\uc790 \ucd94\uac00\ub85c \uc6b8\uc0b0 \uac04\uc808\uacf6\uc5d0\uc11c \uc0ac\uc6a9 \uac00\ub2a5\ud558\uac8c \ub418\uc5c8\ub2e4."} {"question": "\uc778\ubbfc\uad70\uc774 \ud769\uc5b4\uc838 \ub0a8\ucabd\uc73c\ub85c \uacf5\uaca9\uc744 \uc9c0\uc18d\ud558\ub2e4\uac00 12\uc2dc \uc815\uac01\ubd80\ud130 \uc9d1\uc911 \ud3ec\ud654\ud560 \ub54c \uacf5\uaca9 \ubc1b\uc740 \uad70\ub300\ub294?", "paragraph": "\uc720\uc5d4\uad70 \ucca9\ubcf4 \ub2f9\uad6d\uc740 \ub9e4\uc6b0 \ubc00\uc9d1\ub41c \uc778\ubbfc\uad70 \ub300\ubcd1\ub825\uc774 \uad6d\uad70 \uc81c1\uc0ac\ub2e8\uc774 \uc788\ub294 \uacf3\uc744 \ud1b5\ud574 \ub099\ub3d9\uac15\uc744 \uac74\ub108\ub824 \uc2dc\ub3c4\ud560 \uac83\uc774\ub77c\uace0 \uba70\uce60\uc5d0 \uac78\uccd0 \uc54c\ub838\ub2e4. 8\uc6d4 14\uc77c \uc0c8\ubcbd, \uc778\ubbfc\uad70 1\uac1c \uc5f0\ub300\uac00 \uc218\uc911\uad50\ub97c \uc774\uc6a9\ud574 \uc65c\uad00 \ubd81\ucabd 9.7 \ud0ac\ub85c\ubbf8\ud130 \uc9c0\uc810\uc5d0 \ub3c4\ud558\ud588\ub2e4. \uc790\uc815 \uc9c1\ud6c4, \uc720\uc5d4\uad70\uacfc \uad6d\uad70 \uc811\uacbd \uad6c\uc5ed\uc758 \ubc14\ub85c \ubd81\ucabd\uc5d0 \uc704\uce58\ud55c \uace0\uc9c0\ub300\uc758 \uad6d\uad70 \ubcd1\ub825\uc774 \uc774 \uc778\ubbfc\uad70 \uc5f0\ub300\uc5d0\uac8c \uacf5\uaca9\ub2f9\ud588\ub2e4. \ud574\uac00 \ub728\uc790 \uacf5\uc2b5\uc744 \uac00\ud574 \uc218\uc911\uad50\ub97c \ud30c\uad34\ud588\ub2e4. \uc778\ubbfc\uad70\uc740 \uc0b0\uac1c\ud558\uc5ec \ub0a8\ucabd\uc73c\ub85c \uacf5\uaca9\uc744 \uacc4\uc18d\ud588\uace0 \ud55c\uad6d \ud45c\uc900\uc2dc 12:00 \uc815\uac01\uc774 \ub418\uc790 \uc778\ubbfc\uad70 \uc18c\ud654\uae30 \ucd1d\ud654\uac00 G\uc911\ub300 \ucabd\uc73c\ub85c \ud5a5\ud558\uae30 \uc2dc\uc791\ud588\ub2e4. 303\uace0\uc9c0 \uc704\uc758 \uc81c5\uae30\ubcd1\uc5f0\ub300\ub294 \ub2e4\ub978 \ubcd1\ub825\ub4e4\uacfc \ub2ec\ub9ac \ub3d9\ucabd\uc73c\ub85c \uc0b0\uc744 \ub0b4\ub824\uac00\uc9c0 \uc54a\uace0 \ub0a8\ucabd\uc73c\ub85c \ub0b4\ub824\uac00 \uc65c\uad00\uc744 \ud5a5\ud588\ub2e4.", "answer": "G\uc911\ub300", "sentence": "\uc778\ubbfc\uad70\uc740 \uc0b0\uac1c\ud558\uc5ec \ub0a8\ucabd\uc73c\ub85c \uacf5\uaca9\uc744 \uacc4\uc18d\ud588\uace0 \ud55c\uad6d \ud45c\uc900\uc2dc 12:00 \uc815\uac01\uc774 \ub418\uc790 \uc778\ubbfc\uad70 \uc18c\ud654\uae30 \ucd1d\ud654\uac00 G\uc911\ub300 \ucabd\uc73c\ub85c \ud5a5\ud558\uae30 \uc2dc\uc791\ud588\ub2e4.", "paragraph_sentence": "\uc720\uc5d4\uad70 \ucca9\ubcf4 \ub2f9\uad6d\uc740 \ub9e4\uc6b0 \ubc00\uc9d1\ub41c \uc778\ubbfc\uad70 \ub300\ubcd1\ub825\uc774 \uad6d\uad70 \uc81c1\uc0ac\ub2e8\uc774 \uc788\ub294 \uacf3\uc744 \ud1b5\ud574 \ub099\ub3d9\uac15\uc744 \uac74\ub108\ub824 \uc2dc\ub3c4\ud560 \uac83\uc774\ub77c\uace0 \uba70\uce60\uc5d0 \uac78\uccd0 \uc54c\ub838\ub2e4. 8\uc6d4 14\uc77c \uc0c8\ubcbd, \uc778\ubbfc\uad70 1\uac1c \uc5f0\ub300\uac00 \uc218\uc911\uad50\ub97c \uc774\uc6a9\ud574 \uc65c\uad00 \ubd81\ucabd 9.7 \ud0ac\ub85c\ubbf8\ud130 \uc9c0\uc810\uc5d0 \ub3c4\ud558\ud588\ub2e4. \uc790\uc815 \uc9c1\ud6c4, \uc720\uc5d4\uad70\uacfc \uad6d\uad70 \uc811\uacbd \uad6c\uc5ed\uc758 \ubc14\ub85c \ubd81\ucabd\uc5d0 \uc704\uce58\ud55c \uace0\uc9c0\ub300\uc758 \uad6d\uad70 \ubcd1\ub825\uc774 \uc774 \uc778\ubbfc\uad70 \uc5f0\ub300\uc5d0\uac8c \uacf5\uaca9\ub2f9\ud588\ub2e4. \ud574\uac00 \ub728\uc790 \uacf5\uc2b5\uc744 \uac00\ud574 \uc218\uc911\uad50\ub97c \ud30c\uad34\ud588\ub2e4. \uc778\ubbfc\uad70\uc740 \uc0b0\uac1c\ud558\uc5ec \ub0a8\ucabd\uc73c\ub85c \uacf5\uaca9\uc744 \uacc4\uc18d\ud588\uace0 \ud55c\uad6d \ud45c\uc900\uc2dc 12:00 \uc815\uac01\uc774 \ub418\uc790 \uc778\ubbfc\uad70 \uc18c\ud654\uae30 \ucd1d\ud654\uac00 G\uc911\ub300 \ucabd\uc73c\ub85c \ud5a5\ud558\uae30 \uc2dc\uc791\ud588\ub2e4. 303\uace0\uc9c0 \uc704\uc758 \uc81c5\uae30\ubcd1\uc5f0\ub300\ub294 \ub2e4\ub978 \ubcd1\ub825\ub4e4\uacfc \ub2ec\ub9ac \ub3d9\ucabd\uc73c\ub85c \uc0b0\uc744 \ub0b4\ub824\uac00\uc9c0 \uc54a\uace0 \ub0a8\ucabd\uc73c\ub85c \ub0b4\ub824\uac00 \uc65c\uad00\uc744 \ud5a5\ud588\ub2e4.", "paragraph_answer": "\uc720\uc5d4\uad70 \ucca9\ubcf4 \ub2f9\uad6d\uc740 \ub9e4\uc6b0 \ubc00\uc9d1\ub41c \uc778\ubbfc\uad70 \ub300\ubcd1\ub825\uc774 \uad6d\uad70 \uc81c1\uc0ac\ub2e8\uc774 \uc788\ub294 \uacf3\uc744 \ud1b5\ud574 \ub099\ub3d9\uac15\uc744 \uac74\ub108\ub824 \uc2dc\ub3c4\ud560 \uac83\uc774\ub77c\uace0 \uba70\uce60\uc5d0 \uac78\uccd0 \uc54c\ub838\ub2e4. 8\uc6d4 14\uc77c \uc0c8\ubcbd, \uc778\ubbfc\uad70 1\uac1c \uc5f0\ub300\uac00 \uc218\uc911\uad50\ub97c \uc774\uc6a9\ud574 \uc65c\uad00 \ubd81\ucabd 9.7 \ud0ac\ub85c\ubbf8\ud130 \uc9c0\uc810\uc5d0 \ub3c4\ud558\ud588\ub2e4. \uc790\uc815 \uc9c1\ud6c4, \uc720\uc5d4\uad70\uacfc \uad6d\uad70 \uc811\uacbd \uad6c\uc5ed\uc758 \ubc14\ub85c \ubd81\ucabd\uc5d0 \uc704\uce58\ud55c \uace0\uc9c0\ub300\uc758 \uad6d\uad70 \ubcd1\ub825\uc774 \uc774 \uc778\ubbfc\uad70 \uc5f0\ub300\uc5d0\uac8c \uacf5\uaca9\ub2f9\ud588\ub2e4. \ud574\uac00 \ub728\uc790 \uacf5\uc2b5\uc744 \uac00\ud574 \uc218\uc911\uad50\ub97c \ud30c\uad34\ud588\ub2e4. \uc778\ubbfc\uad70\uc740 \uc0b0\uac1c\ud558\uc5ec \ub0a8\ucabd\uc73c\ub85c \uacf5\uaca9\uc744 \uacc4\uc18d\ud588\uace0 \ud55c\uad6d \ud45c\uc900\uc2dc 12:00 \uc815\uac01\uc774 \ub418\uc790 \uc778\ubbfc\uad70 \uc18c\ud654\uae30 \ucd1d\ud654\uac00 G\uc911\ub300 \ucabd\uc73c\ub85c \ud5a5\ud558\uae30 \uc2dc\uc791\ud588\ub2e4. 303\uace0\uc9c0 \uc704\uc758 \uc81c5\uae30\ubcd1\uc5f0\ub300\ub294 \ub2e4\ub978 \ubcd1\ub825\ub4e4\uacfc \ub2ec\ub9ac \ub3d9\ucabd\uc73c\ub85c \uc0b0\uc744 \ub0b4\ub824\uac00\uc9c0 \uc54a\uace0 \ub0a8\ucabd\uc73c\ub85c \ub0b4\ub824\uac00 \uc65c\uad00\uc744 \ud5a5\ud588\ub2e4.", "sentence_answer": "\uc778\ubbfc\uad70\uc740 \uc0b0\uac1c\ud558\uc5ec \ub0a8\ucabd\uc73c\ub85c \uacf5\uaca9\uc744 \uacc4\uc18d\ud588\uace0 \ud55c\uad6d \ud45c\uc900\uc2dc 12:00 \uc815\uac01\uc774 \ub418\uc790 \uc778\ubbfc\uad70 \uc18c\ud654\uae30 \ucd1d\ud654\uac00 G\uc911\ub300 \ucabd\uc73c\ub85c \ud5a5\ud558\uae30 \uc2dc\uc791\ud588\ub2e4.", "paragraph_id": "6470861-3-0", "paragraph_question": "question: \uc778\ubbfc\uad70\uc774 \ud769\uc5b4\uc838 \ub0a8\ucabd\uc73c\ub85c \uacf5\uaca9\uc744 \uc9c0\uc18d\ud558\ub2e4\uac00 12\uc2dc \uc815\uac01\ubd80\ud130 \uc9d1\uc911 \ud3ec\ud654\ud560 \ub54c \uacf5\uaca9 \ubc1b\uc740 \uad70\ub300\ub294?, context: \uc720\uc5d4\uad70 \ucca9\ubcf4 \ub2f9\uad6d\uc740 \ub9e4\uc6b0 \ubc00\uc9d1\ub41c \uc778\ubbfc\uad70 \ub300\ubcd1\ub825\uc774 \uad6d\uad70 \uc81c1\uc0ac\ub2e8\uc774 \uc788\ub294 \uacf3\uc744 \ud1b5\ud574 \ub099\ub3d9\uac15\uc744 \uac74\ub108\ub824 \uc2dc\ub3c4\ud560 \uac83\uc774\ub77c\uace0 \uba70\uce60\uc5d0 \uac78\uccd0 \uc54c\ub838\ub2e4. 8\uc6d4 14\uc77c \uc0c8\ubcbd, \uc778\ubbfc\uad70 1\uac1c \uc5f0\ub300\uac00 \uc218\uc911\uad50\ub97c \uc774\uc6a9\ud574 \uc65c\uad00 \ubd81\ucabd 9.7 \ud0ac\ub85c\ubbf8\ud130 \uc9c0\uc810\uc5d0 \ub3c4\ud558\ud588\ub2e4. \uc790\uc815 \uc9c1\ud6c4, \uc720\uc5d4\uad70\uacfc \uad6d\uad70 \uc811\uacbd \uad6c\uc5ed\uc758 \ubc14\ub85c \ubd81\ucabd\uc5d0 \uc704\uce58\ud55c \uace0\uc9c0\ub300\uc758 \uad6d\uad70 \ubcd1\ub825\uc774 \uc774 \uc778\ubbfc\uad70 \uc5f0\ub300\uc5d0\uac8c \uacf5\uaca9\ub2f9\ud588\ub2e4. \ud574\uac00 \ub728\uc790 \uacf5\uc2b5\uc744 \uac00\ud574 \uc218\uc911\uad50\ub97c \ud30c\uad34\ud588\ub2e4. \uc778\ubbfc\uad70\uc740 \uc0b0\uac1c\ud558\uc5ec \ub0a8\ucabd\uc73c\ub85c \uacf5\uaca9\uc744 \uacc4\uc18d\ud588\uace0 \ud55c\uad6d \ud45c\uc900\uc2dc 12:00 \uc815\uac01\uc774 \ub418\uc790 \uc778\ubbfc\uad70 \uc18c\ud654\uae30 \ucd1d\ud654\uac00 G\uc911\ub300 \ucabd\uc73c\ub85c \ud5a5\ud558\uae30 \uc2dc\uc791\ud588\ub2e4. 303\uace0\uc9c0 \uc704\uc758 \uc81c5\uae30\ubcd1\uc5f0\ub300\ub294 \ub2e4\ub978 \ubcd1\ub825\ub4e4\uacfc \ub2ec\ub9ac \ub3d9\ucabd\uc73c\ub85c \uc0b0\uc744 \ub0b4\ub824\uac00\uc9c0 \uc54a\uace0 \ub0a8\ucabd\uc73c\ub85c \ub0b4\ub824\uac00 \uc65c\uad00\uc744 \ud5a5\ud588\ub2e4."} {"question": "\uae38\ubc84\ud2b8\uac00 \uc12c \ub0a8\ub3d9\ucabd\uc5d0 \uc704\uce58\ud588\uc744\ub54c 252km/h\uc758 \ub3cc\ud48d\uc774 \uad00\uce21\ub41c \ub54c\ub294?", "paragraph": "\ud5c8\ub9ac\ucf00\uc778 \uae38\ubc84\ud2b8\ub294 \uc790\uba54\uc774\uce74\ub97c \ube60\uc838\ub098\uc628 \ub4a4\uc5d0 \ub2e4\uc2dc \uae09\uaca9\ud558\uac8c \uac15\ud574\uc84c\uc73c\uba70 \ucf00\uc774\ub9e8 \uc81c\ub3c4\uc758 \uac00\uc7a5 \ud070 \uc12c\uc778 \uadf8\ub79c\ub4dc \ucf00\uc774\ub9e8(Grand Cayman)\uc12c\uc758 \uae30\uc0c1\uad00\uce21\uc18c\uc5d0\uc11c\ub294 9\uc6d4 13\uc77c, \uae38\ubc84\ud2b8\uac00 \uc12c \ub0a8\ub3d9\ucabd\uc5d0 \uc704\uce58\ud574\uc788\uc744 \ub54c 252km/h(156mph)\uc5d0 \ub2ec\ud558\ub294 \ub3cc\ud48d\uc774 \uad00\uce21\ub418\uae30\ub3c4 \ud558\uc600\ub2e4. \uae38\ubc84\ud2b8\ub294 \uc911\uc2ec\uae30\uc555\uc774 888hPa\uc5d0 \ub3c4\ub2ec\ud558\uae30\uae4c\uc9c0 \uc26c\uc9c0\uc54a\uace0 \uacc4\uc18d \uac15\ud574\uc84c\uace0 \ub9c8\uce68\ub0b4 \ucd5c\uc131\uae30\ub97c \ub9de\uc774\ud558\uc600\uc744 \ub54c\ub294 \uac15\ud48d\ubc18\uacbd \ub0b4\uc5d0\uc11c 295km/h(185mph)\uc758 \ubc14\ub78c\uc774 \ubd88\uc5c8\ub2e4. \uc774 \uc218\uce58\ub294 24\uc2dc\uac04\ub9cc\uc5d0 72hPa\uac00 \ub5a8\uc5b4\uc9c4 \uac83\uc73c\ub85c \uae30\ub85d\ub418\uc5c8\uace0 \uc774\ub294 \ub2f9\uc2dc \uc804\uc5d0 \uc77c\uc5b4\ub0ac\ub358 \ubaa8\ub4e0 \ubd81\ub300\uc11c\uc591 \ud5c8\ub9ac\ucf00\uc778\uc744 \ud1b5\ud2c0\uc5b4\uc11c \uac00\uc7a5 \uac15\ud55c \ud5c8\ub9ac\ucf00\uc778\uc774\uc5c8\ub2e4. \uadf8\ub7ec\ub098 \uc774 \uae30\ub85d\uc740 2005\ub144\uc5d0 \ud5c8\ub9ac\ucf00\uc778 \uc70c\ub9c8\uac00 6hPa \ub0ae\uc740 882hPa\ub97c \uae30\ub85d\ud558\uba74\uc11c \uae68\uc9c0\uac8c \ub418\uc5c8\ub2e4.", "answer": "9\uc6d4 13\uc77c", "sentence": "\ud5c8\ub9ac\ucf00\uc778 \uae38\ubc84\ud2b8\ub294 \uc790\uba54\uc774\uce74\ub97c \ube60\uc838\ub098\uc628 \ub4a4\uc5d0 \ub2e4\uc2dc \uae09\uaca9\ud558\uac8c \uac15\ud574\uc84c\uc73c\uba70 \ucf00\uc774\ub9e8 \uc81c\ub3c4\uc758 \uac00\uc7a5 \ud070 \uc12c\uc778 \uadf8\ub79c\ub4dc \ucf00\uc774\ub9e8(Grand Cayman)\uc12c\uc758 \uae30\uc0c1\uad00\uce21\uc18c\uc5d0\uc11c\ub294 9\uc6d4 13\uc77c ,", "paragraph_sentence": " \ud5c8\ub9ac\ucf00\uc778 \uae38\ubc84\ud2b8\ub294 \uc790\uba54\uc774\uce74\ub97c \ube60\uc838\ub098\uc628 \ub4a4\uc5d0 \ub2e4\uc2dc \uae09\uaca9\ud558\uac8c \uac15\ud574\uc84c\uc73c\uba70 \ucf00\uc774\ub9e8 \uc81c\ub3c4\uc758 \uac00\uc7a5 \ud070 \uc12c\uc778 \uadf8\ub79c\ub4dc \ucf00\uc774\ub9e8(Grand Cayman)\uc12c\uc758 \uae30\uc0c1\uad00\uce21\uc18c\uc5d0\uc11c\ub294 9\uc6d4 13\uc77c , \uae38\ubc84\ud2b8\uac00 \uc12c \ub0a8\ub3d9\ucabd\uc5d0 \uc704\uce58\ud574\uc788\uc744 \ub54c 252km/h(156mph)\uc5d0 \ub2ec\ud558\ub294 \ub3cc\ud48d\uc774 \uad00\uce21\ub418\uae30\ub3c4 \ud558\uc600\ub2e4. \uae38\ubc84\ud2b8\ub294 \uc911\uc2ec\uae30\uc555\uc774 888hPa\uc5d0 \ub3c4\ub2ec\ud558\uae30\uae4c\uc9c0 \uc26c\uc9c0\uc54a\uace0 \uacc4\uc18d \uac15\ud574\uc84c\uace0 \ub9c8\uce68\ub0b4 \ucd5c\uc131\uae30\ub97c \ub9de\uc774\ud558\uc600\uc744 \ub54c\ub294 \uac15\ud48d\ubc18\uacbd \ub0b4\uc5d0\uc11c 295km/h(185mph)\uc758 \ubc14\ub78c\uc774 \ubd88\uc5c8\ub2e4. \uc774 \uc218\uce58\ub294 24\uc2dc\uac04\ub9cc\uc5d0 72hPa\uac00 \ub5a8\uc5b4\uc9c4 \uac83\uc73c\ub85c \uae30\ub85d\ub418\uc5c8\uace0 \uc774\ub294 \ub2f9\uc2dc \uc804\uc5d0 \uc77c\uc5b4\ub0ac\ub358 \ubaa8\ub4e0 \ubd81\ub300\uc11c\uc591 \ud5c8\ub9ac\ucf00\uc778\uc744 \ud1b5\ud2c0\uc5b4\uc11c \uac00\uc7a5 \uac15\ud55c \ud5c8\ub9ac\ucf00\uc778\uc774\uc5c8\ub2e4. \uadf8\ub7ec\ub098 \uc774 \uae30\ub85d\uc740 2005\ub144\uc5d0 \ud5c8\ub9ac\ucf00\uc778 \uc70c\ub9c8\uac00 6hPa \ub0ae\uc740 882hPa\ub97c \uae30\ub85d\ud558\uba74\uc11c \uae68\uc9c0\uac8c \ub418\uc5c8\ub2e4.", "paragraph_answer": "\ud5c8\ub9ac\ucf00\uc778 \uae38\ubc84\ud2b8\ub294 \uc790\uba54\uc774\uce74\ub97c \ube60\uc838\ub098\uc628 \ub4a4\uc5d0 \ub2e4\uc2dc \uae09\uaca9\ud558\uac8c \uac15\ud574\uc84c\uc73c\uba70 \ucf00\uc774\ub9e8 \uc81c\ub3c4\uc758 \uac00\uc7a5 \ud070 \uc12c\uc778 \uadf8\ub79c\ub4dc \ucf00\uc774\ub9e8(Grand Cayman)\uc12c\uc758 \uae30\uc0c1\uad00\uce21\uc18c\uc5d0\uc11c\ub294 9\uc6d4 13\uc77c , \uae38\ubc84\ud2b8\uac00 \uc12c \ub0a8\ub3d9\ucabd\uc5d0 \uc704\uce58\ud574\uc788\uc744 \ub54c 252km/h(156mph)\uc5d0 \ub2ec\ud558\ub294 \ub3cc\ud48d\uc774 \uad00\uce21\ub418\uae30\ub3c4 \ud558\uc600\ub2e4. \uae38\ubc84\ud2b8\ub294 \uc911\uc2ec\uae30\uc555\uc774 888hPa\uc5d0 \ub3c4\ub2ec\ud558\uae30\uae4c\uc9c0 \uc26c\uc9c0\uc54a\uace0 \uacc4\uc18d \uac15\ud574\uc84c\uace0 \ub9c8\uce68\ub0b4 \ucd5c\uc131\uae30\ub97c \ub9de\uc774\ud558\uc600\uc744 \ub54c\ub294 \uac15\ud48d\ubc18\uacbd \ub0b4\uc5d0\uc11c 295km/h(185mph)\uc758 \ubc14\ub78c\uc774 \ubd88\uc5c8\ub2e4. \uc774 \uc218\uce58\ub294 24\uc2dc\uac04\ub9cc\uc5d0 72hPa\uac00 \ub5a8\uc5b4\uc9c4 \uac83\uc73c\ub85c \uae30\ub85d\ub418\uc5c8\uace0 \uc774\ub294 \ub2f9\uc2dc \uc804\uc5d0 \uc77c\uc5b4\ub0ac\ub358 \ubaa8\ub4e0 \ubd81\ub300\uc11c\uc591 \ud5c8\ub9ac\ucf00\uc778\uc744 \ud1b5\ud2c0\uc5b4\uc11c \uac00\uc7a5 \uac15\ud55c \ud5c8\ub9ac\ucf00\uc778\uc774\uc5c8\ub2e4. \uadf8\ub7ec\ub098 \uc774 \uae30\ub85d\uc740 2005\ub144\uc5d0 \ud5c8\ub9ac\ucf00\uc778 \uc70c\ub9c8\uac00 6hPa \ub0ae\uc740 882hPa\ub97c \uae30\ub85d\ud558\uba74\uc11c \uae68\uc9c0\uac8c \ub418\uc5c8\ub2e4.", "sentence_answer": "\ud5c8\ub9ac\ucf00\uc778 \uae38\ubc84\ud2b8\ub294 \uc790\uba54\uc774\uce74\ub97c \ube60\uc838\ub098\uc628 \ub4a4\uc5d0 \ub2e4\uc2dc \uae09\uaca9\ud558\uac8c \uac15\ud574\uc84c\uc73c\uba70 \ucf00\uc774\ub9e8 \uc81c\ub3c4\uc758 \uac00\uc7a5 \ud070 \uc12c\uc778 \uadf8\ub79c\ub4dc \ucf00\uc774\ub9e8(Grand Cayman)\uc12c\uc758 \uae30\uc0c1\uad00\uce21\uc18c\uc5d0\uc11c\ub294 9\uc6d4 13\uc77c ,", "paragraph_id": "6571114-0-1", "paragraph_question": "question: \uae38\ubc84\ud2b8\uac00 \uc12c \ub0a8\ub3d9\ucabd\uc5d0 \uc704\uce58\ud588\uc744\ub54c 252km/h\uc758 \ub3cc\ud48d\uc774 \uad00\uce21\ub41c \ub54c\ub294?, context: \ud5c8\ub9ac\ucf00\uc778 \uae38\ubc84\ud2b8\ub294 \uc790\uba54\uc774\uce74\ub97c \ube60\uc838\ub098\uc628 \ub4a4\uc5d0 \ub2e4\uc2dc \uae09\uaca9\ud558\uac8c \uac15\ud574\uc84c\uc73c\uba70 \ucf00\uc774\ub9e8 \uc81c\ub3c4\uc758 \uac00\uc7a5 \ud070 \uc12c\uc778 \uadf8\ub79c\ub4dc \ucf00\uc774\ub9e8(Grand Cayman)\uc12c\uc758 \uae30\uc0c1\uad00\uce21\uc18c\uc5d0\uc11c\ub294 9\uc6d4 13\uc77c, \uae38\ubc84\ud2b8\uac00 \uc12c \ub0a8\ub3d9\ucabd\uc5d0 \uc704\uce58\ud574\uc788\uc744 \ub54c 252km/h(156mph)\uc5d0 \ub2ec\ud558\ub294 \ub3cc\ud48d\uc774 \uad00\uce21\ub418\uae30\ub3c4 \ud558\uc600\ub2e4. \uae38\ubc84\ud2b8\ub294 \uc911\uc2ec\uae30\uc555\uc774 888hPa\uc5d0 \ub3c4\ub2ec\ud558\uae30\uae4c\uc9c0 \uc26c\uc9c0\uc54a\uace0 \uacc4\uc18d \uac15\ud574\uc84c\uace0 \ub9c8\uce68\ub0b4 \ucd5c\uc131\uae30\ub97c \ub9de\uc774\ud558\uc600\uc744 \ub54c\ub294 \uac15\ud48d\ubc18\uacbd \ub0b4\uc5d0\uc11c 295km/h(185mph)\uc758 \ubc14\ub78c\uc774 \ubd88\uc5c8\ub2e4. \uc774 \uc218\uce58\ub294 24\uc2dc\uac04\ub9cc\uc5d0 72hPa\uac00 \ub5a8\uc5b4\uc9c4 \uac83\uc73c\ub85c \uae30\ub85d\ub418\uc5c8\uace0 \uc774\ub294 \ub2f9\uc2dc \uc804\uc5d0 \uc77c\uc5b4\ub0ac\ub358 \ubaa8\ub4e0 \ubd81\ub300\uc11c\uc591 \ud5c8\ub9ac\ucf00\uc778\uc744 \ud1b5\ud2c0\uc5b4\uc11c \uac00\uc7a5 \uac15\ud55c \ud5c8\ub9ac\ucf00\uc778\uc774\uc5c8\ub2e4. \uadf8\ub7ec\ub098 \uc774 \uae30\ub85d\uc740 2005\ub144\uc5d0 \ud5c8\ub9ac\ucf00\uc778 \uc70c\ub9c8\uac00 6hPa \ub0ae\uc740 882hPa\ub97c \uae30\ub85d\ud558\uba74\uc11c \uae68\uc9c0\uac8c \ub418\uc5c8\ub2e4."} {"question": "\uc591\uacfc \uad50\uc12d\uc774 \ud65c\ubc1c\ud574\ub358 \ub098\ub77c\ub4e4\uc5d0\uc11c \ubb34\uc81c\uc5d0\uac8c \ubcf4\ub0b4\ub294 \uad6d\uc11c\uc5d0 \ubb34\uc81c\ub97c \ubb34\uc5c7\uc73c\ub85c \uce6d\ud588\ub098?", "paragraph": "\uc774\ub7ec\ud55c \ubb34\uc81c\uc758 \ubd88\uad50 \uc2e0\uc559\uc740 \ud45c\uba74\uc801\uc778 \uac83\uc5d0 \uadf8\uce58\uc9c0 \uc54a\uace0 \ubb34\uc81c \uc790\uc2e0\uc774 \ubd88\uad50 \uacbd\uc804\uc5d0 \ub300\ud55c \uc218\ub9ce\uc740 \uc8fc\uc11d\uc11c\ub97c \uc9d1\ud544\ud558\uc600\ub2e4. \ub610\ud55c \ud669\uc81c \uc790\uc2e0\ub3c4 \ubd88\uad50\uc758 \uacc4\uc728\uc744 \ub530\ub974\uba70 \uc18c\uc2dd(\u852c\u98df)\ud558\uae30\ub97c \uacac\uc9c0\ud588\uae30\uc5d0, \ub2f9\uc2dc\uc5d0\ub294 \ud669\uc81c\ub97c \uac00\ub9ac\ucf1c \ud669\uc81c\ubcf4\uc0b4(\u7687\u5e1d\u83e9\u85a9)\ub85c \ubd80\ub97c \uc815\ub3c4\uc600\ub2e4. \uc774\ub294 \ub2f9\uc2dc \uad6d\uac00\ubd88\uad50\ub85c\uc11c\uc758 \uc0c9\ucc44\uac00 \ub18d\ud6c4\ud588\ub358 \ubd81\uc870\uc5d0\uc11c \uc0ac\uc6a9\ud558\ub358 \ud669\uc81c\uc989\uc5ec\ub798(\u7687\u5e1d\u5373\u5982\u6765)\uc5d0 \ub300\ube44\ub418\uc5b4 \ub0a8\uc870\uc758 \ubd88\uad50\ub97c \uc0c1\uc9d5\ud558\ub294 \uce6d\ud638\ub85c\uc11c \ud3c9\uac00\ub418\uace0 \uc788\ub2e4. \uc774\ub7ec\ud55c \ud669\uc81c\uc5d0 \ub300\ud574 \ub2f9\uc2dc \uc591\uacfc \uad50\uc12d\uc774 \ud65c\ubc1c\ud588\ub358 \ub3d9\ub0a8\uc544\uc2dc\uc544\ub098 \uc11c\uc5ed, \ud55c\ubc18\ub3c4\uc758 \ubc31\uc81c\ub098 \uc65c\uad6d \ub4f1 \uc5ec\ub7ec \uad6d\uac00\uc5d0\uc11c\ub294 \ubb34\uc81c\uc5d0\uac8c \ubcf4\ub0b4\ub294 \uad6d\uc11c\uc5d0\uc11c \ubb34\uc81c\ub97c \ubcf4\uc0b4\uc774\ub77c \ubd88\ub800\ub294\ub370, \ub2f9\uc2dc\uc758 \uad6d\uc81c\uc0ac\ud68c\uc5d0\uc11c\ub3c4 \uc591\uc758 \ud669\uc81c\ub294 \ubd88\uad50 \uc2e0\uc559\uc5d0 \uc788\uc5b4 \uace0\uba85\ud55c \uc778\ubb3c\uc758 \ud55c \uc0ac\ub78c\uc73c\ub85c \uaf3d\ud788\uace0 \uc788\uc5c8\uae30 \ub54c\ubb38\uc774\ub2e4.", "answer": "\ubcf4\uc0b4", "sentence": "\ub610\ud55c \ud669\uc81c \uc790\uc2e0\ub3c4 \ubd88\uad50\uc758 \uacc4\uc728\uc744 \ub530\ub974\uba70 \uc18c\uc2dd(\u852c\u98df)\ud558\uae30\ub97c \uacac\uc9c0\ud588\uae30\uc5d0, \ub2f9\uc2dc\uc5d0\ub294 \ud669\uc81c\ub97c \uac00\ub9ac\ucf1c \ud669\uc81c \ubcf4\uc0b4 (\u7687\u5e1d\u83e9\u85a9)\ub85c \ubd80\ub97c \uc815\ub3c4\uc600\ub2e4.", "paragraph_sentence": "\uc774\ub7ec\ud55c \ubb34\uc81c\uc758 \ubd88\uad50 \uc2e0\uc559\uc740 \ud45c\uba74\uc801\uc778 \uac83\uc5d0 \uadf8\uce58\uc9c0 \uc54a\uace0 \ubb34\uc81c \uc790\uc2e0\uc774 \ubd88\uad50 \uacbd\uc804\uc5d0 \ub300\ud55c \uc218\ub9ce\uc740 \uc8fc\uc11d\uc11c\ub97c \uc9d1\ud544\ud558\uc600\ub2e4. \ub610\ud55c \ud669\uc81c \uc790\uc2e0\ub3c4 \ubd88\uad50\uc758 \uacc4\uc728\uc744 \ub530\ub974\uba70 \uc18c\uc2dd(\u852c\u98df)\ud558\uae30\ub97c \uacac\uc9c0\ud588\uae30\uc5d0, \ub2f9\uc2dc\uc5d0\ub294 \ud669\uc81c\ub97c \uac00\ub9ac\ucf1c \ud669\uc81c \ubcf4\uc0b4 (\u7687\u5e1d\u83e9\u85a9)\ub85c \ubd80\ub97c \uc815\ub3c4\uc600\ub2e4. \uc774\ub294 \ub2f9\uc2dc \uad6d\uac00\ubd88\uad50\ub85c\uc11c\uc758 \uc0c9\ucc44\uac00 \ub18d\ud6c4\ud588\ub358 \ubd81\uc870\uc5d0\uc11c \uc0ac\uc6a9\ud558\ub358 \ud669\uc81c\uc989\uc5ec\ub798(\u7687\u5e1d\u5373\u5982\u6765)\uc5d0 \ub300\ube44\ub418\uc5b4 \ub0a8\uc870\uc758 \ubd88\uad50\ub97c \uc0c1\uc9d5\ud558\ub294 \uce6d\ud638\ub85c\uc11c \ud3c9\uac00\ub418\uace0 \uc788\ub2e4. \uc774\ub7ec\ud55c \ud669\uc81c\uc5d0 \ub300\ud574 \ub2f9\uc2dc \uc591\uacfc \uad50\uc12d\uc774 \ud65c\ubc1c\ud588\ub358 \ub3d9\ub0a8\uc544\uc2dc\uc544\ub098 \uc11c\uc5ed, \ud55c\ubc18\ub3c4\uc758 \ubc31\uc81c\ub098 \uc65c\uad6d \ub4f1 \uc5ec\ub7ec \uad6d\uac00\uc5d0\uc11c\ub294 \ubb34\uc81c\uc5d0\uac8c \ubcf4\ub0b4\ub294 \uad6d\uc11c\uc5d0\uc11c \ubb34\uc81c\ub97c \ubcf4\uc0b4\uc774\ub77c \ubd88\ub800\ub294\ub370, \ub2f9\uc2dc\uc758 \uad6d\uc81c\uc0ac\ud68c\uc5d0\uc11c\ub3c4 \uc591\uc758 \ud669\uc81c\ub294 \ubd88\uad50 \uc2e0\uc559\uc5d0 \uc788\uc5b4 \uace0\uba85\ud55c \uc778\ubb3c\uc758 \ud55c \uc0ac\ub78c\uc73c\ub85c \uaf3d\ud788\uace0 \uc788\uc5c8\uae30 \ub54c\ubb38\uc774\ub2e4.", "paragraph_answer": "\uc774\ub7ec\ud55c \ubb34\uc81c\uc758 \ubd88\uad50 \uc2e0\uc559\uc740 \ud45c\uba74\uc801\uc778 \uac83\uc5d0 \uadf8\uce58\uc9c0 \uc54a\uace0 \ubb34\uc81c \uc790\uc2e0\uc774 \ubd88\uad50 \uacbd\uc804\uc5d0 \ub300\ud55c \uc218\ub9ce\uc740 \uc8fc\uc11d\uc11c\ub97c \uc9d1\ud544\ud558\uc600\ub2e4. \ub610\ud55c \ud669\uc81c \uc790\uc2e0\ub3c4 \ubd88\uad50\uc758 \uacc4\uc728\uc744 \ub530\ub974\uba70 \uc18c\uc2dd(\u852c\u98df)\ud558\uae30\ub97c \uacac\uc9c0\ud588\uae30\uc5d0, \ub2f9\uc2dc\uc5d0\ub294 \ud669\uc81c\ub97c \uac00\ub9ac\ucf1c \ud669\uc81c \ubcf4\uc0b4 (\u7687\u5e1d\u83e9\u85a9)\ub85c \ubd80\ub97c \uc815\ub3c4\uc600\ub2e4. \uc774\ub294 \ub2f9\uc2dc \uad6d\uac00\ubd88\uad50\ub85c\uc11c\uc758 \uc0c9\ucc44\uac00 \ub18d\ud6c4\ud588\ub358 \ubd81\uc870\uc5d0\uc11c \uc0ac\uc6a9\ud558\ub358 \ud669\uc81c\uc989\uc5ec\ub798(\u7687\u5e1d\u5373\u5982\u6765)\uc5d0 \ub300\ube44\ub418\uc5b4 \ub0a8\uc870\uc758 \ubd88\uad50\ub97c \uc0c1\uc9d5\ud558\ub294 \uce6d\ud638\ub85c\uc11c \ud3c9\uac00\ub418\uace0 \uc788\ub2e4. \uc774\ub7ec\ud55c \ud669\uc81c\uc5d0 \ub300\ud574 \ub2f9\uc2dc \uc591\uacfc \uad50\uc12d\uc774 \ud65c\ubc1c\ud588\ub358 \ub3d9\ub0a8\uc544\uc2dc\uc544\ub098 \uc11c\uc5ed, \ud55c\ubc18\ub3c4\uc758 \ubc31\uc81c\ub098 \uc65c\uad6d \ub4f1 \uc5ec\ub7ec \uad6d\uac00\uc5d0\uc11c\ub294 \ubb34\uc81c\uc5d0\uac8c \ubcf4\ub0b4\ub294 \uad6d\uc11c\uc5d0\uc11c \ubb34\uc81c\ub97c \ubcf4\uc0b4\uc774\ub77c \ubd88\ub800\ub294\ub370, \ub2f9\uc2dc\uc758 \uad6d\uc81c\uc0ac\ud68c\uc5d0\uc11c\ub3c4 \uc591\uc758 \ud669\uc81c\ub294 \ubd88\uad50 \uc2e0\uc559\uc5d0 \uc788\uc5b4 \uace0\uba85\ud55c \uc778\ubb3c\uc758 \ud55c \uc0ac\ub78c\uc73c\ub85c \uaf3d\ud788\uace0 \uc788\uc5c8\uae30 \ub54c\ubb38\uc774\ub2e4.", "sentence_answer": "\ub610\ud55c \ud669\uc81c \uc790\uc2e0\ub3c4 \ubd88\uad50\uc758 \uacc4\uc728\uc744 \ub530\ub974\uba70 \uc18c\uc2dd(\u852c\u98df)\ud558\uae30\ub97c \uacac\uc9c0\ud588\uae30\uc5d0, \ub2f9\uc2dc\uc5d0\ub294 \ud669\uc81c\ub97c \uac00\ub9ac\ucf1c \ud669\uc81c \ubcf4\uc0b4 (\u7687\u5e1d\u83e9\u85a9)\ub85c \ubd80\ub97c \uc815\ub3c4\uc600\ub2e4.", "paragraph_id": "6457882-1-1", "paragraph_question": "question: \uc591\uacfc \uad50\uc12d\uc774 \ud65c\ubc1c\ud574\ub358 \ub098\ub77c\ub4e4\uc5d0\uc11c \ubb34\uc81c\uc5d0\uac8c \ubcf4\ub0b4\ub294 \uad6d\uc11c\uc5d0 \ubb34\uc81c\ub97c \ubb34\uc5c7\uc73c\ub85c \uce6d\ud588\ub098?, context: \uc774\ub7ec\ud55c \ubb34\uc81c\uc758 \ubd88\uad50 \uc2e0\uc559\uc740 \ud45c\uba74\uc801\uc778 \uac83\uc5d0 \uadf8\uce58\uc9c0 \uc54a\uace0 \ubb34\uc81c \uc790\uc2e0\uc774 \ubd88\uad50 \uacbd\uc804\uc5d0 \ub300\ud55c \uc218\ub9ce\uc740 \uc8fc\uc11d\uc11c\ub97c \uc9d1\ud544\ud558\uc600\ub2e4. \ub610\ud55c \ud669\uc81c \uc790\uc2e0\ub3c4 \ubd88\uad50\uc758 \uacc4\uc728\uc744 \ub530\ub974\uba70 \uc18c\uc2dd(\u852c\u98df)\ud558\uae30\ub97c \uacac\uc9c0\ud588\uae30\uc5d0, \ub2f9\uc2dc\uc5d0\ub294 \ud669\uc81c\ub97c \uac00\ub9ac\ucf1c \ud669\uc81c\ubcf4\uc0b4(\u7687\u5e1d\u83e9\u85a9)\ub85c \ubd80\ub97c \uc815\ub3c4\uc600\ub2e4. \uc774\ub294 \ub2f9\uc2dc \uad6d\uac00\ubd88\uad50\ub85c\uc11c\uc758 \uc0c9\ucc44\uac00 \ub18d\ud6c4\ud588\ub358 \ubd81\uc870\uc5d0\uc11c \uc0ac\uc6a9\ud558\ub358 \ud669\uc81c\uc989\uc5ec\ub798(\u7687\u5e1d\u5373\u5982\u6765)\uc5d0 \ub300\ube44\ub418\uc5b4 \ub0a8\uc870\uc758 \ubd88\uad50\ub97c \uc0c1\uc9d5\ud558\ub294 \uce6d\ud638\ub85c\uc11c \ud3c9\uac00\ub418\uace0 \uc788\ub2e4. \uc774\ub7ec\ud55c \ud669\uc81c\uc5d0 \ub300\ud574 \ub2f9\uc2dc \uc591\uacfc \uad50\uc12d\uc774 \ud65c\ubc1c\ud588\ub358 \ub3d9\ub0a8\uc544\uc2dc\uc544\ub098 \uc11c\uc5ed, \ud55c\ubc18\ub3c4\uc758 \ubc31\uc81c\ub098 \uc65c\uad6d \ub4f1 \uc5ec\ub7ec \uad6d\uac00\uc5d0\uc11c\ub294 \ubb34\uc81c\uc5d0\uac8c \ubcf4\ub0b4\ub294 \uad6d\uc11c\uc5d0\uc11c \ubb34\uc81c\ub97c \ubcf4\uc0b4\uc774\ub77c \ubd88\ub800\ub294\ub370, \ub2f9\uc2dc\uc758 \uad6d\uc81c\uc0ac\ud68c\uc5d0\uc11c\ub3c4 \uc591\uc758 \ud669\uc81c\ub294 \ubd88\uad50 \uc2e0\uc559\uc5d0 \uc788\uc5b4 \uace0\uba85\ud55c \uc778\ubb3c\uc758 \ud55c \uc0ac\ub78c\uc73c\ub85c \uaf3d\ud788\uace0 \uc788\uc5c8\uae30 \ub54c\ubb38\uc774\ub2e4."} {"question": "\ud558\uc778\uce20 \uad6c\ub370\ub9ac\uc548\uc740 \uc5b8\uc81c \ubbf8\uad70\uc5d0\uac8c \ud56d\ubcf5\ud574 \ud3ec\ub85c\uac00 \ub418\uc5c8\ub294\uac00?", "paragraph": "\uad6c\ub370\ub9ac\uc548\uc740 1945\ub144 5\uc6d4 10\uc77c \ubbf8\uad70\uc5d0 \ud56d\ubcf5\ud574 \ud3ec\ub85c\uac00 \ub418\uc5c8\ub2e4. \ub258\ub978\ubca0\ub974\ud06c \uc7ac\ud310\uc774 \uc2dc\uc791\ub418\uc790 \uad6c \uc18c\ub828\uacfc \ud3f4\ub780\ub4dc\ub294 \uc804\uc7c1 \ubc94\uc8c4\uc790\ub85c\uc11c \uad6c\ub370\ub9ac\uc548\uc744 \uae30\uc18c\ud588\uc73c\ub098 \uacb0\uad6d \uae30\uc18c\ub294 \uc911\uc9c0\ub418\uc5c8\uace0 \uad6c\ub370\ub9ac\uc548\uc740 \uc544\ubb34\ub7f0 \ud610\uc758\uac00 \uc5c6\ub294 \uac83\uc73c\ub85c \uacb0\ub860\uc9c0\uc5b4\uc84c\ub2e4. \uadf8\uc758 \uc804\uc7c1\uae30\uac04\ub3d9\uc548\uc758 \ud589\uc704\ub4e4\uc740 \ubaa8\ub450 \uc815\ub2f9\ud55c \uad70\uc778\uc758 \uc790\uc138\uc5d0 \uc785\uac01\ud558\uc600\ub358 \uac83\uc73c\ub85c \ud310\uc815\ub41c \uac83\uc774\ub2e4. \uadf8\ub9ac\uace0 \uc7ac\ud310\uc5d0\uc11c \uad70\uc0ac \uc791\uc804\uc5d0 \ub300\ud55c(\ud2b9\ud788 \ud3f4\ub780\ub4dc \uce68\uacf5\uc5d0 \ub300\ud574\uc11c)\uc99d\uc5b8\uc744 \ub0a8\uacbc\ub2e4. 1948\ub144 6\uc6d4 17\uc77c \uc11d\ubc29\ub41c \ub4a4 \ubbf8\uad70 \uc721\uad70 \uae30\uac11\ud559\uad50\uc5d0\uc11c \uac15\uc758\ub97c \ud558\uba74\uc11c \ud68c\uc0c1\ub85d\uc744 \uc9d1\ud544\ud588\ub2e4. 1954\ub144 5\uc6d4 14\uc77c \uc11c\ub3c5 \ubc14\ubc14\ub9ac\uc544 \ub0a8\ubd80\uc5d0\uc11c \uc0ac\ub9dd\ud558\uc600\uc73c\uba70 \uadf8\uc758 \ub4a4\ub97c \uc774\uc740 \uc544\ub4e4 \uc5ed\uc2dc \uc804\ud6c4 \ub3c5\uc77c \uc5f0\ubc29\uad70\uacfc \ubd81\ub300\uc11c\uc591 \uc870\uc57d \uae30\uad6c\uc5d0\uc11c \ud070 \uc5ed\ud560\uc744 \ub2f4\ub2f9\ud558\uc600\ub2e4.", "answer": "1945\ub144 5\uc6d4 10\uc77c", "sentence": "\uad6c\ub370\ub9ac\uc548\uc740 1945\ub144 5\uc6d4 10\uc77c \ubbf8\uad70\uc5d0 \ud56d\ubcf5\ud574 \ud3ec\ub85c\uac00 \ub418\uc5c8\ub2e4.", "paragraph_sentence": " \uad6c\ub370\ub9ac\uc548\uc740 1945\ub144 5\uc6d4 10\uc77c \ubbf8\uad70\uc5d0 \ud56d\ubcf5\ud574 \ud3ec\ub85c\uac00 \ub418\uc5c8\ub2e4. \ub258\ub978\ubca0\ub974\ud06c \uc7ac\ud310\uc774 \uc2dc\uc791\ub418\uc790 \uad6c \uc18c\ub828\uacfc \ud3f4\ub780\ub4dc\ub294 \uc804\uc7c1 \ubc94\uc8c4\uc790\ub85c\uc11c \uad6c\ub370\ub9ac\uc548\uc744 \uae30\uc18c\ud588\uc73c\ub098 \uacb0\uad6d \uae30\uc18c\ub294 \uc911\uc9c0\ub418\uc5c8\uace0 \uad6c\ub370\ub9ac\uc548\uc740 \uc544\ubb34\ub7f0 \ud610\uc758\uac00 \uc5c6\ub294 \uac83\uc73c\ub85c \uacb0\ub860\uc9c0\uc5b4\uc84c\ub2e4. \uadf8\uc758 \uc804\uc7c1\uae30\uac04\ub3d9\uc548\uc758 \ud589\uc704\ub4e4\uc740 \ubaa8\ub450 \uc815\ub2f9\ud55c \uad70\uc778\uc758 \uc790\uc138\uc5d0 \uc785\uac01\ud558\uc600\ub358 \uac83\uc73c\ub85c \ud310\uc815\ub41c \uac83\uc774\ub2e4. \uadf8\ub9ac\uace0 \uc7ac\ud310\uc5d0\uc11c \uad70\uc0ac \uc791\uc804\uc5d0 \ub300\ud55c(\ud2b9\ud788 \ud3f4\ub780\ub4dc \uce68\uacf5\uc5d0 \ub300\ud574\uc11c)\uc99d\uc5b8\uc744 \ub0a8\uacbc\ub2e4. 1948\ub144 6\uc6d4 17\uc77c \uc11d\ubc29\ub41c \ub4a4 \ubbf8\uad70 \uc721\uad70 \uae30\uac11\ud559\uad50\uc5d0\uc11c \uac15\uc758\ub97c \ud558\uba74\uc11c \ud68c\uc0c1\ub85d\uc744 \uc9d1\ud544\ud588\ub2e4. 1954\ub144 5\uc6d4 14\uc77c \uc11c\ub3c5 \ubc14\ubc14\ub9ac\uc544 \ub0a8\ubd80\uc5d0\uc11c \uc0ac\ub9dd\ud558\uc600\uc73c\uba70 \uadf8\uc758 \ub4a4\ub97c \uc774\uc740 \uc544\ub4e4 \uc5ed\uc2dc \uc804\ud6c4 \ub3c5\uc77c \uc5f0\ubc29\uad70\uacfc \ubd81\ub300\uc11c\uc591 \uc870\uc57d \uae30\uad6c\uc5d0\uc11c \ud070 \uc5ed\ud560\uc744 \ub2f4\ub2f9\ud558\uc600\ub2e4.", "paragraph_answer": "\uad6c\ub370\ub9ac\uc548\uc740 1945\ub144 5\uc6d4 10\uc77c \ubbf8\uad70\uc5d0 \ud56d\ubcf5\ud574 \ud3ec\ub85c\uac00 \ub418\uc5c8\ub2e4. \ub258\ub978\ubca0\ub974\ud06c \uc7ac\ud310\uc774 \uc2dc\uc791\ub418\uc790 \uad6c \uc18c\ub828\uacfc \ud3f4\ub780\ub4dc\ub294 \uc804\uc7c1 \ubc94\uc8c4\uc790\ub85c\uc11c \uad6c\ub370\ub9ac\uc548\uc744 \uae30\uc18c\ud588\uc73c\ub098 \uacb0\uad6d \uae30\uc18c\ub294 \uc911\uc9c0\ub418\uc5c8\uace0 \uad6c\ub370\ub9ac\uc548\uc740 \uc544\ubb34\ub7f0 \ud610\uc758\uac00 \uc5c6\ub294 \uac83\uc73c\ub85c \uacb0\ub860\uc9c0\uc5b4\uc84c\ub2e4. \uadf8\uc758 \uc804\uc7c1\uae30\uac04\ub3d9\uc548\uc758 \ud589\uc704\ub4e4\uc740 \ubaa8\ub450 \uc815\ub2f9\ud55c \uad70\uc778\uc758 \uc790\uc138\uc5d0 \uc785\uac01\ud558\uc600\ub358 \uac83\uc73c\ub85c \ud310\uc815\ub41c \uac83\uc774\ub2e4. \uadf8\ub9ac\uace0 \uc7ac\ud310\uc5d0\uc11c \uad70\uc0ac \uc791\uc804\uc5d0 \ub300\ud55c(\ud2b9\ud788 \ud3f4\ub780\ub4dc \uce68\uacf5\uc5d0 \ub300\ud574\uc11c)\uc99d\uc5b8\uc744 \ub0a8\uacbc\ub2e4. 1948\ub144 6\uc6d4 17\uc77c \uc11d\ubc29\ub41c \ub4a4 \ubbf8\uad70 \uc721\uad70 \uae30\uac11\ud559\uad50\uc5d0\uc11c \uac15\uc758\ub97c \ud558\uba74\uc11c \ud68c\uc0c1\ub85d\uc744 \uc9d1\ud544\ud588\ub2e4. 1954\ub144 5\uc6d4 14\uc77c \uc11c\ub3c5 \ubc14\ubc14\ub9ac\uc544 \ub0a8\ubd80\uc5d0\uc11c \uc0ac\ub9dd\ud558\uc600\uc73c\uba70 \uadf8\uc758 \ub4a4\ub97c \uc774\uc740 \uc544\ub4e4 \uc5ed\uc2dc \uc804\ud6c4 \ub3c5\uc77c \uc5f0\ubc29\uad70\uacfc \ubd81\ub300\uc11c\uc591 \uc870\uc57d \uae30\uad6c\uc5d0\uc11c \ud070 \uc5ed\ud560\uc744 \ub2f4\ub2f9\ud558\uc600\ub2e4.", "sentence_answer": "\uad6c\ub370\ub9ac\uc548\uc740 1945\ub144 5\uc6d4 10\uc77c \ubbf8\uad70\uc5d0 \ud56d\ubcf5\ud574 \ud3ec\ub85c\uac00 \ub418\uc5c8\ub2e4.", "paragraph_id": "6227985-2-0", "paragraph_question": "question: \ud558\uc778\uce20 \uad6c\ub370\ub9ac\uc548\uc740 \uc5b8\uc81c \ubbf8\uad70\uc5d0\uac8c \ud56d\ubcf5\ud574 \ud3ec\ub85c\uac00 \ub418\uc5c8\ub294\uac00?, context: \uad6c\ub370\ub9ac\uc548\uc740 1945\ub144 5\uc6d4 10\uc77c \ubbf8\uad70\uc5d0 \ud56d\ubcf5\ud574 \ud3ec\ub85c\uac00 \ub418\uc5c8\ub2e4. \ub258\ub978\ubca0\ub974\ud06c \uc7ac\ud310\uc774 \uc2dc\uc791\ub418\uc790 \uad6c \uc18c\ub828\uacfc \ud3f4\ub780\ub4dc\ub294 \uc804\uc7c1 \ubc94\uc8c4\uc790\ub85c\uc11c \uad6c\ub370\ub9ac\uc548\uc744 \uae30\uc18c\ud588\uc73c\ub098 \uacb0\uad6d \uae30\uc18c\ub294 \uc911\uc9c0\ub418\uc5c8\uace0 \uad6c\ub370\ub9ac\uc548\uc740 \uc544\ubb34\ub7f0 \ud610\uc758\uac00 \uc5c6\ub294 \uac83\uc73c\ub85c \uacb0\ub860\uc9c0\uc5b4\uc84c\ub2e4. \uadf8\uc758 \uc804\uc7c1\uae30\uac04\ub3d9\uc548\uc758 \ud589\uc704\ub4e4\uc740 \ubaa8\ub450 \uc815\ub2f9\ud55c \uad70\uc778\uc758 \uc790\uc138\uc5d0 \uc785\uac01\ud558\uc600\ub358 \uac83\uc73c\ub85c \ud310\uc815\ub41c \uac83\uc774\ub2e4. \uadf8\ub9ac\uace0 \uc7ac\ud310\uc5d0\uc11c \uad70\uc0ac \uc791\uc804\uc5d0 \ub300\ud55c(\ud2b9\ud788 \ud3f4\ub780\ub4dc \uce68\uacf5\uc5d0 \ub300\ud574\uc11c)\uc99d\uc5b8\uc744 \ub0a8\uacbc\ub2e4. 1948\ub144 6\uc6d4 17\uc77c \uc11d\ubc29\ub41c \ub4a4 \ubbf8\uad70 \uc721\uad70 \uae30\uac11\ud559\uad50\uc5d0\uc11c \uac15\uc758\ub97c \ud558\uba74\uc11c \ud68c\uc0c1\ub85d\uc744 \uc9d1\ud544\ud588\ub2e4. 1954\ub144 5\uc6d4 14\uc77c \uc11c\ub3c5 \ubc14\ubc14\ub9ac\uc544 \ub0a8\ubd80\uc5d0\uc11c \uc0ac\ub9dd\ud558\uc600\uc73c\uba70 \uadf8\uc758 \ub4a4\ub97c \uc774\uc740 \uc544\ub4e4 \uc5ed\uc2dc \uc804\ud6c4 \ub3c5\uc77c \uc5f0\ubc29\uad70\uacfc \ubd81\ub300\uc11c\uc591 \uc870\uc57d \uae30\uad6c\uc5d0\uc11c \ud070 \uc5ed\ud560\uc744 \ub2f4\ub2f9\ud558\uc600\ub2e4."} {"question": "<\uac00\ub514\uc5b8>\uc5d0\uc11c \uc2dc\ud2f0\uc988 \uc2a4\uce74\uc774\ub77c\uc778\uc744 \uc774\uc6a9\ud574 \uce5c\ud658\uacbd \ub3c4\uc2dc\ub97c \ub9cc\ub4e0 \uae30\uc790\ub294?", "paragraph": "\uac00\ub514\uc5b8\uc5d0\uc11c\ub294 \uc2e4\uc81c \ub3c4\uc2dc \uacc4\ud68d\uc0c1\uc758 \uc774\uc0c1\uc774 \ucef4\ud4e8\ud130 \uc2dc\ubbac\ub808\uc774\uc158\uc5d0\uc11c \uc77c\uc5b4\ub098\ub294\uc9c0 \ubcf4\uae30 \uc704\ud574\uc11c \uc2dc\ud2f0\uc988: \uc2a4\uce74\uc774\ub77c\uc778\uc744 \uc774\uc6a9\ud558\uc5ec \ud658\uacbd \uc804\ubb38 \uae30\uc790\uc778 Karl Mathiesen\uc5d0\uac8c \uce5c\ud658\uacbd\uc801\uc778 \ub3c4\uc2dc\ub97c \ub9cc\ub4e4\uc5b4\ubcf4\ub77c\uace0 \uc2dc\ud0a4\uae30\ub3c4 \ud558\uc600\ub2e4. \uae30\uc790\ub294 \uc2dc\ud2f0\uc988: \uc2a4\uce74\uc774\ub77c\uc778\uc73c\ub85c 7\ub9cc 8\ucc9c\uba85\ub418\ub294 \ub3c4\uc2dc\uc758 \uc5d0\ub108\uc9c0\ub97c \ubaa8\ub450 \uc7ac\uc0dd\uac00\ub2a5\ud55c \uc5d0\ub108\uc9c0\ub97c \uc0ac\uc6a9\ud558\uac8c \ud558\uc600\uc73c\uba70, \ud6a8\uacfc\uc801\uc778 \uc218\uc1a1\uacfc \ud558\uc774\ud14c\ud06c\uc640 \uacf5\ud574\ub97c \ubc1c\uc0dd\uc2dc\ud0a4\uc9c0 \uc54a\ub294 \uacf5\uc5c5\uc5d0 \ub300\ud55c \uc88b\uc740 \uc77c\uc790\ub9ac\ub97c \uc870\uac74\uc73c\ub85c \ub3c4\uc2dc\ub97c \ub9cc\ub4e4\uc5c8\ub2e4. \uadf8\ub9ac\uace0 \uc2dc\ud2f0\uc988: \uc2a4\uce74\uc774\ub77c\uc778\uc744 \ub3cc\ub9b0 Karl Mathiesen\uc740 \uc7ac\uc0dd \uac00\ub2a5 \uc5d0\ub108\uc9c0\uac00 \uac8c\uc784\uc0c1\uc758 \ube44\ud604\uc2e4\uc801\uc778 \uc88b\uc740 \ub0a0\uc528\ub85c \uc778\ud574\uc11c \uc800\ube44\uc6a9\uc73c\ub85c \ub098\uc654\ub2e4\uace0 \ub9d0\ud588\uc9c0\ub9cc, \ub3c4\uc2dc\uc758 \ub300\uc911\uad50\ud1b5 \uc2dc\uc2a4\ud15c\uc740 \uac8c\uc784\uc0c1\uc758 \ub3c4\uc2dc\uc640 \ube44\uad50 \ub300\uc0c1\uc73c\ub85c \uc0bc\uc740 \ub374\ub9c8\ud06c \ucf54\ud39c\ud558\uac90\ubcf4\ub2e4 \ub354 \ub5a8\uc5b4\uc84c\uc73c\uba70, \ucf54\ud39c\ud558\uac90\uacfc \ub354\ubd88\uc5b4 \uac8c\uc784\uc5d0\uc11c\ub3c4 \uc4f0\ub808\uae30 \ucc98\ub9ac\uc5d0 \ub300\ud55c \uc7a5\uae30\uc801\uc778 \ud574\uacb0\ucc45\uc744 \uac1c\ubc1c \ubabb\ud55c \uac83\uc774 \ud070 \uc7a5\uc560\ubb3c\uc774\ub77c\uace0 \ud3c9\uac00 \ud588\ub2e4.", "answer": "Karl Mathiesen", "sentence": "\uac00\ub514\uc5b8\uc5d0\uc11c\ub294 \uc2e4\uc81c \ub3c4\uc2dc \uacc4\ud68d\uc0c1\uc758 \uc774\uc0c1\uc774 \ucef4\ud4e8\ud130 \uc2dc\ubbac\ub808\uc774\uc158\uc5d0\uc11c \uc77c\uc5b4\ub098\ub294\uc9c0 \ubcf4\uae30 \uc704\ud574\uc11c \uc2dc\ud2f0\uc988: \uc2a4\uce74\uc774\ub77c\uc778\uc744 \uc774\uc6a9\ud558\uc5ec \ud658\uacbd \uc804\ubb38 \uae30\uc790\uc778 Karl Mathiesen \uc5d0\uac8c \uce5c\ud658\uacbd\uc801\uc778 \ub3c4\uc2dc\ub97c \ub9cc\ub4e4\uc5b4\ubcf4\ub77c\uace0 \uc2dc\ud0a4\uae30\ub3c4 \ud558\uc600\ub2e4.", "paragraph_sentence": " \uac00\ub514\uc5b8\uc5d0\uc11c\ub294 \uc2e4\uc81c \ub3c4\uc2dc \uacc4\ud68d\uc0c1\uc758 \uc774\uc0c1\uc774 \ucef4\ud4e8\ud130 \uc2dc\ubbac\ub808\uc774\uc158\uc5d0\uc11c \uc77c\uc5b4\ub098\ub294\uc9c0 \ubcf4\uae30 \uc704\ud574\uc11c \uc2dc\ud2f0\uc988: \uc2a4\uce74\uc774\ub77c\uc778\uc744 \uc774\uc6a9\ud558\uc5ec \ud658\uacbd \uc804\ubb38 \uae30\uc790\uc778 Karl Mathiesen \uc5d0\uac8c \uce5c\ud658\uacbd\uc801\uc778 \ub3c4\uc2dc\ub97c \ub9cc\ub4e4\uc5b4\ubcf4\ub77c\uace0 \uc2dc\ud0a4\uae30\ub3c4 \ud558\uc600\ub2e4. \uae30\uc790\ub294 \uc2dc\ud2f0\uc988: \uc2a4\uce74\uc774\ub77c\uc778\uc73c\ub85c 7\ub9cc 8\ucc9c\uba85\ub418\ub294 \ub3c4\uc2dc\uc758 \uc5d0\ub108\uc9c0\ub97c \ubaa8\ub450 \uc7ac\uc0dd\uac00\ub2a5\ud55c \uc5d0\ub108\uc9c0\ub97c \uc0ac\uc6a9\ud558\uac8c \ud558\uc600\uc73c\uba70, \ud6a8\uacfc\uc801\uc778 \uc218\uc1a1\uacfc \ud558\uc774\ud14c\ud06c\uc640 \uacf5\ud574\ub97c \ubc1c\uc0dd\uc2dc\ud0a4\uc9c0 \uc54a\ub294 \uacf5\uc5c5\uc5d0 \ub300\ud55c \uc88b\uc740 \uc77c\uc790\ub9ac\ub97c \uc870\uac74\uc73c\ub85c \ub3c4\uc2dc\ub97c \ub9cc\ub4e4\uc5c8\ub2e4. \uadf8\ub9ac\uace0 \uc2dc\ud2f0\uc988: \uc2a4\uce74\uc774\ub77c\uc778\uc744 \ub3cc\ub9b0 Karl Mathiesen\uc740 \uc7ac\uc0dd \uac00\ub2a5 \uc5d0\ub108\uc9c0\uac00 \uac8c\uc784\uc0c1\uc758 \ube44\ud604\uc2e4\uc801\uc778 \uc88b\uc740 \ub0a0\uc528\ub85c \uc778\ud574\uc11c \uc800\ube44\uc6a9\uc73c\ub85c \ub098\uc654\ub2e4\uace0 \ub9d0\ud588\uc9c0\ub9cc, \ub3c4\uc2dc\uc758 \ub300\uc911\uad50\ud1b5 \uc2dc\uc2a4\ud15c\uc740 \uac8c\uc784\uc0c1\uc758 \ub3c4\uc2dc\uc640 \ube44\uad50 \ub300\uc0c1\uc73c\ub85c \uc0bc\uc740 \ub374\ub9c8\ud06c \ucf54\ud39c\ud558\uac90\ubcf4\ub2e4 \ub354 \ub5a8\uc5b4\uc84c\uc73c\uba70, \ucf54\ud39c\ud558\uac90\uacfc \ub354\ubd88\uc5b4 \uac8c\uc784\uc5d0\uc11c\ub3c4 \uc4f0\ub808\uae30 \ucc98\ub9ac\uc5d0 \ub300\ud55c \uc7a5\uae30\uc801\uc778 \ud574\uacb0\ucc45\uc744 \uac1c\ubc1c \ubabb\ud55c \uac83\uc774 \ud070 \uc7a5\uc560\ubb3c\uc774\ub77c\uace0 \ud3c9\uac00 \ud588\ub2e4.", "paragraph_answer": "\uac00\ub514\uc5b8\uc5d0\uc11c\ub294 \uc2e4\uc81c \ub3c4\uc2dc \uacc4\ud68d\uc0c1\uc758 \uc774\uc0c1\uc774 \ucef4\ud4e8\ud130 \uc2dc\ubbac\ub808\uc774\uc158\uc5d0\uc11c \uc77c\uc5b4\ub098\ub294\uc9c0 \ubcf4\uae30 \uc704\ud574\uc11c \uc2dc\ud2f0\uc988: \uc2a4\uce74\uc774\ub77c\uc778\uc744 \uc774\uc6a9\ud558\uc5ec \ud658\uacbd \uc804\ubb38 \uae30\uc790\uc778 Karl Mathiesen \uc5d0\uac8c \uce5c\ud658\uacbd\uc801\uc778 \ub3c4\uc2dc\ub97c \ub9cc\ub4e4\uc5b4\ubcf4\ub77c\uace0 \uc2dc\ud0a4\uae30\ub3c4 \ud558\uc600\ub2e4. \uae30\uc790\ub294 \uc2dc\ud2f0\uc988: \uc2a4\uce74\uc774\ub77c\uc778\uc73c\ub85c 7\ub9cc 8\ucc9c\uba85\ub418\ub294 \ub3c4\uc2dc\uc758 \uc5d0\ub108\uc9c0\ub97c \ubaa8\ub450 \uc7ac\uc0dd\uac00\ub2a5\ud55c \uc5d0\ub108\uc9c0\ub97c \uc0ac\uc6a9\ud558\uac8c \ud558\uc600\uc73c\uba70, \ud6a8\uacfc\uc801\uc778 \uc218\uc1a1\uacfc \ud558\uc774\ud14c\ud06c\uc640 \uacf5\ud574\ub97c \ubc1c\uc0dd\uc2dc\ud0a4\uc9c0 \uc54a\ub294 \uacf5\uc5c5\uc5d0 \ub300\ud55c \uc88b\uc740 \uc77c\uc790\ub9ac\ub97c \uc870\uac74\uc73c\ub85c \ub3c4\uc2dc\ub97c \ub9cc\ub4e4\uc5c8\ub2e4. \uadf8\ub9ac\uace0 \uc2dc\ud2f0\uc988: \uc2a4\uce74\uc774\ub77c\uc778\uc744 \ub3cc\ub9b0 Karl Mathiesen\uc740 \uc7ac\uc0dd \uac00\ub2a5 \uc5d0\ub108\uc9c0\uac00 \uac8c\uc784\uc0c1\uc758 \ube44\ud604\uc2e4\uc801\uc778 \uc88b\uc740 \ub0a0\uc528\ub85c \uc778\ud574\uc11c \uc800\ube44\uc6a9\uc73c\ub85c \ub098\uc654\ub2e4\uace0 \ub9d0\ud588\uc9c0\ub9cc, \ub3c4\uc2dc\uc758 \ub300\uc911\uad50\ud1b5 \uc2dc\uc2a4\ud15c\uc740 \uac8c\uc784\uc0c1\uc758 \ub3c4\uc2dc\uc640 \ube44\uad50 \ub300\uc0c1\uc73c\ub85c \uc0bc\uc740 \ub374\ub9c8\ud06c \ucf54\ud39c\ud558\uac90\ubcf4\ub2e4 \ub354 \ub5a8\uc5b4\uc84c\uc73c\uba70, \ucf54\ud39c\ud558\uac90\uacfc \ub354\ubd88\uc5b4 \uac8c\uc784\uc5d0\uc11c\ub3c4 \uc4f0\ub808\uae30 \ucc98\ub9ac\uc5d0 \ub300\ud55c \uc7a5\uae30\uc801\uc778 \ud574\uacb0\ucc45\uc744 \uac1c\ubc1c \ubabb\ud55c \uac83\uc774 \ud070 \uc7a5\uc560\ubb3c\uc774\ub77c\uace0 \ud3c9\uac00 \ud588\ub2e4.", "sentence_answer": "\uac00\ub514\uc5b8\uc5d0\uc11c\ub294 \uc2e4\uc81c \ub3c4\uc2dc \uacc4\ud68d\uc0c1\uc758 \uc774\uc0c1\uc774 \ucef4\ud4e8\ud130 \uc2dc\ubbac\ub808\uc774\uc158\uc5d0\uc11c \uc77c\uc5b4\ub098\ub294\uc9c0 \ubcf4\uae30 \uc704\ud574\uc11c \uc2dc\ud2f0\uc988: \uc2a4\uce74\uc774\ub77c\uc778\uc744 \uc774\uc6a9\ud558\uc5ec \ud658\uacbd \uc804\ubb38 \uae30\uc790\uc778 Karl Mathiesen \uc5d0\uac8c \uce5c\ud658\uacbd\uc801\uc778 \ub3c4\uc2dc\ub97c \ub9cc\ub4e4\uc5b4\ubcf4\ub77c\uace0 \uc2dc\ud0a4\uae30\ub3c4 \ud558\uc600\ub2e4.", "paragraph_id": "6519573-11-0", "paragraph_question": "question: <\uac00\ub514\uc5b8>\uc5d0\uc11c \uc2dc\ud2f0\uc988 \uc2a4\uce74\uc774\ub77c\uc778\uc744 \uc774\uc6a9\ud574 \uce5c\ud658\uacbd \ub3c4\uc2dc\ub97c \ub9cc\ub4e0 \uae30\uc790\ub294?, context: \uac00\ub514\uc5b8\uc5d0\uc11c\ub294 \uc2e4\uc81c \ub3c4\uc2dc \uacc4\ud68d\uc0c1\uc758 \uc774\uc0c1\uc774 \ucef4\ud4e8\ud130 \uc2dc\ubbac\ub808\uc774\uc158\uc5d0\uc11c \uc77c\uc5b4\ub098\ub294\uc9c0 \ubcf4\uae30 \uc704\ud574\uc11c \uc2dc\ud2f0\uc988: \uc2a4\uce74\uc774\ub77c\uc778\uc744 \uc774\uc6a9\ud558\uc5ec \ud658\uacbd \uc804\ubb38 \uae30\uc790\uc778 Karl Mathiesen\uc5d0\uac8c \uce5c\ud658\uacbd\uc801\uc778 \ub3c4\uc2dc\ub97c \ub9cc\ub4e4\uc5b4\ubcf4\ub77c\uace0 \uc2dc\ud0a4\uae30\ub3c4 \ud558\uc600\ub2e4. \uae30\uc790\ub294 \uc2dc\ud2f0\uc988: \uc2a4\uce74\uc774\ub77c\uc778\uc73c\ub85c 7\ub9cc 8\ucc9c\uba85\ub418\ub294 \ub3c4\uc2dc\uc758 \uc5d0\ub108\uc9c0\ub97c \ubaa8\ub450 \uc7ac\uc0dd\uac00\ub2a5\ud55c \uc5d0\ub108\uc9c0\ub97c \uc0ac\uc6a9\ud558\uac8c \ud558\uc600\uc73c\uba70, \ud6a8\uacfc\uc801\uc778 \uc218\uc1a1\uacfc \ud558\uc774\ud14c\ud06c\uc640 \uacf5\ud574\ub97c \ubc1c\uc0dd\uc2dc\ud0a4\uc9c0 \uc54a\ub294 \uacf5\uc5c5\uc5d0 \ub300\ud55c \uc88b\uc740 \uc77c\uc790\ub9ac\ub97c \uc870\uac74\uc73c\ub85c \ub3c4\uc2dc\ub97c \ub9cc\ub4e4\uc5c8\ub2e4. \uadf8\ub9ac\uace0 \uc2dc\ud2f0\uc988: \uc2a4\uce74\uc774\ub77c\uc778\uc744 \ub3cc\ub9b0 Karl Mathiesen\uc740 \uc7ac\uc0dd \uac00\ub2a5 \uc5d0\ub108\uc9c0\uac00 \uac8c\uc784\uc0c1\uc758 \ube44\ud604\uc2e4\uc801\uc778 \uc88b\uc740 \ub0a0\uc528\ub85c \uc778\ud574\uc11c \uc800\ube44\uc6a9\uc73c\ub85c \ub098\uc654\ub2e4\uace0 \ub9d0\ud588\uc9c0\ub9cc, \ub3c4\uc2dc\uc758 \ub300\uc911\uad50\ud1b5 \uc2dc\uc2a4\ud15c\uc740 \uac8c\uc784\uc0c1\uc758 \ub3c4\uc2dc\uc640 \ube44\uad50 \ub300\uc0c1\uc73c\ub85c \uc0bc\uc740 \ub374\ub9c8\ud06c \ucf54\ud39c\ud558\uac90\ubcf4\ub2e4 \ub354 \ub5a8\uc5b4\uc84c\uc73c\uba70, \ucf54\ud39c\ud558\uac90\uacfc \ub354\ubd88\uc5b4 \uac8c\uc784\uc5d0\uc11c\ub3c4 \uc4f0\ub808\uae30 \ucc98\ub9ac\uc5d0 \ub300\ud55c \uc7a5\uae30\uc801\uc778 \ud574\uacb0\ucc45\uc744 \uac1c\ubc1c \ubabb\ud55c \uac83\uc774 \ud070 \uc7a5\uc560\ubb3c\uc774\ub77c\uace0 \ud3c9\uac00 \ud588\ub2e4."} {"question": "\uc950\ub77c\uae30 \uc911\uae30\uc758 \uc2a4\ud14c\uace0\uc0ac\uc6b0\ub8e8\uc2a4\ubcf4\ub2e4 2000\ub9cc\ub144 \uc55e\uc11c \uc0b4\uc558\ub358 \uae30\ubc18\uad70\uc740?", "paragraph": "\uc774\ubcf4\ub2e4 \uc774\uc804\uc758 \uae30\ubc18\uad70\uc778 \ud654\uc591\uace0\uc0ac\uc6b0\ub8e8\uc2a4 \uc18d\uc740 \uc950\ub77c\uae30 \uc911\uae30 (1\uc5b5 6500\ub9cc \ub144 \uc804)\uc758 \uc911\uad6d\uc5d0\uc11c \ubc1c\uacac\ub418\uc5b4 \uc2a4\ud14c\uace0\uc0ac\uc6b0\ub8e8\uc2a4\ubcf4\ub2e4 2000\ub9cc \ub144 \uc55e\uc11c \uc0b4\uc558\uc73c\uba70 \ud654\uc591\uace0\uc0ac\uc6b0\ub8e8\uc2a4\uacfc\uc758 \uc720\uc77c\ud55c \uc18d\uc774\ub2e4. \uadf8\ubcf4\ub2e4 \ub354 \uc55e\uc120 \uc885\ub958\ub85c\ub294 \uc2a4\ucf08\ub9ac\ub3c4\uc0ac\uc6b0\ub8e8\uc2a4(Scelidosaurus)\uac00 \uc788\ub294\ub370, \uc774 \uc885\uc740 \uc950\ub77c\uae30 \ucd08\uae30 \uc57d 1\uc5b5 9000\ub9cc \ub144 \uc804\uc758 \uc789\uae00\ub79c\ub4dc\uc5d0\uc11c \uc0b4\uc558\ub2e4. \uc7ac\ubbf8\uc788\uac8c\ub3c4 \uc2a4\ucf08\ub9ac\ub3c4\uc0ac\uc6b0\ub8e8\uc2a4\ub294 \uc2a4\ud14c\uace0\uc0ac\uc6b0\ub8e8\uc2a4\uacfc\uc640 \uc548\ud0ac\ub85c\uc0ac\uc6b0\ub8e8\uc2a4\uacfc\uc758 \ud2b9\uc9d5\uc744 \ubaa8\ub450 \uac00\uc9c0\uace0 \uc788\uc5c8\ub2e4. \uc5d0\ub9c8\uc6b0\uc0ac\uc6b0\ub8e8\uc2a4(Emausaurus)\ub294 \ub3c5\uc77c\uc5d0\uc11c \ubc1c\uacac\ub41c \uc791\uc740 \uc0ac\uc871\ubcf4\ud589 \uacf5\ub8e1\uc774\uba70, \ubbf8\uad6d\uc758 \uc560\ub9ac\uc870\ub098\uc5d0\uc11c \ubc1c\uacac\ub41c \uc2a4\ucfe0\ud154\ub85c\uc0ac\uc6b0\ub8e8\uc2a4\uac00 \ub354 \uc624\ub798\ub41c \uc18d\uc73c\ub85c \ud544\uc694\uc5d0 \ub530\ub77c \uc774\uc871\ubcf4\ud589\uc744 \ud588\ub358 \uac83\uc73c\ub85c \ubcf4\uc778\ub2e4. \uc0c1\ub300\uc801\uc73c\ub85c \uac00\ubcbc\uc6b4 \uac11\uc637\uc744 \uac16\ucd98 \uc774 \uc791\uc740 \uacf5\ub8e1\ub4e4\uc740 \uc2a4\ud14c\uace0\uc0ac\uc6b0\ub8e8\uc2a4\uacfc \ubc0f \uc548\ud0ac\ub85c\uc0ac\uc6b0\ub8e8\uc2a4\uacfc \uc591\ucabd\uc758 \uacf5\ud1b5\uc870\uc0c1\uacfc \uac00\uae4c\uc6b4 \uad00\uacc4\uc600\uc744 \uac83\uc774\ub2e4. \ucd08\uae30 \uc7a5\uc21c\ub958 \uacf5\ub8e1\uc758 \ubcf4\ud589\ub82c\uc774 \ud504\ub791\uc2a4\uc758 1\uc5b5 9500\ub9cc \ub144 \uc804 \uc9c0\uce35\uc5d0\uc11c \ubc1c\uacac\ub418\uc5c8\ub2e4.", "answer": "\ud654\uc591\uace0\uc0ac\uc6b0\ub8e8\uc2a4 \uc18d", "sentence": "\uc774\ubcf4\ub2e4 \uc774\uc804\uc758 \uae30\ubc18\uad70\uc778 \ud654\uc591\uace0\uc0ac\uc6b0\ub8e8\uc2a4 \uc18d \uc740 \uc950\ub77c\uae30 \uc911\uae30 (1\uc5b5 6500\ub9cc \ub144 \uc804)\uc758 \uc911\uad6d\uc5d0\uc11c \ubc1c\uacac\ub418\uc5b4 \uc2a4\ud14c\uace0\uc0ac\uc6b0\ub8e8\uc2a4\ubcf4\ub2e4 2000\ub9cc \ub144 \uc55e\uc11c \uc0b4\uc558\uc73c\uba70 \ud654\uc591\uace0\uc0ac\uc6b0\ub8e8\uc2a4\uacfc\uc758 \uc720\uc77c\ud55c \uc18d\uc774\ub2e4.", "paragraph_sentence": " \uc774\ubcf4\ub2e4 \uc774\uc804\uc758 \uae30\ubc18\uad70\uc778 \ud654\uc591\uace0\uc0ac\uc6b0\ub8e8\uc2a4 \uc18d \uc740 \uc950\ub77c\uae30 \uc911\uae30 (1\uc5b5 6500\ub9cc \ub144 \uc804)\uc758 \uc911\uad6d\uc5d0\uc11c \ubc1c\uacac\ub418\uc5b4 \uc2a4\ud14c\uace0\uc0ac\uc6b0\ub8e8\uc2a4\ubcf4\ub2e4 2000\ub9cc \ub144 \uc55e\uc11c \uc0b4\uc558\uc73c\uba70 \ud654\uc591\uace0\uc0ac\uc6b0\ub8e8\uc2a4\uacfc\uc758 \uc720\uc77c\ud55c \uc18d\uc774\ub2e4. \uadf8\ubcf4\ub2e4 \ub354 \uc55e\uc120 \uc885\ub958\ub85c\ub294 \uc2a4\ucf08\ub9ac\ub3c4\uc0ac\uc6b0\ub8e8\uc2a4(Scelidosaurus)\uac00 \uc788\ub294\ub370, \uc774 \uc885\uc740 \uc950\ub77c\uae30 \ucd08\uae30 \uc57d 1\uc5b5 9000\ub9cc \ub144 \uc804\uc758 \uc789\uae00\ub79c\ub4dc\uc5d0\uc11c \uc0b4\uc558\ub2e4. \uc7ac\ubbf8\uc788\uac8c\ub3c4 \uc2a4\ucf08\ub9ac\ub3c4\uc0ac\uc6b0\ub8e8\uc2a4\ub294 \uc2a4\ud14c\uace0\uc0ac\uc6b0\ub8e8\uc2a4\uacfc\uc640 \uc548\ud0ac\ub85c\uc0ac\uc6b0\ub8e8\uc2a4\uacfc\uc758 \ud2b9\uc9d5\uc744 \ubaa8\ub450 \uac00\uc9c0\uace0 \uc788\uc5c8\ub2e4. \uc5d0\ub9c8\uc6b0\uc0ac\uc6b0\ub8e8\uc2a4(Emausaurus)\ub294 \ub3c5\uc77c\uc5d0\uc11c \ubc1c\uacac\ub41c \uc791\uc740 \uc0ac\uc871\ubcf4\ud589 \uacf5\ub8e1\uc774\uba70, \ubbf8\uad6d\uc758 \uc560\ub9ac\uc870\ub098\uc5d0\uc11c \ubc1c\uacac\ub41c \uc2a4\ucfe0\ud154\ub85c\uc0ac\uc6b0\ub8e8\uc2a4\uac00 \ub354 \uc624\ub798\ub41c \uc18d\uc73c\ub85c \ud544\uc694\uc5d0 \ub530\ub77c \uc774\uc871\ubcf4\ud589\uc744 \ud588\ub358 \uac83\uc73c\ub85c \ubcf4\uc778\ub2e4. \uc0c1\ub300\uc801\uc73c\ub85c \uac00\ubcbc\uc6b4 \uac11\uc637\uc744 \uac16\ucd98 \uc774 \uc791\uc740 \uacf5\ub8e1\ub4e4\uc740 \uc2a4\ud14c\uace0\uc0ac\uc6b0\ub8e8\uc2a4\uacfc \ubc0f \uc548\ud0ac\ub85c\uc0ac\uc6b0\ub8e8\uc2a4\uacfc \uc591\ucabd\uc758 \uacf5\ud1b5\uc870\uc0c1\uacfc \uac00\uae4c\uc6b4 \uad00\uacc4\uc600\uc744 \uac83\uc774\ub2e4. \ucd08\uae30 \uc7a5\uc21c\ub958 \uacf5\ub8e1\uc758 \ubcf4\ud589\ub82c\uc774 \ud504\ub791\uc2a4\uc758 1\uc5b5 9500\ub9cc \ub144 \uc804 \uc9c0\uce35\uc5d0\uc11c \ubc1c\uacac\ub418\uc5c8\ub2e4.", "paragraph_answer": "\uc774\ubcf4\ub2e4 \uc774\uc804\uc758 \uae30\ubc18\uad70\uc778 \ud654\uc591\uace0\uc0ac\uc6b0\ub8e8\uc2a4 \uc18d \uc740 \uc950\ub77c\uae30 \uc911\uae30 (1\uc5b5 6500\ub9cc \ub144 \uc804)\uc758 \uc911\uad6d\uc5d0\uc11c \ubc1c\uacac\ub418\uc5b4 \uc2a4\ud14c\uace0\uc0ac\uc6b0\ub8e8\uc2a4\ubcf4\ub2e4 2000\ub9cc \ub144 \uc55e\uc11c \uc0b4\uc558\uc73c\uba70 \ud654\uc591\uace0\uc0ac\uc6b0\ub8e8\uc2a4\uacfc\uc758 \uc720\uc77c\ud55c \uc18d\uc774\ub2e4. \uadf8\ubcf4\ub2e4 \ub354 \uc55e\uc120 \uc885\ub958\ub85c\ub294 \uc2a4\ucf08\ub9ac\ub3c4\uc0ac\uc6b0\ub8e8\uc2a4(Scelidosaurus)\uac00 \uc788\ub294\ub370, \uc774 \uc885\uc740 \uc950\ub77c\uae30 \ucd08\uae30 \uc57d 1\uc5b5 9000\ub9cc \ub144 \uc804\uc758 \uc789\uae00\ub79c\ub4dc\uc5d0\uc11c \uc0b4\uc558\ub2e4. \uc7ac\ubbf8\uc788\uac8c\ub3c4 \uc2a4\ucf08\ub9ac\ub3c4\uc0ac\uc6b0\ub8e8\uc2a4\ub294 \uc2a4\ud14c\uace0\uc0ac\uc6b0\ub8e8\uc2a4\uacfc\uc640 \uc548\ud0ac\ub85c\uc0ac\uc6b0\ub8e8\uc2a4\uacfc\uc758 \ud2b9\uc9d5\uc744 \ubaa8\ub450 \uac00\uc9c0\uace0 \uc788\uc5c8\ub2e4. \uc5d0\ub9c8\uc6b0\uc0ac\uc6b0\ub8e8\uc2a4(Emausaurus)\ub294 \ub3c5\uc77c\uc5d0\uc11c \ubc1c\uacac\ub41c \uc791\uc740 \uc0ac\uc871\ubcf4\ud589 \uacf5\ub8e1\uc774\uba70, \ubbf8\uad6d\uc758 \uc560\ub9ac\uc870\ub098\uc5d0\uc11c \ubc1c\uacac\ub41c \uc2a4\ucfe0\ud154\ub85c\uc0ac\uc6b0\ub8e8\uc2a4\uac00 \ub354 \uc624\ub798\ub41c \uc18d\uc73c\ub85c \ud544\uc694\uc5d0 \ub530\ub77c \uc774\uc871\ubcf4\ud589\uc744 \ud588\ub358 \uac83\uc73c\ub85c \ubcf4\uc778\ub2e4. \uc0c1\ub300\uc801\uc73c\ub85c \uac00\ubcbc\uc6b4 \uac11\uc637\uc744 \uac16\ucd98 \uc774 \uc791\uc740 \uacf5\ub8e1\ub4e4\uc740 \uc2a4\ud14c\uace0\uc0ac\uc6b0\ub8e8\uc2a4\uacfc \ubc0f \uc548\ud0ac\ub85c\uc0ac\uc6b0\ub8e8\uc2a4\uacfc \uc591\ucabd\uc758 \uacf5\ud1b5\uc870\uc0c1\uacfc \uac00\uae4c\uc6b4 \uad00\uacc4\uc600\uc744 \uac83\uc774\ub2e4. \ucd08\uae30 \uc7a5\uc21c\ub958 \uacf5\ub8e1\uc758 \ubcf4\ud589\ub82c\uc774 \ud504\ub791\uc2a4\uc758 1\uc5b5 9500\ub9cc \ub144 \uc804 \uc9c0\uce35\uc5d0\uc11c \ubc1c\uacac\ub418\uc5c8\ub2e4.", "sentence_answer": "\uc774\ubcf4\ub2e4 \uc774\uc804\uc758 \uae30\ubc18\uad70\uc778 \ud654\uc591\uace0\uc0ac\uc6b0\ub8e8\uc2a4 \uc18d \uc740 \uc950\ub77c\uae30 \uc911\uae30 (1\uc5b5 6500\ub9cc \ub144 \uc804)\uc758 \uc911\uad6d\uc5d0\uc11c \ubc1c\uacac\ub418\uc5b4 \uc2a4\ud14c\uace0\uc0ac\uc6b0\ub8e8\uc2a4\ubcf4\ub2e4 2000\ub9cc \ub144 \uc55e\uc11c \uc0b4\uc558\uc73c\uba70 \ud654\uc591\uace0\uc0ac\uc6b0\ub8e8\uc2a4\uacfc\uc758 \uc720\uc77c\ud55c \uc18d\uc774\ub2e4.", "paragraph_id": "6657232-3-0", "paragraph_question": "question: \uc950\ub77c\uae30 \uc911\uae30\uc758 \uc2a4\ud14c\uace0\uc0ac\uc6b0\ub8e8\uc2a4\ubcf4\ub2e4 2000\ub9cc\ub144 \uc55e\uc11c \uc0b4\uc558\ub358 \uae30\ubc18\uad70\uc740?, context: \uc774\ubcf4\ub2e4 \uc774\uc804\uc758 \uae30\ubc18\uad70\uc778 \ud654\uc591\uace0\uc0ac\uc6b0\ub8e8\uc2a4 \uc18d\uc740 \uc950\ub77c\uae30 \uc911\uae30 (1\uc5b5 6500\ub9cc \ub144 \uc804)\uc758 \uc911\uad6d\uc5d0\uc11c \ubc1c\uacac\ub418\uc5b4 \uc2a4\ud14c\uace0\uc0ac\uc6b0\ub8e8\uc2a4\ubcf4\ub2e4 2000\ub9cc \ub144 \uc55e\uc11c \uc0b4\uc558\uc73c\uba70 \ud654\uc591\uace0\uc0ac\uc6b0\ub8e8\uc2a4\uacfc\uc758 \uc720\uc77c\ud55c \uc18d\uc774\ub2e4. \uadf8\ubcf4\ub2e4 \ub354 \uc55e\uc120 \uc885\ub958\ub85c\ub294 \uc2a4\ucf08\ub9ac\ub3c4\uc0ac\uc6b0\ub8e8\uc2a4(Scelidosaurus)\uac00 \uc788\ub294\ub370, \uc774 \uc885\uc740 \uc950\ub77c\uae30 \ucd08\uae30 \uc57d 1\uc5b5 9000\ub9cc \ub144 \uc804\uc758 \uc789\uae00\ub79c\ub4dc\uc5d0\uc11c \uc0b4\uc558\ub2e4. \uc7ac\ubbf8\uc788\uac8c\ub3c4 \uc2a4\ucf08\ub9ac\ub3c4\uc0ac\uc6b0\ub8e8\uc2a4\ub294 \uc2a4\ud14c\uace0\uc0ac\uc6b0\ub8e8\uc2a4\uacfc\uc640 \uc548\ud0ac\ub85c\uc0ac\uc6b0\ub8e8\uc2a4\uacfc\uc758 \ud2b9\uc9d5\uc744 \ubaa8\ub450 \uac00\uc9c0\uace0 \uc788\uc5c8\ub2e4. \uc5d0\ub9c8\uc6b0\uc0ac\uc6b0\ub8e8\uc2a4(Emausaurus)\ub294 \ub3c5\uc77c\uc5d0\uc11c \ubc1c\uacac\ub41c \uc791\uc740 \uc0ac\uc871\ubcf4\ud589 \uacf5\ub8e1\uc774\uba70, \ubbf8\uad6d\uc758 \uc560\ub9ac\uc870\ub098\uc5d0\uc11c \ubc1c\uacac\ub41c \uc2a4\ucfe0\ud154\ub85c\uc0ac\uc6b0\ub8e8\uc2a4\uac00 \ub354 \uc624\ub798\ub41c \uc18d\uc73c\ub85c \ud544\uc694\uc5d0 \ub530\ub77c \uc774\uc871\ubcf4\ud589\uc744 \ud588\ub358 \uac83\uc73c\ub85c \ubcf4\uc778\ub2e4. \uc0c1\ub300\uc801\uc73c\ub85c \uac00\ubcbc\uc6b4 \uac11\uc637\uc744 \uac16\ucd98 \uc774 \uc791\uc740 \uacf5\ub8e1\ub4e4\uc740 \uc2a4\ud14c\uace0\uc0ac\uc6b0\ub8e8\uc2a4\uacfc \ubc0f \uc548\ud0ac\ub85c\uc0ac\uc6b0\ub8e8\uc2a4\uacfc \uc591\ucabd\uc758 \uacf5\ud1b5\uc870\uc0c1\uacfc \uac00\uae4c\uc6b4 \uad00\uacc4\uc600\uc744 \uac83\uc774\ub2e4. \ucd08\uae30 \uc7a5\uc21c\ub958 \uacf5\ub8e1\uc758 \ubcf4\ud589\ub82c\uc774 \ud504\ub791\uc2a4\uc758 1\uc5b5 9500\ub9cc \ub144 \uc804 \uc9c0\uce35\uc5d0\uc11c \ubc1c\uacac\ub418\uc5c8\ub2e4."} {"question": "\uc815\ubbfc\ucca0\uc740 \uc77c\ubcf8\uc758 \uc5b4\ub290 \uad6c\ub2e8\uc5d0 \uc785\ub2e8\ud588\ub098?", "paragraph": "2000\ub144 \uc77c\ubcf8 \ud504\ub85c \uc57c\uad6c \uc694\ubbf8\uc6b0\ub9ac \uc790\uc774\uc5b8\uce20\uc5d0 \uc785\ub2e8\ud558\uc5ec 2\uc2dc\uc98c\uc744 \ubcf4\ub0b8\ub2e4. \uad6d\ub0b4\uc5d0\uc11c\ub294 \ub9ac\uadf8 \ucd5c\uace0 \uc218\uc900\uc758 \uc5d0\uc774\uc2a4\uc600\uc9c0\ub9cc \uc77c\ubcf8 \ub9ac\uadf8\uc5d0\uc11c\ub294 \ud604\uc9c0 \uc801\uc751 \uc2e4\ud328\uc640 \ub9ac\uadf8\uaca9\ucc28\ub85c \uc778\ud574 \ub69c\ub837\ud55c \uc131\uacfc\ub97c \uc62c\ub9ac\uc9c0 \ubabb\ud588\ub2e4. \uc77c\ubcf8 \ub9ac\uadf8\uc5d0\uc11c \uadf8\ub294 2\uc2dc\uc98c \ub3d9\uc548 3\uc2b9 2\ud328\uc758 \ucd08\ub77c\ud55c \uc131\uc801\uc744 \uac70\ub450\uc5c8\uace0, \ubd80\uc9c4\ud588\ub358 \uc8fc\uc694\ud55c \uc6d0\uc778\uc73c\ub85c\ub294 \uae09\uaca9\ud55c \uad6c\uc704 \uc800\ud558, \ud314\uafc8\uce58 \ud1b5\uc99d\uc774\uc5c8\ub2e4. \uadf8\ub807\uc9c0\ub9cc \uc774 \uc548\ud0c0\uae4c\uc6b4 \uacb0\uacfc\uc5d0\ub294 \uc5ec\ub7ec \ub17c\ub780\uc774 \uc788\ub294\ub370 \uc2e4\uc81c\ub85c \uc815\ubbfc\ucca0\uc758 \uc694\ubbf8\uc6b0\ub9ac \uc785\ub2e8 \ud6c4 \uccab \ub4f1\ud310\uc740 \uc57c\ucfe0\ub974\ud2b8\ub97c \uc0c1\ub300\ub85c 7\uc774\ub2dd 1\uc2e4\uc810 \uc2b9\uc73c\ub85c \ud638\ud22c\ud558\uc600\uc73c\ub098 \uacbd\uae30 \uc9c1\ud6c4 2\uad70 \ud1b5\ubcf4\ub97c \ubc1b\uace0, \ub2e4\uc2dc \uc62c\ub77c\uc628 1\uad70 \uacbd\uae30\uc5d0\uc11c \uc694\ucf54\ud558\ub9c8\ub97c \uc0c1\ub300\ub85c 9\uc774\ub2dd \ubb34\uc2e4\uc810\uc73c\ub85c \uc644\ubd09\uc2b9 \ud558\uc600\uc73c\ub098 \uc774\ud6c4\ub85c\ub3c4 \ub4f1\ud310\uae30\ud68c\ub294 \uc880\ucc98\ub7fc \uc8fc\uc5b4\uc9c0\uc9c0 \uc54a\uc558\ub2e4. \uadf8 \ud6c4 \uad6d\ub0b4 \ubcf5\uadc0 20\uc644\ubd09 \ub2ec\uc131 \uc778\ud130\ubdf0\uc5d0\uc11c \"\uc694\ubbf8\uc6b0\ub9ac\uc758 \uc2dc\uc2a4\ud15c\uc774 \ucc38 \uc774\uc0c1\ud588\ub2e4.\"\ub77c\uace0 \ud68c\uace0\ud55c \ubc14 \uc788\ub2e4.", "answer": "\uc694\ubbf8\uc6b0\ub9ac \uc790\uc774\uc5b8\uce20", "sentence": "2000\ub144 \uc77c\ubcf8 \ud504\ub85c \uc57c\uad6c \uc694\ubbf8\uc6b0\ub9ac \uc790\uc774\uc5b8\uce20 \uc5d0 \uc785\ub2e8\ud558\uc5ec 2\uc2dc\uc98c\uc744 \ubcf4\ub0b8\ub2e4.", "paragraph_sentence": " 2000\ub144 \uc77c\ubcf8 \ud504\ub85c \uc57c\uad6c \uc694\ubbf8\uc6b0\ub9ac \uc790\uc774\uc5b8\uce20 \uc5d0 \uc785\ub2e8\ud558\uc5ec 2\uc2dc\uc98c\uc744 \ubcf4\ub0b8\ub2e4. \uad6d\ub0b4\uc5d0\uc11c\ub294 \ub9ac\uadf8 \ucd5c\uace0 \uc218\uc900\uc758 \uc5d0\uc774\uc2a4\uc600\uc9c0\ub9cc \uc77c\ubcf8 \ub9ac\uadf8\uc5d0\uc11c\ub294 \ud604\uc9c0 \uc801\uc751 \uc2e4\ud328\uc640 \ub9ac\uadf8\uaca9\ucc28\ub85c \uc778\ud574 \ub69c\ub837\ud55c \uc131\uacfc\ub97c \uc62c\ub9ac\uc9c0 \ubabb\ud588\ub2e4. \uc77c\ubcf8 \ub9ac\uadf8\uc5d0\uc11c \uadf8\ub294 2\uc2dc\uc98c \ub3d9\uc548 3\uc2b9 2\ud328\uc758 \ucd08\ub77c\ud55c \uc131\uc801\uc744 \uac70\ub450\uc5c8\uace0, \ubd80\uc9c4\ud588\ub358 \uc8fc\uc694\ud55c \uc6d0\uc778\uc73c\ub85c\ub294 \uae09\uaca9\ud55c \uad6c\uc704 \uc800\ud558, \ud314\uafc8\uce58 \ud1b5\uc99d\uc774\uc5c8\ub2e4. \uadf8\ub807\uc9c0\ub9cc \uc774 \uc548\ud0c0\uae4c\uc6b4 \uacb0\uacfc\uc5d0\ub294 \uc5ec\ub7ec \ub17c\ub780\uc774 \uc788\ub294\ub370 \uc2e4\uc81c\ub85c \uc815\ubbfc\ucca0\uc758 \uc694\ubbf8\uc6b0\ub9ac \uc785\ub2e8 \ud6c4 \uccab \ub4f1\ud310\uc740 \uc57c\ucfe0\ub974\ud2b8\ub97c \uc0c1\ub300\ub85c 7\uc774\ub2dd 1\uc2e4\uc810 \uc2b9\uc73c\ub85c \ud638\ud22c\ud558\uc600\uc73c\ub098 \uacbd\uae30 \uc9c1\ud6c4 2\uad70 \ud1b5\ubcf4\ub97c \ubc1b\uace0, \ub2e4\uc2dc \uc62c\ub77c\uc628 1\uad70 \uacbd\uae30\uc5d0\uc11c \uc694\ucf54\ud558\ub9c8\ub97c \uc0c1\ub300\ub85c 9\uc774\ub2dd \ubb34\uc2e4\uc810\uc73c\ub85c \uc644\ubd09\uc2b9 \ud558\uc600\uc73c\ub098 \uc774\ud6c4\ub85c\ub3c4 \ub4f1\ud310\uae30\ud68c\ub294 \uc880\ucc98\ub7fc \uc8fc\uc5b4\uc9c0\uc9c0 \uc54a\uc558\ub2e4. \uadf8 \ud6c4 \uad6d\ub0b4 \ubcf5\uadc0 20\uc644\ubd09 \ub2ec\uc131 \uc778\ud130\ubdf0\uc5d0\uc11c \"\uc694\ubbf8\uc6b0\ub9ac\uc758 \uc2dc\uc2a4\ud15c\uc774 \ucc38 \uc774\uc0c1\ud588\ub2e4. \"\ub77c\uace0 \ud68c\uace0\ud55c \ubc14 \uc788\ub2e4.", "paragraph_answer": "2000\ub144 \uc77c\ubcf8 \ud504\ub85c \uc57c\uad6c \uc694\ubbf8\uc6b0\ub9ac \uc790\uc774\uc5b8\uce20 \uc5d0 \uc785\ub2e8\ud558\uc5ec 2\uc2dc\uc98c\uc744 \ubcf4\ub0b8\ub2e4. \uad6d\ub0b4\uc5d0\uc11c\ub294 \ub9ac\uadf8 \ucd5c\uace0 \uc218\uc900\uc758 \uc5d0\uc774\uc2a4\uc600\uc9c0\ub9cc \uc77c\ubcf8 \ub9ac\uadf8\uc5d0\uc11c\ub294 \ud604\uc9c0 \uc801\uc751 \uc2e4\ud328\uc640 \ub9ac\uadf8\uaca9\ucc28\ub85c \uc778\ud574 \ub69c\ub837\ud55c \uc131\uacfc\ub97c \uc62c\ub9ac\uc9c0 \ubabb\ud588\ub2e4. \uc77c\ubcf8 \ub9ac\uadf8\uc5d0\uc11c \uadf8\ub294 2\uc2dc\uc98c \ub3d9\uc548 3\uc2b9 2\ud328\uc758 \ucd08\ub77c\ud55c \uc131\uc801\uc744 \uac70\ub450\uc5c8\uace0, \ubd80\uc9c4\ud588\ub358 \uc8fc\uc694\ud55c \uc6d0\uc778\uc73c\ub85c\ub294 \uae09\uaca9\ud55c \uad6c\uc704 \uc800\ud558, \ud314\uafc8\uce58 \ud1b5\uc99d\uc774\uc5c8\ub2e4. \uadf8\ub807\uc9c0\ub9cc \uc774 \uc548\ud0c0\uae4c\uc6b4 \uacb0\uacfc\uc5d0\ub294 \uc5ec\ub7ec \ub17c\ub780\uc774 \uc788\ub294\ub370 \uc2e4\uc81c\ub85c \uc815\ubbfc\ucca0\uc758 \uc694\ubbf8\uc6b0\ub9ac \uc785\ub2e8 \ud6c4 \uccab \ub4f1\ud310\uc740 \uc57c\ucfe0\ub974\ud2b8\ub97c \uc0c1\ub300\ub85c 7\uc774\ub2dd 1\uc2e4\uc810 \uc2b9\uc73c\ub85c \ud638\ud22c\ud558\uc600\uc73c\ub098 \uacbd\uae30 \uc9c1\ud6c4 2\uad70 \ud1b5\ubcf4\ub97c \ubc1b\uace0, \ub2e4\uc2dc \uc62c\ub77c\uc628 1\uad70 \uacbd\uae30\uc5d0\uc11c \uc694\ucf54\ud558\ub9c8\ub97c \uc0c1\ub300\ub85c 9\uc774\ub2dd \ubb34\uc2e4\uc810\uc73c\ub85c \uc644\ubd09\uc2b9 \ud558\uc600\uc73c\ub098 \uc774\ud6c4\ub85c\ub3c4 \ub4f1\ud310\uae30\ud68c\ub294 \uc880\ucc98\ub7fc \uc8fc\uc5b4\uc9c0\uc9c0 \uc54a\uc558\ub2e4. \uadf8 \ud6c4 \uad6d\ub0b4 \ubcf5\uadc0 20\uc644\ubd09 \ub2ec\uc131 \uc778\ud130\ubdf0\uc5d0\uc11c \"\uc694\ubbf8\uc6b0\ub9ac\uc758 \uc2dc\uc2a4\ud15c\uc774 \ucc38 \uc774\uc0c1\ud588\ub2e4.\"\ub77c\uace0 \ud68c\uace0\ud55c \ubc14 \uc788\ub2e4.", "sentence_answer": "2000\ub144 \uc77c\ubcf8 \ud504\ub85c \uc57c\uad6c \uc694\ubbf8\uc6b0\ub9ac \uc790\uc774\uc5b8\uce20 \uc5d0 \uc785\ub2e8\ud558\uc5ec 2\uc2dc\uc98c\uc744 \ubcf4\ub0b8\ub2e4.", "paragraph_id": "6550099-2-0", "paragraph_question": "question: \uc815\ubbfc\ucca0\uc740 \uc77c\ubcf8\uc758 \uc5b4\ub290 \uad6c\ub2e8\uc5d0 \uc785\ub2e8\ud588\ub098?, context: 2000\ub144 \uc77c\ubcf8 \ud504\ub85c \uc57c\uad6c \uc694\ubbf8\uc6b0\ub9ac \uc790\uc774\uc5b8\uce20\uc5d0 \uc785\ub2e8\ud558\uc5ec 2\uc2dc\uc98c\uc744 \ubcf4\ub0b8\ub2e4. \uad6d\ub0b4\uc5d0\uc11c\ub294 \ub9ac\uadf8 \ucd5c\uace0 \uc218\uc900\uc758 \uc5d0\uc774\uc2a4\uc600\uc9c0\ub9cc \uc77c\ubcf8 \ub9ac\uadf8\uc5d0\uc11c\ub294 \ud604\uc9c0 \uc801\uc751 \uc2e4\ud328\uc640 \ub9ac\uadf8\uaca9\ucc28\ub85c \uc778\ud574 \ub69c\ub837\ud55c \uc131\uacfc\ub97c \uc62c\ub9ac\uc9c0 \ubabb\ud588\ub2e4. \uc77c\ubcf8 \ub9ac\uadf8\uc5d0\uc11c \uadf8\ub294 2\uc2dc\uc98c \ub3d9\uc548 3\uc2b9 2\ud328\uc758 \ucd08\ub77c\ud55c \uc131\uc801\uc744 \uac70\ub450\uc5c8\uace0, \ubd80\uc9c4\ud588\ub358 \uc8fc\uc694\ud55c \uc6d0\uc778\uc73c\ub85c\ub294 \uae09\uaca9\ud55c \uad6c\uc704 \uc800\ud558, \ud314\uafc8\uce58 \ud1b5\uc99d\uc774\uc5c8\ub2e4. \uadf8\ub807\uc9c0\ub9cc \uc774 \uc548\ud0c0\uae4c\uc6b4 \uacb0\uacfc\uc5d0\ub294 \uc5ec\ub7ec \ub17c\ub780\uc774 \uc788\ub294\ub370 \uc2e4\uc81c\ub85c \uc815\ubbfc\ucca0\uc758 \uc694\ubbf8\uc6b0\ub9ac \uc785\ub2e8 \ud6c4 \uccab \ub4f1\ud310\uc740 \uc57c\ucfe0\ub974\ud2b8\ub97c \uc0c1\ub300\ub85c 7\uc774\ub2dd 1\uc2e4\uc810 \uc2b9\uc73c\ub85c \ud638\ud22c\ud558\uc600\uc73c\ub098 \uacbd\uae30 \uc9c1\ud6c4 2\uad70 \ud1b5\ubcf4\ub97c \ubc1b\uace0, \ub2e4\uc2dc \uc62c\ub77c\uc628 1\uad70 \uacbd\uae30\uc5d0\uc11c \uc694\ucf54\ud558\ub9c8\ub97c \uc0c1\ub300\ub85c 9\uc774\ub2dd \ubb34\uc2e4\uc810\uc73c\ub85c \uc644\ubd09\uc2b9 \ud558\uc600\uc73c\ub098 \uc774\ud6c4\ub85c\ub3c4 \ub4f1\ud310\uae30\ud68c\ub294 \uc880\ucc98\ub7fc \uc8fc\uc5b4\uc9c0\uc9c0 \uc54a\uc558\ub2e4. \uadf8 \ud6c4 \uad6d\ub0b4 \ubcf5\uadc0 20\uc644\ubd09 \ub2ec\uc131 \uc778\ud130\ubdf0\uc5d0\uc11c \"\uc694\ubbf8\uc6b0\ub9ac\uc758 \uc2dc\uc2a4\ud15c\uc774 \ucc38 \uc774\uc0c1\ud588\ub2e4.\"\ub77c\uace0 \ud68c\uace0\ud55c \ubc14 \uc788\ub2e4."} {"question": "\ub9c8\uc2a4\uac00 \uc81c\uc774\uc2a8 \ub9c8\uc758 \ub370\ubdd4\uc2f1\uae00 \uc568\ubc94 \ud53c\ucc98\ub9c1\uc744 \ud55c \ub144\ub3c4\ub294?", "paragraph": "\uc194\ub85c \uc544\ud2f0\uc2a4\ud2b8\uac00 \ub418\uae30\uc804\uc5d0 \ub9c8\uc2a4\ub294 \uc54c\ub809\uc0b0\ub4dc\ub77c \ubc84\ud06c, \ud2b8\ub798\ube44 \ub9e4\ucf54\uc774, \uc560\ub364 \ub9ac\ubc14\uc778, \ube0c\ub79c\ub514 \ub178\uc6b0\ub4dc, \uc200 \ud0b9\uc2a4\ud134, \ud50c\ub85c \ub77c\uc774\ub2e4\ub85c\ubd80\ud130 \uc74c\uc545 \ud504\ub85c\ub4c0\uc11c \ubc0f \uc1a1\ub77c\uc774\ud130\ub85c \uc778\uc815\uc744 \ubc1b\uc558\ub2e4. \ub610\ud55c, \uc288\uac00\ubca0\uc774\ube44\uc2a4\uc758 \ud788\ud2b8 \uc2f1\uae00 \"Get Sexy\"\ub97c \uc791\uc5c5\ud588\uc73c\uba70, \uadf8\ub4e4\uc758 \uc74c\ubc18 Sweet 7\uc5d0\ub3c4 \ubc31\ubcf4\uceec\ub85c \ucc38\uc5ec\ud558\uc600\ub2e4. \ub9c8\uc2a4\uac00 \ucc98\uc74c\uc73c\ub85c \ub178\ub798\ub97c \ubc1c\ub9e4\ud55c \uac83\uc740 \ud30c \uc774\uc2a4\ud2b8 \ubb34\ube0c\uba3c\ud2b8\uc758 \ub178\ub798 \"3D\"\uc758 \ud53c\ucc98\ub9c1\uc73c\ub85c, 2\uc9d1 \uc74c\ubc18 Animal\uc5d0 \uc218\ub85d\ub41c \uace1\uc774\ub2e4. \ub9c8\uc2a4\ub294 \ub610\ud55c 2009\ub144 8\uc6d4\uc5d0 \ubc1c\ub9e4\ub41c \ubaa9\uc0ac\uc774\uc790 \ud799\ud569 \uc544\ud2f0\uc2a4\ud2b8 \uc81c\uc774\uc2a8 \ub9c8\uc758 \ub370\ubdd4 \uc2f1\uae00 \"Love\"\uc5d0\ub3c4 \ud53c\ucc98\ub9c1\uc744 \ud558\uc600\ub2e4. B.o.B\uc758 \"Nothin' on You\"\uc640 \ud2b8\ub798\ube44 \ub9e4\ucf54\uc774\uc758 \"Billionaire\"\uc5d0 \ucc38\uc5ec\ud55c \ub9c8\uc2a4\ub294 \uc774\ub54c\ubd80\ud130 \uc194\ub85c \uc544\ud2f0\uc2a4\ud2b8\ub85c\uc11c \ubcf8\uaca9\uc801\uc73c\ub85c \uc74c\uc545\uc801 \uba85\uc131\uc744 \ub5a8\ucce4\ub2e4. \uc774 \ub450\uace1\uc740 \uc804\uc138\uacc4\uc758 \ub9ce\uc740 \ub098\ub77c\uc5d0\uc11c \ud06c\uac8c \ud788\ud2b8\ub97c \ucce4\ub2e4. \uc774 \uc131\uacf5\uc5d0 \uc774\uc5b4 \ub9c8\uc2a4\ub294 \ub370\ubdd4 EP It's Better If You Don't Understand\ub97c 2010\ub144 5\uc6d4 11\uc77c \ubc1c\ub9e4\ud558\uc600\ub2e4. \uc774 EP\ub294 \ube4c\ubcf4\ub4dc 200\uc5d0\uc11c 99\uc704\ub97c \ud558\uc600\uc73c\uba70, \uc2f1\uae00 \"The Other Side\"\ub294 \uc2dc \ub85c \uadf8\ub9b0\uacfc B.o.B\uac00 \ud53c\ucc98\ub9c1\uc744 \ud558\uc600\ub2e4. \ub9c8\uc2a4\ub294 2010\ub144 8\uc6d4 \ubc1c\ub9e4\ub41c \uc2dc \ub85c \uadf8\ub9b0\uc758 \uc2f1\uae00 \"Fuck You\"\ub97c \uacf5\ub3d9\uc791\uc5c5\ud558\uae30\ub3c4 \ud558\uc600\ub2e4. 2010\ub144 9\uc6d4 12\uc77c 2010\ub144 MTV \ube44\ub514\uc624 \ubba4\uc9c1 \uc5b4\uc6cc\ub4dc\uc5d0\uc11c \ub9c8\uc2a4\ub294 B.o.B, \ud5e4\uc77c\ub9ac \uc70c\ub9ac\uc5c4\uc2a4\uc640 \ud568\uaed8 \"Nothin' on You\"\uc640 \"Airplanes\"\ub97c \ubd88\ub800\ub2e4.", "answer": "2009\ub144", "sentence": "\ub9c8\uc2a4\ub294 \ub610\ud55c 2009\ub144 8\uc6d4\uc5d0 \ubc1c\ub9e4\ub41c \ubaa9\uc0ac\uc774\uc790 \ud799\ud569 \uc544\ud2f0\uc2a4\ud2b8 \uc81c\uc774\uc2a8 \ub9c8\uc758 \ub370\ubdd4 \uc2f1\uae00 \"Love\"\uc5d0\ub3c4 \ud53c\ucc98\ub9c1\uc744 \ud558\uc600\ub2e4.", "paragraph_sentence": "\uc194\ub85c \uc544\ud2f0\uc2a4\ud2b8\uac00 \ub418\uae30\uc804\uc5d0 \ub9c8\uc2a4\ub294 \uc54c\ub809\uc0b0\ub4dc\ub77c \ubc84\ud06c, \ud2b8\ub798\ube44 \ub9e4\ucf54\uc774, \uc560\ub364 \ub9ac\ubc14\uc778, \ube0c\ub79c\ub514 \ub178\uc6b0\ub4dc, \uc200 \ud0b9\uc2a4\ud134, \ud50c\ub85c \ub77c\uc774\ub2e4\ub85c\ubd80\ud130 \uc74c\uc545 \ud504\ub85c\ub4c0\uc11c \ubc0f \uc1a1\ub77c\uc774\ud130\ub85c \uc778\uc815\uc744 \ubc1b\uc558\ub2e4. \ub610\ud55c, \uc288\uac00\ubca0\uc774\ube44\uc2a4\uc758 \ud788\ud2b8 \uc2f1\uae00 \"Get Sexy\"\ub97c \uc791\uc5c5\ud588\uc73c\uba70, \uadf8\ub4e4\uc758 \uc74c\ubc18 Sweet 7\uc5d0\ub3c4 \ubc31\ubcf4\uceec\ub85c \ucc38\uc5ec\ud558\uc600\ub2e4. \ub9c8\uc2a4\uac00 \ucc98\uc74c\uc73c\ub85c \ub178\ub798\ub97c \ubc1c\ub9e4\ud55c \uac83\uc740 \ud30c \uc774\uc2a4\ud2b8 \ubb34\ube0c\uba3c\ud2b8\uc758 \ub178\ub798 \"3D\"\uc758 \ud53c\ucc98\ub9c1\uc73c\ub85c, 2\uc9d1 \uc74c\ubc18 Animal\uc5d0 \uc218\ub85d\ub41c \uace1\uc774\ub2e4. \ub9c8\uc2a4\ub294 \ub610\ud55c 2009\ub144 8\uc6d4\uc5d0 \ubc1c\ub9e4\ub41c \ubaa9\uc0ac\uc774\uc790 \ud799\ud569 \uc544\ud2f0\uc2a4\ud2b8 \uc81c\uc774\uc2a8 \ub9c8\uc758 \ub370\ubdd4 \uc2f1\uae00 \"Love\"\uc5d0\ub3c4 \ud53c\ucc98\ub9c1\uc744 \ud558\uc600\ub2e4. B.o. B\uc758 \"Nothin' on You\"\uc640 \ud2b8\ub798\ube44 \ub9e4\ucf54\uc774\uc758 \"Billionaire\"\uc5d0 \ucc38\uc5ec\ud55c \ub9c8\uc2a4\ub294 \uc774\ub54c\ubd80\ud130 \uc194\ub85c \uc544\ud2f0\uc2a4\ud2b8\ub85c\uc11c \ubcf8\uaca9\uc801\uc73c\ub85c \uc74c\uc545\uc801 \uba85\uc131\uc744 \ub5a8\ucce4\ub2e4. \uc774 \ub450\uace1\uc740 \uc804\uc138\uacc4\uc758 \ub9ce\uc740 \ub098\ub77c\uc5d0\uc11c \ud06c\uac8c \ud788\ud2b8\ub97c \ucce4\ub2e4. \uc774 \uc131\uacf5\uc5d0 \uc774\uc5b4 \ub9c8\uc2a4\ub294 \ub370\ubdd4 EP It's Better If You Don't Understand\ub97c 2010\ub144 5\uc6d4 11\uc77c \ubc1c\ub9e4\ud558\uc600\ub2e4. \uc774 EP\ub294 \ube4c\ubcf4\ub4dc 200\uc5d0\uc11c 99\uc704\ub97c \ud558\uc600\uc73c\uba70, \uc2f1\uae00 \"The Other Side\"\ub294 \uc2dc \ub85c \uadf8\ub9b0\uacfc B.o. B\uac00 \ud53c\ucc98\ub9c1\uc744 \ud558\uc600\ub2e4. \ub9c8\uc2a4\ub294 2010\ub144 8\uc6d4 \ubc1c\ub9e4\ub41c \uc2dc \ub85c \uadf8\ub9b0\uc758 \uc2f1\uae00 \"Fuck You\"\ub97c \uacf5\ub3d9\uc791\uc5c5\ud558\uae30\ub3c4 \ud558\uc600\ub2e4. 2010\ub144 9\uc6d4 12\uc77c 2010\ub144 MTV \ube44\ub514\uc624 \ubba4\uc9c1 \uc5b4\uc6cc\ub4dc\uc5d0\uc11c \ub9c8\uc2a4\ub294 B.o. B, \ud5e4\uc77c\ub9ac \uc70c\ub9ac\uc5c4\uc2a4\uc640 \ud568\uaed8 \"Nothin' on You\"\uc640 \"Airplanes\"\ub97c \ubd88\ub800\ub2e4.", "paragraph_answer": "\uc194\ub85c \uc544\ud2f0\uc2a4\ud2b8\uac00 \ub418\uae30\uc804\uc5d0 \ub9c8\uc2a4\ub294 \uc54c\ub809\uc0b0\ub4dc\ub77c \ubc84\ud06c, \ud2b8\ub798\ube44 \ub9e4\ucf54\uc774, \uc560\ub364 \ub9ac\ubc14\uc778, \ube0c\ub79c\ub514 \ub178\uc6b0\ub4dc, \uc200 \ud0b9\uc2a4\ud134, \ud50c\ub85c \ub77c\uc774\ub2e4\ub85c\ubd80\ud130 \uc74c\uc545 \ud504\ub85c\ub4c0\uc11c \ubc0f \uc1a1\ub77c\uc774\ud130\ub85c \uc778\uc815\uc744 \ubc1b\uc558\ub2e4. \ub610\ud55c, \uc288\uac00\ubca0\uc774\ube44\uc2a4\uc758 \ud788\ud2b8 \uc2f1\uae00 \"Get Sexy\"\ub97c \uc791\uc5c5\ud588\uc73c\uba70, \uadf8\ub4e4\uc758 \uc74c\ubc18 Sweet 7\uc5d0\ub3c4 \ubc31\ubcf4\uceec\ub85c \ucc38\uc5ec\ud558\uc600\ub2e4. \ub9c8\uc2a4\uac00 \ucc98\uc74c\uc73c\ub85c \ub178\ub798\ub97c \ubc1c\ub9e4\ud55c \uac83\uc740 \ud30c \uc774\uc2a4\ud2b8 \ubb34\ube0c\uba3c\ud2b8\uc758 \ub178\ub798 \"3D\"\uc758 \ud53c\ucc98\ub9c1\uc73c\ub85c, 2\uc9d1 \uc74c\ubc18 Animal\uc5d0 \uc218\ub85d\ub41c \uace1\uc774\ub2e4. \ub9c8\uc2a4\ub294 \ub610\ud55c 2009\ub144 8\uc6d4\uc5d0 \ubc1c\ub9e4\ub41c \ubaa9\uc0ac\uc774\uc790 \ud799\ud569 \uc544\ud2f0\uc2a4\ud2b8 \uc81c\uc774\uc2a8 \ub9c8\uc758 \ub370\ubdd4 \uc2f1\uae00 \"Love\"\uc5d0\ub3c4 \ud53c\ucc98\ub9c1\uc744 \ud558\uc600\ub2e4. B.o.B\uc758 \"Nothin' on You\"\uc640 \ud2b8\ub798\ube44 \ub9e4\ucf54\uc774\uc758 \"Billionaire\"\uc5d0 \ucc38\uc5ec\ud55c \ub9c8\uc2a4\ub294 \uc774\ub54c\ubd80\ud130 \uc194\ub85c \uc544\ud2f0\uc2a4\ud2b8\ub85c\uc11c \ubcf8\uaca9\uc801\uc73c\ub85c \uc74c\uc545\uc801 \uba85\uc131\uc744 \ub5a8\ucce4\ub2e4. \uc774 \ub450\uace1\uc740 \uc804\uc138\uacc4\uc758 \ub9ce\uc740 \ub098\ub77c\uc5d0\uc11c \ud06c\uac8c \ud788\ud2b8\ub97c \ucce4\ub2e4. \uc774 \uc131\uacf5\uc5d0 \uc774\uc5b4 \ub9c8\uc2a4\ub294 \ub370\ubdd4 EP It's Better If You Don't Understand\ub97c 2010\ub144 5\uc6d4 11\uc77c \ubc1c\ub9e4\ud558\uc600\ub2e4. \uc774 EP\ub294 \ube4c\ubcf4\ub4dc 200\uc5d0\uc11c 99\uc704\ub97c \ud558\uc600\uc73c\uba70, \uc2f1\uae00 \"The Other Side\"\ub294 \uc2dc \ub85c \uadf8\ub9b0\uacfc B.o.B\uac00 \ud53c\ucc98\ub9c1\uc744 \ud558\uc600\ub2e4. \ub9c8\uc2a4\ub294 2010\ub144 8\uc6d4 \ubc1c\ub9e4\ub41c \uc2dc \ub85c \uadf8\ub9b0\uc758 \uc2f1\uae00 \"Fuck You\"\ub97c \uacf5\ub3d9\uc791\uc5c5\ud558\uae30\ub3c4 \ud558\uc600\ub2e4. 2010\ub144 9\uc6d4 12\uc77c 2010\ub144 MTV \ube44\ub514\uc624 \ubba4\uc9c1 \uc5b4\uc6cc\ub4dc\uc5d0\uc11c \ub9c8\uc2a4\ub294 B.o.B, \ud5e4\uc77c\ub9ac \uc70c\ub9ac\uc5c4\uc2a4\uc640 \ud568\uaed8 \"Nothin' on You\"\uc640 \"Airplanes\"\ub97c \ubd88\ub800\ub2e4.", "sentence_answer": "\ub9c8\uc2a4\ub294 \ub610\ud55c 2009\ub144 8\uc6d4\uc5d0 \ubc1c\ub9e4\ub41c \ubaa9\uc0ac\uc774\uc790 \ud799\ud569 \uc544\ud2f0\uc2a4\ud2b8 \uc81c\uc774\uc2a8 \ub9c8\uc758 \ub370\ubdd4 \uc2f1\uae00 \"Love\"\uc5d0\ub3c4 \ud53c\ucc98\ub9c1\uc744 \ud558\uc600\ub2e4.", "paragraph_id": "6535231-5-0", "paragraph_question": "question: \ub9c8\uc2a4\uac00 \uc81c\uc774\uc2a8 \ub9c8\uc758 \ub370\ubdd4\uc2f1\uae00 \uc568\ubc94 \ud53c\ucc98\ub9c1\uc744 \ud55c \ub144\ub3c4\ub294?, context: \uc194\ub85c \uc544\ud2f0\uc2a4\ud2b8\uac00 \ub418\uae30\uc804\uc5d0 \ub9c8\uc2a4\ub294 \uc54c\ub809\uc0b0\ub4dc\ub77c \ubc84\ud06c, \ud2b8\ub798\ube44 \ub9e4\ucf54\uc774, \uc560\ub364 \ub9ac\ubc14\uc778, \ube0c\ub79c\ub514 \ub178\uc6b0\ub4dc, \uc200 \ud0b9\uc2a4\ud134, \ud50c\ub85c \ub77c\uc774\ub2e4\ub85c\ubd80\ud130 \uc74c\uc545 \ud504\ub85c\ub4c0\uc11c \ubc0f \uc1a1\ub77c\uc774\ud130\ub85c \uc778\uc815\uc744 \ubc1b\uc558\ub2e4. \ub610\ud55c, \uc288\uac00\ubca0\uc774\ube44\uc2a4\uc758 \ud788\ud2b8 \uc2f1\uae00 \"Get Sexy\"\ub97c \uc791\uc5c5\ud588\uc73c\uba70, \uadf8\ub4e4\uc758 \uc74c\ubc18 Sweet 7\uc5d0\ub3c4 \ubc31\ubcf4\uceec\ub85c \ucc38\uc5ec\ud558\uc600\ub2e4. \ub9c8\uc2a4\uac00 \ucc98\uc74c\uc73c\ub85c \ub178\ub798\ub97c \ubc1c\ub9e4\ud55c \uac83\uc740 \ud30c \uc774\uc2a4\ud2b8 \ubb34\ube0c\uba3c\ud2b8\uc758 \ub178\ub798 \"3D\"\uc758 \ud53c\ucc98\ub9c1\uc73c\ub85c, 2\uc9d1 \uc74c\ubc18 Animal\uc5d0 \uc218\ub85d\ub41c \uace1\uc774\ub2e4. \ub9c8\uc2a4\ub294 \ub610\ud55c 2009\ub144 8\uc6d4\uc5d0 \ubc1c\ub9e4\ub41c \ubaa9\uc0ac\uc774\uc790 \ud799\ud569 \uc544\ud2f0\uc2a4\ud2b8 \uc81c\uc774\uc2a8 \ub9c8\uc758 \ub370\ubdd4 \uc2f1\uae00 \"Love\"\uc5d0\ub3c4 \ud53c\ucc98\ub9c1\uc744 \ud558\uc600\ub2e4. B.o.B\uc758 \"Nothin' on You\"\uc640 \ud2b8\ub798\ube44 \ub9e4\ucf54\uc774\uc758 \"Billionaire\"\uc5d0 \ucc38\uc5ec\ud55c \ub9c8\uc2a4\ub294 \uc774\ub54c\ubd80\ud130 \uc194\ub85c \uc544\ud2f0\uc2a4\ud2b8\ub85c\uc11c \ubcf8\uaca9\uc801\uc73c\ub85c \uc74c\uc545\uc801 \uba85\uc131\uc744 \ub5a8\ucce4\ub2e4. \uc774 \ub450\uace1\uc740 \uc804\uc138\uacc4\uc758 \ub9ce\uc740 \ub098\ub77c\uc5d0\uc11c \ud06c\uac8c \ud788\ud2b8\ub97c \ucce4\ub2e4. \uc774 \uc131\uacf5\uc5d0 \uc774\uc5b4 \ub9c8\uc2a4\ub294 \ub370\ubdd4 EP It's Better If You Don't Understand\ub97c 2010\ub144 5\uc6d4 11\uc77c \ubc1c\ub9e4\ud558\uc600\ub2e4. \uc774 EP\ub294 \ube4c\ubcf4\ub4dc 200\uc5d0\uc11c 99\uc704\ub97c \ud558\uc600\uc73c\uba70, \uc2f1\uae00 \"The Other Side\"\ub294 \uc2dc \ub85c \uadf8\ub9b0\uacfc B.o.B\uac00 \ud53c\ucc98\ub9c1\uc744 \ud558\uc600\ub2e4. \ub9c8\uc2a4\ub294 2010\ub144 8\uc6d4 \ubc1c\ub9e4\ub41c \uc2dc \ub85c \uadf8\ub9b0\uc758 \uc2f1\uae00 \"Fuck You\"\ub97c \uacf5\ub3d9\uc791\uc5c5\ud558\uae30\ub3c4 \ud558\uc600\ub2e4. 2010\ub144 9\uc6d4 12\uc77c 2010\ub144 MTV \ube44\ub514\uc624 \ubba4\uc9c1 \uc5b4\uc6cc\ub4dc\uc5d0\uc11c \ub9c8\uc2a4\ub294 B.o.B, \ud5e4\uc77c\ub9ac \uc70c\ub9ac\uc5c4\uc2a4\uc640 \ud568\uaed8 \"Nothin' on You\"\uc640 \"Airplanes\"\ub97c \ubd88\ub800\ub2e4."} {"question": "\uc77c\uc81c\uc758 \uc81c2\ucc28 \uc18c\uac1c\uacf5\uc9c0 \uc0ac\uc5c5\uc774 \ubbf8\uc644\uc131\uc73c\ub85c \ub05d\ub09c \uc774\uc720\ub294 \ubb34\uc5c7\uc778\uac00?", "paragraph": "1945\ub144 3\uc6d4 10\uc77c\uc5d0 \uc788\uc5c8\ub358 \ub3c4\ucfc4 \ub300\uacf5\uc2b5\uc740 \uc77c\ubcf8\uc5d0 \ucda9\uaca9\uc744 \uc8fc\uc5c8\ub2e4. \uc804\uc120\uc5d0\uc11c \uc77c\ubcf8\uc774 \ubd88\ub9ac\ud55c \uc0c1\ud669\uc774\uc5c8\uace0, \ubbf8\uad6d\uc5d0 \uc758\ud55c \ud3ed\uaca9\uc740 \uc55e\uc73c\ub85c\ub3c4 \uacc4\uc18d\ub420 \uac83\uc774 \uc790\uba85\ud588\uc73c\ubbc0\ub85c, \uc77c\ubcf8\uc758 \uac01 \uc9c0\uc5ed\uc5d0\uc11c\ub294 \ud3ed\uaca9\uc5d0 \ub300\ube44\ud558\uae30 \uc704\ud574 \uc544\ubb34\ub7f0 \uac74\ubb3c\ub3c4 \uc9d3\uc9c0 \uc54a\uace0 \uacf5\ud130\ub85c \ub0a8\uaca8\ub450\ub294 \uc18c\uac1c\uacf5\uc9c0 \uc870\uc131 \uc0ac\uc5c5\uc5d0 \ucc29\uc218\ud558\uc600\ub2e4. \uc774 \uc0ac\uc5c5\uc740 \ub2f9\uc2dc \uc2dd\ubbfc\uc9c0\uc778 \ud55c\ubc18\ub3c4\uc5d0\ub3c4 \uc2dc\ud589\ub418\uc5c8\ub294\ub370, 1945\ub144 4\uc6d4\ubd80\ud130 6\uc6d4\uae4c\uc9c0 \uacbd\uc131, \ubd80\uc0b0, \ud3c9\uc591, \ub300\uc804, \ub300\uad6c \ub4f1\uc758 \uc8fc\uc694 \ub3c4\uc2dc\uc5d0 \uac01\uac01 \uc18c\uac1c\uacf5\uc9c0 \ub610\ub294 \uc18c\uac1c\uacf5\uc9c0\ub300\ub77c\ub294 \uac83\uc744 \uc9c0\uc815\ud588\ub2e4. \uacbd\uc131\ubd80\ub0b4\uc758 \uc18c\uac1c\uacf5\uc9c0\ub300\ub294 \ubaa8\ub450 19\uacf3\uc774\uc5c8\uc73c\uba70, \uc81c1\ucc28 \uc18c\uac1c\uc791\uc5c5\uc740 5\uc6d4 11\uc77c\ubd80\ud130 \uc2dc\uc791\ub418\uc5b4 6\uc6d4 \ub9d0\uc5d0 \ub05d\ub0ac\ub2e4. \uadf8 \uc911 \ud55c \uacf3\uc774 \uc885\ubb18 \uc55e\uc5d0\uc11c \ud544\ub3d9\uae4c\uc9c0\uc758 \ub108\ube44 50 m, \uae38\uc774 1180 m\uc758 \ud604 \uc138\uc6b4\uc0c1\uac00 \uc9c0\ub300\uc600\ub2e4. \uace7\ubc14\ub85c \uc81c2\ucc28 \uc18c\uac1c\uc791\uc5c5\uc774 \uc2dc\uc791\ub418\uc5c8\uc73c\ub098 \uc77c\ubcf8\uc758 \ud328\uc804\uc73c\ub85c \uc18c\uac1c\uc791\uc5c5\uc740 \ubbf8\uc644\uc73c\ub85c \ub05d\ub0ac\ub2e4.", "answer": "\uc77c\ubcf8\uc758 \ud328\uc804", "sentence": "\uace7\ubc14\ub85c \uc81c2\ucc28 \uc18c\uac1c\uc791\uc5c5\uc774 \uc2dc\uc791\ub418\uc5c8\uc73c\ub098 \uc77c\ubcf8\uc758 \ud328\uc804 \uc73c\ub85c \uc18c\uac1c\uc791\uc5c5\uc740 \ubbf8\uc644\uc73c\ub85c \ub05d\ub0ac\ub2e4.", "paragraph_sentence": "1945\ub144 3\uc6d4 10\uc77c\uc5d0 \uc788\uc5c8\ub358 \ub3c4\ucfc4 \ub300\uacf5\uc2b5\uc740 \uc77c\ubcf8\uc5d0 \ucda9\uaca9\uc744 \uc8fc\uc5c8\ub2e4. \uc804\uc120\uc5d0\uc11c \uc77c\ubcf8\uc774 \ubd88\ub9ac\ud55c \uc0c1\ud669\uc774\uc5c8\uace0, \ubbf8\uad6d\uc5d0 \uc758\ud55c \ud3ed\uaca9\uc740 \uc55e\uc73c\ub85c\ub3c4 \uacc4\uc18d\ub420 \uac83\uc774 \uc790\uba85\ud588\uc73c\ubbc0\ub85c, \uc77c\ubcf8\uc758 \uac01 \uc9c0\uc5ed\uc5d0\uc11c\ub294 \ud3ed\uaca9\uc5d0 \ub300\ube44\ud558\uae30 \uc704\ud574 \uc544\ubb34\ub7f0 \uac74\ubb3c\ub3c4 \uc9d3\uc9c0 \uc54a\uace0 \uacf5\ud130\ub85c \ub0a8\uaca8\ub450\ub294 \uc18c\uac1c\uacf5\uc9c0 \uc870\uc131 \uc0ac\uc5c5\uc5d0 \ucc29\uc218\ud558\uc600\ub2e4. \uc774 \uc0ac\uc5c5\uc740 \ub2f9\uc2dc \uc2dd\ubbfc\uc9c0\uc778 \ud55c\ubc18\ub3c4\uc5d0\ub3c4 \uc2dc\ud589\ub418\uc5c8\ub294\ub370, 1945\ub144 4\uc6d4\ubd80\ud130 6\uc6d4\uae4c\uc9c0 \uacbd\uc131, \ubd80\uc0b0, \ud3c9\uc591, \ub300\uc804, \ub300\uad6c \ub4f1\uc758 \uc8fc\uc694 \ub3c4\uc2dc\uc5d0 \uac01\uac01 \uc18c\uac1c\uacf5\uc9c0 \ub610\ub294 \uc18c\uac1c\uacf5\uc9c0\ub300\ub77c\ub294 \uac83\uc744 \uc9c0\uc815\ud588\ub2e4. \uacbd\uc131\ubd80\ub0b4\uc758 \uc18c\uac1c\uacf5\uc9c0\ub300\ub294 \ubaa8\ub450 19\uacf3\uc774\uc5c8\uc73c\uba70, \uc81c1\ucc28 \uc18c\uac1c\uc791\uc5c5\uc740 5\uc6d4 11\uc77c\ubd80\ud130 \uc2dc\uc791\ub418\uc5b4 6\uc6d4 \ub9d0\uc5d0 \ub05d\ub0ac\ub2e4. \uadf8 \uc911 \ud55c \uacf3\uc774 \uc885\ubb18 \uc55e\uc5d0\uc11c \ud544\ub3d9\uae4c\uc9c0\uc758 \ub108\ube44 50 m, \uae38\uc774 1180 m\uc758 \ud604 \uc138\uc6b4\uc0c1\uac00 \uc9c0\ub300\uc600\ub2e4. \uace7\ubc14\ub85c \uc81c2\ucc28 \uc18c\uac1c\uc791\uc5c5\uc774 \uc2dc\uc791\ub418\uc5c8\uc73c\ub098 \uc77c\ubcf8\uc758 \ud328\uc804 \uc73c\ub85c \uc18c\uac1c\uc791\uc5c5\uc740 \ubbf8\uc644\uc73c\ub85c \ub05d\ub0ac\ub2e4. ", "paragraph_answer": "1945\ub144 3\uc6d4 10\uc77c\uc5d0 \uc788\uc5c8\ub358 \ub3c4\ucfc4 \ub300\uacf5\uc2b5\uc740 \uc77c\ubcf8\uc5d0 \ucda9\uaca9\uc744 \uc8fc\uc5c8\ub2e4. \uc804\uc120\uc5d0\uc11c \uc77c\ubcf8\uc774 \ubd88\ub9ac\ud55c \uc0c1\ud669\uc774\uc5c8\uace0, \ubbf8\uad6d\uc5d0 \uc758\ud55c \ud3ed\uaca9\uc740 \uc55e\uc73c\ub85c\ub3c4 \uacc4\uc18d\ub420 \uac83\uc774 \uc790\uba85\ud588\uc73c\ubbc0\ub85c, \uc77c\ubcf8\uc758 \uac01 \uc9c0\uc5ed\uc5d0\uc11c\ub294 \ud3ed\uaca9\uc5d0 \ub300\ube44\ud558\uae30 \uc704\ud574 \uc544\ubb34\ub7f0 \uac74\ubb3c\ub3c4 \uc9d3\uc9c0 \uc54a\uace0 \uacf5\ud130\ub85c \ub0a8\uaca8\ub450\ub294 \uc18c\uac1c\uacf5\uc9c0 \uc870\uc131 \uc0ac\uc5c5\uc5d0 \ucc29\uc218\ud558\uc600\ub2e4. \uc774 \uc0ac\uc5c5\uc740 \ub2f9\uc2dc \uc2dd\ubbfc\uc9c0\uc778 \ud55c\ubc18\ub3c4\uc5d0\ub3c4 \uc2dc\ud589\ub418\uc5c8\ub294\ub370, 1945\ub144 4\uc6d4\ubd80\ud130 6\uc6d4\uae4c\uc9c0 \uacbd\uc131, \ubd80\uc0b0, \ud3c9\uc591, \ub300\uc804, \ub300\uad6c \ub4f1\uc758 \uc8fc\uc694 \ub3c4\uc2dc\uc5d0 \uac01\uac01 \uc18c\uac1c\uacf5\uc9c0 \ub610\ub294 \uc18c\uac1c\uacf5\uc9c0\ub300\ub77c\ub294 \uac83\uc744 \uc9c0\uc815\ud588\ub2e4. \uacbd\uc131\ubd80\ub0b4\uc758 \uc18c\uac1c\uacf5\uc9c0\ub300\ub294 \ubaa8\ub450 19\uacf3\uc774\uc5c8\uc73c\uba70, \uc81c1\ucc28 \uc18c\uac1c\uc791\uc5c5\uc740 5\uc6d4 11\uc77c\ubd80\ud130 \uc2dc\uc791\ub418\uc5b4 6\uc6d4 \ub9d0\uc5d0 \ub05d\ub0ac\ub2e4. \uadf8 \uc911 \ud55c \uacf3\uc774 \uc885\ubb18 \uc55e\uc5d0\uc11c \ud544\ub3d9\uae4c\uc9c0\uc758 \ub108\ube44 50 m, \uae38\uc774 1180 m\uc758 \ud604 \uc138\uc6b4\uc0c1\uac00 \uc9c0\ub300\uc600\ub2e4. \uace7\ubc14\ub85c \uc81c2\ucc28 \uc18c\uac1c\uc791\uc5c5\uc774 \uc2dc\uc791\ub418\uc5c8\uc73c\ub098 \uc77c\ubcf8\uc758 \ud328\uc804 \uc73c\ub85c \uc18c\uac1c\uc791\uc5c5\uc740 \ubbf8\uc644\uc73c\ub85c \ub05d\ub0ac\ub2e4.", "sentence_answer": "\uace7\ubc14\ub85c \uc81c2\ucc28 \uc18c\uac1c\uc791\uc5c5\uc774 \uc2dc\uc791\ub418\uc5c8\uc73c\ub098 \uc77c\ubcf8\uc758 \ud328\uc804 \uc73c\ub85c \uc18c\uac1c\uc791\uc5c5\uc740 \ubbf8\uc644\uc73c\ub85c \ub05d\ub0ac\ub2e4.", "paragraph_id": "6532128-0-1", "paragraph_question": "question: \uc77c\uc81c\uc758 \uc81c2\ucc28 \uc18c\uac1c\uacf5\uc9c0 \uc0ac\uc5c5\uc774 \ubbf8\uc644\uc131\uc73c\ub85c \ub05d\ub09c \uc774\uc720\ub294 \ubb34\uc5c7\uc778\uac00?, context: 1945\ub144 3\uc6d4 10\uc77c\uc5d0 \uc788\uc5c8\ub358 \ub3c4\ucfc4 \ub300\uacf5\uc2b5\uc740 \uc77c\ubcf8\uc5d0 \ucda9\uaca9\uc744 \uc8fc\uc5c8\ub2e4. \uc804\uc120\uc5d0\uc11c \uc77c\ubcf8\uc774 \ubd88\ub9ac\ud55c \uc0c1\ud669\uc774\uc5c8\uace0, \ubbf8\uad6d\uc5d0 \uc758\ud55c \ud3ed\uaca9\uc740 \uc55e\uc73c\ub85c\ub3c4 \uacc4\uc18d\ub420 \uac83\uc774 \uc790\uba85\ud588\uc73c\ubbc0\ub85c, \uc77c\ubcf8\uc758 \uac01 \uc9c0\uc5ed\uc5d0\uc11c\ub294 \ud3ed\uaca9\uc5d0 \ub300\ube44\ud558\uae30 \uc704\ud574 \uc544\ubb34\ub7f0 \uac74\ubb3c\ub3c4 \uc9d3\uc9c0 \uc54a\uace0 \uacf5\ud130\ub85c \ub0a8\uaca8\ub450\ub294 \uc18c\uac1c\uacf5\uc9c0 \uc870\uc131 \uc0ac\uc5c5\uc5d0 \ucc29\uc218\ud558\uc600\ub2e4. \uc774 \uc0ac\uc5c5\uc740 \ub2f9\uc2dc \uc2dd\ubbfc\uc9c0\uc778 \ud55c\ubc18\ub3c4\uc5d0\ub3c4 \uc2dc\ud589\ub418\uc5c8\ub294\ub370, 1945\ub144 4\uc6d4\ubd80\ud130 6\uc6d4\uae4c\uc9c0 \uacbd\uc131, \ubd80\uc0b0, \ud3c9\uc591, \ub300\uc804, \ub300\uad6c \ub4f1\uc758 \uc8fc\uc694 \ub3c4\uc2dc\uc5d0 \uac01\uac01 \uc18c\uac1c\uacf5\uc9c0 \ub610\ub294 \uc18c\uac1c\uacf5\uc9c0\ub300\ub77c\ub294 \uac83\uc744 \uc9c0\uc815\ud588\ub2e4. \uacbd\uc131\ubd80\ub0b4\uc758 \uc18c\uac1c\uacf5\uc9c0\ub300\ub294 \ubaa8\ub450 19\uacf3\uc774\uc5c8\uc73c\uba70, \uc81c1\ucc28 \uc18c\uac1c\uc791\uc5c5\uc740 5\uc6d4 11\uc77c\ubd80\ud130 \uc2dc\uc791\ub418\uc5b4 6\uc6d4 \ub9d0\uc5d0 \ub05d\ub0ac\ub2e4. \uadf8 \uc911 \ud55c \uacf3\uc774 \uc885\ubb18 \uc55e\uc5d0\uc11c \ud544\ub3d9\uae4c\uc9c0\uc758 \ub108\ube44 50 m, \uae38\uc774 1180 m\uc758 \ud604 \uc138\uc6b4\uc0c1\uac00 \uc9c0\ub300\uc600\ub2e4. \uace7\ubc14\ub85c \uc81c2\ucc28 \uc18c\uac1c\uc791\uc5c5\uc774 \uc2dc\uc791\ub418\uc5c8\uc73c\ub098 \uc77c\ubcf8\uc758 \ud328\uc804\uc73c\ub85c \uc18c\uac1c\uc791\uc5c5\uc740 \ubbf8\uc644\uc73c\ub85c \ub05d\ub0ac\ub2e4."} {"question": "Locke\ub294 \ubaa9\ud45c\uc124\uc815\uc774\ub860\uc744 \ubc1c\uc804\uc2dc\ud0a4\uba74\uc11c \uc778\uac04 \ud589\ub3d9\uc774 \ubb34\uc5c7\uc5d0 \uc758\ud574 \uacb0\uc815\ub41c\ub2e4\uace0 \uc8fc\uc7a5\ud558\uc600\ub294\uac00?", "paragraph": "\ubaa9\ud45c\uc124\uc815\uc774\ub860\uc740 Edwin A. Locke\uc5d0 \uc758\ud574\uc11c \uc81c\uc2dc\ub41c \uc774\ud6c4 \ud604\uc7ac\uc5d0 \uc774\ub974\uae30\uae4c\uc9c0 \ub3d9\uae30\uc774\ub860 \uc911 \uac00\uc7a5 \ub9ce\uc774 \uc5f0\uad6c\ub418\uc5c8\uc73c\uba70 \uac00\uc7a5 \ud0c0\ub2f9\uc131\uc774 \uc788\ub294 \uac83\uc73c\ub85c \ubc1d\ud600\uc9c4 \uc774\ub860\uc73c\ub85c \ub2e4\ub978 \ub3d9\uae30\uc774\ub860\uacfc\ub294 \ub2ec\ub9ac \uc624\ub298\ub0a0\uc5d0\ub3c4 \uc9c0\uc18d\uc801\uc73c\ub85c \uc5f0\uad6c \ub17c\ubb38\uc774 \ubc1c\ud45c\ub418\uace0 \uc788\ub2e4. Locke\uac00 \uc8fc\uc7a5\ud55c \ubaa9\ud45c \uc124\uc815 \uc774\ub860\uc740 \uc778\uac04 \ud589\ub3d9\uc774 \ubcf8\ub2a5\uacfc \uc695\uad6c\uc5d0 \ub530\ub77c \uac00\uc7a5 \ucf8c\ub77d\uc801\uc778 \ubc29\ud5a5\uc73c\ub85c \ub3d9\uae30\ud654\ub41c\ub2e4\ub294 \uae30\uc874 \uc774\ub860\ub4e4\uc744 \ube44\ud310\ud558\uba74\uc11c \uc2dc\uc791\ub418\uc5c8\ub2e4. Locke\ub294 1960\ub144\ub300 \uc911\ubc18\uc5d0 \ubaa9\ud45c\uc124\uc815\uc774\ub860\uc744 \uc804\uac1c\ud558\uae30 \uc2dc\uc791\ud588\uace0 30\ub144 \ub3d9\uc548 \uc5f0\uad6c\ub97c \ud574\uc654\ub2e4. Locke\ub294 \uadf8\ub54c\ubd80\ud130 \ubaa9\ud45c\uac00 \uc778\uac04\uc758 \ud589\ub3d9\uc5d0 \ubbf8\uce58\ub294 \uc601\ud5a5\uc744 \uc5f0\uad6c\ud558\uae30 \uc2dc\uc791\ud588\ub2e4. \uadf8\ub294 1960\ub144\ub300\uc5d0 \uc790\uc2e0\uc758 \ubaa9\ud45c\uc124\uc815\uc774\ub860\uc744 \ubc1c\uc804\uc2dc\ud0a4\uace0 \uc7ac\uc815\uc758 \ud574\ub098\uac14\uace0, \uc774\ub54c \uc774 \uc8fc\uc81c\uc5d0 \ub300\ud574 \ucc98\uc74c\uc73c\ub85c \uae00\uc744 \uc37c\uace0 \uc774 \uae00\uc740 \ud655\uc2e4\ud55c \ubaa9\ud45c(clearly identified goals)\uc640 \ud589\ub3d9(performance) \uac04\uc758 \uae0d\uc815\uc801\uc778 \uc5f0\uad00\uc131\uc744 \ubcf4\uc5ec\uc8fc\uc5c8\ub2e4. Locke\ub294 \uadf8\ub807\uac8c \uc778\uac04 \ud589\ub3d9\uc774 \ubaa9\ud45c \ud639\uc740 \uc758\ub3c4(intention)\uc5d0 \uc758\ud574 \uacb0\uc815\ub41c\ub2e4\uace0 \uc8fc\uc7a5\ud588\ub2e4.", "answer": "\ubaa9\ud45c \ud639\uc740 \uc758\ub3c4", "sentence": "Locke\ub294 \uadf8\ub807\uac8c \uc778\uac04 \ud589\ub3d9\uc774 \ubaa9\ud45c \ud639\uc740 \uc758\ub3c4 (intention)", "paragraph_sentence": "\ubaa9\ud45c\uc124\uc815\uc774\ub860\uc740 Edwin A. Locke\uc5d0 \uc758\ud574\uc11c \uc81c\uc2dc\ub41c \uc774\ud6c4 \ud604\uc7ac\uc5d0 \uc774\ub974\uae30\uae4c\uc9c0 \ub3d9\uae30\uc774\ub860 \uc911 \uac00\uc7a5 \ub9ce\uc774 \uc5f0\uad6c\ub418\uc5c8\uc73c\uba70 \uac00\uc7a5 \ud0c0\ub2f9\uc131\uc774 \uc788\ub294 \uac83\uc73c\ub85c \ubc1d\ud600\uc9c4 \uc774\ub860\uc73c\ub85c \ub2e4\ub978 \ub3d9\uae30\uc774\ub860\uacfc\ub294 \ub2ec\ub9ac \uc624\ub298\ub0a0\uc5d0\ub3c4 \uc9c0\uc18d\uc801\uc73c\ub85c \uc5f0\uad6c \ub17c\ubb38\uc774 \ubc1c\ud45c\ub418\uace0 \uc788\ub2e4. Locke\uac00 \uc8fc\uc7a5\ud55c \ubaa9\ud45c \uc124\uc815 \uc774\ub860\uc740 \uc778\uac04 \ud589\ub3d9\uc774 \ubcf8\ub2a5\uacfc \uc695\uad6c\uc5d0 \ub530\ub77c \uac00\uc7a5 \ucf8c\ub77d\uc801\uc778 \ubc29\ud5a5\uc73c\ub85c \ub3d9\uae30\ud654\ub41c\ub2e4\ub294 \uae30\uc874 \uc774\ub860\ub4e4\uc744 \ube44\ud310\ud558\uba74\uc11c \uc2dc\uc791\ub418\uc5c8\ub2e4. Locke\ub294 1960\ub144\ub300 \uc911\ubc18\uc5d0 \ubaa9\ud45c\uc124\uc815\uc774\ub860\uc744 \uc804\uac1c\ud558\uae30 \uc2dc\uc791\ud588\uace0 30\ub144 \ub3d9\uc548 \uc5f0\uad6c\ub97c \ud574\uc654\ub2e4. Locke\ub294 \uadf8\ub54c\ubd80\ud130 \ubaa9\ud45c\uac00 \uc778\uac04\uc758 \ud589\ub3d9\uc5d0 \ubbf8\uce58\ub294 \uc601\ud5a5\uc744 \uc5f0\uad6c\ud558\uae30 \uc2dc\uc791\ud588\ub2e4. \uadf8\ub294 1960\ub144\ub300\uc5d0 \uc790\uc2e0\uc758 \ubaa9\ud45c\uc124\uc815\uc774\ub860\uc744 \ubc1c\uc804\uc2dc\ud0a4\uace0 \uc7ac\uc815\uc758 \ud574\ub098\uac14\uace0, \uc774\ub54c \uc774 \uc8fc\uc81c\uc5d0 \ub300\ud574 \ucc98\uc74c\uc73c\ub85c \uae00\uc744 \uc37c\uace0 \uc774 \uae00\uc740 \ud655\uc2e4\ud55c \ubaa9\ud45c(clearly identified goals)\uc640 \ud589\ub3d9(performance) \uac04\uc758 \uae0d\uc815\uc801\uc778 \uc5f0\uad00\uc131\uc744 \ubcf4\uc5ec\uc8fc\uc5c8\ub2e4. Locke\ub294 \uadf8\ub807\uac8c \uc778\uac04 \ud589\ub3d9\uc774 \ubaa9\ud45c \ud639\uc740 \uc758\ub3c4 (intention) \uc5d0 \uc758\ud574 \uacb0\uc815\ub41c\ub2e4\uace0 \uc8fc\uc7a5\ud588\ub2e4.", "paragraph_answer": "\ubaa9\ud45c\uc124\uc815\uc774\ub860\uc740 Edwin A. Locke\uc5d0 \uc758\ud574\uc11c \uc81c\uc2dc\ub41c \uc774\ud6c4 \ud604\uc7ac\uc5d0 \uc774\ub974\uae30\uae4c\uc9c0 \ub3d9\uae30\uc774\ub860 \uc911 \uac00\uc7a5 \ub9ce\uc774 \uc5f0\uad6c\ub418\uc5c8\uc73c\uba70 \uac00\uc7a5 \ud0c0\ub2f9\uc131\uc774 \uc788\ub294 \uac83\uc73c\ub85c \ubc1d\ud600\uc9c4 \uc774\ub860\uc73c\ub85c \ub2e4\ub978 \ub3d9\uae30\uc774\ub860\uacfc\ub294 \ub2ec\ub9ac \uc624\ub298\ub0a0\uc5d0\ub3c4 \uc9c0\uc18d\uc801\uc73c\ub85c \uc5f0\uad6c \ub17c\ubb38\uc774 \ubc1c\ud45c\ub418\uace0 \uc788\ub2e4. Locke\uac00 \uc8fc\uc7a5\ud55c \ubaa9\ud45c \uc124\uc815 \uc774\ub860\uc740 \uc778\uac04 \ud589\ub3d9\uc774 \ubcf8\ub2a5\uacfc \uc695\uad6c\uc5d0 \ub530\ub77c \uac00\uc7a5 \ucf8c\ub77d\uc801\uc778 \ubc29\ud5a5\uc73c\ub85c \ub3d9\uae30\ud654\ub41c\ub2e4\ub294 \uae30\uc874 \uc774\ub860\ub4e4\uc744 \ube44\ud310\ud558\uba74\uc11c \uc2dc\uc791\ub418\uc5c8\ub2e4. Locke\ub294 1960\ub144\ub300 \uc911\ubc18\uc5d0 \ubaa9\ud45c\uc124\uc815\uc774\ub860\uc744 \uc804\uac1c\ud558\uae30 \uc2dc\uc791\ud588\uace0 30\ub144 \ub3d9\uc548 \uc5f0\uad6c\ub97c \ud574\uc654\ub2e4. Locke\ub294 \uadf8\ub54c\ubd80\ud130 \ubaa9\ud45c\uac00 \uc778\uac04\uc758 \ud589\ub3d9\uc5d0 \ubbf8\uce58\ub294 \uc601\ud5a5\uc744 \uc5f0\uad6c\ud558\uae30 \uc2dc\uc791\ud588\ub2e4. \uadf8\ub294 1960\ub144\ub300\uc5d0 \uc790\uc2e0\uc758 \ubaa9\ud45c\uc124\uc815\uc774\ub860\uc744 \ubc1c\uc804\uc2dc\ud0a4\uace0 \uc7ac\uc815\uc758 \ud574\ub098\uac14\uace0, \uc774\ub54c \uc774 \uc8fc\uc81c\uc5d0 \ub300\ud574 \ucc98\uc74c\uc73c\ub85c \uae00\uc744 \uc37c\uace0 \uc774 \uae00\uc740 \ud655\uc2e4\ud55c \ubaa9\ud45c(clearly identified goals)\uc640 \ud589\ub3d9(performance) \uac04\uc758 \uae0d\uc815\uc801\uc778 \uc5f0\uad00\uc131\uc744 \ubcf4\uc5ec\uc8fc\uc5c8\ub2e4. Locke\ub294 \uadf8\ub807\uac8c \uc778\uac04 \ud589\ub3d9\uc774 \ubaa9\ud45c \ud639\uc740 \uc758\ub3c4 (intention)\uc5d0 \uc758\ud574 \uacb0\uc815\ub41c\ub2e4\uace0 \uc8fc\uc7a5\ud588\ub2e4.", "sentence_answer": "Locke\ub294 \uadf8\ub807\uac8c \uc778\uac04 \ud589\ub3d9\uc774 \ubaa9\ud45c \ud639\uc740 \uc758\ub3c4 (intention)", "paragraph_id": "6504583-1-1", "paragraph_question": "question: Locke\ub294 \ubaa9\ud45c\uc124\uc815\uc774\ub860\uc744 \ubc1c\uc804\uc2dc\ud0a4\uba74\uc11c \uc778\uac04 \ud589\ub3d9\uc774 \ubb34\uc5c7\uc5d0 \uc758\ud574 \uacb0\uc815\ub41c\ub2e4\uace0 \uc8fc\uc7a5\ud558\uc600\ub294\uac00?, context: \ubaa9\ud45c\uc124\uc815\uc774\ub860\uc740 Edwin A. Locke\uc5d0 \uc758\ud574\uc11c \uc81c\uc2dc\ub41c \uc774\ud6c4 \ud604\uc7ac\uc5d0 \uc774\ub974\uae30\uae4c\uc9c0 \ub3d9\uae30\uc774\ub860 \uc911 \uac00\uc7a5 \ub9ce\uc774 \uc5f0\uad6c\ub418\uc5c8\uc73c\uba70 \uac00\uc7a5 \ud0c0\ub2f9\uc131\uc774 \uc788\ub294 \uac83\uc73c\ub85c \ubc1d\ud600\uc9c4 \uc774\ub860\uc73c\ub85c \ub2e4\ub978 \ub3d9\uae30\uc774\ub860\uacfc\ub294 \ub2ec\ub9ac \uc624\ub298\ub0a0\uc5d0\ub3c4 \uc9c0\uc18d\uc801\uc73c\ub85c \uc5f0\uad6c \ub17c\ubb38\uc774 \ubc1c\ud45c\ub418\uace0 \uc788\ub2e4. Locke\uac00 \uc8fc\uc7a5\ud55c \ubaa9\ud45c \uc124\uc815 \uc774\ub860\uc740 \uc778\uac04 \ud589\ub3d9\uc774 \ubcf8\ub2a5\uacfc \uc695\uad6c\uc5d0 \ub530\ub77c \uac00\uc7a5 \ucf8c\ub77d\uc801\uc778 \ubc29\ud5a5\uc73c\ub85c \ub3d9\uae30\ud654\ub41c\ub2e4\ub294 \uae30\uc874 \uc774\ub860\ub4e4\uc744 \ube44\ud310\ud558\uba74\uc11c \uc2dc\uc791\ub418\uc5c8\ub2e4. Locke\ub294 1960\ub144\ub300 \uc911\ubc18\uc5d0 \ubaa9\ud45c\uc124\uc815\uc774\ub860\uc744 \uc804\uac1c\ud558\uae30 \uc2dc\uc791\ud588\uace0 30\ub144 \ub3d9\uc548 \uc5f0\uad6c\ub97c \ud574\uc654\ub2e4. Locke\ub294 \uadf8\ub54c\ubd80\ud130 \ubaa9\ud45c\uac00 \uc778\uac04\uc758 \ud589\ub3d9\uc5d0 \ubbf8\uce58\ub294 \uc601\ud5a5\uc744 \uc5f0\uad6c\ud558\uae30 \uc2dc\uc791\ud588\ub2e4. \uadf8\ub294 1960\ub144\ub300\uc5d0 \uc790\uc2e0\uc758 \ubaa9\ud45c\uc124\uc815\uc774\ub860\uc744 \ubc1c\uc804\uc2dc\ud0a4\uace0 \uc7ac\uc815\uc758 \ud574\ub098\uac14\uace0, \uc774\ub54c \uc774 \uc8fc\uc81c\uc5d0 \ub300\ud574 \ucc98\uc74c\uc73c\ub85c \uae00\uc744 \uc37c\uace0 \uc774 \uae00\uc740 \ud655\uc2e4\ud55c \ubaa9\ud45c(clearly identified goals)\uc640 \ud589\ub3d9(performance) \uac04\uc758 \uae0d\uc815\uc801\uc778 \uc5f0\uad00\uc131\uc744 \ubcf4\uc5ec\uc8fc\uc5c8\ub2e4. Locke\ub294 \uadf8\ub807\uac8c \uc778\uac04 \ud589\ub3d9\uc774 \ubaa9\ud45c \ud639\uc740 \uc758\ub3c4(intention)\uc5d0 \uc758\ud574 \uacb0\uc815\ub41c\ub2e4\uace0 \uc8fc\uc7a5\ud588\ub2e4."} {"question": "\ubaa9\uc0ac\ub294 \uac1c\uc2e0\uad50\uc640 \uac00\ud1a8\ub9ad\uc744 \ud654\ud574\uc2dc\ud0a4\ub824\ub294 \uac83\uc740 \uc608\uc218\uc640 \ubb34\uc5c7\uc744 \ud654\ud574\uc2dc\ud0a4\ub824\ub294 \ubc1c\uc0c1\uc774\ub77c\uace0 \ud588\ub294\uac00?", "paragraph": "\ucf00\ud50c\ub7ec\ub294 1630\ub144 10\uc6d4 8\uc77c\uc5d0 \ub2e4\uc2dc \uc5ec\ud589\uc744 \ub5a0\ub0ac\ub2e4. \ub9b0\uce20\uc5d0 \ub4e4\ub7ec\uc11c \ub3c8 \ub3c5\ucd09\uc744 \ud558\uace0, \ub77c\uc774\ud504\uce58\ud788 \ub3c4\uc11c\uc804\uc5d0 \ub4e4\ub7ec \uc7ac\uace0 \ub3c4\uc11c\ub97c \ucc98\ub9ac\ud558\uace0, \ub808\uac90\uc2a4\ubd80\ub974\ud06c\uc5d0\uc11c \uc5f4\ub9b0 1630\ub144 7\uc120\uc81c\ud6c4 \ud68c\uc758\uc5d0 \uc9c1\uc811 \ucc38\uc11d\ud574 \uc815\uce58\uc801 \uc138\ud0dc\ub97c \uc9c0\ucf1c\ubcf4\ub294 \uac83\uc774 \uc5ec\ud589\uc758 \ubaa9\uc801\uc774\uc5c8\ub2e4. \ucf00\ud50c\ub7ec\ub294 \ub9b0\uce20, \ub77c\uc774\ud504\uce58\ud788, \ub258\ub978\ubca0\ub974\ud06c\ub97c \uac70\uccd0 11\uc6d4 2\uc77c\uc5d0 \ub808\uac90\uc2a4\ubd80\ub974\ud06c\uc5d0 \ub3c4\ucc29\ud558\uc600\uc9c0\ub9cc, \ucc28\uac00\uc6b4 \uac00\uc744 \ubc14\ub78c\uc744 \ub9de\uc73c\uba70 \uc5ec\ud589\ud55c \uac83\uc774 \ud654\uadfc\uc774 \ub418\uc5b4 \ubcd1\uc774 \ub4e4\uc5c8\ub2e4. \ucc98\uc74c\uc5d0\ub294 \uac00\ubcbc\uc6b4 \ubcd1\uc778 \uc904 \uc54c\uc558\uc73c\ub098, \uace0\uc5f4\uc774 \ub36e\uce58\uace0 \uc12c\ub9dd \uc0c1\ud0dc\uc5d0 \ube60\uc9c0\ub294 \ub4f1 \uc0c1\ud0dc\uac00 \uc545\ud654\ub418\uc5c8\ub2e4. \uc8fd\uae30 \uc9c1\uc804, \ucf00\ud50c\ub7ec\ub294 \uacc1\uc5d0 \uc788\ub358 \ubaa9\uc0ac\uc5d0\uac8c \uc790\uc2e0\uc740 \uac1c\uc2e0\uad50\uc640 \uac00\ud1a8\ub9ad\uc744 \ud654\ud574\uc2dc\ud0a4\ub824\uace0 \ucd5c\uc120\uc744 \ub2e4\ud588\ub2e4\ub294 \ub73b\uc744 \ubc1d\ud614\ub2e4. \uadf8\ub7ec\uc790 \ubaa9\uc0ac\ub294 \uadf8\uac83\uc740 \uc608\uc218\uc640 \uc0ac\ud0c4\uc744 \ud654\ud574\uc2dc\ud0a4\ub824\ub294 \ubc1c\uc0c1\uc774\ub77c\uace0 \ub300\uafb8\ud588\ub2e4..", "answer": "\uc0ac\ud0c4", "sentence": "\uadf8\ub7ec\uc790 \ubaa9\uc0ac\ub294 \uadf8\uac83\uc740 \uc608\uc218\uc640 \uc0ac\ud0c4 \uc744 \ud654\ud574\uc2dc\ud0a4\ub824\ub294 \ubc1c\uc0c1\uc774\ub77c\uace0 \ub300\uafb8\ud588\ub2e4", "paragraph_sentence": "\ucf00\ud50c\ub7ec\ub294 1630\ub144 10\uc6d4 8\uc77c\uc5d0 \ub2e4\uc2dc \uc5ec\ud589\uc744 \ub5a0\ub0ac\ub2e4. \ub9b0\uce20\uc5d0 \ub4e4\ub7ec\uc11c \ub3c8 \ub3c5\ucd09\uc744 \ud558\uace0, \ub77c\uc774\ud504\uce58\ud788 \ub3c4\uc11c\uc804\uc5d0 \ub4e4\ub7ec \uc7ac\uace0 \ub3c4\uc11c\ub97c \ucc98\ub9ac\ud558\uace0, \ub808\uac90\uc2a4\ubd80\ub974\ud06c\uc5d0\uc11c \uc5f4\ub9b0 1630\ub144 7\uc120\uc81c\ud6c4 \ud68c\uc758\uc5d0 \uc9c1\uc811 \ucc38\uc11d\ud574 \uc815\uce58\uc801 \uc138\ud0dc\ub97c \uc9c0\ucf1c\ubcf4\ub294 \uac83\uc774 \uc5ec\ud589\uc758 \ubaa9\uc801\uc774\uc5c8\ub2e4. \ucf00\ud50c\ub7ec\ub294 \ub9b0\uce20, \ub77c\uc774\ud504\uce58\ud788, \ub258\ub978\ubca0\ub974\ud06c\ub97c \uac70\uccd0 11\uc6d4 2\uc77c\uc5d0 \ub808\uac90\uc2a4\ubd80\ub974\ud06c\uc5d0 \ub3c4\ucc29\ud558\uc600\uc9c0\ub9cc, \ucc28\uac00\uc6b4 \uac00\uc744 \ubc14\ub78c\uc744 \ub9de\uc73c\uba70 \uc5ec\ud589\ud55c \uac83\uc774 \ud654\uadfc\uc774 \ub418\uc5b4 \ubcd1\uc774 \ub4e4\uc5c8\ub2e4. \ucc98\uc74c\uc5d0\ub294 \uac00\ubcbc\uc6b4 \ubcd1\uc778 \uc904 \uc54c\uc558\uc73c\ub098, \uace0\uc5f4\uc774 \ub36e\uce58\uace0 \uc12c\ub9dd \uc0c1\ud0dc\uc5d0 \ube60\uc9c0\ub294 \ub4f1 \uc0c1\ud0dc\uac00 \uc545\ud654\ub418\uc5c8\ub2e4. \uc8fd\uae30 \uc9c1\uc804, \ucf00\ud50c\ub7ec\ub294 \uacc1\uc5d0 \uc788\ub358 \ubaa9\uc0ac\uc5d0\uac8c \uc790\uc2e0\uc740 \uac1c\uc2e0\uad50\uc640 \uac00\ud1a8\ub9ad\uc744 \ud654\ud574\uc2dc\ud0a4\ub824\uace0 \ucd5c\uc120\uc744 \ub2e4\ud588\ub2e4\ub294 \ub73b\uc744 \ubc1d\ud614\ub2e4. \uadf8\ub7ec\uc790 \ubaa9\uc0ac\ub294 \uadf8\uac83\uc740 \uc608\uc218\uc640 \uc0ac\ud0c4 \uc744 \ud654\ud574\uc2dc\ud0a4\ub824\ub294 \ubc1c\uc0c1\uc774\ub77c\uace0 \ub300\uafb8\ud588\ub2e4 ..", "paragraph_answer": "\ucf00\ud50c\ub7ec\ub294 1630\ub144 10\uc6d4 8\uc77c\uc5d0 \ub2e4\uc2dc \uc5ec\ud589\uc744 \ub5a0\ub0ac\ub2e4. \ub9b0\uce20\uc5d0 \ub4e4\ub7ec\uc11c \ub3c8 \ub3c5\ucd09\uc744 \ud558\uace0, \ub77c\uc774\ud504\uce58\ud788 \ub3c4\uc11c\uc804\uc5d0 \ub4e4\ub7ec \uc7ac\uace0 \ub3c4\uc11c\ub97c \ucc98\ub9ac\ud558\uace0, \ub808\uac90\uc2a4\ubd80\ub974\ud06c\uc5d0\uc11c \uc5f4\ub9b0 1630\ub144 7\uc120\uc81c\ud6c4 \ud68c\uc758\uc5d0 \uc9c1\uc811 \ucc38\uc11d\ud574 \uc815\uce58\uc801 \uc138\ud0dc\ub97c \uc9c0\ucf1c\ubcf4\ub294 \uac83\uc774 \uc5ec\ud589\uc758 \ubaa9\uc801\uc774\uc5c8\ub2e4. \ucf00\ud50c\ub7ec\ub294 \ub9b0\uce20, \ub77c\uc774\ud504\uce58\ud788, \ub258\ub978\ubca0\ub974\ud06c\ub97c \uac70\uccd0 11\uc6d4 2\uc77c\uc5d0 \ub808\uac90\uc2a4\ubd80\ub974\ud06c\uc5d0 \ub3c4\ucc29\ud558\uc600\uc9c0\ub9cc, \ucc28\uac00\uc6b4 \uac00\uc744 \ubc14\ub78c\uc744 \ub9de\uc73c\uba70 \uc5ec\ud589\ud55c \uac83\uc774 \ud654\uadfc\uc774 \ub418\uc5b4 \ubcd1\uc774 \ub4e4\uc5c8\ub2e4. \ucc98\uc74c\uc5d0\ub294 \uac00\ubcbc\uc6b4 \ubcd1\uc778 \uc904 \uc54c\uc558\uc73c\ub098, \uace0\uc5f4\uc774 \ub36e\uce58\uace0 \uc12c\ub9dd \uc0c1\ud0dc\uc5d0 \ube60\uc9c0\ub294 \ub4f1 \uc0c1\ud0dc\uac00 \uc545\ud654\ub418\uc5c8\ub2e4. \uc8fd\uae30 \uc9c1\uc804, \ucf00\ud50c\ub7ec\ub294 \uacc1\uc5d0 \uc788\ub358 \ubaa9\uc0ac\uc5d0\uac8c \uc790\uc2e0\uc740 \uac1c\uc2e0\uad50\uc640 \uac00\ud1a8\ub9ad\uc744 \ud654\ud574\uc2dc\ud0a4\ub824\uace0 \ucd5c\uc120\uc744 \ub2e4\ud588\ub2e4\ub294 \ub73b\uc744 \ubc1d\ud614\ub2e4. \uadf8\ub7ec\uc790 \ubaa9\uc0ac\ub294 \uadf8\uac83\uc740 \uc608\uc218\uc640 \uc0ac\ud0c4 \uc744 \ud654\ud574\uc2dc\ud0a4\ub824\ub294 \ubc1c\uc0c1\uc774\ub77c\uace0 \ub300\uafb8\ud588\ub2e4..", "sentence_answer": "\uadf8\ub7ec\uc790 \ubaa9\uc0ac\ub294 \uadf8\uac83\uc740 \uc608\uc218\uc640 \uc0ac\ud0c4 \uc744 \ud654\ud574\uc2dc\ud0a4\ub824\ub294 \ubc1c\uc0c1\uc774\ub77c\uace0 \ub300\uafb8\ud588\ub2e4", "paragraph_id": "6541867-32-1", "paragraph_question": "question: \ubaa9\uc0ac\ub294 \uac1c\uc2e0\uad50\uc640 \uac00\ud1a8\ub9ad\uc744 \ud654\ud574\uc2dc\ud0a4\ub824\ub294 \uac83\uc740 \uc608\uc218\uc640 \ubb34\uc5c7\uc744 \ud654\ud574\uc2dc\ud0a4\ub824\ub294 \ubc1c\uc0c1\uc774\ub77c\uace0 \ud588\ub294\uac00?, context: \ucf00\ud50c\ub7ec\ub294 1630\ub144 10\uc6d4 8\uc77c\uc5d0 \ub2e4\uc2dc \uc5ec\ud589\uc744 \ub5a0\ub0ac\ub2e4. \ub9b0\uce20\uc5d0 \ub4e4\ub7ec\uc11c \ub3c8 \ub3c5\ucd09\uc744 \ud558\uace0, \ub77c\uc774\ud504\uce58\ud788 \ub3c4\uc11c\uc804\uc5d0 \ub4e4\ub7ec \uc7ac\uace0 \ub3c4\uc11c\ub97c \ucc98\ub9ac\ud558\uace0, \ub808\uac90\uc2a4\ubd80\ub974\ud06c\uc5d0\uc11c \uc5f4\ub9b0 1630\ub144 7\uc120\uc81c\ud6c4 \ud68c\uc758\uc5d0 \uc9c1\uc811 \ucc38\uc11d\ud574 \uc815\uce58\uc801 \uc138\ud0dc\ub97c \uc9c0\ucf1c\ubcf4\ub294 \uac83\uc774 \uc5ec\ud589\uc758 \ubaa9\uc801\uc774\uc5c8\ub2e4. \ucf00\ud50c\ub7ec\ub294 \ub9b0\uce20, \ub77c\uc774\ud504\uce58\ud788, \ub258\ub978\ubca0\ub974\ud06c\ub97c \uac70\uccd0 11\uc6d4 2\uc77c\uc5d0 \ub808\uac90\uc2a4\ubd80\ub974\ud06c\uc5d0 \ub3c4\ucc29\ud558\uc600\uc9c0\ub9cc, \ucc28\uac00\uc6b4 \uac00\uc744 \ubc14\ub78c\uc744 \ub9de\uc73c\uba70 \uc5ec\ud589\ud55c \uac83\uc774 \ud654\uadfc\uc774 \ub418\uc5b4 \ubcd1\uc774 \ub4e4\uc5c8\ub2e4. \ucc98\uc74c\uc5d0\ub294 \uac00\ubcbc\uc6b4 \ubcd1\uc778 \uc904 \uc54c\uc558\uc73c\ub098, \uace0\uc5f4\uc774 \ub36e\uce58\uace0 \uc12c\ub9dd \uc0c1\ud0dc\uc5d0 \ube60\uc9c0\ub294 \ub4f1 \uc0c1\ud0dc\uac00 \uc545\ud654\ub418\uc5c8\ub2e4. \uc8fd\uae30 \uc9c1\uc804, \ucf00\ud50c\ub7ec\ub294 \uacc1\uc5d0 \uc788\ub358 \ubaa9\uc0ac\uc5d0\uac8c \uc790\uc2e0\uc740 \uac1c\uc2e0\uad50\uc640 \uac00\ud1a8\ub9ad\uc744 \ud654\ud574\uc2dc\ud0a4\ub824\uace0 \ucd5c\uc120\uc744 \ub2e4\ud588\ub2e4\ub294 \ub73b\uc744 \ubc1d\ud614\ub2e4. \uadf8\ub7ec\uc790 \ubaa9\uc0ac\ub294 \uadf8\uac83\uc740 \uc608\uc218\uc640 \uc0ac\ud0c4\uc744 \ud654\ud574\uc2dc\ud0a4\ub824\ub294 \ubc1c\uc0c1\uc774\ub77c\uace0 \ub300\uafb8\ud588\ub2e4.."} {"question": "\uc2a4\ud0c0\uc27d \uc5d1\uc2a4\ub294 \uc5b8\uc81c\ubd80\ud130 mc\uc778\uc800\uc2a4\ub514\uc2a4\uc640 \ud568\uaed8 \ub7a9 \ub4c0\uc624\ub85c \uacf5\uc5f0\uc744 \ud558\uace0 \uc788\ub294\uac00", "paragraph": "\uacfc\uac70 \uc18c\uc6b8 \ucef4\ud37c\ub2c8 \uc18c\uc18d\uc73c\ub85c \uc18c\uc6b8 \ucef4\ud37c\ub2c8 \ud574\uccb4 \ud6c4 \ud604\uc7ac\uc758 \uc18c\uc18d\uc0ac\ub294 \uc2a4\ud0c0\uc27d \uc5d1\uc2a4\uc774\ub2e4. Mad Clown vs Crucial Star (\ub9e4\ub4dc \ud074\ub77c\uc6b4 vs \ud06c\ub8e8\uc15c \uc2a4\ud0c0)\ub77c\ub294 \ud504\ub85c\uc81d\ud2b8 \uadf8\ub8f9\uc73c\ub85c \ud06c\ub8e8\uc15c \uc2a4\ud0c0\uc640 \ud568\uaed8 \ud65c\ub3d9\ud558\uae30\ub3c4 \ud558\uc600\uc73c\uba70, 2013\ub144\ubd80\ud130\ub294 MC\uc778 \uc800\uc2a4\ub514\uc2a4\uc640 \ud300\uc744 \uc774\ub8e8\uc5b4 \ub7a9 \ub4c0\uc624 \ucee4\uba3c\ucf5c\ub4dc\ub85c \ud65c\ub3d9\ud558\uace0 \uc788\ub2e4. \ub610\ud55c Mnet \u300a\uc1fc\ubbf8\ub354\uba38\ub2c8 2\u300b\uc5d0\uc11c \ucc38\uac00\uc790\ub85c \ucc38\uac00\ud558\uc5ec \ud070 \ud654\uc81c\ub97c \ubaa8\uc558\uc73c\uba70, \u300a\uc1fc\ubbf8\ub354\uba38\ub2c8 5\u300b\uc5d0\uc11c\ub294 \uae38 & \ub9e4\ub4dc \ud074\ub77c\uc6b4 \ud300\uc73c\ub85c \ud504\ub85c\ub4c0\uc11c\ub85c \ucd9c\uc5f0\ud558\uc600\ub2e4., \uc7ac\ubc1c\ub9e4 \ubb3c\ub7c9\ub3c4 \uc644\ud310\ub418\uc5b4 \ucd94\uac00 \uc81c\uc791\uc5d0 \ub4e4\uc5b4\uac14\ub2e4. 2016\ub144 4\uc6d4, \uc18c\uc18d\uc0ac\uc640 \uc790\uc2e0\uc758 SNS\ub97c \ud1b5\ud574 2016\ub144 5\uc6d4 15\uc77c \ud604\uc7ac \uad50\uc81c \uc911\uc778 \uc77c\ubc18\uc778 \uc5ec\uc790\uce5c\uad6c\uc640\uc758 \uacb0\ud63c\uc744 \uacf5\uc2dd\ubc1c\ud45c\ud558\uc600\ub2e4.", "answer": "2013\ub144", "sentence": "Mad Clown vs Crucial Star (\ub9e4\ub4dc \ud074\ub77c\uc6b4 vs \ud06c\ub8e8\uc15c \uc2a4\ud0c0)\ub77c\ub294 \ud504\ub85c\uc81d\ud2b8 \uadf8\ub8f9\uc73c\ub85c \ud06c\ub8e8\uc15c \uc2a4\ud0c0\uc640 \ud568\uaed8 \ud65c\ub3d9\ud558\uae30\ub3c4 \ud558\uc600\uc73c\uba70, 2013\ub144 \ubd80\ud130\ub294 MC\uc778 \uc800\uc2a4\ub514\uc2a4\uc640 \ud300\uc744 \uc774\ub8e8\uc5b4 \ub7a9 \ub4c0\uc624 \ucee4\uba3c\ucf5c\ub4dc\ub85c \ud65c\ub3d9\ud558\uace0 \uc788\ub2e4.", "paragraph_sentence": "\uacfc\uac70 \uc18c\uc6b8 \ucef4\ud37c\ub2c8 \uc18c\uc18d\uc73c\ub85c \uc18c\uc6b8 \ucef4\ud37c\ub2c8 \ud574\uccb4 \ud6c4 \ud604\uc7ac\uc758 \uc18c\uc18d\uc0ac\ub294 \uc2a4\ud0c0\uc27d \uc5d1\uc2a4\uc774\ub2e4. Mad Clown vs Crucial Star (\ub9e4\ub4dc \ud074\ub77c\uc6b4 vs \ud06c\ub8e8\uc15c \uc2a4\ud0c0)\ub77c\ub294 \ud504\ub85c\uc81d\ud2b8 \uadf8\ub8f9\uc73c\ub85c \ud06c\ub8e8\uc15c \uc2a4\ud0c0\uc640 \ud568\uaed8 \ud65c\ub3d9\ud558\uae30\ub3c4 \ud558\uc600\uc73c\uba70, 2013\ub144 \ubd80\ud130\ub294 MC\uc778 \uc800\uc2a4\ub514\uc2a4\uc640 \ud300\uc744 \uc774\ub8e8\uc5b4 \ub7a9 \ub4c0\uc624 \ucee4\uba3c\ucf5c\ub4dc\ub85c \ud65c\ub3d9\ud558\uace0 \uc788\ub2e4. \ub610\ud55c Mnet \u300a\uc1fc\ubbf8\ub354\uba38\ub2c8 2\u300b\uc5d0\uc11c \ucc38\uac00\uc790\ub85c \ucc38\uac00\ud558\uc5ec \ud070 \ud654\uc81c\ub97c \ubaa8\uc558\uc73c\uba70, \u300a\uc1fc\ubbf8\ub354\uba38\ub2c8 5\u300b\uc5d0\uc11c\ub294 \uae38 & \ub9e4\ub4dc \ud074\ub77c\uc6b4 \ud300\uc73c\ub85c \ud504\ub85c\ub4c0\uc11c\ub85c \ucd9c\uc5f0\ud558\uc600\ub2e4. , \uc7ac\ubc1c\ub9e4 \ubb3c\ub7c9\ub3c4 \uc644\ud310\ub418\uc5b4 \ucd94\uac00 \uc81c\uc791\uc5d0 \ub4e4\uc5b4\uac14\ub2e4. 2016\ub144 4\uc6d4, \uc18c\uc18d\uc0ac\uc640 \uc790\uc2e0\uc758 SNS\ub97c \ud1b5\ud574 2016\ub144 5\uc6d4 15\uc77c \ud604\uc7ac \uad50\uc81c \uc911\uc778 \uc77c\ubc18\uc778 \uc5ec\uc790\uce5c\uad6c\uc640\uc758 \uacb0\ud63c\uc744 \uacf5\uc2dd\ubc1c\ud45c\ud558\uc600\ub2e4.", "paragraph_answer": "\uacfc\uac70 \uc18c\uc6b8 \ucef4\ud37c\ub2c8 \uc18c\uc18d\uc73c\ub85c \uc18c\uc6b8 \ucef4\ud37c\ub2c8 \ud574\uccb4 \ud6c4 \ud604\uc7ac\uc758 \uc18c\uc18d\uc0ac\ub294 \uc2a4\ud0c0\uc27d \uc5d1\uc2a4\uc774\ub2e4. Mad Clown vs Crucial Star (\ub9e4\ub4dc \ud074\ub77c\uc6b4 vs \ud06c\ub8e8\uc15c \uc2a4\ud0c0)\ub77c\ub294 \ud504\ub85c\uc81d\ud2b8 \uadf8\ub8f9\uc73c\ub85c \ud06c\ub8e8\uc15c \uc2a4\ud0c0\uc640 \ud568\uaed8 \ud65c\ub3d9\ud558\uae30\ub3c4 \ud558\uc600\uc73c\uba70, 2013\ub144 \ubd80\ud130\ub294 MC\uc778 \uc800\uc2a4\ub514\uc2a4\uc640 \ud300\uc744 \uc774\ub8e8\uc5b4 \ub7a9 \ub4c0\uc624 \ucee4\uba3c\ucf5c\ub4dc\ub85c \ud65c\ub3d9\ud558\uace0 \uc788\ub2e4. \ub610\ud55c Mnet \u300a\uc1fc\ubbf8\ub354\uba38\ub2c8 2\u300b\uc5d0\uc11c \ucc38\uac00\uc790\ub85c \ucc38\uac00\ud558\uc5ec \ud070 \ud654\uc81c\ub97c \ubaa8\uc558\uc73c\uba70, \u300a\uc1fc\ubbf8\ub354\uba38\ub2c8 5\u300b\uc5d0\uc11c\ub294 \uae38 & \ub9e4\ub4dc \ud074\ub77c\uc6b4 \ud300\uc73c\ub85c \ud504\ub85c\ub4c0\uc11c\ub85c \ucd9c\uc5f0\ud558\uc600\ub2e4., \uc7ac\ubc1c\ub9e4 \ubb3c\ub7c9\ub3c4 \uc644\ud310\ub418\uc5b4 \ucd94\uac00 \uc81c\uc791\uc5d0 \ub4e4\uc5b4\uac14\ub2e4. 2016\ub144 4\uc6d4, \uc18c\uc18d\uc0ac\uc640 \uc790\uc2e0\uc758 SNS\ub97c \ud1b5\ud574 2016\ub144 5\uc6d4 15\uc77c \ud604\uc7ac \uad50\uc81c \uc911\uc778 \uc77c\ubc18\uc778 \uc5ec\uc790\uce5c\uad6c\uc640\uc758 \uacb0\ud63c\uc744 \uacf5\uc2dd\ubc1c\ud45c\ud558\uc600\ub2e4.", "sentence_answer": "Mad Clown vs Crucial Star (\ub9e4\ub4dc \ud074\ub77c\uc6b4 vs \ud06c\ub8e8\uc15c \uc2a4\ud0c0)\ub77c\ub294 \ud504\ub85c\uc81d\ud2b8 \uadf8\ub8f9\uc73c\ub85c \ud06c\ub8e8\uc15c \uc2a4\ud0c0\uc640 \ud568\uaed8 \ud65c\ub3d9\ud558\uae30\ub3c4 \ud558\uc600\uc73c\uba70, 2013\ub144 \ubd80\ud130\ub294 MC\uc778 \uc800\uc2a4\ub514\uc2a4\uc640 \ud300\uc744 \uc774\ub8e8\uc5b4 \ub7a9 \ub4c0\uc624 \ucee4\uba3c\ucf5c\ub4dc\ub85c \ud65c\ub3d9\ud558\uace0 \uc788\ub2e4.", "paragraph_id": "6074170-0-1", "paragraph_question": "question: \uc2a4\ud0c0\uc27d \uc5d1\uc2a4\ub294 \uc5b8\uc81c\ubd80\ud130 mc\uc778\uc800\uc2a4\ub514\uc2a4\uc640 \ud568\uaed8 \ub7a9 \ub4c0\uc624\ub85c \uacf5\uc5f0\uc744 \ud558\uace0 \uc788\ub294\uac00, context: \uacfc\uac70 \uc18c\uc6b8 \ucef4\ud37c\ub2c8 \uc18c\uc18d\uc73c\ub85c \uc18c\uc6b8 \ucef4\ud37c\ub2c8 \ud574\uccb4 \ud6c4 \ud604\uc7ac\uc758 \uc18c\uc18d\uc0ac\ub294 \uc2a4\ud0c0\uc27d \uc5d1\uc2a4\uc774\ub2e4. Mad Clown vs Crucial Star (\ub9e4\ub4dc \ud074\ub77c\uc6b4 vs \ud06c\ub8e8\uc15c \uc2a4\ud0c0)\ub77c\ub294 \ud504\ub85c\uc81d\ud2b8 \uadf8\ub8f9\uc73c\ub85c \ud06c\ub8e8\uc15c \uc2a4\ud0c0\uc640 \ud568\uaed8 \ud65c\ub3d9\ud558\uae30\ub3c4 \ud558\uc600\uc73c\uba70, 2013\ub144\ubd80\ud130\ub294 MC\uc778 \uc800\uc2a4\ub514\uc2a4\uc640 \ud300\uc744 \uc774\ub8e8\uc5b4 \ub7a9 \ub4c0\uc624 \ucee4\uba3c\ucf5c\ub4dc\ub85c \ud65c\ub3d9\ud558\uace0 \uc788\ub2e4. \ub610\ud55c Mnet \u300a\uc1fc\ubbf8\ub354\uba38\ub2c8 2\u300b\uc5d0\uc11c \ucc38\uac00\uc790\ub85c \ucc38\uac00\ud558\uc5ec \ud070 \ud654\uc81c\ub97c \ubaa8\uc558\uc73c\uba70, \u300a\uc1fc\ubbf8\ub354\uba38\ub2c8 5\u300b\uc5d0\uc11c\ub294 \uae38 & \ub9e4\ub4dc \ud074\ub77c\uc6b4 \ud300\uc73c\ub85c \ud504\ub85c\ub4c0\uc11c\ub85c \ucd9c\uc5f0\ud558\uc600\ub2e4., \uc7ac\ubc1c\ub9e4 \ubb3c\ub7c9\ub3c4 \uc644\ud310\ub418\uc5b4 \ucd94\uac00 \uc81c\uc791\uc5d0 \ub4e4\uc5b4\uac14\ub2e4. 2016\ub144 4\uc6d4, \uc18c\uc18d\uc0ac\uc640 \uc790\uc2e0\uc758 SNS\ub97c \ud1b5\ud574 2016\ub144 5\uc6d4 15\uc77c \ud604\uc7ac \uad50\uc81c \uc911\uc778 \uc77c\ubc18\uc778 \uc5ec\uc790\uce5c\uad6c\uc640\uc758 \uacb0\ud63c\uc744 \uacf5\uc2dd\ubc1c\ud45c\ud558\uc600\ub2e4."} {"question": "2016\ub144 5\uc6d4 10\uc77c\uc5d0 \uc784\uba85\ub41c \ub300\ud1b5\ub839\ube44\uc11c\uc2e4\uc7a5\uc740 \ub204\uad6c\uc778\uac00?", "paragraph": "2016\ub144 4\uc6d4\uc5d0 \uce58\ub904\uc9c4 20\ub300 \ucd1d\uc120 \uacb0\uacfc \ub354\ubd88\uc5b4\ubbfc\uc8fc\ub2f9\ub3c4 \uc0c8\ub204\ub9ac\ub2f9\ub3c4 \uacfc\ubc18\uc218 \uc758\uc11d\uc744 \ucc28\uc9c0\ud558\uc9c0 \ubabb\ud588\uace0, \ub2e4\uc74c \ud574\uc758 \uc870\uae30 \ub300\uc120\uc5d0\uc11c \ubbfc\uc8fc\ub2f9\uc774 \uc9d1\uad8c\ub2f9\uc774 \ub418\uc5c8\uc9c0\ub9cc \uc5ec\uc804\ud788 \uc5ec\uc18c\uc57c\ub300 \uc0c1\ud669\uc5d0 \ubab0\ub824 \uc788\uc5c8\ub2e4. \uc774\uc5d0 \ubb38\uc7ac\uc778\uc740 \ub2f9\uc120\ub41c \uc9c1\ud6c4\uc778 5\uc6d4 10\uc77c \uc57c\ub2f9 \ub300\ud45c\ub4e4\uc744 \ucc28\ub840\ub85c \uc608\ubc29\ud588\uc73c\uba70, \ub2f9\uc77c \uc784\uba85\ub41c \uc784\uc885\uc11d \ub300\ud1b5\ub839\ube44\uc11c\uc2e4\uc7a5\ub3c4 \uc774\ub4e4\uc744 \ubc29\ubb38\ud588\ub2e4. \ub610\ud55c \uc5ec\uc18c\uc57c\ub300 \uc0c1\ud669\uc744 \uadf9\ubcf5\ud558\uae30 \uc704\ud574\uc11c '\ud611\uce58'\uac00 \uc911\uc694\ud55c \ud654\ub450\ub85c \ub5a0\uc62c\ub790\uc73c\uba70 \uc5ec\ub2f9\uc778 \ubbfc\uc8fc\ub2f9 \uc5ed\uc2dc \uac01\uc885 \uac1c\ud601 \ubc95\uc548\uc758 \ucc98\ub9ac\ub97c \uc704\ud574 \uc57c\ub2f9\uacfc\uc758 \ud611\uc870 \uccb4\uacc4\ub97c \uad6c\ucd95\ud558\ub294 \uac83\uc744 \uc6b0\uc120\ud558\uace0 \uc788\ub2e4. \ub2e4\ub9cc, \uad6d\ubbfc\uc758\ub2f9 \ub4f1 \ub2e4\ub978 \ub2f9\uacfc\uc758 \ud1b5\ud569\uc744 \uc8fc\uc7a5\ud558\ub294 \ubaa9\uc18c\ub9ac\ub294 \uc904\uc5b4\ub4e4\uc5c8\ub294\ub370 \uc774\ub294 \uc5ec\ub860\uc744 \uc758\uc2dd\ud558\uba74\uc11c \ub610\ud55c \ub2f9\uc758 \uc815\uccb4\uc131\uc744 \uc720\uc9c0\ud558\uae30 \uc704\ud568\uc778 \uac83\uc73c\ub85c \ubcf4\uc778\ub2e4. \uc1a1\uc601\uae38 \ubbfc\uc8fc\ub2f9 \uc758\uc6d0\uc740 \"\uad6d\ubbfc\uc758\ub2f9, \uc815\uc758\ub2f9 \uac19\uc774 \uc5f0\uc815\ud574\uc57c \ub41c\ub2e4\ub294 \uc785\uc7a5\uc744 \uac00\uc9c0\uace0 \uc788\ub2e4\"\uace0 \ubc1d\ud614\uc73c\uba70, \ubbfc\ubcd1\ub450 \uc758\uc6d0\ub3c4 \ud1b5\ud569\uae4c\uc9c0\ub294 \ub108\ubb34 \ube60\ub978 \uc774\uc57c\uae30\ub77c\uba70 \"\ud1b5\ud569\uc815\ubd80\ub97c \uad6c\uc131\ud558\ub294\ub370 \uc788\uc5b4\uc11c \uc77c\ubd80\ub97c \uac19\uc774 \ud560 \uac83\uc778\uac00 \uc544\ub2c8\uba74 \uac1c\ud601\uc758 \ud504\ub85c\uadf8\ub7a8\uc744 \uac19\uc774 \ub17c\uc758\ud560 \uac83\uc778\uac00 \uc774\ub7f0 \ub2e8\uacc4\ubd80\ud130 \uc77c\uc774 \uc2dc\uc791\ub418\uc5b4\uc57c \ud558\ub294 \uac83\uc774 \uc544\ub2cc\uac00 \uc0dd\uac01\ud55c\ub2e4\"\uace0 \ub9d0\ud588\ub2e4. \uadf8\ub9ac\uace0 \uc790\uc720\ud55c\uad6d\ub2f9 \ub0b4\uc5d0\ub3c4 \uc774\uc804 \uc815\ubd80\uc758 \ud0c4\ud575\uc5d0 \ucc2c\uc131\ud588\ub358 \uc758\uc6d0\ub4e4\uc774 \uc788\uc73c\ubbc0\ub85c \uac1c\ubcc4\uc801\uc73c\ub85c \ud611\uce58\uc758 \ub300\uc0c1\uc774 \ub420 \uc218 \uc788\ub294 \uac83\uc73c\ub85c \ubcf4\uace0 \uc788\ub2e4. \ud611\uce58\uc758 \uc2dc\ud5d8\uc7a5\uc73c\ub85c \uac00\uc7a5 \uba3c\uc800 \ubb34\ub300\uc5d0 \uc624\ub978 \uac83\uc740 \uc774\ub099\uc5f0 \uad6d\ubb34\ucd1d\ub9ac \ud6c4\ubcf4\uc790\uc758 \uc778\uc0ac\uccad\ubb38\ud68c\uc774\ub2e4. \ucde8\uc784\ud558\uc790\ub9d0\uc790 \uc804\ub77c\ub0a8\ub3c4\uc9c0\uc0ac\uc9c1\uc744 \uc218\ud589\ud558\ub358 \uc774\ub099\uc5f0\uc744 \ucd1d\ub9ac \ud6c4\ubcf4\uc790\ub85c \uc9c0\uba85\ud558\uace0 24\uc77c\ubd80\ud130 \uad6d\ud68c\uc5d0\uc11c \uccad\ubb38\ud68c\ub97c \uc9c4\ud589\ud588\ub2e4. \ud55c\uad6d\ub2f9\uacfc \ubc14\ub978\uc815\ub2f9\uc740 \"\ud655\uc2e4\ud55c \uacb0\uaca9 \uc0ac\uc720\uac00 \uc5c6\ub294 \ud55c \ubc1c\ubaa9\uc740 \uc7a1\uc9c0 \uc54a\uaca0\ub2e4\"\uace0 \ubc1d\ud788\uba74\uc11c \uad6d\ud68c\uc758\uc6d0 \ucd9c\uc2e0\uc774\ub77c \uc6b0\ud638\uc801\uc778 \ud3c9\uac00\uac00 \ub098\uc624\uace0 \uc788\uc73c\uba70, \uad6d\ubbfc\uc758\ub2f9 \uc5ed\uc2dc \ud638\ub0a8 \ucd9c\uc2e0\uc778 \uc774\ub099\uc5f0 \ud6c4\ubcf4\uc790\ub97c \ubc18\ub300\ud558\uae30\uc5d0\ub294 \ubd80\ub2f4\uc774 \ub530\ub97c \uac83\uc774\ub77c\ub294 \uad00\uce21\uc774 \ub9ce\uc558\ub2e4. \ud558\uc9c0\ub9cc \ub300\uc120 \ub2f9\uc2dc \ubb38\uc7ac\uc778\uc774 \uc704\uc7a5\uc804\uc785, \ud0c8\uc138, \ubcd1\ub825\uba74\ud0c8, \ub17c\ubb38 \ud45c\uc808, \ubd80\ub3d9\uc0b0 \ud22c\uae30\uc640 \uad00\ub828\ub41c \uc0ac\ub78c\uc740 \uacf5\uc9c1\uc5d0 \uae30\uc6a9\ud558\uc9c0 \uc54a\uaca0\ub2e4\uace0 \ubc1d\ud78c \uc778\uc0ac\uae30\uc900 5\ub300 \uc6d0\uce59 \uc911 \uc704\uc7a5\uc804\uc785, \uc544\ub4e4\uc758 \ubcd1\uc5ed \ub17c\ub780, \uc77c\ubd80 \uc138\uae08 \ud0c8\ub8e8 \ubb38\uc81c\uac00 \ubd88\uac70\uc9c0\uc790 \uc57c\ub2f9\uc758 \ubc18\ub300\uac00 \uc2ec\ud574\uc84c\uace0 \ubbfc\uc8fc\ub2f9\uc774 \uc774\uc5d0 \ub300\ud574 \"\ud611\uce58\uc640 \ub300\ud1b5\ud569\uc758 \uc2dc\uc791\uc5d0 \uc5ec\uc57c \ud560 \uac83 \uc5c6\uc774 \ub300\uc2b9\uc801\uc778 \uacb0\ub2e8\uc744 \ub0b4\ub824\ub2ec\ub77c\"\uace0 \uad6d\uc815\uc758 \uc815\uc0c1\ud654\ub9cc \uac15\uc870\ud558\uba74\uc11c \uc5ec\uc57c\uac00 \ub300\uce58\ud558\ub294 \ud615\uad6d\uc73c\ub85c \ubc14\ub00c\uc5c8\ub2e4. \uc5ec\uae30\uc5d0 \uc678\uad50\ubd80 \uc7a5\uad00\uc73c\ub85c \uc9c0\uba85\ub41c \uac15\uacbd\ud654\uc640 \uacf5\uc815\uac70\ub798\uc704\uc6d0\ud68c \uc704\uc6d0\uc7a5\uc73c\ub85c \uc9c0\uba85\ub41c \uae40\uc0c1\uc870\uc5d0 \ub300\ud574\uc11c\ub3c4 5\ub300 \uae30\uc900\uc5d0 \uc704\ubc30\ub418\ub294 \ubb38\uc81c\uac00 \ub098\ud0c0\ub098\uba74\uc11c \uc815\uad6d\uc774 \uae09\uaca9\ud788 \ub0c9\ub7ad\ud574\uc84c\ub2e4. \uc774\ud6c4 31\uc77c \uc790\uc720\ud55c\uad6d\ub2f9 \uc758\uc6d0\ub4e4\uc774 \ud1f4\uc7a5\ud55c \uac00\uc6b4\ub370 \uc5ec\uc57c \uc758\uc6d0 188\uba85\ub9cc\uc774 \ucc38\uc11d\ud558\uc5ec \uaca8\uc6b0 \uc778\uc900\uc548\uc744 \ud1b5\uacfc\uc2dc\ud0ac \uc218 \uc788\uc5c8\ub2e4. \uc774\ud6c4 \uc6b0\uc6d0\uc2dd \ubbfc\uc8fc\ub2f9 \uc6d0\ub0b4\ub300\ud45c\ub294 \"\uc6b0\uc5ec\uace1\uc808\uc744 \uacaa\uae34 \ud588\uc9c0\ub9cc \ud611\uce58\uac00 \uc911\ub2e8\ub41c \uac83\uc740 \uc544\ub2c8\ub2e4\"\uace0 \ubc1d\ud614\uc9c0\ub9cc \uc815\uc6b0\ud0dd \ud55c\uad6d\ub2f9 \uc6d0\ub0b4\ub300\ud45c\ub294 \"\ubaa8\ub4e0 \ub300\ud45c\ub294 \ub300\ud1b5\ub839\uc5d0\uac8c \uc788\ub2e4. \uc9c0\uae08 \uc0c1\ud0dc\ub860 (\ud611\uce58\uac00) \uc5b4\ub835\ub2e4\"\uace0 \ub2e8\uc5b8\ud588\ub2e4.", "answer": "\uc784\uc885\uc11d", "sentence": "\uc774\uc5d0 \ubb38\uc7ac\uc778\uc740 \ub2f9\uc120\ub41c \uc9c1\ud6c4\uc778 5\uc6d4 10\uc77c \uc57c\ub2f9 \ub300\ud45c\ub4e4\uc744 \ucc28\ub840\ub85c \uc608\ubc29\ud588\uc73c\uba70, \ub2f9\uc77c \uc784\uba85\ub41c \uc784\uc885\uc11d \ub300\ud1b5\ub839\ube44\uc11c\uc2e4\uc7a5\ub3c4 \uc774\ub4e4\uc744 \ubc29\ubb38\ud588\ub2e4.", "paragraph_sentence": "2016\ub144 4\uc6d4\uc5d0 \uce58\ub904\uc9c4 20\ub300 \ucd1d\uc120 \uacb0\uacfc \ub354\ubd88\uc5b4\ubbfc\uc8fc\ub2f9\ub3c4 \uc0c8\ub204\ub9ac\ub2f9\ub3c4 \uacfc\ubc18\uc218 \uc758\uc11d\uc744 \ucc28\uc9c0\ud558\uc9c0 \ubabb\ud588\uace0, \ub2e4\uc74c \ud574\uc758 \uc870\uae30 \ub300\uc120\uc5d0\uc11c \ubbfc\uc8fc\ub2f9\uc774 \uc9d1\uad8c\ub2f9\uc774 \ub418\uc5c8\uc9c0\ub9cc \uc5ec\uc804\ud788 \uc5ec\uc18c\uc57c\ub300 \uc0c1\ud669\uc5d0 \ubab0\ub824 \uc788\uc5c8\ub2e4. \uc774\uc5d0 \ubb38\uc7ac\uc778\uc740 \ub2f9\uc120\ub41c \uc9c1\ud6c4\uc778 5\uc6d4 10\uc77c \uc57c\ub2f9 \ub300\ud45c\ub4e4\uc744 \ucc28\ub840\ub85c \uc608\ubc29\ud588\uc73c\uba70, \ub2f9\uc77c \uc784\uba85\ub41c \uc784\uc885\uc11d \ub300\ud1b5\ub839\ube44\uc11c\uc2e4\uc7a5\ub3c4 \uc774\ub4e4\uc744 \ubc29\ubb38\ud588\ub2e4. \ub610\ud55c \uc5ec\uc18c\uc57c\ub300 \uc0c1\ud669\uc744 \uadf9\ubcf5\ud558\uae30 \uc704\ud574\uc11c '\ud611\uce58'\uac00 \uc911\uc694\ud55c \ud654\ub450\ub85c \ub5a0\uc62c\ub790\uc73c\uba70 \uc5ec\ub2f9\uc778 \ubbfc\uc8fc\ub2f9 \uc5ed\uc2dc \uac01\uc885 \uac1c\ud601 \ubc95\uc548\uc758 \ucc98\ub9ac\ub97c \uc704\ud574 \uc57c\ub2f9\uacfc\uc758 \ud611\uc870 \uccb4\uacc4\ub97c \uad6c\ucd95\ud558\ub294 \uac83\uc744 \uc6b0\uc120\ud558\uace0 \uc788\ub2e4. \ub2e4\ub9cc, \uad6d\ubbfc\uc758\ub2f9 \ub4f1 \ub2e4\ub978 \ub2f9\uacfc\uc758 \ud1b5\ud569\uc744 \uc8fc\uc7a5\ud558\ub294 \ubaa9\uc18c\ub9ac\ub294 \uc904\uc5b4\ub4e4\uc5c8\ub294\ub370 \uc774\ub294 \uc5ec\ub860\uc744 \uc758\uc2dd\ud558\uba74\uc11c \ub610\ud55c \ub2f9\uc758 \uc815\uccb4\uc131\uc744 \uc720\uc9c0\ud558\uae30 \uc704\ud568\uc778 \uac83\uc73c\ub85c \ubcf4\uc778\ub2e4. \uc1a1\uc601\uae38 \ubbfc\uc8fc\ub2f9 \uc758\uc6d0\uc740 \"\uad6d\ubbfc\uc758\ub2f9, \uc815\uc758\ub2f9 \uac19\uc774 \uc5f0\uc815\ud574\uc57c \ub41c\ub2e4\ub294 \uc785\uc7a5\uc744 \uac00\uc9c0\uace0 \uc788\ub2e4\"\uace0 \ubc1d\ud614\uc73c\uba70, \ubbfc\ubcd1\ub450 \uc758\uc6d0\ub3c4 \ud1b5\ud569\uae4c\uc9c0\ub294 \ub108\ubb34 \ube60\ub978 \uc774\uc57c\uae30\ub77c\uba70 \"\ud1b5\ud569\uc815\ubd80\ub97c \uad6c\uc131\ud558\ub294\ub370 \uc788\uc5b4\uc11c \uc77c\ubd80\ub97c \uac19\uc774 \ud560 \uac83\uc778\uac00 \uc544\ub2c8\uba74 \uac1c\ud601\uc758 \ud504\ub85c\uadf8\ub7a8\uc744 \uac19\uc774 \ub17c\uc758\ud560 \uac83\uc778\uac00 \uc774\ub7f0 \ub2e8\uacc4\ubd80\ud130 \uc77c\uc774 \uc2dc\uc791\ub418\uc5b4\uc57c \ud558\ub294 \uac83\uc774 \uc544\ub2cc\uac00 \uc0dd\uac01\ud55c\ub2e4\"\uace0 \ub9d0\ud588\ub2e4. \uadf8\ub9ac\uace0 \uc790\uc720\ud55c\uad6d\ub2f9 \ub0b4\uc5d0\ub3c4 \uc774\uc804 \uc815\ubd80\uc758 \ud0c4\ud575\uc5d0 \ucc2c\uc131\ud588\ub358 \uc758\uc6d0\ub4e4\uc774 \uc788\uc73c\ubbc0\ub85c \uac1c\ubcc4\uc801\uc73c\ub85c \ud611\uce58\uc758 \ub300\uc0c1\uc774 \ub420 \uc218 \uc788\ub294 \uac83\uc73c\ub85c \ubcf4\uace0 \uc788\ub2e4. \ud611\uce58\uc758 \uc2dc\ud5d8\uc7a5\uc73c\ub85c \uac00\uc7a5 \uba3c\uc800 \ubb34\ub300\uc5d0 \uc624\ub978 \uac83\uc740 \uc774\ub099\uc5f0 \uad6d\ubb34\ucd1d\ub9ac \ud6c4\ubcf4\uc790\uc758 \uc778\uc0ac\uccad\ubb38\ud68c\uc774\ub2e4. \ucde8\uc784\ud558\uc790\ub9d0\uc790 \uc804\ub77c\ub0a8\ub3c4\uc9c0\uc0ac\uc9c1\uc744 \uc218\ud589\ud558\ub358 \uc774\ub099\uc5f0\uc744 \ucd1d\ub9ac \ud6c4\ubcf4\uc790\ub85c \uc9c0\uba85\ud558\uace0 24\uc77c\ubd80\ud130 \uad6d\ud68c\uc5d0\uc11c \uccad\ubb38\ud68c\ub97c \uc9c4\ud589\ud588\ub2e4. \ud55c\uad6d\ub2f9\uacfc \ubc14\ub978\uc815\ub2f9\uc740 \"\ud655\uc2e4\ud55c \uacb0\uaca9 \uc0ac\uc720\uac00 \uc5c6\ub294 \ud55c \ubc1c\ubaa9\uc740 \uc7a1\uc9c0 \uc54a\uaca0\ub2e4\"\uace0 \ubc1d\ud788\uba74\uc11c \uad6d\ud68c\uc758\uc6d0 \ucd9c\uc2e0\uc774\ub77c \uc6b0\ud638\uc801\uc778 \ud3c9\uac00\uac00 \ub098\uc624\uace0 \uc788\uc73c\uba70, \uad6d\ubbfc\uc758\ub2f9 \uc5ed\uc2dc \ud638\ub0a8 \ucd9c\uc2e0\uc778 \uc774\ub099\uc5f0 \ud6c4\ubcf4\uc790\ub97c \ubc18\ub300\ud558\uae30\uc5d0\ub294 \ubd80\ub2f4\uc774 \ub530\ub97c \uac83\uc774\ub77c\ub294 \uad00\uce21\uc774 \ub9ce\uc558\ub2e4. \ud558\uc9c0\ub9cc \ub300\uc120 \ub2f9\uc2dc \ubb38\uc7ac\uc778\uc774 \uc704\uc7a5\uc804\uc785, \ud0c8\uc138, \ubcd1\ub825\uba74\ud0c8, \ub17c\ubb38 \ud45c\uc808, \ubd80\ub3d9\uc0b0 \ud22c\uae30\uc640 \uad00\ub828\ub41c \uc0ac\ub78c\uc740 \uacf5\uc9c1\uc5d0 \uae30\uc6a9\ud558\uc9c0 \uc54a\uaca0\ub2e4\uace0 \ubc1d\ud78c \uc778\uc0ac\uae30\uc900 5\ub300 \uc6d0\uce59 \uc911 \uc704\uc7a5\uc804\uc785, \uc544\ub4e4\uc758 \ubcd1\uc5ed \ub17c\ub780, \uc77c\ubd80 \uc138\uae08 \ud0c8\ub8e8 \ubb38\uc81c\uac00 \ubd88\uac70\uc9c0\uc790 \uc57c\ub2f9\uc758 \ubc18\ub300\uac00 \uc2ec\ud574\uc84c\uace0 \ubbfc\uc8fc\ub2f9\uc774 \uc774\uc5d0 \ub300\ud574 \"\ud611\uce58\uc640 \ub300\ud1b5\ud569\uc758 \uc2dc\uc791\uc5d0 \uc5ec\uc57c \ud560 \uac83 \uc5c6\uc774 \ub300\uc2b9\uc801\uc778 \uacb0\ub2e8\uc744 \ub0b4\ub824\ub2ec\ub77c\"\uace0 \uad6d\uc815\uc758 \uc815\uc0c1\ud654\ub9cc \uac15\uc870\ud558\uba74\uc11c \uc5ec\uc57c\uac00 \ub300\uce58\ud558\ub294 \ud615\uad6d\uc73c\ub85c \ubc14\ub00c\uc5c8\ub2e4. \uc5ec\uae30\uc5d0 \uc678\uad50\ubd80 \uc7a5\uad00\uc73c\ub85c \uc9c0\uba85\ub41c \uac15\uacbd\ud654\uc640 \uacf5\uc815\uac70\ub798\uc704\uc6d0\ud68c \uc704\uc6d0\uc7a5\uc73c\ub85c \uc9c0\uba85\ub41c \uae40\uc0c1\uc870\uc5d0 \ub300\ud574\uc11c\ub3c4 5\ub300 \uae30\uc900\uc5d0 \uc704\ubc30\ub418\ub294 \ubb38\uc81c\uac00 \ub098\ud0c0\ub098\uba74\uc11c \uc815\uad6d\uc774 \uae09\uaca9\ud788 \ub0c9\ub7ad\ud574\uc84c\ub2e4. \uc774\ud6c4 31\uc77c \uc790\uc720\ud55c\uad6d\ub2f9 \uc758\uc6d0\ub4e4\uc774 \ud1f4\uc7a5\ud55c \uac00\uc6b4\ub370 \uc5ec\uc57c \uc758\uc6d0 188\uba85\ub9cc\uc774 \ucc38\uc11d\ud558\uc5ec \uaca8\uc6b0 \uc778\uc900\uc548\uc744 \ud1b5\uacfc\uc2dc\ud0ac \uc218 \uc788\uc5c8\ub2e4. \uc774\ud6c4 \uc6b0\uc6d0\uc2dd \ubbfc\uc8fc\ub2f9 \uc6d0\ub0b4\ub300\ud45c\ub294 \"\uc6b0\uc5ec\uace1\uc808\uc744 \uacaa\uae34 \ud588\uc9c0\ub9cc \ud611\uce58\uac00 \uc911\ub2e8\ub41c \uac83\uc740 \uc544\ub2c8\ub2e4\"\uace0 \ubc1d\ud614\uc9c0\ub9cc \uc815\uc6b0\ud0dd \ud55c\uad6d\ub2f9 \uc6d0\ub0b4\ub300\ud45c\ub294 \"\ubaa8\ub4e0 \ub300\ud45c\ub294 \ub300\ud1b5\ub839\uc5d0\uac8c \uc788\ub2e4. \uc9c0\uae08 \uc0c1\ud0dc\ub860 (\ud611\uce58\uac00) \uc5b4\ub835\ub2e4\"\uace0 \ub2e8\uc5b8\ud588\ub2e4.", "paragraph_answer": "2016\ub144 4\uc6d4\uc5d0 \uce58\ub904\uc9c4 20\ub300 \ucd1d\uc120 \uacb0\uacfc \ub354\ubd88\uc5b4\ubbfc\uc8fc\ub2f9\ub3c4 \uc0c8\ub204\ub9ac\ub2f9\ub3c4 \uacfc\ubc18\uc218 \uc758\uc11d\uc744 \ucc28\uc9c0\ud558\uc9c0 \ubabb\ud588\uace0, \ub2e4\uc74c \ud574\uc758 \uc870\uae30 \ub300\uc120\uc5d0\uc11c \ubbfc\uc8fc\ub2f9\uc774 \uc9d1\uad8c\ub2f9\uc774 \ub418\uc5c8\uc9c0\ub9cc \uc5ec\uc804\ud788 \uc5ec\uc18c\uc57c\ub300 \uc0c1\ud669\uc5d0 \ubab0\ub824 \uc788\uc5c8\ub2e4. \uc774\uc5d0 \ubb38\uc7ac\uc778\uc740 \ub2f9\uc120\ub41c \uc9c1\ud6c4\uc778 5\uc6d4 10\uc77c \uc57c\ub2f9 \ub300\ud45c\ub4e4\uc744 \ucc28\ub840\ub85c \uc608\ubc29\ud588\uc73c\uba70, \ub2f9\uc77c \uc784\uba85\ub41c \uc784\uc885\uc11d \ub300\ud1b5\ub839\ube44\uc11c\uc2e4\uc7a5\ub3c4 \uc774\ub4e4\uc744 \ubc29\ubb38\ud588\ub2e4. \ub610\ud55c \uc5ec\uc18c\uc57c\ub300 \uc0c1\ud669\uc744 \uadf9\ubcf5\ud558\uae30 \uc704\ud574\uc11c '\ud611\uce58'\uac00 \uc911\uc694\ud55c \ud654\ub450\ub85c \ub5a0\uc62c\ub790\uc73c\uba70 \uc5ec\ub2f9\uc778 \ubbfc\uc8fc\ub2f9 \uc5ed\uc2dc \uac01\uc885 \uac1c\ud601 \ubc95\uc548\uc758 \ucc98\ub9ac\ub97c \uc704\ud574 \uc57c\ub2f9\uacfc\uc758 \ud611\uc870 \uccb4\uacc4\ub97c \uad6c\ucd95\ud558\ub294 \uac83\uc744 \uc6b0\uc120\ud558\uace0 \uc788\ub2e4. \ub2e4\ub9cc, \uad6d\ubbfc\uc758\ub2f9 \ub4f1 \ub2e4\ub978 \ub2f9\uacfc\uc758 \ud1b5\ud569\uc744 \uc8fc\uc7a5\ud558\ub294 \ubaa9\uc18c\ub9ac\ub294 \uc904\uc5b4\ub4e4\uc5c8\ub294\ub370 \uc774\ub294 \uc5ec\ub860\uc744 \uc758\uc2dd\ud558\uba74\uc11c \ub610\ud55c \ub2f9\uc758 \uc815\uccb4\uc131\uc744 \uc720\uc9c0\ud558\uae30 \uc704\ud568\uc778 \uac83\uc73c\ub85c \ubcf4\uc778\ub2e4. \uc1a1\uc601\uae38 \ubbfc\uc8fc\ub2f9 \uc758\uc6d0\uc740 \"\uad6d\ubbfc\uc758\ub2f9, \uc815\uc758\ub2f9 \uac19\uc774 \uc5f0\uc815\ud574\uc57c \ub41c\ub2e4\ub294 \uc785\uc7a5\uc744 \uac00\uc9c0\uace0 \uc788\ub2e4\"\uace0 \ubc1d\ud614\uc73c\uba70, \ubbfc\ubcd1\ub450 \uc758\uc6d0\ub3c4 \ud1b5\ud569\uae4c\uc9c0\ub294 \ub108\ubb34 \ube60\ub978 \uc774\uc57c\uae30\ub77c\uba70 \"\ud1b5\ud569\uc815\ubd80\ub97c \uad6c\uc131\ud558\ub294\ub370 \uc788\uc5b4\uc11c \uc77c\ubd80\ub97c \uac19\uc774 \ud560 \uac83\uc778\uac00 \uc544\ub2c8\uba74 \uac1c\ud601\uc758 \ud504\ub85c\uadf8\ub7a8\uc744 \uac19\uc774 \ub17c\uc758\ud560 \uac83\uc778\uac00 \uc774\ub7f0 \ub2e8\uacc4\ubd80\ud130 \uc77c\uc774 \uc2dc\uc791\ub418\uc5b4\uc57c \ud558\ub294 \uac83\uc774 \uc544\ub2cc\uac00 \uc0dd\uac01\ud55c\ub2e4\"\uace0 \ub9d0\ud588\ub2e4. \uadf8\ub9ac\uace0 \uc790\uc720\ud55c\uad6d\ub2f9 \ub0b4\uc5d0\ub3c4 \uc774\uc804 \uc815\ubd80\uc758 \ud0c4\ud575\uc5d0 \ucc2c\uc131\ud588\ub358 \uc758\uc6d0\ub4e4\uc774 \uc788\uc73c\ubbc0\ub85c \uac1c\ubcc4\uc801\uc73c\ub85c \ud611\uce58\uc758 \ub300\uc0c1\uc774 \ub420 \uc218 \uc788\ub294 \uac83\uc73c\ub85c \ubcf4\uace0 \uc788\ub2e4. \ud611\uce58\uc758 \uc2dc\ud5d8\uc7a5\uc73c\ub85c \uac00\uc7a5 \uba3c\uc800 \ubb34\ub300\uc5d0 \uc624\ub978 \uac83\uc740 \uc774\ub099\uc5f0 \uad6d\ubb34\ucd1d\ub9ac \ud6c4\ubcf4\uc790\uc758 \uc778\uc0ac\uccad\ubb38\ud68c\uc774\ub2e4. \ucde8\uc784\ud558\uc790\ub9d0\uc790 \uc804\ub77c\ub0a8\ub3c4\uc9c0\uc0ac\uc9c1\uc744 \uc218\ud589\ud558\ub358 \uc774\ub099\uc5f0\uc744 \ucd1d\ub9ac \ud6c4\ubcf4\uc790\ub85c \uc9c0\uba85\ud558\uace0 24\uc77c\ubd80\ud130 \uad6d\ud68c\uc5d0\uc11c \uccad\ubb38\ud68c\ub97c \uc9c4\ud589\ud588\ub2e4. \ud55c\uad6d\ub2f9\uacfc \ubc14\ub978\uc815\ub2f9\uc740 \"\ud655\uc2e4\ud55c \uacb0\uaca9 \uc0ac\uc720\uac00 \uc5c6\ub294 \ud55c \ubc1c\ubaa9\uc740 \uc7a1\uc9c0 \uc54a\uaca0\ub2e4\"\uace0 \ubc1d\ud788\uba74\uc11c \uad6d\ud68c\uc758\uc6d0 \ucd9c\uc2e0\uc774\ub77c \uc6b0\ud638\uc801\uc778 \ud3c9\uac00\uac00 \ub098\uc624\uace0 \uc788\uc73c\uba70, \uad6d\ubbfc\uc758\ub2f9 \uc5ed\uc2dc \ud638\ub0a8 \ucd9c\uc2e0\uc778 \uc774\ub099\uc5f0 \ud6c4\ubcf4\uc790\ub97c \ubc18\ub300\ud558\uae30\uc5d0\ub294 \ubd80\ub2f4\uc774 \ub530\ub97c \uac83\uc774\ub77c\ub294 \uad00\uce21\uc774 \ub9ce\uc558\ub2e4. \ud558\uc9c0\ub9cc \ub300\uc120 \ub2f9\uc2dc \ubb38\uc7ac\uc778\uc774 \uc704\uc7a5\uc804\uc785, \ud0c8\uc138, \ubcd1\ub825\uba74\ud0c8, \ub17c\ubb38 \ud45c\uc808, \ubd80\ub3d9\uc0b0 \ud22c\uae30\uc640 \uad00\ub828\ub41c \uc0ac\ub78c\uc740 \uacf5\uc9c1\uc5d0 \uae30\uc6a9\ud558\uc9c0 \uc54a\uaca0\ub2e4\uace0 \ubc1d\ud78c \uc778\uc0ac\uae30\uc900 5\ub300 \uc6d0\uce59 \uc911 \uc704\uc7a5\uc804\uc785, \uc544\ub4e4\uc758 \ubcd1\uc5ed \ub17c\ub780, \uc77c\ubd80 \uc138\uae08 \ud0c8\ub8e8 \ubb38\uc81c\uac00 \ubd88\uac70\uc9c0\uc790 \uc57c\ub2f9\uc758 \ubc18\ub300\uac00 \uc2ec\ud574\uc84c\uace0 \ubbfc\uc8fc\ub2f9\uc774 \uc774\uc5d0 \ub300\ud574 \"\ud611\uce58\uc640 \ub300\ud1b5\ud569\uc758 \uc2dc\uc791\uc5d0 \uc5ec\uc57c \ud560 \uac83 \uc5c6\uc774 \ub300\uc2b9\uc801\uc778 \uacb0\ub2e8\uc744 \ub0b4\ub824\ub2ec\ub77c\"\uace0 \uad6d\uc815\uc758 \uc815\uc0c1\ud654\ub9cc \uac15\uc870\ud558\uba74\uc11c \uc5ec\uc57c\uac00 \ub300\uce58\ud558\ub294 \ud615\uad6d\uc73c\ub85c \ubc14\ub00c\uc5c8\ub2e4. \uc5ec\uae30\uc5d0 \uc678\uad50\ubd80 \uc7a5\uad00\uc73c\ub85c \uc9c0\uba85\ub41c \uac15\uacbd\ud654\uc640 \uacf5\uc815\uac70\ub798\uc704\uc6d0\ud68c \uc704\uc6d0\uc7a5\uc73c\ub85c \uc9c0\uba85\ub41c \uae40\uc0c1\uc870\uc5d0 \ub300\ud574\uc11c\ub3c4 5\ub300 \uae30\uc900\uc5d0 \uc704\ubc30\ub418\ub294 \ubb38\uc81c\uac00 \ub098\ud0c0\ub098\uba74\uc11c \uc815\uad6d\uc774 \uae09\uaca9\ud788 \ub0c9\ub7ad\ud574\uc84c\ub2e4. \uc774\ud6c4 31\uc77c \uc790\uc720\ud55c\uad6d\ub2f9 \uc758\uc6d0\ub4e4\uc774 \ud1f4\uc7a5\ud55c \uac00\uc6b4\ub370 \uc5ec\uc57c \uc758\uc6d0 188\uba85\ub9cc\uc774 \ucc38\uc11d\ud558\uc5ec \uaca8\uc6b0 \uc778\uc900\uc548\uc744 \ud1b5\uacfc\uc2dc\ud0ac \uc218 \uc788\uc5c8\ub2e4. \uc774\ud6c4 \uc6b0\uc6d0\uc2dd \ubbfc\uc8fc\ub2f9 \uc6d0\ub0b4\ub300\ud45c\ub294 \"\uc6b0\uc5ec\uace1\uc808\uc744 \uacaa\uae34 \ud588\uc9c0\ub9cc \ud611\uce58\uac00 \uc911\ub2e8\ub41c \uac83\uc740 \uc544\ub2c8\ub2e4\"\uace0 \ubc1d\ud614\uc9c0\ub9cc \uc815\uc6b0\ud0dd \ud55c\uad6d\ub2f9 \uc6d0\ub0b4\ub300\ud45c\ub294 \"\ubaa8\ub4e0 \ub300\ud45c\ub294 \ub300\ud1b5\ub839\uc5d0\uac8c \uc788\ub2e4. \uc9c0\uae08 \uc0c1\ud0dc\ub860 (\ud611\uce58\uac00) \uc5b4\ub835\ub2e4\"\uace0 \ub2e8\uc5b8\ud588\ub2e4.", "sentence_answer": "\uc774\uc5d0 \ubb38\uc7ac\uc778\uc740 \ub2f9\uc120\ub41c \uc9c1\ud6c4\uc778 5\uc6d4 10\uc77c \uc57c\ub2f9 \ub300\ud45c\ub4e4\uc744 \ucc28\ub840\ub85c \uc608\ubc29\ud588\uc73c\uba70, \ub2f9\uc77c \uc784\uba85\ub41c \uc784\uc885\uc11d \ub300\ud1b5\ub839\ube44\uc11c\uc2e4\uc7a5\ub3c4 \uc774\ub4e4\uc744 \ubc29\ubb38\ud588\ub2e4.", "paragraph_id": "6580768-61-0", "paragraph_question": "question: 2016\ub144 5\uc6d4 10\uc77c\uc5d0 \uc784\uba85\ub41c \ub300\ud1b5\ub839\ube44\uc11c\uc2e4\uc7a5\uc740 \ub204\uad6c\uc778\uac00?, context: 2016\ub144 4\uc6d4\uc5d0 \uce58\ub904\uc9c4 20\ub300 \ucd1d\uc120 \uacb0\uacfc \ub354\ubd88\uc5b4\ubbfc\uc8fc\ub2f9\ub3c4 \uc0c8\ub204\ub9ac\ub2f9\ub3c4 \uacfc\ubc18\uc218 \uc758\uc11d\uc744 \ucc28\uc9c0\ud558\uc9c0 \ubabb\ud588\uace0, \ub2e4\uc74c \ud574\uc758 \uc870\uae30 \ub300\uc120\uc5d0\uc11c \ubbfc\uc8fc\ub2f9\uc774 \uc9d1\uad8c\ub2f9\uc774 \ub418\uc5c8\uc9c0\ub9cc \uc5ec\uc804\ud788 \uc5ec\uc18c\uc57c\ub300 \uc0c1\ud669\uc5d0 \ubab0\ub824 \uc788\uc5c8\ub2e4. \uc774\uc5d0 \ubb38\uc7ac\uc778\uc740 \ub2f9\uc120\ub41c \uc9c1\ud6c4\uc778 5\uc6d4 10\uc77c \uc57c\ub2f9 \ub300\ud45c\ub4e4\uc744 \ucc28\ub840\ub85c \uc608\ubc29\ud588\uc73c\uba70, \ub2f9\uc77c \uc784\uba85\ub41c \uc784\uc885\uc11d \ub300\ud1b5\ub839\ube44\uc11c\uc2e4\uc7a5\ub3c4 \uc774\ub4e4\uc744 \ubc29\ubb38\ud588\ub2e4. \ub610\ud55c \uc5ec\uc18c\uc57c\ub300 \uc0c1\ud669\uc744 \uadf9\ubcf5\ud558\uae30 \uc704\ud574\uc11c '\ud611\uce58'\uac00 \uc911\uc694\ud55c \ud654\ub450\ub85c \ub5a0\uc62c\ub790\uc73c\uba70 \uc5ec\ub2f9\uc778 \ubbfc\uc8fc\ub2f9 \uc5ed\uc2dc \uac01\uc885 \uac1c\ud601 \ubc95\uc548\uc758 \ucc98\ub9ac\ub97c \uc704\ud574 \uc57c\ub2f9\uacfc\uc758 \ud611\uc870 \uccb4\uacc4\ub97c \uad6c\ucd95\ud558\ub294 \uac83\uc744 \uc6b0\uc120\ud558\uace0 \uc788\ub2e4. \ub2e4\ub9cc, \uad6d\ubbfc\uc758\ub2f9 \ub4f1 \ub2e4\ub978 \ub2f9\uacfc\uc758 \ud1b5\ud569\uc744 \uc8fc\uc7a5\ud558\ub294 \ubaa9\uc18c\ub9ac\ub294 \uc904\uc5b4\ub4e4\uc5c8\ub294\ub370 \uc774\ub294 \uc5ec\ub860\uc744 \uc758\uc2dd\ud558\uba74\uc11c \ub610\ud55c \ub2f9\uc758 \uc815\uccb4\uc131\uc744 \uc720\uc9c0\ud558\uae30 \uc704\ud568\uc778 \uac83\uc73c\ub85c \ubcf4\uc778\ub2e4. \uc1a1\uc601\uae38 \ubbfc\uc8fc\ub2f9 \uc758\uc6d0\uc740 \"\uad6d\ubbfc\uc758\ub2f9, \uc815\uc758\ub2f9 \uac19\uc774 \uc5f0\uc815\ud574\uc57c \ub41c\ub2e4\ub294 \uc785\uc7a5\uc744 \uac00\uc9c0\uace0 \uc788\ub2e4\"\uace0 \ubc1d\ud614\uc73c\uba70, \ubbfc\ubcd1\ub450 \uc758\uc6d0\ub3c4 \ud1b5\ud569\uae4c\uc9c0\ub294 \ub108\ubb34 \ube60\ub978 \uc774\uc57c\uae30\ub77c\uba70 \"\ud1b5\ud569\uc815\ubd80\ub97c \uad6c\uc131\ud558\ub294\ub370 \uc788\uc5b4\uc11c \uc77c\ubd80\ub97c \uac19\uc774 \ud560 \uac83\uc778\uac00 \uc544\ub2c8\uba74 \uac1c\ud601\uc758 \ud504\ub85c\uadf8\ub7a8\uc744 \uac19\uc774 \ub17c\uc758\ud560 \uac83\uc778\uac00 \uc774\ub7f0 \ub2e8\uacc4\ubd80\ud130 \uc77c\uc774 \uc2dc\uc791\ub418\uc5b4\uc57c \ud558\ub294 \uac83\uc774 \uc544\ub2cc\uac00 \uc0dd\uac01\ud55c\ub2e4\"\uace0 \ub9d0\ud588\ub2e4. \uadf8\ub9ac\uace0 \uc790\uc720\ud55c\uad6d\ub2f9 \ub0b4\uc5d0\ub3c4 \uc774\uc804 \uc815\ubd80\uc758 \ud0c4\ud575\uc5d0 \ucc2c\uc131\ud588\ub358 \uc758\uc6d0\ub4e4\uc774 \uc788\uc73c\ubbc0\ub85c \uac1c\ubcc4\uc801\uc73c\ub85c \ud611\uce58\uc758 \ub300\uc0c1\uc774 \ub420 \uc218 \uc788\ub294 \uac83\uc73c\ub85c \ubcf4\uace0 \uc788\ub2e4. \ud611\uce58\uc758 \uc2dc\ud5d8\uc7a5\uc73c\ub85c \uac00\uc7a5 \uba3c\uc800 \ubb34\ub300\uc5d0 \uc624\ub978 \uac83\uc740 \uc774\ub099\uc5f0 \uad6d\ubb34\ucd1d\ub9ac \ud6c4\ubcf4\uc790\uc758 \uc778\uc0ac\uccad\ubb38\ud68c\uc774\ub2e4. \ucde8\uc784\ud558\uc790\ub9d0\uc790 \uc804\ub77c\ub0a8\ub3c4\uc9c0\uc0ac\uc9c1\uc744 \uc218\ud589\ud558\ub358 \uc774\ub099\uc5f0\uc744 \ucd1d\ub9ac \ud6c4\ubcf4\uc790\ub85c \uc9c0\uba85\ud558\uace0 24\uc77c\ubd80\ud130 \uad6d\ud68c\uc5d0\uc11c \uccad\ubb38\ud68c\ub97c \uc9c4\ud589\ud588\ub2e4. \ud55c\uad6d\ub2f9\uacfc \ubc14\ub978\uc815\ub2f9\uc740 \"\ud655\uc2e4\ud55c \uacb0\uaca9 \uc0ac\uc720\uac00 \uc5c6\ub294 \ud55c \ubc1c\ubaa9\uc740 \uc7a1\uc9c0 \uc54a\uaca0\ub2e4\"\uace0 \ubc1d\ud788\uba74\uc11c \uad6d\ud68c\uc758\uc6d0 \ucd9c\uc2e0\uc774\ub77c \uc6b0\ud638\uc801\uc778 \ud3c9\uac00\uac00 \ub098\uc624\uace0 \uc788\uc73c\uba70, \uad6d\ubbfc\uc758\ub2f9 \uc5ed\uc2dc \ud638\ub0a8 \ucd9c\uc2e0\uc778 \uc774\ub099\uc5f0 \ud6c4\ubcf4\uc790\ub97c \ubc18\ub300\ud558\uae30\uc5d0\ub294 \ubd80\ub2f4\uc774 \ub530\ub97c \uac83\uc774\ub77c\ub294 \uad00\uce21\uc774 \ub9ce\uc558\ub2e4. \ud558\uc9c0\ub9cc \ub300\uc120 \ub2f9\uc2dc \ubb38\uc7ac\uc778\uc774 \uc704\uc7a5\uc804\uc785, \ud0c8\uc138, \ubcd1\ub825\uba74\ud0c8, \ub17c\ubb38 \ud45c\uc808, \ubd80\ub3d9\uc0b0 \ud22c\uae30\uc640 \uad00\ub828\ub41c \uc0ac\ub78c\uc740 \uacf5\uc9c1\uc5d0 \uae30\uc6a9\ud558\uc9c0 \uc54a\uaca0\ub2e4\uace0 \ubc1d\ud78c \uc778\uc0ac\uae30\uc900 5\ub300 \uc6d0\uce59 \uc911 \uc704\uc7a5\uc804\uc785, \uc544\ub4e4\uc758 \ubcd1\uc5ed \ub17c\ub780, \uc77c\ubd80 \uc138\uae08 \ud0c8\ub8e8 \ubb38\uc81c\uac00 \ubd88\uac70\uc9c0\uc790 \uc57c\ub2f9\uc758 \ubc18\ub300\uac00 \uc2ec\ud574\uc84c\uace0 \ubbfc\uc8fc\ub2f9\uc774 \uc774\uc5d0 \ub300\ud574 \"\ud611\uce58\uc640 \ub300\ud1b5\ud569\uc758 \uc2dc\uc791\uc5d0 \uc5ec\uc57c \ud560 \uac83 \uc5c6\uc774 \ub300\uc2b9\uc801\uc778 \uacb0\ub2e8\uc744 \ub0b4\ub824\ub2ec\ub77c\"\uace0 \uad6d\uc815\uc758 \uc815\uc0c1\ud654\ub9cc \uac15\uc870\ud558\uba74\uc11c \uc5ec\uc57c\uac00 \ub300\uce58\ud558\ub294 \ud615\uad6d\uc73c\ub85c \ubc14\ub00c\uc5c8\ub2e4. \uc5ec\uae30\uc5d0 \uc678\uad50\ubd80 \uc7a5\uad00\uc73c\ub85c \uc9c0\uba85\ub41c \uac15\uacbd\ud654\uc640 \uacf5\uc815\uac70\ub798\uc704\uc6d0\ud68c \uc704\uc6d0\uc7a5\uc73c\ub85c \uc9c0\uba85\ub41c \uae40\uc0c1\uc870\uc5d0 \ub300\ud574\uc11c\ub3c4 5\ub300 \uae30\uc900\uc5d0 \uc704\ubc30\ub418\ub294 \ubb38\uc81c\uac00 \ub098\ud0c0\ub098\uba74\uc11c \uc815\uad6d\uc774 \uae09\uaca9\ud788 \ub0c9\ub7ad\ud574\uc84c\ub2e4. \uc774\ud6c4 31\uc77c \uc790\uc720\ud55c\uad6d\ub2f9 \uc758\uc6d0\ub4e4\uc774 \ud1f4\uc7a5\ud55c \uac00\uc6b4\ub370 \uc5ec\uc57c \uc758\uc6d0 188\uba85\ub9cc\uc774 \ucc38\uc11d\ud558\uc5ec \uaca8\uc6b0 \uc778\uc900\uc548\uc744 \ud1b5\uacfc\uc2dc\ud0ac \uc218 \uc788\uc5c8\ub2e4. \uc774\ud6c4 \uc6b0\uc6d0\uc2dd \ubbfc\uc8fc\ub2f9 \uc6d0\ub0b4\ub300\ud45c\ub294 \"\uc6b0\uc5ec\uace1\uc808\uc744 \uacaa\uae34 \ud588\uc9c0\ub9cc \ud611\uce58\uac00 \uc911\ub2e8\ub41c \uac83\uc740 \uc544\ub2c8\ub2e4\"\uace0 \ubc1d\ud614\uc9c0\ub9cc \uc815\uc6b0\ud0dd \ud55c\uad6d\ub2f9 \uc6d0\ub0b4\ub300\ud45c\ub294 \"\ubaa8\ub4e0 \ub300\ud45c\ub294 \ub300\ud1b5\ub839\uc5d0\uac8c \uc788\ub2e4. \uc9c0\uae08 \uc0c1\ud0dc\ub860 (\ud611\uce58\uac00) \uc5b4\ub835\ub2e4\"\uace0 \ub2e8\uc5b8\ud588\ub2e4."} {"question": "single ladies\ub294 MTV \ube44\ub514\uc624 \ubba4\uc9c1 \uc5b4\uc6cc\ub4dc\uc5d0\uc11c \uba87 \uac1c\uc758 \uc0c1\uc744 \uc218\uc0c1\ud588\ub294\uac00?", "paragraph": "\u3008Single Ladies\u3009\ub294 MTV \ube44\ub514\uc624 \ubba4\uc9c1 \uc5b4\uc6cc\ub4dc\uc5d0\uc11c 9\ubd80\ubb38\uc5d0 \ud6c4\ubcf4\ub85c \uc62c\ub790\uace0, \ucd5c\uc885\uc801\uc73c\ub85c \uc62c\ud574\uc758 \ube44\ub514\uc624\ub97c \ud3ec\ud568\ud55c 3\uac1c\uc758 \uc0c1\uc744 \uc218\uc0c1\ud588\ub2e4. \ucd5c\uc6b0\uc218 \uc5ec\uc790 \ube44\ub514\uc624 \ubd80\ubb38\uc5d0\uc11c\ub294 \ucee8\ud2b8\ub9ac \ud31d \uac00\uc218\uc778 \ud14c\uc77c\ub7ec \uc2a4\uc704\ud504\ud2b8\uc758 \"You Belong with Me\"\uac00 \ubc1b\uc558\ub294\ub370, \uc2dc\uc0c1\uc2dd \ub3c4\uc911 \ub17c\ub780\uc774 \uc77c\uc5c8\ub2e4. \uc2a4\uc704\ud504\ud2b8\uac00 \uc218\uc0c1\ud558\uace0 \ub098\uc11c \uc218\uc0c1\uc18c\uac10\uc744 \ub9d0\ud560 \ub54c, \uac11\uc790\uae30 \uce74\ub2c8\uc608 \uc6e8\uc2a4\ud2b8\uac00 \ub098\uc640 \ub9c8\uc774\ud06c\ub97c \ube8f\uc5b4 \"\uc5ed\uc0ac\uc0c1 \ucd5c\uace0\uc758 \ubba4\uc9c1\ube44\ub514\uc624 \uc911 \ud558\ub098\ub294 \u3008Single Ladies\u3009\"\ub77c\uace0 \ub9d0\ud558\uba70 \uc218\uc0c1\uc18c\uac10\uc744 \uc911\ub2e8\uc2dc\ucf30\ub2e4. \uc774\ud6c4 \ube44\uc698\uc138\uac00 \uc218\uc0c1\uc18c\uac10\uc744 \ub9d0\ud560 \ub54c \ub370\uc2a4\ud2f0\ub2c8 \ucc28\uc77c\ub4dc\uac00 \ucc98\uc74c\uc73c\ub85c \ube44\ub514\uc624\uc0c1\uc744 \ubc1b\uc558\uc744 \ub2f9\uc2dc\uc758 \ucd94\uc5b5\uc774 \ub5a0\uc624\ub978\ub2e4\uba70 \ud14c\uc77c\ub7ec \uc2a4\uc704\ud504\ud2b8\ub97c \ubb34\ub300\ub85c \ubd88\ub7ec \ubabb\ud588\ub358 \uc18c\uac10\uc744 \ub9d0\ud558\uac8c \ud574\uc92c\ub2e4. \uce74\ub2c8\uc608 \uc6e8\uc2a4\ud2b8\ub294 \uc774 \ud589\ub3d9 \ub54c\ubb38\uc5d0 \ub9ce\uc740 \ube44\ub09c\uc744 \ubc1b\uc558\uace0, \ub098\uc911\uc5d0 \uc0ac\uacfc\ud588\ub2e4. \uc774 \uc0ac\uac74\uc744 \"\uce74\ub2c8\uc608\uac8c\uc774\ud2b8\"\ub77c\uace0 \ubd80\ub978\ub2e4.", "answer": "3\uac1c", "sentence": "\u3008Single Ladies\u3009\ub294 MTV \ube44\ub514\uc624 \ubba4\uc9c1 \uc5b4\uc6cc\ub4dc\uc5d0\uc11c 9\ubd80\ubb38\uc5d0 \ud6c4\ubcf4\ub85c \uc62c\ub790\uace0, \ucd5c\uc885\uc801\uc73c\ub85c \uc62c\ud574\uc758 \ube44\ub514\uc624\ub97c \ud3ec\ud568\ud55c 3\uac1c \uc758 \uc0c1\uc744 \uc218\uc0c1\ud588\ub2e4.", "paragraph_sentence": " \u3008Single Ladies\u3009\ub294 MTV \ube44\ub514\uc624 \ubba4\uc9c1 \uc5b4\uc6cc\ub4dc\uc5d0\uc11c 9\ubd80\ubb38\uc5d0 \ud6c4\ubcf4\ub85c \uc62c\ub790\uace0, \ucd5c\uc885\uc801\uc73c\ub85c \uc62c\ud574\uc758 \ube44\ub514\uc624\ub97c \ud3ec\ud568\ud55c 3\uac1c \uc758 \uc0c1\uc744 \uc218\uc0c1\ud588\ub2e4. \ucd5c\uc6b0\uc218 \uc5ec\uc790 \ube44\ub514\uc624 \ubd80\ubb38\uc5d0\uc11c\ub294 \ucee8\ud2b8\ub9ac \ud31d \uac00\uc218\uc778 \ud14c\uc77c\ub7ec \uc2a4\uc704\ud504\ud2b8\uc758 \"You Belong with Me\"\uac00 \ubc1b\uc558\ub294\ub370, \uc2dc\uc0c1\uc2dd \ub3c4\uc911 \ub17c\ub780\uc774 \uc77c\uc5c8\ub2e4. \uc2a4\uc704\ud504\ud2b8\uac00 \uc218\uc0c1\ud558\uace0 \ub098\uc11c \uc218\uc0c1\uc18c\uac10\uc744 \ub9d0\ud560 \ub54c, \uac11\uc790\uae30 \uce74\ub2c8\uc608 \uc6e8\uc2a4\ud2b8\uac00 \ub098\uc640 \ub9c8\uc774\ud06c\ub97c \ube8f\uc5b4 \"\uc5ed\uc0ac\uc0c1 \ucd5c\uace0\uc758 \ubba4\uc9c1\ube44\ub514\uc624 \uc911 \ud558\ub098\ub294 \u3008Single Ladies\u3009\"\ub77c\uace0 \ub9d0\ud558\uba70 \uc218\uc0c1\uc18c\uac10\uc744 \uc911\ub2e8\uc2dc\ucf30\ub2e4. \uc774\ud6c4 \ube44\uc698\uc138\uac00 \uc218\uc0c1\uc18c\uac10\uc744 \ub9d0\ud560 \ub54c \ub370\uc2a4\ud2f0\ub2c8 \ucc28\uc77c\ub4dc\uac00 \ucc98\uc74c\uc73c\ub85c \ube44\ub514\uc624\uc0c1\uc744 \ubc1b\uc558\uc744 \ub2f9\uc2dc\uc758 \ucd94\uc5b5\uc774 \ub5a0\uc624\ub978\ub2e4\uba70 \ud14c\uc77c\ub7ec \uc2a4\uc704\ud504\ud2b8\ub97c \ubb34\ub300\ub85c \ubd88\ub7ec \ubabb\ud588\ub358 \uc18c\uac10\uc744 \ub9d0\ud558\uac8c \ud574\uc92c\ub2e4. \uce74\ub2c8\uc608 \uc6e8\uc2a4\ud2b8\ub294 \uc774 \ud589\ub3d9 \ub54c\ubb38\uc5d0 \ub9ce\uc740 \ube44\ub09c\uc744 \ubc1b\uc558\uace0, \ub098\uc911\uc5d0 \uc0ac\uacfc\ud588\ub2e4. \uc774 \uc0ac\uac74\uc744 \"\uce74\ub2c8\uc608\uac8c\uc774\ud2b8\"\ub77c\uace0 \ubd80\ub978\ub2e4.", "paragraph_answer": "\u3008Single Ladies\u3009\ub294 MTV \ube44\ub514\uc624 \ubba4\uc9c1 \uc5b4\uc6cc\ub4dc\uc5d0\uc11c 9\ubd80\ubb38\uc5d0 \ud6c4\ubcf4\ub85c \uc62c\ub790\uace0, \ucd5c\uc885\uc801\uc73c\ub85c \uc62c\ud574\uc758 \ube44\ub514\uc624\ub97c \ud3ec\ud568\ud55c 3\uac1c \uc758 \uc0c1\uc744 \uc218\uc0c1\ud588\ub2e4. \ucd5c\uc6b0\uc218 \uc5ec\uc790 \ube44\ub514\uc624 \ubd80\ubb38\uc5d0\uc11c\ub294 \ucee8\ud2b8\ub9ac \ud31d \uac00\uc218\uc778 \ud14c\uc77c\ub7ec \uc2a4\uc704\ud504\ud2b8\uc758 \"You Belong with Me\"\uac00 \ubc1b\uc558\ub294\ub370, \uc2dc\uc0c1\uc2dd \ub3c4\uc911 \ub17c\ub780\uc774 \uc77c\uc5c8\ub2e4. \uc2a4\uc704\ud504\ud2b8\uac00 \uc218\uc0c1\ud558\uace0 \ub098\uc11c \uc218\uc0c1\uc18c\uac10\uc744 \ub9d0\ud560 \ub54c, \uac11\uc790\uae30 \uce74\ub2c8\uc608 \uc6e8\uc2a4\ud2b8\uac00 \ub098\uc640 \ub9c8\uc774\ud06c\ub97c \ube8f\uc5b4 \"\uc5ed\uc0ac\uc0c1 \ucd5c\uace0\uc758 \ubba4\uc9c1\ube44\ub514\uc624 \uc911 \ud558\ub098\ub294 \u3008Single Ladies\u3009\"\ub77c\uace0 \ub9d0\ud558\uba70 \uc218\uc0c1\uc18c\uac10\uc744 \uc911\ub2e8\uc2dc\ucf30\ub2e4. \uc774\ud6c4 \ube44\uc698\uc138\uac00 \uc218\uc0c1\uc18c\uac10\uc744 \ub9d0\ud560 \ub54c \ub370\uc2a4\ud2f0\ub2c8 \ucc28\uc77c\ub4dc\uac00 \ucc98\uc74c\uc73c\ub85c \ube44\ub514\uc624\uc0c1\uc744 \ubc1b\uc558\uc744 \ub2f9\uc2dc\uc758 \ucd94\uc5b5\uc774 \ub5a0\uc624\ub978\ub2e4\uba70 \ud14c\uc77c\ub7ec \uc2a4\uc704\ud504\ud2b8\ub97c \ubb34\ub300\ub85c \ubd88\ub7ec \ubabb\ud588\ub358 \uc18c\uac10\uc744 \ub9d0\ud558\uac8c \ud574\uc92c\ub2e4. \uce74\ub2c8\uc608 \uc6e8\uc2a4\ud2b8\ub294 \uc774 \ud589\ub3d9 \ub54c\ubb38\uc5d0 \ub9ce\uc740 \ube44\ub09c\uc744 \ubc1b\uc558\uace0, \ub098\uc911\uc5d0 \uc0ac\uacfc\ud588\ub2e4. \uc774 \uc0ac\uac74\uc744 \"\uce74\ub2c8\uc608\uac8c\uc774\ud2b8\"\ub77c\uace0 \ubd80\ub978\ub2e4.", "sentence_answer": "\u3008Single Ladies\u3009\ub294 MTV \ube44\ub514\uc624 \ubba4\uc9c1 \uc5b4\uc6cc\ub4dc\uc5d0\uc11c 9\ubd80\ubb38\uc5d0 \ud6c4\ubcf4\ub85c \uc62c\ub790\uace0, \ucd5c\uc885\uc801\uc73c\ub85c \uc62c\ud574\uc758 \ube44\ub514\uc624\ub97c \ud3ec\ud568\ud55c 3\uac1c \uc758 \uc0c1\uc744 \uc218\uc0c1\ud588\ub2e4.", "paragraph_id": "6488992-22-0", "paragraph_question": "question: single ladies\ub294 MTV \ube44\ub514\uc624 \ubba4\uc9c1 \uc5b4\uc6cc\ub4dc\uc5d0\uc11c \uba87 \uac1c\uc758 \uc0c1\uc744 \uc218\uc0c1\ud588\ub294\uac00?, context: \u3008Single Ladies\u3009\ub294 MTV \ube44\ub514\uc624 \ubba4\uc9c1 \uc5b4\uc6cc\ub4dc\uc5d0\uc11c 9\ubd80\ubb38\uc5d0 \ud6c4\ubcf4\ub85c \uc62c\ub790\uace0, \ucd5c\uc885\uc801\uc73c\ub85c \uc62c\ud574\uc758 \ube44\ub514\uc624\ub97c \ud3ec\ud568\ud55c 3\uac1c\uc758 \uc0c1\uc744 \uc218\uc0c1\ud588\ub2e4. \ucd5c\uc6b0\uc218 \uc5ec\uc790 \ube44\ub514\uc624 \ubd80\ubb38\uc5d0\uc11c\ub294 \ucee8\ud2b8\ub9ac \ud31d \uac00\uc218\uc778 \ud14c\uc77c\ub7ec \uc2a4\uc704\ud504\ud2b8\uc758 \"You Belong with Me\"\uac00 \ubc1b\uc558\ub294\ub370, \uc2dc\uc0c1\uc2dd \ub3c4\uc911 \ub17c\ub780\uc774 \uc77c\uc5c8\ub2e4. \uc2a4\uc704\ud504\ud2b8\uac00 \uc218\uc0c1\ud558\uace0 \ub098\uc11c \uc218\uc0c1\uc18c\uac10\uc744 \ub9d0\ud560 \ub54c, \uac11\uc790\uae30 \uce74\ub2c8\uc608 \uc6e8\uc2a4\ud2b8\uac00 \ub098\uc640 \ub9c8\uc774\ud06c\ub97c \ube8f\uc5b4 \"\uc5ed\uc0ac\uc0c1 \ucd5c\uace0\uc758 \ubba4\uc9c1\ube44\ub514\uc624 \uc911 \ud558\ub098\ub294 \u3008Single Ladies\u3009\"\ub77c\uace0 \ub9d0\ud558\uba70 \uc218\uc0c1\uc18c\uac10\uc744 \uc911\ub2e8\uc2dc\ucf30\ub2e4. \uc774\ud6c4 \ube44\uc698\uc138\uac00 \uc218\uc0c1\uc18c\uac10\uc744 \ub9d0\ud560 \ub54c \ub370\uc2a4\ud2f0\ub2c8 \ucc28\uc77c\ub4dc\uac00 \ucc98\uc74c\uc73c\ub85c \ube44\ub514\uc624\uc0c1\uc744 \ubc1b\uc558\uc744 \ub2f9\uc2dc\uc758 \ucd94\uc5b5\uc774 \ub5a0\uc624\ub978\ub2e4\uba70 \ud14c\uc77c\ub7ec \uc2a4\uc704\ud504\ud2b8\ub97c \ubb34\ub300\ub85c \ubd88\ub7ec \ubabb\ud588\ub358 \uc18c\uac10\uc744 \ub9d0\ud558\uac8c \ud574\uc92c\ub2e4. \uce74\ub2c8\uc608 \uc6e8\uc2a4\ud2b8\ub294 \uc774 \ud589\ub3d9 \ub54c\ubb38\uc5d0 \ub9ce\uc740 \ube44\ub09c\uc744 \ubc1b\uc558\uace0, \ub098\uc911\uc5d0 \uc0ac\uacfc\ud588\ub2e4. \uc774 \uc0ac\uac74\uc744 \"\uce74\ub2c8\uc608\uac8c\uc774\ud2b8\"\ub77c\uace0 \ubd80\ub978\ub2e4."} {"question": "\ubd81\ucc3d\uc6d0\uc5ed \uc5ed\uc138\uad8c \uc885\ud569\uac1c\ubc1c \uc0ac\uc5c5\uc5d0 \ub300\ud558\uc5ec \ucd94\uc9c4\ud560 \uac83\uc774 \ubc1c\ud45c\ub41c \ub0a0\uc740?", "paragraph": "\uc774\ud6c4 2007\ub144 9\uc6d4 27\uc77c, (\uac00\uce6d)\ubd81\ucc3d\uc6d0\uc5ed\uc744 \ube44\ub86f\ud55c \uc6a9\ub3d9 \uc77c\uc6d0\uc758 \uaddc\ubaa8\uc758 \ubd81\ucc3d\uc6d0\uc5ed \uc5ed\uc138\uad8c \uc885\ud569\uac1c\ubc1c \uc0ac\uc5c5\uc744 \ucd94\uc9c4\ud560 \uac83\uc744 \ubc1c\ud45c\ud558\uc5ec \uc774\ub0a0 \ubc15\uc644\uc218 \ucc3d\uc6d0\uc2dc\uc7a5\uacfc \uc2e0\ud76c\ubc94 \uacbd\uc0c1\ub0a8\ub3c4\uac1c\ubc1c\uacf5\uc0ac \uc0ac\uc7a5\uc774 \uc0c1\ud638 \ubd84\ub2f4\uc744 \ud1b5\ud558\uc5ec \ud6a8\uc728\uc801\uc73c\ub85c \ucd94\uc9c4\ud560 \uac83\uc744 \uace8\uc790\ub85c \ud558\ub294 \uc2dc\uc815\ud611\uc57d\uc744 \uccb4\uacb0\ud558\uc600\ub2e4. \uc774 \uc885\ud569\uac1c\ubc1c \uc0ac\uc5c5\uc5d0\ub294 \uace0\uc18d\ucca0\ub3c4 \uac1c\ud1b5\uc73c\ub85c \uc608\uc0c1\ub418\ub294 (\uac00\uce6d)\ubd81\ucc3d\uc6d0\uc5ed \uc9c4\uc785\ub85c \uac1c\uc120 \ubb38\uc81c\uc640 \ud568\uaed8, \uc815\ubcd1\uc0b0\uacfc \ube44\uc74c\uc0b0\uc744 \ucc3e\ub294 \uc2dc\ubbfc\ub4e4\uc758 \uc8fc\ucc28\uacf5\uac04\uc744 \ud655\ubcf4\ud558\uace0 \uc5ec\ub7ec \uc0c1\uc5c5, \uacf5\uacf5\uae30\uad00 \ub4f1\uc744 \uc8fc\ubcc0\uc5d0 \ubc30\uce58\ud558\uc5ec \uacf5\uac04\uccb4\uacc4\uc758 \ud6a8\uc728\uc131 \ub4f1\uc744 \uac1c\uc120 \ud560 \uc218 \uc788\ub3c4\ub85d \ud558\uc600\ub2e4. \uad6c\uccb4\uc801\uc73c\ub85c\ub294 \uc5ed\uc774 \uc2e0\uc124\ub418\ub294 \uc6a9\ub3d9\uc800\uc218\uc9c0 \uc77c\uc6d0 295,110m\uc5d0 1,149\uc5b5\uc6d0\uc758 \uc608\uc0b0\uc744 \ud22c\uc785\ud558\uc5ec, \uadf8 \uac00\uc6b4\ub370 \uc808\ubc18\uc5d0 \ud574\ub2f9\ud558\ub294 15\ub9cc 2\ucc9c\uc5ecm\uc5d0 \ub3c4\uc2dc\uae30\ubc18\uc2dc\uc124\uc744, 6\ub9cc 1\ucc9c\uc5ecm\uc740 \uc0c1\uc5c5\uc6a9\uc9c0\ub85c, \ub098\uba38\uc9c0 8\ub9cc 1\ucc9c\uc5ecm\ub294 \uae30\ud0c0\uc2dc\uc124\uc6a9\uc9c0\ub85c \uad6c\ubd84\ud558\uc5ec \uc5ed\uc138\uad8c\uac1c\ubc1c\uc774 \ucd94\uc9c4\ub418\uc5c8\ub2e4. 2007\ub144 \uc2dc\ud589\ud611\uc57d\uc744 \ud588\ub358 \ub2f9\uc2dc\uc5d0 \uc774 \uc0ac\uc5c5\uc740 2010\ub144 \ucd08\uc5d0 \ucc29\uacf5\ud558\uc5ec 2011\ub144 \ub9d0 \uacbd\uc5d0 \uc644\uacf5 \ub420 \uc608\uc815\uc774\uc5c8\uc9c0\ub9cc, \uadf8\ub9b0\ubca8\ud2b8 \ud574\uc81c\uc5d0 \uc2dc\uac04\uc774 \uc624\ub798 \uac78\ub9ac\uba74\uc11c2012\ub144\uc5d0 \ucc29\uacf5\ud558\uc5ec 2014\ub144\uc5d0 \uc644\uacf5\uc73c\ub85c \uacc4\ud68d\uc774 \ubcc0\uacbd\ub418\uc5c8\uace0, \uc0ac\uc5c5 \uc2dc\uae30\ub3c4 2\ub144 \uc774\uc0c1 \ucc28\uc9c8\uc744 \ube5a\uc5c8\ub2e4. \uc774\uc5d0 \ubc15\uc644\uc218 \ucc3d\uc6d0\uc2dc\uc7a5\uc740 \uc5ed\uc138\uad8c \uac1c\ubc1c\uc5d0 \ub300\ud574 \ud2b9\ubcc4\ud55c \ub300\ucc45\uc744 \uac15\uad6c\ud574\uc57c \ud55c\ub2e4\uace0 \ub9d0\ud588\ub2e4.", "answer": "2007\ub144 9\uc6d4 27\uc77c", "sentence": "\uc774\ud6c4 2007\ub144 9\uc6d4 27\uc77c ,", "paragraph_sentence": " \uc774\ud6c4 2007\ub144 9\uc6d4 27\uc77c , (\uac00\uce6d)\ubd81\ucc3d\uc6d0\uc5ed\uc744 \ube44\ub86f\ud55c \uc6a9\ub3d9 \uc77c\uc6d0\uc758 \uaddc\ubaa8\uc758 \ubd81\ucc3d\uc6d0\uc5ed \uc5ed\uc138\uad8c \uc885\ud569\uac1c\ubc1c \uc0ac\uc5c5\uc744 \ucd94\uc9c4\ud560 \uac83\uc744 \ubc1c\ud45c\ud558\uc5ec \uc774\ub0a0 \ubc15\uc644\uc218 \ucc3d\uc6d0\uc2dc\uc7a5\uacfc \uc2e0\ud76c\ubc94 \uacbd\uc0c1\ub0a8\ub3c4\uac1c\ubc1c\uacf5\uc0ac \uc0ac\uc7a5\uc774 \uc0c1\ud638 \ubd84\ub2f4\uc744 \ud1b5\ud558\uc5ec \ud6a8\uc728\uc801\uc73c\ub85c \ucd94\uc9c4\ud560 \uac83\uc744 \uace8\uc790\ub85c \ud558\ub294 \uc2dc\uc815\ud611\uc57d\uc744 \uccb4\uacb0\ud558\uc600\ub2e4. \uc774 \uc885\ud569\uac1c\ubc1c \uc0ac\uc5c5\uc5d0\ub294 \uace0\uc18d\ucca0\ub3c4 \uac1c\ud1b5\uc73c\ub85c \uc608\uc0c1\ub418\ub294 (\uac00\uce6d)\ubd81\ucc3d\uc6d0\uc5ed \uc9c4\uc785\ub85c \uac1c\uc120 \ubb38\uc81c\uc640 \ud568\uaed8, \uc815\ubcd1\uc0b0\uacfc \ube44\uc74c\uc0b0\uc744 \ucc3e\ub294 \uc2dc\ubbfc\ub4e4\uc758 \uc8fc\ucc28\uacf5\uac04\uc744 \ud655\ubcf4\ud558\uace0 \uc5ec\ub7ec \uc0c1\uc5c5, \uacf5\uacf5\uae30\uad00 \ub4f1\uc744 \uc8fc\ubcc0\uc5d0 \ubc30\uce58\ud558\uc5ec \uacf5\uac04\uccb4\uacc4\uc758 \ud6a8\uc728\uc131 \ub4f1\uc744 \uac1c\uc120 \ud560 \uc218 \uc788\ub3c4\ub85d \ud558\uc600\ub2e4. \uad6c\uccb4\uc801\uc73c\ub85c\ub294 \uc5ed\uc774 \uc2e0\uc124\ub418\ub294 \uc6a9\ub3d9\uc800\uc218\uc9c0 \uc77c\uc6d0 295,110m\uc5d0 1,149\uc5b5\uc6d0\uc758 \uc608\uc0b0\uc744 \ud22c\uc785\ud558\uc5ec, \uadf8 \uac00\uc6b4\ub370 \uc808\ubc18\uc5d0 \ud574\ub2f9\ud558\ub294 15\ub9cc 2\ucc9c\uc5ecm\uc5d0 \ub3c4\uc2dc\uae30\ubc18\uc2dc\uc124\uc744, 6\ub9cc 1\ucc9c\uc5ecm\uc740 \uc0c1\uc5c5\uc6a9\uc9c0\ub85c, \ub098\uba38\uc9c0 8\ub9cc 1\ucc9c\uc5ecm\ub294 \uae30\ud0c0\uc2dc\uc124\uc6a9\uc9c0\ub85c \uad6c\ubd84\ud558\uc5ec \uc5ed\uc138\uad8c\uac1c\ubc1c\uc774 \ucd94\uc9c4\ub418\uc5c8\ub2e4. 2007\ub144 \uc2dc\ud589\ud611\uc57d\uc744 \ud588\ub358 \ub2f9\uc2dc\uc5d0 \uc774 \uc0ac\uc5c5\uc740 2010\ub144 \ucd08\uc5d0 \ucc29\uacf5\ud558\uc5ec 2011\ub144 \ub9d0 \uacbd\uc5d0 \uc644\uacf5 \ub420 \uc608\uc815\uc774\uc5c8\uc9c0\ub9cc, \uadf8\ub9b0\ubca8\ud2b8 \ud574\uc81c\uc5d0 \uc2dc\uac04\uc774 \uc624\ub798 \uac78\ub9ac\uba74\uc11c2012\ub144\uc5d0 \ucc29\uacf5\ud558\uc5ec 2014\ub144\uc5d0 \uc644\uacf5\uc73c\ub85c \uacc4\ud68d\uc774 \ubcc0\uacbd\ub418\uc5c8\uace0, \uc0ac\uc5c5 \uc2dc\uae30\ub3c4 2\ub144 \uc774\uc0c1 \ucc28\uc9c8\uc744 \ube5a\uc5c8\ub2e4. \uc774\uc5d0 \ubc15\uc644\uc218 \ucc3d\uc6d0\uc2dc\uc7a5\uc740 \uc5ed\uc138\uad8c \uac1c\ubc1c\uc5d0 \ub300\ud574 \ud2b9\ubcc4\ud55c \ub300\ucc45\uc744 \uac15\uad6c\ud574\uc57c \ud55c\ub2e4\uace0 \ub9d0\ud588\ub2e4.", "paragraph_answer": "\uc774\ud6c4 2007\ub144 9\uc6d4 27\uc77c , (\uac00\uce6d)\ubd81\ucc3d\uc6d0\uc5ed\uc744 \ube44\ub86f\ud55c \uc6a9\ub3d9 \uc77c\uc6d0\uc758 \uaddc\ubaa8\uc758 \ubd81\ucc3d\uc6d0\uc5ed \uc5ed\uc138\uad8c \uc885\ud569\uac1c\ubc1c \uc0ac\uc5c5\uc744 \ucd94\uc9c4\ud560 \uac83\uc744 \ubc1c\ud45c\ud558\uc5ec \uc774\ub0a0 \ubc15\uc644\uc218 \ucc3d\uc6d0\uc2dc\uc7a5\uacfc \uc2e0\ud76c\ubc94 \uacbd\uc0c1\ub0a8\ub3c4\uac1c\ubc1c\uacf5\uc0ac \uc0ac\uc7a5\uc774 \uc0c1\ud638 \ubd84\ub2f4\uc744 \ud1b5\ud558\uc5ec \ud6a8\uc728\uc801\uc73c\ub85c \ucd94\uc9c4\ud560 \uac83\uc744 \uace8\uc790\ub85c \ud558\ub294 \uc2dc\uc815\ud611\uc57d\uc744 \uccb4\uacb0\ud558\uc600\ub2e4. \uc774 \uc885\ud569\uac1c\ubc1c \uc0ac\uc5c5\uc5d0\ub294 \uace0\uc18d\ucca0\ub3c4 \uac1c\ud1b5\uc73c\ub85c \uc608\uc0c1\ub418\ub294 (\uac00\uce6d)\ubd81\ucc3d\uc6d0\uc5ed \uc9c4\uc785\ub85c \uac1c\uc120 \ubb38\uc81c\uc640 \ud568\uaed8, \uc815\ubcd1\uc0b0\uacfc \ube44\uc74c\uc0b0\uc744 \ucc3e\ub294 \uc2dc\ubbfc\ub4e4\uc758 \uc8fc\ucc28\uacf5\uac04\uc744 \ud655\ubcf4\ud558\uace0 \uc5ec\ub7ec \uc0c1\uc5c5, \uacf5\uacf5\uae30\uad00 \ub4f1\uc744 \uc8fc\ubcc0\uc5d0 \ubc30\uce58\ud558\uc5ec \uacf5\uac04\uccb4\uacc4\uc758 \ud6a8\uc728\uc131 \ub4f1\uc744 \uac1c\uc120 \ud560 \uc218 \uc788\ub3c4\ub85d \ud558\uc600\ub2e4. \uad6c\uccb4\uc801\uc73c\ub85c\ub294 \uc5ed\uc774 \uc2e0\uc124\ub418\ub294 \uc6a9\ub3d9\uc800\uc218\uc9c0 \uc77c\uc6d0 295,110m\uc5d0 1,149\uc5b5\uc6d0\uc758 \uc608\uc0b0\uc744 \ud22c\uc785\ud558\uc5ec, \uadf8 \uac00\uc6b4\ub370 \uc808\ubc18\uc5d0 \ud574\ub2f9\ud558\ub294 15\ub9cc 2\ucc9c\uc5ecm\uc5d0 \ub3c4\uc2dc\uae30\ubc18\uc2dc\uc124\uc744, 6\ub9cc 1\ucc9c\uc5ecm\uc740 \uc0c1\uc5c5\uc6a9\uc9c0\ub85c, \ub098\uba38\uc9c0 8\ub9cc 1\ucc9c\uc5ecm\ub294 \uae30\ud0c0\uc2dc\uc124\uc6a9\uc9c0\ub85c \uad6c\ubd84\ud558\uc5ec \uc5ed\uc138\uad8c\uac1c\ubc1c\uc774 \ucd94\uc9c4\ub418\uc5c8\ub2e4. 2007\ub144 \uc2dc\ud589\ud611\uc57d\uc744 \ud588\ub358 \ub2f9\uc2dc\uc5d0 \uc774 \uc0ac\uc5c5\uc740 2010\ub144 \ucd08\uc5d0 \ucc29\uacf5\ud558\uc5ec 2011\ub144 \ub9d0 \uacbd\uc5d0 \uc644\uacf5 \ub420 \uc608\uc815\uc774\uc5c8\uc9c0\ub9cc, \uadf8\ub9b0\ubca8\ud2b8 \ud574\uc81c\uc5d0 \uc2dc\uac04\uc774 \uc624\ub798 \uac78\ub9ac\uba74\uc11c2012\ub144\uc5d0 \ucc29\uacf5\ud558\uc5ec 2014\ub144\uc5d0 \uc644\uacf5\uc73c\ub85c \uacc4\ud68d\uc774 \ubcc0\uacbd\ub418\uc5c8\uace0, \uc0ac\uc5c5 \uc2dc\uae30\ub3c4 2\ub144 \uc774\uc0c1 \ucc28\uc9c8\uc744 \ube5a\uc5c8\ub2e4. \uc774\uc5d0 \ubc15\uc644\uc218 \ucc3d\uc6d0\uc2dc\uc7a5\uc740 \uc5ed\uc138\uad8c \uac1c\ubc1c\uc5d0 \ub300\ud574 \ud2b9\ubcc4\ud55c \ub300\ucc45\uc744 \uac15\uad6c\ud574\uc57c \ud55c\ub2e4\uace0 \ub9d0\ud588\ub2e4.", "sentence_answer": "\uc774\ud6c4 2007\ub144 9\uc6d4 27\uc77c ,", "paragraph_id": "6558007-3-0", "paragraph_question": "question: \ubd81\ucc3d\uc6d0\uc5ed \uc5ed\uc138\uad8c \uc885\ud569\uac1c\ubc1c \uc0ac\uc5c5\uc5d0 \ub300\ud558\uc5ec \ucd94\uc9c4\ud560 \uac83\uc774 \ubc1c\ud45c\ub41c \ub0a0\uc740?, context: \uc774\ud6c4 2007\ub144 9\uc6d4 27\uc77c, (\uac00\uce6d)\ubd81\ucc3d\uc6d0\uc5ed\uc744 \ube44\ub86f\ud55c \uc6a9\ub3d9 \uc77c\uc6d0\uc758 \uaddc\ubaa8\uc758 \ubd81\ucc3d\uc6d0\uc5ed \uc5ed\uc138\uad8c \uc885\ud569\uac1c\ubc1c \uc0ac\uc5c5\uc744 \ucd94\uc9c4\ud560 \uac83\uc744 \ubc1c\ud45c\ud558\uc5ec \uc774\ub0a0 \ubc15\uc644\uc218 \ucc3d\uc6d0\uc2dc\uc7a5\uacfc \uc2e0\ud76c\ubc94 \uacbd\uc0c1\ub0a8\ub3c4\uac1c\ubc1c\uacf5\uc0ac \uc0ac\uc7a5\uc774 \uc0c1\ud638 \ubd84\ub2f4\uc744 \ud1b5\ud558\uc5ec \ud6a8\uc728\uc801\uc73c\ub85c \ucd94\uc9c4\ud560 \uac83\uc744 \uace8\uc790\ub85c \ud558\ub294 \uc2dc\uc815\ud611\uc57d\uc744 \uccb4\uacb0\ud558\uc600\ub2e4. \uc774 \uc885\ud569\uac1c\ubc1c \uc0ac\uc5c5\uc5d0\ub294 \uace0\uc18d\ucca0\ub3c4 \uac1c\ud1b5\uc73c\ub85c \uc608\uc0c1\ub418\ub294 (\uac00\uce6d)\ubd81\ucc3d\uc6d0\uc5ed \uc9c4\uc785\ub85c \uac1c\uc120 \ubb38\uc81c\uc640 \ud568\uaed8, \uc815\ubcd1\uc0b0\uacfc \ube44\uc74c\uc0b0\uc744 \ucc3e\ub294 \uc2dc\ubbfc\ub4e4\uc758 \uc8fc\ucc28\uacf5\uac04\uc744 \ud655\ubcf4\ud558\uace0 \uc5ec\ub7ec \uc0c1\uc5c5, \uacf5\uacf5\uae30\uad00 \ub4f1\uc744 \uc8fc\ubcc0\uc5d0 \ubc30\uce58\ud558\uc5ec \uacf5\uac04\uccb4\uacc4\uc758 \ud6a8\uc728\uc131 \ub4f1\uc744 \uac1c\uc120 \ud560 \uc218 \uc788\ub3c4\ub85d \ud558\uc600\ub2e4. \uad6c\uccb4\uc801\uc73c\ub85c\ub294 \uc5ed\uc774 \uc2e0\uc124\ub418\ub294 \uc6a9\ub3d9\uc800\uc218\uc9c0 \uc77c\uc6d0 295,110m\uc5d0 1,149\uc5b5\uc6d0\uc758 \uc608\uc0b0\uc744 \ud22c\uc785\ud558\uc5ec, \uadf8 \uac00\uc6b4\ub370 \uc808\ubc18\uc5d0 \ud574\ub2f9\ud558\ub294 15\ub9cc 2\ucc9c\uc5ecm\uc5d0 \ub3c4\uc2dc\uae30\ubc18\uc2dc\uc124\uc744, 6\ub9cc 1\ucc9c\uc5ecm\uc740 \uc0c1\uc5c5\uc6a9\uc9c0\ub85c, \ub098\uba38\uc9c0 8\ub9cc 1\ucc9c\uc5ecm\ub294 \uae30\ud0c0\uc2dc\uc124\uc6a9\uc9c0\ub85c \uad6c\ubd84\ud558\uc5ec \uc5ed\uc138\uad8c\uac1c\ubc1c\uc774 \ucd94\uc9c4\ub418\uc5c8\ub2e4. 2007\ub144 \uc2dc\ud589\ud611\uc57d\uc744 \ud588\ub358 \ub2f9\uc2dc\uc5d0 \uc774 \uc0ac\uc5c5\uc740 2010\ub144 \ucd08\uc5d0 \ucc29\uacf5\ud558\uc5ec 2011\ub144 \ub9d0 \uacbd\uc5d0 \uc644\uacf5 \ub420 \uc608\uc815\uc774\uc5c8\uc9c0\ub9cc, \uadf8\ub9b0\ubca8\ud2b8 \ud574\uc81c\uc5d0 \uc2dc\uac04\uc774 \uc624\ub798 \uac78\ub9ac\uba74\uc11c2012\ub144\uc5d0 \ucc29\uacf5\ud558\uc5ec 2014\ub144\uc5d0 \uc644\uacf5\uc73c\ub85c \uacc4\ud68d\uc774 \ubcc0\uacbd\ub418\uc5c8\uace0, \uc0ac\uc5c5 \uc2dc\uae30\ub3c4 2\ub144 \uc774\uc0c1 \ucc28\uc9c8\uc744 \ube5a\uc5c8\ub2e4. \uc774\uc5d0 \ubc15\uc644\uc218 \ucc3d\uc6d0\uc2dc\uc7a5\uc740 \uc5ed\uc138\uad8c \uac1c\ubc1c\uc5d0 \ub300\ud574 \ud2b9\ubcc4\ud55c \ub300\ucc45\uc744 \uac15\uad6c\ud574\uc57c \ud55c\ub2e4\uace0 \ub9d0\ud588\ub2e4."} {"question": "\uc870\uc131\ubbfc\uc774 \ucd5c\uc9c4\uc2e4\uc758 \uc790\ud0dd\uc744 \ucc3e\uc544\uac00 \ud3ed\ud589\ud55c \uac83\uc740 \uba87 \uc2dc\uc600\ub098?", "paragraph": "2004\ub144 8\uc6d4 1\uc77c \uc0c8\ubcbd 3\uc2dc, \uc870\uc131\ubbfc\uc740 \ucd5c\uc9c4\uc2e4\uc758 \uc790\ud0dd\uc744 \ucc3e\uc544\uac00 \ucd5c\uc9c4\uc2e4\uc744 \ud3ed\ud589\ud558\uc600\ub2e4. \ud568\uaed8 \uae30\uac70\ud558\ub358 \ucd5c\uc9c4\uc2e4\uc758 \uc758\uc81c(\u7fa9\u5f1f) \uc774\uc740\uc815\uc774 \ucd5c\uc9c4\uc601\uacfc \uae30\uac70\ud558\ub358 \uc815\uc625\uc219\uc5d0\uac8c \uc5f0\ub77d\uc744 \ud588\uace0 \uc774\uc5d0 \ucd5c\uc9c4\uc601\uacfc \uc815\uc625\uc219\uc774 \ucd5c\uc9c4\uc2e4\uc758 \uc790\ud0dd\uc73c\ub85c \ub2ec\ub824\uc654\ub2e4. \ucd5c\uc9c4\uc601\uc740 \uc870\uc131\ubbfc\uc758 \ud3ed\ub825 \ud589\uc704\ub97c \uc81c\uc9c0\ud558\uae30 \uc704\ud574 \uc9d1 \uc548\uc73c\ub85c \ub4e4\uc5b4\uac14\ub2e4\uac00 \uc790\uc2e0 \ub610\ud55c \ud3ed\ud589\uc744 \ub2f9\ud588\uace0 \uc815\uc625\uc219\uc740 \uacbd\ucc30\uc5d0 \uc2e0\uace0\ub97c \ud588\ub2e4. \uc870\uc131\ubbfc\uc740 \ucd9c\ub3d9\ud55c \uacbd\ucc30\uc5d0 \ud604\ud589\ubc94\uc73c\ub85c \uccb4\ud3ec\ub418\uc5c8\uace0 \uc774 \uc0ac\uac74\uc5d0 \ub300\ud574 \uc30d\ubc29 \ud3ed\ud589\uc774\ub77c\uace0 \uc8fc\uc7a5\ud558\uc600\ub2e4. \uc774 \ud3ed\ub825 \ud589\uc704\ub294 \ucd5c\uc9c4\uc2e4\uc758 \uc601\ud654 \ucd2c\uc601 \ud06c\ub7ad\ud06c\uc778\uc744 \uc55e\ub450\uace0 \ubc1c\uc0dd\ud55c \uc0ac\uac74\uc774\uc5c8\ub2e4. 2004\ub144 8\uc6d4 2\uc77c, \ucd5c\uc9c4\uc2e4\uc740 \uc790\uc2e0\uc758 \uba4d\ub4e0 \uc5bc\uad74\uacfc \ud30c\uc190\ub41c \uc9d1\uc548 \ub0b4\ubd80 \ub4f1\uc744 \uc5b8\ub860\uc5d0 \uacf5\uac1c\ud558\uc600\uace0 \uc774\ud63c\uc5d0 \ub300\ud574\uc11c\ub294 \uae30\uc874 \uc785\uc7a5\uc744 \uacac\uc9c0\ud558\uc600\ub2e4. 2004\ub144 8\uc6d4 6\uc77c, \uc870\uc131\ubbfc\uc758 \ud3ed\ub825 \ud589\uc704\uc640 \uad00\ub828\ud558\uc5ec '\ucd5c\uc9c4\uc2e4\uc758 \uc790\ud0dd \ub4f1\uc5d0 \ub300\ud55c \uc811\uadfc \uae08\uc9c0' \ubc95\uc6d0 \uacb0\uc815\uc774 \ub0b4\ub824\uc84c\ub2e4. 2004\ub144 8\uc6d4 23\uc77c, \uc870\uc131\ubbfc\uc758 \uc0ac\uae30 \ud589\uc704\uc640 \uad00\ub828\ud558\uc5ec \ubc95\uc6d0 \ud310\uacb0\uc774 \ub0b4\ub824\uc84c\ub2e4. 2004\ub144 9\uc6d4 1\uc77c, \ucd5c\uc9c4\uc2e4\uc740 \uc774\ud63c\uc744 \ub9c8\ubb34\ub9ac \uc9c0\uc5c8\ub2e4. 2004\ub144 11\uc6d4 16\uc77c, \uc2e0\ud55c\uc740 \ucd5c\uc9c4\uc2e4\uc744 \uc0c1\ub300\ub85c 30\uc5b5\uc5ec \uc6d0 \uc0c1\ub2f9\uc758 \uc190\ud574\ubc30\uc0c1\uccad\uad6c\uc18c\uc1a1\uc744 \uc81c\uae30\ud558\uc600\ub2e4. \ucd5c\uc9c4\uc2e4\uc740 2004\ub144 3\uc6d4\uc5d0 \uc544\ud30c\ud2b8 \uad11\uace0 \ubaa8\ub378\ub85c\uc11c \uc2e0\ud55c\uacfc \uacc4\uc57d\uc744 \ub9fa\uc5c8\uc5c8\uace0 \uc774\uc5d0 \uc2e0\ud55c\uc740 \ucd5c\uc9c4\uc2e4\uc5d0\uac8c \ubaa8\ub378\ub8cc 2\uc5b5 5000\ub9cc \uc6d0\uc744 \uc9c0\uae09\ud558\uc600\ub2e4. \uc774 \uacc4\uc57d\uc5d0\ub294 \ucd5c\uc9c4\uc2e4\uc774 \uacc4\uc57d \uae30\uac04 \uc911 \uc0ac\ud68c\uc801\u318d\ub3c4\ub355\uc801 \uba85\uc608\ub97c \ud6fc\uc190\ud568\uc73c\ub85c\uc368 \uae30\uc5c5\uc774\ubbf8\uc9c0\ub97c \ud6fc\uc190\ud574\uc11c\ub294 \uc548 \ub418\uba70, \uc774\ub97c \uc704\ubc18\ud574 \uad11\uace0 \ubaa8\ub378\ub85c \ucd9c\uc5f0\ud560 \uc218 \uc5c6\ub294 \uacb0\uaca9 \uc0ac\uc720\uac00 \ubc1c\uc0dd\ud55c \uacbd\uc6b0 \ubaa8\ub378\ub8cc\uc758 200%\uc778 5\uc5b5 \uc6d0\uc744 \uc190\ud574\ubc30\uc0c1\uae08\uc73c\ub85c \ubb3c\ub3c4\ub85d \ud558\ub294 \ub0b4\uc6a9\uc774 \ub2f4\uaca8 \uc788\uc5c8\ub2e4. \uc774\uc5d0 \uadfc\uac70\ud574 \uc2e0\ud55c\uc740 \"\ud3ed\ud589 \uc0ac\uac74 \ubcf4\ub3c4\ub85c \uae30\uc5c5\uc774\ubbf8\uc9c0\uac00 \uc2e4\ucd94\ub3fc \ub9c9\ub300\ud55c \uc190\ud574\ub97c \uc785\uc5c8\ub2e4. \uc190\ud574\ubc30\uc0c1\uae08 5\uc5b5 \uc6d0, \uc9c0\ucd9c\ud55c \uad11\uace0 \ube44\uc6a9 \ub4f1\uc73c\ub85c \ubc1c\uc0dd\ud55c \uc190\ud574\uc561 21\uc5b5 5245\ub9cc \uc6d0, \uae30\uc5c5\uc774\ubbf8\uc9c0 \ud6fc\uc190\uc5d0 \ub530\ub978 \uc704\uc790\ub8cc 4\uc5b5 \uc6d0 \ub4f1 30\uc5b5 5245\ub9cc \uc6d0\uc744 \uc9c0\uae09\ud558\ub77c\"\ub77c\uace0 \ud558\uba70 \ucd5c\uc9c4\uc2e4\uacfc \ucd5c\uc9c4\uc2e4\uc758 \uc18c\uc18d\uc0ac\ub97c \uc0c1\ub300\ub85c \uc18c\uc1a1\uc744 \uc81c\uae30\ud558\uc600\ub2e4. \uc774\uc5d0 \ub300\ud574 \ucd5c\uc9c4\uc2e4\uc740 \"\ub098\ub294 \ud3ed\ud589 \uc0ac\uac74\uc758 \uc77c\ubc29\uc801 \ud53c\ud574\uc790\ub85c\uc11c \uc0ac\ud68c\uc801\u318d\ub3c4\ub355\uc801 \uba85\uc608\ub97c \ud6fc\uc190\ud588\ub2e4\uace0 \ud560 \uc218 \uc5c6\uace0, \ub610 \uc5b8\ub860\uc5d0 \ubcf4\ub3c4\ub41c \uac83\ub3c4 \uae30\uc790\ub4e4\uc774 \uacbd\ucc30\uc11c\uc5d0 \uac00\uc11c \ucde8\uc7ac\ud55c \ud6c4 \ub0b4\uac00 \uc785\uc6d0\ud55c \ubcd1\uc6d0\uc5d0 \ucc3e\uc544\uc640 \uc2ec\uce35 \ucde8\uc7ac\ud55c \uac83\uc77c \ubfd0 \ub0b4\uac8c \uadc0\ucc45\uc0ac\uc720\uac00 \uc5c6\ub2e4\"\ub77c\uace0 \ub300\uc751\ud558\uc600\ub2e4. 2004\ub144 12\uc6d4 9\uc77c, \uc870\uc131\ubbfc\uc758 \ud3ed\ub825 \ud589\uc704\uc5d0 \ub300\ud574 \uc720\uc8c4 \ud310\uacb0\uc774 \ub0b4\ub824\uc84c\ub2e4. 2004\ub144 12\uc6d4 16\uc77c, \ud5e4\ub7f4\ub4dc\uacbd\uc81c \uae30\uc790 \uc11c\ubcd1\uae30\ub294 \uc774 \uc0ac\uc548\uacfc \uad00\ub828\ud558\uc5ec \ud53c\ud574\uc790 \ucd5c\uc9c4\uc2e4\uc744 \uacf5\uaca9\ud588\ub2e4. 2005\ub144 9\uc6d4 18\uc77c, \ub9c8\uc774\ub370\uc77c\ub9ac \uae30\uc790 \ubc30\uad6d\ub0a8\uc740 \uc774 \uc0ac\uc548\uacfc \uad00\ub828\ud558\uc5ec '\uc0ac\uc0dd\ud65c \ud37c\ud3ec\uba3c\uc2a4' \uc6b4\uc6b4 \ud558\uba70 \ucd5c\uc9c4\uc2e4\uc744 \uc870\ub871\ud588\ub2e4. 2007\ub144 11\uc6d4 2\uc77c, \uc11c\ubcd1\uae30\ub294 \ub610 \uc774 \uc0ac\uc548\uacfc \uad00\ub828\ud558\uc5ec '\ucd94\uc7a1' \uc6b4\uc6b4 \ud558\uba70 \ucd5c\uc9c4\uc2e4\uc744 \uc870\ub871\ud588\ub2e4.", "answer": "\uc0c8\ubcbd 3\uc2dc", "sentence": "2004\ub144 8\uc6d4 1\uc77c \uc0c8\ubcbd 3\uc2dc ,", "paragraph_sentence": " 2004\ub144 8\uc6d4 1\uc77c \uc0c8\ubcbd 3\uc2dc , \uc870\uc131\ubbfc\uc740 \ucd5c\uc9c4\uc2e4\uc758 \uc790\ud0dd\uc744 \ucc3e\uc544\uac00 \ucd5c\uc9c4\uc2e4\uc744 \ud3ed\ud589\ud558\uc600\ub2e4. \ud568\uaed8 \uae30\uac70\ud558\ub358 \ucd5c\uc9c4\uc2e4\uc758 \uc758\uc81c(\u7fa9\u5f1f) \uc774\uc740\uc815\uc774 \ucd5c\uc9c4\uc601\uacfc \uae30\uac70\ud558\ub358 \uc815\uc625\uc219\uc5d0\uac8c \uc5f0\ub77d\uc744 \ud588\uace0 \uc774\uc5d0 \ucd5c\uc9c4\uc601\uacfc \uc815\uc625\uc219\uc774 \ucd5c\uc9c4\uc2e4\uc758 \uc790\ud0dd\uc73c\ub85c \ub2ec\ub824\uc654\ub2e4. \ucd5c\uc9c4\uc601\uc740 \uc870\uc131\ubbfc\uc758 \ud3ed\ub825 \ud589\uc704\ub97c \uc81c\uc9c0\ud558\uae30 \uc704\ud574 \uc9d1 \uc548\uc73c\ub85c \ub4e4\uc5b4\uac14\ub2e4\uac00 \uc790\uc2e0 \ub610\ud55c \ud3ed\ud589\uc744 \ub2f9\ud588\uace0 \uc815\uc625\uc219\uc740 \uacbd\ucc30\uc5d0 \uc2e0\uace0\ub97c \ud588\ub2e4. \uc870\uc131\ubbfc\uc740 \ucd9c\ub3d9\ud55c \uacbd\ucc30\uc5d0 \ud604\ud589\ubc94\uc73c\ub85c \uccb4\ud3ec\ub418\uc5c8\uace0 \uc774 \uc0ac\uac74\uc5d0 \ub300\ud574 \uc30d\ubc29 \ud3ed\ud589\uc774\ub77c\uace0 \uc8fc\uc7a5\ud558\uc600\ub2e4. \uc774 \ud3ed\ub825 \ud589\uc704\ub294 \ucd5c\uc9c4\uc2e4\uc758 \uc601\ud654 \ucd2c\uc601 \ud06c\ub7ad\ud06c\uc778\uc744 \uc55e\ub450\uace0 \ubc1c\uc0dd\ud55c \uc0ac\uac74\uc774\uc5c8\ub2e4. 2004\ub144 8\uc6d4 2\uc77c, \ucd5c\uc9c4\uc2e4\uc740 \uc790\uc2e0\uc758 \uba4d\ub4e0 \uc5bc\uad74\uacfc \ud30c\uc190\ub41c \uc9d1\uc548 \ub0b4\ubd80 \ub4f1\uc744 \uc5b8\ub860\uc5d0 \uacf5\uac1c\ud558\uc600\uace0 \uc774\ud63c\uc5d0 \ub300\ud574\uc11c\ub294 \uae30\uc874 \uc785\uc7a5\uc744 \uacac\uc9c0\ud558\uc600\ub2e4. 2004\ub144 8\uc6d4 6\uc77c, \uc870\uc131\ubbfc\uc758 \ud3ed\ub825 \ud589\uc704\uc640 \uad00\ub828\ud558\uc5ec '\ucd5c\uc9c4\uc2e4\uc758 \uc790\ud0dd \ub4f1\uc5d0 \ub300\ud55c \uc811\uadfc \uae08\uc9c0' \ubc95\uc6d0 \uacb0\uc815\uc774 \ub0b4\ub824\uc84c\ub2e4. 2004\ub144 8\uc6d4 23\uc77c, \uc870\uc131\ubbfc\uc758 \uc0ac\uae30 \ud589\uc704\uc640 \uad00\ub828\ud558\uc5ec \ubc95\uc6d0 \ud310\uacb0\uc774 \ub0b4\ub824\uc84c\ub2e4. 2004\ub144 9\uc6d4 1\uc77c, \ucd5c\uc9c4\uc2e4\uc740 \uc774\ud63c\uc744 \ub9c8\ubb34\ub9ac \uc9c0\uc5c8\ub2e4. 2004\ub144 11\uc6d4 16\uc77c, \uc2e0\ud55c\uc740 \ucd5c\uc9c4\uc2e4\uc744 \uc0c1\ub300\ub85c 30\uc5b5\uc5ec \uc6d0 \uc0c1\ub2f9\uc758 \uc190\ud574\ubc30\uc0c1\uccad\uad6c\uc18c\uc1a1\uc744 \uc81c\uae30\ud558\uc600\ub2e4. \ucd5c\uc9c4\uc2e4\uc740 2004\ub144 3\uc6d4\uc5d0 \uc544\ud30c\ud2b8 \uad11\uace0 \ubaa8\ub378\ub85c\uc11c \uc2e0\ud55c\uacfc \uacc4\uc57d\uc744 \ub9fa\uc5c8\uc5c8\uace0 \uc774\uc5d0 \uc2e0\ud55c\uc740 \ucd5c\uc9c4\uc2e4\uc5d0\uac8c \ubaa8\ub378\ub8cc 2\uc5b5 5000\ub9cc \uc6d0\uc744 \uc9c0\uae09\ud558\uc600\ub2e4. \uc774 \uacc4\uc57d\uc5d0\ub294 \ucd5c\uc9c4\uc2e4\uc774 \uacc4\uc57d \uae30\uac04 \uc911 \uc0ac\ud68c\uc801\u318d\ub3c4\ub355\uc801 \uba85\uc608\ub97c \ud6fc\uc190\ud568\uc73c\ub85c\uc368 \uae30\uc5c5\uc774\ubbf8\uc9c0\ub97c \ud6fc\uc190\ud574\uc11c\ub294 \uc548 \ub418\uba70, \uc774\ub97c \uc704\ubc18\ud574 \uad11\uace0 \ubaa8\ub378\ub85c \ucd9c\uc5f0\ud560 \uc218 \uc5c6\ub294 \uacb0\uaca9 \uc0ac\uc720\uac00 \ubc1c\uc0dd\ud55c \uacbd\uc6b0 \ubaa8\ub378\ub8cc\uc758 200%\uc778 5\uc5b5 \uc6d0\uc744 \uc190\ud574\ubc30\uc0c1\uae08\uc73c\ub85c \ubb3c\ub3c4\ub85d \ud558\ub294 \ub0b4\uc6a9\uc774 \ub2f4\uaca8 \uc788\uc5c8\ub2e4. \uc774\uc5d0 \uadfc\uac70\ud574 \uc2e0\ud55c\uc740 \"\ud3ed\ud589 \uc0ac\uac74 \ubcf4\ub3c4\ub85c \uae30\uc5c5\uc774\ubbf8\uc9c0\uac00 \uc2e4\ucd94\ub3fc \ub9c9\ub300\ud55c \uc190\ud574\ub97c \uc785\uc5c8\ub2e4. \uc190\ud574\ubc30\uc0c1\uae08 5\uc5b5 \uc6d0, \uc9c0\ucd9c\ud55c \uad11\uace0 \ube44\uc6a9 \ub4f1\uc73c\ub85c \ubc1c\uc0dd\ud55c \uc190\ud574\uc561 21\uc5b5 5245\ub9cc \uc6d0, \uae30\uc5c5\uc774\ubbf8\uc9c0 \ud6fc\uc190\uc5d0 \ub530\ub978 \uc704\uc790\ub8cc 4\uc5b5 \uc6d0 \ub4f1 30\uc5b5 5245\ub9cc \uc6d0\uc744 \uc9c0\uae09\ud558\ub77c\"\ub77c\uace0 \ud558\uba70 \ucd5c\uc9c4\uc2e4\uacfc \ucd5c\uc9c4\uc2e4\uc758 \uc18c\uc18d\uc0ac\ub97c \uc0c1\ub300\ub85c \uc18c\uc1a1\uc744 \uc81c\uae30\ud558\uc600\ub2e4. \uc774\uc5d0 \ub300\ud574 \ucd5c\uc9c4\uc2e4\uc740 \"\ub098\ub294 \ud3ed\ud589 \uc0ac\uac74\uc758 \uc77c\ubc29\uc801 \ud53c\ud574\uc790\ub85c\uc11c \uc0ac\ud68c\uc801\u318d\ub3c4\ub355\uc801 \uba85\uc608\ub97c \ud6fc\uc190\ud588\ub2e4\uace0 \ud560 \uc218 \uc5c6\uace0, \ub610 \uc5b8\ub860\uc5d0 \ubcf4\ub3c4\ub41c \uac83\ub3c4 \uae30\uc790\ub4e4\uc774 \uacbd\ucc30\uc11c\uc5d0 \uac00\uc11c \ucde8\uc7ac\ud55c \ud6c4 \ub0b4\uac00 \uc785\uc6d0\ud55c \ubcd1\uc6d0\uc5d0 \ucc3e\uc544\uc640 \uc2ec\uce35 \ucde8\uc7ac\ud55c \uac83\uc77c \ubfd0 \ub0b4\uac8c \uadc0\ucc45\uc0ac\uc720\uac00 \uc5c6\ub2e4\"\ub77c\uace0 \ub300\uc751\ud558\uc600\ub2e4. 2004\ub144 12\uc6d4 9\uc77c, \uc870\uc131\ubbfc\uc758 \ud3ed\ub825 \ud589\uc704\uc5d0 \ub300\ud574 \uc720\uc8c4 \ud310\uacb0\uc774 \ub0b4\ub824\uc84c\ub2e4. 2004\ub144 12\uc6d4 16\uc77c, \ud5e4\ub7f4\ub4dc\uacbd\uc81c \uae30\uc790 \uc11c\ubcd1\uae30\ub294 \uc774 \uc0ac\uc548\uacfc \uad00\ub828\ud558\uc5ec \ud53c\ud574\uc790 \ucd5c\uc9c4\uc2e4\uc744 \uacf5\uaca9\ud588\ub2e4. 2005\ub144 9\uc6d4 18\uc77c, \ub9c8\uc774\ub370\uc77c\ub9ac \uae30\uc790 \ubc30\uad6d\ub0a8\uc740 \uc774 \uc0ac\uc548\uacfc \uad00\ub828\ud558\uc5ec '\uc0ac\uc0dd\ud65c \ud37c\ud3ec\uba3c\uc2a4' \uc6b4\uc6b4 \ud558\uba70 \ucd5c\uc9c4\uc2e4\uc744 \uc870\ub871\ud588\ub2e4. 2007\ub144 11\uc6d4 2\uc77c, \uc11c\ubcd1\uae30\ub294 \ub610 \uc774 \uc0ac\uc548\uacfc \uad00\ub828\ud558\uc5ec '\ucd94\uc7a1' \uc6b4\uc6b4 \ud558\uba70 \ucd5c\uc9c4\uc2e4\uc744 \uc870\ub871\ud588\ub2e4.", "paragraph_answer": "2004\ub144 8\uc6d4 1\uc77c \uc0c8\ubcbd 3\uc2dc , \uc870\uc131\ubbfc\uc740 \ucd5c\uc9c4\uc2e4\uc758 \uc790\ud0dd\uc744 \ucc3e\uc544\uac00 \ucd5c\uc9c4\uc2e4\uc744 \ud3ed\ud589\ud558\uc600\ub2e4. \ud568\uaed8 \uae30\uac70\ud558\ub358 \ucd5c\uc9c4\uc2e4\uc758 \uc758\uc81c(\u7fa9\u5f1f) \uc774\uc740\uc815\uc774 \ucd5c\uc9c4\uc601\uacfc \uae30\uac70\ud558\ub358 \uc815\uc625\uc219\uc5d0\uac8c \uc5f0\ub77d\uc744 \ud588\uace0 \uc774\uc5d0 \ucd5c\uc9c4\uc601\uacfc \uc815\uc625\uc219\uc774 \ucd5c\uc9c4\uc2e4\uc758 \uc790\ud0dd\uc73c\ub85c \ub2ec\ub824\uc654\ub2e4. \ucd5c\uc9c4\uc601\uc740 \uc870\uc131\ubbfc\uc758 \ud3ed\ub825 \ud589\uc704\ub97c \uc81c\uc9c0\ud558\uae30 \uc704\ud574 \uc9d1 \uc548\uc73c\ub85c \ub4e4\uc5b4\uac14\ub2e4\uac00 \uc790\uc2e0 \ub610\ud55c \ud3ed\ud589\uc744 \ub2f9\ud588\uace0 \uc815\uc625\uc219\uc740 \uacbd\ucc30\uc5d0 \uc2e0\uace0\ub97c \ud588\ub2e4. \uc870\uc131\ubbfc\uc740 \ucd9c\ub3d9\ud55c \uacbd\ucc30\uc5d0 \ud604\ud589\ubc94\uc73c\ub85c \uccb4\ud3ec\ub418\uc5c8\uace0 \uc774 \uc0ac\uac74\uc5d0 \ub300\ud574 \uc30d\ubc29 \ud3ed\ud589\uc774\ub77c\uace0 \uc8fc\uc7a5\ud558\uc600\ub2e4. \uc774 \ud3ed\ub825 \ud589\uc704\ub294 \ucd5c\uc9c4\uc2e4\uc758 \uc601\ud654 \ucd2c\uc601 \ud06c\ub7ad\ud06c\uc778\uc744 \uc55e\ub450\uace0 \ubc1c\uc0dd\ud55c \uc0ac\uac74\uc774\uc5c8\ub2e4. 2004\ub144 8\uc6d4 2\uc77c, \ucd5c\uc9c4\uc2e4\uc740 \uc790\uc2e0\uc758 \uba4d\ub4e0 \uc5bc\uad74\uacfc \ud30c\uc190\ub41c \uc9d1\uc548 \ub0b4\ubd80 \ub4f1\uc744 \uc5b8\ub860\uc5d0 \uacf5\uac1c\ud558\uc600\uace0 \uc774\ud63c\uc5d0 \ub300\ud574\uc11c\ub294 \uae30\uc874 \uc785\uc7a5\uc744 \uacac\uc9c0\ud558\uc600\ub2e4. 2004\ub144 8\uc6d4 6\uc77c, \uc870\uc131\ubbfc\uc758 \ud3ed\ub825 \ud589\uc704\uc640 \uad00\ub828\ud558\uc5ec '\ucd5c\uc9c4\uc2e4\uc758 \uc790\ud0dd \ub4f1\uc5d0 \ub300\ud55c \uc811\uadfc \uae08\uc9c0' \ubc95\uc6d0 \uacb0\uc815\uc774 \ub0b4\ub824\uc84c\ub2e4. 2004\ub144 8\uc6d4 23\uc77c, \uc870\uc131\ubbfc\uc758 \uc0ac\uae30 \ud589\uc704\uc640 \uad00\ub828\ud558\uc5ec \ubc95\uc6d0 \ud310\uacb0\uc774 \ub0b4\ub824\uc84c\ub2e4. 2004\ub144 9\uc6d4 1\uc77c, \ucd5c\uc9c4\uc2e4\uc740 \uc774\ud63c\uc744 \ub9c8\ubb34\ub9ac \uc9c0\uc5c8\ub2e4. 2004\ub144 11\uc6d4 16\uc77c, \uc2e0\ud55c\uc740 \ucd5c\uc9c4\uc2e4\uc744 \uc0c1\ub300\ub85c 30\uc5b5\uc5ec \uc6d0 \uc0c1\ub2f9\uc758 \uc190\ud574\ubc30\uc0c1\uccad\uad6c\uc18c\uc1a1\uc744 \uc81c\uae30\ud558\uc600\ub2e4. \ucd5c\uc9c4\uc2e4\uc740 2004\ub144 3\uc6d4\uc5d0 \uc544\ud30c\ud2b8 \uad11\uace0 \ubaa8\ub378\ub85c\uc11c \uc2e0\ud55c\uacfc \uacc4\uc57d\uc744 \ub9fa\uc5c8\uc5c8\uace0 \uc774\uc5d0 \uc2e0\ud55c\uc740 \ucd5c\uc9c4\uc2e4\uc5d0\uac8c \ubaa8\ub378\ub8cc 2\uc5b5 5000\ub9cc \uc6d0\uc744 \uc9c0\uae09\ud558\uc600\ub2e4. \uc774 \uacc4\uc57d\uc5d0\ub294 \ucd5c\uc9c4\uc2e4\uc774 \uacc4\uc57d \uae30\uac04 \uc911 \uc0ac\ud68c\uc801\u318d\ub3c4\ub355\uc801 \uba85\uc608\ub97c \ud6fc\uc190\ud568\uc73c\ub85c\uc368 \uae30\uc5c5\uc774\ubbf8\uc9c0\ub97c \ud6fc\uc190\ud574\uc11c\ub294 \uc548 \ub418\uba70, \uc774\ub97c \uc704\ubc18\ud574 \uad11\uace0 \ubaa8\ub378\ub85c \ucd9c\uc5f0\ud560 \uc218 \uc5c6\ub294 \uacb0\uaca9 \uc0ac\uc720\uac00 \ubc1c\uc0dd\ud55c \uacbd\uc6b0 \ubaa8\ub378\ub8cc\uc758 200%\uc778 5\uc5b5 \uc6d0\uc744 \uc190\ud574\ubc30\uc0c1\uae08\uc73c\ub85c \ubb3c\ub3c4\ub85d \ud558\ub294 \ub0b4\uc6a9\uc774 \ub2f4\uaca8 \uc788\uc5c8\ub2e4. \uc774\uc5d0 \uadfc\uac70\ud574 \uc2e0\ud55c\uc740 \"\ud3ed\ud589 \uc0ac\uac74 \ubcf4\ub3c4\ub85c \uae30\uc5c5\uc774\ubbf8\uc9c0\uac00 \uc2e4\ucd94\ub3fc \ub9c9\ub300\ud55c \uc190\ud574\ub97c \uc785\uc5c8\ub2e4. \uc190\ud574\ubc30\uc0c1\uae08 5\uc5b5 \uc6d0, \uc9c0\ucd9c\ud55c \uad11\uace0 \ube44\uc6a9 \ub4f1\uc73c\ub85c \ubc1c\uc0dd\ud55c \uc190\ud574\uc561 21\uc5b5 5245\ub9cc \uc6d0, \uae30\uc5c5\uc774\ubbf8\uc9c0 \ud6fc\uc190\uc5d0 \ub530\ub978 \uc704\uc790\ub8cc 4\uc5b5 \uc6d0 \ub4f1 30\uc5b5 5245\ub9cc \uc6d0\uc744 \uc9c0\uae09\ud558\ub77c\"\ub77c\uace0 \ud558\uba70 \ucd5c\uc9c4\uc2e4\uacfc \ucd5c\uc9c4\uc2e4\uc758 \uc18c\uc18d\uc0ac\ub97c \uc0c1\ub300\ub85c \uc18c\uc1a1\uc744 \uc81c\uae30\ud558\uc600\ub2e4. \uc774\uc5d0 \ub300\ud574 \ucd5c\uc9c4\uc2e4\uc740 \"\ub098\ub294 \ud3ed\ud589 \uc0ac\uac74\uc758 \uc77c\ubc29\uc801 \ud53c\ud574\uc790\ub85c\uc11c \uc0ac\ud68c\uc801\u318d\ub3c4\ub355\uc801 \uba85\uc608\ub97c \ud6fc\uc190\ud588\ub2e4\uace0 \ud560 \uc218 \uc5c6\uace0, \ub610 \uc5b8\ub860\uc5d0 \ubcf4\ub3c4\ub41c \uac83\ub3c4 \uae30\uc790\ub4e4\uc774 \uacbd\ucc30\uc11c\uc5d0 \uac00\uc11c \ucde8\uc7ac\ud55c \ud6c4 \ub0b4\uac00 \uc785\uc6d0\ud55c \ubcd1\uc6d0\uc5d0 \ucc3e\uc544\uc640 \uc2ec\uce35 \ucde8\uc7ac\ud55c \uac83\uc77c \ubfd0 \ub0b4\uac8c \uadc0\ucc45\uc0ac\uc720\uac00 \uc5c6\ub2e4\"\ub77c\uace0 \ub300\uc751\ud558\uc600\ub2e4. 2004\ub144 12\uc6d4 9\uc77c, \uc870\uc131\ubbfc\uc758 \ud3ed\ub825 \ud589\uc704\uc5d0 \ub300\ud574 \uc720\uc8c4 \ud310\uacb0\uc774 \ub0b4\ub824\uc84c\ub2e4. 2004\ub144 12\uc6d4 16\uc77c, \ud5e4\ub7f4\ub4dc\uacbd\uc81c \uae30\uc790 \uc11c\ubcd1\uae30\ub294 \uc774 \uc0ac\uc548\uacfc \uad00\ub828\ud558\uc5ec \ud53c\ud574\uc790 \ucd5c\uc9c4\uc2e4\uc744 \uacf5\uaca9\ud588\ub2e4. 2005\ub144 9\uc6d4 18\uc77c, \ub9c8\uc774\ub370\uc77c\ub9ac \uae30\uc790 \ubc30\uad6d\ub0a8\uc740 \uc774 \uc0ac\uc548\uacfc \uad00\ub828\ud558\uc5ec '\uc0ac\uc0dd\ud65c \ud37c\ud3ec\uba3c\uc2a4' \uc6b4\uc6b4 \ud558\uba70 \ucd5c\uc9c4\uc2e4\uc744 \uc870\ub871\ud588\ub2e4. 2007\ub144 11\uc6d4 2\uc77c, \uc11c\ubcd1\uae30\ub294 \ub610 \uc774 \uc0ac\uc548\uacfc \uad00\ub828\ud558\uc5ec '\ucd94\uc7a1' \uc6b4\uc6b4 \ud558\uba70 \ucd5c\uc9c4\uc2e4\uc744 \uc870\ub871\ud588\ub2e4.", "sentence_answer": "2004\ub144 8\uc6d4 1\uc77c \uc0c8\ubcbd 3\uc2dc ,", "paragraph_id": "6560245-13-0", "paragraph_question": "question: \uc870\uc131\ubbfc\uc774 \ucd5c\uc9c4\uc2e4\uc758 \uc790\ud0dd\uc744 \ucc3e\uc544\uac00 \ud3ed\ud589\ud55c \uac83\uc740 \uba87 \uc2dc\uc600\ub098?, context: 2004\ub144 8\uc6d4 1\uc77c \uc0c8\ubcbd 3\uc2dc, \uc870\uc131\ubbfc\uc740 \ucd5c\uc9c4\uc2e4\uc758 \uc790\ud0dd\uc744 \ucc3e\uc544\uac00 \ucd5c\uc9c4\uc2e4\uc744 \ud3ed\ud589\ud558\uc600\ub2e4. \ud568\uaed8 \uae30\uac70\ud558\ub358 \ucd5c\uc9c4\uc2e4\uc758 \uc758\uc81c(\u7fa9\u5f1f) \uc774\uc740\uc815\uc774 \ucd5c\uc9c4\uc601\uacfc \uae30\uac70\ud558\ub358 \uc815\uc625\uc219\uc5d0\uac8c \uc5f0\ub77d\uc744 \ud588\uace0 \uc774\uc5d0 \ucd5c\uc9c4\uc601\uacfc \uc815\uc625\uc219\uc774 \ucd5c\uc9c4\uc2e4\uc758 \uc790\ud0dd\uc73c\ub85c \ub2ec\ub824\uc654\ub2e4. \ucd5c\uc9c4\uc601\uc740 \uc870\uc131\ubbfc\uc758 \ud3ed\ub825 \ud589\uc704\ub97c \uc81c\uc9c0\ud558\uae30 \uc704\ud574 \uc9d1 \uc548\uc73c\ub85c \ub4e4\uc5b4\uac14\ub2e4\uac00 \uc790\uc2e0 \ub610\ud55c \ud3ed\ud589\uc744 \ub2f9\ud588\uace0 \uc815\uc625\uc219\uc740 \uacbd\ucc30\uc5d0 \uc2e0\uace0\ub97c \ud588\ub2e4. \uc870\uc131\ubbfc\uc740 \ucd9c\ub3d9\ud55c \uacbd\ucc30\uc5d0 \ud604\ud589\ubc94\uc73c\ub85c \uccb4\ud3ec\ub418\uc5c8\uace0 \uc774 \uc0ac\uac74\uc5d0 \ub300\ud574 \uc30d\ubc29 \ud3ed\ud589\uc774\ub77c\uace0 \uc8fc\uc7a5\ud558\uc600\ub2e4. \uc774 \ud3ed\ub825 \ud589\uc704\ub294 \ucd5c\uc9c4\uc2e4\uc758 \uc601\ud654 \ucd2c\uc601 \ud06c\ub7ad\ud06c\uc778\uc744 \uc55e\ub450\uace0 \ubc1c\uc0dd\ud55c \uc0ac\uac74\uc774\uc5c8\ub2e4. 2004\ub144 8\uc6d4 2\uc77c, \ucd5c\uc9c4\uc2e4\uc740 \uc790\uc2e0\uc758 \uba4d\ub4e0 \uc5bc\uad74\uacfc \ud30c\uc190\ub41c \uc9d1\uc548 \ub0b4\ubd80 \ub4f1\uc744 \uc5b8\ub860\uc5d0 \uacf5\uac1c\ud558\uc600\uace0 \uc774\ud63c\uc5d0 \ub300\ud574\uc11c\ub294 \uae30\uc874 \uc785\uc7a5\uc744 \uacac\uc9c0\ud558\uc600\ub2e4. 2004\ub144 8\uc6d4 6\uc77c, \uc870\uc131\ubbfc\uc758 \ud3ed\ub825 \ud589\uc704\uc640 \uad00\ub828\ud558\uc5ec '\ucd5c\uc9c4\uc2e4\uc758 \uc790\ud0dd \ub4f1\uc5d0 \ub300\ud55c \uc811\uadfc \uae08\uc9c0' \ubc95\uc6d0 \uacb0\uc815\uc774 \ub0b4\ub824\uc84c\ub2e4. 2004\ub144 8\uc6d4 23\uc77c, \uc870\uc131\ubbfc\uc758 \uc0ac\uae30 \ud589\uc704\uc640 \uad00\ub828\ud558\uc5ec \ubc95\uc6d0 \ud310\uacb0\uc774 \ub0b4\ub824\uc84c\ub2e4. 2004\ub144 9\uc6d4 1\uc77c, \ucd5c\uc9c4\uc2e4\uc740 \uc774\ud63c\uc744 \ub9c8\ubb34\ub9ac \uc9c0\uc5c8\ub2e4. 2004\ub144 11\uc6d4 16\uc77c, \uc2e0\ud55c\uc740 \ucd5c\uc9c4\uc2e4\uc744 \uc0c1\ub300\ub85c 30\uc5b5\uc5ec \uc6d0 \uc0c1\ub2f9\uc758 \uc190\ud574\ubc30\uc0c1\uccad\uad6c\uc18c\uc1a1\uc744 \uc81c\uae30\ud558\uc600\ub2e4. \ucd5c\uc9c4\uc2e4\uc740 2004\ub144 3\uc6d4\uc5d0 \uc544\ud30c\ud2b8 \uad11\uace0 \ubaa8\ub378\ub85c\uc11c \uc2e0\ud55c\uacfc \uacc4\uc57d\uc744 \ub9fa\uc5c8\uc5c8\uace0 \uc774\uc5d0 \uc2e0\ud55c\uc740 \ucd5c\uc9c4\uc2e4\uc5d0\uac8c \ubaa8\ub378\ub8cc 2\uc5b5 5000\ub9cc \uc6d0\uc744 \uc9c0\uae09\ud558\uc600\ub2e4. \uc774 \uacc4\uc57d\uc5d0\ub294 \ucd5c\uc9c4\uc2e4\uc774 \uacc4\uc57d \uae30\uac04 \uc911 \uc0ac\ud68c\uc801\u318d\ub3c4\ub355\uc801 \uba85\uc608\ub97c \ud6fc\uc190\ud568\uc73c\ub85c\uc368 \uae30\uc5c5\uc774\ubbf8\uc9c0\ub97c \ud6fc\uc190\ud574\uc11c\ub294 \uc548 \ub418\uba70, \uc774\ub97c \uc704\ubc18\ud574 \uad11\uace0 \ubaa8\ub378\ub85c \ucd9c\uc5f0\ud560 \uc218 \uc5c6\ub294 \uacb0\uaca9 \uc0ac\uc720\uac00 \ubc1c\uc0dd\ud55c \uacbd\uc6b0 \ubaa8\ub378\ub8cc\uc758 200%\uc778 5\uc5b5 \uc6d0\uc744 \uc190\ud574\ubc30\uc0c1\uae08\uc73c\ub85c \ubb3c\ub3c4\ub85d \ud558\ub294 \ub0b4\uc6a9\uc774 \ub2f4\uaca8 \uc788\uc5c8\ub2e4. \uc774\uc5d0 \uadfc\uac70\ud574 \uc2e0\ud55c\uc740 \"\ud3ed\ud589 \uc0ac\uac74 \ubcf4\ub3c4\ub85c \uae30\uc5c5\uc774\ubbf8\uc9c0\uac00 \uc2e4\ucd94\ub3fc \ub9c9\ub300\ud55c \uc190\ud574\ub97c \uc785\uc5c8\ub2e4. \uc190\ud574\ubc30\uc0c1\uae08 5\uc5b5 \uc6d0, \uc9c0\ucd9c\ud55c \uad11\uace0 \ube44\uc6a9 \ub4f1\uc73c\ub85c \ubc1c\uc0dd\ud55c \uc190\ud574\uc561 21\uc5b5 5245\ub9cc \uc6d0, \uae30\uc5c5\uc774\ubbf8\uc9c0 \ud6fc\uc190\uc5d0 \ub530\ub978 \uc704\uc790\ub8cc 4\uc5b5 \uc6d0 \ub4f1 30\uc5b5 5245\ub9cc \uc6d0\uc744 \uc9c0\uae09\ud558\ub77c\"\ub77c\uace0 \ud558\uba70 \ucd5c\uc9c4\uc2e4\uacfc \ucd5c\uc9c4\uc2e4\uc758 \uc18c\uc18d\uc0ac\ub97c \uc0c1\ub300\ub85c \uc18c\uc1a1\uc744 \uc81c\uae30\ud558\uc600\ub2e4. \uc774\uc5d0 \ub300\ud574 \ucd5c\uc9c4\uc2e4\uc740 \"\ub098\ub294 \ud3ed\ud589 \uc0ac\uac74\uc758 \uc77c\ubc29\uc801 \ud53c\ud574\uc790\ub85c\uc11c \uc0ac\ud68c\uc801\u318d\ub3c4\ub355\uc801 \uba85\uc608\ub97c \ud6fc\uc190\ud588\ub2e4\uace0 \ud560 \uc218 \uc5c6\uace0, \ub610 \uc5b8\ub860\uc5d0 \ubcf4\ub3c4\ub41c \uac83\ub3c4 \uae30\uc790\ub4e4\uc774 \uacbd\ucc30\uc11c\uc5d0 \uac00\uc11c \ucde8\uc7ac\ud55c \ud6c4 \ub0b4\uac00 \uc785\uc6d0\ud55c \ubcd1\uc6d0\uc5d0 \ucc3e\uc544\uc640 \uc2ec\uce35 \ucde8\uc7ac\ud55c \uac83\uc77c \ubfd0 \ub0b4\uac8c \uadc0\ucc45\uc0ac\uc720\uac00 \uc5c6\ub2e4\"\ub77c\uace0 \ub300\uc751\ud558\uc600\ub2e4. 2004\ub144 12\uc6d4 9\uc77c, \uc870\uc131\ubbfc\uc758 \ud3ed\ub825 \ud589\uc704\uc5d0 \ub300\ud574 \uc720\uc8c4 \ud310\uacb0\uc774 \ub0b4\ub824\uc84c\ub2e4. 2004\ub144 12\uc6d4 16\uc77c, \ud5e4\ub7f4\ub4dc\uacbd\uc81c \uae30\uc790 \uc11c\ubcd1\uae30\ub294 \uc774 \uc0ac\uc548\uacfc \uad00\ub828\ud558\uc5ec \ud53c\ud574\uc790 \ucd5c\uc9c4\uc2e4\uc744 \uacf5\uaca9\ud588\ub2e4. 2005\ub144 9\uc6d4 18\uc77c, \ub9c8\uc774\ub370\uc77c\ub9ac \uae30\uc790 \ubc30\uad6d\ub0a8\uc740 \uc774 \uc0ac\uc548\uacfc \uad00\ub828\ud558\uc5ec '\uc0ac\uc0dd\ud65c \ud37c\ud3ec\uba3c\uc2a4' \uc6b4\uc6b4 \ud558\uba70 \ucd5c\uc9c4\uc2e4\uc744 \uc870\ub871\ud588\ub2e4. 2007\ub144 11\uc6d4 2\uc77c, \uc11c\ubcd1\uae30\ub294 \ub610 \uc774 \uc0ac\uc548\uacfc \uad00\ub828\ud558\uc5ec '\ucd94\uc7a1' \uc6b4\uc6b4 \ud558\uba70 \ucd5c\uc9c4\uc2e4\uc744 \uc870\ub871\ud588\ub2e4."} {"question": "\ucf00\uc774\ud2f0 \ud398\ub9ac\uac00 2008\ub144\uc5d0 \ubc1c\ud45c\ud55c \uc2f1\uae00 I Kissed A Girl\uc740 \ubb34\uc5c7\uc744 \uc8fc\uc81c\ub85c \uc0bc\uc558\uc2b5\ub2c8\uae4c?", "paragraph": "2008\ub144 \ub3d9\uc131\uc560\ub97c \uc8fc\uc81c\ub85c \uc0bc\uc544 \ub17c\ub780\uc744 \uc77c\uc73c\ud0a8 \uc2f1\uae00 \u3008I Kissed A Girl\u3009\uacfc \ub2e4\uc74c \uc2f1\uae00 \u3008Hot n Cold\u3009, \uadf8\ub9ac\uace0 \ud398\ub9ac\uc758 \ub450 \ubc88\uc9f8 \uc815\uaddc \uc568\ubc94 \u300aOne of the Boys\u300b\ub97c \ubc1c\ub9e4\ud558\uba70 \uc720\uba85\uc138\ub97c \uc5bb\uae30 \uc2dc\uc791\ud588\ub2e4. 2010\ub144\uc5d0\ub294 \uc138 \ubc88\uc9f8 \uc815\uaddc \uc568\ubc94 \u300aTeenage Dream\u300b\uc744 \ubc1c\ub9e4\ud574 \ucc98\uc74c\uc73c\ub85c \ube4c\ubcf4\ub4dc 200 1\uc704\uc5d0 \uc62c\ub790\ub2e4. \uc568\ubc94\uc758 \uc2f1\uae00\uc778 \u3008California Gurls\u3009, \u3008Teenage Dream\u3009, \u3008Firework\u3009, \u3008E.T.\u3009, \u3008Last Friday Night (T.G.I.F.)\u3009\uae4c\uc9c0 \ubaa8\ub450 \ube4c\ubcf4\ub4dc \ud56b 100 1\uc704\uc5d0 \uc62c\ub77c \ub9c8\uc774\ud074 \uc7ad\uc2a8\uc758 \u300aBad\u300b \uc774\ud6c4 20\uc5ec \ub144 \ub9cc\uc5d0 \ud55c \uc568\ubc94\uc5d0\uc11c \ub2e4\uc12f \uc7a5\uc758 \uc2f1\uae00\uc774 \uc5f0\uc18d\uc73c\ub85c 1\uc704\ub97c \ud558\ub294 \uae30\ub85d\uc744 \uc138\uc6e0\ub2e4. 2012\ub144 3\uc6d4\uc5d0\ub294 \ub9ac\ud328\ud0a4\uc9c0 \uc568\ubc94 \u300aTeenage Dream: The Complete Confection\u300b\uc744 \ubc1c\ub9e4\ud574 \uc2f1\uae00 \u3008Part of Me\u3009\ub294 1\uc704, \u3008Wide Awake\u3009\ub294 2\uc704\uc5d0 \uc62c\ub824\ub193\uc558\ub2e4. 2013\ub144\uc5d0\ub294 \ub124 \ubc88\uc9f8 \uc815\uaddc \uc568\ubc94 \u300aPrism\u300b\uc744 \ubc1c\ub9e4\ud588\ub2e4. \uc2f1\uae00 \u3008Roar\u3009\uc640 \u3008Dark Horse\u3009\uc758 \ubba4\uc9c1\ube44\ub514\uc624 \ubaa8\ub450 10\uc5b5 \uac74\uc758 \uc870\ud68c\uc218\ub97c \ub118\uae30\uba74\uc11c 10\uc5b5 \uc774\uc0c1 \uc870\ud68c\uc218 \ubba4\uc9c1\ube44\ub514\uc624\ub97c \ub450 \uac1c \uac00\uc9c4 \ucd5c\ucd08\uc758 \uc74c\uc545\uac00\uac00 \ub418\uc5c8\ub2e4. \u3008Dark Horse\u3009\ub294 \uc870\ud68c\uc218\ub97c 21\uc5b5\ud68c\ub97c \uae30\ub85d\ud558\uace0 \uc788\ub2e4. \ud398\ub9ac\ub294 \ubc1d\uc740 \uc0c9\uc0c1\uacfc \uc74c\uc2dd\uc744 \uc8fc\uc81c\ub85c \ud55c \ud328\uc158, \uadf8\ub9ac\uace0 \ud2b9\uc774\ud558\uace0 \uc720\ucf8c\ud55c \ucee8\uc149\uc73c\ub85c \uc798 \uc54c\ub824\uc838 \uc788\ub2e4. \ub610\ud55c \ucf58\uc11c\ud2b8 \uacf5\uc5f0\uc740 \uc815\uad50\ud558\uace0 \ud2b9\uc0c9\uc788\ub294 \ubb34\ub300 \uc7a5\uce58\uc640 \uc758\uc0c1\uc73c\ub85c \uc774\ub8e8\uc5b4\uc838 \uc788\ub2e4.", "answer": "\ub3d9\uc131\uc560", "sentence": "2008\ub144 \ub3d9\uc131\uc560 \ub97c \uc8fc\uc81c\ub85c \uc0bc\uc544 \ub17c\ub780\uc744 \uc77c\uc73c\ud0a8 \uc2f1\uae00 \u3008I Kissed A Girl\u3009\uacfc \ub2e4\uc74c \uc2f1\uae00 \u3008Hot n Cold\u3009, \uadf8\ub9ac\uace0 \ud398\ub9ac\uc758 \ub450 \ubc88\uc9f8 \uc815\uaddc \uc568\ubc94 \u300aOne of the Boys\u300b\ub97c \ubc1c\ub9e4\ud558\uba70 \uc720\uba85\uc138\ub97c \uc5bb\uae30 \uc2dc\uc791\ud588\ub2e4.", "paragraph_sentence": " 2008\ub144 \ub3d9\uc131\uc560 \ub97c \uc8fc\uc81c\ub85c \uc0bc\uc544 \ub17c\ub780\uc744 \uc77c\uc73c\ud0a8 \uc2f1\uae00 \u3008I Kissed A Girl\u3009\uacfc \ub2e4\uc74c \uc2f1\uae00 \u3008Hot n Cold\u3009, \uadf8\ub9ac\uace0 \ud398\ub9ac\uc758 \ub450 \ubc88\uc9f8 \uc815\uaddc \uc568\ubc94 \u300aOne of the Boys\u300b\ub97c \ubc1c\ub9e4\ud558\uba70 \uc720\uba85\uc138\ub97c \uc5bb\uae30 \uc2dc\uc791\ud588\ub2e4. 2010\ub144\uc5d0\ub294 \uc138 \ubc88\uc9f8 \uc815\uaddc \uc568\ubc94 \u300aTeenage Dream\u300b\uc744 \ubc1c\ub9e4\ud574 \ucc98\uc74c\uc73c\ub85c \ube4c\ubcf4\ub4dc 200 1\uc704\uc5d0 \uc62c\ub790\ub2e4. \uc568\ubc94\uc758 \uc2f1\uae00\uc778 \u3008California Gurls\u3009, \u3008Teenage Dream\u3009, \u3008Firework\u3009, \u3008E.T.\u3009, \u3008Last Friday Night (T.G.I.F.)\u3009\uae4c\uc9c0 \ubaa8\ub450 \ube4c\ubcf4\ub4dc \ud56b 100 1\uc704\uc5d0 \uc62c\ub77c \ub9c8\uc774\ud074 \uc7ad\uc2a8\uc758 \u300aBad\u300b \uc774\ud6c4 20\uc5ec \ub144 \ub9cc\uc5d0 \ud55c \uc568\ubc94\uc5d0\uc11c \ub2e4\uc12f \uc7a5\uc758 \uc2f1\uae00\uc774 \uc5f0\uc18d\uc73c\ub85c 1\uc704\ub97c \ud558\ub294 \uae30\ub85d\uc744 \uc138\uc6e0\ub2e4. 2012\ub144 3\uc6d4\uc5d0\ub294 \ub9ac\ud328\ud0a4\uc9c0 \uc568\ubc94 \u300aTeenage Dream: The Complete Confection\u300b\uc744 \ubc1c\ub9e4\ud574 \uc2f1\uae00 \u3008Part of Me\u3009\ub294 1\uc704, \u3008Wide Awake\u3009\ub294 2\uc704\uc5d0 \uc62c\ub824\ub193\uc558\ub2e4. 2013\ub144\uc5d0\ub294 \ub124 \ubc88\uc9f8 \uc815\uaddc \uc568\ubc94 \u300aPrism\u300b\uc744 \ubc1c\ub9e4\ud588\ub2e4. \uc2f1\uae00 \u3008Roar\u3009\uc640 \u3008Dark Horse\u3009\uc758 \ubba4\uc9c1\ube44\ub514\uc624 \ubaa8\ub450 10\uc5b5 \uac74\uc758 \uc870\ud68c\uc218\ub97c \ub118\uae30\uba74\uc11c 10\uc5b5 \uc774\uc0c1 \uc870\ud68c\uc218 \ubba4\uc9c1\ube44\ub514\uc624\ub97c \ub450 \uac1c \uac00\uc9c4 \ucd5c\ucd08\uc758 \uc74c\uc545\uac00\uac00 \ub418\uc5c8\ub2e4. \u3008Dark Horse\u3009\ub294 \uc870\ud68c\uc218\ub97c 21\uc5b5\ud68c\ub97c \uae30\ub85d\ud558\uace0 \uc788\ub2e4. \ud398\ub9ac\ub294 \ubc1d\uc740 \uc0c9\uc0c1\uacfc \uc74c\uc2dd\uc744 \uc8fc\uc81c\ub85c \ud55c \ud328\uc158, \uadf8\ub9ac\uace0 \ud2b9\uc774\ud558\uace0 \uc720\ucf8c\ud55c \ucee8\uc149\uc73c\ub85c \uc798 \uc54c\ub824\uc838 \uc788\ub2e4. \ub610\ud55c \ucf58\uc11c\ud2b8 \uacf5\uc5f0\uc740 \uc815\uad50\ud558\uace0 \ud2b9\uc0c9\uc788\ub294 \ubb34\ub300 \uc7a5\uce58\uc640 \uc758\uc0c1\uc73c\ub85c \uc774\ub8e8\uc5b4\uc838 \uc788\ub2e4.", "paragraph_answer": "2008\ub144 \ub3d9\uc131\uc560 \ub97c \uc8fc\uc81c\ub85c \uc0bc\uc544 \ub17c\ub780\uc744 \uc77c\uc73c\ud0a8 \uc2f1\uae00 \u3008I Kissed A Girl\u3009\uacfc \ub2e4\uc74c \uc2f1\uae00 \u3008Hot n Cold\u3009, \uadf8\ub9ac\uace0 \ud398\ub9ac\uc758 \ub450 \ubc88\uc9f8 \uc815\uaddc \uc568\ubc94 \u300aOne of the Boys\u300b\ub97c \ubc1c\ub9e4\ud558\uba70 \uc720\uba85\uc138\ub97c \uc5bb\uae30 \uc2dc\uc791\ud588\ub2e4. 2010\ub144\uc5d0\ub294 \uc138 \ubc88\uc9f8 \uc815\uaddc \uc568\ubc94 \u300aTeenage Dream\u300b\uc744 \ubc1c\ub9e4\ud574 \ucc98\uc74c\uc73c\ub85c \ube4c\ubcf4\ub4dc 200 1\uc704\uc5d0 \uc62c\ub790\ub2e4. \uc568\ubc94\uc758 \uc2f1\uae00\uc778 \u3008California Gurls\u3009, \u3008Teenage Dream\u3009, \u3008Firework\u3009, \u3008E.T.\u3009, \u3008Last Friday Night (T.G.I.F.)\u3009\uae4c\uc9c0 \ubaa8\ub450 \ube4c\ubcf4\ub4dc \ud56b 100 1\uc704\uc5d0 \uc62c\ub77c \ub9c8\uc774\ud074 \uc7ad\uc2a8\uc758 \u300aBad\u300b \uc774\ud6c4 20\uc5ec \ub144 \ub9cc\uc5d0 \ud55c \uc568\ubc94\uc5d0\uc11c \ub2e4\uc12f \uc7a5\uc758 \uc2f1\uae00\uc774 \uc5f0\uc18d\uc73c\ub85c 1\uc704\ub97c \ud558\ub294 \uae30\ub85d\uc744 \uc138\uc6e0\ub2e4. 2012\ub144 3\uc6d4\uc5d0\ub294 \ub9ac\ud328\ud0a4\uc9c0 \uc568\ubc94 \u300aTeenage Dream: The Complete Confection\u300b\uc744 \ubc1c\ub9e4\ud574 \uc2f1\uae00 \u3008Part of Me\u3009\ub294 1\uc704, \u3008Wide Awake\u3009\ub294 2\uc704\uc5d0 \uc62c\ub824\ub193\uc558\ub2e4. 2013\ub144\uc5d0\ub294 \ub124 \ubc88\uc9f8 \uc815\uaddc \uc568\ubc94 \u300aPrism\u300b\uc744 \ubc1c\ub9e4\ud588\ub2e4. \uc2f1\uae00 \u3008Roar\u3009\uc640 \u3008Dark Horse\u3009\uc758 \ubba4\uc9c1\ube44\ub514\uc624 \ubaa8\ub450 10\uc5b5 \uac74\uc758 \uc870\ud68c\uc218\ub97c \ub118\uae30\uba74\uc11c 10\uc5b5 \uc774\uc0c1 \uc870\ud68c\uc218 \ubba4\uc9c1\ube44\ub514\uc624\ub97c \ub450 \uac1c \uac00\uc9c4 \ucd5c\ucd08\uc758 \uc74c\uc545\uac00\uac00 \ub418\uc5c8\ub2e4. \u3008Dark Horse\u3009\ub294 \uc870\ud68c\uc218\ub97c 21\uc5b5\ud68c\ub97c \uae30\ub85d\ud558\uace0 \uc788\ub2e4. \ud398\ub9ac\ub294 \ubc1d\uc740 \uc0c9\uc0c1\uacfc \uc74c\uc2dd\uc744 \uc8fc\uc81c\ub85c \ud55c \ud328\uc158, \uadf8\ub9ac\uace0 \ud2b9\uc774\ud558\uace0 \uc720\ucf8c\ud55c \ucee8\uc149\uc73c\ub85c \uc798 \uc54c\ub824\uc838 \uc788\ub2e4. \ub610\ud55c \ucf58\uc11c\ud2b8 \uacf5\uc5f0\uc740 \uc815\uad50\ud558\uace0 \ud2b9\uc0c9\uc788\ub294 \ubb34\ub300 \uc7a5\uce58\uc640 \uc758\uc0c1\uc73c\ub85c \uc774\ub8e8\uc5b4\uc838 \uc788\ub2e4.", "sentence_answer": "2008\ub144 \ub3d9\uc131\uc560 \ub97c \uc8fc\uc81c\ub85c \uc0bc\uc544 \ub17c\ub780\uc744 \uc77c\uc73c\ud0a8 \uc2f1\uae00 \u3008I Kissed A Girl\u3009\uacfc \ub2e4\uc74c \uc2f1\uae00 \u3008Hot n Cold\u3009, \uadf8\ub9ac\uace0 \ud398\ub9ac\uc758 \ub450 \ubc88\uc9f8 \uc815\uaddc \uc568\ubc94 \u300aOne of the Boys\u300b\ub97c \ubc1c\ub9e4\ud558\uba70 \uc720\uba85\uc138\ub97c \uc5bb\uae30 \uc2dc\uc791\ud588\ub2e4.", "paragraph_id": "6561799-1-0", "paragraph_question": "question: \ucf00\uc774\ud2f0 \ud398\ub9ac\uac00 2008\ub144\uc5d0 \ubc1c\ud45c\ud55c \uc2f1\uae00 I Kissed A Girl\uc740 \ubb34\uc5c7\uc744 \uc8fc\uc81c\ub85c \uc0bc\uc558\uc2b5\ub2c8\uae4c?, context: 2008\ub144 \ub3d9\uc131\uc560\ub97c \uc8fc\uc81c\ub85c \uc0bc\uc544 \ub17c\ub780\uc744 \uc77c\uc73c\ud0a8 \uc2f1\uae00 \u3008I Kissed A Girl\u3009\uacfc \ub2e4\uc74c \uc2f1\uae00 \u3008Hot n Cold\u3009, \uadf8\ub9ac\uace0 \ud398\ub9ac\uc758 \ub450 \ubc88\uc9f8 \uc815\uaddc \uc568\ubc94 \u300aOne of the Boys\u300b\ub97c \ubc1c\ub9e4\ud558\uba70 \uc720\uba85\uc138\ub97c \uc5bb\uae30 \uc2dc\uc791\ud588\ub2e4. 2010\ub144\uc5d0\ub294 \uc138 \ubc88\uc9f8 \uc815\uaddc \uc568\ubc94 \u300aTeenage Dream\u300b\uc744 \ubc1c\ub9e4\ud574 \ucc98\uc74c\uc73c\ub85c \ube4c\ubcf4\ub4dc 200 1\uc704\uc5d0 \uc62c\ub790\ub2e4. \uc568\ubc94\uc758 \uc2f1\uae00\uc778 \u3008California Gurls\u3009, \u3008Teenage Dream\u3009, \u3008Firework\u3009, \u3008E.T.\u3009, \u3008Last Friday Night (T.G.I.F.)\u3009\uae4c\uc9c0 \ubaa8\ub450 \ube4c\ubcf4\ub4dc \ud56b 100 1\uc704\uc5d0 \uc62c\ub77c \ub9c8\uc774\ud074 \uc7ad\uc2a8\uc758 \u300aBad\u300b \uc774\ud6c4 20\uc5ec \ub144 \ub9cc\uc5d0 \ud55c \uc568\ubc94\uc5d0\uc11c \ub2e4\uc12f \uc7a5\uc758 \uc2f1\uae00\uc774 \uc5f0\uc18d\uc73c\ub85c 1\uc704\ub97c \ud558\ub294 \uae30\ub85d\uc744 \uc138\uc6e0\ub2e4. 2012\ub144 3\uc6d4\uc5d0\ub294 \ub9ac\ud328\ud0a4\uc9c0 \uc568\ubc94 \u300aTeenage Dream: The Complete Confection\u300b\uc744 \ubc1c\ub9e4\ud574 \uc2f1\uae00 \u3008Part of Me\u3009\ub294 1\uc704, \u3008Wide Awake\u3009\ub294 2\uc704\uc5d0 \uc62c\ub824\ub193\uc558\ub2e4. 2013\ub144\uc5d0\ub294 \ub124 \ubc88\uc9f8 \uc815\uaddc \uc568\ubc94 \u300aPrism\u300b\uc744 \ubc1c\ub9e4\ud588\ub2e4. \uc2f1\uae00 \u3008Roar\u3009\uc640 \u3008Dark Horse\u3009\uc758 \ubba4\uc9c1\ube44\ub514\uc624 \ubaa8\ub450 10\uc5b5 \uac74\uc758 \uc870\ud68c\uc218\ub97c \ub118\uae30\uba74\uc11c 10\uc5b5 \uc774\uc0c1 \uc870\ud68c\uc218 \ubba4\uc9c1\ube44\ub514\uc624\ub97c \ub450 \uac1c \uac00\uc9c4 \ucd5c\ucd08\uc758 \uc74c\uc545\uac00\uac00 \ub418\uc5c8\ub2e4. \u3008Dark Horse\u3009\ub294 \uc870\ud68c\uc218\ub97c 21\uc5b5\ud68c\ub97c \uae30\ub85d\ud558\uace0 \uc788\ub2e4. \ud398\ub9ac\ub294 \ubc1d\uc740 \uc0c9\uc0c1\uacfc \uc74c\uc2dd\uc744 \uc8fc\uc81c\ub85c \ud55c \ud328\uc158, \uadf8\ub9ac\uace0 \ud2b9\uc774\ud558\uace0 \uc720\ucf8c\ud55c \ucee8\uc149\uc73c\ub85c \uc798 \uc54c\ub824\uc838 \uc788\ub2e4. \ub610\ud55c \ucf58\uc11c\ud2b8 \uacf5\uc5f0\uc740 \uc815\uad50\ud558\uace0 \ud2b9\uc0c9\uc788\ub294 \ubb34\ub300 \uc7a5\uce58\uc640 \uc758\uc0c1\uc73c\ub85c \uc774\ub8e8\uc5b4\uc838 \uc788\ub2e4."} {"question": "\uc544\ud0ac\ub808\uc6b0\uc2a4\ub294 \ub204\uad6c\uc758 \uc544\ub4e4\uc778\uac00?", "paragraph": "\uc5b4\ub5a4 \uc774\ub294 \"\uc18c\ud06c\ub77c\ud14c\uc2a4, \uadf8\ub807\ub2e4\uba74 \uc790\ub124\ub294 \uc0ac\ud615 \uc120\uace0\ub97c \ubc1b\uac8c \ub420\uc9c0\ub3c4 \ubaa8\ub974\ub294 \uadf8\ub7f0 \uc0b6\uc744 \uc0b4\uc544 \uc628 \ub370 \ub300\ud574 \ubd80\ub044\ub7ec\uc6c0\uc744 \ub290\ub07c\uc9c0 \uc54a\ub294\uac00?\"\ub77c\uace0 \ubb3c\uc744 \uac83\uc774\uc624. \ub098\ub294 \uadf8 \uc9c8\ubb38\uc5d0 \ub2e4\uc74c\uacfc \uac19\uc774 \ub300\ub2f5\ud558\uaca0\uc18c. \"\uadf8\uac83\uc740 \uc633\uc9c0 \ubabb\ud55c \uacac\ud574\uc694. \uc870\uae08\uc774\ub77c\ub3c4 \ud488\uc704\ub97c \uac00\uc9c0\uace0 \uc788\ub294 \uc0ac\ub78c\uc774\ub77c\uba74 \uc5b4\ub5a4 \uc77c\uc744 \ud560 \ub54c \uadf8\uac83\uc774 \uc633\uc740 \uc77c\uc778\uac00 \uc633\uc9c0 \uc54a\uc740 \uc77c\uc778\uac00, \uc120\ub7c9\ud55c \uc0ac\ub78c\uc774 \ud560 \uc77c\uc778\uac00 \uc545\ud55c \uc0ac\ub78c\uc774 \ud560 \uc77c\uc778\uac00 \ud558\ub294 \uac83\ub9cc\uc744 \uc0dd\uac01\ud574\uc57c \ud558\uba70, \uadf8 \uc77c\uc744 \ud558\uba74 \uc0b4\uac8c \ub418\ub290\ub0d0 \uc8fd\uac8c \ub418\ub290\ub0d0 \ud558\ub294 \uac83\uc744 \uc0dd\uac01\ud574\uc11c\ub294 \uc548 \ub418\uc624. \ub2f9\uc2e0\uc758 \uacac\ud574\ub97c \ub530\ub974\uba74 \uc800 \ud2b8\ub85c\uc774\uc544\uc5d0\uc11c \uc0dd\uc560\ub97c \ub9c8\uce5c \ubc18\uc2e0\ub4e4\uc740 \ud558\ucc2e\uc740 \uc874\uc7ac\ub4e4\uc774 \ub418\ub294 \uc148\uc774\ub2c8\uae4c. \uadf8 \uc911\uc5d0\uc11c\ub3c4 \ud14c\ud2f0\uc2a4\uc758 \uc544\ub4e4\uc778 \uc544\ud0ac\ub808\uc6b0\uc2a4\ub294 \uce58\uc695\uc744 \ucc38\uace0 \uacac\ub514\ub294 \ub370 \ube44\ud558\uba74 \uadf8\ub7f0 \uc704\ud5d8\uc740 \uc544\ubb34\uac83\ub3c4 \uc544\ub2c8\ub77c\uace0 \uc0dd\uac01\ud588\uc18c. \ub098\ub294 \ud5e5\ud1a0\ub974\ub97c \uc8fd\uc774\ub824\uace0 \uc11c\ub450\ub974\ub294 \uadf8\uc5d0\uac8c \uc5b4\uba38\ub2c8\uc778 \uc5ec\uc2e0\uc774 \uc774\ub807\uac8c \ub9d0\ud55c \uac83\uc73c\ub85c \uc54c\uace0 \uc788\uc18c.", "answer": "\ud14c\ud2f0\uc2a4", "sentence": "\uadf8 \uc911\uc5d0\uc11c\ub3c4 \ud14c\ud2f0\uc2a4 \uc758", "paragraph_sentence": "\uc5b4\ub5a4 \uc774\ub294 \"\uc18c\ud06c\ub77c\ud14c\uc2a4, \uadf8\ub807\ub2e4\uba74 \uc790\ub124\ub294 \uc0ac\ud615 \uc120\uace0\ub97c \ubc1b\uac8c \ub420\uc9c0\ub3c4 \ubaa8\ub974\ub294 \uadf8\ub7f0 \uc0b6\uc744 \uc0b4\uc544 \uc628 \ub370 \ub300\ud574 \ubd80\ub044\ub7ec\uc6c0\uc744 \ub290\ub07c\uc9c0 \uc54a\ub294\uac00?\"\ub77c\uace0 \ubb3c\uc744 \uac83\uc774\uc624. \ub098\ub294 \uadf8 \uc9c8\ubb38\uc5d0 \ub2e4\uc74c\uacfc \uac19\uc774 \ub300\ub2f5\ud558\uaca0\uc18c. \"\uadf8\uac83\uc740 \uc633\uc9c0 \ubabb\ud55c \uacac\ud574\uc694. \uc870\uae08\uc774\ub77c\ub3c4 \ud488\uc704\ub97c \uac00\uc9c0\uace0 \uc788\ub294 \uc0ac\ub78c\uc774\ub77c\uba74 \uc5b4\ub5a4 \uc77c\uc744 \ud560 \ub54c \uadf8\uac83\uc774 \uc633\uc740 \uc77c\uc778\uac00 \uc633\uc9c0 \uc54a\uc740 \uc77c\uc778\uac00, \uc120\ub7c9\ud55c \uc0ac\ub78c\uc774 \ud560 \uc77c\uc778\uac00 \uc545\ud55c \uc0ac\ub78c\uc774 \ud560 \uc77c\uc778\uac00 \ud558\ub294 \uac83\ub9cc\uc744 \uc0dd\uac01\ud574\uc57c \ud558\uba70, \uadf8 \uc77c\uc744 \ud558\uba74 \uc0b4\uac8c \ub418\ub290\ub0d0 \uc8fd\uac8c \ub418\ub290\ub0d0 \ud558\ub294 \uac83\uc744 \uc0dd\uac01\ud574\uc11c\ub294 \uc548 \ub418\uc624. \ub2f9\uc2e0\uc758 \uacac\ud574\ub97c \ub530\ub974\uba74 \uc800 \ud2b8\ub85c\uc774\uc544\uc5d0\uc11c \uc0dd\uc560\ub97c \ub9c8\uce5c \ubc18\uc2e0\ub4e4\uc740 \ud558\ucc2e\uc740 \uc874\uc7ac\ub4e4\uc774 \ub418\ub294 \uc148\uc774\ub2c8\uae4c. \uadf8 \uc911\uc5d0\uc11c\ub3c4 \ud14c\ud2f0\uc2a4 \uc758 \uc544\ub4e4\uc778 \uc544\ud0ac\ub808\uc6b0\uc2a4\ub294 \uce58\uc695\uc744 \ucc38\uace0 \uacac\ub514\ub294 \ub370 \ube44\ud558\uba74 \uadf8\ub7f0 \uc704\ud5d8\uc740 \uc544\ubb34\uac83\ub3c4 \uc544\ub2c8\ub77c\uace0 \uc0dd\uac01\ud588\uc18c. \ub098\ub294 \ud5e5\ud1a0\ub974\ub97c \uc8fd\uc774\ub824\uace0 \uc11c\ub450\ub974\ub294 \uadf8\uc5d0\uac8c \uc5b4\uba38\ub2c8\uc778 \uc5ec\uc2e0\uc774 \uc774\ub807\uac8c \ub9d0\ud55c \uac83\uc73c\ub85c \uc54c\uace0 \uc788\uc18c.", "paragraph_answer": "\uc5b4\ub5a4 \uc774\ub294 \"\uc18c\ud06c\ub77c\ud14c\uc2a4, \uadf8\ub807\ub2e4\uba74 \uc790\ub124\ub294 \uc0ac\ud615 \uc120\uace0\ub97c \ubc1b\uac8c \ub420\uc9c0\ub3c4 \ubaa8\ub974\ub294 \uadf8\ub7f0 \uc0b6\uc744 \uc0b4\uc544 \uc628 \ub370 \ub300\ud574 \ubd80\ub044\ub7ec\uc6c0\uc744 \ub290\ub07c\uc9c0 \uc54a\ub294\uac00?\"\ub77c\uace0 \ubb3c\uc744 \uac83\uc774\uc624. \ub098\ub294 \uadf8 \uc9c8\ubb38\uc5d0 \ub2e4\uc74c\uacfc \uac19\uc774 \ub300\ub2f5\ud558\uaca0\uc18c. \"\uadf8\uac83\uc740 \uc633\uc9c0 \ubabb\ud55c \uacac\ud574\uc694. \uc870\uae08\uc774\ub77c\ub3c4 \ud488\uc704\ub97c \uac00\uc9c0\uace0 \uc788\ub294 \uc0ac\ub78c\uc774\ub77c\uba74 \uc5b4\ub5a4 \uc77c\uc744 \ud560 \ub54c \uadf8\uac83\uc774 \uc633\uc740 \uc77c\uc778\uac00 \uc633\uc9c0 \uc54a\uc740 \uc77c\uc778\uac00, \uc120\ub7c9\ud55c \uc0ac\ub78c\uc774 \ud560 \uc77c\uc778\uac00 \uc545\ud55c \uc0ac\ub78c\uc774 \ud560 \uc77c\uc778\uac00 \ud558\ub294 \uac83\ub9cc\uc744 \uc0dd\uac01\ud574\uc57c \ud558\uba70, \uadf8 \uc77c\uc744 \ud558\uba74 \uc0b4\uac8c \ub418\ub290\ub0d0 \uc8fd\uac8c \ub418\ub290\ub0d0 \ud558\ub294 \uac83\uc744 \uc0dd\uac01\ud574\uc11c\ub294 \uc548 \ub418\uc624. \ub2f9\uc2e0\uc758 \uacac\ud574\ub97c \ub530\ub974\uba74 \uc800 \ud2b8\ub85c\uc774\uc544\uc5d0\uc11c \uc0dd\uc560\ub97c \ub9c8\uce5c \ubc18\uc2e0\ub4e4\uc740 \ud558\ucc2e\uc740 \uc874\uc7ac\ub4e4\uc774 \ub418\ub294 \uc148\uc774\ub2c8\uae4c. \uadf8 \uc911\uc5d0\uc11c\ub3c4 \ud14c\ud2f0\uc2a4 \uc758 \uc544\ub4e4\uc778 \uc544\ud0ac\ub808\uc6b0\uc2a4\ub294 \uce58\uc695\uc744 \ucc38\uace0 \uacac\ub514\ub294 \ub370 \ube44\ud558\uba74 \uadf8\ub7f0 \uc704\ud5d8\uc740 \uc544\ubb34\uac83\ub3c4 \uc544\ub2c8\ub77c\uace0 \uc0dd\uac01\ud588\uc18c. \ub098\ub294 \ud5e5\ud1a0\ub974\ub97c \uc8fd\uc774\ub824\uace0 \uc11c\ub450\ub974\ub294 \uadf8\uc5d0\uac8c \uc5b4\uba38\ub2c8\uc778 \uc5ec\uc2e0\uc774 \uc774\ub807\uac8c \ub9d0\ud55c \uac83\uc73c\ub85c \uc54c\uace0 \uc788\uc18c.", "sentence_answer": "\uadf8 \uc911\uc5d0\uc11c\ub3c4 \ud14c\ud2f0\uc2a4 \uc758", "paragraph_id": "6477962-24-0", "paragraph_question": "question: \uc544\ud0ac\ub808\uc6b0\uc2a4\ub294 \ub204\uad6c\uc758 \uc544\ub4e4\uc778\uac00?, context: \uc5b4\ub5a4 \uc774\ub294 \"\uc18c\ud06c\ub77c\ud14c\uc2a4, \uadf8\ub807\ub2e4\uba74 \uc790\ub124\ub294 \uc0ac\ud615 \uc120\uace0\ub97c \ubc1b\uac8c \ub420\uc9c0\ub3c4 \ubaa8\ub974\ub294 \uadf8\ub7f0 \uc0b6\uc744 \uc0b4\uc544 \uc628 \ub370 \ub300\ud574 \ubd80\ub044\ub7ec\uc6c0\uc744 \ub290\ub07c\uc9c0 \uc54a\ub294\uac00?\"\ub77c\uace0 \ubb3c\uc744 \uac83\uc774\uc624. \ub098\ub294 \uadf8 \uc9c8\ubb38\uc5d0 \ub2e4\uc74c\uacfc \uac19\uc774 \ub300\ub2f5\ud558\uaca0\uc18c. \"\uadf8\uac83\uc740 \uc633\uc9c0 \ubabb\ud55c \uacac\ud574\uc694. \uc870\uae08\uc774\ub77c\ub3c4 \ud488\uc704\ub97c \uac00\uc9c0\uace0 \uc788\ub294 \uc0ac\ub78c\uc774\ub77c\uba74 \uc5b4\ub5a4 \uc77c\uc744 \ud560 \ub54c \uadf8\uac83\uc774 \uc633\uc740 \uc77c\uc778\uac00 \uc633\uc9c0 \uc54a\uc740 \uc77c\uc778\uac00, \uc120\ub7c9\ud55c \uc0ac\ub78c\uc774 \ud560 \uc77c\uc778\uac00 \uc545\ud55c \uc0ac\ub78c\uc774 \ud560 \uc77c\uc778\uac00 \ud558\ub294 \uac83\ub9cc\uc744 \uc0dd\uac01\ud574\uc57c \ud558\uba70, \uadf8 \uc77c\uc744 \ud558\uba74 \uc0b4\uac8c \ub418\ub290\ub0d0 \uc8fd\uac8c \ub418\ub290\ub0d0 \ud558\ub294 \uac83\uc744 \uc0dd\uac01\ud574\uc11c\ub294 \uc548 \ub418\uc624. \ub2f9\uc2e0\uc758 \uacac\ud574\ub97c \ub530\ub974\uba74 \uc800 \ud2b8\ub85c\uc774\uc544\uc5d0\uc11c \uc0dd\uc560\ub97c \ub9c8\uce5c \ubc18\uc2e0\ub4e4\uc740 \ud558\ucc2e\uc740 \uc874\uc7ac\ub4e4\uc774 \ub418\ub294 \uc148\uc774\ub2c8\uae4c. \uadf8 \uc911\uc5d0\uc11c\ub3c4 \ud14c\ud2f0\uc2a4\uc758 \uc544\ub4e4\uc778 \uc544\ud0ac\ub808\uc6b0\uc2a4\ub294 \uce58\uc695\uc744 \ucc38\uace0 \uacac\ub514\ub294 \ub370 \ube44\ud558\uba74 \uadf8\ub7f0 \uc704\ud5d8\uc740 \uc544\ubb34\uac83\ub3c4 \uc544\ub2c8\ub77c\uace0 \uc0dd\uac01\ud588\uc18c. \ub098\ub294 \ud5e5\ud1a0\ub974\ub97c \uc8fd\uc774\ub824\uace0 \uc11c\ub450\ub974\ub294 \uadf8\uc5d0\uac8c \uc5b4\uba38\ub2c8\uc778 \uc5ec\uc2e0\uc774 \uc774\ub807\uac8c \ub9d0\ud55c \uac83\uc73c\ub85c \uc54c\uace0 \uc788\uc18c."} {"question": "1950\ub144 \uc911\uad6d \uc81c9\ubcd1\ub2e8\uc774 \ubbf8\uad6d \uc81c10\uad70\ub2e8\uc744 \uacf5\uaca9\ud55c \uc9c0\uc5ed\uc740 \uc5b4\ub514\uc778\uac00?", "paragraph": "1950\ub144 11\uc6d4 27\uc77c, \uc911\uad6d \uc81c9\ubcd1\ub2e8\uc740 \uc7a5\uc9c4\ud638 \uc9c0\uc5ed\uc5d0\uc11c \uc5d0\ub4dc\uc6cc\ub4dc \uc54c\ubaac\ub4dc\uac00 \uc774\ub044\ub294 \ubbf8\uad6d \uc81c10\uad70\ub2e8\uc744 \uae30\uc2b5\uacf5\uaca9\ud588\ub2e4. \ud639\ub3c5\ud55c \uaca8\uc6b8\uc5d0 \uc794\ud639\ud55c 17\uc77c \uac04\uc758 \uc804\ud22c\uac00 \ubc8c\uc5b4\uc84c\ub2e4. 11\uc6d4 27\uc77c\ubd80\ud130 12\uc6d4 13\uc77c\uae4c\uc9c0 30,000\uba85\uc758 \uc720\uc5d4\uc0ac\ub839\ubd80 \ubcd1\ub825\uc774 \uc62c\ub9ac\ubc84 P. \uc2a4\ubbf8\uc2a4 \uc18c\uc7a5\uc758 \uc9c0\ud718\ub97c \ubc1b\uace0 \uc788\uc5c8\uc9c0\ub9cc \uc774\ub4e4\uc740 \uace7 \ud3ec\uc704\ub418\uc5c8\uace0, \ub9c8\uc624\uca4c\ub465\uc758 \uc720\uc5d4\uad70 \uaca9\ud30c \uc9c0\uc2dc\ub97c \ubc1b\uc740 \uc479\uc2a4\ub8ec\uc774 \uc774\ub044\ub294 \uc57d 120,000\uba85\uc758 \uc911\uacf5\uad70\uc774 \uc774\ub4e4\uc744 \uacf5\uaca9\ud588\ub2e4. \uc720\uc5d4\uad70\uc740 \uc774\ub7ec\ud55c \uc0c1\ud669\uc5d0\ub3c4 \ubd88\uad6c\ud558\uace0 \ucca0\uc218\ub97c \uc704\ud574 \uc2f8\uc6c0\uc744 \uc774\uc5b4\ub098\uac14\uc73c\uba70 \uc911\uacf5\uad70\uc758 \uc0ac\uc0c1\uc790\ub97c \ub298\ub824\uac00\uba70 \ud3ec\uc704\ub97c \ub3cc\ud30c\ud588\ub2e4. \uc81c10\uad70\ub2e8\uc758 \ud765\ub0a8 \ucca0\uc218 \uc791\uc804\uc774 \uc720\uc5d4\uad70\uc758 \ubd81\ud55c \ucca0\uc218\uc758 \ub9c8\uc9c0\ub9c9 \ub2e8\uacc4\uc600\ub2e4. \ud574\ubcd1\ub300\uac00 \ucca0\uc218\uc5d0 \uc131\uacf5\ud560 \uc218 \uc788\uc5c8\ub358 \uc774\uc720\ub294 \ub3d9\ucabd\uc5d0\uc11c \uc911\uacf5\uad70\uc758 \uc608\ubd09\uc744 \ub9e1\uc740 \ud398\uc774\uc2a4 \ud2b9\uc218\uc784\ubb34\ubd80\ub300\uc758 \uc5ed\ud560\uc774 \ucef8\ub2e4. \ud398\uc774\uc2a4 \ud2b9\uc218\uc784\ubb34\ubd80\ub300\uac00 \ud76c\uc0dd\uc744 \uce58\ub974\uba70 \ud3ec\ub85c\uac00 \ub418\ub294 \ub3d9\uc548, \uc11c\ucabd\uc758 \ud574\ubcd1\ub300\ub294 \ud3ec\uc704\ub418\uc9c0 \uc54a\uc744 \uc218 \uc788\uc5c8\ub2e4. \uc911\uacf5\uad70\uc740 \uc720\uc5d4\uad70\uc744 \ubd81\ud55c \ub3d9\ubd81\ubd80\uc5d0\uc11c \ubab0\uc544\ub0b4\ub294\ub370 \uc131\uacf5\ud588\uc73c\ub098 \ud070 \ud53c\ud574\ub97c \uc785\uc5c8\ub2e4.", "answer": "\uc7a5\uc9c4\ud638", "sentence": "1950\ub144 11\uc6d4 27\uc77c, \uc911\uad6d \uc81c9\ubcd1\ub2e8\uc740 \uc7a5\uc9c4\ud638 \uc9c0\uc5ed\uc5d0\uc11c \uc5d0\ub4dc\uc6cc\ub4dc \uc54c\ubaac\ub4dc\uac00 \uc774\ub044\ub294 \ubbf8\uad6d \uc81c10\uad70\ub2e8\uc744 \uae30\uc2b5\uacf5\uaca9\ud588\ub2e4.", "paragraph_sentence": " 1950\ub144 11\uc6d4 27\uc77c, \uc911\uad6d \uc81c9\ubcd1\ub2e8\uc740 \uc7a5\uc9c4\ud638 \uc9c0\uc5ed\uc5d0\uc11c \uc5d0\ub4dc\uc6cc\ub4dc \uc54c\ubaac\ub4dc\uac00 \uc774\ub044\ub294 \ubbf8\uad6d \uc81c10\uad70\ub2e8\uc744 \uae30\uc2b5\uacf5\uaca9\ud588\ub2e4. \ud639\ub3c5\ud55c \uaca8\uc6b8\uc5d0 \uc794\ud639\ud55c 17\uc77c \uac04\uc758 \uc804\ud22c\uac00 \ubc8c\uc5b4\uc84c\ub2e4. 11\uc6d4 27\uc77c\ubd80\ud130 12\uc6d4 13\uc77c\uae4c\uc9c0 30,000\uba85\uc758 \uc720\uc5d4\uc0ac\ub839\ubd80 \ubcd1\ub825\uc774 \uc62c\ub9ac\ubc84 P. \uc2a4\ubbf8\uc2a4 \uc18c\uc7a5\uc758 \uc9c0\ud718\ub97c \ubc1b\uace0 \uc788\uc5c8\uc9c0\ub9cc \uc774\ub4e4\uc740 \uace7 \ud3ec\uc704\ub418\uc5c8\uace0, \ub9c8\uc624\uca4c\ub465\uc758 \uc720\uc5d4\uad70 \uaca9\ud30c \uc9c0\uc2dc\ub97c \ubc1b\uc740 \uc479\uc2a4\ub8ec\uc774 \uc774\ub044\ub294 \uc57d 120,000\uba85\uc758 \uc911\uacf5\uad70\uc774 \uc774\ub4e4\uc744 \uacf5\uaca9\ud588\ub2e4. \uc720\uc5d4\uad70\uc740 \uc774\ub7ec\ud55c \uc0c1\ud669\uc5d0\ub3c4 \ubd88\uad6c\ud558\uace0 \ucca0\uc218\ub97c \uc704\ud574 \uc2f8\uc6c0\uc744 \uc774\uc5b4\ub098\uac14\uc73c\uba70 \uc911\uacf5\uad70\uc758 \uc0ac\uc0c1\uc790\ub97c \ub298\ub824\uac00\uba70 \ud3ec\uc704\ub97c \ub3cc\ud30c\ud588\ub2e4. \uc81c10\uad70\ub2e8\uc758 \ud765\ub0a8 \ucca0\uc218 \uc791\uc804\uc774 \uc720\uc5d4\uad70\uc758 \ubd81\ud55c \ucca0\uc218\uc758 \ub9c8\uc9c0\ub9c9 \ub2e8\uacc4\uc600\ub2e4. \ud574\ubcd1\ub300\uac00 \ucca0\uc218\uc5d0 \uc131\uacf5\ud560 \uc218 \uc788\uc5c8\ub358 \uc774\uc720\ub294 \ub3d9\ucabd\uc5d0\uc11c \uc911\uacf5\uad70\uc758 \uc608\ubd09\uc744 \ub9e1\uc740 \ud398\uc774\uc2a4 \ud2b9\uc218\uc784\ubb34\ubd80\ub300\uc758 \uc5ed\ud560\uc774 \ucef8\ub2e4. \ud398\uc774\uc2a4 \ud2b9\uc218\uc784\ubb34\ubd80\ub300\uac00 \ud76c\uc0dd\uc744 \uce58\ub974\uba70 \ud3ec\ub85c\uac00 \ub418\ub294 \ub3d9\uc548, \uc11c\ucabd\uc758 \ud574\ubcd1\ub300\ub294 \ud3ec\uc704\ub418\uc9c0 \uc54a\uc744 \uc218 \uc788\uc5c8\ub2e4. \uc911\uacf5\uad70\uc740 \uc720\uc5d4\uad70\uc744 \ubd81\ud55c \ub3d9\ubd81\ubd80\uc5d0\uc11c \ubab0\uc544\ub0b4\ub294\ub370 \uc131\uacf5\ud588\uc73c\ub098 \ud070 \ud53c\ud574\ub97c \uc785\uc5c8\ub2e4.", "paragraph_answer": "1950\ub144 11\uc6d4 27\uc77c, \uc911\uad6d \uc81c9\ubcd1\ub2e8\uc740 \uc7a5\uc9c4\ud638 \uc9c0\uc5ed\uc5d0\uc11c \uc5d0\ub4dc\uc6cc\ub4dc \uc54c\ubaac\ub4dc\uac00 \uc774\ub044\ub294 \ubbf8\uad6d \uc81c10\uad70\ub2e8\uc744 \uae30\uc2b5\uacf5\uaca9\ud588\ub2e4. \ud639\ub3c5\ud55c \uaca8\uc6b8\uc5d0 \uc794\ud639\ud55c 17\uc77c \uac04\uc758 \uc804\ud22c\uac00 \ubc8c\uc5b4\uc84c\ub2e4. 11\uc6d4 27\uc77c\ubd80\ud130 12\uc6d4 13\uc77c\uae4c\uc9c0 30,000\uba85\uc758 \uc720\uc5d4\uc0ac\ub839\ubd80 \ubcd1\ub825\uc774 \uc62c\ub9ac\ubc84 P. \uc2a4\ubbf8\uc2a4 \uc18c\uc7a5\uc758 \uc9c0\ud718\ub97c \ubc1b\uace0 \uc788\uc5c8\uc9c0\ub9cc \uc774\ub4e4\uc740 \uace7 \ud3ec\uc704\ub418\uc5c8\uace0, \ub9c8\uc624\uca4c\ub465\uc758 \uc720\uc5d4\uad70 \uaca9\ud30c \uc9c0\uc2dc\ub97c \ubc1b\uc740 \uc479\uc2a4\ub8ec\uc774 \uc774\ub044\ub294 \uc57d 120,000\uba85\uc758 \uc911\uacf5\uad70\uc774 \uc774\ub4e4\uc744 \uacf5\uaca9\ud588\ub2e4. \uc720\uc5d4\uad70\uc740 \uc774\ub7ec\ud55c \uc0c1\ud669\uc5d0\ub3c4 \ubd88\uad6c\ud558\uace0 \ucca0\uc218\ub97c \uc704\ud574 \uc2f8\uc6c0\uc744 \uc774\uc5b4\ub098\uac14\uc73c\uba70 \uc911\uacf5\uad70\uc758 \uc0ac\uc0c1\uc790\ub97c \ub298\ub824\uac00\uba70 \ud3ec\uc704\ub97c \ub3cc\ud30c\ud588\ub2e4. \uc81c10\uad70\ub2e8\uc758 \ud765\ub0a8 \ucca0\uc218 \uc791\uc804\uc774 \uc720\uc5d4\uad70\uc758 \ubd81\ud55c \ucca0\uc218\uc758 \ub9c8\uc9c0\ub9c9 \ub2e8\uacc4\uc600\ub2e4. \ud574\ubcd1\ub300\uac00 \ucca0\uc218\uc5d0 \uc131\uacf5\ud560 \uc218 \uc788\uc5c8\ub358 \uc774\uc720\ub294 \ub3d9\ucabd\uc5d0\uc11c \uc911\uacf5\uad70\uc758 \uc608\ubd09\uc744 \ub9e1\uc740 \ud398\uc774\uc2a4 \ud2b9\uc218\uc784\ubb34\ubd80\ub300\uc758 \uc5ed\ud560\uc774 \ucef8\ub2e4. \ud398\uc774\uc2a4 \ud2b9\uc218\uc784\ubb34\ubd80\ub300\uac00 \ud76c\uc0dd\uc744 \uce58\ub974\uba70 \ud3ec\ub85c\uac00 \ub418\ub294 \ub3d9\uc548, \uc11c\ucabd\uc758 \ud574\ubcd1\ub300\ub294 \ud3ec\uc704\ub418\uc9c0 \uc54a\uc744 \uc218 \uc788\uc5c8\ub2e4. \uc911\uacf5\uad70\uc740 \uc720\uc5d4\uad70\uc744 \ubd81\ud55c \ub3d9\ubd81\ubd80\uc5d0\uc11c \ubab0\uc544\ub0b4\ub294\ub370 \uc131\uacf5\ud588\uc73c\ub098 \ud070 \ud53c\ud574\ub97c \uc785\uc5c8\ub2e4.", "sentence_answer": "1950\ub144 11\uc6d4 27\uc77c, \uc911\uad6d \uc81c9\ubcd1\ub2e8\uc740 \uc7a5\uc9c4\ud638 \uc9c0\uc5ed\uc5d0\uc11c \uc5d0\ub4dc\uc6cc\ub4dc \uc54c\ubaac\ub4dc\uac00 \uc774\ub044\ub294 \ubbf8\uad6d \uc81c10\uad70\ub2e8\uc744 \uae30\uc2b5\uacf5\uaca9\ud588\ub2e4.", "paragraph_id": "6592671-0-0", "paragraph_question": "question: 1950\ub144 \uc911\uad6d \uc81c9\ubcd1\ub2e8\uc774 \ubbf8\uad6d \uc81c10\uad70\ub2e8\uc744 \uacf5\uaca9\ud55c \uc9c0\uc5ed\uc740 \uc5b4\ub514\uc778\uac00?, context: 1950\ub144 11\uc6d4 27\uc77c, \uc911\uad6d \uc81c9\ubcd1\ub2e8\uc740 \uc7a5\uc9c4\ud638 \uc9c0\uc5ed\uc5d0\uc11c \uc5d0\ub4dc\uc6cc\ub4dc \uc54c\ubaac\ub4dc\uac00 \uc774\ub044\ub294 \ubbf8\uad6d \uc81c10\uad70\ub2e8\uc744 \uae30\uc2b5\uacf5\uaca9\ud588\ub2e4. \ud639\ub3c5\ud55c \uaca8\uc6b8\uc5d0 \uc794\ud639\ud55c 17\uc77c \uac04\uc758 \uc804\ud22c\uac00 \ubc8c\uc5b4\uc84c\ub2e4. 11\uc6d4 27\uc77c\ubd80\ud130 12\uc6d4 13\uc77c\uae4c\uc9c0 30,000\uba85\uc758 \uc720\uc5d4\uc0ac\ub839\ubd80 \ubcd1\ub825\uc774 \uc62c\ub9ac\ubc84 P. \uc2a4\ubbf8\uc2a4 \uc18c\uc7a5\uc758 \uc9c0\ud718\ub97c \ubc1b\uace0 \uc788\uc5c8\uc9c0\ub9cc \uc774\ub4e4\uc740 \uace7 \ud3ec\uc704\ub418\uc5c8\uace0, \ub9c8\uc624\uca4c\ub465\uc758 \uc720\uc5d4\uad70 \uaca9\ud30c \uc9c0\uc2dc\ub97c \ubc1b\uc740 \uc479\uc2a4\ub8ec\uc774 \uc774\ub044\ub294 \uc57d 120,000\uba85\uc758 \uc911\uacf5\uad70\uc774 \uc774\ub4e4\uc744 \uacf5\uaca9\ud588\ub2e4. \uc720\uc5d4\uad70\uc740 \uc774\ub7ec\ud55c \uc0c1\ud669\uc5d0\ub3c4 \ubd88\uad6c\ud558\uace0 \ucca0\uc218\ub97c \uc704\ud574 \uc2f8\uc6c0\uc744 \uc774\uc5b4\ub098\uac14\uc73c\uba70 \uc911\uacf5\uad70\uc758 \uc0ac\uc0c1\uc790\ub97c \ub298\ub824\uac00\uba70 \ud3ec\uc704\ub97c \ub3cc\ud30c\ud588\ub2e4. \uc81c10\uad70\ub2e8\uc758 \ud765\ub0a8 \ucca0\uc218 \uc791\uc804\uc774 \uc720\uc5d4\uad70\uc758 \ubd81\ud55c \ucca0\uc218\uc758 \ub9c8\uc9c0\ub9c9 \ub2e8\uacc4\uc600\ub2e4. \ud574\ubcd1\ub300\uac00 \ucca0\uc218\uc5d0 \uc131\uacf5\ud560 \uc218 \uc788\uc5c8\ub358 \uc774\uc720\ub294 \ub3d9\ucabd\uc5d0\uc11c \uc911\uacf5\uad70\uc758 \uc608\ubd09\uc744 \ub9e1\uc740 \ud398\uc774\uc2a4 \ud2b9\uc218\uc784\ubb34\ubd80\ub300\uc758 \uc5ed\ud560\uc774 \ucef8\ub2e4. \ud398\uc774\uc2a4 \ud2b9\uc218\uc784\ubb34\ubd80\ub300\uac00 \ud76c\uc0dd\uc744 \uce58\ub974\uba70 \ud3ec\ub85c\uac00 \ub418\ub294 \ub3d9\uc548, \uc11c\ucabd\uc758 \ud574\ubcd1\ub300\ub294 \ud3ec\uc704\ub418\uc9c0 \uc54a\uc744 \uc218 \uc788\uc5c8\ub2e4. \uc911\uacf5\uad70\uc740 \uc720\uc5d4\uad70\uc744 \ubd81\ud55c \ub3d9\ubd81\ubd80\uc5d0\uc11c \ubab0\uc544\ub0b4\ub294\ub370 \uc131\uacf5\ud588\uc73c\ub098 \ud070 \ud53c\ud574\ub97c \uc785\uc5c8\ub2e4."} {"question": "\uc720\uace0\uc2ac\ub77c\ube44\uc544\uc758 \ud559\uc0b4\uc758 \uacc4\uae30\uac00 \ub41c \ub9d0\ub85c \ud788\ud2c0\ub7ec\uac00 \uc778\uac04 \uc774\ud558\ub77c\uace0 \uc5b8\uae09\ud55c \ubbfc\uc871\uc740 \ub204\uad6c\uc778\uac00?", "paragraph": "\ub098\uce58\ub294 \ud788\ud2c0\ub7ec\uc758 \uc758\uc911\uc5d0 \ub530\ub77c \ubc1c\uce78\uc5d0\uc11c 58\ub9cc 1\ucc9c\uba85\uc758 \uc720\uace0\uc2ac\ub77c\ube0c\uc778\uc744 \ud559\uc0b4\ud588\ub2e4. \uc774\uc5d0\ub294 \ud06c\ub85c\uc544\ud2f0\uc544\uc758 \ud30c\uc2dc\uc2a4\ud2b8 \ub3d9\ub9f9\uc73c\ub85c \ubc18\uc720\uace0\uc2ac\ub77c\ube44\uc544 \ubd84\ub9ac\uc8fc\uc758\uc790\uc778 \uc6b0\uc2a4\ud0c0\uc138(Ustase)\uac00 \ud569\uc138\ud558\uc600\ub2e4. \ud788\ud2c0\ub7ec\ub294 \ubc1c\uce78\uc5d0\uc11c \uc720\uace0\uc2ac\ub77c\ube0c\uc778\uc744 \ubaa8\ub450 \ud559\uc0b4\ud558\ub77c\ub294 \uc9c1\uc811\uc801\uc778 \uba85\ub839\uc740 \ub0b4\ub9ac\uc9c0 \uc54a\uc558\uc9c0\ub9cc \u2018\uc138\ub974\ube44\uc544\uc778\ub4e4\uc740 \uc778\uac04 \uc774\ud558(Untermensch)\u2019\ub77c\uace0 \ud45c\uba85\ud568\uc73c\ub85c\uc368 \uac04\uc811\uc801\uc778 \ud559\uc0b4 \uba85\ub839\uc744 \ub0b4\ub9b0 \uac83\uc774\ub098 \ub9c8\ucc2c\uac00\uc9c0\uc600\ub2e4. \uadf8\ub9ac\ud558\uc5ec \uc720\uace0\uc2ac\ub77c\ube44\uc544\uc5d0\uc11c\ub3c4 \uc138\ub974\ube44\uc544\uc778\ub4e4\uc774 \uac00\uc7a5 \ub9ce\uc774 \ud76c\uc0dd\ub418\uc5c8\ub2e4. \ub098\uce58\uc758 \ub3d9\uc870\uc790\uc778 \uc720\uc2a4\ud0c0\uc138\ub294 \ubcc4\ubcc4 \uc218\ub2e8\uacfc \ubc29\ubc95\uc744 \ub3d9\uc6d0\ud558\uc5ec \ub098\uce58\uc758 \uc720\uace0\uc2ac\ub77c\ube44\uc544 \uc810\ub839\uc9c0\uc5ed \uad34\ub8b0\uc815\uad8c\uc73c\ub85c\uc11c \uc138\ub974\ube44\uc544\uc778\ub4e4\uc744 \uccb4\ud3ec\ud558\ub294\ub370 \uc55e\uc7a5\uc130\ub2e4. \ubcf4\uc2a4\ub2c8\uc544\uc778\uacfc \ud06c\ub85c\uc544\ud2f0\uc544\uc778\ub3c4 \uc0c1\ub2f9\uc218\uac00 \ud76c\uc0dd\ub418\uc5c8\ub2e4. \uc8fc\ub85c \uc57c\uc138\ub178\ubc14\ud06c(Jasenovac)\uac15\uc81c\uc218\uc6a9\uc18c\uc5d0\uc11c \uc8fd\uc5c8\ub2e4. \uc720\uace0\uc2ac\ub77c\ube44\uc544\uc758 \uc6b0\uc2a4\ud0c0\uc138 \ub2f9\uad6d\uc740 1941\ub144\ubd80\ud130 45\ub144 \uc0ac\uc774\uc5d0 \ud06c\ub85c\uc544\ud2f0\uc544\uc5d0 \uc5ec\ub7ec \uacf3\uc758 \uac15\uc81c\uc218\uc6a9\uc18c\ub97c \uc124\uce58\ud558\uc600\ub2e4. \uc774\uacf3\uc5d0\uc11c \uc138\ub974\ube44\uc544\uc778, \uc720\ud0dc\uc778, \uc9d1\uc2dc, \ubb34\uc2ac\ub9bc(\ubcf4\uc2a4\ub2c8\uc544\uc778), \uadf8\ub9ac\uace0 \uac00\ud1a8\ub9ad\uad50\ub3c4\uac00 \uc544\ub2cc \uc18c\uc218\ubbfc\uc871\ub4e4\uc774 \ub04c\ub824\uc640 \uc8fd\uc784\uc744 \ub2f9\ud558\uc600\ub2e4. \ubb3c\ub860 \ud06c\ub85c\uc544\ud2f0\uc544\uc758 \uc815\uce58\ubc94\ub4e4\uacfc \ubc18\uc885\uad50\uc778\ub4e4\ub3c4 \ud76c\uc0dd\ub418\uc5c8\ub2e4.", "answer": "\uc138\ub974\ube44\uc544\uc778", "sentence": "\ud788\ud2c0\ub7ec\ub294 \ubc1c\uce78\uc5d0\uc11c \uc720\uace0\uc2ac\ub77c\ube0c\uc778\uc744 \ubaa8\ub450 \ud559\uc0b4\ud558\ub77c\ub294 \uc9c1\uc811\uc801\uc778 \uba85\ub839\uc740 \ub0b4\ub9ac\uc9c0 \uc54a\uc558\uc9c0\ub9cc \u2018 \uc138\ub974\ube44\uc544\uc778 \ub4e4\uc740 \uc778\uac04 \uc774\ud558(Untermensch)\u2019\ub77c\uace0 \ud45c\uba85\ud568\uc73c\ub85c\uc368 \uac04\uc811\uc801\uc778 \ud559\uc0b4 \uba85\ub839\uc744 \ub0b4\ub9b0 \uac83\uc774\ub098 \ub9c8\ucc2c\uac00\uc9c0\uc600\ub2e4.", "paragraph_sentence": "\ub098\uce58\ub294 \ud788\ud2c0\ub7ec\uc758 \uc758\uc911\uc5d0 \ub530\ub77c \ubc1c\uce78\uc5d0\uc11c 58\ub9cc 1\ucc9c\uba85\uc758 \uc720\uace0\uc2ac\ub77c\ube0c\uc778\uc744 \ud559\uc0b4\ud588\ub2e4. \uc774\uc5d0\ub294 \ud06c\ub85c\uc544\ud2f0\uc544\uc758 \ud30c\uc2dc\uc2a4\ud2b8 \ub3d9\ub9f9\uc73c\ub85c \ubc18\uc720\uace0\uc2ac\ub77c\ube44\uc544 \ubd84\ub9ac\uc8fc\uc758\uc790\uc778 \uc6b0\uc2a4\ud0c0\uc138(Ustase)\uac00 \ud569\uc138\ud558\uc600\ub2e4. \ud788\ud2c0\ub7ec\ub294 \ubc1c\uce78\uc5d0\uc11c \uc720\uace0\uc2ac\ub77c\ube0c\uc778\uc744 \ubaa8\ub450 \ud559\uc0b4\ud558\ub77c\ub294 \uc9c1\uc811\uc801\uc778 \uba85\ub839\uc740 \ub0b4\ub9ac\uc9c0 \uc54a\uc558\uc9c0\ub9cc \u2018 \uc138\ub974\ube44\uc544\uc778 \ub4e4\uc740 \uc778\uac04 \uc774\ud558(Untermensch)\u2019\ub77c\uace0 \ud45c\uba85\ud568\uc73c\ub85c\uc368 \uac04\uc811\uc801\uc778 \ud559\uc0b4 \uba85\ub839\uc744 \ub0b4\ub9b0 \uac83\uc774\ub098 \ub9c8\ucc2c\uac00\uc9c0\uc600\ub2e4. \uadf8\ub9ac\ud558\uc5ec \uc720\uace0\uc2ac\ub77c\ube44\uc544\uc5d0\uc11c\ub3c4 \uc138\ub974\ube44\uc544\uc778\ub4e4\uc774 \uac00\uc7a5 \ub9ce\uc774 \ud76c\uc0dd\ub418\uc5c8\ub2e4. \ub098\uce58\uc758 \ub3d9\uc870\uc790\uc778 \uc720\uc2a4\ud0c0\uc138\ub294 \ubcc4\ubcc4 \uc218\ub2e8\uacfc \ubc29\ubc95\uc744 \ub3d9\uc6d0\ud558\uc5ec \ub098\uce58\uc758 \uc720\uace0\uc2ac\ub77c\ube44\uc544 \uc810\ub839\uc9c0\uc5ed \uad34\ub8b0\uc815\uad8c\uc73c\ub85c\uc11c \uc138\ub974\ube44\uc544\uc778\ub4e4\uc744 \uccb4\ud3ec\ud558\ub294\ub370 \uc55e\uc7a5\uc130\ub2e4. \ubcf4\uc2a4\ub2c8\uc544\uc778\uacfc \ud06c\ub85c\uc544\ud2f0\uc544\uc778\ub3c4 \uc0c1\ub2f9\uc218\uac00 \ud76c\uc0dd\ub418\uc5c8\ub2e4. \uc8fc\ub85c \uc57c\uc138\ub178\ubc14\ud06c(Jasenovac)\uac15\uc81c\uc218\uc6a9\uc18c\uc5d0\uc11c \uc8fd\uc5c8\ub2e4. \uc720\uace0\uc2ac\ub77c\ube44\uc544\uc758 \uc6b0\uc2a4\ud0c0\uc138 \ub2f9\uad6d\uc740 1941\ub144\ubd80\ud130 45\ub144 \uc0ac\uc774\uc5d0 \ud06c\ub85c\uc544\ud2f0\uc544\uc5d0 \uc5ec\ub7ec \uacf3\uc758 \uac15\uc81c\uc218\uc6a9\uc18c\ub97c \uc124\uce58\ud558\uc600\ub2e4. \uc774\uacf3\uc5d0\uc11c \uc138\ub974\ube44\uc544\uc778, \uc720\ud0dc\uc778, \uc9d1\uc2dc, \ubb34\uc2ac\ub9bc(\ubcf4\uc2a4\ub2c8\uc544\uc778), \uadf8\ub9ac\uace0 \uac00\ud1a8\ub9ad\uad50\ub3c4\uac00 \uc544\ub2cc \uc18c\uc218\ubbfc\uc871\ub4e4\uc774 \ub04c\ub824\uc640 \uc8fd\uc784\uc744 \ub2f9\ud558\uc600\ub2e4. \ubb3c\ub860 \ud06c\ub85c\uc544\ud2f0\uc544\uc758 \uc815\uce58\ubc94\ub4e4\uacfc \ubc18\uc885\uad50\uc778\ub4e4\ub3c4 \ud76c\uc0dd\ub418\uc5c8\ub2e4.", "paragraph_answer": "\ub098\uce58\ub294 \ud788\ud2c0\ub7ec\uc758 \uc758\uc911\uc5d0 \ub530\ub77c \ubc1c\uce78\uc5d0\uc11c 58\ub9cc 1\ucc9c\uba85\uc758 \uc720\uace0\uc2ac\ub77c\ube0c\uc778\uc744 \ud559\uc0b4\ud588\ub2e4. \uc774\uc5d0\ub294 \ud06c\ub85c\uc544\ud2f0\uc544\uc758 \ud30c\uc2dc\uc2a4\ud2b8 \ub3d9\ub9f9\uc73c\ub85c \ubc18\uc720\uace0\uc2ac\ub77c\ube44\uc544 \ubd84\ub9ac\uc8fc\uc758\uc790\uc778 \uc6b0\uc2a4\ud0c0\uc138(Ustase)\uac00 \ud569\uc138\ud558\uc600\ub2e4. \ud788\ud2c0\ub7ec\ub294 \ubc1c\uce78\uc5d0\uc11c \uc720\uace0\uc2ac\ub77c\ube0c\uc778\uc744 \ubaa8\ub450 \ud559\uc0b4\ud558\ub77c\ub294 \uc9c1\uc811\uc801\uc778 \uba85\ub839\uc740 \ub0b4\ub9ac\uc9c0 \uc54a\uc558\uc9c0\ub9cc \u2018 \uc138\ub974\ube44\uc544\uc778 \ub4e4\uc740 \uc778\uac04 \uc774\ud558(Untermensch)\u2019\ub77c\uace0 \ud45c\uba85\ud568\uc73c\ub85c\uc368 \uac04\uc811\uc801\uc778 \ud559\uc0b4 \uba85\ub839\uc744 \ub0b4\ub9b0 \uac83\uc774\ub098 \ub9c8\ucc2c\uac00\uc9c0\uc600\ub2e4. \uadf8\ub9ac\ud558\uc5ec \uc720\uace0\uc2ac\ub77c\ube44\uc544\uc5d0\uc11c\ub3c4 \uc138\ub974\ube44\uc544\uc778\ub4e4\uc774 \uac00\uc7a5 \ub9ce\uc774 \ud76c\uc0dd\ub418\uc5c8\ub2e4. \ub098\uce58\uc758 \ub3d9\uc870\uc790\uc778 \uc720\uc2a4\ud0c0\uc138\ub294 \ubcc4\ubcc4 \uc218\ub2e8\uacfc \ubc29\ubc95\uc744 \ub3d9\uc6d0\ud558\uc5ec \ub098\uce58\uc758 \uc720\uace0\uc2ac\ub77c\ube44\uc544 \uc810\ub839\uc9c0\uc5ed \uad34\ub8b0\uc815\uad8c\uc73c\ub85c\uc11c \uc138\ub974\ube44\uc544\uc778\ub4e4\uc744 \uccb4\ud3ec\ud558\ub294\ub370 \uc55e\uc7a5\uc130\ub2e4. \ubcf4\uc2a4\ub2c8\uc544\uc778\uacfc \ud06c\ub85c\uc544\ud2f0\uc544\uc778\ub3c4 \uc0c1\ub2f9\uc218\uac00 \ud76c\uc0dd\ub418\uc5c8\ub2e4. \uc8fc\ub85c \uc57c\uc138\ub178\ubc14\ud06c(Jasenovac)\uac15\uc81c\uc218\uc6a9\uc18c\uc5d0\uc11c \uc8fd\uc5c8\ub2e4. \uc720\uace0\uc2ac\ub77c\ube44\uc544\uc758 \uc6b0\uc2a4\ud0c0\uc138 \ub2f9\uad6d\uc740 1941\ub144\ubd80\ud130 45\ub144 \uc0ac\uc774\uc5d0 \ud06c\ub85c\uc544\ud2f0\uc544\uc5d0 \uc5ec\ub7ec \uacf3\uc758 \uac15\uc81c\uc218\uc6a9\uc18c\ub97c \uc124\uce58\ud558\uc600\ub2e4. \uc774\uacf3\uc5d0\uc11c \uc138\ub974\ube44\uc544\uc778, \uc720\ud0dc\uc778, \uc9d1\uc2dc, \ubb34\uc2ac\ub9bc(\ubcf4\uc2a4\ub2c8\uc544\uc778), \uadf8\ub9ac\uace0 \uac00\ud1a8\ub9ad\uad50\ub3c4\uac00 \uc544\ub2cc \uc18c\uc218\ubbfc\uc871\ub4e4\uc774 \ub04c\ub824\uc640 \uc8fd\uc784\uc744 \ub2f9\ud558\uc600\ub2e4. \ubb3c\ub860 \ud06c\ub85c\uc544\ud2f0\uc544\uc758 \uc815\uce58\ubc94\ub4e4\uacfc \ubc18\uc885\uad50\uc778\ub4e4\ub3c4 \ud76c\uc0dd\ub418\uc5c8\ub2e4.", "sentence_answer": "\ud788\ud2c0\ub7ec\ub294 \ubc1c\uce78\uc5d0\uc11c \uc720\uace0\uc2ac\ub77c\ube0c\uc778\uc744 \ubaa8\ub450 \ud559\uc0b4\ud558\ub77c\ub294 \uc9c1\uc811\uc801\uc778 \uba85\ub839\uc740 \ub0b4\ub9ac\uc9c0 \uc54a\uc558\uc9c0\ub9cc \u2018 \uc138\ub974\ube44\uc544\uc778 \ub4e4\uc740 \uc778\uac04 \uc774\ud558(Untermensch)\u2019\ub77c\uace0 \ud45c\uba85\ud568\uc73c\ub85c\uc368 \uac04\uc811\uc801\uc778 \ud559\uc0b4 \uba85\ub839\uc744 \ub0b4\ub9b0 \uac83\uc774\ub098 \ub9c8\ucc2c\uac00\uc9c0\uc600\ub2e4.", "paragraph_id": "6571960-39-2", "paragraph_question": "question: \uc720\uace0\uc2ac\ub77c\ube44\uc544\uc758 \ud559\uc0b4\uc758 \uacc4\uae30\uac00 \ub41c \ub9d0\ub85c \ud788\ud2c0\ub7ec\uac00 \uc778\uac04 \uc774\ud558\ub77c\uace0 \uc5b8\uae09\ud55c \ubbfc\uc871\uc740 \ub204\uad6c\uc778\uac00?, context: \ub098\uce58\ub294 \ud788\ud2c0\ub7ec\uc758 \uc758\uc911\uc5d0 \ub530\ub77c \ubc1c\uce78\uc5d0\uc11c 58\ub9cc 1\ucc9c\uba85\uc758 \uc720\uace0\uc2ac\ub77c\ube0c\uc778\uc744 \ud559\uc0b4\ud588\ub2e4. \uc774\uc5d0\ub294 \ud06c\ub85c\uc544\ud2f0\uc544\uc758 \ud30c\uc2dc\uc2a4\ud2b8 \ub3d9\ub9f9\uc73c\ub85c \ubc18\uc720\uace0\uc2ac\ub77c\ube44\uc544 \ubd84\ub9ac\uc8fc\uc758\uc790\uc778 \uc6b0\uc2a4\ud0c0\uc138(Ustase)\uac00 \ud569\uc138\ud558\uc600\ub2e4. \ud788\ud2c0\ub7ec\ub294 \ubc1c\uce78\uc5d0\uc11c \uc720\uace0\uc2ac\ub77c\ube0c\uc778\uc744 \ubaa8\ub450 \ud559\uc0b4\ud558\ub77c\ub294 \uc9c1\uc811\uc801\uc778 \uba85\ub839\uc740 \ub0b4\ub9ac\uc9c0 \uc54a\uc558\uc9c0\ub9cc \u2018\uc138\ub974\ube44\uc544\uc778\ub4e4\uc740 \uc778\uac04 \uc774\ud558(Untermensch)\u2019\ub77c\uace0 \ud45c\uba85\ud568\uc73c\ub85c\uc368 \uac04\uc811\uc801\uc778 \ud559\uc0b4 \uba85\ub839\uc744 \ub0b4\ub9b0 \uac83\uc774\ub098 \ub9c8\ucc2c\uac00\uc9c0\uc600\ub2e4. \uadf8\ub9ac\ud558\uc5ec \uc720\uace0\uc2ac\ub77c\ube44\uc544\uc5d0\uc11c\ub3c4 \uc138\ub974\ube44\uc544\uc778\ub4e4\uc774 \uac00\uc7a5 \ub9ce\uc774 \ud76c\uc0dd\ub418\uc5c8\ub2e4. \ub098\uce58\uc758 \ub3d9\uc870\uc790\uc778 \uc720\uc2a4\ud0c0\uc138\ub294 \ubcc4\ubcc4 \uc218\ub2e8\uacfc \ubc29\ubc95\uc744 \ub3d9\uc6d0\ud558\uc5ec \ub098\uce58\uc758 \uc720\uace0\uc2ac\ub77c\ube44\uc544 \uc810\ub839\uc9c0\uc5ed \uad34\ub8b0\uc815\uad8c\uc73c\ub85c\uc11c \uc138\ub974\ube44\uc544\uc778\ub4e4\uc744 \uccb4\ud3ec\ud558\ub294\ub370 \uc55e\uc7a5\uc130\ub2e4. \ubcf4\uc2a4\ub2c8\uc544\uc778\uacfc \ud06c\ub85c\uc544\ud2f0\uc544\uc778\ub3c4 \uc0c1\ub2f9\uc218\uac00 \ud76c\uc0dd\ub418\uc5c8\ub2e4. \uc8fc\ub85c \uc57c\uc138\ub178\ubc14\ud06c(Jasenovac)\uac15\uc81c\uc218\uc6a9\uc18c\uc5d0\uc11c \uc8fd\uc5c8\ub2e4. \uc720\uace0\uc2ac\ub77c\ube44\uc544\uc758 \uc6b0\uc2a4\ud0c0\uc138 \ub2f9\uad6d\uc740 1941\ub144\ubd80\ud130 45\ub144 \uc0ac\uc774\uc5d0 \ud06c\ub85c\uc544\ud2f0\uc544\uc5d0 \uc5ec\ub7ec \uacf3\uc758 \uac15\uc81c\uc218\uc6a9\uc18c\ub97c \uc124\uce58\ud558\uc600\ub2e4. \uc774\uacf3\uc5d0\uc11c \uc138\ub974\ube44\uc544\uc778, \uc720\ud0dc\uc778, \uc9d1\uc2dc, \ubb34\uc2ac\ub9bc(\ubcf4\uc2a4\ub2c8\uc544\uc778), \uadf8\ub9ac\uace0 \uac00\ud1a8\ub9ad\uad50\ub3c4\uac00 \uc544\ub2cc \uc18c\uc218\ubbfc\uc871\ub4e4\uc774 \ub04c\ub824\uc640 \uc8fd\uc784\uc744 \ub2f9\ud558\uc600\ub2e4. \ubb3c\ub860 \ud06c\ub85c\uc544\ud2f0\uc544\uc758 \uc815\uce58\ubc94\ub4e4\uacfc \ubc18\uc885\uad50\uc778\ub4e4\ub3c4 \ud76c\uc0dd\ub418\uc5c8\ub2e4."} {"question": "\uc8fc\ubcd1\uc9c4\uc5d0\uac8c \uac1c\uc5c5 \ucd95\ud558\ub97c \ud574\uc8fc\ub7ec \uc654\ub2e4\uac00 \ub3c8\uc744 \ube4c\ub824 \uc900 \uc0ac\ub78c\uc740?", "paragraph": "\uc6b0\uc5ec\uace1\uc808 \ub05d\uc5d0 \uce74\ud398 \uac1c\uc7a5\uc77c\uc774 \ub2e4\uac00\uc640 \uce74\ud398\uc5d0 \ucc3e\uc544\uac00\ubcf4\ub2c8 \uc81c\uc784\uc2a4 \ub518 \uc0ac\uc9c4\ub9cc \uac78\ub824\uc788\uace0 \uc544\ubb34\uac83\ub3c4 \uc5c6\uc5c8\ub2e4. \uace0\uc0ac\ub3c4 \uc9c0\ub0b4\uc57c \ud558\ub294\ub370 \ub3c8\ub3c4 \uc804\ud600 \uc5c6\uc5c8\ub2e4. \uadf8\ub7ec\ub358\uc911 \uce5c\uad6c\uc774\uc790 \ud76c\uadf9\uc778\uc778 \uc7a5\ub450\uc11d\uc774 \uac1c\uc5c5 \ucd95\ud558\ub97c \ud574\uc8fc\ub7ec \uc628 \uac83\uc744 \ubcf4\uace0 \ub3c8\uc744 \ube4c\ub824 \uace0\uc0ac\ub97c \uc9c0\ub0b4\uace0 \ud654\ubd84\ub3c4 \uba87\uac1c \uad6c\uc785\ud574 \uac16\ub2e4\ub193\uc558\uace0 1983\ub144\uc5d0 200\ub9cc\uc6d0\uc744 \ud22c\uc790\ud55c \uce74\ud398\ub97c \uac1c\uc5c5\ud588\ub2e4. \uac1c\uc5c5\ub0a0\ubd80\ud130 \uc0ac\ub78c\ub4e4\uc774 \ubab0\ub824\uc640 24\uc2dc\uac04 \uc601\uc5c5\ud588\ub2e4. \uce74\ud398 \uc601\uc5c5\uc744 \ud558\uba74\uc11c\ub3c4 \ub3c8\uc744 \uafd4\uc900 \uc0ac\ub78c\ub4e4\uc774 \ucc3e\uc544\uc640 \uac1a\uc73c\ub77c\uace0 \ub3c5\ucd09\ud588\uc73c\ub098 \uc7a5\uc0ac\uac00 \uc798\ub41c\ub2e4\ub294 \uc810\uc744 \uac15\uc870\ud558\uba70 \uc548\uc2ec\uc2dc\ucf30\uace0 \ub9e4\uc77c \ub9e4\ucd9c\uc744 \ubd84\ud560\ud558\uc5ec \uac1a\uc73c\uba74\uc11c 6\uac1c\uc6d4 \ub9cc\uc5d0 \ubaa8\ub4e0 \ube5a\uc744 \ub2e4 \uac1a\uc558\ub2e4. \ucc28\ubcc4\ud654\ub97c \uc704\ud574 \uc8fc\ubcd1\uc9c4\uc740 \uce74\uc6b0\ubcf4\uc774 \ubaa8\uc790 \uac04\ud310\uc744 \uc77c\ubc18 \uac04\ud310\ubcf4\ub2e4 5~6\ubc30 \ub354 \ud06c\uace0 \ube44\uc2fc \uac83\uc73c\ub85c \ub2ec\uc544\ub193\uc544 \uadf8 \uadfc\ucc98\ub97c \uc6b4\ud589\ud558\ub294 \ud0dd\uc2dc\uae30\uc0ac\ub4e4\uc774 \uc81c\uc784\uc2a4\ub518 \uce74\ud398\ub97c \uae30\uc900\uc73c\ub85c \uc9c0\ub9ac\ub97c \ucc3e\uc544\uac00\uae30\ub3c4 \ud560 \uc815\ub3c4\uc600\ub2e4. \ud55c\ud3b8, \uce74\ud398\uba85 \uc81c\uc784\uc2a4\ub518\uc740 \uc678\uad6d\uc5b4\uc600\uae30 \ub54c\ubb38\uc5d0 \uc0c1\ud638 \ub4f1\ub85d\uc774 \ubd88\uac00\ub2a5\ud588\ub2e4. \uadf8\ub9ac\ud558\uc5ec \u2018\uc7e4 \uc784\uc528\ub4e0\u2019\uc774\ub77c\ub294 \uc758\ubb38\ud615 \ud55c\uad6d\uc5b4\ub85c \uaca8\uc6b0 \ub4f1\ub85d\uc744 \ud588\ub2e4.", "answer": "\uc7a5\ub450\uc11d", "sentence": "\uadf8\ub7ec\ub358\uc911 \uce5c\uad6c\uc774\uc790 \ud76c\uadf9\uc778\uc778 \uc7a5\ub450\uc11d \uc774 \uac1c\uc5c5 \ucd95\ud558\ub97c \ud574\uc8fc\ub7ec \uc628 \uac83\uc744 \ubcf4\uace0 \ub3c8\uc744 \ube4c\ub824 \uace0\uc0ac\ub97c \uc9c0\ub0b4\uace0 \ud654\ubd84\ub3c4 \uba87\uac1c \uad6c\uc785\ud574 \uac16\ub2e4\ub193\uc558\uace0 1983\ub144\uc5d0 200\ub9cc\uc6d0\uc744 \ud22c\uc790\ud55c \uce74\ud398\ub97c \uac1c\uc5c5\ud588\ub2e4.", "paragraph_sentence": "\uc6b0\uc5ec\uace1\uc808 \ub05d\uc5d0 \uce74\ud398 \uac1c\uc7a5\uc77c\uc774 \ub2e4\uac00\uc640 \uce74\ud398\uc5d0 \ucc3e\uc544\uac00\ubcf4\ub2c8 \uc81c\uc784\uc2a4 \ub518 \uc0ac\uc9c4\ub9cc \uac78\ub824\uc788\uace0 \uc544\ubb34\uac83\ub3c4 \uc5c6\uc5c8\ub2e4. \uace0\uc0ac\ub3c4 \uc9c0\ub0b4\uc57c \ud558\ub294\ub370 \ub3c8\ub3c4 \uc804\ud600 \uc5c6\uc5c8\ub2e4. \uadf8\ub7ec\ub358\uc911 \uce5c\uad6c\uc774\uc790 \ud76c\uadf9\uc778\uc778 \uc7a5\ub450\uc11d \uc774 \uac1c\uc5c5 \ucd95\ud558\ub97c \ud574\uc8fc\ub7ec \uc628 \uac83\uc744 \ubcf4\uace0 \ub3c8\uc744 \ube4c\ub824 \uace0\uc0ac\ub97c \uc9c0\ub0b4\uace0 \ud654\ubd84\ub3c4 \uba87\uac1c \uad6c\uc785\ud574 \uac16\ub2e4\ub193\uc558\uace0 1983\ub144\uc5d0 200\ub9cc\uc6d0\uc744 \ud22c\uc790\ud55c \uce74\ud398\ub97c \uac1c\uc5c5\ud588\ub2e4. \uac1c\uc5c5\ub0a0\ubd80\ud130 \uc0ac\ub78c\ub4e4\uc774 \ubab0\ub824\uc640 24\uc2dc\uac04 \uc601\uc5c5\ud588\ub2e4. \uce74\ud398 \uc601\uc5c5\uc744 \ud558\uba74\uc11c\ub3c4 \ub3c8\uc744 \uafd4\uc900 \uc0ac\ub78c\ub4e4\uc774 \ucc3e\uc544\uc640 \uac1a\uc73c\ub77c\uace0 \ub3c5\ucd09\ud588\uc73c\ub098 \uc7a5\uc0ac\uac00 \uc798\ub41c\ub2e4\ub294 \uc810\uc744 \uac15\uc870\ud558\uba70 \uc548\uc2ec\uc2dc\ucf30\uace0 \ub9e4\uc77c \ub9e4\ucd9c\uc744 \ubd84\ud560\ud558\uc5ec \uac1a\uc73c\uba74\uc11c 6\uac1c\uc6d4 \ub9cc\uc5d0 \ubaa8\ub4e0 \ube5a\uc744 \ub2e4 \uac1a\uc558\ub2e4. \ucc28\ubcc4\ud654\ub97c \uc704\ud574 \uc8fc\ubcd1\uc9c4\uc740 \uce74\uc6b0\ubcf4\uc774 \ubaa8\uc790 \uac04\ud310\uc744 \uc77c\ubc18 \uac04\ud310\ubcf4\ub2e4 5~6\ubc30 \ub354 \ud06c\uace0 \ube44\uc2fc \uac83\uc73c\ub85c \ub2ec\uc544\ub193\uc544 \uadf8 \uadfc\ucc98\ub97c \uc6b4\ud589\ud558\ub294 \ud0dd\uc2dc\uae30\uc0ac\ub4e4\uc774 \uc81c\uc784\uc2a4\ub518 \uce74\ud398\ub97c \uae30\uc900\uc73c\ub85c \uc9c0\ub9ac\ub97c \ucc3e\uc544\uac00\uae30\ub3c4 \ud560 \uc815\ub3c4\uc600\ub2e4. \ud55c\ud3b8, \uce74\ud398\uba85 \uc81c\uc784\uc2a4\ub518\uc740 \uc678\uad6d\uc5b4\uc600\uae30 \ub54c\ubb38\uc5d0 \uc0c1\ud638 \ub4f1\ub85d\uc774 \ubd88\uac00\ub2a5\ud588\ub2e4. \uadf8\ub9ac\ud558\uc5ec \u2018\uc7e4 \uc784\uc528\ub4e0\u2019\uc774\ub77c\ub294 \uc758\ubb38\ud615 \ud55c\uad6d\uc5b4\ub85c \uaca8\uc6b0 \ub4f1\ub85d\uc744 \ud588\ub2e4.", "paragraph_answer": "\uc6b0\uc5ec\uace1\uc808 \ub05d\uc5d0 \uce74\ud398 \uac1c\uc7a5\uc77c\uc774 \ub2e4\uac00\uc640 \uce74\ud398\uc5d0 \ucc3e\uc544\uac00\ubcf4\ub2c8 \uc81c\uc784\uc2a4 \ub518 \uc0ac\uc9c4\ub9cc \uac78\ub824\uc788\uace0 \uc544\ubb34\uac83\ub3c4 \uc5c6\uc5c8\ub2e4. \uace0\uc0ac\ub3c4 \uc9c0\ub0b4\uc57c \ud558\ub294\ub370 \ub3c8\ub3c4 \uc804\ud600 \uc5c6\uc5c8\ub2e4. \uadf8\ub7ec\ub358\uc911 \uce5c\uad6c\uc774\uc790 \ud76c\uadf9\uc778\uc778 \uc7a5\ub450\uc11d \uc774 \uac1c\uc5c5 \ucd95\ud558\ub97c \ud574\uc8fc\ub7ec \uc628 \uac83\uc744 \ubcf4\uace0 \ub3c8\uc744 \ube4c\ub824 \uace0\uc0ac\ub97c \uc9c0\ub0b4\uace0 \ud654\ubd84\ub3c4 \uba87\uac1c \uad6c\uc785\ud574 \uac16\ub2e4\ub193\uc558\uace0 1983\ub144\uc5d0 200\ub9cc\uc6d0\uc744 \ud22c\uc790\ud55c \uce74\ud398\ub97c \uac1c\uc5c5\ud588\ub2e4. \uac1c\uc5c5\ub0a0\ubd80\ud130 \uc0ac\ub78c\ub4e4\uc774 \ubab0\ub824\uc640 24\uc2dc\uac04 \uc601\uc5c5\ud588\ub2e4. \uce74\ud398 \uc601\uc5c5\uc744 \ud558\uba74\uc11c\ub3c4 \ub3c8\uc744 \uafd4\uc900 \uc0ac\ub78c\ub4e4\uc774 \ucc3e\uc544\uc640 \uac1a\uc73c\ub77c\uace0 \ub3c5\ucd09\ud588\uc73c\ub098 \uc7a5\uc0ac\uac00 \uc798\ub41c\ub2e4\ub294 \uc810\uc744 \uac15\uc870\ud558\uba70 \uc548\uc2ec\uc2dc\ucf30\uace0 \ub9e4\uc77c \ub9e4\ucd9c\uc744 \ubd84\ud560\ud558\uc5ec \uac1a\uc73c\uba74\uc11c 6\uac1c\uc6d4 \ub9cc\uc5d0 \ubaa8\ub4e0 \ube5a\uc744 \ub2e4 \uac1a\uc558\ub2e4. \ucc28\ubcc4\ud654\ub97c \uc704\ud574 \uc8fc\ubcd1\uc9c4\uc740 \uce74\uc6b0\ubcf4\uc774 \ubaa8\uc790 \uac04\ud310\uc744 \uc77c\ubc18 \uac04\ud310\ubcf4\ub2e4 5~6\ubc30 \ub354 \ud06c\uace0 \ube44\uc2fc \uac83\uc73c\ub85c \ub2ec\uc544\ub193\uc544 \uadf8 \uadfc\ucc98\ub97c \uc6b4\ud589\ud558\ub294 \ud0dd\uc2dc\uae30\uc0ac\ub4e4\uc774 \uc81c\uc784\uc2a4\ub518 \uce74\ud398\ub97c \uae30\uc900\uc73c\ub85c \uc9c0\ub9ac\ub97c \ucc3e\uc544\uac00\uae30\ub3c4 \ud560 \uc815\ub3c4\uc600\ub2e4. \ud55c\ud3b8, \uce74\ud398\uba85 \uc81c\uc784\uc2a4\ub518\uc740 \uc678\uad6d\uc5b4\uc600\uae30 \ub54c\ubb38\uc5d0 \uc0c1\ud638 \ub4f1\ub85d\uc774 \ubd88\uac00\ub2a5\ud588\ub2e4. \uadf8\ub9ac\ud558\uc5ec \u2018\uc7e4 \uc784\uc528\ub4e0\u2019\uc774\ub77c\ub294 \uc758\ubb38\ud615 \ud55c\uad6d\uc5b4\ub85c \uaca8\uc6b0 \ub4f1\ub85d\uc744 \ud588\ub2e4.", "sentence_answer": "\uadf8\ub7ec\ub358\uc911 \uce5c\uad6c\uc774\uc790 \ud76c\uadf9\uc778\uc778 \uc7a5\ub450\uc11d \uc774 \uac1c\uc5c5 \ucd95\ud558\ub97c \ud574\uc8fc\ub7ec \uc628 \uac83\uc744 \ubcf4\uace0 \ub3c8\uc744 \ube4c\ub824 \uace0\uc0ac\ub97c \uc9c0\ub0b4\uace0 \ud654\ubd84\ub3c4 \uba87\uac1c \uad6c\uc785\ud574 \uac16\ub2e4\ub193\uc558\uace0 1983\ub144\uc5d0 200\ub9cc\uc6d0\uc744 \ud22c\uc790\ud55c \uce74\ud398\ub97c \uac1c\uc5c5\ud588\ub2e4.", "paragraph_id": "6555914-4-0", "paragraph_question": "question: \uc8fc\ubcd1\uc9c4\uc5d0\uac8c \uac1c\uc5c5 \ucd95\ud558\ub97c \ud574\uc8fc\ub7ec \uc654\ub2e4\uac00 \ub3c8\uc744 \ube4c\ub824 \uc900 \uc0ac\ub78c\uc740?, context: \uc6b0\uc5ec\uace1\uc808 \ub05d\uc5d0 \uce74\ud398 \uac1c\uc7a5\uc77c\uc774 \ub2e4\uac00\uc640 \uce74\ud398\uc5d0 \ucc3e\uc544\uac00\ubcf4\ub2c8 \uc81c\uc784\uc2a4 \ub518 \uc0ac\uc9c4\ub9cc \uac78\ub824\uc788\uace0 \uc544\ubb34\uac83\ub3c4 \uc5c6\uc5c8\ub2e4. \uace0\uc0ac\ub3c4 \uc9c0\ub0b4\uc57c \ud558\ub294\ub370 \ub3c8\ub3c4 \uc804\ud600 \uc5c6\uc5c8\ub2e4. \uadf8\ub7ec\ub358\uc911 \uce5c\uad6c\uc774\uc790 \ud76c\uadf9\uc778\uc778 \uc7a5\ub450\uc11d\uc774 \uac1c\uc5c5 \ucd95\ud558\ub97c \ud574\uc8fc\ub7ec \uc628 \uac83\uc744 \ubcf4\uace0 \ub3c8\uc744 \ube4c\ub824 \uace0\uc0ac\ub97c \uc9c0\ub0b4\uace0 \ud654\ubd84\ub3c4 \uba87\uac1c \uad6c\uc785\ud574 \uac16\ub2e4\ub193\uc558\uace0 1983\ub144\uc5d0 200\ub9cc\uc6d0\uc744 \ud22c\uc790\ud55c \uce74\ud398\ub97c \uac1c\uc5c5\ud588\ub2e4. \uac1c\uc5c5\ub0a0\ubd80\ud130 \uc0ac\ub78c\ub4e4\uc774 \ubab0\ub824\uc640 24\uc2dc\uac04 \uc601\uc5c5\ud588\ub2e4. \uce74\ud398 \uc601\uc5c5\uc744 \ud558\uba74\uc11c\ub3c4 \ub3c8\uc744 \uafd4\uc900 \uc0ac\ub78c\ub4e4\uc774 \ucc3e\uc544\uc640 \uac1a\uc73c\ub77c\uace0 \ub3c5\ucd09\ud588\uc73c\ub098 \uc7a5\uc0ac\uac00 \uc798\ub41c\ub2e4\ub294 \uc810\uc744 \uac15\uc870\ud558\uba70 \uc548\uc2ec\uc2dc\ucf30\uace0 \ub9e4\uc77c \ub9e4\ucd9c\uc744 \ubd84\ud560\ud558\uc5ec \uac1a\uc73c\uba74\uc11c 6\uac1c\uc6d4 \ub9cc\uc5d0 \ubaa8\ub4e0 \ube5a\uc744 \ub2e4 \uac1a\uc558\ub2e4. \ucc28\ubcc4\ud654\ub97c \uc704\ud574 \uc8fc\ubcd1\uc9c4\uc740 \uce74\uc6b0\ubcf4\uc774 \ubaa8\uc790 \uac04\ud310\uc744 \uc77c\ubc18 \uac04\ud310\ubcf4\ub2e4 5~6\ubc30 \ub354 \ud06c\uace0 \ube44\uc2fc \uac83\uc73c\ub85c \ub2ec\uc544\ub193\uc544 \uadf8 \uadfc\ucc98\ub97c \uc6b4\ud589\ud558\ub294 \ud0dd\uc2dc\uae30\uc0ac\ub4e4\uc774 \uc81c\uc784\uc2a4\ub518 \uce74\ud398\ub97c \uae30\uc900\uc73c\ub85c \uc9c0\ub9ac\ub97c \ucc3e\uc544\uac00\uae30\ub3c4 \ud560 \uc815\ub3c4\uc600\ub2e4. \ud55c\ud3b8, \uce74\ud398\uba85 \uc81c\uc784\uc2a4\ub518\uc740 \uc678\uad6d\uc5b4\uc600\uae30 \ub54c\ubb38\uc5d0 \uc0c1\ud638 \ub4f1\ub85d\uc774 \ubd88\uac00\ub2a5\ud588\ub2e4. \uadf8\ub9ac\ud558\uc5ec \u2018\uc7e4 \uc784\uc528\ub4e0\u2019\uc774\ub77c\ub294 \uc758\ubb38\ud615 \ud55c\uad6d\uc5b4\ub85c \uaca8\uc6b0 \ub4f1\ub85d\uc744 \ud588\ub2e4."} {"question": "\ud559\uc790\ub4e4\uc774 \uc870\uc9c0\uc544\ub77c\ub294 \uc774\ub984\uc758 \uc720\ub7fd\uc5b4 \uae30\uc6d0\uc124\uc5d0 \ucc98\uc74c\uc73c\ub85c \uc758\ubb38\uc744 \uc81c\uae30\ud55c \uc2dc\uae30\ub294?", "paragraph": "19\uc138\uae30\uc5d0 \ud559\uc790\ub4e4\uc740 \uc774\ub7ec\ud55c \uc720\ub7fd\uc5b4 \uae30\uc6d0\uc124\uc5d0 \ub300\ud574 \ucc98\uc74c\uc73c\ub85c \uc758\ubb38\uc744 \uc81c\uae30\ud55c\ub2e4. \ub9c8\ub9ac-\ud3a0\ub9ac\uc2dc\ud14c \ube0c\ub85c\uc138\ub294 \uc870\uc9c0\uc544\ub780 \uc774\ub984\uc774 Kuros-Cyrus-Kura-Djurzan\ub97c \ud1b5\ud574 \ubbc0\ud2b8\ud06c\ubc14\ub9ac \uac15\uc758 \uc774\ub984\uc5d0\uc11c \ub098\uc628 \uac83\uc774\ub77c\uace0 \uc8fc\uc7a5\ud588\ub2e4. \uc0ac\uc2e4 '\uc870\uc9c0\uc544'\ub294 \uc2e0\ud398\ub974\uc2dc\uc544\uc5b4 gur\u011f/gur\u011f\u0101n\uc5d0\uc11c \ube44\ub86f\ub41c \uc2dc\ub9ac\uc544\uc5b4 gurz-\u0101n/gurz-iy\u0101n\uacfc \uc544\ub78d\uc5b4 \u0135ur\u0135an/\u0135urzan\uc744 11\uc138\uae30\ub098 12\uc138\uae30\uc5d0 \ucc28\uc6a9\ud55c \ub9d0\ub85c \ubcf4\uc774\uba70, \uc2e0\ud398\ub974\uc2dc\uc544\uc5b4 \ub0b1\ub9d0 \uc790\uccb4\ub294 \uace0\ub300 \uc774\ub780\uc5b4\uc640 \uc911\uc138 \ud398\ub974\uc2dc\uc544\uc5b4\ub85c \uae30\uc6d0\uc744 \uc54c \uc218 \uc5c6\ub294 vrk\u0101n/waru\u010d\u0101n\uc5d0\uc11c \ub098\uc628 \ub4ef \ud55c\ub370 \uc774 \ub9d0\uc740 \ub3d9\ubd80 \ud2b8\ub780\uc2a4\uce74\uc2a4\ud53c\uc758 \uc9c0\uba85 \uace0\ub974\uac04(Gorgan, \uc911\uc138 \ud398\ub974\uc2dc\uc544\uc5b4\ub85c '\ub291\ub300\ub4e4\uc758 \ub545'\uc744 \ub73b\ud558\ub294 vark\u00e2na\uc5d0\uc11c \ub098\uc628 \ub9d0)\uacfc \ube44\uc2b7\ud558\ub2e4. \uc774 \ub9d0\uc740 \uc61b \uc544\ub974\uba54\ub2c8\uc544\uc5b4 Virk' (\u054e\u056b\u0580\u0584)\uc640 \uc5b4\uc6d0\uc774 \uac19\uc740 \ub4ef \ud558\uba70, \uadf8\ub9ac\uc2a4-\ub85c\ub9c8\uc2dd \ud45c\uae30\uc778 \uc774\ubca0\ub9ac(\u1f3c\u03b2\u03b7\u03c1\u03b5\u03c2, \uc774\ubca0\ub9ac\uc544 \ubc18\ub3c4\uc758 \uc774\ubca0\ub9ac\uc544\uc778\uc744 \uc774\ub974\ub294 \ub9d0\ub85c \uc774\ubbf8 \uc54c\ub824\uc9c4 \uc774\ub984\uc774\ub2e4)\uc758 \uae30\uc6d0\uc77c \uc218\ub3c4 \uc788\ub2e4.", "answer": "19\uc138\uae30", "sentence": "19\uc138\uae30 \uc5d0 \ud559\uc790\ub4e4\uc740 \uc774\ub7ec\ud55c \uc720\ub7fd\uc5b4 \uae30\uc6d0\uc124\uc5d0 \ub300\ud574 \ucc98\uc74c\uc73c\ub85c \uc758\ubb38\uc744 \uc81c\uae30\ud55c\ub2e4.", "paragraph_sentence": " 19\uc138\uae30 \uc5d0 \ud559\uc790\ub4e4\uc740 \uc774\ub7ec\ud55c \uc720\ub7fd\uc5b4 \uae30\uc6d0\uc124\uc5d0 \ub300\ud574 \ucc98\uc74c\uc73c\ub85c \uc758\ubb38\uc744 \uc81c\uae30\ud55c\ub2e4. \ub9c8\ub9ac-\ud3a0\ub9ac\uc2dc\ud14c \ube0c\ub85c\uc138\ub294 \uc870\uc9c0\uc544\ub780 \uc774\ub984\uc774 Kuros-Cyrus-Kura-Djurzan\ub97c \ud1b5\ud574 \ubbc0\ud2b8\ud06c\ubc14\ub9ac \uac15\uc758 \uc774\ub984\uc5d0\uc11c \ub098\uc628 \uac83\uc774\ub77c\uace0 \uc8fc\uc7a5\ud588\ub2e4. \uc0ac\uc2e4 '\uc870\uc9c0\uc544'\ub294 \uc2e0\ud398\ub974\uc2dc\uc544\uc5b4 gur\u011f/gur\u011f\u0101n\uc5d0\uc11c \ube44\ub86f\ub41c \uc2dc\ub9ac\uc544\uc5b4 gurz-\u0101n/gurz-iy\u0101n\uacfc \uc544\ub78d\uc5b4 \u0135ur\u0135an/\u0135urzan\uc744 11\uc138\uae30\ub098 12\uc138\uae30\uc5d0 \ucc28\uc6a9\ud55c \ub9d0\ub85c \ubcf4\uc774\uba70, \uc2e0\ud398\ub974\uc2dc\uc544\uc5b4 \ub0b1\ub9d0 \uc790\uccb4\ub294 \uace0\ub300 \uc774\ub780\uc5b4\uc640 \uc911\uc138 \ud398\ub974\uc2dc\uc544\uc5b4\ub85c \uae30\uc6d0\uc744 \uc54c \uc218 \uc5c6\ub294 vrk\u0101n/waru\u010d\u0101n\uc5d0\uc11c \ub098\uc628 \ub4ef \ud55c\ub370 \uc774 \ub9d0\uc740 \ub3d9\ubd80 \ud2b8\ub780\uc2a4\uce74\uc2a4\ud53c\uc758 \uc9c0\uba85 \uace0\ub974\uac04(Gorgan, \uc911\uc138 \ud398\ub974\uc2dc\uc544\uc5b4\ub85c '\ub291\ub300\ub4e4\uc758 \ub545'\uc744 \ub73b\ud558\ub294 vark\u00e2na\uc5d0\uc11c \ub098\uc628 \ub9d0)\uacfc \ube44\uc2b7\ud558\ub2e4. \uc774 \ub9d0\uc740 \uc61b \uc544\ub974\uba54\ub2c8\uc544\uc5b4 Virk' (\u054e\u056b\u0580\u0584)\uc640 \uc5b4\uc6d0\uc774 \uac19\uc740 \ub4ef \ud558\uba70, \uadf8\ub9ac\uc2a4-\ub85c\ub9c8\uc2dd \ud45c\uae30\uc778 \uc774\ubca0\ub9ac(\u1f3c\u03b2\u03b7\u03c1\u03b5\u03c2, \uc774\ubca0\ub9ac\uc544 \ubc18\ub3c4\uc758 \uc774\ubca0\ub9ac\uc544\uc778\uc744 \uc774\ub974\ub294 \ub9d0\ub85c \uc774\ubbf8 \uc54c\ub824\uc9c4 \uc774\ub984\uc774\ub2e4)\uc758 \uae30\uc6d0\uc77c \uc218\ub3c4 \uc788\ub2e4.", "paragraph_answer": " 19\uc138\uae30 \uc5d0 \ud559\uc790\ub4e4\uc740 \uc774\ub7ec\ud55c \uc720\ub7fd\uc5b4 \uae30\uc6d0\uc124\uc5d0 \ub300\ud574 \ucc98\uc74c\uc73c\ub85c \uc758\ubb38\uc744 \uc81c\uae30\ud55c\ub2e4. \ub9c8\ub9ac-\ud3a0\ub9ac\uc2dc\ud14c \ube0c\ub85c\uc138\ub294 \uc870\uc9c0\uc544\ub780 \uc774\ub984\uc774 Kuros-Cyrus-Kura-Djurzan\ub97c \ud1b5\ud574 \ubbc0\ud2b8\ud06c\ubc14\ub9ac \uac15\uc758 \uc774\ub984\uc5d0\uc11c \ub098\uc628 \uac83\uc774\ub77c\uace0 \uc8fc\uc7a5\ud588\ub2e4. \uc0ac\uc2e4 '\uc870\uc9c0\uc544'\ub294 \uc2e0\ud398\ub974\uc2dc\uc544\uc5b4 gur\u011f/gur\u011f\u0101n\uc5d0\uc11c \ube44\ub86f\ub41c \uc2dc\ub9ac\uc544\uc5b4 gurz-\u0101n/gurz-iy\u0101n\uacfc \uc544\ub78d\uc5b4 \u0135ur\u0135an/\u0135urzan\uc744 11\uc138\uae30\ub098 12\uc138\uae30\uc5d0 \ucc28\uc6a9\ud55c \ub9d0\ub85c \ubcf4\uc774\uba70, \uc2e0\ud398\ub974\uc2dc\uc544\uc5b4 \ub0b1\ub9d0 \uc790\uccb4\ub294 \uace0\ub300 \uc774\ub780\uc5b4\uc640 \uc911\uc138 \ud398\ub974\uc2dc\uc544\uc5b4\ub85c \uae30\uc6d0\uc744 \uc54c \uc218 \uc5c6\ub294 vrk\u0101n/waru\u010d\u0101n\uc5d0\uc11c \ub098\uc628 \ub4ef \ud55c\ub370 \uc774 \ub9d0\uc740 \ub3d9\ubd80 \ud2b8\ub780\uc2a4\uce74\uc2a4\ud53c\uc758 \uc9c0\uba85 \uace0\ub974\uac04(Gorgan, \uc911\uc138 \ud398\ub974\uc2dc\uc544\uc5b4\ub85c '\ub291\ub300\ub4e4\uc758 \ub545'\uc744 \ub73b\ud558\ub294 vark\u00e2na\uc5d0\uc11c \ub098\uc628 \ub9d0)\uacfc \ube44\uc2b7\ud558\ub2e4. \uc774 \ub9d0\uc740 \uc61b \uc544\ub974\uba54\ub2c8\uc544\uc5b4 Virk' (\u054e\u056b\u0580\u0584)\uc640 \uc5b4\uc6d0\uc774 \uac19\uc740 \ub4ef \ud558\uba70, \uadf8\ub9ac\uc2a4-\ub85c\ub9c8\uc2dd \ud45c\uae30\uc778 \uc774\ubca0\ub9ac(\u1f3c\u03b2\u03b7\u03c1\u03b5\u03c2, \uc774\ubca0\ub9ac\uc544 \ubc18\ub3c4\uc758 \uc774\ubca0\ub9ac\uc544\uc778\uc744 \uc774\ub974\ub294 \ub9d0\ub85c \uc774\ubbf8 \uc54c\ub824\uc9c4 \uc774\ub984\uc774\ub2e4)\uc758 \uae30\uc6d0\uc77c \uc218\ub3c4 \uc788\ub2e4.", "sentence_answer": " 19\uc138\uae30 \uc5d0 \ud559\uc790\ub4e4\uc740 \uc774\ub7ec\ud55c \uc720\ub7fd\uc5b4 \uae30\uc6d0\uc124\uc5d0 \ub300\ud574 \ucc98\uc74c\uc73c\ub85c \uc758\ubb38\uc744 \uc81c\uae30\ud55c\ub2e4.", "paragraph_id": "6554323-2-0", "paragraph_question": "question: \ud559\uc790\ub4e4\uc774 \uc870\uc9c0\uc544\ub77c\ub294 \uc774\ub984\uc758 \uc720\ub7fd\uc5b4 \uae30\uc6d0\uc124\uc5d0 \ucc98\uc74c\uc73c\ub85c \uc758\ubb38\uc744 \uc81c\uae30\ud55c \uc2dc\uae30\ub294?, context: 19\uc138\uae30\uc5d0 \ud559\uc790\ub4e4\uc740 \uc774\ub7ec\ud55c \uc720\ub7fd\uc5b4 \uae30\uc6d0\uc124\uc5d0 \ub300\ud574 \ucc98\uc74c\uc73c\ub85c \uc758\ubb38\uc744 \uc81c\uae30\ud55c\ub2e4. \ub9c8\ub9ac-\ud3a0\ub9ac\uc2dc\ud14c \ube0c\ub85c\uc138\ub294 \uc870\uc9c0\uc544\ub780 \uc774\ub984\uc774 Kuros-Cyrus-Kura-Djurzan\ub97c \ud1b5\ud574 \ubbc0\ud2b8\ud06c\ubc14\ub9ac \uac15\uc758 \uc774\ub984\uc5d0\uc11c \ub098\uc628 \uac83\uc774\ub77c\uace0 \uc8fc\uc7a5\ud588\ub2e4. \uc0ac\uc2e4 '\uc870\uc9c0\uc544'\ub294 \uc2e0\ud398\ub974\uc2dc\uc544\uc5b4 gur\u011f/gur\u011f\u0101n\uc5d0\uc11c \ube44\ub86f\ub41c \uc2dc\ub9ac\uc544\uc5b4 gurz-\u0101n/gurz-iy\u0101n\uacfc \uc544\ub78d\uc5b4 \u0135ur\u0135an/\u0135urzan\uc744 11\uc138\uae30\ub098 12\uc138\uae30\uc5d0 \ucc28\uc6a9\ud55c \ub9d0\ub85c \ubcf4\uc774\uba70, \uc2e0\ud398\ub974\uc2dc\uc544\uc5b4 \ub0b1\ub9d0 \uc790\uccb4\ub294 \uace0\ub300 \uc774\ub780\uc5b4\uc640 \uc911\uc138 \ud398\ub974\uc2dc\uc544\uc5b4\ub85c \uae30\uc6d0\uc744 \uc54c \uc218 \uc5c6\ub294 vrk\u0101n/waru\u010d\u0101n\uc5d0\uc11c \ub098\uc628 \ub4ef \ud55c\ub370 \uc774 \ub9d0\uc740 \ub3d9\ubd80 \ud2b8\ub780\uc2a4\uce74\uc2a4\ud53c\uc758 \uc9c0\uba85 \uace0\ub974\uac04(Gorgan, \uc911\uc138 \ud398\ub974\uc2dc\uc544\uc5b4\ub85c '\ub291\ub300\ub4e4\uc758 \ub545'\uc744 \ub73b\ud558\ub294 vark\u00e2na\uc5d0\uc11c \ub098\uc628 \ub9d0)\uacfc \ube44\uc2b7\ud558\ub2e4. \uc774 \ub9d0\uc740 \uc61b \uc544\ub974\uba54\ub2c8\uc544\uc5b4 Virk' (\u054e\u056b\u0580\u0584)\uc640 \uc5b4\uc6d0\uc774 \uac19\uc740 \ub4ef \ud558\uba70, \uadf8\ub9ac\uc2a4-\ub85c\ub9c8\uc2dd \ud45c\uae30\uc778 \uc774\ubca0\ub9ac(\u1f3c\u03b2\u03b7\u03c1\u03b5\u03c2, \uc774\ubca0\ub9ac\uc544 \ubc18\ub3c4\uc758 \uc774\ubca0\ub9ac\uc544\uc778\uc744 \uc774\ub974\ub294 \ub9d0\ub85c \uc774\ubbf8 \uc54c\ub824\uc9c4 \uc774\ub984\uc774\ub2e4)\uc758 \uae30\uc6d0\uc77c \uc218\ub3c4 \uc788\ub2e4."} {"question": "\uacf5\uc2dd \uc870\uc9c1\uc758 \ubaa9\ud45c\ub97c \ud6a8\uacfc\uc801\uc73c\ub85c \ub2ec\uc131\ud558\uae30 \uc704\ud574 \uac00\uc9c0\uace0 \uc788\ub294 \uac83\uc740?", "paragraph": "\uacf5\uc2dd \uc870\uc9c1\uc740 \uc9c1\ubb34, \ucc45\uc784, \uad8c\ud55c\uc744 \uc911\uc2ec\uc73c\ub85c \uc870\uc9c1 \uad6c\uc131\uc6d0 \ud639\uc740 \uc791\uc5c5 \uc9d1\ub2e8\ub4e4\uc744 \ub098\ub204\uace0 \uacf5\uc801\uc778 \ubaa9\ud45c\ub97c \ub2a5\ub960\uc801\uc73c\ub85c \ub2ec\uc131\ud558\uae30 \uc704\ud574 \uc0ac\ub78c\ub4e4\uc774 \uc0c1\ud638\uad00\uacc4\ub97c \uc774\uc131\uc801, \ud569\ub9ac\uc801, \uc778\uc704\uc801\uc73c\ub85c \uaddc\uc815\ud558\uace0 \uc81c\ub3c4\ud654\ud55c \uc778\uc704\uc801\uc778 \uc870\uc9c1\uc774\ub2e4. \uad8c\ud55c\uc758 \uacc4\ud1b5\uacfc \uae30\ub2a5\uc801 \ubd84\uc5c5\uc5d0 \ub530\ub77c \uc870\uc9c1\uc758 \ubaa9\ud45c\ub97c \ud6a8\uacfc\uc801\uc73c\ub85c \ub2ec\uc131\ud558\uae30 \uc704\ud574 \uc81c\ub3c4\ud654\ub41c \uc870\uc9c1 \uccb4\uacc4\uc640 \uacf5\uc2dd \uaddc\ubc94\uc744 \uac00\uc9c0\uace0 \uc788\uc73c\uba70, \uacf5\uc2dd\ud654\ub41c \ubd84\uc5c5\uc758 \uacc4\ud1b5\uacfc \uc758\uc0ac\uc18c\ud1b5\uc758 \uacbd\ub85c, \uad8c\ud55c\uc758 \ubc30\ubd84\uc5d0 \ub530\ub77c \uac01\uc790 \uba85\ud655\ud55c \uc5ed\ud560\uc774 \uc8fc\uc5b4\uc838 \uc788\ub2e4. \uacf5\uc2dd \uc870\uc9c1\uc740 \ud45c\uc900\ud654\ub41c \uc5c5\ubb34 \uc218\ud589, \uba85\ud655\ud55c \ucc45\uc784 \ubd84\ub2f4 \ub4f1\uc744 \ud2b9\uc9d5\uc73c\ub85c \ud558\uba70 \ub2a5\ub960\uc774\ub098 \ube44\uc6a9\uc758 \ub17c\ub9ac\uc5d0 \uc758\ud574 \uad6c\uc131\ub418\uace0 \uc6b4\uc601\ub41c\ub2e4. \ud558\uc9c0\ub9cc \uacf5\uc2dd \uc870\uc9c1\uc758 \uad6c\uc870\uc801\uc778 \uad00\uacc4\uac00 \uac15\uc555\uc801\uc774\uc9c0 \uc54a\uace0 \ud1b5\uc81c \ubc1b\uc9c0 \uc54a\ub294 \uad00\uacc4\uc5d0 \uc758\ud574 \ub300\uccb4\ub420 \ub54c \ube44\uacf5\uc2dd \uc870\uc9c1\uc73c\ub85c \ubcc0\ud560 \uc218 \uc788\ub2e4. \uacf5\uc2dd \uc870\uc9c1\uc740 \ub300\uccb4\ub85c \ub2e4\uc74c\uacfc \uac19\uc740 \ud2b9\uc9d5\uc744 \uac00\uc9c4\ub2e4.", "answer": "\uc81c\ub3c4\ud654\ub41c \uc870\uc9c1 \uccb4\uacc4\uc640 \uacf5\uc2dd \uaddc\ubc94", "sentence": "\uad8c\ud55c\uc758 \uacc4\ud1b5\uacfc \uae30\ub2a5\uc801 \ubd84\uc5c5\uc5d0 \ub530\ub77c \uc870\uc9c1\uc758 \ubaa9\ud45c\ub97c \ud6a8\uacfc\uc801\uc73c\ub85c \ub2ec\uc131\ud558\uae30 \uc704\ud574 \uc81c\ub3c4\ud654\ub41c \uc870\uc9c1 \uccb4\uacc4\uc640 \uacf5\uc2dd \uaddc\ubc94 \uc744 \uac00\uc9c0\uace0 \uc788\uc73c\uba70, \uacf5\uc2dd\ud654\ub41c \ubd84\uc5c5\uc758 \uacc4\ud1b5\uacfc \uc758\uc0ac\uc18c\ud1b5\uc758 \uacbd\ub85c, \uad8c\ud55c\uc758 \ubc30\ubd84\uc5d0 \ub530\ub77c \uac01\uc790 \uba85\ud655\ud55c \uc5ed\ud560\uc774 \uc8fc\uc5b4\uc838 \uc788\ub2e4.", "paragraph_sentence": "\uacf5\uc2dd \uc870\uc9c1\uc740 \uc9c1\ubb34, \ucc45\uc784, \uad8c\ud55c\uc744 \uc911\uc2ec\uc73c\ub85c \uc870\uc9c1 \uad6c\uc131\uc6d0 \ud639\uc740 \uc791\uc5c5 \uc9d1\ub2e8\ub4e4\uc744 \ub098\ub204\uace0 \uacf5\uc801\uc778 \ubaa9\ud45c\ub97c \ub2a5\ub960\uc801\uc73c\ub85c \ub2ec\uc131\ud558\uae30 \uc704\ud574 \uc0ac\ub78c\ub4e4\uc774 \uc0c1\ud638\uad00\uacc4\ub97c \uc774\uc131\uc801, \ud569\ub9ac\uc801, \uc778\uc704\uc801\uc73c\ub85c \uaddc\uc815\ud558\uace0 \uc81c\ub3c4\ud654\ud55c \uc778\uc704\uc801\uc778 \uc870\uc9c1\uc774\ub2e4. \uad8c\ud55c\uc758 \uacc4\ud1b5\uacfc \uae30\ub2a5\uc801 \ubd84\uc5c5\uc5d0 \ub530\ub77c \uc870\uc9c1\uc758 \ubaa9\ud45c\ub97c \ud6a8\uacfc\uc801\uc73c\ub85c \ub2ec\uc131\ud558\uae30 \uc704\ud574 \uc81c\ub3c4\ud654\ub41c \uc870\uc9c1 \uccb4\uacc4\uc640 \uacf5\uc2dd \uaddc\ubc94 \uc744 \uac00\uc9c0\uace0 \uc788\uc73c\uba70, \uacf5\uc2dd\ud654\ub41c \ubd84\uc5c5\uc758 \uacc4\ud1b5\uacfc \uc758\uc0ac\uc18c\ud1b5\uc758 \uacbd\ub85c, \uad8c\ud55c\uc758 \ubc30\ubd84\uc5d0 \ub530\ub77c \uac01\uc790 \uba85\ud655\ud55c \uc5ed\ud560\uc774 \uc8fc\uc5b4\uc838 \uc788\ub2e4. \uacf5\uc2dd \uc870\uc9c1\uc740 \ud45c\uc900\ud654\ub41c \uc5c5\ubb34 \uc218\ud589, \uba85\ud655\ud55c \ucc45\uc784 \ubd84\ub2f4 \ub4f1\uc744 \ud2b9\uc9d5\uc73c\ub85c \ud558\uba70 \ub2a5\ub960\uc774\ub098 \ube44\uc6a9\uc758 \ub17c\ub9ac\uc5d0 \uc758\ud574 \uad6c\uc131\ub418\uace0 \uc6b4\uc601\ub41c\ub2e4. \ud558\uc9c0\ub9cc \uacf5\uc2dd \uc870\uc9c1\uc758 \uad6c\uc870\uc801\uc778 \uad00\uacc4\uac00 \uac15\uc555\uc801\uc774\uc9c0 \uc54a\uace0 \ud1b5\uc81c \ubc1b\uc9c0 \uc54a\ub294 \uad00\uacc4\uc5d0 \uc758\ud574 \ub300\uccb4\ub420 \ub54c \ube44\uacf5\uc2dd \uc870\uc9c1\uc73c\ub85c \ubcc0\ud560 \uc218 \uc788\ub2e4. \uacf5\uc2dd \uc870\uc9c1\uc740 \ub300\uccb4\ub85c \ub2e4\uc74c\uacfc \uac19\uc740 \ud2b9\uc9d5\uc744 \uac00\uc9c4\ub2e4.", "paragraph_answer": "\uacf5\uc2dd \uc870\uc9c1\uc740 \uc9c1\ubb34, \ucc45\uc784, \uad8c\ud55c\uc744 \uc911\uc2ec\uc73c\ub85c \uc870\uc9c1 \uad6c\uc131\uc6d0 \ud639\uc740 \uc791\uc5c5 \uc9d1\ub2e8\ub4e4\uc744 \ub098\ub204\uace0 \uacf5\uc801\uc778 \ubaa9\ud45c\ub97c \ub2a5\ub960\uc801\uc73c\ub85c \ub2ec\uc131\ud558\uae30 \uc704\ud574 \uc0ac\ub78c\ub4e4\uc774 \uc0c1\ud638\uad00\uacc4\ub97c \uc774\uc131\uc801, \ud569\ub9ac\uc801, \uc778\uc704\uc801\uc73c\ub85c \uaddc\uc815\ud558\uace0 \uc81c\ub3c4\ud654\ud55c \uc778\uc704\uc801\uc778 \uc870\uc9c1\uc774\ub2e4. \uad8c\ud55c\uc758 \uacc4\ud1b5\uacfc \uae30\ub2a5\uc801 \ubd84\uc5c5\uc5d0 \ub530\ub77c \uc870\uc9c1\uc758 \ubaa9\ud45c\ub97c \ud6a8\uacfc\uc801\uc73c\ub85c \ub2ec\uc131\ud558\uae30 \uc704\ud574 \uc81c\ub3c4\ud654\ub41c \uc870\uc9c1 \uccb4\uacc4\uc640 \uacf5\uc2dd \uaddc\ubc94 \uc744 \uac00\uc9c0\uace0 \uc788\uc73c\uba70, \uacf5\uc2dd\ud654\ub41c \ubd84\uc5c5\uc758 \uacc4\ud1b5\uacfc \uc758\uc0ac\uc18c\ud1b5\uc758 \uacbd\ub85c, \uad8c\ud55c\uc758 \ubc30\ubd84\uc5d0 \ub530\ub77c \uac01\uc790 \uba85\ud655\ud55c \uc5ed\ud560\uc774 \uc8fc\uc5b4\uc838 \uc788\ub2e4. \uacf5\uc2dd \uc870\uc9c1\uc740 \ud45c\uc900\ud654\ub41c \uc5c5\ubb34 \uc218\ud589, \uba85\ud655\ud55c \ucc45\uc784 \ubd84\ub2f4 \ub4f1\uc744 \ud2b9\uc9d5\uc73c\ub85c \ud558\uba70 \ub2a5\ub960\uc774\ub098 \ube44\uc6a9\uc758 \ub17c\ub9ac\uc5d0 \uc758\ud574 \uad6c\uc131\ub418\uace0 \uc6b4\uc601\ub41c\ub2e4. \ud558\uc9c0\ub9cc \uacf5\uc2dd \uc870\uc9c1\uc758 \uad6c\uc870\uc801\uc778 \uad00\uacc4\uac00 \uac15\uc555\uc801\uc774\uc9c0 \uc54a\uace0 \ud1b5\uc81c \ubc1b\uc9c0 \uc54a\ub294 \uad00\uacc4\uc5d0 \uc758\ud574 \ub300\uccb4\ub420 \ub54c \ube44\uacf5\uc2dd \uc870\uc9c1\uc73c\ub85c \ubcc0\ud560 \uc218 \uc788\ub2e4. \uacf5\uc2dd \uc870\uc9c1\uc740 \ub300\uccb4\ub85c \ub2e4\uc74c\uacfc \uac19\uc740 \ud2b9\uc9d5\uc744 \uac00\uc9c4\ub2e4.", "sentence_answer": "\uad8c\ud55c\uc758 \uacc4\ud1b5\uacfc \uae30\ub2a5\uc801 \ubd84\uc5c5\uc5d0 \ub530\ub77c \uc870\uc9c1\uc758 \ubaa9\ud45c\ub97c \ud6a8\uacfc\uc801\uc73c\ub85c \ub2ec\uc131\ud558\uae30 \uc704\ud574 \uc81c\ub3c4\ud654\ub41c \uc870\uc9c1 \uccb4\uacc4\uc640 \uacf5\uc2dd \uaddc\ubc94 \uc744 \uac00\uc9c0\uace0 \uc788\uc73c\uba70, \uacf5\uc2dd\ud654\ub41c \ubd84\uc5c5\uc758 \uacc4\ud1b5\uacfc \uc758\uc0ac\uc18c\ud1b5\uc758 \uacbd\ub85c, \uad8c\ud55c\uc758 \ubc30\ubd84\uc5d0 \ub530\ub77c \uac01\uc790 \uba85\ud655\ud55c \uc5ed\ud560\uc774 \uc8fc\uc5b4\uc838 \uc788\ub2e4.", "paragraph_id": "5886327-3-0", "paragraph_question": "question: \uacf5\uc2dd \uc870\uc9c1\uc758 \ubaa9\ud45c\ub97c \ud6a8\uacfc\uc801\uc73c\ub85c \ub2ec\uc131\ud558\uae30 \uc704\ud574 \uac00\uc9c0\uace0 \uc788\ub294 \uac83\uc740?, context: \uacf5\uc2dd \uc870\uc9c1\uc740 \uc9c1\ubb34, \ucc45\uc784, \uad8c\ud55c\uc744 \uc911\uc2ec\uc73c\ub85c \uc870\uc9c1 \uad6c\uc131\uc6d0 \ud639\uc740 \uc791\uc5c5 \uc9d1\ub2e8\ub4e4\uc744 \ub098\ub204\uace0 \uacf5\uc801\uc778 \ubaa9\ud45c\ub97c \ub2a5\ub960\uc801\uc73c\ub85c \ub2ec\uc131\ud558\uae30 \uc704\ud574 \uc0ac\ub78c\ub4e4\uc774 \uc0c1\ud638\uad00\uacc4\ub97c \uc774\uc131\uc801, \ud569\ub9ac\uc801, \uc778\uc704\uc801\uc73c\ub85c \uaddc\uc815\ud558\uace0 \uc81c\ub3c4\ud654\ud55c \uc778\uc704\uc801\uc778 \uc870\uc9c1\uc774\ub2e4. \uad8c\ud55c\uc758 \uacc4\ud1b5\uacfc \uae30\ub2a5\uc801 \ubd84\uc5c5\uc5d0 \ub530\ub77c \uc870\uc9c1\uc758 \ubaa9\ud45c\ub97c \ud6a8\uacfc\uc801\uc73c\ub85c \ub2ec\uc131\ud558\uae30 \uc704\ud574 \uc81c\ub3c4\ud654\ub41c \uc870\uc9c1 \uccb4\uacc4\uc640 \uacf5\uc2dd \uaddc\ubc94\uc744 \uac00\uc9c0\uace0 \uc788\uc73c\uba70, \uacf5\uc2dd\ud654\ub41c \ubd84\uc5c5\uc758 \uacc4\ud1b5\uacfc \uc758\uc0ac\uc18c\ud1b5\uc758 \uacbd\ub85c, \uad8c\ud55c\uc758 \ubc30\ubd84\uc5d0 \ub530\ub77c \uac01\uc790 \uba85\ud655\ud55c \uc5ed\ud560\uc774 \uc8fc\uc5b4\uc838 \uc788\ub2e4. \uacf5\uc2dd \uc870\uc9c1\uc740 \ud45c\uc900\ud654\ub41c \uc5c5\ubb34 \uc218\ud589, \uba85\ud655\ud55c \ucc45\uc784 \ubd84\ub2f4 \ub4f1\uc744 \ud2b9\uc9d5\uc73c\ub85c \ud558\uba70 \ub2a5\ub960\uc774\ub098 \ube44\uc6a9\uc758 \ub17c\ub9ac\uc5d0 \uc758\ud574 \uad6c\uc131\ub418\uace0 \uc6b4\uc601\ub41c\ub2e4. \ud558\uc9c0\ub9cc \uacf5\uc2dd \uc870\uc9c1\uc758 \uad6c\uc870\uc801\uc778 \uad00\uacc4\uac00 \uac15\uc555\uc801\uc774\uc9c0 \uc54a\uace0 \ud1b5\uc81c \ubc1b\uc9c0 \uc54a\ub294 \uad00\uacc4\uc5d0 \uc758\ud574 \ub300\uccb4\ub420 \ub54c \ube44\uacf5\uc2dd \uc870\uc9c1\uc73c\ub85c \ubcc0\ud560 \uc218 \uc788\ub2e4. \uacf5\uc2dd \uc870\uc9c1\uc740 \ub300\uccb4\ub85c \ub2e4\uc74c\uacfc \uac19\uc740 \ud2b9\uc9d5\uc744 \uac00\uc9c4\ub2e4."} {"question": "\ud55c\uad6d \ube14\ub799 \ubba4\uc9c1 \uc0ac\uc774\ud2b8\uc778 \ud799\ud569\ud50c\ub808\uc774\uc57c\uc5d0\uc11c 2007\ub144 \uc62c\ud574\uc758 \uc568\ubc94\uc73c\ub85c \uc120\uc815\ub41c \uc568\ubc94\uc740 \ubb34\uc5c7\uc778\uac00?", "paragraph": "\uc774 \uc74c\ubc18\uc73c\ub85c \uc5d0\ud53d\ud558\uc774\ub294 \uc804\ubb38\uac00\ub4e4\uc774 \ubf51\uc740 2007\ub144 \uc0c1\ubc18\uae30 \ucd5c\uace0\uc758 \uac00\uc218 2\uc704\ub85c \uc120\uc815\ub418\uc5c8\ub2e4. \ub300\ud55c\ud56d\uacf5\uc774 \uc804\ubb38\uac00\ub4e4\uc758 \uc870\uc5b8\uc744 \ubc14\ud0d5\uc73c\ub85c \uc120\uc815\ud558\ub294 \"\uc62c \ud788\ud2b8 \ube0c\ub79c\ub4dc\"\uc5d0\uc11c \uc74c\ubc18 \ubd80\ubb38 \ud788\ud2b8 \ube0c\ub79c\ub4dc\ub85c \uc120\uc815\ub418\uc5c8\ub2e4. 2007\ub144 MKMF \uc2dc\uc0c1\uc2dd\uc5d0\uc11c \uc62c\ud574\uc758 \uc74c\ubc18\uc0c1(\u300aRemapping The Human Soul\u300b), \ud799\ud569 \uc74c\uc545\uc0c1(\u3008Fan\u3009)\uc744 \uc218\uc0c1\ud558\uc600\uace0 \uace8\ub4e0\ub514\uc2a4\ud06c\uc640 \uc11c\uc6b8\uac00\uc694\ub300\uc0c1\uc5d0\uc120 \ubcf8\uc0c1\uc744, \uc81c4\ud68c \ud55c\uad6d\ub300\uc911\uc74c\uc545\uc0c1\uc5d0\uc11c\ub294 \uc774\uc801\uacfc \ud568\uaed8 \ucd5c\ub2e4 \ubd80\ubb38\uc778 6\uac1c \ubd80\ubb38\uc5d0 \ud6c4\ubcf4(\uc62c\ud574\uc758 \uc74c\uc545\uc778, \uc62c\ud574\uc758 \uc74c\ubc18, \uc62c\ud574\uc758 \ub178\ub798, \ucd5c\uc6b0\uc218 \ud799\ud569 \ub178\ub798, \ucd5c\uc6b0\uc218 \ud799\ud569 \uc74c\ubc18, \ub124\ud2f0\uc98c\uc774 \ubf51\uc740 \uc62c\ud574\uc758 \uc74c\uc545\uc778)\ub85c \uc62c\ub77c \ucd5c\uc6b0\uc218 \ud799\ud569 \uc74c\ubc18\uc0c1\uc744 \uc218\uc0c1\ud558\uc600\ub2e4. \ud55c\uad6d \ube14\ub799 \ubba4\uc9c1 \uc0ac\uc774\ud2b8\uc778 \ud799\ud569\ud50c\ub808\uc774\uc57c\uc5d0\uc11c\ub294 \uc774 \uc74c\ubc18\uc744 2007\ub144 \uc62c\ud574\uc758 \uc568\ubc94\uc73c\ub85c \uc120\uc815\ud568\uacfc \ub3d9\uc2dc\uc5d0 \uc5d0\ud53d\ud558\uc774\ub97c \uc62c\ud574\uc758 \uc544\ud2f0\uc2a4\ud2b8\ub85c \uc120\uc815\ud558\uc600\ub2e4.", "answer": "Remapping The Human Soul", "sentence": "2007\ub144 MKMF \uc2dc\uc0c1\uc2dd\uc5d0\uc11c \uc62c\ud574\uc758 \uc74c\ubc18\uc0c1(\u300a Remapping The Human Soul \u300b)", "paragraph_sentence": "\uc774 \uc74c\ubc18\uc73c\ub85c \uc5d0\ud53d\ud558\uc774\ub294 \uc804\ubb38\uac00\ub4e4\uc774 \ubf51\uc740 2007\ub144 \uc0c1\ubc18\uae30 \ucd5c\uace0\uc758 \uac00\uc218 2\uc704\ub85c \uc120\uc815\ub418\uc5c8\ub2e4. \ub300\ud55c\ud56d\uacf5\uc774 \uc804\ubb38\uac00\ub4e4\uc758 \uc870\uc5b8\uc744 \ubc14\ud0d5\uc73c\ub85c \uc120\uc815\ud558\ub294 \"\uc62c \ud788\ud2b8 \ube0c\ub79c\ub4dc\"\uc5d0\uc11c \uc74c\ubc18 \ubd80\ubb38 \ud788\ud2b8 \ube0c\ub79c\ub4dc\ub85c \uc120\uc815\ub418\uc5c8\ub2e4. 2007\ub144 MKMF \uc2dc\uc0c1\uc2dd\uc5d0\uc11c \uc62c\ud574\uc758 \uc74c\ubc18\uc0c1(\u300a Remapping The Human Soul \u300b) , \ud799\ud569 \uc74c\uc545\uc0c1(\u3008Fan\u3009)\uc744 \uc218\uc0c1\ud558\uc600\uace0 \uace8\ub4e0\ub514\uc2a4\ud06c\uc640 \uc11c\uc6b8\uac00\uc694\ub300\uc0c1\uc5d0\uc120 \ubcf8\uc0c1\uc744, \uc81c4\ud68c \ud55c\uad6d\ub300\uc911\uc74c\uc545\uc0c1\uc5d0\uc11c\ub294 \uc774\uc801\uacfc \ud568\uaed8 \ucd5c\ub2e4 \ubd80\ubb38\uc778 6\uac1c \ubd80\ubb38\uc5d0 \ud6c4\ubcf4(\uc62c\ud574\uc758 \uc74c\uc545\uc778, \uc62c\ud574\uc758 \uc74c\ubc18, \uc62c\ud574\uc758 \ub178\ub798, \ucd5c\uc6b0\uc218 \ud799\ud569 \ub178\ub798, \ucd5c\uc6b0\uc218 \ud799\ud569 \uc74c\ubc18, \ub124\ud2f0\uc98c\uc774 \ubf51\uc740 \uc62c\ud574\uc758 \uc74c\uc545\uc778)\ub85c \uc62c\ub77c \ucd5c\uc6b0\uc218 \ud799\ud569 \uc74c\ubc18\uc0c1\uc744 \uc218\uc0c1\ud558\uc600\ub2e4. \ud55c\uad6d \ube14\ub799 \ubba4\uc9c1 \uc0ac\uc774\ud2b8\uc778 \ud799\ud569\ud50c\ub808\uc774\uc57c\uc5d0\uc11c\ub294 \uc774 \uc74c\ubc18\uc744 2007\ub144 \uc62c\ud574\uc758 \uc568\ubc94\uc73c\ub85c \uc120\uc815\ud568\uacfc \ub3d9\uc2dc\uc5d0 \uc5d0\ud53d\ud558\uc774\ub97c \uc62c\ud574\uc758 \uc544\ud2f0\uc2a4\ud2b8\ub85c \uc120\uc815\ud558\uc600\ub2e4.", "paragraph_answer": "\uc774 \uc74c\ubc18\uc73c\ub85c \uc5d0\ud53d\ud558\uc774\ub294 \uc804\ubb38\uac00\ub4e4\uc774 \ubf51\uc740 2007\ub144 \uc0c1\ubc18\uae30 \ucd5c\uace0\uc758 \uac00\uc218 2\uc704\ub85c \uc120\uc815\ub418\uc5c8\ub2e4. \ub300\ud55c\ud56d\uacf5\uc774 \uc804\ubb38\uac00\ub4e4\uc758 \uc870\uc5b8\uc744 \ubc14\ud0d5\uc73c\ub85c \uc120\uc815\ud558\ub294 \"\uc62c \ud788\ud2b8 \ube0c\ub79c\ub4dc\"\uc5d0\uc11c \uc74c\ubc18 \ubd80\ubb38 \ud788\ud2b8 \ube0c\ub79c\ub4dc\ub85c \uc120\uc815\ub418\uc5c8\ub2e4. 2007\ub144 MKMF \uc2dc\uc0c1\uc2dd\uc5d0\uc11c \uc62c\ud574\uc758 \uc74c\ubc18\uc0c1(\u300a Remapping The Human Soul \u300b), \ud799\ud569 \uc74c\uc545\uc0c1(\u3008Fan\u3009)\uc744 \uc218\uc0c1\ud558\uc600\uace0 \uace8\ub4e0\ub514\uc2a4\ud06c\uc640 \uc11c\uc6b8\uac00\uc694\ub300\uc0c1\uc5d0\uc120 \ubcf8\uc0c1\uc744, \uc81c4\ud68c \ud55c\uad6d\ub300\uc911\uc74c\uc545\uc0c1\uc5d0\uc11c\ub294 \uc774\uc801\uacfc \ud568\uaed8 \ucd5c\ub2e4 \ubd80\ubb38\uc778 6\uac1c \ubd80\ubb38\uc5d0 \ud6c4\ubcf4(\uc62c\ud574\uc758 \uc74c\uc545\uc778, \uc62c\ud574\uc758 \uc74c\ubc18, \uc62c\ud574\uc758 \ub178\ub798, \ucd5c\uc6b0\uc218 \ud799\ud569 \ub178\ub798, \ucd5c\uc6b0\uc218 \ud799\ud569 \uc74c\ubc18, \ub124\ud2f0\uc98c\uc774 \ubf51\uc740 \uc62c\ud574\uc758 \uc74c\uc545\uc778)\ub85c \uc62c\ub77c \ucd5c\uc6b0\uc218 \ud799\ud569 \uc74c\ubc18\uc0c1\uc744 \uc218\uc0c1\ud558\uc600\ub2e4. \ud55c\uad6d \ube14\ub799 \ubba4\uc9c1 \uc0ac\uc774\ud2b8\uc778 \ud799\ud569\ud50c\ub808\uc774\uc57c\uc5d0\uc11c\ub294 \uc774 \uc74c\ubc18\uc744 2007\ub144 \uc62c\ud574\uc758 \uc568\ubc94\uc73c\ub85c \uc120\uc815\ud568\uacfc \ub3d9\uc2dc\uc5d0 \uc5d0\ud53d\ud558\uc774\ub97c \uc62c\ud574\uc758 \uc544\ud2f0\uc2a4\ud2b8\ub85c \uc120\uc815\ud558\uc600\ub2e4.", "sentence_answer": "2007\ub144 MKMF \uc2dc\uc0c1\uc2dd\uc5d0\uc11c \uc62c\ud574\uc758 \uc74c\ubc18\uc0c1(\u300a Remapping The Human Soul \u300b)", "paragraph_id": "6470793-5-1", "paragraph_question": "question: \ud55c\uad6d \ube14\ub799 \ubba4\uc9c1 \uc0ac\uc774\ud2b8\uc778 \ud799\ud569\ud50c\ub808\uc774\uc57c\uc5d0\uc11c 2007\ub144 \uc62c\ud574\uc758 \uc568\ubc94\uc73c\ub85c \uc120\uc815\ub41c \uc568\ubc94\uc740 \ubb34\uc5c7\uc778\uac00?, context: \uc774 \uc74c\ubc18\uc73c\ub85c \uc5d0\ud53d\ud558\uc774\ub294 \uc804\ubb38\uac00\ub4e4\uc774 \ubf51\uc740 2007\ub144 \uc0c1\ubc18\uae30 \ucd5c\uace0\uc758 \uac00\uc218 2\uc704\ub85c \uc120\uc815\ub418\uc5c8\ub2e4. \ub300\ud55c\ud56d\uacf5\uc774 \uc804\ubb38\uac00\ub4e4\uc758 \uc870\uc5b8\uc744 \ubc14\ud0d5\uc73c\ub85c \uc120\uc815\ud558\ub294 \"\uc62c \ud788\ud2b8 \ube0c\ub79c\ub4dc\"\uc5d0\uc11c \uc74c\ubc18 \ubd80\ubb38 \ud788\ud2b8 \ube0c\ub79c\ub4dc\ub85c \uc120\uc815\ub418\uc5c8\ub2e4. 2007\ub144 MKMF \uc2dc\uc0c1\uc2dd\uc5d0\uc11c \uc62c\ud574\uc758 \uc74c\ubc18\uc0c1(\u300aRemapping The Human Soul\u300b), \ud799\ud569 \uc74c\uc545\uc0c1(\u3008Fan\u3009)\uc744 \uc218\uc0c1\ud558\uc600\uace0 \uace8\ub4e0\ub514\uc2a4\ud06c\uc640 \uc11c\uc6b8\uac00\uc694\ub300\uc0c1\uc5d0\uc120 \ubcf8\uc0c1\uc744, \uc81c4\ud68c \ud55c\uad6d\ub300\uc911\uc74c\uc545\uc0c1\uc5d0\uc11c\ub294 \uc774\uc801\uacfc \ud568\uaed8 \ucd5c\ub2e4 \ubd80\ubb38\uc778 6\uac1c \ubd80\ubb38\uc5d0 \ud6c4\ubcf4(\uc62c\ud574\uc758 \uc74c\uc545\uc778, \uc62c\ud574\uc758 \uc74c\ubc18, \uc62c\ud574\uc758 \ub178\ub798, \ucd5c\uc6b0\uc218 \ud799\ud569 \ub178\ub798, \ucd5c\uc6b0\uc218 \ud799\ud569 \uc74c\ubc18, \ub124\ud2f0\uc98c\uc774 \ubf51\uc740 \uc62c\ud574\uc758 \uc74c\uc545\uc778)\ub85c \uc62c\ub77c \ucd5c\uc6b0\uc218 \ud799\ud569 \uc74c\ubc18\uc0c1\uc744 \uc218\uc0c1\ud558\uc600\ub2e4. \ud55c\uad6d \ube14\ub799 \ubba4\uc9c1 \uc0ac\uc774\ud2b8\uc778 \ud799\ud569\ud50c\ub808\uc774\uc57c\uc5d0\uc11c\ub294 \uc774 \uc74c\ubc18\uc744 2007\ub144 \uc62c\ud574\uc758 \uc568\ubc94\uc73c\ub85c \uc120\uc815\ud568\uacfc \ub3d9\uc2dc\uc5d0 \uc5d0\ud53d\ud558\uc774\ub97c \uc62c\ud574\uc758 \uc544\ud2f0\uc2a4\ud2b8\ub85c \uc120\uc815\ud558\uc600\ub2e4."} {"question": "\ubc95\uc758 \uc774\ub984\uc73c\ub85c\ub3c4 \ud45c\ud604\ub418\uba70, \ud5cc\ubc95, \ubc95\ub960,\ubc95\uc774\uc0c1\uc758 \ucd08\ubc95\uc801\uc778 \uc790\uc5f0\ubc95\uae4c\uc9c0 \ud3ec\ud568\ud558\uac8c \ub418\ub294\uac83\uc740?", "paragraph": "\ubc95 \uc55e\uc758 \ud3c9\ub4f1(\uc601\uc5b4: Equality before the law)\uc758 \uad00\ub150\uc740 \ud50c\ub77c\ud1a4\uc758'\uad6d\uac00\ub860'\uc5d0\uc11c\uc758 \uc815\uc758\uc774\ub150\uacfc'\uc2e0\uc55e\uc758 \ud3c9\ub4f1'\uc774\ub77c\ub294 \uc885\uad50\uc0ac\uc0c1\uc5d0\uc11c \uc5f0\uc6d0\ud55c\ub2e4. \ub974\ub124\uc0c1\uc2a4\uae30 \uc2e0\ubcf8\uc8fc\uc758\uac00 \ub9c9\uc744 \ub0b4\ub9ac\uace0 \uc778\ubcf8\uc8fc\uc758 \uc2dc\ub300\uac00 \ub3c4\ub798\ud558\uc790 \uc778\uac04 \uc774\uc131\uacfc \uacfc\ud559\uc758 \ud798\uc740 \uc2e0\uc744 \ub300\uccb4\ud558\uc600\ub2e4. \uc774\uc131\uc740 \ubc95(Law)\uc758 \uc774\ub984\uc73c\ub85c\ub3c4 \ud45c\ud604\ub418\uc5b4 \uc774\uc81c \ubc95\uc740 \ub2e8\uc21c\ud788 \ud5cc\ubc95, \ubc95\ub960\ubfd0\ub9cc\uc774 \uc544\ub2c8\ub77c \ubc95\uc774\uc0c1\uc758 \ucd08\ubc95\uc801\uc778 \uc790\uc5f0\ubc95\uae4c\uc9c0 \ud3ec\ud568\ud558\uac8c \ub41c\ub2e4. \uc774\uc5d0 \uc218\ubc18\ub41c \ud3c9\ub4f1\uad8c\ub3c4, \ub2e8\uc9c0 \ud615\ubc8c\uc0c1\uc758 \ud3c9\ub4f1\uc774\ub098 \ubc95\ub960\uc0c1\uc758 \ud3c9\ub4f1\uc774 \uc544\ub2c8\ub77c, \ucd08\ubc95\ub960\uc801 \uc790\uc5f0\uad8c\uc73c\ub85c \ud655\ub300\ub3fc \ud5cc\ubc9510\uc870\uc758 \ubcf4\ud3b8\uc801 \ubd88\uac00\uce68\uc801 \uc874\uc5c4\uad8c, 11\uc870\uc758 \ubc95\uc55e\uc758 \ud3c9\ub4f1\uad8c, 34\uc870\uc758 \uc778\uac04\ub2f5\uac8c \uc0b4 \uad8c\ub9ac\ub85c \ud655\uace0\ud788 \uac01\uc778\ub418\uc5b4 \uc778\uac04\uc740 \ub204\uad6c\ub098 \uc815\uce58, \uacbd\uc81c, \uc0ac\ud68c,\ubb38\ud654\uc801\uc73c\ub85c \uc5b4\ub5a0\ud55c \ucc28\ubcc4\ub3c4 \ubc1b\uc9c0 \uc54a\uc744 \uc790\uc5f0\uad8c\uc801 \ud3c9\ub4f1\uad8c\uc744 \ubcf4\uc7a5\ubc1b\uace0 \uad6d\uac00\ub294 \uc774\ub97c \ubcf4\uc7a5\ud560 \uc758\ubb34\uac00 \uc788\uc74c\uc744 \uaddc\uc815\ud588\ub2e4.", "answer": "\uc774\uc131", "sentence": "\uc778\uac04 \uc774\uc131 \uacfc \uacfc\ud559\uc758 \ud798\uc740 \uc2e0\uc744 \ub300\uccb4\ud558\uc600\ub2e4.", "paragraph_sentence": "\ubc95 \uc55e\uc758 \ud3c9\ub4f1(\uc601\uc5b4: Equality before the law)\uc758 \uad00\ub150\uc740 \ud50c\ub77c\ud1a4\uc758'\uad6d\uac00\ub860'\uc5d0\uc11c\uc758 \uc815\uc758\uc774\ub150\uacfc'\uc2e0\uc55e\uc758 \ud3c9\ub4f1'\uc774\ub77c\ub294 \uc885\uad50\uc0ac\uc0c1\uc5d0\uc11c \uc5f0\uc6d0\ud55c\ub2e4. \ub974\ub124\uc0c1\uc2a4\uae30 \uc2e0\ubcf8\uc8fc\uc758\uac00 \ub9c9\uc744 \ub0b4\ub9ac\uace0 \uc778\ubcf8\uc8fc\uc758 \uc2dc\ub300\uac00 \ub3c4\ub798\ud558\uc790 \uc778\uac04 \uc774\uc131 \uacfc \uacfc\ud559\uc758 \ud798\uc740 \uc2e0\uc744 \ub300\uccb4\ud558\uc600\ub2e4. \uc774\uc131\uc740 \ubc95(Law)\uc758 \uc774\ub984\uc73c\ub85c\ub3c4 \ud45c\ud604\ub418\uc5b4 \uc774\uc81c \ubc95\uc740 \ub2e8\uc21c\ud788 \ud5cc\ubc95, \ubc95\ub960\ubfd0\ub9cc\uc774 \uc544\ub2c8\ub77c \ubc95\uc774\uc0c1\uc758 \ucd08\ubc95\uc801\uc778 \uc790\uc5f0\ubc95\uae4c\uc9c0 \ud3ec\ud568\ud558\uac8c \ub41c\ub2e4. \uc774\uc5d0 \uc218\ubc18\ub41c \ud3c9\ub4f1\uad8c\ub3c4, \ub2e8\uc9c0 \ud615\ubc8c\uc0c1\uc758 \ud3c9\ub4f1\uc774\ub098 \ubc95\ub960\uc0c1\uc758 \ud3c9\ub4f1\uc774 \uc544\ub2c8\ub77c, \ucd08\ubc95\ub960\uc801 \uc790\uc5f0\uad8c\uc73c\ub85c \ud655\ub300\ub3fc \ud5cc\ubc9510\uc870\uc758 \ubcf4\ud3b8\uc801 \ubd88\uac00\uce68\uc801 \uc874\uc5c4\uad8c, 11\uc870\uc758 \ubc95\uc55e\uc758 \ud3c9\ub4f1\uad8c, 34\uc870\uc758 \uc778\uac04\ub2f5\uac8c \uc0b4 \uad8c\ub9ac\ub85c \ud655\uace0\ud788 \uac01\uc778\ub418\uc5b4 \uc778\uac04\uc740 \ub204\uad6c\ub098 \uc815\uce58, \uacbd\uc81c, \uc0ac\ud68c,\ubb38\ud654\uc801\uc73c\ub85c \uc5b4\ub5a0\ud55c \ucc28\ubcc4\ub3c4 \ubc1b\uc9c0 \uc54a\uc744 \uc790\uc5f0\uad8c\uc801 \ud3c9\ub4f1\uad8c\uc744 \ubcf4\uc7a5\ubc1b\uace0 \uad6d\uac00\ub294 \uc774\ub97c \ubcf4\uc7a5\ud560 \uc758\ubb34\uac00 \uc788\uc74c\uc744 \uaddc\uc815\ud588\ub2e4.", "paragraph_answer": "\ubc95 \uc55e\uc758 \ud3c9\ub4f1(\uc601\uc5b4: Equality before the law)\uc758 \uad00\ub150\uc740 \ud50c\ub77c\ud1a4\uc758'\uad6d\uac00\ub860'\uc5d0\uc11c\uc758 \uc815\uc758\uc774\ub150\uacfc'\uc2e0\uc55e\uc758 \ud3c9\ub4f1'\uc774\ub77c\ub294 \uc885\uad50\uc0ac\uc0c1\uc5d0\uc11c \uc5f0\uc6d0\ud55c\ub2e4. \ub974\ub124\uc0c1\uc2a4\uae30 \uc2e0\ubcf8\uc8fc\uc758\uac00 \ub9c9\uc744 \ub0b4\ub9ac\uace0 \uc778\ubcf8\uc8fc\uc758 \uc2dc\ub300\uac00 \ub3c4\ub798\ud558\uc790 \uc778\uac04 \uc774\uc131 \uacfc \uacfc\ud559\uc758 \ud798\uc740 \uc2e0\uc744 \ub300\uccb4\ud558\uc600\ub2e4. \uc774\uc131\uc740 \ubc95(Law)\uc758 \uc774\ub984\uc73c\ub85c\ub3c4 \ud45c\ud604\ub418\uc5b4 \uc774\uc81c \ubc95\uc740 \ub2e8\uc21c\ud788 \ud5cc\ubc95, \ubc95\ub960\ubfd0\ub9cc\uc774 \uc544\ub2c8\ub77c \ubc95\uc774\uc0c1\uc758 \ucd08\ubc95\uc801\uc778 \uc790\uc5f0\ubc95\uae4c\uc9c0 \ud3ec\ud568\ud558\uac8c \ub41c\ub2e4. \uc774\uc5d0 \uc218\ubc18\ub41c \ud3c9\ub4f1\uad8c\ub3c4, \ub2e8\uc9c0 \ud615\ubc8c\uc0c1\uc758 \ud3c9\ub4f1\uc774\ub098 \ubc95\ub960\uc0c1\uc758 \ud3c9\ub4f1\uc774 \uc544\ub2c8\ub77c, \ucd08\ubc95\ub960\uc801 \uc790\uc5f0\uad8c\uc73c\ub85c \ud655\ub300\ub3fc \ud5cc\ubc9510\uc870\uc758 \ubcf4\ud3b8\uc801 \ubd88\uac00\uce68\uc801 \uc874\uc5c4\uad8c, 11\uc870\uc758 \ubc95\uc55e\uc758 \ud3c9\ub4f1\uad8c, 34\uc870\uc758 \uc778\uac04\ub2f5\uac8c \uc0b4 \uad8c\ub9ac\ub85c \ud655\uace0\ud788 \uac01\uc778\ub418\uc5b4 \uc778\uac04\uc740 \ub204\uad6c\ub098 \uc815\uce58, \uacbd\uc81c, \uc0ac\ud68c,\ubb38\ud654\uc801\uc73c\ub85c \uc5b4\ub5a0\ud55c \ucc28\ubcc4\ub3c4 \ubc1b\uc9c0 \uc54a\uc744 \uc790\uc5f0\uad8c\uc801 \ud3c9\ub4f1\uad8c\uc744 \ubcf4\uc7a5\ubc1b\uace0 \uad6d\uac00\ub294 \uc774\ub97c \ubcf4\uc7a5\ud560 \uc758\ubb34\uac00 \uc788\uc74c\uc744 \uaddc\uc815\ud588\ub2e4.", "sentence_answer": "\uc778\uac04 \uc774\uc131 \uacfc \uacfc\ud559\uc758 \ud798\uc740 \uc2e0\uc744 \ub300\uccb4\ud558\uc600\ub2e4.", "paragraph_id": "6109461-0-1", "paragraph_question": "question: \ubc95\uc758 \uc774\ub984\uc73c\ub85c\ub3c4 \ud45c\ud604\ub418\uba70, \ud5cc\ubc95, \ubc95\ub960,\ubc95\uc774\uc0c1\uc758 \ucd08\ubc95\uc801\uc778 \uc790\uc5f0\ubc95\uae4c\uc9c0 \ud3ec\ud568\ud558\uac8c \ub418\ub294\uac83\uc740?, context: \ubc95 \uc55e\uc758 \ud3c9\ub4f1(\uc601\uc5b4: Equality before the law)\uc758 \uad00\ub150\uc740 \ud50c\ub77c\ud1a4\uc758'\uad6d\uac00\ub860'\uc5d0\uc11c\uc758 \uc815\uc758\uc774\ub150\uacfc'\uc2e0\uc55e\uc758 \ud3c9\ub4f1'\uc774\ub77c\ub294 \uc885\uad50\uc0ac\uc0c1\uc5d0\uc11c \uc5f0\uc6d0\ud55c\ub2e4. \ub974\ub124\uc0c1\uc2a4\uae30 \uc2e0\ubcf8\uc8fc\uc758\uac00 \ub9c9\uc744 \ub0b4\ub9ac\uace0 \uc778\ubcf8\uc8fc\uc758 \uc2dc\ub300\uac00 \ub3c4\ub798\ud558\uc790 \uc778\uac04 \uc774\uc131\uacfc \uacfc\ud559\uc758 \ud798\uc740 \uc2e0\uc744 \ub300\uccb4\ud558\uc600\ub2e4. \uc774\uc131\uc740 \ubc95(Law)\uc758 \uc774\ub984\uc73c\ub85c\ub3c4 \ud45c\ud604\ub418\uc5b4 \uc774\uc81c \ubc95\uc740 \ub2e8\uc21c\ud788 \ud5cc\ubc95, \ubc95\ub960\ubfd0\ub9cc\uc774 \uc544\ub2c8\ub77c \ubc95\uc774\uc0c1\uc758 \ucd08\ubc95\uc801\uc778 \uc790\uc5f0\ubc95\uae4c\uc9c0 \ud3ec\ud568\ud558\uac8c \ub41c\ub2e4. \uc774\uc5d0 \uc218\ubc18\ub41c \ud3c9\ub4f1\uad8c\ub3c4, \ub2e8\uc9c0 \ud615\ubc8c\uc0c1\uc758 \ud3c9\ub4f1\uc774\ub098 \ubc95\ub960\uc0c1\uc758 \ud3c9\ub4f1\uc774 \uc544\ub2c8\ub77c, \ucd08\ubc95\ub960\uc801 \uc790\uc5f0\uad8c\uc73c\ub85c \ud655\ub300\ub3fc \ud5cc\ubc9510\uc870\uc758 \ubcf4\ud3b8\uc801 \ubd88\uac00\uce68\uc801 \uc874\uc5c4\uad8c, 11\uc870\uc758 \ubc95\uc55e\uc758 \ud3c9\ub4f1\uad8c, 34\uc870\uc758 \uc778\uac04\ub2f5\uac8c \uc0b4 \uad8c\ub9ac\ub85c \ud655\uace0\ud788 \uac01\uc778\ub418\uc5b4 \uc778\uac04\uc740 \ub204\uad6c\ub098 \uc815\uce58, \uacbd\uc81c, \uc0ac\ud68c,\ubb38\ud654\uc801\uc73c\ub85c \uc5b4\ub5a0\ud55c \ucc28\ubcc4\ub3c4 \ubc1b\uc9c0 \uc54a\uc744 \uc790\uc5f0\uad8c\uc801 \ud3c9\ub4f1\uad8c\uc744 \ubcf4\uc7a5\ubc1b\uace0 \uad6d\uac00\ub294 \uc774\ub97c \ubcf4\uc7a5\ud560 \uc758\ubb34\uac00 \uc788\uc74c\uc744 \uaddc\uc815\ud588\ub2e4."} {"question": "\uae40\uba85\uc218\uac00 \uc8fc\ud55c\ubbf8\uad70\uc5d0 \ub300\ud574 \uc774\ub840\uc801\uc778 \ud310\uacb0\uc744 \ub0b4\ub9b0 \ub144\ub3c4\ub294?", "paragraph": "\ub300\ubc95\uc6d0 \uc7ac\ud310\uc5f0\uad6c\uad00, \uc11c\uc6b8\uc911\uc559\uc9c0\ubc29\ubc95\uc6d0 \ubd80\uc7a5\ud310\uc0ac, \ud2b9\ud5c8\ubc95\uc6d0 \ubd80\uc7a5\ud310\uc0ac, \uc11c\uc6b8\uace0\ub4f1\ubc95\uc6d0 \ubd80\uc7a5\ud310\uc0ac \ub4f1\uc744 \uc9c0\ub0b4\uace0, 2016\ub144\ubd80\ud130 \ucd98\ucc9c\uc9c0\ubc29\ubc95\uc6d0\uc7a5\uc744 \ub9e1\uc558\ub2e4. 2002\ub144\uc5d0\ub294 \uc8fc\ud55c\ubbf8\uad70\uc774 \ub0b8 \uad50\ud1b5\uc0ac\uace0\ub85c \ud53c\ud574\uc790\uac00 \uc2dd\ubb3c\uc778\uac04\uc774 \ub41c \uc0ac\uac74\uc5d0\uc11c \uac00\ud574\uc790\uac00 \"\uc2e0\ud638\uc704\ubc18 \uc0ac\uc2e4\uc744 \ubd80\uc778\ud558\uba70 \ud53c\ud574 \ubc30\uc0c1\uc744 \uc9c0\uccb4\ud558\ub294 \ub4f1 \ubc18\uc131\uc758 \uae30\ubbf8\uac00 \ubcf4\uc774\uc9c0 \uc54a\ub294\ub2e4\"\uba70 \uc9d5\uc5ed 8\uc6d4\uc744 \uc120\uace0\ud558\uc5ec \uc8fc\ud55c\ubbf8\uad70\uc5d0 \ub300\ud574 \uc774\ub840\uc801\uc778 \ud310\uacb0\uc744\ud55c \ubc14\uac00 \uc788\uc73c\uba70 \uc11c\uc6b8\uace0\ub4f1\ubc95\uc6d0 \ubd80\uc7a5\ud310\uc0ac\ub85c \uc7ac\uc9c1\ud558\ub358 2015\ub144\uc5d0 \uc804\uad50\uc870\uac00 \uccad\uad6c\ud55c \ubc95\uc678\ub178\uc870 \ud6a8\ub825\uc815\uc9c0 \uac00\ucc98\ubd84 \uc2e0\uccad \uc0ac\uac74\uc5d0\uc11c \"\uad50\uc6d0\ub178\uc870\ubc95 \uc870\ud56d\uc774 \ud5cc\ubc95\uc5d0 \uc704\ubc18\ub41c\ub2e4\uace0 \uc758\uc2ec\ud560 \ub9cc\ud55c \uc0c1\ub2f9\ud55c \uc774\uc720\uac00 \uc788\uc5b4 \uc704\ud5cc\ubc95\ub960\uc2ec\ud310\uc81c\uccad\uc744 \ud588\ub2e4\ub294 \uc810\uc744 \uc804\uc81c\ub85c \uc804\uad50\uc870 \ubc95\uc678\ub178\uc870 \ud1b5\ubcf4 \ucc98\ubd84\uc5d0 \ub300\ud55c \ud6a8\ub825\uc815\uc9c0 \uc0ac\uc720\uac00 \uc778\uc815\ub41c\ub2e4\uace0 \ubcf8 \uc6d0\uc2ec\uc758 \ud310\ub2e8\uc5d0\ub294 \uc9d1\ud589\uc815\uc9c0\uc5d0 \uad00\ud55c \ubc95\ub9ac\ub97c \uc624\ud574\ud55c \uc704\ubc95\uc774 \uc788\ub2e4\"\uace0 \ud558\uc5ec \uc0ac\uac74\uc744 \ud30c\uae30\ud658\uc1a1\ud55c \ub300\ubc95\uc6d0\uc758 \uc785\uc7a5\uc5d0 \ubc18\ud558\uc5ec \uac00\ucc98\ubd84 \uc2e0\uccad\uc744 \uc778\uc6a9\ud558\uc600\uace0 \uc0bc\uc131\uc5d0\ubc84\ub79c\ub4dc \ub178\ub3d9\uc870\ud569 \ubd80\uc9c0\ud68c\uc7a5\uc758 \ud574\uace0 \uc0ac\uac74\uc5d0\uc11c \ubd80\ub2f9\ud574\uace0 \ud310\uacb0\uc744 \ud588\ub2e4.", "answer": "2002\ub144", "sentence": "2002\ub144 \uc5d0\ub294 \uc8fc\ud55c\ubbf8\uad70\uc774 \ub0b8 \uad50\ud1b5\uc0ac\uace0\ub85c \ud53c\ud574\uc790\uac00 \uc2dd\ubb3c\uc778\uac04\uc774 \ub41c \uc0ac\uac74\uc5d0\uc11c \uac00\ud574\uc790\uac00 \"\uc2e0\ud638\uc704\ubc18 \uc0ac\uc2e4\uc744 \ubd80\uc778\ud558\uba70 \ud53c\ud574 \ubc30\uc0c1\uc744 \uc9c0\uccb4\ud558\ub294 \ub4f1 \ubc18\uc131\uc758 \uae30\ubbf8\uac00 \ubcf4\uc774\uc9c0 \uc54a\ub294\ub2e4\"\uba70 \uc9d5\uc5ed 8\uc6d4\uc744 \uc120\uace0\ud558\uc5ec \uc8fc\ud55c\ubbf8\uad70\uc5d0 \ub300\ud574 \uc774\ub840\uc801\uc778 \ud310\uacb0\uc744\ud55c \ubc14\uac00 \uc788\uc73c\uba70 \uc11c\uc6b8\uace0\ub4f1\ubc95\uc6d0 \ubd80\uc7a5\ud310\uc0ac\ub85c \uc7ac\uc9c1\ud558\ub358 2015\ub144\uc5d0 \uc804\uad50\uc870\uac00 \uccad\uad6c\ud55c \ubc95\uc678\ub178\uc870 \ud6a8\ub825\uc815\uc9c0 \uac00\ucc98\ubd84 \uc2e0\uccad \uc0ac\uac74\uc5d0\uc11c \"\uad50\uc6d0\ub178\uc870\ubc95 \uc870\ud56d\uc774 \ud5cc\ubc95\uc5d0 \uc704\ubc18\ub41c\ub2e4\uace0 \uc758\uc2ec\ud560 \ub9cc\ud55c \uc0c1\ub2f9\ud55c \uc774\uc720\uac00 \uc788\uc5b4 \uc704\ud5cc\ubc95\ub960\uc2ec\ud310\uc81c\uccad\uc744 \ud588\ub2e4\ub294 \uc810\uc744 \uc804\uc81c\ub85c \uc804\uad50\uc870 \ubc95\uc678\ub178\uc870 \ud1b5\ubcf4 \ucc98\ubd84\uc5d0 \ub300\ud55c \ud6a8\ub825\uc815\uc9c0 \uc0ac\uc720\uac00 \uc778\uc815\ub41c\ub2e4\uace0 \ubcf8 \uc6d0\uc2ec\uc758 \ud310\ub2e8\uc5d0\ub294 \uc9d1\ud589\uc815\uc9c0\uc5d0 \uad00\ud55c \ubc95\ub9ac\ub97c \uc624\ud574\ud55c \uc704\ubc95\uc774 \uc788\ub2e4\"\uace0 \ud558\uc5ec \uc0ac\uac74\uc744 \ud30c\uae30\ud658\uc1a1\ud55c \ub300\ubc95\uc6d0\uc758 \uc785\uc7a5\uc5d0 \ubc18\ud558\uc5ec \uac00\ucc98\ubd84 \uc2e0\uccad\uc744 \uc778\uc6a9\ud558\uc600\uace0 \uc0bc\uc131\uc5d0\ubc84\ub79c\ub4dc \ub178\ub3d9\uc870\ud569 \ubd80\uc9c0\ud68c\uc7a5\uc758 \ud574\uace0 \uc0ac\uac74\uc5d0\uc11c \ubd80\ub2f9\ud574\uace0 \ud310\uacb0\uc744 \ud588\ub2e4.", "paragraph_sentence": "\ub300\ubc95\uc6d0 \uc7ac\ud310\uc5f0\uad6c\uad00, \uc11c\uc6b8\uc911\uc559\uc9c0\ubc29\ubc95\uc6d0 \ubd80\uc7a5\ud310\uc0ac, \ud2b9\ud5c8\ubc95\uc6d0 \ubd80\uc7a5\ud310\uc0ac, \uc11c\uc6b8\uace0\ub4f1\ubc95\uc6d0 \ubd80\uc7a5\ud310\uc0ac \ub4f1\uc744 \uc9c0\ub0b4\uace0, 2016\ub144\ubd80\ud130 \ucd98\ucc9c\uc9c0\ubc29\ubc95\uc6d0\uc7a5\uc744 \ub9e1\uc558\ub2e4. 2002\ub144 \uc5d0\ub294 \uc8fc\ud55c\ubbf8\uad70\uc774 \ub0b8 \uad50\ud1b5\uc0ac\uace0\ub85c \ud53c\ud574\uc790\uac00 \uc2dd\ubb3c\uc778\uac04\uc774 \ub41c \uc0ac\uac74\uc5d0\uc11c \uac00\ud574\uc790\uac00 \"\uc2e0\ud638\uc704\ubc18 \uc0ac\uc2e4\uc744 \ubd80\uc778\ud558\uba70 \ud53c\ud574 \ubc30\uc0c1\uc744 \uc9c0\uccb4\ud558\ub294 \ub4f1 \ubc18\uc131\uc758 \uae30\ubbf8\uac00 \ubcf4\uc774\uc9c0 \uc54a\ub294\ub2e4\"\uba70 \uc9d5\uc5ed 8\uc6d4\uc744 \uc120\uace0\ud558\uc5ec \uc8fc\ud55c\ubbf8\uad70\uc5d0 \ub300\ud574 \uc774\ub840\uc801\uc778 \ud310\uacb0\uc744\ud55c \ubc14\uac00 \uc788\uc73c\uba70 \uc11c\uc6b8\uace0\ub4f1\ubc95\uc6d0 \ubd80\uc7a5\ud310\uc0ac\ub85c \uc7ac\uc9c1\ud558\ub358 2015\ub144\uc5d0 \uc804\uad50\uc870\uac00 \uccad\uad6c\ud55c \ubc95\uc678\ub178\uc870 \ud6a8\ub825\uc815\uc9c0 \uac00\ucc98\ubd84 \uc2e0\uccad \uc0ac\uac74\uc5d0\uc11c \"\uad50\uc6d0\ub178\uc870\ubc95 \uc870\ud56d\uc774 \ud5cc\ubc95\uc5d0 \uc704\ubc18\ub41c\ub2e4\uace0 \uc758\uc2ec\ud560 \ub9cc\ud55c \uc0c1\ub2f9\ud55c \uc774\uc720\uac00 \uc788\uc5b4 \uc704\ud5cc\ubc95\ub960\uc2ec\ud310\uc81c\uccad\uc744 \ud588\ub2e4\ub294 \uc810\uc744 \uc804\uc81c\ub85c \uc804\uad50\uc870 \ubc95\uc678\ub178\uc870 \ud1b5\ubcf4 \ucc98\ubd84\uc5d0 \ub300\ud55c \ud6a8\ub825\uc815\uc9c0 \uc0ac\uc720\uac00 \uc778\uc815\ub41c\ub2e4\uace0 \ubcf8 \uc6d0\uc2ec\uc758 \ud310\ub2e8\uc5d0\ub294 \uc9d1\ud589\uc815\uc9c0\uc5d0 \uad00\ud55c \ubc95\ub9ac\ub97c \uc624\ud574\ud55c \uc704\ubc95\uc774 \uc788\ub2e4\"\uace0 \ud558\uc5ec \uc0ac\uac74\uc744 \ud30c\uae30\ud658\uc1a1\ud55c \ub300\ubc95\uc6d0\uc758 \uc785\uc7a5\uc5d0 \ubc18\ud558\uc5ec \uac00\ucc98\ubd84 \uc2e0\uccad\uc744 \uc778\uc6a9\ud558\uc600\uace0 \uc0bc\uc131\uc5d0\ubc84\ub79c\ub4dc \ub178\ub3d9\uc870\ud569 \ubd80\uc9c0\ud68c\uc7a5\uc758 \ud574\uace0 \uc0ac\uac74\uc5d0\uc11c \ubd80\ub2f9\ud574\uace0 \ud310\uacb0\uc744 \ud588\ub2e4. ", "paragraph_answer": "\ub300\ubc95\uc6d0 \uc7ac\ud310\uc5f0\uad6c\uad00, \uc11c\uc6b8\uc911\uc559\uc9c0\ubc29\ubc95\uc6d0 \ubd80\uc7a5\ud310\uc0ac, \ud2b9\ud5c8\ubc95\uc6d0 \ubd80\uc7a5\ud310\uc0ac, \uc11c\uc6b8\uace0\ub4f1\ubc95\uc6d0 \ubd80\uc7a5\ud310\uc0ac \ub4f1\uc744 \uc9c0\ub0b4\uace0, 2016\ub144\ubd80\ud130 \ucd98\ucc9c\uc9c0\ubc29\ubc95\uc6d0\uc7a5\uc744 \ub9e1\uc558\ub2e4. 2002\ub144 \uc5d0\ub294 \uc8fc\ud55c\ubbf8\uad70\uc774 \ub0b8 \uad50\ud1b5\uc0ac\uace0\ub85c \ud53c\ud574\uc790\uac00 \uc2dd\ubb3c\uc778\uac04\uc774 \ub41c \uc0ac\uac74\uc5d0\uc11c \uac00\ud574\uc790\uac00 \"\uc2e0\ud638\uc704\ubc18 \uc0ac\uc2e4\uc744 \ubd80\uc778\ud558\uba70 \ud53c\ud574 \ubc30\uc0c1\uc744 \uc9c0\uccb4\ud558\ub294 \ub4f1 \ubc18\uc131\uc758 \uae30\ubbf8\uac00 \ubcf4\uc774\uc9c0 \uc54a\ub294\ub2e4\"\uba70 \uc9d5\uc5ed 8\uc6d4\uc744 \uc120\uace0\ud558\uc5ec \uc8fc\ud55c\ubbf8\uad70\uc5d0 \ub300\ud574 \uc774\ub840\uc801\uc778 \ud310\uacb0\uc744\ud55c \ubc14\uac00 \uc788\uc73c\uba70 \uc11c\uc6b8\uace0\ub4f1\ubc95\uc6d0 \ubd80\uc7a5\ud310\uc0ac\ub85c \uc7ac\uc9c1\ud558\ub358 2015\ub144\uc5d0 \uc804\uad50\uc870\uac00 \uccad\uad6c\ud55c \ubc95\uc678\ub178\uc870 \ud6a8\ub825\uc815\uc9c0 \uac00\ucc98\ubd84 \uc2e0\uccad \uc0ac\uac74\uc5d0\uc11c \"\uad50\uc6d0\ub178\uc870\ubc95 \uc870\ud56d\uc774 \ud5cc\ubc95\uc5d0 \uc704\ubc18\ub41c\ub2e4\uace0 \uc758\uc2ec\ud560 \ub9cc\ud55c \uc0c1\ub2f9\ud55c \uc774\uc720\uac00 \uc788\uc5b4 \uc704\ud5cc\ubc95\ub960\uc2ec\ud310\uc81c\uccad\uc744 \ud588\ub2e4\ub294 \uc810\uc744 \uc804\uc81c\ub85c \uc804\uad50\uc870 \ubc95\uc678\ub178\uc870 \ud1b5\ubcf4 \ucc98\ubd84\uc5d0 \ub300\ud55c \ud6a8\ub825\uc815\uc9c0 \uc0ac\uc720\uac00 \uc778\uc815\ub41c\ub2e4\uace0 \ubcf8 \uc6d0\uc2ec\uc758 \ud310\ub2e8\uc5d0\ub294 \uc9d1\ud589\uc815\uc9c0\uc5d0 \uad00\ud55c \ubc95\ub9ac\ub97c \uc624\ud574\ud55c \uc704\ubc95\uc774 \uc788\ub2e4\"\uace0 \ud558\uc5ec \uc0ac\uac74\uc744 \ud30c\uae30\ud658\uc1a1\ud55c \ub300\ubc95\uc6d0\uc758 \uc785\uc7a5\uc5d0 \ubc18\ud558\uc5ec \uac00\ucc98\ubd84 \uc2e0\uccad\uc744 \uc778\uc6a9\ud558\uc600\uace0 \uc0bc\uc131\uc5d0\ubc84\ub79c\ub4dc \ub178\ub3d9\uc870\ud569 \ubd80\uc9c0\ud68c\uc7a5\uc758 \ud574\uace0 \uc0ac\uac74\uc5d0\uc11c \ubd80\ub2f9\ud574\uace0 \ud310\uacb0\uc744 \ud588\ub2e4.", "sentence_answer": " 2002\ub144 \uc5d0\ub294 \uc8fc\ud55c\ubbf8\uad70\uc774 \ub0b8 \uad50\ud1b5\uc0ac\uace0\ub85c \ud53c\ud574\uc790\uac00 \uc2dd\ubb3c\uc778\uac04\uc774 \ub41c \uc0ac\uac74\uc5d0\uc11c \uac00\ud574\uc790\uac00 \"\uc2e0\ud638\uc704\ubc18 \uc0ac\uc2e4\uc744 \ubd80\uc778\ud558\uba70 \ud53c\ud574 \ubc30\uc0c1\uc744 \uc9c0\uccb4\ud558\ub294 \ub4f1 \ubc18\uc131\uc758 \uae30\ubbf8\uac00 \ubcf4\uc774\uc9c0 \uc54a\ub294\ub2e4\"\uba70 \uc9d5\uc5ed 8\uc6d4\uc744 \uc120\uace0\ud558\uc5ec \uc8fc\ud55c\ubbf8\uad70\uc5d0 \ub300\ud574 \uc774\ub840\uc801\uc778 \ud310\uacb0\uc744\ud55c \ubc14\uac00 \uc788\uc73c\uba70 \uc11c\uc6b8\uace0\ub4f1\ubc95\uc6d0 \ubd80\uc7a5\ud310\uc0ac\ub85c \uc7ac\uc9c1\ud558\ub358 2015\ub144\uc5d0 \uc804\uad50\uc870\uac00 \uccad\uad6c\ud55c \ubc95\uc678\ub178\uc870 \ud6a8\ub825\uc815\uc9c0 \uac00\ucc98\ubd84 \uc2e0\uccad \uc0ac\uac74\uc5d0\uc11c \"\uad50\uc6d0\ub178\uc870\ubc95 \uc870\ud56d\uc774 \ud5cc\ubc95\uc5d0 \uc704\ubc18\ub41c\ub2e4\uace0 \uc758\uc2ec\ud560 \ub9cc\ud55c \uc0c1\ub2f9\ud55c \uc774\uc720\uac00 \uc788\uc5b4 \uc704\ud5cc\ubc95\ub960\uc2ec\ud310\uc81c\uccad\uc744 \ud588\ub2e4\ub294 \uc810\uc744 \uc804\uc81c\ub85c \uc804\uad50\uc870 \ubc95\uc678\ub178\uc870 \ud1b5\ubcf4 \ucc98\ubd84\uc5d0 \ub300\ud55c \ud6a8\ub825\uc815\uc9c0 \uc0ac\uc720\uac00 \uc778\uc815\ub41c\ub2e4\uace0 \ubcf8 \uc6d0\uc2ec\uc758 \ud310\ub2e8\uc5d0\ub294 \uc9d1\ud589\uc815\uc9c0\uc5d0 \uad00\ud55c \ubc95\ub9ac\ub97c \uc624\ud574\ud55c \uc704\ubc95\uc774 \uc788\ub2e4\"\uace0 \ud558\uc5ec \uc0ac\uac74\uc744 \ud30c\uae30\ud658\uc1a1\ud55c \ub300\ubc95\uc6d0\uc758 \uc785\uc7a5\uc5d0 \ubc18\ud558\uc5ec \uac00\ucc98\ubd84 \uc2e0\uccad\uc744 \uc778\uc6a9\ud558\uc600\uace0 \uc0bc\uc131\uc5d0\ubc84\ub79c\ub4dc \ub178\ub3d9\uc870\ud569 \ubd80\uc9c0\ud68c\uc7a5\uc758 \ud574\uace0 \uc0ac\uac74\uc5d0\uc11c \ubd80\ub2f9\ud574\uace0 \ud310\uacb0\uc744 \ud588\ub2e4.", "paragraph_id": "6481522-0-1", "paragraph_question": "question: \uae40\uba85\uc218\uac00 \uc8fc\ud55c\ubbf8\uad70\uc5d0 \ub300\ud574 \uc774\ub840\uc801\uc778 \ud310\uacb0\uc744 \ub0b4\ub9b0 \ub144\ub3c4\ub294?, context: \ub300\ubc95\uc6d0 \uc7ac\ud310\uc5f0\uad6c\uad00, \uc11c\uc6b8\uc911\uc559\uc9c0\ubc29\ubc95\uc6d0 \ubd80\uc7a5\ud310\uc0ac, \ud2b9\ud5c8\ubc95\uc6d0 \ubd80\uc7a5\ud310\uc0ac, \uc11c\uc6b8\uace0\ub4f1\ubc95\uc6d0 \ubd80\uc7a5\ud310\uc0ac \ub4f1\uc744 \uc9c0\ub0b4\uace0, 2016\ub144\ubd80\ud130 \ucd98\ucc9c\uc9c0\ubc29\ubc95\uc6d0\uc7a5\uc744 \ub9e1\uc558\ub2e4. 2002\ub144\uc5d0\ub294 \uc8fc\ud55c\ubbf8\uad70\uc774 \ub0b8 \uad50\ud1b5\uc0ac\uace0\ub85c \ud53c\ud574\uc790\uac00 \uc2dd\ubb3c\uc778\uac04\uc774 \ub41c \uc0ac\uac74\uc5d0\uc11c \uac00\ud574\uc790\uac00 \"\uc2e0\ud638\uc704\ubc18 \uc0ac\uc2e4\uc744 \ubd80\uc778\ud558\uba70 \ud53c\ud574 \ubc30\uc0c1\uc744 \uc9c0\uccb4\ud558\ub294 \ub4f1 \ubc18\uc131\uc758 \uae30\ubbf8\uac00 \ubcf4\uc774\uc9c0 \uc54a\ub294\ub2e4\"\uba70 \uc9d5\uc5ed 8\uc6d4\uc744 \uc120\uace0\ud558\uc5ec \uc8fc\ud55c\ubbf8\uad70\uc5d0 \ub300\ud574 \uc774\ub840\uc801\uc778 \ud310\uacb0\uc744\ud55c \ubc14\uac00 \uc788\uc73c\uba70 \uc11c\uc6b8\uace0\ub4f1\ubc95\uc6d0 \ubd80\uc7a5\ud310\uc0ac\ub85c \uc7ac\uc9c1\ud558\ub358 2015\ub144\uc5d0 \uc804\uad50\uc870\uac00 \uccad\uad6c\ud55c \ubc95\uc678\ub178\uc870 \ud6a8\ub825\uc815\uc9c0 \uac00\ucc98\ubd84 \uc2e0\uccad \uc0ac\uac74\uc5d0\uc11c \"\uad50\uc6d0\ub178\uc870\ubc95 \uc870\ud56d\uc774 \ud5cc\ubc95\uc5d0 \uc704\ubc18\ub41c\ub2e4\uace0 \uc758\uc2ec\ud560 \ub9cc\ud55c \uc0c1\ub2f9\ud55c \uc774\uc720\uac00 \uc788\uc5b4 \uc704\ud5cc\ubc95\ub960\uc2ec\ud310\uc81c\uccad\uc744 \ud588\ub2e4\ub294 \uc810\uc744 \uc804\uc81c\ub85c \uc804\uad50\uc870 \ubc95\uc678\ub178\uc870 \ud1b5\ubcf4 \ucc98\ubd84\uc5d0 \ub300\ud55c \ud6a8\ub825\uc815\uc9c0 \uc0ac\uc720\uac00 \uc778\uc815\ub41c\ub2e4\uace0 \ubcf8 \uc6d0\uc2ec\uc758 \ud310\ub2e8\uc5d0\ub294 \uc9d1\ud589\uc815\uc9c0\uc5d0 \uad00\ud55c \ubc95\ub9ac\ub97c \uc624\ud574\ud55c \uc704\ubc95\uc774 \uc788\ub2e4\"\uace0 \ud558\uc5ec \uc0ac\uac74\uc744 \ud30c\uae30\ud658\uc1a1\ud55c \ub300\ubc95\uc6d0\uc758 \uc785\uc7a5\uc5d0 \ubc18\ud558\uc5ec \uac00\ucc98\ubd84 \uc2e0\uccad\uc744 \uc778\uc6a9\ud558\uc600\uace0 \uc0bc\uc131\uc5d0\ubc84\ub79c\ub4dc \ub178\ub3d9\uc870\ud569 \ubd80\uc9c0\ud68c\uc7a5\uc758 \ud574\uace0 \uc0ac\uac74\uc5d0\uc11c \ubd80\ub2f9\ud574\uace0 \ud310\uacb0\uc744 \ud588\ub2e4."} {"question": "\ub77c\ubd80\uc544\uc9c0\uc5d0\uac00 \uacf5\uae30\uc758 \uc131\ubd84 \uc911 \ud2b9\uc815 \uc131\ubd84\uc774 \uc0dd\ubb3c\uc758 \ud638\ud761\uacfc \uad00\ub828\ub418\uc5b4\uc788\uc74c\uc744 \ubc1c\uacac\ud55c \uac83\uc740 \uc5b8\uc81c\uc77c\uae4c?", "paragraph": "1775\ub144 5\uc6d4 26\uc77c \ub77c\ubd80\uc544\uc9c0\uc5d0\ub294 \uacf5\uae30\uc758 \uc131\ubd84 \uc911 \ud2b9\uc815 \uc131\ubd84\uc774 \uc0dd\ubb3c\uc758 \ud638\ud761\uacfc \uad00\ub828\ub418\uc5b4\uc788\ub2e4\ub294 \uc0ac\uc2e4\uc744 \ubc1c\uacac\ud558\uace0 \uc774\ub97c '\uc0dd\uba85\uc758 \uacf5\uae30'\ub77c\uace0 \uc774\ub984\uc744 \ubd99\uc600\ub2e4. (\uc0ac\uc2e4 \uc774 \uacf5\uae30\ub294 \uc0b0\uc18c\ub85c, \uc178\ub808\uc640 \ud504\ub9ac\uc2a4\ud2c0\ub9ac\uc5d0 \uc758\ud574\uc11c \uc774\ubbf8 \ubc1c\uacac\ub418\uc5b4\uc788\uc5c8\ub2e4.) 1777\ub144\uc5d0 \uc81c\ucd9c\ud55c \ub17c\ubb38\uc778 M\u00e9moire sur la combustion en g\u00e9n\u00e9ral\uc5d0\uc11c \ub77c\ubd80\uc544\uc9c0\uc5d0\ub294 \ubaa8\ub4e0 \uc0b0\uc740 \uacf5\uae30 \ub0b4\uc758 \ud2b9\uc815 \uc131\ubd84\uc5d0 \uc758\ud574\uc11c \uc0dd\uc131\ub41c\ub2e4\uace0 \uc8fc\uc7a5\ud558\uc600\uace0, \uc774\ub97c principe oxygine\uc774\ub77c\uace0 \uba85\uba85\ud558\uc600\ub2e4. \uc774\ub294 \uadf8\ub9ac\uc2a4\uc5b4\ub85c '\uc0b0\uc744 \uc0dd\uc131\ud558\ub294 \uac83'\uc774\ub77c\ub294 \ub73b\uc73c\ub85c, \uc0b0\uc18c\uc758 \uc5b4\uc6d0\uc774 \ub418\uc5c8\ub2e4. (principe oxygine \uc5ed\uc2dc \uc0b0\uc18c\uc640 \uac19\uc740 \ubb3c\uc9c8\uc774\ub098 \ub77c\ubd80\uc544\uc9c0\uc5d0\ub294 \ucc98\uc74c\uc5d0 \uc774\ub97c \uc54c\uc9c0 \ubabb\ud558\uc600\ub2e4.) \ucd94\uac00\uc801\uc778 \uc5f0\uad6c\ub97c \ud1b5\ud574\uc11c \ub77c\ubd80\uc544\uc9c0\uc5d0\ub294 \uc124\ud0d5\uacfc principe oxygine\uc774 \ubc18\uc751\ud558\uba74 \uc625\uc0b4\uc0b0\uc774 \uc0dd\uc131\ub418\ub294 \ub4f1 \uc5ec\ub7ec \ubb3c\uc9c8\uacfc \ubc18\uc751\ud558\uc5ec \uc0b0\uc744 \uc0dd\uc131\ud55c\ub2e4\ub294 \uc0ac\uc2e4\uc744 \uc54c\uac8c \ub418\uc5c8\ub2e4. \ud604\ub300\uc758 \uc2dc\uac01\uc73c\ub85c \ubcf4\uc558\uc744 \ub54c \uc774 \uc774\ub860\uc740 \uc5fc\ud654 \uc218\uc18c \ub4f1 \uc0b0\uc18c\ub97c \ud3ec\ud568\ud558\uc9c0 \uc54a\uc740 \uc0b0\uc5d0 \uad00\ud574\uc11c\ub294 \ub4e4\uc5b4\ub9de\uc9c0 \uc54a\uc73c\ub098, \uc0b0\uc18c\uc0b0\uc744 \uc124\uba85\ud558\uae30 \uc801\ud569\ud558\ub2e4.", "answer": "1775\ub144 5\uc6d4 26\uc77c", "sentence": "1775\ub144 5\uc6d4 26\uc77c \ub77c\ubd80\uc544\uc9c0\uc5d0\ub294 \uacf5\uae30\uc758 \uc131\ubd84 \uc911 \ud2b9\uc815 \uc131\ubd84\uc774 \uc0dd\ubb3c\uc758 \ud638\ud761\uacfc \uad00\ub828\ub418\uc5b4\uc788\ub2e4\ub294 \uc0ac\uc2e4\uc744 \ubc1c\uacac\ud558\uace0 \uc774\ub97c '\uc0dd\uba85\uc758 \uacf5\uae30'\ub77c\uace0 \uc774\ub984\uc744 \ubd99\uc600\ub2e4.", "paragraph_sentence": " 1775\ub144 5\uc6d4 26\uc77c \ub77c\ubd80\uc544\uc9c0\uc5d0\ub294 \uacf5\uae30\uc758 \uc131\ubd84 \uc911 \ud2b9\uc815 \uc131\ubd84\uc774 \uc0dd\ubb3c\uc758 \ud638\ud761\uacfc \uad00\ub828\ub418\uc5b4\uc788\ub2e4\ub294 \uc0ac\uc2e4\uc744 \ubc1c\uacac\ud558\uace0 \uc774\ub97c '\uc0dd\uba85\uc758 \uacf5\uae30'\ub77c\uace0 \uc774\ub984\uc744 \ubd99\uc600\ub2e4. (\uc0ac\uc2e4 \uc774 \uacf5\uae30\ub294 \uc0b0\uc18c\ub85c, \uc178\ub808\uc640 \ud504\ub9ac\uc2a4\ud2c0\ub9ac\uc5d0 \uc758\ud574\uc11c \uc774\ubbf8 \ubc1c\uacac\ub418\uc5b4\uc788\uc5c8\ub2e4. ) 1777\ub144\uc5d0 \uc81c\ucd9c\ud55c \ub17c\ubb38\uc778 M\u00e9moire sur la combustion en g\u00e9n\u00e9ral\uc5d0\uc11c \ub77c\ubd80\uc544\uc9c0\uc5d0\ub294 \ubaa8\ub4e0 \uc0b0\uc740 \uacf5\uae30 \ub0b4\uc758 \ud2b9\uc815 \uc131\ubd84\uc5d0 \uc758\ud574\uc11c \uc0dd\uc131\ub41c\ub2e4\uace0 \uc8fc\uc7a5\ud558\uc600\uace0, \uc774\ub97c principe oxygine\uc774\ub77c\uace0 \uba85\uba85\ud558\uc600\ub2e4. \uc774\ub294 \uadf8\ub9ac\uc2a4\uc5b4\ub85c '\uc0b0\uc744 \uc0dd\uc131\ud558\ub294 \uac83'\uc774\ub77c\ub294 \ub73b\uc73c\ub85c, \uc0b0\uc18c\uc758 \uc5b4\uc6d0\uc774 \ub418\uc5c8\ub2e4. (principe oxygine \uc5ed\uc2dc \uc0b0\uc18c\uc640 \uac19\uc740 \ubb3c\uc9c8\uc774\ub098 \ub77c\ubd80\uc544\uc9c0\uc5d0\ub294 \ucc98\uc74c\uc5d0 \uc774\ub97c \uc54c\uc9c0 \ubabb\ud558\uc600\ub2e4. ) \ucd94\uac00\uc801\uc778 \uc5f0\uad6c\ub97c \ud1b5\ud574\uc11c \ub77c\ubd80\uc544\uc9c0\uc5d0\ub294 \uc124\ud0d5\uacfc principe oxygine\uc774 \ubc18\uc751\ud558\uba74 \uc625\uc0b4\uc0b0\uc774 \uc0dd\uc131\ub418\ub294 \ub4f1 \uc5ec\ub7ec \ubb3c\uc9c8\uacfc \ubc18\uc751\ud558\uc5ec \uc0b0\uc744 \uc0dd\uc131\ud55c\ub2e4\ub294 \uc0ac\uc2e4\uc744 \uc54c\uac8c \ub418\uc5c8\ub2e4. \ud604\ub300\uc758 \uc2dc\uac01\uc73c\ub85c \ubcf4\uc558\uc744 \ub54c \uc774 \uc774\ub860\uc740 \uc5fc\ud654 \uc218\uc18c \ub4f1 \uc0b0\uc18c\ub97c \ud3ec\ud568\ud558\uc9c0 \uc54a\uc740 \uc0b0\uc5d0 \uad00\ud574\uc11c\ub294 \ub4e4\uc5b4\ub9de\uc9c0 \uc54a\uc73c\ub098, \uc0b0\uc18c\uc0b0\uc744 \uc124\uba85\ud558\uae30 \uc801\ud569\ud558\ub2e4.", "paragraph_answer": " 1775\ub144 5\uc6d4 26\uc77c \ub77c\ubd80\uc544\uc9c0\uc5d0\ub294 \uacf5\uae30\uc758 \uc131\ubd84 \uc911 \ud2b9\uc815 \uc131\ubd84\uc774 \uc0dd\ubb3c\uc758 \ud638\ud761\uacfc \uad00\ub828\ub418\uc5b4\uc788\ub2e4\ub294 \uc0ac\uc2e4\uc744 \ubc1c\uacac\ud558\uace0 \uc774\ub97c '\uc0dd\uba85\uc758 \uacf5\uae30'\ub77c\uace0 \uc774\ub984\uc744 \ubd99\uc600\ub2e4. (\uc0ac\uc2e4 \uc774 \uacf5\uae30\ub294 \uc0b0\uc18c\ub85c, \uc178\ub808\uc640 \ud504\ub9ac\uc2a4\ud2c0\ub9ac\uc5d0 \uc758\ud574\uc11c \uc774\ubbf8 \ubc1c\uacac\ub418\uc5b4\uc788\uc5c8\ub2e4.) 1777\ub144\uc5d0 \uc81c\ucd9c\ud55c \ub17c\ubb38\uc778 M\u00e9moire sur la combustion en g\u00e9n\u00e9ral\uc5d0\uc11c \ub77c\ubd80\uc544\uc9c0\uc5d0\ub294 \ubaa8\ub4e0 \uc0b0\uc740 \uacf5\uae30 \ub0b4\uc758 \ud2b9\uc815 \uc131\ubd84\uc5d0 \uc758\ud574\uc11c \uc0dd\uc131\ub41c\ub2e4\uace0 \uc8fc\uc7a5\ud558\uc600\uace0, \uc774\ub97c principe oxygine\uc774\ub77c\uace0 \uba85\uba85\ud558\uc600\ub2e4. \uc774\ub294 \uadf8\ub9ac\uc2a4\uc5b4\ub85c '\uc0b0\uc744 \uc0dd\uc131\ud558\ub294 \uac83'\uc774\ub77c\ub294 \ub73b\uc73c\ub85c, \uc0b0\uc18c\uc758 \uc5b4\uc6d0\uc774 \ub418\uc5c8\ub2e4. (principe oxygine \uc5ed\uc2dc \uc0b0\uc18c\uc640 \uac19\uc740 \ubb3c\uc9c8\uc774\ub098 \ub77c\ubd80\uc544\uc9c0\uc5d0\ub294 \ucc98\uc74c\uc5d0 \uc774\ub97c \uc54c\uc9c0 \ubabb\ud558\uc600\ub2e4.) \ucd94\uac00\uc801\uc778 \uc5f0\uad6c\ub97c \ud1b5\ud574\uc11c \ub77c\ubd80\uc544\uc9c0\uc5d0\ub294 \uc124\ud0d5\uacfc principe oxygine\uc774 \ubc18\uc751\ud558\uba74 \uc625\uc0b4\uc0b0\uc774 \uc0dd\uc131\ub418\ub294 \ub4f1 \uc5ec\ub7ec \ubb3c\uc9c8\uacfc \ubc18\uc751\ud558\uc5ec \uc0b0\uc744 \uc0dd\uc131\ud55c\ub2e4\ub294 \uc0ac\uc2e4\uc744 \uc54c\uac8c \ub418\uc5c8\ub2e4. \ud604\ub300\uc758 \uc2dc\uac01\uc73c\ub85c \ubcf4\uc558\uc744 \ub54c \uc774 \uc774\ub860\uc740 \uc5fc\ud654 \uc218\uc18c \ub4f1 \uc0b0\uc18c\ub97c \ud3ec\ud568\ud558\uc9c0 \uc54a\uc740 \uc0b0\uc5d0 \uad00\ud574\uc11c\ub294 \ub4e4\uc5b4\ub9de\uc9c0 \uc54a\uc73c\ub098, \uc0b0\uc18c\uc0b0\uc744 \uc124\uba85\ud558\uae30 \uc801\ud569\ud558\ub2e4.", "sentence_answer": " 1775\ub144 5\uc6d4 26\uc77c \ub77c\ubd80\uc544\uc9c0\uc5d0\ub294 \uacf5\uae30\uc758 \uc131\ubd84 \uc911 \ud2b9\uc815 \uc131\ubd84\uc774 \uc0dd\ubb3c\uc758 \ud638\ud761\uacfc \uad00\ub828\ub418\uc5b4\uc788\ub2e4\ub294 \uc0ac\uc2e4\uc744 \ubc1c\uacac\ud558\uace0 \uc774\ub97c '\uc0dd\uba85\uc758 \uacf5\uae30'\ub77c\uace0 \uc774\ub984\uc744 \ubd99\uc600\ub2e4.", "paragraph_id": "6540101-7-0", "paragraph_question": "question: \ub77c\ubd80\uc544\uc9c0\uc5d0\uac00 \uacf5\uae30\uc758 \uc131\ubd84 \uc911 \ud2b9\uc815 \uc131\ubd84\uc774 \uc0dd\ubb3c\uc758 \ud638\ud761\uacfc \uad00\ub828\ub418\uc5b4\uc788\uc74c\uc744 \ubc1c\uacac\ud55c \uac83\uc740 \uc5b8\uc81c\uc77c\uae4c?, context: 1775\ub144 5\uc6d4 26\uc77c \ub77c\ubd80\uc544\uc9c0\uc5d0\ub294 \uacf5\uae30\uc758 \uc131\ubd84 \uc911 \ud2b9\uc815 \uc131\ubd84\uc774 \uc0dd\ubb3c\uc758 \ud638\ud761\uacfc \uad00\ub828\ub418\uc5b4\uc788\ub2e4\ub294 \uc0ac\uc2e4\uc744 \ubc1c\uacac\ud558\uace0 \uc774\ub97c '\uc0dd\uba85\uc758 \uacf5\uae30'\ub77c\uace0 \uc774\ub984\uc744 \ubd99\uc600\ub2e4. (\uc0ac\uc2e4 \uc774 \uacf5\uae30\ub294 \uc0b0\uc18c\ub85c, \uc178\ub808\uc640 \ud504\ub9ac\uc2a4\ud2c0\ub9ac\uc5d0 \uc758\ud574\uc11c \uc774\ubbf8 \ubc1c\uacac\ub418\uc5b4\uc788\uc5c8\ub2e4.) 1777\ub144\uc5d0 \uc81c\ucd9c\ud55c \ub17c\ubb38\uc778 M\u00e9moire sur la combustion en g\u00e9n\u00e9ral\uc5d0\uc11c \ub77c\ubd80\uc544\uc9c0\uc5d0\ub294 \ubaa8\ub4e0 \uc0b0\uc740 \uacf5\uae30 \ub0b4\uc758 \ud2b9\uc815 \uc131\ubd84\uc5d0 \uc758\ud574\uc11c \uc0dd\uc131\ub41c\ub2e4\uace0 \uc8fc\uc7a5\ud558\uc600\uace0, \uc774\ub97c principe oxygine\uc774\ub77c\uace0 \uba85\uba85\ud558\uc600\ub2e4. \uc774\ub294 \uadf8\ub9ac\uc2a4\uc5b4\ub85c '\uc0b0\uc744 \uc0dd\uc131\ud558\ub294 \uac83'\uc774\ub77c\ub294 \ub73b\uc73c\ub85c, \uc0b0\uc18c\uc758 \uc5b4\uc6d0\uc774 \ub418\uc5c8\ub2e4. (principe oxygine \uc5ed\uc2dc \uc0b0\uc18c\uc640 \uac19\uc740 \ubb3c\uc9c8\uc774\ub098 \ub77c\ubd80\uc544\uc9c0\uc5d0\ub294 \ucc98\uc74c\uc5d0 \uc774\ub97c \uc54c\uc9c0 \ubabb\ud558\uc600\ub2e4.) \ucd94\uac00\uc801\uc778 \uc5f0\uad6c\ub97c \ud1b5\ud574\uc11c \ub77c\ubd80\uc544\uc9c0\uc5d0\ub294 \uc124\ud0d5\uacfc principe oxygine\uc774 \ubc18\uc751\ud558\uba74 \uc625\uc0b4\uc0b0\uc774 \uc0dd\uc131\ub418\ub294 \ub4f1 \uc5ec\ub7ec \ubb3c\uc9c8\uacfc \ubc18\uc751\ud558\uc5ec \uc0b0\uc744 \uc0dd\uc131\ud55c\ub2e4\ub294 \uc0ac\uc2e4\uc744 \uc54c\uac8c \ub418\uc5c8\ub2e4. \ud604\ub300\uc758 \uc2dc\uac01\uc73c\ub85c \ubcf4\uc558\uc744 \ub54c \uc774 \uc774\ub860\uc740 \uc5fc\ud654 \uc218\uc18c \ub4f1 \uc0b0\uc18c\ub97c \ud3ec\ud568\ud558\uc9c0 \uc54a\uc740 \uc0b0\uc5d0 \uad00\ud574\uc11c\ub294 \ub4e4\uc5b4\ub9de\uc9c0 \uc54a\uc73c\ub098, \uc0b0\uc18c\uc0b0\uc744 \uc124\uba85\ud558\uae30 \uc801\ud569\ud558\ub2e4."} {"question": "\ud604\uc7ac\uae4c\uc9c0 \uc774\uc5b4\uc9c0\ub294 \ud1b5\uc77c\uc815\ucc45\uc758 \uae30\ubcf8\uc6d0\uce59\uc73c\ub85c \ub0a8\ubd81\uac04 \uacbd\uc9c1\ub41c \uad00\uacc4\ub97c \uac1c\uc120\ud558\uae30 \uc704\ud574 \ubc14\uafbc \uc815\ucc45\uc740?", "paragraph": "\ubbf8\uad6d \ub9ac\ucc98\ub4dc \ub2c9\uc2a8 \ub300\ud1b5\ub839\uc758 \ub2c9\uc2a8 \ub3c5\ud2b8\ub9b0 \uc815\ucc45\uc73c\ub85c \ubca0\ud2b8\ub0a8 \uc804\uc7c1\uc758 \ud3ec\uae30\uc640 \uc911\ud654\uc778\ubbfc\uacf5\ud654\uad6d(\uc911\uacf5)\uacfc\uc758 \uad00\uacc4\uac1c\uc120, \uc8fc\ud55c \ubbf8\uad70\uc758 \ubd80\ubd84\uc801 \ucca0\uc218\ubc0f \ub3d9\uc544\uc2dc\uc544\uc5d0\uc11c \ub0c9\uc804\uae30\ub958\uc758 \ud574\uccb4\uacbd\ud5a5\uc758 \uc601\ud5a5\uc744 \ubc1b\uc544 \ub0a8\ubd81\uac04\uc758 \uad00\uacc4\ub97c \ubaa8\uc0c9\ud558\uace0 \uacbd\uc81c\uc801 \ud604\uc2e4\uc744 \uace0\ub824\ud558\uc5ec '\uc120\uac74\uc124 \ud6c4\ud1b5\uc77c' \uc815\ucc45\uc5d0\uc11c \ud3c9\ud654\ud1b5\uc77c 3\uc6d0\uce59\uc744 \ud1b5\ud574 \ubd81\ud55c\uc758 \uc2e4\uccb4\ub97c \uc778\uc815\ud558\uace0, \ub0a8\ubd81\uac04 \uacbd\uc9c1\ub41c \uad00\uacc4\ub97c \uac1c\uc120\ud558\uae30 \uc704\ud574 '\uc120\ud3c9\ud654 \ud6c4\ud1b5\uc77c' \uc815\ucc45\uc73c\ub85c \ubc14\uafb8\uc5b4 \ud604\uc7ac\uae4c\uc9c0 \ud1b5\uc77c\uc815\ucc45\uc758 \uae30\ubcf8\uc6d0\uce59\uc774 \ub418\uace0 \uc788\ub2e4. 7\u00b74 \ub0a8\ubd81 \uacf5\ub3d9 \uc131\uba85 \ub0a8\ubd81 \uac04 \ud569\uc758\ubb38\uc11c\ub97c \ubc1c\ud45c\ud558\uc600\ub2e4. \uc774\ub97c \uacc4\uae30\ub85c \uad6d\ub0b4\uc678\uc801\uc778 \ud3c9\ud654 \ubd84\uc704\uae30\uac00 \uc870\uc131\ub418\uc5c8\uc9c0\ub9cc, \uace7 \ubc15\uc815\ud76c \uc815\ubd80\ub294 10\uc6d4 \uc720\uc2e0\uc744 \uc120\ud3ec\ud558\uc5ec \uc7a5\uae30 \uc9d1\uad8c\uc744 \uaf80\ud558\uc600\uace0, \uc870\uc120\ubbfc\uc8fc\uc8fc\uc758\uc778\ubbfc\uacf5\ud654\uad6d\ub3c4 \uc8fc\uccb4\uc0ac\uc0c1 \ud5cc\ubc95\uc744 \uac1c\uc815\ud558\uc5ec \uc720\uc77c \uc9c0\ub3c4 \uccb4\uc81c\ub97c \ub354\uc6b1 \uac15\ud654\ud558\uc600\ub2e4.", "answer": "\uc120\ud3c9\ud654 \ud6c4\ud1b5\uc77c", "sentence": "\ubbf8\uad6d \ub9ac\ucc98\ub4dc \ub2c9\uc2a8 \ub300\ud1b5\ub839\uc758 \ub2c9\uc2a8 \ub3c5\ud2b8\ub9b0 \uc815\ucc45\uc73c\ub85c \ubca0\ud2b8\ub0a8 \uc804\uc7c1\uc758 \ud3ec\uae30\uc640 \uc911\ud654\uc778\ubbfc\uacf5\ud654\uad6d(\uc911\uacf5)\uacfc\uc758 \uad00\uacc4\uac1c\uc120, \uc8fc\ud55c \ubbf8\uad70\uc758 \ubd80\ubd84\uc801 \ucca0\uc218\ubc0f \ub3d9\uc544\uc2dc\uc544\uc5d0\uc11c \ub0c9\uc804\uae30\ub958\uc758 \ud574\uccb4\uacbd\ud5a5\uc758 \uc601\ud5a5\uc744 \ubc1b\uc544 \ub0a8\ubd81\uac04\uc758 \uad00\uacc4\ub97c \ubaa8\uc0c9\ud558\uace0 \uacbd\uc81c\uc801 \ud604\uc2e4\uc744 \uace0\ub824\ud558\uc5ec '\uc120\uac74\uc124 \ud6c4\ud1b5\uc77c' \uc815\ucc45\uc5d0\uc11c \ud3c9\ud654\ud1b5\uc77c 3\uc6d0\uce59\uc744 \ud1b5\ud574 \ubd81\ud55c\uc758 \uc2e4\uccb4\ub97c \uc778\uc815\ud558\uace0, \ub0a8\ubd81\uac04 \uacbd\uc9c1\ub41c \uad00\uacc4\ub97c \uac1c\uc120\ud558\uae30 \uc704\ud574 ' \uc120\ud3c9\ud654 \ud6c4\ud1b5\uc77c '", "paragraph_sentence": " \ubbf8\uad6d \ub9ac\ucc98\ub4dc \ub2c9\uc2a8 \ub300\ud1b5\ub839\uc758 \ub2c9\uc2a8 \ub3c5\ud2b8\ub9b0 \uc815\ucc45\uc73c\ub85c \ubca0\ud2b8\ub0a8 \uc804\uc7c1\uc758 \ud3ec\uae30\uc640 \uc911\ud654\uc778\ubbfc\uacf5\ud654\uad6d(\uc911\uacf5)\uacfc\uc758 \uad00\uacc4\uac1c\uc120, \uc8fc\ud55c \ubbf8\uad70\uc758 \ubd80\ubd84\uc801 \ucca0\uc218\ubc0f \ub3d9\uc544\uc2dc\uc544\uc5d0\uc11c \ub0c9\uc804\uae30\ub958\uc758 \ud574\uccb4\uacbd\ud5a5\uc758 \uc601\ud5a5\uc744 \ubc1b\uc544 \ub0a8\ubd81\uac04\uc758 \uad00\uacc4\ub97c \ubaa8\uc0c9\ud558\uace0 \uacbd\uc81c\uc801 \ud604\uc2e4\uc744 \uace0\ub824\ud558\uc5ec '\uc120\uac74\uc124 \ud6c4\ud1b5\uc77c' \uc815\ucc45\uc5d0\uc11c \ud3c9\ud654\ud1b5\uc77c 3\uc6d0\uce59\uc744 \ud1b5\ud574 \ubd81\ud55c\uc758 \uc2e4\uccb4\ub97c \uc778\uc815\ud558\uace0, \ub0a8\ubd81\uac04 \uacbd\uc9c1\ub41c \uad00\uacc4\ub97c \uac1c\uc120\ud558\uae30 \uc704\ud574 ' \uc120\ud3c9\ud654 \ud6c4\ud1b5\uc77c ' \uc815\ucc45\uc73c\ub85c \ubc14\uafb8\uc5b4 \ud604\uc7ac\uae4c\uc9c0 \ud1b5\uc77c\uc815\ucc45\uc758 \uae30\ubcf8\uc6d0\uce59\uc774 \ub418\uace0 \uc788\ub2e4. 7\u00b74 \ub0a8\ubd81 \uacf5\ub3d9 \uc131\uba85 \ub0a8\ubd81 \uac04 \ud569\uc758\ubb38\uc11c\ub97c \ubc1c\ud45c\ud558\uc600\ub2e4. \uc774\ub97c \uacc4\uae30\ub85c \uad6d\ub0b4\uc678\uc801\uc778 \ud3c9\ud654 \ubd84\uc704\uae30\uac00 \uc870\uc131\ub418\uc5c8\uc9c0\ub9cc, \uace7 \ubc15\uc815\ud76c \uc815\ubd80\ub294 10\uc6d4 \uc720\uc2e0\uc744 \uc120\ud3ec\ud558\uc5ec \uc7a5\uae30 \uc9d1\uad8c\uc744 \uaf80\ud558\uc600\uace0, \uc870\uc120\ubbfc\uc8fc\uc8fc\uc758\uc778\ubbfc\uacf5\ud654\uad6d\ub3c4 \uc8fc\uccb4\uc0ac\uc0c1 \ud5cc\ubc95\uc744 \uac1c\uc815\ud558\uc5ec \uc720\uc77c \uc9c0\ub3c4 \uccb4\uc81c\ub97c \ub354\uc6b1 \uac15\ud654\ud558\uc600\ub2e4.", "paragraph_answer": "\ubbf8\uad6d \ub9ac\ucc98\ub4dc \ub2c9\uc2a8 \ub300\ud1b5\ub839\uc758 \ub2c9\uc2a8 \ub3c5\ud2b8\ub9b0 \uc815\ucc45\uc73c\ub85c \ubca0\ud2b8\ub0a8 \uc804\uc7c1\uc758 \ud3ec\uae30\uc640 \uc911\ud654\uc778\ubbfc\uacf5\ud654\uad6d(\uc911\uacf5)\uacfc\uc758 \uad00\uacc4\uac1c\uc120, \uc8fc\ud55c \ubbf8\uad70\uc758 \ubd80\ubd84\uc801 \ucca0\uc218\ubc0f \ub3d9\uc544\uc2dc\uc544\uc5d0\uc11c \ub0c9\uc804\uae30\ub958\uc758 \ud574\uccb4\uacbd\ud5a5\uc758 \uc601\ud5a5\uc744 \ubc1b\uc544 \ub0a8\ubd81\uac04\uc758 \uad00\uacc4\ub97c \ubaa8\uc0c9\ud558\uace0 \uacbd\uc81c\uc801 \ud604\uc2e4\uc744 \uace0\ub824\ud558\uc5ec '\uc120\uac74\uc124 \ud6c4\ud1b5\uc77c' \uc815\ucc45\uc5d0\uc11c \ud3c9\ud654\ud1b5\uc77c 3\uc6d0\uce59\uc744 \ud1b5\ud574 \ubd81\ud55c\uc758 \uc2e4\uccb4\ub97c \uc778\uc815\ud558\uace0, \ub0a8\ubd81\uac04 \uacbd\uc9c1\ub41c \uad00\uacc4\ub97c \uac1c\uc120\ud558\uae30 \uc704\ud574 ' \uc120\ud3c9\ud654 \ud6c4\ud1b5\uc77c ' \uc815\ucc45\uc73c\ub85c \ubc14\uafb8\uc5b4 \ud604\uc7ac\uae4c\uc9c0 \ud1b5\uc77c\uc815\ucc45\uc758 \uae30\ubcf8\uc6d0\uce59\uc774 \ub418\uace0 \uc788\ub2e4. 7\u00b74 \ub0a8\ubd81 \uacf5\ub3d9 \uc131\uba85 \ub0a8\ubd81 \uac04 \ud569\uc758\ubb38\uc11c\ub97c \ubc1c\ud45c\ud558\uc600\ub2e4. \uc774\ub97c \uacc4\uae30\ub85c \uad6d\ub0b4\uc678\uc801\uc778 \ud3c9\ud654 \ubd84\uc704\uae30\uac00 \uc870\uc131\ub418\uc5c8\uc9c0\ub9cc, \uace7 \ubc15\uc815\ud76c \uc815\ubd80\ub294 10\uc6d4 \uc720\uc2e0\uc744 \uc120\ud3ec\ud558\uc5ec \uc7a5\uae30 \uc9d1\uad8c\uc744 \uaf80\ud558\uc600\uace0, \uc870\uc120\ubbfc\uc8fc\uc8fc\uc758\uc778\ubbfc\uacf5\ud654\uad6d\ub3c4 \uc8fc\uccb4\uc0ac\uc0c1 \ud5cc\ubc95\uc744 \uac1c\uc815\ud558\uc5ec \uc720\uc77c \uc9c0\ub3c4 \uccb4\uc81c\ub97c \ub354\uc6b1 \uac15\ud654\ud558\uc600\ub2e4.", "sentence_answer": "\ubbf8\uad6d \ub9ac\ucc98\ub4dc \ub2c9\uc2a8 \ub300\ud1b5\ub839\uc758 \ub2c9\uc2a8 \ub3c5\ud2b8\ub9b0 \uc815\ucc45\uc73c\ub85c \ubca0\ud2b8\ub0a8 \uc804\uc7c1\uc758 \ud3ec\uae30\uc640 \uc911\ud654\uc778\ubbfc\uacf5\ud654\uad6d(\uc911\uacf5)\uacfc\uc758 \uad00\uacc4\uac1c\uc120, \uc8fc\ud55c \ubbf8\uad70\uc758 \ubd80\ubd84\uc801 \ucca0\uc218\ubc0f \ub3d9\uc544\uc2dc\uc544\uc5d0\uc11c \ub0c9\uc804\uae30\ub958\uc758 \ud574\uccb4\uacbd\ud5a5\uc758 \uc601\ud5a5\uc744 \ubc1b\uc544 \ub0a8\ubd81\uac04\uc758 \uad00\uacc4\ub97c \ubaa8\uc0c9\ud558\uace0 \uacbd\uc81c\uc801 \ud604\uc2e4\uc744 \uace0\ub824\ud558\uc5ec '\uc120\uac74\uc124 \ud6c4\ud1b5\uc77c' \uc815\ucc45\uc5d0\uc11c \ud3c9\ud654\ud1b5\uc77c 3\uc6d0\uce59\uc744 \ud1b5\ud574 \ubd81\ud55c\uc758 \uc2e4\uccb4\ub97c \uc778\uc815\ud558\uace0, \ub0a8\ubd81\uac04 \uacbd\uc9c1\ub41c \uad00\uacc4\ub97c \uac1c\uc120\ud558\uae30 \uc704\ud574 ' \uc120\ud3c9\ud654 \ud6c4\ud1b5\uc77c '", "paragraph_id": "6530241-42-0", "paragraph_question": "question: \ud604\uc7ac\uae4c\uc9c0 \uc774\uc5b4\uc9c0\ub294 \ud1b5\uc77c\uc815\ucc45\uc758 \uae30\ubcf8\uc6d0\uce59\uc73c\ub85c \ub0a8\ubd81\uac04 \uacbd\uc9c1\ub41c \uad00\uacc4\ub97c \uac1c\uc120\ud558\uae30 \uc704\ud574 \ubc14\uafbc \uc815\ucc45\uc740?, context: \ubbf8\uad6d \ub9ac\ucc98\ub4dc \ub2c9\uc2a8 \ub300\ud1b5\ub839\uc758 \ub2c9\uc2a8 \ub3c5\ud2b8\ub9b0 \uc815\ucc45\uc73c\ub85c \ubca0\ud2b8\ub0a8 \uc804\uc7c1\uc758 \ud3ec\uae30\uc640 \uc911\ud654\uc778\ubbfc\uacf5\ud654\uad6d(\uc911\uacf5)\uacfc\uc758 \uad00\uacc4\uac1c\uc120, \uc8fc\ud55c \ubbf8\uad70\uc758 \ubd80\ubd84\uc801 \ucca0\uc218\ubc0f \ub3d9\uc544\uc2dc\uc544\uc5d0\uc11c \ub0c9\uc804\uae30\ub958\uc758 \ud574\uccb4\uacbd\ud5a5\uc758 \uc601\ud5a5\uc744 \ubc1b\uc544 \ub0a8\ubd81\uac04\uc758 \uad00\uacc4\ub97c \ubaa8\uc0c9\ud558\uace0 \uacbd\uc81c\uc801 \ud604\uc2e4\uc744 \uace0\ub824\ud558\uc5ec '\uc120\uac74\uc124 \ud6c4\ud1b5\uc77c' \uc815\ucc45\uc5d0\uc11c \ud3c9\ud654\ud1b5\uc77c 3\uc6d0\uce59\uc744 \ud1b5\ud574 \ubd81\ud55c\uc758 \uc2e4\uccb4\ub97c \uc778\uc815\ud558\uace0, \ub0a8\ubd81\uac04 \uacbd\uc9c1\ub41c \uad00\uacc4\ub97c \uac1c\uc120\ud558\uae30 \uc704\ud574 '\uc120\ud3c9\ud654 \ud6c4\ud1b5\uc77c' \uc815\ucc45\uc73c\ub85c \ubc14\uafb8\uc5b4 \ud604\uc7ac\uae4c\uc9c0 \ud1b5\uc77c\uc815\ucc45\uc758 \uae30\ubcf8\uc6d0\uce59\uc774 \ub418\uace0 \uc788\ub2e4. 7\u00b74 \ub0a8\ubd81 \uacf5\ub3d9 \uc131\uba85 \ub0a8\ubd81 \uac04 \ud569\uc758\ubb38\uc11c\ub97c \ubc1c\ud45c\ud558\uc600\ub2e4. \uc774\ub97c \uacc4\uae30\ub85c \uad6d\ub0b4\uc678\uc801\uc778 \ud3c9\ud654 \ubd84\uc704\uae30\uac00 \uc870\uc131\ub418\uc5c8\uc9c0\ub9cc, \uace7 \ubc15\uc815\ud76c \uc815\ubd80\ub294 10\uc6d4 \uc720\uc2e0\uc744 \uc120\ud3ec\ud558\uc5ec \uc7a5\uae30 \uc9d1\uad8c\uc744 \uaf80\ud558\uc600\uace0, \uc870\uc120\ubbfc\uc8fc\uc8fc\uc758\uc778\ubbfc\uacf5\ud654\uad6d\ub3c4 \uc8fc\uccb4\uc0ac\uc0c1 \ud5cc\ubc95\uc744 \uac1c\uc815\ud558\uc5ec \uc720\uc77c \uc9c0\ub3c4 \uccb4\uc81c\ub97c \ub354\uc6b1 \uac15\ud654\ud558\uc600\ub2e4."} {"question": "1729\ub144 \uc124\uce58\ub418\uc5c8\uc73c\ub098 \uace7 \uad70\uae30\ucc98\ub85c \ud655\ub300, \uac1c\ud3b8\ub41c \uacf3\uc740 \uc5b4\ub514\uc778\uac00?", "paragraph": "\uc548\uc73c\ub85c\ub294 \uc774\ubbf8 \ubd80\ud669\uc778 \uac15\ud76c\uc81c\uac00 \uc57d\ud654\uc2dc\ucf30\ub358 \uc758\uc815\uc655\ub300\uc2e0\ud68c\uc758(\u8b70\u653f\u738b\u5927\u81e3\u6703\u8b70)\ub97c \ub354\uc6b1 \uc57d\ud654\uc2dc\ud0a4\uace0 1729\ub144(\uc639\uc815 7\ub144) \uad70\uc218\ubc29(\u8ecd\u9700\u623f)\uc744 \uc124\uce58\ud558\uc600\uc73c\ub098 \uace7 \uad70\uae30\ucc98\ub85c \ud655\ub300\u00b7\uac1c\ud3b8\ud558\uc600\ub2e4. \ucc98\uc74c\uc5d0\ub294 \ub300\uc2e0\ub4e4\uc774 \uad50\ub300\ub85c \uad70\uc0ac\uc5d0 \uad00\ud55c \uae30\ubc00\uc758 \uc0ac\ubb34\ub97c \ub3c4\ub9e1\uc544 \ubcf4\ub294 \uacf3\uc774\uc5c8\uc73c\ub098 \uc810\ucc28 \ub300\uc2e0\ub4e4\uc774 \uad50\ub300\ub85c \uac01\uc9c0\uc5d0\uc11c \ub4e4\uc5b4\uc624\ub294 \uc815\ubcf4\ub97c \uc218\uc9d1\ud558\ub294 \uacf3\uc73c\ub85c \ubcc0\ubaa8\ud558\uc5ec \uc815\ubcf4\ub97c \uc2dc\uc2dc\uac01\uac01 \ud669\uc81c\uc5d0\uac8c \ubcf4\uace0\ud558\uac8c \ud558\uc600\uace0 \uc639\uc815\uc81c\ub294 \ubc1b\uc740 \uc815\ubcf4\uc640 \uc774\uc5d0 \ub300\ud55c \ub2f5\uc744 \ub2e4\uc2dc \uc721\ubd80\ub85c \ubcf4\ub0b4\uc5b4 \uc774\ub97c \ucc98\ub9ac\ud558\uac8c \ud558\uc600\ub2e4. \ubcf8\ub798 \ub300\uc2e0\ub4e4\uc774 \uc815\uc0ac\ub97c \ub17c\uc758\ud558\ub294 \uacf3\uc740 \uc0c1\uc11c\ubc29\uc73c\ub85c \ud0dc\ud654\uc804 \uadfc\ucc98\uc5d0 \uc788\uc5c8\uc73c\ub098 \uc639\uc815\uc81c\ub294 \uad70\uae30\ucc98\ub97c \uc790\uc2e0\uc758 \uce68\uc804\uc778 \uc591\uc2ec\uc804 \uadfc\ucc98\uc5d0 \ub450\uc5b4 \ub300\uc2e0\ub4e4\uc744 \uc2dc\uc2dc\uac01\uac01 \uac10\uc2dc\ud558\uc5ec \ud55c \ub300\uc2e0\uc774 \uc721\ubd80\ub97c \uc870\uc885\ud558\ub294 \uac15\ub825\ud55c \uc2e0\uad8c\uc744 \ucc28\ub2e8\ud558\uace0 \uc2e0\ud558\ub4e4\uc774 \uc790\uc2e0\uc758 \uba85\ub839\uc5d0 \ub530\ub77c \ubcf5\uc885\ud558\uae30\ub97c \uc6d0\ud558\uc600\ub2e4.", "answer": "\uad70\uc218\ubc29", "sentence": "\uc548\uc73c\ub85c\ub294 \uc774\ubbf8 \ubd80\ud669\uc778 \uac15\ud76c\uc81c\uac00 \uc57d\ud654\uc2dc\ucf30\ub358 \uc758\uc815\uc655\ub300\uc2e0\ud68c\uc758(\u8b70\u653f\u738b\u5927\u81e3\u6703\u8b70)\ub97c \ub354\uc6b1 \uc57d\ud654\uc2dc\ud0a4\uace0 1729\ub144(\uc639\uc815 7\ub144) \uad70\uc218\ubc29 (\u8ecd\u9700\u623f)", "paragraph_sentence": " \uc548\uc73c\ub85c\ub294 \uc774\ubbf8 \ubd80\ud669\uc778 \uac15\ud76c\uc81c\uac00 \uc57d\ud654\uc2dc\ucf30\ub358 \uc758\uc815\uc655\ub300\uc2e0\ud68c\uc758(\u8b70\u653f\u738b\u5927\u81e3\u6703\u8b70)\ub97c \ub354\uc6b1 \uc57d\ud654\uc2dc\ud0a4\uace0 1729\ub144(\uc639\uc815 7\ub144) \uad70\uc218\ubc29 (\u8ecd\u9700\u623f) \uc744 \uc124\uce58\ud558\uc600\uc73c\ub098 \uace7 \uad70\uae30\ucc98\ub85c \ud655\ub300\u00b7\uac1c\ud3b8\ud558\uc600\ub2e4. \ucc98\uc74c\uc5d0\ub294 \ub300\uc2e0\ub4e4\uc774 \uad50\ub300\ub85c \uad70\uc0ac\uc5d0 \uad00\ud55c \uae30\ubc00\uc758 \uc0ac\ubb34\ub97c \ub3c4\ub9e1\uc544 \ubcf4\ub294 \uacf3\uc774\uc5c8\uc73c\ub098 \uc810\ucc28 \ub300\uc2e0\ub4e4\uc774 \uad50\ub300\ub85c \uac01\uc9c0\uc5d0\uc11c \ub4e4\uc5b4\uc624\ub294 \uc815\ubcf4\ub97c \uc218\uc9d1\ud558\ub294 \uacf3\uc73c\ub85c \ubcc0\ubaa8\ud558\uc5ec \uc815\ubcf4\ub97c \uc2dc\uc2dc\uac01\uac01 \ud669\uc81c\uc5d0\uac8c \ubcf4\uace0\ud558\uac8c \ud558\uc600\uace0 \uc639\uc815\uc81c\ub294 \ubc1b\uc740 \uc815\ubcf4\uc640 \uc774\uc5d0 \ub300\ud55c \ub2f5\uc744 \ub2e4\uc2dc \uc721\ubd80\ub85c \ubcf4\ub0b4\uc5b4 \uc774\ub97c \ucc98\ub9ac\ud558\uac8c \ud558\uc600\ub2e4. \ubcf8\ub798 \ub300\uc2e0\ub4e4\uc774 \uc815\uc0ac\ub97c \ub17c\uc758\ud558\ub294 \uacf3\uc740 \uc0c1\uc11c\ubc29\uc73c\ub85c \ud0dc\ud654\uc804 \uadfc\ucc98\uc5d0 \uc788\uc5c8\uc73c\ub098 \uc639\uc815\uc81c\ub294 \uad70\uae30\ucc98\ub97c \uc790\uc2e0\uc758 \uce68\uc804\uc778 \uc591\uc2ec\uc804 \uadfc\ucc98\uc5d0 \ub450\uc5b4 \ub300\uc2e0\ub4e4\uc744 \uc2dc\uc2dc\uac01\uac01 \uac10\uc2dc\ud558\uc5ec \ud55c \ub300\uc2e0\uc774 \uc721\ubd80\ub97c \uc870\uc885\ud558\ub294 \uac15\ub825\ud55c \uc2e0\uad8c\uc744 \ucc28\ub2e8\ud558\uace0 \uc2e0\ud558\ub4e4\uc774 \uc790\uc2e0\uc758 \uba85\ub839\uc5d0 \ub530\ub77c \ubcf5\uc885\ud558\uae30\ub97c \uc6d0\ud558\uc600\ub2e4.", "paragraph_answer": "\uc548\uc73c\ub85c\ub294 \uc774\ubbf8 \ubd80\ud669\uc778 \uac15\ud76c\uc81c\uac00 \uc57d\ud654\uc2dc\ucf30\ub358 \uc758\uc815\uc655\ub300\uc2e0\ud68c\uc758(\u8b70\u653f\u738b\u5927\u81e3\u6703\u8b70)\ub97c \ub354\uc6b1 \uc57d\ud654\uc2dc\ud0a4\uace0 1729\ub144(\uc639\uc815 7\ub144) \uad70\uc218\ubc29 (\u8ecd\u9700\u623f)\uc744 \uc124\uce58\ud558\uc600\uc73c\ub098 \uace7 \uad70\uae30\ucc98\ub85c \ud655\ub300\u00b7\uac1c\ud3b8\ud558\uc600\ub2e4. \ucc98\uc74c\uc5d0\ub294 \ub300\uc2e0\ub4e4\uc774 \uad50\ub300\ub85c \uad70\uc0ac\uc5d0 \uad00\ud55c \uae30\ubc00\uc758 \uc0ac\ubb34\ub97c \ub3c4\ub9e1\uc544 \ubcf4\ub294 \uacf3\uc774\uc5c8\uc73c\ub098 \uc810\ucc28 \ub300\uc2e0\ub4e4\uc774 \uad50\ub300\ub85c \uac01\uc9c0\uc5d0\uc11c \ub4e4\uc5b4\uc624\ub294 \uc815\ubcf4\ub97c \uc218\uc9d1\ud558\ub294 \uacf3\uc73c\ub85c \ubcc0\ubaa8\ud558\uc5ec \uc815\ubcf4\ub97c \uc2dc\uc2dc\uac01\uac01 \ud669\uc81c\uc5d0\uac8c \ubcf4\uace0\ud558\uac8c \ud558\uc600\uace0 \uc639\uc815\uc81c\ub294 \ubc1b\uc740 \uc815\ubcf4\uc640 \uc774\uc5d0 \ub300\ud55c \ub2f5\uc744 \ub2e4\uc2dc \uc721\ubd80\ub85c \ubcf4\ub0b4\uc5b4 \uc774\ub97c \ucc98\ub9ac\ud558\uac8c \ud558\uc600\ub2e4. \ubcf8\ub798 \ub300\uc2e0\ub4e4\uc774 \uc815\uc0ac\ub97c \ub17c\uc758\ud558\ub294 \uacf3\uc740 \uc0c1\uc11c\ubc29\uc73c\ub85c \ud0dc\ud654\uc804 \uadfc\ucc98\uc5d0 \uc788\uc5c8\uc73c\ub098 \uc639\uc815\uc81c\ub294 \uad70\uae30\ucc98\ub97c \uc790\uc2e0\uc758 \uce68\uc804\uc778 \uc591\uc2ec\uc804 \uadfc\ucc98\uc5d0 \ub450\uc5b4 \ub300\uc2e0\ub4e4\uc744 \uc2dc\uc2dc\uac01\uac01 \uac10\uc2dc\ud558\uc5ec \ud55c \ub300\uc2e0\uc774 \uc721\ubd80\ub97c \uc870\uc885\ud558\ub294 \uac15\ub825\ud55c \uc2e0\uad8c\uc744 \ucc28\ub2e8\ud558\uace0 \uc2e0\ud558\ub4e4\uc774 \uc790\uc2e0\uc758 \uba85\ub839\uc5d0 \ub530\ub77c \ubcf5\uc885\ud558\uae30\ub97c \uc6d0\ud558\uc600\ub2e4.", "sentence_answer": "\uc548\uc73c\ub85c\ub294 \uc774\ubbf8 \ubd80\ud669\uc778 \uac15\ud76c\uc81c\uac00 \uc57d\ud654\uc2dc\ucf30\ub358 \uc758\uc815\uc655\ub300\uc2e0\ud68c\uc758(\u8b70\u653f\u738b\u5927\u81e3\u6703\u8b70)\ub97c \ub354\uc6b1 \uc57d\ud654\uc2dc\ud0a4\uace0 1729\ub144(\uc639\uc815 7\ub144) \uad70\uc218\ubc29 (\u8ecd\u9700\u623f)", "paragraph_id": "6541455-18-0", "paragraph_question": "question: 1729\ub144 \uc124\uce58\ub418\uc5c8\uc73c\ub098 \uace7 \uad70\uae30\ucc98\ub85c \ud655\ub300, \uac1c\ud3b8\ub41c \uacf3\uc740 \uc5b4\ub514\uc778\uac00?, context: \uc548\uc73c\ub85c\ub294 \uc774\ubbf8 \ubd80\ud669\uc778 \uac15\ud76c\uc81c\uac00 \uc57d\ud654\uc2dc\ucf30\ub358 \uc758\uc815\uc655\ub300\uc2e0\ud68c\uc758(\u8b70\u653f\u738b\u5927\u81e3\u6703\u8b70)\ub97c \ub354\uc6b1 \uc57d\ud654\uc2dc\ud0a4\uace0 1729\ub144(\uc639\uc815 7\ub144) \uad70\uc218\ubc29(\u8ecd\u9700\u623f)\uc744 \uc124\uce58\ud558\uc600\uc73c\ub098 \uace7 \uad70\uae30\ucc98\ub85c \ud655\ub300\u00b7\uac1c\ud3b8\ud558\uc600\ub2e4. \ucc98\uc74c\uc5d0\ub294 \ub300\uc2e0\ub4e4\uc774 \uad50\ub300\ub85c \uad70\uc0ac\uc5d0 \uad00\ud55c \uae30\ubc00\uc758 \uc0ac\ubb34\ub97c \ub3c4\ub9e1\uc544 \ubcf4\ub294 \uacf3\uc774\uc5c8\uc73c\ub098 \uc810\ucc28 \ub300\uc2e0\ub4e4\uc774 \uad50\ub300\ub85c \uac01\uc9c0\uc5d0\uc11c \ub4e4\uc5b4\uc624\ub294 \uc815\ubcf4\ub97c \uc218\uc9d1\ud558\ub294 \uacf3\uc73c\ub85c \ubcc0\ubaa8\ud558\uc5ec \uc815\ubcf4\ub97c \uc2dc\uc2dc\uac01\uac01 \ud669\uc81c\uc5d0\uac8c \ubcf4\uace0\ud558\uac8c \ud558\uc600\uace0 \uc639\uc815\uc81c\ub294 \ubc1b\uc740 \uc815\ubcf4\uc640 \uc774\uc5d0 \ub300\ud55c \ub2f5\uc744 \ub2e4\uc2dc \uc721\ubd80\ub85c \ubcf4\ub0b4\uc5b4 \uc774\ub97c \ucc98\ub9ac\ud558\uac8c \ud558\uc600\ub2e4. \ubcf8\ub798 \ub300\uc2e0\ub4e4\uc774 \uc815\uc0ac\ub97c \ub17c\uc758\ud558\ub294 \uacf3\uc740 \uc0c1\uc11c\ubc29\uc73c\ub85c \ud0dc\ud654\uc804 \uadfc\ucc98\uc5d0 \uc788\uc5c8\uc73c\ub098 \uc639\uc815\uc81c\ub294 \uad70\uae30\ucc98\ub97c \uc790\uc2e0\uc758 \uce68\uc804\uc778 \uc591\uc2ec\uc804 \uadfc\ucc98\uc5d0 \ub450\uc5b4 \ub300\uc2e0\ub4e4\uc744 \uc2dc\uc2dc\uac01\uac01 \uac10\uc2dc\ud558\uc5ec \ud55c \ub300\uc2e0\uc774 \uc721\ubd80\ub97c \uc870\uc885\ud558\ub294 \uac15\ub825\ud55c \uc2e0\uad8c\uc744 \ucc28\ub2e8\ud558\uace0 \uc2e0\ud558\ub4e4\uc774 \uc790\uc2e0\uc758 \uba85\ub839\uc5d0 \ub530\ub77c \ubcf5\uc885\ud558\uae30\ub97c \uc6d0\ud558\uc600\ub2e4."} {"question": "\uad6d\ubbfc\uc758\ud68c\ub294 \ubb34\uc2a8 \uacc4\uae09\uc758 \uc758\uc6d0\uc774 \ub300\ub2e4\uc218\ub97c \ucc28\uc9c0\ud588\ub294\uac00?", "paragraph": "\uad50\uc721\ubc1b\uc740 \ubd80\ub974\uc8fc\uc544 \uacc4\uae09\uc774 \uc758\uc6d0\uc758 \ub300\ub2e4\uc218\ub97c \ucc28\uc9c0\ud588\uae30 \ub54c\ubb38\uc5d0 \uad6d\ubbfc\uc758\ud68c\uc5d0\uc11c \ud5e4\uac8c\ubaa8\ub2c8\ub97c \uc7a5\uc545\ud55c \uc774\ub150 \uc5ed\uc2dc \uad50\uc591\uc2dc\ubbfc\uc758 \uc774\ub150\uc774\uc5c8\ub358 \uc911\ub3c4 \uc6b0\ud30c \uc790\uc720\uc8fc\uc758\uc600\ub2e4. \uc758\uc6d0 \uc911 \uc5ec\ub7ff\uc740 \uc774\ubbf8 \uc5ec\ub7ec \ud574 \ub3d9\uc548 \uadf8\ub4e4\uc758 \uad70\uc8fc\ub4e4\uacfc \ub300\ub9bd\uac01\uc744 \uc138\uc6b0\uace0 \uc788\uc5c8\uc73c\uba70, \uc774\ub4e4 \uc911\uc5d0\uc11c \uac00\uc7a5 \uc720\uba85\ud55c \uc0ac\ub78c\ub4e4\ub85c\ub294 \uad34\ud305\uac90 7\uad50\uc218\uc5d0 \ud3ec\ud568\ub418\ub294 \uad50\uc218\ub4e4\uc778 \uc57c\ucf54\ud504 \uadf8\ub9bc, \ud504\ub9ac\ub4dc\ub9ac\ud788 \ud06c\ub9ac\uc2a4\ud1a0\ud504 \ub2ec\ub9cc, \uac8c\uc624\ub974\ud06c \uace0\ud2b8\ud504\ub9ac\ud2b8 \uac8c\ub974\ube44\ub204\uc2a4, \ube4c\ud5ec\ub984 \uc5d0\ub450\uc544\ub974\ud2b8 \uc54c\ube0c\ub808\ud788\ud2b8 \ubc0f \uc774\ubbf8 20\ub144 \uc774\uc0c1 \ud5cc\ubc95\uc801 \uad8c\ub9ac\ub97c \uc704\ud574 \uc2f8\uc6e0\ub358 \uc120\uad6c\uc790\ub4e4\uc778 \ubca8\ucee4(Welcker)\uc640 \uc774\uce20\uc288\ud0c0\uc778(Johann Adam von Itzstein) \uac19\uc740 \uc815\uce58\uac00\ub4e4\uc774 \uc788\uc5c8\ub2e4. \ub300\ud559\uad50\uc218\ub4e4 \uc911\uc5d0\ub294 \ubc95\ud559\uc790 \uc678\uc5d0\ub3c4 \ub3c5\uc77c\ud559\uc790\ub4e4\uacfc \uc0ac\ud559\uc790\ub4e4\uc758 \ube44\uc911\uc774 \ud2b9\ud788 \ucef8\ub294\ub370, \uc774\ub294 \uad6c\uccb4\uc81c\ub85c\uc758 \ubc18\ub3d9\uae30\uc600\ub358 \uc2dc\ub300\uc801 \ubc30\uacbd\uc5d0\uc11c\ub294 1846\ub144\uacfc 1847\ub144\uc758 \ub3c5\uc77c\ud559\uc790\ub0a0(Germanisten-Tage)\uacfc \uac19\uc740, \uc774\ub7ec\ud55c \uc6d0\uce59\ub4e4\uc5d0 \ub300\ud55c \uc804\ubb38 \ud559\uc220\ud68c\uc758\uac00 \ubbfc\uc871\uad6d\uac00\ub97c \uc8fc\uc81c\ub85c \uc790\uc720\ub86d\uac8c \ud1a0\ub860\ud560 \uc218 \uc788\ub294 \uc720\uc77c\ud55c \uc7a5\uc18c\uc600\uae30 \ub54c\ubb38\uc774\ub2e4. \uc774\ubbf8 \uc5b8\uae09\ud55c \uad50\uc218\ub4e4 \uc678\uc5d0\ub3c4 \uc5ec\uae30\uc5d0\ub294 \ud2b9\ud788 \uc5d0\ub978\uc2a4\ud2b8 \ubaa8\ub9ac\uce20 \uc544\ub978\ud2b8(Ernst Moritz Arndt), \uc694\ud55c \uad6c\uc2a4\ud0c0\ud504 \ub4dc\ub85c\uc774\uc820(Johann Gustav Droysen), \ud558\uc778\ub9ac\ud788 \uce74\ub97c \uc57c\uc6b0\ud504(Heinrich Karl Jaup), \ud504\ub9ac\ub4dc\ub9ac\ud788 \ud14c\uc624\ub3c4\ub974 \ube44\uc154(Friedrich Theodor Vischer), \uac8c\uc624\ub974\ud06c \ubc14\uc774\uce20(Georg Waitz)\uac00 \ud3ec\ud568\ub418\uc5c8\ub2e4.", "answer": "\ubd80\ub974\uc8fc\uc544 \uacc4\uae09", "sentence": "\uad50\uc721\ubc1b\uc740 \ubd80\ub974\uc8fc\uc544 \uacc4\uae09 \uc774 \uc758\uc6d0\uc758 \ub300\ub2e4\uc218\ub97c \ucc28\uc9c0\ud588\uae30 \ub54c\ubb38\uc5d0 \uad6d\ubbfc\uc758\ud68c\uc5d0\uc11c \ud5e4\uac8c\ubaa8\ub2c8\ub97c \uc7a5\uc545\ud55c \uc774\ub150 \uc5ed\uc2dc \uad50\uc591\uc2dc\ubbfc\uc758 \uc774\ub150\uc774\uc5c8\ub358 \uc911\ub3c4 \uc6b0\ud30c \uc790\uc720\uc8fc\uc758\uc600\ub2e4.", "paragraph_sentence": " \uad50\uc721\ubc1b\uc740 \ubd80\ub974\uc8fc\uc544 \uacc4\uae09 \uc774 \uc758\uc6d0\uc758 \ub300\ub2e4\uc218\ub97c \ucc28\uc9c0\ud588\uae30 \ub54c\ubb38\uc5d0 \uad6d\ubbfc\uc758\ud68c\uc5d0\uc11c \ud5e4\uac8c\ubaa8\ub2c8\ub97c \uc7a5\uc545\ud55c \uc774\ub150 \uc5ed\uc2dc \uad50\uc591\uc2dc\ubbfc\uc758 \uc774\ub150\uc774\uc5c8\ub358 \uc911\ub3c4 \uc6b0\ud30c \uc790\uc720\uc8fc\uc758\uc600\ub2e4. \uc758\uc6d0 \uc911 \uc5ec\ub7ff\uc740 \uc774\ubbf8 \uc5ec\ub7ec \ud574 \ub3d9\uc548 \uadf8\ub4e4\uc758 \uad70\uc8fc\ub4e4\uacfc \ub300\ub9bd\uac01\uc744 \uc138\uc6b0\uace0 \uc788\uc5c8\uc73c\uba70, \uc774\ub4e4 \uc911\uc5d0\uc11c \uac00\uc7a5 \uc720\uba85\ud55c \uc0ac\ub78c\ub4e4\ub85c\ub294 \uad34\ud305\uac90 7\uad50\uc218\uc5d0 \ud3ec\ud568\ub418\ub294 \uad50\uc218\ub4e4\uc778 \uc57c\ucf54\ud504 \uadf8\ub9bc, \ud504\ub9ac\ub4dc\ub9ac\ud788 \ud06c\ub9ac\uc2a4\ud1a0\ud504 \ub2ec\ub9cc, \uac8c\uc624\ub974\ud06c \uace0\ud2b8\ud504\ub9ac\ud2b8 \uac8c\ub974\ube44\ub204\uc2a4, \ube4c\ud5ec\ub984 \uc5d0\ub450\uc544\ub974\ud2b8 \uc54c\ube0c\ub808\ud788\ud2b8 \ubc0f \uc774\ubbf8 20\ub144 \uc774\uc0c1 \ud5cc\ubc95\uc801 \uad8c\ub9ac\ub97c \uc704\ud574 \uc2f8\uc6e0\ub358 \uc120\uad6c\uc790\ub4e4\uc778 \ubca8\ucee4(Welcker)\uc640 \uc774\uce20\uc288\ud0c0\uc778(Johann Adam von Itzstein) \uac19\uc740 \uc815\uce58\uac00\ub4e4\uc774 \uc788\uc5c8\ub2e4. \ub300\ud559\uad50\uc218\ub4e4 \uc911\uc5d0\ub294 \ubc95\ud559\uc790 \uc678\uc5d0\ub3c4 \ub3c5\uc77c\ud559\uc790\ub4e4\uacfc \uc0ac\ud559\uc790\ub4e4\uc758 \ube44\uc911\uc774 \ud2b9\ud788 \ucef8\ub294\ub370, \uc774\ub294 \uad6c\uccb4\uc81c\ub85c\uc758 \ubc18\ub3d9\uae30\uc600\ub358 \uc2dc\ub300\uc801 \ubc30\uacbd\uc5d0\uc11c\ub294 1846\ub144\uacfc 1847\ub144\uc758 \ub3c5\uc77c\ud559\uc790\ub0a0(Germanisten-Tage)\uacfc \uac19\uc740, \uc774\ub7ec\ud55c \uc6d0\uce59\ub4e4\uc5d0 \ub300\ud55c \uc804\ubb38 \ud559\uc220\ud68c\uc758\uac00 \ubbfc\uc871\uad6d\uac00\ub97c \uc8fc\uc81c\ub85c \uc790\uc720\ub86d\uac8c \ud1a0\ub860\ud560 \uc218 \uc788\ub294 \uc720\uc77c\ud55c \uc7a5\uc18c\uc600\uae30 \ub54c\ubb38\uc774\ub2e4. \uc774\ubbf8 \uc5b8\uae09\ud55c \uad50\uc218\ub4e4 \uc678\uc5d0\ub3c4 \uc5ec\uae30\uc5d0\ub294 \ud2b9\ud788 \uc5d0\ub978\uc2a4\ud2b8 \ubaa8\ub9ac\uce20 \uc544\ub978\ud2b8(Ernst Moritz Arndt), \uc694\ud55c \uad6c\uc2a4\ud0c0\ud504 \ub4dc\ub85c\uc774\uc820(Johann Gustav Droysen), \ud558\uc778\ub9ac\ud788 \uce74\ub97c \uc57c\uc6b0\ud504(Heinrich Karl Jaup), \ud504\ub9ac\ub4dc\ub9ac\ud788 \ud14c\uc624\ub3c4\ub974 \ube44\uc154(Friedrich Theodor Vischer), \uac8c\uc624\ub974\ud06c \ubc14\uc774\uce20(Georg Waitz)\uac00 \ud3ec\ud568\ub418\uc5c8\ub2e4.", "paragraph_answer": "\uad50\uc721\ubc1b\uc740 \ubd80\ub974\uc8fc\uc544 \uacc4\uae09 \uc774 \uc758\uc6d0\uc758 \ub300\ub2e4\uc218\ub97c \ucc28\uc9c0\ud588\uae30 \ub54c\ubb38\uc5d0 \uad6d\ubbfc\uc758\ud68c\uc5d0\uc11c \ud5e4\uac8c\ubaa8\ub2c8\ub97c \uc7a5\uc545\ud55c \uc774\ub150 \uc5ed\uc2dc \uad50\uc591\uc2dc\ubbfc\uc758 \uc774\ub150\uc774\uc5c8\ub358 \uc911\ub3c4 \uc6b0\ud30c \uc790\uc720\uc8fc\uc758\uc600\ub2e4. \uc758\uc6d0 \uc911 \uc5ec\ub7ff\uc740 \uc774\ubbf8 \uc5ec\ub7ec \ud574 \ub3d9\uc548 \uadf8\ub4e4\uc758 \uad70\uc8fc\ub4e4\uacfc \ub300\ub9bd\uac01\uc744 \uc138\uc6b0\uace0 \uc788\uc5c8\uc73c\uba70, \uc774\ub4e4 \uc911\uc5d0\uc11c \uac00\uc7a5 \uc720\uba85\ud55c \uc0ac\ub78c\ub4e4\ub85c\ub294 \uad34\ud305\uac90 7\uad50\uc218\uc5d0 \ud3ec\ud568\ub418\ub294 \uad50\uc218\ub4e4\uc778 \uc57c\ucf54\ud504 \uadf8\ub9bc, \ud504\ub9ac\ub4dc\ub9ac\ud788 \ud06c\ub9ac\uc2a4\ud1a0\ud504 \ub2ec\ub9cc, \uac8c\uc624\ub974\ud06c \uace0\ud2b8\ud504\ub9ac\ud2b8 \uac8c\ub974\ube44\ub204\uc2a4, \ube4c\ud5ec\ub984 \uc5d0\ub450\uc544\ub974\ud2b8 \uc54c\ube0c\ub808\ud788\ud2b8 \ubc0f \uc774\ubbf8 20\ub144 \uc774\uc0c1 \ud5cc\ubc95\uc801 \uad8c\ub9ac\ub97c \uc704\ud574 \uc2f8\uc6e0\ub358 \uc120\uad6c\uc790\ub4e4\uc778 \ubca8\ucee4(Welcker)\uc640 \uc774\uce20\uc288\ud0c0\uc778(Johann Adam von Itzstein) \uac19\uc740 \uc815\uce58\uac00\ub4e4\uc774 \uc788\uc5c8\ub2e4. \ub300\ud559\uad50\uc218\ub4e4 \uc911\uc5d0\ub294 \ubc95\ud559\uc790 \uc678\uc5d0\ub3c4 \ub3c5\uc77c\ud559\uc790\ub4e4\uacfc \uc0ac\ud559\uc790\ub4e4\uc758 \ube44\uc911\uc774 \ud2b9\ud788 \ucef8\ub294\ub370, \uc774\ub294 \uad6c\uccb4\uc81c\ub85c\uc758 \ubc18\ub3d9\uae30\uc600\ub358 \uc2dc\ub300\uc801 \ubc30\uacbd\uc5d0\uc11c\ub294 1846\ub144\uacfc 1847\ub144\uc758 \ub3c5\uc77c\ud559\uc790\ub0a0(Germanisten-Tage)\uacfc \uac19\uc740, \uc774\ub7ec\ud55c \uc6d0\uce59\ub4e4\uc5d0 \ub300\ud55c \uc804\ubb38 \ud559\uc220\ud68c\uc758\uac00 \ubbfc\uc871\uad6d\uac00\ub97c \uc8fc\uc81c\ub85c \uc790\uc720\ub86d\uac8c \ud1a0\ub860\ud560 \uc218 \uc788\ub294 \uc720\uc77c\ud55c \uc7a5\uc18c\uc600\uae30 \ub54c\ubb38\uc774\ub2e4. \uc774\ubbf8 \uc5b8\uae09\ud55c \uad50\uc218\ub4e4 \uc678\uc5d0\ub3c4 \uc5ec\uae30\uc5d0\ub294 \ud2b9\ud788 \uc5d0\ub978\uc2a4\ud2b8 \ubaa8\ub9ac\uce20 \uc544\ub978\ud2b8(Ernst Moritz Arndt), \uc694\ud55c \uad6c\uc2a4\ud0c0\ud504 \ub4dc\ub85c\uc774\uc820(Johann Gustav Droysen), \ud558\uc778\ub9ac\ud788 \uce74\ub97c \uc57c\uc6b0\ud504(Heinrich Karl Jaup), \ud504\ub9ac\ub4dc\ub9ac\ud788 \ud14c\uc624\ub3c4\ub974 \ube44\uc154(Friedrich Theodor Vischer), \uac8c\uc624\ub974\ud06c \ubc14\uc774\uce20(Georg Waitz)\uac00 \ud3ec\ud568\ub418\uc5c8\ub2e4.", "sentence_answer": "\uad50\uc721\ubc1b\uc740 \ubd80\ub974\uc8fc\uc544 \uacc4\uae09 \uc774 \uc758\uc6d0\uc758 \ub300\ub2e4\uc218\ub97c \ucc28\uc9c0\ud588\uae30 \ub54c\ubb38\uc5d0 \uad6d\ubbfc\uc758\ud68c\uc5d0\uc11c \ud5e4\uac8c\ubaa8\ub2c8\ub97c \uc7a5\uc545\ud55c \uc774\ub150 \uc5ed\uc2dc \uad50\uc591\uc2dc\ubbfc\uc758 \uc774\ub150\uc774\uc5c8\ub358 \uc911\ub3c4 \uc6b0\ud30c \uc790\uc720\uc8fc\uc758\uc600\ub2e4.", "paragraph_id": "6568666-13-0", "paragraph_question": "question: \uad6d\ubbfc\uc758\ud68c\ub294 \ubb34\uc2a8 \uacc4\uae09\uc758 \uc758\uc6d0\uc774 \ub300\ub2e4\uc218\ub97c \ucc28\uc9c0\ud588\ub294\uac00?, context: \uad50\uc721\ubc1b\uc740 \ubd80\ub974\uc8fc\uc544 \uacc4\uae09\uc774 \uc758\uc6d0\uc758 \ub300\ub2e4\uc218\ub97c \ucc28\uc9c0\ud588\uae30 \ub54c\ubb38\uc5d0 \uad6d\ubbfc\uc758\ud68c\uc5d0\uc11c \ud5e4\uac8c\ubaa8\ub2c8\ub97c \uc7a5\uc545\ud55c \uc774\ub150 \uc5ed\uc2dc \uad50\uc591\uc2dc\ubbfc\uc758 \uc774\ub150\uc774\uc5c8\ub358 \uc911\ub3c4 \uc6b0\ud30c \uc790\uc720\uc8fc\uc758\uc600\ub2e4. \uc758\uc6d0 \uc911 \uc5ec\ub7ff\uc740 \uc774\ubbf8 \uc5ec\ub7ec \ud574 \ub3d9\uc548 \uadf8\ub4e4\uc758 \uad70\uc8fc\ub4e4\uacfc \ub300\ub9bd\uac01\uc744 \uc138\uc6b0\uace0 \uc788\uc5c8\uc73c\uba70, \uc774\ub4e4 \uc911\uc5d0\uc11c \uac00\uc7a5 \uc720\uba85\ud55c \uc0ac\ub78c\ub4e4\ub85c\ub294 \uad34\ud305\uac90 7\uad50\uc218\uc5d0 \ud3ec\ud568\ub418\ub294 \uad50\uc218\ub4e4\uc778 \uc57c\ucf54\ud504 \uadf8\ub9bc, \ud504\ub9ac\ub4dc\ub9ac\ud788 \ud06c\ub9ac\uc2a4\ud1a0\ud504 \ub2ec\ub9cc, \uac8c\uc624\ub974\ud06c \uace0\ud2b8\ud504\ub9ac\ud2b8 \uac8c\ub974\ube44\ub204\uc2a4, \ube4c\ud5ec\ub984 \uc5d0\ub450\uc544\ub974\ud2b8 \uc54c\ube0c\ub808\ud788\ud2b8 \ubc0f \uc774\ubbf8 20\ub144 \uc774\uc0c1 \ud5cc\ubc95\uc801 \uad8c\ub9ac\ub97c \uc704\ud574 \uc2f8\uc6e0\ub358 \uc120\uad6c\uc790\ub4e4\uc778 \ubca8\ucee4(Welcker)\uc640 \uc774\uce20\uc288\ud0c0\uc778(Johann Adam von Itzstein) \uac19\uc740 \uc815\uce58\uac00\ub4e4\uc774 \uc788\uc5c8\ub2e4. \ub300\ud559\uad50\uc218\ub4e4 \uc911\uc5d0\ub294 \ubc95\ud559\uc790 \uc678\uc5d0\ub3c4 \ub3c5\uc77c\ud559\uc790\ub4e4\uacfc \uc0ac\ud559\uc790\ub4e4\uc758 \ube44\uc911\uc774 \ud2b9\ud788 \ucef8\ub294\ub370, \uc774\ub294 \uad6c\uccb4\uc81c\ub85c\uc758 \ubc18\ub3d9\uae30\uc600\ub358 \uc2dc\ub300\uc801 \ubc30\uacbd\uc5d0\uc11c\ub294 1846\ub144\uacfc 1847\ub144\uc758 \ub3c5\uc77c\ud559\uc790\ub0a0(Germanisten-Tage)\uacfc \uac19\uc740, \uc774\ub7ec\ud55c \uc6d0\uce59\ub4e4\uc5d0 \ub300\ud55c \uc804\ubb38 \ud559\uc220\ud68c\uc758\uac00 \ubbfc\uc871\uad6d\uac00\ub97c \uc8fc\uc81c\ub85c \uc790\uc720\ub86d\uac8c \ud1a0\ub860\ud560 \uc218 \uc788\ub294 \uc720\uc77c\ud55c \uc7a5\uc18c\uc600\uae30 \ub54c\ubb38\uc774\ub2e4. \uc774\ubbf8 \uc5b8\uae09\ud55c \uad50\uc218\ub4e4 \uc678\uc5d0\ub3c4 \uc5ec\uae30\uc5d0\ub294 \ud2b9\ud788 \uc5d0\ub978\uc2a4\ud2b8 \ubaa8\ub9ac\uce20 \uc544\ub978\ud2b8(Ernst Moritz Arndt), \uc694\ud55c \uad6c\uc2a4\ud0c0\ud504 \ub4dc\ub85c\uc774\uc820(Johann Gustav Droysen), \ud558\uc778\ub9ac\ud788 \uce74\ub97c \uc57c\uc6b0\ud504(Heinrich Karl Jaup), \ud504\ub9ac\ub4dc\ub9ac\ud788 \ud14c\uc624\ub3c4\ub974 \ube44\uc154(Friedrich Theodor Vischer), \uac8c\uc624\ub974\ud06c \ubc14\uc774\uce20(Georg Waitz)\uac00 \ud3ec\ud568\ub418\uc5c8\ub2e4."} {"question": "2001\ub144 \uc560\uc2ac\ub808\ud2f1\uc2a4\uac00 \ub514\ube44\uc804 \uc2dc\ub9ac\uc988\uc5d0\uc11c \ub9cc\ub09c \ud300\uc740?", "paragraph": "2001\ub144 \uc2dc\uc98c \uc624\ud074\ub79c\ub4dc \uc560\uc2ac\ub808\ud2f1\uc2a4\ub294 102\uc2b9 60\ud328\ub97c \uac70\ub450\uc5c8\uc73c\ub098, \uac19\uc740 \uc9c0\uad6c\uc758 \uc2dc\uc560\ud2c0 \ub9e4\ub9ac\ub108\uc2a4\uac00 116\uc2b9\uc744 \uac70\ub450\uba74\uc11c \uc640\uc77c\ub4dc\uce74\ub4dc\ub85c \ud3ec\uc2a4\ud2b8\uc2dc\uc98c\uc5d0 \uc9c4\ucd9c\ud588\ub2e4. \uc624\ud074\ub79c\ub4dc\uac00 \uc18d\ud55c \uc9c0\uad6c \uc774\uc678\uc758 \ub2e4\ub978 \ub2e4\uc12f \uc9c0\uad6c \uc6b0\uc2b9\ud300\ub4e4 \uc911 \uc5d0\uc774\uc2a4\ubcf4\ub2e4 \ub9ce\uc740 \uc2b9\uc218\ub97c \uae30\ub85d\ud55c \ud300\uc740 \uc5c6\uc5c8\ub2e4. \uc560\uc2ac\ub808\ud2f1\uc2a4\ub294 \ub514\ube44\uc804 \uc2dc\ub9ac\uc988\uc5d0\uc11c \uc591\ud0a4\uc2a4\uc640 \uc7ac\ub300\uacb0\uc744 \ud588\ub2e4. \ub871\uc740 1\ucc28\uc804\uc5d0\uc11c \ub450 \uac1c\uc758 \ud648\ub7f0\uc744 \ud130\ub728\ub9ac\uba70 \ud300\uc758 5-3 \uc2b9\ub9ac\ub97c \uc774\ub04c\uc5c8\ub2e4. \uc774\uc740 3\ucc28\uc804\uc5d0\uc11c\ub294 \ud300\uc774 1-0\uc73c\ub85c \uc9c0\uace0 \uc788\ub294 \uc0c1\ud669\uc5d0\uc11c, 2\uc544\uc6c3 1\ub8e8 \uc8fc\uc790 \uc81c\ub808\ubbf8 \uc9c0\uc554\ube44\uac00 \ucd9c\ub8e8\ud55c \uac00\uc6b4\ub370 \ud0c0\uc11d\uc5d0 \ub4e4\uc5b4\uc11c \uc678\uc57c \uc6b0\uce21\uc73c\ub85c \ud0c0\uad6c\ub97c \ub0a0\ub824\ubcf4\ub0c8\ub2e4. \ud558\uc9c0\ub9cc \uc774 \ub54c 3\ub8e8\ub97c \ub3cc\uace0 \ud648\uc73c\ub85c \ub4e4\uc5b4\uc624\ub358 \uc9c0\uc554\ube44\uac00 \ub370\ub9ad \uc9c0\ud130\uc758 \uc720\uba85\ud55c '\ub354 \ud50c\ub9bd'(The Flip) \ud50c\ub808\uc774\ub85c \uc544\uc6c3\uc744 \ub2f9\ud588\uace0, \uc5d0\uc774\uc2a4\ub294 \uadf8 \uc810\uc218 \uadf8\ub300\ub85c \ud328\ubc30\ud588\ub2e4. 5\ucc28\uc804 \uc2dc\ub9ac\uc988\uc5d0\uc11c 2\uc5f0\uc2b9 \ub4a4 3\uc5f0\ud328\ub97c \ub2f9\ud558\uba70 \uc5d0\uc774\uc2a4\ub294 2\ub144 \uc5f0\uc18d \ucc54\ud53c\uc5b8\uc2ed \uc2dc\ub9ac\uc988 \uc9c4\ucd9c\uc5d0 \uc2e4\ud328\ud588\ub2e4. \ub871\uc740 \uc2dc\ub9ac\uc988 5\uacbd\uae30, 7\uc548\ud0c0, 2\ud648\ub7f0, 3\ud0c0\uc810, \ud0c0\uc728 .389\ub97c \uae30\ub85d\ud588\ub2e4.", "answer": "\uc591\ud0a4\uc2a4", "sentence": "\uc560\uc2ac\ub808\ud2f1\uc2a4\ub294 \ub514\ube44\uc804 \uc2dc\ub9ac\uc988\uc5d0\uc11c \uc591\ud0a4\uc2a4 \uc640 \uc7ac\ub300\uacb0\uc744 \ud588\ub2e4.", "paragraph_sentence": "2001\ub144 \uc2dc\uc98c \uc624\ud074\ub79c\ub4dc \uc560\uc2ac\ub808\ud2f1\uc2a4\ub294 102\uc2b9 60\ud328\ub97c \uac70\ub450\uc5c8\uc73c\ub098, \uac19\uc740 \uc9c0\uad6c\uc758 \uc2dc\uc560\ud2c0 \ub9e4\ub9ac\ub108\uc2a4\uac00 116\uc2b9\uc744 \uac70\ub450\uba74\uc11c \uc640\uc77c\ub4dc\uce74\ub4dc\ub85c \ud3ec\uc2a4\ud2b8\uc2dc\uc98c\uc5d0 \uc9c4\ucd9c\ud588\ub2e4. \uc624\ud074\ub79c\ub4dc\uac00 \uc18d\ud55c \uc9c0\uad6c \uc774\uc678\uc758 \ub2e4\ub978 \ub2e4\uc12f \uc9c0\uad6c \uc6b0\uc2b9\ud300\ub4e4 \uc911 \uc5d0\uc774\uc2a4\ubcf4\ub2e4 \ub9ce\uc740 \uc2b9\uc218\ub97c \uae30\ub85d\ud55c \ud300\uc740 \uc5c6\uc5c8\ub2e4. \uc560\uc2ac\ub808\ud2f1\uc2a4\ub294 \ub514\ube44\uc804 \uc2dc\ub9ac\uc988\uc5d0\uc11c \uc591\ud0a4\uc2a4 \uc640 \uc7ac\ub300\uacb0\uc744 \ud588\ub2e4. \ub871\uc740 1\ucc28\uc804\uc5d0\uc11c \ub450 \uac1c\uc758 \ud648\ub7f0\uc744 \ud130\ub728\ub9ac\uba70 \ud300\uc758 5-3 \uc2b9\ub9ac\ub97c \uc774\ub04c\uc5c8\ub2e4. \uc774\uc740 3\ucc28\uc804\uc5d0\uc11c\ub294 \ud300\uc774 1-0\uc73c\ub85c \uc9c0\uace0 \uc788\ub294 \uc0c1\ud669\uc5d0\uc11c, 2\uc544\uc6c3 1\ub8e8 \uc8fc\uc790 \uc81c\ub808\ubbf8 \uc9c0\uc554\ube44\uac00 \ucd9c\ub8e8\ud55c \uac00\uc6b4\ub370 \ud0c0\uc11d\uc5d0 \ub4e4\uc5b4\uc11c \uc678\uc57c \uc6b0\uce21\uc73c\ub85c \ud0c0\uad6c\ub97c \ub0a0\ub824\ubcf4\ub0c8\ub2e4. \ud558\uc9c0\ub9cc \uc774 \ub54c 3\ub8e8\ub97c \ub3cc\uace0 \ud648\uc73c\ub85c \ub4e4\uc5b4\uc624\ub358 \uc9c0\uc554\ube44\uac00 \ub370\ub9ad \uc9c0\ud130\uc758 \uc720\uba85\ud55c '\ub354 \ud50c\ub9bd'(The Flip) \ud50c\ub808\uc774\ub85c \uc544\uc6c3\uc744 \ub2f9\ud588\uace0, \uc5d0\uc774\uc2a4\ub294 \uadf8 \uc810\uc218 \uadf8\ub300\ub85c \ud328\ubc30\ud588\ub2e4. 5\ucc28\uc804 \uc2dc\ub9ac\uc988\uc5d0\uc11c 2\uc5f0\uc2b9 \ub4a4 3\uc5f0\ud328\ub97c \ub2f9\ud558\uba70 \uc5d0\uc774\uc2a4\ub294 2\ub144 \uc5f0\uc18d \ucc54\ud53c\uc5b8\uc2ed \uc2dc\ub9ac\uc988 \uc9c4\ucd9c\uc5d0 \uc2e4\ud328\ud588\ub2e4. \ub871\uc740 \uc2dc\ub9ac\uc988 5\uacbd\uae30, 7\uc548\ud0c0, 2\ud648\ub7f0, 3\ud0c0\uc810, \ud0c0\uc728 .389\ub97c \uae30\ub85d\ud588\ub2e4.", "paragraph_answer": "2001\ub144 \uc2dc\uc98c \uc624\ud074\ub79c\ub4dc \uc560\uc2ac\ub808\ud2f1\uc2a4\ub294 102\uc2b9 60\ud328\ub97c \uac70\ub450\uc5c8\uc73c\ub098, \uac19\uc740 \uc9c0\uad6c\uc758 \uc2dc\uc560\ud2c0 \ub9e4\ub9ac\ub108\uc2a4\uac00 116\uc2b9\uc744 \uac70\ub450\uba74\uc11c \uc640\uc77c\ub4dc\uce74\ub4dc\ub85c \ud3ec\uc2a4\ud2b8\uc2dc\uc98c\uc5d0 \uc9c4\ucd9c\ud588\ub2e4. \uc624\ud074\ub79c\ub4dc\uac00 \uc18d\ud55c \uc9c0\uad6c \uc774\uc678\uc758 \ub2e4\ub978 \ub2e4\uc12f \uc9c0\uad6c \uc6b0\uc2b9\ud300\ub4e4 \uc911 \uc5d0\uc774\uc2a4\ubcf4\ub2e4 \ub9ce\uc740 \uc2b9\uc218\ub97c \uae30\ub85d\ud55c \ud300\uc740 \uc5c6\uc5c8\ub2e4. \uc560\uc2ac\ub808\ud2f1\uc2a4\ub294 \ub514\ube44\uc804 \uc2dc\ub9ac\uc988\uc5d0\uc11c \uc591\ud0a4\uc2a4 \uc640 \uc7ac\ub300\uacb0\uc744 \ud588\ub2e4. \ub871\uc740 1\ucc28\uc804\uc5d0\uc11c \ub450 \uac1c\uc758 \ud648\ub7f0\uc744 \ud130\ub728\ub9ac\uba70 \ud300\uc758 5-3 \uc2b9\ub9ac\ub97c \uc774\ub04c\uc5c8\ub2e4. \uc774\uc740 3\ucc28\uc804\uc5d0\uc11c\ub294 \ud300\uc774 1-0\uc73c\ub85c \uc9c0\uace0 \uc788\ub294 \uc0c1\ud669\uc5d0\uc11c, 2\uc544\uc6c3 1\ub8e8 \uc8fc\uc790 \uc81c\ub808\ubbf8 \uc9c0\uc554\ube44\uac00 \ucd9c\ub8e8\ud55c \uac00\uc6b4\ub370 \ud0c0\uc11d\uc5d0 \ub4e4\uc5b4\uc11c \uc678\uc57c \uc6b0\uce21\uc73c\ub85c \ud0c0\uad6c\ub97c \ub0a0\ub824\ubcf4\ub0c8\ub2e4. \ud558\uc9c0\ub9cc \uc774 \ub54c 3\ub8e8\ub97c \ub3cc\uace0 \ud648\uc73c\ub85c \ub4e4\uc5b4\uc624\ub358 \uc9c0\uc554\ube44\uac00 \ub370\ub9ad \uc9c0\ud130\uc758 \uc720\uba85\ud55c '\ub354 \ud50c\ub9bd'(The Flip) \ud50c\ub808\uc774\ub85c \uc544\uc6c3\uc744 \ub2f9\ud588\uace0, \uc5d0\uc774\uc2a4\ub294 \uadf8 \uc810\uc218 \uadf8\ub300\ub85c \ud328\ubc30\ud588\ub2e4. 5\ucc28\uc804 \uc2dc\ub9ac\uc988\uc5d0\uc11c 2\uc5f0\uc2b9 \ub4a4 3\uc5f0\ud328\ub97c \ub2f9\ud558\uba70 \uc5d0\uc774\uc2a4\ub294 2\ub144 \uc5f0\uc18d \ucc54\ud53c\uc5b8\uc2ed \uc2dc\ub9ac\uc988 \uc9c4\ucd9c\uc5d0 \uc2e4\ud328\ud588\ub2e4. \ub871\uc740 \uc2dc\ub9ac\uc988 5\uacbd\uae30, 7\uc548\ud0c0, 2\ud648\ub7f0, 3\ud0c0\uc810, \ud0c0\uc728 .389\ub97c \uae30\ub85d\ud588\ub2e4.", "sentence_answer": "\uc560\uc2ac\ub808\ud2f1\uc2a4\ub294 \ub514\ube44\uc804 \uc2dc\ub9ac\uc988\uc5d0\uc11c \uc591\ud0a4\uc2a4 \uc640 \uc7ac\ub300\uacb0\uc744 \ud588\ub2e4.", "paragraph_id": "6565664-8-0", "paragraph_question": "question: 2001\ub144 \uc560\uc2ac\ub808\ud2f1\uc2a4\uac00 \ub514\ube44\uc804 \uc2dc\ub9ac\uc988\uc5d0\uc11c \ub9cc\ub09c \ud300\uc740?, context: 2001\ub144 \uc2dc\uc98c \uc624\ud074\ub79c\ub4dc \uc560\uc2ac\ub808\ud2f1\uc2a4\ub294 102\uc2b9 60\ud328\ub97c \uac70\ub450\uc5c8\uc73c\ub098, \uac19\uc740 \uc9c0\uad6c\uc758 \uc2dc\uc560\ud2c0 \ub9e4\ub9ac\ub108\uc2a4\uac00 116\uc2b9\uc744 \uac70\ub450\uba74\uc11c \uc640\uc77c\ub4dc\uce74\ub4dc\ub85c \ud3ec\uc2a4\ud2b8\uc2dc\uc98c\uc5d0 \uc9c4\ucd9c\ud588\ub2e4. \uc624\ud074\ub79c\ub4dc\uac00 \uc18d\ud55c \uc9c0\uad6c \uc774\uc678\uc758 \ub2e4\ub978 \ub2e4\uc12f \uc9c0\uad6c \uc6b0\uc2b9\ud300\ub4e4 \uc911 \uc5d0\uc774\uc2a4\ubcf4\ub2e4 \ub9ce\uc740 \uc2b9\uc218\ub97c \uae30\ub85d\ud55c \ud300\uc740 \uc5c6\uc5c8\ub2e4. \uc560\uc2ac\ub808\ud2f1\uc2a4\ub294 \ub514\ube44\uc804 \uc2dc\ub9ac\uc988\uc5d0\uc11c \uc591\ud0a4\uc2a4\uc640 \uc7ac\ub300\uacb0\uc744 \ud588\ub2e4. \ub871\uc740 1\ucc28\uc804\uc5d0\uc11c \ub450 \uac1c\uc758 \ud648\ub7f0\uc744 \ud130\ub728\ub9ac\uba70 \ud300\uc758 5-3 \uc2b9\ub9ac\ub97c \uc774\ub04c\uc5c8\ub2e4. \uc774\uc740 3\ucc28\uc804\uc5d0\uc11c\ub294 \ud300\uc774 1-0\uc73c\ub85c \uc9c0\uace0 \uc788\ub294 \uc0c1\ud669\uc5d0\uc11c, 2\uc544\uc6c3 1\ub8e8 \uc8fc\uc790 \uc81c\ub808\ubbf8 \uc9c0\uc554\ube44\uac00 \ucd9c\ub8e8\ud55c \uac00\uc6b4\ub370 \ud0c0\uc11d\uc5d0 \ub4e4\uc5b4\uc11c \uc678\uc57c \uc6b0\uce21\uc73c\ub85c \ud0c0\uad6c\ub97c \ub0a0\ub824\ubcf4\ub0c8\ub2e4. \ud558\uc9c0\ub9cc \uc774 \ub54c 3\ub8e8\ub97c \ub3cc\uace0 \ud648\uc73c\ub85c \ub4e4\uc5b4\uc624\ub358 \uc9c0\uc554\ube44\uac00 \ub370\ub9ad \uc9c0\ud130\uc758 \uc720\uba85\ud55c '\ub354 \ud50c\ub9bd'(The Flip) \ud50c\ub808\uc774\ub85c \uc544\uc6c3\uc744 \ub2f9\ud588\uace0, \uc5d0\uc774\uc2a4\ub294 \uadf8 \uc810\uc218 \uadf8\ub300\ub85c \ud328\ubc30\ud588\ub2e4. 5\ucc28\uc804 \uc2dc\ub9ac\uc988\uc5d0\uc11c 2\uc5f0\uc2b9 \ub4a4 3\uc5f0\ud328\ub97c \ub2f9\ud558\uba70 \uc5d0\uc774\uc2a4\ub294 2\ub144 \uc5f0\uc18d \ucc54\ud53c\uc5b8\uc2ed \uc2dc\ub9ac\uc988 \uc9c4\ucd9c\uc5d0 \uc2e4\ud328\ud588\ub2e4. \ub871\uc740 \uc2dc\ub9ac\uc988 5\uacbd\uae30, 7\uc548\ud0c0, 2\ud648\ub7f0, 3\ud0c0\uc810, \ud0c0\uc728 .389\ub97c \uae30\ub85d\ud588\ub2e4."} {"question": "1980\ub144\uc5d0 \ub098\uce74\ubaa8\ub9ac \uc544\ud0a4\ub098\uac00 \uc608\ub2a5\uc2e0\uc778 \ubc1c\uad74 \ud504\ub85c\uadf8\ub7a8\uc5d0\uc11c \ubd80\ub978 \ub178\ub798\ub294?", "paragraph": "13\uc0b4\uc774\uc5c8\ub358 1978\ub144\uc5d0 \ub2c8\ud63c \ud14c\ub808\ube44\uc5d0\uc11c \ubc29\uc601\ud558\ub358 \uc608\ub2a5\uc2e0\uc778 \ubc1c\uad74 \ud504\ub85c\uadf8\ub7a8 \u300a\uc2a4\ud0c0 \ud0c4\uc0dd!\u300b(\u30b9\u30bf\u30fc\u8a95\u751f!)\uc5d0 \ucd9c\uc5f0\ud558\uc5ec \uc774\uc640\uc0ac\ud0a4 \ud788\ub85c\ubbf8\uc758 \u3008\uc5ec\ub984\uc5d0 \uc548\uae30\uc5b4\u3009(\u590f\u306b\u62b1\u308c\u3066)\ub97c \ubd88\ub800\uc73c\ub098 \"\ub098\uc774\uc5d0 \ube44\ud574 \uc74c\uc0c9\uc774 \ub108\ubb34 \uc5b4\ub978\uc2a4\ub7fd\ub2e4\"\ub294 \uc774\uc720\ub85c \ubcf8\uc120\uc5d0\uc11c \ud0c8\ub77d\ud558\uc600\ub2e4. 2\ub144 \ud6c4\uc778 1980\ub144\uc5d0\ub294 \ud6d7\ub0a0 \uc790\uc2e0\uc758 \uac00\uc7a5 \ud070 \ub77c\uc774\ubc8c\uc774 \ub418\ub294 \uc2e0\uc778\uac00\uc218 \ub9c8\uce20\ub2e4 \uc138\uc774\ucf54\uc758 \u3008\ud478\ub978 \uc0b0\ud638\ucd08\u3009(\u9752\u3044\u73ca\u745a\u7901)\ub97c \ubd80\ub97c \ub54c\ub294 \uc2ec\uc0ac\uc704\uc6d0\uc774 \ub178\ub798\ub294 \uc798 \ubd80\ub974\ub294\ub370 \uc5bc\uad74\uc774 \uc544\uc774 \uac19\uc544\uc11c \ucc28\ub77c\ub9ac \ub3d9\uc694\ub97c \ubd80\ub974\ub294\uac8c \ub0ab\ub2e4\uba70 \ud639\ud3c9\uc744 \ud588\ub294\ub370, \uc774\uc5d0 \ub098\uce74\ubaa8\ub9ac\ub294 \uc774 \ud504\ub85c\uadf8\ub7a8\uc5d0\uc11c \ub3d9\uc694\ub294 \ubd80\ub97c \uc218 \uc5c6\uc9c0 \uc54a\ub0d0\uace0 \ubc18\ubb38\ud558\uc600\uc73c\ub098 \ub2e4\uc2dc \ud55c \ubc88 \ubcf8\uc120\uc5d0\uc11c \ub5a8\uc5b4\uc84c\ub2e4. \uadf8\ub7ec\ub098 \uc774\ub4ec\ud574\uc778 1981\ub144 7\uc6d4 11\uc77c, \uc57c\ub9c8\uad6c\uce58 \ubaa8\ubaa8\uc5d0\uc758 \u3008\ubabd\uc120\uc548\ub0b4\uc778\u3009(\u5922\u5148\u6848\u5185\u4eba)\uc73c\ub85c \uc138 \ubc88\uc9f8 \ub3c4\uc804 \ub05d\uc5d0, \ud504\ub85c\uadf8\ub7a8 \ubc29\uc1a1\uc0ac\uc0c1 \ucd5c\uace0 \uc810\uc218\uc778 392\uc810\uc744 \uc5bb\uc73c\uba70 \uc6b0\uc2b9\ud558\uac8c \ub41c\ub2e4. \uc81c\uc791 \uc2a4\ud0dc\ud504 \uc911\uc5d0\uc11c\ub294 \uc774\ubbf8 \ubcf8\uc120\uc5d0\uc11c \ub450 \ubc88\uc774\ub098 \ub5a8\uc5b4\uc9c4 \ub098\uce74\ubaa8\ub9ac\uc758 \uc7ac\ucd9c\uc804\uc5d0 \ub09c\uc0c9\uc744 \ud45c\ud558\ub294 \uc0ac\ub78c\ub3c4 \uc788\uc5c8\ub2e4. \uadf8\ub7ec\ub098 \ub098\uce74\ubaa8\ub9ac\ub294 1,2\ucc28\uc5d0\uc11c\uc758 \uad74\uc695\uc744 \ub9d0\ub054\ud788 \uc53b\uc5b4\ub0b4\uc5c8\uace0, \uc6b0\uc2b9 \ud6c4 1\uac1c\uc6d4 \ub3d9\uc548 \ub2c8\ud63c \ud14c\ub808\ube44 \uc74c\uc545\ud559\uc6d0\uc5d0\uc11c \uac04\ub2e8\ud55c \ubcf4\uceec\ud2b8\ub808\uc774\ub2dd\uacfc \ub304\uc2a4 \uc218\uc5c5\uc744 \ubc1b\uc558\ub2e4. \ub4a4\uc774\uc5b4 \uac19\uc740 \ud574 11\uc6d4 29\uc77c, \uc5f0\ub9d0 \uacb0\uc2b9\uc804\uc5d0 \ud574\ub2f9\ud558\ub294 \uacb0\uc804\ub300\ud68c \uc0ac\uc804\uc2ec\uc0ac\ub97c \uac70\uce58\uace0, 12\uc6d4 6\uc77c\uc5d0 \ubc29\uc1a1\ub41c \uacb0\uc804\ub300\ud68c \ubc29\uc1a1\uc5d0\uc11c \uc5ed\uc2dc \uc57c\ub9c8\uad6c\uce58 \ubaa8\ubaa8\uc5d0\uc758 \u3008\ubabd\uc120\uc548\ub0b4\uc778\u3009\uc744 \ubd88\ub800\ub2e4. \uc774\ud6c4 11\uac1c\uc758 \uc5f0\uc608\uae30\ud68d\uc0ac\uc640 \ub808\ucf54\ub4dc \ub808\uc774\ube14 \uc0ac\ubb34\uc18c\uac00 \ud50c\ub79c\uce74\ub4dc\ub97c \uc62c\ub824 \ub098\uce74\ubaa8\ub9ac\ub97c \uc2a4\uce74\uc6b0\ud2b8\ud558\uaca0\ub2e4\ub294 \uc758\uc0ac\ub97c \ubc1d\ud614\uace0, \ub354\ubd88\uc5b4 \ub300\ud68c\uc5d0\uc11c\ub3c4 \uac00\uc7a5 \ub192\uc740 \uc810\uc218\ub97c \uae30\ub85d\ud558\uc5ec \uacb0\uc804\ub300\ud68c \ucd5c\uc6b0\uc218\uc0c1\uae4c\uc9c0 \uc218\uc0c1\ud558\uc600\ub2e4. \uc774\ud6c4 \ub098\uce74\ubaa8\ub9ac \uc544\ud0a4\ub098\ub294 \uc5f0\uc608\uae30\ud68d\uc0ac\ub294 \ucf04\uc628\uacfc, \ub808\ucf54\ub4dc \ub808\uc774\ube14\uc740 \uc6cc\ub108 \ud30c\uc774\uc624\ub2c8\uc5b4\uc640 \uacc4\uc57d\uc744 \ub9fa\uc5c8\ub2e4. \uacc4\uc57d\uc744 \uccb4\uacb0\ud55c \uc774\ud6c4\uc5d0\ub294 \uc8fc\ub85c \ubcf4\uceec \ud2b8\ub808\uc774\ub108\uc758 \uc804\ub2f4 \ud558\uc5d0 \ubc1c\uc131 \uc5f0\uc2b5\uacfc \uac00\ucc3d\ub825 \ud5a5\uc0c1\uc5d0 \uc9d1\uc911\uc801\uc778 \ud6c8\ub828\uc744 \ubc1b\uc558\ub2e4. \ub098\uce74\ubaa8\ub9ac\ub294 \ubcf8\ub798 \ub367\ub2c8\uac00 \uc788\uc5c8\uc73c\ub098 \ub370\ubdd4 \uc804 \uc18c\uc18d\uc0ac\uc778 \ucf04\uc628\uc774 \uc544\uc774\ub3cc\ub2e4\uc6b0\ub824\uba74 \ub367\ub2c8\ubd80\ud130 \uace0\ub974\uac8c \ud574\uc57c\ud55c\ub2e4 \ud558\uc5ec \uc774\ub97c \uad50\uc815\ud558\uae30\ub3c4 \ud588\ub2e4.", "answer": "\ud478\ub978 \uc0b0\ud638\ucd08", "sentence": "2\ub144 \ud6c4\uc778 1980\ub144\uc5d0\ub294 \ud6d7\ub0a0 \uc790\uc2e0\uc758 \uac00\uc7a5 \ud070 \ub77c\uc774\ubc8c\uc774 \ub418\ub294 \uc2e0\uc778\uac00\uc218 \ub9c8\uce20\ub2e4 \uc138\uc774\ucf54\uc758 \u3008 \ud478\ub978 \uc0b0\ud638\ucd08 \u3009(\u9752\u3044\u73ca\u745a\u7901)\ub97c \ubd80\ub97c \ub54c\ub294 \uc2ec\uc0ac\uc704\uc6d0\uc774 \ub178\ub798\ub294 \uc798 \ubd80\ub974\ub294\ub370 \uc5bc\uad74\uc774 \uc544\uc774 \uac19\uc544\uc11c \ucc28\ub77c\ub9ac \ub3d9\uc694\ub97c \ubd80\ub974\ub294\uac8c \ub0ab\ub2e4\uba70 \ud639\ud3c9\uc744 \ud588\ub294\ub370, \uc774\uc5d0 \ub098\uce74\ubaa8\ub9ac\ub294 \uc774 \ud504\ub85c\uadf8\ub7a8\uc5d0\uc11c \ub3d9\uc694\ub294 \ubd80\ub97c \uc218 \uc5c6\uc9c0 \uc54a\ub0d0\uace0 \ubc18\ubb38\ud558\uc600\uc73c\ub098 \ub2e4\uc2dc \ud55c \ubc88 \ubcf8\uc120\uc5d0\uc11c \ub5a8\uc5b4\uc84c\ub2e4.", "paragraph_sentence": "13\uc0b4\uc774\uc5c8\ub358 1978\ub144\uc5d0 \ub2c8\ud63c \ud14c\ub808\ube44\uc5d0\uc11c \ubc29\uc601\ud558\ub358 \uc608\ub2a5\uc2e0\uc778 \ubc1c\uad74 \ud504\ub85c\uadf8\ub7a8 \u300a\uc2a4\ud0c0 \ud0c4\uc0dd!\u300b(\u30b9\u30bf\u30fc\u8a95\u751f!)\uc5d0 \ucd9c\uc5f0\ud558\uc5ec \uc774\uc640\uc0ac\ud0a4 \ud788\ub85c\ubbf8\uc758 \u3008\uc5ec\ub984\uc5d0 \uc548\uae30\uc5b4\u3009(\u590f\u306b\u62b1\u308c\u3066)\ub97c \ubd88\ub800\uc73c\ub098 \"\ub098\uc774\uc5d0 \ube44\ud574 \uc74c\uc0c9\uc774 \ub108\ubb34 \uc5b4\ub978\uc2a4\ub7fd\ub2e4\"\ub294 \uc774\uc720\ub85c \ubcf8\uc120\uc5d0\uc11c \ud0c8\ub77d\ud558\uc600\ub2e4. 2\ub144 \ud6c4\uc778 1980\ub144\uc5d0\ub294 \ud6d7\ub0a0 \uc790\uc2e0\uc758 \uac00\uc7a5 \ud070 \ub77c\uc774\ubc8c\uc774 \ub418\ub294 \uc2e0\uc778\uac00\uc218 \ub9c8\uce20\ub2e4 \uc138\uc774\ucf54\uc758 \u3008 \ud478\ub978 \uc0b0\ud638\ucd08 \u3009(\u9752\u3044\u73ca\u745a\u7901)\ub97c \ubd80\ub97c \ub54c\ub294 \uc2ec\uc0ac\uc704\uc6d0\uc774 \ub178\ub798\ub294 \uc798 \ubd80\ub974\ub294\ub370 \uc5bc\uad74\uc774 \uc544\uc774 \uac19\uc544\uc11c \ucc28\ub77c\ub9ac \ub3d9\uc694\ub97c \ubd80\ub974\ub294\uac8c \ub0ab\ub2e4\uba70 \ud639\ud3c9\uc744 \ud588\ub294\ub370, \uc774\uc5d0 \ub098\uce74\ubaa8\ub9ac\ub294 \uc774 \ud504\ub85c\uadf8\ub7a8\uc5d0\uc11c \ub3d9\uc694\ub294 \ubd80\ub97c \uc218 \uc5c6\uc9c0 \uc54a\ub0d0\uace0 \ubc18\ubb38\ud558\uc600\uc73c\ub098 \ub2e4\uc2dc \ud55c \ubc88 \ubcf8\uc120\uc5d0\uc11c \ub5a8\uc5b4\uc84c\ub2e4. \uadf8\ub7ec\ub098 \uc774\ub4ec\ud574\uc778 1981\ub144 7\uc6d4 11\uc77c, \uc57c\ub9c8\uad6c\uce58 \ubaa8\ubaa8\uc5d0\uc758 \u3008\ubabd\uc120\uc548\ub0b4\uc778\u3009(\u5922\u5148\u6848\u5185\u4eba)\uc73c\ub85c \uc138 \ubc88\uc9f8 \ub3c4\uc804 \ub05d\uc5d0, \ud504\ub85c\uadf8\ub7a8 \ubc29\uc1a1\uc0ac\uc0c1 \ucd5c\uace0 \uc810\uc218\uc778 392\uc810\uc744 \uc5bb\uc73c\uba70 \uc6b0\uc2b9\ud558\uac8c \ub41c\ub2e4. \uc81c\uc791 \uc2a4\ud0dc\ud504 \uc911\uc5d0\uc11c\ub294 \uc774\ubbf8 \ubcf8\uc120\uc5d0\uc11c \ub450 \ubc88\uc774\ub098 \ub5a8\uc5b4\uc9c4 \ub098\uce74\ubaa8\ub9ac\uc758 \uc7ac\ucd9c\uc804\uc5d0 \ub09c\uc0c9\uc744 \ud45c\ud558\ub294 \uc0ac\ub78c\ub3c4 \uc788\uc5c8\ub2e4. \uadf8\ub7ec\ub098 \ub098\uce74\ubaa8\ub9ac\ub294 1,2\ucc28\uc5d0\uc11c\uc758 \uad74\uc695\uc744 \ub9d0\ub054\ud788 \uc53b\uc5b4\ub0b4\uc5c8\uace0, \uc6b0\uc2b9 \ud6c4 1\uac1c\uc6d4 \ub3d9\uc548 \ub2c8\ud63c \ud14c\ub808\ube44 \uc74c\uc545\ud559\uc6d0\uc5d0\uc11c \uac04\ub2e8\ud55c \ubcf4\uceec\ud2b8\ub808\uc774\ub2dd\uacfc \ub304\uc2a4 \uc218\uc5c5\uc744 \ubc1b\uc558\ub2e4. \ub4a4\uc774\uc5b4 \uac19\uc740 \ud574 11\uc6d4 29\uc77c, \uc5f0\ub9d0 \uacb0\uc2b9\uc804\uc5d0 \ud574\ub2f9\ud558\ub294 \uacb0\uc804\ub300\ud68c \uc0ac\uc804\uc2ec\uc0ac\ub97c \uac70\uce58\uace0, 12\uc6d4 6\uc77c\uc5d0 \ubc29\uc1a1\ub41c \uacb0\uc804\ub300\ud68c \ubc29\uc1a1\uc5d0\uc11c \uc5ed\uc2dc \uc57c\ub9c8\uad6c\uce58 \ubaa8\ubaa8\uc5d0\uc758 \u3008\ubabd\uc120\uc548\ub0b4\uc778\u3009\uc744 \ubd88\ub800\ub2e4. \uc774\ud6c4 11\uac1c\uc758 \uc5f0\uc608\uae30\ud68d\uc0ac\uc640 \ub808\ucf54\ub4dc \ub808\uc774\ube14 \uc0ac\ubb34\uc18c\uac00 \ud50c\ub79c\uce74\ub4dc\ub97c \uc62c\ub824 \ub098\uce74\ubaa8\ub9ac\ub97c \uc2a4\uce74\uc6b0\ud2b8\ud558\uaca0\ub2e4\ub294 \uc758\uc0ac\ub97c \ubc1d\ud614\uace0, \ub354\ubd88\uc5b4 \ub300\ud68c\uc5d0\uc11c\ub3c4 \uac00\uc7a5 \ub192\uc740 \uc810\uc218\ub97c \uae30\ub85d\ud558\uc5ec \uacb0\uc804\ub300\ud68c \ucd5c\uc6b0\uc218\uc0c1\uae4c\uc9c0 \uc218\uc0c1\ud558\uc600\ub2e4. \uc774\ud6c4 \ub098\uce74\ubaa8\ub9ac \uc544\ud0a4\ub098\ub294 \uc5f0\uc608\uae30\ud68d\uc0ac\ub294 \ucf04\uc628\uacfc, \ub808\ucf54\ub4dc \ub808\uc774\ube14\uc740 \uc6cc\ub108 \ud30c\uc774\uc624\ub2c8\uc5b4\uc640 \uacc4\uc57d\uc744 \ub9fa\uc5c8\ub2e4. \uacc4\uc57d\uc744 \uccb4\uacb0\ud55c \uc774\ud6c4\uc5d0\ub294 \uc8fc\ub85c \ubcf4\uceec \ud2b8\ub808\uc774\ub108\uc758 \uc804\ub2f4 \ud558\uc5d0 \ubc1c\uc131 \uc5f0\uc2b5\uacfc \uac00\ucc3d\ub825 \ud5a5\uc0c1\uc5d0 \uc9d1\uc911\uc801\uc778 \ud6c8\ub828\uc744 \ubc1b\uc558\ub2e4. \ub098\uce74\ubaa8\ub9ac\ub294 \ubcf8\ub798 \ub367\ub2c8\uac00 \uc788\uc5c8\uc73c\ub098 \ub370\ubdd4 \uc804 \uc18c\uc18d\uc0ac\uc778 \ucf04\uc628\uc774 \uc544\uc774\ub3cc\ub2e4\uc6b0\ub824\uba74 \ub367\ub2c8\ubd80\ud130 \uace0\ub974\uac8c \ud574\uc57c\ud55c\ub2e4 \ud558\uc5ec \uc774\ub97c \uad50\uc815\ud558\uae30\ub3c4 \ud588\ub2e4.", "paragraph_answer": "13\uc0b4\uc774\uc5c8\ub358 1978\ub144\uc5d0 \ub2c8\ud63c \ud14c\ub808\ube44\uc5d0\uc11c \ubc29\uc601\ud558\ub358 \uc608\ub2a5\uc2e0\uc778 \ubc1c\uad74 \ud504\ub85c\uadf8\ub7a8 \u300a\uc2a4\ud0c0 \ud0c4\uc0dd!\u300b(\u30b9\u30bf\u30fc\u8a95\u751f!)\uc5d0 \ucd9c\uc5f0\ud558\uc5ec \uc774\uc640\uc0ac\ud0a4 \ud788\ub85c\ubbf8\uc758 \u3008\uc5ec\ub984\uc5d0 \uc548\uae30\uc5b4\u3009(\u590f\u306b\u62b1\u308c\u3066)\ub97c \ubd88\ub800\uc73c\ub098 \"\ub098\uc774\uc5d0 \ube44\ud574 \uc74c\uc0c9\uc774 \ub108\ubb34 \uc5b4\ub978\uc2a4\ub7fd\ub2e4\"\ub294 \uc774\uc720\ub85c \ubcf8\uc120\uc5d0\uc11c \ud0c8\ub77d\ud558\uc600\ub2e4. 2\ub144 \ud6c4\uc778 1980\ub144\uc5d0\ub294 \ud6d7\ub0a0 \uc790\uc2e0\uc758 \uac00\uc7a5 \ud070 \ub77c\uc774\ubc8c\uc774 \ub418\ub294 \uc2e0\uc778\uac00\uc218 \ub9c8\uce20\ub2e4 \uc138\uc774\ucf54\uc758 \u3008 \ud478\ub978 \uc0b0\ud638\ucd08 \u3009(\u9752\u3044\u73ca\u745a\u7901)\ub97c \ubd80\ub97c \ub54c\ub294 \uc2ec\uc0ac\uc704\uc6d0\uc774 \ub178\ub798\ub294 \uc798 \ubd80\ub974\ub294\ub370 \uc5bc\uad74\uc774 \uc544\uc774 \uac19\uc544\uc11c \ucc28\ub77c\ub9ac \ub3d9\uc694\ub97c \ubd80\ub974\ub294\uac8c \ub0ab\ub2e4\uba70 \ud639\ud3c9\uc744 \ud588\ub294\ub370, \uc774\uc5d0 \ub098\uce74\ubaa8\ub9ac\ub294 \uc774 \ud504\ub85c\uadf8\ub7a8\uc5d0\uc11c \ub3d9\uc694\ub294 \ubd80\ub97c \uc218 \uc5c6\uc9c0 \uc54a\ub0d0\uace0 \ubc18\ubb38\ud558\uc600\uc73c\ub098 \ub2e4\uc2dc \ud55c \ubc88 \ubcf8\uc120\uc5d0\uc11c \ub5a8\uc5b4\uc84c\ub2e4. \uadf8\ub7ec\ub098 \uc774\ub4ec\ud574\uc778 1981\ub144 7\uc6d4 11\uc77c, \uc57c\ub9c8\uad6c\uce58 \ubaa8\ubaa8\uc5d0\uc758 \u3008\ubabd\uc120\uc548\ub0b4\uc778\u3009(\u5922\u5148\u6848\u5185\u4eba)\uc73c\ub85c \uc138 \ubc88\uc9f8 \ub3c4\uc804 \ub05d\uc5d0, \ud504\ub85c\uadf8\ub7a8 \ubc29\uc1a1\uc0ac\uc0c1 \ucd5c\uace0 \uc810\uc218\uc778 392\uc810\uc744 \uc5bb\uc73c\uba70 \uc6b0\uc2b9\ud558\uac8c \ub41c\ub2e4. \uc81c\uc791 \uc2a4\ud0dc\ud504 \uc911\uc5d0\uc11c\ub294 \uc774\ubbf8 \ubcf8\uc120\uc5d0\uc11c \ub450 \ubc88\uc774\ub098 \ub5a8\uc5b4\uc9c4 \ub098\uce74\ubaa8\ub9ac\uc758 \uc7ac\ucd9c\uc804\uc5d0 \ub09c\uc0c9\uc744 \ud45c\ud558\ub294 \uc0ac\ub78c\ub3c4 \uc788\uc5c8\ub2e4. \uadf8\ub7ec\ub098 \ub098\uce74\ubaa8\ub9ac\ub294 1,2\ucc28\uc5d0\uc11c\uc758 \uad74\uc695\uc744 \ub9d0\ub054\ud788 \uc53b\uc5b4\ub0b4\uc5c8\uace0, \uc6b0\uc2b9 \ud6c4 1\uac1c\uc6d4 \ub3d9\uc548 \ub2c8\ud63c \ud14c\ub808\ube44 \uc74c\uc545\ud559\uc6d0\uc5d0\uc11c \uac04\ub2e8\ud55c \ubcf4\uceec\ud2b8\ub808\uc774\ub2dd\uacfc \ub304\uc2a4 \uc218\uc5c5\uc744 \ubc1b\uc558\ub2e4. \ub4a4\uc774\uc5b4 \uac19\uc740 \ud574 11\uc6d4 29\uc77c, \uc5f0\ub9d0 \uacb0\uc2b9\uc804\uc5d0 \ud574\ub2f9\ud558\ub294 \uacb0\uc804\ub300\ud68c \uc0ac\uc804\uc2ec\uc0ac\ub97c \uac70\uce58\uace0, 12\uc6d4 6\uc77c\uc5d0 \ubc29\uc1a1\ub41c \uacb0\uc804\ub300\ud68c \ubc29\uc1a1\uc5d0\uc11c \uc5ed\uc2dc \uc57c\ub9c8\uad6c\uce58 \ubaa8\ubaa8\uc5d0\uc758 \u3008\ubabd\uc120\uc548\ub0b4\uc778\u3009\uc744 \ubd88\ub800\ub2e4. \uc774\ud6c4 11\uac1c\uc758 \uc5f0\uc608\uae30\ud68d\uc0ac\uc640 \ub808\ucf54\ub4dc \ub808\uc774\ube14 \uc0ac\ubb34\uc18c\uac00 \ud50c\ub79c\uce74\ub4dc\ub97c \uc62c\ub824 \ub098\uce74\ubaa8\ub9ac\ub97c \uc2a4\uce74\uc6b0\ud2b8\ud558\uaca0\ub2e4\ub294 \uc758\uc0ac\ub97c \ubc1d\ud614\uace0, \ub354\ubd88\uc5b4 \ub300\ud68c\uc5d0\uc11c\ub3c4 \uac00\uc7a5 \ub192\uc740 \uc810\uc218\ub97c \uae30\ub85d\ud558\uc5ec \uacb0\uc804\ub300\ud68c \ucd5c\uc6b0\uc218\uc0c1\uae4c\uc9c0 \uc218\uc0c1\ud558\uc600\ub2e4. \uc774\ud6c4 \ub098\uce74\ubaa8\ub9ac \uc544\ud0a4\ub098\ub294 \uc5f0\uc608\uae30\ud68d\uc0ac\ub294 \ucf04\uc628\uacfc, \ub808\ucf54\ub4dc \ub808\uc774\ube14\uc740 \uc6cc\ub108 \ud30c\uc774\uc624\ub2c8\uc5b4\uc640 \uacc4\uc57d\uc744 \ub9fa\uc5c8\ub2e4. \uacc4\uc57d\uc744 \uccb4\uacb0\ud55c \uc774\ud6c4\uc5d0\ub294 \uc8fc\ub85c \ubcf4\uceec \ud2b8\ub808\uc774\ub108\uc758 \uc804\ub2f4 \ud558\uc5d0 \ubc1c\uc131 \uc5f0\uc2b5\uacfc \uac00\ucc3d\ub825 \ud5a5\uc0c1\uc5d0 \uc9d1\uc911\uc801\uc778 \ud6c8\ub828\uc744 \ubc1b\uc558\ub2e4. \ub098\uce74\ubaa8\ub9ac\ub294 \ubcf8\ub798 \ub367\ub2c8\uac00 \uc788\uc5c8\uc73c\ub098 \ub370\ubdd4 \uc804 \uc18c\uc18d\uc0ac\uc778 \ucf04\uc628\uc774 \uc544\uc774\ub3cc\ub2e4\uc6b0\ub824\uba74 \ub367\ub2c8\ubd80\ud130 \uace0\ub974\uac8c \ud574\uc57c\ud55c\ub2e4 \ud558\uc5ec \uc774\ub97c \uad50\uc815\ud558\uae30\ub3c4 \ud588\ub2e4.", "sentence_answer": "2\ub144 \ud6c4\uc778 1980\ub144\uc5d0\ub294 \ud6d7\ub0a0 \uc790\uc2e0\uc758 \uac00\uc7a5 \ud070 \ub77c\uc774\ubc8c\uc774 \ub418\ub294 \uc2e0\uc778\uac00\uc218 \ub9c8\uce20\ub2e4 \uc138\uc774\ucf54\uc758 \u3008 \ud478\ub978 \uc0b0\ud638\ucd08 \u3009(\u9752\u3044\u73ca\u745a\u7901)\ub97c \ubd80\ub97c \ub54c\ub294 \uc2ec\uc0ac\uc704\uc6d0\uc774 \ub178\ub798\ub294 \uc798 \ubd80\ub974\ub294\ub370 \uc5bc\uad74\uc774 \uc544\uc774 \uac19\uc544\uc11c \ucc28\ub77c\ub9ac \ub3d9\uc694\ub97c \ubd80\ub974\ub294\uac8c \ub0ab\ub2e4\uba70 \ud639\ud3c9\uc744 \ud588\ub294\ub370, \uc774\uc5d0 \ub098\uce74\ubaa8\ub9ac\ub294 \uc774 \ud504\ub85c\uadf8\ub7a8\uc5d0\uc11c \ub3d9\uc694\ub294 \ubd80\ub97c \uc218 \uc5c6\uc9c0 \uc54a\ub0d0\uace0 \ubc18\ubb38\ud558\uc600\uc73c\ub098 \ub2e4\uc2dc \ud55c \ubc88 \ubcf8\uc120\uc5d0\uc11c \ub5a8\uc5b4\uc84c\ub2e4.", "paragraph_id": "6511890-3-2", "paragraph_question": "question: 1980\ub144\uc5d0 \ub098\uce74\ubaa8\ub9ac \uc544\ud0a4\ub098\uac00 \uc608\ub2a5\uc2e0\uc778 \ubc1c\uad74 \ud504\ub85c\uadf8\ub7a8\uc5d0\uc11c \ubd80\ub978 \ub178\ub798\ub294?, context: 13\uc0b4\uc774\uc5c8\ub358 1978\ub144\uc5d0 \ub2c8\ud63c \ud14c\ub808\ube44\uc5d0\uc11c \ubc29\uc601\ud558\ub358 \uc608\ub2a5\uc2e0\uc778 \ubc1c\uad74 \ud504\ub85c\uadf8\ub7a8 \u300a\uc2a4\ud0c0 \ud0c4\uc0dd!\u300b(\u30b9\u30bf\u30fc\u8a95\u751f!)\uc5d0 \ucd9c\uc5f0\ud558\uc5ec \uc774\uc640\uc0ac\ud0a4 \ud788\ub85c\ubbf8\uc758 \u3008\uc5ec\ub984\uc5d0 \uc548\uae30\uc5b4\u3009(\u590f\u306b\u62b1\u308c\u3066)\ub97c \ubd88\ub800\uc73c\ub098 \"\ub098\uc774\uc5d0 \ube44\ud574 \uc74c\uc0c9\uc774 \ub108\ubb34 \uc5b4\ub978\uc2a4\ub7fd\ub2e4\"\ub294 \uc774\uc720\ub85c \ubcf8\uc120\uc5d0\uc11c \ud0c8\ub77d\ud558\uc600\ub2e4. 2\ub144 \ud6c4\uc778 1980\ub144\uc5d0\ub294 \ud6d7\ub0a0 \uc790\uc2e0\uc758 \uac00\uc7a5 \ud070 \ub77c\uc774\ubc8c\uc774 \ub418\ub294 \uc2e0\uc778\uac00\uc218 \ub9c8\uce20\ub2e4 \uc138\uc774\ucf54\uc758 \u3008\ud478\ub978 \uc0b0\ud638\ucd08\u3009(\u9752\u3044\u73ca\u745a\u7901)\ub97c \ubd80\ub97c \ub54c\ub294 \uc2ec\uc0ac\uc704\uc6d0\uc774 \ub178\ub798\ub294 \uc798 \ubd80\ub974\ub294\ub370 \uc5bc\uad74\uc774 \uc544\uc774 \uac19\uc544\uc11c \ucc28\ub77c\ub9ac \ub3d9\uc694\ub97c \ubd80\ub974\ub294\uac8c \ub0ab\ub2e4\uba70 \ud639\ud3c9\uc744 \ud588\ub294\ub370, \uc774\uc5d0 \ub098\uce74\ubaa8\ub9ac\ub294 \uc774 \ud504\ub85c\uadf8\ub7a8\uc5d0\uc11c \ub3d9\uc694\ub294 \ubd80\ub97c \uc218 \uc5c6\uc9c0 \uc54a\ub0d0\uace0 \ubc18\ubb38\ud558\uc600\uc73c\ub098 \ub2e4\uc2dc \ud55c \ubc88 \ubcf8\uc120\uc5d0\uc11c \ub5a8\uc5b4\uc84c\ub2e4. \uadf8\ub7ec\ub098 \uc774\ub4ec\ud574\uc778 1981\ub144 7\uc6d4 11\uc77c, \uc57c\ub9c8\uad6c\uce58 \ubaa8\ubaa8\uc5d0\uc758 \u3008\ubabd\uc120\uc548\ub0b4\uc778\u3009(\u5922\u5148\u6848\u5185\u4eba)\uc73c\ub85c \uc138 \ubc88\uc9f8 \ub3c4\uc804 \ub05d\uc5d0, \ud504\ub85c\uadf8\ub7a8 \ubc29\uc1a1\uc0ac\uc0c1 \ucd5c\uace0 \uc810\uc218\uc778 392\uc810\uc744 \uc5bb\uc73c\uba70 \uc6b0\uc2b9\ud558\uac8c \ub41c\ub2e4. \uc81c\uc791 \uc2a4\ud0dc\ud504 \uc911\uc5d0\uc11c\ub294 \uc774\ubbf8 \ubcf8\uc120\uc5d0\uc11c \ub450 \ubc88\uc774\ub098 \ub5a8\uc5b4\uc9c4 \ub098\uce74\ubaa8\ub9ac\uc758 \uc7ac\ucd9c\uc804\uc5d0 \ub09c\uc0c9\uc744 \ud45c\ud558\ub294 \uc0ac\ub78c\ub3c4 \uc788\uc5c8\ub2e4. \uadf8\ub7ec\ub098 \ub098\uce74\ubaa8\ub9ac\ub294 1,2\ucc28\uc5d0\uc11c\uc758 \uad74\uc695\uc744 \ub9d0\ub054\ud788 \uc53b\uc5b4\ub0b4\uc5c8\uace0, \uc6b0\uc2b9 \ud6c4 1\uac1c\uc6d4 \ub3d9\uc548 \ub2c8\ud63c \ud14c\ub808\ube44 \uc74c\uc545\ud559\uc6d0\uc5d0\uc11c \uac04\ub2e8\ud55c \ubcf4\uceec\ud2b8\ub808\uc774\ub2dd\uacfc \ub304\uc2a4 \uc218\uc5c5\uc744 \ubc1b\uc558\ub2e4. \ub4a4\uc774\uc5b4 \uac19\uc740 \ud574 11\uc6d4 29\uc77c, \uc5f0\ub9d0 \uacb0\uc2b9\uc804\uc5d0 \ud574\ub2f9\ud558\ub294 \uacb0\uc804\ub300\ud68c \uc0ac\uc804\uc2ec\uc0ac\ub97c \uac70\uce58\uace0, 12\uc6d4 6\uc77c\uc5d0 \ubc29\uc1a1\ub41c \uacb0\uc804\ub300\ud68c \ubc29\uc1a1\uc5d0\uc11c \uc5ed\uc2dc \uc57c\ub9c8\uad6c\uce58 \ubaa8\ubaa8\uc5d0\uc758 \u3008\ubabd\uc120\uc548\ub0b4\uc778\u3009\uc744 \ubd88\ub800\ub2e4. \uc774\ud6c4 11\uac1c\uc758 \uc5f0\uc608\uae30\ud68d\uc0ac\uc640 \ub808\ucf54\ub4dc \ub808\uc774\ube14 \uc0ac\ubb34\uc18c\uac00 \ud50c\ub79c\uce74\ub4dc\ub97c \uc62c\ub824 \ub098\uce74\ubaa8\ub9ac\ub97c \uc2a4\uce74\uc6b0\ud2b8\ud558\uaca0\ub2e4\ub294 \uc758\uc0ac\ub97c \ubc1d\ud614\uace0, \ub354\ubd88\uc5b4 \ub300\ud68c\uc5d0\uc11c\ub3c4 \uac00\uc7a5 \ub192\uc740 \uc810\uc218\ub97c \uae30\ub85d\ud558\uc5ec \uacb0\uc804\ub300\ud68c \ucd5c\uc6b0\uc218\uc0c1\uae4c\uc9c0 \uc218\uc0c1\ud558\uc600\ub2e4. \uc774\ud6c4 \ub098\uce74\ubaa8\ub9ac \uc544\ud0a4\ub098\ub294 \uc5f0\uc608\uae30\ud68d\uc0ac\ub294 \ucf04\uc628\uacfc, \ub808\ucf54\ub4dc \ub808\uc774\ube14\uc740 \uc6cc\ub108 \ud30c\uc774\uc624\ub2c8\uc5b4\uc640 \uacc4\uc57d\uc744 \ub9fa\uc5c8\ub2e4. \uacc4\uc57d\uc744 \uccb4\uacb0\ud55c \uc774\ud6c4\uc5d0\ub294 \uc8fc\ub85c \ubcf4\uceec \ud2b8\ub808\uc774\ub108\uc758 \uc804\ub2f4 \ud558\uc5d0 \ubc1c\uc131 \uc5f0\uc2b5\uacfc \uac00\ucc3d\ub825 \ud5a5\uc0c1\uc5d0 \uc9d1\uc911\uc801\uc778 \ud6c8\ub828\uc744 \ubc1b\uc558\ub2e4. \ub098\uce74\ubaa8\ub9ac\ub294 \ubcf8\ub798 \ub367\ub2c8\uac00 \uc788\uc5c8\uc73c\ub098 \ub370\ubdd4 \uc804 \uc18c\uc18d\uc0ac\uc778 \ucf04\uc628\uc774 \uc544\uc774\ub3cc\ub2e4\uc6b0\ub824\uba74 \ub367\ub2c8\ubd80\ud130 \uace0\ub974\uac8c \ud574\uc57c\ud55c\ub2e4 \ud558\uc5ec \uc774\ub97c \uad50\uc815\ud558\uae30\ub3c4 \ud588\ub2e4."} {"question": "\uc99d\uad6d\ubc88\uc774 \ud760\ucc28\ub300\uc2e0\uc5d0 \uc784\uba85\ub41c \ub144\ub3c4\ub294?", "paragraph": "\uc911\uc559\uc815\ubd80\ub0b4\uc5d0\uc11c \uc591\ubb34\uc6b4\ub3d9\uc758 \ucd94\uc9c4\uc138\ub825\uc740 \ubcf4\uc218\uc801 \uad00\ub8cc\uc758 \ub04a\uc784\uc5c6\ub294 \ube44\ud310\uacfc \uacac\uc81c\ub97c \ubc1b\uc558\ub2e4. \uadf8\ub7ec\ub098 \uc99d\uad6d\ubc88,\uc774\ud64d\uc7a5,\uc88c\uc885\ub2f9 \ub4f1\uc740 \ud0dc\ud3c9\ucc9c\uad6d\uc758 \ub09c\uc744 \uc9c4\uc555\ud558\ub294 \uacfc\uc815\uc5d0\uc11c \uc815\uce58\uad70\uc0ac\uc801 \uc2e4\ub825\uc744 \uc778\uc815\ubc1b\uc544 \uc801\uadf9\uc801\uc778 \uc591\ubb34\uc6b4\ub3d9\uc744 \ucd94\uc9c4\uc138\ub825\uc73c\ub85c \ubc1c\ud0c1\ub418\uc5c8\ub2e4. \uc99d\uad6d\ubc88\uc740 \ud638\ub0a8 \uc131\ucd9c\uc2e0\uc758 \ud559\uc790\ub85c \uc0c1\uad70\uc744 \uc870\uc9c1\ud558\uc5ec \uc5c4\uaca9\ud55c \ud6c8\ub828\uacfc \uc11c\uc591\uc2dd \ubb34\uae30\ub85c \ubb34\uc7a5\ud558\uc5ec \ud0dc\ud3c9\ucc9c\uad6d \uc9c4\uc555\uc5d0 \uacb0\uc815\uc801\uc778 \uacf5\uc744 \uc138\uc6e0\ub2e4. \uadf8\ub294 \uc591\uad11\ucd1d\ub3c5\uacfc \ud760\ucc28\ub300\uc2e0\uc5d0 1860\ub144 \uc784\uba85\ub41c \ud6c4 1872\ub144 \uc0ac\ub9dd\ud560 \ub54c\uae4c\uc9c0 \uc911\uc559\uc815\ubd80\uc758 \uc815\ucc45\uacb0\uc815\uc5d0 \ucc38\uc5ec\ud558\uc5ec \ucd08\uae30 \uc591\ubb34\uc6b4\ub3d9\uc758 \ud575\uc2ec\uc801\uc778 \uc5ed\ud560\uc744 \ub2f4\ub2f9\ud558\uc600\ub2e4. \uc88c\uc885\ub2f9\ub3c4 \ud638\ub0a8 \uc131 \ucd9c\uc2e0\uc73c\ub85c \ud0dc\ud3c9\ucc9c\uad6d\uc758 \uc9c4\uc555\uc5d0 \uacf5\uc744 \uc138\uc6e0\uace0 1881\ub144,1884\ub144 \ub450\ucc28\ub840 \uad70\uae30\ub300\uc2e0\uc73c\ub85c \ubc1c\ud0c1\ub418\uc5b4 \uad70\uc218\uacf5\uc7a5\uc744 \uac74\uc124\ud558\ub294\ub370 \uc8fc\ub3c4\uc801 \uc5ed\ud560\uc744 \ud558\uc600\ub2e4.", "answer": "1860\ub144", "sentence": "\uadf8\ub294 \uc591\uad11\ucd1d\ub3c5\uacfc \ud760\ucc28\ub300\uc2e0\uc5d0 1860\ub144 \uc784\uba85\ub41c \ud6c4 1872\ub144 \uc0ac\ub9dd\ud560 \ub54c\uae4c\uc9c0 \uc911\uc559\uc815\ubd80\uc758 \uc815\ucc45\uacb0\uc815\uc5d0 \ucc38\uc5ec\ud558\uc5ec \ucd08\uae30 \uc591\ubb34\uc6b4\ub3d9\uc758 \ud575\uc2ec\uc801\uc778 \uc5ed\ud560\uc744 \ub2f4\ub2f9\ud558\uc600\ub2e4.", "paragraph_sentence": "\uc911\uc559\uc815\ubd80\ub0b4\uc5d0\uc11c \uc591\ubb34\uc6b4\ub3d9\uc758 \ucd94\uc9c4\uc138\ub825\uc740 \ubcf4\uc218\uc801 \uad00\ub8cc\uc758 \ub04a\uc784\uc5c6\ub294 \ube44\ud310\uacfc \uacac\uc81c\ub97c \ubc1b\uc558\ub2e4. \uadf8\ub7ec\ub098 \uc99d\uad6d\ubc88,\uc774\ud64d\uc7a5,\uc88c\uc885\ub2f9 \ub4f1\uc740 \ud0dc\ud3c9\ucc9c\uad6d\uc758 \ub09c\uc744 \uc9c4\uc555\ud558\ub294 \uacfc\uc815\uc5d0\uc11c \uc815\uce58\uad70\uc0ac\uc801 \uc2e4\ub825\uc744 \uc778\uc815\ubc1b\uc544 \uc801\uadf9\uc801\uc778 \uc591\ubb34\uc6b4\ub3d9\uc744 \ucd94\uc9c4\uc138\ub825\uc73c\ub85c \ubc1c\ud0c1\ub418\uc5c8\ub2e4. \uc99d\uad6d\ubc88\uc740 \ud638\ub0a8 \uc131\ucd9c\uc2e0\uc758 \ud559\uc790\ub85c \uc0c1\uad70\uc744 \uc870\uc9c1\ud558\uc5ec \uc5c4\uaca9\ud55c \ud6c8\ub828\uacfc \uc11c\uc591\uc2dd \ubb34\uae30\ub85c \ubb34\uc7a5\ud558\uc5ec \ud0dc\ud3c9\ucc9c\uad6d \uc9c4\uc555\uc5d0 \uacb0\uc815\uc801\uc778 \uacf5\uc744 \uc138\uc6e0\ub2e4. \uadf8\ub294 \uc591\uad11\ucd1d\ub3c5\uacfc \ud760\ucc28\ub300\uc2e0\uc5d0 1860\ub144 \uc784\uba85\ub41c \ud6c4 1872\ub144 \uc0ac\ub9dd\ud560 \ub54c\uae4c\uc9c0 \uc911\uc559\uc815\ubd80\uc758 \uc815\ucc45\uacb0\uc815\uc5d0 \ucc38\uc5ec\ud558\uc5ec \ucd08\uae30 \uc591\ubb34\uc6b4\ub3d9\uc758 \ud575\uc2ec\uc801\uc778 \uc5ed\ud560\uc744 \ub2f4\ub2f9\ud558\uc600\ub2e4. \uc88c\uc885\ub2f9\ub3c4 \ud638\ub0a8 \uc131 \ucd9c\uc2e0\uc73c\ub85c \ud0dc\ud3c9\ucc9c\uad6d\uc758 \uc9c4\uc555\uc5d0 \uacf5\uc744 \uc138\uc6e0\uace0 1881\ub144,1884\ub144 \ub450\ucc28\ub840 \uad70\uae30\ub300\uc2e0\uc73c\ub85c \ubc1c\ud0c1\ub418\uc5b4 \uad70\uc218\uacf5\uc7a5\uc744 \uac74\uc124\ud558\ub294\ub370 \uc8fc\ub3c4\uc801 \uc5ed\ud560\uc744 \ud558\uc600\ub2e4.", "paragraph_answer": "\uc911\uc559\uc815\ubd80\ub0b4\uc5d0\uc11c \uc591\ubb34\uc6b4\ub3d9\uc758 \ucd94\uc9c4\uc138\ub825\uc740 \ubcf4\uc218\uc801 \uad00\ub8cc\uc758 \ub04a\uc784\uc5c6\ub294 \ube44\ud310\uacfc \uacac\uc81c\ub97c \ubc1b\uc558\ub2e4. \uadf8\ub7ec\ub098 \uc99d\uad6d\ubc88,\uc774\ud64d\uc7a5,\uc88c\uc885\ub2f9 \ub4f1\uc740 \ud0dc\ud3c9\ucc9c\uad6d\uc758 \ub09c\uc744 \uc9c4\uc555\ud558\ub294 \uacfc\uc815\uc5d0\uc11c \uc815\uce58\uad70\uc0ac\uc801 \uc2e4\ub825\uc744 \uc778\uc815\ubc1b\uc544 \uc801\uadf9\uc801\uc778 \uc591\ubb34\uc6b4\ub3d9\uc744 \ucd94\uc9c4\uc138\ub825\uc73c\ub85c \ubc1c\ud0c1\ub418\uc5c8\ub2e4. \uc99d\uad6d\ubc88\uc740 \ud638\ub0a8 \uc131\ucd9c\uc2e0\uc758 \ud559\uc790\ub85c \uc0c1\uad70\uc744 \uc870\uc9c1\ud558\uc5ec \uc5c4\uaca9\ud55c \ud6c8\ub828\uacfc \uc11c\uc591\uc2dd \ubb34\uae30\ub85c \ubb34\uc7a5\ud558\uc5ec \ud0dc\ud3c9\ucc9c\uad6d \uc9c4\uc555\uc5d0 \uacb0\uc815\uc801\uc778 \uacf5\uc744 \uc138\uc6e0\ub2e4. \uadf8\ub294 \uc591\uad11\ucd1d\ub3c5\uacfc \ud760\ucc28\ub300\uc2e0\uc5d0 1860\ub144 \uc784\uba85\ub41c \ud6c4 1872\ub144 \uc0ac\ub9dd\ud560 \ub54c\uae4c\uc9c0 \uc911\uc559\uc815\ubd80\uc758 \uc815\ucc45\uacb0\uc815\uc5d0 \ucc38\uc5ec\ud558\uc5ec \ucd08\uae30 \uc591\ubb34\uc6b4\ub3d9\uc758 \ud575\uc2ec\uc801\uc778 \uc5ed\ud560\uc744 \ub2f4\ub2f9\ud558\uc600\ub2e4. \uc88c\uc885\ub2f9\ub3c4 \ud638\ub0a8 \uc131 \ucd9c\uc2e0\uc73c\ub85c \ud0dc\ud3c9\ucc9c\uad6d\uc758 \uc9c4\uc555\uc5d0 \uacf5\uc744 \uc138\uc6e0\uace0 1881\ub144,1884\ub144 \ub450\ucc28\ub840 \uad70\uae30\ub300\uc2e0\uc73c\ub85c \ubc1c\ud0c1\ub418\uc5b4 \uad70\uc218\uacf5\uc7a5\uc744 \uac74\uc124\ud558\ub294\ub370 \uc8fc\ub3c4\uc801 \uc5ed\ud560\uc744 \ud558\uc600\ub2e4.", "sentence_answer": "\uadf8\ub294 \uc591\uad11\ucd1d\ub3c5\uacfc \ud760\ucc28\ub300\uc2e0\uc5d0 1860\ub144 \uc784\uba85\ub41c \ud6c4 1872\ub144 \uc0ac\ub9dd\ud560 \ub54c\uae4c\uc9c0 \uc911\uc559\uc815\ubd80\uc758 \uc815\ucc45\uacb0\uc815\uc5d0 \ucc38\uc5ec\ud558\uc5ec \ucd08\uae30 \uc591\ubb34\uc6b4\ub3d9\uc758 \ud575\uc2ec\uc801\uc778 \uc5ed\ud560\uc744 \ub2f4\ub2f9\ud558\uc600\ub2e4.", "paragraph_id": "6522804-1-1", "paragraph_question": "question: \uc99d\uad6d\ubc88\uc774 \ud760\ucc28\ub300\uc2e0\uc5d0 \uc784\uba85\ub41c \ub144\ub3c4\ub294?, context: \uc911\uc559\uc815\ubd80\ub0b4\uc5d0\uc11c \uc591\ubb34\uc6b4\ub3d9\uc758 \ucd94\uc9c4\uc138\ub825\uc740 \ubcf4\uc218\uc801 \uad00\ub8cc\uc758 \ub04a\uc784\uc5c6\ub294 \ube44\ud310\uacfc \uacac\uc81c\ub97c \ubc1b\uc558\ub2e4. \uadf8\ub7ec\ub098 \uc99d\uad6d\ubc88,\uc774\ud64d\uc7a5,\uc88c\uc885\ub2f9 \ub4f1\uc740 \ud0dc\ud3c9\ucc9c\uad6d\uc758 \ub09c\uc744 \uc9c4\uc555\ud558\ub294 \uacfc\uc815\uc5d0\uc11c \uc815\uce58\uad70\uc0ac\uc801 \uc2e4\ub825\uc744 \uc778\uc815\ubc1b\uc544 \uc801\uadf9\uc801\uc778 \uc591\ubb34\uc6b4\ub3d9\uc744 \ucd94\uc9c4\uc138\ub825\uc73c\ub85c \ubc1c\ud0c1\ub418\uc5c8\ub2e4. \uc99d\uad6d\ubc88\uc740 \ud638\ub0a8 \uc131\ucd9c\uc2e0\uc758 \ud559\uc790\ub85c \uc0c1\uad70\uc744 \uc870\uc9c1\ud558\uc5ec \uc5c4\uaca9\ud55c \ud6c8\ub828\uacfc \uc11c\uc591\uc2dd \ubb34\uae30\ub85c \ubb34\uc7a5\ud558\uc5ec \ud0dc\ud3c9\ucc9c\uad6d \uc9c4\uc555\uc5d0 \uacb0\uc815\uc801\uc778 \uacf5\uc744 \uc138\uc6e0\ub2e4. \uadf8\ub294 \uc591\uad11\ucd1d\ub3c5\uacfc \ud760\ucc28\ub300\uc2e0\uc5d0 1860\ub144 \uc784\uba85\ub41c \ud6c4 1872\ub144 \uc0ac\ub9dd\ud560 \ub54c\uae4c\uc9c0 \uc911\uc559\uc815\ubd80\uc758 \uc815\ucc45\uacb0\uc815\uc5d0 \ucc38\uc5ec\ud558\uc5ec \ucd08\uae30 \uc591\ubb34\uc6b4\ub3d9\uc758 \ud575\uc2ec\uc801\uc778 \uc5ed\ud560\uc744 \ub2f4\ub2f9\ud558\uc600\ub2e4. \uc88c\uc885\ub2f9\ub3c4 \ud638\ub0a8 \uc131 \ucd9c\uc2e0\uc73c\ub85c \ud0dc\ud3c9\ucc9c\uad6d\uc758 \uc9c4\uc555\uc5d0 \uacf5\uc744 \uc138\uc6e0\uace0 1881\ub144,1884\ub144 \ub450\ucc28\ub840 \uad70\uae30\ub300\uc2e0\uc73c\ub85c \ubc1c\ud0c1\ub418\uc5b4 \uad70\uc218\uacf5\uc7a5\uc744 \uac74\uc124\ud558\ub294\ub370 \uc8fc\ub3c4\uc801 \uc5ed\ud560\uc744 \ud558\uc600\ub2e4."} {"question": "\uc21c\uc885\uc758 \ub3d9\uc0dd\uc774\ub77c\ub294 \uc9c0\uc704\ub85c \uaca9\ud558\ub41c \uc0ac\ub78c\uc740 \ub204\uad6c\uc778\uac00?", "paragraph": "1906\ub144 12\uc6d4 7\uc77c\ubd80\ud130 \ud669\uc871\uacfc \uadc0\uc871 \uc790\uc81c\ub97c \uad50\uc721\ud558\uae30 \uc704\ud574 \uc124\uce58\ub41c \uadfc\ub300\uc2dd \uad50\uc721\uae30\uad00 \uc218\ud559\uc6d0\uc5d0\uc11c \uac15\ud559\uc744 \ubc1b\uae30 \uc2dc\uc791\ud558\uc600\uc73c\uba70, 1907\ub144 3\uc6d4 11\uc77c\uc5d0 \uacbd\uc6b4\uad81 \uc911\ud654\uc804\uc5d0\uc11c \uad00\ub840\uac00 \ud589\ud574\uc84c\ub2e4. \uac19\uc740 \ud574 8\uc6d4 7\uc77c\uc5d0 \ud0dc\ud669\uc81c \uace0\uc885\uc758 \uc870\uc11c\uc5d0 \ub530\ub77c \uc21c\uc885\uc758 \ud669\ud0dc\uc790\ub85c \uc774\uc740\uc774 \uacb0\uc815\ub418\uc5c8\uace0, 8\uc6d4 16\uc5d0 \ub300\ud55c\uc81c\uad6d \uc721\uad70 \ubcf4\ubcd1 \ucc38\uc704\uc5d0 \uc784\uba85\ub418\uc5c8\uc73c\uba70, 9\uc6d4 7\uc77c\uc5d0 \ud669\ud0dc\uc790\uc5d0 \ucc45\ubd09\ub418\uc5c8\ub2e4. \uc774\ub294 \uc790\uc2e0\uc758 \uc655\uc704\ub97c \uacc4\uc18d \uc704\ud611\ud588\ub358 \uc774\uc900\uc6a9\uacfc \uc758\uce5c\uc655 \uc774\uac15\uc744 \uacac\uc81c\ud558\ub824\ub294 \uace0\uc885\uc758 \uc758\ub3c4\uc640 \uc774\uc900\uc6a9\ud30c\uc640 \uc774\uac15\ud30c\uac00 \ub4dd\uc138\ud558\uba74 \uc790\uc2e0\uc758 \uc2e4\uad8c\uc774 \uc7a0\uc2dd\ub420 \uac83\uc744 \uc6b0\ub824\ud55c \uc774\uc644\uc6a9\uc758 \uc815\ub7b5\uc774 \ub9de\uc544\ub5a8\uc5b4\uc9c4 \uacb0\uacfc\uc600\ub2e4. \uc774\ub85c\uc368 \uc7a5\uae30\uac04 \ud574\uc678 \ub9dd\uba85\uc0dd\ud65c \uc911\uc5d0 \ub04a\uc784\uc5c6\uc774 \uc7a0\uc7ac\uc801 \uc655\uc704\uacc4\uc2b9\uc790\ub85c\uc11c \ub300\uc6b0\uc640 \uc8fc\ubaa9\uacfc \uacac\uc81c\ub97c \ubc1b\uc544\uc654\ub358 \uc774\uc900\uc6a9\uc740 \ud669\ud0dc\uc790\uc758 \uc885\ud615, \uc774\uac15\uc740 \uc21c\uc885\uc758 \ub3d9\uc0dd\uc774\ub77c\ub294 \uc9c0\uc704\ub85c \uaca9\ud558\ub418\uc5c8\ub2e4.", "answer": "\uc774\uac15", "sentence": "\uc774\ub294 \uc790\uc2e0\uc758 \uc655\uc704\ub97c \uacc4\uc18d \uc704\ud611\ud588\ub358 \uc774\uc900\uc6a9\uacfc \uc758\uce5c\uc655 \uc774\uac15 \uc744 \uacac\uc81c\ud558\ub824\ub294 \uace0\uc885\uc758 \uc758\ub3c4\uc640 \uc774\uc900\uc6a9\ud30c\uc640 \uc774\uac15\ud30c\uac00 \ub4dd\uc138\ud558\uba74 \uc790\uc2e0\uc758 \uc2e4\uad8c\uc774 \uc7a0\uc2dd\ub420 \uac83\uc744 \uc6b0\ub824\ud55c \uc774\uc644\uc6a9\uc758 \uc815\ub7b5\uc774 \ub9de\uc544\ub5a8\uc5b4\uc9c4 \uacb0\uacfc\uc600\ub2e4.", "paragraph_sentence": "1906\ub144 12\uc6d4 7\uc77c\ubd80\ud130 \ud669\uc871\uacfc \uadc0\uc871 \uc790\uc81c\ub97c \uad50\uc721\ud558\uae30 \uc704\ud574 \uc124\uce58\ub41c \uadfc\ub300\uc2dd \uad50\uc721\uae30\uad00 \uc218\ud559\uc6d0\uc5d0\uc11c \uac15\ud559\uc744 \ubc1b\uae30 \uc2dc\uc791\ud558\uc600\uc73c\uba70, 1907\ub144 3\uc6d4 11\uc77c\uc5d0 \uacbd\uc6b4\uad81 \uc911\ud654\uc804\uc5d0\uc11c \uad00\ub840\uac00 \ud589\ud574\uc84c\ub2e4. \uac19\uc740 \ud574 8\uc6d4 7\uc77c\uc5d0 \ud0dc\ud669\uc81c \uace0\uc885\uc758 \uc870\uc11c\uc5d0 \ub530\ub77c \uc21c\uc885\uc758 \ud669\ud0dc\uc790\ub85c \uc774\uc740\uc774 \uacb0\uc815\ub418\uc5c8\uace0, 8\uc6d4 16\uc5d0 \ub300\ud55c\uc81c\uad6d \uc721\uad70 \ubcf4\ubcd1 \ucc38\uc704\uc5d0 \uc784\uba85\ub418\uc5c8\uc73c\uba70, 9\uc6d4 7\uc77c\uc5d0 \ud669\ud0dc\uc790\uc5d0 \ucc45\ubd09\ub418\uc5c8\ub2e4. \uc774\ub294 \uc790\uc2e0\uc758 \uc655\uc704\ub97c \uacc4\uc18d \uc704\ud611\ud588\ub358 \uc774\uc900\uc6a9\uacfc \uc758\uce5c\uc655 \uc774\uac15 \uc744 \uacac\uc81c\ud558\ub824\ub294 \uace0\uc885\uc758 \uc758\ub3c4\uc640 \uc774\uc900\uc6a9\ud30c\uc640 \uc774\uac15\ud30c\uac00 \ub4dd\uc138\ud558\uba74 \uc790\uc2e0\uc758 \uc2e4\uad8c\uc774 \uc7a0\uc2dd\ub420 \uac83\uc744 \uc6b0\ub824\ud55c \uc774\uc644\uc6a9\uc758 \uc815\ub7b5\uc774 \ub9de\uc544\ub5a8\uc5b4\uc9c4 \uacb0\uacfc\uc600\ub2e4. \uc774\ub85c\uc368 \uc7a5\uae30\uac04 \ud574\uc678 \ub9dd\uba85\uc0dd\ud65c \uc911\uc5d0 \ub04a\uc784\uc5c6\uc774 \uc7a0\uc7ac\uc801 \uc655\uc704\uacc4\uc2b9\uc790\ub85c\uc11c \ub300\uc6b0\uc640 \uc8fc\ubaa9\uacfc \uacac\uc81c\ub97c \ubc1b\uc544\uc654\ub358 \uc774\uc900\uc6a9\uc740 \ud669\ud0dc\uc790\uc758 \uc885\ud615, \uc774\uac15\uc740 \uc21c\uc885\uc758 \ub3d9\uc0dd\uc774\ub77c\ub294 \uc9c0\uc704\ub85c \uaca9\ud558\ub418\uc5c8\ub2e4.", "paragraph_answer": "1906\ub144 12\uc6d4 7\uc77c\ubd80\ud130 \ud669\uc871\uacfc \uadc0\uc871 \uc790\uc81c\ub97c \uad50\uc721\ud558\uae30 \uc704\ud574 \uc124\uce58\ub41c \uadfc\ub300\uc2dd \uad50\uc721\uae30\uad00 \uc218\ud559\uc6d0\uc5d0\uc11c \uac15\ud559\uc744 \ubc1b\uae30 \uc2dc\uc791\ud558\uc600\uc73c\uba70, 1907\ub144 3\uc6d4 11\uc77c\uc5d0 \uacbd\uc6b4\uad81 \uc911\ud654\uc804\uc5d0\uc11c \uad00\ub840\uac00 \ud589\ud574\uc84c\ub2e4. \uac19\uc740 \ud574 8\uc6d4 7\uc77c\uc5d0 \ud0dc\ud669\uc81c \uace0\uc885\uc758 \uc870\uc11c\uc5d0 \ub530\ub77c \uc21c\uc885\uc758 \ud669\ud0dc\uc790\ub85c \uc774\uc740\uc774 \uacb0\uc815\ub418\uc5c8\uace0, 8\uc6d4 16\uc5d0 \ub300\ud55c\uc81c\uad6d \uc721\uad70 \ubcf4\ubcd1 \ucc38\uc704\uc5d0 \uc784\uba85\ub418\uc5c8\uc73c\uba70, 9\uc6d4 7\uc77c\uc5d0 \ud669\ud0dc\uc790\uc5d0 \ucc45\ubd09\ub418\uc5c8\ub2e4. \uc774\ub294 \uc790\uc2e0\uc758 \uc655\uc704\ub97c \uacc4\uc18d \uc704\ud611\ud588\ub358 \uc774\uc900\uc6a9\uacfc \uc758\uce5c\uc655 \uc774\uac15 \uc744 \uacac\uc81c\ud558\ub824\ub294 \uace0\uc885\uc758 \uc758\ub3c4\uc640 \uc774\uc900\uc6a9\ud30c\uc640 \uc774\uac15\ud30c\uac00 \ub4dd\uc138\ud558\uba74 \uc790\uc2e0\uc758 \uc2e4\uad8c\uc774 \uc7a0\uc2dd\ub420 \uac83\uc744 \uc6b0\ub824\ud55c \uc774\uc644\uc6a9\uc758 \uc815\ub7b5\uc774 \ub9de\uc544\ub5a8\uc5b4\uc9c4 \uacb0\uacfc\uc600\ub2e4. \uc774\ub85c\uc368 \uc7a5\uae30\uac04 \ud574\uc678 \ub9dd\uba85\uc0dd\ud65c \uc911\uc5d0 \ub04a\uc784\uc5c6\uc774 \uc7a0\uc7ac\uc801 \uc655\uc704\uacc4\uc2b9\uc790\ub85c\uc11c \ub300\uc6b0\uc640 \uc8fc\ubaa9\uacfc \uacac\uc81c\ub97c \ubc1b\uc544\uc654\ub358 \uc774\uc900\uc6a9\uc740 \ud669\ud0dc\uc790\uc758 \uc885\ud615, \uc774\uac15\uc740 \uc21c\uc885\uc758 \ub3d9\uc0dd\uc774\ub77c\ub294 \uc9c0\uc704\ub85c \uaca9\ud558\ub418\uc5c8\ub2e4.", "sentence_answer": "\uc774\ub294 \uc790\uc2e0\uc758 \uc655\uc704\ub97c \uacc4\uc18d \uc704\ud611\ud588\ub358 \uc774\uc900\uc6a9\uacfc \uc758\uce5c\uc655 \uc774\uac15 \uc744 \uacac\uc81c\ud558\ub824\ub294 \uace0\uc885\uc758 \uc758\ub3c4\uc640 \uc774\uc900\uc6a9\ud30c\uc640 \uc774\uac15\ud30c\uac00 \ub4dd\uc138\ud558\uba74 \uc790\uc2e0\uc758 \uc2e4\uad8c\uc774 \uc7a0\uc2dd\ub420 \uac83\uc744 \uc6b0\ub824\ud55c \uc774\uc644\uc6a9\uc758 \uc815\ub7b5\uc774 \ub9de\uc544\ub5a8\uc5b4\uc9c4 \uacb0\uacfc\uc600\ub2e4.", "paragraph_id": "5836625-2-2", "paragraph_question": "question: \uc21c\uc885\uc758 \ub3d9\uc0dd\uc774\ub77c\ub294 \uc9c0\uc704\ub85c \uaca9\ud558\ub41c \uc0ac\ub78c\uc740 \ub204\uad6c\uc778\uac00?, context: 1906\ub144 12\uc6d4 7\uc77c\ubd80\ud130 \ud669\uc871\uacfc \uadc0\uc871 \uc790\uc81c\ub97c \uad50\uc721\ud558\uae30 \uc704\ud574 \uc124\uce58\ub41c \uadfc\ub300\uc2dd \uad50\uc721\uae30\uad00 \uc218\ud559\uc6d0\uc5d0\uc11c \uac15\ud559\uc744 \ubc1b\uae30 \uc2dc\uc791\ud558\uc600\uc73c\uba70, 1907\ub144 3\uc6d4 11\uc77c\uc5d0 \uacbd\uc6b4\uad81 \uc911\ud654\uc804\uc5d0\uc11c \uad00\ub840\uac00 \ud589\ud574\uc84c\ub2e4. \uac19\uc740 \ud574 8\uc6d4 7\uc77c\uc5d0 \ud0dc\ud669\uc81c \uace0\uc885\uc758 \uc870\uc11c\uc5d0 \ub530\ub77c \uc21c\uc885\uc758 \ud669\ud0dc\uc790\ub85c \uc774\uc740\uc774 \uacb0\uc815\ub418\uc5c8\uace0, 8\uc6d4 16\uc5d0 \ub300\ud55c\uc81c\uad6d \uc721\uad70 \ubcf4\ubcd1 \ucc38\uc704\uc5d0 \uc784\uba85\ub418\uc5c8\uc73c\uba70, 9\uc6d4 7\uc77c\uc5d0 \ud669\ud0dc\uc790\uc5d0 \ucc45\ubd09\ub418\uc5c8\ub2e4. \uc774\ub294 \uc790\uc2e0\uc758 \uc655\uc704\ub97c \uacc4\uc18d \uc704\ud611\ud588\ub358 \uc774\uc900\uc6a9\uacfc \uc758\uce5c\uc655 \uc774\uac15\uc744 \uacac\uc81c\ud558\ub824\ub294 \uace0\uc885\uc758 \uc758\ub3c4\uc640 \uc774\uc900\uc6a9\ud30c\uc640 \uc774\uac15\ud30c\uac00 \ub4dd\uc138\ud558\uba74 \uc790\uc2e0\uc758 \uc2e4\uad8c\uc774 \uc7a0\uc2dd\ub420 \uac83\uc744 \uc6b0\ub824\ud55c \uc774\uc644\uc6a9\uc758 \uc815\ub7b5\uc774 \ub9de\uc544\ub5a8\uc5b4\uc9c4 \uacb0\uacfc\uc600\ub2e4. \uc774\ub85c\uc368 \uc7a5\uae30\uac04 \ud574\uc678 \ub9dd\uba85\uc0dd\ud65c \uc911\uc5d0 \ub04a\uc784\uc5c6\uc774 \uc7a0\uc7ac\uc801 \uc655\uc704\uacc4\uc2b9\uc790\ub85c\uc11c \ub300\uc6b0\uc640 \uc8fc\ubaa9\uacfc \uacac\uc81c\ub97c \ubc1b\uc544\uc654\ub358 \uc774\uc900\uc6a9\uc740 \ud669\ud0dc\uc790\uc758 \uc885\ud615, \uc774\uac15\uc740 \uc21c\uc885\uc758 \ub3d9\uc0dd\uc774\ub77c\ub294 \uc9c0\uc704\ub85c \uaca9\ud558\ub418\uc5c8\ub2e4."} {"question": "\uc6b0\uc5d0\uc2a4\uae30 \uc820\uc288\uc758 \ub09c\uc774 \uc77c\uc5b4\ub09c \uac83\uc740 \uba87 \ub144\uc778\uac00?", "paragraph": "1410\ub144(\uc624\uc5d0\uc774 17\ub144)\uc5d0\ub294 \ub0a8\uc870 \ucd5c\ud6c4\uc758 \ucc9c\ud669\uc774\uc5c8\ub358 \uace0\uce74\uba54\uc57c\ub9c8 \ucc9c\ud669\uc774 \uc694\uc2dc\ub178\ub85c \ucd9c\ubd84\ud558\uc600\ub2e4(1416\ub144 \uadc0\uacbd). 1414\ub144(\uc624\uc5d0\uc774 21\ub144)\uc5d0 \uc774\uc5d0 \ud638\uc751\ud55c \uae30\ud0c0\ubc14\ud0c0\ucf00 \ubbf8\uc4f0\ub9c8\uc0ac(\u5317\u7560\u6e80\u96c5)\uac00 1412\ub144(\uc624\uc5d0\uc774 19\ub144)\uc758 \uc1fc\ucf54 \ucc9c\ud669\uc758 \uc989\uc704\uc5d0 \ubd88\ubcf5\ud558\uc5ec \uc591\ud1b5 \uc9c8\ub9bd(\u4e21\u7d71\u8fed\u7acb)\uc758 \uc57d\uc18d\uc744 \uc9c0\ud0ac \uac83\uc744 \uc694\uad6c\ud558\uba70 \ubc18\ub780\uc744 \uc77c\uc73c\ucf30\uc73c\ub098, \uae08\uc138 \ud654\ud574\ud558\uc600\ub2e4. \uac19\uc740 \ud574\uc5d0\ub294 \uc694\uc2dc\ubaa8\uce58\ub97c \uc7a5\uae30\uac04 \uc9c0\ud0f1\ud588\ub358 \uc219\ub85c \uc2dc\ubc14 \uc694\uc2dc\uc720\ud0a4\uc758 \uc870\uce74 \uc2dc\ubc14 \ubbf8\uc4f0\ud0c0\ub124(\u65af\u6ce2\u6e80\u7a2e)\uac00 \uc694\uc2dc\ubaa8\uce58\uc758 \ub178\uc5ec\uc6c0\uc744 \uc0ac \uace0\uc57c \uc0b0(\u9ad8\u91ce\u5c71)\uc73c\ub85c \uc740\ud1f4\ud558\ub294 \uc0ac\uac74\uc774 \ubc8c\uc5b4\uc84c\ub2e4. 1416\ub144(\uc624\uc5d0\uc774 23\ub144)\uc5d0\ub294 \uac04\ud1a0 \uc9c0\ubc29\uc5d0\uc11c \uc6b0\uc5d0\uc2a4\uae30 \uc820\uc288\uc758 \ub09c(\u4e0a\u6749\u7985\u79c0\u306e\u4e71)\uc774 \uc77c\uc5b4\ub098 \uc774\uc5d0 \uad00\uc5ec\ud55c \ub3d9\uc0dd \uc694\uc2dc\uc4f0\uad6c\ub97c \uc1fc\ucf54\ucfe0\uc9c0(\u76f8\u56fd\u5bfa)\uc5d0 \uc720\ud3d0\ud558\uace0, 2\ub144 \ub4a4\uc778 1418\ub144(\uc624\uc5d0\uc774 25\ub144)\uc5d0 \uc0b4\ud574\ud588\ub2e4. \ub354\uc6b1\uc774 \uadf8 \uacfc\uc815\uc5d0\uc11c \uc694\uc2dc\ubaa8\uce58 \uce21\uadfc \ub3c4\uac00\uc2dc \ubbf8\uc4f0\ub098\ub9ac(\u5bcc\u6a2b\u6e80\u6210)\ub294 \ud638\uc18c\uce74\uc640 \ubbf8\uc4f0\ubaa8\ud1a0(\u7d30\u5ddd\u6e80\u5143)\uc640 \uc2dc\ubc14 \uc694\uc2dc\uc2dc\uac8c(\u65af\u6ce2\u7fa9\u91cd)\uac00 \uc694\uc2dc\uc4f0\uad6c\uc5d0\uac8c \uac00\ub2f4\ud588\ub2e4\uace0 \uace0\ubc1c\ud558\uc600\ub2e4. \uc77c\uc124\uc5d0\ub294 \uc774 \uace0\ubc1c\uc740 \uc694\uc2dc\ubaa8\uce58\uc758 \uc758\ud5a5\uc5d0 \ub530\ub978 \uac83\uc73c\ub85c \uc720\ub825 \uc288\uace0\uc758 \ubc1c\uc5b8\ub825\uc744 \uc904\uc774\ub824\uace0 \uc694\uc2dc\ubaa8\uce58\uac00 \uc8fc\ub3c4\ud55c \ub9c9\uc815\uc758 \ud655\ub9bd\uc744 \uc9c0\ud5a5\ud55c \uac83\uc774\ub77c\uace0 \ud55c\ub2e4. \uadf8\ub7ec\ub098 \ucd5c\uc885\uc801\uc73c\ub85c\ub294 \ud638\uc18c\uce74\uc640 \ub4f1\uc758 \uace0\ubc1c\uc73c\ub85c \ub3c4\uac00\uc2dc\uac00 \ucd94\ubc29\ub2f9\ud558\uace0 \ud6c4\uc5d0 \uc0b4\ud574\ub41c\ub2e4.", "answer": "1416\ub144", "sentence": "1410\ub144(\uc624\uc5d0\uc774 17\ub144)\uc5d0\ub294 \ub0a8\uc870 \ucd5c\ud6c4\uc758 \ucc9c\ud669\uc774\uc5c8\ub358 \uace0\uce74\uba54\uc57c\ub9c8 \ucc9c\ud669\uc774 \uc694\uc2dc\ub178\ub85c \ucd9c\ubd84\ud558\uc600\ub2e4( 1416\ub144 \uadc0\uacbd)", "paragraph_sentence": " 1410\ub144(\uc624\uc5d0\uc774 17\ub144)\uc5d0\ub294 \ub0a8\uc870 \ucd5c\ud6c4\uc758 \ucc9c\ud669\uc774\uc5c8\ub358 \uace0\uce74\uba54\uc57c\ub9c8 \ucc9c\ud669\uc774 \uc694\uc2dc\ub178\ub85c \ucd9c\ubd84\ud558\uc600\ub2e4( 1416\ub144 \uadc0\uacbd) . 1414\ub144(\uc624\uc5d0\uc774 21\ub144)\uc5d0 \uc774\uc5d0 \ud638\uc751\ud55c \uae30\ud0c0\ubc14\ud0c0\ucf00 \ubbf8\uc4f0\ub9c8\uc0ac(\u5317\u7560\u6e80\u96c5)\uac00 1412\ub144(\uc624\uc5d0\uc774 19\ub144)\uc758 \uc1fc\ucf54 \ucc9c\ud669\uc758 \uc989\uc704\uc5d0 \ubd88\ubcf5\ud558\uc5ec \uc591\ud1b5 \uc9c8\ub9bd(\u4e21\u7d71\u8fed\u7acb)\uc758 \uc57d\uc18d\uc744 \uc9c0\ud0ac \uac83\uc744 \uc694\uad6c\ud558\uba70 \ubc18\ub780\uc744 \uc77c\uc73c\ucf30\uc73c\ub098, \uae08\uc138 \ud654\ud574\ud558\uc600\ub2e4. \uac19\uc740 \ud574\uc5d0\ub294 \uc694\uc2dc\ubaa8\uce58\ub97c \uc7a5\uae30\uac04 \uc9c0\ud0f1\ud588\ub358 \uc219\ub85c \uc2dc\ubc14 \uc694\uc2dc\uc720\ud0a4\uc758 \uc870\uce74 \uc2dc\ubc14 \ubbf8\uc4f0\ud0c0\ub124(\u65af\u6ce2\u6e80\u7a2e)\uac00 \uc694\uc2dc\ubaa8\uce58\uc758 \ub178\uc5ec\uc6c0\uc744 \uc0ac \uace0\uc57c \uc0b0(\u9ad8\u91ce\u5c71)\uc73c\ub85c \uc740\ud1f4\ud558\ub294 \uc0ac\uac74\uc774 \ubc8c\uc5b4\uc84c\ub2e4. 1416\ub144(\uc624\uc5d0\uc774 23\ub144)\uc5d0\ub294 \uac04\ud1a0 \uc9c0\ubc29\uc5d0\uc11c \uc6b0\uc5d0\uc2a4\uae30 \uc820\uc288\uc758 \ub09c(\u4e0a\u6749\u7985\u79c0\u306e\u4e71)\uc774 \uc77c\uc5b4\ub098 \uc774\uc5d0 \uad00\uc5ec\ud55c \ub3d9\uc0dd \uc694\uc2dc\uc4f0\uad6c\ub97c \uc1fc\ucf54\ucfe0\uc9c0(\u76f8\u56fd\u5bfa)\uc5d0 \uc720\ud3d0\ud558\uace0, 2\ub144 \ub4a4\uc778 1418\ub144(\uc624\uc5d0\uc774 25\ub144)\uc5d0 \uc0b4\ud574\ud588\ub2e4. \ub354\uc6b1\uc774 \uadf8 \uacfc\uc815\uc5d0\uc11c \uc694\uc2dc\ubaa8\uce58 \uce21\uadfc \ub3c4\uac00\uc2dc \ubbf8\uc4f0\ub098\ub9ac(\u5bcc\u6a2b\u6e80\u6210)\ub294 \ud638\uc18c\uce74\uc640 \ubbf8\uc4f0\ubaa8\ud1a0(\u7d30\u5ddd\u6e80\u5143)\uc640 \uc2dc\ubc14 \uc694\uc2dc\uc2dc\uac8c(\u65af\u6ce2\u7fa9\u91cd)\uac00 \uc694\uc2dc\uc4f0\uad6c\uc5d0\uac8c \uac00\ub2f4\ud588\ub2e4\uace0 \uace0\ubc1c\ud558\uc600\ub2e4. \uc77c\uc124\uc5d0\ub294 \uc774 \uace0\ubc1c\uc740 \uc694\uc2dc\ubaa8\uce58\uc758 \uc758\ud5a5\uc5d0 \ub530\ub978 \uac83\uc73c\ub85c \uc720\ub825 \uc288\uace0\uc758 \ubc1c\uc5b8\ub825\uc744 \uc904\uc774\ub824\uace0 \uc694\uc2dc\ubaa8\uce58\uac00 \uc8fc\ub3c4\ud55c \ub9c9\uc815\uc758 \ud655\ub9bd\uc744 \uc9c0\ud5a5\ud55c \uac83\uc774\ub77c\uace0 \ud55c\ub2e4. \uadf8\ub7ec\ub098 \ucd5c\uc885\uc801\uc73c\ub85c\ub294 \ud638\uc18c\uce74\uc640 \ub4f1\uc758 \uace0\ubc1c\uc73c\ub85c \ub3c4\uac00\uc2dc\uac00 \ucd94\ubc29\ub2f9\ud558\uace0 \ud6c4\uc5d0 \uc0b4\ud574\ub41c\ub2e4.", "paragraph_answer": "1410\ub144(\uc624\uc5d0\uc774 17\ub144)\uc5d0\ub294 \ub0a8\uc870 \ucd5c\ud6c4\uc758 \ucc9c\ud669\uc774\uc5c8\ub358 \uace0\uce74\uba54\uc57c\ub9c8 \ucc9c\ud669\uc774 \uc694\uc2dc\ub178\ub85c \ucd9c\ubd84\ud558\uc600\ub2e4( 1416\ub144 \uadc0\uacbd). 1414\ub144(\uc624\uc5d0\uc774 21\ub144)\uc5d0 \uc774\uc5d0 \ud638\uc751\ud55c \uae30\ud0c0\ubc14\ud0c0\ucf00 \ubbf8\uc4f0\ub9c8\uc0ac(\u5317\u7560\u6e80\u96c5)\uac00 1412\ub144(\uc624\uc5d0\uc774 19\ub144)\uc758 \uc1fc\ucf54 \ucc9c\ud669\uc758 \uc989\uc704\uc5d0 \ubd88\ubcf5\ud558\uc5ec \uc591\ud1b5 \uc9c8\ub9bd(\u4e21\u7d71\u8fed\u7acb)\uc758 \uc57d\uc18d\uc744 \uc9c0\ud0ac \uac83\uc744 \uc694\uad6c\ud558\uba70 \ubc18\ub780\uc744 \uc77c\uc73c\ucf30\uc73c\ub098, \uae08\uc138 \ud654\ud574\ud558\uc600\ub2e4. \uac19\uc740 \ud574\uc5d0\ub294 \uc694\uc2dc\ubaa8\uce58\ub97c \uc7a5\uae30\uac04 \uc9c0\ud0f1\ud588\ub358 \uc219\ub85c \uc2dc\ubc14 \uc694\uc2dc\uc720\ud0a4\uc758 \uc870\uce74 \uc2dc\ubc14 \ubbf8\uc4f0\ud0c0\ub124(\u65af\u6ce2\u6e80\u7a2e)\uac00 \uc694\uc2dc\ubaa8\uce58\uc758 \ub178\uc5ec\uc6c0\uc744 \uc0ac \uace0\uc57c \uc0b0(\u9ad8\u91ce\u5c71)\uc73c\ub85c \uc740\ud1f4\ud558\ub294 \uc0ac\uac74\uc774 \ubc8c\uc5b4\uc84c\ub2e4. 1416\ub144(\uc624\uc5d0\uc774 23\ub144)\uc5d0\ub294 \uac04\ud1a0 \uc9c0\ubc29\uc5d0\uc11c \uc6b0\uc5d0\uc2a4\uae30 \uc820\uc288\uc758 \ub09c(\u4e0a\u6749\u7985\u79c0\u306e\u4e71)\uc774 \uc77c\uc5b4\ub098 \uc774\uc5d0 \uad00\uc5ec\ud55c \ub3d9\uc0dd \uc694\uc2dc\uc4f0\uad6c\ub97c \uc1fc\ucf54\ucfe0\uc9c0(\u76f8\u56fd\u5bfa)\uc5d0 \uc720\ud3d0\ud558\uace0, 2\ub144 \ub4a4\uc778 1418\ub144(\uc624\uc5d0\uc774 25\ub144)\uc5d0 \uc0b4\ud574\ud588\ub2e4. \ub354\uc6b1\uc774 \uadf8 \uacfc\uc815\uc5d0\uc11c \uc694\uc2dc\ubaa8\uce58 \uce21\uadfc \ub3c4\uac00\uc2dc \ubbf8\uc4f0\ub098\ub9ac(\u5bcc\u6a2b\u6e80\u6210)\ub294 \ud638\uc18c\uce74\uc640 \ubbf8\uc4f0\ubaa8\ud1a0(\u7d30\u5ddd\u6e80\u5143)\uc640 \uc2dc\ubc14 \uc694\uc2dc\uc2dc\uac8c(\u65af\u6ce2\u7fa9\u91cd)\uac00 \uc694\uc2dc\uc4f0\uad6c\uc5d0\uac8c \uac00\ub2f4\ud588\ub2e4\uace0 \uace0\ubc1c\ud558\uc600\ub2e4. \uc77c\uc124\uc5d0\ub294 \uc774 \uace0\ubc1c\uc740 \uc694\uc2dc\ubaa8\uce58\uc758 \uc758\ud5a5\uc5d0 \ub530\ub978 \uac83\uc73c\ub85c \uc720\ub825 \uc288\uace0\uc758 \ubc1c\uc5b8\ub825\uc744 \uc904\uc774\ub824\uace0 \uc694\uc2dc\ubaa8\uce58\uac00 \uc8fc\ub3c4\ud55c \ub9c9\uc815\uc758 \ud655\ub9bd\uc744 \uc9c0\ud5a5\ud55c \uac83\uc774\ub77c\uace0 \ud55c\ub2e4. \uadf8\ub7ec\ub098 \ucd5c\uc885\uc801\uc73c\ub85c\ub294 \ud638\uc18c\uce74\uc640 \ub4f1\uc758 \uace0\ubc1c\uc73c\ub85c \ub3c4\uac00\uc2dc\uac00 \ucd94\ubc29\ub2f9\ud558\uace0 \ud6c4\uc5d0 \uc0b4\ud574\ub41c\ub2e4.", "sentence_answer": "1410\ub144(\uc624\uc5d0\uc774 17\ub144)\uc5d0\ub294 \ub0a8\uc870 \ucd5c\ud6c4\uc758 \ucc9c\ud669\uc774\uc5c8\ub358 \uace0\uce74\uba54\uc57c\ub9c8 \ucc9c\ud669\uc774 \uc694\uc2dc\ub178\ub85c \ucd9c\ubd84\ud558\uc600\ub2e4( 1416\ub144 \uadc0\uacbd)", "paragraph_id": "6604513-2-1", "paragraph_question": "question: \uc6b0\uc5d0\uc2a4\uae30 \uc820\uc288\uc758 \ub09c\uc774 \uc77c\uc5b4\ub09c \uac83\uc740 \uba87 \ub144\uc778\uac00?, context: 1410\ub144(\uc624\uc5d0\uc774 17\ub144)\uc5d0\ub294 \ub0a8\uc870 \ucd5c\ud6c4\uc758 \ucc9c\ud669\uc774\uc5c8\ub358 \uace0\uce74\uba54\uc57c\ub9c8 \ucc9c\ud669\uc774 \uc694\uc2dc\ub178\ub85c \ucd9c\ubd84\ud558\uc600\ub2e4(1416\ub144 \uadc0\uacbd). 1414\ub144(\uc624\uc5d0\uc774 21\ub144)\uc5d0 \uc774\uc5d0 \ud638\uc751\ud55c \uae30\ud0c0\ubc14\ud0c0\ucf00 \ubbf8\uc4f0\ub9c8\uc0ac(\u5317\u7560\u6e80\u96c5)\uac00 1412\ub144(\uc624\uc5d0\uc774 19\ub144)\uc758 \uc1fc\ucf54 \ucc9c\ud669\uc758 \uc989\uc704\uc5d0 \ubd88\ubcf5\ud558\uc5ec \uc591\ud1b5 \uc9c8\ub9bd(\u4e21\u7d71\u8fed\u7acb)\uc758 \uc57d\uc18d\uc744 \uc9c0\ud0ac \uac83\uc744 \uc694\uad6c\ud558\uba70 \ubc18\ub780\uc744 \uc77c\uc73c\ucf30\uc73c\ub098, \uae08\uc138 \ud654\ud574\ud558\uc600\ub2e4. \uac19\uc740 \ud574\uc5d0\ub294 \uc694\uc2dc\ubaa8\uce58\ub97c \uc7a5\uae30\uac04 \uc9c0\ud0f1\ud588\ub358 \uc219\ub85c \uc2dc\ubc14 \uc694\uc2dc\uc720\ud0a4\uc758 \uc870\uce74 \uc2dc\ubc14 \ubbf8\uc4f0\ud0c0\ub124(\u65af\u6ce2\u6e80\u7a2e)\uac00 \uc694\uc2dc\ubaa8\uce58\uc758 \ub178\uc5ec\uc6c0\uc744 \uc0ac \uace0\uc57c \uc0b0(\u9ad8\u91ce\u5c71)\uc73c\ub85c \uc740\ud1f4\ud558\ub294 \uc0ac\uac74\uc774 \ubc8c\uc5b4\uc84c\ub2e4. 1416\ub144(\uc624\uc5d0\uc774 23\ub144)\uc5d0\ub294 \uac04\ud1a0 \uc9c0\ubc29\uc5d0\uc11c \uc6b0\uc5d0\uc2a4\uae30 \uc820\uc288\uc758 \ub09c(\u4e0a\u6749\u7985\u79c0\u306e\u4e71)\uc774 \uc77c\uc5b4\ub098 \uc774\uc5d0 \uad00\uc5ec\ud55c \ub3d9\uc0dd \uc694\uc2dc\uc4f0\uad6c\ub97c \uc1fc\ucf54\ucfe0\uc9c0(\u76f8\u56fd\u5bfa)\uc5d0 \uc720\ud3d0\ud558\uace0, 2\ub144 \ub4a4\uc778 1418\ub144(\uc624\uc5d0\uc774 25\ub144)\uc5d0 \uc0b4\ud574\ud588\ub2e4. \ub354\uc6b1\uc774 \uadf8 \uacfc\uc815\uc5d0\uc11c \uc694\uc2dc\ubaa8\uce58 \uce21\uadfc \ub3c4\uac00\uc2dc \ubbf8\uc4f0\ub098\ub9ac(\u5bcc\u6a2b\u6e80\u6210)\ub294 \ud638\uc18c\uce74\uc640 \ubbf8\uc4f0\ubaa8\ud1a0(\u7d30\u5ddd\u6e80\u5143)\uc640 \uc2dc\ubc14 \uc694\uc2dc\uc2dc\uac8c(\u65af\u6ce2\u7fa9\u91cd)\uac00 \uc694\uc2dc\uc4f0\uad6c\uc5d0\uac8c \uac00\ub2f4\ud588\ub2e4\uace0 \uace0\ubc1c\ud558\uc600\ub2e4. \uc77c\uc124\uc5d0\ub294 \uc774 \uace0\ubc1c\uc740 \uc694\uc2dc\ubaa8\uce58\uc758 \uc758\ud5a5\uc5d0 \ub530\ub978 \uac83\uc73c\ub85c \uc720\ub825 \uc288\uace0\uc758 \ubc1c\uc5b8\ub825\uc744 \uc904\uc774\ub824\uace0 \uc694\uc2dc\ubaa8\uce58\uac00 \uc8fc\ub3c4\ud55c \ub9c9\uc815\uc758 \ud655\ub9bd\uc744 \uc9c0\ud5a5\ud55c \uac83\uc774\ub77c\uace0 \ud55c\ub2e4. \uadf8\ub7ec\ub098 \ucd5c\uc885\uc801\uc73c\ub85c\ub294 \ud638\uc18c\uce74\uc640 \ub4f1\uc758 \uace0\ubc1c\uc73c\ub85c \ub3c4\uac00\uc2dc\uac00 \ucd94\ubc29\ub2f9\ud558\uace0 \ud6c4\uc5d0 \uc0b4\ud574\ub41c\ub2e4."} {"question": "\uae40\ud0dc\ub9ac\uac00 \uc878\uc5c5\ud55c \ub300\ud559\uc740?", "paragraph": "\uae40\ud0dc\ub9ac\ub294 1990\ub144 4\uc6d4 24\uc77c \uc11c\uc6b8\uc5d0\uc11c 1\ub0a8 1\ub140 \uc911 \ub9c9\ub0b4\ub85c \ud0dc\uc5b4\ub0ac\uc73c\uba70, 2\uc0b4 \uc704\uc758 \uc624\ube60\uac00 \uc788\ub2e4. \uc5b4\ub9b0 \uc2dc\uc808\ubd80\ud130 \ub370\ubdd4 \ucd08\uae4c\uc9c0\ub294 \ud560\uba38\ub2c8\uc640 \uc0dd\ud65c\ud588\uace0 \ud604\uc7ac\ub294 \ub3c5\ub9bd\ud574 \ud63c\uc790 \uc0b4\uace0 \uc788\ub2e4. \uae40\ud0dc\ub9ac\ub77c\ub294 \uc774\ub984\uc740 \ubcf8\uba85\uc73c\ub85c '\ud0dc'(\u6cf0)\ub294 \uc9d1\uc548\uc758 \ub3cc\ub9bc\uc790\uace0 '\ub9ac'(\u68a8)\ub294 \ubc30\ub098\ubb34\ub97c \ub73b\ud55c\ub2e4. \ud0dc\uc5b4\ub0a0 \ub54c \ub3d9\ub124\uc5d0 \ubc30\uaf43\uc774 \ub9cc\uac1c\ud55c \uac83\uc5d0\uc11c \ub530\uc628 \uc774\ub984\uc774\ub2e4. \uc5b4\uba38\ub2c8\ub294 \uc815\uce58\ub97c \ud558\uae38 \ubc14\ub77c\ub294 \ub9c8\uc74c\uc5d0\uc11c '\ud0dc\uc815'\uc744 \uc6d0\ud558\uc168\uc9c0\ub9cc \uc544\ubc84\uc9c0\uac00 \ucd9c\uc0dd\uc2e0\uace0 \ud558\ub7ec \uac00\ub294 \uae38\uc5d0 '\ud0dc\ub9ac'\ub85c \ubc14\uafb8\uc168\ub2e4\uace0 \ud55c\ub2e4.\uc601\uc2e0\uc5ec\uc790\uc2e4\uc5c5\uace0\ub4f1\ud559\uad50 \ub514\uc790\uc778\uacfc\uc5d0 \uc9c4\ud559\ud55c \ud6c4, \ub9c9\uc5f0\ud788 \uc544\ub098\uc6b4\uc11c\uac00 \ub418\uace0 \uc2f6\uc740 \uc0dd\uac01\uc5d0 \uacbd\ud76c\ub300\ud559\uad50 \uc2e0\ubb38\ubc29\uc1a1\ud559\uacfc\uc5d0 \uc785\ud559\ud588\ub2e4. \uc774\ub54c\uae4c\uc9c0\ub9cc \ud574\ub3c4 \uc5f0\uae30, \uc5f0\uadf9, \uc601\ud654\uc5d0 \ub300\ud55c \uad00\uc2ec\uc740 \ud558\ub098\ub3c4 \uc5c6\uc5c8\ub2e4. \uae40\ud0dc\ub9ac\uac00 \ucc98\uc74c \uc5f0\uae30\ub97c \uc811\ud55c \uacc4\uae30\ub294 \ub300\ud559 \uc5f0\uadf9 \ub3d9\uc544\ub9ac. 1\ud559\ub144 \ub54c \uc2e0\uc785\ub2e8\uc6d0\uc744 \ubaa8\uc9d1\ud558\ub294 \uc5f0\uadf9\uc744 \ubcf4\uace0 \uc5f0\uadf9 \ub3d9\uc544\ub9ac\uc5d0 \uac00\uc785\ud588\uace0, 2\ud559\ub144 \ub54c \uc9c4\ub85c\ub97c \uc5f0\uadf9\ubc30\uc6b0\ub85c \uacb0\uc815\ud588\ub2e4\uace0 \ud55c\ub2e4. \ub3d9\uc544\ub9ac \ud65c\ub3d9\uc744 \ud558\uba70 \ub3c5\ub9bd \uc601\ud654 \u300a\uc2dc\ubbfc\uc880\ube44\u300b\uc5d0 \ucd9c\uc5f0\ud558\uae30\ub3c4 \ud588\uc73c\uba70, \ub300\ud559\uc744 \uc878\uc5c5\ud560 \uc988\uc74c\uc5d4 \ub300\ud559\ub85c\ub85c \ub098\uac00 \uadf9\ub2e8\uc5d0 \ub4e4\uc5b4\uac14\ub2e4.", "answer": "\uacbd\ud76c\ub300\ud559\uad50", "sentence": "\uc5b4\uba38\ub2c8\ub294 \uc815\uce58\ub97c \ud558\uae38 \ubc14\ub77c\ub294 \ub9c8\uc74c\uc5d0\uc11c '\ud0dc\uc815'\uc744 \uc6d0\ud558\uc168\uc9c0\ub9cc \uc544\ubc84\uc9c0\uac00 \ucd9c\uc0dd\uc2e0\uace0 \ud558\ub7ec \uac00\ub294 \uae38\uc5d0 '\ud0dc\ub9ac'\ub85c \ubc14\uafb8\uc168\ub2e4\uace0 \ud55c\ub2e4.\uc601\uc2e0\uc5ec\uc790\uc2e4\uc5c5\uace0\ub4f1\ud559\uad50 \ub514\uc790\uc778\uacfc\uc5d0 \uc9c4\ud559\ud55c \ud6c4, \ub9c9\uc5f0\ud788 \uc544\ub098\uc6b4\uc11c\uac00 \ub418\uace0 \uc2f6\uc740 \uc0dd\uac01\uc5d0 \uacbd\ud76c\ub300\ud559\uad50 \uc2e0\ubb38\ubc29\uc1a1\ud559\uacfc\uc5d0 \uc785\ud559\ud588\ub2e4.", "paragraph_sentence": "\uae40\ud0dc\ub9ac\ub294 1990\ub144 4\uc6d4 24\uc77c \uc11c\uc6b8\uc5d0\uc11c 1\ub0a8 1\ub140 \uc911 \ub9c9\ub0b4\ub85c \ud0dc\uc5b4\ub0ac\uc73c\uba70, 2\uc0b4 \uc704\uc758 \uc624\ube60\uac00 \uc788\ub2e4. \uc5b4\ub9b0 \uc2dc\uc808\ubd80\ud130 \ub370\ubdd4 \ucd08\uae4c\uc9c0\ub294 \ud560\uba38\ub2c8\uc640 \uc0dd\ud65c\ud588\uace0 \ud604\uc7ac\ub294 \ub3c5\ub9bd\ud574 \ud63c\uc790 \uc0b4\uace0 \uc788\ub2e4. \uae40\ud0dc\ub9ac\ub77c\ub294 \uc774\ub984\uc740 \ubcf8\uba85\uc73c\ub85c '\ud0dc'(\u6cf0)\ub294 \uc9d1\uc548\uc758 \ub3cc\ub9bc\uc790\uace0 '\ub9ac'(\u68a8)\ub294 \ubc30\ub098\ubb34\ub97c \ub73b\ud55c\ub2e4. \ud0dc\uc5b4\ub0a0 \ub54c \ub3d9\ub124\uc5d0 \ubc30\uaf43\uc774 \ub9cc\uac1c\ud55c \uac83\uc5d0\uc11c \ub530\uc628 \uc774\ub984\uc774\ub2e4. \uc5b4\uba38\ub2c8\ub294 \uc815\uce58\ub97c \ud558\uae38 \ubc14\ub77c\ub294 \ub9c8\uc74c\uc5d0\uc11c '\ud0dc\uc815'\uc744 \uc6d0\ud558\uc168\uc9c0\ub9cc \uc544\ubc84\uc9c0\uac00 \ucd9c\uc0dd\uc2e0\uace0 \ud558\ub7ec \uac00\ub294 \uae38\uc5d0 '\ud0dc\ub9ac'\ub85c \ubc14\uafb8\uc168\ub2e4\uace0 \ud55c\ub2e4.\uc601\uc2e0\uc5ec\uc790\uc2e4\uc5c5\uace0\ub4f1\ud559\uad50 \ub514\uc790\uc778\uacfc\uc5d0 \uc9c4\ud559\ud55c \ud6c4, \ub9c9\uc5f0\ud788 \uc544\ub098\uc6b4\uc11c\uac00 \ub418\uace0 \uc2f6\uc740 \uc0dd\uac01\uc5d0 \uacbd\ud76c\ub300\ud559\uad50 \uc2e0\ubb38\ubc29\uc1a1\ud559\uacfc\uc5d0 \uc785\ud559\ud588\ub2e4. \uc774\ub54c\uae4c\uc9c0\ub9cc \ud574\ub3c4 \uc5f0\uae30, \uc5f0\uadf9, \uc601\ud654\uc5d0 \ub300\ud55c \uad00\uc2ec\uc740 \ud558\ub098\ub3c4 \uc5c6\uc5c8\ub2e4. \uae40\ud0dc\ub9ac\uac00 \ucc98\uc74c \uc5f0\uae30\ub97c \uc811\ud55c \uacc4\uae30\ub294 \ub300\ud559 \uc5f0\uadf9 \ub3d9\uc544\ub9ac. 1\ud559\ub144 \ub54c \uc2e0\uc785\ub2e8\uc6d0\uc744 \ubaa8\uc9d1\ud558\ub294 \uc5f0\uadf9\uc744 \ubcf4\uace0 \uc5f0\uadf9 \ub3d9\uc544\ub9ac\uc5d0 \uac00\uc785\ud588\uace0, 2\ud559\ub144 \ub54c \uc9c4\ub85c\ub97c \uc5f0\uadf9\ubc30\uc6b0\ub85c \uacb0\uc815\ud588\ub2e4\uace0 \ud55c\ub2e4. \ub3d9\uc544\ub9ac \ud65c\ub3d9\uc744 \ud558\uba70 \ub3c5\ub9bd \uc601\ud654 \u300a\uc2dc\ubbfc\uc880\ube44\u300b\uc5d0 \ucd9c\uc5f0\ud558\uae30\ub3c4 \ud588\uc73c\uba70, \ub300\ud559\uc744 \uc878\uc5c5\ud560 \uc988\uc74c\uc5d4 \ub300\ud559\ub85c\ub85c \ub098\uac00 \uadf9\ub2e8\uc5d0 \ub4e4\uc5b4\uac14\ub2e4.", "paragraph_answer": "\uae40\ud0dc\ub9ac\ub294 1990\ub144 4\uc6d4 24\uc77c \uc11c\uc6b8\uc5d0\uc11c 1\ub0a8 1\ub140 \uc911 \ub9c9\ub0b4\ub85c \ud0dc\uc5b4\ub0ac\uc73c\uba70, 2\uc0b4 \uc704\uc758 \uc624\ube60\uac00 \uc788\ub2e4. \uc5b4\ub9b0 \uc2dc\uc808\ubd80\ud130 \ub370\ubdd4 \ucd08\uae4c\uc9c0\ub294 \ud560\uba38\ub2c8\uc640 \uc0dd\ud65c\ud588\uace0 \ud604\uc7ac\ub294 \ub3c5\ub9bd\ud574 \ud63c\uc790 \uc0b4\uace0 \uc788\ub2e4. \uae40\ud0dc\ub9ac\ub77c\ub294 \uc774\ub984\uc740 \ubcf8\uba85\uc73c\ub85c '\ud0dc'(\u6cf0)\ub294 \uc9d1\uc548\uc758 \ub3cc\ub9bc\uc790\uace0 '\ub9ac'(\u68a8)\ub294 \ubc30\ub098\ubb34\ub97c \ub73b\ud55c\ub2e4. \ud0dc\uc5b4\ub0a0 \ub54c \ub3d9\ub124\uc5d0 \ubc30\uaf43\uc774 \ub9cc\uac1c\ud55c \uac83\uc5d0\uc11c \ub530\uc628 \uc774\ub984\uc774\ub2e4. \uc5b4\uba38\ub2c8\ub294 \uc815\uce58\ub97c \ud558\uae38 \ubc14\ub77c\ub294 \ub9c8\uc74c\uc5d0\uc11c '\ud0dc\uc815'\uc744 \uc6d0\ud558\uc168\uc9c0\ub9cc \uc544\ubc84\uc9c0\uac00 \ucd9c\uc0dd\uc2e0\uace0 \ud558\ub7ec \uac00\ub294 \uae38\uc5d0 '\ud0dc\ub9ac'\ub85c \ubc14\uafb8\uc168\ub2e4\uace0 \ud55c\ub2e4.\uc601\uc2e0\uc5ec\uc790\uc2e4\uc5c5\uace0\ub4f1\ud559\uad50 \ub514\uc790\uc778\uacfc\uc5d0 \uc9c4\ud559\ud55c \ud6c4, \ub9c9\uc5f0\ud788 \uc544\ub098\uc6b4\uc11c\uac00 \ub418\uace0 \uc2f6\uc740 \uc0dd\uac01\uc5d0 \uacbd\ud76c\ub300\ud559\uad50 \uc2e0\ubb38\ubc29\uc1a1\ud559\uacfc\uc5d0 \uc785\ud559\ud588\ub2e4. \uc774\ub54c\uae4c\uc9c0\ub9cc \ud574\ub3c4 \uc5f0\uae30, \uc5f0\uadf9, \uc601\ud654\uc5d0 \ub300\ud55c \uad00\uc2ec\uc740 \ud558\ub098\ub3c4 \uc5c6\uc5c8\ub2e4. \uae40\ud0dc\ub9ac\uac00 \ucc98\uc74c \uc5f0\uae30\ub97c \uc811\ud55c \uacc4\uae30\ub294 \ub300\ud559 \uc5f0\uadf9 \ub3d9\uc544\ub9ac. 1\ud559\ub144 \ub54c \uc2e0\uc785\ub2e8\uc6d0\uc744 \ubaa8\uc9d1\ud558\ub294 \uc5f0\uadf9\uc744 \ubcf4\uace0 \uc5f0\uadf9 \ub3d9\uc544\ub9ac\uc5d0 \uac00\uc785\ud588\uace0, 2\ud559\ub144 \ub54c \uc9c4\ub85c\ub97c \uc5f0\uadf9\ubc30\uc6b0\ub85c \uacb0\uc815\ud588\ub2e4\uace0 \ud55c\ub2e4. \ub3d9\uc544\ub9ac \ud65c\ub3d9\uc744 \ud558\uba70 \ub3c5\ub9bd \uc601\ud654 \u300a\uc2dc\ubbfc\uc880\ube44\u300b\uc5d0 \ucd9c\uc5f0\ud558\uae30\ub3c4 \ud588\uc73c\uba70, \ub300\ud559\uc744 \uc878\uc5c5\ud560 \uc988\uc74c\uc5d4 \ub300\ud559\ub85c\ub85c \ub098\uac00 \uadf9\ub2e8\uc5d0 \ub4e4\uc5b4\uac14\ub2e4.", "sentence_answer": "\uc5b4\uba38\ub2c8\ub294 \uc815\uce58\ub97c \ud558\uae38 \ubc14\ub77c\ub294 \ub9c8\uc74c\uc5d0\uc11c '\ud0dc\uc815'\uc744 \uc6d0\ud558\uc168\uc9c0\ub9cc \uc544\ubc84\uc9c0\uac00 \ucd9c\uc0dd\uc2e0\uace0 \ud558\ub7ec \uac00\ub294 \uae38\uc5d0 '\ud0dc\ub9ac'\ub85c \ubc14\uafb8\uc168\ub2e4\uace0 \ud55c\ub2e4.\uc601\uc2e0\uc5ec\uc790\uc2e4\uc5c5\uace0\ub4f1\ud559\uad50 \ub514\uc790\uc778\uacfc\uc5d0 \uc9c4\ud559\ud55c \ud6c4, \ub9c9\uc5f0\ud788 \uc544\ub098\uc6b4\uc11c\uac00 \ub418\uace0 \uc2f6\uc740 \uc0dd\uac01\uc5d0 \uacbd\ud76c\ub300\ud559\uad50 \uc2e0\ubb38\ubc29\uc1a1\ud559\uacfc\uc5d0 \uc785\ud559\ud588\ub2e4.", "paragraph_id": "6488631-0-1", "paragraph_question": "question: \uae40\ud0dc\ub9ac\uac00 \uc878\uc5c5\ud55c \ub300\ud559\uc740?, context: \uae40\ud0dc\ub9ac\ub294 1990\ub144 4\uc6d4 24\uc77c \uc11c\uc6b8\uc5d0\uc11c 1\ub0a8 1\ub140 \uc911 \ub9c9\ub0b4\ub85c \ud0dc\uc5b4\ub0ac\uc73c\uba70, 2\uc0b4 \uc704\uc758 \uc624\ube60\uac00 \uc788\ub2e4. \uc5b4\ub9b0 \uc2dc\uc808\ubd80\ud130 \ub370\ubdd4 \ucd08\uae4c\uc9c0\ub294 \ud560\uba38\ub2c8\uc640 \uc0dd\ud65c\ud588\uace0 \ud604\uc7ac\ub294 \ub3c5\ub9bd\ud574 \ud63c\uc790 \uc0b4\uace0 \uc788\ub2e4. \uae40\ud0dc\ub9ac\ub77c\ub294 \uc774\ub984\uc740 \ubcf8\uba85\uc73c\ub85c '\ud0dc'(\u6cf0)\ub294 \uc9d1\uc548\uc758 \ub3cc\ub9bc\uc790\uace0 '\ub9ac'(\u68a8)\ub294 \ubc30\ub098\ubb34\ub97c \ub73b\ud55c\ub2e4. \ud0dc\uc5b4\ub0a0 \ub54c \ub3d9\ub124\uc5d0 \ubc30\uaf43\uc774 \ub9cc\uac1c\ud55c \uac83\uc5d0\uc11c \ub530\uc628 \uc774\ub984\uc774\ub2e4. \uc5b4\uba38\ub2c8\ub294 \uc815\uce58\ub97c \ud558\uae38 \ubc14\ub77c\ub294 \ub9c8\uc74c\uc5d0\uc11c '\ud0dc\uc815'\uc744 \uc6d0\ud558\uc168\uc9c0\ub9cc \uc544\ubc84\uc9c0\uac00 \ucd9c\uc0dd\uc2e0\uace0 \ud558\ub7ec \uac00\ub294 \uae38\uc5d0 '\ud0dc\ub9ac'\ub85c \ubc14\uafb8\uc168\ub2e4\uace0 \ud55c\ub2e4.\uc601\uc2e0\uc5ec\uc790\uc2e4\uc5c5\uace0\ub4f1\ud559\uad50 \ub514\uc790\uc778\uacfc\uc5d0 \uc9c4\ud559\ud55c \ud6c4, \ub9c9\uc5f0\ud788 \uc544\ub098\uc6b4\uc11c\uac00 \ub418\uace0 \uc2f6\uc740 \uc0dd\uac01\uc5d0 \uacbd\ud76c\ub300\ud559\uad50 \uc2e0\ubb38\ubc29\uc1a1\ud559\uacfc\uc5d0 \uc785\ud559\ud588\ub2e4. \uc774\ub54c\uae4c\uc9c0\ub9cc \ud574\ub3c4 \uc5f0\uae30, \uc5f0\uadf9, \uc601\ud654\uc5d0 \ub300\ud55c \uad00\uc2ec\uc740 \ud558\ub098\ub3c4 \uc5c6\uc5c8\ub2e4. \uae40\ud0dc\ub9ac\uac00 \ucc98\uc74c \uc5f0\uae30\ub97c \uc811\ud55c \uacc4\uae30\ub294 \ub300\ud559 \uc5f0\uadf9 \ub3d9\uc544\ub9ac. 1\ud559\ub144 \ub54c \uc2e0\uc785\ub2e8\uc6d0\uc744 \ubaa8\uc9d1\ud558\ub294 \uc5f0\uadf9\uc744 \ubcf4\uace0 \uc5f0\uadf9 \ub3d9\uc544\ub9ac\uc5d0 \uac00\uc785\ud588\uace0, 2\ud559\ub144 \ub54c \uc9c4\ub85c\ub97c \uc5f0\uadf9\ubc30\uc6b0\ub85c \uacb0\uc815\ud588\ub2e4\uace0 \ud55c\ub2e4. \ub3d9\uc544\ub9ac \ud65c\ub3d9\uc744 \ud558\uba70 \ub3c5\ub9bd \uc601\ud654 \u300a\uc2dc\ubbfc\uc880\ube44\u300b\uc5d0 \ucd9c\uc5f0\ud558\uae30\ub3c4 \ud588\uc73c\uba70, \ub300\ud559\uc744 \uc878\uc5c5\ud560 \uc988\uc74c\uc5d4 \ub300\ud559\ub85c\ub85c \ub098\uac00 \uadf9\ub2e8\uc5d0 \ub4e4\uc5b4\uac14\ub2e4."} {"question": "\uccad\ub098\ub77c\uc758 \ud669\uc81c\ub85c \uac15\ud76c\uc81c\uc758 \uc190\uc790\ub294?", "paragraph": "\uac15\ud76c\uc81c\ub294 \ub2e8\ud638\ud55c \uc678\uc815\uacfc\ub294 \ub2ec\ub9ac \ub0b4\uc815\uc5d0\uc11c\ub294 \uc120\uc815\uc744 \ubca0\ud480\uc5c8\ub294\ub370 \ud669\ud558\uc640 \uc7a5\uac15\uc758 \uce58\uc218\uc5d0 \uc131\uacf5\ud558\uc5ec \ub18d\uc0ac\uc5d0 \ucc28\uc9c8\uc774 \uc5c6\ub3c4\ub85d \ud558\uc600\uc73c\uba70 \uc804\ub780\uc758 \uc640\uc911\uc5d0\ub3c4 \uc138\uae08\uc744 \uc904\uc774\uace0 \uc778\ub450\uc138 \uc5ed\uc2dc \uc601\uc6d0\ud788 \ub3d9\uacb0\ud558\uc5ec \ubc31\uc131\ub4e4\uc758 \ub178\uace0\ub97c \ub35c\uc5b4\uc8fc\uc5c8\ub2e4. \ud55c\ud3b8\uc73c\ub85c\ub294 \u300a\uac15\ud76c\uc790\uc804\u300b, \u300a\uace0\uae08\ub3c4\uc11c\uc9d1\uc131\u300b \ub4f1 \ub300\uaddc\ubaa8 \ud3b8\ucc2c\uc0ac\uc5c5\uc744 \ubc8c\uc5ec \ub2f9\ub300\uc758 \ubb38\ud654\ubc1c\uc804\uacfc \ud604\ub300 \uc911\uad6d\uc5b4 \uc5b4\ubc95\uc758 \uae30\ubc18\uc744 \ub9c8\ub828\ud558\uc600\uc73c\uba70 \uc9c0\ubc30\uce35\uc778 \ub9cc\uc8fc\uc871\uacfc \ud53c\uc9c0\ubc30\uce35\uc778 \ud55c\uc871\uc744 \uace0\ub8e8 \ub4f1\uc6a9\ud558\uc5ec \uc870\uc815\uc758 \ud654\ubaa9\uc744 \uaf80\ud558\uc600\ub2e4. \ub610\ud55c \uc911\uad6d \uc5ed\ub300 \ud669\uc870 \uc911 \ub9c8\uc9c0\ub9c9 \ud0dc\ud3c9\uc131\ub300\uc778 \uac15\uac74\uc131\uc138(\u5eb7\u4e7e\u76db\u4e16)\ub97c \uc77c\uc73c\ucf1c \uc544\ub4e4\uc778 \uc639\uc815\uc81c(\u96cd\u6b63\u5e1d), \uc190\uc790\uc778 \uac74\ub96d\uc81c(\u4e7e\u9686\u5e1d)\uae4c\uc9c0 \ud0dc\ud3c9\uc131\ub300\uac00 \uc9c0\uc18d\ub418\uc5c8\ub2e4. \uc911\uad6d \uc5ed\uc0ac\uc0c1 \ucd5c\uace0\uc758 \uc131\uad70\uc774\uc790 \uba85\uad70, \uc989 \ucc9c\uace0\uc77c\uc81c(\u5343\u53e4\u4e00\u5e1d, \ucc9c \ub144\uc5d0 \ud55c \ubc88 \ub098\uc62c \ub9cc\ud55c \ud669\uc81c) \ub610\ub294 \uc5f0\ud638\ub97c \ub530\uc11c \uac15\ud76c\ub300\uc81c(\u5eb7\u7155}\u5927\u5e1d)\ub85c \uce6d\uc1a1\ubc1b\uc73c\uba70 \uc544\uc9c1\ub3c4 \ub9ce\uc740 \uc911\uad6d\uc778\uc5d0\uac8c \ud06c\uac8c \uc874\uacbd\ubc1b\ub294\ub2e4.", "answer": "\uac74\ub96d\uc81c", "sentence": "\ub610\ud55c \uc911\uad6d \uc5ed\ub300 \ud669\uc870 \uc911 \ub9c8\uc9c0\ub9c9 \ud0dc\ud3c9\uc131\ub300\uc778 \uac15\uac74\uc131\uc138(\u5eb7\u4e7e\u76db\u4e16)\ub97c \uc77c\uc73c\ucf1c \uc544\ub4e4\uc778 \uc639\uc815\uc81c(\u96cd\u6b63\u5e1d), \uc190\uc790\uc778 \uac74\ub96d\uc81c (\u4e7e\u9686\u5e1d)", "paragraph_sentence": "\uac15\ud76c\uc81c\ub294 \ub2e8\ud638\ud55c \uc678\uc815\uacfc\ub294 \ub2ec\ub9ac \ub0b4\uc815\uc5d0\uc11c\ub294 \uc120\uc815\uc744 \ubca0\ud480\uc5c8\ub294\ub370 \ud669\ud558\uc640 \uc7a5\uac15\uc758 \uce58\uc218\uc5d0 \uc131\uacf5\ud558\uc5ec \ub18d\uc0ac\uc5d0 \ucc28\uc9c8\uc774 \uc5c6\ub3c4\ub85d \ud558\uc600\uc73c\uba70 \uc804\ub780\uc758 \uc640\uc911\uc5d0\ub3c4 \uc138\uae08\uc744 \uc904\uc774\uace0 \uc778\ub450\uc138 \uc5ed\uc2dc \uc601\uc6d0\ud788 \ub3d9\uacb0\ud558\uc5ec \ubc31\uc131\ub4e4\uc758 \ub178\uace0\ub97c \ub35c\uc5b4\uc8fc\uc5c8\ub2e4. \ud55c\ud3b8\uc73c\ub85c\ub294 \u300a\uac15\ud76c\uc790\uc804\u300b, \u300a\uace0\uae08\ub3c4\uc11c\uc9d1\uc131\u300b \ub4f1 \ub300\uaddc\ubaa8 \ud3b8\ucc2c\uc0ac\uc5c5\uc744 \ubc8c\uc5ec \ub2f9\ub300\uc758 \ubb38\ud654\ubc1c\uc804\uacfc \ud604\ub300 \uc911\uad6d\uc5b4 \uc5b4\ubc95\uc758 \uae30\ubc18\uc744 \ub9c8\ub828\ud558\uc600\uc73c\uba70 \uc9c0\ubc30\uce35\uc778 \ub9cc\uc8fc\uc871\uacfc \ud53c\uc9c0\ubc30\uce35\uc778 \ud55c\uc871\uc744 \uace0\ub8e8 \ub4f1\uc6a9\ud558\uc5ec \uc870\uc815\uc758 \ud654\ubaa9\uc744 \uaf80\ud558\uc600\ub2e4. \ub610\ud55c \uc911\uad6d \uc5ed\ub300 \ud669\uc870 \uc911 \ub9c8\uc9c0\ub9c9 \ud0dc\ud3c9\uc131\ub300\uc778 \uac15\uac74\uc131\uc138(\u5eb7\u4e7e\u76db\u4e16)\ub97c \uc77c\uc73c\ucf1c \uc544\ub4e4\uc778 \uc639\uc815\uc81c(\u96cd\u6b63\u5e1d), \uc190\uc790\uc778 \uac74\ub96d\uc81c (\u4e7e\u9686\u5e1d) \uae4c\uc9c0 \ud0dc\ud3c9\uc131\ub300\uac00 \uc9c0\uc18d\ub418\uc5c8\ub2e4. \uc911\uad6d \uc5ed\uc0ac\uc0c1 \ucd5c\uace0\uc758 \uc131\uad70\uc774\uc790 \uba85\uad70, \uc989 \ucc9c\uace0\uc77c\uc81c(\u5343\u53e4\u4e00\u5e1d, \ucc9c \ub144\uc5d0 \ud55c \ubc88 \ub098\uc62c \ub9cc\ud55c \ud669\uc81c) \ub610\ub294 \uc5f0\ud638\ub97c \ub530\uc11c \uac15\ud76c\ub300\uc81c(\u5eb7\u7155}\u5927\u5e1d)\ub85c \uce6d\uc1a1\ubc1b\uc73c\uba70 \uc544\uc9c1\ub3c4 \ub9ce\uc740 \uc911\uad6d\uc778\uc5d0\uac8c \ud06c\uac8c \uc874\uacbd\ubc1b\ub294\ub2e4.", "paragraph_answer": "\uac15\ud76c\uc81c\ub294 \ub2e8\ud638\ud55c \uc678\uc815\uacfc\ub294 \ub2ec\ub9ac \ub0b4\uc815\uc5d0\uc11c\ub294 \uc120\uc815\uc744 \ubca0\ud480\uc5c8\ub294\ub370 \ud669\ud558\uc640 \uc7a5\uac15\uc758 \uce58\uc218\uc5d0 \uc131\uacf5\ud558\uc5ec \ub18d\uc0ac\uc5d0 \ucc28\uc9c8\uc774 \uc5c6\ub3c4\ub85d \ud558\uc600\uc73c\uba70 \uc804\ub780\uc758 \uc640\uc911\uc5d0\ub3c4 \uc138\uae08\uc744 \uc904\uc774\uace0 \uc778\ub450\uc138 \uc5ed\uc2dc \uc601\uc6d0\ud788 \ub3d9\uacb0\ud558\uc5ec \ubc31\uc131\ub4e4\uc758 \ub178\uace0\ub97c \ub35c\uc5b4\uc8fc\uc5c8\ub2e4. \ud55c\ud3b8\uc73c\ub85c\ub294 \u300a\uac15\ud76c\uc790\uc804\u300b, \u300a\uace0\uae08\ub3c4\uc11c\uc9d1\uc131\u300b \ub4f1 \ub300\uaddc\ubaa8 \ud3b8\ucc2c\uc0ac\uc5c5\uc744 \ubc8c\uc5ec \ub2f9\ub300\uc758 \ubb38\ud654\ubc1c\uc804\uacfc \ud604\ub300 \uc911\uad6d\uc5b4 \uc5b4\ubc95\uc758 \uae30\ubc18\uc744 \ub9c8\ub828\ud558\uc600\uc73c\uba70 \uc9c0\ubc30\uce35\uc778 \ub9cc\uc8fc\uc871\uacfc \ud53c\uc9c0\ubc30\uce35\uc778 \ud55c\uc871\uc744 \uace0\ub8e8 \ub4f1\uc6a9\ud558\uc5ec \uc870\uc815\uc758 \ud654\ubaa9\uc744 \uaf80\ud558\uc600\ub2e4. \ub610\ud55c \uc911\uad6d \uc5ed\ub300 \ud669\uc870 \uc911 \ub9c8\uc9c0\ub9c9 \ud0dc\ud3c9\uc131\ub300\uc778 \uac15\uac74\uc131\uc138(\u5eb7\u4e7e\u76db\u4e16)\ub97c \uc77c\uc73c\ucf1c \uc544\ub4e4\uc778 \uc639\uc815\uc81c(\u96cd\u6b63\u5e1d), \uc190\uc790\uc778 \uac74\ub96d\uc81c (\u4e7e\u9686\u5e1d)\uae4c\uc9c0 \ud0dc\ud3c9\uc131\ub300\uac00 \uc9c0\uc18d\ub418\uc5c8\ub2e4. \uc911\uad6d \uc5ed\uc0ac\uc0c1 \ucd5c\uace0\uc758 \uc131\uad70\uc774\uc790 \uba85\uad70, \uc989 \ucc9c\uace0\uc77c\uc81c(\u5343\u53e4\u4e00\u5e1d, \ucc9c \ub144\uc5d0 \ud55c \ubc88 \ub098\uc62c \ub9cc\ud55c \ud669\uc81c) \ub610\ub294 \uc5f0\ud638\ub97c \ub530\uc11c \uac15\ud76c\ub300\uc81c(\u5eb7\u7155}\u5927\u5e1d)\ub85c \uce6d\uc1a1\ubc1b\uc73c\uba70 \uc544\uc9c1\ub3c4 \ub9ce\uc740 \uc911\uad6d\uc778\uc5d0\uac8c \ud06c\uac8c \uc874\uacbd\ubc1b\ub294\ub2e4.", "sentence_answer": "\ub610\ud55c \uc911\uad6d \uc5ed\ub300 \ud669\uc870 \uc911 \ub9c8\uc9c0\ub9c9 \ud0dc\ud3c9\uc131\ub300\uc778 \uac15\uac74\uc131\uc138(\u5eb7\u4e7e\u76db\u4e16)\ub97c \uc77c\uc73c\ucf1c \uc544\ub4e4\uc778 \uc639\uc815\uc81c(\u96cd\u6b63\u5e1d), \uc190\uc790\uc778 \uac74\ub96d\uc81c (\u4e7e\u9686\u5e1d)", "paragraph_id": "6387920-2-1", "paragraph_question": "question: \uccad\ub098\ub77c\uc758 \ud669\uc81c\ub85c \uac15\ud76c\uc81c\uc758 \uc190\uc790\ub294?, context: \uac15\ud76c\uc81c\ub294 \ub2e8\ud638\ud55c \uc678\uc815\uacfc\ub294 \ub2ec\ub9ac \ub0b4\uc815\uc5d0\uc11c\ub294 \uc120\uc815\uc744 \ubca0\ud480\uc5c8\ub294\ub370 \ud669\ud558\uc640 \uc7a5\uac15\uc758 \uce58\uc218\uc5d0 \uc131\uacf5\ud558\uc5ec \ub18d\uc0ac\uc5d0 \ucc28\uc9c8\uc774 \uc5c6\ub3c4\ub85d \ud558\uc600\uc73c\uba70 \uc804\ub780\uc758 \uc640\uc911\uc5d0\ub3c4 \uc138\uae08\uc744 \uc904\uc774\uace0 \uc778\ub450\uc138 \uc5ed\uc2dc \uc601\uc6d0\ud788 \ub3d9\uacb0\ud558\uc5ec \ubc31\uc131\ub4e4\uc758 \ub178\uace0\ub97c \ub35c\uc5b4\uc8fc\uc5c8\ub2e4. \ud55c\ud3b8\uc73c\ub85c\ub294 \u300a\uac15\ud76c\uc790\uc804\u300b, \u300a\uace0\uae08\ub3c4\uc11c\uc9d1\uc131\u300b \ub4f1 \ub300\uaddc\ubaa8 \ud3b8\ucc2c\uc0ac\uc5c5\uc744 \ubc8c\uc5ec \ub2f9\ub300\uc758 \ubb38\ud654\ubc1c\uc804\uacfc \ud604\ub300 \uc911\uad6d\uc5b4 \uc5b4\ubc95\uc758 \uae30\ubc18\uc744 \ub9c8\ub828\ud558\uc600\uc73c\uba70 \uc9c0\ubc30\uce35\uc778 \ub9cc\uc8fc\uc871\uacfc \ud53c\uc9c0\ubc30\uce35\uc778 \ud55c\uc871\uc744 \uace0\ub8e8 \ub4f1\uc6a9\ud558\uc5ec \uc870\uc815\uc758 \ud654\ubaa9\uc744 \uaf80\ud558\uc600\ub2e4. \ub610\ud55c \uc911\uad6d \uc5ed\ub300 \ud669\uc870 \uc911 \ub9c8\uc9c0\ub9c9 \ud0dc\ud3c9\uc131\ub300\uc778 \uac15\uac74\uc131\uc138(\u5eb7\u4e7e\u76db\u4e16)\ub97c \uc77c\uc73c\ucf1c \uc544\ub4e4\uc778 \uc639\uc815\uc81c(\u96cd\u6b63\u5e1d), \uc190\uc790\uc778 \uac74\ub96d\uc81c(\u4e7e\u9686\u5e1d)\uae4c\uc9c0 \ud0dc\ud3c9\uc131\ub300\uac00 \uc9c0\uc18d\ub418\uc5c8\ub2e4. \uc911\uad6d \uc5ed\uc0ac\uc0c1 \ucd5c\uace0\uc758 \uc131\uad70\uc774\uc790 \uba85\uad70, \uc989 \ucc9c\uace0\uc77c\uc81c(\u5343\u53e4\u4e00\u5e1d, \ucc9c \ub144\uc5d0 \ud55c \ubc88 \ub098\uc62c \ub9cc\ud55c \ud669\uc81c) \ub610\ub294 \uc5f0\ud638\ub97c \ub530\uc11c \uac15\ud76c\ub300\uc81c(\u5eb7\u7155}\u5927\u5e1d)\ub85c \uce6d\uc1a1\ubc1b\uc73c\uba70 \uc544\uc9c1\ub3c4 \ub9ce\uc740 \uc911\uad6d\uc778\uc5d0\uac8c \ud06c\uac8c \uc874\uacbd\ubc1b\ub294\ub2e4."} {"question": "\ud504\ub791\uc2a4 \uc694\ub9ac\uac00 \uc77c\ubcf8\uc5d0 \ud070 \uc601\ud5a5\uc744 \ub07c\uce5c\uac83\uc744 \uc54c \uc218 \uc788\ub294 \uc77c\ubcf8\uc758 \ud154\ub808\ube44\uc83c \uc1fc\ub294?", "paragraph": "\uc591\uad6d \uad00\uacc4\ub294 \uc815\uce58\u30fb\uacbd\uc81c\uc801\uc778 \uba74\ubcf4\ub2e4 \ubb38\ud654\uba74\uc5d0\uc11c\uc758 \uad50\ub958\uac00 \ub354 \ud070 \uacbd\ud5a5\uc5d0 \uc788\ub2e4. \ud504\ub791\uc2a4 \ubb38\ud654\ub294 \ubbf8\uc220\u30fb\uc74c\uc545\u30fb\uc2dd\ubb38\ud654\u30fb\ubb38\uc608 \ub4f1\uc5d0\uc11c \uc77c\ubcf8\uc758 \uadfc\ub300\ud654\uc5d0 \ud070 \uc601\ud5a5\uc744 \ub07c\ucce4\ub2e4. \ub610\ud55c \ud504\ub791\uc2a4\ub294 \uc77c\ubcf8\ubb38\ud654\ub97c \uc9c0\uc9c0\ud558\ub294 \uc0ac\ub78c\ub3c4 \ub9ce\uc544, \ubbf8\uc220\u30fb\uacf5\uc608\u30fb\uc694\ub9ac\u30fb\ub300\uc911\ubb38\ud654 \ub4f1 19\uc138\uae30 \ud6c4\ubc18\uc758 \uc790\ud3ec\ub2c8\uc998\uc744 \uc2dc\uc791\uc73c\ub85c, \ud504\ub791\uc2a4\uc5d0 \ubbf8\uce5c \uc77c\ubcf8\ubb38\ud654\uc758 \uc601\ud5a5\ub3c4 \uc788\ub2e4. \ub610\ud55c \ud30c\ub9ac\uc5d0\ub294 \uc77c\ubcf8 \ubb38\ud654\uc6d0(Maison de la Culture du Japon)\ub3c4 \uc124\ub9bd\ub418\uc5c8\ub2e4. \uc9c0\ubd80\ud2f0, \ub9c8\ub2e4\uac00\uc2a4\uce74\ub974, \uc6b0\uac04\ub2e4 \ub4f1\uc9c0\uc758 \uc5d0\uc774\uc988\ub85c \uc778\ud55c \uc2ec\uac01\ud55c \ubcf4\uac74 \uc0c1\ud669\uacfc \ubbf8\uac1c\ubc1c \uc0c1\ud669\uc744 \ud5a5\uc0c1\ud558\uae30 \uc704\ud558\uc5ec, \ud504\ub791\uc2a4\uc640 \uc77c\ubcf8\uc740 \ud611\ub825\ud558\uace0 \uc788\ub2e4. \uc77c\ubcf8\uacfc \ud504\ub791\uc2a4\ub294 \uc608\uc220, \uc694\ub9ac\uc640 \uac19\uc740 \uc601\uc5ed\uc5d0\uc11c\ub3c4 \uc11c\ub85c \uc544\uc774\ub514\uc5b4\ub97c \uacf5\uc720\ud558\uace0 \uc788\ub294 \uac83\uc73c\ub85c \uc54c\ub824\uc838 \uc788\ub2e4. \u3008\uc694\ub9ac\uc758 \ucca0\uc778(\u6599\u7406\u306e\u9244\u4eba)\u3009\uacfc \uac19\uc740 \ud154\ub808\ube44\uc804 \uc1fc\uc5d0\uc11c \ubcfc \uc218 \uc788\ub4ef\uc774, \ud504\ub791\uc2a4 \uc694\ub9ac\ub294 \uc9c0\ub09c \uc2ed\uc218\ub144\uac04 \uc77c\ubcf8\uc5d0 \ud070 \uc601\ud5a5\uc744 \ub07c\uccd0 \uc654\ub2e4. \uc560\ub2c8\uba54\uc774\uc158\uc740 \ud504\ub791\uc2a4\uc5d0\uc11c\ub3c4 \uc778\uae30\uac00 \uc788\ub294\ub370, \uc911\uc138, \ub974\ub124\uc0c1\uc2a4, \ub098\ud3f4\ub808\uc639 \uc2dc\uae30, \uc138\uacc4\ub300\uc804\uc2dc\uae30\uc5d0 \uc788\uc5b4\uc11c \ud504\ub791\uc2a4\uc758 \uc5ed\uc0ac\uc801 \uc778\ubb3c\uacfc \ubc30\uacbd\ub4e4\uc740 \uc77c\ubcf8\uc758 \uc5d4\ud130\ud14c\uc778\uba3c\ud2b8 \uc0b0\uc5c5\uc5d0 \uc788\uc5b4\uc11c \uc911\uc694\ud55c \uc2a4\ud1a0\ub9ac \ub77c\uc778\uc744 \uad6c\uc131\ud558\uace0 \uc788\ub2e4.", "answer": "\uc694\ub9ac\uc758 \ucca0\uc778", "sentence": "\u3008 \uc694\ub9ac\uc758 \ucca0\uc778 (\u6599\u7406\u306e\u9244\u4eba)\u3009\uacfc \uac19\uc740 \ud154\ub808\ube44\uc804 \uc1fc\uc5d0\uc11c \ubcfc \uc218 \uc788\ub4ef\uc774, \ud504\ub791\uc2a4 \uc694\ub9ac\ub294 \uc9c0\ub09c \uc2ed\uc218\ub144\uac04 \uc77c\ubcf8\uc5d0 \ud070 \uc601\ud5a5\uc744 \ub07c\uccd0 \uc654\ub2e4.", "paragraph_sentence": "\uc591\uad6d \uad00\uacc4\ub294 \uc815\uce58\u30fb\uacbd\uc81c\uc801\uc778 \uba74\ubcf4\ub2e4 \ubb38\ud654\uba74\uc5d0\uc11c\uc758 \uad50\ub958\uac00 \ub354 \ud070 \uacbd\ud5a5\uc5d0 \uc788\ub2e4. \ud504\ub791\uc2a4 \ubb38\ud654\ub294 \ubbf8\uc220\u30fb\uc74c\uc545\u30fb\uc2dd\ubb38\ud654\u30fb\ubb38\uc608 \ub4f1\uc5d0\uc11c \uc77c\ubcf8\uc758 \uadfc\ub300\ud654\uc5d0 \ud070 \uc601\ud5a5\uc744 \ub07c\ucce4\ub2e4. \ub610\ud55c \ud504\ub791\uc2a4\ub294 \uc77c\ubcf8\ubb38\ud654\ub97c \uc9c0\uc9c0\ud558\ub294 \uc0ac\ub78c\ub3c4 \ub9ce\uc544, \ubbf8\uc220\u30fb\uacf5\uc608\u30fb\uc694\ub9ac\u30fb\ub300\uc911\ubb38\ud654 \ub4f1 19\uc138\uae30 \ud6c4\ubc18\uc758 \uc790\ud3ec\ub2c8\uc998\uc744 \uc2dc\uc791\uc73c\ub85c, \ud504\ub791\uc2a4\uc5d0 \ubbf8\uce5c \uc77c\ubcf8\ubb38\ud654\uc758 \uc601\ud5a5\ub3c4 \uc788\ub2e4. \ub610\ud55c \ud30c\ub9ac\uc5d0\ub294 \uc77c\ubcf8 \ubb38\ud654\uc6d0(Maison de la Culture du Japon)\ub3c4 \uc124\ub9bd\ub418\uc5c8\ub2e4. \uc9c0\ubd80\ud2f0, \ub9c8\ub2e4\uac00\uc2a4\uce74\ub974, \uc6b0\uac04\ub2e4 \ub4f1\uc9c0\uc758 \uc5d0\uc774\uc988\ub85c \uc778\ud55c \uc2ec\uac01\ud55c \ubcf4\uac74 \uc0c1\ud669\uacfc \ubbf8\uac1c\ubc1c \uc0c1\ud669\uc744 \ud5a5\uc0c1\ud558\uae30 \uc704\ud558\uc5ec, \ud504\ub791\uc2a4\uc640 \uc77c\ubcf8\uc740 \ud611\ub825\ud558\uace0 \uc788\ub2e4. \uc77c\ubcf8\uacfc \ud504\ub791\uc2a4\ub294 \uc608\uc220, \uc694\ub9ac\uc640 \uac19\uc740 \uc601\uc5ed\uc5d0\uc11c\ub3c4 \uc11c\ub85c \uc544\uc774\ub514\uc5b4\ub97c \uacf5\uc720\ud558\uace0 \uc788\ub294 \uac83\uc73c\ub85c \uc54c\ub824\uc838 \uc788\ub2e4. \u3008 \uc694\ub9ac\uc758 \ucca0\uc778 (\u6599\u7406\u306e\u9244\u4eba)\u3009\uacfc \uac19\uc740 \ud154\ub808\ube44\uc804 \uc1fc\uc5d0\uc11c \ubcfc \uc218 \uc788\ub4ef\uc774, \ud504\ub791\uc2a4 \uc694\ub9ac\ub294 \uc9c0\ub09c \uc2ed\uc218\ub144\uac04 \uc77c\ubcf8\uc5d0 \ud070 \uc601\ud5a5\uc744 \ub07c\uccd0 \uc654\ub2e4. \uc560\ub2c8\uba54\uc774\uc158\uc740 \ud504\ub791\uc2a4\uc5d0\uc11c\ub3c4 \uc778\uae30\uac00 \uc788\ub294\ub370, \uc911\uc138, \ub974\ub124\uc0c1\uc2a4, \ub098\ud3f4\ub808\uc639 \uc2dc\uae30, \uc138\uacc4\ub300\uc804\uc2dc\uae30\uc5d0 \uc788\uc5b4\uc11c \ud504\ub791\uc2a4\uc758 \uc5ed\uc0ac\uc801 \uc778\ubb3c\uacfc \ubc30\uacbd\ub4e4\uc740 \uc77c\ubcf8\uc758 \uc5d4\ud130\ud14c\uc778\uba3c\ud2b8 \uc0b0\uc5c5\uc5d0 \uc788\uc5b4\uc11c \uc911\uc694\ud55c \uc2a4\ud1a0\ub9ac \ub77c\uc778\uc744 \uad6c\uc131\ud558\uace0 \uc788\ub2e4.", "paragraph_answer": "\uc591\uad6d \uad00\uacc4\ub294 \uc815\uce58\u30fb\uacbd\uc81c\uc801\uc778 \uba74\ubcf4\ub2e4 \ubb38\ud654\uba74\uc5d0\uc11c\uc758 \uad50\ub958\uac00 \ub354 \ud070 \uacbd\ud5a5\uc5d0 \uc788\ub2e4. \ud504\ub791\uc2a4 \ubb38\ud654\ub294 \ubbf8\uc220\u30fb\uc74c\uc545\u30fb\uc2dd\ubb38\ud654\u30fb\ubb38\uc608 \ub4f1\uc5d0\uc11c \uc77c\ubcf8\uc758 \uadfc\ub300\ud654\uc5d0 \ud070 \uc601\ud5a5\uc744 \ub07c\ucce4\ub2e4. \ub610\ud55c \ud504\ub791\uc2a4\ub294 \uc77c\ubcf8\ubb38\ud654\ub97c \uc9c0\uc9c0\ud558\ub294 \uc0ac\ub78c\ub3c4 \ub9ce\uc544, \ubbf8\uc220\u30fb\uacf5\uc608\u30fb\uc694\ub9ac\u30fb\ub300\uc911\ubb38\ud654 \ub4f1 19\uc138\uae30 \ud6c4\ubc18\uc758 \uc790\ud3ec\ub2c8\uc998\uc744 \uc2dc\uc791\uc73c\ub85c, \ud504\ub791\uc2a4\uc5d0 \ubbf8\uce5c \uc77c\ubcf8\ubb38\ud654\uc758 \uc601\ud5a5\ub3c4 \uc788\ub2e4. \ub610\ud55c \ud30c\ub9ac\uc5d0\ub294 \uc77c\ubcf8 \ubb38\ud654\uc6d0(Maison de la Culture du Japon)\ub3c4 \uc124\ub9bd\ub418\uc5c8\ub2e4. \uc9c0\ubd80\ud2f0, \ub9c8\ub2e4\uac00\uc2a4\uce74\ub974, \uc6b0\uac04\ub2e4 \ub4f1\uc9c0\uc758 \uc5d0\uc774\uc988\ub85c \uc778\ud55c \uc2ec\uac01\ud55c \ubcf4\uac74 \uc0c1\ud669\uacfc \ubbf8\uac1c\ubc1c \uc0c1\ud669\uc744 \ud5a5\uc0c1\ud558\uae30 \uc704\ud558\uc5ec, \ud504\ub791\uc2a4\uc640 \uc77c\ubcf8\uc740 \ud611\ub825\ud558\uace0 \uc788\ub2e4. \uc77c\ubcf8\uacfc \ud504\ub791\uc2a4\ub294 \uc608\uc220, \uc694\ub9ac\uc640 \uac19\uc740 \uc601\uc5ed\uc5d0\uc11c\ub3c4 \uc11c\ub85c \uc544\uc774\ub514\uc5b4\ub97c \uacf5\uc720\ud558\uace0 \uc788\ub294 \uac83\uc73c\ub85c \uc54c\ub824\uc838 \uc788\ub2e4. \u3008 \uc694\ub9ac\uc758 \ucca0\uc778 (\u6599\u7406\u306e\u9244\u4eba)\u3009\uacfc \uac19\uc740 \ud154\ub808\ube44\uc804 \uc1fc\uc5d0\uc11c \ubcfc \uc218 \uc788\ub4ef\uc774, \ud504\ub791\uc2a4 \uc694\ub9ac\ub294 \uc9c0\ub09c \uc2ed\uc218\ub144\uac04 \uc77c\ubcf8\uc5d0 \ud070 \uc601\ud5a5\uc744 \ub07c\uccd0 \uc654\ub2e4. \uc560\ub2c8\uba54\uc774\uc158\uc740 \ud504\ub791\uc2a4\uc5d0\uc11c\ub3c4 \uc778\uae30\uac00 \uc788\ub294\ub370, \uc911\uc138, \ub974\ub124\uc0c1\uc2a4, \ub098\ud3f4\ub808\uc639 \uc2dc\uae30, \uc138\uacc4\ub300\uc804\uc2dc\uae30\uc5d0 \uc788\uc5b4\uc11c \ud504\ub791\uc2a4\uc758 \uc5ed\uc0ac\uc801 \uc778\ubb3c\uacfc \ubc30\uacbd\ub4e4\uc740 \uc77c\ubcf8\uc758 \uc5d4\ud130\ud14c\uc778\uba3c\ud2b8 \uc0b0\uc5c5\uc5d0 \uc788\uc5b4\uc11c \uc911\uc694\ud55c \uc2a4\ud1a0\ub9ac \ub77c\uc778\uc744 \uad6c\uc131\ud558\uace0 \uc788\ub2e4.", "sentence_answer": "\u3008 \uc694\ub9ac\uc758 \ucca0\uc778 (\u6599\u7406\u306e\u9244\u4eba)\u3009\uacfc \uac19\uc740 \ud154\ub808\ube44\uc804 \uc1fc\uc5d0\uc11c \ubcfc \uc218 \uc788\ub4ef\uc774, \ud504\ub791\uc2a4 \uc694\ub9ac\ub294 \uc9c0\ub09c \uc2ed\uc218\ub144\uac04 \uc77c\ubcf8\uc5d0 \ud070 \uc601\ud5a5\uc744 \ub07c\uccd0 \uc654\ub2e4.", "paragraph_id": "6568624-8-2", "paragraph_question": "question: \ud504\ub791\uc2a4 \uc694\ub9ac\uac00 \uc77c\ubcf8\uc5d0 \ud070 \uc601\ud5a5\uc744 \ub07c\uce5c\uac83\uc744 \uc54c \uc218 \uc788\ub294 \uc77c\ubcf8\uc758 \ud154\ub808\ube44\uc83c \uc1fc\ub294?, context: \uc591\uad6d \uad00\uacc4\ub294 \uc815\uce58\u30fb\uacbd\uc81c\uc801\uc778 \uba74\ubcf4\ub2e4 \ubb38\ud654\uba74\uc5d0\uc11c\uc758 \uad50\ub958\uac00 \ub354 \ud070 \uacbd\ud5a5\uc5d0 \uc788\ub2e4. \ud504\ub791\uc2a4 \ubb38\ud654\ub294 \ubbf8\uc220\u30fb\uc74c\uc545\u30fb\uc2dd\ubb38\ud654\u30fb\ubb38\uc608 \ub4f1\uc5d0\uc11c \uc77c\ubcf8\uc758 \uadfc\ub300\ud654\uc5d0 \ud070 \uc601\ud5a5\uc744 \ub07c\ucce4\ub2e4. \ub610\ud55c \ud504\ub791\uc2a4\ub294 \uc77c\ubcf8\ubb38\ud654\ub97c \uc9c0\uc9c0\ud558\ub294 \uc0ac\ub78c\ub3c4 \ub9ce\uc544, \ubbf8\uc220\u30fb\uacf5\uc608\u30fb\uc694\ub9ac\u30fb\ub300\uc911\ubb38\ud654 \ub4f1 19\uc138\uae30 \ud6c4\ubc18\uc758 \uc790\ud3ec\ub2c8\uc998\uc744 \uc2dc\uc791\uc73c\ub85c, \ud504\ub791\uc2a4\uc5d0 \ubbf8\uce5c \uc77c\ubcf8\ubb38\ud654\uc758 \uc601\ud5a5\ub3c4 \uc788\ub2e4. \ub610\ud55c \ud30c\ub9ac\uc5d0\ub294 \uc77c\ubcf8 \ubb38\ud654\uc6d0(Maison de la Culture du Japon)\ub3c4 \uc124\ub9bd\ub418\uc5c8\ub2e4. \uc9c0\ubd80\ud2f0, \ub9c8\ub2e4\uac00\uc2a4\uce74\ub974, \uc6b0\uac04\ub2e4 \ub4f1\uc9c0\uc758 \uc5d0\uc774\uc988\ub85c \uc778\ud55c \uc2ec\uac01\ud55c \ubcf4\uac74 \uc0c1\ud669\uacfc \ubbf8\uac1c\ubc1c \uc0c1\ud669\uc744 \ud5a5\uc0c1\ud558\uae30 \uc704\ud558\uc5ec, \ud504\ub791\uc2a4\uc640 \uc77c\ubcf8\uc740 \ud611\ub825\ud558\uace0 \uc788\ub2e4. \uc77c\ubcf8\uacfc \ud504\ub791\uc2a4\ub294 \uc608\uc220, \uc694\ub9ac\uc640 \uac19\uc740 \uc601\uc5ed\uc5d0\uc11c\ub3c4 \uc11c\ub85c \uc544\uc774\ub514\uc5b4\ub97c \uacf5\uc720\ud558\uace0 \uc788\ub294 \uac83\uc73c\ub85c \uc54c\ub824\uc838 \uc788\ub2e4. \u3008\uc694\ub9ac\uc758 \ucca0\uc778(\u6599\u7406\u306e\u9244\u4eba)\u3009\uacfc \uac19\uc740 \ud154\ub808\ube44\uc804 \uc1fc\uc5d0\uc11c \ubcfc \uc218 \uc788\ub4ef\uc774, \ud504\ub791\uc2a4 \uc694\ub9ac\ub294 \uc9c0\ub09c \uc2ed\uc218\ub144\uac04 \uc77c\ubcf8\uc5d0 \ud070 \uc601\ud5a5\uc744 \ub07c\uccd0 \uc654\ub2e4. \uc560\ub2c8\uba54\uc774\uc158\uc740 \ud504\ub791\uc2a4\uc5d0\uc11c\ub3c4 \uc778\uae30\uac00 \uc788\ub294\ub370, \uc911\uc138, \ub974\ub124\uc0c1\uc2a4, \ub098\ud3f4\ub808\uc639 \uc2dc\uae30, \uc138\uacc4\ub300\uc804\uc2dc\uae30\uc5d0 \uc788\uc5b4\uc11c \ud504\ub791\uc2a4\uc758 \uc5ed\uc0ac\uc801 \uc778\ubb3c\uacfc \ubc30\uacbd\ub4e4\uc740 \uc77c\ubcf8\uc758 \uc5d4\ud130\ud14c\uc778\uba3c\ud2b8 \uc0b0\uc5c5\uc5d0 \uc788\uc5b4\uc11c \uc911\uc694\ud55c \uc2a4\ud1a0\ub9ac \ub77c\uc778\uc744 \uad6c\uc131\ud558\uace0 \uc788\ub2e4."} {"question": "\uc138\ubc88\uc9f8 \ud0d0\uc0ac \uae30\uac04\uc740?", "paragraph": "1970\ub144\ub300 \ucd08\ubc18 \ud30c\uc774\uc5b4\ub2c8\uc5b4 10\ud638\uc640 11\ud638\uac00 \ubaa9\uc131\uc744 \uadfc\uc811 \ud1b5\uacfc\ud558\uba74\uc11c \uce7c\ub9ac\uc2a4\ud1a0\uc758 \uc815\ubcf4\ub97c \ubcf4\ub0b4\uc654\uc73c\ub098, \ub300\ubd80\ubd84 \uc9c0\uad6c\uc5d0\uc11c\uc758 \uad00\uce21\uc744 \ud1b5\ud574 \uc54c\ub824\uc838 \uc788\ub358 \uc815\ubcf4\ub4e4\uc774\uc5c8\ub2e4. \uc0c8\ub85c\uc6b4 \uc815\ubcf4\ub4e4\uc740 1979\ub144 \ubcf4\uc774\uc800 1\ud638\uc640 2\ud638\uac00 \ubaa9\uc131\uc744 \uc9c0\ub098\uac00\uba74\uc11c \ubcf4\ub0b4\uc654\ub294\ub370, \ubcf4\uc774\uc800 \ud0d0\uc0ac\uc120\ub4e4\uc740 \uce7c\ub9ac\uc2a4\ud1a0\uc758 \ud45c\uba74\uc744 1~2 km\uc758 \ud574\uc0c1\ub3c4\ub85c \ubc18 \ub118\uac8c \ucc0d\uc5c8\uace0, \uce7c\ub9ac\uc2a4\ud1a0\uc758 \uc628\ub3c4, \uc9c8\ub7c9, \ubaa8\uc591\uc744 \uc815\ud655\ud788 \uce21\uc815\ud588\ub2e4. \uc138 \ubc88\uc9f8 \ud0d0\uc0ac\ub294 1994\ub144\ubd80\ud130 2003\ub144\uae4c\uc9c0\uc600\ub2e4. \uac08\ub9b4\ub808\uc624 \ud0d0\uc0ac\uc120\uc740 \uce7c\ub9ac\uc2a4\ud1a0\ub97c 8\ubc88 \uadfc\uc811 \ud1b5\uacfc\ud558\uba70 \uce7c\ub9ac\uc2a4\ud1a0\ub97c \ud0d0\uc0ac\ud588\ub2e4. \ub9c8\uc9c0\ub9c9 \uadfc\uc811 \ud1b5\uacfc\ub294 2001\ub144 C30 \uadfc\uc811\ud1b5\uacfc \ub54c\uc600\uace0, \ud45c\uba74\uc5d0\uc11c 138 km \ub5a8\uc5b4\uc9c4 \uacf3\uc744 \uc9c0\ub0ac\ub2e4. \uac08\ub9b4\ub808\uc624 \ud0d0\uc0ac\uc120\uc740 \uce7c\ub9ac\uc2a4\ud1a0\uc758 \ud45c\uba74 \uc9c0\ub3c4\ub97c \uc644\uc131\ud588\uace0, \ubaa9\ud45c \uc9c0\uc810\uc744 15 m\uc758 \ud574\uc0c1\ub3c4\ub85c \ucc0d\uc5b4 \ubcf4\ub0b4\uc654\ub2e4. 2000\ub144 \ud1a0\uc131\uc73c\ub85c \uac00\ub358 \uce74\uc2dc\ub2c8 \ud0d0\uc0ac\uc120\uc740 \uce7c\ub9ac\uc2a4\ud1a0\ub97c \ud3ec\ud568\ud55c \uac08\ub9b4\ub808\uc774 \uc704\uc131\ub4e4\uc758 \uace0\ud574\uc0c1\ub3c4 \uc801\uc678\uc120 \uc0ac\uc9c4\uc744 \ubcf4\ub0b4\uc654\ub2e4. 2007\ub144 2~3\uc6d4, \uba85\uc655\uc131\uc73c\ub85c \ud5a5\ud558\ub358 \ub274 \ud5c8\ub77c\uc774\uc98c\uc2a4 \ud0d0\uc0ac\uc120\uc740 \uce7c\ub9ac\uc2a4\ud1a0\uc758 \uc0ac\uc9c4\uc744 \ubcf4\ub0b4\uc654\ub2e4.", "answer": "1994\ub144\ubd80\ud130 2003\ub144\uae4c\uc9c0", "sentence": "\uc138 \ubc88\uc9f8 \ud0d0\uc0ac\ub294 1994\ub144\ubd80\ud130 2003\ub144\uae4c\uc9c0 \uc600\ub2e4.", "paragraph_sentence": "1970\ub144\ub300 \ucd08\ubc18 \ud30c\uc774\uc5b4\ub2c8\uc5b4 10\ud638\uc640 11\ud638\uac00 \ubaa9\uc131\uc744 \uadfc\uc811 \ud1b5\uacfc\ud558\uba74\uc11c \uce7c\ub9ac\uc2a4\ud1a0\uc758 \uc815\ubcf4\ub97c \ubcf4\ub0b4\uc654\uc73c\ub098, \ub300\ubd80\ubd84 \uc9c0\uad6c\uc5d0\uc11c\uc758 \uad00\uce21\uc744 \ud1b5\ud574 \uc54c\ub824\uc838 \uc788\ub358 \uc815\ubcf4\ub4e4\uc774\uc5c8\ub2e4. \uc0c8\ub85c\uc6b4 \uc815\ubcf4\ub4e4\uc740 1979\ub144 \ubcf4\uc774\uc800 1\ud638\uc640 2\ud638\uac00 \ubaa9\uc131\uc744 \uc9c0\ub098\uac00\uba74\uc11c \ubcf4\ub0b4\uc654\ub294\ub370, \ubcf4\uc774\uc800 \ud0d0\uc0ac\uc120\ub4e4\uc740 \uce7c\ub9ac\uc2a4\ud1a0\uc758 \ud45c\uba74\uc744 1~2 km\uc758 \ud574\uc0c1\ub3c4\ub85c \ubc18 \ub118\uac8c \ucc0d\uc5c8\uace0, \uce7c\ub9ac\uc2a4\ud1a0\uc758 \uc628\ub3c4, \uc9c8\ub7c9, \ubaa8\uc591\uc744 \uc815\ud655\ud788 \uce21\uc815\ud588\ub2e4. \uc138 \ubc88\uc9f8 \ud0d0\uc0ac\ub294 1994\ub144\ubd80\ud130 2003\ub144\uae4c\uc9c0 \uc600\ub2e4. \uac08\ub9b4\ub808\uc624 \ud0d0\uc0ac\uc120\uc740 \uce7c\ub9ac\uc2a4\ud1a0\ub97c 8\ubc88 \uadfc\uc811 \ud1b5\uacfc\ud558\uba70 \uce7c\ub9ac\uc2a4\ud1a0\ub97c \ud0d0\uc0ac\ud588\ub2e4. \ub9c8\uc9c0\ub9c9 \uadfc\uc811 \ud1b5\uacfc\ub294 2001\ub144 C30 \uadfc\uc811\ud1b5\uacfc \ub54c\uc600\uace0, \ud45c\uba74\uc5d0\uc11c 138 km \ub5a8\uc5b4\uc9c4 \uacf3\uc744 \uc9c0\ub0ac\ub2e4. \uac08\ub9b4\ub808\uc624 \ud0d0\uc0ac\uc120\uc740 \uce7c\ub9ac\uc2a4\ud1a0\uc758 \ud45c\uba74 \uc9c0\ub3c4\ub97c \uc644\uc131\ud588\uace0, \ubaa9\ud45c \uc9c0\uc810\uc744 15 m\uc758 \ud574\uc0c1\ub3c4\ub85c \ucc0d\uc5b4 \ubcf4\ub0b4\uc654\ub2e4. 2000\ub144 \ud1a0\uc131\uc73c\ub85c \uac00\ub358 \uce74\uc2dc\ub2c8 \ud0d0\uc0ac\uc120\uc740 \uce7c\ub9ac\uc2a4\ud1a0\ub97c \ud3ec\ud568\ud55c \uac08\ub9b4\ub808\uc774 \uc704\uc131\ub4e4\uc758 \uace0\ud574\uc0c1\ub3c4 \uc801\uc678\uc120 \uc0ac\uc9c4\uc744 \ubcf4\ub0b4\uc654\ub2e4. 2007\ub144 2~3\uc6d4, \uba85\uc655\uc131\uc73c\ub85c \ud5a5\ud558\ub358 \ub274 \ud5c8\ub77c\uc774\uc98c\uc2a4 \ud0d0\uc0ac\uc120\uc740 \uce7c\ub9ac\uc2a4\ud1a0\uc758 \uc0ac\uc9c4\uc744 \ubcf4\ub0b4\uc654\ub2e4.", "paragraph_answer": "1970\ub144\ub300 \ucd08\ubc18 \ud30c\uc774\uc5b4\ub2c8\uc5b4 10\ud638\uc640 11\ud638\uac00 \ubaa9\uc131\uc744 \uadfc\uc811 \ud1b5\uacfc\ud558\uba74\uc11c \uce7c\ub9ac\uc2a4\ud1a0\uc758 \uc815\ubcf4\ub97c \ubcf4\ub0b4\uc654\uc73c\ub098, \ub300\ubd80\ubd84 \uc9c0\uad6c\uc5d0\uc11c\uc758 \uad00\uce21\uc744 \ud1b5\ud574 \uc54c\ub824\uc838 \uc788\ub358 \uc815\ubcf4\ub4e4\uc774\uc5c8\ub2e4. \uc0c8\ub85c\uc6b4 \uc815\ubcf4\ub4e4\uc740 1979\ub144 \ubcf4\uc774\uc800 1\ud638\uc640 2\ud638\uac00 \ubaa9\uc131\uc744 \uc9c0\ub098\uac00\uba74\uc11c \ubcf4\ub0b4\uc654\ub294\ub370, \ubcf4\uc774\uc800 \ud0d0\uc0ac\uc120\ub4e4\uc740 \uce7c\ub9ac\uc2a4\ud1a0\uc758 \ud45c\uba74\uc744 1~2 km\uc758 \ud574\uc0c1\ub3c4\ub85c \ubc18 \ub118\uac8c \ucc0d\uc5c8\uace0, \uce7c\ub9ac\uc2a4\ud1a0\uc758 \uc628\ub3c4, \uc9c8\ub7c9, \ubaa8\uc591\uc744 \uc815\ud655\ud788 \uce21\uc815\ud588\ub2e4. \uc138 \ubc88\uc9f8 \ud0d0\uc0ac\ub294 1994\ub144\ubd80\ud130 2003\ub144\uae4c\uc9c0 \uc600\ub2e4. \uac08\ub9b4\ub808\uc624 \ud0d0\uc0ac\uc120\uc740 \uce7c\ub9ac\uc2a4\ud1a0\ub97c 8\ubc88 \uadfc\uc811 \ud1b5\uacfc\ud558\uba70 \uce7c\ub9ac\uc2a4\ud1a0\ub97c \ud0d0\uc0ac\ud588\ub2e4. \ub9c8\uc9c0\ub9c9 \uadfc\uc811 \ud1b5\uacfc\ub294 2001\ub144 C30 \uadfc\uc811\ud1b5\uacfc \ub54c\uc600\uace0, \ud45c\uba74\uc5d0\uc11c 138 km \ub5a8\uc5b4\uc9c4 \uacf3\uc744 \uc9c0\ub0ac\ub2e4. \uac08\ub9b4\ub808\uc624 \ud0d0\uc0ac\uc120\uc740 \uce7c\ub9ac\uc2a4\ud1a0\uc758 \ud45c\uba74 \uc9c0\ub3c4\ub97c \uc644\uc131\ud588\uace0, \ubaa9\ud45c \uc9c0\uc810\uc744 15 m\uc758 \ud574\uc0c1\ub3c4\ub85c \ucc0d\uc5b4 \ubcf4\ub0b4\uc654\ub2e4. 2000\ub144 \ud1a0\uc131\uc73c\ub85c \uac00\ub358 \uce74\uc2dc\ub2c8 \ud0d0\uc0ac\uc120\uc740 \uce7c\ub9ac\uc2a4\ud1a0\ub97c \ud3ec\ud568\ud55c \uac08\ub9b4\ub808\uc774 \uc704\uc131\ub4e4\uc758 \uace0\ud574\uc0c1\ub3c4 \uc801\uc678\uc120 \uc0ac\uc9c4\uc744 \ubcf4\ub0b4\uc654\ub2e4. 2007\ub144 2~3\uc6d4, \uba85\uc655\uc131\uc73c\ub85c \ud5a5\ud558\ub358 \ub274 \ud5c8\ub77c\uc774\uc98c\uc2a4 \ud0d0\uc0ac\uc120\uc740 \uce7c\ub9ac\uc2a4\ud1a0\uc758 \uc0ac\uc9c4\uc744 \ubcf4\ub0b4\uc654\ub2e4.", "sentence_answer": "\uc138 \ubc88\uc9f8 \ud0d0\uc0ac\ub294 1994\ub144\ubd80\ud130 2003\ub144\uae4c\uc9c0 \uc600\ub2e4.", "paragraph_id": "6208599-11-2", "paragraph_question": "question: \uc138\ubc88\uc9f8 \ud0d0\uc0ac \uae30\uac04\uc740?, context: 1970\ub144\ub300 \ucd08\ubc18 \ud30c\uc774\uc5b4\ub2c8\uc5b4 10\ud638\uc640 11\ud638\uac00 \ubaa9\uc131\uc744 \uadfc\uc811 \ud1b5\uacfc\ud558\uba74\uc11c \uce7c\ub9ac\uc2a4\ud1a0\uc758 \uc815\ubcf4\ub97c \ubcf4\ub0b4\uc654\uc73c\ub098, \ub300\ubd80\ubd84 \uc9c0\uad6c\uc5d0\uc11c\uc758 \uad00\uce21\uc744 \ud1b5\ud574 \uc54c\ub824\uc838 \uc788\ub358 \uc815\ubcf4\ub4e4\uc774\uc5c8\ub2e4. \uc0c8\ub85c\uc6b4 \uc815\ubcf4\ub4e4\uc740 1979\ub144 \ubcf4\uc774\uc800 1\ud638\uc640 2\ud638\uac00 \ubaa9\uc131\uc744 \uc9c0\ub098\uac00\uba74\uc11c \ubcf4\ub0b4\uc654\ub294\ub370, \ubcf4\uc774\uc800 \ud0d0\uc0ac\uc120\ub4e4\uc740 \uce7c\ub9ac\uc2a4\ud1a0\uc758 \ud45c\uba74\uc744 1~2 km\uc758 \ud574\uc0c1\ub3c4\ub85c \ubc18 \ub118\uac8c \ucc0d\uc5c8\uace0, \uce7c\ub9ac\uc2a4\ud1a0\uc758 \uc628\ub3c4, \uc9c8\ub7c9, \ubaa8\uc591\uc744 \uc815\ud655\ud788 \uce21\uc815\ud588\ub2e4. \uc138 \ubc88\uc9f8 \ud0d0\uc0ac\ub294 1994\ub144\ubd80\ud130 2003\ub144\uae4c\uc9c0\uc600\ub2e4. \uac08\ub9b4\ub808\uc624 \ud0d0\uc0ac\uc120\uc740 \uce7c\ub9ac\uc2a4\ud1a0\ub97c 8\ubc88 \uadfc\uc811 \ud1b5\uacfc\ud558\uba70 \uce7c\ub9ac\uc2a4\ud1a0\ub97c \ud0d0\uc0ac\ud588\ub2e4. \ub9c8\uc9c0\ub9c9 \uadfc\uc811 \ud1b5\uacfc\ub294 2001\ub144 C30 \uadfc\uc811\ud1b5\uacfc \ub54c\uc600\uace0, \ud45c\uba74\uc5d0\uc11c 138 km \ub5a8\uc5b4\uc9c4 \uacf3\uc744 \uc9c0\ub0ac\ub2e4. \uac08\ub9b4\ub808\uc624 \ud0d0\uc0ac\uc120\uc740 \uce7c\ub9ac\uc2a4\ud1a0\uc758 \ud45c\uba74 \uc9c0\ub3c4\ub97c \uc644\uc131\ud588\uace0, \ubaa9\ud45c \uc9c0\uc810\uc744 15 m\uc758 \ud574\uc0c1\ub3c4\ub85c \ucc0d\uc5b4 \ubcf4\ub0b4\uc654\ub2e4. 2000\ub144 \ud1a0\uc131\uc73c\ub85c \uac00\ub358 \uce74\uc2dc\ub2c8 \ud0d0\uc0ac\uc120\uc740 \uce7c\ub9ac\uc2a4\ud1a0\ub97c \ud3ec\ud568\ud55c \uac08\ub9b4\ub808\uc774 \uc704\uc131\ub4e4\uc758 \uace0\ud574\uc0c1\ub3c4 \uc801\uc678\uc120 \uc0ac\uc9c4\uc744 \ubcf4\ub0b4\uc654\ub2e4. 2007\ub144 2~3\uc6d4, \uba85\uc655\uc131\uc73c\ub85c \ud5a5\ud558\ub358 \ub274 \ud5c8\ub77c\uc774\uc98c\uc2a4 \ud0d0\uc0ac\uc120\uc740 \uce7c\ub9ac\uc2a4\ud1a0\uc758 \uc0ac\uc9c4\uc744 \ubcf4\ub0b4\uc654\ub2e4."} {"question": "\ud50c\ub8e8\ud1a0\ub284\uc758 \uc874\uc7ac\uac00 \ubc1c\uacac\ub41c \ud574\ub294 \uc5b8\uc81c\uc778\uac00?", "paragraph": "\uadf8\ub7ec\ub098, \uc5ec\uc804\ud788 \uc54c\ub824\uc9c0\uc9c0 \uc54a\uc740 \uac83\ub4e4\uc774 \uc788\uc5c8\ub2e4. \uc21c\uc218\ud55c \uc6b0\ub77c\ub284-235\uc758 \ubb3c\ub9ac\uc801 \uc131\uc9c8\uc740 \uc544\uc9c1 \uc644\uc804\ud788 \ubc1d\ud600\uc9c0\uc9c0 \uc54a\uc558\uace0, \ud50c\ub8e8\ud1a0\ub284\uc758 \uc874\uc7ac\ub294 1941\ub144\uc5d0 \uc640\uc11c\uc57c \uae00\ub80c \uc2dc\ubcf4\uadf8\uc758 \uc5f0\uad6c\ud300\uc774 \ubc1c\uacac\ud558\uc600\ub2e4. \ubc84\ud074\ub9ac\uc758 \uacfc\ud559\uc790\ub4e4\uc740 \uc6b0\ub77c\ub284-235\uc758 \ud575\ubd84\uc5f4\uc5d0 \uc758\ud55c \ud3ed\ubc1c\uacfc \uc6b0\ub77c\ub284-238\uc744 \ud50c\ub8e8\ud1a0\ub284\uc73c\ub85c \ubcc0\ud658\uc2dc\ud0a8\ud6c4 \ud575\ubd84\uc5f4\uc744 \uc77c\uc73c\ud0a4\ub294 \ubc29\uc2dd\uc758 \ud3ed\ubc1c\uc744 \ub193\uace0 \ud68c\uc758\ub97c \uacc4\uc18d\ud558\uc600\ub2e4. \uac8c\ub2e4\uac00 \ub2f9\uc2dc\uc5d0 \ud50c\ub8e8\ud1a0\ub284\uc740 \uc0ac\uc774\ud074\ub85c\ud2b8\ub860\uc5d0\uc11c \uc18c\ub7c9\uc774 \uac80\ucd9c\ub41c \uc218\uc900\uc5d0 \ubd88\uacfc\ud558\uc600\ub2e4. \uc2ec\uc9c0\uc5b4 1943\ub144 12\uc6d4\uae4c\uc9c0\ub3c4 \ube44\ucd95\ub41c \ud50c\ub8e8\ud1a0\ub284\uc758 \uc591\uc740 2 \ubc00\ub9ac\uadf8\ub7a8\uc5d0 \ubd88\uacfc\ud558\uc600\ub2e4. \ub610\ud55c, \uc784\uacc4 \uc9c8\ub7c9\uc5d0 \uc774\ub974\uac8c \ud558\ub294 \ubc29\ubc95 \uc5ed\uc2dc \ub2e4\uc591\ud55c \uac83\ub4e4\uc774 \uc81c\uc2dc\ub418\uc5c8\ub2e4. \ud55c \ucabd\uc5d0 \"\ubc18\uc751\ubb3c\"\uc744 \ubaa8\uc544\ub193\uace0 \uc6d0\ud1b5 \ubaa8\uc591 \ud50c\ub7ec\uadf8\uc778 \"\uac04\uc12d\uc790\"\ub97c \uc3d8\uc544 \uc784\uacc4 \uc9c8\ub7c9\uc5d0 \ub3c4\ub2ec\ud558\ub294 \ubc29\ubc95\uc5d0\uc11c\ubd80\ud130 \ub458\ub85c \ub098\ub25c \ubc18\uc751\ubb3c\uc744 \uc11c\ub85c \ubbf8\ub04c\uc5b4\uc9c0\ub3c4\ub85d \uac15\ud558\uac8c \uacb0\ud569\uc2dc\ucf1c \uc784\uacc4 \uc9c8\ub7c9\uc5d0 \ub3c4\ub2ec\ud558\ub294 \ubc29\ubc95\uae4c\uc9c0 \ub2e4\uc591\ud55c \ubc29\ubc95\uc774 \uc81c\uc2dc\ub418\uc5c8\ub2e4. \ud3ed\ubc1c \uc591\uc0c1\uc5d0 \ub300\ud574\uc11c\ub294 \ub9ac\ucc98\ub4dc \ud1a8\ub9cc\uc740 \ud0c0\uc6d0\ud615 \ud3ed\ubc1c\uc744 \uc81c\uc2dc\ud558\uc600\ub294\ub370, \uadf8\ub294 \ud0c0\uc6d0\ud615 \ud3ed\ubc1c\uc774 \uc790\uccb4 \ucd09\ub9e4 \uc791\uc6a9\uc744 \ubd88\ub7ec\uc62c \uac00\ub2a5\uc131\uc774 \uc788\uae30 \ub54c\ubb38\uc5d0 \ud3ed\ubc1c\ub825\uc744 \ubcf4\ub2e4 \uac15\ud654\uc2dc\ud0ac \uac83\uc73c\ub85c \ubcf4\uc558\ub2e4.", "answer": "1941\ub144", "sentence": "\uc21c\uc218\ud55c \uc6b0\ub77c\ub284-235\uc758 \ubb3c\ub9ac\uc801 \uc131\uc9c8\uc740 \uc544\uc9c1 \uc644\uc804\ud788 \ubc1d\ud600\uc9c0\uc9c0 \uc54a\uc558\uace0, \ud50c\ub8e8\ud1a0\ub284\uc758 \uc874\uc7ac\ub294 1941\ub144 \uc5d0 \uc640\uc11c\uc57c \uae00\ub80c \uc2dc\ubcf4\uadf8\uc758 \uc5f0\uad6c\ud300\uc774 \ubc1c\uacac\ud558\uc600\ub2e4.", "paragraph_sentence": "\uadf8\ub7ec\ub098, \uc5ec\uc804\ud788 \uc54c\ub824\uc9c0\uc9c0 \uc54a\uc740 \uac83\ub4e4\uc774 \uc788\uc5c8\ub2e4. \uc21c\uc218\ud55c \uc6b0\ub77c\ub284-235\uc758 \ubb3c\ub9ac\uc801 \uc131\uc9c8\uc740 \uc544\uc9c1 \uc644\uc804\ud788 \ubc1d\ud600\uc9c0\uc9c0 \uc54a\uc558\uace0, \ud50c\ub8e8\ud1a0\ub284\uc758 \uc874\uc7ac\ub294 1941\ub144 \uc5d0 \uc640\uc11c\uc57c \uae00\ub80c \uc2dc\ubcf4\uadf8\uc758 \uc5f0\uad6c\ud300\uc774 \ubc1c\uacac\ud558\uc600\ub2e4. \ubc84\ud074\ub9ac\uc758 \uacfc\ud559\uc790\ub4e4\uc740 \uc6b0\ub77c\ub284-235\uc758 \ud575\ubd84\uc5f4\uc5d0 \uc758\ud55c \ud3ed\ubc1c\uacfc \uc6b0\ub77c\ub284-238\uc744 \ud50c\ub8e8\ud1a0\ub284\uc73c\ub85c \ubcc0\ud658\uc2dc\ud0a8\ud6c4 \ud575\ubd84\uc5f4\uc744 \uc77c\uc73c\ud0a4\ub294 \ubc29\uc2dd\uc758 \ud3ed\ubc1c\uc744 \ub193\uace0 \ud68c\uc758\ub97c \uacc4\uc18d\ud558\uc600\ub2e4. \uac8c\ub2e4\uac00 \ub2f9\uc2dc\uc5d0 \ud50c\ub8e8\ud1a0\ub284\uc740 \uc0ac\uc774\ud074\ub85c\ud2b8\ub860\uc5d0\uc11c \uc18c\ub7c9\uc774 \uac80\ucd9c\ub41c \uc218\uc900\uc5d0 \ubd88\uacfc\ud558\uc600\ub2e4. \uc2ec\uc9c0\uc5b4 1943\ub144 12\uc6d4\uae4c\uc9c0\ub3c4 \ube44\ucd95\ub41c \ud50c\ub8e8\ud1a0\ub284\uc758 \uc591\uc740 2 \ubc00\ub9ac\uadf8\ub7a8\uc5d0 \ubd88\uacfc\ud558\uc600\ub2e4. \ub610\ud55c, \uc784\uacc4 \uc9c8\ub7c9\uc5d0 \uc774\ub974\uac8c \ud558\ub294 \ubc29\ubc95 \uc5ed\uc2dc \ub2e4\uc591\ud55c \uac83\ub4e4\uc774 \uc81c\uc2dc\ub418\uc5c8\ub2e4. \ud55c \ucabd\uc5d0 \"\ubc18\uc751\ubb3c\"\uc744 \ubaa8\uc544\ub193\uace0 \uc6d0\ud1b5 \ubaa8\uc591 \ud50c\ub7ec\uadf8\uc778 \"\uac04\uc12d\uc790\"\ub97c \uc3d8\uc544 \uc784\uacc4 \uc9c8\ub7c9\uc5d0 \ub3c4\ub2ec\ud558\ub294 \ubc29\ubc95\uc5d0\uc11c\ubd80\ud130 \ub458\ub85c \ub098\ub25c \ubc18\uc751\ubb3c\uc744 \uc11c\ub85c \ubbf8\ub04c\uc5b4\uc9c0\ub3c4\ub85d \uac15\ud558\uac8c \uacb0\ud569\uc2dc\ucf1c \uc784\uacc4 \uc9c8\ub7c9\uc5d0 \ub3c4\ub2ec\ud558\ub294 \ubc29\ubc95\uae4c\uc9c0 \ub2e4\uc591\ud55c \ubc29\ubc95\uc774 \uc81c\uc2dc\ub418\uc5c8\ub2e4. \ud3ed\ubc1c \uc591\uc0c1\uc5d0 \ub300\ud574\uc11c\ub294 \ub9ac\ucc98\ub4dc \ud1a8\ub9cc\uc740 \ud0c0\uc6d0\ud615 \ud3ed\ubc1c\uc744 \uc81c\uc2dc\ud558\uc600\ub294\ub370, \uadf8\ub294 \ud0c0\uc6d0\ud615 \ud3ed\ubc1c\uc774 \uc790\uccb4 \ucd09\ub9e4 \uc791\uc6a9\uc744 \ubd88\ub7ec\uc62c \uac00\ub2a5\uc131\uc774 \uc788\uae30 \ub54c\ubb38\uc5d0 \ud3ed\ubc1c\ub825\uc744 \ubcf4\ub2e4 \uac15\ud654\uc2dc\ud0ac \uac83\uc73c\ub85c \ubcf4\uc558\ub2e4.", "paragraph_answer": "\uadf8\ub7ec\ub098, \uc5ec\uc804\ud788 \uc54c\ub824\uc9c0\uc9c0 \uc54a\uc740 \uac83\ub4e4\uc774 \uc788\uc5c8\ub2e4. \uc21c\uc218\ud55c \uc6b0\ub77c\ub284-235\uc758 \ubb3c\ub9ac\uc801 \uc131\uc9c8\uc740 \uc544\uc9c1 \uc644\uc804\ud788 \ubc1d\ud600\uc9c0\uc9c0 \uc54a\uc558\uace0, \ud50c\ub8e8\ud1a0\ub284\uc758 \uc874\uc7ac\ub294 1941\ub144 \uc5d0 \uc640\uc11c\uc57c \uae00\ub80c \uc2dc\ubcf4\uadf8\uc758 \uc5f0\uad6c\ud300\uc774 \ubc1c\uacac\ud558\uc600\ub2e4. \ubc84\ud074\ub9ac\uc758 \uacfc\ud559\uc790\ub4e4\uc740 \uc6b0\ub77c\ub284-235\uc758 \ud575\ubd84\uc5f4\uc5d0 \uc758\ud55c \ud3ed\ubc1c\uacfc \uc6b0\ub77c\ub284-238\uc744 \ud50c\ub8e8\ud1a0\ub284\uc73c\ub85c \ubcc0\ud658\uc2dc\ud0a8\ud6c4 \ud575\ubd84\uc5f4\uc744 \uc77c\uc73c\ud0a4\ub294 \ubc29\uc2dd\uc758 \ud3ed\ubc1c\uc744 \ub193\uace0 \ud68c\uc758\ub97c \uacc4\uc18d\ud558\uc600\ub2e4. \uac8c\ub2e4\uac00 \ub2f9\uc2dc\uc5d0 \ud50c\ub8e8\ud1a0\ub284\uc740 \uc0ac\uc774\ud074\ub85c\ud2b8\ub860\uc5d0\uc11c \uc18c\ub7c9\uc774 \uac80\ucd9c\ub41c \uc218\uc900\uc5d0 \ubd88\uacfc\ud558\uc600\ub2e4. \uc2ec\uc9c0\uc5b4 1943\ub144 12\uc6d4\uae4c\uc9c0\ub3c4 \ube44\ucd95\ub41c \ud50c\ub8e8\ud1a0\ub284\uc758 \uc591\uc740 2 \ubc00\ub9ac\uadf8\ub7a8\uc5d0 \ubd88\uacfc\ud558\uc600\ub2e4. \ub610\ud55c, \uc784\uacc4 \uc9c8\ub7c9\uc5d0 \uc774\ub974\uac8c \ud558\ub294 \ubc29\ubc95 \uc5ed\uc2dc \ub2e4\uc591\ud55c \uac83\ub4e4\uc774 \uc81c\uc2dc\ub418\uc5c8\ub2e4. \ud55c \ucabd\uc5d0 \"\ubc18\uc751\ubb3c\"\uc744 \ubaa8\uc544\ub193\uace0 \uc6d0\ud1b5 \ubaa8\uc591 \ud50c\ub7ec\uadf8\uc778 \"\uac04\uc12d\uc790\"\ub97c \uc3d8\uc544 \uc784\uacc4 \uc9c8\ub7c9\uc5d0 \ub3c4\ub2ec\ud558\ub294 \ubc29\ubc95\uc5d0\uc11c\ubd80\ud130 \ub458\ub85c \ub098\ub25c \ubc18\uc751\ubb3c\uc744 \uc11c\ub85c \ubbf8\ub04c\uc5b4\uc9c0\ub3c4\ub85d \uac15\ud558\uac8c \uacb0\ud569\uc2dc\ucf1c \uc784\uacc4 \uc9c8\ub7c9\uc5d0 \ub3c4\ub2ec\ud558\ub294 \ubc29\ubc95\uae4c\uc9c0 \ub2e4\uc591\ud55c \ubc29\ubc95\uc774 \uc81c\uc2dc\ub418\uc5c8\ub2e4. \ud3ed\ubc1c \uc591\uc0c1\uc5d0 \ub300\ud574\uc11c\ub294 \ub9ac\ucc98\ub4dc \ud1a8\ub9cc\uc740 \ud0c0\uc6d0\ud615 \ud3ed\ubc1c\uc744 \uc81c\uc2dc\ud558\uc600\ub294\ub370, \uadf8\ub294 \ud0c0\uc6d0\ud615 \ud3ed\ubc1c\uc774 \uc790\uccb4 \ucd09\ub9e4 \uc791\uc6a9\uc744 \ubd88\ub7ec\uc62c \uac00\ub2a5\uc131\uc774 \uc788\uae30 \ub54c\ubb38\uc5d0 \ud3ed\ubc1c\ub825\uc744 \ubcf4\ub2e4 \uac15\ud654\uc2dc\ud0ac \uac83\uc73c\ub85c \ubcf4\uc558\ub2e4.", "sentence_answer": "\uc21c\uc218\ud55c \uc6b0\ub77c\ub284-235\uc758 \ubb3c\ub9ac\uc801 \uc131\uc9c8\uc740 \uc544\uc9c1 \uc644\uc804\ud788 \ubc1d\ud600\uc9c0\uc9c0 \uc54a\uc558\uace0, \ud50c\ub8e8\ud1a0\ub284\uc758 \uc874\uc7ac\ub294 1941\ub144 \uc5d0 \uc640\uc11c\uc57c \uae00\ub80c \uc2dc\ubcf4\uadf8\uc758 \uc5f0\uad6c\ud300\uc774 \ubc1c\uacac\ud558\uc600\ub2e4.", "paragraph_id": "6502177-6-0", "paragraph_question": "question: \ud50c\ub8e8\ud1a0\ub284\uc758 \uc874\uc7ac\uac00 \ubc1c\uacac\ub41c \ud574\ub294 \uc5b8\uc81c\uc778\uac00?, context: \uadf8\ub7ec\ub098, \uc5ec\uc804\ud788 \uc54c\ub824\uc9c0\uc9c0 \uc54a\uc740 \uac83\ub4e4\uc774 \uc788\uc5c8\ub2e4. \uc21c\uc218\ud55c \uc6b0\ub77c\ub284-235\uc758 \ubb3c\ub9ac\uc801 \uc131\uc9c8\uc740 \uc544\uc9c1 \uc644\uc804\ud788 \ubc1d\ud600\uc9c0\uc9c0 \uc54a\uc558\uace0, \ud50c\ub8e8\ud1a0\ub284\uc758 \uc874\uc7ac\ub294 1941\ub144\uc5d0 \uc640\uc11c\uc57c \uae00\ub80c \uc2dc\ubcf4\uadf8\uc758 \uc5f0\uad6c\ud300\uc774 \ubc1c\uacac\ud558\uc600\ub2e4. \ubc84\ud074\ub9ac\uc758 \uacfc\ud559\uc790\ub4e4\uc740 \uc6b0\ub77c\ub284-235\uc758 \ud575\ubd84\uc5f4\uc5d0 \uc758\ud55c \ud3ed\ubc1c\uacfc \uc6b0\ub77c\ub284-238\uc744 \ud50c\ub8e8\ud1a0\ub284\uc73c\ub85c \ubcc0\ud658\uc2dc\ud0a8\ud6c4 \ud575\ubd84\uc5f4\uc744 \uc77c\uc73c\ud0a4\ub294 \ubc29\uc2dd\uc758 \ud3ed\ubc1c\uc744 \ub193\uace0 \ud68c\uc758\ub97c \uacc4\uc18d\ud558\uc600\ub2e4. \uac8c\ub2e4\uac00 \ub2f9\uc2dc\uc5d0 \ud50c\ub8e8\ud1a0\ub284\uc740 \uc0ac\uc774\ud074\ub85c\ud2b8\ub860\uc5d0\uc11c \uc18c\ub7c9\uc774 \uac80\ucd9c\ub41c \uc218\uc900\uc5d0 \ubd88\uacfc\ud558\uc600\ub2e4. \uc2ec\uc9c0\uc5b4 1943\ub144 12\uc6d4\uae4c\uc9c0\ub3c4 \ube44\ucd95\ub41c \ud50c\ub8e8\ud1a0\ub284\uc758 \uc591\uc740 2 \ubc00\ub9ac\uadf8\ub7a8\uc5d0 \ubd88\uacfc\ud558\uc600\ub2e4. \ub610\ud55c, \uc784\uacc4 \uc9c8\ub7c9\uc5d0 \uc774\ub974\uac8c \ud558\ub294 \ubc29\ubc95 \uc5ed\uc2dc \ub2e4\uc591\ud55c \uac83\ub4e4\uc774 \uc81c\uc2dc\ub418\uc5c8\ub2e4. \ud55c \ucabd\uc5d0 \"\ubc18\uc751\ubb3c\"\uc744 \ubaa8\uc544\ub193\uace0 \uc6d0\ud1b5 \ubaa8\uc591 \ud50c\ub7ec\uadf8\uc778 \"\uac04\uc12d\uc790\"\ub97c \uc3d8\uc544 \uc784\uacc4 \uc9c8\ub7c9\uc5d0 \ub3c4\ub2ec\ud558\ub294 \ubc29\ubc95\uc5d0\uc11c\ubd80\ud130 \ub458\ub85c \ub098\ub25c \ubc18\uc751\ubb3c\uc744 \uc11c\ub85c \ubbf8\ub04c\uc5b4\uc9c0\ub3c4\ub85d \uac15\ud558\uac8c \uacb0\ud569\uc2dc\ucf1c \uc784\uacc4 \uc9c8\ub7c9\uc5d0 \ub3c4\ub2ec\ud558\ub294 \ubc29\ubc95\uae4c\uc9c0 \ub2e4\uc591\ud55c \ubc29\ubc95\uc774 \uc81c\uc2dc\ub418\uc5c8\ub2e4. \ud3ed\ubc1c \uc591\uc0c1\uc5d0 \ub300\ud574\uc11c\ub294 \ub9ac\ucc98\ub4dc \ud1a8\ub9cc\uc740 \ud0c0\uc6d0\ud615 \ud3ed\ubc1c\uc744 \uc81c\uc2dc\ud558\uc600\ub294\ub370, \uadf8\ub294 \ud0c0\uc6d0\ud615 \ud3ed\ubc1c\uc774 \uc790\uccb4 \ucd09\ub9e4 \uc791\uc6a9\uc744 \ubd88\ub7ec\uc62c \uac00\ub2a5\uc131\uc774 \uc788\uae30 \ub54c\ubb38\uc5d0 \ud3ed\ubc1c\ub825\uc744 \ubcf4\ub2e4 \uac15\ud654\uc2dc\ud0ac \uac83\uc73c\ub85c \ubcf4\uc558\ub2e4."} {"question": "\uae40\ub300\uc911\uc758 \uc870\uae30\uc785\ub2f9\uc744 \ubc18\ub300\ud588\ub358 \ub300\ud45c\uc801\uc778 \ube44\uc11c\ub294?", "paragraph": "\ubc18\uba74 \ud55c\ud654\uac11 \ub4f1 \ube44\uc11c\uc9c4\uacfc \uae40\uc885\uc644 \ub4f1\uc744 \uc911\uc2ec\uc73c\ub85c \ud55c \ubbfc\ud5cc\uc5f0 \ub4f1 \uc6d0\uc678\uc778\uc0ac\ub4e4\uc740 \"\uc544\ubb34\ub7f0 \ubcf4\uc7a5 \uc5c6\uc774 \uc785\ub2f9\ud560 \uacbd\uc6b0 \ubbfc\uc8fc\ub2f9 \ud14c\ub450\ub9ac \uc548\uc5d0 \uac07\ud600 \ud589\ub3d9\ubc18\uacbd\uc774 \uc881\uc544\uc9c8 \ubfd0\ub9cc \uc544\ub2c8\ub77c \uc77c\ub2e8 \uce7c\uc790\ub8e8\ub97c \uc954 \uae40\uc601\uc0bc \ucd1d\uc7ac\uac00 \ub05d\uae4c\uc9c0 \ud6c4\ubcf4\ub97c \uace0\uc9d1\ud560 \uacbd\uc6b0 \ub2f9\ub0b4\uc5d0\uc11c\ub294 \ub2e4\ub978 \ubc29\ubc95\uc774 \uc5c6\uac8c \ub41c\ub2e4\"\uba70 \uae40\ub300\uc911\uc758 \uc870\uae30\uc785\ub2f9 \ubc18\ub300\ub97c \uc8fc\uc7a5\ud588\ub2e4. \uae40\ub300\uc911\uc740 \uacc4\ubcf4 \ub0b4\uc758 \ubbfc\uc8fc\ub2f9 \uc785\ub2f9 \ubc18\ub300 \uc8fc\uc7a5\uc744 \ubb3c\ub9ac\uce58\uace0 \uacb0\uad6d \uc785\ub2f9\uc744 \uacb0\uc2ec\ud558\uac8c \ub418\uba70, 8\uc6d4 6\uc77c\uc5d0 \uae40\uc601\uc0bc\uacfc \ub9cc\ub098 \uc785\ub2f9\ubb38\uc81c\ub97c \ud569\uc758\ud55c \ud6c4, 8\uc6d4 8\uc77c \ud1b5\uc77c\ubbfc\uc8fc\ub2f9\uc0ac\uc5d0\uc11c \uc785\ub2f9\uc2dd\uc744 \uac16\uace0 \uace0\ubb38\uc5d0 \ucde8\uc784\ud55c\ub2e4. 8\uc6d4 11\uc77c \uae40\uc601\uc0bc\uacfc \ud68c\ub3d9\uc744 \uac16\uace0 \ub300\ud1b5\ub839 \ud6c4\ubcf4 \ub2e8\uc77c\ud654 \ubb38\uc81c\ub97c \ud611\uc758\ud588\uc73c\ub098, \uae40\uc601\uc0bc\uc758 \uc870\uae30 \ub2e8\uc77c\ud654 \uc8fc\uc7a5\uacfc \uae40\ub300\uc911\uc758 \uac1c\ud5cc \ud611\uc0c1 \ud6c4 \uc870\uc815 \uc8fc\uc7a5\uc774 \ub9de\uc11c\uc11c \ubcc4\ub2e4\ub978 \uc131\uacfc\ub97c \uac70\ub450\uc9c0 \ubabb\ud588\ub2e4.", "answer": "\ud55c\ud654\uac11", "sentence": "\ubc18\uba74 \ud55c\ud654\uac11 \ub4f1 \ube44\uc11c\uc9c4\uacfc \uae40\uc885\uc644 \ub4f1\uc744 \uc911\uc2ec\uc73c\ub85c \ud55c \ubbfc\ud5cc\uc5f0 \ub4f1 \uc6d0\uc678\uc778\uc0ac\ub4e4\uc740 \"\uc544\ubb34\ub7f0 \ubcf4\uc7a5 \uc5c6\uc774 \uc785\ub2f9\ud560 \uacbd\uc6b0 \ubbfc\uc8fc\ub2f9 \ud14c\ub450\ub9ac \uc548\uc5d0 \uac07\ud600 \ud589\ub3d9\ubc18\uacbd\uc774 \uc881\uc544\uc9c8 \ubfd0\ub9cc \uc544\ub2c8\ub77c \uc77c\ub2e8 \uce7c\uc790\ub8e8\ub97c \uc954 \uae40\uc601\uc0bc \ucd1d\uc7ac\uac00 \ub05d\uae4c\uc9c0 \ud6c4\ubcf4\ub97c \uace0\uc9d1\ud560 \uacbd\uc6b0 \ub2f9\ub0b4\uc5d0\uc11c\ub294 \ub2e4\ub978 \ubc29\ubc95\uc774 \uc5c6\uac8c \ub41c\ub2e4\"\uba70 \uae40\ub300\uc911\uc758 \uc870\uae30\uc785\ub2f9 \ubc18\ub300\ub97c \uc8fc\uc7a5\ud588\ub2e4.", "paragraph_sentence": " \ubc18\uba74 \ud55c\ud654\uac11 \ub4f1 \ube44\uc11c\uc9c4\uacfc \uae40\uc885\uc644 \ub4f1\uc744 \uc911\uc2ec\uc73c\ub85c \ud55c \ubbfc\ud5cc\uc5f0 \ub4f1 \uc6d0\uc678\uc778\uc0ac\ub4e4\uc740 \"\uc544\ubb34\ub7f0 \ubcf4\uc7a5 \uc5c6\uc774 \uc785\ub2f9\ud560 \uacbd\uc6b0 \ubbfc\uc8fc\ub2f9 \ud14c\ub450\ub9ac \uc548\uc5d0 \uac07\ud600 \ud589\ub3d9\ubc18\uacbd\uc774 \uc881\uc544\uc9c8 \ubfd0\ub9cc \uc544\ub2c8\ub77c \uc77c\ub2e8 \uce7c\uc790\ub8e8\ub97c \uc954 \uae40\uc601\uc0bc \ucd1d\uc7ac\uac00 \ub05d\uae4c\uc9c0 \ud6c4\ubcf4\ub97c \uace0\uc9d1\ud560 \uacbd\uc6b0 \ub2f9\ub0b4\uc5d0\uc11c\ub294 \ub2e4\ub978 \ubc29\ubc95\uc774 \uc5c6\uac8c \ub41c\ub2e4\"\uba70 \uae40\ub300\uc911\uc758 \uc870\uae30\uc785\ub2f9 \ubc18\ub300\ub97c \uc8fc\uc7a5\ud588\ub2e4. \uae40\ub300\uc911\uc740 \uacc4\ubcf4 \ub0b4\uc758 \ubbfc\uc8fc\ub2f9 \uc785\ub2f9 \ubc18\ub300 \uc8fc\uc7a5\uc744 \ubb3c\ub9ac\uce58\uace0 \uacb0\uad6d \uc785\ub2f9\uc744 \uacb0\uc2ec\ud558\uac8c \ub418\uba70, 8\uc6d4 6\uc77c\uc5d0 \uae40\uc601\uc0bc\uacfc \ub9cc\ub098 \uc785\ub2f9\ubb38\uc81c\ub97c \ud569\uc758\ud55c \ud6c4, 8\uc6d4 8\uc77c \ud1b5\uc77c\ubbfc\uc8fc\ub2f9\uc0ac\uc5d0\uc11c \uc785\ub2f9\uc2dd\uc744 \uac16\uace0 \uace0\ubb38\uc5d0 \ucde8\uc784\ud55c\ub2e4. 8\uc6d4 11\uc77c \uae40\uc601\uc0bc\uacfc \ud68c\ub3d9\uc744 \uac16\uace0 \ub300\ud1b5\ub839 \ud6c4\ubcf4 \ub2e8\uc77c\ud654 \ubb38\uc81c\ub97c \ud611\uc758\ud588\uc73c\ub098, \uae40\uc601\uc0bc\uc758 \uc870\uae30 \ub2e8\uc77c\ud654 \uc8fc\uc7a5\uacfc \uae40\ub300\uc911\uc758 \uac1c\ud5cc \ud611\uc0c1 \ud6c4 \uc870\uc815 \uc8fc\uc7a5\uc774 \ub9de\uc11c\uc11c \ubcc4\ub2e4\ub978 \uc131\uacfc\ub97c \uac70\ub450\uc9c0 \ubabb\ud588\ub2e4.", "paragraph_answer": "\ubc18\uba74 \ud55c\ud654\uac11 \ub4f1 \ube44\uc11c\uc9c4\uacfc \uae40\uc885\uc644 \ub4f1\uc744 \uc911\uc2ec\uc73c\ub85c \ud55c \ubbfc\ud5cc\uc5f0 \ub4f1 \uc6d0\uc678\uc778\uc0ac\ub4e4\uc740 \"\uc544\ubb34\ub7f0 \ubcf4\uc7a5 \uc5c6\uc774 \uc785\ub2f9\ud560 \uacbd\uc6b0 \ubbfc\uc8fc\ub2f9 \ud14c\ub450\ub9ac \uc548\uc5d0 \uac07\ud600 \ud589\ub3d9\ubc18\uacbd\uc774 \uc881\uc544\uc9c8 \ubfd0\ub9cc \uc544\ub2c8\ub77c \uc77c\ub2e8 \uce7c\uc790\ub8e8\ub97c \uc954 \uae40\uc601\uc0bc \ucd1d\uc7ac\uac00 \ub05d\uae4c\uc9c0 \ud6c4\ubcf4\ub97c \uace0\uc9d1\ud560 \uacbd\uc6b0 \ub2f9\ub0b4\uc5d0\uc11c\ub294 \ub2e4\ub978 \ubc29\ubc95\uc774 \uc5c6\uac8c \ub41c\ub2e4\"\uba70 \uae40\ub300\uc911\uc758 \uc870\uae30\uc785\ub2f9 \ubc18\ub300\ub97c \uc8fc\uc7a5\ud588\ub2e4. \uae40\ub300\uc911\uc740 \uacc4\ubcf4 \ub0b4\uc758 \ubbfc\uc8fc\ub2f9 \uc785\ub2f9 \ubc18\ub300 \uc8fc\uc7a5\uc744 \ubb3c\ub9ac\uce58\uace0 \uacb0\uad6d \uc785\ub2f9\uc744 \uacb0\uc2ec\ud558\uac8c \ub418\uba70, 8\uc6d4 6\uc77c\uc5d0 \uae40\uc601\uc0bc\uacfc \ub9cc\ub098 \uc785\ub2f9\ubb38\uc81c\ub97c \ud569\uc758\ud55c \ud6c4, 8\uc6d4 8\uc77c \ud1b5\uc77c\ubbfc\uc8fc\ub2f9\uc0ac\uc5d0\uc11c \uc785\ub2f9\uc2dd\uc744 \uac16\uace0 \uace0\ubb38\uc5d0 \ucde8\uc784\ud55c\ub2e4. 8\uc6d4 11\uc77c \uae40\uc601\uc0bc\uacfc \ud68c\ub3d9\uc744 \uac16\uace0 \ub300\ud1b5\ub839 \ud6c4\ubcf4 \ub2e8\uc77c\ud654 \ubb38\uc81c\ub97c \ud611\uc758\ud588\uc73c\ub098, \uae40\uc601\uc0bc\uc758 \uc870\uae30 \ub2e8\uc77c\ud654 \uc8fc\uc7a5\uacfc \uae40\ub300\uc911\uc758 \uac1c\ud5cc \ud611\uc0c1 \ud6c4 \uc870\uc815 \uc8fc\uc7a5\uc774 \ub9de\uc11c\uc11c \ubcc4\ub2e4\ub978 \uc131\uacfc\ub97c \uac70\ub450\uc9c0 \ubabb\ud588\ub2e4.", "sentence_answer": "\ubc18\uba74 \ud55c\ud654\uac11 \ub4f1 \ube44\uc11c\uc9c4\uacfc \uae40\uc885\uc644 \ub4f1\uc744 \uc911\uc2ec\uc73c\ub85c \ud55c \ubbfc\ud5cc\uc5f0 \ub4f1 \uc6d0\uc678\uc778\uc0ac\ub4e4\uc740 \"\uc544\ubb34\ub7f0 \ubcf4\uc7a5 \uc5c6\uc774 \uc785\ub2f9\ud560 \uacbd\uc6b0 \ubbfc\uc8fc\ub2f9 \ud14c\ub450\ub9ac \uc548\uc5d0 \uac07\ud600 \ud589\ub3d9\ubc18\uacbd\uc774 \uc881\uc544\uc9c8 \ubfd0\ub9cc \uc544\ub2c8\ub77c \uc77c\ub2e8 \uce7c\uc790\ub8e8\ub97c \uc954 \uae40\uc601\uc0bc \ucd1d\uc7ac\uac00 \ub05d\uae4c\uc9c0 \ud6c4\ubcf4\ub97c \uace0\uc9d1\ud560 \uacbd\uc6b0 \ub2f9\ub0b4\uc5d0\uc11c\ub294 \ub2e4\ub978 \ubc29\ubc95\uc774 \uc5c6\uac8c \ub41c\ub2e4\"\uba70 \uae40\ub300\uc911\uc758 \uc870\uae30\uc785\ub2f9 \ubc18\ub300\ub97c \uc8fc\uc7a5\ud588\ub2e4.", "paragraph_id": "6488203-23-1", "paragraph_question": "question: \uae40\ub300\uc911\uc758 \uc870\uae30\uc785\ub2f9\uc744 \ubc18\ub300\ud588\ub358 \ub300\ud45c\uc801\uc778 \ube44\uc11c\ub294?, context: \ubc18\uba74 \ud55c\ud654\uac11 \ub4f1 \ube44\uc11c\uc9c4\uacfc \uae40\uc885\uc644 \ub4f1\uc744 \uc911\uc2ec\uc73c\ub85c \ud55c \ubbfc\ud5cc\uc5f0 \ub4f1 \uc6d0\uc678\uc778\uc0ac\ub4e4\uc740 \"\uc544\ubb34\ub7f0 \ubcf4\uc7a5 \uc5c6\uc774 \uc785\ub2f9\ud560 \uacbd\uc6b0 \ubbfc\uc8fc\ub2f9 \ud14c\ub450\ub9ac \uc548\uc5d0 \uac07\ud600 \ud589\ub3d9\ubc18\uacbd\uc774 \uc881\uc544\uc9c8 \ubfd0\ub9cc \uc544\ub2c8\ub77c \uc77c\ub2e8 \uce7c\uc790\ub8e8\ub97c \uc954 \uae40\uc601\uc0bc \ucd1d\uc7ac\uac00 \ub05d\uae4c\uc9c0 \ud6c4\ubcf4\ub97c \uace0\uc9d1\ud560 \uacbd\uc6b0 \ub2f9\ub0b4\uc5d0\uc11c\ub294 \ub2e4\ub978 \ubc29\ubc95\uc774 \uc5c6\uac8c \ub41c\ub2e4\"\uba70 \uae40\ub300\uc911\uc758 \uc870\uae30\uc785\ub2f9 \ubc18\ub300\ub97c \uc8fc\uc7a5\ud588\ub2e4. \uae40\ub300\uc911\uc740 \uacc4\ubcf4 \ub0b4\uc758 \ubbfc\uc8fc\ub2f9 \uc785\ub2f9 \ubc18\ub300 \uc8fc\uc7a5\uc744 \ubb3c\ub9ac\uce58\uace0 \uacb0\uad6d \uc785\ub2f9\uc744 \uacb0\uc2ec\ud558\uac8c \ub418\uba70, 8\uc6d4 6\uc77c\uc5d0 \uae40\uc601\uc0bc\uacfc \ub9cc\ub098 \uc785\ub2f9\ubb38\uc81c\ub97c \ud569\uc758\ud55c \ud6c4, 8\uc6d4 8\uc77c \ud1b5\uc77c\ubbfc\uc8fc\ub2f9\uc0ac\uc5d0\uc11c \uc785\ub2f9\uc2dd\uc744 \uac16\uace0 \uace0\ubb38\uc5d0 \ucde8\uc784\ud55c\ub2e4. 8\uc6d4 11\uc77c \uae40\uc601\uc0bc\uacfc \ud68c\ub3d9\uc744 \uac16\uace0 \ub300\ud1b5\ub839 \ud6c4\ubcf4 \ub2e8\uc77c\ud654 \ubb38\uc81c\ub97c \ud611\uc758\ud588\uc73c\ub098, \uae40\uc601\uc0bc\uc758 \uc870\uae30 \ub2e8\uc77c\ud654 \uc8fc\uc7a5\uacfc \uae40\ub300\uc911\uc758 \uac1c\ud5cc \ud611\uc0c1 \ud6c4 \uc870\uc815 \uc8fc\uc7a5\uc774 \ub9de\uc11c\uc11c \ubcc4\ub2e4\ub978 \uc131\uacfc\ub97c \uac70\ub450\uc9c0 \ubabb\ud588\ub2e4."} {"question": "\ud55c\uad6d\uc778 \ucd5c\ucd08\ub85c \ud3ec\uc2a4\ud2b8\uc2dc\uc98c \ud648\ub7f0\uc744 \uae30\ub85d\ud55c \ucd94\uc2e0\uc218\uc758 \ud648\ub7f0 \ub2f9\uc2dc \uc0c1\ub300 \ud22c\uc218\ub294?", "paragraph": "2013\ub144 \uc790\uc2e0\uc758 MLB \ucee4\ub9ac\uc5b4 \uc0ac\uc0c1 \uac00\uc7a5 \ub6f0\uc5b4\ub09c \ud55c \ud574\ub97c \ubcf4\ub0c8\ub2e4. \uc120\ubc1c\ucd9c\uc7a5\ud55c \ubaa8\ub4e0\uacbd\uae30\uc5d0\uc11c 1\ubc88\ud0c0\uc790\ub85c \uae30\uc6a9\ub418\uc5c8\ub2e4. \uc218\ube44\uc5d0\uc11c\ub294 \ubd88\uc548\ud55c \ubaa8\uc2b5\uc744 \ub178\ucd9c\ud588\uc9c0\ub9cc \ud0c0\uaca9\uc5d0\uc11c\ub294 \uac1c\uc778\ud1b5\uc0b0 3\ubc88\uc9f8 20-20\uc744 \ub2ec\uc131\ud558\uc600\uace0 \ucd9c\ub8e8\uc728.423, wRC+ 151, wOBA.323, Offensive WAR 6.4\ub97c \uae30\ub85d\ud558\uba70 \uad49\uc7a5\ud788 \uc88b\uc740 \ud65c\uc57d\uc744 \ud3bc\ucce4\ub2e4. \uc640\uc77c\ub4dc\uce74\ub4dc \uacb0\uc815\uc804\uc5d0\uc11c\ub294 \ud53c\uce20\ubc84\uadf8 \ud30c\uc774\ub9ac\uce20\uc758 \ud1a0\ub2c8 \uc653\uc2a8\uc744 \uc0c1\ub300\ub85c \ud648\ub7f0\uc744 \uccd0\ub0b4 \ud55c\uad6d\uc778 \ucd5c\ucd08\ub85c \ud3ec\uc2a4\ud2b8\uc2dc\uc98c \ud648\ub7f0\uc744 \uae30\ub85d\ud558\uae30\ub3c4 \ud588\ub2e4. \ud55c\ud3b8 \ucd94\uc2e0\uc218\uac00 \uc774\uc801\ud55c \ub0b4\uc154\ub110 \ub9ac\uadf8\uc5d0\ub294 \ub958\ud604\uc9c4\uc758 \uc18c\uc18d\ud300\uc778 \ub85c\uc2a4\uc564\uc824\ub808\uc2a4 \ub2e4\uc800\uc2a4\uac00 \uc788\uc5b4\uc11c \ud55c\uad6d\uc778 \ud22c\ud0c0 \ub300\uacb0\uc774 \uad00\uc2ec\uc744 \ubaa8\uc73c\uac8c \ub418\uc5c8\uace0 \ubc00\uc6cc\ud0a4 \ube0c\ub8e8\uc5b4\uc2a4\uc5d0\ub294 \uc77c\ubcf8\uc778 \uc544\uc624\ud0a4 \ub178\ub9ac\uce58\uce74\ub3c4 \uc788\uc5b4\uc11c \ud55c\u30fb\uc77c 1\ubc88 \ud0c0\uc790 \ub9de\ub300\uacb0\ub3c4 \uc8fc\ubaa9\ubc1b\uac8c \ub418\uc5c8\ub2e4. \uc2dc\uc98c\uc774 \ub05d\ub098\uace0 \ub098\uc11c\ub294 FA\uac00 \ub418\uc5c8\ub2e4.", "answer": "\ud1a0\ub2c8 \uc653\uc2a8", "sentence": "\uc640\uc77c\ub4dc\uce74\ub4dc \uacb0\uc815\uc804\uc5d0\uc11c\ub294 \ud53c\uce20\ubc84\uadf8 \ud30c\uc774\ub9ac\uce20\uc758 \ud1a0\ub2c8 \uc653\uc2a8 \uc744 \uc0c1\ub300\ub85c \ud648\ub7f0\uc744 \uccd0\ub0b4 \ud55c\uad6d\uc778 \ucd5c\ucd08\ub85c \ud3ec\uc2a4\ud2b8\uc2dc\uc98c \ud648\ub7f0\uc744 \uae30\ub85d\ud558\uae30\ub3c4 \ud588\ub2e4.", "paragraph_sentence": "2013\ub144 \uc790\uc2e0\uc758 MLB \ucee4\ub9ac\uc5b4 \uc0ac\uc0c1 \uac00\uc7a5 \ub6f0\uc5b4\ub09c \ud55c \ud574\ub97c \ubcf4\ub0c8\ub2e4. \uc120\ubc1c\ucd9c\uc7a5\ud55c \ubaa8\ub4e0\uacbd\uae30\uc5d0\uc11c 1\ubc88\ud0c0\uc790\ub85c \uae30\uc6a9\ub418\uc5c8\ub2e4. \uc218\ube44\uc5d0\uc11c\ub294 \ubd88\uc548\ud55c \ubaa8\uc2b5\uc744 \ub178\ucd9c\ud588\uc9c0\ub9cc \ud0c0\uaca9\uc5d0\uc11c\ub294 \uac1c\uc778\ud1b5\uc0b0 3\ubc88\uc9f8 20-20\uc744 \ub2ec\uc131\ud558\uc600\uace0 \ucd9c\ub8e8\uc728.423, wRC+ 151, wOBA.323, Offensive WAR 6.4\ub97c \uae30\ub85d\ud558\uba70 \uad49\uc7a5\ud788 \uc88b\uc740 \ud65c\uc57d\uc744 \ud3bc\ucce4\ub2e4. \uc640\uc77c\ub4dc\uce74\ub4dc \uacb0\uc815\uc804\uc5d0\uc11c\ub294 \ud53c\uce20\ubc84\uadf8 \ud30c\uc774\ub9ac\uce20\uc758 \ud1a0\ub2c8 \uc653\uc2a8 \uc744 \uc0c1\ub300\ub85c \ud648\ub7f0\uc744 \uccd0\ub0b4 \ud55c\uad6d\uc778 \ucd5c\ucd08\ub85c \ud3ec\uc2a4\ud2b8\uc2dc\uc98c \ud648\ub7f0\uc744 \uae30\ub85d\ud558\uae30\ub3c4 \ud588\ub2e4. \ud55c\ud3b8 \ucd94\uc2e0\uc218\uac00 \uc774\uc801\ud55c \ub0b4\uc154\ub110 \ub9ac\uadf8\uc5d0\ub294 \ub958\ud604\uc9c4\uc758 \uc18c\uc18d\ud300\uc778 \ub85c\uc2a4\uc564\uc824\ub808\uc2a4 \ub2e4\uc800\uc2a4\uac00 \uc788\uc5b4\uc11c \ud55c\uad6d\uc778 \ud22c\ud0c0 \ub300\uacb0\uc774 \uad00\uc2ec\uc744 \ubaa8\uc73c\uac8c \ub418\uc5c8\uace0 \ubc00\uc6cc\ud0a4 \ube0c\ub8e8\uc5b4\uc2a4\uc5d0\ub294 \uc77c\ubcf8\uc778 \uc544\uc624\ud0a4 \ub178\ub9ac\uce58\uce74\ub3c4 \uc788\uc5b4\uc11c \ud55c\u30fb\uc77c 1\ubc88 \ud0c0\uc790 \ub9de\ub300\uacb0\ub3c4 \uc8fc\ubaa9\ubc1b\uac8c \ub418\uc5c8\ub2e4. \uc2dc\uc98c\uc774 \ub05d\ub098\uace0 \ub098\uc11c\ub294 FA\uac00 \ub418\uc5c8\ub2e4.", "paragraph_answer": "2013\ub144 \uc790\uc2e0\uc758 MLB \ucee4\ub9ac\uc5b4 \uc0ac\uc0c1 \uac00\uc7a5 \ub6f0\uc5b4\ub09c \ud55c \ud574\ub97c \ubcf4\ub0c8\ub2e4. \uc120\ubc1c\ucd9c\uc7a5\ud55c \ubaa8\ub4e0\uacbd\uae30\uc5d0\uc11c 1\ubc88\ud0c0\uc790\ub85c \uae30\uc6a9\ub418\uc5c8\ub2e4. \uc218\ube44\uc5d0\uc11c\ub294 \ubd88\uc548\ud55c \ubaa8\uc2b5\uc744 \ub178\ucd9c\ud588\uc9c0\ub9cc \ud0c0\uaca9\uc5d0\uc11c\ub294 \uac1c\uc778\ud1b5\uc0b0 3\ubc88\uc9f8 20-20\uc744 \ub2ec\uc131\ud558\uc600\uace0 \ucd9c\ub8e8\uc728.423, wRC+ 151, wOBA.323, Offensive WAR 6.4\ub97c \uae30\ub85d\ud558\uba70 \uad49\uc7a5\ud788 \uc88b\uc740 \ud65c\uc57d\uc744 \ud3bc\ucce4\ub2e4. \uc640\uc77c\ub4dc\uce74\ub4dc \uacb0\uc815\uc804\uc5d0\uc11c\ub294 \ud53c\uce20\ubc84\uadf8 \ud30c\uc774\ub9ac\uce20\uc758 \ud1a0\ub2c8 \uc653\uc2a8 \uc744 \uc0c1\ub300\ub85c \ud648\ub7f0\uc744 \uccd0\ub0b4 \ud55c\uad6d\uc778 \ucd5c\ucd08\ub85c \ud3ec\uc2a4\ud2b8\uc2dc\uc98c \ud648\ub7f0\uc744 \uae30\ub85d\ud558\uae30\ub3c4 \ud588\ub2e4. \ud55c\ud3b8 \ucd94\uc2e0\uc218\uac00 \uc774\uc801\ud55c \ub0b4\uc154\ub110 \ub9ac\uadf8\uc5d0\ub294 \ub958\ud604\uc9c4\uc758 \uc18c\uc18d\ud300\uc778 \ub85c\uc2a4\uc564\uc824\ub808\uc2a4 \ub2e4\uc800\uc2a4\uac00 \uc788\uc5b4\uc11c \ud55c\uad6d\uc778 \ud22c\ud0c0 \ub300\uacb0\uc774 \uad00\uc2ec\uc744 \ubaa8\uc73c\uac8c \ub418\uc5c8\uace0 \ubc00\uc6cc\ud0a4 \ube0c\ub8e8\uc5b4\uc2a4\uc5d0\ub294 \uc77c\ubcf8\uc778 \uc544\uc624\ud0a4 \ub178\ub9ac\uce58\uce74\ub3c4 \uc788\uc5b4\uc11c \ud55c\u30fb\uc77c 1\ubc88 \ud0c0\uc790 \ub9de\ub300\uacb0\ub3c4 \uc8fc\ubaa9\ubc1b\uac8c \ub418\uc5c8\ub2e4. \uc2dc\uc98c\uc774 \ub05d\ub098\uace0 \ub098\uc11c\ub294 FA\uac00 \ub418\uc5c8\ub2e4.", "sentence_answer": "\uc640\uc77c\ub4dc\uce74\ub4dc \uacb0\uc815\uc804\uc5d0\uc11c\ub294 \ud53c\uce20\ubc84\uadf8 \ud30c\uc774\ub9ac\uce20\uc758 \ud1a0\ub2c8 \uc653\uc2a8 \uc744 \uc0c1\ub300\ub85c \ud648\ub7f0\uc744 \uccd0\ub0b4 \ud55c\uad6d\uc778 \ucd5c\ucd08\ub85c \ud3ec\uc2a4\ud2b8\uc2dc\uc98c \ud648\ub7f0\uc744 \uae30\ub85d\ud558\uae30\ub3c4 \ud588\ub2e4.", "paragraph_id": "6560658-3-1", "paragraph_question": "question: \ud55c\uad6d\uc778 \ucd5c\ucd08\ub85c \ud3ec\uc2a4\ud2b8\uc2dc\uc98c \ud648\ub7f0\uc744 \uae30\ub85d\ud55c \ucd94\uc2e0\uc218\uc758 \ud648\ub7f0 \ub2f9\uc2dc \uc0c1\ub300 \ud22c\uc218\ub294?, context: 2013\ub144 \uc790\uc2e0\uc758 MLB \ucee4\ub9ac\uc5b4 \uc0ac\uc0c1 \uac00\uc7a5 \ub6f0\uc5b4\ub09c \ud55c \ud574\ub97c \ubcf4\ub0c8\ub2e4. \uc120\ubc1c\ucd9c\uc7a5\ud55c \ubaa8\ub4e0\uacbd\uae30\uc5d0\uc11c 1\ubc88\ud0c0\uc790\ub85c \uae30\uc6a9\ub418\uc5c8\ub2e4. \uc218\ube44\uc5d0\uc11c\ub294 \ubd88\uc548\ud55c \ubaa8\uc2b5\uc744 \ub178\ucd9c\ud588\uc9c0\ub9cc \ud0c0\uaca9\uc5d0\uc11c\ub294 \uac1c\uc778\ud1b5\uc0b0 3\ubc88\uc9f8 20-20\uc744 \ub2ec\uc131\ud558\uc600\uace0 \ucd9c\ub8e8\uc728.423, wRC+ 151, wOBA.323, Offensive WAR 6.4\ub97c \uae30\ub85d\ud558\uba70 \uad49\uc7a5\ud788 \uc88b\uc740 \ud65c\uc57d\uc744 \ud3bc\ucce4\ub2e4. \uc640\uc77c\ub4dc\uce74\ub4dc \uacb0\uc815\uc804\uc5d0\uc11c\ub294 \ud53c\uce20\ubc84\uadf8 \ud30c\uc774\ub9ac\uce20\uc758 \ud1a0\ub2c8 \uc653\uc2a8\uc744 \uc0c1\ub300\ub85c \ud648\ub7f0\uc744 \uccd0\ub0b4 \ud55c\uad6d\uc778 \ucd5c\ucd08\ub85c \ud3ec\uc2a4\ud2b8\uc2dc\uc98c \ud648\ub7f0\uc744 \uae30\ub85d\ud558\uae30\ub3c4 \ud588\ub2e4. \ud55c\ud3b8 \ucd94\uc2e0\uc218\uac00 \uc774\uc801\ud55c \ub0b4\uc154\ub110 \ub9ac\uadf8\uc5d0\ub294 \ub958\ud604\uc9c4\uc758 \uc18c\uc18d\ud300\uc778 \ub85c\uc2a4\uc564\uc824\ub808\uc2a4 \ub2e4\uc800\uc2a4\uac00 \uc788\uc5b4\uc11c \ud55c\uad6d\uc778 \ud22c\ud0c0 \ub300\uacb0\uc774 \uad00\uc2ec\uc744 \ubaa8\uc73c\uac8c \ub418\uc5c8\uace0 \ubc00\uc6cc\ud0a4 \ube0c\ub8e8\uc5b4\uc2a4\uc5d0\ub294 \uc77c\ubcf8\uc778 \uc544\uc624\ud0a4 \ub178\ub9ac\uce58\uce74\ub3c4 \uc788\uc5b4\uc11c \ud55c\u30fb\uc77c 1\ubc88 \ud0c0\uc790 \ub9de\ub300\uacb0\ub3c4 \uc8fc\ubaa9\ubc1b\uac8c \ub418\uc5c8\ub2e4. \uc2dc\uc98c\uc774 \ub05d\ub098\uace0 \ub098\uc11c\ub294 FA\uac00 \ub418\uc5c8\ub2e4."} {"question": "\uad11\ud574\uad70\uc740 \uc790\uc2e0\uc758 \uc704\ub825\uc744 \ub354 \ub192\uc774\ub824 \ud558\ub2e4 \ub098\ub77c\uc758 \uacbd\uc81c\uc0c1\ud669\uc744 \uc545\ud654\uc2dc\ud0a4\ub294\ub370 \uc774\ub54c \uad11\ud574\uad70\uc774 \uc790\uc2e0\uc758 \uc704\ub825\uc744 \ub192\uc774\uae30 \uc704\ud574 \ud588\ub358 \uc77c\uc740?", "paragraph": "\uc784\uae08\uc774 \ub41c \uad11\ud574\uad70\uc740 \uc989\uc704 \ucd08\ubd80\ud130 \uc548\uc73c\ub85c\ub294 \uc655\uad8c\uc744 \uac15\ud654\ud558\uae30 \uc704\ud574 \uad81\uad90\uc744 \uc9c0\uc5b4 \uacbd\uc81c\uac00 \ud30c\ud0c4 \ub0ac\uace0 \uc774\uc810\uc740 \uad11\ud574\uad70\uc744 \uae0d\uc815\uc801\uc73c\ub85c \ud3c9\uac00\ud558\ub294 \ud55c\uba85\uae30 \uad50\uc218 \uc870\ucc28\ub3c4 \"\uad11\ud574\uad70\uc740 \ud0c1\uc6d4\ud55c \uc678\uad50 \uc815\ucc45\uc744 \uad81\uad90 \uacf5\uc0ac\ub85c \ubaa8\ub450 \ub9d0\uc544 \uba39\uc5c8\ub2e4\" \ub77c\uba70 \ube44\ud310\ud55c\ub2e4. 1608\ub144 \uc120\ud61c\uccad\uc744 \ub450\uc5b4 \uacbd\uae30\ub3c4\uc5d0\uc11c \uc300\ub85c \uc870\uc138\ub97c \ub0b4\ub3c4\ub85d \ud568\uc73c\ub85c\uc368 \uc18c\ub4dd\uc5d0 \ub530\ub77c \uc138\uae08\uc744 \ub0b4\ub294 \uc870\uc138\uac1c\ud601\uc778 \ub300\ub3d9\ubc95\uc744 \uc5b5\uc9c0\ub85c \uc2dc\ud589\ud588\uc73c\ub098,\ub450 \ubc88\uc774\ub098 \ud3d0\uc9c0\ud558\ub824 \ud588\ub2e4. 1611\ub144 \uc591\uc804 \uc0ac\uc5c5\uc744 \ubc8c\uc600\ub2e4\uace0 \ud558\ub098 \uc120\uc870 \ub300\uc758 \uacc4\ubb18\uc591\uc804, \uc778\uc870 \ub300\uc758 \uac11\uc220\uc591\uc804 \uc774\uc678\uc5d0\ub294 \uc591\uc804\uc744 \uc2dc\ud589\ud55c \uae30\ub85d\uc774 \uc5c6\ub2e4. \uc774\uc5b4 \uc784\uc9c4\uc65c\ub780 \ub54c \ud654\uc7ac\ub85c \uc18c\uc2e4\ub41c \ucc3d\ub355\uad81, \uacbd\ud76c\uad81, \ucc3d\uacbd\uad81\uc744 \uc7ac\uac74\ud558\uace0 \uc778\uacbd\uad81\uc744 \uac74\uc124\ud588\uc73c\uba70, \uc784\uc9c4\uc65c\ub780 \ub54c \uc18c\uc2e4\ub41c \uc11c\uc801 \uac04\ud589\uc5d0\ub3c4 \ud798\uc368 \u300a\uc2e0\uc99d\ub3d9\uad6d\uc5ec\uc9c0\uc2b9\ub78c\u300b, \u300a\uc6a9\ube44\uc5b4\ucc9c\uac00\u300b ,\u300a\ub3d9\uad6d\uc2e0\uc18d\uc0bc\uac15\ud589\uc2e4\u300b \ub4f1\uc744 \ub2e4\uc2dc \uac04\ud589\ud588\ub2e4. \ud5c8\uade0\uc758 \u300a\ud64d\uae38\ub3d9\uc804\u300b, \ud5c8\uc900\uc758 \ud55c\uc758\ud559\ucc45\uc778\u300a\ub3d9\uc758\ubcf4\uac10\u300b \ub4f1\ub3c4 \uc774 \uc2dc\uae30\uc5d0 \uc644\uc131\ub418\uc5c8\ub2e4.", "answer": "\uad81\uad90 \uacf5\uc0ac", "sentence": "\uc784\uae08\uc774 \ub41c \uad11\ud574\uad70\uc740 \uc989\uc704 \ucd08\ubd80\ud130 \uc548\uc73c\ub85c\ub294 \uc655\uad8c\uc744 \uac15\ud654\ud558\uae30 \uc704\ud574 \uad81\uad90\uc744 \uc9c0\uc5b4 \uacbd\uc81c\uac00 \ud30c\ud0c4 \ub0ac\uace0 \uc774\uc810\uc740 \uad11\ud574\uad70\uc744 \uae0d\uc815\uc801\uc73c\ub85c \ud3c9\uac00\ud558\ub294 \ud55c\uba85\uae30 \uad50\uc218 \uc870\ucc28\ub3c4 \"\uad11\ud574\uad70\uc740 \ud0c1\uc6d4\ud55c \uc678\uad50 \uc815\ucc45\uc744 \uad81\uad90 \uacf5\uc0ac \ub85c \ubaa8\ub450 \ub9d0\uc544 \uba39\uc5c8\ub2e4\" \ub77c\uba70 \ube44\ud310\ud55c\ub2e4.", "paragraph_sentence": " \uc784\uae08\uc774 \ub41c \uad11\ud574\uad70\uc740 \uc989\uc704 \ucd08\ubd80\ud130 \uc548\uc73c\ub85c\ub294 \uc655\uad8c\uc744 \uac15\ud654\ud558\uae30 \uc704\ud574 \uad81\uad90\uc744 \uc9c0\uc5b4 \uacbd\uc81c\uac00 \ud30c\ud0c4 \ub0ac\uace0 \uc774\uc810\uc740 \uad11\ud574\uad70\uc744 \uae0d\uc815\uc801\uc73c\ub85c \ud3c9\uac00\ud558\ub294 \ud55c\uba85\uae30 \uad50\uc218 \uc870\ucc28\ub3c4 \"\uad11\ud574\uad70\uc740 \ud0c1\uc6d4\ud55c \uc678\uad50 \uc815\ucc45\uc744 \uad81\uad90 \uacf5\uc0ac \ub85c \ubaa8\ub450 \ub9d0\uc544 \uba39\uc5c8\ub2e4\" \ub77c\uba70 \ube44\ud310\ud55c\ub2e4. 1608\ub144 \uc120\ud61c\uccad\uc744 \ub450\uc5b4 \uacbd\uae30\ub3c4\uc5d0\uc11c \uc300\ub85c \uc870\uc138\ub97c \ub0b4\ub3c4\ub85d \ud568\uc73c\ub85c\uc368 \uc18c\ub4dd\uc5d0 \ub530\ub77c \uc138\uae08\uc744 \ub0b4\ub294 \uc870\uc138\uac1c\ud601\uc778 \ub300\ub3d9\ubc95\uc744 \uc5b5\uc9c0\ub85c \uc2dc\ud589\ud588\uc73c\ub098,\ub450 \ubc88\uc774\ub098 \ud3d0\uc9c0\ud558\ub824 \ud588\ub2e4. 1611\ub144 \uc591\uc804 \uc0ac\uc5c5\uc744 \ubc8c\uc600\ub2e4\uace0 \ud558\ub098 \uc120\uc870 \ub300\uc758 \uacc4\ubb18\uc591\uc804, \uc778\uc870 \ub300\uc758 \uac11\uc220\uc591\uc804 \uc774\uc678\uc5d0\ub294 \uc591\uc804\uc744 \uc2dc\ud589\ud55c \uae30\ub85d\uc774 \uc5c6\ub2e4. \uc774\uc5b4 \uc784\uc9c4\uc65c\ub780 \ub54c \ud654\uc7ac\ub85c \uc18c\uc2e4\ub41c \ucc3d\ub355\uad81, \uacbd\ud76c\uad81, \ucc3d\uacbd\uad81\uc744 \uc7ac\uac74\ud558\uace0 \uc778\uacbd\uad81\uc744 \uac74\uc124\ud588\uc73c\uba70, \uc784\uc9c4\uc65c\ub780 \ub54c \uc18c\uc2e4\ub41c \uc11c\uc801 \uac04\ud589\uc5d0\ub3c4 \ud798\uc368 \u300a\uc2e0\uc99d\ub3d9\uad6d\uc5ec\uc9c0\uc2b9\ub78c\u300b, \u300a\uc6a9\ube44\uc5b4\ucc9c\uac00\u300b ,\u300a\ub3d9\uad6d\uc2e0\uc18d\uc0bc\uac15\ud589\uc2e4\u300b \ub4f1\uc744 \ub2e4\uc2dc \uac04\ud589\ud588\ub2e4. \ud5c8\uade0\uc758 \u300a\ud64d\uae38\ub3d9\uc804\u300b, \ud5c8\uc900\uc758 \ud55c\uc758\ud559\ucc45\uc778\u300a\ub3d9\uc758\ubcf4\uac10\u300b \ub4f1\ub3c4 \uc774 \uc2dc\uae30\uc5d0 \uc644\uc131\ub418\uc5c8\ub2e4.", "paragraph_answer": "\uc784\uae08\uc774 \ub41c \uad11\ud574\uad70\uc740 \uc989\uc704 \ucd08\ubd80\ud130 \uc548\uc73c\ub85c\ub294 \uc655\uad8c\uc744 \uac15\ud654\ud558\uae30 \uc704\ud574 \uad81\uad90\uc744 \uc9c0\uc5b4 \uacbd\uc81c\uac00 \ud30c\ud0c4 \ub0ac\uace0 \uc774\uc810\uc740 \uad11\ud574\uad70\uc744 \uae0d\uc815\uc801\uc73c\ub85c \ud3c9\uac00\ud558\ub294 \ud55c\uba85\uae30 \uad50\uc218 \uc870\ucc28\ub3c4 \"\uad11\ud574\uad70\uc740 \ud0c1\uc6d4\ud55c \uc678\uad50 \uc815\ucc45\uc744 \uad81\uad90 \uacf5\uc0ac \ub85c \ubaa8\ub450 \ub9d0\uc544 \uba39\uc5c8\ub2e4\" \ub77c\uba70 \ube44\ud310\ud55c\ub2e4. 1608\ub144 \uc120\ud61c\uccad\uc744 \ub450\uc5b4 \uacbd\uae30\ub3c4\uc5d0\uc11c \uc300\ub85c \uc870\uc138\ub97c \ub0b4\ub3c4\ub85d \ud568\uc73c\ub85c\uc368 \uc18c\ub4dd\uc5d0 \ub530\ub77c \uc138\uae08\uc744 \ub0b4\ub294 \uc870\uc138\uac1c\ud601\uc778 \ub300\ub3d9\ubc95\uc744 \uc5b5\uc9c0\ub85c \uc2dc\ud589\ud588\uc73c\ub098,\ub450 \ubc88\uc774\ub098 \ud3d0\uc9c0\ud558\ub824 \ud588\ub2e4. 1611\ub144 \uc591\uc804 \uc0ac\uc5c5\uc744 \ubc8c\uc600\ub2e4\uace0 \ud558\ub098 \uc120\uc870 \ub300\uc758 \uacc4\ubb18\uc591\uc804, \uc778\uc870 \ub300\uc758 \uac11\uc220\uc591\uc804 \uc774\uc678\uc5d0\ub294 \uc591\uc804\uc744 \uc2dc\ud589\ud55c \uae30\ub85d\uc774 \uc5c6\ub2e4. \uc774\uc5b4 \uc784\uc9c4\uc65c\ub780 \ub54c \ud654\uc7ac\ub85c \uc18c\uc2e4\ub41c \ucc3d\ub355\uad81, \uacbd\ud76c\uad81, \ucc3d\uacbd\uad81\uc744 \uc7ac\uac74\ud558\uace0 \uc778\uacbd\uad81\uc744 \uac74\uc124\ud588\uc73c\uba70, \uc784\uc9c4\uc65c\ub780 \ub54c \uc18c\uc2e4\ub41c \uc11c\uc801 \uac04\ud589\uc5d0\ub3c4 \ud798\uc368 \u300a\uc2e0\uc99d\ub3d9\uad6d\uc5ec\uc9c0\uc2b9\ub78c\u300b, \u300a\uc6a9\ube44\uc5b4\ucc9c\uac00\u300b ,\u300a\ub3d9\uad6d\uc2e0\uc18d\uc0bc\uac15\ud589\uc2e4\u300b \ub4f1\uc744 \ub2e4\uc2dc \uac04\ud589\ud588\ub2e4. \ud5c8\uade0\uc758 \u300a\ud64d\uae38\ub3d9\uc804\u300b, \ud5c8\uc900\uc758 \ud55c\uc758\ud559\ucc45\uc778\u300a\ub3d9\uc758\ubcf4\uac10\u300b \ub4f1\ub3c4 \uc774 \uc2dc\uae30\uc5d0 \uc644\uc131\ub418\uc5c8\ub2e4.", "sentence_answer": "\uc784\uae08\uc774 \ub41c \uad11\ud574\uad70\uc740 \uc989\uc704 \ucd08\ubd80\ud130 \uc548\uc73c\ub85c\ub294 \uc655\uad8c\uc744 \uac15\ud654\ud558\uae30 \uc704\ud574 \uad81\uad90\uc744 \uc9c0\uc5b4 \uacbd\uc81c\uac00 \ud30c\ud0c4 \ub0ac\uace0 \uc774\uc810\uc740 \uad11\ud574\uad70\uc744 \uae0d\uc815\uc801\uc73c\ub85c \ud3c9\uac00\ud558\ub294 \ud55c\uba85\uae30 \uad50\uc218 \uc870\ucc28\ub3c4 \"\uad11\ud574\uad70\uc740 \ud0c1\uc6d4\ud55c \uc678\uad50 \uc815\ucc45\uc744 \uad81\uad90 \uacf5\uc0ac \ub85c \ubaa8\ub450 \ub9d0\uc544 \uba39\uc5c8\ub2e4\" \ub77c\uba70 \ube44\ud310\ud55c\ub2e4.", "paragraph_id": "6339795-2-1", "paragraph_question": "question: \uad11\ud574\uad70\uc740 \uc790\uc2e0\uc758 \uc704\ub825\uc744 \ub354 \ub192\uc774\ub824 \ud558\ub2e4 \ub098\ub77c\uc758 \uacbd\uc81c\uc0c1\ud669\uc744 \uc545\ud654\uc2dc\ud0a4\ub294\ub370 \uc774\ub54c \uad11\ud574\uad70\uc774 \uc790\uc2e0\uc758 \uc704\ub825\uc744 \ub192\uc774\uae30 \uc704\ud574 \ud588\ub358 \uc77c\uc740?, context: \uc784\uae08\uc774 \ub41c \uad11\ud574\uad70\uc740 \uc989\uc704 \ucd08\ubd80\ud130 \uc548\uc73c\ub85c\ub294 \uc655\uad8c\uc744 \uac15\ud654\ud558\uae30 \uc704\ud574 \uad81\uad90\uc744 \uc9c0\uc5b4 \uacbd\uc81c\uac00 \ud30c\ud0c4 \ub0ac\uace0 \uc774\uc810\uc740 \uad11\ud574\uad70\uc744 \uae0d\uc815\uc801\uc73c\ub85c \ud3c9\uac00\ud558\ub294 \ud55c\uba85\uae30 \uad50\uc218 \uc870\ucc28\ub3c4 \"\uad11\ud574\uad70\uc740 \ud0c1\uc6d4\ud55c \uc678\uad50 \uc815\ucc45\uc744 \uad81\uad90 \uacf5\uc0ac\ub85c \ubaa8\ub450 \ub9d0\uc544 \uba39\uc5c8\ub2e4\" \ub77c\uba70 \ube44\ud310\ud55c\ub2e4. 1608\ub144 \uc120\ud61c\uccad\uc744 \ub450\uc5b4 \uacbd\uae30\ub3c4\uc5d0\uc11c \uc300\ub85c \uc870\uc138\ub97c \ub0b4\ub3c4\ub85d \ud568\uc73c\ub85c\uc368 \uc18c\ub4dd\uc5d0 \ub530\ub77c \uc138\uae08\uc744 \ub0b4\ub294 \uc870\uc138\uac1c\ud601\uc778 \ub300\ub3d9\ubc95\uc744 \uc5b5\uc9c0\ub85c \uc2dc\ud589\ud588\uc73c\ub098,\ub450 \ubc88\uc774\ub098 \ud3d0\uc9c0\ud558\ub824 \ud588\ub2e4. 1611\ub144 \uc591\uc804 \uc0ac\uc5c5\uc744 \ubc8c\uc600\ub2e4\uace0 \ud558\ub098 \uc120\uc870 \ub300\uc758 \uacc4\ubb18\uc591\uc804, \uc778\uc870 \ub300\uc758 \uac11\uc220\uc591\uc804 \uc774\uc678\uc5d0\ub294 \uc591\uc804\uc744 \uc2dc\ud589\ud55c \uae30\ub85d\uc774 \uc5c6\ub2e4. \uc774\uc5b4 \uc784\uc9c4\uc65c\ub780 \ub54c \ud654\uc7ac\ub85c \uc18c\uc2e4\ub41c \ucc3d\ub355\uad81, \uacbd\ud76c\uad81, \ucc3d\uacbd\uad81\uc744 \uc7ac\uac74\ud558\uace0 \uc778\uacbd\uad81\uc744 \uac74\uc124\ud588\uc73c\uba70, \uc784\uc9c4\uc65c\ub780 \ub54c \uc18c\uc2e4\ub41c \uc11c\uc801 \uac04\ud589\uc5d0\ub3c4 \ud798\uc368 \u300a\uc2e0\uc99d\ub3d9\uad6d\uc5ec\uc9c0\uc2b9\ub78c\u300b, \u300a\uc6a9\ube44\uc5b4\ucc9c\uac00\u300b ,\u300a\ub3d9\uad6d\uc2e0\uc18d\uc0bc\uac15\ud589\uc2e4\u300b \ub4f1\uc744 \ub2e4\uc2dc \uac04\ud589\ud588\ub2e4. \ud5c8\uade0\uc758 \u300a\ud64d\uae38\ub3d9\uc804\u300b, \ud5c8\uc900\uc758 \ud55c\uc758\ud559\ucc45\uc778\u300a\ub3d9\uc758\ubcf4\uac10\u300b \ub4f1\ub3c4 \uc774 \uc2dc\uae30\uc5d0 \uc644\uc131\ub418\uc5c8\ub2e4."} {"question": "\uac15\ubaa9\uc744 \uc5f0\uad6c\ud55c \uc9c4\uc81c\uac00 \uc9c0\uc740 \uc5ed\uc0ac\uc11c\ub294?", "paragraph": "\u300a\uc790\uce58\ud1b5\uac10\uac15\ubaa9\u300b\uc758 \uac00\uce58\ub97c \ub192\uac8c \ub9e4\uae30\uac8c \ub41c \uac83\uc740 \ub0a8\uc1a1 \ud6c4\uae30, \uadf8\ub9ac\uace0 \uba85 \uc655\uc870\uc758 \uc2dc\ub300\uc5d0 \ub4e4\uc5b4\uc11c\uc600\ub2e4. \uac15\ubaa9\uc744 \uc5f0\uad6c\ud55c \uc724\uae30\uc2e0(\u5c39\u8d77\u8398)\uc774 \uc4f4 \u300a\ubc1c\uba85(\u767c\u660e)\u300b\uff0c\uc720\uc6b0\uc775(\u5289\u53cb\u76ca)\uc758 \u300a\uac15\ubaa9\uc11c\ubc95(\u7db1\u76ee\u66f8\u6cd5)\u300b\uff0c\uc655\uadf9\uad00(\u6c6a\u514b\u5bec)\uc758 \u300a\uac15\ubaa9\ubc94\ub840\uace0\uc774(\u7db1\u76ee\u51e1\u4f8b\u8003\u7570)\u300b\ub97c \uad6c\uc6b0(\u77bf\u4f51)\uac00 \uad50\uac10\ud574 \uc601\ub77d(\u6c38\u6a02) 8\ub144\uc5d0 \u300a\uc790\uce58\ud1b5\uac10\uac15\ubaa9\uc9d1\ub78c\uc804\uc624\u300b(\u8d44\u6cbb\u901a\u9274\u7eb2\u76ee\u96c6\u89c8\u954c\u8bef)\ub77c\ub294 \uc81c\ubaa9\uc73c\ub85c \uac04\ud589\ud558\uc600\ub2e4. \uc22d\uc815(\u5d07\u798e) 3\ub144\uc5d0 \uc9c4\uc778\uc11d(\u9673\u4ec1\u932b)\uc774 \uc774 \uc138 \uad8c\uc758 \ucc45\uc744 \uac16\uace0 \ub2e4\uc2dc \ud569\uccd0 \uc778\uc1c4\ud560 \ub54c \uc8fc\ud76c\uc640 \uc655\ubc31(\u738b\u67cf), \ubb38\ucc9c\uc6b0(\u6587\u5929\u7950), \uc774\ubc29\uc790(\u674e\u65b9\u5b50), \uc724\uae30\uc2e0, \uc720\uc720\uc775, \uac08\uacc4\uc2e0(\u63ed\u5092\u65af), \uac00\uc120\uac70(\u8cc0\u5584\u6998), \uc608\uc0ac\uc758(\u502a\u58eb\u6bc5), \uc655\uadf9\uad00, \uc655\uc720\ud559, \uc11c\uc18c\ubb38(\u5f90\u662d\u6587), \uc9c4\uc81c(\u9673\u6fdf), \uc591\ubb38\uc815(\u694a\u6587\u8c9e), \ud48d\uc9c0\uc11c(\u99ae\u667a\u8212), \ud669\uc911\ucd08(\u9ec4\u4ef2\u662d), \uc5ec\uc774\ub2a5(\u4f59\u4ee5\u80fd) \ub4f1 \ub9ce\uc740 \uc0ac\ub78c\ub4e4\uc758 \uc11c\ubb38\uc744 \uc2e4\uc5c8\ub2e4. \uadf8 \ubc16\uc5d0 \uc9c4\uc81c(\u9648\u6d4e)\uac00 \uc9c0\uc740 \u300a\uac15\ubaa9\uc9d1\ub78c\uc815\uc624(\u7eb2\u76ee\u96c6\u89c8\u6b63\u8bef)\u300b, \ud48d\uc9c0\ub824(\u51af\u667a\u8212)\uac00 \uc9c0\uc740 \u300a\uac15\ubaa9\uc9c8\uc2e4\u300b(\u7db1\u76ee\u8cea\u5be6), \uc6d0 \uc655\uc870 \uc2dc\ub300 \uc0ac\ub78c\uc778 \uc655\uc720\ud559(\u738b\u5e7c\u5b66)\uc774 \ud3b8\ucc2c\ud55c \u300a\uc790\uce58\ud1b5\uac10\uac15\ubaa9\uc9d1\ub78c(\u8d44\u6cbb\u901a\u9274\u7eb2\u76ee\u96c6\u89c8)\u300b 59\uad8c\uc774 \uc788\uc5c8\ub2e4. \ub610\ud55c \uc9c4\uacbd\uc218(\u9648\u6871\u4fee)\uac00 \uc9c0\uc740 \u300a\ud1b5\uac10\uc18d\ud3b8(\u901a\u9274\u7eed\u7f16)\u300b 24\uad8c, \uc9c4\uade0(\u9648\u5747)\uc774 \uc9c0\uc740 \u300a\ud669\uc870\ud3b8\ub144\uac15\ubaa9\ud53c\uc694(\u7687\u671d\u7f16\u5e74\u7eb2\u76ee\u88ab\u8981)\u300b 30\uad8c\uacfc \u300a\uc911\ud765\uc591\uc870\uac15\ubaa9\ube44\uc694(\u4e2d\u8208\u4e24\u671d\u7eb2\u76ee\u5907\u8981)\u300b18\uad8c\uc774 \uc788\uc5c8\uace0, \uc9c0\uc740\uc774\uac00 \ud655\uc2e4\ud558\uc9c0 \uc54a\uc740 \u300a\uc591\uc870\uac15\ubaa9\ube44\uc694(\u4e24\u671d\u7eb2\u76ee\u5907\u8981)\u300b 16\uad8c\uc774 \uc788\ub2e4.", "answer": "\uac15\ubaa9\uc9d1\ub78c\uc815\uc624", "sentence": "\uadf8 \ubc16\uc5d0 \uc9c4\uc81c(\u9648\u6d4e)\uac00 \uc9c0\uc740 \u300a \uac15\ubaa9\uc9d1\ub78c\uc815\uc624 (\u7eb2\u76ee\u96c6\u89c8\u6b63\u8bef)\u300b, \ud48d\uc9c0\ub824(\u51af\u667a\u8212)\uac00 \uc9c0\uc740 \u300a\uac15\ubaa9\uc9c8\uc2e4\u300b(\u7db1\u76ee\u8cea\u5be6), \uc6d0 \uc655\uc870 \uc2dc\ub300 \uc0ac\ub78c\uc778 \uc655\uc720\ud559(\u738b\u5e7c\u5b66)\uc774 \ud3b8\ucc2c\ud55c \u300a\uc790\uce58\ud1b5\uac10\uac15\ubaa9\uc9d1\ub78c(\u8d44\u6cbb\u901a\u9274\u7eb2\u76ee\u96c6\u89c8)\u300b 59\uad8c\uc774 \uc788\uc5c8\ub2e4.", "paragraph_sentence": "\u300a\uc790\uce58\ud1b5\uac10\uac15\ubaa9\u300b\uc758 \uac00\uce58\ub97c \ub192\uac8c \ub9e4\uae30\uac8c \ub41c \uac83\uc740 \ub0a8\uc1a1 \ud6c4\uae30, \uadf8\ub9ac\uace0 \uba85 \uc655\uc870\uc758 \uc2dc\ub300\uc5d0 \ub4e4\uc5b4\uc11c\uc600\ub2e4. \uac15\ubaa9\uc744 \uc5f0\uad6c\ud55c \uc724\uae30\uc2e0(\u5c39\u8d77\u8398)\uc774 \uc4f4 \u300a\ubc1c\uba85(\u767c\u660e)\u300b\uff0c\uc720\uc6b0\uc775(\u5289\u53cb\u76ca)\uc758 \u300a\uac15\ubaa9\uc11c\ubc95(\u7db1\u76ee\u66f8\u6cd5)\u300b\uff0c\uc655\uadf9\uad00(\u6c6a\u514b\u5bec)\uc758 \u300a\uac15\ubaa9\ubc94\ub840\uace0\uc774(\u7db1\u76ee\u51e1\u4f8b\u8003\u7570)\u300b\ub97c \uad6c\uc6b0(\u77bf\u4f51)\uac00 \uad50\uac10\ud574 \uc601\ub77d(\u6c38\u6a02) 8\ub144\uc5d0 \u300a\uc790\uce58\ud1b5\uac10\uac15\ubaa9\uc9d1\ub78c\uc804\uc624\u300b(\u8d44\u6cbb\u901a\u9274\u7eb2\u76ee\u96c6\u89c8\u954c\u8bef)\ub77c\ub294 \uc81c\ubaa9\uc73c\ub85c \uac04\ud589\ud558\uc600\ub2e4. \uc22d\uc815(\u5d07\u798e) 3\ub144\uc5d0 \uc9c4\uc778\uc11d(\u9673\u4ec1\u932b)\uc774 \uc774 \uc138 \uad8c\uc758 \ucc45\uc744 \uac16\uace0 \ub2e4\uc2dc \ud569\uccd0 \uc778\uc1c4\ud560 \ub54c \uc8fc\ud76c\uc640 \uc655\ubc31(\u738b\u67cf), \ubb38\ucc9c\uc6b0(\u6587\u5929\u7950), \uc774\ubc29\uc790(\u674e\u65b9\u5b50), \uc724\uae30\uc2e0, \uc720\uc720\uc775, \uac08\uacc4\uc2e0(\u63ed\u5092\u65af), \uac00\uc120\uac70(\u8cc0\u5584\u6998), \uc608\uc0ac\uc758(\u502a\u58eb\u6bc5), \uc655\uadf9\uad00, \uc655\uc720\ud559, \uc11c\uc18c\ubb38(\u5f90\u662d\u6587), \uc9c4\uc81c(\u9673\u6fdf), \uc591\ubb38\uc815(\u694a\u6587\u8c9e), \ud48d\uc9c0\uc11c(\u99ae\u667a\u8212), \ud669\uc911\ucd08(\u9ec4\u4ef2\u662d), \uc5ec\uc774\ub2a5(\u4f59\u4ee5\u80fd) \ub4f1 \ub9ce\uc740 \uc0ac\ub78c\ub4e4\uc758 \uc11c\ubb38\uc744 \uc2e4\uc5c8\ub2e4. \uadf8 \ubc16\uc5d0 \uc9c4\uc81c(\u9648\u6d4e)\uac00 \uc9c0\uc740 \u300a \uac15\ubaa9\uc9d1\ub78c\uc815\uc624 (\u7eb2\u76ee\u96c6\u89c8\u6b63\u8bef)\u300b, \ud48d\uc9c0\ub824(\u51af\u667a\u8212)\uac00 \uc9c0\uc740 \u300a\uac15\ubaa9\uc9c8\uc2e4\u300b(\u7db1\u76ee\u8cea\u5be6), \uc6d0 \uc655\uc870 \uc2dc\ub300 \uc0ac\ub78c\uc778 \uc655\uc720\ud559(\u738b\u5e7c\u5b66)\uc774 \ud3b8\ucc2c\ud55c \u300a\uc790\uce58\ud1b5\uac10\uac15\ubaa9\uc9d1\ub78c(\u8d44\u6cbb\u901a\u9274\u7eb2\u76ee\u96c6\u89c8)\u300b 59\uad8c\uc774 \uc788\uc5c8\ub2e4. \ub610\ud55c \uc9c4\uacbd\uc218(\u9648\u6871\u4fee)\uac00 \uc9c0\uc740 \u300a\ud1b5\uac10\uc18d\ud3b8(\u901a\u9274\u7eed\u7f16)\u300b 24\uad8c, \uc9c4\uade0(\u9648\u5747)\uc774 \uc9c0\uc740 \u300a\ud669\uc870\ud3b8\ub144\uac15\ubaa9\ud53c\uc694(\u7687\u671d\u7f16\u5e74\u7eb2\u76ee\u88ab\u8981)\u300b 30\uad8c\uacfc \u300a\uc911\ud765\uc591\uc870\uac15\ubaa9\ube44\uc694(\u4e2d\u8208\u4e24\u671d\u7eb2\u76ee\u5907\u8981)\u300b18\uad8c\uc774 \uc788\uc5c8\uace0, \uc9c0\uc740\uc774\uac00 \ud655\uc2e4\ud558\uc9c0 \uc54a\uc740 \u300a\uc591\uc870\uac15\ubaa9\ube44\uc694(\u4e24\u671d\u7eb2\u76ee\u5907\u8981)\u300b 16\uad8c\uc774 \uc788\ub2e4.", "paragraph_answer": "\u300a\uc790\uce58\ud1b5\uac10\uac15\ubaa9\u300b\uc758 \uac00\uce58\ub97c \ub192\uac8c \ub9e4\uae30\uac8c \ub41c \uac83\uc740 \ub0a8\uc1a1 \ud6c4\uae30, \uadf8\ub9ac\uace0 \uba85 \uc655\uc870\uc758 \uc2dc\ub300\uc5d0 \ub4e4\uc5b4\uc11c\uc600\ub2e4. \uac15\ubaa9\uc744 \uc5f0\uad6c\ud55c \uc724\uae30\uc2e0(\u5c39\u8d77\u8398)\uc774 \uc4f4 \u300a\ubc1c\uba85(\u767c\u660e)\u300b\uff0c\uc720\uc6b0\uc775(\u5289\u53cb\u76ca)\uc758 \u300a\uac15\ubaa9\uc11c\ubc95(\u7db1\u76ee\u66f8\u6cd5)\u300b\uff0c\uc655\uadf9\uad00(\u6c6a\u514b\u5bec)\uc758 \u300a\uac15\ubaa9\ubc94\ub840\uace0\uc774(\u7db1\u76ee\u51e1\u4f8b\u8003\u7570)\u300b\ub97c \uad6c\uc6b0(\u77bf\u4f51)\uac00 \uad50\uac10\ud574 \uc601\ub77d(\u6c38\u6a02) 8\ub144\uc5d0 \u300a\uc790\uce58\ud1b5\uac10\uac15\ubaa9\uc9d1\ub78c\uc804\uc624\u300b(\u8d44\u6cbb\u901a\u9274\u7eb2\u76ee\u96c6\u89c8\u954c\u8bef)\ub77c\ub294 \uc81c\ubaa9\uc73c\ub85c \uac04\ud589\ud558\uc600\ub2e4. \uc22d\uc815(\u5d07\u798e) 3\ub144\uc5d0 \uc9c4\uc778\uc11d(\u9673\u4ec1\u932b)\uc774 \uc774 \uc138 \uad8c\uc758 \ucc45\uc744 \uac16\uace0 \ub2e4\uc2dc \ud569\uccd0 \uc778\uc1c4\ud560 \ub54c \uc8fc\ud76c\uc640 \uc655\ubc31(\u738b\u67cf), \ubb38\ucc9c\uc6b0(\u6587\u5929\u7950), \uc774\ubc29\uc790(\u674e\u65b9\u5b50), \uc724\uae30\uc2e0, \uc720\uc720\uc775, \uac08\uacc4\uc2e0(\u63ed\u5092\u65af), \uac00\uc120\uac70(\u8cc0\u5584\u6998), \uc608\uc0ac\uc758(\u502a\u58eb\u6bc5), \uc655\uadf9\uad00, \uc655\uc720\ud559, \uc11c\uc18c\ubb38(\u5f90\u662d\u6587), \uc9c4\uc81c(\u9673\u6fdf), \uc591\ubb38\uc815(\u694a\u6587\u8c9e), \ud48d\uc9c0\uc11c(\u99ae\u667a\u8212), \ud669\uc911\ucd08(\u9ec4\u4ef2\u662d), \uc5ec\uc774\ub2a5(\u4f59\u4ee5\u80fd) \ub4f1 \ub9ce\uc740 \uc0ac\ub78c\ub4e4\uc758 \uc11c\ubb38\uc744 \uc2e4\uc5c8\ub2e4. \uadf8 \ubc16\uc5d0 \uc9c4\uc81c(\u9648\u6d4e)\uac00 \uc9c0\uc740 \u300a \uac15\ubaa9\uc9d1\ub78c\uc815\uc624 (\u7eb2\u76ee\u96c6\u89c8\u6b63\u8bef)\u300b, \ud48d\uc9c0\ub824(\u51af\u667a\u8212)\uac00 \uc9c0\uc740 \u300a\uac15\ubaa9\uc9c8\uc2e4\u300b(\u7db1\u76ee\u8cea\u5be6), \uc6d0 \uc655\uc870 \uc2dc\ub300 \uc0ac\ub78c\uc778 \uc655\uc720\ud559(\u738b\u5e7c\u5b66)\uc774 \ud3b8\ucc2c\ud55c \u300a\uc790\uce58\ud1b5\uac10\uac15\ubaa9\uc9d1\ub78c(\u8d44\u6cbb\u901a\u9274\u7eb2\u76ee\u96c6\u89c8)\u300b 59\uad8c\uc774 \uc788\uc5c8\ub2e4. \ub610\ud55c \uc9c4\uacbd\uc218(\u9648\u6871\u4fee)\uac00 \uc9c0\uc740 \u300a\ud1b5\uac10\uc18d\ud3b8(\u901a\u9274\u7eed\u7f16)\u300b 24\uad8c, \uc9c4\uade0(\u9648\u5747)\uc774 \uc9c0\uc740 \u300a\ud669\uc870\ud3b8\ub144\uac15\ubaa9\ud53c\uc694(\u7687\u671d\u7f16\u5e74\u7eb2\u76ee\u88ab\u8981)\u300b 30\uad8c\uacfc \u300a\uc911\ud765\uc591\uc870\uac15\ubaa9\ube44\uc694(\u4e2d\u8208\u4e24\u671d\u7eb2\u76ee\u5907\u8981)\u300b18\uad8c\uc774 \uc788\uc5c8\uace0, \uc9c0\uc740\uc774\uac00 \ud655\uc2e4\ud558\uc9c0 \uc54a\uc740 \u300a\uc591\uc870\uac15\ubaa9\ube44\uc694(\u4e24\u671d\u7eb2\u76ee\u5907\u8981)\u300b 16\uad8c\uc774 \uc788\ub2e4.", "sentence_answer": "\uadf8 \ubc16\uc5d0 \uc9c4\uc81c(\u9648\u6d4e)\uac00 \uc9c0\uc740 \u300a \uac15\ubaa9\uc9d1\ub78c\uc815\uc624 (\u7eb2\u76ee\u96c6\u89c8\u6b63\u8bef)\u300b, \ud48d\uc9c0\ub824(\u51af\u667a\u8212)\uac00 \uc9c0\uc740 \u300a\uac15\ubaa9\uc9c8\uc2e4\u300b(\u7db1\u76ee\u8cea\u5be6), \uc6d0 \uc655\uc870 \uc2dc\ub300 \uc0ac\ub78c\uc778 \uc655\uc720\ud559(\u738b\u5e7c\u5b66)\uc774 \ud3b8\ucc2c\ud55c \u300a\uc790\uce58\ud1b5\uac10\uac15\ubaa9\uc9d1\ub78c(\u8d44\u6cbb\u901a\u9274\u7eb2\u76ee\u96c6\u89c8)\u300b 59\uad8c\uc774 \uc788\uc5c8\ub2e4.", "paragraph_id": "6548925-1-1", "paragraph_question": "question: \uac15\ubaa9\uc744 \uc5f0\uad6c\ud55c \uc9c4\uc81c\uac00 \uc9c0\uc740 \uc5ed\uc0ac\uc11c\ub294?, context: \u300a\uc790\uce58\ud1b5\uac10\uac15\ubaa9\u300b\uc758 \uac00\uce58\ub97c \ub192\uac8c \ub9e4\uae30\uac8c \ub41c \uac83\uc740 \ub0a8\uc1a1 \ud6c4\uae30, \uadf8\ub9ac\uace0 \uba85 \uc655\uc870\uc758 \uc2dc\ub300\uc5d0 \ub4e4\uc5b4\uc11c\uc600\ub2e4. \uac15\ubaa9\uc744 \uc5f0\uad6c\ud55c \uc724\uae30\uc2e0(\u5c39\u8d77\u8398)\uc774 \uc4f4 \u300a\ubc1c\uba85(\u767c\u660e)\u300b\uff0c\uc720\uc6b0\uc775(\u5289\u53cb\u76ca)\uc758 \u300a\uac15\ubaa9\uc11c\ubc95(\u7db1\u76ee\u66f8\u6cd5)\u300b\uff0c\uc655\uadf9\uad00(\u6c6a\u514b\u5bec)\uc758 \u300a\uac15\ubaa9\ubc94\ub840\uace0\uc774(\u7db1\u76ee\u51e1\u4f8b\u8003\u7570)\u300b\ub97c \uad6c\uc6b0(\u77bf\u4f51)\uac00 \uad50\uac10\ud574 \uc601\ub77d(\u6c38\u6a02) 8\ub144\uc5d0 \u300a\uc790\uce58\ud1b5\uac10\uac15\ubaa9\uc9d1\ub78c\uc804\uc624\u300b(\u8d44\u6cbb\u901a\u9274\u7eb2\u76ee\u96c6\u89c8\u954c\u8bef)\ub77c\ub294 \uc81c\ubaa9\uc73c\ub85c \uac04\ud589\ud558\uc600\ub2e4. \uc22d\uc815(\u5d07\u798e) 3\ub144\uc5d0 \uc9c4\uc778\uc11d(\u9673\u4ec1\u932b)\uc774 \uc774 \uc138 \uad8c\uc758 \ucc45\uc744 \uac16\uace0 \ub2e4\uc2dc \ud569\uccd0 \uc778\uc1c4\ud560 \ub54c \uc8fc\ud76c\uc640 \uc655\ubc31(\u738b\u67cf), \ubb38\ucc9c\uc6b0(\u6587\u5929\u7950), \uc774\ubc29\uc790(\u674e\u65b9\u5b50), \uc724\uae30\uc2e0, \uc720\uc720\uc775, \uac08\uacc4\uc2e0(\u63ed\u5092\u65af), \uac00\uc120\uac70(\u8cc0\u5584\u6998), \uc608\uc0ac\uc758(\u502a\u58eb\u6bc5), \uc655\uadf9\uad00, \uc655\uc720\ud559, \uc11c\uc18c\ubb38(\u5f90\u662d\u6587), \uc9c4\uc81c(\u9673\u6fdf), \uc591\ubb38\uc815(\u694a\u6587\u8c9e), \ud48d\uc9c0\uc11c(\u99ae\u667a\u8212), \ud669\uc911\ucd08(\u9ec4\u4ef2\u662d), \uc5ec\uc774\ub2a5(\u4f59\u4ee5\u80fd) \ub4f1 \ub9ce\uc740 \uc0ac\ub78c\ub4e4\uc758 \uc11c\ubb38\uc744 \uc2e4\uc5c8\ub2e4. \uadf8 \ubc16\uc5d0 \uc9c4\uc81c(\u9648\u6d4e)\uac00 \uc9c0\uc740 \u300a\uac15\ubaa9\uc9d1\ub78c\uc815\uc624(\u7eb2\u76ee\u96c6\u89c8\u6b63\u8bef)\u300b, \ud48d\uc9c0\ub824(\u51af\u667a\u8212)\uac00 \uc9c0\uc740 \u300a\uac15\ubaa9\uc9c8\uc2e4\u300b(\u7db1\u76ee\u8cea\u5be6), \uc6d0 \uc655\uc870 \uc2dc\ub300 \uc0ac\ub78c\uc778 \uc655\uc720\ud559(\u738b\u5e7c\u5b66)\uc774 \ud3b8\ucc2c\ud55c \u300a\uc790\uce58\ud1b5\uac10\uac15\ubaa9\uc9d1\ub78c(\u8d44\u6cbb\u901a\u9274\u7eb2\u76ee\u96c6\u89c8)\u300b 59\uad8c\uc774 \uc788\uc5c8\ub2e4. \ub610\ud55c \uc9c4\uacbd\uc218(\u9648\u6871\u4fee)\uac00 \uc9c0\uc740 \u300a\ud1b5\uac10\uc18d\ud3b8(\u901a\u9274\u7eed\u7f16)\u300b 24\uad8c, \uc9c4\uade0(\u9648\u5747)\uc774 \uc9c0\uc740 \u300a\ud669\uc870\ud3b8\ub144\uac15\ubaa9\ud53c\uc694(\u7687\u671d\u7f16\u5e74\u7eb2\u76ee\u88ab\u8981)\u300b 30\uad8c\uacfc \u300a\uc911\ud765\uc591\uc870\uac15\ubaa9\ube44\uc694(\u4e2d\u8208\u4e24\u671d\u7eb2\u76ee\u5907\u8981)\u300b18\uad8c\uc774 \uc788\uc5c8\uace0, \uc9c0\uc740\uc774\uac00 \ud655\uc2e4\ud558\uc9c0 \uc54a\uc740 \u300a\uc591\uc870\uac15\ubaa9\ube44\uc694(\u4e24\u671d\u7eb2\u76ee\u5907\u8981)\u300b 16\uad8c\uc774 \uc788\ub2e4."} {"question": "\uc601\uc5b4 \ub2e8\uc5b4 \uc911 guarantee\ub294 \uc5b4\ub514\uc5d0\uc11c \uc720\ub798\ud55c \ub2e8\uc5b4\uc778\uac00?", "paragraph": "\ud604\ub300 \uc601\uc5b4\uc5d0\uc11c\ub3c4 \uc5ec\uc804\ud788 \uac8c\ub974\ub9cc\uc5b4\uc5d0\uc11c \uc720\ub798\ud55c \ub2e8\uc5b4\uc640 \ub3d9\uc758\uc5b4\uc778 \ub77c\ud2f4\uc5b4\uc5d0\uc11c \uc720\ub798\ud55c \ub2e8\uc5b4\ub97c \uc120\ud0dd\ud558\uc5ec \uc0ac\uc6a9\ud560 \uc218 \uc788\ub2e4. \uc608\ub97c \ub4e4\uba74 come\uacfc arrive, sight\uc640 vision, freedom\uacfc liberty \ub4f1\uc774 \uadf8\uac83\uc774\ub2e4. \uc5b4\ub5a4 \uacbd\uc6b0\uc5d0\ub294 \uc5ec\uae30\uc5d0 \ud504\ub791\uc2a4\uc5b4\uc5d0\uc11c \uc720\ub798\ud55c \ub3d9\uc758\uc5b4\uac00 \ucd94\uac00\ub41c\ub2e4. \uc608\ub97c \ub4e4\uc5b4 \uac8c\ub974\ub9cc\uc5b4\uc5d0\uc11c \uc720\ub798\ud55c oversee \ub300\uc2e0 \ub77c\ud2f4\uc5b4\uc5d0\uc11c \uc720\ub798\ud55c supervise\ub098 \ud504\ub791\uc2a4\uc5b4\uc5d0\uc11c \uc720\ub798\ud55c survey\ub97c \ub3d9\uc758\uc5b4\ub85c \uc0ac\uc6a9\ud560 \uc218 \uc788\ub2e4. \uc774\uc678\uc5d0\ub3c4 \uc601\uc5b4\uc5d0\ub294 \ub2e4\uc591\ud55c \ucd9c\ucc98\uc5d0\uc11c \ub4e4\uc5ec\uc628 \ub2e8\uc5b4\ub4e4\uc774 \uc788\ub2e4. \uc608\ub97c \ub4e4\uc5b4 warranty\ub294 \uc575\uae00\ub85c\ub178\ub974\ub9cc\uc5b4\uc5d0\uc11c \uc628 \uac83\uc774\uace0 guarantee\ub294 \ud398\ub974\uc2dc\uc544\uacc4 \ud504\ub791\uc2a4\uc5b4\uc5d0\uc11c \uc628 \uac83\uc774\ub2e4. \uc5b4\ub5a4 \ub2e8\uc5b4\ub4e4\uc740 \uc774\ub807\uac8c \uc5ec\ub7ec \uacf3\uc5d0\uc11c \uc720\ub798\ud55c \ub3d9\uc758\uc5b4\ub4e4\uc774 \ub450\ub8e8 \uc4f0\uc774\uae30\ub3c4 \ud558\ub294\ub370 \uace0\ub300 \uc601\uc5b4\uc5d0\uc11c\ubd80\ud130 \uc0ac\uc6a9\ub41c sick\ub294 \uace0\ub300 \ub178\ub974\ub4dc\uc5b4\uc5d0\uc11c \uc720\ub798\ud55c ill, \ud504\ub791\uc2a4\uc5b4\uc5d0\uc11c \uc720\ub798\ud55c infirm, \ub77c\ud2f4\uc5b4\uc5d0\uc11c \uc720\ub798\ud55c afflicted\uc640 \uac19\uc740 \ub2e8\uc5b4\ub4e4\uacfc \ub3d9\uc758\uc5b4\uad70\uc744 \uc774\ub8e8\uace0 \uc788\ub2e4. \uc601\uc5b4\uc758 \uc0ac\uc6a9\uc5d0\uc11c \uc774\ub7ec\ud55c \ub3d9\uc758\uc5b4\uc758 \uc120\ud0dd\uc740 \ub258\uc559\uc2a4\uc640 \uac19\uc740 \uac83\uc744 \ud568\uaed8 \uc804\ub2ec\ud558\ub294 \uc218\ub2e8\uc774 \ub418\uae30\ub3c4 \ud55c\ub2e4.", "answer": "\ud398\ub974\uc2dc\uc544\uacc4 \ud504\ub791\uc2a4\uc5b4", "sentence": "\uc608\ub97c \ub4e4\uc5b4 warranty\ub294 \uc575\uae00\ub85c\ub178\ub974\ub9cc\uc5b4\uc5d0\uc11c \uc628 \uac83\uc774\uace0 guarantee\ub294 \ud398\ub974\uc2dc\uc544\uacc4 \ud504\ub791\uc2a4\uc5b4 \uc5d0\uc11c \uc628 \uac83\uc774\ub2e4.", "paragraph_sentence": "\ud604\ub300 \uc601\uc5b4\uc5d0\uc11c\ub3c4 \uc5ec\uc804\ud788 \uac8c\ub974\ub9cc\uc5b4\uc5d0\uc11c \uc720\ub798\ud55c \ub2e8\uc5b4\uc640 \ub3d9\uc758\uc5b4\uc778 \ub77c\ud2f4\uc5b4\uc5d0\uc11c \uc720\ub798\ud55c \ub2e8\uc5b4\ub97c \uc120\ud0dd\ud558\uc5ec \uc0ac\uc6a9\ud560 \uc218 \uc788\ub2e4. \uc608\ub97c \ub4e4\uba74 come\uacfc arrive, sight\uc640 vision, freedom\uacfc liberty \ub4f1\uc774 \uadf8\uac83\uc774\ub2e4. \uc5b4\ub5a4 \uacbd\uc6b0\uc5d0\ub294 \uc5ec\uae30\uc5d0 \ud504\ub791\uc2a4\uc5b4\uc5d0\uc11c \uc720\ub798\ud55c \ub3d9\uc758\uc5b4\uac00 \ucd94\uac00\ub41c\ub2e4. \uc608\ub97c \ub4e4\uc5b4 \uac8c\ub974\ub9cc\uc5b4\uc5d0\uc11c \uc720\ub798\ud55c oversee \ub300\uc2e0 \ub77c\ud2f4\uc5b4\uc5d0\uc11c \uc720\ub798\ud55c supervise\ub098 \ud504\ub791\uc2a4\uc5b4\uc5d0\uc11c \uc720\ub798\ud55c survey\ub97c \ub3d9\uc758\uc5b4\ub85c \uc0ac\uc6a9\ud560 \uc218 \uc788\ub2e4. \uc774\uc678\uc5d0\ub3c4 \uc601\uc5b4\uc5d0\ub294 \ub2e4\uc591\ud55c \ucd9c\ucc98\uc5d0\uc11c \ub4e4\uc5ec\uc628 \ub2e8\uc5b4\ub4e4\uc774 \uc788\ub2e4. \uc608\ub97c \ub4e4\uc5b4 warranty\ub294 \uc575\uae00\ub85c\ub178\ub974\ub9cc\uc5b4\uc5d0\uc11c \uc628 \uac83\uc774\uace0 guarantee\ub294 \ud398\ub974\uc2dc\uc544\uacc4 \ud504\ub791\uc2a4\uc5b4 \uc5d0\uc11c \uc628 \uac83\uc774\ub2e4. \uc5b4\ub5a4 \ub2e8\uc5b4\ub4e4\uc740 \uc774\ub807\uac8c \uc5ec\ub7ec \uacf3\uc5d0\uc11c \uc720\ub798\ud55c \ub3d9\uc758\uc5b4\ub4e4\uc774 \ub450\ub8e8 \uc4f0\uc774\uae30\ub3c4 \ud558\ub294\ub370 \uace0\ub300 \uc601\uc5b4\uc5d0\uc11c\ubd80\ud130 \uc0ac\uc6a9\ub41c sick\ub294 \uace0\ub300 \ub178\ub974\ub4dc\uc5b4\uc5d0\uc11c \uc720\ub798\ud55c ill, \ud504\ub791\uc2a4\uc5b4\uc5d0\uc11c \uc720\ub798\ud55c infirm, \ub77c\ud2f4\uc5b4\uc5d0\uc11c \uc720\ub798\ud55c afflicted\uc640 \uac19\uc740 \ub2e8\uc5b4\ub4e4\uacfc \ub3d9\uc758\uc5b4\uad70\uc744 \uc774\ub8e8\uace0 \uc788\ub2e4. \uc601\uc5b4\uc758 \uc0ac\uc6a9\uc5d0\uc11c \uc774\ub7ec\ud55c \ub3d9\uc758\uc5b4\uc758 \uc120\ud0dd\uc740 \ub258\uc559\uc2a4\uc640 \uac19\uc740 \uac83\uc744 \ud568\uaed8 \uc804\ub2ec\ud558\ub294 \uc218\ub2e8\uc774 \ub418\uae30\ub3c4 \ud55c\ub2e4.", "paragraph_answer": "\ud604\ub300 \uc601\uc5b4\uc5d0\uc11c\ub3c4 \uc5ec\uc804\ud788 \uac8c\ub974\ub9cc\uc5b4\uc5d0\uc11c \uc720\ub798\ud55c \ub2e8\uc5b4\uc640 \ub3d9\uc758\uc5b4\uc778 \ub77c\ud2f4\uc5b4\uc5d0\uc11c \uc720\ub798\ud55c \ub2e8\uc5b4\ub97c \uc120\ud0dd\ud558\uc5ec \uc0ac\uc6a9\ud560 \uc218 \uc788\ub2e4. \uc608\ub97c \ub4e4\uba74 come\uacfc arrive, sight\uc640 vision, freedom\uacfc liberty \ub4f1\uc774 \uadf8\uac83\uc774\ub2e4. \uc5b4\ub5a4 \uacbd\uc6b0\uc5d0\ub294 \uc5ec\uae30\uc5d0 \ud504\ub791\uc2a4\uc5b4\uc5d0\uc11c \uc720\ub798\ud55c \ub3d9\uc758\uc5b4\uac00 \ucd94\uac00\ub41c\ub2e4. \uc608\ub97c \ub4e4\uc5b4 \uac8c\ub974\ub9cc\uc5b4\uc5d0\uc11c \uc720\ub798\ud55c oversee \ub300\uc2e0 \ub77c\ud2f4\uc5b4\uc5d0\uc11c \uc720\ub798\ud55c supervise\ub098 \ud504\ub791\uc2a4\uc5b4\uc5d0\uc11c \uc720\ub798\ud55c survey\ub97c \ub3d9\uc758\uc5b4\ub85c \uc0ac\uc6a9\ud560 \uc218 \uc788\ub2e4. \uc774\uc678\uc5d0\ub3c4 \uc601\uc5b4\uc5d0\ub294 \ub2e4\uc591\ud55c \ucd9c\ucc98\uc5d0\uc11c \ub4e4\uc5ec\uc628 \ub2e8\uc5b4\ub4e4\uc774 \uc788\ub2e4. \uc608\ub97c \ub4e4\uc5b4 warranty\ub294 \uc575\uae00\ub85c\ub178\ub974\ub9cc\uc5b4\uc5d0\uc11c \uc628 \uac83\uc774\uace0 guarantee\ub294 \ud398\ub974\uc2dc\uc544\uacc4 \ud504\ub791\uc2a4\uc5b4 \uc5d0\uc11c \uc628 \uac83\uc774\ub2e4. \uc5b4\ub5a4 \ub2e8\uc5b4\ub4e4\uc740 \uc774\ub807\uac8c \uc5ec\ub7ec \uacf3\uc5d0\uc11c \uc720\ub798\ud55c \ub3d9\uc758\uc5b4\ub4e4\uc774 \ub450\ub8e8 \uc4f0\uc774\uae30\ub3c4 \ud558\ub294\ub370 \uace0\ub300 \uc601\uc5b4\uc5d0\uc11c\ubd80\ud130 \uc0ac\uc6a9\ub41c sick\ub294 \uace0\ub300 \ub178\ub974\ub4dc\uc5b4\uc5d0\uc11c \uc720\ub798\ud55c ill, \ud504\ub791\uc2a4\uc5b4\uc5d0\uc11c \uc720\ub798\ud55c infirm, \ub77c\ud2f4\uc5b4\uc5d0\uc11c \uc720\ub798\ud55c afflicted\uc640 \uac19\uc740 \ub2e8\uc5b4\ub4e4\uacfc \ub3d9\uc758\uc5b4\uad70\uc744 \uc774\ub8e8\uace0 \uc788\ub2e4. \uc601\uc5b4\uc758 \uc0ac\uc6a9\uc5d0\uc11c \uc774\ub7ec\ud55c \ub3d9\uc758\uc5b4\uc758 \uc120\ud0dd\uc740 \ub258\uc559\uc2a4\uc640 \uac19\uc740 \uac83\uc744 \ud568\uaed8 \uc804\ub2ec\ud558\ub294 \uc218\ub2e8\uc774 \ub418\uae30\ub3c4 \ud55c\ub2e4.", "sentence_answer": "\uc608\ub97c \ub4e4\uc5b4 warranty\ub294 \uc575\uae00\ub85c\ub178\ub974\ub9cc\uc5b4\uc5d0\uc11c \uc628 \uac83\uc774\uace0 guarantee\ub294 \ud398\ub974\uc2dc\uc544\uacc4 \ud504\ub791\uc2a4\uc5b4 \uc5d0\uc11c \uc628 \uac83\uc774\ub2e4.", "paragraph_id": "6465644-6-1", "paragraph_question": "question: \uc601\uc5b4 \ub2e8\uc5b4 \uc911 guarantee\ub294 \uc5b4\ub514\uc5d0\uc11c \uc720\ub798\ud55c \ub2e8\uc5b4\uc778\uac00?, context: \ud604\ub300 \uc601\uc5b4\uc5d0\uc11c\ub3c4 \uc5ec\uc804\ud788 \uac8c\ub974\ub9cc\uc5b4\uc5d0\uc11c \uc720\ub798\ud55c \ub2e8\uc5b4\uc640 \ub3d9\uc758\uc5b4\uc778 \ub77c\ud2f4\uc5b4\uc5d0\uc11c \uc720\ub798\ud55c \ub2e8\uc5b4\ub97c \uc120\ud0dd\ud558\uc5ec \uc0ac\uc6a9\ud560 \uc218 \uc788\ub2e4. \uc608\ub97c \ub4e4\uba74 come\uacfc arrive, sight\uc640 vision, freedom\uacfc liberty \ub4f1\uc774 \uadf8\uac83\uc774\ub2e4. \uc5b4\ub5a4 \uacbd\uc6b0\uc5d0\ub294 \uc5ec\uae30\uc5d0 \ud504\ub791\uc2a4\uc5b4\uc5d0\uc11c \uc720\ub798\ud55c \ub3d9\uc758\uc5b4\uac00 \ucd94\uac00\ub41c\ub2e4. \uc608\ub97c \ub4e4\uc5b4 \uac8c\ub974\ub9cc\uc5b4\uc5d0\uc11c \uc720\ub798\ud55c oversee \ub300\uc2e0 \ub77c\ud2f4\uc5b4\uc5d0\uc11c \uc720\ub798\ud55c supervise\ub098 \ud504\ub791\uc2a4\uc5b4\uc5d0\uc11c \uc720\ub798\ud55c survey\ub97c \ub3d9\uc758\uc5b4\ub85c \uc0ac\uc6a9\ud560 \uc218 \uc788\ub2e4. \uc774\uc678\uc5d0\ub3c4 \uc601\uc5b4\uc5d0\ub294 \ub2e4\uc591\ud55c \ucd9c\ucc98\uc5d0\uc11c \ub4e4\uc5ec\uc628 \ub2e8\uc5b4\ub4e4\uc774 \uc788\ub2e4. \uc608\ub97c \ub4e4\uc5b4 warranty\ub294 \uc575\uae00\ub85c\ub178\ub974\ub9cc\uc5b4\uc5d0\uc11c \uc628 \uac83\uc774\uace0 guarantee\ub294 \ud398\ub974\uc2dc\uc544\uacc4 \ud504\ub791\uc2a4\uc5b4\uc5d0\uc11c \uc628 \uac83\uc774\ub2e4. \uc5b4\ub5a4 \ub2e8\uc5b4\ub4e4\uc740 \uc774\ub807\uac8c \uc5ec\ub7ec \uacf3\uc5d0\uc11c \uc720\ub798\ud55c \ub3d9\uc758\uc5b4\ub4e4\uc774 \ub450\ub8e8 \uc4f0\uc774\uae30\ub3c4 \ud558\ub294\ub370 \uace0\ub300 \uc601\uc5b4\uc5d0\uc11c\ubd80\ud130 \uc0ac\uc6a9\ub41c sick\ub294 \uace0\ub300 \ub178\ub974\ub4dc\uc5b4\uc5d0\uc11c \uc720\ub798\ud55c ill, \ud504\ub791\uc2a4\uc5b4\uc5d0\uc11c \uc720\ub798\ud55c infirm, \ub77c\ud2f4\uc5b4\uc5d0\uc11c \uc720\ub798\ud55c afflicted\uc640 \uac19\uc740 \ub2e8\uc5b4\ub4e4\uacfc \ub3d9\uc758\uc5b4\uad70\uc744 \uc774\ub8e8\uace0 \uc788\ub2e4. \uc601\uc5b4\uc758 \uc0ac\uc6a9\uc5d0\uc11c \uc774\ub7ec\ud55c \ub3d9\uc758\uc5b4\uc758 \uc120\ud0dd\uc740 \ub258\uc559\uc2a4\uc640 \uac19\uc740 \uac83\uc744 \ud568\uaed8 \uc804\ub2ec\ud558\ub294 \uc218\ub2e8\uc774 \ub418\uae30\ub3c4 \ud55c\ub2e4."} {"question": "\uc81c2\ucc28 \uc138\uacc4\ub300\uc804\uc5d0\uc11c \ub3c5\uc77c\uad70\uc774 \uc774\uc6a9\ud55c \uc911 \uc804\ucc28\ub294 \ubb34\uc5c7\uc778\uac00\uc694?", "paragraph": "4\ud638 \uc804\ucc28(\ub3c5\uc77c\uc5b4: Panzerkampfwagen IV; Pz.Kpfw. IV \ud310\ucc98\uce84\ud504\ubc14\uac90 \ud53c\uc5b4)\ub294 \uc81c2\ucc28 \uc138\uacc4\ub300\uc804 \ub2f9\uc2dc \ub3c5\uc77c\uad70\uc774 \uc0ac\uc6a9\ud588\ub358 \uc911(\u4e2d)\uc804\ucc28\uc774\ub2e4. \ubcf8\ub798 \ubcf4\ubcd1 \uc9c0\uc6d0\uc744 \ubaa9\uc801\uc73c\ub85c \uacc4\ud68d\ub418\uc5c8\uc73c\uba70 \ub300\uc804\ucc28\uc804\uc740 \ubcf8 \uc6a9\ub3c4\uac00 \uc544\ub2c8\uc5c8\uc73c\ub098, \ub300\uc804 \ubc1c\ubc1c\ud6c4 \ub300\uc804\ucc28\uc804\uc744 \ub9e1\uc558\ub358 3\ud638 \uc804\ucc28\uc758 \ubb38\uc81c\uac00 \ub3c4\ucd9c\ub418\uace0 \ub3d9\ubd80\uc804\uc120\uc5d0\uc11c T-34\uc640 \ub9c8\uc8fc\ud558\uac8c \ub418\uba74\uc11c \uad6c\ud615\ud654\ub41c 3\ud638 \uc804\ucc28\ub97c \ub300\uc2e0\ud574 \uc8fc\ub825\uc744 \ub3c4\ub9e1\uac8c \ub418\uc5c8\ub2e4. 1936\ub144 \uac1c\ubc1c\uc774 \uc2dc\uc791\ub41c \uc774\ud6c4\ub85c \uc138\uacc4\ub300\uc804\uc774 \ub05d\ub0a0 \ub54c\uae4c\uc9c0 \ub04a\uc784\uc5c6\uc774 \uc0dd\uc0b0\ub41c \uc720\uc77c\ud55c \uc804\ucc28\ub85c\uc11c \ub3c5\uc77c\uad70\uc774 \ud65c\ub3d9\ud55c \ubaa8\ub4e0 \uc804\uc120\uc5d0 \ud22c\uc785\ub418\uc5c8\uc73c\uba70 \ub3c5\uc77c\uad70 \uc804\ucc28 \uc911\uc5d0\uc11c \uac00\uc7a5 \ub9ce\uc774 \uc0dd\uc0b0\ub41c \uc804\ucc28\ub85c\uc11c \uad6c\ucd95\uc804\ucc28\uc640 \uc790\uc8fc\ub300\uacf5\ud3ec \ub4f1\uacfc \uac19\uc740 \ub2e4\uc218\uc758 \uc804\ud22c \ucc28\ub7c9\uc758 \ubaa8\ud0dc\uac00 \ub418\uae30\ub3c4 \ud558\uc600\ub2e4. \uc57d 300\ub300\uac00 \ub3c5\uc77c \ubc16\uc73c\ub85c \uc218\ucd9c\ub418\uc5c8\ub294\ub370 \uc774\ub4e4 \uc911 \uc77c\ubd80\ub294 \ub2e4\uc2dc \uc2dc\ub9ac\uc544\ub85c \uc218\ucd9c\ub418\uc5b4 6\uc77c \uc804\uc7c1\uc5d0 \ud22c\uc785\ub418\uc5c8\ub2e4.", "answer": "4\ud638 \uc804\ucc28", "sentence": "4\ud638 \uc804\ucc28 (\ub3c5\uc77c\uc5b4: Panzerkampfwagen IV; Pz.", "paragraph_sentence": " 4\ud638 \uc804\ucc28 (\ub3c5\uc77c\uc5b4: Panzerkampfwagen IV; Pz. Kpfw. IV \ud310\ucc98\uce84\ud504\ubc14\uac90 \ud53c\uc5b4)\ub294 \uc81c2\ucc28 \uc138\uacc4\ub300\uc804 \ub2f9\uc2dc \ub3c5\uc77c\uad70\uc774 \uc0ac\uc6a9\ud588\ub358 \uc911(\u4e2d)\uc804\ucc28\uc774\ub2e4. \ubcf8\ub798 \ubcf4\ubcd1 \uc9c0\uc6d0\uc744 \ubaa9\uc801\uc73c\ub85c \uacc4\ud68d\ub418\uc5c8\uc73c\uba70 \ub300\uc804\ucc28\uc804\uc740 \ubcf8 \uc6a9\ub3c4\uac00 \uc544\ub2c8\uc5c8\uc73c\ub098, \ub300\uc804 \ubc1c\ubc1c\ud6c4 \ub300\uc804\ucc28\uc804\uc744 \ub9e1\uc558\ub358 3\ud638 \uc804\ucc28\uc758 \ubb38\uc81c\uac00 \ub3c4\ucd9c\ub418\uace0 \ub3d9\ubd80\uc804\uc120\uc5d0\uc11c T-34\uc640 \ub9c8\uc8fc\ud558\uac8c \ub418\uba74\uc11c \uad6c\ud615\ud654\ub41c 3\ud638 \uc804\ucc28\ub97c \ub300\uc2e0\ud574 \uc8fc\ub825\uc744 \ub3c4\ub9e1\uac8c \ub418\uc5c8\ub2e4. 1936\ub144 \uac1c\ubc1c\uc774 \uc2dc\uc791\ub41c \uc774\ud6c4\ub85c \uc138\uacc4\ub300\uc804\uc774 \ub05d\ub0a0 \ub54c\uae4c\uc9c0 \ub04a\uc784\uc5c6\uc774 \uc0dd\uc0b0\ub41c \uc720\uc77c\ud55c \uc804\ucc28\ub85c\uc11c \ub3c5\uc77c\uad70\uc774 \ud65c\ub3d9\ud55c \ubaa8\ub4e0 \uc804\uc120\uc5d0 \ud22c\uc785\ub418\uc5c8\uc73c\uba70 \ub3c5\uc77c\uad70 \uc804\ucc28 \uc911\uc5d0\uc11c \uac00\uc7a5 \ub9ce\uc774 \uc0dd\uc0b0\ub41c \uc804\ucc28\ub85c\uc11c \uad6c\ucd95\uc804\ucc28\uc640 \uc790\uc8fc\ub300\uacf5\ud3ec \ub4f1\uacfc \uac19\uc740 \ub2e4\uc218\uc758 \uc804\ud22c \ucc28\ub7c9\uc758 \ubaa8\ud0dc\uac00 \ub418\uae30\ub3c4 \ud558\uc600\ub2e4. \uc57d 300\ub300\uac00 \ub3c5\uc77c \ubc16\uc73c\ub85c \uc218\ucd9c\ub418\uc5c8\ub294\ub370 \uc774\ub4e4 \uc911 \uc77c\ubd80\ub294 \ub2e4\uc2dc \uc2dc\ub9ac\uc544\ub85c \uc218\ucd9c\ub418\uc5b4 6\uc77c \uc804\uc7c1\uc5d0 \ud22c\uc785\ub418\uc5c8\ub2e4.", "paragraph_answer": " 4\ud638 \uc804\ucc28 (\ub3c5\uc77c\uc5b4: Panzerkampfwagen IV; Pz.Kpfw. IV \ud310\ucc98\uce84\ud504\ubc14\uac90 \ud53c\uc5b4)\ub294 \uc81c2\ucc28 \uc138\uacc4\ub300\uc804 \ub2f9\uc2dc \ub3c5\uc77c\uad70\uc774 \uc0ac\uc6a9\ud588\ub358 \uc911(\u4e2d)\uc804\ucc28\uc774\ub2e4. \ubcf8\ub798 \ubcf4\ubcd1 \uc9c0\uc6d0\uc744 \ubaa9\uc801\uc73c\ub85c \uacc4\ud68d\ub418\uc5c8\uc73c\uba70 \ub300\uc804\ucc28\uc804\uc740 \ubcf8 \uc6a9\ub3c4\uac00 \uc544\ub2c8\uc5c8\uc73c\ub098, \ub300\uc804 \ubc1c\ubc1c\ud6c4 \ub300\uc804\ucc28\uc804\uc744 \ub9e1\uc558\ub358 3\ud638 \uc804\ucc28\uc758 \ubb38\uc81c\uac00 \ub3c4\ucd9c\ub418\uace0 \ub3d9\ubd80\uc804\uc120\uc5d0\uc11c T-34\uc640 \ub9c8\uc8fc\ud558\uac8c \ub418\uba74\uc11c \uad6c\ud615\ud654\ub41c 3\ud638 \uc804\ucc28\ub97c \ub300\uc2e0\ud574 \uc8fc\ub825\uc744 \ub3c4\ub9e1\uac8c \ub418\uc5c8\ub2e4. 1936\ub144 \uac1c\ubc1c\uc774 \uc2dc\uc791\ub41c \uc774\ud6c4\ub85c \uc138\uacc4\ub300\uc804\uc774 \ub05d\ub0a0 \ub54c\uae4c\uc9c0 \ub04a\uc784\uc5c6\uc774 \uc0dd\uc0b0\ub41c \uc720\uc77c\ud55c \uc804\ucc28\ub85c\uc11c \ub3c5\uc77c\uad70\uc774 \ud65c\ub3d9\ud55c \ubaa8\ub4e0 \uc804\uc120\uc5d0 \ud22c\uc785\ub418\uc5c8\uc73c\uba70 \ub3c5\uc77c\uad70 \uc804\ucc28 \uc911\uc5d0\uc11c \uac00\uc7a5 \ub9ce\uc774 \uc0dd\uc0b0\ub41c \uc804\ucc28\ub85c\uc11c \uad6c\ucd95\uc804\ucc28\uc640 \uc790\uc8fc\ub300\uacf5\ud3ec \ub4f1\uacfc \uac19\uc740 \ub2e4\uc218\uc758 \uc804\ud22c \ucc28\ub7c9\uc758 \ubaa8\ud0dc\uac00 \ub418\uae30\ub3c4 \ud558\uc600\ub2e4. \uc57d 300\ub300\uac00 \ub3c5\uc77c \ubc16\uc73c\ub85c \uc218\ucd9c\ub418\uc5c8\ub294\ub370 \uc774\ub4e4 \uc911 \uc77c\ubd80\ub294 \ub2e4\uc2dc \uc2dc\ub9ac\uc544\ub85c \uc218\ucd9c\ub418\uc5b4 6\uc77c \uc804\uc7c1\uc5d0 \ud22c\uc785\ub418\uc5c8\ub2e4.", "sentence_answer": " 4\ud638 \uc804\ucc28 (\ub3c5\uc77c\uc5b4: Panzerkampfwagen IV; Pz.", "paragraph_id": "6471320-0-0", "paragraph_question": "question: \uc81c2\ucc28 \uc138\uacc4\ub300\uc804\uc5d0\uc11c \ub3c5\uc77c\uad70\uc774 \uc774\uc6a9\ud55c \uc911 \uc804\ucc28\ub294 \ubb34\uc5c7\uc778\uac00\uc694?, context: 4\ud638 \uc804\ucc28(\ub3c5\uc77c\uc5b4: Panzerkampfwagen IV; Pz.Kpfw. IV \ud310\ucc98\uce84\ud504\ubc14\uac90 \ud53c\uc5b4)\ub294 \uc81c2\ucc28 \uc138\uacc4\ub300\uc804 \ub2f9\uc2dc \ub3c5\uc77c\uad70\uc774 \uc0ac\uc6a9\ud588\ub358 \uc911(\u4e2d)\uc804\ucc28\uc774\ub2e4. \ubcf8\ub798 \ubcf4\ubcd1 \uc9c0\uc6d0\uc744 \ubaa9\uc801\uc73c\ub85c \uacc4\ud68d\ub418\uc5c8\uc73c\uba70 \ub300\uc804\ucc28\uc804\uc740 \ubcf8 \uc6a9\ub3c4\uac00 \uc544\ub2c8\uc5c8\uc73c\ub098, \ub300\uc804 \ubc1c\ubc1c\ud6c4 \ub300\uc804\ucc28\uc804\uc744 \ub9e1\uc558\ub358 3\ud638 \uc804\ucc28\uc758 \ubb38\uc81c\uac00 \ub3c4\ucd9c\ub418\uace0 \ub3d9\ubd80\uc804\uc120\uc5d0\uc11c T-34\uc640 \ub9c8\uc8fc\ud558\uac8c \ub418\uba74\uc11c \uad6c\ud615\ud654\ub41c 3\ud638 \uc804\ucc28\ub97c \ub300\uc2e0\ud574 \uc8fc\ub825\uc744 \ub3c4\ub9e1\uac8c \ub418\uc5c8\ub2e4. 1936\ub144 \uac1c\ubc1c\uc774 \uc2dc\uc791\ub41c \uc774\ud6c4\ub85c \uc138\uacc4\ub300\uc804\uc774 \ub05d\ub0a0 \ub54c\uae4c\uc9c0 \ub04a\uc784\uc5c6\uc774 \uc0dd\uc0b0\ub41c \uc720\uc77c\ud55c \uc804\ucc28\ub85c\uc11c \ub3c5\uc77c\uad70\uc774 \ud65c\ub3d9\ud55c \ubaa8\ub4e0 \uc804\uc120\uc5d0 \ud22c\uc785\ub418\uc5c8\uc73c\uba70 \ub3c5\uc77c\uad70 \uc804\ucc28 \uc911\uc5d0\uc11c \uac00\uc7a5 \ub9ce\uc774 \uc0dd\uc0b0\ub41c \uc804\ucc28\ub85c\uc11c \uad6c\ucd95\uc804\ucc28\uc640 \uc790\uc8fc\ub300\uacf5\ud3ec \ub4f1\uacfc \uac19\uc740 \ub2e4\uc218\uc758 \uc804\ud22c \ucc28\ub7c9\uc758 \ubaa8\ud0dc\uac00 \ub418\uae30\ub3c4 \ud558\uc600\ub2e4. \uc57d 300\ub300\uac00 \ub3c5\uc77c \ubc16\uc73c\ub85c \uc218\ucd9c\ub418\uc5c8\ub294\ub370 \uc774\ub4e4 \uc911 \uc77c\ubd80\ub294 \ub2e4\uc2dc \uc2dc\ub9ac\uc544\ub85c \uc218\ucd9c\ub418\uc5b4 6\uc77c \uc804\uc7c1\uc5d0 \ud22c\uc785\ub418\uc5c8\ub2e4."} {"question": "\ud310\ubb38\uc810 \ub3c4\ub07c \uc0b4\uc778 \uc0ac\uac74\uc774 \uc77c\uc5b4\ub09c \ud6c4 \uc2a4\ud2f8\uc6f0 \uc8fc\ud55c\ubbf8\uad70 \uc0ac\ub839\uad00\uc5d0 \ub530\ub77c \uc778\ubbfc\uad70\uc774 \uc124\uce58\ud55c \ubd88\ubc95 \ubc29\ubcbd\uc744 \uc81c\uac70\ud558\ub294 \uc791\uc804\uc774 \ubb34\uc5c7\uc778\uac00?", "paragraph": "\uc0ac\uac74\uc774 \ubc1c\uc0dd\ud558\uc790 \ubbf8\uad6d \ubc31\uc545\uad00\uc5d0\uc11c\ub294 \uc6cc\uc2f1\ud134 \ud2b9\ubcc4 \ub300\ucc45\ubc18\uc774 \uc18c\uc9d1\ub418\uc5c8\uc73c\uba70 \ubbf8\uad6d \uad6d\ubb34\ubd80\uacfc \ud568\uaed8 \"\uc774 \uc0ac\uac74\uc758 \uacb0\uacfc\ub85c \ube5a\uc5b4\uc9c0\ub294 \uc5b4\ub5a0\ud55c \uc0ac\ud0dc\uc5d0 \ub300\ud574\uc11c\ub3c4 \uadf8 \ucc45\uc784\uc740 \uc870\uc120\ubbfc\uc8fc\uc8fc\uc758\uc778\ubbfc\uacf5\ud654\uad6d\uc5d0 \uc788\ub2e4\"\ub294 \uacf5\ub3d9\uc131\uba85\uc744 \ub2f9\uc77c\uc5d0 \ubc1c\ud45c\ud558\uc600\ub2e4. \ub610\ud55c \uc81c\ub7f4\ub4dc \ud3ec\ub4dc \ubbf8\uad6d \ub300\ud1b5\ub839\uc758 \uba85\ub839\uc5d0 \ub530\ub77c \uc2a4\ud2f8\uc6f0 \uc8fc\ud55c\ubbf8\uad70 \uc0ac\ub839\uad00\uc740 \ubb38\uc81c\uc758 \ubbf8\ub8e8\ub098\ubb34\ub97c \ubca0\uace0 \uacf5\ub3d9\uacbd\ube44\uad6c\uc5ed \ub0b4\uc5d0 \uc778\ubbfc\uad70\uc774 \uc124\uce58\ud55c \ubd88\ubc95 \ubc29\ubcbd(\u9632\u58c1; \ubc14\ub9ac\uac8c\uc774\ud2b8 \ub4f1)\uc744 \uc81c\uac70\ud558\uae30 \uc704\ud55c \ud3f4 \ubc84\ub2c8\uc5b8 \uc791\uc804(Operation Paul Bunyan: \ubbf8\uad6d \uc804\uc124\uc5d0 \ub4f1\uc7a5\ud558\ub294 \uac70\uad6c\uc758 \ub098\ubb34\uafbc \ud3f4 \ubc84\ub2c8\uc5b8\uc5d0\uc11c \ub530\uc628 \uc791\uc804\uba85)\uc744 \uae30\ubcf8\uc73c\ub85c F-4, F-111, B-52 \ud3ed\uaca9\uae30, \ubbf8\ub4dc\uc6e8\uc774\ud638 \ub4f1\uc744 \ub3d9\uc6d0\ud558\ub294 \ub300\uaddc\ubaa8 \ubb34\ub825 \uc2dc\uc704 \uacc4\ud68d\uc744 \uc218\ub9bd\ud558\uc600\uace0, \uc804\ud22c\uc900\ube44\ud0dc\uc138\uc778 \ub370\ud504\ucf58 3\uc774 \ubc1c\ub839\ub418\uc5c8\ub2e4.", "answer": "\ud3f4 \ubc84\ub2c8\uc5b8", "sentence": "\ub610\ud55c \uc81c\ub7f4\ub4dc \ud3ec\ub4dc \ubbf8\uad6d \ub300\ud1b5\ub839\uc758 \uba85\ub839\uc5d0 \ub530\ub77c \uc2a4\ud2f8\uc6f0 \uc8fc\ud55c\ubbf8\uad70 \uc0ac\ub839\uad00\uc740 \ubb38\uc81c\uc758 \ubbf8\ub8e8\ub098\ubb34\ub97c \ubca0\uace0 \uacf5\ub3d9\uacbd\ube44\uad6c\uc5ed \ub0b4\uc5d0 \uc778\ubbfc\uad70\uc774 \uc124\uce58\ud55c \ubd88\ubc95 \ubc29\ubcbd(\u9632\u58c1; \ubc14\ub9ac\uac8c\uc774\ud2b8 \ub4f1)\uc744 \uc81c\uac70\ud558\uae30 \uc704\ud55c \ud3f4 \ubc84\ub2c8\uc5b8 \uc791\uc804(Operation Paul Bunyan: \ubbf8\uad6d \uc804\uc124\uc5d0 \ub4f1\uc7a5\ud558\ub294 \uac70\uad6c\uc758 \ub098\ubb34\uafbc \ud3f4 \ubc84\ub2c8\uc5b8\uc5d0\uc11c \ub530\uc628 \uc791\uc804\uba85)\uc744 \uae30\ubcf8\uc73c\ub85c F-4, F-111, B-52 \ud3ed\uaca9\uae30, \ubbf8\ub4dc\uc6e8\uc774\ud638 \ub4f1\uc744 \ub3d9\uc6d0\ud558\ub294 \ub300\uaddc\ubaa8 \ubb34\ub825 \uc2dc\uc704 \uacc4\ud68d\uc744 \uc218\ub9bd\ud558\uc600\uace0, \uc804\ud22c\uc900\ube44\ud0dc\uc138\uc778 \ub370\ud504\ucf58 3\uc774 \ubc1c\ub839\ub418\uc5c8\ub2e4.", "paragraph_sentence": "\uc0ac\uac74\uc774 \ubc1c\uc0dd\ud558\uc790 \ubbf8\uad6d \ubc31\uc545\uad00\uc5d0\uc11c\ub294 \uc6cc\uc2f1\ud134 \ud2b9\ubcc4 \ub300\ucc45\ubc18\uc774 \uc18c\uc9d1\ub418\uc5c8\uc73c\uba70 \ubbf8\uad6d \uad6d\ubb34\ubd80\uacfc \ud568\uaed8 \"\uc774 \uc0ac\uac74\uc758 \uacb0\uacfc\ub85c \ube5a\uc5b4\uc9c0\ub294 \uc5b4\ub5a0\ud55c \uc0ac\ud0dc\uc5d0 \ub300\ud574\uc11c\ub3c4 \uadf8 \ucc45\uc784\uc740 \uc870\uc120\ubbfc\uc8fc\uc8fc\uc758\uc778\ubbfc\uacf5\ud654\uad6d\uc5d0 \uc788\ub2e4\"\ub294 \uacf5\ub3d9\uc131\uba85\uc744 \ub2f9\uc77c\uc5d0 \ubc1c\ud45c\ud558\uc600\ub2e4. \ub610\ud55c \uc81c\ub7f4\ub4dc \ud3ec\ub4dc \ubbf8\uad6d \ub300\ud1b5\ub839\uc758 \uba85\ub839\uc5d0 \ub530\ub77c \uc2a4\ud2f8\uc6f0 \uc8fc\ud55c\ubbf8\uad70 \uc0ac\ub839\uad00\uc740 \ubb38\uc81c\uc758 \ubbf8\ub8e8\ub098\ubb34\ub97c \ubca0\uace0 \uacf5\ub3d9\uacbd\ube44\uad6c\uc5ed \ub0b4\uc5d0 \uc778\ubbfc\uad70\uc774 \uc124\uce58\ud55c \ubd88\ubc95 \ubc29\ubcbd(\u9632\u58c1; \ubc14\ub9ac\uac8c\uc774\ud2b8 \ub4f1)\uc744 \uc81c\uac70\ud558\uae30 \uc704\ud55c \ud3f4 \ubc84\ub2c8\uc5b8 \uc791\uc804(Operation Paul Bunyan: \ubbf8\uad6d \uc804\uc124\uc5d0 \ub4f1\uc7a5\ud558\ub294 \uac70\uad6c\uc758 \ub098\ubb34\uafbc \ud3f4 \ubc84\ub2c8\uc5b8\uc5d0\uc11c \ub530\uc628 \uc791\uc804\uba85)\uc744 \uae30\ubcf8\uc73c\ub85c F-4, F-111, B-52 \ud3ed\uaca9\uae30, \ubbf8\ub4dc\uc6e8\uc774\ud638 \ub4f1\uc744 \ub3d9\uc6d0\ud558\ub294 \ub300\uaddc\ubaa8 \ubb34\ub825 \uc2dc\uc704 \uacc4\ud68d\uc744 \uc218\ub9bd\ud558\uc600\uace0, \uc804\ud22c\uc900\ube44\ud0dc\uc138\uc778 \ub370\ud504\ucf58 3\uc774 \ubc1c\ub839\ub418\uc5c8\ub2e4. ", "paragraph_answer": "\uc0ac\uac74\uc774 \ubc1c\uc0dd\ud558\uc790 \ubbf8\uad6d \ubc31\uc545\uad00\uc5d0\uc11c\ub294 \uc6cc\uc2f1\ud134 \ud2b9\ubcc4 \ub300\ucc45\ubc18\uc774 \uc18c\uc9d1\ub418\uc5c8\uc73c\uba70 \ubbf8\uad6d \uad6d\ubb34\ubd80\uacfc \ud568\uaed8 \"\uc774 \uc0ac\uac74\uc758 \uacb0\uacfc\ub85c \ube5a\uc5b4\uc9c0\ub294 \uc5b4\ub5a0\ud55c \uc0ac\ud0dc\uc5d0 \ub300\ud574\uc11c\ub3c4 \uadf8 \ucc45\uc784\uc740 \uc870\uc120\ubbfc\uc8fc\uc8fc\uc758\uc778\ubbfc\uacf5\ud654\uad6d\uc5d0 \uc788\ub2e4\"\ub294 \uacf5\ub3d9\uc131\uba85\uc744 \ub2f9\uc77c\uc5d0 \ubc1c\ud45c\ud558\uc600\ub2e4. \ub610\ud55c \uc81c\ub7f4\ub4dc \ud3ec\ub4dc \ubbf8\uad6d \ub300\ud1b5\ub839\uc758 \uba85\ub839\uc5d0 \ub530\ub77c \uc2a4\ud2f8\uc6f0 \uc8fc\ud55c\ubbf8\uad70 \uc0ac\ub839\uad00\uc740 \ubb38\uc81c\uc758 \ubbf8\ub8e8\ub098\ubb34\ub97c \ubca0\uace0 \uacf5\ub3d9\uacbd\ube44\uad6c\uc5ed \ub0b4\uc5d0 \uc778\ubbfc\uad70\uc774 \uc124\uce58\ud55c \ubd88\ubc95 \ubc29\ubcbd(\u9632\u58c1; \ubc14\ub9ac\uac8c\uc774\ud2b8 \ub4f1)\uc744 \uc81c\uac70\ud558\uae30 \uc704\ud55c \ud3f4 \ubc84\ub2c8\uc5b8 \uc791\uc804(Operation Paul Bunyan: \ubbf8\uad6d \uc804\uc124\uc5d0 \ub4f1\uc7a5\ud558\ub294 \uac70\uad6c\uc758 \ub098\ubb34\uafbc \ud3f4 \ubc84\ub2c8\uc5b8\uc5d0\uc11c \ub530\uc628 \uc791\uc804\uba85)\uc744 \uae30\ubcf8\uc73c\ub85c F-4, F-111, B-52 \ud3ed\uaca9\uae30, \ubbf8\ub4dc\uc6e8\uc774\ud638 \ub4f1\uc744 \ub3d9\uc6d0\ud558\ub294 \ub300\uaddc\ubaa8 \ubb34\ub825 \uc2dc\uc704 \uacc4\ud68d\uc744 \uc218\ub9bd\ud558\uc600\uace0, \uc804\ud22c\uc900\ube44\ud0dc\uc138\uc778 \ub370\ud504\ucf58 3\uc774 \ubc1c\ub839\ub418\uc5c8\ub2e4.", "sentence_answer": "\ub610\ud55c \uc81c\ub7f4\ub4dc \ud3ec\ub4dc \ubbf8\uad6d \ub300\ud1b5\ub839\uc758 \uba85\ub839\uc5d0 \ub530\ub77c \uc2a4\ud2f8\uc6f0 \uc8fc\ud55c\ubbf8\uad70 \uc0ac\ub839\uad00\uc740 \ubb38\uc81c\uc758 \ubbf8\ub8e8\ub098\ubb34\ub97c \ubca0\uace0 \uacf5\ub3d9\uacbd\ube44\uad6c\uc5ed \ub0b4\uc5d0 \uc778\ubbfc\uad70\uc774 \uc124\uce58\ud55c \ubd88\ubc95 \ubc29\ubcbd(\u9632\u58c1; \ubc14\ub9ac\uac8c\uc774\ud2b8 \ub4f1)\uc744 \uc81c\uac70\ud558\uae30 \uc704\ud55c \ud3f4 \ubc84\ub2c8\uc5b8 \uc791\uc804(Operation Paul Bunyan: \ubbf8\uad6d \uc804\uc124\uc5d0 \ub4f1\uc7a5\ud558\ub294 \uac70\uad6c\uc758 \ub098\ubb34\uafbc \ud3f4 \ubc84\ub2c8\uc5b8\uc5d0\uc11c \ub530\uc628 \uc791\uc804\uba85)\uc744 \uae30\ubcf8\uc73c\ub85c F-4, F-111, B-52 \ud3ed\uaca9\uae30, \ubbf8\ub4dc\uc6e8\uc774\ud638 \ub4f1\uc744 \ub3d9\uc6d0\ud558\ub294 \ub300\uaddc\ubaa8 \ubb34\ub825 \uc2dc\uc704 \uacc4\ud68d\uc744 \uc218\ub9bd\ud558\uc600\uace0, \uc804\ud22c\uc900\ube44\ud0dc\uc138\uc778 \ub370\ud504\ucf58 3\uc774 \ubc1c\ub839\ub418\uc5c8\ub2e4.", "paragraph_id": "6566672-1-1", "paragraph_question": "question: \ud310\ubb38\uc810 \ub3c4\ub07c \uc0b4\uc778 \uc0ac\uac74\uc774 \uc77c\uc5b4\ub09c \ud6c4 \uc2a4\ud2f8\uc6f0 \uc8fc\ud55c\ubbf8\uad70 \uc0ac\ub839\uad00\uc5d0 \ub530\ub77c \uc778\ubbfc\uad70\uc774 \uc124\uce58\ud55c \ubd88\ubc95 \ubc29\ubcbd\uc744 \uc81c\uac70\ud558\ub294 \uc791\uc804\uc774 \ubb34\uc5c7\uc778\uac00?, context: \uc0ac\uac74\uc774 \ubc1c\uc0dd\ud558\uc790 \ubbf8\uad6d \ubc31\uc545\uad00\uc5d0\uc11c\ub294 \uc6cc\uc2f1\ud134 \ud2b9\ubcc4 \ub300\ucc45\ubc18\uc774 \uc18c\uc9d1\ub418\uc5c8\uc73c\uba70 \ubbf8\uad6d \uad6d\ubb34\ubd80\uacfc \ud568\uaed8 \"\uc774 \uc0ac\uac74\uc758 \uacb0\uacfc\ub85c \ube5a\uc5b4\uc9c0\ub294 \uc5b4\ub5a0\ud55c \uc0ac\ud0dc\uc5d0 \ub300\ud574\uc11c\ub3c4 \uadf8 \ucc45\uc784\uc740 \uc870\uc120\ubbfc\uc8fc\uc8fc\uc758\uc778\ubbfc\uacf5\ud654\uad6d\uc5d0 \uc788\ub2e4\"\ub294 \uacf5\ub3d9\uc131\uba85\uc744 \ub2f9\uc77c\uc5d0 \ubc1c\ud45c\ud558\uc600\ub2e4. \ub610\ud55c \uc81c\ub7f4\ub4dc \ud3ec\ub4dc \ubbf8\uad6d \ub300\ud1b5\ub839\uc758 \uba85\ub839\uc5d0 \ub530\ub77c \uc2a4\ud2f8\uc6f0 \uc8fc\ud55c\ubbf8\uad70 \uc0ac\ub839\uad00\uc740 \ubb38\uc81c\uc758 \ubbf8\ub8e8\ub098\ubb34\ub97c \ubca0\uace0 \uacf5\ub3d9\uacbd\ube44\uad6c\uc5ed \ub0b4\uc5d0 \uc778\ubbfc\uad70\uc774 \uc124\uce58\ud55c \ubd88\ubc95 \ubc29\ubcbd(\u9632\u58c1; \ubc14\ub9ac\uac8c\uc774\ud2b8 \ub4f1)\uc744 \uc81c\uac70\ud558\uae30 \uc704\ud55c \ud3f4 \ubc84\ub2c8\uc5b8 \uc791\uc804(Operation Paul Bunyan: \ubbf8\uad6d \uc804\uc124\uc5d0 \ub4f1\uc7a5\ud558\ub294 \uac70\uad6c\uc758 \ub098\ubb34\uafbc \ud3f4 \ubc84\ub2c8\uc5b8\uc5d0\uc11c \ub530\uc628 \uc791\uc804\uba85)\uc744 \uae30\ubcf8\uc73c\ub85c F-4, F-111, B-52 \ud3ed\uaca9\uae30, \ubbf8\ub4dc\uc6e8\uc774\ud638 \ub4f1\uc744 \ub3d9\uc6d0\ud558\ub294 \ub300\uaddc\ubaa8 \ubb34\ub825 \uc2dc\uc704 \uacc4\ud68d\uc744 \uc218\ub9bd\ud558\uc600\uace0, \uc804\ud22c\uc900\ube44\ud0dc\uc138\uc778 \ub370\ud504\ucf58 3\uc774 \ubc1c\ub839\ub418\uc5c8\ub2e4."} {"question": "\ub3c4\ucfe0\uce74\uc640 \ub9c9\ubd80\uac00 \uc1c4\uad6d\uc815\ucc45\uc744 \uc2dc\ud589\ud55c \uacb0\uc815\uc801\uc778 \uc0ac\uac74\uc740?", "paragraph": "\ub3c4\ucfe0\uac00\uc640 \ub9c9\ubd80\uac00 \uc1c4\uad6d\uc744 \ub2e8\ud589\ud55c \uacb0\uc815\uc801\uc778 \uc0ac\uac74\uc740 1637\ub144\uc5d0 \uc77c\uc5b4\ub09c \uc2dc\ub9c8\ubc14\ub77c\uc758 \ub09c\uc774\ub2e4. \uc774 \ub09c\uc5d0 \uc758\ud574 \uae30\ub3c5\uad50\ub294 \ub3c4\ucfe0\uac00\uc640 \ub9c9\ubd80\ub97c \ud754\ub4e4 \uc6d0\ud749\uc73c\ub85c \uac04\uc8fc\ub418\uc5c8\uace0, \uc0c8\ub85c\uc6b4 \ud3ec\uad50 \ud65c\ub3d9\uc774 \ud5a5\ud6c4 \uc77c\uc808 \ud589\ud574\uc9c8 \uc218 \uc5c6\ub3c4\ub85d \uc774\ubca0\ub9ac\uc544 \ubc18\ub3c4 \uc138\ub825\uc744 \ubc30\uc81c\ud588\ub2e4. \ud3ec\ub974\ud22c\uac08\uc740 1636\ub144 \uc774\ud6c4 \ub370\uc9c0\ub9c8\uc5d0\uc11c\ub9cc \uad50\uc5ed\uc774 \ud5c8\uc6a9\ub418\uc5c8\uc9c0\ub9cc, 1639\ub144\uc5d0\ub294 \ud3ec\ub974\ud22c\uac08\uc774 \uc544\uc608 \ucd94\ubc29\ub418\uba74\uc11c \ub370\uc9c0\ub9c8\ub294 \uacf5\ud130\uac00 \ub418\uc5b4 \uc788\uc5c8\ub2e4. 1641\ub144 \ud788\ub77c\ub3c4\uc758 \ub124\ub35c\ub780\ub4dc \uc0c1\uad00 \ucc3d\uace0\uc5d0 \u2018\uc11c\uae30\u2019\uac00 \uc0c8\uaca8\uc838 \uc788\ub2e4\ub294 \uc0ac\uc18c\ud55c \uc774\uc720\ub85c, \ub124\ub35c\ub780\ub4dc\ub294 \ucc3d\uace0\ub97c \ud30c\uac01\ud558\uace0 \ud788\ub77c\ub3c4\uc5d0\uc11c \ub370\uc9c0\ub9c8\ub85c \uc62e\uae38 \uac83\uc744 \uac15\uc694\ub2f9\ud588\ub2e4.(\ud3ec\ub974\ud22c\uac08\uc740 \ub370\uc9c0\ub9c8 \uc0ac\uc6a9\ub8cc\ub97c \uc5f0\uac04 \uc740 80\uad00\uc744 \uc8fc\uace0 \uc788\uc5c8\uc9c0\ub9cc, \ub124\ub35c\ub780\ub4dc\ub294 55\uad00\ub9cc \uc9c0\ubd88\ud558\uac8c \ud588\ub2e4.) \ub610\ud55c \ub3c4\ucfe0\uac00\uc640 \ub9c9\ubd80\uc5d0 \ud3ec\uad50\ub97c \uc77c\uc808\ud558\uc9c0 \uc54a\uaca0\ub2e4\uace0 \uc57d\uc18d\ud588\ub2e4.", "answer": "\uc2dc\ub9c8\ubc14\ub77c\uc758 \ub09c", "sentence": "\ub3c4\ucfe0\uac00\uc640 \ub9c9\ubd80\uac00 \uc1c4\uad6d\uc744 \ub2e8\ud589\ud55c \uacb0\uc815\uc801\uc778 \uc0ac\uac74\uc740 1637\ub144\uc5d0 \uc77c\uc5b4\ub09c \uc2dc\ub9c8\ubc14\ub77c\uc758 \ub09c \uc774\ub2e4.", "paragraph_sentence": " \ub3c4\ucfe0\uac00\uc640 \ub9c9\ubd80\uac00 \uc1c4\uad6d\uc744 \ub2e8\ud589\ud55c \uacb0\uc815\uc801\uc778 \uc0ac\uac74\uc740 1637\ub144\uc5d0 \uc77c\uc5b4\ub09c \uc2dc\ub9c8\ubc14\ub77c\uc758 \ub09c \uc774\ub2e4. \uc774 \ub09c\uc5d0 \uc758\ud574 \uae30\ub3c5\uad50\ub294 \ub3c4\ucfe0\uac00\uc640 \ub9c9\ubd80\ub97c \ud754\ub4e4 \uc6d0\ud749\uc73c\ub85c \uac04\uc8fc\ub418\uc5c8\uace0, \uc0c8\ub85c\uc6b4 \ud3ec\uad50 \ud65c\ub3d9\uc774 \ud5a5\ud6c4 \uc77c\uc808 \ud589\ud574\uc9c8 \uc218 \uc5c6\ub3c4\ub85d \uc774\ubca0\ub9ac\uc544 \ubc18\ub3c4 \uc138\ub825\uc744 \ubc30\uc81c\ud588\ub2e4. \ud3ec\ub974\ud22c\uac08\uc740 1636\ub144 \uc774\ud6c4 \ub370\uc9c0\ub9c8\uc5d0\uc11c\ub9cc \uad50\uc5ed\uc774 \ud5c8\uc6a9\ub418\uc5c8\uc9c0\ub9cc, 1639\ub144\uc5d0\ub294 \ud3ec\ub974\ud22c\uac08\uc774 \uc544\uc608 \ucd94\ubc29\ub418\uba74\uc11c \ub370\uc9c0\ub9c8\ub294 \uacf5\ud130\uac00 \ub418\uc5b4 \uc788\uc5c8\ub2e4. 1641\ub144 \ud788\ub77c\ub3c4\uc758 \ub124\ub35c\ub780\ub4dc \uc0c1\uad00 \ucc3d\uace0\uc5d0 \u2018\uc11c\uae30\u2019\uac00 \uc0c8\uaca8\uc838 \uc788\ub2e4\ub294 \uc0ac\uc18c\ud55c \uc774\uc720\ub85c, \ub124\ub35c\ub780\ub4dc\ub294 \ucc3d\uace0\ub97c \ud30c\uac01\ud558\uace0 \ud788\ub77c\ub3c4\uc5d0\uc11c \ub370\uc9c0\ub9c8\ub85c \uc62e\uae38 \uac83\uc744 \uac15\uc694\ub2f9\ud588\ub2e4.(\ud3ec\ub974\ud22c\uac08\uc740 \ub370\uc9c0\ub9c8 \uc0ac\uc6a9\ub8cc\ub97c \uc5f0\uac04 \uc740 80\uad00\uc744 \uc8fc\uace0 \uc788\uc5c8\uc9c0\ub9cc, \ub124\ub35c\ub780\ub4dc\ub294 55\uad00\ub9cc \uc9c0\ubd88\ud558\uac8c \ud588\ub2e4. ) \ub610\ud55c \ub3c4\ucfe0\uac00\uc640 \ub9c9\ubd80\uc5d0 \ud3ec\uad50\ub97c \uc77c\uc808\ud558\uc9c0 \uc54a\uaca0\ub2e4\uace0 \uc57d\uc18d\ud588\ub2e4.", "paragraph_answer": "\ub3c4\ucfe0\uac00\uc640 \ub9c9\ubd80\uac00 \uc1c4\uad6d\uc744 \ub2e8\ud589\ud55c \uacb0\uc815\uc801\uc778 \uc0ac\uac74\uc740 1637\ub144\uc5d0 \uc77c\uc5b4\ub09c \uc2dc\ub9c8\ubc14\ub77c\uc758 \ub09c \uc774\ub2e4. \uc774 \ub09c\uc5d0 \uc758\ud574 \uae30\ub3c5\uad50\ub294 \ub3c4\ucfe0\uac00\uc640 \ub9c9\ubd80\ub97c \ud754\ub4e4 \uc6d0\ud749\uc73c\ub85c \uac04\uc8fc\ub418\uc5c8\uace0, \uc0c8\ub85c\uc6b4 \ud3ec\uad50 \ud65c\ub3d9\uc774 \ud5a5\ud6c4 \uc77c\uc808 \ud589\ud574\uc9c8 \uc218 \uc5c6\ub3c4\ub85d \uc774\ubca0\ub9ac\uc544 \ubc18\ub3c4 \uc138\ub825\uc744 \ubc30\uc81c\ud588\ub2e4. \ud3ec\ub974\ud22c\uac08\uc740 1636\ub144 \uc774\ud6c4 \ub370\uc9c0\ub9c8\uc5d0\uc11c\ub9cc \uad50\uc5ed\uc774 \ud5c8\uc6a9\ub418\uc5c8\uc9c0\ub9cc, 1639\ub144\uc5d0\ub294 \ud3ec\ub974\ud22c\uac08\uc774 \uc544\uc608 \ucd94\ubc29\ub418\uba74\uc11c \ub370\uc9c0\ub9c8\ub294 \uacf5\ud130\uac00 \ub418\uc5b4 \uc788\uc5c8\ub2e4. 1641\ub144 \ud788\ub77c\ub3c4\uc758 \ub124\ub35c\ub780\ub4dc \uc0c1\uad00 \ucc3d\uace0\uc5d0 \u2018\uc11c\uae30\u2019\uac00 \uc0c8\uaca8\uc838 \uc788\ub2e4\ub294 \uc0ac\uc18c\ud55c \uc774\uc720\ub85c, \ub124\ub35c\ub780\ub4dc\ub294 \ucc3d\uace0\ub97c \ud30c\uac01\ud558\uace0 \ud788\ub77c\ub3c4\uc5d0\uc11c \ub370\uc9c0\ub9c8\ub85c \uc62e\uae38 \uac83\uc744 \uac15\uc694\ub2f9\ud588\ub2e4.(\ud3ec\ub974\ud22c\uac08\uc740 \ub370\uc9c0\ub9c8 \uc0ac\uc6a9\ub8cc\ub97c \uc5f0\uac04 \uc740 80\uad00\uc744 \uc8fc\uace0 \uc788\uc5c8\uc9c0\ub9cc, \ub124\ub35c\ub780\ub4dc\ub294 55\uad00\ub9cc \uc9c0\ubd88\ud558\uac8c \ud588\ub2e4.) \ub610\ud55c \ub3c4\ucfe0\uac00\uc640 \ub9c9\ubd80\uc5d0 \ud3ec\uad50\ub97c \uc77c\uc808\ud558\uc9c0 \uc54a\uaca0\ub2e4\uace0 \uc57d\uc18d\ud588\ub2e4.", "sentence_answer": "\ub3c4\ucfe0\uac00\uc640 \ub9c9\ubd80\uac00 \uc1c4\uad6d\uc744 \ub2e8\ud589\ud55c \uacb0\uc815\uc801\uc778 \uc0ac\uac74\uc740 1637\ub144\uc5d0 \uc77c\uc5b4\ub09c \uc2dc\ub9c8\ubc14\ub77c\uc758 \ub09c \uc774\ub2e4.", "paragraph_id": "6532465-3-0", "paragraph_question": "question: \ub3c4\ucfe0\uce74\uc640 \ub9c9\ubd80\uac00 \uc1c4\uad6d\uc815\ucc45\uc744 \uc2dc\ud589\ud55c \uacb0\uc815\uc801\uc778 \uc0ac\uac74\uc740?, context: \ub3c4\ucfe0\uac00\uc640 \ub9c9\ubd80\uac00 \uc1c4\uad6d\uc744 \ub2e8\ud589\ud55c \uacb0\uc815\uc801\uc778 \uc0ac\uac74\uc740 1637\ub144\uc5d0 \uc77c\uc5b4\ub09c \uc2dc\ub9c8\ubc14\ub77c\uc758 \ub09c\uc774\ub2e4. \uc774 \ub09c\uc5d0 \uc758\ud574 \uae30\ub3c5\uad50\ub294 \ub3c4\ucfe0\uac00\uc640 \ub9c9\ubd80\ub97c \ud754\ub4e4 \uc6d0\ud749\uc73c\ub85c \uac04\uc8fc\ub418\uc5c8\uace0, \uc0c8\ub85c\uc6b4 \ud3ec\uad50 \ud65c\ub3d9\uc774 \ud5a5\ud6c4 \uc77c\uc808 \ud589\ud574\uc9c8 \uc218 \uc5c6\ub3c4\ub85d \uc774\ubca0\ub9ac\uc544 \ubc18\ub3c4 \uc138\ub825\uc744 \ubc30\uc81c\ud588\ub2e4. \ud3ec\ub974\ud22c\uac08\uc740 1636\ub144 \uc774\ud6c4 \ub370\uc9c0\ub9c8\uc5d0\uc11c\ub9cc \uad50\uc5ed\uc774 \ud5c8\uc6a9\ub418\uc5c8\uc9c0\ub9cc, 1639\ub144\uc5d0\ub294 \ud3ec\ub974\ud22c\uac08\uc774 \uc544\uc608 \ucd94\ubc29\ub418\uba74\uc11c \ub370\uc9c0\ub9c8\ub294 \uacf5\ud130\uac00 \ub418\uc5b4 \uc788\uc5c8\ub2e4. 1641\ub144 \ud788\ub77c\ub3c4\uc758 \ub124\ub35c\ub780\ub4dc \uc0c1\uad00 \ucc3d\uace0\uc5d0 \u2018\uc11c\uae30\u2019\uac00 \uc0c8\uaca8\uc838 \uc788\ub2e4\ub294 \uc0ac\uc18c\ud55c \uc774\uc720\ub85c, \ub124\ub35c\ub780\ub4dc\ub294 \ucc3d\uace0\ub97c \ud30c\uac01\ud558\uace0 \ud788\ub77c\ub3c4\uc5d0\uc11c \ub370\uc9c0\ub9c8\ub85c \uc62e\uae38 \uac83\uc744 \uac15\uc694\ub2f9\ud588\ub2e4.(\ud3ec\ub974\ud22c\uac08\uc740 \ub370\uc9c0\ub9c8 \uc0ac\uc6a9\ub8cc\ub97c \uc5f0\uac04 \uc740 80\uad00\uc744 \uc8fc\uace0 \uc788\uc5c8\uc9c0\ub9cc, \ub124\ub35c\ub780\ub4dc\ub294 55\uad00\ub9cc \uc9c0\ubd88\ud558\uac8c \ud588\ub2e4.) \ub610\ud55c \ub3c4\ucfe0\uac00\uc640 \ub9c9\ubd80\uc5d0 \ud3ec\uad50\ub97c \uc77c\uc808\ud558\uc9c0 \uc54a\uaca0\ub2e4\uace0 \uc57d\uc18d\ud588\ub2e4."} {"question": "\ub291\ub300 \ubcf4\ud638 \uc870\uce58\uc640 \ud6a8\uacfc\uc801 \ubc95 \uc9d1\ud589\uc73c\ub85c \uc801\ub2f9\ud55c \ub291\ub300 \uac1c\uccb4\uc218\ub97c \uc720\uc9c0\ud558\uace0 \uc788\ub294 \uacf3\uc740 \uc5b4\ub514\uc778\uac00?", "paragraph": "\ub291\ub300 \uc0c1\ud0dc\uc5d0 \ub300\ud55c \uc2e0\ub8b0\ud560 \uc218 \uc788\ub294 \ub370\uc774\ud130\ub294 \uc911\ub3d9\uc758 \uc774\uc2a4\ub77c\uc5d8\uacfc \uc0ac\uc6b0\ub514\uc544\ub77c\ube44\uc544\ub85c \uc548\uc815\uc801\uc778 \uac1c\uccb4\uc218\ub97c \uc720\uc9c0\ud558\uace0 \uc788\ub2e4. \uc774\uc2a4\ub77c\uc5d8\uc740 \ub291\ub300 \ubcf4\ud638 \uc870\uce58\uc640 \ud6a8\uacfc\uc801 \ubc95 \uc9d1\ud589\uc73c\ub85c \uc801\ub2f9\ud55c \ub291\ub300 \uac1c\uccb4\uc218\ub97c \uc720\uc9c0\ud558\uace0 \uc788\uace0 \uc774\uc6c3 \uad6d\uac00\ub85c \ubc29\ucd9c\ud558\uace0 \uc788\uc73c\uba70, \uc0ac\uc6b0\ub514\uc544\ub77c\ube44\uc544\ub294 3\ubc31-6\ubc31 \ub9c8\ub9ac\uac00 \ubc29\ud574\ubc1b\uc9c0 \uc54a\uc740 \uad11\ub300\ud55c \uc0ac\ub9c9 \uc9c0\uc5ed\uc5d0\uc11c \uc11c\uc2dd\ud558\uace0 \uc788\ub2e4. \uc0ac\uc6b0\ub514\uc544\ub77c\ube44\uc544\uc758 \ub291\ub300\ub294 \uc720\ubaa9\uc758 \uac10\uc18c\uc640 \uc778\uac04 \ud3d0\uae30\ubb3c\uc758 \uc99d\uac00\ub85c \uc5ed\uc0ac\uc801 \ubc94\uc704 \ub0b4\uc5d0\uc11c \uacc4\uc18d \uc11c\uc2dd\ud558\uace0 \uc788\ub2e4. \ud130\ud0a4\uc758 \ub291\ub300\ub294 \uc911\uc559\uc544\uc2dc\uc544\uc5d0\uc11c \uc720\uc785\ub418\ub294 \uac1c\uccb4\ub85c \uc778\ud574 \ub291\ub300\uac00 \uc720\uc9c0\ub418\uace0 \uc788\ub2e4. \ud130\ud0a4 \uc0b0\ub9e5 \uc9c0\uc5ed\uc5d0 \uc0b0 \uba87\uba87 \ub291\ub300\ub4e4\uc740 \uc2dc\ub9ac\uc544\ub85c \uc62e\uaca8\uac00\uae30\ub3c4 \ud55c\ub2e4. \uc791\uc740 \ub291\ub300 \uac1c\uccb4\uac00 \uace8\ub780 \uace0\uc6d0\uc5d0 \uc11c\uc2dd\ud558\uace0 \uc788\uc73c\uba70 \uad70\uc0ac \ud65c\ub3d9\uc73c\ub85c \uc778\ud574 \ubcf4\ud638\ubc1b\uace0 \uc788\ub2e4. \ub124\uac8c\ube0c \uc0ac\ub9c9 \ub0a8\ubd80\uc5d0 \uc11c\uc2dd\ud558\ub294 \ub291\ub300\ub294 \uc774\uc9d1\ud2b8\uc758 \uc2dc\ub098\uc774 \ubc18\ub3c4\ub098 \uc694\ub974\ub2e8\uc5d0\uc11c \uc720\uc785\ub418\ub294 \uac1c\uccb4\ub85c \uc720\uc9c0\ub418\uace0 \uc788\ub2e4. \uc911\ub3d9\uc744 \ud1b5\ud2c0\uc5b4\uc11c \ub291\ub300\ub97c \ubcf4\ud638\ud558\ub294 \uad6d\uac00\ub294 \uc774\uc2a4\ub77c\uc5d8\ubfd0\uc774\ub2e4. \ub2e4\ub978 \uacf3\uc5d0\uc11c\ub294 \ubca0\ub450\uc778\uc871\uc774 \uc5f0\uc911 \ub0b4\ub0b4 \uc0ac\ub0e5\ud560 \uc218 \uc788\ub2e4.", "answer": "\uc774\uc2a4\ub77c\uc5d8", "sentence": "\ub291\ub300 \uc0c1\ud0dc\uc5d0 \ub300\ud55c \uc2e0\ub8b0\ud560 \uc218 \uc788\ub294 \ub370\uc774\ud130\ub294 \uc911\ub3d9\uc758 \uc774\uc2a4\ub77c\uc5d8 \uacfc \uc0ac\uc6b0\ub514\uc544\ub77c\ube44\uc544\ub85c \uc548\uc815\uc801\uc778 \uac1c\uccb4\uc218\ub97c \uc720\uc9c0\ud558\uace0 \uc788\ub2e4.", "paragraph_sentence": " \ub291\ub300 \uc0c1\ud0dc\uc5d0 \ub300\ud55c \uc2e0\ub8b0\ud560 \uc218 \uc788\ub294 \ub370\uc774\ud130\ub294 \uc911\ub3d9\uc758 \uc774\uc2a4\ub77c\uc5d8 \uacfc \uc0ac\uc6b0\ub514\uc544\ub77c\ube44\uc544\ub85c \uc548\uc815\uc801\uc778 \uac1c\uccb4\uc218\ub97c \uc720\uc9c0\ud558\uace0 \uc788\ub2e4. \uc774\uc2a4\ub77c\uc5d8\uc740 \ub291\ub300 \ubcf4\ud638 \uc870\uce58\uc640 \ud6a8\uacfc\uc801 \ubc95 \uc9d1\ud589\uc73c\ub85c \uc801\ub2f9\ud55c \ub291\ub300 \uac1c\uccb4\uc218\ub97c \uc720\uc9c0\ud558\uace0 \uc788\uace0 \uc774\uc6c3 \uad6d\uac00\ub85c \ubc29\ucd9c\ud558\uace0 \uc788\uc73c\uba70, \uc0ac\uc6b0\ub514\uc544\ub77c\ube44\uc544\ub294 3\ubc31-6\ubc31 \ub9c8\ub9ac\uac00 \ubc29\ud574\ubc1b\uc9c0 \uc54a\uc740 \uad11\ub300\ud55c \uc0ac\ub9c9 \uc9c0\uc5ed\uc5d0\uc11c \uc11c\uc2dd\ud558\uace0 \uc788\ub2e4. \uc0ac\uc6b0\ub514\uc544\ub77c\ube44\uc544\uc758 \ub291\ub300\ub294 \uc720\ubaa9\uc758 \uac10\uc18c\uc640 \uc778\uac04 \ud3d0\uae30\ubb3c\uc758 \uc99d\uac00\ub85c \uc5ed\uc0ac\uc801 \ubc94\uc704 \ub0b4\uc5d0\uc11c \uacc4\uc18d \uc11c\uc2dd\ud558\uace0 \uc788\ub2e4. \ud130\ud0a4\uc758 \ub291\ub300\ub294 \uc911\uc559\uc544\uc2dc\uc544\uc5d0\uc11c \uc720\uc785\ub418\ub294 \uac1c\uccb4\ub85c \uc778\ud574 \ub291\ub300\uac00 \uc720\uc9c0\ub418\uace0 \uc788\ub2e4. \ud130\ud0a4 \uc0b0\ub9e5 \uc9c0\uc5ed\uc5d0 \uc0b0 \uba87\uba87 \ub291\ub300\ub4e4\uc740 \uc2dc\ub9ac\uc544\ub85c \uc62e\uaca8\uac00\uae30\ub3c4 \ud55c\ub2e4. \uc791\uc740 \ub291\ub300 \uac1c\uccb4\uac00 \uace8\ub780 \uace0\uc6d0\uc5d0 \uc11c\uc2dd\ud558\uace0 \uc788\uc73c\uba70 \uad70\uc0ac \ud65c\ub3d9\uc73c\ub85c \uc778\ud574 \ubcf4\ud638\ubc1b\uace0 \uc788\ub2e4. \ub124\uac8c\ube0c \uc0ac\ub9c9 \ub0a8\ubd80\uc5d0 \uc11c\uc2dd\ud558\ub294 \ub291\ub300\ub294 \uc774\uc9d1\ud2b8\uc758 \uc2dc\ub098\uc774 \ubc18\ub3c4\ub098 \uc694\ub974\ub2e8\uc5d0\uc11c \uc720\uc785\ub418\ub294 \uac1c\uccb4\ub85c \uc720\uc9c0\ub418\uace0 \uc788\ub2e4. \uc911\ub3d9\uc744 \ud1b5\ud2c0\uc5b4\uc11c \ub291\ub300\ub97c \ubcf4\ud638\ud558\ub294 \uad6d\uac00\ub294 \uc774\uc2a4\ub77c\uc5d8\ubfd0\uc774\ub2e4. \ub2e4\ub978 \uacf3\uc5d0\uc11c\ub294 \ubca0\ub450\uc778\uc871\uc774 \uc5f0\uc911 \ub0b4\ub0b4 \uc0ac\ub0e5\ud560 \uc218 \uc788\ub2e4.", "paragraph_answer": "\ub291\ub300 \uc0c1\ud0dc\uc5d0 \ub300\ud55c \uc2e0\ub8b0\ud560 \uc218 \uc788\ub294 \ub370\uc774\ud130\ub294 \uc911\ub3d9\uc758 \uc774\uc2a4\ub77c\uc5d8 \uacfc \uc0ac\uc6b0\ub514\uc544\ub77c\ube44\uc544\ub85c \uc548\uc815\uc801\uc778 \uac1c\uccb4\uc218\ub97c \uc720\uc9c0\ud558\uace0 \uc788\ub2e4. \uc774\uc2a4\ub77c\uc5d8\uc740 \ub291\ub300 \ubcf4\ud638 \uc870\uce58\uc640 \ud6a8\uacfc\uc801 \ubc95 \uc9d1\ud589\uc73c\ub85c \uc801\ub2f9\ud55c \ub291\ub300 \uac1c\uccb4\uc218\ub97c \uc720\uc9c0\ud558\uace0 \uc788\uace0 \uc774\uc6c3 \uad6d\uac00\ub85c \ubc29\ucd9c\ud558\uace0 \uc788\uc73c\uba70, \uc0ac\uc6b0\ub514\uc544\ub77c\ube44\uc544\ub294 3\ubc31-6\ubc31 \ub9c8\ub9ac\uac00 \ubc29\ud574\ubc1b\uc9c0 \uc54a\uc740 \uad11\ub300\ud55c \uc0ac\ub9c9 \uc9c0\uc5ed\uc5d0\uc11c \uc11c\uc2dd\ud558\uace0 \uc788\ub2e4. \uc0ac\uc6b0\ub514\uc544\ub77c\ube44\uc544\uc758 \ub291\ub300\ub294 \uc720\ubaa9\uc758 \uac10\uc18c\uc640 \uc778\uac04 \ud3d0\uae30\ubb3c\uc758 \uc99d\uac00\ub85c \uc5ed\uc0ac\uc801 \ubc94\uc704 \ub0b4\uc5d0\uc11c \uacc4\uc18d \uc11c\uc2dd\ud558\uace0 \uc788\ub2e4. \ud130\ud0a4\uc758 \ub291\ub300\ub294 \uc911\uc559\uc544\uc2dc\uc544\uc5d0\uc11c \uc720\uc785\ub418\ub294 \uac1c\uccb4\ub85c \uc778\ud574 \ub291\ub300\uac00 \uc720\uc9c0\ub418\uace0 \uc788\ub2e4. \ud130\ud0a4 \uc0b0\ub9e5 \uc9c0\uc5ed\uc5d0 \uc0b0 \uba87\uba87 \ub291\ub300\ub4e4\uc740 \uc2dc\ub9ac\uc544\ub85c \uc62e\uaca8\uac00\uae30\ub3c4 \ud55c\ub2e4. \uc791\uc740 \ub291\ub300 \uac1c\uccb4\uac00 \uace8\ub780 \uace0\uc6d0\uc5d0 \uc11c\uc2dd\ud558\uace0 \uc788\uc73c\uba70 \uad70\uc0ac \ud65c\ub3d9\uc73c\ub85c \uc778\ud574 \ubcf4\ud638\ubc1b\uace0 \uc788\ub2e4. \ub124\uac8c\ube0c \uc0ac\ub9c9 \ub0a8\ubd80\uc5d0 \uc11c\uc2dd\ud558\ub294 \ub291\ub300\ub294 \uc774\uc9d1\ud2b8\uc758 \uc2dc\ub098\uc774 \ubc18\ub3c4\ub098 \uc694\ub974\ub2e8\uc5d0\uc11c \uc720\uc785\ub418\ub294 \uac1c\uccb4\ub85c \uc720\uc9c0\ub418\uace0 \uc788\ub2e4. \uc911\ub3d9\uc744 \ud1b5\ud2c0\uc5b4\uc11c \ub291\ub300\ub97c \ubcf4\ud638\ud558\ub294 \uad6d\uac00\ub294 \uc774\uc2a4\ub77c\uc5d8\ubfd0\uc774\ub2e4. \ub2e4\ub978 \uacf3\uc5d0\uc11c\ub294 \ubca0\ub450\uc778\uc871\uc774 \uc5f0\uc911 \ub0b4\ub0b4 \uc0ac\ub0e5\ud560 \uc218 \uc788\ub2e4.", "sentence_answer": "\ub291\ub300 \uc0c1\ud0dc\uc5d0 \ub300\ud55c \uc2e0\ub8b0\ud560 \uc218 \uc788\ub294 \ub370\uc774\ud130\ub294 \uc911\ub3d9\uc758 \uc774\uc2a4\ub77c\uc5d8 \uacfc \uc0ac\uc6b0\ub514\uc544\ub77c\ube44\uc544\ub85c \uc548\uc815\uc801\uc778 \uac1c\uccb4\uc218\ub97c \uc720\uc9c0\ud558\uace0 \uc788\ub2e4.", "paragraph_id": "6572454-36-0", "paragraph_question": "question: \ub291\ub300 \ubcf4\ud638 \uc870\uce58\uc640 \ud6a8\uacfc\uc801 \ubc95 \uc9d1\ud589\uc73c\ub85c \uc801\ub2f9\ud55c \ub291\ub300 \uac1c\uccb4\uc218\ub97c \uc720\uc9c0\ud558\uace0 \uc788\ub294 \uacf3\uc740 \uc5b4\ub514\uc778\uac00?, context: \ub291\ub300 \uc0c1\ud0dc\uc5d0 \ub300\ud55c \uc2e0\ub8b0\ud560 \uc218 \uc788\ub294 \ub370\uc774\ud130\ub294 \uc911\ub3d9\uc758 \uc774\uc2a4\ub77c\uc5d8\uacfc \uc0ac\uc6b0\ub514\uc544\ub77c\ube44\uc544\ub85c \uc548\uc815\uc801\uc778 \uac1c\uccb4\uc218\ub97c \uc720\uc9c0\ud558\uace0 \uc788\ub2e4. \uc774\uc2a4\ub77c\uc5d8\uc740 \ub291\ub300 \ubcf4\ud638 \uc870\uce58\uc640 \ud6a8\uacfc\uc801 \ubc95 \uc9d1\ud589\uc73c\ub85c \uc801\ub2f9\ud55c \ub291\ub300 \uac1c\uccb4\uc218\ub97c \uc720\uc9c0\ud558\uace0 \uc788\uace0 \uc774\uc6c3 \uad6d\uac00\ub85c \ubc29\ucd9c\ud558\uace0 \uc788\uc73c\uba70, \uc0ac\uc6b0\ub514\uc544\ub77c\ube44\uc544\ub294 3\ubc31-6\ubc31 \ub9c8\ub9ac\uac00 \ubc29\ud574\ubc1b\uc9c0 \uc54a\uc740 \uad11\ub300\ud55c \uc0ac\ub9c9 \uc9c0\uc5ed\uc5d0\uc11c \uc11c\uc2dd\ud558\uace0 \uc788\ub2e4. \uc0ac\uc6b0\ub514\uc544\ub77c\ube44\uc544\uc758 \ub291\ub300\ub294 \uc720\ubaa9\uc758 \uac10\uc18c\uc640 \uc778\uac04 \ud3d0\uae30\ubb3c\uc758 \uc99d\uac00\ub85c \uc5ed\uc0ac\uc801 \ubc94\uc704 \ub0b4\uc5d0\uc11c \uacc4\uc18d \uc11c\uc2dd\ud558\uace0 \uc788\ub2e4. \ud130\ud0a4\uc758 \ub291\ub300\ub294 \uc911\uc559\uc544\uc2dc\uc544\uc5d0\uc11c \uc720\uc785\ub418\ub294 \uac1c\uccb4\ub85c \uc778\ud574 \ub291\ub300\uac00 \uc720\uc9c0\ub418\uace0 \uc788\ub2e4. \ud130\ud0a4 \uc0b0\ub9e5 \uc9c0\uc5ed\uc5d0 \uc0b0 \uba87\uba87 \ub291\ub300\ub4e4\uc740 \uc2dc\ub9ac\uc544\ub85c \uc62e\uaca8\uac00\uae30\ub3c4 \ud55c\ub2e4. \uc791\uc740 \ub291\ub300 \uac1c\uccb4\uac00 \uace8\ub780 \uace0\uc6d0\uc5d0 \uc11c\uc2dd\ud558\uace0 \uc788\uc73c\uba70 \uad70\uc0ac \ud65c\ub3d9\uc73c\ub85c \uc778\ud574 \ubcf4\ud638\ubc1b\uace0 \uc788\ub2e4. \ub124\uac8c\ube0c \uc0ac\ub9c9 \ub0a8\ubd80\uc5d0 \uc11c\uc2dd\ud558\ub294 \ub291\ub300\ub294 \uc774\uc9d1\ud2b8\uc758 \uc2dc\ub098\uc774 \ubc18\ub3c4\ub098 \uc694\ub974\ub2e8\uc5d0\uc11c \uc720\uc785\ub418\ub294 \uac1c\uccb4\ub85c \uc720\uc9c0\ub418\uace0 \uc788\ub2e4. \uc911\ub3d9\uc744 \ud1b5\ud2c0\uc5b4\uc11c \ub291\ub300\ub97c \ubcf4\ud638\ud558\ub294 \uad6d\uac00\ub294 \uc774\uc2a4\ub77c\uc5d8\ubfd0\uc774\ub2e4. \ub2e4\ub978 \uacf3\uc5d0\uc11c\ub294 \ubca0\ub450\uc778\uc871\uc774 \uc5f0\uc911 \ub0b4\ub0b4 \uc0ac\ub0e5\ud560 \uc218 \uc788\ub2e4."} {"question": "\uc704\ubbf8\ub974\uc758 \uc774\ube68\ub85c \uc790\uac08\uacfc \ub3cc\uba69\uc774\ub97c \ub9cc\ub4e0 \uc774\ub4e4\uc740?", "paragraph": "\uac15\uae00\ub808\ub9ac\ub294 \ub192\uc73c\uc2e0 \ubd84\uacfc \uadf8\ub9cc\ud07c \ub192\uc73c\uc2e0 \ubd84\uacfc \uc138\ubc88\uc9f8 \ubd84\uc774 \uc624\ub518 3\ud615\uc81c\ub97c \uc2e0\uc73c\ub85c \uc0dd\uac01\ud55c\ub2e4\uba74, \uadf8 \uc14b\uc774 \ud55c \uc77c\uc774 \ubb50\uac00 \uc788\ub0d0\uace0 \ubb3b\ub294\ub2e4. \ub192\uc73c\uc2e0 \ubd84\uc740 \uc624\ub518 3\ud615\uc81c\uac00 \uc704\ubbf8\ub974\uc758 \uc2dc\uccb4\ub97c \uae34\ub219\uac00\uac00\ud504 \ud55c\uac00\uc6b4\ub370\ub85c \uac00\uc838\uac00 \uc704\ubbf8\ub974\uc758 \uc0b4\ub85c \ub300\uc9c0\ub97c \uc0bc\uace0, \ud53c\ub85c \ubc14\ub2e4\uc640 \ud638\uc218\ub97c \ub9cc\ub4e4\uace0, \ubf08\ub85c \ubc14\uc704\ub97c \ub9cc\ub4e4\uace0, \uc774\ube68\ub85c \uc790\uac08\uacfc \ub3cc\uba69\uc774\ub97c \ub9cc\ub4e4\uc5c8\ub2e4\uace0 \ub9d0\ud55c\ub2e4. \uadf8\ub9cc\ud07c \ub192\uc73c\uc2e0 \ubd84\uc774 \ub9d0\uc744 \ubc1b\uc544 \uc704\ubbf8\ub974\uc758 \uc0c1\ucc98\uc5d0\uc11c \uc3df\uc544\uc9c4 \ud53c\uac00 \ub300\uc9c0\ub97c \ub458\ub7ec\uc2f8\ub294 \ubc14\ub2e4\ub97c \ub9cc\ub4e4\uc5c8\ub2e4\uace0 \ud55c\ub2e4. \uadf8\ub9ac\uace0 \uc138 \ubc88\uc9f8 \ubd84\uc774 3\ud615\uc81c\uac00 \uc704\ubbf8\ub974\uc758 \ub450\uac1c\uace8\uc744 \ub545 \uc704\ub85c \ub4e4\uc5b4\uc62c\ub824 \ud558\ub298\uc744 \uc0bc\uc558\uc73c\uba70, \ub178\ub974\ub4dc\ub9ac, \uc218\ub4dc\ub9ac, \uc544\uc6b0\uc2a4\ud2b8\ub9ac, \ubca0\uc2a4\ud2b8\ub9ac\ub77c\ub294 \ub4dc\ubca0\ub974\uadf8 \ub137\uc744 \ub3d9\uc11c\ub0a8\ubd81 \ub124 \ubc29\ud5a5\uc5d0\uc11c \ud558\ub298\uc744 \ubc1b\uce58\uace0 \uc788\uac8c \ud588\ub2e4\uace0 \ub9d0\ud55c\ub2e4. 3\ud615\uc81c\ub294 \ubb34\uc2a4\ud3a0\uc2a4\ud5e4\uc784\uc5d0\uc11c \ub0a0\uc544\uc628 \ubd88\ud2f0\uc640 \ub179\uc740 \uc870\uac01\ub4e4\uc744 \uac00\uc838\ub2e4\uac00 \uba87 \uac1c\ub294 \ud558\ub298\uc5d0 \ubc15\uace0 \uba87 \uac1c\ub294 \ud558\ub298 \uc544\ub798\uc5d0\uc11c \uc6c0\uc9c1\uc774\uac8c \ud558\uc600\uc73c\ub2c8 \uc774\uac83\uc774 \uac01\uac01 \ubcc4\uacfc \ud574\uc640 \ub2ec\uc774\ub2e4. \uc774 \uc774\uc57c\uae30\ub97c \ud558\uba74\uc11c \u3008\ubb34\ub140\uc758 \uc608\uc5b8\u3009\uc774 \uc778\uc6a9\ub41c\ub2e4.", "answer": "\uc624\ub518 3\ud615\uc81c", "sentence": "\uac15\uae00\ub808\ub9ac\ub294 \ub192\uc73c\uc2e0 \ubd84\uacfc \uadf8\ub9cc\ud07c \ub192\uc73c\uc2e0 \ubd84\uacfc \uc138\ubc88\uc9f8 \ubd84\uc774 \uc624\ub518 3\ud615\uc81c \ub97c \uc2e0\uc73c\ub85c \uc0dd\uac01\ud55c\ub2e4\uba74, \uadf8 \uc14b\uc774 \ud55c \uc77c\uc774 \ubb50\uac00 \uc788\ub0d0\uace0 \ubb3b\ub294\ub2e4.", "paragraph_sentence": " \uac15\uae00\ub808\ub9ac\ub294 \ub192\uc73c\uc2e0 \ubd84\uacfc \uadf8\ub9cc\ud07c \ub192\uc73c\uc2e0 \ubd84\uacfc \uc138\ubc88\uc9f8 \ubd84\uc774 \uc624\ub518 3\ud615\uc81c \ub97c \uc2e0\uc73c\ub85c \uc0dd\uac01\ud55c\ub2e4\uba74, \uadf8 \uc14b\uc774 \ud55c \uc77c\uc774 \ubb50\uac00 \uc788\ub0d0\uace0 \ubb3b\ub294\ub2e4. \ub192\uc73c\uc2e0 \ubd84\uc740 \uc624\ub518 3\ud615\uc81c\uac00 \uc704\ubbf8\ub974\uc758 \uc2dc\uccb4\ub97c \uae34\ub219\uac00\uac00\ud504 \ud55c\uac00\uc6b4\ub370\ub85c \uac00\uc838\uac00 \uc704\ubbf8\ub974\uc758 \uc0b4\ub85c \ub300\uc9c0\ub97c \uc0bc\uace0, \ud53c\ub85c \ubc14\ub2e4\uc640 \ud638\uc218\ub97c \ub9cc\ub4e4\uace0, \ubf08\ub85c \ubc14\uc704\ub97c \ub9cc\ub4e4\uace0, \uc774\ube68\ub85c \uc790\uac08\uacfc \ub3cc\uba69\uc774\ub97c \ub9cc\ub4e4\uc5c8\ub2e4\uace0 \ub9d0\ud55c\ub2e4. \uadf8\ub9cc\ud07c \ub192\uc73c\uc2e0 \ubd84\uc774 \ub9d0\uc744 \ubc1b\uc544 \uc704\ubbf8\ub974\uc758 \uc0c1\ucc98\uc5d0\uc11c \uc3df\uc544\uc9c4 \ud53c\uac00 \ub300\uc9c0\ub97c \ub458\ub7ec\uc2f8\ub294 \ubc14\ub2e4\ub97c \ub9cc\ub4e4\uc5c8\ub2e4\uace0 \ud55c\ub2e4. \uadf8\ub9ac\uace0 \uc138 \ubc88\uc9f8 \ubd84\uc774 3\ud615\uc81c\uac00 \uc704\ubbf8\ub974\uc758 \ub450\uac1c\uace8\uc744 \ub545 \uc704\ub85c \ub4e4\uc5b4\uc62c\ub824 \ud558\ub298\uc744 \uc0bc\uc558\uc73c\uba70, \ub178\ub974\ub4dc\ub9ac, \uc218\ub4dc\ub9ac, \uc544\uc6b0\uc2a4\ud2b8\ub9ac, \ubca0\uc2a4\ud2b8\ub9ac\ub77c\ub294 \ub4dc\ubca0\ub974\uadf8 \ub137\uc744 \ub3d9\uc11c\ub0a8\ubd81 \ub124 \ubc29\ud5a5\uc5d0\uc11c \ud558\ub298\uc744 \ubc1b\uce58\uace0 \uc788\uac8c \ud588\ub2e4\uace0 \ub9d0\ud55c\ub2e4. 3\ud615\uc81c\ub294 \ubb34\uc2a4\ud3a0\uc2a4\ud5e4\uc784\uc5d0\uc11c \ub0a0\uc544\uc628 \ubd88\ud2f0\uc640 \ub179\uc740 \uc870\uac01\ub4e4\uc744 \uac00\uc838\ub2e4\uac00 \uba87 \uac1c\ub294 \ud558\ub298\uc5d0 \ubc15\uace0 \uba87 \uac1c\ub294 \ud558\ub298 \uc544\ub798\uc5d0\uc11c \uc6c0\uc9c1\uc774\uac8c \ud558\uc600\uc73c\ub2c8 \uc774\uac83\uc774 \uac01\uac01 \ubcc4\uacfc \ud574\uc640 \ub2ec\uc774\ub2e4. \uc774 \uc774\uc57c\uae30\ub97c \ud558\uba74\uc11c \u3008\ubb34\ub140\uc758 \uc608\uc5b8\u3009\uc774 \uc778\uc6a9\ub41c\ub2e4.", "paragraph_answer": "\uac15\uae00\ub808\ub9ac\ub294 \ub192\uc73c\uc2e0 \ubd84\uacfc \uadf8\ub9cc\ud07c \ub192\uc73c\uc2e0 \ubd84\uacfc \uc138\ubc88\uc9f8 \ubd84\uc774 \uc624\ub518 3\ud615\uc81c \ub97c \uc2e0\uc73c\ub85c \uc0dd\uac01\ud55c\ub2e4\uba74, \uadf8 \uc14b\uc774 \ud55c \uc77c\uc774 \ubb50\uac00 \uc788\ub0d0\uace0 \ubb3b\ub294\ub2e4. \ub192\uc73c\uc2e0 \ubd84\uc740 \uc624\ub518 3\ud615\uc81c\uac00 \uc704\ubbf8\ub974\uc758 \uc2dc\uccb4\ub97c \uae34\ub219\uac00\uac00\ud504 \ud55c\uac00\uc6b4\ub370\ub85c \uac00\uc838\uac00 \uc704\ubbf8\ub974\uc758 \uc0b4\ub85c \ub300\uc9c0\ub97c \uc0bc\uace0, \ud53c\ub85c \ubc14\ub2e4\uc640 \ud638\uc218\ub97c \ub9cc\ub4e4\uace0, \ubf08\ub85c \ubc14\uc704\ub97c \ub9cc\ub4e4\uace0, \uc774\ube68\ub85c \uc790\uac08\uacfc \ub3cc\uba69\uc774\ub97c \ub9cc\ub4e4\uc5c8\ub2e4\uace0 \ub9d0\ud55c\ub2e4. \uadf8\ub9cc\ud07c \ub192\uc73c\uc2e0 \ubd84\uc774 \ub9d0\uc744 \ubc1b\uc544 \uc704\ubbf8\ub974\uc758 \uc0c1\ucc98\uc5d0\uc11c \uc3df\uc544\uc9c4 \ud53c\uac00 \ub300\uc9c0\ub97c \ub458\ub7ec\uc2f8\ub294 \ubc14\ub2e4\ub97c \ub9cc\ub4e4\uc5c8\ub2e4\uace0 \ud55c\ub2e4. \uadf8\ub9ac\uace0 \uc138 \ubc88\uc9f8 \ubd84\uc774 3\ud615\uc81c\uac00 \uc704\ubbf8\ub974\uc758 \ub450\uac1c\uace8\uc744 \ub545 \uc704\ub85c \ub4e4\uc5b4\uc62c\ub824 \ud558\ub298\uc744 \uc0bc\uc558\uc73c\uba70, \ub178\ub974\ub4dc\ub9ac, \uc218\ub4dc\ub9ac, \uc544\uc6b0\uc2a4\ud2b8\ub9ac, \ubca0\uc2a4\ud2b8\ub9ac\ub77c\ub294 \ub4dc\ubca0\ub974\uadf8 \ub137\uc744 \ub3d9\uc11c\ub0a8\ubd81 \ub124 \ubc29\ud5a5\uc5d0\uc11c \ud558\ub298\uc744 \ubc1b\uce58\uace0 \uc788\uac8c \ud588\ub2e4\uace0 \ub9d0\ud55c\ub2e4. 3\ud615\uc81c\ub294 \ubb34\uc2a4\ud3a0\uc2a4\ud5e4\uc784\uc5d0\uc11c \ub0a0\uc544\uc628 \ubd88\ud2f0\uc640 \ub179\uc740 \uc870\uac01\ub4e4\uc744 \uac00\uc838\ub2e4\uac00 \uba87 \uac1c\ub294 \ud558\ub298\uc5d0 \ubc15\uace0 \uba87 \uac1c\ub294 \ud558\ub298 \uc544\ub798\uc5d0\uc11c \uc6c0\uc9c1\uc774\uac8c \ud558\uc600\uc73c\ub2c8 \uc774\uac83\uc774 \uac01\uac01 \ubcc4\uacfc \ud574\uc640 \ub2ec\uc774\ub2e4. \uc774 \uc774\uc57c\uae30\ub97c \ud558\uba74\uc11c \u3008\ubb34\ub140\uc758 \uc608\uc5b8\u3009\uc774 \uc778\uc6a9\ub41c\ub2e4.", "sentence_answer": "\uac15\uae00\ub808\ub9ac\ub294 \ub192\uc73c\uc2e0 \ubd84\uacfc \uadf8\ub9cc\ud07c \ub192\uc73c\uc2e0 \ubd84\uacfc \uc138\ubc88\uc9f8 \ubd84\uc774 \uc624\ub518 3\ud615\uc81c \ub97c \uc2e0\uc73c\ub85c \uc0dd\uac01\ud55c\ub2e4\uba74, \uadf8 \uc14b\uc774 \ud55c \uc77c\uc774 \ubb50\uac00 \uc788\ub0d0\uace0 \ubb3b\ub294\ub2e4.", "paragraph_id": "6526951-4-0", "paragraph_question": "question: \uc704\ubbf8\ub974\uc758 \uc774\ube68\ub85c \uc790\uac08\uacfc \ub3cc\uba69\uc774\ub97c \ub9cc\ub4e0 \uc774\ub4e4\uc740?, context: \uac15\uae00\ub808\ub9ac\ub294 \ub192\uc73c\uc2e0 \ubd84\uacfc \uadf8\ub9cc\ud07c \ub192\uc73c\uc2e0 \ubd84\uacfc \uc138\ubc88\uc9f8 \ubd84\uc774 \uc624\ub518 3\ud615\uc81c\ub97c \uc2e0\uc73c\ub85c \uc0dd\uac01\ud55c\ub2e4\uba74, \uadf8 \uc14b\uc774 \ud55c \uc77c\uc774 \ubb50\uac00 \uc788\ub0d0\uace0 \ubb3b\ub294\ub2e4. \ub192\uc73c\uc2e0 \ubd84\uc740 \uc624\ub518 3\ud615\uc81c\uac00 \uc704\ubbf8\ub974\uc758 \uc2dc\uccb4\ub97c \uae34\ub219\uac00\uac00\ud504 \ud55c\uac00\uc6b4\ub370\ub85c \uac00\uc838\uac00 \uc704\ubbf8\ub974\uc758 \uc0b4\ub85c \ub300\uc9c0\ub97c \uc0bc\uace0, \ud53c\ub85c \ubc14\ub2e4\uc640 \ud638\uc218\ub97c \ub9cc\ub4e4\uace0, \ubf08\ub85c \ubc14\uc704\ub97c \ub9cc\ub4e4\uace0, \uc774\ube68\ub85c \uc790\uac08\uacfc \ub3cc\uba69\uc774\ub97c \ub9cc\ub4e4\uc5c8\ub2e4\uace0 \ub9d0\ud55c\ub2e4. \uadf8\ub9cc\ud07c \ub192\uc73c\uc2e0 \ubd84\uc774 \ub9d0\uc744 \ubc1b\uc544 \uc704\ubbf8\ub974\uc758 \uc0c1\ucc98\uc5d0\uc11c \uc3df\uc544\uc9c4 \ud53c\uac00 \ub300\uc9c0\ub97c \ub458\ub7ec\uc2f8\ub294 \ubc14\ub2e4\ub97c \ub9cc\ub4e4\uc5c8\ub2e4\uace0 \ud55c\ub2e4. \uadf8\ub9ac\uace0 \uc138 \ubc88\uc9f8 \ubd84\uc774 3\ud615\uc81c\uac00 \uc704\ubbf8\ub974\uc758 \ub450\uac1c\uace8\uc744 \ub545 \uc704\ub85c \ub4e4\uc5b4\uc62c\ub824 \ud558\ub298\uc744 \uc0bc\uc558\uc73c\uba70, \ub178\ub974\ub4dc\ub9ac, \uc218\ub4dc\ub9ac, \uc544\uc6b0\uc2a4\ud2b8\ub9ac, \ubca0\uc2a4\ud2b8\ub9ac\ub77c\ub294 \ub4dc\ubca0\ub974\uadf8 \ub137\uc744 \ub3d9\uc11c\ub0a8\ubd81 \ub124 \ubc29\ud5a5\uc5d0\uc11c \ud558\ub298\uc744 \ubc1b\uce58\uace0 \uc788\uac8c \ud588\ub2e4\uace0 \ub9d0\ud55c\ub2e4. 3\ud615\uc81c\ub294 \ubb34\uc2a4\ud3a0\uc2a4\ud5e4\uc784\uc5d0\uc11c \ub0a0\uc544\uc628 \ubd88\ud2f0\uc640 \ub179\uc740 \uc870\uac01\ub4e4\uc744 \uac00\uc838\ub2e4\uac00 \uba87 \uac1c\ub294 \ud558\ub298\uc5d0 \ubc15\uace0 \uba87 \uac1c\ub294 \ud558\ub298 \uc544\ub798\uc5d0\uc11c \uc6c0\uc9c1\uc774\uac8c \ud558\uc600\uc73c\ub2c8 \uc774\uac83\uc774 \uac01\uac01 \ubcc4\uacfc \ud574\uc640 \ub2ec\uc774\ub2e4. \uc774 \uc774\uc57c\uae30\ub97c \ud558\uba74\uc11c \u3008\ubb34\ub140\uc758 \uc608\uc5b8\u3009\uc774 \uc778\uc6a9\ub41c\ub2e4."} {"question": "1915\ub144 \uc77c\ubc18\uc0c1\ub300\ub860\uc744 \uace0\uc548\ud558\uc5ec \uc911\ub825\uc774 \ube5b\uc758 \uc6b4\ub3d9\uc5d0 \uc601\ud5a5\uc744 \ubbf8\uce68\uc744 \ubcf4\uc778\uac83\uc740 \ub204\uad6c\uc778\uac00?", "paragraph": "1915\ub144, \uc54c\ubca0\ub974\ud2b8 \uc544\uc778\uc288\ud0c0\uc778\uc774 \uc77c\ubc18\uc0c1\ub300\ub860\uc744 \uace0\uc548\ud558\uc5ec \uc911\ub825\uc774 \ube5b\uc758 \uc6b4\ub3d9\uc5d0 \uc601\ud5a5\uc744 \ubbf8\uce68\uc744 \ubcf4\uc600\ub2e4. \ubd88\uacfc \uba87 \uac1c\uc6d4 \ub4a4, \uce74\ub97c \uc288\ubc14\ub974\uce20\uc2e4\ud2b8\uac00 \uc810\uc9c8\ub7c9\uacfc \uad6c\uc9c8\ub7c9\uc758 \uc911\ub825\uc7a5\uc744 \uae30\uc220\ud558\ub294 \uc544\uc778\uc288\ud0c0\uc778 \ubc29\uc815\uc2dd\uc758 \ud574\ub97c \uad6c\ud558\uc600\ub2e4(\uc288\ubc14\ub974\uce20\uc2e4\ud2b8 \uacc4\ub7c9). \uc288\ubc14\ub974\uce20\uc2e4\ud2b8\ub85c\ubd80\ud130 \ub610 \uba87 \uac1c\uc6d4 \ub4a4, \ud5e8\ub4dc\ub9ad \ub85c\ub7f0\uce20\uc758 \uc9c0\ub3c4\ud559\uc0dd\uc778 \uc694\ud558\ub124\uc2a4 \ub4dc\ub85c\uc2a4\ud130\uac00 \uc288\ubc14\ub974\uce20\uc2e4\ud2b8\uc640 \ub3c5\ub9bd\uc801\uc73c\ub85c \uc810\uc9c8\ub7c9\uc5d0 \ub300\ud55c \ub3d9\uc77c\ud55c \ud574\ub97c \uad6c\ud558\uc600\uace0, \uadf8 \uc131\uc9c8\uc744 \ubcf4\ub2e4 \uad11\ubc94\ud558\uac8c \uae30\uc220\ud558\uc600\ub2e4. \uc774 \ud574\ub294 \uc544\uc778\uc288\ud0c0\uc778 \ubc29\uc815\uc2dd\uc758 \uc77c\ubd80 \ud56d\uc774 \ubb34\ud55c\ub300\uac00 \ub418\ub294 \ud2b9\uc774\uc810\uc744 \uac00\uc9c0\ub294 \ud2b9\uc774\ud589\ub3d9\uc744 \ubcf4\uc774\ub294\ub370, \uc774\uac83\uc744 \uc624\ub298\ub0a0 \uc288\ubc14\ub974\uce20\uc2e4\ud2b8 \ubc18\uacbd\uc774\ub77c\uace0 \ubd80\ub978\ub2e4. \uc774\ub54c\uae4c\uc9c0\ub9cc \ud574\ub3c4 \uc774 \ud45c\uba74\uc758 \uc131\uc9c8\uc740 \ud655\uc2e4\ud558\uac8c \uc774\ud574\ub418\uc9c0 \uc54a\uc558\ub2e4. 1924\ub144, \uc544\uc11c \uc5d0\ub529\ud134\uc774 \uc88c\ud45c\uacc4\uc758 \uc218\uc815\uc744 \ud1b5\ud574(\uc5d0\ub529\ud134-\ud540\ucf08\uc2a4\ud0c0\uc778 \uc88c\ud45c\uacc4) \ud2b9\uc774\uc810\uc744 \uc5c6\uc568 \uc218 \uc788\uc74c\uc744 \ubcf4\uc600\uc73c\ub098, \uc774\uac83\uc774 \uc288\ubc14\ub974\uce20\uc2e4\ud2b8 \ubc18\uacbd\uc758 \ud2b9\uc774\uc810\uc774 \ube44\ubb3c\ub9ac\uc801 \uc88c\ud45c \ud2b9\uc774\uc810\uc774\ub77c\ub294 \uac83\uc744 \uc758\ubbf8\ud55c\ub2e4\ub294 \uac83\uc740 1933\ub144\uc5d0\uc57c \uc870\ub974\uc8fc \ub974\uba54\ud2b8\ub974\uac00 \ubc1d\ud600\ub0b4\uc5c8\ub2e4. \ud55c\ud3b8 \uc544\uc11c \uc5d0\ub529\ud134\uc740 1926\ub144 \uc800\uc11c\uc5d0\uc11c \uc5b4\ub5a4 \ubcc4\uc774 \uc288\ubc14\ub974\uce20\uc2e4\ud2b8 \ubc18\uacbd \uc774\ud558\uc758 \ud06c\uae30\ub85c \uc9dc\ubd80\ub77c\ub4e4 \uac00\ub2a5\uc131\uc5d0 \ub300\ud558\uc5ec \ub2e4\uc74c\uacfc \uac19\uc774 \ub17c\ud558\uc600\ub2e4. \uc544\uc778\uc288\ud0c0\uc778\uc758 \uc774\ub860\uc740 \ubca0\ud154\uac8c\uc6b0\uc2a4 \uac19\uc740 \uac70\ub300\ud55c \ubcc4\ub4e4\uc774 \ubb34\uc9c0\ub9c9\uc9c0\ud55c \ubc00\ub3c4\ub97c \uac00\uc9c0\ub294 \uac83\uc744 \ubd88\uac00\ub2a5\ud558\uac8c \ud55c\ub2e4. \uc774\uc720\uc778\uc989 \u201c\ubc18\uacbd 2\uc5b5 5\ucc9c\ub9cc \ud0ac\ub85c\ubbf8\ud130\uc758 \ud56d\uc131\uc740 \ud0dc\uc591\ub9cc\ud07c \ub192\uc740 \ubc00\ub3c4\ub97c \uac00\uc9c8 \uc218 \uc5c6\ub2e4. \uc6b0\uc120 \uc911\ub825\uc774 \ub108\ubb34 \ucee4\uc838\uc11c \ube5b\uc774 \uadf8 \ubcc4\uc5d0\uc11c \ud0c8\ucd9c\ud560 \uc218 \uc5c6\uc744 \uac83\uc774\uba70, \ub9c8\uce58 \uc9c0\uad6c\ub85c \ub418\ub5a8\uc5b4\uc9c0\ub294 \ub3cc\ucc98\ub7fc \ube5b\uc0b4\uc774 \ubcc4\ub85c \ub418\ub5a8\uc5b4\uc9c0\uac8c \ub420 \uac83\uc774\ub2e4. \ub458\uc9f8\ub85c \uc2a4\ud399\ud2b8\ub7fc\uc120\uc758 \uc801\uc0c9\ud3b8\uc774\uac00 \ub108\ubb34 \ucee4\uc11c \uc2a4\ud399\ud2b8\ub7fc\uc774 \uc18c\uba78\ud560 \uc9c0\uacbd\uc73c\ub85c \ud3b8\uc774\ud560 \uac83\uc774\ub2e4. \uc14b\uc9f8\ub85c \uc9c8\ub7c9\uc774 \ub108\ubb34 \ud070 \ub9cc\ud07c \uc2dc\uacf5\uac04\uc5d0 \ub300\ud55c \uc65c\uace1\ub3c4 \ud06c\uac8c \ubc1c\uc0dd\ud558\uc5ec \uacf5\uac04\uc774 \ubcc4\uc744 \uc544\ubb3c\uc5b4 \uc228\uaca8 \ubc84\ub9b4 \uac83\uc774\ub2e4.\"", "answer": "\uc54c\ubca0\ub974\ud2b8 \uc544\uc778\uc288\ud0c0\uc778", "sentence": "1915\ub144, \uc54c\ubca0\ub974\ud2b8 \uc544\uc778\uc288\ud0c0\uc778 \uc774 \uc77c\ubc18\uc0c1\ub300\ub860\uc744 \uace0\uc548\ud558\uc5ec \uc911\ub825\uc774 \ube5b\uc758 \uc6b4\ub3d9\uc5d0 \uc601\ud5a5\uc744 \ubbf8\uce68\uc744 \ubcf4\uc600\ub2e4.", "paragraph_sentence": " 1915\ub144, \uc54c\ubca0\ub974\ud2b8 \uc544\uc778\uc288\ud0c0\uc778 \uc774 \uc77c\ubc18\uc0c1\ub300\ub860\uc744 \uace0\uc548\ud558\uc5ec \uc911\ub825\uc774 \ube5b\uc758 \uc6b4\ub3d9\uc5d0 \uc601\ud5a5\uc744 \ubbf8\uce68\uc744 \ubcf4\uc600\ub2e4. \ubd88\uacfc \uba87 \uac1c\uc6d4 \ub4a4, \uce74\ub97c \uc288\ubc14\ub974\uce20\uc2e4\ud2b8\uac00 \uc810\uc9c8\ub7c9\uacfc \uad6c\uc9c8\ub7c9\uc758 \uc911\ub825\uc7a5\uc744 \uae30\uc220\ud558\ub294 \uc544\uc778\uc288\ud0c0\uc778 \ubc29\uc815\uc2dd\uc758 \ud574\ub97c \uad6c\ud558\uc600\ub2e4(\uc288\ubc14\ub974\uce20\uc2e4\ud2b8 \uacc4\ub7c9). \uc288\ubc14\ub974\uce20\uc2e4\ud2b8\ub85c\ubd80\ud130 \ub610 \uba87 \uac1c\uc6d4 \ub4a4, \ud5e8\ub4dc\ub9ad \ub85c\ub7f0\uce20\uc758 \uc9c0\ub3c4\ud559\uc0dd\uc778 \uc694\ud558\ub124\uc2a4 \ub4dc\ub85c\uc2a4\ud130\uac00 \uc288\ubc14\ub974\uce20\uc2e4\ud2b8\uc640 \ub3c5\ub9bd\uc801\uc73c\ub85c \uc810\uc9c8\ub7c9\uc5d0 \ub300\ud55c \ub3d9\uc77c\ud55c \ud574\ub97c \uad6c\ud558\uc600\uace0, \uadf8 \uc131\uc9c8\uc744 \ubcf4\ub2e4 \uad11\ubc94\ud558\uac8c \uae30\uc220\ud558\uc600\ub2e4. \uc774 \ud574\ub294 \uc544\uc778\uc288\ud0c0\uc778 \ubc29\uc815\uc2dd\uc758 \uc77c\ubd80 \ud56d\uc774 \ubb34\ud55c\ub300\uac00 \ub418\ub294 \ud2b9\uc774\uc810\uc744 \uac00\uc9c0\ub294 \ud2b9\uc774\ud589\ub3d9\uc744 \ubcf4\uc774\ub294\ub370, \uc774\uac83\uc744 \uc624\ub298\ub0a0 \uc288\ubc14\ub974\uce20\uc2e4\ud2b8 \ubc18\uacbd\uc774\ub77c\uace0 \ubd80\ub978\ub2e4. \uc774\ub54c\uae4c\uc9c0\ub9cc \ud574\ub3c4 \uc774 \ud45c\uba74\uc758 \uc131\uc9c8\uc740 \ud655\uc2e4\ud558\uac8c \uc774\ud574\ub418\uc9c0 \uc54a\uc558\ub2e4. 1924\ub144, \uc544\uc11c \uc5d0\ub529\ud134\uc774 \uc88c\ud45c\uacc4\uc758 \uc218\uc815\uc744 \ud1b5\ud574(\uc5d0\ub529\ud134-\ud540\ucf08\uc2a4\ud0c0\uc778 \uc88c\ud45c\uacc4) \ud2b9\uc774\uc810\uc744 \uc5c6\uc568 \uc218 \uc788\uc74c\uc744 \ubcf4\uc600\uc73c\ub098, \uc774\uac83\uc774 \uc288\ubc14\ub974\uce20\uc2e4\ud2b8 \ubc18\uacbd\uc758 \ud2b9\uc774\uc810\uc774 \ube44\ubb3c\ub9ac\uc801 \uc88c\ud45c \ud2b9\uc774\uc810\uc774\ub77c\ub294 \uac83\uc744 \uc758\ubbf8\ud55c\ub2e4\ub294 \uac83\uc740 1933\ub144\uc5d0\uc57c \uc870\ub974\uc8fc \ub974\uba54\ud2b8\ub974\uac00 \ubc1d\ud600\ub0b4\uc5c8\ub2e4. \ud55c\ud3b8 \uc544\uc11c \uc5d0\ub529\ud134\uc740 1926\ub144 \uc800\uc11c\uc5d0\uc11c \uc5b4\ub5a4 \ubcc4\uc774 \uc288\ubc14\ub974\uce20\uc2e4\ud2b8 \ubc18\uacbd \uc774\ud558\uc758 \ud06c\uae30\ub85c \uc9dc\ubd80\ub77c\ub4e4 \uac00\ub2a5\uc131\uc5d0 \ub300\ud558\uc5ec \ub2e4\uc74c\uacfc \uac19\uc774 \ub17c\ud558\uc600\ub2e4. \uc544\uc778\uc288\ud0c0\uc778\uc758 \uc774\ub860\uc740 \ubca0\ud154\uac8c\uc6b0\uc2a4 \uac19\uc740 \uac70\ub300\ud55c \ubcc4\ub4e4\uc774 \ubb34\uc9c0\ub9c9\uc9c0\ud55c \ubc00\ub3c4\ub97c \uac00\uc9c0\ub294 \uac83\uc744 \ubd88\uac00\ub2a5\ud558\uac8c \ud55c\ub2e4. \uc774\uc720\uc778\uc989 \u201c\ubc18\uacbd 2\uc5b5 5\ucc9c\ub9cc \ud0ac\ub85c\ubbf8\ud130\uc758 \ud56d\uc131\uc740 \ud0dc\uc591\ub9cc\ud07c \ub192\uc740 \ubc00\ub3c4\ub97c \uac00\uc9c8 \uc218 \uc5c6\ub2e4. \uc6b0\uc120 \uc911\ub825\uc774 \ub108\ubb34 \ucee4\uc838\uc11c \ube5b\uc774 \uadf8 \ubcc4\uc5d0\uc11c \ud0c8\ucd9c\ud560 \uc218 \uc5c6\uc744 \uac83\uc774\uba70, \ub9c8\uce58 \uc9c0\uad6c\ub85c \ub418\ub5a8\uc5b4\uc9c0\ub294 \ub3cc\ucc98\ub7fc \ube5b\uc0b4\uc774 \ubcc4\ub85c \ub418\ub5a8\uc5b4\uc9c0\uac8c \ub420 \uac83\uc774\ub2e4. \ub458\uc9f8\ub85c \uc2a4\ud399\ud2b8\ub7fc\uc120\uc758 \uc801\uc0c9\ud3b8\uc774\uac00 \ub108\ubb34 \ucee4\uc11c \uc2a4\ud399\ud2b8\ub7fc\uc774 \uc18c\uba78\ud560 \uc9c0\uacbd\uc73c\ub85c \ud3b8\uc774\ud560 \uac83\uc774\ub2e4. \uc14b\uc9f8\ub85c \uc9c8\ub7c9\uc774 \ub108\ubb34 \ud070 \ub9cc\ud07c \uc2dc\uacf5\uac04\uc5d0 \ub300\ud55c \uc65c\uace1\ub3c4 \ud06c\uac8c \ubc1c\uc0dd\ud558\uc5ec \uacf5\uac04\uc774 \ubcc4\uc744 \uc544\ubb3c\uc5b4 \uc228\uaca8 \ubc84\ub9b4 \uac83\uc774\ub2e4. \"", "paragraph_answer": "1915\ub144, \uc54c\ubca0\ub974\ud2b8 \uc544\uc778\uc288\ud0c0\uc778 \uc774 \uc77c\ubc18\uc0c1\ub300\ub860\uc744 \uace0\uc548\ud558\uc5ec \uc911\ub825\uc774 \ube5b\uc758 \uc6b4\ub3d9\uc5d0 \uc601\ud5a5\uc744 \ubbf8\uce68\uc744 \ubcf4\uc600\ub2e4. \ubd88\uacfc \uba87 \uac1c\uc6d4 \ub4a4, \uce74\ub97c \uc288\ubc14\ub974\uce20\uc2e4\ud2b8\uac00 \uc810\uc9c8\ub7c9\uacfc \uad6c\uc9c8\ub7c9\uc758 \uc911\ub825\uc7a5\uc744 \uae30\uc220\ud558\ub294 \uc544\uc778\uc288\ud0c0\uc778 \ubc29\uc815\uc2dd\uc758 \ud574\ub97c \uad6c\ud558\uc600\ub2e4(\uc288\ubc14\ub974\uce20\uc2e4\ud2b8 \uacc4\ub7c9). \uc288\ubc14\ub974\uce20\uc2e4\ud2b8\ub85c\ubd80\ud130 \ub610 \uba87 \uac1c\uc6d4 \ub4a4, \ud5e8\ub4dc\ub9ad \ub85c\ub7f0\uce20\uc758 \uc9c0\ub3c4\ud559\uc0dd\uc778 \uc694\ud558\ub124\uc2a4 \ub4dc\ub85c\uc2a4\ud130\uac00 \uc288\ubc14\ub974\uce20\uc2e4\ud2b8\uc640 \ub3c5\ub9bd\uc801\uc73c\ub85c \uc810\uc9c8\ub7c9\uc5d0 \ub300\ud55c \ub3d9\uc77c\ud55c \ud574\ub97c \uad6c\ud558\uc600\uace0, \uadf8 \uc131\uc9c8\uc744 \ubcf4\ub2e4 \uad11\ubc94\ud558\uac8c \uae30\uc220\ud558\uc600\ub2e4. \uc774 \ud574\ub294 \uc544\uc778\uc288\ud0c0\uc778 \ubc29\uc815\uc2dd\uc758 \uc77c\ubd80 \ud56d\uc774 \ubb34\ud55c\ub300\uac00 \ub418\ub294 \ud2b9\uc774\uc810\uc744 \uac00\uc9c0\ub294 \ud2b9\uc774\ud589\ub3d9\uc744 \ubcf4\uc774\ub294\ub370, \uc774\uac83\uc744 \uc624\ub298\ub0a0 \uc288\ubc14\ub974\uce20\uc2e4\ud2b8 \ubc18\uacbd\uc774\ub77c\uace0 \ubd80\ub978\ub2e4. \uc774\ub54c\uae4c\uc9c0\ub9cc \ud574\ub3c4 \uc774 \ud45c\uba74\uc758 \uc131\uc9c8\uc740 \ud655\uc2e4\ud558\uac8c \uc774\ud574\ub418\uc9c0 \uc54a\uc558\ub2e4. 1924\ub144, \uc544\uc11c \uc5d0\ub529\ud134\uc774 \uc88c\ud45c\uacc4\uc758 \uc218\uc815\uc744 \ud1b5\ud574(\uc5d0\ub529\ud134-\ud540\ucf08\uc2a4\ud0c0\uc778 \uc88c\ud45c\uacc4) \ud2b9\uc774\uc810\uc744 \uc5c6\uc568 \uc218 \uc788\uc74c\uc744 \ubcf4\uc600\uc73c\ub098, \uc774\uac83\uc774 \uc288\ubc14\ub974\uce20\uc2e4\ud2b8 \ubc18\uacbd\uc758 \ud2b9\uc774\uc810\uc774 \ube44\ubb3c\ub9ac\uc801 \uc88c\ud45c \ud2b9\uc774\uc810\uc774\ub77c\ub294 \uac83\uc744 \uc758\ubbf8\ud55c\ub2e4\ub294 \uac83\uc740 1933\ub144\uc5d0\uc57c \uc870\ub974\uc8fc \ub974\uba54\ud2b8\ub974\uac00 \ubc1d\ud600\ub0b4\uc5c8\ub2e4. \ud55c\ud3b8 \uc544\uc11c \uc5d0\ub529\ud134\uc740 1926\ub144 \uc800\uc11c\uc5d0\uc11c \uc5b4\ub5a4 \ubcc4\uc774 \uc288\ubc14\ub974\uce20\uc2e4\ud2b8 \ubc18\uacbd \uc774\ud558\uc758 \ud06c\uae30\ub85c \uc9dc\ubd80\ub77c\ub4e4 \uac00\ub2a5\uc131\uc5d0 \ub300\ud558\uc5ec \ub2e4\uc74c\uacfc \uac19\uc774 \ub17c\ud558\uc600\ub2e4. \uc544\uc778\uc288\ud0c0\uc778\uc758 \uc774\ub860\uc740 \ubca0\ud154\uac8c\uc6b0\uc2a4 \uac19\uc740 \uac70\ub300\ud55c \ubcc4\ub4e4\uc774 \ubb34\uc9c0\ub9c9\uc9c0\ud55c \ubc00\ub3c4\ub97c \uac00\uc9c0\ub294 \uac83\uc744 \ubd88\uac00\ub2a5\ud558\uac8c \ud55c\ub2e4. \uc774\uc720\uc778\uc989 \u201c\ubc18\uacbd 2\uc5b5 5\ucc9c\ub9cc \ud0ac\ub85c\ubbf8\ud130\uc758 \ud56d\uc131\uc740 \ud0dc\uc591\ub9cc\ud07c \ub192\uc740 \ubc00\ub3c4\ub97c \uac00\uc9c8 \uc218 \uc5c6\ub2e4. \uc6b0\uc120 \uc911\ub825\uc774 \ub108\ubb34 \ucee4\uc838\uc11c \ube5b\uc774 \uadf8 \ubcc4\uc5d0\uc11c \ud0c8\ucd9c\ud560 \uc218 \uc5c6\uc744 \uac83\uc774\uba70, \ub9c8\uce58 \uc9c0\uad6c\ub85c \ub418\ub5a8\uc5b4\uc9c0\ub294 \ub3cc\ucc98\ub7fc \ube5b\uc0b4\uc774 \ubcc4\ub85c \ub418\ub5a8\uc5b4\uc9c0\uac8c \ub420 \uac83\uc774\ub2e4. \ub458\uc9f8\ub85c \uc2a4\ud399\ud2b8\ub7fc\uc120\uc758 \uc801\uc0c9\ud3b8\uc774\uac00 \ub108\ubb34 \ucee4\uc11c \uc2a4\ud399\ud2b8\ub7fc\uc774 \uc18c\uba78\ud560 \uc9c0\uacbd\uc73c\ub85c \ud3b8\uc774\ud560 \uac83\uc774\ub2e4. \uc14b\uc9f8\ub85c \uc9c8\ub7c9\uc774 \ub108\ubb34 \ud070 \ub9cc\ud07c \uc2dc\uacf5\uac04\uc5d0 \ub300\ud55c \uc65c\uace1\ub3c4 \ud06c\uac8c \ubc1c\uc0dd\ud558\uc5ec \uacf5\uac04\uc774 \ubcc4\uc744 \uc544\ubb3c\uc5b4 \uc228\uaca8 \ubc84\ub9b4 \uac83\uc774\ub2e4.\"", "sentence_answer": "1915\ub144, \uc54c\ubca0\ub974\ud2b8 \uc544\uc778\uc288\ud0c0\uc778 \uc774 \uc77c\ubc18\uc0c1\ub300\ub860\uc744 \uace0\uc548\ud558\uc5ec \uc911\ub825\uc774 \ube5b\uc758 \uc6b4\ub3d9\uc5d0 \uc601\ud5a5\uc744 \ubbf8\uce68\uc744 \ubcf4\uc600\ub2e4.", "paragraph_id": "6583522-2-0", "paragraph_question": "question: 1915\ub144 \uc77c\ubc18\uc0c1\ub300\ub860\uc744 \uace0\uc548\ud558\uc5ec \uc911\ub825\uc774 \ube5b\uc758 \uc6b4\ub3d9\uc5d0 \uc601\ud5a5\uc744 \ubbf8\uce68\uc744 \ubcf4\uc778\uac83\uc740 \ub204\uad6c\uc778\uac00?, context: 1915\ub144, \uc54c\ubca0\ub974\ud2b8 \uc544\uc778\uc288\ud0c0\uc778\uc774 \uc77c\ubc18\uc0c1\ub300\ub860\uc744 \uace0\uc548\ud558\uc5ec \uc911\ub825\uc774 \ube5b\uc758 \uc6b4\ub3d9\uc5d0 \uc601\ud5a5\uc744 \ubbf8\uce68\uc744 \ubcf4\uc600\ub2e4. \ubd88\uacfc \uba87 \uac1c\uc6d4 \ub4a4, \uce74\ub97c \uc288\ubc14\ub974\uce20\uc2e4\ud2b8\uac00 \uc810\uc9c8\ub7c9\uacfc \uad6c\uc9c8\ub7c9\uc758 \uc911\ub825\uc7a5\uc744 \uae30\uc220\ud558\ub294 \uc544\uc778\uc288\ud0c0\uc778 \ubc29\uc815\uc2dd\uc758 \ud574\ub97c \uad6c\ud558\uc600\ub2e4(\uc288\ubc14\ub974\uce20\uc2e4\ud2b8 \uacc4\ub7c9). \uc288\ubc14\ub974\uce20\uc2e4\ud2b8\ub85c\ubd80\ud130 \ub610 \uba87 \uac1c\uc6d4 \ub4a4, \ud5e8\ub4dc\ub9ad \ub85c\ub7f0\uce20\uc758 \uc9c0\ub3c4\ud559\uc0dd\uc778 \uc694\ud558\ub124\uc2a4 \ub4dc\ub85c\uc2a4\ud130\uac00 \uc288\ubc14\ub974\uce20\uc2e4\ud2b8\uc640 \ub3c5\ub9bd\uc801\uc73c\ub85c \uc810\uc9c8\ub7c9\uc5d0 \ub300\ud55c \ub3d9\uc77c\ud55c \ud574\ub97c \uad6c\ud558\uc600\uace0, \uadf8 \uc131\uc9c8\uc744 \ubcf4\ub2e4 \uad11\ubc94\ud558\uac8c \uae30\uc220\ud558\uc600\ub2e4. \uc774 \ud574\ub294 \uc544\uc778\uc288\ud0c0\uc778 \ubc29\uc815\uc2dd\uc758 \uc77c\ubd80 \ud56d\uc774 \ubb34\ud55c\ub300\uac00 \ub418\ub294 \ud2b9\uc774\uc810\uc744 \uac00\uc9c0\ub294 \ud2b9\uc774\ud589\ub3d9\uc744 \ubcf4\uc774\ub294\ub370, \uc774\uac83\uc744 \uc624\ub298\ub0a0 \uc288\ubc14\ub974\uce20\uc2e4\ud2b8 \ubc18\uacbd\uc774\ub77c\uace0 \ubd80\ub978\ub2e4. \uc774\ub54c\uae4c\uc9c0\ub9cc \ud574\ub3c4 \uc774 \ud45c\uba74\uc758 \uc131\uc9c8\uc740 \ud655\uc2e4\ud558\uac8c \uc774\ud574\ub418\uc9c0 \uc54a\uc558\ub2e4. 1924\ub144, \uc544\uc11c \uc5d0\ub529\ud134\uc774 \uc88c\ud45c\uacc4\uc758 \uc218\uc815\uc744 \ud1b5\ud574(\uc5d0\ub529\ud134-\ud540\ucf08\uc2a4\ud0c0\uc778 \uc88c\ud45c\uacc4) \ud2b9\uc774\uc810\uc744 \uc5c6\uc568 \uc218 \uc788\uc74c\uc744 \ubcf4\uc600\uc73c\ub098, \uc774\uac83\uc774 \uc288\ubc14\ub974\uce20\uc2e4\ud2b8 \ubc18\uacbd\uc758 \ud2b9\uc774\uc810\uc774 \ube44\ubb3c\ub9ac\uc801 \uc88c\ud45c \ud2b9\uc774\uc810\uc774\ub77c\ub294 \uac83\uc744 \uc758\ubbf8\ud55c\ub2e4\ub294 \uac83\uc740 1933\ub144\uc5d0\uc57c \uc870\ub974\uc8fc \ub974\uba54\ud2b8\ub974\uac00 \ubc1d\ud600\ub0b4\uc5c8\ub2e4. \ud55c\ud3b8 \uc544\uc11c \uc5d0\ub529\ud134\uc740 1926\ub144 \uc800\uc11c\uc5d0\uc11c \uc5b4\ub5a4 \ubcc4\uc774 \uc288\ubc14\ub974\uce20\uc2e4\ud2b8 \ubc18\uacbd \uc774\ud558\uc758 \ud06c\uae30\ub85c \uc9dc\ubd80\ub77c\ub4e4 \uac00\ub2a5\uc131\uc5d0 \ub300\ud558\uc5ec \ub2e4\uc74c\uacfc \uac19\uc774 \ub17c\ud558\uc600\ub2e4. \uc544\uc778\uc288\ud0c0\uc778\uc758 \uc774\ub860\uc740 \ubca0\ud154\uac8c\uc6b0\uc2a4 \uac19\uc740 \uac70\ub300\ud55c \ubcc4\ub4e4\uc774 \ubb34\uc9c0\ub9c9\uc9c0\ud55c \ubc00\ub3c4\ub97c \uac00\uc9c0\ub294 \uac83\uc744 \ubd88\uac00\ub2a5\ud558\uac8c \ud55c\ub2e4. \uc774\uc720\uc778\uc989 \u201c\ubc18\uacbd 2\uc5b5 5\ucc9c\ub9cc \ud0ac\ub85c\ubbf8\ud130\uc758 \ud56d\uc131\uc740 \ud0dc\uc591\ub9cc\ud07c \ub192\uc740 \ubc00\ub3c4\ub97c \uac00\uc9c8 \uc218 \uc5c6\ub2e4. \uc6b0\uc120 \uc911\ub825\uc774 \ub108\ubb34 \ucee4\uc838\uc11c \ube5b\uc774 \uadf8 \ubcc4\uc5d0\uc11c \ud0c8\ucd9c\ud560 \uc218 \uc5c6\uc744 \uac83\uc774\uba70, \ub9c8\uce58 \uc9c0\uad6c\ub85c \ub418\ub5a8\uc5b4\uc9c0\ub294 \ub3cc\ucc98\ub7fc \ube5b\uc0b4\uc774 \ubcc4\ub85c \ub418\ub5a8\uc5b4\uc9c0\uac8c \ub420 \uac83\uc774\ub2e4. \ub458\uc9f8\ub85c \uc2a4\ud399\ud2b8\ub7fc\uc120\uc758 \uc801\uc0c9\ud3b8\uc774\uac00 \ub108\ubb34 \ucee4\uc11c \uc2a4\ud399\ud2b8\ub7fc\uc774 \uc18c\uba78\ud560 \uc9c0\uacbd\uc73c\ub85c \ud3b8\uc774\ud560 \uac83\uc774\ub2e4. \uc14b\uc9f8\ub85c \uc9c8\ub7c9\uc774 \ub108\ubb34 \ud070 \ub9cc\ud07c \uc2dc\uacf5\uac04\uc5d0 \ub300\ud55c \uc65c\uace1\ub3c4 \ud06c\uac8c \ubc1c\uc0dd\ud558\uc5ec \uacf5\uac04\uc774 \ubcc4\uc744 \uc544\ubb3c\uc5b4 \uc228\uaca8 \ubc84\ub9b4 \uac83\uc774\ub2e4.\""} {"question": "2005\ub144 5\uc6d4 \ubd81\ud55c\uc758 \ud3c9\uc591\ud559\uc0dd\uc18c\ub144\uc608\uc220\ub2e8\uc740 \ud55c\uad6d \uc5b4\ub514\uc758 \uc624\ud398\ub77c\uadf9\uc7a5\uc5d0\uc11c \uacf5\uc5f0\ud558\uc600\ub294\uac00?", "paragraph": "\uccad\ubd09\uc545\ub2e8\uc758 \uac00\uc218\ub4e4\uc778 \uae40\uc8fc\ud5a5, \uc1a1\uc601, \ub9ac\uc218\uacbd, \uae40\uccad, \uae40\uc131\uc2ec, \ub85c\uacbd\ubbf8, \uad8c\ud5a5\ub9bc \uadf8\ub9ac\uace0 \uccad\ubd09\uc545\ub2e8\uacfc \ubaa8\ub780\ubd09\uc545\ub2e8\uc744 \uc624\uac00\uba70 \ub9f9\ub82c\ud55c \ud65c\uc57d\uc744 \ubcf4\uc5ec\uc8fc\uace0 \uc788\ub294 \uc911\uacac\uac00\uc218 \uae40\uc625\uc8fc\ub294 \uc5ed\uc2dc \uc9c4\ub2ec\ub798\ube5b \ud55c\ubcf5\uc744 \uc785\uace0 \ub4f1\uc7a5\ud558\uc600\ub294\ub370 \ubd81\uce21 \ub178\ub798 \u3008\ubc18\uac11\uc2b5\ub2c8\ub2e4\u3009\ub85c \uacf5\uc5f0\uc744 \uc2dc\uc791\ud558\uc5ec \ubd81\uce21 \uc0c8\ud574 \uacf5\uc5f0\uc5d0\uc11c \ub298 \ube60\uc9d0\uc5c6\uc774 \ubd80\ub974\ub294 \u3008\uc124\ub208\uc544 \ub0b4\ub824\ub77c\u3009\ub85c \uc774\uc5b4\uc84c\ub2e4. \ub610\ud55c \uae40\uc625\uc8fc\uc640 \uc1a1\uc601\uc740 \ub0a8\uce21 \ub178\ub798\uc778 \u3008J\uc5d0\uac8c\u3009\ub97c \ud568\uaed8 \ubd88\ub800\uc73c\uba70, \uae40\uc625\uc8fc\ub294 \u3008\uc5ec\uc815\u3009\uacfc \u3008\uc0c8\ubcc4\u3009\uc744 \ub3c5\ucc3d\uc73c\ub85c \ubd88\ub7ec\uc11c \uc65c \uadf8\ub140\uac00 \uccad\ubd09\uc545\ub2e8\uacfc \ubaa8\ub780\ubd09\uc545\ub2e8\uc744 \ub204\ube44\uba70 \ud65c\uc57d\ud558\uace0 \uc788\ub294\uc9c0 \ub2e8\uc801\uc73c\ub85c \uc99d\uba85\ud574\uc8fc\uc5c8\ub2e4. \uccad\ubd09\uc545\ub2e8\uc758 \uac00\uc218\uc778 \uae40\uc8fc\ud5a5\uc740 \uc774\ubc88\uc774 \ub450 \ubc88\uc9f8 \uc608\uc220\ub2e8 \ubc29\ubb38\uc73c\ub85c \uc9c0\ub09c 2000\ub144 5\uc6d4 \ud3c9\uc591\ud559\uc0dd\uc18c\ub144\uc608\uc220\ub2e8\uc758 \uc77c\uc6d0\uc73c\ub85c \uc11c\uc6b8 \uc608\uc220\uc758\uc804\ub2f9 \uc624\ud398\ub77c\uadf9\uc7a5 \uacf5\uc5f0\uc5d0\uc11c \ub178\ub798\ub97c \ubd80\ub978 \uc801\uc774 \uc788\uc5c8\ub294\ub370, \ub2f9\uc2dc \uae40\uc8fc\ud5a5\uc758 \ub098\uc774\ub294 8\uc0b4\uc774\uc5c8\ub2e4.", "answer": "\uc608\uc220\uc758\uc804\ub2f9", "sentence": "\uccad\ubd09\uc545\ub2e8\uc758 \uac00\uc218\uc778 \uae40\uc8fc\ud5a5\uc740 \uc774\ubc88\uc774 \ub450 \ubc88\uc9f8 \uc608\uc220\ub2e8 \ubc29\ubb38\uc73c\ub85c \uc9c0\ub09c 2000\ub144 5\uc6d4 \ud3c9\uc591\ud559\uc0dd\uc18c\ub144\uc608\uc220\ub2e8\uc758 \uc77c\uc6d0\uc73c\ub85c \uc11c\uc6b8 \uc608\uc220\uc758\uc804\ub2f9 \uc624\ud398\ub77c\uadf9\uc7a5 \uacf5\uc5f0\uc5d0\uc11c \ub178\ub798\ub97c \ubd80\ub978 \uc801\uc774 \uc788\uc5c8\ub294\ub370, \ub2f9\uc2dc \uae40\uc8fc\ud5a5\uc758 \ub098\uc774\ub294 8\uc0b4\uc774\uc5c8\ub2e4.", "paragraph_sentence": "\uccad\ubd09\uc545\ub2e8\uc758 \uac00\uc218\ub4e4\uc778 \uae40\uc8fc\ud5a5, \uc1a1\uc601, \ub9ac\uc218\uacbd, \uae40\uccad, \uae40\uc131\uc2ec, \ub85c\uacbd\ubbf8, \uad8c\ud5a5\ub9bc \uadf8\ub9ac\uace0 \uccad\ubd09\uc545\ub2e8\uacfc \ubaa8\ub780\ubd09\uc545\ub2e8\uc744 \uc624\uac00\uba70 \ub9f9\ub82c\ud55c \ud65c\uc57d\uc744 \ubcf4\uc5ec\uc8fc\uace0 \uc788\ub294 \uc911\uacac\uac00\uc218 \uae40\uc625\uc8fc\ub294 \uc5ed\uc2dc \uc9c4\ub2ec\ub798\ube5b \ud55c\ubcf5\uc744 \uc785\uace0 \ub4f1\uc7a5\ud558\uc600\ub294\ub370 \ubd81\uce21 \ub178\ub798 \u3008\ubc18\uac11\uc2b5\ub2c8\ub2e4\u3009\ub85c \uacf5\uc5f0\uc744 \uc2dc\uc791\ud558\uc5ec \ubd81\uce21 \uc0c8\ud574 \uacf5\uc5f0\uc5d0\uc11c \ub298 \ube60\uc9d0\uc5c6\uc774 \ubd80\ub974\ub294 \u3008\uc124\ub208\uc544 \ub0b4\ub824\ub77c\u3009\ub85c \uc774\uc5b4\uc84c\ub2e4. \ub610\ud55c \uae40\uc625\uc8fc\uc640 \uc1a1\uc601\uc740 \ub0a8\uce21 \ub178\ub798\uc778 \u3008J\uc5d0\uac8c\u3009\ub97c \ud568\uaed8 \ubd88\ub800\uc73c\uba70, \uae40\uc625\uc8fc\ub294 \u3008\uc5ec\uc815\u3009\uacfc \u3008\uc0c8\ubcc4\u3009\uc744 \ub3c5\ucc3d\uc73c\ub85c \ubd88\ub7ec\uc11c \uc65c \uadf8\ub140\uac00 \uccad\ubd09\uc545\ub2e8\uacfc \ubaa8\ub780\ubd09\uc545\ub2e8\uc744 \ub204\ube44\uba70 \ud65c\uc57d\ud558\uace0 \uc788\ub294\uc9c0 \ub2e8\uc801\uc73c\ub85c \uc99d\uba85\ud574\uc8fc\uc5c8\ub2e4. \uccad\ubd09\uc545\ub2e8\uc758 \uac00\uc218\uc778 \uae40\uc8fc\ud5a5\uc740 \uc774\ubc88\uc774 \ub450 \ubc88\uc9f8 \uc608\uc220\ub2e8 \ubc29\ubb38\uc73c\ub85c \uc9c0\ub09c 2000\ub144 5\uc6d4 \ud3c9\uc591\ud559\uc0dd\uc18c\ub144\uc608\uc220\ub2e8\uc758 \uc77c\uc6d0\uc73c\ub85c \uc11c\uc6b8 \uc608\uc220\uc758\uc804\ub2f9 \uc624\ud398\ub77c\uadf9\uc7a5 \uacf5\uc5f0\uc5d0\uc11c \ub178\ub798\ub97c \ubd80\ub978 \uc801\uc774 \uc788\uc5c8\ub294\ub370, \ub2f9\uc2dc \uae40\uc8fc\ud5a5\uc758 \ub098\uc774\ub294 8\uc0b4\uc774\uc5c8\ub2e4. ", "paragraph_answer": "\uccad\ubd09\uc545\ub2e8\uc758 \uac00\uc218\ub4e4\uc778 \uae40\uc8fc\ud5a5, \uc1a1\uc601, \ub9ac\uc218\uacbd, \uae40\uccad, \uae40\uc131\uc2ec, \ub85c\uacbd\ubbf8, \uad8c\ud5a5\ub9bc \uadf8\ub9ac\uace0 \uccad\ubd09\uc545\ub2e8\uacfc \ubaa8\ub780\ubd09\uc545\ub2e8\uc744 \uc624\uac00\uba70 \ub9f9\ub82c\ud55c \ud65c\uc57d\uc744 \ubcf4\uc5ec\uc8fc\uace0 \uc788\ub294 \uc911\uacac\uac00\uc218 \uae40\uc625\uc8fc\ub294 \uc5ed\uc2dc \uc9c4\ub2ec\ub798\ube5b \ud55c\ubcf5\uc744 \uc785\uace0 \ub4f1\uc7a5\ud558\uc600\ub294\ub370 \ubd81\uce21 \ub178\ub798 \u3008\ubc18\uac11\uc2b5\ub2c8\ub2e4\u3009\ub85c \uacf5\uc5f0\uc744 \uc2dc\uc791\ud558\uc5ec \ubd81\uce21 \uc0c8\ud574 \uacf5\uc5f0\uc5d0\uc11c \ub298 \ube60\uc9d0\uc5c6\uc774 \ubd80\ub974\ub294 \u3008\uc124\ub208\uc544 \ub0b4\ub824\ub77c\u3009\ub85c \uc774\uc5b4\uc84c\ub2e4. \ub610\ud55c \uae40\uc625\uc8fc\uc640 \uc1a1\uc601\uc740 \ub0a8\uce21 \ub178\ub798\uc778 \u3008J\uc5d0\uac8c\u3009\ub97c \ud568\uaed8 \ubd88\ub800\uc73c\uba70, \uae40\uc625\uc8fc\ub294 \u3008\uc5ec\uc815\u3009\uacfc \u3008\uc0c8\ubcc4\u3009\uc744 \ub3c5\ucc3d\uc73c\ub85c \ubd88\ub7ec\uc11c \uc65c \uadf8\ub140\uac00 \uccad\ubd09\uc545\ub2e8\uacfc \ubaa8\ub780\ubd09\uc545\ub2e8\uc744 \ub204\ube44\uba70 \ud65c\uc57d\ud558\uace0 \uc788\ub294\uc9c0 \ub2e8\uc801\uc73c\ub85c \uc99d\uba85\ud574\uc8fc\uc5c8\ub2e4. \uccad\ubd09\uc545\ub2e8\uc758 \uac00\uc218\uc778 \uae40\uc8fc\ud5a5\uc740 \uc774\ubc88\uc774 \ub450 \ubc88\uc9f8 \uc608\uc220\ub2e8 \ubc29\ubb38\uc73c\ub85c \uc9c0\ub09c 2000\ub144 5\uc6d4 \ud3c9\uc591\ud559\uc0dd\uc18c\ub144\uc608\uc220\ub2e8\uc758 \uc77c\uc6d0\uc73c\ub85c \uc11c\uc6b8 \uc608\uc220\uc758\uc804\ub2f9 \uc624\ud398\ub77c\uadf9\uc7a5 \uacf5\uc5f0\uc5d0\uc11c \ub178\ub798\ub97c \ubd80\ub978 \uc801\uc774 \uc788\uc5c8\ub294\ub370, \ub2f9\uc2dc \uae40\uc8fc\ud5a5\uc758 \ub098\uc774\ub294 8\uc0b4\uc774\uc5c8\ub2e4.", "sentence_answer": "\uccad\ubd09\uc545\ub2e8\uc758 \uac00\uc218\uc778 \uae40\uc8fc\ud5a5\uc740 \uc774\ubc88\uc774 \ub450 \ubc88\uc9f8 \uc608\uc220\ub2e8 \ubc29\ubb38\uc73c\ub85c \uc9c0\ub09c 2000\ub144 5\uc6d4 \ud3c9\uc591\ud559\uc0dd\uc18c\ub144\uc608\uc220\ub2e8\uc758 \uc77c\uc6d0\uc73c\ub85c \uc11c\uc6b8 \uc608\uc220\uc758\uc804\ub2f9 \uc624\ud398\ub77c\uadf9\uc7a5 \uacf5\uc5f0\uc5d0\uc11c \ub178\ub798\ub97c \ubd80\ub978 \uc801\uc774 \uc788\uc5c8\ub294\ub370, \ub2f9\uc2dc \uae40\uc8fc\ud5a5\uc758 \ub098\uc774\ub294 8\uc0b4\uc774\uc5c8\ub2e4.", "paragraph_id": "6535636-1-2", "paragraph_question": "question: 2005\ub144 5\uc6d4 \ubd81\ud55c\uc758 \ud3c9\uc591\ud559\uc0dd\uc18c\ub144\uc608\uc220\ub2e8\uc740 \ud55c\uad6d \uc5b4\ub514\uc758 \uc624\ud398\ub77c\uadf9\uc7a5\uc5d0\uc11c \uacf5\uc5f0\ud558\uc600\ub294\uac00?, context: \uccad\ubd09\uc545\ub2e8\uc758 \uac00\uc218\ub4e4\uc778 \uae40\uc8fc\ud5a5, \uc1a1\uc601, \ub9ac\uc218\uacbd, \uae40\uccad, \uae40\uc131\uc2ec, \ub85c\uacbd\ubbf8, \uad8c\ud5a5\ub9bc \uadf8\ub9ac\uace0 \uccad\ubd09\uc545\ub2e8\uacfc \ubaa8\ub780\ubd09\uc545\ub2e8\uc744 \uc624\uac00\uba70 \ub9f9\ub82c\ud55c \ud65c\uc57d\uc744 \ubcf4\uc5ec\uc8fc\uace0 \uc788\ub294 \uc911\uacac\uac00\uc218 \uae40\uc625\uc8fc\ub294 \uc5ed\uc2dc \uc9c4\ub2ec\ub798\ube5b \ud55c\ubcf5\uc744 \uc785\uace0 \ub4f1\uc7a5\ud558\uc600\ub294\ub370 \ubd81\uce21 \ub178\ub798 \u3008\ubc18\uac11\uc2b5\ub2c8\ub2e4\u3009\ub85c \uacf5\uc5f0\uc744 \uc2dc\uc791\ud558\uc5ec \ubd81\uce21 \uc0c8\ud574 \uacf5\uc5f0\uc5d0\uc11c \ub298 \ube60\uc9d0\uc5c6\uc774 \ubd80\ub974\ub294 \u3008\uc124\ub208\uc544 \ub0b4\ub824\ub77c\u3009\ub85c \uc774\uc5b4\uc84c\ub2e4. \ub610\ud55c \uae40\uc625\uc8fc\uc640 \uc1a1\uc601\uc740 \ub0a8\uce21 \ub178\ub798\uc778 \u3008J\uc5d0\uac8c\u3009\ub97c \ud568\uaed8 \ubd88\ub800\uc73c\uba70, \uae40\uc625\uc8fc\ub294 \u3008\uc5ec\uc815\u3009\uacfc \u3008\uc0c8\ubcc4\u3009\uc744 \ub3c5\ucc3d\uc73c\ub85c \ubd88\ub7ec\uc11c \uc65c \uadf8\ub140\uac00 \uccad\ubd09\uc545\ub2e8\uacfc \ubaa8\ub780\ubd09\uc545\ub2e8\uc744 \ub204\ube44\uba70 \ud65c\uc57d\ud558\uace0 \uc788\ub294\uc9c0 \ub2e8\uc801\uc73c\ub85c \uc99d\uba85\ud574\uc8fc\uc5c8\ub2e4. \uccad\ubd09\uc545\ub2e8\uc758 \uac00\uc218\uc778 \uae40\uc8fc\ud5a5\uc740 \uc774\ubc88\uc774 \ub450 \ubc88\uc9f8 \uc608\uc220\ub2e8 \ubc29\ubb38\uc73c\ub85c \uc9c0\ub09c 2000\ub144 5\uc6d4 \ud3c9\uc591\ud559\uc0dd\uc18c\ub144\uc608\uc220\ub2e8\uc758 \uc77c\uc6d0\uc73c\ub85c \uc11c\uc6b8 \uc608\uc220\uc758\uc804\ub2f9 \uc624\ud398\ub77c\uadf9\uc7a5 \uacf5\uc5f0\uc5d0\uc11c \ub178\ub798\ub97c \ubd80\ub978 \uc801\uc774 \uc788\uc5c8\ub294\ub370, \ub2f9\uc2dc \uae40\uc8fc\ud5a5\uc758 \ub098\uc774\ub294 8\uc0b4\uc774\uc5c8\ub2e4."} {"question": "BBC \ud154\ub808\ube44\uc804 \uc2dc\uc5b4\ud130\uc5d0\uc11c \ucc0d\uc740 \uc601\ud654\ub294 \uc5b4\ub5a4 \ud22c\uc5b4\ub97c \uc704\ud574 \ucc0d\uc740 \uac83\uc778\uac00?", "paragraph": "1970\ub144\ub300\uc758 \ubca0\ub9ac\ub294 \uadf8\uc758 \ucd08\uae30 \uc131\uacf5\uc774 \ub4b7\ubc1b\uce68\ub418\uc5b4 \ud22c\uc5b4\ub97c \uc131\uc0ac\uc2dc\ucf30\ub2e4. \uadf8\ub294 \uc218\ub144\uac04 \uae41\uc2a8 \uae30\ud0c0\ub97c \uc9ca\uc5b4\uc9c0\uace0 \uc5b4\ub514\ub85c \uac00\ub4e0 \ubc34\ub4dc\uac00 \uc790\uc2e0\uc758 \uc74c\uc545\uc744 \uc54c\uace0 \uc788\uc744 \uac83\uc774\ub77c\uace0 \ud655\uc2e0\ud558\uba74\uc11c \uc774\ub4e4\uc744 \uace0\uc6a9\ud558\uba70 \uba3c \uae38\uc744 \ub5a0\ub0ac\ub2e4. \uc62c\ubba4\uc9c1\uc740 \uc774 \uc2dc\uae30 \uadf8\uc758 \"\ub77c\uc774\ube0c \uacf5\uc5f0\uc740 \uacc4\uc18d\ud574\uc11c \ubd88\uaddc\uce59\ud574\uc84c\ub2e4 ... \ub054\ucc0d\ud55c \ubc31\uc5c5 \ubc34\ub4dc\uc640 \ud611\uc5f0\ud558\uc5ec \uc124\ub801\ud558\uba70, \uac00\ub77d \uc548 \ub9de\ub294 \uacf5\uc5f0\uc744 \ubcf4\uc5ec\uc92c\ub2e4.\"\uba74\uc11c \uadf8\uc758 \uba85\uc131\uc5d0 \uba39\uce60\uc744 \ud588\ub2e4\uace0 \ud3c9\ub860\ud588\ub2e4. 1972\ub144 3\uc6d4\uc5d0\ub294 \uc250\ud37c\ub4dc \ubd80\uc2dc\uc5d0 \uc704\uce58\ud55c BBC \ud154\ub808\ube44\uc804 \uc2dc\uc5b4\ud130\uc5d0\uc11c, \ubc34\ub4dc \ub77c\ud0b9 \ud638\uc2a4(Rocking Horse)\uac00 \ubc31\uc5c5\ud55c 60\uc77c\uc758 \ud22c\uc5b4 \uc911 \uc77c\ubd80\uc778 \u300a\ucc99 \ubca0\ub9ac \uc778 \ucf58\uc11c\ud2b8\u300b\ub97c \uc704\ud574 \uc601\ud654\ub97c \ucc0d\ub294\ub2e4. 1970\ub144\ub300 \uadf8\uc640 \ud568\uaed8\ud55c \uc218\ub9ce\uc740 \ubc31\uc5c5 \ubba4\uc9c0\uc158 \uc911\uc5d0\uc11c\ub294 \ub9c9 \uc74c\uc545\uc744 \uc2dc\uc791\ud55c \ube0c\ub8e8\uc2a4 \uc2a4\ud504\ub9c1\uc2a4\ud2f4\uacfc \uc2a4\ud2f0\ube0c \ubc00\ub7ec\uac00 \uc788\uc5c8\ub2e4. \uc2a4\ud504\ub9c1\uc2a4\ud2f4\uc740 \ub2e4\ud050\uba58\ud130\ub9ac \uc601\ud654 \u300a\ud5e4\uc77c! \ud5e4\uc77c! \ub85c\ud070\ub864\u300b\uc5d0\uc11c \ubca0\ub9ac\uac00 \ubc34\ub4dc\uc5d0\uac8c \uc138\ud2b8 \ub9ac\uc2a4\ud2b8\ub97c \uc8fc\uc9c0 \uc54a\uc558\uc73c\uba70, \uadf8\ub294 \uc790\uc2e0\uc758 \uae30\ud0c0 \uc778\ud2b8\ub85c\ub97c \ub4e3\uace0 \uadf8\ub4e4\uc774 \uc54c\uc544\uc11c \uc790\uc2e0\uc5d0\uac8c \ub530\ub77c\uc62c \uac83\uc774\ub77c \uae30\ub300\ud588\ub2e4\uace0 \ub9d0\ud588\ub2e4. \ubca0\ub9ac\ub294 \uacf5\uc5f0 \uc774\ud6c4 \ubc34\ub4dc\uc5d0\uac8c \uace0\ub9d9\ub2e4\ub294 \uc778\uc0ac \ud558\ub098 \uc5c6\uc5c8\ub2e4. \uadf8\ub807\uc9c0\ub9cc, \uc2a4\ud504\ub9c1\uc2a4\ud2f4\uc740 \uadf8\uac00 1995\ub144\uc5d0 \ub85c\ud070\ub864 \uba85\uc608\uc758 \uc804\ub2f9 \ucf58\uc11c\ud2b8\ub97c \ud560 \ub54c \ubca0\ub9ac\uc758 \ubc31\uc5c5\uc744 \ub2e4\uc2dc\uae08 \ub9e1\uc544\uc8fc\uc5c8\ub2e4. \uc9c0\ubbf8 \uce74\ud130\uc758 \uc694\uccad\uc5d0 \uc758\ud574 \ubca0\ub9ac\ub294 1979\ub144 6\uc6d4 1\uc77c \ubc31\uc545\uad00\uc5d0\uc11c \uacf5\uc5f0\ud558\uae30\ub3c4 \ud588\ub2e4.", "answer": "\ucc99 \ubca0\ub9ac \uc778 \ucf58\uc11c\ud2b8", "sentence": "1972\ub144 3\uc6d4\uc5d0\ub294 \uc250\ud37c\ub4dc \ubd80\uc2dc\uc5d0 \uc704\uce58\ud55c BBC \ud154\ub808\ube44\uc804 \uc2dc\uc5b4\ud130\uc5d0\uc11c, \ubc34\ub4dc \ub77c\ud0b9 \ud638\uc2a4(Rocking Horse)\uac00 \ubc31\uc5c5\ud55c 60\uc77c\uc758 \ud22c\uc5b4 \uc911 \uc77c\ubd80\uc778 \u300a \ucc99 \ubca0\ub9ac \uc778 \ucf58\uc11c\ud2b8 \u300b\ub97c \uc704\ud574 \uc601\ud654\ub97c \ucc0d\ub294\ub2e4.", "paragraph_sentence": "1970\ub144\ub300\uc758 \ubca0\ub9ac\ub294 \uadf8\uc758 \ucd08\uae30 \uc131\uacf5\uc774 \ub4b7\ubc1b\uce68\ub418\uc5b4 \ud22c\uc5b4\ub97c \uc131\uc0ac\uc2dc\ucf30\ub2e4. \uadf8\ub294 \uc218\ub144\uac04 \uae41\uc2a8 \uae30\ud0c0\ub97c \uc9ca\uc5b4\uc9c0\uace0 \uc5b4\ub514\ub85c \uac00\ub4e0 \ubc34\ub4dc\uac00 \uc790\uc2e0\uc758 \uc74c\uc545\uc744 \uc54c\uace0 \uc788\uc744 \uac83\uc774\ub77c\uace0 \ud655\uc2e0\ud558\uba74\uc11c \uc774\ub4e4\uc744 \uace0\uc6a9\ud558\uba70 \uba3c \uae38\uc744 \ub5a0\ub0ac\ub2e4. \uc62c\ubba4\uc9c1\uc740 \uc774 \uc2dc\uae30 \uadf8\uc758 \"\ub77c\uc774\ube0c \uacf5\uc5f0\uc740 \uacc4\uc18d\ud574\uc11c \ubd88\uaddc\uce59\ud574\uc84c\ub2e4 ... \ub054\ucc0d\ud55c \ubc31\uc5c5 \ubc34\ub4dc\uc640 \ud611\uc5f0\ud558\uc5ec \uc124\ub801\ud558\uba70, \uac00\ub77d \uc548 \ub9de\ub294 \uacf5\uc5f0\uc744 \ubcf4\uc5ec\uc92c\ub2e4. \"\uba74\uc11c \uadf8\uc758 \uba85\uc131\uc5d0 \uba39\uce60\uc744 \ud588\ub2e4\uace0 \ud3c9\ub860\ud588\ub2e4. 1972\ub144 3\uc6d4\uc5d0\ub294 \uc250\ud37c\ub4dc \ubd80\uc2dc\uc5d0 \uc704\uce58\ud55c BBC \ud154\ub808\ube44\uc804 \uc2dc\uc5b4\ud130\uc5d0\uc11c, \ubc34\ub4dc \ub77c\ud0b9 \ud638\uc2a4(Rocking Horse)\uac00 \ubc31\uc5c5\ud55c 60\uc77c\uc758 \ud22c\uc5b4 \uc911 \uc77c\ubd80\uc778 \u300a \ucc99 \ubca0\ub9ac \uc778 \ucf58\uc11c\ud2b8 \u300b\ub97c \uc704\ud574 \uc601\ud654\ub97c \ucc0d\ub294\ub2e4. 1970\ub144\ub300 \uadf8\uc640 \ud568\uaed8\ud55c \uc218\ub9ce\uc740 \ubc31\uc5c5 \ubba4\uc9c0\uc158 \uc911\uc5d0\uc11c\ub294 \ub9c9 \uc74c\uc545\uc744 \uc2dc\uc791\ud55c \ube0c\ub8e8\uc2a4 \uc2a4\ud504\ub9c1\uc2a4\ud2f4\uacfc \uc2a4\ud2f0\ube0c \ubc00\ub7ec\uac00 \uc788\uc5c8\ub2e4. \uc2a4\ud504\ub9c1\uc2a4\ud2f4\uc740 \ub2e4\ud050\uba58\ud130\ub9ac \uc601\ud654 \u300a\ud5e4\uc77c! \ud5e4\uc77c! \ub85c\ud070\ub864\u300b\uc5d0\uc11c \ubca0\ub9ac\uac00 \ubc34\ub4dc\uc5d0\uac8c \uc138\ud2b8 \ub9ac\uc2a4\ud2b8\ub97c \uc8fc\uc9c0 \uc54a\uc558\uc73c\uba70, \uadf8\ub294 \uc790\uc2e0\uc758 \uae30\ud0c0 \uc778\ud2b8\ub85c\ub97c \ub4e3\uace0 \uadf8\ub4e4\uc774 \uc54c\uc544\uc11c \uc790\uc2e0\uc5d0\uac8c \ub530\ub77c\uc62c \uac83\uc774\ub77c \uae30\ub300\ud588\ub2e4\uace0 \ub9d0\ud588\ub2e4. \ubca0\ub9ac\ub294 \uacf5\uc5f0 \uc774\ud6c4 \ubc34\ub4dc\uc5d0\uac8c \uace0\ub9d9\ub2e4\ub294 \uc778\uc0ac \ud558\ub098 \uc5c6\uc5c8\ub2e4. \uadf8\ub807\uc9c0\ub9cc, \uc2a4\ud504\ub9c1\uc2a4\ud2f4\uc740 \uadf8\uac00 1995\ub144\uc5d0 \ub85c\ud070\ub864 \uba85\uc608\uc758 \uc804\ub2f9 \ucf58\uc11c\ud2b8\ub97c \ud560 \ub54c \ubca0\ub9ac\uc758 \ubc31\uc5c5\uc744 \ub2e4\uc2dc\uae08 \ub9e1\uc544\uc8fc\uc5c8\ub2e4. \uc9c0\ubbf8 \uce74\ud130\uc758 \uc694\uccad\uc5d0 \uc758\ud574 \ubca0\ub9ac\ub294 1979\ub144 6\uc6d4 1\uc77c \ubc31\uc545\uad00\uc5d0\uc11c \uacf5\uc5f0\ud558\uae30\ub3c4 \ud588\ub2e4.", "paragraph_answer": "1970\ub144\ub300\uc758 \ubca0\ub9ac\ub294 \uadf8\uc758 \ucd08\uae30 \uc131\uacf5\uc774 \ub4b7\ubc1b\uce68\ub418\uc5b4 \ud22c\uc5b4\ub97c \uc131\uc0ac\uc2dc\ucf30\ub2e4. \uadf8\ub294 \uc218\ub144\uac04 \uae41\uc2a8 \uae30\ud0c0\ub97c \uc9ca\uc5b4\uc9c0\uace0 \uc5b4\ub514\ub85c \uac00\ub4e0 \ubc34\ub4dc\uac00 \uc790\uc2e0\uc758 \uc74c\uc545\uc744 \uc54c\uace0 \uc788\uc744 \uac83\uc774\ub77c\uace0 \ud655\uc2e0\ud558\uba74\uc11c \uc774\ub4e4\uc744 \uace0\uc6a9\ud558\uba70 \uba3c \uae38\uc744 \ub5a0\ub0ac\ub2e4. \uc62c\ubba4\uc9c1\uc740 \uc774 \uc2dc\uae30 \uadf8\uc758 \"\ub77c\uc774\ube0c \uacf5\uc5f0\uc740 \uacc4\uc18d\ud574\uc11c \ubd88\uaddc\uce59\ud574\uc84c\ub2e4 ... \ub054\ucc0d\ud55c \ubc31\uc5c5 \ubc34\ub4dc\uc640 \ud611\uc5f0\ud558\uc5ec \uc124\ub801\ud558\uba70, \uac00\ub77d \uc548 \ub9de\ub294 \uacf5\uc5f0\uc744 \ubcf4\uc5ec\uc92c\ub2e4.\"\uba74\uc11c \uadf8\uc758 \uba85\uc131\uc5d0 \uba39\uce60\uc744 \ud588\ub2e4\uace0 \ud3c9\ub860\ud588\ub2e4. 1972\ub144 3\uc6d4\uc5d0\ub294 \uc250\ud37c\ub4dc \ubd80\uc2dc\uc5d0 \uc704\uce58\ud55c BBC \ud154\ub808\ube44\uc804 \uc2dc\uc5b4\ud130\uc5d0\uc11c, \ubc34\ub4dc \ub77c\ud0b9 \ud638\uc2a4(Rocking Horse)\uac00 \ubc31\uc5c5\ud55c 60\uc77c\uc758 \ud22c\uc5b4 \uc911 \uc77c\ubd80\uc778 \u300a \ucc99 \ubca0\ub9ac \uc778 \ucf58\uc11c\ud2b8 \u300b\ub97c \uc704\ud574 \uc601\ud654\ub97c \ucc0d\ub294\ub2e4. 1970\ub144\ub300 \uadf8\uc640 \ud568\uaed8\ud55c \uc218\ub9ce\uc740 \ubc31\uc5c5 \ubba4\uc9c0\uc158 \uc911\uc5d0\uc11c\ub294 \ub9c9 \uc74c\uc545\uc744 \uc2dc\uc791\ud55c \ube0c\ub8e8\uc2a4 \uc2a4\ud504\ub9c1\uc2a4\ud2f4\uacfc \uc2a4\ud2f0\ube0c \ubc00\ub7ec\uac00 \uc788\uc5c8\ub2e4. \uc2a4\ud504\ub9c1\uc2a4\ud2f4\uc740 \ub2e4\ud050\uba58\ud130\ub9ac \uc601\ud654 \u300a\ud5e4\uc77c! \ud5e4\uc77c! \ub85c\ud070\ub864\u300b\uc5d0\uc11c \ubca0\ub9ac\uac00 \ubc34\ub4dc\uc5d0\uac8c \uc138\ud2b8 \ub9ac\uc2a4\ud2b8\ub97c \uc8fc\uc9c0 \uc54a\uc558\uc73c\uba70, \uadf8\ub294 \uc790\uc2e0\uc758 \uae30\ud0c0 \uc778\ud2b8\ub85c\ub97c \ub4e3\uace0 \uadf8\ub4e4\uc774 \uc54c\uc544\uc11c \uc790\uc2e0\uc5d0\uac8c \ub530\ub77c\uc62c \uac83\uc774\ub77c \uae30\ub300\ud588\ub2e4\uace0 \ub9d0\ud588\ub2e4. \ubca0\ub9ac\ub294 \uacf5\uc5f0 \uc774\ud6c4 \ubc34\ub4dc\uc5d0\uac8c \uace0\ub9d9\ub2e4\ub294 \uc778\uc0ac \ud558\ub098 \uc5c6\uc5c8\ub2e4. \uadf8\ub807\uc9c0\ub9cc, \uc2a4\ud504\ub9c1\uc2a4\ud2f4\uc740 \uadf8\uac00 1995\ub144\uc5d0 \ub85c\ud070\ub864 \uba85\uc608\uc758 \uc804\ub2f9 \ucf58\uc11c\ud2b8\ub97c \ud560 \ub54c \ubca0\ub9ac\uc758 \ubc31\uc5c5\uc744 \ub2e4\uc2dc\uae08 \ub9e1\uc544\uc8fc\uc5c8\ub2e4. \uc9c0\ubbf8 \uce74\ud130\uc758 \uc694\uccad\uc5d0 \uc758\ud574 \ubca0\ub9ac\ub294 1979\ub144 6\uc6d4 1\uc77c \ubc31\uc545\uad00\uc5d0\uc11c \uacf5\uc5f0\ud558\uae30\ub3c4 \ud588\ub2e4.", "sentence_answer": "1972\ub144 3\uc6d4\uc5d0\ub294 \uc250\ud37c\ub4dc \ubd80\uc2dc\uc5d0 \uc704\uce58\ud55c BBC \ud154\ub808\ube44\uc804 \uc2dc\uc5b4\ud130\uc5d0\uc11c, \ubc34\ub4dc \ub77c\ud0b9 \ud638\uc2a4(Rocking Horse)\uac00 \ubc31\uc5c5\ud55c 60\uc77c\uc758 \ud22c\uc5b4 \uc911 \uc77c\ubd80\uc778 \u300a \ucc99 \ubca0\ub9ac \uc778 \ucf58\uc11c\ud2b8 \u300b\ub97c \uc704\ud574 \uc601\ud654\ub97c \ucc0d\ub294\ub2e4.", "paragraph_id": "6596446-11-0", "paragraph_question": "question: BBC \ud154\ub808\ube44\uc804 \uc2dc\uc5b4\ud130\uc5d0\uc11c \ucc0d\uc740 \uc601\ud654\ub294 \uc5b4\ub5a4 \ud22c\uc5b4\ub97c \uc704\ud574 \ucc0d\uc740 \uac83\uc778\uac00?, context: 1970\ub144\ub300\uc758 \ubca0\ub9ac\ub294 \uadf8\uc758 \ucd08\uae30 \uc131\uacf5\uc774 \ub4b7\ubc1b\uce68\ub418\uc5b4 \ud22c\uc5b4\ub97c \uc131\uc0ac\uc2dc\ucf30\ub2e4. \uadf8\ub294 \uc218\ub144\uac04 \uae41\uc2a8 \uae30\ud0c0\ub97c \uc9ca\uc5b4\uc9c0\uace0 \uc5b4\ub514\ub85c \uac00\ub4e0 \ubc34\ub4dc\uac00 \uc790\uc2e0\uc758 \uc74c\uc545\uc744 \uc54c\uace0 \uc788\uc744 \uac83\uc774\ub77c\uace0 \ud655\uc2e0\ud558\uba74\uc11c \uc774\ub4e4\uc744 \uace0\uc6a9\ud558\uba70 \uba3c \uae38\uc744 \ub5a0\ub0ac\ub2e4. \uc62c\ubba4\uc9c1\uc740 \uc774 \uc2dc\uae30 \uadf8\uc758 \"\ub77c\uc774\ube0c \uacf5\uc5f0\uc740 \uacc4\uc18d\ud574\uc11c \ubd88\uaddc\uce59\ud574\uc84c\ub2e4 ... \ub054\ucc0d\ud55c \ubc31\uc5c5 \ubc34\ub4dc\uc640 \ud611\uc5f0\ud558\uc5ec \uc124\ub801\ud558\uba70, \uac00\ub77d \uc548 \ub9de\ub294 \uacf5\uc5f0\uc744 \ubcf4\uc5ec\uc92c\ub2e4.\"\uba74\uc11c \uadf8\uc758 \uba85\uc131\uc5d0 \uba39\uce60\uc744 \ud588\ub2e4\uace0 \ud3c9\ub860\ud588\ub2e4. 1972\ub144 3\uc6d4\uc5d0\ub294 \uc250\ud37c\ub4dc \ubd80\uc2dc\uc5d0 \uc704\uce58\ud55c BBC \ud154\ub808\ube44\uc804 \uc2dc\uc5b4\ud130\uc5d0\uc11c, \ubc34\ub4dc \ub77c\ud0b9 \ud638\uc2a4(Rocking Horse)\uac00 \ubc31\uc5c5\ud55c 60\uc77c\uc758 \ud22c\uc5b4 \uc911 \uc77c\ubd80\uc778 \u300a\ucc99 \ubca0\ub9ac \uc778 \ucf58\uc11c\ud2b8\u300b\ub97c \uc704\ud574 \uc601\ud654\ub97c \ucc0d\ub294\ub2e4. 1970\ub144\ub300 \uadf8\uc640 \ud568\uaed8\ud55c \uc218\ub9ce\uc740 \ubc31\uc5c5 \ubba4\uc9c0\uc158 \uc911\uc5d0\uc11c\ub294 \ub9c9 \uc74c\uc545\uc744 \uc2dc\uc791\ud55c \ube0c\ub8e8\uc2a4 \uc2a4\ud504\ub9c1\uc2a4\ud2f4\uacfc \uc2a4\ud2f0\ube0c \ubc00\ub7ec\uac00 \uc788\uc5c8\ub2e4. \uc2a4\ud504\ub9c1\uc2a4\ud2f4\uc740 \ub2e4\ud050\uba58\ud130\ub9ac \uc601\ud654 \u300a\ud5e4\uc77c! \ud5e4\uc77c! \ub85c\ud070\ub864\u300b\uc5d0\uc11c \ubca0\ub9ac\uac00 \ubc34\ub4dc\uc5d0\uac8c \uc138\ud2b8 \ub9ac\uc2a4\ud2b8\ub97c \uc8fc\uc9c0 \uc54a\uc558\uc73c\uba70, \uadf8\ub294 \uc790\uc2e0\uc758 \uae30\ud0c0 \uc778\ud2b8\ub85c\ub97c \ub4e3\uace0 \uadf8\ub4e4\uc774 \uc54c\uc544\uc11c \uc790\uc2e0\uc5d0\uac8c \ub530\ub77c\uc62c \uac83\uc774\ub77c \uae30\ub300\ud588\ub2e4\uace0 \ub9d0\ud588\ub2e4. \ubca0\ub9ac\ub294 \uacf5\uc5f0 \uc774\ud6c4 \ubc34\ub4dc\uc5d0\uac8c \uace0\ub9d9\ub2e4\ub294 \uc778\uc0ac \ud558\ub098 \uc5c6\uc5c8\ub2e4. \uadf8\ub807\uc9c0\ub9cc, \uc2a4\ud504\ub9c1\uc2a4\ud2f4\uc740 \uadf8\uac00 1995\ub144\uc5d0 \ub85c\ud070\ub864 \uba85\uc608\uc758 \uc804\ub2f9 \ucf58\uc11c\ud2b8\ub97c \ud560 \ub54c \ubca0\ub9ac\uc758 \ubc31\uc5c5\uc744 \ub2e4\uc2dc\uae08 \ub9e1\uc544\uc8fc\uc5c8\ub2e4. \uc9c0\ubbf8 \uce74\ud130\uc758 \uc694\uccad\uc5d0 \uc758\ud574 \ubca0\ub9ac\ub294 1979\ub144 6\uc6d4 1\uc77c \ubc31\uc545\uad00\uc5d0\uc11c \uacf5\uc5f0\ud558\uae30\ub3c4 \ud588\ub2e4."} {"question": "\ud654\uc7a5\uc758 \uc2b5\uad00\uc774 \ub05d\ub09c \ub54c\ub294?", "paragraph": "\uc5d0\ud2b8\ub8e8\ub9ac\uc544\uc758 \uc870\uac01\uc740 \uc810\ucc28\uc801\uc73c\ub85c \uae30\uc6d0\uc804 6\uc138\uae30 \ub9d0\uc774 \ub418\uc5b4\uc11c \ub3c5\ub9bd\ub41c \ub300\uc870\uac01\uc774 \uc81c\uc791\ub418\uc5c8\ub2e4. \uadf8 \uc774\uc804\uc758 \uac83\uc740 \ud0a4\uc6b0\uc2dc \ubc16\uc758 \ubd84\ubb18\uc5d0\uc11c \ubc1c\uacac\ub41c \ucd08\uae30\uc758 \ud14c\ub77c\ucf54\ud0c0\uc758 \ubf08\ud56d\uc544\ub9ac\uc5d0\uc11c \ubc1c\uacac\ub41c\ub2e4. \ucd08\uae30\uc758 \ubf08\ud56d\uc544\ub9ac\uc5d0\ub294 \ub69c\uaed1 \uc704\uc5d0 \uc791\uc740 \uc0ac\ub78c\uc774\ub098 \ub3d9\ubb3c\uc758 \uc0c1(\u50cf)\uc744 \ub9cc\ub4e0 \uac83\uc744 \ubcfc \uc218 \uc788\ub294\ub370, \uae30\uc6d0\uc804 7\uc138\uae30\uacbd\uc774 \ub418\uba74 \ubf08\ud56d\uc774\ub9ac \uc804\uccb4\uac00 \ud558\ub098\uc758 \uc778\uccb4 \ud615\uc0c1\uc774 \ub418\uc5c8\ub2e4. \uadf8\ub7ec\ub098 \ud654\uc7a5(\u706b\u846c)\uc758 \uc2b5\uad00\uc740 \uae30\uc6d0\uc804 6\uc138\uae30 \uc911\uc5fd\uc5d0 \ub05d\ub098\uace0, \ub3d9\uc2dc\uc5d0 \ubf08\ud56d\uc544\ub9ac\ub3c4 \ucc28\ucc28 \ubaa8\uc2b5\uc744 \uac10\ucd94\uace0, \uc774\uc5d0 \ub300\uc2e0\ud558\uc5ec \uad00(\u68fa) \uc704\uc5d0 \ubd80\ucc98(\u592b\u59bb)\uac00 \uac00\uae4c\uc774 \ub2e4\uac00 \uc788\ub294 \ubc18\uc640\uc0c1(\u534a\u81e5\u50cf)\uc774 \uc788\ub294 \ud14c\ub77c\ucf54\ud0c0 \uad00\uc774 \uc81c\uc791\ub418\uac8c \ub418\uc5c8\ub2e4. \ud604\uc7ac \ub8e8\ube0c\ub974 \ubbf8\uc220\uad00\uacfc \ub85c\ub9c8\uc758 \ube4c\ub77c\uc9c0\uc6b8\ub9ac\uc544 \ubbf8\uc220\uad00\uc5d0 \uc788\ub294 \uccb4\ub974\ubca0\ud2b8\ub9ac\uc5d0\uc11c \ucd9c\ud1a0\ud55c \ud14c\ub77c\ucf54\ud0c0 \uad00\uc758 <\ubd80\ucc98\uc758 \ubc18\uc640\uc0c1>\uc740 \uac00\uc7a5 \ud6cc \ub96d\ud55c \ubcf4\uae30\uc778\ub370, \uc81c\uc791 \uc5f0\ub300\ub294 \uc5b4\ub290 \uac83\uc774\ub098 \uae30\uc6d0\uc804 520\ub144\uacbd\uc774\ub77c\uace0 \ucd94\uc815\ub41c\ub2e4. \ub300\uc601 \ubc15\ubb3c\uad00\uc5d0 \uc788\ub294 \ub450\uac1c\uc758 \ud14c\ub77c\ucf54\ud0c0 \uc5ec\uc778 \uc88c\uc0c1(\u5973\u4eba\u5750\u50cf)\uc774\ub098 \ud53c\ub80c\uccb4 \uace0\uace0(\u8003\u53e4) \ubc15\ubb3c\uad00\uc758 \uccad\ub3d9 \uc804\uc0ac\uc0c1(\u6230\u58eb\u50cf)\uc774\ub098 \uc5ec\uc778\uc0c1\uc740 \uc5d0\ud2b8\ub8e8\ub9ac\uc544 \uc870\uac01\uc758 \ucd08\uae30\uc758 \uac83\uc73c\ub85c \uc88b\uc740 \uc608\ub2e4. \uc544\ub974\uce74\uc774\ud06c \uc591\uc2dd\uc774\ub77c \ubd88\ub9ac\ub294 \uae30\uc6d0\uc804 6\uc138\uae30\uc5d0\uc11c \uae30\uc6d0\uc804 5\uc138\uae30 \uc911\uc5fd\uae4c\uc9c0\uc758 \uc2dc\uae30\ub294 \uc5d0\ud2b8\ub8e8\ub9ac\uc544 \uc870\uac01\uc758 \ucd5c\uc131\uae30\ub85c\uc11c, \ud55c\ud3b8\uc73c\ub85c\ub294 \uadf8\ub9ac\uc2a4 \ubbf8\uc220\uc758 \uc601\ud5a5\uc744 \ubc1b\uc73c\uba74\uc11c, \ub610 \ub2e4\ub978 \ud55c\ud3b8\uc73c\ub85c\ub294 \uadf8 \ud45c\ud604\uacfc \uc591\uc2dd\uc5d0 \uc788\uc5b4\uc11c \uc5d0\ud2b8\ub8e8\ub9ac\uc544 \uc870\uac01\uc758 \ub3c5\uc790\uc131\uc744 \ubc1c\uc804\uc2dc\ucf30\ub2e4. \ubca0\uc774\uc624\uc758 \ubbf8\ub124\ub974\ubc14 \uc2e0\uc804\uc758 \uc9c0\ubd95\uc744 \uafb8\ubbfc <\uc544\ud3f4\ub860\uacfc \ud5e4\ub77c\ud074\ub808\uc2a4\uc758 \uad70\uc0c1(\u7fa4\u50cf)>(\ube4c\ub77c\uc9c0\uc6b8\ub9ac\uc544\uc758 \ubbf8\uc220\uad00), <\uc11c \uc788\ub294 \uc18c\ub140\uc0c1(\u5c11\u5973\u50cf)>(\ucf54\ud39c\ud558\uac90 \ub2c8\uce74\ub8e8\ub974\uc2a4\ubca0\ub85c\uadf8 \uc9c4\uc5f4\uad00)\uc740 \uad00(\u68fa)\uc758 \uc0c1(\u50cf)\uacfc \ub354\ubd88\uc5b4 \uc774 \uae30(\u671f)\uc758 \ud14c\ub77c\ucf54\ud0c0 \uc870\uac01\uc744 \ub300\ud45c\ud558\ub294 \uac78\uc791\uc774\ub2e4. \uadf8 \uc911\uc5d0\uc11c\ub3c4 \uc544\ud3f4\ub860 \uc0c1\uc740 \uc5d0\ud2b8\ub8e8\ub9ac\uc544\uc758 \uc608\uc220\uac00 \uc911\uc5d0\uc11c \uadf8 \uc774\ub984\uc774 \uc54c\ub824\uc9c4 \uc720\uc77c\ud55c \uc870\uac01\uac00 \uc6b0\ub974\uce74\uc758 \uc791\ud488\uc73c\ub85c \ucd94\uc815\ub41c\ub2e4. \uc774\ub4e4 \uc870\uac01\uc5d0\uc11c \ubcf4\uc774\ub294 \ud6cc\ub96d\ud55c \uae30\ub2a5\uc740 \ud14c\ub77c\ucf54\ud0c0 \uc870\uac01\uacfc \ub354\ubd88\uc5b4 \uccad\ub3d9\uc870\uac01\uc5d0\uc11c\ub3c4 \uc5ff\ubcf4\uc778\ub2e4. \ub85c\ub9c8 \uac74\uad6d\uc758 \uac74\uc124\uc5d0 \uc5bd\ud78c <\uc5b4\ubbf8 \uc2b9\ub0e5\uc774>(\ub85c\ub9c8 \ucf58\uc138\ub974\ubc14\ud1a0\ub9ac) \ubc0f \uad34\uc218(\u602a\u7378) <\ud0a4\ub9c8\uc774\ub77c>(\ud53c\ub80c\uccb4 \uace0\uace0\ubc15\ubb3c\uad00)\ub294, \uc815\uad50\ud55c \uc8fc\uc870 \uae30\uc220\uacfc \uc0ac\uc2e4\uc801\uc778 \uac15\ub825\ud55c \ud45c\ud604\ub825\uc744 \uac00\uc9c4 \uae30\uc6d0\uc804 5\uc138\uae30\uc758 \uc5d0\ud2b8\ub8e8\ub9ac\uc544 \uccad\ub3d9 \uc870\uac01\uc758 \uac78\uc791\uc774\ub2e4. \ubc15\uc9c4\uac10\uc774 \ub118\uce58\ub294 \uc774\ub4e4 \uc870\uac01\uc5d0 \ub300\ud558\uc5ec, \uc774 \uae30\uac04\uc5d0\ub294 \ub450\ubd80(\u982d\u90e8)\uc5d0\uc11c \ud558\ubd80(\u4e0b\u90e8)\uac00 \ub69c\ub837\uc774 \uac00\ub298\uace0 \uae34 \ubd09\ub0a9(\u5949\u7d0d)\uc758 \uccad\ub3d9 \uc18c\uc0c1(\u5c0f\u50cf)\uc774 \ub9ce\uc774 \uc81c\uc791\ub418\uc5c8\ub2e4. \uadf8 \uac00\ub298\uace0 \uae34 \uc2e0\uccb4\ub294 \uc0ac\uc2e4\uc801\uc778 \ub450\ubd80\ub97c \uc81c\uc678\ud558\uace0, \uadf9\ud788 \ub2e8\uc21c\ud558\uc5ec \uc5d0\ud2b8\ub8e8\ub9ac\uc544 \uc870\uac01 \uc911\uc5d0\uc11c\ub294 \uc774\uc9c8\uc801\uc778 \ud45c\ud604\uc778 \uac83\uc774\ub2e4. \uadf8 \ub54c\ubb38\uc5d0 \uc774\ub4e4 \uc791\uc740 \uc0c1\uc740 \uc5d0\ud2b8\ub8e8\ub9ac\uc544\uc801\uc774\ub77c\uace0 \ud558\uae30\ubcf4\ub2e4 \uc624\ud788\ub824 \ud1a0\ucc29\uc758 \uc774\ud0c8\ub9ac\uc544\uc801 \ud45c\ud604\uc5d0 \uba38\ubb3c\uace0 \uc788\ub2e4\uace0\ub3c4 \uc0dd\uac01\ub41c\ub2e4. \uae30\uc6d0\uc804 4\uc138\uae30 \uc774\ud6c4\uc5d0 \uc81c\uc791\ub41c \ub300\ubd80\ubd84\uc758 \uc791\ud488\uc740 \uc804\uc138\uae30\uc5d0\uc11c \ubcf4\uc5ec\uc900 \uac83 \uac19\uc740 \uc0dd\uae30\uc640 \ub3c5\uc790\uc131\uc744 \uc783\uace0, \ucc28\ucc28 \uadf8\ub9ac\uc2a4\uc640 \ub85c\ub9c8\uc5d0\uc758 \ub3d9\ud654(\u540c\u5316)\uac00 \ud604\uc800\ud574\uc84c\ub2e4. \uc6c0\ube0c\ub9ac\uc544\uc5d0\uc11c \ubc1c\uacac\ub41c <\ub9c8\ub974\uc2a4 \uc0c1>(\ubc14\ud2f0\uce78 \ubbf8\uc220\uad00)\uc774\ub098, \uc0c1\uae30 \ub124\ud2b8\uc5d0\uc11c \ucd9c\ud1a0\ud55c \uc720\uba85\ud55c <\uc5f0\uc124\uc790>(\ud53c\ub80c\uccb4 \uace0\uace0\ubc15\ubb3c\uad00)\ub294 \uc5b4\ub290 \uac83\uc774\ub098 \ud6c4\uae30 \uc5d0\ud2b8\ub8e8\ub9ac\uc544 \uc870\uac01\uc744 \ub300\ud45c\ud558\ub294 \uac83\uc778\ub370, \uc774\uac83\ub4e4\uc5d0\ub294 \ubd84\uba85\ud788 \ud5ec\ub808\ub2c8\uc2a4\ud2f1 \uc591\uc2dd\uc744 \uacc4\uc2b9\ud55c \ub85c\ub9c8 \uc870\uac01\uc758 \uc601\ud5a5\uc774 \ubcf4\uc778\ub2e4. \uadf8\ub9ac\uace0 \ud6c4\uae30\uac00 \ub418\uc5b4, \uadf8 \uc885\uad50\uad00\uc758 \uc774\ud589\uc73c\ub85c \ub9c8\uc2e0(\u9b54\u795e), \uacf5\uc0c1\uc801 \ub3d9\ubb3c, \ube44\uadf9\uc801\uc778 \uc8fc\uc81c\ub97c \ub2e4\ub8ec \ubd80\uc870\uc640 \uc870\uac01\uc774 \ub9ce\uc544\uc84c\ub2e4.", "answer": "\uae30\uc6d0\uc804 6\uc138\uae30 \uc911\uc5fd", "sentence": "\uadf8\ub7ec\ub098 \ud654\uc7a5(\u706b\u846c)\uc758 \uc2b5\uad00\uc740 \uae30\uc6d0\uc804 6\uc138\uae30 \uc911\uc5fd \uc5d0 \ub05d\ub098\uace0, \ub3d9\uc2dc\uc5d0 \ubf08\ud56d\uc544\ub9ac\ub3c4 \ucc28\ucc28 \ubaa8\uc2b5\uc744 \uac10\ucd94\uace0, \uc774\uc5d0 \ub300\uc2e0\ud558\uc5ec \uad00(\u68fa) \uc704\uc5d0 \ubd80\ucc98(\u592b\u59bb)\uac00 \uac00\uae4c\uc774 \ub2e4\uac00 \uc788\ub294 \ubc18\uc640\uc0c1(\u534a\u81e5\u50cf)\uc774 \uc788\ub294 \ud14c\ub77c\ucf54\ud0c0 \uad00\uc774 \uc81c\uc791\ub418\uac8c \ub418\uc5c8\ub2e4.", "paragraph_sentence": "\uc5d0\ud2b8\ub8e8\ub9ac\uc544\uc758 \uc870\uac01\uc740 \uc810\ucc28\uc801\uc73c\ub85c \uae30\uc6d0\uc804 6\uc138\uae30 \ub9d0\uc774 \ub418\uc5b4\uc11c \ub3c5\ub9bd\ub41c \ub300\uc870\uac01\uc774 \uc81c\uc791\ub418\uc5c8\ub2e4. \uadf8 \uc774\uc804\uc758 \uac83\uc740 \ud0a4\uc6b0\uc2dc \ubc16\uc758 \ubd84\ubb18\uc5d0\uc11c \ubc1c\uacac\ub41c \ucd08\uae30\uc758 \ud14c\ub77c\ucf54\ud0c0\uc758 \ubf08\ud56d\uc544\ub9ac\uc5d0\uc11c \ubc1c\uacac\ub41c\ub2e4. \ucd08\uae30\uc758 \ubf08\ud56d\uc544\ub9ac\uc5d0\ub294 \ub69c\uaed1 \uc704\uc5d0 \uc791\uc740 \uc0ac\ub78c\uc774\ub098 \ub3d9\ubb3c\uc758 \uc0c1(\u50cf)\uc744 \ub9cc\ub4e0 \uac83\uc744 \ubcfc \uc218 \uc788\ub294\ub370, \uae30\uc6d0\uc804 7\uc138\uae30\uacbd\uc774 \ub418\uba74 \ubf08\ud56d\uc774\ub9ac \uc804\uccb4\uac00 \ud558\ub098\uc758 \uc778\uccb4 \ud615\uc0c1\uc774 \ub418\uc5c8\ub2e4. \uadf8\ub7ec\ub098 \ud654\uc7a5(\u706b\u846c)\uc758 \uc2b5\uad00\uc740 \uae30\uc6d0\uc804 6\uc138\uae30 \uc911\uc5fd \uc5d0 \ub05d\ub098\uace0, \ub3d9\uc2dc\uc5d0 \ubf08\ud56d\uc544\ub9ac\ub3c4 \ucc28\ucc28 \ubaa8\uc2b5\uc744 \uac10\ucd94\uace0, \uc774\uc5d0 \ub300\uc2e0\ud558\uc5ec \uad00(\u68fa) \uc704\uc5d0 \ubd80\ucc98(\u592b\u59bb)\uac00 \uac00\uae4c\uc774 \ub2e4\uac00 \uc788\ub294 \ubc18\uc640\uc0c1(\u534a\u81e5\u50cf)\uc774 \uc788\ub294 \ud14c\ub77c\ucf54\ud0c0 \uad00\uc774 \uc81c\uc791\ub418\uac8c \ub418\uc5c8\ub2e4. \ud604\uc7ac \ub8e8\ube0c\ub974 \ubbf8\uc220\uad00\uacfc \ub85c\ub9c8\uc758 \ube4c\ub77c\uc9c0\uc6b8\ub9ac\uc544 \ubbf8\uc220\uad00\uc5d0 \uc788\ub294 \uccb4\ub974\ubca0\ud2b8\ub9ac\uc5d0\uc11c \ucd9c\ud1a0\ud55c \ud14c\ub77c\ucf54\ud0c0 \uad00\uc758 <\ubd80\ucc98\uc758 \ubc18\uc640\uc0c1>\uc740 \uac00\uc7a5 \ud6cc \ub96d\ud55c \ubcf4\uae30\uc778\ub370, \uc81c\uc791 \uc5f0\ub300\ub294 \uc5b4\ub290 \uac83\uc774\ub098 \uae30\uc6d0\uc804 520\ub144\uacbd\uc774\ub77c\uace0 \ucd94\uc815\ub41c\ub2e4. \ub300\uc601 \ubc15\ubb3c\uad00\uc5d0 \uc788\ub294 \ub450\uac1c\uc758 \ud14c\ub77c\ucf54\ud0c0 \uc5ec\uc778 \uc88c\uc0c1(\u5973\u4eba\u5750\u50cf)\uc774\ub098 \ud53c\ub80c\uccb4 \uace0\uace0(\u8003\u53e4) \ubc15\ubb3c\uad00\uc758 \uccad\ub3d9 \uc804\uc0ac\uc0c1(\u6230\u58eb\u50cf)\uc774\ub098 \uc5ec\uc778\uc0c1\uc740 \uc5d0\ud2b8\ub8e8\ub9ac\uc544 \uc870\uac01\uc758 \ucd08\uae30\uc758 \uac83\uc73c\ub85c \uc88b\uc740 \uc608\ub2e4. \uc544\ub974\uce74\uc774\ud06c \uc591\uc2dd\uc774\ub77c \ubd88\ub9ac\ub294 \uae30\uc6d0\uc804 6\uc138\uae30\uc5d0\uc11c \uae30\uc6d0\uc804 5\uc138\uae30 \uc911\uc5fd\uae4c\uc9c0\uc758 \uc2dc\uae30\ub294 \uc5d0\ud2b8\ub8e8\ub9ac\uc544 \uc870\uac01\uc758 \ucd5c\uc131\uae30\ub85c\uc11c, \ud55c\ud3b8\uc73c\ub85c\ub294 \uadf8\ub9ac\uc2a4 \ubbf8\uc220\uc758 \uc601\ud5a5\uc744 \ubc1b\uc73c\uba74\uc11c, \ub610 \ub2e4\ub978 \ud55c\ud3b8\uc73c\ub85c\ub294 \uadf8 \ud45c\ud604\uacfc \uc591\uc2dd\uc5d0 \uc788\uc5b4\uc11c \uc5d0\ud2b8\ub8e8\ub9ac\uc544 \uc870\uac01\uc758 \ub3c5\uc790\uc131\uc744 \ubc1c\uc804\uc2dc\ucf30\ub2e4. \ubca0\uc774\uc624\uc758 \ubbf8\ub124\ub974\ubc14 \uc2e0\uc804\uc758 \uc9c0\ubd95\uc744 \uafb8\ubbfc <\uc544\ud3f4\ub860\uacfc \ud5e4\ub77c\ud074\ub808\uc2a4\uc758 \uad70\uc0c1(\u7fa4\u50cf)>(\ube4c\ub77c\uc9c0\uc6b8\ub9ac\uc544\uc758 \ubbf8\uc220\uad00), <\uc11c \uc788\ub294 \uc18c\ub140\uc0c1(\u5c11\u5973\u50cf)>(\ucf54\ud39c\ud558\uac90 \ub2c8\uce74\ub8e8\ub974\uc2a4\ubca0\ub85c\uadf8 \uc9c4\uc5f4\uad00)\uc740 \uad00(\u68fa)\uc758 \uc0c1(\u50cf)\uacfc \ub354\ubd88\uc5b4 \uc774 \uae30(\u671f)\uc758 \ud14c\ub77c\ucf54\ud0c0 \uc870\uac01\uc744 \ub300\ud45c\ud558\ub294 \uac78\uc791\uc774\ub2e4. \uadf8 \uc911\uc5d0\uc11c\ub3c4 \uc544\ud3f4\ub860 \uc0c1\uc740 \uc5d0\ud2b8\ub8e8\ub9ac\uc544\uc758 \uc608\uc220\uac00 \uc911\uc5d0\uc11c \uadf8 \uc774\ub984\uc774 \uc54c\ub824\uc9c4 \uc720\uc77c\ud55c \uc870\uac01\uac00 \uc6b0\ub974\uce74\uc758 \uc791\ud488\uc73c\ub85c \ucd94\uc815\ub41c\ub2e4. \uc774\ub4e4 \uc870\uac01\uc5d0\uc11c \ubcf4\uc774\ub294 \ud6cc\ub96d\ud55c \uae30\ub2a5\uc740 \ud14c\ub77c\ucf54\ud0c0 \uc870\uac01\uacfc \ub354\ubd88\uc5b4 \uccad\ub3d9\uc870\uac01\uc5d0\uc11c\ub3c4 \uc5ff\ubcf4\uc778\ub2e4. \ub85c\ub9c8 \uac74\uad6d\uc758 \uac74\uc124\uc5d0 \uc5bd\ud78c <\uc5b4\ubbf8 \uc2b9\ub0e5\uc774>(\ub85c\ub9c8 \ucf58\uc138\ub974\ubc14\ud1a0\ub9ac) \ubc0f \uad34\uc218(\u602a\u7378) <\ud0a4\ub9c8\uc774\ub77c>(\ud53c\ub80c\uccb4 \uace0\uace0\ubc15\ubb3c\uad00)\ub294, \uc815\uad50\ud55c \uc8fc\uc870 \uae30\uc220\uacfc \uc0ac\uc2e4\uc801\uc778 \uac15\ub825\ud55c \ud45c\ud604\ub825\uc744 \uac00\uc9c4 \uae30\uc6d0\uc804 5\uc138\uae30\uc758 \uc5d0\ud2b8\ub8e8\ub9ac\uc544 \uccad\ub3d9 \uc870\uac01\uc758 \uac78\uc791\uc774\ub2e4. \ubc15\uc9c4\uac10\uc774 \ub118\uce58\ub294 \uc774\ub4e4 \uc870\uac01\uc5d0 \ub300\ud558\uc5ec, \uc774 \uae30\uac04\uc5d0\ub294 \ub450\ubd80(\u982d\u90e8)\uc5d0\uc11c \ud558\ubd80(\u4e0b\u90e8)\uac00 \ub69c\ub837\uc774 \uac00\ub298\uace0 \uae34 \ubd09\ub0a9(\u5949\u7d0d)\uc758 \uccad\ub3d9 \uc18c\uc0c1(\u5c0f\u50cf)\uc774 \ub9ce\uc774 \uc81c\uc791\ub418\uc5c8\ub2e4. \uadf8 \uac00\ub298\uace0 \uae34 \uc2e0\uccb4\ub294 \uc0ac\uc2e4\uc801\uc778 \ub450\ubd80\ub97c \uc81c\uc678\ud558\uace0, \uadf9\ud788 \ub2e8\uc21c\ud558\uc5ec \uc5d0\ud2b8\ub8e8\ub9ac\uc544 \uc870\uac01 \uc911\uc5d0\uc11c\ub294 \uc774\uc9c8\uc801\uc778 \ud45c\ud604\uc778 \uac83\uc774\ub2e4. \uadf8 \ub54c\ubb38\uc5d0 \uc774\ub4e4 \uc791\uc740 \uc0c1\uc740 \uc5d0\ud2b8\ub8e8\ub9ac\uc544\uc801\uc774\ub77c\uace0 \ud558\uae30\ubcf4\ub2e4 \uc624\ud788\ub824 \ud1a0\ucc29\uc758 \uc774\ud0c8\ub9ac\uc544\uc801 \ud45c\ud604\uc5d0 \uba38\ubb3c\uace0 \uc788\ub2e4\uace0\ub3c4 \uc0dd\uac01\ub41c\ub2e4. \uae30\uc6d0\uc804 4\uc138\uae30 \uc774\ud6c4\uc5d0 \uc81c\uc791\ub41c \ub300\ubd80\ubd84\uc758 \uc791\ud488\uc740 \uc804\uc138\uae30\uc5d0\uc11c \ubcf4\uc5ec\uc900 \uac83 \uac19\uc740 \uc0dd\uae30\uc640 \ub3c5\uc790\uc131\uc744 \uc783\uace0, \ucc28\ucc28 \uadf8\ub9ac\uc2a4\uc640 \ub85c\ub9c8\uc5d0\uc758 \ub3d9\ud654(\u540c\u5316)\uac00 \ud604\uc800\ud574\uc84c\ub2e4. \uc6c0\ube0c\ub9ac\uc544\uc5d0\uc11c \ubc1c\uacac\ub41c <\ub9c8\ub974\uc2a4 \uc0c1>(\ubc14\ud2f0\uce78 \ubbf8\uc220\uad00)\uc774\ub098, \uc0c1\uae30 \ub124\ud2b8\uc5d0\uc11c \ucd9c\ud1a0\ud55c \uc720\uba85\ud55c <\uc5f0\uc124\uc790>(\ud53c\ub80c\uccb4 \uace0\uace0\ubc15\ubb3c\uad00)\ub294 \uc5b4\ub290 \uac83\uc774\ub098 \ud6c4\uae30 \uc5d0\ud2b8\ub8e8\ub9ac\uc544 \uc870\uac01\uc744 \ub300\ud45c\ud558\ub294 \uac83\uc778\ub370, \uc774\uac83\ub4e4\uc5d0\ub294 \ubd84\uba85\ud788 \ud5ec\ub808\ub2c8\uc2a4\ud2f1 \uc591\uc2dd\uc744 \uacc4\uc2b9\ud55c \ub85c\ub9c8 \uc870\uac01\uc758 \uc601\ud5a5\uc774 \ubcf4\uc778\ub2e4. \uadf8\ub9ac\uace0 \ud6c4\uae30\uac00 \ub418\uc5b4, \uadf8 \uc885\uad50\uad00\uc758 \uc774\ud589\uc73c\ub85c \ub9c8\uc2e0(\u9b54\u795e), \uacf5\uc0c1\uc801 \ub3d9\ubb3c, \ube44\uadf9\uc801\uc778 \uc8fc\uc81c\ub97c \ub2e4\ub8ec \ubd80\uc870\uc640 \uc870\uac01\uc774 \ub9ce\uc544\uc84c\ub2e4.", "paragraph_answer": "\uc5d0\ud2b8\ub8e8\ub9ac\uc544\uc758 \uc870\uac01\uc740 \uc810\ucc28\uc801\uc73c\ub85c \uae30\uc6d0\uc804 6\uc138\uae30 \ub9d0\uc774 \ub418\uc5b4\uc11c \ub3c5\ub9bd\ub41c \ub300\uc870\uac01\uc774 \uc81c\uc791\ub418\uc5c8\ub2e4. \uadf8 \uc774\uc804\uc758 \uac83\uc740 \ud0a4\uc6b0\uc2dc \ubc16\uc758 \ubd84\ubb18\uc5d0\uc11c \ubc1c\uacac\ub41c \ucd08\uae30\uc758 \ud14c\ub77c\ucf54\ud0c0\uc758 \ubf08\ud56d\uc544\ub9ac\uc5d0\uc11c \ubc1c\uacac\ub41c\ub2e4. \ucd08\uae30\uc758 \ubf08\ud56d\uc544\ub9ac\uc5d0\ub294 \ub69c\uaed1 \uc704\uc5d0 \uc791\uc740 \uc0ac\ub78c\uc774\ub098 \ub3d9\ubb3c\uc758 \uc0c1(\u50cf)\uc744 \ub9cc\ub4e0 \uac83\uc744 \ubcfc \uc218 \uc788\ub294\ub370, \uae30\uc6d0\uc804 7\uc138\uae30\uacbd\uc774 \ub418\uba74 \ubf08\ud56d\uc774\ub9ac \uc804\uccb4\uac00 \ud558\ub098\uc758 \uc778\uccb4 \ud615\uc0c1\uc774 \ub418\uc5c8\ub2e4. \uadf8\ub7ec\ub098 \ud654\uc7a5(\u706b\u846c)\uc758 \uc2b5\uad00\uc740 \uae30\uc6d0\uc804 6\uc138\uae30 \uc911\uc5fd \uc5d0 \ub05d\ub098\uace0, \ub3d9\uc2dc\uc5d0 \ubf08\ud56d\uc544\ub9ac\ub3c4 \ucc28\ucc28 \ubaa8\uc2b5\uc744 \uac10\ucd94\uace0, \uc774\uc5d0 \ub300\uc2e0\ud558\uc5ec \uad00(\u68fa) \uc704\uc5d0 \ubd80\ucc98(\u592b\u59bb)\uac00 \uac00\uae4c\uc774 \ub2e4\uac00 \uc788\ub294 \ubc18\uc640\uc0c1(\u534a\u81e5\u50cf)\uc774 \uc788\ub294 \ud14c\ub77c\ucf54\ud0c0 \uad00\uc774 \uc81c\uc791\ub418\uac8c \ub418\uc5c8\ub2e4. \ud604\uc7ac \ub8e8\ube0c\ub974 \ubbf8\uc220\uad00\uacfc \ub85c\ub9c8\uc758 \ube4c\ub77c\uc9c0\uc6b8\ub9ac\uc544 \ubbf8\uc220\uad00\uc5d0 \uc788\ub294 \uccb4\ub974\ubca0\ud2b8\ub9ac\uc5d0\uc11c \ucd9c\ud1a0\ud55c \ud14c\ub77c\ucf54\ud0c0 \uad00\uc758 <\ubd80\ucc98\uc758 \ubc18\uc640\uc0c1>\uc740 \uac00\uc7a5 \ud6cc \ub96d\ud55c \ubcf4\uae30\uc778\ub370, \uc81c\uc791 \uc5f0\ub300\ub294 \uc5b4\ub290 \uac83\uc774\ub098 \uae30\uc6d0\uc804 520\ub144\uacbd\uc774\ub77c\uace0 \ucd94\uc815\ub41c\ub2e4. \ub300\uc601 \ubc15\ubb3c\uad00\uc5d0 \uc788\ub294 \ub450\uac1c\uc758 \ud14c\ub77c\ucf54\ud0c0 \uc5ec\uc778 \uc88c\uc0c1(\u5973\u4eba\u5750\u50cf)\uc774\ub098 \ud53c\ub80c\uccb4 \uace0\uace0(\u8003\u53e4) \ubc15\ubb3c\uad00\uc758 \uccad\ub3d9 \uc804\uc0ac\uc0c1(\u6230\u58eb\u50cf)\uc774\ub098 \uc5ec\uc778\uc0c1\uc740 \uc5d0\ud2b8\ub8e8\ub9ac\uc544 \uc870\uac01\uc758 \ucd08\uae30\uc758 \uac83\uc73c\ub85c \uc88b\uc740 \uc608\ub2e4. \uc544\ub974\uce74\uc774\ud06c \uc591\uc2dd\uc774\ub77c \ubd88\ub9ac\ub294 \uae30\uc6d0\uc804 6\uc138\uae30\uc5d0\uc11c \uae30\uc6d0\uc804 5\uc138\uae30 \uc911\uc5fd\uae4c\uc9c0\uc758 \uc2dc\uae30\ub294 \uc5d0\ud2b8\ub8e8\ub9ac\uc544 \uc870\uac01\uc758 \ucd5c\uc131\uae30\ub85c\uc11c, \ud55c\ud3b8\uc73c\ub85c\ub294 \uadf8\ub9ac\uc2a4 \ubbf8\uc220\uc758 \uc601\ud5a5\uc744 \ubc1b\uc73c\uba74\uc11c, \ub610 \ub2e4\ub978 \ud55c\ud3b8\uc73c\ub85c\ub294 \uadf8 \ud45c\ud604\uacfc \uc591\uc2dd\uc5d0 \uc788\uc5b4\uc11c \uc5d0\ud2b8\ub8e8\ub9ac\uc544 \uc870\uac01\uc758 \ub3c5\uc790\uc131\uc744 \ubc1c\uc804\uc2dc\ucf30\ub2e4. \ubca0\uc774\uc624\uc758 \ubbf8\ub124\ub974\ubc14 \uc2e0\uc804\uc758 \uc9c0\ubd95\uc744 \uafb8\ubbfc <\uc544\ud3f4\ub860\uacfc \ud5e4\ub77c\ud074\ub808\uc2a4\uc758 \uad70\uc0c1(\u7fa4\u50cf)>(\ube4c\ub77c\uc9c0\uc6b8\ub9ac\uc544\uc758 \ubbf8\uc220\uad00), <\uc11c \uc788\ub294 \uc18c\ub140\uc0c1(\u5c11\u5973\u50cf)>(\ucf54\ud39c\ud558\uac90 \ub2c8\uce74\ub8e8\ub974\uc2a4\ubca0\ub85c\uadf8 \uc9c4\uc5f4\uad00)\uc740 \uad00(\u68fa)\uc758 \uc0c1(\u50cf)\uacfc \ub354\ubd88\uc5b4 \uc774 \uae30(\u671f)\uc758 \ud14c\ub77c\ucf54\ud0c0 \uc870\uac01\uc744 \ub300\ud45c\ud558\ub294 \uac78\uc791\uc774\ub2e4. \uadf8 \uc911\uc5d0\uc11c\ub3c4 \uc544\ud3f4\ub860 \uc0c1\uc740 \uc5d0\ud2b8\ub8e8\ub9ac\uc544\uc758 \uc608\uc220\uac00 \uc911\uc5d0\uc11c \uadf8 \uc774\ub984\uc774 \uc54c\ub824\uc9c4 \uc720\uc77c\ud55c \uc870\uac01\uac00 \uc6b0\ub974\uce74\uc758 \uc791\ud488\uc73c\ub85c \ucd94\uc815\ub41c\ub2e4. \uc774\ub4e4 \uc870\uac01\uc5d0\uc11c \ubcf4\uc774\ub294 \ud6cc\ub96d\ud55c \uae30\ub2a5\uc740 \ud14c\ub77c\ucf54\ud0c0 \uc870\uac01\uacfc \ub354\ubd88\uc5b4 \uccad\ub3d9\uc870\uac01\uc5d0\uc11c\ub3c4 \uc5ff\ubcf4\uc778\ub2e4. \ub85c\ub9c8 \uac74\uad6d\uc758 \uac74\uc124\uc5d0 \uc5bd\ud78c <\uc5b4\ubbf8 \uc2b9\ub0e5\uc774>(\ub85c\ub9c8 \ucf58\uc138\ub974\ubc14\ud1a0\ub9ac) \ubc0f \uad34\uc218(\u602a\u7378) <\ud0a4\ub9c8\uc774\ub77c>(\ud53c\ub80c\uccb4 \uace0\uace0\ubc15\ubb3c\uad00)\ub294, \uc815\uad50\ud55c \uc8fc\uc870 \uae30\uc220\uacfc \uc0ac\uc2e4\uc801\uc778 \uac15\ub825\ud55c \ud45c\ud604\ub825\uc744 \uac00\uc9c4 \uae30\uc6d0\uc804 5\uc138\uae30\uc758 \uc5d0\ud2b8\ub8e8\ub9ac\uc544 \uccad\ub3d9 \uc870\uac01\uc758 \uac78\uc791\uc774\ub2e4. \ubc15\uc9c4\uac10\uc774 \ub118\uce58\ub294 \uc774\ub4e4 \uc870\uac01\uc5d0 \ub300\ud558\uc5ec, \uc774 \uae30\uac04\uc5d0\ub294 \ub450\ubd80(\u982d\u90e8)\uc5d0\uc11c \ud558\ubd80(\u4e0b\u90e8)\uac00 \ub69c\ub837\uc774 \uac00\ub298\uace0 \uae34 \ubd09\ub0a9(\u5949\u7d0d)\uc758 \uccad\ub3d9 \uc18c\uc0c1(\u5c0f\u50cf)\uc774 \ub9ce\uc774 \uc81c\uc791\ub418\uc5c8\ub2e4. \uadf8 \uac00\ub298\uace0 \uae34 \uc2e0\uccb4\ub294 \uc0ac\uc2e4\uc801\uc778 \ub450\ubd80\ub97c \uc81c\uc678\ud558\uace0, \uadf9\ud788 \ub2e8\uc21c\ud558\uc5ec \uc5d0\ud2b8\ub8e8\ub9ac\uc544 \uc870\uac01 \uc911\uc5d0\uc11c\ub294 \uc774\uc9c8\uc801\uc778 \ud45c\ud604\uc778 \uac83\uc774\ub2e4. \uadf8 \ub54c\ubb38\uc5d0 \uc774\ub4e4 \uc791\uc740 \uc0c1\uc740 \uc5d0\ud2b8\ub8e8\ub9ac\uc544\uc801\uc774\ub77c\uace0 \ud558\uae30\ubcf4\ub2e4 \uc624\ud788\ub824 \ud1a0\ucc29\uc758 \uc774\ud0c8\ub9ac\uc544\uc801 \ud45c\ud604\uc5d0 \uba38\ubb3c\uace0 \uc788\ub2e4\uace0\ub3c4 \uc0dd\uac01\ub41c\ub2e4. \uae30\uc6d0\uc804 4\uc138\uae30 \uc774\ud6c4\uc5d0 \uc81c\uc791\ub41c \ub300\ubd80\ubd84\uc758 \uc791\ud488\uc740 \uc804\uc138\uae30\uc5d0\uc11c \ubcf4\uc5ec\uc900 \uac83 \uac19\uc740 \uc0dd\uae30\uc640 \ub3c5\uc790\uc131\uc744 \uc783\uace0, \ucc28\ucc28 \uadf8\ub9ac\uc2a4\uc640 \ub85c\ub9c8\uc5d0\uc758 \ub3d9\ud654(\u540c\u5316)\uac00 \ud604\uc800\ud574\uc84c\ub2e4. \uc6c0\ube0c\ub9ac\uc544\uc5d0\uc11c \ubc1c\uacac\ub41c <\ub9c8\ub974\uc2a4 \uc0c1>(\ubc14\ud2f0\uce78 \ubbf8\uc220\uad00)\uc774\ub098, \uc0c1\uae30 \ub124\ud2b8\uc5d0\uc11c \ucd9c\ud1a0\ud55c \uc720\uba85\ud55c <\uc5f0\uc124\uc790>(\ud53c\ub80c\uccb4 \uace0\uace0\ubc15\ubb3c\uad00)\ub294 \uc5b4\ub290 \uac83\uc774\ub098 \ud6c4\uae30 \uc5d0\ud2b8\ub8e8\ub9ac\uc544 \uc870\uac01\uc744 \ub300\ud45c\ud558\ub294 \uac83\uc778\ub370, \uc774\uac83\ub4e4\uc5d0\ub294 \ubd84\uba85\ud788 \ud5ec\ub808\ub2c8\uc2a4\ud2f1 \uc591\uc2dd\uc744 \uacc4\uc2b9\ud55c \ub85c\ub9c8 \uc870\uac01\uc758 \uc601\ud5a5\uc774 \ubcf4\uc778\ub2e4. \uadf8\ub9ac\uace0 \ud6c4\uae30\uac00 \ub418\uc5b4, \uadf8 \uc885\uad50\uad00\uc758 \uc774\ud589\uc73c\ub85c \ub9c8\uc2e0(\u9b54\u795e), \uacf5\uc0c1\uc801 \ub3d9\ubb3c, \ube44\uadf9\uc801\uc778 \uc8fc\uc81c\ub97c \ub2e4\ub8ec \ubd80\uc870\uc640 \uc870\uac01\uc774 \ub9ce\uc544\uc84c\ub2e4.", "sentence_answer": "\uadf8\ub7ec\ub098 \ud654\uc7a5(\u706b\u846c)\uc758 \uc2b5\uad00\uc740 \uae30\uc6d0\uc804 6\uc138\uae30 \uc911\uc5fd \uc5d0 \ub05d\ub098\uace0, \ub3d9\uc2dc\uc5d0 \ubf08\ud56d\uc544\ub9ac\ub3c4 \ucc28\ucc28 \ubaa8\uc2b5\uc744 \uac10\ucd94\uace0, \uc774\uc5d0 \ub300\uc2e0\ud558\uc5ec \uad00(\u68fa) \uc704\uc5d0 \ubd80\ucc98(\u592b\u59bb)\uac00 \uac00\uae4c\uc774 \ub2e4\uac00 \uc788\ub294 \ubc18\uc640\uc0c1(\u534a\u81e5\u50cf)\uc774 \uc788\ub294 \ud14c\ub77c\ucf54\ud0c0 \uad00\uc774 \uc81c\uc791\ub418\uac8c \ub418\uc5c8\ub2e4.", "paragraph_id": "6586884-1-1", "paragraph_question": "question: \ud654\uc7a5\uc758 \uc2b5\uad00\uc774 \ub05d\ub09c \ub54c\ub294?, context: \uc5d0\ud2b8\ub8e8\ub9ac\uc544\uc758 \uc870\uac01\uc740 \uc810\ucc28\uc801\uc73c\ub85c \uae30\uc6d0\uc804 6\uc138\uae30 \ub9d0\uc774 \ub418\uc5b4\uc11c \ub3c5\ub9bd\ub41c \ub300\uc870\uac01\uc774 \uc81c\uc791\ub418\uc5c8\ub2e4. \uadf8 \uc774\uc804\uc758 \uac83\uc740 \ud0a4\uc6b0\uc2dc \ubc16\uc758 \ubd84\ubb18\uc5d0\uc11c \ubc1c\uacac\ub41c \ucd08\uae30\uc758 \ud14c\ub77c\ucf54\ud0c0\uc758 \ubf08\ud56d\uc544\ub9ac\uc5d0\uc11c \ubc1c\uacac\ub41c\ub2e4. \ucd08\uae30\uc758 \ubf08\ud56d\uc544\ub9ac\uc5d0\ub294 \ub69c\uaed1 \uc704\uc5d0 \uc791\uc740 \uc0ac\ub78c\uc774\ub098 \ub3d9\ubb3c\uc758 \uc0c1(\u50cf)\uc744 \ub9cc\ub4e0 \uac83\uc744 \ubcfc \uc218 \uc788\ub294\ub370, \uae30\uc6d0\uc804 7\uc138\uae30\uacbd\uc774 \ub418\uba74 \ubf08\ud56d\uc774\ub9ac \uc804\uccb4\uac00 \ud558\ub098\uc758 \uc778\uccb4 \ud615\uc0c1\uc774 \ub418\uc5c8\ub2e4. \uadf8\ub7ec\ub098 \ud654\uc7a5(\u706b\u846c)\uc758 \uc2b5\uad00\uc740 \uae30\uc6d0\uc804 6\uc138\uae30 \uc911\uc5fd\uc5d0 \ub05d\ub098\uace0, \ub3d9\uc2dc\uc5d0 \ubf08\ud56d\uc544\ub9ac\ub3c4 \ucc28\ucc28 \ubaa8\uc2b5\uc744 \uac10\ucd94\uace0, \uc774\uc5d0 \ub300\uc2e0\ud558\uc5ec \uad00(\u68fa) \uc704\uc5d0 \ubd80\ucc98(\u592b\u59bb)\uac00 \uac00\uae4c\uc774 \ub2e4\uac00 \uc788\ub294 \ubc18\uc640\uc0c1(\u534a\u81e5\u50cf)\uc774 \uc788\ub294 \ud14c\ub77c\ucf54\ud0c0 \uad00\uc774 \uc81c\uc791\ub418\uac8c \ub418\uc5c8\ub2e4. \ud604\uc7ac \ub8e8\ube0c\ub974 \ubbf8\uc220\uad00\uacfc \ub85c\ub9c8\uc758 \ube4c\ub77c\uc9c0\uc6b8\ub9ac\uc544 \ubbf8\uc220\uad00\uc5d0 \uc788\ub294 \uccb4\ub974\ubca0\ud2b8\ub9ac\uc5d0\uc11c \ucd9c\ud1a0\ud55c \ud14c\ub77c\ucf54\ud0c0 \uad00\uc758 <\ubd80\ucc98\uc758 \ubc18\uc640\uc0c1>\uc740 \uac00\uc7a5 \ud6cc \ub96d\ud55c \ubcf4\uae30\uc778\ub370, \uc81c\uc791 \uc5f0\ub300\ub294 \uc5b4\ub290 \uac83\uc774\ub098 \uae30\uc6d0\uc804 520\ub144\uacbd\uc774\ub77c\uace0 \ucd94\uc815\ub41c\ub2e4. \ub300\uc601 \ubc15\ubb3c\uad00\uc5d0 \uc788\ub294 \ub450\uac1c\uc758 \ud14c\ub77c\ucf54\ud0c0 \uc5ec\uc778 \uc88c\uc0c1(\u5973\u4eba\u5750\u50cf)\uc774\ub098 \ud53c\ub80c\uccb4 \uace0\uace0(\u8003\u53e4) \ubc15\ubb3c\uad00\uc758 \uccad\ub3d9 \uc804\uc0ac\uc0c1(\u6230\u58eb\u50cf)\uc774\ub098 \uc5ec\uc778\uc0c1\uc740 \uc5d0\ud2b8\ub8e8\ub9ac\uc544 \uc870\uac01\uc758 \ucd08\uae30\uc758 \uac83\uc73c\ub85c \uc88b\uc740 \uc608\ub2e4. \uc544\ub974\uce74\uc774\ud06c \uc591\uc2dd\uc774\ub77c \ubd88\ub9ac\ub294 \uae30\uc6d0\uc804 6\uc138\uae30\uc5d0\uc11c \uae30\uc6d0\uc804 5\uc138\uae30 \uc911\uc5fd\uae4c\uc9c0\uc758 \uc2dc\uae30\ub294 \uc5d0\ud2b8\ub8e8\ub9ac\uc544 \uc870\uac01\uc758 \ucd5c\uc131\uae30\ub85c\uc11c, \ud55c\ud3b8\uc73c\ub85c\ub294 \uadf8\ub9ac\uc2a4 \ubbf8\uc220\uc758 \uc601\ud5a5\uc744 \ubc1b\uc73c\uba74\uc11c, \ub610 \ub2e4\ub978 \ud55c\ud3b8\uc73c\ub85c\ub294 \uadf8 \ud45c\ud604\uacfc \uc591\uc2dd\uc5d0 \uc788\uc5b4\uc11c \uc5d0\ud2b8\ub8e8\ub9ac\uc544 \uc870\uac01\uc758 \ub3c5\uc790\uc131\uc744 \ubc1c\uc804\uc2dc\ucf30\ub2e4. \ubca0\uc774\uc624\uc758 \ubbf8\ub124\ub974\ubc14 \uc2e0\uc804\uc758 \uc9c0\ubd95\uc744 \uafb8\ubbfc <\uc544\ud3f4\ub860\uacfc \ud5e4\ub77c\ud074\ub808\uc2a4\uc758 \uad70\uc0c1(\u7fa4\u50cf)>(\ube4c\ub77c\uc9c0\uc6b8\ub9ac\uc544\uc758 \ubbf8\uc220\uad00), <\uc11c \uc788\ub294 \uc18c\ub140\uc0c1(\u5c11\u5973\u50cf)>(\ucf54\ud39c\ud558\uac90 \ub2c8\uce74\ub8e8\ub974\uc2a4\ubca0\ub85c\uadf8 \uc9c4\uc5f4\uad00)\uc740 \uad00(\u68fa)\uc758 \uc0c1(\u50cf)\uacfc \ub354\ubd88\uc5b4 \uc774 \uae30(\u671f)\uc758 \ud14c\ub77c\ucf54\ud0c0 \uc870\uac01\uc744 \ub300\ud45c\ud558\ub294 \uac78\uc791\uc774\ub2e4. \uadf8 \uc911\uc5d0\uc11c\ub3c4 \uc544\ud3f4\ub860 \uc0c1\uc740 \uc5d0\ud2b8\ub8e8\ub9ac\uc544\uc758 \uc608\uc220\uac00 \uc911\uc5d0\uc11c \uadf8 \uc774\ub984\uc774 \uc54c\ub824\uc9c4 \uc720\uc77c\ud55c \uc870\uac01\uac00 \uc6b0\ub974\uce74\uc758 \uc791\ud488\uc73c\ub85c \ucd94\uc815\ub41c\ub2e4. \uc774\ub4e4 \uc870\uac01\uc5d0\uc11c \ubcf4\uc774\ub294 \ud6cc\ub96d\ud55c \uae30\ub2a5\uc740 \ud14c\ub77c\ucf54\ud0c0 \uc870\uac01\uacfc \ub354\ubd88\uc5b4 \uccad\ub3d9\uc870\uac01\uc5d0\uc11c\ub3c4 \uc5ff\ubcf4\uc778\ub2e4. \ub85c\ub9c8 \uac74\uad6d\uc758 \uac74\uc124\uc5d0 \uc5bd\ud78c <\uc5b4\ubbf8 \uc2b9\ub0e5\uc774>(\ub85c\ub9c8 \ucf58\uc138\ub974\ubc14\ud1a0\ub9ac) \ubc0f \uad34\uc218(\u602a\u7378) <\ud0a4\ub9c8\uc774\ub77c>(\ud53c\ub80c\uccb4 \uace0\uace0\ubc15\ubb3c\uad00)\ub294, \uc815\uad50\ud55c \uc8fc\uc870 \uae30\uc220\uacfc \uc0ac\uc2e4\uc801\uc778 \uac15\ub825\ud55c \ud45c\ud604\ub825\uc744 \uac00\uc9c4 \uae30\uc6d0\uc804 5\uc138\uae30\uc758 \uc5d0\ud2b8\ub8e8\ub9ac\uc544 \uccad\ub3d9 \uc870\uac01\uc758 \uac78\uc791\uc774\ub2e4. \ubc15\uc9c4\uac10\uc774 \ub118\uce58\ub294 \uc774\ub4e4 \uc870\uac01\uc5d0 \ub300\ud558\uc5ec, \uc774 \uae30\uac04\uc5d0\ub294 \ub450\ubd80(\u982d\u90e8)\uc5d0\uc11c \ud558\ubd80(\u4e0b\u90e8)\uac00 \ub69c\ub837\uc774 \uac00\ub298\uace0 \uae34 \ubd09\ub0a9(\u5949\u7d0d)\uc758 \uccad\ub3d9 \uc18c\uc0c1(\u5c0f\u50cf)\uc774 \ub9ce\uc774 \uc81c\uc791\ub418\uc5c8\ub2e4. \uadf8 \uac00\ub298\uace0 \uae34 \uc2e0\uccb4\ub294 \uc0ac\uc2e4\uc801\uc778 \ub450\ubd80\ub97c \uc81c\uc678\ud558\uace0, \uadf9\ud788 \ub2e8\uc21c\ud558\uc5ec \uc5d0\ud2b8\ub8e8\ub9ac\uc544 \uc870\uac01 \uc911\uc5d0\uc11c\ub294 \uc774\uc9c8\uc801\uc778 \ud45c\ud604\uc778 \uac83\uc774\ub2e4. \uadf8 \ub54c\ubb38\uc5d0 \uc774\ub4e4 \uc791\uc740 \uc0c1\uc740 \uc5d0\ud2b8\ub8e8\ub9ac\uc544\uc801\uc774\ub77c\uace0 \ud558\uae30\ubcf4\ub2e4 \uc624\ud788\ub824 \ud1a0\ucc29\uc758 \uc774\ud0c8\ub9ac\uc544\uc801 \ud45c\ud604\uc5d0 \uba38\ubb3c\uace0 \uc788\ub2e4\uace0\ub3c4 \uc0dd\uac01\ub41c\ub2e4. \uae30\uc6d0\uc804 4\uc138\uae30 \uc774\ud6c4\uc5d0 \uc81c\uc791\ub41c \ub300\ubd80\ubd84\uc758 \uc791\ud488\uc740 \uc804\uc138\uae30\uc5d0\uc11c \ubcf4\uc5ec\uc900 \uac83 \uac19\uc740 \uc0dd\uae30\uc640 \ub3c5\uc790\uc131\uc744 \uc783\uace0, \ucc28\ucc28 \uadf8\ub9ac\uc2a4\uc640 \ub85c\ub9c8\uc5d0\uc758 \ub3d9\ud654(\u540c\u5316)\uac00 \ud604\uc800\ud574\uc84c\ub2e4. \uc6c0\ube0c\ub9ac\uc544\uc5d0\uc11c \ubc1c\uacac\ub41c <\ub9c8\ub974\uc2a4 \uc0c1>(\ubc14\ud2f0\uce78 \ubbf8\uc220\uad00)\uc774\ub098, \uc0c1\uae30 \ub124\ud2b8\uc5d0\uc11c \ucd9c\ud1a0\ud55c \uc720\uba85\ud55c <\uc5f0\uc124\uc790>(\ud53c\ub80c\uccb4 \uace0\uace0\ubc15\ubb3c\uad00)\ub294 \uc5b4\ub290 \uac83\uc774\ub098 \ud6c4\uae30 \uc5d0\ud2b8\ub8e8\ub9ac\uc544 \uc870\uac01\uc744 \ub300\ud45c\ud558\ub294 \uac83\uc778\ub370, \uc774\uac83\ub4e4\uc5d0\ub294 \ubd84\uba85\ud788 \ud5ec\ub808\ub2c8\uc2a4\ud2f1 \uc591\uc2dd\uc744 \uacc4\uc2b9\ud55c \ub85c\ub9c8 \uc870\uac01\uc758 \uc601\ud5a5\uc774 \ubcf4\uc778\ub2e4. \uadf8\ub9ac\uace0 \ud6c4\uae30\uac00 \ub418\uc5b4, \uadf8 \uc885\uad50\uad00\uc758 \uc774\ud589\uc73c\ub85c \ub9c8\uc2e0(\u9b54\u795e), \uacf5\uc0c1\uc801 \ub3d9\ubb3c, \ube44\uadf9\uc801\uc778 \uc8fc\uc81c\ub97c \ub2e4\ub8ec \ubd80\uc870\uc640 \uc870\uac01\uc774 \ub9ce\uc544\uc84c\ub2e4."} {"question": "2017\ub144 \uacbd\uc81c\uacc4\uc5d0\uc11c \ud65c\uc57d\ud558\ub294 \ud559\uc0ac\uc7a5\uad50 \ucd9c\uc2e0\uc758 \uc218\ub294 \ub300\ub7b5 \uba87 \uba85\uc778\uac00?", "paragraph": "\ud559\uc0ac\uc7a5\uad50\ub294 \ud559\uad70\uc7a5\uad50\uc640 \ube44\uc2b7\ud558\uac8c \ub300\ubd80\ubd84 \ub2e8\uae30 \uc758\ubb34\ubcf5\ubb34 \ud6c4 \uc804\uc5ed\ud558\uc5ec \uc0ac\ud68c \uac01\uacc4\uac01\uce35\uc5d0\uc11c \ud65c\uc57d\ud558\uace0 \uc788\ub2e4. \ud559\uc0ac\uc7a5\uad50 \uc81c\ub3c4\uac00 \uc815\ucc29\ub418\uae30 \uc2dc\uc791\ud55c 80\ub144\ub300 \ub9d0\ubd80\ud130 \uc7a5\uae30\ubcf5\ubb34\uc790\uac00 \ub298\uc5b4\ub098\uae30 \uc2dc\uc791\ud558\uc5ec 2017\ub144 \ud604\uc7ac 10,000\uc5ec \uba85\uc774 \ud604\uc5ed\uc73c\ub85c \ubcf5\ubb34\ud558\uace0 \uc788\uc73c\uba70, 8\uba85\uc758 \uc7a5\uc131, \uc57d 3,000\uc5ec \uba85\uc758 \uc601\uad00\uc7a5\uad50, \uc911 \u00b7 \uc18c\ub300\uc7a5\uc758 40%\ub97c \ub2f4\ub2f9\ud558\uace0 \uc788\ub2e4. \uc758\ubb34\ubcf5\ubb34\ub97c \ub9c8\uce58\uace0 \uc0ac\ud68c\uc5d0 \uc9c4\ucd9c\ud55c \ud559\uc0ac\uc7a5\uad50\ub294 \uc804\ubb38\ubd84\uc57c\uc758 \uc9c0\uc2dd\uacfc \uad70\uc5d0\uc11c \uccb4\ub4dd\ud55c \uc7a5\uad50\uac00 \uc9c0\ub140\uc57c \ud560 \uc9c0\ub3c4\ub825\uacfc \uad70\uc778\uc815\uc2e0\uc744 \ubc14\ud0d5\uc73c\ub85c \uc0ac\ud68c \uac01\uacc4\uac01\uce35\uc5d0 \uc9c4\ucd9c\ud574 \uc788\ub2e4. \uc815\uad00\uacc4\uc5d0\uc11c \u524d\ucd1d\ub3d9\ubb38\ud68c\uc7a5\uc778 \uc774\uc885\ubc30(1\uae30) \uc790\uc720\ud55c\uad6d\ub2f9 \uc758\uc6d0, \uc815\uc591\uc11d(1\uae30) \ubc14\ub978\uc815\ub2f9 \uc758\uc6d0, \ubc15\uc131\uc911(1\uae30) \uc790\uc720\ud55c\uad6d\ub2f9 \uc758\uc6d0, \uae40\ub3d9\uc644(1\uae30) \u524d\uc0c8\ub204\ub9ac\ub2f9 \uc758\uc6d0, \u524d\uc0c8\uc815\uce58\ubbfc\uc8fc\uc5f0\ud569 \uc6d0\ub0b4\ub300\ud45c\uc778 \uc804\ubcd1\ud5cc(1\uae30) \u524d\ub354\ubd88\uc5b4\ubbfc\uc8fc\ub2f9 \uc758\uc6d0, \uae40\ud76c\uad6d(2\uae30) \u524d\uc0c8\ub204\ub9ac\ub2f9 \uc758\uc6d0 \ub4f1\uc774 \uc788\ub2e4. \ub610\ud55c \uc815\uc2b9(1\uae30) \uc804 \uc2dd\uc57d\ucc98\uc7a5, \uc815\uc7ac\uadfc(5\uae30) \ud589\uc815\uc790\uce58\ubd80 \ucc28\uad00, \uae40\ub355\uc911(7\uae30) \uc804 \uad6d\uc138\uccad\uc7a5, \uc774\uc885\ud638(7\uae30) \uc911\ubd80\uc9c0\ubc29\uad6d\uc138\uccad\uc7a5, \uc5ec\ud76c\uad11(7\uae30) \ub300\uad6c\uad11\uc5ed\uc2dc \ud589\uc815\ubd80\uc2dc\uc7a5, \ubc15\ub3d9\ud6c8(8\uae30) \uccad\uc640\ub300 \ud589\uc815\uc790\uce58\ube44\uc11c\uad00, \uae40\ub099\ud68c(8\uae30) \uad00\uc138\uccad\uc7a5, \uc624\uade0(9\uae30) \uccad\uc640\ub300 \uad6d\uc815\uacfc\uc81c\ube44\uc11c\uad00, \uae40\uc900\ub3d9(13\uae30) \uc0b0\uc5c5\ud1b5\uc0c1\uc790\uc6d0\ubd80 \uae30\ud68d\uc870\uc815\uc2e4\uc7a5, \uae40\uc758\ub3c4(17\uae30) \ud1b5\uc77c\ubd80 \ub300\ubcc0\uc778 \ub4f1 5\uae09 \uc774\uc0c1\uc758 \uacf5\uc9c1\uc790\uac00 1,000\uc5ec \uba85\uc774 \uc788\ub2e4. \ud604\uc9c1 \ub2e8\uccb4\uc7a5 \uac00\uc6b4\ub370\ub294 \uc720\uc815\ubcf5(1\uae30) \uc778\ucc9c\uad11\uc5ed\uc2dc\uc7a5, \uc774\uc131(1\uae30) \uc11c\uc6b8 \uad6c\ub85c\uad6c\uccad\uc7a5, \ubc15\uc131\uc77c(1\uae30) \uc644\uc8fc\uad70\uc218, \ubc30\uad11\uc2dd(4\uae30) \ub300\uad6c \ubd81\uad6c\uccad\uc7a5 \ub4f1\uc774 \uc788\ub2e4. \ubc95\uc870\uacc4\ub294 \uc591\ubd80\ub0a8(5\uae30) \ub300\uad6c\uc9c0\uac80 2\ucc28\uc7a5\uac80\uc0ac\ub97c \ud3ec\ud568\ud55c 40\uc5ec \uba85, \uacbd\ucc30\uc740 \uc724\uc885\uae30(5\uae30) \uc778\ucc9c\uc9c0\ubc29\uacbd\ucc30\uccad \uccad\uc7a5 \ub4f1 \uc57d 300\uc5ec \uba85, \uad50\uc721\uacc4\uc5d0\ub294 \ub178\ub3d9\uc77c(1\uae30) \uacbd\ud76c\ub300 \ubc95\ub300 \uad50\uc218\uc744 \ud3ec\ud568\ud55c \ub300\ud559\uad50\uc218 \uc57d 600\uc5ec \uba85, \uae40\uc625\uae30(2\uae30) \uc5f0\uc138\uc911 \uad50\uc7a5\uc744 \ud3ec\ud568\ud55c \uad50\uc0ac 800\uc5ec \uba85, \uacbd\uc81c\uacc4\uc5d0\ub294 \uc724\ud64d\uadfc(1\uae30) \uc81c\ub124\uc2dc\uc2a4 \ud68c\uc7a5\uc744 \ud3ec\ud568\ud55c 4,000\uc5ec \uba85 \ub4f1\uc774 \ud65c\uc57d\ud558\uace0 \uc788\ub2e4.", "answer": "4,000\uc5ec \uba85", "sentence": "\ubc95\uc870\uacc4\ub294 \uc591\ubd80\ub0a8(5\uae30) \ub300\uad6c\uc9c0\uac80 2\ucc28\uc7a5\uac80\uc0ac\ub97c \ud3ec\ud568\ud55c 40\uc5ec \uba85, \uacbd\ucc30\uc740 \uc724\uc885\uae30(5\uae30) \uc778\ucc9c\uc9c0\ubc29\uacbd\ucc30\uccad \uccad\uc7a5 \ub4f1 \uc57d 300\uc5ec \uba85, \uad50\uc721\uacc4\uc5d0\ub294 \ub178\ub3d9\uc77c(1\uae30) \uacbd\ud76c\ub300 \ubc95\ub300 \uad50\uc218\uc744 \ud3ec\ud568\ud55c \ub300\ud559\uad50\uc218 \uc57d 600\uc5ec \uba85, \uae40\uc625\uae30(2\uae30) \uc5f0\uc138\uc911 \uad50\uc7a5\uc744 \ud3ec\ud568\ud55c \uad50\uc0ac 800\uc5ec \uba85, \uacbd\uc81c\uacc4\uc5d0\ub294 \uc724\ud64d\uadfc(1\uae30) \uc81c\ub124\uc2dc\uc2a4 \ud68c\uc7a5\uc744 \ud3ec\ud568\ud55c 4,000\uc5ec \uba85 \ub4f1\uc774 \ud65c\uc57d\ud558\uace0 \uc788\ub2e4.", "paragraph_sentence": "\ud559\uc0ac\uc7a5\uad50\ub294 \ud559\uad70\uc7a5\uad50\uc640 \ube44\uc2b7\ud558\uac8c \ub300\ubd80\ubd84 \ub2e8\uae30 \uc758\ubb34\ubcf5\ubb34 \ud6c4 \uc804\uc5ed\ud558\uc5ec \uc0ac\ud68c \uac01\uacc4\uac01\uce35\uc5d0\uc11c \ud65c\uc57d\ud558\uace0 \uc788\ub2e4. \ud559\uc0ac\uc7a5\uad50 \uc81c\ub3c4\uac00 \uc815\ucc29\ub418\uae30 \uc2dc\uc791\ud55c 80\ub144\ub300 \ub9d0\ubd80\ud130 \uc7a5\uae30\ubcf5\ubb34\uc790\uac00 \ub298\uc5b4\ub098\uae30 \uc2dc\uc791\ud558\uc5ec 2017\ub144 \ud604\uc7ac 10,000\uc5ec \uba85\uc774 \ud604\uc5ed\uc73c\ub85c \ubcf5\ubb34\ud558\uace0 \uc788\uc73c\uba70, 8\uba85\uc758 \uc7a5\uc131, \uc57d 3,000\uc5ec \uba85\uc758 \uc601\uad00\uc7a5\uad50, \uc911 \u00b7 \uc18c\ub300\uc7a5\uc758 40%\ub97c \ub2f4\ub2f9\ud558\uace0 \uc788\ub2e4. \uc758\ubb34\ubcf5\ubb34\ub97c \ub9c8\uce58\uace0 \uc0ac\ud68c\uc5d0 \uc9c4\ucd9c\ud55c \ud559\uc0ac\uc7a5\uad50\ub294 \uc804\ubb38\ubd84\uc57c\uc758 \uc9c0\uc2dd\uacfc \uad70\uc5d0\uc11c \uccb4\ub4dd\ud55c \uc7a5\uad50\uac00 \uc9c0\ub140\uc57c \ud560 \uc9c0\ub3c4\ub825\uacfc \uad70\uc778\uc815\uc2e0\uc744 \ubc14\ud0d5\uc73c\ub85c \uc0ac\ud68c \uac01\uacc4\uac01\uce35\uc5d0 \uc9c4\ucd9c\ud574 \uc788\ub2e4. \uc815\uad00\uacc4\uc5d0\uc11c \u524d\ucd1d\ub3d9\ubb38\ud68c\uc7a5\uc778 \uc774\uc885\ubc30(1\uae30) \uc790\uc720\ud55c\uad6d\ub2f9 \uc758\uc6d0, \uc815\uc591\uc11d(1\uae30) \ubc14\ub978\uc815\ub2f9 \uc758\uc6d0, \ubc15\uc131\uc911(1\uae30) \uc790\uc720\ud55c\uad6d\ub2f9 \uc758\uc6d0, \uae40\ub3d9\uc644(1\uae30) \u524d\uc0c8\ub204\ub9ac\ub2f9 \uc758\uc6d0, \u524d\uc0c8\uc815\uce58\ubbfc\uc8fc\uc5f0\ud569 \uc6d0\ub0b4\ub300\ud45c\uc778 \uc804\ubcd1\ud5cc(1\uae30) \u524d\ub354\ubd88\uc5b4\ubbfc\uc8fc\ub2f9 \uc758\uc6d0, \uae40\ud76c\uad6d(2\uae30) \u524d\uc0c8\ub204\ub9ac\ub2f9 \uc758\uc6d0 \ub4f1\uc774 \uc788\ub2e4. \ub610\ud55c \uc815\uc2b9(1\uae30) \uc804 \uc2dd\uc57d\ucc98\uc7a5, \uc815\uc7ac\uadfc(5\uae30) \ud589\uc815\uc790\uce58\ubd80 \ucc28\uad00, \uae40\ub355\uc911(7\uae30) \uc804 \uad6d\uc138\uccad\uc7a5, \uc774\uc885\ud638(7\uae30) \uc911\ubd80\uc9c0\ubc29\uad6d\uc138\uccad\uc7a5, \uc5ec\ud76c\uad11(7\uae30) \ub300\uad6c\uad11\uc5ed\uc2dc \ud589\uc815\ubd80\uc2dc\uc7a5, \ubc15\ub3d9\ud6c8(8\uae30) \uccad\uc640\ub300 \ud589\uc815\uc790\uce58\ube44\uc11c\uad00, \uae40\ub099\ud68c(8\uae30) \uad00\uc138\uccad\uc7a5, \uc624\uade0(9\uae30) \uccad\uc640\ub300 \uad6d\uc815\uacfc\uc81c\ube44\uc11c\uad00, \uae40\uc900\ub3d9(13\uae30) \uc0b0\uc5c5\ud1b5\uc0c1\uc790\uc6d0\ubd80 \uae30\ud68d\uc870\uc815\uc2e4\uc7a5, \uae40\uc758\ub3c4(17\uae30) \ud1b5\uc77c\ubd80 \ub300\ubcc0\uc778 \ub4f1 5\uae09 \uc774\uc0c1\uc758 \uacf5\uc9c1\uc790\uac00 1,000\uc5ec \uba85\uc774 \uc788\ub2e4. \ud604\uc9c1 \ub2e8\uccb4\uc7a5 \uac00\uc6b4\ub370\ub294 \uc720\uc815\ubcf5(1\uae30) \uc778\ucc9c\uad11\uc5ed\uc2dc\uc7a5, \uc774\uc131(1\uae30) \uc11c\uc6b8 \uad6c\ub85c\uad6c\uccad\uc7a5, \ubc15\uc131\uc77c(1\uae30) \uc644\uc8fc\uad70\uc218, \ubc30\uad11\uc2dd(4\uae30) \ub300\uad6c \ubd81\uad6c\uccad\uc7a5 \ub4f1\uc774 \uc788\ub2e4. \ubc95\uc870\uacc4\ub294 \uc591\ubd80\ub0a8(5\uae30) \ub300\uad6c\uc9c0\uac80 2\ucc28\uc7a5\uac80\uc0ac\ub97c \ud3ec\ud568\ud55c 40\uc5ec \uba85, \uacbd\ucc30\uc740 \uc724\uc885\uae30(5\uae30) \uc778\ucc9c\uc9c0\ubc29\uacbd\ucc30\uccad \uccad\uc7a5 \ub4f1 \uc57d 300\uc5ec \uba85, \uad50\uc721\uacc4\uc5d0\ub294 \ub178\ub3d9\uc77c(1\uae30) \uacbd\ud76c\ub300 \ubc95\ub300 \uad50\uc218\uc744 \ud3ec\ud568\ud55c \ub300\ud559\uad50\uc218 \uc57d 600\uc5ec \uba85, \uae40\uc625\uae30(2\uae30) \uc5f0\uc138\uc911 \uad50\uc7a5\uc744 \ud3ec\ud568\ud55c \uad50\uc0ac 800\uc5ec \uba85, \uacbd\uc81c\uacc4\uc5d0\ub294 \uc724\ud64d\uadfc(1\uae30) \uc81c\ub124\uc2dc\uc2a4 \ud68c\uc7a5\uc744 \ud3ec\ud568\ud55c 4,000\uc5ec \uba85 \ub4f1\uc774 \ud65c\uc57d\ud558\uace0 \uc788\ub2e4. ", "paragraph_answer": "\ud559\uc0ac\uc7a5\uad50\ub294 \ud559\uad70\uc7a5\uad50\uc640 \ube44\uc2b7\ud558\uac8c \ub300\ubd80\ubd84 \ub2e8\uae30 \uc758\ubb34\ubcf5\ubb34 \ud6c4 \uc804\uc5ed\ud558\uc5ec \uc0ac\ud68c \uac01\uacc4\uac01\uce35\uc5d0\uc11c \ud65c\uc57d\ud558\uace0 \uc788\ub2e4. \ud559\uc0ac\uc7a5\uad50 \uc81c\ub3c4\uac00 \uc815\ucc29\ub418\uae30 \uc2dc\uc791\ud55c 80\ub144\ub300 \ub9d0\ubd80\ud130 \uc7a5\uae30\ubcf5\ubb34\uc790\uac00 \ub298\uc5b4\ub098\uae30 \uc2dc\uc791\ud558\uc5ec 2017\ub144 \ud604\uc7ac 10,000\uc5ec \uba85\uc774 \ud604\uc5ed\uc73c\ub85c \ubcf5\ubb34\ud558\uace0 \uc788\uc73c\uba70, 8\uba85\uc758 \uc7a5\uc131, \uc57d 3,000\uc5ec \uba85\uc758 \uc601\uad00\uc7a5\uad50, \uc911 \u00b7 \uc18c\ub300\uc7a5\uc758 40%\ub97c \ub2f4\ub2f9\ud558\uace0 \uc788\ub2e4. \uc758\ubb34\ubcf5\ubb34\ub97c \ub9c8\uce58\uace0 \uc0ac\ud68c\uc5d0 \uc9c4\ucd9c\ud55c \ud559\uc0ac\uc7a5\uad50\ub294 \uc804\ubb38\ubd84\uc57c\uc758 \uc9c0\uc2dd\uacfc \uad70\uc5d0\uc11c \uccb4\ub4dd\ud55c \uc7a5\uad50\uac00 \uc9c0\ub140\uc57c \ud560 \uc9c0\ub3c4\ub825\uacfc \uad70\uc778\uc815\uc2e0\uc744 \ubc14\ud0d5\uc73c\ub85c \uc0ac\ud68c \uac01\uacc4\uac01\uce35\uc5d0 \uc9c4\ucd9c\ud574 \uc788\ub2e4. \uc815\uad00\uacc4\uc5d0\uc11c \u524d\ucd1d\ub3d9\ubb38\ud68c\uc7a5\uc778 \uc774\uc885\ubc30(1\uae30) \uc790\uc720\ud55c\uad6d\ub2f9 \uc758\uc6d0, \uc815\uc591\uc11d(1\uae30) \ubc14\ub978\uc815\ub2f9 \uc758\uc6d0, \ubc15\uc131\uc911(1\uae30) \uc790\uc720\ud55c\uad6d\ub2f9 \uc758\uc6d0, \uae40\ub3d9\uc644(1\uae30) \u524d\uc0c8\ub204\ub9ac\ub2f9 \uc758\uc6d0, \u524d\uc0c8\uc815\uce58\ubbfc\uc8fc\uc5f0\ud569 \uc6d0\ub0b4\ub300\ud45c\uc778 \uc804\ubcd1\ud5cc(1\uae30) \u524d\ub354\ubd88\uc5b4\ubbfc\uc8fc\ub2f9 \uc758\uc6d0, \uae40\ud76c\uad6d(2\uae30) \u524d\uc0c8\ub204\ub9ac\ub2f9 \uc758\uc6d0 \ub4f1\uc774 \uc788\ub2e4. \ub610\ud55c \uc815\uc2b9(1\uae30) \uc804 \uc2dd\uc57d\ucc98\uc7a5, \uc815\uc7ac\uadfc(5\uae30) \ud589\uc815\uc790\uce58\ubd80 \ucc28\uad00, \uae40\ub355\uc911(7\uae30) \uc804 \uad6d\uc138\uccad\uc7a5, \uc774\uc885\ud638(7\uae30) \uc911\ubd80\uc9c0\ubc29\uad6d\uc138\uccad\uc7a5, \uc5ec\ud76c\uad11(7\uae30) \ub300\uad6c\uad11\uc5ed\uc2dc \ud589\uc815\ubd80\uc2dc\uc7a5, \ubc15\ub3d9\ud6c8(8\uae30) \uccad\uc640\ub300 \ud589\uc815\uc790\uce58\ube44\uc11c\uad00, \uae40\ub099\ud68c(8\uae30) \uad00\uc138\uccad\uc7a5, \uc624\uade0(9\uae30) \uccad\uc640\ub300 \uad6d\uc815\uacfc\uc81c\ube44\uc11c\uad00, \uae40\uc900\ub3d9(13\uae30) \uc0b0\uc5c5\ud1b5\uc0c1\uc790\uc6d0\ubd80 \uae30\ud68d\uc870\uc815\uc2e4\uc7a5, \uae40\uc758\ub3c4(17\uae30) \ud1b5\uc77c\ubd80 \ub300\ubcc0\uc778 \ub4f1 5\uae09 \uc774\uc0c1\uc758 \uacf5\uc9c1\uc790\uac00 1,000\uc5ec \uba85\uc774 \uc788\ub2e4. \ud604\uc9c1 \ub2e8\uccb4\uc7a5 \uac00\uc6b4\ub370\ub294 \uc720\uc815\ubcf5(1\uae30) \uc778\ucc9c\uad11\uc5ed\uc2dc\uc7a5, \uc774\uc131(1\uae30) \uc11c\uc6b8 \uad6c\ub85c\uad6c\uccad\uc7a5, \ubc15\uc131\uc77c(1\uae30) \uc644\uc8fc\uad70\uc218, \ubc30\uad11\uc2dd(4\uae30) \ub300\uad6c \ubd81\uad6c\uccad\uc7a5 \ub4f1\uc774 \uc788\ub2e4. \ubc95\uc870\uacc4\ub294 \uc591\ubd80\ub0a8(5\uae30) \ub300\uad6c\uc9c0\uac80 2\ucc28\uc7a5\uac80\uc0ac\ub97c \ud3ec\ud568\ud55c 40\uc5ec \uba85, \uacbd\ucc30\uc740 \uc724\uc885\uae30(5\uae30) \uc778\ucc9c\uc9c0\ubc29\uacbd\ucc30\uccad \uccad\uc7a5 \ub4f1 \uc57d 300\uc5ec \uba85, \uad50\uc721\uacc4\uc5d0\ub294 \ub178\ub3d9\uc77c(1\uae30) \uacbd\ud76c\ub300 \ubc95\ub300 \uad50\uc218\uc744 \ud3ec\ud568\ud55c \ub300\ud559\uad50\uc218 \uc57d 600\uc5ec \uba85, \uae40\uc625\uae30(2\uae30) \uc5f0\uc138\uc911 \uad50\uc7a5\uc744 \ud3ec\ud568\ud55c \uad50\uc0ac 800\uc5ec \uba85, \uacbd\uc81c\uacc4\uc5d0\ub294 \uc724\ud64d\uadfc(1\uae30) \uc81c\ub124\uc2dc\uc2a4 \ud68c\uc7a5\uc744 \ud3ec\ud568\ud55c 4,000\uc5ec \uba85 \ub4f1\uc774 \ud65c\uc57d\ud558\uace0 \uc788\ub2e4.", "sentence_answer": "\ubc95\uc870\uacc4\ub294 \uc591\ubd80\ub0a8(5\uae30) \ub300\uad6c\uc9c0\uac80 2\ucc28\uc7a5\uac80\uc0ac\ub97c \ud3ec\ud568\ud55c 40\uc5ec \uba85, \uacbd\ucc30\uc740 \uc724\uc885\uae30(5\uae30) \uc778\ucc9c\uc9c0\ubc29\uacbd\ucc30\uccad \uccad\uc7a5 \ub4f1 \uc57d 300\uc5ec \uba85, \uad50\uc721\uacc4\uc5d0\ub294 \ub178\ub3d9\uc77c(1\uae30) \uacbd\ud76c\ub300 \ubc95\ub300 \uad50\uc218\uc744 \ud3ec\ud568\ud55c \ub300\ud559\uad50\uc218 \uc57d 600\uc5ec \uba85, \uae40\uc625\uae30(2\uae30) \uc5f0\uc138\uc911 \uad50\uc7a5\uc744 \ud3ec\ud568\ud55c \uad50\uc0ac 800\uc5ec \uba85, \uacbd\uc81c\uacc4\uc5d0\ub294 \uc724\ud64d\uadfc(1\uae30) \uc81c\ub124\uc2dc\uc2a4 \ud68c\uc7a5\uc744 \ud3ec\ud568\ud55c 4,000\uc5ec \uba85 \ub4f1\uc774 \ud65c\uc57d\ud558\uace0 \uc788\ub2e4.", "paragraph_id": "5833774-2-2", "paragraph_question": "question: 2017\ub144 \uacbd\uc81c\uacc4\uc5d0\uc11c \ud65c\uc57d\ud558\ub294 \ud559\uc0ac\uc7a5\uad50 \ucd9c\uc2e0\uc758 \uc218\ub294 \ub300\ub7b5 \uba87 \uba85\uc778\uac00?, context: \ud559\uc0ac\uc7a5\uad50\ub294 \ud559\uad70\uc7a5\uad50\uc640 \ube44\uc2b7\ud558\uac8c \ub300\ubd80\ubd84 \ub2e8\uae30 \uc758\ubb34\ubcf5\ubb34 \ud6c4 \uc804\uc5ed\ud558\uc5ec \uc0ac\ud68c \uac01\uacc4\uac01\uce35\uc5d0\uc11c \ud65c\uc57d\ud558\uace0 \uc788\ub2e4. \ud559\uc0ac\uc7a5\uad50 \uc81c\ub3c4\uac00 \uc815\ucc29\ub418\uae30 \uc2dc\uc791\ud55c 80\ub144\ub300 \ub9d0\ubd80\ud130 \uc7a5\uae30\ubcf5\ubb34\uc790\uac00 \ub298\uc5b4\ub098\uae30 \uc2dc\uc791\ud558\uc5ec 2017\ub144 \ud604\uc7ac 10,000\uc5ec \uba85\uc774 \ud604\uc5ed\uc73c\ub85c \ubcf5\ubb34\ud558\uace0 \uc788\uc73c\uba70, 8\uba85\uc758 \uc7a5\uc131, \uc57d 3,000\uc5ec \uba85\uc758 \uc601\uad00\uc7a5\uad50, \uc911 \u00b7 \uc18c\ub300\uc7a5\uc758 40%\ub97c \ub2f4\ub2f9\ud558\uace0 \uc788\ub2e4. \uc758\ubb34\ubcf5\ubb34\ub97c \ub9c8\uce58\uace0 \uc0ac\ud68c\uc5d0 \uc9c4\ucd9c\ud55c \ud559\uc0ac\uc7a5\uad50\ub294 \uc804\ubb38\ubd84\uc57c\uc758 \uc9c0\uc2dd\uacfc \uad70\uc5d0\uc11c \uccb4\ub4dd\ud55c \uc7a5\uad50\uac00 \uc9c0\ub140\uc57c \ud560 \uc9c0\ub3c4\ub825\uacfc \uad70\uc778\uc815\uc2e0\uc744 \ubc14\ud0d5\uc73c\ub85c \uc0ac\ud68c \uac01\uacc4\uac01\uce35\uc5d0 \uc9c4\ucd9c\ud574 \uc788\ub2e4. \uc815\uad00\uacc4\uc5d0\uc11c \u524d\ucd1d\ub3d9\ubb38\ud68c\uc7a5\uc778 \uc774\uc885\ubc30(1\uae30) \uc790\uc720\ud55c\uad6d\ub2f9 \uc758\uc6d0, \uc815\uc591\uc11d(1\uae30) \ubc14\ub978\uc815\ub2f9 \uc758\uc6d0, \ubc15\uc131\uc911(1\uae30) \uc790\uc720\ud55c\uad6d\ub2f9 \uc758\uc6d0, \uae40\ub3d9\uc644(1\uae30) \u524d\uc0c8\ub204\ub9ac\ub2f9 \uc758\uc6d0, \u524d\uc0c8\uc815\uce58\ubbfc\uc8fc\uc5f0\ud569 \uc6d0\ub0b4\ub300\ud45c\uc778 \uc804\ubcd1\ud5cc(1\uae30) \u524d\ub354\ubd88\uc5b4\ubbfc\uc8fc\ub2f9 \uc758\uc6d0, \uae40\ud76c\uad6d(2\uae30) \u524d\uc0c8\ub204\ub9ac\ub2f9 \uc758\uc6d0 \ub4f1\uc774 \uc788\ub2e4. \ub610\ud55c \uc815\uc2b9(1\uae30) \uc804 \uc2dd\uc57d\ucc98\uc7a5, \uc815\uc7ac\uadfc(5\uae30) \ud589\uc815\uc790\uce58\ubd80 \ucc28\uad00, \uae40\ub355\uc911(7\uae30) \uc804 \uad6d\uc138\uccad\uc7a5, \uc774\uc885\ud638(7\uae30) \uc911\ubd80\uc9c0\ubc29\uad6d\uc138\uccad\uc7a5, \uc5ec\ud76c\uad11(7\uae30) \ub300\uad6c\uad11\uc5ed\uc2dc \ud589\uc815\ubd80\uc2dc\uc7a5, \ubc15\ub3d9\ud6c8(8\uae30) \uccad\uc640\ub300 \ud589\uc815\uc790\uce58\ube44\uc11c\uad00, \uae40\ub099\ud68c(8\uae30) \uad00\uc138\uccad\uc7a5, \uc624\uade0(9\uae30) \uccad\uc640\ub300 \uad6d\uc815\uacfc\uc81c\ube44\uc11c\uad00, \uae40\uc900\ub3d9(13\uae30) \uc0b0\uc5c5\ud1b5\uc0c1\uc790\uc6d0\ubd80 \uae30\ud68d\uc870\uc815\uc2e4\uc7a5, \uae40\uc758\ub3c4(17\uae30) \ud1b5\uc77c\ubd80 \ub300\ubcc0\uc778 \ub4f1 5\uae09 \uc774\uc0c1\uc758 \uacf5\uc9c1\uc790\uac00 1,000\uc5ec \uba85\uc774 \uc788\ub2e4. \ud604\uc9c1 \ub2e8\uccb4\uc7a5 \uac00\uc6b4\ub370\ub294 \uc720\uc815\ubcf5(1\uae30) \uc778\ucc9c\uad11\uc5ed\uc2dc\uc7a5, \uc774\uc131(1\uae30) \uc11c\uc6b8 \uad6c\ub85c\uad6c\uccad\uc7a5, \ubc15\uc131\uc77c(1\uae30) \uc644\uc8fc\uad70\uc218, \ubc30\uad11\uc2dd(4\uae30) \ub300\uad6c \ubd81\uad6c\uccad\uc7a5 \ub4f1\uc774 \uc788\ub2e4. \ubc95\uc870\uacc4\ub294 \uc591\ubd80\ub0a8(5\uae30) \ub300\uad6c\uc9c0\uac80 2\ucc28\uc7a5\uac80\uc0ac\ub97c \ud3ec\ud568\ud55c 40\uc5ec \uba85, \uacbd\ucc30\uc740 \uc724\uc885\uae30(5\uae30) \uc778\ucc9c\uc9c0\ubc29\uacbd\ucc30\uccad \uccad\uc7a5 \ub4f1 \uc57d 300\uc5ec \uba85, \uad50\uc721\uacc4\uc5d0\ub294 \ub178\ub3d9\uc77c(1\uae30) \uacbd\ud76c\ub300 \ubc95\ub300 \uad50\uc218\uc744 \ud3ec\ud568\ud55c \ub300\ud559\uad50\uc218 \uc57d 600\uc5ec \uba85, \uae40\uc625\uae30(2\uae30) \uc5f0\uc138\uc911 \uad50\uc7a5\uc744 \ud3ec\ud568\ud55c \uad50\uc0ac 800\uc5ec \uba85, \uacbd\uc81c\uacc4\uc5d0\ub294 \uc724\ud64d\uadfc(1\uae30) \uc81c\ub124\uc2dc\uc2a4 \ud68c\uc7a5\uc744 \ud3ec\ud568\ud55c 4,000\uc5ec \uba85 \ub4f1\uc774 \ud65c\uc57d\ud558\uace0 \uc788\ub2e4."} {"question": "\ubbf8\uad6d\uc5d0\uc11c \uc8fc\ub85c \ub3d9\ubd80 \ubc0f \uc11c\ubd80 \ub300\ud559\uc5d0 \uc9c4\ud559\ud558\uae30 \uc704\ud574 \ubcf4\ub294 \uc2dc\ud5d8\uc740?", "paragraph": "\ubbf8\uad6d\uc758 \uad50\uc721\uc740 \ucd08\uae30 \uc2dd\ubbfc\uc9c0 \uc2dc\uc808\ubd80\ud130 \uc911\uc694\uc2dc\ub418\uc5b4\uc654\ub294\ub370, \uace0\ub4f1\uad50\uc721\uae30\uad00\uc758 \ubc1c\uc804\uc740 \uc804\uc7c1\uacfc \uacfc\ud559 \uc5f0\uad6c \ub4f1\uc5d0 \uc788\uc5b4 \ubbf8\uad6d\uc758 \uc5ed\uc0ac\uc640 \ud568\uaed8\ud574\uc654\ub2e4. \ucd08\uae30\uc5d0\uc11c\ubd80\ud130 \ud604\uc7ac\uae4c\uc9c0 \uad50\uc721\uc5d0 \uc788\uc5b4 \uc885\uad50\uc758 \uc601\ud5a5\uc740 \ub9e4\uc6b0 \ud06c\uba70, \uc5d8\ub9ac\ud2b8\ub4e4\uc758 \uad6d\uac00 \uacbd\uc601\uc774 \uc7a5\ub824\ub418\ub294 \uc0ac\ud68c\uc5ec\uc11c, \uc0ac\ud559\uc774 \ubc1c\ub2ec\ud588\ub2e4. \ud06c\uac8c \uc0ac\ub9bd\uacfc \uc8fc\ub9bd \ud639\uc740 \uad6d\uacf5\ub9bd \uad50\uc721\uae30\uad00\uc73c\ub85c \ub098\ub258\uba70, \ub300\ubd80\ubd84\uc758 \uc8fc\uc5d0\uc11c\ub294 6\uc138\uc5d0\uc11c 16\uc138\uae4c\uc9c0 \ubb34\uc0c1\u00b7\uc758\ubb34 \uad50\uc721\uc744 \uc2e4\uc2dc\ud55c\ub2e4. \ubbf8\uad6d \ud559\uc0dd\ub4e4\uc758 \uc808\ub300 \ub2e4\uc218\uac00 \uc911\ub4f1\uad50\uc721\uc744 \ub9c8\uce58\ub294 17, 18\uc138 (K-12 \ud559\uc81c \uc0c1 \uace0\ub4f1\ud559\uad50 \uc878\uc5c5\ubc18)\uae4c\uc9c0 \ud559\uad50\uc5d0 \ub2e4\ub2cc\ub2e4. \ubd80\uc790\ub4e4\uc740 \ub300\uccb4\ub85c \uc0ac\ub9bd \ud559\uad50\uc5d0 \ub2e4\ub2cc\ub2e4. \uc2e4\uc6a9\uc801\uc778 \uad50\uc721 \ucca0\ud559\uc740 \uad50\uc721\uc758 \ub9c8\uc9c0\ub9c9 \uae30\uac04\uc778 \ub300\ud559\uad50\uc640 \ub300\ud559\uc6d0\uc758 \uc6b0\uc218\uc131\uc5d0\uc11c \uc54c \uc218 \uc788\ub294\ub370, \ud2b9\ud788 \ub300\ud559\uad50\uc640 \ub300\ud559\uc6d0 \ub4f1 \uace0\ub4f1\uad50\uc721\uc740 \uadf8 \uba85\uc131\uacfc \ud559\uc5f4, \ud559\uc0dd \uc218\uc900, \uadf8\ub9ac\uace0 \uc5f0\uad6c \uc2e4\uc801\uc5d0\uc11c \uc138\uacc4 \uc5ec\ub290 \ub098\ub77c\uc758 \uace0\ub4f1\uad50\uc721\uae30\uad00\uc744 \uc555\ub3c4\ud55c\ub2e4. \ubbf8\uad6d\uc5d0\uc11c \ub300\ud559\uc5d0 \uc9c4\ud559\ud558\ub824\uba74 ACT(\uc8fc\ub85c \uc911\ubd80 \ucabd \ub300\ud559)\ub098 SAT(\uc8fc\ub85c \ub3d9\ubd80, \uc11c\ubd80 \ucabd \ub300\ud559)\ub97c \uce58\ub7ec\uc57c \ud55c\ub2e4. \ub2e4\ub978 \uc720\ub7fd\uc758 \uad6d\uac00\ub4e4\ucc98\ub7fc \ubbf8\uad6d\ub3c4 \uc911\ub4f1 \uad50\uc721 \ub2e8\uacc4\ubd80\ud130 \ud559\uc810\uc81c\ub97c \ucc44\ud0dd\ud55c\ub2e4. \uad50\uc721\uc5d0\uc11c\ub294 \uc601\uc5b4\ub97c \uc0ac\uc6a9\ud558\uace0, \uc678\uad6d\uc5b4\ub85c\ub294 \ub3c5\uc77c\uc5b4, \ud504\ub791\uc2a4\uc5b4, \uc2a4\ud398\uc778\uc5b4, \ub77c\ud2f4\uc5b4, \uadf8\ub9ac\uc2a4\uc5b4, \ud788\ube0c\ub9ac\uc5b4, \uc774\ud0c8\ub9ac\uc544\uc5b4, \uc911\uad6d\uc5b4, \uc77c\ubcf8\uc5b4, \ud55c\uad6d\uc5b4 \uc911 \ud558\ub098\ub97c \uc120\ud0dd\ud55c\ub2e4.", "answer": "SAT", "sentence": "\ubbf8\uad6d\uc5d0\uc11c \ub300\ud559\uc5d0 \uc9c4\ud559\ud558\ub824\uba74 ACT(\uc8fc\ub85c \uc911\ubd80 \ucabd \ub300\ud559)\ub098 SAT (\uc8fc\ub85c \ub3d9\ubd80, \uc11c\ubd80 \ucabd \ub300\ud559)\ub97c \uce58\ub7ec\uc57c \ud55c\ub2e4.", "paragraph_sentence": "\ubbf8\uad6d\uc758 \uad50\uc721\uc740 \ucd08\uae30 \uc2dd\ubbfc\uc9c0 \uc2dc\uc808\ubd80\ud130 \uc911\uc694\uc2dc\ub418\uc5b4\uc654\ub294\ub370, \uace0\ub4f1\uad50\uc721\uae30\uad00\uc758 \ubc1c\uc804\uc740 \uc804\uc7c1\uacfc \uacfc\ud559 \uc5f0\uad6c \ub4f1\uc5d0 \uc788\uc5b4 \ubbf8\uad6d\uc758 \uc5ed\uc0ac\uc640 \ud568\uaed8\ud574\uc654\ub2e4. \ucd08\uae30\uc5d0\uc11c\ubd80\ud130 \ud604\uc7ac\uae4c\uc9c0 \uad50\uc721\uc5d0 \uc788\uc5b4 \uc885\uad50\uc758 \uc601\ud5a5\uc740 \ub9e4\uc6b0 \ud06c\uba70, \uc5d8\ub9ac\ud2b8\ub4e4\uc758 \uad6d\uac00 \uacbd\uc601\uc774 \uc7a5\ub824\ub418\ub294 \uc0ac\ud68c\uc5ec\uc11c, \uc0ac\ud559\uc774 \ubc1c\ub2ec\ud588\ub2e4. \ud06c\uac8c \uc0ac\ub9bd\uacfc \uc8fc\ub9bd \ud639\uc740 \uad6d\uacf5\ub9bd \uad50\uc721\uae30\uad00\uc73c\ub85c \ub098\ub258\uba70, \ub300\ubd80\ubd84\uc758 \uc8fc\uc5d0\uc11c\ub294 6\uc138\uc5d0\uc11c 16\uc138\uae4c\uc9c0 \ubb34\uc0c1\u00b7\uc758\ubb34 \uad50\uc721\uc744 \uc2e4\uc2dc\ud55c\ub2e4. \ubbf8\uad6d \ud559\uc0dd\ub4e4\uc758 \uc808\ub300 \ub2e4\uc218\uac00 \uc911\ub4f1\uad50\uc721\uc744 \ub9c8\uce58\ub294 17, 18\uc138 (K-12 \ud559\uc81c \uc0c1 \uace0\ub4f1\ud559\uad50 \uc878\uc5c5\ubc18)\uae4c\uc9c0 \ud559\uad50\uc5d0 \ub2e4\ub2cc\ub2e4. \ubd80\uc790\ub4e4\uc740 \ub300\uccb4\ub85c \uc0ac\ub9bd \ud559\uad50\uc5d0 \ub2e4\ub2cc\ub2e4. \uc2e4\uc6a9\uc801\uc778 \uad50\uc721 \ucca0\ud559\uc740 \uad50\uc721\uc758 \ub9c8\uc9c0\ub9c9 \uae30\uac04\uc778 \ub300\ud559\uad50\uc640 \ub300\ud559\uc6d0\uc758 \uc6b0\uc218\uc131\uc5d0\uc11c \uc54c \uc218 \uc788\ub294\ub370, \ud2b9\ud788 \ub300\ud559\uad50\uc640 \ub300\ud559\uc6d0 \ub4f1 \uace0\ub4f1\uad50\uc721\uc740 \uadf8 \uba85\uc131\uacfc \ud559\uc5f4, \ud559\uc0dd \uc218\uc900, \uadf8\ub9ac\uace0 \uc5f0\uad6c \uc2e4\uc801\uc5d0\uc11c \uc138\uacc4 \uc5ec\ub290 \ub098\ub77c\uc758 \uace0\ub4f1\uad50\uc721\uae30\uad00\uc744 \uc555\ub3c4\ud55c\ub2e4. \ubbf8\uad6d\uc5d0\uc11c \ub300\ud559\uc5d0 \uc9c4\ud559\ud558\ub824\uba74 ACT(\uc8fc\ub85c \uc911\ubd80 \ucabd \ub300\ud559)\ub098 SAT (\uc8fc\ub85c \ub3d9\ubd80, \uc11c\ubd80 \ucabd \ub300\ud559)\ub97c \uce58\ub7ec\uc57c \ud55c\ub2e4. \ub2e4\ub978 \uc720\ub7fd\uc758 \uad6d\uac00\ub4e4\ucc98\ub7fc \ubbf8\uad6d\ub3c4 \uc911\ub4f1 \uad50\uc721 \ub2e8\uacc4\ubd80\ud130 \ud559\uc810\uc81c\ub97c \ucc44\ud0dd\ud55c\ub2e4. \uad50\uc721\uc5d0\uc11c\ub294 \uc601\uc5b4\ub97c \uc0ac\uc6a9\ud558\uace0, \uc678\uad6d\uc5b4\ub85c\ub294 \ub3c5\uc77c\uc5b4, \ud504\ub791\uc2a4\uc5b4, \uc2a4\ud398\uc778\uc5b4, \ub77c\ud2f4\uc5b4, \uadf8\ub9ac\uc2a4\uc5b4, \ud788\ube0c\ub9ac\uc5b4, \uc774\ud0c8\ub9ac\uc544\uc5b4, \uc911\uad6d\uc5b4, \uc77c\ubcf8\uc5b4, \ud55c\uad6d\uc5b4 \uc911 \ud558\ub098\ub97c \uc120\ud0dd\ud55c\ub2e4.", "paragraph_answer": "\ubbf8\uad6d\uc758 \uad50\uc721\uc740 \ucd08\uae30 \uc2dd\ubbfc\uc9c0 \uc2dc\uc808\ubd80\ud130 \uc911\uc694\uc2dc\ub418\uc5b4\uc654\ub294\ub370, \uace0\ub4f1\uad50\uc721\uae30\uad00\uc758 \ubc1c\uc804\uc740 \uc804\uc7c1\uacfc \uacfc\ud559 \uc5f0\uad6c \ub4f1\uc5d0 \uc788\uc5b4 \ubbf8\uad6d\uc758 \uc5ed\uc0ac\uc640 \ud568\uaed8\ud574\uc654\ub2e4. \ucd08\uae30\uc5d0\uc11c\ubd80\ud130 \ud604\uc7ac\uae4c\uc9c0 \uad50\uc721\uc5d0 \uc788\uc5b4 \uc885\uad50\uc758 \uc601\ud5a5\uc740 \ub9e4\uc6b0 \ud06c\uba70, \uc5d8\ub9ac\ud2b8\ub4e4\uc758 \uad6d\uac00 \uacbd\uc601\uc774 \uc7a5\ub824\ub418\ub294 \uc0ac\ud68c\uc5ec\uc11c, \uc0ac\ud559\uc774 \ubc1c\ub2ec\ud588\ub2e4. \ud06c\uac8c \uc0ac\ub9bd\uacfc \uc8fc\ub9bd \ud639\uc740 \uad6d\uacf5\ub9bd \uad50\uc721\uae30\uad00\uc73c\ub85c \ub098\ub258\uba70, \ub300\ubd80\ubd84\uc758 \uc8fc\uc5d0\uc11c\ub294 6\uc138\uc5d0\uc11c 16\uc138\uae4c\uc9c0 \ubb34\uc0c1\u00b7\uc758\ubb34 \uad50\uc721\uc744 \uc2e4\uc2dc\ud55c\ub2e4. \ubbf8\uad6d \ud559\uc0dd\ub4e4\uc758 \uc808\ub300 \ub2e4\uc218\uac00 \uc911\ub4f1\uad50\uc721\uc744 \ub9c8\uce58\ub294 17, 18\uc138 (K-12 \ud559\uc81c \uc0c1 \uace0\ub4f1\ud559\uad50 \uc878\uc5c5\ubc18)\uae4c\uc9c0 \ud559\uad50\uc5d0 \ub2e4\ub2cc\ub2e4. \ubd80\uc790\ub4e4\uc740 \ub300\uccb4\ub85c \uc0ac\ub9bd \ud559\uad50\uc5d0 \ub2e4\ub2cc\ub2e4. \uc2e4\uc6a9\uc801\uc778 \uad50\uc721 \ucca0\ud559\uc740 \uad50\uc721\uc758 \ub9c8\uc9c0\ub9c9 \uae30\uac04\uc778 \ub300\ud559\uad50\uc640 \ub300\ud559\uc6d0\uc758 \uc6b0\uc218\uc131\uc5d0\uc11c \uc54c \uc218 \uc788\ub294\ub370, \ud2b9\ud788 \ub300\ud559\uad50\uc640 \ub300\ud559\uc6d0 \ub4f1 \uace0\ub4f1\uad50\uc721\uc740 \uadf8 \uba85\uc131\uacfc \ud559\uc5f4, \ud559\uc0dd \uc218\uc900, \uadf8\ub9ac\uace0 \uc5f0\uad6c \uc2e4\uc801\uc5d0\uc11c \uc138\uacc4 \uc5ec\ub290 \ub098\ub77c\uc758 \uace0\ub4f1\uad50\uc721\uae30\uad00\uc744 \uc555\ub3c4\ud55c\ub2e4. \ubbf8\uad6d\uc5d0\uc11c \ub300\ud559\uc5d0 \uc9c4\ud559\ud558\ub824\uba74 ACT(\uc8fc\ub85c \uc911\ubd80 \ucabd \ub300\ud559)\ub098 SAT (\uc8fc\ub85c \ub3d9\ubd80, \uc11c\ubd80 \ucabd \ub300\ud559)\ub97c \uce58\ub7ec\uc57c \ud55c\ub2e4. \ub2e4\ub978 \uc720\ub7fd\uc758 \uad6d\uac00\ub4e4\ucc98\ub7fc \ubbf8\uad6d\ub3c4 \uc911\ub4f1 \uad50\uc721 \ub2e8\uacc4\ubd80\ud130 \ud559\uc810\uc81c\ub97c \ucc44\ud0dd\ud55c\ub2e4. \uad50\uc721\uc5d0\uc11c\ub294 \uc601\uc5b4\ub97c \uc0ac\uc6a9\ud558\uace0, \uc678\uad6d\uc5b4\ub85c\ub294 \ub3c5\uc77c\uc5b4, \ud504\ub791\uc2a4\uc5b4, \uc2a4\ud398\uc778\uc5b4, \ub77c\ud2f4\uc5b4, \uadf8\ub9ac\uc2a4\uc5b4, \ud788\ube0c\ub9ac\uc5b4, \uc774\ud0c8\ub9ac\uc544\uc5b4, \uc911\uad6d\uc5b4, \uc77c\ubcf8\uc5b4, \ud55c\uad6d\uc5b4 \uc911 \ud558\ub098\ub97c \uc120\ud0dd\ud55c\ub2e4.", "sentence_answer": "\ubbf8\uad6d\uc5d0\uc11c \ub300\ud559\uc5d0 \uc9c4\ud559\ud558\ub824\uba74 ACT(\uc8fc\ub85c \uc911\ubd80 \ucabd \ub300\ud559)\ub098 SAT (\uc8fc\ub85c \ub3d9\ubd80, \uc11c\ubd80 \ucabd \ub300\ud559)\ub97c \uce58\ub7ec\uc57c \ud55c\ub2e4.", "paragraph_id": "6412933-32-1", "paragraph_question": "question: \ubbf8\uad6d\uc5d0\uc11c \uc8fc\ub85c \ub3d9\ubd80 \ubc0f \uc11c\ubd80 \ub300\ud559\uc5d0 \uc9c4\ud559\ud558\uae30 \uc704\ud574 \ubcf4\ub294 \uc2dc\ud5d8\uc740?, context: \ubbf8\uad6d\uc758 \uad50\uc721\uc740 \ucd08\uae30 \uc2dd\ubbfc\uc9c0 \uc2dc\uc808\ubd80\ud130 \uc911\uc694\uc2dc\ub418\uc5b4\uc654\ub294\ub370, \uace0\ub4f1\uad50\uc721\uae30\uad00\uc758 \ubc1c\uc804\uc740 \uc804\uc7c1\uacfc \uacfc\ud559 \uc5f0\uad6c \ub4f1\uc5d0 \uc788\uc5b4 \ubbf8\uad6d\uc758 \uc5ed\uc0ac\uc640 \ud568\uaed8\ud574\uc654\ub2e4. \ucd08\uae30\uc5d0\uc11c\ubd80\ud130 \ud604\uc7ac\uae4c\uc9c0 \uad50\uc721\uc5d0 \uc788\uc5b4 \uc885\uad50\uc758 \uc601\ud5a5\uc740 \ub9e4\uc6b0 \ud06c\uba70, \uc5d8\ub9ac\ud2b8\ub4e4\uc758 \uad6d\uac00 \uacbd\uc601\uc774 \uc7a5\ub824\ub418\ub294 \uc0ac\ud68c\uc5ec\uc11c, \uc0ac\ud559\uc774 \ubc1c\ub2ec\ud588\ub2e4. \ud06c\uac8c \uc0ac\ub9bd\uacfc \uc8fc\ub9bd \ud639\uc740 \uad6d\uacf5\ub9bd \uad50\uc721\uae30\uad00\uc73c\ub85c \ub098\ub258\uba70, \ub300\ubd80\ubd84\uc758 \uc8fc\uc5d0\uc11c\ub294 6\uc138\uc5d0\uc11c 16\uc138\uae4c\uc9c0 \ubb34\uc0c1\u00b7\uc758\ubb34 \uad50\uc721\uc744 \uc2e4\uc2dc\ud55c\ub2e4. \ubbf8\uad6d \ud559\uc0dd\ub4e4\uc758 \uc808\ub300 \ub2e4\uc218\uac00 \uc911\ub4f1\uad50\uc721\uc744 \ub9c8\uce58\ub294 17, 18\uc138 (K-12 \ud559\uc81c \uc0c1 \uace0\ub4f1\ud559\uad50 \uc878\uc5c5\ubc18)\uae4c\uc9c0 \ud559\uad50\uc5d0 \ub2e4\ub2cc\ub2e4. \ubd80\uc790\ub4e4\uc740 \ub300\uccb4\ub85c \uc0ac\ub9bd \ud559\uad50\uc5d0 \ub2e4\ub2cc\ub2e4. \uc2e4\uc6a9\uc801\uc778 \uad50\uc721 \ucca0\ud559\uc740 \uad50\uc721\uc758 \ub9c8\uc9c0\ub9c9 \uae30\uac04\uc778 \ub300\ud559\uad50\uc640 \ub300\ud559\uc6d0\uc758 \uc6b0\uc218\uc131\uc5d0\uc11c \uc54c \uc218 \uc788\ub294\ub370, \ud2b9\ud788 \ub300\ud559\uad50\uc640 \ub300\ud559\uc6d0 \ub4f1 \uace0\ub4f1\uad50\uc721\uc740 \uadf8 \uba85\uc131\uacfc \ud559\uc5f4, \ud559\uc0dd \uc218\uc900, \uadf8\ub9ac\uace0 \uc5f0\uad6c \uc2e4\uc801\uc5d0\uc11c \uc138\uacc4 \uc5ec\ub290 \ub098\ub77c\uc758 \uace0\ub4f1\uad50\uc721\uae30\uad00\uc744 \uc555\ub3c4\ud55c\ub2e4. \ubbf8\uad6d\uc5d0\uc11c \ub300\ud559\uc5d0 \uc9c4\ud559\ud558\ub824\uba74 ACT(\uc8fc\ub85c \uc911\ubd80 \ucabd \ub300\ud559)\ub098 SAT(\uc8fc\ub85c \ub3d9\ubd80, \uc11c\ubd80 \ucabd \ub300\ud559)\ub97c \uce58\ub7ec\uc57c \ud55c\ub2e4. \ub2e4\ub978 \uc720\ub7fd\uc758 \uad6d\uac00\ub4e4\ucc98\ub7fc \ubbf8\uad6d\ub3c4 \uc911\ub4f1 \uad50\uc721 \ub2e8\uacc4\ubd80\ud130 \ud559\uc810\uc81c\ub97c \ucc44\ud0dd\ud55c\ub2e4. \uad50\uc721\uc5d0\uc11c\ub294 \uc601\uc5b4\ub97c \uc0ac\uc6a9\ud558\uace0, \uc678\uad6d\uc5b4\ub85c\ub294 \ub3c5\uc77c\uc5b4, \ud504\ub791\uc2a4\uc5b4, \uc2a4\ud398\uc778\uc5b4, \ub77c\ud2f4\uc5b4, \uadf8\ub9ac\uc2a4\uc5b4, \ud788\ube0c\ub9ac\uc5b4, \uc774\ud0c8\ub9ac\uc544\uc5b4, \uc911\uad6d\uc5b4, \uc77c\ubcf8\uc5b4, \ud55c\uad6d\uc5b4 \uc911 \ud558\ub098\ub97c \uc120\ud0dd\ud55c\ub2e4."} {"question": "\ud604\uc7ac \ub300\ud55c\ubbfc\uad6d\uc740 \uc77c\ubc18 \ub3d9\uc0b0\ubb38\ud654\uc7ac\ub97c \ud574\uc678\ub85c \ubc18\ucd9c\ud560 \ub54c \ub204\uad6c\uc758 \ud5c8\uac00\ub97c \ubc1b\uc544\uc57c\ud558\ub294\uac00?", "paragraph": "\ud604\ud589 \ub300\ud55c\ubbfc\uad6d\uc758 \ubb38\ud654\uc7ac\ubcf4\ud638\ubc95\uc5d0 \ub530\ub974\uba74 \ube44\uc9c0\uc815\ubb38\ud654\uc7ac\ub97c \ud3ec\ud568\ud55c \uc77c\ubc18 \ub3d9\uc0b0\ubb38\ud654\uc7ac\ub97c \ud574\uc678\ub85c \ubc18\ucd9c\ud560 \uacbd\uc6b0\uc5d0\ub294 \uc0ac\uc804\uc5d0 \ubb38\ud654\uc7ac\uccad\uc7a5\uc758 \ud5c8\uac00\ub97c \ubc1b\ub3c4\ub85d \ud558\uace0 \uc788\ub2e4. \uc6d0\uce59\uc801\uc73c\ub85c \uad6d\ubcf4, \ubcf4\ubb3c, \ucc9c\uc5f0\uae30\ub150\ubb3c \ub610\ub294 \uc911\uc694\ubbfc\uc18d\ubb38\ud654\uc7ac\ub294 \ud574\uc678\ub85c \uc218\ucd9c\ud558\uac70\ub098 \ubc18\ucd9c\ud560 \uc218 \uc5c6\ub2e4. \uc608\uc678\uc801\uc73c\ub85c \ubb38\ud654\uc7ac\ub97c \uc804\uc2dc \ub4f1 \uad6d\uc81c\uc801 \ubb38\ud654 \uad50\ub958\ub97c \ubaa9\uc801\uc73c\ub85c \ubc18\ucd9c\ud558\ub294 \uacbd\uc6b0\uc5d0\ub294 \uadf8 \ubc18\ucd9c\ub41c \ub0a0\ubd80\ud130 2\ub144 \uc774\ub0b4\uc5d0 \ub2e4\uc2dc \ubc18\uc785\ub420 \uac83\uc744 \uc870\uac74\uc73c\ub85c \ubb38\ud654\uc7ac\uccad\uc7a5\uc758 \ud5c8\uac00\ub97c \ubc1b\uc744 \uc218 \uc788\uc73c\uba70, \ud544\uc694\ud55c \uacbd\uc6b0\uc5d0\ub294 2\ub144\uc758 \ubc94\uc704\uc5d0\uc11c \uae30\uac04\uc744 \uc5f0\uc7a5\ud560 \uc218 \uc788\ub2e4. \ube44\uc9c0\uc815\ubb38\ud654\uc7ac\ub294 \uad6d\ub0b4 \ubc15\ubb3c\uad00 \ub4f1\uc774 \uc678\uad6d\uc758 \ubc15\ubb3c\uad00 \ub4f1\uc5d0\uac8c \ubc18\ucd9c\ud55c \ub0a0\ubd80\ud130 10\ub144 \uc774\ub0b4\uc5d0 \ub2e4\uc2dc \ubc18\uc785\ud558\ub294 \uacbd\uc6b0\uc640 \uc678\uad6d \uc815\ubd80\uac00 \uc778\uc99d\ud558\ub294 \ubc15\ubb3c\uad00\uc774\ub098 \ubb38\ud654\uc7ac \uad00\ub828 \ub2e8\uccb4\uac00 \uc790\uad6d\uc758 \ubc15\ubb3c\uad00 \ub4f1\uc5d0\uc11c \uc804\uc2dc\ud560 \ubaa9\uc801\uc73c\ub85c \uad6c\uc785 \ub610\ub294 \uae30\uc99d\ubc1b\uc544 \ubc18\ucd9c\ud558\ub294 \uacbd\uc6b0\uc5d0\ub294 \ubb38\ud654\uc7ac\uccad\uc7a5\uc758 \ud5c8\uac00\ub97c \ud1b5\ud574\uc11c \ud574\uc678 \ubc18\ucd9c\uc774 \uc778\uc815\ub41c\ub2e4.", "answer": "\ubb38\ud654\uc7ac\uccad\uc7a5", "sentence": "\ube44\uc9c0\uc815\ubb38\ud654\uc7ac\ub97c \ud3ec\ud568\ud55c \uc77c\ubc18 \ub3d9\uc0b0\ubb38\ud654\uc7ac\ub97c \ud574\uc678\ub85c \ubc18\ucd9c\ud560 \uacbd\uc6b0\uc5d0\ub294 \uc0ac\uc804\uc5d0 \ubb38\ud654\uc7ac\uccad\uc7a5 \uc758 \ud5c8\uac00\ub97c \ubc1b\ub3c4\ub85d \ud558\uace0 \uc788\ub2e4.", "paragraph_sentence": "\ud604\ud589 \ub300\ud55c\ubbfc\uad6d\uc758 \ubb38\ud654\uc7ac\ubcf4\ud638\ubc95\uc5d0 \ub530\ub974\uba74 \ube44\uc9c0\uc815\ubb38\ud654\uc7ac\ub97c \ud3ec\ud568\ud55c \uc77c\ubc18 \ub3d9\uc0b0\ubb38\ud654\uc7ac\ub97c \ud574\uc678\ub85c \ubc18\ucd9c\ud560 \uacbd\uc6b0\uc5d0\ub294 \uc0ac\uc804\uc5d0 \ubb38\ud654\uc7ac\uccad\uc7a5 \uc758 \ud5c8\uac00\ub97c \ubc1b\ub3c4\ub85d \ud558\uace0 \uc788\ub2e4. \uc6d0\uce59\uc801\uc73c\ub85c \uad6d\ubcf4, \ubcf4\ubb3c, \ucc9c\uc5f0\uae30\ub150\ubb3c \ub610\ub294 \uc911\uc694\ubbfc\uc18d\ubb38\ud654\uc7ac\ub294 \ud574\uc678\ub85c \uc218\ucd9c\ud558\uac70\ub098 \ubc18\ucd9c\ud560 \uc218 \uc5c6\ub2e4. \uc608\uc678\uc801\uc73c\ub85c \ubb38\ud654\uc7ac\ub97c \uc804\uc2dc \ub4f1 \uad6d\uc81c\uc801 \ubb38\ud654 \uad50\ub958\ub97c \ubaa9\uc801\uc73c\ub85c \ubc18\ucd9c\ud558\ub294 \uacbd\uc6b0\uc5d0\ub294 \uadf8 \ubc18\ucd9c\ub41c \ub0a0\ubd80\ud130 2\ub144 \uc774\ub0b4\uc5d0 \ub2e4\uc2dc \ubc18\uc785\ub420 \uac83\uc744 \uc870\uac74\uc73c\ub85c \ubb38\ud654\uc7ac\uccad\uc7a5\uc758 \ud5c8\uac00\ub97c \ubc1b\uc744 \uc218 \uc788\uc73c\uba70, \ud544\uc694\ud55c \uacbd\uc6b0\uc5d0\ub294 2\ub144\uc758 \ubc94\uc704\uc5d0\uc11c \uae30\uac04\uc744 \uc5f0\uc7a5\ud560 \uc218 \uc788\ub2e4. \ube44\uc9c0\uc815\ubb38\ud654\uc7ac\ub294 \uad6d\ub0b4 \ubc15\ubb3c\uad00 \ub4f1\uc774 \uc678\uad6d\uc758 \ubc15\ubb3c\uad00 \ub4f1\uc5d0\uac8c \ubc18\ucd9c\ud55c \ub0a0\ubd80\ud130 10\ub144 \uc774\ub0b4\uc5d0 \ub2e4\uc2dc \ubc18\uc785\ud558\ub294 \uacbd\uc6b0\uc640 \uc678\uad6d \uc815\ubd80\uac00 \uc778\uc99d\ud558\ub294 \ubc15\ubb3c\uad00\uc774\ub098 \ubb38\ud654\uc7ac \uad00\ub828 \ub2e8\uccb4\uac00 \uc790\uad6d\uc758 \ubc15\ubb3c\uad00 \ub4f1\uc5d0\uc11c \uc804\uc2dc\ud560 \ubaa9\uc801\uc73c\ub85c \uad6c\uc785 \ub610\ub294 \uae30\uc99d\ubc1b\uc544 \ubc18\ucd9c\ud558\ub294 \uacbd\uc6b0\uc5d0\ub294 \ubb38\ud654\uc7ac\uccad\uc7a5\uc758 \ud5c8\uac00\ub97c \ud1b5\ud574\uc11c \ud574\uc678 \ubc18\ucd9c\uc774 \uc778\uc815\ub41c\ub2e4.", "paragraph_answer": "\ud604\ud589 \ub300\ud55c\ubbfc\uad6d\uc758 \ubb38\ud654\uc7ac\ubcf4\ud638\ubc95\uc5d0 \ub530\ub974\uba74 \ube44\uc9c0\uc815\ubb38\ud654\uc7ac\ub97c \ud3ec\ud568\ud55c \uc77c\ubc18 \ub3d9\uc0b0\ubb38\ud654\uc7ac\ub97c \ud574\uc678\ub85c \ubc18\ucd9c\ud560 \uacbd\uc6b0\uc5d0\ub294 \uc0ac\uc804\uc5d0 \ubb38\ud654\uc7ac\uccad\uc7a5 \uc758 \ud5c8\uac00\ub97c \ubc1b\ub3c4\ub85d \ud558\uace0 \uc788\ub2e4. \uc6d0\uce59\uc801\uc73c\ub85c \uad6d\ubcf4, \ubcf4\ubb3c, \ucc9c\uc5f0\uae30\ub150\ubb3c \ub610\ub294 \uc911\uc694\ubbfc\uc18d\ubb38\ud654\uc7ac\ub294 \ud574\uc678\ub85c \uc218\ucd9c\ud558\uac70\ub098 \ubc18\ucd9c\ud560 \uc218 \uc5c6\ub2e4. \uc608\uc678\uc801\uc73c\ub85c \ubb38\ud654\uc7ac\ub97c \uc804\uc2dc \ub4f1 \uad6d\uc81c\uc801 \ubb38\ud654 \uad50\ub958\ub97c \ubaa9\uc801\uc73c\ub85c \ubc18\ucd9c\ud558\ub294 \uacbd\uc6b0\uc5d0\ub294 \uadf8 \ubc18\ucd9c\ub41c \ub0a0\ubd80\ud130 2\ub144 \uc774\ub0b4\uc5d0 \ub2e4\uc2dc \ubc18\uc785\ub420 \uac83\uc744 \uc870\uac74\uc73c\ub85c \ubb38\ud654\uc7ac\uccad\uc7a5\uc758 \ud5c8\uac00\ub97c \ubc1b\uc744 \uc218 \uc788\uc73c\uba70, \ud544\uc694\ud55c \uacbd\uc6b0\uc5d0\ub294 2\ub144\uc758 \ubc94\uc704\uc5d0\uc11c \uae30\uac04\uc744 \uc5f0\uc7a5\ud560 \uc218 \uc788\ub2e4. \ube44\uc9c0\uc815\ubb38\ud654\uc7ac\ub294 \uad6d\ub0b4 \ubc15\ubb3c\uad00 \ub4f1\uc774 \uc678\uad6d\uc758 \ubc15\ubb3c\uad00 \ub4f1\uc5d0\uac8c \ubc18\ucd9c\ud55c \ub0a0\ubd80\ud130 10\ub144 \uc774\ub0b4\uc5d0 \ub2e4\uc2dc \ubc18\uc785\ud558\ub294 \uacbd\uc6b0\uc640 \uc678\uad6d \uc815\ubd80\uac00 \uc778\uc99d\ud558\ub294 \ubc15\ubb3c\uad00\uc774\ub098 \ubb38\ud654\uc7ac \uad00\ub828 \ub2e8\uccb4\uac00 \uc790\uad6d\uc758 \ubc15\ubb3c\uad00 \ub4f1\uc5d0\uc11c \uc804\uc2dc\ud560 \ubaa9\uc801\uc73c\ub85c \uad6c\uc785 \ub610\ub294 \uae30\uc99d\ubc1b\uc544 \ubc18\ucd9c\ud558\ub294 \uacbd\uc6b0\uc5d0\ub294 \ubb38\ud654\uc7ac\uccad\uc7a5\uc758 \ud5c8\uac00\ub97c \ud1b5\ud574\uc11c \ud574\uc678 \ubc18\ucd9c\uc774 \uc778\uc815\ub41c\ub2e4.", "sentence_answer": "\ube44\uc9c0\uc815\ubb38\ud654\uc7ac\ub97c \ud3ec\ud568\ud55c \uc77c\ubc18 \ub3d9\uc0b0\ubb38\ud654\uc7ac\ub97c \ud574\uc678\ub85c \ubc18\ucd9c\ud560 \uacbd\uc6b0\uc5d0\ub294 \uc0ac\uc804\uc5d0 \ubb38\ud654\uc7ac\uccad\uc7a5 \uc758 \ud5c8\uac00\ub97c \ubc1b\ub3c4\ub85d \ud558\uace0 \uc788\ub2e4.", "paragraph_id": "6529514-7-0", "paragraph_question": "question: \ud604\uc7ac \ub300\ud55c\ubbfc\uad6d\uc740 \uc77c\ubc18 \ub3d9\uc0b0\ubb38\ud654\uc7ac\ub97c \ud574\uc678\ub85c \ubc18\ucd9c\ud560 \ub54c \ub204\uad6c\uc758 \ud5c8\uac00\ub97c \ubc1b\uc544\uc57c\ud558\ub294\uac00?, context: \ud604\ud589 \ub300\ud55c\ubbfc\uad6d\uc758 \ubb38\ud654\uc7ac\ubcf4\ud638\ubc95\uc5d0 \ub530\ub974\uba74 \ube44\uc9c0\uc815\ubb38\ud654\uc7ac\ub97c \ud3ec\ud568\ud55c \uc77c\ubc18 \ub3d9\uc0b0\ubb38\ud654\uc7ac\ub97c \ud574\uc678\ub85c \ubc18\ucd9c\ud560 \uacbd\uc6b0\uc5d0\ub294 \uc0ac\uc804\uc5d0 \ubb38\ud654\uc7ac\uccad\uc7a5\uc758 \ud5c8\uac00\ub97c \ubc1b\ub3c4\ub85d \ud558\uace0 \uc788\ub2e4. \uc6d0\uce59\uc801\uc73c\ub85c \uad6d\ubcf4, \ubcf4\ubb3c, \ucc9c\uc5f0\uae30\ub150\ubb3c \ub610\ub294 \uc911\uc694\ubbfc\uc18d\ubb38\ud654\uc7ac\ub294 \ud574\uc678\ub85c \uc218\ucd9c\ud558\uac70\ub098 \ubc18\ucd9c\ud560 \uc218 \uc5c6\ub2e4. \uc608\uc678\uc801\uc73c\ub85c \ubb38\ud654\uc7ac\ub97c \uc804\uc2dc \ub4f1 \uad6d\uc81c\uc801 \ubb38\ud654 \uad50\ub958\ub97c \ubaa9\uc801\uc73c\ub85c \ubc18\ucd9c\ud558\ub294 \uacbd\uc6b0\uc5d0\ub294 \uadf8 \ubc18\ucd9c\ub41c \ub0a0\ubd80\ud130 2\ub144 \uc774\ub0b4\uc5d0 \ub2e4\uc2dc \ubc18\uc785\ub420 \uac83\uc744 \uc870\uac74\uc73c\ub85c \ubb38\ud654\uc7ac\uccad\uc7a5\uc758 \ud5c8\uac00\ub97c \ubc1b\uc744 \uc218 \uc788\uc73c\uba70, \ud544\uc694\ud55c \uacbd\uc6b0\uc5d0\ub294 2\ub144\uc758 \ubc94\uc704\uc5d0\uc11c \uae30\uac04\uc744 \uc5f0\uc7a5\ud560 \uc218 \uc788\ub2e4. \ube44\uc9c0\uc815\ubb38\ud654\uc7ac\ub294 \uad6d\ub0b4 \ubc15\ubb3c\uad00 \ub4f1\uc774 \uc678\uad6d\uc758 \ubc15\ubb3c\uad00 \ub4f1\uc5d0\uac8c \ubc18\ucd9c\ud55c \ub0a0\ubd80\ud130 10\ub144 \uc774\ub0b4\uc5d0 \ub2e4\uc2dc \ubc18\uc785\ud558\ub294 \uacbd\uc6b0\uc640 \uc678\uad6d \uc815\ubd80\uac00 \uc778\uc99d\ud558\ub294 \ubc15\ubb3c\uad00\uc774\ub098 \ubb38\ud654\uc7ac \uad00\ub828 \ub2e8\uccb4\uac00 \uc790\uad6d\uc758 \ubc15\ubb3c\uad00 \ub4f1\uc5d0\uc11c \uc804\uc2dc\ud560 \ubaa9\uc801\uc73c\ub85c \uad6c\uc785 \ub610\ub294 \uae30\uc99d\ubc1b\uc544 \ubc18\ucd9c\ud558\ub294 \uacbd\uc6b0\uc5d0\ub294 \ubb38\ud654\uc7ac\uccad\uc7a5\uc758 \ud5c8\uac00\ub97c \ud1b5\ud574\uc11c \ud574\uc678 \ubc18\ucd9c\uc774 \uc778\uc815\ub41c\ub2e4."} {"question": "\ub2e8\uad70\uc5d0 \uc774\uc5b4 \uace0\uc870\uc120\uc744 \uc774\ub048 \uc655\uc740?", "paragraph": "\uadfc\ub300 \uc5ed\uc0ac\ud559\uc774 \ud615\uc131\ub418\uae30 \uc774\uc804\uc5d0\ub294 \uc9c0\ubc30\uc790\uc758 \uc774\ub984\uc5d0 \ub530\ub77c \uace0\uc870\uc120\uc758 \uc5ed\uc0ac\ub97c \ub2e8\uad70\uc870\uc120, \uae30\uc790\uc870\uc120, \uc704\ub9cc\uc870\uc120\uc758 3\uac1c \uc2dc\uae30\ub85c \uad6c\ubd84\ud558\uc600\ub2e4. \ub2e8\uad70\uc870\uc120\uc740 \ub2e8\uad70\uc774 \ubb34\uc9c4\ub144(\uae30\uc6d0\uc804 2333\ub144)\uc5d0 \uc544\uc0ac\ub2ec(\u963f\u65af\u9054) \ub610\ub294 \ud3c9\uc591(\u5e73\u58e4)\uc5d0\uc11c \uac74\uad6d\ud558\uc600\ub2e4\uace0 \ud55c\ub2e4. \ub2e8\uad70\uc740 \uc57d 1000\uc5ec \ub144 \ub3d9\uc548 \ub2e8\uad70\uc870\uc120\uc744 \ub2e4\uc2a4\ub9ac\ub2e4\uac00 \uae30\ubb18\ub144(\uae30\uc6d0\uc804 1122\ub144)\uc5d0 \uae30\uc790\uc5d0\uac8c \uc655\uc704\ub97c \ubb3c\ub824\uc8fc\uace0 \uc544\uc0ac\ub2ec\uc758 \uc0b0\uc2e0\uc774 \ub418\uc5c8\ub2e4. \ub610\ub294 \uae30\uc790\uac00 \uc624\uae30 164\ub144 \uc804\uc778 \uae30\uc6d0\uc804 1286\ub144\uc5d0 \uc655\uc704\ub97c \ub5a0\ub0ac\ub2e4\uace0 \ud558\uae30\ub3c4 \ud55c\ub2e4. \ud604\ub300 \ud55c\uad6d\uc758 \uc5ed\uc0ac\ud559\uacc4\uc5d0\uc11c\ub294 \uae30\uc790\uc870\uc120\uc744 \uc778\uc815\ud558\uc9c0 \uc54a\uae30 \ub54c\ubb38\uc5d0 \uae30\uc6d0\uc804 11\uc138\uae30 \ubb34\ub835\uc5d0 \ub2e8\uad70\uc870\uc120\uc774 \uc18c\uba78\ub418\uc5c8\ub2e4\ub294 \ub0b4\uc6a9 \uc5ed\uc2dc \uc778\uc815\ub418\uc9c0 \uc54a\uc73c\uba70 \uc704\ub9cc\uc870\uc120\uc774 \uc131\ub9bd\ub418\ub294 \uae30\uc6d0\uc804 194\ub144\uae4c\uc9c0\uc758 \uc5ed\uc0ac\ub97c \ubaa8\ub450 \ub2e8\uad70\uc870\uc120 \ubc0f \uace0\uc870\uc120\uc758 \uc5ed\uc0ac\ub85c \ubcf4\uace0 \uc788\ub2e4.", "answer": "\uae30\uc790", "sentence": "\uadfc\ub300 \uc5ed\uc0ac\ud559\uc774 \ud615\uc131\ub418\uae30 \uc774\uc804\uc5d0\ub294 \uc9c0\ubc30\uc790\uc758 \uc774\ub984\uc5d0 \ub530\ub77c \uace0\uc870\uc120\uc758 \uc5ed\uc0ac\ub97c \ub2e8\uad70\uc870\uc120, \uae30\uc790 \uc870\uc120, \uc704\ub9cc\uc870\uc120\uc758 3\uac1c \uc2dc\uae30\ub85c \uad6c\ubd84\ud558\uc600\ub2e4.", "paragraph_sentence": " \uadfc\ub300 \uc5ed\uc0ac\ud559\uc774 \ud615\uc131\ub418\uae30 \uc774\uc804\uc5d0\ub294 \uc9c0\ubc30\uc790\uc758 \uc774\ub984\uc5d0 \ub530\ub77c \uace0\uc870\uc120\uc758 \uc5ed\uc0ac\ub97c \ub2e8\uad70\uc870\uc120, \uae30\uc790 \uc870\uc120, \uc704\ub9cc\uc870\uc120\uc758 3\uac1c \uc2dc\uae30\ub85c \uad6c\ubd84\ud558\uc600\ub2e4. \ub2e8\uad70\uc870\uc120\uc740 \ub2e8\uad70\uc774 \ubb34\uc9c4\ub144(\uae30\uc6d0\uc804 2333\ub144)\uc5d0 \uc544\uc0ac\ub2ec(\u963f\u65af\u9054) \ub610\ub294 \ud3c9\uc591(\u5e73\u58e4)\uc5d0\uc11c \uac74\uad6d\ud558\uc600\ub2e4\uace0 \ud55c\ub2e4. \ub2e8\uad70\uc740 \uc57d 1000\uc5ec \ub144 \ub3d9\uc548 \ub2e8\uad70\uc870\uc120\uc744 \ub2e4\uc2a4\ub9ac\ub2e4\uac00 \uae30\ubb18\ub144(\uae30\uc6d0\uc804 1122\ub144)\uc5d0 \uae30\uc790\uc5d0\uac8c \uc655\uc704\ub97c \ubb3c\ub824\uc8fc\uace0 \uc544\uc0ac\ub2ec\uc758 \uc0b0\uc2e0\uc774 \ub418\uc5c8\ub2e4. \ub610\ub294 \uae30\uc790\uac00 \uc624\uae30 164\ub144 \uc804\uc778 \uae30\uc6d0\uc804 1286\ub144\uc5d0 \uc655\uc704\ub97c \ub5a0\ub0ac\ub2e4\uace0 \ud558\uae30\ub3c4 \ud55c\ub2e4. \ud604\ub300 \ud55c\uad6d\uc758 \uc5ed\uc0ac\ud559\uacc4\uc5d0\uc11c\ub294 \uae30\uc790\uc870\uc120\uc744 \uc778\uc815\ud558\uc9c0 \uc54a\uae30 \ub54c\ubb38\uc5d0 \uae30\uc6d0\uc804 11\uc138\uae30 \ubb34\ub835\uc5d0 \ub2e8\uad70\uc870\uc120\uc774 \uc18c\uba78\ub418\uc5c8\ub2e4\ub294 \ub0b4\uc6a9 \uc5ed\uc2dc \uc778\uc815\ub418\uc9c0 \uc54a\uc73c\uba70 \uc704\ub9cc\uc870\uc120\uc774 \uc131\ub9bd\ub418\ub294 \uae30\uc6d0\uc804 194\ub144\uae4c\uc9c0\uc758 \uc5ed\uc0ac\ub97c \ubaa8\ub450 \ub2e8\uad70\uc870\uc120 \ubc0f \uace0\uc870\uc120\uc758 \uc5ed\uc0ac\ub85c \ubcf4\uace0 \uc788\ub2e4.", "paragraph_answer": "\uadfc\ub300 \uc5ed\uc0ac\ud559\uc774 \ud615\uc131\ub418\uae30 \uc774\uc804\uc5d0\ub294 \uc9c0\ubc30\uc790\uc758 \uc774\ub984\uc5d0 \ub530\ub77c \uace0\uc870\uc120\uc758 \uc5ed\uc0ac\ub97c \ub2e8\uad70\uc870\uc120, \uae30\uc790 \uc870\uc120, \uc704\ub9cc\uc870\uc120\uc758 3\uac1c \uc2dc\uae30\ub85c \uad6c\ubd84\ud558\uc600\ub2e4. \ub2e8\uad70\uc870\uc120\uc740 \ub2e8\uad70\uc774 \ubb34\uc9c4\ub144(\uae30\uc6d0\uc804 2333\ub144)\uc5d0 \uc544\uc0ac\ub2ec(\u963f\u65af\u9054) \ub610\ub294 \ud3c9\uc591(\u5e73\u58e4)\uc5d0\uc11c \uac74\uad6d\ud558\uc600\ub2e4\uace0 \ud55c\ub2e4. \ub2e8\uad70\uc740 \uc57d 1000\uc5ec \ub144 \ub3d9\uc548 \ub2e8\uad70\uc870\uc120\uc744 \ub2e4\uc2a4\ub9ac\ub2e4\uac00 \uae30\ubb18\ub144(\uae30\uc6d0\uc804 1122\ub144)\uc5d0 \uae30\uc790\uc5d0\uac8c \uc655\uc704\ub97c \ubb3c\ub824\uc8fc\uace0 \uc544\uc0ac\ub2ec\uc758 \uc0b0\uc2e0\uc774 \ub418\uc5c8\ub2e4. \ub610\ub294 \uae30\uc790\uac00 \uc624\uae30 164\ub144 \uc804\uc778 \uae30\uc6d0\uc804 1286\ub144\uc5d0 \uc655\uc704\ub97c \ub5a0\ub0ac\ub2e4\uace0 \ud558\uae30\ub3c4 \ud55c\ub2e4. \ud604\ub300 \ud55c\uad6d\uc758 \uc5ed\uc0ac\ud559\uacc4\uc5d0\uc11c\ub294 \uae30\uc790\uc870\uc120\uc744 \uc778\uc815\ud558\uc9c0 \uc54a\uae30 \ub54c\ubb38\uc5d0 \uae30\uc6d0\uc804 11\uc138\uae30 \ubb34\ub835\uc5d0 \ub2e8\uad70\uc870\uc120\uc774 \uc18c\uba78\ub418\uc5c8\ub2e4\ub294 \ub0b4\uc6a9 \uc5ed\uc2dc \uc778\uc815\ub418\uc9c0 \uc54a\uc73c\uba70 \uc704\ub9cc\uc870\uc120\uc774 \uc131\ub9bd\ub418\ub294 \uae30\uc6d0\uc804 194\ub144\uae4c\uc9c0\uc758 \uc5ed\uc0ac\ub97c \ubaa8\ub450 \ub2e8\uad70\uc870\uc120 \ubc0f \uace0\uc870\uc120\uc758 \uc5ed\uc0ac\ub85c \ubcf4\uace0 \uc788\ub2e4.", "sentence_answer": "\uadfc\ub300 \uc5ed\uc0ac\ud559\uc774 \ud615\uc131\ub418\uae30 \uc774\uc804\uc5d0\ub294 \uc9c0\ubc30\uc790\uc758 \uc774\ub984\uc5d0 \ub530\ub77c \uace0\uc870\uc120\uc758 \uc5ed\uc0ac\ub97c \ub2e8\uad70\uc870\uc120, \uae30\uc790 \uc870\uc120, \uc704\ub9cc\uc870\uc120\uc758 3\uac1c \uc2dc\uae30\ub85c \uad6c\ubd84\ud558\uc600\ub2e4.", "paragraph_id": "6490822-0-0", "paragraph_question": "question: \ub2e8\uad70\uc5d0 \uc774\uc5b4 \uace0\uc870\uc120\uc744 \uc774\ub048 \uc655\uc740?, context: \uadfc\ub300 \uc5ed\uc0ac\ud559\uc774 \ud615\uc131\ub418\uae30 \uc774\uc804\uc5d0\ub294 \uc9c0\ubc30\uc790\uc758 \uc774\ub984\uc5d0 \ub530\ub77c \uace0\uc870\uc120\uc758 \uc5ed\uc0ac\ub97c \ub2e8\uad70\uc870\uc120, \uae30\uc790\uc870\uc120, \uc704\ub9cc\uc870\uc120\uc758 3\uac1c \uc2dc\uae30\ub85c \uad6c\ubd84\ud558\uc600\ub2e4. \ub2e8\uad70\uc870\uc120\uc740 \ub2e8\uad70\uc774 \ubb34\uc9c4\ub144(\uae30\uc6d0\uc804 2333\ub144)\uc5d0 \uc544\uc0ac\ub2ec(\u963f\u65af\u9054) \ub610\ub294 \ud3c9\uc591(\u5e73\u58e4)\uc5d0\uc11c \uac74\uad6d\ud558\uc600\ub2e4\uace0 \ud55c\ub2e4. \ub2e8\uad70\uc740 \uc57d 1000\uc5ec \ub144 \ub3d9\uc548 \ub2e8\uad70\uc870\uc120\uc744 \ub2e4\uc2a4\ub9ac\ub2e4\uac00 \uae30\ubb18\ub144(\uae30\uc6d0\uc804 1122\ub144)\uc5d0 \uae30\uc790\uc5d0\uac8c \uc655\uc704\ub97c \ubb3c\ub824\uc8fc\uace0 \uc544\uc0ac\ub2ec\uc758 \uc0b0\uc2e0\uc774 \ub418\uc5c8\ub2e4. \ub610\ub294 \uae30\uc790\uac00 \uc624\uae30 164\ub144 \uc804\uc778 \uae30\uc6d0\uc804 1286\ub144\uc5d0 \uc655\uc704\ub97c \ub5a0\ub0ac\ub2e4\uace0 \ud558\uae30\ub3c4 \ud55c\ub2e4. \ud604\ub300 \ud55c\uad6d\uc758 \uc5ed\uc0ac\ud559\uacc4\uc5d0\uc11c\ub294 \uae30\uc790\uc870\uc120\uc744 \uc778\uc815\ud558\uc9c0 \uc54a\uae30 \ub54c\ubb38\uc5d0 \uae30\uc6d0\uc804 11\uc138\uae30 \ubb34\ub835\uc5d0 \ub2e8\uad70\uc870\uc120\uc774 \uc18c\uba78\ub418\uc5c8\ub2e4\ub294 \ub0b4\uc6a9 \uc5ed\uc2dc \uc778\uc815\ub418\uc9c0 \uc54a\uc73c\uba70 \uc704\ub9cc\uc870\uc120\uc774 \uc131\ub9bd\ub418\ub294 \uae30\uc6d0\uc804 194\ub144\uae4c\uc9c0\uc758 \uc5ed\uc0ac\ub97c \ubaa8\ub450 \ub2e8\uad70\uc870\uc120 \ubc0f \uace0\uc870\uc120\uc758 \uc5ed\uc0ac\ub85c \ubcf4\uace0 \uc788\ub2e4."} {"question": "\ud504\ub780\ucc28\uc774\uc988 \uacc4\uc57d\uad00\uacc4\uac00 \ud574\ub2f9\ud558\ub294 \uacc4\uc57d\uad00\uacc4\ub294?", "paragraph": "\uc874\uc18d\uae30\uac04\uc758 \uc815\ud568\uc774 \uc788\ub294 \uacc4\uc18d\uc801 \uacc4\uc57d\uad00\uacc4\ub294 \uadf8 \uae30\uac04\uc774 \ub9cc\ub8cc\ub418\uba74 \uc885\ub8cc\ud55c\ub2e4. \ud55c\ud3b8 \uadf8 \uacc4\uc57d\uc5d0\uc11c \uacc4\uc57d\uc758 \uac31\uc2e0 \ub610\ub294 \uc874\uc18d\uae30\uac04\uc758 \uc5f0\uc7a5\uc5d0 \uad00\ud558\uc5ec \ubcc4\ub3c4\uc758 \uc57d\uc815\uc774 \uc788\ub294 \uacbd\uc6b0\uc5d0\ub294 \uadf8 \uc57d\uc815\uc774 \uc815\ud558\ub294 \ubc14\uc5d0 \ub530\ub77c \uacc4\uc57d\uc774 \uac31\uc2e0\ub418\uac70\ub098 \uc874\uc18d\uae30\uac04\uc774 \uc5f0\uc7a5\ub418\uace0, \uadf8\ub7ec\ud55c \uc57d\uc815\uc774 \uc5c6\ub294 \uacbd\uc6b0\uc5d0\ub294 \ubc95\uc815\uac31\uc2e0 \ub4f1\uc5d0 \uad00\ud55c \ubcc4\ub3c4\uc758 \ubc95\uaddc\uc815\uc774 \uc5c6\ub294 \ud55c \ub2f9\uc0ac\uc790\uac00 \uc0c8\ub85c\uc774 \uacc4\uc57d\uc758 \uac31\uc2e0 \ub4f1\uc5d0 \uad00\ud558\uc5ec \ud569\uc758\ud558\uc5ec\uc57c \ud55c\ub2e4. \uc774\ub294 \uacc4\uc18d\uc801 \uacc4\uc57d\uad00\uacc4\uc5d0 \ud574\ub2f9\ud558\ub294 \uac00\ub9f9\uc810(\ud504\ub79c\ucc28\uc774\uc988)\uacc4\uc57d\uad00\uacc4\uc5d0\uc11c\ub3c4 \ub2e4\ub97c \ubc14 \uc5c6\ub2e4. \ub530\ub77c\uc11c \ubc95 \uaddc\uc815 \ub610\ub294 \ub2f9\ud574 \uac00\ub9f9\uc810\uacc4\uc57d\uc758 \ud574\uc11d\uc5d0 \uc887\uc544 \uac00\ub9f9\uc810\uc0ac\uc5c5\uc790\uac00 \uac00\ub9f9\ubcf8\ubd80\uc5d0 \ub300\ud558\uc5ec \uac31\uc2e0\uc744 \uccad\uad6c\ud560 \uad8c\ub9ac\ub97c \uac00\uc9c0\uac70\ub098, \uac00\ub9f9\ubcf8\ubd80\uc758 \uac31\uc2e0 \uac70\uc808\uc774 \ub2f9\ud574 \uac00\ub9f9\uc810\uacc4\uc57d\uc758 \uccb4\uacb0 \uacbd\uc704\u00b7\ubaa9\uc801\uc774\ub098 \ub0b4\uc6a9, \uadf8 \uacc4\uc57d\uad00\uacc4\uc758 \uc804\uac1c \uc591\uc0c1, \ub2f9\uc0ac\uc790\uc758 \uc774\uc775 \uc0c1\ud669 \ubc0f \uac00\ub9f9\uc810\uacc4\uc57d \uc77c\ubc18\uc758 \uace0\uc720\ud55c \ud2b9\uc131 \ub4f1\uc5d0 \ube44\ucd94\uc5b4 \uc2e0\uc758\uce59\uc5d0 \ubc18\ud558\uc5ec \ud5c8\uc6a9\ub418\uc9c0 \uc544\ub2c8\ud558\ub294 \ub4f1\uc758 \ud2b9\ubcc4\ud55c \uc0ac\uc815\uc774 \uc5c6\ub294 \ud55c, \uac00\ub9f9\ubcf8\ubd80\ub294 \uac00\ub9f9\uc810\uc0ac\uc5c5\uc790\uc758 \uac31\uc2e0 \uc694\uccad\uc744 \ubc1b\uc544\ub4e4\uc5ec \uac31\uc2e0 \ub4f1\uc5d0 \ud569\uc758\ud560 \uac83\uc778\uc9c0 \uc5ec\ubd80\ub97c \uc2a4\uc2a4\ub85c \ud310\ub2e8\u00b7\uacb0\uc815\ud560 \uc790\uc720\ub97c \uac00\uc9c0\uba70, \uadf8\uc5d0 \uc788\uc5b4\uc11c \uc815\ub2f9\ud55c \uc0ac\uc720 \ub610\ub294 \ud569\ub9ac\uc801 \uc0ac\uc720\uac00 \uc788\ub294 \uacbd\uc6b0\uc5d0 \ud55c\ud558\uc5ec \uac31\uc2e0\uc744 \uac70\uc808\ud560 \uc218 \uc788\ub294 \uac83\uc740 \uc544\ub2c8\ub2e4.", "answer": "\uacc4\uc18d\uc801 \uacc4\uc57d\uad00\uacc4", "sentence": "\uc874\uc18d\uae30\uac04\uc758 \uc815\ud568\uc774 \uc788\ub294 \uacc4\uc18d\uc801 \uacc4\uc57d\uad00\uacc4 \ub294 \uadf8 \uae30\uac04\uc774 \ub9cc\ub8cc\ub418\uba74 \uc885\ub8cc\ud55c\ub2e4.", "paragraph_sentence": " \uc874\uc18d\uae30\uac04\uc758 \uc815\ud568\uc774 \uc788\ub294 \uacc4\uc18d\uc801 \uacc4\uc57d\uad00\uacc4 \ub294 \uadf8 \uae30\uac04\uc774 \ub9cc\ub8cc\ub418\uba74 \uc885\ub8cc\ud55c\ub2e4. \ud55c\ud3b8 \uadf8 \uacc4\uc57d\uc5d0\uc11c \uacc4\uc57d\uc758 \uac31\uc2e0 \ub610\ub294 \uc874\uc18d\uae30\uac04\uc758 \uc5f0\uc7a5\uc5d0 \uad00\ud558\uc5ec \ubcc4\ub3c4\uc758 \uc57d\uc815\uc774 \uc788\ub294 \uacbd\uc6b0\uc5d0\ub294 \uadf8 \uc57d\uc815\uc774 \uc815\ud558\ub294 \ubc14\uc5d0 \ub530\ub77c \uacc4\uc57d\uc774 \uac31\uc2e0\ub418\uac70\ub098 \uc874\uc18d\uae30\uac04\uc774 \uc5f0\uc7a5\ub418\uace0, \uadf8\ub7ec\ud55c \uc57d\uc815\uc774 \uc5c6\ub294 \uacbd\uc6b0\uc5d0\ub294 \ubc95\uc815\uac31\uc2e0 \ub4f1\uc5d0 \uad00\ud55c \ubcc4\ub3c4\uc758 \ubc95\uaddc\uc815\uc774 \uc5c6\ub294 \ud55c \ub2f9\uc0ac\uc790\uac00 \uc0c8\ub85c\uc774 \uacc4\uc57d\uc758 \uac31\uc2e0 \ub4f1\uc5d0 \uad00\ud558\uc5ec \ud569\uc758\ud558\uc5ec\uc57c \ud55c\ub2e4. \uc774\ub294 \uacc4\uc18d\uc801 \uacc4\uc57d\uad00\uacc4\uc5d0 \ud574\ub2f9\ud558\ub294 \uac00\ub9f9\uc810(\ud504\ub79c\ucc28\uc774\uc988)\uacc4\uc57d\uad00\uacc4\uc5d0\uc11c\ub3c4 \ub2e4\ub97c \ubc14 \uc5c6\ub2e4. \ub530\ub77c\uc11c \ubc95 \uaddc\uc815 \ub610\ub294 \ub2f9\ud574 \uac00\ub9f9\uc810\uacc4\uc57d\uc758 \ud574\uc11d\uc5d0 \uc887\uc544 \uac00\ub9f9\uc810\uc0ac\uc5c5\uc790\uac00 \uac00\ub9f9\ubcf8\ubd80\uc5d0 \ub300\ud558\uc5ec \uac31\uc2e0\uc744 \uccad\uad6c\ud560 \uad8c\ub9ac\ub97c \uac00\uc9c0\uac70\ub098, \uac00\ub9f9\ubcf8\ubd80\uc758 \uac31\uc2e0 \uac70\uc808\uc774 \ub2f9\ud574 \uac00\ub9f9\uc810\uacc4\uc57d\uc758 \uccb4\uacb0 \uacbd\uc704\u00b7\ubaa9\uc801\uc774\ub098 \ub0b4\uc6a9, \uadf8 \uacc4\uc57d\uad00\uacc4\uc758 \uc804\uac1c \uc591\uc0c1, \ub2f9\uc0ac\uc790\uc758 \uc774\uc775 \uc0c1\ud669 \ubc0f \uac00\ub9f9\uc810\uacc4\uc57d \uc77c\ubc18\uc758 \uace0\uc720\ud55c \ud2b9\uc131 \ub4f1\uc5d0 \ube44\ucd94\uc5b4 \uc2e0\uc758\uce59\uc5d0 \ubc18\ud558\uc5ec \ud5c8\uc6a9\ub418\uc9c0 \uc544\ub2c8\ud558\ub294 \ub4f1\uc758 \ud2b9\ubcc4\ud55c \uc0ac\uc815\uc774 \uc5c6\ub294 \ud55c, \uac00\ub9f9\ubcf8\ubd80\ub294 \uac00\ub9f9\uc810\uc0ac\uc5c5\uc790\uc758 \uac31\uc2e0 \uc694\uccad\uc744 \ubc1b\uc544\ub4e4\uc5ec \uac31\uc2e0 \ub4f1\uc5d0 \ud569\uc758\ud560 \uac83\uc778\uc9c0 \uc5ec\ubd80\ub97c \uc2a4\uc2a4\ub85c \ud310\ub2e8\u00b7\uacb0\uc815\ud560 \uc790\uc720\ub97c \uac00\uc9c0\uba70, \uadf8\uc5d0 \uc788\uc5b4\uc11c \uc815\ub2f9\ud55c \uc0ac\uc720 \ub610\ub294 \ud569\ub9ac\uc801 \uc0ac\uc720\uac00 \uc788\ub294 \uacbd\uc6b0\uc5d0 \ud55c\ud558\uc5ec \uac31\uc2e0\uc744 \uac70\uc808\ud560 \uc218 \uc788\ub294 \uac83\uc740 \uc544\ub2c8\ub2e4.", "paragraph_answer": "\uc874\uc18d\uae30\uac04\uc758 \uc815\ud568\uc774 \uc788\ub294 \uacc4\uc18d\uc801 \uacc4\uc57d\uad00\uacc4 \ub294 \uadf8 \uae30\uac04\uc774 \ub9cc\ub8cc\ub418\uba74 \uc885\ub8cc\ud55c\ub2e4. \ud55c\ud3b8 \uadf8 \uacc4\uc57d\uc5d0\uc11c \uacc4\uc57d\uc758 \uac31\uc2e0 \ub610\ub294 \uc874\uc18d\uae30\uac04\uc758 \uc5f0\uc7a5\uc5d0 \uad00\ud558\uc5ec \ubcc4\ub3c4\uc758 \uc57d\uc815\uc774 \uc788\ub294 \uacbd\uc6b0\uc5d0\ub294 \uadf8 \uc57d\uc815\uc774 \uc815\ud558\ub294 \ubc14\uc5d0 \ub530\ub77c \uacc4\uc57d\uc774 \uac31\uc2e0\ub418\uac70\ub098 \uc874\uc18d\uae30\uac04\uc774 \uc5f0\uc7a5\ub418\uace0, \uadf8\ub7ec\ud55c \uc57d\uc815\uc774 \uc5c6\ub294 \uacbd\uc6b0\uc5d0\ub294 \ubc95\uc815\uac31\uc2e0 \ub4f1\uc5d0 \uad00\ud55c \ubcc4\ub3c4\uc758 \ubc95\uaddc\uc815\uc774 \uc5c6\ub294 \ud55c \ub2f9\uc0ac\uc790\uac00 \uc0c8\ub85c\uc774 \uacc4\uc57d\uc758 \uac31\uc2e0 \ub4f1\uc5d0 \uad00\ud558\uc5ec \ud569\uc758\ud558\uc5ec\uc57c \ud55c\ub2e4. \uc774\ub294 \uacc4\uc18d\uc801 \uacc4\uc57d\uad00\uacc4\uc5d0 \ud574\ub2f9\ud558\ub294 \uac00\ub9f9\uc810(\ud504\ub79c\ucc28\uc774\uc988)\uacc4\uc57d\uad00\uacc4\uc5d0\uc11c\ub3c4 \ub2e4\ub97c \ubc14 \uc5c6\ub2e4. \ub530\ub77c\uc11c \ubc95 \uaddc\uc815 \ub610\ub294 \ub2f9\ud574 \uac00\ub9f9\uc810\uacc4\uc57d\uc758 \ud574\uc11d\uc5d0 \uc887\uc544 \uac00\ub9f9\uc810\uc0ac\uc5c5\uc790\uac00 \uac00\ub9f9\ubcf8\ubd80\uc5d0 \ub300\ud558\uc5ec \uac31\uc2e0\uc744 \uccad\uad6c\ud560 \uad8c\ub9ac\ub97c \uac00\uc9c0\uac70\ub098, \uac00\ub9f9\ubcf8\ubd80\uc758 \uac31\uc2e0 \uac70\uc808\uc774 \ub2f9\ud574 \uac00\ub9f9\uc810\uacc4\uc57d\uc758 \uccb4\uacb0 \uacbd\uc704\u00b7\ubaa9\uc801\uc774\ub098 \ub0b4\uc6a9, \uadf8 \uacc4\uc57d\uad00\uacc4\uc758 \uc804\uac1c \uc591\uc0c1, \ub2f9\uc0ac\uc790\uc758 \uc774\uc775 \uc0c1\ud669 \ubc0f \uac00\ub9f9\uc810\uacc4\uc57d \uc77c\ubc18\uc758 \uace0\uc720\ud55c \ud2b9\uc131 \ub4f1\uc5d0 \ube44\ucd94\uc5b4 \uc2e0\uc758\uce59\uc5d0 \ubc18\ud558\uc5ec \ud5c8\uc6a9\ub418\uc9c0 \uc544\ub2c8\ud558\ub294 \ub4f1\uc758 \ud2b9\ubcc4\ud55c \uc0ac\uc815\uc774 \uc5c6\ub294 \ud55c, \uac00\ub9f9\ubcf8\ubd80\ub294 \uac00\ub9f9\uc810\uc0ac\uc5c5\uc790\uc758 \uac31\uc2e0 \uc694\uccad\uc744 \ubc1b\uc544\ub4e4\uc5ec \uac31\uc2e0 \ub4f1\uc5d0 \ud569\uc758\ud560 \uac83\uc778\uc9c0 \uc5ec\ubd80\ub97c \uc2a4\uc2a4\ub85c \ud310\ub2e8\u00b7\uacb0\uc815\ud560 \uc790\uc720\ub97c \uac00\uc9c0\uba70, \uadf8\uc5d0 \uc788\uc5b4\uc11c \uc815\ub2f9\ud55c \uc0ac\uc720 \ub610\ub294 \ud569\ub9ac\uc801 \uc0ac\uc720\uac00 \uc788\ub294 \uacbd\uc6b0\uc5d0 \ud55c\ud558\uc5ec \uac31\uc2e0\uc744 \uac70\uc808\ud560 \uc218 \uc788\ub294 \uac83\uc740 \uc544\ub2c8\ub2e4.", "sentence_answer": "\uc874\uc18d\uae30\uac04\uc758 \uc815\ud568\uc774 \uc788\ub294 \uacc4\uc18d\uc801 \uacc4\uc57d\uad00\uacc4 \ub294 \uadf8 \uae30\uac04\uc774 \ub9cc\ub8cc\ub418\uba74 \uc885\ub8cc\ud55c\ub2e4.", "paragraph_id": "6527244-16-1", "paragraph_question": "question: \ud504\ub780\ucc28\uc774\uc988 \uacc4\uc57d\uad00\uacc4\uac00 \ud574\ub2f9\ud558\ub294 \uacc4\uc57d\uad00\uacc4\ub294?, context: \uc874\uc18d\uae30\uac04\uc758 \uc815\ud568\uc774 \uc788\ub294 \uacc4\uc18d\uc801 \uacc4\uc57d\uad00\uacc4\ub294 \uadf8 \uae30\uac04\uc774 \ub9cc\ub8cc\ub418\uba74 \uc885\ub8cc\ud55c\ub2e4. \ud55c\ud3b8 \uadf8 \uacc4\uc57d\uc5d0\uc11c \uacc4\uc57d\uc758 \uac31\uc2e0 \ub610\ub294 \uc874\uc18d\uae30\uac04\uc758 \uc5f0\uc7a5\uc5d0 \uad00\ud558\uc5ec \ubcc4\ub3c4\uc758 \uc57d\uc815\uc774 \uc788\ub294 \uacbd\uc6b0\uc5d0\ub294 \uadf8 \uc57d\uc815\uc774 \uc815\ud558\ub294 \ubc14\uc5d0 \ub530\ub77c \uacc4\uc57d\uc774 \uac31\uc2e0\ub418\uac70\ub098 \uc874\uc18d\uae30\uac04\uc774 \uc5f0\uc7a5\ub418\uace0, \uadf8\ub7ec\ud55c \uc57d\uc815\uc774 \uc5c6\ub294 \uacbd\uc6b0\uc5d0\ub294 \ubc95\uc815\uac31\uc2e0 \ub4f1\uc5d0 \uad00\ud55c \ubcc4\ub3c4\uc758 \ubc95\uaddc\uc815\uc774 \uc5c6\ub294 \ud55c \ub2f9\uc0ac\uc790\uac00 \uc0c8\ub85c\uc774 \uacc4\uc57d\uc758 \uac31\uc2e0 \ub4f1\uc5d0 \uad00\ud558\uc5ec \ud569\uc758\ud558\uc5ec\uc57c \ud55c\ub2e4. \uc774\ub294 \uacc4\uc18d\uc801 \uacc4\uc57d\uad00\uacc4\uc5d0 \ud574\ub2f9\ud558\ub294 \uac00\ub9f9\uc810(\ud504\ub79c\ucc28\uc774\uc988)\uacc4\uc57d\uad00\uacc4\uc5d0\uc11c\ub3c4 \ub2e4\ub97c \ubc14 \uc5c6\ub2e4. \ub530\ub77c\uc11c \ubc95 \uaddc\uc815 \ub610\ub294 \ub2f9\ud574 \uac00\ub9f9\uc810\uacc4\uc57d\uc758 \ud574\uc11d\uc5d0 \uc887\uc544 \uac00\ub9f9\uc810\uc0ac\uc5c5\uc790\uac00 \uac00\ub9f9\ubcf8\ubd80\uc5d0 \ub300\ud558\uc5ec \uac31\uc2e0\uc744 \uccad\uad6c\ud560 \uad8c\ub9ac\ub97c \uac00\uc9c0\uac70\ub098, \uac00\ub9f9\ubcf8\ubd80\uc758 \uac31\uc2e0 \uac70\uc808\uc774 \ub2f9\ud574 \uac00\ub9f9\uc810\uacc4\uc57d\uc758 \uccb4\uacb0 \uacbd\uc704\u00b7\ubaa9\uc801\uc774\ub098 \ub0b4\uc6a9, \uadf8 \uacc4\uc57d\uad00\uacc4\uc758 \uc804\uac1c \uc591\uc0c1, \ub2f9\uc0ac\uc790\uc758 \uc774\uc775 \uc0c1\ud669 \ubc0f \uac00\ub9f9\uc810\uacc4\uc57d \uc77c\ubc18\uc758 \uace0\uc720\ud55c \ud2b9\uc131 \ub4f1\uc5d0 \ube44\ucd94\uc5b4 \uc2e0\uc758\uce59\uc5d0 \ubc18\ud558\uc5ec \ud5c8\uc6a9\ub418\uc9c0 \uc544\ub2c8\ud558\ub294 \ub4f1\uc758 \ud2b9\ubcc4\ud55c \uc0ac\uc815\uc774 \uc5c6\ub294 \ud55c, \uac00\ub9f9\ubcf8\ubd80\ub294 \uac00\ub9f9\uc810\uc0ac\uc5c5\uc790\uc758 \uac31\uc2e0 \uc694\uccad\uc744 \ubc1b\uc544\ub4e4\uc5ec \uac31\uc2e0 \ub4f1\uc5d0 \ud569\uc758\ud560 \uac83\uc778\uc9c0 \uc5ec\ubd80\ub97c \uc2a4\uc2a4\ub85c \ud310\ub2e8\u00b7\uacb0\uc815\ud560 \uc790\uc720\ub97c \uac00\uc9c0\uba70, \uadf8\uc5d0 \uc788\uc5b4\uc11c \uc815\ub2f9\ud55c \uc0ac\uc720 \ub610\ub294 \ud569\ub9ac\uc801 \uc0ac\uc720\uac00 \uc788\ub294 \uacbd\uc6b0\uc5d0 \ud55c\ud558\uc5ec \uac31\uc2e0\uc744 \uac70\uc808\ud560 \uc218 \uc788\ub294 \uac83\uc740 \uc544\ub2c8\ub2e4."} {"question": "\ud0dc\uc591\uc774 \uc790\uc804\ud558\uba74\uc11c \uc790\uae30\uc7a5\uc774 \ub4a4\ud2c0\ub9ac\uba74\uc11c \uadf8 \ub4a4\ud2c0\ub9bc\uc774 \ucd5c\ub300\uac00 \ub418\ub294 \uacf3\uc5d0 \ubc1c\uc0dd\ud558\ub294 \uac83\uc740?", "paragraph": "\ub458\uc9f8, \ud0dc\uc591\uc758 \ubcc0\ud654\ub85c \uc778\ud55c \uaca9\ubcc0\uc758 \uac00\ub2a5\uc131\uc774\ub2e4. \ud0dc\uc591\uc740 \uae30\uccb4\ub85c \ub418\uc5b4 \uc788\uc9c0\ub9cc \uc628\ub3c4\ub098 \uc555\ub825\uc774 \uc5c4\uccad\ub098\uae30 \ub54c\ubb38\uc5d0 \uc911\uc2ec\uc5d0\uc11c \uc6d0\uc790\ub4e4\uc774 \uc11c\ub85c \uc735\ud569\ud558\ub294 \ud575\uc735\ud569 \ubc18\uc751\uc744 \uc77c\uc73c\ud0a8\ub2e4. \ud0dc\uc591\uc5d0\uc11c\ub294 1\ucd08\uc5d0 4-5\uc5b5\ud1a4 \uc9c8\ub7c9\uc774 \uc5d0\ub108\uc9c0\ub85c \ubc14\ub00c\uba74\uc11c \uc9c0\uad6c \uc0dd\uba85\uccb4\ub4e4\uc774 \uc0b4\uc544\uac00\ub294\ub370 \ud544\uc694\ud55c \uc5f4\uacfc \ube5b\uc744 \ubc29\ucd9c\ud558\uace0 \uc788\ub2e4. \ub610\ud55c \ud0dc\uc591\uc740 \uc911\uc2ec\uc73c\ub85c\ubd80\ud130 \uac15\ub825\ud55c \uc790\uae30\uc7a5\uc744 \ubc29\ucd9c\ud558\uc5ec \uc6b0\uc8fc \uacf5\uac04\uc73c\ub85c \uc3d8\uc544 \ubcf4\ub0b4\uace0 \uc788\ub2e4. \uc774\ub7ec\ud55c \ud0dc\uc591\uc774 \uc790\uc804\ud558\uac8c \ub418\uba74 \ud0dc\uc591\uc758 \uc801\ub3c4\uc9c0\ubc29\uc774 \uadf9\uc9c0\ubc29\uc5d0 \ube44\ud574 \uc18d\ub3c4\uac00 \ube60\ub974\uace0 \uc774\ub7ec\ud55c \uc18d\ub3c4\uc758 \ucc28\uc774\ub85c \uc778\ud574 \uc790\uae30\uc7a5\uc774 \ub4a4\ud2c0\ub9ac\uac8c \ub41c\ub2e4. \uadf8\ub9ac\uace0 \uc774 \ub4a4\ud2c0\ub9bc\uc774 \ucd5c\ub300\uac00 \ub418\ub294 \uacf3\uc5d0 \uc6b0\ub9ac\uac00 \ud754\ud788 \ud0dc\uc591\uc758 \ud751\uc810\uc774\ub77c\uace0 \ub9d0\ud558\ub294 \uac83\uc774 \ud615\uc131\ub41c\ub2e4. \ud751\uc810\uc758 \uc22b\uc790\ub294 \ucd5c\ub300\uc640 \ucd5c\uc18c \uc0ac\uc774\ub97c \uc8fc\uae30\uc801\uc73c\ub85c \ubc18\ubcf5\ud55c\ub2e4. \uacfc\ud559\uc790\ub4e4\uc740 \ud751\uc810 \uc8fc\uae30\uc640 \uc9c0\uad6c\uc758 \uae30\ud6c4\uc758 \ubcc0\ud654 \uc0ac\uc774\uc5d0 \uc5b4\ub5a4 \uad00\uacc4\uac00 \uc788\uc9c0 \uc54a\uc740\uc9c0\ub97c \uc5f0\uad6c\ud55c\ub2e4. \ub9cc\uc77c \uc815\ub9d0 \uc5b4\ub5a4 \uad00\uacc4\uac00 \uc874\uc7ac\ud55c\ub2e4\uba74 \ud751\uc815 \uc8fc\uae30\uc758 \ubcc0\ud654\ub85c \uc778\ud574 \uc18c\ube59\ud558\uae30\uc640 \uc18c\uac04\ube59\uae30\uac00 \ubc18\ubcf5\ub420 \uac83\uc774\ub2e4.", "answer": "\ud751\uc810", "sentence": "\uadf8\ub9ac\uace0 \uc774 \ub4a4\ud2c0\ub9bc\uc774 \ucd5c\ub300\uac00 \ub418\ub294 \uacf3\uc5d0 \uc6b0\ub9ac\uac00 \ud754\ud788 \ud0dc\uc591\uc758 \ud751\uc810 \uc774\ub77c\uace0 \ub9d0\ud558\ub294 \uac83\uc774 \ud615\uc131\ub41c\ub2e4.", "paragraph_sentence": "\ub458\uc9f8, \ud0dc\uc591\uc758 \ubcc0\ud654\ub85c \uc778\ud55c \uaca9\ubcc0\uc758 \uac00\ub2a5\uc131\uc774\ub2e4. \ud0dc\uc591\uc740 \uae30\uccb4\ub85c \ub418\uc5b4 \uc788\uc9c0\ub9cc \uc628\ub3c4\ub098 \uc555\ub825\uc774 \uc5c4\uccad\ub098\uae30 \ub54c\ubb38\uc5d0 \uc911\uc2ec\uc5d0\uc11c \uc6d0\uc790\ub4e4\uc774 \uc11c\ub85c \uc735\ud569\ud558\ub294 \ud575\uc735\ud569 \ubc18\uc751\uc744 \uc77c\uc73c\ud0a8\ub2e4. \ud0dc\uc591\uc5d0\uc11c\ub294 1\ucd08\uc5d0 4-5\uc5b5\ud1a4 \uc9c8\ub7c9\uc774 \uc5d0\ub108\uc9c0\ub85c \ubc14\ub00c\uba74\uc11c \uc9c0\uad6c \uc0dd\uba85\uccb4\ub4e4\uc774 \uc0b4\uc544\uac00\ub294\ub370 \ud544\uc694\ud55c \uc5f4\uacfc \ube5b\uc744 \ubc29\ucd9c\ud558\uace0 \uc788\ub2e4. \ub610\ud55c \ud0dc\uc591\uc740 \uc911\uc2ec\uc73c\ub85c\ubd80\ud130 \uac15\ub825\ud55c \uc790\uae30\uc7a5\uc744 \ubc29\ucd9c\ud558\uc5ec \uc6b0\uc8fc \uacf5\uac04\uc73c\ub85c \uc3d8\uc544 \ubcf4\ub0b4\uace0 \uc788\ub2e4. \uc774\ub7ec\ud55c \ud0dc\uc591\uc774 \uc790\uc804\ud558\uac8c \ub418\uba74 \ud0dc\uc591\uc758 \uc801\ub3c4\uc9c0\ubc29\uc774 \uadf9\uc9c0\ubc29\uc5d0 \ube44\ud574 \uc18d\ub3c4\uac00 \ube60\ub974\uace0 \uc774\ub7ec\ud55c \uc18d\ub3c4\uc758 \ucc28\uc774\ub85c \uc778\ud574 \uc790\uae30\uc7a5\uc774 \ub4a4\ud2c0\ub9ac\uac8c \ub41c\ub2e4. \uadf8\ub9ac\uace0 \uc774 \ub4a4\ud2c0\ub9bc\uc774 \ucd5c\ub300\uac00 \ub418\ub294 \uacf3\uc5d0 \uc6b0\ub9ac\uac00 \ud754\ud788 \ud0dc\uc591\uc758 \ud751\uc810 \uc774\ub77c\uace0 \ub9d0\ud558\ub294 \uac83\uc774 \ud615\uc131\ub41c\ub2e4. \ud751\uc810\uc758 \uc22b\uc790\ub294 \ucd5c\ub300\uc640 \ucd5c\uc18c \uc0ac\uc774\ub97c \uc8fc\uae30\uc801\uc73c\ub85c \ubc18\ubcf5\ud55c\ub2e4. \uacfc\ud559\uc790\ub4e4\uc740 \ud751\uc810 \uc8fc\uae30\uc640 \uc9c0\uad6c\uc758 \uae30\ud6c4\uc758 \ubcc0\ud654 \uc0ac\uc774\uc5d0 \uc5b4\ub5a4 \uad00\uacc4\uac00 \uc788\uc9c0 \uc54a\uc740\uc9c0\ub97c \uc5f0\uad6c\ud55c\ub2e4. \ub9cc\uc77c \uc815\ub9d0 \uc5b4\ub5a4 \uad00\uacc4\uac00 \uc874\uc7ac\ud55c\ub2e4\uba74 \ud751\uc815 \uc8fc\uae30\uc758 \ubcc0\ud654\ub85c \uc778\ud574 \uc18c\ube59\ud558\uae30\uc640 \uc18c\uac04\ube59\uae30\uac00 \ubc18\ubcf5\ub420 \uac83\uc774\ub2e4.", "paragraph_answer": "\ub458\uc9f8, \ud0dc\uc591\uc758 \ubcc0\ud654\ub85c \uc778\ud55c \uaca9\ubcc0\uc758 \uac00\ub2a5\uc131\uc774\ub2e4. \ud0dc\uc591\uc740 \uae30\uccb4\ub85c \ub418\uc5b4 \uc788\uc9c0\ub9cc \uc628\ub3c4\ub098 \uc555\ub825\uc774 \uc5c4\uccad\ub098\uae30 \ub54c\ubb38\uc5d0 \uc911\uc2ec\uc5d0\uc11c \uc6d0\uc790\ub4e4\uc774 \uc11c\ub85c \uc735\ud569\ud558\ub294 \ud575\uc735\ud569 \ubc18\uc751\uc744 \uc77c\uc73c\ud0a8\ub2e4. \ud0dc\uc591\uc5d0\uc11c\ub294 1\ucd08\uc5d0 4-5\uc5b5\ud1a4 \uc9c8\ub7c9\uc774 \uc5d0\ub108\uc9c0\ub85c \ubc14\ub00c\uba74\uc11c \uc9c0\uad6c \uc0dd\uba85\uccb4\ub4e4\uc774 \uc0b4\uc544\uac00\ub294\ub370 \ud544\uc694\ud55c \uc5f4\uacfc \ube5b\uc744 \ubc29\ucd9c\ud558\uace0 \uc788\ub2e4. \ub610\ud55c \ud0dc\uc591\uc740 \uc911\uc2ec\uc73c\ub85c\ubd80\ud130 \uac15\ub825\ud55c \uc790\uae30\uc7a5\uc744 \ubc29\ucd9c\ud558\uc5ec \uc6b0\uc8fc \uacf5\uac04\uc73c\ub85c \uc3d8\uc544 \ubcf4\ub0b4\uace0 \uc788\ub2e4. \uc774\ub7ec\ud55c \ud0dc\uc591\uc774 \uc790\uc804\ud558\uac8c \ub418\uba74 \ud0dc\uc591\uc758 \uc801\ub3c4\uc9c0\ubc29\uc774 \uadf9\uc9c0\ubc29\uc5d0 \ube44\ud574 \uc18d\ub3c4\uac00 \ube60\ub974\uace0 \uc774\ub7ec\ud55c \uc18d\ub3c4\uc758 \ucc28\uc774\ub85c \uc778\ud574 \uc790\uae30\uc7a5\uc774 \ub4a4\ud2c0\ub9ac\uac8c \ub41c\ub2e4. \uadf8\ub9ac\uace0 \uc774 \ub4a4\ud2c0\ub9bc\uc774 \ucd5c\ub300\uac00 \ub418\ub294 \uacf3\uc5d0 \uc6b0\ub9ac\uac00 \ud754\ud788 \ud0dc\uc591\uc758 \ud751\uc810 \uc774\ub77c\uace0 \ub9d0\ud558\ub294 \uac83\uc774 \ud615\uc131\ub41c\ub2e4. \ud751\uc810\uc758 \uc22b\uc790\ub294 \ucd5c\ub300\uc640 \ucd5c\uc18c \uc0ac\uc774\ub97c \uc8fc\uae30\uc801\uc73c\ub85c \ubc18\ubcf5\ud55c\ub2e4. \uacfc\ud559\uc790\ub4e4\uc740 \ud751\uc810 \uc8fc\uae30\uc640 \uc9c0\uad6c\uc758 \uae30\ud6c4\uc758 \ubcc0\ud654 \uc0ac\uc774\uc5d0 \uc5b4\ub5a4 \uad00\uacc4\uac00 \uc788\uc9c0 \uc54a\uc740\uc9c0\ub97c \uc5f0\uad6c\ud55c\ub2e4. \ub9cc\uc77c \uc815\ub9d0 \uc5b4\ub5a4 \uad00\uacc4\uac00 \uc874\uc7ac\ud55c\ub2e4\uba74 \ud751\uc815 \uc8fc\uae30\uc758 \ubcc0\ud654\ub85c \uc778\ud574 \uc18c\ube59\ud558\uae30\uc640 \uc18c\uac04\ube59\uae30\uac00 \ubc18\ubcf5\ub420 \uac83\uc774\ub2e4.", "sentence_answer": "\uadf8\ub9ac\uace0 \uc774 \ub4a4\ud2c0\ub9bc\uc774 \ucd5c\ub300\uac00 \ub418\ub294 \uacf3\uc5d0 \uc6b0\ub9ac\uac00 \ud754\ud788 \ud0dc\uc591\uc758 \ud751\uc810 \uc774\ub77c\uace0 \ub9d0\ud558\ub294 \uac83\uc774 \ud615\uc131\ub41c\ub2e4.", "paragraph_id": "6472705-4-0", "paragraph_question": "question: \ud0dc\uc591\uc774 \uc790\uc804\ud558\uba74\uc11c \uc790\uae30\uc7a5\uc774 \ub4a4\ud2c0\ub9ac\uba74\uc11c \uadf8 \ub4a4\ud2c0\ub9bc\uc774 \ucd5c\ub300\uac00 \ub418\ub294 \uacf3\uc5d0 \ubc1c\uc0dd\ud558\ub294 \uac83\uc740?, context: \ub458\uc9f8, \ud0dc\uc591\uc758 \ubcc0\ud654\ub85c \uc778\ud55c \uaca9\ubcc0\uc758 \uac00\ub2a5\uc131\uc774\ub2e4. \ud0dc\uc591\uc740 \uae30\uccb4\ub85c \ub418\uc5b4 \uc788\uc9c0\ub9cc \uc628\ub3c4\ub098 \uc555\ub825\uc774 \uc5c4\uccad\ub098\uae30 \ub54c\ubb38\uc5d0 \uc911\uc2ec\uc5d0\uc11c \uc6d0\uc790\ub4e4\uc774 \uc11c\ub85c \uc735\ud569\ud558\ub294 \ud575\uc735\ud569 \ubc18\uc751\uc744 \uc77c\uc73c\ud0a8\ub2e4. \ud0dc\uc591\uc5d0\uc11c\ub294 1\ucd08\uc5d0 4-5\uc5b5\ud1a4 \uc9c8\ub7c9\uc774 \uc5d0\ub108\uc9c0\ub85c \ubc14\ub00c\uba74\uc11c \uc9c0\uad6c \uc0dd\uba85\uccb4\ub4e4\uc774 \uc0b4\uc544\uac00\ub294\ub370 \ud544\uc694\ud55c \uc5f4\uacfc \ube5b\uc744 \ubc29\ucd9c\ud558\uace0 \uc788\ub2e4. \ub610\ud55c \ud0dc\uc591\uc740 \uc911\uc2ec\uc73c\ub85c\ubd80\ud130 \uac15\ub825\ud55c \uc790\uae30\uc7a5\uc744 \ubc29\ucd9c\ud558\uc5ec \uc6b0\uc8fc \uacf5\uac04\uc73c\ub85c \uc3d8\uc544 \ubcf4\ub0b4\uace0 \uc788\ub2e4. \uc774\ub7ec\ud55c \ud0dc\uc591\uc774 \uc790\uc804\ud558\uac8c \ub418\uba74 \ud0dc\uc591\uc758 \uc801\ub3c4\uc9c0\ubc29\uc774 \uadf9\uc9c0\ubc29\uc5d0 \ube44\ud574 \uc18d\ub3c4\uac00 \ube60\ub974\uace0 \uc774\ub7ec\ud55c \uc18d\ub3c4\uc758 \ucc28\uc774\ub85c \uc778\ud574 \uc790\uae30\uc7a5\uc774 \ub4a4\ud2c0\ub9ac\uac8c \ub41c\ub2e4. \uadf8\ub9ac\uace0 \uc774 \ub4a4\ud2c0\ub9bc\uc774 \ucd5c\ub300\uac00 \ub418\ub294 \uacf3\uc5d0 \uc6b0\ub9ac\uac00 \ud754\ud788 \ud0dc\uc591\uc758 \ud751\uc810\uc774\ub77c\uace0 \ub9d0\ud558\ub294 \uac83\uc774 \ud615\uc131\ub41c\ub2e4. \ud751\uc810\uc758 \uc22b\uc790\ub294 \ucd5c\ub300\uc640 \ucd5c\uc18c \uc0ac\uc774\ub97c \uc8fc\uae30\uc801\uc73c\ub85c \ubc18\ubcf5\ud55c\ub2e4. \uacfc\ud559\uc790\ub4e4\uc740 \ud751\uc810 \uc8fc\uae30\uc640 \uc9c0\uad6c\uc758 \uae30\ud6c4\uc758 \ubcc0\ud654 \uc0ac\uc774\uc5d0 \uc5b4\ub5a4 \uad00\uacc4\uac00 \uc788\uc9c0 \uc54a\uc740\uc9c0\ub97c \uc5f0\uad6c\ud55c\ub2e4. \ub9cc\uc77c \uc815\ub9d0 \uc5b4\ub5a4 \uad00\uacc4\uac00 \uc874\uc7ac\ud55c\ub2e4\uba74 \ud751\uc815 \uc8fc\uae30\uc758 \ubcc0\ud654\ub85c \uc778\ud574 \uc18c\ube59\ud558\uae30\uc640 \uc18c\uac04\ube59\uae30\uac00 \ubc18\ubcf5\ub420 \uac83\uc774\ub2e4."} {"question": "\uc0c8\uc815\uce58\ubbfc\uc8fc\uc5f0\ud569\uc740 \ubbfc\uc8fc\ub2f9\uacfc \uc5b4\ub5a4 \ub2f9\uc758 \ud569\ub2f9\uc73c\ub85c \ucc3d\ub2f9\ub418\uc5c8\ub294\uac00?", "paragraph": "\uc0c8\uc815\uce58\ubbfc\uc8fc\uc5f0\ud569\uc740 \ubbfc\uc8fc\ub2f9\uacfc \uc0c8\uc815\uce58\uc5f0\ud569\uc758 \ud569\ub2f9\uc73c\ub85c \ucc3d\ub2f9\ub418\uc5c8\ub2e4. 2014\ub144 3\uc6d4 16\uc77c, \ucc3d\ub2f9 \ubc1c\uae30\uc778\ub300\ud68c\ub97c \uac1c\ucd5c\ud558\uc5ec \uc0c8\uc815\uce58\ubbfc\uc8fc\uc5f0\ud569\uc774\ub77c\ub294 \ub2f9\uba85\uc744 \ubc1c\ud45c\ud558\uc600\uc73c\uba70, \uacf5\ub3d9\ucc3d\ub2f9\uc900\ube44\uc704\uc6d0\uc7a5\uc5d0 \uae40\ud55c\uae38 \ubbfc\uc8fc\ub2f9 \ub300\ud45c\uc640 \uc548\ucca0\uc218 \uc0c8\uc815\uce58\uc5f0\ud569 \ucc3d\ub2f9\uc900\ube44\uc704\uc6d0\ud68c \uc911\uc559\uc6b4\uc601\uc704\uc6d0\uc7a5\uc774 \uc120\ucd9c\ub418\uc5c8\ub2e4. \uc774\uc5d0 \ub530\ub77c \uc0c8\uc815\uce58\ubbfc\uc8fc\uc5f0\ud569\uc740 \ubbfc\uc8fc\ub2f9\uc758 126\uc11d\uacfc \uc0c8\uc815\uce58\uc5f0\ud569\uc758 2\uc11d, \ubc1c\uae30\uc778 \ub300\ud68c\uc5d0 \ucc38\uc5ec\ud55c \ubb34\uc18c\uc18d \ubc15\uc8fc\uc120, \uac15\ub3d9\uc6d0 \uc758\uc6d0\uc744 \ud569\ud558\uc5ec \ucd1d 130\uc11d\uc758 \uc6d0\ub0b4 2\ub2f9\uc774 \ub418\uc5c8\ub2e4. \ud1b5\ud569 \ubc29\uc2dd\uc740 \ucc3d\ub2f9\uc900\ube44\uc704\uc6d0\ud68c \ub2e8\uacc4\uc778 \uc0c8\uc815\uce58\uc5f0\ud569\uacfc \ubbfc\uc8fc\ub2f9\uacfc \ud1b5\ud569\uc744 \ud558\uace0, \ub2f9\uba85\uc744 \uc0c8\uc815\uce58\ubbfc\uc8fc\uc5f0\ud569\uc73c\ub85c \uc815\ud558\uc600\ub2e4. 2014\ub144 3\uc6d4 26\uc77c, \uc0c8\uc815\uce58\ubbfc\uc8fc\uc5f0\ud569\uc740 \uae40\ud55c\uae38\uacfc \uc548\ucca0\uc218\ub97c \uacf5\ub3d9 \ub300\ud45c\ub85c \ud558\uc5ec \ucc3d\ub2f9\ud558\uc600\ub2e4. \uc774\ub294 2007\ub144 \ub300\ud1b5\ud569\ubbfc\uc8fc\uc2e0\ub2f9\uc774 \uc5f4\ub9b0\uc6b0\ub9ac\ub2f9\uc744 \ub2f9\ub300\ub2f9 \ud569\ub2f9\ud558\uc600\ub358 \uacfc\uc815\uacfc \ube44\uc2b7\ud558\ub2e4.", "answer": "\uc0c8\uc815\uce58\uc5f0\ud569", "sentence": "\uc0c8\uc815\uce58\ubbfc\uc8fc\uc5f0\ud569\uc740 \ubbfc\uc8fc\ub2f9\uacfc \uc0c8\uc815\uce58\uc5f0\ud569 \uc758 \ud569\ub2f9\uc73c\ub85c \ucc3d\ub2f9\ub418\uc5c8\ub2e4.", "paragraph_sentence": " \uc0c8\uc815\uce58\ubbfc\uc8fc\uc5f0\ud569\uc740 \ubbfc\uc8fc\ub2f9\uacfc \uc0c8\uc815\uce58\uc5f0\ud569 \uc758 \ud569\ub2f9\uc73c\ub85c \ucc3d\ub2f9\ub418\uc5c8\ub2e4. 2014\ub144 3\uc6d4 16\uc77c, \ucc3d\ub2f9 \ubc1c\uae30\uc778\ub300\ud68c\ub97c \uac1c\ucd5c\ud558\uc5ec \uc0c8\uc815\uce58\ubbfc\uc8fc\uc5f0\ud569\uc774\ub77c\ub294 \ub2f9\uba85\uc744 \ubc1c\ud45c\ud558\uc600\uc73c\uba70, \uacf5\ub3d9\ucc3d\ub2f9\uc900\ube44\uc704\uc6d0\uc7a5\uc5d0 \uae40\ud55c\uae38 \ubbfc\uc8fc\ub2f9 \ub300\ud45c\uc640 \uc548\ucca0\uc218 \uc0c8\uc815\uce58\uc5f0\ud569 \ucc3d\ub2f9\uc900\ube44\uc704\uc6d0\ud68c \uc911\uc559\uc6b4\uc601\uc704\uc6d0\uc7a5\uc774 \uc120\ucd9c\ub418\uc5c8\ub2e4. \uc774\uc5d0 \ub530\ub77c \uc0c8\uc815\uce58\ubbfc\uc8fc\uc5f0\ud569\uc740 \ubbfc\uc8fc\ub2f9\uc758 126\uc11d\uacfc \uc0c8\uc815\uce58\uc5f0\ud569\uc758 2\uc11d, \ubc1c\uae30\uc778 \ub300\ud68c\uc5d0 \ucc38\uc5ec\ud55c \ubb34\uc18c\uc18d \ubc15\uc8fc\uc120, \uac15\ub3d9\uc6d0 \uc758\uc6d0\uc744 \ud569\ud558\uc5ec \ucd1d 130\uc11d\uc758 \uc6d0\ub0b4 2\ub2f9\uc774 \ub418\uc5c8\ub2e4. \ud1b5\ud569 \ubc29\uc2dd\uc740 \ucc3d\ub2f9\uc900\ube44\uc704\uc6d0\ud68c \ub2e8\uacc4\uc778 \uc0c8\uc815\uce58\uc5f0\ud569\uacfc \ubbfc\uc8fc\ub2f9\uacfc \ud1b5\ud569\uc744 \ud558\uace0, \ub2f9\uba85\uc744 \uc0c8\uc815\uce58\ubbfc\uc8fc\uc5f0\ud569\uc73c\ub85c \uc815\ud558\uc600\ub2e4. 2014\ub144 3\uc6d4 26\uc77c, \uc0c8\uc815\uce58\ubbfc\uc8fc\uc5f0\ud569\uc740 \uae40\ud55c\uae38\uacfc \uc548\ucca0\uc218\ub97c \uacf5\ub3d9 \ub300\ud45c\ub85c \ud558\uc5ec \ucc3d\ub2f9\ud558\uc600\ub2e4. \uc774\ub294 2007\ub144 \ub300\ud1b5\ud569\ubbfc\uc8fc\uc2e0\ub2f9\uc774 \uc5f4\ub9b0\uc6b0\ub9ac\ub2f9\uc744 \ub2f9\ub300\ub2f9 \ud569\ub2f9\ud558\uc600\ub358 \uacfc\uc815\uacfc \ube44\uc2b7\ud558\ub2e4.", "paragraph_answer": "\uc0c8\uc815\uce58\ubbfc\uc8fc\uc5f0\ud569\uc740 \ubbfc\uc8fc\ub2f9\uacfc \uc0c8\uc815\uce58\uc5f0\ud569 \uc758 \ud569\ub2f9\uc73c\ub85c \ucc3d\ub2f9\ub418\uc5c8\ub2e4. 2014\ub144 3\uc6d4 16\uc77c, \ucc3d\ub2f9 \ubc1c\uae30\uc778\ub300\ud68c\ub97c \uac1c\ucd5c\ud558\uc5ec \uc0c8\uc815\uce58\ubbfc\uc8fc\uc5f0\ud569\uc774\ub77c\ub294 \ub2f9\uba85\uc744 \ubc1c\ud45c\ud558\uc600\uc73c\uba70, \uacf5\ub3d9\ucc3d\ub2f9\uc900\ube44\uc704\uc6d0\uc7a5\uc5d0 \uae40\ud55c\uae38 \ubbfc\uc8fc\ub2f9 \ub300\ud45c\uc640 \uc548\ucca0\uc218 \uc0c8\uc815\uce58\uc5f0\ud569 \ucc3d\ub2f9\uc900\ube44\uc704\uc6d0\ud68c \uc911\uc559\uc6b4\uc601\uc704\uc6d0\uc7a5\uc774 \uc120\ucd9c\ub418\uc5c8\ub2e4. \uc774\uc5d0 \ub530\ub77c \uc0c8\uc815\uce58\ubbfc\uc8fc\uc5f0\ud569\uc740 \ubbfc\uc8fc\ub2f9\uc758 126\uc11d\uacfc \uc0c8\uc815\uce58\uc5f0\ud569\uc758 2\uc11d, \ubc1c\uae30\uc778 \ub300\ud68c\uc5d0 \ucc38\uc5ec\ud55c \ubb34\uc18c\uc18d \ubc15\uc8fc\uc120, \uac15\ub3d9\uc6d0 \uc758\uc6d0\uc744 \ud569\ud558\uc5ec \ucd1d 130\uc11d\uc758 \uc6d0\ub0b4 2\ub2f9\uc774 \ub418\uc5c8\ub2e4. \ud1b5\ud569 \ubc29\uc2dd\uc740 \ucc3d\ub2f9\uc900\ube44\uc704\uc6d0\ud68c \ub2e8\uacc4\uc778 \uc0c8\uc815\uce58\uc5f0\ud569\uacfc \ubbfc\uc8fc\ub2f9\uacfc \ud1b5\ud569\uc744 \ud558\uace0, \ub2f9\uba85\uc744 \uc0c8\uc815\uce58\ubbfc\uc8fc\uc5f0\ud569\uc73c\ub85c \uc815\ud558\uc600\ub2e4. 2014\ub144 3\uc6d4 26\uc77c, \uc0c8\uc815\uce58\ubbfc\uc8fc\uc5f0\ud569\uc740 \uae40\ud55c\uae38\uacfc \uc548\ucca0\uc218\ub97c \uacf5\ub3d9 \ub300\ud45c\ub85c \ud558\uc5ec \ucc3d\ub2f9\ud558\uc600\ub2e4. \uc774\ub294 2007\ub144 \ub300\ud1b5\ud569\ubbfc\uc8fc\uc2e0\ub2f9\uc774 \uc5f4\ub9b0\uc6b0\ub9ac\ub2f9\uc744 \ub2f9\ub300\ub2f9 \ud569\ub2f9\ud558\uc600\ub358 \uacfc\uc815\uacfc \ube44\uc2b7\ud558\ub2e4.", "sentence_answer": "\uc0c8\uc815\uce58\ubbfc\uc8fc\uc5f0\ud569\uc740 \ubbfc\uc8fc\ub2f9\uacfc \uc0c8\uc815\uce58\uc5f0\ud569 \uc758 \ud569\ub2f9\uc73c\ub85c \ucc3d\ub2f9\ub418\uc5c8\ub2e4.", "paragraph_id": "6486812-0-0", "paragraph_question": "question: \uc0c8\uc815\uce58\ubbfc\uc8fc\uc5f0\ud569\uc740 \ubbfc\uc8fc\ub2f9\uacfc \uc5b4\ub5a4 \ub2f9\uc758 \ud569\ub2f9\uc73c\ub85c \ucc3d\ub2f9\ub418\uc5c8\ub294\uac00?, context: \uc0c8\uc815\uce58\ubbfc\uc8fc\uc5f0\ud569\uc740 \ubbfc\uc8fc\ub2f9\uacfc \uc0c8\uc815\uce58\uc5f0\ud569\uc758 \ud569\ub2f9\uc73c\ub85c \ucc3d\ub2f9\ub418\uc5c8\ub2e4. 2014\ub144 3\uc6d4 16\uc77c, \ucc3d\ub2f9 \ubc1c\uae30\uc778\ub300\ud68c\ub97c \uac1c\ucd5c\ud558\uc5ec \uc0c8\uc815\uce58\ubbfc\uc8fc\uc5f0\ud569\uc774\ub77c\ub294 \ub2f9\uba85\uc744 \ubc1c\ud45c\ud558\uc600\uc73c\uba70, \uacf5\ub3d9\ucc3d\ub2f9\uc900\ube44\uc704\uc6d0\uc7a5\uc5d0 \uae40\ud55c\uae38 \ubbfc\uc8fc\ub2f9 \ub300\ud45c\uc640 \uc548\ucca0\uc218 \uc0c8\uc815\uce58\uc5f0\ud569 \ucc3d\ub2f9\uc900\ube44\uc704\uc6d0\ud68c \uc911\uc559\uc6b4\uc601\uc704\uc6d0\uc7a5\uc774 \uc120\ucd9c\ub418\uc5c8\ub2e4. \uc774\uc5d0 \ub530\ub77c \uc0c8\uc815\uce58\ubbfc\uc8fc\uc5f0\ud569\uc740 \ubbfc\uc8fc\ub2f9\uc758 126\uc11d\uacfc \uc0c8\uc815\uce58\uc5f0\ud569\uc758 2\uc11d, \ubc1c\uae30\uc778 \ub300\ud68c\uc5d0 \ucc38\uc5ec\ud55c \ubb34\uc18c\uc18d \ubc15\uc8fc\uc120, \uac15\ub3d9\uc6d0 \uc758\uc6d0\uc744 \ud569\ud558\uc5ec \ucd1d 130\uc11d\uc758 \uc6d0\ub0b4 2\ub2f9\uc774 \ub418\uc5c8\ub2e4. \ud1b5\ud569 \ubc29\uc2dd\uc740 \ucc3d\ub2f9\uc900\ube44\uc704\uc6d0\ud68c \ub2e8\uacc4\uc778 \uc0c8\uc815\uce58\uc5f0\ud569\uacfc \ubbfc\uc8fc\ub2f9\uacfc \ud1b5\ud569\uc744 \ud558\uace0, \ub2f9\uba85\uc744 \uc0c8\uc815\uce58\ubbfc\uc8fc\uc5f0\ud569\uc73c\ub85c \uc815\ud558\uc600\ub2e4. 2014\ub144 3\uc6d4 26\uc77c, \uc0c8\uc815\uce58\ubbfc\uc8fc\uc5f0\ud569\uc740 \uae40\ud55c\uae38\uacfc \uc548\ucca0\uc218\ub97c \uacf5\ub3d9 \ub300\ud45c\ub85c \ud558\uc5ec \ucc3d\ub2f9\ud558\uc600\ub2e4. \uc774\ub294 2007\ub144 \ub300\ud1b5\ud569\ubbfc\uc8fc\uc2e0\ub2f9\uc774 \uc5f4\ub9b0\uc6b0\ub9ac\ub2f9\uc744 \ub2f9\ub300\ub2f9 \ud569\ub2f9\ud558\uc600\ub358 \uacfc\uc815\uacfc \ube44\uc2b7\ud558\ub2e4."} {"question": "\uc0ac\ud68c \uc774\uc288\ub4e4\uc5d0 \uad00\ud574 \uc77c\ubc18 \uc5ec\ub860\uc744 \ub3c4\ucd9c\ud558\uae30 \uc704\ud574 \uc219\uc758 \ubbfc\uc8fc\uc8fc\uc758\uc5d0\uc11c \uc0ac\uc6a9\ud558\ub294 \uac83\uc740?", "paragraph": "\uc219\uc758 \ubbfc\uc8fc\uc8fc\uc758\ub294 \ubbfc\uc8fc\uc801 \uacb0\uc815\uc774 \uc801\ubc95\ud558\uae30 \uc704\ud574\uc11c\ub294, \ub2e8\uc9c0 \ud22c\ud45c\uc5d0\uc11c \ub098\ud0c0\ub098\ub294 \uc120\ud638\ub3c4\uc758 \ucd1d\ud569\uc774 \uc544\ub2c8\ub77c, \uc2e4\uc81c\uc801\uc778 \uc219\uc758\uac00 \uc120\ud589\ub418\uc5b4\uc57c \ud55c\ub2e4\uace0 \uac04\uc8fc\ud55c\ub2e4. \uc2e4\uc81c\uc801 \uc219\uc758\ub780, \uc774\ub97c\ud14c\uba74 \uacbd\uc81c\uc801 \ubd80\ub098 \ub610\ub294 \uc774\ud574 \uc9d1\ub2e8\ub4e4\uc758 \uc9c0\uc6d0\uc744 \ud1b5\ud574 \uc758\uc0ac\uacb0\uc815\uc790\ub4e4\uc774 \ubcf4\uc720\ud558\ub294, \uc815\uce58\uc801 \uad8c\ud55c\uc758 \ubd88\ud3c9\ub4f1\ud55c \uc0ac\uc774\uc5d0 \uc774\ub8e8\uc5b4\uc9c0\ub294 \uc219\uc758\uc774\ub2e4. \uc758\uc0ac\uacb0\uc815\uc790\ub4e4\uc774 \uc5b4\ub5a4 \uc81c\uc548\uc5d0 \ub300\ud574 \uc2e4\uc81c\uc801\uc73c\ub85c \uc219\uc758\ud55c \ud6c4\uc5d0 \ud569\uc758\uc5d0 \ub3c4\ub2ec\ud560 \uc218 \uc5c6\ub2e4\uba74, \uadf8\ub4e4\uc740 \ub2e4\uc218\uacb0 \uc6d0\ub9ac\ub77c\ub294 \ud615\uc2dd\uc73c\ub85c \uadf8 \uc81c\uc548\uc5d0 \ub300\ud574 \ud22c\ud45c\ud55c\ub2e4. \uc219\uc758 \ubbfc\uc8fc\uc8fc\uc758\ub294 \ub300\uc758 \uc788\ub2e4. \uc5d8\ub9ac\ud2b8 \uc219\uc758 \ubbfc\uc8fc\uc8fc\uc758(elitist deliberative democracy)\uc5d0\uc11c \uc785\ubc95\ubd80\uc640 \ubc95\uc6d0\ub4e4\uacfc \uac19\uc740, \uc5d8\ub9ac\ud2b8\uc801\uc6a9\ub41c\ub2e4; \ub300\uc911\uc801(populist) \uc219\uc758 \ubbfc\uc8fc\uc8fc\uc758\uc5d0\uc11c \uc219\uc758 \ubbfc\uc8fc\uc8fc\uc8fc\uc758 \uc6d0\ub9ac\ub4e4\uc740 \uc758\uc0ac\uacb0\uc815\uc744 \uc704\uc784 \ubc1b\uc740 \uc77c\ubc18 \uc2dc\ubbfc\ub4e4\uc758 \ubaa8\uc784\uc5d0 \uc801\uc6a9\ub41c\ub2e4. \ub300\uc911\uc801 \uc219\uc758 \ubbfc\uc8fc\uc8fc\uc758\uc758 \ud55c \uac00\uc9c0 \ubaa9\uc801\uc740 \uc0ac\ud68c \uc774\uc288\ub4e4\uc5d0 \uad00\ud558\uc5ec \uad6c\uc18d\ub825 \uac83\uc774 \uc544\ub2c8\ub77c \ubcf4\ub2e4 \uc77c\ubc18 \uc5ec\ub860\uc744 \ub3c4\ucd9c(distill)\ud558\ub294\ub370 \uc0ac\uc6a9\ub420 \uc218 \uc788\ub2e4; \uc219\uc758 \uc5ec\ub860\uc870\uc0ac(deliberative opinion poll)\uacfc \uac19\uc740 \uc218\ub2e8\ub4e4\uc740 \uc774\ub7ec\ud55c \ubaa9\ud45c\ub97c \ub2ec\uc131\ud558\ub294\ub370 \uc124\uacc4\ub418\uc5b4\uc654\ub2e4. \ub300\uc911\uc801 \uc219\uc758 \ubbfc\uc8fc\uc8fc\uc758\uc758 \ub610 \ub2e4\ub978 \ubaa9\uc801\uc740 \uc9c1\uc811 \ubbfc\uc8fc\uc8fc\uc758 \ud615\ud0dc\ub85c \uc5ed\ud560 \ud560 \uc218 \uc219\uc758\ub294 \uc77c\ubc18 \uc2dc\ubbfc\ub4e4\uc758 \ubaa8\uc784 \uac00\uc6b4\ub370 \u201c\uacf5\uacf5\uc758 \uc758\uc9c0\u201d\ub97c \ud615\uc131\ud558\uace0 \uad6c\uc18d\ub825 \uc788\ub294 \ubc95\uc744 \uc9c1\uc811 \ub9cc\ub4e4\uac8c \ub41c\ub2e4. \uc815\uce58\uc801 \uacb0\uc815\ub4e4\uc774 \uad6d\ubbfc \uc2a4\uc2a4\ub85c \ub610\ub294 \uadf8\ub4e4\uc5d0 \uc758\ud574 \uc120\ucd9c\ub41c \ub300\ud45c\uc790\ub4e4\uc5d0 \uc758\ud558\uc9c0 \uc54a\uace0 \uc219\uc758\uc5d0 \uc758\ud574 \ub9cc\ub4e4\uc5b4\uc9c0\uba74, \uac70\uae30\uc5d0\ub294 \ubbfc\uc8fc\uc801 \uc694\uc18c\uac00 \uc5c6\uac8c \ub41c\ub2e4; \uc774\ub7ec\ud55c \uc219\uc758 \uacfc\uc815\uc740 \uc5d8\ub9ac\ud2b8 \uc219\uc758(elite deliberation)\uc774\ub77c\uace0 \ubd88\ub9b0\ub2e4.", "answer": "\uc219\uc758 \uc5ec\ub860\uc870\uc0ac", "sentence": "\ub300\uc911\uc801 \uc219\uc758 \ubbfc\uc8fc\uc8fc\uc758\uc758 \ud55c \uac00\uc9c0 \ubaa9\uc801\uc740 \uc0ac\ud68c \uc774\uc288\ub4e4\uc5d0 \uad00\ud558\uc5ec \uad6c\uc18d\ub825 \uac83\uc774 \uc544\ub2c8\ub77c \ubcf4\ub2e4 \uc77c\ubc18 \uc5ec\ub860\uc744 \ub3c4\ucd9c(distill)\ud558\ub294\ub370 \uc0ac\uc6a9\ub420 \uc218 \uc788\ub2e4; \uc219\uc758 \uc5ec\ub860\uc870\uc0ac (deliberative opinion poll)\uacfc", "paragraph_sentence": "\uc219\uc758 \ubbfc\uc8fc\uc8fc\uc758\ub294 \ubbfc\uc8fc\uc801 \uacb0\uc815\uc774 \uc801\ubc95\ud558\uae30 \uc704\ud574\uc11c\ub294, \ub2e8\uc9c0 \ud22c\ud45c\uc5d0\uc11c \ub098\ud0c0\ub098\ub294 \uc120\ud638\ub3c4\uc758 \ucd1d\ud569\uc774 \uc544\ub2c8\ub77c, \uc2e4\uc81c\uc801\uc778 \uc219\uc758\uac00 \uc120\ud589\ub418\uc5b4\uc57c \ud55c\ub2e4\uace0 \uac04\uc8fc\ud55c\ub2e4. \uc2e4\uc81c\uc801 \uc219\uc758\ub780, \uc774\ub97c\ud14c\uba74 \uacbd\uc81c\uc801 \ubd80\ub098 \ub610\ub294 \uc774\ud574 \uc9d1\ub2e8\ub4e4\uc758 \uc9c0\uc6d0\uc744 \ud1b5\ud574 \uc758\uc0ac\uacb0\uc815\uc790\ub4e4\uc774 \ubcf4\uc720\ud558\ub294, \uc815\uce58\uc801 \uad8c\ud55c\uc758 \ubd88\ud3c9\ub4f1\ud55c \uc0ac\uc774\uc5d0 \uc774\ub8e8\uc5b4\uc9c0\ub294 \uc219\uc758\uc774\ub2e4. \uc758\uc0ac\uacb0\uc815\uc790\ub4e4\uc774 \uc5b4\ub5a4 \uc81c\uc548\uc5d0 \ub300\ud574 \uc2e4\uc81c\uc801\uc73c\ub85c \uc219\uc758\ud55c \ud6c4\uc5d0 \ud569\uc758\uc5d0 \ub3c4\ub2ec\ud560 \uc218 \uc5c6\ub2e4\uba74, \uadf8\ub4e4\uc740 \ub2e4\uc218\uacb0 \uc6d0\ub9ac\ub77c\ub294 \ud615\uc2dd\uc73c\ub85c \uadf8 \uc81c\uc548\uc5d0 \ub300\ud574 \ud22c\ud45c\ud55c\ub2e4. \uc219\uc758 \ubbfc\uc8fc\uc8fc\uc758\ub294 \ub300\uc758 \uc788\ub2e4. \uc5d8\ub9ac\ud2b8 \uc219\uc758 \ubbfc\uc8fc\uc8fc\uc758(elitist deliberative democracy)\uc5d0\uc11c \uc785\ubc95\ubd80\uc640 \ubc95\uc6d0\ub4e4\uacfc \uac19\uc740, \uc5d8\ub9ac\ud2b8\uc801\uc6a9\ub41c\ub2e4; \ub300\uc911\uc801(populist) \uc219\uc758 \ubbfc\uc8fc\uc8fc\uc758\uc5d0\uc11c \uc219\uc758 \ubbfc\uc8fc\uc8fc\uc8fc\uc758 \uc6d0\ub9ac\ub4e4\uc740 \uc758\uc0ac\uacb0\uc815\uc744 \uc704\uc784 \ubc1b\uc740 \uc77c\ubc18 \uc2dc\ubbfc\ub4e4\uc758 \ubaa8\uc784\uc5d0 \uc801\uc6a9\ub41c\ub2e4. \ub300\uc911\uc801 \uc219\uc758 \ubbfc\uc8fc\uc8fc\uc758\uc758 \ud55c \uac00\uc9c0 \ubaa9\uc801\uc740 \uc0ac\ud68c \uc774\uc288\ub4e4\uc5d0 \uad00\ud558\uc5ec \uad6c\uc18d\ub825 \uac83\uc774 \uc544\ub2c8\ub77c \ubcf4\ub2e4 \uc77c\ubc18 \uc5ec\ub860\uc744 \ub3c4\ucd9c(distill)\ud558\ub294\ub370 \uc0ac\uc6a9\ub420 \uc218 \uc788\ub2e4; \uc219\uc758 \uc5ec\ub860\uc870\uc0ac (deliberative opinion poll)\uacfc \uac19\uc740 \uc218\ub2e8\ub4e4\uc740 \uc774\ub7ec\ud55c \ubaa9\ud45c\ub97c \ub2ec\uc131\ud558\ub294\ub370 \uc124\uacc4\ub418\uc5b4\uc654\ub2e4. \ub300\uc911\uc801 \uc219\uc758 \ubbfc\uc8fc\uc8fc\uc758\uc758 \ub610 \ub2e4\ub978 \ubaa9\uc801\uc740 \uc9c1\uc811 \ubbfc\uc8fc\uc8fc\uc758 \ud615\ud0dc\ub85c \uc5ed\ud560 \ud560 \uc218 \uc219\uc758\ub294 \uc77c\ubc18 \uc2dc\ubbfc\ub4e4\uc758 \ubaa8\uc784 \uac00\uc6b4\ub370 \u201c\uacf5\uacf5\uc758 \uc758\uc9c0\u201d\ub97c \ud615\uc131\ud558\uace0 \uad6c\uc18d\ub825 \uc788\ub294 \ubc95\uc744 \uc9c1\uc811 \ub9cc\ub4e4\uac8c \ub41c\ub2e4. \uc815\uce58\uc801 \uacb0\uc815\ub4e4\uc774 \uad6d\ubbfc \uc2a4\uc2a4\ub85c \ub610\ub294 \uadf8\ub4e4\uc5d0 \uc758\ud574 \uc120\ucd9c\ub41c \ub300\ud45c\uc790\ub4e4\uc5d0 \uc758\ud558\uc9c0 \uc54a\uace0 \uc219\uc758\uc5d0 \uc758\ud574 \ub9cc\ub4e4\uc5b4\uc9c0\uba74, \uac70\uae30\uc5d0\ub294 \ubbfc\uc8fc\uc801 \uc694\uc18c\uac00 \uc5c6\uac8c \ub41c\ub2e4; \uc774\ub7ec\ud55c \uc219\uc758 \uacfc\uc815\uc740 \uc5d8\ub9ac\ud2b8 \uc219\uc758(elite deliberation)\uc774\ub77c\uace0 \ubd88\ub9b0\ub2e4.", "paragraph_answer": "\uc219\uc758 \ubbfc\uc8fc\uc8fc\uc758\ub294 \ubbfc\uc8fc\uc801 \uacb0\uc815\uc774 \uc801\ubc95\ud558\uae30 \uc704\ud574\uc11c\ub294, \ub2e8\uc9c0 \ud22c\ud45c\uc5d0\uc11c \ub098\ud0c0\ub098\ub294 \uc120\ud638\ub3c4\uc758 \ucd1d\ud569\uc774 \uc544\ub2c8\ub77c, \uc2e4\uc81c\uc801\uc778 \uc219\uc758\uac00 \uc120\ud589\ub418\uc5b4\uc57c \ud55c\ub2e4\uace0 \uac04\uc8fc\ud55c\ub2e4. \uc2e4\uc81c\uc801 \uc219\uc758\ub780, \uc774\ub97c\ud14c\uba74 \uacbd\uc81c\uc801 \ubd80\ub098 \ub610\ub294 \uc774\ud574 \uc9d1\ub2e8\ub4e4\uc758 \uc9c0\uc6d0\uc744 \ud1b5\ud574 \uc758\uc0ac\uacb0\uc815\uc790\ub4e4\uc774 \ubcf4\uc720\ud558\ub294, \uc815\uce58\uc801 \uad8c\ud55c\uc758 \ubd88\ud3c9\ub4f1\ud55c \uc0ac\uc774\uc5d0 \uc774\ub8e8\uc5b4\uc9c0\ub294 \uc219\uc758\uc774\ub2e4. \uc758\uc0ac\uacb0\uc815\uc790\ub4e4\uc774 \uc5b4\ub5a4 \uc81c\uc548\uc5d0 \ub300\ud574 \uc2e4\uc81c\uc801\uc73c\ub85c \uc219\uc758\ud55c \ud6c4\uc5d0 \ud569\uc758\uc5d0 \ub3c4\ub2ec\ud560 \uc218 \uc5c6\ub2e4\uba74, \uadf8\ub4e4\uc740 \ub2e4\uc218\uacb0 \uc6d0\ub9ac\ub77c\ub294 \ud615\uc2dd\uc73c\ub85c \uadf8 \uc81c\uc548\uc5d0 \ub300\ud574 \ud22c\ud45c\ud55c\ub2e4. \uc219\uc758 \ubbfc\uc8fc\uc8fc\uc758\ub294 \ub300\uc758 \uc788\ub2e4. \uc5d8\ub9ac\ud2b8 \uc219\uc758 \ubbfc\uc8fc\uc8fc\uc758(elitist deliberative democracy)\uc5d0\uc11c \uc785\ubc95\ubd80\uc640 \ubc95\uc6d0\ub4e4\uacfc \uac19\uc740, \uc5d8\ub9ac\ud2b8\uc801\uc6a9\ub41c\ub2e4; \ub300\uc911\uc801(populist) \uc219\uc758 \ubbfc\uc8fc\uc8fc\uc758\uc5d0\uc11c \uc219\uc758 \ubbfc\uc8fc\uc8fc\uc8fc\uc758 \uc6d0\ub9ac\ub4e4\uc740 \uc758\uc0ac\uacb0\uc815\uc744 \uc704\uc784 \ubc1b\uc740 \uc77c\ubc18 \uc2dc\ubbfc\ub4e4\uc758 \ubaa8\uc784\uc5d0 \uc801\uc6a9\ub41c\ub2e4. \ub300\uc911\uc801 \uc219\uc758 \ubbfc\uc8fc\uc8fc\uc758\uc758 \ud55c \uac00\uc9c0 \ubaa9\uc801\uc740 \uc0ac\ud68c \uc774\uc288\ub4e4\uc5d0 \uad00\ud558\uc5ec \uad6c\uc18d\ub825 \uac83\uc774 \uc544\ub2c8\ub77c \ubcf4\ub2e4 \uc77c\ubc18 \uc5ec\ub860\uc744 \ub3c4\ucd9c(distill)\ud558\ub294\ub370 \uc0ac\uc6a9\ub420 \uc218 \uc788\ub2e4; \uc219\uc758 \uc5ec\ub860\uc870\uc0ac (deliberative opinion poll)\uacfc \uac19\uc740 \uc218\ub2e8\ub4e4\uc740 \uc774\ub7ec\ud55c \ubaa9\ud45c\ub97c \ub2ec\uc131\ud558\ub294\ub370 \uc124\uacc4\ub418\uc5b4\uc654\ub2e4. \ub300\uc911\uc801 \uc219\uc758 \ubbfc\uc8fc\uc8fc\uc758\uc758 \ub610 \ub2e4\ub978 \ubaa9\uc801\uc740 \uc9c1\uc811 \ubbfc\uc8fc\uc8fc\uc758 \ud615\ud0dc\ub85c \uc5ed\ud560 \ud560 \uc218 \uc219\uc758\ub294 \uc77c\ubc18 \uc2dc\ubbfc\ub4e4\uc758 \ubaa8\uc784 \uac00\uc6b4\ub370 \u201c\uacf5\uacf5\uc758 \uc758\uc9c0\u201d\ub97c \ud615\uc131\ud558\uace0 \uad6c\uc18d\ub825 \uc788\ub294 \ubc95\uc744 \uc9c1\uc811 \ub9cc\ub4e4\uac8c \ub41c\ub2e4. \uc815\uce58\uc801 \uacb0\uc815\ub4e4\uc774 \uad6d\ubbfc \uc2a4\uc2a4\ub85c \ub610\ub294 \uadf8\ub4e4\uc5d0 \uc758\ud574 \uc120\ucd9c\ub41c \ub300\ud45c\uc790\ub4e4\uc5d0 \uc758\ud558\uc9c0 \uc54a\uace0 \uc219\uc758\uc5d0 \uc758\ud574 \ub9cc\ub4e4\uc5b4\uc9c0\uba74, \uac70\uae30\uc5d0\ub294 \ubbfc\uc8fc\uc801 \uc694\uc18c\uac00 \uc5c6\uac8c \ub41c\ub2e4; \uc774\ub7ec\ud55c \uc219\uc758 \uacfc\uc815\uc740 \uc5d8\ub9ac\ud2b8 \uc219\uc758(elite deliberation)\uc774\ub77c\uace0 \ubd88\ub9b0\ub2e4.", "sentence_answer": "\ub300\uc911\uc801 \uc219\uc758 \ubbfc\uc8fc\uc8fc\uc758\uc758 \ud55c \uac00\uc9c0 \ubaa9\uc801\uc740 \uc0ac\ud68c \uc774\uc288\ub4e4\uc5d0 \uad00\ud558\uc5ec \uad6c\uc18d\ub825 \uac83\uc774 \uc544\ub2c8\ub77c \ubcf4\ub2e4 \uc77c\ubc18 \uc5ec\ub860\uc744 \ub3c4\ucd9c(distill)\ud558\ub294\ub370 \uc0ac\uc6a9\ub420 \uc218 \uc788\ub2e4; \uc219\uc758 \uc5ec\ub860\uc870\uc0ac (deliberative opinion poll)\uacfc", "paragraph_id": "6547107-0-1", "paragraph_question": "question: \uc0ac\ud68c \uc774\uc288\ub4e4\uc5d0 \uad00\ud574 \uc77c\ubc18 \uc5ec\ub860\uc744 \ub3c4\ucd9c\ud558\uae30 \uc704\ud574 \uc219\uc758 \ubbfc\uc8fc\uc8fc\uc758\uc5d0\uc11c \uc0ac\uc6a9\ud558\ub294 \uac83\uc740?, context: \uc219\uc758 \ubbfc\uc8fc\uc8fc\uc758\ub294 \ubbfc\uc8fc\uc801 \uacb0\uc815\uc774 \uc801\ubc95\ud558\uae30 \uc704\ud574\uc11c\ub294, \ub2e8\uc9c0 \ud22c\ud45c\uc5d0\uc11c \ub098\ud0c0\ub098\ub294 \uc120\ud638\ub3c4\uc758 \ucd1d\ud569\uc774 \uc544\ub2c8\ub77c, \uc2e4\uc81c\uc801\uc778 \uc219\uc758\uac00 \uc120\ud589\ub418\uc5b4\uc57c \ud55c\ub2e4\uace0 \uac04\uc8fc\ud55c\ub2e4. \uc2e4\uc81c\uc801 \uc219\uc758\ub780, \uc774\ub97c\ud14c\uba74 \uacbd\uc81c\uc801 \ubd80\ub098 \ub610\ub294 \uc774\ud574 \uc9d1\ub2e8\ub4e4\uc758 \uc9c0\uc6d0\uc744 \ud1b5\ud574 \uc758\uc0ac\uacb0\uc815\uc790\ub4e4\uc774 \ubcf4\uc720\ud558\ub294, \uc815\uce58\uc801 \uad8c\ud55c\uc758 \ubd88\ud3c9\ub4f1\ud55c \uc0ac\uc774\uc5d0 \uc774\ub8e8\uc5b4\uc9c0\ub294 \uc219\uc758\uc774\ub2e4. \uc758\uc0ac\uacb0\uc815\uc790\ub4e4\uc774 \uc5b4\ub5a4 \uc81c\uc548\uc5d0 \ub300\ud574 \uc2e4\uc81c\uc801\uc73c\ub85c \uc219\uc758\ud55c \ud6c4\uc5d0 \ud569\uc758\uc5d0 \ub3c4\ub2ec\ud560 \uc218 \uc5c6\ub2e4\uba74, \uadf8\ub4e4\uc740 \ub2e4\uc218\uacb0 \uc6d0\ub9ac\ub77c\ub294 \ud615\uc2dd\uc73c\ub85c \uadf8 \uc81c\uc548\uc5d0 \ub300\ud574 \ud22c\ud45c\ud55c\ub2e4. \uc219\uc758 \ubbfc\uc8fc\uc8fc\uc758\ub294 \ub300\uc758 \uc788\ub2e4. \uc5d8\ub9ac\ud2b8 \uc219\uc758 \ubbfc\uc8fc\uc8fc\uc758(elitist deliberative democracy)\uc5d0\uc11c \uc785\ubc95\ubd80\uc640 \ubc95\uc6d0\ub4e4\uacfc \uac19\uc740, \uc5d8\ub9ac\ud2b8\uc801\uc6a9\ub41c\ub2e4; \ub300\uc911\uc801(populist) \uc219\uc758 \ubbfc\uc8fc\uc8fc\uc758\uc5d0\uc11c \uc219\uc758 \ubbfc\uc8fc\uc8fc\uc8fc\uc758 \uc6d0\ub9ac\ub4e4\uc740 \uc758\uc0ac\uacb0\uc815\uc744 \uc704\uc784 \ubc1b\uc740 \uc77c\ubc18 \uc2dc\ubbfc\ub4e4\uc758 \ubaa8\uc784\uc5d0 \uc801\uc6a9\ub41c\ub2e4. \ub300\uc911\uc801 \uc219\uc758 \ubbfc\uc8fc\uc8fc\uc758\uc758 \ud55c \uac00\uc9c0 \ubaa9\uc801\uc740 \uc0ac\ud68c \uc774\uc288\ub4e4\uc5d0 \uad00\ud558\uc5ec \uad6c\uc18d\ub825 \uac83\uc774 \uc544\ub2c8\ub77c \ubcf4\ub2e4 \uc77c\ubc18 \uc5ec\ub860\uc744 \ub3c4\ucd9c(distill)\ud558\ub294\ub370 \uc0ac\uc6a9\ub420 \uc218 \uc788\ub2e4; \uc219\uc758 \uc5ec\ub860\uc870\uc0ac(deliberative opinion poll)\uacfc \uac19\uc740 \uc218\ub2e8\ub4e4\uc740 \uc774\ub7ec\ud55c \ubaa9\ud45c\ub97c \ub2ec\uc131\ud558\ub294\ub370 \uc124\uacc4\ub418\uc5b4\uc654\ub2e4. \ub300\uc911\uc801 \uc219\uc758 \ubbfc\uc8fc\uc8fc\uc758\uc758 \ub610 \ub2e4\ub978 \ubaa9\uc801\uc740 \uc9c1\uc811 \ubbfc\uc8fc\uc8fc\uc758 \ud615\ud0dc\ub85c \uc5ed\ud560 \ud560 \uc218 \uc219\uc758\ub294 \uc77c\ubc18 \uc2dc\ubbfc\ub4e4\uc758 \ubaa8\uc784 \uac00\uc6b4\ub370 \u201c\uacf5\uacf5\uc758 \uc758\uc9c0\u201d\ub97c \ud615\uc131\ud558\uace0 \uad6c\uc18d\ub825 \uc788\ub294 \ubc95\uc744 \uc9c1\uc811 \ub9cc\ub4e4\uac8c \ub41c\ub2e4. \uc815\uce58\uc801 \uacb0\uc815\ub4e4\uc774 \uad6d\ubbfc \uc2a4\uc2a4\ub85c \ub610\ub294 \uadf8\ub4e4\uc5d0 \uc758\ud574 \uc120\ucd9c\ub41c \ub300\ud45c\uc790\ub4e4\uc5d0 \uc758\ud558\uc9c0 \uc54a\uace0 \uc219\uc758\uc5d0 \uc758\ud574 \ub9cc\ub4e4\uc5b4\uc9c0\uba74, \uac70\uae30\uc5d0\ub294 \ubbfc\uc8fc\uc801 \uc694\uc18c\uac00 \uc5c6\uac8c \ub41c\ub2e4; \uc774\ub7ec\ud55c \uc219\uc758 \uacfc\uc815\uc740 \uc5d8\ub9ac\ud2b8 \uc219\uc758(elite deliberation)\uc774\ub77c\uace0 \ubd88\ub9b0\ub2e4."} {"question": "\uc601\uad6d\uad70\uc744 \ud328\ud1f4\uc2dc\ud0a8 \uc804\ud22c\ub294?", "paragraph": "\ud3f0\ud2f0\uc561\uc740 \uae30\uc2b5\uc73c\ub85c \uac15\ub825\ud55c \uc694\uc0c8\ub97c \ud568\ub77d\uc2dc\ud0ac \uacc4\ud68d\uc744 \uc138\uc6e0\uace0, 5\uc6d4 7\uc77c \ubb34\uae30\ub97c \uc228\uae34 \uc57d 300\uba85\uc758 \uc804\uc0ac\uc640 \ud568\uaed8 \ub514\ud2b8\ub85c\uc774\ud2b8 \uc694\uc0c8\ub85c \ub4e4\uc5b4\uac14\ub2e4. \ud558\uc9c0\ub9cc \uc601\uad6d\uad70 \uc9c0\ud718\uad00\uc740 \uc774\ubbf8 \uc778\ub514\uc5b8 \uc5f0\ud569 \uc804\ub7b5\uc744 \uc219\uc9c0\ud558\uace0, \ubb34\uc7a5 \uacf5\uaca9\uc5d0 \ub300\ube44\ud588\ub2e4. \uc778\ub514\uc5b8\ub4e4\uc740 \uc791\uc804\uc774 \uc2e4\ud328\ud588\ub2e4\ub294 \uac83\uc744 \uc54c\uac8c \ub418\uc790 \uac04\ub2e8\ud788 \ud68c\uc758\ub97c \ud558\uace0 \uc774\ud2c0 \ub9cc\uc5d0 \uc694\uc0c8\ub97c \ud3ec\uc704\ud588\ub2e4. \ud3f0\ud2f0\uc561 \uc77c\ub2f9\uc740 \uc694\uc0c8 \ubc14\uae65\uc5d0\uc11c \ucc3e\uc544\ub0b8 \uc601\uad6d\uad70\uacfc \uc815\ucc29\ubbfc\ub4e4\uc5d0 \ub300\ud574\uc11c\ub294 \uc5ec\uc790\ub098 \uc544\uc774\ub97c \uac00\ub9ac\uc9c0 \uc54a\uace0, \ubaa8\ub450 \uc8fd\uc600\ub2e4. \uad70\uc778\ub4e4 \uc911 \ud55c \uba85\uc740 \uc624\ub300\ud638 \uc9c0\uc5ed\uc758 \uc778\ub514\uc5b8 \ubd80\uc871\uc758 \uad00\uc2b5\uc5d0 \ub530\ub77c \uc758\uc2dd\uc758 \uc81c\ubb3c\ub85c \uc2dd\uc721\ub418\uc5c8\ub2e4. \ud3ed\ub825\uc740 \uc601\uad6d\uc778\uc5d0\uac8c\ub9cc \ud574\ub2f9\ub418\uc5c8\uace0, \ud504\ub791\uc2a4 \uc815\ucc29\ubbfc\uc740 \uc77c\ubc18\uc801\uc73c\ub85c \uc548\uc804\uc744 \uc704\ud611\ubc1b\uc9c0 \uc54a\uc558\ub2e4. \uacb0\uad6d \uc5ec\uc12f \uc9c0\ud30c 900\uba85 \uc774\uc0c1\uc758 \uc804\uc0ac\uac00 \ud3ec\uc704\uc804\uc5d0 \ucc38\uac00\ud588\ub2e4. \uc601\uad6d\uad70\uc740 \uc6d0\uad70\uc758 \ub3c4\ucc29 \ud6c4, \ud3f0\ud2f0\uc561\uc758 \uc219\uc601\uc9c0\ub97c \uae09\uc2b5\ud588\uc9c0\ub9cc, \uc774\ubbf8 \ud3f0\ud2f0\uc561\uc774 \uc54c\uace0 \uae30\ub2e4\ub9ac\uace0 \uc788\uc5c8\uc73c\uba70, 7\uc6d4 31\uc77c\uc758 \u2018\ube14\ub7ec\ub514 \ub7f0 \uc804\ud22c\u2019\uc5d0\uc11c \uc601\uad6d\uad70\uc744 \ud328\ud1f4\uc2dc\ucf30\ub2e4.", "answer": "\ube14\ub7ec\ub514 \ub7f0 \uc804\ud22c", "sentence": "\uc601\uad6d\uad70\uc740 \uc6d0\uad70\uc758 \ub3c4\ucc29 \ud6c4, \ud3f0\ud2f0\uc561\uc758 \uc219\uc601\uc9c0\ub97c \uae09\uc2b5\ud588\uc9c0\ub9cc, \uc774\ubbf8 \ud3f0\ud2f0\uc561\uc774 \uc54c\uace0 \uae30\ub2e4\ub9ac\uace0 \uc788\uc5c8\uc73c\uba70, 7\uc6d4 31\uc77c\uc758 \u2018 \ube14\ub7ec\ub514 \ub7f0 \uc804\ud22c \u2019\uc5d0\uc11c \uc601\uad6d\uad70\uc744 \ud328\ud1f4\uc2dc\ucf30\ub2e4.", "paragraph_sentence": "\ud3f0\ud2f0\uc561\uc740 \uae30\uc2b5\uc73c\ub85c \uac15\ub825\ud55c \uc694\uc0c8\ub97c \ud568\ub77d\uc2dc\ud0ac \uacc4\ud68d\uc744 \uc138\uc6e0\uace0, 5\uc6d4 7\uc77c \ubb34\uae30\ub97c \uc228\uae34 \uc57d 300\uba85\uc758 \uc804\uc0ac\uc640 \ud568\uaed8 \ub514\ud2b8\ub85c\uc774\ud2b8 \uc694\uc0c8\ub85c \ub4e4\uc5b4\uac14\ub2e4. \ud558\uc9c0\ub9cc \uc601\uad6d\uad70 \uc9c0\ud718\uad00\uc740 \uc774\ubbf8 \uc778\ub514\uc5b8 \uc5f0\ud569 \uc804\ub7b5\uc744 \uc219\uc9c0\ud558\uace0, \ubb34\uc7a5 \uacf5\uaca9\uc5d0 \ub300\ube44\ud588\ub2e4. \uc778\ub514\uc5b8\ub4e4\uc740 \uc791\uc804\uc774 \uc2e4\ud328\ud588\ub2e4\ub294 \uac83\uc744 \uc54c\uac8c \ub418\uc790 \uac04\ub2e8\ud788 \ud68c\uc758\ub97c \ud558\uace0 \uc774\ud2c0 \ub9cc\uc5d0 \uc694\uc0c8\ub97c \ud3ec\uc704\ud588\ub2e4. \ud3f0\ud2f0\uc561 \uc77c\ub2f9\uc740 \uc694\uc0c8 \ubc14\uae65\uc5d0\uc11c \ucc3e\uc544\ub0b8 \uc601\uad6d\uad70\uacfc \uc815\ucc29\ubbfc\ub4e4\uc5d0 \ub300\ud574\uc11c\ub294 \uc5ec\uc790\ub098 \uc544\uc774\ub97c \uac00\ub9ac\uc9c0 \uc54a\uace0, \ubaa8\ub450 \uc8fd\uc600\ub2e4. \uad70\uc778\ub4e4 \uc911 \ud55c \uba85\uc740 \uc624\ub300\ud638 \uc9c0\uc5ed\uc758 \uc778\ub514\uc5b8 \ubd80\uc871\uc758 \uad00\uc2b5\uc5d0 \ub530\ub77c \uc758\uc2dd\uc758 \uc81c\ubb3c\ub85c \uc2dd\uc721\ub418\uc5c8\ub2e4. \ud3ed\ub825\uc740 \uc601\uad6d\uc778\uc5d0\uac8c\ub9cc \ud574\ub2f9\ub418\uc5c8\uace0, \ud504\ub791\uc2a4 \uc815\ucc29\ubbfc\uc740 \uc77c\ubc18\uc801\uc73c\ub85c \uc548\uc804\uc744 \uc704\ud611\ubc1b\uc9c0 \uc54a\uc558\ub2e4. \uacb0\uad6d \uc5ec\uc12f \uc9c0\ud30c 900\uba85 \uc774\uc0c1\uc758 \uc804\uc0ac\uac00 \ud3ec\uc704\uc804\uc5d0 \ucc38\uac00\ud588\ub2e4. \uc601\uad6d\uad70\uc740 \uc6d0\uad70\uc758 \ub3c4\ucc29 \ud6c4, \ud3f0\ud2f0\uc561\uc758 \uc219\uc601\uc9c0\ub97c \uae09\uc2b5\ud588\uc9c0\ub9cc, \uc774\ubbf8 \ud3f0\ud2f0\uc561\uc774 \uc54c\uace0 \uae30\ub2e4\ub9ac\uace0 \uc788\uc5c8\uc73c\uba70, 7\uc6d4 31\uc77c\uc758 \u2018 \ube14\ub7ec\ub514 \ub7f0 \uc804\ud22c \u2019\uc5d0\uc11c \uc601\uad6d\uad70\uc744 \ud328\ud1f4\uc2dc\ucf30\ub2e4. ", "paragraph_answer": "\ud3f0\ud2f0\uc561\uc740 \uae30\uc2b5\uc73c\ub85c \uac15\ub825\ud55c \uc694\uc0c8\ub97c \ud568\ub77d\uc2dc\ud0ac \uacc4\ud68d\uc744 \uc138\uc6e0\uace0, 5\uc6d4 7\uc77c \ubb34\uae30\ub97c \uc228\uae34 \uc57d 300\uba85\uc758 \uc804\uc0ac\uc640 \ud568\uaed8 \ub514\ud2b8\ub85c\uc774\ud2b8 \uc694\uc0c8\ub85c \ub4e4\uc5b4\uac14\ub2e4. \ud558\uc9c0\ub9cc \uc601\uad6d\uad70 \uc9c0\ud718\uad00\uc740 \uc774\ubbf8 \uc778\ub514\uc5b8 \uc5f0\ud569 \uc804\ub7b5\uc744 \uc219\uc9c0\ud558\uace0, \ubb34\uc7a5 \uacf5\uaca9\uc5d0 \ub300\ube44\ud588\ub2e4. \uc778\ub514\uc5b8\ub4e4\uc740 \uc791\uc804\uc774 \uc2e4\ud328\ud588\ub2e4\ub294 \uac83\uc744 \uc54c\uac8c \ub418\uc790 \uac04\ub2e8\ud788 \ud68c\uc758\ub97c \ud558\uace0 \uc774\ud2c0 \ub9cc\uc5d0 \uc694\uc0c8\ub97c \ud3ec\uc704\ud588\ub2e4. \ud3f0\ud2f0\uc561 \uc77c\ub2f9\uc740 \uc694\uc0c8 \ubc14\uae65\uc5d0\uc11c \ucc3e\uc544\ub0b8 \uc601\uad6d\uad70\uacfc \uc815\ucc29\ubbfc\ub4e4\uc5d0 \ub300\ud574\uc11c\ub294 \uc5ec\uc790\ub098 \uc544\uc774\ub97c \uac00\ub9ac\uc9c0 \uc54a\uace0, \ubaa8\ub450 \uc8fd\uc600\ub2e4. \uad70\uc778\ub4e4 \uc911 \ud55c \uba85\uc740 \uc624\ub300\ud638 \uc9c0\uc5ed\uc758 \uc778\ub514\uc5b8 \ubd80\uc871\uc758 \uad00\uc2b5\uc5d0 \ub530\ub77c \uc758\uc2dd\uc758 \uc81c\ubb3c\ub85c \uc2dd\uc721\ub418\uc5c8\ub2e4. \ud3ed\ub825\uc740 \uc601\uad6d\uc778\uc5d0\uac8c\ub9cc \ud574\ub2f9\ub418\uc5c8\uace0, \ud504\ub791\uc2a4 \uc815\ucc29\ubbfc\uc740 \uc77c\ubc18\uc801\uc73c\ub85c \uc548\uc804\uc744 \uc704\ud611\ubc1b\uc9c0 \uc54a\uc558\ub2e4. \uacb0\uad6d \uc5ec\uc12f \uc9c0\ud30c 900\uba85 \uc774\uc0c1\uc758 \uc804\uc0ac\uac00 \ud3ec\uc704\uc804\uc5d0 \ucc38\uac00\ud588\ub2e4. \uc601\uad6d\uad70\uc740 \uc6d0\uad70\uc758 \ub3c4\ucc29 \ud6c4, \ud3f0\ud2f0\uc561\uc758 \uc219\uc601\uc9c0\ub97c \uae09\uc2b5\ud588\uc9c0\ub9cc, \uc774\ubbf8 \ud3f0\ud2f0\uc561\uc774 \uc54c\uace0 \uae30\ub2e4\ub9ac\uace0 \uc788\uc5c8\uc73c\uba70, 7\uc6d4 31\uc77c\uc758 \u2018 \ube14\ub7ec\ub514 \ub7f0 \uc804\ud22c \u2019\uc5d0\uc11c \uc601\uad6d\uad70\uc744 \ud328\ud1f4\uc2dc\ucf30\ub2e4.", "sentence_answer": "\uc601\uad6d\uad70\uc740 \uc6d0\uad70\uc758 \ub3c4\ucc29 \ud6c4, \ud3f0\ud2f0\uc561\uc758 \uc219\uc601\uc9c0\ub97c \uae09\uc2b5\ud588\uc9c0\ub9cc, \uc774\ubbf8 \ud3f0\ud2f0\uc561\uc774 \uc54c\uace0 \uae30\ub2e4\ub9ac\uace0 \uc788\uc5c8\uc73c\uba70, 7\uc6d4 31\uc77c\uc758 \u2018 \ube14\ub7ec\ub514 \ub7f0 \uc804\ud22c \u2019\uc5d0\uc11c \uc601\uad6d\uad70\uc744 \ud328\ud1f4\uc2dc\ucf30\ub2e4.", "paragraph_id": "6567978-11-0", "paragraph_question": "question: \uc601\uad6d\uad70\uc744 \ud328\ud1f4\uc2dc\ud0a8 \uc804\ud22c\ub294?, context: \ud3f0\ud2f0\uc561\uc740 \uae30\uc2b5\uc73c\ub85c \uac15\ub825\ud55c \uc694\uc0c8\ub97c \ud568\ub77d\uc2dc\ud0ac \uacc4\ud68d\uc744 \uc138\uc6e0\uace0, 5\uc6d4 7\uc77c \ubb34\uae30\ub97c \uc228\uae34 \uc57d 300\uba85\uc758 \uc804\uc0ac\uc640 \ud568\uaed8 \ub514\ud2b8\ub85c\uc774\ud2b8 \uc694\uc0c8\ub85c \ub4e4\uc5b4\uac14\ub2e4. \ud558\uc9c0\ub9cc \uc601\uad6d\uad70 \uc9c0\ud718\uad00\uc740 \uc774\ubbf8 \uc778\ub514\uc5b8 \uc5f0\ud569 \uc804\ub7b5\uc744 \uc219\uc9c0\ud558\uace0, \ubb34\uc7a5 \uacf5\uaca9\uc5d0 \ub300\ube44\ud588\ub2e4. \uc778\ub514\uc5b8\ub4e4\uc740 \uc791\uc804\uc774 \uc2e4\ud328\ud588\ub2e4\ub294 \uac83\uc744 \uc54c\uac8c \ub418\uc790 \uac04\ub2e8\ud788 \ud68c\uc758\ub97c \ud558\uace0 \uc774\ud2c0 \ub9cc\uc5d0 \uc694\uc0c8\ub97c \ud3ec\uc704\ud588\ub2e4. \ud3f0\ud2f0\uc561 \uc77c\ub2f9\uc740 \uc694\uc0c8 \ubc14\uae65\uc5d0\uc11c \ucc3e\uc544\ub0b8 \uc601\uad6d\uad70\uacfc \uc815\ucc29\ubbfc\ub4e4\uc5d0 \ub300\ud574\uc11c\ub294 \uc5ec\uc790\ub098 \uc544\uc774\ub97c \uac00\ub9ac\uc9c0 \uc54a\uace0, \ubaa8\ub450 \uc8fd\uc600\ub2e4. \uad70\uc778\ub4e4 \uc911 \ud55c \uba85\uc740 \uc624\ub300\ud638 \uc9c0\uc5ed\uc758 \uc778\ub514\uc5b8 \ubd80\uc871\uc758 \uad00\uc2b5\uc5d0 \ub530\ub77c \uc758\uc2dd\uc758 \uc81c\ubb3c\ub85c \uc2dd\uc721\ub418\uc5c8\ub2e4. \ud3ed\ub825\uc740 \uc601\uad6d\uc778\uc5d0\uac8c\ub9cc \ud574\ub2f9\ub418\uc5c8\uace0, \ud504\ub791\uc2a4 \uc815\ucc29\ubbfc\uc740 \uc77c\ubc18\uc801\uc73c\ub85c \uc548\uc804\uc744 \uc704\ud611\ubc1b\uc9c0 \uc54a\uc558\ub2e4. \uacb0\uad6d \uc5ec\uc12f \uc9c0\ud30c 900\uba85 \uc774\uc0c1\uc758 \uc804\uc0ac\uac00 \ud3ec\uc704\uc804\uc5d0 \ucc38\uac00\ud588\ub2e4. \uc601\uad6d\uad70\uc740 \uc6d0\uad70\uc758 \ub3c4\ucc29 \ud6c4, \ud3f0\ud2f0\uc561\uc758 \uc219\uc601\uc9c0\ub97c \uae09\uc2b5\ud588\uc9c0\ub9cc, \uc774\ubbf8 \ud3f0\ud2f0\uc561\uc774 \uc54c\uace0 \uae30\ub2e4\ub9ac\uace0 \uc788\uc5c8\uc73c\uba70, 7\uc6d4 31\uc77c\uc758 \u2018\ube14\ub7ec\ub514 \ub7f0 \uc804\ud22c\u2019\uc5d0\uc11c \uc601\uad6d\uad70\uc744 \ud328\ud1f4\uc2dc\ucf30\ub2e4."} {"question": "\ubbf8\uad6d\uc758 \ub274\uc695 \ud0c0\uc784\uc2a4\uc5d0\uc11c \ucd5c\uc9c4\uc2e4\uc744 \ubb34\uc5c7\uc758 \ud53c\ud574\uc790\ub77c\uace0 \ud558\uc600\ub294\uac00?", "paragraph": "\ud55c\ub098\ub77c\ub2f9 \ud64d\uc900\ud45c \uc6d0\ub0b4 \ub300\ud45c\ub294 2008\ub144 10\uc6d4 5\uc77c\uc5d0 \uc788\uc740 \uae30\uc790\uac04\ub2f4\ud68c\uc5d0\uc11c \"\uc815\ubd80\uac00 \ucd94\uc9c4 \uc911\uc778 \uc0ac\uc774\ubc84 \ubaa8\uc695\uc8c4\uc640 \uc778\ud130\ub137 \uc2e4\uba85\uc81c\uac00 \uc774\ubc88 \uc815\uae30 \uad6d\ud68c\uc5d0\uc11c \ubc18\ub4dc\uc2dc \ud1b5\uacfc\ub418\ub3c4\ub85d \ub178\ub825\ud560 \uac83\uc774\ub2e4. \uc775\uba85\uc131\uc5d0 \uc228\uc740 \uc0ac\uc774\ubc84 \ud3ed\ub825\uc774 \ub09c\ubb34\ud558\ub294 \uac83\uc740 \ucc38\uc73c\ub85c \uc798\ubabb\ub41c \uac83\uc774\ub2e4. \ubbf8\uad6d\uc758 \ub274\uc695 \ud0c0\uc784\uc2a4\uc5d0\uc11c\ub3c4 \uc5b4\uc81c \ucd5c\uc9c4\uc2e4 \uc528\ub97c \uc778\ud130\ub137 \uc545\ud50c\uc758 \ud53c\ud574\uc790\ub77c\uace0 \ud588\ub2e4. \uc778\ud130\ub137 \uc545\ud50c\uc740 \ucc38\uc73c\ub85c \ube44\uac81\ud55c \uc9d3\uc774\uba70, \uc778\ud130\ub137 \uacf5\uac04\uc774 \ub9c8\uce58 \ud654\uc7a5\uc2e4 \ub2f4\ubcbc\ub77d\ucc98\ub7fc \uadf8\ub807\uac8c \ucd94\uc545\ud55c \uacf5\uac04\uc73c\ub85c \ub098\ud0c0\ub098\ub294 \uac83\uc740 \uc633\uc9c0 \uc54a\ub2e4\"\ub77c\uace0 \ub9d0\ud588\ub2e4. \uc774\uc5d0 \uac19\uc740 \ub0a0 \ubbfc\uc8fc\ub2f9 \uc18c\uc18d \uad6d\ud68c \ubb38\ud654\uccb4\uc721\uad00\uad11\ubc29\uc1a1\ud1b5\uc2e0\uc704\uc6d0\ud68c \uc704\uc6d0 \uc77c\ub3d9\uc740 \uc131\uba85\uc11c\uc5d0\uc11c \"\uc775\uba85\uc131\uc774 \ud2b9\uc9d5\uc778 \uc778\ud130\ub137 \uacf5\uac04\uc5d0\uc11c \ubd88\uc21c\ud55c \uc758\ub3c4\ub85c \uc545\uc131 \ub313\uae00\uc744 \uc62c\ub824 \uc0c1\ub300\ubc29\uc5d0\uac8c \ud68c\ubcf5\ud560 \uc218 \uc5c6\ub294 \uc544\ud514\uc744 \uc8fc\ub294 \uac83\uc5d0 \ub300\ud55c \uc77c\uc815 \ubd80\ubd84\uc758 \uaddc\uc81c\uc5d0 \ubc18\ub300\ud558\uc9c0 \uc54a\ub294\ub2e4. \uadf8\ub7ec\ub098 \uc815\ubd80 \uc5ec\ub2f9\uc774 \uc778\ud130\ub137 \uc2e4\uba85\uc81c\ub97c \uc2e4\uc2dc\ud558\ub824\ub294 \uc228\uc740 \uc758\ub3c4\ub294 \uc815\ubd80 \ube44\ud310\uc801, \ubc18\uc815\ubd80\uc801 \uc5ec\ub860 \uc8fc\ub3c4\uc790\ub4e4\uc744 \uc9e7\uc740 \uc2dc\uac04 \uc548\uc5d0 \uc2e0\uc18d\ud558\uac8c \uc0c9\ucd9c\ud574 \ucc98\ubc8c\ud558\ub824\ub294 \uac83\uc774\ub2e4. \uc5c4\uc911 \ucc98\ubc8c\ud558\ub294 \ud604\ud589\ubc95\uc774 \uc788\uc74c\uc5d0\ub3c4 \ubd88\uad6c\ud558\uace0 \ubcc4\ub3c4\ub85c \uc0ac\uc774\ubc84 \ubaa8\uc695\uc8c4\ub97c \uc2e0\uc124\ud55c\ub2e4\ub294 \uac83\uc740 \uacb0\uad6d \ub124\ud2f0\uc98c\ub4e4\uc774 \uc815\ubd80 \ube44\ud310\uc801 \uc5ec\ub860 \ud615\uc131\uc5d0 \uac1c\uc785\ud558\ub294 \uac83\uc744 \ub9c9\uae30 \uc704\ud55c \ud611\ubc15\uc131 \ucc98\ubc8c \uaddc\uc815\uc744 \ub9cc\ub4dc\ub294 \uac83\uc77c \ubfd0\uc774\ub2e4. \ub354\uc6b1 \uc548\ud0c0\uae5d\uace0 \ubd84\ub178\uc2a4\ub7ec\uc6b4 \uac83\uc740 \uc0ac\ub9dd\ud55c \ucd5c\uc9c4\uc2e4 \uc528\ub97c \ud551\uacc4\uc0bc\uc544 \uc790\uc2e0\ub4e4\uc758 \uc18d\ub0b4\ub97c \uc228\uae34 \ucc44 \ub124\ud2f0\uc98c\uc744 \ud1b5\uc81c\ud558\uaca0\ub2e4\ub294 \ubc18\uc724\ub9ac\uc801 \ubc1c\uc0c1\uc774\ub2e4\"\ub77c\uace0 \ub9d0\ud588\ub2e4.", "answer": "\uc778\ud130\ub137 \uc545\ud50c", "sentence": "\ubbf8\uad6d\uc758 \ub274\uc695 \ud0c0\uc784\uc2a4\uc5d0\uc11c\ub3c4 \uc5b4\uc81c \ucd5c\uc9c4\uc2e4 \uc528\ub97c \uc778\ud130\ub137 \uc545\ud50c \uc758 \ud53c\ud574\uc790\ub77c\uace0 \ud588\ub2e4.", "paragraph_sentence": "\ud55c\ub098\ub77c\ub2f9 \ud64d\uc900\ud45c \uc6d0\ub0b4 \ub300\ud45c\ub294 2008\ub144 10\uc6d4 5\uc77c\uc5d0 \uc788\uc740 \uae30\uc790\uac04\ub2f4\ud68c\uc5d0\uc11c \"\uc815\ubd80\uac00 \ucd94\uc9c4 \uc911\uc778 \uc0ac\uc774\ubc84 \ubaa8\uc695\uc8c4\uc640 \uc778\ud130\ub137 \uc2e4\uba85\uc81c\uac00 \uc774\ubc88 \uc815\uae30 \uad6d\ud68c\uc5d0\uc11c \ubc18\ub4dc\uc2dc \ud1b5\uacfc\ub418\ub3c4\ub85d \ub178\ub825\ud560 \uac83\uc774\ub2e4. \uc775\uba85\uc131\uc5d0 \uc228\uc740 \uc0ac\uc774\ubc84 \ud3ed\ub825\uc774 \ub09c\ubb34\ud558\ub294 \uac83\uc740 \ucc38\uc73c\ub85c \uc798\ubabb\ub41c \uac83\uc774\ub2e4. \ubbf8\uad6d\uc758 \ub274\uc695 \ud0c0\uc784\uc2a4\uc5d0\uc11c\ub3c4 \uc5b4\uc81c \ucd5c\uc9c4\uc2e4 \uc528\ub97c \uc778\ud130\ub137 \uc545\ud50c \uc758 \ud53c\ud574\uc790\ub77c\uace0 \ud588\ub2e4. \uc778\ud130\ub137 \uc545\ud50c\uc740 \ucc38\uc73c\ub85c \ube44\uac81\ud55c \uc9d3\uc774\uba70, \uc778\ud130\ub137 \uacf5\uac04\uc774 \ub9c8\uce58 \ud654\uc7a5\uc2e4 \ub2f4\ubcbc\ub77d\ucc98\ub7fc \uadf8\ub807\uac8c \ucd94\uc545\ud55c \uacf5\uac04\uc73c\ub85c \ub098\ud0c0\ub098\ub294 \uac83\uc740 \uc633\uc9c0 \uc54a\ub2e4\"\ub77c\uace0 \ub9d0\ud588\ub2e4. \uc774\uc5d0 \uac19\uc740 \ub0a0 \ubbfc\uc8fc\ub2f9 \uc18c\uc18d \uad6d\ud68c \ubb38\ud654\uccb4\uc721\uad00\uad11\ubc29\uc1a1\ud1b5\uc2e0\uc704\uc6d0\ud68c \uc704\uc6d0 \uc77c\ub3d9\uc740 \uc131\uba85\uc11c\uc5d0\uc11c \"\uc775\uba85\uc131\uc774 \ud2b9\uc9d5\uc778 \uc778\ud130\ub137 \uacf5\uac04\uc5d0\uc11c \ubd88\uc21c\ud55c \uc758\ub3c4\ub85c \uc545\uc131 \ub313\uae00\uc744 \uc62c\ub824 \uc0c1\ub300\ubc29\uc5d0\uac8c \ud68c\ubcf5\ud560 \uc218 \uc5c6\ub294 \uc544\ud514\uc744 \uc8fc\ub294 \uac83\uc5d0 \ub300\ud55c \uc77c\uc815 \ubd80\ubd84\uc758 \uaddc\uc81c\uc5d0 \ubc18\ub300\ud558\uc9c0 \uc54a\ub294\ub2e4. \uadf8\ub7ec\ub098 \uc815\ubd80 \uc5ec\ub2f9\uc774 \uc778\ud130\ub137 \uc2e4\uba85\uc81c\ub97c \uc2e4\uc2dc\ud558\ub824\ub294 \uc228\uc740 \uc758\ub3c4\ub294 \uc815\ubd80 \ube44\ud310\uc801, \ubc18\uc815\ubd80\uc801 \uc5ec\ub860 \uc8fc\ub3c4\uc790\ub4e4\uc744 \uc9e7\uc740 \uc2dc\uac04 \uc548\uc5d0 \uc2e0\uc18d\ud558\uac8c \uc0c9\ucd9c\ud574 \ucc98\ubc8c\ud558\ub824\ub294 \uac83\uc774\ub2e4. \uc5c4\uc911 \ucc98\ubc8c\ud558\ub294 \ud604\ud589\ubc95\uc774 \uc788\uc74c\uc5d0\ub3c4 \ubd88\uad6c\ud558\uace0 \ubcc4\ub3c4\ub85c \uc0ac\uc774\ubc84 \ubaa8\uc695\uc8c4\ub97c \uc2e0\uc124\ud55c\ub2e4\ub294 \uac83\uc740 \uacb0\uad6d \ub124\ud2f0\uc98c\ub4e4\uc774 \uc815\ubd80 \ube44\ud310\uc801 \uc5ec\ub860 \ud615\uc131\uc5d0 \uac1c\uc785\ud558\ub294 \uac83\uc744 \ub9c9\uae30 \uc704\ud55c \ud611\ubc15\uc131 \ucc98\ubc8c \uaddc\uc815\uc744 \ub9cc\ub4dc\ub294 \uac83\uc77c \ubfd0\uc774\ub2e4. \ub354\uc6b1 \uc548\ud0c0\uae5d\uace0 \ubd84\ub178\uc2a4\ub7ec\uc6b4 \uac83\uc740 \uc0ac\ub9dd\ud55c \ucd5c\uc9c4\uc2e4 \uc528\ub97c \ud551\uacc4\uc0bc\uc544 \uc790\uc2e0\ub4e4\uc758 \uc18d\ub0b4\ub97c \uc228\uae34 \ucc44 \ub124\ud2f0\uc98c\uc744 \ud1b5\uc81c\ud558\uaca0\ub2e4\ub294 \ubc18\uc724\ub9ac\uc801 \ubc1c\uc0c1\uc774\ub2e4\"\ub77c\uace0 \ub9d0\ud588\ub2e4.", "paragraph_answer": "\ud55c\ub098\ub77c\ub2f9 \ud64d\uc900\ud45c \uc6d0\ub0b4 \ub300\ud45c\ub294 2008\ub144 10\uc6d4 5\uc77c\uc5d0 \uc788\uc740 \uae30\uc790\uac04\ub2f4\ud68c\uc5d0\uc11c \"\uc815\ubd80\uac00 \ucd94\uc9c4 \uc911\uc778 \uc0ac\uc774\ubc84 \ubaa8\uc695\uc8c4\uc640 \uc778\ud130\ub137 \uc2e4\uba85\uc81c\uac00 \uc774\ubc88 \uc815\uae30 \uad6d\ud68c\uc5d0\uc11c \ubc18\ub4dc\uc2dc \ud1b5\uacfc\ub418\ub3c4\ub85d \ub178\ub825\ud560 \uac83\uc774\ub2e4. \uc775\uba85\uc131\uc5d0 \uc228\uc740 \uc0ac\uc774\ubc84 \ud3ed\ub825\uc774 \ub09c\ubb34\ud558\ub294 \uac83\uc740 \ucc38\uc73c\ub85c \uc798\ubabb\ub41c \uac83\uc774\ub2e4. \ubbf8\uad6d\uc758 \ub274\uc695 \ud0c0\uc784\uc2a4\uc5d0\uc11c\ub3c4 \uc5b4\uc81c \ucd5c\uc9c4\uc2e4 \uc528\ub97c \uc778\ud130\ub137 \uc545\ud50c \uc758 \ud53c\ud574\uc790\ub77c\uace0 \ud588\ub2e4. \uc778\ud130\ub137 \uc545\ud50c\uc740 \ucc38\uc73c\ub85c \ube44\uac81\ud55c \uc9d3\uc774\uba70, \uc778\ud130\ub137 \uacf5\uac04\uc774 \ub9c8\uce58 \ud654\uc7a5\uc2e4 \ub2f4\ubcbc\ub77d\ucc98\ub7fc \uadf8\ub807\uac8c \ucd94\uc545\ud55c \uacf5\uac04\uc73c\ub85c \ub098\ud0c0\ub098\ub294 \uac83\uc740 \uc633\uc9c0 \uc54a\ub2e4\"\ub77c\uace0 \ub9d0\ud588\ub2e4. \uc774\uc5d0 \uac19\uc740 \ub0a0 \ubbfc\uc8fc\ub2f9 \uc18c\uc18d \uad6d\ud68c \ubb38\ud654\uccb4\uc721\uad00\uad11\ubc29\uc1a1\ud1b5\uc2e0\uc704\uc6d0\ud68c \uc704\uc6d0 \uc77c\ub3d9\uc740 \uc131\uba85\uc11c\uc5d0\uc11c \"\uc775\uba85\uc131\uc774 \ud2b9\uc9d5\uc778 \uc778\ud130\ub137 \uacf5\uac04\uc5d0\uc11c \ubd88\uc21c\ud55c \uc758\ub3c4\ub85c \uc545\uc131 \ub313\uae00\uc744 \uc62c\ub824 \uc0c1\ub300\ubc29\uc5d0\uac8c \ud68c\ubcf5\ud560 \uc218 \uc5c6\ub294 \uc544\ud514\uc744 \uc8fc\ub294 \uac83\uc5d0 \ub300\ud55c \uc77c\uc815 \ubd80\ubd84\uc758 \uaddc\uc81c\uc5d0 \ubc18\ub300\ud558\uc9c0 \uc54a\ub294\ub2e4. \uadf8\ub7ec\ub098 \uc815\ubd80 \uc5ec\ub2f9\uc774 \uc778\ud130\ub137 \uc2e4\uba85\uc81c\ub97c \uc2e4\uc2dc\ud558\ub824\ub294 \uc228\uc740 \uc758\ub3c4\ub294 \uc815\ubd80 \ube44\ud310\uc801, \ubc18\uc815\ubd80\uc801 \uc5ec\ub860 \uc8fc\ub3c4\uc790\ub4e4\uc744 \uc9e7\uc740 \uc2dc\uac04 \uc548\uc5d0 \uc2e0\uc18d\ud558\uac8c \uc0c9\ucd9c\ud574 \ucc98\ubc8c\ud558\ub824\ub294 \uac83\uc774\ub2e4. \uc5c4\uc911 \ucc98\ubc8c\ud558\ub294 \ud604\ud589\ubc95\uc774 \uc788\uc74c\uc5d0\ub3c4 \ubd88\uad6c\ud558\uace0 \ubcc4\ub3c4\ub85c \uc0ac\uc774\ubc84 \ubaa8\uc695\uc8c4\ub97c \uc2e0\uc124\ud55c\ub2e4\ub294 \uac83\uc740 \uacb0\uad6d \ub124\ud2f0\uc98c\ub4e4\uc774 \uc815\ubd80 \ube44\ud310\uc801 \uc5ec\ub860 \ud615\uc131\uc5d0 \uac1c\uc785\ud558\ub294 \uac83\uc744 \ub9c9\uae30 \uc704\ud55c \ud611\ubc15\uc131 \ucc98\ubc8c \uaddc\uc815\uc744 \ub9cc\ub4dc\ub294 \uac83\uc77c \ubfd0\uc774\ub2e4. \ub354\uc6b1 \uc548\ud0c0\uae5d\uace0 \ubd84\ub178\uc2a4\ub7ec\uc6b4 \uac83\uc740 \uc0ac\ub9dd\ud55c \ucd5c\uc9c4\uc2e4 \uc528\ub97c \ud551\uacc4\uc0bc\uc544 \uc790\uc2e0\ub4e4\uc758 \uc18d\ub0b4\ub97c \uc228\uae34 \ucc44 \ub124\ud2f0\uc98c\uc744 \ud1b5\uc81c\ud558\uaca0\ub2e4\ub294 \ubc18\uc724\ub9ac\uc801 \ubc1c\uc0c1\uc774\ub2e4\"\ub77c\uace0 \ub9d0\ud588\ub2e4.", "sentence_answer": "\ubbf8\uad6d\uc758 \ub274\uc695 \ud0c0\uc784\uc2a4\uc5d0\uc11c\ub3c4 \uc5b4\uc81c \ucd5c\uc9c4\uc2e4 \uc528\ub97c \uc778\ud130\ub137 \uc545\ud50c \uc758 \ud53c\ud574\uc790\ub77c\uace0 \ud588\ub2e4.", "paragraph_id": "6560245-24-1", "paragraph_question": "question: \ubbf8\uad6d\uc758 \ub274\uc695 \ud0c0\uc784\uc2a4\uc5d0\uc11c \ucd5c\uc9c4\uc2e4\uc744 \ubb34\uc5c7\uc758 \ud53c\ud574\uc790\ub77c\uace0 \ud558\uc600\ub294\uac00?, context: \ud55c\ub098\ub77c\ub2f9 \ud64d\uc900\ud45c \uc6d0\ub0b4 \ub300\ud45c\ub294 2008\ub144 10\uc6d4 5\uc77c\uc5d0 \uc788\uc740 \uae30\uc790\uac04\ub2f4\ud68c\uc5d0\uc11c \"\uc815\ubd80\uac00 \ucd94\uc9c4 \uc911\uc778 \uc0ac\uc774\ubc84 \ubaa8\uc695\uc8c4\uc640 \uc778\ud130\ub137 \uc2e4\uba85\uc81c\uac00 \uc774\ubc88 \uc815\uae30 \uad6d\ud68c\uc5d0\uc11c \ubc18\ub4dc\uc2dc \ud1b5\uacfc\ub418\ub3c4\ub85d \ub178\ub825\ud560 \uac83\uc774\ub2e4. \uc775\uba85\uc131\uc5d0 \uc228\uc740 \uc0ac\uc774\ubc84 \ud3ed\ub825\uc774 \ub09c\ubb34\ud558\ub294 \uac83\uc740 \ucc38\uc73c\ub85c \uc798\ubabb\ub41c \uac83\uc774\ub2e4. \ubbf8\uad6d\uc758 \ub274\uc695 \ud0c0\uc784\uc2a4\uc5d0\uc11c\ub3c4 \uc5b4\uc81c \ucd5c\uc9c4\uc2e4 \uc528\ub97c \uc778\ud130\ub137 \uc545\ud50c\uc758 \ud53c\ud574\uc790\ub77c\uace0 \ud588\ub2e4. \uc778\ud130\ub137 \uc545\ud50c\uc740 \ucc38\uc73c\ub85c \ube44\uac81\ud55c \uc9d3\uc774\uba70, \uc778\ud130\ub137 \uacf5\uac04\uc774 \ub9c8\uce58 \ud654\uc7a5\uc2e4 \ub2f4\ubcbc\ub77d\ucc98\ub7fc \uadf8\ub807\uac8c \ucd94\uc545\ud55c \uacf5\uac04\uc73c\ub85c \ub098\ud0c0\ub098\ub294 \uac83\uc740 \uc633\uc9c0 \uc54a\ub2e4\"\ub77c\uace0 \ub9d0\ud588\ub2e4. \uc774\uc5d0 \uac19\uc740 \ub0a0 \ubbfc\uc8fc\ub2f9 \uc18c\uc18d \uad6d\ud68c \ubb38\ud654\uccb4\uc721\uad00\uad11\ubc29\uc1a1\ud1b5\uc2e0\uc704\uc6d0\ud68c \uc704\uc6d0 \uc77c\ub3d9\uc740 \uc131\uba85\uc11c\uc5d0\uc11c \"\uc775\uba85\uc131\uc774 \ud2b9\uc9d5\uc778 \uc778\ud130\ub137 \uacf5\uac04\uc5d0\uc11c \ubd88\uc21c\ud55c \uc758\ub3c4\ub85c \uc545\uc131 \ub313\uae00\uc744 \uc62c\ub824 \uc0c1\ub300\ubc29\uc5d0\uac8c \ud68c\ubcf5\ud560 \uc218 \uc5c6\ub294 \uc544\ud514\uc744 \uc8fc\ub294 \uac83\uc5d0 \ub300\ud55c \uc77c\uc815 \ubd80\ubd84\uc758 \uaddc\uc81c\uc5d0 \ubc18\ub300\ud558\uc9c0 \uc54a\ub294\ub2e4. \uadf8\ub7ec\ub098 \uc815\ubd80 \uc5ec\ub2f9\uc774 \uc778\ud130\ub137 \uc2e4\uba85\uc81c\ub97c \uc2e4\uc2dc\ud558\ub824\ub294 \uc228\uc740 \uc758\ub3c4\ub294 \uc815\ubd80 \ube44\ud310\uc801, \ubc18\uc815\ubd80\uc801 \uc5ec\ub860 \uc8fc\ub3c4\uc790\ub4e4\uc744 \uc9e7\uc740 \uc2dc\uac04 \uc548\uc5d0 \uc2e0\uc18d\ud558\uac8c \uc0c9\ucd9c\ud574 \ucc98\ubc8c\ud558\ub824\ub294 \uac83\uc774\ub2e4. \uc5c4\uc911 \ucc98\ubc8c\ud558\ub294 \ud604\ud589\ubc95\uc774 \uc788\uc74c\uc5d0\ub3c4 \ubd88\uad6c\ud558\uace0 \ubcc4\ub3c4\ub85c \uc0ac\uc774\ubc84 \ubaa8\uc695\uc8c4\ub97c \uc2e0\uc124\ud55c\ub2e4\ub294 \uac83\uc740 \uacb0\uad6d \ub124\ud2f0\uc98c\ub4e4\uc774 \uc815\ubd80 \ube44\ud310\uc801 \uc5ec\ub860 \ud615\uc131\uc5d0 \uac1c\uc785\ud558\ub294 \uac83\uc744 \ub9c9\uae30 \uc704\ud55c \ud611\ubc15\uc131 \ucc98\ubc8c \uaddc\uc815\uc744 \ub9cc\ub4dc\ub294 \uac83\uc77c \ubfd0\uc774\ub2e4. \ub354\uc6b1 \uc548\ud0c0\uae5d\uace0 \ubd84\ub178\uc2a4\ub7ec\uc6b4 \uac83\uc740 \uc0ac\ub9dd\ud55c \ucd5c\uc9c4\uc2e4 \uc528\ub97c \ud551\uacc4\uc0bc\uc544 \uc790\uc2e0\ub4e4\uc758 \uc18d\ub0b4\ub97c \uc228\uae34 \ucc44 \ub124\ud2f0\uc98c\uc744 \ud1b5\uc81c\ud558\uaca0\ub2e4\ub294 \ubc18\uc724\ub9ac\uc801 \ubc1c\uc0c1\uc774\ub2e4\"\ub77c\uace0 \ub9d0\ud588\ub2e4."} {"question": "1941\ub144 \ub3c5\uc77c\uc758 \uc18c\ub828 \uce68\uacf5 \uacc4\ud68d\uc758 \uc554\ud638\uba85\uc740?", "paragraph": "1941\ub144 \ucd08 \ub3c5\uc77c \ucd5c\uace0\uc0ac\ub839\ubd80\ub294 \uc18c\ub828 \uce68\uacf5 \uacc4\ud68d\uc744 \uc2dc\uc791\ud588\ub2e4. \uc554\ud638\uba85\uc740 \ubc14\ub974\ubc14\ub85c\uc0ac \uc791\uc804. 5\uc6d4 15\uc77c \ub9cc\uc288\ud0c0\uc778\uc740 \uc81c56\uae30\uac11\uad70\ub2e8 \uad70\ub2e8\uc7a5\uc73c\ub85c \uc784\uba85\ub418\uc5c8\ub2e4. \uadf8\ub294 5\uc6d4\uc5d0 \ud5a5\ud6c4 \ub300\uacf5\uc138\uac00 \uc2dc\uc791\ub41c\ub2e4\ub294 \uc0c1\uc138\ud55c \uacc4\ud68d\uc5d0 \ub300\ud55c \ube0c\ub9ac\ud551\uc744 \ubc1b\uc740 250\uba85 \uc7a5\uc131 \uc911 \ud55c \uba85\uc774\uc5c8\ub2e4. \ub9cc\uc288\ud0c0\uc778\uc758 56\uad70\ub2e8\uc740 \uc5d0\ub9ac\ud788 \ud68c\ud504\ub108 \ub300\uc7a5\uc758 \uc81c4\uae30\uac11\uad70\uc5d0 \ubc30\uc18d\ub418\uc5c8\uace0, \uc81c4\uae30\uac11\uad70\uc740 \ube4c\ud5ec\ub984 \ub9ac\ud130 \ud3f0 \ub808\ud504 \uc6d0\uc218\uc758 \ubd81\ubd80 \uc9d1\ub2e8\uad70\uc5d0 \ubc30\uc18d\ub418\uc5c8\ub2e4. \ubd81\ubd80 \uc9d1\ub2e8\uad70\uc758 \uc784\ubb34\ub294 \ubc1c\ud2b8 3\uad6d\uc744 \uac70\uccd0 \ub808\ub2cc\uadf8\ub77c\ub4dc\ub85c \uc9c4\uaca9\ud558\ub294 \uac83\uc774\uc5c8\ub2e4. \ub9cc\uc288\ud0c0\uc778\uc740 \uacf5\uc138 \uc2dc\uc791 \ubd88\uacfc 6\uc77c \uc804\uc5d0\uc57c \uc804\uc120\uc5d0 \ub3c4\ucc29\ud588\ub2e4. 1941\ub144 6\uc6d4 22\uc77c \ubc14\ub974\ubc14\ub85c\uc0ac \uc791\uc804\uc774 \uac1c\uc2dc\ub418\uc5b4 \ub3d9\ubd80\uc804\uc120 \uc804\uccb4\uc5d0\uc11c \ub300\uaddc\ubaa8\uc758 \ub3c5\uc77c\uad70 \uacf5\uaca9\uc774 \uc2dc\uc791\ub418\uc5c8\ub2e4. \ub9cc\uc288\ud0c0\uc778\uc758 56\uad70\ub2e8\uc740 \uac8c\uc624\ub974\ud06c\ud55c\uc2a4 \ud558\uc778\ud558\ub974\ud2b8\uc758 \uc81c41\uae30\uac11\uad70\ub2e8\uacfc \ud568\uaed8 \ub2e4\uc6b0\uac00\ud504\ud544\uc2a4 \uadfc\uad50\uc758 \uad50\ub7c9\ub4e4\uc744 \ud655\ubcf4\ud558\uae30 \uc704\ud574 \ub4dc\ub2c8\ubc14 \uac15 \ucabd\uc73c\ub85c \uc9c4\uaca9\ud588\ub2e4. \uc18c\ub828\uad70\uc774 \uc5ec\ub7ec \ucc28\ub840 \ubc18\uaca9\uc744 \uac00\ud588\uc73c\ub098 \uacf5\uaca9\uc744 \ubc1b\uc740 \uac83\uc740 \ub77c\uc778\ud558\ub974\ud2b8\uc758 \uad70\ub2e8 \ubfd0(\ub77c\uc138\uc774\ub2c8\uc544\uc774 \uc804\ud22c), \ub9cc\uc288\ud0c0\uc778\uc758 \uad70\ub2e8\uc740 315 \ud0ac\ub85c\ubbf8\ud130\ub97c \ubd88\uacfc 100 \uc2dc\uac04\ub9cc\uc5d0 \ucf8c\uc9c4\uaca9\ud558\uba70 \ub4dc\ub2c8\ubc14 \uac15\uc5d0 \ub3c4\ub2ec\ud588\ub2e4. \uc9c0\ub098\uce58\uac8c \uc55e\uc11c\ub098\uac04 \ub9cc\uc288\ud0c0\uc778\uc758 \uad70\ub2e8\uc740 \ub2e4\ub978 \uc544\uad70\ub4e4\uacfc \ub5a8\uc5b4\uc838 \uc788\uc5c8\uace0, \uc0c1\ub2f9\ud55c \ud68c\uc218\uc758 \uc18c\ub828\uad70\uc758 \uc5ed\uc2b5\uc744 \ubc1b\uc558\uc73c\ub098 \ubaa8\ub450 \ubb3c\ub9ac\ucce4\ub2e4. \ub77c\uc778\ud558\ub974\ud2b8\uc758 \uad70\ub2e8\uc774 \ub4a4\ub530\ub77c \uc624\uc790 2\uac1c \uad70\ub2e8\uc740 \ub8e8\uac00\uc758 \uc18c\ub828\uad70 \uc9c4\ud615\uc744 \uc591\uba74\ud611\uacf5\ud558\uc5ec \ud3ec\uc704\ud558\uae30\ub85c \ud588\ub2e4.", "answer": "\ubc14\ub974\ubc14\ub85c\uc0ac \uc791\uc804", "sentence": "\uc554\ud638\uba85\uc740 \ubc14\ub974\ubc14\ub85c\uc0ac \uc791\uc804 .", "paragraph_sentence": "1941\ub144 \ucd08 \ub3c5\uc77c \ucd5c\uace0\uc0ac\ub839\ubd80\ub294 \uc18c\ub828 \uce68\uacf5 \uacc4\ud68d\uc744 \uc2dc\uc791\ud588\ub2e4. \uc554\ud638\uba85\uc740 \ubc14\ub974\ubc14\ub85c\uc0ac \uc791\uc804 . 5\uc6d4 15\uc77c \ub9cc\uc288\ud0c0\uc778\uc740 \uc81c56\uae30\uac11\uad70\ub2e8 \uad70\ub2e8\uc7a5\uc73c\ub85c \uc784\uba85\ub418\uc5c8\ub2e4. \uadf8\ub294 5\uc6d4\uc5d0 \ud5a5\ud6c4 \ub300\uacf5\uc138\uac00 \uc2dc\uc791\ub41c\ub2e4\ub294 \uc0c1\uc138\ud55c \uacc4\ud68d\uc5d0 \ub300\ud55c \ube0c\ub9ac\ud551\uc744 \ubc1b\uc740 250\uba85 \uc7a5\uc131 \uc911 \ud55c \uba85\uc774\uc5c8\ub2e4. \ub9cc\uc288\ud0c0\uc778\uc758 56\uad70\ub2e8\uc740 \uc5d0\ub9ac\ud788 \ud68c\ud504\ub108 \ub300\uc7a5\uc758 \uc81c4\uae30\uac11\uad70\uc5d0 \ubc30\uc18d\ub418\uc5c8\uace0, \uc81c4\uae30\uac11\uad70\uc740 \ube4c\ud5ec\ub984 \ub9ac\ud130 \ud3f0 \ub808\ud504 \uc6d0\uc218\uc758 \ubd81\ubd80 \uc9d1\ub2e8\uad70\uc5d0 \ubc30\uc18d\ub418\uc5c8\ub2e4. \ubd81\ubd80 \uc9d1\ub2e8\uad70\uc758 \uc784\ubb34\ub294 \ubc1c\ud2b8 3\uad6d\uc744 \uac70\uccd0 \ub808\ub2cc\uadf8\ub77c\ub4dc\ub85c \uc9c4\uaca9\ud558\ub294 \uac83\uc774\uc5c8\ub2e4. \ub9cc\uc288\ud0c0\uc778\uc740 \uacf5\uc138 \uc2dc\uc791 \ubd88\uacfc 6\uc77c \uc804\uc5d0\uc57c \uc804\uc120\uc5d0 \ub3c4\ucc29\ud588\ub2e4. 1941\ub144 6\uc6d4 22\uc77c \ubc14\ub974\ubc14\ub85c\uc0ac \uc791\uc804\uc774 \uac1c\uc2dc\ub418\uc5b4 \ub3d9\ubd80\uc804\uc120 \uc804\uccb4\uc5d0\uc11c \ub300\uaddc\ubaa8\uc758 \ub3c5\uc77c\uad70 \uacf5\uaca9\uc774 \uc2dc\uc791\ub418\uc5c8\ub2e4. \ub9cc\uc288\ud0c0\uc778\uc758 56\uad70\ub2e8\uc740 \uac8c\uc624\ub974\ud06c\ud55c\uc2a4 \ud558\uc778\ud558\ub974\ud2b8\uc758 \uc81c41\uae30\uac11\uad70\ub2e8\uacfc \ud568\uaed8 \ub2e4\uc6b0\uac00\ud504\ud544\uc2a4 \uadfc\uad50\uc758 \uad50\ub7c9\ub4e4\uc744 \ud655\ubcf4\ud558\uae30 \uc704\ud574 \ub4dc\ub2c8\ubc14 \uac15 \ucabd\uc73c\ub85c \uc9c4\uaca9\ud588\ub2e4. \uc18c\ub828\uad70\uc774 \uc5ec\ub7ec \ucc28\ub840 \ubc18\uaca9\uc744 \uac00\ud588\uc73c\ub098 \uacf5\uaca9\uc744 \ubc1b\uc740 \uac83\uc740 \ub77c\uc778\ud558\ub974\ud2b8\uc758 \uad70\ub2e8 \ubfd0(\ub77c\uc138\uc774\ub2c8\uc544\uc774 \uc804\ud22c), \ub9cc\uc288\ud0c0\uc778\uc758 \uad70\ub2e8\uc740 315 \ud0ac\ub85c\ubbf8\ud130\ub97c \ubd88\uacfc 100 \uc2dc\uac04\ub9cc\uc5d0 \ucf8c\uc9c4\uaca9\ud558\uba70 \ub4dc\ub2c8\ubc14 \uac15\uc5d0 \ub3c4\ub2ec\ud588\ub2e4. \uc9c0\ub098\uce58\uac8c \uc55e\uc11c\ub098\uac04 \ub9cc\uc288\ud0c0\uc778\uc758 \uad70\ub2e8\uc740 \ub2e4\ub978 \uc544\uad70\ub4e4\uacfc \ub5a8\uc5b4\uc838 \uc788\uc5c8\uace0, \uc0c1\ub2f9\ud55c \ud68c\uc218\uc758 \uc18c\ub828\uad70\uc758 \uc5ed\uc2b5\uc744 \ubc1b\uc558\uc73c\ub098 \ubaa8\ub450 \ubb3c\ub9ac\ucce4\ub2e4. \ub77c\uc778\ud558\ub974\ud2b8\uc758 \uad70\ub2e8\uc774 \ub4a4\ub530\ub77c \uc624\uc790 2\uac1c \uad70\ub2e8\uc740 \ub8e8\uac00\uc758 \uc18c\ub828\uad70 \uc9c4\ud615\uc744 \uc591\uba74\ud611\uacf5\ud558\uc5ec \ud3ec\uc704\ud558\uae30\ub85c \ud588\ub2e4.", "paragraph_answer": "1941\ub144 \ucd08 \ub3c5\uc77c \ucd5c\uace0\uc0ac\ub839\ubd80\ub294 \uc18c\ub828 \uce68\uacf5 \uacc4\ud68d\uc744 \uc2dc\uc791\ud588\ub2e4. \uc554\ud638\uba85\uc740 \ubc14\ub974\ubc14\ub85c\uc0ac \uc791\uc804 . 5\uc6d4 15\uc77c \ub9cc\uc288\ud0c0\uc778\uc740 \uc81c56\uae30\uac11\uad70\ub2e8 \uad70\ub2e8\uc7a5\uc73c\ub85c \uc784\uba85\ub418\uc5c8\ub2e4. \uadf8\ub294 5\uc6d4\uc5d0 \ud5a5\ud6c4 \ub300\uacf5\uc138\uac00 \uc2dc\uc791\ub41c\ub2e4\ub294 \uc0c1\uc138\ud55c \uacc4\ud68d\uc5d0 \ub300\ud55c \ube0c\ub9ac\ud551\uc744 \ubc1b\uc740 250\uba85 \uc7a5\uc131 \uc911 \ud55c \uba85\uc774\uc5c8\ub2e4. \ub9cc\uc288\ud0c0\uc778\uc758 56\uad70\ub2e8\uc740 \uc5d0\ub9ac\ud788 \ud68c\ud504\ub108 \ub300\uc7a5\uc758 \uc81c4\uae30\uac11\uad70\uc5d0 \ubc30\uc18d\ub418\uc5c8\uace0, \uc81c4\uae30\uac11\uad70\uc740 \ube4c\ud5ec\ub984 \ub9ac\ud130 \ud3f0 \ub808\ud504 \uc6d0\uc218\uc758 \ubd81\ubd80 \uc9d1\ub2e8\uad70\uc5d0 \ubc30\uc18d\ub418\uc5c8\ub2e4. \ubd81\ubd80 \uc9d1\ub2e8\uad70\uc758 \uc784\ubb34\ub294 \ubc1c\ud2b8 3\uad6d\uc744 \uac70\uccd0 \ub808\ub2cc\uadf8\ub77c\ub4dc\ub85c \uc9c4\uaca9\ud558\ub294 \uac83\uc774\uc5c8\ub2e4. \ub9cc\uc288\ud0c0\uc778\uc740 \uacf5\uc138 \uc2dc\uc791 \ubd88\uacfc 6\uc77c \uc804\uc5d0\uc57c \uc804\uc120\uc5d0 \ub3c4\ucc29\ud588\ub2e4. 1941\ub144 6\uc6d4 22\uc77c \ubc14\ub974\ubc14\ub85c\uc0ac \uc791\uc804\uc774 \uac1c\uc2dc\ub418\uc5b4 \ub3d9\ubd80\uc804\uc120 \uc804\uccb4\uc5d0\uc11c \ub300\uaddc\ubaa8\uc758 \ub3c5\uc77c\uad70 \uacf5\uaca9\uc774 \uc2dc\uc791\ub418\uc5c8\ub2e4. \ub9cc\uc288\ud0c0\uc778\uc758 56\uad70\ub2e8\uc740 \uac8c\uc624\ub974\ud06c\ud55c\uc2a4 \ud558\uc778\ud558\ub974\ud2b8\uc758 \uc81c41\uae30\uac11\uad70\ub2e8\uacfc \ud568\uaed8 \ub2e4\uc6b0\uac00\ud504\ud544\uc2a4 \uadfc\uad50\uc758 \uad50\ub7c9\ub4e4\uc744 \ud655\ubcf4\ud558\uae30 \uc704\ud574 \ub4dc\ub2c8\ubc14 \uac15 \ucabd\uc73c\ub85c \uc9c4\uaca9\ud588\ub2e4. \uc18c\ub828\uad70\uc774 \uc5ec\ub7ec \ucc28\ub840 \ubc18\uaca9\uc744 \uac00\ud588\uc73c\ub098 \uacf5\uaca9\uc744 \ubc1b\uc740 \uac83\uc740 \ub77c\uc778\ud558\ub974\ud2b8\uc758 \uad70\ub2e8 \ubfd0(\ub77c\uc138\uc774\ub2c8\uc544\uc774 \uc804\ud22c), \ub9cc\uc288\ud0c0\uc778\uc758 \uad70\ub2e8\uc740 315 \ud0ac\ub85c\ubbf8\ud130\ub97c \ubd88\uacfc 100 \uc2dc\uac04\ub9cc\uc5d0 \ucf8c\uc9c4\uaca9\ud558\uba70 \ub4dc\ub2c8\ubc14 \uac15\uc5d0 \ub3c4\ub2ec\ud588\ub2e4. \uc9c0\ub098\uce58\uac8c \uc55e\uc11c\ub098\uac04 \ub9cc\uc288\ud0c0\uc778\uc758 \uad70\ub2e8\uc740 \ub2e4\ub978 \uc544\uad70\ub4e4\uacfc \ub5a8\uc5b4\uc838 \uc788\uc5c8\uace0, \uc0c1\ub2f9\ud55c \ud68c\uc218\uc758 \uc18c\ub828\uad70\uc758 \uc5ed\uc2b5\uc744 \ubc1b\uc558\uc73c\ub098 \ubaa8\ub450 \ubb3c\ub9ac\ucce4\ub2e4. \ub77c\uc778\ud558\ub974\ud2b8\uc758 \uad70\ub2e8\uc774 \ub4a4\ub530\ub77c \uc624\uc790 2\uac1c \uad70\ub2e8\uc740 \ub8e8\uac00\uc758 \uc18c\ub828\uad70 \uc9c4\ud615\uc744 \uc591\uba74\ud611\uacf5\ud558\uc5ec \ud3ec\uc704\ud558\uae30\ub85c \ud588\ub2e4.", "sentence_answer": "\uc554\ud638\uba85\uc740 \ubc14\ub974\ubc14\ub85c\uc0ac \uc791\uc804 .", "paragraph_id": "6586570-14-0", "paragraph_question": "question: 1941\ub144 \ub3c5\uc77c\uc758 \uc18c\ub828 \uce68\uacf5 \uacc4\ud68d\uc758 \uc554\ud638\uba85\uc740?, context: 1941\ub144 \ucd08 \ub3c5\uc77c \ucd5c\uace0\uc0ac\ub839\ubd80\ub294 \uc18c\ub828 \uce68\uacf5 \uacc4\ud68d\uc744 \uc2dc\uc791\ud588\ub2e4. \uc554\ud638\uba85\uc740 \ubc14\ub974\ubc14\ub85c\uc0ac \uc791\uc804. 5\uc6d4 15\uc77c \ub9cc\uc288\ud0c0\uc778\uc740 \uc81c56\uae30\uac11\uad70\ub2e8 \uad70\ub2e8\uc7a5\uc73c\ub85c \uc784\uba85\ub418\uc5c8\ub2e4. \uadf8\ub294 5\uc6d4\uc5d0 \ud5a5\ud6c4 \ub300\uacf5\uc138\uac00 \uc2dc\uc791\ub41c\ub2e4\ub294 \uc0c1\uc138\ud55c \uacc4\ud68d\uc5d0 \ub300\ud55c \ube0c\ub9ac\ud551\uc744 \ubc1b\uc740 250\uba85 \uc7a5\uc131 \uc911 \ud55c \uba85\uc774\uc5c8\ub2e4. \ub9cc\uc288\ud0c0\uc778\uc758 56\uad70\ub2e8\uc740 \uc5d0\ub9ac\ud788 \ud68c\ud504\ub108 \ub300\uc7a5\uc758 \uc81c4\uae30\uac11\uad70\uc5d0 \ubc30\uc18d\ub418\uc5c8\uace0, \uc81c4\uae30\uac11\uad70\uc740 \ube4c\ud5ec\ub984 \ub9ac\ud130 \ud3f0 \ub808\ud504 \uc6d0\uc218\uc758 \ubd81\ubd80 \uc9d1\ub2e8\uad70\uc5d0 \ubc30\uc18d\ub418\uc5c8\ub2e4. \ubd81\ubd80 \uc9d1\ub2e8\uad70\uc758 \uc784\ubb34\ub294 \ubc1c\ud2b8 3\uad6d\uc744 \uac70\uccd0 \ub808\ub2cc\uadf8\ub77c\ub4dc\ub85c \uc9c4\uaca9\ud558\ub294 \uac83\uc774\uc5c8\ub2e4. \ub9cc\uc288\ud0c0\uc778\uc740 \uacf5\uc138 \uc2dc\uc791 \ubd88\uacfc 6\uc77c \uc804\uc5d0\uc57c \uc804\uc120\uc5d0 \ub3c4\ucc29\ud588\ub2e4. 1941\ub144 6\uc6d4 22\uc77c \ubc14\ub974\ubc14\ub85c\uc0ac \uc791\uc804\uc774 \uac1c\uc2dc\ub418\uc5b4 \ub3d9\ubd80\uc804\uc120 \uc804\uccb4\uc5d0\uc11c \ub300\uaddc\ubaa8\uc758 \ub3c5\uc77c\uad70 \uacf5\uaca9\uc774 \uc2dc\uc791\ub418\uc5c8\ub2e4. \ub9cc\uc288\ud0c0\uc778\uc758 56\uad70\ub2e8\uc740 \uac8c\uc624\ub974\ud06c\ud55c\uc2a4 \ud558\uc778\ud558\ub974\ud2b8\uc758 \uc81c41\uae30\uac11\uad70\ub2e8\uacfc \ud568\uaed8 \ub2e4\uc6b0\uac00\ud504\ud544\uc2a4 \uadfc\uad50\uc758 \uad50\ub7c9\ub4e4\uc744 \ud655\ubcf4\ud558\uae30 \uc704\ud574 \ub4dc\ub2c8\ubc14 \uac15 \ucabd\uc73c\ub85c \uc9c4\uaca9\ud588\ub2e4. \uc18c\ub828\uad70\uc774 \uc5ec\ub7ec \ucc28\ub840 \ubc18\uaca9\uc744 \uac00\ud588\uc73c\ub098 \uacf5\uaca9\uc744 \ubc1b\uc740 \uac83\uc740 \ub77c\uc778\ud558\ub974\ud2b8\uc758 \uad70\ub2e8 \ubfd0(\ub77c\uc138\uc774\ub2c8\uc544\uc774 \uc804\ud22c), \ub9cc\uc288\ud0c0\uc778\uc758 \uad70\ub2e8\uc740 315 \ud0ac\ub85c\ubbf8\ud130\ub97c \ubd88\uacfc 100 \uc2dc\uac04\ub9cc\uc5d0 \ucf8c\uc9c4\uaca9\ud558\uba70 \ub4dc\ub2c8\ubc14 \uac15\uc5d0 \ub3c4\ub2ec\ud588\ub2e4. \uc9c0\ub098\uce58\uac8c \uc55e\uc11c\ub098\uac04 \ub9cc\uc288\ud0c0\uc778\uc758 \uad70\ub2e8\uc740 \ub2e4\ub978 \uc544\uad70\ub4e4\uacfc \ub5a8\uc5b4\uc838 \uc788\uc5c8\uace0, \uc0c1\ub2f9\ud55c \ud68c\uc218\uc758 \uc18c\ub828\uad70\uc758 \uc5ed\uc2b5\uc744 \ubc1b\uc558\uc73c\ub098 \ubaa8\ub450 \ubb3c\ub9ac\ucce4\ub2e4. \ub77c\uc778\ud558\ub974\ud2b8\uc758 \uad70\ub2e8\uc774 \ub4a4\ub530\ub77c \uc624\uc790 2\uac1c \uad70\ub2e8\uc740 \ub8e8\uac00\uc758 \uc18c\ub828\uad70 \uc9c4\ud615\uc744 \uc591\uba74\ud611\uacf5\ud558\uc5ec \ud3ec\uc704\ud558\uae30\ub85c \ud588\ub2e4."} {"question": "\ubc15\uc815\ud76c\uac00 \uc77c\ubcf8 \uc721\uad70\uc0ac\uad00\ud559\uad50\uc5d0 \uc785\ud559\ud55c \uae30\uc218\ub294?", "paragraph": "\ub300\uad6c\uc0ac\ubc94\ud559\uad50\ub97c \uc878\uc5c5\ud558\uace0 3\ub144\uac04 \uad50\uc0ac\ub85c \uc7ac\uc9c1\ud558\ub2e4 \ub9cc\uc8fc\uad6d \uc721\uad70\uad70\uad00\ud559\uad50\uc5d0 \uc785\ud559\ud558\uc600\ub2e4. \uc878\uc5c5 \uc131\uc801 \uc11d\ucc28 2\ub4f1\uc73c\ub85c \ub9cc\uc8fc\uad6d \uad70\uad00\ud559\uad50\ub97c \uc878\uc5c5\ud55c \ud6c4, \uc131\uc801\uc6b0\uc218\uc790 \ucd94\ucc9c\uc744 \ubc1b\uc544, \uc77c\ubcf8 \uc721\uad70\uc0ac\uad00\ud559\uad50\uc5d0 57\uae30\ub85c \uc785\ud559\ud55c \ud6c4 1944\ub144 \uc131\uc801 \uc11d\ucc28 1\ub4f1\uc73c\ub85c \uc878\uc5c5\ud588\ub2e4. \uc77c\ubcf8\uc774 \uc81c2\ucc28 \uc138\uacc4 \ub300\uc804\uc5d0\uc11c \ud328\ub9dd\ud560 \ub54c\uae4c\uc9c0 \uc77c\ubcf8 \uc81c\uad6d\uc774 \uc218\ub9bd\ud55c \ub9cc\uc8fc\uad6d\uc758 \uc77c\uc81c\uad00\ub3d9\uad70\uc7a5\uad50\ub85c \uadfc\ubb34\ud558\uc600\ub2e4. \uc77c\uc81c\uac00 \ud328\ub9dd\ud558\uace0 1946\ub144 7\uc6d4\uc5d0 \uadc0\uad6d\ud558\uc600\ub2e4. \uadc0\uad6d \uc774\ud6c4 \ub300\ud55c\ubbfc\uad6d \uad6d\uad70 \uc7a5\uad50\ub85c \uc9c0\ub0b4\ub358\uc911, \ud615\uc778 \ubc15\uc0c1\ud76c\uac00 \uacbd\ucc30\uc5d0\uac8c \uc0ac\uc0b4\ub2f9\ud558\uc790 \uc774\uc7ac\ubcf5\uc758 \uad8c\uc720\ub85c \ubcf5\uc218\uc2ec\uc5d0 \ub0a8\uc870\uc120\ub85c\ub3d9\ub2f9\uc5d0 \uc785\ub2f9\ud558\uc5ec \ud65c\ub3d9\ud558\uc600\ub2e4\uac00 \uae40\ucc3d\ub8e1\uc774 \uc8fc\ub3c4\ud55c \uc219\uad70\uc5d0\uc11c \uc5ec\uc218\u00b7\uc21c\ucc9c \uc0ac\uac74 \uc5f0\ub8e8 \ud610\uc758\ub85c \uccb4\ud3ec\ub418\uc5b4 \uc0ac\ud615\uc744 \uc120\uace0\ubc1b\uc558\ub2e4. \uc815\ubcf4\uad6d\uc5d0 \ub0a8\uc870\uc120\ub85c\ub3d9\ub2f9 \uc870\uc9c1\uacfc \ub3d9\ub8cc\ub4e4\uc744 \uc99d\uc5b8\ud55c \ud6c4, \uc721\uad70\ubcf8\ubd80 \uc815\ubcf4\uad6d\uc7a5\uc774\uc5c8\ub358 \ubc31\uc120\uc5fd\uc758 \ucd5c\uc885 \uba74\ub2f4\uc5d0\uc11c \uc0ac\ud615\uc744 \uba74\ud558\uc600\ub2e4. \ud55c\uad6d \uc804\uc7c1\uc774 \ub098\uc790 \ub300\ud55c\ubbfc\uad6d \uad6d\uad70 \uc7a5\uad50\ub85c \ucc38\uc804\ud558\uc600\ub2e4.", "answer": "57\uae30", "sentence": "\uc878\uc5c5 \uc131\uc801 \uc11d\ucc28 2\ub4f1\uc73c\ub85c \ub9cc\uc8fc\uad6d \uad70\uad00\ud559\uad50\ub97c \uc878\uc5c5\ud55c \ud6c4, \uc131\uc801\uc6b0\uc218\uc790 \ucd94\ucc9c\uc744 \ubc1b\uc544, \uc77c\ubcf8 \uc721\uad70\uc0ac\uad00\ud559\uad50\uc5d0 57\uae30 \ub85c \uc785\ud559\ud55c \ud6c4 1944\ub144 \uc131\uc801 \uc11d\ucc28 1\ub4f1\uc73c\ub85c \uc878\uc5c5\ud588\ub2e4.", "paragraph_sentence": "\ub300\uad6c\uc0ac\ubc94\ud559\uad50\ub97c \uc878\uc5c5\ud558\uace0 3\ub144\uac04 \uad50\uc0ac\ub85c \uc7ac\uc9c1\ud558\ub2e4 \ub9cc\uc8fc\uad6d \uc721\uad70\uad70\uad00\ud559\uad50\uc5d0 \uc785\ud559\ud558\uc600\ub2e4. \uc878\uc5c5 \uc131\uc801 \uc11d\ucc28 2\ub4f1\uc73c\ub85c \ub9cc\uc8fc\uad6d \uad70\uad00\ud559\uad50\ub97c \uc878\uc5c5\ud55c \ud6c4, \uc131\uc801\uc6b0\uc218\uc790 \ucd94\ucc9c\uc744 \ubc1b\uc544, \uc77c\ubcf8 \uc721\uad70\uc0ac\uad00\ud559\uad50\uc5d0 57\uae30 \ub85c \uc785\ud559\ud55c \ud6c4 1944\ub144 \uc131\uc801 \uc11d\ucc28 1\ub4f1\uc73c\ub85c \uc878\uc5c5\ud588\ub2e4. \uc77c\ubcf8\uc774 \uc81c2\ucc28 \uc138\uacc4 \ub300\uc804\uc5d0\uc11c \ud328\ub9dd\ud560 \ub54c\uae4c\uc9c0 \uc77c\ubcf8 \uc81c\uad6d\uc774 \uc218\ub9bd\ud55c \ub9cc\uc8fc\uad6d\uc758 \uc77c\uc81c\uad00\ub3d9\uad70\uc7a5\uad50\ub85c \uadfc\ubb34\ud558\uc600\ub2e4. \uc77c\uc81c\uac00 \ud328\ub9dd\ud558\uace0 1946\ub144 7\uc6d4\uc5d0 \uadc0\uad6d\ud558\uc600\ub2e4. \uadc0\uad6d \uc774\ud6c4 \ub300\ud55c\ubbfc\uad6d \uad6d\uad70 \uc7a5\uad50\ub85c \uc9c0\ub0b4\ub358\uc911, \ud615\uc778 \ubc15\uc0c1\ud76c\uac00 \uacbd\ucc30\uc5d0\uac8c \uc0ac\uc0b4\ub2f9\ud558\uc790 \uc774\uc7ac\ubcf5\uc758 \uad8c\uc720\ub85c \ubcf5\uc218\uc2ec\uc5d0 \ub0a8\uc870\uc120\ub85c\ub3d9\ub2f9\uc5d0 \uc785\ub2f9\ud558\uc5ec \ud65c\ub3d9\ud558\uc600\ub2e4\uac00 \uae40\ucc3d\ub8e1\uc774 \uc8fc\ub3c4\ud55c \uc219\uad70\uc5d0\uc11c \uc5ec\uc218\u00b7\uc21c\ucc9c \uc0ac\uac74 \uc5f0\ub8e8 \ud610\uc758\ub85c \uccb4\ud3ec\ub418\uc5b4 \uc0ac\ud615\uc744 \uc120\uace0\ubc1b\uc558\ub2e4. \uc815\ubcf4\uad6d\uc5d0 \ub0a8\uc870\uc120\ub85c\ub3d9\ub2f9 \uc870\uc9c1\uacfc \ub3d9\ub8cc\ub4e4\uc744 \uc99d\uc5b8\ud55c \ud6c4, \uc721\uad70\ubcf8\ubd80 \uc815\ubcf4\uad6d\uc7a5\uc774\uc5c8\ub358 \ubc31\uc120\uc5fd\uc758 \ucd5c\uc885 \uba74\ub2f4\uc5d0\uc11c \uc0ac\ud615\uc744 \uba74\ud558\uc600\ub2e4. \ud55c\uad6d \uc804\uc7c1\uc774 \ub098\uc790 \ub300\ud55c\ubbfc\uad6d \uad6d\uad70 \uc7a5\uad50\ub85c \ucc38\uc804\ud558\uc600\ub2e4.", "paragraph_answer": "\ub300\uad6c\uc0ac\ubc94\ud559\uad50\ub97c \uc878\uc5c5\ud558\uace0 3\ub144\uac04 \uad50\uc0ac\ub85c \uc7ac\uc9c1\ud558\ub2e4 \ub9cc\uc8fc\uad6d \uc721\uad70\uad70\uad00\ud559\uad50\uc5d0 \uc785\ud559\ud558\uc600\ub2e4. \uc878\uc5c5 \uc131\uc801 \uc11d\ucc28 2\ub4f1\uc73c\ub85c \ub9cc\uc8fc\uad6d \uad70\uad00\ud559\uad50\ub97c \uc878\uc5c5\ud55c \ud6c4, \uc131\uc801\uc6b0\uc218\uc790 \ucd94\ucc9c\uc744 \ubc1b\uc544, \uc77c\ubcf8 \uc721\uad70\uc0ac\uad00\ud559\uad50\uc5d0 57\uae30 \ub85c \uc785\ud559\ud55c \ud6c4 1944\ub144 \uc131\uc801 \uc11d\ucc28 1\ub4f1\uc73c\ub85c \uc878\uc5c5\ud588\ub2e4. \uc77c\ubcf8\uc774 \uc81c2\ucc28 \uc138\uacc4 \ub300\uc804\uc5d0\uc11c \ud328\ub9dd\ud560 \ub54c\uae4c\uc9c0 \uc77c\ubcf8 \uc81c\uad6d\uc774 \uc218\ub9bd\ud55c \ub9cc\uc8fc\uad6d\uc758 \uc77c\uc81c\uad00\ub3d9\uad70\uc7a5\uad50\ub85c \uadfc\ubb34\ud558\uc600\ub2e4. \uc77c\uc81c\uac00 \ud328\ub9dd\ud558\uace0 1946\ub144 7\uc6d4\uc5d0 \uadc0\uad6d\ud558\uc600\ub2e4. \uadc0\uad6d \uc774\ud6c4 \ub300\ud55c\ubbfc\uad6d \uad6d\uad70 \uc7a5\uad50\ub85c \uc9c0\ub0b4\ub358\uc911, \ud615\uc778 \ubc15\uc0c1\ud76c\uac00 \uacbd\ucc30\uc5d0\uac8c \uc0ac\uc0b4\ub2f9\ud558\uc790 \uc774\uc7ac\ubcf5\uc758 \uad8c\uc720\ub85c \ubcf5\uc218\uc2ec\uc5d0 \ub0a8\uc870\uc120\ub85c\ub3d9\ub2f9\uc5d0 \uc785\ub2f9\ud558\uc5ec \ud65c\ub3d9\ud558\uc600\ub2e4\uac00 \uae40\ucc3d\ub8e1\uc774 \uc8fc\ub3c4\ud55c \uc219\uad70\uc5d0\uc11c \uc5ec\uc218\u00b7\uc21c\ucc9c \uc0ac\uac74 \uc5f0\ub8e8 \ud610\uc758\ub85c \uccb4\ud3ec\ub418\uc5b4 \uc0ac\ud615\uc744 \uc120\uace0\ubc1b\uc558\ub2e4. \uc815\ubcf4\uad6d\uc5d0 \ub0a8\uc870\uc120\ub85c\ub3d9\ub2f9 \uc870\uc9c1\uacfc \ub3d9\ub8cc\ub4e4\uc744 \uc99d\uc5b8\ud55c \ud6c4, \uc721\uad70\ubcf8\ubd80 \uc815\ubcf4\uad6d\uc7a5\uc774\uc5c8\ub358 \ubc31\uc120\uc5fd\uc758 \ucd5c\uc885 \uba74\ub2f4\uc5d0\uc11c \uc0ac\ud615\uc744 \uba74\ud558\uc600\ub2e4. \ud55c\uad6d \uc804\uc7c1\uc774 \ub098\uc790 \ub300\ud55c\ubbfc\uad6d \uad6d\uad70 \uc7a5\uad50\ub85c \ucc38\uc804\ud558\uc600\ub2e4.", "sentence_answer": "\uc878\uc5c5 \uc131\uc801 \uc11d\ucc28 2\ub4f1\uc73c\ub85c \ub9cc\uc8fc\uad6d \uad70\uad00\ud559\uad50\ub97c \uc878\uc5c5\ud55c \ud6c4, \uc131\uc801\uc6b0\uc218\uc790 \ucd94\ucc9c\uc744 \ubc1b\uc544, \uc77c\ubcf8 \uc721\uad70\uc0ac\uad00\ud559\uad50\uc5d0 57\uae30 \ub85c \uc785\ud559\ud55c \ud6c4 1944\ub144 \uc131\uc801 \uc11d\ucc28 1\ub4f1\uc73c\ub85c \uc878\uc5c5\ud588\ub2e4.", "paragraph_id": "6530241-0-0", "paragraph_question": "question: \ubc15\uc815\ud76c\uac00 \uc77c\ubcf8 \uc721\uad70\uc0ac\uad00\ud559\uad50\uc5d0 \uc785\ud559\ud55c \uae30\uc218\ub294?, context: \ub300\uad6c\uc0ac\ubc94\ud559\uad50\ub97c \uc878\uc5c5\ud558\uace0 3\ub144\uac04 \uad50\uc0ac\ub85c \uc7ac\uc9c1\ud558\ub2e4 \ub9cc\uc8fc\uad6d \uc721\uad70\uad70\uad00\ud559\uad50\uc5d0 \uc785\ud559\ud558\uc600\ub2e4. \uc878\uc5c5 \uc131\uc801 \uc11d\ucc28 2\ub4f1\uc73c\ub85c \ub9cc\uc8fc\uad6d \uad70\uad00\ud559\uad50\ub97c \uc878\uc5c5\ud55c \ud6c4, \uc131\uc801\uc6b0\uc218\uc790 \ucd94\ucc9c\uc744 \ubc1b\uc544, \uc77c\ubcf8 \uc721\uad70\uc0ac\uad00\ud559\uad50\uc5d0 57\uae30\ub85c \uc785\ud559\ud55c \ud6c4 1944\ub144 \uc131\uc801 \uc11d\ucc28 1\ub4f1\uc73c\ub85c \uc878\uc5c5\ud588\ub2e4. \uc77c\ubcf8\uc774 \uc81c2\ucc28 \uc138\uacc4 \ub300\uc804\uc5d0\uc11c \ud328\ub9dd\ud560 \ub54c\uae4c\uc9c0 \uc77c\ubcf8 \uc81c\uad6d\uc774 \uc218\ub9bd\ud55c \ub9cc\uc8fc\uad6d\uc758 \uc77c\uc81c\uad00\ub3d9\uad70\uc7a5\uad50\ub85c \uadfc\ubb34\ud558\uc600\ub2e4. \uc77c\uc81c\uac00 \ud328\ub9dd\ud558\uace0 1946\ub144 7\uc6d4\uc5d0 \uadc0\uad6d\ud558\uc600\ub2e4. \uadc0\uad6d \uc774\ud6c4 \ub300\ud55c\ubbfc\uad6d \uad6d\uad70 \uc7a5\uad50\ub85c \uc9c0\ub0b4\ub358\uc911, \ud615\uc778 \ubc15\uc0c1\ud76c\uac00 \uacbd\ucc30\uc5d0\uac8c \uc0ac\uc0b4\ub2f9\ud558\uc790 \uc774\uc7ac\ubcf5\uc758 \uad8c\uc720\ub85c \ubcf5\uc218\uc2ec\uc5d0 \ub0a8\uc870\uc120\ub85c\ub3d9\ub2f9\uc5d0 \uc785\ub2f9\ud558\uc5ec \ud65c\ub3d9\ud558\uc600\ub2e4\uac00 \uae40\ucc3d\ub8e1\uc774 \uc8fc\ub3c4\ud55c \uc219\uad70\uc5d0\uc11c \uc5ec\uc218\u00b7\uc21c\ucc9c \uc0ac\uac74 \uc5f0\ub8e8 \ud610\uc758\ub85c \uccb4\ud3ec\ub418\uc5b4 \uc0ac\ud615\uc744 \uc120\uace0\ubc1b\uc558\ub2e4. \uc815\ubcf4\uad6d\uc5d0 \ub0a8\uc870\uc120\ub85c\ub3d9\ub2f9 \uc870\uc9c1\uacfc \ub3d9\ub8cc\ub4e4\uc744 \uc99d\uc5b8\ud55c \ud6c4, \uc721\uad70\ubcf8\ubd80 \uc815\ubcf4\uad6d\uc7a5\uc774\uc5c8\ub358 \ubc31\uc120\uc5fd\uc758 \ucd5c\uc885 \uba74\ub2f4\uc5d0\uc11c \uc0ac\ud615\uc744 \uba74\ud558\uc600\ub2e4. \ud55c\uad6d \uc804\uc7c1\uc774 \ub098\uc790 \ub300\ud55c\ubbfc\uad6d \uad6d\uad70 \uc7a5\uad50\ub85c \ucc38\uc804\ud558\uc600\ub2e4."} {"question": "\uadf8\uac00 2015\ub144 \uc2dc\uc98c\uc5d0\uc11c \ucd9c\uc804\ud574\uc11c \uc5bb\uc740 \ub4dd\uc810\uc740 \ucd1d \uba87\uc810\uc778\uac00?", "paragraph": "2015\ub144 \uc2dc\uc98c\uc5d0 \uc774\ud638\uc900\uc758 \ub4a4\ub97c \uc774\uc5b4 NC \ub2e4\uc774\ub178\uc2a4\uc758 \uc0c8 \uc8fc\uc7a5\uc774 \ub418\uc5c8\ub2e4. \uadf8\ub294 \u201c\uc774\ud638\uc900 \uc120\ubc30\uc758 \ub4a4\ub97c \uc787\ub294\ub2e4\ub294 \uc810\uc5d0\uc11c \ubd80\ub2f4\uc774 \ud06c\uc9c0\ub9cc \uc88b\uc740 \ud2c0\uc744 \ub9cc\ub4e4\uc5b4\ub193\uc740 \ub9cc\ud07c \uc804\ud1b5\uc744 \uc798 \uc787\ub294 \uc8fc\uc7a5\uc774 \ub418\uaca0\ub2e4\u201d\uba70 \u201c\ucc45\uc784\uac10\uc744 \uac00\uc9c0\uace0 \uc5f4\uc2ec\ud788 \ud558\uaca0\ub2e4\u201d\uace0 \uc18c\uac10\uc744 \ubc1d\ud614\ub2e4. 4\uc6d4 1\uc77c \ub125\uc13c \ud788\uc5b4\ub85c\uc988\uc640\uc758 \uacbd\uae30\uc5d0\uc11c \ud53c\uc5b4\ubc34\ub4dc\ub97c \uc0c1\ub300\ub85c 4\ud68c 2\uc0ac\uc5d0\uc11c \uc88c\uc804 \uc548\ud0c0\ub97c \ucce4\ub2e4. \uc774 \uc548\ud0c0\ub85c \uc5ed\ub300 63\ubc88\uc9f8 \ud1b5\uc0b0 1,100\uc548\ud0c0\ub97c \uae30\ub85d\ud588\ub2e4. 4\uc6d4 15\uc77c \ub86f\ub370 \uc790\uc774\uc5b8\uce20\uc640\uc758 \uacbd\uae30\uc5d0\uc11c 2\ub8e8 \ub3c4\ub8e8\uc5d0 \uc131\uacf5\ud558\uba74\uc11c \uc5ed\ub300 9\ubc88\uc9f8 \ud1b5\uc0b0 300\ub3c4\ub8e8\ub97c \uae30\ub85d\ud588\ub2e4. 6\uc6d4 18\uc77c kt \uc704\uc988\uc640\uc758 \uacbd\uae30\uc5d0\uc11c \uc5ed\ub300 88\ubc88\uc9f8 \ud1b5\uc0b0 1100\uacbd\uae30 \ucd9c\uc7a5\uc744 \uae30\ub85d\ud588\ub2e4. 6\uc6d4 21\uc77c \ud55c\ud654 \uc774\uae00\uc2a4\uc640\uc758 \uacbd\uae30\uc5d0\uc11c 2\ub8e8 \ub3c4\ub8e8\ub97c \uc131\uacf5\ud558\uba74\uc11c \ud1b5\uc0b0 9\ubc88\uc9f8 10\ub144 \uc5f0\uc18d \ub450 \uc790\ub9bf\uc218 \ub3c4\ub8e8\ub97c \ub2ec\uc131\ud588\ub2e4. \uc804\ubc18\uae30 81\uacbd\uae30 293\ud0c0\uc218 79\uc548\ud0c0 \ud0c0\uc728 0.270, 45\ub4dd\uc810, 2\ud648\ub7f0, 31\ud0c0\uc810, 14\ub3c4\ub8e8, 37\ubcfc\ub137, 48\ud0c8\uc0bc\uc9c4\uc744 \uae30\ub85d\ud588\ub2e4. 8\uc6d4 22\uc77c SK \uc640\uc774\ubc88\uc2a4\uc640\uc758 \uacbd\uae30\uc5d0\uc11c \uc2e0\uc7ac\uc6c5\uc744 \uc0c1\ub300\ub85c \uc88c\uc775\uc218 \ud0a4\ub97c \ub118\uae30\ub294 2\ub8e8\ud0c0\ub97c \uce58\uba74\uc11c KBO \ub9ac\uadf8 \ud1b5\uc0b0 52\ubc88\uc9f8 1200\uc548\ud0c0\ub97c \uae30\ub85d\ud588\ub2e4. 2015 \uc2dc\uc98c 2\ud560\ub300 \ud0c0\uc728, 125\uacbd\uae30, 440\ud0c0\uc218, 118\uc548\ud0c0(5\ud648\ub7f0), 63\ub4dd\uc810, 52\ud0c0\uc810, 17\ub3c4\ub8e8\ub97c \uae30\ub85d\ud588\ub2e4. \ud50c\ub808\uc774\uc624\ud504 1, 2\ucc28\uc804\uc5d0\uc11c 3\ubc88\ud0c0\uc21c\uc5d0 \uae30\uc6a9\ub410\uc73c\ub098 \ucd9c\ub8e8\uc870\ucc28 \ud558\uc9c0 \ubabb\ud558\uace0 \ubd80\uc9c4\ud588\ub2e4. 3\ucc28\uc804\uc5d0\uc11c\ub294 5\ud0c0\uc218 2\uc548\ud0c0 2\ud0c0\uc810\uc73c\ub85c \ud300\uc758 \uc2b9\ub9ac\ub97c \uc774\ub04c\uc5c8\uace0, 4\ucc28\uc804\uc5d0\uc11c\ub3c4 \uc548\ud0c0\ub97c 1\uac1c \ucce4\ub2e4. \uadf8\ub7ec\ub098 5\ucc28\uc804\uc5d0\uc11c \uc548\ud0c0\ub97c \uce58\uc9c0 \ubabb\ud558\uba74\uc11c \ud50c\ub808\uc774\uc624\ud504 \ucd5c\uc885 \uc131\uc801 18\ud0c0\uc218 3\uc548\ud0c0 \ud0c0\uc728 0.166 2\ud0c0\uc810\uc744 \uae30\ub85d\ud588\ub2e4.", "answer": "63\ub4dd\uc810", "sentence": "2015 \uc2dc\uc98c 2\ud560\ub300 \ud0c0\uc728, 125\uacbd\uae30, 440\ud0c0\uc218, 118\uc548\ud0c0(5\ud648\ub7f0), 63\ub4dd\uc810 ,", "paragraph_sentence": "2015\ub144 \uc2dc\uc98c\uc5d0 \uc774\ud638\uc900\uc758 \ub4a4\ub97c \uc774\uc5b4 NC \ub2e4\uc774\ub178\uc2a4\uc758 \uc0c8 \uc8fc\uc7a5\uc774 \ub418\uc5c8\ub2e4. \uadf8\ub294 \u201c\uc774\ud638\uc900 \uc120\ubc30\uc758 \ub4a4\ub97c \uc787\ub294\ub2e4\ub294 \uc810\uc5d0\uc11c \ubd80\ub2f4\uc774 \ud06c\uc9c0\ub9cc \uc88b\uc740 \ud2c0\uc744 \ub9cc\ub4e4\uc5b4\ub193\uc740 \ub9cc\ud07c \uc804\ud1b5\uc744 \uc798 \uc787\ub294 \uc8fc\uc7a5\uc774 \ub418\uaca0\ub2e4\u201d\uba70 \u201c\ucc45\uc784\uac10\uc744 \uac00\uc9c0\uace0 \uc5f4\uc2ec\ud788 \ud558\uaca0\ub2e4\u201d\uace0 \uc18c\uac10\uc744 \ubc1d\ud614\ub2e4. 4\uc6d4 1\uc77c \ub125\uc13c \ud788\uc5b4\ub85c\uc988\uc640\uc758 \uacbd\uae30\uc5d0\uc11c \ud53c\uc5b4\ubc34\ub4dc\ub97c \uc0c1\ub300\ub85c 4\ud68c 2\uc0ac\uc5d0\uc11c \uc88c\uc804 \uc548\ud0c0\ub97c \ucce4\ub2e4. \uc774 \uc548\ud0c0\ub85c \uc5ed\ub300 63\ubc88\uc9f8 \ud1b5\uc0b0 1,100\uc548\ud0c0\ub97c \uae30\ub85d\ud588\ub2e4. 4\uc6d4 15\uc77c \ub86f\ub370 \uc790\uc774\uc5b8\uce20\uc640\uc758 \uacbd\uae30\uc5d0\uc11c 2\ub8e8 \ub3c4\ub8e8\uc5d0 \uc131\uacf5\ud558\uba74\uc11c \uc5ed\ub300 9\ubc88\uc9f8 \ud1b5\uc0b0 300\ub3c4\ub8e8\ub97c \uae30\ub85d\ud588\ub2e4. 6\uc6d4 18\uc77c kt \uc704\uc988\uc640\uc758 \uacbd\uae30\uc5d0\uc11c \uc5ed\ub300 88\ubc88\uc9f8 \ud1b5\uc0b0 1100\uacbd\uae30 \ucd9c\uc7a5\uc744 \uae30\ub85d\ud588\ub2e4. 6\uc6d4 21\uc77c \ud55c\ud654 \uc774\uae00\uc2a4\uc640\uc758 \uacbd\uae30\uc5d0\uc11c 2\ub8e8 \ub3c4\ub8e8\ub97c \uc131\uacf5\ud558\uba74\uc11c \ud1b5\uc0b0 9\ubc88\uc9f8 10\ub144 \uc5f0\uc18d \ub450 \uc790\ub9bf\uc218 \ub3c4\ub8e8\ub97c \ub2ec\uc131\ud588\ub2e4. \uc804\ubc18\uae30 81\uacbd\uae30 293\ud0c0\uc218 79\uc548\ud0c0 \ud0c0\uc728 0.270, 45\ub4dd\uc810, 2\ud648\ub7f0, 31\ud0c0\uc810, 14\ub3c4\ub8e8, 37\ubcfc\ub137, 48\ud0c8\uc0bc\uc9c4\uc744 \uae30\ub85d\ud588\ub2e4. 8\uc6d4 22\uc77c SK \uc640\uc774\ubc88\uc2a4\uc640\uc758 \uacbd\uae30\uc5d0\uc11c \uc2e0\uc7ac\uc6c5\uc744 \uc0c1\ub300\ub85c \uc88c\uc775\uc218 \ud0a4\ub97c \ub118\uae30\ub294 2\ub8e8\ud0c0\ub97c \uce58\uba74\uc11c KBO \ub9ac\uadf8 \ud1b5\uc0b0 52\ubc88\uc9f8 1200\uc548\ud0c0\ub97c \uae30\ub85d\ud588\ub2e4. 2015 \uc2dc\uc98c 2\ud560\ub300 \ud0c0\uc728, 125\uacbd\uae30, 440\ud0c0\uc218, 118\uc548\ud0c0(5\ud648\ub7f0), 63\ub4dd\uc810 , 52\ud0c0\uc810, 17\ub3c4\ub8e8\ub97c \uae30\ub85d\ud588\ub2e4. \ud50c\ub808\uc774\uc624\ud504 1, 2\ucc28\uc804\uc5d0\uc11c 3\ubc88\ud0c0\uc21c\uc5d0 \uae30\uc6a9\ub410\uc73c\ub098 \ucd9c\ub8e8\uc870\ucc28 \ud558\uc9c0 \ubabb\ud558\uace0 \ubd80\uc9c4\ud588\ub2e4. 3\ucc28\uc804\uc5d0\uc11c\ub294 5\ud0c0\uc218 2\uc548\ud0c0 2\ud0c0\uc810\uc73c\ub85c \ud300\uc758 \uc2b9\ub9ac\ub97c \uc774\ub04c\uc5c8\uace0, 4\ucc28\uc804\uc5d0\uc11c\ub3c4 \uc548\ud0c0\ub97c 1\uac1c \ucce4\ub2e4. \uadf8\ub7ec\ub098 5\ucc28\uc804\uc5d0\uc11c \uc548\ud0c0\ub97c \uce58\uc9c0 \ubabb\ud558\uba74\uc11c \ud50c\ub808\uc774\uc624\ud504 \ucd5c\uc885 \uc131\uc801 18\ud0c0\uc218 3\uc548\ud0c0 \ud0c0\uc728 0.166 2\ud0c0\uc810\uc744 \uae30\ub85d\ud588\ub2e4.", "paragraph_answer": "2015\ub144 \uc2dc\uc98c\uc5d0 \uc774\ud638\uc900\uc758 \ub4a4\ub97c \uc774\uc5b4 NC \ub2e4\uc774\ub178\uc2a4\uc758 \uc0c8 \uc8fc\uc7a5\uc774 \ub418\uc5c8\ub2e4. \uadf8\ub294 \u201c\uc774\ud638\uc900 \uc120\ubc30\uc758 \ub4a4\ub97c \uc787\ub294\ub2e4\ub294 \uc810\uc5d0\uc11c \ubd80\ub2f4\uc774 \ud06c\uc9c0\ub9cc \uc88b\uc740 \ud2c0\uc744 \ub9cc\ub4e4\uc5b4\ub193\uc740 \ub9cc\ud07c \uc804\ud1b5\uc744 \uc798 \uc787\ub294 \uc8fc\uc7a5\uc774 \ub418\uaca0\ub2e4\u201d\uba70 \u201c\ucc45\uc784\uac10\uc744 \uac00\uc9c0\uace0 \uc5f4\uc2ec\ud788 \ud558\uaca0\ub2e4\u201d\uace0 \uc18c\uac10\uc744 \ubc1d\ud614\ub2e4. 4\uc6d4 1\uc77c \ub125\uc13c \ud788\uc5b4\ub85c\uc988\uc640\uc758 \uacbd\uae30\uc5d0\uc11c \ud53c\uc5b4\ubc34\ub4dc\ub97c \uc0c1\ub300\ub85c 4\ud68c 2\uc0ac\uc5d0\uc11c \uc88c\uc804 \uc548\ud0c0\ub97c \ucce4\ub2e4. \uc774 \uc548\ud0c0\ub85c \uc5ed\ub300 63\ubc88\uc9f8 \ud1b5\uc0b0 1,100\uc548\ud0c0\ub97c \uae30\ub85d\ud588\ub2e4. 4\uc6d4 15\uc77c \ub86f\ub370 \uc790\uc774\uc5b8\uce20\uc640\uc758 \uacbd\uae30\uc5d0\uc11c 2\ub8e8 \ub3c4\ub8e8\uc5d0 \uc131\uacf5\ud558\uba74\uc11c \uc5ed\ub300 9\ubc88\uc9f8 \ud1b5\uc0b0 300\ub3c4\ub8e8\ub97c \uae30\ub85d\ud588\ub2e4. 6\uc6d4 18\uc77c kt \uc704\uc988\uc640\uc758 \uacbd\uae30\uc5d0\uc11c \uc5ed\ub300 88\ubc88\uc9f8 \ud1b5\uc0b0 1100\uacbd\uae30 \ucd9c\uc7a5\uc744 \uae30\ub85d\ud588\ub2e4. 6\uc6d4 21\uc77c \ud55c\ud654 \uc774\uae00\uc2a4\uc640\uc758 \uacbd\uae30\uc5d0\uc11c 2\ub8e8 \ub3c4\ub8e8\ub97c \uc131\uacf5\ud558\uba74\uc11c \ud1b5\uc0b0 9\ubc88\uc9f8 10\ub144 \uc5f0\uc18d \ub450 \uc790\ub9bf\uc218 \ub3c4\ub8e8\ub97c \ub2ec\uc131\ud588\ub2e4. \uc804\ubc18\uae30 81\uacbd\uae30 293\ud0c0\uc218 79\uc548\ud0c0 \ud0c0\uc728 0.270, 45\ub4dd\uc810, 2\ud648\ub7f0, 31\ud0c0\uc810, 14\ub3c4\ub8e8, 37\ubcfc\ub137, 48\ud0c8\uc0bc\uc9c4\uc744 \uae30\ub85d\ud588\ub2e4. 8\uc6d4 22\uc77c SK \uc640\uc774\ubc88\uc2a4\uc640\uc758 \uacbd\uae30\uc5d0\uc11c \uc2e0\uc7ac\uc6c5\uc744 \uc0c1\ub300\ub85c \uc88c\uc775\uc218 \ud0a4\ub97c \ub118\uae30\ub294 2\ub8e8\ud0c0\ub97c \uce58\uba74\uc11c KBO \ub9ac\uadf8 \ud1b5\uc0b0 52\ubc88\uc9f8 1200\uc548\ud0c0\ub97c \uae30\ub85d\ud588\ub2e4. 2015 \uc2dc\uc98c 2\ud560\ub300 \ud0c0\uc728, 125\uacbd\uae30, 440\ud0c0\uc218, 118\uc548\ud0c0(5\ud648\ub7f0), 63\ub4dd\uc810 , 52\ud0c0\uc810, 17\ub3c4\ub8e8\ub97c \uae30\ub85d\ud588\ub2e4. \ud50c\ub808\uc774\uc624\ud504 1, 2\ucc28\uc804\uc5d0\uc11c 3\ubc88\ud0c0\uc21c\uc5d0 \uae30\uc6a9\ub410\uc73c\ub098 \ucd9c\ub8e8\uc870\ucc28 \ud558\uc9c0 \ubabb\ud558\uace0 \ubd80\uc9c4\ud588\ub2e4. 3\ucc28\uc804\uc5d0\uc11c\ub294 5\ud0c0\uc218 2\uc548\ud0c0 2\ud0c0\uc810\uc73c\ub85c \ud300\uc758 \uc2b9\ub9ac\ub97c \uc774\ub04c\uc5c8\uace0, 4\ucc28\uc804\uc5d0\uc11c\ub3c4 \uc548\ud0c0\ub97c 1\uac1c \ucce4\ub2e4. \uadf8\ub7ec\ub098 5\ucc28\uc804\uc5d0\uc11c \uc548\ud0c0\ub97c \uce58\uc9c0 \ubabb\ud558\uba74\uc11c \ud50c\ub808\uc774\uc624\ud504 \ucd5c\uc885 \uc131\uc801 18\ud0c0\uc218 3\uc548\ud0c0 \ud0c0\uc728 0.166 2\ud0c0\uc810\uc744 \uae30\ub85d\ud588\ub2e4.", "sentence_answer": "2015 \uc2dc\uc98c 2\ud560\ub300 \ud0c0\uc728, 125\uacbd\uae30, 440\ud0c0\uc218, 118\uc548\ud0c0(5\ud648\ub7f0), 63\ub4dd\uc810 ,", "paragraph_id": "6547804-10-2", "paragraph_question": "question: \uadf8\uac00 2015\ub144 \uc2dc\uc98c\uc5d0\uc11c \ucd9c\uc804\ud574\uc11c \uc5bb\uc740 \ub4dd\uc810\uc740 \ucd1d \uba87\uc810\uc778\uac00?, context: 2015\ub144 \uc2dc\uc98c\uc5d0 \uc774\ud638\uc900\uc758 \ub4a4\ub97c \uc774\uc5b4 NC \ub2e4\uc774\ub178\uc2a4\uc758 \uc0c8 \uc8fc\uc7a5\uc774 \ub418\uc5c8\ub2e4. \uadf8\ub294 \u201c\uc774\ud638\uc900 \uc120\ubc30\uc758 \ub4a4\ub97c \uc787\ub294\ub2e4\ub294 \uc810\uc5d0\uc11c \ubd80\ub2f4\uc774 \ud06c\uc9c0\ub9cc \uc88b\uc740 \ud2c0\uc744 \ub9cc\ub4e4\uc5b4\ub193\uc740 \ub9cc\ud07c \uc804\ud1b5\uc744 \uc798 \uc787\ub294 \uc8fc\uc7a5\uc774 \ub418\uaca0\ub2e4\u201d\uba70 \u201c\ucc45\uc784\uac10\uc744 \uac00\uc9c0\uace0 \uc5f4\uc2ec\ud788 \ud558\uaca0\ub2e4\u201d\uace0 \uc18c\uac10\uc744 \ubc1d\ud614\ub2e4. 4\uc6d4 1\uc77c \ub125\uc13c \ud788\uc5b4\ub85c\uc988\uc640\uc758 \uacbd\uae30\uc5d0\uc11c \ud53c\uc5b4\ubc34\ub4dc\ub97c \uc0c1\ub300\ub85c 4\ud68c 2\uc0ac\uc5d0\uc11c \uc88c\uc804 \uc548\ud0c0\ub97c \ucce4\ub2e4. \uc774 \uc548\ud0c0\ub85c \uc5ed\ub300 63\ubc88\uc9f8 \ud1b5\uc0b0 1,100\uc548\ud0c0\ub97c \uae30\ub85d\ud588\ub2e4. 4\uc6d4 15\uc77c \ub86f\ub370 \uc790\uc774\uc5b8\uce20\uc640\uc758 \uacbd\uae30\uc5d0\uc11c 2\ub8e8 \ub3c4\ub8e8\uc5d0 \uc131\uacf5\ud558\uba74\uc11c \uc5ed\ub300 9\ubc88\uc9f8 \ud1b5\uc0b0 300\ub3c4\ub8e8\ub97c \uae30\ub85d\ud588\ub2e4. 6\uc6d4 18\uc77c kt \uc704\uc988\uc640\uc758 \uacbd\uae30\uc5d0\uc11c \uc5ed\ub300 88\ubc88\uc9f8 \ud1b5\uc0b0 1100\uacbd\uae30 \ucd9c\uc7a5\uc744 \uae30\ub85d\ud588\ub2e4. 6\uc6d4 21\uc77c \ud55c\ud654 \uc774\uae00\uc2a4\uc640\uc758 \uacbd\uae30\uc5d0\uc11c 2\ub8e8 \ub3c4\ub8e8\ub97c \uc131\uacf5\ud558\uba74\uc11c \ud1b5\uc0b0 9\ubc88\uc9f8 10\ub144 \uc5f0\uc18d \ub450 \uc790\ub9bf\uc218 \ub3c4\ub8e8\ub97c \ub2ec\uc131\ud588\ub2e4. \uc804\ubc18\uae30 81\uacbd\uae30 293\ud0c0\uc218 79\uc548\ud0c0 \ud0c0\uc728 0.270, 45\ub4dd\uc810, 2\ud648\ub7f0, 31\ud0c0\uc810, 14\ub3c4\ub8e8, 37\ubcfc\ub137, 48\ud0c8\uc0bc\uc9c4\uc744 \uae30\ub85d\ud588\ub2e4. 8\uc6d4 22\uc77c SK \uc640\uc774\ubc88\uc2a4\uc640\uc758 \uacbd\uae30\uc5d0\uc11c \uc2e0\uc7ac\uc6c5\uc744 \uc0c1\ub300\ub85c \uc88c\uc775\uc218 \ud0a4\ub97c \ub118\uae30\ub294 2\ub8e8\ud0c0\ub97c \uce58\uba74\uc11c KBO \ub9ac\uadf8 \ud1b5\uc0b0 52\ubc88\uc9f8 1200\uc548\ud0c0\ub97c \uae30\ub85d\ud588\ub2e4. 2015 \uc2dc\uc98c 2\ud560\ub300 \ud0c0\uc728, 125\uacbd\uae30, 440\ud0c0\uc218, 118\uc548\ud0c0(5\ud648\ub7f0), 63\ub4dd\uc810, 52\ud0c0\uc810, 17\ub3c4\ub8e8\ub97c \uae30\ub85d\ud588\ub2e4. \ud50c\ub808\uc774\uc624\ud504 1, 2\ucc28\uc804\uc5d0\uc11c 3\ubc88\ud0c0\uc21c\uc5d0 \uae30\uc6a9\ub410\uc73c\ub098 \ucd9c\ub8e8\uc870\ucc28 \ud558\uc9c0 \ubabb\ud558\uace0 \ubd80\uc9c4\ud588\ub2e4. 3\ucc28\uc804\uc5d0\uc11c\ub294 5\ud0c0\uc218 2\uc548\ud0c0 2\ud0c0\uc810\uc73c\ub85c \ud300\uc758 \uc2b9\ub9ac\ub97c \uc774\ub04c\uc5c8\uace0, 4\ucc28\uc804\uc5d0\uc11c\ub3c4 \uc548\ud0c0\ub97c 1\uac1c \ucce4\ub2e4. \uadf8\ub7ec\ub098 5\ucc28\uc804\uc5d0\uc11c \uc548\ud0c0\ub97c \uce58\uc9c0 \ubabb\ud558\uba74\uc11c \ud50c\ub808\uc774\uc624\ud504 \ucd5c\uc885 \uc131\uc801 18\ud0c0\uc218 3\uc548\ud0c0 \ud0c0\uc728 0.166 2\ud0c0\uc810\uc744 \uae30\ub85d\ud588\ub2e4."} {"question": "\uc774\uc7ac\uc624\ub97c \uad6d\uad70\ubcf4\uc548\uc0ac\ub839\ubd80\uc758 \uc0ac\ucc30 \ub300\uc0c1\uc5d0 \ud3ec\ud568\uc2dc\ucf1c \uac10\uc2dc\ud588\ub358 \uc815\ubd80\uc758 \ub300\ud1b5\ub839 \uc774\ub984\uc740?", "paragraph": "\ubbfc\uc8fc\uc8fc\uc758\uc640 \ud1b5\uc77c\uc744 \uc2e4\ud604\ud558\uae30 \uc704\ud55c \ub178\ub825\uc740 \ub3c5\uc7ac\uc815\uad8c \uc544\ub798\uc11c \ub2e4\uc12f\ubc88\uc758 \uc218\ud615 \uc0dd\ud65c\ub85c \uc774\uc5b4\uc84c\ub2e4. \ubbfc\uc8fc\ud1b5\uc77c\ubbfc\uc911\uc6b4\ub3d9\uc5f0\ud569 \ubbfc\uc871\ud1b5\uc77c\uc704\uc6d0\uc7a5\uc73c\ub85c \ud65c\ub3d9\ud588\uace0, 6\uc6d4 \ud56d\uc7c1 \ub2f9\uc2dc\uc5d0\ub294 \ubbfc\uc8fc\ud5cc\ubc95\uc7c1\ucde8\uad6d\ubbfc\uc6b4\ub3d9\ubcf8\ubd80 \uc0c1\uc784 \uc9d1\ud589\uc704\uc6d0\uc73c\ub85c \ud56d\uc7c1\uc744 \uc8fc\ub3c4\ud588\uc73c\uba70, \uad70\uc815\uc885\uc2dd \ud6c4\ubcf4\ub2e8\uc77c\ud654 \uc7c1\ucde8 \uad6d\ubbfc\ud611\uc758\ud68c(\uad6d\ud611)\uc744 \uacb0\uc131\ud558\uace0 \ud6c4\ubcf4\ub2e8\uc77c\ud654 \uc6b4\ub3d9\uc5d0 \ub098\uc130\ub2e4. \uadf8\ub294 \uc774\ud6c4 1989\ub144 \uc804\uad6d\ubbfc\uc871\ubbfc\uc8fc\uc6b4\ub3d9\uc5f0\ud569 \uc870\uad6d\ud1b5\uc77c\uc704\uc6d0\uc7a5\uc744 \uac70\uce5c \ud6c4 \uc9c4\ubcf4\uc815\ub2f9\uc744 \ucc3d\ub2f9\ud558\uae30 \uc704\ud574 \ubbfc\uc5f0\ucd94\ub97c \uacb0\uc131\ud558\ub098 \uc774\ubd80\uc601\uc744 \uc9c0\uc9c0\ud558\ub294 \uc138\ub825\uacfc \ubd84\uc5f4\ud558\uba70 1990\ub144 \uae40\ubb38\uc218, \uc7a5\uae30\ud45c \ub4f1\uacfc \ud568\uaed8 \ubbfc\uc911\ub2f9 \ucc3d\ub2f9\uc5d0 \ucc38\uc5ec\ud558\uc600\ub2e4. \uc5ec\uae30\uc5d0\ub294 \uae30\ub3c5\uad50\uc2dc\ubbfc\uc6b4\ub3d9\uacc4\uc5f4 \uc778\uc0ac\ub4e4\uc774 \ucc38\uc5ec\ud558\uc600\uace0 \ubb38\uc775\ud658 \ubaa9\uc0ac \ub4f1\ub3c4 \uc9c0\uc9c0\ub97c \ubcf4\ub0b4\uc8fc\uc5c8\ub2e4. \uc774\uc7ac\uc624\ub294 \uc774\uc5b4 \ubbfc\uc911\ub2f9 \uc0ac\ubb34\ucd1d\uc7a5\uc5d0 \uc120\ucd9c\ub418\uace0 1992\ub144 48\uc138\uc5d0 \uc81c14\ub300 \ucd1d\uc120\uc5d0\uc11c \uc11c\uc6b8 \uc740\ud3c9\uad6c \uc744\uc5d0 \ucd9c\ub9c8\ud588\uc73c\ub098 3\uc704\ub85c \ub099\uc120\ud558\uba70 \ud604\uc2e4\uc815\uce58\uc758 \ubcbd\uc744 \ub118\uc9c0 \ubabb\ud588\ub2e4. \ud55c\ud3b8 \uc774\uc7ac\uc624\ub294 \uad6d\uad70\ubcf4\uc548\uc0ac\ub839\ubd80\uc758 \uc0ac\ucc30 \ub300\uc0c1 \uc911 \ud55c \uc0ac\ub78c\uc774 \ub418\uc5b4 \ub178\ud0dc\uc6b0 \uc815\ubd80\ub85c\ubd80\ud130 \uac10\uc2dc\ub2f9\ud558\uae30\ub3c4 \ud588\ub2e4.", "answer": "\ub178\ud0dc\uc6b0", "sentence": "\ud55c\ud3b8 \uc774\uc7ac\uc624\ub294 \uad6d\uad70\ubcf4\uc548\uc0ac\ub839\ubd80\uc758 \uc0ac\ucc30 \ub300\uc0c1 \uc911 \ud55c \uc0ac\ub78c\uc774 \ub418\uc5b4 \ub178\ud0dc\uc6b0 \uc815\ubd80\ub85c\ubd80\ud130 \uac10\uc2dc\ub2f9\ud558\uae30\ub3c4 \ud588\ub2e4.", "paragraph_sentence": "\ubbfc\uc8fc\uc8fc\uc758\uc640 \ud1b5\uc77c\uc744 \uc2e4\ud604\ud558\uae30 \uc704\ud55c \ub178\ub825\uc740 \ub3c5\uc7ac\uc815\uad8c \uc544\ub798\uc11c \ub2e4\uc12f\ubc88\uc758 \uc218\ud615 \uc0dd\ud65c\ub85c \uc774\uc5b4\uc84c\ub2e4. \ubbfc\uc8fc\ud1b5\uc77c\ubbfc\uc911\uc6b4\ub3d9\uc5f0\ud569 \ubbfc\uc871\ud1b5\uc77c\uc704\uc6d0\uc7a5\uc73c\ub85c \ud65c\ub3d9\ud588\uace0, 6\uc6d4 \ud56d\uc7c1 \ub2f9\uc2dc\uc5d0\ub294 \ubbfc\uc8fc\ud5cc\ubc95\uc7c1\ucde8\uad6d\ubbfc\uc6b4\ub3d9\ubcf8\ubd80 \uc0c1\uc784 \uc9d1\ud589\uc704\uc6d0\uc73c\ub85c \ud56d\uc7c1\uc744 \uc8fc\ub3c4\ud588\uc73c\uba70, \uad70\uc815\uc885\uc2dd \ud6c4\ubcf4\ub2e8\uc77c\ud654 \uc7c1\ucde8 \uad6d\ubbfc\ud611\uc758\ud68c(\uad6d\ud611)\uc744 \uacb0\uc131\ud558\uace0 \ud6c4\ubcf4\ub2e8\uc77c\ud654 \uc6b4\ub3d9\uc5d0 \ub098\uc130\ub2e4. \uadf8\ub294 \uc774\ud6c4 1989\ub144 \uc804\uad6d\ubbfc\uc871\ubbfc\uc8fc\uc6b4\ub3d9\uc5f0\ud569 \uc870\uad6d\ud1b5\uc77c\uc704\uc6d0\uc7a5\uc744 \uac70\uce5c \ud6c4 \uc9c4\ubcf4\uc815\ub2f9\uc744 \ucc3d\ub2f9\ud558\uae30 \uc704\ud574 \ubbfc\uc5f0\ucd94\ub97c \uacb0\uc131\ud558\ub098 \uc774\ubd80\uc601\uc744 \uc9c0\uc9c0\ud558\ub294 \uc138\ub825\uacfc \ubd84\uc5f4\ud558\uba70 1990\ub144 \uae40\ubb38\uc218, \uc7a5\uae30\ud45c \ub4f1\uacfc \ud568\uaed8 \ubbfc\uc911\ub2f9 \ucc3d\ub2f9\uc5d0 \ucc38\uc5ec\ud558\uc600\ub2e4. \uc5ec\uae30\uc5d0\ub294 \uae30\ub3c5\uad50\uc2dc\ubbfc\uc6b4\ub3d9\uacc4\uc5f4 \uc778\uc0ac\ub4e4\uc774 \ucc38\uc5ec\ud558\uc600\uace0 \ubb38\uc775\ud658 \ubaa9\uc0ac \ub4f1\ub3c4 \uc9c0\uc9c0\ub97c \ubcf4\ub0b4\uc8fc\uc5c8\ub2e4. \uc774\uc7ac\uc624\ub294 \uc774\uc5b4 \ubbfc\uc911\ub2f9 \uc0ac\ubb34\ucd1d\uc7a5\uc5d0 \uc120\ucd9c\ub418\uace0 1992\ub144 48\uc138\uc5d0 \uc81c14\ub300 \ucd1d\uc120\uc5d0\uc11c \uc11c\uc6b8 \uc740\ud3c9\uad6c \uc744\uc5d0 \ucd9c\ub9c8\ud588\uc73c\ub098 3\uc704\ub85c \ub099\uc120\ud558\uba70 \ud604\uc2e4\uc815\uce58\uc758 \ubcbd\uc744 \ub118\uc9c0 \ubabb\ud588\ub2e4. \ud55c\ud3b8 \uc774\uc7ac\uc624\ub294 \uad6d\uad70\ubcf4\uc548\uc0ac\ub839\ubd80\uc758 \uc0ac\ucc30 \ub300\uc0c1 \uc911 \ud55c \uc0ac\ub78c\uc774 \ub418\uc5b4 \ub178\ud0dc\uc6b0 \uc815\ubd80\ub85c\ubd80\ud130 \uac10\uc2dc\ub2f9\ud558\uae30\ub3c4 \ud588\ub2e4. ", "paragraph_answer": "\ubbfc\uc8fc\uc8fc\uc758\uc640 \ud1b5\uc77c\uc744 \uc2e4\ud604\ud558\uae30 \uc704\ud55c \ub178\ub825\uc740 \ub3c5\uc7ac\uc815\uad8c \uc544\ub798\uc11c \ub2e4\uc12f\ubc88\uc758 \uc218\ud615 \uc0dd\ud65c\ub85c \uc774\uc5b4\uc84c\ub2e4. \ubbfc\uc8fc\ud1b5\uc77c\ubbfc\uc911\uc6b4\ub3d9\uc5f0\ud569 \ubbfc\uc871\ud1b5\uc77c\uc704\uc6d0\uc7a5\uc73c\ub85c \ud65c\ub3d9\ud588\uace0, 6\uc6d4 \ud56d\uc7c1 \ub2f9\uc2dc\uc5d0\ub294 \ubbfc\uc8fc\ud5cc\ubc95\uc7c1\ucde8\uad6d\ubbfc\uc6b4\ub3d9\ubcf8\ubd80 \uc0c1\uc784 \uc9d1\ud589\uc704\uc6d0\uc73c\ub85c \ud56d\uc7c1\uc744 \uc8fc\ub3c4\ud588\uc73c\uba70, \uad70\uc815\uc885\uc2dd \ud6c4\ubcf4\ub2e8\uc77c\ud654 \uc7c1\ucde8 \uad6d\ubbfc\ud611\uc758\ud68c(\uad6d\ud611)\uc744 \uacb0\uc131\ud558\uace0 \ud6c4\ubcf4\ub2e8\uc77c\ud654 \uc6b4\ub3d9\uc5d0 \ub098\uc130\ub2e4. \uadf8\ub294 \uc774\ud6c4 1989\ub144 \uc804\uad6d\ubbfc\uc871\ubbfc\uc8fc\uc6b4\ub3d9\uc5f0\ud569 \uc870\uad6d\ud1b5\uc77c\uc704\uc6d0\uc7a5\uc744 \uac70\uce5c \ud6c4 \uc9c4\ubcf4\uc815\ub2f9\uc744 \ucc3d\ub2f9\ud558\uae30 \uc704\ud574 \ubbfc\uc5f0\ucd94\ub97c \uacb0\uc131\ud558\ub098 \uc774\ubd80\uc601\uc744 \uc9c0\uc9c0\ud558\ub294 \uc138\ub825\uacfc \ubd84\uc5f4\ud558\uba70 1990\ub144 \uae40\ubb38\uc218, \uc7a5\uae30\ud45c \ub4f1\uacfc \ud568\uaed8 \ubbfc\uc911\ub2f9 \ucc3d\ub2f9\uc5d0 \ucc38\uc5ec\ud558\uc600\ub2e4. \uc5ec\uae30\uc5d0\ub294 \uae30\ub3c5\uad50\uc2dc\ubbfc\uc6b4\ub3d9\uacc4\uc5f4 \uc778\uc0ac\ub4e4\uc774 \ucc38\uc5ec\ud558\uc600\uace0 \ubb38\uc775\ud658 \ubaa9\uc0ac \ub4f1\ub3c4 \uc9c0\uc9c0\ub97c \ubcf4\ub0b4\uc8fc\uc5c8\ub2e4. \uc774\uc7ac\uc624\ub294 \uc774\uc5b4 \ubbfc\uc911\ub2f9 \uc0ac\ubb34\ucd1d\uc7a5\uc5d0 \uc120\ucd9c\ub418\uace0 1992\ub144 48\uc138\uc5d0 \uc81c14\ub300 \ucd1d\uc120\uc5d0\uc11c \uc11c\uc6b8 \uc740\ud3c9\uad6c \uc744\uc5d0 \ucd9c\ub9c8\ud588\uc73c\ub098 3\uc704\ub85c \ub099\uc120\ud558\uba70 \ud604\uc2e4\uc815\uce58\uc758 \ubcbd\uc744 \ub118\uc9c0 \ubabb\ud588\ub2e4. \ud55c\ud3b8 \uc774\uc7ac\uc624\ub294 \uad6d\uad70\ubcf4\uc548\uc0ac\ub839\ubd80\uc758 \uc0ac\ucc30 \ub300\uc0c1 \uc911 \ud55c \uc0ac\ub78c\uc774 \ub418\uc5b4 \ub178\ud0dc\uc6b0 \uc815\ubd80\ub85c\ubd80\ud130 \uac10\uc2dc\ub2f9\ud558\uae30\ub3c4 \ud588\ub2e4.", "sentence_answer": "\ud55c\ud3b8 \uc774\uc7ac\uc624\ub294 \uad6d\uad70\ubcf4\uc548\uc0ac\ub839\ubd80\uc758 \uc0ac\ucc30 \ub300\uc0c1 \uc911 \ud55c \uc0ac\ub78c\uc774 \ub418\uc5b4 \ub178\ud0dc\uc6b0 \uc815\ubd80\ub85c\ubd80\ud130 \uac10\uc2dc\ub2f9\ud558\uae30\ub3c4 \ud588\ub2e4.", "paragraph_id": "6547718-1-2", "paragraph_question": "question: \uc774\uc7ac\uc624\ub97c \uad6d\uad70\ubcf4\uc548\uc0ac\ub839\ubd80\uc758 \uc0ac\ucc30 \ub300\uc0c1\uc5d0 \ud3ec\ud568\uc2dc\ucf1c \uac10\uc2dc\ud588\ub358 \uc815\ubd80\uc758 \ub300\ud1b5\ub839 \uc774\ub984\uc740?, context: \ubbfc\uc8fc\uc8fc\uc758\uc640 \ud1b5\uc77c\uc744 \uc2e4\ud604\ud558\uae30 \uc704\ud55c \ub178\ub825\uc740 \ub3c5\uc7ac\uc815\uad8c \uc544\ub798\uc11c \ub2e4\uc12f\ubc88\uc758 \uc218\ud615 \uc0dd\ud65c\ub85c \uc774\uc5b4\uc84c\ub2e4. \ubbfc\uc8fc\ud1b5\uc77c\ubbfc\uc911\uc6b4\ub3d9\uc5f0\ud569 \ubbfc\uc871\ud1b5\uc77c\uc704\uc6d0\uc7a5\uc73c\ub85c \ud65c\ub3d9\ud588\uace0, 6\uc6d4 \ud56d\uc7c1 \ub2f9\uc2dc\uc5d0\ub294 \ubbfc\uc8fc\ud5cc\ubc95\uc7c1\ucde8\uad6d\ubbfc\uc6b4\ub3d9\ubcf8\ubd80 \uc0c1\uc784 \uc9d1\ud589\uc704\uc6d0\uc73c\ub85c \ud56d\uc7c1\uc744 \uc8fc\ub3c4\ud588\uc73c\uba70, \uad70\uc815\uc885\uc2dd \ud6c4\ubcf4\ub2e8\uc77c\ud654 \uc7c1\ucde8 \uad6d\ubbfc\ud611\uc758\ud68c(\uad6d\ud611)\uc744 \uacb0\uc131\ud558\uace0 \ud6c4\ubcf4\ub2e8\uc77c\ud654 \uc6b4\ub3d9\uc5d0 \ub098\uc130\ub2e4. \uadf8\ub294 \uc774\ud6c4 1989\ub144 \uc804\uad6d\ubbfc\uc871\ubbfc\uc8fc\uc6b4\ub3d9\uc5f0\ud569 \uc870\uad6d\ud1b5\uc77c\uc704\uc6d0\uc7a5\uc744 \uac70\uce5c \ud6c4 \uc9c4\ubcf4\uc815\ub2f9\uc744 \ucc3d\ub2f9\ud558\uae30 \uc704\ud574 \ubbfc\uc5f0\ucd94\ub97c \uacb0\uc131\ud558\ub098 \uc774\ubd80\uc601\uc744 \uc9c0\uc9c0\ud558\ub294 \uc138\ub825\uacfc \ubd84\uc5f4\ud558\uba70 1990\ub144 \uae40\ubb38\uc218, \uc7a5\uae30\ud45c \ub4f1\uacfc \ud568\uaed8 \ubbfc\uc911\ub2f9 \ucc3d\ub2f9\uc5d0 \ucc38\uc5ec\ud558\uc600\ub2e4. \uc5ec\uae30\uc5d0\ub294 \uae30\ub3c5\uad50\uc2dc\ubbfc\uc6b4\ub3d9\uacc4\uc5f4 \uc778\uc0ac\ub4e4\uc774 \ucc38\uc5ec\ud558\uc600\uace0 \ubb38\uc775\ud658 \ubaa9\uc0ac \ub4f1\ub3c4 \uc9c0\uc9c0\ub97c \ubcf4\ub0b4\uc8fc\uc5c8\ub2e4. \uc774\uc7ac\uc624\ub294 \uc774\uc5b4 \ubbfc\uc911\ub2f9 \uc0ac\ubb34\ucd1d\uc7a5\uc5d0 \uc120\ucd9c\ub418\uace0 1992\ub144 48\uc138\uc5d0 \uc81c14\ub300 \ucd1d\uc120\uc5d0\uc11c \uc11c\uc6b8 \uc740\ud3c9\uad6c \uc744\uc5d0 \ucd9c\ub9c8\ud588\uc73c\ub098 3\uc704\ub85c \ub099\uc120\ud558\uba70 \ud604\uc2e4\uc815\uce58\uc758 \ubcbd\uc744 \ub118\uc9c0 \ubabb\ud588\ub2e4. \ud55c\ud3b8 \uc774\uc7ac\uc624\ub294 \uad6d\uad70\ubcf4\uc548\uc0ac\ub839\ubd80\uc758 \uc0ac\ucc30 \ub300\uc0c1 \uc911 \ud55c \uc0ac\ub78c\uc774 \ub418\uc5b4 \ub178\ud0dc\uc6b0 \uc815\ubd80\ub85c\ubd80\ud130 \uac10\uc2dc\ub2f9\ud558\uae30\ub3c4 \ud588\ub2e4."} {"question": "\uc774\ub77c\ud06c\uc804 \ub54c \uc138\ubc88\uc9f8\ub85c \ub9ce\uc740 \ubcd1\ub825\uc744 \ubcf4\ub0b8 \ub098\ub77c\ub294?", "paragraph": "2004\ub144 12\uc6d4\uc5d0\ub294 \uc774\ub77c\ud06c \ub2e4\uad6d\uc801\uad70\uc0ac\ub839\ubd80 \uc120\uac70\uc9c0\uc6d0\uacfc\uc7a5\uc73c\ub85c \uadfc\ubb34\ud558\uba74\uc11c \uacf5\uc815\ud55c \uc120\uac70\uc5c5\ubb34\uc758 \uc8fc\uc5ed\uc73c\ub85c \uc678\uad6d\uad70(\ubbf8\uad6d, \uc601\uad6d, \ud638\uc8fc, \ub8e8\ub9c8\ub2c8\uc544)\uc744 \uc9c0\ud718\u00b7\uac10\ub3c5, \uc774\ub77c\ud06c \ubbfc\uc8fc\uc8fc\uc758 \ud0c4\uc0dd\uc5d0 \uae30\uc5ec\ud55c \uac83\uc73c\ub85c \ud3c9\uac00\ubc1b\uc558\ub2e4. \ud55c\uad6d\uc774 \uc774\ub77c\ud06c\uc804 \ub54c \uc138 \ubc88\uc9f8\ub85c \ub9ce\uc740 \ubcd1\ub825\uc744 \ubcf4\ub0c8\ub294\ub370 \ub2e4\uad6d\uc801\uad70\uc0ac\ub839\ubd80 \ucc38\ubaa8 \uc911\uc5d0 \ud55c\uad6d\uad70\uc774 \uadf8 \uc678\uc5d0\ub294 \uc5c6\uc5b4 \ucc98\uc74c\uc5d0\ub294 \uc9c1\uc18d \ubd80\ud558\ub4e4\uc774 \uc790\uc2e0\uc744 \ubb34\uc2dc\ud588\ub2e4\uace0 \ub290\ub080 \uadf8\uc600\ub2e4. \uadf8\ub798\uc11c \ubc14\uc058\ub2e4\ub294 \uc678\uad6d\uad70 \uc7a5\uad70\ub4e4 \ub300\uc2e0 \ube0c\ub9ac\ud551\uc744 \ud558\ub294 \ub4f1 \uc88b\uc740 \ud3c9\uac00\ub97c \ubc1b\uace0\uc790 \ub178\ub825\ud574 \uc810\ucc28 \uc870\uc9c1\uc744 \uc7a5\uc545\ud558\uc600\ub2e4\uace0 \ubc1d\ud614\ub2e4. \uadf8\ub294 \ubbf8\uad70\uacfc \uc601\uad6d\uad70, \ud638\uc8fc\uad70 \uc7a5\uad50 10\uc5ec \uba85\uc744 \uc9c0\ud718\ud574 5400\uac1c \uc120\uac70\uad6c\uc5d0\uc11c \uce58\ub7ec\uc9c0\ub294 \uc774\ub77c\ud06c \uc720\uad8c\uc790 1400\ub9cc \uba85\uc758 \ud22c\ud45c\ub97c \uc9c0\uc6d0 \ubc0f \uac10\ub3c5\ud558\uc600\uc73c\uba70, 3500t\uc5d0 \uc774\ub974\ub294 \uc120\uac70 \uad00\ub828 \ubb3c\uc790\ub97c \uac01 \ud22c\ud45c\uc18c\uc5d0 \uacf5\uae09\ud558\uace0 \uce58\uc548 \uc720\uc9c0\ub97c \uc9c0\uc6d0\ud558\ub294 \uc77c\ub3c4 \ub9e1\uc558\ub2e4. \uc120\uac70\uac00 \ubb34\uc0ac\ud788 \ub05d\ub09c \ub4a4 \uc774\ub77c\ud06c \ud3c9\ud654\uc815\ucc29\uc5d0 \uae30\uc5ec\ud55c \uacf5\ub85c\ub85c \ub300\ud55c\ubbfc\uad6d \uc815\ubd80\ub85c\ubd80\ud130 \ud654\ub791\ubb34\uacf5\ud6c8\uc7a5\uc744, \ud55c\uad6d \uad70\uc778\uc73c\ub85c\uc11c\ub294 \ucd5c\ucd08\ub85c \ubbf8\uad6d \uc815\ubd80\ub85c\ubd80\ud130 \ub3d9\uc131\ud6c8\uc7a5(Bronze Star Medal)\uc744 \ubc1b\uc558\ub2e4.", "answer": "\ud55c\uad6d", "sentence": "\ud55c\uad6d \uc774 \uc774\ub77c\ud06c\uc804 \ub54c \uc138 \ubc88\uc9f8\ub85c \ub9ce\uc740 \ubcd1\ub825\uc744 \ubcf4\ub0c8\ub294\ub370 \ub2e4\uad6d\uc801\uad70\uc0ac\ub839\ubd80 \ucc38\ubaa8 \uc911\uc5d0 \ud55c\uad6d\uad70\uc774 \uadf8 \uc678\uc5d0\ub294 \uc5c6\uc5b4 \ucc98\uc74c\uc5d0\ub294 \uc9c1\uc18d \ubd80\ud558\ub4e4\uc774 \uc790\uc2e0\uc744 \ubb34\uc2dc\ud588\ub2e4\uace0 \ub290\ub080 \uadf8\uc600\ub2e4.", "paragraph_sentence": "2004\ub144 12\uc6d4\uc5d0\ub294 \uc774\ub77c\ud06c \ub2e4\uad6d\uc801\uad70\uc0ac\ub839\ubd80 \uc120\uac70\uc9c0\uc6d0\uacfc\uc7a5\uc73c\ub85c \uadfc\ubb34\ud558\uba74\uc11c \uacf5\uc815\ud55c \uc120\uac70\uc5c5\ubb34\uc758 \uc8fc\uc5ed\uc73c\ub85c \uc678\uad6d\uad70(\ubbf8\uad6d, \uc601\uad6d, \ud638\uc8fc, \ub8e8\ub9c8\ub2c8\uc544)\uc744 \uc9c0\ud718\u00b7\uac10\ub3c5, \uc774\ub77c\ud06c \ubbfc\uc8fc\uc8fc\uc758 \ud0c4\uc0dd\uc5d0 \uae30\uc5ec\ud55c \uac83\uc73c\ub85c \ud3c9\uac00\ubc1b\uc558\ub2e4. \ud55c\uad6d \uc774 \uc774\ub77c\ud06c\uc804 \ub54c \uc138 \ubc88\uc9f8\ub85c \ub9ce\uc740 \ubcd1\ub825\uc744 \ubcf4\ub0c8\ub294\ub370 \ub2e4\uad6d\uc801\uad70\uc0ac\ub839\ubd80 \ucc38\ubaa8 \uc911\uc5d0 \ud55c\uad6d\uad70\uc774 \uadf8 \uc678\uc5d0\ub294 \uc5c6\uc5b4 \ucc98\uc74c\uc5d0\ub294 \uc9c1\uc18d \ubd80\ud558\ub4e4\uc774 \uc790\uc2e0\uc744 \ubb34\uc2dc\ud588\ub2e4\uace0 \ub290\ub080 \uadf8\uc600\ub2e4. \uadf8\ub798\uc11c \ubc14\uc058\ub2e4\ub294 \uc678\uad6d\uad70 \uc7a5\uad70\ub4e4 \ub300\uc2e0 \ube0c\ub9ac\ud551\uc744 \ud558\ub294 \ub4f1 \uc88b\uc740 \ud3c9\uac00\ub97c \ubc1b\uace0\uc790 \ub178\ub825\ud574 \uc810\ucc28 \uc870\uc9c1\uc744 \uc7a5\uc545\ud558\uc600\ub2e4\uace0 \ubc1d\ud614\ub2e4. \uadf8\ub294 \ubbf8\uad70\uacfc \uc601\uad6d\uad70, \ud638\uc8fc\uad70 \uc7a5\uad50 10\uc5ec \uba85\uc744 \uc9c0\ud718\ud574 5400\uac1c \uc120\uac70\uad6c\uc5d0\uc11c \uce58\ub7ec\uc9c0\ub294 \uc774\ub77c\ud06c \uc720\uad8c\uc790 1400\ub9cc \uba85\uc758 \ud22c\ud45c\ub97c \uc9c0\uc6d0 \ubc0f \uac10\ub3c5\ud558\uc600\uc73c\uba70, 3500t\uc5d0 \uc774\ub974\ub294 \uc120\uac70 \uad00\ub828 \ubb3c\uc790\ub97c \uac01 \ud22c\ud45c\uc18c\uc5d0 \uacf5\uae09\ud558\uace0 \uce58\uc548 \uc720\uc9c0\ub97c \uc9c0\uc6d0\ud558\ub294 \uc77c\ub3c4 \ub9e1\uc558\ub2e4. \uc120\uac70\uac00 \ubb34\uc0ac\ud788 \ub05d\ub09c \ub4a4 \uc774\ub77c\ud06c \ud3c9\ud654\uc815\ucc29\uc5d0 \uae30\uc5ec\ud55c \uacf5\ub85c\ub85c \ub300\ud55c\ubbfc\uad6d \uc815\ubd80\ub85c\ubd80\ud130 \ud654\ub791\ubb34\uacf5\ud6c8\uc7a5\uc744, \ud55c\uad6d \uad70\uc778\uc73c\ub85c\uc11c\ub294 \ucd5c\ucd08\ub85c \ubbf8\uad6d \uc815\ubd80\ub85c\ubd80\ud130 \ub3d9\uc131\ud6c8\uc7a5(Bronze Star Medal)\uc744 \ubc1b\uc558\ub2e4.", "paragraph_answer": "2004\ub144 12\uc6d4\uc5d0\ub294 \uc774\ub77c\ud06c \ub2e4\uad6d\uc801\uad70\uc0ac\ub839\ubd80 \uc120\uac70\uc9c0\uc6d0\uacfc\uc7a5\uc73c\ub85c \uadfc\ubb34\ud558\uba74\uc11c \uacf5\uc815\ud55c \uc120\uac70\uc5c5\ubb34\uc758 \uc8fc\uc5ed\uc73c\ub85c \uc678\uad6d\uad70(\ubbf8\uad6d, \uc601\uad6d, \ud638\uc8fc, \ub8e8\ub9c8\ub2c8\uc544)\uc744 \uc9c0\ud718\u00b7\uac10\ub3c5, \uc774\ub77c\ud06c \ubbfc\uc8fc\uc8fc\uc758 \ud0c4\uc0dd\uc5d0 \uae30\uc5ec\ud55c \uac83\uc73c\ub85c \ud3c9\uac00\ubc1b\uc558\ub2e4. \ud55c\uad6d \uc774 \uc774\ub77c\ud06c\uc804 \ub54c \uc138 \ubc88\uc9f8\ub85c \ub9ce\uc740 \ubcd1\ub825\uc744 \ubcf4\ub0c8\ub294\ub370 \ub2e4\uad6d\uc801\uad70\uc0ac\ub839\ubd80 \ucc38\ubaa8 \uc911\uc5d0 \ud55c\uad6d\uad70\uc774 \uadf8 \uc678\uc5d0\ub294 \uc5c6\uc5b4 \ucc98\uc74c\uc5d0\ub294 \uc9c1\uc18d \ubd80\ud558\ub4e4\uc774 \uc790\uc2e0\uc744 \ubb34\uc2dc\ud588\ub2e4\uace0 \ub290\ub080 \uadf8\uc600\ub2e4. \uadf8\ub798\uc11c \ubc14\uc058\ub2e4\ub294 \uc678\uad6d\uad70 \uc7a5\uad70\ub4e4 \ub300\uc2e0 \ube0c\ub9ac\ud551\uc744 \ud558\ub294 \ub4f1 \uc88b\uc740 \ud3c9\uac00\ub97c \ubc1b\uace0\uc790 \ub178\ub825\ud574 \uc810\ucc28 \uc870\uc9c1\uc744 \uc7a5\uc545\ud558\uc600\ub2e4\uace0 \ubc1d\ud614\ub2e4. \uadf8\ub294 \ubbf8\uad70\uacfc \uc601\uad6d\uad70, \ud638\uc8fc\uad70 \uc7a5\uad50 10\uc5ec \uba85\uc744 \uc9c0\ud718\ud574 5400\uac1c \uc120\uac70\uad6c\uc5d0\uc11c \uce58\ub7ec\uc9c0\ub294 \uc774\ub77c\ud06c \uc720\uad8c\uc790 1400\ub9cc \uba85\uc758 \ud22c\ud45c\ub97c \uc9c0\uc6d0 \ubc0f \uac10\ub3c5\ud558\uc600\uc73c\uba70, 3500t\uc5d0 \uc774\ub974\ub294 \uc120\uac70 \uad00\ub828 \ubb3c\uc790\ub97c \uac01 \ud22c\ud45c\uc18c\uc5d0 \uacf5\uae09\ud558\uace0 \uce58\uc548 \uc720\uc9c0\ub97c \uc9c0\uc6d0\ud558\ub294 \uc77c\ub3c4 \ub9e1\uc558\ub2e4. \uc120\uac70\uac00 \ubb34\uc0ac\ud788 \ub05d\ub09c \ub4a4 \uc774\ub77c\ud06c \ud3c9\ud654\uc815\ucc29\uc5d0 \uae30\uc5ec\ud55c \uacf5\ub85c\ub85c \ub300\ud55c\ubbfc\uad6d \uc815\ubd80\ub85c\ubd80\ud130 \ud654\ub791\ubb34\uacf5\ud6c8\uc7a5\uc744, \ud55c\uad6d \uad70\uc778\uc73c\ub85c\uc11c\ub294 \ucd5c\ucd08\ub85c \ubbf8\uad6d \uc815\ubd80\ub85c\ubd80\ud130 \ub3d9\uc131\ud6c8\uc7a5(Bronze Star Medal)\uc744 \ubc1b\uc558\ub2e4.", "sentence_answer": " \ud55c\uad6d \uc774 \uc774\ub77c\ud06c\uc804 \ub54c \uc138 \ubc88\uc9f8\ub85c \ub9ce\uc740 \ubcd1\ub825\uc744 \ubcf4\ub0c8\ub294\ub370 \ub2e4\uad6d\uc801\uad70\uc0ac\ub839\ubd80 \ucc38\ubaa8 \uc911\uc5d0 \ud55c\uad6d\uad70\uc774 \uadf8 \uc678\uc5d0\ub294 \uc5c6\uc5b4 \ucc98\uc74c\uc5d0\ub294 \uc9c1\uc18d \ubd80\ud558\ub4e4\uc774 \uc790\uc2e0\uc744 \ubb34\uc2dc\ud588\ub2e4\uace0 \ub290\ub080 \uadf8\uc600\ub2e4.", "paragraph_id": "6593261-3-1", "paragraph_question": "question: \uc774\ub77c\ud06c\uc804 \ub54c \uc138\ubc88\uc9f8\ub85c \ub9ce\uc740 \ubcd1\ub825\uc744 \ubcf4\ub0b8 \ub098\ub77c\ub294?, context: 2004\ub144 12\uc6d4\uc5d0\ub294 \uc774\ub77c\ud06c \ub2e4\uad6d\uc801\uad70\uc0ac\ub839\ubd80 \uc120\uac70\uc9c0\uc6d0\uacfc\uc7a5\uc73c\ub85c \uadfc\ubb34\ud558\uba74\uc11c \uacf5\uc815\ud55c \uc120\uac70\uc5c5\ubb34\uc758 \uc8fc\uc5ed\uc73c\ub85c \uc678\uad6d\uad70(\ubbf8\uad6d, \uc601\uad6d, \ud638\uc8fc, \ub8e8\ub9c8\ub2c8\uc544)\uc744 \uc9c0\ud718\u00b7\uac10\ub3c5, \uc774\ub77c\ud06c \ubbfc\uc8fc\uc8fc\uc758 \ud0c4\uc0dd\uc5d0 \uae30\uc5ec\ud55c \uac83\uc73c\ub85c \ud3c9\uac00\ubc1b\uc558\ub2e4. \ud55c\uad6d\uc774 \uc774\ub77c\ud06c\uc804 \ub54c \uc138 \ubc88\uc9f8\ub85c \ub9ce\uc740 \ubcd1\ub825\uc744 \ubcf4\ub0c8\ub294\ub370 \ub2e4\uad6d\uc801\uad70\uc0ac\ub839\ubd80 \ucc38\ubaa8 \uc911\uc5d0 \ud55c\uad6d\uad70\uc774 \uadf8 \uc678\uc5d0\ub294 \uc5c6\uc5b4 \ucc98\uc74c\uc5d0\ub294 \uc9c1\uc18d \ubd80\ud558\ub4e4\uc774 \uc790\uc2e0\uc744 \ubb34\uc2dc\ud588\ub2e4\uace0 \ub290\ub080 \uadf8\uc600\ub2e4. \uadf8\ub798\uc11c \ubc14\uc058\ub2e4\ub294 \uc678\uad6d\uad70 \uc7a5\uad70\ub4e4 \ub300\uc2e0 \ube0c\ub9ac\ud551\uc744 \ud558\ub294 \ub4f1 \uc88b\uc740 \ud3c9\uac00\ub97c \ubc1b\uace0\uc790 \ub178\ub825\ud574 \uc810\ucc28 \uc870\uc9c1\uc744 \uc7a5\uc545\ud558\uc600\ub2e4\uace0 \ubc1d\ud614\ub2e4. \uadf8\ub294 \ubbf8\uad70\uacfc \uc601\uad6d\uad70, \ud638\uc8fc\uad70 \uc7a5\uad50 10\uc5ec \uba85\uc744 \uc9c0\ud718\ud574 5400\uac1c \uc120\uac70\uad6c\uc5d0\uc11c \uce58\ub7ec\uc9c0\ub294 \uc774\ub77c\ud06c \uc720\uad8c\uc790 1400\ub9cc \uba85\uc758 \ud22c\ud45c\ub97c \uc9c0\uc6d0 \ubc0f \uac10\ub3c5\ud558\uc600\uc73c\uba70, 3500t\uc5d0 \uc774\ub974\ub294 \uc120\uac70 \uad00\ub828 \ubb3c\uc790\ub97c \uac01 \ud22c\ud45c\uc18c\uc5d0 \uacf5\uae09\ud558\uace0 \uce58\uc548 \uc720\uc9c0\ub97c \uc9c0\uc6d0\ud558\ub294 \uc77c\ub3c4 \ub9e1\uc558\ub2e4. \uc120\uac70\uac00 \ubb34\uc0ac\ud788 \ub05d\ub09c \ub4a4 \uc774\ub77c\ud06c \ud3c9\ud654\uc815\ucc29\uc5d0 \uae30\uc5ec\ud55c \uacf5\ub85c\ub85c \ub300\ud55c\ubbfc\uad6d \uc815\ubd80\ub85c\ubd80\ud130 \ud654\ub791\ubb34\uacf5\ud6c8\uc7a5\uc744, \ud55c\uad6d \uad70\uc778\uc73c\ub85c\uc11c\ub294 \ucd5c\ucd08\ub85c \ubbf8\uad6d \uc815\ubd80\ub85c\ubd80\ud130 \ub3d9\uc131\ud6c8\uc7a5(Bronze Star Medal)\uc744 \ubc1b\uc558\ub2e4."} {"question": "\ubba4\uc9c1\ubc45\ud06c\uc5d0\uc11c \uac15\ub0a8\uc2a4\ud0c0\uc77c\uc740 \uba87 \uc8fc \uc5f0\uc18d 1\uc704\ub97c \ucc28\uc9c0\ud558\uc600\ub294\uac00?", "paragraph": "\uc2f8\uc774\ub294 2012\ub144 8\uc6d4 17\uc77c\ubd80\ud130 \ubc29\uc1a1\uc774 \uc2dc\uc791\ub41c \ub300\uad6d\ubbfc \uc624\ub514\uc158 \ud504\ub85c\uadf8\ub7a8 \u300a\uc288\ud37c\uc2a4\ud0c0K 4\u300b\uc758 \uba54\uc778 \uc2ec\uc0ac\uc704\uc6d0\uc73c\ub85c\ub3c4 \ud65c\ub3d9\ud558\uc600\ub2e4. 2012\ub144 11\uc6d4 16\uc77c, KBS2 \u300a\ubba4\uc9c1\ubc45\ud06c\u300b\uc5d0\uc11c \u3008\uac15\ub0a8\uc2a4\ud0c0\uc77c\u3009\ub85c 10\uc8fc \uc5f0\uc18d 1\uc704 \uae30\ub85d\uc744 \uc138\uc6e0\uc73c\uba70, \uc774\uac83\uc740 \uc885\uc804\uc758 \uc774 \ud504\ub85c\uadf8\ub7a8\uc5d0\uc11c \ucd5c\ub2e4 \uc5f0\uc18d \uae30\ub85d\uc774\uc5c8\ub358 \uc18c\ub140\uc2dc\ub300 \u3008Gee\u3009\uc758 9\uc8fc \uc5f0\uc18d 1\uc704 \uae30\ub85d\uc744 \uae6c\uac83\uacfc \ub3d9\uc2dc\uc5d0 \uc5ed\ub300 \uac00\uc694 \uc21c\uc704\ud504\ub85c\uadf8\ub7a8\uc73c\ub85c\uc11c\ub294 1992\ub144 \uac00\uc218 \uc2e0\uc2b9\ud6c8\uc774 \u300aSBS \uc778\uae30\uac00\uc694\u300b\uc5d0\uc11c \u3008\ubcf4\uc774\uc9c0 \uc54a\ub294 \uc0ac\ub791\u3009\uc73c\ub85c \uc138\uc6b4 14\uc8fc \uc5f0\uc18d 1\uc704 \uae30\ub85d\uc5d0 \uc774\uc740 \ub450\ubc88\uc9f8 \uae30\ub85d\uc774\uae30\ub3c4 \ud558\ub2e4. \u3008\uac15\ub0a8\uc2a4\ud0c0\uc77c\u3009\uc740 \uc774\ud6c4\uc5d0\ub3c4 1\uc704\ub97c \ub2e4\uc2dc \uc7ac\ud0c8\ud658\ud558\uba74\uc11c 2013\ub144\ucd08\uae4c\uc9c0 \ud1b5\uc0b0 16\uc8fc 1\uc704\ub97c \uae30\ub85d\ud558\uc5ec \ub2e8\uc77c\uace1 \ud1b5\uc0b0 1\uc704\ub85c\ub294 \uc5ed\ub300 \ucd5c\uace0 \uae30\ub85d\uc744 \uc218\ub9bd\ud558\uc600\ub2e4. 2012\ub144 12\uc6d4 31\uc77c, \uc2f8\uc774\ub294 \ub274\uc695 \ud0c0\uc784\uc2a4 \uc2a4\ud018\uc5b4\uc5d0\uc11c \uc5f4\ub9b0 \uc5f0\ub9d0 \ud589\uc0ac '\ub515 \ud074\ub77d\uc2a4 \ub274 \uc774\uc5bc\uc2a4 \ub85c\ud0b9 \uc774\ube0c'\uc5d0\uc11c MC \ud574\uba38\uc640 \ubba4\uc9c1\ube44\ub514\uc624\uc5d0 \ucd9c\uc5f0\ud55c \uc720\uc7ac\uc11d\uacfc \ub178\ud64d\ucca0\uacfc \uacf5\uc5f0\uc744 \ud3bc\ucce4\ub2e4.", "answer": "10\uc8fc", "sentence": "2012\ub144 11\uc6d4 16\uc77c, KBS2 \u300a\ubba4\uc9c1\ubc45\ud06c\u300b\uc5d0\uc11c \u3008\uac15\ub0a8\uc2a4\ud0c0\uc77c\u3009\ub85c 10\uc8fc \uc5f0\uc18d 1\uc704 \uae30\ub85d\uc744 \uc138\uc6e0\uc73c\uba70, \uc774\uac83\uc740 \uc885\uc804\uc758 \uc774 \ud504\ub85c\uadf8\ub7a8\uc5d0\uc11c \ucd5c\ub2e4 \uc5f0\uc18d \uae30\ub85d\uc774\uc5c8\ub358 \uc18c\ub140\uc2dc\ub300 \u3008Gee\u3009\uc758 9\uc8fc \uc5f0\uc18d 1\uc704 \uae30\ub85d\uc744 \uae6c\uac83\uacfc \ub3d9\uc2dc\uc5d0 \uc5ed\ub300 \uac00\uc694 \uc21c\uc704\ud504\ub85c\uadf8\ub7a8\uc73c\ub85c\uc11c\ub294 1992\ub144 \uac00\uc218 \uc2e0\uc2b9\ud6c8\uc774 \u300aSBS \uc778\uae30\uac00\uc694\u300b\uc5d0\uc11c \u3008\ubcf4\uc774\uc9c0 \uc54a\ub294 \uc0ac\ub791\u3009\uc73c\ub85c \uc138\uc6b4 14\uc8fc \uc5f0\uc18d 1\uc704 \uae30\ub85d\uc5d0 \uc774\uc740 \ub450\ubc88\uc9f8 \uae30\ub85d\uc774\uae30\ub3c4 \ud558\ub2e4.", "paragraph_sentence": "\uc2f8\uc774\ub294 2012\ub144 8\uc6d4 17\uc77c\ubd80\ud130 \ubc29\uc1a1\uc774 \uc2dc\uc791\ub41c \ub300\uad6d\ubbfc \uc624\ub514\uc158 \ud504\ub85c\uadf8\ub7a8 \u300a\uc288\ud37c\uc2a4\ud0c0K 4\u300b\uc758 \uba54\uc778 \uc2ec\uc0ac\uc704\uc6d0\uc73c\ub85c\ub3c4 \ud65c\ub3d9\ud558\uc600\ub2e4. 2012\ub144 11\uc6d4 16\uc77c, KBS2 \u300a\ubba4\uc9c1\ubc45\ud06c\u300b\uc5d0\uc11c \u3008\uac15\ub0a8\uc2a4\ud0c0\uc77c\u3009\ub85c 10\uc8fc \uc5f0\uc18d 1\uc704 \uae30\ub85d\uc744 \uc138\uc6e0\uc73c\uba70, \uc774\uac83\uc740 \uc885\uc804\uc758 \uc774 \ud504\ub85c\uadf8\ub7a8\uc5d0\uc11c \ucd5c\ub2e4 \uc5f0\uc18d \uae30\ub85d\uc774\uc5c8\ub358 \uc18c\ub140\uc2dc\ub300 \u3008Gee\u3009\uc758 9\uc8fc \uc5f0\uc18d 1\uc704 \uae30\ub85d\uc744 \uae6c\uac83\uacfc \ub3d9\uc2dc\uc5d0 \uc5ed\ub300 \uac00\uc694 \uc21c\uc704\ud504\ub85c\uadf8\ub7a8\uc73c\ub85c\uc11c\ub294 1992\ub144 \uac00\uc218 \uc2e0\uc2b9\ud6c8\uc774 \u300aSBS \uc778\uae30\uac00\uc694\u300b\uc5d0\uc11c \u3008\ubcf4\uc774\uc9c0 \uc54a\ub294 \uc0ac\ub791\u3009\uc73c\ub85c \uc138\uc6b4 14\uc8fc \uc5f0\uc18d 1\uc704 \uae30\ub85d\uc5d0 \uc774\uc740 \ub450\ubc88\uc9f8 \uae30\ub85d\uc774\uae30\ub3c4 \ud558\ub2e4. \u3008\uac15\ub0a8\uc2a4\ud0c0\uc77c\u3009\uc740 \uc774\ud6c4\uc5d0\ub3c4 1\uc704\ub97c \ub2e4\uc2dc \uc7ac\ud0c8\ud658\ud558\uba74\uc11c 2013\ub144\ucd08\uae4c\uc9c0 \ud1b5\uc0b0 16\uc8fc 1\uc704\ub97c \uae30\ub85d\ud558\uc5ec \ub2e8\uc77c\uace1 \ud1b5\uc0b0 1\uc704\ub85c\ub294 \uc5ed\ub300 \ucd5c\uace0 \uae30\ub85d\uc744 \uc218\ub9bd\ud558\uc600\ub2e4. 2012\ub144 12\uc6d4 31\uc77c, \uc2f8\uc774\ub294 \ub274\uc695 \ud0c0\uc784\uc2a4 \uc2a4\ud018\uc5b4\uc5d0\uc11c \uc5f4\ub9b0 \uc5f0\ub9d0 \ud589\uc0ac '\ub515 \ud074\ub77d\uc2a4 \ub274 \uc774\uc5bc\uc2a4 \ub85c\ud0b9 \uc774\ube0c'\uc5d0\uc11c MC \ud574\uba38\uc640 \ubba4\uc9c1\ube44\ub514\uc624\uc5d0 \ucd9c\uc5f0\ud55c \uc720\uc7ac\uc11d\uacfc \ub178\ud64d\ucca0\uacfc \uacf5\uc5f0\uc744 \ud3bc\ucce4\ub2e4.", "paragraph_answer": "\uc2f8\uc774\ub294 2012\ub144 8\uc6d4 17\uc77c\ubd80\ud130 \ubc29\uc1a1\uc774 \uc2dc\uc791\ub41c \ub300\uad6d\ubbfc \uc624\ub514\uc158 \ud504\ub85c\uadf8\ub7a8 \u300a\uc288\ud37c\uc2a4\ud0c0K 4\u300b\uc758 \uba54\uc778 \uc2ec\uc0ac\uc704\uc6d0\uc73c\ub85c\ub3c4 \ud65c\ub3d9\ud558\uc600\ub2e4. 2012\ub144 11\uc6d4 16\uc77c, KBS2 \u300a\ubba4\uc9c1\ubc45\ud06c\u300b\uc5d0\uc11c \u3008\uac15\ub0a8\uc2a4\ud0c0\uc77c\u3009\ub85c 10\uc8fc \uc5f0\uc18d 1\uc704 \uae30\ub85d\uc744 \uc138\uc6e0\uc73c\uba70, \uc774\uac83\uc740 \uc885\uc804\uc758 \uc774 \ud504\ub85c\uadf8\ub7a8\uc5d0\uc11c \ucd5c\ub2e4 \uc5f0\uc18d \uae30\ub85d\uc774\uc5c8\ub358 \uc18c\ub140\uc2dc\ub300 \u3008Gee\u3009\uc758 9\uc8fc \uc5f0\uc18d 1\uc704 \uae30\ub85d\uc744 \uae6c\uac83\uacfc \ub3d9\uc2dc\uc5d0 \uc5ed\ub300 \uac00\uc694 \uc21c\uc704\ud504\ub85c\uadf8\ub7a8\uc73c\ub85c\uc11c\ub294 1992\ub144 \uac00\uc218 \uc2e0\uc2b9\ud6c8\uc774 \u300aSBS \uc778\uae30\uac00\uc694\u300b\uc5d0\uc11c \u3008\ubcf4\uc774\uc9c0 \uc54a\ub294 \uc0ac\ub791\u3009\uc73c\ub85c \uc138\uc6b4 14\uc8fc \uc5f0\uc18d 1\uc704 \uae30\ub85d\uc5d0 \uc774\uc740 \ub450\ubc88\uc9f8 \uae30\ub85d\uc774\uae30\ub3c4 \ud558\ub2e4. \u3008\uac15\ub0a8\uc2a4\ud0c0\uc77c\u3009\uc740 \uc774\ud6c4\uc5d0\ub3c4 1\uc704\ub97c \ub2e4\uc2dc \uc7ac\ud0c8\ud658\ud558\uba74\uc11c 2013\ub144\ucd08\uae4c\uc9c0 \ud1b5\uc0b0 16\uc8fc 1\uc704\ub97c \uae30\ub85d\ud558\uc5ec \ub2e8\uc77c\uace1 \ud1b5\uc0b0 1\uc704\ub85c\ub294 \uc5ed\ub300 \ucd5c\uace0 \uae30\ub85d\uc744 \uc218\ub9bd\ud558\uc600\ub2e4. 2012\ub144 12\uc6d4 31\uc77c, \uc2f8\uc774\ub294 \ub274\uc695 \ud0c0\uc784\uc2a4 \uc2a4\ud018\uc5b4\uc5d0\uc11c \uc5f4\ub9b0 \uc5f0\ub9d0 \ud589\uc0ac '\ub515 \ud074\ub77d\uc2a4 \ub274 \uc774\uc5bc\uc2a4 \ub85c\ud0b9 \uc774\ube0c'\uc5d0\uc11c MC \ud574\uba38\uc640 \ubba4\uc9c1\ube44\ub514\uc624\uc5d0 \ucd9c\uc5f0\ud55c \uc720\uc7ac\uc11d\uacfc \ub178\ud64d\ucca0\uacfc \uacf5\uc5f0\uc744 \ud3bc\ucce4\ub2e4.", "sentence_answer": "2012\ub144 11\uc6d4 16\uc77c, KBS2 \u300a\ubba4\uc9c1\ubc45\ud06c\u300b\uc5d0\uc11c \u3008\uac15\ub0a8\uc2a4\ud0c0\uc77c\u3009\ub85c 10\uc8fc \uc5f0\uc18d 1\uc704 \uae30\ub85d\uc744 \uc138\uc6e0\uc73c\uba70, \uc774\uac83\uc740 \uc885\uc804\uc758 \uc774 \ud504\ub85c\uadf8\ub7a8\uc5d0\uc11c \ucd5c\ub2e4 \uc5f0\uc18d \uae30\ub85d\uc774\uc5c8\ub358 \uc18c\ub140\uc2dc\ub300 \u3008Gee\u3009\uc758 9\uc8fc \uc5f0\uc18d 1\uc704 \uae30\ub85d\uc744 \uae6c\uac83\uacfc \ub3d9\uc2dc\uc5d0 \uc5ed\ub300 \uac00\uc694 \uc21c\uc704\ud504\ub85c\uadf8\ub7a8\uc73c\ub85c\uc11c\ub294 1992\ub144 \uac00\uc218 \uc2e0\uc2b9\ud6c8\uc774 \u300aSBS \uc778\uae30\uac00\uc694\u300b\uc5d0\uc11c \u3008\ubcf4\uc774\uc9c0 \uc54a\ub294 \uc0ac\ub791\u3009\uc73c\ub85c \uc138\uc6b4 14\uc8fc \uc5f0\uc18d 1\uc704 \uae30\ub85d\uc5d0 \uc774\uc740 \ub450\ubc88\uc9f8 \uae30\ub85d\uc774\uae30\ub3c4 \ud558\ub2e4.", "paragraph_id": "6457532-6-1", "paragraph_question": "question: \ubba4\uc9c1\ubc45\ud06c\uc5d0\uc11c \uac15\ub0a8\uc2a4\ud0c0\uc77c\uc740 \uba87 \uc8fc \uc5f0\uc18d 1\uc704\ub97c \ucc28\uc9c0\ud558\uc600\ub294\uac00?, context: \uc2f8\uc774\ub294 2012\ub144 8\uc6d4 17\uc77c\ubd80\ud130 \ubc29\uc1a1\uc774 \uc2dc\uc791\ub41c \ub300\uad6d\ubbfc \uc624\ub514\uc158 \ud504\ub85c\uadf8\ub7a8 \u300a\uc288\ud37c\uc2a4\ud0c0K 4\u300b\uc758 \uba54\uc778 \uc2ec\uc0ac\uc704\uc6d0\uc73c\ub85c\ub3c4 \ud65c\ub3d9\ud558\uc600\ub2e4. 2012\ub144 11\uc6d4 16\uc77c, KBS2 \u300a\ubba4\uc9c1\ubc45\ud06c\u300b\uc5d0\uc11c \u3008\uac15\ub0a8\uc2a4\ud0c0\uc77c\u3009\ub85c 10\uc8fc \uc5f0\uc18d 1\uc704 \uae30\ub85d\uc744 \uc138\uc6e0\uc73c\uba70, \uc774\uac83\uc740 \uc885\uc804\uc758 \uc774 \ud504\ub85c\uadf8\ub7a8\uc5d0\uc11c \ucd5c\ub2e4 \uc5f0\uc18d \uae30\ub85d\uc774\uc5c8\ub358 \uc18c\ub140\uc2dc\ub300 \u3008Gee\u3009\uc758 9\uc8fc \uc5f0\uc18d 1\uc704 \uae30\ub85d\uc744 \uae6c\uac83\uacfc \ub3d9\uc2dc\uc5d0 \uc5ed\ub300 \uac00\uc694 \uc21c\uc704\ud504\ub85c\uadf8\ub7a8\uc73c\ub85c\uc11c\ub294 1992\ub144 \uac00\uc218 \uc2e0\uc2b9\ud6c8\uc774 \u300aSBS \uc778\uae30\uac00\uc694\u300b\uc5d0\uc11c \u3008\ubcf4\uc774\uc9c0 \uc54a\ub294 \uc0ac\ub791\u3009\uc73c\ub85c \uc138\uc6b4 14\uc8fc \uc5f0\uc18d 1\uc704 \uae30\ub85d\uc5d0 \uc774\uc740 \ub450\ubc88\uc9f8 \uae30\ub85d\uc774\uae30\ub3c4 \ud558\ub2e4. \u3008\uac15\ub0a8\uc2a4\ud0c0\uc77c\u3009\uc740 \uc774\ud6c4\uc5d0\ub3c4 1\uc704\ub97c \ub2e4\uc2dc \uc7ac\ud0c8\ud658\ud558\uba74\uc11c 2013\ub144\ucd08\uae4c\uc9c0 \ud1b5\uc0b0 16\uc8fc 1\uc704\ub97c \uae30\ub85d\ud558\uc5ec \ub2e8\uc77c\uace1 \ud1b5\uc0b0 1\uc704\ub85c\ub294 \uc5ed\ub300 \ucd5c\uace0 \uae30\ub85d\uc744 \uc218\ub9bd\ud558\uc600\ub2e4. 2012\ub144 12\uc6d4 31\uc77c, \uc2f8\uc774\ub294 \ub274\uc695 \ud0c0\uc784\uc2a4 \uc2a4\ud018\uc5b4\uc5d0\uc11c \uc5f4\ub9b0 \uc5f0\ub9d0 \ud589\uc0ac '\ub515 \ud074\ub77d\uc2a4 \ub274 \uc774\uc5bc\uc2a4 \ub85c\ud0b9 \uc774\ube0c'\uc5d0\uc11c MC \ud574\uba38\uc640 \ubba4\uc9c1\ube44\ub514\uc624\uc5d0 \ucd9c\uc5f0\ud55c \uc720\uc7ac\uc11d\uacfc \ub178\ud64d\ucca0\uacfc \uacf5\uc5f0\uc744 \ud3bc\ucce4\ub2e4."} {"question": "\uc7a5 \ud3f4 \uace0\ud2f0\uc5d0\uc758 \uccab \uceec\ub809\uc158\uc774 \uc131\uacf5\ud55c \ub54c\ub294?", "paragraph": "1990\ub144\ub300\uc5d0\ub294 \uc7a5 \ud3f4 \uace0\ud2f0\uc5d0 \u2018\ud5a5\uc218\u2019\uac00 \ud0c4\uc0dd\ud588\ub2e4. \uadf8\uac83\uc740 \ub9c8\ub3c8\ub098\uac00 \uc6d0\ucd94\ud615 \ucef5\uc73c\ub85c \uc720\uba85\ud55c \ucf54\ub974\uc14b\uc5d0\uc11c \uc720\ub798\ud55c \uc637\uc744 \uc785\uc740 \uc2dc\uc810\uc774\uae30\ub3c4 \ud588\ub2e4. 1980\ub144\ub300 \uc138\ub300\uc758 \ub514\uc790\uc774\ub108\ub4e4\uc774 \uc774\ube0c \uc0dd\ub85c\ub791\uc774\ub098 \uae4c\ub974\ubcb5(Carven) \uac19\uc740 \uc778\ubb3c\uc774 \uc2b9\ud654\ub41c \uc624\ud2b8 \uafb8\ub6f0\ub974\uc5d0 \uc2a4\ud0c0\uc77c\uc5d0 \uc21c\uc751\ud558\uc9c0 \uc54a\uc558\uc9c0\ub9cc \uc7a5 \ud3f4 \uace0\ud2f0\uc5d0 (Jean Paul Gaultier)\ub294 1997\ub144 \uadf8\uc758 \uc774\ub984\uc73c\ub85c \ub41c \uadf8\uc758 \uccab \uceec\ub809\uc158\uc744 \uc131\uacf5\uc801\uc73c\ub85c \uc774\ub048\ub2e4. CFDA\uc5d0 \uac1c\ucd5c\ub418\ub294 \ud328\uc158\uc2dc\uc0c1\uc2dd\uc5d0\uc11c \uace7 \uc0c1\uc744 \ubc1b\uac8c \ub41c\ub2e4. 2004\ub144 \uc7a5 \ud3f4 \uace0\ud2f0\uc5d0\ub294 \uc790\uc2e0\uc758 \uc774\ub984\uc73c\ub85c \ud65c\ub3d9\ud558\uba74\uc11c 6\ub144\ub3d9\uc548 \uc5d0\ub974\uba54\uc2a4\uc758 \uae30\uc131\ubcf5\ub4e4\uc744 \ub3d9\uc2dc\uc5d0 \ub9cc\ub4e4\uc5c8\ub2e4. \uc218\ub144 \ub3d9\uc548, \uadf8\ub294 \uc544\ub3d9\ubcf5 \uace0\ud2f0\uc5d0, \uace0\ud2f0\uc5d0 \uccad\ubc14\uc9c0 \ub610\ub294 \ub0a8\ub140 \uacf5\uc6a9 \uace0\ud2f0\uc5d0 \ub77c\uc778\uacfc \uac19\uc740 \ubcf4\uc644 \ub77c\uc778\uc744 \ube44\ub86f\ud558\uc5ec \uc774\ud0c8\ub9ac\uc544 \uc81c\uc870\uc5c5\uccb4 \uc778 \ud398\ub97c\ub77c\uc640 \ud568\uaed8 \uace0\uae09\uc2a4\ub7ec\uc6b4 \ub780\uc81c\ub9ac \uceec\ub809\uc158\uc744 \uc2dc\uc791\ud55c\ub2e4.", "answer": "1997\ub144", "sentence": "1980\ub144\ub300 \uc138\ub300\uc758 \ub514\uc790\uc774\ub108\ub4e4\uc774 \uc774\ube0c \uc0dd\ub85c\ub791\uc774\ub098 \uae4c\ub974\ubcb5(Carven) \uac19\uc740 \uc778\ubb3c\uc774 \uc2b9\ud654\ub41c \uc624\ud2b8 \uafb8\ub6f0\ub974\uc5d0 \uc2a4\ud0c0\uc77c\uc5d0 \uc21c\uc751\ud558\uc9c0 \uc54a\uc558\uc9c0\ub9cc \uc7a5 \ud3f4 \uace0\ud2f0\uc5d0 (Jean Paul Gaultier)\ub294 1997\ub144 \uadf8\uc758 \uc774\ub984\uc73c\ub85c \ub41c \uadf8\uc758 \uccab \uceec\ub809\uc158\uc744 \uc131\uacf5\uc801\uc73c\ub85c \uc774\ub048\ub2e4.", "paragraph_sentence": "1990\ub144\ub300\uc5d0\ub294 \uc7a5 \ud3f4 \uace0\ud2f0\uc5d0 \u2018\ud5a5\uc218\u2019\uac00 \ud0c4\uc0dd\ud588\ub2e4. \uadf8\uac83\uc740 \ub9c8\ub3c8\ub098\uac00 \uc6d0\ucd94\ud615 \ucef5\uc73c\ub85c \uc720\uba85\ud55c \ucf54\ub974\uc14b\uc5d0\uc11c \uc720\ub798\ud55c \uc637\uc744 \uc785\uc740 \uc2dc\uc810\uc774\uae30\ub3c4 \ud588\ub2e4. 1980\ub144\ub300 \uc138\ub300\uc758 \ub514\uc790\uc774\ub108\ub4e4\uc774 \uc774\ube0c \uc0dd\ub85c\ub791\uc774\ub098 \uae4c\ub974\ubcb5(Carven) \uac19\uc740 \uc778\ubb3c\uc774 \uc2b9\ud654\ub41c \uc624\ud2b8 \uafb8\ub6f0\ub974\uc5d0 \uc2a4\ud0c0\uc77c\uc5d0 \uc21c\uc751\ud558\uc9c0 \uc54a\uc558\uc9c0\ub9cc \uc7a5 \ud3f4 \uace0\ud2f0\uc5d0 (Jean Paul Gaultier)\ub294 1997\ub144 \uadf8\uc758 \uc774\ub984\uc73c\ub85c \ub41c \uadf8\uc758 \uccab \uceec\ub809\uc158\uc744 \uc131\uacf5\uc801\uc73c\ub85c \uc774\ub048\ub2e4. CFDA\uc5d0 \uac1c\ucd5c\ub418\ub294 \ud328\uc158\uc2dc\uc0c1\uc2dd\uc5d0\uc11c \uace7 \uc0c1\uc744 \ubc1b\uac8c \ub41c\ub2e4. 2004\ub144 \uc7a5 \ud3f4 \uace0\ud2f0\uc5d0\ub294 \uc790\uc2e0\uc758 \uc774\ub984\uc73c\ub85c \ud65c\ub3d9\ud558\uba74\uc11c 6\ub144\ub3d9\uc548 \uc5d0\ub974\uba54\uc2a4\uc758 \uae30\uc131\ubcf5\ub4e4\uc744 \ub3d9\uc2dc\uc5d0 \ub9cc\ub4e4\uc5c8\ub2e4. \uc218\ub144 \ub3d9\uc548, \uadf8\ub294 \uc544\ub3d9\ubcf5 \uace0\ud2f0\uc5d0, \uace0\ud2f0\uc5d0 \uccad\ubc14\uc9c0 \ub610\ub294 \ub0a8\ub140 \uacf5\uc6a9 \uace0\ud2f0\uc5d0 \ub77c\uc778\uacfc \uac19\uc740 \ubcf4\uc644 \ub77c\uc778\uc744 \ube44\ub86f\ud558\uc5ec \uc774\ud0c8\ub9ac\uc544 \uc81c\uc870\uc5c5\uccb4 \uc778 \ud398\ub97c\ub77c\uc640 \ud568\uaed8 \uace0\uae09\uc2a4\ub7ec\uc6b4 \ub780\uc81c\ub9ac \uceec\ub809\uc158\uc744 \uc2dc\uc791\ud55c\ub2e4.", "paragraph_answer": "1990\ub144\ub300\uc5d0\ub294 \uc7a5 \ud3f4 \uace0\ud2f0\uc5d0 \u2018\ud5a5\uc218\u2019\uac00 \ud0c4\uc0dd\ud588\ub2e4. \uadf8\uac83\uc740 \ub9c8\ub3c8\ub098\uac00 \uc6d0\ucd94\ud615 \ucef5\uc73c\ub85c \uc720\uba85\ud55c \ucf54\ub974\uc14b\uc5d0\uc11c \uc720\ub798\ud55c \uc637\uc744 \uc785\uc740 \uc2dc\uc810\uc774\uae30\ub3c4 \ud588\ub2e4. 1980\ub144\ub300 \uc138\ub300\uc758 \ub514\uc790\uc774\ub108\ub4e4\uc774 \uc774\ube0c \uc0dd\ub85c\ub791\uc774\ub098 \uae4c\ub974\ubcb5(Carven) \uac19\uc740 \uc778\ubb3c\uc774 \uc2b9\ud654\ub41c \uc624\ud2b8 \uafb8\ub6f0\ub974\uc5d0 \uc2a4\ud0c0\uc77c\uc5d0 \uc21c\uc751\ud558\uc9c0 \uc54a\uc558\uc9c0\ub9cc \uc7a5 \ud3f4 \uace0\ud2f0\uc5d0 (Jean Paul Gaultier)\ub294 1997\ub144 \uadf8\uc758 \uc774\ub984\uc73c\ub85c \ub41c \uadf8\uc758 \uccab \uceec\ub809\uc158\uc744 \uc131\uacf5\uc801\uc73c\ub85c \uc774\ub048\ub2e4. CFDA\uc5d0 \uac1c\ucd5c\ub418\ub294 \ud328\uc158\uc2dc\uc0c1\uc2dd\uc5d0\uc11c \uace7 \uc0c1\uc744 \ubc1b\uac8c \ub41c\ub2e4. 2004\ub144 \uc7a5 \ud3f4 \uace0\ud2f0\uc5d0\ub294 \uc790\uc2e0\uc758 \uc774\ub984\uc73c\ub85c \ud65c\ub3d9\ud558\uba74\uc11c 6\ub144\ub3d9\uc548 \uc5d0\ub974\uba54\uc2a4\uc758 \uae30\uc131\ubcf5\ub4e4\uc744 \ub3d9\uc2dc\uc5d0 \ub9cc\ub4e4\uc5c8\ub2e4. \uc218\ub144 \ub3d9\uc548, \uadf8\ub294 \uc544\ub3d9\ubcf5 \uace0\ud2f0\uc5d0, \uace0\ud2f0\uc5d0 \uccad\ubc14\uc9c0 \ub610\ub294 \ub0a8\ub140 \uacf5\uc6a9 \uace0\ud2f0\uc5d0 \ub77c\uc778\uacfc \uac19\uc740 \ubcf4\uc644 \ub77c\uc778\uc744 \ube44\ub86f\ud558\uc5ec \uc774\ud0c8\ub9ac\uc544 \uc81c\uc870\uc5c5\uccb4 \uc778 \ud398\ub97c\ub77c\uc640 \ud568\uaed8 \uace0\uae09\uc2a4\ub7ec\uc6b4 \ub780\uc81c\ub9ac \uceec\ub809\uc158\uc744 \uc2dc\uc791\ud55c\ub2e4.", "sentence_answer": "1980\ub144\ub300 \uc138\ub300\uc758 \ub514\uc790\uc774\ub108\ub4e4\uc774 \uc774\ube0c \uc0dd\ub85c\ub791\uc774\ub098 \uae4c\ub974\ubcb5(Carven) \uac19\uc740 \uc778\ubb3c\uc774 \uc2b9\ud654\ub41c \uc624\ud2b8 \uafb8\ub6f0\ub974\uc5d0 \uc2a4\ud0c0\uc77c\uc5d0 \uc21c\uc751\ud558\uc9c0 \uc54a\uc558\uc9c0\ub9cc \uc7a5 \ud3f4 \uace0\ud2f0\uc5d0 (Jean Paul Gaultier)\ub294 1997\ub144 \uadf8\uc758 \uc774\ub984\uc73c\ub85c \ub41c \uadf8\uc758 \uccab \uceec\ub809\uc158\uc744 \uc131\uacf5\uc801\uc73c\ub85c \uc774\ub048\ub2e4.", "paragraph_id": "5859484-1-0", "paragraph_question": "question: \uc7a5 \ud3f4 \uace0\ud2f0\uc5d0\uc758 \uccab \uceec\ub809\uc158\uc774 \uc131\uacf5\ud55c \ub54c\ub294?, context: 1990\ub144\ub300\uc5d0\ub294 \uc7a5 \ud3f4 \uace0\ud2f0\uc5d0 \u2018\ud5a5\uc218\u2019\uac00 \ud0c4\uc0dd\ud588\ub2e4. \uadf8\uac83\uc740 \ub9c8\ub3c8\ub098\uac00 \uc6d0\ucd94\ud615 \ucef5\uc73c\ub85c \uc720\uba85\ud55c \ucf54\ub974\uc14b\uc5d0\uc11c \uc720\ub798\ud55c \uc637\uc744 \uc785\uc740 \uc2dc\uc810\uc774\uae30\ub3c4 \ud588\ub2e4. 1980\ub144\ub300 \uc138\ub300\uc758 \ub514\uc790\uc774\ub108\ub4e4\uc774 \uc774\ube0c \uc0dd\ub85c\ub791\uc774\ub098 \uae4c\ub974\ubcb5(Carven) \uac19\uc740 \uc778\ubb3c\uc774 \uc2b9\ud654\ub41c \uc624\ud2b8 \uafb8\ub6f0\ub974\uc5d0 \uc2a4\ud0c0\uc77c\uc5d0 \uc21c\uc751\ud558\uc9c0 \uc54a\uc558\uc9c0\ub9cc \uc7a5 \ud3f4 \uace0\ud2f0\uc5d0 (Jean Paul Gaultier)\ub294 1997\ub144 \uadf8\uc758 \uc774\ub984\uc73c\ub85c \ub41c \uadf8\uc758 \uccab \uceec\ub809\uc158\uc744 \uc131\uacf5\uc801\uc73c\ub85c \uc774\ub048\ub2e4. CFDA\uc5d0 \uac1c\ucd5c\ub418\ub294 \ud328\uc158\uc2dc\uc0c1\uc2dd\uc5d0\uc11c \uace7 \uc0c1\uc744 \ubc1b\uac8c \ub41c\ub2e4. 2004\ub144 \uc7a5 \ud3f4 \uace0\ud2f0\uc5d0\ub294 \uc790\uc2e0\uc758 \uc774\ub984\uc73c\ub85c \ud65c\ub3d9\ud558\uba74\uc11c 6\ub144\ub3d9\uc548 \uc5d0\ub974\uba54\uc2a4\uc758 \uae30\uc131\ubcf5\ub4e4\uc744 \ub3d9\uc2dc\uc5d0 \ub9cc\ub4e4\uc5c8\ub2e4. \uc218\ub144 \ub3d9\uc548, \uadf8\ub294 \uc544\ub3d9\ubcf5 \uace0\ud2f0\uc5d0, \uace0\ud2f0\uc5d0 \uccad\ubc14\uc9c0 \ub610\ub294 \ub0a8\ub140 \uacf5\uc6a9 \uace0\ud2f0\uc5d0 \ub77c\uc778\uacfc \uac19\uc740 \ubcf4\uc644 \ub77c\uc778\uc744 \ube44\ub86f\ud558\uc5ec \uc774\ud0c8\ub9ac\uc544 \uc81c\uc870\uc5c5\uccb4 \uc778 \ud398\ub97c\ub77c\uc640 \ud568\uaed8 \uace0\uae09\uc2a4\ub7ec\uc6b4 \ub780\uc81c\ub9ac \uceec\ub809\uc158\uc744 \uc2dc\uc791\ud55c\ub2e4."} {"question": "\uc2a4\ud398\uc778 \uc815\uaddc \ubcd1\uc0ac\ub4e4\uc758 \uaddc\ubaa8\ub294 \uc5b4\ub290 \uc815\ub3c4\uc600\ub098?", "paragraph": "\uae30\ub3c5\uad50 \ub3d9\ub9f9 \ud568\ub300\ub294 12,920\uba85\uc758 \uc120\uc6d0\uc73c\ub85c \uad6c\uc131\ub418\uc5b4 \uc788\uc5c8\ub2e4. \uc5ec\uae30\uc5d0 \ub354\ud558\uc5ec \uac70\uc758 28,000\uba85\uc5d0 \uc774\ub974\ub294 \ubcd1\uc0ac\ub97c \uc218\uc1a1\ud558\uace0 \uc788\uc5c8\ub2e4. \uc798 \ud6c8\ub828\ubc1b\uc740 \uc2a4\ud398\uc778 \uc815\uaddc \ubcd1\uc0ac\ub4e4\uc740 10,000\uba85 \uc815\ub3c4\uc758 \uaddc\ubaa8\uc600\uace0, \uadf8 \uc678\uc5d0\ub294 7,000\uba85\uc758 \ub3c5\uc77c \uc6a9\ubcd1\uacfc 6,000\uba85 \uc774\ud0c8\ub9ac\uc544 \uc6a9\ubcd1, \uadf8\ub9ac\uace0 5,000\uba85\uc758 \ubca0\ub124\uce58\uc544 \ubcd1\uc0ac\ub4e4\ub85c \uad6c\uc131\ub418\uc5b4 \uc788\uc5c8\ub2e4. \ub610 \ubca0\ub124\uce58\uc544\uc758 \ub178\uc7a1\uc774\ub4e4\uc740 \uc624\uc2a4\ub9cc \ud568\ub300\uc5d0 \uc5f4 \uc9c0\uc5b4 \uc788\ub358 \ub178\uc608\ub4e4\uacfc \ub2e4\ub974\uac8c \uc790\uc720 \uc2dc\ubbfc\ub4e4\ub85c \uad6c\uc131\ub418\uc5b4 \uc788\uc5c8\uc73c\ubbc0\ub85c \ubb34\uc7a5\uc744 \ud558\uace0 \uc788\uc5c8\uace0, \uc870\uad6d\uc744 \uc704\ud574 \uc2f8\uc6b8 \uc218 \uc788\uc5c8\ub2e4. \uc790\uc720\ubbfc \ub178\uc7a1\uc774\ub4e4\uc740 \uc77c\ubc18\uc801\uc73c\ub85c \ub6f0\uc5b4\ub09c \uc2e4\ub825\uc774 \uc788\ub2e4\uace0 \uc778\uc815\ubc1b\uc558\uc73c\ub098, 16\uc138\uae30 \ub3d9\uc548 \ubaa8\ub4e0 \uac24\ub9ac\uc120 \ud568\ub300\uc5d0\uc11c \ub178\uc608\uc640 \ubc94\uc8c4\uc790 \uc804\uc7c1 \ud3ec\ub85c\ub4e4\ub85c \uc11c\uc11c\ud788 \ub300\uccb4\ub418\uc5c8\ub294\ub370, \uc790\uc720\ubbfc \ub178\uc7a1\uc774\ub4e4\uc758 \uc784\uae08\uc774 \uc9c0\ub098\uce58\uac8c \ube44\uc30c\uae30 \ub54c\ubb38\uc774\ub2e4.", "answer": "10,000\uba85 \uc815\ub3c4", "sentence": "\uc798 \ud6c8\ub828\ubc1b\uc740 \uc2a4\ud398\uc778 \uc815\uaddc \ubcd1\uc0ac\ub4e4\uc740 10,000\uba85 \uc815\ub3c4 \uc758 \uaddc\ubaa8\uc600\uace0, \uadf8 \uc678\uc5d0\ub294 7,000\uba85\uc758 \ub3c5\uc77c \uc6a9\ubcd1\uacfc 6,000\uba85 \uc774\ud0c8\ub9ac\uc544 \uc6a9\ubcd1, \uadf8\ub9ac\uace0 5,000\uba85\uc758 \ubca0\ub124\uce58\uc544 \ubcd1\uc0ac\ub4e4\ub85c \uad6c\uc131\ub418\uc5b4 \uc788\uc5c8\ub2e4.", "paragraph_sentence": "\uae30\ub3c5\uad50 \ub3d9\ub9f9 \ud568\ub300\ub294 12,920\uba85\uc758 \uc120\uc6d0\uc73c\ub85c \uad6c\uc131\ub418\uc5b4 \uc788\uc5c8\ub2e4. \uc5ec\uae30\uc5d0 \ub354\ud558\uc5ec \uac70\uc758 28,000\uba85\uc5d0 \uc774\ub974\ub294 \ubcd1\uc0ac\ub97c \uc218\uc1a1\ud558\uace0 \uc788\uc5c8\ub2e4. \uc798 \ud6c8\ub828\ubc1b\uc740 \uc2a4\ud398\uc778 \uc815\uaddc \ubcd1\uc0ac\ub4e4\uc740 10,000\uba85 \uc815\ub3c4 \uc758 \uaddc\ubaa8\uc600\uace0, \uadf8 \uc678\uc5d0\ub294 7,000\uba85\uc758 \ub3c5\uc77c \uc6a9\ubcd1\uacfc 6,000\uba85 \uc774\ud0c8\ub9ac\uc544 \uc6a9\ubcd1, \uadf8\ub9ac\uace0 5,000\uba85\uc758 \ubca0\ub124\uce58\uc544 \ubcd1\uc0ac\ub4e4\ub85c \uad6c\uc131\ub418\uc5b4 \uc788\uc5c8\ub2e4. \ub610 \ubca0\ub124\uce58\uc544\uc758 \ub178\uc7a1\uc774\ub4e4\uc740 \uc624\uc2a4\ub9cc \ud568\ub300\uc5d0 \uc5f4 \uc9c0\uc5b4 \uc788\ub358 \ub178\uc608\ub4e4\uacfc \ub2e4\ub974\uac8c \uc790\uc720 \uc2dc\ubbfc\ub4e4\ub85c \uad6c\uc131\ub418\uc5b4 \uc788\uc5c8\uc73c\ubbc0\ub85c \ubb34\uc7a5\uc744 \ud558\uace0 \uc788\uc5c8\uace0, \uc870\uad6d\uc744 \uc704\ud574 \uc2f8\uc6b8 \uc218 \uc788\uc5c8\ub2e4. \uc790\uc720\ubbfc \ub178\uc7a1\uc774\ub4e4\uc740 \uc77c\ubc18\uc801\uc73c\ub85c \ub6f0\uc5b4\ub09c \uc2e4\ub825\uc774 \uc788\ub2e4\uace0 \uc778\uc815\ubc1b\uc558\uc73c\ub098, 16\uc138\uae30 \ub3d9\uc548 \ubaa8\ub4e0 \uac24\ub9ac\uc120 \ud568\ub300\uc5d0\uc11c \ub178\uc608\uc640 \ubc94\uc8c4\uc790 \uc804\uc7c1 \ud3ec\ub85c\ub4e4\ub85c \uc11c\uc11c\ud788 \ub300\uccb4\ub418\uc5c8\ub294\ub370, \uc790\uc720\ubbfc \ub178\uc7a1\uc774\ub4e4\uc758 \uc784\uae08\uc774 \uc9c0\ub098\uce58\uac8c \ube44\uc30c\uae30 \ub54c\ubb38\uc774\ub2e4.", "paragraph_answer": "\uae30\ub3c5\uad50 \ub3d9\ub9f9 \ud568\ub300\ub294 12,920\uba85\uc758 \uc120\uc6d0\uc73c\ub85c \uad6c\uc131\ub418\uc5b4 \uc788\uc5c8\ub2e4. \uc5ec\uae30\uc5d0 \ub354\ud558\uc5ec \uac70\uc758 28,000\uba85\uc5d0 \uc774\ub974\ub294 \ubcd1\uc0ac\ub97c \uc218\uc1a1\ud558\uace0 \uc788\uc5c8\ub2e4. \uc798 \ud6c8\ub828\ubc1b\uc740 \uc2a4\ud398\uc778 \uc815\uaddc \ubcd1\uc0ac\ub4e4\uc740 10,000\uba85 \uc815\ub3c4 \uc758 \uaddc\ubaa8\uc600\uace0, \uadf8 \uc678\uc5d0\ub294 7,000\uba85\uc758 \ub3c5\uc77c \uc6a9\ubcd1\uacfc 6,000\uba85 \uc774\ud0c8\ub9ac\uc544 \uc6a9\ubcd1, \uadf8\ub9ac\uace0 5,000\uba85\uc758 \ubca0\ub124\uce58\uc544 \ubcd1\uc0ac\ub4e4\ub85c \uad6c\uc131\ub418\uc5b4 \uc788\uc5c8\ub2e4. \ub610 \ubca0\ub124\uce58\uc544\uc758 \ub178\uc7a1\uc774\ub4e4\uc740 \uc624\uc2a4\ub9cc \ud568\ub300\uc5d0 \uc5f4 \uc9c0\uc5b4 \uc788\ub358 \ub178\uc608\ub4e4\uacfc \ub2e4\ub974\uac8c \uc790\uc720 \uc2dc\ubbfc\ub4e4\ub85c \uad6c\uc131\ub418\uc5b4 \uc788\uc5c8\uc73c\ubbc0\ub85c \ubb34\uc7a5\uc744 \ud558\uace0 \uc788\uc5c8\uace0, \uc870\uad6d\uc744 \uc704\ud574 \uc2f8\uc6b8 \uc218 \uc788\uc5c8\ub2e4. \uc790\uc720\ubbfc \ub178\uc7a1\uc774\ub4e4\uc740 \uc77c\ubc18\uc801\uc73c\ub85c \ub6f0\uc5b4\ub09c \uc2e4\ub825\uc774 \uc788\ub2e4\uace0 \uc778\uc815\ubc1b\uc558\uc73c\ub098, 16\uc138\uae30 \ub3d9\uc548 \ubaa8\ub4e0 \uac24\ub9ac\uc120 \ud568\ub300\uc5d0\uc11c \ub178\uc608\uc640 \ubc94\uc8c4\uc790 \uc804\uc7c1 \ud3ec\ub85c\ub4e4\ub85c \uc11c\uc11c\ud788 \ub300\uccb4\ub418\uc5c8\ub294\ub370, \uc790\uc720\ubbfc \ub178\uc7a1\uc774\ub4e4\uc758 \uc784\uae08\uc774 \uc9c0\ub098\uce58\uac8c \ube44\uc30c\uae30 \ub54c\ubb38\uc774\ub2e4.", "sentence_answer": "\uc798 \ud6c8\ub828\ubc1b\uc740 \uc2a4\ud398\uc778 \uc815\uaddc \ubcd1\uc0ac\ub4e4\uc740 10,000\uba85 \uc815\ub3c4 \uc758 \uaddc\ubaa8\uc600\uace0, \uadf8 \uc678\uc5d0\ub294 7,000\uba85\uc758 \ub3c5\uc77c \uc6a9\ubcd1\uacfc 6,000\uba85 \uc774\ud0c8\ub9ac\uc544 \uc6a9\ubcd1, \uadf8\ub9ac\uace0 5,000\uba85\uc758 \ubca0\ub124\uce58\uc544 \ubcd1\uc0ac\ub4e4\ub85c \uad6c\uc131\ub418\uc5b4 \uc788\uc5c8\ub2e4.", "paragraph_id": "6306274-2-1", "paragraph_question": "question: \uc2a4\ud398\uc778 \uc815\uaddc \ubcd1\uc0ac\ub4e4\uc758 \uaddc\ubaa8\ub294 \uc5b4\ub290 \uc815\ub3c4\uc600\ub098?, context: \uae30\ub3c5\uad50 \ub3d9\ub9f9 \ud568\ub300\ub294 12,920\uba85\uc758 \uc120\uc6d0\uc73c\ub85c \uad6c\uc131\ub418\uc5b4 \uc788\uc5c8\ub2e4. \uc5ec\uae30\uc5d0 \ub354\ud558\uc5ec \uac70\uc758 28,000\uba85\uc5d0 \uc774\ub974\ub294 \ubcd1\uc0ac\ub97c \uc218\uc1a1\ud558\uace0 \uc788\uc5c8\ub2e4. \uc798 \ud6c8\ub828\ubc1b\uc740 \uc2a4\ud398\uc778 \uc815\uaddc \ubcd1\uc0ac\ub4e4\uc740 10,000\uba85 \uc815\ub3c4\uc758 \uaddc\ubaa8\uc600\uace0, \uadf8 \uc678\uc5d0\ub294 7,000\uba85\uc758 \ub3c5\uc77c \uc6a9\ubcd1\uacfc 6,000\uba85 \uc774\ud0c8\ub9ac\uc544 \uc6a9\ubcd1, \uadf8\ub9ac\uace0 5,000\uba85\uc758 \ubca0\ub124\uce58\uc544 \ubcd1\uc0ac\ub4e4\ub85c \uad6c\uc131\ub418\uc5b4 \uc788\uc5c8\ub2e4. \ub610 \ubca0\ub124\uce58\uc544\uc758 \ub178\uc7a1\uc774\ub4e4\uc740 \uc624\uc2a4\ub9cc \ud568\ub300\uc5d0 \uc5f4 \uc9c0\uc5b4 \uc788\ub358 \ub178\uc608\ub4e4\uacfc \ub2e4\ub974\uac8c \uc790\uc720 \uc2dc\ubbfc\ub4e4\ub85c \uad6c\uc131\ub418\uc5b4 \uc788\uc5c8\uc73c\ubbc0\ub85c \ubb34\uc7a5\uc744 \ud558\uace0 \uc788\uc5c8\uace0, \uc870\uad6d\uc744 \uc704\ud574 \uc2f8\uc6b8 \uc218 \uc788\uc5c8\ub2e4. \uc790\uc720\ubbfc \ub178\uc7a1\uc774\ub4e4\uc740 \uc77c\ubc18\uc801\uc73c\ub85c \ub6f0\uc5b4\ub09c \uc2e4\ub825\uc774 \uc788\ub2e4\uace0 \uc778\uc815\ubc1b\uc558\uc73c\ub098, 16\uc138\uae30 \ub3d9\uc548 \ubaa8\ub4e0 \uac24\ub9ac\uc120 \ud568\ub300\uc5d0\uc11c \ub178\uc608\uc640 \ubc94\uc8c4\uc790 \uc804\uc7c1 \ud3ec\ub85c\ub4e4\ub85c \uc11c\uc11c\ud788 \ub300\uccb4\ub418\uc5c8\ub294\ub370, \uc790\uc720\ubbfc \ub178\uc7a1\uc774\ub4e4\uc758 \uc784\uae08\uc774 \uc9c0\ub098\uce58\uac8c \ube44\uc30c\uae30 \ub54c\ubb38\uc774\ub2e4."} {"question": "\ub808\ud53c\ub450\uc2a4\ub294 \uc625\ud0c0\ube44\uc544\ub204\uc2a4\uc5d0\uac8c \ud22c\ud56d\ud558\uace0 \uc5b4\ub5a4 \uc9c1\ud568\uc744 \uc0ac\uc6a9\ud558\ub3c4\ub85d \ud5c8\ub77d\ubc1b\ub098?", "paragraph": "\uae30\uc6d0\uc804 36\ub144 \uc625\ud0c0\ube44\uc544\ub204\uc2a4\uc640 \ub808\ud53c\ub450\uc2a4\ub294 \uc139\uc2a4\ud22c\uc2a4 \ud3fc\ud398\uc774\uc6b0\uc2a4\ub97c \uce58\uae30 \uc704\ud574 \ud569\ub3d9 \uc791\uc804\uc744 \uac1c\uc2dc\ud558\uc600\ub2e4. \uc625\ud0c0\ube44\uc544\ub204\uc2a4\ub97c \uc774\uacbc\uc74c\uc5d0\ub3c4 \ubd88\uad6c\ud558\uace0 \uc139\uc2a4\ud22c\uc2a4 \ud3fc\ud398\uc774\uc6b0\uc2a4\uc758 \ud568\ub300\ub294 \uae30\uc6d0\uc804 36\ub144 9\uc6d4 3\uc77c \ub098\uc6b0\ub85c\ucfe0\uc2a4 \uc804\ud22c\uc5d0\uc11c \uc544\uadf8\ub9ac\ud30c\uc758 \uad70\ub300\uc5d0\uac8c \uac70\uc758 \uc644\ud30c\ub418\uc5c8\ub2e4. \uc139\uc2a4\ud22c\uc2a4\ub294 \ub0a8\uc740 \uad70\ub300\ub97c \uc774\ub04c\uace0 \ub3d9\ucabd\uc73c\ub85c \ud1f4\uac01\ud558\uc600\ub294\ub370, \uadf8 \ub2e4\uc74c\ud574\uc5d0 \ud3fc\ud398\uc774\uc6b0\uc2a4\ub294 \uc548\ud1a0\ub2c8\uc6b0\uc2a4 \ud718\ud558\uc758 \uc7a5\uad70\uc5d0\uac8c \uc7a1\ud600 \ucc98\ud615\ub418\uc5c8\ub2e4. \uc625\ud0c0\ube44\uc544\ub204\uc2a4\uc640 \ub808\ud53c\ub450\uc2a4\ub294 \ud56d\ubcf5\ud55c \ud3fc\ud398\uc774\uc6b0\uc2a4\uc758 \ubcd1\uc0ac\ub97c \uc190\uc5d0 \ub123\uc5c8\ub294\ub370 \uba38\uc9c0\uc54a\uc544 \ub808\ud53c\ub450\uc2a4\ub294 \uc790\uc2e0\uc774 \uc2dc\uce60\ub9ac\uc544\ub97c \ub2e4\uc2a4\ub9b4 \ucda9\ubd84\ud55c \ub2a5\ub825\uc774 \ub41c\ub2e4 \uc790\ubd80\ud558\uace0 \uc625\ud0c0\ube44\uc544\ub204\uc2a4\uc5d0\uac8c \ub5a0\ub098\ub77c \uba85\ub839\ud558\uc600\ub2e4. \uadf8\ub7ec\ub098 \ub808\ud53c\ub450\uc2a4\uc758 \uad70\ub300\ub294 \uc790\uc2e0\ub4e4\uc774 \uc2f8\uc6b0\ub294 \ub370 \uc9c0\uce58\uace0 \uc625\ud0c0\ube44\uc544\ub204\uc2a4\uac00 \uadf8\ub4e4\uc744 \ub3c8\uc73c\ub85c \uc720\ud639\ud558\uc790 \ub808\ud53c\ub450\uc2a4\ub97c \ubc84\ub9ac\uace0 \uc625\ud0c0\ube44\uc544\ub204\uc2a4\uc5d0\uac8c\ub85c \ud22c\ud56d\ud558\uc600\ub2e4. \ub808\ud53c\ub450\uc2a4\ub294 \uc625\ud0c0\ube44\uc544\ub204\uc2a4\uc5d0\uac8c \ud56d\ubcf5\ud558\uace0 \ucd5c\uace0 \uc81c\uc0ac\uc7a5(pontifex maximus)\uc758 \uc9c1\ud568\uc744 \uc0ac\uc6a9\ud560 \uc218 \uc788\ub3c4\ub85d \ud5c8\ub77d\ubc1b\uc558\uc73c\ub098 \uc0bc\ub450 \uc815\uce58 \uccb4\uc81c\uc5d0\uc11c \ucad3\uaca8\ub098\uace0 \uacf5\uc9c1 \uc0dd\ud65c \uc5ed\uc2dc \ub05d\uc774 \ub098\uba74\uc11c \uc774\ud0c8\ub9ac\uc544\uc758 \uce74\ud398 \ud0a4\ub974\ucf00\uc774\uc5d0 \uc788\ub294 \uc7a5\uc6d0\uc73c\ub85c \uc720\ubc30\ub418\uc5c8\ub2e4. \ub85c\ub9c8\uc758 \ud1b5\uce58\uad8c\uc740 \uc774\uc81c \uc11c\ubc29\uc758 \uc625\ud0c0\ube44\uc544\ub204\uc2a4\uc640 \ub3d9\ubc29\uc758 \uc548\ud1a0\ub2c8\uc6b0\uc2a4\uc5d0\uac8c\ub85c \uc8fc\uc5b4\uc84c\uace0 \uc591\ubd84\ub418\uc5c8\ub2e4. \uacf5\ud654\uad6d\uc758 \ud3c9\ud654\uc640 \uc548\uc815\uc744 \uc9c0\ud0a4\uae30 \uc704\ud574 \uc625\ud0c0\ube44\uc544\ub204\uc2a4\ub294 \ub85c\ub9c8 \uc2dc\ubbfc\uc758 \uc7ac\uc0b0\uad8c\uc744 \ubcf4\uc7a5\ud574 \uc8fc\uc5c8\ub2e4. \uc625\ud0c0\ube44\uc544\ub204\uc2a4\uac00 \uc804\uc5ed\ud55c \ubcd1\uc0ac\ub4e4\uc5d0\uac8c \uc774\ud0c8\ub9ac\uc544 \uc678\uacfd\uc5d0 \uc790\ub9ac\ub97c \uc7a1\uac8c \ud574\uc8fc\ub294 \uc0ac\uc774, \uacfc\uac70 \ud3fc\ud398\uc774\uc6b0\uc2a4\uc758 \uad70\ub300\uc5d0 \ucc38\uac00\ud558\uae30 \uc704\ud574 \ub85c\ub9c8\ub97c \ub5a0\ub0ac\ub358 3\ub9cc\uc5ec \uba85\uc758 \ub178\uc608\ub97c \ubaa8\ub450 \uc8fc\uc778\uc5d0\uac8c \ub3cc\ub824\uc8fc\uc5c8\ub2e4. \uc625\ud0c0\ube44\uc544\ub204\uc2a4\ub294 \ub85c\ub9c8\ub85c \ub3cc\uc544\uc624\uc790 \uc790\uc2e0\uacfc \ub9ac\ube44\uc544, \uc625\ud0c0\ube44\uc544\uc758 \uc2e0\ubcc0 \uc548\uc804\uc758 \ubcf4\uc7a5\uc744 \uc6d0\ub85c\uc6d0\uc5d0 \uc694\uccad\ud558\uc600\ub2e4. \uadf8 \uacb0\uacfc \uc625\ud0c0\ube44\uc544\ub204\uc2a4\uc640 \uadf8\uc758 \uc544\ub0b4 \ub9ac\ube44\uc544, \ub204\uc774\uc778 \uc625\ud0c0\ube44\uc544, \ub538\uc778 \uc728\ub9ac\uc544\ub294 \uc8fc\uad8c\uba74\uc81c\uad8c\uc744 \ubd80\uc5ec\ubc1b\uc558\ub2e4.", "answer": "\ucd5c\uace0 \uc81c\uc0ac\uc7a5", "sentence": "\ub808\ud53c\ub450\uc2a4\ub294 \uc625\ud0c0\ube44\uc544\ub204\uc2a4\uc5d0\uac8c \ud56d\ubcf5\ud558\uace0 \ucd5c\uace0 \uc81c\uc0ac\uc7a5 (pontifex maximus)", "paragraph_sentence": "\uae30\uc6d0\uc804 36\ub144 \uc625\ud0c0\ube44\uc544\ub204\uc2a4\uc640 \ub808\ud53c\ub450\uc2a4\ub294 \uc139\uc2a4\ud22c\uc2a4 \ud3fc\ud398\uc774\uc6b0\uc2a4\ub97c \uce58\uae30 \uc704\ud574 \ud569\ub3d9 \uc791\uc804\uc744 \uac1c\uc2dc\ud558\uc600\ub2e4. \uc625\ud0c0\ube44\uc544\ub204\uc2a4\ub97c \uc774\uacbc\uc74c\uc5d0\ub3c4 \ubd88\uad6c\ud558\uace0 \uc139\uc2a4\ud22c\uc2a4 \ud3fc\ud398\uc774\uc6b0\uc2a4\uc758 \ud568\ub300\ub294 \uae30\uc6d0\uc804 36\ub144 9\uc6d4 3\uc77c \ub098\uc6b0\ub85c\ucfe0\uc2a4 \uc804\ud22c\uc5d0\uc11c \uc544\uadf8\ub9ac\ud30c\uc758 \uad70\ub300\uc5d0\uac8c \uac70\uc758 \uc644\ud30c\ub418\uc5c8\ub2e4. \uc139\uc2a4\ud22c\uc2a4\ub294 \ub0a8\uc740 \uad70\ub300\ub97c \uc774\ub04c\uace0 \ub3d9\ucabd\uc73c\ub85c \ud1f4\uac01\ud558\uc600\ub294\ub370, \uadf8 \ub2e4\uc74c\ud574\uc5d0 \ud3fc\ud398\uc774\uc6b0\uc2a4\ub294 \uc548\ud1a0\ub2c8\uc6b0\uc2a4 \ud718\ud558\uc758 \uc7a5\uad70\uc5d0\uac8c \uc7a1\ud600 \ucc98\ud615\ub418\uc5c8\ub2e4. \uc625\ud0c0\ube44\uc544\ub204\uc2a4\uc640 \ub808\ud53c\ub450\uc2a4\ub294 \ud56d\ubcf5\ud55c \ud3fc\ud398\uc774\uc6b0\uc2a4\uc758 \ubcd1\uc0ac\ub97c \uc190\uc5d0 \ub123\uc5c8\ub294\ub370 \uba38\uc9c0\uc54a\uc544 \ub808\ud53c\ub450\uc2a4\ub294 \uc790\uc2e0\uc774 \uc2dc\uce60\ub9ac\uc544\ub97c \ub2e4\uc2a4\ub9b4 \ucda9\ubd84\ud55c \ub2a5\ub825\uc774 \ub41c\ub2e4 \uc790\ubd80\ud558\uace0 \uc625\ud0c0\ube44\uc544\ub204\uc2a4\uc5d0\uac8c \ub5a0\ub098\ub77c \uba85\ub839\ud558\uc600\ub2e4. \uadf8\ub7ec\ub098 \ub808\ud53c\ub450\uc2a4\uc758 \uad70\ub300\ub294 \uc790\uc2e0\ub4e4\uc774 \uc2f8\uc6b0\ub294 \ub370 \uc9c0\uce58\uace0 \uc625\ud0c0\ube44\uc544\ub204\uc2a4\uac00 \uadf8\ub4e4\uc744 \ub3c8\uc73c\ub85c \uc720\ud639\ud558\uc790 \ub808\ud53c\ub450\uc2a4\ub97c \ubc84\ub9ac\uace0 \uc625\ud0c0\ube44\uc544\ub204\uc2a4\uc5d0\uac8c\ub85c \ud22c\ud56d\ud558\uc600\ub2e4. \ub808\ud53c\ub450\uc2a4\ub294 \uc625\ud0c0\ube44\uc544\ub204\uc2a4\uc5d0\uac8c \ud56d\ubcf5\ud558\uace0 \ucd5c\uace0 \uc81c\uc0ac\uc7a5 (pontifex maximus) \uc758 \uc9c1\ud568\uc744 \uc0ac\uc6a9\ud560 \uc218 \uc788\ub3c4\ub85d \ud5c8\ub77d\ubc1b\uc558\uc73c\ub098 \uc0bc\ub450 \uc815\uce58 \uccb4\uc81c\uc5d0\uc11c \ucad3\uaca8\ub098\uace0 \uacf5\uc9c1 \uc0dd\ud65c \uc5ed\uc2dc \ub05d\uc774 \ub098\uba74\uc11c \uc774\ud0c8\ub9ac\uc544\uc758 \uce74\ud398 \ud0a4\ub974\ucf00\uc774\uc5d0 \uc788\ub294 \uc7a5\uc6d0\uc73c\ub85c \uc720\ubc30\ub418\uc5c8\ub2e4. \ub85c\ub9c8\uc758 \ud1b5\uce58\uad8c\uc740 \uc774\uc81c \uc11c\ubc29\uc758 \uc625\ud0c0\ube44\uc544\ub204\uc2a4\uc640 \ub3d9\ubc29\uc758 \uc548\ud1a0\ub2c8\uc6b0\uc2a4\uc5d0\uac8c\ub85c \uc8fc\uc5b4\uc84c\uace0 \uc591\ubd84\ub418\uc5c8\ub2e4. \uacf5\ud654\uad6d\uc758 \ud3c9\ud654\uc640 \uc548\uc815\uc744 \uc9c0\ud0a4\uae30 \uc704\ud574 \uc625\ud0c0\ube44\uc544\ub204\uc2a4\ub294 \ub85c\ub9c8 \uc2dc\ubbfc\uc758 \uc7ac\uc0b0\uad8c\uc744 \ubcf4\uc7a5\ud574 \uc8fc\uc5c8\ub2e4. \uc625\ud0c0\ube44\uc544\ub204\uc2a4\uac00 \uc804\uc5ed\ud55c \ubcd1\uc0ac\ub4e4\uc5d0\uac8c \uc774\ud0c8\ub9ac\uc544 \uc678\uacfd\uc5d0 \uc790\ub9ac\ub97c \uc7a1\uac8c \ud574\uc8fc\ub294 \uc0ac\uc774, \uacfc\uac70 \ud3fc\ud398\uc774\uc6b0\uc2a4\uc758 \uad70\ub300\uc5d0 \ucc38\uac00\ud558\uae30 \uc704\ud574 \ub85c\ub9c8\ub97c \ub5a0\ub0ac\ub358 3\ub9cc\uc5ec \uba85\uc758 \ub178\uc608\ub97c \ubaa8\ub450 \uc8fc\uc778\uc5d0\uac8c \ub3cc\ub824\uc8fc\uc5c8\ub2e4. \uc625\ud0c0\ube44\uc544\ub204\uc2a4\ub294 \ub85c\ub9c8\ub85c \ub3cc\uc544\uc624\uc790 \uc790\uc2e0\uacfc \ub9ac\ube44\uc544, \uc625\ud0c0\ube44\uc544\uc758 \uc2e0\ubcc0 \uc548\uc804\uc758 \ubcf4\uc7a5\uc744 \uc6d0\ub85c\uc6d0\uc5d0 \uc694\uccad\ud558\uc600\ub2e4. \uadf8 \uacb0\uacfc \uc625\ud0c0\ube44\uc544\ub204\uc2a4\uc640 \uadf8\uc758 \uc544\ub0b4 \ub9ac\ube44\uc544, \ub204\uc774\uc778 \uc625\ud0c0\ube44\uc544, \ub538\uc778 \uc728\ub9ac\uc544\ub294 \uc8fc\uad8c\uba74\uc81c\uad8c\uc744 \ubd80\uc5ec\ubc1b\uc558\ub2e4.", "paragraph_answer": "\uae30\uc6d0\uc804 36\ub144 \uc625\ud0c0\ube44\uc544\ub204\uc2a4\uc640 \ub808\ud53c\ub450\uc2a4\ub294 \uc139\uc2a4\ud22c\uc2a4 \ud3fc\ud398\uc774\uc6b0\uc2a4\ub97c \uce58\uae30 \uc704\ud574 \ud569\ub3d9 \uc791\uc804\uc744 \uac1c\uc2dc\ud558\uc600\ub2e4. \uc625\ud0c0\ube44\uc544\ub204\uc2a4\ub97c \uc774\uacbc\uc74c\uc5d0\ub3c4 \ubd88\uad6c\ud558\uace0 \uc139\uc2a4\ud22c\uc2a4 \ud3fc\ud398\uc774\uc6b0\uc2a4\uc758 \ud568\ub300\ub294 \uae30\uc6d0\uc804 36\ub144 9\uc6d4 3\uc77c \ub098\uc6b0\ub85c\ucfe0\uc2a4 \uc804\ud22c\uc5d0\uc11c \uc544\uadf8\ub9ac\ud30c\uc758 \uad70\ub300\uc5d0\uac8c \uac70\uc758 \uc644\ud30c\ub418\uc5c8\ub2e4. \uc139\uc2a4\ud22c\uc2a4\ub294 \ub0a8\uc740 \uad70\ub300\ub97c \uc774\ub04c\uace0 \ub3d9\ucabd\uc73c\ub85c \ud1f4\uac01\ud558\uc600\ub294\ub370, \uadf8 \ub2e4\uc74c\ud574\uc5d0 \ud3fc\ud398\uc774\uc6b0\uc2a4\ub294 \uc548\ud1a0\ub2c8\uc6b0\uc2a4 \ud718\ud558\uc758 \uc7a5\uad70\uc5d0\uac8c \uc7a1\ud600 \ucc98\ud615\ub418\uc5c8\ub2e4. \uc625\ud0c0\ube44\uc544\ub204\uc2a4\uc640 \ub808\ud53c\ub450\uc2a4\ub294 \ud56d\ubcf5\ud55c \ud3fc\ud398\uc774\uc6b0\uc2a4\uc758 \ubcd1\uc0ac\ub97c \uc190\uc5d0 \ub123\uc5c8\ub294\ub370 \uba38\uc9c0\uc54a\uc544 \ub808\ud53c\ub450\uc2a4\ub294 \uc790\uc2e0\uc774 \uc2dc\uce60\ub9ac\uc544\ub97c \ub2e4\uc2a4\ub9b4 \ucda9\ubd84\ud55c \ub2a5\ub825\uc774 \ub41c\ub2e4 \uc790\ubd80\ud558\uace0 \uc625\ud0c0\ube44\uc544\ub204\uc2a4\uc5d0\uac8c \ub5a0\ub098\ub77c \uba85\ub839\ud558\uc600\ub2e4. \uadf8\ub7ec\ub098 \ub808\ud53c\ub450\uc2a4\uc758 \uad70\ub300\ub294 \uc790\uc2e0\ub4e4\uc774 \uc2f8\uc6b0\ub294 \ub370 \uc9c0\uce58\uace0 \uc625\ud0c0\ube44\uc544\ub204\uc2a4\uac00 \uadf8\ub4e4\uc744 \ub3c8\uc73c\ub85c \uc720\ud639\ud558\uc790 \ub808\ud53c\ub450\uc2a4\ub97c \ubc84\ub9ac\uace0 \uc625\ud0c0\ube44\uc544\ub204\uc2a4\uc5d0\uac8c\ub85c \ud22c\ud56d\ud558\uc600\ub2e4. \ub808\ud53c\ub450\uc2a4\ub294 \uc625\ud0c0\ube44\uc544\ub204\uc2a4\uc5d0\uac8c \ud56d\ubcf5\ud558\uace0 \ucd5c\uace0 \uc81c\uc0ac\uc7a5 (pontifex maximus)\uc758 \uc9c1\ud568\uc744 \uc0ac\uc6a9\ud560 \uc218 \uc788\ub3c4\ub85d \ud5c8\ub77d\ubc1b\uc558\uc73c\ub098 \uc0bc\ub450 \uc815\uce58 \uccb4\uc81c\uc5d0\uc11c \ucad3\uaca8\ub098\uace0 \uacf5\uc9c1 \uc0dd\ud65c \uc5ed\uc2dc \ub05d\uc774 \ub098\uba74\uc11c \uc774\ud0c8\ub9ac\uc544\uc758 \uce74\ud398 \ud0a4\ub974\ucf00\uc774\uc5d0 \uc788\ub294 \uc7a5\uc6d0\uc73c\ub85c \uc720\ubc30\ub418\uc5c8\ub2e4. \ub85c\ub9c8\uc758 \ud1b5\uce58\uad8c\uc740 \uc774\uc81c \uc11c\ubc29\uc758 \uc625\ud0c0\ube44\uc544\ub204\uc2a4\uc640 \ub3d9\ubc29\uc758 \uc548\ud1a0\ub2c8\uc6b0\uc2a4\uc5d0\uac8c\ub85c \uc8fc\uc5b4\uc84c\uace0 \uc591\ubd84\ub418\uc5c8\ub2e4. \uacf5\ud654\uad6d\uc758 \ud3c9\ud654\uc640 \uc548\uc815\uc744 \uc9c0\ud0a4\uae30 \uc704\ud574 \uc625\ud0c0\ube44\uc544\ub204\uc2a4\ub294 \ub85c\ub9c8 \uc2dc\ubbfc\uc758 \uc7ac\uc0b0\uad8c\uc744 \ubcf4\uc7a5\ud574 \uc8fc\uc5c8\ub2e4. \uc625\ud0c0\ube44\uc544\ub204\uc2a4\uac00 \uc804\uc5ed\ud55c \ubcd1\uc0ac\ub4e4\uc5d0\uac8c \uc774\ud0c8\ub9ac\uc544 \uc678\uacfd\uc5d0 \uc790\ub9ac\ub97c \uc7a1\uac8c \ud574\uc8fc\ub294 \uc0ac\uc774, \uacfc\uac70 \ud3fc\ud398\uc774\uc6b0\uc2a4\uc758 \uad70\ub300\uc5d0 \ucc38\uac00\ud558\uae30 \uc704\ud574 \ub85c\ub9c8\ub97c \ub5a0\ub0ac\ub358 3\ub9cc\uc5ec \uba85\uc758 \ub178\uc608\ub97c \ubaa8\ub450 \uc8fc\uc778\uc5d0\uac8c \ub3cc\ub824\uc8fc\uc5c8\ub2e4. \uc625\ud0c0\ube44\uc544\ub204\uc2a4\ub294 \ub85c\ub9c8\ub85c \ub3cc\uc544\uc624\uc790 \uc790\uc2e0\uacfc \ub9ac\ube44\uc544, \uc625\ud0c0\ube44\uc544\uc758 \uc2e0\ubcc0 \uc548\uc804\uc758 \ubcf4\uc7a5\uc744 \uc6d0\ub85c\uc6d0\uc5d0 \uc694\uccad\ud558\uc600\ub2e4. \uadf8 \uacb0\uacfc \uc625\ud0c0\ube44\uc544\ub204\uc2a4\uc640 \uadf8\uc758 \uc544\ub0b4 \ub9ac\ube44\uc544, \ub204\uc774\uc778 \uc625\ud0c0\ube44\uc544, \ub538\uc778 \uc728\ub9ac\uc544\ub294 \uc8fc\uad8c\uba74\uc81c\uad8c\uc744 \ubd80\uc5ec\ubc1b\uc558\ub2e4.", "sentence_answer": "\ub808\ud53c\ub450\uc2a4\ub294 \uc625\ud0c0\ube44\uc544\ub204\uc2a4\uc5d0\uac8c \ud56d\ubcf5\ud558\uace0 \ucd5c\uace0 \uc81c\uc0ac\uc7a5 (pontifex maximus)", "paragraph_id": "6585254-16-1", "paragraph_question": "question: \ub808\ud53c\ub450\uc2a4\ub294 \uc625\ud0c0\ube44\uc544\ub204\uc2a4\uc5d0\uac8c \ud22c\ud56d\ud558\uace0 \uc5b4\ub5a4 \uc9c1\ud568\uc744 \uc0ac\uc6a9\ud558\ub3c4\ub85d \ud5c8\ub77d\ubc1b\ub098?, context: \uae30\uc6d0\uc804 36\ub144 \uc625\ud0c0\ube44\uc544\ub204\uc2a4\uc640 \ub808\ud53c\ub450\uc2a4\ub294 \uc139\uc2a4\ud22c\uc2a4 \ud3fc\ud398\uc774\uc6b0\uc2a4\ub97c \uce58\uae30 \uc704\ud574 \ud569\ub3d9 \uc791\uc804\uc744 \uac1c\uc2dc\ud558\uc600\ub2e4. \uc625\ud0c0\ube44\uc544\ub204\uc2a4\ub97c \uc774\uacbc\uc74c\uc5d0\ub3c4 \ubd88\uad6c\ud558\uace0 \uc139\uc2a4\ud22c\uc2a4 \ud3fc\ud398\uc774\uc6b0\uc2a4\uc758 \ud568\ub300\ub294 \uae30\uc6d0\uc804 36\ub144 9\uc6d4 3\uc77c \ub098\uc6b0\ub85c\ucfe0\uc2a4 \uc804\ud22c\uc5d0\uc11c \uc544\uadf8\ub9ac\ud30c\uc758 \uad70\ub300\uc5d0\uac8c \uac70\uc758 \uc644\ud30c\ub418\uc5c8\ub2e4. \uc139\uc2a4\ud22c\uc2a4\ub294 \ub0a8\uc740 \uad70\ub300\ub97c \uc774\ub04c\uace0 \ub3d9\ucabd\uc73c\ub85c \ud1f4\uac01\ud558\uc600\ub294\ub370, \uadf8 \ub2e4\uc74c\ud574\uc5d0 \ud3fc\ud398\uc774\uc6b0\uc2a4\ub294 \uc548\ud1a0\ub2c8\uc6b0\uc2a4 \ud718\ud558\uc758 \uc7a5\uad70\uc5d0\uac8c \uc7a1\ud600 \ucc98\ud615\ub418\uc5c8\ub2e4. \uc625\ud0c0\ube44\uc544\ub204\uc2a4\uc640 \ub808\ud53c\ub450\uc2a4\ub294 \ud56d\ubcf5\ud55c \ud3fc\ud398\uc774\uc6b0\uc2a4\uc758 \ubcd1\uc0ac\ub97c \uc190\uc5d0 \ub123\uc5c8\ub294\ub370 \uba38\uc9c0\uc54a\uc544 \ub808\ud53c\ub450\uc2a4\ub294 \uc790\uc2e0\uc774 \uc2dc\uce60\ub9ac\uc544\ub97c \ub2e4\uc2a4\ub9b4 \ucda9\ubd84\ud55c \ub2a5\ub825\uc774 \ub41c\ub2e4 \uc790\ubd80\ud558\uace0 \uc625\ud0c0\ube44\uc544\ub204\uc2a4\uc5d0\uac8c \ub5a0\ub098\ub77c \uba85\ub839\ud558\uc600\ub2e4. \uadf8\ub7ec\ub098 \ub808\ud53c\ub450\uc2a4\uc758 \uad70\ub300\ub294 \uc790\uc2e0\ub4e4\uc774 \uc2f8\uc6b0\ub294 \ub370 \uc9c0\uce58\uace0 \uc625\ud0c0\ube44\uc544\ub204\uc2a4\uac00 \uadf8\ub4e4\uc744 \ub3c8\uc73c\ub85c \uc720\ud639\ud558\uc790 \ub808\ud53c\ub450\uc2a4\ub97c \ubc84\ub9ac\uace0 \uc625\ud0c0\ube44\uc544\ub204\uc2a4\uc5d0\uac8c\ub85c \ud22c\ud56d\ud558\uc600\ub2e4. \ub808\ud53c\ub450\uc2a4\ub294 \uc625\ud0c0\ube44\uc544\ub204\uc2a4\uc5d0\uac8c \ud56d\ubcf5\ud558\uace0 \ucd5c\uace0 \uc81c\uc0ac\uc7a5(pontifex maximus)\uc758 \uc9c1\ud568\uc744 \uc0ac\uc6a9\ud560 \uc218 \uc788\ub3c4\ub85d \ud5c8\ub77d\ubc1b\uc558\uc73c\ub098 \uc0bc\ub450 \uc815\uce58 \uccb4\uc81c\uc5d0\uc11c \ucad3\uaca8\ub098\uace0 \uacf5\uc9c1 \uc0dd\ud65c \uc5ed\uc2dc \ub05d\uc774 \ub098\uba74\uc11c \uc774\ud0c8\ub9ac\uc544\uc758 \uce74\ud398 \ud0a4\ub974\ucf00\uc774\uc5d0 \uc788\ub294 \uc7a5\uc6d0\uc73c\ub85c \uc720\ubc30\ub418\uc5c8\ub2e4. \ub85c\ub9c8\uc758 \ud1b5\uce58\uad8c\uc740 \uc774\uc81c \uc11c\ubc29\uc758 \uc625\ud0c0\ube44\uc544\ub204\uc2a4\uc640 \ub3d9\ubc29\uc758 \uc548\ud1a0\ub2c8\uc6b0\uc2a4\uc5d0\uac8c\ub85c \uc8fc\uc5b4\uc84c\uace0 \uc591\ubd84\ub418\uc5c8\ub2e4. \uacf5\ud654\uad6d\uc758 \ud3c9\ud654\uc640 \uc548\uc815\uc744 \uc9c0\ud0a4\uae30 \uc704\ud574 \uc625\ud0c0\ube44\uc544\ub204\uc2a4\ub294 \ub85c\ub9c8 \uc2dc\ubbfc\uc758 \uc7ac\uc0b0\uad8c\uc744 \ubcf4\uc7a5\ud574 \uc8fc\uc5c8\ub2e4. \uc625\ud0c0\ube44\uc544\ub204\uc2a4\uac00 \uc804\uc5ed\ud55c \ubcd1\uc0ac\ub4e4\uc5d0\uac8c \uc774\ud0c8\ub9ac\uc544 \uc678\uacfd\uc5d0 \uc790\ub9ac\ub97c \uc7a1\uac8c \ud574\uc8fc\ub294 \uc0ac\uc774, \uacfc\uac70 \ud3fc\ud398\uc774\uc6b0\uc2a4\uc758 \uad70\ub300\uc5d0 \ucc38\uac00\ud558\uae30 \uc704\ud574 \ub85c\ub9c8\ub97c \ub5a0\ub0ac\ub358 3\ub9cc\uc5ec \uba85\uc758 \ub178\uc608\ub97c \ubaa8\ub450 \uc8fc\uc778\uc5d0\uac8c \ub3cc\ub824\uc8fc\uc5c8\ub2e4. \uc625\ud0c0\ube44\uc544\ub204\uc2a4\ub294 \ub85c\ub9c8\ub85c \ub3cc\uc544\uc624\uc790 \uc790\uc2e0\uacfc \ub9ac\ube44\uc544, \uc625\ud0c0\ube44\uc544\uc758 \uc2e0\ubcc0 \uc548\uc804\uc758 \ubcf4\uc7a5\uc744 \uc6d0\ub85c\uc6d0\uc5d0 \uc694\uccad\ud558\uc600\ub2e4. \uadf8 \uacb0\uacfc \uc625\ud0c0\ube44\uc544\ub204\uc2a4\uc640 \uadf8\uc758 \uc544\ub0b4 \ub9ac\ube44\uc544, \ub204\uc774\uc778 \uc625\ud0c0\ube44\uc544, \ub538\uc778 \uc728\ub9ac\uc544\ub294 \uc8fc\uad8c\uba74\uc81c\uad8c\uc744 \ubd80\uc5ec\ubc1b\uc558\ub2e4."} {"question": "\uae40\ud55c\uc194\uc774 \uc878\uc5c5\ud55c \ub300\ud559 \uba85\uc740?", "paragraph": "\uae40\ud55c\uc194\uc740 1995\ub144 6\uc6d4 16\uc77c\uc5d0 \ud3c9\uc591\uc5d0\uc11c \ud0dc\uc5b4\ub0ac\ub2e4. \uc544\ubc84\uc9c0 \uae40\uc815\ub0a8\uc774 \ud6c4\uacc4 \uad6c\ub3c4\uc5d0\uc11c \ubc00\ub9ac\uc790 \uc5ec\ub7ec\ub098\ub77c\ub97c \ub5a0\ub3cc\uc544 \ub2e4\ub154\uace0, \uc774\ud6c4 2011\ub144 10\uc6d4 \ubcf4\uc2a4\ub2c8\uc544 \ud5e4\ub974\uccb4\uace0\ube44\ub098\uc758 \uad6d\uc81c\ud559\uad50\uc5d0 \uc785\ud559\ud588\uace0 \ubcf4\uc2a4\ub2c8\uc544 \ub2f9\uad6d\uc774 \ucde8\ud559 \ube44\uc790\ub97c \ubc1c\uae09\ud574 \uc8fc\uc5c8\ub2e4. \uadf8\ub294 \ub2f9\ucd08 \ud64d\ucf69\uc758 \ub9ac\ud3ec\ucd98 \uc720\ub098\uc774\ud2f0\ub4dc \uc6d4\ub4dc\uce7c\ub9ac\uc9c0(Li Po Chun United World College)\uc5d0 \ub4f1\ub85d\ud560 \uc608\uc815\uc774\uc5c8\uc9c0\ub9cc \ud64d\ucf69 \ub2f9\uad6d\uc774 \ube44\uc790 \ubc1c\uae09\uc744 \uac70\uc808\ud558\uc5ec 2012\ub144\ubd80\ud130 \uc720\ub7fd\uc5d0\uc11c \ud559\uad50\ub97c \ub2e4\ub2c8\uae30\ub85c \ud558\uc600\uace0, \ubcf4\uc2a4\ub2c8\uc544\ud589\uc744 \uacb0\uc815\ud558\uc600\ub2e4\uace0 \ud55c\ub2e4. 2013\ub144 5\uc6d4 30\uc77c\uc5d0 \uad6d\uc81c\ud559\uad50\ub97c \uc878\uc5c5\ud588\uace0 \uadf8\ud574 2013\ub144 8\uc6d4\uc5d0 \ud30c\ub9ac \uc815\uce58 \ub300\ud559\uc5d0 \uc9c4\ud559, 2016\ub144 \uc878\uc5c5\ud588\ub2e4. 2017\ub144 1\uc6d4\uc758 \uc544\ubc84\uc9c0 \uae40\uc815\ub0a8\uc774 \ubd81\ud55c\uc758 \uc0ac\uc8fc\ub97c \ubc1b\uc740 \uac83\uc73c\ub85c \ucd94\uc815\ub418\ub294 \ubca0\ud2b8\ub0a8 \uc778\ub3c4\ub124\uc2dc\uc544 \uad6d\uc801\uc758 \uc5ec\uc131\ub4e4\uc5d0\uac8c \uc0b4\ud574\ub2f9\ud55c \ud6c4\uc5d0 \ub9dd\uba85\uc744 \ud558\uc600\uc9c0\ub9cc, 2018\ub144 \ud604\uc7ac\uae4c\uc9c0 \uadf8\uc758 \uc18c\uc7ac\ub294 \uba85\ud655\ud558\uac8c \ud655\uc778\ub418\uace0 \uc788\uc9c0 \uc54a\ub2e4.", "answer": "\ud30c\ub9ac \uc815\uce58 \ub300\ud559", "sentence": "\uadf8\ud574 2013\ub144 8\uc6d4\uc5d0 \ud30c\ub9ac \uc815\uce58 \ub300\ud559 \uc5d0 \uc9c4\ud559, 2016\ub144 \uc878\uc5c5\ud588\ub2e4.", "paragraph_sentence": "\uae40\ud55c\uc194\uc740 1995\ub144 6\uc6d4 16\uc77c\uc5d0 \ud3c9\uc591\uc5d0\uc11c \ud0dc\uc5b4\ub0ac\ub2e4. \uc544\ubc84\uc9c0 \uae40\uc815\ub0a8\uc774 \ud6c4\uacc4 \uad6c\ub3c4\uc5d0\uc11c \ubc00\ub9ac\uc790 \uc5ec\ub7ec\ub098\ub77c\ub97c \ub5a0\ub3cc\uc544 \ub2e4\ub154\uace0, \uc774\ud6c4 2011\ub144 10\uc6d4 \ubcf4\uc2a4\ub2c8\uc544 \ud5e4\ub974\uccb4\uace0\ube44\ub098\uc758 \uad6d\uc81c\ud559\uad50\uc5d0 \uc785\ud559\ud588\uace0 \ubcf4\uc2a4\ub2c8\uc544 \ub2f9\uad6d\uc774 \ucde8\ud559 \ube44\uc790\ub97c \ubc1c\uae09\ud574 \uc8fc\uc5c8\ub2e4. \uadf8\ub294 \ub2f9\ucd08 \ud64d\ucf69\uc758 \ub9ac\ud3ec\ucd98 \uc720\ub098\uc774\ud2f0\ub4dc \uc6d4\ub4dc\uce7c\ub9ac\uc9c0(Li Po Chun United World College)\uc5d0 \ub4f1\ub85d\ud560 \uc608\uc815\uc774\uc5c8\uc9c0\ub9cc \ud64d\ucf69 \ub2f9\uad6d\uc774 \ube44\uc790 \ubc1c\uae09\uc744 \uac70\uc808\ud558\uc5ec 2012\ub144\ubd80\ud130 \uc720\ub7fd\uc5d0\uc11c \ud559\uad50\ub97c \ub2e4\ub2c8\uae30\ub85c \ud558\uc600\uace0, \ubcf4\uc2a4\ub2c8\uc544\ud589\uc744 \uacb0\uc815\ud558\uc600\ub2e4\uace0 \ud55c\ub2e4. 2013\ub144 5\uc6d4 30\uc77c\uc5d0 \uad6d\uc81c\ud559\uad50\ub97c \uc878\uc5c5\ud588\uace0 \uadf8\ud574 2013\ub144 8\uc6d4\uc5d0 \ud30c\ub9ac \uc815\uce58 \ub300\ud559 \uc5d0 \uc9c4\ud559, 2016\ub144 \uc878\uc5c5\ud588\ub2e4. 2017\ub144 1\uc6d4\uc758 \uc544\ubc84\uc9c0 \uae40\uc815\ub0a8\uc774 \ubd81\ud55c\uc758 \uc0ac\uc8fc\ub97c \ubc1b\uc740 \uac83\uc73c\ub85c \ucd94\uc815\ub418\ub294 \ubca0\ud2b8\ub0a8 \uc778\ub3c4\ub124\uc2dc\uc544 \uad6d\uc801\uc758 \uc5ec\uc131\ub4e4\uc5d0\uac8c \uc0b4\ud574\ub2f9\ud55c \ud6c4\uc5d0 \ub9dd\uba85\uc744 \ud558\uc600\uc9c0\ub9cc, 2018\ub144 \ud604\uc7ac\uae4c\uc9c0 \uadf8\uc758 \uc18c\uc7ac\ub294 \uba85\ud655\ud558\uac8c \ud655\uc778\ub418\uace0 \uc788\uc9c0 \uc54a\ub2e4.", "paragraph_answer": "\uae40\ud55c\uc194\uc740 1995\ub144 6\uc6d4 16\uc77c\uc5d0 \ud3c9\uc591\uc5d0\uc11c \ud0dc\uc5b4\ub0ac\ub2e4. \uc544\ubc84\uc9c0 \uae40\uc815\ub0a8\uc774 \ud6c4\uacc4 \uad6c\ub3c4\uc5d0\uc11c \ubc00\ub9ac\uc790 \uc5ec\ub7ec\ub098\ub77c\ub97c \ub5a0\ub3cc\uc544 \ub2e4\ub154\uace0, \uc774\ud6c4 2011\ub144 10\uc6d4 \ubcf4\uc2a4\ub2c8\uc544 \ud5e4\ub974\uccb4\uace0\ube44\ub098\uc758 \uad6d\uc81c\ud559\uad50\uc5d0 \uc785\ud559\ud588\uace0 \ubcf4\uc2a4\ub2c8\uc544 \ub2f9\uad6d\uc774 \ucde8\ud559 \ube44\uc790\ub97c \ubc1c\uae09\ud574 \uc8fc\uc5c8\ub2e4. \uadf8\ub294 \ub2f9\ucd08 \ud64d\ucf69\uc758 \ub9ac\ud3ec\ucd98 \uc720\ub098\uc774\ud2f0\ub4dc \uc6d4\ub4dc\uce7c\ub9ac\uc9c0(Li Po Chun United World College)\uc5d0 \ub4f1\ub85d\ud560 \uc608\uc815\uc774\uc5c8\uc9c0\ub9cc \ud64d\ucf69 \ub2f9\uad6d\uc774 \ube44\uc790 \ubc1c\uae09\uc744 \uac70\uc808\ud558\uc5ec 2012\ub144\ubd80\ud130 \uc720\ub7fd\uc5d0\uc11c \ud559\uad50\ub97c \ub2e4\ub2c8\uae30\ub85c \ud558\uc600\uace0, \ubcf4\uc2a4\ub2c8\uc544\ud589\uc744 \uacb0\uc815\ud558\uc600\ub2e4\uace0 \ud55c\ub2e4. 2013\ub144 5\uc6d4 30\uc77c\uc5d0 \uad6d\uc81c\ud559\uad50\ub97c \uc878\uc5c5\ud588\uace0 \uadf8\ud574 2013\ub144 8\uc6d4\uc5d0 \ud30c\ub9ac \uc815\uce58 \ub300\ud559 \uc5d0 \uc9c4\ud559, 2016\ub144 \uc878\uc5c5\ud588\ub2e4. 2017\ub144 1\uc6d4\uc758 \uc544\ubc84\uc9c0 \uae40\uc815\ub0a8\uc774 \ubd81\ud55c\uc758 \uc0ac\uc8fc\ub97c \ubc1b\uc740 \uac83\uc73c\ub85c \ucd94\uc815\ub418\ub294 \ubca0\ud2b8\ub0a8 \uc778\ub3c4\ub124\uc2dc\uc544 \uad6d\uc801\uc758 \uc5ec\uc131\ub4e4\uc5d0\uac8c \uc0b4\ud574\ub2f9\ud55c \ud6c4\uc5d0 \ub9dd\uba85\uc744 \ud558\uc600\uc9c0\ub9cc, 2018\ub144 \ud604\uc7ac\uae4c\uc9c0 \uadf8\uc758 \uc18c\uc7ac\ub294 \uba85\ud655\ud558\uac8c \ud655\uc778\ub418\uace0 \uc788\uc9c0 \uc54a\ub2e4.", "sentence_answer": "\uadf8\ud574 2013\ub144 8\uc6d4\uc5d0 \ud30c\ub9ac \uc815\uce58 \ub300\ud559 \uc5d0 \uc9c4\ud559, 2016\ub144 \uc878\uc5c5\ud588\ub2e4.", "paragraph_id": "6482923-0-1", "paragraph_question": "question: \uae40\ud55c\uc194\uc774 \uc878\uc5c5\ud55c \ub300\ud559 \uba85\uc740?, context: \uae40\ud55c\uc194\uc740 1995\ub144 6\uc6d4 16\uc77c\uc5d0 \ud3c9\uc591\uc5d0\uc11c \ud0dc\uc5b4\ub0ac\ub2e4. \uc544\ubc84\uc9c0 \uae40\uc815\ub0a8\uc774 \ud6c4\uacc4 \uad6c\ub3c4\uc5d0\uc11c \ubc00\ub9ac\uc790 \uc5ec\ub7ec\ub098\ub77c\ub97c \ub5a0\ub3cc\uc544 \ub2e4\ub154\uace0, \uc774\ud6c4 2011\ub144 10\uc6d4 \ubcf4\uc2a4\ub2c8\uc544 \ud5e4\ub974\uccb4\uace0\ube44\ub098\uc758 \uad6d\uc81c\ud559\uad50\uc5d0 \uc785\ud559\ud588\uace0 \ubcf4\uc2a4\ub2c8\uc544 \ub2f9\uad6d\uc774 \ucde8\ud559 \ube44\uc790\ub97c \ubc1c\uae09\ud574 \uc8fc\uc5c8\ub2e4. \uadf8\ub294 \ub2f9\ucd08 \ud64d\ucf69\uc758 \ub9ac\ud3ec\ucd98 \uc720\ub098\uc774\ud2f0\ub4dc \uc6d4\ub4dc\uce7c\ub9ac\uc9c0(Li Po Chun United World College)\uc5d0 \ub4f1\ub85d\ud560 \uc608\uc815\uc774\uc5c8\uc9c0\ub9cc \ud64d\ucf69 \ub2f9\uad6d\uc774 \ube44\uc790 \ubc1c\uae09\uc744 \uac70\uc808\ud558\uc5ec 2012\ub144\ubd80\ud130 \uc720\ub7fd\uc5d0\uc11c \ud559\uad50\ub97c \ub2e4\ub2c8\uae30\ub85c \ud558\uc600\uace0, \ubcf4\uc2a4\ub2c8\uc544\ud589\uc744 \uacb0\uc815\ud558\uc600\ub2e4\uace0 \ud55c\ub2e4. 2013\ub144 5\uc6d4 30\uc77c\uc5d0 \uad6d\uc81c\ud559\uad50\ub97c \uc878\uc5c5\ud588\uace0 \uadf8\ud574 2013\ub144 8\uc6d4\uc5d0 \ud30c\ub9ac \uc815\uce58 \ub300\ud559\uc5d0 \uc9c4\ud559, 2016\ub144 \uc878\uc5c5\ud588\ub2e4. 2017\ub144 1\uc6d4\uc758 \uc544\ubc84\uc9c0 \uae40\uc815\ub0a8\uc774 \ubd81\ud55c\uc758 \uc0ac\uc8fc\ub97c \ubc1b\uc740 \uac83\uc73c\ub85c \ucd94\uc815\ub418\ub294 \ubca0\ud2b8\ub0a8 \uc778\ub3c4\ub124\uc2dc\uc544 \uad6d\uc801\uc758 \uc5ec\uc131\ub4e4\uc5d0\uac8c \uc0b4\ud574\ub2f9\ud55c \ud6c4\uc5d0 \ub9dd\uba85\uc744 \ud558\uc600\uc9c0\ub9cc, 2018\ub144 \ud604\uc7ac\uae4c\uc9c0 \uadf8\uc758 \uc18c\uc7ac\ub294 \uba85\ud655\ud558\uac8c \ud655\uc778\ub418\uace0 \uc788\uc9c0 \uc54a\ub2e4."} {"question": "\ubd80\ud65c\uc808\uc740 \ub204\uad6c\uc758 \ubd80\ud65c\uc744 \uae30\ub150\ud558\ub294 \ub0a0\uc778\uac00?", "paragraph": "\ubd80\ud65c\uc808\uc740 \uc608\uc218\uac00 \uc2ed\uc790\uac00\uc5d0 \ubabb\ubc15\ud600 \uc8fd\uc740 \uc9c0 3\uc77c \uc9f8 \ub418\ub294 \ub0a0, \uadf8\uac00 \ub2e4\uc2dc \uc0b4\uc544\ub0ac\ub2e4\ub294 \uac83\uc744 \uae30\ub150\ud558\ub294 \ub0a0\uc774\ub2e4. \uae30\ub3c5\uad50\uc758 \uacbd\uc804\uc778 \uc2e0\uc57d\uc131\uc11c\uc758 \ubcf5\uc74c\uc11c \uc911 \ub9c8\uac00(\ub9c8\ub974\ucf54), \ub9c8\ud0dc(\ub9c8\ud0dc\uc624), \ub204\uac00(\ub8e8\uac00), \uc694\ud55c \ubcf5\uc74c\uc11c\uc758 \uae30\ub85d\uc744 \ub530\ub978\ub2e4. \uac01\ubcf5\uc74c\uc11c\uc758 \uae30\ub85d\uc5d0\ub294 \uc57d\uac04\uc774 \ucc28\uc774\uac00 \uc788\uc9c0\ub9cc \uc804\uccb4\ub97c \ud1b5\ud569\ud558\uc5ec \ubd80\ud65c\uc808\uc758 \uc804\uc2b9\uc73c\ub85c \uc774\ud574\ud558\uace0, \uadf8 \uc804\uc2b9\uc5d0 \ub530\ub77c \ubd80\ud65c\uc808 \uc608\ubc30\uc640 \uc608\uc2dd, \ud589\uc0ac \ub4f1\uc774 \uc0dd\uaca8\ub0ac\ub2e4.\ubcf5\uc74c\uc11c\uac00 \uc791\uc131\ub41c 1\uc138\uae30\uc640 2\uc138\uae30\uc778 \ucd08\ub300\uad50\ud68c\uc5d0\uc11c\ub3c4 \uc608\uc218\uc758 \ubd80\ud65c\uc5d0 \ub300\ud55c \uac15\ud55c \uc2e0\ub150\uc744 \uc9c0\ub154\uc73c\uba70, \ubd80\ud65c\uc808\uc740 \uad6c\uc57d\uc131\uc11c\ub97c \ud1b5\ud55c \uc804\uc2b9\uacfc \uc608\uc218\uac00 \ubcf5\uc74c\uc73c\ub85c \uc804\ud55c \ud558\ub298\ub098\ub77c\uc640 \uc5d0\uc138\ub124\ud30c\uac00 \uc911\uc694\uc2dc\ud588\ub358 \ubab8\uc758 \ubd80\ud65c\uacfc \ubb35\uc2dc\ub860\uacfc \ubc18\uc81c\uad6d\uc8fc\uc758 \uc0ac\uc0c1\uacfc \ud3c9\ud654\uc8fc\uc758 \uc0ac\uc0c1\ub4e4\uc774 \ubcf5\ud569\uc801\uc73c\ub85c \uad6c\uc131\ub41c \uc0ac\uac74\uc73c\ub85c \ubcf8\ub2e4.", "answer": "\uc608\uc218", "sentence": "\ubd80\ud65c\uc808\uc740 \uc608\uc218 \uac00 \uc2ed\uc790\uac00\uc5d0 \ubabb\ubc15\ud600 \uc8fd\uc740 \uc9c0 3\uc77c \uc9f8 \ub418\ub294 \ub0a0, \uadf8\uac00 \ub2e4\uc2dc \uc0b4\uc544\ub0ac\ub2e4\ub294 \uac83\uc744 \uae30\ub150\ud558\ub294 \ub0a0\uc774\ub2e4.", "paragraph_sentence": " \ubd80\ud65c\uc808\uc740 \uc608\uc218 \uac00 \uc2ed\uc790\uac00\uc5d0 \ubabb\ubc15\ud600 \uc8fd\uc740 \uc9c0 3\uc77c \uc9f8 \ub418\ub294 \ub0a0, \uadf8\uac00 \ub2e4\uc2dc \uc0b4\uc544\ub0ac\ub2e4\ub294 \uac83\uc744 \uae30\ub150\ud558\ub294 \ub0a0\uc774\ub2e4. \uae30\ub3c5\uad50\uc758 \uacbd\uc804\uc778 \uc2e0\uc57d\uc131\uc11c\uc758 \ubcf5\uc74c\uc11c \uc911 \ub9c8\uac00(\ub9c8\ub974\ucf54), \ub9c8\ud0dc(\ub9c8\ud0dc\uc624), \ub204\uac00(\ub8e8\uac00), \uc694\ud55c \ubcf5\uc74c\uc11c\uc758 \uae30\ub85d\uc744 \ub530\ub978\ub2e4. \uac01\ubcf5\uc74c\uc11c\uc758 \uae30\ub85d\uc5d0\ub294 \uc57d\uac04\uc774 \ucc28\uc774\uac00 \uc788\uc9c0\ub9cc \uc804\uccb4\ub97c \ud1b5\ud569\ud558\uc5ec \ubd80\ud65c\uc808\uc758 \uc804\uc2b9\uc73c\ub85c \uc774\ud574\ud558\uace0, \uadf8 \uc804\uc2b9\uc5d0 \ub530\ub77c \ubd80\ud65c\uc808 \uc608\ubc30\uc640 \uc608\uc2dd, \ud589\uc0ac \ub4f1\uc774 \uc0dd\uaca8\ub0ac\ub2e4.\ubcf5\uc74c\uc11c\uac00 \uc791\uc131\ub41c 1\uc138\uae30\uc640 2\uc138\uae30\uc778 \ucd08\ub300\uad50\ud68c\uc5d0\uc11c\ub3c4 \uc608\uc218\uc758 \ubd80\ud65c\uc5d0 \ub300\ud55c \uac15\ud55c \uc2e0\ub150\uc744 \uc9c0\ub154\uc73c\uba70, \ubd80\ud65c\uc808\uc740 \uad6c\uc57d\uc131\uc11c\ub97c \ud1b5\ud55c \uc804\uc2b9\uacfc \uc608\uc218\uac00 \ubcf5\uc74c\uc73c\ub85c \uc804\ud55c \ud558\ub298\ub098\ub77c\uc640 \uc5d0\uc138\ub124\ud30c\uac00 \uc911\uc694\uc2dc\ud588\ub358 \ubab8\uc758 \ubd80\ud65c\uacfc \ubb35\uc2dc\ub860\uacfc \ubc18\uc81c\uad6d\uc8fc\uc758 \uc0ac\uc0c1\uacfc \ud3c9\ud654\uc8fc\uc758 \uc0ac\uc0c1\ub4e4\uc774 \ubcf5\ud569\uc801\uc73c\ub85c \uad6c\uc131\ub41c \uc0ac\uac74\uc73c\ub85c \ubcf8\ub2e4.", "paragraph_answer": "\ubd80\ud65c\uc808\uc740 \uc608\uc218 \uac00 \uc2ed\uc790\uac00\uc5d0 \ubabb\ubc15\ud600 \uc8fd\uc740 \uc9c0 3\uc77c \uc9f8 \ub418\ub294 \ub0a0, \uadf8\uac00 \ub2e4\uc2dc \uc0b4\uc544\ub0ac\ub2e4\ub294 \uac83\uc744 \uae30\ub150\ud558\ub294 \ub0a0\uc774\ub2e4. \uae30\ub3c5\uad50\uc758 \uacbd\uc804\uc778 \uc2e0\uc57d\uc131\uc11c\uc758 \ubcf5\uc74c\uc11c \uc911 \ub9c8\uac00(\ub9c8\ub974\ucf54), \ub9c8\ud0dc(\ub9c8\ud0dc\uc624), \ub204\uac00(\ub8e8\uac00), \uc694\ud55c \ubcf5\uc74c\uc11c\uc758 \uae30\ub85d\uc744 \ub530\ub978\ub2e4. \uac01\ubcf5\uc74c\uc11c\uc758 \uae30\ub85d\uc5d0\ub294 \uc57d\uac04\uc774 \ucc28\uc774\uac00 \uc788\uc9c0\ub9cc \uc804\uccb4\ub97c \ud1b5\ud569\ud558\uc5ec \ubd80\ud65c\uc808\uc758 \uc804\uc2b9\uc73c\ub85c \uc774\ud574\ud558\uace0, \uadf8 \uc804\uc2b9\uc5d0 \ub530\ub77c \ubd80\ud65c\uc808 \uc608\ubc30\uc640 \uc608\uc2dd, \ud589\uc0ac \ub4f1\uc774 \uc0dd\uaca8\ub0ac\ub2e4.\ubcf5\uc74c\uc11c\uac00 \uc791\uc131\ub41c 1\uc138\uae30\uc640 2\uc138\uae30\uc778 \ucd08\ub300\uad50\ud68c\uc5d0\uc11c\ub3c4 \uc608\uc218\uc758 \ubd80\ud65c\uc5d0 \ub300\ud55c \uac15\ud55c \uc2e0\ub150\uc744 \uc9c0\ub154\uc73c\uba70, \ubd80\ud65c\uc808\uc740 \uad6c\uc57d\uc131\uc11c\ub97c \ud1b5\ud55c \uc804\uc2b9\uacfc \uc608\uc218\uac00 \ubcf5\uc74c\uc73c\ub85c \uc804\ud55c \ud558\ub298\ub098\ub77c\uc640 \uc5d0\uc138\ub124\ud30c\uac00 \uc911\uc694\uc2dc\ud588\ub358 \ubab8\uc758 \ubd80\ud65c\uacfc \ubb35\uc2dc\ub860\uacfc \ubc18\uc81c\uad6d\uc8fc\uc758 \uc0ac\uc0c1\uacfc \ud3c9\ud654\uc8fc\uc758 \uc0ac\uc0c1\ub4e4\uc774 \ubcf5\ud569\uc801\uc73c\ub85c \uad6c\uc131\ub41c \uc0ac\uac74\uc73c\ub85c \ubcf8\ub2e4.", "sentence_answer": "\ubd80\ud65c\uc808\uc740 \uc608\uc218 \uac00 \uc2ed\uc790\uac00\uc5d0 \ubabb\ubc15\ud600 \uc8fd\uc740 \uc9c0 3\uc77c \uc9f8 \ub418\ub294 \ub0a0, \uadf8\uac00 \ub2e4\uc2dc \uc0b4\uc544\ub0ac\ub2e4\ub294 \uac83\uc744 \uae30\ub150\ud558\ub294 \ub0a0\uc774\ub2e4.", "paragraph_id": "6509111-0-1", "paragraph_question": "question: \ubd80\ud65c\uc808\uc740 \ub204\uad6c\uc758 \ubd80\ud65c\uc744 \uae30\ub150\ud558\ub294 \ub0a0\uc778\uac00?, context: \ubd80\ud65c\uc808\uc740 \uc608\uc218\uac00 \uc2ed\uc790\uac00\uc5d0 \ubabb\ubc15\ud600 \uc8fd\uc740 \uc9c0 3\uc77c \uc9f8 \ub418\ub294 \ub0a0, \uadf8\uac00 \ub2e4\uc2dc \uc0b4\uc544\ub0ac\ub2e4\ub294 \uac83\uc744 \uae30\ub150\ud558\ub294 \ub0a0\uc774\ub2e4. \uae30\ub3c5\uad50\uc758 \uacbd\uc804\uc778 \uc2e0\uc57d\uc131\uc11c\uc758 \ubcf5\uc74c\uc11c \uc911 \ub9c8\uac00(\ub9c8\ub974\ucf54), \ub9c8\ud0dc(\ub9c8\ud0dc\uc624), \ub204\uac00(\ub8e8\uac00), \uc694\ud55c \ubcf5\uc74c\uc11c\uc758 \uae30\ub85d\uc744 \ub530\ub978\ub2e4. \uac01\ubcf5\uc74c\uc11c\uc758 \uae30\ub85d\uc5d0\ub294 \uc57d\uac04\uc774 \ucc28\uc774\uac00 \uc788\uc9c0\ub9cc \uc804\uccb4\ub97c \ud1b5\ud569\ud558\uc5ec \ubd80\ud65c\uc808\uc758 \uc804\uc2b9\uc73c\ub85c \uc774\ud574\ud558\uace0, \uadf8 \uc804\uc2b9\uc5d0 \ub530\ub77c \ubd80\ud65c\uc808 \uc608\ubc30\uc640 \uc608\uc2dd, \ud589\uc0ac \ub4f1\uc774 \uc0dd\uaca8\ub0ac\ub2e4.\ubcf5\uc74c\uc11c\uac00 \uc791\uc131\ub41c 1\uc138\uae30\uc640 2\uc138\uae30\uc778 \ucd08\ub300\uad50\ud68c\uc5d0\uc11c\ub3c4 \uc608\uc218\uc758 \ubd80\ud65c\uc5d0 \ub300\ud55c \uac15\ud55c \uc2e0\ub150\uc744 \uc9c0\ub154\uc73c\uba70, \ubd80\ud65c\uc808\uc740 \uad6c\uc57d\uc131\uc11c\ub97c \ud1b5\ud55c \uc804\uc2b9\uacfc \uc608\uc218\uac00 \ubcf5\uc74c\uc73c\ub85c \uc804\ud55c \ud558\ub298\ub098\ub77c\uc640 \uc5d0\uc138\ub124\ud30c\uac00 \uc911\uc694\uc2dc\ud588\ub358 \ubab8\uc758 \ubd80\ud65c\uacfc \ubb35\uc2dc\ub860\uacfc \ubc18\uc81c\uad6d\uc8fc\uc758 \uc0ac\uc0c1\uacfc \ud3c9\ud654\uc8fc\uc758 \uc0ac\uc0c1\ub4e4\uc774 \ubcf5\ud569\uc801\uc73c\ub85c \uad6c\uc131\ub41c \uc0ac\uac74\uc73c\ub85c \ubcf8\ub2e4."} {"question": "351\uacc4\uac00 \ud655\ub9bd\ud55c \uc138\uc774\ubd80 \ucca0\ub3c4 \uc18c\uc18d \uc804\ub3d9\ucc28\uc758 \uae30\ubcf8 \ub514\uc790\uc778\uc740?", "paragraph": "\uadf8\ub7ec\ub098, 1954\ub144\uc5d0 \uc911\uad6d\uc2dd\uc774\ub098 \uc61b\ub0a0 \ucc28\uccb4\uc5d0 \ub300\ud55c \uc0c8\ub85c\uc6b4 \uc81c\uc791\uc774 \uc2dc\uc791\ub418\uc5c8\ub2e4. \uc774 \ub54c, \ub4f1\uc7a5\ud55c 351\uacc4(\ub4f1\uc7a5 \ub2f9\uc2dc 501\uacc4)\ub294 \uc9c0\uae08\ubd80\ud130 \uc7a5\uae30\uac04\uc5d0 \uac78\uce58\ub294 \uc138\uc774\ubd80 \ucca0\ub3c4 \uc18c\uc18d \uc804\ub3d9\ucc28\uc758 \uae30\ubcf8 \ub514\uc790\uc778\uc778 \u201c\uc1fc\ub09c \ub514\uc790\uc778(\u6e58\u5357\u30c7\u30b6\u30a4\u30f3)\u201d\uc744 \ud655\ub9bd\ud588\ub2e4. \uc774 \ub54c\uc758 \uc138\uc774\ubd80 \ucca0\ub3c4 \uad6c\uac04\uc744 \uc6b4\ud589\ud558\ub294 \uc804\ucca0\uc744 \ub9d0\ud55c\ub2e4\uba74, \uc7a1\ub2e4\ud55c \ud615\ud0dc\uc758 \uc804\ub3d9\ucc28\ub97c \uc5f0\uacb0\ud558\uc5ec \ubb34\uc0ac\uc2dc\ub178\ub2e4\uc774\uc9c0(\u6b66\u8535\u91ce\u53f0\u5730)\ub97c \ub2e4\ub2c8\ub294 \uac83\uc73c\ub85c \ubb18\uc0ac\ub418\uc5c8\ub2e4. 2\ucc28\ub300\uc804 \ud6c4\uc758 \uc138\uc774\ubd80 \ucca0\ub3c4\uc758 \ubc29\uce68\uc740 \u201c\uc9c8\ubcf4\ub2e4\ub294 \uc591\u201d\uc73c\ub85c, \uc774\ub294 \ucc28\ub7c9\uc758 \uace0\uc18d\ud654\ub97c \uc6b0\uc120\uc73c\ub85c \ub0b4\uc138\uc6cc\uc11c 1950\ub144\ub300 \uc911\ubc18\uc5d0 \ub4f1\uc7a5\ud55c \uc18c\uc704 \uace0\uc131\ub2a5\ucc28\uc758 \ub3c4\uc785\uc740 \ud558\uc9c0 \uc54a\uace0, \ub2e4\ub978 \uacc4\uc5f4\uacfc\uc758 \ubcd1\uacb0\uc744 \uace0\ub824\ud558\uc5ec \uc131\ub2a5\uc758 \ub2e8\uc77c\ud654\ub97c \ucd94\uc9c4\ud558\ub294 \uac83\ubd80\ud130 \uc77c\uad00\ud558\uc5ec \uc774\uc804\uc758 \u201c\ub9e4\ub2ec\ub9ac\uace0 \uac78\uce58\ub294 \uad6c\ub3d9\ubc29\uc2dd(\u540a\u308a\u639b\u3051\u99c6\u52d5\u65b9\u5f0f)\"\uc744 \uacc4\uc18d \ud558\uace0 \uc788\ub358 \ub2e4\ub978 \ud68c\uc0ac\uc640\ub294 \ub2e4\ub978 \ubc29\uc2dd\uc774\uc5c8\ub2e4. \uadf8 \ud6c4 \ub4f1\uc7a5\ud55c \uc138\uc774\ubd80 \ucca0\ub3c4 \uccab \uace0\uc131\ub2a5\ucc28\ub77c\uace0 \ub9d0\ud560 \uc218 \uc788\ub294 \uce74\ub974\ub2e8 \uad6c\ub3d9 \ubc29\uc2dd\uc758 601\uacc4, 701\uacc4 \uc804\ucca0\ub85c\ub3c4, \uc774\uc804\uc758 \uad6c\ub3d9 \ubc29\uc2dd\uc744 \uc9c0\ub2cc \ucc28\ub7c9\uacfc\uc758 \ubcd1\uacb0\uc744 \uc804\uc81c\ub85c, \uae30\ub2a5\uc801\uc73c\ub85c\ub294 \ub3d9\ub825 \uc804\ub2ec \ubc29\uc2dd\uc744 \ub192\uc778 \ub300\uccb4\ud488\uc77c \ubfd0\uc774\uc5c8\uc73c\uba70, \uc774\uc804\uc758 \ubc29\uc2dd\uc744 \uc9c0\ub2cc \ucc28\ub7c9\uacfc \ubcd1\uacb0 \ud558\ub294 \uac83\uc73c\ub85c \uc778\ud558\uc5ec \uce74\ub974\ub2e8 \uad6c\ub3d9 \ubc29\uc2dd \ubcf8\ub798\uc758 \uc131\ub2a5\uc740 \ubc1c\ud718 \ub418\uc5b4 \uc788\uc9c0 \uc54a\uc558\ub2e4. \uc804\ub3d9\ucc28\uc758 \ucc28\uccb4\ub97c \uc9c0\ud0f1\ud558\ub294 \ubd80\ubd84\uc740 \uc0c8\ub86d\uac8c \uc81c\uc791\ub418\uc5c8\uc9c0\ub9cc, \ubd80\uc218\ucc28\uc758 \ucc28\uccb4\ub97c \uc9c0\ud0f1\ud558\ub294 \ubd80\ubd84\uc740 \uc774\uc804\uc758 \uade0\ud615\ub300\ub4e4\ubcf4\uc2dd\uc744 \uc9c0\ub2cc TR11\uacc4\uc758 \uac1c\uc870\ud488\uc73c\ub85c \uad6d\ucca0\ub85c\ubd80\ud130 \uc591\ub3c4\ub41c \uac83\uc774\uc5c8\ub2e4. \uc774\uc804 \ucc28\ub7c9\uacfc \uac19\uc774 \uc4f0\ub294 \uac83\uc744 \uace0\ub824\ud558\uc9c0 \uc54a\uace0, \uc804\uae30 \uc81c\ub3d9\uc774 \uac00\ub2a5\ud558\uac8c \ud55c \uc9c4\uc815\ud55c \uc758\ubbf8\ub85c\uc758 \uace0\uc131\ub2a5\ucc28\uac00 \ub3c4\uc785\ub41c \uac83\uc740, \uc138\uc774\ubd80 \uc9c0\uce58\ubd80 \uc120\uc774 \uac1c\ud1b5\uc744 \uc55e\ub454 \ub54c\uc600\ub2e4.", "answer": "\uc1fc\ub09c \ub514\uc790\uc778", "sentence": "\uc774 \ub54c, \ub4f1\uc7a5\ud55c 351\uacc4(\ub4f1\uc7a5 \ub2f9\uc2dc 501\uacc4)\ub294 \uc9c0\uae08\ubd80\ud130 \uc7a5\uae30\uac04\uc5d0 \uac78\uce58\ub294 \uc138\uc774\ubd80 \ucca0\ub3c4 \uc18c\uc18d \uc804\ub3d9\ucc28\uc758 \uae30\ubcf8 \ub514\uc790\uc778\uc778 \u201c \uc1fc\ub09c \ub514\uc790\uc778 (\u6e58\u5357\u30c7\u30b6\u30a4\u30f3)\u201d\uc744 \ud655\ub9bd\ud588\ub2e4.", "paragraph_sentence": "\uadf8\ub7ec\ub098, 1954\ub144\uc5d0 \uc911\uad6d\uc2dd\uc774\ub098 \uc61b\ub0a0 \ucc28\uccb4\uc5d0 \ub300\ud55c \uc0c8\ub85c\uc6b4 \uc81c\uc791\uc774 \uc2dc\uc791\ub418\uc5c8\ub2e4. \uc774 \ub54c, \ub4f1\uc7a5\ud55c 351\uacc4(\ub4f1\uc7a5 \ub2f9\uc2dc 501\uacc4)\ub294 \uc9c0\uae08\ubd80\ud130 \uc7a5\uae30\uac04\uc5d0 \uac78\uce58\ub294 \uc138\uc774\ubd80 \ucca0\ub3c4 \uc18c\uc18d \uc804\ub3d9\ucc28\uc758 \uae30\ubcf8 \ub514\uc790\uc778\uc778 \u201c \uc1fc\ub09c \ub514\uc790\uc778 (\u6e58\u5357\u30c7\u30b6\u30a4\u30f3)\u201d\uc744 \ud655\ub9bd\ud588\ub2e4. \uc774 \ub54c\uc758 \uc138\uc774\ubd80 \ucca0\ub3c4 \uad6c\uac04\uc744 \uc6b4\ud589\ud558\ub294 \uc804\ucca0\uc744 \ub9d0\ud55c\ub2e4\uba74, \uc7a1\ub2e4\ud55c \ud615\ud0dc\uc758 \uc804\ub3d9\ucc28\ub97c \uc5f0\uacb0\ud558\uc5ec \ubb34\uc0ac\uc2dc\ub178\ub2e4\uc774\uc9c0(\u6b66\u8535\u91ce\u53f0\u5730)\ub97c \ub2e4\ub2c8\ub294 \uac83\uc73c\ub85c \ubb18\uc0ac\ub418\uc5c8\ub2e4. 2\ucc28\ub300\uc804 \ud6c4\uc758 \uc138\uc774\ubd80 \ucca0\ub3c4\uc758 \ubc29\uce68\uc740 \u201c\uc9c8\ubcf4\ub2e4\ub294 \uc591\u201d\uc73c\ub85c, \uc774\ub294 \ucc28\ub7c9\uc758 \uace0\uc18d\ud654\ub97c \uc6b0\uc120\uc73c\ub85c \ub0b4\uc138\uc6cc\uc11c 1950\ub144\ub300 \uc911\ubc18\uc5d0 \ub4f1\uc7a5\ud55c \uc18c\uc704 \uace0\uc131\ub2a5\ucc28\uc758 \ub3c4\uc785\uc740 \ud558\uc9c0 \uc54a\uace0, \ub2e4\ub978 \uacc4\uc5f4\uacfc\uc758 \ubcd1\uacb0\uc744 \uace0\ub824\ud558\uc5ec \uc131\ub2a5\uc758 \ub2e8\uc77c\ud654\ub97c \ucd94\uc9c4\ud558\ub294 \uac83\ubd80\ud130 \uc77c\uad00\ud558\uc5ec \uc774\uc804\uc758 \u201c\ub9e4\ub2ec\ub9ac\uace0 \uac78\uce58\ub294 \uad6c\ub3d9\ubc29\uc2dd(\u540a\u308a\u639b\u3051\u99c6\u52d5\u65b9\u5f0f)\"\uc744 \uacc4\uc18d \ud558\uace0 \uc788\ub358 \ub2e4\ub978 \ud68c\uc0ac\uc640\ub294 \ub2e4\ub978 \ubc29\uc2dd\uc774\uc5c8\ub2e4. \uadf8 \ud6c4 \ub4f1\uc7a5\ud55c \uc138\uc774\ubd80 \ucca0\ub3c4 \uccab \uace0\uc131\ub2a5\ucc28\ub77c\uace0 \ub9d0\ud560 \uc218 \uc788\ub294 \uce74\ub974\ub2e8 \uad6c\ub3d9 \ubc29\uc2dd\uc758 601\uacc4, 701\uacc4 \uc804\ucca0\ub85c\ub3c4, \uc774\uc804\uc758 \uad6c\ub3d9 \ubc29\uc2dd\uc744 \uc9c0\ub2cc \ucc28\ub7c9\uacfc\uc758 \ubcd1\uacb0\uc744 \uc804\uc81c\ub85c, \uae30\ub2a5\uc801\uc73c\ub85c\ub294 \ub3d9\ub825 \uc804\ub2ec \ubc29\uc2dd\uc744 \ub192\uc778 \ub300\uccb4\ud488\uc77c \ubfd0\uc774\uc5c8\uc73c\uba70, \uc774\uc804\uc758 \ubc29\uc2dd\uc744 \uc9c0\ub2cc \ucc28\ub7c9\uacfc \ubcd1\uacb0 \ud558\ub294 \uac83\uc73c\ub85c \uc778\ud558\uc5ec \uce74\ub974\ub2e8 \uad6c\ub3d9 \ubc29\uc2dd \ubcf8\ub798\uc758 \uc131\ub2a5\uc740 \ubc1c\ud718 \ub418\uc5b4 \uc788\uc9c0 \uc54a\uc558\ub2e4. \uc804\ub3d9\ucc28\uc758 \ucc28\uccb4\ub97c \uc9c0\ud0f1\ud558\ub294 \ubd80\ubd84\uc740 \uc0c8\ub86d\uac8c \uc81c\uc791\ub418\uc5c8\uc9c0\ub9cc, \ubd80\uc218\ucc28\uc758 \ucc28\uccb4\ub97c \uc9c0\ud0f1\ud558\ub294 \ubd80\ubd84\uc740 \uc774\uc804\uc758 \uade0\ud615\ub300\ub4e4\ubcf4\uc2dd\uc744 \uc9c0\ub2cc TR11\uacc4\uc758 \uac1c\uc870\ud488\uc73c\ub85c \uad6d\ucca0\ub85c\ubd80\ud130 \uc591\ub3c4\ub41c \uac83\uc774\uc5c8\ub2e4. \uc774\uc804 \ucc28\ub7c9\uacfc \uac19\uc774 \uc4f0\ub294 \uac83\uc744 \uace0\ub824\ud558\uc9c0 \uc54a\uace0, \uc804\uae30 \uc81c\ub3d9\uc774 \uac00\ub2a5\ud558\uac8c \ud55c \uc9c4\uc815\ud55c \uc758\ubbf8\ub85c\uc758 \uace0\uc131\ub2a5\ucc28\uac00 \ub3c4\uc785\ub41c \uac83\uc740, \uc138\uc774\ubd80 \uc9c0\uce58\ubd80 \uc120\uc774 \uac1c\ud1b5\uc744 \uc55e\ub454 \ub54c\uc600\ub2e4.", "paragraph_answer": "\uadf8\ub7ec\ub098, 1954\ub144\uc5d0 \uc911\uad6d\uc2dd\uc774\ub098 \uc61b\ub0a0 \ucc28\uccb4\uc5d0 \ub300\ud55c \uc0c8\ub85c\uc6b4 \uc81c\uc791\uc774 \uc2dc\uc791\ub418\uc5c8\ub2e4. \uc774 \ub54c, \ub4f1\uc7a5\ud55c 351\uacc4(\ub4f1\uc7a5 \ub2f9\uc2dc 501\uacc4)\ub294 \uc9c0\uae08\ubd80\ud130 \uc7a5\uae30\uac04\uc5d0 \uac78\uce58\ub294 \uc138\uc774\ubd80 \ucca0\ub3c4 \uc18c\uc18d \uc804\ub3d9\ucc28\uc758 \uae30\ubcf8 \ub514\uc790\uc778\uc778 \u201c \uc1fc\ub09c \ub514\uc790\uc778 (\u6e58\u5357\u30c7\u30b6\u30a4\u30f3)\u201d\uc744 \ud655\ub9bd\ud588\ub2e4. \uc774 \ub54c\uc758 \uc138\uc774\ubd80 \ucca0\ub3c4 \uad6c\uac04\uc744 \uc6b4\ud589\ud558\ub294 \uc804\ucca0\uc744 \ub9d0\ud55c\ub2e4\uba74, \uc7a1\ub2e4\ud55c \ud615\ud0dc\uc758 \uc804\ub3d9\ucc28\ub97c \uc5f0\uacb0\ud558\uc5ec \ubb34\uc0ac\uc2dc\ub178\ub2e4\uc774\uc9c0(\u6b66\u8535\u91ce\u53f0\u5730)\ub97c \ub2e4\ub2c8\ub294 \uac83\uc73c\ub85c \ubb18\uc0ac\ub418\uc5c8\ub2e4. 2\ucc28\ub300\uc804 \ud6c4\uc758 \uc138\uc774\ubd80 \ucca0\ub3c4\uc758 \ubc29\uce68\uc740 \u201c\uc9c8\ubcf4\ub2e4\ub294 \uc591\u201d\uc73c\ub85c, \uc774\ub294 \ucc28\ub7c9\uc758 \uace0\uc18d\ud654\ub97c \uc6b0\uc120\uc73c\ub85c \ub0b4\uc138\uc6cc\uc11c 1950\ub144\ub300 \uc911\ubc18\uc5d0 \ub4f1\uc7a5\ud55c \uc18c\uc704 \uace0\uc131\ub2a5\ucc28\uc758 \ub3c4\uc785\uc740 \ud558\uc9c0 \uc54a\uace0, \ub2e4\ub978 \uacc4\uc5f4\uacfc\uc758 \ubcd1\uacb0\uc744 \uace0\ub824\ud558\uc5ec \uc131\ub2a5\uc758 \ub2e8\uc77c\ud654\ub97c \ucd94\uc9c4\ud558\ub294 \uac83\ubd80\ud130 \uc77c\uad00\ud558\uc5ec \uc774\uc804\uc758 \u201c\ub9e4\ub2ec\ub9ac\uace0 \uac78\uce58\ub294 \uad6c\ub3d9\ubc29\uc2dd(\u540a\u308a\u639b\u3051\u99c6\u52d5\u65b9\u5f0f)\"\uc744 \uacc4\uc18d \ud558\uace0 \uc788\ub358 \ub2e4\ub978 \ud68c\uc0ac\uc640\ub294 \ub2e4\ub978 \ubc29\uc2dd\uc774\uc5c8\ub2e4. \uadf8 \ud6c4 \ub4f1\uc7a5\ud55c \uc138\uc774\ubd80 \ucca0\ub3c4 \uccab \uace0\uc131\ub2a5\ucc28\ub77c\uace0 \ub9d0\ud560 \uc218 \uc788\ub294 \uce74\ub974\ub2e8 \uad6c\ub3d9 \ubc29\uc2dd\uc758 601\uacc4, 701\uacc4 \uc804\ucca0\ub85c\ub3c4, \uc774\uc804\uc758 \uad6c\ub3d9 \ubc29\uc2dd\uc744 \uc9c0\ub2cc \ucc28\ub7c9\uacfc\uc758 \ubcd1\uacb0\uc744 \uc804\uc81c\ub85c, \uae30\ub2a5\uc801\uc73c\ub85c\ub294 \ub3d9\ub825 \uc804\ub2ec \ubc29\uc2dd\uc744 \ub192\uc778 \ub300\uccb4\ud488\uc77c \ubfd0\uc774\uc5c8\uc73c\uba70, \uc774\uc804\uc758 \ubc29\uc2dd\uc744 \uc9c0\ub2cc \ucc28\ub7c9\uacfc \ubcd1\uacb0 \ud558\ub294 \uac83\uc73c\ub85c \uc778\ud558\uc5ec \uce74\ub974\ub2e8 \uad6c\ub3d9 \ubc29\uc2dd \ubcf8\ub798\uc758 \uc131\ub2a5\uc740 \ubc1c\ud718 \ub418\uc5b4 \uc788\uc9c0 \uc54a\uc558\ub2e4. \uc804\ub3d9\ucc28\uc758 \ucc28\uccb4\ub97c \uc9c0\ud0f1\ud558\ub294 \ubd80\ubd84\uc740 \uc0c8\ub86d\uac8c \uc81c\uc791\ub418\uc5c8\uc9c0\ub9cc, \ubd80\uc218\ucc28\uc758 \ucc28\uccb4\ub97c \uc9c0\ud0f1\ud558\ub294 \ubd80\ubd84\uc740 \uc774\uc804\uc758 \uade0\ud615\ub300\ub4e4\ubcf4\uc2dd\uc744 \uc9c0\ub2cc TR11\uacc4\uc758 \uac1c\uc870\ud488\uc73c\ub85c \uad6d\ucca0\ub85c\ubd80\ud130 \uc591\ub3c4\ub41c \uac83\uc774\uc5c8\ub2e4. \uc774\uc804 \ucc28\ub7c9\uacfc \uac19\uc774 \uc4f0\ub294 \uac83\uc744 \uace0\ub824\ud558\uc9c0 \uc54a\uace0, \uc804\uae30 \uc81c\ub3d9\uc774 \uac00\ub2a5\ud558\uac8c \ud55c \uc9c4\uc815\ud55c \uc758\ubbf8\ub85c\uc758 \uace0\uc131\ub2a5\ucc28\uac00 \ub3c4\uc785\ub41c \uac83\uc740, \uc138\uc774\ubd80 \uc9c0\uce58\ubd80 \uc120\uc774 \uac1c\ud1b5\uc744 \uc55e\ub454 \ub54c\uc600\ub2e4.", "sentence_answer": "\uc774 \ub54c, \ub4f1\uc7a5\ud55c 351\uacc4(\ub4f1\uc7a5 \ub2f9\uc2dc 501\uacc4)\ub294 \uc9c0\uae08\ubd80\ud130 \uc7a5\uae30\uac04\uc5d0 \uac78\uce58\ub294 \uc138\uc774\ubd80 \ucca0\ub3c4 \uc18c\uc18d \uc804\ub3d9\ucc28\uc758 \uae30\ubcf8 \ub514\uc790\uc778\uc778 \u201c \uc1fc\ub09c \ub514\uc790\uc778 (\u6e58\u5357\u30c7\u30b6\u30a4\u30f3)\u201d\uc744 \ud655\ub9bd\ud588\ub2e4.", "paragraph_id": "6603770-5-0", "paragraph_question": "question: 351\uacc4\uac00 \ud655\ub9bd\ud55c \uc138\uc774\ubd80 \ucca0\ub3c4 \uc18c\uc18d \uc804\ub3d9\ucc28\uc758 \uae30\ubcf8 \ub514\uc790\uc778\uc740?, context: \uadf8\ub7ec\ub098, 1954\ub144\uc5d0 \uc911\uad6d\uc2dd\uc774\ub098 \uc61b\ub0a0 \ucc28\uccb4\uc5d0 \ub300\ud55c \uc0c8\ub85c\uc6b4 \uc81c\uc791\uc774 \uc2dc\uc791\ub418\uc5c8\ub2e4. \uc774 \ub54c, \ub4f1\uc7a5\ud55c 351\uacc4(\ub4f1\uc7a5 \ub2f9\uc2dc 501\uacc4)\ub294 \uc9c0\uae08\ubd80\ud130 \uc7a5\uae30\uac04\uc5d0 \uac78\uce58\ub294 \uc138\uc774\ubd80 \ucca0\ub3c4 \uc18c\uc18d \uc804\ub3d9\ucc28\uc758 \uae30\ubcf8 \ub514\uc790\uc778\uc778 \u201c\uc1fc\ub09c \ub514\uc790\uc778(\u6e58\u5357\u30c7\u30b6\u30a4\u30f3)\u201d\uc744 \ud655\ub9bd\ud588\ub2e4. \uc774 \ub54c\uc758 \uc138\uc774\ubd80 \ucca0\ub3c4 \uad6c\uac04\uc744 \uc6b4\ud589\ud558\ub294 \uc804\ucca0\uc744 \ub9d0\ud55c\ub2e4\uba74, \uc7a1\ub2e4\ud55c \ud615\ud0dc\uc758 \uc804\ub3d9\ucc28\ub97c \uc5f0\uacb0\ud558\uc5ec \ubb34\uc0ac\uc2dc\ub178\ub2e4\uc774\uc9c0(\u6b66\u8535\u91ce\u53f0\u5730)\ub97c \ub2e4\ub2c8\ub294 \uac83\uc73c\ub85c \ubb18\uc0ac\ub418\uc5c8\ub2e4. 2\ucc28\ub300\uc804 \ud6c4\uc758 \uc138\uc774\ubd80 \ucca0\ub3c4\uc758 \ubc29\uce68\uc740 \u201c\uc9c8\ubcf4\ub2e4\ub294 \uc591\u201d\uc73c\ub85c, \uc774\ub294 \ucc28\ub7c9\uc758 \uace0\uc18d\ud654\ub97c \uc6b0\uc120\uc73c\ub85c \ub0b4\uc138\uc6cc\uc11c 1950\ub144\ub300 \uc911\ubc18\uc5d0 \ub4f1\uc7a5\ud55c \uc18c\uc704 \uace0\uc131\ub2a5\ucc28\uc758 \ub3c4\uc785\uc740 \ud558\uc9c0 \uc54a\uace0, \ub2e4\ub978 \uacc4\uc5f4\uacfc\uc758 \ubcd1\uacb0\uc744 \uace0\ub824\ud558\uc5ec \uc131\ub2a5\uc758 \ub2e8\uc77c\ud654\ub97c \ucd94\uc9c4\ud558\ub294 \uac83\ubd80\ud130 \uc77c\uad00\ud558\uc5ec \uc774\uc804\uc758 \u201c\ub9e4\ub2ec\ub9ac\uace0 \uac78\uce58\ub294 \uad6c\ub3d9\ubc29\uc2dd(\u540a\u308a\u639b\u3051\u99c6\u52d5\u65b9\u5f0f)\"\uc744 \uacc4\uc18d \ud558\uace0 \uc788\ub358 \ub2e4\ub978 \ud68c\uc0ac\uc640\ub294 \ub2e4\ub978 \ubc29\uc2dd\uc774\uc5c8\ub2e4. \uadf8 \ud6c4 \ub4f1\uc7a5\ud55c \uc138\uc774\ubd80 \ucca0\ub3c4 \uccab \uace0\uc131\ub2a5\ucc28\ub77c\uace0 \ub9d0\ud560 \uc218 \uc788\ub294 \uce74\ub974\ub2e8 \uad6c\ub3d9 \ubc29\uc2dd\uc758 601\uacc4, 701\uacc4 \uc804\ucca0\ub85c\ub3c4, \uc774\uc804\uc758 \uad6c\ub3d9 \ubc29\uc2dd\uc744 \uc9c0\ub2cc \ucc28\ub7c9\uacfc\uc758 \ubcd1\uacb0\uc744 \uc804\uc81c\ub85c, \uae30\ub2a5\uc801\uc73c\ub85c\ub294 \ub3d9\ub825 \uc804\ub2ec \ubc29\uc2dd\uc744 \ub192\uc778 \ub300\uccb4\ud488\uc77c \ubfd0\uc774\uc5c8\uc73c\uba70, \uc774\uc804\uc758 \ubc29\uc2dd\uc744 \uc9c0\ub2cc \ucc28\ub7c9\uacfc \ubcd1\uacb0 \ud558\ub294 \uac83\uc73c\ub85c \uc778\ud558\uc5ec \uce74\ub974\ub2e8 \uad6c\ub3d9 \ubc29\uc2dd \ubcf8\ub798\uc758 \uc131\ub2a5\uc740 \ubc1c\ud718 \ub418\uc5b4 \uc788\uc9c0 \uc54a\uc558\ub2e4. \uc804\ub3d9\ucc28\uc758 \ucc28\uccb4\ub97c \uc9c0\ud0f1\ud558\ub294 \ubd80\ubd84\uc740 \uc0c8\ub86d\uac8c \uc81c\uc791\ub418\uc5c8\uc9c0\ub9cc, \ubd80\uc218\ucc28\uc758 \ucc28\uccb4\ub97c \uc9c0\ud0f1\ud558\ub294 \ubd80\ubd84\uc740 \uc774\uc804\uc758 \uade0\ud615\ub300\ub4e4\ubcf4\uc2dd\uc744 \uc9c0\ub2cc TR11\uacc4\uc758 \uac1c\uc870\ud488\uc73c\ub85c \uad6d\ucca0\ub85c\ubd80\ud130 \uc591\ub3c4\ub41c \uac83\uc774\uc5c8\ub2e4. \uc774\uc804 \ucc28\ub7c9\uacfc \uac19\uc774 \uc4f0\ub294 \uac83\uc744 \uace0\ub824\ud558\uc9c0 \uc54a\uace0, \uc804\uae30 \uc81c\ub3d9\uc774 \uac00\ub2a5\ud558\uac8c \ud55c \uc9c4\uc815\ud55c \uc758\ubbf8\ub85c\uc758 \uace0\uc131\ub2a5\ucc28\uac00 \ub3c4\uc785\ub41c \uac83\uc740, \uc138\uc774\ubd80 \uc9c0\uce58\ubd80 \uc120\uc774 \uac1c\ud1b5\uc744 \uc55e\ub454 \ub54c\uc600\ub2e4."} {"question": "\ub9e4\ub044\ub7ec\uc6b4 \ud3c9\uc9c0\ub77c\ub294 \ub2e8\uc5b4\ub294 \ub2e4\uc591\ud55c \ud06c\uae30\uc758 \uc5b4\ub5a4\uac83\uc774 \uc874\uc7ac\ud558\ub294\uac00?", "paragraph": "\"\ub9e4\ub044\ub7ec\uc6b4 \ud3c9\uc9c0(smooth plain)\"\ub294 \ub2e4\uc591\ud55c \ud06c\uae30\uc758 \ud568\ubab0\uc9c0\uac00 \uc874\uc7ac\ud558\uba70, \ub2ec\uc758 \ubc14\ub2e4\uc640 \ub9ce\uc740 \uc720\uc0ac\uc131\uc744 \ub760\ub294 \uacf3 \uc774\ub2e4. \ud2b9\ud788, \uc8fc\ubaa9\ud560 \ub9cc\ud55c \uc810\uc740 \uce7c\ub85c\ub9ac\uc2a4 \ubd84\uc9c0\ub97c \ub458\ub7ec\uc2f8\uace0 \uc788\ub294 \ub113\uc740 \uace0\ub9ac\uc774\ub2e4. \ub2ec\uc758 \ubc14\ub2e4\uc640 \ub2ec\ub9ac, \uc218\uc131\uc758 \"\ub9e4\ub044\ub7ec\uc6b4 \ud3c9\uc9c0\" \ub294 \ubaa8\ub450 \uc54c\ubca0\ub3c4\uac00 \uac19\ub2e4. \uc774 \uc810\uc740 \ud654\uc0b0 \ud65c\ub3d9 \ud754\uc801\uc740 \ub9ce\uc9c0 \uc54a\uc544\ub3c4 \ud654\uc0b0 \ud65c\ub3d9\uc774 \ud65c\ubc1c\ud558\uac8c \uc774\ub8e8\uc5b4\uc84c\ub2e4\ub294 \uc0ac\uc2e4\uc744 \uac15\ud558\uac8c \ub4b7\ubc1b\uce68\ud55c\ub2e4. \uc218\uc131\uc758 \"\ub9e4\ub044\ub7ec\uc6b4 \ud3c9\uc9c0\"\ub294 \uce7c\ub85c\ub9ac\uc2a4 \ubd84\uc9c0\uac00 \ud615\uc131\ub41c \ud6c4\uc5d0 \ud615\uc131\ub410\ub2e4. \uc774 \uc0ac\uc2e4\uc740 \uce7c\ub85c\ub9ac\uc2a4 \ubd84\ucd9c\ubb3c \ub36e\uac1c\uc758 \ubc00\ub3c4\ubcf4\ub2e4 \ub0ae\uc740 \ucda9\ub3cc\uad6c\uac00 \uc99d\uac70\uac00 \ub41c\ub2e4. \ud3c9\uc9c0\uc5d0 \ub36e\uc778 \uce7c\ub85c\ub9ac\uc2a4 \ubd84\uc9c0\uc758 \uce35(\u5c64)\uc740, \uc9c0\uc9c8\ud559\uc801\uc73c\ub85c \ubcf4\uc558\uc744 \ub54c \uc0b0\ub9e5 \ud615\ud0dc\uc640 \ub2e4\uac01\ud615 \ud615\ud0dc\ub85c \uade0\uc5f4\uc774 \uac14\ub2e4. \uadf8\ub7ec\ub098 \uc774 \uc5ed\uc2dc \ucda9\ub3cc\ub85c \uc6a9\uc554\uc774 \ubd84\ucd9c\ud55c \uac83\uc778\uc9c0, \ucda9\ub3cc\ub85c \ub179\uc740 \uac83\uc778\uc9c0\ub294 \ud655\uc2e4\ud558\uc9c0 \uc54a\ub2e4.", "answer": "\ud568\ubab0\uc9c0", "sentence": "\"\ub9e4\ub044\ub7ec\uc6b4 \ud3c9\uc9c0(smooth plain)\"\ub294 \ub2e4\uc591\ud55c \ud06c\uae30\uc758 \ud568\ubab0\uc9c0 \uac00 \uc874\uc7ac\ud558\uba70, \ub2ec\uc758 \ubc14\ub2e4\uc640 \ub9ce\uc740 \uc720\uc0ac\uc131\uc744 \ub760\ub294 \uacf3 \uc774\ub2e4.", "paragraph_sentence": " \"\ub9e4\ub044\ub7ec\uc6b4 \ud3c9\uc9c0(smooth plain)\"\ub294 \ub2e4\uc591\ud55c \ud06c\uae30\uc758 \ud568\ubab0\uc9c0 \uac00 \uc874\uc7ac\ud558\uba70, \ub2ec\uc758 \ubc14\ub2e4\uc640 \ub9ce\uc740 \uc720\uc0ac\uc131\uc744 \ub760\ub294 \uacf3 \uc774\ub2e4. \ud2b9\ud788, \uc8fc\ubaa9\ud560 \ub9cc\ud55c \uc810\uc740 \uce7c\ub85c\ub9ac\uc2a4 \ubd84\uc9c0\ub97c \ub458\ub7ec\uc2f8\uace0 \uc788\ub294 \ub113\uc740 \uace0\ub9ac\uc774\ub2e4. \ub2ec\uc758 \ubc14\ub2e4\uc640 \ub2ec\ub9ac, \uc218\uc131\uc758 \"\ub9e4\ub044\ub7ec\uc6b4 \ud3c9\uc9c0\" \ub294 \ubaa8\ub450 \uc54c\ubca0\ub3c4\uac00 \uac19\ub2e4. \uc774 \uc810\uc740 \ud654\uc0b0 \ud65c\ub3d9 \ud754\uc801\uc740 \ub9ce\uc9c0 \uc54a\uc544\ub3c4 \ud654\uc0b0 \ud65c\ub3d9\uc774 \ud65c\ubc1c\ud558\uac8c \uc774\ub8e8\uc5b4\uc84c\ub2e4\ub294 \uc0ac\uc2e4\uc744 \uac15\ud558\uac8c \ub4b7\ubc1b\uce68\ud55c\ub2e4. \uc218\uc131\uc758 \"\ub9e4\ub044\ub7ec\uc6b4 \ud3c9\uc9c0\"\ub294 \uce7c\ub85c\ub9ac\uc2a4 \ubd84\uc9c0\uac00 \ud615\uc131\ub41c \ud6c4\uc5d0 \ud615\uc131\ub410\ub2e4. \uc774 \uc0ac\uc2e4\uc740 \uce7c\ub85c\ub9ac\uc2a4 \ubd84\ucd9c\ubb3c \ub36e\uac1c\uc758 \ubc00\ub3c4\ubcf4\ub2e4 \ub0ae\uc740 \ucda9\ub3cc\uad6c\uac00 \uc99d\uac70\uac00 \ub41c\ub2e4. \ud3c9\uc9c0\uc5d0 \ub36e\uc778 \uce7c\ub85c\ub9ac\uc2a4 \ubd84\uc9c0\uc758 \uce35(\u5c64)\uc740, \uc9c0\uc9c8\ud559\uc801\uc73c\ub85c \ubcf4\uc558\uc744 \ub54c \uc0b0\ub9e5 \ud615\ud0dc\uc640 \ub2e4\uac01\ud615 \ud615\ud0dc\ub85c \uade0\uc5f4\uc774 \uac14\ub2e4. \uadf8\ub7ec\ub098 \uc774 \uc5ed\uc2dc \ucda9\ub3cc\ub85c \uc6a9\uc554\uc774 \ubd84\ucd9c\ud55c \uac83\uc778\uc9c0, \ucda9\ub3cc\ub85c \ub179\uc740 \uac83\uc778\uc9c0\ub294 \ud655\uc2e4\ud558\uc9c0 \uc54a\ub2e4.", "paragraph_answer": "\"\ub9e4\ub044\ub7ec\uc6b4 \ud3c9\uc9c0(smooth plain)\"\ub294 \ub2e4\uc591\ud55c \ud06c\uae30\uc758 \ud568\ubab0\uc9c0 \uac00 \uc874\uc7ac\ud558\uba70, \ub2ec\uc758 \ubc14\ub2e4\uc640 \ub9ce\uc740 \uc720\uc0ac\uc131\uc744 \ub760\ub294 \uacf3 \uc774\ub2e4. \ud2b9\ud788, \uc8fc\ubaa9\ud560 \ub9cc\ud55c \uc810\uc740 \uce7c\ub85c\ub9ac\uc2a4 \ubd84\uc9c0\ub97c \ub458\ub7ec\uc2f8\uace0 \uc788\ub294 \ub113\uc740 \uace0\ub9ac\uc774\ub2e4. \ub2ec\uc758 \ubc14\ub2e4\uc640 \ub2ec\ub9ac, \uc218\uc131\uc758 \"\ub9e4\ub044\ub7ec\uc6b4 \ud3c9\uc9c0\" \ub294 \ubaa8\ub450 \uc54c\ubca0\ub3c4\uac00 \uac19\ub2e4. \uc774 \uc810\uc740 \ud654\uc0b0 \ud65c\ub3d9 \ud754\uc801\uc740 \ub9ce\uc9c0 \uc54a\uc544\ub3c4 \ud654\uc0b0 \ud65c\ub3d9\uc774 \ud65c\ubc1c\ud558\uac8c \uc774\ub8e8\uc5b4\uc84c\ub2e4\ub294 \uc0ac\uc2e4\uc744 \uac15\ud558\uac8c \ub4b7\ubc1b\uce68\ud55c\ub2e4. \uc218\uc131\uc758 \"\ub9e4\ub044\ub7ec\uc6b4 \ud3c9\uc9c0\"\ub294 \uce7c\ub85c\ub9ac\uc2a4 \ubd84\uc9c0\uac00 \ud615\uc131\ub41c \ud6c4\uc5d0 \ud615\uc131\ub410\ub2e4. \uc774 \uc0ac\uc2e4\uc740 \uce7c\ub85c\ub9ac\uc2a4 \ubd84\ucd9c\ubb3c \ub36e\uac1c\uc758 \ubc00\ub3c4\ubcf4\ub2e4 \ub0ae\uc740 \ucda9\ub3cc\uad6c\uac00 \uc99d\uac70\uac00 \ub41c\ub2e4. \ud3c9\uc9c0\uc5d0 \ub36e\uc778 \uce7c\ub85c\ub9ac\uc2a4 \ubd84\uc9c0\uc758 \uce35(\u5c64)\uc740, \uc9c0\uc9c8\ud559\uc801\uc73c\ub85c \ubcf4\uc558\uc744 \ub54c \uc0b0\ub9e5 \ud615\ud0dc\uc640 \ub2e4\uac01\ud615 \ud615\ud0dc\ub85c \uade0\uc5f4\uc774 \uac14\ub2e4. \uadf8\ub7ec\ub098 \uc774 \uc5ed\uc2dc \ucda9\ub3cc\ub85c \uc6a9\uc554\uc774 \ubd84\ucd9c\ud55c \uac83\uc778\uc9c0, \ucda9\ub3cc\ub85c \ub179\uc740 \uac83\uc778\uc9c0\ub294 \ud655\uc2e4\ud558\uc9c0 \uc54a\ub2e4.", "sentence_answer": "\"\ub9e4\ub044\ub7ec\uc6b4 \ud3c9\uc9c0(smooth plain)\"\ub294 \ub2e4\uc591\ud55c \ud06c\uae30\uc758 \ud568\ubab0\uc9c0 \uac00 \uc874\uc7ac\ud558\uba70, \ub2ec\uc758 \ubc14\ub2e4\uc640 \ub9ce\uc740 \uc720\uc0ac\uc131\uc744 \ub760\ub294 \uacf3 \uc774\ub2e4.", "paragraph_id": "6532492-2-0", "paragraph_question": "question: \ub9e4\ub044\ub7ec\uc6b4 \ud3c9\uc9c0\ub77c\ub294 \ub2e8\uc5b4\ub294 \ub2e4\uc591\ud55c \ud06c\uae30\uc758 \uc5b4\ub5a4\uac83\uc774 \uc874\uc7ac\ud558\ub294\uac00?, context: \"\ub9e4\ub044\ub7ec\uc6b4 \ud3c9\uc9c0(smooth plain)\"\ub294 \ub2e4\uc591\ud55c \ud06c\uae30\uc758 \ud568\ubab0\uc9c0\uac00 \uc874\uc7ac\ud558\uba70, \ub2ec\uc758 \ubc14\ub2e4\uc640 \ub9ce\uc740 \uc720\uc0ac\uc131\uc744 \ub760\ub294 \uacf3 \uc774\ub2e4. \ud2b9\ud788, \uc8fc\ubaa9\ud560 \ub9cc\ud55c \uc810\uc740 \uce7c\ub85c\ub9ac\uc2a4 \ubd84\uc9c0\ub97c \ub458\ub7ec\uc2f8\uace0 \uc788\ub294 \ub113\uc740 \uace0\ub9ac\uc774\ub2e4. \ub2ec\uc758 \ubc14\ub2e4\uc640 \ub2ec\ub9ac, \uc218\uc131\uc758 \"\ub9e4\ub044\ub7ec\uc6b4 \ud3c9\uc9c0\" \ub294 \ubaa8\ub450 \uc54c\ubca0\ub3c4\uac00 \uac19\ub2e4. \uc774 \uc810\uc740 \ud654\uc0b0 \ud65c\ub3d9 \ud754\uc801\uc740 \ub9ce\uc9c0 \uc54a\uc544\ub3c4 \ud654\uc0b0 \ud65c\ub3d9\uc774 \ud65c\ubc1c\ud558\uac8c \uc774\ub8e8\uc5b4\uc84c\ub2e4\ub294 \uc0ac\uc2e4\uc744 \uac15\ud558\uac8c \ub4b7\ubc1b\uce68\ud55c\ub2e4. \uc218\uc131\uc758 \"\ub9e4\ub044\ub7ec\uc6b4 \ud3c9\uc9c0\"\ub294 \uce7c\ub85c\ub9ac\uc2a4 \ubd84\uc9c0\uac00 \ud615\uc131\ub41c \ud6c4\uc5d0 \ud615\uc131\ub410\ub2e4. \uc774 \uc0ac\uc2e4\uc740 \uce7c\ub85c\ub9ac\uc2a4 \ubd84\ucd9c\ubb3c \ub36e\uac1c\uc758 \ubc00\ub3c4\ubcf4\ub2e4 \ub0ae\uc740 \ucda9\ub3cc\uad6c\uac00 \uc99d\uac70\uac00 \ub41c\ub2e4. \ud3c9\uc9c0\uc5d0 \ub36e\uc778 \uce7c\ub85c\ub9ac\uc2a4 \ubd84\uc9c0\uc758 \uce35(\u5c64)\uc740, \uc9c0\uc9c8\ud559\uc801\uc73c\ub85c \ubcf4\uc558\uc744 \ub54c \uc0b0\ub9e5 \ud615\ud0dc\uc640 \ub2e4\uac01\ud615 \ud615\ud0dc\ub85c \uade0\uc5f4\uc774 \uac14\ub2e4. \uadf8\ub7ec\ub098 \uc774 \uc5ed\uc2dc \ucda9\ub3cc\ub85c \uc6a9\uc554\uc774 \ubd84\ucd9c\ud55c \uac83\uc778\uc9c0, \ucda9\ub3cc\ub85c \ub179\uc740 \uac83\uc778\uc9c0\ub294 \ud655\uc2e4\ud558\uc9c0 \uc54a\ub2e4."} {"question": "\uac00\uc7a5 \uc720\uba85\ud55c \uad6c\ubbf8\ubca0\uc5b4\uc778 \uace8\ub4dc\ubca0\uc5b4 \uc2dc\ub9ac\uc988\uac00 \ub9cc\ub4e4\uc5b4\uc9c4 \ub144\ub3c4\ub294 \uc5b8\uc81c\uc778\uac00?", "paragraph": "1920\ub144 \uc81c\uacfc\uc5c5\uc744 \ud558\ub358 \ud55c\uc2a4 \ub9ac\uac94(Hans Riegel)\uc774 \uc124\ub9bd\ud55c \uc0ac\ud0d5 \uc81c\uc870 \ud68c\uc0ac\uc778 \ud558\ub9ac\ubcf4(HARIBO)\uc5d0\uc11c \ub9cc\ub4e4\uc5b4\uc9c4 \uac83\uc774 \ucd5c\ucd08\uc758 \uc81c\ud488\uc774\ub77c \ud560 \uc218 \uc788\ub2e4. \ub3c5\uc77c \ubcf8(Bonn)\uc5d0\uc11c \ucd5c\ucd08\uc758 \uacf5\uc7a5\uc774 \uc124\ub9bd\ub418\uc5c8\uc73c\ub098, 1922\ub144 \uc138\uacc4 2\ucc28 \ub300\uc804\uc73c\ub85c \uc778\ud574 \uc124\ub9bd\uc790\uac00 \uc0ac\ub9dd\ud558\uba70 \uc824\ub9ac \uc81c\uc870\uc758 \uc704\uae30\ub97c \ub9de\uac8c \ub41c\ub2e4. \ud558\uc9c0\ub9cc \uc124\ub9bd\uc790\uc758 \uc544\ub4e4\uc778 \ud55c\uc2a4 \ub9ac\uac94 \uc8fc\ub2c8\uc5b4\uc640 \ud3f4 \ub9ac\uac94\uc774 \ud68c\uc0ac\ub97c \uc774\uc5b4\ubc1b\uc544 \uc704\uae30 \uadf9\ubcf5 \ud6c4, \ucd5c\ucd08\ub85c \uc138\uacc4\uc801\uc73c\ub85c \uc120\ud48d\uc801\uc778 \uc778\uae30\ub97c \ub044\ub294 \uc81c\ud488\uc778 \ub304\uc2f1\ubca0\uc5b4(DANCING BEAR)\ub97c \ub9cc\ub4e4\uac8c \ub418\uba70 \uc774\uac83\uc774 \uad6c\ubbf8\ubca0\uc5b4\uc758 \uc2dc\ucd08\uac00 \ub41c\ub2e4. \uc774\ud6c4 \ube14\ub799\ubca0\uc5b4(BLACK BEAR) \ub4f1\uc744 \ub9cc\ub4e4\uba70 \uc778\uae30\ub97c \uc774\uc5b4\ub098\uac14\uc73c\uba70, 1967\ub144 \uc138\uacc4\uc801\uc73c\ub85c \uac00\uc7a5 \uc720\uba85\ud55c \uad6c\ubbf8\ubca0\uc5b4 \uc911 \ud558\ub098\uc778 \uace8\ub4dc\ubca0\uc5b4(GOLD BEAR) \uc2dc\ub9ac\uc988\ub97c \ub9cc\ub4e4\uba74\uc11c \uac00\uc7a5 \uc720\uba85\ud55c \uc8fc\uc804\ubd80\ub9ac\ub85c \uc790\ub9ac\uc7a1\ub294\ub2e4.", "answer": "1967\ub144", "sentence": "\uc774\ud6c4 \ube14\ub799\ubca0\uc5b4(BLACK BEAR) \ub4f1\uc744 \ub9cc\ub4e4\uba70 \uc778\uae30\ub97c \uc774\uc5b4\ub098\uac14\uc73c\uba70, 1967\ub144 \uc138\uacc4\uc801\uc73c\ub85c \uac00\uc7a5 \uc720\uba85\ud55c \uad6c\ubbf8\ubca0\uc5b4 \uc911 \ud558\ub098\uc778 \uace8\ub4dc\ubca0\uc5b4(GOLD BEAR) \uc2dc\ub9ac\uc988\ub97c \ub9cc\ub4e4\uba74\uc11c \uac00\uc7a5 \uc720\uba85\ud55c \uc8fc\uc804\ubd80\ub9ac\ub85c \uc790\ub9ac\uc7a1\ub294\ub2e4.", "paragraph_sentence": "1920\ub144 \uc81c\uacfc\uc5c5\uc744 \ud558\ub358 \ud55c\uc2a4 \ub9ac\uac94(Hans Riegel)\uc774 \uc124\ub9bd\ud55c \uc0ac\ud0d5 \uc81c\uc870 \ud68c\uc0ac\uc778 \ud558\ub9ac\ubcf4(HARIBO)\uc5d0\uc11c \ub9cc\ub4e4\uc5b4\uc9c4 \uac83\uc774 \ucd5c\ucd08\uc758 \uc81c\ud488\uc774\ub77c \ud560 \uc218 \uc788\ub2e4. \ub3c5\uc77c \ubcf8(Bonn)\uc5d0\uc11c \ucd5c\ucd08\uc758 \uacf5\uc7a5\uc774 \uc124\ub9bd\ub418\uc5c8\uc73c\ub098, 1922\ub144 \uc138\uacc4 2\ucc28 \ub300\uc804\uc73c\ub85c \uc778\ud574 \uc124\ub9bd\uc790\uac00 \uc0ac\ub9dd\ud558\uba70 \uc824\ub9ac \uc81c\uc870\uc758 \uc704\uae30\ub97c \ub9de\uac8c \ub41c\ub2e4. \ud558\uc9c0\ub9cc \uc124\ub9bd\uc790\uc758 \uc544\ub4e4\uc778 \ud55c\uc2a4 \ub9ac\uac94 \uc8fc\ub2c8\uc5b4\uc640 \ud3f4 \ub9ac\uac94\uc774 \ud68c\uc0ac\ub97c \uc774\uc5b4\ubc1b\uc544 \uc704\uae30 \uadf9\ubcf5 \ud6c4, \ucd5c\ucd08\ub85c \uc138\uacc4\uc801\uc73c\ub85c \uc120\ud48d\uc801\uc778 \uc778\uae30\ub97c \ub044\ub294 \uc81c\ud488\uc778 \ub304\uc2f1\ubca0\uc5b4(DANCING BEAR)\ub97c \ub9cc\ub4e4\uac8c \ub418\uba70 \uc774\uac83\uc774 \uad6c\ubbf8\ubca0\uc5b4\uc758 \uc2dc\ucd08\uac00 \ub41c\ub2e4. \uc774\ud6c4 \ube14\ub799\ubca0\uc5b4(BLACK BEAR) \ub4f1\uc744 \ub9cc\ub4e4\uba70 \uc778\uae30\ub97c \uc774\uc5b4\ub098\uac14\uc73c\uba70, 1967\ub144 \uc138\uacc4\uc801\uc73c\ub85c \uac00\uc7a5 \uc720\uba85\ud55c \uad6c\ubbf8\ubca0\uc5b4 \uc911 \ud558\ub098\uc778 \uace8\ub4dc\ubca0\uc5b4(GOLD BEAR) \uc2dc\ub9ac\uc988\ub97c \ub9cc\ub4e4\uba74\uc11c \uac00\uc7a5 \uc720\uba85\ud55c \uc8fc\uc804\ubd80\ub9ac\ub85c \uc790\ub9ac\uc7a1\ub294\ub2e4. ", "paragraph_answer": "1920\ub144 \uc81c\uacfc\uc5c5\uc744 \ud558\ub358 \ud55c\uc2a4 \ub9ac\uac94(Hans Riegel)\uc774 \uc124\ub9bd\ud55c \uc0ac\ud0d5 \uc81c\uc870 \ud68c\uc0ac\uc778 \ud558\ub9ac\ubcf4(HARIBO)\uc5d0\uc11c \ub9cc\ub4e4\uc5b4\uc9c4 \uac83\uc774 \ucd5c\ucd08\uc758 \uc81c\ud488\uc774\ub77c \ud560 \uc218 \uc788\ub2e4. \ub3c5\uc77c \ubcf8(Bonn)\uc5d0\uc11c \ucd5c\ucd08\uc758 \uacf5\uc7a5\uc774 \uc124\ub9bd\ub418\uc5c8\uc73c\ub098, 1922\ub144 \uc138\uacc4 2\ucc28 \ub300\uc804\uc73c\ub85c \uc778\ud574 \uc124\ub9bd\uc790\uac00 \uc0ac\ub9dd\ud558\uba70 \uc824\ub9ac \uc81c\uc870\uc758 \uc704\uae30\ub97c \ub9de\uac8c \ub41c\ub2e4. \ud558\uc9c0\ub9cc \uc124\ub9bd\uc790\uc758 \uc544\ub4e4\uc778 \ud55c\uc2a4 \ub9ac\uac94 \uc8fc\ub2c8\uc5b4\uc640 \ud3f4 \ub9ac\uac94\uc774 \ud68c\uc0ac\ub97c \uc774\uc5b4\ubc1b\uc544 \uc704\uae30 \uadf9\ubcf5 \ud6c4, \ucd5c\ucd08\ub85c \uc138\uacc4\uc801\uc73c\ub85c \uc120\ud48d\uc801\uc778 \uc778\uae30\ub97c \ub044\ub294 \uc81c\ud488\uc778 \ub304\uc2f1\ubca0\uc5b4(DANCING BEAR)\ub97c \ub9cc\ub4e4\uac8c \ub418\uba70 \uc774\uac83\uc774 \uad6c\ubbf8\ubca0\uc5b4\uc758 \uc2dc\ucd08\uac00 \ub41c\ub2e4. \uc774\ud6c4 \ube14\ub799\ubca0\uc5b4(BLACK BEAR) \ub4f1\uc744 \ub9cc\ub4e4\uba70 \uc778\uae30\ub97c \uc774\uc5b4\ub098\uac14\uc73c\uba70, 1967\ub144 \uc138\uacc4\uc801\uc73c\ub85c \uac00\uc7a5 \uc720\uba85\ud55c \uad6c\ubbf8\ubca0\uc5b4 \uc911 \ud558\ub098\uc778 \uace8\ub4dc\ubca0\uc5b4(GOLD BEAR) \uc2dc\ub9ac\uc988\ub97c \ub9cc\ub4e4\uba74\uc11c \uac00\uc7a5 \uc720\uba85\ud55c \uc8fc\uc804\ubd80\ub9ac\ub85c \uc790\ub9ac\uc7a1\ub294\ub2e4.", "sentence_answer": "\uc774\ud6c4 \ube14\ub799\ubca0\uc5b4(BLACK BEAR) \ub4f1\uc744 \ub9cc\ub4e4\uba70 \uc778\uae30\ub97c \uc774\uc5b4\ub098\uac14\uc73c\uba70, 1967\ub144 \uc138\uacc4\uc801\uc73c\ub85c \uac00\uc7a5 \uc720\uba85\ud55c \uad6c\ubbf8\ubca0\uc5b4 \uc911 \ud558\ub098\uc778 \uace8\ub4dc\ubca0\uc5b4(GOLD BEAR) \uc2dc\ub9ac\uc988\ub97c \ub9cc\ub4e4\uba74\uc11c \uac00\uc7a5 \uc720\uba85\ud55c \uc8fc\uc804\ubd80\ub9ac\ub85c \uc790\ub9ac\uc7a1\ub294\ub2e4.", "paragraph_id": "6257694-0-1", "paragraph_question": "question: \uac00\uc7a5 \uc720\uba85\ud55c \uad6c\ubbf8\ubca0\uc5b4\uc778 \uace8\ub4dc\ubca0\uc5b4 \uc2dc\ub9ac\uc988\uac00 \ub9cc\ub4e4\uc5b4\uc9c4 \ub144\ub3c4\ub294 \uc5b8\uc81c\uc778\uac00?, context: 1920\ub144 \uc81c\uacfc\uc5c5\uc744 \ud558\ub358 \ud55c\uc2a4 \ub9ac\uac94(Hans Riegel)\uc774 \uc124\ub9bd\ud55c \uc0ac\ud0d5 \uc81c\uc870 \ud68c\uc0ac\uc778 \ud558\ub9ac\ubcf4(HARIBO)\uc5d0\uc11c \ub9cc\ub4e4\uc5b4\uc9c4 \uac83\uc774 \ucd5c\ucd08\uc758 \uc81c\ud488\uc774\ub77c \ud560 \uc218 \uc788\ub2e4. \ub3c5\uc77c \ubcf8(Bonn)\uc5d0\uc11c \ucd5c\ucd08\uc758 \uacf5\uc7a5\uc774 \uc124\ub9bd\ub418\uc5c8\uc73c\ub098, 1922\ub144 \uc138\uacc4 2\ucc28 \ub300\uc804\uc73c\ub85c \uc778\ud574 \uc124\ub9bd\uc790\uac00 \uc0ac\ub9dd\ud558\uba70 \uc824\ub9ac \uc81c\uc870\uc758 \uc704\uae30\ub97c \ub9de\uac8c \ub41c\ub2e4. \ud558\uc9c0\ub9cc \uc124\ub9bd\uc790\uc758 \uc544\ub4e4\uc778 \ud55c\uc2a4 \ub9ac\uac94 \uc8fc\ub2c8\uc5b4\uc640 \ud3f4 \ub9ac\uac94\uc774 \ud68c\uc0ac\ub97c \uc774\uc5b4\ubc1b\uc544 \uc704\uae30 \uadf9\ubcf5 \ud6c4, \ucd5c\ucd08\ub85c \uc138\uacc4\uc801\uc73c\ub85c \uc120\ud48d\uc801\uc778 \uc778\uae30\ub97c \ub044\ub294 \uc81c\ud488\uc778 \ub304\uc2f1\ubca0\uc5b4(DANCING BEAR)\ub97c \ub9cc\ub4e4\uac8c \ub418\uba70 \uc774\uac83\uc774 \uad6c\ubbf8\ubca0\uc5b4\uc758 \uc2dc\ucd08\uac00 \ub41c\ub2e4. \uc774\ud6c4 \ube14\ub799\ubca0\uc5b4(BLACK BEAR) \ub4f1\uc744 \ub9cc\ub4e4\uba70 \uc778\uae30\ub97c \uc774\uc5b4\ub098\uac14\uc73c\uba70, 1967\ub144 \uc138\uacc4\uc801\uc73c\ub85c \uac00\uc7a5 \uc720\uba85\ud55c \uad6c\ubbf8\ubca0\uc5b4 \uc911 \ud558\ub098\uc778 \uace8\ub4dc\ubca0\uc5b4(GOLD BEAR) \uc2dc\ub9ac\uc988\ub97c \ub9cc\ub4e4\uba74\uc11c \uac00\uc7a5 \uc720\uba85\ud55c \uc8fc\uc804\ubd80\ub9ac\ub85c \uc790\ub9ac\uc7a1\ub294\ub2e4."} {"question": "\uc54c\uce20\ud558\uc774\uba38\ubcd1\uc740 \uce58\ub9e4 \ud658\uc790\uc758 \uba87 \ud504\ub85c\uac00 \uac00\uc9c0\uace0 \uc788\ub294 \ubcd1\uc778\uac00?", "paragraph": "\uc54c\uce20\ud558\uc774\uba38\ubcd1(Alzheimer's disease (AD)=Alzheimer disease)\uc740 \uce58\ub9e4\uc758 \uac00\uc7a5 \ud754\ud55c \ud615\ud0dc\uc774\uba70 75%\uc758 \uce58\ub9e4 \ud658\uc790\uac00 \uc54c\uce20\ud558\uc774\uba38\ubcd1\uc774\ub2e4. \ud604\ub300 \uc758\ud559\uc5d0\uc11c\ub294 \uce58\ub8cc\ud560 \uc218 \uc5c6\ub294 \uc9c8\ubcd1\uc73c\ub85c, \uc2dc\uac04\uc774 \uc9c0\ub0a0\uc218\ub85d \uc99d\uc0c1\uc774 \uc545\ud654\ub418\uba70 \uacb0\uacfc\uc801\uc73c\ub85c \uc8fd\uc74c\uc5d0 \uc774\ub978\ub2e4. 1906\ub144 \ub3c5\uc77c\uc758 \uc815\uc2e0\uacfc \uc758\uc0ac\uc778 \uc54c\ub85c\uc774\uc2a4 \uc54c\uce20\ud558\uc774\uba38(Alois Alzheimer)\uc5d0 \uc758\ud574 \uc54c\ub824\uc84c\ub2e4. \ub300\ubd80\ubd84\uc758 \uacbd\uc6b0 \uc54c\uce20\ud558\uc774\uba38\ubcd1\uc740 65\uc138\uac00 \ub118\uc5b4 \ubc1c\ubcd1\ud558\uc9c0\ub9cc, \ub4dc\ubb3c\uac8c \uadf8 \uc774\uc804\uc5d0 \ubc1c\ubcd1\ud560 \uc218 \uc788\ub2e4. \ubbf8\uad6d\uc5d0\uc11c\ub294 65~74\uc138 \uc778\uad6c\uc758 \uc57d 3%, 75~84\uc138 \uc778\uad6c\uc758 \uc57d 19%, 85\uc138 \uc774\uc0c1 \uc778\uad6c\uc758 50%\uac00 \uc774 \ubcd1\uc744 \uc553\uace0 \uc788\ub2e4. \ud55c\uad6d\uc5d0\uc11c\ub294 \ucd5c\uadfc \ud55c \ub18d\ucd0c \uc9c0\uc5ed\uc744 \uc911\uc2ec\uc73c\ub85c \ud55c \uc5f0\uad6c \ubcf4\uace0\uc5d0 \uc758\ud558\uba74, \ub18d\ucd0c \uc9c0\uc5ed 60\uc138 \uc774\uc0c1\uc758 \uc778\uad6c\uc5d0\uc11c \uc57d 21%\uac00 \uce58\ub9e4 \uc591\uc0c1\uc744 \ubcf4\uc774\uace0, \uc774 \uc911 63%\uac00 \uc54c\uce20\ud558\uc774\uba38\ud615 \uce58\ub9e4\uc778 \uac83\uc73c\ub85c \ubcf4\uace0\ub418\uace0 \uc788\ub2e4. 2006\ub144 \uc804 \uc138\uacc4 26.6\ub9cc\uba85\uc774 \uac00\uc9c4 \uc9c8\ubcd1\uc774\ub2e4. 2050\ub144\uc5d0\ub294 85\uba85\uc911 1\uba85\uaf34\ub85c \ubc1c\ubcd1\ub420 \uac83\uc73c\ub85c \uc608\uce21\ub41c\ub2e4.", "answer": "75%", "sentence": "\uc54c\uce20\ud558\uc774\uba38\ubcd1(Alzheimer's disease (AD)=Alzheimer disease)\uc740 \uce58\ub9e4\uc758 \uac00\uc7a5 \ud754\ud55c \ud615\ud0dc\uc774\uba70 75% \uc758 \uce58\ub9e4 \ud658\uc790\uac00 \uc54c\uce20\ud558\uc774\uba38\ubcd1\uc774\ub2e4.", "paragraph_sentence": " \uc54c\uce20\ud558\uc774\uba38\ubcd1(Alzheimer's disease (AD)=Alzheimer disease)\uc740 \uce58\ub9e4\uc758 \uac00\uc7a5 \ud754\ud55c \ud615\ud0dc\uc774\uba70 75% \uc758 \uce58\ub9e4 \ud658\uc790\uac00 \uc54c\uce20\ud558\uc774\uba38\ubcd1\uc774\ub2e4. \ud604\ub300 \uc758\ud559\uc5d0\uc11c\ub294 \uce58\ub8cc\ud560 \uc218 \uc5c6\ub294 \uc9c8\ubcd1\uc73c\ub85c, \uc2dc\uac04\uc774 \uc9c0\ub0a0\uc218\ub85d \uc99d\uc0c1\uc774 \uc545\ud654\ub418\uba70 \uacb0\uacfc\uc801\uc73c\ub85c \uc8fd\uc74c\uc5d0 \uc774\ub978\ub2e4. 1906\ub144 \ub3c5\uc77c\uc758 \uc815\uc2e0\uacfc \uc758\uc0ac\uc778 \uc54c\ub85c\uc774\uc2a4 \uc54c\uce20\ud558\uc774\uba38(Alois Alzheimer)\uc5d0 \uc758\ud574 \uc54c\ub824\uc84c\ub2e4. \ub300\ubd80\ubd84\uc758 \uacbd\uc6b0 \uc54c\uce20\ud558\uc774\uba38\ubcd1\uc740 65\uc138\uac00 \ub118\uc5b4 \ubc1c\ubcd1\ud558\uc9c0\ub9cc, \ub4dc\ubb3c\uac8c \uadf8 \uc774\uc804\uc5d0 \ubc1c\ubcd1\ud560 \uc218 \uc788\ub2e4. \ubbf8\uad6d\uc5d0\uc11c\ub294 65~74\uc138 \uc778\uad6c\uc758 \uc57d 3%, 75~84\uc138 \uc778\uad6c\uc758 \uc57d 19%, 85\uc138 \uc774\uc0c1 \uc778\uad6c\uc758 50%\uac00 \uc774 \ubcd1\uc744 \uc553\uace0 \uc788\ub2e4. \ud55c\uad6d\uc5d0\uc11c\ub294 \ucd5c\uadfc \ud55c \ub18d\ucd0c \uc9c0\uc5ed\uc744 \uc911\uc2ec\uc73c\ub85c \ud55c \uc5f0\uad6c \ubcf4\uace0\uc5d0 \uc758\ud558\uba74, \ub18d\ucd0c \uc9c0\uc5ed 60\uc138 \uc774\uc0c1\uc758 \uc778\uad6c\uc5d0\uc11c \uc57d 21%\uac00 \uce58\ub9e4 \uc591\uc0c1\uc744 \ubcf4\uc774\uace0, \uc774 \uc911 63%\uac00 \uc54c\uce20\ud558\uc774\uba38\ud615 \uce58\ub9e4\uc778 \uac83\uc73c\ub85c \ubcf4\uace0\ub418\uace0 \uc788\ub2e4. 2006\ub144 \uc804 \uc138\uacc4 26.6\ub9cc\uba85\uc774 \uac00\uc9c4 \uc9c8\ubcd1\uc774\ub2e4. 2050\ub144\uc5d0\ub294 85\uba85\uc911 1\uba85\uaf34\ub85c \ubc1c\ubcd1\ub420 \uac83\uc73c\ub85c \uc608\uce21\ub41c\ub2e4.", "paragraph_answer": "\uc54c\uce20\ud558\uc774\uba38\ubcd1(Alzheimer's disease (AD)=Alzheimer disease)\uc740 \uce58\ub9e4\uc758 \uac00\uc7a5 \ud754\ud55c \ud615\ud0dc\uc774\uba70 75% \uc758 \uce58\ub9e4 \ud658\uc790\uac00 \uc54c\uce20\ud558\uc774\uba38\ubcd1\uc774\ub2e4. \ud604\ub300 \uc758\ud559\uc5d0\uc11c\ub294 \uce58\ub8cc\ud560 \uc218 \uc5c6\ub294 \uc9c8\ubcd1\uc73c\ub85c, \uc2dc\uac04\uc774 \uc9c0\ub0a0\uc218\ub85d \uc99d\uc0c1\uc774 \uc545\ud654\ub418\uba70 \uacb0\uacfc\uc801\uc73c\ub85c \uc8fd\uc74c\uc5d0 \uc774\ub978\ub2e4. 1906\ub144 \ub3c5\uc77c\uc758 \uc815\uc2e0\uacfc \uc758\uc0ac\uc778 \uc54c\ub85c\uc774\uc2a4 \uc54c\uce20\ud558\uc774\uba38(Alois Alzheimer)\uc5d0 \uc758\ud574 \uc54c\ub824\uc84c\ub2e4. \ub300\ubd80\ubd84\uc758 \uacbd\uc6b0 \uc54c\uce20\ud558\uc774\uba38\ubcd1\uc740 65\uc138\uac00 \ub118\uc5b4 \ubc1c\ubcd1\ud558\uc9c0\ub9cc, \ub4dc\ubb3c\uac8c \uadf8 \uc774\uc804\uc5d0 \ubc1c\ubcd1\ud560 \uc218 \uc788\ub2e4. \ubbf8\uad6d\uc5d0\uc11c\ub294 65~74\uc138 \uc778\uad6c\uc758 \uc57d 3%, 75~84\uc138 \uc778\uad6c\uc758 \uc57d 19%, 85\uc138 \uc774\uc0c1 \uc778\uad6c\uc758 50%\uac00 \uc774 \ubcd1\uc744 \uc553\uace0 \uc788\ub2e4. \ud55c\uad6d\uc5d0\uc11c\ub294 \ucd5c\uadfc \ud55c \ub18d\ucd0c \uc9c0\uc5ed\uc744 \uc911\uc2ec\uc73c\ub85c \ud55c \uc5f0\uad6c \ubcf4\uace0\uc5d0 \uc758\ud558\uba74, \ub18d\ucd0c \uc9c0\uc5ed 60\uc138 \uc774\uc0c1\uc758 \uc778\uad6c\uc5d0\uc11c \uc57d 21%\uac00 \uce58\ub9e4 \uc591\uc0c1\uc744 \ubcf4\uc774\uace0, \uc774 \uc911 63%\uac00 \uc54c\uce20\ud558\uc774\uba38\ud615 \uce58\ub9e4\uc778 \uac83\uc73c\ub85c \ubcf4\uace0\ub418\uace0 \uc788\ub2e4. 2006\ub144 \uc804 \uc138\uacc4 26.6\ub9cc\uba85\uc774 \uac00\uc9c4 \uc9c8\ubcd1\uc774\ub2e4. 2050\ub144\uc5d0\ub294 85\uba85\uc911 1\uba85\uaf34\ub85c \ubc1c\ubcd1\ub420 \uac83\uc73c\ub85c \uc608\uce21\ub41c\ub2e4.", "sentence_answer": "\uc54c\uce20\ud558\uc774\uba38\ubcd1(Alzheimer's disease (AD)=Alzheimer disease)\uc740 \uce58\ub9e4\uc758 \uac00\uc7a5 \ud754\ud55c \ud615\ud0dc\uc774\uba70 75% \uc758 \uce58\ub9e4 \ud658\uc790\uac00 \uc54c\uce20\ud558\uc774\uba38\ubcd1\uc774\ub2e4.", "paragraph_id": "6539479-0-0", "paragraph_question": "question: \uc54c\uce20\ud558\uc774\uba38\ubcd1\uc740 \uce58\ub9e4 \ud658\uc790\uc758 \uba87 \ud504\ub85c\uac00 \uac00\uc9c0\uace0 \uc788\ub294 \ubcd1\uc778\uac00?, context: \uc54c\uce20\ud558\uc774\uba38\ubcd1(Alzheimer's disease (AD)=Alzheimer disease)\uc740 \uce58\ub9e4\uc758 \uac00\uc7a5 \ud754\ud55c \ud615\ud0dc\uc774\uba70 75%\uc758 \uce58\ub9e4 \ud658\uc790\uac00 \uc54c\uce20\ud558\uc774\uba38\ubcd1\uc774\ub2e4. \ud604\ub300 \uc758\ud559\uc5d0\uc11c\ub294 \uce58\ub8cc\ud560 \uc218 \uc5c6\ub294 \uc9c8\ubcd1\uc73c\ub85c, \uc2dc\uac04\uc774 \uc9c0\ub0a0\uc218\ub85d \uc99d\uc0c1\uc774 \uc545\ud654\ub418\uba70 \uacb0\uacfc\uc801\uc73c\ub85c \uc8fd\uc74c\uc5d0 \uc774\ub978\ub2e4. 1906\ub144 \ub3c5\uc77c\uc758 \uc815\uc2e0\uacfc \uc758\uc0ac\uc778 \uc54c\ub85c\uc774\uc2a4 \uc54c\uce20\ud558\uc774\uba38(Alois Alzheimer)\uc5d0 \uc758\ud574 \uc54c\ub824\uc84c\ub2e4. \ub300\ubd80\ubd84\uc758 \uacbd\uc6b0 \uc54c\uce20\ud558\uc774\uba38\ubcd1\uc740 65\uc138\uac00 \ub118\uc5b4 \ubc1c\ubcd1\ud558\uc9c0\ub9cc, \ub4dc\ubb3c\uac8c \uadf8 \uc774\uc804\uc5d0 \ubc1c\ubcd1\ud560 \uc218 \uc788\ub2e4. \ubbf8\uad6d\uc5d0\uc11c\ub294 65~74\uc138 \uc778\uad6c\uc758 \uc57d 3%, 75~84\uc138 \uc778\uad6c\uc758 \uc57d 19%, 85\uc138 \uc774\uc0c1 \uc778\uad6c\uc758 50%\uac00 \uc774 \ubcd1\uc744 \uc553\uace0 \uc788\ub2e4. \ud55c\uad6d\uc5d0\uc11c\ub294 \ucd5c\uadfc \ud55c \ub18d\ucd0c \uc9c0\uc5ed\uc744 \uc911\uc2ec\uc73c\ub85c \ud55c \uc5f0\uad6c \ubcf4\uace0\uc5d0 \uc758\ud558\uba74, \ub18d\ucd0c \uc9c0\uc5ed 60\uc138 \uc774\uc0c1\uc758 \uc778\uad6c\uc5d0\uc11c \uc57d 21%\uac00 \uce58\ub9e4 \uc591\uc0c1\uc744 \ubcf4\uc774\uace0, \uc774 \uc911 63%\uac00 \uc54c\uce20\ud558\uc774\uba38\ud615 \uce58\ub9e4\uc778 \uac83\uc73c\ub85c \ubcf4\uace0\ub418\uace0 \uc788\ub2e4. 2006\ub144 \uc804 \uc138\uacc4 26.6\ub9cc\uba85\uc774 \uac00\uc9c4 \uc9c8\ubcd1\uc774\ub2e4. 2050\ub144\uc5d0\ub294 85\uba85\uc911 1\uba85\uaf34\ub85c \ubc1c\ubcd1\ub420 \uac83\uc73c\ub85c \uc608\uce21\ub41c\ub2e4."} {"question": "\uc2ec\uc2dc\ud2f0 \uc2dc\ub9ac\uc988\uac00 \uc815\ub9bd\ud55c \ub3c4\uc2dc \uac74\uc124 \uc2dc\ubbac\ub808\uc774\uc158\uc758 \uc911\uc810\uc744 \ub450 \uac00\uc9c0\ub85c \uc815\ub9ac\ud55c \ub9ac\ubdf0\uc5b4\uc758 \uc18c\uc18d \ub9e4\uccb4\ub294?", "paragraph": "\uc778\ubca4\uc740 \uc218\ub9ce\uc740 \uac8c\uc784\uc774 \uc3df\uc544\uc838 \ub098\uc624\ub294 \ube44\ub514\uc624 \uac8c\uc784\uc5d0\uc11c \uc2ec\uc2dc\ud2f0 \uc2dc\ub9ac\uc988\uac00 \ub3c4\uc2dc \uc2dc\ubbac\ub808\uc774\uc158\uc758 \ucd5c\uace0 \uc790\ub9ac\ub97c \uace0\uc218\ud588\ub2e4\ub294 \uac74 \ub180\ub77c\uc6b4 \uc77c\uc774\ub77c\uace0 \ud558\uba74\uc11c, \uc774\ub294 \ud50c\ub808\uc774\uc5b4\uac00 \uae30\ub300\ud558\ub358 \uac83\uc774 \uc774\ubbf8 '\uc2ec\uc2dc\ud2f0' \uc2dc\ub9ac\uc988\uc5d0 \ub4e4\uc5b4\uac00 \uc788\uc5c8\uae30 \ub54c\ubb38\uc774\ub77c\uace0 \ud3c9\uac00\ud558\uace0 \uc788\ub2e4. \uc2ec\uc2dc\ud2f0\uc758 \uacbd\uc7c1\uc790\ub85c 'Anno' \uc2dc\ub9ac\uc988\ub77c\ub358\uac00, '\uc2dc\ud2f0\uc988 XL' \uc2dc\ub9ac\uc988 \uac19\uc740 \ub3c4\uc804\uc790\ub4e4\uc774 \uc788\uc5c8\uc9c0\ub9cc, \uc774 \ub3c4\uc804\uc790\ub4e4\uc740 \ud50c\ub808\uc774\uc5b4\uc758 \uc695\uad6c\ub97c \ucda9\uc871\ud574\uc8fc\uc9c0 \ubabb\ud588\ub2e4\uace0 \uc124\uba85\ud558\uace0 \uc788\ub2e4. \ub610\ud55c \uc778\ubca4 \ub9ac\ubdf0\uc5b4\ub294 \uc2ec\uc2dc\ud2f0 \uc2dc\ub9ac\uc988\uac00 \uc815\ub9bd\ud55c \ub3c4\uc2dc \uac74\uc124 \uc2dc\ubbac\ub808\uc774\uc158\uc5d0\uc11c\uc758 \uc911\uc694\ud55c \uc810\uc740 '\ub113\uace0 \uba4b\uc9c4 \ub3c4\uc2dc \uafb8\ubbf8\uae30'\uc640 '\uc138\uc138\ud55c \ub3c4\uc2dc\uacc4\ud68d'\uc774\ub77c\uace0 \ub9d0\ud558\uba74\uc11c, \uc774 \ub458\uc744 \ub3d9\uc2dc\uc5d0 \ucd94\uad6c\ud558\ub294 \uac83\uc774 \ub3c4\uc2dc \uac74\uc124 \uc2dc\ubbac\ub808\uc774\uc158 \ud50c\ub808\uc774\uc5b4\uc758 \uc18c\ub9dd\uc774\uc790 \uac8c\uc784\uc758 \ubaa9\ud45c\ub77c\uace0 \ub9d0\ud558\uace0 \uc788\ub2e4. \uc774\ub7f0 \uc694\uc18c\ub97c \ucda9\uc871\ud558\uae30 \uc704\ud574\uc11c \ub3c4\uc2dc \uc2dc\ubbac\ub808\uc774\uc158 \uac8c\uc784\uc740 \uce58\ubc00\ud55c \uc54c\uace0\ub9ac\uc998\uacfc \ub354\ubd88\uc5b4 \ub9c9\ub300\ud55c \uc5f0\uc0b0\ub7c9\uc744 \uc694\uad6c\ud558\uba70, \ud6cc\ub96d\ud55c \ub3c4\uc2dc \uac74\uc124 \uc2dc\ubbac\ub808\uc774\uc158\uc774 \ub418\uae30 \uc704\ud574\uc120 \uc5ec\ub7ec \ubd80\ubd84\uc5d0\uc11c \uc644\ubcbd\ud568\uc744 \ucd94\uad6c\ud574\uc57c \ud55c\ub2e4\ub77c\uace0 \uc778\ubca4\uc740 \ub9d0\ud558\uace0 \uc788\ub2e4.", "answer": "\uc778\ubca4", "sentence": "\uc778\ubca4 \uc740 \uc218\ub9ce\uc740 \uac8c\uc784\uc774 \uc3df\uc544\uc838 \ub098\uc624\ub294 \ube44\ub514\uc624 \uac8c\uc784\uc5d0\uc11c \uc2ec\uc2dc\ud2f0 \uc2dc\ub9ac\uc988\uac00 \ub3c4\uc2dc \uc2dc\ubbac\ub808\uc774\uc158\uc758 \ucd5c\uace0 \uc790\ub9ac\ub97c \uace0\uc218\ud588\ub2e4\ub294 \uac74 \ub180\ub77c\uc6b4 \uc77c\uc774\ub77c\uace0 \ud558\uba74\uc11c, \uc774\ub294 \ud50c\ub808\uc774\uc5b4\uac00 \uae30\ub300\ud558\ub358 \uac83\uc774 \uc774\ubbf8 '\uc2ec\uc2dc\ud2f0' \uc2dc\ub9ac\uc988\uc5d0 \ub4e4\uc5b4\uac00 \uc788\uc5c8\uae30 \ub54c\ubb38\uc774\ub77c\uace0 \ud3c9\uac00\ud558\uace0 \uc788\ub2e4.", "paragraph_sentence": " \uc778\ubca4 \uc740 \uc218\ub9ce\uc740 \uac8c\uc784\uc774 \uc3df\uc544\uc838 \ub098\uc624\ub294 \ube44\ub514\uc624 \uac8c\uc784\uc5d0\uc11c \uc2ec\uc2dc\ud2f0 \uc2dc\ub9ac\uc988\uac00 \ub3c4\uc2dc \uc2dc\ubbac\ub808\uc774\uc158\uc758 \ucd5c\uace0 \uc790\ub9ac\ub97c \uace0\uc218\ud588\ub2e4\ub294 \uac74 \ub180\ub77c\uc6b4 \uc77c\uc774\ub77c\uace0 \ud558\uba74\uc11c, \uc774\ub294 \ud50c\ub808\uc774\uc5b4\uac00 \uae30\ub300\ud558\ub358 \uac83\uc774 \uc774\ubbf8 '\uc2ec\uc2dc\ud2f0' \uc2dc\ub9ac\uc988\uc5d0 \ub4e4\uc5b4\uac00 \uc788\uc5c8\uae30 \ub54c\ubb38\uc774\ub77c\uace0 \ud3c9\uac00\ud558\uace0 \uc788\ub2e4. \uc2ec\uc2dc\ud2f0\uc758 \uacbd\uc7c1\uc790\ub85c 'Anno' \uc2dc\ub9ac\uc988\ub77c\ub358\uac00, '\uc2dc\ud2f0\uc988 XL' \uc2dc\ub9ac\uc988 \uac19\uc740 \ub3c4\uc804\uc790\ub4e4\uc774 \uc788\uc5c8\uc9c0\ub9cc, \uc774 \ub3c4\uc804\uc790\ub4e4\uc740 \ud50c\ub808\uc774\uc5b4\uc758 \uc695\uad6c\ub97c \ucda9\uc871\ud574\uc8fc\uc9c0 \ubabb\ud588\ub2e4\uace0 \uc124\uba85\ud558\uace0 \uc788\ub2e4. \ub610\ud55c \uc778\ubca4 \ub9ac\ubdf0\uc5b4\ub294 \uc2ec\uc2dc\ud2f0 \uc2dc\ub9ac\uc988\uac00 \uc815\ub9bd\ud55c \ub3c4\uc2dc \uac74\uc124 \uc2dc\ubbac\ub808\uc774\uc158\uc5d0\uc11c\uc758 \uc911\uc694\ud55c \uc810\uc740 '\ub113\uace0 \uba4b\uc9c4 \ub3c4\uc2dc \uafb8\ubbf8\uae30'\uc640 '\uc138\uc138\ud55c \ub3c4\uc2dc\uacc4\ud68d'\uc774\ub77c\uace0 \ub9d0\ud558\uba74\uc11c, \uc774 \ub458\uc744 \ub3d9\uc2dc\uc5d0 \ucd94\uad6c\ud558\ub294 \uac83\uc774 \ub3c4\uc2dc \uac74\uc124 \uc2dc\ubbac\ub808\uc774\uc158 \ud50c\ub808\uc774\uc5b4\uc758 \uc18c\ub9dd\uc774\uc790 \uac8c\uc784\uc758 \ubaa9\ud45c\ub77c\uace0 \ub9d0\ud558\uace0 \uc788\ub2e4. \uc774\ub7f0 \uc694\uc18c\ub97c \ucda9\uc871\ud558\uae30 \uc704\ud574\uc11c \ub3c4\uc2dc \uc2dc\ubbac\ub808\uc774\uc158 \uac8c\uc784\uc740 \uce58\ubc00\ud55c \uc54c\uace0\ub9ac\uc998\uacfc \ub354\ubd88\uc5b4 \ub9c9\ub300\ud55c \uc5f0\uc0b0\ub7c9\uc744 \uc694\uad6c\ud558\uba70, \ud6cc\ub96d\ud55c \ub3c4\uc2dc \uac74\uc124 \uc2dc\ubbac\ub808\uc774\uc158\uc774 \ub418\uae30 \uc704\ud574\uc120 \uc5ec\ub7ec \ubd80\ubd84\uc5d0\uc11c \uc644\ubcbd\ud568\uc744 \ucd94\uad6c\ud574\uc57c \ud55c\ub2e4\ub77c\uace0 \uc778\ubca4\uc740 \ub9d0\ud558\uace0 \uc788\ub2e4.", "paragraph_answer": " \uc778\ubca4 \uc740 \uc218\ub9ce\uc740 \uac8c\uc784\uc774 \uc3df\uc544\uc838 \ub098\uc624\ub294 \ube44\ub514\uc624 \uac8c\uc784\uc5d0\uc11c \uc2ec\uc2dc\ud2f0 \uc2dc\ub9ac\uc988\uac00 \ub3c4\uc2dc \uc2dc\ubbac\ub808\uc774\uc158\uc758 \ucd5c\uace0 \uc790\ub9ac\ub97c \uace0\uc218\ud588\ub2e4\ub294 \uac74 \ub180\ub77c\uc6b4 \uc77c\uc774\ub77c\uace0 \ud558\uba74\uc11c, \uc774\ub294 \ud50c\ub808\uc774\uc5b4\uac00 \uae30\ub300\ud558\ub358 \uac83\uc774 \uc774\ubbf8 '\uc2ec\uc2dc\ud2f0' \uc2dc\ub9ac\uc988\uc5d0 \ub4e4\uc5b4\uac00 \uc788\uc5c8\uae30 \ub54c\ubb38\uc774\ub77c\uace0 \ud3c9\uac00\ud558\uace0 \uc788\ub2e4. \uc2ec\uc2dc\ud2f0\uc758 \uacbd\uc7c1\uc790\ub85c 'Anno' \uc2dc\ub9ac\uc988\ub77c\ub358\uac00, '\uc2dc\ud2f0\uc988 XL' \uc2dc\ub9ac\uc988 \uac19\uc740 \ub3c4\uc804\uc790\ub4e4\uc774 \uc788\uc5c8\uc9c0\ub9cc, \uc774 \ub3c4\uc804\uc790\ub4e4\uc740 \ud50c\ub808\uc774\uc5b4\uc758 \uc695\uad6c\ub97c \ucda9\uc871\ud574\uc8fc\uc9c0 \ubabb\ud588\ub2e4\uace0 \uc124\uba85\ud558\uace0 \uc788\ub2e4. \ub610\ud55c \uc778\ubca4 \ub9ac\ubdf0\uc5b4\ub294 \uc2ec\uc2dc\ud2f0 \uc2dc\ub9ac\uc988\uac00 \uc815\ub9bd\ud55c \ub3c4\uc2dc \uac74\uc124 \uc2dc\ubbac\ub808\uc774\uc158\uc5d0\uc11c\uc758 \uc911\uc694\ud55c \uc810\uc740 '\ub113\uace0 \uba4b\uc9c4 \ub3c4\uc2dc \uafb8\ubbf8\uae30'\uc640 '\uc138\uc138\ud55c \ub3c4\uc2dc\uacc4\ud68d'\uc774\ub77c\uace0 \ub9d0\ud558\uba74\uc11c, \uc774 \ub458\uc744 \ub3d9\uc2dc\uc5d0 \ucd94\uad6c\ud558\ub294 \uac83\uc774 \ub3c4\uc2dc \uac74\uc124 \uc2dc\ubbac\ub808\uc774\uc158 \ud50c\ub808\uc774\uc5b4\uc758 \uc18c\ub9dd\uc774\uc790 \uac8c\uc784\uc758 \ubaa9\ud45c\ub77c\uace0 \ub9d0\ud558\uace0 \uc788\ub2e4. \uc774\ub7f0 \uc694\uc18c\ub97c \ucda9\uc871\ud558\uae30 \uc704\ud574\uc11c \ub3c4\uc2dc \uc2dc\ubbac\ub808\uc774\uc158 \uac8c\uc784\uc740 \uce58\ubc00\ud55c \uc54c\uace0\ub9ac\uc998\uacfc \ub354\ubd88\uc5b4 \ub9c9\ub300\ud55c \uc5f0\uc0b0\ub7c9\uc744 \uc694\uad6c\ud558\uba70, \ud6cc\ub96d\ud55c \ub3c4\uc2dc \uac74\uc124 \uc2dc\ubbac\ub808\uc774\uc158\uc774 \ub418\uae30 \uc704\ud574\uc120 \uc5ec\ub7ec \ubd80\ubd84\uc5d0\uc11c \uc644\ubcbd\ud568\uc744 \ucd94\uad6c\ud574\uc57c \ud55c\ub2e4\ub77c\uace0 \uc778\ubca4\uc740 \ub9d0\ud558\uace0 \uc788\ub2e4.", "sentence_answer": " \uc778\ubca4 \uc740 \uc218\ub9ce\uc740 \uac8c\uc784\uc774 \uc3df\uc544\uc838 \ub098\uc624\ub294 \ube44\ub514\uc624 \uac8c\uc784\uc5d0\uc11c \uc2ec\uc2dc\ud2f0 \uc2dc\ub9ac\uc988\uac00 \ub3c4\uc2dc \uc2dc\ubbac\ub808\uc774\uc158\uc758 \ucd5c\uace0 \uc790\ub9ac\ub97c \uace0\uc218\ud588\ub2e4\ub294 \uac74 \ub180\ub77c\uc6b4 \uc77c\uc774\ub77c\uace0 \ud558\uba74\uc11c, \uc774\ub294 \ud50c\ub808\uc774\uc5b4\uac00 \uae30\ub300\ud558\ub358 \uac83\uc774 \uc774\ubbf8 '\uc2ec\uc2dc\ud2f0' \uc2dc\ub9ac\uc988\uc5d0 \ub4e4\uc5b4\uac00 \uc788\uc5c8\uae30 \ub54c\ubb38\uc774\ub77c\uace0 \ud3c9\uac00\ud558\uace0 \uc788\ub2e4.", "paragraph_id": "6519573-16-1", "paragraph_question": "question: \uc2ec\uc2dc\ud2f0 \uc2dc\ub9ac\uc988\uac00 \uc815\ub9bd\ud55c \ub3c4\uc2dc \uac74\uc124 \uc2dc\ubbac\ub808\uc774\uc158\uc758 \uc911\uc810\uc744 \ub450 \uac00\uc9c0\ub85c \uc815\ub9ac\ud55c \ub9ac\ubdf0\uc5b4\uc758 \uc18c\uc18d \ub9e4\uccb4\ub294?, context: \uc778\ubca4\uc740 \uc218\ub9ce\uc740 \uac8c\uc784\uc774 \uc3df\uc544\uc838 \ub098\uc624\ub294 \ube44\ub514\uc624 \uac8c\uc784\uc5d0\uc11c \uc2ec\uc2dc\ud2f0 \uc2dc\ub9ac\uc988\uac00 \ub3c4\uc2dc \uc2dc\ubbac\ub808\uc774\uc158\uc758 \ucd5c\uace0 \uc790\ub9ac\ub97c \uace0\uc218\ud588\ub2e4\ub294 \uac74 \ub180\ub77c\uc6b4 \uc77c\uc774\ub77c\uace0 \ud558\uba74\uc11c, \uc774\ub294 \ud50c\ub808\uc774\uc5b4\uac00 \uae30\ub300\ud558\ub358 \uac83\uc774 \uc774\ubbf8 '\uc2ec\uc2dc\ud2f0' \uc2dc\ub9ac\uc988\uc5d0 \ub4e4\uc5b4\uac00 \uc788\uc5c8\uae30 \ub54c\ubb38\uc774\ub77c\uace0 \ud3c9\uac00\ud558\uace0 \uc788\ub2e4. \uc2ec\uc2dc\ud2f0\uc758 \uacbd\uc7c1\uc790\ub85c 'Anno' \uc2dc\ub9ac\uc988\ub77c\ub358\uac00, '\uc2dc\ud2f0\uc988 XL' \uc2dc\ub9ac\uc988 \uac19\uc740 \ub3c4\uc804\uc790\ub4e4\uc774 \uc788\uc5c8\uc9c0\ub9cc, \uc774 \ub3c4\uc804\uc790\ub4e4\uc740 \ud50c\ub808\uc774\uc5b4\uc758 \uc695\uad6c\ub97c \ucda9\uc871\ud574\uc8fc\uc9c0 \ubabb\ud588\ub2e4\uace0 \uc124\uba85\ud558\uace0 \uc788\ub2e4. \ub610\ud55c \uc778\ubca4 \ub9ac\ubdf0\uc5b4\ub294 \uc2ec\uc2dc\ud2f0 \uc2dc\ub9ac\uc988\uac00 \uc815\ub9bd\ud55c \ub3c4\uc2dc \uac74\uc124 \uc2dc\ubbac\ub808\uc774\uc158\uc5d0\uc11c\uc758 \uc911\uc694\ud55c \uc810\uc740 '\ub113\uace0 \uba4b\uc9c4 \ub3c4\uc2dc \uafb8\ubbf8\uae30'\uc640 '\uc138\uc138\ud55c \ub3c4\uc2dc\uacc4\ud68d'\uc774\ub77c\uace0 \ub9d0\ud558\uba74\uc11c, \uc774 \ub458\uc744 \ub3d9\uc2dc\uc5d0 \ucd94\uad6c\ud558\ub294 \uac83\uc774 \ub3c4\uc2dc \uac74\uc124 \uc2dc\ubbac\ub808\uc774\uc158 \ud50c\ub808\uc774\uc5b4\uc758 \uc18c\ub9dd\uc774\uc790 \uac8c\uc784\uc758 \ubaa9\ud45c\ub77c\uace0 \ub9d0\ud558\uace0 \uc788\ub2e4. \uc774\ub7f0 \uc694\uc18c\ub97c \ucda9\uc871\ud558\uae30 \uc704\ud574\uc11c \ub3c4\uc2dc \uc2dc\ubbac\ub808\uc774\uc158 \uac8c\uc784\uc740 \uce58\ubc00\ud55c \uc54c\uace0\ub9ac\uc998\uacfc \ub354\ubd88\uc5b4 \ub9c9\ub300\ud55c \uc5f0\uc0b0\ub7c9\uc744 \uc694\uad6c\ud558\uba70, \ud6cc\ub96d\ud55c \ub3c4\uc2dc \uac74\uc124 \uc2dc\ubbac\ub808\uc774\uc158\uc774 \ub418\uae30 \uc704\ud574\uc120 \uc5ec\ub7ec \ubd80\ubd84\uc5d0\uc11c \uc644\ubcbd\ud568\uc744 \ucd94\uad6c\ud574\uc57c \ud55c\ub2e4\ub77c\uace0 \uc778\ubca4\uc740 \ub9d0\ud558\uace0 \uc788\ub2e4."} {"question": "\ud53c\ud574\uc790\uc758 \ucd94\ubaa8 \uc6b4\ub3d9\uc774 \uc2dc\uc791\ub418\uc5b4 \uc5b4\ub514\uc5d0 \uc218 \ub9ce\uc740 \ud3ec\uc2a4\ud2b8\uc787\uacfc \uad6d\ud654 \uaf43\uc774 \ub193\uc544\uc84c\ub098?", "paragraph": "\ud55c \ub124\ud2f0\uc98c\uc758 \uc81c\uc548\uc73c\ub85c \ud53c\ud574\uc790\uc758 \ucd94\ubaa8 \uc6b4\ub3d9\uc774 \uc2dc\uc791\ub418\uc5b4 \uc0ac\uac74 \ud604\uc7a5\uacfc \uac00\uae4c\uc6b4 \uac15\ub0a8\uc5ed 10\ubc88 \ucd9c\uad6c\uc5d0\ub294 \u201c\uc5ec\uc131 \ud610\uc624\ub294 \uc0ac\ud68c\uc801 \ubb38\uc81c\u201d, \u201c\ub0a8\uc544 \uc788\ub294 \uc5ec\uc131\ub4e4\uc774 \ub354 \uc88b\uc740 \uc138\uc0c1 \ub9cc\ub4e4\uac8c\uc694\u201d \ub4f1 \uc5ec\uc131 \ud610\uc624 \ubb38\uc81c\ub97c \uc9c0\uc801\ud558\ub294 \ub0b4\uc6a9\uc758 \ucabd\uc9c0\ub4e4\uc774 \ubd99\uc5c8\ub2e4. \uac15\ub0a8\uc5ed 10\ubc88 \ucd9c\uad6c \uc0ac\uac74 \ud604\uc7a5\uc744 \uc911\uc2ec\uc73c\ub85c \ud53c\ud574\uc790\ub97c \ucd94\ubaa8\ud558\ub294 \ub73b\uc744 \uc804\ud558\ub294 \ud3ec\uc2a4\ud2b8\uc787\uacfc \uad6d\ud654 \uaf43\uc774 \ub193\uc5ec\uc9c0\ub294 \ub4f1 \uc5ec\uc131 \ud610\uc624 \ubb38\uc81c\uc5d0 \ub300\ud558\uc5ec \uc0ac\ud68c\uc801 \ub17c\ub780\uc774 \uc77c\uc5c8\ub2e4. \ud55c\ud3b8 \uc774 \uc0ac\uac74\uc744 \uc5ec\uc131 \ud610\uc624 \ubc94\uc8c4\ub85c \ud574\uc11d\ud558\ub294 \uac83\uc744 \ube44\uc57d\uc774\ub77c\uace0 \uc8fc\uc7a5\ud558\uba70, \u201c\ud53c\ud574\uc758\uc2dd\uc774 \uc788\ub294 \uc5ec\uc131\ub4e4\uc774 \uc0ac\uac74\uc744 \uc9c0\ub098\uce58\uac8c \ud655\ub300\ud558\uace0 \uc788\ub2e4\u201d, \u201c\ub0a8\uc790\ub77c\ub294 \uc774\uc720\ub9cc\uc73c\ub85c \uc7a0\uc7ac\uc801 \ubc94\uc8c4\uc790 \ucde8\uae09\uc744 \ubc1b\uc544\uc57c \ud558\ub294 \uac70\ub0d0\u201d, \u201c\uac00\ub09c\ud55c \uc0ac\ub78c\uc774 \ubd80\uc790\ub97c \uc0b4\ud574\ud558\uba74 \uac00\ub09c\ud55c \uc0ac\ub78c\ub3c4 \uc7a0\uc7ac\uc801 \uc0b4\uc778\uc790\ub0d0\u201d \ub4f1\uc758 \uae00\uc774 SNS\uc5d0 \uc62c\ub77c\uc624\uace0 \uac15\ub0a8\uc5ed\uc5d0\ub294 \"\ub0a8\uc790\ub77c\uc11c \uc8fd\uc740 \ucc9c\uc548\ud568 \uc6a9\uc0ac\ub4e4\uc744 \uc78a\uc9c0 \ub9d9\uc2dc\ub2e4\"\uc774\ub77c\uace0 \uc4f0\uc5ec\uc9c4 \ub9ac\ubcf8\uc774 \ub2ec\ub9b0 \ud654\ud658\uc774 5\uc6d4 19\uc77c\uc5d0 \ub3c4\ucc29\ud558\uc600\uc73c\ub098, \uc774\ud6c4 \ud53c\ud574 \uc5ec\uc131\uc744 \ucd94\ubaa8\ud558\ub294 \uc2dc\ubbfc\uc5d0 \uc758\ud558\uc5ec \ucd94\ubaa8 \ud3ec\uc2a4\ud2b8\uc787\uc73c\ub85c \ub9ac\ubcf8\uc774 \uac00\ub824\uc84c\uace0, \uacb0\uad6d \ub9ac\ubcf8\uc774 \uc81c\uac70\ub418\uc5c8\ub2e4.", "answer": "\uac15\ub0a8\uc5ed 10\ubc88 \ucd9c\uad6c", "sentence": "\ud55c \ub124\ud2f0\uc98c\uc758 \uc81c\uc548\uc73c\ub85c \ud53c\ud574\uc790\uc758 \ucd94\ubaa8 \uc6b4\ub3d9\uc774 \uc2dc\uc791\ub418\uc5b4 \uc0ac\uac74 \ud604\uc7a5\uacfc \uac00\uae4c\uc6b4 \uac15\ub0a8\uc5ed 10\ubc88 \ucd9c\uad6c \uc5d0\ub294 \u201c\uc5ec\uc131 \ud610\uc624\ub294 \uc0ac\ud68c\uc801 \ubb38\uc81c\u201d, \u201c\ub0a8\uc544 \uc788\ub294 \uc5ec\uc131\ub4e4\uc774 \ub354 \uc88b\uc740 \uc138\uc0c1 \ub9cc\ub4e4\uac8c\uc694\u201d \ub4f1 \uc5ec\uc131 \ud610\uc624 \ubb38\uc81c\ub97c \uc9c0\uc801\ud558\ub294 \ub0b4\uc6a9\uc758 \ucabd\uc9c0\ub4e4\uc774 \ubd99\uc5c8\ub2e4.", "paragraph_sentence": " \ud55c \ub124\ud2f0\uc98c\uc758 \uc81c\uc548\uc73c\ub85c \ud53c\ud574\uc790\uc758 \ucd94\ubaa8 \uc6b4\ub3d9\uc774 \uc2dc\uc791\ub418\uc5b4 \uc0ac\uac74 \ud604\uc7a5\uacfc \uac00\uae4c\uc6b4 \uac15\ub0a8\uc5ed 10\ubc88 \ucd9c\uad6c \uc5d0\ub294 \u201c\uc5ec\uc131 \ud610\uc624\ub294 \uc0ac\ud68c\uc801 \ubb38\uc81c\u201d, \u201c\ub0a8\uc544 \uc788\ub294 \uc5ec\uc131\ub4e4\uc774 \ub354 \uc88b\uc740 \uc138\uc0c1 \ub9cc\ub4e4\uac8c\uc694\u201d \ub4f1 \uc5ec\uc131 \ud610\uc624 \ubb38\uc81c\ub97c \uc9c0\uc801\ud558\ub294 \ub0b4\uc6a9\uc758 \ucabd\uc9c0\ub4e4\uc774 \ubd99\uc5c8\ub2e4. \uac15\ub0a8\uc5ed 10\ubc88 \ucd9c\uad6c \uc0ac\uac74 \ud604\uc7a5\uc744 \uc911\uc2ec\uc73c\ub85c \ud53c\ud574\uc790\ub97c \ucd94\ubaa8\ud558\ub294 \ub73b\uc744 \uc804\ud558\ub294 \ud3ec\uc2a4\ud2b8\uc787\uacfc \uad6d\ud654 \uaf43\uc774 \ub193\uc5ec\uc9c0\ub294 \ub4f1 \uc5ec\uc131 \ud610\uc624 \ubb38\uc81c\uc5d0 \ub300\ud558\uc5ec \uc0ac\ud68c\uc801 \ub17c\ub780\uc774 \uc77c\uc5c8\ub2e4. \ud55c\ud3b8 \uc774 \uc0ac\uac74\uc744 \uc5ec\uc131 \ud610\uc624 \ubc94\uc8c4\ub85c \ud574\uc11d\ud558\ub294 \uac83\uc744 \ube44\uc57d\uc774\ub77c\uace0 \uc8fc\uc7a5\ud558\uba70, \u201c\ud53c\ud574\uc758\uc2dd\uc774 \uc788\ub294 \uc5ec\uc131\ub4e4\uc774 \uc0ac\uac74\uc744 \uc9c0\ub098\uce58\uac8c \ud655\ub300\ud558\uace0 \uc788\ub2e4\u201d, \u201c\ub0a8\uc790\ub77c\ub294 \uc774\uc720\ub9cc\uc73c\ub85c \uc7a0\uc7ac\uc801 \ubc94\uc8c4\uc790 \ucde8\uae09\uc744 \ubc1b\uc544\uc57c \ud558\ub294 \uac70\ub0d0\u201d, \u201c\uac00\ub09c\ud55c \uc0ac\ub78c\uc774 \ubd80\uc790\ub97c \uc0b4\ud574\ud558\uba74 \uac00\ub09c\ud55c \uc0ac\ub78c\ub3c4 \uc7a0\uc7ac\uc801 \uc0b4\uc778\uc790\ub0d0\u201d \ub4f1\uc758 \uae00\uc774 SNS\uc5d0 \uc62c\ub77c\uc624\uace0 \uac15\ub0a8\uc5ed\uc5d0\ub294 \"\ub0a8\uc790\ub77c\uc11c \uc8fd\uc740 \ucc9c\uc548\ud568 \uc6a9\uc0ac\ub4e4\uc744 \uc78a\uc9c0 \ub9d9\uc2dc\ub2e4\"\uc774\ub77c\uace0 \uc4f0\uc5ec\uc9c4 \ub9ac\ubcf8\uc774 \ub2ec\ub9b0 \ud654\ud658\uc774 5\uc6d4 19\uc77c\uc5d0 \ub3c4\ucc29\ud558\uc600\uc73c\ub098, \uc774\ud6c4 \ud53c\ud574 \uc5ec\uc131\uc744 \ucd94\ubaa8\ud558\ub294 \uc2dc\ubbfc\uc5d0 \uc758\ud558\uc5ec \ucd94\ubaa8 \ud3ec\uc2a4\ud2b8\uc787\uc73c\ub85c \ub9ac\ubcf8\uc774 \uac00\ub824\uc84c\uace0, \uacb0\uad6d \ub9ac\ubcf8\uc774 \uc81c\uac70\ub418\uc5c8\ub2e4.", "paragraph_answer": "\ud55c \ub124\ud2f0\uc98c\uc758 \uc81c\uc548\uc73c\ub85c \ud53c\ud574\uc790\uc758 \ucd94\ubaa8 \uc6b4\ub3d9\uc774 \uc2dc\uc791\ub418\uc5b4 \uc0ac\uac74 \ud604\uc7a5\uacfc \uac00\uae4c\uc6b4 \uac15\ub0a8\uc5ed 10\ubc88 \ucd9c\uad6c \uc5d0\ub294 \u201c\uc5ec\uc131 \ud610\uc624\ub294 \uc0ac\ud68c\uc801 \ubb38\uc81c\u201d, \u201c\ub0a8\uc544 \uc788\ub294 \uc5ec\uc131\ub4e4\uc774 \ub354 \uc88b\uc740 \uc138\uc0c1 \ub9cc\ub4e4\uac8c\uc694\u201d \ub4f1 \uc5ec\uc131 \ud610\uc624 \ubb38\uc81c\ub97c \uc9c0\uc801\ud558\ub294 \ub0b4\uc6a9\uc758 \ucabd\uc9c0\ub4e4\uc774 \ubd99\uc5c8\ub2e4. \uac15\ub0a8\uc5ed 10\ubc88 \ucd9c\uad6c \uc0ac\uac74 \ud604\uc7a5\uc744 \uc911\uc2ec\uc73c\ub85c \ud53c\ud574\uc790\ub97c \ucd94\ubaa8\ud558\ub294 \ub73b\uc744 \uc804\ud558\ub294 \ud3ec\uc2a4\ud2b8\uc787\uacfc \uad6d\ud654 \uaf43\uc774 \ub193\uc5ec\uc9c0\ub294 \ub4f1 \uc5ec\uc131 \ud610\uc624 \ubb38\uc81c\uc5d0 \ub300\ud558\uc5ec \uc0ac\ud68c\uc801 \ub17c\ub780\uc774 \uc77c\uc5c8\ub2e4. \ud55c\ud3b8 \uc774 \uc0ac\uac74\uc744 \uc5ec\uc131 \ud610\uc624 \ubc94\uc8c4\ub85c \ud574\uc11d\ud558\ub294 \uac83\uc744 \ube44\uc57d\uc774\ub77c\uace0 \uc8fc\uc7a5\ud558\uba70, \u201c\ud53c\ud574\uc758\uc2dd\uc774 \uc788\ub294 \uc5ec\uc131\ub4e4\uc774 \uc0ac\uac74\uc744 \uc9c0\ub098\uce58\uac8c \ud655\ub300\ud558\uace0 \uc788\ub2e4\u201d, \u201c\ub0a8\uc790\ub77c\ub294 \uc774\uc720\ub9cc\uc73c\ub85c \uc7a0\uc7ac\uc801 \ubc94\uc8c4\uc790 \ucde8\uae09\uc744 \ubc1b\uc544\uc57c \ud558\ub294 \uac70\ub0d0\u201d, \u201c\uac00\ub09c\ud55c \uc0ac\ub78c\uc774 \ubd80\uc790\ub97c \uc0b4\ud574\ud558\uba74 \uac00\ub09c\ud55c \uc0ac\ub78c\ub3c4 \uc7a0\uc7ac\uc801 \uc0b4\uc778\uc790\ub0d0\u201d \ub4f1\uc758 \uae00\uc774 SNS\uc5d0 \uc62c\ub77c\uc624\uace0 \uac15\ub0a8\uc5ed\uc5d0\ub294 \"\ub0a8\uc790\ub77c\uc11c \uc8fd\uc740 \ucc9c\uc548\ud568 \uc6a9\uc0ac\ub4e4\uc744 \uc78a\uc9c0 \ub9d9\uc2dc\ub2e4\"\uc774\ub77c\uace0 \uc4f0\uc5ec\uc9c4 \ub9ac\ubcf8\uc774 \ub2ec\ub9b0 \ud654\ud658\uc774 5\uc6d4 19\uc77c\uc5d0 \ub3c4\ucc29\ud558\uc600\uc73c\ub098, \uc774\ud6c4 \ud53c\ud574 \uc5ec\uc131\uc744 \ucd94\ubaa8\ud558\ub294 \uc2dc\ubbfc\uc5d0 \uc758\ud558\uc5ec \ucd94\ubaa8 \ud3ec\uc2a4\ud2b8\uc787\uc73c\ub85c \ub9ac\ubcf8\uc774 \uac00\ub824\uc84c\uace0, \uacb0\uad6d \ub9ac\ubcf8\uc774 \uc81c\uac70\ub418\uc5c8\ub2e4.", "sentence_answer": "\ud55c \ub124\ud2f0\uc98c\uc758 \uc81c\uc548\uc73c\ub85c \ud53c\ud574\uc790\uc758 \ucd94\ubaa8 \uc6b4\ub3d9\uc774 \uc2dc\uc791\ub418\uc5b4 \uc0ac\uac74 \ud604\uc7a5\uacfc \uac00\uae4c\uc6b4 \uac15\ub0a8\uc5ed 10\ubc88 \ucd9c\uad6c \uc5d0\ub294 \u201c\uc5ec\uc131 \ud610\uc624\ub294 \uc0ac\ud68c\uc801 \ubb38\uc81c\u201d, \u201c\ub0a8\uc544 \uc788\ub294 \uc5ec\uc131\ub4e4\uc774 \ub354 \uc88b\uc740 \uc138\uc0c1 \ub9cc\ub4e4\uac8c\uc694\u201d \ub4f1 \uc5ec\uc131 \ud610\uc624 \ubb38\uc81c\ub97c \uc9c0\uc801\ud558\ub294 \ub0b4\uc6a9\uc758 \ucabd\uc9c0\ub4e4\uc774 \ubd99\uc5c8\ub2e4.", "paragraph_id": "6601077-3-0", "paragraph_question": "question: \ud53c\ud574\uc790\uc758 \ucd94\ubaa8 \uc6b4\ub3d9\uc774 \uc2dc\uc791\ub418\uc5b4 \uc5b4\ub514\uc5d0 \uc218 \ub9ce\uc740 \ud3ec\uc2a4\ud2b8\uc787\uacfc \uad6d\ud654 \uaf43\uc774 \ub193\uc544\uc84c\ub098?, context: \ud55c \ub124\ud2f0\uc98c\uc758 \uc81c\uc548\uc73c\ub85c \ud53c\ud574\uc790\uc758 \ucd94\ubaa8 \uc6b4\ub3d9\uc774 \uc2dc\uc791\ub418\uc5b4 \uc0ac\uac74 \ud604\uc7a5\uacfc \uac00\uae4c\uc6b4 \uac15\ub0a8\uc5ed 10\ubc88 \ucd9c\uad6c\uc5d0\ub294 \u201c\uc5ec\uc131 \ud610\uc624\ub294 \uc0ac\ud68c\uc801 \ubb38\uc81c\u201d, \u201c\ub0a8\uc544 \uc788\ub294 \uc5ec\uc131\ub4e4\uc774 \ub354 \uc88b\uc740 \uc138\uc0c1 \ub9cc\ub4e4\uac8c\uc694\u201d \ub4f1 \uc5ec\uc131 \ud610\uc624 \ubb38\uc81c\ub97c \uc9c0\uc801\ud558\ub294 \ub0b4\uc6a9\uc758 \ucabd\uc9c0\ub4e4\uc774 \ubd99\uc5c8\ub2e4. \uac15\ub0a8\uc5ed 10\ubc88 \ucd9c\uad6c \uc0ac\uac74 \ud604\uc7a5\uc744 \uc911\uc2ec\uc73c\ub85c \ud53c\ud574\uc790\ub97c \ucd94\ubaa8\ud558\ub294 \ub73b\uc744 \uc804\ud558\ub294 \ud3ec\uc2a4\ud2b8\uc787\uacfc \uad6d\ud654 \uaf43\uc774 \ub193\uc5ec\uc9c0\ub294 \ub4f1 \uc5ec\uc131 \ud610\uc624 \ubb38\uc81c\uc5d0 \ub300\ud558\uc5ec \uc0ac\ud68c\uc801 \ub17c\ub780\uc774 \uc77c\uc5c8\ub2e4. \ud55c\ud3b8 \uc774 \uc0ac\uac74\uc744 \uc5ec\uc131 \ud610\uc624 \ubc94\uc8c4\ub85c \ud574\uc11d\ud558\ub294 \uac83\uc744 \ube44\uc57d\uc774\ub77c\uace0 \uc8fc\uc7a5\ud558\uba70, \u201c\ud53c\ud574\uc758\uc2dd\uc774 \uc788\ub294 \uc5ec\uc131\ub4e4\uc774 \uc0ac\uac74\uc744 \uc9c0\ub098\uce58\uac8c \ud655\ub300\ud558\uace0 \uc788\ub2e4\u201d, \u201c\ub0a8\uc790\ub77c\ub294 \uc774\uc720\ub9cc\uc73c\ub85c \uc7a0\uc7ac\uc801 \ubc94\uc8c4\uc790 \ucde8\uae09\uc744 \ubc1b\uc544\uc57c \ud558\ub294 \uac70\ub0d0\u201d, \u201c\uac00\ub09c\ud55c \uc0ac\ub78c\uc774 \ubd80\uc790\ub97c \uc0b4\ud574\ud558\uba74 \uac00\ub09c\ud55c \uc0ac\ub78c\ub3c4 \uc7a0\uc7ac\uc801 \uc0b4\uc778\uc790\ub0d0\u201d \ub4f1\uc758 \uae00\uc774 SNS\uc5d0 \uc62c\ub77c\uc624\uace0 \uac15\ub0a8\uc5ed\uc5d0\ub294 \"\ub0a8\uc790\ub77c\uc11c \uc8fd\uc740 \ucc9c\uc548\ud568 \uc6a9\uc0ac\ub4e4\uc744 \uc78a\uc9c0 \ub9d9\uc2dc\ub2e4\"\uc774\ub77c\uace0 \uc4f0\uc5ec\uc9c4 \ub9ac\ubcf8\uc774 \ub2ec\ub9b0 \ud654\ud658\uc774 5\uc6d4 19\uc77c\uc5d0 \ub3c4\ucc29\ud558\uc600\uc73c\ub098, \uc774\ud6c4 \ud53c\ud574 \uc5ec\uc131\uc744 \ucd94\ubaa8\ud558\ub294 \uc2dc\ubbfc\uc5d0 \uc758\ud558\uc5ec \ucd94\ubaa8 \ud3ec\uc2a4\ud2b8\uc787\uc73c\ub85c \ub9ac\ubcf8\uc774 \uac00\ub824\uc84c\uace0, \uacb0\uad6d \ub9ac\ubcf8\uc774 \uc81c\uac70\ub418\uc5c8\ub2e4."} {"question": "\uc0c1\uc5b4\uac00\uc624\ub9ac\ub098 \ubed8\ud64d\uc5b4\ub85c\ub3c4 \ubd88\ub9ac\ub294 \uc0dd\ubb3c\uc740?", "paragraph": "\ub3c5\uc77c\uc758 \ubc15\ubb3c\ud559\uc790 \ub9c8\ub974\ucfe0\uc2a4 \uc5d8\ub808\uc138\ub974 \ube14\ub85c\ud750\uc640 \uc694\ud55c \uace0\ud2c0\ub85c\ud504 \uc288\ub098\uc774\ub354\uac00 1801\ub144 Systema Ichthyologiae\uc5d0 \ubaa9\ud0c1\uc218\uad6c\ub9ac\ub97c \ucc98\uc74c \uae30\uc7ac\ud588\ub2e4. \ubaa8\uc2dd\ud45c\ubcf8\uc740 \uc778\ub3c4 \ucf54\ub85c\ub9cc\ub378 \ud574\uc548\uc5d0\uc11c \ud3ec\ud68d\ub41c \uc2e0\uc7a5 51 \uc13c\ud2f0\ubbf8\ud130\uc758 \uac1c\uccb4\uc600\ub294\ub370, \ud604\uc7ac\ub294 \uc18c\uc2e4\ub418\uc5c8\ub2e4. \uc18d\uba85 \"\ub9ac\ub098(Rhina)\ub294 \uace0\ub300 \uadf8\ub9ac\uc2a4\uc5b4\ub85c \"(\ub3fc\uc9c0 \ub4f1\uc758) \uc8fc\ub465\uc774\"\ub77c\ub294 \ub73b\uc758 \"\ub974\ud788\ub178\uc2a4(\u03c1\u03b9\u03bd\u03bf\u03c3 rhinos)\"\uc5d0\uc11c \uc720\ub798\ud588\uace0, \uc885\uba85 \"\uc548\ud0ac\ub85c\uc2a4\ud1a0\ub9c8(ancylostoma)\"\ub294 \uadf8\ub9ac\uc2a4\uc5b4\ub85c \"\ub4a4\ud2c0\ub9b0\"\uc774\ub77c\ub294 \ub73b\uc758 \"\uc548\ud03c\ub85c\uc2a4(\u03b1\u03bd\u03ba\u03c5\u03bb\u03bf\u03c3 ankylos)\"\uc640 \"\uc785\"\uc774\ub77c\ub294 \ub73b\uc758 \"\uc2a4\ud1a0\ub9c8(\u03c3\u03c4\u03bf\u03bc\u03b1 stoma)\uc5d0\uc11c \uc720\ub798\ud588\ub2e4. \ube14\ub85c\ud750\uc640 \uc288\ub098\uc774\ub354\ub294 \"\uc548\ud0ac\ub85c\uc2a4\ud1a0\ubb34\uc2a4(ancylostomus)\"\ub77c\uace0 \uc37c\uace0 \uadf8\ub807\uac8c \uc4f0\uc778 \ubb38\ud5cc\uc774 \uc77c\ubd80 \uc874\uc7ac\ud558\uc9c0\ub9cc, \uc624\ub298\ub0a0\uc5d0\ub294 \ub300\ubd80\ubd84 \"\uc548\ud0ac\ub85c\uc2a4\ud1a0\ub9c8\"\uac00 \uc62c\ubc14\ub978 \uac83\uc73c\ub85c \uc5ec\uaca8\uc9c4\ub2e4. \uadf8 \uc678\uc5d0 \ubaa9\ud0c1\uc218\uad6c\ub9ac\ub97c \uac00\ub9ac\ud0a4\ub294 \ud1b5\uc0c1\uba85\uc73c\ub85c\ub294 \uc0c1\uc5b4\uac00\uc624\ub9ac(shark ray), \ubed8\ud64d\uc5b4(mud skate), \uc9e7\uc740\ucf54\ubed8\ud64d\uc5b4(shortnose mud skate), \ud65c\uc8fc\ub465\uc774\ucc9c\uc0ac\uace0\uae30(bow-mouthed angel fish), \ud65c\uc8fc\ub465\uc774\ucc9c\uc0ac\uc0c1\uc5b4(bow-mouthed angel shark) \ub4f1\uc774 \uc788\ub2e4.", "answer": "\ubaa9\ud0c1\uc218\uad6c\ub9ac", "sentence": "\ub3c5\uc77c\uc758 \ubc15\ubb3c\ud559\uc790 \ub9c8\ub974\ucfe0\uc2a4 \uc5d8\ub808\uc138\ub974 \ube14\ub85c\ud750\uc640 \uc694\ud55c \uace0\ud2c0\ub85c\ud504 \uc288\ub098\uc774\ub354\uac00 1801\ub144 Systema Ichthyologiae\uc5d0 \ubaa9\ud0c1\uc218\uad6c\ub9ac \ub97c \ucc98\uc74c \uae30\uc7ac\ud588\ub2e4.", "paragraph_sentence": " \ub3c5\uc77c\uc758 \ubc15\ubb3c\ud559\uc790 \ub9c8\ub974\ucfe0\uc2a4 \uc5d8\ub808\uc138\ub974 \ube14\ub85c\ud750\uc640 \uc694\ud55c \uace0\ud2c0\ub85c\ud504 \uc288\ub098\uc774\ub354\uac00 1801\ub144 Systema Ichthyologiae\uc5d0 \ubaa9\ud0c1\uc218\uad6c\ub9ac \ub97c \ucc98\uc74c \uae30\uc7ac\ud588\ub2e4. \ubaa8\uc2dd\ud45c\ubcf8\uc740 \uc778\ub3c4 \ucf54\ub85c\ub9cc\ub378 \ud574\uc548\uc5d0\uc11c \ud3ec\ud68d\ub41c \uc2e0\uc7a5 51 \uc13c\ud2f0\ubbf8\ud130\uc758 \uac1c\uccb4\uc600\ub294\ub370, \ud604\uc7ac\ub294 \uc18c\uc2e4\ub418\uc5c8\ub2e4. \uc18d\uba85 \"\ub9ac\ub098(Rhina)\ub294 \uace0\ub300 \uadf8\ub9ac\uc2a4\uc5b4\ub85c \"(\ub3fc\uc9c0 \ub4f1\uc758) \uc8fc\ub465\uc774\"\ub77c\ub294 \ub73b\uc758 \"\ub974\ud788\ub178\uc2a4(\u03c1\u03b9\u03bd\u03bf\u03c3 rhinos)\"\uc5d0\uc11c \uc720\ub798\ud588\uace0, \uc885\uba85 \"\uc548\ud0ac\ub85c\uc2a4\ud1a0\ub9c8(ancylostoma)\"\ub294 \uadf8\ub9ac\uc2a4\uc5b4\ub85c \"\ub4a4\ud2c0\ub9b0\"\uc774\ub77c\ub294 \ub73b\uc758 \"\uc548\ud03c\ub85c\uc2a4(\u03b1\u03bd\u03ba\u03c5\u03bb\u03bf\u03c3 ankylos)\"\uc640 \"\uc785\"\uc774\ub77c\ub294 \ub73b\uc758 \"\uc2a4\ud1a0\ub9c8(\u03c3\u03c4\u03bf\u03bc\u03b1 stoma)\uc5d0\uc11c \uc720\ub798\ud588\ub2e4. \ube14\ub85c\ud750\uc640 \uc288\ub098\uc774\ub354\ub294 \"\uc548\ud0ac\ub85c\uc2a4\ud1a0\ubb34\uc2a4(ancylostomus)\"\ub77c\uace0 \uc37c\uace0 \uadf8\ub807\uac8c \uc4f0\uc778 \ubb38\ud5cc\uc774 \uc77c\ubd80 \uc874\uc7ac\ud558\uc9c0\ub9cc, \uc624\ub298\ub0a0\uc5d0\ub294 \ub300\ubd80\ubd84 \"\uc548\ud0ac\ub85c\uc2a4\ud1a0\ub9c8\"\uac00 \uc62c\ubc14\ub978 \uac83\uc73c\ub85c \uc5ec\uaca8\uc9c4\ub2e4. \uadf8 \uc678\uc5d0 \ubaa9\ud0c1\uc218\uad6c\ub9ac\ub97c \uac00\ub9ac\ud0a4\ub294 \ud1b5\uc0c1\uba85\uc73c\ub85c\ub294 \uc0c1\uc5b4\uac00\uc624\ub9ac(shark ray), \ubed8\ud64d\uc5b4(mud skate), \uc9e7\uc740\ucf54\ubed8\ud64d\uc5b4(shortnose mud skate), \ud65c\uc8fc\ub465\uc774\ucc9c\uc0ac\uace0\uae30(bow-mouthed angel fish), \ud65c\uc8fc\ub465\uc774\ucc9c\uc0ac\uc0c1\uc5b4(bow-mouthed angel shark) \ub4f1\uc774 \uc788\ub2e4.", "paragraph_answer": "\ub3c5\uc77c\uc758 \ubc15\ubb3c\ud559\uc790 \ub9c8\ub974\ucfe0\uc2a4 \uc5d8\ub808\uc138\ub974 \ube14\ub85c\ud750\uc640 \uc694\ud55c \uace0\ud2c0\ub85c\ud504 \uc288\ub098\uc774\ub354\uac00 1801\ub144 Systema Ichthyologiae\uc5d0 \ubaa9\ud0c1\uc218\uad6c\ub9ac \ub97c \ucc98\uc74c \uae30\uc7ac\ud588\ub2e4. \ubaa8\uc2dd\ud45c\ubcf8\uc740 \uc778\ub3c4 \ucf54\ub85c\ub9cc\ub378 \ud574\uc548\uc5d0\uc11c \ud3ec\ud68d\ub41c \uc2e0\uc7a5 51 \uc13c\ud2f0\ubbf8\ud130\uc758 \uac1c\uccb4\uc600\ub294\ub370, \ud604\uc7ac\ub294 \uc18c\uc2e4\ub418\uc5c8\ub2e4. \uc18d\uba85 \"\ub9ac\ub098(Rhina)\ub294 \uace0\ub300 \uadf8\ub9ac\uc2a4\uc5b4\ub85c \"(\ub3fc\uc9c0 \ub4f1\uc758) \uc8fc\ub465\uc774\"\ub77c\ub294 \ub73b\uc758 \"\ub974\ud788\ub178\uc2a4(\u03c1\u03b9\u03bd\u03bf\u03c3 rhinos)\"\uc5d0\uc11c \uc720\ub798\ud588\uace0, \uc885\uba85 \"\uc548\ud0ac\ub85c\uc2a4\ud1a0\ub9c8(ancylostoma)\"\ub294 \uadf8\ub9ac\uc2a4\uc5b4\ub85c \"\ub4a4\ud2c0\ub9b0\"\uc774\ub77c\ub294 \ub73b\uc758 \"\uc548\ud03c\ub85c\uc2a4(\u03b1\u03bd\u03ba\u03c5\u03bb\u03bf\u03c3 ankylos)\"\uc640 \"\uc785\"\uc774\ub77c\ub294 \ub73b\uc758 \"\uc2a4\ud1a0\ub9c8(\u03c3\u03c4\u03bf\u03bc\u03b1 stoma)\uc5d0\uc11c \uc720\ub798\ud588\ub2e4. \ube14\ub85c\ud750\uc640 \uc288\ub098\uc774\ub354\ub294 \"\uc548\ud0ac\ub85c\uc2a4\ud1a0\ubb34\uc2a4(ancylostomus)\"\ub77c\uace0 \uc37c\uace0 \uadf8\ub807\uac8c \uc4f0\uc778 \ubb38\ud5cc\uc774 \uc77c\ubd80 \uc874\uc7ac\ud558\uc9c0\ub9cc, \uc624\ub298\ub0a0\uc5d0\ub294 \ub300\ubd80\ubd84 \"\uc548\ud0ac\ub85c\uc2a4\ud1a0\ub9c8\"\uac00 \uc62c\ubc14\ub978 \uac83\uc73c\ub85c \uc5ec\uaca8\uc9c4\ub2e4. \uadf8 \uc678\uc5d0 \ubaa9\ud0c1\uc218\uad6c\ub9ac\ub97c \uac00\ub9ac\ud0a4\ub294 \ud1b5\uc0c1\uba85\uc73c\ub85c\ub294 \uc0c1\uc5b4\uac00\uc624\ub9ac(shark ray), \ubed8\ud64d\uc5b4(mud skate), \uc9e7\uc740\ucf54\ubed8\ud64d\uc5b4(shortnose mud skate), \ud65c\uc8fc\ub465\uc774\ucc9c\uc0ac\uace0\uae30(bow-mouthed angel fish), \ud65c\uc8fc\ub465\uc774\ucc9c\uc0ac\uc0c1\uc5b4(bow-mouthed angel shark) \ub4f1\uc774 \uc788\ub2e4.", "sentence_answer": "\ub3c5\uc77c\uc758 \ubc15\ubb3c\ud559\uc790 \ub9c8\ub974\ucfe0\uc2a4 \uc5d8\ub808\uc138\ub974 \ube14\ub85c\ud750\uc640 \uc694\ud55c \uace0\ud2c0\ub85c\ud504 \uc288\ub098\uc774\ub354\uac00 1801\ub144 Systema Ichthyologiae\uc5d0 \ubaa9\ud0c1\uc218\uad6c\ub9ac \ub97c \ucc98\uc74c \uae30\uc7ac\ud588\ub2e4.", "paragraph_id": "6504330-2-2", "paragraph_question": "question: \uc0c1\uc5b4\uac00\uc624\ub9ac\ub098 \ubed8\ud64d\uc5b4\ub85c\ub3c4 \ubd88\ub9ac\ub294 \uc0dd\ubb3c\uc740?, context: \ub3c5\uc77c\uc758 \ubc15\ubb3c\ud559\uc790 \ub9c8\ub974\ucfe0\uc2a4 \uc5d8\ub808\uc138\ub974 \ube14\ub85c\ud750\uc640 \uc694\ud55c \uace0\ud2c0\ub85c\ud504 \uc288\ub098\uc774\ub354\uac00 1801\ub144 Systema Ichthyologiae\uc5d0 \ubaa9\ud0c1\uc218\uad6c\ub9ac\ub97c \ucc98\uc74c \uae30\uc7ac\ud588\ub2e4. \ubaa8\uc2dd\ud45c\ubcf8\uc740 \uc778\ub3c4 \ucf54\ub85c\ub9cc\ub378 \ud574\uc548\uc5d0\uc11c \ud3ec\ud68d\ub41c \uc2e0\uc7a5 51 \uc13c\ud2f0\ubbf8\ud130\uc758 \uac1c\uccb4\uc600\ub294\ub370, \ud604\uc7ac\ub294 \uc18c\uc2e4\ub418\uc5c8\ub2e4. \uc18d\uba85 \"\ub9ac\ub098(Rhina)\ub294 \uace0\ub300 \uadf8\ub9ac\uc2a4\uc5b4\ub85c \"(\ub3fc\uc9c0 \ub4f1\uc758) \uc8fc\ub465\uc774\"\ub77c\ub294 \ub73b\uc758 \"\ub974\ud788\ub178\uc2a4(\u03c1\u03b9\u03bd\u03bf\u03c3 rhinos)\"\uc5d0\uc11c \uc720\ub798\ud588\uace0, \uc885\uba85 \"\uc548\ud0ac\ub85c\uc2a4\ud1a0\ub9c8(ancylostoma)\"\ub294 \uadf8\ub9ac\uc2a4\uc5b4\ub85c \"\ub4a4\ud2c0\ub9b0\"\uc774\ub77c\ub294 \ub73b\uc758 \"\uc548\ud03c\ub85c\uc2a4(\u03b1\u03bd\u03ba\u03c5\u03bb\u03bf\u03c3 ankylos)\"\uc640 \"\uc785\"\uc774\ub77c\ub294 \ub73b\uc758 \"\uc2a4\ud1a0\ub9c8(\u03c3\u03c4\u03bf\u03bc\u03b1 stoma)\uc5d0\uc11c \uc720\ub798\ud588\ub2e4. \ube14\ub85c\ud750\uc640 \uc288\ub098\uc774\ub354\ub294 \"\uc548\ud0ac\ub85c\uc2a4\ud1a0\ubb34\uc2a4(ancylostomus)\"\ub77c\uace0 \uc37c\uace0 \uadf8\ub807\uac8c \uc4f0\uc778 \ubb38\ud5cc\uc774 \uc77c\ubd80 \uc874\uc7ac\ud558\uc9c0\ub9cc, \uc624\ub298\ub0a0\uc5d0\ub294 \ub300\ubd80\ubd84 \"\uc548\ud0ac\ub85c\uc2a4\ud1a0\ub9c8\"\uac00 \uc62c\ubc14\ub978 \uac83\uc73c\ub85c \uc5ec\uaca8\uc9c4\ub2e4. \uadf8 \uc678\uc5d0 \ubaa9\ud0c1\uc218\uad6c\ub9ac\ub97c \uac00\ub9ac\ud0a4\ub294 \ud1b5\uc0c1\uba85\uc73c\ub85c\ub294 \uc0c1\uc5b4\uac00\uc624\ub9ac(shark ray), \ubed8\ud64d\uc5b4(mud skate), \uc9e7\uc740\ucf54\ubed8\ud64d\uc5b4(shortnose mud skate), \ud65c\uc8fc\ub465\uc774\ucc9c\uc0ac\uace0\uae30(bow-mouthed angel fish), \ud65c\uc8fc\ub465\uc774\ucc9c\uc0ac\uc0c1\uc5b4(bow-mouthed angel shark) \ub4f1\uc774 \uc788\ub2e4."} {"question": "\ub098\uce58\uc758 \uc720\ub300\uc778 \ud559\uc0b4\uc740 \uc5b4\ub5a4 \ud504\ub85c\uc81d\ud2b8\uac00 \uc2e4\ud328\ud55c \ud6c4\ubd80\ud130 \uc9c4\ud589\ub418\uc5c8\ub294\uac00?", "paragraph": "\ub3c5\uc77c(\uadf8\ub9ac\uace0 \ub2e4\ub978 \uc5b4\ub514\ub4e0)\uc758 \uac00\uc7a5 \ub3c5\uc124\uc5d0 \ucc2c \uc138 \ubc88\uc9f8 \ubc18\uc720\ub300\uc8fc\uc758\ub294 \ubbfc\uc871\uc8fc\uc758 \ubc18\uc720\ub300\uc8fc\uc758 \ud639\uc740 \uc778\uc885 \ucc28\ubcc4\uc8fc\uc758\ub77c \ubd88\ub9ac\ub294\ub370, \uc774\ub294 \ub9e8 \ucc98\uc74c\uc73c\ub85c \ud3ed\ub825\uc744 \uc815\ub2f9\ud654\ud588\ub2e4. \uc5b4\ub5bb\uac8c\ub4e0 1938\ub144\uc774\ub098 1939\ub144\uc5d0 \ud788\ud2c0\ub7ec\uac00 \ub3c5\uc77c \uc720\ub300\uc778\ub4e4\uc744 \uc5c6\uc560\uae30 \uc704\ud574 \uac15\uc81c\ub85c \uc774\uc8fc\ud558\ub294 \uac83\uc744 \uc54c\uc544\ucc28\ub838\uc5b4\uc57c \ud588\ub2e4. \uadf8\ub54c\ub294 \uc544\uc9c1 \uc720\ub300\uc778\uc744 \uc8fd\uc774\ub294 \uc5b4\ub5a0\ud55c \ud589\uc704\ub3c4 \uc77c\uc5b4\ub098\uc9c0 \uc54a\uc558\uc5c8\ub2e4. \uadf8\ub7ec\ub098 \uc774\uac83\uc740 \ub098\uce58\ub098 \ub2e4\ub978 \uacf3\uc5d0\uc11c \ud3ed\ub825\uc801\uc778 \ubc29\uc2dd\uc744 \uc0ac\uc6a9\ud558\ub294 \uac83\uc744 \ub9dd\uc124\uc774\uc9c0 \uc54a\uc558\ub2e4\uac70\ub098 \uc720\ub300\uc778\uc5d0 \ubc18\ud558\uace0 \uc720\ub300\uc778\uc758 \uc0c1\uc810\uc5d0 \uce68\ud22c\ud558\ub294 \uac83\uc744 \ub9dd\uc124\uc774\uc9c0 \uc54a\uc558\ub2e4\ub294 \uac83\uc744 \ub73b\ud558\uc9c0\ub294 \uc54a\ub294\ub2e4. \ud558\uc9c0\ub9cc \uc804\uc7c1\uc758 \ub450 \ubc88\uc9f8 \ud574\uae4c\uc9c0\ub294 \uc5b4\ub5a0\ud55c \uacf5\uc2dd\uc801\uc778 \ud559\uc0b4\ub3c4 \uc77c\uc5b4\ub098\uc9c0 \uc54a\uc558\uc5c8\ub2e4. \uc774\uac83\uc740 \u2018\uc608\uc57d(reservation)' \ud504\ub85c\uc81d\ud2b8\uac00 \uc2e4\ud328\ud55c \uc774\ud6c4 \uc9c4\ud589\ub418\uc5c8\ub2e4. \uadf8\ub807\ub2e4\uace0 \uc774\uac83\ub4e4\uc774 \uce58\uba85\uc801\uc778 \uc694\uc18c\ub97c \ud3ec\ud568\ud558\uc9c0 \uc54a\uc558\ub2e4\ub294 \uac83\uc744 \ub73b\ud558\uc9c0\ub294 \uc54a\ub294\ub2e4.", "answer": "\uc608\uc57d", "sentence": "\uc774\uac83\uc740 \u2018 \uc608\uc57d (reservation)' \ud504\ub85c\uc81d\ud2b8\uac00 \uc2e4\ud328\ud55c \uc774\ud6c4 \uc9c4\ud589\ub418\uc5c8\ub2e4.", "paragraph_sentence": "\ub3c5\uc77c(\uadf8\ub9ac\uace0 \ub2e4\ub978 \uc5b4\ub514\ub4e0)\uc758 \uac00\uc7a5 \ub3c5\uc124\uc5d0 \ucc2c \uc138 \ubc88\uc9f8 \ubc18\uc720\ub300\uc8fc\uc758\ub294 \ubbfc\uc871\uc8fc\uc758 \ubc18\uc720\ub300\uc8fc\uc758 \ud639\uc740 \uc778\uc885 \ucc28\ubcc4\uc8fc\uc758\ub77c \ubd88\ub9ac\ub294\ub370, \uc774\ub294 \ub9e8 \ucc98\uc74c\uc73c\ub85c \ud3ed\ub825\uc744 \uc815\ub2f9\ud654\ud588\ub2e4. \uc5b4\ub5bb\uac8c\ub4e0 1938\ub144\uc774\ub098 1939\ub144\uc5d0 \ud788\ud2c0\ub7ec\uac00 \ub3c5\uc77c \uc720\ub300\uc778\ub4e4\uc744 \uc5c6\uc560\uae30 \uc704\ud574 \uac15\uc81c\ub85c \uc774\uc8fc\ud558\ub294 \uac83\uc744 \uc54c\uc544\ucc28\ub838\uc5b4\uc57c \ud588\ub2e4. \uadf8\ub54c\ub294 \uc544\uc9c1 \uc720\ub300\uc778\uc744 \uc8fd\uc774\ub294 \uc5b4\ub5a0\ud55c \ud589\uc704\ub3c4 \uc77c\uc5b4\ub098\uc9c0 \uc54a\uc558\uc5c8\ub2e4. \uadf8\ub7ec\ub098 \uc774\uac83\uc740 \ub098\uce58\ub098 \ub2e4\ub978 \uacf3\uc5d0\uc11c \ud3ed\ub825\uc801\uc778 \ubc29\uc2dd\uc744 \uc0ac\uc6a9\ud558\ub294 \uac83\uc744 \ub9dd\uc124\uc774\uc9c0 \uc54a\uc558\ub2e4\uac70\ub098 \uc720\ub300\uc778\uc5d0 \ubc18\ud558\uace0 \uc720\ub300\uc778\uc758 \uc0c1\uc810\uc5d0 \uce68\ud22c\ud558\ub294 \uac83\uc744 \ub9dd\uc124\uc774\uc9c0 \uc54a\uc558\ub2e4\ub294 \uac83\uc744 \ub73b\ud558\uc9c0\ub294 \uc54a\ub294\ub2e4. \ud558\uc9c0\ub9cc \uc804\uc7c1\uc758 \ub450 \ubc88\uc9f8 \ud574\uae4c\uc9c0\ub294 \uc5b4\ub5a0\ud55c \uacf5\uc2dd\uc801\uc778 \ud559\uc0b4\ub3c4 \uc77c\uc5b4\ub098\uc9c0 \uc54a\uc558\uc5c8\ub2e4. \uc774\uac83\uc740 \u2018 \uc608\uc57d (reservation)' \ud504\ub85c\uc81d\ud2b8\uac00 \uc2e4\ud328\ud55c \uc774\ud6c4 \uc9c4\ud589\ub418\uc5c8\ub2e4. \uadf8\ub807\ub2e4\uace0 \uc774\uac83\ub4e4\uc774 \uce58\uba85\uc801\uc778 \uc694\uc18c\ub97c \ud3ec\ud568\ud558\uc9c0 \uc54a\uc558\ub2e4\ub294 \uac83\uc744 \ub73b\ud558\uc9c0\ub294 \uc54a\ub294\ub2e4.", "paragraph_answer": "\ub3c5\uc77c(\uadf8\ub9ac\uace0 \ub2e4\ub978 \uc5b4\ub514\ub4e0)\uc758 \uac00\uc7a5 \ub3c5\uc124\uc5d0 \ucc2c \uc138 \ubc88\uc9f8 \ubc18\uc720\ub300\uc8fc\uc758\ub294 \ubbfc\uc871\uc8fc\uc758 \ubc18\uc720\ub300\uc8fc\uc758 \ud639\uc740 \uc778\uc885 \ucc28\ubcc4\uc8fc\uc758\ub77c \ubd88\ub9ac\ub294\ub370, \uc774\ub294 \ub9e8 \ucc98\uc74c\uc73c\ub85c \ud3ed\ub825\uc744 \uc815\ub2f9\ud654\ud588\ub2e4. \uc5b4\ub5bb\uac8c\ub4e0 1938\ub144\uc774\ub098 1939\ub144\uc5d0 \ud788\ud2c0\ub7ec\uac00 \ub3c5\uc77c \uc720\ub300\uc778\ub4e4\uc744 \uc5c6\uc560\uae30 \uc704\ud574 \uac15\uc81c\ub85c \uc774\uc8fc\ud558\ub294 \uac83\uc744 \uc54c\uc544\ucc28\ub838\uc5b4\uc57c \ud588\ub2e4. \uadf8\ub54c\ub294 \uc544\uc9c1 \uc720\ub300\uc778\uc744 \uc8fd\uc774\ub294 \uc5b4\ub5a0\ud55c \ud589\uc704\ub3c4 \uc77c\uc5b4\ub098\uc9c0 \uc54a\uc558\uc5c8\ub2e4. \uadf8\ub7ec\ub098 \uc774\uac83\uc740 \ub098\uce58\ub098 \ub2e4\ub978 \uacf3\uc5d0\uc11c \ud3ed\ub825\uc801\uc778 \ubc29\uc2dd\uc744 \uc0ac\uc6a9\ud558\ub294 \uac83\uc744 \ub9dd\uc124\uc774\uc9c0 \uc54a\uc558\ub2e4\uac70\ub098 \uc720\ub300\uc778\uc5d0 \ubc18\ud558\uace0 \uc720\ub300\uc778\uc758 \uc0c1\uc810\uc5d0 \uce68\ud22c\ud558\ub294 \uac83\uc744 \ub9dd\uc124\uc774\uc9c0 \uc54a\uc558\ub2e4\ub294 \uac83\uc744 \ub73b\ud558\uc9c0\ub294 \uc54a\ub294\ub2e4. \ud558\uc9c0\ub9cc \uc804\uc7c1\uc758 \ub450 \ubc88\uc9f8 \ud574\uae4c\uc9c0\ub294 \uc5b4\ub5a0\ud55c \uacf5\uc2dd\uc801\uc778 \ud559\uc0b4\ub3c4 \uc77c\uc5b4\ub098\uc9c0 \uc54a\uc558\uc5c8\ub2e4. \uc774\uac83\uc740 \u2018 \uc608\uc57d (reservation)' \ud504\ub85c\uc81d\ud2b8\uac00 \uc2e4\ud328\ud55c \uc774\ud6c4 \uc9c4\ud589\ub418\uc5c8\ub2e4. \uadf8\ub807\ub2e4\uace0 \uc774\uac83\ub4e4\uc774 \uce58\uba85\uc801\uc778 \uc694\uc18c\ub97c \ud3ec\ud568\ud558\uc9c0 \uc54a\uc558\ub2e4\ub294 \uac83\uc744 \ub73b\ud558\uc9c0\ub294 \uc54a\ub294\ub2e4.", "sentence_answer": "\uc774\uac83\uc740 \u2018 \uc608\uc57d (reservation)' \ud504\ub85c\uc81d\ud2b8\uac00 \uc2e4\ud328\ud55c \uc774\ud6c4 \uc9c4\ud589\ub418\uc5c8\ub2e4.", "paragraph_id": "6571960-5-2", "paragraph_question": "question: \ub098\uce58\uc758 \uc720\ub300\uc778 \ud559\uc0b4\uc740 \uc5b4\ub5a4 \ud504\ub85c\uc81d\ud2b8\uac00 \uc2e4\ud328\ud55c \ud6c4\ubd80\ud130 \uc9c4\ud589\ub418\uc5c8\ub294\uac00?, context: \ub3c5\uc77c(\uadf8\ub9ac\uace0 \ub2e4\ub978 \uc5b4\ub514\ub4e0)\uc758 \uac00\uc7a5 \ub3c5\uc124\uc5d0 \ucc2c \uc138 \ubc88\uc9f8 \ubc18\uc720\ub300\uc8fc\uc758\ub294 \ubbfc\uc871\uc8fc\uc758 \ubc18\uc720\ub300\uc8fc\uc758 \ud639\uc740 \uc778\uc885 \ucc28\ubcc4\uc8fc\uc758\ub77c \ubd88\ub9ac\ub294\ub370, \uc774\ub294 \ub9e8 \ucc98\uc74c\uc73c\ub85c \ud3ed\ub825\uc744 \uc815\ub2f9\ud654\ud588\ub2e4. \uc5b4\ub5bb\uac8c\ub4e0 1938\ub144\uc774\ub098 1939\ub144\uc5d0 \ud788\ud2c0\ub7ec\uac00 \ub3c5\uc77c \uc720\ub300\uc778\ub4e4\uc744 \uc5c6\uc560\uae30 \uc704\ud574 \uac15\uc81c\ub85c \uc774\uc8fc\ud558\ub294 \uac83\uc744 \uc54c\uc544\ucc28\ub838\uc5b4\uc57c \ud588\ub2e4. \uadf8\ub54c\ub294 \uc544\uc9c1 \uc720\ub300\uc778\uc744 \uc8fd\uc774\ub294 \uc5b4\ub5a0\ud55c \ud589\uc704\ub3c4 \uc77c\uc5b4\ub098\uc9c0 \uc54a\uc558\uc5c8\ub2e4. \uadf8\ub7ec\ub098 \uc774\uac83\uc740 \ub098\uce58\ub098 \ub2e4\ub978 \uacf3\uc5d0\uc11c \ud3ed\ub825\uc801\uc778 \ubc29\uc2dd\uc744 \uc0ac\uc6a9\ud558\ub294 \uac83\uc744 \ub9dd\uc124\uc774\uc9c0 \uc54a\uc558\ub2e4\uac70\ub098 \uc720\ub300\uc778\uc5d0 \ubc18\ud558\uace0 \uc720\ub300\uc778\uc758 \uc0c1\uc810\uc5d0 \uce68\ud22c\ud558\ub294 \uac83\uc744 \ub9dd\uc124\uc774\uc9c0 \uc54a\uc558\ub2e4\ub294 \uac83\uc744 \ub73b\ud558\uc9c0\ub294 \uc54a\ub294\ub2e4. \ud558\uc9c0\ub9cc \uc804\uc7c1\uc758 \ub450 \ubc88\uc9f8 \ud574\uae4c\uc9c0\ub294 \uc5b4\ub5a0\ud55c \uacf5\uc2dd\uc801\uc778 \ud559\uc0b4\ub3c4 \uc77c\uc5b4\ub098\uc9c0 \uc54a\uc558\uc5c8\ub2e4. \uc774\uac83\uc740 \u2018\uc608\uc57d(reservation)' \ud504\ub85c\uc81d\ud2b8\uac00 \uc2e4\ud328\ud55c \uc774\ud6c4 \uc9c4\ud589\ub418\uc5c8\ub2e4. \uadf8\ub807\ub2e4\uace0 \uc774\uac83\ub4e4\uc774 \uce58\uba85\uc801\uc778 \uc694\uc18c\ub97c \ud3ec\ud568\ud558\uc9c0 \uc54a\uc558\ub2e4\ub294 \uac83\uc744 \ub73b\ud558\uc9c0\ub294 \uc54a\ub294\ub2e4."} {"question": "\ubbf8\ub974\ud0c0\uc790\ud540\uc774 \uc18d\ud574 \uc788\ub294 \ud56d\uc6b0\uc6b8\uc81c \uacc4\ud1b5\uc740?", "paragraph": "\uc0bc\ud658\uacc4 \ud56d\uc6b0\uc6b8\uc81c\uc778 \ud2b8\ub77c\uc870\ub3c8\uacfc \ub178\ub974\uc544\ub4dc\ub808\ub0a0\ub9b0 \ud2b9\uc815 \uc138\ub85c\ud1a0\ub2cc \uc0ac\ud658\uacc4 \ud56d\uc6b0\uc6b8\uc81c(NaSSA)\uc778 \ubbf8\ub974\ud0c0\uc790\ud540(mirtazpine) \ub4f1 \uc9c4\uc815 \ud6a8\uacfc\uac00 \uac15\ud55c \uc57d\ubb3c\ub4e4\uc740 \uc548\uc804\uc131, \ubd80\uc791\uc6a9 \ub4f1\uc758 \ubb38\uc81c\uac00 \uc81c\uae30\ub418\uace0 \uc788\ub294\ub370, \ud2b9\ud788 \uace0\ub839\uc790\uac00 \ubcf5\uc6a9\ud560 \uacbd\uc6b0\uc5d0 \ub1cc\uc878\uc911 \ubc1c\uacac\uc774 \ub2a6\uc5b4\uc9c0\uac70\ub098 \ub118\uc5b4\uc838 \uace8\uc808\ub420 \uc704\ud5d8\uc774 \uc788\uc5b4, \uc774\ub54c\ubb38\uc5d0 \uc0ac\ub9dd \uc704\ud5d8\uae4c\uc9c0 \ub192\uc77c \uac00\ub2a5\uc131\uc774 \uc788\uc73c\ubbc0\ub85c \ub098\uc774 \ub4e0 \uc0ac\ub78c\ub4e4\uc5d0\uac8c \ucc98\ubc29\uc2dc \uc8fc\uc758\ud574\uc57c \ud55c\ub2e4. \ub610\ud55c, \uc870\ud604\ubcd1 \uce58\ub8cc\uc81c \uc911 \ube44\uc815\ud615 \ud56d\uc815\uc2e0\ubcd1\uc81c\uc81c(\ud478\ub9c8\ub974\uc0b0 \ucfe0\uc5d0\ud2f0\uc544\ud540, \uc544\ub9ac\ud53c\ud504\ub77c\uc878)\uc758 \uacbd\uc6b0 \uc77c\ubd80 \uc81c\uc81c\uc640 \ubcd1\uc6a9 \ud22c\uc5ec\uc2dc \ub2e4\uc591\ud55c \ubd80\uc791\uc6a9\uc774 \ub098\ud0c0\ub0a0 \uc218 \uc788\ub2e4. \uc774\uc5d0 \uc2dd\uc57d\ucc98 \uad00\uacc4\uc790\ub294 \"\uc5b4\ub5a4 \uc57d\ubb3c\uc774\uac74 \ubd80\uc791\uc6a9\uc774 \uc5c6\uc744 \uc218\ub294 \uc5c6\ub2e4\"\uba70 \"\uc774\ub4e4 \uc57d\ubb3c\uc5d0 \ub300\ud55c \uc548\uc804\uc131 \ub4f1\uc5d0 \ubb38\uc81c\uac00 \uc0dd\uae34\ub2e4\uba74 \uc870\uce58\ud558\uaca0\ub2e4\"\uace0 \ub9d0\ud558\uc600\ub2e4. \uc57d\ubb3c \ubd80\uc791\uc6a9\uc740 \uc815\ub3c4\uc758 \ucc28\uc774\uac00 \uc788\uc744 \ubfd0 \ud53c\ud560 \uc218 \uc5c6\ub294 \uac83\uc73c\ub85c, \uc774\ub294 \uc57d\ub9ac\ud559\uc801 \ud6a8\uacfc\uc5d0 \ud544\uc5f0\uc801\uc73c\ub85c \ub3d9\ubc18\ub418\ub294 \uac83\uc774\ub2e4. \ub530\ub77c\uc11c, \uce58\ub8cc \ucd08\uae30\uc5d0\ub294 \ud658\uc790\uc758 \uc99d\uc0c1\uacfc \uc870\uac74\uc5d0 \ub530\ub77c \uc801\uc808\ud55c \uc57d\ubb3c\uacfc \uc6a9\ub7c9\uc744 \ucc3e\uae30 \uc704\ud55c \uae30\uac04\uc774 \ud544\uc694\ud560 \uc218 \ubc16\uc5d0 \uc5c6\ub2e4. \ubd80\uc791\uc6a9\uc73c\ub85c \uc758\uc2ec\ub418\ub294 \ud604\uc0c1\uc774 \uc0dd\uae38 \uacbd\uc6b0, \uc989\uac01 \uc815\uc2e0\uac74\uac15\uc758\ud559\uacfc \uc804\ubb38\uc758\ub97c \ucc3e\uc544\uac00 \uc0c1\uc758\ud558\uc5ec \uc57d\ubb3c \ubcc0\uacbd\uc744 \uace0\ub824\ud558\ub294 \uac8c \ubc14\ub78c\uc9c1\ud558\ub2e4.", "answer": "NaSSA", "sentence": "\uc0bc\ud658\uacc4 \ud56d\uc6b0\uc6b8\uc81c\uc778 \ud2b8\ub77c\uc870\ub3c8\uacfc \ub178\ub974\uc544\ub4dc\ub808\ub0a0\ub9b0 \ud2b9\uc815 \uc138\ub85c\ud1a0\ub2cc \uc0ac\ud658\uacc4 \ud56d\uc6b0\uc6b8\uc81c( NaSSA )", "paragraph_sentence": " \uc0bc\ud658\uacc4 \ud56d\uc6b0\uc6b8\uc81c\uc778 \ud2b8\ub77c\uc870\ub3c8\uacfc \ub178\ub974\uc544\ub4dc\ub808\ub0a0\ub9b0 \ud2b9\uc815 \uc138\ub85c\ud1a0\ub2cc \uc0ac\ud658\uacc4 \ud56d\uc6b0\uc6b8\uc81c( NaSSA ) \uc778 \ubbf8\ub974\ud0c0\uc790\ud540(mirtazpine) \ub4f1 \uc9c4\uc815 \ud6a8\uacfc\uac00 \uac15\ud55c \uc57d\ubb3c\ub4e4\uc740 \uc548\uc804\uc131, \ubd80\uc791\uc6a9 \ub4f1\uc758 \ubb38\uc81c\uac00 \uc81c\uae30\ub418\uace0 \uc788\ub294\ub370, \ud2b9\ud788 \uace0\ub839\uc790\uac00 \ubcf5\uc6a9\ud560 \uacbd\uc6b0\uc5d0 \ub1cc\uc878\uc911 \ubc1c\uacac\uc774 \ub2a6\uc5b4\uc9c0\uac70\ub098 \ub118\uc5b4\uc838 \uace8\uc808\ub420 \uc704\ud5d8\uc774 \uc788\uc5b4, \uc774\ub54c\ubb38\uc5d0 \uc0ac\ub9dd \uc704\ud5d8\uae4c\uc9c0 \ub192\uc77c \uac00\ub2a5\uc131\uc774 \uc788\uc73c\ubbc0\ub85c \ub098\uc774 \ub4e0 \uc0ac\ub78c\ub4e4\uc5d0\uac8c \ucc98\ubc29\uc2dc \uc8fc\uc758\ud574\uc57c \ud55c\ub2e4. \ub610\ud55c, \uc870\ud604\ubcd1 \uce58\ub8cc\uc81c \uc911 \ube44\uc815\ud615 \ud56d\uc815\uc2e0\ubcd1\uc81c\uc81c(\ud478\ub9c8\ub974\uc0b0 \ucfe0\uc5d0\ud2f0\uc544\ud540, \uc544\ub9ac\ud53c\ud504\ub77c\uc878)\uc758 \uacbd\uc6b0 \uc77c\ubd80 \uc81c\uc81c\uc640 \ubcd1\uc6a9 \ud22c\uc5ec\uc2dc \ub2e4\uc591\ud55c \ubd80\uc791\uc6a9\uc774 \ub098\ud0c0\ub0a0 \uc218 \uc788\ub2e4. \uc774\uc5d0 \uc2dd\uc57d\ucc98 \uad00\uacc4\uc790\ub294 \"\uc5b4\ub5a4 \uc57d\ubb3c\uc774\uac74 \ubd80\uc791\uc6a9\uc774 \uc5c6\uc744 \uc218\ub294 \uc5c6\ub2e4\"\uba70 \"\uc774\ub4e4 \uc57d\ubb3c\uc5d0 \ub300\ud55c \uc548\uc804\uc131 \ub4f1\uc5d0 \ubb38\uc81c\uac00 \uc0dd\uae34\ub2e4\uba74 \uc870\uce58\ud558\uaca0\ub2e4\"\uace0 \ub9d0\ud558\uc600\ub2e4. \uc57d\ubb3c \ubd80\uc791\uc6a9\uc740 \uc815\ub3c4\uc758 \ucc28\uc774\uac00 \uc788\uc744 \ubfd0 \ud53c\ud560 \uc218 \uc5c6\ub294 \uac83\uc73c\ub85c, \uc774\ub294 \uc57d\ub9ac\ud559\uc801 \ud6a8\uacfc\uc5d0 \ud544\uc5f0\uc801\uc73c\ub85c \ub3d9\ubc18\ub418\ub294 \uac83\uc774\ub2e4. \ub530\ub77c\uc11c, \uce58\ub8cc \ucd08\uae30\uc5d0\ub294 \ud658\uc790\uc758 \uc99d\uc0c1\uacfc \uc870\uac74\uc5d0 \ub530\ub77c \uc801\uc808\ud55c \uc57d\ubb3c\uacfc \uc6a9\ub7c9\uc744 \ucc3e\uae30 \uc704\ud55c \uae30\uac04\uc774 \ud544\uc694\ud560 \uc218 \ubc16\uc5d0 \uc5c6\ub2e4. \ubd80\uc791\uc6a9\uc73c\ub85c \uc758\uc2ec\ub418\ub294 \ud604\uc0c1\uc774 \uc0dd\uae38 \uacbd\uc6b0, \uc989\uac01 \uc815\uc2e0\uac74\uac15\uc758\ud559\uacfc \uc804\ubb38\uc758\ub97c \ucc3e\uc544\uac00 \uc0c1\uc758\ud558\uc5ec \uc57d\ubb3c \ubcc0\uacbd\uc744 \uace0\ub824\ud558\ub294 \uac8c \ubc14\ub78c\uc9c1\ud558\ub2e4.", "paragraph_answer": "\uc0bc\ud658\uacc4 \ud56d\uc6b0\uc6b8\uc81c\uc778 \ud2b8\ub77c\uc870\ub3c8\uacfc \ub178\ub974\uc544\ub4dc\ub808\ub0a0\ub9b0 \ud2b9\uc815 \uc138\ub85c\ud1a0\ub2cc \uc0ac\ud658\uacc4 \ud56d\uc6b0\uc6b8\uc81c( NaSSA )\uc778 \ubbf8\ub974\ud0c0\uc790\ud540(mirtazpine) \ub4f1 \uc9c4\uc815 \ud6a8\uacfc\uac00 \uac15\ud55c \uc57d\ubb3c\ub4e4\uc740 \uc548\uc804\uc131, \ubd80\uc791\uc6a9 \ub4f1\uc758 \ubb38\uc81c\uac00 \uc81c\uae30\ub418\uace0 \uc788\ub294\ub370, \ud2b9\ud788 \uace0\ub839\uc790\uac00 \ubcf5\uc6a9\ud560 \uacbd\uc6b0\uc5d0 \ub1cc\uc878\uc911 \ubc1c\uacac\uc774 \ub2a6\uc5b4\uc9c0\uac70\ub098 \ub118\uc5b4\uc838 \uace8\uc808\ub420 \uc704\ud5d8\uc774 \uc788\uc5b4, \uc774\ub54c\ubb38\uc5d0 \uc0ac\ub9dd \uc704\ud5d8\uae4c\uc9c0 \ub192\uc77c \uac00\ub2a5\uc131\uc774 \uc788\uc73c\ubbc0\ub85c \ub098\uc774 \ub4e0 \uc0ac\ub78c\ub4e4\uc5d0\uac8c \ucc98\ubc29\uc2dc \uc8fc\uc758\ud574\uc57c \ud55c\ub2e4. \ub610\ud55c, \uc870\ud604\ubcd1 \uce58\ub8cc\uc81c \uc911 \ube44\uc815\ud615 \ud56d\uc815\uc2e0\ubcd1\uc81c\uc81c(\ud478\ub9c8\ub974\uc0b0 \ucfe0\uc5d0\ud2f0\uc544\ud540, \uc544\ub9ac\ud53c\ud504\ub77c\uc878)\uc758 \uacbd\uc6b0 \uc77c\ubd80 \uc81c\uc81c\uc640 \ubcd1\uc6a9 \ud22c\uc5ec\uc2dc \ub2e4\uc591\ud55c \ubd80\uc791\uc6a9\uc774 \ub098\ud0c0\ub0a0 \uc218 \uc788\ub2e4. \uc774\uc5d0 \uc2dd\uc57d\ucc98 \uad00\uacc4\uc790\ub294 \"\uc5b4\ub5a4 \uc57d\ubb3c\uc774\uac74 \ubd80\uc791\uc6a9\uc774 \uc5c6\uc744 \uc218\ub294 \uc5c6\ub2e4\"\uba70 \"\uc774\ub4e4 \uc57d\ubb3c\uc5d0 \ub300\ud55c \uc548\uc804\uc131 \ub4f1\uc5d0 \ubb38\uc81c\uac00 \uc0dd\uae34\ub2e4\uba74 \uc870\uce58\ud558\uaca0\ub2e4\"\uace0 \ub9d0\ud558\uc600\ub2e4. \uc57d\ubb3c \ubd80\uc791\uc6a9\uc740 \uc815\ub3c4\uc758 \ucc28\uc774\uac00 \uc788\uc744 \ubfd0 \ud53c\ud560 \uc218 \uc5c6\ub294 \uac83\uc73c\ub85c, \uc774\ub294 \uc57d\ub9ac\ud559\uc801 \ud6a8\uacfc\uc5d0 \ud544\uc5f0\uc801\uc73c\ub85c \ub3d9\ubc18\ub418\ub294 \uac83\uc774\ub2e4. \ub530\ub77c\uc11c, \uce58\ub8cc \ucd08\uae30\uc5d0\ub294 \ud658\uc790\uc758 \uc99d\uc0c1\uacfc \uc870\uac74\uc5d0 \ub530\ub77c \uc801\uc808\ud55c \uc57d\ubb3c\uacfc \uc6a9\ub7c9\uc744 \ucc3e\uae30 \uc704\ud55c \uae30\uac04\uc774 \ud544\uc694\ud560 \uc218 \ubc16\uc5d0 \uc5c6\ub2e4. \ubd80\uc791\uc6a9\uc73c\ub85c \uc758\uc2ec\ub418\ub294 \ud604\uc0c1\uc774 \uc0dd\uae38 \uacbd\uc6b0, \uc989\uac01 \uc815\uc2e0\uac74\uac15\uc758\ud559\uacfc \uc804\ubb38\uc758\ub97c \ucc3e\uc544\uac00 \uc0c1\uc758\ud558\uc5ec \uc57d\ubb3c \ubcc0\uacbd\uc744 \uace0\ub824\ud558\ub294 \uac8c \ubc14\ub78c\uc9c1\ud558\ub2e4.", "sentence_answer": "\uc0bc\ud658\uacc4 \ud56d\uc6b0\uc6b8\uc81c\uc778 \ud2b8\ub77c\uc870\ub3c8\uacfc \ub178\ub974\uc544\ub4dc\ub808\ub0a0\ub9b0 \ud2b9\uc815 \uc138\ub85c\ud1a0\ub2cc \uc0ac\ud658\uacc4 \ud56d\uc6b0\uc6b8\uc81c( NaSSA )", "paragraph_id": "6526295-1-1", "paragraph_question": "question: \ubbf8\ub974\ud0c0\uc790\ud540\uc774 \uc18d\ud574 \uc788\ub294 \ud56d\uc6b0\uc6b8\uc81c \uacc4\ud1b5\uc740?, context: \uc0bc\ud658\uacc4 \ud56d\uc6b0\uc6b8\uc81c\uc778 \ud2b8\ub77c\uc870\ub3c8\uacfc \ub178\ub974\uc544\ub4dc\ub808\ub0a0\ub9b0 \ud2b9\uc815 \uc138\ub85c\ud1a0\ub2cc \uc0ac\ud658\uacc4 \ud56d\uc6b0\uc6b8\uc81c(NaSSA)\uc778 \ubbf8\ub974\ud0c0\uc790\ud540(mirtazpine) \ub4f1 \uc9c4\uc815 \ud6a8\uacfc\uac00 \uac15\ud55c \uc57d\ubb3c\ub4e4\uc740 \uc548\uc804\uc131, \ubd80\uc791\uc6a9 \ub4f1\uc758 \ubb38\uc81c\uac00 \uc81c\uae30\ub418\uace0 \uc788\ub294\ub370, \ud2b9\ud788 \uace0\ub839\uc790\uac00 \ubcf5\uc6a9\ud560 \uacbd\uc6b0\uc5d0 \ub1cc\uc878\uc911 \ubc1c\uacac\uc774 \ub2a6\uc5b4\uc9c0\uac70\ub098 \ub118\uc5b4\uc838 \uace8\uc808\ub420 \uc704\ud5d8\uc774 \uc788\uc5b4, \uc774\ub54c\ubb38\uc5d0 \uc0ac\ub9dd \uc704\ud5d8\uae4c\uc9c0 \ub192\uc77c \uac00\ub2a5\uc131\uc774 \uc788\uc73c\ubbc0\ub85c \ub098\uc774 \ub4e0 \uc0ac\ub78c\ub4e4\uc5d0\uac8c \ucc98\ubc29\uc2dc \uc8fc\uc758\ud574\uc57c \ud55c\ub2e4. \ub610\ud55c, \uc870\ud604\ubcd1 \uce58\ub8cc\uc81c \uc911 \ube44\uc815\ud615 \ud56d\uc815\uc2e0\ubcd1\uc81c\uc81c(\ud478\ub9c8\ub974\uc0b0 \ucfe0\uc5d0\ud2f0\uc544\ud540, \uc544\ub9ac\ud53c\ud504\ub77c\uc878)\uc758 \uacbd\uc6b0 \uc77c\ubd80 \uc81c\uc81c\uc640 \ubcd1\uc6a9 \ud22c\uc5ec\uc2dc \ub2e4\uc591\ud55c \ubd80\uc791\uc6a9\uc774 \ub098\ud0c0\ub0a0 \uc218 \uc788\ub2e4. \uc774\uc5d0 \uc2dd\uc57d\ucc98 \uad00\uacc4\uc790\ub294 \"\uc5b4\ub5a4 \uc57d\ubb3c\uc774\uac74 \ubd80\uc791\uc6a9\uc774 \uc5c6\uc744 \uc218\ub294 \uc5c6\ub2e4\"\uba70 \"\uc774\ub4e4 \uc57d\ubb3c\uc5d0 \ub300\ud55c \uc548\uc804\uc131 \ub4f1\uc5d0 \ubb38\uc81c\uac00 \uc0dd\uae34\ub2e4\uba74 \uc870\uce58\ud558\uaca0\ub2e4\"\uace0 \ub9d0\ud558\uc600\ub2e4. \uc57d\ubb3c \ubd80\uc791\uc6a9\uc740 \uc815\ub3c4\uc758 \ucc28\uc774\uac00 \uc788\uc744 \ubfd0 \ud53c\ud560 \uc218 \uc5c6\ub294 \uac83\uc73c\ub85c, \uc774\ub294 \uc57d\ub9ac\ud559\uc801 \ud6a8\uacfc\uc5d0 \ud544\uc5f0\uc801\uc73c\ub85c \ub3d9\ubc18\ub418\ub294 \uac83\uc774\ub2e4. \ub530\ub77c\uc11c, \uce58\ub8cc \ucd08\uae30\uc5d0\ub294 \ud658\uc790\uc758 \uc99d\uc0c1\uacfc \uc870\uac74\uc5d0 \ub530\ub77c \uc801\uc808\ud55c \uc57d\ubb3c\uacfc \uc6a9\ub7c9\uc744 \ucc3e\uae30 \uc704\ud55c \uae30\uac04\uc774 \ud544\uc694\ud560 \uc218 \ubc16\uc5d0 \uc5c6\ub2e4. \ubd80\uc791\uc6a9\uc73c\ub85c \uc758\uc2ec\ub418\ub294 \ud604\uc0c1\uc774 \uc0dd\uae38 \uacbd\uc6b0, \uc989\uac01 \uc815\uc2e0\uac74\uac15\uc758\ud559\uacfc \uc804\ubb38\uc758\ub97c \ucc3e\uc544\uac00 \uc0c1\uc758\ud558\uc5ec \uc57d\ubb3c \ubcc0\uacbd\uc744 \uace0\ub824\ud558\ub294 \uac8c \ubc14\ub78c\uc9c1\ud558\ub2e4."} {"question": "\uc774\uc815\ud76c\uac00 6.25\uac00 \ub0a8\uce68\uc778\uc9c0 \ubd81\uce68\uc778\uc9c0\uc5d0 \ub300\ud55c \ub2f5\uc744 \ub0b4\ub193\uc740 \uc2dc\uae30\ub294 \uc5b8\uc81c\uc778\uac00?", "paragraph": "\uc774\uc815\ud76c\ub294 2010\ub144 8\uc6d4 4\uc77c KBS \ub77c\ub514\uc624 '\uc5f4\ub9b0\ud1a0\ub860'\uc5d0 \ucd9c\uc5f0, \u201c6\u00b725\uac00 \ub0a8\uce68\uc774\ub0d0, \ubd81\uce68\uc774\ub0d0\u201d\ub294 \uccad\ucde8\uc790\uc758 \uc9c8\ubb38\uc5d0 \uc774\ub807\uac8c \ub2f5\ud588\ub2e4. \u201c\uc5ed\uc0ac\uc801\uc778 \ub17c\uc7c1\ub4e4\uc774 \uc788\ub294 \uac83\uc73c\ub85c \uc54c\uace0 \uc788\uc2b5\ub2c8\ub2e4.\u2026 \uadf8 \ubb38\uc81c\ub294 \uc880 \ub354 \uce58\ubc00\ud558\uac8c \uc0dd\uac01\ud574\uc11c \ub098\uc911\uc5d0 \ube14\ub85c\uadf8\uc5d0 \ub2f5\uc744 \ub4dc\ub9ac\ub294 \uac83\uc73c\ub85c \ud558\uaca0\uc2b5\ub2c8\ub2e4.\u201d\uace0 \ub2f5\ud588\ub2e4. 2011\ub144 4\uc6d4 1\uc77c KBS \ub77c\ub514\uc624'\uc548\ub155\ud558\uc2ed\ub2c8\uae4c \ud64d\uc9c0\uba85\uc785\ub2c8\ub2e4'\uc5d0 \ucd9c\uc5f0, \"\ube14\ub85c\uadf8\uc5d0 \ub2f5\uc744 \uc62c\ub838\ub290\ub0d0\"\ub294 \uc9c4\ud589\uc790\uc758 \uc9c8\ubb38\uc5d0 \"\uc5ed\uc0ac\uc801\uc73c\ub85c\ub294 \uadf8\uac83\uc774 \ub0a8\uce68\uc778 \uac70\ub77c\ub294 \uac8c \ud655\uc778\ub41c \uc77c\uc774\uc9c0\ub9cc \uc6b0\ub9ac\uac00 \uadf8 \uc18d\uc5d0\uc11c \uad50\ud6c8\uc744 \ucc3e\uc744 \uac83\uc740 \ubcc4\ub3c4\ub85c \uc788\ub294 \uac83 \uac19\ub2e4. \uadf8\ub54c \uc6b0\ub9ac \ubbfc\uc871\uc774 \uc804\uccb4\uac00 \uc788\ub294 \uacfc\uc81c\ub294 \uc5b4\ub5bb\uac8c \uce5c\uc77c \uccad\uc0b0\ud558\uace0 \uadf8\ub9ac\uace0 \ubbfc\uc8fc\uc8fc\uc758\uc801\uc73c\ub85c \ubc1c\uc804\ud574 \ub098\uac08 \uac83\uc778\uac00, \ud558\ub294 \ubb38\uc81c\uc600\ub294\ub370 \uadf8\ub807\uac8c \ud558\uae30 \uc704\ud574\uc11c \ud798\uc744 \ubaa8\uc73c\uc9c0 \ubabb\ud55c \uac83\uc740 \ub300\ub2e8\ud788 \uc548\ud0c0\uae5d\ub2e4. \uc5b4\ub5bb\uac8c \ud798\uc744 \ubaa8\uc744\uc9c0, \uc5ed\uc0ac\uc801 \uacfc\uc81c\ub97c \uc5b4\ub5bb\uac8c \uc124\uc815\ud560\uc9c0, \uadf8\uac83\uc774 \uc6b0\ub9ac\uc758 \uacfc\uc81c\ub2e4.\"\ub77c\uace0 \ub2f5\ud588\ub2e4.", "answer": "2011\ub144", "sentence": "2011\ub144 4\uc6d4 1\uc77c KBS \ub77c\ub514\uc624'\uc548\ub155\ud558\uc2ed\ub2c8\uae4c \ud64d\uc9c0\uba85\uc785\ub2c8\ub2e4'\uc5d0 \ucd9c\uc5f0, \"\ube14\ub85c\uadf8\uc5d0 \ub2f5\uc744 \uc62c\ub838\ub290\ub0d0\"\ub294 \uc9c4\ud589\uc790\uc758 \uc9c8\ubb38\uc5d0 \"\uc5ed\uc0ac\uc801\uc73c\ub85c\ub294 \uadf8\uac83\uc774 \ub0a8\uce68\uc778 \uac70\ub77c\ub294 \uac8c \ud655\uc778\ub41c \uc77c\uc774\uc9c0\ub9cc \uc6b0\ub9ac\uac00 \uadf8 \uc18d\uc5d0\uc11c \uad50\ud6c8\uc744 \ucc3e\uc744 \uac83\uc740 \ubcc4\ub3c4\ub85c \uc788\ub294 \uac83 \uac19\ub2e4.", "paragraph_sentence": "\uc774\uc815\ud76c\ub294 2010\ub144 8\uc6d4 4\uc77c KBS \ub77c\ub514\uc624 '\uc5f4\ub9b0\ud1a0\ub860'\uc5d0 \ucd9c\uc5f0, \u201c6\u00b725\uac00 \ub0a8\uce68\uc774\ub0d0, \ubd81\uce68\uc774\ub0d0\u201d\ub294 \uccad\ucde8\uc790\uc758 \uc9c8\ubb38\uc5d0 \uc774\ub807\uac8c \ub2f5\ud588\ub2e4. \u201c\uc5ed\uc0ac\uc801\uc778 \ub17c\uc7c1\ub4e4\uc774 \uc788\ub294 \uac83\uc73c\ub85c \uc54c\uace0 \uc788\uc2b5\ub2c8\ub2e4. \u2026 \uadf8 \ubb38\uc81c\ub294 \uc880 \ub354 \uce58\ubc00\ud558\uac8c \uc0dd\uac01\ud574\uc11c \ub098\uc911\uc5d0 \ube14\ub85c\uadf8\uc5d0 \ub2f5\uc744 \ub4dc\ub9ac\ub294 \uac83\uc73c\ub85c \ud558\uaca0\uc2b5\ub2c8\ub2e4. \u201d\uace0 \ub2f5\ud588\ub2e4. 2011\ub144 4\uc6d4 1\uc77c KBS \ub77c\ub514\uc624'\uc548\ub155\ud558\uc2ed\ub2c8\uae4c \ud64d\uc9c0\uba85\uc785\ub2c8\ub2e4'\uc5d0 \ucd9c\uc5f0, \"\ube14\ub85c\uadf8\uc5d0 \ub2f5\uc744 \uc62c\ub838\ub290\ub0d0\"\ub294 \uc9c4\ud589\uc790\uc758 \uc9c8\ubb38\uc5d0 \"\uc5ed\uc0ac\uc801\uc73c\ub85c\ub294 \uadf8\uac83\uc774 \ub0a8\uce68\uc778 \uac70\ub77c\ub294 \uac8c \ud655\uc778\ub41c \uc77c\uc774\uc9c0\ub9cc \uc6b0\ub9ac\uac00 \uadf8 \uc18d\uc5d0\uc11c \uad50\ud6c8\uc744 \ucc3e\uc744 \uac83\uc740 \ubcc4\ub3c4\ub85c \uc788\ub294 \uac83 \uac19\ub2e4. \uadf8\ub54c \uc6b0\ub9ac \ubbfc\uc871\uc774 \uc804\uccb4\uac00 \uc788\ub294 \uacfc\uc81c\ub294 \uc5b4\ub5bb\uac8c \uce5c\uc77c \uccad\uc0b0\ud558\uace0 \uadf8\ub9ac\uace0 \ubbfc\uc8fc\uc8fc\uc758\uc801\uc73c\ub85c \ubc1c\uc804\ud574 \ub098\uac08 \uac83\uc778\uac00, \ud558\ub294 \ubb38\uc81c\uc600\ub294\ub370 \uadf8\ub807\uac8c \ud558\uae30 \uc704\ud574\uc11c \ud798\uc744 \ubaa8\uc73c\uc9c0 \ubabb\ud55c \uac83\uc740 \ub300\ub2e8\ud788 \uc548\ud0c0\uae5d\ub2e4. \uc5b4\ub5bb\uac8c \ud798\uc744 \ubaa8\uc744\uc9c0, \uc5ed\uc0ac\uc801 \uacfc\uc81c\ub97c \uc5b4\ub5bb\uac8c \uc124\uc815\ud560\uc9c0, \uadf8\uac83\uc774 \uc6b0\ub9ac\uc758 \uacfc\uc81c\ub2e4. \"\ub77c\uace0 \ub2f5\ud588\ub2e4.", "paragraph_answer": "\uc774\uc815\ud76c\ub294 2010\ub144 8\uc6d4 4\uc77c KBS \ub77c\ub514\uc624 '\uc5f4\ub9b0\ud1a0\ub860'\uc5d0 \ucd9c\uc5f0, \u201c6\u00b725\uac00 \ub0a8\uce68\uc774\ub0d0, \ubd81\uce68\uc774\ub0d0\u201d\ub294 \uccad\ucde8\uc790\uc758 \uc9c8\ubb38\uc5d0 \uc774\ub807\uac8c \ub2f5\ud588\ub2e4. \u201c\uc5ed\uc0ac\uc801\uc778 \ub17c\uc7c1\ub4e4\uc774 \uc788\ub294 \uac83\uc73c\ub85c \uc54c\uace0 \uc788\uc2b5\ub2c8\ub2e4.\u2026 \uadf8 \ubb38\uc81c\ub294 \uc880 \ub354 \uce58\ubc00\ud558\uac8c \uc0dd\uac01\ud574\uc11c \ub098\uc911\uc5d0 \ube14\ub85c\uadf8\uc5d0 \ub2f5\uc744 \ub4dc\ub9ac\ub294 \uac83\uc73c\ub85c \ud558\uaca0\uc2b5\ub2c8\ub2e4.\u201d\uace0 \ub2f5\ud588\ub2e4. 2011\ub144 4\uc6d4 1\uc77c KBS \ub77c\ub514\uc624'\uc548\ub155\ud558\uc2ed\ub2c8\uae4c \ud64d\uc9c0\uba85\uc785\ub2c8\ub2e4'\uc5d0 \ucd9c\uc5f0, \"\ube14\ub85c\uadf8\uc5d0 \ub2f5\uc744 \uc62c\ub838\ub290\ub0d0\"\ub294 \uc9c4\ud589\uc790\uc758 \uc9c8\ubb38\uc5d0 \"\uc5ed\uc0ac\uc801\uc73c\ub85c\ub294 \uadf8\uac83\uc774 \ub0a8\uce68\uc778 \uac70\ub77c\ub294 \uac8c \ud655\uc778\ub41c \uc77c\uc774\uc9c0\ub9cc \uc6b0\ub9ac\uac00 \uadf8 \uc18d\uc5d0\uc11c \uad50\ud6c8\uc744 \ucc3e\uc744 \uac83\uc740 \ubcc4\ub3c4\ub85c \uc788\ub294 \uac83 \uac19\ub2e4. \uadf8\ub54c \uc6b0\ub9ac \ubbfc\uc871\uc774 \uc804\uccb4\uac00 \uc788\ub294 \uacfc\uc81c\ub294 \uc5b4\ub5bb\uac8c \uce5c\uc77c \uccad\uc0b0\ud558\uace0 \uadf8\ub9ac\uace0 \ubbfc\uc8fc\uc8fc\uc758\uc801\uc73c\ub85c \ubc1c\uc804\ud574 \ub098\uac08 \uac83\uc778\uac00, \ud558\ub294 \ubb38\uc81c\uc600\ub294\ub370 \uadf8\ub807\uac8c \ud558\uae30 \uc704\ud574\uc11c \ud798\uc744 \ubaa8\uc73c\uc9c0 \ubabb\ud55c \uac83\uc740 \ub300\ub2e8\ud788 \uc548\ud0c0\uae5d\ub2e4. \uc5b4\ub5bb\uac8c \ud798\uc744 \ubaa8\uc744\uc9c0, \uc5ed\uc0ac\uc801 \uacfc\uc81c\ub97c \uc5b4\ub5bb\uac8c \uc124\uc815\ud560\uc9c0, \uadf8\uac83\uc774 \uc6b0\ub9ac\uc758 \uacfc\uc81c\ub2e4.\"\ub77c\uace0 \ub2f5\ud588\ub2e4.", "sentence_answer": " 2011\ub144 4\uc6d4 1\uc77c KBS \ub77c\ub514\uc624'\uc548\ub155\ud558\uc2ed\ub2c8\uae4c \ud64d\uc9c0\uba85\uc785\ub2c8\ub2e4'\uc5d0 \ucd9c\uc5f0, \"\ube14\ub85c\uadf8\uc5d0 \ub2f5\uc744 \uc62c\ub838\ub290\ub0d0\"\ub294 \uc9c4\ud589\uc790\uc758 \uc9c8\ubb38\uc5d0 \"\uc5ed\uc0ac\uc801\uc73c\ub85c\ub294 \uadf8\uac83\uc774 \ub0a8\uce68\uc778 \uac70\ub77c\ub294 \uac8c \ud655\uc778\ub41c \uc77c\uc774\uc9c0\ub9cc \uc6b0\ub9ac\uac00 \uadf8 \uc18d\uc5d0\uc11c \uad50\ud6c8\uc744 \ucc3e\uc744 \uac83\uc740 \ubcc4\ub3c4\ub85c \uc788\ub294 \uac83 \uac19\ub2e4.", "paragraph_id": "6163941-9-1", "paragraph_question": "question: \uc774\uc815\ud76c\uac00 6.25\uac00 \ub0a8\uce68\uc778\uc9c0 \ubd81\uce68\uc778\uc9c0\uc5d0 \ub300\ud55c \ub2f5\uc744 \ub0b4\ub193\uc740 \uc2dc\uae30\ub294 \uc5b8\uc81c\uc778\uac00?, context: \uc774\uc815\ud76c\ub294 2010\ub144 8\uc6d4 4\uc77c KBS \ub77c\ub514\uc624 '\uc5f4\ub9b0\ud1a0\ub860'\uc5d0 \ucd9c\uc5f0, \u201c6\u00b725\uac00 \ub0a8\uce68\uc774\ub0d0, \ubd81\uce68\uc774\ub0d0\u201d\ub294 \uccad\ucde8\uc790\uc758 \uc9c8\ubb38\uc5d0 \uc774\ub807\uac8c \ub2f5\ud588\ub2e4. \u201c\uc5ed\uc0ac\uc801\uc778 \ub17c\uc7c1\ub4e4\uc774 \uc788\ub294 \uac83\uc73c\ub85c \uc54c\uace0 \uc788\uc2b5\ub2c8\ub2e4.\u2026 \uadf8 \ubb38\uc81c\ub294 \uc880 \ub354 \uce58\ubc00\ud558\uac8c \uc0dd\uac01\ud574\uc11c \ub098\uc911\uc5d0 \ube14\ub85c\uadf8\uc5d0 \ub2f5\uc744 \ub4dc\ub9ac\ub294 \uac83\uc73c\ub85c \ud558\uaca0\uc2b5\ub2c8\ub2e4.\u201d\uace0 \ub2f5\ud588\ub2e4. 2011\ub144 4\uc6d4 1\uc77c KBS \ub77c\ub514\uc624'\uc548\ub155\ud558\uc2ed\ub2c8\uae4c \ud64d\uc9c0\uba85\uc785\ub2c8\ub2e4'\uc5d0 \ucd9c\uc5f0, \"\ube14\ub85c\uadf8\uc5d0 \ub2f5\uc744 \uc62c\ub838\ub290\ub0d0\"\ub294 \uc9c4\ud589\uc790\uc758 \uc9c8\ubb38\uc5d0 \"\uc5ed\uc0ac\uc801\uc73c\ub85c\ub294 \uadf8\uac83\uc774 \ub0a8\uce68\uc778 \uac70\ub77c\ub294 \uac8c \ud655\uc778\ub41c \uc77c\uc774\uc9c0\ub9cc \uc6b0\ub9ac\uac00 \uadf8 \uc18d\uc5d0\uc11c \uad50\ud6c8\uc744 \ucc3e\uc744 \uac83\uc740 \ubcc4\ub3c4\ub85c \uc788\ub294 \uac83 \uac19\ub2e4. \uadf8\ub54c \uc6b0\ub9ac \ubbfc\uc871\uc774 \uc804\uccb4\uac00 \uc788\ub294 \uacfc\uc81c\ub294 \uc5b4\ub5bb\uac8c \uce5c\uc77c \uccad\uc0b0\ud558\uace0 \uadf8\ub9ac\uace0 \ubbfc\uc8fc\uc8fc\uc758\uc801\uc73c\ub85c \ubc1c\uc804\ud574 \ub098\uac08 \uac83\uc778\uac00, \ud558\ub294 \ubb38\uc81c\uc600\ub294\ub370 \uadf8\ub807\uac8c \ud558\uae30 \uc704\ud574\uc11c \ud798\uc744 \ubaa8\uc73c\uc9c0 \ubabb\ud55c \uac83\uc740 \ub300\ub2e8\ud788 \uc548\ud0c0\uae5d\ub2e4. \uc5b4\ub5bb\uac8c \ud798\uc744 \ubaa8\uc744\uc9c0, \uc5ed\uc0ac\uc801 \uacfc\uc81c\ub97c \uc5b4\ub5bb\uac8c \uc124\uc815\ud560\uc9c0, \uadf8\uac83\uc774 \uc6b0\ub9ac\uc758 \uacfc\uc81c\ub2e4.\"\ub77c\uace0 \ub2f5\ud588\ub2e4."} {"question": "\uba85\ub098\ub77c\uc5d0\uc11c \uc758\ud559\uc740 \ud64d\ubb34 \uba87\ub144\uc5d0 \ucc3d\uc124\ub418\uc5c8\ub294\uac00?", "paragraph": "\uba85\ub098\ub77c \uc9c0\ubc29\ud589\uc815 \uad6c\uc5ed\uc740 \ub0b4\uc9c0(\u5167\u5730)\uc5d0 \uc18d\ud55c \uc131(\u7701)\u00b7\ubd80(\u5e9c)\u00b7\uc8fc(\u5dde)\u00b7\ud604(\u7e23)\uacfc \ubcc0\uac15(\u908a\u7586)\uc5d0 \uc18d\ud55c \ubd84\ubcc0(\u5206\u908a)\u00b7\uc704\uc18c(\u885b\u6240) \ub4f1\uc774 \uc788\uc5c8\ub2e4. \uc804\uc790\uc758 \ud589\uc815\uad6c\uc5ed\uc5d4 \uc131\uc744 \uc81c\uc678\ud558\uace0\ub294 \ubaa8\ub450 \ud559\uad50\uac00 \uc788\uc5c8\ub294\ub370, \ubd80\ud559(\u5e9c\u5b78)\u00b7\uc8fc\ud559(\u5dde\u5b78)\u00b7\ud604\ud559(\u7e23\u5b78) \ub4f1\uc774 \uadf8\uac83\uc774\ub2e4. \ud6c4\uc790\uc5d0\ub294 \uc704\ud559(\u885b\u5b78)\uc744 \uc124\ub9bd\ud558\uc600\uc73c\uba70, \uc804\uc790\uc640 \ud6c4\uc790\ub97c \ud1b5\uce6d\ud558\uc5ec \uc720\ud559(\u5112\u5b78)\uc774\ub77c \ud55c\ub2e4. \uc804\uad6d\uc5d0 1\ucc9c 579\uac1c\uc18c\uc758 \uc720\ud559\uc774 \uc788\uc5c8\ub2e4. \uc704\ud559\uc740 4\uac1c \uc704(\u885b) \ub610\ub294 2-3\uac1c\uc758 \uc704\uac00 \ud558\ub098\uc758 \ud559\uad50\uad6c\uac00 \ub418\uc5b4 1\uac1c\uc758 \ud559\uad50\ub97c \uc138\uc6e0\ub2e4. \uc804\uad6d 493\uac1c \uc704\uc5d0 180\uc5ec\uac1c \uc18c\uc758 \uc704\ud559\uc774 \uc788\uc5c8\ub2e4. \ubd80\ud559\u00b7\uc8fc\ud559\u00b7\ud604\ud559\uc740 \uac01\uac01 \uaddc\ubaa8\uac00 \ub2ec\ub790\uc9c0\ub9cc \uc0c1\ud558(\u4e0a\u4e0b)\uc758 \uad6c\ubcc4\uc5c6\uc774 \uc911\uc559\uc758 \uad6d\uc790\uac10\uc5d0 \uc9c4\ud559\ud560 \uc790\uaca9\uc744 \uc778\uc815\ubc1b\uc558\ub2e4. \ubd80\ud559\uc740 \uad50\uc218 1\uc778, \ud6c8\ub3c4(\u8a13\u5c0e) 4\uc778\uc774 \uad50\uc721\uc744 \ub9e1\uace0, \uc8fc\ud559\uc740 \ud559\uc815(\u5b78\u6b63) 1\uc778, \ud6c8\ub3c4 3\uc778, \ud604\ud559\uc740 \uad50\uc720(\u654e\u8aed) 1\uc778, \ud6c8\ub3c4 2\uc778\uc774 \uac01\uac01 \uad50\uc721\uc744 \ub2f4\ub2f9\ud588\ub2e4. \uc601\uc885(\u82f1\u5b97) \uc815\ud1b5(\u6b63\u7d71) \uc6d0\ub144\uc5d0 \uc81c\uc870\ud559\uad50\uad00(\u63d0\u8abf\u5b78\u6821\u5b98)\uc744 \uc124\uce58, \uace0\uc2dc\uc640 \uad50\uad00(\u654e\u5b98)\uc744 \uac10\ub3c5\ud558\uace0 \ud559\uc0dd\ub4e4\uc744 \ud6c8\ub3c4\ud558\ub294 \uc77c \ub4f1\uc744 \ub2f4\ub2f9\ucf00 \ud588\ub2e4. \ud559\uc0dd\uc740 3\uae09\uc73c\ub85c \ub098\ub204\uc5b4 1\uae09\uc744 \ub2a0\uc120\uc0dd, 2\uae09\uc744 \uc99d\uad11\uc0dd(\u589e\u5ee3\u751f), 3\uae09\uc744 \ubd80\ud559\uc0dd(\u5e9c\u5b78\u751f)\uc774\ub77c \ud558\uc600\ub2e4. \uc2e0\uc785\uc0dd\uc740 \ubd80\ud559\uc0dd\uc774 \ub418\uba70, \uc5f0\ub9d0\uc2dc\ud5d8\uc778 \uc138\uace0(\u6b72\u8003)\uc758 \uc131\uc801\uc5d0 \ub530\ub77c \uc99d\uad11\uc0dd\u00b7\ub2a0\uc120\uc0dd\uc73c\ub85c \uc2b9\uae09\ud558\uc600\ub2e4. \uc774\ucc98\ub7fc \uc2b9\uae09\ud568\uc73c\ub85c\uc368 \uc815\uc2dd \ud559\uc0dd\uc790\uaca9\uc744 \uc5bb\ub294\ub370, \uc774\ub7ec\ud55c \ud559\uc0dd\uc774 7\ub9cc\uc5ec\uba85\uc5d0 \uc774\ub974\ub800\ub2e4\uace0 \ud55c\ub2e4. \ud64d\ubb34(\u6d2a\u6b66) \ucd08\ub144\uc5d0 \uad50\uacfc\uacfc\uc815\uc774 \ud655\uc815\ub418\uace0 \ud559\uc0dd\uc740 \uac01\uac01 1\uacbd(\u4e00\u7d93)\uc529\uc744 \uc804\uacf5\ud558\uace0 \uc608\u00b7\uc545\u00b7\uc0ac\u00b7\uc5b4\u00b7\uc11c\u00b7\uc218(\u79ae\u6a02\u5c04\u5fa1\u66f8\u6578)\ub97c \ubd84\uacfc(\u5206\u79d1)\ud558\uc5ec \uad50\uc721\ud588\ub2e4. \uc9c0\ubc29\ud559\uad50\ub85c\uc11c \uc9c1\uc5c5\uad50\uc721\uc5d0 \uc18d\ud55c \uac83\uc73c\ub85c \uacbd\uc704\ubb34\ud559(\u4eac\u885b\u6b66\u5b78)\u00b7\uc704\ubb34\ud559(\u885b\u6b66\u5b78)\u00b7\uc758\ud559(\u91ab\u5b78) \ubc0f \uc74c\uc591\ud559(\u9670\u967d\u5b78)\uc758 4\uc885\uc774 \uc788\uc5c8\ub2e4. \uacbd\uc704\ubb34\ud559 \ubc0f \uc704\ubb34\ud559\uc740 \uac01 \uc704(\u885b)\uc758 \ubb34\uc7a5(\u6b66\u5c07)\uc758 \uc790\uc81c\uac00 \uc785\ud559\ud560 \uc218 \uc788\uc5c8\uace0 \uace0\uc2dc(\u8003\u8a66)\ub294 \ubcd1\ubd80(\u5175\u90e8)\uc5d0\uc11c \uc8fc\uad00\ud588\ub2e4. \uc758\ud559\uc740 \ud64d\ubb34 17\ub144\uc5d0 \ucc3d\uc124\ub418\uc5c8\ub294\ub370 \ubd80\uc815\uacfc(\u5e9c\u6b63\u79d1)\u00b7\uc8fc\uc804\uacfc(\u5dde\u5178\u79d1)\u00b7\ud604\ud6c8\uacfc(\u7e23\u8a13\u79d1) \ub4f1\uc758 \ud559\uad00(\u5b78\u5b98)\uc774 \uc788\uc5c8\ub2e4. \uc74c\uc591\ud559\uc740 \uc6d0\ub098\ub77c \uc81c\ub3c4\ub97c \uc804\uc2b9\ud55c \uac83\uc73c\ub85c \uc5ed\uc2dc \ud64d\ubb34 17\ub144\uc5d0 \uc124\ub9bd\ud588\uace0, \ubd80\uc815\uc220(\u5e9c\u6b63\u8853)\u00b7\uc8fc\uc804\uc220(\u5dde\u5178\u8853)\u00b7\ud604\ud6c8\uc220(\u7e23\u8a13\u8853) \ub4f1\uc758 \ud559\uad00\uc774 \uad00\uc7a5\ud588\ub2e4.", "answer": "17\ub144", "sentence": "\uc758\ud559\uc740 \ud64d\ubb34 17\ub144 \uc5d0 \ucc3d\uc124\ub418\uc5c8\ub294\ub370 \ubd80\uc815\uacfc(\u5e9c\u6b63\u79d1)\u00b7\uc8fc\uc804\uacfc(\u5dde\u5178\u79d1)\u00b7\ud604\ud6c8\uacfc(\u7e23\u8a13\u79d1) \ub4f1\uc758 \ud559\uad00(\u5b78\u5b98)\uc774 \uc788\uc5c8\ub2e4.", "paragraph_sentence": "\uba85\ub098\ub77c \uc9c0\ubc29\ud589\uc815 \uad6c\uc5ed\uc740 \ub0b4\uc9c0(\u5167\u5730)\uc5d0 \uc18d\ud55c \uc131(\u7701)\u00b7\ubd80(\u5e9c)\u00b7\uc8fc(\u5dde)\u00b7\ud604(\u7e23)\uacfc \ubcc0\uac15(\u908a\u7586)\uc5d0 \uc18d\ud55c \ubd84\ubcc0(\u5206\u908a)\u00b7\uc704\uc18c(\u885b\u6240) \ub4f1\uc774 \uc788\uc5c8\ub2e4. \uc804\uc790\uc758 \ud589\uc815\uad6c\uc5ed\uc5d4 \uc131\uc744 \uc81c\uc678\ud558\uace0\ub294 \ubaa8\ub450 \ud559\uad50\uac00 \uc788\uc5c8\ub294\ub370, \ubd80\ud559(\u5e9c\u5b78)\u00b7\uc8fc\ud559(\u5dde\u5b78)\u00b7\ud604\ud559(\u7e23\u5b78) \ub4f1\uc774 \uadf8\uac83\uc774\ub2e4. \ud6c4\uc790\uc5d0\ub294 \uc704\ud559(\u885b\u5b78)\uc744 \uc124\ub9bd\ud558\uc600\uc73c\uba70, \uc804\uc790\uc640 \ud6c4\uc790\ub97c \ud1b5\uce6d\ud558\uc5ec \uc720\ud559(\u5112\u5b78)\uc774\ub77c \ud55c\ub2e4. \uc804\uad6d\uc5d0 1\ucc9c 579\uac1c\uc18c\uc758 \uc720\ud559\uc774 \uc788\uc5c8\ub2e4. \uc704\ud559\uc740 4\uac1c \uc704(\u885b) \ub610\ub294 2-3\uac1c\uc758 \uc704\uac00 \ud558\ub098\uc758 \ud559\uad50\uad6c\uac00 \ub418\uc5b4 1\uac1c\uc758 \ud559\uad50\ub97c \uc138\uc6e0\ub2e4. \uc804\uad6d 493\uac1c \uc704\uc5d0 180\uc5ec\uac1c \uc18c\uc758 \uc704\ud559\uc774 \uc788\uc5c8\ub2e4. \ubd80\ud559\u00b7\uc8fc\ud559\u00b7\ud604\ud559\uc740 \uac01\uac01 \uaddc\ubaa8\uac00 \ub2ec\ub790\uc9c0\ub9cc \uc0c1\ud558(\u4e0a\u4e0b)\uc758 \uad6c\ubcc4\uc5c6\uc774 \uc911\uc559\uc758 \uad6d\uc790\uac10\uc5d0 \uc9c4\ud559\ud560 \uc790\uaca9\uc744 \uc778\uc815\ubc1b\uc558\ub2e4. \ubd80\ud559\uc740 \uad50\uc218 1\uc778, \ud6c8\ub3c4(\u8a13\u5c0e) 4\uc778\uc774 \uad50\uc721\uc744 \ub9e1\uace0, \uc8fc\ud559\uc740 \ud559\uc815(\u5b78\u6b63) 1\uc778, \ud6c8\ub3c4 3\uc778, \ud604\ud559\uc740 \uad50\uc720(\u654e\u8aed) 1\uc778, \ud6c8\ub3c4 2\uc778\uc774 \uac01\uac01 \uad50\uc721\uc744 \ub2f4\ub2f9\ud588\ub2e4. \uc601\uc885(\u82f1\u5b97) \uc815\ud1b5(\u6b63\u7d71) \uc6d0\ub144\uc5d0 \uc81c\uc870\ud559\uad50\uad00(\u63d0\u8abf\u5b78\u6821\u5b98)\uc744 \uc124\uce58, \uace0\uc2dc\uc640 \uad50\uad00(\u654e\u5b98)\uc744 \uac10\ub3c5\ud558\uace0 \ud559\uc0dd\ub4e4\uc744 \ud6c8\ub3c4\ud558\ub294 \uc77c \ub4f1\uc744 \ub2f4\ub2f9\ucf00 \ud588\ub2e4. \ud559\uc0dd\uc740 3\uae09\uc73c\ub85c \ub098\ub204\uc5b4 1\uae09\uc744 \ub2a0\uc120\uc0dd, 2\uae09\uc744 \uc99d\uad11\uc0dd(\u589e\u5ee3\u751f), 3\uae09\uc744 \ubd80\ud559\uc0dd(\u5e9c\u5b78\u751f)\uc774\ub77c \ud558\uc600\ub2e4. \uc2e0\uc785\uc0dd\uc740 \ubd80\ud559\uc0dd\uc774 \ub418\uba70, \uc5f0\ub9d0\uc2dc\ud5d8\uc778 \uc138\uace0(\u6b72\u8003)\uc758 \uc131\uc801\uc5d0 \ub530\ub77c \uc99d\uad11\uc0dd\u00b7\ub2a0\uc120\uc0dd\uc73c\ub85c \uc2b9\uae09\ud558\uc600\ub2e4. \uc774\ucc98\ub7fc \uc2b9\uae09\ud568\uc73c\ub85c\uc368 \uc815\uc2dd \ud559\uc0dd\uc790\uaca9\uc744 \uc5bb\ub294\ub370, \uc774\ub7ec\ud55c \ud559\uc0dd\uc774 7\ub9cc\uc5ec\uba85\uc5d0 \uc774\ub974\ub800\ub2e4\uace0 \ud55c\ub2e4. \ud64d\ubb34(\u6d2a\u6b66) \ucd08\ub144\uc5d0 \uad50\uacfc\uacfc\uc815\uc774 \ud655\uc815\ub418\uace0 \ud559\uc0dd\uc740 \uac01\uac01 1\uacbd(\u4e00\u7d93)\uc529\uc744 \uc804\uacf5\ud558\uace0 \uc608\u00b7\uc545\u00b7\uc0ac\u00b7\uc5b4\u00b7\uc11c\u00b7\uc218(\u79ae\u6a02\u5c04\u5fa1\u66f8\u6578)\ub97c \ubd84\uacfc(\u5206\u79d1)\ud558\uc5ec \uad50\uc721\ud588\ub2e4. \uc9c0\ubc29\ud559\uad50\ub85c\uc11c \uc9c1\uc5c5\uad50\uc721\uc5d0 \uc18d\ud55c \uac83\uc73c\ub85c \uacbd\uc704\ubb34\ud559(\u4eac\u885b\u6b66\u5b78)\u00b7\uc704\ubb34\ud559(\u885b\u6b66\u5b78)\u00b7\uc758\ud559(\u91ab\u5b78) \ubc0f \uc74c\uc591\ud559(\u9670\u967d\u5b78)\uc758 4\uc885\uc774 \uc788\uc5c8\ub2e4. \uacbd\uc704\ubb34\ud559 \ubc0f \uc704\ubb34\ud559\uc740 \uac01 \uc704(\u885b)\uc758 \ubb34\uc7a5(\u6b66\u5c07)\uc758 \uc790\uc81c\uac00 \uc785\ud559\ud560 \uc218 \uc788\uc5c8\uace0 \uace0\uc2dc(\u8003\u8a66)\ub294 \ubcd1\ubd80(\u5175\u90e8)\uc5d0\uc11c \uc8fc\uad00\ud588\ub2e4. \uc758\ud559\uc740 \ud64d\ubb34 17\ub144 \uc5d0 \ucc3d\uc124\ub418\uc5c8\ub294\ub370 \ubd80\uc815\uacfc(\u5e9c\u6b63\u79d1)\u00b7\uc8fc\uc804\uacfc(\u5dde\u5178\u79d1)\u00b7\ud604\ud6c8\uacfc(\u7e23\u8a13\u79d1) \ub4f1\uc758 \ud559\uad00(\u5b78\u5b98)\uc774 \uc788\uc5c8\ub2e4. \uc74c\uc591\ud559\uc740 \uc6d0\ub098\ub77c \uc81c\ub3c4\ub97c \uc804\uc2b9\ud55c \uac83\uc73c\ub85c \uc5ed\uc2dc \ud64d\ubb34 17\ub144\uc5d0 \uc124\ub9bd\ud588\uace0, \ubd80\uc815\uc220(\u5e9c\u6b63\u8853)\u00b7\uc8fc\uc804\uc220(\u5dde\u5178\u8853)\u00b7\ud604\ud6c8\uc220(\u7e23\u8a13\u8853) \ub4f1\uc758 \ud559\uad00\uc774 \uad00\uc7a5\ud588\ub2e4.", "paragraph_answer": "\uba85\ub098\ub77c \uc9c0\ubc29\ud589\uc815 \uad6c\uc5ed\uc740 \ub0b4\uc9c0(\u5167\u5730)\uc5d0 \uc18d\ud55c \uc131(\u7701)\u00b7\ubd80(\u5e9c)\u00b7\uc8fc(\u5dde)\u00b7\ud604(\u7e23)\uacfc \ubcc0\uac15(\u908a\u7586)\uc5d0 \uc18d\ud55c \ubd84\ubcc0(\u5206\u908a)\u00b7\uc704\uc18c(\u885b\u6240) \ub4f1\uc774 \uc788\uc5c8\ub2e4. \uc804\uc790\uc758 \ud589\uc815\uad6c\uc5ed\uc5d4 \uc131\uc744 \uc81c\uc678\ud558\uace0\ub294 \ubaa8\ub450 \ud559\uad50\uac00 \uc788\uc5c8\ub294\ub370, \ubd80\ud559(\u5e9c\u5b78)\u00b7\uc8fc\ud559(\u5dde\u5b78)\u00b7\ud604\ud559(\u7e23\u5b78) \ub4f1\uc774 \uadf8\uac83\uc774\ub2e4. \ud6c4\uc790\uc5d0\ub294 \uc704\ud559(\u885b\u5b78)\uc744 \uc124\ub9bd\ud558\uc600\uc73c\uba70, \uc804\uc790\uc640 \ud6c4\uc790\ub97c \ud1b5\uce6d\ud558\uc5ec \uc720\ud559(\u5112\u5b78)\uc774\ub77c \ud55c\ub2e4. \uc804\uad6d\uc5d0 1\ucc9c 579\uac1c\uc18c\uc758 \uc720\ud559\uc774 \uc788\uc5c8\ub2e4. \uc704\ud559\uc740 4\uac1c \uc704(\u885b) \ub610\ub294 2-3\uac1c\uc758 \uc704\uac00 \ud558\ub098\uc758 \ud559\uad50\uad6c\uac00 \ub418\uc5b4 1\uac1c\uc758 \ud559\uad50\ub97c \uc138\uc6e0\ub2e4. \uc804\uad6d 493\uac1c \uc704\uc5d0 180\uc5ec\uac1c \uc18c\uc758 \uc704\ud559\uc774 \uc788\uc5c8\ub2e4. \ubd80\ud559\u00b7\uc8fc\ud559\u00b7\ud604\ud559\uc740 \uac01\uac01 \uaddc\ubaa8\uac00 \ub2ec\ub790\uc9c0\ub9cc \uc0c1\ud558(\u4e0a\u4e0b)\uc758 \uad6c\ubcc4\uc5c6\uc774 \uc911\uc559\uc758 \uad6d\uc790\uac10\uc5d0 \uc9c4\ud559\ud560 \uc790\uaca9\uc744 \uc778\uc815\ubc1b\uc558\ub2e4. \ubd80\ud559\uc740 \uad50\uc218 1\uc778, \ud6c8\ub3c4(\u8a13\u5c0e) 4\uc778\uc774 \uad50\uc721\uc744 \ub9e1\uace0, \uc8fc\ud559\uc740 \ud559\uc815(\u5b78\u6b63) 1\uc778, \ud6c8\ub3c4 3\uc778, \ud604\ud559\uc740 \uad50\uc720(\u654e\u8aed) 1\uc778, \ud6c8\ub3c4 2\uc778\uc774 \uac01\uac01 \uad50\uc721\uc744 \ub2f4\ub2f9\ud588\ub2e4. \uc601\uc885(\u82f1\u5b97) \uc815\ud1b5(\u6b63\u7d71) \uc6d0\ub144\uc5d0 \uc81c\uc870\ud559\uad50\uad00(\u63d0\u8abf\u5b78\u6821\u5b98)\uc744 \uc124\uce58, \uace0\uc2dc\uc640 \uad50\uad00(\u654e\u5b98)\uc744 \uac10\ub3c5\ud558\uace0 \ud559\uc0dd\ub4e4\uc744 \ud6c8\ub3c4\ud558\ub294 \uc77c \ub4f1\uc744 \ub2f4\ub2f9\ucf00 \ud588\ub2e4. \ud559\uc0dd\uc740 3\uae09\uc73c\ub85c \ub098\ub204\uc5b4 1\uae09\uc744 \ub2a0\uc120\uc0dd, 2\uae09\uc744 \uc99d\uad11\uc0dd(\u589e\u5ee3\u751f), 3\uae09\uc744 \ubd80\ud559\uc0dd(\u5e9c\u5b78\u751f)\uc774\ub77c \ud558\uc600\ub2e4. \uc2e0\uc785\uc0dd\uc740 \ubd80\ud559\uc0dd\uc774 \ub418\uba70, \uc5f0\ub9d0\uc2dc\ud5d8\uc778 \uc138\uace0(\u6b72\u8003)\uc758 \uc131\uc801\uc5d0 \ub530\ub77c \uc99d\uad11\uc0dd\u00b7\ub2a0\uc120\uc0dd\uc73c\ub85c \uc2b9\uae09\ud558\uc600\ub2e4. \uc774\ucc98\ub7fc \uc2b9\uae09\ud568\uc73c\ub85c\uc368 \uc815\uc2dd \ud559\uc0dd\uc790\uaca9\uc744 \uc5bb\ub294\ub370, \uc774\ub7ec\ud55c \ud559\uc0dd\uc774 7\ub9cc\uc5ec\uba85\uc5d0 \uc774\ub974\ub800\ub2e4\uace0 \ud55c\ub2e4. \ud64d\ubb34(\u6d2a\u6b66) \ucd08\ub144\uc5d0 \uad50\uacfc\uacfc\uc815\uc774 \ud655\uc815\ub418\uace0 \ud559\uc0dd\uc740 \uac01\uac01 1\uacbd(\u4e00\u7d93)\uc529\uc744 \uc804\uacf5\ud558\uace0 \uc608\u00b7\uc545\u00b7\uc0ac\u00b7\uc5b4\u00b7\uc11c\u00b7\uc218(\u79ae\u6a02\u5c04\u5fa1\u66f8\u6578)\ub97c \ubd84\uacfc(\u5206\u79d1)\ud558\uc5ec \uad50\uc721\ud588\ub2e4. \uc9c0\ubc29\ud559\uad50\ub85c\uc11c \uc9c1\uc5c5\uad50\uc721\uc5d0 \uc18d\ud55c \uac83\uc73c\ub85c \uacbd\uc704\ubb34\ud559(\u4eac\u885b\u6b66\u5b78)\u00b7\uc704\ubb34\ud559(\u885b\u6b66\u5b78)\u00b7\uc758\ud559(\u91ab\u5b78) \ubc0f \uc74c\uc591\ud559(\u9670\u967d\u5b78)\uc758 4\uc885\uc774 \uc788\uc5c8\ub2e4. \uacbd\uc704\ubb34\ud559 \ubc0f \uc704\ubb34\ud559\uc740 \uac01 \uc704(\u885b)\uc758 \ubb34\uc7a5(\u6b66\u5c07)\uc758 \uc790\uc81c\uac00 \uc785\ud559\ud560 \uc218 \uc788\uc5c8\uace0 \uace0\uc2dc(\u8003\u8a66)\ub294 \ubcd1\ubd80(\u5175\u90e8)\uc5d0\uc11c \uc8fc\uad00\ud588\ub2e4. \uc758\ud559\uc740 \ud64d\ubb34 17\ub144 \uc5d0 \ucc3d\uc124\ub418\uc5c8\ub294\ub370 \ubd80\uc815\uacfc(\u5e9c\u6b63\u79d1)\u00b7\uc8fc\uc804\uacfc(\u5dde\u5178\u79d1)\u00b7\ud604\ud6c8\uacfc(\u7e23\u8a13\u79d1) \ub4f1\uc758 \ud559\uad00(\u5b78\u5b98)\uc774 \uc788\uc5c8\ub2e4. \uc74c\uc591\ud559\uc740 \uc6d0\ub098\ub77c \uc81c\ub3c4\ub97c \uc804\uc2b9\ud55c \uac83\uc73c\ub85c \uc5ed\uc2dc \ud64d\ubb34 17\ub144\uc5d0 \uc124\ub9bd\ud588\uace0, \ubd80\uc815\uc220(\u5e9c\u6b63\u8853)\u00b7\uc8fc\uc804\uc220(\u5dde\u5178\u8853)\u00b7\ud604\ud6c8\uc220(\u7e23\u8a13\u8853) \ub4f1\uc758 \ud559\uad00\uc774 \uad00\uc7a5\ud588\ub2e4.", "sentence_answer": "\uc758\ud559\uc740 \ud64d\ubb34 17\ub144 \uc5d0 \ucc3d\uc124\ub418\uc5c8\ub294\ub370 \ubd80\uc815\uacfc(\u5e9c\u6b63\u79d1)\u00b7\uc8fc\uc804\uacfc(\u5dde\u5178\u79d1)\u00b7\ud604\ud6c8\uacfc(\u7e23\u8a13\u79d1) \ub4f1\uc758 \ud559\uad00(\u5b78\u5b98)\uc774 \uc788\uc5c8\ub2e4.", "paragraph_id": "6503200-1-1", "paragraph_question": "question: \uba85\ub098\ub77c\uc5d0\uc11c \uc758\ud559\uc740 \ud64d\ubb34 \uba87\ub144\uc5d0 \ucc3d\uc124\ub418\uc5c8\ub294\uac00?, context: \uba85\ub098\ub77c \uc9c0\ubc29\ud589\uc815 \uad6c\uc5ed\uc740 \ub0b4\uc9c0(\u5167\u5730)\uc5d0 \uc18d\ud55c \uc131(\u7701)\u00b7\ubd80(\u5e9c)\u00b7\uc8fc(\u5dde)\u00b7\ud604(\u7e23)\uacfc \ubcc0\uac15(\u908a\u7586)\uc5d0 \uc18d\ud55c \ubd84\ubcc0(\u5206\u908a)\u00b7\uc704\uc18c(\u885b\u6240) \ub4f1\uc774 \uc788\uc5c8\ub2e4. \uc804\uc790\uc758 \ud589\uc815\uad6c\uc5ed\uc5d4 \uc131\uc744 \uc81c\uc678\ud558\uace0\ub294 \ubaa8\ub450 \ud559\uad50\uac00 \uc788\uc5c8\ub294\ub370, \ubd80\ud559(\u5e9c\u5b78)\u00b7\uc8fc\ud559(\u5dde\u5b78)\u00b7\ud604\ud559(\u7e23\u5b78) \ub4f1\uc774 \uadf8\uac83\uc774\ub2e4. \ud6c4\uc790\uc5d0\ub294 \uc704\ud559(\u885b\u5b78)\uc744 \uc124\ub9bd\ud558\uc600\uc73c\uba70, \uc804\uc790\uc640 \ud6c4\uc790\ub97c \ud1b5\uce6d\ud558\uc5ec \uc720\ud559(\u5112\u5b78)\uc774\ub77c \ud55c\ub2e4. \uc804\uad6d\uc5d0 1\ucc9c 579\uac1c\uc18c\uc758 \uc720\ud559\uc774 \uc788\uc5c8\ub2e4. \uc704\ud559\uc740 4\uac1c \uc704(\u885b) \ub610\ub294 2-3\uac1c\uc758 \uc704\uac00 \ud558\ub098\uc758 \ud559\uad50\uad6c\uac00 \ub418\uc5b4 1\uac1c\uc758 \ud559\uad50\ub97c \uc138\uc6e0\ub2e4. \uc804\uad6d 493\uac1c \uc704\uc5d0 180\uc5ec\uac1c \uc18c\uc758 \uc704\ud559\uc774 \uc788\uc5c8\ub2e4. \ubd80\ud559\u00b7\uc8fc\ud559\u00b7\ud604\ud559\uc740 \uac01\uac01 \uaddc\ubaa8\uac00 \ub2ec\ub790\uc9c0\ub9cc \uc0c1\ud558(\u4e0a\u4e0b)\uc758 \uad6c\ubcc4\uc5c6\uc774 \uc911\uc559\uc758 \uad6d\uc790\uac10\uc5d0 \uc9c4\ud559\ud560 \uc790\uaca9\uc744 \uc778\uc815\ubc1b\uc558\ub2e4. \ubd80\ud559\uc740 \uad50\uc218 1\uc778, \ud6c8\ub3c4(\u8a13\u5c0e) 4\uc778\uc774 \uad50\uc721\uc744 \ub9e1\uace0, \uc8fc\ud559\uc740 \ud559\uc815(\u5b78\u6b63) 1\uc778, \ud6c8\ub3c4 3\uc778, \ud604\ud559\uc740 \uad50\uc720(\u654e\u8aed) 1\uc778, \ud6c8\ub3c4 2\uc778\uc774 \uac01\uac01 \uad50\uc721\uc744 \ub2f4\ub2f9\ud588\ub2e4. \uc601\uc885(\u82f1\u5b97) \uc815\ud1b5(\u6b63\u7d71) \uc6d0\ub144\uc5d0 \uc81c\uc870\ud559\uad50\uad00(\u63d0\u8abf\u5b78\u6821\u5b98)\uc744 \uc124\uce58, \uace0\uc2dc\uc640 \uad50\uad00(\u654e\u5b98)\uc744 \uac10\ub3c5\ud558\uace0 \ud559\uc0dd\ub4e4\uc744 \ud6c8\ub3c4\ud558\ub294 \uc77c \ub4f1\uc744 \ub2f4\ub2f9\ucf00 \ud588\ub2e4. \ud559\uc0dd\uc740 3\uae09\uc73c\ub85c \ub098\ub204\uc5b4 1\uae09\uc744 \ub2a0\uc120\uc0dd, 2\uae09\uc744 \uc99d\uad11\uc0dd(\u589e\u5ee3\u751f), 3\uae09\uc744 \ubd80\ud559\uc0dd(\u5e9c\u5b78\u751f)\uc774\ub77c \ud558\uc600\ub2e4. \uc2e0\uc785\uc0dd\uc740 \ubd80\ud559\uc0dd\uc774 \ub418\uba70, \uc5f0\ub9d0\uc2dc\ud5d8\uc778 \uc138\uace0(\u6b72\u8003)\uc758 \uc131\uc801\uc5d0 \ub530\ub77c \uc99d\uad11\uc0dd\u00b7\ub2a0\uc120\uc0dd\uc73c\ub85c \uc2b9\uae09\ud558\uc600\ub2e4. \uc774\ucc98\ub7fc \uc2b9\uae09\ud568\uc73c\ub85c\uc368 \uc815\uc2dd \ud559\uc0dd\uc790\uaca9\uc744 \uc5bb\ub294\ub370, \uc774\ub7ec\ud55c \ud559\uc0dd\uc774 7\ub9cc\uc5ec\uba85\uc5d0 \uc774\ub974\ub800\ub2e4\uace0 \ud55c\ub2e4. \ud64d\ubb34(\u6d2a\u6b66) \ucd08\ub144\uc5d0 \uad50\uacfc\uacfc\uc815\uc774 \ud655\uc815\ub418\uace0 \ud559\uc0dd\uc740 \uac01\uac01 1\uacbd(\u4e00\u7d93)\uc529\uc744 \uc804\uacf5\ud558\uace0 \uc608\u00b7\uc545\u00b7\uc0ac\u00b7\uc5b4\u00b7\uc11c\u00b7\uc218(\u79ae\u6a02\u5c04\u5fa1\u66f8\u6578)\ub97c \ubd84\uacfc(\u5206\u79d1)\ud558\uc5ec \uad50\uc721\ud588\ub2e4. \uc9c0\ubc29\ud559\uad50\ub85c\uc11c \uc9c1\uc5c5\uad50\uc721\uc5d0 \uc18d\ud55c \uac83\uc73c\ub85c \uacbd\uc704\ubb34\ud559(\u4eac\u885b\u6b66\u5b78)\u00b7\uc704\ubb34\ud559(\u885b\u6b66\u5b78)\u00b7\uc758\ud559(\u91ab\u5b78) \ubc0f \uc74c\uc591\ud559(\u9670\u967d\u5b78)\uc758 4\uc885\uc774 \uc788\uc5c8\ub2e4. \uacbd\uc704\ubb34\ud559 \ubc0f \uc704\ubb34\ud559\uc740 \uac01 \uc704(\u885b)\uc758 \ubb34\uc7a5(\u6b66\u5c07)\uc758 \uc790\uc81c\uac00 \uc785\ud559\ud560 \uc218 \uc788\uc5c8\uace0 \uace0\uc2dc(\u8003\u8a66)\ub294 \ubcd1\ubd80(\u5175\u90e8)\uc5d0\uc11c \uc8fc\uad00\ud588\ub2e4. \uc758\ud559\uc740 \ud64d\ubb34 17\ub144\uc5d0 \ucc3d\uc124\ub418\uc5c8\ub294\ub370 \ubd80\uc815\uacfc(\u5e9c\u6b63\u79d1)\u00b7\uc8fc\uc804\uacfc(\u5dde\u5178\u79d1)\u00b7\ud604\ud6c8\uacfc(\u7e23\u8a13\u79d1) \ub4f1\uc758 \ud559\uad00(\u5b78\u5b98)\uc774 \uc788\uc5c8\ub2e4. \uc74c\uc591\ud559\uc740 \uc6d0\ub098\ub77c \uc81c\ub3c4\ub97c \uc804\uc2b9\ud55c \uac83\uc73c\ub85c \uc5ed\uc2dc \ud64d\ubb34 17\ub144\uc5d0 \uc124\ub9bd\ud588\uace0, \ubd80\uc815\uc220(\u5e9c\u6b63\u8853)\u00b7\uc8fc\uc804\uc220(\u5dde\u5178\u8853)\u00b7\ud604\ud6c8\uc220(\u7e23\u8a13\u8853) \ub4f1\uc758 \ud559\uad00\uc774 \uad00\uc7a5\ud588\ub2e4."} {"question": "\uc18c\ud589\uc131\uccb4\uc13c\ud130\uc7a5\uc740 \uc138\ub4dc\ub098\ub77c\ub294 \uc774\ub984\uc744 \ubbf8\ub9ac \ubd99\uc778 \uc5f0\uad6c\uc9c4\uc5d0\uac8c \ub204\uac00 \uc774 \uc774\ub984\uc5d0 \ubc18\ub300\ud45c\ub97c \ub358\uc9c8 \uac00\ub2a5\uc131\uc774 \uc788\ub2e4\uace0 \uacbd\uace0\ud558\uc600\ub294\uac00?", "paragraph": "\ub9c8\uc774\ud074 \ube0c\ub77c\uc6b4\uc740 \uc790\uae30 \uc6f9\uc0ac\uc774\ud2b8\uc5d0 \u201c\uc774\ubc88\uc5d0 \uc0c8\ub85c \ubc1c\uacac\ub41c \ucc9c\uccb4\ub294 \uc9c0\uae08\uae4c\uc9c0 \ud0dc\uc591\uacc4\uc5d0\uc11c \ubc1c\uacac\ub41c \ucc9c\uccb4 \uc911 \uac00\uc7a5 \ucda5\uace0 \uac00\uc7a5 \uba3c \uacf3\uc5d0 \uc788\ub2e4,\u201d \u201c\uadf8\ub798\uc11c \uc6b0\ub9ac\ub294 \uc774\ub204\uc774\ud2b8 \uc2e0\ud654\uc5d0\uc11c \uc5bc\uc5b4\ubd99\uc740 \ubd81\ube59\uc591 \ubc14\ub2e5\uc5d0 \uc0b0\ub2e4\uace0 \ud558\ub294 \ubc14\ub2e4\uc758 \uc5ec\uc2e0 \uc138\ub4dc\ub098\uc758 \uc774\ub984\uc744 \ubd99\uc774\ub294 \uac83\uc774 \uc801\uc808\ud558\ub2e4\uace0 \uc0dd\uac01\ud588\ub2e4\u201d\ub77c\uace0 \uc368\uc11c \uc62c\ub838\ub2e4. \ube0c\ub77c\uc6b4\uc740 \uad6d\uc81c\ucc9c\ubb38\uc5f0\ub9f9(IAU) \uc18c\ud589\uc131\uccb4\uc13c\ud130\uc5d0 \uc55e\uc73c\ub85c \uc138\ub4dc\ub098\uc758 \uada4\ub3c4 \uc9c0\uc5ed\uc5d0\uc11c \ubc1c\uacac\ub418\ub294 \ucc9c\uccb4\ub4e4\uc740 \ubd81\uadf9 \uc2e0\ud654\uc5d0\uc11c \uc774\ub984\uc744 \ub530\uc11c \uba85\uba85\ud574\uc57c \ud55c\ub2e4\uace0 \uc8fc\uc7a5\ud588\ub2e4. \uc5f0\uad6c\uc9c4\uc740 \uacf5\uc2dd \ubc88\ud638\uac00 \ub9e4\uaca8\uc9c0\uae30\ub3c4 \uc804\uc5d0 \"\uc138\ub4dc\ub098\"\ub77c\ub294 \uc774\ub984\ubd80\ud130 \uba3c\uc800 \uc9c0\uc5b4\uc11c \ubc1c\ud45c\ud588\ub2e4. \uc18c\ud589\uc131\uccb4\uc13c\ud130\uc7a5 \ube0c\ub77c\uc774\uc5b8 G. \ub9c8\uc2a4\ub374\uc740 \uc774\ub7f0 \ud589\ub3d9\uc740 \uaddc\uc57d \uc704\ubc18\uc774\uba70, IAU \ud68c\uc6d0 \uc77c\ubd80\ub294 \ubc18\ub300\ud45c\ub97c \ub358\uc9c8 \uc218 \uc788\ub2e4\uace0 \uacbd\uace0\ud588\ub2e4. \uadf8\ub7ec\ub098 \uc138\ub4dc\ub098\ub77c\ub294 \uc774\ub984\uc5d0 \ubc18\ub300\ud558\ub294 \uc758\uacac\uc740 \uc5c6\uc5c8\uace0, \uadf8 \uc678\uc5d0 \ub2e4\ub978 \uacbd\ud569\ud560 \ub9cc\ud55c \uc774\ub984\uc774 \uc81c\uc2dc\ub418\uc9c0\ub3c4 \uc54a\uc558\ub2e4. IAU \uc18c\ucc9c\uccb4\uba85\uba85\uc704\uc6d0\ud68c\ub294 \uc138\ub4dc\ub098\ub77c\ub294 \uc774\ub984\uc744 2004\ub144 9\uc6d4\uc5d0 \uc815\uc2dd\uc73c\ub85c \ubc1b\uc544\ub4e4\uc774\uace0, \uc55e\uc73c\ub85c\ub3c4 \uc774\uc640 \uac19\uc774 \ube44\uc0c1\ud55c \uad00\uc2ec\uc758 \ub300\uc0c1\uc778 \uacbd\uc6b0 \ubc88\ud638\uac00 \ub9e4\uaca8\uc9c0\uae30 \uc804\uc5d0 \uc774\ub984\uc744 \uba3c\uc800 \ubc1c\ud45c\ud560 \uc218\ub3c4 \uc788\ub2e4\ub294 \uacb0\ub860\uc744 \ub0b4\ub838\ub2e4.", "answer": "IAU \ud68c\uc6d0", "sentence": "\uc18c\ud589\uc131\uccb4\uc13c\ud130\uc7a5 \ube0c\ub77c\uc774\uc5b8 G. \ub9c8\uc2a4\ub374\uc740 \uc774\ub7f0 \ud589\ub3d9\uc740 \uaddc\uc57d \uc704\ubc18\uc774\uba70, IAU \ud68c\uc6d0 \uc77c\ubd80\ub294 \ubc18\ub300\ud45c\ub97c \ub358\uc9c8 \uc218 \uc788\ub2e4\uace0 \uacbd\uace0\ud588\ub2e4.", "paragraph_sentence": "\ub9c8\uc774\ud074 \ube0c\ub77c\uc6b4\uc740 \uc790\uae30 \uc6f9\uc0ac\uc774\ud2b8\uc5d0 \u201c\uc774\ubc88\uc5d0 \uc0c8\ub85c \ubc1c\uacac\ub41c \ucc9c\uccb4\ub294 \uc9c0\uae08\uae4c\uc9c0 \ud0dc\uc591\uacc4\uc5d0\uc11c \ubc1c\uacac\ub41c \ucc9c\uccb4 \uc911 \uac00\uc7a5 \ucda5\uace0 \uac00\uc7a5 \uba3c \uacf3\uc5d0 \uc788\ub2e4,\u201d \u201c\uadf8\ub798\uc11c \uc6b0\ub9ac\ub294 \uc774\ub204\uc774\ud2b8 \uc2e0\ud654\uc5d0\uc11c \uc5bc\uc5b4\ubd99\uc740 \ubd81\ube59\uc591 \ubc14\ub2e5\uc5d0 \uc0b0\ub2e4\uace0 \ud558\ub294 \ubc14\ub2e4\uc758 \uc5ec\uc2e0 \uc138\ub4dc\ub098\uc758 \uc774\ub984\uc744 \ubd99\uc774\ub294 \uac83\uc774 \uc801\uc808\ud558\ub2e4\uace0 \uc0dd\uac01\ud588\ub2e4\u201d\ub77c\uace0 \uc368\uc11c \uc62c\ub838\ub2e4. \ube0c\ub77c\uc6b4\uc740 \uad6d\uc81c\ucc9c\ubb38\uc5f0\ub9f9(IAU) \uc18c\ud589\uc131\uccb4\uc13c\ud130\uc5d0 \uc55e\uc73c\ub85c \uc138\ub4dc\ub098\uc758 \uada4\ub3c4 \uc9c0\uc5ed\uc5d0\uc11c \ubc1c\uacac\ub418\ub294 \ucc9c\uccb4\ub4e4\uc740 \ubd81\uadf9 \uc2e0\ud654\uc5d0\uc11c \uc774\ub984\uc744 \ub530\uc11c \uba85\uba85\ud574\uc57c \ud55c\ub2e4\uace0 \uc8fc\uc7a5\ud588\ub2e4. \uc5f0\uad6c\uc9c4\uc740 \uacf5\uc2dd \ubc88\ud638\uac00 \ub9e4\uaca8\uc9c0\uae30\ub3c4 \uc804\uc5d0 \"\uc138\ub4dc\ub098\"\ub77c\ub294 \uc774\ub984\ubd80\ud130 \uba3c\uc800 \uc9c0\uc5b4\uc11c \ubc1c\ud45c\ud588\ub2e4. \uc18c\ud589\uc131\uccb4\uc13c\ud130\uc7a5 \ube0c\ub77c\uc774\uc5b8 G. \ub9c8\uc2a4\ub374\uc740 \uc774\ub7f0 \ud589\ub3d9\uc740 \uaddc\uc57d \uc704\ubc18\uc774\uba70, IAU \ud68c\uc6d0 \uc77c\ubd80\ub294 \ubc18\ub300\ud45c\ub97c \ub358\uc9c8 \uc218 \uc788\ub2e4\uace0 \uacbd\uace0\ud588\ub2e4. \uadf8\ub7ec\ub098 \uc138\ub4dc\ub098\ub77c\ub294 \uc774\ub984\uc5d0 \ubc18\ub300\ud558\ub294 \uc758\uacac\uc740 \uc5c6\uc5c8\uace0, \uadf8 \uc678\uc5d0 \ub2e4\ub978 \uacbd\ud569\ud560 \ub9cc\ud55c \uc774\ub984\uc774 \uc81c\uc2dc\ub418\uc9c0\ub3c4 \uc54a\uc558\ub2e4. IAU \uc18c\ucc9c\uccb4\uba85\uba85\uc704\uc6d0\ud68c\ub294 \uc138\ub4dc\ub098\ub77c\ub294 \uc774\ub984\uc744 2004\ub144 9\uc6d4\uc5d0 \uc815\uc2dd\uc73c\ub85c \ubc1b\uc544\ub4e4\uc774\uace0, \uc55e\uc73c\ub85c\ub3c4 \uc774\uc640 \uac19\uc774 \ube44\uc0c1\ud55c \uad00\uc2ec\uc758 \ub300\uc0c1\uc778 \uacbd\uc6b0 \ubc88\ud638\uac00 \ub9e4\uaca8\uc9c0\uae30 \uc804\uc5d0 \uc774\ub984\uc744 \uba3c\uc800 \ubc1c\ud45c\ud560 \uc218\ub3c4 \uc788\ub2e4\ub294 \uacb0\ub860\uc744 \ub0b4\ub838\ub2e4.", "paragraph_answer": "\ub9c8\uc774\ud074 \ube0c\ub77c\uc6b4\uc740 \uc790\uae30 \uc6f9\uc0ac\uc774\ud2b8\uc5d0 \u201c\uc774\ubc88\uc5d0 \uc0c8\ub85c \ubc1c\uacac\ub41c \ucc9c\uccb4\ub294 \uc9c0\uae08\uae4c\uc9c0 \ud0dc\uc591\uacc4\uc5d0\uc11c \ubc1c\uacac\ub41c \ucc9c\uccb4 \uc911 \uac00\uc7a5 \ucda5\uace0 \uac00\uc7a5 \uba3c \uacf3\uc5d0 \uc788\ub2e4,\u201d \u201c\uadf8\ub798\uc11c \uc6b0\ub9ac\ub294 \uc774\ub204\uc774\ud2b8 \uc2e0\ud654\uc5d0\uc11c \uc5bc\uc5b4\ubd99\uc740 \ubd81\ube59\uc591 \ubc14\ub2e5\uc5d0 \uc0b0\ub2e4\uace0 \ud558\ub294 \ubc14\ub2e4\uc758 \uc5ec\uc2e0 \uc138\ub4dc\ub098\uc758 \uc774\ub984\uc744 \ubd99\uc774\ub294 \uac83\uc774 \uc801\uc808\ud558\ub2e4\uace0 \uc0dd\uac01\ud588\ub2e4\u201d\ub77c\uace0 \uc368\uc11c \uc62c\ub838\ub2e4. \ube0c\ub77c\uc6b4\uc740 \uad6d\uc81c\ucc9c\ubb38\uc5f0\ub9f9(IAU) \uc18c\ud589\uc131\uccb4\uc13c\ud130\uc5d0 \uc55e\uc73c\ub85c \uc138\ub4dc\ub098\uc758 \uada4\ub3c4 \uc9c0\uc5ed\uc5d0\uc11c \ubc1c\uacac\ub418\ub294 \ucc9c\uccb4\ub4e4\uc740 \ubd81\uadf9 \uc2e0\ud654\uc5d0\uc11c \uc774\ub984\uc744 \ub530\uc11c \uba85\uba85\ud574\uc57c \ud55c\ub2e4\uace0 \uc8fc\uc7a5\ud588\ub2e4. \uc5f0\uad6c\uc9c4\uc740 \uacf5\uc2dd \ubc88\ud638\uac00 \ub9e4\uaca8\uc9c0\uae30\ub3c4 \uc804\uc5d0 \"\uc138\ub4dc\ub098\"\ub77c\ub294 \uc774\ub984\ubd80\ud130 \uba3c\uc800 \uc9c0\uc5b4\uc11c \ubc1c\ud45c\ud588\ub2e4. \uc18c\ud589\uc131\uccb4\uc13c\ud130\uc7a5 \ube0c\ub77c\uc774\uc5b8 G. \ub9c8\uc2a4\ub374\uc740 \uc774\ub7f0 \ud589\ub3d9\uc740 \uaddc\uc57d \uc704\ubc18\uc774\uba70, IAU \ud68c\uc6d0 \uc77c\ubd80\ub294 \ubc18\ub300\ud45c\ub97c \ub358\uc9c8 \uc218 \uc788\ub2e4\uace0 \uacbd\uace0\ud588\ub2e4. \uadf8\ub7ec\ub098 \uc138\ub4dc\ub098\ub77c\ub294 \uc774\ub984\uc5d0 \ubc18\ub300\ud558\ub294 \uc758\uacac\uc740 \uc5c6\uc5c8\uace0, \uadf8 \uc678\uc5d0 \ub2e4\ub978 \uacbd\ud569\ud560 \ub9cc\ud55c \uc774\ub984\uc774 \uc81c\uc2dc\ub418\uc9c0\ub3c4 \uc54a\uc558\ub2e4. IAU \uc18c\ucc9c\uccb4\uba85\uba85\uc704\uc6d0\ud68c\ub294 \uc138\ub4dc\ub098\ub77c\ub294 \uc774\ub984\uc744 2004\ub144 9\uc6d4\uc5d0 \uc815\uc2dd\uc73c\ub85c \ubc1b\uc544\ub4e4\uc774\uace0, \uc55e\uc73c\ub85c\ub3c4 \uc774\uc640 \uac19\uc774 \ube44\uc0c1\ud55c \uad00\uc2ec\uc758 \ub300\uc0c1\uc778 \uacbd\uc6b0 \ubc88\ud638\uac00 \ub9e4\uaca8\uc9c0\uae30 \uc804\uc5d0 \uc774\ub984\uc744 \uba3c\uc800 \ubc1c\ud45c\ud560 \uc218\ub3c4 \uc788\ub2e4\ub294 \uacb0\ub860\uc744 \ub0b4\ub838\ub2e4.", "sentence_answer": "\uc18c\ud589\uc131\uccb4\uc13c\ud130\uc7a5 \ube0c\ub77c\uc774\uc5b8 G. \ub9c8\uc2a4\ub374\uc740 \uc774\ub7f0 \ud589\ub3d9\uc740 \uaddc\uc57d \uc704\ubc18\uc774\uba70, IAU \ud68c\uc6d0 \uc77c\ubd80\ub294 \ubc18\ub300\ud45c\ub97c \ub358\uc9c8 \uc218 \uc788\ub2e4\uace0 \uacbd\uace0\ud588\ub2e4.", "paragraph_id": "6568339-2-2", "paragraph_question": "question: \uc18c\ud589\uc131\uccb4\uc13c\ud130\uc7a5\uc740 \uc138\ub4dc\ub098\ub77c\ub294 \uc774\ub984\uc744 \ubbf8\ub9ac \ubd99\uc778 \uc5f0\uad6c\uc9c4\uc5d0\uac8c \ub204\uac00 \uc774 \uc774\ub984\uc5d0 \ubc18\ub300\ud45c\ub97c \ub358\uc9c8 \uac00\ub2a5\uc131\uc774 \uc788\ub2e4\uace0 \uacbd\uace0\ud558\uc600\ub294\uac00?, context: \ub9c8\uc774\ud074 \ube0c\ub77c\uc6b4\uc740 \uc790\uae30 \uc6f9\uc0ac\uc774\ud2b8\uc5d0 \u201c\uc774\ubc88\uc5d0 \uc0c8\ub85c \ubc1c\uacac\ub41c \ucc9c\uccb4\ub294 \uc9c0\uae08\uae4c\uc9c0 \ud0dc\uc591\uacc4\uc5d0\uc11c \ubc1c\uacac\ub41c \ucc9c\uccb4 \uc911 \uac00\uc7a5 \ucda5\uace0 \uac00\uc7a5 \uba3c \uacf3\uc5d0 \uc788\ub2e4,\u201d \u201c\uadf8\ub798\uc11c \uc6b0\ub9ac\ub294 \uc774\ub204\uc774\ud2b8 \uc2e0\ud654\uc5d0\uc11c \uc5bc\uc5b4\ubd99\uc740 \ubd81\ube59\uc591 \ubc14\ub2e5\uc5d0 \uc0b0\ub2e4\uace0 \ud558\ub294 \ubc14\ub2e4\uc758 \uc5ec\uc2e0 \uc138\ub4dc\ub098\uc758 \uc774\ub984\uc744 \ubd99\uc774\ub294 \uac83\uc774 \uc801\uc808\ud558\ub2e4\uace0 \uc0dd\uac01\ud588\ub2e4\u201d\ub77c\uace0 \uc368\uc11c \uc62c\ub838\ub2e4. \ube0c\ub77c\uc6b4\uc740 \uad6d\uc81c\ucc9c\ubb38\uc5f0\ub9f9(IAU) \uc18c\ud589\uc131\uccb4\uc13c\ud130\uc5d0 \uc55e\uc73c\ub85c \uc138\ub4dc\ub098\uc758 \uada4\ub3c4 \uc9c0\uc5ed\uc5d0\uc11c \ubc1c\uacac\ub418\ub294 \ucc9c\uccb4\ub4e4\uc740 \ubd81\uadf9 \uc2e0\ud654\uc5d0\uc11c \uc774\ub984\uc744 \ub530\uc11c \uba85\uba85\ud574\uc57c \ud55c\ub2e4\uace0 \uc8fc\uc7a5\ud588\ub2e4. \uc5f0\uad6c\uc9c4\uc740 \uacf5\uc2dd \ubc88\ud638\uac00 \ub9e4\uaca8\uc9c0\uae30\ub3c4 \uc804\uc5d0 \"\uc138\ub4dc\ub098\"\ub77c\ub294 \uc774\ub984\ubd80\ud130 \uba3c\uc800 \uc9c0\uc5b4\uc11c \ubc1c\ud45c\ud588\ub2e4. \uc18c\ud589\uc131\uccb4\uc13c\ud130\uc7a5 \ube0c\ub77c\uc774\uc5b8 G. \ub9c8\uc2a4\ub374\uc740 \uc774\ub7f0 \ud589\ub3d9\uc740 \uaddc\uc57d \uc704\ubc18\uc774\uba70, IAU \ud68c\uc6d0 \uc77c\ubd80\ub294 \ubc18\ub300\ud45c\ub97c \ub358\uc9c8 \uc218 \uc788\ub2e4\uace0 \uacbd\uace0\ud588\ub2e4. \uadf8\ub7ec\ub098 \uc138\ub4dc\ub098\ub77c\ub294 \uc774\ub984\uc5d0 \ubc18\ub300\ud558\ub294 \uc758\uacac\uc740 \uc5c6\uc5c8\uace0, \uadf8 \uc678\uc5d0 \ub2e4\ub978 \uacbd\ud569\ud560 \ub9cc\ud55c \uc774\ub984\uc774 \uc81c\uc2dc\ub418\uc9c0\ub3c4 \uc54a\uc558\ub2e4. IAU \uc18c\ucc9c\uccb4\uba85\uba85\uc704\uc6d0\ud68c\ub294 \uc138\ub4dc\ub098\ub77c\ub294 \uc774\ub984\uc744 2004\ub144 9\uc6d4\uc5d0 \uc815\uc2dd\uc73c\ub85c \ubc1b\uc544\ub4e4\uc774\uace0, \uc55e\uc73c\ub85c\ub3c4 \uc774\uc640 \uac19\uc774 \ube44\uc0c1\ud55c \uad00\uc2ec\uc758 \ub300\uc0c1\uc778 \uacbd\uc6b0 \ubc88\ud638\uac00 \ub9e4\uaca8\uc9c0\uae30 \uc804\uc5d0 \uc774\ub984\uc744 \uba3c\uc800 \ubc1c\ud45c\ud560 \uc218\ub3c4 \uc788\ub2e4\ub294 \uacb0\ub860\uc744 \ub0b4\ub838\ub2e4."} {"question": "\ub124\ubc84\uc6d0\ud130 \ub098\uc774\uce20\uc758 \ud655\uc7a5\ud329\uc758 \ubd84\ub7c9\uc740?", "paragraph": "\uac8c\uc784\uc758 \ubd84\ub7c9\uc740 \uae34 \ud3b8\uc73c\ub85c \uc624\ub9ac\uc9c0\ub110 \u300a\ub124\ubc84\uc708\ud130 \ub098\uc774\uce20\u300b\uac00 CD 3\uc7a5 \ubd84\ub7c9\uc774\uba70, \uac01\uac01\uc758 \ud655\uc7a5\ud329\uc740 CD 1\uc7a5 \ubd84\ub7c9\uc774\ub2e4. \uc624\ub9ac\uc9c0\ub110 \ucea0\ud328\uc778\uc740 \uc9e7\uc740 \uc11c\uc7a5\uacfc \ub124 \uac1c\uc758 \uc7a5\uc73c\ub85c \uad6c\uc131\ub418\uc5b4 \uc788\uc73c\uba70, \uac01 \uc7a5\uc5d0\ub294 \uc7a5 \uc804\ubc18\uc5d0 \uac78\uce5c \uc804\uccb4\uc801\uc778 \uc774\uc57c\uae30\uac00 \uc874\uc7ac\ud558\uace0(\uc608\ub97c \ub4e4\uc5b4, \uccab \ubc88\uc9f8 \uc7a5\uc740 \ub124\ubc84\uc708\ud130\ub97c \ub36e\uce5c \uc218\uc218\uaed8\ub07c\uc758 \uc804\uc5fc\ubcd1\uc5d0 \ub300\ud56d\ud558\ub294 \uc774\uc57c\uae30\uc774\ub2e4), \ub9ce\uc740 \ud018\uc2a4\ud2b8\uc640 \uc0ac\uc774\ub4dc \ud018\uc2a4\ud2b8, \uc9e7\uc740 \uc774\uc57c\uae30\ub4e4\ub85c \uad6c\uc131\ub41c\ub2e4. \ud2b9\uc815\ud55c \ud018\uc2a4\ud2b8\ub97c \uc218\ud589\ud588\ub294\uc9c0\uc5d0 \ub530\ub77c\uc11c, \ub610\ub294 \ud2b9\uc815\ud55c \uc544\uc774\ud15c\uc744 \uc18c\uc9c0\ud588\ub294\uc9c0\uc5d0 \ub530\ub77c \uc77c\ubd80 \uc774\uc57c\uae30\uac00 \ub2ec\ub77c\uc9c0\uae30\ub3c4 \ud55c\ub2e4(\ud5e8\uce58\ub9e8\uc774\ub098 \uc544\ub9ac\ubca0\uc2a4\uc758 \uc774\uc57c\uae30 \ub4f1). \uc0ac\uc774\ub4dc \ud018\uc2a4\ud2b8\ub97c \ub9ce\uc774 \uc218\ud589\ud558\uba74 \uce90\ub9ad\ud130\ub294 \ub354 \ub9ce\uc740 \uacbd\ud5d8\uce58\uc640 \ud2b9\ubcc4\ud55c \uc544\uc774\ud15c\uc744 \uc5bb\uc744 \uc218 \uc788\uc73c\uba70, \uc774\ub97c \ud1b5\ud574\uc11c \ub354 \ube68\ub9ac \ub808\ubca8\uc744 \uc62c\ub9ac\uace0 \uac8c\uc784\uc744 \uc880 \ub354 \uc27d\uac8c \ud480\uc5b4\ub098\uac08 \uc218 \uc788\uac8c \ub41c\ub2e4. \uc608\ub97c \ub4e4\uc5b4, 1\uc7a5\uacfc 2\uc7a5\uc758 \ubaa8\ub4e0 \ud018\uc2a4\ud2b8\ub97c \uc644\uc218\ud588\uc744 \uacbd\uc6b0, 13\ub808\ubca8 \uce90\ub9ad\ud130\ub85c 3\uc7a5\uc744 \uc2dc\uc791\ud560 \uc218 \uc788\ub2e4.", "answer": "CD 1\uc7a5", "sentence": "\uac8c\uc784\uc758 \ubd84\ub7c9\uc740 \uae34 \ud3b8\uc73c\ub85c \uc624\ub9ac\uc9c0\ub110 \u300a\ub124\ubc84\uc708\ud130 \ub098\uc774\uce20\u300b\uac00 CD 3\uc7a5 \ubd84\ub7c9\uc774\uba70, \uac01\uac01\uc758 \ud655\uc7a5\ud329\uc740 CD 1\uc7a5 \ubd84\ub7c9\uc774\ub2e4.", "paragraph_sentence": " \uac8c\uc784\uc758 \ubd84\ub7c9\uc740 \uae34 \ud3b8\uc73c\ub85c \uc624\ub9ac\uc9c0\ub110 \u300a\ub124\ubc84\uc708\ud130 \ub098\uc774\uce20\u300b\uac00 CD 3\uc7a5 \ubd84\ub7c9\uc774\uba70, \uac01\uac01\uc758 \ud655\uc7a5\ud329\uc740 CD 1\uc7a5 \ubd84\ub7c9\uc774\ub2e4. \uc624\ub9ac\uc9c0\ub110 \ucea0\ud328\uc778\uc740 \uc9e7\uc740 \uc11c\uc7a5\uacfc \ub124 \uac1c\uc758 \uc7a5\uc73c\ub85c \uad6c\uc131\ub418\uc5b4 \uc788\uc73c\uba70, \uac01 \uc7a5\uc5d0\ub294 \uc7a5 \uc804\ubc18\uc5d0 \uac78\uce5c \uc804\uccb4\uc801\uc778 \uc774\uc57c\uae30\uac00 \uc874\uc7ac\ud558\uace0(\uc608\ub97c \ub4e4\uc5b4, \uccab \ubc88\uc9f8 \uc7a5\uc740 \ub124\ubc84\uc708\ud130\ub97c \ub36e\uce5c \uc218\uc218\uaed8\ub07c\uc758 \uc804\uc5fc\ubcd1\uc5d0 \ub300\ud56d\ud558\ub294 \uc774\uc57c\uae30\uc774\ub2e4), \ub9ce\uc740 \ud018\uc2a4\ud2b8\uc640 \uc0ac\uc774\ub4dc \ud018\uc2a4\ud2b8, \uc9e7\uc740 \uc774\uc57c\uae30\ub4e4\ub85c \uad6c\uc131\ub41c\ub2e4. \ud2b9\uc815\ud55c \ud018\uc2a4\ud2b8\ub97c \uc218\ud589\ud588\ub294\uc9c0\uc5d0 \ub530\ub77c\uc11c, \ub610\ub294 \ud2b9\uc815\ud55c \uc544\uc774\ud15c\uc744 \uc18c\uc9c0\ud588\ub294\uc9c0\uc5d0 \ub530\ub77c \uc77c\ubd80 \uc774\uc57c\uae30\uac00 \ub2ec\ub77c\uc9c0\uae30\ub3c4 \ud55c\ub2e4(\ud5e8\uce58\ub9e8\uc774\ub098 \uc544\ub9ac\ubca0\uc2a4\uc758 \uc774\uc57c\uae30 \ub4f1). \uc0ac\uc774\ub4dc \ud018\uc2a4\ud2b8\ub97c \ub9ce\uc774 \uc218\ud589\ud558\uba74 \uce90\ub9ad\ud130\ub294 \ub354 \ub9ce\uc740 \uacbd\ud5d8\uce58\uc640 \ud2b9\ubcc4\ud55c \uc544\uc774\ud15c\uc744 \uc5bb\uc744 \uc218 \uc788\uc73c\uba70, \uc774\ub97c \ud1b5\ud574\uc11c \ub354 \ube68\ub9ac \ub808\ubca8\uc744 \uc62c\ub9ac\uace0 \uac8c\uc784\uc744 \uc880 \ub354 \uc27d\uac8c \ud480\uc5b4\ub098\uac08 \uc218 \uc788\uac8c \ub41c\ub2e4. \uc608\ub97c \ub4e4\uc5b4, 1\uc7a5\uacfc 2\uc7a5\uc758 \ubaa8\ub4e0 \ud018\uc2a4\ud2b8\ub97c \uc644\uc218\ud588\uc744 \uacbd\uc6b0, 13\ub808\ubca8 \uce90\ub9ad\ud130\ub85c 3\uc7a5\uc744 \uc2dc\uc791\ud560 \uc218 \uc788\ub2e4.", "paragraph_answer": "\uac8c\uc784\uc758 \ubd84\ub7c9\uc740 \uae34 \ud3b8\uc73c\ub85c \uc624\ub9ac\uc9c0\ub110 \u300a\ub124\ubc84\uc708\ud130 \ub098\uc774\uce20\u300b\uac00 CD 3\uc7a5 \ubd84\ub7c9\uc774\uba70, \uac01\uac01\uc758 \ud655\uc7a5\ud329\uc740 CD 1\uc7a5 \ubd84\ub7c9\uc774\ub2e4. \uc624\ub9ac\uc9c0\ub110 \ucea0\ud328\uc778\uc740 \uc9e7\uc740 \uc11c\uc7a5\uacfc \ub124 \uac1c\uc758 \uc7a5\uc73c\ub85c \uad6c\uc131\ub418\uc5b4 \uc788\uc73c\uba70, \uac01 \uc7a5\uc5d0\ub294 \uc7a5 \uc804\ubc18\uc5d0 \uac78\uce5c \uc804\uccb4\uc801\uc778 \uc774\uc57c\uae30\uac00 \uc874\uc7ac\ud558\uace0(\uc608\ub97c \ub4e4\uc5b4, \uccab \ubc88\uc9f8 \uc7a5\uc740 \ub124\ubc84\uc708\ud130\ub97c \ub36e\uce5c \uc218\uc218\uaed8\ub07c\uc758 \uc804\uc5fc\ubcd1\uc5d0 \ub300\ud56d\ud558\ub294 \uc774\uc57c\uae30\uc774\ub2e4), \ub9ce\uc740 \ud018\uc2a4\ud2b8\uc640 \uc0ac\uc774\ub4dc \ud018\uc2a4\ud2b8, \uc9e7\uc740 \uc774\uc57c\uae30\ub4e4\ub85c \uad6c\uc131\ub41c\ub2e4. \ud2b9\uc815\ud55c \ud018\uc2a4\ud2b8\ub97c \uc218\ud589\ud588\ub294\uc9c0\uc5d0 \ub530\ub77c\uc11c, \ub610\ub294 \ud2b9\uc815\ud55c \uc544\uc774\ud15c\uc744 \uc18c\uc9c0\ud588\ub294\uc9c0\uc5d0 \ub530\ub77c \uc77c\ubd80 \uc774\uc57c\uae30\uac00 \ub2ec\ub77c\uc9c0\uae30\ub3c4 \ud55c\ub2e4(\ud5e8\uce58\ub9e8\uc774\ub098 \uc544\ub9ac\ubca0\uc2a4\uc758 \uc774\uc57c\uae30 \ub4f1). \uc0ac\uc774\ub4dc \ud018\uc2a4\ud2b8\ub97c \ub9ce\uc774 \uc218\ud589\ud558\uba74 \uce90\ub9ad\ud130\ub294 \ub354 \ub9ce\uc740 \uacbd\ud5d8\uce58\uc640 \ud2b9\ubcc4\ud55c \uc544\uc774\ud15c\uc744 \uc5bb\uc744 \uc218 \uc788\uc73c\uba70, \uc774\ub97c \ud1b5\ud574\uc11c \ub354 \ube68\ub9ac \ub808\ubca8\uc744 \uc62c\ub9ac\uace0 \uac8c\uc784\uc744 \uc880 \ub354 \uc27d\uac8c \ud480\uc5b4\ub098\uac08 \uc218 \uc788\uac8c \ub41c\ub2e4. \uc608\ub97c \ub4e4\uc5b4, 1\uc7a5\uacfc 2\uc7a5\uc758 \ubaa8\ub4e0 \ud018\uc2a4\ud2b8\ub97c \uc644\uc218\ud588\uc744 \uacbd\uc6b0, 13\ub808\ubca8 \uce90\ub9ad\ud130\ub85c 3\uc7a5\uc744 \uc2dc\uc791\ud560 \uc218 \uc788\ub2e4.", "sentence_answer": "\uac8c\uc784\uc758 \ubd84\ub7c9\uc740 \uae34 \ud3b8\uc73c\ub85c \uc624\ub9ac\uc9c0\ub110 \u300a\ub124\ubc84\uc708\ud130 \ub098\uc774\uce20\u300b\uac00 CD 3\uc7a5 \ubd84\ub7c9\uc774\uba70, \uac01\uac01\uc758 \ud655\uc7a5\ud329\uc740 CD 1\uc7a5 \ubd84\ub7c9\uc774\ub2e4.", "paragraph_id": "6488864-2-1", "paragraph_question": "question: \ub124\ubc84\uc6d0\ud130 \ub098\uc774\uce20\uc758 \ud655\uc7a5\ud329\uc758 \ubd84\ub7c9\uc740?, context: \uac8c\uc784\uc758 \ubd84\ub7c9\uc740 \uae34 \ud3b8\uc73c\ub85c \uc624\ub9ac\uc9c0\ub110 \u300a\ub124\ubc84\uc708\ud130 \ub098\uc774\uce20\u300b\uac00 CD 3\uc7a5 \ubd84\ub7c9\uc774\uba70, \uac01\uac01\uc758 \ud655\uc7a5\ud329\uc740 CD 1\uc7a5 \ubd84\ub7c9\uc774\ub2e4. \uc624\ub9ac\uc9c0\ub110 \ucea0\ud328\uc778\uc740 \uc9e7\uc740 \uc11c\uc7a5\uacfc \ub124 \uac1c\uc758 \uc7a5\uc73c\ub85c \uad6c\uc131\ub418\uc5b4 \uc788\uc73c\uba70, \uac01 \uc7a5\uc5d0\ub294 \uc7a5 \uc804\ubc18\uc5d0 \uac78\uce5c \uc804\uccb4\uc801\uc778 \uc774\uc57c\uae30\uac00 \uc874\uc7ac\ud558\uace0(\uc608\ub97c \ub4e4\uc5b4, \uccab \ubc88\uc9f8 \uc7a5\uc740 \ub124\ubc84\uc708\ud130\ub97c \ub36e\uce5c \uc218\uc218\uaed8\ub07c\uc758 \uc804\uc5fc\ubcd1\uc5d0 \ub300\ud56d\ud558\ub294 \uc774\uc57c\uae30\uc774\ub2e4), \ub9ce\uc740 \ud018\uc2a4\ud2b8\uc640 \uc0ac\uc774\ub4dc \ud018\uc2a4\ud2b8, \uc9e7\uc740 \uc774\uc57c\uae30\ub4e4\ub85c \uad6c\uc131\ub41c\ub2e4. \ud2b9\uc815\ud55c \ud018\uc2a4\ud2b8\ub97c \uc218\ud589\ud588\ub294\uc9c0\uc5d0 \ub530\ub77c\uc11c, \ub610\ub294 \ud2b9\uc815\ud55c \uc544\uc774\ud15c\uc744 \uc18c\uc9c0\ud588\ub294\uc9c0\uc5d0 \ub530\ub77c \uc77c\ubd80 \uc774\uc57c\uae30\uac00 \ub2ec\ub77c\uc9c0\uae30\ub3c4 \ud55c\ub2e4(\ud5e8\uce58\ub9e8\uc774\ub098 \uc544\ub9ac\ubca0\uc2a4\uc758 \uc774\uc57c\uae30 \ub4f1). \uc0ac\uc774\ub4dc \ud018\uc2a4\ud2b8\ub97c \ub9ce\uc774 \uc218\ud589\ud558\uba74 \uce90\ub9ad\ud130\ub294 \ub354 \ub9ce\uc740 \uacbd\ud5d8\uce58\uc640 \ud2b9\ubcc4\ud55c \uc544\uc774\ud15c\uc744 \uc5bb\uc744 \uc218 \uc788\uc73c\uba70, \uc774\ub97c \ud1b5\ud574\uc11c \ub354 \ube68\ub9ac \ub808\ubca8\uc744 \uc62c\ub9ac\uace0 \uac8c\uc784\uc744 \uc880 \ub354 \uc27d\uac8c \ud480\uc5b4\ub098\uac08 \uc218 \uc788\uac8c \ub41c\ub2e4. \uc608\ub97c \ub4e4\uc5b4, 1\uc7a5\uacfc 2\uc7a5\uc758 \ubaa8\ub4e0 \ud018\uc2a4\ud2b8\ub97c \uc644\uc218\ud588\uc744 \uacbd\uc6b0, 13\ub808\ubca8 \uce90\ub9ad\ud130\ub85c 3\uc7a5\uc744 \uc2dc\uc791\ud560 \uc218 \uc788\ub2e4."} {"question": "WCW\ub97c \ub300\uccb4\ud558\uc5ec ECW\uc740 \uc5b4\ub514\uc758 \uc0b0\ud558\uae30\uad00\uc73c\ub85c \ud65c\ub3d9\ud558\uc600\ub294\uac00?", "paragraph": "1992\ub144 \ub2f9\uc2dc \ud504\ub85c\ubaa8\ud130\uc600\ub358 \ud1a0\ub4dc \uace0\ub358\uc740 \ud544\ub77c\ub378\ud53c\uc544\ub97c \ubcf8\uac70\uc9c0\ub85c\ud55c \ub3d9\ubd80\uc9c0\uc5ed \ud504\ub85c\ub808\uc2ac\ub9c1 \ub2e8\uccb4\uc778 \uc774\uc2a4\ud134 \ucc54\ud53c\uc5b8\uc2ed \ub808\uc2ac\ub9c1(Eastern Championsip Wrestling)\uc744 \uc138\uc6b0\uac8c \ub41c\ub2e4. \ub2f9\uc2dc \ub124\uc154\ub110 \ub808\uc2ac\ub9c1 \uc5bc\ub77c\uc774\uc5b8\uc2a4 (National Wrestling Alliance) \uc774\ud558 NWA\ub294 \uc0b0\ud558 \ub2e8\uccb4\uc600\ub358 WCW\uac00 \ud0c8\ud1f4\ub97c \ud588\ub358 \ud130\ub77c \uc8fc\ubaa9\ubc1b\uc744 \ub9cc\ud55c \uc288\ud37c\uc2a4\ud0c0\uac00 \uc5c6\ub294 \uc2ec\uac01\ud55c \uc0c1\ud669\uc5d0\uc11c \ub370\ub2c8\uc988 \uce7c\ub8e8\uc18c\uac00 \uaca8\uc6b0 \uba54\uafb8\uc5b4 \ub098\uac00\ub294 \ub370\uc5d0 \uc8fc\ub825\ud558\ub358 \uc911\uc774\uc5c8\uace0 \uadf8\ub7ec\ud55c \uc2dc\uae30\uc5d0 ECW\ub294 WCW\ub97c \ub300\uccb4\ud560 NWA\uc758 \uc0b0\ud558 \ub2e8\uccb4\uac00 \ub418\uc5c8\ub2e4. \uc124\ub9bd \ub2f9\uc2dc\uae4c\uc9c0\ub9cc \ud574\ub3c4 ECW\ub294 \uc5ec\ub290 \ub2e8\uccb4\uc640 \ub9c8\ucc2c\uac00\uc9c0\ub85c \uc815\ud1b5 \ud504\ub85c\ub808\uc2ac\ub9c1\uc744 \uc9c0\ud5a5\ud558\ub294 \ub2e8\uccb4\uc600\uc73c\uba70 \uc124\ub9bd\ud558\uace0 \ub098\uc11c \ud5a5\ud6c4 2\ub144\uac04\uc740 NWA\uc758 \uc804\ud1b5\uc801\uc778 \ub8f0\uc744 \ub530\ub974\ub358 \ub2e8\uccb4\uc600\ub2e4.", "answer": "NWA", "sentence": "\ub2f9\uc2dc \ub124\uc154\ub110 \ub808\uc2ac\ub9c1 \uc5bc\ub77c\uc774\uc5b8\uc2a4 (National Wrestling Alliance) \uc774\ud558 NWA \ub294 \uc0b0\ud558 \ub2e8\uccb4\uc600\ub358 WCW\uac00 \ud0c8\ud1f4\ub97c \ud588\ub358 \ud130\ub77c \uc8fc\ubaa9\ubc1b\uc744 \ub9cc\ud55c \uc288\ud37c\uc2a4\ud0c0\uac00 \uc5c6\ub294 \uc2ec\uac01\ud55c \uc0c1\ud669\uc5d0\uc11c \ub370\ub2c8\uc988 \uce7c\ub8e8\uc18c\uac00 \uaca8\uc6b0 \uba54\uafb8\uc5b4 \ub098\uac00\ub294 \ub370\uc5d0 \uc8fc\ub825\ud558\ub358 \uc911\uc774\uc5c8\uace0 \uadf8\ub7ec\ud55c \uc2dc\uae30\uc5d0 ECW\ub294 WCW\ub97c \ub300\uccb4\ud560 NWA\uc758 \uc0b0\ud558 \ub2e8\uccb4\uac00 \ub418\uc5c8\ub2e4.", "paragraph_sentence": "1992\ub144 \ub2f9\uc2dc \ud504\ub85c\ubaa8\ud130\uc600\ub358 \ud1a0\ub4dc \uace0\ub358\uc740 \ud544\ub77c\ub378\ud53c\uc544\ub97c \ubcf8\uac70\uc9c0\ub85c\ud55c \ub3d9\ubd80\uc9c0\uc5ed \ud504\ub85c\ub808\uc2ac\ub9c1 \ub2e8\uccb4\uc778 \uc774\uc2a4\ud134 \ucc54\ud53c\uc5b8\uc2ed \ub808\uc2ac\ub9c1(Eastern Championsip Wrestling)\uc744 \uc138\uc6b0\uac8c \ub41c\ub2e4. \ub2f9\uc2dc \ub124\uc154\ub110 \ub808\uc2ac\ub9c1 \uc5bc\ub77c\uc774\uc5b8\uc2a4 (National Wrestling Alliance) \uc774\ud558 NWA \ub294 \uc0b0\ud558 \ub2e8\uccb4\uc600\ub358 WCW\uac00 \ud0c8\ud1f4\ub97c \ud588\ub358 \ud130\ub77c \uc8fc\ubaa9\ubc1b\uc744 \ub9cc\ud55c \uc288\ud37c\uc2a4\ud0c0\uac00 \uc5c6\ub294 \uc2ec\uac01\ud55c \uc0c1\ud669\uc5d0\uc11c \ub370\ub2c8\uc988 \uce7c\ub8e8\uc18c\uac00 \uaca8\uc6b0 \uba54\uafb8\uc5b4 \ub098\uac00\ub294 \ub370\uc5d0 \uc8fc\ub825\ud558\ub358 \uc911\uc774\uc5c8\uace0 \uadf8\ub7ec\ud55c \uc2dc\uae30\uc5d0 ECW\ub294 WCW\ub97c \ub300\uccb4\ud560 NWA\uc758 \uc0b0\ud558 \ub2e8\uccb4\uac00 \ub418\uc5c8\ub2e4. \uc124\ub9bd \ub2f9\uc2dc\uae4c\uc9c0\ub9cc \ud574\ub3c4 ECW\ub294 \uc5ec\ub290 \ub2e8\uccb4\uc640 \ub9c8\ucc2c\uac00\uc9c0\ub85c \uc815\ud1b5 \ud504\ub85c\ub808\uc2ac\ub9c1\uc744 \uc9c0\ud5a5\ud558\ub294 \ub2e8\uccb4\uc600\uc73c\uba70 \uc124\ub9bd\ud558\uace0 \ub098\uc11c \ud5a5\ud6c4 2\ub144\uac04\uc740 NWA\uc758 \uc804\ud1b5\uc801\uc778 \ub8f0\uc744 \ub530\ub974\ub358 \ub2e8\uccb4\uc600\ub2e4.", "paragraph_answer": "1992\ub144 \ub2f9\uc2dc \ud504\ub85c\ubaa8\ud130\uc600\ub358 \ud1a0\ub4dc \uace0\ub358\uc740 \ud544\ub77c\ub378\ud53c\uc544\ub97c \ubcf8\uac70\uc9c0\ub85c\ud55c \ub3d9\ubd80\uc9c0\uc5ed \ud504\ub85c\ub808\uc2ac\ub9c1 \ub2e8\uccb4\uc778 \uc774\uc2a4\ud134 \ucc54\ud53c\uc5b8\uc2ed \ub808\uc2ac\ub9c1(Eastern Championsip Wrestling)\uc744 \uc138\uc6b0\uac8c \ub41c\ub2e4. \ub2f9\uc2dc \ub124\uc154\ub110 \ub808\uc2ac\ub9c1 \uc5bc\ub77c\uc774\uc5b8\uc2a4 (National Wrestling Alliance) \uc774\ud558 NWA \ub294 \uc0b0\ud558 \ub2e8\uccb4\uc600\ub358 WCW\uac00 \ud0c8\ud1f4\ub97c \ud588\ub358 \ud130\ub77c \uc8fc\ubaa9\ubc1b\uc744 \ub9cc\ud55c \uc288\ud37c\uc2a4\ud0c0\uac00 \uc5c6\ub294 \uc2ec\uac01\ud55c \uc0c1\ud669\uc5d0\uc11c \ub370\ub2c8\uc988 \uce7c\ub8e8\uc18c\uac00 \uaca8\uc6b0 \uba54\uafb8\uc5b4 \ub098\uac00\ub294 \ub370\uc5d0 \uc8fc\ub825\ud558\ub358 \uc911\uc774\uc5c8\uace0 \uadf8\ub7ec\ud55c \uc2dc\uae30\uc5d0 ECW\ub294 WCW\ub97c \ub300\uccb4\ud560 NWA\uc758 \uc0b0\ud558 \ub2e8\uccb4\uac00 \ub418\uc5c8\ub2e4. \uc124\ub9bd \ub2f9\uc2dc\uae4c\uc9c0\ub9cc \ud574\ub3c4 ECW\ub294 \uc5ec\ub290 \ub2e8\uccb4\uc640 \ub9c8\ucc2c\uac00\uc9c0\ub85c \uc815\ud1b5 \ud504\ub85c\ub808\uc2ac\ub9c1\uc744 \uc9c0\ud5a5\ud558\ub294 \ub2e8\uccb4\uc600\uc73c\uba70 \uc124\ub9bd\ud558\uace0 \ub098\uc11c \ud5a5\ud6c4 2\ub144\uac04\uc740 NWA\uc758 \uc804\ud1b5\uc801\uc778 \ub8f0\uc744 \ub530\ub974\ub358 \ub2e8\uccb4\uc600\ub2e4.", "sentence_answer": "\ub2f9\uc2dc \ub124\uc154\ub110 \ub808\uc2ac\ub9c1 \uc5bc\ub77c\uc774\uc5b8\uc2a4 (National Wrestling Alliance) \uc774\ud558 NWA \ub294 \uc0b0\ud558 \ub2e8\uccb4\uc600\ub358 WCW\uac00 \ud0c8\ud1f4\ub97c \ud588\ub358 \ud130\ub77c \uc8fc\ubaa9\ubc1b\uc744 \ub9cc\ud55c \uc288\ud37c\uc2a4\ud0c0\uac00 \uc5c6\ub294 \uc2ec\uac01\ud55c \uc0c1\ud669\uc5d0\uc11c \ub370\ub2c8\uc988 \uce7c\ub8e8\uc18c\uac00 \uaca8\uc6b0 \uba54\uafb8\uc5b4 \ub098\uac00\ub294 \ub370\uc5d0 \uc8fc\ub825\ud558\ub358 \uc911\uc774\uc5c8\uace0 \uadf8\ub7ec\ud55c \uc2dc\uae30\uc5d0 ECW\ub294 WCW\ub97c \ub300\uccb4\ud560 NWA\uc758 \uc0b0\ud558 \ub2e8\uccb4\uac00 \ub418\uc5c8\ub2e4.", "paragraph_id": "6548069-0-1", "paragraph_question": "question: WCW\ub97c \ub300\uccb4\ud558\uc5ec ECW\uc740 \uc5b4\ub514\uc758 \uc0b0\ud558\uae30\uad00\uc73c\ub85c \ud65c\ub3d9\ud558\uc600\ub294\uac00?, context: 1992\ub144 \ub2f9\uc2dc \ud504\ub85c\ubaa8\ud130\uc600\ub358 \ud1a0\ub4dc \uace0\ub358\uc740 \ud544\ub77c\ub378\ud53c\uc544\ub97c \ubcf8\uac70\uc9c0\ub85c\ud55c \ub3d9\ubd80\uc9c0\uc5ed \ud504\ub85c\ub808\uc2ac\ub9c1 \ub2e8\uccb4\uc778 \uc774\uc2a4\ud134 \ucc54\ud53c\uc5b8\uc2ed \ub808\uc2ac\ub9c1(Eastern Championsip Wrestling)\uc744 \uc138\uc6b0\uac8c \ub41c\ub2e4. \ub2f9\uc2dc \ub124\uc154\ub110 \ub808\uc2ac\ub9c1 \uc5bc\ub77c\uc774\uc5b8\uc2a4 (National Wrestling Alliance) \uc774\ud558 NWA\ub294 \uc0b0\ud558 \ub2e8\uccb4\uc600\ub358 WCW\uac00 \ud0c8\ud1f4\ub97c \ud588\ub358 \ud130\ub77c \uc8fc\ubaa9\ubc1b\uc744 \ub9cc\ud55c \uc288\ud37c\uc2a4\ud0c0\uac00 \uc5c6\ub294 \uc2ec\uac01\ud55c \uc0c1\ud669\uc5d0\uc11c \ub370\ub2c8\uc988 \uce7c\ub8e8\uc18c\uac00 \uaca8\uc6b0 \uba54\uafb8\uc5b4 \ub098\uac00\ub294 \ub370\uc5d0 \uc8fc\ub825\ud558\ub358 \uc911\uc774\uc5c8\uace0 \uadf8\ub7ec\ud55c \uc2dc\uae30\uc5d0 ECW\ub294 WCW\ub97c \ub300\uccb4\ud560 NWA\uc758 \uc0b0\ud558 \ub2e8\uccb4\uac00 \ub418\uc5c8\ub2e4. \uc124\ub9bd \ub2f9\uc2dc\uae4c\uc9c0\ub9cc \ud574\ub3c4 ECW\ub294 \uc5ec\ub290 \ub2e8\uccb4\uc640 \ub9c8\ucc2c\uac00\uc9c0\ub85c \uc815\ud1b5 \ud504\ub85c\ub808\uc2ac\ub9c1\uc744 \uc9c0\ud5a5\ud558\ub294 \ub2e8\uccb4\uc600\uc73c\uba70 \uc124\ub9bd\ud558\uace0 \ub098\uc11c \ud5a5\ud6c4 2\ub144\uac04\uc740 NWA\uc758 \uc804\ud1b5\uc801\uc778 \ub8f0\uc744 \ub530\ub974\ub358 \ub2e8\uccb4\uc600\ub2e4."} {"question": "\ub178\ube44\ub4e4\uc758 \uacc4\ubcf4\uac00 \ubaa8\uacc4\ub85c \uc774\uc5b4\uc9c0\uac8c \ub418\ub294\ub370 \uc601\ud5a5\uc744 \ubbf8\uce5c \ubc95\uc740 \ubb34\uc5c7\uc778\uac00?", "paragraph": "\uc774\uc0c9\uc801\uc73c\ub85c \ud658\uad00\uc758 \uc9d1\uc548\uc5d0\uc11c \uc871\ubcf4\ub97c \uc81c\uc791\ud55c \uacbd\uc6b0\ub3c4 \uc788\uc5c8\ub294\ub370, \uc774\ub97c \u300a\ub0b4\uc2dc\ubcf4\u300b\ub77c \ud558\uc600\ub2e4. \uc77c\uc81c \uc2dc\ub300\uc5d0 \uc791\uc131\ub41c \u300a\uc591\uc138\uacc4\ubcf4(\u990a\u4e16\u7cfb\u8b5c)\u300b(1920\ub144)\uc640 \u300a\uc5f0\uc591\uad70\uc138\uacc4\u300b\uac00 \ub0a8\uc544 \uc788\uc73c\uba70, \ud544\uc0ac\ubcf8\uc774\ub2e4. \uc774 \uacbd\uc6b0 \ud658\uad00\uc774\ub77c\ub294 \uc9c1\uc5c5\uc801 \ud2b9\uc131\uc0c1 \uc591\uc790\ub97c \ud1b5\ud574 \ub0b4\uc2dc\uc9c1\uc744 \uc138\uc2b5\ud558\uc5ec \uc138\uacc4\ub97c \ub0b4\ub824\uc624\uba74\uc11c \uc591\uc790\uc758 \uc131\ub3c4 \uadf8\ub300\ub85c \uc720\uc9c0\ud558\uace0 \ubcf8\uad00\ub3c4 \uc801\uc5b4\uc11c \uacc4\ucd9c\uc744 \ubc1d\ud78c \uc810\uc774 \ud2b9\uc9d5\uc774\ub2e4. \uc591\ubc18\uc774\ub098 \uc911\uc778\uc774 \uc544\ub2cc \uc2e0\ubd84\uc73c\ub85c \uacc4\ubcf4\ub97c \ub0a8\uae34 \uc0ac\ud68c\uce35\uc740 \ub178\ube44\uc600\ub2e4. \ub178\ube44\uac00 \ub9cc\ub4e0 \uac83\uc774 \uc544\ub2c8\ub77c \ub178\ube44\ub97c \uad00\ub9ac\ud558\ub294 \uc790\ub4e4\uc774 \ub9cc\ub4e0 \uac83\uc73c\ub85c, \uc804\ub77c\ub0a8\ub3c4 \uc7a5\uc131\uad70\uc758 \ud544\uc554\uc11c\uc6d0 \uc18c\uc7a5 \ubb38\uc11c \uac00\uc6b4\ub370 \u300a\ub178\ube44\ubcf4\u300b\uac00 \ub0a8\uc544\uc788\ub2e4. 18\uc138\uae30\uacbd \ud544\uc554\uc11c\uc6d0\uc5d0 \uc18c\uc18d\ub41c \ub178\ube44\ub4e4\uc758 \uc778\uc801 \uc0ac\ud56d\uc744 \uc801\uc740 \uacc4\ubcf4\ub294 \ubaa8\uacc4\ub85c \uc774\uc5b4\uc9c0\ub294 \uc131\uaca9\uc744 \uac00\uc9c0\uace0 \uc788\ub294\ub370, \uc870\uc120\uc758 \uc2e0\ubd84 \uad00\ub828 \ubc95\ub960\uc5d0\uc11c \uc885\ubaa8\ubc95(\u5f9e\u6bcd\u6cd5)\uc744 \ud0dd\ud588\ub358 \uc0ac\uc815\uc5d0 \uae30\uc778\ud55c \uac83\uc774\ub2e4.", "answer": "\uc885\ubaa8\ubc95", "sentence": "18\uc138\uae30\uacbd \ud544\uc554\uc11c\uc6d0\uc5d0 \uc18c\uc18d\ub41c \ub178\ube44\ub4e4\uc758 \uc778\uc801 \uc0ac\ud56d\uc744 \uc801\uc740 \uacc4\ubcf4\ub294 \ubaa8\uacc4\ub85c \uc774\uc5b4\uc9c0\ub294 \uc131\uaca9\uc744 \uac00\uc9c0\uace0 \uc788\ub294\ub370, \uc870\uc120\uc758 \uc2e0\ubd84 \uad00\ub828 \ubc95\ub960\uc5d0\uc11c \uc885\ubaa8\ubc95 (\u5f9e\u6bcd\u6cd5)", "paragraph_sentence": "\uc774\uc0c9\uc801\uc73c\ub85c \ud658\uad00\uc758 \uc9d1\uc548\uc5d0\uc11c \uc871\ubcf4\ub97c \uc81c\uc791\ud55c \uacbd\uc6b0\ub3c4 \uc788\uc5c8\ub294\ub370, \uc774\ub97c \u300a\ub0b4\uc2dc\ubcf4\u300b\ub77c \ud558\uc600\ub2e4. \uc77c\uc81c \uc2dc\ub300\uc5d0 \uc791\uc131\ub41c \u300a\uc591\uc138\uacc4\ubcf4(\u990a\u4e16\u7cfb\u8b5c)\u300b(1920\ub144)\uc640 \u300a\uc5f0\uc591\uad70\uc138\uacc4\u300b\uac00 \ub0a8\uc544 \uc788\uc73c\uba70, \ud544\uc0ac\ubcf8\uc774\ub2e4. \uc774 \uacbd\uc6b0 \ud658\uad00\uc774\ub77c\ub294 \uc9c1\uc5c5\uc801 \ud2b9\uc131\uc0c1 \uc591\uc790\ub97c \ud1b5\ud574 \ub0b4\uc2dc\uc9c1\uc744 \uc138\uc2b5\ud558\uc5ec \uc138\uacc4\ub97c \ub0b4\ub824\uc624\uba74\uc11c \uc591\uc790\uc758 \uc131\ub3c4 \uadf8\ub300\ub85c \uc720\uc9c0\ud558\uace0 \ubcf8\uad00\ub3c4 \uc801\uc5b4\uc11c \uacc4\ucd9c\uc744 \ubc1d\ud78c \uc810\uc774 \ud2b9\uc9d5\uc774\ub2e4. \uc591\ubc18\uc774\ub098 \uc911\uc778\uc774 \uc544\ub2cc \uc2e0\ubd84\uc73c\ub85c \uacc4\ubcf4\ub97c \ub0a8\uae34 \uc0ac\ud68c\uce35\uc740 \ub178\ube44\uc600\ub2e4. \ub178\ube44\uac00 \ub9cc\ub4e0 \uac83\uc774 \uc544\ub2c8\ub77c \ub178\ube44\ub97c \uad00\ub9ac\ud558\ub294 \uc790\ub4e4\uc774 \ub9cc\ub4e0 \uac83\uc73c\ub85c, \uc804\ub77c\ub0a8\ub3c4 \uc7a5\uc131\uad70\uc758 \ud544\uc554\uc11c\uc6d0 \uc18c\uc7a5 \ubb38\uc11c \uac00\uc6b4\ub370 \u300a\ub178\ube44\ubcf4\u300b\uac00 \ub0a8\uc544\uc788\ub2e4. 18\uc138\uae30\uacbd \ud544\uc554\uc11c\uc6d0\uc5d0 \uc18c\uc18d\ub41c \ub178\ube44\ub4e4\uc758 \uc778\uc801 \uc0ac\ud56d\uc744 \uc801\uc740 \uacc4\ubcf4\ub294 \ubaa8\uacc4\ub85c \uc774\uc5b4\uc9c0\ub294 \uc131\uaca9\uc744 \uac00\uc9c0\uace0 \uc788\ub294\ub370, \uc870\uc120\uc758 \uc2e0\ubd84 \uad00\ub828 \ubc95\ub960\uc5d0\uc11c \uc885\ubaa8\ubc95 (\u5f9e\u6bcd\u6cd5) \uc744 \ud0dd\ud588\ub358 \uc0ac\uc815\uc5d0 \uae30\uc778\ud55c \uac83\uc774\ub2e4.", "paragraph_answer": "\uc774\uc0c9\uc801\uc73c\ub85c \ud658\uad00\uc758 \uc9d1\uc548\uc5d0\uc11c \uc871\ubcf4\ub97c \uc81c\uc791\ud55c \uacbd\uc6b0\ub3c4 \uc788\uc5c8\ub294\ub370, \uc774\ub97c \u300a\ub0b4\uc2dc\ubcf4\u300b\ub77c \ud558\uc600\ub2e4. \uc77c\uc81c \uc2dc\ub300\uc5d0 \uc791\uc131\ub41c \u300a\uc591\uc138\uacc4\ubcf4(\u990a\u4e16\u7cfb\u8b5c)\u300b(1920\ub144)\uc640 \u300a\uc5f0\uc591\uad70\uc138\uacc4\u300b\uac00 \ub0a8\uc544 \uc788\uc73c\uba70, \ud544\uc0ac\ubcf8\uc774\ub2e4. \uc774 \uacbd\uc6b0 \ud658\uad00\uc774\ub77c\ub294 \uc9c1\uc5c5\uc801 \ud2b9\uc131\uc0c1 \uc591\uc790\ub97c \ud1b5\ud574 \ub0b4\uc2dc\uc9c1\uc744 \uc138\uc2b5\ud558\uc5ec \uc138\uacc4\ub97c \ub0b4\ub824\uc624\uba74\uc11c \uc591\uc790\uc758 \uc131\ub3c4 \uadf8\ub300\ub85c \uc720\uc9c0\ud558\uace0 \ubcf8\uad00\ub3c4 \uc801\uc5b4\uc11c \uacc4\ucd9c\uc744 \ubc1d\ud78c \uc810\uc774 \ud2b9\uc9d5\uc774\ub2e4. \uc591\ubc18\uc774\ub098 \uc911\uc778\uc774 \uc544\ub2cc \uc2e0\ubd84\uc73c\ub85c \uacc4\ubcf4\ub97c \ub0a8\uae34 \uc0ac\ud68c\uce35\uc740 \ub178\ube44\uc600\ub2e4. \ub178\ube44\uac00 \ub9cc\ub4e0 \uac83\uc774 \uc544\ub2c8\ub77c \ub178\ube44\ub97c \uad00\ub9ac\ud558\ub294 \uc790\ub4e4\uc774 \ub9cc\ub4e0 \uac83\uc73c\ub85c, \uc804\ub77c\ub0a8\ub3c4 \uc7a5\uc131\uad70\uc758 \ud544\uc554\uc11c\uc6d0 \uc18c\uc7a5 \ubb38\uc11c \uac00\uc6b4\ub370 \u300a\ub178\ube44\ubcf4\u300b\uac00 \ub0a8\uc544\uc788\ub2e4. 18\uc138\uae30\uacbd \ud544\uc554\uc11c\uc6d0\uc5d0 \uc18c\uc18d\ub41c \ub178\ube44\ub4e4\uc758 \uc778\uc801 \uc0ac\ud56d\uc744 \uc801\uc740 \uacc4\ubcf4\ub294 \ubaa8\uacc4\ub85c \uc774\uc5b4\uc9c0\ub294 \uc131\uaca9\uc744 \uac00\uc9c0\uace0 \uc788\ub294\ub370, \uc870\uc120\uc758 \uc2e0\ubd84 \uad00\ub828 \ubc95\ub960\uc5d0\uc11c \uc885\ubaa8\ubc95 (\u5f9e\u6bcd\u6cd5)\uc744 \ud0dd\ud588\ub358 \uc0ac\uc815\uc5d0 \uae30\uc778\ud55c \uac83\uc774\ub2e4.", "sentence_answer": "18\uc138\uae30\uacbd \ud544\uc554\uc11c\uc6d0\uc5d0 \uc18c\uc18d\ub41c \ub178\ube44\ub4e4\uc758 \uc778\uc801 \uc0ac\ud56d\uc744 \uc801\uc740 \uacc4\ubcf4\ub294 \ubaa8\uacc4\ub85c \uc774\uc5b4\uc9c0\ub294 \uc131\uaca9\uc744 \uac00\uc9c0\uace0 \uc788\ub294\ub370, \uc870\uc120\uc758 \uc2e0\ubd84 \uad00\ub828 \ubc95\ub960\uc5d0\uc11c \uc885\ubaa8\ubc95 (\u5f9e\u6bcd\u6cd5)", "paragraph_id": "6459958-4-1", "paragraph_question": "question: \ub178\ube44\ub4e4\uc758 \uacc4\ubcf4\uac00 \ubaa8\uacc4\ub85c \uc774\uc5b4\uc9c0\uac8c \ub418\ub294\ub370 \uc601\ud5a5\uc744 \ubbf8\uce5c \ubc95\uc740 \ubb34\uc5c7\uc778\uac00?, context: \uc774\uc0c9\uc801\uc73c\ub85c \ud658\uad00\uc758 \uc9d1\uc548\uc5d0\uc11c \uc871\ubcf4\ub97c \uc81c\uc791\ud55c \uacbd\uc6b0\ub3c4 \uc788\uc5c8\ub294\ub370, \uc774\ub97c \u300a\ub0b4\uc2dc\ubcf4\u300b\ub77c \ud558\uc600\ub2e4. \uc77c\uc81c \uc2dc\ub300\uc5d0 \uc791\uc131\ub41c \u300a\uc591\uc138\uacc4\ubcf4(\u990a\u4e16\u7cfb\u8b5c)\u300b(1920\ub144)\uc640 \u300a\uc5f0\uc591\uad70\uc138\uacc4\u300b\uac00 \ub0a8\uc544 \uc788\uc73c\uba70, \ud544\uc0ac\ubcf8\uc774\ub2e4. \uc774 \uacbd\uc6b0 \ud658\uad00\uc774\ub77c\ub294 \uc9c1\uc5c5\uc801 \ud2b9\uc131\uc0c1 \uc591\uc790\ub97c \ud1b5\ud574 \ub0b4\uc2dc\uc9c1\uc744 \uc138\uc2b5\ud558\uc5ec \uc138\uacc4\ub97c \ub0b4\ub824\uc624\uba74\uc11c \uc591\uc790\uc758 \uc131\ub3c4 \uadf8\ub300\ub85c \uc720\uc9c0\ud558\uace0 \ubcf8\uad00\ub3c4 \uc801\uc5b4\uc11c \uacc4\ucd9c\uc744 \ubc1d\ud78c \uc810\uc774 \ud2b9\uc9d5\uc774\ub2e4. \uc591\ubc18\uc774\ub098 \uc911\uc778\uc774 \uc544\ub2cc \uc2e0\ubd84\uc73c\ub85c \uacc4\ubcf4\ub97c \ub0a8\uae34 \uc0ac\ud68c\uce35\uc740 \ub178\ube44\uc600\ub2e4. \ub178\ube44\uac00 \ub9cc\ub4e0 \uac83\uc774 \uc544\ub2c8\ub77c \ub178\ube44\ub97c \uad00\ub9ac\ud558\ub294 \uc790\ub4e4\uc774 \ub9cc\ub4e0 \uac83\uc73c\ub85c, \uc804\ub77c\ub0a8\ub3c4 \uc7a5\uc131\uad70\uc758 \ud544\uc554\uc11c\uc6d0 \uc18c\uc7a5 \ubb38\uc11c \uac00\uc6b4\ub370 \u300a\ub178\ube44\ubcf4\u300b\uac00 \ub0a8\uc544\uc788\ub2e4. 18\uc138\uae30\uacbd \ud544\uc554\uc11c\uc6d0\uc5d0 \uc18c\uc18d\ub41c \ub178\ube44\ub4e4\uc758 \uc778\uc801 \uc0ac\ud56d\uc744 \uc801\uc740 \uacc4\ubcf4\ub294 \ubaa8\uacc4\ub85c \uc774\uc5b4\uc9c0\ub294 \uc131\uaca9\uc744 \uac00\uc9c0\uace0 \uc788\ub294\ub370, \uc870\uc120\uc758 \uc2e0\ubd84 \uad00\ub828 \ubc95\ub960\uc5d0\uc11c \uc885\ubaa8\ubc95(\u5f9e\u6bcd\u6cd5)\uc744 \ud0dd\ud588\ub358 \uc0ac\uc815\uc5d0 \uae30\uc778\ud55c \uac83\uc774\ub2e4."} {"question": "1989\ub144 \uacbd\uae30\uc758 2\ubd80 \ub9ac\uadf8\uc5d0\uc11c \ub9e8\uccb4\uc2a4\ud130 \uc2dc\ud2f0\ub97c \uac10\ub3c5\ud55c \uc0ac\ub78c\uc740?", "paragraph": "1989\ub144, \uba5c \ub9c8\uce5c\uc758 \ub9e8\uccb4\uc2a4\ud130 \uc2dc\ud2f0\ub294 2\ubd80\ub9ac\uadf8\uc5d0\uc11c 2\uc704\ub97c \uae30\ub85d\ud558\uba70 1\ubd80\ub9ac\uadf8\uc5d0 \ub2e4\uc2dc \uc2b9\uaca9\ud558\uac8c \ub41c\ub2e4. 1989 - 90\uc2dc\uc98c\uc740 \uc5b4\ub824\uc6b4 \uc2dc\uc98c\uc774 \ub418\uc5c8\uace0 89\ub144 11\uc6d4, \ub610\ub2e4\uc2dc \uac15\ub4f1 \uac00\ub2a5\uc131\uc774 \ub192\uc544\uc9d0\uc5d0 \ub530\ub77c \ub9c8\uce5c\uc740 \ud574\uace0 \ub2f9\ud558\uac8c \ub41c\ub2e4. \u524d \uc5d0\ubc84\ud134 \uac10\ub3c5\uc774\uc5c8\ub358 \ud558\uc6cc\ub4dc \ucf04\ub2ec\uc774 \ud300\uc744 \ub9e1\uc544 \uacb0\uad6d \uac15\ub4f1\uc5d0 \uad6c\ud574\ub0b4\uc5c8\uace0 \ub2e4\uc74c\uc2dc\uc98c\uc778 1990 - 91\uc2dc\uc98c\uc5d0\ub294 \uc88b\uc740 \ucd9c\ubc1c\uc744 \ubcf4\uc774\uace0 \uc788\uc5c8\ub2e4. \ud558\uc9c0\ub9cc 1990\ub144 11\uc6d4, \ucf04\ub2ec\uc740 \uc5d0\ubc84\ud134\uc73c\ub85c \ub3cc\uc544\uac00\uac8c \ub418\uc5c8\uace0 34\uc0b4\uc758 \ubbf8\ub4dc\ud544\ub354\uc778 \ud53c\ud130 \ub9ac\ub4dc\uac00 \uc120\uc218 \uacb8 \uac10\ub3c5\uc774 \ub41c\ub2e4. \ub9ac\ub4dc\uc758 \uccab \ub450 \uc2dc\uc98c\uc740 5\uc704\ub97c \uae30\ub85d\ud588\uc9c0\ub9cc UEFA \ucef5\uc5d0\ub294 \uc9c4\ucd9c\ud558\uc9c0 \ubabb\ud588\ub358\uac8c \uc774 \ub2f9\uc2dc\uc5d0\ub294 2, 3\uc704 \ud300\ub9cc \uc9c4\ucd9c\ud560 \uc218 \uc558\uc5c8\uae30 \ub54c\ubb38\uc774\uc5c8\ub2e4. 1992 - 93\uc2dc\uc98c, \ub9e8\uccb4\uc2a4\ud130 \uc2dc\ud2f0\ub294 9\uc704\ub97c \uae30\ub85d\ud588\uc9c0\ub9cc 1993 - 94\uc2dc\uc98c, \ub2e8 \ub450 \uacbd\uae30\ub9cc\uc744 \uce58\ub974\uace0 \ub9ac\ub4dc\ub294 \ud574\uc784 \ub2f9\ud558\uac8c \ub41c\ub2e4.", "answer": "\uba5c \ub9c8\uce5c", "sentence": "1989\ub144, \uba5c \ub9c8\uce5c \uc758 \ub9e8\uccb4\uc2a4\ud130 \uc2dc\ud2f0\ub294 2\ubd80\ub9ac\uadf8\uc5d0\uc11c 2\uc704\ub97c \uae30\ub85d\ud558\uba70 1\ubd80\ub9ac\uadf8\uc5d0 \ub2e4\uc2dc \uc2b9\uaca9\ud558\uac8c \ub41c\ub2e4.", "paragraph_sentence": " 1989\ub144, \uba5c \ub9c8\uce5c \uc758 \ub9e8\uccb4\uc2a4\ud130 \uc2dc\ud2f0\ub294 2\ubd80\ub9ac\uadf8\uc5d0\uc11c 2\uc704\ub97c \uae30\ub85d\ud558\uba70 1\ubd80\ub9ac\uadf8\uc5d0 \ub2e4\uc2dc \uc2b9\uaca9\ud558\uac8c \ub41c\ub2e4. 1989 - 90\uc2dc\uc98c\uc740 \uc5b4\ub824\uc6b4 \uc2dc\uc98c\uc774 \ub418\uc5c8\uace0 89\ub144 11\uc6d4, \ub610\ub2e4\uc2dc \uac15\ub4f1 \uac00\ub2a5\uc131\uc774 \ub192\uc544\uc9d0\uc5d0 \ub530\ub77c \ub9c8\uce5c\uc740 \ud574\uace0 \ub2f9\ud558\uac8c \ub41c\ub2e4. \u524d \uc5d0\ubc84\ud134 \uac10\ub3c5\uc774\uc5c8\ub358 \ud558\uc6cc\ub4dc \ucf04\ub2ec\uc774 \ud300\uc744 \ub9e1\uc544 \uacb0\uad6d \uac15\ub4f1\uc5d0 \uad6c\ud574\ub0b4\uc5c8\uace0 \ub2e4\uc74c\uc2dc\uc98c\uc778 1990 - 91\uc2dc\uc98c\uc5d0\ub294 \uc88b\uc740 \ucd9c\ubc1c\uc744 \ubcf4\uc774\uace0 \uc788\uc5c8\ub2e4. \ud558\uc9c0\ub9cc 1990\ub144 11\uc6d4, \ucf04\ub2ec\uc740 \uc5d0\ubc84\ud134\uc73c\ub85c \ub3cc\uc544\uac00\uac8c \ub418\uc5c8\uace0 34\uc0b4\uc758 \ubbf8\ub4dc\ud544\ub354\uc778 \ud53c\ud130 \ub9ac\ub4dc\uac00 \uc120\uc218 \uacb8 \uac10\ub3c5\uc774 \ub41c\ub2e4. \ub9ac\ub4dc\uc758 \uccab \ub450 \uc2dc\uc98c\uc740 5\uc704\ub97c \uae30\ub85d\ud588\uc9c0\ub9cc UEFA \ucef5\uc5d0\ub294 \uc9c4\ucd9c\ud558\uc9c0 \ubabb\ud588\ub358\uac8c \uc774 \ub2f9\uc2dc\uc5d0\ub294 2, 3\uc704 \ud300\ub9cc \uc9c4\ucd9c\ud560 \uc218 \uc558\uc5c8\uae30 \ub54c\ubb38\uc774\uc5c8\ub2e4. 1992 - 93\uc2dc\uc98c, \ub9e8\uccb4\uc2a4\ud130 \uc2dc\ud2f0\ub294 9\uc704\ub97c \uae30\ub85d\ud588\uc9c0\ub9cc 1993 - 94\uc2dc\uc98c, \ub2e8 \ub450 \uacbd\uae30\ub9cc\uc744 \uce58\ub974\uace0 \ub9ac\ub4dc\ub294 \ud574\uc784 \ub2f9\ud558\uac8c \ub41c\ub2e4.", "paragraph_answer": "1989\ub144, \uba5c \ub9c8\uce5c \uc758 \ub9e8\uccb4\uc2a4\ud130 \uc2dc\ud2f0\ub294 2\ubd80\ub9ac\uadf8\uc5d0\uc11c 2\uc704\ub97c \uae30\ub85d\ud558\uba70 1\ubd80\ub9ac\uadf8\uc5d0 \ub2e4\uc2dc \uc2b9\uaca9\ud558\uac8c \ub41c\ub2e4. 1989 - 90\uc2dc\uc98c\uc740 \uc5b4\ub824\uc6b4 \uc2dc\uc98c\uc774 \ub418\uc5c8\uace0 89\ub144 11\uc6d4, \ub610\ub2e4\uc2dc \uac15\ub4f1 \uac00\ub2a5\uc131\uc774 \ub192\uc544\uc9d0\uc5d0 \ub530\ub77c \ub9c8\uce5c\uc740 \ud574\uace0 \ub2f9\ud558\uac8c \ub41c\ub2e4. \u524d \uc5d0\ubc84\ud134 \uac10\ub3c5\uc774\uc5c8\ub358 \ud558\uc6cc\ub4dc \ucf04\ub2ec\uc774 \ud300\uc744 \ub9e1\uc544 \uacb0\uad6d \uac15\ub4f1\uc5d0 \uad6c\ud574\ub0b4\uc5c8\uace0 \ub2e4\uc74c\uc2dc\uc98c\uc778 1990 - 91\uc2dc\uc98c\uc5d0\ub294 \uc88b\uc740 \ucd9c\ubc1c\uc744 \ubcf4\uc774\uace0 \uc788\uc5c8\ub2e4. \ud558\uc9c0\ub9cc 1990\ub144 11\uc6d4, \ucf04\ub2ec\uc740 \uc5d0\ubc84\ud134\uc73c\ub85c \ub3cc\uc544\uac00\uac8c \ub418\uc5c8\uace0 34\uc0b4\uc758 \ubbf8\ub4dc\ud544\ub354\uc778 \ud53c\ud130 \ub9ac\ub4dc\uac00 \uc120\uc218 \uacb8 \uac10\ub3c5\uc774 \ub41c\ub2e4. \ub9ac\ub4dc\uc758 \uccab \ub450 \uc2dc\uc98c\uc740 5\uc704\ub97c \uae30\ub85d\ud588\uc9c0\ub9cc UEFA \ucef5\uc5d0\ub294 \uc9c4\ucd9c\ud558\uc9c0 \ubabb\ud588\ub358\uac8c \uc774 \ub2f9\uc2dc\uc5d0\ub294 2, 3\uc704 \ud300\ub9cc \uc9c4\ucd9c\ud560 \uc218 \uc558\uc5c8\uae30 \ub54c\ubb38\uc774\uc5c8\ub2e4. 1992 - 93\uc2dc\uc98c, \ub9e8\uccb4\uc2a4\ud130 \uc2dc\ud2f0\ub294 9\uc704\ub97c \uae30\ub85d\ud588\uc9c0\ub9cc 1993 - 94\uc2dc\uc98c, \ub2e8 \ub450 \uacbd\uae30\ub9cc\uc744 \uce58\ub974\uace0 \ub9ac\ub4dc\ub294 \ud574\uc784 \ub2f9\ud558\uac8c \ub41c\ub2e4.", "sentence_answer": "1989\ub144, \uba5c \ub9c8\uce5c \uc758 \ub9e8\uccb4\uc2a4\ud130 \uc2dc\ud2f0\ub294 2\ubd80\ub9ac\uadf8\uc5d0\uc11c 2\uc704\ub97c \uae30\ub85d\ud558\uba70 1\ubd80\ub9ac\uadf8\uc5d0 \ub2e4\uc2dc \uc2b9\uaca9\ud558\uac8c \ub41c\ub2e4.", "paragraph_id": "6465982-10-0", "paragraph_question": "question: 1989\ub144 \uacbd\uae30\uc758 2\ubd80 \ub9ac\uadf8\uc5d0\uc11c \ub9e8\uccb4\uc2a4\ud130 \uc2dc\ud2f0\ub97c \uac10\ub3c5\ud55c \uc0ac\ub78c\uc740?, context: 1989\ub144, \uba5c \ub9c8\uce5c\uc758 \ub9e8\uccb4\uc2a4\ud130 \uc2dc\ud2f0\ub294 2\ubd80\ub9ac\uadf8\uc5d0\uc11c 2\uc704\ub97c \uae30\ub85d\ud558\uba70 1\ubd80\ub9ac\uadf8\uc5d0 \ub2e4\uc2dc \uc2b9\uaca9\ud558\uac8c \ub41c\ub2e4. 1989 - 90\uc2dc\uc98c\uc740 \uc5b4\ub824\uc6b4 \uc2dc\uc98c\uc774 \ub418\uc5c8\uace0 89\ub144 11\uc6d4, \ub610\ub2e4\uc2dc \uac15\ub4f1 \uac00\ub2a5\uc131\uc774 \ub192\uc544\uc9d0\uc5d0 \ub530\ub77c \ub9c8\uce5c\uc740 \ud574\uace0 \ub2f9\ud558\uac8c \ub41c\ub2e4. \u524d \uc5d0\ubc84\ud134 \uac10\ub3c5\uc774\uc5c8\ub358 \ud558\uc6cc\ub4dc \ucf04\ub2ec\uc774 \ud300\uc744 \ub9e1\uc544 \uacb0\uad6d \uac15\ub4f1\uc5d0 \uad6c\ud574\ub0b4\uc5c8\uace0 \ub2e4\uc74c\uc2dc\uc98c\uc778 1990 - 91\uc2dc\uc98c\uc5d0\ub294 \uc88b\uc740 \ucd9c\ubc1c\uc744 \ubcf4\uc774\uace0 \uc788\uc5c8\ub2e4. \ud558\uc9c0\ub9cc 1990\ub144 11\uc6d4, \ucf04\ub2ec\uc740 \uc5d0\ubc84\ud134\uc73c\ub85c \ub3cc\uc544\uac00\uac8c \ub418\uc5c8\uace0 34\uc0b4\uc758 \ubbf8\ub4dc\ud544\ub354\uc778 \ud53c\ud130 \ub9ac\ub4dc\uac00 \uc120\uc218 \uacb8 \uac10\ub3c5\uc774 \ub41c\ub2e4. \ub9ac\ub4dc\uc758 \uccab \ub450 \uc2dc\uc98c\uc740 5\uc704\ub97c \uae30\ub85d\ud588\uc9c0\ub9cc UEFA \ucef5\uc5d0\ub294 \uc9c4\ucd9c\ud558\uc9c0 \ubabb\ud588\ub358\uac8c \uc774 \ub2f9\uc2dc\uc5d0\ub294 2, 3\uc704 \ud300\ub9cc \uc9c4\ucd9c\ud560 \uc218 \uc558\uc5c8\uae30 \ub54c\ubb38\uc774\uc5c8\ub2e4. 1992 - 93\uc2dc\uc98c, \ub9e8\uccb4\uc2a4\ud130 \uc2dc\ud2f0\ub294 9\uc704\ub97c \uae30\ub85d\ud588\uc9c0\ub9cc 1993 - 94\uc2dc\uc98c, \ub2e8 \ub450 \uacbd\uae30\ub9cc\uc744 \uce58\ub974\uace0 \ub9ac\ub4dc\ub294 \ud574\uc784 \ub2f9\ud558\uac8c \ub41c\ub2e4."} {"question": "\ub0a8\uc790\ub4e4 \uc0ac\uc774\uc5d0\uc11c \uc11c\ub85c\uc758 \uacb0\uc18d\ub825\uc744 \ub2e4\uc838\uc8fc\ub294 \uac83\uc740?", "paragraph": "\uad50\uc218\uc640 \uadf8\ub140\uc758 \ub3d9\ub8cc\ub4e4\uc740 \uc6c3\uc74c\uc18c\ub9ac\uac00 \uc0ac\ud68c\uc801 \uc2e0\ud638\uc774\uba70 \ub0a8\uc790\uc640 \uc5ec\uc790\uac00 \uc778\uac04\uad00\uacc4\ub97c \ub300\ud558\ub294 \ubc29\uc2dd\uc774 \ub2e4\ub974\ub4ef \uc6c3\uc74c\uc744 \uc774\uc6a9\ud558\ub294 \ubc29\uc2dd \ub610\ud55c \ub2e4\ub974\uae30 \ub54c\ubb38\uc774\ub77c\uace0 \uc124\uba85\ud588\ub2e4. \ub0a8\uc790\ub4e4\uc740 \ud06c\uac8c \uc6c3\uac70\ub098 \uc18c\ub9ac\uce58\ub294 \ubaa8\uc2b5\uc774 \uadf8\ub2e4\uc9c0 \ub9e4\ub825\uc801\uc73c\ub85c \ubcf4\uc774\uc9c0 \uc54a\uae30 \ub54c\ubb38\uc5d0 \uc5ec\uc790\ub4e4 \uc55e\uc5d0\uc11c \uc790\uc81c\ud558\uc9c0\ub9cc, \ub0a8\uc790\ub4e4 \uc0ac\uc774\uc5d0\uc120 \ucee4\ub2e4\ub780 \uc6c3\uc74c\uc774 \uc11c\ub85c\uc758 \uacb0\uc18d\ub825\uc744 \uac15\ud654\uc2dc\ucf1c\uc8fc\uae30 \ub54c\ubb38\uc5d0 \ub0a8\uc790 \ub3d9\ub8cc\ub4e4\uacfc \uc788\uc744 \ub54c\ub294 \ud06c\uac8c \uc6c3\ub294\ub2e4. \ubc18\uba74 \uc5ec\uc790\ub4e4\uc740 \uc790\uc2e0\uc758 \uc6c3\uc74c\uc774 \ub0af\uc120 \uc0ac\ub78c\uc774\ub098 \ub0a8\uc131\uacfc \ud568\uaed8 \uc788\uc744 \ub54c \ubc29\uc548\uc5d0 \uac10\ub3c4\ub294 \uc5b4\uc0c9\ud568\uc774\ub098 \uae34\uc7a5\uac10\uc744 \uae68\uc904 \uc218 \uc788\ub2e4\ub294 \uac83\uc744 \uc798 \uc54c\uace0 \uc788\ub2e4. \uadf8\ub798\uc11c \uadf8\ub4e4\uc740 \ub0a8\uc790\uc640 \ud568\uaed8 \uc788\uc744 \ub54c \ud2b9\ud788 \ub0af\uc120 \uc0ac\ub78c\uacfc \ud568\uaed8 \uc788\uc744 \ub54c \ub354 \ud06c\uac8c \uc6c3\ub294\ub2e4. \uc774\ub7ec\ud55c \uc774\uc720 \ub54c\ubb38\uc5d0 \uc6c3\uc74c \ud2b8\ub799\uc744 \ub9cc\ub4e4 \ub54c(\uc6c3\uc74c \ub354\ube59\uc744 \ud560 \ub54c) \uc5ec\uc131 \ucc38\uac00\uc790\uac00 \ub354 \uc120\ud638\ub418\ub294 \uac83\uc774\ub2e4.", "answer": "\ucee4\ub2e4\ub780 \uc6c3\uc74c", "sentence": "\ub0a8\uc790\ub4e4 \uc0ac\uc774\uc5d0\uc120 \ucee4\ub2e4\ub780 \uc6c3\uc74c \uc774 \uc11c\ub85c\uc758 \uacb0\uc18d\ub825\uc744 \uac15\ud654\uc2dc\ucf1c\uc8fc\uae30 \ub54c\ubb38\uc5d0 \ub0a8\uc790 \ub3d9\ub8cc\ub4e4\uacfc \uc788\uc744 \ub54c\ub294 \ud06c\uac8c \uc6c3\ub294\ub2e4.", "paragraph_sentence": "\uad50\uc218\uc640 \uadf8\ub140\uc758 \ub3d9\ub8cc\ub4e4\uc740 \uc6c3\uc74c\uc18c\ub9ac\uac00 \uc0ac\ud68c\uc801 \uc2e0\ud638\uc774\uba70 \ub0a8\uc790\uc640 \uc5ec\uc790\uac00 \uc778\uac04\uad00\uacc4\ub97c \ub300\ud558\ub294 \ubc29\uc2dd\uc774 \ub2e4\ub974\ub4ef \uc6c3\uc74c\uc744 \uc774\uc6a9\ud558\ub294 \ubc29\uc2dd \ub610\ud55c \ub2e4\ub974\uae30 \ub54c\ubb38\uc774\ub77c\uace0 \uc124\uba85\ud588\ub2e4. \ub0a8\uc790\ub4e4\uc740 \ud06c\uac8c \uc6c3\uac70\ub098 \uc18c\ub9ac\uce58\ub294 \ubaa8\uc2b5\uc774 \uadf8\ub2e4\uc9c0 \ub9e4\ub825\uc801\uc73c\ub85c \ubcf4\uc774\uc9c0 \uc54a\uae30 \ub54c\ubb38\uc5d0 \uc5ec\uc790\ub4e4 \uc55e\uc5d0\uc11c \uc790\uc81c\ud558\uc9c0\ub9cc, \ub0a8\uc790\ub4e4 \uc0ac\uc774\uc5d0\uc120 \ucee4\ub2e4\ub780 \uc6c3\uc74c \uc774 \uc11c\ub85c\uc758 \uacb0\uc18d\ub825\uc744 \uac15\ud654\uc2dc\ucf1c\uc8fc\uae30 \ub54c\ubb38\uc5d0 \ub0a8\uc790 \ub3d9\ub8cc\ub4e4\uacfc \uc788\uc744 \ub54c\ub294 \ud06c\uac8c \uc6c3\ub294\ub2e4. \ubc18\uba74 \uc5ec\uc790\ub4e4\uc740 \uc790\uc2e0\uc758 \uc6c3\uc74c\uc774 \ub0af\uc120 \uc0ac\ub78c\uc774\ub098 \ub0a8\uc131\uacfc \ud568\uaed8 \uc788\uc744 \ub54c \ubc29\uc548\uc5d0 \uac10\ub3c4\ub294 \uc5b4\uc0c9\ud568\uc774\ub098 \uae34\uc7a5\uac10\uc744 \uae68\uc904 \uc218 \uc788\ub2e4\ub294 \uac83\uc744 \uc798 \uc54c\uace0 \uc788\ub2e4. \uadf8\ub798\uc11c \uadf8\ub4e4\uc740 \ub0a8\uc790\uc640 \ud568\uaed8 \uc788\uc744 \ub54c \ud2b9\ud788 \ub0af\uc120 \uc0ac\ub78c\uacfc \ud568\uaed8 \uc788\uc744 \ub54c \ub354 \ud06c\uac8c \uc6c3\ub294\ub2e4. \uc774\ub7ec\ud55c \uc774\uc720 \ub54c\ubb38\uc5d0 \uc6c3\uc74c \ud2b8\ub799\uc744 \ub9cc\ub4e4 \ub54c(\uc6c3\uc74c \ub354\ube59\uc744 \ud560 \ub54c) \uc5ec\uc131 \ucc38\uac00\uc790\uac00 \ub354 \uc120\ud638\ub418\ub294 \uac83\uc774\ub2e4.", "paragraph_answer": "\uad50\uc218\uc640 \uadf8\ub140\uc758 \ub3d9\ub8cc\ub4e4\uc740 \uc6c3\uc74c\uc18c\ub9ac\uac00 \uc0ac\ud68c\uc801 \uc2e0\ud638\uc774\uba70 \ub0a8\uc790\uc640 \uc5ec\uc790\uac00 \uc778\uac04\uad00\uacc4\ub97c \ub300\ud558\ub294 \ubc29\uc2dd\uc774 \ub2e4\ub974\ub4ef \uc6c3\uc74c\uc744 \uc774\uc6a9\ud558\ub294 \ubc29\uc2dd \ub610\ud55c \ub2e4\ub974\uae30 \ub54c\ubb38\uc774\ub77c\uace0 \uc124\uba85\ud588\ub2e4. \ub0a8\uc790\ub4e4\uc740 \ud06c\uac8c \uc6c3\uac70\ub098 \uc18c\ub9ac\uce58\ub294 \ubaa8\uc2b5\uc774 \uadf8\ub2e4\uc9c0 \ub9e4\ub825\uc801\uc73c\ub85c \ubcf4\uc774\uc9c0 \uc54a\uae30 \ub54c\ubb38\uc5d0 \uc5ec\uc790\ub4e4 \uc55e\uc5d0\uc11c \uc790\uc81c\ud558\uc9c0\ub9cc, \ub0a8\uc790\ub4e4 \uc0ac\uc774\uc5d0\uc120 \ucee4\ub2e4\ub780 \uc6c3\uc74c \uc774 \uc11c\ub85c\uc758 \uacb0\uc18d\ub825\uc744 \uac15\ud654\uc2dc\ucf1c\uc8fc\uae30 \ub54c\ubb38\uc5d0 \ub0a8\uc790 \ub3d9\ub8cc\ub4e4\uacfc \uc788\uc744 \ub54c\ub294 \ud06c\uac8c \uc6c3\ub294\ub2e4. \ubc18\uba74 \uc5ec\uc790\ub4e4\uc740 \uc790\uc2e0\uc758 \uc6c3\uc74c\uc774 \ub0af\uc120 \uc0ac\ub78c\uc774\ub098 \ub0a8\uc131\uacfc \ud568\uaed8 \uc788\uc744 \ub54c \ubc29\uc548\uc5d0 \uac10\ub3c4\ub294 \uc5b4\uc0c9\ud568\uc774\ub098 \uae34\uc7a5\uac10\uc744 \uae68\uc904 \uc218 \uc788\ub2e4\ub294 \uac83\uc744 \uc798 \uc54c\uace0 \uc788\ub2e4. \uadf8\ub798\uc11c \uadf8\ub4e4\uc740 \ub0a8\uc790\uc640 \ud568\uaed8 \uc788\uc744 \ub54c \ud2b9\ud788 \ub0af\uc120 \uc0ac\ub78c\uacfc \ud568\uaed8 \uc788\uc744 \ub54c \ub354 \ud06c\uac8c \uc6c3\ub294\ub2e4. \uc774\ub7ec\ud55c \uc774\uc720 \ub54c\ubb38\uc5d0 \uc6c3\uc74c \ud2b8\ub799\uc744 \ub9cc\ub4e4 \ub54c(\uc6c3\uc74c \ub354\ube59\uc744 \ud560 \ub54c) \uc5ec\uc131 \ucc38\uac00\uc790\uac00 \ub354 \uc120\ud638\ub418\ub294 \uac83\uc774\ub2e4.", "sentence_answer": "\ub0a8\uc790\ub4e4 \uc0ac\uc774\uc5d0\uc120 \ucee4\ub2e4\ub780 \uc6c3\uc74c \uc774 \uc11c\ub85c\uc758 \uacb0\uc18d\ub825\uc744 \uac15\ud654\uc2dc\ucf1c\uc8fc\uae30 \ub54c\ubb38\uc5d0 \ub0a8\uc790 \ub3d9\ub8cc\ub4e4\uacfc \uc788\uc744 \ub54c\ub294 \ud06c\uac8c \uc6c3\ub294\ub2e4.", "paragraph_id": "6542091-2-1", "paragraph_question": "question: \ub0a8\uc790\ub4e4 \uc0ac\uc774\uc5d0\uc11c \uc11c\ub85c\uc758 \uacb0\uc18d\ub825\uc744 \ub2e4\uc838\uc8fc\ub294 \uac83\uc740?, context: \uad50\uc218\uc640 \uadf8\ub140\uc758 \ub3d9\ub8cc\ub4e4\uc740 \uc6c3\uc74c\uc18c\ub9ac\uac00 \uc0ac\ud68c\uc801 \uc2e0\ud638\uc774\uba70 \ub0a8\uc790\uc640 \uc5ec\uc790\uac00 \uc778\uac04\uad00\uacc4\ub97c \ub300\ud558\ub294 \ubc29\uc2dd\uc774 \ub2e4\ub974\ub4ef \uc6c3\uc74c\uc744 \uc774\uc6a9\ud558\ub294 \ubc29\uc2dd \ub610\ud55c \ub2e4\ub974\uae30 \ub54c\ubb38\uc774\ub77c\uace0 \uc124\uba85\ud588\ub2e4. \ub0a8\uc790\ub4e4\uc740 \ud06c\uac8c \uc6c3\uac70\ub098 \uc18c\ub9ac\uce58\ub294 \ubaa8\uc2b5\uc774 \uadf8\ub2e4\uc9c0 \ub9e4\ub825\uc801\uc73c\ub85c \ubcf4\uc774\uc9c0 \uc54a\uae30 \ub54c\ubb38\uc5d0 \uc5ec\uc790\ub4e4 \uc55e\uc5d0\uc11c \uc790\uc81c\ud558\uc9c0\ub9cc, \ub0a8\uc790\ub4e4 \uc0ac\uc774\uc5d0\uc120 \ucee4\ub2e4\ub780 \uc6c3\uc74c\uc774 \uc11c\ub85c\uc758 \uacb0\uc18d\ub825\uc744 \uac15\ud654\uc2dc\ucf1c\uc8fc\uae30 \ub54c\ubb38\uc5d0 \ub0a8\uc790 \ub3d9\ub8cc\ub4e4\uacfc \uc788\uc744 \ub54c\ub294 \ud06c\uac8c \uc6c3\ub294\ub2e4. \ubc18\uba74 \uc5ec\uc790\ub4e4\uc740 \uc790\uc2e0\uc758 \uc6c3\uc74c\uc774 \ub0af\uc120 \uc0ac\ub78c\uc774\ub098 \ub0a8\uc131\uacfc \ud568\uaed8 \uc788\uc744 \ub54c \ubc29\uc548\uc5d0 \uac10\ub3c4\ub294 \uc5b4\uc0c9\ud568\uc774\ub098 \uae34\uc7a5\uac10\uc744 \uae68\uc904 \uc218 \uc788\ub2e4\ub294 \uac83\uc744 \uc798 \uc54c\uace0 \uc788\ub2e4. \uadf8\ub798\uc11c \uadf8\ub4e4\uc740 \ub0a8\uc790\uc640 \ud568\uaed8 \uc788\uc744 \ub54c \ud2b9\ud788 \ub0af\uc120 \uc0ac\ub78c\uacfc \ud568\uaed8 \uc788\uc744 \ub54c \ub354 \ud06c\uac8c \uc6c3\ub294\ub2e4. \uc774\ub7ec\ud55c \uc774\uc720 \ub54c\ubb38\uc5d0 \uc6c3\uc74c \ud2b8\ub799\uc744 \ub9cc\ub4e4 \ub54c(\uc6c3\uc74c \ub354\ube59\uc744 \ud560 \ub54c) \uc5ec\uc131 \ucc38\uac00\uc790\uac00 \ub354 \uc120\ud638\ub418\ub294 \uac83\uc774\ub2e4."} {"question": "\uc784\uae08\uc18c\ub4dd \uc678 \ubcf4\ud5d8\ub8cc\uac00 \ubd80\uacfc\ub418\uc9c0 \uc54a\ub294 \uc885\ud569\uc18c\ub4dd\uc561\uc740?", "paragraph": "\ud604\ud589 \uac74\uac15\ubcf4\ud5d8\ub8cc \ubd80\uacfc\uccb4\uacc4\ub294 \uc9c1\uc7a5\uac00\uc785\uc790\uc640 \uc9c0\uc5ed\uac00\uc785\uc790 \uac04 \uac74\uac15\ubcf4\ud5d8\ub8cc \uc0b0\uc815\ubc29\uc2dd\uc5d0 \ucc28\uc774\ub97c \ub454\ub2e4. \ud2b9\ud788 \uc9c0\uc5ed\uac00\uc785\uc790\uc758 \uacbd\uc6b0 \uc18c\ub4dd \uc678\uc5d0\ub3c4 \uc790\ub3d9\ucc28\uc640 \uc7ac\uc0b0\uc5d0\ub3c4 \uac74\uac15\ubcf4\ud5d8\ub8cc\uac00 \ubd80\uacfc\ub428\uc740 \ubb3c\ub860, \uac00\uc871\uc758 \ub098\uc774\uc640 \uc131\ubcc4, \uac00\uad6c\uc6d0\uc218\uc5d0 \ub530\ub77c \ub610 \uac74\uac15\ubcf4\ud5d8\ub8cc\uac00 \ucc28\ub4f1\uc801\uc73c\ub85c \ubd80\uacfc\ub41c\ub2e4. \uadf8\ub7ec\ub098 \uc774\uc640\ub294 \ub2ec\ub9ac \uc9c1\uc7a5\uac00\uc785\uc790\uc758 \uacbd\uc6b0 \uc784\uae08\uc18c\ub4dd\uc5d0 \ub530\ub77c \uac74\uac15\ubcf4\ud5d8\ub8cc\uac00 \ubd80\uacfc\ub418\uace0, \uc784\uae08\uc18c\ub4dd \uc678\uc5d0 \uc885\ud569\uc18c\ub4dd\uc774 \uc5f0\uac04 7,200\ub9cc\uc6d0\uc744 \ub118\uc9c0 \uc54a\uc73c\uba74 \uac74\uac15\ubcf4\ud5d8\ub8cc\uac00 \ubd80\uacfc\ub418\uc9c0 \uc54a\uc558\ub2e4. \ub54c\ubb38\uc5d0 \uc77c\ubd80 \uace0\uc18c\ub4dd\uce35\uc740 \uc774\uc790\uc18c\ub4dd, \ubc30\ub2f9\uc18c\ub4dd, \uc784\ub300\uc18c\ub4dd \ub4f1 \uc801\uc9c0 \uc54a\uc740 \uc885\ud569\uc18c\ub4dd\uc744 \uac00\uc9c0\uace0 \uc788\uc74c\uc5d0\ub3c4 \ubd88\uad6c\ud558\uace0 \uac74\uac15\ubcf4\ud5d8\ub8cc\ub97c \ub0a9\ubd80\ud558\uc9c0 \uc54a\uc558\ub2e4. \uc11c\ubbfc\ub4e4\uc740 \uc18c\ub4dd\uc5d0 \ube44\ud574 \ub9ce\uc740 \uac74\uac15\ubcf4\ud5d8\ub8cc\ub97c \ubd80\ub2f4\ud558\uace0, \uc77c\ubd80 \uace0\uc18c\ub4dd\uce35\uc740 \uc18c\ub4dd\uc5d0 \ube44\ud574 \uc801\uac8c \ubd80\ub2f4\ud558\ub294 \uc148\uc774\ub2e4. \uc774 \uac19\uc740 \ud3d0\ud574\ub97c \ubc29\uc9c0\ud558\uae30 \uc704\ud574 \uac74\uac15\ubcf4\ud5d8\ub8cc \ubd80\uacfc\uae30\uc900\uc744 \uc18c\ub4dd \uae30\uc900\uc73c\ub85c \ub2e8\uc77c\ud654\ud558\uace0\uc790 \ud558\ub294 \uac83\uc774\ub2e4.", "answer": "7,200\ub9cc\uc6d0", "sentence": "\uc5f0\uac04 7,200\ub9cc\uc6d0 \uc744 \ub118\uc9c0 \uc54a\uc73c\uba74 \uac74\uac15\ubcf4\ud5d8\ub8cc\uac00 \ubd80\uacfc\ub418\uc9c0 \uc54a\uc558\ub2e4.", "paragraph_sentence": "\ud604\ud589 \uac74\uac15\ubcf4\ud5d8\ub8cc \ubd80\uacfc\uccb4\uacc4\ub294 \uc9c1\uc7a5\uac00\uc785\uc790\uc640 \uc9c0\uc5ed\uac00\uc785\uc790 \uac04 \uac74\uac15\ubcf4\ud5d8\ub8cc \uc0b0\uc815\ubc29\uc2dd\uc5d0 \ucc28\uc774\ub97c \ub454\ub2e4. \ud2b9\ud788 \uc9c0\uc5ed\uac00\uc785\uc790\uc758 \uacbd\uc6b0 \uc18c\ub4dd \uc678\uc5d0\ub3c4 \uc790\ub3d9\ucc28\uc640 \uc7ac\uc0b0\uc5d0\ub3c4 \uac74\uac15\ubcf4\ud5d8\ub8cc\uac00 \ubd80\uacfc\ub428\uc740 \ubb3c\ub860, \uac00\uc871\uc758 \ub098\uc774\uc640 \uc131\ubcc4, \uac00\uad6c\uc6d0\uc218\uc5d0 \ub530\ub77c \ub610 \uac74\uac15\ubcf4\ud5d8\ub8cc\uac00 \ucc28\ub4f1\uc801\uc73c\ub85c \ubd80\uacfc\ub41c\ub2e4. \uadf8\ub7ec\ub098 \uc774\uc640\ub294 \ub2ec\ub9ac \uc9c1\uc7a5\uac00\uc785\uc790\uc758 \uacbd\uc6b0 \uc784\uae08\uc18c\ub4dd\uc5d0 \ub530\ub77c \uac74\uac15\ubcf4\ud5d8\ub8cc\uac00 \ubd80\uacfc\ub418\uace0, \uc784\uae08\uc18c\ub4dd \uc678\uc5d0 \uc885\ud569\uc18c\ub4dd\uc774 \uc5f0\uac04 7,200\ub9cc\uc6d0 \uc744 \ub118\uc9c0 \uc54a\uc73c\uba74 \uac74\uac15\ubcf4\ud5d8\ub8cc\uac00 \ubd80\uacfc\ub418\uc9c0 \uc54a\uc558\ub2e4. \ub54c\ubb38\uc5d0 \uc77c\ubd80 \uace0\uc18c\ub4dd\uce35\uc740 \uc774\uc790\uc18c\ub4dd, \ubc30\ub2f9\uc18c\ub4dd, \uc784\ub300\uc18c\ub4dd \ub4f1 \uc801\uc9c0 \uc54a\uc740 \uc885\ud569\uc18c\ub4dd\uc744 \uac00\uc9c0\uace0 \uc788\uc74c\uc5d0\ub3c4 \ubd88\uad6c\ud558\uace0 \uac74\uac15\ubcf4\ud5d8\ub8cc\ub97c \ub0a9\ubd80\ud558\uc9c0 \uc54a\uc558\ub2e4. \uc11c\ubbfc\ub4e4\uc740 \uc18c\ub4dd\uc5d0 \ube44\ud574 \ub9ce\uc740 \uac74\uac15\ubcf4\ud5d8\ub8cc\ub97c \ubd80\ub2f4\ud558\uace0, \uc77c\ubd80 \uace0\uc18c\ub4dd\uce35\uc740 \uc18c\ub4dd\uc5d0 \ube44\ud574 \uc801\uac8c \ubd80\ub2f4\ud558\ub294 \uc148\uc774\ub2e4. \uc774 \uac19\uc740 \ud3d0\ud574\ub97c \ubc29\uc9c0\ud558\uae30 \uc704\ud574 \uac74\uac15\ubcf4\ud5d8\ub8cc \ubd80\uacfc\uae30\uc900\uc744 \uc18c\ub4dd \uae30\uc900\uc73c\ub85c \ub2e8\uc77c\ud654\ud558\uace0\uc790 \ud558\ub294 \uac83\uc774\ub2e4.", "paragraph_answer": "\ud604\ud589 \uac74\uac15\ubcf4\ud5d8\ub8cc \ubd80\uacfc\uccb4\uacc4\ub294 \uc9c1\uc7a5\uac00\uc785\uc790\uc640 \uc9c0\uc5ed\uac00\uc785\uc790 \uac04 \uac74\uac15\ubcf4\ud5d8\ub8cc \uc0b0\uc815\ubc29\uc2dd\uc5d0 \ucc28\uc774\ub97c \ub454\ub2e4. \ud2b9\ud788 \uc9c0\uc5ed\uac00\uc785\uc790\uc758 \uacbd\uc6b0 \uc18c\ub4dd \uc678\uc5d0\ub3c4 \uc790\ub3d9\ucc28\uc640 \uc7ac\uc0b0\uc5d0\ub3c4 \uac74\uac15\ubcf4\ud5d8\ub8cc\uac00 \ubd80\uacfc\ub428\uc740 \ubb3c\ub860, \uac00\uc871\uc758 \ub098\uc774\uc640 \uc131\ubcc4, \uac00\uad6c\uc6d0\uc218\uc5d0 \ub530\ub77c \ub610 \uac74\uac15\ubcf4\ud5d8\ub8cc\uac00 \ucc28\ub4f1\uc801\uc73c\ub85c \ubd80\uacfc\ub41c\ub2e4. \uadf8\ub7ec\ub098 \uc774\uc640\ub294 \ub2ec\ub9ac \uc9c1\uc7a5\uac00\uc785\uc790\uc758 \uacbd\uc6b0 \uc784\uae08\uc18c\ub4dd\uc5d0 \ub530\ub77c \uac74\uac15\ubcf4\ud5d8\ub8cc\uac00 \ubd80\uacfc\ub418\uace0, \uc784\uae08\uc18c\ub4dd \uc678\uc5d0 \uc885\ud569\uc18c\ub4dd\uc774 \uc5f0\uac04 7,200\ub9cc\uc6d0 \uc744 \ub118\uc9c0 \uc54a\uc73c\uba74 \uac74\uac15\ubcf4\ud5d8\ub8cc\uac00 \ubd80\uacfc\ub418\uc9c0 \uc54a\uc558\ub2e4. \ub54c\ubb38\uc5d0 \uc77c\ubd80 \uace0\uc18c\ub4dd\uce35\uc740 \uc774\uc790\uc18c\ub4dd, \ubc30\ub2f9\uc18c\ub4dd, \uc784\ub300\uc18c\ub4dd \ub4f1 \uc801\uc9c0 \uc54a\uc740 \uc885\ud569\uc18c\ub4dd\uc744 \uac00\uc9c0\uace0 \uc788\uc74c\uc5d0\ub3c4 \ubd88\uad6c\ud558\uace0 \uac74\uac15\ubcf4\ud5d8\ub8cc\ub97c \ub0a9\ubd80\ud558\uc9c0 \uc54a\uc558\ub2e4. \uc11c\ubbfc\ub4e4\uc740 \uc18c\ub4dd\uc5d0 \ube44\ud574 \ub9ce\uc740 \uac74\uac15\ubcf4\ud5d8\ub8cc\ub97c \ubd80\ub2f4\ud558\uace0, \uc77c\ubd80 \uace0\uc18c\ub4dd\uce35\uc740 \uc18c\ub4dd\uc5d0 \ube44\ud574 \uc801\uac8c \ubd80\ub2f4\ud558\ub294 \uc148\uc774\ub2e4. \uc774 \uac19\uc740 \ud3d0\ud574\ub97c \ubc29\uc9c0\ud558\uae30 \uc704\ud574 \uac74\uac15\ubcf4\ud5d8\ub8cc \ubd80\uacfc\uae30\uc900\uc744 \uc18c\ub4dd \uae30\uc900\uc73c\ub85c \ub2e8\uc77c\ud654\ud558\uace0\uc790 \ud558\ub294 \uac83\uc774\ub2e4.", "sentence_answer": "\uc5f0\uac04 7,200\ub9cc\uc6d0 \uc744 \ub118\uc9c0 \uc54a\uc73c\uba74 \uac74\uac15\ubcf4\ud5d8\ub8cc\uac00 \ubd80\uacfc\ub418\uc9c0 \uc54a\uc558\ub2e4.", "paragraph_id": "6589598-0-1", "paragraph_question": "question: \uc784\uae08\uc18c\ub4dd \uc678 \ubcf4\ud5d8\ub8cc\uac00 \ubd80\uacfc\ub418\uc9c0 \uc54a\ub294 \uc885\ud569\uc18c\ub4dd\uc561\uc740?, context: \ud604\ud589 \uac74\uac15\ubcf4\ud5d8\ub8cc \ubd80\uacfc\uccb4\uacc4\ub294 \uc9c1\uc7a5\uac00\uc785\uc790\uc640 \uc9c0\uc5ed\uac00\uc785\uc790 \uac04 \uac74\uac15\ubcf4\ud5d8\ub8cc \uc0b0\uc815\ubc29\uc2dd\uc5d0 \ucc28\uc774\ub97c \ub454\ub2e4. \ud2b9\ud788 \uc9c0\uc5ed\uac00\uc785\uc790\uc758 \uacbd\uc6b0 \uc18c\ub4dd \uc678\uc5d0\ub3c4 \uc790\ub3d9\ucc28\uc640 \uc7ac\uc0b0\uc5d0\ub3c4 \uac74\uac15\ubcf4\ud5d8\ub8cc\uac00 \ubd80\uacfc\ub428\uc740 \ubb3c\ub860, \uac00\uc871\uc758 \ub098\uc774\uc640 \uc131\ubcc4, \uac00\uad6c\uc6d0\uc218\uc5d0 \ub530\ub77c \ub610 \uac74\uac15\ubcf4\ud5d8\ub8cc\uac00 \ucc28\ub4f1\uc801\uc73c\ub85c \ubd80\uacfc\ub41c\ub2e4. \uadf8\ub7ec\ub098 \uc774\uc640\ub294 \ub2ec\ub9ac \uc9c1\uc7a5\uac00\uc785\uc790\uc758 \uacbd\uc6b0 \uc784\uae08\uc18c\ub4dd\uc5d0 \ub530\ub77c \uac74\uac15\ubcf4\ud5d8\ub8cc\uac00 \ubd80\uacfc\ub418\uace0, \uc784\uae08\uc18c\ub4dd \uc678\uc5d0 \uc885\ud569\uc18c\ub4dd\uc774 \uc5f0\uac04 7,200\ub9cc\uc6d0\uc744 \ub118\uc9c0 \uc54a\uc73c\uba74 \uac74\uac15\ubcf4\ud5d8\ub8cc\uac00 \ubd80\uacfc\ub418\uc9c0 \uc54a\uc558\ub2e4. \ub54c\ubb38\uc5d0 \uc77c\ubd80 \uace0\uc18c\ub4dd\uce35\uc740 \uc774\uc790\uc18c\ub4dd, \ubc30\ub2f9\uc18c\ub4dd, \uc784\ub300\uc18c\ub4dd \ub4f1 \uc801\uc9c0 \uc54a\uc740 \uc885\ud569\uc18c\ub4dd\uc744 \uac00\uc9c0\uace0 \uc788\uc74c\uc5d0\ub3c4 \ubd88\uad6c\ud558\uace0 \uac74\uac15\ubcf4\ud5d8\ub8cc\ub97c \ub0a9\ubd80\ud558\uc9c0 \uc54a\uc558\ub2e4. \uc11c\ubbfc\ub4e4\uc740 \uc18c\ub4dd\uc5d0 \ube44\ud574 \ub9ce\uc740 \uac74\uac15\ubcf4\ud5d8\ub8cc\ub97c \ubd80\ub2f4\ud558\uace0, \uc77c\ubd80 \uace0\uc18c\ub4dd\uce35\uc740 \uc18c\ub4dd\uc5d0 \ube44\ud574 \uc801\uac8c \ubd80\ub2f4\ud558\ub294 \uc148\uc774\ub2e4. \uc774 \uac19\uc740 \ud3d0\ud574\ub97c \ubc29\uc9c0\ud558\uae30 \uc704\ud574 \uac74\uac15\ubcf4\ud5d8\ub8cc \ubd80\uacfc\uae30\uc900\uc744 \uc18c\ub4dd \uae30\uc900\uc73c\ub85c \ub2e8\uc77c\ud654\ud558\uace0\uc790 \ud558\ub294 \uac83\uc774\ub2e4."} {"question": "\ub3d9\uacc4\uc62c\ub9bc\ud53d \ub0a8\uc790 \ucd5c\uc5f0\uc18c \uae08\uba54\ub2ec\ub9ac\uc2a4\ud2b8\uc778 \ud540\ub780\ub4dc\uc758 \uc2a4\ud0a4\uc810\ud504 \uc120\uc218 \ud1a0\ub2c8 \ub2c8\uc5d0\ubbf8\ub128\uc774 \uae08\uba54\ub2ec\uc744 \ud68d\ub4dd\ud55c \ub098\uc774\ub294?", "paragraph": "1992\ub144 \ub3d9\uacc4 \uc62c\ub9bc\ud53d\uc740 \ud558\uacc4 \uc62c\ub9bc\ud53d\uacfc \uac19\uc740 \ud574\uc5d0 \uce58\ub7ec\uc9c4 \ub9c8\uc9c0\ub9c9 \ub300\ud68c\uc600\ub2e4. \uc774\ubc88\uc5d0\ub294 \ud504\ub791\uc2a4 \uc0ac\ubd80\uc544\uc758 \uc911\uc2ec\uc9c0 \uc54c\ubca0\ub974\ube4c\uc5d0\uc11c \uc5f4\ub838\uc9c0\ub9cc, \uc815\uc791 \uc54c\ubca0\ub974\ube4c \uc2dc\ub0b4\uc5d0\uc11c\ub294 18\uac1c \uacbd\uae30\ubc16\uc5d0 \uc9c4\ud589\ub418\uc9c0 \uc54a\uc558\ub2e4. \ub098\uba38\uc9c0 \uacbd\uae30\ub4e4\uc740 \uc0ac\ubd80\uc544 \uc804\uc5ed\uc5d0 \uac78\uccd0 \ubd84\uc0b0 \uac1c\ucd5c\ub418\uc5c8\ub2e4. \ud55c\ud3b8 \uacf5\uc0b0\uad8c \ubd95\uad34\uc640 \ubca0\ub97c\ub9b0 \uc7a5\ubcbd\uc758 \ubd95\uad34 \ub4f1\uc73c\ub85c \uc778\ud55c \uc815\uce58\uc0c1\ud669 \uae09\ubcc0\uc73c\ub85c \uc62c\ub9bc\ud53d \uc120\uc218\ub2e8\uc5d0\ub3c4 \uc601\ud5a5\uc774 \ub098\ud0c0\ub0ac\ub294\ub370, \ub3c5\uc77c\uc758 \uacbd\uc6b0 1964\ub144 \ub300\ud68c \uc774\ud6c4 \ucc98\uc74c\uc73c\ub85c \uc11c\ub3d9\ub3c5\uc774 \ud55c \ud300\uc774 \ub418\uc5b4 \ucd9c\uc804\ud558\uac8c \ub418\uc5c8\uc73c\uba70 \uad6c \uc720\uace0\uc2ac\ub77c\ube44\uc544\uc5d0 \uc18d\ud588\ub358 \ud06c\ub85c\uc544\ud2f0\uc544\uc640 \uc2ac\ub85c\ubca0\ub2c8\uc544\ub294 \ub3c5\ub9bd\uad6d\uc73c\ub85c\uc11c \uc62c\ub9bc\ud53d \ubb34\ub300\uc5d0 \uccab \uc9c4\ucd9c\ud558\uc600\ub2e4. \uad6c\uc18c\ub828 \uc9c4\uc601 \uad6d\uac00\ub4e4\uc740 \ub300\ubd80\ubd84 \uc5f0\ud569 \uc120\uc218\ub2e8\uc774\ub77c\ub294 \uc774\ub984\uc758 \ub2e8\uc77c\ud300\uc73c\ub85c \ucd9c\uc804\ud558\uc600\uc9c0\ub9cc \ubc1c\ud2b83\uad6d\uc740 2\ucc28 \uc138\uacc4\ub300\uc804 \uc774\ud6c4 \ucc98\uc74c\uc73c\ub85c \uc628\uc804\ud55c \ub3c5\ub9bd\uad6d\uc73c\ub85c\uc11c \ucd9c\uc804\ud558\uae30\ub3c4 \ud558\uc600\ub2e4. \ud540\ub780\ub4dc\uc758 \uc2a4\ud0a4\uc810\ud504 \uc120\uc218 \ud1a0\ub2c8 \ub2c8\uc5d0\ubbf8\ub128\uc740 \ubd88\uacfc 16\uc0b4\uc758 \ub098\uc774\ub85c \uae08\uba54\ub2ec\uc744 \ud68d\ub4dd\ud574 \ub3d9\uacc4\uc62c\ub9bc\ud53d \ub0a8\uc790 \uae08\uba54\ub2ec\ub9ac\uc2a4\ud2b8 \uc911 \ucd5c\uc5f0\uc18c\ub77c\ub294 \uc601\uc608\ub97c \uc548\uac8c \ub418\uc5c8\uace0, \ub274\uc9c8\ub79c\ub4dc\uc758 \uc564\ub9ac\uc2a4 \ucf54\ubc84\uac70\ub294 \uc54c\ud30c\uc778\uc2a4\ud0a4 \uc5ec\uc790 \ud68c\uc804 \uc885\ubaa9\uc5d0\uc11c \uc740\uba54\ub2ec\uc744 \ud68d\ub4dd\ud574 \ub0a8\ubc18\uad6c \ucd9c\uc2e0\uc73c\ub85c\ub294 \ucc98\uc74c\uc73c\ub85c \ub3d9\uacc4\uc62c\ub9bc\ud53d \uba54\ub2ec\uc744 \ud68d\ub4dd\ud558\uac8c \ub418\uc5c8\ub2e4. \ub610\ud55c \ub300\ud55c\ubbfc\uad6d\uc758 \uae40\uc724\ub9cc\uc774 \uc2a4\ud53c\ub4dc\uc2a4\ucf00\uc774\ud305\uc5d0\uc11c \uc740\uba54\ub2ec\uc744 \ub530\ub0c8\uace0, \ub4a4\uc774\uc5b4 \uc1fc\ud2b8\ud2b8\ub799\uc758 \uae40\uae30\ud6c8\uc774 \uae08\uba54\ub2ec\uc744 \ucc28\uc9c0\ud558\uc5ec \ub3d9\uacc4\uc62c\ub9bc\ud53d \uccab \uba54\ub2ec\uacfc \uccab \uae08\uba54\ub2ec\uc744 \ub3d9\uc2dc\uc5d0 \ub2ec\uc131\ud55c \ub300\ud68c\uac00 \ub418\uc5c8\ub2e4.", "answer": "16\uc0b4", "sentence": "\ud540\ub780\ub4dc\uc758 \uc2a4\ud0a4\uc810\ud504 \uc120\uc218 \ud1a0\ub2c8 \ub2c8\uc5d0\ubbf8\ub128\uc740 \ubd88\uacfc 16\uc0b4 \uc758 \ub098\uc774\ub85c \uae08\uba54\ub2ec\uc744 \ud68d\ub4dd\ud574 \ub3d9\uacc4\uc62c\ub9bc\ud53d \ub0a8\uc790 \uae08\uba54\ub2ec\ub9ac\uc2a4\ud2b8 \uc911 \ucd5c\uc5f0\uc18c\ub77c\ub294 \uc601\uc608\ub97c \uc548\uac8c \ub418\uc5c8\uace0, \ub274\uc9c8\ub79c\ub4dc\uc758 \uc564\ub9ac\uc2a4 \ucf54\ubc84\uac70\ub294 \uc54c\ud30c\uc778\uc2a4\ud0a4 \uc5ec\uc790 \ud68c\uc804 \uc885\ubaa9\uc5d0\uc11c \uc740\uba54\ub2ec\uc744 \ud68d\ub4dd\ud574 \ub0a8\ubc18\uad6c \ucd9c\uc2e0\uc73c\ub85c\ub294 \ucc98\uc74c\uc73c\ub85c \ub3d9\uacc4\uc62c\ub9bc\ud53d \uba54\ub2ec\uc744 \ud68d\ub4dd\ud558\uac8c \ub418\uc5c8\ub2e4.", "paragraph_sentence": "1992\ub144 \ub3d9\uacc4 \uc62c\ub9bc\ud53d\uc740 \ud558\uacc4 \uc62c\ub9bc\ud53d\uacfc \uac19\uc740 \ud574\uc5d0 \uce58\ub7ec\uc9c4 \ub9c8\uc9c0\ub9c9 \ub300\ud68c\uc600\ub2e4. \uc774\ubc88\uc5d0\ub294 \ud504\ub791\uc2a4 \uc0ac\ubd80\uc544\uc758 \uc911\uc2ec\uc9c0 \uc54c\ubca0\ub974\ube4c\uc5d0\uc11c \uc5f4\ub838\uc9c0\ub9cc, \uc815\uc791 \uc54c\ubca0\ub974\ube4c \uc2dc\ub0b4\uc5d0\uc11c\ub294 18\uac1c \uacbd\uae30\ubc16\uc5d0 \uc9c4\ud589\ub418\uc9c0 \uc54a\uc558\ub2e4. \ub098\uba38\uc9c0 \uacbd\uae30\ub4e4\uc740 \uc0ac\ubd80\uc544 \uc804\uc5ed\uc5d0 \uac78\uccd0 \ubd84\uc0b0 \uac1c\ucd5c\ub418\uc5c8\ub2e4. \ud55c\ud3b8 \uacf5\uc0b0\uad8c \ubd95\uad34\uc640 \ubca0\ub97c\ub9b0 \uc7a5\ubcbd\uc758 \ubd95\uad34 \ub4f1\uc73c\ub85c \uc778\ud55c \uc815\uce58\uc0c1\ud669 \uae09\ubcc0\uc73c\ub85c \uc62c\ub9bc\ud53d \uc120\uc218\ub2e8\uc5d0\ub3c4 \uc601\ud5a5\uc774 \ub098\ud0c0\ub0ac\ub294\ub370, \ub3c5\uc77c\uc758 \uacbd\uc6b0 1964\ub144 \ub300\ud68c \uc774\ud6c4 \ucc98\uc74c\uc73c\ub85c \uc11c\ub3d9\ub3c5\uc774 \ud55c \ud300\uc774 \ub418\uc5b4 \ucd9c\uc804\ud558\uac8c \ub418\uc5c8\uc73c\uba70 \uad6c \uc720\uace0\uc2ac\ub77c\ube44\uc544\uc5d0 \uc18d\ud588\ub358 \ud06c\ub85c\uc544\ud2f0\uc544\uc640 \uc2ac\ub85c\ubca0\ub2c8\uc544\ub294 \ub3c5\ub9bd\uad6d\uc73c\ub85c\uc11c \uc62c\ub9bc\ud53d \ubb34\ub300\uc5d0 \uccab \uc9c4\ucd9c\ud558\uc600\ub2e4. \uad6c\uc18c\ub828 \uc9c4\uc601 \uad6d\uac00\ub4e4\uc740 \ub300\ubd80\ubd84 \uc5f0\ud569 \uc120\uc218\ub2e8\uc774\ub77c\ub294 \uc774\ub984\uc758 \ub2e8\uc77c\ud300\uc73c\ub85c \ucd9c\uc804\ud558\uc600\uc9c0\ub9cc \ubc1c\ud2b83\uad6d\uc740 2\ucc28 \uc138\uacc4\ub300\uc804 \uc774\ud6c4 \ucc98\uc74c\uc73c\ub85c \uc628\uc804\ud55c \ub3c5\ub9bd\uad6d\uc73c\ub85c\uc11c \ucd9c\uc804\ud558\uae30\ub3c4 \ud558\uc600\ub2e4. \ud540\ub780\ub4dc\uc758 \uc2a4\ud0a4\uc810\ud504 \uc120\uc218 \ud1a0\ub2c8 \ub2c8\uc5d0\ubbf8\ub128\uc740 \ubd88\uacfc 16\uc0b4 \uc758 \ub098\uc774\ub85c \uae08\uba54\ub2ec\uc744 \ud68d\ub4dd\ud574 \ub3d9\uacc4\uc62c\ub9bc\ud53d \ub0a8\uc790 \uae08\uba54\ub2ec\ub9ac\uc2a4\ud2b8 \uc911 \ucd5c\uc5f0\uc18c\ub77c\ub294 \uc601\uc608\ub97c \uc548\uac8c \ub418\uc5c8\uace0, \ub274\uc9c8\ub79c\ub4dc\uc758 \uc564\ub9ac\uc2a4 \ucf54\ubc84\uac70\ub294 \uc54c\ud30c\uc778\uc2a4\ud0a4 \uc5ec\uc790 \ud68c\uc804 \uc885\ubaa9\uc5d0\uc11c \uc740\uba54\ub2ec\uc744 \ud68d\ub4dd\ud574 \ub0a8\ubc18\uad6c \ucd9c\uc2e0\uc73c\ub85c\ub294 \ucc98\uc74c\uc73c\ub85c \ub3d9\uacc4\uc62c\ub9bc\ud53d \uba54\ub2ec\uc744 \ud68d\ub4dd\ud558\uac8c \ub418\uc5c8\ub2e4. \ub610\ud55c \ub300\ud55c\ubbfc\uad6d\uc758 \uae40\uc724\ub9cc\uc774 \uc2a4\ud53c\ub4dc\uc2a4\ucf00\uc774\ud305\uc5d0\uc11c \uc740\uba54\ub2ec\uc744 \ub530\ub0c8\uace0, \ub4a4\uc774\uc5b4 \uc1fc\ud2b8\ud2b8\ub799\uc758 \uae40\uae30\ud6c8\uc774 \uae08\uba54\ub2ec\uc744 \ucc28\uc9c0\ud558\uc5ec \ub3d9\uacc4\uc62c\ub9bc\ud53d \uccab \uba54\ub2ec\uacfc \uccab \uae08\uba54\ub2ec\uc744 \ub3d9\uc2dc\uc5d0 \ub2ec\uc131\ud55c \ub300\ud68c\uac00 \ub418\uc5c8\ub2e4.", "paragraph_answer": "1992\ub144 \ub3d9\uacc4 \uc62c\ub9bc\ud53d\uc740 \ud558\uacc4 \uc62c\ub9bc\ud53d\uacfc \uac19\uc740 \ud574\uc5d0 \uce58\ub7ec\uc9c4 \ub9c8\uc9c0\ub9c9 \ub300\ud68c\uc600\ub2e4. \uc774\ubc88\uc5d0\ub294 \ud504\ub791\uc2a4 \uc0ac\ubd80\uc544\uc758 \uc911\uc2ec\uc9c0 \uc54c\ubca0\ub974\ube4c\uc5d0\uc11c \uc5f4\ub838\uc9c0\ub9cc, \uc815\uc791 \uc54c\ubca0\ub974\ube4c \uc2dc\ub0b4\uc5d0\uc11c\ub294 18\uac1c \uacbd\uae30\ubc16\uc5d0 \uc9c4\ud589\ub418\uc9c0 \uc54a\uc558\ub2e4. \ub098\uba38\uc9c0 \uacbd\uae30\ub4e4\uc740 \uc0ac\ubd80\uc544 \uc804\uc5ed\uc5d0 \uac78\uccd0 \ubd84\uc0b0 \uac1c\ucd5c\ub418\uc5c8\ub2e4. \ud55c\ud3b8 \uacf5\uc0b0\uad8c \ubd95\uad34\uc640 \ubca0\ub97c\ub9b0 \uc7a5\ubcbd\uc758 \ubd95\uad34 \ub4f1\uc73c\ub85c \uc778\ud55c \uc815\uce58\uc0c1\ud669 \uae09\ubcc0\uc73c\ub85c \uc62c\ub9bc\ud53d \uc120\uc218\ub2e8\uc5d0\ub3c4 \uc601\ud5a5\uc774 \ub098\ud0c0\ub0ac\ub294\ub370, \ub3c5\uc77c\uc758 \uacbd\uc6b0 1964\ub144 \ub300\ud68c \uc774\ud6c4 \ucc98\uc74c\uc73c\ub85c \uc11c\ub3d9\ub3c5\uc774 \ud55c \ud300\uc774 \ub418\uc5b4 \ucd9c\uc804\ud558\uac8c \ub418\uc5c8\uc73c\uba70 \uad6c \uc720\uace0\uc2ac\ub77c\ube44\uc544\uc5d0 \uc18d\ud588\ub358 \ud06c\ub85c\uc544\ud2f0\uc544\uc640 \uc2ac\ub85c\ubca0\ub2c8\uc544\ub294 \ub3c5\ub9bd\uad6d\uc73c\ub85c\uc11c \uc62c\ub9bc\ud53d \ubb34\ub300\uc5d0 \uccab \uc9c4\ucd9c\ud558\uc600\ub2e4. \uad6c\uc18c\ub828 \uc9c4\uc601 \uad6d\uac00\ub4e4\uc740 \ub300\ubd80\ubd84 \uc5f0\ud569 \uc120\uc218\ub2e8\uc774\ub77c\ub294 \uc774\ub984\uc758 \ub2e8\uc77c\ud300\uc73c\ub85c \ucd9c\uc804\ud558\uc600\uc9c0\ub9cc \ubc1c\ud2b83\uad6d\uc740 2\ucc28 \uc138\uacc4\ub300\uc804 \uc774\ud6c4 \ucc98\uc74c\uc73c\ub85c \uc628\uc804\ud55c \ub3c5\ub9bd\uad6d\uc73c\ub85c\uc11c \ucd9c\uc804\ud558\uae30\ub3c4 \ud558\uc600\ub2e4. \ud540\ub780\ub4dc\uc758 \uc2a4\ud0a4\uc810\ud504 \uc120\uc218 \ud1a0\ub2c8 \ub2c8\uc5d0\ubbf8\ub128\uc740 \ubd88\uacfc 16\uc0b4 \uc758 \ub098\uc774\ub85c \uae08\uba54\ub2ec\uc744 \ud68d\ub4dd\ud574 \ub3d9\uacc4\uc62c\ub9bc\ud53d \ub0a8\uc790 \uae08\uba54\ub2ec\ub9ac\uc2a4\ud2b8 \uc911 \ucd5c\uc5f0\uc18c\ub77c\ub294 \uc601\uc608\ub97c \uc548\uac8c \ub418\uc5c8\uace0, \ub274\uc9c8\ub79c\ub4dc\uc758 \uc564\ub9ac\uc2a4 \ucf54\ubc84\uac70\ub294 \uc54c\ud30c\uc778\uc2a4\ud0a4 \uc5ec\uc790 \ud68c\uc804 \uc885\ubaa9\uc5d0\uc11c \uc740\uba54\ub2ec\uc744 \ud68d\ub4dd\ud574 \ub0a8\ubc18\uad6c \ucd9c\uc2e0\uc73c\ub85c\ub294 \ucc98\uc74c\uc73c\ub85c \ub3d9\uacc4\uc62c\ub9bc\ud53d \uba54\ub2ec\uc744 \ud68d\ub4dd\ud558\uac8c \ub418\uc5c8\ub2e4. \ub610\ud55c \ub300\ud55c\ubbfc\uad6d\uc758 \uae40\uc724\ub9cc\uc774 \uc2a4\ud53c\ub4dc\uc2a4\ucf00\uc774\ud305\uc5d0\uc11c \uc740\uba54\ub2ec\uc744 \ub530\ub0c8\uace0, \ub4a4\uc774\uc5b4 \uc1fc\ud2b8\ud2b8\ub799\uc758 \uae40\uae30\ud6c8\uc774 \uae08\uba54\ub2ec\uc744 \ucc28\uc9c0\ud558\uc5ec \ub3d9\uacc4\uc62c\ub9bc\ud53d \uccab \uba54\ub2ec\uacfc \uccab \uae08\uba54\ub2ec\uc744 \ub3d9\uc2dc\uc5d0 \ub2ec\uc131\ud55c \ub300\ud68c\uac00 \ub418\uc5c8\ub2e4.", "sentence_answer": "\ud540\ub780\ub4dc\uc758 \uc2a4\ud0a4\uc810\ud504 \uc120\uc218 \ud1a0\ub2c8 \ub2c8\uc5d0\ubbf8\ub128\uc740 \ubd88\uacfc 16\uc0b4 \uc758 \ub098\uc774\ub85c \uae08\uba54\ub2ec\uc744 \ud68d\ub4dd\ud574 \ub3d9\uacc4\uc62c\ub9bc\ud53d \ub0a8\uc790 \uae08\uba54\ub2ec\ub9ac\uc2a4\ud2b8 \uc911 \ucd5c\uc5f0\uc18c\ub77c\ub294 \uc601\uc608\ub97c \uc548\uac8c \ub418\uc5c8\uace0, \ub274\uc9c8\ub79c\ub4dc\uc758 \uc564\ub9ac\uc2a4 \ucf54\ubc84\uac70\ub294 \uc54c\ud30c\uc778\uc2a4\ud0a4 \uc5ec\uc790 \ud68c\uc804 \uc885\ubaa9\uc5d0\uc11c \uc740\uba54\ub2ec\uc744 \ud68d\ub4dd\ud574 \ub0a8\ubc18\uad6c \ucd9c\uc2e0\uc73c\ub85c\ub294 \ucc98\uc74c\uc73c\ub85c \ub3d9\uacc4\uc62c\ub9bc\ud53d \uba54\ub2ec\uc744 \ud68d\ub4dd\ud558\uac8c \ub418\uc5c8\ub2e4.", "paragraph_id": "6047160-11-1", "paragraph_question": "question: \ub3d9\uacc4\uc62c\ub9bc\ud53d \ub0a8\uc790 \ucd5c\uc5f0\uc18c \uae08\uba54\ub2ec\ub9ac\uc2a4\ud2b8\uc778 \ud540\ub780\ub4dc\uc758 \uc2a4\ud0a4\uc810\ud504 \uc120\uc218 \ud1a0\ub2c8 \ub2c8\uc5d0\ubbf8\ub128\uc774 \uae08\uba54\ub2ec\uc744 \ud68d\ub4dd\ud55c \ub098\uc774\ub294?, context: 1992\ub144 \ub3d9\uacc4 \uc62c\ub9bc\ud53d\uc740 \ud558\uacc4 \uc62c\ub9bc\ud53d\uacfc \uac19\uc740 \ud574\uc5d0 \uce58\ub7ec\uc9c4 \ub9c8\uc9c0\ub9c9 \ub300\ud68c\uc600\ub2e4. \uc774\ubc88\uc5d0\ub294 \ud504\ub791\uc2a4 \uc0ac\ubd80\uc544\uc758 \uc911\uc2ec\uc9c0 \uc54c\ubca0\ub974\ube4c\uc5d0\uc11c \uc5f4\ub838\uc9c0\ub9cc, \uc815\uc791 \uc54c\ubca0\ub974\ube4c \uc2dc\ub0b4\uc5d0\uc11c\ub294 18\uac1c \uacbd\uae30\ubc16\uc5d0 \uc9c4\ud589\ub418\uc9c0 \uc54a\uc558\ub2e4. \ub098\uba38\uc9c0 \uacbd\uae30\ub4e4\uc740 \uc0ac\ubd80\uc544 \uc804\uc5ed\uc5d0 \uac78\uccd0 \ubd84\uc0b0 \uac1c\ucd5c\ub418\uc5c8\ub2e4. \ud55c\ud3b8 \uacf5\uc0b0\uad8c \ubd95\uad34\uc640 \ubca0\ub97c\ub9b0 \uc7a5\ubcbd\uc758 \ubd95\uad34 \ub4f1\uc73c\ub85c \uc778\ud55c \uc815\uce58\uc0c1\ud669 \uae09\ubcc0\uc73c\ub85c \uc62c\ub9bc\ud53d \uc120\uc218\ub2e8\uc5d0\ub3c4 \uc601\ud5a5\uc774 \ub098\ud0c0\ub0ac\ub294\ub370, \ub3c5\uc77c\uc758 \uacbd\uc6b0 1964\ub144 \ub300\ud68c \uc774\ud6c4 \ucc98\uc74c\uc73c\ub85c \uc11c\ub3d9\ub3c5\uc774 \ud55c \ud300\uc774 \ub418\uc5b4 \ucd9c\uc804\ud558\uac8c \ub418\uc5c8\uc73c\uba70 \uad6c \uc720\uace0\uc2ac\ub77c\ube44\uc544\uc5d0 \uc18d\ud588\ub358 \ud06c\ub85c\uc544\ud2f0\uc544\uc640 \uc2ac\ub85c\ubca0\ub2c8\uc544\ub294 \ub3c5\ub9bd\uad6d\uc73c\ub85c\uc11c \uc62c\ub9bc\ud53d \ubb34\ub300\uc5d0 \uccab \uc9c4\ucd9c\ud558\uc600\ub2e4. \uad6c\uc18c\ub828 \uc9c4\uc601 \uad6d\uac00\ub4e4\uc740 \ub300\ubd80\ubd84 \uc5f0\ud569 \uc120\uc218\ub2e8\uc774\ub77c\ub294 \uc774\ub984\uc758 \ub2e8\uc77c\ud300\uc73c\ub85c \ucd9c\uc804\ud558\uc600\uc9c0\ub9cc \ubc1c\ud2b83\uad6d\uc740 2\ucc28 \uc138\uacc4\ub300\uc804 \uc774\ud6c4 \ucc98\uc74c\uc73c\ub85c \uc628\uc804\ud55c \ub3c5\ub9bd\uad6d\uc73c\ub85c\uc11c \ucd9c\uc804\ud558\uae30\ub3c4 \ud558\uc600\ub2e4. \ud540\ub780\ub4dc\uc758 \uc2a4\ud0a4\uc810\ud504 \uc120\uc218 \ud1a0\ub2c8 \ub2c8\uc5d0\ubbf8\ub128\uc740 \ubd88\uacfc 16\uc0b4\uc758 \ub098\uc774\ub85c \uae08\uba54\ub2ec\uc744 \ud68d\ub4dd\ud574 \ub3d9\uacc4\uc62c\ub9bc\ud53d \ub0a8\uc790 \uae08\uba54\ub2ec\ub9ac\uc2a4\ud2b8 \uc911 \ucd5c\uc5f0\uc18c\ub77c\ub294 \uc601\uc608\ub97c \uc548\uac8c \ub418\uc5c8\uace0, \ub274\uc9c8\ub79c\ub4dc\uc758 \uc564\ub9ac\uc2a4 \ucf54\ubc84\uac70\ub294 \uc54c\ud30c\uc778\uc2a4\ud0a4 \uc5ec\uc790 \ud68c\uc804 \uc885\ubaa9\uc5d0\uc11c \uc740\uba54\ub2ec\uc744 \ud68d\ub4dd\ud574 \ub0a8\ubc18\uad6c \ucd9c\uc2e0\uc73c\ub85c\ub294 \ucc98\uc74c\uc73c\ub85c \ub3d9\uacc4\uc62c\ub9bc\ud53d \uba54\ub2ec\uc744 \ud68d\ub4dd\ud558\uac8c \ub418\uc5c8\ub2e4. \ub610\ud55c \ub300\ud55c\ubbfc\uad6d\uc758 \uae40\uc724\ub9cc\uc774 \uc2a4\ud53c\ub4dc\uc2a4\ucf00\uc774\ud305\uc5d0\uc11c \uc740\uba54\ub2ec\uc744 \ub530\ub0c8\uace0, \ub4a4\uc774\uc5b4 \uc1fc\ud2b8\ud2b8\ub799\uc758 \uae40\uae30\ud6c8\uc774 \uae08\uba54\ub2ec\uc744 \ucc28\uc9c0\ud558\uc5ec \ub3d9\uacc4\uc62c\ub9bc\ud53d \uccab \uba54\ub2ec\uacfc \uccab \uae08\uba54\ub2ec\uc744 \ub3d9\uc2dc\uc5d0 \ub2ec\uc131\ud55c \ub300\ud68c\uac00 \ub418\uc5c8\ub2e4."} {"question": "\ucd08\uace0\ubc00\ub3c4 \ud30c\uc7a5 \ubd84\ud560 \ub2e4\uc911\ud654\uc5d0\uc11c \uc4f0\ub294 \uac04\uaca9\uc740?", "paragraph": "WDM \uc2dc\uc2a4\ud15c\uc740 \ubc00\uc9d1 \ud30c\uc7a5 \ubd84\ud560 \ub2e4\uc911\ud654(DWDM)\uc640 \uc804\ud1b5\uc801\uc778 \ud30c\uc7a5 \ubd84\ud560 \ub2e4\uc911\ud654\ub85c \ub098\ub258\uc5b4\uc838 \uc788\ub2e4. \uae30\uc874\uc758 WDM \uc2dc\uc2a4\ud15c\uc740 1550nm \ubd80\uadfc\uc758 3\uac1c\uc758 \uc804\uc1a1 \ucc3d\uc744 \ud1b5\ud574 8\uac1c\uc758 \ucc44\ub110\uc744 \uc81c\uacf5\ud588\ub2e4. \uace0\ubc00\ub3c4 \ud30c\uc7a5 \ubd84\ud560 \ub2e4\uc911\ud654 (DWDM) \uac19\uc740 \uc804\uc1a1 \ucc3d\uc744 \uc0ac\uc6a9\ud558\uc9c0\ub9cc \uc880 \ub354 \uc881\uc740 \ucc44\ub110 \uac04\uaca9\uc744 \uc0ac\uc6a9\ud588\ub2e4. \ucc44\ub110 \uad6c\uc131\uc740 \ub2e4\ub974\uc9c0\ub9cc \uc77c\ubc18\uc801\uc778 \uc2dc\uc2a4\ud15c\uc740 100 GHz \uac04\uaca9\uc73c\ub85c 40 \ucc44\ub110, \ub610\ub294 50 GHz \uac04\uaca9\uc73c\ub85c 80 \ucc44\ub110\uc744 \uc0ac\uc6a9\ud558\uac8c \ub41c\ub2e4. \uc77c\ubd80 \uae30\uc220\uc740 12.5 GHz \uac04\uaca9\uc744 \uc0ac\uc6a9\ud558\uae30\ub3c4 \ud55c\ub2e4(\uc774\uac83\uc740 \ucd08\uace0\ubc00\ub3c4 \ud30c\uc7a5 \ubd84\ud560 \ub2e4\uc911\ud654\uc774\ub2e4). \uc774\ub7ec\ud55c \uac04\uaca9\uc740 \uc790\uc720 \uacf5\uac04 \uad11\ud559(\u81ea\u7531\u7a7a\u9593\u5149\u5b78)\uc5d0 \uc758\ud574 \ub2ec\uc131\ub41c \uac83\uc774\ub2e4. \uc774\uc81c L-\uc8fc\ud30c\uc218\ub300\uc758 \ud3ed\uc744 2(n\uc740 \uc790\uc5f0\uc218)\ubc30\ub85c \uc870\uc815\ud568\uc73c\ub85c\uc368 \ucc44\ub110 \ud3ed\uc744 \uc870\uc815\ud558\ub294 \ubc29\ubc95 \ub610\ud55c \ub098\uc654\ub2e4.", "answer": "12.5 GHz", "sentence": "\uc77c\ubd80 \uae30\uc220\uc740 12.5 GHz \uac04\uaca9\uc744 \uc0ac\uc6a9\ud558\uae30\ub3c4 \ud55c\ub2e4(\uc774\uac83\uc740 \ucd08\uace0\ubc00\ub3c4 \ud30c\uc7a5 \ubd84\ud560 \ub2e4\uc911\ud654\uc774\ub2e4).", "paragraph_sentence": "WDM \uc2dc\uc2a4\ud15c\uc740 \ubc00\uc9d1 \ud30c\uc7a5 \ubd84\ud560 \ub2e4\uc911\ud654(DWDM)\uc640 \uc804\ud1b5\uc801\uc778 \ud30c\uc7a5 \ubd84\ud560 \ub2e4\uc911\ud654\ub85c \ub098\ub258\uc5b4\uc838 \uc788\ub2e4. \uae30\uc874\uc758 WDM \uc2dc\uc2a4\ud15c\uc740 1550nm \ubd80\uadfc\uc758 3\uac1c\uc758 \uc804\uc1a1 \ucc3d\uc744 \ud1b5\ud574 8\uac1c\uc758 \ucc44\ub110\uc744 \uc81c\uacf5\ud588\ub2e4. \uace0\ubc00\ub3c4 \ud30c\uc7a5 \ubd84\ud560 \ub2e4\uc911\ud654 (DWDM) \uac19\uc740 \uc804\uc1a1 \ucc3d\uc744 \uc0ac\uc6a9\ud558\uc9c0\ub9cc \uc880 \ub354 \uc881\uc740 \ucc44\ub110 \uac04\uaca9\uc744 \uc0ac\uc6a9\ud588\ub2e4. \ucc44\ub110 \uad6c\uc131\uc740 \ub2e4\ub974\uc9c0\ub9cc \uc77c\ubc18\uc801\uc778 \uc2dc\uc2a4\ud15c\uc740 100 GHz \uac04\uaca9\uc73c\ub85c 40 \ucc44\ub110, \ub610\ub294 50 GHz \uac04\uaca9\uc73c\ub85c 80 \ucc44\ub110\uc744 \uc0ac\uc6a9\ud558\uac8c \ub41c\ub2e4. \uc77c\ubd80 \uae30\uc220\uc740 12.5 GHz \uac04\uaca9\uc744 \uc0ac\uc6a9\ud558\uae30\ub3c4 \ud55c\ub2e4(\uc774\uac83\uc740 \ucd08\uace0\ubc00\ub3c4 \ud30c\uc7a5 \ubd84\ud560 \ub2e4\uc911\ud654\uc774\ub2e4). \uc774\ub7ec\ud55c \uac04\uaca9\uc740 \uc790\uc720 \uacf5\uac04 \uad11\ud559(\u81ea\u7531\u7a7a\u9593\u5149\u5b78)\uc5d0 \uc758\ud574 \ub2ec\uc131\ub41c \uac83\uc774\ub2e4. \uc774\uc81c L-\uc8fc\ud30c\uc218\ub300\uc758 \ud3ed\uc744 2(n\uc740 \uc790\uc5f0\uc218)\ubc30\ub85c \uc870\uc815\ud568\uc73c\ub85c\uc368 \ucc44\ub110 \ud3ed\uc744 \uc870\uc815\ud558\ub294 \ubc29\ubc95 \ub610\ud55c \ub098\uc654\ub2e4.", "paragraph_answer": "WDM \uc2dc\uc2a4\ud15c\uc740 \ubc00\uc9d1 \ud30c\uc7a5 \ubd84\ud560 \ub2e4\uc911\ud654(DWDM)\uc640 \uc804\ud1b5\uc801\uc778 \ud30c\uc7a5 \ubd84\ud560 \ub2e4\uc911\ud654\ub85c \ub098\ub258\uc5b4\uc838 \uc788\ub2e4. \uae30\uc874\uc758 WDM \uc2dc\uc2a4\ud15c\uc740 1550nm \ubd80\uadfc\uc758 3\uac1c\uc758 \uc804\uc1a1 \ucc3d\uc744 \ud1b5\ud574 8\uac1c\uc758 \ucc44\ub110\uc744 \uc81c\uacf5\ud588\ub2e4. \uace0\ubc00\ub3c4 \ud30c\uc7a5 \ubd84\ud560 \ub2e4\uc911\ud654 (DWDM) \uac19\uc740 \uc804\uc1a1 \ucc3d\uc744 \uc0ac\uc6a9\ud558\uc9c0\ub9cc \uc880 \ub354 \uc881\uc740 \ucc44\ub110 \uac04\uaca9\uc744 \uc0ac\uc6a9\ud588\ub2e4. \ucc44\ub110 \uad6c\uc131\uc740 \ub2e4\ub974\uc9c0\ub9cc \uc77c\ubc18\uc801\uc778 \uc2dc\uc2a4\ud15c\uc740 100 GHz \uac04\uaca9\uc73c\ub85c 40 \ucc44\ub110, \ub610\ub294 50 GHz \uac04\uaca9\uc73c\ub85c 80 \ucc44\ub110\uc744 \uc0ac\uc6a9\ud558\uac8c \ub41c\ub2e4. \uc77c\ubd80 \uae30\uc220\uc740 12.5 GHz \uac04\uaca9\uc744 \uc0ac\uc6a9\ud558\uae30\ub3c4 \ud55c\ub2e4(\uc774\uac83\uc740 \ucd08\uace0\ubc00\ub3c4 \ud30c\uc7a5 \ubd84\ud560 \ub2e4\uc911\ud654\uc774\ub2e4). \uc774\ub7ec\ud55c \uac04\uaca9\uc740 \uc790\uc720 \uacf5\uac04 \uad11\ud559(\u81ea\u7531\u7a7a\u9593\u5149\u5b78)\uc5d0 \uc758\ud574 \ub2ec\uc131\ub41c \uac83\uc774\ub2e4. \uc774\uc81c L-\uc8fc\ud30c\uc218\ub300\uc758 \ud3ed\uc744 2(n\uc740 \uc790\uc5f0\uc218)\ubc30\ub85c \uc870\uc815\ud568\uc73c\ub85c\uc368 \ucc44\ub110 \ud3ed\uc744 \uc870\uc815\ud558\ub294 \ubc29\ubc95 \ub610\ud55c \ub098\uc654\ub2e4.", "sentence_answer": "\uc77c\ubd80 \uae30\uc220\uc740 12.5 GHz \uac04\uaca9\uc744 \uc0ac\uc6a9\ud558\uae30\ub3c4 \ud55c\ub2e4(\uc774\uac83\uc740 \ucd08\uace0\ubc00\ub3c4 \ud30c\uc7a5 \ubd84\ud560 \ub2e4\uc911\ud654\uc774\ub2e4).", "paragraph_id": "6566531-2-1", "paragraph_question": "question: \ucd08\uace0\ubc00\ub3c4 \ud30c\uc7a5 \ubd84\ud560 \ub2e4\uc911\ud654\uc5d0\uc11c \uc4f0\ub294 \uac04\uaca9\uc740?, context: WDM \uc2dc\uc2a4\ud15c\uc740 \ubc00\uc9d1 \ud30c\uc7a5 \ubd84\ud560 \ub2e4\uc911\ud654(DWDM)\uc640 \uc804\ud1b5\uc801\uc778 \ud30c\uc7a5 \ubd84\ud560 \ub2e4\uc911\ud654\ub85c \ub098\ub258\uc5b4\uc838 \uc788\ub2e4. \uae30\uc874\uc758 WDM \uc2dc\uc2a4\ud15c\uc740 1550nm \ubd80\uadfc\uc758 3\uac1c\uc758 \uc804\uc1a1 \ucc3d\uc744 \ud1b5\ud574 8\uac1c\uc758 \ucc44\ub110\uc744 \uc81c\uacf5\ud588\ub2e4. \uace0\ubc00\ub3c4 \ud30c\uc7a5 \ubd84\ud560 \ub2e4\uc911\ud654 (DWDM) \uac19\uc740 \uc804\uc1a1 \ucc3d\uc744 \uc0ac\uc6a9\ud558\uc9c0\ub9cc \uc880 \ub354 \uc881\uc740 \ucc44\ub110 \uac04\uaca9\uc744 \uc0ac\uc6a9\ud588\ub2e4. \ucc44\ub110 \uad6c\uc131\uc740 \ub2e4\ub974\uc9c0\ub9cc \uc77c\ubc18\uc801\uc778 \uc2dc\uc2a4\ud15c\uc740 100 GHz \uac04\uaca9\uc73c\ub85c 40 \ucc44\ub110, \ub610\ub294 50 GHz \uac04\uaca9\uc73c\ub85c 80 \ucc44\ub110\uc744 \uc0ac\uc6a9\ud558\uac8c \ub41c\ub2e4. \uc77c\ubd80 \uae30\uc220\uc740 12.5 GHz \uac04\uaca9\uc744 \uc0ac\uc6a9\ud558\uae30\ub3c4 \ud55c\ub2e4(\uc774\uac83\uc740 \ucd08\uace0\ubc00\ub3c4 \ud30c\uc7a5 \ubd84\ud560 \ub2e4\uc911\ud654\uc774\ub2e4). \uc774\ub7ec\ud55c \uac04\uaca9\uc740 \uc790\uc720 \uacf5\uac04 \uad11\ud559(\u81ea\u7531\u7a7a\u9593\u5149\u5b78)\uc5d0 \uc758\ud574 \ub2ec\uc131\ub41c \uac83\uc774\ub2e4. \uc774\uc81c L-\uc8fc\ud30c\uc218\ub300\uc758 \ud3ed\uc744 2(n\uc740 \uc790\uc5f0\uc218)\ubc30\ub85c \uc870\uc815\ud568\uc73c\ub85c\uc368 \ucc44\ub110 \ud3ed\uc744 \uc870\uc815\ud558\ub294 \ubc29\ubc95 \ub610\ud55c \ub098\uc654\ub2e4."} {"question": "\ube4c\uc774 \ub274\uc695\uac70\ub9ac\uc5d0\uc11c \ub9cc\ub098 \ub3c8\ub9cc \uc9c0\ubd88\ud588\ub358 \ucc3d\ub140\uc758 \uc774\ub984\uc740?", "paragraph": "\ub2e4\uc74c \ub0a0 \ubc24, \ub9c8\ub9ac\ud654\ub098\uc5d0 \ubc18\ucbe4 \ucde8\ud55c \ub450 \uc0ac\ub78c\uc740 \uc11c\ub85c \uc804\ub0a0\uc5d0 \uac19\uc774 \uc788\ub358 \uc0ac\ub78c\uc774 \ub204\uad6c\uc600\ub0d0\uace0 \ubb3c\uc5b4\ubcf4\uace0 \uadf8 \ucd94\uad81\uc740 \uace7 \ub0a8\uc790\uc640 \uc5ec\uc790\uc5d0 \ubcf8\uc131\uc5d0 \ub300\ud55c \ub17c\uc7c1\uc73c\ub85c \uc774\uc5b4\uc9c4\ub2e4. \uadf8\ub7ec\ub358 \ub3c4\uc911, \uc5d8\ub9ac\uc2a4\ub294 \uc791\ub144 \uc5ec\ub984\uc5d0 \uac14\ub358 \ud734\uac00\uc5d0\uc11c \ubcf4\uc558\ub358 \ud574\uad70 \uc7a5\uad50 \ubd24\ub358 \uc21c\uac04\ubd80\ud130 \uc131\uad00\uacc4\ub97c \ub9fa\uace0 \uc2f6\uc5c8\uc73c\uba70 \uadf8\ub97c \uc704\ud574\uc11c\ub294 \uac00\uc871\uc744 \ub2e4 \ubc84\ub9b4 \uc218 \uc788\uc5c8\uc744 \uac83\uc774\ub77c\uace0 \uace0\ubc31\ud55c\ub2e4. \uadf8 \uc21c\uac04, \ube4c\uc740 \uadf8\uc758 \ud658\uc790\uc774\ub358 \ub8e8 \ub124\uc774\ub2e8\uc2a8\uc758 \ubd80\uace0 \uc18c\uc2dd\uc744 \uc811\ud558\uac8c \ub418\uba70, \uadf8 \uc9d1\uc5d0 \ucc3e\uc544\uac00\uac8c \ub41c\ub2e4. \uadf8 \uc9d1\uc5d0\uc11c \ube4c\uc740 \ub8e8 \ub124\uc774\uc2a8\uc758 \ub538\uc5d0\uac8c \uac11\uc791\uc2a4\ub7ec\uc6b4 \uc0ac\ub791 \uace0\ubc31\uc744 \ub4e3\uac8c \ub418\uba70 \ud669\uae09\ud788 \uadf8 \uc9d1\uc744 \ube60\uc838\ub098\uc628\ub2e4. \ub274\uc695 \uac70\ub9ac\ub97c \uc9c0\ub098 \uc9d1\uc73c\ub85c \ub3cc\uc544\uc624\ub358 \uc911 \ube4c\uc740 \ucc3d\ub140\uc778 \ub3c4\ubbf8\ub178\ub97c \ub9cc\ub098\uac8c \ub418\uc5b4 \ubc24\uc744 \uac19\uc774 \uc9c0\ub0bc \ubed4 \ud558\uc9c0\ub9cc \uc8c4\ucc45\uac10\uc5d0 \uc2dc\ub2ec\ub824 \uacb0\uad6d \ub3c8\ub9cc \uc9c0\ubd88\ud558\uace0 \ub098\uc624\uac8c \ub41c\ub2e4.", "answer": "\ub3c4\ubbf8\ub178", "sentence": "\ub274\uc695 \uac70\ub9ac\ub97c \uc9c0\ub098 \uc9d1\uc73c\ub85c \ub3cc\uc544\uc624\ub358 \uc911 \ube4c\uc740 \ucc3d\ub140\uc778 \ub3c4\ubbf8\ub178 \ub97c \ub9cc\ub098\uac8c \ub418\uc5b4 \ubc24\uc744 \uac19\uc774 \uc9c0\ub0bc \ubed4 \ud558\uc9c0\ub9cc \uc8c4\ucc45\uac10\uc5d0 \uc2dc\ub2ec\ub824 \uacb0\uad6d \ub3c8\ub9cc \uc9c0\ubd88\ud558\uace0 \ub098\uc624\uac8c \ub41c\ub2e4.", "paragraph_sentence": "\ub2e4\uc74c \ub0a0 \ubc24, \ub9c8\ub9ac\ud654\ub098\uc5d0 \ubc18\ucbe4 \ucde8\ud55c \ub450 \uc0ac\ub78c\uc740 \uc11c\ub85c \uc804\ub0a0\uc5d0 \uac19\uc774 \uc788\ub358 \uc0ac\ub78c\uc774 \ub204\uad6c\uc600\ub0d0\uace0 \ubb3c\uc5b4\ubcf4\uace0 \uadf8 \ucd94\uad81\uc740 \uace7 \ub0a8\uc790\uc640 \uc5ec\uc790\uc5d0 \ubcf8\uc131\uc5d0 \ub300\ud55c \ub17c\uc7c1\uc73c\ub85c \uc774\uc5b4\uc9c4\ub2e4. \uadf8\ub7ec\ub358 \ub3c4\uc911, \uc5d8\ub9ac\uc2a4\ub294 \uc791\ub144 \uc5ec\ub984\uc5d0 \uac14\ub358 \ud734\uac00\uc5d0\uc11c \ubcf4\uc558\ub358 \ud574\uad70 \uc7a5\uad50 \ubd24\ub358 \uc21c\uac04\ubd80\ud130 \uc131\uad00\uacc4\ub97c \ub9fa\uace0 \uc2f6\uc5c8\uc73c\uba70 \uadf8\ub97c \uc704\ud574\uc11c\ub294 \uac00\uc871\uc744 \ub2e4 \ubc84\ub9b4 \uc218 \uc788\uc5c8\uc744 \uac83\uc774\ub77c\uace0 \uace0\ubc31\ud55c\ub2e4. \uadf8 \uc21c\uac04, \ube4c\uc740 \uadf8\uc758 \ud658\uc790\uc774\ub358 \ub8e8 \ub124\uc774\ub2e8\uc2a8\uc758 \ubd80\uace0 \uc18c\uc2dd\uc744 \uc811\ud558\uac8c \ub418\uba70, \uadf8 \uc9d1\uc5d0 \ucc3e\uc544\uac00\uac8c \ub41c\ub2e4. \uadf8 \uc9d1\uc5d0\uc11c \ube4c\uc740 \ub8e8 \ub124\uc774\uc2a8\uc758 \ub538\uc5d0\uac8c \uac11\uc791\uc2a4\ub7ec\uc6b4 \uc0ac\ub791 \uace0\ubc31\uc744 \ub4e3\uac8c \ub418\uba70 \ud669\uae09\ud788 \uadf8 \uc9d1\uc744 \ube60\uc838\ub098\uc628\ub2e4. \ub274\uc695 \uac70\ub9ac\ub97c \uc9c0\ub098 \uc9d1\uc73c\ub85c \ub3cc\uc544\uc624\ub358 \uc911 \ube4c\uc740 \ucc3d\ub140\uc778 \ub3c4\ubbf8\ub178 \ub97c \ub9cc\ub098\uac8c \ub418\uc5b4 \ubc24\uc744 \uac19\uc774 \uc9c0\ub0bc \ubed4 \ud558\uc9c0\ub9cc \uc8c4\ucc45\uac10\uc5d0 \uc2dc\ub2ec\ub824 \uacb0\uad6d \ub3c8\ub9cc \uc9c0\ubd88\ud558\uace0 \ub098\uc624\uac8c \ub41c\ub2e4. ", "paragraph_answer": "\ub2e4\uc74c \ub0a0 \ubc24, \ub9c8\ub9ac\ud654\ub098\uc5d0 \ubc18\ucbe4 \ucde8\ud55c \ub450 \uc0ac\ub78c\uc740 \uc11c\ub85c \uc804\ub0a0\uc5d0 \uac19\uc774 \uc788\ub358 \uc0ac\ub78c\uc774 \ub204\uad6c\uc600\ub0d0\uace0 \ubb3c\uc5b4\ubcf4\uace0 \uadf8 \ucd94\uad81\uc740 \uace7 \ub0a8\uc790\uc640 \uc5ec\uc790\uc5d0 \ubcf8\uc131\uc5d0 \ub300\ud55c \ub17c\uc7c1\uc73c\ub85c \uc774\uc5b4\uc9c4\ub2e4. \uadf8\ub7ec\ub358 \ub3c4\uc911, \uc5d8\ub9ac\uc2a4\ub294 \uc791\ub144 \uc5ec\ub984\uc5d0 \uac14\ub358 \ud734\uac00\uc5d0\uc11c \ubcf4\uc558\ub358 \ud574\uad70 \uc7a5\uad50 \ubd24\ub358 \uc21c\uac04\ubd80\ud130 \uc131\uad00\uacc4\ub97c \ub9fa\uace0 \uc2f6\uc5c8\uc73c\uba70 \uadf8\ub97c \uc704\ud574\uc11c\ub294 \uac00\uc871\uc744 \ub2e4 \ubc84\ub9b4 \uc218 \uc788\uc5c8\uc744 \uac83\uc774\ub77c\uace0 \uace0\ubc31\ud55c\ub2e4. \uadf8 \uc21c\uac04, \ube4c\uc740 \uadf8\uc758 \ud658\uc790\uc774\ub358 \ub8e8 \ub124\uc774\ub2e8\uc2a8\uc758 \ubd80\uace0 \uc18c\uc2dd\uc744 \uc811\ud558\uac8c \ub418\uba70, \uadf8 \uc9d1\uc5d0 \ucc3e\uc544\uac00\uac8c \ub41c\ub2e4. \uadf8 \uc9d1\uc5d0\uc11c \ube4c\uc740 \ub8e8 \ub124\uc774\uc2a8\uc758 \ub538\uc5d0\uac8c \uac11\uc791\uc2a4\ub7ec\uc6b4 \uc0ac\ub791 \uace0\ubc31\uc744 \ub4e3\uac8c \ub418\uba70 \ud669\uae09\ud788 \uadf8 \uc9d1\uc744 \ube60\uc838\ub098\uc628\ub2e4. \ub274\uc695 \uac70\ub9ac\ub97c \uc9c0\ub098 \uc9d1\uc73c\ub85c \ub3cc\uc544\uc624\ub358 \uc911 \ube4c\uc740 \ucc3d\ub140\uc778 \ub3c4\ubbf8\ub178 \ub97c \ub9cc\ub098\uac8c \ub418\uc5b4 \ubc24\uc744 \uac19\uc774 \uc9c0\ub0bc \ubed4 \ud558\uc9c0\ub9cc \uc8c4\ucc45\uac10\uc5d0 \uc2dc\ub2ec\ub824 \uacb0\uad6d \ub3c8\ub9cc \uc9c0\ubd88\ud558\uace0 \ub098\uc624\uac8c \ub41c\ub2e4.", "sentence_answer": "\ub274\uc695 \uac70\ub9ac\ub97c \uc9c0\ub098 \uc9d1\uc73c\ub85c \ub3cc\uc544\uc624\ub358 \uc911 \ube4c\uc740 \ucc3d\ub140\uc778 \ub3c4\ubbf8\ub178 \ub97c \ub9cc\ub098\uac8c \ub418\uc5b4 \ubc24\uc744 \uac19\uc774 \uc9c0\ub0bc \ubed4 \ud558\uc9c0\ub9cc \uc8c4\ucc45\uac10\uc5d0 \uc2dc\ub2ec\ub824 \uacb0\uad6d \ub3c8\ub9cc \uc9c0\ubd88\ud558\uace0 \ub098\uc624\uac8c \ub41c\ub2e4.", "paragraph_id": "6538652-0-2", "paragraph_question": "question: \ube4c\uc774 \ub274\uc695\uac70\ub9ac\uc5d0\uc11c \ub9cc\ub098 \ub3c8\ub9cc \uc9c0\ubd88\ud588\ub358 \ucc3d\ub140\uc758 \uc774\ub984\uc740?, context: \ub2e4\uc74c \ub0a0 \ubc24, \ub9c8\ub9ac\ud654\ub098\uc5d0 \ubc18\ucbe4 \ucde8\ud55c \ub450 \uc0ac\ub78c\uc740 \uc11c\ub85c \uc804\ub0a0\uc5d0 \uac19\uc774 \uc788\ub358 \uc0ac\ub78c\uc774 \ub204\uad6c\uc600\ub0d0\uace0 \ubb3c\uc5b4\ubcf4\uace0 \uadf8 \ucd94\uad81\uc740 \uace7 \ub0a8\uc790\uc640 \uc5ec\uc790\uc5d0 \ubcf8\uc131\uc5d0 \ub300\ud55c \ub17c\uc7c1\uc73c\ub85c \uc774\uc5b4\uc9c4\ub2e4. \uadf8\ub7ec\ub358 \ub3c4\uc911, \uc5d8\ub9ac\uc2a4\ub294 \uc791\ub144 \uc5ec\ub984\uc5d0 \uac14\ub358 \ud734\uac00\uc5d0\uc11c \ubcf4\uc558\ub358 \ud574\uad70 \uc7a5\uad50 \ubd24\ub358 \uc21c\uac04\ubd80\ud130 \uc131\uad00\uacc4\ub97c \ub9fa\uace0 \uc2f6\uc5c8\uc73c\uba70 \uadf8\ub97c \uc704\ud574\uc11c\ub294 \uac00\uc871\uc744 \ub2e4 \ubc84\ub9b4 \uc218 \uc788\uc5c8\uc744 \uac83\uc774\ub77c\uace0 \uace0\ubc31\ud55c\ub2e4. \uadf8 \uc21c\uac04, \ube4c\uc740 \uadf8\uc758 \ud658\uc790\uc774\ub358 \ub8e8 \ub124\uc774\ub2e8\uc2a8\uc758 \ubd80\uace0 \uc18c\uc2dd\uc744 \uc811\ud558\uac8c \ub418\uba70, \uadf8 \uc9d1\uc5d0 \ucc3e\uc544\uac00\uac8c \ub41c\ub2e4. \uadf8 \uc9d1\uc5d0\uc11c \ube4c\uc740 \ub8e8 \ub124\uc774\uc2a8\uc758 \ub538\uc5d0\uac8c \uac11\uc791\uc2a4\ub7ec\uc6b4 \uc0ac\ub791 \uace0\ubc31\uc744 \ub4e3\uac8c \ub418\uba70 \ud669\uae09\ud788 \uadf8 \uc9d1\uc744 \ube60\uc838\ub098\uc628\ub2e4. \ub274\uc695 \uac70\ub9ac\ub97c \uc9c0\ub098 \uc9d1\uc73c\ub85c \ub3cc\uc544\uc624\ub358 \uc911 \ube4c\uc740 \ucc3d\ub140\uc778 \ub3c4\ubbf8\ub178\ub97c \ub9cc\ub098\uac8c \ub418\uc5b4 \ubc24\uc744 \uac19\uc774 \uc9c0\ub0bc \ubed4 \ud558\uc9c0\ub9cc \uc8c4\ucc45\uac10\uc5d0 \uc2dc\ub2ec\ub824 \uacb0\uad6d \ub3c8\ub9cc \uc9c0\ubd88\ud558\uace0 \ub098\uc624\uac8c \ub41c\ub2e4."} {"question": "\ub9c8\ub974\ud06c\uc2a4\ub294 \ubb34\uc2a8 \ud559\ud30c\uc600\uc744\uae4c?", "paragraph": "\uc774 \ub17c\uc7c1\uc758 \uc2dc\uc791\uc740 Engels\uc758 \u300aThe Condition of the Working Class in England\u300b\ub77c\ub294 \ucc45\uc73c\ub85c \ubcf4\ub294 \uac83\uc774 \uc77c\ubc18\uc801\uc778\ub370 \uc774 \uc800\uc11c\ub294 \ube45\ud1a0\ub9ac\uc544 \uc2dc\ub300 \ub178\ub3d9\uacc4\uae09\uc758 \uc0dd\ud65c\uc218\uc900\uc5d0 \ub300\ud55c \uc5f0\uad6c\ub85c\uc11c, \uc5e5\uac94\uc2a4\ub294 \uac1c\uc778\uc801\uc778 \uad00\ucc30\uacfc \uc0c1\uc138\ud55c \uc2e4\ud0dc \ubcf4\uace0\ub97c \ud1a0\ub300\ub85c \uc0b0\uc5c5\ud601\uba85 \ud6c4\uae30 \ub178\ub3d9\uacc4\uae09\uc758 \uc0dd\ud65c\uc218\uc900\uc774 \ub9e4\uc6b0 \uc5f4\uc545\ud588\uc74c\uc744 \uc8fc\uc7a5\ud55c\ub2e4. \uc774\ub294 \ub9c8\ub974\ud06c\uc2a4 \ub4f1 \uc0ac\ud68c\uc8fc\uc758\ud559\ud30c\uc758 \uc8fc\uc7a5\uacfc \uacb0\ubd80\ub418\uc5b4 \uc790\ubcf8\uc8fc\uc758 \ubaa8\uc21c\uc5d0 \ub300\ud55c \ube44\ud310\uc73c\ub85c \uc774\uc5b4\uc9c0\uac8c \ub41c\ub2e4. \ub610\ud55c 20\uc138\uae30 \ucd08\uae4c\uc9c0 \uc5e5\uac94\uc2a4\uc758 \ube44\ud310\ub860\uc801 \uc785\uc7a5\uc740 \uc0b0\uc5c5\ud601\uba85\uc5d0 \uc758\ud55c \ubd84\ubc30\uc758 \ubd88\ud3c9\ub4f1 \uc2ec\ud654\ub97c \uc8fc\uc7a5\ud55c \uacbd\uc81c\uc0ac\ud559\uc790 \ud1a0\uc778\ube44 \uadf8\ub9ac\uace0 \uc0b0\uc5c5\ud601\uba85\uae30 \ub178\ub3d9\uacc4\uae09\uc740 \ubd80 \ucd95\uc801\uc758 \uc218\ub2e8\uc774\uc5c8\ub2e4\uace0 \uc8fc\uc7a5\ud55c \uc874 \ud574\uba3c\ub4dc\uc640 \ubc14\ubc14\ub77c \ud574\uba3c\ub4dc \ub4f1\uc758 \uc5f0\uad6c\uc5d0 \uc601\ud5a5\uc744 \uc8fc\uac8c \ub41c\ub2e4. 1926\ub144 \ud074\ub798\ud38c(John Clapham)\uc774 \uc0b0\uc5c5\ud601\uba85\uae30 \uc0dd\ud65c\uc218\uc900\uc5d0 \ub300\ud55c \ub099\uad00\ub860\uc801 \uc785\uc7a5\uc744 \uc0c8\ub86d\uac8c \ubc1c\uad70\ud55c \uc784\uae08 \ud1b5\uacc4\uc640 \uc0dd\uacc4\ube44 \uc9c0\uc218\ub97c \ubcf4\uace0 \uc0b0\uc5c5\ud601\uba85\uae30 \ub3c4\uc2dc\uc640 \ub18d\ucd0c \ub178\ub3d9\uc790\ub4e4\uc758 \uad6c\ub9e4\ub825\uc774 \ud06c\uac8c \uc0c1\uc2b9\ud55c \uac83\uc744 \ubc14\ud0d5\uc73c\ub85c \uc8fc\uc7a5\ud55c\ub2e4. \uadf8\ub7ec\ub098 \uc784\uae08 \ubc0f \ubb3c\uac00\uc790\ub8cc\uc758 \ud55c\uacc4\uc640 \ubb38\uc81c\ub85c \ub17c\ub780\uc744 \uac70\ub4ed\ud558\uac8c \ub41c\ub2e4. 20\uc138\uae30 \uc911\ubc18 \uc774\ud6c4 \ub2e8\uae30\ubcf4\ub2e4\ub294 \uc7a5\uae30\uc801\uc778 \ucd94\uc774\uc5d0 \uc9d1\uc911\ud558\uac8c \ub41c\ub2e4. \uacbd\uc81c\ud559\uc790\ub4e4\uc740 18\uc138\uae30 \uc804\ubc18\uae30 \uc2e4\uc9c8\uc784\uae08\uc740 \uc99d\uac00 \ucd94\uc138\uc5d0 \uc788\uc5c8\uc73c\uba70 1750\ub144 \uc774\ud6c4 1820\ub144\uae4c\uc9c0\ub294 \uc815\uccb4\ud558\uac70\ub098 \ud558\ub77d\ud558\uc600\ub2e4\ub294 \uc0ac\uc2e4\uc5d0 \uc758\uacac\uc77c\uce58\ub97c \ubcf4\uc600\ub2e4. \uadf8\ub9ac\uace0 Lindert and Williamson\uc740 \uc0c8\ub85c\uc6b4 \uc0ac\ub8cc\ub97c \ubc14\ud0d5\uc73c\ub85c 1820~1850\ub144\uc5d0\ub294 \uc2e4\uc9c8\uc784\uae08\uc774 \uc774\uc804 \uc2dc\uae30\uc5d0 \ube44\ud574 2\ubc30 \uc0c1\uc2b9\ud558\uc600\ub2e4\ub294 \uac83\uc744 \ubcf4\uc784\uc73c\ub85c\uc368 \uc801\uc5b4\ub3c4 \uc0b0\uc5c5\ud601\uba85 \ud6c4\uae30\ubd80\ud130 \ubb3c\uc9c8\uc801 \uce21\uba74\uc5d0\uc11c\uc758 \uc0dd\ud65c\uc218\uc900\uc774 \uc0c1\uc2b9\ud558\uc600\ub2e4\uace0 \uc8fc\uc7a5\ud558\uc600\ub2e4. \uadf8\ub7ec\ub098 \uc2e4\uc9c8\uc784\uae08\uc758 \uc0c1\uc2b9\uc774 1820\ub144\uc774 \uc544\ub2cc \uc0b0\uc5c5\ud601\uba85 \ucd08\uae30\ubd80\ud130 \uc774\ub8e8\uc5b4\uc838 \uc654\ub2e4\ub294 \uc8fc\uc7a5 \ub610\ud55c \uc0c8\ub86d\uac8c \uc81c\uae30\ub418\uc5c8\ub294\ub370 Feinstein\uc740 1770~1818\ub144 \ud3c9\uade0 \uc2e4\uc9c8\uc784\uae08\uc740 \uc57d 12.5% \uc0c1\uc2b9\ud558\uc600\uace0, 1818~1848\ub144\uc5d0\ub294 23.1% \uc0c1\uc2b9\ud558\uc600\uc74c\uc744 \ubcf4\uc600\ub2e4. Clark\uc640 Allen\uc740 \ud30c\uc778\uc288\ud0c0\uc778\uc758 \uc784\uae08 \ucd94\uacc4\uac00 \uacfc\ub300 \ucd94\uc815\ub418\uc5c8\uc744 \uac00\ub2a5\uc131\uc744 \uc81c\uae30\ud558\uc600\uc9c0\ub9cc, \uadf8\ub7fc\uc5d0\ub3c4 \ubd88\uad6c\ud558\uace0 \ucd08\ucc3d\uae30 \uacbd\uc81c\uc0ac \ud559\uc790\ub4e4\uc758 \ube44\uad00\ub860\uc801 \uc785\uc7a5\ubcf4\ub2e4\ub294 \ub35c \ube44\uad00\uc801\uc774\ub77c\uace0 \uacb0\ub860\uc9c0\uc5c8\ub2e4.", "answer": "\uc0ac\ud68c\uc8fc\uc758\ud559\ud30c", "sentence": "\ub9c8\ub974\ud06c\uc2a4 \ub4f1 \uc0ac\ud68c\uc8fc\uc758\ud559\ud30c \uc758 \uc8fc\uc7a5\uacfc \uacb0\ubd80\ub418\uc5b4 \uc790\ubcf8\uc8fc\uc758 \ubaa8\uc21c\uc5d0 \ub300\ud55c \ube44\ud310\uc73c\ub85c \uc774\uc5b4\uc9c0\uac8c \ub41c\ub2e4.", "paragraph_sentence": "\uc774 \ub17c\uc7c1\uc758 \uc2dc\uc791\uc740 Engels\uc758 \u300aThe Condition of the Working Class in England\u300b\ub77c\ub294 \ucc45\uc73c\ub85c \ubcf4\ub294 \uac83\uc774 \uc77c\ubc18\uc801\uc778\ub370 \uc774 \uc800\uc11c\ub294 \ube45\ud1a0\ub9ac\uc544 \uc2dc\ub300 \ub178\ub3d9\uacc4\uae09\uc758 \uc0dd\ud65c\uc218\uc900\uc5d0 \ub300\ud55c \uc5f0\uad6c\ub85c\uc11c, \uc5e5\uac94\uc2a4\ub294 \uac1c\uc778\uc801\uc778 \uad00\ucc30\uacfc \uc0c1\uc138\ud55c \uc2e4\ud0dc \ubcf4\uace0\ub97c \ud1a0\ub300\ub85c \uc0b0\uc5c5\ud601\uba85 \ud6c4\uae30 \ub178\ub3d9\uacc4\uae09\uc758 \uc0dd\ud65c\uc218\uc900\uc774 \ub9e4\uc6b0 \uc5f4\uc545\ud588\uc74c\uc744 \uc8fc\uc7a5\ud55c\ub2e4. \uc774\ub294 \ub9c8\ub974\ud06c\uc2a4 \ub4f1 \uc0ac\ud68c\uc8fc\uc758\ud559\ud30c \uc758 \uc8fc\uc7a5\uacfc \uacb0\ubd80\ub418\uc5b4 \uc790\ubcf8\uc8fc\uc758 \ubaa8\uc21c\uc5d0 \ub300\ud55c \ube44\ud310\uc73c\ub85c \uc774\uc5b4\uc9c0\uac8c \ub41c\ub2e4. \ub610\ud55c 20\uc138\uae30 \ucd08\uae4c\uc9c0 \uc5e5\uac94\uc2a4\uc758 \ube44\ud310\ub860\uc801 \uc785\uc7a5\uc740 \uc0b0\uc5c5\ud601\uba85\uc5d0 \uc758\ud55c \ubd84\ubc30\uc758 \ubd88\ud3c9\ub4f1 \uc2ec\ud654\ub97c \uc8fc\uc7a5\ud55c \uacbd\uc81c\uc0ac\ud559\uc790 \ud1a0\uc778\ube44 \uadf8\ub9ac\uace0 \uc0b0\uc5c5\ud601\uba85\uae30 \ub178\ub3d9\uacc4\uae09\uc740 \ubd80 \ucd95\uc801\uc758 \uc218\ub2e8\uc774\uc5c8\ub2e4\uace0 \uc8fc\uc7a5\ud55c \uc874 \ud574\uba3c\ub4dc\uc640 \ubc14\ubc14\ub77c \ud574\uba3c\ub4dc \ub4f1\uc758 \uc5f0\uad6c\uc5d0 \uc601\ud5a5\uc744 \uc8fc\uac8c \ub41c\ub2e4. 1926\ub144 \ud074\ub798\ud38c(John Clapham)\uc774 \uc0b0\uc5c5\ud601\uba85\uae30 \uc0dd\ud65c\uc218\uc900\uc5d0 \ub300\ud55c \ub099\uad00\ub860\uc801 \uc785\uc7a5\uc744 \uc0c8\ub86d\uac8c \ubc1c\uad70\ud55c \uc784\uae08 \ud1b5\uacc4\uc640 \uc0dd\uacc4\ube44 \uc9c0\uc218\ub97c \ubcf4\uace0 \uc0b0\uc5c5\ud601\uba85\uae30 \ub3c4\uc2dc\uc640 \ub18d\ucd0c \ub178\ub3d9\uc790\ub4e4\uc758 \uad6c\ub9e4\ub825\uc774 \ud06c\uac8c \uc0c1\uc2b9\ud55c \uac83\uc744 \ubc14\ud0d5\uc73c\ub85c \uc8fc\uc7a5\ud55c\ub2e4. \uadf8\ub7ec\ub098 \uc784\uae08 \ubc0f \ubb3c\uac00\uc790\ub8cc\uc758 \ud55c\uacc4\uc640 \ubb38\uc81c\ub85c \ub17c\ub780\uc744 \uac70\ub4ed\ud558\uac8c \ub41c\ub2e4. 20\uc138\uae30 \uc911\ubc18 \uc774\ud6c4 \ub2e8\uae30\ubcf4\ub2e4\ub294 \uc7a5\uae30\uc801\uc778 \ucd94\uc774\uc5d0 \uc9d1\uc911\ud558\uac8c \ub41c\ub2e4. \uacbd\uc81c\ud559\uc790\ub4e4\uc740 18\uc138\uae30 \uc804\ubc18\uae30 \uc2e4\uc9c8\uc784\uae08\uc740 \uc99d\uac00 \ucd94\uc138\uc5d0 \uc788\uc5c8\uc73c\uba70 1750\ub144 \uc774\ud6c4 1820\ub144\uae4c\uc9c0\ub294 \uc815\uccb4\ud558\uac70\ub098 \ud558\ub77d\ud558\uc600\ub2e4\ub294 \uc0ac\uc2e4\uc5d0 \uc758\uacac\uc77c\uce58\ub97c \ubcf4\uc600\ub2e4. \uadf8\ub9ac\uace0 Lindert and Williamson\uc740 \uc0c8\ub85c\uc6b4 \uc0ac\ub8cc\ub97c \ubc14\ud0d5\uc73c\ub85c 1820~1850\ub144\uc5d0\ub294 \uc2e4\uc9c8\uc784\uae08\uc774 \uc774\uc804 \uc2dc\uae30\uc5d0 \ube44\ud574 2\ubc30 \uc0c1\uc2b9\ud558\uc600\ub2e4\ub294 \uac83\uc744 \ubcf4\uc784\uc73c\ub85c\uc368 \uc801\uc5b4\ub3c4 \uc0b0\uc5c5\ud601\uba85 \ud6c4\uae30\ubd80\ud130 \ubb3c\uc9c8\uc801 \uce21\uba74\uc5d0\uc11c\uc758 \uc0dd\ud65c\uc218\uc900\uc774 \uc0c1\uc2b9\ud558\uc600\ub2e4\uace0 \uc8fc\uc7a5\ud558\uc600\ub2e4. \uadf8\ub7ec\ub098 \uc2e4\uc9c8\uc784\uae08\uc758 \uc0c1\uc2b9\uc774 1820\ub144\uc774 \uc544\ub2cc \uc0b0\uc5c5\ud601\uba85 \ucd08\uae30\ubd80\ud130 \uc774\ub8e8\uc5b4\uc838 \uc654\ub2e4\ub294 \uc8fc\uc7a5 \ub610\ud55c \uc0c8\ub86d\uac8c \uc81c\uae30\ub418\uc5c8\ub294\ub370 Feinstein\uc740 1770~1818\ub144 \ud3c9\uade0 \uc2e4\uc9c8\uc784\uae08\uc740 \uc57d 12.5% \uc0c1\uc2b9\ud558\uc600\uace0, 1818~1848\ub144\uc5d0\ub294 23.1% \uc0c1\uc2b9\ud558\uc600\uc74c\uc744 \ubcf4\uc600\ub2e4. Clark\uc640 Allen\uc740 \ud30c\uc778\uc288\ud0c0\uc778\uc758 \uc784\uae08 \ucd94\uacc4\uac00 \uacfc\ub300 \ucd94\uc815\ub418\uc5c8\uc744 \uac00\ub2a5\uc131\uc744 \uc81c\uae30\ud558\uc600\uc9c0\ub9cc, \uadf8\ub7fc\uc5d0\ub3c4 \ubd88\uad6c\ud558\uace0 \ucd08\ucc3d\uae30 \uacbd\uc81c\uc0ac \ud559\uc790\ub4e4\uc758 \ube44\uad00\ub860\uc801 \uc785\uc7a5\ubcf4\ub2e4\ub294 \ub35c \ube44\uad00\uc801\uc774\ub77c\uace0 \uacb0\ub860\uc9c0\uc5c8\ub2e4.", "paragraph_answer": "\uc774 \ub17c\uc7c1\uc758 \uc2dc\uc791\uc740 Engels\uc758 \u300aThe Condition of the Working Class in England\u300b\ub77c\ub294 \ucc45\uc73c\ub85c \ubcf4\ub294 \uac83\uc774 \uc77c\ubc18\uc801\uc778\ub370 \uc774 \uc800\uc11c\ub294 \ube45\ud1a0\ub9ac\uc544 \uc2dc\ub300 \ub178\ub3d9\uacc4\uae09\uc758 \uc0dd\ud65c\uc218\uc900\uc5d0 \ub300\ud55c \uc5f0\uad6c\ub85c\uc11c, \uc5e5\uac94\uc2a4\ub294 \uac1c\uc778\uc801\uc778 \uad00\ucc30\uacfc \uc0c1\uc138\ud55c \uc2e4\ud0dc \ubcf4\uace0\ub97c \ud1a0\ub300\ub85c \uc0b0\uc5c5\ud601\uba85 \ud6c4\uae30 \ub178\ub3d9\uacc4\uae09\uc758 \uc0dd\ud65c\uc218\uc900\uc774 \ub9e4\uc6b0 \uc5f4\uc545\ud588\uc74c\uc744 \uc8fc\uc7a5\ud55c\ub2e4. \uc774\ub294 \ub9c8\ub974\ud06c\uc2a4 \ub4f1 \uc0ac\ud68c\uc8fc\uc758\ud559\ud30c \uc758 \uc8fc\uc7a5\uacfc \uacb0\ubd80\ub418\uc5b4 \uc790\ubcf8\uc8fc\uc758 \ubaa8\uc21c\uc5d0 \ub300\ud55c \ube44\ud310\uc73c\ub85c \uc774\uc5b4\uc9c0\uac8c \ub41c\ub2e4. \ub610\ud55c 20\uc138\uae30 \ucd08\uae4c\uc9c0 \uc5e5\uac94\uc2a4\uc758 \ube44\ud310\ub860\uc801 \uc785\uc7a5\uc740 \uc0b0\uc5c5\ud601\uba85\uc5d0 \uc758\ud55c \ubd84\ubc30\uc758 \ubd88\ud3c9\ub4f1 \uc2ec\ud654\ub97c \uc8fc\uc7a5\ud55c \uacbd\uc81c\uc0ac\ud559\uc790 \ud1a0\uc778\ube44 \uadf8\ub9ac\uace0 \uc0b0\uc5c5\ud601\uba85\uae30 \ub178\ub3d9\uacc4\uae09\uc740 \ubd80 \ucd95\uc801\uc758 \uc218\ub2e8\uc774\uc5c8\ub2e4\uace0 \uc8fc\uc7a5\ud55c \uc874 \ud574\uba3c\ub4dc\uc640 \ubc14\ubc14\ub77c \ud574\uba3c\ub4dc \ub4f1\uc758 \uc5f0\uad6c\uc5d0 \uc601\ud5a5\uc744 \uc8fc\uac8c \ub41c\ub2e4. 1926\ub144 \ud074\ub798\ud38c(John Clapham)\uc774 \uc0b0\uc5c5\ud601\uba85\uae30 \uc0dd\ud65c\uc218\uc900\uc5d0 \ub300\ud55c \ub099\uad00\ub860\uc801 \uc785\uc7a5\uc744 \uc0c8\ub86d\uac8c \ubc1c\uad70\ud55c \uc784\uae08 \ud1b5\uacc4\uc640 \uc0dd\uacc4\ube44 \uc9c0\uc218\ub97c \ubcf4\uace0 \uc0b0\uc5c5\ud601\uba85\uae30 \ub3c4\uc2dc\uc640 \ub18d\ucd0c \ub178\ub3d9\uc790\ub4e4\uc758 \uad6c\ub9e4\ub825\uc774 \ud06c\uac8c \uc0c1\uc2b9\ud55c \uac83\uc744 \ubc14\ud0d5\uc73c\ub85c \uc8fc\uc7a5\ud55c\ub2e4. \uadf8\ub7ec\ub098 \uc784\uae08 \ubc0f \ubb3c\uac00\uc790\ub8cc\uc758 \ud55c\uacc4\uc640 \ubb38\uc81c\ub85c \ub17c\ub780\uc744 \uac70\ub4ed\ud558\uac8c \ub41c\ub2e4. 20\uc138\uae30 \uc911\ubc18 \uc774\ud6c4 \ub2e8\uae30\ubcf4\ub2e4\ub294 \uc7a5\uae30\uc801\uc778 \ucd94\uc774\uc5d0 \uc9d1\uc911\ud558\uac8c \ub41c\ub2e4. \uacbd\uc81c\ud559\uc790\ub4e4\uc740 18\uc138\uae30 \uc804\ubc18\uae30 \uc2e4\uc9c8\uc784\uae08\uc740 \uc99d\uac00 \ucd94\uc138\uc5d0 \uc788\uc5c8\uc73c\uba70 1750\ub144 \uc774\ud6c4 1820\ub144\uae4c\uc9c0\ub294 \uc815\uccb4\ud558\uac70\ub098 \ud558\ub77d\ud558\uc600\ub2e4\ub294 \uc0ac\uc2e4\uc5d0 \uc758\uacac\uc77c\uce58\ub97c \ubcf4\uc600\ub2e4. \uadf8\ub9ac\uace0 Lindert and Williamson\uc740 \uc0c8\ub85c\uc6b4 \uc0ac\ub8cc\ub97c \ubc14\ud0d5\uc73c\ub85c 1820~1850\ub144\uc5d0\ub294 \uc2e4\uc9c8\uc784\uae08\uc774 \uc774\uc804 \uc2dc\uae30\uc5d0 \ube44\ud574 2\ubc30 \uc0c1\uc2b9\ud558\uc600\ub2e4\ub294 \uac83\uc744 \ubcf4\uc784\uc73c\ub85c\uc368 \uc801\uc5b4\ub3c4 \uc0b0\uc5c5\ud601\uba85 \ud6c4\uae30\ubd80\ud130 \ubb3c\uc9c8\uc801 \uce21\uba74\uc5d0\uc11c\uc758 \uc0dd\ud65c\uc218\uc900\uc774 \uc0c1\uc2b9\ud558\uc600\ub2e4\uace0 \uc8fc\uc7a5\ud558\uc600\ub2e4. \uadf8\ub7ec\ub098 \uc2e4\uc9c8\uc784\uae08\uc758 \uc0c1\uc2b9\uc774 1820\ub144\uc774 \uc544\ub2cc \uc0b0\uc5c5\ud601\uba85 \ucd08\uae30\ubd80\ud130 \uc774\ub8e8\uc5b4\uc838 \uc654\ub2e4\ub294 \uc8fc\uc7a5 \ub610\ud55c \uc0c8\ub86d\uac8c \uc81c\uae30\ub418\uc5c8\ub294\ub370 Feinstein\uc740 1770~1818\ub144 \ud3c9\uade0 \uc2e4\uc9c8\uc784\uae08\uc740 \uc57d 12.5% \uc0c1\uc2b9\ud558\uc600\uace0, 1818~1848\ub144\uc5d0\ub294 23.1% \uc0c1\uc2b9\ud558\uc600\uc74c\uc744 \ubcf4\uc600\ub2e4. Clark\uc640 Allen\uc740 \ud30c\uc778\uc288\ud0c0\uc778\uc758 \uc784\uae08 \ucd94\uacc4\uac00 \uacfc\ub300 \ucd94\uc815\ub418\uc5c8\uc744 \uac00\ub2a5\uc131\uc744 \uc81c\uae30\ud558\uc600\uc9c0\ub9cc, \uadf8\ub7fc\uc5d0\ub3c4 \ubd88\uad6c\ud558\uace0 \ucd08\ucc3d\uae30 \uacbd\uc81c\uc0ac \ud559\uc790\ub4e4\uc758 \ube44\uad00\ub860\uc801 \uc785\uc7a5\ubcf4\ub2e4\ub294 \ub35c \ube44\uad00\uc801\uc774\ub77c\uace0 \uacb0\ub860\uc9c0\uc5c8\ub2e4.", "sentence_answer": "\ub9c8\ub974\ud06c\uc2a4 \ub4f1 \uc0ac\ud68c\uc8fc\uc758\ud559\ud30c \uc758 \uc8fc\uc7a5\uacfc \uacb0\ubd80\ub418\uc5b4 \uc790\ubcf8\uc8fc\uc758 \ubaa8\uc21c\uc5d0 \ub300\ud55c \ube44\ud310\uc73c\ub85c \uc774\uc5b4\uc9c0\uac8c \ub41c\ub2e4.", "paragraph_id": "6657194-3-1", "paragraph_question": "question: \ub9c8\ub974\ud06c\uc2a4\ub294 \ubb34\uc2a8 \ud559\ud30c\uc600\uc744\uae4c?, context: \uc774 \ub17c\uc7c1\uc758 \uc2dc\uc791\uc740 Engels\uc758 \u300aThe Condition of the Working Class in England\u300b\ub77c\ub294 \ucc45\uc73c\ub85c \ubcf4\ub294 \uac83\uc774 \uc77c\ubc18\uc801\uc778\ub370 \uc774 \uc800\uc11c\ub294 \ube45\ud1a0\ub9ac\uc544 \uc2dc\ub300 \ub178\ub3d9\uacc4\uae09\uc758 \uc0dd\ud65c\uc218\uc900\uc5d0 \ub300\ud55c \uc5f0\uad6c\ub85c\uc11c, \uc5e5\uac94\uc2a4\ub294 \uac1c\uc778\uc801\uc778 \uad00\ucc30\uacfc \uc0c1\uc138\ud55c \uc2e4\ud0dc \ubcf4\uace0\ub97c \ud1a0\ub300\ub85c \uc0b0\uc5c5\ud601\uba85 \ud6c4\uae30 \ub178\ub3d9\uacc4\uae09\uc758 \uc0dd\ud65c\uc218\uc900\uc774 \ub9e4\uc6b0 \uc5f4\uc545\ud588\uc74c\uc744 \uc8fc\uc7a5\ud55c\ub2e4. \uc774\ub294 \ub9c8\ub974\ud06c\uc2a4 \ub4f1 \uc0ac\ud68c\uc8fc\uc758\ud559\ud30c\uc758 \uc8fc\uc7a5\uacfc \uacb0\ubd80\ub418\uc5b4 \uc790\ubcf8\uc8fc\uc758 \ubaa8\uc21c\uc5d0 \ub300\ud55c \ube44\ud310\uc73c\ub85c \uc774\uc5b4\uc9c0\uac8c \ub41c\ub2e4. \ub610\ud55c 20\uc138\uae30 \ucd08\uae4c\uc9c0 \uc5e5\uac94\uc2a4\uc758 \ube44\ud310\ub860\uc801 \uc785\uc7a5\uc740 \uc0b0\uc5c5\ud601\uba85\uc5d0 \uc758\ud55c \ubd84\ubc30\uc758 \ubd88\ud3c9\ub4f1 \uc2ec\ud654\ub97c \uc8fc\uc7a5\ud55c \uacbd\uc81c\uc0ac\ud559\uc790 \ud1a0\uc778\ube44 \uadf8\ub9ac\uace0 \uc0b0\uc5c5\ud601\uba85\uae30 \ub178\ub3d9\uacc4\uae09\uc740 \ubd80 \ucd95\uc801\uc758 \uc218\ub2e8\uc774\uc5c8\ub2e4\uace0 \uc8fc\uc7a5\ud55c \uc874 \ud574\uba3c\ub4dc\uc640 \ubc14\ubc14\ub77c \ud574\uba3c\ub4dc \ub4f1\uc758 \uc5f0\uad6c\uc5d0 \uc601\ud5a5\uc744 \uc8fc\uac8c \ub41c\ub2e4. 1926\ub144 \ud074\ub798\ud38c(John Clapham)\uc774 \uc0b0\uc5c5\ud601\uba85\uae30 \uc0dd\ud65c\uc218\uc900\uc5d0 \ub300\ud55c \ub099\uad00\ub860\uc801 \uc785\uc7a5\uc744 \uc0c8\ub86d\uac8c \ubc1c\uad70\ud55c \uc784\uae08 \ud1b5\uacc4\uc640 \uc0dd\uacc4\ube44 \uc9c0\uc218\ub97c \ubcf4\uace0 \uc0b0\uc5c5\ud601\uba85\uae30 \ub3c4\uc2dc\uc640 \ub18d\ucd0c \ub178\ub3d9\uc790\ub4e4\uc758 \uad6c\ub9e4\ub825\uc774 \ud06c\uac8c \uc0c1\uc2b9\ud55c \uac83\uc744 \ubc14\ud0d5\uc73c\ub85c \uc8fc\uc7a5\ud55c\ub2e4. \uadf8\ub7ec\ub098 \uc784\uae08 \ubc0f \ubb3c\uac00\uc790\ub8cc\uc758 \ud55c\uacc4\uc640 \ubb38\uc81c\ub85c \ub17c\ub780\uc744 \uac70\ub4ed\ud558\uac8c \ub41c\ub2e4. 20\uc138\uae30 \uc911\ubc18 \uc774\ud6c4 \ub2e8\uae30\ubcf4\ub2e4\ub294 \uc7a5\uae30\uc801\uc778 \ucd94\uc774\uc5d0 \uc9d1\uc911\ud558\uac8c \ub41c\ub2e4. \uacbd\uc81c\ud559\uc790\ub4e4\uc740 18\uc138\uae30 \uc804\ubc18\uae30 \uc2e4\uc9c8\uc784\uae08\uc740 \uc99d\uac00 \ucd94\uc138\uc5d0 \uc788\uc5c8\uc73c\uba70 1750\ub144 \uc774\ud6c4 1820\ub144\uae4c\uc9c0\ub294 \uc815\uccb4\ud558\uac70\ub098 \ud558\ub77d\ud558\uc600\ub2e4\ub294 \uc0ac\uc2e4\uc5d0 \uc758\uacac\uc77c\uce58\ub97c \ubcf4\uc600\ub2e4. \uadf8\ub9ac\uace0 Lindert and Williamson\uc740 \uc0c8\ub85c\uc6b4 \uc0ac\ub8cc\ub97c \ubc14\ud0d5\uc73c\ub85c 1820~1850\ub144\uc5d0\ub294 \uc2e4\uc9c8\uc784\uae08\uc774 \uc774\uc804 \uc2dc\uae30\uc5d0 \ube44\ud574 2\ubc30 \uc0c1\uc2b9\ud558\uc600\ub2e4\ub294 \uac83\uc744 \ubcf4\uc784\uc73c\ub85c\uc368 \uc801\uc5b4\ub3c4 \uc0b0\uc5c5\ud601\uba85 \ud6c4\uae30\ubd80\ud130 \ubb3c\uc9c8\uc801 \uce21\uba74\uc5d0\uc11c\uc758 \uc0dd\ud65c\uc218\uc900\uc774 \uc0c1\uc2b9\ud558\uc600\ub2e4\uace0 \uc8fc\uc7a5\ud558\uc600\ub2e4. \uadf8\ub7ec\ub098 \uc2e4\uc9c8\uc784\uae08\uc758 \uc0c1\uc2b9\uc774 1820\ub144\uc774 \uc544\ub2cc \uc0b0\uc5c5\ud601\uba85 \ucd08\uae30\ubd80\ud130 \uc774\ub8e8\uc5b4\uc838 \uc654\ub2e4\ub294 \uc8fc\uc7a5 \ub610\ud55c \uc0c8\ub86d\uac8c \uc81c\uae30\ub418\uc5c8\ub294\ub370 Feinstein\uc740 1770~1818\ub144 \ud3c9\uade0 \uc2e4\uc9c8\uc784\uae08\uc740 \uc57d 12.5% \uc0c1\uc2b9\ud558\uc600\uace0, 1818~1848\ub144\uc5d0\ub294 23.1% \uc0c1\uc2b9\ud558\uc600\uc74c\uc744 \ubcf4\uc600\ub2e4. Clark\uc640 Allen\uc740 \ud30c\uc778\uc288\ud0c0\uc778\uc758 \uc784\uae08 \ucd94\uacc4\uac00 \uacfc\ub300 \ucd94\uc815\ub418\uc5c8\uc744 \uac00\ub2a5\uc131\uc744 \uc81c\uae30\ud558\uc600\uc9c0\ub9cc, \uadf8\ub7fc\uc5d0\ub3c4 \ubd88\uad6c\ud558\uace0 \ucd08\ucc3d\uae30 \uacbd\uc81c\uc0ac \ud559\uc790\ub4e4\uc758 \ube44\uad00\ub860\uc801 \uc785\uc7a5\ubcf4\ub2e4\ub294 \ub35c \ube44\uad00\uc801\uc774\ub77c\uace0 \uacb0\ub860\uc9c0\uc5c8\ub2e4."} {"question": "\uc0ac\ub4dc \uccb4\uacc4\uc758 \ub208\uc778 \ub808\uc774\ub2e4\uc5d0\ub294 \uba87 \uac1c\uc758 \uc804\ud30c \uc1a1\uc218\uc2e0\uc7a5\uce58\uac00 \ubd80\ucc29\ub3fc \uc788\ub098?", "paragraph": "\uc0ac\ub4dc \uccb4\uacc4\uc758 \ub208\uc774\ub77c \ud560 \uc218 \uc788\ub294 \ub808\uc774\ub2e4\ub294 2\ub9cc5\ucc9c \uac1c\uc758 \uc804\ud30c \uc1a1\uc218\uc2e0\uc7a5\uce58\uac00 \ubd80\ucc29\ub41c \uace0\uae30\ub2a5 \uc7a5\ube44\ub85c, \uc804\ubc29 120\ub3c4 \uac01\ub3c4\uc5d0\uc11c \ud0d0\uc9c0\uac70\ub9ac\uac00 600~800km\ub85c \ubd81\ud55c \uc804\uc5ed\uc774 \uac10\uc2dc\uad8c\uc5d0 \ub4e4\uc5b4\uc628\ub2e4. \uc694\uaca9 \ubbf8\uc0ac\uc77c\uc744 \ubc1c\uc0ac\ud558\uba74 \uc801\uc678\uc120 \uc13c\uc11c\ub97c \uc774\uc6a9\ud574 \uc801\uc758 \ud0c4\ub3c4\ubbf8\uc0ac\uc77c\uc744 \ucd94\uaca9\ud558\ub2e4\uac00 \uace0\ub3c4 10 km \uc5d0\uc11c 150 km \uc0ac\uc774\uc5d0\uc11c \uc9c1\uc811 \ud0c0\uaca9\ud55c\ub2e4. \ub9cc\uc57d\uc5d0\ub77c\ub3c4 \uc0ac\ub4dc\uac00 \uc774 \uc9c0\uc810\uc5d0\uc11c \uc801\uc758 \ubbf8\uc0ac\uc77c\uc744 \ubabb\uc7a1\uc73c\uba74 \ud328\ud2b8\ub9ac\uc5c7 \ubbf8\uc0ac\uc77c\uc774 \uc800\uace0\ub3c4\uc5d0\uc11c 40 km \uc774\ud558\ub97c \ubc29\uc5b4\ud55c\ub2e4. \uc0ac\ub4dc \ubc30\uce58\ub85c \uace0\uace0\ub3c4\uc640 \uc800\uace0\ub3c4\uc5d0\uc11c \uac01\uac01 \uc694\uaca9 \uae30\ud68c\ub97c \uac16\uac8c \ub418\uba74\uc11c, \ub2e4\uce35 \ubc29\uc5b4\uccb4\uacc4\ub97c \uad6c\ucd95\ud560 \uc218 \uc788\ub2e4. \uc0ac\ub4dc\ub294 \ubbf8\uad70\ud14c\uc2a4\ud2b8\ub97c \ud1b5\ud574 \uc694\uaca9\uc2dc\ud5d8\uc5d0\uc11c \ubaa8\ub450 \uc131\uacf5\ud588\ub2e4. \ub9cc\uc57d\uc5d0 \uc0ac\ub4dc \ubcf4\ub2e4 \uc131\ub2a5\uc774 \ub354 \uc88b\uc740 \ub2e4\ub978 \ubc29\uc5b4\uccb4\uacc4\uac00 \uc874\uc7ac\ud558\uac70\ub098 \uc0ac\ub4dc\ub97c \ub300\uccb4\ud560 \uc218 \uc788\ub294 \ub2e4\ub978 \ub300\uc548\uc774 \uc874\uc7ac\ud55c\ub2e4\uba74 \uc0ac\ub4dc\ub97c \ucca0\ud68c\ud558\uace0 \uc0ac\ub4dc \ubcf4\ub2e4 \uc131\ub2a5\uc774 \ub354 \uc88b\uc740 \ub2e4\ub978 \ubc29\uc5b4\uccb4\uacc4\ub97c \ub3c4\uc785\ud558\uba74 \ub418\uaca0\uc9c0\ub9cc \ud604\uc7ac\ub85c\uc11c\ub294 \uc0ac\ub4dc \ubcf4\ub2e4 \uc131\ub2a5\uc774 \ub354 \uc88b\uc740 \ub2e4\ub978 \ubc29\uc5b4\uccb4\uacc4\ub294 \uc874\uc7ac\ud558\uc9c0 \uc54a\uae30 \ub54c\ubb38\uc5d0 \ud604\uc7ac\ub85c\uc11c\ub294 \uc0ac\ub4dc\uac00 \ucd5c\uc120\uc758 \uc120\ud0dd\uc774\ub2e4.", "answer": "2\ub9cc5\ucc9c \uac1c", "sentence": "\uc0ac\ub4dc \uccb4\uacc4\uc758 \ub208\uc774\ub77c \ud560 \uc218 \uc788\ub294 \ub808\uc774\ub2e4\ub294 2\ub9cc5\ucc9c \uac1c \uc758 \uc804\ud30c \uc1a1\uc218\uc2e0\uc7a5\uce58\uac00 \ubd80\ucc29\ub41c \uace0\uae30\ub2a5 \uc7a5\ube44\ub85c, \uc804\ubc29 120\ub3c4 \uac01\ub3c4\uc5d0\uc11c \ud0d0\uc9c0\uac70\ub9ac\uac00 600~800km\ub85c \ubd81\ud55c \uc804\uc5ed\uc774 \uac10\uc2dc\uad8c\uc5d0 \ub4e4\uc5b4\uc628\ub2e4.", "paragraph_sentence": " \uc0ac\ub4dc \uccb4\uacc4\uc758 \ub208\uc774\ub77c \ud560 \uc218 \uc788\ub294 \ub808\uc774\ub2e4\ub294 2\ub9cc5\ucc9c \uac1c \uc758 \uc804\ud30c \uc1a1\uc218\uc2e0\uc7a5\uce58\uac00 \ubd80\ucc29\ub41c \uace0\uae30\ub2a5 \uc7a5\ube44\ub85c, \uc804\ubc29 120\ub3c4 \uac01\ub3c4\uc5d0\uc11c \ud0d0\uc9c0\uac70\ub9ac\uac00 600~800km\ub85c \ubd81\ud55c \uc804\uc5ed\uc774 \uac10\uc2dc\uad8c\uc5d0 \ub4e4\uc5b4\uc628\ub2e4. \uc694\uaca9 \ubbf8\uc0ac\uc77c\uc744 \ubc1c\uc0ac\ud558\uba74 \uc801\uc678\uc120 \uc13c\uc11c\ub97c \uc774\uc6a9\ud574 \uc801\uc758 \ud0c4\ub3c4\ubbf8\uc0ac\uc77c\uc744 \ucd94\uaca9\ud558\ub2e4\uac00 \uace0\ub3c4 10 km \uc5d0\uc11c 150 km \uc0ac\uc774\uc5d0\uc11c \uc9c1\uc811 \ud0c0\uaca9\ud55c\ub2e4. \ub9cc\uc57d\uc5d0\ub77c\ub3c4 \uc0ac\ub4dc\uac00 \uc774 \uc9c0\uc810\uc5d0\uc11c \uc801\uc758 \ubbf8\uc0ac\uc77c\uc744 \ubabb\uc7a1\uc73c\uba74 \ud328\ud2b8\ub9ac\uc5c7 \ubbf8\uc0ac\uc77c\uc774 \uc800\uace0\ub3c4\uc5d0\uc11c 40 km \uc774\ud558\ub97c \ubc29\uc5b4\ud55c\ub2e4. \uc0ac\ub4dc \ubc30\uce58\ub85c \uace0\uace0\ub3c4\uc640 \uc800\uace0\ub3c4\uc5d0\uc11c \uac01\uac01 \uc694\uaca9 \uae30\ud68c\ub97c \uac16\uac8c \ub418\uba74\uc11c, \ub2e4\uce35 \ubc29\uc5b4\uccb4\uacc4\ub97c \uad6c\ucd95\ud560 \uc218 \uc788\ub2e4. \uc0ac\ub4dc\ub294 \ubbf8\uad70\ud14c\uc2a4\ud2b8\ub97c \ud1b5\ud574 \uc694\uaca9\uc2dc\ud5d8\uc5d0\uc11c \ubaa8\ub450 \uc131\uacf5\ud588\ub2e4. \ub9cc\uc57d\uc5d0 \uc0ac\ub4dc \ubcf4\ub2e4 \uc131\ub2a5\uc774 \ub354 \uc88b\uc740 \ub2e4\ub978 \ubc29\uc5b4\uccb4\uacc4\uac00 \uc874\uc7ac\ud558\uac70\ub098 \uc0ac\ub4dc\ub97c \ub300\uccb4\ud560 \uc218 \uc788\ub294 \ub2e4\ub978 \ub300\uc548\uc774 \uc874\uc7ac\ud55c\ub2e4\uba74 \uc0ac\ub4dc\ub97c \ucca0\ud68c\ud558\uace0 \uc0ac\ub4dc \ubcf4\ub2e4 \uc131\ub2a5\uc774 \ub354 \uc88b\uc740 \ub2e4\ub978 \ubc29\uc5b4\uccb4\uacc4\ub97c \ub3c4\uc785\ud558\uba74 \ub418\uaca0\uc9c0\ub9cc \ud604\uc7ac\ub85c\uc11c\ub294 \uc0ac\ub4dc \ubcf4\ub2e4 \uc131\ub2a5\uc774 \ub354 \uc88b\uc740 \ub2e4\ub978 \ubc29\uc5b4\uccb4\uacc4\ub294 \uc874\uc7ac\ud558\uc9c0 \uc54a\uae30 \ub54c\ubb38\uc5d0 \ud604\uc7ac\ub85c\uc11c\ub294 \uc0ac\ub4dc\uac00 \ucd5c\uc120\uc758 \uc120\ud0dd\uc774\ub2e4.", "paragraph_answer": "\uc0ac\ub4dc \uccb4\uacc4\uc758 \ub208\uc774\ub77c \ud560 \uc218 \uc788\ub294 \ub808\uc774\ub2e4\ub294 2\ub9cc5\ucc9c \uac1c \uc758 \uc804\ud30c \uc1a1\uc218\uc2e0\uc7a5\uce58\uac00 \ubd80\ucc29\ub41c \uace0\uae30\ub2a5 \uc7a5\ube44\ub85c, \uc804\ubc29 120\ub3c4 \uac01\ub3c4\uc5d0\uc11c \ud0d0\uc9c0\uac70\ub9ac\uac00 600~800km\ub85c \ubd81\ud55c \uc804\uc5ed\uc774 \uac10\uc2dc\uad8c\uc5d0 \ub4e4\uc5b4\uc628\ub2e4. \uc694\uaca9 \ubbf8\uc0ac\uc77c\uc744 \ubc1c\uc0ac\ud558\uba74 \uc801\uc678\uc120 \uc13c\uc11c\ub97c \uc774\uc6a9\ud574 \uc801\uc758 \ud0c4\ub3c4\ubbf8\uc0ac\uc77c\uc744 \ucd94\uaca9\ud558\ub2e4\uac00 \uace0\ub3c4 10 km \uc5d0\uc11c 150 km \uc0ac\uc774\uc5d0\uc11c \uc9c1\uc811 \ud0c0\uaca9\ud55c\ub2e4. \ub9cc\uc57d\uc5d0\ub77c\ub3c4 \uc0ac\ub4dc\uac00 \uc774 \uc9c0\uc810\uc5d0\uc11c \uc801\uc758 \ubbf8\uc0ac\uc77c\uc744 \ubabb\uc7a1\uc73c\uba74 \ud328\ud2b8\ub9ac\uc5c7 \ubbf8\uc0ac\uc77c\uc774 \uc800\uace0\ub3c4\uc5d0\uc11c 40 km \uc774\ud558\ub97c \ubc29\uc5b4\ud55c\ub2e4. \uc0ac\ub4dc \ubc30\uce58\ub85c \uace0\uace0\ub3c4\uc640 \uc800\uace0\ub3c4\uc5d0\uc11c \uac01\uac01 \uc694\uaca9 \uae30\ud68c\ub97c \uac16\uac8c \ub418\uba74\uc11c, \ub2e4\uce35 \ubc29\uc5b4\uccb4\uacc4\ub97c \uad6c\ucd95\ud560 \uc218 \uc788\ub2e4. \uc0ac\ub4dc\ub294 \ubbf8\uad70\ud14c\uc2a4\ud2b8\ub97c \ud1b5\ud574 \uc694\uaca9\uc2dc\ud5d8\uc5d0\uc11c \ubaa8\ub450 \uc131\uacf5\ud588\ub2e4. \ub9cc\uc57d\uc5d0 \uc0ac\ub4dc \ubcf4\ub2e4 \uc131\ub2a5\uc774 \ub354 \uc88b\uc740 \ub2e4\ub978 \ubc29\uc5b4\uccb4\uacc4\uac00 \uc874\uc7ac\ud558\uac70\ub098 \uc0ac\ub4dc\ub97c \ub300\uccb4\ud560 \uc218 \uc788\ub294 \ub2e4\ub978 \ub300\uc548\uc774 \uc874\uc7ac\ud55c\ub2e4\uba74 \uc0ac\ub4dc\ub97c \ucca0\ud68c\ud558\uace0 \uc0ac\ub4dc \ubcf4\ub2e4 \uc131\ub2a5\uc774 \ub354 \uc88b\uc740 \ub2e4\ub978 \ubc29\uc5b4\uccb4\uacc4\ub97c \ub3c4\uc785\ud558\uba74 \ub418\uaca0\uc9c0\ub9cc \ud604\uc7ac\ub85c\uc11c\ub294 \uc0ac\ub4dc \ubcf4\ub2e4 \uc131\ub2a5\uc774 \ub354 \uc88b\uc740 \ub2e4\ub978 \ubc29\uc5b4\uccb4\uacc4\ub294 \uc874\uc7ac\ud558\uc9c0 \uc54a\uae30 \ub54c\ubb38\uc5d0 \ud604\uc7ac\ub85c\uc11c\ub294 \uc0ac\ub4dc\uac00 \ucd5c\uc120\uc758 \uc120\ud0dd\uc774\ub2e4.", "sentence_answer": "\uc0ac\ub4dc \uccb4\uacc4\uc758 \ub208\uc774\ub77c \ud560 \uc218 \uc788\ub294 \ub808\uc774\ub2e4\ub294 2\ub9cc5\ucc9c \uac1c \uc758 \uc804\ud30c \uc1a1\uc218\uc2e0\uc7a5\uce58\uac00 \ubd80\ucc29\ub41c \uace0\uae30\ub2a5 \uc7a5\ube44\ub85c, \uc804\ubc29 120\ub3c4 \uac01\ub3c4\uc5d0\uc11c \ud0d0\uc9c0\uac70\ub9ac\uac00 600~800km\ub85c \ubd81\ud55c \uc804\uc5ed\uc774 \uac10\uc2dc\uad8c\uc5d0 \ub4e4\uc5b4\uc628\ub2e4.", "paragraph_id": "6486360-4-0", "paragraph_question": "question: \uc0ac\ub4dc \uccb4\uacc4\uc758 \ub208\uc778 \ub808\uc774\ub2e4\uc5d0\ub294 \uba87 \uac1c\uc758 \uc804\ud30c \uc1a1\uc218\uc2e0\uc7a5\uce58\uac00 \ubd80\ucc29\ub3fc \uc788\ub098?, context: \uc0ac\ub4dc \uccb4\uacc4\uc758 \ub208\uc774\ub77c \ud560 \uc218 \uc788\ub294 \ub808\uc774\ub2e4\ub294 2\ub9cc5\ucc9c \uac1c\uc758 \uc804\ud30c \uc1a1\uc218\uc2e0\uc7a5\uce58\uac00 \ubd80\ucc29\ub41c \uace0\uae30\ub2a5 \uc7a5\ube44\ub85c, \uc804\ubc29 120\ub3c4 \uac01\ub3c4\uc5d0\uc11c \ud0d0\uc9c0\uac70\ub9ac\uac00 600~800km\ub85c \ubd81\ud55c \uc804\uc5ed\uc774 \uac10\uc2dc\uad8c\uc5d0 \ub4e4\uc5b4\uc628\ub2e4. \uc694\uaca9 \ubbf8\uc0ac\uc77c\uc744 \ubc1c\uc0ac\ud558\uba74 \uc801\uc678\uc120 \uc13c\uc11c\ub97c \uc774\uc6a9\ud574 \uc801\uc758 \ud0c4\ub3c4\ubbf8\uc0ac\uc77c\uc744 \ucd94\uaca9\ud558\ub2e4\uac00 \uace0\ub3c4 10 km \uc5d0\uc11c 150 km \uc0ac\uc774\uc5d0\uc11c \uc9c1\uc811 \ud0c0\uaca9\ud55c\ub2e4. \ub9cc\uc57d\uc5d0\ub77c\ub3c4 \uc0ac\ub4dc\uac00 \uc774 \uc9c0\uc810\uc5d0\uc11c \uc801\uc758 \ubbf8\uc0ac\uc77c\uc744 \ubabb\uc7a1\uc73c\uba74 \ud328\ud2b8\ub9ac\uc5c7 \ubbf8\uc0ac\uc77c\uc774 \uc800\uace0\ub3c4\uc5d0\uc11c 40 km \uc774\ud558\ub97c \ubc29\uc5b4\ud55c\ub2e4. \uc0ac\ub4dc \ubc30\uce58\ub85c \uace0\uace0\ub3c4\uc640 \uc800\uace0\ub3c4\uc5d0\uc11c \uac01\uac01 \uc694\uaca9 \uae30\ud68c\ub97c \uac16\uac8c \ub418\uba74\uc11c, \ub2e4\uce35 \ubc29\uc5b4\uccb4\uacc4\ub97c \uad6c\ucd95\ud560 \uc218 \uc788\ub2e4. \uc0ac\ub4dc\ub294 \ubbf8\uad70\ud14c\uc2a4\ud2b8\ub97c \ud1b5\ud574 \uc694\uaca9\uc2dc\ud5d8\uc5d0\uc11c \ubaa8\ub450 \uc131\uacf5\ud588\ub2e4. \ub9cc\uc57d\uc5d0 \uc0ac\ub4dc \ubcf4\ub2e4 \uc131\ub2a5\uc774 \ub354 \uc88b\uc740 \ub2e4\ub978 \ubc29\uc5b4\uccb4\uacc4\uac00 \uc874\uc7ac\ud558\uac70\ub098 \uc0ac\ub4dc\ub97c \ub300\uccb4\ud560 \uc218 \uc788\ub294 \ub2e4\ub978 \ub300\uc548\uc774 \uc874\uc7ac\ud55c\ub2e4\uba74 \uc0ac\ub4dc\ub97c \ucca0\ud68c\ud558\uace0 \uc0ac\ub4dc \ubcf4\ub2e4 \uc131\ub2a5\uc774 \ub354 \uc88b\uc740 \ub2e4\ub978 \ubc29\uc5b4\uccb4\uacc4\ub97c \ub3c4\uc785\ud558\uba74 \ub418\uaca0\uc9c0\ub9cc \ud604\uc7ac\ub85c\uc11c\ub294 \uc0ac\ub4dc \ubcf4\ub2e4 \uc131\ub2a5\uc774 \ub354 \uc88b\uc740 \ub2e4\ub978 \ubc29\uc5b4\uccb4\uacc4\ub294 \uc874\uc7ac\ud558\uc9c0 \uc54a\uae30 \ub54c\ubb38\uc5d0 \ud604\uc7ac\ub85c\uc11c\ub294 \uc0ac\ub4dc\uac00 \ucd5c\uc120\uc758 \uc120\ud0dd\uc774\ub2e4."} {"question": "\uc2f8\uc774\uac00 \ub300\ub9c8\ucd08\ub97c \ud53c\uc6b4 \ud610\uc758\ub85c \uac80\uac70 \ub41c \ub0a0\uc9dc\ub294 \uc5b8\uc81c\uc778\uac00?", "paragraph": "2001\ub144 1\uc6d4, \uc2f8\uc774\ub294 \uadf8\uc758 \uccab \ubc88\uc9f8 \uc815\uaddc \uc568\ubc94 \u300aPSY... From the Psycho World\u300b\uc744 \ubc1c\ub9e4\ud558\uba70 \ub370\ubdd4\ud558\uc600\ub2e4, \ud558\uc9c0\ub9cc \uc774 \uc74c\ubc18\uc758 \uac00\uc0ac \uc18d\uc5d0 \ubd80\uc801\uc808\ud55c \ub0b4\uc6a9\uc774 \ud3ec\ud568\ub418\uc5b4 \uc788\ub2e4\ub294 \uc774\uc720\ub85c \ubc8c\uae08\uc744 \ub0c8\ub2e4. \uc2f8\uc774\ub294 \ud1b5\ud1b5\ud55c \ubab8\ub9e4\uc5d0 \ubc88\uca4d\uc774\ub294 \uc7ac\ud0b7\uc744 \uc785\uace0 \ubb34\ub300\uc5d0 \uc62c\ub790\uace0 \ub2e4\uc18c \uc704\ud611\uc801\uc778 \ub7a9\uacfc \uc5fd\uae30\uc801\uc778 \ucda4\uc73c\ub85c \u201c\uc5fd\uae30 \uac00\uc218\u201d\ub85c \ubd88\ub838\ub2e4. \uadf8\uc758 \ub370\ubdd4\uace1 \u3008\uc0c8\u3009\ub294 \ubc1c\ub9e4\uc640 \ub3d9\uc2dc\uc5d0 \ud070 \uc778\uae30\ub97c \uc5bb\uc73c\uba70 \uc77c\uc57d \uc2a4\ud0c0\ub364\uc5d0 \uc62c\ub790\ub2e4. \uadf8 \ud574 11\uc6d4 15\uc77c, \uc2f8\uc774\ub294 \ub300\ub9c8\ucd08\ub97c \ud53c\uc6b4 \ud610\uc758\ub85c \uac80\uac70\ub418\uc5c8\ub2e4. \uc774\ub85c \uc778\ud574 \ub2f9\uc2dc \uc5f0\ub9d0 \uac00\uc694 \uc2dc\uc0c1\uc2dd\uc5d0\uc11c \uc720\ub825\ud55c \uc2e0\uc778\uc0c1 \ud6c4\ubcf4\uc600\ub358 \uadf8\ub294 \uc218\uc0c1\uc758 \uae30\uc068\uc744 \ub9db\ubcf4\uc9c0 \ubabb\ud558\uace0 \uc138\ud654\uace0\ub4f1\ud559\uad50 2\ub144 \ud6c4\ubc30\uc778 \uc131\uc2dc\uacbd\uc774 \uc2e0\uc778\uc0c1\uc744 \ubc1b\ub294 \ubaa8\uc2b5\uc744 \uc9c0\ucf1c\ubcf4\uc544\uc57c \ud588\ub2e4. \uadf8\uc758 \ub450 \ubc88\uc9f8 \uc815\uaddc \uc568\ubc94 \u300a\uc2f82\u300b\ub294 2002\ub144 1\uc6d4 \ubc1c\ub9e4 \ub418\uc5c8\uace0, \uc774 \uc74c\ubc18 \uc5ed\uc2dc \uc120\uc815\uc801\uc778 \uac00\uc0ac\ub85c \uc778\ud574 \uc5b4\ub9b0\uc774\uc640 \uccad\uc18c\ub144\uc5d0 \ubbf8\uce60 \uc601\ud5a5\uc5d0 \ub300\ud55c \uc2dc\ubbfc \ub2e8\uccb4\uc758 \ud56d\uc758\ub85c 19\uc138 \ubbf8\ub9cc \uccad\ucde8 \uae08\uc9c0 \ud310\uc815\uc744 \ubc1b\uc544 \ud310\ub9e4\uac00 \uae08\uc9c0\ub418\uba74\uc11c \ub2e4\uc2dc \ud55c\ubc88 \ub17c\ub780\uc774 \uc77c\uae30\ub3c4 \ud588\ub2e4. \uac19\uc740 \ud574 9\uc6d4, \uc2f8\uc774\ub294 \uc138 \ubc88\uc9f8 \uc815\uaddc \uc568\ubc94 \u300a3\ub9c8\uc774\u300b\ub97c \ubc1c\ud45c\ud588\ub2e4. \uc774 \uc74c\ubc18\uc758 \ud0c0\uc774\ud2c0 \uace1 \u3008\ucc54\ud53c\uc5b8\u3009\uc740 \ud55c\uc77c \uc6d4\ub4dc\ucef5\uc758 \uad11\uace0 \ud2b9\uc218\ub97c \ub204\ub824 \ud070 \uc131\uacf5\uc744 \uac70\ub480\ub2e4. \uadf8\uc758 \uc74c\uc545\uc744 \ub458\ub7ec\uc2fc \uc0c1\ub2f9\ud55c \ub17c\uc7c1\uc5d0\ub3c4 \ubd88\uad6c\ud558\uace0, \uc2f8\uc774\ub294 \ud55c\uad6d \uc74c\uc545 \uc0b0\uc5c5\uc5d0\uc11c \uc790\uc2e0\ub9cc\uc758 \ub3c5\ud2b9\ud558\uace0 \ub69d\uc2ec\uc788\ub294 \uc74c\uc545\uc73c\ub85c \uc11c\uc6b8\uac00\uc694\ub300\uc0c1\uc5d0\uc11c \uc791\uace1\uac00\uc0c1\uc744 \uc218\uc0c1\ud588\ub2e4.", "answer": "11\uc6d4 15\uc77c", "sentence": "\uadf8 \ud574 11\uc6d4 15\uc77c ,", "paragraph_sentence": "2001\ub144 1\uc6d4, \uc2f8\uc774\ub294 \uadf8\uc758 \uccab \ubc88\uc9f8 \uc815\uaddc \uc568\ubc94 \u300aPSY... From the Psycho World\u300b\uc744 \ubc1c\ub9e4\ud558\uba70 \ub370\ubdd4\ud558\uc600\ub2e4, \ud558\uc9c0\ub9cc \uc774 \uc74c\ubc18\uc758 \uac00\uc0ac \uc18d\uc5d0 \ubd80\uc801\uc808\ud55c \ub0b4\uc6a9\uc774 \ud3ec\ud568\ub418\uc5b4 \uc788\ub2e4\ub294 \uc774\uc720\ub85c \ubc8c\uae08\uc744 \ub0c8\ub2e4. \uc2f8\uc774\ub294 \ud1b5\ud1b5\ud55c \ubab8\ub9e4\uc5d0 \ubc88\uca4d\uc774\ub294 \uc7ac\ud0b7\uc744 \uc785\uace0 \ubb34\ub300\uc5d0 \uc62c\ub790\uace0 \ub2e4\uc18c \uc704\ud611\uc801\uc778 \ub7a9\uacfc \uc5fd\uae30\uc801\uc778 \ucda4\uc73c\ub85c \u201c\uc5fd\uae30 \uac00\uc218\u201d\ub85c \ubd88\ub838\ub2e4. \uadf8\uc758 \ub370\ubdd4\uace1 \u3008\uc0c8\u3009\ub294 \ubc1c\ub9e4\uc640 \ub3d9\uc2dc\uc5d0 \ud070 \uc778\uae30\ub97c \uc5bb\uc73c\uba70 \uc77c\uc57d \uc2a4\ud0c0\ub364\uc5d0 \uc62c\ub790\ub2e4. \uadf8 \ud574 11\uc6d4 15\uc77c , \uc2f8\uc774\ub294 \ub300\ub9c8\ucd08\ub97c \ud53c\uc6b4 \ud610\uc758\ub85c \uac80\uac70\ub418\uc5c8\ub2e4. \uc774\ub85c \uc778\ud574 \ub2f9\uc2dc \uc5f0\ub9d0 \uac00\uc694 \uc2dc\uc0c1\uc2dd\uc5d0\uc11c \uc720\ub825\ud55c \uc2e0\uc778\uc0c1 \ud6c4\ubcf4\uc600\ub358 \uadf8\ub294 \uc218\uc0c1\uc758 \uae30\uc068\uc744 \ub9db\ubcf4\uc9c0 \ubabb\ud558\uace0 \uc138\ud654\uace0\ub4f1\ud559\uad50 2\ub144 \ud6c4\ubc30\uc778 \uc131\uc2dc\uacbd\uc774 \uc2e0\uc778\uc0c1\uc744 \ubc1b\ub294 \ubaa8\uc2b5\uc744 \uc9c0\ucf1c\ubcf4\uc544\uc57c \ud588\ub2e4. \uadf8\uc758 \ub450 \ubc88\uc9f8 \uc815\uaddc \uc568\ubc94 \u300a\uc2f82\u300b\ub294 2002\ub144 1\uc6d4 \ubc1c\ub9e4 \ub418\uc5c8\uace0, \uc774 \uc74c\ubc18 \uc5ed\uc2dc \uc120\uc815\uc801\uc778 \uac00\uc0ac\ub85c \uc778\ud574 \uc5b4\ub9b0\uc774\uc640 \uccad\uc18c\ub144\uc5d0 \ubbf8\uce60 \uc601\ud5a5\uc5d0 \ub300\ud55c \uc2dc\ubbfc \ub2e8\uccb4\uc758 \ud56d\uc758\ub85c 19\uc138 \ubbf8\ub9cc \uccad\ucde8 \uae08\uc9c0 \ud310\uc815\uc744 \ubc1b\uc544 \ud310\ub9e4\uac00 \uae08\uc9c0\ub418\uba74\uc11c \ub2e4\uc2dc \ud55c\ubc88 \ub17c\ub780\uc774 \uc77c\uae30\ub3c4 \ud588\ub2e4. \uac19\uc740 \ud574 9\uc6d4, \uc2f8\uc774\ub294 \uc138 \ubc88\uc9f8 \uc815\uaddc \uc568\ubc94 \u300a3\ub9c8\uc774\u300b\ub97c \ubc1c\ud45c\ud588\ub2e4. \uc774 \uc74c\ubc18\uc758 \ud0c0\uc774\ud2c0 \uace1 \u3008\ucc54\ud53c\uc5b8\u3009\uc740 \ud55c\uc77c \uc6d4\ub4dc\ucef5\uc758 \uad11\uace0 \ud2b9\uc218\ub97c \ub204\ub824 \ud070 \uc131\uacf5\uc744 \uac70\ub480\ub2e4. \uadf8\uc758 \uc74c\uc545\uc744 \ub458\ub7ec\uc2fc \uc0c1\ub2f9\ud55c \ub17c\uc7c1\uc5d0\ub3c4 \ubd88\uad6c\ud558\uace0, \uc2f8\uc774\ub294 \ud55c\uad6d \uc74c\uc545 \uc0b0\uc5c5\uc5d0\uc11c \uc790\uc2e0\ub9cc\uc758 \ub3c5\ud2b9\ud558\uace0 \ub69d\uc2ec\uc788\ub294 \uc74c\uc545\uc73c\ub85c \uc11c\uc6b8\uac00\uc694\ub300\uc0c1\uc5d0\uc11c \uc791\uace1\uac00\uc0c1\uc744 \uc218\uc0c1\ud588\ub2e4.", "paragraph_answer": "2001\ub144 1\uc6d4, \uc2f8\uc774\ub294 \uadf8\uc758 \uccab \ubc88\uc9f8 \uc815\uaddc \uc568\ubc94 \u300aPSY... From the Psycho World\u300b\uc744 \ubc1c\ub9e4\ud558\uba70 \ub370\ubdd4\ud558\uc600\ub2e4, \ud558\uc9c0\ub9cc \uc774 \uc74c\ubc18\uc758 \uac00\uc0ac \uc18d\uc5d0 \ubd80\uc801\uc808\ud55c \ub0b4\uc6a9\uc774 \ud3ec\ud568\ub418\uc5b4 \uc788\ub2e4\ub294 \uc774\uc720\ub85c \ubc8c\uae08\uc744 \ub0c8\ub2e4. \uc2f8\uc774\ub294 \ud1b5\ud1b5\ud55c \ubab8\ub9e4\uc5d0 \ubc88\uca4d\uc774\ub294 \uc7ac\ud0b7\uc744 \uc785\uace0 \ubb34\ub300\uc5d0 \uc62c\ub790\uace0 \ub2e4\uc18c \uc704\ud611\uc801\uc778 \ub7a9\uacfc \uc5fd\uae30\uc801\uc778 \ucda4\uc73c\ub85c \u201c\uc5fd\uae30 \uac00\uc218\u201d\ub85c \ubd88\ub838\ub2e4. \uadf8\uc758 \ub370\ubdd4\uace1 \u3008\uc0c8\u3009\ub294 \ubc1c\ub9e4\uc640 \ub3d9\uc2dc\uc5d0 \ud070 \uc778\uae30\ub97c \uc5bb\uc73c\uba70 \uc77c\uc57d \uc2a4\ud0c0\ub364\uc5d0 \uc62c\ub790\ub2e4. \uadf8 \ud574 11\uc6d4 15\uc77c , \uc2f8\uc774\ub294 \ub300\ub9c8\ucd08\ub97c \ud53c\uc6b4 \ud610\uc758\ub85c \uac80\uac70\ub418\uc5c8\ub2e4. \uc774\ub85c \uc778\ud574 \ub2f9\uc2dc \uc5f0\ub9d0 \uac00\uc694 \uc2dc\uc0c1\uc2dd\uc5d0\uc11c \uc720\ub825\ud55c \uc2e0\uc778\uc0c1 \ud6c4\ubcf4\uc600\ub358 \uadf8\ub294 \uc218\uc0c1\uc758 \uae30\uc068\uc744 \ub9db\ubcf4\uc9c0 \ubabb\ud558\uace0 \uc138\ud654\uace0\ub4f1\ud559\uad50 2\ub144 \ud6c4\ubc30\uc778 \uc131\uc2dc\uacbd\uc774 \uc2e0\uc778\uc0c1\uc744 \ubc1b\ub294 \ubaa8\uc2b5\uc744 \uc9c0\ucf1c\ubcf4\uc544\uc57c \ud588\ub2e4. \uadf8\uc758 \ub450 \ubc88\uc9f8 \uc815\uaddc \uc568\ubc94 \u300a\uc2f82\u300b\ub294 2002\ub144 1\uc6d4 \ubc1c\ub9e4 \ub418\uc5c8\uace0, \uc774 \uc74c\ubc18 \uc5ed\uc2dc \uc120\uc815\uc801\uc778 \uac00\uc0ac\ub85c \uc778\ud574 \uc5b4\ub9b0\uc774\uc640 \uccad\uc18c\ub144\uc5d0 \ubbf8\uce60 \uc601\ud5a5\uc5d0 \ub300\ud55c \uc2dc\ubbfc \ub2e8\uccb4\uc758 \ud56d\uc758\ub85c 19\uc138 \ubbf8\ub9cc \uccad\ucde8 \uae08\uc9c0 \ud310\uc815\uc744 \ubc1b\uc544 \ud310\ub9e4\uac00 \uae08\uc9c0\ub418\uba74\uc11c \ub2e4\uc2dc \ud55c\ubc88 \ub17c\ub780\uc774 \uc77c\uae30\ub3c4 \ud588\ub2e4. \uac19\uc740 \ud574 9\uc6d4, \uc2f8\uc774\ub294 \uc138 \ubc88\uc9f8 \uc815\uaddc \uc568\ubc94 \u300a3\ub9c8\uc774\u300b\ub97c \ubc1c\ud45c\ud588\ub2e4. \uc774 \uc74c\ubc18\uc758 \ud0c0\uc774\ud2c0 \uace1 \u3008\ucc54\ud53c\uc5b8\u3009\uc740 \ud55c\uc77c \uc6d4\ub4dc\ucef5\uc758 \uad11\uace0 \ud2b9\uc218\ub97c \ub204\ub824 \ud070 \uc131\uacf5\uc744 \uac70\ub480\ub2e4. \uadf8\uc758 \uc74c\uc545\uc744 \ub458\ub7ec\uc2fc \uc0c1\ub2f9\ud55c \ub17c\uc7c1\uc5d0\ub3c4 \ubd88\uad6c\ud558\uace0, \uc2f8\uc774\ub294 \ud55c\uad6d \uc74c\uc545 \uc0b0\uc5c5\uc5d0\uc11c \uc790\uc2e0\ub9cc\uc758 \ub3c5\ud2b9\ud558\uace0 \ub69d\uc2ec\uc788\ub294 \uc74c\uc545\uc73c\ub85c \uc11c\uc6b8\uac00\uc694\ub300\uc0c1\uc5d0\uc11c \uc791\uace1\uac00\uc0c1\uc744 \uc218\uc0c1\ud588\ub2e4.", "sentence_answer": "\uadf8 \ud574 11\uc6d4 15\uc77c ,", "paragraph_id": "6547275-1-2", "paragraph_question": "question: \uc2f8\uc774\uac00 \ub300\ub9c8\ucd08\ub97c \ud53c\uc6b4 \ud610\uc758\ub85c \uac80\uac70 \ub41c \ub0a0\uc9dc\ub294 \uc5b8\uc81c\uc778\uac00?, context: 2001\ub144 1\uc6d4, \uc2f8\uc774\ub294 \uadf8\uc758 \uccab \ubc88\uc9f8 \uc815\uaddc \uc568\ubc94 \u300aPSY... From the Psycho World\u300b\uc744 \ubc1c\ub9e4\ud558\uba70 \ub370\ubdd4\ud558\uc600\ub2e4, \ud558\uc9c0\ub9cc \uc774 \uc74c\ubc18\uc758 \uac00\uc0ac \uc18d\uc5d0 \ubd80\uc801\uc808\ud55c \ub0b4\uc6a9\uc774 \ud3ec\ud568\ub418\uc5b4 \uc788\ub2e4\ub294 \uc774\uc720\ub85c \ubc8c\uae08\uc744 \ub0c8\ub2e4. \uc2f8\uc774\ub294 \ud1b5\ud1b5\ud55c \ubab8\ub9e4\uc5d0 \ubc88\uca4d\uc774\ub294 \uc7ac\ud0b7\uc744 \uc785\uace0 \ubb34\ub300\uc5d0 \uc62c\ub790\uace0 \ub2e4\uc18c \uc704\ud611\uc801\uc778 \ub7a9\uacfc \uc5fd\uae30\uc801\uc778 \ucda4\uc73c\ub85c \u201c\uc5fd\uae30 \uac00\uc218\u201d\ub85c \ubd88\ub838\ub2e4. \uadf8\uc758 \ub370\ubdd4\uace1 \u3008\uc0c8\u3009\ub294 \ubc1c\ub9e4\uc640 \ub3d9\uc2dc\uc5d0 \ud070 \uc778\uae30\ub97c \uc5bb\uc73c\uba70 \uc77c\uc57d \uc2a4\ud0c0\ub364\uc5d0 \uc62c\ub790\ub2e4. \uadf8 \ud574 11\uc6d4 15\uc77c, \uc2f8\uc774\ub294 \ub300\ub9c8\ucd08\ub97c \ud53c\uc6b4 \ud610\uc758\ub85c \uac80\uac70\ub418\uc5c8\ub2e4. \uc774\ub85c \uc778\ud574 \ub2f9\uc2dc \uc5f0\ub9d0 \uac00\uc694 \uc2dc\uc0c1\uc2dd\uc5d0\uc11c \uc720\ub825\ud55c \uc2e0\uc778\uc0c1 \ud6c4\ubcf4\uc600\ub358 \uadf8\ub294 \uc218\uc0c1\uc758 \uae30\uc068\uc744 \ub9db\ubcf4\uc9c0 \ubabb\ud558\uace0 \uc138\ud654\uace0\ub4f1\ud559\uad50 2\ub144 \ud6c4\ubc30\uc778 \uc131\uc2dc\uacbd\uc774 \uc2e0\uc778\uc0c1\uc744 \ubc1b\ub294 \ubaa8\uc2b5\uc744 \uc9c0\ucf1c\ubcf4\uc544\uc57c \ud588\ub2e4. \uadf8\uc758 \ub450 \ubc88\uc9f8 \uc815\uaddc \uc568\ubc94 \u300a\uc2f82\u300b\ub294 2002\ub144 1\uc6d4 \ubc1c\ub9e4 \ub418\uc5c8\uace0, \uc774 \uc74c\ubc18 \uc5ed\uc2dc \uc120\uc815\uc801\uc778 \uac00\uc0ac\ub85c \uc778\ud574 \uc5b4\ub9b0\uc774\uc640 \uccad\uc18c\ub144\uc5d0 \ubbf8\uce60 \uc601\ud5a5\uc5d0 \ub300\ud55c \uc2dc\ubbfc \ub2e8\uccb4\uc758 \ud56d\uc758\ub85c 19\uc138 \ubbf8\ub9cc \uccad\ucde8 \uae08\uc9c0 \ud310\uc815\uc744 \ubc1b\uc544 \ud310\ub9e4\uac00 \uae08\uc9c0\ub418\uba74\uc11c \ub2e4\uc2dc \ud55c\ubc88 \ub17c\ub780\uc774 \uc77c\uae30\ub3c4 \ud588\ub2e4. \uac19\uc740 \ud574 9\uc6d4, \uc2f8\uc774\ub294 \uc138 \ubc88\uc9f8 \uc815\uaddc \uc568\ubc94 \u300a3\ub9c8\uc774\u300b\ub97c \ubc1c\ud45c\ud588\ub2e4. \uc774 \uc74c\ubc18\uc758 \ud0c0\uc774\ud2c0 \uace1 \u3008\ucc54\ud53c\uc5b8\u3009\uc740 \ud55c\uc77c \uc6d4\ub4dc\ucef5\uc758 \uad11\uace0 \ud2b9\uc218\ub97c \ub204\ub824 \ud070 \uc131\uacf5\uc744 \uac70\ub480\ub2e4. \uadf8\uc758 \uc74c\uc545\uc744 \ub458\ub7ec\uc2fc \uc0c1\ub2f9\ud55c \ub17c\uc7c1\uc5d0\ub3c4 \ubd88\uad6c\ud558\uace0, \uc2f8\uc774\ub294 \ud55c\uad6d \uc74c\uc545 \uc0b0\uc5c5\uc5d0\uc11c \uc790\uc2e0\ub9cc\uc758 \ub3c5\ud2b9\ud558\uace0 \ub69d\uc2ec\uc788\ub294 \uc74c\uc545\uc73c\ub85c \uc11c\uc6b8\uac00\uc694\ub300\uc0c1\uc5d0\uc11c \uc791\uace1\uac00\uc0c1\uc744 \uc218\uc0c1\ud588\ub2e4."} {"question": "\uc774\uc2ac\ub78c\uc758 \ud669\uae08 \uc2dc\ub300\ub54c\uc758 \uce7c\ub9ac\ud504 \ub9c8\ubb38\uc740 \uc9c0\ud61c\uc758 \uc9d1\uc744 \uc5b4\ub290 \uc9c0\uc5ed\uc5d0 \uc138\uc6e0\ub098?", "paragraph": "\uc774\uc2ac\ub78c\uc758 \ud669\uae08 \uc2dc\ub300\ub294 \uc774\uc2ac\ub78c \ubb38\ud654\uc640 \uc774\uc2ac\ub78c\uc744 \uae30\ucd08\ub85c \ud558\uc5ec \uadf8\ub9ac\uc2a4\u00b7\ub85c\ub9c8\uc758 \uace0\uc804 \ubb38\ud654\ub098, \ud398\ub974\uc2dc\uc544\u00b7\uc778\ub3c4\uc758 \uc5ec\ub7ec \ubb38\ud654\ub97c \ubc1b\uc544\ub4e4\uc5ec \uc131\ub9bd\ub41c \uc77c\ub300 \ucd1d\ud569 \ubb38\ud654\uc774\uba70, \uc911\uc138 \ubc0f \ub974\ub124\uc0c1\uc2a4\uae30\uc758 \uc11c\uad6c \ubb38\ud654\uc5d0 \uacb0\uc815\uc801\uc778 \uc601\ud5a5\uc744 \uc90c\uc73c\ub85c\uc368 \uadf8 \uc5ed\uc0ac\uc801\uc778 \uc758\uc758\ub294 \ub9e4\uc6b0 \ud06c\ub2e4. \ubb34\ud568\ub9c8\ub4dc\uc758 \uc0ac\ud6c4 \u300a\uafb8\ub780\u300b\uc758 \ud574\uc11d\uc744 \ud1b5\ud574 \ubc95\ud559\uc774\ub098 \uc2e0\ud559\uc774 \uc131\ub9bd\ub418\uc5c8\uc9c0\ub9cc, \uc634\ubbf8\uc544\ub4dc \uc655\uc870 \ub54c\ubd80\ud130 \uc678\ub798 \ud559\ubb38\uc758 \uc774\uc2dd\uc774 \uc2dc\uc791\ub418\uace0, \uc544\ubc14\uc2a4 \uc655\uc870 \uc2dc\ub300\uac00 \ub418\uba74\uc11c \uadf8\ub9ac\uc2a4\uc5b4 \ubb38\ud5cc \ubc88\uc5ed\uc774 \uc801\uadf9\uc801\uc73c\ub85c \uc9c4\ud589\ub418\uace0 \ucca0\ud559\u00b7\uc218\ud559\u00b7\uacfc\ud559\uc758 \uc5f0\uad6c\uac00 \ubc1c\ub2ec\ud588\ub2e4. \ud2b9\ud788 \uc81c7\ub300 \uce7c\ub9ac\ud504\uc778 \ub9c8\ubb38(\uc7ac\uc704 813~833)\uc774 \ubc14\uadf8\ub2e4\ub4dc\uc5d0 '\uc9c0\ud61c\uc758 \uc9d1'\uc744 \uc138\uc6b0\uace0, \uadf8\ub9ac\uc2a4\uc5b4 \ubb38\ud5cc\uc758 \uc218\uc9d1 \ubc0f \ubc88\uc5ed\uc744 \uc7a5\ub824\ud558\uac8c \ub418\uc790 \uc808\uc815\uc5d0 \uc774\ub974\ub800\ub2e4. \uc774\ub4e4 \ubb38\ud654 \ud65c\ub3d9\uc5d0 \uc885\uc0ac\ud55c \uc0ac\ub78c\uc740 \uc544\ub78d\uc778\ubcf4\ub2e4\ub3c4 \uc624\ud788\ub824 \ud53c\uc815\ubcf5\ubbfc\uc774\uc5c8\uc73c\uba70, \ud2b9\ud788 \ud398\ub974\uc2dc\uc544\uc778\uc758 \uc5ed\ud560\uc774 \ucef8\ub2e4. \uc774\uc2ac\ub78c \ubb38\ud654\uc758 \uc804\uc131\uae30\ub294 9\uc138\uae30\ubd80\ud130 11\uc138\uae30\uc5d0 \uac78\uccd0\uc11c\uc774\uba70, 12\uc138\uae30\uc5d0\ub294 \uc1e0\ud1f4\ud558\uae30 \uc2dc\uc791\ud588\ub2e4.", "answer": "\ubc14\uadf8\ub2e4\ub4dc", "sentence": "\ud2b9\ud788 \uc81c7\ub300 \uce7c\ub9ac\ud504\uc778 \ub9c8\ubb38(\uc7ac\uc704 813~833)\uc774 \ubc14\uadf8\ub2e4\ub4dc \uc5d0", "paragraph_sentence": "\uc774\uc2ac\ub78c\uc758 \ud669\uae08 \uc2dc\ub300\ub294 \uc774\uc2ac\ub78c \ubb38\ud654\uc640 \uc774\uc2ac\ub78c\uc744 \uae30\ucd08\ub85c \ud558\uc5ec \uadf8\ub9ac\uc2a4\u00b7\ub85c\ub9c8\uc758 \uace0\uc804 \ubb38\ud654\ub098, \ud398\ub974\uc2dc\uc544\u00b7\uc778\ub3c4\uc758 \uc5ec\ub7ec \ubb38\ud654\ub97c \ubc1b\uc544\ub4e4\uc5ec \uc131\ub9bd\ub41c \uc77c\ub300 \ucd1d\ud569 \ubb38\ud654\uc774\uba70, \uc911\uc138 \ubc0f \ub974\ub124\uc0c1\uc2a4\uae30\uc758 \uc11c\uad6c \ubb38\ud654\uc5d0 \uacb0\uc815\uc801\uc778 \uc601\ud5a5\uc744 \uc90c\uc73c\ub85c\uc368 \uadf8 \uc5ed\uc0ac\uc801\uc778 \uc758\uc758\ub294 \ub9e4\uc6b0 \ud06c\ub2e4. \ubb34\ud568\ub9c8\ub4dc\uc758 \uc0ac\ud6c4 \u300a\uafb8\ub780\u300b\uc758 \ud574\uc11d\uc744 \ud1b5\ud574 \ubc95\ud559\uc774\ub098 \uc2e0\ud559\uc774 \uc131\ub9bd\ub418\uc5c8\uc9c0\ub9cc, \uc634\ubbf8\uc544\ub4dc \uc655\uc870 \ub54c\ubd80\ud130 \uc678\ub798 \ud559\ubb38\uc758 \uc774\uc2dd\uc774 \uc2dc\uc791\ub418\uace0, \uc544\ubc14\uc2a4 \uc655\uc870 \uc2dc\ub300\uac00 \ub418\uba74\uc11c \uadf8\ub9ac\uc2a4\uc5b4 \ubb38\ud5cc \ubc88\uc5ed\uc774 \uc801\uadf9\uc801\uc73c\ub85c \uc9c4\ud589\ub418\uace0 \ucca0\ud559\u00b7\uc218\ud559\u00b7\uacfc\ud559\uc758 \uc5f0\uad6c\uac00 \ubc1c\ub2ec\ud588\ub2e4. \ud2b9\ud788 \uc81c7\ub300 \uce7c\ub9ac\ud504\uc778 \ub9c8\ubb38(\uc7ac\uc704 813~833)\uc774 \ubc14\uadf8\ub2e4\ub4dc \uc5d0 '\uc9c0\ud61c\uc758 \uc9d1'\uc744 \uc138\uc6b0\uace0, \uadf8\ub9ac\uc2a4\uc5b4 \ubb38\ud5cc\uc758 \uc218\uc9d1 \ubc0f \ubc88\uc5ed\uc744 \uc7a5\ub824\ud558\uac8c \ub418\uc790 \uc808\uc815\uc5d0 \uc774\ub974\ub800\ub2e4. \uc774\ub4e4 \ubb38\ud654 \ud65c\ub3d9\uc5d0 \uc885\uc0ac\ud55c \uc0ac\ub78c\uc740 \uc544\ub78d\uc778\ubcf4\ub2e4\ub3c4 \uc624\ud788\ub824 \ud53c\uc815\ubcf5\ubbfc\uc774\uc5c8\uc73c\uba70, \ud2b9\ud788 \ud398\ub974\uc2dc\uc544\uc778\uc758 \uc5ed\ud560\uc774 \ucef8\ub2e4. \uc774\uc2ac\ub78c \ubb38\ud654\uc758 \uc804\uc131\uae30\ub294 9\uc138\uae30\ubd80\ud130 11\uc138\uae30\uc5d0 \uac78\uccd0\uc11c\uc774\uba70, 12\uc138\uae30\uc5d0\ub294 \uc1e0\ud1f4\ud558\uae30 \uc2dc\uc791\ud588\ub2e4.", "paragraph_answer": "\uc774\uc2ac\ub78c\uc758 \ud669\uae08 \uc2dc\ub300\ub294 \uc774\uc2ac\ub78c \ubb38\ud654\uc640 \uc774\uc2ac\ub78c\uc744 \uae30\ucd08\ub85c \ud558\uc5ec \uadf8\ub9ac\uc2a4\u00b7\ub85c\ub9c8\uc758 \uace0\uc804 \ubb38\ud654\ub098, \ud398\ub974\uc2dc\uc544\u00b7\uc778\ub3c4\uc758 \uc5ec\ub7ec \ubb38\ud654\ub97c \ubc1b\uc544\ub4e4\uc5ec \uc131\ub9bd\ub41c \uc77c\ub300 \ucd1d\ud569 \ubb38\ud654\uc774\uba70, \uc911\uc138 \ubc0f \ub974\ub124\uc0c1\uc2a4\uae30\uc758 \uc11c\uad6c \ubb38\ud654\uc5d0 \uacb0\uc815\uc801\uc778 \uc601\ud5a5\uc744 \uc90c\uc73c\ub85c\uc368 \uadf8 \uc5ed\uc0ac\uc801\uc778 \uc758\uc758\ub294 \ub9e4\uc6b0 \ud06c\ub2e4. \ubb34\ud568\ub9c8\ub4dc\uc758 \uc0ac\ud6c4 \u300a\uafb8\ub780\u300b\uc758 \ud574\uc11d\uc744 \ud1b5\ud574 \ubc95\ud559\uc774\ub098 \uc2e0\ud559\uc774 \uc131\ub9bd\ub418\uc5c8\uc9c0\ub9cc, \uc634\ubbf8\uc544\ub4dc \uc655\uc870 \ub54c\ubd80\ud130 \uc678\ub798 \ud559\ubb38\uc758 \uc774\uc2dd\uc774 \uc2dc\uc791\ub418\uace0, \uc544\ubc14\uc2a4 \uc655\uc870 \uc2dc\ub300\uac00 \ub418\uba74\uc11c \uadf8\ub9ac\uc2a4\uc5b4 \ubb38\ud5cc \ubc88\uc5ed\uc774 \uc801\uadf9\uc801\uc73c\ub85c \uc9c4\ud589\ub418\uace0 \ucca0\ud559\u00b7\uc218\ud559\u00b7\uacfc\ud559\uc758 \uc5f0\uad6c\uac00 \ubc1c\ub2ec\ud588\ub2e4. \ud2b9\ud788 \uc81c7\ub300 \uce7c\ub9ac\ud504\uc778 \ub9c8\ubb38(\uc7ac\uc704 813~833)\uc774 \ubc14\uadf8\ub2e4\ub4dc \uc5d0 '\uc9c0\ud61c\uc758 \uc9d1'\uc744 \uc138\uc6b0\uace0, \uadf8\ub9ac\uc2a4\uc5b4 \ubb38\ud5cc\uc758 \uc218\uc9d1 \ubc0f \ubc88\uc5ed\uc744 \uc7a5\ub824\ud558\uac8c \ub418\uc790 \uc808\uc815\uc5d0 \uc774\ub974\ub800\ub2e4. \uc774\ub4e4 \ubb38\ud654 \ud65c\ub3d9\uc5d0 \uc885\uc0ac\ud55c \uc0ac\ub78c\uc740 \uc544\ub78d\uc778\ubcf4\ub2e4\ub3c4 \uc624\ud788\ub824 \ud53c\uc815\ubcf5\ubbfc\uc774\uc5c8\uc73c\uba70, \ud2b9\ud788 \ud398\ub974\uc2dc\uc544\uc778\uc758 \uc5ed\ud560\uc774 \ucef8\ub2e4. \uc774\uc2ac\ub78c \ubb38\ud654\uc758 \uc804\uc131\uae30\ub294 9\uc138\uae30\ubd80\ud130 11\uc138\uae30\uc5d0 \uac78\uccd0\uc11c\uc774\uba70, 12\uc138\uae30\uc5d0\ub294 \uc1e0\ud1f4\ud558\uae30 \uc2dc\uc791\ud588\ub2e4.", "sentence_answer": "\ud2b9\ud788 \uc81c7\ub300 \uce7c\ub9ac\ud504\uc778 \ub9c8\ubb38(\uc7ac\uc704 813~833)\uc774 \ubc14\uadf8\ub2e4\ub4dc \uc5d0", "paragraph_id": "6590047-0-1", "paragraph_question": "question: \uc774\uc2ac\ub78c\uc758 \ud669\uae08 \uc2dc\ub300\ub54c\uc758 \uce7c\ub9ac\ud504 \ub9c8\ubb38\uc740 \uc9c0\ud61c\uc758 \uc9d1\uc744 \uc5b4\ub290 \uc9c0\uc5ed\uc5d0 \uc138\uc6e0\ub098?, context: \uc774\uc2ac\ub78c\uc758 \ud669\uae08 \uc2dc\ub300\ub294 \uc774\uc2ac\ub78c \ubb38\ud654\uc640 \uc774\uc2ac\ub78c\uc744 \uae30\ucd08\ub85c \ud558\uc5ec \uadf8\ub9ac\uc2a4\u00b7\ub85c\ub9c8\uc758 \uace0\uc804 \ubb38\ud654\ub098, \ud398\ub974\uc2dc\uc544\u00b7\uc778\ub3c4\uc758 \uc5ec\ub7ec \ubb38\ud654\ub97c \ubc1b\uc544\ub4e4\uc5ec \uc131\ub9bd\ub41c \uc77c\ub300 \ucd1d\ud569 \ubb38\ud654\uc774\uba70, \uc911\uc138 \ubc0f \ub974\ub124\uc0c1\uc2a4\uae30\uc758 \uc11c\uad6c \ubb38\ud654\uc5d0 \uacb0\uc815\uc801\uc778 \uc601\ud5a5\uc744 \uc90c\uc73c\ub85c\uc368 \uadf8 \uc5ed\uc0ac\uc801\uc778 \uc758\uc758\ub294 \ub9e4\uc6b0 \ud06c\ub2e4. \ubb34\ud568\ub9c8\ub4dc\uc758 \uc0ac\ud6c4 \u300a\uafb8\ub780\u300b\uc758 \ud574\uc11d\uc744 \ud1b5\ud574 \ubc95\ud559\uc774\ub098 \uc2e0\ud559\uc774 \uc131\ub9bd\ub418\uc5c8\uc9c0\ub9cc, \uc634\ubbf8\uc544\ub4dc \uc655\uc870 \ub54c\ubd80\ud130 \uc678\ub798 \ud559\ubb38\uc758 \uc774\uc2dd\uc774 \uc2dc\uc791\ub418\uace0, \uc544\ubc14\uc2a4 \uc655\uc870 \uc2dc\ub300\uac00 \ub418\uba74\uc11c \uadf8\ub9ac\uc2a4\uc5b4 \ubb38\ud5cc \ubc88\uc5ed\uc774 \uc801\uadf9\uc801\uc73c\ub85c \uc9c4\ud589\ub418\uace0 \ucca0\ud559\u00b7\uc218\ud559\u00b7\uacfc\ud559\uc758 \uc5f0\uad6c\uac00 \ubc1c\ub2ec\ud588\ub2e4. \ud2b9\ud788 \uc81c7\ub300 \uce7c\ub9ac\ud504\uc778 \ub9c8\ubb38(\uc7ac\uc704 813~833)\uc774 \ubc14\uadf8\ub2e4\ub4dc\uc5d0 '\uc9c0\ud61c\uc758 \uc9d1'\uc744 \uc138\uc6b0\uace0, \uadf8\ub9ac\uc2a4\uc5b4 \ubb38\ud5cc\uc758 \uc218\uc9d1 \ubc0f \ubc88\uc5ed\uc744 \uc7a5\ub824\ud558\uac8c \ub418\uc790 \uc808\uc815\uc5d0 \uc774\ub974\ub800\ub2e4. \uc774\ub4e4 \ubb38\ud654 \ud65c\ub3d9\uc5d0 \uc885\uc0ac\ud55c \uc0ac\ub78c\uc740 \uc544\ub78d\uc778\ubcf4\ub2e4\ub3c4 \uc624\ud788\ub824 \ud53c\uc815\ubcf5\ubbfc\uc774\uc5c8\uc73c\uba70, \ud2b9\ud788 \ud398\ub974\uc2dc\uc544\uc778\uc758 \uc5ed\ud560\uc774 \ucef8\ub2e4. \uc774\uc2ac\ub78c \ubb38\ud654\uc758 \uc804\uc131\uae30\ub294 9\uc138\uae30\ubd80\ud130 11\uc138\uae30\uc5d0 \uac78\uccd0\uc11c\uc774\uba70, 12\uc138\uae30\uc5d0\ub294 \uc1e0\ud1f4\ud558\uae30 \uc2dc\uc791\ud588\ub2e4."} {"question": "19\uc138\uae30 \ucd08 \uc601\uad6d\uc774 \ub9cc\ub4e0 \ubc95\uc740 \ubb34\uc5c7\uc778\uac00?", "paragraph": "\uc601\uad6d\uc740 \ucca0\ud559\uc801\uc774\uace0 \uacfc\ud559\uc801\uc778 \uc815\ubcf4 \ubc0f \uc720\ub825\ud55c \ubb38\ud559\uacfc \uc5f0\uadf9\uc758 \uc804\ud1b5\uc744 \uac00\uc9c4 \uacc4\ubabd\uc2dc\ub300\uc758 \uc911\uc694\ud55c \uad6d\uac00\uc600\ub2e4. \uadf8 \ub2e4\uc74c \uc138\uae30 \ub0b4\ub0b4 \uc601\uad6d\uc740 \ubb38\ud559, \uc608\uc220 \ubc0f \uacfc\ud559\uc5d0 \ud68d\uae30\uc801\uc73c\ub85c \uacf5\ud5cc\ud55c \uc758\ud68c \ubbfc\uc8fc\uc8fc\uc758\uc640 \uac19\uc740 \uc11c\uc591 \uc0ac\uc0c1\uc758 \ubc1c\uc804\uc5d0\uc11c \uc8fc\uc5ed\uc774 \ub418\uc5c8\ub2e4. \ucd08\uae30 \ub300\uc601 \uc81c\uad6d\uc758 \ubd80\ub294 \ub2e4\ub978 \uac15\uad6d\ucc98\ub7fc \uc5ed\uc2dc 1750\ub144 \uc774\ud6c4\uc5d0 \uc788\uc5c8\ub358 \ub178\uc608\ubb34\uc5ed\uc758 \uc0b0\uc5c5\ud654\ub97c \ud3ec\ud568\ud55c \uc2dd\ubbfc \ucc29\ucde8\uc5d0 \uc758\ud574 \uadf8 \uc77c\ubd80\ub294 \uc0dd\uc131\ub418\uc5c8\ub2e4. 18\uc138\uae30 \uc601\uad6d\uc758 \ub178\uc608 \ub9e4\ub9e4\ub294 \uc138\uacc4\uc5d0\uc11c \uac00\uc7a5 \uac15\ub825\ud588\ub358 \uc601\uad6d \ud568\ub300\uac00 \uc544\ud504\ub9ac\uce74 \ub178\uc608\ub97c \uc544\uba54\ub9ac\uce74 \ub300\ub959\uc73c\ub85c \ubcf4\ub0b4\ub294 \uc545\uba85 \ub192\uc740 \uc0bc\uac01\ubb34\uc5ed\uc758 \uc77c\ubd80\uc600\ub2e4. \uadf8\ub7ec\ub098 19\uc138\uae30 \ucd08\uc5d0 \uc601\uad6d\uc740 \ub178\uc608\ubb34\uc5ed\ubc95\uc744 \ub9cc\ub4e4\uc5b4, \uc601\uc6d0\ud788 \ub178\uc608 \ubb34\uc5ed\uc744 \uae08\uc9c0\ud558\ub294 \uccab \ubc88\uc9f8 \uad6d\uac00\uac00 \ub418\uc5c8\ub2e4. \uc601\uad6d\uc740 \ubbf8\uad6d\uc758 \uc2dc\uce74\uace0, \ud074\ub9ac\ube14\ub79c\ub4dc, \ub514\ud2b8\ub85c\uc774\ud2b8, \ud53c\uce20\ubc84\uadf8, \ud544\ub77c\ub378\ud53c\uc544, \ub274\uc695, \ubcfc\ud2f0\ubaa8\uc5b4, \ubcf4\uc2a4\ud134, \uc6cc\uc2f1\ud134 D.C., \uc560\ud2c0\ub79c\ud0c0 \ub4f1\uc744 \ucc28\uc9c0\ud588\ub2e4\uac00 18\uc138\uae30 \ub9d0\uc5d0 \ub3c5\ub9bd\uc2dc\ucf30\ub2e4.", "answer": "\ub178\uc608\ubb34\uc5ed\ubc95", "sentence": "\uc601\uad6d\uc740 \ub178\uc608\ubb34\uc5ed\ubc95 \uc744 \ub9cc\ub4e4\uc5b4, \uc601\uc6d0\ud788 \ub178\uc608 \ubb34\uc5ed\uc744 \uae08\uc9c0\ud558\ub294 \uccab \ubc88\uc9f8 \uad6d\uac00\uac00 \ub418\uc5c8\ub2e4.", "paragraph_sentence": "\uc601\uad6d\uc740 \ucca0\ud559\uc801\uc774\uace0 \uacfc\ud559\uc801\uc778 \uc815\ubcf4 \ubc0f \uc720\ub825\ud55c \ubb38\ud559\uacfc \uc5f0\uadf9\uc758 \uc804\ud1b5\uc744 \uac00\uc9c4 \uacc4\ubabd\uc2dc\ub300\uc758 \uc911\uc694\ud55c \uad6d\uac00\uc600\ub2e4. \uadf8 \ub2e4\uc74c \uc138\uae30 \ub0b4\ub0b4 \uc601\uad6d\uc740 \ubb38\ud559, \uc608\uc220 \ubc0f \uacfc\ud559\uc5d0 \ud68d\uae30\uc801\uc73c\ub85c \uacf5\ud5cc\ud55c \uc758\ud68c \ubbfc\uc8fc\uc8fc\uc758\uc640 \uac19\uc740 \uc11c\uc591 \uc0ac\uc0c1\uc758 \ubc1c\uc804\uc5d0\uc11c \uc8fc\uc5ed\uc774 \ub418\uc5c8\ub2e4. \ucd08\uae30 \ub300\uc601 \uc81c\uad6d\uc758 \ubd80\ub294 \ub2e4\ub978 \uac15\uad6d\ucc98\ub7fc \uc5ed\uc2dc 1750\ub144 \uc774\ud6c4\uc5d0 \uc788\uc5c8\ub358 \ub178\uc608\ubb34\uc5ed\uc758 \uc0b0\uc5c5\ud654\ub97c \ud3ec\ud568\ud55c \uc2dd\ubbfc \ucc29\ucde8\uc5d0 \uc758\ud574 \uadf8 \uc77c\ubd80\ub294 \uc0dd\uc131\ub418\uc5c8\ub2e4. 18\uc138\uae30 \uc601\uad6d\uc758 \ub178\uc608 \ub9e4\ub9e4\ub294 \uc138\uacc4\uc5d0\uc11c \uac00\uc7a5 \uac15\ub825\ud588\ub358 \uc601\uad6d \ud568\ub300\uac00 \uc544\ud504\ub9ac\uce74 \ub178\uc608\ub97c \uc544\uba54\ub9ac\uce74 \ub300\ub959\uc73c\ub85c \ubcf4\ub0b4\ub294 \uc545\uba85 \ub192\uc740 \uc0bc\uac01\ubb34\uc5ed\uc758 \uc77c\ubd80\uc600\ub2e4. \uadf8\ub7ec\ub098 19\uc138\uae30 \ucd08\uc5d0 \uc601\uad6d\uc740 \ub178\uc608\ubb34\uc5ed\ubc95 \uc744 \ub9cc\ub4e4\uc5b4, \uc601\uc6d0\ud788 \ub178\uc608 \ubb34\uc5ed\uc744 \uae08\uc9c0\ud558\ub294 \uccab \ubc88\uc9f8 \uad6d\uac00\uac00 \ub418\uc5c8\ub2e4. \uc601\uad6d\uc740 \ubbf8\uad6d\uc758 \uc2dc\uce74\uace0, \ud074\ub9ac\ube14\ub79c\ub4dc, \ub514\ud2b8\ub85c\uc774\ud2b8, \ud53c\uce20\ubc84\uadf8, \ud544\ub77c\ub378\ud53c\uc544, \ub274\uc695, \ubcfc\ud2f0\ubaa8\uc5b4, \ubcf4\uc2a4\ud134, \uc6cc\uc2f1\ud134 D.C., \uc560\ud2c0\ub79c\ud0c0 \ub4f1\uc744 \ucc28\uc9c0\ud588\ub2e4\uac00 18\uc138\uae30 \ub9d0\uc5d0 \ub3c5\ub9bd\uc2dc\ucf30\ub2e4.", "paragraph_answer": "\uc601\uad6d\uc740 \ucca0\ud559\uc801\uc774\uace0 \uacfc\ud559\uc801\uc778 \uc815\ubcf4 \ubc0f \uc720\ub825\ud55c \ubb38\ud559\uacfc \uc5f0\uadf9\uc758 \uc804\ud1b5\uc744 \uac00\uc9c4 \uacc4\ubabd\uc2dc\ub300\uc758 \uc911\uc694\ud55c \uad6d\uac00\uc600\ub2e4. \uadf8 \ub2e4\uc74c \uc138\uae30 \ub0b4\ub0b4 \uc601\uad6d\uc740 \ubb38\ud559, \uc608\uc220 \ubc0f \uacfc\ud559\uc5d0 \ud68d\uae30\uc801\uc73c\ub85c \uacf5\ud5cc\ud55c \uc758\ud68c \ubbfc\uc8fc\uc8fc\uc758\uc640 \uac19\uc740 \uc11c\uc591 \uc0ac\uc0c1\uc758 \ubc1c\uc804\uc5d0\uc11c \uc8fc\uc5ed\uc774 \ub418\uc5c8\ub2e4. \ucd08\uae30 \ub300\uc601 \uc81c\uad6d\uc758 \ubd80\ub294 \ub2e4\ub978 \uac15\uad6d\ucc98\ub7fc \uc5ed\uc2dc 1750\ub144 \uc774\ud6c4\uc5d0 \uc788\uc5c8\ub358 \ub178\uc608\ubb34\uc5ed\uc758 \uc0b0\uc5c5\ud654\ub97c \ud3ec\ud568\ud55c \uc2dd\ubbfc \ucc29\ucde8\uc5d0 \uc758\ud574 \uadf8 \uc77c\ubd80\ub294 \uc0dd\uc131\ub418\uc5c8\ub2e4. 18\uc138\uae30 \uc601\uad6d\uc758 \ub178\uc608 \ub9e4\ub9e4\ub294 \uc138\uacc4\uc5d0\uc11c \uac00\uc7a5 \uac15\ub825\ud588\ub358 \uc601\uad6d \ud568\ub300\uac00 \uc544\ud504\ub9ac\uce74 \ub178\uc608\ub97c \uc544\uba54\ub9ac\uce74 \ub300\ub959\uc73c\ub85c \ubcf4\ub0b4\ub294 \uc545\uba85 \ub192\uc740 \uc0bc\uac01\ubb34\uc5ed\uc758 \uc77c\ubd80\uc600\ub2e4. \uadf8\ub7ec\ub098 19\uc138\uae30 \ucd08\uc5d0 \uc601\uad6d\uc740 \ub178\uc608\ubb34\uc5ed\ubc95 \uc744 \ub9cc\ub4e4\uc5b4, \uc601\uc6d0\ud788 \ub178\uc608 \ubb34\uc5ed\uc744 \uae08\uc9c0\ud558\ub294 \uccab \ubc88\uc9f8 \uad6d\uac00\uac00 \ub418\uc5c8\ub2e4. \uc601\uad6d\uc740 \ubbf8\uad6d\uc758 \uc2dc\uce74\uace0, \ud074\ub9ac\ube14\ub79c\ub4dc, \ub514\ud2b8\ub85c\uc774\ud2b8, \ud53c\uce20\ubc84\uadf8, \ud544\ub77c\ub378\ud53c\uc544, \ub274\uc695, \ubcfc\ud2f0\ubaa8\uc5b4, \ubcf4\uc2a4\ud134, \uc6cc\uc2f1\ud134 D.C., \uc560\ud2c0\ub79c\ud0c0 \ub4f1\uc744 \ucc28\uc9c0\ud588\ub2e4\uac00 18\uc138\uae30 \ub9d0\uc5d0 \ub3c5\ub9bd\uc2dc\ucf30\ub2e4.", "sentence_answer": "\uc601\uad6d\uc740 \ub178\uc608\ubb34\uc5ed\ubc95 \uc744 \ub9cc\ub4e4\uc5b4, \uc601\uc6d0\ud788 \ub178\uc608 \ubb34\uc5ed\uc744 \uae08\uc9c0\ud558\ub294 \uccab \ubc88\uc9f8 \uad6d\uac00\uac00 \ub418\uc5c8\ub2e4.", "paragraph_id": "6458616-3-0", "paragraph_question": "question: 19\uc138\uae30 \ucd08 \uc601\uad6d\uc774 \ub9cc\ub4e0 \ubc95\uc740 \ubb34\uc5c7\uc778\uac00?, context: \uc601\uad6d\uc740 \ucca0\ud559\uc801\uc774\uace0 \uacfc\ud559\uc801\uc778 \uc815\ubcf4 \ubc0f \uc720\ub825\ud55c \ubb38\ud559\uacfc \uc5f0\uadf9\uc758 \uc804\ud1b5\uc744 \uac00\uc9c4 \uacc4\ubabd\uc2dc\ub300\uc758 \uc911\uc694\ud55c \uad6d\uac00\uc600\ub2e4. \uadf8 \ub2e4\uc74c \uc138\uae30 \ub0b4\ub0b4 \uc601\uad6d\uc740 \ubb38\ud559, \uc608\uc220 \ubc0f \uacfc\ud559\uc5d0 \ud68d\uae30\uc801\uc73c\ub85c \uacf5\ud5cc\ud55c \uc758\ud68c \ubbfc\uc8fc\uc8fc\uc758\uc640 \uac19\uc740 \uc11c\uc591 \uc0ac\uc0c1\uc758 \ubc1c\uc804\uc5d0\uc11c \uc8fc\uc5ed\uc774 \ub418\uc5c8\ub2e4. \ucd08\uae30 \ub300\uc601 \uc81c\uad6d\uc758 \ubd80\ub294 \ub2e4\ub978 \uac15\uad6d\ucc98\ub7fc \uc5ed\uc2dc 1750\ub144 \uc774\ud6c4\uc5d0 \uc788\uc5c8\ub358 \ub178\uc608\ubb34\uc5ed\uc758 \uc0b0\uc5c5\ud654\ub97c \ud3ec\ud568\ud55c \uc2dd\ubbfc \ucc29\ucde8\uc5d0 \uc758\ud574 \uadf8 \uc77c\ubd80\ub294 \uc0dd\uc131\ub418\uc5c8\ub2e4. 18\uc138\uae30 \uc601\uad6d\uc758 \ub178\uc608 \ub9e4\ub9e4\ub294 \uc138\uacc4\uc5d0\uc11c \uac00\uc7a5 \uac15\ub825\ud588\ub358 \uc601\uad6d \ud568\ub300\uac00 \uc544\ud504\ub9ac\uce74 \ub178\uc608\ub97c \uc544\uba54\ub9ac\uce74 \ub300\ub959\uc73c\ub85c \ubcf4\ub0b4\ub294 \uc545\uba85 \ub192\uc740 \uc0bc\uac01\ubb34\uc5ed\uc758 \uc77c\ubd80\uc600\ub2e4. \uadf8\ub7ec\ub098 19\uc138\uae30 \ucd08\uc5d0 \uc601\uad6d\uc740 \ub178\uc608\ubb34\uc5ed\ubc95\uc744 \ub9cc\ub4e4\uc5b4, \uc601\uc6d0\ud788 \ub178\uc608 \ubb34\uc5ed\uc744 \uae08\uc9c0\ud558\ub294 \uccab \ubc88\uc9f8 \uad6d\uac00\uac00 \ub418\uc5c8\ub2e4. \uc601\uad6d\uc740 \ubbf8\uad6d\uc758 \uc2dc\uce74\uace0, \ud074\ub9ac\ube14\ub79c\ub4dc, \ub514\ud2b8\ub85c\uc774\ud2b8, \ud53c\uce20\ubc84\uadf8, \ud544\ub77c\ub378\ud53c\uc544, \ub274\uc695, \ubcfc\ud2f0\ubaa8\uc5b4, \ubcf4\uc2a4\ud134, \uc6cc\uc2f1\ud134 D.C., \uc560\ud2c0\ub79c\ud0c0 \ub4f1\uc744 \ucc28\uc9c0\ud588\ub2e4\uac00 18\uc138\uae30 \ub9d0\uc5d0 \ub3c5\ub9bd\uc2dc\ucf30\ub2e4."} {"question": "\ud070\uacf0\uc774 \ub291\ub300\ubcf4\ub2e4 \uc6b0\uc704\uc5d0 \uc788\ub294 \uac83\uc740?", "paragraph": "\ub291\ub300\ub294 \uc720\ub77c\uc2dc\uc544\uc640 \ubd81\ubbf8\uc5d0\uc11c \ud070\uacf0\uacfc \ub9cc\ub09c\ub2e4. \ud070\uacf0\uc740 \ubd80\uc721 \uacbd\uc7c1\uc5d0\uc11c \ub291\ub300\uc5d0 \uc6b0\uc704 \uad00\uacc4\uc774\uc9c0\ub9cc \ub291\ub300\ub294 \uc11c\uc2dd\uc9c0\uc5d0\uc11c \ud070\uacf0\ubcf4\ub2e4 \uc6b0\uc704\uc5d0 \uc788\ub2e4. \ub450 \uc885\uc740 \uc11c\ub85c\uc758 \uc0c8\ub07c\ub97c \uc8fd\uc778\ub2e4. \ub291\ub300\uac00 \ud070\uacf0 \uc0c8\ub07c\ub97c \uba39\uace0, \ud070\uacf0\ub3c4 \uc8fc\uc694 \uba39\uc774\uac00 \ub291\ub300 \uc0c8\ub07c\uc774\ub2e4. \ubbf8\uad6d\uc5d0\uc11c\ub294 \uc544\uba54\ub9ac\uce74\ud751\uacf0\ub3c4 \ub9c8\uc8fc\uce5c\ub2e4. \ud751\uacf0\uacfc \ub291\ub300\uac00 \ub9cc\ub098\ub294 \uc77c\uc740 \uc11c\uc2dd\uc9c0 \ucc28\uc774\ub85c \uc778\ud574 \ud070\uacf0\uacfc \ub9cc\ub098\ub294 \uc77c\ubcf4\ub2e4 \ub4dc\ubb3c\ub2e4. \ud751\uacf0\uacfc \ub291\ub300\uac00 \ub9cc\ub09c \uac83 \ub300\ubd80\ubd84\uc740 \uba55\uc2dc\ucf54 \ubd81\ubd80\uc5d0\uc11c \uae30\ub85d\ub41c \uac83\uc774\ub2e4. \ub291\ub300\ub294 \uc218\ub9ce\uc740 \uacbd\uc6b0\uc5d0 \ud751\uacf0\uc758 \ub465\uc9c0\ub97c \uacf5\uaca9\ud558\uac70\ub098 \uba39\uac70\ub098 \uc8fd\uc774\ub824 \ud558\uc9c0 \uc54a\ub294\ub2e4\uace0 \uae30\ub85d\ub418\uc5c8\ub2e4. \ud070\uacf0\uacfc\ub294 \ub2ec\ub9ac, \ud751\uacf0\uacfc \ub291\ub300\uc758 \uc0b4\ud574 \ub17c\uc7c1\uc740 \uc544\uc9c1\ub3c4 \uc9c0\uc18d\ub418\uace0 \uc788\ub2e4. \uc77c\ubc18\uc801\uc73c\ub85c \ud751\uacf0\uacfc \ud070\uacf0\uacfc\uc758 \uc870\uc6b0\ub294 \ub9ce\uc740 \ud3b8\uc774\uc9c0\ub9cc, \ubd81\uadf9\uacf0\uc740 \ub291\ub300 2\ubb34\ub9ac\uac00 \ubd81\uadf9\uacf0\uc744 \uc8fd\uc600\ub2e4\ub294 \uae30\ub85d\uc774 \uc788\uc73c\ub098 \ub9cc\ub0ac\ub2e4\ub294 \uae30\ub85d\uc740 \uac70\uc758 \uc5c6\ub2e4. \ub291\ub300\ub294 \uc544\uc2dc\uc544\ud751\uacf0\uc758 \uc0c8\ub07c\ub97c \uc8fd\uc600\ub2e4\ub294 \uae30\ub85d\uc774 \uc788\ub2e4.", "answer": "\ubd80\uc721 \uacbd\uc7c1", "sentence": "\ud070\uacf0\uc740 \ubd80\uc721 \uacbd\uc7c1 \uc5d0\uc11c \ub291\ub300\uc5d0 \uc6b0\uc704 \uad00\uacc4\uc774\uc9c0\ub9cc \ub291\ub300\ub294 \uc11c\uc2dd\uc9c0\uc5d0\uc11c \ud070\uacf0\ubcf4\ub2e4 \uc6b0\uc704\uc5d0 \uc788\ub2e4.", "paragraph_sentence": "\ub291\ub300\ub294 \uc720\ub77c\uc2dc\uc544\uc640 \ubd81\ubbf8\uc5d0\uc11c \ud070\uacf0\uacfc \ub9cc\ub09c\ub2e4. \ud070\uacf0\uc740 \ubd80\uc721 \uacbd\uc7c1 \uc5d0\uc11c \ub291\ub300\uc5d0 \uc6b0\uc704 \uad00\uacc4\uc774\uc9c0\ub9cc \ub291\ub300\ub294 \uc11c\uc2dd\uc9c0\uc5d0\uc11c \ud070\uacf0\ubcf4\ub2e4 \uc6b0\uc704\uc5d0 \uc788\ub2e4. \ub450 \uc885\uc740 \uc11c\ub85c\uc758 \uc0c8\ub07c\ub97c \uc8fd\uc778\ub2e4. \ub291\ub300\uac00 \ud070\uacf0 \uc0c8\ub07c\ub97c \uba39\uace0, \ud070\uacf0\ub3c4 \uc8fc\uc694 \uba39\uc774\uac00 \ub291\ub300 \uc0c8\ub07c\uc774\ub2e4. \ubbf8\uad6d\uc5d0\uc11c\ub294 \uc544\uba54\ub9ac\uce74\ud751\uacf0\ub3c4 \ub9c8\uc8fc\uce5c\ub2e4. \ud751\uacf0\uacfc \ub291\ub300\uac00 \ub9cc\ub098\ub294 \uc77c\uc740 \uc11c\uc2dd\uc9c0 \ucc28\uc774\ub85c \uc778\ud574 \ud070\uacf0\uacfc \ub9cc\ub098\ub294 \uc77c\ubcf4\ub2e4 \ub4dc\ubb3c\ub2e4. \ud751\uacf0\uacfc \ub291\ub300\uac00 \ub9cc\ub09c \uac83 \ub300\ubd80\ubd84\uc740 \uba55\uc2dc\ucf54 \ubd81\ubd80\uc5d0\uc11c \uae30\ub85d\ub41c \uac83\uc774\ub2e4. \ub291\ub300\ub294 \uc218\ub9ce\uc740 \uacbd\uc6b0\uc5d0 \ud751\uacf0\uc758 \ub465\uc9c0\ub97c \uacf5\uaca9\ud558\uac70\ub098 \uba39\uac70\ub098 \uc8fd\uc774\ub824 \ud558\uc9c0 \uc54a\ub294\ub2e4\uace0 \uae30\ub85d\ub418\uc5c8\ub2e4. \ud070\uacf0\uacfc\ub294 \ub2ec\ub9ac, \ud751\uacf0\uacfc \ub291\ub300\uc758 \uc0b4\ud574 \ub17c\uc7c1\uc740 \uc544\uc9c1\ub3c4 \uc9c0\uc18d\ub418\uace0 \uc788\ub2e4. \uc77c\ubc18\uc801\uc73c\ub85c \ud751\uacf0\uacfc \ud070\uacf0\uacfc\uc758 \uc870\uc6b0\ub294 \ub9ce\uc740 \ud3b8\uc774\uc9c0\ub9cc, \ubd81\uadf9\uacf0\uc740 \ub291\ub300 2\ubb34\ub9ac\uac00 \ubd81\uadf9\uacf0\uc744 \uc8fd\uc600\ub2e4\ub294 \uae30\ub85d\uc774 \uc788\uc73c\ub098 \ub9cc\ub0ac\ub2e4\ub294 \uae30\ub85d\uc740 \uac70\uc758 \uc5c6\ub2e4. \ub291\ub300\ub294 \uc544\uc2dc\uc544\ud751\uacf0\uc758 \uc0c8\ub07c\ub97c \uc8fd\uc600\ub2e4\ub294 \uae30\ub85d\uc774 \uc788\ub2e4.", "paragraph_answer": "\ub291\ub300\ub294 \uc720\ub77c\uc2dc\uc544\uc640 \ubd81\ubbf8\uc5d0\uc11c \ud070\uacf0\uacfc \ub9cc\ub09c\ub2e4. \ud070\uacf0\uc740 \ubd80\uc721 \uacbd\uc7c1 \uc5d0\uc11c \ub291\ub300\uc5d0 \uc6b0\uc704 \uad00\uacc4\uc774\uc9c0\ub9cc \ub291\ub300\ub294 \uc11c\uc2dd\uc9c0\uc5d0\uc11c \ud070\uacf0\ubcf4\ub2e4 \uc6b0\uc704\uc5d0 \uc788\ub2e4. \ub450 \uc885\uc740 \uc11c\ub85c\uc758 \uc0c8\ub07c\ub97c \uc8fd\uc778\ub2e4. \ub291\ub300\uac00 \ud070\uacf0 \uc0c8\ub07c\ub97c \uba39\uace0, \ud070\uacf0\ub3c4 \uc8fc\uc694 \uba39\uc774\uac00 \ub291\ub300 \uc0c8\ub07c\uc774\ub2e4. \ubbf8\uad6d\uc5d0\uc11c\ub294 \uc544\uba54\ub9ac\uce74\ud751\uacf0\ub3c4 \ub9c8\uc8fc\uce5c\ub2e4. \ud751\uacf0\uacfc \ub291\ub300\uac00 \ub9cc\ub098\ub294 \uc77c\uc740 \uc11c\uc2dd\uc9c0 \ucc28\uc774\ub85c \uc778\ud574 \ud070\uacf0\uacfc \ub9cc\ub098\ub294 \uc77c\ubcf4\ub2e4 \ub4dc\ubb3c\ub2e4. \ud751\uacf0\uacfc \ub291\ub300\uac00 \ub9cc\ub09c \uac83 \ub300\ubd80\ubd84\uc740 \uba55\uc2dc\ucf54 \ubd81\ubd80\uc5d0\uc11c \uae30\ub85d\ub41c \uac83\uc774\ub2e4. \ub291\ub300\ub294 \uc218\ub9ce\uc740 \uacbd\uc6b0\uc5d0 \ud751\uacf0\uc758 \ub465\uc9c0\ub97c \uacf5\uaca9\ud558\uac70\ub098 \uba39\uac70\ub098 \uc8fd\uc774\ub824 \ud558\uc9c0 \uc54a\ub294\ub2e4\uace0 \uae30\ub85d\ub418\uc5c8\ub2e4. \ud070\uacf0\uacfc\ub294 \ub2ec\ub9ac, \ud751\uacf0\uacfc \ub291\ub300\uc758 \uc0b4\ud574 \ub17c\uc7c1\uc740 \uc544\uc9c1\ub3c4 \uc9c0\uc18d\ub418\uace0 \uc788\ub2e4. \uc77c\ubc18\uc801\uc73c\ub85c \ud751\uacf0\uacfc \ud070\uacf0\uacfc\uc758 \uc870\uc6b0\ub294 \ub9ce\uc740 \ud3b8\uc774\uc9c0\ub9cc, \ubd81\uadf9\uacf0\uc740 \ub291\ub300 2\ubb34\ub9ac\uac00 \ubd81\uadf9\uacf0\uc744 \uc8fd\uc600\ub2e4\ub294 \uae30\ub85d\uc774 \uc788\uc73c\ub098 \ub9cc\ub0ac\ub2e4\ub294 \uae30\ub85d\uc740 \uac70\uc758 \uc5c6\ub2e4. \ub291\ub300\ub294 \uc544\uc2dc\uc544\ud751\uacf0\uc758 \uc0c8\ub07c\ub97c \uc8fd\uc600\ub2e4\ub294 \uae30\ub85d\uc774 \uc788\ub2e4.", "sentence_answer": "\ud070\uacf0\uc740 \ubd80\uc721 \uacbd\uc7c1 \uc5d0\uc11c \ub291\ub300\uc5d0 \uc6b0\uc704 \uad00\uacc4\uc774\uc9c0\ub9cc \ub291\ub300\ub294 \uc11c\uc2dd\uc9c0\uc5d0\uc11c \ud070\uacf0\ubcf4\ub2e4 \uc6b0\uc704\uc5d0 \uc788\ub2e4.", "paragraph_id": "6246005-29-1", "paragraph_question": "question: \ud070\uacf0\uc774 \ub291\ub300\ubcf4\ub2e4 \uc6b0\uc704\uc5d0 \uc788\ub294 \uac83\uc740?, context: \ub291\ub300\ub294 \uc720\ub77c\uc2dc\uc544\uc640 \ubd81\ubbf8\uc5d0\uc11c \ud070\uacf0\uacfc \ub9cc\ub09c\ub2e4. \ud070\uacf0\uc740 \ubd80\uc721 \uacbd\uc7c1\uc5d0\uc11c \ub291\ub300\uc5d0 \uc6b0\uc704 \uad00\uacc4\uc774\uc9c0\ub9cc \ub291\ub300\ub294 \uc11c\uc2dd\uc9c0\uc5d0\uc11c \ud070\uacf0\ubcf4\ub2e4 \uc6b0\uc704\uc5d0 \uc788\ub2e4. \ub450 \uc885\uc740 \uc11c\ub85c\uc758 \uc0c8\ub07c\ub97c \uc8fd\uc778\ub2e4. \ub291\ub300\uac00 \ud070\uacf0 \uc0c8\ub07c\ub97c \uba39\uace0, \ud070\uacf0\ub3c4 \uc8fc\uc694 \uba39\uc774\uac00 \ub291\ub300 \uc0c8\ub07c\uc774\ub2e4. \ubbf8\uad6d\uc5d0\uc11c\ub294 \uc544\uba54\ub9ac\uce74\ud751\uacf0\ub3c4 \ub9c8\uc8fc\uce5c\ub2e4. \ud751\uacf0\uacfc \ub291\ub300\uac00 \ub9cc\ub098\ub294 \uc77c\uc740 \uc11c\uc2dd\uc9c0 \ucc28\uc774\ub85c \uc778\ud574 \ud070\uacf0\uacfc \ub9cc\ub098\ub294 \uc77c\ubcf4\ub2e4 \ub4dc\ubb3c\ub2e4. \ud751\uacf0\uacfc \ub291\ub300\uac00 \ub9cc\ub09c \uac83 \ub300\ubd80\ubd84\uc740 \uba55\uc2dc\ucf54 \ubd81\ubd80\uc5d0\uc11c \uae30\ub85d\ub41c \uac83\uc774\ub2e4. \ub291\ub300\ub294 \uc218\ub9ce\uc740 \uacbd\uc6b0\uc5d0 \ud751\uacf0\uc758 \ub465\uc9c0\ub97c \uacf5\uaca9\ud558\uac70\ub098 \uba39\uac70\ub098 \uc8fd\uc774\ub824 \ud558\uc9c0 \uc54a\ub294\ub2e4\uace0 \uae30\ub85d\ub418\uc5c8\ub2e4. \ud070\uacf0\uacfc\ub294 \ub2ec\ub9ac, \ud751\uacf0\uacfc \ub291\ub300\uc758 \uc0b4\ud574 \ub17c\uc7c1\uc740 \uc544\uc9c1\ub3c4 \uc9c0\uc18d\ub418\uace0 \uc788\ub2e4. \uc77c\ubc18\uc801\uc73c\ub85c \ud751\uacf0\uacfc \ud070\uacf0\uacfc\uc758 \uc870\uc6b0\ub294 \ub9ce\uc740 \ud3b8\uc774\uc9c0\ub9cc, \ubd81\uadf9\uacf0\uc740 \ub291\ub300 2\ubb34\ub9ac\uac00 \ubd81\uadf9\uacf0\uc744 \uc8fd\uc600\ub2e4\ub294 \uae30\ub85d\uc774 \uc788\uc73c\ub098 \ub9cc\ub0ac\ub2e4\ub294 \uae30\ub85d\uc740 \uac70\uc758 \uc5c6\ub2e4. \ub291\ub300\ub294 \uc544\uc2dc\uc544\ud751\uacf0\uc758 \uc0c8\ub07c\ub97c \uc8fd\uc600\ub2e4\ub294 \uae30\ub85d\uc774 \uc788\ub2e4."} {"question": "\ub274 SM7 \ub178\ubc14 1\ud638 \ucc28\ub97c \uad6c\uc785\ud55c \uc0ac\ub78c\uc740?", "paragraph": "2014\ub144 9\uc6d4 2\uc77c\uc5d0 \ucd9c\uc2dc\ub41c \ub274 SM7 \ub178\ubc14\ub294 \ubc94\ud37c\uc640 \ub77c\ub514\uc5d0\uc774\ud130 \uadf8\ub9b4\uc5d0 \ubcc0\ud654\ub97c \uc8fc\uc5b4 QM3\ubd80\ud130 \uc2dc\uc791\ub41c \ub974\ub178\uc0bc\uc131\uc790\ub3d9\ucc28\uc758 \ud328\ubc00\ub9ac \ub8e9\uc774 \uc801\uc6a9\ub418\uc5c8\uace0, LED \uc8fc\uac04 \uc8fc\ud589\ub4f1\uc774 \uc801\uc6a9\ub418\uc5c8\ub2e4. \ub178\ubc14\ub77c\ub294 \uc11c\ube0c \ub124\uc784\uc740 \uc2e0\uc131\uc774\ub77c\ub294 \uc758\ubbf8\uc758 \ub77c\ud2f4\uc5b4\ub85c, \uc0c8\ub86d\uac8c \ub5a0\uc624\ub974\ub294 \uc720\ub7ec\ud53c\uc5b8 \ud504\ub9ac\ubbf8\uc5c4 \uc2b9\uc6a9\ucc28\ub97c \uc758\ubbf8\ud55c\ub2e4. \ub300\ud55c\ubbfc\uad6d\uc0b0 \uc790\ub3d9\ucc28 \ucd5c\ucd08\ub85c \uc640\uc774\ud30c\uc774\ub97c \ud65c\uc6a9\ud55c \uc2a4\ub9c8\ud2b8 \ubbf8\ub7ec\ub9c1 \uc2dc\uc2a4\ud15c(\uc2a4\ub9c8\ud2b8 \ud3f0\uacfc \ucc28\ub7c9\uc758 \ubaa8\ub2c8\ud130\ub97c \uc640\uc774\ud30c\uc774\ub85c \uc5f0\uacb0\ud558\ub294 \uc2dc\uc2a4\ud15c)\uc774 \uc801\uc6a9\ub418\uc5c8\uace0, \uc548\ub4dc\ub85c\uc774\ub4dc \ud3f0\uc5d0\uc11c\ub294 \ucc28\ub7c9\uc758 \ubaa8\ub2c8\ud130\uc640 \uc2a4\ub9c8\ud2b8 \ud3f0 \uac04 \uc591\ubc29\ud5a5 \uc870\uc791\uae4c\uc9c0 \ud560 \uc218 \uc788\ub2e4. \uac19\uc740 \ud574 9\uc6d4 4\uc77c\uc5d0\ub294 \uc774 \ucc28\uc758 1\ud638\ucc28\uac00 \uc11c\ubcd1\uc218 \ubd80\uc0b0\uad11\uc5ed\uc2dc\uc7a5\uc5d0\uac8c \uad00\uc6a9\ucc28\ub85c \uc804\ub2ec\ub418\uc5c8\ub2e4. \uae30\uc874\uc5d0\ub294 \ud604\ub300 \uc5d0\ucfe0\uc2a4\ub97c \uad00\uc6a9\ucc28\ub85c \uc774\uc6a9\ud558\ub358 \uc11c \uc2dc\uc7a5\uc740 \ub974\ub178\uc0bc\uc131\uc790\ub3d9\ucc28\ub97c \ube44\ub86f\ud55c \ubd80\uc0b0 \ud5a5\ud1a0 \uae30\uc5c5\uc744 \uc0b4\ub9ac\uaca0\ub2e4\ub294 \ucde8\uc9c0\ub85c \uc758\uc804\ucc28\ub97c \ub274 SM7 \ub178\ubc14\ub85c \uad50\uccb4\ud558\uc600\ub2e4\uace0 \ubc1d\ud614\ub2e4. 2015\ub144 8\uc6d4 3\uc77c\ubd80\ud130 I4 2.0\u2113 LPG \uc5d4\uc9c4\uc774 \ucd94\uac00\ub418\uc5b4 \uc7a5\uc560\uc778\uc6a9\uacfc \ub80c\ud130\uce74\ub85c \ud310\ub9e4\uac00 \uc2dc\uc791\ub418\uc5c8\ub2e4. \uc2a4\ud398\uc5b4 \ud0c0\uc774\uc5b4 \uacf5\uac04\uc5d0 \ub3c4\ub11b\u00ae \ud0f1\ud06c\uac00 \uc801\uc6a9\ub418\uc5b4 \ud2b8\ub801\ud06c \uacf5\uac04\uc744 \ub113\ud614\uace0, \uc2a4\ud0a4\uc2a4\ub8e8 \uc2dc\ud2b8\uac00 \uc801\uc6a9\ub418\uc5b4 \ud65c\uc6a9\ub3c4\ub97c \ub192\uc600\ub2e4. 2016\ub144 7\uc6d4 4\uc77c\uc5d0 \uc120\ubcf4\uc778 2017\ub144\ud615\uc740 \ub77c\uc778\uc5c5\uc774 \ub2e8\uc21c\ud654\ub418\uc5c8\uace0, \uc2a4\ub9c8\ud2b8 \ucee4\ub125\ud2b8\uc640 \ud53c\uc544\ub178 \ube14\ub799 \uc778\ud14c\ub9ac\uc5b4\ub97c \uae30\ubcf8 \uc801\uc6a9\ub428\uacfc \ub3d9\uc2dc\uc5d0 2\uac00\uc9c0 \ub514\uc790\uc778\uc758 18\uc778\uce58 \uc54c\ub8e8\ubbf8\ub284 \ud720\uc744 \ubb34\uc0c1 \uc120\ud0dd\ud560 \uc218 \uc788\ub3c4\ub85d \ud558\uc600\ub2e4. \uc544\uc6b8\ub7ec I4 2.0\u2113 LPG \uc5d4\uc9c4\uc774 \uc7a5\ucc29\ub41c \ud0dd\uc2dc\uac00 \ucd9c\uc2dc\ub418\uc5c8\ub2e4.", "answer": "\uc11c\ubcd1\uc218", "sentence": "\uac19\uc740 \ud574 9\uc6d4 4\uc77c\uc5d0\ub294 \uc774 \ucc28\uc758 1\ud638\ucc28\uac00 \uc11c\ubcd1\uc218 \ubd80\uc0b0\uad11\uc5ed\uc2dc\uc7a5\uc5d0\uac8c \uad00\uc6a9\ucc28\ub85c \uc804\ub2ec\ub418\uc5c8\ub2e4.", "paragraph_sentence": "2014\ub144 9\uc6d4 2\uc77c\uc5d0 \ucd9c\uc2dc\ub41c \ub274 SM7 \ub178\ubc14\ub294 \ubc94\ud37c\uc640 \ub77c\ub514\uc5d0\uc774\ud130 \uadf8\ub9b4\uc5d0 \ubcc0\ud654\ub97c \uc8fc\uc5b4 QM3\ubd80\ud130 \uc2dc\uc791\ub41c \ub974\ub178\uc0bc\uc131\uc790\ub3d9\ucc28\uc758 \ud328\ubc00\ub9ac \ub8e9\uc774 \uc801\uc6a9\ub418\uc5c8\uace0, LED \uc8fc\uac04 \uc8fc\ud589\ub4f1\uc774 \uc801\uc6a9\ub418\uc5c8\ub2e4. \ub178\ubc14\ub77c\ub294 \uc11c\ube0c \ub124\uc784\uc740 \uc2e0\uc131\uc774\ub77c\ub294 \uc758\ubbf8\uc758 \ub77c\ud2f4\uc5b4\ub85c, \uc0c8\ub86d\uac8c \ub5a0\uc624\ub974\ub294 \uc720\ub7ec\ud53c\uc5b8 \ud504\ub9ac\ubbf8\uc5c4 \uc2b9\uc6a9\ucc28\ub97c \uc758\ubbf8\ud55c\ub2e4. \ub300\ud55c\ubbfc\uad6d\uc0b0 \uc790\ub3d9\ucc28 \ucd5c\ucd08\ub85c \uc640\uc774\ud30c\uc774\ub97c \ud65c\uc6a9\ud55c \uc2a4\ub9c8\ud2b8 \ubbf8\ub7ec\ub9c1 \uc2dc\uc2a4\ud15c(\uc2a4\ub9c8\ud2b8 \ud3f0\uacfc \ucc28\ub7c9\uc758 \ubaa8\ub2c8\ud130\ub97c \uc640\uc774\ud30c\uc774\ub85c \uc5f0\uacb0\ud558\ub294 \uc2dc\uc2a4\ud15c)\uc774 \uc801\uc6a9\ub418\uc5c8\uace0, \uc548\ub4dc\ub85c\uc774\ub4dc \ud3f0\uc5d0\uc11c\ub294 \ucc28\ub7c9\uc758 \ubaa8\ub2c8\ud130\uc640 \uc2a4\ub9c8\ud2b8 \ud3f0 \uac04 \uc591\ubc29\ud5a5 \uc870\uc791\uae4c\uc9c0 \ud560 \uc218 \uc788\ub2e4. \uac19\uc740 \ud574 9\uc6d4 4\uc77c\uc5d0\ub294 \uc774 \ucc28\uc758 1\ud638\ucc28\uac00 \uc11c\ubcd1\uc218 \ubd80\uc0b0\uad11\uc5ed\uc2dc\uc7a5\uc5d0\uac8c \uad00\uc6a9\ucc28\ub85c \uc804\ub2ec\ub418\uc5c8\ub2e4. \uae30\uc874\uc5d0\ub294 \ud604\ub300 \uc5d0\ucfe0\uc2a4\ub97c \uad00\uc6a9\ucc28\ub85c \uc774\uc6a9\ud558\ub358 \uc11c \uc2dc\uc7a5\uc740 \ub974\ub178\uc0bc\uc131\uc790\ub3d9\ucc28\ub97c \ube44\ub86f\ud55c \ubd80\uc0b0 \ud5a5\ud1a0 \uae30\uc5c5\uc744 \uc0b4\ub9ac\uaca0\ub2e4\ub294 \ucde8\uc9c0\ub85c \uc758\uc804\ucc28\ub97c \ub274 SM7 \ub178\ubc14\ub85c \uad50\uccb4\ud558\uc600\ub2e4\uace0 \ubc1d\ud614\ub2e4. 2015\ub144 8\uc6d4 3\uc77c\ubd80\ud130 I4 2.0\u2113 LPG \uc5d4\uc9c4\uc774 \ucd94\uac00\ub418\uc5b4 \uc7a5\uc560\uc778\uc6a9\uacfc \ub80c\ud130\uce74\ub85c \ud310\ub9e4\uac00 \uc2dc\uc791\ub418\uc5c8\ub2e4. \uc2a4\ud398\uc5b4 \ud0c0\uc774\uc5b4 \uacf5\uac04\uc5d0 \ub3c4\ub11b\u00ae \ud0f1\ud06c\uac00 \uc801\uc6a9\ub418\uc5b4 \ud2b8\ub801\ud06c \uacf5\uac04\uc744 \ub113\ud614\uace0, \uc2a4\ud0a4\uc2a4\ub8e8 \uc2dc\ud2b8\uac00 \uc801\uc6a9\ub418\uc5b4 \ud65c\uc6a9\ub3c4\ub97c \ub192\uc600\ub2e4. 2016\ub144 7\uc6d4 4\uc77c\uc5d0 \uc120\ubcf4\uc778 2017\ub144\ud615\uc740 \ub77c\uc778\uc5c5\uc774 \ub2e8\uc21c\ud654\ub418\uc5c8\uace0, \uc2a4\ub9c8\ud2b8 \ucee4\ub125\ud2b8\uc640 \ud53c\uc544\ub178 \ube14\ub799 \uc778\ud14c\ub9ac\uc5b4\ub97c \uae30\ubcf8 \uc801\uc6a9\ub428\uacfc \ub3d9\uc2dc\uc5d0 2\uac00\uc9c0 \ub514\uc790\uc778\uc758 18\uc778\uce58 \uc54c\ub8e8\ubbf8\ub284 \ud720\uc744 \ubb34\uc0c1 \uc120\ud0dd\ud560 \uc218 \uc788\ub3c4\ub85d \ud558\uc600\ub2e4. \uc544\uc6b8\ub7ec I4 2.0\u2113 LPG \uc5d4\uc9c4\uc774 \uc7a5\ucc29\ub41c \ud0dd\uc2dc\uac00 \ucd9c\uc2dc\ub418\uc5c8\ub2e4.", "paragraph_answer": "2014\ub144 9\uc6d4 2\uc77c\uc5d0 \ucd9c\uc2dc\ub41c \ub274 SM7 \ub178\ubc14\ub294 \ubc94\ud37c\uc640 \ub77c\ub514\uc5d0\uc774\ud130 \uadf8\ub9b4\uc5d0 \ubcc0\ud654\ub97c \uc8fc\uc5b4 QM3\ubd80\ud130 \uc2dc\uc791\ub41c \ub974\ub178\uc0bc\uc131\uc790\ub3d9\ucc28\uc758 \ud328\ubc00\ub9ac \ub8e9\uc774 \uc801\uc6a9\ub418\uc5c8\uace0, LED \uc8fc\uac04 \uc8fc\ud589\ub4f1\uc774 \uc801\uc6a9\ub418\uc5c8\ub2e4. \ub178\ubc14\ub77c\ub294 \uc11c\ube0c \ub124\uc784\uc740 \uc2e0\uc131\uc774\ub77c\ub294 \uc758\ubbf8\uc758 \ub77c\ud2f4\uc5b4\ub85c, \uc0c8\ub86d\uac8c \ub5a0\uc624\ub974\ub294 \uc720\ub7ec\ud53c\uc5b8 \ud504\ub9ac\ubbf8\uc5c4 \uc2b9\uc6a9\ucc28\ub97c \uc758\ubbf8\ud55c\ub2e4. \ub300\ud55c\ubbfc\uad6d\uc0b0 \uc790\ub3d9\ucc28 \ucd5c\ucd08\ub85c \uc640\uc774\ud30c\uc774\ub97c \ud65c\uc6a9\ud55c \uc2a4\ub9c8\ud2b8 \ubbf8\ub7ec\ub9c1 \uc2dc\uc2a4\ud15c(\uc2a4\ub9c8\ud2b8 \ud3f0\uacfc \ucc28\ub7c9\uc758 \ubaa8\ub2c8\ud130\ub97c \uc640\uc774\ud30c\uc774\ub85c \uc5f0\uacb0\ud558\ub294 \uc2dc\uc2a4\ud15c)\uc774 \uc801\uc6a9\ub418\uc5c8\uace0, \uc548\ub4dc\ub85c\uc774\ub4dc \ud3f0\uc5d0\uc11c\ub294 \ucc28\ub7c9\uc758 \ubaa8\ub2c8\ud130\uc640 \uc2a4\ub9c8\ud2b8 \ud3f0 \uac04 \uc591\ubc29\ud5a5 \uc870\uc791\uae4c\uc9c0 \ud560 \uc218 \uc788\ub2e4. \uac19\uc740 \ud574 9\uc6d4 4\uc77c\uc5d0\ub294 \uc774 \ucc28\uc758 1\ud638\ucc28\uac00 \uc11c\ubcd1\uc218 \ubd80\uc0b0\uad11\uc5ed\uc2dc\uc7a5\uc5d0\uac8c \uad00\uc6a9\ucc28\ub85c \uc804\ub2ec\ub418\uc5c8\ub2e4. \uae30\uc874\uc5d0\ub294 \ud604\ub300 \uc5d0\ucfe0\uc2a4\ub97c \uad00\uc6a9\ucc28\ub85c \uc774\uc6a9\ud558\ub358 \uc11c \uc2dc\uc7a5\uc740 \ub974\ub178\uc0bc\uc131\uc790\ub3d9\ucc28\ub97c \ube44\ub86f\ud55c \ubd80\uc0b0 \ud5a5\ud1a0 \uae30\uc5c5\uc744 \uc0b4\ub9ac\uaca0\ub2e4\ub294 \ucde8\uc9c0\ub85c \uc758\uc804\ucc28\ub97c \ub274 SM7 \ub178\ubc14\ub85c \uad50\uccb4\ud558\uc600\ub2e4\uace0 \ubc1d\ud614\ub2e4. 2015\ub144 8\uc6d4 3\uc77c\ubd80\ud130 I4 2.0\u2113 LPG \uc5d4\uc9c4\uc774 \ucd94\uac00\ub418\uc5b4 \uc7a5\uc560\uc778\uc6a9\uacfc \ub80c\ud130\uce74\ub85c \ud310\ub9e4\uac00 \uc2dc\uc791\ub418\uc5c8\ub2e4. \uc2a4\ud398\uc5b4 \ud0c0\uc774\uc5b4 \uacf5\uac04\uc5d0 \ub3c4\ub11b\u00ae \ud0f1\ud06c\uac00 \uc801\uc6a9\ub418\uc5b4 \ud2b8\ub801\ud06c \uacf5\uac04\uc744 \ub113\ud614\uace0, \uc2a4\ud0a4\uc2a4\ub8e8 \uc2dc\ud2b8\uac00 \uc801\uc6a9\ub418\uc5b4 \ud65c\uc6a9\ub3c4\ub97c \ub192\uc600\ub2e4. 2016\ub144 7\uc6d4 4\uc77c\uc5d0 \uc120\ubcf4\uc778 2017\ub144\ud615\uc740 \ub77c\uc778\uc5c5\uc774 \ub2e8\uc21c\ud654\ub418\uc5c8\uace0, \uc2a4\ub9c8\ud2b8 \ucee4\ub125\ud2b8\uc640 \ud53c\uc544\ub178 \ube14\ub799 \uc778\ud14c\ub9ac\uc5b4\ub97c \uae30\ubcf8 \uc801\uc6a9\ub428\uacfc \ub3d9\uc2dc\uc5d0 2\uac00\uc9c0 \ub514\uc790\uc778\uc758 18\uc778\uce58 \uc54c\ub8e8\ubbf8\ub284 \ud720\uc744 \ubb34\uc0c1 \uc120\ud0dd\ud560 \uc218 \uc788\ub3c4\ub85d \ud558\uc600\ub2e4. \uc544\uc6b8\ub7ec I4 2.0\u2113 LPG \uc5d4\uc9c4\uc774 \uc7a5\ucc29\ub41c \ud0dd\uc2dc\uac00 \ucd9c\uc2dc\ub418\uc5c8\ub2e4.", "sentence_answer": "\uac19\uc740 \ud574 9\uc6d4 4\uc77c\uc5d0\ub294 \uc774 \ucc28\uc758 1\ud638\ucc28\uac00 \uc11c\ubcd1\uc218 \ubd80\uc0b0\uad11\uc5ed\uc2dc\uc7a5\uc5d0\uac8c \uad00\uc6a9\ucc28\ub85c \uc804\ub2ec\ub418\uc5c8\ub2e4.", "paragraph_id": "6498341-2-2", "paragraph_question": "question: \ub274 SM7 \ub178\ubc14 1\ud638 \ucc28\ub97c \uad6c\uc785\ud55c \uc0ac\ub78c\uc740?, context: 2014\ub144 9\uc6d4 2\uc77c\uc5d0 \ucd9c\uc2dc\ub41c \ub274 SM7 \ub178\ubc14\ub294 \ubc94\ud37c\uc640 \ub77c\ub514\uc5d0\uc774\ud130 \uadf8\ub9b4\uc5d0 \ubcc0\ud654\ub97c \uc8fc\uc5b4 QM3\ubd80\ud130 \uc2dc\uc791\ub41c \ub974\ub178\uc0bc\uc131\uc790\ub3d9\ucc28\uc758 \ud328\ubc00\ub9ac \ub8e9\uc774 \uc801\uc6a9\ub418\uc5c8\uace0, LED \uc8fc\uac04 \uc8fc\ud589\ub4f1\uc774 \uc801\uc6a9\ub418\uc5c8\ub2e4. \ub178\ubc14\ub77c\ub294 \uc11c\ube0c \ub124\uc784\uc740 \uc2e0\uc131\uc774\ub77c\ub294 \uc758\ubbf8\uc758 \ub77c\ud2f4\uc5b4\ub85c, \uc0c8\ub86d\uac8c \ub5a0\uc624\ub974\ub294 \uc720\ub7ec\ud53c\uc5b8 \ud504\ub9ac\ubbf8\uc5c4 \uc2b9\uc6a9\ucc28\ub97c \uc758\ubbf8\ud55c\ub2e4. \ub300\ud55c\ubbfc\uad6d\uc0b0 \uc790\ub3d9\ucc28 \ucd5c\ucd08\ub85c \uc640\uc774\ud30c\uc774\ub97c \ud65c\uc6a9\ud55c \uc2a4\ub9c8\ud2b8 \ubbf8\ub7ec\ub9c1 \uc2dc\uc2a4\ud15c(\uc2a4\ub9c8\ud2b8 \ud3f0\uacfc \ucc28\ub7c9\uc758 \ubaa8\ub2c8\ud130\ub97c \uc640\uc774\ud30c\uc774\ub85c \uc5f0\uacb0\ud558\ub294 \uc2dc\uc2a4\ud15c)\uc774 \uc801\uc6a9\ub418\uc5c8\uace0, \uc548\ub4dc\ub85c\uc774\ub4dc \ud3f0\uc5d0\uc11c\ub294 \ucc28\ub7c9\uc758 \ubaa8\ub2c8\ud130\uc640 \uc2a4\ub9c8\ud2b8 \ud3f0 \uac04 \uc591\ubc29\ud5a5 \uc870\uc791\uae4c\uc9c0 \ud560 \uc218 \uc788\ub2e4. \uac19\uc740 \ud574 9\uc6d4 4\uc77c\uc5d0\ub294 \uc774 \ucc28\uc758 1\ud638\ucc28\uac00 \uc11c\ubcd1\uc218 \ubd80\uc0b0\uad11\uc5ed\uc2dc\uc7a5\uc5d0\uac8c \uad00\uc6a9\ucc28\ub85c \uc804\ub2ec\ub418\uc5c8\ub2e4. \uae30\uc874\uc5d0\ub294 \ud604\ub300 \uc5d0\ucfe0\uc2a4\ub97c \uad00\uc6a9\ucc28\ub85c \uc774\uc6a9\ud558\ub358 \uc11c \uc2dc\uc7a5\uc740 \ub974\ub178\uc0bc\uc131\uc790\ub3d9\ucc28\ub97c \ube44\ub86f\ud55c \ubd80\uc0b0 \ud5a5\ud1a0 \uae30\uc5c5\uc744 \uc0b4\ub9ac\uaca0\ub2e4\ub294 \ucde8\uc9c0\ub85c \uc758\uc804\ucc28\ub97c \ub274 SM7 \ub178\ubc14\ub85c \uad50\uccb4\ud558\uc600\ub2e4\uace0 \ubc1d\ud614\ub2e4. 2015\ub144 8\uc6d4 3\uc77c\ubd80\ud130 I4 2.0\u2113 LPG \uc5d4\uc9c4\uc774 \ucd94\uac00\ub418\uc5b4 \uc7a5\uc560\uc778\uc6a9\uacfc \ub80c\ud130\uce74\ub85c \ud310\ub9e4\uac00 \uc2dc\uc791\ub418\uc5c8\ub2e4. \uc2a4\ud398\uc5b4 \ud0c0\uc774\uc5b4 \uacf5\uac04\uc5d0 \ub3c4\ub11b\u00ae \ud0f1\ud06c\uac00 \uc801\uc6a9\ub418\uc5b4 \ud2b8\ub801\ud06c \uacf5\uac04\uc744 \ub113\ud614\uace0, \uc2a4\ud0a4\uc2a4\ub8e8 \uc2dc\ud2b8\uac00 \uc801\uc6a9\ub418\uc5b4 \ud65c\uc6a9\ub3c4\ub97c \ub192\uc600\ub2e4. 2016\ub144 7\uc6d4 4\uc77c\uc5d0 \uc120\ubcf4\uc778 2017\ub144\ud615\uc740 \ub77c\uc778\uc5c5\uc774 \ub2e8\uc21c\ud654\ub418\uc5c8\uace0, \uc2a4\ub9c8\ud2b8 \ucee4\ub125\ud2b8\uc640 \ud53c\uc544\ub178 \ube14\ub799 \uc778\ud14c\ub9ac\uc5b4\ub97c \uae30\ubcf8 \uc801\uc6a9\ub428\uacfc \ub3d9\uc2dc\uc5d0 2\uac00\uc9c0 \ub514\uc790\uc778\uc758 18\uc778\uce58 \uc54c\ub8e8\ubbf8\ub284 \ud720\uc744 \ubb34\uc0c1 \uc120\ud0dd\ud560 \uc218 \uc788\ub3c4\ub85d \ud558\uc600\ub2e4. \uc544\uc6b8\ub7ec I4 2.0\u2113 LPG \uc5d4\uc9c4\uc774 \uc7a5\ucc29\ub41c \ud0dd\uc2dc\uac00 \ucd9c\uc2dc\ub418\uc5c8\ub2e4."} {"question": "\uc6a9\uc218\uac00 \ub9d0\ud558\ub294 \ucd9c\uc138\uac04\uc758 \uc120\ubc95\uc740?", "paragraph": "\uc774\uc640 \uad00\ub828\ud558\uc5ec \ub300\uc2b9\ubd88\uad50\uc758 \uc8fc\uc694 \uc778\ubb3c\uc778 \uc6a9\uc218\ub294 \u300a\ub300\uc9c0\ub3c4\ub860\u300b \uc81c30\uad8c\uc5d0\uc11c \ub2e4\uc74c\uacfc \uac19\uc774 \ub9d0\ud558\uace0 \uc788\ub2e4. \uc774 \uc778\uc6a9\ubb38\uc5d0\uc11c \uc6a9\uc218\ub294 \ucd9c\uc138\uac04\uc758 \uc120\ubc95\uc73c\ub85c 37\ub3c4\ud488\uc744 \ub9d0\ud558\uace0 \uc788\ub294\ub370, \ucd9c\uc138\uac04\uc758 \uc120\ubc95\uc5d0\ub294 4\uc131\uc81c(\u56db\u8056\u8ae6) \u00b7 3\ud559(\u4e09\u5b78) \u00b7 5\uc628\uc124(\u4e94\u860a\u8aaa) \u00b7 12\uc5f0\uae30\uc124(\u5341\u4e8c\u7de3\u8d77\u8aaa) \u00b7 8\uc815\ub3c4(\u516b\u6b63\u9053) \u00b7 37\ub3c4\ud488(\u4e09\u5341\u4e03\u9053\u54c1) \u00b7 6\ubc14\ub77c\ubc00(\u516d\u6ce2\u7f85\u871c) \ub4f1\uc774 \uc788\ub2e4. \uc774\ub4e4 \uc911\uc5d0\uc11c \uc5c4\ubc00\ud788 \uc218\ud589\ubc95\ub9cc\uc744 \ub9d0\ud558\uc790\uba74, 8\uc815\ub3c4\ub294 37\ub3c4\ud488\uc758 7\ubc88\uc9f8 \uadf8\ub8f9\uc5d0 \ud574\ub2f9\ud558\uace0, 6\ubc14\ub77c\ubc00\uc740 \ucd08\uae30\ubd88\uad50\uc758 37\ub3c4\ud488\uc774 \ub300\uc2b9\ubd88\uad50\uc5d0 \uc758\ud574 \uc885\ud569\ub418\uc5b4 \uc0c8\ub85c\uc6b4 \ud615\ud0dc\ub85c \uc81c\ucd9c\ub41c \uac83\uc774\ubbc0\ub85c, \ucd9c\uc138\uac04\uc758 \uc120\ubc95\uc774\ub780 37\ub3c4\ud488\uc744 \ub9d0\ud558\ub294 \uac83\uc774\ub77c \ud560 \uc218 \uc788\ub2e4. \uc774\ub7ec\ud55c \ucde8\uc9c0\uc5d0\uc11c \uc6a9\uc218\ub294 \uc774 \uc778\uc6a9\ubb38\uc5d0\uc11c \ucd9c\uc138\uac04\uc758 \uc120\ubc95\uc73c\ub85c 37\ub3c4\ud488\uc744 \ub9d0\ud558\uace0 \uc788\ub2e4.", "answer": "37\ub3c4\ud488", "sentence": "\uc774 \uc778\uc6a9\ubb38\uc5d0\uc11c \uc6a9\uc218\ub294 \ucd9c\uc138\uac04\uc758 \uc120\ubc95\uc73c\ub85c 37\ub3c4\ud488 \uc744 \ub9d0\ud558\uace0 \uc788\ub294\ub370, \ucd9c\uc138\uac04\uc758 \uc120\ubc95\uc5d0\ub294 4\uc131\uc81c(\u56db\u8056\u8ae6) \u00b7 3\ud559(\u4e09\u5b78) \u00b7 5\uc628\uc124(\u4e94\u860a\u8aaa) \u00b7 12\uc5f0\uae30\uc124(\u5341\u4e8c\u7de3\u8d77\u8aaa) \u00b7 8\uc815\ub3c4(\u516b\u6b63\u9053) \u00b7 37\ub3c4\ud488(\u4e09\u5341\u4e03\u9053\u54c1) \u00b7 6\ubc14\ub77c\ubc00(\u516d\u6ce2\u7f85\u871c) \ub4f1\uc774 \uc788\ub2e4.", "paragraph_sentence": "\uc774\uc640 \uad00\ub828\ud558\uc5ec \ub300\uc2b9\ubd88\uad50\uc758 \uc8fc\uc694 \uc778\ubb3c\uc778 \uc6a9\uc218\ub294 \u300a\ub300\uc9c0\ub3c4\ub860\u300b \uc81c30\uad8c\uc5d0\uc11c \ub2e4\uc74c\uacfc \uac19\uc774 \ub9d0\ud558\uace0 \uc788\ub2e4. \uc774 \uc778\uc6a9\ubb38\uc5d0\uc11c \uc6a9\uc218\ub294 \ucd9c\uc138\uac04\uc758 \uc120\ubc95\uc73c\ub85c 37\ub3c4\ud488 \uc744 \ub9d0\ud558\uace0 \uc788\ub294\ub370, \ucd9c\uc138\uac04\uc758 \uc120\ubc95\uc5d0\ub294 4\uc131\uc81c(\u56db\u8056\u8ae6) \u00b7 3\ud559(\u4e09\u5b78) \u00b7 5\uc628\uc124(\u4e94\u860a\u8aaa) \u00b7 12\uc5f0\uae30\uc124(\u5341\u4e8c\u7de3\u8d77\u8aaa) \u00b7 8\uc815\ub3c4(\u516b\u6b63\u9053) \u00b7 37\ub3c4\ud488(\u4e09\u5341\u4e03\u9053\u54c1) \u00b7 6\ubc14\ub77c\ubc00(\u516d\u6ce2\u7f85\u871c) \ub4f1\uc774 \uc788\ub2e4. \uc774\ub4e4 \uc911\uc5d0\uc11c \uc5c4\ubc00\ud788 \uc218\ud589\ubc95\ub9cc\uc744 \ub9d0\ud558\uc790\uba74, 8\uc815\ub3c4\ub294 37\ub3c4\ud488\uc758 7\ubc88\uc9f8 \uadf8\ub8f9\uc5d0 \ud574\ub2f9\ud558\uace0, 6\ubc14\ub77c\ubc00\uc740 \ucd08\uae30\ubd88\uad50\uc758 37\ub3c4\ud488\uc774 \ub300\uc2b9\ubd88\uad50\uc5d0 \uc758\ud574 \uc885\ud569\ub418\uc5b4 \uc0c8\ub85c\uc6b4 \ud615\ud0dc\ub85c \uc81c\ucd9c\ub41c \uac83\uc774\ubbc0\ub85c, \ucd9c\uc138\uac04\uc758 \uc120\ubc95\uc774\ub780 37\ub3c4\ud488\uc744 \ub9d0\ud558\ub294 \uac83\uc774\ub77c \ud560 \uc218 \uc788\ub2e4. \uc774\ub7ec\ud55c \ucde8\uc9c0\uc5d0\uc11c \uc6a9\uc218\ub294 \uc774 \uc778\uc6a9\ubb38\uc5d0\uc11c \ucd9c\uc138\uac04\uc758 \uc120\ubc95\uc73c\ub85c 37\ub3c4\ud488\uc744 \ub9d0\ud558\uace0 \uc788\ub2e4.", "paragraph_answer": "\uc774\uc640 \uad00\ub828\ud558\uc5ec \ub300\uc2b9\ubd88\uad50\uc758 \uc8fc\uc694 \uc778\ubb3c\uc778 \uc6a9\uc218\ub294 \u300a\ub300\uc9c0\ub3c4\ub860\u300b \uc81c30\uad8c\uc5d0\uc11c \ub2e4\uc74c\uacfc \uac19\uc774 \ub9d0\ud558\uace0 \uc788\ub2e4. \uc774 \uc778\uc6a9\ubb38\uc5d0\uc11c \uc6a9\uc218\ub294 \ucd9c\uc138\uac04\uc758 \uc120\ubc95\uc73c\ub85c 37\ub3c4\ud488 \uc744 \ub9d0\ud558\uace0 \uc788\ub294\ub370, \ucd9c\uc138\uac04\uc758 \uc120\ubc95\uc5d0\ub294 4\uc131\uc81c(\u56db\u8056\u8ae6) \u00b7 3\ud559(\u4e09\u5b78) \u00b7 5\uc628\uc124(\u4e94\u860a\u8aaa) \u00b7 12\uc5f0\uae30\uc124(\u5341\u4e8c\u7de3\u8d77\u8aaa) \u00b7 8\uc815\ub3c4(\u516b\u6b63\u9053) \u00b7 37\ub3c4\ud488(\u4e09\u5341\u4e03\u9053\u54c1) \u00b7 6\ubc14\ub77c\ubc00(\u516d\u6ce2\u7f85\u871c) \ub4f1\uc774 \uc788\ub2e4. \uc774\ub4e4 \uc911\uc5d0\uc11c \uc5c4\ubc00\ud788 \uc218\ud589\ubc95\ub9cc\uc744 \ub9d0\ud558\uc790\uba74, 8\uc815\ub3c4\ub294 37\ub3c4\ud488\uc758 7\ubc88\uc9f8 \uadf8\ub8f9\uc5d0 \ud574\ub2f9\ud558\uace0, 6\ubc14\ub77c\ubc00\uc740 \ucd08\uae30\ubd88\uad50\uc758 37\ub3c4\ud488\uc774 \ub300\uc2b9\ubd88\uad50\uc5d0 \uc758\ud574 \uc885\ud569\ub418\uc5b4 \uc0c8\ub85c\uc6b4 \ud615\ud0dc\ub85c \uc81c\ucd9c\ub41c \uac83\uc774\ubbc0\ub85c, \ucd9c\uc138\uac04\uc758 \uc120\ubc95\uc774\ub780 37\ub3c4\ud488\uc744 \ub9d0\ud558\ub294 \uac83\uc774\ub77c \ud560 \uc218 \uc788\ub2e4. \uc774\ub7ec\ud55c \ucde8\uc9c0\uc5d0\uc11c \uc6a9\uc218\ub294 \uc774 \uc778\uc6a9\ubb38\uc5d0\uc11c \ucd9c\uc138\uac04\uc758 \uc120\ubc95\uc73c\ub85c 37\ub3c4\ud488\uc744 \ub9d0\ud558\uace0 \uc788\ub2e4.", "sentence_answer": "\uc774 \uc778\uc6a9\ubb38\uc5d0\uc11c \uc6a9\uc218\ub294 \ucd9c\uc138\uac04\uc758 \uc120\ubc95\uc73c\ub85c 37\ub3c4\ud488 \uc744 \ub9d0\ud558\uace0 \uc788\ub294\ub370, \ucd9c\uc138\uac04\uc758 \uc120\ubc95\uc5d0\ub294 4\uc131\uc81c(\u56db\u8056\u8ae6) \u00b7 3\ud559(\u4e09\u5b78) \u00b7 5\uc628\uc124(\u4e94\u860a\u8aaa) \u00b7 12\uc5f0\uae30\uc124(\u5341\u4e8c\u7de3\u8d77\u8aaa) \u00b7 8\uc815\ub3c4(\u516b\u6b63\u9053) \u00b7 37\ub3c4\ud488(\u4e09\u5341\u4e03\u9053\u54c1) \u00b7 6\ubc14\ub77c\ubc00(\u516d\u6ce2\u7f85\u871c) \ub4f1\uc774 \uc788\ub2e4.", "paragraph_id": "6596528-0-1", "paragraph_question": "question: \uc6a9\uc218\uac00 \ub9d0\ud558\ub294 \ucd9c\uc138\uac04\uc758 \uc120\ubc95\uc740?, context: \uc774\uc640 \uad00\ub828\ud558\uc5ec \ub300\uc2b9\ubd88\uad50\uc758 \uc8fc\uc694 \uc778\ubb3c\uc778 \uc6a9\uc218\ub294 \u300a\ub300\uc9c0\ub3c4\ub860\u300b \uc81c30\uad8c\uc5d0\uc11c \ub2e4\uc74c\uacfc \uac19\uc774 \ub9d0\ud558\uace0 \uc788\ub2e4. \uc774 \uc778\uc6a9\ubb38\uc5d0\uc11c \uc6a9\uc218\ub294 \ucd9c\uc138\uac04\uc758 \uc120\ubc95\uc73c\ub85c 37\ub3c4\ud488\uc744 \ub9d0\ud558\uace0 \uc788\ub294\ub370, \ucd9c\uc138\uac04\uc758 \uc120\ubc95\uc5d0\ub294 4\uc131\uc81c(\u56db\u8056\u8ae6) \u00b7 3\ud559(\u4e09\u5b78) \u00b7 5\uc628\uc124(\u4e94\u860a\u8aaa) \u00b7 12\uc5f0\uae30\uc124(\u5341\u4e8c\u7de3\u8d77\u8aaa) \u00b7 8\uc815\ub3c4(\u516b\u6b63\u9053) \u00b7 37\ub3c4\ud488(\u4e09\u5341\u4e03\u9053\u54c1) \u00b7 6\ubc14\ub77c\ubc00(\u516d\u6ce2\u7f85\u871c) \ub4f1\uc774 \uc788\ub2e4. \uc774\ub4e4 \uc911\uc5d0\uc11c \uc5c4\ubc00\ud788 \uc218\ud589\ubc95\ub9cc\uc744 \ub9d0\ud558\uc790\uba74, 8\uc815\ub3c4\ub294 37\ub3c4\ud488\uc758 7\ubc88\uc9f8 \uadf8\ub8f9\uc5d0 \ud574\ub2f9\ud558\uace0, 6\ubc14\ub77c\ubc00\uc740 \ucd08\uae30\ubd88\uad50\uc758 37\ub3c4\ud488\uc774 \ub300\uc2b9\ubd88\uad50\uc5d0 \uc758\ud574 \uc885\ud569\ub418\uc5b4 \uc0c8\ub85c\uc6b4 \ud615\ud0dc\ub85c \uc81c\ucd9c\ub41c \uac83\uc774\ubbc0\ub85c, \ucd9c\uc138\uac04\uc758 \uc120\ubc95\uc774\ub780 37\ub3c4\ud488\uc744 \ub9d0\ud558\ub294 \uac83\uc774\ub77c \ud560 \uc218 \uc788\ub2e4. \uc774\ub7ec\ud55c \ucde8\uc9c0\uc5d0\uc11c \uc6a9\uc218\ub294 \uc774 \uc778\uc6a9\ubb38\uc5d0\uc11c \ucd9c\uc138\uac04\uc758 \uc120\ubc95\uc73c\ub85c 37\ub3c4\ud488\uc744 \ub9d0\ud558\uace0 \uc788\ub2e4."} {"question": "\ubbf8\uc18c\uacf5\uc704 \ucc38\uad00 \ubb38\uc548 \uc791\uc131 \uc911 \uae40\uc900\uc5f0\uc774 \ubb38\uc81c\uc0bc\uc740 \ub2e8\uc5b4\ub294?", "paragraph": "1946\ub144 3\uc6d4 \ubbf8\uc18c\uacf5\uc704 \ucc38\uad00 \ud55c\uc778\uc758 \ub300\ud45c\uc790\ub85c \uc120\ucd9c\ub410\ub2e4. \ubbf8\uc18c\uacf5\uc704 \ucc38\uad00 \ubb38\uc548 \uc791\uc131 \uc911 \uae40\uc900\uc5f0(\u91d1\u4fca\u6df5)\uc758 \ubc29\ubb38\uc744 \ubc1b\uc558\ub2e4. \uadf8\ub7ec\ub098 \ud0c0\uc774\ud504\ub85c \uc2e0\ud0c1\ud1b5\uce58 \uad00\ub828 \ubb38\uc11c\ub97c \uc791\uc131\ud558\ub358 \uc911 \ucc2c\uc131\uc774\ub77c\ub294 \ub2e8\uc5b4\ub97c \ubcf4\uace0 \uae40\uc900\uc5f0(\u91d1\u4fca\u6df5)\uc740 \uc774\ub97c \ubb38\uc81c\uc0bc\uae30\ub3c4 \ud588\ub2e4. \ub610\ud55c \ubbf8\uad70\uc815\uccad\uc758 \uc0ac\ub839\uad00 \uc874 \ud558\uc9c0 \uc7a5\uad70\uc740 \uc774\uc2b9\ub9cc\uc758 \ub3d9\uc758\ub97c \uc5bb\uc5b4 \uae40\uaddc\uc2dd\uc744 \uc88c\uc6b0\ud569\uc791\uc704\uc6d0\ud68c\uc758 \uc704\uc6d0\uc7a5\uc73c\ub85c \uc784\uba85\ud558\uc600\uc73c\uba70, \uacfc\ub3c4\uc785\ubc95\uc758\uc6d0\uc758 \uc758\uc7a5\uc774 \ub418\uac8c \ud558\uc600\ub2e4. 1946\ub144 2\uc6d4 14\uc77c \ubbfc\uc871\ud601\uba85\ub2f9 \ub2f9\uc218\uc9c1\uc744 \uc0ac\ud1f4\ud558\uc600\ub2e4. \uae40\uc131\uc219, \uc7a5\uac74\uc0c1, \uae40\uc6d0\ubd09 \ub4f1\uc774 \uc784\uc815\uc744 \ub5a0\ub098 \ubbfc\uc871\uc8fc\uc758\ubbfc\uc8fc\uc804\uc120\uc5d0 \uac00\ub2f4\ud558\uc790, \ubbfc\uc871\ud601\uba85\ub2f9\uc744 \ud0c8\ub2f9\ud558\uc600\uc73c\uba70 \uc5ec\uc6b4\ud615\uacfc \ud568\uaed8 \uc88c\uc6b0\ud569\uc791\uc6b4\ub3d9\uc744 \uc9c4\ud589\ud558\uc600\ub2e4. \uc88c\uc6b0\ud569\uc791 \ucc38\uc5ec \ubc30\uacbd\uc740 \ubbf8\uad6d \uad6d\ubb34\uc131\uc758 \uc694\uad6c\ub85c \uc2e4\uc2dc\ub41c \uac83\uc5d0\uc11c \ube44\ub86f\ub418\uc5c8\ub2e4.", "answer": "\ucc2c\uc131", "sentence": "\uadf8\ub7ec\ub098 \ud0c0\uc774\ud504\ub85c \uc2e0\ud0c1\ud1b5\uce58 \uad00\ub828 \ubb38\uc11c\ub97c \uc791\uc131\ud558\ub358 \uc911 \ucc2c\uc131 \uc774\ub77c\ub294 \ub2e8\uc5b4\ub97c \ubcf4\uace0 \uae40\uc900\uc5f0(\u91d1\u4fca\u6df5)\uc740 \uc774\ub97c \ubb38\uc81c\uc0bc\uae30\ub3c4 \ud588\ub2e4.", "paragraph_sentence": "1946\ub144 3\uc6d4 \ubbf8\uc18c\uacf5\uc704 \ucc38\uad00 \ud55c\uc778\uc758 \ub300\ud45c\uc790\ub85c \uc120\ucd9c\ub410\ub2e4. \ubbf8\uc18c\uacf5\uc704 \ucc38\uad00 \ubb38\uc548 \uc791\uc131 \uc911 \uae40\uc900\uc5f0(\u91d1\u4fca\u6df5)\uc758 \ubc29\ubb38\uc744 \ubc1b\uc558\ub2e4. \uadf8\ub7ec\ub098 \ud0c0\uc774\ud504\ub85c \uc2e0\ud0c1\ud1b5\uce58 \uad00\ub828 \ubb38\uc11c\ub97c \uc791\uc131\ud558\ub358 \uc911 \ucc2c\uc131 \uc774\ub77c\ub294 \ub2e8\uc5b4\ub97c \ubcf4\uace0 \uae40\uc900\uc5f0(\u91d1\u4fca\u6df5)\uc740 \uc774\ub97c \ubb38\uc81c\uc0bc\uae30\ub3c4 \ud588\ub2e4. \ub610\ud55c \ubbf8\uad70\uc815\uccad\uc758 \uc0ac\ub839\uad00 \uc874 \ud558\uc9c0 \uc7a5\uad70\uc740 \uc774\uc2b9\ub9cc\uc758 \ub3d9\uc758\ub97c \uc5bb\uc5b4 \uae40\uaddc\uc2dd\uc744 \uc88c\uc6b0\ud569\uc791\uc704\uc6d0\ud68c\uc758 \uc704\uc6d0\uc7a5\uc73c\ub85c \uc784\uba85\ud558\uc600\uc73c\uba70, \uacfc\ub3c4\uc785\ubc95\uc758\uc6d0\uc758 \uc758\uc7a5\uc774 \ub418\uac8c \ud558\uc600\ub2e4. 1946\ub144 2\uc6d4 14\uc77c \ubbfc\uc871\ud601\uba85\ub2f9 \ub2f9\uc218\uc9c1\uc744 \uc0ac\ud1f4\ud558\uc600\ub2e4. \uae40\uc131\uc219, \uc7a5\uac74\uc0c1, \uae40\uc6d0\ubd09 \ub4f1\uc774 \uc784\uc815\uc744 \ub5a0\ub098 \ubbfc\uc871\uc8fc\uc758\ubbfc\uc8fc\uc804\uc120\uc5d0 \uac00\ub2f4\ud558\uc790, \ubbfc\uc871\ud601\uba85\ub2f9\uc744 \ud0c8\ub2f9\ud558\uc600\uc73c\uba70 \uc5ec\uc6b4\ud615\uacfc \ud568\uaed8 \uc88c\uc6b0\ud569\uc791\uc6b4\ub3d9\uc744 \uc9c4\ud589\ud558\uc600\ub2e4. \uc88c\uc6b0\ud569\uc791 \ucc38\uc5ec \ubc30\uacbd\uc740 \ubbf8\uad6d \uad6d\ubb34\uc131\uc758 \uc694\uad6c\ub85c \uc2e4\uc2dc\ub41c \uac83\uc5d0\uc11c \ube44\ub86f\ub418\uc5c8\ub2e4.", "paragraph_answer": "1946\ub144 3\uc6d4 \ubbf8\uc18c\uacf5\uc704 \ucc38\uad00 \ud55c\uc778\uc758 \ub300\ud45c\uc790\ub85c \uc120\ucd9c\ub410\ub2e4. \ubbf8\uc18c\uacf5\uc704 \ucc38\uad00 \ubb38\uc548 \uc791\uc131 \uc911 \uae40\uc900\uc5f0(\u91d1\u4fca\u6df5)\uc758 \ubc29\ubb38\uc744 \ubc1b\uc558\ub2e4. \uadf8\ub7ec\ub098 \ud0c0\uc774\ud504\ub85c \uc2e0\ud0c1\ud1b5\uce58 \uad00\ub828 \ubb38\uc11c\ub97c \uc791\uc131\ud558\ub358 \uc911 \ucc2c\uc131 \uc774\ub77c\ub294 \ub2e8\uc5b4\ub97c \ubcf4\uace0 \uae40\uc900\uc5f0(\u91d1\u4fca\u6df5)\uc740 \uc774\ub97c \ubb38\uc81c\uc0bc\uae30\ub3c4 \ud588\ub2e4. \ub610\ud55c \ubbf8\uad70\uc815\uccad\uc758 \uc0ac\ub839\uad00 \uc874 \ud558\uc9c0 \uc7a5\uad70\uc740 \uc774\uc2b9\ub9cc\uc758 \ub3d9\uc758\ub97c \uc5bb\uc5b4 \uae40\uaddc\uc2dd\uc744 \uc88c\uc6b0\ud569\uc791\uc704\uc6d0\ud68c\uc758 \uc704\uc6d0\uc7a5\uc73c\ub85c \uc784\uba85\ud558\uc600\uc73c\uba70, \uacfc\ub3c4\uc785\ubc95\uc758\uc6d0\uc758 \uc758\uc7a5\uc774 \ub418\uac8c \ud558\uc600\ub2e4. 1946\ub144 2\uc6d4 14\uc77c \ubbfc\uc871\ud601\uba85\ub2f9 \ub2f9\uc218\uc9c1\uc744 \uc0ac\ud1f4\ud558\uc600\ub2e4. \uae40\uc131\uc219, \uc7a5\uac74\uc0c1, \uae40\uc6d0\ubd09 \ub4f1\uc774 \uc784\uc815\uc744 \ub5a0\ub098 \ubbfc\uc871\uc8fc\uc758\ubbfc\uc8fc\uc804\uc120\uc5d0 \uac00\ub2f4\ud558\uc790, \ubbfc\uc871\ud601\uba85\ub2f9\uc744 \ud0c8\ub2f9\ud558\uc600\uc73c\uba70 \uc5ec\uc6b4\ud615\uacfc \ud568\uaed8 \uc88c\uc6b0\ud569\uc791\uc6b4\ub3d9\uc744 \uc9c4\ud589\ud558\uc600\ub2e4. \uc88c\uc6b0\ud569\uc791 \ucc38\uc5ec \ubc30\uacbd\uc740 \ubbf8\uad6d \uad6d\ubb34\uc131\uc758 \uc694\uad6c\ub85c \uc2e4\uc2dc\ub41c \uac83\uc5d0\uc11c \ube44\ub86f\ub418\uc5c8\ub2e4.", "sentence_answer": "\uadf8\ub7ec\ub098 \ud0c0\uc774\ud504\ub85c \uc2e0\ud0c1\ud1b5\uce58 \uad00\ub828 \ubb38\uc11c\ub97c \uc791\uc131\ud558\ub358 \uc911 \ucc2c\uc131 \uc774\ub77c\ub294 \ub2e8\uc5b4\ub97c \ubcf4\uace0 \uae40\uc900\uc5f0(\u91d1\u4fca\u6df5)\uc740 \uc774\ub97c \ubb38\uc81c\uc0bc\uae30\ub3c4 \ud588\ub2e4.", "paragraph_id": "6533880-27-1", "paragraph_question": "question: \ubbf8\uc18c\uacf5\uc704 \ucc38\uad00 \ubb38\uc548 \uc791\uc131 \uc911 \uae40\uc900\uc5f0\uc774 \ubb38\uc81c\uc0bc\uc740 \ub2e8\uc5b4\ub294?, context: 1946\ub144 3\uc6d4 \ubbf8\uc18c\uacf5\uc704 \ucc38\uad00 \ud55c\uc778\uc758 \ub300\ud45c\uc790\ub85c \uc120\ucd9c\ub410\ub2e4. \ubbf8\uc18c\uacf5\uc704 \ucc38\uad00 \ubb38\uc548 \uc791\uc131 \uc911 \uae40\uc900\uc5f0(\u91d1\u4fca\u6df5)\uc758 \ubc29\ubb38\uc744 \ubc1b\uc558\ub2e4. \uadf8\ub7ec\ub098 \ud0c0\uc774\ud504\ub85c \uc2e0\ud0c1\ud1b5\uce58 \uad00\ub828 \ubb38\uc11c\ub97c \uc791\uc131\ud558\ub358 \uc911 \ucc2c\uc131\uc774\ub77c\ub294 \ub2e8\uc5b4\ub97c \ubcf4\uace0 \uae40\uc900\uc5f0(\u91d1\u4fca\u6df5)\uc740 \uc774\ub97c \ubb38\uc81c\uc0bc\uae30\ub3c4 \ud588\ub2e4. \ub610\ud55c \ubbf8\uad70\uc815\uccad\uc758 \uc0ac\ub839\uad00 \uc874 \ud558\uc9c0 \uc7a5\uad70\uc740 \uc774\uc2b9\ub9cc\uc758 \ub3d9\uc758\ub97c \uc5bb\uc5b4 \uae40\uaddc\uc2dd\uc744 \uc88c\uc6b0\ud569\uc791\uc704\uc6d0\ud68c\uc758 \uc704\uc6d0\uc7a5\uc73c\ub85c \uc784\uba85\ud558\uc600\uc73c\uba70, \uacfc\ub3c4\uc785\ubc95\uc758\uc6d0\uc758 \uc758\uc7a5\uc774 \ub418\uac8c \ud558\uc600\ub2e4. 1946\ub144 2\uc6d4 14\uc77c \ubbfc\uc871\ud601\uba85\ub2f9 \ub2f9\uc218\uc9c1\uc744 \uc0ac\ud1f4\ud558\uc600\ub2e4. \uae40\uc131\uc219, \uc7a5\uac74\uc0c1, \uae40\uc6d0\ubd09 \ub4f1\uc774 \uc784\uc815\uc744 \ub5a0\ub098 \ubbfc\uc871\uc8fc\uc758\ubbfc\uc8fc\uc804\uc120\uc5d0 \uac00\ub2f4\ud558\uc790, \ubbfc\uc871\ud601\uba85\ub2f9\uc744 \ud0c8\ub2f9\ud558\uc600\uc73c\uba70 \uc5ec\uc6b4\ud615\uacfc \ud568\uaed8 \uc88c\uc6b0\ud569\uc791\uc6b4\ub3d9\uc744 \uc9c4\ud589\ud558\uc600\ub2e4. \uc88c\uc6b0\ud569\uc791 \ucc38\uc5ec \ubc30\uacbd\uc740 \ubbf8\uad6d \uad6d\ubb34\uc131\uc758 \uc694\uad6c\ub85c \uc2e4\uc2dc\ub41c \uac83\uc5d0\uc11c \ube44\ub86f\ub418\uc5c8\ub2e4."} {"question": "1649\ub144 5\uc6d4 Huge Peters\uac00 \ub9b4\ubc88\uc744 \ub9cc\ub0ac\uc744 \ub54c \uadf8\ub294 \uc9c0\uae08\uc758 \uc815\ubd80\uc5d0 1\ub144 \uc788\ub294\uac83\ubcf4\ub2e4 \uc655 \uc544\ub798\uc5d0\uc11c \uba87 \ub144 \uc788\ub294 \uac83\uc774 \ub0ab\ub2e4\ub77c \ub9d0\ud588\ub098?", "paragraph": "\u2018Huge Peters\u2019\uac00 1649\ub144 5\uc6d4 25\uc77c \ud0d1 \uc548\uc5d0\uc11c \uc874 \ub9b4\ubc88\uc744 \ub9cc\ub0a0 \ub2f9\uc2dc, \ub9b4\ubc88\uc740 \uadf8\uc5d0\uac8c \uc790\uc2e0\uc740 \ud604 \uc815\uad8c\uc5d0 1\ub144 \uc788\uae30\ubcf4\ub2e4\ub294 \uc655\uc758 \ubc11\uc5d0 7\ub144 \ub3d9\uc548 \uc788\ub294 \uac83\uc774 \ub098\uc744 \uac83\uc774\ub77c\uace0 \ud588\uace0, \ud604 \uc815\uad8c\uc774 \uc5ec\uc804\ud788 \uc555\uc81c\uc801\uc774\ub77c\uba74, \uc0ac\ub78c\ub4e4\uc740 \u2018\ucc30\uc2a4 \uc655\uc790\u2019\uc5d0 \ub9de\uc11c \uc2f8\uc6cc\uc57c \ud55c\ub2e4\uace0 \uc8fc\uc7a5\ud588\ub2e4. 3\ub2ec \ud6c4 \u2018Outcry of the Apprentices to the Soldiers\u2019\uc5d0\uc11c \ub9b4\ubc88\uc740 \uacac\uc2b5\uc0dd\ub4e4\uacfc \uad70\uc778\ub4e4\uc740 \ucf54\uba3c\uc6f0\uc2a4\uc758 \ud5cc\ubc95\uc758 \uae30\ubcf8\uad8c\uacfc \uc758\ud68c\uc5d0\uc11c\uc758 \uc0ac\ub78c\ub4e4\uc758 \uad8c\ub9ac\ub97c \uc815\ubd80\uac00 \ubc31\uc131\ub4e4\uc744 \ubc18\ud558\uc9c0 \uc54a\uac8c \uaddc\uc81c\ud558\ub294 \uc2dd\uc73c\ub85c \uc2f8\uc6cc\uc57c \ud55c\ub2e4\uace0 \uc8fc\uc7a5\ud588\ub2e4. 1648\ub144 \ube0c\ub85c\ub4dc\uc6e8\uc774 \ud68c\uc758\uc774\ud6c4\uc5d0 \uc218\ud3c9\ud30c\ub4e4\uc774 \uc655\ub2f9\ud30c\ub4e4\uacfc \uc0c8\ub85c\uc6b4 \uacf5\ud654\uad6d\uc744 \ud0c0\ub3c4\ud558\uae30 \uc704\ud574 \uc74c\ubaa8\ub97c \uafb8\ubbfc\ub2e4\ub294 \uc18c\ubb38\ub4e4\uc774 \ub3cc\uc558\ub2e4. \uc625\uc2a4\ud37c\ub4dc \ubc18\ub780 \uc911\uc5d0 \uc774\uac83\uc740 \uc758\ud68c\uac00 \u2018Tower of London to Lord Cottington\u2019\uc5d0\uc11c \ucc30\uc2a4 2\uc138\uc758 \ucd94\ubc29\uacfc \uad00\ub828\ub41c \uace0\ubb38\uc778 \uc655\ub2f9\ud30c \uc778\uc9c8\ub85c\ubd80\ud130 \ud3b8\uc9c0\ub97c \ud68d\ub4dd\ud588\uc744 \ub54c \ud655\uc778\ub418\uc5c8\ub2e4. \uadf8 \ud3b8\uc9c0\uc5d0\uc11c\ub294 \uc655\ub2f9\ud30c\ub4e4\uc740 \ucc30\uc2a4\uac00 \uc655\uc704\ub97c \ub418\ucc3e\uac8c \ud558\uae30 \uc704\ud574\uc11c \uc218\ud3c9\ud30c\uc5d0 \uc6d0\uc870\ud574\uc57c\ud55c\ub2e4\ub294 \ub0b4\uc6a9\uc774\uc5c8\ub2e4. \uc774 \uc99d\uac70\ub85c \uc758\ud68c\ub294 \ub9b4\ubc88\uc744 \ub192\uc740 \uc218\uc704\uc758 \ubc18\uc5ed\uc8c4\ub85c \ucc45\uc815\ud558\ub294 \uc81c\uc758\ub97c \ubc1b\uc544\ub4e4\uc600\ub2e4. \ucc30\uc2a4 1\uc138\ub97c \uc2ec\ud310\ud588\ub358 \ubc95\uc6d0\uc744 \uc0ac\uc6a9\ud558\uba74\uc11c \uc655\uc758 \uc7ac\ud310\ucc98\ub7fc \uc9c0\uba85\ub41c \uc704\uc6d0\ub4e4\uc774 \ud615\ubcc4\uc744 \uacb0\uc815\ud558\ub824\ud588\uc73c\ub098 \uadf8\uc640 \ub2e4\ub974\uac8c 12\uba85\uc758 \ubc30\uc2ec\uc6d0\ub4e4\uc774 \ubb34\uc8c4\ub77c\uace0 \uacb0\uc815\ud588\ub2e4. \ubc30\uc2ec\uc6d0\ub4e4\uc774 \uadf8\uac00 \ubb34\uc8c4\uc784\uc744 \uc120\uace0\ud588\uc744 \ub54c, \uc0ac\ub78c\ub4e4\uc740 \uadf8\ub4e4\uc758 \uc2b9\uc778\uc5d0 \ub300\ud574 \uc5c4\uccad\ub098\uac8c \uc18c\ub9ac \uc9c8\ub800\uace0, \uc774 \uc7ac\ud310\uc774 \uacf5\uc2dd\uc801\uc73c\ub85c \ub05d\ub098\uae30\uae4c\uc9c0 1\uc2dc\uac04 \ubc18\uc774 \uac78\ub838\ub2e4. \ub9b4\ubc88\uc740 \uc989\uc2dc \ud480\ub824\ub098\uc9c0 \uc54a\uc558\uace0 \ub300\uc911\ub4e4\uc774\ub098 \uc758\ud68c\uc758 \uce5c\uad6c\ub4e4\uc774 \uadf8\uc758 \uc11d\ubc29\uc744 \ud655\ubcf4\ud558\uae30\uae4c\uc9c0 2\uc8fc\uac00 \uac78\ub838\ub2e4. \uc758\ud68c\uc758 \uc0ac\ub78c\ub4e4 \uc911 \uba87\uba87\uc740 \ub9b4\ubc88\uc758 \uc11d\ubc29\uc5d0 \ub300\ud574 \ubd88\ub9cc\uc744 \ud488\uc5c8\uc9c0\ub9cc \uc758\ud68c\ub294 \uc218\ud3c9\ud30c\uc5d0 \ub300\ud55c \ubc18\ub300\uac00 \uc0dd\uae30\ub3c4\ub85d \uc5b5\uc555\ud558\ub294\ub370 \uc131\uacf5\ud588\ub2e4. \uc218\ud3c9\ud30c\ub4e4\uc740 \uad6d\uac00\uc640 \uad70\uc778\ub4e4\uacfc \ubc18\ub780\uc744 \uc77c\uc73c\ud0a4\ub824\ub294 \uc2dc\ub3c4\ub97c \uc911\uc9c0\ud558\uac8c \ub418\uc5c8\uace0 \uc74c\ubaa8\ub3c4 \ube44\ubc00\ub9ac\uc5d0 \uc9dc\uae30 \uc2dc\uc791\ud588\ub2e4.", "answer": "7\ub144", "sentence": "\u2018Huge Peters\u2019\uac00 1649\ub144 5\uc6d4 25\uc77c \ud0d1 \uc548\uc5d0\uc11c \uc874 \ub9b4\ubc88\uc744 \ub9cc\ub0a0 \ub2f9\uc2dc, \ub9b4\ubc88\uc740 \uadf8\uc5d0\uac8c \uc790\uc2e0\uc740 \ud604 \uc815\uad8c\uc5d0 1\ub144 \uc788\uae30\ubcf4\ub2e4\ub294 \uc655\uc758 \ubc11\uc5d0 7\ub144 \ub3d9\uc548 \uc788\ub294 \uac83\uc774 \ub098\uc744 \uac83\uc774\ub77c\uace0 \ud588\uace0, \ud604 \uc815\uad8c\uc774 \uc5ec\uc804\ud788 \uc555\uc81c\uc801\uc774\ub77c\uba74, \uc0ac\ub78c\ub4e4\uc740 \u2018\ucc30\uc2a4 \uc655\uc790\u2019\uc5d0 \ub9de\uc11c \uc2f8\uc6cc\uc57c \ud55c\ub2e4\uace0 \uc8fc\uc7a5\ud588\ub2e4.", "paragraph_sentence": " \u2018Huge Peters\u2019\uac00 1649\ub144 5\uc6d4 25\uc77c \ud0d1 \uc548\uc5d0\uc11c \uc874 \ub9b4\ubc88\uc744 \ub9cc\ub0a0 \ub2f9\uc2dc, \ub9b4\ubc88\uc740 \uadf8\uc5d0\uac8c \uc790\uc2e0\uc740 \ud604 \uc815\uad8c\uc5d0 1\ub144 \uc788\uae30\ubcf4\ub2e4\ub294 \uc655\uc758 \ubc11\uc5d0 7\ub144 \ub3d9\uc548 \uc788\ub294 \uac83\uc774 \ub098\uc744 \uac83\uc774\ub77c\uace0 \ud588\uace0, \ud604 \uc815\uad8c\uc774 \uc5ec\uc804\ud788 \uc555\uc81c\uc801\uc774\ub77c\uba74, \uc0ac\ub78c\ub4e4\uc740 \u2018\ucc30\uc2a4 \uc655\uc790\u2019\uc5d0 \ub9de\uc11c \uc2f8\uc6cc\uc57c \ud55c\ub2e4\uace0 \uc8fc\uc7a5\ud588\ub2e4. 3\ub2ec \ud6c4 \u2018Outcry of the Apprentices to the Soldiers\u2019\uc5d0\uc11c \ub9b4\ubc88\uc740 \uacac\uc2b5\uc0dd\ub4e4\uacfc \uad70\uc778\ub4e4\uc740 \ucf54\uba3c\uc6f0\uc2a4\uc758 \ud5cc\ubc95\uc758 \uae30\ubcf8\uad8c\uacfc \uc758\ud68c\uc5d0\uc11c\uc758 \uc0ac\ub78c\ub4e4\uc758 \uad8c\ub9ac\ub97c \uc815\ubd80\uac00 \ubc31\uc131\ub4e4\uc744 \ubc18\ud558\uc9c0 \uc54a\uac8c \uaddc\uc81c\ud558\ub294 \uc2dd\uc73c\ub85c \uc2f8\uc6cc\uc57c \ud55c\ub2e4\uace0 \uc8fc\uc7a5\ud588\ub2e4. 1648\ub144 \ube0c\ub85c\ub4dc\uc6e8\uc774 \ud68c\uc758\uc774\ud6c4\uc5d0 \uc218\ud3c9\ud30c\ub4e4\uc774 \uc655\ub2f9\ud30c\ub4e4\uacfc \uc0c8\ub85c\uc6b4 \uacf5\ud654\uad6d\uc744 \ud0c0\ub3c4\ud558\uae30 \uc704\ud574 \uc74c\ubaa8\ub97c \uafb8\ubbfc\ub2e4\ub294 \uc18c\ubb38\ub4e4\uc774 \ub3cc\uc558\ub2e4. \uc625\uc2a4\ud37c\ub4dc \ubc18\ub780 \uc911\uc5d0 \uc774\uac83\uc740 \uc758\ud68c\uac00 \u2018Tower of London to Lord Cottington\u2019\uc5d0\uc11c \ucc30\uc2a4 2\uc138\uc758 \ucd94\ubc29\uacfc \uad00\ub828\ub41c \uace0\ubb38\uc778 \uc655\ub2f9\ud30c \uc778\uc9c8\ub85c\ubd80\ud130 \ud3b8\uc9c0\ub97c \ud68d\ub4dd\ud588\uc744 \ub54c \ud655\uc778\ub418\uc5c8\ub2e4. \uadf8 \ud3b8\uc9c0\uc5d0\uc11c\ub294 \uc655\ub2f9\ud30c\ub4e4\uc740 \ucc30\uc2a4\uac00 \uc655\uc704\ub97c \ub418\ucc3e\uac8c \ud558\uae30 \uc704\ud574\uc11c \uc218\ud3c9\ud30c\uc5d0 \uc6d0\uc870\ud574\uc57c\ud55c\ub2e4\ub294 \ub0b4\uc6a9\uc774\uc5c8\ub2e4. \uc774 \uc99d\uac70\ub85c \uc758\ud68c\ub294 \ub9b4\ubc88\uc744 \ub192\uc740 \uc218\uc704\uc758 \ubc18\uc5ed\uc8c4\ub85c \ucc45\uc815\ud558\ub294 \uc81c\uc758\ub97c \ubc1b\uc544\ub4e4\uc600\ub2e4. \ucc30\uc2a4 1\uc138\ub97c \uc2ec\ud310\ud588\ub358 \ubc95\uc6d0\uc744 \uc0ac\uc6a9\ud558\uba74\uc11c \uc655\uc758 \uc7ac\ud310\ucc98\ub7fc \uc9c0\uba85\ub41c \uc704\uc6d0\ub4e4\uc774 \ud615\ubcc4\uc744 \uacb0\uc815\ud558\ub824\ud588\uc73c\ub098 \uadf8\uc640 \ub2e4\ub974\uac8c 12\uba85\uc758 \ubc30\uc2ec\uc6d0\ub4e4\uc774 \ubb34\uc8c4\ub77c\uace0 \uacb0\uc815\ud588\ub2e4. \ubc30\uc2ec\uc6d0\ub4e4\uc774 \uadf8\uac00 \ubb34\uc8c4\uc784\uc744 \uc120\uace0\ud588\uc744 \ub54c, \uc0ac\ub78c\ub4e4\uc740 \uadf8\ub4e4\uc758 \uc2b9\uc778\uc5d0 \ub300\ud574 \uc5c4\uccad\ub098\uac8c \uc18c\ub9ac \uc9c8\ub800\uace0, \uc774 \uc7ac\ud310\uc774 \uacf5\uc2dd\uc801\uc73c\ub85c \ub05d\ub098\uae30\uae4c\uc9c0 1\uc2dc\uac04 \ubc18\uc774 \uac78\ub838\ub2e4. \ub9b4\ubc88\uc740 \uc989\uc2dc \ud480\ub824\ub098\uc9c0 \uc54a\uc558\uace0 \ub300\uc911\ub4e4\uc774\ub098 \uc758\ud68c\uc758 \uce5c\uad6c\ub4e4\uc774 \uadf8\uc758 \uc11d\ubc29\uc744 \ud655\ubcf4\ud558\uae30\uae4c\uc9c0 2\uc8fc\uac00 \uac78\ub838\ub2e4. \uc758\ud68c\uc758 \uc0ac\ub78c\ub4e4 \uc911 \uba87\uba87\uc740 \ub9b4\ubc88\uc758 \uc11d\ubc29\uc5d0 \ub300\ud574 \ubd88\ub9cc\uc744 \ud488\uc5c8\uc9c0\ub9cc \uc758\ud68c\ub294 \uc218\ud3c9\ud30c\uc5d0 \ub300\ud55c \ubc18\ub300\uac00 \uc0dd\uae30\ub3c4\ub85d \uc5b5\uc555\ud558\ub294\ub370 \uc131\uacf5\ud588\ub2e4. \uc218\ud3c9\ud30c\ub4e4\uc740 \uad6d\uac00\uc640 \uad70\uc778\ub4e4\uacfc \ubc18\ub780\uc744 \uc77c\uc73c\ud0a4\ub824\ub294 \uc2dc\ub3c4\ub97c \uc911\uc9c0\ud558\uac8c \ub418\uc5c8\uace0 \uc74c\ubaa8\ub3c4 \ube44\ubc00\ub9ac\uc5d0 \uc9dc\uae30 \uc2dc\uc791\ud588\ub2e4.", "paragraph_answer": "\u2018Huge Peters\u2019\uac00 1649\ub144 5\uc6d4 25\uc77c \ud0d1 \uc548\uc5d0\uc11c \uc874 \ub9b4\ubc88\uc744 \ub9cc\ub0a0 \ub2f9\uc2dc, \ub9b4\ubc88\uc740 \uadf8\uc5d0\uac8c \uc790\uc2e0\uc740 \ud604 \uc815\uad8c\uc5d0 1\ub144 \uc788\uae30\ubcf4\ub2e4\ub294 \uc655\uc758 \ubc11\uc5d0 7\ub144 \ub3d9\uc548 \uc788\ub294 \uac83\uc774 \ub098\uc744 \uac83\uc774\ub77c\uace0 \ud588\uace0, \ud604 \uc815\uad8c\uc774 \uc5ec\uc804\ud788 \uc555\uc81c\uc801\uc774\ub77c\uba74, \uc0ac\ub78c\ub4e4\uc740 \u2018\ucc30\uc2a4 \uc655\uc790\u2019\uc5d0 \ub9de\uc11c \uc2f8\uc6cc\uc57c \ud55c\ub2e4\uace0 \uc8fc\uc7a5\ud588\ub2e4. 3\ub2ec \ud6c4 \u2018Outcry of the Apprentices to the Soldiers\u2019\uc5d0\uc11c \ub9b4\ubc88\uc740 \uacac\uc2b5\uc0dd\ub4e4\uacfc \uad70\uc778\ub4e4\uc740 \ucf54\uba3c\uc6f0\uc2a4\uc758 \ud5cc\ubc95\uc758 \uae30\ubcf8\uad8c\uacfc \uc758\ud68c\uc5d0\uc11c\uc758 \uc0ac\ub78c\ub4e4\uc758 \uad8c\ub9ac\ub97c \uc815\ubd80\uac00 \ubc31\uc131\ub4e4\uc744 \ubc18\ud558\uc9c0 \uc54a\uac8c \uaddc\uc81c\ud558\ub294 \uc2dd\uc73c\ub85c \uc2f8\uc6cc\uc57c \ud55c\ub2e4\uace0 \uc8fc\uc7a5\ud588\ub2e4. 1648\ub144 \ube0c\ub85c\ub4dc\uc6e8\uc774 \ud68c\uc758\uc774\ud6c4\uc5d0 \uc218\ud3c9\ud30c\ub4e4\uc774 \uc655\ub2f9\ud30c\ub4e4\uacfc \uc0c8\ub85c\uc6b4 \uacf5\ud654\uad6d\uc744 \ud0c0\ub3c4\ud558\uae30 \uc704\ud574 \uc74c\ubaa8\ub97c \uafb8\ubbfc\ub2e4\ub294 \uc18c\ubb38\ub4e4\uc774 \ub3cc\uc558\ub2e4. \uc625\uc2a4\ud37c\ub4dc \ubc18\ub780 \uc911\uc5d0 \uc774\uac83\uc740 \uc758\ud68c\uac00 \u2018Tower of London to Lord Cottington\u2019\uc5d0\uc11c \ucc30\uc2a4 2\uc138\uc758 \ucd94\ubc29\uacfc \uad00\ub828\ub41c \uace0\ubb38\uc778 \uc655\ub2f9\ud30c \uc778\uc9c8\ub85c\ubd80\ud130 \ud3b8\uc9c0\ub97c \ud68d\ub4dd\ud588\uc744 \ub54c \ud655\uc778\ub418\uc5c8\ub2e4. \uadf8 \ud3b8\uc9c0\uc5d0\uc11c\ub294 \uc655\ub2f9\ud30c\ub4e4\uc740 \ucc30\uc2a4\uac00 \uc655\uc704\ub97c \ub418\ucc3e\uac8c \ud558\uae30 \uc704\ud574\uc11c \uc218\ud3c9\ud30c\uc5d0 \uc6d0\uc870\ud574\uc57c\ud55c\ub2e4\ub294 \ub0b4\uc6a9\uc774\uc5c8\ub2e4. \uc774 \uc99d\uac70\ub85c \uc758\ud68c\ub294 \ub9b4\ubc88\uc744 \ub192\uc740 \uc218\uc704\uc758 \ubc18\uc5ed\uc8c4\ub85c \ucc45\uc815\ud558\ub294 \uc81c\uc758\ub97c \ubc1b\uc544\ub4e4\uc600\ub2e4. \ucc30\uc2a4 1\uc138\ub97c \uc2ec\ud310\ud588\ub358 \ubc95\uc6d0\uc744 \uc0ac\uc6a9\ud558\uba74\uc11c \uc655\uc758 \uc7ac\ud310\ucc98\ub7fc \uc9c0\uba85\ub41c \uc704\uc6d0\ub4e4\uc774 \ud615\ubcc4\uc744 \uacb0\uc815\ud558\ub824\ud588\uc73c\ub098 \uadf8\uc640 \ub2e4\ub974\uac8c 12\uba85\uc758 \ubc30\uc2ec\uc6d0\ub4e4\uc774 \ubb34\uc8c4\ub77c\uace0 \uacb0\uc815\ud588\ub2e4. \ubc30\uc2ec\uc6d0\ub4e4\uc774 \uadf8\uac00 \ubb34\uc8c4\uc784\uc744 \uc120\uace0\ud588\uc744 \ub54c, \uc0ac\ub78c\ub4e4\uc740 \uadf8\ub4e4\uc758 \uc2b9\uc778\uc5d0 \ub300\ud574 \uc5c4\uccad\ub098\uac8c \uc18c\ub9ac \uc9c8\ub800\uace0, \uc774 \uc7ac\ud310\uc774 \uacf5\uc2dd\uc801\uc73c\ub85c \ub05d\ub098\uae30\uae4c\uc9c0 1\uc2dc\uac04 \ubc18\uc774 \uac78\ub838\ub2e4. \ub9b4\ubc88\uc740 \uc989\uc2dc \ud480\ub824\ub098\uc9c0 \uc54a\uc558\uace0 \ub300\uc911\ub4e4\uc774\ub098 \uc758\ud68c\uc758 \uce5c\uad6c\ub4e4\uc774 \uadf8\uc758 \uc11d\ubc29\uc744 \ud655\ubcf4\ud558\uae30\uae4c\uc9c0 2\uc8fc\uac00 \uac78\ub838\ub2e4. \uc758\ud68c\uc758 \uc0ac\ub78c\ub4e4 \uc911 \uba87\uba87\uc740 \ub9b4\ubc88\uc758 \uc11d\ubc29\uc5d0 \ub300\ud574 \ubd88\ub9cc\uc744 \ud488\uc5c8\uc9c0\ub9cc \uc758\ud68c\ub294 \uc218\ud3c9\ud30c\uc5d0 \ub300\ud55c \ubc18\ub300\uac00 \uc0dd\uae30\ub3c4\ub85d \uc5b5\uc555\ud558\ub294\ub370 \uc131\uacf5\ud588\ub2e4. \uc218\ud3c9\ud30c\ub4e4\uc740 \uad6d\uac00\uc640 \uad70\uc778\ub4e4\uacfc \ubc18\ub780\uc744 \uc77c\uc73c\ud0a4\ub824\ub294 \uc2dc\ub3c4\ub97c \uc911\uc9c0\ud558\uac8c \ub418\uc5c8\uace0 \uc74c\ubaa8\ub3c4 \ube44\ubc00\ub9ac\uc5d0 \uc9dc\uae30 \uc2dc\uc791\ud588\ub2e4.", "sentence_answer": "\u2018Huge Peters\u2019\uac00 1649\ub144 5\uc6d4 25\uc77c \ud0d1 \uc548\uc5d0\uc11c \uc874 \ub9b4\ubc88\uc744 \ub9cc\ub0a0 \ub2f9\uc2dc, \ub9b4\ubc88\uc740 \uadf8\uc5d0\uac8c \uc790\uc2e0\uc740 \ud604 \uc815\uad8c\uc5d0 1\ub144 \uc788\uae30\ubcf4\ub2e4\ub294 \uc655\uc758 \ubc11\uc5d0 7\ub144 \ub3d9\uc548 \uc788\ub294 \uac83\uc774 \ub098\uc744 \uac83\uc774\ub77c\uace0 \ud588\uace0, \ud604 \uc815\uad8c\uc774 \uc5ec\uc804\ud788 \uc555\uc81c\uc801\uc774\ub77c\uba74, \uc0ac\ub78c\ub4e4\uc740 \u2018\ucc30\uc2a4 \uc655\uc790\u2019\uc5d0 \ub9de\uc11c \uc2f8\uc6cc\uc57c \ud55c\ub2e4\uace0 \uc8fc\uc7a5\ud588\ub2e4.", "paragraph_id": "6555421-3-0", "paragraph_question": "question: 1649\ub144 5\uc6d4 Huge Peters\uac00 \ub9b4\ubc88\uc744 \ub9cc\ub0ac\uc744 \ub54c \uadf8\ub294 \uc9c0\uae08\uc758 \uc815\ubd80\uc5d0 1\ub144 \uc788\ub294\uac83\ubcf4\ub2e4 \uc655 \uc544\ub798\uc5d0\uc11c \uba87 \ub144 \uc788\ub294 \uac83\uc774 \ub0ab\ub2e4\ub77c \ub9d0\ud588\ub098?, context: \u2018Huge Peters\u2019\uac00 1649\ub144 5\uc6d4 25\uc77c \ud0d1 \uc548\uc5d0\uc11c \uc874 \ub9b4\ubc88\uc744 \ub9cc\ub0a0 \ub2f9\uc2dc, \ub9b4\ubc88\uc740 \uadf8\uc5d0\uac8c \uc790\uc2e0\uc740 \ud604 \uc815\uad8c\uc5d0 1\ub144 \uc788\uae30\ubcf4\ub2e4\ub294 \uc655\uc758 \ubc11\uc5d0 7\ub144 \ub3d9\uc548 \uc788\ub294 \uac83\uc774 \ub098\uc744 \uac83\uc774\ub77c\uace0 \ud588\uace0, \ud604 \uc815\uad8c\uc774 \uc5ec\uc804\ud788 \uc555\uc81c\uc801\uc774\ub77c\uba74, \uc0ac\ub78c\ub4e4\uc740 \u2018\ucc30\uc2a4 \uc655\uc790\u2019\uc5d0 \ub9de\uc11c \uc2f8\uc6cc\uc57c \ud55c\ub2e4\uace0 \uc8fc\uc7a5\ud588\ub2e4. 3\ub2ec \ud6c4 \u2018Outcry of the Apprentices to the Soldiers\u2019\uc5d0\uc11c \ub9b4\ubc88\uc740 \uacac\uc2b5\uc0dd\ub4e4\uacfc \uad70\uc778\ub4e4\uc740 \ucf54\uba3c\uc6f0\uc2a4\uc758 \ud5cc\ubc95\uc758 \uae30\ubcf8\uad8c\uacfc \uc758\ud68c\uc5d0\uc11c\uc758 \uc0ac\ub78c\ub4e4\uc758 \uad8c\ub9ac\ub97c \uc815\ubd80\uac00 \ubc31\uc131\ub4e4\uc744 \ubc18\ud558\uc9c0 \uc54a\uac8c \uaddc\uc81c\ud558\ub294 \uc2dd\uc73c\ub85c \uc2f8\uc6cc\uc57c \ud55c\ub2e4\uace0 \uc8fc\uc7a5\ud588\ub2e4. 1648\ub144 \ube0c\ub85c\ub4dc\uc6e8\uc774 \ud68c\uc758\uc774\ud6c4\uc5d0 \uc218\ud3c9\ud30c\ub4e4\uc774 \uc655\ub2f9\ud30c\ub4e4\uacfc \uc0c8\ub85c\uc6b4 \uacf5\ud654\uad6d\uc744 \ud0c0\ub3c4\ud558\uae30 \uc704\ud574 \uc74c\ubaa8\ub97c \uafb8\ubbfc\ub2e4\ub294 \uc18c\ubb38\ub4e4\uc774 \ub3cc\uc558\ub2e4. \uc625\uc2a4\ud37c\ub4dc \ubc18\ub780 \uc911\uc5d0 \uc774\uac83\uc740 \uc758\ud68c\uac00 \u2018Tower of London to Lord Cottington\u2019\uc5d0\uc11c \ucc30\uc2a4 2\uc138\uc758 \ucd94\ubc29\uacfc \uad00\ub828\ub41c \uace0\ubb38\uc778 \uc655\ub2f9\ud30c \uc778\uc9c8\ub85c\ubd80\ud130 \ud3b8\uc9c0\ub97c \ud68d\ub4dd\ud588\uc744 \ub54c \ud655\uc778\ub418\uc5c8\ub2e4. \uadf8 \ud3b8\uc9c0\uc5d0\uc11c\ub294 \uc655\ub2f9\ud30c\ub4e4\uc740 \ucc30\uc2a4\uac00 \uc655\uc704\ub97c \ub418\ucc3e\uac8c \ud558\uae30 \uc704\ud574\uc11c \uc218\ud3c9\ud30c\uc5d0 \uc6d0\uc870\ud574\uc57c\ud55c\ub2e4\ub294 \ub0b4\uc6a9\uc774\uc5c8\ub2e4. \uc774 \uc99d\uac70\ub85c \uc758\ud68c\ub294 \ub9b4\ubc88\uc744 \ub192\uc740 \uc218\uc704\uc758 \ubc18\uc5ed\uc8c4\ub85c \ucc45\uc815\ud558\ub294 \uc81c\uc758\ub97c \ubc1b\uc544\ub4e4\uc600\ub2e4. \ucc30\uc2a4 1\uc138\ub97c \uc2ec\ud310\ud588\ub358 \ubc95\uc6d0\uc744 \uc0ac\uc6a9\ud558\uba74\uc11c \uc655\uc758 \uc7ac\ud310\ucc98\ub7fc \uc9c0\uba85\ub41c \uc704\uc6d0\ub4e4\uc774 \ud615\ubcc4\uc744 \uacb0\uc815\ud558\ub824\ud588\uc73c\ub098 \uadf8\uc640 \ub2e4\ub974\uac8c 12\uba85\uc758 \ubc30\uc2ec\uc6d0\ub4e4\uc774 \ubb34\uc8c4\ub77c\uace0 \uacb0\uc815\ud588\ub2e4. \ubc30\uc2ec\uc6d0\ub4e4\uc774 \uadf8\uac00 \ubb34\uc8c4\uc784\uc744 \uc120\uace0\ud588\uc744 \ub54c, \uc0ac\ub78c\ub4e4\uc740 \uadf8\ub4e4\uc758 \uc2b9\uc778\uc5d0 \ub300\ud574 \uc5c4\uccad\ub098\uac8c \uc18c\ub9ac \uc9c8\ub800\uace0, \uc774 \uc7ac\ud310\uc774 \uacf5\uc2dd\uc801\uc73c\ub85c \ub05d\ub098\uae30\uae4c\uc9c0 1\uc2dc\uac04 \ubc18\uc774 \uac78\ub838\ub2e4. \ub9b4\ubc88\uc740 \uc989\uc2dc \ud480\ub824\ub098\uc9c0 \uc54a\uc558\uace0 \ub300\uc911\ub4e4\uc774\ub098 \uc758\ud68c\uc758 \uce5c\uad6c\ub4e4\uc774 \uadf8\uc758 \uc11d\ubc29\uc744 \ud655\ubcf4\ud558\uae30\uae4c\uc9c0 2\uc8fc\uac00 \uac78\ub838\ub2e4. \uc758\ud68c\uc758 \uc0ac\ub78c\ub4e4 \uc911 \uba87\uba87\uc740 \ub9b4\ubc88\uc758 \uc11d\ubc29\uc5d0 \ub300\ud574 \ubd88\ub9cc\uc744 \ud488\uc5c8\uc9c0\ub9cc \uc758\ud68c\ub294 \uc218\ud3c9\ud30c\uc5d0 \ub300\ud55c \ubc18\ub300\uac00 \uc0dd\uae30\ub3c4\ub85d \uc5b5\uc555\ud558\ub294\ub370 \uc131\uacf5\ud588\ub2e4. \uc218\ud3c9\ud30c\ub4e4\uc740 \uad6d\uac00\uc640 \uad70\uc778\ub4e4\uacfc \ubc18\ub780\uc744 \uc77c\uc73c\ud0a4\ub824\ub294 \uc2dc\ub3c4\ub97c \uc911\uc9c0\ud558\uac8c \ub418\uc5c8\uace0 \uc74c\ubaa8\ub3c4 \ube44\ubc00\ub9ac\uc5d0 \uc9dc\uae30 \uc2dc\uc791\ud588\ub2e4."} {"question": "\ud55c\uad6d\uc5d0\uc11c \ubc95\ud559\uc801\uc131\uc2dc\ud5d8\uc758 \uc810\uc218\uac00 \uc720\ud6a8\ud55c \uae30\uac04\uc740?", "paragraph": "\ubbf8\uad6d\uc758 \ubc95\ud559\uc804\ubb38\ub300\ud559\uc6d0\uc740 \ud55c\uad6d\ucc98\ub7fc \uc5c4\uaca9\ud55c \uc815\uc6d0\uc81c\ub098 \ub300\ud559\uc6d0 \uc9c0\uc6d0\uac00\ub2a5\ud559\uad50\uc5d0 \uc81c\ud55c\uc744 \ub450\uace0 \uc788\uc9c0 \uc54a\uc544 \ubaa8\ub4e0 \ub85c\uc2a4\ucfe8\uc5d0 \ub3d9\uc2dc \uc9c0\uc6d0\ud558\ub294 \uac83\uc774 \uac00\ub2a5\ud558\ub2e4. \ubc95\ud559\uc801\uc131\uc2dc\ud5d8\uc5d0 \ud574\ub2f9\ud558\ub294 LSAT\uc740 2\ub144\uc5d0 3\ubc88 \ubcfc \uc218 \uc788\uc5c8\uc73c\ub098, 2017\ub144\ubd80\ud130 \uc751\uc2dc \ud69f\uc218 \uc81c\ud55c\uc774 \ud3d0\uc9c0\ub418\uc5c8\ub2e4. LSAT \uc810\uc218\ub294 5\ub144\uac04 \uc810\uc218\uac00 \uc720\ud6a8\ud558\uace0 \ub300\ubd80\ubd84\uc758 \ud559\uad50\uac00 \ucd5c\uace0\uc810\uc744 \uc778\uc815\ud55c\ub2e4. \ud55c\uad6d\uc740 \uc77c\ub144\uc5d0 \ud55c\ubc88 \ub9cc \ubcfc \uc218 \uc788\uc73c\uba70 \uc810\uc218\ub294 \ub2f9\ud574\ub144\ub3c4\uc5d0\ub9cc \uc720\ud6a8\ud558\ub2e4. \ud3c9\uac00\ub294 \ud55c\uad6d\ucc98\ub7fc \uc804\uad6d\uc801\uc73c\ub85c \ub3d9\uc77c\ud55c \uc0c1\ub300\ud3c9\uac00\ub97c \uc2e4\uc2dc\ud558\uc9c0\ub294 \uc54a\uc73c\ub098 \ub300\ubd80\ubd84\uc758 \ubbf8\uad6d \ub85c\uc2a4\ucfe8\uc5d0\uc11c\ub3c4 \uc0c1\ub300\ud3c9\uac00\ub97c \uc0ac\uc6a9\ud55c\ub2e4. JD \ud559\uc704\ub97c \ucde8\ub4dd\ud55c \ud559\uc0dd\ub4e4\uc774 \ubc95\ud559\uc790\uac00 \ub418\ub824\uba74 \ubc95\ud559 \ubd84\uc57c\uc5d0\uc11c \ubc15\uc0ac \ud559\uc704\ub97c \ucde8\ub4dd\ud558\ub294\uac8c \uc77c\ubc18\uc801\uc778 \ud55c\uad6d\uc5d0\uc11c\ub294 JD \ud559\uc704\ub97c \uc11d\uc0ac \ud559\uc704\ub85c \ubd84\ub958\ud558\ub098, \uc2e4\ubb34\uc640 \ub2e4\ub978 \ud559\ubb38\uacfc\uc758 \uc811\ubaa9\uc744 \uc911\uc2dc\ud558\uc5ec \ub300\ubd80\ubd84\uc758 \ubc95\ud559\uc790\ub4e4\uc774 JD, LLB(1960\ub144\ub300\ub9d0\uc5d0 JD\ub85c \uac1c\uba85) \ud639\uc740 \ub2e4\ub978 \ubd84\uc57c PhD \ud559\uc704\ub97c \ucd5c\uc885\ud559\uc704\ub85c \uc18c\uc9c0\ud558\uace0 \uc788\ub294 \ubbf8\uad6d\uc5d0\uc11c\ub294 JD\ub97c \uc804\ubb38\ubc15\uc0ac \ud559\uc704\ub85c \ubd84\ub958\ud55c\ub2e4.", "answer": "\ub2f9\ud574\ub144\ub3c4", "sentence": "\uc810\uc218\ub294 \ub2f9\ud574\ub144\ub3c4 \uc5d0\ub9cc \uc720\ud6a8\ud558\ub2e4.", "paragraph_sentence": "\ubbf8\uad6d\uc758 \ubc95\ud559\uc804\ubb38\ub300\ud559\uc6d0\uc740 \ud55c\uad6d\ucc98\ub7fc \uc5c4\uaca9\ud55c \uc815\uc6d0\uc81c\ub098 \ub300\ud559\uc6d0 \uc9c0\uc6d0\uac00\ub2a5\ud559\uad50\uc5d0 \uc81c\ud55c\uc744 \ub450\uace0 \uc788\uc9c0 \uc54a\uc544 \ubaa8\ub4e0 \ub85c\uc2a4\ucfe8\uc5d0 \ub3d9\uc2dc \uc9c0\uc6d0\ud558\ub294 \uac83\uc774 \uac00\ub2a5\ud558\ub2e4. \ubc95\ud559\uc801\uc131\uc2dc\ud5d8\uc5d0 \ud574\ub2f9\ud558\ub294 LSAT\uc740 2\ub144\uc5d0 3\ubc88 \ubcfc \uc218 \uc788\uc5c8\uc73c\ub098, 2017\ub144\ubd80\ud130 \uc751\uc2dc \ud69f\uc218 \uc81c\ud55c\uc774 \ud3d0\uc9c0\ub418\uc5c8\ub2e4. LSAT \uc810\uc218\ub294 5\ub144\uac04 \uc810\uc218\uac00 \uc720\ud6a8\ud558\uace0 \ub300\ubd80\ubd84\uc758 \ud559\uad50\uac00 \ucd5c\uace0\uc810\uc744 \uc778\uc815\ud55c\ub2e4. \ud55c\uad6d\uc740 \uc77c\ub144\uc5d0 \ud55c\ubc88 \ub9cc \ubcfc \uc218 \uc788\uc73c\uba70 \uc810\uc218\ub294 \ub2f9\ud574\ub144\ub3c4 \uc5d0\ub9cc \uc720\ud6a8\ud558\ub2e4. \ud3c9\uac00\ub294 \ud55c\uad6d\ucc98\ub7fc \uc804\uad6d\uc801\uc73c\ub85c \ub3d9\uc77c\ud55c \uc0c1\ub300\ud3c9\uac00\ub97c \uc2e4\uc2dc\ud558\uc9c0\ub294 \uc54a\uc73c\ub098 \ub300\ubd80\ubd84\uc758 \ubbf8\uad6d \ub85c\uc2a4\ucfe8\uc5d0\uc11c\ub3c4 \uc0c1\ub300\ud3c9\uac00\ub97c \uc0ac\uc6a9\ud55c\ub2e4. JD \ud559\uc704\ub97c \ucde8\ub4dd\ud55c \ud559\uc0dd\ub4e4\uc774 \ubc95\ud559\uc790\uac00 \ub418\ub824\uba74 \ubc95\ud559 \ubd84\uc57c\uc5d0\uc11c \ubc15\uc0ac \ud559\uc704\ub97c \ucde8\ub4dd\ud558\ub294\uac8c \uc77c\ubc18\uc801\uc778 \ud55c\uad6d\uc5d0\uc11c\ub294 JD \ud559\uc704\ub97c \uc11d\uc0ac \ud559\uc704\ub85c \ubd84\ub958\ud558\ub098, \uc2e4\ubb34\uc640 \ub2e4\ub978 \ud559\ubb38\uacfc\uc758 \uc811\ubaa9\uc744 \uc911\uc2dc\ud558\uc5ec \ub300\ubd80\ubd84\uc758 \ubc95\ud559\uc790\ub4e4\uc774 JD, LLB(1960\ub144\ub300\ub9d0\uc5d0 JD\ub85c \uac1c\uba85) \ud639\uc740 \ub2e4\ub978 \ubd84\uc57c PhD \ud559\uc704\ub97c \ucd5c\uc885\ud559\uc704\ub85c \uc18c\uc9c0\ud558\uace0 \uc788\ub294 \ubbf8\uad6d\uc5d0\uc11c\ub294 JD\ub97c \uc804\ubb38\ubc15\uc0ac \ud559\uc704\ub85c \ubd84\ub958\ud55c\ub2e4.", "paragraph_answer": "\ubbf8\uad6d\uc758 \ubc95\ud559\uc804\ubb38\ub300\ud559\uc6d0\uc740 \ud55c\uad6d\ucc98\ub7fc \uc5c4\uaca9\ud55c \uc815\uc6d0\uc81c\ub098 \ub300\ud559\uc6d0 \uc9c0\uc6d0\uac00\ub2a5\ud559\uad50\uc5d0 \uc81c\ud55c\uc744 \ub450\uace0 \uc788\uc9c0 \uc54a\uc544 \ubaa8\ub4e0 \ub85c\uc2a4\ucfe8\uc5d0 \ub3d9\uc2dc \uc9c0\uc6d0\ud558\ub294 \uac83\uc774 \uac00\ub2a5\ud558\ub2e4. \ubc95\ud559\uc801\uc131\uc2dc\ud5d8\uc5d0 \ud574\ub2f9\ud558\ub294 LSAT\uc740 2\ub144\uc5d0 3\ubc88 \ubcfc \uc218 \uc788\uc5c8\uc73c\ub098, 2017\ub144\ubd80\ud130 \uc751\uc2dc \ud69f\uc218 \uc81c\ud55c\uc774 \ud3d0\uc9c0\ub418\uc5c8\ub2e4. LSAT \uc810\uc218\ub294 5\ub144\uac04 \uc810\uc218\uac00 \uc720\ud6a8\ud558\uace0 \ub300\ubd80\ubd84\uc758 \ud559\uad50\uac00 \ucd5c\uace0\uc810\uc744 \uc778\uc815\ud55c\ub2e4. \ud55c\uad6d\uc740 \uc77c\ub144\uc5d0 \ud55c\ubc88 \ub9cc \ubcfc \uc218 \uc788\uc73c\uba70 \uc810\uc218\ub294 \ub2f9\ud574\ub144\ub3c4 \uc5d0\ub9cc \uc720\ud6a8\ud558\ub2e4. \ud3c9\uac00\ub294 \ud55c\uad6d\ucc98\ub7fc \uc804\uad6d\uc801\uc73c\ub85c \ub3d9\uc77c\ud55c \uc0c1\ub300\ud3c9\uac00\ub97c \uc2e4\uc2dc\ud558\uc9c0\ub294 \uc54a\uc73c\ub098 \ub300\ubd80\ubd84\uc758 \ubbf8\uad6d \ub85c\uc2a4\ucfe8\uc5d0\uc11c\ub3c4 \uc0c1\ub300\ud3c9\uac00\ub97c \uc0ac\uc6a9\ud55c\ub2e4. JD \ud559\uc704\ub97c \ucde8\ub4dd\ud55c \ud559\uc0dd\ub4e4\uc774 \ubc95\ud559\uc790\uac00 \ub418\ub824\uba74 \ubc95\ud559 \ubd84\uc57c\uc5d0\uc11c \ubc15\uc0ac \ud559\uc704\ub97c \ucde8\ub4dd\ud558\ub294\uac8c \uc77c\ubc18\uc801\uc778 \ud55c\uad6d\uc5d0\uc11c\ub294 JD \ud559\uc704\ub97c \uc11d\uc0ac \ud559\uc704\ub85c \ubd84\ub958\ud558\ub098, \uc2e4\ubb34\uc640 \ub2e4\ub978 \ud559\ubb38\uacfc\uc758 \uc811\ubaa9\uc744 \uc911\uc2dc\ud558\uc5ec \ub300\ubd80\ubd84\uc758 \ubc95\ud559\uc790\ub4e4\uc774 JD, LLB(1960\ub144\ub300\ub9d0\uc5d0 JD\ub85c \uac1c\uba85) \ud639\uc740 \ub2e4\ub978 \ubd84\uc57c PhD \ud559\uc704\ub97c \ucd5c\uc885\ud559\uc704\ub85c \uc18c\uc9c0\ud558\uace0 \uc788\ub294 \ubbf8\uad6d\uc5d0\uc11c\ub294 JD\ub97c \uc804\ubb38\ubc15\uc0ac \ud559\uc704\ub85c \ubd84\ub958\ud55c\ub2e4.", "sentence_answer": "\uc810\uc218\ub294 \ub2f9\ud574\ub144\ub3c4 \uc5d0\ub9cc \uc720\ud6a8\ud558\ub2e4.", "paragraph_id": "6602021-11-1", "paragraph_question": "question: \ud55c\uad6d\uc5d0\uc11c \ubc95\ud559\uc801\uc131\uc2dc\ud5d8\uc758 \uc810\uc218\uac00 \uc720\ud6a8\ud55c \uae30\uac04\uc740?, context: \ubbf8\uad6d\uc758 \ubc95\ud559\uc804\ubb38\ub300\ud559\uc6d0\uc740 \ud55c\uad6d\ucc98\ub7fc \uc5c4\uaca9\ud55c \uc815\uc6d0\uc81c\ub098 \ub300\ud559\uc6d0 \uc9c0\uc6d0\uac00\ub2a5\ud559\uad50\uc5d0 \uc81c\ud55c\uc744 \ub450\uace0 \uc788\uc9c0 \uc54a\uc544 \ubaa8\ub4e0 \ub85c\uc2a4\ucfe8\uc5d0 \ub3d9\uc2dc \uc9c0\uc6d0\ud558\ub294 \uac83\uc774 \uac00\ub2a5\ud558\ub2e4. \ubc95\ud559\uc801\uc131\uc2dc\ud5d8\uc5d0 \ud574\ub2f9\ud558\ub294 LSAT\uc740 2\ub144\uc5d0 3\ubc88 \ubcfc \uc218 \uc788\uc5c8\uc73c\ub098, 2017\ub144\ubd80\ud130 \uc751\uc2dc \ud69f\uc218 \uc81c\ud55c\uc774 \ud3d0\uc9c0\ub418\uc5c8\ub2e4. LSAT \uc810\uc218\ub294 5\ub144\uac04 \uc810\uc218\uac00 \uc720\ud6a8\ud558\uace0 \ub300\ubd80\ubd84\uc758 \ud559\uad50\uac00 \ucd5c\uace0\uc810\uc744 \uc778\uc815\ud55c\ub2e4. \ud55c\uad6d\uc740 \uc77c\ub144\uc5d0 \ud55c\ubc88 \ub9cc \ubcfc \uc218 \uc788\uc73c\uba70 \uc810\uc218\ub294 \ub2f9\ud574\ub144\ub3c4\uc5d0\ub9cc \uc720\ud6a8\ud558\ub2e4. \ud3c9\uac00\ub294 \ud55c\uad6d\ucc98\ub7fc \uc804\uad6d\uc801\uc73c\ub85c \ub3d9\uc77c\ud55c \uc0c1\ub300\ud3c9\uac00\ub97c \uc2e4\uc2dc\ud558\uc9c0\ub294 \uc54a\uc73c\ub098 \ub300\ubd80\ubd84\uc758 \ubbf8\uad6d \ub85c\uc2a4\ucfe8\uc5d0\uc11c\ub3c4 \uc0c1\ub300\ud3c9\uac00\ub97c \uc0ac\uc6a9\ud55c\ub2e4. JD \ud559\uc704\ub97c \ucde8\ub4dd\ud55c \ud559\uc0dd\ub4e4\uc774 \ubc95\ud559\uc790\uac00 \ub418\ub824\uba74 \ubc95\ud559 \ubd84\uc57c\uc5d0\uc11c \ubc15\uc0ac \ud559\uc704\ub97c \ucde8\ub4dd\ud558\ub294\uac8c \uc77c\ubc18\uc801\uc778 \ud55c\uad6d\uc5d0\uc11c\ub294 JD \ud559\uc704\ub97c \uc11d\uc0ac \ud559\uc704\ub85c \ubd84\ub958\ud558\ub098, \uc2e4\ubb34\uc640 \ub2e4\ub978 \ud559\ubb38\uacfc\uc758 \uc811\ubaa9\uc744 \uc911\uc2dc\ud558\uc5ec \ub300\ubd80\ubd84\uc758 \ubc95\ud559\uc790\ub4e4\uc774 JD, LLB(1960\ub144\ub300\ub9d0\uc5d0 JD\ub85c \uac1c\uba85) \ud639\uc740 \ub2e4\ub978 \ubd84\uc57c PhD \ud559\uc704\ub97c \ucd5c\uc885\ud559\uc704\ub85c \uc18c\uc9c0\ud558\uace0 \uc788\ub294 \ubbf8\uad6d\uc5d0\uc11c\ub294 JD\ub97c \uc804\ubb38\ubc15\uc0ac \ud559\uc704\ub85c \ubd84\ub958\ud55c\ub2e4."} {"question": "\ubb18\ud638 \uc911, \ub4a4\ub97c \uc774\uc740 \uad70\uc8fc\uc640 \uc2e0\ud558\ub4e4\uc774 \uacb0\uc815\ud558\uc5ec \uc62c\ub9ac\ub294 \uac83\uc740?", "paragraph": "\ubb18\ud638\ub294 \uad70\uc8fc\uc758 \uce58\uc138\ub97c \ub098\ud0c0\ub0b4\ub294 \uae00\uc790\uc778 \uc2dc\uc790\uc640 \uc885\ud638, 2\uae00\uc790\ub85c \uc774\ub8e8\uc5b4\uc9c4\ub2e4. \uc2dc\uc790\ub294 \uc2dc\ud638\ub97c \uc815\ud560 \ub54c \uc4f0\ub294 \uc2dc\ubc95(\u8ae1\u6cd5)\uc5d0 \uc900\ud558\uc5ec \ub4a4\ub97c \uc774\uc740 \uad70\uc8fc\uc640 \uc2e0\ud558\ub4e4\uc774 \uacb0\uc815\ud558\uc5ec \uc62c\ub9ac\uac8c \ub41c\ub2e4. \ud754\ud788 \uc2dc\ubc95\uc740 \uace0\ub300 \uc911\uad6d\uc758 \uc8fc\uacf5 \ub2e8\uc774 \uc815\ud55c \uc2dc\ubc95\uc5d0 \uc758\uac70\ud558\uc5ec \uc815\ud588\ub2e4. \uc885\ud638\ub294 \u2018\uc870\u2019(\u7956) \ub610\ub294 \u2018\uc885\u2019(\u5b97) \uc911 \ud558\ub098\ub97c \uace8\ub77c \uc815\ud558\uac8c \ub41c\ub2e4. \ubb18\ud638 \uc911 \uc885\ud638\uc758 \uc720\ub798\ub294 \uace0\ub300 \uc911\uad6d\uc758 \uc0ac\ud68c \uc81c\ub3c4\uc778 \uc885\ubc95\uc81c\uc5d0\uc11c \ube44\ub86f\ub418\uc5c8\ub294\ub370 \uc120\uc655\uc758 \uc544\ub4e4\uc774\uc790 \ud604\uc655\uc758 \ud615\uc81c\ub85c\uc11c \ubd09\ud1a0\ub97c \ubc1b\uc544 \ucc98\uc74c \uc81c\ud6c4\uc5d0 \ucc45\ubd09\ub41c \uc774, \uc989 \ubcc4\uc790(\u5225\u5b50)\ub294 \u2018\uc870\u2019\uac00 \ub418\uace0 \uadf8 \ud6c4\uc608, \uc989 \ubcc4\uc790\uc758 \uc7a5\uc790\ub294 \u2018\uc885\u2019\uc774 \ub418\uba70, \ubcc4\uc790\uc758 \ucc28\ub0a8\uc744 \ube44\ub86f\ud55c \ub098\uba38\uc9c0 \uc544\ub4e4\ub4e4, \uc989 \uc911\uc790(\u8846\u5b50)\ub294 \u2018\uc18c\uc885\u2019(\u5c0f\u5b97)\uc774 \ub41c\ub2e4\uace0 \uc124\uba85\ud55c\ub2e4. \ub610\ud55c \ubcc4\uc790\ub97c \uc9c1\uc811 \uacc4\uc2b9\ud55c \uc774\ub294 \ubd88\ucc9c\uc704(\u4e0d\u9077\u4f4d)\ub77c \ud558\uc5ec \uadf8 \uc704\ud328\ub97c \ub2e4\ub978 \uc0ac\ub2f9\uc73c\ub85c \ubaa8\uc2dc\uc9c0 \uc54a\uc558\uace0, \ub2e8\uc21c\ud788 \uc870\uc0c1\uc758 \ud608\ud1b5\uc5d0 \ub530\ub77c \uacc4\uc2b9\ud55c \uc774\ub294 5\ub300\uac00 \ub118\uc73c\uba74 \ub2e4\ub978 \uc0ac\ub2f9\uc73c\ub85c \ubaa8\uc154\uc84c\ub2e4. \ubd88\ucc9c\uc704\ub85c \ubaa8\uc154\uc9c4 \u2018\uc885\u2019\uc740 5\ub300\uac00 \ub118\uc5b4 \uc885\ubb18\uc5d0\uc11c \ub0b4\uccd0\uc9c4 \u2018\uc885\u2019\ubcf4\ub2e4 \ub354 \ub192\uc740 \uaca9\uc870\ub97c \uac00\uc84c\uace0 \uc774\ub7ec\ud55c \ud608\uc5f0\uc801, \uc218\uc9c1\uc801 \uacc4\uc2b9\uad00\uacc4\ub97c \ud615\uc131\ud558\uc5ec \uc8fc\ub098\ub77c\uc5d0 \uc774\ub974\ub7ec\uc11c\ub294 \uc885\ubc95\uc81c\uc758 \uae30\ubcf8 \uccb4\uc81c\uac00 \uc644\uc131\ub418\uc5c8\ub2e4. \uc774 \uc81c\ub3c4\uac00 \uc810\ucc28 \uc2dc\ud638 \uc81c\ub3c4\uc640 \uacb0\ubd80\ub418\uba74\uc11c, \ub610\ud55c \uc885\ud638 \uc55e\uc5d0 \uc2dc\uc790\ub97c \ubd99\uc5ec\uc4f0\uba74\uc11c \uc655\uc2e4\uc5d0\uc11c \ud655\ub9bd\ud654\ub41c \uac83\uc774 \ubc14\ub85c \ubb18\ud638\uc774\ub2e4.", "answer": "\uc2dc\uc790", "sentence": "\ubb18\ud638\ub294 \uad70\uc8fc\uc758 \uce58\uc138\ub97c \ub098\ud0c0\ub0b4\ub294 \uae00\uc790\uc778 \uc2dc\uc790 \uc640 \uc885\ud638, 2\uae00\uc790\ub85c \uc774\ub8e8\uc5b4\uc9c4\ub2e4.", "paragraph_sentence": " \ubb18\ud638\ub294 \uad70\uc8fc\uc758 \uce58\uc138\ub97c \ub098\ud0c0\ub0b4\ub294 \uae00\uc790\uc778 \uc2dc\uc790 \uc640 \uc885\ud638, 2\uae00\uc790\ub85c \uc774\ub8e8\uc5b4\uc9c4\ub2e4. \uc2dc\uc790\ub294 \uc2dc\ud638\ub97c \uc815\ud560 \ub54c \uc4f0\ub294 \uc2dc\ubc95(\u8ae1\u6cd5)\uc5d0 \uc900\ud558\uc5ec \ub4a4\ub97c \uc774\uc740 \uad70\uc8fc\uc640 \uc2e0\ud558\ub4e4\uc774 \uacb0\uc815\ud558\uc5ec \uc62c\ub9ac\uac8c \ub41c\ub2e4. \ud754\ud788 \uc2dc\ubc95\uc740 \uace0\ub300 \uc911\uad6d\uc758 \uc8fc\uacf5 \ub2e8\uc774 \uc815\ud55c \uc2dc\ubc95\uc5d0 \uc758\uac70\ud558\uc5ec \uc815\ud588\ub2e4. \uc885\ud638\ub294 \u2018\uc870\u2019(\u7956) \ub610\ub294 \u2018\uc885\u2019(\u5b97) \uc911 \ud558\ub098\ub97c \uace8\ub77c \uc815\ud558\uac8c \ub41c\ub2e4. \ubb18\ud638 \uc911 \uc885\ud638\uc758 \uc720\ub798\ub294 \uace0\ub300 \uc911\uad6d\uc758 \uc0ac\ud68c \uc81c\ub3c4\uc778 \uc885\ubc95\uc81c\uc5d0\uc11c \ube44\ub86f\ub418\uc5c8\ub294\ub370 \uc120\uc655\uc758 \uc544\ub4e4\uc774\uc790 \ud604\uc655\uc758 \ud615\uc81c\ub85c\uc11c \ubd09\ud1a0\ub97c \ubc1b\uc544 \ucc98\uc74c \uc81c\ud6c4\uc5d0 \ucc45\ubd09\ub41c \uc774, \uc989 \ubcc4\uc790(\u5225\u5b50)\ub294 \u2018\uc870\u2019\uac00 \ub418\uace0 \uadf8 \ud6c4\uc608, \uc989 \ubcc4\uc790\uc758 \uc7a5\uc790\ub294 \u2018\uc885\u2019\uc774 \ub418\uba70, \ubcc4\uc790\uc758 \ucc28\ub0a8\uc744 \ube44\ub86f\ud55c \ub098\uba38\uc9c0 \uc544\ub4e4\ub4e4, \uc989 \uc911\uc790(\u8846\u5b50)\ub294 \u2018\uc18c\uc885\u2019(\u5c0f\u5b97)\uc774 \ub41c\ub2e4\uace0 \uc124\uba85\ud55c\ub2e4. \ub610\ud55c \ubcc4\uc790\ub97c \uc9c1\uc811 \uacc4\uc2b9\ud55c \uc774\ub294 \ubd88\ucc9c\uc704(\u4e0d\u9077\u4f4d)\ub77c \ud558\uc5ec \uadf8 \uc704\ud328\ub97c \ub2e4\ub978 \uc0ac\ub2f9\uc73c\ub85c \ubaa8\uc2dc\uc9c0 \uc54a\uc558\uace0, \ub2e8\uc21c\ud788 \uc870\uc0c1\uc758 \ud608\ud1b5\uc5d0 \ub530\ub77c \uacc4\uc2b9\ud55c \uc774\ub294 5\ub300\uac00 \ub118\uc73c\uba74 \ub2e4\ub978 \uc0ac\ub2f9\uc73c\ub85c \ubaa8\uc154\uc84c\ub2e4. \ubd88\ucc9c\uc704\ub85c \ubaa8\uc154\uc9c4 \u2018\uc885\u2019\uc740 5\ub300\uac00 \ub118\uc5b4 \uc885\ubb18\uc5d0\uc11c \ub0b4\uccd0\uc9c4 \u2018\uc885\u2019\ubcf4\ub2e4 \ub354 \ub192\uc740 \uaca9\uc870\ub97c \uac00\uc84c\uace0 \uc774\ub7ec\ud55c \ud608\uc5f0\uc801, \uc218\uc9c1\uc801 \uacc4\uc2b9\uad00\uacc4\ub97c \ud615\uc131\ud558\uc5ec \uc8fc\ub098\ub77c\uc5d0 \uc774\ub974\ub7ec\uc11c\ub294 \uc885\ubc95\uc81c\uc758 \uae30\ubcf8 \uccb4\uc81c\uac00 \uc644\uc131\ub418\uc5c8\ub2e4. \uc774 \uc81c\ub3c4\uac00 \uc810\ucc28 \uc2dc\ud638 \uc81c\ub3c4\uc640 \uacb0\ubd80\ub418\uba74\uc11c, \ub610\ud55c \uc885\ud638 \uc55e\uc5d0 \uc2dc\uc790\ub97c \ubd99\uc5ec\uc4f0\uba74\uc11c \uc655\uc2e4\uc5d0\uc11c \ud655\ub9bd\ud654\ub41c \uac83\uc774 \ubc14\ub85c \ubb18\ud638\uc774\ub2e4.", "paragraph_answer": "\ubb18\ud638\ub294 \uad70\uc8fc\uc758 \uce58\uc138\ub97c \ub098\ud0c0\ub0b4\ub294 \uae00\uc790\uc778 \uc2dc\uc790 \uc640 \uc885\ud638, 2\uae00\uc790\ub85c \uc774\ub8e8\uc5b4\uc9c4\ub2e4. \uc2dc\uc790\ub294 \uc2dc\ud638\ub97c \uc815\ud560 \ub54c \uc4f0\ub294 \uc2dc\ubc95(\u8ae1\u6cd5)\uc5d0 \uc900\ud558\uc5ec \ub4a4\ub97c \uc774\uc740 \uad70\uc8fc\uc640 \uc2e0\ud558\ub4e4\uc774 \uacb0\uc815\ud558\uc5ec \uc62c\ub9ac\uac8c \ub41c\ub2e4. \ud754\ud788 \uc2dc\ubc95\uc740 \uace0\ub300 \uc911\uad6d\uc758 \uc8fc\uacf5 \ub2e8\uc774 \uc815\ud55c \uc2dc\ubc95\uc5d0 \uc758\uac70\ud558\uc5ec \uc815\ud588\ub2e4. \uc885\ud638\ub294 \u2018\uc870\u2019(\u7956) \ub610\ub294 \u2018\uc885\u2019(\u5b97) \uc911 \ud558\ub098\ub97c \uace8\ub77c \uc815\ud558\uac8c \ub41c\ub2e4. \ubb18\ud638 \uc911 \uc885\ud638\uc758 \uc720\ub798\ub294 \uace0\ub300 \uc911\uad6d\uc758 \uc0ac\ud68c \uc81c\ub3c4\uc778 \uc885\ubc95\uc81c\uc5d0\uc11c \ube44\ub86f\ub418\uc5c8\ub294\ub370 \uc120\uc655\uc758 \uc544\ub4e4\uc774\uc790 \ud604\uc655\uc758 \ud615\uc81c\ub85c\uc11c \ubd09\ud1a0\ub97c \ubc1b\uc544 \ucc98\uc74c \uc81c\ud6c4\uc5d0 \ucc45\ubd09\ub41c \uc774, \uc989 \ubcc4\uc790(\u5225\u5b50)\ub294 \u2018\uc870\u2019\uac00 \ub418\uace0 \uadf8 \ud6c4\uc608, \uc989 \ubcc4\uc790\uc758 \uc7a5\uc790\ub294 \u2018\uc885\u2019\uc774 \ub418\uba70, \ubcc4\uc790\uc758 \ucc28\ub0a8\uc744 \ube44\ub86f\ud55c \ub098\uba38\uc9c0 \uc544\ub4e4\ub4e4, \uc989 \uc911\uc790(\u8846\u5b50)\ub294 \u2018\uc18c\uc885\u2019(\u5c0f\u5b97)\uc774 \ub41c\ub2e4\uace0 \uc124\uba85\ud55c\ub2e4. \ub610\ud55c \ubcc4\uc790\ub97c \uc9c1\uc811 \uacc4\uc2b9\ud55c \uc774\ub294 \ubd88\ucc9c\uc704(\u4e0d\u9077\u4f4d)\ub77c \ud558\uc5ec \uadf8 \uc704\ud328\ub97c \ub2e4\ub978 \uc0ac\ub2f9\uc73c\ub85c \ubaa8\uc2dc\uc9c0 \uc54a\uc558\uace0, \ub2e8\uc21c\ud788 \uc870\uc0c1\uc758 \ud608\ud1b5\uc5d0 \ub530\ub77c \uacc4\uc2b9\ud55c \uc774\ub294 5\ub300\uac00 \ub118\uc73c\uba74 \ub2e4\ub978 \uc0ac\ub2f9\uc73c\ub85c \ubaa8\uc154\uc84c\ub2e4. \ubd88\ucc9c\uc704\ub85c \ubaa8\uc154\uc9c4 \u2018\uc885\u2019\uc740 5\ub300\uac00 \ub118\uc5b4 \uc885\ubb18\uc5d0\uc11c \ub0b4\uccd0\uc9c4 \u2018\uc885\u2019\ubcf4\ub2e4 \ub354 \ub192\uc740 \uaca9\uc870\ub97c \uac00\uc84c\uace0 \uc774\ub7ec\ud55c \ud608\uc5f0\uc801, \uc218\uc9c1\uc801 \uacc4\uc2b9\uad00\uacc4\ub97c \ud615\uc131\ud558\uc5ec \uc8fc\ub098\ub77c\uc5d0 \uc774\ub974\ub7ec\uc11c\ub294 \uc885\ubc95\uc81c\uc758 \uae30\ubcf8 \uccb4\uc81c\uac00 \uc644\uc131\ub418\uc5c8\ub2e4. \uc774 \uc81c\ub3c4\uac00 \uc810\ucc28 \uc2dc\ud638 \uc81c\ub3c4\uc640 \uacb0\ubd80\ub418\uba74\uc11c, \ub610\ud55c \uc885\ud638 \uc55e\uc5d0 \uc2dc\uc790\ub97c \ubd99\uc5ec\uc4f0\uba74\uc11c \uc655\uc2e4\uc5d0\uc11c \ud655\ub9bd\ud654\ub41c \uac83\uc774 \ubc14\ub85c \ubb18\ud638\uc774\ub2e4.", "sentence_answer": "\ubb18\ud638\ub294 \uad70\uc8fc\uc758 \uce58\uc138\ub97c \ub098\ud0c0\ub0b4\ub294 \uae00\uc790\uc778 \uc2dc\uc790 \uc640 \uc885\ud638, 2\uae00\uc790\ub85c \uc774\ub8e8\uc5b4\uc9c4\ub2e4.", "paragraph_id": "6504692-1-1", "paragraph_question": "question: \ubb18\ud638 \uc911, \ub4a4\ub97c \uc774\uc740 \uad70\uc8fc\uc640 \uc2e0\ud558\ub4e4\uc774 \uacb0\uc815\ud558\uc5ec \uc62c\ub9ac\ub294 \uac83\uc740?, context: \ubb18\ud638\ub294 \uad70\uc8fc\uc758 \uce58\uc138\ub97c \ub098\ud0c0\ub0b4\ub294 \uae00\uc790\uc778 \uc2dc\uc790\uc640 \uc885\ud638, 2\uae00\uc790\ub85c \uc774\ub8e8\uc5b4\uc9c4\ub2e4. \uc2dc\uc790\ub294 \uc2dc\ud638\ub97c \uc815\ud560 \ub54c \uc4f0\ub294 \uc2dc\ubc95(\u8ae1\u6cd5)\uc5d0 \uc900\ud558\uc5ec \ub4a4\ub97c \uc774\uc740 \uad70\uc8fc\uc640 \uc2e0\ud558\ub4e4\uc774 \uacb0\uc815\ud558\uc5ec \uc62c\ub9ac\uac8c \ub41c\ub2e4. \ud754\ud788 \uc2dc\ubc95\uc740 \uace0\ub300 \uc911\uad6d\uc758 \uc8fc\uacf5 \ub2e8\uc774 \uc815\ud55c \uc2dc\ubc95\uc5d0 \uc758\uac70\ud558\uc5ec \uc815\ud588\ub2e4. \uc885\ud638\ub294 \u2018\uc870\u2019(\u7956) \ub610\ub294 \u2018\uc885\u2019(\u5b97) \uc911 \ud558\ub098\ub97c \uace8\ub77c \uc815\ud558\uac8c \ub41c\ub2e4. \ubb18\ud638 \uc911 \uc885\ud638\uc758 \uc720\ub798\ub294 \uace0\ub300 \uc911\uad6d\uc758 \uc0ac\ud68c \uc81c\ub3c4\uc778 \uc885\ubc95\uc81c\uc5d0\uc11c \ube44\ub86f\ub418\uc5c8\ub294\ub370 \uc120\uc655\uc758 \uc544\ub4e4\uc774\uc790 \ud604\uc655\uc758 \ud615\uc81c\ub85c\uc11c \ubd09\ud1a0\ub97c \ubc1b\uc544 \ucc98\uc74c \uc81c\ud6c4\uc5d0 \ucc45\ubd09\ub41c \uc774, \uc989 \ubcc4\uc790(\u5225\u5b50)\ub294 \u2018\uc870\u2019\uac00 \ub418\uace0 \uadf8 \ud6c4\uc608, \uc989 \ubcc4\uc790\uc758 \uc7a5\uc790\ub294 \u2018\uc885\u2019\uc774 \ub418\uba70, \ubcc4\uc790\uc758 \ucc28\ub0a8\uc744 \ube44\ub86f\ud55c \ub098\uba38\uc9c0 \uc544\ub4e4\ub4e4, \uc989 \uc911\uc790(\u8846\u5b50)\ub294 \u2018\uc18c\uc885\u2019(\u5c0f\u5b97)\uc774 \ub41c\ub2e4\uace0 \uc124\uba85\ud55c\ub2e4. \ub610\ud55c \ubcc4\uc790\ub97c \uc9c1\uc811 \uacc4\uc2b9\ud55c \uc774\ub294 \ubd88\ucc9c\uc704(\u4e0d\u9077\u4f4d)\ub77c \ud558\uc5ec \uadf8 \uc704\ud328\ub97c \ub2e4\ub978 \uc0ac\ub2f9\uc73c\ub85c \ubaa8\uc2dc\uc9c0 \uc54a\uc558\uace0, \ub2e8\uc21c\ud788 \uc870\uc0c1\uc758 \ud608\ud1b5\uc5d0 \ub530\ub77c \uacc4\uc2b9\ud55c \uc774\ub294 5\ub300\uac00 \ub118\uc73c\uba74 \ub2e4\ub978 \uc0ac\ub2f9\uc73c\ub85c \ubaa8\uc154\uc84c\ub2e4. \ubd88\ucc9c\uc704\ub85c \ubaa8\uc154\uc9c4 \u2018\uc885\u2019\uc740 5\ub300\uac00 \ub118\uc5b4 \uc885\ubb18\uc5d0\uc11c \ub0b4\uccd0\uc9c4 \u2018\uc885\u2019\ubcf4\ub2e4 \ub354 \ub192\uc740 \uaca9\uc870\ub97c \uac00\uc84c\uace0 \uc774\ub7ec\ud55c \ud608\uc5f0\uc801, \uc218\uc9c1\uc801 \uacc4\uc2b9\uad00\uacc4\ub97c \ud615\uc131\ud558\uc5ec \uc8fc\ub098\ub77c\uc5d0 \uc774\ub974\ub7ec\uc11c\ub294 \uc885\ubc95\uc81c\uc758 \uae30\ubcf8 \uccb4\uc81c\uac00 \uc644\uc131\ub418\uc5c8\ub2e4. \uc774 \uc81c\ub3c4\uac00 \uc810\ucc28 \uc2dc\ud638 \uc81c\ub3c4\uc640 \uacb0\ubd80\ub418\uba74\uc11c, \ub610\ud55c \uc885\ud638 \uc55e\uc5d0 \uc2dc\uc790\ub97c \ubd99\uc5ec\uc4f0\uba74\uc11c \uc655\uc2e4\uc5d0\uc11c \ud655\ub9bd\ud654\ub41c \uac83\uc774 \ubc14\ub85c \ubb18\ud638\uc774\ub2e4."} {"question": "\ub178\uba3c \uc18c\ud504\ub294 \uc8fc\uac70\ube44\uc790 \ub54c\ubb38\uc5d0 \uba87\uc77c\ub9c8\ub2e4 \ucd9c\uc785\uad6d\uc744 \ubc18\ubcf5\ud588\ub098\uc694?", "paragraph": "\ud55c\uad6d\uc804\uc7c1 \uc774\ud6c4 \uc11c\uc6b8 \uc0c1\uc8fc \ud2b9\ud30c\uc6d0\uc73c\ub85c \uc77c\ud55c \ucd5c\ucd08\uc758 \uc11c\uc591\uc778 \uae30\uc790\uc778 \ub178\uba3c \uc18c\ud504\ub294 \uc544\uc2dc\uc544\uc6d4\uc2a4\ud2b8\ub9ac\ud2b8\uc800\ub110(AWSJ) \uae30\uc790\ub85c \uadfc\ubb34\ud558\ub358 \uc911 5\u00b718 \ubbfc\uc8fc\ud654 \uc6b4\ub3d9\uc744 \uacaa\uc5c8\ub2e4. \uadf8\ub294 \"5\u00b718 \ubbfc\uc8fc\ud654 \uc6b4\ub3d9\uc740 \ub300\ud55c\ubbfc\uad6d\uc758 \ubbfc\uc8fc\ud654\ub97c \ud5a5\ud55c \uae38\uace0 \uae34 \ud22c\uc7c1\uc758 \uc77c\ubd80\ubd84\uc774\ub2e4. \uc55e \uc138\ub300\uac00 \uc790\uc720\uc120\uac70\ub97c \ud655\ub9bd\ud558\uace0 \ubbfc\uc8fc\uc8fc\uc758\ub97c \uaf43\ud53c\uc6b0\ub824\uace0 \uc5bc\ub9c8\ub098 \ub9ce\uc740 \uc5b4\ub824\uc6c0\uc744 \uacaa\uc5c8\ub294\uc9c0 \uc9c0\uae08 \uc80a\uc740 \uc138\ub300\uac00 \ubc30\uc6b0\uace0 \uc9c4\uc2ec\uc73c\ub85c \uac10\uc0ac\ud558\uae38 \ubc14\ub780\ub2e4\"\uba70 \"\uc9c0\uae08\uc740 \uc5b8\ub860\uacc4\uc5d0\uc11c \uc740\ud1f4\ud588\uc9c0\ub9cc \ubbfc\uc8fc\ud654 \uc6b4\ub3d9 \ub54c \ub300\ud55c\ubbfc\uad6d \uc815\ubd80\uc758 \ub9ce\uc740 \uc555\ub825\uc5d0\ub3c4 \uc800\ub97c \ube44\ub86f\ud55c \ub9ce\uc740 \ud5cc\uc2e0\uc801\uc778 \uae30\uc790\ub4e4\uc774 \uc678\uad6d \ub3c5\uc790\ub4e4\uc5d0\uac8c \ub300\ud55c\ubbfc\uad6d \uc0c1\ud669\uc744 \uc54c\ub9ac\ub824\uace0 \ub178\ub825\ud588\ub2e4. \uc2dc\ubbfc\uc744 \uc9c4\uc555\ud558\ub358 \uad70\uc778\ub4e4\uc758 \uc794\ud639\ud568\uc774 \uc2dc\uc704\ub97c \ucd09\ubc1c\ud588\ub2e4\ub294 \uac83\uc744 \uc54c\uac8c \ub410\ub2e4\"\ub77c\uace0 \ub9d0\ud588\ub2e4. \ub610\ud55c \"\uc678\uad6d \uc7a1\uc9c0\ub098 \uc2e0\ubb38\uc744 \ub300\ud55c\ubbfc\uad6d\uc5d0 \ub4e4\uc5ec\uc624\ub294 \uacfc\uc815\uc5d0\uc11c \uad70 \uad00\uacc4\uc790\ub4e4\uc774 \ubc30\ud3ec \uc804 \ubaa8\ub4e0 \uc790\ub8cc\ub97c \uac80\uc5f4\ud588\ub2e4. \uad11\uc8fc\uc5d0 \ub300\ud55c \uae30\uc0ac\ub098 \uc804\ub450\ud658\uc744 \ube44\ud310\ud558\ub294 \uae30\uc0ac \ub4f1 \ub9c8\uc74c\uc5d0 \ub4e4\uc9c0 \uc54a\ub294 \uae30\uc0ac\uac00 \uc788\uc744 \ub54c\ub294 \ubc30\ud3ec \uc804 \uc9c0\uba74\uc5d0\uc11c \ube60\uc84c\ub2e4. \uc678\uc2e0 \uae30\uc790\ub4e4\uc774 \uc4f0\ub294 \uae30\uc0ac\ub3c4 \uac10\uc2dc\uc758 \ub300\uc0c1\uc774\uc5c8\ub2e4. \uc8fc\uac70 \ube44\uc790\uac00 \ub098\uc624\ub294 \uac83\ub3c4 \uba87 \ub2ec\uc529 \uac78\ub824 30\uc77c\ub9c8\ub2e4 \ucd9c\uc785\uad6d\uc744 \ubc18\ubcf5\ud588\ub2e4\"\ub77c\uace0 \ub2f9\uc2dc\ub97c \ud68c\uc0c1\ud588\ub2e4.", "answer": "30\uc77c", "sentence": "\uc8fc\uac70 \ube44\uc790\uac00 \ub098\uc624\ub294 \uac83\ub3c4 \uba87 \ub2ec\uc529 \uac78\ub824 30\uc77c \ub9c8\ub2e4 \ucd9c\uc785\uad6d\uc744 \ubc18\ubcf5\ud588\ub2e4\"\ub77c\uace0 \ub2f9\uc2dc\ub97c \ud68c\uc0c1\ud588\ub2e4.", "paragraph_sentence": "\ud55c\uad6d\uc804\uc7c1 \uc774\ud6c4 \uc11c\uc6b8 \uc0c1\uc8fc \ud2b9\ud30c\uc6d0\uc73c\ub85c \uc77c\ud55c \ucd5c\ucd08\uc758 \uc11c\uc591\uc778 \uae30\uc790\uc778 \ub178\uba3c \uc18c\ud504\ub294 \uc544\uc2dc\uc544\uc6d4\uc2a4\ud2b8\ub9ac\ud2b8\uc800\ub110(AWSJ) \uae30\uc790\ub85c \uadfc\ubb34\ud558\ub358 \uc911 5\u00b718 \ubbfc\uc8fc\ud654 \uc6b4\ub3d9\uc744 \uacaa\uc5c8\ub2e4. \uadf8\ub294 \"5\u00b718 \ubbfc\uc8fc\ud654 \uc6b4\ub3d9\uc740 \ub300\ud55c\ubbfc\uad6d\uc758 \ubbfc\uc8fc\ud654\ub97c \ud5a5\ud55c \uae38\uace0 \uae34 \ud22c\uc7c1\uc758 \uc77c\ubd80\ubd84\uc774\ub2e4. \uc55e \uc138\ub300\uac00 \uc790\uc720\uc120\uac70\ub97c \ud655\ub9bd\ud558\uace0 \ubbfc\uc8fc\uc8fc\uc758\ub97c \uaf43\ud53c\uc6b0\ub824\uace0 \uc5bc\ub9c8\ub098 \ub9ce\uc740 \uc5b4\ub824\uc6c0\uc744 \uacaa\uc5c8\ub294\uc9c0 \uc9c0\uae08 \uc80a\uc740 \uc138\ub300\uac00 \ubc30\uc6b0\uace0 \uc9c4\uc2ec\uc73c\ub85c \uac10\uc0ac\ud558\uae38 \ubc14\ub780\ub2e4\"\uba70 \"\uc9c0\uae08\uc740 \uc5b8\ub860\uacc4\uc5d0\uc11c \uc740\ud1f4\ud588\uc9c0\ub9cc \ubbfc\uc8fc\ud654 \uc6b4\ub3d9 \ub54c \ub300\ud55c\ubbfc\uad6d \uc815\ubd80\uc758 \ub9ce\uc740 \uc555\ub825\uc5d0\ub3c4 \uc800\ub97c \ube44\ub86f\ud55c \ub9ce\uc740 \ud5cc\uc2e0\uc801\uc778 \uae30\uc790\ub4e4\uc774 \uc678\uad6d \ub3c5\uc790\ub4e4\uc5d0\uac8c \ub300\ud55c\ubbfc\uad6d \uc0c1\ud669\uc744 \uc54c\ub9ac\ub824\uace0 \ub178\ub825\ud588\ub2e4. \uc2dc\ubbfc\uc744 \uc9c4\uc555\ud558\ub358 \uad70\uc778\ub4e4\uc758 \uc794\ud639\ud568\uc774 \uc2dc\uc704\ub97c \ucd09\ubc1c\ud588\ub2e4\ub294 \uac83\uc744 \uc54c\uac8c \ub410\ub2e4\"\ub77c\uace0 \ub9d0\ud588\ub2e4. \ub610\ud55c \"\uc678\uad6d \uc7a1\uc9c0\ub098 \uc2e0\ubb38\uc744 \ub300\ud55c\ubbfc\uad6d\uc5d0 \ub4e4\uc5ec\uc624\ub294 \uacfc\uc815\uc5d0\uc11c \uad70 \uad00\uacc4\uc790\ub4e4\uc774 \ubc30\ud3ec \uc804 \ubaa8\ub4e0 \uc790\ub8cc\ub97c \uac80\uc5f4\ud588\ub2e4. \uad11\uc8fc\uc5d0 \ub300\ud55c \uae30\uc0ac\ub098 \uc804\ub450\ud658\uc744 \ube44\ud310\ud558\ub294 \uae30\uc0ac \ub4f1 \ub9c8\uc74c\uc5d0 \ub4e4\uc9c0 \uc54a\ub294 \uae30\uc0ac\uac00 \uc788\uc744 \ub54c\ub294 \ubc30\ud3ec \uc804 \uc9c0\uba74\uc5d0\uc11c \ube60\uc84c\ub2e4. \uc678\uc2e0 \uae30\uc790\ub4e4\uc774 \uc4f0\ub294 \uae30\uc0ac\ub3c4 \uac10\uc2dc\uc758 \ub300\uc0c1\uc774\uc5c8\ub2e4. \uc8fc\uac70 \ube44\uc790\uac00 \ub098\uc624\ub294 \uac83\ub3c4 \uba87 \ub2ec\uc529 \uac78\ub824 30\uc77c \ub9c8\ub2e4 \ucd9c\uc785\uad6d\uc744 \ubc18\ubcf5\ud588\ub2e4\"\ub77c\uace0 \ub2f9\uc2dc\ub97c \ud68c\uc0c1\ud588\ub2e4. ", "paragraph_answer": "\ud55c\uad6d\uc804\uc7c1 \uc774\ud6c4 \uc11c\uc6b8 \uc0c1\uc8fc \ud2b9\ud30c\uc6d0\uc73c\ub85c \uc77c\ud55c \ucd5c\ucd08\uc758 \uc11c\uc591\uc778 \uae30\uc790\uc778 \ub178\uba3c \uc18c\ud504\ub294 \uc544\uc2dc\uc544\uc6d4\uc2a4\ud2b8\ub9ac\ud2b8\uc800\ub110(AWSJ) \uae30\uc790\ub85c \uadfc\ubb34\ud558\ub358 \uc911 5\u00b718 \ubbfc\uc8fc\ud654 \uc6b4\ub3d9\uc744 \uacaa\uc5c8\ub2e4. \uadf8\ub294 \"5\u00b718 \ubbfc\uc8fc\ud654 \uc6b4\ub3d9\uc740 \ub300\ud55c\ubbfc\uad6d\uc758 \ubbfc\uc8fc\ud654\ub97c \ud5a5\ud55c \uae38\uace0 \uae34 \ud22c\uc7c1\uc758 \uc77c\ubd80\ubd84\uc774\ub2e4. \uc55e \uc138\ub300\uac00 \uc790\uc720\uc120\uac70\ub97c \ud655\ub9bd\ud558\uace0 \ubbfc\uc8fc\uc8fc\uc758\ub97c \uaf43\ud53c\uc6b0\ub824\uace0 \uc5bc\ub9c8\ub098 \ub9ce\uc740 \uc5b4\ub824\uc6c0\uc744 \uacaa\uc5c8\ub294\uc9c0 \uc9c0\uae08 \uc80a\uc740 \uc138\ub300\uac00 \ubc30\uc6b0\uace0 \uc9c4\uc2ec\uc73c\ub85c \uac10\uc0ac\ud558\uae38 \ubc14\ub780\ub2e4\"\uba70 \"\uc9c0\uae08\uc740 \uc5b8\ub860\uacc4\uc5d0\uc11c \uc740\ud1f4\ud588\uc9c0\ub9cc \ubbfc\uc8fc\ud654 \uc6b4\ub3d9 \ub54c \ub300\ud55c\ubbfc\uad6d \uc815\ubd80\uc758 \ub9ce\uc740 \uc555\ub825\uc5d0\ub3c4 \uc800\ub97c \ube44\ub86f\ud55c \ub9ce\uc740 \ud5cc\uc2e0\uc801\uc778 \uae30\uc790\ub4e4\uc774 \uc678\uad6d \ub3c5\uc790\ub4e4\uc5d0\uac8c \ub300\ud55c\ubbfc\uad6d \uc0c1\ud669\uc744 \uc54c\ub9ac\ub824\uace0 \ub178\ub825\ud588\ub2e4. \uc2dc\ubbfc\uc744 \uc9c4\uc555\ud558\ub358 \uad70\uc778\ub4e4\uc758 \uc794\ud639\ud568\uc774 \uc2dc\uc704\ub97c \ucd09\ubc1c\ud588\ub2e4\ub294 \uac83\uc744 \uc54c\uac8c \ub410\ub2e4\"\ub77c\uace0 \ub9d0\ud588\ub2e4. \ub610\ud55c \"\uc678\uad6d \uc7a1\uc9c0\ub098 \uc2e0\ubb38\uc744 \ub300\ud55c\ubbfc\uad6d\uc5d0 \ub4e4\uc5ec\uc624\ub294 \uacfc\uc815\uc5d0\uc11c \uad70 \uad00\uacc4\uc790\ub4e4\uc774 \ubc30\ud3ec \uc804 \ubaa8\ub4e0 \uc790\ub8cc\ub97c \uac80\uc5f4\ud588\ub2e4. \uad11\uc8fc\uc5d0 \ub300\ud55c \uae30\uc0ac\ub098 \uc804\ub450\ud658\uc744 \ube44\ud310\ud558\ub294 \uae30\uc0ac \ub4f1 \ub9c8\uc74c\uc5d0 \ub4e4\uc9c0 \uc54a\ub294 \uae30\uc0ac\uac00 \uc788\uc744 \ub54c\ub294 \ubc30\ud3ec \uc804 \uc9c0\uba74\uc5d0\uc11c \ube60\uc84c\ub2e4. \uc678\uc2e0 \uae30\uc790\ub4e4\uc774 \uc4f0\ub294 \uae30\uc0ac\ub3c4 \uac10\uc2dc\uc758 \ub300\uc0c1\uc774\uc5c8\ub2e4. \uc8fc\uac70 \ube44\uc790\uac00 \ub098\uc624\ub294 \uac83\ub3c4 \uba87 \ub2ec\uc529 \uac78\ub824 30\uc77c \ub9c8\ub2e4 \ucd9c\uc785\uad6d\uc744 \ubc18\ubcf5\ud588\ub2e4\"\ub77c\uace0 \ub2f9\uc2dc\ub97c \ud68c\uc0c1\ud588\ub2e4.", "sentence_answer": "\uc8fc\uac70 \ube44\uc790\uac00 \ub098\uc624\ub294 \uac83\ub3c4 \uba87 \ub2ec\uc529 \uac78\ub824 30\uc77c \ub9c8\ub2e4 \ucd9c\uc785\uad6d\uc744 \ubc18\ubcf5\ud588\ub2e4\"\ub77c\uace0 \ub2f9\uc2dc\ub97c \ud68c\uc0c1\ud588\ub2e4.", "paragraph_id": "5801882-24-2", "paragraph_question": "question: \ub178\uba3c \uc18c\ud504\ub294 \uc8fc\uac70\ube44\uc790 \ub54c\ubb38\uc5d0 \uba87\uc77c\ub9c8\ub2e4 \ucd9c\uc785\uad6d\uc744 \ubc18\ubcf5\ud588\ub098\uc694?, context: \ud55c\uad6d\uc804\uc7c1 \uc774\ud6c4 \uc11c\uc6b8 \uc0c1\uc8fc \ud2b9\ud30c\uc6d0\uc73c\ub85c \uc77c\ud55c \ucd5c\ucd08\uc758 \uc11c\uc591\uc778 \uae30\uc790\uc778 \ub178\uba3c \uc18c\ud504\ub294 \uc544\uc2dc\uc544\uc6d4\uc2a4\ud2b8\ub9ac\ud2b8\uc800\ub110(AWSJ) \uae30\uc790\ub85c \uadfc\ubb34\ud558\ub358 \uc911 5\u00b718 \ubbfc\uc8fc\ud654 \uc6b4\ub3d9\uc744 \uacaa\uc5c8\ub2e4. \uadf8\ub294 \"5\u00b718 \ubbfc\uc8fc\ud654 \uc6b4\ub3d9\uc740 \ub300\ud55c\ubbfc\uad6d\uc758 \ubbfc\uc8fc\ud654\ub97c \ud5a5\ud55c \uae38\uace0 \uae34 \ud22c\uc7c1\uc758 \uc77c\ubd80\ubd84\uc774\ub2e4. \uc55e \uc138\ub300\uac00 \uc790\uc720\uc120\uac70\ub97c \ud655\ub9bd\ud558\uace0 \ubbfc\uc8fc\uc8fc\uc758\ub97c \uaf43\ud53c\uc6b0\ub824\uace0 \uc5bc\ub9c8\ub098 \ub9ce\uc740 \uc5b4\ub824\uc6c0\uc744 \uacaa\uc5c8\ub294\uc9c0 \uc9c0\uae08 \uc80a\uc740 \uc138\ub300\uac00 \ubc30\uc6b0\uace0 \uc9c4\uc2ec\uc73c\ub85c \uac10\uc0ac\ud558\uae38 \ubc14\ub780\ub2e4\"\uba70 \"\uc9c0\uae08\uc740 \uc5b8\ub860\uacc4\uc5d0\uc11c \uc740\ud1f4\ud588\uc9c0\ub9cc \ubbfc\uc8fc\ud654 \uc6b4\ub3d9 \ub54c \ub300\ud55c\ubbfc\uad6d \uc815\ubd80\uc758 \ub9ce\uc740 \uc555\ub825\uc5d0\ub3c4 \uc800\ub97c \ube44\ub86f\ud55c \ub9ce\uc740 \ud5cc\uc2e0\uc801\uc778 \uae30\uc790\ub4e4\uc774 \uc678\uad6d \ub3c5\uc790\ub4e4\uc5d0\uac8c \ub300\ud55c\ubbfc\uad6d \uc0c1\ud669\uc744 \uc54c\ub9ac\ub824\uace0 \ub178\ub825\ud588\ub2e4. \uc2dc\ubbfc\uc744 \uc9c4\uc555\ud558\ub358 \uad70\uc778\ub4e4\uc758 \uc794\ud639\ud568\uc774 \uc2dc\uc704\ub97c \ucd09\ubc1c\ud588\ub2e4\ub294 \uac83\uc744 \uc54c\uac8c \ub410\ub2e4\"\ub77c\uace0 \ub9d0\ud588\ub2e4. \ub610\ud55c \"\uc678\uad6d \uc7a1\uc9c0\ub098 \uc2e0\ubb38\uc744 \ub300\ud55c\ubbfc\uad6d\uc5d0 \ub4e4\uc5ec\uc624\ub294 \uacfc\uc815\uc5d0\uc11c \uad70 \uad00\uacc4\uc790\ub4e4\uc774 \ubc30\ud3ec \uc804 \ubaa8\ub4e0 \uc790\ub8cc\ub97c \uac80\uc5f4\ud588\ub2e4. \uad11\uc8fc\uc5d0 \ub300\ud55c \uae30\uc0ac\ub098 \uc804\ub450\ud658\uc744 \ube44\ud310\ud558\ub294 \uae30\uc0ac \ub4f1 \ub9c8\uc74c\uc5d0 \ub4e4\uc9c0 \uc54a\ub294 \uae30\uc0ac\uac00 \uc788\uc744 \ub54c\ub294 \ubc30\ud3ec \uc804 \uc9c0\uba74\uc5d0\uc11c \ube60\uc84c\ub2e4. \uc678\uc2e0 \uae30\uc790\ub4e4\uc774 \uc4f0\ub294 \uae30\uc0ac\ub3c4 \uac10\uc2dc\uc758 \ub300\uc0c1\uc774\uc5c8\ub2e4. \uc8fc\uac70 \ube44\uc790\uac00 \ub098\uc624\ub294 \uac83\ub3c4 \uba87 \ub2ec\uc529 \uac78\ub824 30\uc77c\ub9c8\ub2e4 \ucd9c\uc785\uad6d\uc744 \ubc18\ubcf5\ud588\ub2e4\"\ub77c\uace0 \ub2f9\uc2dc\ub97c \ud68c\uc0c1\ud588\ub2e4."} {"question": "\uac80\ucc30\uc774 \ubd80\ub3d9\uc0b0 \ub9e4\ub9e4 \uc758\ud639\uac00 \uad00\ub828\ud574 \ubb34\ud610\uc758 \ucc98\ubd84\uc744 \ub0b4\ub9b0 \ub0a0\uc740 \uc5b8\uc81c\uc778\uac00?", "paragraph": "2016\ub144 7\uc6d4 18\uc77c \uc870\uc120\uc77c\ubcf4\uc758 \uae30\uc0ac\uc5d0 \ub530\ub974\uba74 \uc7a5\uc778\uc778 \uc774\uc0c1\ub2ec \ud68c\uc7a5\uc774 \uc790\uc2e0\uc758 \ub124 \ub538\uc5d0\uac8c \uc0c1\uc18d\ud55c 1300\uc5b5 \uc6d0\ub300 \ubd80\ub3d9\uc0b0\uc744 \ub125\uc2a8\uc774 \ub9e4\uc785\ud574\uc92c\ub2e4\ub294 \uae30\uc0ac\uac00 \ubcf4\ub3c4\ub418\uc5c8\ub2e4. \uc774\uc0c1\ub2ec\uc758 \uc0ac\ub9dd \ud6c4 \uc0c1\uc18d\uc138 \ub0a9\ubd80 \ub4f1\uc744 \uc704\ud574 \uc774 \ubd80\ub3d9\uc0b0\uc744 \ud314\ub824\uace0 \ub0b4\ub193\uc558\uc9c0\ub9cc 2\ub144 \ub118\uac8c \ud314\ub9ac\uc9c0 \uc54a\uc544 \uac70\uc561\uc758 \uc0c1\uc18d\uc138\ub85c \uace0\ubbfc\ud558\ub358 \uc911\uc5d0 \ub125\uc2a8\uc774 \uc774 \ubd80\uc9c0\ub97c \ub9e4\uc785\ud574 \uc92c\ub2e4\ub294 \uac83\uc774\ub2e4. \uadf8\ub7fc\uc73c\ub85c\uc368 \uae40\uc815\uc8fc \ub125\uc2a8 \ub300\ud45c\uc640 \uc808\uce5c\ud55c \uc0ac\uc774\uc600\ub358 \uc9c4\uacbd\uc900 \uac80\uc0ac\uc7a5\uc758 \uc8fc\uc120\uc73c\ub85c \ubd80\ub3d9\uc0b0 \uac70\ub798\uac00 \uc774\ub8e8\uc5b4\uc9c4 \uac83\uc774 \uc544\ub2c8\ub0d0\ub294 \uc758\ud639\uc774 \uc77c\uc5c8\ub2e4. \uc774\ubbf8 \ub125\uc2a8\uc740 \uc9c4\uacbd\uc900\uacfc\uc758 '\uc8fc\uc2dd \ud30c\ubb38'\uc73c\ub85c \ubb38\uc81c\uac00 \ub418\uace0 \uc788\ub294 \uc640\uc911\uc774\uc5c8\ub294\ub370, \uc774 \uc0ac\uac74\uc774 \ubd88\uac70\uc9c0\uba74\uc11c \uae40\uc815\uc8fc\uac00 \uc9c4\uacbd\uc900 \uc678\uc5d0\ub3c4 \uc6b0\ubcd1\uc6b0\uc640\ub3c4 \uc704\ubc95\uc801 \uc5f0\uacb0\uc774 \uc788\ub294 \uac83\uc774 \uc544\ub2c8\ub0d0\ub294 \uc758\uc2ec\uc744 \uc0c0\ub2e4. \uc774\uc5d0 \uc6b0\ubcd1\uc6b0 \ubbfc\uc815\uc218\uc11d\uc740 \uc870\uc120\uc77c\ubcf4 \ubcf4\ub3c4\ub294 \uc0ac\uc2e4\ubb34\uadfc\uc774\ub77c\uba70 \ubaa8\ub4e0 \uc758\ud639\uc744 \uc77c\ucd95\ud558\uace0 \uc758\ud639\uc744 \ubcf4\ub3c4\ud55c \uc870\uc120\uc77c\ubcf4 \uae30\uc790\ub97c \ucd9c\ud310\ubb3c\uc5d0 \uc758\ud55c \uba85\uc608\ud6fc\uc190\uc73c\ub85c \uc11c\uc6b8\uc911\uc559\uc9c0\uac80\uc5d0 \uace0\uc18c\ud588\ub2e4. 2017\ub144 4\uc6d4 17\uc77c \uac80\ucc30\uc740 \ubd80\ub3d9\uc0b0 \ub9e4\ub9e4 \uc758\ud639\uc5d0 \uad00\ud574 \ubb34\ud610\uc758 \ucc98\ubd84\uc744 \ub0b4\ub838\ub2e4.", "answer": "2017\ub144 4\uc6d4 17\uc77c", "sentence": "2017\ub144 4\uc6d4 17\uc77c \uac80\ucc30\uc740 \ubd80\ub3d9\uc0b0 \ub9e4\ub9e4 \uc758\ud639\uc5d0 \uad00\ud574 \ubb34\ud610\uc758 \ucc98\ubd84\uc744 \ub0b4\ub838\ub2e4.", "paragraph_sentence": "2016\ub144 7\uc6d4 18\uc77c \uc870\uc120\uc77c\ubcf4\uc758 \uae30\uc0ac\uc5d0 \ub530\ub974\uba74 \uc7a5\uc778\uc778 \uc774\uc0c1\ub2ec \ud68c\uc7a5\uc774 \uc790\uc2e0\uc758 \ub124 \ub538\uc5d0\uac8c \uc0c1\uc18d\ud55c 1300\uc5b5 \uc6d0\ub300 \ubd80\ub3d9\uc0b0\uc744 \ub125\uc2a8\uc774 \ub9e4\uc785\ud574\uc92c\ub2e4\ub294 \uae30\uc0ac\uac00 \ubcf4\ub3c4\ub418\uc5c8\ub2e4. \uc774\uc0c1\ub2ec\uc758 \uc0ac\ub9dd \ud6c4 \uc0c1\uc18d\uc138 \ub0a9\ubd80 \ub4f1\uc744 \uc704\ud574 \uc774 \ubd80\ub3d9\uc0b0\uc744 \ud314\ub824\uace0 \ub0b4\ub193\uc558\uc9c0\ub9cc 2\ub144 \ub118\uac8c \ud314\ub9ac\uc9c0 \uc54a\uc544 \uac70\uc561\uc758 \uc0c1\uc18d\uc138\ub85c \uace0\ubbfc\ud558\ub358 \uc911\uc5d0 \ub125\uc2a8\uc774 \uc774 \ubd80\uc9c0\ub97c \ub9e4\uc785\ud574 \uc92c\ub2e4\ub294 \uac83\uc774\ub2e4. \uadf8\ub7fc\uc73c\ub85c\uc368 \uae40\uc815\uc8fc \ub125\uc2a8 \ub300\ud45c\uc640 \uc808\uce5c\ud55c \uc0ac\uc774\uc600\ub358 \uc9c4\uacbd\uc900 \uac80\uc0ac\uc7a5\uc758 \uc8fc\uc120\uc73c\ub85c \ubd80\ub3d9\uc0b0 \uac70\ub798\uac00 \uc774\ub8e8\uc5b4\uc9c4 \uac83\uc774 \uc544\ub2c8\ub0d0\ub294 \uc758\ud639\uc774 \uc77c\uc5c8\ub2e4. \uc774\ubbf8 \ub125\uc2a8\uc740 \uc9c4\uacbd\uc900\uacfc\uc758 '\uc8fc\uc2dd \ud30c\ubb38'\uc73c\ub85c \ubb38\uc81c\uac00 \ub418\uace0 \uc788\ub294 \uc640\uc911\uc774\uc5c8\ub294\ub370, \uc774 \uc0ac\uac74\uc774 \ubd88\uac70\uc9c0\uba74\uc11c \uae40\uc815\uc8fc\uac00 \uc9c4\uacbd\uc900 \uc678\uc5d0\ub3c4 \uc6b0\ubcd1\uc6b0\uc640\ub3c4 \uc704\ubc95\uc801 \uc5f0\uacb0\uc774 \uc788\ub294 \uac83\uc774 \uc544\ub2c8\ub0d0\ub294 \uc758\uc2ec\uc744 \uc0c0\ub2e4. \uc774\uc5d0 \uc6b0\ubcd1\uc6b0 \ubbfc\uc815\uc218\uc11d\uc740 \uc870\uc120\uc77c\ubcf4 \ubcf4\ub3c4\ub294 \uc0ac\uc2e4\ubb34\uadfc\uc774\ub77c\uba70 \ubaa8\ub4e0 \uc758\ud639\uc744 \uc77c\ucd95\ud558\uace0 \uc758\ud639\uc744 \ubcf4\ub3c4\ud55c \uc870\uc120\uc77c\ubcf4 \uae30\uc790\ub97c \ucd9c\ud310\ubb3c\uc5d0 \uc758\ud55c \uba85\uc608\ud6fc\uc190\uc73c\ub85c \uc11c\uc6b8\uc911\uc559\uc9c0\uac80\uc5d0 \uace0\uc18c\ud588\ub2e4. 2017\ub144 4\uc6d4 17\uc77c \uac80\ucc30\uc740 \ubd80\ub3d9\uc0b0 \ub9e4\ub9e4 \uc758\ud639\uc5d0 \uad00\ud574 \ubb34\ud610\uc758 \ucc98\ubd84\uc744 \ub0b4\ub838\ub2e4. ", "paragraph_answer": "2016\ub144 7\uc6d4 18\uc77c \uc870\uc120\uc77c\ubcf4\uc758 \uae30\uc0ac\uc5d0 \ub530\ub974\uba74 \uc7a5\uc778\uc778 \uc774\uc0c1\ub2ec \ud68c\uc7a5\uc774 \uc790\uc2e0\uc758 \ub124 \ub538\uc5d0\uac8c \uc0c1\uc18d\ud55c 1300\uc5b5 \uc6d0\ub300 \ubd80\ub3d9\uc0b0\uc744 \ub125\uc2a8\uc774 \ub9e4\uc785\ud574\uc92c\ub2e4\ub294 \uae30\uc0ac\uac00 \ubcf4\ub3c4\ub418\uc5c8\ub2e4. \uc774\uc0c1\ub2ec\uc758 \uc0ac\ub9dd \ud6c4 \uc0c1\uc18d\uc138 \ub0a9\ubd80 \ub4f1\uc744 \uc704\ud574 \uc774 \ubd80\ub3d9\uc0b0\uc744 \ud314\ub824\uace0 \ub0b4\ub193\uc558\uc9c0\ub9cc 2\ub144 \ub118\uac8c \ud314\ub9ac\uc9c0 \uc54a\uc544 \uac70\uc561\uc758 \uc0c1\uc18d\uc138\ub85c \uace0\ubbfc\ud558\ub358 \uc911\uc5d0 \ub125\uc2a8\uc774 \uc774 \ubd80\uc9c0\ub97c \ub9e4\uc785\ud574 \uc92c\ub2e4\ub294 \uac83\uc774\ub2e4. \uadf8\ub7fc\uc73c\ub85c\uc368 \uae40\uc815\uc8fc \ub125\uc2a8 \ub300\ud45c\uc640 \uc808\uce5c\ud55c \uc0ac\uc774\uc600\ub358 \uc9c4\uacbd\uc900 \uac80\uc0ac\uc7a5\uc758 \uc8fc\uc120\uc73c\ub85c \ubd80\ub3d9\uc0b0 \uac70\ub798\uac00 \uc774\ub8e8\uc5b4\uc9c4 \uac83\uc774 \uc544\ub2c8\ub0d0\ub294 \uc758\ud639\uc774 \uc77c\uc5c8\ub2e4. \uc774\ubbf8 \ub125\uc2a8\uc740 \uc9c4\uacbd\uc900\uacfc\uc758 '\uc8fc\uc2dd \ud30c\ubb38'\uc73c\ub85c \ubb38\uc81c\uac00 \ub418\uace0 \uc788\ub294 \uc640\uc911\uc774\uc5c8\ub294\ub370, \uc774 \uc0ac\uac74\uc774 \ubd88\uac70\uc9c0\uba74\uc11c \uae40\uc815\uc8fc\uac00 \uc9c4\uacbd\uc900 \uc678\uc5d0\ub3c4 \uc6b0\ubcd1\uc6b0\uc640\ub3c4 \uc704\ubc95\uc801 \uc5f0\uacb0\uc774 \uc788\ub294 \uac83\uc774 \uc544\ub2c8\ub0d0\ub294 \uc758\uc2ec\uc744 \uc0c0\ub2e4. \uc774\uc5d0 \uc6b0\ubcd1\uc6b0 \ubbfc\uc815\uc218\uc11d\uc740 \uc870\uc120\uc77c\ubcf4 \ubcf4\ub3c4\ub294 \uc0ac\uc2e4\ubb34\uadfc\uc774\ub77c\uba70 \ubaa8\ub4e0 \uc758\ud639\uc744 \uc77c\ucd95\ud558\uace0 \uc758\ud639\uc744 \ubcf4\ub3c4\ud55c \uc870\uc120\uc77c\ubcf4 \uae30\uc790\ub97c \ucd9c\ud310\ubb3c\uc5d0 \uc758\ud55c \uba85\uc608\ud6fc\uc190\uc73c\ub85c \uc11c\uc6b8\uc911\uc559\uc9c0\uac80\uc5d0 \uace0\uc18c\ud588\ub2e4. 2017\ub144 4\uc6d4 17\uc77c \uac80\ucc30\uc740 \ubd80\ub3d9\uc0b0 \ub9e4\ub9e4 \uc758\ud639\uc5d0 \uad00\ud574 \ubb34\ud610\uc758 \ucc98\ubd84\uc744 \ub0b4\ub838\ub2e4.", "sentence_answer": " 2017\ub144 4\uc6d4 17\uc77c \uac80\ucc30\uc740 \ubd80\ub3d9\uc0b0 \ub9e4\ub9e4 \uc758\ud639\uc5d0 \uad00\ud574 \ubb34\ud610\uc758 \ucc98\ubd84\uc744 \ub0b4\ub838\ub2e4.", "paragraph_id": "6526264-3-1", "paragraph_question": "question: \uac80\ucc30\uc774 \ubd80\ub3d9\uc0b0 \ub9e4\ub9e4 \uc758\ud639\uac00 \uad00\ub828\ud574 \ubb34\ud610\uc758 \ucc98\ubd84\uc744 \ub0b4\ub9b0 \ub0a0\uc740 \uc5b8\uc81c\uc778\uac00?, context: 2016\ub144 7\uc6d4 18\uc77c \uc870\uc120\uc77c\ubcf4\uc758 \uae30\uc0ac\uc5d0 \ub530\ub974\uba74 \uc7a5\uc778\uc778 \uc774\uc0c1\ub2ec \ud68c\uc7a5\uc774 \uc790\uc2e0\uc758 \ub124 \ub538\uc5d0\uac8c \uc0c1\uc18d\ud55c 1300\uc5b5 \uc6d0\ub300 \ubd80\ub3d9\uc0b0\uc744 \ub125\uc2a8\uc774 \ub9e4\uc785\ud574\uc92c\ub2e4\ub294 \uae30\uc0ac\uac00 \ubcf4\ub3c4\ub418\uc5c8\ub2e4. \uc774\uc0c1\ub2ec\uc758 \uc0ac\ub9dd \ud6c4 \uc0c1\uc18d\uc138 \ub0a9\ubd80 \ub4f1\uc744 \uc704\ud574 \uc774 \ubd80\ub3d9\uc0b0\uc744 \ud314\ub824\uace0 \ub0b4\ub193\uc558\uc9c0\ub9cc 2\ub144 \ub118\uac8c \ud314\ub9ac\uc9c0 \uc54a\uc544 \uac70\uc561\uc758 \uc0c1\uc18d\uc138\ub85c \uace0\ubbfc\ud558\ub358 \uc911\uc5d0 \ub125\uc2a8\uc774 \uc774 \ubd80\uc9c0\ub97c \ub9e4\uc785\ud574 \uc92c\ub2e4\ub294 \uac83\uc774\ub2e4. \uadf8\ub7fc\uc73c\ub85c\uc368 \uae40\uc815\uc8fc \ub125\uc2a8 \ub300\ud45c\uc640 \uc808\uce5c\ud55c \uc0ac\uc774\uc600\ub358 \uc9c4\uacbd\uc900 \uac80\uc0ac\uc7a5\uc758 \uc8fc\uc120\uc73c\ub85c \ubd80\ub3d9\uc0b0 \uac70\ub798\uac00 \uc774\ub8e8\uc5b4\uc9c4 \uac83\uc774 \uc544\ub2c8\ub0d0\ub294 \uc758\ud639\uc774 \uc77c\uc5c8\ub2e4. \uc774\ubbf8 \ub125\uc2a8\uc740 \uc9c4\uacbd\uc900\uacfc\uc758 '\uc8fc\uc2dd \ud30c\ubb38'\uc73c\ub85c \ubb38\uc81c\uac00 \ub418\uace0 \uc788\ub294 \uc640\uc911\uc774\uc5c8\ub294\ub370, \uc774 \uc0ac\uac74\uc774 \ubd88\uac70\uc9c0\uba74\uc11c \uae40\uc815\uc8fc\uac00 \uc9c4\uacbd\uc900 \uc678\uc5d0\ub3c4 \uc6b0\ubcd1\uc6b0\uc640\ub3c4 \uc704\ubc95\uc801 \uc5f0\uacb0\uc774 \uc788\ub294 \uac83\uc774 \uc544\ub2c8\ub0d0\ub294 \uc758\uc2ec\uc744 \uc0c0\ub2e4. \uc774\uc5d0 \uc6b0\ubcd1\uc6b0 \ubbfc\uc815\uc218\uc11d\uc740 \uc870\uc120\uc77c\ubcf4 \ubcf4\ub3c4\ub294 \uc0ac\uc2e4\ubb34\uadfc\uc774\ub77c\uba70 \ubaa8\ub4e0 \uc758\ud639\uc744 \uc77c\ucd95\ud558\uace0 \uc758\ud639\uc744 \ubcf4\ub3c4\ud55c \uc870\uc120\uc77c\ubcf4 \uae30\uc790\ub97c \ucd9c\ud310\ubb3c\uc5d0 \uc758\ud55c \uba85\uc608\ud6fc\uc190\uc73c\ub85c \uc11c\uc6b8\uc911\uc559\uc9c0\uac80\uc5d0 \uace0\uc18c\ud588\ub2e4. 2017\ub144 4\uc6d4 17\uc77c \uac80\ucc30\uc740 \ubd80\ub3d9\uc0b0 \ub9e4\ub9e4 \uc758\ud639\uc5d0 \uad00\ud574 \ubb34\ud610\uc758 \ucc98\ubd84\uc744 \ub0b4\ub838\ub2e4."} {"question": "\ud0a4\uc608\ud504 \uc804\ud22c\uac00 \uc9c4\ud589\ub41c \uc9c0\uc5ed\uc740?", "paragraph": "\ud0a4\uc608\ud504 \uc804\ud22c(\ub7ec\uc2dc\uc544\uc5b4: \u0411\u0438\u0442\u0432\u0430 \u043f\u043e\u0434 \u041a\u0438\u0435\u0432\u043e\u043c, \ub3c5\uc77c\uc5b4: Schlacht um Kiew)\ub294 \uc81c2\ucc28 \uc138\uacc4 \ub300\uc804 \uc911 \ub3c5\uc77c\uad70\uc774 \uc6b0\ud06c\ub77c\uc774\ub098\uc5d0\uc11c \uc9c4\ud589\ud55c \uc18c\ub828 \uad70\ub300\uc5d0 \ub300\ud55c \uac70\ub300\ud55c \ud3ec\uc704\uc804\uc758 \uc774\ub984\uc774\ub2e4. \uc774 \uc804\ud22c\ub294 \uc5ed\uc0ac\uc801\uc73c\ub85c \uac00\uc7a5 \ub9ce\uc740 \uad70\uc774 \ud3ec\uc704\ub41c \uc804\ud22c\ub85c \uc54c\ub824\uc838 \uc788\ub2e4. \uc774 \uc791\uc804\uc740 \ubc14\ub974\ubc14\ub85c\uc0ac \uc791\uc804\uc758 \uc77c\ubd80\ub85c 1941\ub144 8\uc6d4 23\uc77c\ubd80\ud130 9\uc6d4 26\uc77c\uae4c\uc9c0 \uc9c4\ud589\ub418\uc5c8\ub2e4. \uc18c\ub828 \uad70\uc0ac\uc0ac\uc5d0\uc11c\ub294 \uc774 \uc804\ud22c\ub97c \ud0a4\uc608\ud504 \ubc29\uc5b4\uc804(\ub7ec\uc2dc\uc544\uc5b4: \u041a\u0438\u0435\u0432\u0441\u043a\u0430\u044f \u043e\u0431\u043e\u0440\u043e\u043d\u0438\u0442\u0435\u043b\u044c\u043d\u0430\u044f \u043e\u043f\u0435\u0440\u0430\u0446\u0438\u044f) \ub610\ub294 \ud0a4\uc608\ud504 \uc804\ub7b5\ubc29\uc5b4\uc804(\ub7ec\uc2dc\uc544\uc5b4: \u041a\u0438\u0435\u0432\u0441\u043a\u0430\u044f \u0441\u0442\u0440\u0430\u0442\u0435\u0433\u0438\u0447\u0435\u0441\u043a\u0430\u044f \u043e\u0431\u043e\u0440\u043e\u043d\u0438\u0442\u0435\u043b\u044c\u043d\u0430\u044f \u043e\u043f\u0435\u0440\u0430\u0446\u0438\u044f)\uc774\ub77c\uace0 \ubd80\ub974\uba70, \ub0a0\uc9dc\ub85c 7\uc6d4 7\uc77c\ubd80\ud130 9\uc6d4 26\uc77c\uae4c\uc9c0 \ub2e4\ub974\uac8c \uc7a1\uace0 \uc788\ub2e4.", "answer": "\uc6b0\ud06c\ub77c\uc774\ub098", "sentence": "\ud0a4\uc608\ud504 \uc804\ud22c(\ub7ec\uc2dc\uc544\uc5b4: \u0411\u0438\u0442\u0432\u0430 \u043f\u043e\u0434 \u041a\u0438\u0435\u0432\u043e\u043c, \ub3c5\uc77c\uc5b4: Schlacht um Kiew)\ub294 \uc81c2\ucc28 \uc138\uacc4 \ub300\uc804 \uc911 \ub3c5\uc77c\uad70\uc774 \uc6b0\ud06c\ub77c\uc774\ub098 \uc5d0\uc11c \uc9c4\ud589\ud55c \uc18c\ub828 \uad70\ub300\uc5d0 \ub300\ud55c \uac70\ub300\ud55c \ud3ec\uc704\uc804\uc758 \uc774\ub984\uc774\ub2e4.", "paragraph_sentence": " \ud0a4\uc608\ud504 \uc804\ud22c(\ub7ec\uc2dc\uc544\uc5b4: \u0411\u0438\u0442\u0432\u0430 \u043f\u043e\u0434 \u041a\u0438\u0435\u0432\u043e\u043c, \ub3c5\uc77c\uc5b4: Schlacht um Kiew)\ub294 \uc81c2\ucc28 \uc138\uacc4 \ub300\uc804 \uc911 \ub3c5\uc77c\uad70\uc774 \uc6b0\ud06c\ub77c\uc774\ub098 \uc5d0\uc11c \uc9c4\ud589\ud55c \uc18c\ub828 \uad70\ub300\uc5d0 \ub300\ud55c \uac70\ub300\ud55c \ud3ec\uc704\uc804\uc758 \uc774\ub984\uc774\ub2e4. \uc774 \uc804\ud22c\ub294 \uc5ed\uc0ac\uc801\uc73c\ub85c \uac00\uc7a5 \ub9ce\uc740 \uad70\uc774 \ud3ec\uc704\ub41c \uc804\ud22c\ub85c \uc54c\ub824\uc838 \uc788\ub2e4. \uc774 \uc791\uc804\uc740 \ubc14\ub974\ubc14\ub85c\uc0ac \uc791\uc804\uc758 \uc77c\ubd80\ub85c 1941\ub144 8\uc6d4 23\uc77c\ubd80\ud130 9\uc6d4 26\uc77c\uae4c\uc9c0 \uc9c4\ud589\ub418\uc5c8\ub2e4. \uc18c\ub828 \uad70\uc0ac\uc0ac\uc5d0\uc11c\ub294 \uc774 \uc804\ud22c\ub97c \ud0a4\uc608\ud504 \ubc29\uc5b4\uc804(\ub7ec\uc2dc\uc544\uc5b4: \u041a\u0438\u0435\u0432\u0441\u043a\u0430\u044f \u043e\u0431\u043e\u0440\u043e\u043d\u0438\u0442\u0435\u043b\u044c\u043d\u0430\u044f \u043e\u043f\u0435\u0440\u0430\u0446\u0438\u044f) \ub610\ub294 \ud0a4\uc608\ud504 \uc804\ub7b5\ubc29\uc5b4\uc804(\ub7ec\uc2dc\uc544\uc5b4: \u041a\u0438\u0435\u0432\u0441\u043a\u0430\u044f \u0441\u0442\u0440\u0430\u0442\u0435\u0433\u0438\u0447\u0435\u0441\u043a\u0430\u044f \u043e\u0431\u043e\u0440\u043e\u043d\u0438\u0442\u0435\u043b\u044c\u043d\u0430\u044f \u043e\u043f\u0435\u0440\u0430\u0446\u0438\u044f)\uc774\ub77c\uace0 \ubd80\ub974\uba70, \ub0a0\uc9dc\ub85c 7\uc6d4 7\uc77c\ubd80\ud130 9\uc6d4 26\uc77c\uae4c\uc9c0 \ub2e4\ub974\uac8c \uc7a1\uace0 \uc788\ub2e4.", "paragraph_answer": "\ud0a4\uc608\ud504 \uc804\ud22c(\ub7ec\uc2dc\uc544\uc5b4: \u0411\u0438\u0442\u0432\u0430 \u043f\u043e\u0434 \u041a\u0438\u0435\u0432\u043e\u043c, \ub3c5\uc77c\uc5b4: Schlacht um Kiew)\ub294 \uc81c2\ucc28 \uc138\uacc4 \ub300\uc804 \uc911 \ub3c5\uc77c\uad70\uc774 \uc6b0\ud06c\ub77c\uc774\ub098 \uc5d0\uc11c \uc9c4\ud589\ud55c \uc18c\ub828 \uad70\ub300\uc5d0 \ub300\ud55c \uac70\ub300\ud55c \ud3ec\uc704\uc804\uc758 \uc774\ub984\uc774\ub2e4. \uc774 \uc804\ud22c\ub294 \uc5ed\uc0ac\uc801\uc73c\ub85c \uac00\uc7a5 \ub9ce\uc740 \uad70\uc774 \ud3ec\uc704\ub41c \uc804\ud22c\ub85c \uc54c\ub824\uc838 \uc788\ub2e4. \uc774 \uc791\uc804\uc740 \ubc14\ub974\ubc14\ub85c\uc0ac \uc791\uc804\uc758 \uc77c\ubd80\ub85c 1941\ub144 8\uc6d4 23\uc77c\ubd80\ud130 9\uc6d4 26\uc77c\uae4c\uc9c0 \uc9c4\ud589\ub418\uc5c8\ub2e4. \uc18c\ub828 \uad70\uc0ac\uc0ac\uc5d0\uc11c\ub294 \uc774 \uc804\ud22c\ub97c \ud0a4\uc608\ud504 \ubc29\uc5b4\uc804(\ub7ec\uc2dc\uc544\uc5b4: \u041a\u0438\u0435\u0432\u0441\u043a\u0430\u044f \u043e\u0431\u043e\u0440\u043e\u043d\u0438\u0442\u0435\u043b\u044c\u043d\u0430\u044f \u043e\u043f\u0435\u0440\u0430\u0446\u0438\u044f) \ub610\ub294 \ud0a4\uc608\ud504 \uc804\ub7b5\ubc29\uc5b4\uc804(\ub7ec\uc2dc\uc544\uc5b4: \u041a\u0438\u0435\u0432\u0441\u043a\u0430\u044f \u0441\u0442\u0440\u0430\u0442\u0435\u0433\u0438\u0447\u0435\u0441\u043a\u0430\u044f \u043e\u0431\u043e\u0440\u043e\u043d\u0438\u0442\u0435\u043b\u044c\u043d\u0430\u044f \u043e\u043f\u0435\u0440\u0430\u0446\u0438\u044f)\uc774\ub77c\uace0 \ubd80\ub974\uba70, \ub0a0\uc9dc\ub85c 7\uc6d4 7\uc77c\ubd80\ud130 9\uc6d4 26\uc77c\uae4c\uc9c0 \ub2e4\ub974\uac8c \uc7a1\uace0 \uc788\ub2e4.", "sentence_answer": "\ud0a4\uc608\ud504 \uc804\ud22c(\ub7ec\uc2dc\uc544\uc5b4: \u0411\u0438\u0442\u0432\u0430 \u043f\u043e\u0434 \u041a\u0438\u0435\u0432\u043e\u043c, \ub3c5\uc77c\uc5b4: Schlacht um Kiew)\ub294 \uc81c2\ucc28 \uc138\uacc4 \ub300\uc804 \uc911 \ub3c5\uc77c\uad70\uc774 \uc6b0\ud06c\ub77c\uc774\ub098 \uc5d0\uc11c \uc9c4\ud589\ud55c \uc18c\ub828 \uad70\ub300\uc5d0 \ub300\ud55c \uac70\ub300\ud55c \ud3ec\uc704\uc804\uc758 \uc774\ub984\uc774\ub2e4.", "paragraph_id": "6564958-0-1", "paragraph_question": "question: \ud0a4\uc608\ud504 \uc804\ud22c\uac00 \uc9c4\ud589\ub41c \uc9c0\uc5ed\uc740?, context: \ud0a4\uc608\ud504 \uc804\ud22c(\ub7ec\uc2dc\uc544\uc5b4: \u0411\u0438\u0442\u0432\u0430 \u043f\u043e\u0434 \u041a\u0438\u0435\u0432\u043e\u043c, \ub3c5\uc77c\uc5b4: Schlacht um Kiew)\ub294 \uc81c2\ucc28 \uc138\uacc4 \ub300\uc804 \uc911 \ub3c5\uc77c\uad70\uc774 \uc6b0\ud06c\ub77c\uc774\ub098\uc5d0\uc11c \uc9c4\ud589\ud55c \uc18c\ub828 \uad70\ub300\uc5d0 \ub300\ud55c \uac70\ub300\ud55c \ud3ec\uc704\uc804\uc758 \uc774\ub984\uc774\ub2e4. \uc774 \uc804\ud22c\ub294 \uc5ed\uc0ac\uc801\uc73c\ub85c \uac00\uc7a5 \ub9ce\uc740 \uad70\uc774 \ud3ec\uc704\ub41c \uc804\ud22c\ub85c \uc54c\ub824\uc838 \uc788\ub2e4. \uc774 \uc791\uc804\uc740 \ubc14\ub974\ubc14\ub85c\uc0ac \uc791\uc804\uc758 \uc77c\ubd80\ub85c 1941\ub144 8\uc6d4 23\uc77c\ubd80\ud130 9\uc6d4 26\uc77c\uae4c\uc9c0 \uc9c4\ud589\ub418\uc5c8\ub2e4. \uc18c\ub828 \uad70\uc0ac\uc0ac\uc5d0\uc11c\ub294 \uc774 \uc804\ud22c\ub97c \ud0a4\uc608\ud504 \ubc29\uc5b4\uc804(\ub7ec\uc2dc\uc544\uc5b4: \u041a\u0438\u0435\u0432\u0441\u043a\u0430\u044f \u043e\u0431\u043e\u0440\u043e\u043d\u0438\u0442\u0435\u043b\u044c\u043d\u0430\u044f \u043e\u043f\u0435\u0440\u0430\u0446\u0438\u044f) \ub610\ub294 \ud0a4\uc608\ud504 \uc804\ub7b5\ubc29\uc5b4\uc804(\ub7ec\uc2dc\uc544\uc5b4: \u041a\u0438\u0435\u0432\u0441\u043a\u0430\u044f \u0441\u0442\u0440\u0430\u0442\u0435\u0433\u0438\u0447\u0435\u0441\u043a\u0430\u044f \u043e\u0431\u043e\u0440\u043e\u043d\u0438\u0442\u0435\u043b\u044c\u043d\u0430\u044f \u043e\u043f\u0435\u0440\u0430\u0446\u0438\u044f)\uc774\ub77c\uace0 \ubd80\ub974\uba70, \ub0a0\uc9dc\ub85c 7\uc6d4 7\uc77c\ubd80\ud130 9\uc6d4 26\uc77c\uae4c\uc9c0 \ub2e4\ub974\uac8c \uc7a1\uace0 \uc788\ub2e4."} {"question": "1981\ub144 \ud55c \ud574 \ub3d9\uc548 \uce58\uc548\uc720\uc9c0 \uad70\uacbd\uc774 \ubc1c\uc0ac\ud55c \uace0\ubb34\ud0c4\uc5d0 \uc758\ud574 \uc0ac\ub9dd\ud55c \uc0ac\ub78c\uc758 \uc218\ub294?", "paragraph": "\uc601\uad6d \uc5b8\ub860\ub4e4\uc740 \ub2e8\uc2dd\ud22c\uc7c1 \uacb0\uacfc \ub300\ucc98\uac00 \uc2b9\ub9ac\ud55c \uac83\ucc98\ub7fc \ub300\uc11c\ud2b9\ud544\ud588\ub2e4. \u300a\uac00\ub514\uc5b8\u300b\uc740 \u201c\uc815\ubd80\uac00 \uce68\ud574\ub418\uc9c0 \uc54a\uc744 \ub2e8\ud638\ud55c \uc758\uc9c0\ub97c \ud45c\ud604\ud568\uc73c\ub85c\uc368 \ub2e8\uc2dd\ud22c\uc7c1\uc744 \uadf9\ubcf5\ud588\ub2e4.\u201d\ub77c\uace0 \uc37c\ub2e4. \uadf8\ub7ec\ub098 \ub2e8\uc2dd\ud22c\uc7c1\uc740 \ub300\ucc98\uc640 \uc601\uad6d \uc815\ubd80\uc758 \uc785\uc7a5\uc5d0\uc11c\ub294 \ud53c\ub85c\uc2a4\uc758 \uc2b9\ub9ac\uc5d0 \ubd88\uacfc\ud588\ub2e4. \ub300\ucc98\ub294 \uc544\uc77c\ub79c\ub4dc \uacf5\ud654\uc8fc\uc758\uc790\ub4e4\uc5d0\uac8c \uc544\uc77c\ub79c\ub4dc\ub97c \uc815\ubcf5\ud55c \uc62c\ub9ac\ubc84 \ud06c\ub86c\uc6f0\uc5d0 \ub9de\uba39\ub294 \ub300\uc545\ub2f9\uc73c\ub85c \ub0a8\uac8c \ub418\uc5c8\ub2e4. \ub300\ub2c8 \ubaa8\ub9ac\uc2a8\uc740 \ub300\ub193\uace0 \uc721\ub450\ubb38\uc790\ub97c \uc368\uc11c \ub300\ucc98\ub97c \u201c\uc6b0\ub9ac\uac00 \uc544\ub294 \ucd5c\uc545\uc758 \uc30d\ub144bastard\u201d\uc774\ub77c\uace0 \ubd88\ub800\ub2e4. \ub2e8\uc2dd\ud22c\uc7c1\uc774 \uc9c4\ud589\ub418\ub358 \ub2f9\uc2dc\uc5d0\ub294 \ud22c\uc7c1\uc774 \uacf5\ud654\uc8fc\uc758\uc790\ub4e4\uc758 \uc644\ud328\ub85c \uc5ec\uaca8\uc84c\uace0, \uc774\uac83\uc774 IRA\uc640 \uc2e0\ud398\uc778 \ub0b4\ubd80\uc5d0\uc11c \uacf5\uc720\ub41c \uc2dc\uac01\uc774\uae30\ub3c4 \ud588\ub2e4. \uadf8\ub7ec\ub098 \uc0cc\uc988\uc758 \ubcf4\uad90\uc120\uac70 \uc625\uc911 \ucd9c\ub9c8\uc640 \ub2f9\uc120\uc740 \uc120\ub3d9\uc801 \uc2b9\ub9ac\uc600\ub2e4. 1971\ub144\uc758 \ub4dc\ubbf8\ud2b8\ub9ac\uc6b0\uc2a4 \uc791\uc804\uacfc 1972\ub144\uc758 \ud53c\uc758 \uc77c\uc694\uc77c \ub54c\uc640 \ub9c8\ucc2c\uac00\uc9c0\ub85c IRA\uc5d0 \uac00\uc785\ud558\ub294 \uc0ac\ub78c\uc758 \uc218\uac00 \ub298\uc5b4\ub0ac\uc73c\uba70, IRA\uc758 \uc900\uad70\uc0ac \uc900\ub3d9\uc774 \uc7ac\ucc28 \uae09\uc99d\ud558\ub294 \uacb0\uacfc\ub97c \ub0b3\uc558\ub2e4. \ube44\uad50\uc801 \uc870\uc6a9\ud588\ub358 1970\ub144\ub300 \ub9d0\ubcf4\ub2e4 \ud3ed\ub825\uc758 \ube48\ub3c4\uac00 \ub300\ud3ed \uc99d\uac00\ud588\uc73c\uba70, \ubd81\uc544\uc77c\ub79c\ub4dc \uacf3\uacf3\uc5d0\uc11c \uad11\ubc94\ud55c \uc18c\uc694\uac00 \uc77c\uc5b4\ub098\uace0 \ub354\ube14\ub9b0\uc758 \uc601\uad6d \ub300\uc0ac\uad00 \uc55e\uc5d0\uc11c\ub294 \ud3ed\ub3d9\uc774 \ub4a4\ub530\ub790\ub2e4. 1981\ub144 \ud55c \ud574 \ub3d9\uc548 \uce58\uc548\uc720\uc9c0 \uad70\uacbd\uc740 29,695\ubc1c\uc758 \uace0\ubb34\ud0c4\uc744 \ubc1c\uc0ac\ud588\uace0, \uadf8 \uacb0\uacfc 7\uba85\uc774 \uc0ac\ub9dd\ud588\ub2e4. \uadf8\ub9ac\uace0 \uadf8 \ub4a4\ub85c\ub3c4 8\ub144 \ub3d9\uc548 16,000\uc5ec \ubc1c\uc758 \uace0\ubb34\ud0c4\uc774 \ubc1c\uc0ac\ub418\uc5b4 4\uba85\uc744 \uc8fd\uac8c \ud558\uc600\ub2e4. IRA\ub294 \ub2e8\uc2dd\ud22c\uc7c1\uc774 \uc9c4\ud589\uc911\uc774\ub358 7\uac1c\uc6d4 \ub3d9\uc548 \ubb34\uc7a5\ud22c\uc7c1\uc744 \uacc4\uc18d\ud588\uace0, \uc601\uad6d \uacbd\ucc30 13\uba85, \uad70\uc778 13\uba85(\uadf8\uc911 \uc5bc\uc2a4\ud130 \ubc29\uc704\uc5f0\ub300 \uc18c\uc18d\uc774 5\uba85), \ubbfc\uac04\uc778 5\uba85\uc744 \uc8fd\uc600\ub2e4. \uc774 7\uac1c\uc6d4\uc740 \ubd81\uc544\uc77c\ub79c\ub4dc \ubd84\uc7c1 30\ub144 \ub3d9\uc548 \uac00\uc7a5 \uc720\ud608\uc774 \ub0ad\uc790\ud55c \uae30\uac04 \uc911 \ud558\ub098\ub85c \ucd1d 61\uba85\uc774 \uc0ac\ub9dd\ud588\uace0 \uadf8\uc911 34\uba85\uc774 \ubbfc\uac04\uc778\uc774\uc5c8\ub2e4. 3\ub144 \ub4a4 IRA\ub294 \ub300\ucc98\uc5d0\uac8c \ubcf5\uc218\ud558\uae30 \uc704\ud574 \ubcf4\uc218\ub2f9 \ub300\ud68c\uac00 \uc5f4\ub9ac\ub358 \uadf8\ub79c\ub4dc\ud638\ud154\uc5d0 \uc2dc\ud55c\ud3ed\ud0c4\uc744 \uc7a5\uce58\ud574 \ud638\ud154\uc744 \ub0a0\ub824 \ubc84\ub838\uace0(\ube0c\ub77c\uc774\ud2bc \ud638\ud154 \ud3ed\ud30c) \uadf8 \uacb0\uacfc 5\uba85\uc774 \uc0ac\ub9dd\ud588\ub2e4. \uc774\ub54c \ub300\ucc98 \ubcf8\uc778\ub3c4 \uac04\uc2e0\ud788 \ubaa9\uc228\ub9cc \uac74\uc838 \ub2ec\uc544\ub0ac\ub2e4.", "answer": "7\uba85", "sentence": "1981\ub144 \ud55c \ud574 \ub3d9\uc548 \uce58\uc548\uc720\uc9c0 \uad70\uacbd\uc740 29,695\ubc1c\uc758 \uace0\ubb34\ud0c4\uc744 \ubc1c\uc0ac\ud588\uace0, \uadf8 \uacb0\uacfc 7\uba85 \uc774 \uc0ac\ub9dd\ud588\ub2e4.", "paragraph_sentence": "\uc601\uad6d \uc5b8\ub860\ub4e4\uc740 \ub2e8\uc2dd\ud22c\uc7c1 \uacb0\uacfc \ub300\ucc98\uac00 \uc2b9\ub9ac\ud55c \uac83\ucc98\ub7fc \ub300\uc11c\ud2b9\ud544\ud588\ub2e4. \u300a\uac00\ub514\uc5b8\u300b\uc740 \u201c\uc815\ubd80\uac00 \uce68\ud574\ub418\uc9c0 \uc54a\uc744 \ub2e8\ud638\ud55c \uc758\uc9c0\ub97c \ud45c\ud604\ud568\uc73c\ub85c\uc368 \ub2e8\uc2dd\ud22c\uc7c1\uc744 \uadf9\ubcf5\ud588\ub2e4. \u201d\ub77c\uace0 \uc37c\ub2e4. \uadf8\ub7ec\ub098 \ub2e8\uc2dd\ud22c\uc7c1\uc740 \ub300\ucc98\uc640 \uc601\uad6d \uc815\ubd80\uc758 \uc785\uc7a5\uc5d0\uc11c\ub294 \ud53c\ub85c\uc2a4\uc758 \uc2b9\ub9ac\uc5d0 \ubd88\uacfc\ud588\ub2e4. \ub300\ucc98\ub294 \uc544\uc77c\ub79c\ub4dc \uacf5\ud654\uc8fc\uc758\uc790\ub4e4\uc5d0\uac8c \uc544\uc77c\ub79c\ub4dc\ub97c \uc815\ubcf5\ud55c \uc62c\ub9ac\ubc84 \ud06c\ub86c\uc6f0\uc5d0 \ub9de\uba39\ub294 \ub300\uc545\ub2f9\uc73c\ub85c \ub0a8\uac8c \ub418\uc5c8\ub2e4. \ub300\ub2c8 \ubaa8\ub9ac\uc2a8\uc740 \ub300\ub193\uace0 \uc721\ub450\ubb38\uc790\ub97c \uc368\uc11c \ub300\ucc98\ub97c \u201c\uc6b0\ub9ac\uac00 \uc544\ub294 \ucd5c\uc545\uc758 \uc30d\ub144bastard\u201d\uc774\ub77c\uace0 \ubd88\ub800\ub2e4. \ub2e8\uc2dd\ud22c\uc7c1\uc774 \uc9c4\ud589\ub418\ub358 \ub2f9\uc2dc\uc5d0\ub294 \ud22c\uc7c1\uc774 \uacf5\ud654\uc8fc\uc758\uc790\ub4e4\uc758 \uc644\ud328\ub85c \uc5ec\uaca8\uc84c\uace0, \uc774\uac83\uc774 IRA\uc640 \uc2e0\ud398\uc778 \ub0b4\ubd80\uc5d0\uc11c \uacf5\uc720\ub41c \uc2dc\uac01\uc774\uae30\ub3c4 \ud588\ub2e4. \uadf8\ub7ec\ub098 \uc0cc\uc988\uc758 \ubcf4\uad90\uc120\uac70 \uc625\uc911 \ucd9c\ub9c8\uc640 \ub2f9\uc120\uc740 \uc120\ub3d9\uc801 \uc2b9\ub9ac\uc600\ub2e4. 1971\ub144\uc758 \ub4dc\ubbf8\ud2b8\ub9ac\uc6b0\uc2a4 \uc791\uc804\uacfc 1972\ub144\uc758 \ud53c\uc758 \uc77c\uc694\uc77c \ub54c\uc640 \ub9c8\ucc2c\uac00\uc9c0\ub85c IRA\uc5d0 \uac00\uc785\ud558\ub294 \uc0ac\ub78c\uc758 \uc218\uac00 \ub298\uc5b4\ub0ac\uc73c\uba70, IRA\uc758 \uc900\uad70\uc0ac \uc900\ub3d9\uc774 \uc7ac\ucc28 \uae09\uc99d\ud558\ub294 \uacb0\uacfc\ub97c \ub0b3\uc558\ub2e4. \ube44\uad50\uc801 \uc870\uc6a9\ud588\ub358 1970\ub144\ub300 \ub9d0\ubcf4\ub2e4 \ud3ed\ub825\uc758 \ube48\ub3c4\uac00 \ub300\ud3ed \uc99d\uac00\ud588\uc73c\uba70, \ubd81\uc544\uc77c\ub79c\ub4dc \uacf3\uacf3\uc5d0\uc11c \uad11\ubc94\ud55c \uc18c\uc694\uac00 \uc77c\uc5b4\ub098\uace0 \ub354\ube14\ub9b0\uc758 \uc601\uad6d \ub300\uc0ac\uad00 \uc55e\uc5d0\uc11c\ub294 \ud3ed\ub3d9\uc774 \ub4a4\ub530\ub790\ub2e4. 1981\ub144 \ud55c \ud574 \ub3d9\uc548 \uce58\uc548\uc720\uc9c0 \uad70\uacbd\uc740 29,695\ubc1c\uc758 \uace0\ubb34\ud0c4\uc744 \ubc1c\uc0ac\ud588\uace0, \uadf8 \uacb0\uacfc 7\uba85 \uc774 \uc0ac\ub9dd\ud588\ub2e4. \uadf8\ub9ac\uace0 \uadf8 \ub4a4\ub85c\ub3c4 8\ub144 \ub3d9\uc548 16,000\uc5ec \ubc1c\uc758 \uace0\ubb34\ud0c4\uc774 \ubc1c\uc0ac\ub418\uc5b4 4\uba85\uc744 \uc8fd\uac8c \ud558\uc600\ub2e4. IRA\ub294 \ub2e8\uc2dd\ud22c\uc7c1\uc774 \uc9c4\ud589\uc911\uc774\ub358 7\uac1c\uc6d4 \ub3d9\uc548 \ubb34\uc7a5\ud22c\uc7c1\uc744 \uacc4\uc18d\ud588\uace0, \uc601\uad6d \uacbd\ucc30 13\uba85, \uad70\uc778 13\uba85(\uadf8\uc911 \uc5bc\uc2a4\ud130 \ubc29\uc704\uc5f0\ub300 \uc18c\uc18d\uc774 5\uba85), \ubbfc\uac04\uc778 5\uba85\uc744 \uc8fd\uc600\ub2e4. \uc774 7\uac1c\uc6d4\uc740 \ubd81\uc544\uc77c\ub79c\ub4dc \ubd84\uc7c1 30\ub144 \ub3d9\uc548 \uac00\uc7a5 \uc720\ud608\uc774 \ub0ad\uc790\ud55c \uae30\uac04 \uc911 \ud558\ub098\ub85c \ucd1d 61\uba85\uc774 \uc0ac\ub9dd\ud588\uace0 \uadf8\uc911 34\uba85\uc774 \ubbfc\uac04\uc778\uc774\uc5c8\ub2e4. 3\ub144 \ub4a4 IRA\ub294 \ub300\ucc98\uc5d0\uac8c \ubcf5\uc218\ud558\uae30 \uc704\ud574 \ubcf4\uc218\ub2f9 \ub300\ud68c\uac00 \uc5f4\ub9ac\ub358 \uadf8\ub79c\ub4dc\ud638\ud154\uc5d0 \uc2dc\ud55c\ud3ed\ud0c4\uc744 \uc7a5\uce58\ud574 \ud638\ud154\uc744 \ub0a0\ub824 \ubc84\ub838\uace0(\ube0c\ub77c\uc774\ud2bc \ud638\ud154 \ud3ed\ud30c) \uadf8 \uacb0\uacfc 5\uba85\uc774 \uc0ac\ub9dd\ud588\ub2e4. \uc774\ub54c \ub300\ucc98 \ubcf8\uc778\ub3c4 \uac04\uc2e0\ud788 \ubaa9\uc228\ub9cc \uac74\uc838 \ub2ec\uc544\ub0ac\ub2e4.", "paragraph_answer": "\uc601\uad6d \uc5b8\ub860\ub4e4\uc740 \ub2e8\uc2dd\ud22c\uc7c1 \uacb0\uacfc \ub300\ucc98\uac00 \uc2b9\ub9ac\ud55c \uac83\ucc98\ub7fc \ub300\uc11c\ud2b9\ud544\ud588\ub2e4. \u300a\uac00\ub514\uc5b8\u300b\uc740 \u201c\uc815\ubd80\uac00 \uce68\ud574\ub418\uc9c0 \uc54a\uc744 \ub2e8\ud638\ud55c \uc758\uc9c0\ub97c \ud45c\ud604\ud568\uc73c\ub85c\uc368 \ub2e8\uc2dd\ud22c\uc7c1\uc744 \uadf9\ubcf5\ud588\ub2e4.\u201d\ub77c\uace0 \uc37c\ub2e4. \uadf8\ub7ec\ub098 \ub2e8\uc2dd\ud22c\uc7c1\uc740 \ub300\ucc98\uc640 \uc601\uad6d \uc815\ubd80\uc758 \uc785\uc7a5\uc5d0\uc11c\ub294 \ud53c\ub85c\uc2a4\uc758 \uc2b9\ub9ac\uc5d0 \ubd88\uacfc\ud588\ub2e4. \ub300\ucc98\ub294 \uc544\uc77c\ub79c\ub4dc \uacf5\ud654\uc8fc\uc758\uc790\ub4e4\uc5d0\uac8c \uc544\uc77c\ub79c\ub4dc\ub97c \uc815\ubcf5\ud55c \uc62c\ub9ac\ubc84 \ud06c\ub86c\uc6f0\uc5d0 \ub9de\uba39\ub294 \ub300\uc545\ub2f9\uc73c\ub85c \ub0a8\uac8c \ub418\uc5c8\ub2e4. \ub300\ub2c8 \ubaa8\ub9ac\uc2a8\uc740 \ub300\ub193\uace0 \uc721\ub450\ubb38\uc790\ub97c \uc368\uc11c \ub300\ucc98\ub97c \u201c\uc6b0\ub9ac\uac00 \uc544\ub294 \ucd5c\uc545\uc758 \uc30d\ub144bastard\u201d\uc774\ub77c\uace0 \ubd88\ub800\ub2e4. \ub2e8\uc2dd\ud22c\uc7c1\uc774 \uc9c4\ud589\ub418\ub358 \ub2f9\uc2dc\uc5d0\ub294 \ud22c\uc7c1\uc774 \uacf5\ud654\uc8fc\uc758\uc790\ub4e4\uc758 \uc644\ud328\ub85c \uc5ec\uaca8\uc84c\uace0, \uc774\uac83\uc774 IRA\uc640 \uc2e0\ud398\uc778 \ub0b4\ubd80\uc5d0\uc11c \uacf5\uc720\ub41c \uc2dc\uac01\uc774\uae30\ub3c4 \ud588\ub2e4. \uadf8\ub7ec\ub098 \uc0cc\uc988\uc758 \ubcf4\uad90\uc120\uac70 \uc625\uc911 \ucd9c\ub9c8\uc640 \ub2f9\uc120\uc740 \uc120\ub3d9\uc801 \uc2b9\ub9ac\uc600\ub2e4. 1971\ub144\uc758 \ub4dc\ubbf8\ud2b8\ub9ac\uc6b0\uc2a4 \uc791\uc804\uacfc 1972\ub144\uc758 \ud53c\uc758 \uc77c\uc694\uc77c \ub54c\uc640 \ub9c8\ucc2c\uac00\uc9c0\ub85c IRA\uc5d0 \uac00\uc785\ud558\ub294 \uc0ac\ub78c\uc758 \uc218\uac00 \ub298\uc5b4\ub0ac\uc73c\uba70, IRA\uc758 \uc900\uad70\uc0ac \uc900\ub3d9\uc774 \uc7ac\ucc28 \uae09\uc99d\ud558\ub294 \uacb0\uacfc\ub97c \ub0b3\uc558\ub2e4. \ube44\uad50\uc801 \uc870\uc6a9\ud588\ub358 1970\ub144\ub300 \ub9d0\ubcf4\ub2e4 \ud3ed\ub825\uc758 \ube48\ub3c4\uac00 \ub300\ud3ed \uc99d\uac00\ud588\uc73c\uba70, \ubd81\uc544\uc77c\ub79c\ub4dc \uacf3\uacf3\uc5d0\uc11c \uad11\ubc94\ud55c \uc18c\uc694\uac00 \uc77c\uc5b4\ub098\uace0 \ub354\ube14\ub9b0\uc758 \uc601\uad6d \ub300\uc0ac\uad00 \uc55e\uc5d0\uc11c\ub294 \ud3ed\ub3d9\uc774 \ub4a4\ub530\ub790\ub2e4. 1981\ub144 \ud55c \ud574 \ub3d9\uc548 \uce58\uc548\uc720\uc9c0 \uad70\uacbd\uc740 29,695\ubc1c\uc758 \uace0\ubb34\ud0c4\uc744 \ubc1c\uc0ac\ud588\uace0, \uadf8 \uacb0\uacfc 7\uba85 \uc774 \uc0ac\ub9dd\ud588\ub2e4. \uadf8\ub9ac\uace0 \uadf8 \ub4a4\ub85c\ub3c4 8\ub144 \ub3d9\uc548 16,000\uc5ec \ubc1c\uc758 \uace0\ubb34\ud0c4\uc774 \ubc1c\uc0ac\ub418\uc5b4 4\uba85\uc744 \uc8fd\uac8c \ud558\uc600\ub2e4. IRA\ub294 \ub2e8\uc2dd\ud22c\uc7c1\uc774 \uc9c4\ud589\uc911\uc774\ub358 7\uac1c\uc6d4 \ub3d9\uc548 \ubb34\uc7a5\ud22c\uc7c1\uc744 \uacc4\uc18d\ud588\uace0, \uc601\uad6d \uacbd\ucc30 13\uba85, \uad70\uc778 13\uba85(\uadf8\uc911 \uc5bc\uc2a4\ud130 \ubc29\uc704\uc5f0\ub300 \uc18c\uc18d\uc774 5\uba85), \ubbfc\uac04\uc778 5\uba85\uc744 \uc8fd\uc600\ub2e4. \uc774 7\uac1c\uc6d4\uc740 \ubd81\uc544\uc77c\ub79c\ub4dc \ubd84\uc7c1 30\ub144 \ub3d9\uc548 \uac00\uc7a5 \uc720\ud608\uc774 \ub0ad\uc790\ud55c \uae30\uac04 \uc911 \ud558\ub098\ub85c \ucd1d 61\uba85\uc774 \uc0ac\ub9dd\ud588\uace0 \uadf8\uc911 34\uba85\uc774 \ubbfc\uac04\uc778\uc774\uc5c8\ub2e4. 3\ub144 \ub4a4 IRA\ub294 \ub300\ucc98\uc5d0\uac8c \ubcf5\uc218\ud558\uae30 \uc704\ud574 \ubcf4\uc218\ub2f9 \ub300\ud68c\uac00 \uc5f4\ub9ac\ub358 \uadf8\ub79c\ub4dc\ud638\ud154\uc5d0 \uc2dc\ud55c\ud3ed\ud0c4\uc744 \uc7a5\uce58\ud574 \ud638\ud154\uc744 \ub0a0\ub824 \ubc84\ub838\uace0(\ube0c\ub77c\uc774\ud2bc \ud638\ud154 \ud3ed\ud30c) \uadf8 \uacb0\uacfc 5\uba85\uc774 \uc0ac\ub9dd\ud588\ub2e4. \uc774\ub54c \ub300\ucc98 \ubcf8\uc778\ub3c4 \uac04\uc2e0\ud788 \ubaa9\uc228\ub9cc \uac74\uc838 \ub2ec\uc544\ub0ac\ub2e4.", "sentence_answer": "1981\ub144 \ud55c \ud574 \ub3d9\uc548 \uce58\uc548\uc720\uc9c0 \uad70\uacbd\uc740 29,695\ubc1c\uc758 \uace0\ubb34\ud0c4\uc744 \ubc1c\uc0ac\ud588\uace0, \uadf8 \uacb0\uacfc 7\uba85 \uc774 \uc0ac\ub9dd\ud588\ub2e4.", "paragraph_id": "6521117-11-0", "paragraph_question": "question: 1981\ub144 \ud55c \ud574 \ub3d9\uc548 \uce58\uc548\uc720\uc9c0 \uad70\uacbd\uc774 \ubc1c\uc0ac\ud55c \uace0\ubb34\ud0c4\uc5d0 \uc758\ud574 \uc0ac\ub9dd\ud55c \uc0ac\ub78c\uc758 \uc218\ub294?, context: \uc601\uad6d \uc5b8\ub860\ub4e4\uc740 \ub2e8\uc2dd\ud22c\uc7c1 \uacb0\uacfc \ub300\ucc98\uac00 \uc2b9\ub9ac\ud55c \uac83\ucc98\ub7fc \ub300\uc11c\ud2b9\ud544\ud588\ub2e4. \u300a\uac00\ub514\uc5b8\u300b\uc740 \u201c\uc815\ubd80\uac00 \uce68\ud574\ub418\uc9c0 \uc54a\uc744 \ub2e8\ud638\ud55c \uc758\uc9c0\ub97c \ud45c\ud604\ud568\uc73c\ub85c\uc368 \ub2e8\uc2dd\ud22c\uc7c1\uc744 \uadf9\ubcf5\ud588\ub2e4.\u201d\ub77c\uace0 \uc37c\ub2e4. \uadf8\ub7ec\ub098 \ub2e8\uc2dd\ud22c\uc7c1\uc740 \ub300\ucc98\uc640 \uc601\uad6d \uc815\ubd80\uc758 \uc785\uc7a5\uc5d0\uc11c\ub294 \ud53c\ub85c\uc2a4\uc758 \uc2b9\ub9ac\uc5d0 \ubd88\uacfc\ud588\ub2e4. \ub300\ucc98\ub294 \uc544\uc77c\ub79c\ub4dc \uacf5\ud654\uc8fc\uc758\uc790\ub4e4\uc5d0\uac8c \uc544\uc77c\ub79c\ub4dc\ub97c \uc815\ubcf5\ud55c \uc62c\ub9ac\ubc84 \ud06c\ub86c\uc6f0\uc5d0 \ub9de\uba39\ub294 \ub300\uc545\ub2f9\uc73c\ub85c \ub0a8\uac8c \ub418\uc5c8\ub2e4. \ub300\ub2c8 \ubaa8\ub9ac\uc2a8\uc740 \ub300\ub193\uace0 \uc721\ub450\ubb38\uc790\ub97c \uc368\uc11c \ub300\ucc98\ub97c \u201c\uc6b0\ub9ac\uac00 \uc544\ub294 \ucd5c\uc545\uc758 \uc30d\ub144bastard\u201d\uc774\ub77c\uace0 \ubd88\ub800\ub2e4. \ub2e8\uc2dd\ud22c\uc7c1\uc774 \uc9c4\ud589\ub418\ub358 \ub2f9\uc2dc\uc5d0\ub294 \ud22c\uc7c1\uc774 \uacf5\ud654\uc8fc\uc758\uc790\ub4e4\uc758 \uc644\ud328\ub85c \uc5ec\uaca8\uc84c\uace0, \uc774\uac83\uc774 IRA\uc640 \uc2e0\ud398\uc778 \ub0b4\ubd80\uc5d0\uc11c \uacf5\uc720\ub41c \uc2dc\uac01\uc774\uae30\ub3c4 \ud588\ub2e4. \uadf8\ub7ec\ub098 \uc0cc\uc988\uc758 \ubcf4\uad90\uc120\uac70 \uc625\uc911 \ucd9c\ub9c8\uc640 \ub2f9\uc120\uc740 \uc120\ub3d9\uc801 \uc2b9\ub9ac\uc600\ub2e4. 1971\ub144\uc758 \ub4dc\ubbf8\ud2b8\ub9ac\uc6b0\uc2a4 \uc791\uc804\uacfc 1972\ub144\uc758 \ud53c\uc758 \uc77c\uc694\uc77c \ub54c\uc640 \ub9c8\ucc2c\uac00\uc9c0\ub85c IRA\uc5d0 \uac00\uc785\ud558\ub294 \uc0ac\ub78c\uc758 \uc218\uac00 \ub298\uc5b4\ub0ac\uc73c\uba70, IRA\uc758 \uc900\uad70\uc0ac \uc900\ub3d9\uc774 \uc7ac\ucc28 \uae09\uc99d\ud558\ub294 \uacb0\uacfc\ub97c \ub0b3\uc558\ub2e4. \ube44\uad50\uc801 \uc870\uc6a9\ud588\ub358 1970\ub144\ub300 \ub9d0\ubcf4\ub2e4 \ud3ed\ub825\uc758 \ube48\ub3c4\uac00 \ub300\ud3ed \uc99d\uac00\ud588\uc73c\uba70, \ubd81\uc544\uc77c\ub79c\ub4dc \uacf3\uacf3\uc5d0\uc11c \uad11\ubc94\ud55c \uc18c\uc694\uac00 \uc77c\uc5b4\ub098\uace0 \ub354\ube14\ub9b0\uc758 \uc601\uad6d \ub300\uc0ac\uad00 \uc55e\uc5d0\uc11c\ub294 \ud3ed\ub3d9\uc774 \ub4a4\ub530\ub790\ub2e4. 1981\ub144 \ud55c \ud574 \ub3d9\uc548 \uce58\uc548\uc720\uc9c0 \uad70\uacbd\uc740 29,695\ubc1c\uc758 \uace0\ubb34\ud0c4\uc744 \ubc1c\uc0ac\ud588\uace0, \uadf8 \uacb0\uacfc 7\uba85\uc774 \uc0ac\ub9dd\ud588\ub2e4. \uadf8\ub9ac\uace0 \uadf8 \ub4a4\ub85c\ub3c4 8\ub144 \ub3d9\uc548 16,000\uc5ec \ubc1c\uc758 \uace0\ubb34\ud0c4\uc774 \ubc1c\uc0ac\ub418\uc5b4 4\uba85\uc744 \uc8fd\uac8c \ud558\uc600\ub2e4. IRA\ub294 \ub2e8\uc2dd\ud22c\uc7c1\uc774 \uc9c4\ud589\uc911\uc774\ub358 7\uac1c\uc6d4 \ub3d9\uc548 \ubb34\uc7a5\ud22c\uc7c1\uc744 \uacc4\uc18d\ud588\uace0, \uc601\uad6d \uacbd\ucc30 13\uba85, \uad70\uc778 13\uba85(\uadf8\uc911 \uc5bc\uc2a4\ud130 \ubc29\uc704\uc5f0\ub300 \uc18c\uc18d\uc774 5\uba85), \ubbfc\uac04\uc778 5\uba85\uc744 \uc8fd\uc600\ub2e4. \uc774 7\uac1c\uc6d4\uc740 \ubd81\uc544\uc77c\ub79c\ub4dc \ubd84\uc7c1 30\ub144 \ub3d9\uc548 \uac00\uc7a5 \uc720\ud608\uc774 \ub0ad\uc790\ud55c \uae30\uac04 \uc911 \ud558\ub098\ub85c \ucd1d 61\uba85\uc774 \uc0ac\ub9dd\ud588\uace0 \uadf8\uc911 34\uba85\uc774 \ubbfc\uac04\uc778\uc774\uc5c8\ub2e4. 3\ub144 \ub4a4 IRA\ub294 \ub300\ucc98\uc5d0\uac8c \ubcf5\uc218\ud558\uae30 \uc704\ud574 \ubcf4\uc218\ub2f9 \ub300\ud68c\uac00 \uc5f4\ub9ac\ub358 \uadf8\ub79c\ub4dc\ud638\ud154\uc5d0 \uc2dc\ud55c\ud3ed\ud0c4\uc744 \uc7a5\uce58\ud574 \ud638\ud154\uc744 \ub0a0\ub824 \ubc84\ub838\uace0(\ube0c\ub77c\uc774\ud2bc \ud638\ud154 \ud3ed\ud30c) \uadf8 \uacb0\uacfc 5\uba85\uc774 \uc0ac\ub9dd\ud588\ub2e4. \uc774\ub54c \ub300\ucc98 \ubcf8\uc778\ub3c4 \uac04\uc2e0\ud788 \ubaa9\uc228\ub9cc \uac74\uc838 \ub2ec\uc544\ub0ac\ub2e4."} {"question": "\ub354\ubd88\uc5b4\ubbfc\uc8fc\ub2f9\uc744 \ud0c8\ub2f9\ud558\uace0 \ub09c \ub300\ubd80\ubd84\uc774 \uc785\ub2f9\ud55c \ub2f9\uc758 \uc774\ub984\uc740?", "paragraph": "2015\ub144 12\uc6d4 13\uc77c, \uc81c3\uc138\ub825\uacfc \uc911\ub3c4\uc138\ub825\uc744 \ud45c\ubc29\ud55c \uc815\ub2f9 \ucc3d\ub2f9\uc744 \uc120\uc5b8\ud55c \uc548\ucca0\uc218 \uc758\uc6d0\uc758 \ud0c8\ub2f9\uc744 \uc911\uc2ec\uc73c\ub85c \ubb38\uc7ac\uc778 \ub300\ud45c\uc758 \ud1f4\uc9c4\uc744 \uc694\uad6c\ud558\ub358 \uc0c8\uc815\uce58\ubbfc\uc8fc\uc5f0\ud569 \ube44\uc8fc\ub958 \uc138\ub825\uc774 \ub300\uac70 \ud0c8\ub2f9\ud588\ub2e4. \uc774\ud6c4 \ud638\ub0a8\ud5a5\uc6b0\ud68c, \ub3d9\uad50\ub3d9\uacc4 \ub4f1 \ud638\ub0a8 \uc8fc\ucd95 \uc138\ub825\ub4e4\uc758 \ud0c8\ub2f9\uc744 \ud544\ub450\ub85c, \uae40\ud55c\uae38, \uae40\uc601\ud658 \uc758\uc6d0 \ub4f1 \ube44\uc8fc\ub958 \uc778\uc0ac\uc640, \uc8fc\uc2b9\uc6a9, \uc7a5\ubcd1\uc644 , \uad8c\uc740\ud76c \uc758\uc6d0 \ub4f1 \ud638\ub0a8 \uc758\uc6d0\ub4e4\uc774 \ub354\ubd88\uc5b4\ubbfc\uc8fc\ub2f9\uc744 \ud0c8\ub2f9\ud558\uba74\uc11c \ucd1d\uc120\uc744 \uc55e\ub450\uace0 \ud070 \ud63c\ub780\uc774 \ubc1c\uc0dd\ud588\ub2e4. \ud0c8\ub2f9\uc758 \uc774\uc720\ub85c \ub354\ubd88\uc5b4\ubbfc\uc8fc\ub2f9\uc5d0 \ub300\ud55c \ud638\ub0a8\uc758 \uc9c0\uc9c0\uc728 \ud558\ub77d, \uce5c\ub178 \uc778\uc0ac\ub4e4\uc758 \ud328\uad8c\uc8fc\uc758 \ub4f1\uc744 \ub4e4\uc5c8\ub2e4. \ub300\ub2e4\uc218 \uc758\uc6d0\ub4e4\uc740 \uc548\ucca0\uc218 \uc758\uc6d0\uc774 \uc8fc\ucd95\uc73c\ub85c \ucd94\uc9c4\ud55c \uad6d\ubbfc\uc758\ub2f9\uc73c\ub85c \uc785\ub2f9\ud558\uc600\uace0, \ud638\ub0a8\uad8c \uc138\ub825\uc740 \ucc9c\uc815\ubc30 \uc758\uc6d0\uc774 \uc774\ub044\ub294 \uad6d\ubbfc\ud68c\uc758, \ubc15\uc8fc\uc120 \uc758\uc6d0\uc758 \ud1b5\ud569\uc2e0\ub2f9 \ub4f1\uc758 \uc815\ub2f9\uc73c\ub85c \uc785\ub2f9\ud558\uc600\ub2e4. \uc870\uacbd\ud0dc \uc758\uc6d0\uc740 \ub354\ubbfc\uc8fc\uac00 \uc57c\ub2f9\uc758 \uc5ed\ud560\uc744 \uc81c\ub300\ub85c \ubabb\ud55c \uac83\uc744 \uc774\uc720\ub85c \uc0c8\ub204\ub9ac\ub2f9\uc5d0 \uc785\ub2f9\ud558\uc600\uc73c\uba70, \ud638\ub0a8 \uc138\ub825\uc758 \uc8fc\ucd95\uc778 \ubc15\uc9c0\uc6d0 \uc758\uc6d0\ub3c4 \ud1b5\ud569\uc758 \ubc00\uc54c\uc774 \ub418\uaca0\ub2e4\ub294 \uc774\uc720\ub85c \ud0c8\ub2f9\ud558\uc600\ub2e4.", "answer": "\uad6d\ubbfc\uc758\ub2f9", "sentence": "\ub300\ub2e4\uc218 \uc758\uc6d0\ub4e4\uc740 \uc548\ucca0\uc218 \uc758\uc6d0\uc774 \uc8fc\ucd95\uc73c\ub85c \ucd94\uc9c4\ud55c \uad6d\ubbfc\uc758\ub2f9 \uc73c\ub85c \uc785\ub2f9\ud558\uc600\uace0, \ud638\ub0a8\uad8c \uc138\ub825\uc740 \ucc9c\uc815\ubc30 \uc758\uc6d0\uc774 \uc774\ub044\ub294 \uad6d\ubbfc\ud68c\uc758, \ubc15\uc8fc\uc120 \uc758\uc6d0\uc758 \ud1b5\ud569\uc2e0\ub2f9 \ub4f1\uc758 \uc815\ub2f9\uc73c\ub85c \uc785\ub2f9\ud558\uc600\ub2e4.", "paragraph_sentence": "2015\ub144 12\uc6d4 13\uc77c, \uc81c3\uc138\ub825\uacfc \uc911\ub3c4\uc138\ub825\uc744 \ud45c\ubc29\ud55c \uc815\ub2f9 \ucc3d\ub2f9\uc744 \uc120\uc5b8\ud55c \uc548\ucca0\uc218 \uc758\uc6d0\uc758 \ud0c8\ub2f9\uc744 \uc911\uc2ec\uc73c\ub85c \ubb38\uc7ac\uc778 \ub300\ud45c\uc758 \ud1f4\uc9c4\uc744 \uc694\uad6c\ud558\ub358 \uc0c8\uc815\uce58\ubbfc\uc8fc\uc5f0\ud569 \ube44\uc8fc\ub958 \uc138\ub825\uc774 \ub300\uac70 \ud0c8\ub2f9\ud588\ub2e4. \uc774\ud6c4 \ud638\ub0a8\ud5a5\uc6b0\ud68c, \ub3d9\uad50\ub3d9\uacc4 \ub4f1 \ud638\ub0a8 \uc8fc\ucd95 \uc138\ub825\ub4e4\uc758 \ud0c8\ub2f9\uc744 \ud544\ub450\ub85c, \uae40\ud55c\uae38, \uae40\uc601\ud658 \uc758\uc6d0 \ub4f1 \ube44\uc8fc\ub958 \uc778\uc0ac\uc640, \uc8fc\uc2b9\uc6a9, \uc7a5\ubcd1\uc644 , \uad8c\uc740\ud76c \uc758\uc6d0 \ub4f1 \ud638\ub0a8 \uc758\uc6d0\ub4e4\uc774 \ub354\ubd88\uc5b4\ubbfc\uc8fc\ub2f9\uc744 \ud0c8\ub2f9\ud558\uba74\uc11c \ucd1d\uc120\uc744 \uc55e\ub450\uace0 \ud070 \ud63c\ub780\uc774 \ubc1c\uc0dd\ud588\ub2e4. \ud0c8\ub2f9\uc758 \uc774\uc720\ub85c \ub354\ubd88\uc5b4\ubbfc\uc8fc\ub2f9\uc5d0 \ub300\ud55c \ud638\ub0a8\uc758 \uc9c0\uc9c0\uc728 \ud558\ub77d, \uce5c\ub178 \uc778\uc0ac\ub4e4\uc758 \ud328\uad8c\uc8fc\uc758 \ub4f1\uc744 \ub4e4\uc5c8\ub2e4. \ub300\ub2e4\uc218 \uc758\uc6d0\ub4e4\uc740 \uc548\ucca0\uc218 \uc758\uc6d0\uc774 \uc8fc\ucd95\uc73c\ub85c \ucd94\uc9c4\ud55c \uad6d\ubbfc\uc758\ub2f9 \uc73c\ub85c \uc785\ub2f9\ud558\uc600\uace0, \ud638\ub0a8\uad8c \uc138\ub825\uc740 \ucc9c\uc815\ubc30 \uc758\uc6d0\uc774 \uc774\ub044\ub294 \uad6d\ubbfc\ud68c\uc758, \ubc15\uc8fc\uc120 \uc758\uc6d0\uc758 \ud1b5\ud569\uc2e0\ub2f9 \ub4f1\uc758 \uc815\ub2f9\uc73c\ub85c \uc785\ub2f9\ud558\uc600\ub2e4. \uc870\uacbd\ud0dc \uc758\uc6d0\uc740 \ub354\ubbfc\uc8fc\uac00 \uc57c\ub2f9\uc758 \uc5ed\ud560\uc744 \uc81c\ub300\ub85c \ubabb\ud55c \uac83\uc744 \uc774\uc720\ub85c \uc0c8\ub204\ub9ac\ub2f9\uc5d0 \uc785\ub2f9\ud558\uc600\uc73c\uba70, \ud638\ub0a8 \uc138\ub825\uc758 \uc8fc\ucd95\uc778 \ubc15\uc9c0\uc6d0 \uc758\uc6d0\ub3c4 \ud1b5\ud569\uc758 \ubc00\uc54c\uc774 \ub418\uaca0\ub2e4\ub294 \uc774\uc720\ub85c \ud0c8\ub2f9\ud558\uc600\ub2e4.", "paragraph_answer": "2015\ub144 12\uc6d4 13\uc77c, \uc81c3\uc138\ub825\uacfc \uc911\ub3c4\uc138\ub825\uc744 \ud45c\ubc29\ud55c \uc815\ub2f9 \ucc3d\ub2f9\uc744 \uc120\uc5b8\ud55c \uc548\ucca0\uc218 \uc758\uc6d0\uc758 \ud0c8\ub2f9\uc744 \uc911\uc2ec\uc73c\ub85c \ubb38\uc7ac\uc778 \ub300\ud45c\uc758 \ud1f4\uc9c4\uc744 \uc694\uad6c\ud558\ub358 \uc0c8\uc815\uce58\ubbfc\uc8fc\uc5f0\ud569 \ube44\uc8fc\ub958 \uc138\ub825\uc774 \ub300\uac70 \ud0c8\ub2f9\ud588\ub2e4. \uc774\ud6c4 \ud638\ub0a8\ud5a5\uc6b0\ud68c, \ub3d9\uad50\ub3d9\uacc4 \ub4f1 \ud638\ub0a8 \uc8fc\ucd95 \uc138\ub825\ub4e4\uc758 \ud0c8\ub2f9\uc744 \ud544\ub450\ub85c, \uae40\ud55c\uae38, \uae40\uc601\ud658 \uc758\uc6d0 \ub4f1 \ube44\uc8fc\ub958 \uc778\uc0ac\uc640, \uc8fc\uc2b9\uc6a9, \uc7a5\ubcd1\uc644 , \uad8c\uc740\ud76c \uc758\uc6d0 \ub4f1 \ud638\ub0a8 \uc758\uc6d0\ub4e4\uc774 \ub354\ubd88\uc5b4\ubbfc\uc8fc\ub2f9\uc744 \ud0c8\ub2f9\ud558\uba74\uc11c \ucd1d\uc120\uc744 \uc55e\ub450\uace0 \ud070 \ud63c\ub780\uc774 \ubc1c\uc0dd\ud588\ub2e4. \ud0c8\ub2f9\uc758 \uc774\uc720\ub85c \ub354\ubd88\uc5b4\ubbfc\uc8fc\ub2f9\uc5d0 \ub300\ud55c \ud638\ub0a8\uc758 \uc9c0\uc9c0\uc728 \ud558\ub77d, \uce5c\ub178 \uc778\uc0ac\ub4e4\uc758 \ud328\uad8c\uc8fc\uc758 \ub4f1\uc744 \ub4e4\uc5c8\ub2e4. \ub300\ub2e4\uc218 \uc758\uc6d0\ub4e4\uc740 \uc548\ucca0\uc218 \uc758\uc6d0\uc774 \uc8fc\ucd95\uc73c\ub85c \ucd94\uc9c4\ud55c \uad6d\ubbfc\uc758\ub2f9 \uc73c\ub85c \uc785\ub2f9\ud558\uc600\uace0, \ud638\ub0a8\uad8c \uc138\ub825\uc740 \ucc9c\uc815\ubc30 \uc758\uc6d0\uc774 \uc774\ub044\ub294 \uad6d\ubbfc\ud68c\uc758, \ubc15\uc8fc\uc120 \uc758\uc6d0\uc758 \ud1b5\ud569\uc2e0\ub2f9 \ub4f1\uc758 \uc815\ub2f9\uc73c\ub85c \uc785\ub2f9\ud558\uc600\ub2e4. \uc870\uacbd\ud0dc \uc758\uc6d0\uc740 \ub354\ubbfc\uc8fc\uac00 \uc57c\ub2f9\uc758 \uc5ed\ud560\uc744 \uc81c\ub300\ub85c \ubabb\ud55c \uac83\uc744 \uc774\uc720\ub85c \uc0c8\ub204\ub9ac\ub2f9\uc5d0 \uc785\ub2f9\ud558\uc600\uc73c\uba70, \ud638\ub0a8 \uc138\ub825\uc758 \uc8fc\ucd95\uc778 \ubc15\uc9c0\uc6d0 \uc758\uc6d0\ub3c4 \ud1b5\ud569\uc758 \ubc00\uc54c\uc774 \ub418\uaca0\ub2e4\ub294 \uc774\uc720\ub85c \ud0c8\ub2f9\ud558\uc600\ub2e4.", "sentence_answer": "\ub300\ub2e4\uc218 \uc758\uc6d0\ub4e4\uc740 \uc548\ucca0\uc218 \uc758\uc6d0\uc774 \uc8fc\ucd95\uc73c\ub85c \ucd94\uc9c4\ud55c \uad6d\ubbfc\uc758\ub2f9 \uc73c\ub85c \uc785\ub2f9\ud558\uc600\uace0, \ud638\ub0a8\uad8c \uc138\ub825\uc740 \ucc9c\uc815\ubc30 \uc758\uc6d0\uc774 \uc774\ub044\ub294 \uad6d\ubbfc\ud68c\uc758, \ubc15\uc8fc\uc120 \uc758\uc6d0\uc758 \ud1b5\ud569\uc2e0\ub2f9 \ub4f1\uc758 \uc815\ub2f9\uc73c\ub85c \uc785\ub2f9\ud558\uc600\ub2e4.", "paragraph_id": "6290061-9-1", "paragraph_question": "question: \ub354\ubd88\uc5b4\ubbfc\uc8fc\ub2f9\uc744 \ud0c8\ub2f9\ud558\uace0 \ub09c \ub300\ubd80\ubd84\uc774 \uc785\ub2f9\ud55c \ub2f9\uc758 \uc774\ub984\uc740?, context: 2015\ub144 12\uc6d4 13\uc77c, \uc81c3\uc138\ub825\uacfc \uc911\ub3c4\uc138\ub825\uc744 \ud45c\ubc29\ud55c \uc815\ub2f9 \ucc3d\ub2f9\uc744 \uc120\uc5b8\ud55c \uc548\ucca0\uc218 \uc758\uc6d0\uc758 \ud0c8\ub2f9\uc744 \uc911\uc2ec\uc73c\ub85c \ubb38\uc7ac\uc778 \ub300\ud45c\uc758 \ud1f4\uc9c4\uc744 \uc694\uad6c\ud558\ub358 \uc0c8\uc815\uce58\ubbfc\uc8fc\uc5f0\ud569 \ube44\uc8fc\ub958 \uc138\ub825\uc774 \ub300\uac70 \ud0c8\ub2f9\ud588\ub2e4. \uc774\ud6c4 \ud638\ub0a8\ud5a5\uc6b0\ud68c, \ub3d9\uad50\ub3d9\uacc4 \ub4f1 \ud638\ub0a8 \uc8fc\ucd95 \uc138\ub825\ub4e4\uc758 \ud0c8\ub2f9\uc744 \ud544\ub450\ub85c, \uae40\ud55c\uae38, \uae40\uc601\ud658 \uc758\uc6d0 \ub4f1 \ube44\uc8fc\ub958 \uc778\uc0ac\uc640, \uc8fc\uc2b9\uc6a9, \uc7a5\ubcd1\uc644 , \uad8c\uc740\ud76c \uc758\uc6d0 \ub4f1 \ud638\ub0a8 \uc758\uc6d0\ub4e4\uc774 \ub354\ubd88\uc5b4\ubbfc\uc8fc\ub2f9\uc744 \ud0c8\ub2f9\ud558\uba74\uc11c \ucd1d\uc120\uc744 \uc55e\ub450\uace0 \ud070 \ud63c\ub780\uc774 \ubc1c\uc0dd\ud588\ub2e4. \ud0c8\ub2f9\uc758 \uc774\uc720\ub85c \ub354\ubd88\uc5b4\ubbfc\uc8fc\ub2f9\uc5d0 \ub300\ud55c \ud638\ub0a8\uc758 \uc9c0\uc9c0\uc728 \ud558\ub77d, \uce5c\ub178 \uc778\uc0ac\ub4e4\uc758 \ud328\uad8c\uc8fc\uc758 \ub4f1\uc744 \ub4e4\uc5c8\ub2e4. \ub300\ub2e4\uc218 \uc758\uc6d0\ub4e4\uc740 \uc548\ucca0\uc218 \uc758\uc6d0\uc774 \uc8fc\ucd95\uc73c\ub85c \ucd94\uc9c4\ud55c \uad6d\ubbfc\uc758\ub2f9\uc73c\ub85c \uc785\ub2f9\ud558\uc600\uace0, \ud638\ub0a8\uad8c \uc138\ub825\uc740 \ucc9c\uc815\ubc30 \uc758\uc6d0\uc774 \uc774\ub044\ub294 \uad6d\ubbfc\ud68c\uc758, \ubc15\uc8fc\uc120 \uc758\uc6d0\uc758 \ud1b5\ud569\uc2e0\ub2f9 \ub4f1\uc758 \uc815\ub2f9\uc73c\ub85c \uc785\ub2f9\ud558\uc600\ub2e4. \uc870\uacbd\ud0dc \uc758\uc6d0\uc740 \ub354\ubbfc\uc8fc\uac00 \uc57c\ub2f9\uc758 \uc5ed\ud560\uc744 \uc81c\ub300\ub85c \ubabb\ud55c \uac83\uc744 \uc774\uc720\ub85c \uc0c8\ub204\ub9ac\ub2f9\uc5d0 \uc785\ub2f9\ud558\uc600\uc73c\uba70, \ud638\ub0a8 \uc138\ub825\uc758 \uc8fc\ucd95\uc778 \ubc15\uc9c0\uc6d0 \uc758\uc6d0\ub3c4 \ud1b5\ud569\uc758 \ubc00\uc54c\uc774 \ub418\uaca0\ub2e4\ub294 \uc774\uc720\ub85c \ud0c8\ub2f9\ud558\uc600\ub2e4."} {"question": "\ubc15\ud14c\ub9ac\uc544\uac00 NDM-1 \uc720\uc804\uc790\ub97c \uac00\uc9c0\uace0 \uc788\uc744\ub54c \ubc1c\uc0dd\ud558\ub294 \uc804\ud30c\ub97c \ubd84\uc11d\ud558\uae30\uc704\ud574 \uc0ac\uc6a9\ub41c \uc601\uad6d \ucf00\uc774\uc2a4\ub294 \uba87\uac74\uc778\uac00?", "paragraph": "2010\ub144 8\uc6d4, \uc800\ub110 The Lancet Infectious Diseases\uc5d0 \ucd5c\uadfc \ud589\ud574\uc9c4 \ub2e4\uad6d\uc801 \uc5f0\uad6c \uacb0\uacfc\uac00 \uc2e4\ub9b0 \ubc14 \uc788\ub2e4. \uc774 \uc5f0\uad6c\ub294 NDM-1 \uc720\uc804\uc790\ub97c \uac00\uc9c4 \ubc15\ud14c\ub9ac\uc544\uc758 \ubc1c\uc0dd\uacfc \uc804\ud30c\ub97c \ubd84\uc11d\ud558\uc600\ub2e4. \uc5f0\uad6c\uc5d0 \uc0ac\uc6a9\ub41c \ucf00\uc774\uc2a4\ub294 \uc601\uad6d 37\uac74, \uccb8\ub098\uc774 26\uac74, \uc778\ub3c4 \ud558\ub9ac\uc544\ub098 \uc8fc 26\uac74, \uadf8 \uc678 \uc778\ub3c4, \ud30c\ud0a4\uc2a4\ud0c4\uc758 \uc5ec\ub7ec \uc9c0\uc5ed 73\uac74 \ub4f1\uc774\uc5c8\ub2e4. \uc800\uc790\uc758 \ub9d0\uc5d0 \ub530\ub974\uba74, \ub9ce\uc740 \uade0\uc8fc\ub4e4\uc774 NDM-1 \uc720\uc804\uc790\ub97c \ud50c\ub77c\uc2a4\ubbf8\ub4dc \uc0c1\uc5d0 \uac00\uc9c0\uace0 \uc788\uc5c8\uc73c\uba70, \uc774 \ub54c\ubb38\uc5d0 \uade0\uacfc \uade0 \uc0ac\uc774\uc758 \uc720\uc804\uc790 \uc804\ub2ec (gene transfer)\uc774 \uc27d\uac8c \uac00\ub2a5\ud560 \uac83\uc73c\ub85c \ubcf4\uc558\ub2e4. \uc5f0\uad6c \uc2dc \ub2e4\ub904\uc9c4 \ubaa8\ub4e0 \uade0\uc8fc\ub294 \ubca0\ud0c0-\ub77d\ud0d0\uacc4\uc5f4 \ud56d\uc0dd\uc81c, \ud034\ub180\ub860 (quinolone) \uacc4\uc5f4 \ud56d\uc0dd\uc81c, \uc544\ubbf8\ub178\uae00\ub9ac\ucf54\uc0ac\uc774\ub4dc \ub4f1 \uc5ec\ub7ec \ud56d\uc0dd\uc81c\uc5d0 \ub0b4\uc131\uc744 \ubcf4\uc600\uc73c\ub098, \ub300\ubd80\ubd84 \ud3f4\ub9ac\ubbf9\uc2e0 (polymyxin) \uacc4\uc5f4 \ud56d\uc0dd\uc81c \ucf5c\ub9ac\uc2a4\ud2f4 (colistin)\uc5d0\ub294 \uac10\uc218\uc131\uc744 \ubcf4\uc600\ub2e4.", "answer": "37\uac74", "sentence": "\uc601\uad6d 37\uac74 ,", "paragraph_sentence": "2010\ub144 8\uc6d4, \uc800\ub110 The Lancet Infectious Diseases\uc5d0 \ucd5c\uadfc \ud589\ud574\uc9c4 \ub2e4\uad6d\uc801 \uc5f0\uad6c \uacb0\uacfc\uac00 \uc2e4\ub9b0 \ubc14 \uc788\ub2e4. \uc774 \uc5f0\uad6c\ub294 NDM-1 \uc720\uc804\uc790\ub97c \uac00\uc9c4 \ubc15\ud14c\ub9ac\uc544\uc758 \ubc1c\uc0dd\uacfc \uc804\ud30c\ub97c \ubd84\uc11d\ud558\uc600\ub2e4. \uc5f0\uad6c\uc5d0 \uc0ac\uc6a9\ub41c \ucf00\uc774\uc2a4\ub294 \uc601\uad6d 37\uac74 , \uccb8\ub098\uc774 26\uac74, \uc778\ub3c4 \ud558\ub9ac\uc544\ub098 \uc8fc 26\uac74, \uadf8 \uc678 \uc778\ub3c4, \ud30c\ud0a4\uc2a4\ud0c4\uc758 \uc5ec\ub7ec \uc9c0\uc5ed 73\uac74 \ub4f1\uc774\uc5c8\ub2e4. \uc800\uc790\uc758 \ub9d0\uc5d0 \ub530\ub974\uba74, \ub9ce\uc740 \uade0\uc8fc\ub4e4\uc774 NDM-1 \uc720\uc804\uc790\ub97c \ud50c\ub77c\uc2a4\ubbf8\ub4dc \uc0c1\uc5d0 \uac00\uc9c0\uace0 \uc788\uc5c8\uc73c\uba70, \uc774 \ub54c\ubb38\uc5d0 \uade0\uacfc \uade0 \uc0ac\uc774\uc758 \uc720\uc804\uc790 \uc804\ub2ec (gene transfer)\uc774 \uc27d\uac8c \uac00\ub2a5\ud560 \uac83\uc73c\ub85c \ubcf4\uc558\ub2e4. \uc5f0\uad6c \uc2dc \ub2e4\ub904\uc9c4 \ubaa8\ub4e0 \uade0\uc8fc\ub294 \ubca0\ud0c0-\ub77d\ud0d0\uacc4\uc5f4 \ud56d\uc0dd\uc81c, \ud034\ub180\ub860 (quinolone) \uacc4\uc5f4 \ud56d\uc0dd\uc81c, \uc544\ubbf8\ub178\uae00\ub9ac\ucf54\uc0ac\uc774\ub4dc \ub4f1 \uc5ec\ub7ec \ud56d\uc0dd\uc81c\uc5d0 \ub0b4\uc131\uc744 \ubcf4\uc600\uc73c\ub098, \ub300\ubd80\ubd84 \ud3f4\ub9ac\ubbf9\uc2e0 (polymyxin) \uacc4\uc5f4 \ud56d\uc0dd\uc81c \ucf5c\ub9ac\uc2a4\ud2f4 (colistin)\uc5d0\ub294 \uac10\uc218\uc131\uc744 \ubcf4\uc600\ub2e4.", "paragraph_answer": "2010\ub144 8\uc6d4, \uc800\ub110 The Lancet Infectious Diseases\uc5d0 \ucd5c\uadfc \ud589\ud574\uc9c4 \ub2e4\uad6d\uc801 \uc5f0\uad6c \uacb0\uacfc\uac00 \uc2e4\ub9b0 \ubc14 \uc788\ub2e4. \uc774 \uc5f0\uad6c\ub294 NDM-1 \uc720\uc804\uc790\ub97c \uac00\uc9c4 \ubc15\ud14c\ub9ac\uc544\uc758 \ubc1c\uc0dd\uacfc \uc804\ud30c\ub97c \ubd84\uc11d\ud558\uc600\ub2e4. \uc5f0\uad6c\uc5d0 \uc0ac\uc6a9\ub41c \ucf00\uc774\uc2a4\ub294 \uc601\uad6d 37\uac74 , \uccb8\ub098\uc774 26\uac74, \uc778\ub3c4 \ud558\ub9ac\uc544\ub098 \uc8fc 26\uac74, \uadf8 \uc678 \uc778\ub3c4, \ud30c\ud0a4\uc2a4\ud0c4\uc758 \uc5ec\ub7ec \uc9c0\uc5ed 73\uac74 \ub4f1\uc774\uc5c8\ub2e4. \uc800\uc790\uc758 \ub9d0\uc5d0 \ub530\ub974\uba74, \ub9ce\uc740 \uade0\uc8fc\ub4e4\uc774 NDM-1 \uc720\uc804\uc790\ub97c \ud50c\ub77c\uc2a4\ubbf8\ub4dc \uc0c1\uc5d0 \uac00\uc9c0\uace0 \uc788\uc5c8\uc73c\uba70, \uc774 \ub54c\ubb38\uc5d0 \uade0\uacfc \uade0 \uc0ac\uc774\uc758 \uc720\uc804\uc790 \uc804\ub2ec (gene transfer)\uc774 \uc27d\uac8c \uac00\ub2a5\ud560 \uac83\uc73c\ub85c \ubcf4\uc558\ub2e4. \uc5f0\uad6c \uc2dc \ub2e4\ub904\uc9c4 \ubaa8\ub4e0 \uade0\uc8fc\ub294 \ubca0\ud0c0-\ub77d\ud0d0\uacc4\uc5f4 \ud56d\uc0dd\uc81c, \ud034\ub180\ub860 (quinolone) \uacc4\uc5f4 \ud56d\uc0dd\uc81c, \uc544\ubbf8\ub178\uae00\ub9ac\ucf54\uc0ac\uc774\ub4dc \ub4f1 \uc5ec\ub7ec \ud56d\uc0dd\uc81c\uc5d0 \ub0b4\uc131\uc744 \ubcf4\uc600\uc73c\ub098, \ub300\ubd80\ubd84 \ud3f4\ub9ac\ubbf9\uc2e0 (polymyxin) \uacc4\uc5f4 \ud56d\uc0dd\uc81c \ucf5c\ub9ac\uc2a4\ud2f4 (colistin)\uc5d0\ub294 \uac10\uc218\uc131\uc744 \ubcf4\uc600\ub2e4.", "sentence_answer": "\uc601\uad6d 37\uac74 ,", "paragraph_id": "6467478-2-1", "paragraph_question": "question: \ubc15\ud14c\ub9ac\uc544\uac00 NDM-1 \uc720\uc804\uc790\ub97c \uac00\uc9c0\uace0 \uc788\uc744\ub54c \ubc1c\uc0dd\ud558\ub294 \uc804\ud30c\ub97c \ubd84\uc11d\ud558\uae30\uc704\ud574 \uc0ac\uc6a9\ub41c \uc601\uad6d \ucf00\uc774\uc2a4\ub294 \uba87\uac74\uc778\uac00?, context: 2010\ub144 8\uc6d4, \uc800\ub110 The Lancet Infectious Diseases\uc5d0 \ucd5c\uadfc \ud589\ud574\uc9c4 \ub2e4\uad6d\uc801 \uc5f0\uad6c \uacb0\uacfc\uac00 \uc2e4\ub9b0 \ubc14 \uc788\ub2e4. \uc774 \uc5f0\uad6c\ub294 NDM-1 \uc720\uc804\uc790\ub97c \uac00\uc9c4 \ubc15\ud14c\ub9ac\uc544\uc758 \ubc1c\uc0dd\uacfc \uc804\ud30c\ub97c \ubd84\uc11d\ud558\uc600\ub2e4. \uc5f0\uad6c\uc5d0 \uc0ac\uc6a9\ub41c \ucf00\uc774\uc2a4\ub294 \uc601\uad6d 37\uac74, \uccb8\ub098\uc774 26\uac74, \uc778\ub3c4 \ud558\ub9ac\uc544\ub098 \uc8fc 26\uac74, \uadf8 \uc678 \uc778\ub3c4, \ud30c\ud0a4\uc2a4\ud0c4\uc758 \uc5ec\ub7ec \uc9c0\uc5ed 73\uac74 \ub4f1\uc774\uc5c8\ub2e4. \uc800\uc790\uc758 \ub9d0\uc5d0 \ub530\ub974\uba74, \ub9ce\uc740 \uade0\uc8fc\ub4e4\uc774 NDM-1 \uc720\uc804\uc790\ub97c \ud50c\ub77c\uc2a4\ubbf8\ub4dc \uc0c1\uc5d0 \uac00\uc9c0\uace0 \uc788\uc5c8\uc73c\uba70, \uc774 \ub54c\ubb38\uc5d0 \uade0\uacfc \uade0 \uc0ac\uc774\uc758 \uc720\uc804\uc790 \uc804\ub2ec (gene transfer)\uc774 \uc27d\uac8c \uac00\ub2a5\ud560 \uac83\uc73c\ub85c \ubcf4\uc558\ub2e4. \uc5f0\uad6c \uc2dc \ub2e4\ub904\uc9c4 \ubaa8\ub4e0 \uade0\uc8fc\ub294 \ubca0\ud0c0-\ub77d\ud0d0\uacc4\uc5f4 \ud56d\uc0dd\uc81c, \ud034\ub180\ub860 (quinolone) \uacc4\uc5f4 \ud56d\uc0dd\uc81c, \uc544\ubbf8\ub178\uae00\ub9ac\ucf54\uc0ac\uc774\ub4dc \ub4f1 \uc5ec\ub7ec \ud56d\uc0dd\uc81c\uc5d0 \ub0b4\uc131\uc744 \ubcf4\uc600\uc73c\ub098, \ub300\ubd80\ubd84 \ud3f4\ub9ac\ubbf9\uc2e0 (polymyxin) \uacc4\uc5f4 \ud56d\uc0dd\uc81c \ucf5c\ub9ac\uc2a4\ud2f4 (colistin)\uc5d0\ub294 \uac10\uc218\uc131\uc744 \ubcf4\uc600\ub2e4."} {"question": "\uc0dd\ud65c\uc218\uc900\uc758 \uac1c\ub150\uc744 \uc870\uc815\ud574 \ud654\ud3d0\uc18c\ub4dd\uc758 \ud55c\uacc4\ub97c \uadf9\ubcf5\ud558\ub824 \ud558\uc600\uc73c\uba70 \uc720\ubcd1\ub960\uacfc \uc0ac\ub9dd\ub960\uc744 \uace0\ub824\ud55c \uc5f0\uad6c\ub97c \uc9c4\ud589\ud588\ub358 \ud559\uc790\ub294?", "paragraph": "\uad6d\ubbfc\uc18c\ub4dd \uadf8\ub9ac\uace0 \uc784\uae08 \ub4f1\uc774 \uac16\ub294 \ub2e8\uc810\uc744 \uadf9\ubcf5\ud558\uace0 \ubcf4\ub2e4 \ud3ec\uad04\uc801\uc73c\ub85c \uc0dd\ud65c\uc218\uc900\uc744 \uce21\uc815\ud558\uace0\uc790 \ud558\uc600\ub2e4. \uc6b0\uc120 \uad6d\ubbfc\uc18c\ub4dd\uc774 \uac16\ub294 \ub2e8\uc810\uc744 \ubcf4\uc644\ud558\uae30 \uc704\ud574 Nordhaus and Tobin\uacfc Usher\ub294 \uac00\uc0ac\ub178\ub3d9, \uc5ec\uac00 \ub4f1\uc758 \uac00\uce58\ub97c \uace0\ub824\ud574 \ubcf4\uc644\uc801 \uad6d\ubbfc\uc18c\ub4dd\uc758 \uac1c\ub150\uc744 \uc81c\uc2dc\ud588\uc73c\uba70, Williamson\uc740 \uc720\ubcd1\ub960\uacfc \uc0ac\ub9dd\ub960\uc744 \uace0\ub824\ud55c \uc5f0\uad6c\ub97c \uc9c4\ud589\ud588\ub2e4. \ud654\ud3d0\uc18c\ub4dd\uc758 \ud55c\uacc4\ub97c \uadf9\ubcf5\ud558\uae30 \uc704\ud574 \uc0dd\ud65c\uc218\uc900\uc758 \uac1c\ub150\uc744 \ub2ec\ub9ac\ud558\uae30\ub3c4 \ud558\uc600\ub2e4. Amartya Sen\uc740 \ud654\ud3d0\uc18c\ub4dd\uacfc \uc790\uc6d0\uc758 \uc9c0\ubc30\ub825\uc744 \uac15\uc870\ud558\ub294 \uae30\uc874\uc758 \uc0dd\ud65c\uc218\uc900 \uc9c0\ud45c\ub97c \ube44\ud310\ud558\uace0 \ucda9\ub9cc\ud55c \uc0b6\uc744 \uc0b4\ub3c4\ub85d \ud560 \uc218 \uc788\ub294 \uc0ac\ud68c\uc758 \uc5ed\ub7c9\uacfc \uc7a0\uc7ac\ub825\uc744 \uac15\uc870\ud588\ub2e4. \ub610\ud55c \uad50\uc721, \uac74\uac15 \ub4f1\uc5d0 \ub300\ud55c \ud22c\uc790\ub97c \ud1b5\ud574 \ubbf8\ub798\uc758 \uc5ed\ub7c9\uc744 \uac15\ud654\ud560 \ub54c \uc0dd\ud65c\uc218\uc900\uc774 \ud5a5\uc0c1\ub41c\ub2e4\uace0 \ud558\uc600\ub2e4.", "answer": "Williamson", "sentence": "\uc6b0\uc120 \uad6d\ubbfc\uc18c\ub4dd\uc774 \uac16\ub294 \ub2e8\uc810\uc744 \ubcf4\uc644\ud558\uae30 \uc704\ud574 Nordhaus and Tobin\uacfc Usher\ub294 \uac00\uc0ac\ub178\ub3d9, \uc5ec\uac00 \ub4f1\uc758 \uac00\uce58\ub97c \uace0\ub824\ud574 \ubcf4\uc644\uc801 \uad6d\ubbfc\uc18c\ub4dd\uc758 \uac1c\ub150\uc744 \uc81c\uc2dc\ud588\uc73c\uba70, Williamson \uc740 \uc720\ubcd1\ub960\uacfc \uc0ac\ub9dd\ub960\uc744 \uace0\ub824\ud55c \uc5f0\uad6c\ub97c \uc9c4\ud589\ud588\ub2e4.", "paragraph_sentence": "\uad6d\ubbfc\uc18c\ub4dd \uadf8\ub9ac\uace0 \uc784\uae08 \ub4f1\uc774 \uac16\ub294 \ub2e8\uc810\uc744 \uadf9\ubcf5\ud558\uace0 \ubcf4\ub2e4 \ud3ec\uad04\uc801\uc73c\ub85c \uc0dd\ud65c\uc218\uc900\uc744 \uce21\uc815\ud558\uace0\uc790 \ud558\uc600\ub2e4. \uc6b0\uc120 \uad6d\ubbfc\uc18c\ub4dd\uc774 \uac16\ub294 \ub2e8\uc810\uc744 \ubcf4\uc644\ud558\uae30 \uc704\ud574 Nordhaus and Tobin\uacfc Usher\ub294 \uac00\uc0ac\ub178\ub3d9, \uc5ec\uac00 \ub4f1\uc758 \uac00\uce58\ub97c \uace0\ub824\ud574 \ubcf4\uc644\uc801 \uad6d\ubbfc\uc18c\ub4dd\uc758 \uac1c\ub150\uc744 \uc81c\uc2dc\ud588\uc73c\uba70, Williamson \uc740 \uc720\ubcd1\ub960\uacfc \uc0ac\ub9dd\ub960\uc744 \uace0\ub824\ud55c \uc5f0\uad6c\ub97c \uc9c4\ud589\ud588\ub2e4. \ud654\ud3d0\uc18c\ub4dd\uc758 \ud55c\uacc4\ub97c \uadf9\ubcf5\ud558\uae30 \uc704\ud574 \uc0dd\ud65c\uc218\uc900\uc758 \uac1c\ub150\uc744 \ub2ec\ub9ac\ud558\uae30\ub3c4 \ud558\uc600\ub2e4. Amartya Sen\uc740 \ud654\ud3d0\uc18c\ub4dd\uacfc \uc790\uc6d0\uc758 \uc9c0\ubc30\ub825\uc744 \uac15\uc870\ud558\ub294 \uae30\uc874\uc758 \uc0dd\ud65c\uc218\uc900 \uc9c0\ud45c\ub97c \ube44\ud310\ud558\uace0 \ucda9\ub9cc\ud55c \uc0b6\uc744 \uc0b4\ub3c4\ub85d \ud560 \uc218 \uc788\ub294 \uc0ac\ud68c\uc758 \uc5ed\ub7c9\uacfc \uc7a0\uc7ac\ub825\uc744 \uac15\uc870\ud588\ub2e4. \ub610\ud55c \uad50\uc721, \uac74\uac15 \ub4f1\uc5d0 \ub300\ud55c \ud22c\uc790\ub97c \ud1b5\ud574 \ubbf8\ub798\uc758 \uc5ed\ub7c9\uc744 \uac15\ud654\ud560 \ub54c \uc0dd\ud65c\uc218\uc900\uc774 \ud5a5\uc0c1\ub41c\ub2e4\uace0 \ud558\uc600\ub2e4.", "paragraph_answer": "\uad6d\ubbfc\uc18c\ub4dd \uadf8\ub9ac\uace0 \uc784\uae08 \ub4f1\uc774 \uac16\ub294 \ub2e8\uc810\uc744 \uadf9\ubcf5\ud558\uace0 \ubcf4\ub2e4 \ud3ec\uad04\uc801\uc73c\ub85c \uc0dd\ud65c\uc218\uc900\uc744 \uce21\uc815\ud558\uace0\uc790 \ud558\uc600\ub2e4. \uc6b0\uc120 \uad6d\ubbfc\uc18c\ub4dd\uc774 \uac16\ub294 \ub2e8\uc810\uc744 \ubcf4\uc644\ud558\uae30 \uc704\ud574 Nordhaus and Tobin\uacfc Usher\ub294 \uac00\uc0ac\ub178\ub3d9, \uc5ec\uac00 \ub4f1\uc758 \uac00\uce58\ub97c \uace0\ub824\ud574 \ubcf4\uc644\uc801 \uad6d\ubbfc\uc18c\ub4dd\uc758 \uac1c\ub150\uc744 \uc81c\uc2dc\ud588\uc73c\uba70, Williamson \uc740 \uc720\ubcd1\ub960\uacfc \uc0ac\ub9dd\ub960\uc744 \uace0\ub824\ud55c \uc5f0\uad6c\ub97c \uc9c4\ud589\ud588\ub2e4. \ud654\ud3d0\uc18c\ub4dd\uc758 \ud55c\uacc4\ub97c \uadf9\ubcf5\ud558\uae30 \uc704\ud574 \uc0dd\ud65c\uc218\uc900\uc758 \uac1c\ub150\uc744 \ub2ec\ub9ac\ud558\uae30\ub3c4 \ud558\uc600\ub2e4. Amartya Sen\uc740 \ud654\ud3d0\uc18c\ub4dd\uacfc \uc790\uc6d0\uc758 \uc9c0\ubc30\ub825\uc744 \uac15\uc870\ud558\ub294 \uae30\uc874\uc758 \uc0dd\ud65c\uc218\uc900 \uc9c0\ud45c\ub97c \ube44\ud310\ud558\uace0 \ucda9\ub9cc\ud55c \uc0b6\uc744 \uc0b4\ub3c4\ub85d \ud560 \uc218 \uc788\ub294 \uc0ac\ud68c\uc758 \uc5ed\ub7c9\uacfc \uc7a0\uc7ac\ub825\uc744 \uac15\uc870\ud588\ub2e4. \ub610\ud55c \uad50\uc721, \uac74\uac15 \ub4f1\uc5d0 \ub300\ud55c \ud22c\uc790\ub97c \ud1b5\ud574 \ubbf8\ub798\uc758 \uc5ed\ub7c9\uc744 \uac15\ud654\ud560 \ub54c \uc0dd\ud65c\uc218\uc900\uc774 \ud5a5\uc0c1\ub41c\ub2e4\uace0 \ud558\uc600\ub2e4.", "sentence_answer": "\uc6b0\uc120 \uad6d\ubbfc\uc18c\ub4dd\uc774 \uac16\ub294 \ub2e8\uc810\uc744 \ubcf4\uc644\ud558\uae30 \uc704\ud574 Nordhaus and Tobin\uacfc Usher\ub294 \uac00\uc0ac\ub178\ub3d9, \uc5ec\uac00 \ub4f1\uc758 \uac00\uce58\ub97c \uace0\ub824\ud574 \ubcf4\uc644\uc801 \uad6d\ubbfc\uc18c\ub4dd\uc758 \uac1c\ub150\uc744 \uc81c\uc2dc\ud588\uc73c\uba70, Williamson \uc740 \uc720\ubcd1\ub960\uacfc \uc0ac\ub9dd\ub960\uc744 \uace0\ub824\ud55c \uc5f0\uad6c\ub97c \uc9c4\ud589\ud588\ub2e4.", "paragraph_id": "6511379-2-1", "paragraph_question": "question: \uc0dd\ud65c\uc218\uc900\uc758 \uac1c\ub150\uc744 \uc870\uc815\ud574 \ud654\ud3d0\uc18c\ub4dd\uc758 \ud55c\uacc4\ub97c \uadf9\ubcf5\ud558\ub824 \ud558\uc600\uc73c\uba70 \uc720\ubcd1\ub960\uacfc \uc0ac\ub9dd\ub960\uc744 \uace0\ub824\ud55c \uc5f0\uad6c\ub97c \uc9c4\ud589\ud588\ub358 \ud559\uc790\ub294?, context: \uad6d\ubbfc\uc18c\ub4dd \uadf8\ub9ac\uace0 \uc784\uae08 \ub4f1\uc774 \uac16\ub294 \ub2e8\uc810\uc744 \uadf9\ubcf5\ud558\uace0 \ubcf4\ub2e4 \ud3ec\uad04\uc801\uc73c\ub85c \uc0dd\ud65c\uc218\uc900\uc744 \uce21\uc815\ud558\uace0\uc790 \ud558\uc600\ub2e4. \uc6b0\uc120 \uad6d\ubbfc\uc18c\ub4dd\uc774 \uac16\ub294 \ub2e8\uc810\uc744 \ubcf4\uc644\ud558\uae30 \uc704\ud574 Nordhaus and Tobin\uacfc Usher\ub294 \uac00\uc0ac\ub178\ub3d9, \uc5ec\uac00 \ub4f1\uc758 \uac00\uce58\ub97c \uace0\ub824\ud574 \ubcf4\uc644\uc801 \uad6d\ubbfc\uc18c\ub4dd\uc758 \uac1c\ub150\uc744 \uc81c\uc2dc\ud588\uc73c\uba70, Williamson\uc740 \uc720\ubcd1\ub960\uacfc \uc0ac\ub9dd\ub960\uc744 \uace0\ub824\ud55c \uc5f0\uad6c\ub97c \uc9c4\ud589\ud588\ub2e4. \ud654\ud3d0\uc18c\ub4dd\uc758 \ud55c\uacc4\ub97c \uadf9\ubcf5\ud558\uae30 \uc704\ud574 \uc0dd\ud65c\uc218\uc900\uc758 \uac1c\ub150\uc744 \ub2ec\ub9ac\ud558\uae30\ub3c4 \ud558\uc600\ub2e4. Amartya Sen\uc740 \ud654\ud3d0\uc18c\ub4dd\uacfc \uc790\uc6d0\uc758 \uc9c0\ubc30\ub825\uc744 \uac15\uc870\ud558\ub294 \uae30\uc874\uc758 \uc0dd\ud65c\uc218\uc900 \uc9c0\ud45c\ub97c \ube44\ud310\ud558\uace0 \ucda9\ub9cc\ud55c \uc0b6\uc744 \uc0b4\ub3c4\ub85d \ud560 \uc218 \uc788\ub294 \uc0ac\ud68c\uc758 \uc5ed\ub7c9\uacfc \uc7a0\uc7ac\ub825\uc744 \uac15\uc870\ud588\ub2e4. \ub610\ud55c \uad50\uc721, \uac74\uac15 \ub4f1\uc5d0 \ub300\ud55c \ud22c\uc790\ub97c \ud1b5\ud574 \ubbf8\ub798\uc758 \uc5ed\ub7c9\uc744 \uac15\ud654\ud560 \ub54c \uc0dd\ud65c\uc218\uc900\uc774 \ud5a5\uc0c1\ub41c\ub2e4\uace0 \ud558\uc600\ub2e4."} {"question": "2013\ub144 3\uc6d4 \uc5ec\uc131\ub3d9\uc544\uc640\uc758 \uc778\ud130\ubdf0\uc5d0\uc11c \ubd80\ubaa8\ub2d8 \uc9d1\uc744 \uc0ac\ub4dc\ub9ac\uace0\ub3c4 \ub0a8\uc744\ub9cc\ud07c \ub3c8\uc744 \ubc8c\uc5c8\ub2e4\uace0 \ubc1d\ud78c \uc81d\uc2a4\ud0a4\uc2a4 \uba64\ubc84\ub294?", "paragraph": "2013\ub144 3\uc6d4 \uc5ec\uc131\ub3d9\uc544\uc640\uc758 \uc778\ud130\ubdf0\uc5d0\uc11c \uc774\uc7ac\uc9c4\uc740 \ub2e4\uc74c\uacfc \uac19\uc774 \ub9d0\ud588\ub2e4.[21]. \uc6d0\ubb38: \uadf8\ub7f0 \uc774\ubbf8\uc9c0 \ub54c\ubb38\uc5d0 \u2018\ub3c8 \ud544\uc694\ud558\uc9c0?\u2019 \ud558\uba74\uc11c \uc811\uadfc\ud558\ub294 \uc0ac\ub78c\ub4e4\uc774 \ub9ce\uc558\uc5b4\uc694. \uadf8\ub7f0\ub370 \uc815\uc791 \uadf8\ub54c\ub294 \uacbd\uc81c\uc801\uc778 \uc5b4\ub824\uc6c0\uc5d0\uc11c \ubc97\uc5b4\ub09c \uc0c1\ud0dc\uc600\uac70\ub4e0\uc694. \uc81d\uc2a4\ud0a4\uc2a4 \ub370\ubdd4 \ud6c4 \uba87 \uac1c\uc6d4 \ub9cc\uc5d0 \ubd80\ubaa8\ub2d8\uaed8 \uc9d1\uc744 \uc0ac\ub4dc\ub9ac\uace0\ub3c4 \ub0a8\uc744 \ub9cc\ud07c \ubc8c\uc5c8\uc73c\ub2c8\uae4c\uc694. \ud558\uc9c0\ub9cc \ub298 \uc800\ud76c \uc9d1 \uac00\uc815 \ud615\ud3b8\uc774 \uc5c4\uccad \uc5b4\ub824\uc6b4 \uac83\ucc98\ub7fc \ubc29\uc1a1\uc5d0\uc11c \uadf8\ub824\uc9c0\uace4 \ud588\uc5b4\uc694. \uc815\uc791 \uc800\uc5d0\uac8c \uac00\uc218 \ud65c\ub3d9\uc73c\ub85c \ubc8c\uc5c8\ub358 \ub3c8\uc740 \ubb34\ucc99 \ud070\ub3c8\uc774\uc5c8\uace0, \ubd80\ubaa8\ub2d8\uaed8 \uc804\ubd80 \ub4dc\ub824\uc11c \ucc28\uace1\ucc28\uace1 \ubaa8\uc544\uc654\uc5c8\uc8e0. \ub450 \ubd84\uc774 \ub3cc\uc544\uac00\uc2e0 \uc9c0\uae08\ub3c4 \uadf8 \ub3c8\uc740 \uadf8\ub300\ub85c \ub0a8\uc544 \uc788\uc73c\ub2c8\uae4c\uc694. (\uc911\ub7b5) \ub2e4\ud050\uba58\ud130\ub9ac\ub97c \ucc0d\uc740 \uc801\uc774 \uc788\uc5c8\uc5b4\uc694. \uc9d1\uc5d0 \uac00\uc11c \ub77c\uba74\uc744 \ub053\uc5ec \uba39\uc73c\ub77c\ub294 \ub465, \ubc84\uc2a4\ub97c \ud0c0\ub77c\ub294 \ub465, \ud559\uad50\uc5d0 \uac00\uc11c \uc591\uce58\uc9c8\uc744 \ud558\ub77c\ub294 \ub465, \uc5c9\ub6b1\ud55c \uc8fc\ubb38\uc744 \ub9ce\uc774 \ud588\uc8e0. \ub204\uad70\uac00\uc5d0\uac8c\ub294 \uc77c\uc0c1\uc801\uc778 \uc77c\uc778\ub370, \uc5f0\uc608\uc778\uc774 \uc591\uc740 \ub0c4\ube44 \ub69c\uaed1\uc5d0 \ub77c\uba74\uc744 \ub35c\uc5b4 \uba39\ub2e4 \ud758\ub9ac\uace0, \ubc84\uc2a4\ub97c \ud0c0\ub824\ub2e4\uac00 \uc0ac\ub78c\ub4e4 \uc2dc\uc120\uc774 \ub450\ub824\uc6cc \ud0dd\uc2dc\ub97c \ud0c0\uace0 \uc591\uce58\uc9c8\ub3c4 \ud559\uad50\uc5d0\uc11c\ub098 \uac00\uc11c \ud560 \uc218 \uc788\ub2e4\ub294 \uc0c1\ud669\uc73c\ub85c \ub9cc\ub4e4\uc5b4\uac00\ub354\ub77c\uace0\uc694(\uc6c3\uc74c).", "answer": "\uc774\uc7ac\uc9c4", "sentence": "2013\ub144 3\uc6d4 \uc5ec\uc131\ub3d9\uc544\uc640\uc758 \uc778\ud130\ubdf0\uc5d0\uc11c \uc774\uc7ac\uc9c4 \uc740 \ub2e4\uc74c\uacfc \uac19\uc774 \ub9d0\ud588\ub2e4.[21].", "paragraph_sentence": " 2013\ub144 3\uc6d4 \uc5ec\uc131\ub3d9\uc544\uc640\uc758 \uc778\ud130\ubdf0\uc5d0\uc11c \uc774\uc7ac\uc9c4 \uc740 \ub2e4\uc74c\uacfc \uac19\uc774 \ub9d0\ud588\ub2e4.[21]. \uc6d0\ubb38: \uadf8\ub7f0 \uc774\ubbf8\uc9c0 \ub54c\ubb38\uc5d0 \u2018\ub3c8 \ud544\uc694\ud558\uc9c0? \u2019 \ud558\uba74\uc11c \uc811\uadfc\ud558\ub294 \uc0ac\ub78c\ub4e4\uc774 \ub9ce\uc558\uc5b4\uc694. \uadf8\ub7f0\ub370 \uc815\uc791 \uadf8\ub54c\ub294 \uacbd\uc81c\uc801\uc778 \uc5b4\ub824\uc6c0\uc5d0\uc11c \ubc97\uc5b4\ub09c \uc0c1\ud0dc\uc600\uac70\ub4e0\uc694. \uc81d\uc2a4\ud0a4\uc2a4 \ub370\ubdd4 \ud6c4 \uba87 \uac1c\uc6d4 \ub9cc\uc5d0 \ubd80\ubaa8\ub2d8\uaed8 \uc9d1\uc744 \uc0ac\ub4dc\ub9ac\uace0\ub3c4 \ub0a8\uc744 \ub9cc\ud07c \ubc8c\uc5c8\uc73c\ub2c8\uae4c\uc694. \ud558\uc9c0\ub9cc \ub298 \uc800\ud76c \uc9d1 \uac00\uc815 \ud615\ud3b8\uc774 \uc5c4\uccad \uc5b4\ub824\uc6b4 \uac83\ucc98\ub7fc \ubc29\uc1a1\uc5d0\uc11c \uadf8\ub824\uc9c0\uace4 \ud588\uc5b4\uc694. \uc815\uc791 \uc800\uc5d0\uac8c \uac00\uc218 \ud65c\ub3d9\uc73c\ub85c \ubc8c\uc5c8\ub358 \ub3c8\uc740 \ubb34\ucc99 \ud070\ub3c8\uc774\uc5c8\uace0, \ubd80\ubaa8\ub2d8\uaed8 \uc804\ubd80 \ub4dc\ub824\uc11c \ucc28\uace1\ucc28\uace1 \ubaa8\uc544\uc654\uc5c8\uc8e0. \ub450 \ubd84\uc774 \ub3cc\uc544\uac00\uc2e0 \uc9c0\uae08\ub3c4 \uadf8 \ub3c8\uc740 \uadf8\ub300\ub85c \ub0a8\uc544 \uc788\uc73c\ub2c8\uae4c\uc694. (\uc911\ub7b5) \ub2e4\ud050\uba58\ud130\ub9ac\ub97c \ucc0d\uc740 \uc801\uc774 \uc788\uc5c8\uc5b4\uc694. \uc9d1\uc5d0 \uac00\uc11c \ub77c\uba74\uc744 \ub053\uc5ec \uba39\uc73c\ub77c\ub294 \ub465, \ubc84\uc2a4\ub97c \ud0c0\ub77c\ub294 \ub465, \ud559\uad50\uc5d0 \uac00\uc11c \uc591\uce58\uc9c8\uc744 \ud558\ub77c\ub294 \ub465, \uc5c9\ub6b1\ud55c \uc8fc\ubb38\uc744 \ub9ce\uc774 \ud588\uc8e0. \ub204\uad70\uac00\uc5d0\uac8c\ub294 \uc77c\uc0c1\uc801\uc778 \uc77c\uc778\ub370, \uc5f0\uc608\uc778\uc774 \uc591\uc740 \ub0c4\ube44 \ub69c\uaed1\uc5d0 \ub77c\uba74\uc744 \ub35c\uc5b4 \uba39\ub2e4 \ud758\ub9ac\uace0, \ubc84\uc2a4\ub97c \ud0c0\ub824\ub2e4\uac00 \uc0ac\ub78c\ub4e4 \uc2dc\uc120\uc774 \ub450\ub824\uc6cc \ud0dd\uc2dc\ub97c \ud0c0\uace0 \uc591\uce58\uc9c8\ub3c4 \ud559\uad50\uc5d0\uc11c\ub098 \uac00\uc11c \ud560 \uc218 \uc788\ub2e4\ub294 \uc0c1\ud669\uc73c\ub85c \ub9cc\ub4e4\uc5b4\uac00\ub354\ub77c\uace0\uc694(\uc6c3\uc74c).", "paragraph_answer": "2013\ub144 3\uc6d4 \uc5ec\uc131\ub3d9\uc544\uc640\uc758 \uc778\ud130\ubdf0\uc5d0\uc11c \uc774\uc7ac\uc9c4 \uc740 \ub2e4\uc74c\uacfc \uac19\uc774 \ub9d0\ud588\ub2e4.[21]. \uc6d0\ubb38: \uadf8\ub7f0 \uc774\ubbf8\uc9c0 \ub54c\ubb38\uc5d0 \u2018\ub3c8 \ud544\uc694\ud558\uc9c0?\u2019 \ud558\uba74\uc11c \uc811\uadfc\ud558\ub294 \uc0ac\ub78c\ub4e4\uc774 \ub9ce\uc558\uc5b4\uc694. \uadf8\ub7f0\ub370 \uc815\uc791 \uadf8\ub54c\ub294 \uacbd\uc81c\uc801\uc778 \uc5b4\ub824\uc6c0\uc5d0\uc11c \ubc97\uc5b4\ub09c \uc0c1\ud0dc\uc600\uac70\ub4e0\uc694. \uc81d\uc2a4\ud0a4\uc2a4 \ub370\ubdd4 \ud6c4 \uba87 \uac1c\uc6d4 \ub9cc\uc5d0 \ubd80\ubaa8\ub2d8\uaed8 \uc9d1\uc744 \uc0ac\ub4dc\ub9ac\uace0\ub3c4 \ub0a8\uc744 \ub9cc\ud07c \ubc8c\uc5c8\uc73c\ub2c8\uae4c\uc694. \ud558\uc9c0\ub9cc \ub298 \uc800\ud76c \uc9d1 \uac00\uc815 \ud615\ud3b8\uc774 \uc5c4\uccad \uc5b4\ub824\uc6b4 \uac83\ucc98\ub7fc \ubc29\uc1a1\uc5d0\uc11c \uadf8\ub824\uc9c0\uace4 \ud588\uc5b4\uc694. \uc815\uc791 \uc800\uc5d0\uac8c \uac00\uc218 \ud65c\ub3d9\uc73c\ub85c \ubc8c\uc5c8\ub358 \ub3c8\uc740 \ubb34\ucc99 \ud070\ub3c8\uc774\uc5c8\uace0, \ubd80\ubaa8\ub2d8\uaed8 \uc804\ubd80 \ub4dc\ub824\uc11c \ucc28\uace1\ucc28\uace1 \ubaa8\uc544\uc654\uc5c8\uc8e0. \ub450 \ubd84\uc774 \ub3cc\uc544\uac00\uc2e0 \uc9c0\uae08\ub3c4 \uadf8 \ub3c8\uc740 \uadf8\ub300\ub85c \ub0a8\uc544 \uc788\uc73c\ub2c8\uae4c\uc694. (\uc911\ub7b5) \ub2e4\ud050\uba58\ud130\ub9ac\ub97c \ucc0d\uc740 \uc801\uc774 \uc788\uc5c8\uc5b4\uc694. \uc9d1\uc5d0 \uac00\uc11c \ub77c\uba74\uc744 \ub053\uc5ec \uba39\uc73c\ub77c\ub294 \ub465, \ubc84\uc2a4\ub97c \ud0c0\ub77c\ub294 \ub465, \ud559\uad50\uc5d0 \uac00\uc11c \uc591\uce58\uc9c8\uc744 \ud558\ub77c\ub294 \ub465, \uc5c9\ub6b1\ud55c \uc8fc\ubb38\uc744 \ub9ce\uc774 \ud588\uc8e0. \ub204\uad70\uac00\uc5d0\uac8c\ub294 \uc77c\uc0c1\uc801\uc778 \uc77c\uc778\ub370, \uc5f0\uc608\uc778\uc774 \uc591\uc740 \ub0c4\ube44 \ub69c\uaed1\uc5d0 \ub77c\uba74\uc744 \ub35c\uc5b4 \uba39\ub2e4 \ud758\ub9ac\uace0, \ubc84\uc2a4\ub97c \ud0c0\ub824\ub2e4\uac00 \uc0ac\ub78c\ub4e4 \uc2dc\uc120\uc774 \ub450\ub824\uc6cc \ud0dd\uc2dc\ub97c \ud0c0\uace0 \uc591\uce58\uc9c8\ub3c4 \ud559\uad50\uc5d0\uc11c\ub098 \uac00\uc11c \ud560 \uc218 \uc788\ub2e4\ub294 \uc0c1\ud669\uc73c\ub85c \ub9cc\ub4e4\uc5b4\uac00\ub354\ub77c\uace0\uc694(\uc6c3\uc74c).", "sentence_answer": "2013\ub144 3\uc6d4 \uc5ec\uc131\ub3d9\uc544\uc640\uc758 \uc778\ud130\ubdf0\uc5d0\uc11c \uc774\uc7ac\uc9c4 \uc740 \ub2e4\uc74c\uacfc \uac19\uc774 \ub9d0\ud588\ub2e4.[21].", "paragraph_id": "6187340-4-0", "paragraph_question": "question: 2013\ub144 3\uc6d4 \uc5ec\uc131\ub3d9\uc544\uc640\uc758 \uc778\ud130\ubdf0\uc5d0\uc11c \ubd80\ubaa8\ub2d8 \uc9d1\uc744 \uc0ac\ub4dc\ub9ac\uace0\ub3c4 \ub0a8\uc744\ub9cc\ud07c \ub3c8\uc744 \ubc8c\uc5c8\ub2e4\uace0 \ubc1d\ud78c \uc81d\uc2a4\ud0a4\uc2a4 \uba64\ubc84\ub294?, context: 2013\ub144 3\uc6d4 \uc5ec\uc131\ub3d9\uc544\uc640\uc758 \uc778\ud130\ubdf0\uc5d0\uc11c \uc774\uc7ac\uc9c4\uc740 \ub2e4\uc74c\uacfc \uac19\uc774 \ub9d0\ud588\ub2e4.[21]. \uc6d0\ubb38: \uadf8\ub7f0 \uc774\ubbf8\uc9c0 \ub54c\ubb38\uc5d0 \u2018\ub3c8 \ud544\uc694\ud558\uc9c0?\u2019 \ud558\uba74\uc11c \uc811\uadfc\ud558\ub294 \uc0ac\ub78c\ub4e4\uc774 \ub9ce\uc558\uc5b4\uc694. \uadf8\ub7f0\ub370 \uc815\uc791 \uadf8\ub54c\ub294 \uacbd\uc81c\uc801\uc778 \uc5b4\ub824\uc6c0\uc5d0\uc11c \ubc97\uc5b4\ub09c \uc0c1\ud0dc\uc600\uac70\ub4e0\uc694. \uc81d\uc2a4\ud0a4\uc2a4 \ub370\ubdd4 \ud6c4 \uba87 \uac1c\uc6d4 \ub9cc\uc5d0 \ubd80\ubaa8\ub2d8\uaed8 \uc9d1\uc744 \uc0ac\ub4dc\ub9ac\uace0\ub3c4 \ub0a8\uc744 \ub9cc\ud07c \ubc8c\uc5c8\uc73c\ub2c8\uae4c\uc694. \ud558\uc9c0\ub9cc \ub298 \uc800\ud76c \uc9d1 \uac00\uc815 \ud615\ud3b8\uc774 \uc5c4\uccad \uc5b4\ub824\uc6b4 \uac83\ucc98\ub7fc \ubc29\uc1a1\uc5d0\uc11c \uadf8\ub824\uc9c0\uace4 \ud588\uc5b4\uc694. \uc815\uc791 \uc800\uc5d0\uac8c \uac00\uc218 \ud65c\ub3d9\uc73c\ub85c \ubc8c\uc5c8\ub358 \ub3c8\uc740 \ubb34\ucc99 \ud070\ub3c8\uc774\uc5c8\uace0, \ubd80\ubaa8\ub2d8\uaed8 \uc804\ubd80 \ub4dc\ub824\uc11c \ucc28\uace1\ucc28\uace1 \ubaa8\uc544\uc654\uc5c8\uc8e0. \ub450 \ubd84\uc774 \ub3cc\uc544\uac00\uc2e0 \uc9c0\uae08\ub3c4 \uadf8 \ub3c8\uc740 \uadf8\ub300\ub85c \ub0a8\uc544 \uc788\uc73c\ub2c8\uae4c\uc694. (\uc911\ub7b5) \ub2e4\ud050\uba58\ud130\ub9ac\ub97c \ucc0d\uc740 \uc801\uc774 \uc788\uc5c8\uc5b4\uc694. \uc9d1\uc5d0 \uac00\uc11c \ub77c\uba74\uc744 \ub053\uc5ec \uba39\uc73c\ub77c\ub294 \ub465, \ubc84\uc2a4\ub97c \ud0c0\ub77c\ub294 \ub465, \ud559\uad50\uc5d0 \uac00\uc11c \uc591\uce58\uc9c8\uc744 \ud558\ub77c\ub294 \ub465, \uc5c9\ub6b1\ud55c \uc8fc\ubb38\uc744 \ub9ce\uc774 \ud588\uc8e0. \ub204\uad70\uac00\uc5d0\uac8c\ub294 \uc77c\uc0c1\uc801\uc778 \uc77c\uc778\ub370, \uc5f0\uc608\uc778\uc774 \uc591\uc740 \ub0c4\ube44 \ub69c\uaed1\uc5d0 \ub77c\uba74\uc744 \ub35c\uc5b4 \uba39\ub2e4 \ud758\ub9ac\uace0, \ubc84\uc2a4\ub97c \ud0c0\ub824\ub2e4\uac00 \uc0ac\ub78c\ub4e4 \uc2dc\uc120\uc774 \ub450\ub824\uc6cc \ud0dd\uc2dc\ub97c \ud0c0\uace0 \uc591\uce58\uc9c8\ub3c4 \ud559\uad50\uc5d0\uc11c\ub098 \uac00\uc11c \ud560 \uc218 \uc788\ub2e4\ub294 \uc0c1\ud669\uc73c\ub85c \ub9cc\ub4e4\uc5b4\uac00\ub354\ub77c\uace0\uc694(\uc6c3\uc74c)."} {"question": "\ud575\uc735\ud569 \ubc18\uc751\uc744 \uc77c\uc73c\ud0a4\uae30 \uc704\ud574 \ud544\uc694\ud55c \uccab \ubc88\uc9f8 \uae30\uc220\ub85c \ud50c\ub77c\uc988\ub9c8\ub97c \uac00\uc5f4\ud558\ub294 \uae30\uc220\uc774 \ud544\uc694\ud558\ub2e4. \uc5ec\uae30\uc11c \ud50c\ub77c\uc988\ub9c8\ub97c \uba87 \ub3c4 \uc774\uc0c1\uc73c\ub85c \uac00\uc5f4\ud574\uc57c \ud558\ub294\uac00?", "paragraph": "\ud575\uc735\ud569\uc740 \ub450 \uac1c\uc758 \uc6d0\uc790\uac00 \ud569\uccd0\uc838 \uc0c8\ub85c\uc6b4 \ubb3c\uc9c8\uc744 \ub9cc\ub4e4 \ub54c \uc9c8\ub7c9\uc774 \uc190\uc2e4\ub418\uc5b4 \uc5d0\ub108\uc9c0\uac00 \ubc29\ucd9c\ub418\ub294 \uac83\uc744 \ub9d0\ud55c\ub2e4. \ube44\uc2b7\ud55c \uc6d0\ub9ac\ub85c \uc6d0\uc790\ub825\uc774\ub77c\uace0 \ubd88\ub9ac\ub294 \ud575\ubd84\uc5f4 \ubc18\uc751\uc740 \ubb34\uac70\uc6b4 \uc6d0\uc790\uac00 \uc678\ubd80 \uc790\uadf9\uc5d0 \uc758\ud574 \ubd84\uc5f4\ud558\uac8c \ub420 \ub54c \uc9c8\ub7c9\uc190\uc2e4\uc774 \uc77c\uc5b4\ub098\uac8c \ub418\uc5b4 \uc544\uc778\uc288\ud0c0\uc778\uc758 \uc0c1\ub300\uc131 \uc6d0\ub9ac \uacf5\uc2dd\uc5d0 \ub530\ub77c \uc5d0\ub108\uc9c0\ub97c \ubc29\ucd9c\ud55c\ub2e4. \uc608\ub97c \ub4e4\uba74 \ud604\uc7ac \ud55c\uad6d\uc5d0\uc11c \uac00\ub3d9 \uc911\uc778 \ud575\ubd84\uc5f4 \uc6d0\uc790\ub825 \ubc1c\uc804\uc18c\uc5d0\uc11c\ub294 \uc6b0\ub77c\ub284\uc744 \ubd84\ud574\ud558\uc5ec \uc804\uae30\ub97c \uc5bb\uace0 \uc788\ub2e4. \ud575\uc735\ud569 \ubc18\uc751\uc744 \uac00\uc7a5 \uc27d\uac8c \ubcfc \uc218 \uc788\ub294 \uac83\uc740 \ud0dc\uc591\uc774\ub2e4. \ud0dc\uc591\uc774 \ubd88\ud0c0\uace0 \uc788\ub294 \uc774\uc720\ub294 \ud0dc\uc591 \ub0b4\ubd80\uc5d0\uc11c \uc218\uc18c, \ud5ec\ub968\uc774 \uc11c\ub85c \ud575\uc735\ud569 \ubc18\uc751\uc744 \ud558\uace0 \uc788\uae30 \ub54c\ubb38\uc774\ub2e4. \uc774 \uacfc\uc815\uc744 \ud1b5\ud574 \ud0dc\uc591\uc740 \ucd08\ub2f9 4\uc870W\uc758 100\uc870\ubc30\uc5d0 \ub2ec\ud558\ub294 \uc5d0\ub108\uc9c0\ub97c \ubc29\ucd9c\ud558\uace0 \uc788\ub2e4. \uc774\ub7f0 \ud575\uc735\ud569 \ubc18\uc751\uc740 \uc911\uc218\uc18c\uc640 \uc0bc\uc911\uc218\uc18c\uc758 \ubc18\uc751, \uc911\uc218\uc18c\uc640 \uc911\uc218\uc18c\uc758 \ubc18\uc751, \uc911\uc218\uc18c\uc640 \ud5ec\ub968\uc758 \ubc18\uc751, \uc911\uc218\uc18c\uc640 \ub9ac\ud2ac\uc758 \ubc18\uc751 \ub4f1 \uc5ec\ub7ec \uac00\uc9c0 \ud615\ud0dc\ub85c \ub098\ud0c0\ub0a0 \uc218 \uc788\ub2e4. \uc774 \uc911 \ud575\uc735\ud569 \ubc1c\uc804\uc744 \uc704\ud574 \uac00\ub2a5\ud55c \uc5f0\ub8cc\ub85c\ub294 \uc9c0\uad6c \uc0c1\uc5d0\uc11c \ub204\uad6c\ub098 \uc27d\uac8c \uc5bb\uc744 \uc218 \uc788\uc5b4\uc57c \ud558\uba70 \ud575\uc735\ud569 \ubc18\uc751\uc774 \uc27d\uac8c \uc77c\uc5b4\ub0a0 \uc218 \uc788\uc5b4\uc57c \ud55c\ub2e4\ub294 \uc870\uac74\uc774 \uc788\uc5b4\uc57c \ud55c\ub2e4. \uc774\ub97c \ub9cc\uc871\ud558\ub294 \uc5f0\ub8cc\ub85c \ud604\uc7ac\ub294 \uc911\uc218\uc18c\uc640 \uc0bc\uc911\uc218\uc18c\uc758 \ubc18\uc751\uc774 \ud575\uc735\ud569 \ubc18\uc751\uc73c\ub85c \ucd94\ub300\ub418\uace0 \uc788\ub2e4. \uc65c\ub0d0\ud558\uba74 \uc911\uc218\uc18c\ub294 \ubc14\ub2f7\ubb3c\uc5d0\uc11c \uc544\uc8fc \uc27d\uace0 \uc800\ub834\ud558\uac8c \uad6c\ud560 \uc218 \uc788\uc73c\uba70 \uc0bc\uc911\uc218\uc18c \ub610\ud55c \ubc14\ub2f7\ubb3c\uc5d0\uc11c \uad6c\ud55c \ub9ac\ud2ac\uc744 \ud1b5\ud574 \uc27d\uac8c \ub9cc\ub4e4 \uc218 \uc788\uae30 \ub54c\ubb38\uc774\ub2e4. \uc774\ub7f0 \ud575\uc735\ud569 \ubc18\uc751\uc774 \uc77c\uc5b4\ub098\uae30 \uc704\ud574\uc11c\ub294 \ub450 \uac00\uc9c0 \uc911\uc694\ud55c \uae30\uc220\uc774 \ud544\uc694\ud558\ub2e4. \uccab \ubc88\uc9f8 \uae30\uc220\uc740 \ud50c\ub77c\uc988\ub9c8\ub97c 1\uc5b5\ub3c4\uc528 \uc774\uc0c1\uc73c\ub85c \uac00\uc5f4\ud558\ub294 \uae30\uc220\uc774\uace0 \ub450 \ubc88\uc9f8\ub294 \uc774\ub7f0 \ucd08\uace0\uc628 \ud50c\ub77c\uc988\ub9c8\ub97c \ud575\uc735\ud569 \uc7a5\uce58\uc5d0 \uac00\ub450\ub294 \uae30\uc220\uc774 \ud544\uc694\ud558\ub2e4.", "answer": "1\uc5b5\ub3c4\uc528", "sentence": "\uccab \ubc88\uc9f8 \uae30\uc220\uc740 \ud50c\ub77c\uc988\ub9c8\ub97c 1\uc5b5\ub3c4\uc528 \uc774\uc0c1\uc73c\ub85c \uac00\uc5f4\ud558\ub294 \uae30\uc220\uc774\uace0 \ub450 \ubc88\uc9f8\ub294 \uc774\ub7f0 \ucd08\uace0\uc628 \ud50c\ub77c\uc988\ub9c8\ub97c \ud575\uc735\ud569 \uc7a5\uce58\uc5d0 \uac00\ub450\ub294 \uae30\uc220\uc774 \ud544\uc694\ud558\ub2e4.", "paragraph_sentence": "\ud575\uc735\ud569\uc740 \ub450 \uac1c\uc758 \uc6d0\uc790\uac00 \ud569\uccd0\uc838 \uc0c8\ub85c\uc6b4 \ubb3c\uc9c8\uc744 \ub9cc\ub4e4 \ub54c \uc9c8\ub7c9\uc774 \uc190\uc2e4\ub418\uc5b4 \uc5d0\ub108\uc9c0\uac00 \ubc29\ucd9c\ub418\ub294 \uac83\uc744 \ub9d0\ud55c\ub2e4. \ube44\uc2b7\ud55c \uc6d0\ub9ac\ub85c \uc6d0\uc790\ub825\uc774\ub77c\uace0 \ubd88\ub9ac\ub294 \ud575\ubd84\uc5f4 \ubc18\uc751\uc740 \ubb34\uac70\uc6b4 \uc6d0\uc790\uac00 \uc678\ubd80 \uc790\uadf9\uc5d0 \uc758\ud574 \ubd84\uc5f4\ud558\uac8c \ub420 \ub54c \uc9c8\ub7c9\uc190\uc2e4\uc774 \uc77c\uc5b4\ub098\uac8c \ub418\uc5b4 \uc544\uc778\uc288\ud0c0\uc778\uc758 \uc0c1\ub300\uc131 \uc6d0\ub9ac \uacf5\uc2dd\uc5d0 \ub530\ub77c \uc5d0\ub108\uc9c0\ub97c \ubc29\ucd9c\ud55c\ub2e4. \uc608\ub97c \ub4e4\uba74 \ud604\uc7ac \ud55c\uad6d\uc5d0\uc11c \uac00\ub3d9 \uc911\uc778 \ud575\ubd84\uc5f4 \uc6d0\uc790\ub825 \ubc1c\uc804\uc18c\uc5d0\uc11c\ub294 \uc6b0\ub77c\ub284\uc744 \ubd84\ud574\ud558\uc5ec \uc804\uae30\ub97c \uc5bb\uace0 \uc788\ub2e4. \ud575\uc735\ud569 \ubc18\uc751\uc744 \uac00\uc7a5 \uc27d\uac8c \ubcfc \uc218 \uc788\ub294 \uac83\uc740 \ud0dc\uc591\uc774\ub2e4. \ud0dc\uc591\uc774 \ubd88\ud0c0\uace0 \uc788\ub294 \uc774\uc720\ub294 \ud0dc\uc591 \ub0b4\ubd80\uc5d0\uc11c \uc218\uc18c, \ud5ec\ub968\uc774 \uc11c\ub85c \ud575\uc735\ud569 \ubc18\uc751\uc744 \ud558\uace0 \uc788\uae30 \ub54c\ubb38\uc774\ub2e4. \uc774 \uacfc\uc815\uc744 \ud1b5\ud574 \ud0dc\uc591\uc740 \ucd08\ub2f9 4\uc870W\uc758 100\uc870\ubc30\uc5d0 \ub2ec\ud558\ub294 \uc5d0\ub108\uc9c0\ub97c \ubc29\ucd9c\ud558\uace0 \uc788\ub2e4. \uc774\ub7f0 \ud575\uc735\ud569 \ubc18\uc751\uc740 \uc911\uc218\uc18c\uc640 \uc0bc\uc911\uc218\uc18c\uc758 \ubc18\uc751, \uc911\uc218\uc18c\uc640 \uc911\uc218\uc18c\uc758 \ubc18\uc751, \uc911\uc218\uc18c\uc640 \ud5ec\ub968\uc758 \ubc18\uc751, \uc911\uc218\uc18c\uc640 \ub9ac\ud2ac\uc758 \ubc18\uc751 \ub4f1 \uc5ec\ub7ec \uac00\uc9c0 \ud615\ud0dc\ub85c \ub098\ud0c0\ub0a0 \uc218 \uc788\ub2e4. \uc774 \uc911 \ud575\uc735\ud569 \ubc1c\uc804\uc744 \uc704\ud574 \uac00\ub2a5\ud55c \uc5f0\ub8cc\ub85c\ub294 \uc9c0\uad6c \uc0c1\uc5d0\uc11c \ub204\uad6c\ub098 \uc27d\uac8c \uc5bb\uc744 \uc218 \uc788\uc5b4\uc57c \ud558\uba70 \ud575\uc735\ud569 \ubc18\uc751\uc774 \uc27d\uac8c \uc77c\uc5b4\ub0a0 \uc218 \uc788\uc5b4\uc57c \ud55c\ub2e4\ub294 \uc870\uac74\uc774 \uc788\uc5b4\uc57c \ud55c\ub2e4. \uc774\ub97c \ub9cc\uc871\ud558\ub294 \uc5f0\ub8cc\ub85c \ud604\uc7ac\ub294 \uc911\uc218\uc18c\uc640 \uc0bc\uc911\uc218\uc18c\uc758 \ubc18\uc751\uc774 \ud575\uc735\ud569 \ubc18\uc751\uc73c\ub85c \ucd94\ub300\ub418\uace0 \uc788\ub2e4. \uc65c\ub0d0\ud558\uba74 \uc911\uc218\uc18c\ub294 \ubc14\ub2f7\ubb3c\uc5d0\uc11c \uc544\uc8fc \uc27d\uace0 \uc800\ub834\ud558\uac8c \uad6c\ud560 \uc218 \uc788\uc73c\uba70 \uc0bc\uc911\uc218\uc18c \ub610\ud55c \ubc14\ub2f7\ubb3c\uc5d0\uc11c \uad6c\ud55c \ub9ac\ud2ac\uc744 \ud1b5\ud574 \uc27d\uac8c \ub9cc\ub4e4 \uc218 \uc788\uae30 \ub54c\ubb38\uc774\ub2e4. \uc774\ub7f0 \ud575\uc735\ud569 \ubc18\uc751\uc774 \uc77c\uc5b4\ub098\uae30 \uc704\ud574\uc11c\ub294 \ub450 \uac00\uc9c0 \uc911\uc694\ud55c \uae30\uc220\uc774 \ud544\uc694\ud558\ub2e4. \uccab \ubc88\uc9f8 \uae30\uc220\uc740 \ud50c\ub77c\uc988\ub9c8\ub97c 1\uc5b5\ub3c4\uc528 \uc774\uc0c1\uc73c\ub85c \uac00\uc5f4\ud558\ub294 \uae30\uc220\uc774\uace0 \ub450 \ubc88\uc9f8\ub294 \uc774\ub7f0 \ucd08\uace0\uc628 \ud50c\ub77c\uc988\ub9c8\ub97c \ud575\uc735\ud569 \uc7a5\uce58\uc5d0 \uac00\ub450\ub294 \uae30\uc220\uc774 \ud544\uc694\ud558\ub2e4. ", "paragraph_answer": "\ud575\uc735\ud569\uc740 \ub450 \uac1c\uc758 \uc6d0\uc790\uac00 \ud569\uccd0\uc838 \uc0c8\ub85c\uc6b4 \ubb3c\uc9c8\uc744 \ub9cc\ub4e4 \ub54c \uc9c8\ub7c9\uc774 \uc190\uc2e4\ub418\uc5b4 \uc5d0\ub108\uc9c0\uac00 \ubc29\ucd9c\ub418\ub294 \uac83\uc744 \ub9d0\ud55c\ub2e4. \ube44\uc2b7\ud55c \uc6d0\ub9ac\ub85c \uc6d0\uc790\ub825\uc774\ub77c\uace0 \ubd88\ub9ac\ub294 \ud575\ubd84\uc5f4 \ubc18\uc751\uc740 \ubb34\uac70\uc6b4 \uc6d0\uc790\uac00 \uc678\ubd80 \uc790\uadf9\uc5d0 \uc758\ud574 \ubd84\uc5f4\ud558\uac8c \ub420 \ub54c \uc9c8\ub7c9\uc190\uc2e4\uc774 \uc77c\uc5b4\ub098\uac8c \ub418\uc5b4 \uc544\uc778\uc288\ud0c0\uc778\uc758 \uc0c1\ub300\uc131 \uc6d0\ub9ac \uacf5\uc2dd\uc5d0 \ub530\ub77c \uc5d0\ub108\uc9c0\ub97c \ubc29\ucd9c\ud55c\ub2e4. \uc608\ub97c \ub4e4\uba74 \ud604\uc7ac \ud55c\uad6d\uc5d0\uc11c \uac00\ub3d9 \uc911\uc778 \ud575\ubd84\uc5f4 \uc6d0\uc790\ub825 \ubc1c\uc804\uc18c\uc5d0\uc11c\ub294 \uc6b0\ub77c\ub284\uc744 \ubd84\ud574\ud558\uc5ec \uc804\uae30\ub97c \uc5bb\uace0 \uc788\ub2e4. \ud575\uc735\ud569 \ubc18\uc751\uc744 \uac00\uc7a5 \uc27d\uac8c \ubcfc \uc218 \uc788\ub294 \uac83\uc740 \ud0dc\uc591\uc774\ub2e4. \ud0dc\uc591\uc774 \ubd88\ud0c0\uace0 \uc788\ub294 \uc774\uc720\ub294 \ud0dc\uc591 \ub0b4\ubd80\uc5d0\uc11c \uc218\uc18c, \ud5ec\ub968\uc774 \uc11c\ub85c \ud575\uc735\ud569 \ubc18\uc751\uc744 \ud558\uace0 \uc788\uae30 \ub54c\ubb38\uc774\ub2e4. \uc774 \uacfc\uc815\uc744 \ud1b5\ud574 \ud0dc\uc591\uc740 \ucd08\ub2f9 4\uc870W\uc758 100\uc870\ubc30\uc5d0 \ub2ec\ud558\ub294 \uc5d0\ub108\uc9c0\ub97c \ubc29\ucd9c\ud558\uace0 \uc788\ub2e4. \uc774\ub7f0 \ud575\uc735\ud569 \ubc18\uc751\uc740 \uc911\uc218\uc18c\uc640 \uc0bc\uc911\uc218\uc18c\uc758 \ubc18\uc751, \uc911\uc218\uc18c\uc640 \uc911\uc218\uc18c\uc758 \ubc18\uc751, \uc911\uc218\uc18c\uc640 \ud5ec\ub968\uc758 \ubc18\uc751, \uc911\uc218\uc18c\uc640 \ub9ac\ud2ac\uc758 \ubc18\uc751 \ub4f1 \uc5ec\ub7ec \uac00\uc9c0 \ud615\ud0dc\ub85c \ub098\ud0c0\ub0a0 \uc218 \uc788\ub2e4. \uc774 \uc911 \ud575\uc735\ud569 \ubc1c\uc804\uc744 \uc704\ud574 \uac00\ub2a5\ud55c \uc5f0\ub8cc\ub85c\ub294 \uc9c0\uad6c \uc0c1\uc5d0\uc11c \ub204\uad6c\ub098 \uc27d\uac8c \uc5bb\uc744 \uc218 \uc788\uc5b4\uc57c \ud558\uba70 \ud575\uc735\ud569 \ubc18\uc751\uc774 \uc27d\uac8c \uc77c\uc5b4\ub0a0 \uc218 \uc788\uc5b4\uc57c \ud55c\ub2e4\ub294 \uc870\uac74\uc774 \uc788\uc5b4\uc57c \ud55c\ub2e4. \uc774\ub97c \ub9cc\uc871\ud558\ub294 \uc5f0\ub8cc\ub85c \ud604\uc7ac\ub294 \uc911\uc218\uc18c\uc640 \uc0bc\uc911\uc218\uc18c\uc758 \ubc18\uc751\uc774 \ud575\uc735\ud569 \ubc18\uc751\uc73c\ub85c \ucd94\ub300\ub418\uace0 \uc788\ub2e4. \uc65c\ub0d0\ud558\uba74 \uc911\uc218\uc18c\ub294 \ubc14\ub2f7\ubb3c\uc5d0\uc11c \uc544\uc8fc \uc27d\uace0 \uc800\ub834\ud558\uac8c \uad6c\ud560 \uc218 \uc788\uc73c\uba70 \uc0bc\uc911\uc218\uc18c \ub610\ud55c \ubc14\ub2f7\ubb3c\uc5d0\uc11c \uad6c\ud55c \ub9ac\ud2ac\uc744 \ud1b5\ud574 \uc27d\uac8c \ub9cc\ub4e4 \uc218 \uc788\uae30 \ub54c\ubb38\uc774\ub2e4. \uc774\ub7f0 \ud575\uc735\ud569 \ubc18\uc751\uc774 \uc77c\uc5b4\ub098\uae30 \uc704\ud574\uc11c\ub294 \ub450 \uac00\uc9c0 \uc911\uc694\ud55c \uae30\uc220\uc774 \ud544\uc694\ud558\ub2e4. \uccab \ubc88\uc9f8 \uae30\uc220\uc740 \ud50c\ub77c\uc988\ub9c8\ub97c 1\uc5b5\ub3c4\uc528 \uc774\uc0c1\uc73c\ub85c \uac00\uc5f4\ud558\ub294 \uae30\uc220\uc774\uace0 \ub450 \ubc88\uc9f8\ub294 \uc774\ub7f0 \ucd08\uace0\uc628 \ud50c\ub77c\uc988\ub9c8\ub97c \ud575\uc735\ud569 \uc7a5\uce58\uc5d0 \uac00\ub450\ub294 \uae30\uc220\uc774 \ud544\uc694\ud558\ub2e4.", "sentence_answer": "\uccab \ubc88\uc9f8 \uae30\uc220\uc740 \ud50c\ub77c\uc988\ub9c8\ub97c 1\uc5b5\ub3c4\uc528 \uc774\uc0c1\uc73c\ub85c \uac00\uc5f4\ud558\ub294 \uae30\uc220\uc774\uace0 \ub450 \ubc88\uc9f8\ub294 \uc774\ub7f0 \ucd08\uace0\uc628 \ud50c\ub77c\uc988\ub9c8\ub97c \ud575\uc735\ud569 \uc7a5\uce58\uc5d0 \uac00\ub450\ub294 \uae30\uc220\uc774 \ud544\uc694\ud558\ub2e4.", "paragraph_id": "5915496-0-2", "paragraph_question": "question: \ud575\uc735\ud569 \ubc18\uc751\uc744 \uc77c\uc73c\ud0a4\uae30 \uc704\ud574 \ud544\uc694\ud55c \uccab \ubc88\uc9f8 \uae30\uc220\ub85c \ud50c\ub77c\uc988\ub9c8\ub97c \uac00\uc5f4\ud558\ub294 \uae30\uc220\uc774 \ud544\uc694\ud558\ub2e4. \uc5ec\uae30\uc11c \ud50c\ub77c\uc988\ub9c8\ub97c \uba87 \ub3c4 \uc774\uc0c1\uc73c\ub85c \uac00\uc5f4\ud574\uc57c \ud558\ub294\uac00?, context: \ud575\uc735\ud569\uc740 \ub450 \uac1c\uc758 \uc6d0\uc790\uac00 \ud569\uccd0\uc838 \uc0c8\ub85c\uc6b4 \ubb3c\uc9c8\uc744 \ub9cc\ub4e4 \ub54c \uc9c8\ub7c9\uc774 \uc190\uc2e4\ub418\uc5b4 \uc5d0\ub108\uc9c0\uac00 \ubc29\ucd9c\ub418\ub294 \uac83\uc744 \ub9d0\ud55c\ub2e4. \ube44\uc2b7\ud55c \uc6d0\ub9ac\ub85c \uc6d0\uc790\ub825\uc774\ub77c\uace0 \ubd88\ub9ac\ub294 \ud575\ubd84\uc5f4 \ubc18\uc751\uc740 \ubb34\uac70\uc6b4 \uc6d0\uc790\uac00 \uc678\ubd80 \uc790\uadf9\uc5d0 \uc758\ud574 \ubd84\uc5f4\ud558\uac8c \ub420 \ub54c \uc9c8\ub7c9\uc190\uc2e4\uc774 \uc77c\uc5b4\ub098\uac8c \ub418\uc5b4 \uc544\uc778\uc288\ud0c0\uc778\uc758 \uc0c1\ub300\uc131 \uc6d0\ub9ac \uacf5\uc2dd\uc5d0 \ub530\ub77c \uc5d0\ub108\uc9c0\ub97c \ubc29\ucd9c\ud55c\ub2e4. \uc608\ub97c \ub4e4\uba74 \ud604\uc7ac \ud55c\uad6d\uc5d0\uc11c \uac00\ub3d9 \uc911\uc778 \ud575\ubd84\uc5f4 \uc6d0\uc790\ub825 \ubc1c\uc804\uc18c\uc5d0\uc11c\ub294 \uc6b0\ub77c\ub284\uc744 \ubd84\ud574\ud558\uc5ec \uc804\uae30\ub97c \uc5bb\uace0 \uc788\ub2e4. \ud575\uc735\ud569 \ubc18\uc751\uc744 \uac00\uc7a5 \uc27d\uac8c \ubcfc \uc218 \uc788\ub294 \uac83\uc740 \ud0dc\uc591\uc774\ub2e4. \ud0dc\uc591\uc774 \ubd88\ud0c0\uace0 \uc788\ub294 \uc774\uc720\ub294 \ud0dc\uc591 \ub0b4\ubd80\uc5d0\uc11c \uc218\uc18c, \ud5ec\ub968\uc774 \uc11c\ub85c \ud575\uc735\ud569 \ubc18\uc751\uc744 \ud558\uace0 \uc788\uae30 \ub54c\ubb38\uc774\ub2e4. \uc774 \uacfc\uc815\uc744 \ud1b5\ud574 \ud0dc\uc591\uc740 \ucd08\ub2f9 4\uc870W\uc758 100\uc870\ubc30\uc5d0 \ub2ec\ud558\ub294 \uc5d0\ub108\uc9c0\ub97c \ubc29\ucd9c\ud558\uace0 \uc788\ub2e4. \uc774\ub7f0 \ud575\uc735\ud569 \ubc18\uc751\uc740 \uc911\uc218\uc18c\uc640 \uc0bc\uc911\uc218\uc18c\uc758 \ubc18\uc751, \uc911\uc218\uc18c\uc640 \uc911\uc218\uc18c\uc758 \ubc18\uc751, \uc911\uc218\uc18c\uc640 \ud5ec\ub968\uc758 \ubc18\uc751, \uc911\uc218\uc18c\uc640 \ub9ac\ud2ac\uc758 \ubc18\uc751 \ub4f1 \uc5ec\ub7ec \uac00\uc9c0 \ud615\ud0dc\ub85c \ub098\ud0c0\ub0a0 \uc218 \uc788\ub2e4. \uc774 \uc911 \ud575\uc735\ud569 \ubc1c\uc804\uc744 \uc704\ud574 \uac00\ub2a5\ud55c \uc5f0\ub8cc\ub85c\ub294 \uc9c0\uad6c \uc0c1\uc5d0\uc11c \ub204\uad6c\ub098 \uc27d\uac8c \uc5bb\uc744 \uc218 \uc788\uc5b4\uc57c \ud558\uba70 \ud575\uc735\ud569 \ubc18\uc751\uc774 \uc27d\uac8c \uc77c\uc5b4\ub0a0 \uc218 \uc788\uc5b4\uc57c \ud55c\ub2e4\ub294 \uc870\uac74\uc774 \uc788\uc5b4\uc57c \ud55c\ub2e4. \uc774\ub97c \ub9cc\uc871\ud558\ub294 \uc5f0\ub8cc\ub85c \ud604\uc7ac\ub294 \uc911\uc218\uc18c\uc640 \uc0bc\uc911\uc218\uc18c\uc758 \ubc18\uc751\uc774 \ud575\uc735\ud569 \ubc18\uc751\uc73c\ub85c \ucd94\ub300\ub418\uace0 \uc788\ub2e4. \uc65c\ub0d0\ud558\uba74 \uc911\uc218\uc18c\ub294 \ubc14\ub2f7\ubb3c\uc5d0\uc11c \uc544\uc8fc \uc27d\uace0 \uc800\ub834\ud558\uac8c \uad6c\ud560 \uc218 \uc788\uc73c\uba70 \uc0bc\uc911\uc218\uc18c \ub610\ud55c \ubc14\ub2f7\ubb3c\uc5d0\uc11c \uad6c\ud55c \ub9ac\ud2ac\uc744 \ud1b5\ud574 \uc27d\uac8c \ub9cc\ub4e4 \uc218 \uc788\uae30 \ub54c\ubb38\uc774\ub2e4. \uc774\ub7f0 \ud575\uc735\ud569 \ubc18\uc751\uc774 \uc77c\uc5b4\ub098\uae30 \uc704\ud574\uc11c\ub294 \ub450 \uac00\uc9c0 \uc911\uc694\ud55c \uae30\uc220\uc774 \ud544\uc694\ud558\ub2e4. \uccab \ubc88\uc9f8 \uae30\uc220\uc740 \ud50c\ub77c\uc988\ub9c8\ub97c 1\uc5b5\ub3c4\uc528 \uc774\uc0c1\uc73c\ub85c \uac00\uc5f4\ud558\ub294 \uae30\uc220\uc774\uace0 \ub450 \ubc88\uc9f8\ub294 \uc774\ub7f0 \ucd08\uace0\uc628 \ud50c\ub77c\uc988\ub9c8\ub97c \ud575\uc735\ud569 \uc7a5\uce58\uc5d0 \uac00\ub450\ub294 \uae30\uc220\uc774 \ud544\uc694\ud558\ub2e4."} {"question": "\ud3ec\uba54\ub77c\ub2c8\uc548\uc744 \ud63c\uc790\uc788\uc744 \uc218 \uc788\uac8c \ud6c8\ub828\uc2dc\ud0a4\uc9c0 \uc54a\uc73c\uba74 \ubb34\uc5c7\uc5d0 \uc2dc\ub2ec\ub9b4 \uc218 \uc788\ub098?", "paragraph": "\ud3ec\uba54\ub77c\ub2c8\uc548\uc740 \uc77c\ubc18\uc801\uc73c\ub85c \ub9e4\uc6b0 \uce5c\uadfc\ud558\uace0 \uc6d0\uae30\uc655\uc131\ud55c \uac1c\uc774\ub2e4. \uc774 \uac1c\ub294 \uc8fc\uc778\uc758 \uc8fc\ubcc0\uc5d0 \ud568\uaed8 \uc788\ub294 \uac83\uc744 \uc88b\uc544\ud558\uace0, \ubcf4\ud638\ud574\uc8fc\ub294 \uac83\uc73c\ub85c \uc54c\ub824\uc838 \uc788\ub2e4. \uc774 \ud488\uc885\uc740 \uc8fc\uc778\uacfc \uc720\ub300\uac10\uc774 \ube68\ub9ac \ud615\uc131\ub3fc \ud63c\uc790 \uc2dc\uac04\uc744 \ubcf4\ub0bc \uc218 \uc788\ub3c4\ub85d \ud6c8\ub828 \uc2dc\ud0a4\uc9c0 \uc54a\ub294\ub2e4\uba74, \ubd84\ub9ac \ubd88\uc548\uc5d0 \uc2dc\ub2ec\ub9b4 \uc218 \uc788\ub2e4. \ud3ec\uba54\ub77c\ub2c8\uc548\uc740 \uacbd\uacc4\uc640 \ud658\uacbd\uc758 \ubcc0\ud654\ub97c \uc54c\uc544\ucc44\uace0 \uc0c8\ub85c\uc6b4 \uc790\uadf9\uc5d0 \uc9d6\ub294 \ud589\ub3d9\uc740 \uc5b4\ub5a4 \uc0c1\ud669\uc5d0\uc11c\ub3c4 \uacfc\ub3c4\ud558\uac8c \uc9d6\ub294 \uc2b5\uad00\uc73c\ub85c \uc774\uc5b4 \uc9c8 \uc218 \uc788\ub2e4. \ud3ec\uba54\ub77c\ub2c8\uc548\uc740 \ub611\ub611\ud55c \uac1c\ub85c \ud6c8\ub828 \ubc18\uc751\uc774 \uc88b\uc740\ub370, \uc8fc\uc778\uc5d0\uac8c \uc5b4\ub5bb\uac8c \ud6c8\ub828\uc744 \ubc1b\ub290\ub0d0\uc5d0 \ub530\ub77c \ub9e4\uc6b0 \uc131\uacf5\uc801\uc778 \uc131\uacfc\ub97c \uc5bb\uc744 \uc218 \uc788\ub2e4. \uadf8\ub7ec\ub098 \ud6c8\ub828\uc744 \uc2dc\ud0a4\uc9c0 \uc54a\ub294\ub2e4\uba74, \uc9c0\ubc30\uc801\uc774\uace0 \uacf5\uaca9\uc801\uc73c\ub85c \ub420 \uc218 \uc788\ub2e4. \ub610\ud55c \ud765\ubd84\uc744 \uc798\ud558\uba70 \uae09\ud558\uace0 \ud638\uae30\uc2ec\uc774 \ub9ce\uc544 \ud0c0 \uacac\uc885\uc5d0\uac8c \uad00\uc2ec\uc774 \ub9ce\ub2e4.", "answer": "\ubd84\ub9ac \ubd88\uc548", "sentence": "\uc774 \ud488\uc885\uc740 \uc8fc\uc778\uacfc \uc720\ub300\uac10\uc774 \ube68\ub9ac \ud615\uc131\ub3fc \ud63c\uc790 \uc2dc\uac04\uc744 \ubcf4\ub0bc \uc218 \uc788\ub3c4\ub85d \ud6c8\ub828 \uc2dc\ud0a4\uc9c0 \uc54a\ub294\ub2e4\uba74, \ubd84\ub9ac \ubd88\uc548 \uc5d0 \uc2dc\ub2ec\ub9b4 \uc218 \uc788\ub2e4.", "paragraph_sentence": "\ud3ec\uba54\ub77c\ub2c8\uc548\uc740 \uc77c\ubc18\uc801\uc73c\ub85c \ub9e4\uc6b0 \uce5c\uadfc\ud558\uace0 \uc6d0\uae30\uc655\uc131\ud55c \uac1c\uc774\ub2e4. \uc774 \uac1c\ub294 \uc8fc\uc778\uc758 \uc8fc\ubcc0\uc5d0 \ud568\uaed8 \uc788\ub294 \uac83\uc744 \uc88b\uc544\ud558\uace0, \ubcf4\ud638\ud574\uc8fc\ub294 \uac83\uc73c\ub85c \uc54c\ub824\uc838 \uc788\ub2e4. \uc774 \ud488\uc885\uc740 \uc8fc\uc778\uacfc \uc720\ub300\uac10\uc774 \ube68\ub9ac \ud615\uc131\ub3fc \ud63c\uc790 \uc2dc\uac04\uc744 \ubcf4\ub0bc \uc218 \uc788\ub3c4\ub85d \ud6c8\ub828 \uc2dc\ud0a4\uc9c0 \uc54a\ub294\ub2e4\uba74, \ubd84\ub9ac \ubd88\uc548 \uc5d0 \uc2dc\ub2ec\ub9b4 \uc218 \uc788\ub2e4. \ud3ec\uba54\ub77c\ub2c8\uc548\uc740 \uacbd\uacc4\uc640 \ud658\uacbd\uc758 \ubcc0\ud654\ub97c \uc54c\uc544\ucc44\uace0 \uc0c8\ub85c\uc6b4 \uc790\uadf9\uc5d0 \uc9d6\ub294 \ud589\ub3d9\uc740 \uc5b4\ub5a4 \uc0c1\ud669\uc5d0\uc11c\ub3c4 \uacfc\ub3c4\ud558\uac8c \uc9d6\ub294 \uc2b5\uad00\uc73c\ub85c \uc774\uc5b4 \uc9c8 \uc218 \uc788\ub2e4. \ud3ec\uba54\ub77c\ub2c8\uc548\uc740 \ub611\ub611\ud55c \uac1c\ub85c \ud6c8\ub828 \ubc18\uc751\uc774 \uc88b\uc740\ub370, \uc8fc\uc778\uc5d0\uac8c \uc5b4\ub5bb\uac8c \ud6c8\ub828\uc744 \ubc1b\ub290\ub0d0\uc5d0 \ub530\ub77c \ub9e4\uc6b0 \uc131\uacf5\uc801\uc778 \uc131\uacfc\ub97c \uc5bb\uc744 \uc218 \uc788\ub2e4. \uadf8\ub7ec\ub098 \ud6c8\ub828\uc744 \uc2dc\ud0a4\uc9c0 \uc54a\ub294\ub2e4\uba74, \uc9c0\ubc30\uc801\uc774\uace0 \uacf5\uaca9\uc801\uc73c\ub85c \ub420 \uc218 \uc788\ub2e4. \ub610\ud55c \ud765\ubd84\uc744 \uc798\ud558\uba70 \uae09\ud558\uace0 \ud638\uae30\uc2ec\uc774 \ub9ce\uc544 \ud0c0 \uacac\uc885\uc5d0\uac8c \uad00\uc2ec\uc774 \ub9ce\ub2e4.", "paragraph_answer": "\ud3ec\uba54\ub77c\ub2c8\uc548\uc740 \uc77c\ubc18\uc801\uc73c\ub85c \ub9e4\uc6b0 \uce5c\uadfc\ud558\uace0 \uc6d0\uae30\uc655\uc131\ud55c \uac1c\uc774\ub2e4. \uc774 \uac1c\ub294 \uc8fc\uc778\uc758 \uc8fc\ubcc0\uc5d0 \ud568\uaed8 \uc788\ub294 \uac83\uc744 \uc88b\uc544\ud558\uace0, \ubcf4\ud638\ud574\uc8fc\ub294 \uac83\uc73c\ub85c \uc54c\ub824\uc838 \uc788\ub2e4. \uc774 \ud488\uc885\uc740 \uc8fc\uc778\uacfc \uc720\ub300\uac10\uc774 \ube68\ub9ac \ud615\uc131\ub3fc \ud63c\uc790 \uc2dc\uac04\uc744 \ubcf4\ub0bc \uc218 \uc788\ub3c4\ub85d \ud6c8\ub828 \uc2dc\ud0a4\uc9c0 \uc54a\ub294\ub2e4\uba74, \ubd84\ub9ac \ubd88\uc548 \uc5d0 \uc2dc\ub2ec\ub9b4 \uc218 \uc788\ub2e4. \ud3ec\uba54\ub77c\ub2c8\uc548\uc740 \uacbd\uacc4\uc640 \ud658\uacbd\uc758 \ubcc0\ud654\ub97c \uc54c\uc544\ucc44\uace0 \uc0c8\ub85c\uc6b4 \uc790\uadf9\uc5d0 \uc9d6\ub294 \ud589\ub3d9\uc740 \uc5b4\ub5a4 \uc0c1\ud669\uc5d0\uc11c\ub3c4 \uacfc\ub3c4\ud558\uac8c \uc9d6\ub294 \uc2b5\uad00\uc73c\ub85c \uc774\uc5b4 \uc9c8 \uc218 \uc788\ub2e4. \ud3ec\uba54\ub77c\ub2c8\uc548\uc740 \ub611\ub611\ud55c \uac1c\ub85c \ud6c8\ub828 \ubc18\uc751\uc774 \uc88b\uc740\ub370, \uc8fc\uc778\uc5d0\uac8c \uc5b4\ub5bb\uac8c \ud6c8\ub828\uc744 \ubc1b\ub290\ub0d0\uc5d0 \ub530\ub77c \ub9e4\uc6b0 \uc131\uacf5\uc801\uc778 \uc131\uacfc\ub97c \uc5bb\uc744 \uc218 \uc788\ub2e4. \uadf8\ub7ec\ub098 \ud6c8\ub828\uc744 \uc2dc\ud0a4\uc9c0 \uc54a\ub294\ub2e4\uba74, \uc9c0\ubc30\uc801\uc774\uace0 \uacf5\uaca9\uc801\uc73c\ub85c \ub420 \uc218 \uc788\ub2e4. \ub610\ud55c \ud765\ubd84\uc744 \uc798\ud558\uba70 \uae09\ud558\uace0 \ud638\uae30\uc2ec\uc774 \ub9ce\uc544 \ud0c0 \uacac\uc885\uc5d0\uac8c \uad00\uc2ec\uc774 \ub9ce\ub2e4.", "sentence_answer": "\uc774 \ud488\uc885\uc740 \uc8fc\uc778\uacfc \uc720\ub300\uac10\uc774 \ube68\ub9ac \ud615\uc131\ub3fc \ud63c\uc790 \uc2dc\uac04\uc744 \ubcf4\ub0bc \uc218 \uc788\ub3c4\ub85d \ud6c8\ub828 \uc2dc\ud0a4\uc9c0 \uc54a\ub294\ub2e4\uba74, \ubd84\ub9ac \ubd88\uc548 \uc5d0 \uc2dc\ub2ec\ub9b4 \uc218 \uc788\ub2e4.", "paragraph_id": "6567268-0-0", "paragraph_question": "question: \ud3ec\uba54\ub77c\ub2c8\uc548\uc744 \ud63c\uc790\uc788\uc744 \uc218 \uc788\uac8c \ud6c8\ub828\uc2dc\ud0a4\uc9c0 \uc54a\uc73c\uba74 \ubb34\uc5c7\uc5d0 \uc2dc\ub2ec\ub9b4 \uc218 \uc788\ub098?, context: \ud3ec\uba54\ub77c\ub2c8\uc548\uc740 \uc77c\ubc18\uc801\uc73c\ub85c \ub9e4\uc6b0 \uce5c\uadfc\ud558\uace0 \uc6d0\uae30\uc655\uc131\ud55c \uac1c\uc774\ub2e4. \uc774 \uac1c\ub294 \uc8fc\uc778\uc758 \uc8fc\ubcc0\uc5d0 \ud568\uaed8 \uc788\ub294 \uac83\uc744 \uc88b\uc544\ud558\uace0, \ubcf4\ud638\ud574\uc8fc\ub294 \uac83\uc73c\ub85c \uc54c\ub824\uc838 \uc788\ub2e4. \uc774 \ud488\uc885\uc740 \uc8fc\uc778\uacfc \uc720\ub300\uac10\uc774 \ube68\ub9ac \ud615\uc131\ub3fc \ud63c\uc790 \uc2dc\uac04\uc744 \ubcf4\ub0bc \uc218 \uc788\ub3c4\ub85d \ud6c8\ub828 \uc2dc\ud0a4\uc9c0 \uc54a\ub294\ub2e4\uba74, \ubd84\ub9ac \ubd88\uc548\uc5d0 \uc2dc\ub2ec\ub9b4 \uc218 \uc788\ub2e4. \ud3ec\uba54\ub77c\ub2c8\uc548\uc740 \uacbd\uacc4\uc640 \ud658\uacbd\uc758 \ubcc0\ud654\ub97c \uc54c\uc544\ucc44\uace0 \uc0c8\ub85c\uc6b4 \uc790\uadf9\uc5d0 \uc9d6\ub294 \ud589\ub3d9\uc740 \uc5b4\ub5a4 \uc0c1\ud669\uc5d0\uc11c\ub3c4 \uacfc\ub3c4\ud558\uac8c \uc9d6\ub294 \uc2b5\uad00\uc73c\ub85c \uc774\uc5b4 \uc9c8 \uc218 \uc788\ub2e4. \ud3ec\uba54\ub77c\ub2c8\uc548\uc740 \ub611\ub611\ud55c \uac1c\ub85c \ud6c8\ub828 \ubc18\uc751\uc774 \uc88b\uc740\ub370, \uc8fc\uc778\uc5d0\uac8c \uc5b4\ub5bb\uac8c \ud6c8\ub828\uc744 \ubc1b\ub290\ub0d0\uc5d0 \ub530\ub77c \ub9e4\uc6b0 \uc131\uacf5\uc801\uc778 \uc131\uacfc\ub97c \uc5bb\uc744 \uc218 \uc788\ub2e4. \uadf8\ub7ec\ub098 \ud6c8\ub828\uc744 \uc2dc\ud0a4\uc9c0 \uc54a\ub294\ub2e4\uba74, \uc9c0\ubc30\uc801\uc774\uace0 \uacf5\uaca9\uc801\uc73c\ub85c \ub420 \uc218 \uc788\ub2e4. \ub610\ud55c \ud765\ubd84\uc744 \uc798\ud558\uba70 \uae09\ud558\uace0 \ud638\uae30\uc2ec\uc774 \ub9ce\uc544 \ud0c0 \uacac\uc885\uc5d0\uac8c \uad00\uc2ec\uc774 \ub9ce\ub2e4."} {"question": "\ud558\uc9c0\uc6d0\uc774 \ubc1c\ub9ac\uc5d0\uc11c \uc0dd\uae34 \uc77c\ub85c \uc5f0\uae30\ub825\uc744 \uc778\uc815\ubc1b\ub294 \ud574\uc5d0 \uac1c\ubd09\ud55c \ub450 \uc601\ud654\uc758 \uc7a5\ub974\ub294?", "paragraph": "2003\ub144 6\uc6d4, \uac1c\ubd09 \ub41c \uc601\ud654 \u300a\uc5ed\uc804\uc5d0 \uc0b0\ub2e4\u300b\uc5d0\uc11c \ud558\uc9c0\uc6d0\uc740 \uc5b4\ub9b4 \uc801 \uace8\ud504 \uc2e0\ub3d9\uc5d0\uc11c \ud604\uc7ac\uc5d0\ub294 \ubcc4 \ubcfc\uc77c \uc5c6\ub294 \uc778\uc0dd\uc744 \uc0b4\uace0 \uc788\ub294 \uc99d\uad8c\uc0ac \uc601\uc5c5\uc0ac\uc6d0 \uac15\uc2b9\uc644(\uae40\uc2b9\uc6b0 \ubd84)\uc758 \ub2e4\ub978 \uc138\uacc4\uc758 \uc544\ub0b4 \ud55c\uc9c0\uc601 \uc5ed\uc744 \uc5f0\uae30\ud558\uba70, \ub370\ubdd4 \ucc98\uc74c\uc73c\ub85c \uc2f8\uc774\uac00 \uc791\uace1\ud55c \uc601\ud654 \uc0bd\uc785\uace1\uc778 '\ud648\ub7f0'\uc774\ub77c\ub294 \uace1\uc744 \ud1b5\ud574 \uac00\uc218\ub85c \ud65c\ub3d9\ud558\uae30\ub3c4 \ud588\ub2e4. \ud558\uc9c0\uc6d0\uc740 2003\ub144 7\uc6d4\ubd80\ud130 \ubc29\uc601 \ub41c \ud4e8\uc804 \uc0ac\uadf9 \u300a\ub2e4\ubaa8\u300b\uc5d0 \uce90\uc2a4\ud305 \ub418\uc5c8\uace0, \uc774 \uc791\ud488\uc5d0\uc11c \uc88c\ud3ec\uccad \uc18c\uc18d \uc5ec\uc131 \uc218\uc0ac\uad00 \ub2e4\ubaa8 \ucc44\uc625 \uc5ed\uc744 \ub9e1\uc544 \ud669\ubcf4\uc724(\uc774\uc11c\uc9c4 \ubd84)\uacfc \uc560\uc808\ud55c \uba5c\ub85c \uc5f0\uae30\ub97c \ud3bc\uce58\uba70, \uc77c\uba85 \u2018\ub2e4\ubaa8\ud3d0\uc778\u2019\uc774\ub77c\ub294 \ub9d0\uc744 \ub9cc\ub4e4\uc5b4 \ub0bc \uc815\ub3c4\ub85c \ud070 \uc778\uae30\ub97c \ub04c\uc5c8\ub2e4. \uadf9\uc911 \uc774\uc11c\uc9c4\uc774 \"\uc544\ud504\ub0d0, \ub098\ub3c4 \uc544\ud504\ub2e4\"\ub77c\uba70 \ucc44\uc625\uc758 \uc0c1\ucc98\ub97c \ub3cc\ubd10\uc8fc\uace0 \ub300\ub193\uace0 \ub4dc\ub7ec\ub0bc \uc218 \uc5c6\ub294 \ub9c8\uc74c\uc5d0 \ud558\uc9c0\uc6d0\uc774 \uc870\uc6a9\ud788 \ub208\ubb3c\uc744 \ud758\ub9ac\ub294 \uc7a5\uba74\uc740 \uc9c0\uae08\uae4c\uc9c0\ub3c4 \ud68c\uc790\ub418\ub294 \uba85\uc7a5\uba74\uc73c\ub85c \uaf3d\ud78c\ub2e4. \uc5f0\ub9d0 MBC \uc5f0\uae30\ub300\uc0c1\uc5d0\uc11c \ubca0\uc2a4\ud2b8\ucee4\ud50c\uc0c1, \uc5ec\uc790\uc778\uae30\uc0c1\uacfc \uc5ec\uc790 \ucd5c\uc6b0\uc218\uc5f0\uae30\uc0c1\uc744 \uccab \uc218\uc0c1\ud558\uc600\ub2e4. \ud558\uc9c0\uc6d0\uc740 \uc774\ub4ec \ud574, 2004\ub144 \uba5c\ub85c \ub4dc\ub77c\ub9c8 \u300a\ubc1c\ub9ac\uc5d0\uc11c \uc0dd\uae34 \uc77c\u300b\uc5d0\uc11c \ud604\uc2e4\uc744 \ubb34\uc2dc\ud560 \uc218 \uc5c6\uc5b4 \ub450 \ub0a8\uc790 \uc815\uc7ac\ubbfc(\uc870\uc778\uc131 \ubd84), \uac15\uc778\uc6b1(\uc18c\uc9c0\uc12d \ubd84) \uc0ac\uc774\uc5d0\uc11c \uac08\ud321\uc9c8\ud321\ud558\ub294 \ube44\ub828\uc758 \uc5ec\uc8fc\uc778\uacf5 \uc774\uc218\uc815 \uc5ed\uc744 \ub9e1\uc544 \uac10\uc131 \ub0b4\uba74 \uc5f0\uae30\ub85c \ud638\ud3c9\uc744 \ubc1b\uc544 \uc81c40\ud68c \ubc31\uc0c1\uc608\uc220\ub300\uc0c1 TV\ubd80\ubb38 \uc5ec\uc790\ucd5c\uc6b0\uc218\uc5f0\uae30\uc0c1\uacfc SBS \uc5f0\uae30\ub300\uc0c1 \uc5ec\uc790\ucd5c\uc6b0\uc218\uc5f0\uae30\uc0c1\uc744 \uc218\uc0c1\ud558\uba70 \uc5f0\uae30\ub825\uc744 \uc778\uc815 \ubc1b\uc558\ub2e4. \ub610\ud55c \uac19\uc740 \ud574\uc5d0\ub294 \uc8fc\uc5f0\uc744 \ub9e1\uc740 \ub450 \ud3b8\uc758 \ub85c\ub9e8\ud2f1 \ucf54\ubbf8\ub514 \uc601\ud654 \u300a\ub0b4 \uc0ac\ub791 \uc2f8\uac00\uc9c0\u300b\uc640 \u300a\uc2e0\ubd80\uc218\uc5c5\u300b\uc774 \uac1c\ubd09 \ub418\uc5c8\ub2e4.", "answer": "\ub85c\ub9e8\ud2f1 \ucf54\ubbf8\ub514", "sentence": "\ub610\ud55c \uac19\uc740 \ud574\uc5d0\ub294 \uc8fc\uc5f0\uc744 \ub9e1\uc740 \ub450 \ud3b8\uc758 \ub85c\ub9e8\ud2f1 \ucf54\ubbf8\ub514 \uc601\ud654 \u300a\ub0b4 \uc0ac\ub791 \uc2f8\uac00\uc9c0\u300b\uc640 \u300a\uc2e0\ubd80\uc218\uc5c5\u300b\uc774 \uac1c\ubd09 \ub418\uc5c8\ub2e4.", "paragraph_sentence": "2003\ub144 6\uc6d4, \uac1c\ubd09 \ub41c \uc601\ud654 \u300a\uc5ed\uc804\uc5d0 \uc0b0\ub2e4\u300b\uc5d0\uc11c \ud558\uc9c0\uc6d0\uc740 \uc5b4\ub9b4 \uc801 \uace8\ud504 \uc2e0\ub3d9\uc5d0\uc11c \ud604\uc7ac\uc5d0\ub294 \ubcc4 \ubcfc\uc77c \uc5c6\ub294 \uc778\uc0dd\uc744 \uc0b4\uace0 \uc788\ub294 \uc99d\uad8c\uc0ac \uc601\uc5c5\uc0ac\uc6d0 \uac15\uc2b9\uc644(\uae40\uc2b9\uc6b0 \ubd84)\uc758 \ub2e4\ub978 \uc138\uacc4\uc758 \uc544\ub0b4 \ud55c\uc9c0\uc601 \uc5ed\uc744 \uc5f0\uae30\ud558\uba70, \ub370\ubdd4 \ucc98\uc74c\uc73c\ub85c \uc2f8\uc774\uac00 \uc791\uace1\ud55c \uc601\ud654 \uc0bd\uc785\uace1\uc778 '\ud648\ub7f0'\uc774\ub77c\ub294 \uace1\uc744 \ud1b5\ud574 \uac00\uc218\ub85c \ud65c\ub3d9\ud558\uae30\ub3c4 \ud588\ub2e4. \ud558\uc9c0\uc6d0\uc740 2003\ub144 7\uc6d4\ubd80\ud130 \ubc29\uc601 \ub41c \ud4e8\uc804 \uc0ac\uadf9 \u300a\ub2e4\ubaa8\u300b\uc5d0 \uce90\uc2a4\ud305 \ub418\uc5c8\uace0, \uc774 \uc791\ud488\uc5d0\uc11c \uc88c\ud3ec\uccad \uc18c\uc18d \uc5ec\uc131 \uc218\uc0ac\uad00 \ub2e4\ubaa8 \ucc44\uc625 \uc5ed\uc744 \ub9e1\uc544 \ud669\ubcf4\uc724(\uc774\uc11c\uc9c4 \ubd84)\uacfc \uc560\uc808\ud55c \uba5c\ub85c \uc5f0\uae30\ub97c \ud3bc\uce58\uba70, \uc77c\uba85 \u2018\ub2e4\ubaa8\ud3d0\uc778\u2019\uc774\ub77c\ub294 \ub9d0\uc744 \ub9cc\ub4e4\uc5b4 \ub0bc \uc815\ub3c4\ub85c \ud070 \uc778\uae30\ub97c \ub04c\uc5c8\ub2e4. \uadf9\uc911 \uc774\uc11c\uc9c4\uc774 \"\uc544\ud504\ub0d0, \ub098\ub3c4 \uc544\ud504\ub2e4\"\ub77c\uba70 \ucc44\uc625\uc758 \uc0c1\ucc98\ub97c \ub3cc\ubd10\uc8fc\uace0 \ub300\ub193\uace0 \ub4dc\ub7ec\ub0bc \uc218 \uc5c6\ub294 \ub9c8\uc74c\uc5d0 \ud558\uc9c0\uc6d0\uc774 \uc870\uc6a9\ud788 \ub208\ubb3c\uc744 \ud758\ub9ac\ub294 \uc7a5\uba74\uc740 \uc9c0\uae08\uae4c\uc9c0\ub3c4 \ud68c\uc790\ub418\ub294 \uba85\uc7a5\uba74\uc73c\ub85c \uaf3d\ud78c\ub2e4. \uc5f0\ub9d0 MBC \uc5f0\uae30\ub300\uc0c1\uc5d0\uc11c \ubca0\uc2a4\ud2b8\ucee4\ud50c\uc0c1, \uc5ec\uc790\uc778\uae30\uc0c1\uacfc \uc5ec\uc790 \ucd5c\uc6b0\uc218\uc5f0\uae30\uc0c1\uc744 \uccab \uc218\uc0c1\ud558\uc600\ub2e4. \ud558\uc9c0\uc6d0\uc740 \uc774\ub4ec \ud574, 2004\ub144 \uba5c\ub85c \ub4dc\ub77c\ub9c8 \u300a\ubc1c\ub9ac\uc5d0\uc11c \uc0dd\uae34 \uc77c\u300b\uc5d0\uc11c \ud604\uc2e4\uc744 \ubb34\uc2dc\ud560 \uc218 \uc5c6\uc5b4 \ub450 \ub0a8\uc790 \uc815\uc7ac\ubbfc(\uc870\uc778\uc131 \ubd84), \uac15\uc778\uc6b1(\uc18c\uc9c0\uc12d \ubd84) \uc0ac\uc774\uc5d0\uc11c \uac08\ud321\uc9c8\ud321\ud558\ub294 \ube44\ub828\uc758 \uc5ec\uc8fc\uc778\uacf5 \uc774\uc218\uc815 \uc5ed\uc744 \ub9e1\uc544 \uac10\uc131 \ub0b4\uba74 \uc5f0\uae30\ub85c \ud638\ud3c9\uc744 \ubc1b\uc544 \uc81c40\ud68c \ubc31\uc0c1\uc608\uc220\ub300\uc0c1 TV\ubd80\ubb38 \uc5ec\uc790\ucd5c\uc6b0\uc218\uc5f0\uae30\uc0c1\uacfc SBS \uc5f0\uae30\ub300\uc0c1 \uc5ec\uc790\ucd5c\uc6b0\uc218\uc5f0\uae30\uc0c1\uc744 \uc218\uc0c1\ud558\uba70 \uc5f0\uae30\ub825\uc744 \uc778\uc815 \ubc1b\uc558\ub2e4. \ub610\ud55c \uac19\uc740 \ud574\uc5d0\ub294 \uc8fc\uc5f0\uc744 \ub9e1\uc740 \ub450 \ud3b8\uc758 \ub85c\ub9e8\ud2f1 \ucf54\ubbf8\ub514 \uc601\ud654 \u300a\ub0b4 \uc0ac\ub791 \uc2f8\uac00\uc9c0\u300b\uc640 \u300a\uc2e0\ubd80\uc218\uc5c5\u300b\uc774 \uac1c\ubd09 \ub418\uc5c8\ub2e4. ", "paragraph_answer": "2003\ub144 6\uc6d4, \uac1c\ubd09 \ub41c \uc601\ud654 \u300a\uc5ed\uc804\uc5d0 \uc0b0\ub2e4\u300b\uc5d0\uc11c \ud558\uc9c0\uc6d0\uc740 \uc5b4\ub9b4 \uc801 \uace8\ud504 \uc2e0\ub3d9\uc5d0\uc11c \ud604\uc7ac\uc5d0\ub294 \ubcc4 \ubcfc\uc77c \uc5c6\ub294 \uc778\uc0dd\uc744 \uc0b4\uace0 \uc788\ub294 \uc99d\uad8c\uc0ac \uc601\uc5c5\uc0ac\uc6d0 \uac15\uc2b9\uc644(\uae40\uc2b9\uc6b0 \ubd84)\uc758 \ub2e4\ub978 \uc138\uacc4\uc758 \uc544\ub0b4 \ud55c\uc9c0\uc601 \uc5ed\uc744 \uc5f0\uae30\ud558\uba70, \ub370\ubdd4 \ucc98\uc74c\uc73c\ub85c \uc2f8\uc774\uac00 \uc791\uace1\ud55c \uc601\ud654 \uc0bd\uc785\uace1\uc778 '\ud648\ub7f0'\uc774\ub77c\ub294 \uace1\uc744 \ud1b5\ud574 \uac00\uc218\ub85c \ud65c\ub3d9\ud558\uae30\ub3c4 \ud588\ub2e4. \ud558\uc9c0\uc6d0\uc740 2003\ub144 7\uc6d4\ubd80\ud130 \ubc29\uc601 \ub41c \ud4e8\uc804 \uc0ac\uadf9 \u300a\ub2e4\ubaa8\u300b\uc5d0 \uce90\uc2a4\ud305 \ub418\uc5c8\uace0, \uc774 \uc791\ud488\uc5d0\uc11c \uc88c\ud3ec\uccad \uc18c\uc18d \uc5ec\uc131 \uc218\uc0ac\uad00 \ub2e4\ubaa8 \ucc44\uc625 \uc5ed\uc744 \ub9e1\uc544 \ud669\ubcf4\uc724(\uc774\uc11c\uc9c4 \ubd84)\uacfc \uc560\uc808\ud55c \uba5c\ub85c \uc5f0\uae30\ub97c \ud3bc\uce58\uba70, \uc77c\uba85 \u2018\ub2e4\ubaa8\ud3d0\uc778\u2019\uc774\ub77c\ub294 \ub9d0\uc744 \ub9cc\ub4e4\uc5b4 \ub0bc \uc815\ub3c4\ub85c \ud070 \uc778\uae30\ub97c \ub04c\uc5c8\ub2e4. \uadf9\uc911 \uc774\uc11c\uc9c4\uc774 \"\uc544\ud504\ub0d0, \ub098\ub3c4 \uc544\ud504\ub2e4\"\ub77c\uba70 \ucc44\uc625\uc758 \uc0c1\ucc98\ub97c \ub3cc\ubd10\uc8fc\uace0 \ub300\ub193\uace0 \ub4dc\ub7ec\ub0bc \uc218 \uc5c6\ub294 \ub9c8\uc74c\uc5d0 \ud558\uc9c0\uc6d0\uc774 \uc870\uc6a9\ud788 \ub208\ubb3c\uc744 \ud758\ub9ac\ub294 \uc7a5\uba74\uc740 \uc9c0\uae08\uae4c\uc9c0\ub3c4 \ud68c\uc790\ub418\ub294 \uba85\uc7a5\uba74\uc73c\ub85c \uaf3d\ud78c\ub2e4. \uc5f0\ub9d0 MBC \uc5f0\uae30\ub300\uc0c1\uc5d0\uc11c \ubca0\uc2a4\ud2b8\ucee4\ud50c\uc0c1, \uc5ec\uc790\uc778\uae30\uc0c1\uacfc \uc5ec\uc790 \ucd5c\uc6b0\uc218\uc5f0\uae30\uc0c1\uc744 \uccab \uc218\uc0c1\ud558\uc600\ub2e4. \ud558\uc9c0\uc6d0\uc740 \uc774\ub4ec \ud574, 2004\ub144 \uba5c\ub85c \ub4dc\ub77c\ub9c8 \u300a\ubc1c\ub9ac\uc5d0\uc11c \uc0dd\uae34 \uc77c\u300b\uc5d0\uc11c \ud604\uc2e4\uc744 \ubb34\uc2dc\ud560 \uc218 \uc5c6\uc5b4 \ub450 \ub0a8\uc790 \uc815\uc7ac\ubbfc(\uc870\uc778\uc131 \ubd84), \uac15\uc778\uc6b1(\uc18c\uc9c0\uc12d \ubd84) \uc0ac\uc774\uc5d0\uc11c \uac08\ud321\uc9c8\ud321\ud558\ub294 \ube44\ub828\uc758 \uc5ec\uc8fc\uc778\uacf5 \uc774\uc218\uc815 \uc5ed\uc744 \ub9e1\uc544 \uac10\uc131 \ub0b4\uba74 \uc5f0\uae30\ub85c \ud638\ud3c9\uc744 \ubc1b\uc544 \uc81c40\ud68c \ubc31\uc0c1\uc608\uc220\ub300\uc0c1 TV\ubd80\ubb38 \uc5ec\uc790\ucd5c\uc6b0\uc218\uc5f0\uae30\uc0c1\uacfc SBS \uc5f0\uae30\ub300\uc0c1 \uc5ec\uc790\ucd5c\uc6b0\uc218\uc5f0\uae30\uc0c1\uc744 \uc218\uc0c1\ud558\uba70 \uc5f0\uae30\ub825\uc744 \uc778\uc815 \ubc1b\uc558\ub2e4. \ub610\ud55c \uac19\uc740 \ud574\uc5d0\ub294 \uc8fc\uc5f0\uc744 \ub9e1\uc740 \ub450 \ud3b8\uc758 \ub85c\ub9e8\ud2f1 \ucf54\ubbf8\ub514 \uc601\ud654 \u300a\ub0b4 \uc0ac\ub791 \uc2f8\uac00\uc9c0\u300b\uc640 \u300a\uc2e0\ubd80\uc218\uc5c5\u300b\uc774 \uac1c\ubd09 \ub418\uc5c8\ub2e4.", "sentence_answer": "\ub610\ud55c \uac19\uc740 \ud574\uc5d0\ub294 \uc8fc\uc5f0\uc744 \ub9e1\uc740 \ub450 \ud3b8\uc758 \ub85c\ub9e8\ud2f1 \ucf54\ubbf8\ub514 \uc601\ud654 \u300a\ub0b4 \uc0ac\ub791 \uc2f8\uac00\uc9c0\u300b\uc640 \u300a\uc2e0\ubd80\uc218\uc5c5\u300b\uc774 \uac1c\ubd09 \ub418\uc5c8\ub2e4.", "paragraph_id": "5934842-2-1", "paragraph_question": "question: \ud558\uc9c0\uc6d0\uc774 \ubc1c\ub9ac\uc5d0\uc11c \uc0dd\uae34 \uc77c\ub85c \uc5f0\uae30\ub825\uc744 \uc778\uc815\ubc1b\ub294 \ud574\uc5d0 \uac1c\ubd09\ud55c \ub450 \uc601\ud654\uc758 \uc7a5\ub974\ub294?, context: 2003\ub144 6\uc6d4, \uac1c\ubd09 \ub41c \uc601\ud654 \u300a\uc5ed\uc804\uc5d0 \uc0b0\ub2e4\u300b\uc5d0\uc11c \ud558\uc9c0\uc6d0\uc740 \uc5b4\ub9b4 \uc801 \uace8\ud504 \uc2e0\ub3d9\uc5d0\uc11c \ud604\uc7ac\uc5d0\ub294 \ubcc4 \ubcfc\uc77c \uc5c6\ub294 \uc778\uc0dd\uc744 \uc0b4\uace0 \uc788\ub294 \uc99d\uad8c\uc0ac \uc601\uc5c5\uc0ac\uc6d0 \uac15\uc2b9\uc644(\uae40\uc2b9\uc6b0 \ubd84)\uc758 \ub2e4\ub978 \uc138\uacc4\uc758 \uc544\ub0b4 \ud55c\uc9c0\uc601 \uc5ed\uc744 \uc5f0\uae30\ud558\uba70, \ub370\ubdd4 \ucc98\uc74c\uc73c\ub85c \uc2f8\uc774\uac00 \uc791\uace1\ud55c \uc601\ud654 \uc0bd\uc785\uace1\uc778 '\ud648\ub7f0'\uc774\ub77c\ub294 \uace1\uc744 \ud1b5\ud574 \uac00\uc218\ub85c \ud65c\ub3d9\ud558\uae30\ub3c4 \ud588\ub2e4. \ud558\uc9c0\uc6d0\uc740 2003\ub144 7\uc6d4\ubd80\ud130 \ubc29\uc601 \ub41c \ud4e8\uc804 \uc0ac\uadf9 \u300a\ub2e4\ubaa8\u300b\uc5d0 \uce90\uc2a4\ud305 \ub418\uc5c8\uace0, \uc774 \uc791\ud488\uc5d0\uc11c \uc88c\ud3ec\uccad \uc18c\uc18d \uc5ec\uc131 \uc218\uc0ac\uad00 \ub2e4\ubaa8 \ucc44\uc625 \uc5ed\uc744 \ub9e1\uc544 \ud669\ubcf4\uc724(\uc774\uc11c\uc9c4 \ubd84)\uacfc \uc560\uc808\ud55c \uba5c\ub85c \uc5f0\uae30\ub97c \ud3bc\uce58\uba70, \uc77c\uba85 \u2018\ub2e4\ubaa8\ud3d0\uc778\u2019\uc774\ub77c\ub294 \ub9d0\uc744 \ub9cc\ub4e4\uc5b4 \ub0bc \uc815\ub3c4\ub85c \ud070 \uc778\uae30\ub97c \ub04c\uc5c8\ub2e4. \uadf9\uc911 \uc774\uc11c\uc9c4\uc774 \"\uc544\ud504\ub0d0, \ub098\ub3c4 \uc544\ud504\ub2e4\"\ub77c\uba70 \ucc44\uc625\uc758 \uc0c1\ucc98\ub97c \ub3cc\ubd10\uc8fc\uace0 \ub300\ub193\uace0 \ub4dc\ub7ec\ub0bc \uc218 \uc5c6\ub294 \ub9c8\uc74c\uc5d0 \ud558\uc9c0\uc6d0\uc774 \uc870\uc6a9\ud788 \ub208\ubb3c\uc744 \ud758\ub9ac\ub294 \uc7a5\uba74\uc740 \uc9c0\uae08\uae4c\uc9c0\ub3c4 \ud68c\uc790\ub418\ub294 \uba85\uc7a5\uba74\uc73c\ub85c \uaf3d\ud78c\ub2e4. \uc5f0\ub9d0 MBC \uc5f0\uae30\ub300\uc0c1\uc5d0\uc11c \ubca0\uc2a4\ud2b8\ucee4\ud50c\uc0c1, \uc5ec\uc790\uc778\uae30\uc0c1\uacfc \uc5ec\uc790 \ucd5c\uc6b0\uc218\uc5f0\uae30\uc0c1\uc744 \uccab \uc218\uc0c1\ud558\uc600\ub2e4. \ud558\uc9c0\uc6d0\uc740 \uc774\ub4ec \ud574, 2004\ub144 \uba5c\ub85c \ub4dc\ub77c\ub9c8 \u300a\ubc1c\ub9ac\uc5d0\uc11c \uc0dd\uae34 \uc77c\u300b\uc5d0\uc11c \ud604\uc2e4\uc744 \ubb34\uc2dc\ud560 \uc218 \uc5c6\uc5b4 \ub450 \ub0a8\uc790 \uc815\uc7ac\ubbfc(\uc870\uc778\uc131 \ubd84), \uac15\uc778\uc6b1(\uc18c\uc9c0\uc12d \ubd84) \uc0ac\uc774\uc5d0\uc11c \uac08\ud321\uc9c8\ud321\ud558\ub294 \ube44\ub828\uc758 \uc5ec\uc8fc\uc778\uacf5 \uc774\uc218\uc815 \uc5ed\uc744 \ub9e1\uc544 \uac10\uc131 \ub0b4\uba74 \uc5f0\uae30\ub85c \ud638\ud3c9\uc744 \ubc1b\uc544 \uc81c40\ud68c \ubc31\uc0c1\uc608\uc220\ub300\uc0c1 TV\ubd80\ubb38 \uc5ec\uc790\ucd5c\uc6b0\uc218\uc5f0\uae30\uc0c1\uacfc SBS \uc5f0\uae30\ub300\uc0c1 \uc5ec\uc790\ucd5c\uc6b0\uc218\uc5f0\uae30\uc0c1\uc744 \uc218\uc0c1\ud558\uba70 \uc5f0\uae30\ub825\uc744 \uc778\uc815 \ubc1b\uc558\ub2e4. \ub610\ud55c \uac19\uc740 \ud574\uc5d0\ub294 \uc8fc\uc5f0\uc744 \ub9e1\uc740 \ub450 \ud3b8\uc758 \ub85c\ub9e8\ud2f1 \ucf54\ubbf8\ub514 \uc601\ud654 \u300a\ub0b4 \uc0ac\ub791 \uc2f8\uac00\uc9c0\u300b\uc640 \u300a\uc2e0\ubd80\uc218\uc5c5\u300b\uc774 \uac1c\ubd09 \ub418\uc5c8\ub2e4."} {"question": "\ubc29\uc0ac\ud654\ud559\uc5d0 \uad00\uc2ec\uc774 \ub9ce\uc558\ub358 \uc624\ud1a0 \ud55c\uc774 \ub9e5\uae38 \ub300\ud559\uc5d0\uc11c \ubc1c\uacac\ud55c \uac83\uc740?", "paragraph": "\uba3c\uc800 \ud300\uc7a5\uc778 \uc624\ud1a0 \ud55c\uc740 \uc720\uae30\ud654\ud559\uc790 \ucd9c\uc2e0\uc774\uc5c8\uc9c0\ub9cc 1901\ub144 \ubc15\uc0ac\ud559\uc704\ub97c \ub9c8\uce58\uace0 \ubcd1\uc5ed\uc744 \ub9c8\uce5c \ud6c4, \ube44\ud65c\uc131 \uae30\uccb4\uc758 \uc5f0\uad6c\ub85c \uc720\uba85\ud558\ub358 \uc601\uad6d\uc758 \uc70c\ub9ac\uc5c4 \ub7a8\uc9c0 \uc5f0\uad6c\uc2e4\uc5d0\uc11c \uacf5\ubd80\ud558\uac8c \ub418\uc5c8\ub2e4. \ubc29\uc0ac\ub2a5\uc5d0 \uad00\uc2ec\uc788\ub358 \ub7a8\uc9c0\uac00 \uc624\ud1a0 \ud55c\uc5d0\uac8c \ud034\ub9ac\uc758 \ubc29\ubc95\uc744 \uc774\uc6a9\ud558\uc5ec \ubc14\ub968\uc5fc\uc73c\ub85c\ubd80\ud130 \ub77c\ub4d0\uc744 \ucd94\ucd9c\ud558\ub77c\ub294 \ubd80\ud0c1\uc744 \ubc1b\uace0 \uc5f0\uad6c\ub97c \ud558\ub2e4\uac00 \ubc29\uc0ac\ud1a0\ub968\uc744 \ubc1c\uacac\ud558\uac8c \ub418\uc5c8\ub2e4. \uc774\ub97c \uacc4\uae30\ub85c \ud558\uc5ec \ubc29\uc0ac\ud654\ud559\uc73c\ub85c \uae38\uc744 \ubc14\uafbc \uc624\ud1a0 \ud55c\uc740 1905\ub144 9\uc6d4 \ubaac\ud2b8\ub9ac\uc62c\uc758 \ub9e5\uae38 \ub300\ud559\uc5d0 \uc788\ub358 \ub7ec\ub354\ud37c\ub4dc\ub97c \ucc3e\uc544\uac14\ub2e4. \uc774\uacf3\uc5d0\uc11c\ub3c4 \uadf8\ub294 \ubc29\uc0ac\uc545\ud2f0\ub284 \uc744 \ubc1c\uacac\ud574\ub0c8\ub2e4. 1906\ub144 \uac00\uc744 \ubca0\ub97c\ub9b0\uc758 \uc5d0\ubc00 \ud53c\uc154 \uc5f0\uad6c\uc2e4\uc5d0\uc11c \uc77c\ud558\uac8c \ub41c \uadf8\ub294 \uba54\uc870\ud1a0\ub968\uc744 \ubd84\ub9ac\ud574\ub0b4\ub294 \ub4f1 \ubc29\uc0ac\ud654\ud559 \ubd84\uc57c\uc5d0\uc11c \uacc4\uc18d \uc88b\uc740 \uc5f0\uad6c \uc5c5\uc801\uc744 \ub0c8\ub2e4. 1910\ub144\uc5d0 \ubca0\ub97c\ub9b0 \ub300\ud559 \ud654\ud559\uacfc \uad50\uc218\uac00 \ub41c \uadf8\ub294 \uc5ec\ub7ec \ubb3c\ub9ac\ud559\uc790\ub4e4\uacfc \uad50\ub958\ud558\uc600\ub294\ub370 \uadf8 \uc911 \ud55c \uba85\uc774 \ub9ac\uc81c \ub9c8\uc774\ud2b8\ub108\uc600\ub2e4.", "answer": "\ubc29\uc0ac\uc545\ud2f0\ub284", "sentence": "\uc774\uacf3\uc5d0\uc11c\ub3c4 \uadf8\ub294 \ubc29\uc0ac\uc545\ud2f0\ub284 \uc744 \ubc1c\uacac\ud574\ub0c8\ub2e4.", "paragraph_sentence": "\uba3c\uc800 \ud300\uc7a5\uc778 \uc624\ud1a0 \ud55c\uc740 \uc720\uae30\ud654\ud559\uc790 \ucd9c\uc2e0\uc774\uc5c8\uc9c0\ub9cc 1901\ub144 \ubc15\uc0ac\ud559\uc704\ub97c \ub9c8\uce58\uace0 \ubcd1\uc5ed\uc744 \ub9c8\uce5c \ud6c4, \ube44\ud65c\uc131 \uae30\uccb4\uc758 \uc5f0\uad6c\ub85c \uc720\uba85\ud558\ub358 \uc601\uad6d\uc758 \uc70c\ub9ac\uc5c4 \ub7a8\uc9c0 \uc5f0\uad6c\uc2e4\uc5d0\uc11c \uacf5\ubd80\ud558\uac8c \ub418\uc5c8\ub2e4. \ubc29\uc0ac\ub2a5\uc5d0 \uad00\uc2ec\uc788\ub358 \ub7a8\uc9c0\uac00 \uc624\ud1a0 \ud55c\uc5d0\uac8c \ud034\ub9ac\uc758 \ubc29\ubc95\uc744 \uc774\uc6a9\ud558\uc5ec \ubc14\ub968\uc5fc\uc73c\ub85c\ubd80\ud130 \ub77c\ub4d0\uc744 \ucd94\ucd9c\ud558\ub77c\ub294 \ubd80\ud0c1\uc744 \ubc1b\uace0 \uc5f0\uad6c\ub97c \ud558\ub2e4\uac00 \ubc29\uc0ac\ud1a0\ub968\uc744 \ubc1c\uacac\ud558\uac8c \ub418\uc5c8\ub2e4. \uc774\ub97c \uacc4\uae30\ub85c \ud558\uc5ec \ubc29\uc0ac\ud654\ud559\uc73c\ub85c \uae38\uc744 \ubc14\uafbc \uc624\ud1a0 \ud55c\uc740 1905\ub144 9\uc6d4 \ubaac\ud2b8\ub9ac\uc62c\uc758 \ub9e5\uae38 \ub300\ud559\uc5d0 \uc788\ub358 \ub7ec\ub354\ud37c\ub4dc\ub97c \ucc3e\uc544\uac14\ub2e4. \uc774\uacf3\uc5d0\uc11c\ub3c4 \uadf8\ub294 \ubc29\uc0ac\uc545\ud2f0\ub284 \uc744 \ubc1c\uacac\ud574\ub0c8\ub2e4. 1906\ub144 \uac00\uc744 \ubca0\ub97c\ub9b0\uc758 \uc5d0\ubc00 \ud53c\uc154 \uc5f0\uad6c\uc2e4\uc5d0\uc11c \uc77c\ud558\uac8c \ub41c \uadf8\ub294 \uba54\uc870\ud1a0\ub968\uc744 \ubd84\ub9ac\ud574\ub0b4\ub294 \ub4f1 \ubc29\uc0ac\ud654\ud559 \ubd84\uc57c\uc5d0\uc11c \uacc4\uc18d \uc88b\uc740 \uc5f0\uad6c \uc5c5\uc801\uc744 \ub0c8\ub2e4. 1910\ub144\uc5d0 \ubca0\ub97c\ub9b0 \ub300\ud559 \ud654\ud559\uacfc \uad50\uc218\uac00 \ub41c \uadf8\ub294 \uc5ec\ub7ec \ubb3c\ub9ac\ud559\uc790\ub4e4\uacfc \uad50\ub958\ud558\uc600\ub294\ub370 \uadf8 \uc911 \ud55c \uba85\uc774 \ub9ac\uc81c \ub9c8\uc774\ud2b8\ub108\uc600\ub2e4.", "paragraph_answer": "\uba3c\uc800 \ud300\uc7a5\uc778 \uc624\ud1a0 \ud55c\uc740 \uc720\uae30\ud654\ud559\uc790 \ucd9c\uc2e0\uc774\uc5c8\uc9c0\ub9cc 1901\ub144 \ubc15\uc0ac\ud559\uc704\ub97c \ub9c8\uce58\uace0 \ubcd1\uc5ed\uc744 \ub9c8\uce5c \ud6c4, \ube44\ud65c\uc131 \uae30\uccb4\uc758 \uc5f0\uad6c\ub85c \uc720\uba85\ud558\ub358 \uc601\uad6d\uc758 \uc70c\ub9ac\uc5c4 \ub7a8\uc9c0 \uc5f0\uad6c\uc2e4\uc5d0\uc11c \uacf5\ubd80\ud558\uac8c \ub418\uc5c8\ub2e4. \ubc29\uc0ac\ub2a5\uc5d0 \uad00\uc2ec\uc788\ub358 \ub7a8\uc9c0\uac00 \uc624\ud1a0 \ud55c\uc5d0\uac8c \ud034\ub9ac\uc758 \ubc29\ubc95\uc744 \uc774\uc6a9\ud558\uc5ec \ubc14\ub968\uc5fc\uc73c\ub85c\ubd80\ud130 \ub77c\ub4d0\uc744 \ucd94\ucd9c\ud558\ub77c\ub294 \ubd80\ud0c1\uc744 \ubc1b\uace0 \uc5f0\uad6c\ub97c \ud558\ub2e4\uac00 \ubc29\uc0ac\ud1a0\ub968\uc744 \ubc1c\uacac\ud558\uac8c \ub418\uc5c8\ub2e4. \uc774\ub97c \uacc4\uae30\ub85c \ud558\uc5ec \ubc29\uc0ac\ud654\ud559\uc73c\ub85c \uae38\uc744 \ubc14\uafbc \uc624\ud1a0 \ud55c\uc740 1905\ub144 9\uc6d4 \ubaac\ud2b8\ub9ac\uc62c\uc758 \ub9e5\uae38 \ub300\ud559\uc5d0 \uc788\ub358 \ub7ec\ub354\ud37c\ub4dc\ub97c \ucc3e\uc544\uac14\ub2e4. \uc774\uacf3\uc5d0\uc11c\ub3c4 \uadf8\ub294 \ubc29\uc0ac\uc545\ud2f0\ub284 \uc744 \ubc1c\uacac\ud574\ub0c8\ub2e4. 1906\ub144 \uac00\uc744 \ubca0\ub97c\ub9b0\uc758 \uc5d0\ubc00 \ud53c\uc154 \uc5f0\uad6c\uc2e4\uc5d0\uc11c \uc77c\ud558\uac8c \ub41c \uadf8\ub294 \uba54\uc870\ud1a0\ub968\uc744 \ubd84\ub9ac\ud574\ub0b4\ub294 \ub4f1 \ubc29\uc0ac\ud654\ud559 \ubd84\uc57c\uc5d0\uc11c \uacc4\uc18d \uc88b\uc740 \uc5f0\uad6c \uc5c5\uc801\uc744 \ub0c8\ub2e4. 1910\ub144\uc5d0 \ubca0\ub97c\ub9b0 \ub300\ud559 \ud654\ud559\uacfc \uad50\uc218\uac00 \ub41c \uadf8\ub294 \uc5ec\ub7ec \ubb3c\ub9ac\ud559\uc790\ub4e4\uacfc \uad50\ub958\ud558\uc600\ub294\ub370 \uadf8 \uc911 \ud55c \uba85\uc774 \ub9ac\uc81c \ub9c8\uc774\ud2b8\ub108\uc600\ub2e4.", "sentence_answer": "\uc774\uacf3\uc5d0\uc11c\ub3c4 \uadf8\ub294 \ubc29\uc0ac\uc545\ud2f0\ub284 \uc744 \ubc1c\uacac\ud574\ub0c8\ub2e4.", "paragraph_id": "6589809-2-1", "paragraph_question": "question: \ubc29\uc0ac\ud654\ud559\uc5d0 \uad00\uc2ec\uc774 \ub9ce\uc558\ub358 \uc624\ud1a0 \ud55c\uc774 \ub9e5\uae38 \ub300\ud559\uc5d0\uc11c \ubc1c\uacac\ud55c \uac83\uc740?, context: \uba3c\uc800 \ud300\uc7a5\uc778 \uc624\ud1a0 \ud55c\uc740 \uc720\uae30\ud654\ud559\uc790 \ucd9c\uc2e0\uc774\uc5c8\uc9c0\ub9cc 1901\ub144 \ubc15\uc0ac\ud559\uc704\ub97c \ub9c8\uce58\uace0 \ubcd1\uc5ed\uc744 \ub9c8\uce5c \ud6c4, \ube44\ud65c\uc131 \uae30\uccb4\uc758 \uc5f0\uad6c\ub85c \uc720\uba85\ud558\ub358 \uc601\uad6d\uc758 \uc70c\ub9ac\uc5c4 \ub7a8\uc9c0 \uc5f0\uad6c\uc2e4\uc5d0\uc11c \uacf5\ubd80\ud558\uac8c \ub418\uc5c8\ub2e4. \ubc29\uc0ac\ub2a5\uc5d0 \uad00\uc2ec\uc788\ub358 \ub7a8\uc9c0\uac00 \uc624\ud1a0 \ud55c\uc5d0\uac8c \ud034\ub9ac\uc758 \ubc29\ubc95\uc744 \uc774\uc6a9\ud558\uc5ec \ubc14\ub968\uc5fc\uc73c\ub85c\ubd80\ud130 \ub77c\ub4d0\uc744 \ucd94\ucd9c\ud558\ub77c\ub294 \ubd80\ud0c1\uc744 \ubc1b\uace0 \uc5f0\uad6c\ub97c \ud558\ub2e4\uac00 \ubc29\uc0ac\ud1a0\ub968\uc744 \ubc1c\uacac\ud558\uac8c \ub418\uc5c8\ub2e4. \uc774\ub97c \uacc4\uae30\ub85c \ud558\uc5ec \ubc29\uc0ac\ud654\ud559\uc73c\ub85c \uae38\uc744 \ubc14\uafbc \uc624\ud1a0 \ud55c\uc740 1905\ub144 9\uc6d4 \ubaac\ud2b8\ub9ac\uc62c\uc758 \ub9e5\uae38 \ub300\ud559\uc5d0 \uc788\ub358 \ub7ec\ub354\ud37c\ub4dc\ub97c \ucc3e\uc544\uac14\ub2e4. \uc774\uacf3\uc5d0\uc11c\ub3c4 \uadf8\ub294 \ubc29\uc0ac\uc545\ud2f0\ub284 \uc744 \ubc1c\uacac\ud574\ub0c8\ub2e4. 1906\ub144 \uac00\uc744 \ubca0\ub97c\ub9b0\uc758 \uc5d0\ubc00 \ud53c\uc154 \uc5f0\uad6c\uc2e4\uc5d0\uc11c \uc77c\ud558\uac8c \ub41c \uadf8\ub294 \uba54\uc870\ud1a0\ub968\uc744 \ubd84\ub9ac\ud574\ub0b4\ub294 \ub4f1 \ubc29\uc0ac\ud654\ud559 \ubd84\uc57c\uc5d0\uc11c \uacc4\uc18d \uc88b\uc740 \uc5f0\uad6c \uc5c5\uc801\uc744 \ub0c8\ub2e4. 1910\ub144\uc5d0 \ubca0\ub97c\ub9b0 \ub300\ud559 \ud654\ud559\uacfc \uad50\uc218\uac00 \ub41c \uadf8\ub294 \uc5ec\ub7ec \ubb3c\ub9ac\ud559\uc790\ub4e4\uacfc \uad50\ub958\ud558\uc600\ub294\ub370 \uadf8 \uc911 \ud55c \uba85\uc774 \ub9ac\uc81c \ub9c8\uc774\ud2b8\ub108\uc600\ub2e4."} {"question": "\uc624\uc2a4\ud2b8\ub9ac\uc544-\uc2e0\uc131\ub85c\ub9c8\uc81c\uad6d \uc5f0\ud569\uad70\uc774 \ubb3c\ub7ec\ub09c \uacf3\uc740?", "paragraph": "\uc624\uc2a4\ud2b8\ub9ac\uc544-\uc2e0\uc131\ub85c\ub9c8\uc81c\uad6d \uc5f0\ud569\uad70\uc740 \ud53c\ub974\ub098(Pirna)\ub85c \ubb3c\ub7ec\ub0ac\uace0, \ud504\ub85c\uc774\uc13c\uad70\uc740 \ucc98\uc74c\uc5d0\ub294 \uc870\uc2ec\uc2a4\ub7fd\uac8c \ub4a4\ub97c \ub530\ub974\ub2e4\uac00 \uc5bc\ub9c8 \uc548 \uc788\uc5b4 \uc880 \ub354 \ub300\ub2f4\ud558\uac8c \ucd94\uaca9\uc744 \ud558\uae30 \uc2dc\uc791\ud588\ub2e4. \ud074\ub77c\uc774\uc2a4\ud2b8\ub294 11\uc6d4\uc758 \uccab \uc8fc \ub3d9\uc548 \uc791\uc13c\uc5d0 \uc788\ub294 \uc624\uc2a4\ud2b8\ub9ac\uc544\uad70\uc758 \ubcf4\uae09 \ucc3d\uace0\ub97c \uacf5\uaca9\ud558\uae30 \uc704\ud574 \ud30c\uacac\ub418\uc5c8\uace0, \uc791\uc804\uc744 \uc131\uacf5\uc2dc\ucf30\ub2e4. \uadf8\ub294 \ub610 \ud558\uc778\ub9ac\ud788 \uacf5\uc758 \uba85\ub839\uc5d0 \ub530\ub77c \ud669\uc81c\ud30c\ub97c \uaca9\ud30c\ud558\uae30 \uc704\ud574 \uc624\ub79c \uae30\uac04 \ud589\uad70\ud558\uc5ec \ud504\ub791\ucf54\ub2c8\uc544(Franconia)\uc5d0 \uc9c4\uc785\ud558\uc600\ub2e4. \ub098\uc6c0\ubca0\ub974\ud06c(Naumberg), \ubdd4\ub974\uce20\ubd80\ub974\ud06c(W\u00fcrzburg), \ub808\uac90\uc2a4\ubd80\ub974\ud06c(Regensburg)\ub294 \ud504\ub85c\uc774\uc13c\uad70\uc5d0\uac8c \uc810\ub839\ub2f9\ud588\uace0, \uc810\ub839\uad70\uc744 \uc9c0\ud718\ud558\ub294 \ud558\uc778\ub9ac\ud788 \uacf5\uc73c\ub85c\ubd80\ud130 \uae30\ubd80\uae08\uc744 \ucc28\ucd9c\ub2f9\ud588\ub2e4. \uc2a4\ud1a8\ubca0\ub974\ud06c\ub294 \ud558\ub514\ud06c(Hadik)\uc640 \ud568\uaed8 \uadc0\ud658\ud558\uc5ec \uace0\ud5a5\uc744 \ubc29\uc5b4\ud558\uae30 \uc704\ud55c \ubb34\uc758\ubbf8\ud55c \ub178\ub825\uc744 \ud558\uace0 \uc788\uc5c8\ub2e4. \uadf8\ub7ec\ub098 \ud558\ub514\ud06c\ub294 \uc2a4\ud1a8\ubca0\ub974\ud06c \ubab0\ub798 \ud504\ub9ac\ub4dc\ub9ac\ud788 \ub300\uc655\uc5d0\uac8c \ubc00\uc0ac\ub97c \ubcf4\ub0c8\uace0, \uc774\ub4e4\uc740 \uc81c\uad6d\uad70\uc758 \ud574\uc0b0\uc744 \uace8\uc790\ub85c \ud558\ub294 \ud611\uc0c1\uc744 \uccb4\uacb0\ud558\uc600\ub2e4.", "answer": "\ud53c\ub974\ub098", "sentence": "\uc624\uc2a4\ud2b8\ub9ac\uc544-\uc2e0\uc131\ub85c\ub9c8\uc81c\uad6d \uc5f0\ud569\uad70\uc740 \ud53c\ub974\ub098 (Pirna)\ub85c \ubb3c\ub7ec\ub0ac\uace0, \ud504\ub85c\uc774\uc13c\uad70\uc740 \ucc98\uc74c\uc5d0\ub294 \uc870\uc2ec\uc2a4\ub7fd\uac8c \ub4a4\ub97c \ub530\ub974\ub2e4\uac00 \uc5bc\ub9c8 \uc548 \uc788\uc5b4 \uc880 \ub354 \ub300\ub2f4\ud558\uac8c \ucd94\uaca9\uc744 \ud558\uae30 \uc2dc\uc791\ud588\ub2e4.", "paragraph_sentence": " \uc624\uc2a4\ud2b8\ub9ac\uc544-\uc2e0\uc131\ub85c\ub9c8\uc81c\uad6d \uc5f0\ud569\uad70\uc740 \ud53c\ub974\ub098 (Pirna)\ub85c \ubb3c\ub7ec\ub0ac\uace0, \ud504\ub85c\uc774\uc13c\uad70\uc740 \ucc98\uc74c\uc5d0\ub294 \uc870\uc2ec\uc2a4\ub7fd\uac8c \ub4a4\ub97c \ub530\ub974\ub2e4\uac00 \uc5bc\ub9c8 \uc548 \uc788\uc5b4 \uc880 \ub354 \ub300\ub2f4\ud558\uac8c \ucd94\uaca9\uc744 \ud558\uae30 \uc2dc\uc791\ud588\ub2e4. \ud074\ub77c\uc774\uc2a4\ud2b8\ub294 11\uc6d4\uc758 \uccab \uc8fc \ub3d9\uc548 \uc791\uc13c\uc5d0 \uc788\ub294 \uc624\uc2a4\ud2b8\ub9ac\uc544\uad70\uc758 \ubcf4\uae09 \ucc3d\uace0\ub97c \uacf5\uaca9\ud558\uae30 \uc704\ud574 \ud30c\uacac\ub418\uc5c8\uace0, \uc791\uc804\uc744 \uc131\uacf5\uc2dc\ucf30\ub2e4. \uadf8\ub294 \ub610 \ud558\uc778\ub9ac\ud788 \uacf5\uc758 \uba85\ub839\uc5d0 \ub530\ub77c \ud669\uc81c\ud30c\ub97c \uaca9\ud30c\ud558\uae30 \uc704\ud574 \uc624\ub79c \uae30\uac04 \ud589\uad70\ud558\uc5ec \ud504\ub791\ucf54\ub2c8\uc544(Franconia)\uc5d0 \uc9c4\uc785\ud558\uc600\ub2e4. \ub098\uc6c0\ubca0\ub974\ud06c(Naumberg), \ubdd4\ub974\uce20\ubd80\ub974\ud06c(W\u00fcrzburg), \ub808\uac90\uc2a4\ubd80\ub974\ud06c(Regensburg)\ub294 \ud504\ub85c\uc774\uc13c\uad70\uc5d0\uac8c \uc810\ub839\ub2f9\ud588\uace0, \uc810\ub839\uad70\uc744 \uc9c0\ud718\ud558\ub294 \ud558\uc778\ub9ac\ud788 \uacf5\uc73c\ub85c\ubd80\ud130 \uae30\ubd80\uae08\uc744 \ucc28\ucd9c\ub2f9\ud588\ub2e4. \uc2a4\ud1a8\ubca0\ub974\ud06c\ub294 \ud558\ub514\ud06c(Hadik)\uc640 \ud568\uaed8 \uadc0\ud658\ud558\uc5ec \uace0\ud5a5\uc744 \ubc29\uc5b4\ud558\uae30 \uc704\ud55c \ubb34\uc758\ubbf8\ud55c \ub178\ub825\uc744 \ud558\uace0 \uc788\uc5c8\ub2e4. \uadf8\ub7ec\ub098 \ud558\ub514\ud06c\ub294 \uc2a4\ud1a8\ubca0\ub974\ud06c \ubab0\ub798 \ud504\ub9ac\ub4dc\ub9ac\ud788 \ub300\uc655\uc5d0\uac8c \ubc00\uc0ac\ub97c \ubcf4\ub0c8\uace0, \uc774\ub4e4\uc740 \uc81c\uad6d\uad70\uc758 \ud574\uc0b0\uc744 \uace8\uc790\ub85c \ud558\ub294 \ud611\uc0c1\uc744 \uccb4\uacb0\ud558\uc600\ub2e4.", "paragraph_answer": "\uc624\uc2a4\ud2b8\ub9ac\uc544-\uc2e0\uc131\ub85c\ub9c8\uc81c\uad6d \uc5f0\ud569\uad70\uc740 \ud53c\ub974\ub098 (Pirna)\ub85c \ubb3c\ub7ec\ub0ac\uace0, \ud504\ub85c\uc774\uc13c\uad70\uc740 \ucc98\uc74c\uc5d0\ub294 \uc870\uc2ec\uc2a4\ub7fd\uac8c \ub4a4\ub97c \ub530\ub974\ub2e4\uac00 \uc5bc\ub9c8 \uc548 \uc788\uc5b4 \uc880 \ub354 \ub300\ub2f4\ud558\uac8c \ucd94\uaca9\uc744 \ud558\uae30 \uc2dc\uc791\ud588\ub2e4. \ud074\ub77c\uc774\uc2a4\ud2b8\ub294 11\uc6d4\uc758 \uccab \uc8fc \ub3d9\uc548 \uc791\uc13c\uc5d0 \uc788\ub294 \uc624\uc2a4\ud2b8\ub9ac\uc544\uad70\uc758 \ubcf4\uae09 \ucc3d\uace0\ub97c \uacf5\uaca9\ud558\uae30 \uc704\ud574 \ud30c\uacac\ub418\uc5c8\uace0, \uc791\uc804\uc744 \uc131\uacf5\uc2dc\ucf30\ub2e4. \uadf8\ub294 \ub610 \ud558\uc778\ub9ac\ud788 \uacf5\uc758 \uba85\ub839\uc5d0 \ub530\ub77c \ud669\uc81c\ud30c\ub97c \uaca9\ud30c\ud558\uae30 \uc704\ud574 \uc624\ub79c \uae30\uac04 \ud589\uad70\ud558\uc5ec \ud504\ub791\ucf54\ub2c8\uc544(Franconia)\uc5d0 \uc9c4\uc785\ud558\uc600\ub2e4. \ub098\uc6c0\ubca0\ub974\ud06c(Naumberg), \ubdd4\ub974\uce20\ubd80\ub974\ud06c(W\u00fcrzburg), \ub808\uac90\uc2a4\ubd80\ub974\ud06c(Regensburg)\ub294 \ud504\ub85c\uc774\uc13c\uad70\uc5d0\uac8c \uc810\ub839\ub2f9\ud588\uace0, \uc810\ub839\uad70\uc744 \uc9c0\ud718\ud558\ub294 \ud558\uc778\ub9ac\ud788 \uacf5\uc73c\ub85c\ubd80\ud130 \uae30\ubd80\uae08\uc744 \ucc28\ucd9c\ub2f9\ud588\ub2e4. \uc2a4\ud1a8\ubca0\ub974\ud06c\ub294 \ud558\ub514\ud06c(Hadik)\uc640 \ud568\uaed8 \uadc0\ud658\ud558\uc5ec \uace0\ud5a5\uc744 \ubc29\uc5b4\ud558\uae30 \uc704\ud55c \ubb34\uc758\ubbf8\ud55c \ub178\ub825\uc744 \ud558\uace0 \uc788\uc5c8\ub2e4. \uadf8\ub7ec\ub098 \ud558\ub514\ud06c\ub294 \uc2a4\ud1a8\ubca0\ub974\ud06c \ubab0\ub798 \ud504\ub9ac\ub4dc\ub9ac\ud788 \ub300\uc655\uc5d0\uac8c \ubc00\uc0ac\ub97c \ubcf4\ub0c8\uace0, \uc774\ub4e4\uc740 \uc81c\uad6d\uad70\uc758 \ud574\uc0b0\uc744 \uace8\uc790\ub85c \ud558\ub294 \ud611\uc0c1\uc744 \uccb4\uacb0\ud558\uc600\ub2e4.", "sentence_answer": "\uc624\uc2a4\ud2b8\ub9ac\uc544-\uc2e0\uc131\ub85c\ub9c8\uc81c\uad6d \uc5f0\ud569\uad70\uc740 \ud53c\ub974\ub098 (Pirna)\ub85c \ubb3c\ub7ec\ub0ac\uace0, \ud504\ub85c\uc774\uc13c\uad70\uc740 \ucc98\uc74c\uc5d0\ub294 \uc870\uc2ec\uc2a4\ub7fd\uac8c \ub4a4\ub97c \ub530\ub974\ub2e4\uac00 \uc5bc\ub9c8 \uc548 \uc788\uc5b4 \uc880 \ub354 \ub300\ub2f4\ud558\uac8c \ucd94\uaca9\uc744 \ud558\uae30 \uc2dc\uc791\ud588\ub2e4.", "paragraph_id": "6568515-4-0", "paragraph_question": "question: \uc624\uc2a4\ud2b8\ub9ac\uc544-\uc2e0\uc131\ub85c\ub9c8\uc81c\uad6d \uc5f0\ud569\uad70\uc774 \ubb3c\ub7ec\ub09c \uacf3\uc740?, context: \uc624\uc2a4\ud2b8\ub9ac\uc544-\uc2e0\uc131\ub85c\ub9c8\uc81c\uad6d \uc5f0\ud569\uad70\uc740 \ud53c\ub974\ub098(Pirna)\ub85c \ubb3c\ub7ec\ub0ac\uace0, \ud504\ub85c\uc774\uc13c\uad70\uc740 \ucc98\uc74c\uc5d0\ub294 \uc870\uc2ec\uc2a4\ub7fd\uac8c \ub4a4\ub97c \ub530\ub974\ub2e4\uac00 \uc5bc\ub9c8 \uc548 \uc788\uc5b4 \uc880 \ub354 \ub300\ub2f4\ud558\uac8c \ucd94\uaca9\uc744 \ud558\uae30 \uc2dc\uc791\ud588\ub2e4. \ud074\ub77c\uc774\uc2a4\ud2b8\ub294 11\uc6d4\uc758 \uccab \uc8fc \ub3d9\uc548 \uc791\uc13c\uc5d0 \uc788\ub294 \uc624\uc2a4\ud2b8\ub9ac\uc544\uad70\uc758 \ubcf4\uae09 \ucc3d\uace0\ub97c \uacf5\uaca9\ud558\uae30 \uc704\ud574 \ud30c\uacac\ub418\uc5c8\uace0, \uc791\uc804\uc744 \uc131\uacf5\uc2dc\ucf30\ub2e4. \uadf8\ub294 \ub610 \ud558\uc778\ub9ac\ud788 \uacf5\uc758 \uba85\ub839\uc5d0 \ub530\ub77c \ud669\uc81c\ud30c\ub97c \uaca9\ud30c\ud558\uae30 \uc704\ud574 \uc624\ub79c \uae30\uac04 \ud589\uad70\ud558\uc5ec \ud504\ub791\ucf54\ub2c8\uc544(Franconia)\uc5d0 \uc9c4\uc785\ud558\uc600\ub2e4. \ub098\uc6c0\ubca0\ub974\ud06c(Naumberg), \ubdd4\ub974\uce20\ubd80\ub974\ud06c(W\u00fcrzburg), \ub808\uac90\uc2a4\ubd80\ub974\ud06c(Regensburg)\ub294 \ud504\ub85c\uc774\uc13c\uad70\uc5d0\uac8c \uc810\ub839\ub2f9\ud588\uace0, \uc810\ub839\uad70\uc744 \uc9c0\ud718\ud558\ub294 \ud558\uc778\ub9ac\ud788 \uacf5\uc73c\ub85c\ubd80\ud130 \uae30\ubd80\uae08\uc744 \ucc28\ucd9c\ub2f9\ud588\ub2e4. \uc2a4\ud1a8\ubca0\ub974\ud06c\ub294 \ud558\ub514\ud06c(Hadik)\uc640 \ud568\uaed8 \uadc0\ud658\ud558\uc5ec \uace0\ud5a5\uc744 \ubc29\uc5b4\ud558\uae30 \uc704\ud55c \ubb34\uc758\ubbf8\ud55c \ub178\ub825\uc744 \ud558\uace0 \uc788\uc5c8\ub2e4. \uadf8\ub7ec\ub098 \ud558\ub514\ud06c\ub294 \uc2a4\ud1a8\ubca0\ub974\ud06c \ubab0\ub798 \ud504\ub9ac\ub4dc\ub9ac\ud788 \ub300\uc655\uc5d0\uac8c \ubc00\uc0ac\ub97c \ubcf4\ub0c8\uace0, \uc774\ub4e4\uc740 \uc81c\uad6d\uad70\uc758 \ud574\uc0b0\uc744 \uace8\uc790\ub85c \ud558\ub294 \ud611\uc0c1\uc744 \uccb4\uacb0\ud558\uc600\ub2e4."} {"question": "\uc2e0\uacf5\ud56d \uc0ac\uc5c5\uc774 \uc644\uc804\ud788 \ud3d0\uae30\ub41c \uac00\uc7a5 \ud070 \uc774\uc720\ub294 \ubb34\uc5c7\uc774 \uc5c6\ub2e4\ub294 \ud310\ub2e8 \ub54c\ubb38\uc774\uc5c8\ub098?", "paragraph": "2011\ub144 3\uc6d4 30\uc77c \uad6d\ud1a0\ud574\uc591\ubd80\ub294 \"\ub450 \ud6c4\ubcf4\uc9c0 \ubaa8\ub450 \ubd88\ub9ac\ud55c \uc9c0\ud615\uc870\uac74\uc73c\ub85c \uc778\ud55c \ud658\uacbd\ubb38\uc81c, \uc0ac\uc5c5\ube44 \uacfc\ub2e4, \uacbd\uc81c\uc131 \ubbf8\ud761 \ub4f1\uc73c\ub85c \ud604 \uc2dc\uc810\uc5d0\uc11c \uc0ac\uc5c5 \ucd94\uc9c4 \uc5ec\uac74\uc774 \uc801\ud569\uce58 \uc54a\ub2e4\ub294 \uacb0\ub860\uc5d0 \ub3c4\ucd9c\ud588\ub2e4\"\uba74\uc11c \ubc31\uc9c0\ud654\ub97c \ubc1c\ud45c\ud588\ub2e4. 2003\ub144\ubd80\ud130 \ucd94\uc9c4\ub41c \ub3d9\ub0a8\uad8c \uc2e0\uacf5\ud56d \uc0ac\uc5c5\uc774 \uc644\uc804\ud788 \ud3d0\uae30\ub41c \uac83\uc774\ub2e4. \uacbd\uc81c\uc801 \ud0c0\ub2f9\uc131\uc774 \uc5c6\ub2e4\ub294 \uac83\uc774 \uac00\uc7a5 \ud070 \uc774\uc720\uc600\ub2e4. \uadf8\ub7ec\ub098 \uacbd\uc81c\uc131\ub9cc\uc744 \uac80\ud1a0\ud55c \ubc31\uc9c0\ud654\ub294 \uc798\ubabb\ub410\ub2e4\ub294 \uc8fc\uc7a5\ub3c4 \uc788\ub2e4. \ubd80\uc0b0 \uc218\uc601\uad6c\uac00 \uc9c0\uc5ed\uad6c\uc778 \ud55c\ub098\ub77c\ub2f9 \uc720\uc7ac\uc911 \uc758\uc6d0\uc740 \"B/C\ub9cc \ub530\uc838\uc11c\ub294 \uc548\ub41c\ub2e4. \uc6d0\ub798 1\uc774\uc0c1 \ub098\uc62c\uc218\uac00 \uc5c6\ub2e4. \ud604\uc7ac \ud638\ub0a8\uace0\uc18d\ub3c4\ub85c, \uc11c\ud574\uc548\uace0\uc18d\ub3c4\ub85c\ub97c \ubcf4\uba74 B/C(\ube44\uc6a9 \ub300\ube44 \ud3b8\uc775)\uac00 0.39\uc774\uba70, \uace0\uc18d\ucca0\ub3c4 B/C\uac00 0.39\ubc16\uc5d0 \ub098\uc624\uc9c0 \uc54a\uc558\uc9c0\ub9cc \uc0ac\uc5c5\uc744 \ucd94\uc9c4\ud588\ub2e4.\"\ub77c\uba74\uc11c \"\uc120\ub3c4 \ud504\ub85c\uc81d\ud2b8 \uc0ac\uc5c5\uc740 \uade0\ud615\ubc1c\uc804\uc744 \ud558\uaca0\ub2e4\ub294 \uc0ac\uc5c5\uc774\ub2e4. \uadf8\uac83\uc744 \uacbd\uc81c\uc131\uc774 \uc5c6\ub2e4\uace0 \uc548\ud558\uba74 \uc9c0\ubc29\uc740 \ud560 \uc0ac\uc5c5\uc774 \uc5c6\ub294\uac83\uc774\ub2e4\"\ub77c\uace0 \ubaa9\uc18c\ub9ac\ub97c \ub192\uc600\ub2e4. \uc815\ubd80\uac00 \ub108\ubb34 \ub9ce\uc774 \uc2dc\uac04\uc744 \ub04c\uc5c8\ub358\uac83\uc5d0 \ub300\ud55c \ube44\ud310\ub3c4 \uc788\ub2e4. \uac15\uc8fc\uc5f4 \uc2e0\uacf5\ud56d\ubc00\uc591\uc720\uce58 \uacb0\uc0ac\ucd94\uc9c4\uc704\uc6d0\ud68c\ub294 \"\uc544\uc608 \uc548\ud560\uac83\uac19\uc73c\uba74 \uc9c4\uc791 \ud3ec\uae30\ud574\uc57c \ud558\ub294\ub370 \uc138\ubc88\uc774\ub098 \uc5f0\uae30\ud558\ub2c8 '\uc6b0\ub9ac\uac00 \uc5f4\ub9dd\uc774 \uc801\uc5c8\ub098' \uc2f6\uc5b4\uc11c \uc2f8\uc6c0\uc774 \ubc8c\uc5b4\uc9c4 \uac83\uc774\ub2e4.\"\ub77c\uace0 \uc9c0\uc801\ud588\ub2e4.", "answer": "\uacbd\uc81c\uc801 \ud0c0\ub2f9\uc131", "sentence": "\uacbd\uc81c\uc801 \ud0c0\ub2f9\uc131 \uc774 \uc5c6\ub2e4\ub294 \uac83\uc774 \uac00\uc7a5 \ud070 \uc774\uc720\uc600\ub2e4.", "paragraph_sentence": "2011\ub144 3\uc6d4 30\uc77c \uad6d\ud1a0\ud574\uc591\ubd80\ub294 \"\ub450 \ud6c4\ubcf4\uc9c0 \ubaa8\ub450 \ubd88\ub9ac\ud55c \uc9c0\ud615\uc870\uac74\uc73c\ub85c \uc778\ud55c \ud658\uacbd\ubb38\uc81c, \uc0ac\uc5c5\ube44 \uacfc\ub2e4, \uacbd\uc81c\uc131 \ubbf8\ud761 \ub4f1\uc73c\ub85c \ud604 \uc2dc\uc810\uc5d0\uc11c \uc0ac\uc5c5 \ucd94\uc9c4 \uc5ec\uac74\uc774 \uc801\ud569\uce58 \uc54a\ub2e4\ub294 \uacb0\ub860\uc5d0 \ub3c4\ucd9c\ud588\ub2e4\"\uba74\uc11c \ubc31\uc9c0\ud654\ub97c \ubc1c\ud45c\ud588\ub2e4. 2003\ub144\ubd80\ud130 \ucd94\uc9c4\ub41c \ub3d9\ub0a8\uad8c \uc2e0\uacf5\ud56d \uc0ac\uc5c5\uc774 \uc644\uc804\ud788 \ud3d0\uae30\ub41c \uac83\uc774\ub2e4. \uacbd\uc81c\uc801 \ud0c0\ub2f9\uc131 \uc774 \uc5c6\ub2e4\ub294 \uac83\uc774 \uac00\uc7a5 \ud070 \uc774\uc720\uc600\ub2e4. \uadf8\ub7ec\ub098 \uacbd\uc81c\uc131\ub9cc\uc744 \uac80\ud1a0\ud55c \ubc31\uc9c0\ud654\ub294 \uc798\ubabb\ub410\ub2e4\ub294 \uc8fc\uc7a5\ub3c4 \uc788\ub2e4. \ubd80\uc0b0 \uc218\uc601\uad6c\uac00 \uc9c0\uc5ed\uad6c\uc778 \ud55c\ub098\ub77c\ub2f9 \uc720\uc7ac\uc911 \uc758\uc6d0\uc740 \"B/C\ub9cc \ub530\uc838\uc11c\ub294 \uc548\ub41c\ub2e4. \uc6d0\ub798 1\uc774\uc0c1 \ub098\uc62c\uc218\uac00 \uc5c6\ub2e4. \ud604\uc7ac \ud638\ub0a8\uace0\uc18d\ub3c4\ub85c, \uc11c\ud574\uc548\uace0\uc18d\ub3c4\ub85c\ub97c \ubcf4\uba74 B/C(\ube44\uc6a9 \ub300\ube44 \ud3b8\uc775)\uac00 0.39\uc774\uba70, \uace0\uc18d\ucca0\ub3c4 B/C\uac00 0.39\ubc16\uc5d0 \ub098\uc624\uc9c0 \uc54a\uc558\uc9c0\ub9cc \uc0ac\uc5c5\uc744 \ucd94\uc9c4\ud588\ub2e4. \"\ub77c\uba74\uc11c \"\uc120\ub3c4 \ud504\ub85c\uc81d\ud2b8 \uc0ac\uc5c5\uc740 \uade0\ud615\ubc1c\uc804\uc744 \ud558\uaca0\ub2e4\ub294 \uc0ac\uc5c5\uc774\ub2e4. \uadf8\uac83\uc744 \uacbd\uc81c\uc131\uc774 \uc5c6\ub2e4\uace0 \uc548\ud558\uba74 \uc9c0\ubc29\uc740 \ud560 \uc0ac\uc5c5\uc774 \uc5c6\ub294\uac83\uc774\ub2e4\"\ub77c\uace0 \ubaa9\uc18c\ub9ac\ub97c \ub192\uc600\ub2e4. \uc815\ubd80\uac00 \ub108\ubb34 \ub9ce\uc774 \uc2dc\uac04\uc744 \ub04c\uc5c8\ub358\uac83\uc5d0 \ub300\ud55c \ube44\ud310\ub3c4 \uc788\ub2e4. \uac15\uc8fc\uc5f4 \uc2e0\uacf5\ud56d\ubc00\uc591\uc720\uce58 \uacb0\uc0ac\ucd94\uc9c4\uc704\uc6d0\ud68c\ub294 \"\uc544\uc608 \uc548\ud560\uac83\uac19\uc73c\uba74 \uc9c4\uc791 \ud3ec\uae30\ud574\uc57c \ud558\ub294\ub370 \uc138\ubc88\uc774\ub098 \uc5f0\uae30\ud558\ub2c8 '\uc6b0\ub9ac\uac00 \uc5f4\ub9dd\uc774 \uc801\uc5c8\ub098' \uc2f6\uc5b4\uc11c \uc2f8\uc6c0\uc774 \ubc8c\uc5b4\uc9c4 \uac83\uc774\ub2e4. \"\ub77c\uace0 \uc9c0\uc801\ud588\ub2e4.", "paragraph_answer": "2011\ub144 3\uc6d4 30\uc77c \uad6d\ud1a0\ud574\uc591\ubd80\ub294 \"\ub450 \ud6c4\ubcf4\uc9c0 \ubaa8\ub450 \ubd88\ub9ac\ud55c \uc9c0\ud615\uc870\uac74\uc73c\ub85c \uc778\ud55c \ud658\uacbd\ubb38\uc81c, \uc0ac\uc5c5\ube44 \uacfc\ub2e4, \uacbd\uc81c\uc131 \ubbf8\ud761 \ub4f1\uc73c\ub85c \ud604 \uc2dc\uc810\uc5d0\uc11c \uc0ac\uc5c5 \ucd94\uc9c4 \uc5ec\uac74\uc774 \uc801\ud569\uce58 \uc54a\ub2e4\ub294 \uacb0\ub860\uc5d0 \ub3c4\ucd9c\ud588\ub2e4\"\uba74\uc11c \ubc31\uc9c0\ud654\ub97c \ubc1c\ud45c\ud588\ub2e4. 2003\ub144\ubd80\ud130 \ucd94\uc9c4\ub41c \ub3d9\ub0a8\uad8c \uc2e0\uacf5\ud56d \uc0ac\uc5c5\uc774 \uc644\uc804\ud788 \ud3d0\uae30\ub41c \uac83\uc774\ub2e4. \uacbd\uc81c\uc801 \ud0c0\ub2f9\uc131 \uc774 \uc5c6\ub2e4\ub294 \uac83\uc774 \uac00\uc7a5 \ud070 \uc774\uc720\uc600\ub2e4. \uadf8\ub7ec\ub098 \uacbd\uc81c\uc131\ub9cc\uc744 \uac80\ud1a0\ud55c \ubc31\uc9c0\ud654\ub294 \uc798\ubabb\ub410\ub2e4\ub294 \uc8fc\uc7a5\ub3c4 \uc788\ub2e4. \ubd80\uc0b0 \uc218\uc601\uad6c\uac00 \uc9c0\uc5ed\uad6c\uc778 \ud55c\ub098\ub77c\ub2f9 \uc720\uc7ac\uc911 \uc758\uc6d0\uc740 \"B/C\ub9cc \ub530\uc838\uc11c\ub294 \uc548\ub41c\ub2e4. \uc6d0\ub798 1\uc774\uc0c1 \ub098\uc62c\uc218\uac00 \uc5c6\ub2e4. \ud604\uc7ac \ud638\ub0a8\uace0\uc18d\ub3c4\ub85c, \uc11c\ud574\uc548\uace0\uc18d\ub3c4\ub85c\ub97c \ubcf4\uba74 B/C(\ube44\uc6a9 \ub300\ube44 \ud3b8\uc775)\uac00 0.39\uc774\uba70, \uace0\uc18d\ucca0\ub3c4 B/C\uac00 0.39\ubc16\uc5d0 \ub098\uc624\uc9c0 \uc54a\uc558\uc9c0\ub9cc \uc0ac\uc5c5\uc744 \ucd94\uc9c4\ud588\ub2e4.\"\ub77c\uba74\uc11c \"\uc120\ub3c4 \ud504\ub85c\uc81d\ud2b8 \uc0ac\uc5c5\uc740 \uade0\ud615\ubc1c\uc804\uc744 \ud558\uaca0\ub2e4\ub294 \uc0ac\uc5c5\uc774\ub2e4. \uadf8\uac83\uc744 \uacbd\uc81c\uc131\uc774 \uc5c6\ub2e4\uace0 \uc548\ud558\uba74 \uc9c0\ubc29\uc740 \ud560 \uc0ac\uc5c5\uc774 \uc5c6\ub294\uac83\uc774\ub2e4\"\ub77c\uace0 \ubaa9\uc18c\ub9ac\ub97c \ub192\uc600\ub2e4. \uc815\ubd80\uac00 \ub108\ubb34 \ub9ce\uc774 \uc2dc\uac04\uc744 \ub04c\uc5c8\ub358\uac83\uc5d0 \ub300\ud55c \ube44\ud310\ub3c4 \uc788\ub2e4. \uac15\uc8fc\uc5f4 \uc2e0\uacf5\ud56d\ubc00\uc591\uc720\uce58 \uacb0\uc0ac\ucd94\uc9c4\uc704\uc6d0\ud68c\ub294 \"\uc544\uc608 \uc548\ud560\uac83\uac19\uc73c\uba74 \uc9c4\uc791 \ud3ec\uae30\ud574\uc57c \ud558\ub294\ub370 \uc138\ubc88\uc774\ub098 \uc5f0\uae30\ud558\ub2c8 '\uc6b0\ub9ac\uac00 \uc5f4\ub9dd\uc774 \uc801\uc5c8\ub098' \uc2f6\uc5b4\uc11c \uc2f8\uc6c0\uc774 \ubc8c\uc5b4\uc9c4 \uac83\uc774\ub2e4.\"\ub77c\uace0 \uc9c0\uc801\ud588\ub2e4.", "sentence_answer": " \uacbd\uc81c\uc801 \ud0c0\ub2f9\uc131 \uc774 \uc5c6\ub2e4\ub294 \uac83\uc774 \uac00\uc7a5 \ud070 \uc774\uc720\uc600\ub2e4.", "paragraph_id": "6524258-9-2", "paragraph_question": "question: \uc2e0\uacf5\ud56d \uc0ac\uc5c5\uc774 \uc644\uc804\ud788 \ud3d0\uae30\ub41c \uac00\uc7a5 \ud070 \uc774\uc720\ub294 \ubb34\uc5c7\uc774 \uc5c6\ub2e4\ub294 \ud310\ub2e8 \ub54c\ubb38\uc774\uc5c8\ub098?, context: 2011\ub144 3\uc6d4 30\uc77c \uad6d\ud1a0\ud574\uc591\ubd80\ub294 \"\ub450 \ud6c4\ubcf4\uc9c0 \ubaa8\ub450 \ubd88\ub9ac\ud55c \uc9c0\ud615\uc870\uac74\uc73c\ub85c \uc778\ud55c \ud658\uacbd\ubb38\uc81c, \uc0ac\uc5c5\ube44 \uacfc\ub2e4, \uacbd\uc81c\uc131 \ubbf8\ud761 \ub4f1\uc73c\ub85c \ud604 \uc2dc\uc810\uc5d0\uc11c \uc0ac\uc5c5 \ucd94\uc9c4 \uc5ec\uac74\uc774 \uc801\ud569\uce58 \uc54a\ub2e4\ub294 \uacb0\ub860\uc5d0 \ub3c4\ucd9c\ud588\ub2e4\"\uba74\uc11c \ubc31\uc9c0\ud654\ub97c \ubc1c\ud45c\ud588\ub2e4. 2003\ub144\ubd80\ud130 \ucd94\uc9c4\ub41c \ub3d9\ub0a8\uad8c \uc2e0\uacf5\ud56d \uc0ac\uc5c5\uc774 \uc644\uc804\ud788 \ud3d0\uae30\ub41c \uac83\uc774\ub2e4. \uacbd\uc81c\uc801 \ud0c0\ub2f9\uc131\uc774 \uc5c6\ub2e4\ub294 \uac83\uc774 \uac00\uc7a5 \ud070 \uc774\uc720\uc600\ub2e4. \uadf8\ub7ec\ub098 \uacbd\uc81c\uc131\ub9cc\uc744 \uac80\ud1a0\ud55c \ubc31\uc9c0\ud654\ub294 \uc798\ubabb\ub410\ub2e4\ub294 \uc8fc\uc7a5\ub3c4 \uc788\ub2e4. \ubd80\uc0b0 \uc218\uc601\uad6c\uac00 \uc9c0\uc5ed\uad6c\uc778 \ud55c\ub098\ub77c\ub2f9 \uc720\uc7ac\uc911 \uc758\uc6d0\uc740 \"B/C\ub9cc \ub530\uc838\uc11c\ub294 \uc548\ub41c\ub2e4. \uc6d0\ub798 1\uc774\uc0c1 \ub098\uc62c\uc218\uac00 \uc5c6\ub2e4. \ud604\uc7ac \ud638\ub0a8\uace0\uc18d\ub3c4\ub85c, \uc11c\ud574\uc548\uace0\uc18d\ub3c4\ub85c\ub97c \ubcf4\uba74 B/C(\ube44\uc6a9 \ub300\ube44 \ud3b8\uc775)\uac00 0.39\uc774\uba70, \uace0\uc18d\ucca0\ub3c4 B/C\uac00 0.39\ubc16\uc5d0 \ub098\uc624\uc9c0 \uc54a\uc558\uc9c0\ub9cc \uc0ac\uc5c5\uc744 \ucd94\uc9c4\ud588\ub2e4.\"\ub77c\uba74\uc11c \"\uc120\ub3c4 \ud504\ub85c\uc81d\ud2b8 \uc0ac\uc5c5\uc740 \uade0\ud615\ubc1c\uc804\uc744 \ud558\uaca0\ub2e4\ub294 \uc0ac\uc5c5\uc774\ub2e4. \uadf8\uac83\uc744 \uacbd\uc81c\uc131\uc774 \uc5c6\ub2e4\uace0 \uc548\ud558\uba74 \uc9c0\ubc29\uc740 \ud560 \uc0ac\uc5c5\uc774 \uc5c6\ub294\uac83\uc774\ub2e4\"\ub77c\uace0 \ubaa9\uc18c\ub9ac\ub97c \ub192\uc600\ub2e4. \uc815\ubd80\uac00 \ub108\ubb34 \ub9ce\uc774 \uc2dc\uac04\uc744 \ub04c\uc5c8\ub358\uac83\uc5d0 \ub300\ud55c \ube44\ud310\ub3c4 \uc788\ub2e4. \uac15\uc8fc\uc5f4 \uc2e0\uacf5\ud56d\ubc00\uc591\uc720\uce58 \uacb0\uc0ac\ucd94\uc9c4\uc704\uc6d0\ud68c\ub294 \"\uc544\uc608 \uc548\ud560\uac83\uac19\uc73c\uba74 \uc9c4\uc791 \ud3ec\uae30\ud574\uc57c \ud558\ub294\ub370 \uc138\ubc88\uc774\ub098 \uc5f0\uae30\ud558\ub2c8 '\uc6b0\ub9ac\uac00 \uc5f4\ub9dd\uc774 \uc801\uc5c8\ub098' \uc2f6\uc5b4\uc11c \uc2f8\uc6c0\uc774 \ubc8c\uc5b4\uc9c4 \uac83\uc774\ub2e4.\"\ub77c\uace0 \uc9c0\uc801\ud588\ub2e4."} {"question": "\uc778\uc81c\uc5d0 \uc720\ub09c\ud788 \ub9ce\uc73c\uba70 \uad6d\uad8c\ud68c\ubcf5\uc744 \uc758\ubbf8\ud558\ub294 \uc9c0\uba85\uc740 \ubb34\uc5c7\uc778\uac00?", "paragraph": "\uadf8 \uac00\uc6b4\ub370 \ud765\ubbf8\ub85c\uc6b4 \uac83\uc740 \ub9c8\uc758\ud0dc\uc790(\u9ebb\u8863\u592a\u5b50)\uac00 \ub2e8\uc9c0 \uc740\uac70\ud558\ub294 \ub370\uc5d0 \uadf8\uce58\uc9c0 \uc54a\uace0 \uace0\ub824\uc5d0 \uc758\ud574 \uba78\ub9dd\ud55c(\uc2e4\uc740 \uc2a4\uc2a4\ub85c \ud56d\ubcf5\ud55c) \uc870\uad6d \uc2e0\ub77c\uc758 \ubd80\ud65c\uc744 \uc704\ud574 \ubd80\ud765\uc6b4\ub3d9\uc744 \uc900\ube44\ud588\ub2e4\ub294 \uc804\uc2b9\uc774\ub2e4. \ud604\uc9c0 \uc804\uc2b9\uc5d0 \ub530\ub974\uba74 \ub9c8\uc758 \ud0dc\uc790\ub294 \ud63c\uc790 \uc218\ub3c4\ub97c \ub5a0\ub09c \uac83\uc774 \uc544\ub2c8\ub77c \uc77c\uad70\uc758 \ubb34\ub9ac\ub97c \uc774\ub04c\uace0 \uc788\uc5c8\uace0 \uadf8 \uac00\uc6b4\ub370 \ud55c \uc0ac\ub78c\uc774\uc5c8\ub358 \ub9f9\uc7a5\uad70\uc774\ub77c\ub294 \uc0ac\ub78c\uc758 \uc774\ub984\uc744 \ub530\uc11c \ubd99\uc5ec\uc9c4 \ub9f9\uac1c\uace8\uc774\ub77c\ub294 \ub9c8\uc744\uc774\ub984\uc774\ub098 \uc2e0\ub77c \ubd80\ud765 \uc6b4\ub3d9\uc744 \uc704\ud574 \uad70\ub7c9\ubbf8\ub97c \ubaa8\uc544 \uc800\uc7a5\ud558\uc600\ub2e4\ub294 '\uad70\ub7c9\ub9ac'\ub77c\ub294 \ub9c8\uc744 \uc9c0\uba85\ub3c4 \ub0a8\uc544 \uc788\ub2e4\uace0 \ud55c\ub2e4. \ub610\ud55c \uc778\uc81c\uc5d0 \uc720\ub09c\ud788 \ub9ce\uc740 '\ub2e4\ubb34\ub9ac'\ub77c\ub294 \uc9c0\uba85\uc740 '\uad6d\uad8c \ud68c\ubcf5'\uc744 \ub73b\ud558\ub294 \uac83\uc73c\ub85c(\u300a\uc0bc\uad6d\uc0ac\uae30\u300b\uc5d0\uc11c \uc774\ubbf8 '\ub3c4\ub85c \ub418\ucc3e\uc740 \ub545'\uc774\ub77c\ub294 \ub73b\uc758 '\ub2e4\ubb3c'\uc774\ub77c\ub294 \ub9d0\uc774 \uc5b8\uae09\ub41c \uc608\uac00 \uc788\ub2e4) \uc774\ub97c \uadfc\uac70\ub85c \ub9c8\uc758\ud0dc\uc790\uac00 \uc2e0\ub77c\ubd80\ud765\uc6b4\ub3d9\uc744 \uc900\ube44\ud558\uace0 \uc788\uc5c8\ub2e4\uace0 \uc8fc\uc7a5\ud558\ub294 \uc124\uc774 \uc788\ub2e4.", "answer": "\ub2e4\ubb34\ub9ac", "sentence": "\ub610\ud55c \uc778\uc81c\uc5d0 \uc720\ub09c\ud788 \ub9ce\uc740 ' \ub2e4\ubb34\ub9ac '\ub77c\ub294 \uc9c0\uba85\uc740 '\uad6d\uad8c \ud68c\ubcf5'\uc744 \ub73b\ud558\ub294 \uac83\uc73c\ub85c(\u300a\uc0bc\uad6d\uc0ac\uae30\u300b\uc5d0\uc11c \uc774\ubbf8 '\ub3c4\ub85c \ub418\ucc3e\uc740 \ub545'\uc774\ub77c\ub294 \ub73b\uc758 '\ub2e4\ubb3c'\uc774\ub77c\ub294 \ub9d0\uc774 \uc5b8\uae09\ub41c \uc608\uac00 \uc788\ub2e4)", "paragraph_sentence": "\uadf8 \uac00\uc6b4\ub370 \ud765\ubbf8\ub85c\uc6b4 \uac83\uc740 \ub9c8\uc758\ud0dc\uc790(\u9ebb\u8863\u592a\u5b50)\uac00 \ub2e8\uc9c0 \uc740\uac70\ud558\ub294 \ub370\uc5d0 \uadf8\uce58\uc9c0 \uc54a\uace0 \uace0\ub824\uc5d0 \uc758\ud574 \uba78\ub9dd\ud55c(\uc2e4\uc740 \uc2a4\uc2a4\ub85c \ud56d\ubcf5\ud55c) \uc870\uad6d \uc2e0\ub77c\uc758 \ubd80\ud65c\uc744 \uc704\ud574 \ubd80\ud765\uc6b4\ub3d9\uc744 \uc900\ube44\ud588\ub2e4\ub294 \uc804\uc2b9\uc774\ub2e4. \ud604\uc9c0 \uc804\uc2b9\uc5d0 \ub530\ub974\uba74 \ub9c8\uc758 \ud0dc\uc790\ub294 \ud63c\uc790 \uc218\ub3c4\ub97c \ub5a0\ub09c \uac83\uc774 \uc544\ub2c8\ub77c \uc77c\uad70\uc758 \ubb34\ub9ac\ub97c \uc774\ub04c\uace0 \uc788\uc5c8\uace0 \uadf8 \uac00\uc6b4\ub370 \ud55c \uc0ac\ub78c\uc774\uc5c8\ub358 \ub9f9\uc7a5\uad70\uc774\ub77c\ub294 \uc0ac\ub78c\uc758 \uc774\ub984\uc744 \ub530\uc11c \ubd99\uc5ec\uc9c4 \ub9f9\uac1c\uace8\uc774\ub77c\ub294 \ub9c8\uc744\uc774\ub984\uc774\ub098 \uc2e0\ub77c \ubd80\ud765 \uc6b4\ub3d9\uc744 \uc704\ud574 \uad70\ub7c9\ubbf8\ub97c \ubaa8\uc544 \uc800\uc7a5\ud558\uc600\ub2e4\ub294 '\uad70\ub7c9\ub9ac'\ub77c\ub294 \ub9c8\uc744 \uc9c0\uba85\ub3c4 \ub0a8\uc544 \uc788\ub2e4\uace0 \ud55c\ub2e4. \ub610\ud55c \uc778\uc81c\uc5d0 \uc720\ub09c\ud788 \ub9ce\uc740 ' \ub2e4\ubb34\ub9ac '\ub77c\ub294 \uc9c0\uba85\uc740 '\uad6d\uad8c \ud68c\ubcf5'\uc744 \ub73b\ud558\ub294 \uac83\uc73c\ub85c(\u300a\uc0bc\uad6d\uc0ac\uae30\u300b\uc5d0\uc11c \uc774\ubbf8 '\ub3c4\ub85c \ub418\ucc3e\uc740 \ub545'\uc774\ub77c\ub294 \ub73b\uc758 '\ub2e4\ubb3c'\uc774\ub77c\ub294 \ub9d0\uc774 \uc5b8\uae09\ub41c \uc608\uac00 \uc788\ub2e4) \uc774\ub97c \uadfc\uac70\ub85c \ub9c8\uc758\ud0dc\uc790\uac00 \uc2e0\ub77c\ubd80\ud765\uc6b4\ub3d9\uc744 \uc900\ube44\ud558\uace0 \uc788\uc5c8\ub2e4\uace0 \uc8fc\uc7a5\ud558\ub294 \uc124\uc774 \uc788\ub2e4.", "paragraph_answer": "\uadf8 \uac00\uc6b4\ub370 \ud765\ubbf8\ub85c\uc6b4 \uac83\uc740 \ub9c8\uc758\ud0dc\uc790(\u9ebb\u8863\u592a\u5b50)\uac00 \ub2e8\uc9c0 \uc740\uac70\ud558\ub294 \ub370\uc5d0 \uadf8\uce58\uc9c0 \uc54a\uace0 \uace0\ub824\uc5d0 \uc758\ud574 \uba78\ub9dd\ud55c(\uc2e4\uc740 \uc2a4\uc2a4\ub85c \ud56d\ubcf5\ud55c) \uc870\uad6d \uc2e0\ub77c\uc758 \ubd80\ud65c\uc744 \uc704\ud574 \ubd80\ud765\uc6b4\ub3d9\uc744 \uc900\ube44\ud588\ub2e4\ub294 \uc804\uc2b9\uc774\ub2e4. \ud604\uc9c0 \uc804\uc2b9\uc5d0 \ub530\ub974\uba74 \ub9c8\uc758 \ud0dc\uc790\ub294 \ud63c\uc790 \uc218\ub3c4\ub97c \ub5a0\ub09c \uac83\uc774 \uc544\ub2c8\ub77c \uc77c\uad70\uc758 \ubb34\ub9ac\ub97c \uc774\ub04c\uace0 \uc788\uc5c8\uace0 \uadf8 \uac00\uc6b4\ub370 \ud55c \uc0ac\ub78c\uc774\uc5c8\ub358 \ub9f9\uc7a5\uad70\uc774\ub77c\ub294 \uc0ac\ub78c\uc758 \uc774\ub984\uc744 \ub530\uc11c \ubd99\uc5ec\uc9c4 \ub9f9\uac1c\uace8\uc774\ub77c\ub294 \ub9c8\uc744\uc774\ub984\uc774\ub098 \uc2e0\ub77c \ubd80\ud765 \uc6b4\ub3d9\uc744 \uc704\ud574 \uad70\ub7c9\ubbf8\ub97c \ubaa8\uc544 \uc800\uc7a5\ud558\uc600\ub2e4\ub294 '\uad70\ub7c9\ub9ac'\ub77c\ub294 \ub9c8\uc744 \uc9c0\uba85\ub3c4 \ub0a8\uc544 \uc788\ub2e4\uace0 \ud55c\ub2e4. \ub610\ud55c \uc778\uc81c\uc5d0 \uc720\ub09c\ud788 \ub9ce\uc740 ' \ub2e4\ubb34\ub9ac '\ub77c\ub294 \uc9c0\uba85\uc740 '\uad6d\uad8c \ud68c\ubcf5'\uc744 \ub73b\ud558\ub294 \uac83\uc73c\ub85c(\u300a\uc0bc\uad6d\uc0ac\uae30\u300b\uc5d0\uc11c \uc774\ubbf8 '\ub3c4\ub85c \ub418\ucc3e\uc740 \ub545'\uc774\ub77c\ub294 \ub73b\uc758 '\ub2e4\ubb3c'\uc774\ub77c\ub294 \ub9d0\uc774 \uc5b8\uae09\ub41c \uc608\uac00 \uc788\ub2e4) \uc774\ub97c \uadfc\uac70\ub85c \ub9c8\uc758\ud0dc\uc790\uac00 \uc2e0\ub77c\ubd80\ud765\uc6b4\ub3d9\uc744 \uc900\ube44\ud558\uace0 \uc788\uc5c8\ub2e4\uace0 \uc8fc\uc7a5\ud558\ub294 \uc124\uc774 \uc788\ub2e4.", "sentence_answer": "\ub610\ud55c \uc778\uc81c\uc5d0 \uc720\ub09c\ud788 \ub9ce\uc740 ' \ub2e4\ubb34\ub9ac '\ub77c\ub294 \uc9c0\uba85\uc740 '\uad6d\uad8c \ud68c\ubcf5'\uc744 \ub73b\ud558\ub294 \uac83\uc73c\ub85c(\u300a\uc0bc\uad6d\uc0ac\uae30\u300b\uc5d0\uc11c \uc774\ubbf8 '\ub3c4\ub85c \ub418\ucc3e\uc740 \ub545'\uc774\ub77c\ub294 \ub73b\uc758 '\ub2e4\ubb3c'\uc774\ub77c\ub294 \ub9d0\uc774 \uc5b8\uae09\ub41c \uc608\uac00 \uc788\ub2e4)", "paragraph_id": "6500216-1-0", "paragraph_question": "question: \uc778\uc81c\uc5d0 \uc720\ub09c\ud788 \ub9ce\uc73c\uba70 \uad6d\uad8c\ud68c\ubcf5\uc744 \uc758\ubbf8\ud558\ub294 \uc9c0\uba85\uc740 \ubb34\uc5c7\uc778\uac00?, context: \uadf8 \uac00\uc6b4\ub370 \ud765\ubbf8\ub85c\uc6b4 \uac83\uc740 \ub9c8\uc758\ud0dc\uc790(\u9ebb\u8863\u592a\u5b50)\uac00 \ub2e8\uc9c0 \uc740\uac70\ud558\ub294 \ub370\uc5d0 \uadf8\uce58\uc9c0 \uc54a\uace0 \uace0\ub824\uc5d0 \uc758\ud574 \uba78\ub9dd\ud55c(\uc2e4\uc740 \uc2a4\uc2a4\ub85c \ud56d\ubcf5\ud55c) \uc870\uad6d \uc2e0\ub77c\uc758 \ubd80\ud65c\uc744 \uc704\ud574 \ubd80\ud765\uc6b4\ub3d9\uc744 \uc900\ube44\ud588\ub2e4\ub294 \uc804\uc2b9\uc774\ub2e4. \ud604\uc9c0 \uc804\uc2b9\uc5d0 \ub530\ub974\uba74 \ub9c8\uc758 \ud0dc\uc790\ub294 \ud63c\uc790 \uc218\ub3c4\ub97c \ub5a0\ub09c \uac83\uc774 \uc544\ub2c8\ub77c \uc77c\uad70\uc758 \ubb34\ub9ac\ub97c \uc774\ub04c\uace0 \uc788\uc5c8\uace0 \uadf8 \uac00\uc6b4\ub370 \ud55c \uc0ac\ub78c\uc774\uc5c8\ub358 \ub9f9\uc7a5\uad70\uc774\ub77c\ub294 \uc0ac\ub78c\uc758 \uc774\ub984\uc744 \ub530\uc11c \ubd99\uc5ec\uc9c4 \ub9f9\uac1c\uace8\uc774\ub77c\ub294 \ub9c8\uc744\uc774\ub984\uc774\ub098 \uc2e0\ub77c \ubd80\ud765 \uc6b4\ub3d9\uc744 \uc704\ud574 \uad70\ub7c9\ubbf8\ub97c \ubaa8\uc544 \uc800\uc7a5\ud558\uc600\ub2e4\ub294 '\uad70\ub7c9\ub9ac'\ub77c\ub294 \ub9c8\uc744 \uc9c0\uba85\ub3c4 \ub0a8\uc544 \uc788\ub2e4\uace0 \ud55c\ub2e4. \ub610\ud55c \uc778\uc81c\uc5d0 \uc720\ub09c\ud788 \ub9ce\uc740 '\ub2e4\ubb34\ub9ac'\ub77c\ub294 \uc9c0\uba85\uc740 '\uad6d\uad8c \ud68c\ubcf5'\uc744 \ub73b\ud558\ub294 \uac83\uc73c\ub85c(\u300a\uc0bc\uad6d\uc0ac\uae30\u300b\uc5d0\uc11c \uc774\ubbf8 '\ub3c4\ub85c \ub418\ucc3e\uc740 \ub545'\uc774\ub77c\ub294 \ub73b\uc758 '\ub2e4\ubb3c'\uc774\ub77c\ub294 \ub9d0\uc774 \uc5b8\uae09\ub41c \uc608\uac00 \uc788\ub2e4) \uc774\ub97c \uadfc\uac70\ub85c \ub9c8\uc758\ud0dc\uc790\uac00 \uc2e0\ub77c\ubd80\ud765\uc6b4\ub3d9\uc744 \uc900\ube44\ud558\uace0 \uc788\uc5c8\ub2e4\uace0 \uc8fc\uc7a5\ud558\ub294 \uc124\uc774 \uc788\ub2e4."} {"question": "\ud5c8\uc218\uc544\ube44\uc655\uacfc \uc591\ucca0 \ub098\ubb34\uafbc\ud669\uc81c\ub97c \ub450\ub824\uc6cc\ud55c \uc0ac\ub78c\uc740?", "paragraph": "\ud55c\ud3b8 \uc601\uc6c5\uc73c\ub85c \uc54c\ub824\uc9c4 \ud5c8\uc218\uc544\ube44\uc655\uacfc \uc591\ucca0 \ub098\ubb34\uafbc\ud669\uc81c\uac00 \ud568\uaed8 \uc62c \uac83\uc744 \ub450\ub824\uc6cc\ud55c \uc9c4\uc800\uc7a5\uad70\uc740 \ubab8\ube44\ub97c \ubd80\ub974\uace0, \ubab8\ube44\ub294 \ud301\uc77c\ud589\uc774 \uc624\ub294 \uae38\uc5d0 \ub9c8\ubc95\uc73c\ub85c \ud658\uc0c1\uc744 \ub9cc\ub4e4\uc5b4 \ub0b4\uc5b4 \uae38\uc744 \uc783\uac8c \ub9cc\ub4e0\ub2e4. \ud5c8\uc218\uc544\ube44\uc758 \uc9c0\ud61c\ub85c \uae38\uc744 \ud5e4\uccd0\uac00\ub358 \uc77c\ud589\uc740 \ubaa9\ub9c8\uac00 \ud1a0\ub07c\uad6c\uba4d\uc5d0 \ube60\uc838 \ub2e4\ub9ac\uac00 \ubd80\ub7ec\uc9c0\ub294 \ubc14\ub78c\uc5d0 \uba48\ucd94\uac8c \ub418\uace0, \uadf8\uacf3\uc5d0\uc11c \uc0ac\ub78c\ub9cc\ud07c \ud070\ub370\ub2e4 \uc591\uc7a5\uae4c\uc9c0 \ucc28\ub824\uc785\uc740 \uc6cc\uae00\ubc8c\ub808\ub97c \ub9cc\ub098\uac8c \ub41c\ub2e4. \uc774 \uc6cc\uae00\ubc8c\ub808\ub294 \uc6b0\uc5f0\ud788 \ud559\uad50\uc5d0 \uac14\ub2e4\uac00 \ubcbd\ub09c\ub85c\ud2c8\uc5d0\uc11c 3\ub144\uac04 \uc0b4\uba70 \ub178\uc704\ud2c0\uad50\uc218\uc758 \uc218\uc5c5\uc744 \ub4e4\uc5c8\uace0, \uc790\uc2e0\uc744 '\uc644\uc804\ud558\uac8c \uad50\uc721\uc744 \ubc1b\uc740' \ud559\uc704\uc788\ub294 \ubc8c\ub808\ub77c\uace0 \uc18c\uac1c\ud55c\ub2e4. \ub610\ud55c \uadf8\ub294 \uad50\uc218\uc5d0\uac8c \uc7a1\ud614\ub2e4\uac00 \uad50\uc7ac\uc6a9\uc73c\ub85c \ud655\ub300\uacbd \uc704\uc5d0 \uc62c\ub790\uc5c8\ub294\ub370, \uc774 \ub54c \ub3c4\ub9dd\uccd0 \uc9c0\uae08\ucc98\ub7fc \ud070 \ubab8\uc744 \uac00\uc838 '\ub300\ub2e8\ud788 \uc704\ub300\ud55c' \uc6cc\uae00\ubc8c\ub808\uac00 \ub418\uc5c8\ub2e4\ub294 \uac83\uc774\ub2e4. \uc0c8\ub85c\uc6b4 \uc77c\ud589\uc774 \ub41c \uc6cc\uae00\ubc8c\ub808\ub294 \ud559\uc2dd\uacfc \uad50\uc591\uc744 \uac16\ucd98 \uc9c0\uc2dd\uc778\uc774\ub77c\uba74 \ub18d\ub2f4\uc744 \ud1b5\ud574 \ucc9c\uc7ac\uc131\uc744 \ub4dc\ub7ec\ub0b8\ub2e4\uba70 \ub9d0\uc7a5\ub09c\uc744 \ud558\uc9c0\ub9cc \uc77c\ud589\uc740 \ubaa8\ub450 \ubd88\ucf8c\ud574 \ud55c\ub2e4.", "answer": "\uc9c4\uc800\uc7a5\uad70", "sentence": "\ud55c\ud3b8 \uc601\uc6c5\uc73c\ub85c \uc54c\ub824\uc9c4 \ud5c8\uc218\uc544\ube44\uc655\uacfc \uc591\ucca0 \ub098\ubb34\uafbc\ud669\uc81c\uac00 \ud568\uaed8 \uc62c \uac83\uc744 \ub450\ub824\uc6cc\ud55c \uc9c4\uc800\uc7a5\uad70 \uc740 \ubab8\ube44\ub97c \ubd80\ub974\uace0, \ubab8\ube44\ub294 \ud301\uc77c\ud589\uc774 \uc624\ub294 \uae38\uc5d0 \ub9c8\ubc95\uc73c\ub85c \ud658\uc0c1\uc744 \ub9cc\ub4e4\uc5b4 \ub0b4\uc5b4 \uae38\uc744 \uc783\uac8c \ub9cc\ub4e0\ub2e4.", "paragraph_sentence": " \ud55c\ud3b8 \uc601\uc6c5\uc73c\ub85c \uc54c\ub824\uc9c4 \ud5c8\uc218\uc544\ube44\uc655\uacfc \uc591\ucca0 \ub098\ubb34\uafbc\ud669\uc81c\uac00 \ud568\uaed8 \uc62c \uac83\uc744 \ub450\ub824\uc6cc\ud55c \uc9c4\uc800\uc7a5\uad70 \uc740 \ubab8\ube44\ub97c \ubd80\ub974\uace0, \ubab8\ube44\ub294 \ud301\uc77c\ud589\uc774 \uc624\ub294 \uae38\uc5d0 \ub9c8\ubc95\uc73c\ub85c \ud658\uc0c1\uc744 \ub9cc\ub4e4\uc5b4 \ub0b4\uc5b4 \uae38\uc744 \uc783\uac8c \ub9cc\ub4e0\ub2e4. \ud5c8\uc218\uc544\ube44\uc758 \uc9c0\ud61c\ub85c \uae38\uc744 \ud5e4\uccd0\uac00\ub358 \uc77c\ud589\uc740 \ubaa9\ub9c8\uac00 \ud1a0\ub07c\uad6c\uba4d\uc5d0 \ube60\uc838 \ub2e4\ub9ac\uac00 \ubd80\ub7ec\uc9c0\ub294 \ubc14\ub78c\uc5d0 \uba48\ucd94\uac8c \ub418\uace0, \uadf8\uacf3\uc5d0\uc11c \uc0ac\ub78c\ub9cc\ud07c \ud070\ub370\ub2e4 \uc591\uc7a5\uae4c\uc9c0 \ucc28\ub824\uc785\uc740 \uc6cc\uae00\ubc8c\ub808\ub97c \ub9cc\ub098\uac8c \ub41c\ub2e4. \uc774 \uc6cc\uae00\ubc8c\ub808\ub294 \uc6b0\uc5f0\ud788 \ud559\uad50\uc5d0 \uac14\ub2e4\uac00 \ubcbd\ub09c\ub85c\ud2c8\uc5d0\uc11c 3\ub144\uac04 \uc0b4\uba70 \ub178\uc704\ud2c0\uad50\uc218\uc758 \uc218\uc5c5\uc744 \ub4e4\uc5c8\uace0, \uc790\uc2e0\uc744 '\uc644\uc804\ud558\uac8c \uad50\uc721\uc744 \ubc1b\uc740' \ud559\uc704\uc788\ub294 \ubc8c\ub808\ub77c\uace0 \uc18c\uac1c\ud55c\ub2e4. \ub610\ud55c \uadf8\ub294 \uad50\uc218\uc5d0\uac8c \uc7a1\ud614\ub2e4\uac00 \uad50\uc7ac\uc6a9\uc73c\ub85c \ud655\ub300\uacbd \uc704\uc5d0 \uc62c\ub790\uc5c8\ub294\ub370, \uc774 \ub54c \ub3c4\ub9dd\uccd0 \uc9c0\uae08\ucc98\ub7fc \ud070 \ubab8\uc744 \uac00\uc838 '\ub300\ub2e8\ud788 \uc704\ub300\ud55c' \uc6cc\uae00\ubc8c\ub808\uac00 \ub418\uc5c8\ub2e4\ub294 \uac83\uc774\ub2e4. \uc0c8\ub85c\uc6b4 \uc77c\ud589\uc774 \ub41c \uc6cc\uae00\ubc8c\ub808\ub294 \ud559\uc2dd\uacfc \uad50\uc591\uc744 \uac16\ucd98 \uc9c0\uc2dd\uc778\uc774\ub77c\uba74 \ub18d\ub2f4\uc744 \ud1b5\ud574 \ucc9c\uc7ac\uc131\uc744 \ub4dc\ub7ec\ub0b8\ub2e4\uba70 \ub9d0\uc7a5\ub09c\uc744 \ud558\uc9c0\ub9cc \uc77c\ud589\uc740 \ubaa8\ub450 \ubd88\ucf8c\ud574 \ud55c\ub2e4.", "paragraph_answer": "\ud55c\ud3b8 \uc601\uc6c5\uc73c\ub85c \uc54c\ub824\uc9c4 \ud5c8\uc218\uc544\ube44\uc655\uacfc \uc591\ucca0 \ub098\ubb34\uafbc\ud669\uc81c\uac00 \ud568\uaed8 \uc62c \uac83\uc744 \ub450\ub824\uc6cc\ud55c \uc9c4\uc800\uc7a5\uad70 \uc740 \ubab8\ube44\ub97c \ubd80\ub974\uace0, \ubab8\ube44\ub294 \ud301\uc77c\ud589\uc774 \uc624\ub294 \uae38\uc5d0 \ub9c8\ubc95\uc73c\ub85c \ud658\uc0c1\uc744 \ub9cc\ub4e4\uc5b4 \ub0b4\uc5b4 \uae38\uc744 \uc783\uac8c \ub9cc\ub4e0\ub2e4. \ud5c8\uc218\uc544\ube44\uc758 \uc9c0\ud61c\ub85c \uae38\uc744 \ud5e4\uccd0\uac00\ub358 \uc77c\ud589\uc740 \ubaa9\ub9c8\uac00 \ud1a0\ub07c\uad6c\uba4d\uc5d0 \ube60\uc838 \ub2e4\ub9ac\uac00 \ubd80\ub7ec\uc9c0\ub294 \ubc14\ub78c\uc5d0 \uba48\ucd94\uac8c \ub418\uace0, \uadf8\uacf3\uc5d0\uc11c \uc0ac\ub78c\ub9cc\ud07c \ud070\ub370\ub2e4 \uc591\uc7a5\uae4c\uc9c0 \ucc28\ub824\uc785\uc740 \uc6cc\uae00\ubc8c\ub808\ub97c \ub9cc\ub098\uac8c \ub41c\ub2e4. \uc774 \uc6cc\uae00\ubc8c\ub808\ub294 \uc6b0\uc5f0\ud788 \ud559\uad50\uc5d0 \uac14\ub2e4\uac00 \ubcbd\ub09c\ub85c\ud2c8\uc5d0\uc11c 3\ub144\uac04 \uc0b4\uba70 \ub178\uc704\ud2c0\uad50\uc218\uc758 \uc218\uc5c5\uc744 \ub4e4\uc5c8\uace0, \uc790\uc2e0\uc744 '\uc644\uc804\ud558\uac8c \uad50\uc721\uc744 \ubc1b\uc740' \ud559\uc704\uc788\ub294 \ubc8c\ub808\ub77c\uace0 \uc18c\uac1c\ud55c\ub2e4. \ub610\ud55c \uadf8\ub294 \uad50\uc218\uc5d0\uac8c \uc7a1\ud614\ub2e4\uac00 \uad50\uc7ac\uc6a9\uc73c\ub85c \ud655\ub300\uacbd \uc704\uc5d0 \uc62c\ub790\uc5c8\ub294\ub370, \uc774 \ub54c \ub3c4\ub9dd\uccd0 \uc9c0\uae08\ucc98\ub7fc \ud070 \ubab8\uc744 \uac00\uc838 '\ub300\ub2e8\ud788 \uc704\ub300\ud55c' \uc6cc\uae00\ubc8c\ub808\uac00 \ub418\uc5c8\ub2e4\ub294 \uac83\uc774\ub2e4. \uc0c8\ub85c\uc6b4 \uc77c\ud589\uc774 \ub41c \uc6cc\uae00\ubc8c\ub808\ub294 \ud559\uc2dd\uacfc \uad50\uc591\uc744 \uac16\ucd98 \uc9c0\uc2dd\uc778\uc774\ub77c\uba74 \ub18d\ub2f4\uc744 \ud1b5\ud574 \ucc9c\uc7ac\uc131\uc744 \ub4dc\ub7ec\ub0b8\ub2e4\uba70 \ub9d0\uc7a5\ub09c\uc744 \ud558\uc9c0\ub9cc \uc77c\ud589\uc740 \ubaa8\ub450 \ubd88\ucf8c\ud574 \ud55c\ub2e4.", "sentence_answer": "\ud55c\ud3b8 \uc601\uc6c5\uc73c\ub85c \uc54c\ub824\uc9c4 \ud5c8\uc218\uc544\ube44\uc655\uacfc \uc591\ucca0 \ub098\ubb34\uafbc\ud669\uc81c\uac00 \ud568\uaed8 \uc62c \uac83\uc744 \ub450\ub824\uc6cc\ud55c \uc9c4\uc800\uc7a5\uad70 \uc740 \ubab8\ube44\ub97c \ubd80\ub974\uace0, \ubab8\ube44\ub294 \ud301\uc77c\ud589\uc774 \uc624\ub294 \uae38\uc5d0 \ub9c8\ubc95\uc73c\ub85c \ud658\uc0c1\uc744 \ub9cc\ub4e4\uc5b4 \ub0b4\uc5b4 \uae38\uc744 \uc783\uac8c \ub9cc\ub4e0\ub2e4.", "paragraph_id": "6541299-4-0", "paragraph_question": "question: \ud5c8\uc218\uc544\ube44\uc655\uacfc \uc591\ucca0 \ub098\ubb34\uafbc\ud669\uc81c\ub97c \ub450\ub824\uc6cc\ud55c \uc0ac\ub78c\uc740?, context: \ud55c\ud3b8 \uc601\uc6c5\uc73c\ub85c \uc54c\ub824\uc9c4 \ud5c8\uc218\uc544\ube44\uc655\uacfc \uc591\ucca0 \ub098\ubb34\uafbc\ud669\uc81c\uac00 \ud568\uaed8 \uc62c \uac83\uc744 \ub450\ub824\uc6cc\ud55c \uc9c4\uc800\uc7a5\uad70\uc740 \ubab8\ube44\ub97c \ubd80\ub974\uace0, \ubab8\ube44\ub294 \ud301\uc77c\ud589\uc774 \uc624\ub294 \uae38\uc5d0 \ub9c8\ubc95\uc73c\ub85c \ud658\uc0c1\uc744 \ub9cc\ub4e4\uc5b4 \ub0b4\uc5b4 \uae38\uc744 \uc783\uac8c \ub9cc\ub4e0\ub2e4. \ud5c8\uc218\uc544\ube44\uc758 \uc9c0\ud61c\ub85c \uae38\uc744 \ud5e4\uccd0\uac00\ub358 \uc77c\ud589\uc740 \ubaa9\ub9c8\uac00 \ud1a0\ub07c\uad6c\uba4d\uc5d0 \ube60\uc838 \ub2e4\ub9ac\uac00 \ubd80\ub7ec\uc9c0\ub294 \ubc14\ub78c\uc5d0 \uba48\ucd94\uac8c \ub418\uace0, \uadf8\uacf3\uc5d0\uc11c \uc0ac\ub78c\ub9cc\ud07c \ud070\ub370\ub2e4 \uc591\uc7a5\uae4c\uc9c0 \ucc28\ub824\uc785\uc740 \uc6cc\uae00\ubc8c\ub808\ub97c \ub9cc\ub098\uac8c \ub41c\ub2e4. \uc774 \uc6cc\uae00\ubc8c\ub808\ub294 \uc6b0\uc5f0\ud788 \ud559\uad50\uc5d0 \uac14\ub2e4\uac00 \ubcbd\ub09c\ub85c\ud2c8\uc5d0\uc11c 3\ub144\uac04 \uc0b4\uba70 \ub178\uc704\ud2c0\uad50\uc218\uc758 \uc218\uc5c5\uc744 \ub4e4\uc5c8\uace0, \uc790\uc2e0\uc744 '\uc644\uc804\ud558\uac8c \uad50\uc721\uc744 \ubc1b\uc740' \ud559\uc704\uc788\ub294 \ubc8c\ub808\ub77c\uace0 \uc18c\uac1c\ud55c\ub2e4. \ub610\ud55c \uadf8\ub294 \uad50\uc218\uc5d0\uac8c \uc7a1\ud614\ub2e4\uac00 \uad50\uc7ac\uc6a9\uc73c\ub85c \ud655\ub300\uacbd \uc704\uc5d0 \uc62c\ub790\uc5c8\ub294\ub370, \uc774 \ub54c \ub3c4\ub9dd\uccd0 \uc9c0\uae08\ucc98\ub7fc \ud070 \ubab8\uc744 \uac00\uc838 '\ub300\ub2e8\ud788 \uc704\ub300\ud55c' \uc6cc\uae00\ubc8c\ub808\uac00 \ub418\uc5c8\ub2e4\ub294 \uac83\uc774\ub2e4. \uc0c8\ub85c\uc6b4 \uc77c\ud589\uc774 \ub41c \uc6cc\uae00\ubc8c\ub808\ub294 \ud559\uc2dd\uacfc \uad50\uc591\uc744 \uac16\ucd98 \uc9c0\uc2dd\uc778\uc774\ub77c\uba74 \ub18d\ub2f4\uc744 \ud1b5\ud574 \ucc9c\uc7ac\uc131\uc744 \ub4dc\ub7ec\ub0b8\ub2e4\uba70 \ub9d0\uc7a5\ub09c\uc744 \ud558\uc9c0\ub9cc \uc77c\ud589\uc740 \ubaa8\ub450 \ubd88\ucf8c\ud574 \ud55c\ub2e4."} {"question": "1917\ub144 \uc624\ub9ac\uac74 \ub18d\uc5c5 \ub300\ud559\uc758 \uc704\uce58\ub294?", "paragraph": "1917\ub144 \ud3f4\ub9c1\uc740 \ucf54\ubc1c\ub9ac\uc2a4(Corvallis)\uc5d0 \uc704\uce58\ud55c \ud604\uc7ac \uc624\ub9ac\uac74 \uc8fc\ub9bd \ub300\ud559\uc758 \uc804\uc2e0\uc778 \uc624\ub9ac\uac74 \ub18d\uc5c5 \ub300\ud559(the Oregon Agricultural College)\uc5d0 \ub4e4\uc5b4\uac14\ub2e4. \uadf8\ub7ec\ub098 \uacbd\uc81c\uc801 \uc5b4\ub824\uc6c0 \ub54c\ubb38\uc5d0 \uadf8\ub294 full-time \uc73c\ub85c \uc77c\uc744 \ud574\uc57c \ud588\ub2e4. \uadf8\ub294 \uc7a5\uc791\uc744 \ucabc\uac1c\uace0, \uccad\uc18c\ub97c \ud558\uace0, \uc5ec\ud559\uc0dd \ud68c\uad00\uc758 \uc8fc\ubc29\uc5d0\uc11c \uace0\uae30\ub97c \uc368\ub294 \ub4f1\uc758 \uc77c\uc744 \ud558\uc600\ub2e4. \ub610\ud55c 2\ud559\ub144 \uc5ec\ub984\ubc29\ud559\uc5d0\ub294 \uc544\uc2a4\ud314\ud2b8 \uc131\ubd84\uc744 \uc2dc\ud5d8\ud558\ub294 \uc77c\uc790\ub9ac\ub97c \uad6c\ud588\uace0 \uc4f0\uace0 \ub0a8\uc740 \ub3c8\uc744 \uc9d1\uc5d0 \ubcf4\ub0c8\ub2e4. \uadf8\ub7ec\ub098 2\ub144\uc758 \ud559\uc5c5\uc744 \ub9c8\uce58\uace0 \uc5b4\uba38\ub2c8 \ubca8\uc740 \uc544\ub4e4 \ud3f4\ub9c1\uc5d0\uac8c 1\ub144\ub3d9\uc548 \ud734\ud559\uc744 \ud558\uace0 \uc77c\uc744 \ub3c4\uc640\ub2ec\ub77c\uace0 \ud558\uc600\ub2e4. \uadf8\ub7f0 \uc0c1\ud669\uc5d0\uc11c \ud3f4\ub9c1\uc740 \ud654\ud559\uad50\uc218\ub85c\ubd80\ud130 1\ub144\ub3d9\uc548 \uc815\ub7c9\ud654\ud559\uc744 \uac00\ub974\uce58\ub294 \uc77c\uc790\ub9ac\ub97c \ubc1b\uac8c \ub418\uace0 \ud559\uad50\uc5d0 \uacc4\uc18d \ub0a8\uc544 \ub3c8\ub3c4 \ubc8c\uace0 \uacf5\ubd80\ub97c \uacc4\uc18d \ud560 \uc218 \uc788\uac8c \ub41c\ub2e4. \ud3f4\ub9c1\uc740 \uac15\uc758 \uc900\ube44\uc5d0 \ub9ce\uc740 \uc5f4\uc815\uc744 \uc3df\uc544\ubd80\uc5c8\uace0 \uadf8\uc758 \uac15\uc758\ub294 \ud559\uc0dd\ub4e4\uc5d0\uac8c \uc810\ucc28 \uc778\uae30\ub97c \uc5bb\uace0 \uc778\uc815\uc744 \ubc1b\uc558\ub2e4. 1922\ub144 \uaca8\uc6b8\ud559\uae30 \uac00\uc815\uacbd\uc81c\ud559\uacfc\uc758 \ud654\ud559\uc218\uc5c5\uc744 \ub9de\uac8c \ub41c \ud3f4\ub9c1\uc740 \uc218\uc5c5\uc5d0\uc11c \uc544\ubc14 \ud5ec\ub80c \ubc00\ub7ec\ub97c \ub9cc\ub098\uac8c \ub418\uace0, \ub458\uc740 1923\ub144 6\uc6d4 17\uc77c\uc5d0 \uacb0\ud63c\ud558\uac8c \ub41c\ub2e4.", "answer": "\ucf54\ubc1c\ub9ac\uc2a4", "sentence": "1917\ub144 \ud3f4\ub9c1\uc740 \ucf54\ubc1c\ub9ac\uc2a4 (Corvallis)", "paragraph_sentence": " 1917\ub144 \ud3f4\ub9c1\uc740 \ucf54\ubc1c\ub9ac\uc2a4 (Corvallis) \uc5d0 \uc704\uce58\ud55c \ud604\uc7ac \uc624\ub9ac\uac74 \uc8fc\ub9bd \ub300\ud559\uc758 \uc804\uc2e0\uc778 \uc624\ub9ac\uac74 \ub18d\uc5c5 \ub300\ud559(the Oregon Agricultural College)\uc5d0 \ub4e4\uc5b4\uac14\ub2e4. \uadf8\ub7ec\ub098 \uacbd\uc81c\uc801 \uc5b4\ub824\uc6c0 \ub54c\ubb38\uc5d0 \uadf8\ub294 full-time \uc73c\ub85c \uc77c\uc744 \ud574\uc57c \ud588\ub2e4. \uadf8\ub294 \uc7a5\uc791\uc744 \ucabc\uac1c\uace0, \uccad\uc18c\ub97c \ud558\uace0, \uc5ec\ud559\uc0dd \ud68c\uad00\uc758 \uc8fc\ubc29\uc5d0\uc11c \uace0\uae30\ub97c \uc368\ub294 \ub4f1\uc758 \uc77c\uc744 \ud558\uc600\ub2e4. \ub610\ud55c 2\ud559\ub144 \uc5ec\ub984\ubc29\ud559\uc5d0\ub294 \uc544\uc2a4\ud314\ud2b8 \uc131\ubd84\uc744 \uc2dc\ud5d8\ud558\ub294 \uc77c\uc790\ub9ac\ub97c \uad6c\ud588\uace0 \uc4f0\uace0 \ub0a8\uc740 \ub3c8\uc744 \uc9d1\uc5d0 \ubcf4\ub0c8\ub2e4. \uadf8\ub7ec\ub098 2\ub144\uc758 \ud559\uc5c5\uc744 \ub9c8\uce58\uace0 \uc5b4\uba38\ub2c8 \ubca8\uc740 \uc544\ub4e4 \ud3f4\ub9c1\uc5d0\uac8c 1\ub144\ub3d9\uc548 \ud734\ud559\uc744 \ud558\uace0 \uc77c\uc744 \ub3c4\uc640\ub2ec\ub77c\uace0 \ud558\uc600\ub2e4. \uadf8\ub7f0 \uc0c1\ud669\uc5d0\uc11c \ud3f4\ub9c1\uc740 \ud654\ud559\uad50\uc218\ub85c\ubd80\ud130 1\ub144\ub3d9\uc548 \uc815\ub7c9\ud654\ud559\uc744 \uac00\ub974\uce58\ub294 \uc77c\uc790\ub9ac\ub97c \ubc1b\uac8c \ub418\uace0 \ud559\uad50\uc5d0 \uacc4\uc18d \ub0a8\uc544 \ub3c8\ub3c4 \ubc8c\uace0 \uacf5\ubd80\ub97c \uacc4\uc18d \ud560 \uc218 \uc788\uac8c \ub41c\ub2e4. \ud3f4\ub9c1\uc740 \uac15\uc758 \uc900\ube44\uc5d0 \ub9ce\uc740 \uc5f4\uc815\uc744 \uc3df\uc544\ubd80\uc5c8\uace0 \uadf8\uc758 \uac15\uc758\ub294 \ud559\uc0dd\ub4e4\uc5d0\uac8c \uc810\ucc28 \uc778\uae30\ub97c \uc5bb\uace0 \uc778\uc815\uc744 \ubc1b\uc558\ub2e4. 1922\ub144 \uaca8\uc6b8\ud559\uae30 \uac00\uc815\uacbd\uc81c\ud559\uacfc\uc758 \ud654\ud559\uc218\uc5c5\uc744 \ub9de\uac8c \ub41c \ud3f4\ub9c1\uc740 \uc218\uc5c5\uc5d0\uc11c \uc544\ubc14 \ud5ec\ub80c \ubc00\ub7ec\ub97c \ub9cc\ub098\uac8c \ub418\uace0, \ub458\uc740 1923\ub144 6\uc6d4 17\uc77c\uc5d0 \uacb0\ud63c\ud558\uac8c \ub41c\ub2e4.", "paragraph_answer": "1917\ub144 \ud3f4\ub9c1\uc740 \ucf54\ubc1c\ub9ac\uc2a4 (Corvallis)\uc5d0 \uc704\uce58\ud55c \ud604\uc7ac \uc624\ub9ac\uac74 \uc8fc\ub9bd \ub300\ud559\uc758 \uc804\uc2e0\uc778 \uc624\ub9ac\uac74 \ub18d\uc5c5 \ub300\ud559(the Oregon Agricultural College)\uc5d0 \ub4e4\uc5b4\uac14\ub2e4. \uadf8\ub7ec\ub098 \uacbd\uc81c\uc801 \uc5b4\ub824\uc6c0 \ub54c\ubb38\uc5d0 \uadf8\ub294 full-time \uc73c\ub85c \uc77c\uc744 \ud574\uc57c \ud588\ub2e4. \uadf8\ub294 \uc7a5\uc791\uc744 \ucabc\uac1c\uace0, \uccad\uc18c\ub97c \ud558\uace0, \uc5ec\ud559\uc0dd \ud68c\uad00\uc758 \uc8fc\ubc29\uc5d0\uc11c \uace0\uae30\ub97c \uc368\ub294 \ub4f1\uc758 \uc77c\uc744 \ud558\uc600\ub2e4. \ub610\ud55c 2\ud559\ub144 \uc5ec\ub984\ubc29\ud559\uc5d0\ub294 \uc544\uc2a4\ud314\ud2b8 \uc131\ubd84\uc744 \uc2dc\ud5d8\ud558\ub294 \uc77c\uc790\ub9ac\ub97c \uad6c\ud588\uace0 \uc4f0\uace0 \ub0a8\uc740 \ub3c8\uc744 \uc9d1\uc5d0 \ubcf4\ub0c8\ub2e4. \uadf8\ub7ec\ub098 2\ub144\uc758 \ud559\uc5c5\uc744 \ub9c8\uce58\uace0 \uc5b4\uba38\ub2c8 \ubca8\uc740 \uc544\ub4e4 \ud3f4\ub9c1\uc5d0\uac8c 1\ub144\ub3d9\uc548 \ud734\ud559\uc744 \ud558\uace0 \uc77c\uc744 \ub3c4\uc640\ub2ec\ub77c\uace0 \ud558\uc600\ub2e4. \uadf8\ub7f0 \uc0c1\ud669\uc5d0\uc11c \ud3f4\ub9c1\uc740 \ud654\ud559\uad50\uc218\ub85c\ubd80\ud130 1\ub144\ub3d9\uc548 \uc815\ub7c9\ud654\ud559\uc744 \uac00\ub974\uce58\ub294 \uc77c\uc790\ub9ac\ub97c \ubc1b\uac8c \ub418\uace0 \ud559\uad50\uc5d0 \uacc4\uc18d \ub0a8\uc544 \ub3c8\ub3c4 \ubc8c\uace0 \uacf5\ubd80\ub97c \uacc4\uc18d \ud560 \uc218 \uc788\uac8c \ub41c\ub2e4. \ud3f4\ub9c1\uc740 \uac15\uc758 \uc900\ube44\uc5d0 \ub9ce\uc740 \uc5f4\uc815\uc744 \uc3df\uc544\ubd80\uc5c8\uace0 \uadf8\uc758 \uac15\uc758\ub294 \ud559\uc0dd\ub4e4\uc5d0\uac8c \uc810\ucc28 \uc778\uae30\ub97c \uc5bb\uace0 \uc778\uc815\uc744 \ubc1b\uc558\ub2e4. 1922\ub144 \uaca8\uc6b8\ud559\uae30 \uac00\uc815\uacbd\uc81c\ud559\uacfc\uc758 \ud654\ud559\uc218\uc5c5\uc744 \ub9de\uac8c \ub41c \ud3f4\ub9c1\uc740 \uc218\uc5c5\uc5d0\uc11c \uc544\ubc14 \ud5ec\ub80c \ubc00\ub7ec\ub97c \ub9cc\ub098\uac8c \ub418\uace0, \ub458\uc740 1923\ub144 6\uc6d4 17\uc77c\uc5d0 \uacb0\ud63c\ud558\uac8c \ub41c\ub2e4.", "sentence_answer": "1917\ub144 \ud3f4\ub9c1\uc740 \ucf54\ubc1c\ub9ac\uc2a4 (Corvallis)", "paragraph_id": "6579230-6-0", "paragraph_question": "question: 1917\ub144 \uc624\ub9ac\uac74 \ub18d\uc5c5 \ub300\ud559\uc758 \uc704\uce58\ub294?, context: 1917\ub144 \ud3f4\ub9c1\uc740 \ucf54\ubc1c\ub9ac\uc2a4(Corvallis)\uc5d0 \uc704\uce58\ud55c \ud604\uc7ac \uc624\ub9ac\uac74 \uc8fc\ub9bd \ub300\ud559\uc758 \uc804\uc2e0\uc778 \uc624\ub9ac\uac74 \ub18d\uc5c5 \ub300\ud559(the Oregon Agricultural College)\uc5d0 \ub4e4\uc5b4\uac14\ub2e4. \uadf8\ub7ec\ub098 \uacbd\uc81c\uc801 \uc5b4\ub824\uc6c0 \ub54c\ubb38\uc5d0 \uadf8\ub294 full-time \uc73c\ub85c \uc77c\uc744 \ud574\uc57c \ud588\ub2e4. \uadf8\ub294 \uc7a5\uc791\uc744 \ucabc\uac1c\uace0, \uccad\uc18c\ub97c \ud558\uace0, \uc5ec\ud559\uc0dd \ud68c\uad00\uc758 \uc8fc\ubc29\uc5d0\uc11c \uace0\uae30\ub97c \uc368\ub294 \ub4f1\uc758 \uc77c\uc744 \ud558\uc600\ub2e4. \ub610\ud55c 2\ud559\ub144 \uc5ec\ub984\ubc29\ud559\uc5d0\ub294 \uc544\uc2a4\ud314\ud2b8 \uc131\ubd84\uc744 \uc2dc\ud5d8\ud558\ub294 \uc77c\uc790\ub9ac\ub97c \uad6c\ud588\uace0 \uc4f0\uace0 \ub0a8\uc740 \ub3c8\uc744 \uc9d1\uc5d0 \ubcf4\ub0c8\ub2e4. \uadf8\ub7ec\ub098 2\ub144\uc758 \ud559\uc5c5\uc744 \ub9c8\uce58\uace0 \uc5b4\uba38\ub2c8 \ubca8\uc740 \uc544\ub4e4 \ud3f4\ub9c1\uc5d0\uac8c 1\ub144\ub3d9\uc548 \ud734\ud559\uc744 \ud558\uace0 \uc77c\uc744 \ub3c4\uc640\ub2ec\ub77c\uace0 \ud558\uc600\ub2e4. \uadf8\ub7f0 \uc0c1\ud669\uc5d0\uc11c \ud3f4\ub9c1\uc740 \ud654\ud559\uad50\uc218\ub85c\ubd80\ud130 1\ub144\ub3d9\uc548 \uc815\ub7c9\ud654\ud559\uc744 \uac00\ub974\uce58\ub294 \uc77c\uc790\ub9ac\ub97c \ubc1b\uac8c \ub418\uace0 \ud559\uad50\uc5d0 \uacc4\uc18d \ub0a8\uc544 \ub3c8\ub3c4 \ubc8c\uace0 \uacf5\ubd80\ub97c \uacc4\uc18d \ud560 \uc218 \uc788\uac8c \ub41c\ub2e4. \ud3f4\ub9c1\uc740 \uac15\uc758 \uc900\ube44\uc5d0 \ub9ce\uc740 \uc5f4\uc815\uc744 \uc3df\uc544\ubd80\uc5c8\uace0 \uadf8\uc758 \uac15\uc758\ub294 \ud559\uc0dd\ub4e4\uc5d0\uac8c \uc810\ucc28 \uc778\uae30\ub97c \uc5bb\uace0 \uc778\uc815\uc744 \ubc1b\uc558\ub2e4. 1922\ub144 \uaca8\uc6b8\ud559\uae30 \uac00\uc815\uacbd\uc81c\ud559\uacfc\uc758 \ud654\ud559\uc218\uc5c5\uc744 \ub9de\uac8c \ub41c \ud3f4\ub9c1\uc740 \uc218\uc5c5\uc5d0\uc11c \uc544\ubc14 \ud5ec\ub80c \ubc00\ub7ec\ub97c \ub9cc\ub098\uac8c \ub418\uace0, \ub458\uc740 1923\ub144 6\uc6d4 17\uc77c\uc5d0 \uacb0\ud63c\ud558\uac8c \ub41c\ub2e4."} {"question": "\ud649\ud0a8\uacfc \ud5c9\uc2ac\ub9ac\uac00 \uc218\uc0c1\ud55c \uc0c1 \uc774\ub984\uc740?", "paragraph": "\uc548\ucca0\uc218\uc758 \ubc15\uc0ac\ud559\uc704 \ub17c\ubb38\uc774 \uc548\ucca0\uc218 \ub17c\ubb38\uc5d0 \ube44\ud574 2\ub144 \uc55e\uc11c \ubc15\uc0ac\ud559\uc704\ub97c \ubc1b\uc740 \uc11c\uc6b8\ub300\ud559\uad50 \uc11c \ubaa8 \uad50\uc218\uc758 \ubc15\uc0ac \ub17c\ubb38\uc744 \ud45c\uc808\ud588\ub2e4\ub294 \uc758\ud639\uc774 \uc788\ub2e4. \ubcfc\uce20\ub9cc \uace1\uc120\uc744 \uc720\ub3c4\ud558\ub294 \uc124\uba85\uc5d0\uc11c \uc720\ub3c4\uc2dd\uc744 \uc11c \uad50\uc218 \ub17c\ubb38\uc5d0\uc11c \uac70\uc758 \ubcf5\uc0ac \uc218\uc900\uc73c\ub85c 3\ud398\uc774\uc9c0\uc5d0 \uac78\uccd0 \ubca0\uaf08\ub2e4\ub294 \uac83\uc774\ub2e4. \ub354\uad70\ub2e4\ub098 \uc11c \uad50\uc218\uc758 \ub17c\ubb38\uc5d0\uc11c \uae30\uc220\ud55c \ubcfc\uce20\ub9cc \uacf5\uc2dd\uc740 \ub300\uad04\ud638\uac00 \ud558\ub098 \ube60\uc838\uc11c \uc624\ub958\uac00 \uc788\ub294 \uacf5\uc2dd\uc778\ub370, \uacf5\uad50\ub86d\uac8c\ub3c4 \uc548\ucca0\uc218\uc758 \ub17c\ubb38\uc5d0\uc11c\ub3c4 \uac19\uc740 \uacf5\uc2dd\uc5d0 \ub300\uad04\ud638\uac00 \ube60\uc838 \uc788\uc5c8\ub2e4. \ub530\ub77c\uc11c \ub17c\ubb38\uc744 \ubca0\ub07c\ub2e4\uac00 \uc624\ub958\uae4c\uc9c0 \ubca0\uaf08\ub2e4\ub294 \uc758\ud639\uc774 \uc788\ub2e4. \uadf8\ub7ec\ub098 \uc548\ucca0\uc218 \uce21\uc740 \ubcfc\uce20\ub9cc \uacf5\uc2dd\uc740 \ubcf4\ud3b8\uc801\uc73c\ub85c \uc54c\ub824\uc9c4 \uacf5\uc2dd\uc774\ub77c\uba70 \ub178\ubca8\uc0c1\uc744 \uc218\uc0c1\ud55c \ud649\ud0a8\uacfc \ud5c9\uc2ac\ub9ac\uc758 \ub17c\ubb38\uc5d0\uc11c\ub3c4 \ubcfc\uce20\ub9cc \uacf5\uc2dd\uc744 \uc0ac\uc6a9\ud558\ub294\ub370 \uc788\uc5b4\uc11c \uc6d0\uc800\ub97c \ubc1d\ud788\uc9c0 \uc54a\uc558\uace0, \uadf8\uac83\uc774 \uad00\ud589\uc774\ub77c\uace0 \ubc18\ubc15\ud558\uc600\ub2e4.", "answer": "\ub178\ubca8\uc0c1", "sentence": "\uadf8\ub7ec\ub098 \uc548\ucca0\uc218 \uce21\uc740 \ubcfc\uce20\ub9cc \uacf5\uc2dd\uc740 \ubcf4\ud3b8\uc801\uc73c\ub85c \uc54c\ub824\uc9c4 \uacf5\uc2dd\uc774\ub77c\uba70 \ub178\ubca8\uc0c1 \uc744 \uc218\uc0c1\ud55c \ud649\ud0a8\uacfc \ud5c9\uc2ac\ub9ac\uc758 \ub17c\ubb38\uc5d0\uc11c\ub3c4 \ubcfc\uce20\ub9cc \uacf5\uc2dd\uc744 \uc0ac\uc6a9\ud558\ub294\ub370 \uc788\uc5b4\uc11c \uc6d0\uc800\ub97c \ubc1d\ud788\uc9c0 \uc54a\uc558\uace0, \uadf8\uac83\uc774 \uad00\ud589\uc774\ub77c\uace0 \ubc18\ubc15\ud558\uc600\ub2e4.", "paragraph_sentence": "\uc548\ucca0\uc218\uc758 \ubc15\uc0ac\ud559\uc704 \ub17c\ubb38\uc774 \uc548\ucca0\uc218 \ub17c\ubb38\uc5d0 \ube44\ud574 2\ub144 \uc55e\uc11c \ubc15\uc0ac\ud559\uc704\ub97c \ubc1b\uc740 \uc11c\uc6b8\ub300\ud559\uad50 \uc11c \ubaa8 \uad50\uc218\uc758 \ubc15\uc0ac \ub17c\ubb38\uc744 \ud45c\uc808\ud588\ub2e4\ub294 \uc758\ud639\uc774 \uc788\ub2e4. \ubcfc\uce20\ub9cc \uace1\uc120\uc744 \uc720\ub3c4\ud558\ub294 \uc124\uba85\uc5d0\uc11c \uc720\ub3c4\uc2dd\uc744 \uc11c \uad50\uc218 \ub17c\ubb38\uc5d0\uc11c \uac70\uc758 \ubcf5\uc0ac \uc218\uc900\uc73c\ub85c 3\ud398\uc774\uc9c0\uc5d0 \uac78\uccd0 \ubca0\uaf08\ub2e4\ub294 \uac83\uc774\ub2e4. \ub354\uad70\ub2e4\ub098 \uc11c \uad50\uc218\uc758 \ub17c\ubb38\uc5d0\uc11c \uae30\uc220\ud55c \ubcfc\uce20\ub9cc \uacf5\uc2dd\uc740 \ub300\uad04\ud638\uac00 \ud558\ub098 \ube60\uc838\uc11c \uc624\ub958\uac00 \uc788\ub294 \uacf5\uc2dd\uc778\ub370, \uacf5\uad50\ub86d\uac8c\ub3c4 \uc548\ucca0\uc218\uc758 \ub17c\ubb38\uc5d0\uc11c\ub3c4 \uac19\uc740 \uacf5\uc2dd\uc5d0 \ub300\uad04\ud638\uac00 \ube60\uc838 \uc788\uc5c8\ub2e4. \ub530\ub77c\uc11c \ub17c\ubb38\uc744 \ubca0\ub07c\ub2e4\uac00 \uc624\ub958\uae4c\uc9c0 \ubca0\uaf08\ub2e4\ub294 \uc758\ud639\uc774 \uc788\ub2e4. \uadf8\ub7ec\ub098 \uc548\ucca0\uc218 \uce21\uc740 \ubcfc\uce20\ub9cc \uacf5\uc2dd\uc740 \ubcf4\ud3b8\uc801\uc73c\ub85c \uc54c\ub824\uc9c4 \uacf5\uc2dd\uc774\ub77c\uba70 \ub178\ubca8\uc0c1 \uc744 \uc218\uc0c1\ud55c \ud649\ud0a8\uacfc \ud5c9\uc2ac\ub9ac\uc758 \ub17c\ubb38\uc5d0\uc11c\ub3c4 \ubcfc\uce20\ub9cc \uacf5\uc2dd\uc744 \uc0ac\uc6a9\ud558\ub294\ub370 \uc788\uc5b4\uc11c \uc6d0\uc800\ub97c \ubc1d\ud788\uc9c0 \uc54a\uc558\uace0, \uadf8\uac83\uc774 \uad00\ud589\uc774\ub77c\uace0 \ubc18\ubc15\ud558\uc600\ub2e4. ", "paragraph_answer": "\uc548\ucca0\uc218\uc758 \ubc15\uc0ac\ud559\uc704 \ub17c\ubb38\uc774 \uc548\ucca0\uc218 \ub17c\ubb38\uc5d0 \ube44\ud574 2\ub144 \uc55e\uc11c \ubc15\uc0ac\ud559\uc704\ub97c \ubc1b\uc740 \uc11c\uc6b8\ub300\ud559\uad50 \uc11c \ubaa8 \uad50\uc218\uc758 \ubc15\uc0ac \ub17c\ubb38\uc744 \ud45c\uc808\ud588\ub2e4\ub294 \uc758\ud639\uc774 \uc788\ub2e4. \ubcfc\uce20\ub9cc \uace1\uc120\uc744 \uc720\ub3c4\ud558\ub294 \uc124\uba85\uc5d0\uc11c \uc720\ub3c4\uc2dd\uc744 \uc11c \uad50\uc218 \ub17c\ubb38\uc5d0\uc11c \uac70\uc758 \ubcf5\uc0ac \uc218\uc900\uc73c\ub85c 3\ud398\uc774\uc9c0\uc5d0 \uac78\uccd0 \ubca0\uaf08\ub2e4\ub294 \uac83\uc774\ub2e4. \ub354\uad70\ub2e4\ub098 \uc11c \uad50\uc218\uc758 \ub17c\ubb38\uc5d0\uc11c \uae30\uc220\ud55c \ubcfc\uce20\ub9cc \uacf5\uc2dd\uc740 \ub300\uad04\ud638\uac00 \ud558\ub098 \ube60\uc838\uc11c \uc624\ub958\uac00 \uc788\ub294 \uacf5\uc2dd\uc778\ub370, \uacf5\uad50\ub86d\uac8c\ub3c4 \uc548\ucca0\uc218\uc758 \ub17c\ubb38\uc5d0\uc11c\ub3c4 \uac19\uc740 \uacf5\uc2dd\uc5d0 \ub300\uad04\ud638\uac00 \ube60\uc838 \uc788\uc5c8\ub2e4. \ub530\ub77c\uc11c \ub17c\ubb38\uc744 \ubca0\ub07c\ub2e4\uac00 \uc624\ub958\uae4c\uc9c0 \ubca0\uaf08\ub2e4\ub294 \uc758\ud639\uc774 \uc788\ub2e4. \uadf8\ub7ec\ub098 \uc548\ucca0\uc218 \uce21\uc740 \ubcfc\uce20\ub9cc \uacf5\uc2dd\uc740 \ubcf4\ud3b8\uc801\uc73c\ub85c \uc54c\ub824\uc9c4 \uacf5\uc2dd\uc774\ub77c\uba70 \ub178\ubca8\uc0c1 \uc744 \uc218\uc0c1\ud55c \ud649\ud0a8\uacfc \ud5c9\uc2ac\ub9ac\uc758 \ub17c\ubb38\uc5d0\uc11c\ub3c4 \ubcfc\uce20\ub9cc \uacf5\uc2dd\uc744 \uc0ac\uc6a9\ud558\ub294\ub370 \uc788\uc5b4\uc11c \uc6d0\uc800\ub97c \ubc1d\ud788\uc9c0 \uc54a\uc558\uace0, \uadf8\uac83\uc774 \uad00\ud589\uc774\ub77c\uace0 \ubc18\ubc15\ud558\uc600\ub2e4.", "sentence_answer": "\uadf8\ub7ec\ub098 \uc548\ucca0\uc218 \uce21\uc740 \ubcfc\uce20\ub9cc \uacf5\uc2dd\uc740 \ubcf4\ud3b8\uc801\uc73c\ub85c \uc54c\ub824\uc9c4 \uacf5\uc2dd\uc774\ub77c\uba70 \ub178\ubca8\uc0c1 \uc744 \uc218\uc0c1\ud55c \ud649\ud0a8\uacfc \ud5c9\uc2ac\ub9ac\uc758 \ub17c\ubb38\uc5d0\uc11c\ub3c4 \ubcfc\uce20\ub9cc \uacf5\uc2dd\uc744 \uc0ac\uc6a9\ud558\ub294\ub370 \uc788\uc5b4\uc11c \uc6d0\uc800\ub97c \ubc1d\ud788\uc9c0 \uc54a\uc558\uace0, \uadf8\uac83\uc774 \uad00\ud589\uc774\ub77c\uace0 \ubc18\ubc15\ud558\uc600\ub2e4.", "paragraph_id": "6547507-25-1", "paragraph_question": "question: \ud649\ud0a8\uacfc \ud5c9\uc2ac\ub9ac\uac00 \uc218\uc0c1\ud55c \uc0c1 \uc774\ub984\uc740?, context: \uc548\ucca0\uc218\uc758 \ubc15\uc0ac\ud559\uc704 \ub17c\ubb38\uc774 \uc548\ucca0\uc218 \ub17c\ubb38\uc5d0 \ube44\ud574 2\ub144 \uc55e\uc11c \ubc15\uc0ac\ud559\uc704\ub97c \ubc1b\uc740 \uc11c\uc6b8\ub300\ud559\uad50 \uc11c \ubaa8 \uad50\uc218\uc758 \ubc15\uc0ac \ub17c\ubb38\uc744 \ud45c\uc808\ud588\ub2e4\ub294 \uc758\ud639\uc774 \uc788\ub2e4. \ubcfc\uce20\ub9cc \uace1\uc120\uc744 \uc720\ub3c4\ud558\ub294 \uc124\uba85\uc5d0\uc11c \uc720\ub3c4\uc2dd\uc744 \uc11c \uad50\uc218 \ub17c\ubb38\uc5d0\uc11c \uac70\uc758 \ubcf5\uc0ac \uc218\uc900\uc73c\ub85c 3\ud398\uc774\uc9c0\uc5d0 \uac78\uccd0 \ubca0\uaf08\ub2e4\ub294 \uac83\uc774\ub2e4. \ub354\uad70\ub2e4\ub098 \uc11c \uad50\uc218\uc758 \ub17c\ubb38\uc5d0\uc11c \uae30\uc220\ud55c \ubcfc\uce20\ub9cc \uacf5\uc2dd\uc740 \ub300\uad04\ud638\uac00 \ud558\ub098 \ube60\uc838\uc11c \uc624\ub958\uac00 \uc788\ub294 \uacf5\uc2dd\uc778\ub370, \uacf5\uad50\ub86d\uac8c\ub3c4 \uc548\ucca0\uc218\uc758 \ub17c\ubb38\uc5d0\uc11c\ub3c4 \uac19\uc740 \uacf5\uc2dd\uc5d0 \ub300\uad04\ud638\uac00 \ube60\uc838 \uc788\uc5c8\ub2e4. \ub530\ub77c\uc11c \ub17c\ubb38\uc744 \ubca0\ub07c\ub2e4\uac00 \uc624\ub958\uae4c\uc9c0 \ubca0\uaf08\ub2e4\ub294 \uc758\ud639\uc774 \uc788\ub2e4. \uadf8\ub7ec\ub098 \uc548\ucca0\uc218 \uce21\uc740 \ubcfc\uce20\ub9cc \uacf5\uc2dd\uc740 \ubcf4\ud3b8\uc801\uc73c\ub85c \uc54c\ub824\uc9c4 \uacf5\uc2dd\uc774\ub77c\uba70 \ub178\ubca8\uc0c1\uc744 \uc218\uc0c1\ud55c \ud649\ud0a8\uacfc \ud5c9\uc2ac\ub9ac\uc758 \ub17c\ubb38\uc5d0\uc11c\ub3c4 \ubcfc\uce20\ub9cc \uacf5\uc2dd\uc744 \uc0ac\uc6a9\ud558\ub294\ub370 \uc788\uc5b4\uc11c \uc6d0\uc800\ub97c \ubc1d\ud788\uc9c0 \uc54a\uc558\uace0, \uadf8\uac83\uc774 \uad00\ud589\uc774\ub77c\uace0 \ubc18\ubc15\ud558\uc600\ub2e4."} {"question": "12\uc138\uae30 \uacbd \uc5ec\ub7f7 \uc218\ub3c4\uc6d0\ub4e4\uc740 \uc5b4\ub514\uc5d0 \uc788\uc5c8\uc744\uae4c?", "paragraph": "\uc870\uc9c0\uc544 \uc655\uc2e4\uc740 \uba3c\uc800 \uc131\uc9c0\uc5d0 \uc788\ub294 \uc870\uc9c0\uc544\uc758 \uc218\ub3c4\uc6d0 \uc911\uc2ec\uc9c0\ub4e4\uc758 \ubcf4\ud638\uc5d0 \ub300\ud574 \uad00\uc2ec\uc744 \uae30\uc6b8\ub838\ub2e4. 12\uc138\uae30 \uacbd\uc5d0\ub294, \uc5ec\ub35f \uc218\ub3c4\uc6d0\ub4e4\uc774 \uc81c\ub8e8\uc0b4\ub818\uc5d0 \uc788\uc5c8\ub2e4. \uc0b4\ub77c\ub518\uc758 \uc804\uae30 \uc791\uac00 \ubc14\ud558 \uc544\ub4dc-\ub518 \uc774\ube10 \uc0e4\ub4dc\ub2e4\ub4dc\uc758 \ubcf4\uace0\uc5d0 \ub530\ub974\uba74, 1187\ub144\uc5d0 \uc544\uc774\uc720\ube0c\uc870\uac00 \uc608\ub8e8\uc0b4\ub818\uc744 \uc815\ubcf5\ud55c \ud6c4\uc5d0, \ud0c0\ub9c8\ub974\uac00 \uc220\ud0c4\uc5d0\uac8c \uc678\uad50 \uc0ac\uc808\uc744 \ubcf4\ub0b4 \uc608\ub8e8\uc0b4\ub818\uc5d0 \uc788\ub294 \uadf8\ub4e4\uc774 \ubab0\uc218\ud55c \uc870\uc9c0\uc544\uc758 \uc218\ub3c4\uc6d0\ub4e4\uc758 \uc18c\uc720\uc9c0\ub4e4\uc744 \ubc18\ud658\ud560 \uac83\uc744 \uc694\uad6c\ud588\ub2e4. \uc0b4\ub77c\ub518\uc758 \ub300\uafb8\ub294 \uc5c6\uc5c8\uc9c0\ub9cc, \uc5ec\uc655\uc73c\uc758 \ub178\ub825\uc740 \uc131\uacf5\uc744 \uac70\ub450\uc5b4 \uac14\ub2e4. \ud0c0\ub9c8\ub974\uac00 \uc8fd\uc740 \ud6c4\uc5d0 \ub2e8\uae30\uac04\uc758 \uc544\ud06c\ub808\uc758 \uc8fc\uad50\ub85c \uc788\ub294 \uc790\ucfe0\uc5d0\uc2a4 \ub370 \ube44\ud2b8\ub9ac\ub294 \uc608\ub8e8\uc0b4\ub818\uc5d0\uc11c\uc758 \uc870\uc9c0\uc544\uc758 \uc874\uc7ac\uc5d0 \ub300\ud558\uc5ec \ub354 \ub098\uc740 \uadfc\uac70\ub97c \uc81c\uc2dc\ud588\ub2e4. \uadf8\ub294 \ub2e4\ub978 \uadf8\ub9ac\uc2a4\ub3c4\uad50 \uc21c\ub840\uc790\ub4e4\uacfc\ub294 \ub300\uc870\ub418\uac8c \uc870\uc9c0\uc544\uc778\ub4e4\uc740 \uadf8\ub4e4\uc758 \uae43\ubc1c\uc744 \ud718\ub0a0\ub9ac\uba70 \ub3c4\uc2dc\ub85c\uc758 \uc790\uc720\ub85c\uc6b4 \uc655\ub798\uac00 \ud5c8\uc6a9\ub418\uc5c8\uc5c8\ub2e4\uace0 \uae30\uc220\ud588\ub2e4. \ub354\uad70\ub2e4\ub098 \uc774\ube10 \uc0ac\ub4dc\ub2e4\ub4dc\ub294 \ud558\ud2f4 \uc804\ud22c\uc5d0\uc11c \uc57d\ud0c8\ud55c \uc720\ubb3c\uc744 \uac00\uc9c0\uace0 \uc788\ub358 \uc0b4\ub77c\ub518\uc5d0\uac8c \ud0c0\ub9c8\ub974\uac00 \uae08\uad34 20\ub9cc \ucabd\uc744 \ud5cc\ub0a9\ud560 \ub9cc\ud07c \uc131\uc2ed\uc790\uac00\uc758 \uc720\ubb3c\uc744 \ud68d\ub4dd\ud558\ub824\ub294 \uadf8\ub140\uc758 \ub178\ub825\uc774 \ube44\uc794\ud2f0\uc6c0 \ud669\uc81c\ub97c \ub2a5\uac00\ud588\ub2e4\uace0 \uc8fc\uc7a5\ud588\ub2e4\uace0 \ud588\ub2e4. \uadf8\ub7ec\ub098, \uadf8\uc758 \ub178\ub825\uc740 \ud5c8\uc0ac\ub85c \ub3cc\uc544\uac14\ub2e4.", "answer": "\uc81c\ub8e8\uc0b4\ub818", "sentence": "12\uc138\uae30 \uacbd\uc5d0\ub294, \uc5ec\ub35f \uc218\ub3c4\uc6d0\ub4e4\uc774 \uc81c\ub8e8\uc0b4\ub818 \uc5d0 \uc788\uc5c8\ub2e4.", "paragraph_sentence": "\uc870\uc9c0\uc544 \uc655\uc2e4\uc740 \uba3c\uc800 \uc131\uc9c0\uc5d0 \uc788\ub294 \uc870\uc9c0\uc544\uc758 \uc218\ub3c4\uc6d0 \uc911\uc2ec\uc9c0\ub4e4\uc758 \ubcf4\ud638\uc5d0 \ub300\ud574 \uad00\uc2ec\uc744 \uae30\uc6b8\ub838\ub2e4. 12\uc138\uae30 \uacbd\uc5d0\ub294, \uc5ec\ub35f \uc218\ub3c4\uc6d0\ub4e4\uc774 \uc81c\ub8e8\uc0b4\ub818 \uc5d0 \uc788\uc5c8\ub2e4. \uc0b4\ub77c\ub518\uc758 \uc804\uae30 \uc791\uac00 \ubc14\ud558 \uc544\ub4dc-\ub518 \uc774\ube10 \uc0e4\ub4dc\ub2e4\ub4dc\uc758 \ubcf4\uace0\uc5d0 \ub530\ub974\uba74, 1187\ub144\uc5d0 \uc544\uc774\uc720\ube0c\uc870\uac00 \uc608\ub8e8\uc0b4\ub818\uc744 \uc815\ubcf5\ud55c \ud6c4\uc5d0, \ud0c0\ub9c8\ub974\uac00 \uc220\ud0c4\uc5d0\uac8c \uc678\uad50 \uc0ac\uc808\uc744 \ubcf4\ub0b4 \uc608\ub8e8\uc0b4\ub818\uc5d0 \uc788\ub294 \uadf8\ub4e4\uc774 \ubab0\uc218\ud55c \uc870\uc9c0\uc544\uc758 \uc218\ub3c4\uc6d0\ub4e4\uc758 \uc18c\uc720\uc9c0\ub4e4\uc744 \ubc18\ud658\ud560 \uac83\uc744 \uc694\uad6c\ud588\ub2e4. \uc0b4\ub77c\ub518\uc758 \ub300\uafb8\ub294 \uc5c6\uc5c8\uc9c0\ub9cc, \uc5ec\uc655\uc73c\uc758 \ub178\ub825\uc740 \uc131\uacf5\uc744 \uac70\ub450\uc5b4 \uac14\ub2e4. \ud0c0\ub9c8\ub974\uac00 \uc8fd\uc740 \ud6c4\uc5d0 \ub2e8\uae30\uac04\uc758 \uc544\ud06c\ub808\uc758 \uc8fc\uad50\ub85c \uc788\ub294 \uc790\ucfe0\uc5d0\uc2a4 \ub370 \ube44\ud2b8\ub9ac\ub294 \uc608\ub8e8\uc0b4\ub818\uc5d0\uc11c\uc758 \uc870\uc9c0\uc544\uc758 \uc874\uc7ac\uc5d0 \ub300\ud558\uc5ec \ub354 \ub098\uc740 \uadfc\uac70\ub97c \uc81c\uc2dc\ud588\ub2e4. \uadf8\ub294 \ub2e4\ub978 \uadf8\ub9ac\uc2a4\ub3c4\uad50 \uc21c\ub840\uc790\ub4e4\uacfc\ub294 \ub300\uc870\ub418\uac8c \uc870\uc9c0\uc544\uc778\ub4e4\uc740 \uadf8\ub4e4\uc758 \uae43\ubc1c\uc744 \ud718\ub0a0\ub9ac\uba70 \ub3c4\uc2dc\ub85c\uc758 \uc790\uc720\ub85c\uc6b4 \uc655\ub798\uac00 \ud5c8\uc6a9\ub418\uc5c8\uc5c8\ub2e4\uace0 \uae30\uc220\ud588\ub2e4. \ub354\uad70\ub2e4\ub098 \uc774\ube10 \uc0ac\ub4dc\ub2e4\ub4dc\ub294 \ud558\ud2f4 \uc804\ud22c\uc5d0\uc11c \uc57d\ud0c8\ud55c \uc720\ubb3c\uc744 \uac00\uc9c0\uace0 \uc788\ub358 \uc0b4\ub77c\ub518\uc5d0\uac8c \ud0c0\ub9c8\ub974\uac00 \uae08\uad34 20\ub9cc \ucabd\uc744 \ud5cc\ub0a9\ud560 \ub9cc\ud07c \uc131\uc2ed\uc790\uac00\uc758 \uc720\ubb3c\uc744 \ud68d\ub4dd\ud558\ub824\ub294 \uadf8\ub140\uc758 \ub178\ub825\uc774 \ube44\uc794\ud2f0\uc6c0 \ud669\uc81c\ub97c \ub2a5\uac00\ud588\ub2e4\uace0 \uc8fc\uc7a5\ud588\ub2e4\uace0 \ud588\ub2e4. \uadf8\ub7ec\ub098, \uadf8\uc758 \ub178\ub825\uc740 \ud5c8\uc0ac\ub85c \ub3cc\uc544\uac14\ub2e4.", "paragraph_answer": "\uc870\uc9c0\uc544 \uc655\uc2e4\uc740 \uba3c\uc800 \uc131\uc9c0\uc5d0 \uc788\ub294 \uc870\uc9c0\uc544\uc758 \uc218\ub3c4\uc6d0 \uc911\uc2ec\uc9c0\ub4e4\uc758 \ubcf4\ud638\uc5d0 \ub300\ud574 \uad00\uc2ec\uc744 \uae30\uc6b8\ub838\ub2e4. 12\uc138\uae30 \uacbd\uc5d0\ub294, \uc5ec\ub35f \uc218\ub3c4\uc6d0\ub4e4\uc774 \uc81c\ub8e8\uc0b4\ub818 \uc5d0 \uc788\uc5c8\ub2e4. \uc0b4\ub77c\ub518\uc758 \uc804\uae30 \uc791\uac00 \ubc14\ud558 \uc544\ub4dc-\ub518 \uc774\ube10 \uc0e4\ub4dc\ub2e4\ub4dc\uc758 \ubcf4\uace0\uc5d0 \ub530\ub974\uba74, 1187\ub144\uc5d0 \uc544\uc774\uc720\ube0c\uc870\uac00 \uc608\ub8e8\uc0b4\ub818\uc744 \uc815\ubcf5\ud55c \ud6c4\uc5d0, \ud0c0\ub9c8\ub974\uac00 \uc220\ud0c4\uc5d0\uac8c \uc678\uad50 \uc0ac\uc808\uc744 \ubcf4\ub0b4 \uc608\ub8e8\uc0b4\ub818\uc5d0 \uc788\ub294 \uadf8\ub4e4\uc774 \ubab0\uc218\ud55c \uc870\uc9c0\uc544\uc758 \uc218\ub3c4\uc6d0\ub4e4\uc758 \uc18c\uc720\uc9c0\ub4e4\uc744 \ubc18\ud658\ud560 \uac83\uc744 \uc694\uad6c\ud588\ub2e4. \uc0b4\ub77c\ub518\uc758 \ub300\uafb8\ub294 \uc5c6\uc5c8\uc9c0\ub9cc, \uc5ec\uc655\uc73c\uc758 \ub178\ub825\uc740 \uc131\uacf5\uc744 \uac70\ub450\uc5b4 \uac14\ub2e4. \ud0c0\ub9c8\ub974\uac00 \uc8fd\uc740 \ud6c4\uc5d0 \ub2e8\uae30\uac04\uc758 \uc544\ud06c\ub808\uc758 \uc8fc\uad50\ub85c \uc788\ub294 \uc790\ucfe0\uc5d0\uc2a4 \ub370 \ube44\ud2b8\ub9ac\ub294 \uc608\ub8e8\uc0b4\ub818\uc5d0\uc11c\uc758 \uc870\uc9c0\uc544\uc758 \uc874\uc7ac\uc5d0 \ub300\ud558\uc5ec \ub354 \ub098\uc740 \uadfc\uac70\ub97c \uc81c\uc2dc\ud588\ub2e4. \uadf8\ub294 \ub2e4\ub978 \uadf8\ub9ac\uc2a4\ub3c4\uad50 \uc21c\ub840\uc790\ub4e4\uacfc\ub294 \ub300\uc870\ub418\uac8c \uc870\uc9c0\uc544\uc778\ub4e4\uc740 \uadf8\ub4e4\uc758 \uae43\ubc1c\uc744 \ud718\ub0a0\ub9ac\uba70 \ub3c4\uc2dc\ub85c\uc758 \uc790\uc720\ub85c\uc6b4 \uc655\ub798\uac00 \ud5c8\uc6a9\ub418\uc5c8\uc5c8\ub2e4\uace0 \uae30\uc220\ud588\ub2e4. \ub354\uad70\ub2e4\ub098 \uc774\ube10 \uc0ac\ub4dc\ub2e4\ub4dc\ub294 \ud558\ud2f4 \uc804\ud22c\uc5d0\uc11c \uc57d\ud0c8\ud55c \uc720\ubb3c\uc744 \uac00\uc9c0\uace0 \uc788\ub358 \uc0b4\ub77c\ub518\uc5d0\uac8c \ud0c0\ub9c8\ub974\uac00 \uae08\uad34 20\ub9cc \ucabd\uc744 \ud5cc\ub0a9\ud560 \ub9cc\ud07c \uc131\uc2ed\uc790\uac00\uc758 \uc720\ubb3c\uc744 \ud68d\ub4dd\ud558\ub824\ub294 \uadf8\ub140\uc758 \ub178\ub825\uc774 \ube44\uc794\ud2f0\uc6c0 \ud669\uc81c\ub97c \ub2a5\uac00\ud588\ub2e4\uace0 \uc8fc\uc7a5\ud588\ub2e4\uace0 \ud588\ub2e4. \uadf8\ub7ec\ub098, \uadf8\uc758 \ub178\ub825\uc740 \ud5c8\uc0ac\ub85c \ub3cc\uc544\uac14\ub2e4.", "sentence_answer": "12\uc138\uae30 \uacbd\uc5d0\ub294, \uc5ec\ub35f \uc218\ub3c4\uc6d0\ub4e4\uc774 \uc81c\ub8e8\uc0b4\ub818 \uc5d0 \uc788\uc5c8\ub2e4.", "paragraph_id": "6554557-7-0", "paragraph_question": "question: 12\uc138\uae30 \uacbd \uc5ec\ub7f7 \uc218\ub3c4\uc6d0\ub4e4\uc740 \uc5b4\ub514\uc5d0 \uc788\uc5c8\uc744\uae4c?, context: \uc870\uc9c0\uc544 \uc655\uc2e4\uc740 \uba3c\uc800 \uc131\uc9c0\uc5d0 \uc788\ub294 \uc870\uc9c0\uc544\uc758 \uc218\ub3c4\uc6d0 \uc911\uc2ec\uc9c0\ub4e4\uc758 \ubcf4\ud638\uc5d0 \ub300\ud574 \uad00\uc2ec\uc744 \uae30\uc6b8\ub838\ub2e4. 12\uc138\uae30 \uacbd\uc5d0\ub294, \uc5ec\ub35f \uc218\ub3c4\uc6d0\ub4e4\uc774 \uc81c\ub8e8\uc0b4\ub818\uc5d0 \uc788\uc5c8\ub2e4. \uc0b4\ub77c\ub518\uc758 \uc804\uae30 \uc791\uac00 \ubc14\ud558 \uc544\ub4dc-\ub518 \uc774\ube10 \uc0e4\ub4dc\ub2e4\ub4dc\uc758 \ubcf4\uace0\uc5d0 \ub530\ub974\uba74, 1187\ub144\uc5d0 \uc544\uc774\uc720\ube0c\uc870\uac00 \uc608\ub8e8\uc0b4\ub818\uc744 \uc815\ubcf5\ud55c \ud6c4\uc5d0, \ud0c0\ub9c8\ub974\uac00 \uc220\ud0c4\uc5d0\uac8c \uc678\uad50 \uc0ac\uc808\uc744 \ubcf4\ub0b4 \uc608\ub8e8\uc0b4\ub818\uc5d0 \uc788\ub294 \uadf8\ub4e4\uc774 \ubab0\uc218\ud55c \uc870\uc9c0\uc544\uc758 \uc218\ub3c4\uc6d0\ub4e4\uc758 \uc18c\uc720\uc9c0\ub4e4\uc744 \ubc18\ud658\ud560 \uac83\uc744 \uc694\uad6c\ud588\ub2e4. \uc0b4\ub77c\ub518\uc758 \ub300\uafb8\ub294 \uc5c6\uc5c8\uc9c0\ub9cc, \uc5ec\uc655\uc73c\uc758 \ub178\ub825\uc740 \uc131\uacf5\uc744 \uac70\ub450\uc5b4 \uac14\ub2e4. \ud0c0\ub9c8\ub974\uac00 \uc8fd\uc740 \ud6c4\uc5d0 \ub2e8\uae30\uac04\uc758 \uc544\ud06c\ub808\uc758 \uc8fc\uad50\ub85c \uc788\ub294 \uc790\ucfe0\uc5d0\uc2a4 \ub370 \ube44\ud2b8\ub9ac\ub294 \uc608\ub8e8\uc0b4\ub818\uc5d0\uc11c\uc758 \uc870\uc9c0\uc544\uc758 \uc874\uc7ac\uc5d0 \ub300\ud558\uc5ec \ub354 \ub098\uc740 \uadfc\uac70\ub97c \uc81c\uc2dc\ud588\ub2e4. \uadf8\ub294 \ub2e4\ub978 \uadf8\ub9ac\uc2a4\ub3c4\uad50 \uc21c\ub840\uc790\ub4e4\uacfc\ub294 \ub300\uc870\ub418\uac8c \uc870\uc9c0\uc544\uc778\ub4e4\uc740 \uadf8\ub4e4\uc758 \uae43\ubc1c\uc744 \ud718\ub0a0\ub9ac\uba70 \ub3c4\uc2dc\ub85c\uc758 \uc790\uc720\ub85c\uc6b4 \uc655\ub798\uac00 \ud5c8\uc6a9\ub418\uc5c8\uc5c8\ub2e4\uace0 \uae30\uc220\ud588\ub2e4. \ub354\uad70\ub2e4\ub098 \uc774\ube10 \uc0ac\ub4dc\ub2e4\ub4dc\ub294 \ud558\ud2f4 \uc804\ud22c\uc5d0\uc11c \uc57d\ud0c8\ud55c \uc720\ubb3c\uc744 \uac00\uc9c0\uace0 \uc788\ub358 \uc0b4\ub77c\ub518\uc5d0\uac8c \ud0c0\ub9c8\ub974\uac00 \uae08\uad34 20\ub9cc \ucabd\uc744 \ud5cc\ub0a9\ud560 \ub9cc\ud07c \uc131\uc2ed\uc790\uac00\uc758 \uc720\ubb3c\uc744 \ud68d\ub4dd\ud558\ub824\ub294 \uadf8\ub140\uc758 \ub178\ub825\uc774 \ube44\uc794\ud2f0\uc6c0 \ud669\uc81c\ub97c \ub2a5\uac00\ud588\ub2e4\uace0 \uc8fc\uc7a5\ud588\ub2e4\uace0 \ud588\ub2e4. \uadf8\ub7ec\ub098, \uadf8\uc758 \ub178\ub825\uc740 \ud5c8\uc0ac\ub85c \ub3cc\uc544\uac14\ub2e4."} {"question": "\ubcf4\uc988\ub124\uc13c\uc2a4\ud0a4\uc758 \uc5b8\uc5b4 \uc608\uc220\uc758 \uc9c4\uc218\ub97c \ubcf4\uc5ec\uc900 \ubc29\uc2dd\uc758 \uc7a5\ub974\ub294 \ubb34\uc5c7\uc778\uac00\uc694?", "paragraph": "\uc2dc\uc778\uc73c\ub85c\uc11c \ubcf4\uc988\ub124\uc13c\uc2a4\ud0a4\uc758 \uc778\uae30\ub97c \uc54c\uae30 \uc704\ud574\uc11c\ub294 \uba3c\uc800 \ub7ec\uc2dc\uc544\uc5d0\uc11c \uc2dc\uc778\uc774 \ucc28\uc9c0\ud558\uace0 \uc788\ub294 \uc704\uce58\ub97c \uc54c\uc544\uc57c \ud55c\ub2e4. \ubcf4\uc988\ub124\uc13c\uc2a4\ud0a4\ub294 \uacfc\uac70 \uc18c\ub828\uc5d0\uc11c \uac00\uc7a5 \uc778\uae30 \uc788\ub294 \uc2dc\uc778\uc774\uc5c8\ub2e4. \ub3c5\uc7ac\uc790 \uc2a4\ud0c8\ub9b0\uc758 \uc0ac\ub9dd \ud6c4 \uc800\ud56d \uc2dc\uc778 \uc619\ud22c\uc174\ucf54\uc640 \ub354\ubd88\uc5b4 \u2018\uc80a\uc740 \uc2dc\uc778\ub4e4\u2019\uc758 \uc120\ub450 \uc8fc\uc790\uc778 \ubcf4\uc988\ub124\uc13c\uc2a4\ud0a4\ub294 1950\ub144\ub300 \ud6c4\ubc18\ubd80\ud130 \uc9c0\uae08\uae4c\uc9c0 \uc904\uace7 \ud654\uc81c\uc758 \uc911\uc2ec\uc774 \ub418\uc5b4\uc628 \ub7ec\uc2dc\uc544\uc758 \ub300\ud45c\uc801\uc778 \uc2dc\uc778\uc73c\ub85c\uc11c \uc138\uacc4\uc801 \uba85\uc131\uc744 \uc5bb\uc5c8\ub2e4. \uc2dc \ub0ad\uc1a1\ud68c, \ud154\ub808\ube44\uc804\uacfc \uc2e0\ubb38 \ub4f1 \uac01\uc885 \ub9e4\uc2a4\ucef4\uc744 \ud1b5\ud574 \ub300\uc911\uacfc \uce5c\uc219\ud55c \uc774 \uc2dc\uc778\uc740 \ub7ec\uc2dc\uc544 \uc2dc \uc804\ud1b5 \uc18d\uc5d0\uc11c \ub3c5\uc790\uc801\uc778 \uc2dc\uc5b4\ub97c \uac1c\ubc1c\ud558\uace0 \uc790\uc791\uc2dc \ub0ad\uc1a1\uc744 \ud1b5\ud558\uc5ec \uc5b8\uc5b4 \uc608\uc220\uc758 \uc9c4\uc218\ub97c \ubcf4\uc5ec\uc900\ub2e4. \uc218\ub9ce\uc740 \uccad\uc911\uc758 \uac08\ucc44\uc640 \ud658\ud638 \uc18d\uc5d0\uc11c \ud589\ud574\uc9c0\ub294 \uadf8\uc758 \uc2dc \ub0ad\uc1a1\uc740 \uc0c8\ub85c\uc6b4 \uc608\uc220 \uc7a5\ub974, \uc989 \ub0ad\uc1a1 \uc608\uc220\uc758 \ucc3d\uc870\ub77c \ud560 \uc218 \uc788\ub2e4.", "answer": "\ub0ad\uc1a1 \uc608\uc220", "sentence": "\uc218\ub9ce\uc740 \uccad\uc911\uc758 \uac08\ucc44\uc640 \ud658\ud638 \uc18d\uc5d0\uc11c \ud589\ud574\uc9c0\ub294 \uadf8\uc758 \uc2dc \ub0ad\uc1a1\uc740 \uc0c8\ub85c\uc6b4 \uc608\uc220 \uc7a5\ub974, \uc989 \ub0ad\uc1a1 \uc608\uc220 \uc758 \ucc3d\uc870\ub77c \ud560 \uc218 \uc788\ub2e4.", "paragraph_sentence": "\uc2dc\uc778\uc73c\ub85c\uc11c \ubcf4\uc988\ub124\uc13c\uc2a4\ud0a4\uc758 \uc778\uae30\ub97c \uc54c\uae30 \uc704\ud574\uc11c\ub294 \uba3c\uc800 \ub7ec\uc2dc\uc544\uc5d0\uc11c \uc2dc\uc778\uc774 \ucc28\uc9c0\ud558\uace0 \uc788\ub294 \uc704\uce58\ub97c \uc54c\uc544\uc57c \ud55c\ub2e4. \ubcf4\uc988\ub124\uc13c\uc2a4\ud0a4\ub294 \uacfc\uac70 \uc18c\ub828\uc5d0\uc11c \uac00\uc7a5 \uc778\uae30 \uc788\ub294 \uc2dc\uc778\uc774\uc5c8\ub2e4. \ub3c5\uc7ac\uc790 \uc2a4\ud0c8\ub9b0\uc758 \uc0ac\ub9dd \ud6c4 \uc800\ud56d \uc2dc\uc778 \uc619\ud22c\uc174\ucf54\uc640 \ub354\ubd88\uc5b4 \u2018\uc80a\uc740 \uc2dc\uc778\ub4e4\u2019\uc758 \uc120\ub450 \uc8fc\uc790\uc778 \ubcf4\uc988\ub124\uc13c\uc2a4\ud0a4\ub294 1950\ub144\ub300 \ud6c4\ubc18\ubd80\ud130 \uc9c0\uae08\uae4c\uc9c0 \uc904\uace7 \ud654\uc81c\uc758 \uc911\uc2ec\uc774 \ub418\uc5b4\uc628 \ub7ec\uc2dc\uc544\uc758 \ub300\ud45c\uc801\uc778 \uc2dc\uc778\uc73c\ub85c\uc11c \uc138\uacc4\uc801 \uba85\uc131\uc744 \uc5bb\uc5c8\ub2e4. \uc2dc \ub0ad\uc1a1\ud68c, \ud154\ub808\ube44\uc804\uacfc \uc2e0\ubb38 \ub4f1 \uac01\uc885 \ub9e4\uc2a4\ucef4\uc744 \ud1b5\ud574 \ub300\uc911\uacfc \uce5c\uc219\ud55c \uc774 \uc2dc\uc778\uc740 \ub7ec\uc2dc\uc544 \uc2dc \uc804\ud1b5 \uc18d\uc5d0\uc11c \ub3c5\uc790\uc801\uc778 \uc2dc\uc5b4\ub97c \uac1c\ubc1c\ud558\uace0 \uc790\uc791\uc2dc \ub0ad\uc1a1\uc744 \ud1b5\ud558\uc5ec \uc5b8\uc5b4 \uc608\uc220\uc758 \uc9c4\uc218\ub97c \ubcf4\uc5ec\uc900\ub2e4. \uc218\ub9ce\uc740 \uccad\uc911\uc758 \uac08\ucc44\uc640 \ud658\ud638 \uc18d\uc5d0\uc11c \ud589\ud574\uc9c0\ub294 \uadf8\uc758 \uc2dc \ub0ad\uc1a1\uc740 \uc0c8\ub85c\uc6b4 \uc608\uc220 \uc7a5\ub974, \uc989 \ub0ad\uc1a1 \uc608\uc220 \uc758 \ucc3d\uc870\ub77c \ud560 \uc218 \uc788\ub2e4. ", "paragraph_answer": "\uc2dc\uc778\uc73c\ub85c\uc11c \ubcf4\uc988\ub124\uc13c\uc2a4\ud0a4\uc758 \uc778\uae30\ub97c \uc54c\uae30 \uc704\ud574\uc11c\ub294 \uba3c\uc800 \ub7ec\uc2dc\uc544\uc5d0\uc11c \uc2dc\uc778\uc774 \ucc28\uc9c0\ud558\uace0 \uc788\ub294 \uc704\uce58\ub97c \uc54c\uc544\uc57c \ud55c\ub2e4. \ubcf4\uc988\ub124\uc13c\uc2a4\ud0a4\ub294 \uacfc\uac70 \uc18c\ub828\uc5d0\uc11c \uac00\uc7a5 \uc778\uae30 \uc788\ub294 \uc2dc\uc778\uc774\uc5c8\ub2e4. \ub3c5\uc7ac\uc790 \uc2a4\ud0c8\ub9b0\uc758 \uc0ac\ub9dd \ud6c4 \uc800\ud56d \uc2dc\uc778 \uc619\ud22c\uc174\ucf54\uc640 \ub354\ubd88\uc5b4 \u2018\uc80a\uc740 \uc2dc\uc778\ub4e4\u2019\uc758 \uc120\ub450 \uc8fc\uc790\uc778 \ubcf4\uc988\ub124\uc13c\uc2a4\ud0a4\ub294 1950\ub144\ub300 \ud6c4\ubc18\ubd80\ud130 \uc9c0\uae08\uae4c\uc9c0 \uc904\uace7 \ud654\uc81c\uc758 \uc911\uc2ec\uc774 \ub418\uc5b4\uc628 \ub7ec\uc2dc\uc544\uc758 \ub300\ud45c\uc801\uc778 \uc2dc\uc778\uc73c\ub85c\uc11c \uc138\uacc4\uc801 \uba85\uc131\uc744 \uc5bb\uc5c8\ub2e4. \uc2dc \ub0ad\uc1a1\ud68c, \ud154\ub808\ube44\uc804\uacfc \uc2e0\ubb38 \ub4f1 \uac01\uc885 \ub9e4\uc2a4\ucef4\uc744 \ud1b5\ud574 \ub300\uc911\uacfc \uce5c\uc219\ud55c \uc774 \uc2dc\uc778\uc740 \ub7ec\uc2dc\uc544 \uc2dc \uc804\ud1b5 \uc18d\uc5d0\uc11c \ub3c5\uc790\uc801\uc778 \uc2dc\uc5b4\ub97c \uac1c\ubc1c\ud558\uace0 \uc790\uc791\uc2dc \ub0ad\uc1a1\uc744 \ud1b5\ud558\uc5ec \uc5b8\uc5b4 \uc608\uc220\uc758 \uc9c4\uc218\ub97c \ubcf4\uc5ec\uc900\ub2e4. \uc218\ub9ce\uc740 \uccad\uc911\uc758 \uac08\ucc44\uc640 \ud658\ud638 \uc18d\uc5d0\uc11c \ud589\ud574\uc9c0\ub294 \uadf8\uc758 \uc2dc \ub0ad\uc1a1\uc740 \uc0c8\ub85c\uc6b4 \uc608\uc220 \uc7a5\ub974, \uc989 \ub0ad\uc1a1 \uc608\uc220 \uc758 \ucc3d\uc870\ub77c \ud560 \uc218 \uc788\ub2e4.", "sentence_answer": "\uc218\ub9ce\uc740 \uccad\uc911\uc758 \uac08\ucc44\uc640 \ud658\ud638 \uc18d\uc5d0\uc11c \ud589\ud574\uc9c0\ub294 \uadf8\uc758 \uc2dc \ub0ad\uc1a1\uc740 \uc0c8\ub85c\uc6b4 \uc608\uc220 \uc7a5\ub974, \uc989 \ub0ad\uc1a1 \uc608\uc220 \uc758 \ucc3d\uc870\ub77c \ud560 \uc218 \uc788\ub2e4.", "paragraph_id": "6547501-3-1", "paragraph_question": "question: \ubcf4\uc988\ub124\uc13c\uc2a4\ud0a4\uc758 \uc5b8\uc5b4 \uc608\uc220\uc758 \uc9c4\uc218\ub97c \ubcf4\uc5ec\uc900 \ubc29\uc2dd\uc758 \uc7a5\ub974\ub294 \ubb34\uc5c7\uc778\uac00\uc694?, context: \uc2dc\uc778\uc73c\ub85c\uc11c \ubcf4\uc988\ub124\uc13c\uc2a4\ud0a4\uc758 \uc778\uae30\ub97c \uc54c\uae30 \uc704\ud574\uc11c\ub294 \uba3c\uc800 \ub7ec\uc2dc\uc544\uc5d0\uc11c \uc2dc\uc778\uc774 \ucc28\uc9c0\ud558\uace0 \uc788\ub294 \uc704\uce58\ub97c \uc54c\uc544\uc57c \ud55c\ub2e4. \ubcf4\uc988\ub124\uc13c\uc2a4\ud0a4\ub294 \uacfc\uac70 \uc18c\ub828\uc5d0\uc11c \uac00\uc7a5 \uc778\uae30 \uc788\ub294 \uc2dc\uc778\uc774\uc5c8\ub2e4. \ub3c5\uc7ac\uc790 \uc2a4\ud0c8\ub9b0\uc758 \uc0ac\ub9dd \ud6c4 \uc800\ud56d \uc2dc\uc778 \uc619\ud22c\uc174\ucf54\uc640 \ub354\ubd88\uc5b4 \u2018\uc80a\uc740 \uc2dc\uc778\ub4e4\u2019\uc758 \uc120\ub450 \uc8fc\uc790\uc778 \ubcf4\uc988\ub124\uc13c\uc2a4\ud0a4\ub294 1950\ub144\ub300 \ud6c4\ubc18\ubd80\ud130 \uc9c0\uae08\uae4c\uc9c0 \uc904\uace7 \ud654\uc81c\uc758 \uc911\uc2ec\uc774 \ub418\uc5b4\uc628 \ub7ec\uc2dc\uc544\uc758 \ub300\ud45c\uc801\uc778 \uc2dc\uc778\uc73c\ub85c\uc11c \uc138\uacc4\uc801 \uba85\uc131\uc744 \uc5bb\uc5c8\ub2e4. \uc2dc \ub0ad\uc1a1\ud68c, \ud154\ub808\ube44\uc804\uacfc \uc2e0\ubb38 \ub4f1 \uac01\uc885 \ub9e4\uc2a4\ucef4\uc744 \ud1b5\ud574 \ub300\uc911\uacfc \uce5c\uc219\ud55c \uc774 \uc2dc\uc778\uc740 \ub7ec\uc2dc\uc544 \uc2dc \uc804\ud1b5 \uc18d\uc5d0\uc11c \ub3c5\uc790\uc801\uc778 \uc2dc\uc5b4\ub97c \uac1c\ubc1c\ud558\uace0 \uc790\uc791\uc2dc \ub0ad\uc1a1\uc744 \ud1b5\ud558\uc5ec \uc5b8\uc5b4 \uc608\uc220\uc758 \uc9c4\uc218\ub97c \ubcf4\uc5ec\uc900\ub2e4. \uc218\ub9ce\uc740 \uccad\uc911\uc758 \uac08\ucc44\uc640 \ud658\ud638 \uc18d\uc5d0\uc11c \ud589\ud574\uc9c0\ub294 \uadf8\uc758 \uc2dc \ub0ad\uc1a1\uc740 \uc0c8\ub85c\uc6b4 \uc608\uc220 \uc7a5\ub974, \uc989 \ub0ad\uc1a1 \uc608\uc220\uc758 \ucc3d\uc870\ub77c \ud560 \uc218 \uc788\ub2e4."} {"question": "\uc0c8\ub204\ub9ac\ub2f9 \uad00\uacc4\uc790\ub97c \ub9c9\uae30 \uc704\ud574 \uc11c\uc6b8\uc2dc\uccad\uc774 \ub3d9\uc6d0\ud55c \uac83\uc740 \ubb34\uc5c7\uc778\uac00?", "paragraph": "\uc0c8\ub204\ub9ac\ub2f9 \uad00\uacc4\uc790\ub4e4\uc740 2013\ub144 8\uc6d4 2\uc77c\uc5d0 \uc11c\uc6b8\ud2b9\ubcc4\uc2dc\uccad\uc744 \ubc29\ubb38\ud558\uace0 \ubc15\uc6d0\uc21c \uc2dc\uc7a5\uc5d0\uac8c \uac74\uc124\ud604\uc7a5 \uc0ac\uace0\uc5d0 \uad00\ud55c \ucc45\uc784\uc744 \ubb3b\ub294 \ud56d\uc758 \uc11c\ud55c\uc744 \uc81c\ucd9c\ud558\ub824\uace0 \ud558\uc600\uc73c\ub098, \ubc15\uc6d0\uc21c\uacfc \uccad\uc6d0\uacbd\ucc30\uc758 \uac70\ubd80\ud589\uc704\ub85c \uc778\ud55c \ucda9\ub3cc\uc774 \uc788\uc5c8\ub2e4. \ud76c\uc0dd\uc790 \uc720\uac00\uc871\ub4e4\uc744 \ube44\ub86f\ud55c \uc0c8\ub204\ub9ac\ub2f9 \uad00\uacc4\uc790\ub4e4\uc740 \ubc15\uc6d0\uc21c\uc5d0\uac8c \ud56d\uc758 \uc11c\ud55c\uc744 \uc804\ub2ec\ud558\ub824\ub294 \ubaa9\uc801\uc774 \uc788\uc5c8\uc73c\ub098, \uc774\ub97c \uac70\ubd80\ud55c \uc11c\uc6b8\uc2dc\uccad \uce21\uc740 \uccad\uc6d0\uacbd\ucc30\uc744 \ub3d9\uc6d0\ud558\uc5ec \ubc29\ubb38\uc744 \ub9c9\uc558\uace0 \uc774 \uacfc\uc815\uc5d0\uc11c \ubab8\uc2f8\uc6c0\uc774 \ubc8c\uc5b4\uc838 \uccad\uc6d0\uacbd\ucc30\uc774 \ubd80\uc0c1\uc744 \uc785\uc5c8\ub2e4. \uc774\uc5d0 \ub300\ud574 \uc11c\uc6b8\uc2dc \uce21\uc740, \uc11c\uc6b8\uc2dc \uc9c1\uc6d0\uc774 \uc0c1\ud574\ub97c \uc785\uc740 \uac83\uc5d0 \ub300\ud574 \uc720\uac10\uc758 \ub73b\uc744 \ud45c\uba85\ud558\uc600\ub2e4. \ubc18\uba74 \uae40\uc131\ud0dc \uc758\uc6d0\uc2e4 \uce21\uc740, \uc11c\uc6b8\uc2dc\ubbfc\uc758 \uc548\uc804\uc744 \uc9c0\ud0a4\uc9c0 \ubabb\ud558\uace0 \uc720\uac00\uc871\uacfc \ubbfc\uc6d0\uc778\uc758 \ud56d\uc758 \ubc29\ubb38\uc744 \ud68c\ud53c\ud55c \ubc15\uc6d0\uc21c \uc2dc\uc7a5\uc774 \ucc45\uc784\uc744 \uc838\uc57c\ud55c\ub2e4\uace0 \uc8fc\uc7a5\ud558\uc600\ub2e4. \uc774\ub7ec\ud55c \uc77c\ub828\uc758 \uc0ac\uac74 \uc911 \uae30\ud0c0 \uc2e0\ubb38\uae30\uc0ac\ub098 \uc778\ud130\ub137 \ub4f1\uc9c0\uc5d0\uc11c \uc0c8\ub204\ub9ac\ub2f9 \uad00\uacc4\uc790\ub97c \uae40\uc131\ud0dc \uc758\uc6d0\uc2e4 \uad00\uacc4\uc790\ub85c \uc798\ubabb \ud45c\uae30\ud558\ub294 \uc624\ubcf4\uac00 \ubc1c\uc0dd\ud588\ub2e4.", "answer": "\uccad\uc6d0\uacbd\ucc30", "sentence": "\ubc15\uc6d0\uc21c\uacfc \uccad\uc6d0\uacbd\ucc30 \uc758 \uac70\ubd80\ud589\uc704\ub85c \uc778\ud55c \ucda9\ub3cc\uc774 \uc788\uc5c8\ub2e4.", "paragraph_sentence": "\uc0c8\ub204\ub9ac\ub2f9 \uad00\uacc4\uc790\ub4e4\uc740 2013\ub144 8\uc6d4 2\uc77c\uc5d0 \uc11c\uc6b8\ud2b9\ubcc4\uc2dc\uccad\uc744 \ubc29\ubb38\ud558\uace0 \ubc15\uc6d0\uc21c \uc2dc\uc7a5\uc5d0\uac8c \uac74\uc124\ud604\uc7a5 \uc0ac\uace0\uc5d0 \uad00\ud55c \ucc45\uc784\uc744 \ubb3b\ub294 \ud56d\uc758 \uc11c\ud55c\uc744 \uc81c\ucd9c\ud558\ub824\uace0 \ud558\uc600\uc73c\ub098, \ubc15\uc6d0\uc21c\uacfc \uccad\uc6d0\uacbd\ucc30 \uc758 \uac70\ubd80\ud589\uc704\ub85c \uc778\ud55c \ucda9\ub3cc\uc774 \uc788\uc5c8\ub2e4. \ud76c\uc0dd\uc790 \uc720\uac00\uc871\ub4e4\uc744 \ube44\ub86f\ud55c \uc0c8\ub204\ub9ac\ub2f9 \uad00\uacc4\uc790\ub4e4\uc740 \ubc15\uc6d0\uc21c\uc5d0\uac8c \ud56d\uc758 \uc11c\ud55c\uc744 \uc804\ub2ec\ud558\ub824\ub294 \ubaa9\uc801\uc774 \uc788\uc5c8\uc73c\ub098, \uc774\ub97c \uac70\ubd80\ud55c \uc11c\uc6b8\uc2dc\uccad \uce21\uc740 \uccad\uc6d0\uacbd\ucc30\uc744 \ub3d9\uc6d0\ud558\uc5ec \ubc29\ubb38\uc744 \ub9c9\uc558\uace0 \uc774 \uacfc\uc815\uc5d0\uc11c \ubab8\uc2f8\uc6c0\uc774 \ubc8c\uc5b4\uc838 \uccad\uc6d0\uacbd\ucc30\uc774 \ubd80\uc0c1\uc744 \uc785\uc5c8\ub2e4. \uc774\uc5d0 \ub300\ud574 \uc11c\uc6b8\uc2dc \uce21\uc740, \uc11c\uc6b8\uc2dc \uc9c1\uc6d0\uc774 \uc0c1\ud574\ub97c \uc785\uc740 \uac83\uc5d0 \ub300\ud574 \uc720\uac10\uc758 \ub73b\uc744 \ud45c\uba85\ud558\uc600\ub2e4. \ubc18\uba74 \uae40\uc131\ud0dc \uc758\uc6d0\uc2e4 \uce21\uc740, \uc11c\uc6b8\uc2dc\ubbfc\uc758 \uc548\uc804\uc744 \uc9c0\ud0a4\uc9c0 \ubabb\ud558\uace0 \uc720\uac00\uc871\uacfc \ubbfc\uc6d0\uc778\uc758 \ud56d\uc758 \ubc29\ubb38\uc744 \ud68c\ud53c\ud55c \ubc15\uc6d0\uc21c \uc2dc\uc7a5\uc774 \ucc45\uc784\uc744 \uc838\uc57c\ud55c\ub2e4\uace0 \uc8fc\uc7a5\ud558\uc600\ub2e4. \uc774\ub7ec\ud55c \uc77c\ub828\uc758 \uc0ac\uac74 \uc911 \uae30\ud0c0 \uc2e0\ubb38\uae30\uc0ac\ub098 \uc778\ud130\ub137 \ub4f1\uc9c0\uc5d0\uc11c \uc0c8\ub204\ub9ac\ub2f9 \uad00\uacc4\uc790\ub97c \uae40\uc131\ud0dc \uc758\uc6d0\uc2e4 \uad00\uacc4\uc790\ub85c \uc798\ubabb \ud45c\uae30\ud558\ub294 \uc624\ubcf4\uac00 \ubc1c\uc0dd\ud588\ub2e4.", "paragraph_answer": "\uc0c8\ub204\ub9ac\ub2f9 \uad00\uacc4\uc790\ub4e4\uc740 2013\ub144 8\uc6d4 2\uc77c\uc5d0 \uc11c\uc6b8\ud2b9\ubcc4\uc2dc\uccad\uc744 \ubc29\ubb38\ud558\uace0 \ubc15\uc6d0\uc21c \uc2dc\uc7a5\uc5d0\uac8c \uac74\uc124\ud604\uc7a5 \uc0ac\uace0\uc5d0 \uad00\ud55c \ucc45\uc784\uc744 \ubb3b\ub294 \ud56d\uc758 \uc11c\ud55c\uc744 \uc81c\ucd9c\ud558\ub824\uace0 \ud558\uc600\uc73c\ub098, \ubc15\uc6d0\uc21c\uacfc \uccad\uc6d0\uacbd\ucc30 \uc758 \uac70\ubd80\ud589\uc704\ub85c \uc778\ud55c \ucda9\ub3cc\uc774 \uc788\uc5c8\ub2e4. \ud76c\uc0dd\uc790 \uc720\uac00\uc871\ub4e4\uc744 \ube44\ub86f\ud55c \uc0c8\ub204\ub9ac\ub2f9 \uad00\uacc4\uc790\ub4e4\uc740 \ubc15\uc6d0\uc21c\uc5d0\uac8c \ud56d\uc758 \uc11c\ud55c\uc744 \uc804\ub2ec\ud558\ub824\ub294 \ubaa9\uc801\uc774 \uc788\uc5c8\uc73c\ub098, \uc774\ub97c \uac70\ubd80\ud55c \uc11c\uc6b8\uc2dc\uccad \uce21\uc740 \uccad\uc6d0\uacbd\ucc30\uc744 \ub3d9\uc6d0\ud558\uc5ec \ubc29\ubb38\uc744 \ub9c9\uc558\uace0 \uc774 \uacfc\uc815\uc5d0\uc11c \ubab8\uc2f8\uc6c0\uc774 \ubc8c\uc5b4\uc838 \uccad\uc6d0\uacbd\ucc30\uc774 \ubd80\uc0c1\uc744 \uc785\uc5c8\ub2e4. \uc774\uc5d0 \ub300\ud574 \uc11c\uc6b8\uc2dc \uce21\uc740, \uc11c\uc6b8\uc2dc \uc9c1\uc6d0\uc774 \uc0c1\ud574\ub97c \uc785\uc740 \uac83\uc5d0 \ub300\ud574 \uc720\uac10\uc758 \ub73b\uc744 \ud45c\uba85\ud558\uc600\ub2e4. \ubc18\uba74 \uae40\uc131\ud0dc \uc758\uc6d0\uc2e4 \uce21\uc740, \uc11c\uc6b8\uc2dc\ubbfc\uc758 \uc548\uc804\uc744 \uc9c0\ud0a4\uc9c0 \ubabb\ud558\uace0 \uc720\uac00\uc871\uacfc \ubbfc\uc6d0\uc778\uc758 \ud56d\uc758 \ubc29\ubb38\uc744 \ud68c\ud53c\ud55c \ubc15\uc6d0\uc21c \uc2dc\uc7a5\uc774 \ucc45\uc784\uc744 \uc838\uc57c\ud55c\ub2e4\uace0 \uc8fc\uc7a5\ud558\uc600\ub2e4. \uc774\ub7ec\ud55c \uc77c\ub828\uc758 \uc0ac\uac74 \uc911 \uae30\ud0c0 \uc2e0\ubb38\uae30\uc0ac\ub098 \uc778\ud130\ub137 \ub4f1\uc9c0\uc5d0\uc11c \uc0c8\ub204\ub9ac\ub2f9 \uad00\uacc4\uc790\ub97c \uae40\uc131\ud0dc \uc758\uc6d0\uc2e4 \uad00\uacc4\uc790\ub85c \uc798\ubabb \ud45c\uae30\ud558\ub294 \uc624\ubcf4\uac00 \ubc1c\uc0dd\ud588\ub2e4.", "sentence_answer": "\ubc15\uc6d0\uc21c\uacfc \uccad\uc6d0\uacbd\ucc30 \uc758 \uac70\ubd80\ud589\uc704\ub85c \uc778\ud55c \ucda9\ub3cc\uc774 \uc788\uc5c8\ub2e4.", "paragraph_id": "6577053-1-1", "paragraph_question": "question: \uc0c8\ub204\ub9ac\ub2f9 \uad00\uacc4\uc790\ub97c \ub9c9\uae30 \uc704\ud574 \uc11c\uc6b8\uc2dc\uccad\uc774 \ub3d9\uc6d0\ud55c \uac83\uc740 \ubb34\uc5c7\uc778\uac00?, context: \uc0c8\ub204\ub9ac\ub2f9 \uad00\uacc4\uc790\ub4e4\uc740 2013\ub144 8\uc6d4 2\uc77c\uc5d0 \uc11c\uc6b8\ud2b9\ubcc4\uc2dc\uccad\uc744 \ubc29\ubb38\ud558\uace0 \ubc15\uc6d0\uc21c \uc2dc\uc7a5\uc5d0\uac8c \uac74\uc124\ud604\uc7a5 \uc0ac\uace0\uc5d0 \uad00\ud55c \ucc45\uc784\uc744 \ubb3b\ub294 \ud56d\uc758 \uc11c\ud55c\uc744 \uc81c\ucd9c\ud558\ub824\uace0 \ud558\uc600\uc73c\ub098, \ubc15\uc6d0\uc21c\uacfc \uccad\uc6d0\uacbd\ucc30\uc758 \uac70\ubd80\ud589\uc704\ub85c \uc778\ud55c \ucda9\ub3cc\uc774 \uc788\uc5c8\ub2e4. \ud76c\uc0dd\uc790 \uc720\uac00\uc871\ub4e4\uc744 \ube44\ub86f\ud55c \uc0c8\ub204\ub9ac\ub2f9 \uad00\uacc4\uc790\ub4e4\uc740 \ubc15\uc6d0\uc21c\uc5d0\uac8c \ud56d\uc758 \uc11c\ud55c\uc744 \uc804\ub2ec\ud558\ub824\ub294 \ubaa9\uc801\uc774 \uc788\uc5c8\uc73c\ub098, \uc774\ub97c \uac70\ubd80\ud55c \uc11c\uc6b8\uc2dc\uccad \uce21\uc740 \uccad\uc6d0\uacbd\ucc30\uc744 \ub3d9\uc6d0\ud558\uc5ec \ubc29\ubb38\uc744 \ub9c9\uc558\uace0 \uc774 \uacfc\uc815\uc5d0\uc11c \ubab8\uc2f8\uc6c0\uc774 \ubc8c\uc5b4\uc838 \uccad\uc6d0\uacbd\ucc30\uc774 \ubd80\uc0c1\uc744 \uc785\uc5c8\ub2e4. \uc774\uc5d0 \ub300\ud574 \uc11c\uc6b8\uc2dc \uce21\uc740, \uc11c\uc6b8\uc2dc \uc9c1\uc6d0\uc774 \uc0c1\ud574\ub97c \uc785\uc740 \uac83\uc5d0 \ub300\ud574 \uc720\uac10\uc758 \ub73b\uc744 \ud45c\uba85\ud558\uc600\ub2e4. \ubc18\uba74 \uae40\uc131\ud0dc \uc758\uc6d0\uc2e4 \uce21\uc740, \uc11c\uc6b8\uc2dc\ubbfc\uc758 \uc548\uc804\uc744 \uc9c0\ud0a4\uc9c0 \ubabb\ud558\uace0 \uc720\uac00\uc871\uacfc \ubbfc\uc6d0\uc778\uc758 \ud56d\uc758 \ubc29\ubb38\uc744 \ud68c\ud53c\ud55c \ubc15\uc6d0\uc21c \uc2dc\uc7a5\uc774 \ucc45\uc784\uc744 \uc838\uc57c\ud55c\ub2e4\uace0 \uc8fc\uc7a5\ud558\uc600\ub2e4. \uc774\ub7ec\ud55c \uc77c\ub828\uc758 \uc0ac\uac74 \uc911 \uae30\ud0c0 \uc2e0\ubb38\uae30\uc0ac\ub098 \uc778\ud130\ub137 \ub4f1\uc9c0\uc5d0\uc11c \uc0c8\ub204\ub9ac\ub2f9 \uad00\uacc4\uc790\ub97c \uae40\uc131\ud0dc \uc758\uc6d0\uc2e4 \uad00\uacc4\uc790\ub85c \uc798\ubabb \ud45c\uae30\ud558\ub294 \uc624\ubcf4\uac00 \ubc1c\uc0dd\ud588\ub2e4."} {"question": "\uc218\ub3c4\uad8c\uc9c0\uc5ed\uc5d0 \ub3c4\uc2dc\ucca0\ub3c4\ub294 \ucd1d \uba87 \uac1c\uc758 \ub178\uc120\uc774 \uc6b4\ud589\ub418\uace0 \uc788\ub294\uac00?", "paragraph": "\ub300\ud55c\ubbfc\uad6d\uc758 \ub3c4\uc2dc\ucca0\ub3c4\ub294 \uc218\ub3c4\uad8c\uc9c0\uc5ed\uacfc \ubd80\uc0b0\uc9c0\uc5ed, \ub300\uad6c\uad11\uc5ed\uc2dc, \ub300\uc804\uad11\uc5ed\uc2dc, \uad11\uc8fc\uad11\uc5ed\uc2dc \ub4f1\uc5d0\uc11c \uc6b4\ud589\ub418\uace0 \uc788\ub2e4. \uc2dc\ub0b4\ub97c \uc6b4\ud589\ud558\ub294 \ub3c4\uc2dc\ucca0\ub3c4\ub97c \uc77c\uceec\uc5b4 '\uc9c0\ud558\ucca0' \ub610\ub294 '\uc804\ucca0' \ub4f1\uc73c\ub85c \ubd80\ub974\uae30\ub3c4 \ud55c\ub2e4. 1974\ub144 8\uc6d4 15\uc77c \ucc98\uc74c\uc73c\ub85c \uac1c\ud1b5\ub41c \uc218\ub3c4\uad8c\uc9c0\uc5ed\uc740 1~9\ud638\uc120\uacfc \ubd84\ub2f9\uc120, \uc911\uc559\uc120, \uacbd\uc758\uc120, \uacbd\ucd98\uc120, \uc778\ucc9c 1\ud638\uc120, \uc778\ucc9c\uad6d\uc81c\uacf5\ud56d\ucca0\ub3c4, \uc2e0\ubd84\ub2f9\uc120, \uc218\uc778\uc120, \uc6a9\uc778\uacbd\uc804\ucca0, \uc758\uc815\ubd80\uacbd\uc804\ucca0 \uae4c\uc9c0 \ucd1d 19\uac1c \ub178\uc120\uc774 \uc6b4\ud589\ub418\uace0 \uc788\uc73c\uba70 \uc804\uad6d\uc5d0\uc11c \uac00\uc7a5 \uaddc\ubaa8\uac00 \ud06c\ub2e4. \uae30\ubcf8\uc694\uae08\uc740 \ud3c9\uade0\uc801\uc73c\ub85c \uad50\ud1b5\uce74\ub4dc \uc774\uc6a9\uc2dc \ucd5c\ucd08 10km\uae4c\uc9c0 1050\uc6d0, \uadf8\uc774\ud6c4 5;km \ucd08\uacfc\uc2dc\ub9c8\ub2e4 100\uc6d0\uc529 \ucd94\uac00\ub41c\ub2e4. \uadf8\ub7ec\ub098 \ubbfc\uc601 \ub3c4\uc2dc\ucca0\ub3c4\uc778 \uc2e0\ubd84\ub2f9\uc120\uc740 \uae30\ubcf8\uc694\uae08\uc774 1,750\uc6d0\uc774\uace0, \ub2e4\ub978 \ub178\uc120\uc5d0\uc11c \uc2e0\ubd84\ub2f9\uc120\uc73c\ub85c \ud658\uc2b9\ud560 \uacbd\uc6b0 700\uc6d0\uc774 \ucd94\uac00\ub41c\ub2e4. \uc218\ub3c4\uad8c \uc804\ucca0\uc740 \uc778\ucc9c \ub3c4\uc2dc\ucca0\ub3c4, \uc778\ucc9c\uacf5\ud56d, \uae40\ud3ec\uacf5\ud56d\uacfc\ub3c4 \uc5f0\uacb0\ub41c\ub2e4. 1985\ub144 \uac1c\ud1b5\ub41c \ub450 \ubc88\uc9f8\ub85c \ud070 \ubd80\uc0b0 \ub3c4\uc2dc\ucca0\ub3c4\ub294 \ucd1d \uae38\uc774 131.7km\uc5d0 1\ud638\uc120, 2\ud638\uc120, 3\ud638\uc120, 4\ud638\uc120, \ubd80\uc0b0\uae40\ud574\uacbd\uc804\ucca0 5\uac1c \ub178\uc120\uc774 \uc6b4\ud589\ub418\uace0 \uc788\uc73c\uba70, \uc774\uc678\uc5d0\ub3c4 \ub300\uad6c\uc9c0\ud558\ucca0 1\ud638\uc120, 2\ud638\uc120 2\uac1c \ub178\uc120\uc774 \uc6b4\ud589\ub418\uace0 \ub300\uc804\uc9c0\ud558\ucca0 1\ud638\uc120 1\uac1c\ub178\uc120\uc774 \uc6b4\uc601, \uad11\uc8fc1\ud638\uc120 1\uac1c \ub178\uc120 \uc6b4\ud589 \ub4f1 \uc77c\ubd80 \uad11\uc5ed\uc2dc\ub4e4\uc744 \uc911\uc2ec\uc73c\ub85c \ub3c4\uc2dc\ucca0\ub3c4\uac00 \uacc4\uc18d\ub418\uc5b4 \uac74\uc124\uc911\uc774\ub2e4. \uc694\uae08\uc740 \uac01 \uc9c0\uc790\uccb4\uc5d0\uc11c \ub530\ub85c \uaddc\uc815\ud558\uc5ec \uc870\uae08\uc529 \ub2e4\ub974\ub2e4. \ubcf5\uc9c0 \uc815\ucc45\uc73c\ub85c 65\uc138 \uc774\uc0c1\uc758 \ub178\uc778\uacfc \uc7a5\uc560\uc778, \uad6d\uac00\uc720\uacf5\uc790\ub294 \ubb34\uc784\uc73c\ub85c \uc2b9\ucc28\ud560 \uc218 \uc788\ub2e4.(\ubd80\uc0b0-\uae40\ud574 \uacbd\uc804\ucca0\uc740 \ub178\uc778\ub3c4 \uc720\uc784\uc2b9\ucc28)", "answer": "19\uac1c", "sentence": "1974\ub144 8\uc6d4 15\uc77c \ucc98\uc74c\uc73c\ub85c \uac1c\ud1b5\ub41c \uc218\ub3c4\uad8c\uc9c0\uc5ed\uc740 1~9\ud638\uc120\uacfc \ubd84\ub2f9\uc120, \uc911\uc559\uc120, \uacbd\uc758\uc120, \uacbd\ucd98\uc120, \uc778\ucc9c 1\ud638\uc120, \uc778\ucc9c\uad6d\uc81c\uacf5\ud56d\ucca0\ub3c4, \uc2e0\ubd84\ub2f9\uc120, \uc218\uc778\uc120, \uc6a9\uc778\uacbd\uc804\ucca0, \uc758\uc815\ubd80\uacbd\uc804\ucca0 \uae4c\uc9c0 \ucd1d 19\uac1c \ub178\uc120\uc774 \uc6b4\ud589\ub418\uace0 \uc788\uc73c\uba70 \uc804\uad6d\uc5d0\uc11c \uac00\uc7a5 \uaddc\ubaa8\uac00 \ud06c\ub2e4.", "paragraph_sentence": "\ub300\ud55c\ubbfc\uad6d\uc758 \ub3c4\uc2dc\ucca0\ub3c4\ub294 \uc218\ub3c4\uad8c\uc9c0\uc5ed\uacfc \ubd80\uc0b0\uc9c0\uc5ed, \ub300\uad6c\uad11\uc5ed\uc2dc, \ub300\uc804\uad11\uc5ed\uc2dc, \uad11\uc8fc\uad11\uc5ed\uc2dc \ub4f1\uc5d0\uc11c \uc6b4\ud589\ub418\uace0 \uc788\ub2e4. \uc2dc\ub0b4\ub97c \uc6b4\ud589\ud558\ub294 \ub3c4\uc2dc\ucca0\ub3c4\ub97c \uc77c\uceec\uc5b4 '\uc9c0\ud558\ucca0' \ub610\ub294 '\uc804\ucca0' \ub4f1\uc73c\ub85c \ubd80\ub974\uae30\ub3c4 \ud55c\ub2e4. 1974\ub144 8\uc6d4 15\uc77c \ucc98\uc74c\uc73c\ub85c \uac1c\ud1b5\ub41c \uc218\ub3c4\uad8c\uc9c0\uc5ed\uc740 1~9\ud638\uc120\uacfc \ubd84\ub2f9\uc120, \uc911\uc559\uc120, \uacbd\uc758\uc120, \uacbd\ucd98\uc120, \uc778\ucc9c 1\ud638\uc120, \uc778\ucc9c\uad6d\uc81c\uacf5\ud56d\ucca0\ub3c4, \uc2e0\ubd84\ub2f9\uc120, \uc218\uc778\uc120, \uc6a9\uc778\uacbd\uc804\ucca0, \uc758\uc815\ubd80\uacbd\uc804\ucca0 \uae4c\uc9c0 \ucd1d 19\uac1c \ub178\uc120\uc774 \uc6b4\ud589\ub418\uace0 \uc788\uc73c\uba70 \uc804\uad6d\uc5d0\uc11c \uac00\uc7a5 \uaddc\ubaa8\uac00 \ud06c\ub2e4. \uae30\ubcf8\uc694\uae08\uc740 \ud3c9\uade0\uc801\uc73c\ub85c \uad50\ud1b5\uce74\ub4dc \uc774\uc6a9\uc2dc \ucd5c\ucd08 10km\uae4c\uc9c0 1050\uc6d0, \uadf8\uc774\ud6c4 5;km \ucd08\uacfc\uc2dc\ub9c8\ub2e4 100\uc6d0\uc529 \ucd94\uac00\ub41c\ub2e4. \uadf8\ub7ec\ub098 \ubbfc\uc601 \ub3c4\uc2dc\ucca0\ub3c4\uc778 \uc2e0\ubd84\ub2f9\uc120\uc740 \uae30\ubcf8\uc694\uae08\uc774 1,750\uc6d0\uc774\uace0, \ub2e4\ub978 \ub178\uc120\uc5d0\uc11c \uc2e0\ubd84\ub2f9\uc120\uc73c\ub85c \ud658\uc2b9\ud560 \uacbd\uc6b0 700\uc6d0\uc774 \ucd94\uac00\ub41c\ub2e4. \uc218\ub3c4\uad8c \uc804\ucca0\uc740 \uc778\ucc9c \ub3c4\uc2dc\ucca0\ub3c4, \uc778\ucc9c\uacf5\ud56d, \uae40\ud3ec\uacf5\ud56d\uacfc\ub3c4 \uc5f0\uacb0\ub41c\ub2e4. 1985\ub144 \uac1c\ud1b5\ub41c \ub450 \ubc88\uc9f8\ub85c \ud070 \ubd80\uc0b0 \ub3c4\uc2dc\ucca0\ub3c4\ub294 \ucd1d \uae38\uc774 131.7km\uc5d0 1\ud638\uc120, 2\ud638\uc120, 3\ud638\uc120, 4\ud638\uc120, \ubd80\uc0b0\uae40\ud574\uacbd\uc804\ucca0 5\uac1c \ub178\uc120\uc774 \uc6b4\ud589\ub418\uace0 \uc788\uc73c\uba70, \uc774\uc678\uc5d0\ub3c4 \ub300\uad6c\uc9c0\ud558\ucca0 1\ud638\uc120, 2\ud638\uc120 2\uac1c \ub178\uc120\uc774 \uc6b4\ud589\ub418\uace0 \ub300\uc804\uc9c0\ud558\ucca0 1\ud638\uc120 1\uac1c\ub178\uc120\uc774 \uc6b4\uc601, \uad11\uc8fc1\ud638\uc120 1\uac1c \ub178\uc120 \uc6b4\ud589 \ub4f1 \uc77c\ubd80 \uad11\uc5ed\uc2dc\ub4e4\uc744 \uc911\uc2ec\uc73c\ub85c \ub3c4\uc2dc\ucca0\ub3c4\uac00 \uacc4\uc18d\ub418\uc5b4 \uac74\uc124\uc911\uc774\ub2e4. \uc694\uae08\uc740 \uac01 \uc9c0\uc790\uccb4\uc5d0\uc11c \ub530\ub85c \uaddc\uc815\ud558\uc5ec \uc870\uae08\uc529 \ub2e4\ub974\ub2e4. \ubcf5\uc9c0 \uc815\ucc45\uc73c\ub85c 65\uc138 \uc774\uc0c1\uc758 \ub178\uc778\uacfc \uc7a5\uc560\uc778, \uad6d\uac00\uc720\uacf5\uc790\ub294 \ubb34\uc784\uc73c\ub85c \uc2b9\ucc28\ud560 \uc218 \uc788\ub2e4.(\ubd80\uc0b0-\uae40\ud574 \uacbd\uc804\ucca0\uc740 \ub178\uc778\ub3c4 \uc720\uc784\uc2b9\ucc28)", "paragraph_answer": "\ub300\ud55c\ubbfc\uad6d\uc758 \ub3c4\uc2dc\ucca0\ub3c4\ub294 \uc218\ub3c4\uad8c\uc9c0\uc5ed\uacfc \ubd80\uc0b0\uc9c0\uc5ed, \ub300\uad6c\uad11\uc5ed\uc2dc, \ub300\uc804\uad11\uc5ed\uc2dc, \uad11\uc8fc\uad11\uc5ed\uc2dc \ub4f1\uc5d0\uc11c \uc6b4\ud589\ub418\uace0 \uc788\ub2e4. \uc2dc\ub0b4\ub97c \uc6b4\ud589\ud558\ub294 \ub3c4\uc2dc\ucca0\ub3c4\ub97c \uc77c\uceec\uc5b4 '\uc9c0\ud558\ucca0' \ub610\ub294 '\uc804\ucca0' \ub4f1\uc73c\ub85c \ubd80\ub974\uae30\ub3c4 \ud55c\ub2e4. 1974\ub144 8\uc6d4 15\uc77c \ucc98\uc74c\uc73c\ub85c \uac1c\ud1b5\ub41c \uc218\ub3c4\uad8c\uc9c0\uc5ed\uc740 1~9\ud638\uc120\uacfc \ubd84\ub2f9\uc120, \uc911\uc559\uc120, \uacbd\uc758\uc120, \uacbd\ucd98\uc120, \uc778\ucc9c 1\ud638\uc120, \uc778\ucc9c\uad6d\uc81c\uacf5\ud56d\ucca0\ub3c4, \uc2e0\ubd84\ub2f9\uc120, \uc218\uc778\uc120, \uc6a9\uc778\uacbd\uc804\ucca0, \uc758\uc815\ubd80\uacbd\uc804\ucca0 \uae4c\uc9c0 \ucd1d 19\uac1c \ub178\uc120\uc774 \uc6b4\ud589\ub418\uace0 \uc788\uc73c\uba70 \uc804\uad6d\uc5d0\uc11c \uac00\uc7a5 \uaddc\ubaa8\uac00 \ud06c\ub2e4. \uae30\ubcf8\uc694\uae08\uc740 \ud3c9\uade0\uc801\uc73c\ub85c \uad50\ud1b5\uce74\ub4dc \uc774\uc6a9\uc2dc \ucd5c\ucd08 10km\uae4c\uc9c0 1050\uc6d0, \uadf8\uc774\ud6c4 5;km \ucd08\uacfc\uc2dc\ub9c8\ub2e4 100\uc6d0\uc529 \ucd94\uac00\ub41c\ub2e4. \uadf8\ub7ec\ub098 \ubbfc\uc601 \ub3c4\uc2dc\ucca0\ub3c4\uc778 \uc2e0\ubd84\ub2f9\uc120\uc740 \uae30\ubcf8\uc694\uae08\uc774 1,750\uc6d0\uc774\uace0, \ub2e4\ub978 \ub178\uc120\uc5d0\uc11c \uc2e0\ubd84\ub2f9\uc120\uc73c\ub85c \ud658\uc2b9\ud560 \uacbd\uc6b0 700\uc6d0\uc774 \ucd94\uac00\ub41c\ub2e4. \uc218\ub3c4\uad8c \uc804\ucca0\uc740 \uc778\ucc9c \ub3c4\uc2dc\ucca0\ub3c4, \uc778\ucc9c\uacf5\ud56d, \uae40\ud3ec\uacf5\ud56d\uacfc\ub3c4 \uc5f0\uacb0\ub41c\ub2e4. 1985\ub144 \uac1c\ud1b5\ub41c \ub450 \ubc88\uc9f8\ub85c \ud070 \ubd80\uc0b0 \ub3c4\uc2dc\ucca0\ub3c4\ub294 \ucd1d \uae38\uc774 131.7km\uc5d0 1\ud638\uc120, 2\ud638\uc120, 3\ud638\uc120, 4\ud638\uc120, \ubd80\uc0b0\uae40\ud574\uacbd\uc804\ucca0 5\uac1c \ub178\uc120\uc774 \uc6b4\ud589\ub418\uace0 \uc788\uc73c\uba70, \uc774\uc678\uc5d0\ub3c4 \ub300\uad6c\uc9c0\ud558\ucca0 1\ud638\uc120, 2\ud638\uc120 2\uac1c \ub178\uc120\uc774 \uc6b4\ud589\ub418\uace0 \ub300\uc804\uc9c0\ud558\ucca0 1\ud638\uc120 1\uac1c\ub178\uc120\uc774 \uc6b4\uc601, \uad11\uc8fc1\ud638\uc120 1\uac1c \ub178\uc120 \uc6b4\ud589 \ub4f1 \uc77c\ubd80 \uad11\uc5ed\uc2dc\ub4e4\uc744 \uc911\uc2ec\uc73c\ub85c \ub3c4\uc2dc\ucca0\ub3c4\uac00 \uacc4\uc18d\ub418\uc5b4 \uac74\uc124\uc911\uc774\ub2e4. \uc694\uae08\uc740 \uac01 \uc9c0\uc790\uccb4\uc5d0\uc11c \ub530\ub85c \uaddc\uc815\ud558\uc5ec \uc870\uae08\uc529 \ub2e4\ub974\ub2e4. \ubcf5\uc9c0 \uc815\ucc45\uc73c\ub85c 65\uc138 \uc774\uc0c1\uc758 \ub178\uc778\uacfc \uc7a5\uc560\uc778, \uad6d\uac00\uc720\uacf5\uc790\ub294 \ubb34\uc784\uc73c\ub85c \uc2b9\ucc28\ud560 \uc218 \uc788\ub2e4.(\ubd80\uc0b0-\uae40\ud574 \uacbd\uc804\ucca0\uc740 \ub178\uc778\ub3c4 \uc720\uc784\uc2b9\ucc28)", "sentence_answer": "1974\ub144 8\uc6d4 15\uc77c \ucc98\uc74c\uc73c\ub85c \uac1c\ud1b5\ub41c \uc218\ub3c4\uad8c\uc9c0\uc5ed\uc740 1~9\ud638\uc120\uacfc \ubd84\ub2f9\uc120, \uc911\uc559\uc120, \uacbd\uc758\uc120, \uacbd\ucd98\uc120, \uc778\ucc9c 1\ud638\uc120, \uc778\ucc9c\uad6d\uc81c\uacf5\ud56d\ucca0\ub3c4, \uc2e0\ubd84\ub2f9\uc120, \uc218\uc778\uc120, \uc6a9\uc778\uacbd\uc804\ucca0, \uc758\uc815\ubd80\uacbd\uc804\ucca0 \uae4c\uc9c0 \ucd1d 19\uac1c \ub178\uc120\uc774 \uc6b4\ud589\ub418\uace0 \uc788\uc73c\uba70 \uc804\uad6d\uc5d0\uc11c \uac00\uc7a5 \uaddc\ubaa8\uac00 \ud06c\ub2e4.", "paragraph_id": "6490879-27-1", "paragraph_question": "question: \uc218\ub3c4\uad8c\uc9c0\uc5ed\uc5d0 \ub3c4\uc2dc\ucca0\ub3c4\ub294 \ucd1d \uba87 \uac1c\uc758 \ub178\uc120\uc774 \uc6b4\ud589\ub418\uace0 \uc788\ub294\uac00?, context: \ub300\ud55c\ubbfc\uad6d\uc758 \ub3c4\uc2dc\ucca0\ub3c4\ub294 \uc218\ub3c4\uad8c\uc9c0\uc5ed\uacfc \ubd80\uc0b0\uc9c0\uc5ed, \ub300\uad6c\uad11\uc5ed\uc2dc, \ub300\uc804\uad11\uc5ed\uc2dc, \uad11\uc8fc\uad11\uc5ed\uc2dc \ub4f1\uc5d0\uc11c \uc6b4\ud589\ub418\uace0 \uc788\ub2e4. \uc2dc\ub0b4\ub97c \uc6b4\ud589\ud558\ub294 \ub3c4\uc2dc\ucca0\ub3c4\ub97c \uc77c\uceec\uc5b4 '\uc9c0\ud558\ucca0' \ub610\ub294 '\uc804\ucca0' \ub4f1\uc73c\ub85c \ubd80\ub974\uae30\ub3c4 \ud55c\ub2e4. 1974\ub144 8\uc6d4 15\uc77c \ucc98\uc74c\uc73c\ub85c \uac1c\ud1b5\ub41c \uc218\ub3c4\uad8c\uc9c0\uc5ed\uc740 1~9\ud638\uc120\uacfc \ubd84\ub2f9\uc120, \uc911\uc559\uc120, \uacbd\uc758\uc120, \uacbd\ucd98\uc120, \uc778\ucc9c 1\ud638\uc120, \uc778\ucc9c\uad6d\uc81c\uacf5\ud56d\ucca0\ub3c4, \uc2e0\ubd84\ub2f9\uc120, \uc218\uc778\uc120, \uc6a9\uc778\uacbd\uc804\ucca0, \uc758\uc815\ubd80\uacbd\uc804\ucca0 \uae4c\uc9c0 \ucd1d 19\uac1c \ub178\uc120\uc774 \uc6b4\ud589\ub418\uace0 \uc788\uc73c\uba70 \uc804\uad6d\uc5d0\uc11c \uac00\uc7a5 \uaddc\ubaa8\uac00 \ud06c\ub2e4. \uae30\ubcf8\uc694\uae08\uc740 \ud3c9\uade0\uc801\uc73c\ub85c \uad50\ud1b5\uce74\ub4dc \uc774\uc6a9\uc2dc \ucd5c\ucd08 10km\uae4c\uc9c0 1050\uc6d0, \uadf8\uc774\ud6c4 5;km \ucd08\uacfc\uc2dc\ub9c8\ub2e4 100\uc6d0\uc529 \ucd94\uac00\ub41c\ub2e4. \uadf8\ub7ec\ub098 \ubbfc\uc601 \ub3c4\uc2dc\ucca0\ub3c4\uc778 \uc2e0\ubd84\ub2f9\uc120\uc740 \uae30\ubcf8\uc694\uae08\uc774 1,750\uc6d0\uc774\uace0, \ub2e4\ub978 \ub178\uc120\uc5d0\uc11c \uc2e0\ubd84\ub2f9\uc120\uc73c\ub85c \ud658\uc2b9\ud560 \uacbd\uc6b0 700\uc6d0\uc774 \ucd94\uac00\ub41c\ub2e4. \uc218\ub3c4\uad8c \uc804\ucca0\uc740 \uc778\ucc9c \ub3c4\uc2dc\ucca0\ub3c4, \uc778\ucc9c\uacf5\ud56d, \uae40\ud3ec\uacf5\ud56d\uacfc\ub3c4 \uc5f0\uacb0\ub41c\ub2e4. 1985\ub144 \uac1c\ud1b5\ub41c \ub450 \ubc88\uc9f8\ub85c \ud070 \ubd80\uc0b0 \ub3c4\uc2dc\ucca0\ub3c4\ub294 \ucd1d \uae38\uc774 131.7km\uc5d0 1\ud638\uc120, 2\ud638\uc120, 3\ud638\uc120, 4\ud638\uc120, \ubd80\uc0b0\uae40\ud574\uacbd\uc804\ucca0 5\uac1c \ub178\uc120\uc774 \uc6b4\ud589\ub418\uace0 \uc788\uc73c\uba70, \uc774\uc678\uc5d0\ub3c4 \ub300\uad6c\uc9c0\ud558\ucca0 1\ud638\uc120, 2\ud638\uc120 2\uac1c \ub178\uc120\uc774 \uc6b4\ud589\ub418\uace0 \ub300\uc804\uc9c0\ud558\ucca0 1\ud638\uc120 1\uac1c\ub178\uc120\uc774 \uc6b4\uc601, \uad11\uc8fc1\ud638\uc120 1\uac1c \ub178\uc120 \uc6b4\ud589 \ub4f1 \uc77c\ubd80 \uad11\uc5ed\uc2dc\ub4e4\uc744 \uc911\uc2ec\uc73c\ub85c \ub3c4\uc2dc\ucca0\ub3c4\uac00 \uacc4\uc18d\ub418\uc5b4 \uac74\uc124\uc911\uc774\ub2e4. \uc694\uae08\uc740 \uac01 \uc9c0\uc790\uccb4\uc5d0\uc11c \ub530\ub85c \uaddc\uc815\ud558\uc5ec \uc870\uae08\uc529 \ub2e4\ub974\ub2e4. \ubcf5\uc9c0 \uc815\ucc45\uc73c\ub85c 65\uc138 \uc774\uc0c1\uc758 \ub178\uc778\uacfc \uc7a5\uc560\uc778, \uad6d\uac00\uc720\uacf5\uc790\ub294 \ubb34\uc784\uc73c\ub85c \uc2b9\ucc28\ud560 \uc218 \uc788\ub2e4.(\ubd80\uc0b0-\uae40\ud574 \uacbd\uc804\ucca0\uc740 \ub178\uc778\ub3c4 \uc720\uc784\uc2b9\ucc28)"} {"question": "2004\ub144\uc5d0 \ubbf8\uad6d\uc758 \ubca8\ub77c\ub8e8\uc2a4 \ubbfc\uc8fc\uc8fc\uc758 \ubc95\uc744 \ube44\ub09c\ud558\ub294 \ud3b8\uc9c0\uc5d0 \uc11c\uba85\ud55c \uc81c1\uc11c\uae30 \uc774\ub984\uc740?", "paragraph": "\ubca8\ub77c\ub8e8\uc2a4 \uacf5\ud654\uad6d \uccad\ub144\ub2e8\uc740 \ubca8\ub77c\ub8e8\uc2a4 \uccad\uc18c\ub144 \ucf58\uc11c\ud2b8\uc640 \uac19\uc740 \uc0ac\ud68c \ud589\uc0ac\ub97c \uc8fc\ucd5c\ud55c\ub2e4. \uadf8\ub7ec\ub098 \ubca8\ub77c\ub8e8\uc2a4 \uacf5\ud654\uad6d \uccad\ub144\ub2e8\uc774 \ud6c4\uc6d0\ud558\ub294 \ucf58\uc11c\ud2b8\uc5d0\uc11c\ub294 \uc2e0\ub098\uce58\uc8fc\uc758\uc790\ub4e4\ub3c4 \ucc38\uc11d\ud560 \ubfd0\ub9cc \uc544\ub2c8\ub77c \uc774\ub4e4\uc758 \uacf5\uc5f0\uc744 \ubcf4\uc5ec\uc8fc\uae30\ub3c4 \ud55c\ub2e4. \uc774\ub85c \ubca8\ub77c\ub8e8\uc2a4 \uacf5\ud654\uad6d \uccad\ub144\ub2e8\uc740 \uc9c0\uc5ed \uc9c0\ub3c4\uc790\ub4e4\uacfc \ud1f4\uc5ed \uad70\uc778\ub4e4\uc5d0\uac8c \ub9ce\uc740 \ube44\ub09c\uc744 \ubc1b\uae30\ub3c4 \ud55c\ub2e4. \ub610\ud55c \ubca8\ub77c\ub8e8\uc2a4 \uacf5\ud654\uad6d \uccad\ub144\ub2e8\uc740 2004\ub144\uc5d0 \ubbf8\uc2a4 \ubca8\ub77c\ub8e8\uc2a4 \uc120\ubc1c \ub300\ud68c\uc758 \uc8fc\ucd5c \uac00\uc6b4\ub370 \ud558\ub098\ub85c \ub098\uc11c\uba74\uc11c \ubbf8\uc2a4 \uc544\uba54\ub9ac\uce74\uc640 \ubbf8\uc2a4 \uc720\ub2c8\ubc84\uc2a4\uc640 \uac19\uc740 \ubc29\uc2dd\uc73c\ub85c \uc9c4\ud589\ud558\uae30\ub3c4 \ud558\uc600\uc73c\uba70 \ubca8\ub77c\ub8e8\uc2a4 \uacf5\ud654\uad6d \uccad\ub144\ub2e8\uc774 \uc815\uce58\uc5d0 \uac1c\uc785\ud558\uc9c0 \uc54a\ub294 \ub3d9\uc548 \uc81c1\uc11c\uae30\ub97c \uc9c0\ub0c8\ub358 \ubbf8\ud558\uc77c \uc624\ub974\ub2e4\ub294 \uc5ec\ub7ec \uacf5\ubb34\uc6d0\ub4e4\uacfc \ud568\uaed8 \ubbf8\uad6d\uc758 2004\ub144 \ubca8\ub77c\ub8e8\uc2a4 \ubbfc\uc8fc\uc8fc\uc758 \ubc95\uc744 \ube44\ub09c\ud558\ub294 \ud3b8\uc9c0\uc5d0 \uc11c\uba85\ud558\uae30\ub3c4 \ud558\uc600\ub2e4. \ubca8\ub77c\ub8e8\uc2a4 \uacf5\ud654\uad6d \uccad\ub144\ub2e8\uc758 \ud559\uc0dd \uac74\uc124 \uc5ec\ub2e8\uc740 \uc18c\ube44\uc5d0\ud2b8 \uc5f0\ubc29\uc758 \uccad\ub144 \ub2e8\uccb4\uc5d0 \uae30\uc6d0\uc744 \ub454\ub2e4.", "answer": "\ubbf8\ud558\uc77c \uc624\ub974\ub2e4", "sentence": "\ub610\ud55c \ubca8\ub77c\ub8e8\uc2a4 \uacf5\ud654\uad6d \uccad\ub144\ub2e8\uc740 2004\ub144\uc5d0 \ubbf8\uc2a4 \ubca8\ub77c\ub8e8\uc2a4 \uc120\ubc1c \ub300\ud68c\uc758 \uc8fc\ucd5c \uac00\uc6b4\ub370 \ud558\ub098\ub85c \ub098\uc11c\uba74\uc11c \ubbf8\uc2a4 \uc544\uba54\ub9ac\uce74\uc640 \ubbf8\uc2a4 \uc720\ub2c8\ubc84\uc2a4\uc640 \uac19\uc740 \ubc29\uc2dd\uc73c\ub85c \uc9c4\ud589\ud558\uae30\ub3c4 \ud558\uc600\uc73c\uba70 \ubca8\ub77c\ub8e8\uc2a4 \uacf5\ud654\uad6d \uccad\ub144\ub2e8\uc774 \uc815\uce58\uc5d0 \uac1c\uc785\ud558\uc9c0 \uc54a\ub294 \ub3d9\uc548 \uc81c1\uc11c\uae30\ub97c \uc9c0\ub0c8\ub358 \ubbf8\ud558\uc77c \uc624\ub974\ub2e4 \ub294 \uc5ec\ub7ec \uacf5\ubb34\uc6d0\ub4e4\uacfc \ud568\uaed8 \ubbf8\uad6d\uc758 2004\ub144 \ubca8\ub77c\ub8e8\uc2a4 \ubbfc\uc8fc\uc8fc\uc758 \ubc95\uc744 \ube44\ub09c\ud558\ub294 \ud3b8\uc9c0\uc5d0 \uc11c\uba85\ud558\uae30\ub3c4 \ud558\uc600\ub2e4.", "paragraph_sentence": "\ubca8\ub77c\ub8e8\uc2a4 \uacf5\ud654\uad6d \uccad\ub144\ub2e8\uc740 \ubca8\ub77c\ub8e8\uc2a4 \uccad\uc18c\ub144 \ucf58\uc11c\ud2b8\uc640 \uac19\uc740 \uc0ac\ud68c \ud589\uc0ac\ub97c \uc8fc\ucd5c\ud55c\ub2e4. \uadf8\ub7ec\ub098 \ubca8\ub77c\ub8e8\uc2a4 \uacf5\ud654\uad6d \uccad\ub144\ub2e8\uc774 \ud6c4\uc6d0\ud558\ub294 \ucf58\uc11c\ud2b8\uc5d0\uc11c\ub294 \uc2e0\ub098\uce58\uc8fc\uc758\uc790\ub4e4\ub3c4 \ucc38\uc11d\ud560 \ubfd0\ub9cc \uc544\ub2c8\ub77c \uc774\ub4e4\uc758 \uacf5\uc5f0\uc744 \ubcf4\uc5ec\uc8fc\uae30\ub3c4 \ud55c\ub2e4. \uc774\ub85c \ubca8\ub77c\ub8e8\uc2a4 \uacf5\ud654\uad6d \uccad\ub144\ub2e8\uc740 \uc9c0\uc5ed \uc9c0\ub3c4\uc790\ub4e4\uacfc \ud1f4\uc5ed \uad70\uc778\ub4e4\uc5d0\uac8c \ub9ce\uc740 \ube44\ub09c\uc744 \ubc1b\uae30\ub3c4 \ud55c\ub2e4. \ub610\ud55c \ubca8\ub77c\ub8e8\uc2a4 \uacf5\ud654\uad6d \uccad\ub144\ub2e8\uc740 2004\ub144\uc5d0 \ubbf8\uc2a4 \ubca8\ub77c\ub8e8\uc2a4 \uc120\ubc1c \ub300\ud68c\uc758 \uc8fc\ucd5c \uac00\uc6b4\ub370 \ud558\ub098\ub85c \ub098\uc11c\uba74\uc11c \ubbf8\uc2a4 \uc544\uba54\ub9ac\uce74\uc640 \ubbf8\uc2a4 \uc720\ub2c8\ubc84\uc2a4\uc640 \uac19\uc740 \ubc29\uc2dd\uc73c\ub85c \uc9c4\ud589\ud558\uae30\ub3c4 \ud558\uc600\uc73c\uba70 \ubca8\ub77c\ub8e8\uc2a4 \uacf5\ud654\uad6d \uccad\ub144\ub2e8\uc774 \uc815\uce58\uc5d0 \uac1c\uc785\ud558\uc9c0 \uc54a\ub294 \ub3d9\uc548 \uc81c1\uc11c\uae30\ub97c \uc9c0\ub0c8\ub358 \ubbf8\ud558\uc77c \uc624\ub974\ub2e4 \ub294 \uc5ec\ub7ec \uacf5\ubb34\uc6d0\ub4e4\uacfc \ud568\uaed8 \ubbf8\uad6d\uc758 2004\ub144 \ubca8\ub77c\ub8e8\uc2a4 \ubbfc\uc8fc\uc8fc\uc758 \ubc95\uc744 \ube44\ub09c\ud558\ub294 \ud3b8\uc9c0\uc5d0 \uc11c\uba85\ud558\uae30\ub3c4 \ud558\uc600\ub2e4. \ubca8\ub77c\ub8e8\uc2a4 \uacf5\ud654\uad6d \uccad\ub144\ub2e8\uc758 \ud559\uc0dd \uac74\uc124 \uc5ec\ub2e8\uc740 \uc18c\ube44\uc5d0\ud2b8 \uc5f0\ubc29\uc758 \uccad\ub144 \ub2e8\uccb4\uc5d0 \uae30\uc6d0\uc744 \ub454\ub2e4.", "paragraph_answer": "\ubca8\ub77c\ub8e8\uc2a4 \uacf5\ud654\uad6d \uccad\ub144\ub2e8\uc740 \ubca8\ub77c\ub8e8\uc2a4 \uccad\uc18c\ub144 \ucf58\uc11c\ud2b8\uc640 \uac19\uc740 \uc0ac\ud68c \ud589\uc0ac\ub97c \uc8fc\ucd5c\ud55c\ub2e4. \uadf8\ub7ec\ub098 \ubca8\ub77c\ub8e8\uc2a4 \uacf5\ud654\uad6d \uccad\ub144\ub2e8\uc774 \ud6c4\uc6d0\ud558\ub294 \ucf58\uc11c\ud2b8\uc5d0\uc11c\ub294 \uc2e0\ub098\uce58\uc8fc\uc758\uc790\ub4e4\ub3c4 \ucc38\uc11d\ud560 \ubfd0\ub9cc \uc544\ub2c8\ub77c \uc774\ub4e4\uc758 \uacf5\uc5f0\uc744 \ubcf4\uc5ec\uc8fc\uae30\ub3c4 \ud55c\ub2e4. \uc774\ub85c \ubca8\ub77c\ub8e8\uc2a4 \uacf5\ud654\uad6d \uccad\ub144\ub2e8\uc740 \uc9c0\uc5ed \uc9c0\ub3c4\uc790\ub4e4\uacfc \ud1f4\uc5ed \uad70\uc778\ub4e4\uc5d0\uac8c \ub9ce\uc740 \ube44\ub09c\uc744 \ubc1b\uae30\ub3c4 \ud55c\ub2e4. \ub610\ud55c \ubca8\ub77c\ub8e8\uc2a4 \uacf5\ud654\uad6d \uccad\ub144\ub2e8\uc740 2004\ub144\uc5d0 \ubbf8\uc2a4 \ubca8\ub77c\ub8e8\uc2a4 \uc120\ubc1c \ub300\ud68c\uc758 \uc8fc\ucd5c \uac00\uc6b4\ub370 \ud558\ub098\ub85c \ub098\uc11c\uba74\uc11c \ubbf8\uc2a4 \uc544\uba54\ub9ac\uce74\uc640 \ubbf8\uc2a4 \uc720\ub2c8\ubc84\uc2a4\uc640 \uac19\uc740 \ubc29\uc2dd\uc73c\ub85c \uc9c4\ud589\ud558\uae30\ub3c4 \ud558\uc600\uc73c\uba70 \ubca8\ub77c\ub8e8\uc2a4 \uacf5\ud654\uad6d \uccad\ub144\ub2e8\uc774 \uc815\uce58\uc5d0 \uac1c\uc785\ud558\uc9c0 \uc54a\ub294 \ub3d9\uc548 \uc81c1\uc11c\uae30\ub97c \uc9c0\ub0c8\ub358 \ubbf8\ud558\uc77c \uc624\ub974\ub2e4 \ub294 \uc5ec\ub7ec \uacf5\ubb34\uc6d0\ub4e4\uacfc \ud568\uaed8 \ubbf8\uad6d\uc758 2004\ub144 \ubca8\ub77c\ub8e8\uc2a4 \ubbfc\uc8fc\uc8fc\uc758 \ubc95\uc744 \ube44\ub09c\ud558\ub294 \ud3b8\uc9c0\uc5d0 \uc11c\uba85\ud558\uae30\ub3c4 \ud558\uc600\ub2e4. \ubca8\ub77c\ub8e8\uc2a4 \uacf5\ud654\uad6d \uccad\ub144\ub2e8\uc758 \ud559\uc0dd \uac74\uc124 \uc5ec\ub2e8\uc740 \uc18c\ube44\uc5d0\ud2b8 \uc5f0\ubc29\uc758 \uccad\ub144 \ub2e8\uccb4\uc5d0 \uae30\uc6d0\uc744 \ub454\ub2e4.", "sentence_answer": "\ub610\ud55c \ubca8\ub77c\ub8e8\uc2a4 \uacf5\ud654\uad6d \uccad\ub144\ub2e8\uc740 2004\ub144\uc5d0 \ubbf8\uc2a4 \ubca8\ub77c\ub8e8\uc2a4 \uc120\ubc1c \ub300\ud68c\uc758 \uc8fc\ucd5c \uac00\uc6b4\ub370 \ud558\ub098\ub85c \ub098\uc11c\uba74\uc11c \ubbf8\uc2a4 \uc544\uba54\ub9ac\uce74\uc640 \ubbf8\uc2a4 \uc720\ub2c8\ubc84\uc2a4\uc640 \uac19\uc740 \ubc29\uc2dd\uc73c\ub85c \uc9c4\ud589\ud558\uae30\ub3c4 \ud558\uc600\uc73c\uba70 \ubca8\ub77c\ub8e8\uc2a4 \uacf5\ud654\uad6d \uccad\ub144\ub2e8\uc774 \uc815\uce58\uc5d0 \uac1c\uc785\ud558\uc9c0 \uc54a\ub294 \ub3d9\uc548 \uc81c1\uc11c\uae30\ub97c \uc9c0\ub0c8\ub358 \ubbf8\ud558\uc77c \uc624\ub974\ub2e4 \ub294 \uc5ec\ub7ec \uacf5\ubb34\uc6d0\ub4e4\uacfc \ud568\uaed8 \ubbf8\uad6d\uc758 2004\ub144 \ubca8\ub77c\ub8e8\uc2a4 \ubbfc\uc8fc\uc8fc\uc758 \ubc95\uc744 \ube44\ub09c\ud558\ub294 \ud3b8\uc9c0\uc5d0 \uc11c\uba85\ud558\uae30\ub3c4 \ud558\uc600\ub2e4.", "paragraph_id": "6657150-0-1", "paragraph_question": "question: 2004\ub144\uc5d0 \ubbf8\uad6d\uc758 \ubca8\ub77c\ub8e8\uc2a4 \ubbfc\uc8fc\uc8fc\uc758 \ubc95\uc744 \ube44\ub09c\ud558\ub294 \ud3b8\uc9c0\uc5d0 \uc11c\uba85\ud55c \uc81c1\uc11c\uae30 \uc774\ub984\uc740?, context: \ubca8\ub77c\ub8e8\uc2a4 \uacf5\ud654\uad6d \uccad\ub144\ub2e8\uc740 \ubca8\ub77c\ub8e8\uc2a4 \uccad\uc18c\ub144 \ucf58\uc11c\ud2b8\uc640 \uac19\uc740 \uc0ac\ud68c \ud589\uc0ac\ub97c \uc8fc\ucd5c\ud55c\ub2e4. \uadf8\ub7ec\ub098 \ubca8\ub77c\ub8e8\uc2a4 \uacf5\ud654\uad6d \uccad\ub144\ub2e8\uc774 \ud6c4\uc6d0\ud558\ub294 \ucf58\uc11c\ud2b8\uc5d0\uc11c\ub294 \uc2e0\ub098\uce58\uc8fc\uc758\uc790\ub4e4\ub3c4 \ucc38\uc11d\ud560 \ubfd0\ub9cc \uc544\ub2c8\ub77c \uc774\ub4e4\uc758 \uacf5\uc5f0\uc744 \ubcf4\uc5ec\uc8fc\uae30\ub3c4 \ud55c\ub2e4. \uc774\ub85c \ubca8\ub77c\ub8e8\uc2a4 \uacf5\ud654\uad6d \uccad\ub144\ub2e8\uc740 \uc9c0\uc5ed \uc9c0\ub3c4\uc790\ub4e4\uacfc \ud1f4\uc5ed \uad70\uc778\ub4e4\uc5d0\uac8c \ub9ce\uc740 \ube44\ub09c\uc744 \ubc1b\uae30\ub3c4 \ud55c\ub2e4. \ub610\ud55c \ubca8\ub77c\ub8e8\uc2a4 \uacf5\ud654\uad6d \uccad\ub144\ub2e8\uc740 2004\ub144\uc5d0 \ubbf8\uc2a4 \ubca8\ub77c\ub8e8\uc2a4 \uc120\ubc1c \ub300\ud68c\uc758 \uc8fc\ucd5c \uac00\uc6b4\ub370 \ud558\ub098\ub85c \ub098\uc11c\uba74\uc11c \ubbf8\uc2a4 \uc544\uba54\ub9ac\uce74\uc640 \ubbf8\uc2a4 \uc720\ub2c8\ubc84\uc2a4\uc640 \uac19\uc740 \ubc29\uc2dd\uc73c\ub85c \uc9c4\ud589\ud558\uae30\ub3c4 \ud558\uc600\uc73c\uba70 \ubca8\ub77c\ub8e8\uc2a4 \uacf5\ud654\uad6d \uccad\ub144\ub2e8\uc774 \uc815\uce58\uc5d0 \uac1c\uc785\ud558\uc9c0 \uc54a\ub294 \ub3d9\uc548 \uc81c1\uc11c\uae30\ub97c \uc9c0\ub0c8\ub358 \ubbf8\ud558\uc77c \uc624\ub974\ub2e4\ub294 \uc5ec\ub7ec \uacf5\ubb34\uc6d0\ub4e4\uacfc \ud568\uaed8 \ubbf8\uad6d\uc758 2004\ub144 \ubca8\ub77c\ub8e8\uc2a4 \ubbfc\uc8fc\uc8fc\uc758 \ubc95\uc744 \ube44\ub09c\ud558\ub294 \ud3b8\uc9c0\uc5d0 \uc11c\uba85\ud558\uae30\ub3c4 \ud558\uc600\ub2e4. \ubca8\ub77c\ub8e8\uc2a4 \uacf5\ud654\uad6d \uccad\ub144\ub2e8\uc758 \ud559\uc0dd \uac74\uc124 \uc5ec\ub2e8\uc740 \uc18c\ube44\uc5d0\ud2b8 \uc5f0\ubc29\uc758 \uccad\ub144 \ub2e8\uccb4\uc5d0 \uae30\uc6d0\uc744 \ub454\ub2e4."} {"question": "2015\ub144 12\uc6d4 11\uc77c \uc11c\uc6b8\ub0a8\ub300\ubb38\uacbd\ucc30\uc11c\uc5d0\uc11c \u3131\uc528\uac00 \uc870\uc0ac\ubc1b\uc740 \uc2dc\uac04\uc740?", "paragraph": "2016\ub144 10\uc6d4 19\uc77c \uc11c\uc6b8\ud2b9\ubcc4\uc2dc \uc911\uad6c \ud504\ub808\uc2a4\uc13c\ud130\uc5d0\uc11c '\ube68\uac04\uc6b0\uc758' \ub2f9\uc0ac\uc790\uc778 \uc804\uad6d\uacf5\uacf5\uc6b4\uc218\ub178\ub3d9\uc870\ud569 \uad11\uc8fc\u2027\uc804\ub0a8\uc9c0\ubd80 \uac04\ubd80 \ucd9c\uc2e0 \uc870\ud569\uc6d0\uc778 40\ub300 \u3131\uc528\uac00 \uae30\uc790\ud68c\uacac\uc744 \uc5f4\uc5b4 \uacf5\uc2dd \uc785\uc7a5\uc744 \ubc1d\ud614\ub2e4. \u3131\uc528\ub294 2015\ub144 12\uc6d4 11\uc77c \uc11c\uc6b8\ub0a8\ub300\ubb38\uacbd\ucc30\uc11c\uc5d0\uc11c 4\uc2dc\uac04 \ub3d9\uc548 \uc870\uc0ac\ub97c \ubc1b\uac8c \ub418\uc5c8\ub2e4. \u3131\uc528\ub294 \uc870\uc0ac \ub2f9\uc2dc \uacbd\ucc30\uc740 \ubbfc\uc911\ucd1d\uad90\uae30\ub54c \uc790\uc2e0\uc774 \ube68\uac04 \uc6b0\uc758\ub97c \ucc29\uc6a9\ud558\uace0 \uc788\uc5c8\ub2e4\ub294 \uac83\uc744 \uc54c\uace0 \uc788\uc5c8\uace0, \ucc44\uc99d\ud55c \uc99d\uac70\uc790\ub8cc\ub97c \ubcf4\uc5ec\uc8fc\uba74\uc11c \uc870\uc0ac\ud558\uc600\uc73c\uba70, \"\uc608\uc0c1 \ubc16\uc73c\ub85c \ubc31\ub0a8\uae30 \uc5b4\ub974\uc2e0\uacfc \uad00\ub828\ub41c (\ubb3c\ub300\ud3ec\uc5d0 \uc4f0\ub7ec\uc9c4) \uc2dc\uac04\ub300 \uc0ac\uc9c4\ub9cc \uc81c\uc2dc\ud558\uc9c0 \uc54a\uc558\uace0, \ubb3b\uc9c0\ub3c4 \uc54a\uc558\ub2e4. \uc55e\ub4a4 \uc0c1\ud669\uc740 \ub2e4 \ubb3c\uc5c8\ub2e4\" \uace0 \ubc1d\ud614\ub2e4.\u3131\uc528\ub294 \u201c\uadf8\ub3d9\uc548 \uc77c\ubca0 \ub4f1 \uc77c\ubd80\uc758 (\ube68\uac04\uc6b0\uc758 \uc870\ud569\uc6d0\uc774 \ubc31\ub0a8\uae30 \ub18d\ubbfc\uc744 \uac00\uaca9\ud574 \uc0ac\ub9dd\uc758 \uc6d0\uc778\uc774 \ub418\uc5c8\ub2e4\ub294) \uc8fc\uc7a5\uc740 \ub108\ubb34 \uc5c9\ud130\ub9ac\ub77c \uad73\uc774 \ub300\uc751\ud558\uc5ec \uad6d\uac00\ud3ed\ub825 \uc0b4\uc778\uc774\ub77c\ub294 \ucd08\uc810\uc744 \ud750\ub9ac\uae30\ub97c \ubc14\ub77c\uc9c0 \uc54a\uc544 \uce68\ubb35\ud588\ub2e4\u201d\uace0 \ubc1d\ud614\ub2e4. \uadf8\ub294 \ubc31\ub0a8\uae30 \uc528\uc5d0 \ub300\ud574 \"\ucc98\uc74c \ubd59\ub294 \ubd84\uc774\uc600\ub2e4. \uc0ac\ub294 \uc9c0\uc5ed\uc774 \uac19\ub2e4\ub294 \uac83\uc740 \ub098\uc911\uc5d0 \uadf8\ubd84\uc758 \uc2e0\uc0c1\uc774 \ubcf4\ub3c4\ub418\uba74\uc11c \uc54c\uc558\ub2e4\u201d\uace0 \ubc1d\ud614\ub2e4.", "answer": "4\uc2dc\uac04", "sentence": "\u3131\uc528\ub294 2015\ub144 12\uc6d4 11\uc77c \uc11c\uc6b8\ub0a8\ub300\ubb38\uacbd\ucc30\uc11c\uc5d0\uc11c 4\uc2dc\uac04 \ub3d9\uc548 \uc870\uc0ac\ub97c \ubc1b\uac8c \ub418\uc5c8\ub2e4.", "paragraph_sentence": "2016\ub144 10\uc6d4 19\uc77c \uc11c\uc6b8\ud2b9\ubcc4\uc2dc \uc911\uad6c \ud504\ub808\uc2a4\uc13c\ud130\uc5d0\uc11c '\ube68\uac04\uc6b0\uc758' \ub2f9\uc0ac\uc790\uc778 \uc804\uad6d\uacf5\uacf5\uc6b4\uc218\ub178\ub3d9\uc870\ud569 \uad11\uc8fc\u2027\uc804\ub0a8\uc9c0\ubd80 \uac04\ubd80 \ucd9c\uc2e0 \uc870\ud569\uc6d0\uc778 40\ub300 \u3131\uc528\uac00 \uae30\uc790\ud68c\uacac\uc744 \uc5f4\uc5b4 \uacf5\uc2dd \uc785\uc7a5\uc744 \ubc1d\ud614\ub2e4. \u3131\uc528\ub294 2015\ub144 12\uc6d4 11\uc77c \uc11c\uc6b8\ub0a8\ub300\ubb38\uacbd\ucc30\uc11c\uc5d0\uc11c 4\uc2dc\uac04 \ub3d9\uc548 \uc870\uc0ac\ub97c \ubc1b\uac8c \ub418\uc5c8\ub2e4. \u3131\uc528\ub294 \uc870\uc0ac \ub2f9\uc2dc \uacbd\ucc30\uc740 \ubbfc\uc911\ucd1d\uad90\uae30\ub54c \uc790\uc2e0\uc774 \ube68\uac04 \uc6b0\uc758\ub97c \ucc29\uc6a9\ud558\uace0 \uc788\uc5c8\ub2e4\ub294 \uac83\uc744 \uc54c\uace0 \uc788\uc5c8\uace0, \ucc44\uc99d\ud55c \uc99d\uac70\uc790\ub8cc\ub97c \ubcf4\uc5ec\uc8fc\uba74\uc11c \uc870\uc0ac\ud558\uc600\uc73c\uba70, \"\uc608\uc0c1 \ubc16\uc73c\ub85c \ubc31\ub0a8\uae30 \uc5b4\ub974\uc2e0\uacfc \uad00\ub828\ub41c (\ubb3c\ub300\ud3ec\uc5d0 \uc4f0\ub7ec\uc9c4) \uc2dc\uac04\ub300 \uc0ac\uc9c4\ub9cc \uc81c\uc2dc\ud558\uc9c0 \uc54a\uc558\uace0, \ubb3b\uc9c0\ub3c4 \uc54a\uc558\ub2e4. \uc55e\ub4a4 \uc0c1\ud669\uc740 \ub2e4 \ubb3c\uc5c8\ub2e4\" \uace0 \ubc1d\ud614\ub2e4.\u3131\uc528\ub294 \u201c\uadf8\ub3d9\uc548 \uc77c\ubca0 \ub4f1 \uc77c\ubd80\uc758 (\ube68\uac04\uc6b0\uc758 \uc870\ud569\uc6d0\uc774 \ubc31\ub0a8\uae30 \ub18d\ubbfc\uc744 \uac00\uaca9\ud574 \uc0ac\ub9dd\uc758 \uc6d0\uc778\uc774 \ub418\uc5c8\ub2e4\ub294) \uc8fc\uc7a5\uc740 \ub108\ubb34 \uc5c9\ud130\ub9ac\ub77c \uad73\uc774 \ub300\uc751\ud558\uc5ec \uad6d\uac00\ud3ed\ub825 \uc0b4\uc778\uc774\ub77c\ub294 \ucd08\uc810\uc744 \ud750\ub9ac\uae30\ub97c \ubc14\ub77c\uc9c0 \uc54a\uc544 \uce68\ubb35\ud588\ub2e4\u201d\uace0 \ubc1d\ud614\ub2e4. \uadf8\ub294 \ubc31\ub0a8\uae30 \uc528\uc5d0 \ub300\ud574 \"\ucc98\uc74c \ubd59\ub294 \ubd84\uc774\uc600\ub2e4. \uc0ac\ub294 \uc9c0\uc5ed\uc774 \uac19\ub2e4\ub294 \uac83\uc740 \ub098\uc911\uc5d0 \uadf8\ubd84\uc758 \uc2e0\uc0c1\uc774 \ubcf4\ub3c4\ub418\uba74\uc11c \uc54c\uc558\ub2e4\u201d\uace0 \ubc1d\ud614\ub2e4.", "paragraph_answer": "2016\ub144 10\uc6d4 19\uc77c \uc11c\uc6b8\ud2b9\ubcc4\uc2dc \uc911\uad6c \ud504\ub808\uc2a4\uc13c\ud130\uc5d0\uc11c '\ube68\uac04\uc6b0\uc758' \ub2f9\uc0ac\uc790\uc778 \uc804\uad6d\uacf5\uacf5\uc6b4\uc218\ub178\ub3d9\uc870\ud569 \uad11\uc8fc\u2027\uc804\ub0a8\uc9c0\ubd80 \uac04\ubd80 \ucd9c\uc2e0 \uc870\ud569\uc6d0\uc778 40\ub300 \u3131\uc528\uac00 \uae30\uc790\ud68c\uacac\uc744 \uc5f4\uc5b4 \uacf5\uc2dd \uc785\uc7a5\uc744 \ubc1d\ud614\ub2e4. \u3131\uc528\ub294 2015\ub144 12\uc6d4 11\uc77c \uc11c\uc6b8\ub0a8\ub300\ubb38\uacbd\ucc30\uc11c\uc5d0\uc11c 4\uc2dc\uac04 \ub3d9\uc548 \uc870\uc0ac\ub97c \ubc1b\uac8c \ub418\uc5c8\ub2e4. \u3131\uc528\ub294 \uc870\uc0ac \ub2f9\uc2dc \uacbd\ucc30\uc740 \ubbfc\uc911\ucd1d\uad90\uae30\ub54c \uc790\uc2e0\uc774 \ube68\uac04 \uc6b0\uc758\ub97c \ucc29\uc6a9\ud558\uace0 \uc788\uc5c8\ub2e4\ub294 \uac83\uc744 \uc54c\uace0 \uc788\uc5c8\uace0, \ucc44\uc99d\ud55c \uc99d\uac70\uc790\ub8cc\ub97c \ubcf4\uc5ec\uc8fc\uba74\uc11c \uc870\uc0ac\ud558\uc600\uc73c\uba70, \"\uc608\uc0c1 \ubc16\uc73c\ub85c \ubc31\ub0a8\uae30 \uc5b4\ub974\uc2e0\uacfc \uad00\ub828\ub41c (\ubb3c\ub300\ud3ec\uc5d0 \uc4f0\ub7ec\uc9c4) \uc2dc\uac04\ub300 \uc0ac\uc9c4\ub9cc \uc81c\uc2dc\ud558\uc9c0 \uc54a\uc558\uace0, \ubb3b\uc9c0\ub3c4 \uc54a\uc558\ub2e4. \uc55e\ub4a4 \uc0c1\ud669\uc740 \ub2e4 \ubb3c\uc5c8\ub2e4\" \uace0 \ubc1d\ud614\ub2e4.\u3131\uc528\ub294 \u201c\uadf8\ub3d9\uc548 \uc77c\ubca0 \ub4f1 \uc77c\ubd80\uc758 (\ube68\uac04\uc6b0\uc758 \uc870\ud569\uc6d0\uc774 \ubc31\ub0a8\uae30 \ub18d\ubbfc\uc744 \uac00\uaca9\ud574 \uc0ac\ub9dd\uc758 \uc6d0\uc778\uc774 \ub418\uc5c8\ub2e4\ub294) \uc8fc\uc7a5\uc740 \ub108\ubb34 \uc5c9\ud130\ub9ac\ub77c \uad73\uc774 \ub300\uc751\ud558\uc5ec \uad6d\uac00\ud3ed\ub825 \uc0b4\uc778\uc774\ub77c\ub294 \ucd08\uc810\uc744 \ud750\ub9ac\uae30\ub97c \ubc14\ub77c\uc9c0 \uc54a\uc544 \uce68\ubb35\ud588\ub2e4\u201d\uace0 \ubc1d\ud614\ub2e4. \uadf8\ub294 \ubc31\ub0a8\uae30 \uc528\uc5d0 \ub300\ud574 \"\ucc98\uc74c \ubd59\ub294 \ubd84\uc774\uc600\ub2e4. \uc0ac\ub294 \uc9c0\uc5ed\uc774 \uac19\ub2e4\ub294 \uac83\uc740 \ub098\uc911\uc5d0 \uadf8\ubd84\uc758 \uc2e0\uc0c1\uc774 \ubcf4\ub3c4\ub418\uba74\uc11c \uc54c\uc558\ub2e4\u201d\uace0 \ubc1d\ud614\ub2e4.", "sentence_answer": "\u3131\uc528\ub294 2015\ub144 12\uc6d4 11\uc77c \uc11c\uc6b8\ub0a8\ub300\ubb38\uacbd\ucc30\uc11c\uc5d0\uc11c 4\uc2dc\uac04 \ub3d9\uc548 \uc870\uc0ac\ub97c \ubc1b\uac8c \ub418\uc5c8\ub2e4.", "paragraph_id": "6581582-2-0", "paragraph_question": "question: 2015\ub144 12\uc6d4 11\uc77c \uc11c\uc6b8\ub0a8\ub300\ubb38\uacbd\ucc30\uc11c\uc5d0\uc11c \u3131\uc528\uac00 \uc870\uc0ac\ubc1b\uc740 \uc2dc\uac04\uc740?, context: 2016\ub144 10\uc6d4 19\uc77c \uc11c\uc6b8\ud2b9\ubcc4\uc2dc \uc911\uad6c \ud504\ub808\uc2a4\uc13c\ud130\uc5d0\uc11c '\ube68\uac04\uc6b0\uc758' \ub2f9\uc0ac\uc790\uc778 \uc804\uad6d\uacf5\uacf5\uc6b4\uc218\ub178\ub3d9\uc870\ud569 \uad11\uc8fc\u2027\uc804\ub0a8\uc9c0\ubd80 \uac04\ubd80 \ucd9c\uc2e0 \uc870\ud569\uc6d0\uc778 40\ub300 \u3131\uc528\uac00 \uae30\uc790\ud68c\uacac\uc744 \uc5f4\uc5b4 \uacf5\uc2dd \uc785\uc7a5\uc744 \ubc1d\ud614\ub2e4. \u3131\uc528\ub294 2015\ub144 12\uc6d4 11\uc77c \uc11c\uc6b8\ub0a8\ub300\ubb38\uacbd\ucc30\uc11c\uc5d0\uc11c 4\uc2dc\uac04 \ub3d9\uc548 \uc870\uc0ac\ub97c \ubc1b\uac8c \ub418\uc5c8\ub2e4. \u3131\uc528\ub294 \uc870\uc0ac \ub2f9\uc2dc \uacbd\ucc30\uc740 \ubbfc\uc911\ucd1d\uad90\uae30\ub54c \uc790\uc2e0\uc774 \ube68\uac04 \uc6b0\uc758\ub97c \ucc29\uc6a9\ud558\uace0 \uc788\uc5c8\ub2e4\ub294 \uac83\uc744 \uc54c\uace0 \uc788\uc5c8\uace0, \ucc44\uc99d\ud55c \uc99d\uac70\uc790\ub8cc\ub97c \ubcf4\uc5ec\uc8fc\uba74\uc11c \uc870\uc0ac\ud558\uc600\uc73c\uba70, \"\uc608\uc0c1 \ubc16\uc73c\ub85c \ubc31\ub0a8\uae30 \uc5b4\ub974\uc2e0\uacfc \uad00\ub828\ub41c (\ubb3c\ub300\ud3ec\uc5d0 \uc4f0\ub7ec\uc9c4) \uc2dc\uac04\ub300 \uc0ac\uc9c4\ub9cc \uc81c\uc2dc\ud558\uc9c0 \uc54a\uc558\uace0, \ubb3b\uc9c0\ub3c4 \uc54a\uc558\ub2e4. \uc55e\ub4a4 \uc0c1\ud669\uc740 \ub2e4 \ubb3c\uc5c8\ub2e4\" \uace0 \ubc1d\ud614\ub2e4.\u3131\uc528\ub294 \u201c\uadf8\ub3d9\uc548 \uc77c\ubca0 \ub4f1 \uc77c\ubd80\uc758 (\ube68\uac04\uc6b0\uc758 \uc870\ud569\uc6d0\uc774 \ubc31\ub0a8\uae30 \ub18d\ubbfc\uc744 \uac00\uaca9\ud574 \uc0ac\ub9dd\uc758 \uc6d0\uc778\uc774 \ub418\uc5c8\ub2e4\ub294) \uc8fc\uc7a5\uc740 \ub108\ubb34 \uc5c9\ud130\ub9ac\ub77c \uad73\uc774 \ub300\uc751\ud558\uc5ec \uad6d\uac00\ud3ed\ub825 \uc0b4\uc778\uc774\ub77c\ub294 \ucd08\uc810\uc744 \ud750\ub9ac\uae30\ub97c \ubc14\ub77c\uc9c0 \uc54a\uc544 \uce68\ubb35\ud588\ub2e4\u201d\uace0 \ubc1d\ud614\ub2e4. \uadf8\ub294 \ubc31\ub0a8\uae30 \uc528\uc5d0 \ub300\ud574 \"\ucc98\uc74c \ubd59\ub294 \ubd84\uc774\uc600\ub2e4. \uc0ac\ub294 \uc9c0\uc5ed\uc774 \uac19\ub2e4\ub294 \uac83\uc740 \ub098\uc911\uc5d0 \uadf8\ubd84\uc758 \uc2e0\uc0c1\uc774 \ubcf4\ub3c4\ub418\uba74\uc11c \uc54c\uc558\ub2e4\u201d\uace0 \ubc1d\ud614\ub2e4."} {"question": "\uc568\ubc94 \uc7a5\uae30\ud558\uc640 \uc5bc\uad74\ub4e4\uc758 LP\ud55c\uc815\ubc18\uc744 \uc81c\uc791\ud55c \uc2a4\ud29c\ub514\uc624\ub294 \uc5b4\ub514\uc57c?", "paragraph": "\u300a\uc7a5\uae30\ud558\uc640 \uc5bc\uad74\ub4e4\u300b\uc740 \ubc1c\ud45c \ub2e4\uc74c\uc77c\uc5d0 \uc8fc\uc694 \uc628\ub77c\uc778 \ud310\ub9e4\ucc98\uc778 YES24, \uc54c\ub77c\ub518, \uc778\ud130\ub137 \uad50\ubcf4\ubb38\uace0 \ub4f1\uc5d0\uc11c \ud310\ub9e4\uc728 1\uc704\ub97c \uae30\ub85d\ud588\ub2e4. \uc608\uc57d\ud310\ub9e4 \ub9cc\uc73c\ub85c 1\ub9cc 5\ucc9c\uc7a5\uc774 \ub9e4\uc9c4\ub418\uc5b4 \ubc1c\ub9e4 \uccab\ub0a0 \uae34\uae09\ud558\uac8c 1\ub9cc\uc7a5\uc744 \ucd94\uac00 \uc81c\uc791\ud588\ub2e4. \ucd94\uac00\ub85c \ub098\uc628 5000\uc7a5\ub3c4 \ubc14\ub85c \ub9e4\uc9c4\ub418\uc5c8\ub2e4. 6\uc6d4 10\uc77c\uc5d0 \uc568\ubc94 \ud0c0\uc774\ud2c0 \uace1 \u3008\uadf8\ub807\uace0 \uadf8\ub7f0 \uc0ac\uc774\u3009\ub294 \ubc85\uc2a4\uc5d0\uc11c 1\uc704\ub97c, \uc218\ub85d\uace1 \u3008TV\ub97c \ubd24\ub124\u3009\ub294 7\uc704\uc5d0 \uc62c\ub790\ub2e4. \ub610 \uad6d\ub0b4 \ucd5c\ub300 \uc74c\ubc18\ud310\ub9e4\ub7c9 \uc9d1\uacc4 \uc0ac\uc774\ud2b8 \ud55c\ud130\ucc28\ud2b8\uc5d0\uc11c\ub294 5\uc704\ub2e4. 2014\ub144 1\uc6d4 \ube14\ub8e8, \ube14\ub799\uc73c\ub85c \uad6c\uc131\ub41c \ud55c\uc815\ud310 LP\ub85c \ubc1c\ub9e4\ub418\uc5c8\ub2e4. LP \ud55c\uc815\ubc18\uc744 \uc81c\uc791\ud558\uae30 \uc704\ud574 \uc804\uace1 \ub9ac\ub9c8\uc2a4\ud130\ub9c1 \uc791\uc5c5\uacfc \ub799\ucee4 \uc81c\uc791\uc774 \uc601\uad6d \uc560\ube44 \ub85c\ub4dc \uc2a4\ud29c\ub514\uc624\uc5d0\uc11c \uc9c4\ud589\ub418\uc5c8\ub2e4. \uc791\uc5c5\uc744 \ud568\uaed8\ud55c \uc5d4\uc9c0\ub2c8\uc5b4 \uc158 \ub9e4\uae30\ub294 \uc74c\ubc18\uc5d0 \ub300\ud574 \"\uc544\uc8fc \uc88b\ub2e4. \ub2e4\uc74c \uc791\ud488\ub3c4 \uc791\uc5c5\ud574 \ubcf4\uace0 \uc2f6\ub2e4. \uc62c\ud574 \uc791\uc5c5\ud588\ub358 \ucd5c\uace0\uc758 \uc74c\ubc18\ub4e4 \uc911 \ud558\ub098\ub2e4. \uc0ac\uc6b4\ub4dc\ub3c4, \uc74c\uc545\ub3c4, \uc5f0\uc8fc\ub3c4 \ub9e4\uc6b0 \uc88b\ub2e4\"\uace0 \uadf9\ucc2c\ud588\ub2e4.", "answer": "\uc601\uad6d \uc560\ube44 \ub85c\ub4dc \uc2a4\ud29c\ub514\uc624", "sentence": "LP \ud55c\uc815\ubc18\uc744 \uc81c\uc791\ud558\uae30 \uc704\ud574 \uc804\uace1 \ub9ac\ub9c8\uc2a4\ud130\ub9c1 \uc791\uc5c5\uacfc \ub799\ucee4 \uc81c\uc791\uc774 \uc601\uad6d \uc560\ube44 \ub85c\ub4dc \uc2a4\ud29c\ub514\uc624 \uc5d0\uc11c \uc9c4\ud589\ub418\uc5c8\ub2e4.", "paragraph_sentence": "\u300a\uc7a5\uae30\ud558\uc640 \uc5bc\uad74\ub4e4\u300b\uc740 \ubc1c\ud45c \ub2e4\uc74c\uc77c\uc5d0 \uc8fc\uc694 \uc628\ub77c\uc778 \ud310\ub9e4\ucc98\uc778 YES24, \uc54c\ub77c\ub518, \uc778\ud130\ub137 \uad50\ubcf4\ubb38\uace0 \ub4f1\uc5d0\uc11c \ud310\ub9e4\uc728 1\uc704\ub97c \uae30\ub85d\ud588\ub2e4. \uc608\uc57d\ud310\ub9e4 \ub9cc\uc73c\ub85c 1\ub9cc 5\ucc9c\uc7a5\uc774 \ub9e4\uc9c4\ub418\uc5b4 \ubc1c\ub9e4 \uccab\ub0a0 \uae34\uae09\ud558\uac8c 1\ub9cc\uc7a5\uc744 \ucd94\uac00 \uc81c\uc791\ud588\ub2e4. \ucd94\uac00\ub85c \ub098\uc628 5000\uc7a5\ub3c4 \ubc14\ub85c \ub9e4\uc9c4\ub418\uc5c8\ub2e4. 6\uc6d4 10\uc77c\uc5d0 \uc568\ubc94 \ud0c0\uc774\ud2c0 \uace1 \u3008\uadf8\ub807\uace0 \uadf8\ub7f0 \uc0ac\uc774\u3009\ub294 \ubc85\uc2a4\uc5d0\uc11c 1\uc704\ub97c, \uc218\ub85d\uace1 \u3008TV\ub97c \ubd24\ub124\u3009\ub294 7\uc704\uc5d0 \uc62c\ub790\ub2e4. \ub610 \uad6d\ub0b4 \ucd5c\ub300 \uc74c\ubc18\ud310\ub9e4\ub7c9 \uc9d1\uacc4 \uc0ac\uc774\ud2b8 \ud55c\ud130\ucc28\ud2b8\uc5d0\uc11c\ub294 5\uc704\ub2e4. 2014\ub144 1\uc6d4 \ube14\ub8e8, \ube14\ub799\uc73c\ub85c \uad6c\uc131\ub41c \ud55c\uc815\ud310 LP\ub85c \ubc1c\ub9e4\ub418\uc5c8\ub2e4. LP \ud55c\uc815\ubc18\uc744 \uc81c\uc791\ud558\uae30 \uc704\ud574 \uc804\uace1 \ub9ac\ub9c8\uc2a4\ud130\ub9c1 \uc791\uc5c5\uacfc \ub799\ucee4 \uc81c\uc791\uc774 \uc601\uad6d \uc560\ube44 \ub85c\ub4dc \uc2a4\ud29c\ub514\uc624 \uc5d0\uc11c \uc9c4\ud589\ub418\uc5c8\ub2e4. \uc791\uc5c5\uc744 \ud568\uaed8\ud55c \uc5d4\uc9c0\ub2c8\uc5b4 \uc158 \ub9e4\uae30\ub294 \uc74c\ubc18\uc5d0 \ub300\ud574 \"\uc544\uc8fc \uc88b\ub2e4. \ub2e4\uc74c \uc791\ud488\ub3c4 \uc791\uc5c5\ud574 \ubcf4\uace0 \uc2f6\ub2e4. \uc62c\ud574 \uc791\uc5c5\ud588\ub358 \ucd5c\uace0\uc758 \uc74c\ubc18\ub4e4 \uc911 \ud558\ub098\ub2e4. \uc0ac\uc6b4\ub4dc\ub3c4, \uc74c\uc545\ub3c4, \uc5f0\uc8fc\ub3c4 \ub9e4\uc6b0 \uc88b\ub2e4\"\uace0 \uadf9\ucc2c\ud588\ub2e4.", "paragraph_answer": "\u300a\uc7a5\uae30\ud558\uc640 \uc5bc\uad74\ub4e4\u300b\uc740 \ubc1c\ud45c \ub2e4\uc74c\uc77c\uc5d0 \uc8fc\uc694 \uc628\ub77c\uc778 \ud310\ub9e4\ucc98\uc778 YES24, \uc54c\ub77c\ub518, \uc778\ud130\ub137 \uad50\ubcf4\ubb38\uace0 \ub4f1\uc5d0\uc11c \ud310\ub9e4\uc728 1\uc704\ub97c \uae30\ub85d\ud588\ub2e4. \uc608\uc57d\ud310\ub9e4 \ub9cc\uc73c\ub85c 1\ub9cc 5\ucc9c\uc7a5\uc774 \ub9e4\uc9c4\ub418\uc5b4 \ubc1c\ub9e4 \uccab\ub0a0 \uae34\uae09\ud558\uac8c 1\ub9cc\uc7a5\uc744 \ucd94\uac00 \uc81c\uc791\ud588\ub2e4. \ucd94\uac00\ub85c \ub098\uc628 5000\uc7a5\ub3c4 \ubc14\ub85c \ub9e4\uc9c4\ub418\uc5c8\ub2e4. 6\uc6d4 10\uc77c\uc5d0 \uc568\ubc94 \ud0c0\uc774\ud2c0 \uace1 \u3008\uadf8\ub807\uace0 \uadf8\ub7f0 \uc0ac\uc774\u3009\ub294 \ubc85\uc2a4\uc5d0\uc11c 1\uc704\ub97c, \uc218\ub85d\uace1 \u3008TV\ub97c \ubd24\ub124\u3009\ub294 7\uc704\uc5d0 \uc62c\ub790\ub2e4. \ub610 \uad6d\ub0b4 \ucd5c\ub300 \uc74c\ubc18\ud310\ub9e4\ub7c9 \uc9d1\uacc4 \uc0ac\uc774\ud2b8 \ud55c\ud130\ucc28\ud2b8\uc5d0\uc11c\ub294 5\uc704\ub2e4. 2014\ub144 1\uc6d4 \ube14\ub8e8, \ube14\ub799\uc73c\ub85c \uad6c\uc131\ub41c \ud55c\uc815\ud310 LP\ub85c \ubc1c\ub9e4\ub418\uc5c8\ub2e4. LP \ud55c\uc815\ubc18\uc744 \uc81c\uc791\ud558\uae30 \uc704\ud574 \uc804\uace1 \ub9ac\ub9c8\uc2a4\ud130\ub9c1 \uc791\uc5c5\uacfc \ub799\ucee4 \uc81c\uc791\uc774 \uc601\uad6d \uc560\ube44 \ub85c\ub4dc \uc2a4\ud29c\ub514\uc624 \uc5d0\uc11c \uc9c4\ud589\ub418\uc5c8\ub2e4. \uc791\uc5c5\uc744 \ud568\uaed8\ud55c \uc5d4\uc9c0\ub2c8\uc5b4 \uc158 \ub9e4\uae30\ub294 \uc74c\ubc18\uc5d0 \ub300\ud574 \"\uc544\uc8fc \uc88b\ub2e4. \ub2e4\uc74c \uc791\ud488\ub3c4 \uc791\uc5c5\ud574 \ubcf4\uace0 \uc2f6\ub2e4. \uc62c\ud574 \uc791\uc5c5\ud588\ub358 \ucd5c\uace0\uc758 \uc74c\ubc18\ub4e4 \uc911 \ud558\ub098\ub2e4. \uc0ac\uc6b4\ub4dc\ub3c4, \uc74c\uc545\ub3c4, \uc5f0\uc8fc\ub3c4 \ub9e4\uc6b0 \uc88b\ub2e4\"\uace0 \uadf9\ucc2c\ud588\ub2e4.", "sentence_answer": "LP \ud55c\uc815\ubc18\uc744 \uc81c\uc791\ud558\uae30 \uc704\ud574 \uc804\uace1 \ub9ac\ub9c8\uc2a4\ud130\ub9c1 \uc791\uc5c5\uacfc \ub799\ucee4 \uc81c\uc791\uc774 \uc601\uad6d \uc560\ube44 \ub85c\ub4dc \uc2a4\ud29c\ub514\uc624 \uc5d0\uc11c \uc9c4\ud589\ub418\uc5c8\ub2e4.", "paragraph_id": "6177395-10-2", "paragraph_question": "question: \uc568\ubc94 \uc7a5\uae30\ud558\uc640 \uc5bc\uad74\ub4e4\uc758 LP\ud55c\uc815\ubc18\uc744 \uc81c\uc791\ud55c \uc2a4\ud29c\ub514\uc624\ub294 \uc5b4\ub514\uc57c?, context: \u300a\uc7a5\uae30\ud558\uc640 \uc5bc\uad74\ub4e4\u300b\uc740 \ubc1c\ud45c \ub2e4\uc74c\uc77c\uc5d0 \uc8fc\uc694 \uc628\ub77c\uc778 \ud310\ub9e4\ucc98\uc778 YES24, \uc54c\ub77c\ub518, \uc778\ud130\ub137 \uad50\ubcf4\ubb38\uace0 \ub4f1\uc5d0\uc11c \ud310\ub9e4\uc728 1\uc704\ub97c \uae30\ub85d\ud588\ub2e4. \uc608\uc57d\ud310\ub9e4 \ub9cc\uc73c\ub85c 1\ub9cc 5\ucc9c\uc7a5\uc774 \ub9e4\uc9c4\ub418\uc5b4 \ubc1c\ub9e4 \uccab\ub0a0 \uae34\uae09\ud558\uac8c 1\ub9cc\uc7a5\uc744 \ucd94\uac00 \uc81c\uc791\ud588\ub2e4. \ucd94\uac00\ub85c \ub098\uc628 5000\uc7a5\ub3c4 \ubc14\ub85c \ub9e4\uc9c4\ub418\uc5c8\ub2e4. 6\uc6d4 10\uc77c\uc5d0 \uc568\ubc94 \ud0c0\uc774\ud2c0 \uace1 \u3008\uadf8\ub807\uace0 \uadf8\ub7f0 \uc0ac\uc774\u3009\ub294 \ubc85\uc2a4\uc5d0\uc11c 1\uc704\ub97c, \uc218\ub85d\uace1 \u3008TV\ub97c \ubd24\ub124\u3009\ub294 7\uc704\uc5d0 \uc62c\ub790\ub2e4. \ub610 \uad6d\ub0b4 \ucd5c\ub300 \uc74c\ubc18\ud310\ub9e4\ub7c9 \uc9d1\uacc4 \uc0ac\uc774\ud2b8 \ud55c\ud130\ucc28\ud2b8\uc5d0\uc11c\ub294 5\uc704\ub2e4. 2014\ub144 1\uc6d4 \ube14\ub8e8, \ube14\ub799\uc73c\ub85c \uad6c\uc131\ub41c \ud55c\uc815\ud310 LP\ub85c \ubc1c\ub9e4\ub418\uc5c8\ub2e4. LP \ud55c\uc815\ubc18\uc744 \uc81c\uc791\ud558\uae30 \uc704\ud574 \uc804\uace1 \ub9ac\ub9c8\uc2a4\ud130\ub9c1 \uc791\uc5c5\uacfc \ub799\ucee4 \uc81c\uc791\uc774 \uc601\uad6d \uc560\ube44 \ub85c\ub4dc \uc2a4\ud29c\ub514\uc624\uc5d0\uc11c \uc9c4\ud589\ub418\uc5c8\ub2e4. \uc791\uc5c5\uc744 \ud568\uaed8\ud55c \uc5d4\uc9c0\ub2c8\uc5b4 \uc158 \ub9e4\uae30\ub294 \uc74c\ubc18\uc5d0 \ub300\ud574 \"\uc544\uc8fc \uc88b\ub2e4. \ub2e4\uc74c \uc791\ud488\ub3c4 \uc791\uc5c5\ud574 \ubcf4\uace0 \uc2f6\ub2e4. \uc62c\ud574 \uc791\uc5c5\ud588\ub358 \ucd5c\uace0\uc758 \uc74c\ubc18\ub4e4 \uc911 \ud558\ub098\ub2e4. \uc0ac\uc6b4\ub4dc\ub3c4, \uc74c\uc545\ub3c4, \uc5f0\uc8fc\ub3c4 \ub9e4\uc6b0 \uc88b\ub2e4\"\uace0 \uadf9\ucc2c\ud588\ub2e4."} {"question": "1990\ub144\ub300 \uce98\ub9ac\ud3ec\ub2c8\uc544\ud574\ub2ec \uac1c\uccb4\ub4e4 \uc911 \uc0ac\ub9dd\ub960\uc774 \uc720\ub2ec\ub9ac \ub192\uc740 \uac1c\uccb4\ub294 \uc5b4\ub5a4 \uac1c\uccb4\uc778\uac00?", "paragraph": "\ub610 \ub2e4\ub978 \uc694\uc8fc\uc758 \uc9c0\uc5ed\uc740 \uce98\ub9ac\ud3ec\ub2c8\uc544\ub85c, 1990\ub144\ub300 \ud574\ub2ec \ubcf5\uc6d0 \uc815\ub3c4\uac00 \ub4f1\ub77d\uc744 \uac70\ub4ed\ud558\uac70\ub098 \uac10\uc18c\ud558\uace0 \uc788\ub2e4. \uc131\uccb4 \ubc0f \uc131\uc7a5\uae30\uac00 \uac70\uc758 \ub05d\ub09c \uac1c\uccb4\uc758 \uc0ac\ub9dd\ub960\uc774 \uc720\ub2ec\ub9ac \ub192\uc73c\uba70, \ud2b9\ud788 \uc554\ucef7\uc758 \uc0ac\ub9dd\ub960\uc774 \ub192\uc74c\uc774 \ubcf4\uace0\ub418\uc5c8\ub2e4. \uc8fd\uc740 \uc2dc\uccb4\uc758 \ubd80\uac80 \uacb0\uacfc \uc9c8\ubcd1, \ud2b9\ud788 \ud1a1\uc18c\ud50c\ub77c\uc988\ub9c8 \uace4\ub514\uc774(Toxoplasma gondii)\uc640 \uad6c\ub450\ucda9 \uae30\uc0dd\uc774 \uce98\ub9ac\ud3ec\ub2c8\uc544 \ud574\ub2ec\uc758 \uc8fc\uc694 \ud3d0\uc0ac \uc6d0\uc778\uc73c\ub85c \ubc1d\ud600\uc84c\ub2e4. \ud574\ub2ec\uc5d0\uac8c \ub9e4\uc6b0 \uce58\uba85\uc801\uc778 \ud1a1\uc18c\ud50c\ub77c\uc988\ub9c8 \uae30\uc0dd\ucda9\uc740 \uc57c\uc0dd\uace0\uc591\uc774 \ubc0f \uc9d1\uace0\uc591\uc774\uc640 \uc8fc\uba38\ub2c8\uc950 \ub4f1\uc774 \uc219\uc8fc\uc774\uba70, \uc9d1\uace0\uc591\uc774\uc758 \ubd84\ube44\ubb3c\uc774 \ud558\uc218\ub3c4\ub97c \ud1b5\ud574 \ubc14\ub2e4\ub85c \ud758\ub7ec\ub4e4\uc5b4\uac00 \ud574\ub2ec\uc5d0\uac8c \uac10\uc5fc\ub420 \uc218 \uc788\ub2e4. \uce98\ub9ac\ud3ec\ub2c8\uc544\ud574\ub2ec\uc758 \ud3d0\uc0ac\uc5d0 \uc9c8\ubcd1\uc774 \ud070 \uac83\uc740 \ud655\uc2e4\ud558\uc9c0\ub9cc, \uc5b4\uc9f8\uc11c \ub2e4\ub978 \uc9c0\uc5ed\uc5d0 \ube44\ud574 \uce98\ub9ac\ud3ec\ub2c8\uc544\uc758 \ud574\ub2ec \uac1c\uccb4\uad70\uc774 \ub2e4\ub978 \uc9c0\uc5ed\uc758 \ud574\ub2ec \uac1c\uccb4\uad70\ubcf4\ub2e4 \uc9c8\ubcd1\uc5d0 \ub354 \ud070 \uc601\ud5a5\uc744 \ubc1b\ub294\uc9c0\ub294 \uc544\uc9c1 \ubc1d\ud600\uc9c0\uc9c0 \uc54a\uc558\ub2e4.", "answer": "\uc131\uccb4", "sentence": "\uc131\uccb4 \ubc0f \uc131\uc7a5\uae30\uac00 \uac70\uc758 \ub05d\ub09c \uac1c\uccb4\uc758 \uc0ac\ub9dd\ub960\uc774 \uc720\ub2ec\ub9ac \ub192\uc73c\uba70, \ud2b9\ud788 \uc554\ucef7\uc758 \uc0ac\ub9dd\ub960\uc774 \ub192\uc74c\uc774 \ubcf4\uace0\ub418\uc5c8\ub2e4.", "paragraph_sentence": "\ub610 \ub2e4\ub978 \uc694\uc8fc\uc758 \uc9c0\uc5ed\uc740 \uce98\ub9ac\ud3ec\ub2c8\uc544\ub85c, 1990\ub144\ub300 \ud574\ub2ec \ubcf5\uc6d0 \uc815\ub3c4\uac00 \ub4f1\ub77d\uc744 \uac70\ub4ed\ud558\uac70\ub098 \uac10\uc18c\ud558\uace0 \uc788\ub2e4. \uc131\uccb4 \ubc0f \uc131\uc7a5\uae30\uac00 \uac70\uc758 \ub05d\ub09c \uac1c\uccb4\uc758 \uc0ac\ub9dd\ub960\uc774 \uc720\ub2ec\ub9ac \ub192\uc73c\uba70, \ud2b9\ud788 \uc554\ucef7\uc758 \uc0ac\ub9dd\ub960\uc774 \ub192\uc74c\uc774 \ubcf4\uace0\ub418\uc5c8\ub2e4. \uc8fd\uc740 \uc2dc\uccb4\uc758 \ubd80\uac80 \uacb0\uacfc \uc9c8\ubcd1, \ud2b9\ud788 \ud1a1\uc18c\ud50c\ub77c\uc988\ub9c8 \uace4\ub514\uc774(Toxoplasma gondii)\uc640 \uad6c\ub450\ucda9 \uae30\uc0dd\uc774 \uce98\ub9ac\ud3ec\ub2c8\uc544 \ud574\ub2ec\uc758 \uc8fc\uc694 \ud3d0\uc0ac \uc6d0\uc778\uc73c\ub85c \ubc1d\ud600\uc84c\ub2e4. \ud574\ub2ec\uc5d0\uac8c \ub9e4\uc6b0 \uce58\uba85\uc801\uc778 \ud1a1\uc18c\ud50c\ub77c\uc988\ub9c8 \uae30\uc0dd\ucda9\uc740 \uc57c\uc0dd\uace0\uc591\uc774 \ubc0f \uc9d1\uace0\uc591\uc774\uc640 \uc8fc\uba38\ub2c8\uc950 \ub4f1\uc774 \uc219\uc8fc\uc774\uba70, \uc9d1\uace0\uc591\uc774\uc758 \ubd84\ube44\ubb3c\uc774 \ud558\uc218\ub3c4\ub97c \ud1b5\ud574 \ubc14\ub2e4\ub85c \ud758\ub7ec\ub4e4\uc5b4\uac00 \ud574\ub2ec\uc5d0\uac8c \uac10\uc5fc\ub420 \uc218 \uc788\ub2e4. \uce98\ub9ac\ud3ec\ub2c8\uc544\ud574\ub2ec\uc758 \ud3d0\uc0ac\uc5d0 \uc9c8\ubcd1\uc774 \ud070 \uac83\uc740 \ud655\uc2e4\ud558\uc9c0\ub9cc, \uc5b4\uc9f8\uc11c \ub2e4\ub978 \uc9c0\uc5ed\uc5d0 \ube44\ud574 \uce98\ub9ac\ud3ec\ub2c8\uc544\uc758 \ud574\ub2ec \uac1c\uccb4\uad70\uc774 \ub2e4\ub978 \uc9c0\uc5ed\uc758 \ud574\ub2ec \uac1c\uccb4\uad70\ubcf4\ub2e4 \uc9c8\ubcd1\uc5d0 \ub354 \ud070 \uc601\ud5a5\uc744 \ubc1b\ub294\uc9c0\ub294 \uc544\uc9c1 \ubc1d\ud600\uc9c0\uc9c0 \uc54a\uc558\ub2e4.", "paragraph_answer": "\ub610 \ub2e4\ub978 \uc694\uc8fc\uc758 \uc9c0\uc5ed\uc740 \uce98\ub9ac\ud3ec\ub2c8\uc544\ub85c, 1990\ub144\ub300 \ud574\ub2ec \ubcf5\uc6d0 \uc815\ub3c4\uac00 \ub4f1\ub77d\uc744 \uac70\ub4ed\ud558\uac70\ub098 \uac10\uc18c\ud558\uace0 \uc788\ub2e4. \uc131\uccb4 \ubc0f \uc131\uc7a5\uae30\uac00 \uac70\uc758 \ub05d\ub09c \uac1c\uccb4\uc758 \uc0ac\ub9dd\ub960\uc774 \uc720\ub2ec\ub9ac \ub192\uc73c\uba70, \ud2b9\ud788 \uc554\ucef7\uc758 \uc0ac\ub9dd\ub960\uc774 \ub192\uc74c\uc774 \ubcf4\uace0\ub418\uc5c8\ub2e4. \uc8fd\uc740 \uc2dc\uccb4\uc758 \ubd80\uac80 \uacb0\uacfc \uc9c8\ubcd1, \ud2b9\ud788 \ud1a1\uc18c\ud50c\ub77c\uc988\ub9c8 \uace4\ub514\uc774(Toxoplasma gondii)\uc640 \uad6c\ub450\ucda9 \uae30\uc0dd\uc774 \uce98\ub9ac\ud3ec\ub2c8\uc544 \ud574\ub2ec\uc758 \uc8fc\uc694 \ud3d0\uc0ac \uc6d0\uc778\uc73c\ub85c \ubc1d\ud600\uc84c\ub2e4. \ud574\ub2ec\uc5d0\uac8c \ub9e4\uc6b0 \uce58\uba85\uc801\uc778 \ud1a1\uc18c\ud50c\ub77c\uc988\ub9c8 \uae30\uc0dd\ucda9\uc740 \uc57c\uc0dd\uace0\uc591\uc774 \ubc0f \uc9d1\uace0\uc591\uc774\uc640 \uc8fc\uba38\ub2c8\uc950 \ub4f1\uc774 \uc219\uc8fc\uc774\uba70, \uc9d1\uace0\uc591\uc774\uc758 \ubd84\ube44\ubb3c\uc774 \ud558\uc218\ub3c4\ub97c \ud1b5\ud574 \ubc14\ub2e4\ub85c \ud758\ub7ec\ub4e4\uc5b4\uac00 \ud574\ub2ec\uc5d0\uac8c \uac10\uc5fc\ub420 \uc218 \uc788\ub2e4. \uce98\ub9ac\ud3ec\ub2c8\uc544\ud574\ub2ec\uc758 \ud3d0\uc0ac\uc5d0 \uc9c8\ubcd1\uc774 \ud070 \uac83\uc740 \ud655\uc2e4\ud558\uc9c0\ub9cc, \uc5b4\uc9f8\uc11c \ub2e4\ub978 \uc9c0\uc5ed\uc5d0 \ube44\ud574 \uce98\ub9ac\ud3ec\ub2c8\uc544\uc758 \ud574\ub2ec \uac1c\uccb4\uad70\uc774 \ub2e4\ub978 \uc9c0\uc5ed\uc758 \ud574\ub2ec \uac1c\uccb4\uad70\ubcf4\ub2e4 \uc9c8\ubcd1\uc5d0 \ub354 \ud070 \uc601\ud5a5\uc744 \ubc1b\ub294\uc9c0\ub294 \uc544\uc9c1 \ubc1d\ud600\uc9c0\uc9c0 \uc54a\uc558\ub2e4.", "sentence_answer": " \uc131\uccb4 \ubc0f \uc131\uc7a5\uae30\uac00 \uac70\uc758 \ub05d\ub09c \uac1c\uccb4\uc758 \uc0ac\ub9dd\ub960\uc774 \uc720\ub2ec\ub9ac \ub192\uc73c\uba70, \ud2b9\ud788 \uc554\ucef7\uc758 \uc0ac\ub9dd\ub960\uc774 \ub192\uc74c\uc774 \ubcf4\uace0\ub418\uc5c8\ub2e4.", "paragraph_id": "6570443-20-1", "paragraph_question": "question: 1990\ub144\ub300 \uce98\ub9ac\ud3ec\ub2c8\uc544\ud574\ub2ec \uac1c\uccb4\ub4e4 \uc911 \uc0ac\ub9dd\ub960\uc774 \uc720\ub2ec\ub9ac \ub192\uc740 \uac1c\uccb4\ub294 \uc5b4\ub5a4 \uac1c\uccb4\uc778\uac00?, context: \ub610 \ub2e4\ub978 \uc694\uc8fc\uc758 \uc9c0\uc5ed\uc740 \uce98\ub9ac\ud3ec\ub2c8\uc544\ub85c, 1990\ub144\ub300 \ud574\ub2ec \ubcf5\uc6d0 \uc815\ub3c4\uac00 \ub4f1\ub77d\uc744 \uac70\ub4ed\ud558\uac70\ub098 \uac10\uc18c\ud558\uace0 \uc788\ub2e4. \uc131\uccb4 \ubc0f \uc131\uc7a5\uae30\uac00 \uac70\uc758 \ub05d\ub09c \uac1c\uccb4\uc758 \uc0ac\ub9dd\ub960\uc774 \uc720\ub2ec\ub9ac \ub192\uc73c\uba70, \ud2b9\ud788 \uc554\ucef7\uc758 \uc0ac\ub9dd\ub960\uc774 \ub192\uc74c\uc774 \ubcf4\uace0\ub418\uc5c8\ub2e4. \uc8fd\uc740 \uc2dc\uccb4\uc758 \ubd80\uac80 \uacb0\uacfc \uc9c8\ubcd1, \ud2b9\ud788 \ud1a1\uc18c\ud50c\ub77c\uc988\ub9c8 \uace4\ub514\uc774(Toxoplasma gondii)\uc640 \uad6c\ub450\ucda9 \uae30\uc0dd\uc774 \uce98\ub9ac\ud3ec\ub2c8\uc544 \ud574\ub2ec\uc758 \uc8fc\uc694 \ud3d0\uc0ac \uc6d0\uc778\uc73c\ub85c \ubc1d\ud600\uc84c\ub2e4. \ud574\ub2ec\uc5d0\uac8c \ub9e4\uc6b0 \uce58\uba85\uc801\uc778 \ud1a1\uc18c\ud50c\ub77c\uc988\ub9c8 \uae30\uc0dd\ucda9\uc740 \uc57c\uc0dd\uace0\uc591\uc774 \ubc0f \uc9d1\uace0\uc591\uc774\uc640 \uc8fc\uba38\ub2c8\uc950 \ub4f1\uc774 \uc219\uc8fc\uc774\uba70, \uc9d1\uace0\uc591\uc774\uc758 \ubd84\ube44\ubb3c\uc774 \ud558\uc218\ub3c4\ub97c \ud1b5\ud574 \ubc14\ub2e4\ub85c \ud758\ub7ec\ub4e4\uc5b4\uac00 \ud574\ub2ec\uc5d0\uac8c \uac10\uc5fc\ub420 \uc218 \uc788\ub2e4. \uce98\ub9ac\ud3ec\ub2c8\uc544\ud574\ub2ec\uc758 \ud3d0\uc0ac\uc5d0 \uc9c8\ubcd1\uc774 \ud070 \uac83\uc740 \ud655\uc2e4\ud558\uc9c0\ub9cc, \uc5b4\uc9f8\uc11c \ub2e4\ub978 \uc9c0\uc5ed\uc5d0 \ube44\ud574 \uce98\ub9ac\ud3ec\ub2c8\uc544\uc758 \ud574\ub2ec \uac1c\uccb4\uad70\uc774 \ub2e4\ub978 \uc9c0\uc5ed\uc758 \ud574\ub2ec \uac1c\uccb4\uad70\ubcf4\ub2e4 \uc9c8\ubcd1\uc5d0 \ub354 \ud070 \uc601\ud5a5\uc744 \ubc1b\ub294\uc9c0\ub294 \uc544\uc9c1 \ubc1d\ud600\uc9c0\uc9c0 \uc54a\uc558\ub2e4."} {"question": "\uc815\ucc45\ud559\uc740 \uc5b4\ub290 \ubd84\uc57c\ub85c \uc131\ub9bd\ub3fc?", "paragraph": "\uc815\ucc45\ud559(\u653f\u7b56\u5b78, policy science)\uc740 \uadf8 \uc8fc\ub41c \uc5f0\uad6c\uad00\uc2ec \ub300\uc0c1\uacfc \ud750\ub984\uc5d0 \ub530\ub77c \uc815\ucc45\uacfc\uc815, \uc815\ucc45\ud615\uc131, \uc815\ucc45\uc9d1\ud589, \uc815\ucc45\ubd84\uc11d, \uc815\ucc45\ud3c9\uac00 \ub4f1\uc758 \uc138\ubd80 \ubd84\uc57c\ub85c \ubc1c\uc804\ud574\uc654\ub2e4. \uc815\ucc45\ud559\uc774 \ub3c5\uc790\uc801\uc778 \uc0ac\ud68c\uacfc\ud559\uc758 \ud55c \ubd84\uc57c\ub85c \uc131\ub9bd\ud560 \uc218 \uc788\ub294 \uc774\uc720\ub294 \uadf8\uac83\uc774 \uc815\ucc45\uc774\ub77c\ub294 \uc911\uc694\ud55c \uc5f0\uad6c\ub300\uc0c1\uc744 \uc5ec\ub7ec\uac00\uc9c0 \uacfc\ud559\uc801\uc778 \ubc29\ubc95\ub860\uacfc \uc774\ub860\uc744 \uac00\uc9c0\uace0 \uc5f0\uad6c\ud55c\ub2e4\ub294 \uc810 \ubfd0\ub9cc \uc544\ub2c8\ub77c \uadf8\uac83\uc774 \uac00\uce58\uc9c0\ud5a5\uc801 \uc989 \uc815\ucc45\ud559 \uc5f0\uad6c\uc5d0\uc11c \ubbfc\uc8fc\uc801 \uac00\uce58\uc640 \uc0ac\ud68c\uc801 \uac00\uce58\ub97c \uc774\ud574\ud560 \uac83\uc778\uac00\uc758 \ubb38\uc81c \uc5ed\uc2dc \uade0\ud615\ub418\uac8c \uc811\uadfc \ud55c\ub2e4\ub294 \uc810\uc5d0\uc11c\ub3c4 \ucc3e\uc744 \uc218 \uc788\uc744 \uac83\uc774\ub2e4. \uc774\ub7ec\ud55c \uc810\uc5d0\uc11c \ud574\ub7f4\ub4dc \ub77c\uc2a4\uc6f0(Harold Lasswell, 1902-1978)\uc774 \uc81c\uc2dc\ud55c \uc815\ucc45 \uacfc\ud559 \uac1c\ub150\uc758 \uc911\uc694\uc131\uc740 \uc5ec\uc804\ud788 21\uc138\uae30\uc5d0\ub3c4 \uc720\ud6a8\ud558\ub2e4\uace0 \ud560 \uac83\uc774\ub2e4.", "answer": "\uc0ac\ud68c\uacfc\ud559", "sentence": "\uc815\ucc45\ud559\uc774 \ub3c5\uc790\uc801\uc778 \uc0ac\ud68c\uacfc\ud559 \uc758 \ud55c \ubd84\uc57c\ub85c \uc131\ub9bd\ud560 \uc218 \uc788\ub294 \uc774\uc720\ub294 \uadf8\uac83\uc774 \uc815\ucc45\uc774\ub77c\ub294 \uc911\uc694\ud55c \uc5f0\uad6c\ub300\uc0c1\uc744 \uc5ec\ub7ec\uac00\uc9c0 \uacfc\ud559\uc801\uc778 \ubc29\ubc95\ub860\uacfc \uc774\ub860\uc744 \uac00\uc9c0\uace0 \uc5f0\uad6c\ud55c\ub2e4\ub294 \uc810 \ubfd0\ub9cc \uc544\ub2c8\ub77c \uadf8\uac83\uc774 \uac00\uce58\uc9c0\ud5a5\uc801 \uc989 \uc815\ucc45\ud559 \uc5f0\uad6c\uc5d0\uc11c \ubbfc\uc8fc\uc801 \uac00\uce58\uc640 \uc0ac\ud68c\uc801 \uac00\uce58\ub97c \uc774\ud574\ud560 \uac83\uc778\uac00\uc758 \ubb38\uc81c \uc5ed\uc2dc \uade0\ud615\ub418\uac8c \uc811\uadfc \ud55c\ub2e4\ub294 \uc810\uc5d0\uc11c\ub3c4 \ucc3e\uc744 \uc218 \uc788\uc744 \uac83\uc774\ub2e4.", "paragraph_sentence": "\uc815\ucc45\ud559(\u653f\u7b56\u5b78, policy science)\uc740 \uadf8 \uc8fc\ub41c \uc5f0\uad6c\uad00\uc2ec \ub300\uc0c1\uacfc \ud750\ub984\uc5d0 \ub530\ub77c \uc815\ucc45\uacfc\uc815, \uc815\ucc45\ud615\uc131, \uc815\ucc45\uc9d1\ud589, \uc815\ucc45\ubd84\uc11d, \uc815\ucc45\ud3c9\uac00 \ub4f1\uc758 \uc138\ubd80 \ubd84\uc57c\ub85c \ubc1c\uc804\ud574\uc654\ub2e4. \uc815\ucc45\ud559\uc774 \ub3c5\uc790\uc801\uc778 \uc0ac\ud68c\uacfc\ud559 \uc758 \ud55c \ubd84\uc57c\ub85c \uc131\ub9bd\ud560 \uc218 \uc788\ub294 \uc774\uc720\ub294 \uadf8\uac83\uc774 \uc815\ucc45\uc774\ub77c\ub294 \uc911\uc694\ud55c \uc5f0\uad6c\ub300\uc0c1\uc744 \uc5ec\ub7ec\uac00\uc9c0 \uacfc\ud559\uc801\uc778 \ubc29\ubc95\ub860\uacfc \uc774\ub860\uc744 \uac00\uc9c0\uace0 \uc5f0\uad6c\ud55c\ub2e4\ub294 \uc810 \ubfd0\ub9cc \uc544\ub2c8\ub77c \uadf8\uac83\uc774 \uac00\uce58\uc9c0\ud5a5\uc801 \uc989 \uc815\ucc45\ud559 \uc5f0\uad6c\uc5d0\uc11c \ubbfc\uc8fc\uc801 \uac00\uce58\uc640 \uc0ac\ud68c\uc801 \uac00\uce58\ub97c \uc774\ud574\ud560 \uac83\uc778\uac00\uc758 \ubb38\uc81c \uc5ed\uc2dc \uade0\ud615\ub418\uac8c \uc811\uadfc \ud55c\ub2e4\ub294 \uc810\uc5d0\uc11c\ub3c4 \ucc3e\uc744 \uc218 \uc788\uc744 \uac83\uc774\ub2e4. \uc774\ub7ec\ud55c \uc810\uc5d0\uc11c \ud574\ub7f4\ub4dc \ub77c\uc2a4\uc6f0(Harold Lasswell, 1902-1978)\uc774 \uc81c\uc2dc\ud55c \uc815\ucc45 \uacfc\ud559 \uac1c\ub150\uc758 \uc911\uc694\uc131\uc740 \uc5ec\uc804\ud788 21\uc138\uae30\uc5d0\ub3c4 \uc720\ud6a8\ud558\ub2e4\uace0 \ud560 \uac83\uc774\ub2e4.", "paragraph_answer": "\uc815\ucc45\ud559(\u653f\u7b56\u5b78, policy science)\uc740 \uadf8 \uc8fc\ub41c \uc5f0\uad6c\uad00\uc2ec \ub300\uc0c1\uacfc \ud750\ub984\uc5d0 \ub530\ub77c \uc815\ucc45\uacfc\uc815, \uc815\ucc45\ud615\uc131, \uc815\ucc45\uc9d1\ud589, \uc815\ucc45\ubd84\uc11d, \uc815\ucc45\ud3c9\uac00 \ub4f1\uc758 \uc138\ubd80 \ubd84\uc57c\ub85c \ubc1c\uc804\ud574\uc654\ub2e4. \uc815\ucc45\ud559\uc774 \ub3c5\uc790\uc801\uc778 \uc0ac\ud68c\uacfc\ud559 \uc758 \ud55c \ubd84\uc57c\ub85c \uc131\ub9bd\ud560 \uc218 \uc788\ub294 \uc774\uc720\ub294 \uadf8\uac83\uc774 \uc815\ucc45\uc774\ub77c\ub294 \uc911\uc694\ud55c \uc5f0\uad6c\ub300\uc0c1\uc744 \uc5ec\ub7ec\uac00\uc9c0 \uacfc\ud559\uc801\uc778 \ubc29\ubc95\ub860\uacfc \uc774\ub860\uc744 \uac00\uc9c0\uace0 \uc5f0\uad6c\ud55c\ub2e4\ub294 \uc810 \ubfd0\ub9cc \uc544\ub2c8\ub77c \uadf8\uac83\uc774 \uac00\uce58\uc9c0\ud5a5\uc801 \uc989 \uc815\ucc45\ud559 \uc5f0\uad6c\uc5d0\uc11c \ubbfc\uc8fc\uc801 \uac00\uce58\uc640 \uc0ac\ud68c\uc801 \uac00\uce58\ub97c \uc774\ud574\ud560 \uac83\uc778\uac00\uc758 \ubb38\uc81c \uc5ed\uc2dc \uade0\ud615\ub418\uac8c \uc811\uadfc \ud55c\ub2e4\ub294 \uc810\uc5d0\uc11c\ub3c4 \ucc3e\uc744 \uc218 \uc788\uc744 \uac83\uc774\ub2e4. \uc774\ub7ec\ud55c \uc810\uc5d0\uc11c \ud574\ub7f4\ub4dc \ub77c\uc2a4\uc6f0(Harold Lasswell, 1902-1978)\uc774 \uc81c\uc2dc\ud55c \uc815\ucc45 \uacfc\ud559 \uac1c\ub150\uc758 \uc911\uc694\uc131\uc740 \uc5ec\uc804\ud788 21\uc138\uae30\uc5d0\ub3c4 \uc720\ud6a8\ud558\ub2e4\uace0 \ud560 \uac83\uc774\ub2e4.", "sentence_answer": "\uc815\ucc45\ud559\uc774 \ub3c5\uc790\uc801\uc778 \uc0ac\ud68c\uacfc\ud559 \uc758 \ud55c \ubd84\uc57c\ub85c \uc131\ub9bd\ud560 \uc218 \uc788\ub294 \uc774\uc720\ub294 \uadf8\uac83\uc774 \uc815\ucc45\uc774\ub77c\ub294 \uc911\uc694\ud55c \uc5f0\uad6c\ub300\uc0c1\uc744 \uc5ec\ub7ec\uac00\uc9c0 \uacfc\ud559\uc801\uc778 \ubc29\ubc95\ub860\uacfc \uc774\ub860\uc744 \uac00\uc9c0\uace0 \uc5f0\uad6c\ud55c\ub2e4\ub294 \uc810 \ubfd0\ub9cc \uc544\ub2c8\ub77c \uadf8\uac83\uc774 \uac00\uce58\uc9c0\ud5a5\uc801 \uc989 \uc815\ucc45\ud559 \uc5f0\uad6c\uc5d0\uc11c \ubbfc\uc8fc\uc801 \uac00\uce58\uc640 \uc0ac\ud68c\uc801 \uac00\uce58\ub97c \uc774\ud574\ud560 \uac83\uc778\uac00\uc758 \ubb38\uc81c \uc5ed\uc2dc \uade0\ud615\ub418\uac8c \uc811\uadfc \ud55c\ub2e4\ub294 \uc810\uc5d0\uc11c\ub3c4 \ucc3e\uc744 \uc218 \uc788\uc744 \uac83\uc774\ub2e4.", "paragraph_id": "6184291-0-0", "paragraph_question": "question: \uc815\ucc45\ud559\uc740 \uc5b4\ub290 \ubd84\uc57c\ub85c \uc131\ub9bd\ub3fc?, context: \uc815\ucc45\ud559(\u653f\u7b56\u5b78, policy science)\uc740 \uadf8 \uc8fc\ub41c \uc5f0\uad6c\uad00\uc2ec \ub300\uc0c1\uacfc \ud750\ub984\uc5d0 \ub530\ub77c \uc815\ucc45\uacfc\uc815, \uc815\ucc45\ud615\uc131, \uc815\ucc45\uc9d1\ud589, \uc815\ucc45\ubd84\uc11d, \uc815\ucc45\ud3c9\uac00 \ub4f1\uc758 \uc138\ubd80 \ubd84\uc57c\ub85c \ubc1c\uc804\ud574\uc654\ub2e4. \uc815\ucc45\ud559\uc774 \ub3c5\uc790\uc801\uc778 \uc0ac\ud68c\uacfc\ud559\uc758 \ud55c \ubd84\uc57c\ub85c \uc131\ub9bd\ud560 \uc218 \uc788\ub294 \uc774\uc720\ub294 \uadf8\uac83\uc774 \uc815\ucc45\uc774\ub77c\ub294 \uc911\uc694\ud55c \uc5f0\uad6c\ub300\uc0c1\uc744 \uc5ec\ub7ec\uac00\uc9c0 \uacfc\ud559\uc801\uc778 \ubc29\ubc95\ub860\uacfc \uc774\ub860\uc744 \uac00\uc9c0\uace0 \uc5f0\uad6c\ud55c\ub2e4\ub294 \uc810 \ubfd0\ub9cc \uc544\ub2c8\ub77c \uadf8\uac83\uc774 \uac00\uce58\uc9c0\ud5a5\uc801 \uc989 \uc815\ucc45\ud559 \uc5f0\uad6c\uc5d0\uc11c \ubbfc\uc8fc\uc801 \uac00\uce58\uc640 \uc0ac\ud68c\uc801 \uac00\uce58\ub97c \uc774\ud574\ud560 \uac83\uc778\uac00\uc758 \ubb38\uc81c \uc5ed\uc2dc \uade0\ud615\ub418\uac8c \uc811\uadfc \ud55c\ub2e4\ub294 \uc810\uc5d0\uc11c\ub3c4 \ucc3e\uc744 \uc218 \uc788\uc744 \uac83\uc774\ub2e4. \uc774\ub7ec\ud55c \uc810\uc5d0\uc11c \ud574\ub7f4\ub4dc \ub77c\uc2a4\uc6f0(Harold Lasswell, 1902-1978)\uc774 \uc81c\uc2dc\ud55c \uc815\ucc45 \uacfc\ud559 \uac1c\ub150\uc758 \uc911\uc694\uc131\uc740 \uc5ec\uc804\ud788 21\uc138\uae30\uc5d0\ub3c4 \uc720\ud6a8\ud558\ub2e4\uace0 \ud560 \uac83\uc774\ub2e4."} {"question": "\uc625\ud0c0\ube44\uc544\ub204\uc2a4\uac00 \uce74\uc774\uc0ac\ub974\uc758 \ud788\uc2a4\ud30c\ub2c8\uc544 \uc6d0\uc815\uc5d0 \ucc38\uc5ec\ud560 \uc218 \uc5c6\uc5c8\ub358 \uc774\uc720\ub294?", "paragraph": "\uae30\uc6d0\uc804 51\ub144, \uc728\ub9ac\uc544 \uce74\uc774\uc0ac\ub9ac\uc2a4\uac00 \uc8fd\uc790, \uc625\ud0c0\ube44\uc544\ub204\uc2a4\ub294 \uc678\ud560\uba38\ub2c8\uc758 \uc7a5\ub840\uc2dd\uc5d0\uc11c \ucd94\ubaa8\uc0ac\ub97c \ub0ad\ub3c5\ud558\uc600\ub2e4. \uc774\ub54c\ubd80\ud130 \uc5b4\uba38\ub2c8 \uc544\ud2f0\uc544\uc640 \uacc4\ubd80 \ud544\ub9ac\ud478\uc2a4\ub294 \uc625\ud0c0\ube44\uc544\ub204\uc2a4\ub97c \ub2e8\ub828\uc2dc\ud0a4\uae30 \uc704\ud574 \uc801\uadf9\uc801\uc73c\ub85c \ub098\uc130\ub2e4. \uc625\ud0c0\ube44\uc544\ub204\uc2a4\ub294 4\ub144 \ub4a4\uc778 \uae30\uc6d0\uc804 47\ub144\ubd80\ud130 \ud1a0\uac00\ub97c \uc785\uac8c \ub418\uc5c8\uace0 \uad6d\uac00 \uc0ac\uc81c\ub2e8(Collegium Pontificum)\uc5d0 \uc120\ucd9c\ub418\uc5c8\ub2e4. \ub2e4\uc74c \ud574\uc5d0 \uc625\ud0c0\ube44\uc544\ub204\uc2a4\ub294 \uce74\uc774\uc0ac\ub974\uac00 \uc138\uc6b4 \ubca0\ub204\uc2a4 \uac8c\ub124\ud2b8\ub9ad\uc2a4 \uc2e0\uc804\uc5d0\uc11c \ud589\ud574\uc9c0\ub294 \uadf8\ub9ac\uc2a4 \uacbd\uae30\ub97c \uad00\uc7a5\ud558\uc600\ub2e4. \ub2e4\ub9c8\uc2a4\ucfe0\uc2a4\uc758 \ub2c8\ucf5c\ub77c\uc624\uc2a4\uc5d0 \ub530\ub974\uba74, \uc625\ud0c0\ube44\uc544\ub204\uc2a4\ub294 \uce74\uc774\uc0ac\ub974\uc758 \uc544\ud504\ub9ac\uce74 \uc6d0\uc815\uc5d0 \ucc38\uc5ec\ud558\uace0 \uc2f6\uc5c8\uc73c\ub098 \uc5b4\uba38\ub2c8\uc758 \ubc18\ub300\ub85c \ubb34\uc0b0\ub418\uc5c8\ub2e4\uace0 \ud55c\ub2e4. \uae30\uc6d0\uc804 46\ub144, \uc544\ud2f0\uc544\ub294 \uc625\ud0c0\ube44\uc544\ub204\uc2a4\uac00 \uce74\uc774\uc0ac\ub974\uc758 \ud788\uc2a4\ud30c\ub2c8\uc544 \uc6d0\uc815\uc5d0 \ucc38\uc5ec\ud558\ub294 \uac83\uc5d0 \ub3d9\uc758\ud558\uc600\ub294\ub370, \uc774 \uc6d0\uc815\uc5d0\uc11c \uce74\uc774\uc0ac\ub974\ub294 \uc624\ub79c \uc219\uc801\uc774\uc5c8\ub358 \ud3fc\ud398\uc774\uc6b0\uc2a4 \ub9c8\uadf8\ub204\uc2a4\uc758 \uc544\ub4e4 \uadf8\ub098\uc774\uc6b0\uc2a4 \ud3fc\ud398\uc774\uc6b0\uc2a4\uc640 \uc2f8\uc6b0\ub824 \ud558\uc600\ub2e4. \ud558\uc9c0\ub9cc \uc625\ud0c0\ube44\uc544\ub204\uc2a4\ub294 \ubcd1\uc774 \ub4e4\uc5b4 \uc6d0\uc815\uc5d0 \ucc38\uc5ec\ud560 \uc218 \uc5c6\uac8c \ub418\uc5c8\ub2e4.", "answer": "\ubcd1", "sentence": "\uc625\ud0c0\ube44\uc544\ub204\uc2a4\ub294 \ubcd1 \uc774 \ub4e4\uc5b4 \uc6d0\uc815\uc5d0 \ucc38\uc5ec\ud560 \uc218 \uc5c6\uac8c \ub418\uc5c8\ub2e4.", "paragraph_sentence": "\uae30\uc6d0\uc804 51\ub144, \uc728\ub9ac\uc544 \uce74\uc774\uc0ac\ub9ac\uc2a4\uac00 \uc8fd\uc790, \uc625\ud0c0\ube44\uc544\ub204\uc2a4\ub294 \uc678\ud560\uba38\ub2c8\uc758 \uc7a5\ub840\uc2dd\uc5d0\uc11c \ucd94\ubaa8\uc0ac\ub97c \ub0ad\ub3c5\ud558\uc600\ub2e4. \uc774\ub54c\ubd80\ud130 \uc5b4\uba38\ub2c8 \uc544\ud2f0\uc544\uc640 \uacc4\ubd80 \ud544\ub9ac\ud478\uc2a4\ub294 \uc625\ud0c0\ube44\uc544\ub204\uc2a4\ub97c \ub2e8\ub828\uc2dc\ud0a4\uae30 \uc704\ud574 \uc801\uadf9\uc801\uc73c\ub85c \ub098\uc130\ub2e4. \uc625\ud0c0\ube44\uc544\ub204\uc2a4\ub294 4\ub144 \ub4a4\uc778 \uae30\uc6d0\uc804 47\ub144\ubd80\ud130 \ud1a0\uac00\ub97c \uc785\uac8c \ub418\uc5c8\uace0 \uad6d\uac00 \uc0ac\uc81c\ub2e8(Collegium Pontificum)\uc5d0 \uc120\ucd9c\ub418\uc5c8\ub2e4. \ub2e4\uc74c \ud574\uc5d0 \uc625\ud0c0\ube44\uc544\ub204\uc2a4\ub294 \uce74\uc774\uc0ac\ub974\uac00 \uc138\uc6b4 \ubca0\ub204\uc2a4 \uac8c\ub124\ud2b8\ub9ad\uc2a4 \uc2e0\uc804\uc5d0\uc11c \ud589\ud574\uc9c0\ub294 \uadf8\ub9ac\uc2a4 \uacbd\uae30\ub97c \uad00\uc7a5\ud558\uc600\ub2e4. \ub2e4\ub9c8\uc2a4\ucfe0\uc2a4\uc758 \ub2c8\ucf5c\ub77c\uc624\uc2a4\uc5d0 \ub530\ub974\uba74, \uc625\ud0c0\ube44\uc544\ub204\uc2a4\ub294 \uce74\uc774\uc0ac\ub974\uc758 \uc544\ud504\ub9ac\uce74 \uc6d0\uc815\uc5d0 \ucc38\uc5ec\ud558\uace0 \uc2f6\uc5c8\uc73c\ub098 \uc5b4\uba38\ub2c8\uc758 \ubc18\ub300\ub85c \ubb34\uc0b0\ub418\uc5c8\ub2e4\uace0 \ud55c\ub2e4. \uae30\uc6d0\uc804 46\ub144, \uc544\ud2f0\uc544\ub294 \uc625\ud0c0\ube44\uc544\ub204\uc2a4\uac00 \uce74\uc774\uc0ac\ub974\uc758 \ud788\uc2a4\ud30c\ub2c8\uc544 \uc6d0\uc815\uc5d0 \ucc38\uc5ec\ud558\ub294 \uac83\uc5d0 \ub3d9\uc758\ud558\uc600\ub294\ub370, \uc774 \uc6d0\uc815\uc5d0\uc11c \uce74\uc774\uc0ac\ub974\ub294 \uc624\ub79c \uc219\uc801\uc774\uc5c8\ub358 \ud3fc\ud398\uc774\uc6b0\uc2a4 \ub9c8\uadf8\ub204\uc2a4\uc758 \uc544\ub4e4 \uadf8\ub098\uc774\uc6b0\uc2a4 \ud3fc\ud398\uc774\uc6b0\uc2a4\uc640 \uc2f8\uc6b0\ub824 \ud558\uc600\ub2e4. \ud558\uc9c0\ub9cc \uc625\ud0c0\ube44\uc544\ub204\uc2a4\ub294 \ubcd1 \uc774 \ub4e4\uc5b4 \uc6d0\uc815\uc5d0 \ucc38\uc5ec\ud560 \uc218 \uc5c6\uac8c \ub418\uc5c8\ub2e4. ", "paragraph_answer": "\uae30\uc6d0\uc804 51\ub144, \uc728\ub9ac\uc544 \uce74\uc774\uc0ac\ub9ac\uc2a4\uac00 \uc8fd\uc790, \uc625\ud0c0\ube44\uc544\ub204\uc2a4\ub294 \uc678\ud560\uba38\ub2c8\uc758 \uc7a5\ub840\uc2dd\uc5d0\uc11c \ucd94\ubaa8\uc0ac\ub97c \ub0ad\ub3c5\ud558\uc600\ub2e4. \uc774\ub54c\ubd80\ud130 \uc5b4\uba38\ub2c8 \uc544\ud2f0\uc544\uc640 \uacc4\ubd80 \ud544\ub9ac\ud478\uc2a4\ub294 \uc625\ud0c0\ube44\uc544\ub204\uc2a4\ub97c \ub2e8\ub828\uc2dc\ud0a4\uae30 \uc704\ud574 \uc801\uadf9\uc801\uc73c\ub85c \ub098\uc130\ub2e4. \uc625\ud0c0\ube44\uc544\ub204\uc2a4\ub294 4\ub144 \ub4a4\uc778 \uae30\uc6d0\uc804 47\ub144\ubd80\ud130 \ud1a0\uac00\ub97c \uc785\uac8c \ub418\uc5c8\uace0 \uad6d\uac00 \uc0ac\uc81c\ub2e8(Collegium Pontificum)\uc5d0 \uc120\ucd9c\ub418\uc5c8\ub2e4. \ub2e4\uc74c \ud574\uc5d0 \uc625\ud0c0\ube44\uc544\ub204\uc2a4\ub294 \uce74\uc774\uc0ac\ub974\uac00 \uc138\uc6b4 \ubca0\ub204\uc2a4 \uac8c\ub124\ud2b8\ub9ad\uc2a4 \uc2e0\uc804\uc5d0\uc11c \ud589\ud574\uc9c0\ub294 \uadf8\ub9ac\uc2a4 \uacbd\uae30\ub97c \uad00\uc7a5\ud558\uc600\ub2e4. \ub2e4\ub9c8\uc2a4\ucfe0\uc2a4\uc758 \ub2c8\ucf5c\ub77c\uc624\uc2a4\uc5d0 \ub530\ub974\uba74, \uc625\ud0c0\ube44\uc544\ub204\uc2a4\ub294 \uce74\uc774\uc0ac\ub974\uc758 \uc544\ud504\ub9ac\uce74 \uc6d0\uc815\uc5d0 \ucc38\uc5ec\ud558\uace0 \uc2f6\uc5c8\uc73c\ub098 \uc5b4\uba38\ub2c8\uc758 \ubc18\ub300\ub85c \ubb34\uc0b0\ub418\uc5c8\ub2e4\uace0 \ud55c\ub2e4. \uae30\uc6d0\uc804 46\ub144, \uc544\ud2f0\uc544\ub294 \uc625\ud0c0\ube44\uc544\ub204\uc2a4\uac00 \uce74\uc774\uc0ac\ub974\uc758 \ud788\uc2a4\ud30c\ub2c8\uc544 \uc6d0\uc815\uc5d0 \ucc38\uc5ec\ud558\ub294 \uac83\uc5d0 \ub3d9\uc758\ud558\uc600\ub294\ub370, \uc774 \uc6d0\uc815\uc5d0\uc11c \uce74\uc774\uc0ac\ub974\ub294 \uc624\ub79c \uc219\uc801\uc774\uc5c8\ub358 \ud3fc\ud398\uc774\uc6b0\uc2a4 \ub9c8\uadf8\ub204\uc2a4\uc758 \uc544\ub4e4 \uadf8\ub098\uc774\uc6b0\uc2a4 \ud3fc\ud398\uc774\uc6b0\uc2a4\uc640 \uc2f8\uc6b0\ub824 \ud558\uc600\ub2e4. \ud558\uc9c0\ub9cc \uc625\ud0c0\ube44\uc544\ub204\uc2a4\ub294 \ubcd1 \uc774 \ub4e4\uc5b4 \uc6d0\uc815\uc5d0 \ucc38\uc5ec\ud560 \uc218 \uc5c6\uac8c \ub418\uc5c8\ub2e4.", "sentence_answer": "\uc625\ud0c0\ube44\uc544\ub204\uc2a4\ub294 \ubcd1 \uc774 \ub4e4\uc5b4 \uc6d0\uc815\uc5d0 \ucc38\uc5ec\ud560 \uc218 \uc5c6\uac8c \ub418\uc5c8\ub2e4.", "paragraph_id": "6520922-3-2", "paragraph_question": "question: \uc625\ud0c0\ube44\uc544\ub204\uc2a4\uac00 \uce74\uc774\uc0ac\ub974\uc758 \ud788\uc2a4\ud30c\ub2c8\uc544 \uc6d0\uc815\uc5d0 \ucc38\uc5ec\ud560 \uc218 \uc5c6\uc5c8\ub358 \uc774\uc720\ub294?, context: \uae30\uc6d0\uc804 51\ub144, \uc728\ub9ac\uc544 \uce74\uc774\uc0ac\ub9ac\uc2a4\uac00 \uc8fd\uc790, \uc625\ud0c0\ube44\uc544\ub204\uc2a4\ub294 \uc678\ud560\uba38\ub2c8\uc758 \uc7a5\ub840\uc2dd\uc5d0\uc11c \ucd94\ubaa8\uc0ac\ub97c \ub0ad\ub3c5\ud558\uc600\ub2e4. \uc774\ub54c\ubd80\ud130 \uc5b4\uba38\ub2c8 \uc544\ud2f0\uc544\uc640 \uacc4\ubd80 \ud544\ub9ac\ud478\uc2a4\ub294 \uc625\ud0c0\ube44\uc544\ub204\uc2a4\ub97c \ub2e8\ub828\uc2dc\ud0a4\uae30 \uc704\ud574 \uc801\uadf9\uc801\uc73c\ub85c \ub098\uc130\ub2e4. \uc625\ud0c0\ube44\uc544\ub204\uc2a4\ub294 4\ub144 \ub4a4\uc778 \uae30\uc6d0\uc804 47\ub144\ubd80\ud130 \ud1a0\uac00\ub97c \uc785\uac8c \ub418\uc5c8\uace0 \uad6d\uac00 \uc0ac\uc81c\ub2e8(Collegium Pontificum)\uc5d0 \uc120\ucd9c\ub418\uc5c8\ub2e4. \ub2e4\uc74c \ud574\uc5d0 \uc625\ud0c0\ube44\uc544\ub204\uc2a4\ub294 \uce74\uc774\uc0ac\ub974\uac00 \uc138\uc6b4 \ubca0\ub204\uc2a4 \uac8c\ub124\ud2b8\ub9ad\uc2a4 \uc2e0\uc804\uc5d0\uc11c \ud589\ud574\uc9c0\ub294 \uadf8\ub9ac\uc2a4 \uacbd\uae30\ub97c \uad00\uc7a5\ud558\uc600\ub2e4. \ub2e4\ub9c8\uc2a4\ucfe0\uc2a4\uc758 \ub2c8\ucf5c\ub77c\uc624\uc2a4\uc5d0 \ub530\ub974\uba74, \uc625\ud0c0\ube44\uc544\ub204\uc2a4\ub294 \uce74\uc774\uc0ac\ub974\uc758 \uc544\ud504\ub9ac\uce74 \uc6d0\uc815\uc5d0 \ucc38\uc5ec\ud558\uace0 \uc2f6\uc5c8\uc73c\ub098 \uc5b4\uba38\ub2c8\uc758 \ubc18\ub300\ub85c \ubb34\uc0b0\ub418\uc5c8\ub2e4\uace0 \ud55c\ub2e4. \uae30\uc6d0\uc804 46\ub144, \uc544\ud2f0\uc544\ub294 \uc625\ud0c0\ube44\uc544\ub204\uc2a4\uac00 \uce74\uc774\uc0ac\ub974\uc758 \ud788\uc2a4\ud30c\ub2c8\uc544 \uc6d0\uc815\uc5d0 \ucc38\uc5ec\ud558\ub294 \uac83\uc5d0 \ub3d9\uc758\ud558\uc600\ub294\ub370, \uc774 \uc6d0\uc815\uc5d0\uc11c \uce74\uc774\uc0ac\ub974\ub294 \uc624\ub79c \uc219\uc801\uc774\uc5c8\ub358 \ud3fc\ud398\uc774\uc6b0\uc2a4 \ub9c8\uadf8\ub204\uc2a4\uc758 \uc544\ub4e4 \uadf8\ub098\uc774\uc6b0\uc2a4 \ud3fc\ud398\uc774\uc6b0\uc2a4\uc640 \uc2f8\uc6b0\ub824 \ud558\uc600\ub2e4. \ud558\uc9c0\ub9cc \uc625\ud0c0\ube44\uc544\ub204\uc2a4\ub294 \ubcd1\uc774 \ub4e4\uc5b4 \uc6d0\uc815\uc5d0 \ucc38\uc5ec\ud560 \uc218 \uc5c6\uac8c \ub418\uc5c8\ub2e4."} {"question": "\ub300\uc911\ub4e4\uc740 \uc5b8\uc81c\ubd80\ud130 \ub300\uc911\ubb38\ud654\uc5d0 \ub300\ud574 \ub354 \ub9ce\uc740 \uad00\uc2ec\uc744 \uac00\uc9c0\uae30 \uc2dc\uc791\ud558\uc600\ub294\uac00?", "paragraph": "1987\ub144 6\u00b729 \uc120\uc5b8\uc73c\ub85c \uc778\ud574 \ub300\ud55c\ubbfc\uad6d\uc5d0 \ubbfc\uc8fc\uc8fc\uc758\uac00 \ub4e4\uc5b4\uc11c\uae30 \uc2dc\uc791\ud558\uba74\uc11c \ub300\uc911\ub4e4\uc740 \ub300\uc911\ubb38\ud654\uc5d0 \ub300\ud574 \ub354 \ub9ce\uc740 \uad00\uc2ec\uc744 \uac00\uc9c0\uae30 \uc2dc\uc791\ud588\ub2e4. \uc774\uc640 \ud568\uaed8 SBS\ub97c \ud3ec\ud568\ud55c \uc5ec\ub7ec \ucf00\uc774\ube14 TV, \uc704\uc131\ubc29\uc1a1 \ucc44\ub110 \ub4f1\uc774 \uac1c\uad6d\ud558\uba74\uc11c \ubcf8\uaca9\uc801\uc778 \uc0c1\uc5c5 \ubc29\uc1a1\uc774 \ud65c\uc131\ud654\ub418\uc5c8\uace0, \uc774\ub294 \uace7 \ubc29\uc1a1 \ud658\uacbd\uc758 \ubc1c\ub2ec\ub85c \uc774\uc5b4\uc84c\ub2e4. \ubc29\uc1a1 \ud658\uacbd\uc758 \ubc1c\ub2ec\uc740 \ub300\uae30\uc5c5 \uc790\ubcf8\uc758 \ubb38\ud654 \uc0b0\uc5c5 \ucc38\uc5ec\ub97c \uc720\ub3c4\ud588\uace0, \uadf8 \uacb0\uacfc \uccb4\uacc4\uc801\uc778 \uc0dd\uc0b0 \uacfc\uc815\uc758 \ub3c4\uc785 \ub4f1 \ubb38\ud654 \ucf58\ud150\uce20\uc758 \uc9c8\uc744 \ub192\uc774\ub294 \uc5ec\ub7ec \ubcc0\ud654\uac00 \uc77c\uc5b4\ub0ac\ub2e4. \uc774 \uacfc\uc815\uc5d0\uc11c \ub300\ud55c\ubbfc\uad6d\uc758 \ubb38\ud654 \uc0b0\uc5c5\uc5d0\ub294 \u2018\uae30\ud68d\u2019\uc774\ub77c\ub294 \uac1c\ub150\uc774 \ubcf8\uaca9\uc801\uc73c\ub85c \uc790\ub9ac \uc7a1\uae30 \uc2dc\uc791\ud588\ub294\ub370, \uadf8 \uc608\ub85c 1990\ub144\ub300\uc5d0 \uc124\ub9bd\ub41c \uae30\ud68d \uc601\ud654 \uc81c\uc791\uc0ac \u2018\uc2e0\uc528\ub124\u2019, \uae30\ud68d \uc544\uc774\ub3cc \uc18c\uc18d\uc0ac \u2018SM\uc5d4\ud130\ud14c\uc778\uba3c\ud2b8\u2019 \ub4f1\uc774 \uc788\ub2e4.", "answer": "\ub300\ud55c\ubbfc\uad6d\uc5d0 \ubbfc\uc8fc\uc8fc\uc758\uac00 \ub4e4\uc5b4\uc11c\uae30 \uc2dc\uc791\ud558\uba74\uc11c", "sentence": "1987\ub144 6\u00b729 \uc120\uc5b8\uc73c\ub85c \uc778\ud574 \ub300\ud55c\ubbfc\uad6d\uc5d0 \ubbfc\uc8fc\uc8fc\uc758\uac00 \ub4e4\uc5b4\uc11c\uae30 \uc2dc\uc791\ud558\uba74\uc11c \ub300\uc911\ub4e4\uc740 \ub300\uc911\ubb38\ud654\uc5d0 \ub300\ud574 \ub354 \ub9ce\uc740 \uad00\uc2ec\uc744 \uac00\uc9c0\uae30 \uc2dc\uc791\ud588\ub2e4.", "paragraph_sentence": " 1987\ub144 6\u00b729 \uc120\uc5b8\uc73c\ub85c \uc778\ud574 \ub300\ud55c\ubbfc\uad6d\uc5d0 \ubbfc\uc8fc\uc8fc\uc758\uac00 \ub4e4\uc5b4\uc11c\uae30 \uc2dc\uc791\ud558\uba74\uc11c \ub300\uc911\ub4e4\uc740 \ub300\uc911\ubb38\ud654\uc5d0 \ub300\ud574 \ub354 \ub9ce\uc740 \uad00\uc2ec\uc744 \uac00\uc9c0\uae30 \uc2dc\uc791\ud588\ub2e4. \uc774\uc640 \ud568\uaed8 SBS\ub97c \ud3ec\ud568\ud55c \uc5ec\ub7ec \ucf00\uc774\ube14 TV, \uc704\uc131\ubc29\uc1a1 \ucc44\ub110 \ub4f1\uc774 \uac1c\uad6d\ud558\uba74\uc11c \ubcf8\uaca9\uc801\uc778 \uc0c1\uc5c5 \ubc29\uc1a1\uc774 \ud65c\uc131\ud654\ub418\uc5c8\uace0, \uc774\ub294 \uace7 \ubc29\uc1a1 \ud658\uacbd\uc758 \ubc1c\ub2ec\ub85c \uc774\uc5b4\uc84c\ub2e4. \ubc29\uc1a1 \ud658\uacbd\uc758 \ubc1c\ub2ec\uc740 \ub300\uae30\uc5c5 \uc790\ubcf8\uc758 \ubb38\ud654 \uc0b0\uc5c5 \ucc38\uc5ec\ub97c \uc720\ub3c4\ud588\uace0, \uadf8 \uacb0\uacfc \uccb4\uacc4\uc801\uc778 \uc0dd\uc0b0 \uacfc\uc815\uc758 \ub3c4\uc785 \ub4f1 \ubb38\ud654 \ucf58\ud150\uce20\uc758 \uc9c8\uc744 \ub192\uc774\ub294 \uc5ec\ub7ec \ubcc0\ud654\uac00 \uc77c\uc5b4\ub0ac\ub2e4. \uc774 \uacfc\uc815\uc5d0\uc11c \ub300\ud55c\ubbfc\uad6d\uc758 \ubb38\ud654 \uc0b0\uc5c5\uc5d0\ub294 \u2018\uae30\ud68d\u2019\uc774\ub77c\ub294 \uac1c\ub150\uc774 \ubcf8\uaca9\uc801\uc73c\ub85c \uc790\ub9ac \uc7a1\uae30 \uc2dc\uc791\ud588\ub294\ub370, \uadf8 \uc608\ub85c 1990\ub144\ub300\uc5d0 \uc124\ub9bd\ub41c \uae30\ud68d \uc601\ud654 \uc81c\uc791\uc0ac \u2018\uc2e0\uc528\ub124\u2019, \uae30\ud68d \uc544\uc774\ub3cc \uc18c\uc18d\uc0ac \u2018SM\uc5d4\ud130\ud14c\uc778\uba3c\ud2b8\u2019 \ub4f1\uc774 \uc788\ub2e4.", "paragraph_answer": "1987\ub144 6\u00b729 \uc120\uc5b8\uc73c\ub85c \uc778\ud574 \ub300\ud55c\ubbfc\uad6d\uc5d0 \ubbfc\uc8fc\uc8fc\uc758\uac00 \ub4e4\uc5b4\uc11c\uae30 \uc2dc\uc791\ud558\uba74\uc11c \ub300\uc911\ub4e4\uc740 \ub300\uc911\ubb38\ud654\uc5d0 \ub300\ud574 \ub354 \ub9ce\uc740 \uad00\uc2ec\uc744 \uac00\uc9c0\uae30 \uc2dc\uc791\ud588\ub2e4. \uc774\uc640 \ud568\uaed8 SBS\ub97c \ud3ec\ud568\ud55c \uc5ec\ub7ec \ucf00\uc774\ube14 TV, \uc704\uc131\ubc29\uc1a1 \ucc44\ub110 \ub4f1\uc774 \uac1c\uad6d\ud558\uba74\uc11c \ubcf8\uaca9\uc801\uc778 \uc0c1\uc5c5 \ubc29\uc1a1\uc774 \ud65c\uc131\ud654\ub418\uc5c8\uace0, \uc774\ub294 \uace7 \ubc29\uc1a1 \ud658\uacbd\uc758 \ubc1c\ub2ec\ub85c \uc774\uc5b4\uc84c\ub2e4. \ubc29\uc1a1 \ud658\uacbd\uc758 \ubc1c\ub2ec\uc740 \ub300\uae30\uc5c5 \uc790\ubcf8\uc758 \ubb38\ud654 \uc0b0\uc5c5 \ucc38\uc5ec\ub97c \uc720\ub3c4\ud588\uace0, \uadf8 \uacb0\uacfc \uccb4\uacc4\uc801\uc778 \uc0dd\uc0b0 \uacfc\uc815\uc758 \ub3c4\uc785 \ub4f1 \ubb38\ud654 \ucf58\ud150\uce20\uc758 \uc9c8\uc744 \ub192\uc774\ub294 \uc5ec\ub7ec \ubcc0\ud654\uac00 \uc77c\uc5b4\ub0ac\ub2e4. \uc774 \uacfc\uc815\uc5d0\uc11c \ub300\ud55c\ubbfc\uad6d\uc758 \ubb38\ud654 \uc0b0\uc5c5\uc5d0\ub294 \u2018\uae30\ud68d\u2019\uc774\ub77c\ub294 \uac1c\ub150\uc774 \ubcf8\uaca9\uc801\uc73c\ub85c \uc790\ub9ac \uc7a1\uae30 \uc2dc\uc791\ud588\ub294\ub370, \uadf8 \uc608\ub85c 1990\ub144\ub300\uc5d0 \uc124\ub9bd\ub41c \uae30\ud68d \uc601\ud654 \uc81c\uc791\uc0ac \u2018\uc2e0\uc528\ub124\u2019, \uae30\ud68d \uc544\uc774\ub3cc \uc18c\uc18d\uc0ac \u2018SM\uc5d4\ud130\ud14c\uc778\uba3c\ud2b8\u2019 \ub4f1\uc774 \uc788\ub2e4.", "sentence_answer": "1987\ub144 6\u00b729 \uc120\uc5b8\uc73c\ub85c \uc778\ud574 \ub300\ud55c\ubbfc\uad6d\uc5d0 \ubbfc\uc8fc\uc8fc\uc758\uac00 \ub4e4\uc5b4\uc11c\uae30 \uc2dc\uc791\ud558\uba74\uc11c \ub300\uc911\ub4e4\uc740 \ub300\uc911\ubb38\ud654\uc5d0 \ub300\ud574 \ub354 \ub9ce\uc740 \uad00\uc2ec\uc744 \uac00\uc9c0\uae30 \uc2dc\uc791\ud588\ub2e4.", "paragraph_id": "5936907-1-0", "paragraph_question": "question: \ub300\uc911\ub4e4\uc740 \uc5b8\uc81c\ubd80\ud130 \ub300\uc911\ubb38\ud654\uc5d0 \ub300\ud574 \ub354 \ub9ce\uc740 \uad00\uc2ec\uc744 \uac00\uc9c0\uae30 \uc2dc\uc791\ud558\uc600\ub294\uac00?, context: 1987\ub144 6\u00b729 \uc120\uc5b8\uc73c\ub85c \uc778\ud574 \ub300\ud55c\ubbfc\uad6d\uc5d0 \ubbfc\uc8fc\uc8fc\uc758\uac00 \ub4e4\uc5b4\uc11c\uae30 \uc2dc\uc791\ud558\uba74\uc11c \ub300\uc911\ub4e4\uc740 \ub300\uc911\ubb38\ud654\uc5d0 \ub300\ud574 \ub354 \ub9ce\uc740 \uad00\uc2ec\uc744 \uac00\uc9c0\uae30 \uc2dc\uc791\ud588\ub2e4. \uc774\uc640 \ud568\uaed8 SBS\ub97c \ud3ec\ud568\ud55c \uc5ec\ub7ec \ucf00\uc774\ube14 TV, \uc704\uc131\ubc29\uc1a1 \ucc44\ub110 \ub4f1\uc774 \uac1c\uad6d\ud558\uba74\uc11c \ubcf8\uaca9\uc801\uc778 \uc0c1\uc5c5 \ubc29\uc1a1\uc774 \ud65c\uc131\ud654\ub418\uc5c8\uace0, \uc774\ub294 \uace7 \ubc29\uc1a1 \ud658\uacbd\uc758 \ubc1c\ub2ec\ub85c \uc774\uc5b4\uc84c\ub2e4. \ubc29\uc1a1 \ud658\uacbd\uc758 \ubc1c\ub2ec\uc740 \ub300\uae30\uc5c5 \uc790\ubcf8\uc758 \ubb38\ud654 \uc0b0\uc5c5 \ucc38\uc5ec\ub97c \uc720\ub3c4\ud588\uace0, \uadf8 \uacb0\uacfc \uccb4\uacc4\uc801\uc778 \uc0dd\uc0b0 \uacfc\uc815\uc758 \ub3c4\uc785 \ub4f1 \ubb38\ud654 \ucf58\ud150\uce20\uc758 \uc9c8\uc744 \ub192\uc774\ub294 \uc5ec\ub7ec \ubcc0\ud654\uac00 \uc77c\uc5b4\ub0ac\ub2e4. \uc774 \uacfc\uc815\uc5d0\uc11c \ub300\ud55c\ubbfc\uad6d\uc758 \ubb38\ud654 \uc0b0\uc5c5\uc5d0\ub294 \u2018\uae30\ud68d\u2019\uc774\ub77c\ub294 \uac1c\ub150\uc774 \ubcf8\uaca9\uc801\uc73c\ub85c \uc790\ub9ac \uc7a1\uae30 \uc2dc\uc791\ud588\ub294\ub370, \uadf8 \uc608\ub85c 1990\ub144\ub300\uc5d0 \uc124\ub9bd\ub41c \uae30\ud68d \uc601\ud654 \uc81c\uc791\uc0ac \u2018\uc2e0\uc528\ub124\u2019, \uae30\ud68d \uc544\uc774\ub3cc \uc18c\uc18d\uc0ac \u2018SM\uc5d4\ud130\ud14c\uc778\uba3c\ud2b8\u2019 \ub4f1\uc774 \uc788\ub2e4."} {"question": "\uc11c\uc6d0\ud6c8\uc758 \uc9c1\uc5c5\uc740 \ubb34\uc5c7\uc778\uac00\uc694?", "paragraph": "1904\ub144 \uc544\ubc84\uc9c0 \uc548\ud0dc\ud6c8\uacfc \uccad\ub098\ub77c \uc758\uc0ac \uc0ac\uc774\uc5d0 \ubc8c\uc5b4\uc9c4 \uc2f8\uc6c0\uc5d0 \ud718\ub9d0\ub838\ub2e4\uac00 \uc678\uad50 \ubb38\uc81c\ub85c \ube44\ud654\ub418\uae30\ub3c4 \ud588\ub2e4. \uc544\ubc84\uc9c0 \uc548\ud0dc\ud6c8\uc740 1904\ub144 4\uc6d4 20\uc77c \uc548\uc545\uc74d\uc5d0 \uc0ac\ub294 \uccad\ub098\ub77c \uc758\uc0ac \uc11c\uc6d0\ud6c8(\u8212\u5143\u52db)\uacfc \uc2dc\ube44\uac00 \uc788\uc5c8\ub2e4. \uc544\ubc84\uc9c0 \uc548\ud0dc\ud6c8\uc740 \ubcd1\uc744 \uce58\ub8cc\ud558\uae30 \uc704\ud574 \uce5c\uc6b0 \uc774\uc6a9\uc77c(\u674e\u9f8d\u4e00)\uc744 \ub370\ub9ac\uace0 \uc11c\uc6d0\ud6c8\uc744 \ucc3e\uc544\uac14\ub2e4. \uc591\uce21\uc774 \ud544\ub2f4\uc744 \ub098\ub204\ub294 \uc0ac\uc774\uc5d0 \uc548\ud0dc\ud6c8\uce21\uc774 \uc5b4\ub5a4 \uc2e4\uc218\ub97c \ud588\ub294\uc9c0 \uc11c\uc6d0\ud6c8\uc774 \uac11\uc790\uae30 \uc77c\uc5b4\ub098 \uc548\ud0dc\ud6c8\uc758 \uac00\uc2b4\uc744 \ubc1c\ub85c \uac77\uc5b4\ucc3c\ub2e4. \uc774\uc5d0 \uc548\ud0dc\ud6c8\uc740 \uc11c\uc6d0\ud6c8\uc5d0\uac8c \ud654\ud574\ub97c \uccad\ud558\uace0 \ubb3c\ub7ec\ub098\uc654\ub2e4. \uadf8\ub7ec\ub098 \uc774 \uc18c\uc2dd\uc744 \ub4e4\uc740 \uc548\uc911\uadfc\uc774 4\uc6d4 29\uc77c \uc774\uc6a9\ub4f1 10\uc5ec \uba85\uacfc \ud568\uaed8 \ubb34\uae30\ub97c \ub4e4\uace0 \uc57c\ubc24\uc5d0 \uc11c\uc6d0\ud6c8\uc744 \uc7a1\uc544\ub2e4\uac00 \uae38\uac00\uc5d0\uc11c \ubb34\uc218\ud788 \ub09c\ud0c0\ud558\uc5ec \uac70\ub3d9\uc774 \uc5b4\ub835\uac8c \ub9cc\ub4e4\uc5c8\uace0, \uc774\uc5d0 \ub300\ud55c \ubc18\ubc1c\ub85c 1904\ub144 5\uc6d4 2\uc77c \uccad\uad6d\uc778 7~8\uba85\uc774 \uc774\uc6a9\uc77c\uc758 \uc9d1\uc5d0 \ub09c\uc785\ud558\uc5ec \uadf8\ub97c \ub9c8\uad6c \uad6c\ud0c0\ud558\uace0 \uc7a1\uc544\uac00\ub824\uace0 \ud588\ub2e4.", "answer": "\uccad\ub098\ub77c \uc758\uc0ac", "sentence": "1904\ub144 \uc544\ubc84\uc9c0 \uc548\ud0dc\ud6c8\uacfc \uccad\ub098\ub77c \uc758\uc0ac \uc0ac\uc774\uc5d0 \ubc8c\uc5b4\uc9c4 \uc2f8\uc6c0\uc5d0 \ud718\ub9d0\ub838\ub2e4\uac00 \uc678\uad50 \ubb38\uc81c\ub85c \ube44\ud654\ub418\uae30\ub3c4 \ud588\ub2e4.", "paragraph_sentence": " 1904\ub144 \uc544\ubc84\uc9c0 \uc548\ud0dc\ud6c8\uacfc \uccad\ub098\ub77c \uc758\uc0ac \uc0ac\uc774\uc5d0 \ubc8c\uc5b4\uc9c4 \uc2f8\uc6c0\uc5d0 \ud718\ub9d0\ub838\ub2e4\uac00 \uc678\uad50 \ubb38\uc81c\ub85c \ube44\ud654\ub418\uae30\ub3c4 \ud588\ub2e4. \uc544\ubc84\uc9c0 \uc548\ud0dc\ud6c8\uc740 1904\ub144 4\uc6d4 20\uc77c \uc548\uc545\uc74d\uc5d0 \uc0ac\ub294 \uccad\ub098\ub77c \uc758\uc0ac \uc11c\uc6d0\ud6c8(\u8212\u5143\u52db)\uacfc \uc2dc\ube44\uac00 \uc788\uc5c8\ub2e4. \uc544\ubc84\uc9c0 \uc548\ud0dc\ud6c8\uc740 \ubcd1\uc744 \uce58\ub8cc\ud558\uae30 \uc704\ud574 \uce5c\uc6b0 \uc774\uc6a9\uc77c(\u674e\u9f8d\u4e00)\uc744 \ub370\ub9ac\uace0 \uc11c\uc6d0\ud6c8\uc744 \ucc3e\uc544\uac14\ub2e4. \uc591\uce21\uc774 \ud544\ub2f4\uc744 \ub098\ub204\ub294 \uc0ac\uc774\uc5d0 \uc548\ud0dc\ud6c8\uce21\uc774 \uc5b4\ub5a4 \uc2e4\uc218\ub97c \ud588\ub294\uc9c0 \uc11c\uc6d0\ud6c8\uc774 \uac11\uc790\uae30 \uc77c\uc5b4\ub098 \uc548\ud0dc\ud6c8\uc758 \uac00\uc2b4\uc744 \ubc1c\ub85c \uac77\uc5b4\ucc3c\ub2e4. \uc774\uc5d0 \uc548\ud0dc\ud6c8\uc740 \uc11c\uc6d0\ud6c8\uc5d0\uac8c \ud654\ud574\ub97c \uccad\ud558\uace0 \ubb3c\ub7ec\ub098\uc654\ub2e4. \uadf8\ub7ec\ub098 \uc774 \uc18c\uc2dd\uc744 \ub4e4\uc740 \uc548\uc911\uadfc\uc774 4\uc6d4 29\uc77c \uc774\uc6a9\ub4f1 10\uc5ec \uba85\uacfc \ud568\uaed8 \ubb34\uae30\ub97c \ub4e4\uace0 \uc57c\ubc24\uc5d0 \uc11c\uc6d0\ud6c8\uc744 \uc7a1\uc544\ub2e4\uac00 \uae38\uac00\uc5d0\uc11c \ubb34\uc218\ud788 \ub09c\ud0c0\ud558\uc5ec \uac70\ub3d9\uc774 \uc5b4\ub835\uac8c \ub9cc\ub4e4\uc5c8\uace0, \uc774\uc5d0 \ub300\ud55c \ubc18\ubc1c\ub85c 1904\ub144 5\uc6d4 2\uc77c \uccad\uad6d\uc778 7~8\uba85\uc774 \uc774\uc6a9\uc77c\uc758 \uc9d1\uc5d0 \ub09c\uc785\ud558\uc5ec \uadf8\ub97c \ub9c8\uad6c \uad6c\ud0c0\ud558\uace0 \uc7a1\uc544\uac00\ub824\uace0 \ud588\ub2e4.", "paragraph_answer": "1904\ub144 \uc544\ubc84\uc9c0 \uc548\ud0dc\ud6c8\uacfc \uccad\ub098\ub77c \uc758\uc0ac \uc0ac\uc774\uc5d0 \ubc8c\uc5b4\uc9c4 \uc2f8\uc6c0\uc5d0 \ud718\ub9d0\ub838\ub2e4\uac00 \uc678\uad50 \ubb38\uc81c\ub85c \ube44\ud654\ub418\uae30\ub3c4 \ud588\ub2e4. \uc544\ubc84\uc9c0 \uc548\ud0dc\ud6c8\uc740 1904\ub144 4\uc6d4 20\uc77c \uc548\uc545\uc74d\uc5d0 \uc0ac\ub294 \uccad\ub098\ub77c \uc758\uc0ac \uc11c\uc6d0\ud6c8(\u8212\u5143\u52db)\uacfc \uc2dc\ube44\uac00 \uc788\uc5c8\ub2e4. \uc544\ubc84\uc9c0 \uc548\ud0dc\ud6c8\uc740 \ubcd1\uc744 \uce58\ub8cc\ud558\uae30 \uc704\ud574 \uce5c\uc6b0 \uc774\uc6a9\uc77c(\u674e\u9f8d\u4e00)\uc744 \ub370\ub9ac\uace0 \uc11c\uc6d0\ud6c8\uc744 \ucc3e\uc544\uac14\ub2e4. \uc591\uce21\uc774 \ud544\ub2f4\uc744 \ub098\ub204\ub294 \uc0ac\uc774\uc5d0 \uc548\ud0dc\ud6c8\uce21\uc774 \uc5b4\ub5a4 \uc2e4\uc218\ub97c \ud588\ub294\uc9c0 \uc11c\uc6d0\ud6c8\uc774 \uac11\uc790\uae30 \uc77c\uc5b4\ub098 \uc548\ud0dc\ud6c8\uc758 \uac00\uc2b4\uc744 \ubc1c\ub85c \uac77\uc5b4\ucc3c\ub2e4. \uc774\uc5d0 \uc548\ud0dc\ud6c8\uc740 \uc11c\uc6d0\ud6c8\uc5d0\uac8c \ud654\ud574\ub97c \uccad\ud558\uace0 \ubb3c\ub7ec\ub098\uc654\ub2e4. \uadf8\ub7ec\ub098 \uc774 \uc18c\uc2dd\uc744 \ub4e4\uc740 \uc548\uc911\uadfc\uc774 4\uc6d4 29\uc77c \uc774\uc6a9\ub4f1 10\uc5ec \uba85\uacfc \ud568\uaed8 \ubb34\uae30\ub97c \ub4e4\uace0 \uc57c\ubc24\uc5d0 \uc11c\uc6d0\ud6c8\uc744 \uc7a1\uc544\ub2e4\uac00 \uae38\uac00\uc5d0\uc11c \ubb34\uc218\ud788 \ub09c\ud0c0\ud558\uc5ec \uac70\ub3d9\uc774 \uc5b4\ub835\uac8c \ub9cc\ub4e4\uc5c8\uace0, \uc774\uc5d0 \ub300\ud55c \ubc18\ubc1c\ub85c 1904\ub144 5\uc6d4 2\uc77c \uccad\uad6d\uc778 7~8\uba85\uc774 \uc774\uc6a9\uc77c\uc758 \uc9d1\uc5d0 \ub09c\uc785\ud558\uc5ec \uadf8\ub97c \ub9c8\uad6c \uad6c\ud0c0\ud558\uace0 \uc7a1\uc544\uac00\ub824\uace0 \ud588\ub2e4.", "sentence_answer": "1904\ub144 \uc544\ubc84\uc9c0 \uc548\ud0dc\ud6c8\uacfc \uccad\ub098\ub77c \uc758\uc0ac \uc0ac\uc774\uc5d0 \ubc8c\uc5b4\uc9c4 \uc2f8\uc6c0\uc5d0 \ud718\ub9d0\ub838\ub2e4\uac00 \uc678\uad50 \ubb38\uc81c\ub85c \ube44\ud654\ub418\uae30\ub3c4 \ud588\ub2e4.", "paragraph_id": "6521585-3-0", "paragraph_question": "question: \uc11c\uc6d0\ud6c8\uc758 \uc9c1\uc5c5\uc740 \ubb34\uc5c7\uc778\uac00\uc694?, context: 1904\ub144 \uc544\ubc84\uc9c0 \uc548\ud0dc\ud6c8\uacfc \uccad\ub098\ub77c \uc758\uc0ac \uc0ac\uc774\uc5d0 \ubc8c\uc5b4\uc9c4 \uc2f8\uc6c0\uc5d0 \ud718\ub9d0\ub838\ub2e4\uac00 \uc678\uad50 \ubb38\uc81c\ub85c \ube44\ud654\ub418\uae30\ub3c4 \ud588\ub2e4. \uc544\ubc84\uc9c0 \uc548\ud0dc\ud6c8\uc740 1904\ub144 4\uc6d4 20\uc77c \uc548\uc545\uc74d\uc5d0 \uc0ac\ub294 \uccad\ub098\ub77c \uc758\uc0ac \uc11c\uc6d0\ud6c8(\u8212\u5143\u52db)\uacfc \uc2dc\ube44\uac00 \uc788\uc5c8\ub2e4. \uc544\ubc84\uc9c0 \uc548\ud0dc\ud6c8\uc740 \ubcd1\uc744 \uce58\ub8cc\ud558\uae30 \uc704\ud574 \uce5c\uc6b0 \uc774\uc6a9\uc77c(\u674e\u9f8d\u4e00)\uc744 \ub370\ub9ac\uace0 \uc11c\uc6d0\ud6c8\uc744 \ucc3e\uc544\uac14\ub2e4. \uc591\uce21\uc774 \ud544\ub2f4\uc744 \ub098\ub204\ub294 \uc0ac\uc774\uc5d0 \uc548\ud0dc\ud6c8\uce21\uc774 \uc5b4\ub5a4 \uc2e4\uc218\ub97c \ud588\ub294\uc9c0 \uc11c\uc6d0\ud6c8\uc774 \uac11\uc790\uae30 \uc77c\uc5b4\ub098 \uc548\ud0dc\ud6c8\uc758 \uac00\uc2b4\uc744 \ubc1c\ub85c \uac77\uc5b4\ucc3c\ub2e4. \uc774\uc5d0 \uc548\ud0dc\ud6c8\uc740 \uc11c\uc6d0\ud6c8\uc5d0\uac8c \ud654\ud574\ub97c \uccad\ud558\uace0 \ubb3c\ub7ec\ub098\uc654\ub2e4. \uadf8\ub7ec\ub098 \uc774 \uc18c\uc2dd\uc744 \ub4e4\uc740 \uc548\uc911\uadfc\uc774 4\uc6d4 29\uc77c \uc774\uc6a9\ub4f1 10\uc5ec \uba85\uacfc \ud568\uaed8 \ubb34\uae30\ub97c \ub4e4\uace0 \uc57c\ubc24\uc5d0 \uc11c\uc6d0\ud6c8\uc744 \uc7a1\uc544\ub2e4\uac00 \uae38\uac00\uc5d0\uc11c \ubb34\uc218\ud788 \ub09c\ud0c0\ud558\uc5ec \uac70\ub3d9\uc774 \uc5b4\ub835\uac8c \ub9cc\ub4e4\uc5c8\uace0, \uc774\uc5d0 \ub300\ud55c \ubc18\ubc1c\ub85c 1904\ub144 5\uc6d4 2\uc77c \uccad\uad6d\uc778 7~8\uba85\uc774 \uc774\uc6a9\uc77c\uc758 \uc9d1\uc5d0 \ub09c\uc785\ud558\uc5ec \uadf8\ub97c \ub9c8\uad6c \uad6c\ud0c0\ud558\uace0 \uc7a1\uc544\uac00\ub824\uace0 \ud588\ub2e4."} {"question": "\uc2e0\uc804\ud658\uc740 \ud55c \ubc88\uc5d0 \ubca0\ud305\ud55c \uae08\uc561\uc758 \uc561\uc218\ub294?", "paragraph": "\uc2e0\uc815\ud658\uc740 2010\ub144 9\uc6d4 5\uc77c\uc5d0 MBC\uc758 \ucd94\uc11d \ud2b9\uc9d1 \ubc29\uc1a1 \ub179\ud654\uc5d0 \uc0ac\uc804\uc5d0 \ud1b5\ubcf4 \uc5c6\uc774 \ubd88\ucc38\ud588\uace0, 6\uc77c\uc5d0\ub294 KBS\uc758\u300a\uc2a4\ud0c0 \uace8\ub4e0\ubca8 1\ud559\ub144 1\ubc18\u300b, 7\uc77c\uc5d0\ub294 MBC\uc758\u300a\uccad\ucd98 \ubc84\ub77c\uc774\uc5b4\ud2f0 \uaf43\ub2e4\ubc1c\u300b\uc5d0 \uc0ac\uc804 \ud1b5\ubcf4 \uc5c6\uc774 \ubc29\uc1a1 \ub179\ud654\uc5d0 \ubd88\ucc38\ud558\uac8c \ub418\uc790, \uc7a0\uc801\uc124\uc744 \ube44\ub86f\ud55c \uc5ec\ub7ec \ub8e8\uba38\uac00 \uc81c\uae30\ub410\ub2e4. \uc774\uc5d0 \ub300\ud574 \uc18c\uc18d\uc0ac\uc5d0\uc11c\ub294 \uc2e0\uc815\ud658\uc774 \uacfc\ub85c\ub85c \uc778\ud574 \ubc29\uc1a1 \ub179\ud654\uc5d0 \ubd88\ucc38\ud558\uac8c \ub410\ub2e4\uace0 \ud574\uba85\ud588\uc9c0\ub9cc, SBS\ub294 \ud544\ub9ac\ud540\uc5d0\uc11c \ub3c4\ubc15\ube5a \ub54c\ubb38\uc5d0 \uc5b5\ub958\ub418\uc5b4 \uc788\ub294 \uac83\uc73c\ub85c \ubcf4\uc778\ub2e4\uace0 \ubcf4\ub3c4\ud558\uc600\uace0, MBC\ub294 \uc5ec\uad8c\uc744 \ub9e1\uae30\uace0 \ub3c8\uc744 \ube4c\ub838\ub2e4\uac00 \uadc0\uad6d\uc744 \ubabb\ud558\uace0 \uc788\ub2e4\uba70, \uc815\ud669\uc73c\ub85c \ubcfc \ub54c \uc6d0\uc815 \ub3c4\ubc15\uc744 \uac14\ub2e4 \ub3c8\uc744 \uac1a\uc9c0 \ubabb\ud574 \ucd9c\uad6d\ud558\uc9c0 \ubabb\ud588\uc744 \uac00\ub2a5\uc131\uc774 \uc788\ub2e4\uace0 \ubcf4\ub3c4\ud558\uc600\ub2e4. \uc774\ud6c4 \uc2e0\uc815\ud658\uc740 \ud32c \uce74\ud398\ub97c \ud1b5\ud574 \ud544\ub9ac\ud540 \uc138\ubd80\uc5d0\uc11c \uc77c\ud589\ub4e4\uacfc \ud568\uaed8 \uce74\uc9c0\ub178\uc5d0 \ubc29\ubb38\ud55c \uac83\uc740 \uc0ac\uc2e4\uc774\ub098, \uad00\uad11 \ubaa9\uc801\uc73c\ub85c \ubc29\ubb38\ud588\uace0 \uc5ec\ud589 \ub3c4\uc911 \ub385\uae30\uc5f4\uc5d0 \uac78\ub824 \ubcd1\uc6d0\uc5d0\uc11c \uc9c0\ub0b4\uc654\ub2e4\uba70 \ud604\uc7a5 \uc0ac\uc9c4\uacfc \ud568\uaed8 \ub3c4\ubc15\uc124\uc744 \ubd80\uc778\ud588\ub2e4. \ud558\uc9c0\ub9cc \uc77c\ubd80 \ub204\ub9ac\uafbc\ub4e4\uc740 \uacf5\uac1c\ud55c \uc0ac\uc9c4 \uc18d\uc758 \ubaa8\uc2b5\uc740 \uc2ec\uc804\ub3c4 \uac80\uc0ac\ub97c \ud558\ub294 \ubaa8\uc2b5\uc774\uace0, \ub385\uae30\uc5f4\uc73c\ub85c \uc778\ud55c \uc785\uc6d0 \uc2dc\uc810\uc744 \uc815\ud655\ud788 \ubc1d\ud790 \ud544\uc694\uac00 \uc788\ub2e4\uace0 \ud53c\ub825\ud588\ub2e4. \ud544\ub9ac\ud540 \ud604\uc9c0\uc5d0\uc11c \uc2e0\uc815\ud658\uc774 \uc785\uc6d0\ud574\uc788\ub294 \ubcd1\uc6d0\uc744 \ucde8\uc7ac\ud55c \uacb0\uacfc \ub2f4\ub2f9 \uc758\uc0ac\ub294 \"\ub385\uae30\uc5f4 \uc99d\uc0c1\uc774 \uc544\ub2c8\uba70 \uc815\uc0c1\"\uc774\ub77c\uace0 \uc99d\uc5b8\ud558\uc790 \ub124\ud2f0\uc98c\ub4e4\uc740 \uc2e0\uc815\ud658\uc774 \uc0ac\uac74\uc744 \uc870\uc791\ud588\ub2e4\uba70 \ube44\ud310\ud588\ub2e4. \uc2e0\uc815\ud658\uc740 \ub610\ud55c \uac70\uc561\uc758 \ub3c4\ubc15\uc744 \ud558\uc9c0 \uc54a\uc558\ub2e4\uace0 \ubc1d\ud614\uc9c0\ub9cc, \uc911\uad6d\uc778 \uc5c5\uc790\uc758 \ub3c8\uc744 \ube4c\ub824\uc11c \uac70\uc561\uc758 \ub3c4\ubc15\uc744 \ud588\ub2e4\ub294 \uc99d\uc5b8\ub3c4 \ub098\uc624\uba74\uc11c \uc774 \uc5ed\uc2dc \uac70\uc9d3\ud574\uba85\uc774 \uc544\ub2c8\ub0d0\ub294 \ub17c\ub780\uc774 \uc77c\uc5c8\ub2e4. \uc120\uc774\uc790\ub97c \ub5bc\uace0 \ube4c\ub824\uc8fc\ub294 \uc77c\uba85 \uaf41\uc9d3\ub3c8\uc744 \ube4c\ub824 \ud55c \ubc88\uc5d0 1500\ub9cc\uc6d0\uc529 \ubca0\ud305\ud55c \uc0ac\uc2e4\ub3c4 \ub4dc\ub7ec\ub0ac\ub2e4. \uc5b8\ub860\ub4e4\uc758 \ucde8\uc7ac\uacb0\uacfc \uc2e0\uc815\ud658\uc740 \uc790\uc2e0\uc774 \ucd9c\uc5f0\ud55c \ubc29\uc1a1 \ub4f1\uc5d0 \ub300\ud55c \ucd9c\uc5f0\ub8cc\uac00 \uc555\ub958\ub41c \uac83\uc73c\ub85c \uc54c\ub824\uc84c\uc73c\uba70, 1\uc5b5\uc6d0\uc758 \ube5a\uc744 \uac1a\uc9c0 \uc54a\uc558\ub2e4\ub294 \uc774\uc720\ub85c \uace0\uc18c\ub97c \ub2f9\ud558\uae30\ub3c4 \ud588\ub2e4. \uc774 \uacfc\uc815\uc5d0\uc11c \uc2e0\uc815\ud658\uc758 \uc8fc\ub2f9 \ucd9c\uc5f0\ub8cc\uac00 2500\ub9cc\uc6d0\uc774\ub77c\ub294 \uac83\ub3c4 \ubc1d\ud600\uc84c\ub2e4.", "answer": "1500\ub9cc\uc6d0", "sentence": "\uc120\uc774\uc790\ub97c \ub5bc\uace0 \ube4c\ub824\uc8fc\ub294 \uc77c\uba85 \uaf41\uc9d3\ub3c8\uc744 \ube4c\ub824 \ud55c \ubc88\uc5d0 1500\ub9cc\uc6d0 \uc529 \ubca0\ud305\ud55c \uc0ac\uc2e4\ub3c4 \ub4dc\ub7ec\ub0ac\ub2e4.", "paragraph_sentence": "\uc2e0\uc815\ud658\uc740 2010\ub144 9\uc6d4 5\uc77c\uc5d0 MBC\uc758 \ucd94\uc11d \ud2b9\uc9d1 \ubc29\uc1a1 \ub179\ud654\uc5d0 \uc0ac\uc804\uc5d0 \ud1b5\ubcf4 \uc5c6\uc774 \ubd88\ucc38\ud588\uace0, 6\uc77c\uc5d0\ub294 KBS\uc758\u300a\uc2a4\ud0c0 \uace8\ub4e0\ubca8 1\ud559\ub144 1\ubc18\u300b, 7\uc77c\uc5d0\ub294 MBC\uc758\u300a\uccad\ucd98 \ubc84\ub77c\uc774\uc5b4\ud2f0 \uaf43\ub2e4\ubc1c\u300b\uc5d0 \uc0ac\uc804 \ud1b5\ubcf4 \uc5c6\uc774 \ubc29\uc1a1 \ub179\ud654\uc5d0 \ubd88\ucc38\ud558\uac8c \ub418\uc790, \uc7a0\uc801\uc124\uc744 \ube44\ub86f\ud55c \uc5ec\ub7ec \ub8e8\uba38\uac00 \uc81c\uae30\ub410\ub2e4. \uc774\uc5d0 \ub300\ud574 \uc18c\uc18d\uc0ac\uc5d0\uc11c\ub294 \uc2e0\uc815\ud658\uc774 \uacfc\ub85c\ub85c \uc778\ud574 \ubc29\uc1a1 \ub179\ud654\uc5d0 \ubd88\ucc38\ud558\uac8c \ub410\ub2e4\uace0 \ud574\uba85\ud588\uc9c0\ub9cc, SBS\ub294 \ud544\ub9ac\ud540\uc5d0\uc11c \ub3c4\ubc15\ube5a \ub54c\ubb38\uc5d0 \uc5b5\ub958\ub418\uc5b4 \uc788\ub294 \uac83\uc73c\ub85c \ubcf4\uc778\ub2e4\uace0 \ubcf4\ub3c4\ud558\uc600\uace0, MBC\ub294 \uc5ec\uad8c\uc744 \ub9e1\uae30\uace0 \ub3c8\uc744 \ube4c\ub838\ub2e4\uac00 \uadc0\uad6d\uc744 \ubabb\ud558\uace0 \uc788\ub2e4\uba70, \uc815\ud669\uc73c\ub85c \ubcfc \ub54c \uc6d0\uc815 \ub3c4\ubc15\uc744 \uac14\ub2e4 \ub3c8\uc744 \uac1a\uc9c0 \ubabb\ud574 \ucd9c\uad6d\ud558\uc9c0 \ubabb\ud588\uc744 \uac00\ub2a5\uc131\uc774 \uc788\ub2e4\uace0 \ubcf4\ub3c4\ud558\uc600\ub2e4. \uc774\ud6c4 \uc2e0\uc815\ud658\uc740 \ud32c \uce74\ud398\ub97c \ud1b5\ud574 \ud544\ub9ac\ud540 \uc138\ubd80\uc5d0\uc11c \uc77c\ud589\ub4e4\uacfc \ud568\uaed8 \uce74\uc9c0\ub178\uc5d0 \ubc29\ubb38\ud55c \uac83\uc740 \uc0ac\uc2e4\uc774\ub098, \uad00\uad11 \ubaa9\uc801\uc73c\ub85c \ubc29\ubb38\ud588\uace0 \uc5ec\ud589 \ub3c4\uc911 \ub385\uae30\uc5f4\uc5d0 \uac78\ub824 \ubcd1\uc6d0\uc5d0\uc11c \uc9c0\ub0b4\uc654\ub2e4\uba70 \ud604\uc7a5 \uc0ac\uc9c4\uacfc \ud568\uaed8 \ub3c4\ubc15\uc124\uc744 \ubd80\uc778\ud588\ub2e4. \ud558\uc9c0\ub9cc \uc77c\ubd80 \ub204\ub9ac\uafbc\ub4e4\uc740 \uacf5\uac1c\ud55c \uc0ac\uc9c4 \uc18d\uc758 \ubaa8\uc2b5\uc740 \uc2ec\uc804\ub3c4 \uac80\uc0ac\ub97c \ud558\ub294 \ubaa8\uc2b5\uc774\uace0, \ub385\uae30\uc5f4\uc73c\ub85c \uc778\ud55c \uc785\uc6d0 \uc2dc\uc810\uc744 \uc815\ud655\ud788 \ubc1d\ud790 \ud544\uc694\uac00 \uc788\ub2e4\uace0 \ud53c\ub825\ud588\ub2e4. \ud544\ub9ac\ud540 \ud604\uc9c0\uc5d0\uc11c \uc2e0\uc815\ud658\uc774 \uc785\uc6d0\ud574\uc788\ub294 \ubcd1\uc6d0\uc744 \ucde8\uc7ac\ud55c \uacb0\uacfc \ub2f4\ub2f9 \uc758\uc0ac\ub294 \"\ub385\uae30\uc5f4 \uc99d\uc0c1\uc774 \uc544\ub2c8\uba70 \uc815\uc0c1\"\uc774\ub77c\uace0 \uc99d\uc5b8\ud558\uc790 \ub124\ud2f0\uc98c\ub4e4\uc740 \uc2e0\uc815\ud658\uc774 \uc0ac\uac74\uc744 \uc870\uc791\ud588\ub2e4\uba70 \ube44\ud310\ud588\ub2e4. \uc2e0\uc815\ud658\uc740 \ub610\ud55c \uac70\uc561\uc758 \ub3c4\ubc15\uc744 \ud558\uc9c0 \uc54a\uc558\ub2e4\uace0 \ubc1d\ud614\uc9c0\ub9cc, \uc911\uad6d\uc778 \uc5c5\uc790\uc758 \ub3c8\uc744 \ube4c\ub824\uc11c \uac70\uc561\uc758 \ub3c4\ubc15\uc744 \ud588\ub2e4\ub294 \uc99d\uc5b8\ub3c4 \ub098\uc624\uba74\uc11c \uc774 \uc5ed\uc2dc \uac70\uc9d3\ud574\uba85\uc774 \uc544\ub2c8\ub0d0\ub294 \ub17c\ub780\uc774 \uc77c\uc5c8\ub2e4. \uc120\uc774\uc790\ub97c \ub5bc\uace0 \ube4c\ub824\uc8fc\ub294 \uc77c\uba85 \uaf41\uc9d3\ub3c8\uc744 \ube4c\ub824 \ud55c \ubc88\uc5d0 1500\ub9cc\uc6d0 \uc529 \ubca0\ud305\ud55c \uc0ac\uc2e4\ub3c4 \ub4dc\ub7ec\ub0ac\ub2e4. \uc5b8\ub860\ub4e4\uc758 \ucde8\uc7ac\uacb0\uacfc \uc2e0\uc815\ud658\uc740 \uc790\uc2e0\uc774 \ucd9c\uc5f0\ud55c \ubc29\uc1a1 \ub4f1\uc5d0 \ub300\ud55c \ucd9c\uc5f0\ub8cc\uac00 \uc555\ub958\ub41c \uac83\uc73c\ub85c \uc54c\ub824\uc84c\uc73c\uba70, 1\uc5b5\uc6d0\uc758 \ube5a\uc744 \uac1a\uc9c0 \uc54a\uc558\ub2e4\ub294 \uc774\uc720\ub85c \uace0\uc18c\ub97c \ub2f9\ud558\uae30\ub3c4 \ud588\ub2e4. \uc774 \uacfc\uc815\uc5d0\uc11c \uc2e0\uc815\ud658\uc758 \uc8fc\ub2f9 \ucd9c\uc5f0\ub8cc\uac00 2500\ub9cc\uc6d0\uc774\ub77c\ub294 \uac83\ub3c4 \ubc1d\ud600\uc84c\ub2e4.", "paragraph_answer": "\uc2e0\uc815\ud658\uc740 2010\ub144 9\uc6d4 5\uc77c\uc5d0 MBC\uc758 \ucd94\uc11d \ud2b9\uc9d1 \ubc29\uc1a1 \ub179\ud654\uc5d0 \uc0ac\uc804\uc5d0 \ud1b5\ubcf4 \uc5c6\uc774 \ubd88\ucc38\ud588\uace0, 6\uc77c\uc5d0\ub294 KBS\uc758\u300a\uc2a4\ud0c0 \uace8\ub4e0\ubca8 1\ud559\ub144 1\ubc18\u300b, 7\uc77c\uc5d0\ub294 MBC\uc758\u300a\uccad\ucd98 \ubc84\ub77c\uc774\uc5b4\ud2f0 \uaf43\ub2e4\ubc1c\u300b\uc5d0 \uc0ac\uc804 \ud1b5\ubcf4 \uc5c6\uc774 \ubc29\uc1a1 \ub179\ud654\uc5d0 \ubd88\ucc38\ud558\uac8c \ub418\uc790, \uc7a0\uc801\uc124\uc744 \ube44\ub86f\ud55c \uc5ec\ub7ec \ub8e8\uba38\uac00 \uc81c\uae30\ub410\ub2e4. \uc774\uc5d0 \ub300\ud574 \uc18c\uc18d\uc0ac\uc5d0\uc11c\ub294 \uc2e0\uc815\ud658\uc774 \uacfc\ub85c\ub85c \uc778\ud574 \ubc29\uc1a1 \ub179\ud654\uc5d0 \ubd88\ucc38\ud558\uac8c \ub410\ub2e4\uace0 \ud574\uba85\ud588\uc9c0\ub9cc, SBS\ub294 \ud544\ub9ac\ud540\uc5d0\uc11c \ub3c4\ubc15\ube5a \ub54c\ubb38\uc5d0 \uc5b5\ub958\ub418\uc5b4 \uc788\ub294 \uac83\uc73c\ub85c \ubcf4\uc778\ub2e4\uace0 \ubcf4\ub3c4\ud558\uc600\uace0, MBC\ub294 \uc5ec\uad8c\uc744 \ub9e1\uae30\uace0 \ub3c8\uc744 \ube4c\ub838\ub2e4\uac00 \uadc0\uad6d\uc744 \ubabb\ud558\uace0 \uc788\ub2e4\uba70, \uc815\ud669\uc73c\ub85c \ubcfc \ub54c \uc6d0\uc815 \ub3c4\ubc15\uc744 \uac14\ub2e4 \ub3c8\uc744 \uac1a\uc9c0 \ubabb\ud574 \ucd9c\uad6d\ud558\uc9c0 \ubabb\ud588\uc744 \uac00\ub2a5\uc131\uc774 \uc788\ub2e4\uace0 \ubcf4\ub3c4\ud558\uc600\ub2e4. \uc774\ud6c4 \uc2e0\uc815\ud658\uc740 \ud32c \uce74\ud398\ub97c \ud1b5\ud574 \ud544\ub9ac\ud540 \uc138\ubd80\uc5d0\uc11c \uc77c\ud589\ub4e4\uacfc \ud568\uaed8 \uce74\uc9c0\ub178\uc5d0 \ubc29\ubb38\ud55c \uac83\uc740 \uc0ac\uc2e4\uc774\ub098, \uad00\uad11 \ubaa9\uc801\uc73c\ub85c \ubc29\ubb38\ud588\uace0 \uc5ec\ud589 \ub3c4\uc911 \ub385\uae30\uc5f4\uc5d0 \uac78\ub824 \ubcd1\uc6d0\uc5d0\uc11c \uc9c0\ub0b4\uc654\ub2e4\uba70 \ud604\uc7a5 \uc0ac\uc9c4\uacfc \ud568\uaed8 \ub3c4\ubc15\uc124\uc744 \ubd80\uc778\ud588\ub2e4. \ud558\uc9c0\ub9cc \uc77c\ubd80 \ub204\ub9ac\uafbc\ub4e4\uc740 \uacf5\uac1c\ud55c \uc0ac\uc9c4 \uc18d\uc758 \ubaa8\uc2b5\uc740 \uc2ec\uc804\ub3c4 \uac80\uc0ac\ub97c \ud558\ub294 \ubaa8\uc2b5\uc774\uace0, \ub385\uae30\uc5f4\uc73c\ub85c \uc778\ud55c \uc785\uc6d0 \uc2dc\uc810\uc744 \uc815\ud655\ud788 \ubc1d\ud790 \ud544\uc694\uac00 \uc788\ub2e4\uace0 \ud53c\ub825\ud588\ub2e4. \ud544\ub9ac\ud540 \ud604\uc9c0\uc5d0\uc11c \uc2e0\uc815\ud658\uc774 \uc785\uc6d0\ud574\uc788\ub294 \ubcd1\uc6d0\uc744 \ucde8\uc7ac\ud55c \uacb0\uacfc \ub2f4\ub2f9 \uc758\uc0ac\ub294 \"\ub385\uae30\uc5f4 \uc99d\uc0c1\uc774 \uc544\ub2c8\uba70 \uc815\uc0c1\"\uc774\ub77c\uace0 \uc99d\uc5b8\ud558\uc790 \ub124\ud2f0\uc98c\ub4e4\uc740 \uc2e0\uc815\ud658\uc774 \uc0ac\uac74\uc744 \uc870\uc791\ud588\ub2e4\uba70 \ube44\ud310\ud588\ub2e4. \uc2e0\uc815\ud658\uc740 \ub610\ud55c \uac70\uc561\uc758 \ub3c4\ubc15\uc744 \ud558\uc9c0 \uc54a\uc558\ub2e4\uace0 \ubc1d\ud614\uc9c0\ub9cc, \uc911\uad6d\uc778 \uc5c5\uc790\uc758 \ub3c8\uc744 \ube4c\ub824\uc11c \uac70\uc561\uc758 \ub3c4\ubc15\uc744 \ud588\ub2e4\ub294 \uc99d\uc5b8\ub3c4 \ub098\uc624\uba74\uc11c \uc774 \uc5ed\uc2dc \uac70\uc9d3\ud574\uba85\uc774 \uc544\ub2c8\ub0d0\ub294 \ub17c\ub780\uc774 \uc77c\uc5c8\ub2e4. \uc120\uc774\uc790\ub97c \ub5bc\uace0 \ube4c\ub824\uc8fc\ub294 \uc77c\uba85 \uaf41\uc9d3\ub3c8\uc744 \ube4c\ub824 \ud55c \ubc88\uc5d0 1500\ub9cc\uc6d0 \uc529 \ubca0\ud305\ud55c \uc0ac\uc2e4\ub3c4 \ub4dc\ub7ec\ub0ac\ub2e4. \uc5b8\ub860\ub4e4\uc758 \ucde8\uc7ac\uacb0\uacfc \uc2e0\uc815\ud658\uc740 \uc790\uc2e0\uc774 \ucd9c\uc5f0\ud55c \ubc29\uc1a1 \ub4f1\uc5d0 \ub300\ud55c \ucd9c\uc5f0\ub8cc\uac00 \uc555\ub958\ub41c \uac83\uc73c\ub85c \uc54c\ub824\uc84c\uc73c\uba70, 1\uc5b5\uc6d0\uc758 \ube5a\uc744 \uac1a\uc9c0 \uc54a\uc558\ub2e4\ub294 \uc774\uc720\ub85c \uace0\uc18c\ub97c \ub2f9\ud558\uae30\ub3c4 \ud588\ub2e4. \uc774 \uacfc\uc815\uc5d0\uc11c \uc2e0\uc815\ud658\uc758 \uc8fc\ub2f9 \ucd9c\uc5f0\ub8cc\uac00 2500\ub9cc\uc6d0\uc774\ub77c\ub294 \uac83\ub3c4 \ubc1d\ud600\uc84c\ub2e4.", "sentence_answer": "\uc120\uc774\uc790\ub97c \ub5bc\uace0 \ube4c\ub824\uc8fc\ub294 \uc77c\uba85 \uaf41\uc9d3\ub3c8\uc744 \ube4c\ub824 \ud55c \ubc88\uc5d0 1500\ub9cc\uc6d0 \uc529 \ubca0\ud305\ud55c \uc0ac\uc2e4\ub3c4 \ub4dc\ub7ec\ub0ac\ub2e4.", "paragraph_id": "6584943-0-2", "paragraph_question": "question: \uc2e0\uc804\ud658\uc740 \ud55c \ubc88\uc5d0 \ubca0\ud305\ud55c \uae08\uc561\uc758 \uc561\uc218\ub294?, context: \uc2e0\uc815\ud658\uc740 2010\ub144 9\uc6d4 5\uc77c\uc5d0 MBC\uc758 \ucd94\uc11d \ud2b9\uc9d1 \ubc29\uc1a1 \ub179\ud654\uc5d0 \uc0ac\uc804\uc5d0 \ud1b5\ubcf4 \uc5c6\uc774 \ubd88\ucc38\ud588\uace0, 6\uc77c\uc5d0\ub294 KBS\uc758\u300a\uc2a4\ud0c0 \uace8\ub4e0\ubca8 1\ud559\ub144 1\ubc18\u300b, 7\uc77c\uc5d0\ub294 MBC\uc758\u300a\uccad\ucd98 \ubc84\ub77c\uc774\uc5b4\ud2f0 \uaf43\ub2e4\ubc1c\u300b\uc5d0 \uc0ac\uc804 \ud1b5\ubcf4 \uc5c6\uc774 \ubc29\uc1a1 \ub179\ud654\uc5d0 \ubd88\ucc38\ud558\uac8c \ub418\uc790, \uc7a0\uc801\uc124\uc744 \ube44\ub86f\ud55c \uc5ec\ub7ec \ub8e8\uba38\uac00 \uc81c\uae30\ub410\ub2e4. \uc774\uc5d0 \ub300\ud574 \uc18c\uc18d\uc0ac\uc5d0\uc11c\ub294 \uc2e0\uc815\ud658\uc774 \uacfc\ub85c\ub85c \uc778\ud574 \ubc29\uc1a1 \ub179\ud654\uc5d0 \ubd88\ucc38\ud558\uac8c \ub410\ub2e4\uace0 \ud574\uba85\ud588\uc9c0\ub9cc, SBS\ub294 \ud544\ub9ac\ud540\uc5d0\uc11c \ub3c4\ubc15\ube5a \ub54c\ubb38\uc5d0 \uc5b5\ub958\ub418\uc5b4 \uc788\ub294 \uac83\uc73c\ub85c \ubcf4\uc778\ub2e4\uace0 \ubcf4\ub3c4\ud558\uc600\uace0, MBC\ub294 \uc5ec\uad8c\uc744 \ub9e1\uae30\uace0 \ub3c8\uc744 \ube4c\ub838\ub2e4\uac00 \uadc0\uad6d\uc744 \ubabb\ud558\uace0 \uc788\ub2e4\uba70, \uc815\ud669\uc73c\ub85c \ubcfc \ub54c \uc6d0\uc815 \ub3c4\ubc15\uc744 \uac14\ub2e4 \ub3c8\uc744 \uac1a\uc9c0 \ubabb\ud574 \ucd9c\uad6d\ud558\uc9c0 \ubabb\ud588\uc744 \uac00\ub2a5\uc131\uc774 \uc788\ub2e4\uace0 \ubcf4\ub3c4\ud558\uc600\ub2e4. \uc774\ud6c4 \uc2e0\uc815\ud658\uc740 \ud32c \uce74\ud398\ub97c \ud1b5\ud574 \ud544\ub9ac\ud540 \uc138\ubd80\uc5d0\uc11c \uc77c\ud589\ub4e4\uacfc \ud568\uaed8 \uce74\uc9c0\ub178\uc5d0 \ubc29\ubb38\ud55c \uac83\uc740 \uc0ac\uc2e4\uc774\ub098, \uad00\uad11 \ubaa9\uc801\uc73c\ub85c \ubc29\ubb38\ud588\uace0 \uc5ec\ud589 \ub3c4\uc911 \ub385\uae30\uc5f4\uc5d0 \uac78\ub824 \ubcd1\uc6d0\uc5d0\uc11c \uc9c0\ub0b4\uc654\ub2e4\uba70 \ud604\uc7a5 \uc0ac\uc9c4\uacfc \ud568\uaed8 \ub3c4\ubc15\uc124\uc744 \ubd80\uc778\ud588\ub2e4. \ud558\uc9c0\ub9cc \uc77c\ubd80 \ub204\ub9ac\uafbc\ub4e4\uc740 \uacf5\uac1c\ud55c \uc0ac\uc9c4 \uc18d\uc758 \ubaa8\uc2b5\uc740 \uc2ec\uc804\ub3c4 \uac80\uc0ac\ub97c \ud558\ub294 \ubaa8\uc2b5\uc774\uace0, \ub385\uae30\uc5f4\uc73c\ub85c \uc778\ud55c \uc785\uc6d0 \uc2dc\uc810\uc744 \uc815\ud655\ud788 \ubc1d\ud790 \ud544\uc694\uac00 \uc788\ub2e4\uace0 \ud53c\ub825\ud588\ub2e4. \ud544\ub9ac\ud540 \ud604\uc9c0\uc5d0\uc11c \uc2e0\uc815\ud658\uc774 \uc785\uc6d0\ud574\uc788\ub294 \ubcd1\uc6d0\uc744 \ucde8\uc7ac\ud55c \uacb0\uacfc \ub2f4\ub2f9 \uc758\uc0ac\ub294 \"\ub385\uae30\uc5f4 \uc99d\uc0c1\uc774 \uc544\ub2c8\uba70 \uc815\uc0c1\"\uc774\ub77c\uace0 \uc99d\uc5b8\ud558\uc790 \ub124\ud2f0\uc98c\ub4e4\uc740 \uc2e0\uc815\ud658\uc774 \uc0ac\uac74\uc744 \uc870\uc791\ud588\ub2e4\uba70 \ube44\ud310\ud588\ub2e4. \uc2e0\uc815\ud658\uc740 \ub610\ud55c \uac70\uc561\uc758 \ub3c4\ubc15\uc744 \ud558\uc9c0 \uc54a\uc558\ub2e4\uace0 \ubc1d\ud614\uc9c0\ub9cc, \uc911\uad6d\uc778 \uc5c5\uc790\uc758 \ub3c8\uc744 \ube4c\ub824\uc11c \uac70\uc561\uc758 \ub3c4\ubc15\uc744 \ud588\ub2e4\ub294 \uc99d\uc5b8\ub3c4 \ub098\uc624\uba74\uc11c \uc774 \uc5ed\uc2dc \uac70\uc9d3\ud574\uba85\uc774 \uc544\ub2c8\ub0d0\ub294 \ub17c\ub780\uc774 \uc77c\uc5c8\ub2e4. \uc120\uc774\uc790\ub97c \ub5bc\uace0 \ube4c\ub824\uc8fc\ub294 \uc77c\uba85 \uaf41\uc9d3\ub3c8\uc744 \ube4c\ub824 \ud55c \ubc88\uc5d0 1500\ub9cc\uc6d0\uc529 \ubca0\ud305\ud55c \uc0ac\uc2e4\ub3c4 \ub4dc\ub7ec\ub0ac\ub2e4. \uc5b8\ub860\ub4e4\uc758 \ucde8\uc7ac\uacb0\uacfc \uc2e0\uc815\ud658\uc740 \uc790\uc2e0\uc774 \ucd9c\uc5f0\ud55c \ubc29\uc1a1 \ub4f1\uc5d0 \ub300\ud55c \ucd9c\uc5f0\ub8cc\uac00 \uc555\ub958\ub41c \uac83\uc73c\ub85c \uc54c\ub824\uc84c\uc73c\uba70, 1\uc5b5\uc6d0\uc758 \ube5a\uc744 \uac1a\uc9c0 \uc54a\uc558\ub2e4\ub294 \uc774\uc720\ub85c \uace0\uc18c\ub97c \ub2f9\ud558\uae30\ub3c4 \ud588\ub2e4. \uc774 \uacfc\uc815\uc5d0\uc11c \uc2e0\uc815\ud658\uc758 \uc8fc\ub2f9 \ucd9c\uc5f0\ub8cc\uac00 2500\ub9cc\uc6d0\uc774\ub77c\ub294 \uac83\ub3c4 \ubc1d\ud600\uc84c\ub2e4."} {"question": "\uc774\uc2dc\ubbf8\ub124 \uac00\uc988\ud788\ucf54\uac00 1981\ub144 3\uc810 \ud648\ub828\uc744 \uce5c \uc2dc\ubc94 \uacbd\uae30\uc758 \uc0c1\ub300\ud300\uc740?", "paragraph": "\ud504\ub85c 2\ub144\uc9f8\uc778 1980\ub144 2\uc6d4\uc5d0 \ud1f4\uc6d0\ud55c \ub4a4 \uc6d0\uc0c1\ubcf5\uadc0\ub418\uc9c0 \uc54a\ub294 \ubc18\ub2ec\ud310\uc744 \uc9c0\ud0f1\ud558\uae30 \uc704\ud574 \uc8fc\ubcc0 \uadfc\uc721\uc744 \uacc4\uc18d \ub2e8\ub828\ud588\ub2e4. \uc6e8\uc2a4\ud134 \ub9ac\uadf8\uc5d0\uc11c\ub294 \ubb34\ub98e\uc5d0 \ubd80\ub2f4\uc744 \uc8fc\uc9c0 \uc54a\ub3c4\ub85d \ub300\ud0c0\ub97c \uc911\uc2ec\uc73c\ub85c \uae30\uc6a9\ub418\uba74\uc11c 82\ud0c0\uc218 21\uc548\ud0c0\ub85c \ud0c0\uc728\uc740 2\ud560 5\ud47c 6\ub9ac\ub85c \ub0ae\uc544\uc84c\uc9c0\ub9cc \ud648\ub7f0\uc740 4\uac1c\ub85c \uc99d\uac00\ud574 \uc7a5\uac70\ub9ac\ud3ec\uc758 \ud3b8\ub9b0\uc744 \uc5ff\ubcf4\uac8c \ud588\ub2e4. \uc774\ub4ec\ud574 1981\ub144\uc5d0\ub294 \ud788\ub85c\uc2dc\ub9c8\uc640\uc758 \uc2dc\ubc94 \uacbd\uae30\uc5d0\uc11c 7\uc810 \ucc28\ub85c \ub4a4\uc9c4 9\ud68c\ub9d0\uc5d0 \ub300\ud0c0\ub85c\uc11c \ucd9c\uc804\ud574 \uc774 \ud0c0\uc11d\uc5d0\uc11c \ubcfc\ub137\uc744 \uc120\ud0dd\ud55c \uac83\uc744 \uae30\uc810\uc73c\ub85c \ud0c0\uc21c\uc774 \ud55c \ubc14\ud034 \ub3c4\ub294 \ub9f9\uacf5\uaca9\uc774 \uc2dc\uc791\ub410\uace0 9\ud68c\uc5d0 \ub2e4\uc2dc \ub3cc\uc544\uc628 \ud0c0\uc11d\uc5d0\uc11c \ub77c\uc774\ub108\uc131 \ub05d\ub0b4\uae30 3\uc810 \ud648\ub7f0\uc744 \ub54c\ub824\ub0c8\ub2e4. \uc774\uac83\uc744 \uacc4\uae30\ub85c \uadf8 \ud574\ubd80\ud130 \uac10\ub3c5\uc73c\ub85c \ucde8\uc784\ud55c \uc6b0\uc5d0\ub2e4 \ub3c4\uc2dc\ud558\ub8e8\ub85c\ubd80\ud130 \ud380\uce58\ub825\uc744 \ud3c9\uac00\ubc1b\uc544 \ub098\uce74\uc790\uc640 \uc2e0\uc9c0\ub098 \uc0ac\uc0ac\ubaa8\ud1a0 \uc2e0\uc9c0, \uac00\ud0c0\uc624\uce74 \uc2e0\ub178\uc2a4\ucf00, \uac00\uc640\ubb34\ub77c \uac90\uc774\uce58\ub85c\uc5d0 \uc774\uc5b4 \uc774\ub840\uc801\uc73c\ub85c 5\ubc88\uc9f8 \ud3ec\uc218\ub85c\uc11c \uac1c\ub9c9\uc804 1\uad70 \uc5d4\ud2b8\ub9ac\uc5d0 \uc9c4\uc785\ud588\ub2e4. \uac19\uc740 \ud574 36\uacbd\uae30\uc5d0 \ucd9c\uc804\ud574 \ud0c0\uc728 2\ud560 1\ud47c 4\ub9ac\ub97c \uae30\ub85d\ud588\uace0 \ud3ec\uc218\ub85c\uc11c\ub294 15\uacbd\uae30\uc5d0 \ucd9c\uc804\ud588\ub2e4.", "answer": "\ud788\ub85c\uc2dc\ub9c8", "sentence": "\uc774\ub4ec\ud574 1981\ub144\uc5d0\ub294 \ud788\ub85c\uc2dc\ub9c8 \uc640\uc758 \uc2dc\ubc94 \uacbd\uae30\uc5d0\uc11c 7\uc810 \ucc28\ub85c \ub4a4\uc9c4 9\ud68c\ub9d0\uc5d0 \ub300\ud0c0\ub85c\uc11c \ucd9c\uc804\ud574 \uc774 \ud0c0\uc11d\uc5d0\uc11c \ubcfc\ub137\uc744 \uc120\ud0dd\ud55c \uac83\uc744 \uae30\uc810\uc73c\ub85c \ud0c0\uc21c\uc774 \ud55c \ubc14\ud034 \ub3c4\ub294 \ub9f9\uacf5\uaca9\uc774 \uc2dc\uc791\ub410\uace0 9\ud68c\uc5d0 \ub2e4\uc2dc \ub3cc\uc544\uc628 \ud0c0\uc11d\uc5d0\uc11c \ub77c\uc774\ub108\uc131 \ub05d\ub0b4\uae30 3\uc810 \ud648\ub7f0\uc744 \ub54c\ub824\ub0c8\ub2e4.", "paragraph_sentence": "\ud504\ub85c 2\ub144\uc9f8\uc778 1980\ub144 2\uc6d4\uc5d0 \ud1f4\uc6d0\ud55c \ub4a4 \uc6d0\uc0c1\ubcf5\uadc0\ub418\uc9c0 \uc54a\ub294 \ubc18\ub2ec\ud310\uc744 \uc9c0\ud0f1\ud558\uae30 \uc704\ud574 \uc8fc\ubcc0 \uadfc\uc721\uc744 \uacc4\uc18d \ub2e8\ub828\ud588\ub2e4. \uc6e8\uc2a4\ud134 \ub9ac\uadf8\uc5d0\uc11c\ub294 \ubb34\ub98e\uc5d0 \ubd80\ub2f4\uc744 \uc8fc\uc9c0 \uc54a\ub3c4\ub85d \ub300\ud0c0\ub97c \uc911\uc2ec\uc73c\ub85c \uae30\uc6a9\ub418\uba74\uc11c 82\ud0c0\uc218 21\uc548\ud0c0\ub85c \ud0c0\uc728\uc740 2\ud560 5\ud47c 6\ub9ac\ub85c \ub0ae\uc544\uc84c\uc9c0\ub9cc \ud648\ub7f0\uc740 4\uac1c\ub85c \uc99d\uac00\ud574 \uc7a5\uac70\ub9ac\ud3ec\uc758 \ud3b8\ub9b0\uc744 \uc5ff\ubcf4\uac8c \ud588\ub2e4. \uc774\ub4ec\ud574 1981\ub144\uc5d0\ub294 \ud788\ub85c\uc2dc\ub9c8 \uc640\uc758 \uc2dc\ubc94 \uacbd\uae30\uc5d0\uc11c 7\uc810 \ucc28\ub85c \ub4a4\uc9c4 9\ud68c\ub9d0\uc5d0 \ub300\ud0c0\ub85c\uc11c \ucd9c\uc804\ud574 \uc774 \ud0c0\uc11d\uc5d0\uc11c \ubcfc\ub137\uc744 \uc120\ud0dd\ud55c \uac83\uc744 \uae30\uc810\uc73c\ub85c \ud0c0\uc21c\uc774 \ud55c \ubc14\ud034 \ub3c4\ub294 \ub9f9\uacf5\uaca9\uc774 \uc2dc\uc791\ub410\uace0 9\ud68c\uc5d0 \ub2e4\uc2dc \ub3cc\uc544\uc628 \ud0c0\uc11d\uc5d0\uc11c \ub77c\uc774\ub108\uc131 \ub05d\ub0b4\uae30 3\uc810 \ud648\ub7f0\uc744 \ub54c\ub824\ub0c8\ub2e4. \uc774\uac83\uc744 \uacc4\uae30\ub85c \uadf8 \ud574\ubd80\ud130 \uac10\ub3c5\uc73c\ub85c \ucde8\uc784\ud55c \uc6b0\uc5d0\ub2e4 \ub3c4\uc2dc\ud558\ub8e8\ub85c\ubd80\ud130 \ud380\uce58\ub825\uc744 \ud3c9\uac00\ubc1b\uc544 \ub098\uce74\uc790\uc640 \uc2e0\uc9c0\ub098 \uc0ac\uc0ac\ubaa8\ud1a0 \uc2e0\uc9c0, \uac00\ud0c0\uc624\uce74 \uc2e0\ub178\uc2a4\ucf00, \uac00\uc640\ubb34\ub77c \uac90\uc774\uce58\ub85c\uc5d0 \uc774\uc5b4 \uc774\ub840\uc801\uc73c\ub85c 5\ubc88\uc9f8 \ud3ec\uc218\ub85c\uc11c \uac1c\ub9c9\uc804 1\uad70 \uc5d4\ud2b8\ub9ac\uc5d0 \uc9c4\uc785\ud588\ub2e4. \uac19\uc740 \ud574 36\uacbd\uae30\uc5d0 \ucd9c\uc804\ud574 \ud0c0\uc728 2\ud560 1\ud47c 4\ub9ac\ub97c \uae30\ub85d\ud588\uace0 \ud3ec\uc218\ub85c\uc11c\ub294 15\uacbd\uae30\uc5d0 \ucd9c\uc804\ud588\ub2e4.", "paragraph_answer": "\ud504\ub85c 2\ub144\uc9f8\uc778 1980\ub144 2\uc6d4\uc5d0 \ud1f4\uc6d0\ud55c \ub4a4 \uc6d0\uc0c1\ubcf5\uadc0\ub418\uc9c0 \uc54a\ub294 \ubc18\ub2ec\ud310\uc744 \uc9c0\ud0f1\ud558\uae30 \uc704\ud574 \uc8fc\ubcc0 \uadfc\uc721\uc744 \uacc4\uc18d \ub2e8\ub828\ud588\ub2e4. \uc6e8\uc2a4\ud134 \ub9ac\uadf8\uc5d0\uc11c\ub294 \ubb34\ub98e\uc5d0 \ubd80\ub2f4\uc744 \uc8fc\uc9c0 \uc54a\ub3c4\ub85d \ub300\ud0c0\ub97c \uc911\uc2ec\uc73c\ub85c \uae30\uc6a9\ub418\uba74\uc11c 82\ud0c0\uc218 21\uc548\ud0c0\ub85c \ud0c0\uc728\uc740 2\ud560 5\ud47c 6\ub9ac\ub85c \ub0ae\uc544\uc84c\uc9c0\ub9cc \ud648\ub7f0\uc740 4\uac1c\ub85c \uc99d\uac00\ud574 \uc7a5\uac70\ub9ac\ud3ec\uc758 \ud3b8\ub9b0\uc744 \uc5ff\ubcf4\uac8c \ud588\ub2e4. \uc774\ub4ec\ud574 1981\ub144\uc5d0\ub294 \ud788\ub85c\uc2dc\ub9c8 \uc640\uc758 \uc2dc\ubc94 \uacbd\uae30\uc5d0\uc11c 7\uc810 \ucc28\ub85c \ub4a4\uc9c4 9\ud68c\ub9d0\uc5d0 \ub300\ud0c0\ub85c\uc11c \ucd9c\uc804\ud574 \uc774 \ud0c0\uc11d\uc5d0\uc11c \ubcfc\ub137\uc744 \uc120\ud0dd\ud55c \uac83\uc744 \uae30\uc810\uc73c\ub85c \ud0c0\uc21c\uc774 \ud55c \ubc14\ud034 \ub3c4\ub294 \ub9f9\uacf5\uaca9\uc774 \uc2dc\uc791\ub410\uace0 9\ud68c\uc5d0 \ub2e4\uc2dc \ub3cc\uc544\uc628 \ud0c0\uc11d\uc5d0\uc11c \ub77c\uc774\ub108\uc131 \ub05d\ub0b4\uae30 3\uc810 \ud648\ub7f0\uc744 \ub54c\ub824\ub0c8\ub2e4. \uc774\uac83\uc744 \uacc4\uae30\ub85c \uadf8 \ud574\ubd80\ud130 \uac10\ub3c5\uc73c\ub85c \ucde8\uc784\ud55c \uc6b0\uc5d0\ub2e4 \ub3c4\uc2dc\ud558\ub8e8\ub85c\ubd80\ud130 \ud380\uce58\ub825\uc744 \ud3c9\uac00\ubc1b\uc544 \ub098\uce74\uc790\uc640 \uc2e0\uc9c0\ub098 \uc0ac\uc0ac\ubaa8\ud1a0 \uc2e0\uc9c0, \uac00\ud0c0\uc624\uce74 \uc2e0\ub178\uc2a4\ucf00, \uac00\uc640\ubb34\ub77c \uac90\uc774\uce58\ub85c\uc5d0 \uc774\uc5b4 \uc774\ub840\uc801\uc73c\ub85c 5\ubc88\uc9f8 \ud3ec\uc218\ub85c\uc11c \uac1c\ub9c9\uc804 1\uad70 \uc5d4\ud2b8\ub9ac\uc5d0 \uc9c4\uc785\ud588\ub2e4. \uac19\uc740 \ud574 36\uacbd\uae30\uc5d0 \ucd9c\uc804\ud574 \ud0c0\uc728 2\ud560 1\ud47c 4\ub9ac\ub97c \uae30\ub85d\ud588\uace0 \ud3ec\uc218\ub85c\uc11c\ub294 15\uacbd\uae30\uc5d0 \ucd9c\uc804\ud588\ub2e4.", "sentence_answer": "\uc774\ub4ec\ud574 1981\ub144\uc5d0\ub294 \ud788\ub85c\uc2dc\ub9c8 \uc640\uc758 \uc2dc\ubc94 \uacbd\uae30\uc5d0\uc11c 7\uc810 \ucc28\ub85c \ub4a4\uc9c4 9\ud68c\ub9d0\uc5d0 \ub300\ud0c0\ub85c\uc11c \ucd9c\uc804\ud574 \uc774 \ud0c0\uc11d\uc5d0\uc11c \ubcfc\ub137\uc744 \uc120\ud0dd\ud55c \uac83\uc744 \uae30\uc810\uc73c\ub85c \ud0c0\uc21c\uc774 \ud55c \ubc14\ud034 \ub3c4\ub294 \ub9f9\uacf5\uaca9\uc774 \uc2dc\uc791\ub410\uace0 9\ud68c\uc5d0 \ub2e4\uc2dc \ub3cc\uc544\uc628 \ud0c0\uc11d\uc5d0\uc11c \ub77c\uc774\ub108\uc131 \ub05d\ub0b4\uae30 3\uc810 \ud648\ub7f0\uc744 \ub54c\ub824\ub0c8\ub2e4.", "paragraph_id": "6545783-1-0", "paragraph_question": "question: \uc774\uc2dc\ubbf8\ub124 \uac00\uc988\ud788\ucf54\uac00 1981\ub144 3\uc810 \ud648\ub828\uc744 \uce5c \uc2dc\ubc94 \uacbd\uae30\uc758 \uc0c1\ub300\ud300\uc740?, context: \ud504\ub85c 2\ub144\uc9f8\uc778 1980\ub144 2\uc6d4\uc5d0 \ud1f4\uc6d0\ud55c \ub4a4 \uc6d0\uc0c1\ubcf5\uadc0\ub418\uc9c0 \uc54a\ub294 \ubc18\ub2ec\ud310\uc744 \uc9c0\ud0f1\ud558\uae30 \uc704\ud574 \uc8fc\ubcc0 \uadfc\uc721\uc744 \uacc4\uc18d \ub2e8\ub828\ud588\ub2e4. \uc6e8\uc2a4\ud134 \ub9ac\uadf8\uc5d0\uc11c\ub294 \ubb34\ub98e\uc5d0 \ubd80\ub2f4\uc744 \uc8fc\uc9c0 \uc54a\ub3c4\ub85d \ub300\ud0c0\ub97c \uc911\uc2ec\uc73c\ub85c \uae30\uc6a9\ub418\uba74\uc11c 82\ud0c0\uc218 21\uc548\ud0c0\ub85c \ud0c0\uc728\uc740 2\ud560 5\ud47c 6\ub9ac\ub85c \ub0ae\uc544\uc84c\uc9c0\ub9cc \ud648\ub7f0\uc740 4\uac1c\ub85c \uc99d\uac00\ud574 \uc7a5\uac70\ub9ac\ud3ec\uc758 \ud3b8\ub9b0\uc744 \uc5ff\ubcf4\uac8c \ud588\ub2e4. \uc774\ub4ec\ud574 1981\ub144\uc5d0\ub294 \ud788\ub85c\uc2dc\ub9c8\uc640\uc758 \uc2dc\ubc94 \uacbd\uae30\uc5d0\uc11c 7\uc810 \ucc28\ub85c \ub4a4\uc9c4 9\ud68c\ub9d0\uc5d0 \ub300\ud0c0\ub85c\uc11c \ucd9c\uc804\ud574 \uc774 \ud0c0\uc11d\uc5d0\uc11c \ubcfc\ub137\uc744 \uc120\ud0dd\ud55c \uac83\uc744 \uae30\uc810\uc73c\ub85c \ud0c0\uc21c\uc774 \ud55c \ubc14\ud034 \ub3c4\ub294 \ub9f9\uacf5\uaca9\uc774 \uc2dc\uc791\ub410\uace0 9\ud68c\uc5d0 \ub2e4\uc2dc \ub3cc\uc544\uc628 \ud0c0\uc11d\uc5d0\uc11c \ub77c\uc774\ub108\uc131 \ub05d\ub0b4\uae30 3\uc810 \ud648\ub7f0\uc744 \ub54c\ub824\ub0c8\ub2e4. \uc774\uac83\uc744 \uacc4\uae30\ub85c \uadf8 \ud574\ubd80\ud130 \uac10\ub3c5\uc73c\ub85c \ucde8\uc784\ud55c \uc6b0\uc5d0\ub2e4 \ub3c4\uc2dc\ud558\ub8e8\ub85c\ubd80\ud130 \ud380\uce58\ub825\uc744 \ud3c9\uac00\ubc1b\uc544 \ub098\uce74\uc790\uc640 \uc2e0\uc9c0\ub098 \uc0ac\uc0ac\ubaa8\ud1a0 \uc2e0\uc9c0, \uac00\ud0c0\uc624\uce74 \uc2e0\ub178\uc2a4\ucf00, \uac00\uc640\ubb34\ub77c \uac90\uc774\uce58\ub85c\uc5d0 \uc774\uc5b4 \uc774\ub840\uc801\uc73c\ub85c 5\ubc88\uc9f8 \ud3ec\uc218\ub85c\uc11c \uac1c\ub9c9\uc804 1\uad70 \uc5d4\ud2b8\ub9ac\uc5d0 \uc9c4\uc785\ud588\ub2e4. \uac19\uc740 \ud574 36\uacbd\uae30\uc5d0 \ucd9c\uc804\ud574 \ud0c0\uc728 2\ud560 1\ud47c 4\ub9ac\ub97c \uae30\ub85d\ud588\uace0 \ud3ec\uc218\ub85c\uc11c\ub294 15\uacbd\uae30\uc5d0 \ucd9c\uc804\ud588\ub2e4."} {"question": "\uc720\uc2dc\ubbfc\uc758 \uc544\ubc84\uc9c0\ub294?", "paragraph": "1959\ub144 7\uc6d4 28\uc77c\uc5d0 \uacbd\uc0c1\ubd81\ub3c4 \uacbd\uc8fc\uc5d0\uc11c \uc544\ubc84\uc9c0 \ub958\ud0dc\uc6b0\uc640 \uc5b4\uba38\ub2c8 \uc11c\ub3d9\ud544 \uc0ac\uc774\uc5d0\uc11c 4\ub0a8 2\ub140 \uc911 4\ub0a8\uc73c\ub85c \ud0dc\uc5b4\ub0ac\ub2e4. 13\ub300\uc870\ub294 \uc784\uc9c4\uc65c\ub780\uc744 \uadf9\ubcf5\ud558\ub294 \ub370 \ud798\uc4f4 \uc911\ud765\uc758 \uba85\uc7ac\uc0c1 \uc758\uc815\ubd80 \uc601\uc758\uc815 \uc11c\uc560 \ub958\uc131\ub8e1\uc774\ub2e4. 12\ub300\uc870\ub294 \uc7a5\uc218\ucc30\ubc29\uc744 \uc9c0\ub0b8 \ub958\uc5ec\uc774\uba70, 11\ub300\uc870\ub294 \uc870\uc120\uc911\uae30\uc758 \ub300\ud559\uc790\ub85c \uc548\uae30 \ucc30\ubc29\uc744 \uc9c0\ub0b8 \uc878\uc7ac \ub958\uc6d0\uc9c0\uc774\uace0, 10\ub300\uc870\ub294 \ubd80\ud638\uad70\uc744 \uc9c0\ub0b8 \ub958\ub9cc\ud558, 9\ub300\uc870\ub294 \uc138\uc790 \uc2dc\uac15\uc6d0 \uc790\uc758\ub97c \uc9c0\ub0b8 \uc8fc\uc77c\uc7ac \ub958\ud6c4\uc7a5\uc774\ub2e4. \uc870\ubd80 \ub300\uc5d0 \uc774\ub974\ub7ec \uc548\ub3d9\uc5d0\uc11c \ucc98\uac00\uac00 \uc788\ub294 \uacbd\uc8fc\ub85c \uc774\uac70\ud558\uc600\ub2e4. \uc544\ubc84\uc9c0 \ub958\ud0dc\uc6b0\ub294 \uacbd\uc8fc\uc5ec\uc790\uace0\ub4f1\ud559\uad50\uc758 \uc5ed\uc0ac \uad50\uc0ac\uc600\ub2e4.\ud070\uc544\ubc84\uc9c0 \ub958\uc11d\uc6b0\ub294 \uacf5\ubb34\uc6d0\uc73c\ub85c \ub0b4\ub0a8\uba74 \uba74\uc7a5\uc744 \uc9c0\ub0b4\uae30\ub3c4 \ud588\ub2e4.\ub958\ud0dc\uc6b0\uac00 \ub9cc\uc8fc \uc18c\ud559\uad50\uc758 \uad50\uc9c1\uc6d0\uc73c\ub85c \uadfc\ubb34\ud55c \uc77c\uc740 \ud6c4\uc5d0 \uc758\ud639\uc73c\ub85c \uc81c\uae30\ub418\uc5c8\ub294\ub370, \uc774\ub97c \ub450\uace0 \uc720\uc2dc\ubbfc\uc740 \"\ubc31\ubd80\uac00 \uba74\uc7a5\uc744 \ud55c \uac83\uc740 \ub9de\uc9c0\ub9cc \uc120\uce5c\uc740 \uc77c\uc81c \uce58\ud558 \ub54c \uad50\uc0ac\ub97c \ud558\uc9c0 \uc54a\uc558\ub2e4. \ucc98\uc74c \ub4e3\ub294 \uc598\uae30\ub77c \uc9d1\uc548 \uc5b4\ub974\uc2e0\ub4e4\uaed8 \ud655\uc778\ud574\ubcf8 \uacb0\uacfc \uc120\uce5c\uc740 \ud574\ubc29 \uc9c1\ud6c4 \ubbf8 \uad70\uc815\uc774 \uad50\uc0ac \uc694\uc6d0\uc744 \uacf5\ucc44\ud588\uc744 \ub54c \ub3d9\uc591\uc0ac \ubd84\uc57c\uc5d0 \uc751\uc2dc\ud574 \ud569\uaca9\ud588\uace0 6\uac1c\uc6d4 \uc5f0\uc218 \ud6c4 \uacbd\uc8fc\uc5ec\uc790\uc911\ud559\uad50\uc5d0 \ubd80\uc784\ud588\ub2e4.\"\uace0 \ub9d0\ud558\uba70 \uad00\ub828 \uc0ac\uc2e4\uc5d0 \ub300\ud574 \ud574\uba85\ud588\ub2e4.", "answer": "\ub958\ud0dc\uc6b0", "sentence": "1959\ub144 7\uc6d4 28\uc77c\uc5d0 \uacbd\uc0c1\ubd81\ub3c4 \uacbd\uc8fc\uc5d0\uc11c \uc544\ubc84\uc9c0 \ub958\ud0dc\uc6b0 \uc640 \uc5b4\uba38\ub2c8 \uc11c\ub3d9\ud544 \uc0ac\uc774\uc5d0\uc11c 4\ub0a8 2\ub140 \uc911 4\ub0a8\uc73c\ub85c \ud0dc\uc5b4\ub0ac\ub2e4.", "paragraph_sentence": " 1959\ub144 7\uc6d4 28\uc77c\uc5d0 \uacbd\uc0c1\ubd81\ub3c4 \uacbd\uc8fc\uc5d0\uc11c \uc544\ubc84\uc9c0 \ub958\ud0dc\uc6b0 \uc640 \uc5b4\uba38\ub2c8 \uc11c\ub3d9\ud544 \uc0ac\uc774\uc5d0\uc11c 4\ub0a8 2\ub140 \uc911 4\ub0a8\uc73c\ub85c \ud0dc\uc5b4\ub0ac\ub2e4. 13\ub300\uc870\ub294 \uc784\uc9c4\uc65c\ub780\uc744 \uadf9\ubcf5\ud558\ub294 \ub370 \ud798\uc4f4 \uc911\ud765\uc758 \uba85\uc7ac\uc0c1 \uc758\uc815\ubd80 \uc601\uc758\uc815 \uc11c\uc560 \ub958\uc131\ub8e1\uc774\ub2e4. 12\ub300\uc870\ub294 \uc7a5\uc218\ucc30\ubc29\uc744 \uc9c0\ub0b8 \ub958\uc5ec\uc774\uba70, 11\ub300\uc870\ub294 \uc870\uc120\uc911\uae30\uc758 \ub300\ud559\uc790\ub85c \uc548\uae30 \ucc30\ubc29\uc744 \uc9c0\ub0b8 \uc878\uc7ac \ub958\uc6d0\uc9c0\uc774\uace0, 10\ub300\uc870\ub294 \ubd80\ud638\uad70\uc744 \uc9c0\ub0b8 \ub958\ub9cc\ud558, 9\ub300\uc870\ub294 \uc138\uc790 \uc2dc\uac15\uc6d0 \uc790\uc758\ub97c \uc9c0\ub0b8 \uc8fc\uc77c\uc7ac \ub958\ud6c4\uc7a5\uc774\ub2e4. \uc870\ubd80 \ub300\uc5d0 \uc774\ub974\ub7ec \uc548\ub3d9\uc5d0\uc11c \ucc98\uac00\uac00 \uc788\ub294 \uacbd\uc8fc\ub85c \uc774\uac70\ud558\uc600\ub2e4. \uc544\ubc84\uc9c0 \ub958\ud0dc\uc6b0\ub294 \uacbd\uc8fc\uc5ec\uc790\uace0\ub4f1\ud559\uad50\uc758 \uc5ed\uc0ac \uad50\uc0ac\uc600\ub2e4.\ud070\uc544\ubc84\uc9c0 \ub958\uc11d\uc6b0\ub294 \uacf5\ubb34\uc6d0\uc73c\ub85c \ub0b4\ub0a8\uba74 \uba74\uc7a5\uc744 \uc9c0\ub0b4\uae30\ub3c4 \ud588\ub2e4.\ub958\ud0dc\uc6b0\uac00 \ub9cc\uc8fc \uc18c\ud559\uad50\uc758 \uad50\uc9c1\uc6d0\uc73c\ub85c \uadfc\ubb34\ud55c \uc77c\uc740 \ud6c4\uc5d0 \uc758\ud639\uc73c\ub85c \uc81c\uae30\ub418\uc5c8\ub294\ub370, \uc774\ub97c \ub450\uace0 \uc720\uc2dc\ubbfc\uc740 \"\ubc31\ubd80\uac00 \uba74\uc7a5\uc744 \ud55c \uac83\uc740 \ub9de\uc9c0\ub9cc \uc120\uce5c\uc740 \uc77c\uc81c \uce58\ud558 \ub54c \uad50\uc0ac\ub97c \ud558\uc9c0 \uc54a\uc558\ub2e4. \ucc98\uc74c \ub4e3\ub294 \uc598\uae30\ub77c \uc9d1\uc548 \uc5b4\ub974\uc2e0\ub4e4\uaed8 \ud655\uc778\ud574\ubcf8 \uacb0\uacfc \uc120\uce5c\uc740 \ud574\ubc29 \uc9c1\ud6c4 \ubbf8 \uad70\uc815\uc774 \uad50\uc0ac \uc694\uc6d0\uc744 \uacf5\ucc44\ud588\uc744 \ub54c \ub3d9\uc591\uc0ac \ubd84\uc57c\uc5d0 \uc751\uc2dc\ud574 \ud569\uaca9\ud588\uace0 6\uac1c\uc6d4 \uc5f0\uc218 \ud6c4 \uacbd\uc8fc\uc5ec\uc790\uc911\ud559\uad50\uc5d0 \ubd80\uc784\ud588\ub2e4. \"\uace0 \ub9d0\ud558\uba70 \uad00\ub828 \uc0ac\uc2e4\uc5d0 \ub300\ud574 \ud574\uba85\ud588\ub2e4.", "paragraph_answer": "1959\ub144 7\uc6d4 28\uc77c\uc5d0 \uacbd\uc0c1\ubd81\ub3c4 \uacbd\uc8fc\uc5d0\uc11c \uc544\ubc84\uc9c0 \ub958\ud0dc\uc6b0 \uc640 \uc5b4\uba38\ub2c8 \uc11c\ub3d9\ud544 \uc0ac\uc774\uc5d0\uc11c 4\ub0a8 2\ub140 \uc911 4\ub0a8\uc73c\ub85c \ud0dc\uc5b4\ub0ac\ub2e4. 13\ub300\uc870\ub294 \uc784\uc9c4\uc65c\ub780\uc744 \uadf9\ubcf5\ud558\ub294 \ub370 \ud798\uc4f4 \uc911\ud765\uc758 \uba85\uc7ac\uc0c1 \uc758\uc815\ubd80 \uc601\uc758\uc815 \uc11c\uc560 \ub958\uc131\ub8e1\uc774\ub2e4. 12\ub300\uc870\ub294 \uc7a5\uc218\ucc30\ubc29\uc744 \uc9c0\ub0b8 \ub958\uc5ec\uc774\uba70, 11\ub300\uc870\ub294 \uc870\uc120\uc911\uae30\uc758 \ub300\ud559\uc790\ub85c \uc548\uae30 \ucc30\ubc29\uc744 \uc9c0\ub0b8 \uc878\uc7ac \ub958\uc6d0\uc9c0\uc774\uace0, 10\ub300\uc870\ub294 \ubd80\ud638\uad70\uc744 \uc9c0\ub0b8 \ub958\ub9cc\ud558, 9\ub300\uc870\ub294 \uc138\uc790 \uc2dc\uac15\uc6d0 \uc790\uc758\ub97c \uc9c0\ub0b8 \uc8fc\uc77c\uc7ac \ub958\ud6c4\uc7a5\uc774\ub2e4. \uc870\ubd80 \ub300\uc5d0 \uc774\ub974\ub7ec \uc548\ub3d9\uc5d0\uc11c \ucc98\uac00\uac00 \uc788\ub294 \uacbd\uc8fc\ub85c \uc774\uac70\ud558\uc600\ub2e4. \uc544\ubc84\uc9c0 \ub958\ud0dc\uc6b0\ub294 \uacbd\uc8fc\uc5ec\uc790\uace0\ub4f1\ud559\uad50\uc758 \uc5ed\uc0ac \uad50\uc0ac\uc600\ub2e4.\ud070\uc544\ubc84\uc9c0 \ub958\uc11d\uc6b0\ub294 \uacf5\ubb34\uc6d0\uc73c\ub85c \ub0b4\ub0a8\uba74 \uba74\uc7a5\uc744 \uc9c0\ub0b4\uae30\ub3c4 \ud588\ub2e4.\ub958\ud0dc\uc6b0\uac00 \ub9cc\uc8fc \uc18c\ud559\uad50\uc758 \uad50\uc9c1\uc6d0\uc73c\ub85c \uadfc\ubb34\ud55c \uc77c\uc740 \ud6c4\uc5d0 \uc758\ud639\uc73c\ub85c \uc81c\uae30\ub418\uc5c8\ub294\ub370, \uc774\ub97c \ub450\uace0 \uc720\uc2dc\ubbfc\uc740 \"\ubc31\ubd80\uac00 \uba74\uc7a5\uc744 \ud55c \uac83\uc740 \ub9de\uc9c0\ub9cc \uc120\uce5c\uc740 \uc77c\uc81c \uce58\ud558 \ub54c \uad50\uc0ac\ub97c \ud558\uc9c0 \uc54a\uc558\ub2e4. \ucc98\uc74c \ub4e3\ub294 \uc598\uae30\ub77c \uc9d1\uc548 \uc5b4\ub974\uc2e0\ub4e4\uaed8 \ud655\uc778\ud574\ubcf8 \uacb0\uacfc \uc120\uce5c\uc740 \ud574\ubc29 \uc9c1\ud6c4 \ubbf8 \uad70\uc815\uc774 \uad50\uc0ac \uc694\uc6d0\uc744 \uacf5\ucc44\ud588\uc744 \ub54c \ub3d9\uc591\uc0ac \ubd84\uc57c\uc5d0 \uc751\uc2dc\ud574 \ud569\uaca9\ud588\uace0 6\uac1c\uc6d4 \uc5f0\uc218 \ud6c4 \uacbd\uc8fc\uc5ec\uc790\uc911\ud559\uad50\uc5d0 \ubd80\uc784\ud588\ub2e4.\"\uace0 \ub9d0\ud558\uba70 \uad00\ub828 \uc0ac\uc2e4\uc5d0 \ub300\ud574 \ud574\uba85\ud588\ub2e4.", "sentence_answer": "1959\ub144 7\uc6d4 28\uc77c\uc5d0 \uacbd\uc0c1\ubd81\ub3c4 \uacbd\uc8fc\uc5d0\uc11c \uc544\ubc84\uc9c0 \ub958\ud0dc\uc6b0 \uc640 \uc5b4\uba38\ub2c8 \uc11c\ub3d9\ud544 \uc0ac\uc774\uc5d0\uc11c 4\ub0a8 2\ub140 \uc911 4\ub0a8\uc73c\ub85c \ud0dc\uc5b4\ub0ac\ub2e4.", "paragraph_id": "6543184-0-0", "paragraph_question": "question: \uc720\uc2dc\ubbfc\uc758 \uc544\ubc84\uc9c0\ub294?, context: 1959\ub144 7\uc6d4 28\uc77c\uc5d0 \uacbd\uc0c1\ubd81\ub3c4 \uacbd\uc8fc\uc5d0\uc11c \uc544\ubc84\uc9c0 \ub958\ud0dc\uc6b0\uc640 \uc5b4\uba38\ub2c8 \uc11c\ub3d9\ud544 \uc0ac\uc774\uc5d0\uc11c 4\ub0a8 2\ub140 \uc911 4\ub0a8\uc73c\ub85c \ud0dc\uc5b4\ub0ac\ub2e4. 13\ub300\uc870\ub294 \uc784\uc9c4\uc65c\ub780\uc744 \uadf9\ubcf5\ud558\ub294 \ub370 \ud798\uc4f4 \uc911\ud765\uc758 \uba85\uc7ac\uc0c1 \uc758\uc815\ubd80 \uc601\uc758\uc815 \uc11c\uc560 \ub958\uc131\ub8e1\uc774\ub2e4. 12\ub300\uc870\ub294 \uc7a5\uc218\ucc30\ubc29\uc744 \uc9c0\ub0b8 \ub958\uc5ec\uc774\uba70, 11\ub300\uc870\ub294 \uc870\uc120\uc911\uae30\uc758 \ub300\ud559\uc790\ub85c \uc548\uae30 \ucc30\ubc29\uc744 \uc9c0\ub0b8 \uc878\uc7ac \ub958\uc6d0\uc9c0\uc774\uace0, 10\ub300\uc870\ub294 \ubd80\ud638\uad70\uc744 \uc9c0\ub0b8 \ub958\ub9cc\ud558, 9\ub300\uc870\ub294 \uc138\uc790 \uc2dc\uac15\uc6d0 \uc790\uc758\ub97c \uc9c0\ub0b8 \uc8fc\uc77c\uc7ac \ub958\ud6c4\uc7a5\uc774\ub2e4. \uc870\ubd80 \ub300\uc5d0 \uc774\ub974\ub7ec \uc548\ub3d9\uc5d0\uc11c \ucc98\uac00\uac00 \uc788\ub294 \uacbd\uc8fc\ub85c \uc774\uac70\ud558\uc600\ub2e4. \uc544\ubc84\uc9c0 \ub958\ud0dc\uc6b0\ub294 \uacbd\uc8fc\uc5ec\uc790\uace0\ub4f1\ud559\uad50\uc758 \uc5ed\uc0ac \uad50\uc0ac\uc600\ub2e4.\ud070\uc544\ubc84\uc9c0 \ub958\uc11d\uc6b0\ub294 \uacf5\ubb34\uc6d0\uc73c\ub85c \ub0b4\ub0a8\uba74 \uba74\uc7a5\uc744 \uc9c0\ub0b4\uae30\ub3c4 \ud588\ub2e4.\ub958\ud0dc\uc6b0\uac00 \ub9cc\uc8fc \uc18c\ud559\uad50\uc758 \uad50\uc9c1\uc6d0\uc73c\ub85c \uadfc\ubb34\ud55c \uc77c\uc740 \ud6c4\uc5d0 \uc758\ud639\uc73c\ub85c \uc81c\uae30\ub418\uc5c8\ub294\ub370, \uc774\ub97c \ub450\uace0 \uc720\uc2dc\ubbfc\uc740 \"\ubc31\ubd80\uac00 \uba74\uc7a5\uc744 \ud55c \uac83\uc740 \ub9de\uc9c0\ub9cc \uc120\uce5c\uc740 \uc77c\uc81c \uce58\ud558 \ub54c \uad50\uc0ac\ub97c \ud558\uc9c0 \uc54a\uc558\ub2e4. \ucc98\uc74c \ub4e3\ub294 \uc598\uae30\ub77c \uc9d1\uc548 \uc5b4\ub974\uc2e0\ub4e4\uaed8 \ud655\uc778\ud574\ubcf8 \uacb0\uacfc \uc120\uce5c\uc740 \ud574\ubc29 \uc9c1\ud6c4 \ubbf8 \uad70\uc815\uc774 \uad50\uc0ac \uc694\uc6d0\uc744 \uacf5\ucc44\ud588\uc744 \ub54c \ub3d9\uc591\uc0ac \ubd84\uc57c\uc5d0 \uc751\uc2dc\ud574 \ud569\uaca9\ud588\uace0 6\uac1c\uc6d4 \uc5f0\uc218 \ud6c4 \uacbd\uc8fc\uc5ec\uc790\uc911\ud559\uad50\uc5d0 \ubd80\uc784\ud588\ub2e4.\"\uace0 \ub9d0\ud558\uba70 \uad00\ub828 \uc0ac\uc2e4\uc5d0 \ub300\ud574 \ud574\uba85\ud588\ub2e4."} {"question": "MC \uba54\ud0c0\ub294 \uc5b4\ub514\uc758 \ub300\ud45c \ub798\ud37c\ub85c \uc778\uc815\ubc1b\ub294\uac00?", "paragraph": "\ud55c\uad6d\ub300\uc911\uc74c\uc545 100\ub300 \uba85\ubc18\uc5d0\ub3c4 \uc624\ub97c \uc815\ub3c4\ub85c \uba85\ubc18\uc774\uc5c8\ub358 1\uc9d1 Garion\uc758 \uc601\uad11\uc744 \ub4a4\ub85c \ud558\uace0 \uac00\ub9ac\uc628\uc774 \ub9cc\ub4e0 \u300a\uac00\ub9ac\uc6282\u300b\ub294 \ud504\ub85c\ub4c0\uc11c\uc778 J.U.\uac00 \ub5a0\ub098\uace0 \uac70\uc758 \ubaa8\ub4e0 \uace1\uc5d0\uc11c \ub2e4\ub978 \ud504\ub85c\ub4c0\uc11c\ub4e4\uc744 \uc0ac\uc6a9\ud588\uc9c0\ub9cc \ud070 \uc2a4\ud0c0\uc77c\uc801\uc778 \ubcc0\ud654\ub97c \uaf80\ud558\uc9c0\ub294 \uc54a\uc558\ub2e4. \uc720\uae30\uc801\uc778 \uad6c\uc131\uc73c\ub85c \ub418\uc5b4\uc788\ub294 \uc774 \uc74c\ubc18\uc740 10\ub144\uc774 \ub118\ub294 \uc138\uc6d4 \ub3d9\uc548 \ubb34\ub108\uc9c0\uc9c0 \uc54a\uace0 \ud0c4\ud0c4\ud55c \uae30\ubcf8\uae30\ub97c \ubc14\ud0d5\uc73c\ub85c \uad73\uac74\ud55c \uc2a4\ud0c0\uc77c\uc744 \ubc14\ud0d5\uc73c\ub85c \ud558\uace0 \uc788\uc5b4 '\uba85\ubd88\ud5c8\uc804'\uc774\ub77c\ub294 \ud3c9\uac00\ub97c \ubc1b\uc558\uc73c\ub098 \uadf8\uc640 \ubc18\ub300\ub85c \uc77c\uad00\ub41c \ub7a9 \uc2a4\ud0c0\uc77c\uacfc \uc804\uc791\uacfc \ud070 \uad6c\ubcc4\uc810\uc774 \uc5c6\ub294 \uac83 \ub4f1\uc740 \ube44\ud310\uc744 \ubc1b\uc558\ub2e4. \uc5b8\ub354\uadf8\ub77c\uc6b4\ub4dc\uc758 \ub300\ud45c \ub798\ud37c\ub85c \uc778\uc815\ubc1b\ub294 MC \uba54\ud0c0\uc640 \ud0c4\ud0c4\ud55c \uae30\ubcf8\uae30\ub97c \uc9c0\ub2cc \ub098\ucc30\uc758 \uc791\uc0ac \ub2a5\ub825\ub3c4 \uc791\ud488\uc5d0 \uc131\uacfc\ub97c \ub354\ud574\uc8fc\uc5c8\ub2e4. \uc774 \ub458\uc740 \ud488\uaca9\uc788\ub294 \uc791\uc0ac\ubc95\uc73c\ub85c \uc8fc\uc81c\ub97c \ub108\ubb34 \uc9c1\uc124\uc801\uc774\uac70\ub098 \ub108\ubb34 \ub3cc\ub824\ub9d0\ud558\uc9c0 \uc54a\uace0 \uc801\uc808\ud55c \ub2e8\uc5b4\ub4e4\ub85c \ud3ec\uc7a5\ud574 \ubb38\ud559\uc801\uc73c\ub85c \ud45c\ud604\ud55c\ub2e4\ub294 \ud3c9\uac00\ub97c \ubc1b\uc558\ub2e4. \uac00\uc0ac\uc801\uc778 \ubd80\ubd84 \uc678\uc5d0\ub3c4 \ud50c\ub85c\uc6b0, \ub77c\uc774\ubc0d, \ub9ac\ub4ec\uac10 \ub4f1\uc5d0\uc11c \ube44\ub86f\ub41c \ub7a9 \uc2e4\ub825\uc758 \uc9c4\ubcf4\ub3c4 \uc74c\ubc18\uc744 \ub4b7\ubc1b\uce68\ud574\uc8fc\ub294 \uc88b\uc740 \ubb34\uae30\uac00 \ub418\uc5c8\ub2e4\uace0 \ud638\ud3c9\uc744 \ubc1b\uc558\ub2e4.", "answer": "\uc5b8\ub354\uadf8\ub77c\uc6b4\ub4dc", "sentence": "\uc5b8\ub354\uadf8\ub77c\uc6b4\ub4dc \uc758 \ub300\ud45c \ub798\ud37c\ub85c \uc778\uc815\ubc1b\ub294 MC \uba54\ud0c0\uc640 \ud0c4\ud0c4\ud55c \uae30\ubcf8\uae30\ub97c \uc9c0\ub2cc \ub098\ucc30\uc758 \uc791\uc0ac \ub2a5\ub825\ub3c4 \uc791\ud488\uc5d0 \uc131\uacfc\ub97c \ub354\ud574\uc8fc\uc5c8\ub2e4.", "paragraph_sentence": "\ud55c\uad6d\ub300\uc911\uc74c\uc545 100\ub300 \uba85\ubc18\uc5d0\ub3c4 \uc624\ub97c \uc815\ub3c4\ub85c \uba85\ubc18\uc774\uc5c8\ub358 1\uc9d1 Garion\uc758 \uc601\uad11\uc744 \ub4a4\ub85c \ud558\uace0 \uac00\ub9ac\uc628\uc774 \ub9cc\ub4e0 \u300a\uac00\ub9ac\uc6282\u300b\ub294 \ud504\ub85c\ub4c0\uc11c\uc778 J.U.\uac00 \ub5a0\ub098\uace0 \uac70\uc758 \ubaa8\ub4e0 \uace1\uc5d0\uc11c \ub2e4\ub978 \ud504\ub85c\ub4c0\uc11c\ub4e4\uc744 \uc0ac\uc6a9\ud588\uc9c0\ub9cc \ud070 \uc2a4\ud0c0\uc77c\uc801\uc778 \ubcc0\ud654\ub97c \uaf80\ud558\uc9c0\ub294 \uc54a\uc558\ub2e4. \uc720\uae30\uc801\uc778 \uad6c\uc131\uc73c\ub85c \ub418\uc5b4\uc788\ub294 \uc774 \uc74c\ubc18\uc740 10\ub144\uc774 \ub118\ub294 \uc138\uc6d4 \ub3d9\uc548 \ubb34\ub108\uc9c0\uc9c0 \uc54a\uace0 \ud0c4\ud0c4\ud55c \uae30\ubcf8\uae30\ub97c \ubc14\ud0d5\uc73c\ub85c \uad73\uac74\ud55c \uc2a4\ud0c0\uc77c\uc744 \ubc14\ud0d5\uc73c\ub85c \ud558\uace0 \uc788\uc5b4 '\uba85\ubd88\ud5c8\uc804'\uc774\ub77c\ub294 \ud3c9\uac00\ub97c \ubc1b\uc558\uc73c\ub098 \uadf8\uc640 \ubc18\ub300\ub85c \uc77c\uad00\ub41c \ub7a9 \uc2a4\ud0c0\uc77c\uacfc \uc804\uc791\uacfc \ud070 \uad6c\ubcc4\uc810\uc774 \uc5c6\ub294 \uac83 \ub4f1\uc740 \ube44\ud310\uc744 \ubc1b\uc558\ub2e4. \uc5b8\ub354\uadf8\ub77c\uc6b4\ub4dc \uc758 \ub300\ud45c \ub798\ud37c\ub85c \uc778\uc815\ubc1b\ub294 MC \uba54\ud0c0\uc640 \ud0c4\ud0c4\ud55c \uae30\ubcf8\uae30\ub97c \uc9c0\ub2cc \ub098\ucc30\uc758 \uc791\uc0ac \ub2a5\ub825\ub3c4 \uc791\ud488\uc5d0 \uc131\uacfc\ub97c \ub354\ud574\uc8fc\uc5c8\ub2e4. \uc774 \ub458\uc740 \ud488\uaca9\uc788\ub294 \uc791\uc0ac\ubc95\uc73c\ub85c \uc8fc\uc81c\ub97c \ub108\ubb34 \uc9c1\uc124\uc801\uc774\uac70\ub098 \ub108\ubb34 \ub3cc\ub824\ub9d0\ud558\uc9c0 \uc54a\uace0 \uc801\uc808\ud55c \ub2e8\uc5b4\ub4e4\ub85c \ud3ec\uc7a5\ud574 \ubb38\ud559\uc801\uc73c\ub85c \ud45c\ud604\ud55c\ub2e4\ub294 \ud3c9\uac00\ub97c \ubc1b\uc558\ub2e4. \uac00\uc0ac\uc801\uc778 \ubd80\ubd84 \uc678\uc5d0\ub3c4 \ud50c\ub85c\uc6b0, \ub77c\uc774\ubc0d, \ub9ac\ub4ec\uac10 \ub4f1\uc5d0\uc11c \ube44\ub86f\ub41c \ub7a9 \uc2e4\ub825\uc758 \uc9c4\ubcf4\ub3c4 \uc74c\ubc18\uc744 \ub4b7\ubc1b\uce68\ud574\uc8fc\ub294 \uc88b\uc740 \ubb34\uae30\uac00 \ub418\uc5c8\ub2e4\uace0 \ud638\ud3c9\uc744 \ubc1b\uc558\ub2e4.", "paragraph_answer": "\ud55c\uad6d\ub300\uc911\uc74c\uc545 100\ub300 \uba85\ubc18\uc5d0\ub3c4 \uc624\ub97c \uc815\ub3c4\ub85c \uba85\ubc18\uc774\uc5c8\ub358 1\uc9d1 Garion\uc758 \uc601\uad11\uc744 \ub4a4\ub85c \ud558\uace0 \uac00\ub9ac\uc628\uc774 \ub9cc\ub4e0 \u300a\uac00\ub9ac\uc6282\u300b\ub294 \ud504\ub85c\ub4c0\uc11c\uc778 J.U.\uac00 \ub5a0\ub098\uace0 \uac70\uc758 \ubaa8\ub4e0 \uace1\uc5d0\uc11c \ub2e4\ub978 \ud504\ub85c\ub4c0\uc11c\ub4e4\uc744 \uc0ac\uc6a9\ud588\uc9c0\ub9cc \ud070 \uc2a4\ud0c0\uc77c\uc801\uc778 \ubcc0\ud654\ub97c \uaf80\ud558\uc9c0\ub294 \uc54a\uc558\ub2e4. \uc720\uae30\uc801\uc778 \uad6c\uc131\uc73c\ub85c \ub418\uc5b4\uc788\ub294 \uc774 \uc74c\ubc18\uc740 10\ub144\uc774 \ub118\ub294 \uc138\uc6d4 \ub3d9\uc548 \ubb34\ub108\uc9c0\uc9c0 \uc54a\uace0 \ud0c4\ud0c4\ud55c \uae30\ubcf8\uae30\ub97c \ubc14\ud0d5\uc73c\ub85c \uad73\uac74\ud55c \uc2a4\ud0c0\uc77c\uc744 \ubc14\ud0d5\uc73c\ub85c \ud558\uace0 \uc788\uc5b4 '\uba85\ubd88\ud5c8\uc804'\uc774\ub77c\ub294 \ud3c9\uac00\ub97c \ubc1b\uc558\uc73c\ub098 \uadf8\uc640 \ubc18\ub300\ub85c \uc77c\uad00\ub41c \ub7a9 \uc2a4\ud0c0\uc77c\uacfc \uc804\uc791\uacfc \ud070 \uad6c\ubcc4\uc810\uc774 \uc5c6\ub294 \uac83 \ub4f1\uc740 \ube44\ud310\uc744 \ubc1b\uc558\ub2e4. \uc5b8\ub354\uadf8\ub77c\uc6b4\ub4dc \uc758 \ub300\ud45c \ub798\ud37c\ub85c \uc778\uc815\ubc1b\ub294 MC \uba54\ud0c0\uc640 \ud0c4\ud0c4\ud55c \uae30\ubcf8\uae30\ub97c \uc9c0\ub2cc \ub098\ucc30\uc758 \uc791\uc0ac \ub2a5\ub825\ub3c4 \uc791\ud488\uc5d0 \uc131\uacfc\ub97c \ub354\ud574\uc8fc\uc5c8\ub2e4. \uc774 \ub458\uc740 \ud488\uaca9\uc788\ub294 \uc791\uc0ac\ubc95\uc73c\ub85c \uc8fc\uc81c\ub97c \ub108\ubb34 \uc9c1\uc124\uc801\uc774\uac70\ub098 \ub108\ubb34 \ub3cc\ub824\ub9d0\ud558\uc9c0 \uc54a\uace0 \uc801\uc808\ud55c \ub2e8\uc5b4\ub4e4\ub85c \ud3ec\uc7a5\ud574 \ubb38\ud559\uc801\uc73c\ub85c \ud45c\ud604\ud55c\ub2e4\ub294 \ud3c9\uac00\ub97c \ubc1b\uc558\ub2e4. \uac00\uc0ac\uc801\uc778 \ubd80\ubd84 \uc678\uc5d0\ub3c4 \ud50c\ub85c\uc6b0, \ub77c\uc774\ubc0d, \ub9ac\ub4ec\uac10 \ub4f1\uc5d0\uc11c \ube44\ub86f\ub41c \ub7a9 \uc2e4\ub825\uc758 \uc9c4\ubcf4\ub3c4 \uc74c\ubc18\uc744 \ub4b7\ubc1b\uce68\ud574\uc8fc\ub294 \uc88b\uc740 \ubb34\uae30\uac00 \ub418\uc5c8\ub2e4\uace0 \ud638\ud3c9\uc744 \ubc1b\uc558\ub2e4.", "sentence_answer": " \uc5b8\ub354\uadf8\ub77c\uc6b4\ub4dc \uc758 \ub300\ud45c \ub798\ud37c\ub85c \uc778\uc815\ubc1b\ub294 MC \uba54\ud0c0\uc640 \ud0c4\ud0c4\ud55c \uae30\ubcf8\uae30\ub97c \uc9c0\ub2cc \ub098\ucc30\uc758 \uc791\uc0ac \ub2a5\ub825\ub3c4 \uc791\ud488\uc5d0 \uc131\uacfc\ub97c \ub354\ud574\uc8fc\uc5c8\ub2e4.", "paragraph_id": "6598007-3-0", "paragraph_question": "question: MC \uba54\ud0c0\ub294 \uc5b4\ub514\uc758 \ub300\ud45c \ub798\ud37c\ub85c \uc778\uc815\ubc1b\ub294\uac00?, context: \ud55c\uad6d\ub300\uc911\uc74c\uc545 100\ub300 \uba85\ubc18\uc5d0\ub3c4 \uc624\ub97c \uc815\ub3c4\ub85c \uba85\ubc18\uc774\uc5c8\ub358 1\uc9d1 Garion\uc758 \uc601\uad11\uc744 \ub4a4\ub85c \ud558\uace0 \uac00\ub9ac\uc628\uc774 \ub9cc\ub4e0 \u300a\uac00\ub9ac\uc6282\u300b\ub294 \ud504\ub85c\ub4c0\uc11c\uc778 J.U.\uac00 \ub5a0\ub098\uace0 \uac70\uc758 \ubaa8\ub4e0 \uace1\uc5d0\uc11c \ub2e4\ub978 \ud504\ub85c\ub4c0\uc11c\ub4e4\uc744 \uc0ac\uc6a9\ud588\uc9c0\ub9cc \ud070 \uc2a4\ud0c0\uc77c\uc801\uc778 \ubcc0\ud654\ub97c \uaf80\ud558\uc9c0\ub294 \uc54a\uc558\ub2e4. \uc720\uae30\uc801\uc778 \uad6c\uc131\uc73c\ub85c \ub418\uc5b4\uc788\ub294 \uc774 \uc74c\ubc18\uc740 10\ub144\uc774 \ub118\ub294 \uc138\uc6d4 \ub3d9\uc548 \ubb34\ub108\uc9c0\uc9c0 \uc54a\uace0 \ud0c4\ud0c4\ud55c \uae30\ubcf8\uae30\ub97c \ubc14\ud0d5\uc73c\ub85c \uad73\uac74\ud55c \uc2a4\ud0c0\uc77c\uc744 \ubc14\ud0d5\uc73c\ub85c \ud558\uace0 \uc788\uc5b4 '\uba85\ubd88\ud5c8\uc804'\uc774\ub77c\ub294 \ud3c9\uac00\ub97c \ubc1b\uc558\uc73c\ub098 \uadf8\uc640 \ubc18\ub300\ub85c \uc77c\uad00\ub41c \ub7a9 \uc2a4\ud0c0\uc77c\uacfc \uc804\uc791\uacfc \ud070 \uad6c\ubcc4\uc810\uc774 \uc5c6\ub294 \uac83 \ub4f1\uc740 \ube44\ud310\uc744 \ubc1b\uc558\ub2e4. \uc5b8\ub354\uadf8\ub77c\uc6b4\ub4dc\uc758 \ub300\ud45c \ub798\ud37c\ub85c \uc778\uc815\ubc1b\ub294 MC \uba54\ud0c0\uc640 \ud0c4\ud0c4\ud55c \uae30\ubcf8\uae30\ub97c \uc9c0\ub2cc \ub098\ucc30\uc758 \uc791\uc0ac \ub2a5\ub825\ub3c4 \uc791\ud488\uc5d0 \uc131\uacfc\ub97c \ub354\ud574\uc8fc\uc5c8\ub2e4. \uc774 \ub458\uc740 \ud488\uaca9\uc788\ub294 \uc791\uc0ac\ubc95\uc73c\ub85c \uc8fc\uc81c\ub97c \ub108\ubb34 \uc9c1\uc124\uc801\uc774\uac70\ub098 \ub108\ubb34 \ub3cc\ub824\ub9d0\ud558\uc9c0 \uc54a\uace0 \uc801\uc808\ud55c \ub2e8\uc5b4\ub4e4\ub85c \ud3ec\uc7a5\ud574 \ubb38\ud559\uc801\uc73c\ub85c \ud45c\ud604\ud55c\ub2e4\ub294 \ud3c9\uac00\ub97c \ubc1b\uc558\ub2e4. \uac00\uc0ac\uc801\uc778 \ubd80\ubd84 \uc678\uc5d0\ub3c4 \ud50c\ub85c\uc6b0, \ub77c\uc774\ubc0d, \ub9ac\ub4ec\uac10 \ub4f1\uc5d0\uc11c \ube44\ub86f\ub41c \ub7a9 \uc2e4\ub825\uc758 \uc9c4\ubcf4\ub3c4 \uc74c\ubc18\uc744 \ub4b7\ubc1b\uce68\ud574\uc8fc\ub294 \uc88b\uc740 \ubb34\uae30\uac00 \ub418\uc5c8\ub2e4\uace0 \ud638\ud3c9\uc744 \ubc1b\uc558\ub2e4."} {"question": "\ub85c\uc800 \ud398\ub354\ub7ec\uac00 \uacb0\ud63c\uc2dd\uc744 \uc62c\ub9b0 \uc7a5\uc18c\ub294?", "paragraph": "\ud398\ub354\ub7ec\ub294 2009\ub144 4\uc6d4 \uc804 \uc5ec\uc790 \ud504\ub85c \ud14c\ub2c8\uc2a4 \uc120\uc218\uc774\uc790 \uadf8\uc758 \uc624\ub79c \uc5f0\uc778\uc774\uc5c8\ub358 \ubbf8\ub974\uce74 \ubc14\ube0c\ub9ac\ub125\uacfc \uacb0\ud63c\ud558\uc600\ub2e4. \ub450 \uc0ac\ub78c\uc740 2000\ub144 \uc2dc\ub4dc\ub2c8 \uc62c\ub9bc\ud53d\uc5d0\uc11c \ud14c\ub2c8\uc2a4 \uc885\ubaa9 \uc2a4\uc704\uc2a4 \uad6d\uac00\ub300\ud45c\ub85c \ud568\uaed8 \ucd9c\uc804\ud558\uba74\uc11c \ub9cc\ub098\uac8c \ub418\uc5c8\ub2e4. \ubc14\ube0c\ub9ac\ub125\uc740 \uace0\uc9c8\uc801\uc778 \ubc1c \ubd80\uc0c1 \ub54c\ubb38\uc5d0 2002\ub144 \ud22c\uc5b4\uc5d0\uc11c \uc740\ud1f4\ud558\uc600\uace0 \uc774\ud6c4 \ud398\ub354\ub7ec\uc758 \ud64d\ubcf4 \ub9e4\ub2c8\uc800 \uc5ed\ud560\uc744 \ub2f4\ub2f9\ud558\uc600\ub2e4. \uc624\ub79c \uc5f0\uc778 \uad00\uacc4\ub97c \uc9c0\uc18d\ud558\ub358 \uc774\ub4e4\uc740 2009\ub144 3\uc6d4\uc5d0 \ud398\ub354\ub7ec\uc758 \ud648\ud398\uc774\uc9c0\ub97c \ud1b5\ud574 \ubc14\ube0c\ub9ac\ub125\uc758 \uc784\uc2e0\uc744 \ubc1c\ud45c\ud558\uc600\uc73c\uba70, \uc774\ud6c4 \ub450 \uc0ac\ub78c\uc740 2009\ub144 4\uc6d4 11\uc77c \ubc14\uc824\uc5d0\uc11c \uac00\uae4c\uc6b4 \uce5c\ucc99\ub4e4\uacfc \uce5c\uad6c\ub4e4\ub9cc\uc774 \ucc38\uc11d\ud55c \uac00\uc6b4\ub370 \uc870\ucd10\ud55c \uacb0\ud63c\uc2dd\uc744 \uc62c\ub838\ub2e4. 2009\ub144 7\uc6d4 24\uc77c, \ud398\ub354\ub7ec\ub294 \uc790\uc2e0\uc758 \ud398\uc774\uc2a4\ubd81 \ud648\ud398\uc774\uc9c0\ub97c \ud1b5\ud574 \ub450 \uc30d\ub465\uc774 \ub538 \ubc00\ub77c \ub85c\uc988(Myla Rose)\uc640 \uc0e4\ub97c\ub80c \ub9ac\ubc14(Charlene Riva)\uac00 7\uc6d4 23\uc77c\uc5d0 \ud0c4\uc0dd\ud588\uc74c\uc744 \uc54c\ub838\ub2e4.", "answer": "\ubc14\uc824", "sentence": "\uc774\ud6c4 \ub450 \uc0ac\ub78c\uc740 2009\ub144 4\uc6d4 11\uc77c \ubc14\uc824 \uc5d0\uc11c \uac00\uae4c\uc6b4 \uce5c\ucc99\ub4e4\uacfc \uce5c\uad6c\ub4e4\ub9cc\uc774 \ucc38\uc11d\ud55c \uac00\uc6b4\ub370 \uc870\ucd10\ud55c \uacb0\ud63c\uc2dd\uc744 \uc62c\ub838\ub2e4.", "paragraph_sentence": "\ud398\ub354\ub7ec\ub294 2009\ub144 4\uc6d4 \uc804 \uc5ec\uc790 \ud504\ub85c \ud14c\ub2c8\uc2a4 \uc120\uc218\uc774\uc790 \uadf8\uc758 \uc624\ub79c \uc5f0\uc778\uc774\uc5c8\ub358 \ubbf8\ub974\uce74 \ubc14\ube0c\ub9ac\ub125\uacfc \uacb0\ud63c\ud558\uc600\ub2e4. \ub450 \uc0ac\ub78c\uc740 2000\ub144 \uc2dc\ub4dc\ub2c8 \uc62c\ub9bc\ud53d\uc5d0\uc11c \ud14c\ub2c8\uc2a4 \uc885\ubaa9 \uc2a4\uc704\uc2a4 \uad6d\uac00\ub300\ud45c\ub85c \ud568\uaed8 \ucd9c\uc804\ud558\uba74\uc11c \ub9cc\ub098\uac8c \ub418\uc5c8\ub2e4. \ubc14\ube0c\ub9ac\ub125\uc740 \uace0\uc9c8\uc801\uc778 \ubc1c \ubd80\uc0c1 \ub54c\ubb38\uc5d0 2002\ub144 \ud22c\uc5b4\uc5d0\uc11c \uc740\ud1f4\ud558\uc600\uace0 \uc774\ud6c4 \ud398\ub354\ub7ec\uc758 \ud64d\ubcf4 \ub9e4\ub2c8\uc800 \uc5ed\ud560\uc744 \ub2f4\ub2f9\ud558\uc600\ub2e4. \uc624\ub79c \uc5f0\uc778 \uad00\uacc4\ub97c \uc9c0\uc18d\ud558\ub358 \uc774\ub4e4\uc740 2009\ub144 3\uc6d4\uc5d0 \ud398\ub354\ub7ec\uc758 \ud648\ud398\uc774\uc9c0\ub97c \ud1b5\ud574 \ubc14\ube0c\ub9ac\ub125\uc758 \uc784\uc2e0\uc744 \ubc1c\ud45c\ud558\uc600\uc73c\uba70, \uc774\ud6c4 \ub450 \uc0ac\ub78c\uc740 2009\ub144 4\uc6d4 11\uc77c \ubc14\uc824 \uc5d0\uc11c \uac00\uae4c\uc6b4 \uce5c\ucc99\ub4e4\uacfc \uce5c\uad6c\ub4e4\ub9cc\uc774 \ucc38\uc11d\ud55c \uac00\uc6b4\ub370 \uc870\ucd10\ud55c \uacb0\ud63c\uc2dd\uc744 \uc62c\ub838\ub2e4. 2009\ub144 7\uc6d4 24\uc77c, \ud398\ub354\ub7ec\ub294 \uc790\uc2e0\uc758 \ud398\uc774\uc2a4\ubd81 \ud648\ud398\uc774\uc9c0\ub97c \ud1b5\ud574 \ub450 \uc30d\ub465\uc774 \ub538 \ubc00\ub77c \ub85c\uc988(Myla Rose)\uc640 \uc0e4\ub97c\ub80c \ub9ac\ubc14(Charlene Riva)\uac00 7\uc6d4 23\uc77c\uc5d0 \ud0c4\uc0dd\ud588\uc74c\uc744 \uc54c\ub838\ub2e4.", "paragraph_answer": "\ud398\ub354\ub7ec\ub294 2009\ub144 4\uc6d4 \uc804 \uc5ec\uc790 \ud504\ub85c \ud14c\ub2c8\uc2a4 \uc120\uc218\uc774\uc790 \uadf8\uc758 \uc624\ub79c \uc5f0\uc778\uc774\uc5c8\ub358 \ubbf8\ub974\uce74 \ubc14\ube0c\ub9ac\ub125\uacfc \uacb0\ud63c\ud558\uc600\ub2e4. \ub450 \uc0ac\ub78c\uc740 2000\ub144 \uc2dc\ub4dc\ub2c8 \uc62c\ub9bc\ud53d\uc5d0\uc11c \ud14c\ub2c8\uc2a4 \uc885\ubaa9 \uc2a4\uc704\uc2a4 \uad6d\uac00\ub300\ud45c\ub85c \ud568\uaed8 \ucd9c\uc804\ud558\uba74\uc11c \ub9cc\ub098\uac8c \ub418\uc5c8\ub2e4. \ubc14\ube0c\ub9ac\ub125\uc740 \uace0\uc9c8\uc801\uc778 \ubc1c \ubd80\uc0c1 \ub54c\ubb38\uc5d0 2002\ub144 \ud22c\uc5b4\uc5d0\uc11c \uc740\ud1f4\ud558\uc600\uace0 \uc774\ud6c4 \ud398\ub354\ub7ec\uc758 \ud64d\ubcf4 \ub9e4\ub2c8\uc800 \uc5ed\ud560\uc744 \ub2f4\ub2f9\ud558\uc600\ub2e4. \uc624\ub79c \uc5f0\uc778 \uad00\uacc4\ub97c \uc9c0\uc18d\ud558\ub358 \uc774\ub4e4\uc740 2009\ub144 3\uc6d4\uc5d0 \ud398\ub354\ub7ec\uc758 \ud648\ud398\uc774\uc9c0\ub97c \ud1b5\ud574 \ubc14\ube0c\ub9ac\ub125\uc758 \uc784\uc2e0\uc744 \ubc1c\ud45c\ud558\uc600\uc73c\uba70, \uc774\ud6c4 \ub450 \uc0ac\ub78c\uc740 2009\ub144 4\uc6d4 11\uc77c \ubc14\uc824 \uc5d0\uc11c \uac00\uae4c\uc6b4 \uce5c\ucc99\ub4e4\uacfc \uce5c\uad6c\ub4e4\ub9cc\uc774 \ucc38\uc11d\ud55c \uac00\uc6b4\ub370 \uc870\ucd10\ud55c \uacb0\ud63c\uc2dd\uc744 \uc62c\ub838\ub2e4. 2009\ub144 7\uc6d4 24\uc77c, \ud398\ub354\ub7ec\ub294 \uc790\uc2e0\uc758 \ud398\uc774\uc2a4\ubd81 \ud648\ud398\uc774\uc9c0\ub97c \ud1b5\ud574 \ub450 \uc30d\ub465\uc774 \ub538 \ubc00\ub77c \ub85c\uc988(Myla Rose)\uc640 \uc0e4\ub97c\ub80c \ub9ac\ubc14(Charlene Riva)\uac00 7\uc6d4 23\uc77c\uc5d0 \ud0c4\uc0dd\ud588\uc74c\uc744 \uc54c\ub838\ub2e4.", "sentence_answer": "\uc774\ud6c4 \ub450 \uc0ac\ub78c\uc740 2009\ub144 4\uc6d4 11\uc77c \ubc14\uc824 \uc5d0\uc11c \uac00\uae4c\uc6b4 \uce5c\ucc99\ub4e4\uacfc \uce5c\uad6c\ub4e4\ub9cc\uc774 \ucc38\uc11d\ud55c \uac00\uc6b4\ub370 \uc870\ucd10\ud55c \uacb0\ud63c\uc2dd\uc744 \uc62c\ub838\ub2e4.", "paragraph_id": "6496547-1-2", "paragraph_question": "question: \ub85c\uc800 \ud398\ub354\ub7ec\uac00 \uacb0\ud63c\uc2dd\uc744 \uc62c\ub9b0 \uc7a5\uc18c\ub294?, context: \ud398\ub354\ub7ec\ub294 2009\ub144 4\uc6d4 \uc804 \uc5ec\uc790 \ud504\ub85c \ud14c\ub2c8\uc2a4 \uc120\uc218\uc774\uc790 \uadf8\uc758 \uc624\ub79c \uc5f0\uc778\uc774\uc5c8\ub358 \ubbf8\ub974\uce74 \ubc14\ube0c\ub9ac\ub125\uacfc \uacb0\ud63c\ud558\uc600\ub2e4. \ub450 \uc0ac\ub78c\uc740 2000\ub144 \uc2dc\ub4dc\ub2c8 \uc62c\ub9bc\ud53d\uc5d0\uc11c \ud14c\ub2c8\uc2a4 \uc885\ubaa9 \uc2a4\uc704\uc2a4 \uad6d\uac00\ub300\ud45c\ub85c \ud568\uaed8 \ucd9c\uc804\ud558\uba74\uc11c \ub9cc\ub098\uac8c \ub418\uc5c8\ub2e4. \ubc14\ube0c\ub9ac\ub125\uc740 \uace0\uc9c8\uc801\uc778 \ubc1c \ubd80\uc0c1 \ub54c\ubb38\uc5d0 2002\ub144 \ud22c\uc5b4\uc5d0\uc11c \uc740\ud1f4\ud558\uc600\uace0 \uc774\ud6c4 \ud398\ub354\ub7ec\uc758 \ud64d\ubcf4 \ub9e4\ub2c8\uc800 \uc5ed\ud560\uc744 \ub2f4\ub2f9\ud558\uc600\ub2e4. \uc624\ub79c \uc5f0\uc778 \uad00\uacc4\ub97c \uc9c0\uc18d\ud558\ub358 \uc774\ub4e4\uc740 2009\ub144 3\uc6d4\uc5d0 \ud398\ub354\ub7ec\uc758 \ud648\ud398\uc774\uc9c0\ub97c \ud1b5\ud574 \ubc14\ube0c\ub9ac\ub125\uc758 \uc784\uc2e0\uc744 \ubc1c\ud45c\ud558\uc600\uc73c\uba70, \uc774\ud6c4 \ub450 \uc0ac\ub78c\uc740 2009\ub144 4\uc6d4 11\uc77c \ubc14\uc824\uc5d0\uc11c \uac00\uae4c\uc6b4 \uce5c\ucc99\ub4e4\uacfc \uce5c\uad6c\ub4e4\ub9cc\uc774 \ucc38\uc11d\ud55c \uac00\uc6b4\ub370 \uc870\ucd10\ud55c \uacb0\ud63c\uc2dd\uc744 \uc62c\ub838\ub2e4. 2009\ub144 7\uc6d4 24\uc77c, \ud398\ub354\ub7ec\ub294 \uc790\uc2e0\uc758 \ud398\uc774\uc2a4\ubd81 \ud648\ud398\uc774\uc9c0\ub97c \ud1b5\ud574 \ub450 \uc30d\ub465\uc774 \ub538 \ubc00\ub77c \ub85c\uc988(Myla Rose)\uc640 \uc0e4\ub97c\ub80c \ub9ac\ubc14(Charlene Riva)\uac00 7\uc6d4 23\uc77c\uc5d0 \ud0c4\uc0dd\ud588\uc74c\uc744 \uc54c\ub838\ub2e4."} {"question": "\ub2e5\ud130\ud6c4 \uc2dc\ub9ac\uc988\uc758 \uc6b0\ub7ec\ub4dc\ud22c\uc5b4\uc5d0 \ub300\ud55c \uc5d0\ud53c\uc18c\ub4dc\ub97c \ub2e4\ub8ec \ub2e4\ud050\uba58\ud130\ub9ac\ub97c \ubc29\uc601\ud55c \ub300\ud55c\ubbfc\uad6d\uc758 \ubc29\uc1a1\uad6d\uc740?", "paragraph": "\ucef4\ud328\ub2c8\uc5b8\uacfc \ub2e5\ud130 \ubcf8\uc778\uc774 '\ub2e5\ud130' \ub97c \ubc14\ub77c\ubcf4\ub294 \uad00\uc810\uacfc \uadf8\ub85c \uc778\ud55c \uac08\ub4f1\ub4e4\uc744 \uc911\uc810\uc801\uc73c\ub85c \ub2e4\ub8ec \uc2dc\ub9ac\uc988 8\uc740, \uae30\uc874\uc758 '13 \uc815\uaddc \uc5d0\ud53c\uc18c\ub4dc + 1 \uc2a4\ud398\uc15c' \uad6c\uc131\uc774 \uc544\ub2cc '12 \uc815\uaddc \uc5d0\ud53c\uc18c\ub4dc + 1 \uc2a4\ud398\uc15c' \uad6c\uc131\uc73c\ub85c \ucd95\uc18c\ub41c \ub300\uc2e0 \uc55e\uc120 \ub450 \uc2dc\uc98c\ucc98\ub7fc \ub450 \ud30c\ud2b8\ub85c \ub098\ub204\uc9c0 \uc54a\uace0 \uae30\uc874\ucc98\ub7fc \uc5f0\uc18d\uc801\uc73c\ub85c \ubc29\uc1a1\ub418\uc5c8\ub2e4. \ud55c\ud3b8, \ub300\ud55c\ubbfc\uad6d\uc758 \uacbd\uc6b0, \uc2dc\uc98c 8 \ud504\ub9ac\ubbf8\uc5b4\ub97c \uc704\ud55c \uc6d4\ub4dc\ud22c\uc5b4\uc5d0 \uc11c\uc6b8\uc774 \ud3ec\ud568\ub418\uba74\uc11c \uc5d0\ud53c\uc18c\ub4dc 1 '\uc228\uc744 \uba48\ucdb0\ub77c Deep Breath' \uc790\ub9c9\ud310\uc774 2014\ub144 8\uc6d4 9\uc77c \uc77c\ubd80 \ud32c\ub4e4\uc5d0\uac8c \uc120\ud589 \uacf5\uac1c \ub418\uae30\ub3c4 \ud588\ub2e4. \uc774 \uc2dc\uc0ac\ud68c\ub97c \ud3ec\ud568\ud55c \uc6d4\ub4dc\ud22c\uc5b4\uc5d0 \ub300\ud55c \uc5d0\ud53c\uc18c\ub4dc\ub4e4\uc744 \ub2e4\ub8ec \ub2e4\ud050\uba58\ud130\ub9ac\ub294 2015\ub144 1\uc6d4 18\uc77c \ubc24\uc5d0 KBS2 <\uc138\uc0c1\uc758 \ubaa8\ub4e0 \ub2e4\ud050> \ud504\ub85c\uadf8\ub7a8\uc744 \ud1b5\ud574 \uc790\ub9c9 \ubc29\uc601\ub418\uc5c8\ub2e4. \ud55c\ud3b8, \uc774\uc804 \ub2e5\ud130\uc758 \ub9c8\uc9c0\ub9c9 \uc5d0\ud53c\uc18c\ub4dc\uc778 '\ub2e5\ud130\uc758 \uc2dc\uac04 the Time of the Doctor' \uc744 \ud3ec\ud568\ud55c \uc2dc\uc98c 8\uc758 \ubc29\uc601\ubd84 \ubd80\ud130\ub294 KBS\uc758 \ud3b8\uc131 \ubcc0\uacbd\uc73c\ub85c KBS 1TV \ub85c \ucc44\ub110\uc744 \uc62e\uaca8 \ubc29\uc601\ud558\uace0 \uc788\ub2e4.", "answer": "KBS2", "sentence": "\uc774 \uc2dc\uc0ac\ud68c\ub97c \ud3ec\ud568\ud55c \uc6d4\ub4dc\ud22c\uc5b4\uc5d0 \ub300\ud55c \uc5d0\ud53c\uc18c\ub4dc\ub4e4\uc744 \ub2e4\ub8ec \ub2e4\ud050\uba58\ud130\ub9ac\ub294 2015\ub144 1\uc6d4 18\uc77c \ubc24\uc5d0 KBS2 <\uc138\uc0c1\uc758 \ubaa8\ub4e0 \ub2e4\ud050> \ud504\ub85c\uadf8\ub7a8\uc744 \ud1b5\ud574 \uc790\ub9c9 \ubc29\uc601\ub418\uc5c8\ub2e4.", "paragraph_sentence": "\ucef4\ud328\ub2c8\uc5b8\uacfc \ub2e5\ud130 \ubcf8\uc778\uc774 '\ub2e5\ud130' \ub97c \ubc14\ub77c\ubcf4\ub294 \uad00\uc810\uacfc \uadf8\ub85c \uc778\ud55c \uac08\ub4f1\ub4e4\uc744 \uc911\uc810\uc801\uc73c\ub85c \ub2e4\ub8ec \uc2dc\ub9ac\uc988 8\uc740, \uae30\uc874\uc758 '13 \uc815\uaddc \uc5d0\ud53c\uc18c\ub4dc + 1 \uc2a4\ud398\uc15c' \uad6c\uc131\uc774 \uc544\ub2cc '12 \uc815\uaddc \uc5d0\ud53c\uc18c\ub4dc + 1 \uc2a4\ud398\uc15c' \uad6c\uc131\uc73c\ub85c \ucd95\uc18c\ub41c \ub300\uc2e0 \uc55e\uc120 \ub450 \uc2dc\uc98c\ucc98\ub7fc \ub450 \ud30c\ud2b8\ub85c \ub098\ub204\uc9c0 \uc54a\uace0 \uae30\uc874\ucc98\ub7fc \uc5f0\uc18d\uc801\uc73c\ub85c \ubc29\uc1a1\ub418\uc5c8\ub2e4. \ud55c\ud3b8, \ub300\ud55c\ubbfc\uad6d\uc758 \uacbd\uc6b0, \uc2dc\uc98c 8 \ud504\ub9ac\ubbf8\uc5b4\ub97c \uc704\ud55c \uc6d4\ub4dc\ud22c\uc5b4\uc5d0 \uc11c\uc6b8\uc774 \ud3ec\ud568\ub418\uba74\uc11c \uc5d0\ud53c\uc18c\ub4dc 1 '\uc228\uc744 \uba48\ucdb0\ub77c Deep Breath' \uc790\ub9c9\ud310\uc774 2014\ub144 8\uc6d4 9\uc77c \uc77c\ubd80 \ud32c\ub4e4\uc5d0\uac8c \uc120\ud589 \uacf5\uac1c \ub418\uae30\ub3c4 \ud588\ub2e4. \uc774 \uc2dc\uc0ac\ud68c\ub97c \ud3ec\ud568\ud55c \uc6d4\ub4dc\ud22c\uc5b4\uc5d0 \ub300\ud55c \uc5d0\ud53c\uc18c\ub4dc\ub4e4\uc744 \ub2e4\ub8ec \ub2e4\ud050\uba58\ud130\ub9ac\ub294 2015\ub144 1\uc6d4 18\uc77c \ubc24\uc5d0 KBS2 <\uc138\uc0c1\uc758 \ubaa8\ub4e0 \ub2e4\ud050> \ud504\ub85c\uadf8\ub7a8\uc744 \ud1b5\ud574 \uc790\ub9c9 \ubc29\uc601\ub418\uc5c8\ub2e4. \ud55c\ud3b8, \uc774\uc804 \ub2e5\ud130\uc758 \ub9c8\uc9c0\ub9c9 \uc5d0\ud53c\uc18c\ub4dc\uc778 '\ub2e5\ud130\uc758 \uc2dc\uac04 the Time of the Doctor' \uc744 \ud3ec\ud568\ud55c \uc2dc\uc98c 8\uc758 \ubc29\uc601\ubd84 \ubd80\ud130\ub294 KBS\uc758 \ud3b8\uc131 \ubcc0\uacbd\uc73c\ub85c KBS 1TV \ub85c \ucc44\ub110\uc744 \uc62e\uaca8 \ubc29\uc601\ud558\uace0 \uc788\ub2e4.", "paragraph_answer": "\ucef4\ud328\ub2c8\uc5b8\uacfc \ub2e5\ud130 \ubcf8\uc778\uc774 '\ub2e5\ud130' \ub97c \ubc14\ub77c\ubcf4\ub294 \uad00\uc810\uacfc \uadf8\ub85c \uc778\ud55c \uac08\ub4f1\ub4e4\uc744 \uc911\uc810\uc801\uc73c\ub85c \ub2e4\ub8ec \uc2dc\ub9ac\uc988 8\uc740, \uae30\uc874\uc758 '13 \uc815\uaddc \uc5d0\ud53c\uc18c\ub4dc + 1 \uc2a4\ud398\uc15c' \uad6c\uc131\uc774 \uc544\ub2cc '12 \uc815\uaddc \uc5d0\ud53c\uc18c\ub4dc + 1 \uc2a4\ud398\uc15c' \uad6c\uc131\uc73c\ub85c \ucd95\uc18c\ub41c \ub300\uc2e0 \uc55e\uc120 \ub450 \uc2dc\uc98c\ucc98\ub7fc \ub450 \ud30c\ud2b8\ub85c \ub098\ub204\uc9c0 \uc54a\uace0 \uae30\uc874\ucc98\ub7fc \uc5f0\uc18d\uc801\uc73c\ub85c \ubc29\uc1a1\ub418\uc5c8\ub2e4. \ud55c\ud3b8, \ub300\ud55c\ubbfc\uad6d\uc758 \uacbd\uc6b0, \uc2dc\uc98c 8 \ud504\ub9ac\ubbf8\uc5b4\ub97c \uc704\ud55c \uc6d4\ub4dc\ud22c\uc5b4\uc5d0 \uc11c\uc6b8\uc774 \ud3ec\ud568\ub418\uba74\uc11c \uc5d0\ud53c\uc18c\ub4dc 1 '\uc228\uc744 \uba48\ucdb0\ub77c Deep Breath' \uc790\ub9c9\ud310\uc774 2014\ub144 8\uc6d4 9\uc77c \uc77c\ubd80 \ud32c\ub4e4\uc5d0\uac8c \uc120\ud589 \uacf5\uac1c \ub418\uae30\ub3c4 \ud588\ub2e4. \uc774 \uc2dc\uc0ac\ud68c\ub97c \ud3ec\ud568\ud55c \uc6d4\ub4dc\ud22c\uc5b4\uc5d0 \ub300\ud55c \uc5d0\ud53c\uc18c\ub4dc\ub4e4\uc744 \ub2e4\ub8ec \ub2e4\ud050\uba58\ud130\ub9ac\ub294 2015\ub144 1\uc6d4 18\uc77c \ubc24\uc5d0 KBS2 <\uc138\uc0c1\uc758 \ubaa8\ub4e0 \ub2e4\ud050> \ud504\ub85c\uadf8\ub7a8\uc744 \ud1b5\ud574 \uc790\ub9c9 \ubc29\uc601\ub418\uc5c8\ub2e4. \ud55c\ud3b8, \uc774\uc804 \ub2e5\ud130\uc758 \ub9c8\uc9c0\ub9c9 \uc5d0\ud53c\uc18c\ub4dc\uc778 '\ub2e5\ud130\uc758 \uc2dc\uac04 the Time of the Doctor' \uc744 \ud3ec\ud568\ud55c \uc2dc\uc98c 8\uc758 \ubc29\uc601\ubd84 \ubd80\ud130\ub294 KBS\uc758 \ud3b8\uc131 \ubcc0\uacbd\uc73c\ub85c KBS 1TV \ub85c \ucc44\ub110\uc744 \uc62e\uaca8 \ubc29\uc601\ud558\uace0 \uc788\ub2e4.", "sentence_answer": "\uc774 \uc2dc\uc0ac\ud68c\ub97c \ud3ec\ud568\ud55c \uc6d4\ub4dc\ud22c\uc5b4\uc5d0 \ub300\ud55c \uc5d0\ud53c\uc18c\ub4dc\ub4e4\uc744 \ub2e4\ub8ec \ub2e4\ud050\uba58\ud130\ub9ac\ub294 2015\ub144 1\uc6d4 18\uc77c \ubc24\uc5d0 KBS2 <\uc138\uc0c1\uc758 \ubaa8\ub4e0 \ub2e4\ud050> \ud504\ub85c\uadf8\ub7a8\uc744 \ud1b5\ud574 \uc790\ub9c9 \ubc29\uc601\ub418\uc5c8\ub2e4.", "paragraph_id": "6485131-2-1", "paragraph_question": "question: \ub2e5\ud130\ud6c4 \uc2dc\ub9ac\uc988\uc758 \uc6b0\ub7ec\ub4dc\ud22c\uc5b4\uc5d0 \ub300\ud55c \uc5d0\ud53c\uc18c\ub4dc\ub97c \ub2e4\ub8ec \ub2e4\ud050\uba58\ud130\ub9ac\ub97c \ubc29\uc601\ud55c \ub300\ud55c\ubbfc\uad6d\uc758 \ubc29\uc1a1\uad6d\uc740?, context: \ucef4\ud328\ub2c8\uc5b8\uacfc \ub2e5\ud130 \ubcf8\uc778\uc774 '\ub2e5\ud130' \ub97c \ubc14\ub77c\ubcf4\ub294 \uad00\uc810\uacfc \uadf8\ub85c \uc778\ud55c \uac08\ub4f1\ub4e4\uc744 \uc911\uc810\uc801\uc73c\ub85c \ub2e4\ub8ec \uc2dc\ub9ac\uc988 8\uc740, \uae30\uc874\uc758 '13 \uc815\uaddc \uc5d0\ud53c\uc18c\ub4dc + 1 \uc2a4\ud398\uc15c' \uad6c\uc131\uc774 \uc544\ub2cc '12 \uc815\uaddc \uc5d0\ud53c\uc18c\ub4dc + 1 \uc2a4\ud398\uc15c' \uad6c\uc131\uc73c\ub85c \ucd95\uc18c\ub41c \ub300\uc2e0 \uc55e\uc120 \ub450 \uc2dc\uc98c\ucc98\ub7fc \ub450 \ud30c\ud2b8\ub85c \ub098\ub204\uc9c0 \uc54a\uace0 \uae30\uc874\ucc98\ub7fc \uc5f0\uc18d\uc801\uc73c\ub85c \ubc29\uc1a1\ub418\uc5c8\ub2e4. \ud55c\ud3b8, \ub300\ud55c\ubbfc\uad6d\uc758 \uacbd\uc6b0, \uc2dc\uc98c 8 \ud504\ub9ac\ubbf8\uc5b4\ub97c \uc704\ud55c \uc6d4\ub4dc\ud22c\uc5b4\uc5d0 \uc11c\uc6b8\uc774 \ud3ec\ud568\ub418\uba74\uc11c \uc5d0\ud53c\uc18c\ub4dc 1 '\uc228\uc744 \uba48\ucdb0\ub77c Deep Breath' \uc790\ub9c9\ud310\uc774 2014\ub144 8\uc6d4 9\uc77c \uc77c\ubd80 \ud32c\ub4e4\uc5d0\uac8c \uc120\ud589 \uacf5\uac1c \ub418\uae30\ub3c4 \ud588\ub2e4. \uc774 \uc2dc\uc0ac\ud68c\ub97c \ud3ec\ud568\ud55c \uc6d4\ub4dc\ud22c\uc5b4\uc5d0 \ub300\ud55c \uc5d0\ud53c\uc18c\ub4dc\ub4e4\uc744 \ub2e4\ub8ec \ub2e4\ud050\uba58\ud130\ub9ac\ub294 2015\ub144 1\uc6d4 18\uc77c \ubc24\uc5d0 KBS2 <\uc138\uc0c1\uc758 \ubaa8\ub4e0 \ub2e4\ud050> \ud504\ub85c\uadf8\ub7a8\uc744 \ud1b5\ud574 \uc790\ub9c9 \ubc29\uc601\ub418\uc5c8\ub2e4. \ud55c\ud3b8, \uc774\uc804 \ub2e5\ud130\uc758 \ub9c8\uc9c0\ub9c9 \uc5d0\ud53c\uc18c\ub4dc\uc778 '\ub2e5\ud130\uc758 \uc2dc\uac04 the Time of the Doctor' \uc744 \ud3ec\ud568\ud55c \uc2dc\uc98c 8\uc758 \ubc29\uc601\ubd84 \ubd80\ud130\ub294 KBS\uc758 \ud3b8\uc131 \ubcc0\uacbd\uc73c\ub85c KBS 1TV \ub85c \ucc44\ub110\uc744 \uc62e\uaca8 \ubc29\uc601\ud558\uace0 \uc788\ub2e4."} {"question": "\uc288\ubca0\ub9b0\uc73c\ub85c \ub3cc\uc544\uc628 \ub9ac\ud558\ub974\ud2b8\uac00 \ud568\uaed8 \uc0b4\uac8c\ub41c \ub450 \uc0ac\ub78c \uc911 The Inchtaboktables\uc758 \uba64\ubc84\ub294?", "paragraph": "\uc288\ubca0\ub9b0\uc73c\ub85c \ub3cc\uc544\uc628 \ub9ac\ud558\ub974\ud2b8 \ud06c\ub8e8\uc2a4\ud398\ub294 The Inchtabokatables\uc758 \uc62c\ub9ac\ubc84 \ub9ac\ub378\uacfc Die Firma\uc758 \ud06c\ub9ac\uc2a4\ud1a0\ud504 \"\ub460\" \uc288\ub098\uc774\ub354\uc640 \ud568\uaed8 \uc0b4\uac8c\ub418\uc5c8\ub2e4. \ub450 \uc0ac\ub78c\uacfc\uc758 \ub3d9\uac70\uc911 \ud06c\ub8e8\uc2a4\ud398\ub294 \uadf8\ub3d9\uc548 \uc790\uc2e0\uc774 \ud574\uc654\ub358 \uc74c\uc545\ud65c\ub3d9\uc774 \uc2a4\uc2a4\ub85c\uc5d0\uac8c \ub9de\uc9c0 \uc54a\ub294\ub2e4\ub294 \uac83\uc744 \uae68\ub2eb\uac8c \ub41c\ub2e4. \ud06c\ub8e8\uc2a4\ud398\uac00 \uc6d0\ud558\ub294 \uac83\uc740 \uae30\uacc4\uc801\uc778 \uc18c\ub9ac\uc640 \ubb34\uac70\uc6b4 \uae30\ud0c0\uc74c\uc744 \uacb0\ud569\ud55c \uc885\ub958\uc758 \uc74c\uc545\uc774\uc5c8\ub2e4. \uc62c\ub9ac\ubc84 \ub9ac\ub378, \ud06c\ub9ac\uc2a4\ud1a0\ud504 \uc288\ub098\uc774\ub354, \uadf8\ub9ac\uace0 \ub9ac\ud558\ub974\ud2b8 \ud06c\ub8e8\uc2a4\ud398 \uc774 \uc138 \uc74c\uc545\uc778\ub4e4\uc740 \uc0c8\ub85c\uc6b4 \ubc34\ub4dc \ud504\ub85c\uc81d\ud2b8\uc5d0 \ucc29\uc218\ud558\uac8c \ub41c\ub2e4. \uae08\uc138 \ud06c\ub8e8\uc2a4\ud398\ub294 \uc791\uace1\uacfc \uc791\uc0ac\ub97c \ud55c\uaebc\ubc88\uc5d0 \ud55c\ub2e4\ub294 \uac83\uc774 \uc27d\uc9c0 \uc54a\ub2e4\ub294 \uac83\uc744 \uae68\ub2ec\uc558\uace0, \uc774\ub85c \uc778\ud574 \ud2f8 \ub9b0\ub370\ub9cc\uc774 \ub78c\uc288\ud0c0\uc778\uc5d0 \ucc38\uc5ec\ud558\uac8c \ub41c\ub2e4. \ud06c\ub8e8\uc2a4\ud398\ub294 \ud2f8 \ub9b0\ub370\ub9cc\uc774 \uc77c\ud558\ub294 \ub3c4\uc911 \ub178\ub798\ubd80\ub974\ub294 \uac83\uc744 \ub4e3\uace0 \ub9b0\ub370\ub9cc\uc758 \uc7ac\ub2a5\uc744 \uc54c\uc544\ubcf4\uc558\ub2e4\uace0 \ud55c\ub2e4.", "answer": "\uc62c\ub9ac\ubc84 \ub9ac\ub378", "sentence": "\uc288\ubca0\ub9b0\uc73c\ub85c \ub3cc\uc544\uc628 \ub9ac\ud558\ub974\ud2b8 \ud06c\ub8e8\uc2a4\ud398\ub294 The Inchtabokatables\uc758 \uc62c\ub9ac\ubc84 \ub9ac\ub378 \uacfc Die Firma\uc758 \ud06c\ub9ac\uc2a4\ud1a0\ud504 \"\ub460\" \uc288\ub098\uc774\ub354\uc640 \ud568\uaed8 \uc0b4\uac8c\ub418\uc5c8\ub2e4.", "paragraph_sentence": " \uc288\ubca0\ub9b0\uc73c\ub85c \ub3cc\uc544\uc628 \ub9ac\ud558\ub974\ud2b8 \ud06c\ub8e8\uc2a4\ud398\ub294 The Inchtabokatables\uc758 \uc62c\ub9ac\ubc84 \ub9ac\ub378 \uacfc Die Firma\uc758 \ud06c\ub9ac\uc2a4\ud1a0\ud504 \"\ub460\" \uc288\ub098\uc774\ub354\uc640 \ud568\uaed8 \uc0b4\uac8c\ub418\uc5c8\ub2e4. \ub450 \uc0ac\ub78c\uacfc\uc758 \ub3d9\uac70\uc911 \ud06c\ub8e8\uc2a4\ud398\ub294 \uadf8\ub3d9\uc548 \uc790\uc2e0\uc774 \ud574\uc654\ub358 \uc74c\uc545\ud65c\ub3d9\uc774 \uc2a4\uc2a4\ub85c\uc5d0\uac8c \ub9de\uc9c0 \uc54a\ub294\ub2e4\ub294 \uac83\uc744 \uae68\ub2eb\uac8c \ub41c\ub2e4. \ud06c\ub8e8\uc2a4\ud398\uac00 \uc6d0\ud558\ub294 \uac83\uc740 \uae30\uacc4\uc801\uc778 \uc18c\ub9ac\uc640 \ubb34\uac70\uc6b4 \uae30\ud0c0\uc74c\uc744 \uacb0\ud569\ud55c \uc885\ub958\uc758 \uc74c\uc545\uc774\uc5c8\ub2e4. \uc62c\ub9ac\ubc84 \ub9ac\ub378, \ud06c\ub9ac\uc2a4\ud1a0\ud504 \uc288\ub098\uc774\ub354, \uadf8\ub9ac\uace0 \ub9ac\ud558\ub974\ud2b8 \ud06c\ub8e8\uc2a4\ud398 \uc774 \uc138 \uc74c\uc545\uc778\ub4e4\uc740 \uc0c8\ub85c\uc6b4 \ubc34\ub4dc \ud504\ub85c\uc81d\ud2b8\uc5d0 \ucc29\uc218\ud558\uac8c \ub41c\ub2e4. \uae08\uc138 \ud06c\ub8e8\uc2a4\ud398\ub294 \uc791\uace1\uacfc \uc791\uc0ac\ub97c \ud55c\uaebc\ubc88\uc5d0 \ud55c\ub2e4\ub294 \uac83\uc774 \uc27d\uc9c0 \uc54a\ub2e4\ub294 \uac83\uc744 \uae68\ub2ec\uc558\uace0, \uc774\ub85c \uc778\ud574 \ud2f8 \ub9b0\ub370\ub9cc\uc774 \ub78c\uc288\ud0c0\uc778\uc5d0 \ucc38\uc5ec\ud558\uac8c \ub41c\ub2e4. \ud06c\ub8e8\uc2a4\ud398\ub294 \ud2f8 \ub9b0\ub370\ub9cc\uc774 \uc77c\ud558\ub294 \ub3c4\uc911 \ub178\ub798\ubd80\ub974\ub294 \uac83\uc744 \ub4e3\uace0 \ub9b0\ub370\ub9cc\uc758 \uc7ac\ub2a5\uc744 \uc54c\uc544\ubcf4\uc558\ub2e4\uace0 \ud55c\ub2e4.", "paragraph_answer": "\uc288\ubca0\ub9b0\uc73c\ub85c \ub3cc\uc544\uc628 \ub9ac\ud558\ub974\ud2b8 \ud06c\ub8e8\uc2a4\ud398\ub294 The Inchtabokatables\uc758 \uc62c\ub9ac\ubc84 \ub9ac\ub378 \uacfc Die Firma\uc758 \ud06c\ub9ac\uc2a4\ud1a0\ud504 \"\ub460\" \uc288\ub098\uc774\ub354\uc640 \ud568\uaed8 \uc0b4\uac8c\ub418\uc5c8\ub2e4. \ub450 \uc0ac\ub78c\uacfc\uc758 \ub3d9\uac70\uc911 \ud06c\ub8e8\uc2a4\ud398\ub294 \uadf8\ub3d9\uc548 \uc790\uc2e0\uc774 \ud574\uc654\ub358 \uc74c\uc545\ud65c\ub3d9\uc774 \uc2a4\uc2a4\ub85c\uc5d0\uac8c \ub9de\uc9c0 \uc54a\ub294\ub2e4\ub294 \uac83\uc744 \uae68\ub2eb\uac8c \ub41c\ub2e4. \ud06c\ub8e8\uc2a4\ud398\uac00 \uc6d0\ud558\ub294 \uac83\uc740 \uae30\uacc4\uc801\uc778 \uc18c\ub9ac\uc640 \ubb34\uac70\uc6b4 \uae30\ud0c0\uc74c\uc744 \uacb0\ud569\ud55c \uc885\ub958\uc758 \uc74c\uc545\uc774\uc5c8\ub2e4. \uc62c\ub9ac\ubc84 \ub9ac\ub378, \ud06c\ub9ac\uc2a4\ud1a0\ud504 \uc288\ub098\uc774\ub354, \uadf8\ub9ac\uace0 \ub9ac\ud558\ub974\ud2b8 \ud06c\ub8e8\uc2a4\ud398 \uc774 \uc138 \uc74c\uc545\uc778\ub4e4\uc740 \uc0c8\ub85c\uc6b4 \ubc34\ub4dc \ud504\ub85c\uc81d\ud2b8\uc5d0 \ucc29\uc218\ud558\uac8c \ub41c\ub2e4. \uae08\uc138 \ud06c\ub8e8\uc2a4\ud398\ub294 \uc791\uace1\uacfc \uc791\uc0ac\ub97c \ud55c\uaebc\ubc88\uc5d0 \ud55c\ub2e4\ub294 \uac83\uc774 \uc27d\uc9c0 \uc54a\ub2e4\ub294 \uac83\uc744 \uae68\ub2ec\uc558\uace0, \uc774\ub85c \uc778\ud574 \ud2f8 \ub9b0\ub370\ub9cc\uc774 \ub78c\uc288\ud0c0\uc778\uc5d0 \ucc38\uc5ec\ud558\uac8c \ub41c\ub2e4. \ud06c\ub8e8\uc2a4\ud398\ub294 \ud2f8 \ub9b0\ub370\ub9cc\uc774 \uc77c\ud558\ub294 \ub3c4\uc911 \ub178\ub798\ubd80\ub974\ub294 \uac83\uc744 \ub4e3\uace0 \ub9b0\ub370\ub9cc\uc758 \uc7ac\ub2a5\uc744 \uc54c\uc544\ubcf4\uc558\ub2e4\uace0 \ud55c\ub2e4.", "sentence_answer": "\uc288\ubca0\ub9b0\uc73c\ub85c \ub3cc\uc544\uc628 \ub9ac\ud558\ub974\ud2b8 \ud06c\ub8e8\uc2a4\ud398\ub294 The Inchtabokatables\uc758 \uc62c\ub9ac\ubc84 \ub9ac\ub378 \uacfc Die Firma\uc758 \ud06c\ub9ac\uc2a4\ud1a0\ud504 \"\ub460\" \uc288\ub098\uc774\ub354\uc640 \ud568\uaed8 \uc0b4\uac8c\ub418\uc5c8\ub2e4.", "paragraph_id": "6534087-0-0", "paragraph_question": "question: \uc288\ubca0\ub9b0\uc73c\ub85c \ub3cc\uc544\uc628 \ub9ac\ud558\ub974\ud2b8\uac00 \ud568\uaed8 \uc0b4\uac8c\ub41c \ub450 \uc0ac\ub78c \uc911 The Inchtaboktables\uc758 \uba64\ubc84\ub294?, context: \uc288\ubca0\ub9b0\uc73c\ub85c \ub3cc\uc544\uc628 \ub9ac\ud558\ub974\ud2b8 \ud06c\ub8e8\uc2a4\ud398\ub294 The Inchtabokatables\uc758 \uc62c\ub9ac\ubc84 \ub9ac\ub378\uacfc Die Firma\uc758 \ud06c\ub9ac\uc2a4\ud1a0\ud504 \"\ub460\" \uc288\ub098\uc774\ub354\uc640 \ud568\uaed8 \uc0b4\uac8c\ub418\uc5c8\ub2e4. \ub450 \uc0ac\ub78c\uacfc\uc758 \ub3d9\uac70\uc911 \ud06c\ub8e8\uc2a4\ud398\ub294 \uadf8\ub3d9\uc548 \uc790\uc2e0\uc774 \ud574\uc654\ub358 \uc74c\uc545\ud65c\ub3d9\uc774 \uc2a4\uc2a4\ub85c\uc5d0\uac8c \ub9de\uc9c0 \uc54a\ub294\ub2e4\ub294 \uac83\uc744 \uae68\ub2eb\uac8c \ub41c\ub2e4. \ud06c\ub8e8\uc2a4\ud398\uac00 \uc6d0\ud558\ub294 \uac83\uc740 \uae30\uacc4\uc801\uc778 \uc18c\ub9ac\uc640 \ubb34\uac70\uc6b4 \uae30\ud0c0\uc74c\uc744 \uacb0\ud569\ud55c \uc885\ub958\uc758 \uc74c\uc545\uc774\uc5c8\ub2e4. \uc62c\ub9ac\ubc84 \ub9ac\ub378, \ud06c\ub9ac\uc2a4\ud1a0\ud504 \uc288\ub098\uc774\ub354, \uadf8\ub9ac\uace0 \ub9ac\ud558\ub974\ud2b8 \ud06c\ub8e8\uc2a4\ud398 \uc774 \uc138 \uc74c\uc545\uc778\ub4e4\uc740 \uc0c8\ub85c\uc6b4 \ubc34\ub4dc \ud504\ub85c\uc81d\ud2b8\uc5d0 \ucc29\uc218\ud558\uac8c \ub41c\ub2e4. \uae08\uc138 \ud06c\ub8e8\uc2a4\ud398\ub294 \uc791\uace1\uacfc \uc791\uc0ac\ub97c \ud55c\uaebc\ubc88\uc5d0 \ud55c\ub2e4\ub294 \uac83\uc774 \uc27d\uc9c0 \uc54a\ub2e4\ub294 \uac83\uc744 \uae68\ub2ec\uc558\uace0, \uc774\ub85c \uc778\ud574 \ud2f8 \ub9b0\ub370\ub9cc\uc774 \ub78c\uc288\ud0c0\uc778\uc5d0 \ucc38\uc5ec\ud558\uac8c \ub41c\ub2e4. \ud06c\ub8e8\uc2a4\ud398\ub294 \ud2f8 \ub9b0\ub370\ub9cc\uc774 \uc77c\ud558\ub294 \ub3c4\uc911 \ub178\ub798\ubd80\ub974\ub294 \uac83\uc744 \ub4e3\uace0 \ub9b0\ub370\ub9cc\uc758 \uc7ac\ub2a5\uc744 \uc54c\uc544\ubcf4\uc558\ub2e4\uace0 \ud55c\ub2e4."} {"question": "\ub9e8 \uc624\ube0c \ub354 \ub9e4\uce58\ub97c \uc218\uc0c1\ud55c \ub54c\ub294?", "paragraph": "2009\ub144 7\uc6d4 1\uc77c \ud300\uacfc \uc7ac\uacc4\uc57d\uc744 \uccb4\uacb0\ud558\uba70 \uc7a5\uae30\uacc4\uc57d\uc744 \ub9fa\uc5c8\ub2e4. 8\uc6d4 22\uc77c \ud3ec\uce20\uba38\uc2a4 \ub370\uc774\ube44\ub4dc \uc81c\uc784\uc2a4 \uace8\ud0a4\ud37c\ub97c \uc0c1\ub300\ub85c 1:1 \uc0c1\ud669\uc5d0\uc11c \uae54\ub054\ud558\uac8c \ub9c8\ubb34\ub9ac \uc9c0\uc73c\uba70 \ub9ac\uadf8 \uccab \uace8\uc744 \ub123\uc5b4 \ud300\uc758 4:1 \uc2b9\ub9ac\uc5d0 \uae30\uc5ec\ud588\ub2e4. \uc774\uc5b4 \uc5d0\ubbf8\ub808\uc774\uce20 \uc2a4\ud0c0\ub514\uc6c0\uc5d0\uc11c \uc5f4\ub9b0 \ucc54\ud53c\uc5b8\uc2a4\ub9ac\uadf8 \ucd5c\uc885\uc608\uc120 \uc140\ud2f1\uc804\uc5d0\uc11c \uc548\ub4dc\ub808\uc774 \uc544\ub974\uc0e4\ube48\uc758 \uace8\uc5d0 \uc5b4\uc2dc\uc2a4\ud2b8\ub97c \uae30\ub85d\ud574 \ud300\uc758 3:1 \uc2b9\ub9ac\uc5d0 \ud568\uaed8\ud588\ub2e4. 10\uc6d4 28\uc77c \ub9ac\uadf8\ucef5 4\ub77c\uc6b4\ub4dc \ub9ac\ubc84\ud480\uc804\uc5d0\uc11c \ub2c8\ud074\ub77c\uc2a4 \ubca4\ud2b8\ub108\uc758 \uace8\uc5d0 \uc5b4\uc2dc\uc2a4\ud2b8\ub97c \uae30\ub85d\ud558\uba70 \ub9e8 \uc624\ube0c \ub354 \ub9e4\uce58\ub97c \uc218\uc0c1\ud588\ub2e4. \uc2dc\uc98c \uccab \ub9ac\uadf8 \uc120\ubc1c \ucd9c\uc7a5\uc740 4:1\ub85c \uc2b9\ub9ac\ud55c 11\uc6d4 7\uc77c \uc6b8\ubc84\ud584\ud2bc \uc6d0\ub354\ub7ec\uc2a4 \uc6d0\uc815\uacbd\uae30\uc5d0\uc11c \ud480\ud0c0\uc784 \ucd9c\uc804\ud558\uba70 \uae30\ub85d\ud55c\ub2e4. \uacbd\uae30 4\uc77c \ud6c4 \uc62c\ud574\uc758 \uc6e8\uc77c\uc2a4 \uc5b4\ub9b0 \uc120\uc218 \uc0c1\uc744 \uc218\uc0c1\ud588\ub2e4. 12\uc6d4 9\uc77c \uc2a4\ud1a0\ud06c \uc2dc\ud2f0\ub97c \uc0c1\ub300\ub85c \uc2dc\uc98c \ub450 \ubc88\uc9f8 \ub9ac\uadf8\uace8\uc744 \uae30\ub85d\ud558\uba70 \ud300\uc740 \uae54\ub054\ud55c 2:0 \uc2b9\ub9ac\ub97c \uae30\ub85d\ud55c\ub2e4. 12\uc6d4 30\uc77c \uc2dc\uc98c 3\ubc88\uc9f8 \uace8\uc744 \ub123\uace0 \ud558\ub098\uc758 \uc5b4\uc2dc\uc2a4\ud2b8 \uc5ed\uc2dc \uae30\ub85d\ud558\uba70 \ud3ec\uce20\uba38\uc2a4\ub97c \uc0c1\ub300\ub85c \ud55c 4:1 \uc2b9\ub9ac\uc5d0 \uc5ed\ud560\ud588\ub2e4. 2010\ub144 1\uc6d4 3\uc77c FA\ucef5 3\ub77c\uc6b4\ub4dc \uc5c5\ud2bc\ud30c\ud06c\uc5d0\uc11c \uc5f4\ub9b0 \uc6e8\uc2a4\ud2b8\ud584 \uc6d0\uc815\uacbd\uae30\uc5d0\uc11c 78\ubd84 \ub3d9\uc810\uace8\uc744 \uae30\ub85d\ud558\uba70 83\ubd84 \uc5d0\ub450\uc544\ub974\ub450 \ub2e4 \uc2e4\ubc14\uac00 \ud130\ud2b8\ub9b0 \uacb0\uc2b9 \ud5e4\ub529 \uace8\ub85c \uc5ed\uc804\uc2b9\uc744 \ub9cc\ub4e4\uc5b4\ub0b8\ub2e4. \uc774 \uacbd\uae30 MVP\ub85c \uc120\uc815\ub410\uc73c\uba70 \uc544\ub974\uc13c \ubcb5\uac70 \uac10\ub3c5\uc740 \uacbd\uae30 \ud6c4 \"\uacf5\uaca9\uc801\uc778 \ub85c\uc774 \ud0a8, \uc62c\ub77c\uc6b4\ub4dc \ud50c\ub808\uc774\uc5b4\ub85c \ud658\uc0c1\uc801\uc778 \ud65c\ub3d9\ub7c9\uc744 \uac00\uc84c\ub2e4\"\ub294 \ucc2c\uc0ac\ub97c \ud588\ub2e4. 2\uc6d4 27\uc77c \ube0c\ub9ac\ud0c0\ub2c8\uc544 \uc2a4\ud0c0\ub514\uc6c0\uc5d0\uc11c \uc5f4\ub9b0 \uc2a4\ud1a0\ud06c \uc2dc\ud2f0\uc640\uc758 \uacbd\uae30\uc5d0\uc11c \uc2a4\ud1a0\ud06c \uc218\ube44\uc218 \ub77c\uc774\uc5b8 \uc1fc\ud06c\ub85c\uc2a4\uc758 \ud0dc\ud074\ub85c \uc624\ub978\ucabd \ub2e4\ub9ac\uac00 \uc774\uc911\uc73c\ub85c \uace8\uc808\ub418\ub294 \ub054\ucc0d\ud55c \ubd80\uc0c1\uc744 \ub2f9\ud588\ub2e4. \uc989\uc2dc \ubcd1\uc6d0\uc73c\ub85c \uc62e\uaca8\uc84c\uc73c\ub098 \uc815\uac15\uc774\ubf08\uc640 \uc885\uc544\ub9ac\ubf08 \uc5ed\uc2dc \uace8\uc808\ub41c \uc0ac\uc2e4\uc774 \uc54c\ub824\uc84c\uc73c\uba70 \ubcf5\uadc0 \ub0a0\uc9dc \uc5ed\uc2dc \ubd88\ud22c\uba85\ud560 \uc815\ub3c4\uc758 \uc2ec\uac01\ud55c \ubd80\uc0c1\uc774\uc600\ub2e4. \ub2e4\ud589\ud788\ub3c4 2010\ub144 3\uc6d4 20\uc77c \ubaa9\ubc1c \uc5c6\uc774 \uac78\uc5b4\ub2e4\ub2c8\uae30 \uc2dc\uc791\ud574 10\uc6d4 \ud6c8\ub828\uc5d0 \ubcf5\uadc0\ud558\uc600\uc73c\uba70 11\uc6d4 23\uc77c \uc544\uc2a4\ub110\uc758 \ucf5c\ub2c8 \ud6c8\ub828\uc7a5\uc5d0\uc11c \uc5f4\ub9b0 \ub9ac\uc800\ube0c \ud300 \uacbd\uae30, \uc6b8\ubc84\ud584\ud2bc \uc6d0\ub354\ub7ec\uc2a4\uc640\uc758 \uacbd\uae30\uc5d0\uc11c \ubcf5\uadc0\uc804\uc744 \uce58\ub918\ub2e4. 2010\ub144 6\uc6d4 1\uc77c\uc5d0\ub294 \uc7ac\uacc4\uc57d\uc744 \uccb4\uacb0\ud558\uba70 \uc0c8\ub85c\uc6b4 \uc7a5\uae30\uacc4\uc57d\uc744 \ub9fa\uc5c8\ub2e4.", "answer": "10\uc6d4 28\uc77c", "sentence": "10\uc6d4 28\uc77c \ub9ac\uadf8\ucef5 4\ub77c\uc6b4\ub4dc \ub9ac\ubc84\ud480\uc804\uc5d0\uc11c \ub2c8\ud074\ub77c\uc2a4 \ubca4\ud2b8\ub108\uc758 \uace8\uc5d0 \uc5b4\uc2dc\uc2a4\ud2b8\ub97c \uae30\ub85d\ud558\uba70 \ub9e8 \uc624\ube0c \ub354 \ub9e4\uce58\ub97c \uc218\uc0c1\ud588\ub2e4.", "paragraph_sentence": "2009\ub144 7\uc6d4 1\uc77c \ud300\uacfc \uc7ac\uacc4\uc57d\uc744 \uccb4\uacb0\ud558\uba70 \uc7a5\uae30\uacc4\uc57d\uc744 \ub9fa\uc5c8\ub2e4. 8\uc6d4 22\uc77c \ud3ec\uce20\uba38\uc2a4 \ub370\uc774\ube44\ub4dc \uc81c\uc784\uc2a4 \uace8\ud0a4\ud37c\ub97c \uc0c1\ub300\ub85c 1:1 \uc0c1\ud669\uc5d0\uc11c \uae54\ub054\ud558\uac8c \ub9c8\ubb34\ub9ac \uc9c0\uc73c\uba70 \ub9ac\uadf8 \uccab \uace8\uc744 \ub123\uc5b4 \ud300\uc758 4:1 \uc2b9\ub9ac\uc5d0 \uae30\uc5ec\ud588\ub2e4. \uc774\uc5b4 \uc5d0\ubbf8\ub808\uc774\uce20 \uc2a4\ud0c0\ub514\uc6c0\uc5d0\uc11c \uc5f4\ub9b0 \ucc54\ud53c\uc5b8\uc2a4\ub9ac\uadf8 \ucd5c\uc885\uc608\uc120 \uc140\ud2f1\uc804\uc5d0\uc11c \uc548\ub4dc\ub808\uc774 \uc544\ub974\uc0e4\ube48\uc758 \uace8\uc5d0 \uc5b4\uc2dc\uc2a4\ud2b8\ub97c \uae30\ub85d\ud574 \ud300\uc758 3:1 \uc2b9\ub9ac\uc5d0 \ud568\uaed8\ud588\ub2e4. 10\uc6d4 28\uc77c \ub9ac\uadf8\ucef5 4\ub77c\uc6b4\ub4dc \ub9ac\ubc84\ud480\uc804\uc5d0\uc11c \ub2c8\ud074\ub77c\uc2a4 \ubca4\ud2b8\ub108\uc758 \uace8\uc5d0 \uc5b4\uc2dc\uc2a4\ud2b8\ub97c \uae30\ub85d\ud558\uba70 \ub9e8 \uc624\ube0c \ub354 \ub9e4\uce58\ub97c \uc218\uc0c1\ud588\ub2e4. \uc2dc\uc98c \uccab \ub9ac\uadf8 \uc120\ubc1c \ucd9c\uc7a5\uc740 4:1\ub85c \uc2b9\ub9ac\ud55c 11\uc6d4 7\uc77c \uc6b8\ubc84\ud584\ud2bc \uc6d0\ub354\ub7ec\uc2a4 \uc6d0\uc815\uacbd\uae30\uc5d0\uc11c \ud480\ud0c0\uc784 \ucd9c\uc804\ud558\uba70 \uae30\ub85d\ud55c\ub2e4. \uacbd\uae30 4\uc77c \ud6c4 \uc62c\ud574\uc758 \uc6e8\uc77c\uc2a4 \uc5b4\ub9b0 \uc120\uc218 \uc0c1\uc744 \uc218\uc0c1\ud588\ub2e4. 12\uc6d4 9\uc77c \uc2a4\ud1a0\ud06c \uc2dc\ud2f0\ub97c \uc0c1\ub300\ub85c \uc2dc\uc98c \ub450 \ubc88\uc9f8 \ub9ac\uadf8\uace8\uc744 \uae30\ub85d\ud558\uba70 \ud300\uc740 \uae54\ub054\ud55c 2:0 \uc2b9\ub9ac\ub97c \uae30\ub85d\ud55c\ub2e4. 12\uc6d4 30\uc77c \uc2dc\uc98c 3\ubc88\uc9f8 \uace8\uc744 \ub123\uace0 \ud558\ub098\uc758 \uc5b4\uc2dc\uc2a4\ud2b8 \uc5ed\uc2dc \uae30\ub85d\ud558\uba70 \ud3ec\uce20\uba38\uc2a4\ub97c \uc0c1\ub300\ub85c \ud55c 4:1 \uc2b9\ub9ac\uc5d0 \uc5ed\ud560\ud588\ub2e4. 2010\ub144 1\uc6d4 3\uc77c FA\ucef5 3\ub77c\uc6b4\ub4dc \uc5c5\ud2bc\ud30c\ud06c\uc5d0\uc11c \uc5f4\ub9b0 \uc6e8\uc2a4\ud2b8\ud584 \uc6d0\uc815\uacbd\uae30\uc5d0\uc11c 78\ubd84 \ub3d9\uc810\uace8\uc744 \uae30\ub85d\ud558\uba70 83\ubd84 \uc5d0\ub450\uc544\ub974\ub450 \ub2e4 \uc2e4\ubc14\uac00 \ud130\ud2b8\ub9b0 \uacb0\uc2b9 \ud5e4\ub529 \uace8\ub85c \uc5ed\uc804\uc2b9\uc744 \ub9cc\ub4e4\uc5b4\ub0b8\ub2e4. \uc774 \uacbd\uae30 MVP\ub85c \uc120\uc815\ub410\uc73c\uba70 \uc544\ub974\uc13c \ubcb5\uac70 \uac10\ub3c5\uc740 \uacbd\uae30 \ud6c4 \"\uacf5\uaca9\uc801\uc778 \ub85c\uc774 \ud0a8, \uc62c\ub77c\uc6b4\ub4dc \ud50c\ub808\uc774\uc5b4\ub85c \ud658\uc0c1\uc801\uc778 \ud65c\ub3d9\ub7c9\uc744 \uac00\uc84c\ub2e4\"\ub294 \ucc2c\uc0ac\ub97c \ud588\ub2e4. 2\uc6d4 27\uc77c \ube0c\ub9ac\ud0c0\ub2c8\uc544 \uc2a4\ud0c0\ub514\uc6c0\uc5d0\uc11c \uc5f4\ub9b0 \uc2a4\ud1a0\ud06c \uc2dc\ud2f0\uc640\uc758 \uacbd\uae30\uc5d0\uc11c \uc2a4\ud1a0\ud06c \uc218\ube44\uc218 \ub77c\uc774\uc5b8 \uc1fc\ud06c\ub85c\uc2a4\uc758 \ud0dc\ud074\ub85c \uc624\ub978\ucabd \ub2e4\ub9ac\uac00 \uc774\uc911\uc73c\ub85c \uace8\uc808\ub418\ub294 \ub054\ucc0d\ud55c \ubd80\uc0c1\uc744 \ub2f9\ud588\ub2e4. \uc989\uc2dc \ubcd1\uc6d0\uc73c\ub85c \uc62e\uaca8\uc84c\uc73c\ub098 \uc815\uac15\uc774\ubf08\uc640 \uc885\uc544\ub9ac\ubf08 \uc5ed\uc2dc \uace8\uc808\ub41c \uc0ac\uc2e4\uc774 \uc54c\ub824\uc84c\uc73c\uba70 \ubcf5\uadc0 \ub0a0\uc9dc \uc5ed\uc2dc \ubd88\ud22c\uba85\ud560 \uc815\ub3c4\uc758 \uc2ec\uac01\ud55c \ubd80\uc0c1\uc774\uc600\ub2e4. \ub2e4\ud589\ud788\ub3c4 2010\ub144 3\uc6d4 20\uc77c \ubaa9\ubc1c \uc5c6\uc774 \uac78\uc5b4\ub2e4\ub2c8\uae30 \uc2dc\uc791\ud574 10\uc6d4 \ud6c8\ub828\uc5d0 \ubcf5\uadc0\ud558\uc600\uc73c\uba70 11\uc6d4 23\uc77c \uc544\uc2a4\ub110\uc758 \ucf5c\ub2c8 \ud6c8\ub828\uc7a5\uc5d0\uc11c \uc5f4\ub9b0 \ub9ac\uc800\ube0c \ud300 \uacbd\uae30, \uc6b8\ubc84\ud584\ud2bc \uc6d0\ub354\ub7ec\uc2a4\uc640\uc758 \uacbd\uae30\uc5d0\uc11c \ubcf5\uadc0\uc804\uc744 \uce58\ub918\ub2e4. 2010\ub144 6\uc6d4 1\uc77c\uc5d0\ub294 \uc7ac\uacc4\uc57d\uc744 \uccb4\uacb0\ud558\uba70 \uc0c8\ub85c\uc6b4 \uc7a5\uae30\uacc4\uc57d\uc744 \ub9fa\uc5c8\ub2e4.", "paragraph_answer": "2009\ub144 7\uc6d4 1\uc77c \ud300\uacfc \uc7ac\uacc4\uc57d\uc744 \uccb4\uacb0\ud558\uba70 \uc7a5\uae30\uacc4\uc57d\uc744 \ub9fa\uc5c8\ub2e4. 8\uc6d4 22\uc77c \ud3ec\uce20\uba38\uc2a4 \ub370\uc774\ube44\ub4dc \uc81c\uc784\uc2a4 \uace8\ud0a4\ud37c\ub97c \uc0c1\ub300\ub85c 1:1 \uc0c1\ud669\uc5d0\uc11c \uae54\ub054\ud558\uac8c \ub9c8\ubb34\ub9ac \uc9c0\uc73c\uba70 \ub9ac\uadf8 \uccab \uace8\uc744 \ub123\uc5b4 \ud300\uc758 4:1 \uc2b9\ub9ac\uc5d0 \uae30\uc5ec\ud588\ub2e4. \uc774\uc5b4 \uc5d0\ubbf8\ub808\uc774\uce20 \uc2a4\ud0c0\ub514\uc6c0\uc5d0\uc11c \uc5f4\ub9b0 \ucc54\ud53c\uc5b8\uc2a4\ub9ac\uadf8 \ucd5c\uc885\uc608\uc120 \uc140\ud2f1\uc804\uc5d0\uc11c \uc548\ub4dc\ub808\uc774 \uc544\ub974\uc0e4\ube48\uc758 \uace8\uc5d0 \uc5b4\uc2dc\uc2a4\ud2b8\ub97c \uae30\ub85d\ud574 \ud300\uc758 3:1 \uc2b9\ub9ac\uc5d0 \ud568\uaed8\ud588\ub2e4. 10\uc6d4 28\uc77c \ub9ac\uadf8\ucef5 4\ub77c\uc6b4\ub4dc \ub9ac\ubc84\ud480\uc804\uc5d0\uc11c \ub2c8\ud074\ub77c\uc2a4 \ubca4\ud2b8\ub108\uc758 \uace8\uc5d0 \uc5b4\uc2dc\uc2a4\ud2b8\ub97c \uae30\ub85d\ud558\uba70 \ub9e8 \uc624\ube0c \ub354 \ub9e4\uce58\ub97c \uc218\uc0c1\ud588\ub2e4. \uc2dc\uc98c \uccab \ub9ac\uadf8 \uc120\ubc1c \ucd9c\uc7a5\uc740 4:1\ub85c \uc2b9\ub9ac\ud55c 11\uc6d4 7\uc77c \uc6b8\ubc84\ud584\ud2bc \uc6d0\ub354\ub7ec\uc2a4 \uc6d0\uc815\uacbd\uae30\uc5d0\uc11c \ud480\ud0c0\uc784 \ucd9c\uc804\ud558\uba70 \uae30\ub85d\ud55c\ub2e4. \uacbd\uae30 4\uc77c \ud6c4 \uc62c\ud574\uc758 \uc6e8\uc77c\uc2a4 \uc5b4\ub9b0 \uc120\uc218 \uc0c1\uc744 \uc218\uc0c1\ud588\ub2e4. 12\uc6d4 9\uc77c \uc2a4\ud1a0\ud06c \uc2dc\ud2f0\ub97c \uc0c1\ub300\ub85c \uc2dc\uc98c \ub450 \ubc88\uc9f8 \ub9ac\uadf8\uace8\uc744 \uae30\ub85d\ud558\uba70 \ud300\uc740 \uae54\ub054\ud55c 2:0 \uc2b9\ub9ac\ub97c \uae30\ub85d\ud55c\ub2e4. 12\uc6d4 30\uc77c \uc2dc\uc98c 3\ubc88\uc9f8 \uace8\uc744 \ub123\uace0 \ud558\ub098\uc758 \uc5b4\uc2dc\uc2a4\ud2b8 \uc5ed\uc2dc \uae30\ub85d\ud558\uba70 \ud3ec\uce20\uba38\uc2a4\ub97c \uc0c1\ub300\ub85c \ud55c 4:1 \uc2b9\ub9ac\uc5d0 \uc5ed\ud560\ud588\ub2e4. 2010\ub144 1\uc6d4 3\uc77c FA\ucef5 3\ub77c\uc6b4\ub4dc \uc5c5\ud2bc\ud30c\ud06c\uc5d0\uc11c \uc5f4\ub9b0 \uc6e8\uc2a4\ud2b8\ud584 \uc6d0\uc815\uacbd\uae30\uc5d0\uc11c 78\ubd84 \ub3d9\uc810\uace8\uc744 \uae30\ub85d\ud558\uba70 83\ubd84 \uc5d0\ub450\uc544\ub974\ub450 \ub2e4 \uc2e4\ubc14\uac00 \ud130\ud2b8\ub9b0 \uacb0\uc2b9 \ud5e4\ub529 \uace8\ub85c \uc5ed\uc804\uc2b9\uc744 \ub9cc\ub4e4\uc5b4\ub0b8\ub2e4. \uc774 \uacbd\uae30 MVP\ub85c \uc120\uc815\ub410\uc73c\uba70 \uc544\ub974\uc13c \ubcb5\uac70 \uac10\ub3c5\uc740 \uacbd\uae30 \ud6c4 \"\uacf5\uaca9\uc801\uc778 \ub85c\uc774 \ud0a8, \uc62c\ub77c\uc6b4\ub4dc \ud50c\ub808\uc774\uc5b4\ub85c \ud658\uc0c1\uc801\uc778 \ud65c\ub3d9\ub7c9\uc744 \uac00\uc84c\ub2e4\"\ub294 \ucc2c\uc0ac\ub97c \ud588\ub2e4. 2\uc6d4 27\uc77c \ube0c\ub9ac\ud0c0\ub2c8\uc544 \uc2a4\ud0c0\ub514\uc6c0\uc5d0\uc11c \uc5f4\ub9b0 \uc2a4\ud1a0\ud06c \uc2dc\ud2f0\uc640\uc758 \uacbd\uae30\uc5d0\uc11c \uc2a4\ud1a0\ud06c \uc218\ube44\uc218 \ub77c\uc774\uc5b8 \uc1fc\ud06c\ub85c\uc2a4\uc758 \ud0dc\ud074\ub85c \uc624\ub978\ucabd \ub2e4\ub9ac\uac00 \uc774\uc911\uc73c\ub85c \uace8\uc808\ub418\ub294 \ub054\ucc0d\ud55c \ubd80\uc0c1\uc744 \ub2f9\ud588\ub2e4. \uc989\uc2dc \ubcd1\uc6d0\uc73c\ub85c \uc62e\uaca8\uc84c\uc73c\ub098 \uc815\uac15\uc774\ubf08\uc640 \uc885\uc544\ub9ac\ubf08 \uc5ed\uc2dc \uace8\uc808\ub41c \uc0ac\uc2e4\uc774 \uc54c\ub824\uc84c\uc73c\uba70 \ubcf5\uadc0 \ub0a0\uc9dc \uc5ed\uc2dc \ubd88\ud22c\uba85\ud560 \uc815\ub3c4\uc758 \uc2ec\uac01\ud55c \ubd80\uc0c1\uc774\uc600\ub2e4. \ub2e4\ud589\ud788\ub3c4 2010\ub144 3\uc6d4 20\uc77c \ubaa9\ubc1c \uc5c6\uc774 \uac78\uc5b4\ub2e4\ub2c8\uae30 \uc2dc\uc791\ud574 10\uc6d4 \ud6c8\ub828\uc5d0 \ubcf5\uadc0\ud558\uc600\uc73c\uba70 11\uc6d4 23\uc77c \uc544\uc2a4\ub110\uc758 \ucf5c\ub2c8 \ud6c8\ub828\uc7a5\uc5d0\uc11c \uc5f4\ub9b0 \ub9ac\uc800\ube0c \ud300 \uacbd\uae30, \uc6b8\ubc84\ud584\ud2bc \uc6d0\ub354\ub7ec\uc2a4\uc640\uc758 \uacbd\uae30\uc5d0\uc11c \ubcf5\uadc0\uc804\uc744 \uce58\ub918\ub2e4. 2010\ub144 6\uc6d4 1\uc77c\uc5d0\ub294 \uc7ac\uacc4\uc57d\uc744 \uccb4\uacb0\ud558\uba70 \uc0c8\ub85c\uc6b4 \uc7a5\uae30\uacc4\uc57d\uc744 \ub9fa\uc5c8\ub2e4.", "sentence_answer": " 10\uc6d4 28\uc77c \ub9ac\uadf8\ucef5 4\ub77c\uc6b4\ub4dc \ub9ac\ubc84\ud480\uc804\uc5d0\uc11c \ub2c8\ud074\ub77c\uc2a4 \ubca4\ud2b8\ub108\uc758 \uace8\uc5d0 \uc5b4\uc2dc\uc2a4\ud2b8\ub97c \uae30\ub85d\ud558\uba70 \ub9e8 \uc624\ube0c \ub354 \ub9e4\uce58\ub97c \uc218\uc0c1\ud588\ub2e4.", "paragraph_id": "6586552-2-2", "paragraph_question": "question: \ub9e8 \uc624\ube0c \ub354 \ub9e4\uce58\ub97c \uc218\uc0c1\ud55c \ub54c\ub294?, context: 2009\ub144 7\uc6d4 1\uc77c \ud300\uacfc \uc7ac\uacc4\uc57d\uc744 \uccb4\uacb0\ud558\uba70 \uc7a5\uae30\uacc4\uc57d\uc744 \ub9fa\uc5c8\ub2e4. 8\uc6d4 22\uc77c \ud3ec\uce20\uba38\uc2a4 \ub370\uc774\ube44\ub4dc \uc81c\uc784\uc2a4 \uace8\ud0a4\ud37c\ub97c \uc0c1\ub300\ub85c 1:1 \uc0c1\ud669\uc5d0\uc11c \uae54\ub054\ud558\uac8c \ub9c8\ubb34\ub9ac \uc9c0\uc73c\uba70 \ub9ac\uadf8 \uccab \uace8\uc744 \ub123\uc5b4 \ud300\uc758 4:1 \uc2b9\ub9ac\uc5d0 \uae30\uc5ec\ud588\ub2e4. \uc774\uc5b4 \uc5d0\ubbf8\ub808\uc774\uce20 \uc2a4\ud0c0\ub514\uc6c0\uc5d0\uc11c \uc5f4\ub9b0 \ucc54\ud53c\uc5b8\uc2a4\ub9ac\uadf8 \ucd5c\uc885\uc608\uc120 \uc140\ud2f1\uc804\uc5d0\uc11c \uc548\ub4dc\ub808\uc774 \uc544\ub974\uc0e4\ube48\uc758 \uace8\uc5d0 \uc5b4\uc2dc\uc2a4\ud2b8\ub97c \uae30\ub85d\ud574 \ud300\uc758 3:1 \uc2b9\ub9ac\uc5d0 \ud568\uaed8\ud588\ub2e4. 10\uc6d4 28\uc77c \ub9ac\uadf8\ucef5 4\ub77c\uc6b4\ub4dc \ub9ac\ubc84\ud480\uc804\uc5d0\uc11c \ub2c8\ud074\ub77c\uc2a4 \ubca4\ud2b8\ub108\uc758 \uace8\uc5d0 \uc5b4\uc2dc\uc2a4\ud2b8\ub97c \uae30\ub85d\ud558\uba70 \ub9e8 \uc624\ube0c \ub354 \ub9e4\uce58\ub97c \uc218\uc0c1\ud588\ub2e4. \uc2dc\uc98c \uccab \ub9ac\uadf8 \uc120\ubc1c \ucd9c\uc7a5\uc740 4:1\ub85c \uc2b9\ub9ac\ud55c 11\uc6d4 7\uc77c \uc6b8\ubc84\ud584\ud2bc \uc6d0\ub354\ub7ec\uc2a4 \uc6d0\uc815\uacbd\uae30\uc5d0\uc11c \ud480\ud0c0\uc784 \ucd9c\uc804\ud558\uba70 \uae30\ub85d\ud55c\ub2e4. \uacbd\uae30 4\uc77c \ud6c4 \uc62c\ud574\uc758 \uc6e8\uc77c\uc2a4 \uc5b4\ub9b0 \uc120\uc218 \uc0c1\uc744 \uc218\uc0c1\ud588\ub2e4. 12\uc6d4 9\uc77c \uc2a4\ud1a0\ud06c \uc2dc\ud2f0\ub97c \uc0c1\ub300\ub85c \uc2dc\uc98c \ub450 \ubc88\uc9f8 \ub9ac\uadf8\uace8\uc744 \uae30\ub85d\ud558\uba70 \ud300\uc740 \uae54\ub054\ud55c 2:0 \uc2b9\ub9ac\ub97c \uae30\ub85d\ud55c\ub2e4. 12\uc6d4 30\uc77c \uc2dc\uc98c 3\ubc88\uc9f8 \uace8\uc744 \ub123\uace0 \ud558\ub098\uc758 \uc5b4\uc2dc\uc2a4\ud2b8 \uc5ed\uc2dc \uae30\ub85d\ud558\uba70 \ud3ec\uce20\uba38\uc2a4\ub97c \uc0c1\ub300\ub85c \ud55c 4:1 \uc2b9\ub9ac\uc5d0 \uc5ed\ud560\ud588\ub2e4. 2010\ub144 1\uc6d4 3\uc77c FA\ucef5 3\ub77c\uc6b4\ub4dc \uc5c5\ud2bc\ud30c\ud06c\uc5d0\uc11c \uc5f4\ub9b0 \uc6e8\uc2a4\ud2b8\ud584 \uc6d0\uc815\uacbd\uae30\uc5d0\uc11c 78\ubd84 \ub3d9\uc810\uace8\uc744 \uae30\ub85d\ud558\uba70 83\ubd84 \uc5d0\ub450\uc544\ub974\ub450 \ub2e4 \uc2e4\ubc14\uac00 \ud130\ud2b8\ub9b0 \uacb0\uc2b9 \ud5e4\ub529 \uace8\ub85c \uc5ed\uc804\uc2b9\uc744 \ub9cc\ub4e4\uc5b4\ub0b8\ub2e4. \uc774 \uacbd\uae30 MVP\ub85c \uc120\uc815\ub410\uc73c\uba70 \uc544\ub974\uc13c \ubcb5\uac70 \uac10\ub3c5\uc740 \uacbd\uae30 \ud6c4 \"\uacf5\uaca9\uc801\uc778 \ub85c\uc774 \ud0a8, \uc62c\ub77c\uc6b4\ub4dc \ud50c\ub808\uc774\uc5b4\ub85c \ud658\uc0c1\uc801\uc778 \ud65c\ub3d9\ub7c9\uc744 \uac00\uc84c\ub2e4\"\ub294 \ucc2c\uc0ac\ub97c \ud588\ub2e4. 2\uc6d4 27\uc77c \ube0c\ub9ac\ud0c0\ub2c8\uc544 \uc2a4\ud0c0\ub514\uc6c0\uc5d0\uc11c \uc5f4\ub9b0 \uc2a4\ud1a0\ud06c \uc2dc\ud2f0\uc640\uc758 \uacbd\uae30\uc5d0\uc11c \uc2a4\ud1a0\ud06c \uc218\ube44\uc218 \ub77c\uc774\uc5b8 \uc1fc\ud06c\ub85c\uc2a4\uc758 \ud0dc\ud074\ub85c \uc624\ub978\ucabd \ub2e4\ub9ac\uac00 \uc774\uc911\uc73c\ub85c \uace8\uc808\ub418\ub294 \ub054\ucc0d\ud55c \ubd80\uc0c1\uc744 \ub2f9\ud588\ub2e4. \uc989\uc2dc \ubcd1\uc6d0\uc73c\ub85c \uc62e\uaca8\uc84c\uc73c\ub098 \uc815\uac15\uc774\ubf08\uc640 \uc885\uc544\ub9ac\ubf08 \uc5ed\uc2dc \uace8\uc808\ub41c \uc0ac\uc2e4\uc774 \uc54c\ub824\uc84c\uc73c\uba70 \ubcf5\uadc0 \ub0a0\uc9dc \uc5ed\uc2dc \ubd88\ud22c\uba85\ud560 \uc815\ub3c4\uc758 \uc2ec\uac01\ud55c \ubd80\uc0c1\uc774\uc600\ub2e4. \ub2e4\ud589\ud788\ub3c4 2010\ub144 3\uc6d4 20\uc77c \ubaa9\ubc1c \uc5c6\uc774 \uac78\uc5b4\ub2e4\ub2c8\uae30 \uc2dc\uc791\ud574 10\uc6d4 \ud6c8\ub828\uc5d0 \ubcf5\uadc0\ud558\uc600\uc73c\uba70 11\uc6d4 23\uc77c \uc544\uc2a4\ub110\uc758 \ucf5c\ub2c8 \ud6c8\ub828\uc7a5\uc5d0\uc11c \uc5f4\ub9b0 \ub9ac\uc800\ube0c \ud300 \uacbd\uae30, \uc6b8\ubc84\ud584\ud2bc \uc6d0\ub354\ub7ec\uc2a4\uc640\uc758 \uacbd\uae30\uc5d0\uc11c \ubcf5\uadc0\uc804\uc744 \uce58\ub918\ub2e4. 2010\ub144 6\uc6d4 1\uc77c\uc5d0\ub294 \uc7ac\uacc4\uc57d\uc744 \uccb4\uacb0\ud558\uba70 \uc0c8\ub85c\uc6b4 \uc7a5\uae30\uacc4\uc57d\uc744 \ub9fa\uc5c8\ub2e4."} {"question": "\ub274\ube0c\ub9ac\uc9c0\uce90\ud53c\ud0c8\uc774 \uc81c\uc77c\uc740\ud589\uc744 \ud314\uc544\ub118\uae34 \uc0c1\ub300\ub294?", "paragraph": "\uc774\ub7ec\ud55c \ub17c\ub780\uc758 \uc815\uc810\uc5d0\ub294 98\ub144 \uc81c\uc77c\uc740\ud589\uc758 \ub274\ube0c\ub9ac\uc9c0\uce90\ud53c\ud0c8 \ub9e4\uac01 \uac74\uc774 \uc788\uc5c8\ub2e4. 1999\ub144 \ub274\ube0c\ub9ac\uc9c0\uce90\ud53c\ud0c8\uc740 8\uc870 4,000\uc5b5\uc6d0\uc758 \uacf5\uc801\uc790\uae08\uc774 \ud22c\uc785\ub41c \uc81c\uc77c\uc740\ud589\uc758 \uc9c0\ubd84 48.56%\ub97c 5000\uc5b5\uc6d0\uc5d0 \uad6c\uc785\ud558\uc5ec \uc774\ub97c 5\ub144 \ub4a4\uc778 2004\ub144\uc5d0 \uc2a4\ud0e0\ub2e4\ub4dc\ucc28\ud0c0\ub4dc \uc740\ud589\uc5d0 3\uc870 4,000\uc5b5\uc5d0 \ub418\ud314\uc544 1\uc870 5000\uc5b5\uc758 \ucc28\uc775\uc744 \ub0a8\uae30\uac8c \ub41c\ub2e4. 1999\ub144 \ub9e4\uac01 \ub2f9\uc2dc \uae40\ub300\uc911 \uc815\ubd80\ub294 \uc9c0\ubd84 70% \uad6c\uc785 \uc758\uc0ac\ub97c \ubcf4\uc600\ub358 HSBC\uc758 \uc81c\uc548\uc744 \uac70\uc808\ud558\uace0 \uc9c0\ubd84 48.56%\ub97c 5000\uc5b5\uc6d0\uc5d0 \uad6c\uc785\ud558\uaca0\ub2e4\ub294 \ub274\ube0c\ub9ac\uc9c0\uce90\ud53c\ud0c8\uc758 \uc81c\uc548\uc5d0 \ub9de\ucd94\uae30 \uc704\ud574 \ub300\uaddc\ubaa8 \uc720\uc0c1 \uac10\uc790\uc640 \uc561\uba74 \ubcd1\ud569\uc744 \uac70\uccd0 \uc81c\uc77c\uc740\ud589\uc758 \ucd1d \uc790\ubcf8\uae08\uc744 4\uc870 4806\uc5b5\uc6d0\uc5d0\uc11c 9806\uc5b5\uc6d0\uc73c\ub85c \uc904\uc600\uc73c\uba70 \ub9e4\uac01 \ud6c4 3\ub144 \ub0b4 \ubc1c\uc0dd\ud558\ub294 \ubaa8\ub4e0 \ubd80\uc2e4 \uc5ec\uc2e0\uc5d0 \uc190\uc2e4\uc774 \uc788\uc744 \uacbd\uc6b0 \uc815\ubd80\uac00 \uc190\uc2e4\uc744 \ubcf4\uc804\ud574 \uc8fc\ub294 \ud48b\ubc31 \uc635\uc158\uae4c\uc9c0 \ucd94\uac00\ud558\uc600\ub2e4.", "answer": "\uc2a4\ud0e0\ub2e4\ub4dc\ucc28\ud0c0\ub4dc \uc740\ud589", "sentence": "1999\ub144 \ub274\ube0c\ub9ac\uc9c0\uce90\ud53c\ud0c8\uc740 8\uc870 4,000\uc5b5\uc6d0\uc758 \uacf5\uc801\uc790\uae08\uc774 \ud22c\uc785\ub41c \uc81c\uc77c\uc740\ud589\uc758 \uc9c0\ubd84 48.56%\ub97c 5000\uc5b5\uc6d0\uc5d0 \uad6c\uc785\ud558\uc5ec \uc774\ub97c 5\ub144 \ub4a4\uc778 2004\ub144\uc5d0 \uc2a4\ud0e0\ub2e4\ub4dc\ucc28\ud0c0\ub4dc \uc740\ud589 \uc5d0 3\uc870 4,000\uc5b5\uc5d0 \ub418\ud314\uc544 1\uc870 5000\uc5b5\uc758 \ucc28\uc775\uc744 \ub0a8\uae30\uac8c \ub41c\ub2e4.", "paragraph_sentence": "\uc774\ub7ec\ud55c \ub17c\ub780\uc758 \uc815\uc810\uc5d0\ub294 98\ub144 \uc81c\uc77c\uc740\ud589\uc758 \ub274\ube0c\ub9ac\uc9c0\uce90\ud53c\ud0c8 \ub9e4\uac01 \uac74\uc774 \uc788\uc5c8\ub2e4. 1999\ub144 \ub274\ube0c\ub9ac\uc9c0\uce90\ud53c\ud0c8\uc740 8\uc870 4,000\uc5b5\uc6d0\uc758 \uacf5\uc801\uc790\uae08\uc774 \ud22c\uc785\ub41c \uc81c\uc77c\uc740\ud589\uc758 \uc9c0\ubd84 48.56%\ub97c 5000\uc5b5\uc6d0\uc5d0 \uad6c\uc785\ud558\uc5ec \uc774\ub97c 5\ub144 \ub4a4\uc778 2004\ub144\uc5d0 \uc2a4\ud0e0\ub2e4\ub4dc\ucc28\ud0c0\ub4dc \uc740\ud589 \uc5d0 3\uc870 4,000\uc5b5\uc5d0 \ub418\ud314\uc544 1\uc870 5000\uc5b5\uc758 \ucc28\uc775\uc744 \ub0a8\uae30\uac8c \ub41c\ub2e4. 1999\ub144 \ub9e4\uac01 \ub2f9\uc2dc \uae40\ub300\uc911 \uc815\ubd80\ub294 \uc9c0\ubd84 70% \uad6c\uc785 \uc758\uc0ac\ub97c \ubcf4\uc600\ub358 HSBC\uc758 \uc81c\uc548\uc744 \uac70\uc808\ud558\uace0 \uc9c0\ubd84 48.56%\ub97c 5000\uc5b5\uc6d0\uc5d0 \uad6c\uc785\ud558\uaca0\ub2e4\ub294 \ub274\ube0c\ub9ac\uc9c0\uce90\ud53c\ud0c8\uc758 \uc81c\uc548\uc5d0 \ub9de\ucd94\uae30 \uc704\ud574 \ub300\uaddc\ubaa8 \uc720\uc0c1 \uac10\uc790\uc640 \uc561\uba74 \ubcd1\ud569\uc744 \uac70\uccd0 \uc81c\uc77c\uc740\ud589\uc758 \ucd1d \uc790\ubcf8\uae08\uc744 4\uc870 4806\uc5b5\uc6d0\uc5d0\uc11c 9806\uc5b5\uc6d0\uc73c\ub85c \uc904\uc600\uc73c\uba70 \ub9e4\uac01 \ud6c4 3\ub144 \ub0b4 \ubc1c\uc0dd\ud558\ub294 \ubaa8\ub4e0 \ubd80\uc2e4 \uc5ec\uc2e0\uc5d0 \uc190\uc2e4\uc774 \uc788\uc744 \uacbd\uc6b0 \uc815\ubd80\uac00 \uc190\uc2e4\uc744 \ubcf4\uc804\ud574 \uc8fc\ub294 \ud48b\ubc31 \uc635\uc158\uae4c\uc9c0 \ucd94\uac00\ud558\uc600\ub2e4.", "paragraph_answer": "\uc774\ub7ec\ud55c \ub17c\ub780\uc758 \uc815\uc810\uc5d0\ub294 98\ub144 \uc81c\uc77c\uc740\ud589\uc758 \ub274\ube0c\ub9ac\uc9c0\uce90\ud53c\ud0c8 \ub9e4\uac01 \uac74\uc774 \uc788\uc5c8\ub2e4. 1999\ub144 \ub274\ube0c\ub9ac\uc9c0\uce90\ud53c\ud0c8\uc740 8\uc870 4,000\uc5b5\uc6d0\uc758 \uacf5\uc801\uc790\uae08\uc774 \ud22c\uc785\ub41c \uc81c\uc77c\uc740\ud589\uc758 \uc9c0\ubd84 48.56%\ub97c 5000\uc5b5\uc6d0\uc5d0 \uad6c\uc785\ud558\uc5ec \uc774\ub97c 5\ub144 \ub4a4\uc778 2004\ub144\uc5d0 \uc2a4\ud0e0\ub2e4\ub4dc\ucc28\ud0c0\ub4dc \uc740\ud589 \uc5d0 3\uc870 4,000\uc5b5\uc5d0 \ub418\ud314\uc544 1\uc870 5000\uc5b5\uc758 \ucc28\uc775\uc744 \ub0a8\uae30\uac8c \ub41c\ub2e4. 1999\ub144 \ub9e4\uac01 \ub2f9\uc2dc \uae40\ub300\uc911 \uc815\ubd80\ub294 \uc9c0\ubd84 70% \uad6c\uc785 \uc758\uc0ac\ub97c \ubcf4\uc600\ub358 HSBC\uc758 \uc81c\uc548\uc744 \uac70\uc808\ud558\uace0 \uc9c0\ubd84 48.56%\ub97c 5000\uc5b5\uc6d0\uc5d0 \uad6c\uc785\ud558\uaca0\ub2e4\ub294 \ub274\ube0c\ub9ac\uc9c0\uce90\ud53c\ud0c8\uc758 \uc81c\uc548\uc5d0 \ub9de\ucd94\uae30 \uc704\ud574 \ub300\uaddc\ubaa8 \uc720\uc0c1 \uac10\uc790\uc640 \uc561\uba74 \ubcd1\ud569\uc744 \uac70\uccd0 \uc81c\uc77c\uc740\ud589\uc758 \ucd1d \uc790\ubcf8\uae08\uc744 4\uc870 4806\uc5b5\uc6d0\uc5d0\uc11c 9806\uc5b5\uc6d0\uc73c\ub85c \uc904\uc600\uc73c\uba70 \ub9e4\uac01 \ud6c4 3\ub144 \ub0b4 \ubc1c\uc0dd\ud558\ub294 \ubaa8\ub4e0 \ubd80\uc2e4 \uc5ec\uc2e0\uc5d0 \uc190\uc2e4\uc774 \uc788\uc744 \uacbd\uc6b0 \uc815\ubd80\uac00 \uc190\uc2e4\uc744 \ubcf4\uc804\ud574 \uc8fc\ub294 \ud48b\ubc31 \uc635\uc158\uae4c\uc9c0 \ucd94\uac00\ud558\uc600\ub2e4.", "sentence_answer": "1999\ub144 \ub274\ube0c\ub9ac\uc9c0\uce90\ud53c\ud0c8\uc740 8\uc870 4,000\uc5b5\uc6d0\uc758 \uacf5\uc801\uc790\uae08\uc774 \ud22c\uc785\ub41c \uc81c\uc77c\uc740\ud589\uc758 \uc9c0\ubd84 48.56%\ub97c 5000\uc5b5\uc6d0\uc5d0 \uad6c\uc785\ud558\uc5ec \uc774\ub97c 5\ub144 \ub4a4\uc778 2004\ub144\uc5d0 \uc2a4\ud0e0\ub2e4\ub4dc\ucc28\ud0c0\ub4dc \uc740\ud589 \uc5d0 3\uc870 4,000\uc5b5\uc5d0 \ub418\ud314\uc544 1\uc870 5000\uc5b5\uc758 \ucc28\uc775\uc744 \ub0a8\uae30\uac8c \ub41c\ub2e4.", "paragraph_id": "6488203-75-0", "paragraph_question": "question: \ub274\ube0c\ub9ac\uc9c0\uce90\ud53c\ud0c8\uc774 \uc81c\uc77c\uc740\ud589\uc744 \ud314\uc544\ub118\uae34 \uc0c1\ub300\ub294?, context: \uc774\ub7ec\ud55c \ub17c\ub780\uc758 \uc815\uc810\uc5d0\ub294 98\ub144 \uc81c\uc77c\uc740\ud589\uc758 \ub274\ube0c\ub9ac\uc9c0\uce90\ud53c\ud0c8 \ub9e4\uac01 \uac74\uc774 \uc788\uc5c8\ub2e4. 1999\ub144 \ub274\ube0c\ub9ac\uc9c0\uce90\ud53c\ud0c8\uc740 8\uc870 4,000\uc5b5\uc6d0\uc758 \uacf5\uc801\uc790\uae08\uc774 \ud22c\uc785\ub41c \uc81c\uc77c\uc740\ud589\uc758 \uc9c0\ubd84 48.56%\ub97c 5000\uc5b5\uc6d0\uc5d0 \uad6c\uc785\ud558\uc5ec \uc774\ub97c 5\ub144 \ub4a4\uc778 2004\ub144\uc5d0 \uc2a4\ud0e0\ub2e4\ub4dc\ucc28\ud0c0\ub4dc \uc740\ud589\uc5d0 3\uc870 4,000\uc5b5\uc5d0 \ub418\ud314\uc544 1\uc870 5000\uc5b5\uc758 \ucc28\uc775\uc744 \ub0a8\uae30\uac8c \ub41c\ub2e4. 1999\ub144 \ub9e4\uac01 \ub2f9\uc2dc \uae40\ub300\uc911 \uc815\ubd80\ub294 \uc9c0\ubd84 70% \uad6c\uc785 \uc758\uc0ac\ub97c \ubcf4\uc600\ub358 HSBC\uc758 \uc81c\uc548\uc744 \uac70\uc808\ud558\uace0 \uc9c0\ubd84 48.56%\ub97c 5000\uc5b5\uc6d0\uc5d0 \uad6c\uc785\ud558\uaca0\ub2e4\ub294 \ub274\ube0c\ub9ac\uc9c0\uce90\ud53c\ud0c8\uc758 \uc81c\uc548\uc5d0 \ub9de\ucd94\uae30 \uc704\ud574 \ub300\uaddc\ubaa8 \uc720\uc0c1 \uac10\uc790\uc640 \uc561\uba74 \ubcd1\ud569\uc744 \uac70\uccd0 \uc81c\uc77c\uc740\ud589\uc758 \ucd1d \uc790\ubcf8\uae08\uc744 4\uc870 4806\uc5b5\uc6d0\uc5d0\uc11c 9806\uc5b5\uc6d0\uc73c\ub85c \uc904\uc600\uc73c\uba70 \ub9e4\uac01 \ud6c4 3\ub144 \ub0b4 \ubc1c\uc0dd\ud558\ub294 \ubaa8\ub4e0 \ubd80\uc2e4 \uc5ec\uc2e0\uc5d0 \uc190\uc2e4\uc774 \uc788\uc744 \uacbd\uc6b0 \uc815\ubd80\uac00 \uc190\uc2e4\uc744 \ubcf4\uc804\ud574 \uc8fc\ub294 \ud48b\ubc31 \uc635\uc158\uae4c\uc9c0 \ucd94\uac00\ud558\uc600\ub2e4."} {"question": "\uc774\ud718\uc18c\uac00 \ubc15\uc0ac \ub17c\ubb38\uc744 \uc900\ube44\ud558\uba70 \uacf5\ub3d9\uc73c\ub85c \uc5f0\uad6c\ub97c \uc218\ud589\ud558\ub358 \uc778\ubb3c\uc740 \ub204\uad6c\uc778\uac00?", "paragraph": "\ud074\ub77c\uc778\uc740 \ub2f9\uc2dc \uc11c\ub978 \uc138 \uc0b4\uc758 \uc80a\uc740 \uad50\uc218\uc600\ub294\ub370, \uc774\ud718\uc18c\ub294 \ud074\ub77c\uc778\uacfc \ud568\uaed8 \uacf5\ub3d9 \uc5f0\uad6c\ub97c \uc218\ud589\ud558\uba74\uc11c \uc790\uc2e0\uc758 \ubc15\uc0ac \ud559\uc704 \ub17c\ubb38\uc5d0 \uc870\uae08\uc529 \uc811\uadfc\ud574 \ub098\uac14\ub2e4. \uadf8\ub7ec\ub2e4 1960\ub144 11\uc6d4\uc5d0 \u3008K \uc911\uac04\uc790\uc640 \ud575\uc790 \uc0b0\ub780 \ud604\uc0c1\uc758 \uc774\uc911 \ubd84\uc0b0 \uad00\uacc4\u3009(Study of K Scattering in the Double Dispersion Representation)\uc73c\ub85c \ubb3c\ub9ac\ud559 \ubc15\uc0ac(Doctor of Philosophy) \ud559\uc704\ub97c \ubc1b\uc558\ub294\ub370 \uc774 \ub54c \ub098\uc774\ub294 \ubd88\uacfc 25\uc138\uc600\ub2e4. \uadf8\uac00 \ubc15\uc0ac \ud559\uc704\uc99d\uc744 \uacf5\uc2dd\uc801\uc73c\ub85c \ubc1b\uc740 \uac83\uc740 1961\ub144 2\uc6d4 4\uc77c\uc774\uc5c8\ub2e4. \ubc15\uc0ac \ub17c\ubb38 \ub514\ud39c\uc2a4\uac00 \ub05d\ub09c 11\uc6d4\ubd80\ud130 1961\ub144 8\uc6d4\uae4c\uc9c0, \uc774\ud718\uc18c\ub294 \ud39c\uc2e4\ubca0\uc774\ub2c8\uc544 \ub300\ud559\uad50\uc758 \ubc15\uc0ac\ud6c4 \uc5f0\uad6c\uc6d0 \ubc0f \uc804\uc784 \uac15\uc0ac\ub85c \uc784\uc6a9\ub410\ub2e4.", "answer": "\ud074\ub77c\uc778", "sentence": "\ud074\ub77c\uc778 \uc740 \ub2f9\uc2dc \uc11c\ub978 \uc138 \uc0b4\uc758 \uc80a\uc740 \uad50\uc218\uc600\ub294\ub370, \uc774\ud718\uc18c\ub294 \ud074\ub77c\uc778\uacfc \ud568\uaed8 \uacf5\ub3d9 \uc5f0\uad6c\ub97c \uc218\ud589\ud558\uba74\uc11c \uc790\uc2e0\uc758 \ubc15\uc0ac \ud559\uc704 \ub17c\ubb38\uc5d0 \uc870\uae08\uc529 \uc811\uadfc\ud574 \ub098\uac14\ub2e4.", "paragraph_sentence": " \ud074\ub77c\uc778 \uc740 \ub2f9\uc2dc \uc11c\ub978 \uc138 \uc0b4\uc758 \uc80a\uc740 \uad50\uc218\uc600\ub294\ub370, \uc774\ud718\uc18c\ub294 \ud074\ub77c\uc778\uacfc \ud568\uaed8 \uacf5\ub3d9 \uc5f0\uad6c\ub97c \uc218\ud589\ud558\uba74\uc11c \uc790\uc2e0\uc758 \ubc15\uc0ac \ud559\uc704 \ub17c\ubb38\uc5d0 \uc870\uae08\uc529 \uc811\uadfc\ud574 \ub098\uac14\ub2e4. \uadf8\ub7ec\ub2e4 1960\ub144 11\uc6d4\uc5d0 \u3008K \uc911\uac04\uc790\uc640 \ud575\uc790 \uc0b0\ub780 \ud604\uc0c1\uc758 \uc774\uc911 \ubd84\uc0b0 \uad00\uacc4\u3009(Study of K Scattering in the Double Dispersion Representation)\uc73c\ub85c \ubb3c\ub9ac\ud559 \ubc15\uc0ac(Doctor of Philosophy) \ud559\uc704\ub97c \ubc1b\uc558\ub294\ub370 \uc774 \ub54c \ub098\uc774\ub294 \ubd88\uacfc 25\uc138\uc600\ub2e4. \uadf8\uac00 \ubc15\uc0ac \ud559\uc704\uc99d\uc744 \uacf5\uc2dd\uc801\uc73c\ub85c \ubc1b\uc740 \uac83\uc740 1961\ub144 2\uc6d4 4\uc77c\uc774\uc5c8\ub2e4. \ubc15\uc0ac \ub17c\ubb38 \ub514\ud39c\uc2a4\uac00 \ub05d\ub09c 11\uc6d4\ubd80\ud130 1961\ub144 8\uc6d4\uae4c\uc9c0, \uc774\ud718\uc18c\ub294 \ud39c\uc2e4\ubca0\uc774\ub2c8\uc544 \ub300\ud559\uad50\uc758 \ubc15\uc0ac\ud6c4 \uc5f0\uad6c\uc6d0 \ubc0f \uc804\uc784 \uac15\uc0ac\ub85c \uc784\uc6a9\ub410\ub2e4.", "paragraph_answer": " \ud074\ub77c\uc778 \uc740 \ub2f9\uc2dc \uc11c\ub978 \uc138 \uc0b4\uc758 \uc80a\uc740 \uad50\uc218\uc600\ub294\ub370, \uc774\ud718\uc18c\ub294 \ud074\ub77c\uc778\uacfc \ud568\uaed8 \uacf5\ub3d9 \uc5f0\uad6c\ub97c \uc218\ud589\ud558\uba74\uc11c \uc790\uc2e0\uc758 \ubc15\uc0ac \ud559\uc704 \ub17c\ubb38\uc5d0 \uc870\uae08\uc529 \uc811\uadfc\ud574 \ub098\uac14\ub2e4. \uadf8\ub7ec\ub2e4 1960\ub144 11\uc6d4\uc5d0 \u3008K \uc911\uac04\uc790\uc640 \ud575\uc790 \uc0b0\ub780 \ud604\uc0c1\uc758 \uc774\uc911 \ubd84\uc0b0 \uad00\uacc4\u3009(Study of K Scattering in the Double Dispersion Representation)\uc73c\ub85c \ubb3c\ub9ac\ud559 \ubc15\uc0ac(Doctor of Philosophy) \ud559\uc704\ub97c \ubc1b\uc558\ub294\ub370 \uc774 \ub54c \ub098\uc774\ub294 \ubd88\uacfc 25\uc138\uc600\ub2e4. \uadf8\uac00 \ubc15\uc0ac \ud559\uc704\uc99d\uc744 \uacf5\uc2dd\uc801\uc73c\ub85c \ubc1b\uc740 \uac83\uc740 1961\ub144 2\uc6d4 4\uc77c\uc774\uc5c8\ub2e4. \ubc15\uc0ac \ub17c\ubb38 \ub514\ud39c\uc2a4\uac00 \ub05d\ub09c 11\uc6d4\ubd80\ud130 1961\ub144 8\uc6d4\uae4c\uc9c0, \uc774\ud718\uc18c\ub294 \ud39c\uc2e4\ubca0\uc774\ub2c8\uc544 \ub300\ud559\uad50\uc758 \ubc15\uc0ac\ud6c4 \uc5f0\uad6c\uc6d0 \ubc0f \uc804\uc784 \uac15\uc0ac\ub85c \uc784\uc6a9\ub410\ub2e4.", "sentence_answer": " \ud074\ub77c\uc778 \uc740 \ub2f9\uc2dc \uc11c\ub978 \uc138 \uc0b4\uc758 \uc80a\uc740 \uad50\uc218\uc600\ub294\ub370, \uc774\ud718\uc18c\ub294 \ud074\ub77c\uc778\uacfc \ud568\uaed8 \uacf5\ub3d9 \uc5f0\uad6c\ub97c \uc218\ud589\ud558\uba74\uc11c \uc790\uc2e0\uc758 \ubc15\uc0ac \ud559\uc704 \ub17c\ubb38\uc5d0 \uc870\uae08\uc529 \uc811\uadfc\ud574 \ub098\uac14\ub2e4.", "paragraph_id": "6547981-5-0", "paragraph_question": "question: \uc774\ud718\uc18c\uac00 \ubc15\uc0ac \ub17c\ubb38\uc744 \uc900\ube44\ud558\uba70 \uacf5\ub3d9\uc73c\ub85c \uc5f0\uad6c\ub97c \uc218\ud589\ud558\ub358 \uc778\ubb3c\uc740 \ub204\uad6c\uc778\uac00?, context: \ud074\ub77c\uc778\uc740 \ub2f9\uc2dc \uc11c\ub978 \uc138 \uc0b4\uc758 \uc80a\uc740 \uad50\uc218\uc600\ub294\ub370, \uc774\ud718\uc18c\ub294 \ud074\ub77c\uc778\uacfc \ud568\uaed8 \uacf5\ub3d9 \uc5f0\uad6c\ub97c \uc218\ud589\ud558\uba74\uc11c \uc790\uc2e0\uc758 \ubc15\uc0ac \ud559\uc704 \ub17c\ubb38\uc5d0 \uc870\uae08\uc529 \uc811\uadfc\ud574 \ub098\uac14\ub2e4. \uadf8\ub7ec\ub2e4 1960\ub144 11\uc6d4\uc5d0 \u3008K \uc911\uac04\uc790\uc640 \ud575\uc790 \uc0b0\ub780 \ud604\uc0c1\uc758 \uc774\uc911 \ubd84\uc0b0 \uad00\uacc4\u3009(Study of K Scattering in the Double Dispersion Representation)\uc73c\ub85c \ubb3c\ub9ac\ud559 \ubc15\uc0ac(Doctor of Philosophy) \ud559\uc704\ub97c \ubc1b\uc558\ub294\ub370 \uc774 \ub54c \ub098\uc774\ub294 \ubd88\uacfc 25\uc138\uc600\ub2e4. \uadf8\uac00 \ubc15\uc0ac \ud559\uc704\uc99d\uc744 \uacf5\uc2dd\uc801\uc73c\ub85c \ubc1b\uc740 \uac83\uc740 1961\ub144 2\uc6d4 4\uc77c\uc774\uc5c8\ub2e4. \ubc15\uc0ac \ub17c\ubb38 \ub514\ud39c\uc2a4\uac00 \ub05d\ub09c 11\uc6d4\ubd80\ud130 1961\ub144 8\uc6d4\uae4c\uc9c0, \uc774\ud718\uc18c\ub294 \ud39c\uc2e4\ubca0\uc774\ub2c8\uc544 \ub300\ud559\uad50\uc758 \ubc15\uc0ac\ud6c4 \uc5f0\uad6c\uc6d0 \ubc0f \uc804\uc784 \uac15\uc0ac\ub85c \uc784\uc6a9\ub410\ub2e4."} {"question": "\uc81c 9\ud68c \ub3d9\uc544\uc2dc\uc544 \uad50\uc0ac\uad50\uc721 \uad6d\uc81c\uc2ec\ud3ec\uc9c0\uc5c4\uc744 \uc8fc\uad00\ud588\ub358 \ub144\ub3c4\ub294?", "paragraph": "\uc544\uc2dc\uc544 \uad50\uc6d0\uad50\uc721 \ud611\uc758\uccb4 \uad6c\ucd95\uacfc \ud574\uc678 \uc0ac\ubc94\ub300\ud559\uacfc\uc758 \ubcf5\uc218 \ud559\uc704\uc81c \uc2dc\ud589 \ub4f1 \uad6d\u00b7\ub0b4\uc678 \uc678\ubd80 \uae30\uad00\uacfc \ud559\uc220 \uad50\ub958 \uad00\uacc4\ub97c \ub9fa\uace0 \uc788\ub2e4. \uc911\uad6d\uc758 \ubd81\uacbd\uc0ac\ubc94\ub300\ud559, \uc77c\ubcf8\uc758 \ub3c4\ucfc4 \uac00\ucfe0\uac8c\uc774 \ub300\ud559, \ubbf8\uad6d\uc758 \ucf04\ud130\ud0a4 \uc8fc\ub9bd \ub300\ud559\uad50 \ub4f1 35\uac1c \uc678\uad6d \ub300\ud559\uacfc \ud559\uc220 \uad50\ub958 \ud611\uc815\uc744 \uccb4\uacb0\ud558\uace0 \uc788\uc73c\uba70 2009\ub144 12\uc6d4 \uacb0\uc131\ub41c \ub300\ud559\uac04 \ud611\uc758\uccb4\uc778 \ub3d9\uc544\uc2dc\uc544 \uad50\uc6d0\uc591\uc131 \uad6d\uc81c\ucee8\uc18c\uc2dc\uc5c4\uc758 \ud68c\uc6d0 \ub300\ud559\uc73c\ub85c \ucc38\uc5ec\ud558\uace0 \uc788\ub2e4. 2014\ub144 11\uc6d4\uc5d0\ub294 \u2018\ub514\uc9c0\ud138 \uc2dc\ub300\uc758 \uc2a4\ub9c8\ud2b8\uad50\uc721\uacfc \uad50\uc0ac\uad50\uc721\u2019\uc744 \uc8fc\uc81c\ub85c \uc81c9\ud68c \ub3d9\uc544\uc2dc\uc544 \uad50\uc0ac\uad50\uc721 \uad6d\uc81c\uc2ec\ud3ec\uc9c0\uc5c4\uc744 \uc8fc\uad00\ud558\uae30\ub3c4 \ud588\ub2e4. \ud55c\uad6d\uad50\uc721\uacfc\uc815\ud3c9\uac00\uc6d0, \ud55c\uad6d\uad50\uc721\uac1c\ubc1c\uc6d0, \ud55c\uad6d\uc9c1\uc5c5\ub2a5\ub825\uac1c\ubc1c\uc6d0, 17\uac1c \uc2dc\u00b7\ub3c4 \uad50\uc721\uccad \ub4f1 \uad6d\ub0b4 \uad50\uc721 \uad00\ub828 \uae30\uad00\uacfc 52\uac1c \uad6d\ub0b4 \ub300\ud559\uacfc\ub3c4 \ud611\ub825 \uad00\uacc4\ub97c \ub9fa\uace0 \uc788\ub2e4.", "answer": "2014\ub144", "sentence": "2014\ub144 11\uc6d4\uc5d0\ub294 \u2018\ub514\uc9c0\ud138 \uc2dc\ub300\uc758 \uc2a4\ub9c8\ud2b8\uad50\uc721\uacfc \uad50\uc0ac\uad50\uc721\u2019\uc744 \uc8fc\uc81c\ub85c \uc81c9\ud68c \ub3d9\uc544\uc2dc\uc544 \uad50\uc0ac\uad50\uc721 \uad6d\uc81c\uc2ec\ud3ec\uc9c0\uc5c4\uc744 \uc8fc\uad00\ud558\uae30\ub3c4 \ud588\ub2e4.", "paragraph_sentence": "\uc544\uc2dc\uc544 \uad50\uc6d0\uad50\uc721 \ud611\uc758\uccb4 \uad6c\ucd95\uacfc \ud574\uc678 \uc0ac\ubc94\ub300\ud559\uacfc\uc758 \ubcf5\uc218 \ud559\uc704\uc81c \uc2dc\ud589 \ub4f1 \uad6d\u00b7\ub0b4\uc678 \uc678\ubd80 \uae30\uad00\uacfc \ud559\uc220 \uad50\ub958 \uad00\uacc4\ub97c \ub9fa\uace0 \uc788\ub2e4. \uc911\uad6d\uc758 \ubd81\uacbd\uc0ac\ubc94\ub300\ud559, \uc77c\ubcf8\uc758 \ub3c4\ucfc4 \uac00\ucfe0\uac8c\uc774 \ub300\ud559, \ubbf8\uad6d\uc758 \ucf04\ud130\ud0a4 \uc8fc\ub9bd \ub300\ud559\uad50 \ub4f1 35\uac1c \uc678\uad6d \ub300\ud559\uacfc \ud559\uc220 \uad50\ub958 \ud611\uc815\uc744 \uccb4\uacb0\ud558\uace0 \uc788\uc73c\uba70 2009\ub144 12\uc6d4 \uacb0\uc131\ub41c \ub300\ud559\uac04 \ud611\uc758\uccb4\uc778 \ub3d9\uc544\uc2dc\uc544 \uad50\uc6d0\uc591\uc131 \uad6d\uc81c\ucee8\uc18c\uc2dc\uc5c4\uc758 \ud68c\uc6d0 \ub300\ud559\uc73c\ub85c \ucc38\uc5ec\ud558\uace0 \uc788\ub2e4. 2014\ub144 11\uc6d4\uc5d0\ub294 \u2018\ub514\uc9c0\ud138 \uc2dc\ub300\uc758 \uc2a4\ub9c8\ud2b8\uad50\uc721\uacfc \uad50\uc0ac\uad50\uc721\u2019\uc744 \uc8fc\uc81c\ub85c \uc81c9\ud68c \ub3d9\uc544\uc2dc\uc544 \uad50\uc0ac\uad50\uc721 \uad6d\uc81c\uc2ec\ud3ec\uc9c0\uc5c4\uc744 \uc8fc\uad00\ud558\uae30\ub3c4 \ud588\ub2e4. \ud55c\uad6d\uad50\uc721\uacfc\uc815\ud3c9\uac00\uc6d0, \ud55c\uad6d\uad50\uc721\uac1c\ubc1c\uc6d0, \ud55c\uad6d\uc9c1\uc5c5\ub2a5\ub825\uac1c\ubc1c\uc6d0, 17\uac1c \uc2dc\u00b7\ub3c4 \uad50\uc721\uccad \ub4f1 \uad6d\ub0b4 \uad50\uc721 \uad00\ub828 \uae30\uad00\uacfc 52\uac1c \uad6d\ub0b4 \ub300\ud559\uacfc\ub3c4 \ud611\ub825 \uad00\uacc4\ub97c \ub9fa\uace0 \uc788\ub2e4.", "paragraph_answer": "\uc544\uc2dc\uc544 \uad50\uc6d0\uad50\uc721 \ud611\uc758\uccb4 \uad6c\ucd95\uacfc \ud574\uc678 \uc0ac\ubc94\ub300\ud559\uacfc\uc758 \ubcf5\uc218 \ud559\uc704\uc81c \uc2dc\ud589 \ub4f1 \uad6d\u00b7\ub0b4\uc678 \uc678\ubd80 \uae30\uad00\uacfc \ud559\uc220 \uad50\ub958 \uad00\uacc4\ub97c \ub9fa\uace0 \uc788\ub2e4. \uc911\uad6d\uc758 \ubd81\uacbd\uc0ac\ubc94\ub300\ud559, \uc77c\ubcf8\uc758 \ub3c4\ucfc4 \uac00\ucfe0\uac8c\uc774 \ub300\ud559, \ubbf8\uad6d\uc758 \ucf04\ud130\ud0a4 \uc8fc\ub9bd \ub300\ud559\uad50 \ub4f1 35\uac1c \uc678\uad6d \ub300\ud559\uacfc \ud559\uc220 \uad50\ub958 \ud611\uc815\uc744 \uccb4\uacb0\ud558\uace0 \uc788\uc73c\uba70 2009\ub144 12\uc6d4 \uacb0\uc131\ub41c \ub300\ud559\uac04 \ud611\uc758\uccb4\uc778 \ub3d9\uc544\uc2dc\uc544 \uad50\uc6d0\uc591\uc131 \uad6d\uc81c\ucee8\uc18c\uc2dc\uc5c4\uc758 \ud68c\uc6d0 \ub300\ud559\uc73c\ub85c \ucc38\uc5ec\ud558\uace0 \uc788\ub2e4. 2014\ub144 11\uc6d4\uc5d0\ub294 \u2018\ub514\uc9c0\ud138 \uc2dc\ub300\uc758 \uc2a4\ub9c8\ud2b8\uad50\uc721\uacfc \uad50\uc0ac\uad50\uc721\u2019\uc744 \uc8fc\uc81c\ub85c \uc81c9\ud68c \ub3d9\uc544\uc2dc\uc544 \uad50\uc0ac\uad50\uc721 \uad6d\uc81c\uc2ec\ud3ec\uc9c0\uc5c4\uc744 \uc8fc\uad00\ud558\uae30\ub3c4 \ud588\ub2e4. \ud55c\uad6d\uad50\uc721\uacfc\uc815\ud3c9\uac00\uc6d0, \ud55c\uad6d\uad50\uc721\uac1c\ubc1c\uc6d0, \ud55c\uad6d\uc9c1\uc5c5\ub2a5\ub825\uac1c\ubc1c\uc6d0, 17\uac1c \uc2dc\u00b7\ub3c4 \uad50\uc721\uccad \ub4f1 \uad6d\ub0b4 \uad50\uc721 \uad00\ub828 \uae30\uad00\uacfc 52\uac1c \uad6d\ub0b4 \ub300\ud559\uacfc\ub3c4 \ud611\ub825 \uad00\uacc4\ub97c \ub9fa\uace0 \uc788\ub2e4.", "sentence_answer": " 2014\ub144 11\uc6d4\uc5d0\ub294 \u2018\ub514\uc9c0\ud138 \uc2dc\ub300\uc758 \uc2a4\ub9c8\ud2b8\uad50\uc721\uacfc \uad50\uc0ac\uad50\uc721\u2019\uc744 \uc8fc\uc81c\ub85c \uc81c9\ud68c \ub3d9\uc544\uc2dc\uc544 \uad50\uc0ac\uad50\uc721 \uad6d\uc81c\uc2ec\ud3ec\uc9c0\uc5c4\uc744 \uc8fc\uad00\ud558\uae30\ub3c4 \ud588\ub2e4.", "paragraph_id": "6569875-9-2", "paragraph_question": "question: \uc81c 9\ud68c \ub3d9\uc544\uc2dc\uc544 \uad50\uc0ac\uad50\uc721 \uad6d\uc81c\uc2ec\ud3ec\uc9c0\uc5c4\uc744 \uc8fc\uad00\ud588\ub358 \ub144\ub3c4\ub294?, context: \uc544\uc2dc\uc544 \uad50\uc6d0\uad50\uc721 \ud611\uc758\uccb4 \uad6c\ucd95\uacfc \ud574\uc678 \uc0ac\ubc94\ub300\ud559\uacfc\uc758 \ubcf5\uc218 \ud559\uc704\uc81c \uc2dc\ud589 \ub4f1 \uad6d\u00b7\ub0b4\uc678 \uc678\ubd80 \uae30\uad00\uacfc \ud559\uc220 \uad50\ub958 \uad00\uacc4\ub97c \ub9fa\uace0 \uc788\ub2e4. \uc911\uad6d\uc758 \ubd81\uacbd\uc0ac\ubc94\ub300\ud559, \uc77c\ubcf8\uc758 \ub3c4\ucfc4 \uac00\ucfe0\uac8c\uc774 \ub300\ud559, \ubbf8\uad6d\uc758 \ucf04\ud130\ud0a4 \uc8fc\ub9bd \ub300\ud559\uad50 \ub4f1 35\uac1c \uc678\uad6d \ub300\ud559\uacfc \ud559\uc220 \uad50\ub958 \ud611\uc815\uc744 \uccb4\uacb0\ud558\uace0 \uc788\uc73c\uba70 2009\ub144 12\uc6d4 \uacb0\uc131\ub41c \ub300\ud559\uac04 \ud611\uc758\uccb4\uc778 \ub3d9\uc544\uc2dc\uc544 \uad50\uc6d0\uc591\uc131 \uad6d\uc81c\ucee8\uc18c\uc2dc\uc5c4\uc758 \ud68c\uc6d0 \ub300\ud559\uc73c\ub85c \ucc38\uc5ec\ud558\uace0 \uc788\ub2e4. 2014\ub144 11\uc6d4\uc5d0\ub294 \u2018\ub514\uc9c0\ud138 \uc2dc\ub300\uc758 \uc2a4\ub9c8\ud2b8\uad50\uc721\uacfc \uad50\uc0ac\uad50\uc721\u2019\uc744 \uc8fc\uc81c\ub85c \uc81c9\ud68c \ub3d9\uc544\uc2dc\uc544 \uad50\uc0ac\uad50\uc721 \uad6d\uc81c\uc2ec\ud3ec\uc9c0\uc5c4\uc744 \uc8fc\uad00\ud558\uae30\ub3c4 \ud588\ub2e4. \ud55c\uad6d\uad50\uc721\uacfc\uc815\ud3c9\uac00\uc6d0, \ud55c\uad6d\uad50\uc721\uac1c\ubc1c\uc6d0, \ud55c\uad6d\uc9c1\uc5c5\ub2a5\ub825\uac1c\ubc1c\uc6d0, 17\uac1c \uc2dc\u00b7\ub3c4 \uad50\uc721\uccad \ub4f1 \uad6d\ub0b4 \uad50\uc721 \uad00\ub828 \uae30\uad00\uacfc 52\uac1c \uad6d\ub0b4 \ub300\ud559\uacfc\ub3c4 \ud611\ub825 \uad00\uacc4\ub97c \ub9fa\uace0 \uc788\ub2e4."} {"question": "\uc624\ub7ab\ub3d9\uc548 \ub0c9\uac01\ub418\uc5b4 \uc788\ub358 \ud55c\uc77c \uad00\uacc4\uc758 \ubb38\uc81c\ub294?", "paragraph": "\uc704\uc548\ubd80 \ud611\uc0c1 \ubb38\uc81c\ub85c \ud55c\uc77c \uad00\uacc4\ub294 \ubb38\uc7ac\uc778 \uc815\ubd80 \ucd9c\ubc94\uc744 \uc804\ud6c4\ud558\uc5ec \uc624\ub7ab\ub3d9\uc548 \ub0c9\uac01\ub418\uc5b4 \uc788\uc5c8\ub2e4. \uc774\ub97c \ud574\uacb0\ud558\uace0\uc790 2017\ub144 12\uc6d4 19\uc77c \uac15\uacbd\ud654 \uc678\uad50\ubd80 \uc7a5\uad00\uc774 1\ubc15 2\uc77c \uac04\uc758 \ubc29\uc77c\uc744 \ud558\uc600\ub2e4. \uc544\ubca0 \uc2e0\uc870 \ucd1d\ub9ac\ub300\uc2e0\uacfc \uace0\ub178 \ub2e4\ub85c \uc678\ubb34\ub300\uc2e0\uc744 \ub9cc\ub09c \uc790\ub9ac\uc5d0\uc11c \uac15\uacbd\ud654\ub294 \"\uacfc\uac70\uc0ac\ub85c\ubd80\ud130 \ube44\ub86f\ub418\ub294 \uc5b4\ub824\uc6b4 \ubb38\uc81c\ub4e4\uc774 \uc788\uc9c0\ub9cc, \uadf8\ub7ec\ud55c \uc5b4\ub824\uc6c0\ub4e4\uc740 \uae34\ubc00\ud55c \uc18c\ud1b5\uc744 \ud1b5\ud574 \uc9c0\ud61c\ub86d\uac8c \uadf9\ubcf5\ud574 \ub098\uac00\uae38 \ud76c\ub9dd\ud55c\ub2e4\"\ub294 \ub300\ud1b5\ub839\uc758 \uba54\uc2dc\uc9c0\ub97c \uc804\ub2ec\ud588\uace0 \uc544\ubca0\ub294 \"\ud55c\uad6d\uc740 \uc804\ub7b5\uc801 \uc774\uc775\uc744 \uacf5\uc720\ud558\ub294, \uc77c\ubcf8\uc5d0 \uac00\uc7a5 \uc911\uc694\ud55c \uc774\uc6c3\"\uc774\ub77c\uace0 \ud654\ub2f5\ud588\ub2e4. \uc911\uc694 \ud604\uc548\uc73c\ub85c\ub294 \ubd81\ud575 \ubb38\uc81c\uc640 \ud568\uaed8 \uc704\uc548\ubd80 \uc598\uae30\uac00 \ub2e4\ub904\uc84c\ub294\ub370, \uace0\ub178 \ub300\uc2e0\uc740 \"\ud55c\u00b7\uc77c \uc704\uc548\ubd80 \ud569\uc758\uc758 \ucc29\uc2e4\ud55c \uc774\ud589\uc774 \uc911\uc694\ud558\ub2e4\"\uace0 \uac15\uc870\ud588\uace0 \uac15 \uc7a5\uad00\uc740 \"(27\uc77c \ubc1c\ud45c\ub418\ub294 TF) \ubcf4\uace0\uc11c \uacb0\uacfc\ub97c \uadf8\ub300\ub85c \ud55c\uad6d \uc815\ubd80\uc758 \uc785\uc7a5\uc73c\ub85c \ucc44\ud0dd\ud558\uc9c0\ub294 \uc54a\uc744 \uac83\"\uc774\ub77c\ub294 \ub73b\uc744 \uc804\ub2ec\ud588\ub2e4. \uc774\ub294 \uc544\ubca0 \ucd1d\ub9ac\uc758 \ud3c9\ucc3d \ub3d9\uacc4\uc62c\ub9bc\ud53d \ucc38\uc11d\uc744 \uc694\uccad\ud558\uba74\uc11c \ud55c\uc77c \uad00\uacc4\uc758 \ubbf8\ub798\ub97c \uc704\ud574 \ud30c\uad6d\uc801 \uacb0\uc815\uc744 \ud558\uc9c0 \uc54a\uc744 \ub73b\uc744 \ub0b4\ube44\uce5c \uac83\uc73c\ub85c \ubcf4\uc778\ub2e4.", "answer": "\uc704\uc548\ubd80 \ud611\uc0c1 \ubb38\uc81c", "sentence": "\uc704\uc548\ubd80 \ud611\uc0c1 \ubb38\uc81c \ub85c \ud55c\uc77c \uad00\uacc4\ub294 \ubb38\uc7ac\uc778 \uc815\ubd80 \ucd9c\ubc94\uc744 \uc804\ud6c4\ud558\uc5ec \uc624\ub7ab\ub3d9\uc548 \ub0c9\uac01\ub418\uc5b4 \uc788\uc5c8\ub2e4.", "paragraph_sentence": " \uc704\uc548\ubd80 \ud611\uc0c1 \ubb38\uc81c \ub85c \ud55c\uc77c \uad00\uacc4\ub294 \ubb38\uc7ac\uc778 \uc815\ubd80 \ucd9c\ubc94\uc744 \uc804\ud6c4\ud558\uc5ec \uc624\ub7ab\ub3d9\uc548 \ub0c9\uac01\ub418\uc5b4 \uc788\uc5c8\ub2e4. \uc774\ub97c \ud574\uacb0\ud558\uace0\uc790 2017\ub144 12\uc6d4 19\uc77c \uac15\uacbd\ud654 \uc678\uad50\ubd80 \uc7a5\uad00\uc774 1\ubc15 2\uc77c \uac04\uc758 \ubc29\uc77c\uc744 \ud558\uc600\ub2e4. \uc544\ubca0 \uc2e0\uc870 \ucd1d\ub9ac\ub300\uc2e0\uacfc \uace0\ub178 \ub2e4\ub85c \uc678\ubb34\ub300\uc2e0\uc744 \ub9cc\ub09c \uc790\ub9ac\uc5d0\uc11c \uac15\uacbd\ud654\ub294 \"\uacfc\uac70\uc0ac\ub85c\ubd80\ud130 \ube44\ub86f\ub418\ub294 \uc5b4\ub824\uc6b4 \ubb38\uc81c\ub4e4\uc774 \uc788\uc9c0\ub9cc, \uadf8\ub7ec\ud55c \uc5b4\ub824\uc6c0\ub4e4\uc740 \uae34\ubc00\ud55c \uc18c\ud1b5\uc744 \ud1b5\ud574 \uc9c0\ud61c\ub86d\uac8c \uadf9\ubcf5\ud574 \ub098\uac00\uae38 \ud76c\ub9dd\ud55c\ub2e4\"\ub294 \ub300\ud1b5\ub839\uc758 \uba54\uc2dc\uc9c0\ub97c \uc804\ub2ec\ud588\uace0 \uc544\ubca0\ub294 \"\ud55c\uad6d\uc740 \uc804\ub7b5\uc801 \uc774\uc775\uc744 \uacf5\uc720\ud558\ub294, \uc77c\ubcf8\uc5d0 \uac00\uc7a5 \uc911\uc694\ud55c \uc774\uc6c3\"\uc774\ub77c\uace0 \ud654\ub2f5\ud588\ub2e4. \uc911\uc694 \ud604\uc548\uc73c\ub85c\ub294 \ubd81\ud575 \ubb38\uc81c\uc640 \ud568\uaed8 \uc704\uc548\ubd80 \uc598\uae30\uac00 \ub2e4\ub904\uc84c\ub294\ub370, \uace0\ub178 \ub300\uc2e0\uc740 \"\ud55c\u00b7\uc77c \uc704\uc548\ubd80 \ud569\uc758\uc758 \ucc29\uc2e4\ud55c \uc774\ud589\uc774 \uc911\uc694\ud558\ub2e4\"\uace0 \uac15\uc870\ud588\uace0 \uac15 \uc7a5\uad00\uc740 \"(27\uc77c \ubc1c\ud45c\ub418\ub294 TF) \ubcf4\uace0\uc11c \uacb0\uacfc\ub97c \uadf8\ub300\ub85c \ud55c\uad6d \uc815\ubd80\uc758 \uc785\uc7a5\uc73c\ub85c \ucc44\ud0dd\ud558\uc9c0\ub294 \uc54a\uc744 \uac83\"\uc774\ub77c\ub294 \ub73b\uc744 \uc804\ub2ec\ud588\ub2e4. \uc774\ub294 \uc544\ubca0 \ucd1d\ub9ac\uc758 \ud3c9\ucc3d \ub3d9\uacc4\uc62c\ub9bc\ud53d \ucc38\uc11d\uc744 \uc694\uccad\ud558\uba74\uc11c \ud55c\uc77c \uad00\uacc4\uc758 \ubbf8\ub798\ub97c \uc704\ud574 \ud30c\uad6d\uc801 \uacb0\uc815\uc744 \ud558\uc9c0 \uc54a\uc744 \ub73b\uc744 \ub0b4\ube44\uce5c \uac83\uc73c\ub85c \ubcf4\uc778\ub2e4.", "paragraph_answer": " \uc704\uc548\ubd80 \ud611\uc0c1 \ubb38\uc81c \ub85c \ud55c\uc77c \uad00\uacc4\ub294 \ubb38\uc7ac\uc778 \uc815\ubd80 \ucd9c\ubc94\uc744 \uc804\ud6c4\ud558\uc5ec \uc624\ub7ab\ub3d9\uc548 \ub0c9\uac01\ub418\uc5b4 \uc788\uc5c8\ub2e4. \uc774\ub97c \ud574\uacb0\ud558\uace0\uc790 2017\ub144 12\uc6d4 19\uc77c \uac15\uacbd\ud654 \uc678\uad50\ubd80 \uc7a5\uad00\uc774 1\ubc15 2\uc77c \uac04\uc758 \ubc29\uc77c\uc744 \ud558\uc600\ub2e4. \uc544\ubca0 \uc2e0\uc870 \ucd1d\ub9ac\ub300\uc2e0\uacfc \uace0\ub178 \ub2e4\ub85c \uc678\ubb34\ub300\uc2e0\uc744 \ub9cc\ub09c \uc790\ub9ac\uc5d0\uc11c \uac15\uacbd\ud654\ub294 \"\uacfc\uac70\uc0ac\ub85c\ubd80\ud130 \ube44\ub86f\ub418\ub294 \uc5b4\ub824\uc6b4 \ubb38\uc81c\ub4e4\uc774 \uc788\uc9c0\ub9cc, \uadf8\ub7ec\ud55c \uc5b4\ub824\uc6c0\ub4e4\uc740 \uae34\ubc00\ud55c \uc18c\ud1b5\uc744 \ud1b5\ud574 \uc9c0\ud61c\ub86d\uac8c \uadf9\ubcf5\ud574 \ub098\uac00\uae38 \ud76c\ub9dd\ud55c\ub2e4\"\ub294 \ub300\ud1b5\ub839\uc758 \uba54\uc2dc\uc9c0\ub97c \uc804\ub2ec\ud588\uace0 \uc544\ubca0\ub294 \"\ud55c\uad6d\uc740 \uc804\ub7b5\uc801 \uc774\uc775\uc744 \uacf5\uc720\ud558\ub294, \uc77c\ubcf8\uc5d0 \uac00\uc7a5 \uc911\uc694\ud55c \uc774\uc6c3\"\uc774\ub77c\uace0 \ud654\ub2f5\ud588\ub2e4. \uc911\uc694 \ud604\uc548\uc73c\ub85c\ub294 \ubd81\ud575 \ubb38\uc81c\uc640 \ud568\uaed8 \uc704\uc548\ubd80 \uc598\uae30\uac00 \ub2e4\ub904\uc84c\ub294\ub370, \uace0\ub178 \ub300\uc2e0\uc740 \"\ud55c\u00b7\uc77c \uc704\uc548\ubd80 \ud569\uc758\uc758 \ucc29\uc2e4\ud55c \uc774\ud589\uc774 \uc911\uc694\ud558\ub2e4\"\uace0 \uac15\uc870\ud588\uace0 \uac15 \uc7a5\uad00\uc740 \"(27\uc77c \ubc1c\ud45c\ub418\ub294 TF) \ubcf4\uace0\uc11c \uacb0\uacfc\ub97c \uadf8\ub300\ub85c \ud55c\uad6d \uc815\ubd80\uc758 \uc785\uc7a5\uc73c\ub85c \ucc44\ud0dd\ud558\uc9c0\ub294 \uc54a\uc744 \uac83\"\uc774\ub77c\ub294 \ub73b\uc744 \uc804\ub2ec\ud588\ub2e4. \uc774\ub294 \uc544\ubca0 \ucd1d\ub9ac\uc758 \ud3c9\ucc3d \ub3d9\uacc4\uc62c\ub9bc\ud53d \ucc38\uc11d\uc744 \uc694\uccad\ud558\uba74\uc11c \ud55c\uc77c \uad00\uacc4\uc758 \ubbf8\ub798\ub97c \uc704\ud574 \ud30c\uad6d\uc801 \uacb0\uc815\uc744 \ud558\uc9c0 \uc54a\uc744 \ub73b\uc744 \ub0b4\ube44\uce5c \uac83\uc73c\ub85c \ubcf4\uc778\ub2e4.", "sentence_answer": " \uc704\uc548\ubd80 \ud611\uc0c1 \ubb38\uc81c \ub85c \ud55c\uc77c \uad00\uacc4\ub294 \ubb38\uc7ac\uc778 \uc815\ubd80 \ucd9c\ubc94\uc744 \uc804\ud6c4\ud558\uc5ec \uc624\ub7ab\ub3d9\uc548 \ub0c9\uac01\ub418\uc5b4 \uc788\uc5c8\ub2e4.", "paragraph_id": "6580792-13-0", "paragraph_question": "question: \uc624\ub7ab\ub3d9\uc548 \ub0c9\uac01\ub418\uc5b4 \uc788\ub358 \ud55c\uc77c \uad00\uacc4\uc758 \ubb38\uc81c\ub294?, context: \uc704\uc548\ubd80 \ud611\uc0c1 \ubb38\uc81c\ub85c \ud55c\uc77c \uad00\uacc4\ub294 \ubb38\uc7ac\uc778 \uc815\ubd80 \ucd9c\ubc94\uc744 \uc804\ud6c4\ud558\uc5ec \uc624\ub7ab\ub3d9\uc548 \ub0c9\uac01\ub418\uc5b4 \uc788\uc5c8\ub2e4. \uc774\ub97c \ud574\uacb0\ud558\uace0\uc790 2017\ub144 12\uc6d4 19\uc77c \uac15\uacbd\ud654 \uc678\uad50\ubd80 \uc7a5\uad00\uc774 1\ubc15 2\uc77c \uac04\uc758 \ubc29\uc77c\uc744 \ud558\uc600\ub2e4. \uc544\ubca0 \uc2e0\uc870 \ucd1d\ub9ac\ub300\uc2e0\uacfc \uace0\ub178 \ub2e4\ub85c \uc678\ubb34\ub300\uc2e0\uc744 \ub9cc\ub09c \uc790\ub9ac\uc5d0\uc11c \uac15\uacbd\ud654\ub294 \"\uacfc\uac70\uc0ac\ub85c\ubd80\ud130 \ube44\ub86f\ub418\ub294 \uc5b4\ub824\uc6b4 \ubb38\uc81c\ub4e4\uc774 \uc788\uc9c0\ub9cc, \uadf8\ub7ec\ud55c \uc5b4\ub824\uc6c0\ub4e4\uc740 \uae34\ubc00\ud55c \uc18c\ud1b5\uc744 \ud1b5\ud574 \uc9c0\ud61c\ub86d\uac8c \uadf9\ubcf5\ud574 \ub098\uac00\uae38 \ud76c\ub9dd\ud55c\ub2e4\"\ub294 \ub300\ud1b5\ub839\uc758 \uba54\uc2dc\uc9c0\ub97c \uc804\ub2ec\ud588\uace0 \uc544\ubca0\ub294 \"\ud55c\uad6d\uc740 \uc804\ub7b5\uc801 \uc774\uc775\uc744 \uacf5\uc720\ud558\ub294, \uc77c\ubcf8\uc5d0 \uac00\uc7a5 \uc911\uc694\ud55c \uc774\uc6c3\"\uc774\ub77c\uace0 \ud654\ub2f5\ud588\ub2e4. \uc911\uc694 \ud604\uc548\uc73c\ub85c\ub294 \ubd81\ud575 \ubb38\uc81c\uc640 \ud568\uaed8 \uc704\uc548\ubd80 \uc598\uae30\uac00 \ub2e4\ub904\uc84c\ub294\ub370, \uace0\ub178 \ub300\uc2e0\uc740 \"\ud55c\u00b7\uc77c \uc704\uc548\ubd80 \ud569\uc758\uc758 \ucc29\uc2e4\ud55c \uc774\ud589\uc774 \uc911\uc694\ud558\ub2e4\"\uace0 \uac15\uc870\ud588\uace0 \uac15 \uc7a5\uad00\uc740 \"(27\uc77c \ubc1c\ud45c\ub418\ub294 TF) \ubcf4\uace0\uc11c \uacb0\uacfc\ub97c \uadf8\ub300\ub85c \ud55c\uad6d \uc815\ubd80\uc758 \uc785\uc7a5\uc73c\ub85c \ucc44\ud0dd\ud558\uc9c0\ub294 \uc54a\uc744 \uac83\"\uc774\ub77c\ub294 \ub73b\uc744 \uc804\ub2ec\ud588\ub2e4. \uc774\ub294 \uc544\ubca0 \ucd1d\ub9ac\uc758 \ud3c9\ucc3d \ub3d9\uacc4\uc62c\ub9bc\ud53d \ucc38\uc11d\uc744 \uc694\uccad\ud558\uba74\uc11c \ud55c\uc77c \uad00\uacc4\uc758 \ubbf8\ub798\ub97c \uc704\ud574 \ud30c\uad6d\uc801 \uacb0\uc815\uc744 \ud558\uc9c0 \uc54a\uc744 \ub73b\uc744 \ub0b4\ube44\uce5c \uac83\uc73c\ub85c \ubcf4\uc778\ub2e4."} {"question": "\uc5b8\ud504\ub9ac\ud2f0 \ub7a9\uc2a4\ud0c0 3 \uc81c 1\ud654\uc758 \ubbf8\uc158\uc5d0\uc11c \ucd5c\uace0\uc758 \ub798\ud37c\ub85c \ubf51\ud78c \uc0ac\ub78c\uc740?", "paragraph": "\ud504\ub85c\ub4c0\uc11c\uc778 \uae38\uc774 \uc120\uc815\ud55c \uc138 \uace1\uc758 \ud799\ud569 \uace1\uc744 \uc138 \ud300\uc73c\ub85c \ub098\ub220\uc11c \uacf5\uc5f0\ud558\ub294 \ubbf8\uc158\uc774 \uc8fc\uc6cc\uc9c4\ub2e4. \uacf5\uc5f0\uc5d0\uc11c \uad00\uac1d\ub4e4\ub85c\ubd80\ud130 \uac00\uc7a5 \ub9ce\uc740 \ud45c\ub97c \ubc1b\uc740 1\ub4f1 \ud300\uc740 \uc804\uc6d0 \uc194\ub85c\uc5d0 \uc9c4\ucd9c\ud558\uace0, 2\ub4f1 \ud300\uc5d0\uc11c\ub294 1\uba85\ub9cc\uc774 \uc194\ub85c \ubc30\ud2c0\uc5d0 \uc9c4\ucd9c\ud55c\ub2e4. \uae38\uc740 \ud5c8\ub2c8 \ud328\ubc00\ub9ac\uc758 \u3008\ub0a8\uc790 \uc774\uc57c\uae30\u3009, \ub9ac\uc30d\uc758 \u3008\ub0b4\uac00 \uc6c3\ub294\uac8c \uc544\ub2c8\uc57c\u3009, \ubcf4\uc774\ube44\uc758 \u3008\ud638\ub791\ub098\ube44\u3009\ub97c \uacf5\uc5f0\ud560 \uace1\uc73c\ub85c \uc81c\uc2dc\ud55c\ub2e4. \uc81c1\ud654\uc758 \ubbf8\uc158\uc5d0\uc11c \ucd5c\uace0\uc758 \ub798\ud37c\ub85c \ubf51\ud78c \uc790\uc774\uc5b8\ud2b8 \ud551\ud06c\uc5d0\uac8c \ub178\ub798\uc640 \ud300\uc6d0\uc744 \uc120\ud0dd\ud560 \uc218 \uc788\ub294 \uad8c\ud55c\uc774 \uc8fc\uc6cc\uc9c4\ub2e4. \uc790\uc774\uc5b8\ud2b8 \ud551\ud06c\ub294 \ud300\uc6d0\uc73c\ub85c \uc721\uc9c0\ub2f4\uacfc \ub098\ub2e4\ub97c \uc120\ud0dd\ud558\uace0 \u3008\ud638\ub791\ub098\ube44\u3009\ub97c \uacf5\uc5f0 \uace1\uc73c\ub85c \uc120\ud0dd\ud55c\ub2e4. \uc804\uc18c\uc5f0, \ubbf8\ub8cc, \ud558\uc8fc\uc5f0\uc774 \ud55c \ud300\uc73c\ub85c, \uadf8\ub808\uc774\uc2a4, \uc720\ub098\ud0b4, \ucf00\uc774\uc2dc\uac00 \ud55c \ud300\uc73c\ub85c \ubb36\uc774\uace0, \uacf5\uc5f0 \uace1\uc73c\ub85c \uac01\uac01 \u3008\ub0a8\uc790 \uc774\uc57c\uae30\u3009, \u3008\ub0b4\uac00 \uc6c3\ub294 \uac8c \uc544\ub2c8\uc57c\u3009\ub97c \uc120\ud0dd\ud55c\ub2e4. \uc544\ubb34\uc5d0\uac8c\ub3c4 \uc120\ud0dd\ubc1b\uc9c0 \ubabb\ud55c \uc81c\uc774\ub2c8\ub294 \uc790\uc774\uc5b8\ud2b8 \ud551\ud06c\uc758 \uad8c\ud55c\uc73c\ub85c \uadf8\ub808\uc774\uc2a4 \ud300\uc5d0 \ub4e4\uc5b4\uac04\ub2e4. \uacf5\uc5f0\uc5d0\uc11c \uc790\uc774\uc5b8\ud2b8 \ud551\ud06c \ud300\uc774 1\uc704\ub97c \ud558\uc5ec \ud300\uc6d0 \uc804\uc6d0\uc774 \uc194\ub85c \ubc30\ud2c0\uc5d0 \uc9c4\ucd9c\ud558\uace0, \ud504\ub85c\ub4c0\uc11c \uae38\uc740 2\ub4f1\uc744 \ud55c \ubbf8\ub8cc \ud300\uc5d0\uc11c \ubbf8\ub8cc\ub97c \uc194\ub85c \ubc30\ud2c0\uc5d0 \uc9c4\ucd9c\ud560 \ub798\ud37c\ub85c \uc120\ud0dd\ud55c\ub2e4. \uc194\ub85c \ubc30\ud2c0\uc5d0\uc11c \ud504\ub85c\ub4c0\uc11c \uae38\uacfc \ud53c\ucc98\ub9c1 \ub798\ud37c \ub9e4\ub4dc \ud074\ub77c\uc6b4\uc740 \uc721\uc9c0\ub2f4\uc744 \uc6b0\uc2b9\uc790\ub85c \uc120\uc815\ud55c\ub2e4.", "answer": "\uc790\uc774\uc5b8\ud2b8 \ud551\ud06c", "sentence": "\uc81c1\ud654\uc758 \ubbf8\uc158\uc5d0\uc11c \ucd5c\uace0\uc758 \ub798\ud37c\ub85c \ubf51\ud78c \uc790\uc774\uc5b8\ud2b8 \ud551\ud06c \uc5d0\uac8c \ub178\ub798\uc640 \ud300\uc6d0\uc744 \uc120\ud0dd\ud560 \uc218 \uc788\ub294 \uad8c\ud55c\uc774 \uc8fc\uc6cc\uc9c4\ub2e4.", "paragraph_sentence": "\ud504\ub85c\ub4c0\uc11c\uc778 \uae38\uc774 \uc120\uc815\ud55c \uc138 \uace1\uc758 \ud799\ud569 \uace1\uc744 \uc138 \ud300\uc73c\ub85c \ub098\ub220\uc11c \uacf5\uc5f0\ud558\ub294 \ubbf8\uc158\uc774 \uc8fc\uc6cc\uc9c4\ub2e4. \uacf5\uc5f0\uc5d0\uc11c \uad00\uac1d\ub4e4\ub85c\ubd80\ud130 \uac00\uc7a5 \ub9ce\uc740 \ud45c\ub97c \ubc1b\uc740 1\ub4f1 \ud300\uc740 \uc804\uc6d0 \uc194\ub85c\uc5d0 \uc9c4\ucd9c\ud558\uace0, 2\ub4f1 \ud300\uc5d0\uc11c\ub294 1\uba85\ub9cc\uc774 \uc194\ub85c \ubc30\ud2c0\uc5d0 \uc9c4\ucd9c\ud55c\ub2e4. \uae38\uc740 \ud5c8\ub2c8 \ud328\ubc00\ub9ac\uc758 \u3008\ub0a8\uc790 \uc774\uc57c\uae30\u3009, \ub9ac\uc30d\uc758 \u3008\ub0b4\uac00 \uc6c3\ub294\uac8c \uc544\ub2c8\uc57c\u3009, \ubcf4\uc774\ube44\uc758 \u3008\ud638\ub791\ub098\ube44\u3009\ub97c \uacf5\uc5f0\ud560 \uace1\uc73c\ub85c \uc81c\uc2dc\ud55c\ub2e4. \uc81c1\ud654\uc758 \ubbf8\uc158\uc5d0\uc11c \ucd5c\uace0\uc758 \ub798\ud37c\ub85c \ubf51\ud78c \uc790\uc774\uc5b8\ud2b8 \ud551\ud06c \uc5d0\uac8c \ub178\ub798\uc640 \ud300\uc6d0\uc744 \uc120\ud0dd\ud560 \uc218 \uc788\ub294 \uad8c\ud55c\uc774 \uc8fc\uc6cc\uc9c4\ub2e4. \uc790\uc774\uc5b8\ud2b8 \ud551\ud06c\ub294 \ud300\uc6d0\uc73c\ub85c \uc721\uc9c0\ub2f4\uacfc \ub098\ub2e4\ub97c \uc120\ud0dd\ud558\uace0 \u3008\ud638\ub791\ub098\ube44\u3009\ub97c \uacf5\uc5f0 \uace1\uc73c\ub85c \uc120\ud0dd\ud55c\ub2e4. \uc804\uc18c\uc5f0, \ubbf8\ub8cc, \ud558\uc8fc\uc5f0\uc774 \ud55c \ud300\uc73c\ub85c, \uadf8\ub808\uc774\uc2a4, \uc720\ub098\ud0b4, \ucf00\uc774\uc2dc\uac00 \ud55c \ud300\uc73c\ub85c \ubb36\uc774\uace0, \uacf5\uc5f0 \uace1\uc73c\ub85c \uac01\uac01 \u3008\ub0a8\uc790 \uc774\uc57c\uae30\u3009, \u3008\ub0b4\uac00 \uc6c3\ub294 \uac8c \uc544\ub2c8\uc57c\u3009\ub97c \uc120\ud0dd\ud55c\ub2e4. \uc544\ubb34\uc5d0\uac8c\ub3c4 \uc120\ud0dd\ubc1b\uc9c0 \ubabb\ud55c \uc81c\uc774\ub2c8\ub294 \uc790\uc774\uc5b8\ud2b8 \ud551\ud06c\uc758 \uad8c\ud55c\uc73c\ub85c \uadf8\ub808\uc774\uc2a4 \ud300\uc5d0 \ub4e4\uc5b4\uac04\ub2e4. \uacf5\uc5f0\uc5d0\uc11c \uc790\uc774\uc5b8\ud2b8 \ud551\ud06c \ud300\uc774 1\uc704\ub97c \ud558\uc5ec \ud300\uc6d0 \uc804\uc6d0\uc774 \uc194\ub85c \ubc30\ud2c0\uc5d0 \uc9c4\ucd9c\ud558\uace0, \ud504\ub85c\ub4c0\uc11c \uae38\uc740 2\ub4f1\uc744 \ud55c \ubbf8\ub8cc \ud300\uc5d0\uc11c \ubbf8\ub8cc\ub97c \uc194\ub85c \ubc30\ud2c0\uc5d0 \uc9c4\ucd9c\ud560 \ub798\ud37c\ub85c \uc120\ud0dd\ud55c\ub2e4. \uc194\ub85c \ubc30\ud2c0\uc5d0\uc11c \ud504\ub85c\ub4c0\uc11c \uae38\uacfc \ud53c\ucc98\ub9c1 \ub798\ud37c \ub9e4\ub4dc \ud074\ub77c\uc6b4\uc740 \uc721\uc9c0\ub2f4\uc744 \uc6b0\uc2b9\uc790\ub85c \uc120\uc815\ud55c\ub2e4.", "paragraph_answer": "\ud504\ub85c\ub4c0\uc11c\uc778 \uae38\uc774 \uc120\uc815\ud55c \uc138 \uace1\uc758 \ud799\ud569 \uace1\uc744 \uc138 \ud300\uc73c\ub85c \ub098\ub220\uc11c \uacf5\uc5f0\ud558\ub294 \ubbf8\uc158\uc774 \uc8fc\uc6cc\uc9c4\ub2e4. \uacf5\uc5f0\uc5d0\uc11c \uad00\uac1d\ub4e4\ub85c\ubd80\ud130 \uac00\uc7a5 \ub9ce\uc740 \ud45c\ub97c \ubc1b\uc740 1\ub4f1 \ud300\uc740 \uc804\uc6d0 \uc194\ub85c\uc5d0 \uc9c4\ucd9c\ud558\uace0, 2\ub4f1 \ud300\uc5d0\uc11c\ub294 1\uba85\ub9cc\uc774 \uc194\ub85c \ubc30\ud2c0\uc5d0 \uc9c4\ucd9c\ud55c\ub2e4. \uae38\uc740 \ud5c8\ub2c8 \ud328\ubc00\ub9ac\uc758 \u3008\ub0a8\uc790 \uc774\uc57c\uae30\u3009, \ub9ac\uc30d\uc758 \u3008\ub0b4\uac00 \uc6c3\ub294\uac8c \uc544\ub2c8\uc57c\u3009, \ubcf4\uc774\ube44\uc758 \u3008\ud638\ub791\ub098\ube44\u3009\ub97c \uacf5\uc5f0\ud560 \uace1\uc73c\ub85c \uc81c\uc2dc\ud55c\ub2e4. \uc81c1\ud654\uc758 \ubbf8\uc158\uc5d0\uc11c \ucd5c\uace0\uc758 \ub798\ud37c\ub85c \ubf51\ud78c \uc790\uc774\uc5b8\ud2b8 \ud551\ud06c \uc5d0\uac8c \ub178\ub798\uc640 \ud300\uc6d0\uc744 \uc120\ud0dd\ud560 \uc218 \uc788\ub294 \uad8c\ud55c\uc774 \uc8fc\uc6cc\uc9c4\ub2e4. \uc790\uc774\uc5b8\ud2b8 \ud551\ud06c\ub294 \ud300\uc6d0\uc73c\ub85c \uc721\uc9c0\ub2f4\uacfc \ub098\ub2e4\ub97c \uc120\ud0dd\ud558\uace0 \u3008\ud638\ub791\ub098\ube44\u3009\ub97c \uacf5\uc5f0 \uace1\uc73c\ub85c \uc120\ud0dd\ud55c\ub2e4. \uc804\uc18c\uc5f0, \ubbf8\ub8cc, \ud558\uc8fc\uc5f0\uc774 \ud55c \ud300\uc73c\ub85c, \uadf8\ub808\uc774\uc2a4, \uc720\ub098\ud0b4, \ucf00\uc774\uc2dc\uac00 \ud55c \ud300\uc73c\ub85c \ubb36\uc774\uace0, \uacf5\uc5f0 \uace1\uc73c\ub85c \uac01\uac01 \u3008\ub0a8\uc790 \uc774\uc57c\uae30\u3009, \u3008\ub0b4\uac00 \uc6c3\ub294 \uac8c \uc544\ub2c8\uc57c\u3009\ub97c \uc120\ud0dd\ud55c\ub2e4. \uc544\ubb34\uc5d0\uac8c\ub3c4 \uc120\ud0dd\ubc1b\uc9c0 \ubabb\ud55c \uc81c\uc774\ub2c8\ub294 \uc790\uc774\uc5b8\ud2b8 \ud551\ud06c\uc758 \uad8c\ud55c\uc73c\ub85c \uadf8\ub808\uc774\uc2a4 \ud300\uc5d0 \ub4e4\uc5b4\uac04\ub2e4. \uacf5\uc5f0\uc5d0\uc11c \uc790\uc774\uc5b8\ud2b8 \ud551\ud06c \ud300\uc774 1\uc704\ub97c \ud558\uc5ec \ud300\uc6d0 \uc804\uc6d0\uc774 \uc194\ub85c \ubc30\ud2c0\uc5d0 \uc9c4\ucd9c\ud558\uace0, \ud504\ub85c\ub4c0\uc11c \uae38\uc740 2\ub4f1\uc744 \ud55c \ubbf8\ub8cc \ud300\uc5d0\uc11c \ubbf8\ub8cc\ub97c \uc194\ub85c \ubc30\ud2c0\uc5d0 \uc9c4\ucd9c\ud560 \ub798\ud37c\ub85c \uc120\ud0dd\ud55c\ub2e4. \uc194\ub85c \ubc30\ud2c0\uc5d0\uc11c \ud504\ub85c\ub4c0\uc11c \uae38\uacfc \ud53c\ucc98\ub9c1 \ub798\ud37c \ub9e4\ub4dc \ud074\ub77c\uc6b4\uc740 \uc721\uc9c0\ub2f4\uc744 \uc6b0\uc2b9\uc790\ub85c \uc120\uc815\ud55c\ub2e4.", "sentence_answer": "\uc81c1\ud654\uc758 \ubbf8\uc158\uc5d0\uc11c \ucd5c\uace0\uc758 \ub798\ud37c\ub85c \ubf51\ud78c \uc790\uc774\uc5b8\ud2b8 \ud551\ud06c \uc5d0\uac8c \ub178\ub798\uc640 \ud300\uc6d0\uc744 \uc120\ud0dd\ud560 \uc218 \uc788\ub294 \uad8c\ud55c\uc774 \uc8fc\uc6cc\uc9c4\ub2e4.", "paragraph_id": "6657344-1-1", "paragraph_question": "question: \uc5b8\ud504\ub9ac\ud2f0 \ub7a9\uc2a4\ud0c0 3 \uc81c 1\ud654\uc758 \ubbf8\uc158\uc5d0\uc11c \ucd5c\uace0\uc758 \ub798\ud37c\ub85c \ubf51\ud78c \uc0ac\ub78c\uc740?, context: \ud504\ub85c\ub4c0\uc11c\uc778 \uae38\uc774 \uc120\uc815\ud55c \uc138 \uace1\uc758 \ud799\ud569 \uace1\uc744 \uc138 \ud300\uc73c\ub85c \ub098\ub220\uc11c \uacf5\uc5f0\ud558\ub294 \ubbf8\uc158\uc774 \uc8fc\uc6cc\uc9c4\ub2e4. \uacf5\uc5f0\uc5d0\uc11c \uad00\uac1d\ub4e4\ub85c\ubd80\ud130 \uac00\uc7a5 \ub9ce\uc740 \ud45c\ub97c \ubc1b\uc740 1\ub4f1 \ud300\uc740 \uc804\uc6d0 \uc194\ub85c\uc5d0 \uc9c4\ucd9c\ud558\uace0, 2\ub4f1 \ud300\uc5d0\uc11c\ub294 1\uba85\ub9cc\uc774 \uc194\ub85c \ubc30\ud2c0\uc5d0 \uc9c4\ucd9c\ud55c\ub2e4. \uae38\uc740 \ud5c8\ub2c8 \ud328\ubc00\ub9ac\uc758 \u3008\ub0a8\uc790 \uc774\uc57c\uae30\u3009, \ub9ac\uc30d\uc758 \u3008\ub0b4\uac00 \uc6c3\ub294\uac8c \uc544\ub2c8\uc57c\u3009, \ubcf4\uc774\ube44\uc758 \u3008\ud638\ub791\ub098\ube44\u3009\ub97c \uacf5\uc5f0\ud560 \uace1\uc73c\ub85c \uc81c\uc2dc\ud55c\ub2e4. \uc81c1\ud654\uc758 \ubbf8\uc158\uc5d0\uc11c \ucd5c\uace0\uc758 \ub798\ud37c\ub85c \ubf51\ud78c \uc790\uc774\uc5b8\ud2b8 \ud551\ud06c\uc5d0\uac8c \ub178\ub798\uc640 \ud300\uc6d0\uc744 \uc120\ud0dd\ud560 \uc218 \uc788\ub294 \uad8c\ud55c\uc774 \uc8fc\uc6cc\uc9c4\ub2e4. \uc790\uc774\uc5b8\ud2b8 \ud551\ud06c\ub294 \ud300\uc6d0\uc73c\ub85c \uc721\uc9c0\ub2f4\uacfc \ub098\ub2e4\ub97c \uc120\ud0dd\ud558\uace0 \u3008\ud638\ub791\ub098\ube44\u3009\ub97c \uacf5\uc5f0 \uace1\uc73c\ub85c \uc120\ud0dd\ud55c\ub2e4. \uc804\uc18c\uc5f0, \ubbf8\ub8cc, \ud558\uc8fc\uc5f0\uc774 \ud55c \ud300\uc73c\ub85c, \uadf8\ub808\uc774\uc2a4, \uc720\ub098\ud0b4, \ucf00\uc774\uc2dc\uac00 \ud55c \ud300\uc73c\ub85c \ubb36\uc774\uace0, \uacf5\uc5f0 \uace1\uc73c\ub85c \uac01\uac01 \u3008\ub0a8\uc790 \uc774\uc57c\uae30\u3009, \u3008\ub0b4\uac00 \uc6c3\ub294 \uac8c \uc544\ub2c8\uc57c\u3009\ub97c \uc120\ud0dd\ud55c\ub2e4. \uc544\ubb34\uc5d0\uac8c\ub3c4 \uc120\ud0dd\ubc1b\uc9c0 \ubabb\ud55c \uc81c\uc774\ub2c8\ub294 \uc790\uc774\uc5b8\ud2b8 \ud551\ud06c\uc758 \uad8c\ud55c\uc73c\ub85c \uadf8\ub808\uc774\uc2a4 \ud300\uc5d0 \ub4e4\uc5b4\uac04\ub2e4. \uacf5\uc5f0\uc5d0\uc11c \uc790\uc774\uc5b8\ud2b8 \ud551\ud06c \ud300\uc774 1\uc704\ub97c \ud558\uc5ec \ud300\uc6d0 \uc804\uc6d0\uc774 \uc194\ub85c \ubc30\ud2c0\uc5d0 \uc9c4\ucd9c\ud558\uace0, \ud504\ub85c\ub4c0\uc11c \uae38\uc740 2\ub4f1\uc744 \ud55c \ubbf8\ub8cc \ud300\uc5d0\uc11c \ubbf8\ub8cc\ub97c \uc194\ub85c \ubc30\ud2c0\uc5d0 \uc9c4\ucd9c\ud560 \ub798\ud37c\ub85c \uc120\ud0dd\ud55c\ub2e4. \uc194\ub85c \ubc30\ud2c0\uc5d0\uc11c \ud504\ub85c\ub4c0\uc11c \uae38\uacfc \ud53c\ucc98\ub9c1 \ub798\ud37c \ub9e4\ub4dc \ud074\ub77c\uc6b4\uc740 \uc721\uc9c0\ub2f4\uc744 \uc6b0\uc2b9\uc790\ub85c \uc120\uc815\ud55c\ub2e4."} {"question": "\uc0b0\uc11c \uc131\uc5d0\uc11c \uccad\uad70\uc5d0\uac8c \uad70\ub300\ub97c \ucca0\uc218\ud558\ub77c\uace0 \ud55c \uc778\ubb3c\uc740?", "paragraph": "\uac15\uc758(\u525b\u6bc5)\ub294 \uadf8\uacf3\uc5d0\uc11c \uc758\ud654\ub2e8\uc744 \uc9c4\uc555\ud558\ub294 \uccad\uad70\uc5d0\uac8c '\ud1a0\ubc8c\uc744 \uc911\uc9c0\ud1a0\ub85d \ud558\uace0 \uad70\ub300\ub97c \ucca0\uc218\ud558\ub77c' \uc870\uce58\ub97c \ucde8\ud558\uc600\ub2e4. \uadf8\ub9ac\uace0 \ubca0\uc774\uc9d5\uc73c\ub85c \uadc0\uacbd\ud558\uc5ec '\uc758\ud654\ub2e8\uc744 \ud574\uc0b0\ud560 \uc218 \uc5c6\ub2e4'\uace0 \uac74\uc758\ud558\uc600\ub2e4. \uadf8\ub7f0\ub370 \ud55c\ud3b8, \uc11c\uad6c \uc5f4\uac15\uc5d0\uc11c \ubca0\uc774\uc9d5 \uc8fc\uc7ac \uc678\uad6d \uacf5\uc0ac\uad00\uacfc \uc678\uad6d\uc778\ub4e4\uc758 \uc548\uc804\uc744 \uc6b0\ub824\ud558\uac8c \ub418\uc5c8\ub2e4. 6\uc6d4 10\uc77c \uc601\uad6d\uc758 \uc8fc\uc7a5\uc544\ub798 \uac01 \uad6d\uc758 \uad70\ub300\ub97c \ub354 \ub3d9\uc6d0\ud55c\ub2e4\uace0 \ubc1c\ud45c\ud558\uace0, \uc601\uad6d \uacf5\uc0ac\ub294 \ud188\uc9c4\uc5d0 \uc788\ub358 \uc601\uad6d\uad70 \ud574\uad70 \uc81c\ub3c5 \uc2dc\ubaa8\uc5b4(E.H. Seymour)\ub97c \uc9c0\ud718\uad00\uc73c\ub85c \uc5f0\ud569\uad70\uc744 \uc870\uc9c1\ud558\uc5ec 2129\uba85\uc774 \uae30\ucc28\ub85c \ud188\uc9c4\uc744 \ub5a0\ub098 \ubca0\uc774\uc9d5\uc73c\ub85c \ud5a5\ud558\uac8c \ub418\uc5c8\ub2e4. \uccad\uc870\uc5d0\uc11c\ub294 \uc774\ub97c \uc800\uc9c0\ud558\ub77c\uace0 \uc720\ub85d\uc5d0\uac8c \uc9c0\uc2dc\ud558\uace0 \uc6d0\ub798 \uc758\ud654\ub2e8\uc744 \uc9c4\uc555\ud558\uace0 \uc788\ub358 \uc12d\uc0ac\uc131 \ubd80\ub300\ub3c4 \uc774\ub4e4\uc758 \ubca0\uc774\uc9d5 \uc9c4\uc785\uc744 \uc800\uc9c0\ud558\ub3c4\ub85d \uc774\ub3d9\uc2dc\ucf30\ub2e4. \uc774\ub294 \uc11c\ud0dc\ud6c4\uc758 \ud0dc\ub3c4\uac00 \ud06c\uac8c \ubc14\ub00c\uc5b4 \uc804\uc7c1 \uc900\ube44\ub85c \ub4e4\uc5b4\uac04 \uac83\uc774\ub2e4. \ub530\ub77c\uc11c \uc601\uad6d\uad70 \ud574\uad70 \uc81c\ub3c5 \uc2dc\ubaa8\uc5b4(E.H. Seymour)\uac00 \uc9c0\ud718\ud55c \uad70\ub300\ub294 5\uc77c \ub3d9\uc548\uc5d0 \ud188\uc9c4\uc5d0\uc11c \ubca0\uc774\uc9d5\uae4c\uc9c0 \ubc18\ub3c4 \ub098\uac00\uc9c0 \ubabb\ud588\uc744 \ubfd0\ub9cc \uc544\ub2c8\ub77c \uc624\ud788\ub824 \ubb3c\ub7ec\ub098\uc57c \ud558\uc600\ub2e4.", "answer": "\uac15\uc758", "sentence": "\uac15\uc758 (\u525b\u6bc5)", "paragraph_sentence": " \uac15\uc758 (\u525b\u6bc5) \ub294 \uadf8\uacf3\uc5d0\uc11c \uc758\ud654\ub2e8\uc744 \uc9c4\uc555\ud558\ub294 \uccad\uad70\uc5d0\uac8c '\ud1a0\ubc8c\uc744 \uc911\uc9c0\ud1a0\ub85d \ud558\uace0 \uad70\ub300\ub97c \ucca0\uc218\ud558\ub77c' \uc870\uce58\ub97c \ucde8\ud558\uc600\ub2e4. \uadf8\ub9ac\uace0 \ubca0\uc774\uc9d5\uc73c\ub85c \uadc0\uacbd\ud558\uc5ec '\uc758\ud654\ub2e8\uc744 \ud574\uc0b0\ud560 \uc218 \uc5c6\ub2e4'\uace0 \uac74\uc758\ud558\uc600\ub2e4. \uadf8\ub7f0\ub370 \ud55c\ud3b8, \uc11c\uad6c \uc5f4\uac15\uc5d0\uc11c \ubca0\uc774\uc9d5 \uc8fc\uc7ac \uc678\uad6d \uacf5\uc0ac\uad00\uacfc \uc678\uad6d\uc778\ub4e4\uc758 \uc548\uc804\uc744 \uc6b0\ub824\ud558\uac8c \ub418\uc5c8\ub2e4. 6\uc6d4 10\uc77c \uc601\uad6d\uc758 \uc8fc\uc7a5\uc544\ub798 \uac01 \uad6d\uc758 \uad70\ub300\ub97c \ub354 \ub3d9\uc6d0\ud55c\ub2e4\uace0 \ubc1c\ud45c\ud558\uace0, \uc601\uad6d \uacf5\uc0ac\ub294 \ud188\uc9c4\uc5d0 \uc788\ub358 \uc601\uad6d\uad70 \ud574\uad70 \uc81c\ub3c5 \uc2dc\ubaa8\uc5b4(E.H. Seymour)\ub97c \uc9c0\ud718\uad00\uc73c\ub85c \uc5f0\ud569\uad70\uc744 \uc870\uc9c1\ud558\uc5ec 2129\uba85\uc774 \uae30\ucc28\ub85c \ud188\uc9c4\uc744 \ub5a0\ub098 \ubca0\uc774\uc9d5\uc73c\ub85c \ud5a5\ud558\uac8c \ub418\uc5c8\ub2e4. \uccad\uc870\uc5d0\uc11c\ub294 \uc774\ub97c \uc800\uc9c0\ud558\ub77c\uace0 \uc720\ub85d\uc5d0\uac8c \uc9c0\uc2dc\ud558\uace0 \uc6d0\ub798 \uc758\ud654\ub2e8\uc744 \uc9c4\uc555\ud558\uace0 \uc788\ub358 \uc12d\uc0ac\uc131 \ubd80\ub300\ub3c4 \uc774\ub4e4\uc758 \ubca0\uc774\uc9d5 \uc9c4\uc785\uc744 \uc800\uc9c0\ud558\ub3c4\ub85d \uc774\ub3d9\uc2dc\ucf30\ub2e4. \uc774\ub294 \uc11c\ud0dc\ud6c4\uc758 \ud0dc\ub3c4\uac00 \ud06c\uac8c \ubc14\ub00c\uc5b4 \uc804\uc7c1 \uc900\ube44\ub85c \ub4e4\uc5b4\uac04 \uac83\uc774\ub2e4. \ub530\ub77c\uc11c \uc601\uad6d\uad70 \ud574\uad70 \uc81c\ub3c5 \uc2dc\ubaa8\uc5b4(E.H. Seymour)\uac00 \uc9c0\ud718\ud55c \uad70\ub300\ub294 5\uc77c \ub3d9\uc548\uc5d0 \ud188\uc9c4\uc5d0\uc11c \ubca0\uc774\uc9d5\uae4c\uc9c0 \ubc18\ub3c4 \ub098\uac00\uc9c0 \ubabb\ud588\uc744 \ubfd0\ub9cc \uc544\ub2c8\ub77c \uc624\ud788\ub824 \ubb3c\ub7ec\ub098\uc57c \ud558\uc600\ub2e4.", "paragraph_answer": " \uac15\uc758 (\u525b\u6bc5)\ub294 \uadf8\uacf3\uc5d0\uc11c \uc758\ud654\ub2e8\uc744 \uc9c4\uc555\ud558\ub294 \uccad\uad70\uc5d0\uac8c '\ud1a0\ubc8c\uc744 \uc911\uc9c0\ud1a0\ub85d \ud558\uace0 \uad70\ub300\ub97c \ucca0\uc218\ud558\ub77c' \uc870\uce58\ub97c \ucde8\ud558\uc600\ub2e4. \uadf8\ub9ac\uace0 \ubca0\uc774\uc9d5\uc73c\ub85c \uadc0\uacbd\ud558\uc5ec '\uc758\ud654\ub2e8\uc744 \ud574\uc0b0\ud560 \uc218 \uc5c6\ub2e4'\uace0 \uac74\uc758\ud558\uc600\ub2e4. \uadf8\ub7f0\ub370 \ud55c\ud3b8, \uc11c\uad6c \uc5f4\uac15\uc5d0\uc11c \ubca0\uc774\uc9d5 \uc8fc\uc7ac \uc678\uad6d \uacf5\uc0ac\uad00\uacfc \uc678\uad6d\uc778\ub4e4\uc758 \uc548\uc804\uc744 \uc6b0\ub824\ud558\uac8c \ub418\uc5c8\ub2e4. 6\uc6d4 10\uc77c \uc601\uad6d\uc758 \uc8fc\uc7a5\uc544\ub798 \uac01 \uad6d\uc758 \uad70\ub300\ub97c \ub354 \ub3d9\uc6d0\ud55c\ub2e4\uace0 \ubc1c\ud45c\ud558\uace0, \uc601\uad6d \uacf5\uc0ac\ub294 \ud188\uc9c4\uc5d0 \uc788\ub358 \uc601\uad6d\uad70 \ud574\uad70 \uc81c\ub3c5 \uc2dc\ubaa8\uc5b4(E.H. Seymour)\ub97c \uc9c0\ud718\uad00\uc73c\ub85c \uc5f0\ud569\uad70\uc744 \uc870\uc9c1\ud558\uc5ec 2129\uba85\uc774 \uae30\ucc28\ub85c \ud188\uc9c4\uc744 \ub5a0\ub098 \ubca0\uc774\uc9d5\uc73c\ub85c \ud5a5\ud558\uac8c \ub418\uc5c8\ub2e4. \uccad\uc870\uc5d0\uc11c\ub294 \uc774\ub97c \uc800\uc9c0\ud558\ub77c\uace0 \uc720\ub85d\uc5d0\uac8c \uc9c0\uc2dc\ud558\uace0 \uc6d0\ub798 \uc758\ud654\ub2e8\uc744 \uc9c4\uc555\ud558\uace0 \uc788\ub358 \uc12d\uc0ac\uc131 \ubd80\ub300\ub3c4 \uc774\ub4e4\uc758 \ubca0\uc774\uc9d5 \uc9c4\uc785\uc744 \uc800\uc9c0\ud558\ub3c4\ub85d \uc774\ub3d9\uc2dc\ucf30\ub2e4. \uc774\ub294 \uc11c\ud0dc\ud6c4\uc758 \ud0dc\ub3c4\uac00 \ud06c\uac8c \ubc14\ub00c\uc5b4 \uc804\uc7c1 \uc900\ube44\ub85c \ub4e4\uc5b4\uac04 \uac83\uc774\ub2e4. \ub530\ub77c\uc11c \uc601\uad6d\uad70 \ud574\uad70 \uc81c\ub3c5 \uc2dc\ubaa8\uc5b4(E.H. Seymour)\uac00 \uc9c0\ud718\ud55c \uad70\ub300\ub294 5\uc77c \ub3d9\uc548\uc5d0 \ud188\uc9c4\uc5d0\uc11c \ubca0\uc774\uc9d5\uae4c\uc9c0 \ubc18\ub3c4 \ub098\uac00\uc9c0 \ubabb\ud588\uc744 \ubfd0\ub9cc \uc544\ub2c8\ub77c \uc624\ud788\ub824 \ubb3c\ub7ec\ub098\uc57c \ud558\uc600\ub2e4.", "sentence_answer": " \uac15\uc758 (\u525b\u6bc5)", "paragraph_id": "6326091-10-0", "paragraph_question": "question: \uc0b0\uc11c \uc131\uc5d0\uc11c \uccad\uad70\uc5d0\uac8c \uad70\ub300\ub97c \ucca0\uc218\ud558\ub77c\uace0 \ud55c \uc778\ubb3c\uc740?, context: \uac15\uc758(\u525b\u6bc5)\ub294 \uadf8\uacf3\uc5d0\uc11c \uc758\ud654\ub2e8\uc744 \uc9c4\uc555\ud558\ub294 \uccad\uad70\uc5d0\uac8c '\ud1a0\ubc8c\uc744 \uc911\uc9c0\ud1a0\ub85d \ud558\uace0 \uad70\ub300\ub97c \ucca0\uc218\ud558\ub77c' \uc870\uce58\ub97c \ucde8\ud558\uc600\ub2e4. \uadf8\ub9ac\uace0 \ubca0\uc774\uc9d5\uc73c\ub85c \uadc0\uacbd\ud558\uc5ec '\uc758\ud654\ub2e8\uc744 \ud574\uc0b0\ud560 \uc218 \uc5c6\ub2e4'\uace0 \uac74\uc758\ud558\uc600\ub2e4. \uadf8\ub7f0\ub370 \ud55c\ud3b8, \uc11c\uad6c \uc5f4\uac15\uc5d0\uc11c \ubca0\uc774\uc9d5 \uc8fc\uc7ac \uc678\uad6d \uacf5\uc0ac\uad00\uacfc \uc678\uad6d\uc778\ub4e4\uc758 \uc548\uc804\uc744 \uc6b0\ub824\ud558\uac8c \ub418\uc5c8\ub2e4. 6\uc6d4 10\uc77c \uc601\uad6d\uc758 \uc8fc\uc7a5\uc544\ub798 \uac01 \uad6d\uc758 \uad70\ub300\ub97c \ub354 \ub3d9\uc6d0\ud55c\ub2e4\uace0 \ubc1c\ud45c\ud558\uace0, \uc601\uad6d \uacf5\uc0ac\ub294 \ud188\uc9c4\uc5d0 \uc788\ub358 \uc601\uad6d\uad70 \ud574\uad70 \uc81c\ub3c5 \uc2dc\ubaa8\uc5b4(E.H. Seymour)\ub97c \uc9c0\ud718\uad00\uc73c\ub85c \uc5f0\ud569\uad70\uc744 \uc870\uc9c1\ud558\uc5ec 2129\uba85\uc774 \uae30\ucc28\ub85c \ud188\uc9c4\uc744 \ub5a0\ub098 \ubca0\uc774\uc9d5\uc73c\ub85c \ud5a5\ud558\uac8c \ub418\uc5c8\ub2e4. \uccad\uc870\uc5d0\uc11c\ub294 \uc774\ub97c \uc800\uc9c0\ud558\ub77c\uace0 \uc720\ub85d\uc5d0\uac8c \uc9c0\uc2dc\ud558\uace0 \uc6d0\ub798 \uc758\ud654\ub2e8\uc744 \uc9c4\uc555\ud558\uace0 \uc788\ub358 \uc12d\uc0ac\uc131 \ubd80\ub300\ub3c4 \uc774\ub4e4\uc758 \ubca0\uc774\uc9d5 \uc9c4\uc785\uc744 \uc800\uc9c0\ud558\ub3c4\ub85d \uc774\ub3d9\uc2dc\ucf30\ub2e4. \uc774\ub294 \uc11c\ud0dc\ud6c4\uc758 \ud0dc\ub3c4\uac00 \ud06c\uac8c \ubc14\ub00c\uc5b4 \uc804\uc7c1 \uc900\ube44\ub85c \ub4e4\uc5b4\uac04 \uac83\uc774\ub2e4. \ub530\ub77c\uc11c \uc601\uad6d\uad70 \ud574\uad70 \uc81c\ub3c5 \uc2dc\ubaa8\uc5b4(E.H. Seymour)\uac00 \uc9c0\ud718\ud55c \uad70\ub300\ub294 5\uc77c \ub3d9\uc548\uc5d0 \ud188\uc9c4\uc5d0\uc11c \ubca0\uc774\uc9d5\uae4c\uc9c0 \ubc18\ub3c4 \ub098\uac00\uc9c0 \ubabb\ud588\uc744 \ubfd0\ub9cc \uc544\ub2c8\ub77c \uc624\ud788\ub824 \ubb3c\ub7ec\ub098\uc57c \ud558\uc600\ub2e4."} {"question": "\uadf8\ub8e8\uc9c0\uc57c SSR\uc758 \ub0b4\ubb34\ubd80 \uc7a5\uad00\uc73c\ub85c \ud2b9\ubcc4 \ubcf4\uace0\uc11c\ub97c \uc4f4 \uc0ac\ub78c\uc740?", "paragraph": "\uadf8\ub8e8\uc9c0\uc57c SSR\uc758 \ub0b4\ubb34\ubd80 \uc7a5\uad00 \ube14\ub77c\ub514\ubbf8\ub974 \uc790\ub2c8\ud30c\ubc14\uc758 \ud2b9\ubcc4 \ubcf4\uace0\uc11c\uc5d0 \ub530\ub974\uba74, \ud2b8\ube4c\ub9ac\uc2dc \ub3c4\uc2ec\uc758 \ucfe0\ub77c \uc81c\ubc29\uc5d0 \uc788\ub294 \uc2a4\ud0c8\ub9b0 \uae30\ub150\ube44 \uc55e\uc5d0\uc11c \uc2a4\ud0c8\ub9b0 \uc11c\uac70 3\uc8fc\ub144\uc744 \uae30\ub150\ud558\uae30 \uc704\ud574 \ud559\uc0dd \ub2e8\uccb4\ub4e4\uc774 \ubaa8\uc5ec\ub4e4\ub358 1956\ub144 3\uc6d4 4\uc77c\uc5d0 \uc815\uce58\uc801\uc778 \ubd88\uc548\uc774 \uc2dc\uc791\ub418\uc5c8\ub2e4\uace0 \ud55c\ub2e4. \ub2c8\ud0a4\ud0c0 \ud750\ub8e8\uc1fc\ud504\uc758 \uc5f0\uc124\uc5d0 \ubd84\uac1c\ud55c \uadf8\ub4e4\uc740 \uadf8 \uc9c0\uc5ed\uc5d0 \uc800\uc9c0\uc120\uc744 \uad6c\ucd95\ud558\uace0 \uc788\ub294 \uc778\ubbfc \uacbd\ucc30\ub4e4\uc5d0\uac8c \ud638\uc804\uc801\uc774\uc5c8\ub2e4. \uadf8\ub8e8\uc9c0\uc57c \uacf5\uc0b0\ub2f9\uc6d0 \ud30c\ub77c\uc2a4\ud2f0\uc288\ube4c\ub9ac\ub294 \uc220\uc5d0 \ucde8\ud574 \uc220\ubcd1\uc744 \ub4e0\ucc44\ub85c \uc2a4\ud0c8\ub9b0 \uae30\ub150\ube44\uc5d0 \uc62c\ub77c\uac00 \uc220\ubcd1\uc744 \uae68\uace0 \uc774\ub807\uac8c \ub9d0\ud558\uc600\ub2e4. \"\uc2a4\ud0c8\ub9b0\uc758 \uc801\ub4e4\uc740 \uc8fd\uc5b4\ubc84\ub824\ub77c. \uc774 \uc220\ubcd1\ucc98\ub7fc.\" \uc0ac\ub78c\ub4e4\uc740 \uc2a4\ud0c8\ub9b0 \uae30\ub150\ube44\uc5d0 \ud654\ud658\uc744 \ub4e4\uace0\uc654\uace0, \ub354\ub354\uc6b1 \ub9ce\uc740 \uc0ac\ub78c\ub4e4\uc744 \ubaa8\uc5ec\ub4e4\uc5c8\uc73c\uba70, \uc2dc\uc704\uc758 \uaddc\ubaa8\ub294 \uc810\uc9c4\uc801\uc73c\ub85c \ucee4\uc838\uac14\ub2e4. \ud63c\ub780\uc5d0 \ube60\uc9c4 \uc9c0\ubc29 \ub2f9\uad6d\uc740 \uadf8\ub7ec\ud55c \ud65c\ub3d9\ub4e4\uc744 \uc801\uadf9\uc801\uc73c\ub85c \ubc29\ud574\ud558\uc9c0 \uc54a\uc558\ub2e4. \uc218\ub3c4\uc5d0\uc11c\uc758 \uc2dc\uc704\ub294 \uace0\ub9ac, \ucfe0\ud0c0\uc774\uc2dc, \ub8e8\uc2a4\ud0c0\ube44, \uc218\ud6c4\ubbf8\uc640 \uac19\uc740 \uacf5\ud654\uad6d\uc758 \ub2e4\ub978 \uc9c0\uc5ed\ub4e4\uc5d0\uc11c\ub3c4 \ud761\uc0ac\ud55c \uc800\ud56d\uc5d0 \ubc29\uc544\uc1e0\ub97c \ub2f9\uacbc\ub2e4.", "answer": "\ube14\ub77c\ub514\ubbf8\ub974 \uc790\ub2c8\ud30c\ubc14", "sentence": "\uadf8\ub8e8\uc9c0\uc57c SSR\uc758 \ub0b4\ubb34\ubd80 \uc7a5\uad00 \ube14\ub77c\ub514\ubbf8\ub974 \uc790\ub2c8\ud30c\ubc14 \uc758 \ud2b9\ubcc4 \ubcf4\uace0\uc11c\uc5d0 \ub530\ub974\uba74, \ud2b8\ube4c\ub9ac\uc2dc \ub3c4\uc2ec\uc758 \ucfe0\ub77c \uc81c\ubc29\uc5d0 \uc788\ub294 \uc2a4\ud0c8\ub9b0 \uae30\ub150\ube44 \uc55e\uc5d0\uc11c \uc2a4\ud0c8\ub9b0 \uc11c\uac70 3\uc8fc\ub144\uc744 \uae30\ub150\ud558\uae30 \uc704\ud574 \ud559\uc0dd \ub2e8\uccb4\ub4e4\uc774 \ubaa8\uc5ec\ub4e4\ub358 1956\ub144 3\uc6d4 4\uc77c\uc5d0 \uc815\uce58\uc801\uc778 \ubd88\uc548\uc774 \uc2dc\uc791\ub418\uc5c8\ub2e4\uace0 \ud55c\ub2e4.", "paragraph_sentence": " \uadf8\ub8e8\uc9c0\uc57c SSR\uc758 \ub0b4\ubb34\ubd80 \uc7a5\uad00 \ube14\ub77c\ub514\ubbf8\ub974 \uc790\ub2c8\ud30c\ubc14 \uc758 \ud2b9\ubcc4 \ubcf4\uace0\uc11c\uc5d0 \ub530\ub974\uba74, \ud2b8\ube4c\ub9ac\uc2dc \ub3c4\uc2ec\uc758 \ucfe0\ub77c \uc81c\ubc29\uc5d0 \uc788\ub294 \uc2a4\ud0c8\ub9b0 \uae30\ub150\ube44 \uc55e\uc5d0\uc11c \uc2a4\ud0c8\ub9b0 \uc11c\uac70 3\uc8fc\ub144\uc744 \uae30\ub150\ud558\uae30 \uc704\ud574 \ud559\uc0dd \ub2e8\uccb4\ub4e4\uc774 \ubaa8\uc5ec\ub4e4\ub358 1956\ub144 3\uc6d4 4\uc77c\uc5d0 \uc815\uce58\uc801\uc778 \ubd88\uc548\uc774 \uc2dc\uc791\ub418\uc5c8\ub2e4\uace0 \ud55c\ub2e4. \ub2c8\ud0a4\ud0c0 \ud750\ub8e8\uc1fc\ud504\uc758 \uc5f0\uc124\uc5d0 \ubd84\uac1c\ud55c \uadf8\ub4e4\uc740 \uadf8 \uc9c0\uc5ed\uc5d0 \uc800\uc9c0\uc120\uc744 \uad6c\ucd95\ud558\uace0 \uc788\ub294 \uc778\ubbfc \uacbd\ucc30\ub4e4\uc5d0\uac8c \ud638\uc804\uc801\uc774\uc5c8\ub2e4. \uadf8\ub8e8\uc9c0\uc57c \uacf5\uc0b0\ub2f9\uc6d0 \ud30c\ub77c\uc2a4\ud2f0\uc288\ube4c\ub9ac\ub294 \uc220\uc5d0 \ucde8\ud574 \uc220\ubcd1\uc744 \ub4e0\ucc44\ub85c \uc2a4\ud0c8\ub9b0 \uae30\ub150\ube44\uc5d0 \uc62c\ub77c\uac00 \uc220\ubcd1\uc744 \uae68\uace0 \uc774\ub807\uac8c \ub9d0\ud558\uc600\ub2e4. \"\uc2a4\ud0c8\ub9b0\uc758 \uc801\ub4e4\uc740 \uc8fd\uc5b4\ubc84\ub824\ub77c. \uc774 \uc220\ubcd1\ucc98\ub7fc. \" \uc0ac\ub78c\ub4e4\uc740 \uc2a4\ud0c8\ub9b0 \uae30\ub150\ube44\uc5d0 \ud654\ud658\uc744 \ub4e4\uace0\uc654\uace0, \ub354\ub354\uc6b1 \ub9ce\uc740 \uc0ac\ub78c\ub4e4\uc744 \ubaa8\uc5ec\ub4e4\uc5c8\uc73c\uba70, \uc2dc\uc704\uc758 \uaddc\ubaa8\ub294 \uc810\uc9c4\uc801\uc73c\ub85c \ucee4\uc838\uac14\ub2e4. \ud63c\ub780\uc5d0 \ube60\uc9c4 \uc9c0\ubc29 \ub2f9\uad6d\uc740 \uadf8\ub7ec\ud55c \ud65c\ub3d9\ub4e4\uc744 \uc801\uadf9\uc801\uc73c\ub85c \ubc29\ud574\ud558\uc9c0 \uc54a\uc558\ub2e4. \uc218\ub3c4\uc5d0\uc11c\uc758 \uc2dc\uc704\ub294 \uace0\ub9ac, \ucfe0\ud0c0\uc774\uc2dc, \ub8e8\uc2a4\ud0c0\ube44, \uc218\ud6c4\ubbf8\uc640 \uac19\uc740 \uacf5\ud654\uad6d\uc758 \ub2e4\ub978 \uc9c0\uc5ed\ub4e4\uc5d0\uc11c\ub3c4 \ud761\uc0ac\ud55c \uc800\ud56d\uc5d0 \ubc29\uc544\uc1e0\ub97c \ub2f9\uacbc\ub2e4.", "paragraph_answer": "\uadf8\ub8e8\uc9c0\uc57c SSR\uc758 \ub0b4\ubb34\ubd80 \uc7a5\uad00 \ube14\ub77c\ub514\ubbf8\ub974 \uc790\ub2c8\ud30c\ubc14 \uc758 \ud2b9\ubcc4 \ubcf4\uace0\uc11c\uc5d0 \ub530\ub974\uba74, \ud2b8\ube4c\ub9ac\uc2dc \ub3c4\uc2ec\uc758 \ucfe0\ub77c \uc81c\ubc29\uc5d0 \uc788\ub294 \uc2a4\ud0c8\ub9b0 \uae30\ub150\ube44 \uc55e\uc5d0\uc11c \uc2a4\ud0c8\ub9b0 \uc11c\uac70 3\uc8fc\ub144\uc744 \uae30\ub150\ud558\uae30 \uc704\ud574 \ud559\uc0dd \ub2e8\uccb4\ub4e4\uc774 \ubaa8\uc5ec\ub4e4\ub358 1956\ub144 3\uc6d4 4\uc77c\uc5d0 \uc815\uce58\uc801\uc778 \ubd88\uc548\uc774 \uc2dc\uc791\ub418\uc5c8\ub2e4\uace0 \ud55c\ub2e4. \ub2c8\ud0a4\ud0c0 \ud750\ub8e8\uc1fc\ud504\uc758 \uc5f0\uc124\uc5d0 \ubd84\uac1c\ud55c \uadf8\ub4e4\uc740 \uadf8 \uc9c0\uc5ed\uc5d0 \uc800\uc9c0\uc120\uc744 \uad6c\ucd95\ud558\uace0 \uc788\ub294 \uc778\ubbfc \uacbd\ucc30\ub4e4\uc5d0\uac8c \ud638\uc804\uc801\uc774\uc5c8\ub2e4. \uadf8\ub8e8\uc9c0\uc57c \uacf5\uc0b0\ub2f9\uc6d0 \ud30c\ub77c\uc2a4\ud2f0\uc288\ube4c\ub9ac\ub294 \uc220\uc5d0 \ucde8\ud574 \uc220\ubcd1\uc744 \ub4e0\ucc44\ub85c \uc2a4\ud0c8\ub9b0 \uae30\ub150\ube44\uc5d0 \uc62c\ub77c\uac00 \uc220\ubcd1\uc744 \uae68\uace0 \uc774\ub807\uac8c \ub9d0\ud558\uc600\ub2e4. \"\uc2a4\ud0c8\ub9b0\uc758 \uc801\ub4e4\uc740 \uc8fd\uc5b4\ubc84\ub824\ub77c. \uc774 \uc220\ubcd1\ucc98\ub7fc.\" \uc0ac\ub78c\ub4e4\uc740 \uc2a4\ud0c8\ub9b0 \uae30\ub150\ube44\uc5d0 \ud654\ud658\uc744 \ub4e4\uace0\uc654\uace0, \ub354\ub354\uc6b1 \ub9ce\uc740 \uc0ac\ub78c\ub4e4\uc744 \ubaa8\uc5ec\ub4e4\uc5c8\uc73c\uba70, \uc2dc\uc704\uc758 \uaddc\ubaa8\ub294 \uc810\uc9c4\uc801\uc73c\ub85c \ucee4\uc838\uac14\ub2e4. \ud63c\ub780\uc5d0 \ube60\uc9c4 \uc9c0\ubc29 \ub2f9\uad6d\uc740 \uadf8\ub7ec\ud55c \ud65c\ub3d9\ub4e4\uc744 \uc801\uadf9\uc801\uc73c\ub85c \ubc29\ud574\ud558\uc9c0 \uc54a\uc558\ub2e4. \uc218\ub3c4\uc5d0\uc11c\uc758 \uc2dc\uc704\ub294 \uace0\ub9ac, \ucfe0\ud0c0\uc774\uc2dc, \ub8e8\uc2a4\ud0c0\ube44, \uc218\ud6c4\ubbf8\uc640 \uac19\uc740 \uacf5\ud654\uad6d\uc758 \ub2e4\ub978 \uc9c0\uc5ed\ub4e4\uc5d0\uc11c\ub3c4 \ud761\uc0ac\ud55c \uc800\ud56d\uc5d0 \ubc29\uc544\uc1e0\ub97c \ub2f9\uacbc\ub2e4.", "sentence_answer": "\uadf8\ub8e8\uc9c0\uc57c SSR\uc758 \ub0b4\ubb34\ubd80 \uc7a5\uad00 \ube14\ub77c\ub514\ubbf8\ub974 \uc790\ub2c8\ud30c\ubc14 \uc758 \ud2b9\ubcc4 \ubcf4\uace0\uc11c\uc5d0 \ub530\ub974\uba74, \ud2b8\ube4c\ub9ac\uc2dc \ub3c4\uc2ec\uc758 \ucfe0\ub77c \uc81c\ubc29\uc5d0 \uc788\ub294 \uc2a4\ud0c8\ub9b0 \uae30\ub150\ube44 \uc55e\uc5d0\uc11c \uc2a4\ud0c8\ub9b0 \uc11c\uac70 3\uc8fc\ub144\uc744 \uae30\ub150\ud558\uae30 \uc704\ud574 \ud559\uc0dd \ub2e8\uccb4\ub4e4\uc774 \ubaa8\uc5ec\ub4e4\ub358 1956\ub144 3\uc6d4 4\uc77c\uc5d0 \uc815\uce58\uc801\uc778 \ubd88\uc548\uc774 \uc2dc\uc791\ub418\uc5c8\ub2e4\uace0 \ud55c\ub2e4.", "paragraph_id": "5783174-2-0", "paragraph_question": "question: \uadf8\ub8e8\uc9c0\uc57c SSR\uc758 \ub0b4\ubb34\ubd80 \uc7a5\uad00\uc73c\ub85c \ud2b9\ubcc4 \ubcf4\uace0\uc11c\ub97c \uc4f4 \uc0ac\ub78c\uc740?, context: \uadf8\ub8e8\uc9c0\uc57c SSR\uc758 \ub0b4\ubb34\ubd80 \uc7a5\uad00 \ube14\ub77c\ub514\ubbf8\ub974 \uc790\ub2c8\ud30c\ubc14\uc758 \ud2b9\ubcc4 \ubcf4\uace0\uc11c\uc5d0 \ub530\ub974\uba74, \ud2b8\ube4c\ub9ac\uc2dc \ub3c4\uc2ec\uc758 \ucfe0\ub77c \uc81c\ubc29\uc5d0 \uc788\ub294 \uc2a4\ud0c8\ub9b0 \uae30\ub150\ube44 \uc55e\uc5d0\uc11c \uc2a4\ud0c8\ub9b0 \uc11c\uac70 3\uc8fc\ub144\uc744 \uae30\ub150\ud558\uae30 \uc704\ud574 \ud559\uc0dd \ub2e8\uccb4\ub4e4\uc774 \ubaa8\uc5ec\ub4e4\ub358 1956\ub144 3\uc6d4 4\uc77c\uc5d0 \uc815\uce58\uc801\uc778 \ubd88\uc548\uc774 \uc2dc\uc791\ub418\uc5c8\ub2e4\uace0 \ud55c\ub2e4. \ub2c8\ud0a4\ud0c0 \ud750\ub8e8\uc1fc\ud504\uc758 \uc5f0\uc124\uc5d0 \ubd84\uac1c\ud55c \uadf8\ub4e4\uc740 \uadf8 \uc9c0\uc5ed\uc5d0 \uc800\uc9c0\uc120\uc744 \uad6c\ucd95\ud558\uace0 \uc788\ub294 \uc778\ubbfc \uacbd\ucc30\ub4e4\uc5d0\uac8c \ud638\uc804\uc801\uc774\uc5c8\ub2e4. \uadf8\ub8e8\uc9c0\uc57c \uacf5\uc0b0\ub2f9\uc6d0 \ud30c\ub77c\uc2a4\ud2f0\uc288\ube4c\ub9ac\ub294 \uc220\uc5d0 \ucde8\ud574 \uc220\ubcd1\uc744 \ub4e0\ucc44\ub85c \uc2a4\ud0c8\ub9b0 \uae30\ub150\ube44\uc5d0 \uc62c\ub77c\uac00 \uc220\ubcd1\uc744 \uae68\uace0 \uc774\ub807\uac8c \ub9d0\ud558\uc600\ub2e4. \"\uc2a4\ud0c8\ub9b0\uc758 \uc801\ub4e4\uc740 \uc8fd\uc5b4\ubc84\ub824\ub77c. \uc774 \uc220\ubcd1\ucc98\ub7fc.\" \uc0ac\ub78c\ub4e4\uc740 \uc2a4\ud0c8\ub9b0 \uae30\ub150\ube44\uc5d0 \ud654\ud658\uc744 \ub4e4\uace0\uc654\uace0, \ub354\ub354\uc6b1 \ub9ce\uc740 \uc0ac\ub78c\ub4e4\uc744 \ubaa8\uc5ec\ub4e4\uc5c8\uc73c\uba70, \uc2dc\uc704\uc758 \uaddc\ubaa8\ub294 \uc810\uc9c4\uc801\uc73c\ub85c \ucee4\uc838\uac14\ub2e4. \ud63c\ub780\uc5d0 \ube60\uc9c4 \uc9c0\ubc29 \ub2f9\uad6d\uc740 \uadf8\ub7ec\ud55c \ud65c\ub3d9\ub4e4\uc744 \uc801\uadf9\uc801\uc73c\ub85c \ubc29\ud574\ud558\uc9c0 \uc54a\uc558\ub2e4. \uc218\ub3c4\uc5d0\uc11c\uc758 \uc2dc\uc704\ub294 \uace0\ub9ac, \ucfe0\ud0c0\uc774\uc2dc, \ub8e8\uc2a4\ud0c0\ube44, \uc218\ud6c4\ubbf8\uc640 \uac19\uc740 \uacf5\ud654\uad6d\uc758 \ub2e4\ub978 \uc9c0\uc5ed\ub4e4\uc5d0\uc11c\ub3c4 \ud761\uc0ac\ud55c \uc800\ud56d\uc5d0 \ubc29\uc544\uc1e0\ub97c \ub2f9\uacbc\ub2e4."} {"question": "\ubcf4\uc2a4\ud134 \uad6c\uc6b4\ucf69, \uacfc\uc77c \uc2a4\ud2b8\ub77c\uc774\ud504, \ub808\ubaac\ud5e4\ub4dc, \uc288\ud37c\ubc84\ube14 \ub4f1\uc758 \uc824\ub9ac\ub97c \ud310\ub9e4\ud558\ub294 \ud68c\uc0ac\ub294?", "paragraph": "\uad6c\ubbf8\ubca0\uc5b4\ub97c \ub9cc\ub4dc\ub294 \uc81c\uc870\uc0ac\ub4e4\uc740 \ub610\ud55c \ub2e4\ub978 \uc885\ub958\uc758 \uc824\ub9ac\ub3c4 \ud310\ub9e4\ud558\uace0 \uc788\ub2e4. \uc608\ub85c \ubbf8\uad6d\uc5d0\uc11c \ud310\ub9e4\ub418\ub294 \ud558\ub9ac\ubcf4 \uc0ac\uc758 \uc81c\ud488\ub4e4\uc744 \ubcf4\uba74 \uad6c\ubbf8\ubca0\uc5b4 \uc774\uc678\uc5d0\ub3c4 \uc2a4\uba38\ud504(The Smurfs), \uacf5\ub8e1(Dinosaurs), \ucf5c\ub77c(Happy Cola), \uac1c\uad6c\ub9ac(Frogs) \ub4f1 \uc5ec\ub7ec \uc885\ub958\ub4e4\uc774 \uc788\ub2e4. \ud2b8\ub864\ub9ac \uc0ac\uc5d0\ub294 \ubcf4\uc2a4\ud134 \uad6c\uc6b4 \ucf69(Boston Baked Beans), \uacfc\uc77c \uc2a4\ud2b8\ub77c\uc774\ud504(Fruit Stripe), \ub808\ubaac\ud5e4\ub4dc(Lemonhead), \uc288\ud37c\ubc84\ube14(Super Bubble) \ub4f1\uc774 \uc788\ub2e4. \ub610\ud55c \ud310\ub9e4\ub418\ub294 \uc9c0\uc5ed\uc5d0 \ub530\ub77c \ud310\ub9e4\ub418\ub294 \uc885\ub958\uc5d0 \ucc28\uc774\uac00 \uc788\ub294\ub370, \ubbf8\uad6d\uc5d0\uc11c\ub294 \ud310\ub9e4\ub418\uc9c0 \uc54a\uc9c0\ub9cc \uc720\ub7fd \ub4f1 \uc138\uacc4\uc5d0\uc11c \ud310\ub9e4\ub418\ub294 \uac83\ub4e4\uc5d0\ub294 \uc2a4\ud0c0\ubbf9\uc2a4(Starmix), \ud574\uc801(Piratos), \ubc40\ud30c\uc774\uc5b4(Vampire), \ubc00\ud06c\uc250\uc774\ud06c(Milkshakes), \uc774\ube68(Teeth) \ub4f1\uc774 \uc788\ub2e4.", "answer": "\ud2b8\ub864\ub9ac \uc0ac", "sentence": "\ud2b8\ub864\ub9ac \uc0ac \uc5d0\ub294 \ubcf4\uc2a4\ud134 \uad6c\uc6b4 \ucf69(Boston Baked Beans), \uacfc\uc77c \uc2a4\ud2b8\ub77c\uc774\ud504(Fruit Stripe), \ub808\ubaac\ud5e4\ub4dc(Lemonhead), \uc288\ud37c\ubc84\ube14(Super Bubble) \ub4f1\uc774 \uc788\ub2e4.", "paragraph_sentence": "\uad6c\ubbf8\ubca0\uc5b4\ub97c \ub9cc\ub4dc\ub294 \uc81c\uc870\uc0ac\ub4e4\uc740 \ub610\ud55c \ub2e4\ub978 \uc885\ub958\uc758 \uc824\ub9ac\ub3c4 \ud310\ub9e4\ud558\uace0 \uc788\ub2e4. \uc608\ub85c \ubbf8\uad6d\uc5d0\uc11c \ud310\ub9e4\ub418\ub294 \ud558\ub9ac\ubcf4 \uc0ac\uc758 \uc81c\ud488\ub4e4\uc744 \ubcf4\uba74 \uad6c\ubbf8\ubca0\uc5b4 \uc774\uc678\uc5d0\ub3c4 \uc2a4\uba38\ud504(The Smurfs), \uacf5\ub8e1(Dinosaurs), \ucf5c\ub77c(Happy Cola), \uac1c\uad6c\ub9ac(Frogs) \ub4f1 \uc5ec\ub7ec \uc885\ub958\ub4e4\uc774 \uc788\ub2e4. \ud2b8\ub864\ub9ac \uc0ac \uc5d0\ub294 \ubcf4\uc2a4\ud134 \uad6c\uc6b4 \ucf69(Boston Baked Beans), \uacfc\uc77c \uc2a4\ud2b8\ub77c\uc774\ud504(Fruit Stripe), \ub808\ubaac\ud5e4\ub4dc(Lemonhead), \uc288\ud37c\ubc84\ube14(Super Bubble) \ub4f1\uc774 \uc788\ub2e4. \ub610\ud55c \ud310\ub9e4\ub418\ub294 \uc9c0\uc5ed\uc5d0 \ub530\ub77c \ud310\ub9e4\ub418\ub294 \uc885\ub958\uc5d0 \ucc28\uc774\uac00 \uc788\ub294\ub370, \ubbf8\uad6d\uc5d0\uc11c\ub294 \ud310\ub9e4\ub418\uc9c0 \uc54a\uc9c0\ub9cc \uc720\ub7fd \ub4f1 \uc138\uacc4\uc5d0\uc11c \ud310\ub9e4\ub418\ub294 \uac83\ub4e4\uc5d0\ub294 \uc2a4\ud0c0\ubbf9\uc2a4(Starmix), \ud574\uc801(Piratos), \ubc40\ud30c\uc774\uc5b4(Vampire), \ubc00\ud06c\uc250\uc774\ud06c(Milkshakes), \uc774\ube68(Teeth) \ub4f1\uc774 \uc788\ub2e4.", "paragraph_answer": "\uad6c\ubbf8\ubca0\uc5b4\ub97c \ub9cc\ub4dc\ub294 \uc81c\uc870\uc0ac\ub4e4\uc740 \ub610\ud55c \ub2e4\ub978 \uc885\ub958\uc758 \uc824\ub9ac\ub3c4 \ud310\ub9e4\ud558\uace0 \uc788\ub2e4. \uc608\ub85c \ubbf8\uad6d\uc5d0\uc11c \ud310\ub9e4\ub418\ub294 \ud558\ub9ac\ubcf4 \uc0ac\uc758 \uc81c\ud488\ub4e4\uc744 \ubcf4\uba74 \uad6c\ubbf8\ubca0\uc5b4 \uc774\uc678\uc5d0\ub3c4 \uc2a4\uba38\ud504(The Smurfs), \uacf5\ub8e1(Dinosaurs), \ucf5c\ub77c(Happy Cola), \uac1c\uad6c\ub9ac(Frogs) \ub4f1 \uc5ec\ub7ec \uc885\ub958\ub4e4\uc774 \uc788\ub2e4. \ud2b8\ub864\ub9ac \uc0ac \uc5d0\ub294 \ubcf4\uc2a4\ud134 \uad6c\uc6b4 \ucf69(Boston Baked Beans), \uacfc\uc77c \uc2a4\ud2b8\ub77c\uc774\ud504(Fruit Stripe), \ub808\ubaac\ud5e4\ub4dc(Lemonhead), \uc288\ud37c\ubc84\ube14(Super Bubble) \ub4f1\uc774 \uc788\ub2e4. \ub610\ud55c \ud310\ub9e4\ub418\ub294 \uc9c0\uc5ed\uc5d0 \ub530\ub77c \ud310\ub9e4\ub418\ub294 \uc885\ub958\uc5d0 \ucc28\uc774\uac00 \uc788\ub294\ub370, \ubbf8\uad6d\uc5d0\uc11c\ub294 \ud310\ub9e4\ub418\uc9c0 \uc54a\uc9c0\ub9cc \uc720\ub7fd \ub4f1 \uc138\uacc4\uc5d0\uc11c \ud310\ub9e4\ub418\ub294 \uac83\ub4e4\uc5d0\ub294 \uc2a4\ud0c0\ubbf9\uc2a4(Starmix), \ud574\uc801(Piratos), \ubc40\ud30c\uc774\uc5b4(Vampire), \ubc00\ud06c\uc250\uc774\ud06c(Milkshakes), \uc774\ube68(Teeth) \ub4f1\uc774 \uc788\ub2e4.", "sentence_answer": " \ud2b8\ub864\ub9ac \uc0ac \uc5d0\ub294 \ubcf4\uc2a4\ud134 \uad6c\uc6b4 \ucf69(Boston Baked Beans), \uacfc\uc77c \uc2a4\ud2b8\ub77c\uc774\ud504(Fruit Stripe), \ub808\ubaac\ud5e4\ub4dc(Lemonhead), \uc288\ud37c\ubc84\ube14(Super Bubble) \ub4f1\uc774 \uc788\ub2e4.", "paragraph_id": "6476868-2-1", "paragraph_question": "question: \ubcf4\uc2a4\ud134 \uad6c\uc6b4\ucf69, \uacfc\uc77c \uc2a4\ud2b8\ub77c\uc774\ud504, \ub808\ubaac\ud5e4\ub4dc, \uc288\ud37c\ubc84\ube14 \ub4f1\uc758 \uc824\ub9ac\ub97c \ud310\ub9e4\ud558\ub294 \ud68c\uc0ac\ub294?, context: \uad6c\ubbf8\ubca0\uc5b4\ub97c \ub9cc\ub4dc\ub294 \uc81c\uc870\uc0ac\ub4e4\uc740 \ub610\ud55c \ub2e4\ub978 \uc885\ub958\uc758 \uc824\ub9ac\ub3c4 \ud310\ub9e4\ud558\uace0 \uc788\ub2e4. \uc608\ub85c \ubbf8\uad6d\uc5d0\uc11c \ud310\ub9e4\ub418\ub294 \ud558\ub9ac\ubcf4 \uc0ac\uc758 \uc81c\ud488\ub4e4\uc744 \ubcf4\uba74 \uad6c\ubbf8\ubca0\uc5b4 \uc774\uc678\uc5d0\ub3c4 \uc2a4\uba38\ud504(The Smurfs), \uacf5\ub8e1(Dinosaurs), \ucf5c\ub77c(Happy Cola), \uac1c\uad6c\ub9ac(Frogs) \ub4f1 \uc5ec\ub7ec \uc885\ub958\ub4e4\uc774 \uc788\ub2e4. \ud2b8\ub864\ub9ac \uc0ac\uc5d0\ub294 \ubcf4\uc2a4\ud134 \uad6c\uc6b4 \ucf69(Boston Baked Beans), \uacfc\uc77c \uc2a4\ud2b8\ub77c\uc774\ud504(Fruit Stripe), \ub808\ubaac\ud5e4\ub4dc(Lemonhead), \uc288\ud37c\ubc84\ube14(Super Bubble) \ub4f1\uc774 \uc788\ub2e4. \ub610\ud55c \ud310\ub9e4\ub418\ub294 \uc9c0\uc5ed\uc5d0 \ub530\ub77c \ud310\ub9e4\ub418\ub294 \uc885\ub958\uc5d0 \ucc28\uc774\uac00 \uc788\ub294\ub370, \ubbf8\uad6d\uc5d0\uc11c\ub294 \ud310\ub9e4\ub418\uc9c0 \uc54a\uc9c0\ub9cc \uc720\ub7fd \ub4f1 \uc138\uacc4\uc5d0\uc11c \ud310\ub9e4\ub418\ub294 \uac83\ub4e4\uc5d0\ub294 \uc2a4\ud0c0\ubbf9\uc2a4(Starmix), \ud574\uc801(Piratos), \ubc40\ud30c\uc774\uc5b4(Vampire), \ubc00\ud06c\uc250\uc774\ud06c(Milkshakes), \uc774\ube68(Teeth) \ub4f1\uc774 \uc788\ub2e4."} {"question": "\ubc15\uc815\ud76c \uc815\ubd80\uac00 \uccad\uc640\ub300 \ucd9c\uc785 \uae30\uc790\uc758 \ucd9c\uc785 \uc5ec\ubd80\ub97c \uc9c1\uc811 \uc2b9\uc778\ud558\uae30 \uc704\ud574 1963\ub144 \uac1c\uc124\ud55c \uc81c\ub3c4\uc758 \uc774\ub984\uc740?", "paragraph": "1950\ub144\ub300\uae4c\uc9c0 \uc5b8\ub860 \ud658\uacbd\uc5d0\ub294 \ud070 \ubcc0\ud654\uac00 \uc5c6\uc5c8\uc73c\ub098, 1961\ub144 5\u00b716 \uad70\uc0ac \uc815\ubcc0 \uc774\ud6c4 \uc5b8\ub860 \ud658\uacbd\uc740 \uae09\ubcc0\ud558\uac8c \ub418\uc5c8\ub2e4. 4\u00b719 \ud601\uba85 \uc774\ud6c4 \uc5b8\ub860\uc0ac\uac00 \ub09c\ub9bd\ud558\uc5ec \uc0ac\uc774\ube44 \uae30\uc790\uac00 \uc0dd\uae30\uace0 \ucd9c\uc785 \uae30\uc790\ub2e8\uc758 \ubd80\ud328\uac00 \uc54c\ub824\uc838 \uc5b8\ub860 \ub9e4\uccb4\uc640 \uae30\uc790\uc5d0 \ub300\ud55c \ubd80\uc815\uc801\uc778 \uc778\uc2dd\uc774 \ud33d\ubc30\ud558\uc600\ub2e4. \uad70\ubd80\ub294 \uc774\uac83\uc744 \uba85\ubd84\uc73c\ub85c \uc0bc\uc544 \uc5b8\ub860 \ub9e4\uccb4\ub97c \ubd80\ud328 \uc9d1\ub2e8\uc73c\ub85c \uaddc\uc815\ud558\uace0 \uac15\ub825\ud558\uac8c \ud1b5\uc81c\ud558\ub824 \ud558\uc600\ub2e4. \ubc15\uc815\ud76c \uc815\ubd80\ub294 \uc790\uc758\uc801\uc73c\ub85c \uccad\uc640\ub300\uc5d0 \ucd9c\uc785\ud558\ub294 \uae30\uc790\uc758 \uc218\ub97c \uc870\uc815\ud558\uc5ec \uc27d\uac8c \uc5b8\ub860 \ub9e4\uccb4\uc640 \ubcf4\ub3c4\ub97c \ud1b5\uc81c\ud558\uae30 \uc704\ud558\uc5ec 1963\ub144 \uccad\uc640\ub300\uc5d0 \uae30\uc790\uc2e4\uc744 \uc124\uce58\ud558\uace0, \ucd9c\uc785 \uc5ec\ubd80\ub97c \uccad\uc640\ub300\uac00 \uc2b9\uc778\ud558\ub294 \uc18c\uc704 \uc544\uadf8\ub808\ub9dd \uc81c\ub3c4\ub97c \ub3c4\uc785\ud558\uc600\ub2e4. \ub2f9\uc2dc \uae30\uc790\uc2e4 \uc774\uc678\uc5d0 \ubcf4\ub3c4 \uc790\ub8cc\ub97c \uc5bb\uc744 \ubc29\ubc95\uc774 \uc5c6\ub358 \uae30\uc790\ub4e4\uc740 \uc815\ubd80\uc758 \uc815\ucc45\uc744 \uc218\uc6a9\ud560 \uc218\ubc16\uc5d0 \uc5c6\uc5c8\uc73c\uba70, \uc774\ub54c\ubd80\ud130 \uc815\ubd80 \uae30\uad00 \ub300\ubcc0\uc778\uc758 \ubc1c\ud45c\ub97c \uae30\uc790\ub2e8\uc774 \ucde8\uc7ac\ud558\ub294 \uac83\uc774 \uad00\ud589\uc774 \ub418\uc5c8\uace0, \uc774 \uad00\ud589\uc740 1967\ub144 \uc815\ubd80\uac00 \uac01 \ubd80\ucc98\uc5d0 \uacf5\ubcf4\uad00\uc744 \uacf5\uc2dd\uc801\uc73c\ub85c \ub450\uc5b4 \uc81c\ub3c4\ub85c\uc11c \uc815\ucc29\ud558\uc600\ub2e4.", "answer": "\uc544\uadf8\ub808\ub9dd \uc81c\ub3c4", "sentence": "\uc18c\uc704 \uc544\uadf8\ub808\ub9dd \uc81c\ub3c4 \ub97c \ub3c4\uc785\ud558\uc600\ub2e4.", "paragraph_sentence": "1950\ub144\ub300\uae4c\uc9c0 \uc5b8\ub860 \ud658\uacbd\uc5d0\ub294 \ud070 \ubcc0\ud654\uac00 \uc5c6\uc5c8\uc73c\ub098, 1961\ub144 5\u00b716 \uad70\uc0ac \uc815\ubcc0 \uc774\ud6c4 \uc5b8\ub860 \ud658\uacbd\uc740 \uae09\ubcc0\ud558\uac8c \ub418\uc5c8\ub2e4. 4\u00b719 \ud601\uba85 \uc774\ud6c4 \uc5b8\ub860\uc0ac\uac00 \ub09c\ub9bd\ud558\uc5ec \uc0ac\uc774\ube44 \uae30\uc790\uac00 \uc0dd\uae30\uace0 \ucd9c\uc785 \uae30\uc790\ub2e8\uc758 \ubd80\ud328\uac00 \uc54c\ub824\uc838 \uc5b8\ub860 \ub9e4\uccb4\uc640 \uae30\uc790\uc5d0 \ub300\ud55c \ubd80\uc815\uc801\uc778 \uc778\uc2dd\uc774 \ud33d\ubc30\ud558\uc600\ub2e4. \uad70\ubd80\ub294 \uc774\uac83\uc744 \uba85\ubd84\uc73c\ub85c \uc0bc\uc544 \uc5b8\ub860 \ub9e4\uccb4\ub97c \ubd80\ud328 \uc9d1\ub2e8\uc73c\ub85c \uaddc\uc815\ud558\uace0 \uac15\ub825\ud558\uac8c \ud1b5\uc81c\ud558\ub824 \ud558\uc600\ub2e4. \ubc15\uc815\ud76c \uc815\ubd80\ub294 \uc790\uc758\uc801\uc73c\ub85c \uccad\uc640\ub300\uc5d0 \ucd9c\uc785\ud558\ub294 \uae30\uc790\uc758 \uc218\ub97c \uc870\uc815\ud558\uc5ec \uc27d\uac8c \uc5b8\ub860 \ub9e4\uccb4\uc640 \ubcf4\ub3c4\ub97c \ud1b5\uc81c\ud558\uae30 \uc704\ud558\uc5ec 1963\ub144 \uccad\uc640\ub300\uc5d0 \uae30\uc790\uc2e4\uc744 \uc124\uce58\ud558\uace0, \ucd9c\uc785 \uc5ec\ubd80\ub97c \uccad\uc640\ub300\uac00 \uc2b9\uc778\ud558\ub294 \uc18c\uc704 \uc544\uadf8\ub808\ub9dd \uc81c\ub3c4 \ub97c \ub3c4\uc785\ud558\uc600\ub2e4. \ub2f9\uc2dc \uae30\uc790\uc2e4 \uc774\uc678\uc5d0 \ubcf4\ub3c4 \uc790\ub8cc\ub97c \uc5bb\uc744 \ubc29\ubc95\uc774 \uc5c6\ub358 \uae30\uc790\ub4e4\uc740 \uc815\ubd80\uc758 \uc815\ucc45\uc744 \uc218\uc6a9\ud560 \uc218\ubc16\uc5d0 \uc5c6\uc5c8\uc73c\uba70, \uc774\ub54c\ubd80\ud130 \uc815\ubd80 \uae30\uad00 \ub300\ubcc0\uc778\uc758 \ubc1c\ud45c\ub97c \uae30\uc790\ub2e8\uc774 \ucde8\uc7ac\ud558\ub294 \uac83\uc774 \uad00\ud589\uc774 \ub418\uc5c8\uace0, \uc774 \uad00\ud589\uc740 1967\ub144 \uc815\ubd80\uac00 \uac01 \ubd80\ucc98\uc5d0 \uacf5\ubcf4\uad00\uc744 \uacf5\uc2dd\uc801\uc73c\ub85c \ub450\uc5b4 \uc81c\ub3c4\ub85c\uc11c \uc815\ucc29\ud558\uc600\ub2e4.", "paragraph_answer": "1950\ub144\ub300\uae4c\uc9c0 \uc5b8\ub860 \ud658\uacbd\uc5d0\ub294 \ud070 \ubcc0\ud654\uac00 \uc5c6\uc5c8\uc73c\ub098, 1961\ub144 5\u00b716 \uad70\uc0ac \uc815\ubcc0 \uc774\ud6c4 \uc5b8\ub860 \ud658\uacbd\uc740 \uae09\ubcc0\ud558\uac8c \ub418\uc5c8\ub2e4. 4\u00b719 \ud601\uba85 \uc774\ud6c4 \uc5b8\ub860\uc0ac\uac00 \ub09c\ub9bd\ud558\uc5ec \uc0ac\uc774\ube44 \uae30\uc790\uac00 \uc0dd\uae30\uace0 \ucd9c\uc785 \uae30\uc790\ub2e8\uc758 \ubd80\ud328\uac00 \uc54c\ub824\uc838 \uc5b8\ub860 \ub9e4\uccb4\uc640 \uae30\uc790\uc5d0 \ub300\ud55c \ubd80\uc815\uc801\uc778 \uc778\uc2dd\uc774 \ud33d\ubc30\ud558\uc600\ub2e4. \uad70\ubd80\ub294 \uc774\uac83\uc744 \uba85\ubd84\uc73c\ub85c \uc0bc\uc544 \uc5b8\ub860 \ub9e4\uccb4\ub97c \ubd80\ud328 \uc9d1\ub2e8\uc73c\ub85c \uaddc\uc815\ud558\uace0 \uac15\ub825\ud558\uac8c \ud1b5\uc81c\ud558\ub824 \ud558\uc600\ub2e4. \ubc15\uc815\ud76c \uc815\ubd80\ub294 \uc790\uc758\uc801\uc73c\ub85c \uccad\uc640\ub300\uc5d0 \ucd9c\uc785\ud558\ub294 \uae30\uc790\uc758 \uc218\ub97c \uc870\uc815\ud558\uc5ec \uc27d\uac8c \uc5b8\ub860 \ub9e4\uccb4\uc640 \ubcf4\ub3c4\ub97c \ud1b5\uc81c\ud558\uae30 \uc704\ud558\uc5ec 1963\ub144 \uccad\uc640\ub300\uc5d0 \uae30\uc790\uc2e4\uc744 \uc124\uce58\ud558\uace0, \ucd9c\uc785 \uc5ec\ubd80\ub97c \uccad\uc640\ub300\uac00 \uc2b9\uc778\ud558\ub294 \uc18c\uc704 \uc544\uadf8\ub808\ub9dd \uc81c\ub3c4 \ub97c \ub3c4\uc785\ud558\uc600\ub2e4. \ub2f9\uc2dc \uae30\uc790\uc2e4 \uc774\uc678\uc5d0 \ubcf4\ub3c4 \uc790\ub8cc\ub97c \uc5bb\uc744 \ubc29\ubc95\uc774 \uc5c6\ub358 \uae30\uc790\ub4e4\uc740 \uc815\ubd80\uc758 \uc815\ucc45\uc744 \uc218\uc6a9\ud560 \uc218\ubc16\uc5d0 \uc5c6\uc5c8\uc73c\uba70, \uc774\ub54c\ubd80\ud130 \uc815\ubd80 \uae30\uad00 \ub300\ubcc0\uc778\uc758 \ubc1c\ud45c\ub97c \uae30\uc790\ub2e8\uc774 \ucde8\uc7ac\ud558\ub294 \uac83\uc774 \uad00\ud589\uc774 \ub418\uc5c8\uace0, \uc774 \uad00\ud589\uc740 1967\ub144 \uc815\ubd80\uac00 \uac01 \ubd80\ucc98\uc5d0 \uacf5\ubcf4\uad00\uc744 \uacf5\uc2dd\uc801\uc73c\ub85c \ub450\uc5b4 \uc81c\ub3c4\ub85c\uc11c \uc815\ucc29\ud558\uc600\ub2e4.", "sentence_answer": "\uc18c\uc704 \uc544\uadf8\ub808\ub9dd \uc81c\ub3c4 \ub97c \ub3c4\uc785\ud558\uc600\ub2e4.", "paragraph_id": "6486011-0-1", "paragraph_question": "question: \ubc15\uc815\ud76c \uc815\ubd80\uac00 \uccad\uc640\ub300 \ucd9c\uc785 \uae30\uc790\uc758 \ucd9c\uc785 \uc5ec\ubd80\ub97c \uc9c1\uc811 \uc2b9\uc778\ud558\uae30 \uc704\ud574 1963\ub144 \uac1c\uc124\ud55c \uc81c\ub3c4\uc758 \uc774\ub984\uc740?, context: 1950\ub144\ub300\uae4c\uc9c0 \uc5b8\ub860 \ud658\uacbd\uc5d0\ub294 \ud070 \ubcc0\ud654\uac00 \uc5c6\uc5c8\uc73c\ub098, 1961\ub144 5\u00b716 \uad70\uc0ac \uc815\ubcc0 \uc774\ud6c4 \uc5b8\ub860 \ud658\uacbd\uc740 \uae09\ubcc0\ud558\uac8c \ub418\uc5c8\ub2e4. 4\u00b719 \ud601\uba85 \uc774\ud6c4 \uc5b8\ub860\uc0ac\uac00 \ub09c\ub9bd\ud558\uc5ec \uc0ac\uc774\ube44 \uae30\uc790\uac00 \uc0dd\uae30\uace0 \ucd9c\uc785 \uae30\uc790\ub2e8\uc758 \ubd80\ud328\uac00 \uc54c\ub824\uc838 \uc5b8\ub860 \ub9e4\uccb4\uc640 \uae30\uc790\uc5d0 \ub300\ud55c \ubd80\uc815\uc801\uc778 \uc778\uc2dd\uc774 \ud33d\ubc30\ud558\uc600\ub2e4. \uad70\ubd80\ub294 \uc774\uac83\uc744 \uba85\ubd84\uc73c\ub85c \uc0bc\uc544 \uc5b8\ub860 \ub9e4\uccb4\ub97c \ubd80\ud328 \uc9d1\ub2e8\uc73c\ub85c \uaddc\uc815\ud558\uace0 \uac15\ub825\ud558\uac8c \ud1b5\uc81c\ud558\ub824 \ud558\uc600\ub2e4. \ubc15\uc815\ud76c \uc815\ubd80\ub294 \uc790\uc758\uc801\uc73c\ub85c \uccad\uc640\ub300\uc5d0 \ucd9c\uc785\ud558\ub294 \uae30\uc790\uc758 \uc218\ub97c \uc870\uc815\ud558\uc5ec \uc27d\uac8c \uc5b8\ub860 \ub9e4\uccb4\uc640 \ubcf4\ub3c4\ub97c \ud1b5\uc81c\ud558\uae30 \uc704\ud558\uc5ec 1963\ub144 \uccad\uc640\ub300\uc5d0 \uae30\uc790\uc2e4\uc744 \uc124\uce58\ud558\uace0, \ucd9c\uc785 \uc5ec\ubd80\ub97c \uccad\uc640\ub300\uac00 \uc2b9\uc778\ud558\ub294 \uc18c\uc704 \uc544\uadf8\ub808\ub9dd \uc81c\ub3c4\ub97c \ub3c4\uc785\ud558\uc600\ub2e4. \ub2f9\uc2dc \uae30\uc790\uc2e4 \uc774\uc678\uc5d0 \ubcf4\ub3c4 \uc790\ub8cc\ub97c \uc5bb\uc744 \ubc29\ubc95\uc774 \uc5c6\ub358 \uae30\uc790\ub4e4\uc740 \uc815\ubd80\uc758 \uc815\ucc45\uc744 \uc218\uc6a9\ud560 \uc218\ubc16\uc5d0 \uc5c6\uc5c8\uc73c\uba70, \uc774\ub54c\ubd80\ud130 \uc815\ubd80 \uae30\uad00 \ub300\ubcc0\uc778\uc758 \ubc1c\ud45c\ub97c \uae30\uc790\ub2e8\uc774 \ucde8\uc7ac\ud558\ub294 \uac83\uc774 \uad00\ud589\uc774 \ub418\uc5c8\uace0, \uc774 \uad00\ud589\uc740 1967\ub144 \uc815\ubd80\uac00 \uac01 \ubd80\ucc98\uc5d0 \uacf5\ubcf4\uad00\uc744 \uacf5\uc2dd\uc801\uc73c\ub85c \ub450\uc5b4 \uc81c\ub3c4\ub85c\uc11c \uc815\ucc29\ud558\uc600\ub2e4."} {"question": "\ub300\uc804\ubcf5\ud569\ud130\ubbf8\ub110\uacfc \ubd81\ub300\uc804\uc815\ub958\uc18c\uc758 \uc5b4\ub5a4 \uc804\uc0b0\uc2dc\uc2a4\ud15c\uc774 \ub2ec\ub77c \uc815\uc0c1\uc801\uc778 \uc2b9\ucc28\uad8c \ubc1c\ub9e4\uac00 \uc774\ub8e8\uc5b4\uc9c0\uc9c0 \uc54a\uc558\ub294\uac00?", "paragraph": "\uc815\ub958\uc18c \uc124\uce58 \uc9c1\uc804\uc5d0\ub294 \ub300\uc804\ubcf5\ud569\ud130\ubbf8\ub110 \uc8fc\uc2dd\ud68c\uc0ac\uc640 \ub300\uc804\uc11c\ub0a8\ubd80\ud130\ubbf8\ub110\uc744 \uc6b4\uc601\ud558\ub294 \ub8e8\uc2dc\ub4dc\uc5d0\uc11c \uc5ec\uac1d\uc6b4\uc218\uc0ac\uc5c5\ubc95\uc744 \uadfc\uac70\ub85c \ud558\uc5ec \"\uc2b9\ucc28\uad8c \ud310\ub9e4\uc640 \uc124\uce58\u00b7\uc6b4\uc601\uc740 \ub2f9\uc5f0\ud788 \ud130\ubbf8\ub110\uc5d0\uc11c \ub9e1\uc544\uc57c \ud55c\ub2e4\"\uace0 \uc8fc\uc7a5\ud558\uace0 \ub300\uc804\uc2dc\uac00 \ubc95\uc870\ud56d\uc5d0 \ub530\ub978 \ud569\uc758\uc640 \ud589\uc815\uc870\uce58\uc5d0 \uc18c\ud640\ud588\ub2e4\uace0 \uc8fc\uc7a5\ud558\uba74\uc11c \ubc18\ubc1c\ud588\ub2e4. \ub610\ud55c \ub300\uc804\ubcf5\ud569\ud130\ubbf8\ub110\uacfc \ubd81\ub300\uc804\uc815\ub958\uc18c\uc758 \uc2dc\uc678\ubc84\uc2a4 \uc608\ub9e4 \uc804\uc0b0\uc2dc\uc2a4\ud15c\uc774 \uc11c\ub85c \ub2ec\ub77c \ud638\ud658\uc774 \uc774\ub8e8\uc5b4\uc9c0\uc9c0 \uc54a\uc544 \uc77c\ubd80 \ub178\uc120\uc5d0\uc11c \uc815\uc0c1\uc801\uc778 \uc2b9\ucc28\uad8c \ubc1c\ub9e4\uac00 \uc774\ub8e8\uc5b4\uc9c0\uc9c0 \uc54a\uc558\ub2e4. \uc774\ub85c \uc778\ud574 \ub300\uc804\ubcf5\ud569\ud130\ubbf8\ub110\uacfc \ub8e8\uc2dc\ub4dc \uce21\uc5d0\uc11c\ub294 \uc815\ub958\uc18c \uc0ac\uc5c5\uc790\uc778 \uae08\ub0a8\uace0\uc18d\uc744 \uc0c1\ub300\ub85c '\ubd81\ub300\uc804 IC\uc815\ub958\uc18c \uc2b9\ucc28\uad8c \ud310\ub9e4 \uac00\ucc98\ubd84\uc2e0\uccad'\uc744 \uc81c\uae30\ud588\uc73c\uba70 5\uc6d4 17\uc77c \ub300\uc804\uc9c0\ubc29\ubc95\uc6d0\uc5d0\uc11c\ub294 \uac00\ucc98\ubd84 \uc2e0\uccad\uc744 \uc778\uc6a9 \uacb0\uc815\ud558\uba74\uc11c \uae08\ub0a8\uace0\uc18d\uc5d0\uc11c \ub9e4\ud45c \ubc1c\uad8c\uc744 \uc911\ub2e8\ud588\ub2e4.", "answer": "\uc2dc\uc678\ubc84\uc2a4 \uc608\ub9e4", "sentence": "\ub610\ud55c \ub300\uc804\ubcf5\ud569\ud130\ubbf8\ub110\uacfc \ubd81\ub300\uc804\uc815\ub958\uc18c\uc758 \uc2dc\uc678\ubc84\uc2a4 \uc608\ub9e4 \uc804\uc0b0\uc2dc\uc2a4\ud15c\uc774 \uc11c\ub85c \ub2ec\ub77c \ud638\ud658\uc774 \uc774\ub8e8\uc5b4\uc9c0\uc9c0 \uc54a\uc544 \uc77c\ubd80 \ub178\uc120\uc5d0\uc11c \uc815\uc0c1\uc801\uc778 \uc2b9\ucc28\uad8c \ubc1c\ub9e4\uac00 \uc774\ub8e8\uc5b4\uc9c0\uc9c0 \uc54a\uc558\ub2e4.", "paragraph_sentence": "\uc815\ub958\uc18c \uc124\uce58 \uc9c1\uc804\uc5d0\ub294 \ub300\uc804\ubcf5\ud569\ud130\ubbf8\ub110 \uc8fc\uc2dd\ud68c\uc0ac\uc640 \ub300\uc804\uc11c\ub0a8\ubd80\ud130\ubbf8\ub110\uc744 \uc6b4\uc601\ud558\ub294 \ub8e8\uc2dc\ub4dc\uc5d0\uc11c \uc5ec\uac1d\uc6b4\uc218\uc0ac\uc5c5\ubc95\uc744 \uadfc\uac70\ub85c \ud558\uc5ec \"\uc2b9\ucc28\uad8c \ud310\ub9e4\uc640 \uc124\uce58\u00b7\uc6b4\uc601\uc740 \ub2f9\uc5f0\ud788 \ud130\ubbf8\ub110\uc5d0\uc11c \ub9e1\uc544\uc57c \ud55c\ub2e4\"\uace0 \uc8fc\uc7a5\ud558\uace0 \ub300\uc804\uc2dc\uac00 \ubc95\uc870\ud56d\uc5d0 \ub530\ub978 \ud569\uc758\uc640 \ud589\uc815\uc870\uce58\uc5d0 \uc18c\ud640\ud588\ub2e4\uace0 \uc8fc\uc7a5\ud558\uba74\uc11c \ubc18\ubc1c\ud588\ub2e4. \ub610\ud55c \ub300\uc804\ubcf5\ud569\ud130\ubbf8\ub110\uacfc \ubd81\ub300\uc804\uc815\ub958\uc18c\uc758 \uc2dc\uc678\ubc84\uc2a4 \uc608\ub9e4 \uc804\uc0b0\uc2dc\uc2a4\ud15c\uc774 \uc11c\ub85c \ub2ec\ub77c \ud638\ud658\uc774 \uc774\ub8e8\uc5b4\uc9c0\uc9c0 \uc54a\uc544 \uc77c\ubd80 \ub178\uc120\uc5d0\uc11c \uc815\uc0c1\uc801\uc778 \uc2b9\ucc28\uad8c \ubc1c\ub9e4\uac00 \uc774\ub8e8\uc5b4\uc9c0\uc9c0 \uc54a\uc558\ub2e4. \uc774\ub85c \uc778\ud574 \ub300\uc804\ubcf5\ud569\ud130\ubbf8\ub110\uacfc \ub8e8\uc2dc\ub4dc \uce21\uc5d0\uc11c\ub294 \uc815\ub958\uc18c \uc0ac\uc5c5\uc790\uc778 \uae08\ub0a8\uace0\uc18d\uc744 \uc0c1\ub300\ub85c '\ubd81\ub300\uc804 IC\uc815\ub958\uc18c \uc2b9\ucc28\uad8c \ud310\ub9e4 \uac00\ucc98\ubd84\uc2e0\uccad'\uc744 \uc81c\uae30\ud588\uc73c\uba70 5\uc6d4 17\uc77c \ub300\uc804\uc9c0\ubc29\ubc95\uc6d0\uc5d0\uc11c\ub294 \uac00\ucc98\ubd84 \uc2e0\uccad\uc744 \uc778\uc6a9 \uacb0\uc815\ud558\uba74\uc11c \uae08\ub0a8\uace0\uc18d\uc5d0\uc11c \ub9e4\ud45c \ubc1c\uad8c\uc744 \uc911\ub2e8\ud588\ub2e4.", "paragraph_answer": "\uc815\ub958\uc18c \uc124\uce58 \uc9c1\uc804\uc5d0\ub294 \ub300\uc804\ubcf5\ud569\ud130\ubbf8\ub110 \uc8fc\uc2dd\ud68c\uc0ac\uc640 \ub300\uc804\uc11c\ub0a8\ubd80\ud130\ubbf8\ub110\uc744 \uc6b4\uc601\ud558\ub294 \ub8e8\uc2dc\ub4dc\uc5d0\uc11c \uc5ec\uac1d\uc6b4\uc218\uc0ac\uc5c5\ubc95\uc744 \uadfc\uac70\ub85c \ud558\uc5ec \"\uc2b9\ucc28\uad8c \ud310\ub9e4\uc640 \uc124\uce58\u00b7\uc6b4\uc601\uc740 \ub2f9\uc5f0\ud788 \ud130\ubbf8\ub110\uc5d0\uc11c \ub9e1\uc544\uc57c \ud55c\ub2e4\"\uace0 \uc8fc\uc7a5\ud558\uace0 \ub300\uc804\uc2dc\uac00 \ubc95\uc870\ud56d\uc5d0 \ub530\ub978 \ud569\uc758\uc640 \ud589\uc815\uc870\uce58\uc5d0 \uc18c\ud640\ud588\ub2e4\uace0 \uc8fc\uc7a5\ud558\uba74\uc11c \ubc18\ubc1c\ud588\ub2e4. \ub610\ud55c \ub300\uc804\ubcf5\ud569\ud130\ubbf8\ub110\uacfc \ubd81\ub300\uc804\uc815\ub958\uc18c\uc758 \uc2dc\uc678\ubc84\uc2a4 \uc608\ub9e4 \uc804\uc0b0\uc2dc\uc2a4\ud15c\uc774 \uc11c\ub85c \ub2ec\ub77c \ud638\ud658\uc774 \uc774\ub8e8\uc5b4\uc9c0\uc9c0 \uc54a\uc544 \uc77c\ubd80 \ub178\uc120\uc5d0\uc11c \uc815\uc0c1\uc801\uc778 \uc2b9\ucc28\uad8c \ubc1c\ub9e4\uac00 \uc774\ub8e8\uc5b4\uc9c0\uc9c0 \uc54a\uc558\ub2e4. \uc774\ub85c \uc778\ud574 \ub300\uc804\ubcf5\ud569\ud130\ubbf8\ub110\uacfc \ub8e8\uc2dc\ub4dc \uce21\uc5d0\uc11c\ub294 \uc815\ub958\uc18c \uc0ac\uc5c5\uc790\uc778 \uae08\ub0a8\uace0\uc18d\uc744 \uc0c1\ub300\ub85c '\ubd81\ub300\uc804 IC\uc815\ub958\uc18c \uc2b9\ucc28\uad8c \ud310\ub9e4 \uac00\ucc98\ubd84\uc2e0\uccad'\uc744 \uc81c\uae30\ud588\uc73c\uba70 5\uc6d4 17\uc77c \ub300\uc804\uc9c0\ubc29\ubc95\uc6d0\uc5d0\uc11c\ub294 \uac00\ucc98\ubd84 \uc2e0\uccad\uc744 \uc778\uc6a9 \uacb0\uc815\ud558\uba74\uc11c \uae08\ub0a8\uace0\uc18d\uc5d0\uc11c \ub9e4\ud45c \ubc1c\uad8c\uc744 \uc911\ub2e8\ud588\ub2e4.", "sentence_answer": "\ub610\ud55c \ub300\uc804\ubcf5\ud569\ud130\ubbf8\ub110\uacfc \ubd81\ub300\uc804\uc815\ub958\uc18c\uc758 \uc2dc\uc678\ubc84\uc2a4 \uc608\ub9e4 \uc804\uc0b0\uc2dc\uc2a4\ud15c\uc774 \uc11c\ub85c \ub2ec\ub77c \ud638\ud658\uc774 \uc774\ub8e8\uc5b4\uc9c0\uc9c0 \uc54a\uc544 \uc77c\ubd80 \ub178\uc120\uc5d0\uc11c \uc815\uc0c1\uc801\uc778 \uc2b9\ucc28\uad8c \ubc1c\ub9e4\uac00 \uc774\ub8e8\uc5b4\uc9c0\uc9c0 \uc54a\uc558\ub2e4.", "paragraph_id": "6582407-1-1", "paragraph_question": "question: \ub300\uc804\ubcf5\ud569\ud130\ubbf8\ub110\uacfc \ubd81\ub300\uc804\uc815\ub958\uc18c\uc758 \uc5b4\ub5a4 \uc804\uc0b0\uc2dc\uc2a4\ud15c\uc774 \ub2ec\ub77c \uc815\uc0c1\uc801\uc778 \uc2b9\ucc28\uad8c \ubc1c\ub9e4\uac00 \uc774\ub8e8\uc5b4\uc9c0\uc9c0 \uc54a\uc558\ub294\uac00?, context: \uc815\ub958\uc18c \uc124\uce58 \uc9c1\uc804\uc5d0\ub294 \ub300\uc804\ubcf5\ud569\ud130\ubbf8\ub110 \uc8fc\uc2dd\ud68c\uc0ac\uc640 \ub300\uc804\uc11c\ub0a8\ubd80\ud130\ubbf8\ub110\uc744 \uc6b4\uc601\ud558\ub294 \ub8e8\uc2dc\ub4dc\uc5d0\uc11c \uc5ec\uac1d\uc6b4\uc218\uc0ac\uc5c5\ubc95\uc744 \uadfc\uac70\ub85c \ud558\uc5ec \"\uc2b9\ucc28\uad8c \ud310\ub9e4\uc640 \uc124\uce58\u00b7\uc6b4\uc601\uc740 \ub2f9\uc5f0\ud788 \ud130\ubbf8\ub110\uc5d0\uc11c \ub9e1\uc544\uc57c \ud55c\ub2e4\"\uace0 \uc8fc\uc7a5\ud558\uace0 \ub300\uc804\uc2dc\uac00 \ubc95\uc870\ud56d\uc5d0 \ub530\ub978 \ud569\uc758\uc640 \ud589\uc815\uc870\uce58\uc5d0 \uc18c\ud640\ud588\ub2e4\uace0 \uc8fc\uc7a5\ud558\uba74\uc11c \ubc18\ubc1c\ud588\ub2e4. \ub610\ud55c \ub300\uc804\ubcf5\ud569\ud130\ubbf8\ub110\uacfc \ubd81\ub300\uc804\uc815\ub958\uc18c\uc758 \uc2dc\uc678\ubc84\uc2a4 \uc608\ub9e4 \uc804\uc0b0\uc2dc\uc2a4\ud15c\uc774 \uc11c\ub85c \ub2ec\ub77c \ud638\ud658\uc774 \uc774\ub8e8\uc5b4\uc9c0\uc9c0 \uc54a\uc544 \uc77c\ubd80 \ub178\uc120\uc5d0\uc11c \uc815\uc0c1\uc801\uc778 \uc2b9\ucc28\uad8c \ubc1c\ub9e4\uac00 \uc774\ub8e8\uc5b4\uc9c0\uc9c0 \uc54a\uc558\ub2e4. \uc774\ub85c \uc778\ud574 \ub300\uc804\ubcf5\ud569\ud130\ubbf8\ub110\uacfc \ub8e8\uc2dc\ub4dc \uce21\uc5d0\uc11c\ub294 \uc815\ub958\uc18c \uc0ac\uc5c5\uc790\uc778 \uae08\ub0a8\uace0\uc18d\uc744 \uc0c1\ub300\ub85c '\ubd81\ub300\uc804 IC\uc815\ub958\uc18c \uc2b9\ucc28\uad8c \ud310\ub9e4 \uac00\ucc98\ubd84\uc2e0\uccad'\uc744 \uc81c\uae30\ud588\uc73c\uba70 5\uc6d4 17\uc77c \ub300\uc804\uc9c0\ubc29\ubc95\uc6d0\uc5d0\uc11c\ub294 \uac00\ucc98\ubd84 \uc2e0\uccad\uc744 \uc778\uc6a9 \uacb0\uc815\ud558\uba74\uc11c \uae08\ub0a8\uace0\uc18d\uc5d0\uc11c \ub9e4\ud45c \ubc1c\uad8c\uc744 \uc911\ub2e8\ud588\ub2e4."} {"question": "\uac1c\ucd5c\uc9c0 \uc120\uc815\uc5d0 \uac78\ub9ac\ub294 \uc2dc\uac04\uc740?", "paragraph": "\uc62c\ub9bc\ud53d \uac1c\ucd5c\uc9c0\ub294 \ud574\ub2f9 \uc62c\ub9bc\ud53d \uac1c\ucd5c 7\ub144 \uc804\uc5d0 IOC \uc704\uc6d0\ub4e4\uc758 \ud22c\ud45c\ub85c \uacb0\uc815\ub41c\ub2e4. \uac1c\ucd5c\uc9c0 \uc120\uc815\uc5d0\ub294 \uc57d 2\ub144\uc774 \uac78\ub9b0\ub2e4. \uc720\uce58\ub97c \ud76c\ub9dd\ud558\ub294 \ub3c4\uc2dc\ub294 \uc6b0\uc120 \uc790\uad6d\uc758 \uc62c\ub9bc\ud53d \uc704\uc6d0\ud68c\uc5d0 \uc2e0\uccad\uc744 \ud574\uc57c \ud55c\ub2e4. \ub9cc\uc57d \ud55c \uad6d\uac00\uc5d0\uc11c \ub450 \ub3c4\uc2dc \uc774\uc0c1\uc774 \uc720\uce58\ub97c \ud76c\ub9dd\ud55c\ub2e4\uba74, \ud55c \uad6d\uac00\ub2f9 \ud55c \ub3c4\uc2dc\ub9cc \ud6c4\ubcf4\uac00 \ub420 \uc218 \uc788\ub2e4\ub294 \uaddc\uce59\uc5d0 \ub530\ub77c \ub0b4\ubd80\uc801\uc73c\ub85c \ud6c4\ubcf4 \ub3c4\uc2dc\ub97c \uacb0\uc815\ud574\uc57c \ud55c\ub2e4. \ud6c4\ubcf4 \ub3c4\uc2dc\uac00 \uacb0\uc815\ub418\uba74 \ud6c4\ubcf4 \ub3c4\uc2dc\uac00 \uc18c\uc18d\ub41c \uad6d\uac00\uc758 \uc62c\ub9bc\ud53d \uc704\uc6d0\ud68c\ub294 IOC\uc5d0 \uac1c\ucd5c \uc2e0\uccad\uc744 \ud558\uace0, \uc2e0\uccad \ud6c4\uc5d0\ub294 \uc62c\ub9bc\ud53d \uac1c\ucd5c\uc5d0 \ub300\ud55c \uc9c8\uc758 \uc751\ub2f5\uc11c\ub97c \ubcf4\ub0b4\uc57c \ud55c\ub2e4. \uc774 \uc9c8\uc758\uc751\ub2f5\uc11c\uc5d0\uc11c \uc2e0\uccad\ud55c \ub3c4\uc2dc\ub294 \uc62c\ub9bc\ud53d \ud5cc\uc7a5\uc744 \uc900\uc218\ud558\uba70 IOC \uc0c1\uc784\uc774\uc0ac\ud68c\uc5d0 \uc758\ud55c \ub2e4\ub978 \uaddc\uc815\ub4e4\uc744 \uc9c0\ud0ac \uac83\uc774\ub77c\ub294 \ud655\uc2e0\uc744 \uc8fc\uc5b4\uc57c \ud55c\ub2e4. \uc774 \uc9c8\uc758\uc751\ub2f5\uc11c\ub294 \uc804\ubb38\uac00\ub4e4\uc774 \uac80\ud1a0\ud558\uc5ec \uc2e0\uccad \ub3c4\uc2dc\ub4e4\uc758 \uc7a0\uc7ac\uc131\uacfc \uacc4\ud68d\uc744 \ud3c9\uac00\ud55c\ub2e4. \uc774 \uc804\ubb38\uc801\uc778 \ud3c9\uac00\ub97c \ubc14\ud0d5\uc73c\ub85c IOC \uc0c1\uc784\uc774\uc0ac\ud68c\uc5d0\uc11c\ub294 \uc2e0\uccad\ub3c4\uc2dc \uc911\uc5d0\uc11c \ud6c4\ubcf4\ub3c4\uc2dc\ub97c \uace0\ub978\ub2e4.", "answer": "\uc57d 2\ub144", "sentence": "\uac1c\ucd5c\uc9c0 \uc120\uc815\uc5d0\ub294 \uc57d 2\ub144 \uc774 \uac78\ub9b0\ub2e4.", "paragraph_sentence": "\uc62c\ub9bc\ud53d \uac1c\ucd5c\uc9c0\ub294 \ud574\ub2f9 \uc62c\ub9bc\ud53d \uac1c\ucd5c 7\ub144 \uc804\uc5d0 IOC \uc704\uc6d0\ub4e4\uc758 \ud22c\ud45c\ub85c \uacb0\uc815\ub41c\ub2e4. \uac1c\ucd5c\uc9c0 \uc120\uc815\uc5d0\ub294 \uc57d 2\ub144 \uc774 \uac78\ub9b0\ub2e4. \uc720\uce58\ub97c \ud76c\ub9dd\ud558\ub294 \ub3c4\uc2dc\ub294 \uc6b0\uc120 \uc790\uad6d\uc758 \uc62c\ub9bc\ud53d \uc704\uc6d0\ud68c\uc5d0 \uc2e0\uccad\uc744 \ud574\uc57c \ud55c\ub2e4. \ub9cc\uc57d \ud55c \uad6d\uac00\uc5d0\uc11c \ub450 \ub3c4\uc2dc \uc774\uc0c1\uc774 \uc720\uce58\ub97c \ud76c\ub9dd\ud55c\ub2e4\uba74, \ud55c \uad6d\uac00\ub2f9 \ud55c \ub3c4\uc2dc\ub9cc \ud6c4\ubcf4\uac00 \ub420 \uc218 \uc788\ub2e4\ub294 \uaddc\uce59\uc5d0 \ub530\ub77c \ub0b4\ubd80\uc801\uc73c\ub85c \ud6c4\ubcf4 \ub3c4\uc2dc\ub97c \uacb0\uc815\ud574\uc57c \ud55c\ub2e4. \ud6c4\ubcf4 \ub3c4\uc2dc\uac00 \uacb0\uc815\ub418\uba74 \ud6c4\ubcf4 \ub3c4\uc2dc\uac00 \uc18c\uc18d\ub41c \uad6d\uac00\uc758 \uc62c\ub9bc\ud53d \uc704\uc6d0\ud68c\ub294 IOC\uc5d0 \uac1c\ucd5c \uc2e0\uccad\uc744 \ud558\uace0, \uc2e0\uccad \ud6c4\uc5d0\ub294 \uc62c\ub9bc\ud53d \uac1c\ucd5c\uc5d0 \ub300\ud55c \uc9c8\uc758 \uc751\ub2f5\uc11c\ub97c \ubcf4\ub0b4\uc57c \ud55c\ub2e4. \uc774 \uc9c8\uc758\uc751\ub2f5\uc11c\uc5d0\uc11c \uc2e0\uccad\ud55c \ub3c4\uc2dc\ub294 \uc62c\ub9bc\ud53d \ud5cc\uc7a5\uc744 \uc900\uc218\ud558\uba70 IOC \uc0c1\uc784\uc774\uc0ac\ud68c\uc5d0 \uc758\ud55c \ub2e4\ub978 \uaddc\uc815\ub4e4\uc744 \uc9c0\ud0ac \uac83\uc774\ub77c\ub294 \ud655\uc2e0\uc744 \uc8fc\uc5b4\uc57c \ud55c\ub2e4. \uc774 \uc9c8\uc758\uc751\ub2f5\uc11c\ub294 \uc804\ubb38\uac00\ub4e4\uc774 \uac80\ud1a0\ud558\uc5ec \uc2e0\uccad \ub3c4\uc2dc\ub4e4\uc758 \uc7a0\uc7ac\uc131\uacfc \uacc4\ud68d\uc744 \ud3c9\uac00\ud55c\ub2e4. \uc774 \uc804\ubb38\uc801\uc778 \ud3c9\uac00\ub97c \ubc14\ud0d5\uc73c\ub85c IOC \uc0c1\uc784\uc774\uc0ac\ud68c\uc5d0\uc11c\ub294 \uc2e0\uccad\ub3c4\uc2dc \uc911\uc5d0\uc11c \ud6c4\ubcf4\ub3c4\uc2dc\ub97c \uace0\ub978\ub2e4.", "paragraph_answer": "\uc62c\ub9bc\ud53d \uac1c\ucd5c\uc9c0\ub294 \ud574\ub2f9 \uc62c\ub9bc\ud53d \uac1c\ucd5c 7\ub144 \uc804\uc5d0 IOC \uc704\uc6d0\ub4e4\uc758 \ud22c\ud45c\ub85c \uacb0\uc815\ub41c\ub2e4. \uac1c\ucd5c\uc9c0 \uc120\uc815\uc5d0\ub294 \uc57d 2\ub144 \uc774 \uac78\ub9b0\ub2e4. \uc720\uce58\ub97c \ud76c\ub9dd\ud558\ub294 \ub3c4\uc2dc\ub294 \uc6b0\uc120 \uc790\uad6d\uc758 \uc62c\ub9bc\ud53d \uc704\uc6d0\ud68c\uc5d0 \uc2e0\uccad\uc744 \ud574\uc57c \ud55c\ub2e4. \ub9cc\uc57d \ud55c \uad6d\uac00\uc5d0\uc11c \ub450 \ub3c4\uc2dc \uc774\uc0c1\uc774 \uc720\uce58\ub97c \ud76c\ub9dd\ud55c\ub2e4\uba74, \ud55c \uad6d\uac00\ub2f9 \ud55c \ub3c4\uc2dc\ub9cc \ud6c4\ubcf4\uac00 \ub420 \uc218 \uc788\ub2e4\ub294 \uaddc\uce59\uc5d0 \ub530\ub77c \ub0b4\ubd80\uc801\uc73c\ub85c \ud6c4\ubcf4 \ub3c4\uc2dc\ub97c \uacb0\uc815\ud574\uc57c \ud55c\ub2e4. \ud6c4\ubcf4 \ub3c4\uc2dc\uac00 \uacb0\uc815\ub418\uba74 \ud6c4\ubcf4 \ub3c4\uc2dc\uac00 \uc18c\uc18d\ub41c \uad6d\uac00\uc758 \uc62c\ub9bc\ud53d \uc704\uc6d0\ud68c\ub294 IOC\uc5d0 \uac1c\ucd5c \uc2e0\uccad\uc744 \ud558\uace0, \uc2e0\uccad \ud6c4\uc5d0\ub294 \uc62c\ub9bc\ud53d \uac1c\ucd5c\uc5d0 \ub300\ud55c \uc9c8\uc758 \uc751\ub2f5\uc11c\ub97c \ubcf4\ub0b4\uc57c \ud55c\ub2e4. \uc774 \uc9c8\uc758\uc751\ub2f5\uc11c\uc5d0\uc11c \uc2e0\uccad\ud55c \ub3c4\uc2dc\ub294 \uc62c\ub9bc\ud53d \ud5cc\uc7a5\uc744 \uc900\uc218\ud558\uba70 IOC \uc0c1\uc784\uc774\uc0ac\ud68c\uc5d0 \uc758\ud55c \ub2e4\ub978 \uaddc\uc815\ub4e4\uc744 \uc9c0\ud0ac \uac83\uc774\ub77c\ub294 \ud655\uc2e0\uc744 \uc8fc\uc5b4\uc57c \ud55c\ub2e4. \uc774 \uc9c8\uc758\uc751\ub2f5\uc11c\ub294 \uc804\ubb38\uac00\ub4e4\uc774 \uac80\ud1a0\ud558\uc5ec \uc2e0\uccad \ub3c4\uc2dc\ub4e4\uc758 \uc7a0\uc7ac\uc131\uacfc \uacc4\ud68d\uc744 \ud3c9\uac00\ud55c\ub2e4. \uc774 \uc804\ubb38\uc801\uc778 \ud3c9\uac00\ub97c \ubc14\ud0d5\uc73c\ub85c IOC \uc0c1\uc784\uc774\uc0ac\ud68c\uc5d0\uc11c\ub294 \uc2e0\uccad\ub3c4\uc2dc \uc911\uc5d0\uc11c \ud6c4\ubcf4\ub3c4\uc2dc\ub97c \uace0\ub978\ub2e4.", "sentence_answer": "\uac1c\ucd5c\uc9c0 \uc120\uc815\uc5d0\ub294 \uc57d 2\ub144 \uc774 \uac78\ub9b0\ub2e4.", "paragraph_id": "6588307-31-1", "paragraph_question": "question: \uac1c\ucd5c\uc9c0 \uc120\uc815\uc5d0 \uac78\ub9ac\ub294 \uc2dc\uac04\uc740?, context: \uc62c\ub9bc\ud53d \uac1c\ucd5c\uc9c0\ub294 \ud574\ub2f9 \uc62c\ub9bc\ud53d \uac1c\ucd5c 7\ub144 \uc804\uc5d0 IOC \uc704\uc6d0\ub4e4\uc758 \ud22c\ud45c\ub85c \uacb0\uc815\ub41c\ub2e4. \uac1c\ucd5c\uc9c0 \uc120\uc815\uc5d0\ub294 \uc57d 2\ub144\uc774 \uac78\ub9b0\ub2e4. \uc720\uce58\ub97c \ud76c\ub9dd\ud558\ub294 \ub3c4\uc2dc\ub294 \uc6b0\uc120 \uc790\uad6d\uc758 \uc62c\ub9bc\ud53d \uc704\uc6d0\ud68c\uc5d0 \uc2e0\uccad\uc744 \ud574\uc57c \ud55c\ub2e4. \ub9cc\uc57d \ud55c \uad6d\uac00\uc5d0\uc11c \ub450 \ub3c4\uc2dc \uc774\uc0c1\uc774 \uc720\uce58\ub97c \ud76c\ub9dd\ud55c\ub2e4\uba74, \ud55c \uad6d\uac00\ub2f9 \ud55c \ub3c4\uc2dc\ub9cc \ud6c4\ubcf4\uac00 \ub420 \uc218 \uc788\ub2e4\ub294 \uaddc\uce59\uc5d0 \ub530\ub77c \ub0b4\ubd80\uc801\uc73c\ub85c \ud6c4\ubcf4 \ub3c4\uc2dc\ub97c \uacb0\uc815\ud574\uc57c \ud55c\ub2e4. \ud6c4\ubcf4 \ub3c4\uc2dc\uac00 \uacb0\uc815\ub418\uba74 \ud6c4\ubcf4 \ub3c4\uc2dc\uac00 \uc18c\uc18d\ub41c \uad6d\uac00\uc758 \uc62c\ub9bc\ud53d \uc704\uc6d0\ud68c\ub294 IOC\uc5d0 \uac1c\ucd5c \uc2e0\uccad\uc744 \ud558\uace0, \uc2e0\uccad \ud6c4\uc5d0\ub294 \uc62c\ub9bc\ud53d \uac1c\ucd5c\uc5d0 \ub300\ud55c \uc9c8\uc758 \uc751\ub2f5\uc11c\ub97c \ubcf4\ub0b4\uc57c \ud55c\ub2e4. \uc774 \uc9c8\uc758\uc751\ub2f5\uc11c\uc5d0\uc11c \uc2e0\uccad\ud55c \ub3c4\uc2dc\ub294 \uc62c\ub9bc\ud53d \ud5cc\uc7a5\uc744 \uc900\uc218\ud558\uba70 IOC \uc0c1\uc784\uc774\uc0ac\ud68c\uc5d0 \uc758\ud55c \ub2e4\ub978 \uaddc\uc815\ub4e4\uc744 \uc9c0\ud0ac \uac83\uc774\ub77c\ub294 \ud655\uc2e0\uc744 \uc8fc\uc5b4\uc57c \ud55c\ub2e4. \uc774 \uc9c8\uc758\uc751\ub2f5\uc11c\ub294 \uc804\ubb38\uac00\ub4e4\uc774 \uac80\ud1a0\ud558\uc5ec \uc2e0\uccad \ub3c4\uc2dc\ub4e4\uc758 \uc7a0\uc7ac\uc131\uacfc \uacc4\ud68d\uc744 \ud3c9\uac00\ud55c\ub2e4. \uc774 \uc804\ubb38\uc801\uc778 \ud3c9\uac00\ub97c \ubc14\ud0d5\uc73c\ub85c IOC \uc0c1\uc784\uc774\uc0ac\ud68c\uc5d0\uc11c\ub294 \uc2e0\uccad\ub3c4\uc2dc \uc911\uc5d0\uc11c \ud6c4\ubcf4\ub3c4\uc2dc\ub97c \uace0\ub978\ub2e4."} {"question": "2006\ub144 10\uc6d4 \ud6c4\uc548 \uc6b0\ub9ac\ubca0\ub294 \uc65c \uace0\uc18c\ub97c \ub2f9\ud588\ub294\uac00?", "paragraph": "2006\ub144 10\uc6d4 \uace0\ud5a5 \ub3c4\ubbf8\ub2c8\uce74 \uacf5\ud654\uad6d\uc5d0\uc11c \ucd1d\uaca9\uc744 \uac00\ud55c \ud610\uc758\ub97c \ubc1b\uc558\ub2e4. \uc548\ud1a0\ub2c8\uc624 \uace4\uc798\ub808\uc2a4\ub77c\ub294 \uc774\ub984\uc758 \ub18d\ubd80\uac00 \uacf5\uae30\ucd1d\uc5d0 \ub9de\uc544 \ubd80\uc0c1\ub2f9\ud588\ub2e4\uba70 \uadf8\ub97c \uace0\uc18c\ud588\ub2e4. \uc6b0\ub9ac\ubca0\uac00 \uadf8 \uc0ac\uac74\uc5d0 \uad00\ub828\uc774 \uc788\ub2e4\ub294 \uc5b4\ub5a4 \uba85\ud655\ud55c \uc99d\uac70\ub3c4 \uc5c6\ub2e4\ub294 \uc9c0\ubc29 \uac80\ucc30\uc758 \uc8fc\uc7a5\uc5d0\ub3c4 \ubd88\uad6c\ud558\uace0 \ub3c4\ubbf8\ub2c8\uce74 \ub2f9\uad6d\uc774 \uc0ac\uac74\uc744 \uc7ac\uc870\uc0ac\ud558\uae30\ub85c \uacb0\uc815\ud558\uba74\uc11c, \uc6b0\ub9ac\ubca0\ub294 \ucd9c\uad6d \uae08\uc9c0\ub97c \ub2f9\ud588\uace0 2007\ub144 1\uc6d4 5\uc77c\uc5d0 \uc0ac\uac74\uc774 \uc885\uacb0\ub418\uae30 \uc804\uae4c\uc9c0 \ub9e4\ub2ec 15\uc77c\uacfc 30\uc77c\uc5d0 \ubc95\uc6d0\uc5d0 \ucd9c\ub450\ud558\ub77c\ub294 \uba85\ub839\uc744 \ubc1b\uc558\ub2e4. \uc6b0\ub9ac\ubca0\ub294 \uacc4\uc18d\ud574\uc11c \uc0ac\uac74\uacfc \uc544\ubb34 \uad00\ub828\uc774 \uc5c6\uc74c\uc744 \uc8fc\uc7a5\ud558\uba74\uc11c, \ubc95\uc815 \uc808\ucc28\uac00 \ub05d\ub0a0 \ub54c\uae4c\uc9c0 \uc57c\uad6c\ub97c \ubabb\ud558\uac8c \ub420 \uac83\uc744 \uc6b0\ub824\ud588\ub2e4. \uc6b0\ub9ac\ubca0\ub294 \ubc95\uc815\uc5d0\uc11c \uc790\uc2e0\uc774 \uc774\uae38 \uac83\uc774\ub77c\uace0 \uc790\uc2e0\ud588\uace0, \uc18c\uc18d\ud300 \ud654\uc774\ud2b8\uc0ad\uc2a4\ub294 \uadf8\uac00 \uc2a4\ud504\ub9c1 \ud2b8\ub808\uc774\ub2dd\uc5d0 \ucc38\uc5ec\ud560 \uc218 \uc788\uc744 \uac83\uc73c\ub85c \ubcf4\uc558\ub2e4. 2\uc6d4 \ub2ec\uc774 \ub418\uc5b4\uc11c \uadf8\uac00 \uc0ac\uac74\uacfc \uc544\ubb34\ub7f0 \uad00\ub828\uc774 \uc5c6\uc74c\uc774 \ubc1d\ud600\uc84c\uc73c\uba70, \ubc95\uc6d0\uc740 \ub354 \uc774\uc0c1 \ubc95\uc815\uc5d0 \ucd9c\ub450\ud560 \ud544\uc694\uac00 \uc5c6\ub2e4\uace0 \uba85\ub839\ud588\ub2e4. \uc6b0\ub9ac\ubca0\ub294 \uc774 \uc0ac\uac74\uc744 \ub450\uace0 '\ube14\ub799\uba54\uc77c'(blackmail)\ub77c\ub294 \ud45c\ud604\uc744 \uc4f0\uba74\uc11c \ub3c8\uc744 \ub72f\uc5b4\uac00\ub824\ub294 \ub178\ub9bc\uc218\uc5d0 \ub2f9\ud55c \uac83\uc774\ub77c\uace0 \ub9d0\ud588\ub2e4.", "answer": "\ucd1d\uaca9\uc744 \uac00\ud55c \ud610\uc758", "sentence": "2006\ub144 10\uc6d4 \uace0\ud5a5 \ub3c4\ubbf8\ub2c8\uce74 \uacf5\ud654\uad6d\uc5d0\uc11c \ucd1d\uaca9\uc744 \uac00\ud55c \ud610\uc758 \ub97c \ubc1b\uc558\ub2e4.", "paragraph_sentence": " 2006\ub144 10\uc6d4 \uace0\ud5a5 \ub3c4\ubbf8\ub2c8\uce74 \uacf5\ud654\uad6d\uc5d0\uc11c \ucd1d\uaca9\uc744 \uac00\ud55c \ud610\uc758 \ub97c \ubc1b\uc558\ub2e4. \uc548\ud1a0\ub2c8\uc624 \uace4\uc798\ub808\uc2a4\ub77c\ub294 \uc774\ub984\uc758 \ub18d\ubd80\uac00 \uacf5\uae30\ucd1d\uc5d0 \ub9de\uc544 \ubd80\uc0c1\ub2f9\ud588\ub2e4\uba70 \uadf8\ub97c \uace0\uc18c\ud588\ub2e4. \uc6b0\ub9ac\ubca0\uac00 \uadf8 \uc0ac\uac74\uc5d0 \uad00\ub828\uc774 \uc788\ub2e4\ub294 \uc5b4\ub5a4 \uba85\ud655\ud55c \uc99d\uac70\ub3c4 \uc5c6\ub2e4\ub294 \uc9c0\ubc29 \uac80\ucc30\uc758 \uc8fc\uc7a5\uc5d0\ub3c4 \ubd88\uad6c\ud558\uace0 \ub3c4\ubbf8\ub2c8\uce74 \ub2f9\uad6d\uc774 \uc0ac\uac74\uc744 \uc7ac\uc870\uc0ac\ud558\uae30\ub85c \uacb0\uc815\ud558\uba74\uc11c, \uc6b0\ub9ac\ubca0\ub294 \ucd9c\uad6d \uae08\uc9c0\ub97c \ub2f9\ud588\uace0 2007\ub144 1\uc6d4 5\uc77c\uc5d0 \uc0ac\uac74\uc774 \uc885\uacb0\ub418\uae30 \uc804\uae4c\uc9c0 \ub9e4\ub2ec 15\uc77c\uacfc 30\uc77c\uc5d0 \ubc95\uc6d0\uc5d0 \ucd9c\ub450\ud558\ub77c\ub294 \uba85\ub839\uc744 \ubc1b\uc558\ub2e4. \uc6b0\ub9ac\ubca0\ub294 \uacc4\uc18d\ud574\uc11c \uc0ac\uac74\uacfc \uc544\ubb34 \uad00\ub828\uc774 \uc5c6\uc74c\uc744 \uc8fc\uc7a5\ud558\uba74\uc11c, \ubc95\uc815 \uc808\ucc28\uac00 \ub05d\ub0a0 \ub54c\uae4c\uc9c0 \uc57c\uad6c\ub97c \ubabb\ud558\uac8c \ub420 \uac83\uc744 \uc6b0\ub824\ud588\ub2e4. \uc6b0\ub9ac\ubca0\ub294 \ubc95\uc815\uc5d0\uc11c \uc790\uc2e0\uc774 \uc774\uae38 \uac83\uc774\ub77c\uace0 \uc790\uc2e0\ud588\uace0, \uc18c\uc18d\ud300 \ud654\uc774\ud2b8\uc0ad\uc2a4\ub294 \uadf8\uac00 \uc2a4\ud504\ub9c1 \ud2b8\ub808\uc774\ub2dd\uc5d0 \ucc38\uc5ec\ud560 \uc218 \uc788\uc744 \uac83\uc73c\ub85c \ubcf4\uc558\ub2e4. 2\uc6d4 \ub2ec\uc774 \ub418\uc5b4\uc11c \uadf8\uac00 \uc0ac\uac74\uacfc \uc544\ubb34\ub7f0 \uad00\ub828\uc774 \uc5c6\uc74c\uc774 \ubc1d\ud600\uc84c\uc73c\uba70, \ubc95\uc6d0\uc740 \ub354 \uc774\uc0c1 \ubc95\uc815\uc5d0 \ucd9c\ub450\ud560 \ud544\uc694\uac00 \uc5c6\ub2e4\uace0 \uba85\ub839\ud588\ub2e4. \uc6b0\ub9ac\ubca0\ub294 \uc774 \uc0ac\uac74\uc744 \ub450\uace0 '\ube14\ub799\uba54\uc77c'(blackmail)\ub77c\ub294 \ud45c\ud604\uc744 \uc4f0\uba74\uc11c \ub3c8\uc744 \ub72f\uc5b4\uac00\ub824\ub294 \ub178\ub9bc\uc218\uc5d0 \ub2f9\ud55c \uac83\uc774\ub77c\uace0 \ub9d0\ud588\ub2e4.", "paragraph_answer": "2006\ub144 10\uc6d4 \uace0\ud5a5 \ub3c4\ubbf8\ub2c8\uce74 \uacf5\ud654\uad6d\uc5d0\uc11c \ucd1d\uaca9\uc744 \uac00\ud55c \ud610\uc758 \ub97c \ubc1b\uc558\ub2e4. \uc548\ud1a0\ub2c8\uc624 \uace4\uc798\ub808\uc2a4\ub77c\ub294 \uc774\ub984\uc758 \ub18d\ubd80\uac00 \uacf5\uae30\ucd1d\uc5d0 \ub9de\uc544 \ubd80\uc0c1\ub2f9\ud588\ub2e4\uba70 \uadf8\ub97c \uace0\uc18c\ud588\ub2e4. \uc6b0\ub9ac\ubca0\uac00 \uadf8 \uc0ac\uac74\uc5d0 \uad00\ub828\uc774 \uc788\ub2e4\ub294 \uc5b4\ub5a4 \uba85\ud655\ud55c \uc99d\uac70\ub3c4 \uc5c6\ub2e4\ub294 \uc9c0\ubc29 \uac80\ucc30\uc758 \uc8fc\uc7a5\uc5d0\ub3c4 \ubd88\uad6c\ud558\uace0 \ub3c4\ubbf8\ub2c8\uce74 \ub2f9\uad6d\uc774 \uc0ac\uac74\uc744 \uc7ac\uc870\uc0ac\ud558\uae30\ub85c \uacb0\uc815\ud558\uba74\uc11c, \uc6b0\ub9ac\ubca0\ub294 \ucd9c\uad6d \uae08\uc9c0\ub97c \ub2f9\ud588\uace0 2007\ub144 1\uc6d4 5\uc77c\uc5d0 \uc0ac\uac74\uc774 \uc885\uacb0\ub418\uae30 \uc804\uae4c\uc9c0 \ub9e4\ub2ec 15\uc77c\uacfc 30\uc77c\uc5d0 \ubc95\uc6d0\uc5d0 \ucd9c\ub450\ud558\ub77c\ub294 \uba85\ub839\uc744 \ubc1b\uc558\ub2e4. \uc6b0\ub9ac\ubca0\ub294 \uacc4\uc18d\ud574\uc11c \uc0ac\uac74\uacfc \uc544\ubb34 \uad00\ub828\uc774 \uc5c6\uc74c\uc744 \uc8fc\uc7a5\ud558\uba74\uc11c, \ubc95\uc815 \uc808\ucc28\uac00 \ub05d\ub0a0 \ub54c\uae4c\uc9c0 \uc57c\uad6c\ub97c \ubabb\ud558\uac8c \ub420 \uac83\uc744 \uc6b0\ub824\ud588\ub2e4. \uc6b0\ub9ac\ubca0\ub294 \ubc95\uc815\uc5d0\uc11c \uc790\uc2e0\uc774 \uc774\uae38 \uac83\uc774\ub77c\uace0 \uc790\uc2e0\ud588\uace0, \uc18c\uc18d\ud300 \ud654\uc774\ud2b8\uc0ad\uc2a4\ub294 \uadf8\uac00 \uc2a4\ud504\ub9c1 \ud2b8\ub808\uc774\ub2dd\uc5d0 \ucc38\uc5ec\ud560 \uc218 \uc788\uc744 \uac83\uc73c\ub85c \ubcf4\uc558\ub2e4. 2\uc6d4 \ub2ec\uc774 \ub418\uc5b4\uc11c \uadf8\uac00 \uc0ac\uac74\uacfc \uc544\ubb34\ub7f0 \uad00\ub828\uc774 \uc5c6\uc74c\uc774 \ubc1d\ud600\uc84c\uc73c\uba70, \ubc95\uc6d0\uc740 \ub354 \uc774\uc0c1 \ubc95\uc815\uc5d0 \ucd9c\ub450\ud560 \ud544\uc694\uac00 \uc5c6\ub2e4\uace0 \uba85\ub839\ud588\ub2e4. \uc6b0\ub9ac\ubca0\ub294 \uc774 \uc0ac\uac74\uc744 \ub450\uace0 '\ube14\ub799\uba54\uc77c'(blackmail)\ub77c\ub294 \ud45c\ud604\uc744 \uc4f0\uba74\uc11c \ub3c8\uc744 \ub72f\uc5b4\uac00\ub824\ub294 \ub178\ub9bc\uc218\uc5d0 \ub2f9\ud55c \uac83\uc774\ub77c\uace0 \ub9d0\ud588\ub2e4.", "sentence_answer": "2006\ub144 10\uc6d4 \uace0\ud5a5 \ub3c4\ubbf8\ub2c8\uce74 \uacf5\ud654\uad6d\uc5d0\uc11c \ucd1d\uaca9\uc744 \uac00\ud55c \ud610\uc758 \ub97c \ubc1b\uc558\ub2e4.", "paragraph_id": "6246299-9-0", "paragraph_question": "question: 2006\ub144 10\uc6d4 \ud6c4\uc548 \uc6b0\ub9ac\ubca0\ub294 \uc65c \uace0\uc18c\ub97c \ub2f9\ud588\ub294\uac00?, context: 2006\ub144 10\uc6d4 \uace0\ud5a5 \ub3c4\ubbf8\ub2c8\uce74 \uacf5\ud654\uad6d\uc5d0\uc11c \ucd1d\uaca9\uc744 \uac00\ud55c \ud610\uc758\ub97c \ubc1b\uc558\ub2e4. \uc548\ud1a0\ub2c8\uc624 \uace4\uc798\ub808\uc2a4\ub77c\ub294 \uc774\ub984\uc758 \ub18d\ubd80\uac00 \uacf5\uae30\ucd1d\uc5d0 \ub9de\uc544 \ubd80\uc0c1\ub2f9\ud588\ub2e4\uba70 \uadf8\ub97c \uace0\uc18c\ud588\ub2e4. \uc6b0\ub9ac\ubca0\uac00 \uadf8 \uc0ac\uac74\uc5d0 \uad00\ub828\uc774 \uc788\ub2e4\ub294 \uc5b4\ub5a4 \uba85\ud655\ud55c \uc99d\uac70\ub3c4 \uc5c6\ub2e4\ub294 \uc9c0\ubc29 \uac80\ucc30\uc758 \uc8fc\uc7a5\uc5d0\ub3c4 \ubd88\uad6c\ud558\uace0 \ub3c4\ubbf8\ub2c8\uce74 \ub2f9\uad6d\uc774 \uc0ac\uac74\uc744 \uc7ac\uc870\uc0ac\ud558\uae30\ub85c \uacb0\uc815\ud558\uba74\uc11c, \uc6b0\ub9ac\ubca0\ub294 \ucd9c\uad6d \uae08\uc9c0\ub97c \ub2f9\ud588\uace0 2007\ub144 1\uc6d4 5\uc77c\uc5d0 \uc0ac\uac74\uc774 \uc885\uacb0\ub418\uae30 \uc804\uae4c\uc9c0 \ub9e4\ub2ec 15\uc77c\uacfc 30\uc77c\uc5d0 \ubc95\uc6d0\uc5d0 \ucd9c\ub450\ud558\ub77c\ub294 \uba85\ub839\uc744 \ubc1b\uc558\ub2e4. \uc6b0\ub9ac\ubca0\ub294 \uacc4\uc18d\ud574\uc11c \uc0ac\uac74\uacfc \uc544\ubb34 \uad00\ub828\uc774 \uc5c6\uc74c\uc744 \uc8fc\uc7a5\ud558\uba74\uc11c, \ubc95\uc815 \uc808\ucc28\uac00 \ub05d\ub0a0 \ub54c\uae4c\uc9c0 \uc57c\uad6c\ub97c \ubabb\ud558\uac8c \ub420 \uac83\uc744 \uc6b0\ub824\ud588\ub2e4. \uc6b0\ub9ac\ubca0\ub294 \ubc95\uc815\uc5d0\uc11c \uc790\uc2e0\uc774 \uc774\uae38 \uac83\uc774\ub77c\uace0 \uc790\uc2e0\ud588\uace0, \uc18c\uc18d\ud300 \ud654\uc774\ud2b8\uc0ad\uc2a4\ub294 \uadf8\uac00 \uc2a4\ud504\ub9c1 \ud2b8\ub808\uc774\ub2dd\uc5d0 \ucc38\uc5ec\ud560 \uc218 \uc788\uc744 \uac83\uc73c\ub85c \ubcf4\uc558\ub2e4. 2\uc6d4 \ub2ec\uc774 \ub418\uc5b4\uc11c \uadf8\uac00 \uc0ac\uac74\uacfc \uc544\ubb34\ub7f0 \uad00\ub828\uc774 \uc5c6\uc74c\uc774 \ubc1d\ud600\uc84c\uc73c\uba70, \ubc95\uc6d0\uc740 \ub354 \uc774\uc0c1 \ubc95\uc815\uc5d0 \ucd9c\ub450\ud560 \ud544\uc694\uac00 \uc5c6\ub2e4\uace0 \uba85\ub839\ud588\ub2e4. \uc6b0\ub9ac\ubca0\ub294 \uc774 \uc0ac\uac74\uc744 \ub450\uace0 '\ube14\ub799\uba54\uc77c'(blackmail)\ub77c\ub294 \ud45c\ud604\uc744 \uc4f0\uba74\uc11c \ub3c8\uc744 \ub72f\uc5b4\uac00\ub824\ub294 \ub178\ub9bc\uc218\uc5d0 \ub2f9\ud55c \uac83\uc774\ub77c\uace0 \ub9d0\ud588\ub2e4."} {"question": "\ud3ed\uc2a4\uc758 \uc2dc\ud2b8\ucf64 \ud0a4\uce5c \ucee8\ud53c\ub374\uc15c\uc740 \uc88b\uc740 \ud3c9\uc5d0\ub3c4 \ubd88\uad6c\ud558\uace0 \uba87 \ud68c\ub9cc \ubc29\uc601\ud558\uc600\ub294\uac00?", "paragraph": "2005\ub144, \ucfe0\ud37c\ub294 \u300a\uc131\ubc94\uc8c4\uc218\uc0ac\ub300: SVU\u300b\uc640 \u300aLaw & Order: Trial by Jury\u300b\uc758 \uac01 \uc5d0\ud53c\uc18c\ub4dc\uc5d0\uc11c \uc81c\uc774\uc2a8 \ud718\ud130\ucee4 \uc5ed\uc73c\ub85c \ucd9c\uc5f0\ud588\ub2e4. 2005\ub144 9\uc6d4, \ud3ed\uc2a4\uc758 \uc2dc\ud2b8\ucf64 \u300a\ud0a4\uce5c \ucee8\ud53c\ub374\uc15c\u300b\uc5d0\uc11c \uc8fc\uc778\uacf5 \uc7ad \ubcf4\ub518 \uc5ed\uc744 \uc5f0\uae30\ud588\ub2e4. \uc774 \uc2dc\ud2b8\ucf64\uc740 \uc564\uc11c\ub2c8 \ubcf4\ub518\uc758 \ub3d9\uba85 \ud68c\uace0\ub85d\uc744 \uc6d0\uc791\uc73c\ub85c \ud55c \uac83\uc774\ub2e4. \uc2dc\ud2b8\ucf64\uc5d0 \ub300\ud574 \uc88b\uc740 \ud3c9\uc744 \ubc1b\uc558\uc9c0\ub9cc 4\ud68c\ub9cc \ubc29\uc601\ub418\uace0 \ucde8\uc18c\ub418\uc5c8\ub2e4. 2006\ub144 3\uc6d4, \ube0c\ub85c\ub4dc\uc6e8\uc774 \uc5f0\uadf9 \u300a\uc2a4\ub9ac \ub370\uc774\uc2a4 \uc624\ube0c \ub808\uc778\u300b\uc744 \uc904\ub9ac\uc544 \ub85c\ubc84\uce20\uc640 \ud3f4 \ub7ec\ub4dc\uc640 \ud568\uaed8 \ubc84\ub098\ub4dc B. \uc81c\uc774\ucef5\uc2a4 \uadf9\uc7a5\uc5d0\uc11c \uc5f0\uae30\ud588\ub2e4. \ucfe0\ud37c\ub294 \ub610\ud55c 2008\ub144 \ud14c\ub9ac\uc0ac \ub9ac\ubca0\ud06c\uc758 \uc5f0\uadf9 \u300a\ub514 \uc5b8\ub354\uc2a4\ud130\ub514\u300b\ub97c \uc70c\ub9ac\uc5c4\uc2a4\ud0c0\uc6b4 \uc5f0\uadf9\uc81c\uc5d0\uc11c \ud06c\ub9ac\uc2a4\ud2f4 \uc874\uc2a4\ud134\uacfc \uac19\uc774 \uc5f0\uae30\ud588\ub2e4. 2007\ub144 \ub4dc\ub77c\ub9c8 \u300a\ub2d9\ud131\u300b \uc2dc\uc98c 5\uc5d0\uc11c \ud558\uce20 \uc564 \uc2a4\uce7c\ud3a0\uc2a4\ub77c\ub294 \uac00\uc0c1\uc758 \ud504\ub85c\uadf8\ub7a8 \uc2a4\ud0c0\uc778 \uc5d0\uc774\ub358 \uc2a4\ud1a4 \uc5ed\uc73c\ub85c 6\uac1c\uc758 \uc5d0\ud53c\uc18c\ub4dc\uc5d0 \ucd9c\uc5f0\ud588\ub2e4. 2008\ub144\uc5d0\ub294 \uc601\ud654 \u300a\ub85d\ucee4\u300b\uc5d0\uc11c \ud2b8\ub798\uc2dc \uadf8\ub77c\uc774\uc2a4 \uc5ed\uc73c\ub85c \ucd9c\uc5f0\ud558\uc600\ub2e4. \uac19\uc740 \ud574, \u300a\uc608\uc2a4 \ub9e8\u300b\uc5d0\uc11c \uc9d0 \uce90\ub9ac\uc640 \uac19\uc774 \uc5f0\uae30\ud558\uc600\ub2e4. \ub610\ud55c, \ubbf8\uc2a4\ud130\ub9ac \uc2a4\ub9b4\ub7ec \uc601\ud654 \u300a\ubbf8\ub4dc\ub098\uc787 \ubbf8\ud2b8 \ud2b8\ub808\uc778\u300b\uc5d0\uc11c \ub9ac\uc5b8 \ucf54\ud504\uba3c \uc5ed\uc744 \ub9e1\uc558\ub2e4. \uc774 \uc601\ud654\ub294 \ud074\ub77c\uc774\ube0c \ubc14\ucee4\uc758 \ub2e8\ud3b8 \uc18c\uc124\uc744 \uc601\ud654\ud654\ud55c \uac83\uc73c\ub85c, \uc88b\uc740 \ud3c9\uc744 \ubc1b\uc558\ub2e4.", "answer": "4\ud68c", "sentence": "\uc2dc\ud2b8\ucf64\uc5d0 \ub300\ud574 \uc88b\uc740 \ud3c9\uc744 \ubc1b\uc558\uc9c0\ub9cc 4\ud68c \ub9cc \ubc29\uc601\ub418\uace0 \ucde8\uc18c\ub418\uc5c8\ub2e4.", "paragraph_sentence": "2005\ub144, \ucfe0\ud37c\ub294 \u300a\uc131\ubc94\uc8c4\uc218\uc0ac\ub300: SVU\u300b\uc640 \u300aLaw & Order: Trial by Jury\u300b\uc758 \uac01 \uc5d0\ud53c\uc18c\ub4dc\uc5d0\uc11c \uc81c\uc774\uc2a8 \ud718\ud130\ucee4 \uc5ed\uc73c\ub85c \ucd9c\uc5f0\ud588\ub2e4. 2005\ub144 9\uc6d4, \ud3ed\uc2a4\uc758 \uc2dc\ud2b8\ucf64 \u300a\ud0a4\uce5c \ucee8\ud53c\ub374\uc15c\u300b\uc5d0\uc11c \uc8fc\uc778\uacf5 \uc7ad \ubcf4\ub518 \uc5ed\uc744 \uc5f0\uae30\ud588\ub2e4. \uc774 \uc2dc\ud2b8\ucf64\uc740 \uc564\uc11c\ub2c8 \ubcf4\ub518\uc758 \ub3d9\uba85 \ud68c\uace0\ub85d\uc744 \uc6d0\uc791\uc73c\ub85c \ud55c \uac83\uc774\ub2e4. \uc2dc\ud2b8\ucf64\uc5d0 \ub300\ud574 \uc88b\uc740 \ud3c9\uc744 \ubc1b\uc558\uc9c0\ub9cc 4\ud68c \ub9cc \ubc29\uc601\ub418\uace0 \ucde8\uc18c\ub418\uc5c8\ub2e4. 2006\ub144 3\uc6d4, \ube0c\ub85c\ub4dc\uc6e8\uc774 \uc5f0\uadf9 \u300a\uc2a4\ub9ac \ub370\uc774\uc2a4 \uc624\ube0c \ub808\uc778\u300b\uc744 \uc904\ub9ac\uc544 \ub85c\ubc84\uce20\uc640 \ud3f4 \ub7ec\ub4dc\uc640 \ud568\uaed8 \ubc84\ub098\ub4dc B. \uc81c\uc774\ucef5\uc2a4 \uadf9\uc7a5\uc5d0\uc11c \uc5f0\uae30\ud588\ub2e4. \ucfe0\ud37c\ub294 \ub610\ud55c 2008\ub144 \ud14c\ub9ac\uc0ac \ub9ac\ubca0\ud06c\uc758 \uc5f0\uadf9 \u300a\ub514 \uc5b8\ub354\uc2a4\ud130\ub514\u300b\ub97c \uc70c\ub9ac\uc5c4\uc2a4\ud0c0\uc6b4 \uc5f0\uadf9\uc81c\uc5d0\uc11c \ud06c\ub9ac\uc2a4\ud2f4 \uc874\uc2a4\ud134\uacfc \uac19\uc774 \uc5f0\uae30\ud588\ub2e4. 2007\ub144 \ub4dc\ub77c\ub9c8 \u300a\ub2d9\ud131\u300b \uc2dc\uc98c 5\uc5d0\uc11c \ud558\uce20 \uc564 \uc2a4\uce7c\ud3a0\uc2a4\ub77c\ub294 \uac00\uc0c1\uc758 \ud504\ub85c\uadf8\ub7a8 \uc2a4\ud0c0\uc778 \uc5d0\uc774\ub358 \uc2a4\ud1a4 \uc5ed\uc73c\ub85c 6\uac1c\uc758 \uc5d0\ud53c\uc18c\ub4dc\uc5d0 \ucd9c\uc5f0\ud588\ub2e4. 2008\ub144\uc5d0\ub294 \uc601\ud654 \u300a\ub85d\ucee4\u300b\uc5d0\uc11c \ud2b8\ub798\uc2dc \uadf8\ub77c\uc774\uc2a4 \uc5ed\uc73c\ub85c \ucd9c\uc5f0\ud558\uc600\ub2e4. \uac19\uc740 \ud574, \u300a\uc608\uc2a4 \ub9e8\u300b\uc5d0\uc11c \uc9d0 \uce90\ub9ac\uc640 \uac19\uc774 \uc5f0\uae30\ud558\uc600\ub2e4. \ub610\ud55c, \ubbf8\uc2a4\ud130\ub9ac \uc2a4\ub9b4\ub7ec \uc601\ud654 \u300a\ubbf8\ub4dc\ub098\uc787 \ubbf8\ud2b8 \ud2b8\ub808\uc778\u300b\uc5d0\uc11c \ub9ac\uc5b8 \ucf54\ud504\uba3c \uc5ed\uc744 \ub9e1\uc558\ub2e4. \uc774 \uc601\ud654\ub294 \ud074\ub77c\uc774\ube0c \ubc14\ucee4\uc758 \ub2e8\ud3b8 \uc18c\uc124\uc744 \uc601\ud654\ud654\ud55c \uac83\uc73c\ub85c, \uc88b\uc740 \ud3c9\uc744 \ubc1b\uc558\ub2e4.", "paragraph_answer": "2005\ub144, \ucfe0\ud37c\ub294 \u300a\uc131\ubc94\uc8c4\uc218\uc0ac\ub300: SVU\u300b\uc640 \u300aLaw & Order: Trial by Jury\u300b\uc758 \uac01 \uc5d0\ud53c\uc18c\ub4dc\uc5d0\uc11c \uc81c\uc774\uc2a8 \ud718\ud130\ucee4 \uc5ed\uc73c\ub85c \ucd9c\uc5f0\ud588\ub2e4. 2005\ub144 9\uc6d4, \ud3ed\uc2a4\uc758 \uc2dc\ud2b8\ucf64 \u300a\ud0a4\uce5c \ucee8\ud53c\ub374\uc15c\u300b\uc5d0\uc11c \uc8fc\uc778\uacf5 \uc7ad \ubcf4\ub518 \uc5ed\uc744 \uc5f0\uae30\ud588\ub2e4. \uc774 \uc2dc\ud2b8\ucf64\uc740 \uc564\uc11c\ub2c8 \ubcf4\ub518\uc758 \ub3d9\uba85 \ud68c\uace0\ub85d\uc744 \uc6d0\uc791\uc73c\ub85c \ud55c \uac83\uc774\ub2e4. \uc2dc\ud2b8\ucf64\uc5d0 \ub300\ud574 \uc88b\uc740 \ud3c9\uc744 \ubc1b\uc558\uc9c0\ub9cc 4\ud68c \ub9cc \ubc29\uc601\ub418\uace0 \ucde8\uc18c\ub418\uc5c8\ub2e4. 2006\ub144 3\uc6d4, \ube0c\ub85c\ub4dc\uc6e8\uc774 \uc5f0\uadf9 \u300a\uc2a4\ub9ac \ub370\uc774\uc2a4 \uc624\ube0c \ub808\uc778\u300b\uc744 \uc904\ub9ac\uc544 \ub85c\ubc84\uce20\uc640 \ud3f4 \ub7ec\ub4dc\uc640 \ud568\uaed8 \ubc84\ub098\ub4dc B. \uc81c\uc774\ucef5\uc2a4 \uadf9\uc7a5\uc5d0\uc11c \uc5f0\uae30\ud588\ub2e4. \ucfe0\ud37c\ub294 \ub610\ud55c 2008\ub144 \ud14c\ub9ac\uc0ac \ub9ac\ubca0\ud06c\uc758 \uc5f0\uadf9 \u300a\ub514 \uc5b8\ub354\uc2a4\ud130\ub514\u300b\ub97c \uc70c\ub9ac\uc5c4\uc2a4\ud0c0\uc6b4 \uc5f0\uadf9\uc81c\uc5d0\uc11c \ud06c\ub9ac\uc2a4\ud2f4 \uc874\uc2a4\ud134\uacfc \uac19\uc774 \uc5f0\uae30\ud588\ub2e4. 2007\ub144 \ub4dc\ub77c\ub9c8 \u300a\ub2d9\ud131\u300b \uc2dc\uc98c 5\uc5d0\uc11c \ud558\uce20 \uc564 \uc2a4\uce7c\ud3a0\uc2a4\ub77c\ub294 \uac00\uc0c1\uc758 \ud504\ub85c\uadf8\ub7a8 \uc2a4\ud0c0\uc778 \uc5d0\uc774\ub358 \uc2a4\ud1a4 \uc5ed\uc73c\ub85c 6\uac1c\uc758 \uc5d0\ud53c\uc18c\ub4dc\uc5d0 \ucd9c\uc5f0\ud588\ub2e4. 2008\ub144\uc5d0\ub294 \uc601\ud654 \u300a\ub85d\ucee4\u300b\uc5d0\uc11c \ud2b8\ub798\uc2dc \uadf8\ub77c\uc774\uc2a4 \uc5ed\uc73c\ub85c \ucd9c\uc5f0\ud558\uc600\ub2e4. \uac19\uc740 \ud574, \u300a\uc608\uc2a4 \ub9e8\u300b\uc5d0\uc11c \uc9d0 \uce90\ub9ac\uc640 \uac19\uc774 \uc5f0\uae30\ud558\uc600\ub2e4. \ub610\ud55c, \ubbf8\uc2a4\ud130\ub9ac \uc2a4\ub9b4\ub7ec \uc601\ud654 \u300a\ubbf8\ub4dc\ub098\uc787 \ubbf8\ud2b8 \ud2b8\ub808\uc778\u300b\uc5d0\uc11c \ub9ac\uc5b8 \ucf54\ud504\uba3c \uc5ed\uc744 \ub9e1\uc558\ub2e4. \uc774 \uc601\ud654\ub294 \ud074\ub77c\uc774\ube0c \ubc14\ucee4\uc758 \ub2e8\ud3b8 \uc18c\uc124\uc744 \uc601\ud654\ud654\ud55c \uac83\uc73c\ub85c, \uc88b\uc740 \ud3c9\uc744 \ubc1b\uc558\ub2e4.", "sentence_answer": "\uc2dc\ud2b8\ucf64\uc5d0 \ub300\ud574 \uc88b\uc740 \ud3c9\uc744 \ubc1b\uc558\uc9c0\ub9cc 4\ud68c \ub9cc \ubc29\uc601\ub418\uace0 \ucde8\uc18c\ub418\uc5c8\ub2e4.", "paragraph_id": "6509370-2-0", "paragraph_question": "question: \ud3ed\uc2a4\uc758 \uc2dc\ud2b8\ucf64 \ud0a4\uce5c \ucee8\ud53c\ub374\uc15c\uc740 \uc88b\uc740 \ud3c9\uc5d0\ub3c4 \ubd88\uad6c\ud558\uace0 \uba87 \ud68c\ub9cc \ubc29\uc601\ud558\uc600\ub294\uac00?, context: 2005\ub144, \ucfe0\ud37c\ub294 \u300a\uc131\ubc94\uc8c4\uc218\uc0ac\ub300: SVU\u300b\uc640 \u300aLaw & Order: Trial by Jury\u300b\uc758 \uac01 \uc5d0\ud53c\uc18c\ub4dc\uc5d0\uc11c \uc81c\uc774\uc2a8 \ud718\ud130\ucee4 \uc5ed\uc73c\ub85c \ucd9c\uc5f0\ud588\ub2e4. 2005\ub144 9\uc6d4, \ud3ed\uc2a4\uc758 \uc2dc\ud2b8\ucf64 \u300a\ud0a4\uce5c \ucee8\ud53c\ub374\uc15c\u300b\uc5d0\uc11c \uc8fc\uc778\uacf5 \uc7ad \ubcf4\ub518 \uc5ed\uc744 \uc5f0\uae30\ud588\ub2e4. \uc774 \uc2dc\ud2b8\ucf64\uc740 \uc564\uc11c\ub2c8 \ubcf4\ub518\uc758 \ub3d9\uba85 \ud68c\uace0\ub85d\uc744 \uc6d0\uc791\uc73c\ub85c \ud55c \uac83\uc774\ub2e4. \uc2dc\ud2b8\ucf64\uc5d0 \ub300\ud574 \uc88b\uc740 \ud3c9\uc744 \ubc1b\uc558\uc9c0\ub9cc 4\ud68c\ub9cc \ubc29\uc601\ub418\uace0 \ucde8\uc18c\ub418\uc5c8\ub2e4. 2006\ub144 3\uc6d4, \ube0c\ub85c\ub4dc\uc6e8\uc774 \uc5f0\uadf9 \u300a\uc2a4\ub9ac \ub370\uc774\uc2a4 \uc624\ube0c \ub808\uc778\u300b\uc744 \uc904\ub9ac\uc544 \ub85c\ubc84\uce20\uc640 \ud3f4 \ub7ec\ub4dc\uc640 \ud568\uaed8 \ubc84\ub098\ub4dc B. \uc81c\uc774\ucef5\uc2a4 \uadf9\uc7a5\uc5d0\uc11c \uc5f0\uae30\ud588\ub2e4. \ucfe0\ud37c\ub294 \ub610\ud55c 2008\ub144 \ud14c\ub9ac\uc0ac \ub9ac\ubca0\ud06c\uc758 \uc5f0\uadf9 \u300a\ub514 \uc5b8\ub354\uc2a4\ud130\ub514\u300b\ub97c \uc70c\ub9ac\uc5c4\uc2a4\ud0c0\uc6b4 \uc5f0\uadf9\uc81c\uc5d0\uc11c \ud06c\ub9ac\uc2a4\ud2f4 \uc874\uc2a4\ud134\uacfc \uac19\uc774 \uc5f0\uae30\ud588\ub2e4. 2007\ub144 \ub4dc\ub77c\ub9c8 \u300a\ub2d9\ud131\u300b \uc2dc\uc98c 5\uc5d0\uc11c \ud558\uce20 \uc564 \uc2a4\uce7c\ud3a0\uc2a4\ub77c\ub294 \uac00\uc0c1\uc758 \ud504\ub85c\uadf8\ub7a8 \uc2a4\ud0c0\uc778 \uc5d0\uc774\ub358 \uc2a4\ud1a4 \uc5ed\uc73c\ub85c 6\uac1c\uc758 \uc5d0\ud53c\uc18c\ub4dc\uc5d0 \ucd9c\uc5f0\ud588\ub2e4. 2008\ub144\uc5d0\ub294 \uc601\ud654 \u300a\ub85d\ucee4\u300b\uc5d0\uc11c \ud2b8\ub798\uc2dc \uadf8\ub77c\uc774\uc2a4 \uc5ed\uc73c\ub85c \ucd9c\uc5f0\ud558\uc600\ub2e4. \uac19\uc740 \ud574, \u300a\uc608\uc2a4 \ub9e8\u300b\uc5d0\uc11c \uc9d0 \uce90\ub9ac\uc640 \uac19\uc774 \uc5f0\uae30\ud558\uc600\ub2e4. \ub610\ud55c, \ubbf8\uc2a4\ud130\ub9ac \uc2a4\ub9b4\ub7ec \uc601\ud654 \u300a\ubbf8\ub4dc\ub098\uc787 \ubbf8\ud2b8 \ud2b8\ub808\uc778\u300b\uc5d0\uc11c \ub9ac\uc5b8 \ucf54\ud504\uba3c \uc5ed\uc744 \ub9e1\uc558\ub2e4. \uc774 \uc601\ud654\ub294 \ud074\ub77c\uc774\ube0c \ubc14\ucee4\uc758 \ub2e8\ud3b8 \uc18c\uc124\uc744 \uc601\ud654\ud654\ud55c \uac83\uc73c\ub85c, \uc88b\uc740 \ud3c9\uc744 \ubc1b\uc558\ub2e4."} {"question": "\uc720\uae38\uc900\uc774 \uad00\ub9ac\ub4e4\uc744 \ud1b5\ud574 \uac00\uc704\ub97c \ub4e4\uace0 \ubc31\uc131\ub4e4\uc758 \uba38\ub9ac\ub97c \uae4d\uc774\ub3c4\ub85d \uc9c0\uc2dc\ud558\uace0 \ub0b4\ubd80\uace0\uc2dc\ub97c \ud1b5\ud574 12\uc6d4 30\uc77c \ud3ec\uace0\ud55c\uac83\uc740 \ubb34\uc5c7\uc778\uac00?", "paragraph": "12\uc6d4 30\uc77c(\uc74c\ub825 11\uc6d4 15\uc77c) \uc624\ud6c4 \uc720\uae38\uc900\uc740 \ub0b4\ubd80\uace0\uc2dc(\u5167\u90e8\u544a\u793a)\ub97c \ud1b5\ud574 \ub2f9\uc77c\ub85c \uc804\uad6d \ubc29\ubc29\uace1\uace1\uc5d0 \uc77c\uc81c\ud788 \ub2e8\ubc1c\ub839\uc744 \ud3ec\uace0\ud558\uc600\ub2e4. \uc720\uae38\uc900\uc740 \ubc14\ub85c \uad00\ub9ac\ub4e4\ub85c \ud558\uc5ec\uae08 \uac00\uc704\ub97c \ub4e4\uace0 \ud55c\uc131\ubd80\uc758 \uac70\ub9ac\ub098 4\ub300\ubb38, 4\uc18c\ubb38 \ub4f1 \ub3c4\uc131\uc5d0 \ub4e4\uc5b4\uc624\ub294 \uc131\ubb38 \ub4f1\uc5d0\uc11c \ubc31\uc131\ub4e4\uc758 \uba38\ub9ac\ub97c \uae4e\uc774\ub3c4\ub85d \uc9c0\uc2dc\ud558\uc600\uace0, \uc720\uae38\uc900 \uc790\uc2e0\ub3c4 \ubc31\uc131\ub4e4\uc758 \ub2e8\ubc1c \uacfc\uc815\uc744 \uc9c0\ub3c4, \uac10\ub3c5\ud558\uc600\ub2e4. \uc774\uc5b4 \uadf8\ub294 \ub2e8\ubc1c\uc5d0 \uc800\ud56d\ud558\ub294 \uc790\ub294 \uac15\uc81c\ub85c \uba38\ub9ac\ub97c \uae4e\ub3c4\ub85d \uc9c0\uc2dc\ud55c\ub2e4. \uadf8\uc640 \ub3d9\uc2dc\uc5d0 \uadf8\ub0a0 \ubc24\ubd80\ud130 12\uc6d4 31\uc77c(\uc74c\ub825 11\uc6d4 16\uc77c) \uc544\uce68\uc5d0 \uac78\uccd0 \uc815\ubd80 \uac01\ubd80\uc758 \uad00\ub8cc\uc640 \uc774\uc18d, \uadf8\ub9ac\uace0 \uad70\uc778, \uc21c\uac80 \ub4f1\uc758 \uad00\uc778\ub4e4\uc5d0\uac8c \uc6b0\uc120\uc801\uc73c\ub85c \ub2e8\ubc1c\uc744 \ub2e8\ud589\ud558\uc600\ub2e4. \ub2e8\ubc1c\uc758 \uba85\uc744 \ubc1b\uc740 \uad00\ub9ac\ub4e4 \uc911\uc5d0\uc11c\ub3c4 \ucc28\ub9c8 \uba38\ub9ac\ub97c \uc790\ub974\uc9c0 \ubabb\ud558\ub294 \uc790\uac00 \ub9ce\uc558\ub2e4. \uc720\uae38\uc900\uc740 \uc9c1\uc811 \uac00\uc704\uc640 \uce7c\uc744 \ub3d9\uc6d0\ud574, \uad00\ub8cc\ub4e4\uc758 \ud314\ub2e4\ub9ac\ub97c \uc7a1\uac8c \ud55c \ub4a4 \uba38\ub9ac\uce74\ub77d\uc744 \uc190\uc218 \uc798\ub790\ub2e4. 12\uc6d4 29\uc77c(\uc74c\ub825 11\uc6d4 14\uc77c) \uc720\uae38\uc900\uc740 \uc2dc\uc704\ub300\uc640 \uc77c\ubcf8\uad70\uc744 \ub300\uad90 \uc8fc\ubcc0\uc5d0 \uc0c1\uc8fc\uc2dc\ud0a4\uace0 \ub300\ud3ec\ub97c \uc124\uce58\ud558\uc5ec, \ub2e8\ubc1c\ub839 \ubc18\ubc1c\uc5d0 \ub300\ud55c \ub9cc\uc77c\uc758 \uc0ac\ud0dc\uc5d0 \ub300\ube44\ud558\uc600\ub2e4.", "answer": "\ub2e8\ubc1c\ub839", "sentence": "\uc77c\uc81c\ud788 \ub2e8\ubc1c\ub839 \uc744 \ud3ec\uace0\ud558\uc600\ub2e4.", "paragraph_sentence": "12\uc6d4 30\uc77c(\uc74c\ub825 11\uc6d4 15\uc77c) \uc624\ud6c4 \uc720\uae38\uc900\uc740 \ub0b4\ubd80\uace0\uc2dc(\u5167\u90e8\u544a\u793a)\ub97c \ud1b5\ud574 \ub2f9\uc77c\ub85c \uc804\uad6d \ubc29\ubc29\uace1\uace1\uc5d0 \uc77c\uc81c\ud788 \ub2e8\ubc1c\ub839 \uc744 \ud3ec\uace0\ud558\uc600\ub2e4. \uc720\uae38\uc900\uc740 \ubc14\ub85c \uad00\ub9ac\ub4e4\ub85c \ud558\uc5ec\uae08 \uac00\uc704\ub97c \ub4e4\uace0 \ud55c\uc131\ubd80\uc758 \uac70\ub9ac\ub098 4\ub300\ubb38, 4\uc18c\ubb38 \ub4f1 \ub3c4\uc131\uc5d0 \ub4e4\uc5b4\uc624\ub294 \uc131\ubb38 \ub4f1\uc5d0\uc11c \ubc31\uc131\ub4e4\uc758 \uba38\ub9ac\ub97c \uae4e\uc774\ub3c4\ub85d \uc9c0\uc2dc\ud558\uc600\uace0, \uc720\uae38\uc900 \uc790\uc2e0\ub3c4 \ubc31\uc131\ub4e4\uc758 \ub2e8\ubc1c \uacfc\uc815\uc744 \uc9c0\ub3c4, \uac10\ub3c5\ud558\uc600\ub2e4. \uc774\uc5b4 \uadf8\ub294 \ub2e8\ubc1c\uc5d0 \uc800\ud56d\ud558\ub294 \uc790\ub294 \uac15\uc81c\ub85c \uba38\ub9ac\ub97c \uae4e\ub3c4\ub85d \uc9c0\uc2dc\ud55c\ub2e4. \uadf8\uc640 \ub3d9\uc2dc\uc5d0 \uadf8\ub0a0 \ubc24\ubd80\ud130 12\uc6d4 31\uc77c(\uc74c\ub825 11\uc6d4 16\uc77c) \uc544\uce68\uc5d0 \uac78\uccd0 \uc815\ubd80 \uac01\ubd80\uc758 \uad00\ub8cc\uc640 \uc774\uc18d, \uadf8\ub9ac\uace0 \uad70\uc778, \uc21c\uac80 \ub4f1\uc758 \uad00\uc778\ub4e4\uc5d0\uac8c \uc6b0\uc120\uc801\uc73c\ub85c \ub2e8\ubc1c\uc744 \ub2e8\ud589\ud558\uc600\ub2e4. \ub2e8\ubc1c\uc758 \uba85\uc744 \ubc1b\uc740 \uad00\ub9ac\ub4e4 \uc911\uc5d0\uc11c\ub3c4 \ucc28\ub9c8 \uba38\ub9ac\ub97c \uc790\ub974\uc9c0 \ubabb\ud558\ub294 \uc790\uac00 \ub9ce\uc558\ub2e4. \uc720\uae38\uc900\uc740 \uc9c1\uc811 \uac00\uc704\uc640 \uce7c\uc744 \ub3d9\uc6d0\ud574, \uad00\ub8cc\ub4e4\uc758 \ud314\ub2e4\ub9ac\ub97c \uc7a1\uac8c \ud55c \ub4a4 \uba38\ub9ac\uce74\ub77d\uc744 \uc190\uc218 \uc798\ub790\ub2e4. 12\uc6d4 29\uc77c(\uc74c\ub825 11\uc6d4 14\uc77c) \uc720\uae38\uc900\uc740 \uc2dc\uc704\ub300\uc640 \uc77c\ubcf8\uad70\uc744 \ub300\uad90 \uc8fc\ubcc0\uc5d0 \uc0c1\uc8fc\uc2dc\ud0a4\uace0 \ub300\ud3ec\ub97c \uc124\uce58\ud558\uc5ec, \ub2e8\ubc1c\ub839 \ubc18\ubc1c\uc5d0 \ub300\ud55c \ub9cc\uc77c\uc758 \uc0ac\ud0dc\uc5d0 \ub300\ube44\ud558\uc600\ub2e4.", "paragraph_answer": "12\uc6d4 30\uc77c(\uc74c\ub825 11\uc6d4 15\uc77c) \uc624\ud6c4 \uc720\uae38\uc900\uc740 \ub0b4\ubd80\uace0\uc2dc(\u5167\u90e8\u544a\u793a)\ub97c \ud1b5\ud574 \ub2f9\uc77c\ub85c \uc804\uad6d \ubc29\ubc29\uace1\uace1\uc5d0 \uc77c\uc81c\ud788 \ub2e8\ubc1c\ub839 \uc744 \ud3ec\uace0\ud558\uc600\ub2e4. \uc720\uae38\uc900\uc740 \ubc14\ub85c \uad00\ub9ac\ub4e4\ub85c \ud558\uc5ec\uae08 \uac00\uc704\ub97c \ub4e4\uace0 \ud55c\uc131\ubd80\uc758 \uac70\ub9ac\ub098 4\ub300\ubb38, 4\uc18c\ubb38 \ub4f1 \ub3c4\uc131\uc5d0 \ub4e4\uc5b4\uc624\ub294 \uc131\ubb38 \ub4f1\uc5d0\uc11c \ubc31\uc131\ub4e4\uc758 \uba38\ub9ac\ub97c \uae4e\uc774\ub3c4\ub85d \uc9c0\uc2dc\ud558\uc600\uace0, \uc720\uae38\uc900 \uc790\uc2e0\ub3c4 \ubc31\uc131\ub4e4\uc758 \ub2e8\ubc1c \uacfc\uc815\uc744 \uc9c0\ub3c4, \uac10\ub3c5\ud558\uc600\ub2e4. \uc774\uc5b4 \uadf8\ub294 \ub2e8\ubc1c\uc5d0 \uc800\ud56d\ud558\ub294 \uc790\ub294 \uac15\uc81c\ub85c \uba38\ub9ac\ub97c \uae4e\ub3c4\ub85d \uc9c0\uc2dc\ud55c\ub2e4. \uadf8\uc640 \ub3d9\uc2dc\uc5d0 \uadf8\ub0a0 \ubc24\ubd80\ud130 12\uc6d4 31\uc77c(\uc74c\ub825 11\uc6d4 16\uc77c) \uc544\uce68\uc5d0 \uac78\uccd0 \uc815\ubd80 \uac01\ubd80\uc758 \uad00\ub8cc\uc640 \uc774\uc18d, \uadf8\ub9ac\uace0 \uad70\uc778, \uc21c\uac80 \ub4f1\uc758 \uad00\uc778\ub4e4\uc5d0\uac8c \uc6b0\uc120\uc801\uc73c\ub85c \ub2e8\ubc1c\uc744 \ub2e8\ud589\ud558\uc600\ub2e4. \ub2e8\ubc1c\uc758 \uba85\uc744 \ubc1b\uc740 \uad00\ub9ac\ub4e4 \uc911\uc5d0\uc11c\ub3c4 \ucc28\ub9c8 \uba38\ub9ac\ub97c \uc790\ub974\uc9c0 \ubabb\ud558\ub294 \uc790\uac00 \ub9ce\uc558\ub2e4. \uc720\uae38\uc900\uc740 \uc9c1\uc811 \uac00\uc704\uc640 \uce7c\uc744 \ub3d9\uc6d0\ud574, \uad00\ub8cc\ub4e4\uc758 \ud314\ub2e4\ub9ac\ub97c \uc7a1\uac8c \ud55c \ub4a4 \uba38\ub9ac\uce74\ub77d\uc744 \uc190\uc218 \uc798\ub790\ub2e4. 12\uc6d4 29\uc77c(\uc74c\ub825 11\uc6d4 14\uc77c) \uc720\uae38\uc900\uc740 \uc2dc\uc704\ub300\uc640 \uc77c\ubcf8\uad70\uc744 \ub300\uad90 \uc8fc\ubcc0\uc5d0 \uc0c1\uc8fc\uc2dc\ud0a4\uace0 \ub300\ud3ec\ub97c \uc124\uce58\ud558\uc5ec, \ub2e8\ubc1c\ub839 \ubc18\ubc1c\uc5d0 \ub300\ud55c \ub9cc\uc77c\uc758 \uc0ac\ud0dc\uc5d0 \ub300\ube44\ud558\uc600\ub2e4.", "sentence_answer": "\uc77c\uc81c\ud788 \ub2e8\ubc1c\ub839 \uc744 \ud3ec\uace0\ud558\uc600\ub2e4.", "paragraph_id": "6542752-18-1", "paragraph_question": "question: \uc720\uae38\uc900\uc774 \uad00\ub9ac\ub4e4\uc744 \ud1b5\ud574 \uac00\uc704\ub97c \ub4e4\uace0 \ubc31\uc131\ub4e4\uc758 \uba38\ub9ac\ub97c \uae4d\uc774\ub3c4\ub85d \uc9c0\uc2dc\ud558\uace0 \ub0b4\ubd80\uace0\uc2dc\ub97c \ud1b5\ud574 12\uc6d4 30\uc77c \ud3ec\uace0\ud55c\uac83\uc740 \ubb34\uc5c7\uc778\uac00?, context: 12\uc6d4 30\uc77c(\uc74c\ub825 11\uc6d4 15\uc77c) \uc624\ud6c4 \uc720\uae38\uc900\uc740 \ub0b4\ubd80\uace0\uc2dc(\u5167\u90e8\u544a\u793a)\ub97c \ud1b5\ud574 \ub2f9\uc77c\ub85c \uc804\uad6d \ubc29\ubc29\uace1\uace1\uc5d0 \uc77c\uc81c\ud788 \ub2e8\ubc1c\ub839\uc744 \ud3ec\uace0\ud558\uc600\ub2e4. \uc720\uae38\uc900\uc740 \ubc14\ub85c \uad00\ub9ac\ub4e4\ub85c \ud558\uc5ec\uae08 \uac00\uc704\ub97c \ub4e4\uace0 \ud55c\uc131\ubd80\uc758 \uac70\ub9ac\ub098 4\ub300\ubb38, 4\uc18c\ubb38 \ub4f1 \ub3c4\uc131\uc5d0 \ub4e4\uc5b4\uc624\ub294 \uc131\ubb38 \ub4f1\uc5d0\uc11c \ubc31\uc131\ub4e4\uc758 \uba38\ub9ac\ub97c \uae4e\uc774\ub3c4\ub85d \uc9c0\uc2dc\ud558\uc600\uace0, \uc720\uae38\uc900 \uc790\uc2e0\ub3c4 \ubc31\uc131\ub4e4\uc758 \ub2e8\ubc1c \uacfc\uc815\uc744 \uc9c0\ub3c4, \uac10\ub3c5\ud558\uc600\ub2e4. \uc774\uc5b4 \uadf8\ub294 \ub2e8\ubc1c\uc5d0 \uc800\ud56d\ud558\ub294 \uc790\ub294 \uac15\uc81c\ub85c \uba38\ub9ac\ub97c \uae4e\ub3c4\ub85d \uc9c0\uc2dc\ud55c\ub2e4. \uadf8\uc640 \ub3d9\uc2dc\uc5d0 \uadf8\ub0a0 \ubc24\ubd80\ud130 12\uc6d4 31\uc77c(\uc74c\ub825 11\uc6d4 16\uc77c) \uc544\uce68\uc5d0 \uac78\uccd0 \uc815\ubd80 \uac01\ubd80\uc758 \uad00\ub8cc\uc640 \uc774\uc18d, \uadf8\ub9ac\uace0 \uad70\uc778, \uc21c\uac80 \ub4f1\uc758 \uad00\uc778\ub4e4\uc5d0\uac8c \uc6b0\uc120\uc801\uc73c\ub85c \ub2e8\ubc1c\uc744 \ub2e8\ud589\ud558\uc600\ub2e4. \ub2e8\ubc1c\uc758 \uba85\uc744 \ubc1b\uc740 \uad00\ub9ac\ub4e4 \uc911\uc5d0\uc11c\ub3c4 \ucc28\ub9c8 \uba38\ub9ac\ub97c \uc790\ub974\uc9c0 \ubabb\ud558\ub294 \uc790\uac00 \ub9ce\uc558\ub2e4. \uc720\uae38\uc900\uc740 \uc9c1\uc811 \uac00\uc704\uc640 \uce7c\uc744 \ub3d9\uc6d0\ud574, \uad00\ub8cc\ub4e4\uc758 \ud314\ub2e4\ub9ac\ub97c \uc7a1\uac8c \ud55c \ub4a4 \uba38\ub9ac\uce74\ub77d\uc744 \uc190\uc218 \uc798\ub790\ub2e4. 12\uc6d4 29\uc77c(\uc74c\ub825 11\uc6d4 14\uc77c) \uc720\uae38\uc900\uc740 \uc2dc\uc704\ub300\uc640 \uc77c\ubcf8\uad70\uc744 \ub300\uad90 \uc8fc\ubcc0\uc5d0 \uc0c1\uc8fc\uc2dc\ud0a4\uace0 \ub300\ud3ec\ub97c \uc124\uce58\ud558\uc5ec, \ub2e8\ubc1c\ub839 \ubc18\ubc1c\uc5d0 \ub300\ud55c \ub9cc\uc77c\uc758 \uc0ac\ud0dc\uc5d0 \ub300\ube44\ud558\uc600\ub2e4."} {"question": "\ud5a5\uc2dc\uc5d0 \ud569\uaca9\uc5d0 \uac70\uc778\uc774 \ub418\uba74 \ubb34\uc5c7\uc5d0 \uc751\uc2dc\ud560 \uc790\uaca9\uc774 \uc8fc\uc5b4\uc9c0\ub098?", "paragraph": "\uba85\ub300\uc758 \uacfc\uac70\uc81c\ub3c4\ub294 \uc1a1\u00b7\uc6d0(\u5b8b\u5143)\uc758 \uc81c\ub3c4\ub97c \uacc4\uc2b9\ud558\uc5ec \ud5a5\uc2dc(\u9115\u8a66)\u00b7\ud68c\uc2dc(\u6703\u8a66)\u00b7\uc804\uc2dc(\u6bbf\u8a66)\uc758 3\uc885\uc774 \uc788\ub2e4. \ud5a5\uc2dc\ub294 \uac01\uc131(\u5404\u7701)\uc5d0\uc11c, \ud68c\uc2dc\ub294 \uc608\ubd80(\u79ae\u90e8)\uc5d0\uc11c \uc804\uc2dc\ub294 \uad81\uc804(\u5bae\u6bbf)\uc5d0\uc11c \uc2dc\ud589\ud558\uc600\ub2e4. \uac01 \ubd80\u00b7\uc8fc\u00b7\ud604\uc758 \ud559\uc0dd\uc740 \uc81c\uac70\ud559\uad00(\u63d0\u64e7\u5b78\u5b98)\uc774 \uc8fc\uad00\ud558\ub294 \uc138\uace0(\u6b72\u8003)\uc5d0 1\uae09\uc73c\ub85c \ud569\uaca9\ud568\uc73c\ub85c\uc368 \ud5a5\uc2dc\uc5d0 \uc751\uc2dc\ud560 \uc790\uaca9\uc744 \uc5bb\ub294\ub2e4. \ud5a5\uc2dc\uc5d0 \ud569\uaca9\ud558\uba74 \uac70\uc778(\u64e7\u4eba)\uc774 \ub418\uace0 \ud68c\uc2dc(\u6703\u8a66)\uc5d0 \uc751\uc2dc\ud560 \uc790\uaca9\uc744 \uc5bb\ub294\ub2e4. \ub610 \ud68c\uc2dc\uc5d0 \ud569\uaca9\ud558\uba74 \ucc9c\uc790(\u5929\u5b50)\uac00 \uc784\uc11d\ud558\uc5ec \uc2dc\ud589\ud558\ub294 \uc804\uc2dc\uc5d0 \uc751\uc2dc\ud55c\ub2e4. \uace0\uc2dc\ub0b4\uc6a9\uc740 \uacbd\uc758(\u7d93\u7fa9)\u00b7\uc18c\uace0\uc728\ub839(\u8a34\u8aa5\u5f8b\u4ee4)\u00b7\uacbd\uc0ac\uc2dc\ubb34\ucc45(\u7d93\u53f2\u6642\u52d9\u7b56) \ub4f1 3\uc885\uc73c\ub85c, \uacbd\uc758\ub294 4\uc11c(\u56db\u66f8) \ubc0f \uc5ed\u00b7\uc11c\u00b7\uc2dc\u00b7\uc608\u00b7\ucd98\ucd94(\u6613\u00b7\u66f8\u00b7\u8a69\u00b7\u79ae\u00b7\u6625\u79cb)\uc758 5\uacbd\uc5d0\uc11c \ucd9c\uc81c\ud55c\ub2e4. \ud5a5\uc2dc\uc640 \ud68c\uc2dc\ub294 \uac01\uac01 3\ucc28\uc758 \uc2dc\ud5d8\uc744 \uac70\uce5c\ub2e4. 1\ucc28\ub294 4\uc11c\uc758(\u56db\u66f8\u7fa9) 3\ubb38\uc81c\u00b7\uacbd\uc758 4\ubb38\uc81c\ub97c \uc791\uc131\ud558\ub294 \uac83\uc774\ub2e4. 2\ucc28\ub294 \ub17c\ubb38 \ud55c \ud3b8\uc744 300\uc790 \uc774\uc0c1 \uc791\uc131\ud558\ub294 \uac83\uacfc \uc728\ub839\uc5d0 \uad00\ud55c \uac83 \ud55c \ud3b8, \uc81c\ub3c4\uc5d0 \uad00\ud55c \uac83 5\uc870\ubaa9\uc744 \uc4f0\ub294 \uac83\uc774\ub2e4. 3\ucc28\ub294 \uacbd\uc0ac\uc2dc\ubb34\ucc45(\u7d93\u53f2\u6642\u52d9\u7b56) 5\ubb38\uc81c\ub97c \uc791\uc131\ud558\ub294 \uac83\uc774\ub2e4. \uadf8\uc911 1\ucc28 \uc2dc\ud5d8\uc774 \uac00\uc7a5 \uc911\uc2dc\ub418\uc5c8\uace0, \ub2f5\uc548 \uc791\uc131\uc740 \uc61b \uc0ac\ub78c\uc758 \ubb38\uccb4\ub97c \ubaa8\ubc29\ud558\uc5ec \uaca9\ub960(\u683c\u5f8b)\uc774 \uc5c4\uaca9\ud558\uac8c \uc815\ud574\uc9c4 8\uace0\ubb38(\u516b\u80a1\u6587)\uc73c\ub85c \ud558\uc5ec\uc57c \ud588\ub2e4. 8\uace0\ubb38\uc740 \uaca9\uc2dd\uc744 \uc815\ud558\uc5ec, \uc124\uba85\uc5d0 \ud3b8\ub9ac\ud558\uace0 \uc6b4\uc728\uc5d0 \ub9de\ucd94\uc5b4 \ub4e3\uae30 \uc88b\ub3c4\ub85d \ud558\uae30 \uc704\ud574 \ub9cc\ub4e4\uc5b4\uc9c4 \uc77c\uc885\uc758 \uc218\uc0ac\ubb38(\u4fee\u8a5e\u6587)\uc774\uc9c0\ub9cc, \uc9c0\ub098\uce58\uac8c \uaddc\uaca9\ud654\ud568\uc73c\ub85c\uc368 \ub9ce\uc740 \ud3d0\ub2e8\uc744 \ub0b3\uc558\ub2e4. 8\uace0\ubb38\uc774 \uba85\ub098\ub77c\uc758 \uacfc\uac70\uc2dc\ud5d8\uc5d0 \uc801\uc6a9\ub428\uc73c\ub85c\uc368 \uad50\uc721\uacfc \ud559\uc220\ubd84\uc57c\uc5d0 \ub07c\uce5c \ud574\ub3c5\uc740 \ub9e4\uc6b0 \ucef8\ub2e4. \uadf8\ub798\uc11c \uacbd\uc11c(\u7d93\u66f8)\uc758 \ub73b\uc744 \ud640\uc2dc(\u5ffd\u8996)\ud558\uace0 \ubb38\uc7a5\uc758 \uc218\uc2dd\uc5d0\ub9cc \uc804\ub150\ud558\ub294 \ub098\uc05c \ud559\ud48d\uc744 \uc870\uc131\ud558\uc600\ub2e4.", "answer": "\ud68c\uc2dc", "sentence": "\uba85\ub300\uc758 \uacfc\uac70\uc81c\ub3c4\ub294 \uc1a1\u00b7\uc6d0(\u5b8b\u5143)\uc758 \uc81c\ub3c4\ub97c \uacc4\uc2b9\ud558\uc5ec \ud5a5\uc2dc(\u9115\u8a66)\u00b7 \ud68c\uc2dc (\u6703\u8a66)", "paragraph_sentence": " \uba85\ub300\uc758 \uacfc\uac70\uc81c\ub3c4\ub294 \uc1a1\u00b7\uc6d0(\u5b8b\u5143)\uc758 \uc81c\ub3c4\ub97c \uacc4\uc2b9\ud558\uc5ec \ud5a5\uc2dc(\u9115\u8a66)\u00b7 \ud68c\uc2dc (\u6703\u8a66) \u00b7\uc804\uc2dc(\u6bbf\u8a66)\uc758 3\uc885\uc774 \uc788\ub2e4. \ud5a5\uc2dc\ub294 \uac01\uc131(\u5404\u7701)\uc5d0\uc11c, \ud68c\uc2dc\ub294 \uc608\ubd80(\u79ae\u90e8)\uc5d0\uc11c \uc804\uc2dc\ub294 \uad81\uc804(\u5bae\u6bbf)\uc5d0\uc11c \uc2dc\ud589\ud558\uc600\ub2e4. \uac01 \ubd80\u00b7\uc8fc\u00b7\ud604\uc758 \ud559\uc0dd\uc740 \uc81c\uac70\ud559\uad00(\u63d0\u64e7\u5b78\u5b98)\uc774 \uc8fc\uad00\ud558\ub294 \uc138\uace0(\u6b72\u8003)\uc5d0 1\uae09\uc73c\ub85c \ud569\uaca9\ud568\uc73c\ub85c\uc368 \ud5a5\uc2dc\uc5d0 \uc751\uc2dc\ud560 \uc790\uaca9\uc744 \uc5bb\ub294\ub2e4. \ud5a5\uc2dc\uc5d0 \ud569\uaca9\ud558\uba74 \uac70\uc778(\u64e7\u4eba)\uc774 \ub418\uace0 \ud68c\uc2dc(\u6703\u8a66)\uc5d0 \uc751\uc2dc\ud560 \uc790\uaca9\uc744 \uc5bb\ub294\ub2e4. \ub610 \ud68c\uc2dc\uc5d0 \ud569\uaca9\ud558\uba74 \ucc9c\uc790(\u5929\u5b50)\uac00 \uc784\uc11d\ud558\uc5ec \uc2dc\ud589\ud558\ub294 \uc804\uc2dc\uc5d0 \uc751\uc2dc\ud55c\ub2e4. \uace0\uc2dc\ub0b4\uc6a9\uc740 \uacbd\uc758(\u7d93\u7fa9)\u00b7\uc18c\uace0\uc728\ub839(\u8a34\u8aa5\u5f8b\u4ee4)\u00b7\uacbd\uc0ac\uc2dc\ubb34\ucc45(\u7d93\u53f2\u6642\u52d9\u7b56) \ub4f1 3\uc885\uc73c\ub85c, \uacbd\uc758\ub294 4\uc11c(\u56db\u66f8) \ubc0f \uc5ed\u00b7\uc11c\u00b7\uc2dc\u00b7\uc608\u00b7\ucd98\ucd94(\u6613\u00b7\u66f8\u00b7\u8a69\u00b7\u79ae\u00b7\u6625\u79cb)\uc758 5\uacbd\uc5d0\uc11c \ucd9c\uc81c\ud55c\ub2e4. \ud5a5\uc2dc\uc640 \ud68c\uc2dc\ub294 \uac01\uac01 3\ucc28\uc758 \uc2dc\ud5d8\uc744 \uac70\uce5c\ub2e4. 1\ucc28\ub294 4\uc11c\uc758(\u56db\u66f8\u7fa9) 3\ubb38\uc81c\u00b7\uacbd\uc758 4\ubb38\uc81c\ub97c \uc791\uc131\ud558\ub294 \uac83\uc774\ub2e4. 2\ucc28\ub294 \ub17c\ubb38 \ud55c \ud3b8\uc744 300\uc790 \uc774\uc0c1 \uc791\uc131\ud558\ub294 \uac83\uacfc \uc728\ub839\uc5d0 \uad00\ud55c \uac83 \ud55c \ud3b8, \uc81c\ub3c4\uc5d0 \uad00\ud55c \uac83 5\uc870\ubaa9\uc744 \uc4f0\ub294 \uac83\uc774\ub2e4. 3\ucc28\ub294 \uacbd\uc0ac\uc2dc\ubb34\ucc45(\u7d93\u53f2\u6642\u52d9\u7b56) 5\ubb38\uc81c\ub97c \uc791\uc131\ud558\ub294 \uac83\uc774\ub2e4. \uadf8\uc911 1\ucc28 \uc2dc\ud5d8\uc774 \uac00\uc7a5 \uc911\uc2dc\ub418\uc5c8\uace0, \ub2f5\uc548 \uc791\uc131\uc740 \uc61b \uc0ac\ub78c\uc758 \ubb38\uccb4\ub97c \ubaa8\ubc29\ud558\uc5ec \uaca9\ub960(\u683c\u5f8b)\uc774 \uc5c4\uaca9\ud558\uac8c \uc815\ud574\uc9c4 8\uace0\ubb38(\u516b\u80a1\u6587)\uc73c\ub85c \ud558\uc5ec\uc57c \ud588\ub2e4. 8\uace0\ubb38\uc740 \uaca9\uc2dd\uc744 \uc815\ud558\uc5ec, \uc124\uba85\uc5d0 \ud3b8\ub9ac\ud558\uace0 \uc6b4\uc728\uc5d0 \ub9de\ucd94\uc5b4 \ub4e3\uae30 \uc88b\ub3c4\ub85d \ud558\uae30 \uc704\ud574 \ub9cc\ub4e4\uc5b4\uc9c4 \uc77c\uc885\uc758 \uc218\uc0ac\ubb38(\u4fee\u8a5e\u6587)\uc774\uc9c0\ub9cc, \uc9c0\ub098\uce58\uac8c \uaddc\uaca9\ud654\ud568\uc73c\ub85c\uc368 \ub9ce\uc740 \ud3d0\ub2e8\uc744 \ub0b3\uc558\ub2e4. 8\uace0\ubb38\uc774 \uba85\ub098\ub77c\uc758 \uacfc\uac70\uc2dc\ud5d8\uc5d0 \uc801\uc6a9\ub428\uc73c\ub85c\uc368 \uad50\uc721\uacfc \ud559\uc220\ubd84\uc57c\uc5d0 \ub07c\uce5c \ud574\ub3c5\uc740 \ub9e4\uc6b0 \ucef8\ub2e4. \uadf8\ub798\uc11c \uacbd\uc11c(\u7d93\u66f8)\uc758 \ub73b\uc744 \ud640\uc2dc(\u5ffd\u8996)\ud558\uace0 \ubb38\uc7a5\uc758 \uc218\uc2dd\uc5d0\ub9cc \uc804\ub150\ud558\ub294 \ub098\uc05c \ud559\ud48d\uc744 \uc870\uc131\ud558\uc600\ub2e4.", "paragraph_answer": "\uba85\ub300\uc758 \uacfc\uac70\uc81c\ub3c4\ub294 \uc1a1\u00b7\uc6d0(\u5b8b\u5143)\uc758 \uc81c\ub3c4\ub97c \uacc4\uc2b9\ud558\uc5ec \ud5a5\uc2dc(\u9115\u8a66)\u00b7 \ud68c\uc2dc (\u6703\u8a66)\u00b7\uc804\uc2dc(\u6bbf\u8a66)\uc758 3\uc885\uc774 \uc788\ub2e4. \ud5a5\uc2dc\ub294 \uac01\uc131(\u5404\u7701)\uc5d0\uc11c, \ud68c\uc2dc\ub294 \uc608\ubd80(\u79ae\u90e8)\uc5d0\uc11c \uc804\uc2dc\ub294 \uad81\uc804(\u5bae\u6bbf)\uc5d0\uc11c \uc2dc\ud589\ud558\uc600\ub2e4. \uac01 \ubd80\u00b7\uc8fc\u00b7\ud604\uc758 \ud559\uc0dd\uc740 \uc81c\uac70\ud559\uad00(\u63d0\u64e7\u5b78\u5b98)\uc774 \uc8fc\uad00\ud558\ub294 \uc138\uace0(\u6b72\u8003)\uc5d0 1\uae09\uc73c\ub85c \ud569\uaca9\ud568\uc73c\ub85c\uc368 \ud5a5\uc2dc\uc5d0 \uc751\uc2dc\ud560 \uc790\uaca9\uc744 \uc5bb\ub294\ub2e4. \ud5a5\uc2dc\uc5d0 \ud569\uaca9\ud558\uba74 \uac70\uc778(\u64e7\u4eba)\uc774 \ub418\uace0 \ud68c\uc2dc(\u6703\u8a66)\uc5d0 \uc751\uc2dc\ud560 \uc790\uaca9\uc744 \uc5bb\ub294\ub2e4. \ub610 \ud68c\uc2dc\uc5d0 \ud569\uaca9\ud558\uba74 \ucc9c\uc790(\u5929\u5b50)\uac00 \uc784\uc11d\ud558\uc5ec \uc2dc\ud589\ud558\ub294 \uc804\uc2dc\uc5d0 \uc751\uc2dc\ud55c\ub2e4. \uace0\uc2dc\ub0b4\uc6a9\uc740 \uacbd\uc758(\u7d93\u7fa9)\u00b7\uc18c\uace0\uc728\ub839(\u8a34\u8aa5\u5f8b\u4ee4)\u00b7\uacbd\uc0ac\uc2dc\ubb34\ucc45(\u7d93\u53f2\u6642\u52d9\u7b56) \ub4f1 3\uc885\uc73c\ub85c, \uacbd\uc758\ub294 4\uc11c(\u56db\u66f8) \ubc0f \uc5ed\u00b7\uc11c\u00b7\uc2dc\u00b7\uc608\u00b7\ucd98\ucd94(\u6613\u00b7\u66f8\u00b7\u8a69\u00b7\u79ae\u00b7\u6625\u79cb)\uc758 5\uacbd\uc5d0\uc11c \ucd9c\uc81c\ud55c\ub2e4. \ud5a5\uc2dc\uc640 \ud68c\uc2dc\ub294 \uac01\uac01 3\ucc28\uc758 \uc2dc\ud5d8\uc744 \uac70\uce5c\ub2e4. 1\ucc28\ub294 4\uc11c\uc758(\u56db\u66f8\u7fa9) 3\ubb38\uc81c\u00b7\uacbd\uc758 4\ubb38\uc81c\ub97c \uc791\uc131\ud558\ub294 \uac83\uc774\ub2e4. 2\ucc28\ub294 \ub17c\ubb38 \ud55c \ud3b8\uc744 300\uc790 \uc774\uc0c1 \uc791\uc131\ud558\ub294 \uac83\uacfc \uc728\ub839\uc5d0 \uad00\ud55c \uac83 \ud55c \ud3b8, \uc81c\ub3c4\uc5d0 \uad00\ud55c \uac83 5\uc870\ubaa9\uc744 \uc4f0\ub294 \uac83\uc774\ub2e4. 3\ucc28\ub294 \uacbd\uc0ac\uc2dc\ubb34\ucc45(\u7d93\u53f2\u6642\u52d9\u7b56) 5\ubb38\uc81c\ub97c \uc791\uc131\ud558\ub294 \uac83\uc774\ub2e4. \uadf8\uc911 1\ucc28 \uc2dc\ud5d8\uc774 \uac00\uc7a5 \uc911\uc2dc\ub418\uc5c8\uace0, \ub2f5\uc548 \uc791\uc131\uc740 \uc61b \uc0ac\ub78c\uc758 \ubb38\uccb4\ub97c \ubaa8\ubc29\ud558\uc5ec \uaca9\ub960(\u683c\u5f8b)\uc774 \uc5c4\uaca9\ud558\uac8c \uc815\ud574\uc9c4 8\uace0\ubb38(\u516b\u80a1\u6587)\uc73c\ub85c \ud558\uc5ec\uc57c \ud588\ub2e4. 8\uace0\ubb38\uc740 \uaca9\uc2dd\uc744 \uc815\ud558\uc5ec, \uc124\uba85\uc5d0 \ud3b8\ub9ac\ud558\uace0 \uc6b4\uc728\uc5d0 \ub9de\ucd94\uc5b4 \ub4e3\uae30 \uc88b\ub3c4\ub85d \ud558\uae30 \uc704\ud574 \ub9cc\ub4e4\uc5b4\uc9c4 \uc77c\uc885\uc758 \uc218\uc0ac\ubb38(\u4fee\u8a5e\u6587)\uc774\uc9c0\ub9cc, \uc9c0\ub098\uce58\uac8c \uaddc\uaca9\ud654\ud568\uc73c\ub85c\uc368 \ub9ce\uc740 \ud3d0\ub2e8\uc744 \ub0b3\uc558\ub2e4. 8\uace0\ubb38\uc774 \uba85\ub098\ub77c\uc758 \uacfc\uac70\uc2dc\ud5d8\uc5d0 \uc801\uc6a9\ub428\uc73c\ub85c\uc368 \uad50\uc721\uacfc \ud559\uc220\ubd84\uc57c\uc5d0 \ub07c\uce5c \ud574\ub3c5\uc740 \ub9e4\uc6b0 \ucef8\ub2e4. \uadf8\ub798\uc11c \uacbd\uc11c(\u7d93\u66f8)\uc758 \ub73b\uc744 \ud640\uc2dc(\u5ffd\u8996)\ud558\uace0 \ubb38\uc7a5\uc758 \uc218\uc2dd\uc5d0\ub9cc \uc804\ub150\ud558\ub294 \ub098\uc05c \ud559\ud48d\uc744 \uc870\uc131\ud558\uc600\ub2e4.", "sentence_answer": "\uba85\ub300\uc758 \uacfc\uac70\uc81c\ub3c4\ub294 \uc1a1\u00b7\uc6d0(\u5b8b\u5143)\uc758 \uc81c\ub3c4\ub97c \uacc4\uc2b9\ud558\uc5ec \ud5a5\uc2dc(\u9115\u8a66)\u00b7 \ud68c\uc2dc (\u6703\u8a66)", "paragraph_id": "6503076-2-2", "paragraph_question": "question: \ud5a5\uc2dc\uc5d0 \ud569\uaca9\uc5d0 \uac70\uc778\uc774 \ub418\uba74 \ubb34\uc5c7\uc5d0 \uc751\uc2dc\ud560 \uc790\uaca9\uc774 \uc8fc\uc5b4\uc9c0\ub098?, context: \uba85\ub300\uc758 \uacfc\uac70\uc81c\ub3c4\ub294 \uc1a1\u00b7\uc6d0(\u5b8b\u5143)\uc758 \uc81c\ub3c4\ub97c \uacc4\uc2b9\ud558\uc5ec \ud5a5\uc2dc(\u9115\u8a66)\u00b7\ud68c\uc2dc(\u6703\u8a66)\u00b7\uc804\uc2dc(\u6bbf\u8a66)\uc758 3\uc885\uc774 \uc788\ub2e4. \ud5a5\uc2dc\ub294 \uac01\uc131(\u5404\u7701)\uc5d0\uc11c, \ud68c\uc2dc\ub294 \uc608\ubd80(\u79ae\u90e8)\uc5d0\uc11c \uc804\uc2dc\ub294 \uad81\uc804(\u5bae\u6bbf)\uc5d0\uc11c \uc2dc\ud589\ud558\uc600\ub2e4. \uac01 \ubd80\u00b7\uc8fc\u00b7\ud604\uc758 \ud559\uc0dd\uc740 \uc81c\uac70\ud559\uad00(\u63d0\u64e7\u5b78\u5b98)\uc774 \uc8fc\uad00\ud558\ub294 \uc138\uace0(\u6b72\u8003)\uc5d0 1\uae09\uc73c\ub85c \ud569\uaca9\ud568\uc73c\ub85c\uc368 \ud5a5\uc2dc\uc5d0 \uc751\uc2dc\ud560 \uc790\uaca9\uc744 \uc5bb\ub294\ub2e4. \ud5a5\uc2dc\uc5d0 \ud569\uaca9\ud558\uba74 \uac70\uc778(\u64e7\u4eba)\uc774 \ub418\uace0 \ud68c\uc2dc(\u6703\u8a66)\uc5d0 \uc751\uc2dc\ud560 \uc790\uaca9\uc744 \uc5bb\ub294\ub2e4. \ub610 \ud68c\uc2dc\uc5d0 \ud569\uaca9\ud558\uba74 \ucc9c\uc790(\u5929\u5b50)\uac00 \uc784\uc11d\ud558\uc5ec \uc2dc\ud589\ud558\ub294 \uc804\uc2dc\uc5d0 \uc751\uc2dc\ud55c\ub2e4. \uace0\uc2dc\ub0b4\uc6a9\uc740 \uacbd\uc758(\u7d93\u7fa9)\u00b7\uc18c\uace0\uc728\ub839(\u8a34\u8aa5\u5f8b\u4ee4)\u00b7\uacbd\uc0ac\uc2dc\ubb34\ucc45(\u7d93\u53f2\u6642\u52d9\u7b56) \ub4f1 3\uc885\uc73c\ub85c, \uacbd\uc758\ub294 4\uc11c(\u56db\u66f8) \ubc0f \uc5ed\u00b7\uc11c\u00b7\uc2dc\u00b7\uc608\u00b7\ucd98\ucd94(\u6613\u00b7\u66f8\u00b7\u8a69\u00b7\u79ae\u00b7\u6625\u79cb)\uc758 5\uacbd\uc5d0\uc11c \ucd9c\uc81c\ud55c\ub2e4. \ud5a5\uc2dc\uc640 \ud68c\uc2dc\ub294 \uac01\uac01 3\ucc28\uc758 \uc2dc\ud5d8\uc744 \uac70\uce5c\ub2e4. 1\ucc28\ub294 4\uc11c\uc758(\u56db\u66f8\u7fa9) 3\ubb38\uc81c\u00b7\uacbd\uc758 4\ubb38\uc81c\ub97c \uc791\uc131\ud558\ub294 \uac83\uc774\ub2e4. 2\ucc28\ub294 \ub17c\ubb38 \ud55c \ud3b8\uc744 300\uc790 \uc774\uc0c1 \uc791\uc131\ud558\ub294 \uac83\uacfc \uc728\ub839\uc5d0 \uad00\ud55c \uac83 \ud55c \ud3b8, \uc81c\ub3c4\uc5d0 \uad00\ud55c \uac83 5\uc870\ubaa9\uc744 \uc4f0\ub294 \uac83\uc774\ub2e4. 3\ucc28\ub294 \uacbd\uc0ac\uc2dc\ubb34\ucc45(\u7d93\u53f2\u6642\u52d9\u7b56) 5\ubb38\uc81c\ub97c \uc791\uc131\ud558\ub294 \uac83\uc774\ub2e4. \uadf8\uc911 1\ucc28 \uc2dc\ud5d8\uc774 \uac00\uc7a5 \uc911\uc2dc\ub418\uc5c8\uace0, \ub2f5\uc548 \uc791\uc131\uc740 \uc61b \uc0ac\ub78c\uc758 \ubb38\uccb4\ub97c \ubaa8\ubc29\ud558\uc5ec \uaca9\ub960(\u683c\u5f8b)\uc774 \uc5c4\uaca9\ud558\uac8c \uc815\ud574\uc9c4 8\uace0\ubb38(\u516b\u80a1\u6587)\uc73c\ub85c \ud558\uc5ec\uc57c \ud588\ub2e4. 8\uace0\ubb38\uc740 \uaca9\uc2dd\uc744 \uc815\ud558\uc5ec, \uc124\uba85\uc5d0 \ud3b8\ub9ac\ud558\uace0 \uc6b4\uc728\uc5d0 \ub9de\ucd94\uc5b4 \ub4e3\uae30 \uc88b\ub3c4\ub85d \ud558\uae30 \uc704\ud574 \ub9cc\ub4e4\uc5b4\uc9c4 \uc77c\uc885\uc758 \uc218\uc0ac\ubb38(\u4fee\u8a5e\u6587)\uc774\uc9c0\ub9cc, \uc9c0\ub098\uce58\uac8c \uaddc\uaca9\ud654\ud568\uc73c\ub85c\uc368 \ub9ce\uc740 \ud3d0\ub2e8\uc744 \ub0b3\uc558\ub2e4. 8\uace0\ubb38\uc774 \uba85\ub098\ub77c\uc758 \uacfc\uac70\uc2dc\ud5d8\uc5d0 \uc801\uc6a9\ub428\uc73c\ub85c\uc368 \uad50\uc721\uacfc \ud559\uc220\ubd84\uc57c\uc5d0 \ub07c\uce5c \ud574\ub3c5\uc740 \ub9e4\uc6b0 \ucef8\ub2e4. \uadf8\ub798\uc11c \uacbd\uc11c(\u7d93\u66f8)\uc758 \ub73b\uc744 \ud640\uc2dc(\u5ffd\u8996)\ud558\uace0 \ubb38\uc7a5\uc758 \uc218\uc2dd\uc5d0\ub9cc \uc804\ub150\ud558\ub294 \ub098\uc05c \ud559\ud48d\uc744 \uc870\uc131\ud558\uc600\ub2e4."} {"question": "\uad8c\ub3c4\uc6d0\uc5d0 \uc758\ud574 \ucc3d\uc2dc\ub41c 8\uccb4\uc9c8\uc758 \ud310\ubcc4\uc740 \uc624\uc9c1 \ubb34\uc5c7\uc758 \ub9e5\uc9c4\uc744 \uae30\ubcf8\uc73c\ub85c \uc774\ub8e8\uc5b4\uc9c0\ub294\uac00?", "paragraph": "8\uccb4\uc9c8 \uc758\ud559(\ud314\uccb4\uc9c8\uc758\ud559, \u516b\u9ad4\u8cea\u91ab\u5b78, Eight-Constitution Medicine)\uc740 \uc0ac\ub78c\uc758 \uccb4\uc9c8\uc744 \uc624\uc7a5\uc721\ubd80\uc758 \uac15\uc57d\ubc30\uc5f4\uc5d0 \ub530\ub77c \ubaa9\uc591(\u967d\u967d), \ubaa9\uc74c(\u6728\u9670), \uc218\uc591(\u6c34\u967d), \uc218\uc74c(\u6c34\u9670), \ud1a0\uc591(\u571f\u967d), \ud1a0\uc74c(\u571f\u9670), \uae08\uc591(\u91d1\u967d), \uae08\uc74c(\u91d1\u9670)\uc758 8\uac00\uc9c0\ub85c \uad6c\ubd84\ud558\ub294 \uccb4\uc9c8\uc758\ud559\ub860\uc774\ub2e4. 1965\ub144 10\uc6d4 24\uc77c \ub300\ud55c\ubbfc\uad6d\uc758 \ud55c\uc758\uc0ac \uad8c\ub3c4\uc6d0(\u6b0a\u5ea6\u676c)\uc774 \uc77c\ubcf8 \ub3c4\ucfc4\uc5d0\uc11c \uac1c\ucd5c\ub41c \uc81c1\ud68c \uad6d\uc81c \uce68\uad6c\ud559\ud68c(The International Congress of Acupuncture)\uc5d0\uc11c \ucd5c\ucd08\ub85c \ubc1c\ud45c\ud558\uc600\ub2e4.[1] \uc804\ud1b5\ud55c\uc758\ud559\uc5d0\uc11c\ub294 \ubb38\uc9c4, \uc124\ubb38\uc870\uc0ac, \uce68, \uc57d \ub4f1\uc758 \ubc29\ubc95\uc73c\ub85c \uc0ac\uc0c1\uccb4\uc9c8\uc744 \ud310\ubcc4\ud55c\ub2e4. \uadf8\ub7ec\ub098 \uad8c\ub3c4\uc6d0\uc5d0 \uc758\ud574 \ucc3d\uc2dc\ub41c 8\uccb4\uc9c8\uc758 \ud310\ubcc4\uc740 \uc624\uc9c1 \uc694\uace8\ub3d9\ub9e5\uc758 \ub9e5\uc9c4\uc744 \uae30\ubcf8\uc73c\ub85c \ud558\uc5ec \uc774\ub8e8\uc5b4\uc9c4\ub2e4. \ud55c \ub54c \u2018\uc774\uc81c\ub9c8\uc758 \uc0ac\uc0c1\uc758\ud559\uacfc \ub2e4\ub974\ub2e4\u2019\uac70\ub098, \u2018\uacfc\ud559\uc801\uc778 \uadfc\uac70\uac00 \ubd80\uc871\ud558\ub2e4\u2019\ub294 \uc774\uc720\ub85c \ud55c\uc758\ud559\uacc4\uc5d0\uc11c \uc774\ub2e8\uc2dc\ub418\uae30\ub3c4 \ud558\uc600\ub2e4. \uadf8\ub7ec\ub098 \uadf8\uac00 \ub3c4\ucfc4\uc5d0\uc11c \ucd5c\ucd08\ub85c \ubc1c\ud45c\ud55c 8\uccb4\uc9c8 \uc758\ud559\uc740 \uc138\uacc4 \uc758\ud559\uacc4\uc5d0 \ud070 \ubc18\ud5a5\uc744 \uc77c\uc73c\ucf30\ub2e4\uace0 \ud55c\ub2e4. \ubbf8\uad6d \uce98\ub9ac\ud3ec\ub2c8\uc544\uc8fc(\u5dde)\ub294 \uadf8\uc5d0\uac8c \ud658\uc790\ub97c \uce58\ub8cc\ud560 \uc218 \uc788\ub294 \uc815\uc2dd \uba74\ud5c8\ub97c \ud5c8\uac00\ud588\uace0, \uad6d\ub0b4\uc5d0\uc120 \ub3d9\uc591\uc758\ub300(\uacbd\ud76c\ub300 \ud55c\uc758\uacfc\ub300\ud559\uc758 \uc804\uc2e0)\uc5d0\uc11c \uac15\uc758\ub3c4 \uc694\uccad\ud574 \uc654\ub2e4. \uba85\uc9c0\ub300\ub294 \uadf8\uc5d0\uac8c \uba85\uc608\ucca0\ud559\ubc15\uc0ac \ud559\uc704\ub97c \uc218\uc5ec\ud588\ub2e4.[2]", "answer": "\uc694\uace8\ub3d9\ub9e5", "sentence": "\uadf8\ub7ec\ub098 \uad8c\ub3c4\uc6d0\uc5d0 \uc758\ud574 \ucc3d\uc2dc\ub41c 8\uccb4\uc9c8\uc758 \ud310\ubcc4\uc740 \uc624\uc9c1 \uc694\uace8\ub3d9\ub9e5 \uc758 \ub9e5\uc9c4\uc744 \uae30\ubcf8\uc73c\ub85c \ud558\uc5ec \uc774\ub8e8\uc5b4\uc9c4\ub2e4.", "paragraph_sentence": "8\uccb4\uc9c8 \uc758\ud559(\ud314\uccb4\uc9c8\uc758\ud559, \u516b\u9ad4\u8cea\u91ab\u5b78, Eight-Constitution Medicine)\uc740 \uc0ac\ub78c\uc758 \uccb4\uc9c8\uc744 \uc624\uc7a5\uc721\ubd80\uc758 \uac15\uc57d\ubc30\uc5f4\uc5d0 \ub530\ub77c \ubaa9\uc591(\u967d\u967d), \ubaa9\uc74c(\u6728\u9670), \uc218\uc591(\u6c34\u967d), \uc218\uc74c(\u6c34\u9670), \ud1a0\uc591(\u571f\u967d), \ud1a0\uc74c(\u571f\u9670), \uae08\uc591(\u91d1\u967d), \uae08\uc74c(\u91d1\u9670)\uc758 8\uac00\uc9c0\ub85c \uad6c\ubd84\ud558\ub294 \uccb4\uc9c8\uc758\ud559\ub860\uc774\ub2e4. 1965\ub144 10\uc6d4 24\uc77c \ub300\ud55c\ubbfc\uad6d\uc758 \ud55c\uc758\uc0ac \uad8c\ub3c4\uc6d0(\u6b0a\u5ea6\u676c)\uc774 \uc77c\ubcf8 \ub3c4\ucfc4\uc5d0\uc11c \uac1c\ucd5c\ub41c \uc81c1\ud68c \uad6d\uc81c \uce68\uad6c\ud559\ud68c(The International Congress of Acupuncture)\uc5d0\uc11c \ucd5c\ucd08\ub85c \ubc1c\ud45c\ud558\uc600\ub2e4.[1] \uc804\ud1b5\ud55c\uc758\ud559\uc5d0\uc11c\ub294 \ubb38\uc9c4, \uc124\ubb38\uc870\uc0ac, \uce68, \uc57d \ub4f1\uc758 \ubc29\ubc95\uc73c\ub85c \uc0ac\uc0c1\uccb4\uc9c8\uc744 \ud310\ubcc4\ud55c\ub2e4. \uadf8\ub7ec\ub098 \uad8c\ub3c4\uc6d0\uc5d0 \uc758\ud574 \ucc3d\uc2dc\ub41c 8\uccb4\uc9c8\uc758 \ud310\ubcc4\uc740 \uc624\uc9c1 \uc694\uace8\ub3d9\ub9e5 \uc758 \ub9e5\uc9c4\uc744 \uae30\ubcf8\uc73c\ub85c \ud558\uc5ec \uc774\ub8e8\uc5b4\uc9c4\ub2e4. \ud55c \ub54c \u2018\uc774\uc81c\ub9c8\uc758 \uc0ac\uc0c1\uc758\ud559\uacfc \ub2e4\ub974\ub2e4\u2019\uac70\ub098, \u2018\uacfc\ud559\uc801\uc778 \uadfc\uac70\uac00 \ubd80\uc871\ud558\ub2e4\u2019\ub294 \uc774\uc720\ub85c \ud55c\uc758\ud559\uacc4\uc5d0\uc11c \uc774\ub2e8\uc2dc\ub418\uae30\ub3c4 \ud558\uc600\ub2e4. \uadf8\ub7ec\ub098 \uadf8\uac00 \ub3c4\ucfc4\uc5d0\uc11c \ucd5c\ucd08\ub85c \ubc1c\ud45c\ud55c 8\uccb4\uc9c8 \uc758\ud559\uc740 \uc138\uacc4 \uc758\ud559\uacc4\uc5d0 \ud070 \ubc18\ud5a5\uc744 \uc77c\uc73c\ucf30\ub2e4\uace0 \ud55c\ub2e4. \ubbf8\uad6d \uce98\ub9ac\ud3ec\ub2c8\uc544\uc8fc(\u5dde)\ub294 \uadf8\uc5d0\uac8c \ud658\uc790\ub97c \uce58\ub8cc\ud560 \uc218 \uc788\ub294 \uc815\uc2dd \uba74\ud5c8\ub97c \ud5c8\uac00\ud588\uace0, \uad6d\ub0b4\uc5d0\uc120 \ub3d9\uc591\uc758\ub300(\uacbd\ud76c\ub300 \ud55c\uc758\uacfc\ub300\ud559\uc758 \uc804\uc2e0)\uc5d0\uc11c \uac15\uc758\ub3c4 \uc694\uccad\ud574 \uc654\ub2e4. \uba85\uc9c0\ub300\ub294 \uadf8\uc5d0\uac8c \uba85\uc608\ucca0\ud559\ubc15\uc0ac \ud559\uc704\ub97c \uc218\uc5ec\ud588\ub2e4.[2]", "paragraph_answer": "8\uccb4\uc9c8 \uc758\ud559(\ud314\uccb4\uc9c8\uc758\ud559, \u516b\u9ad4\u8cea\u91ab\u5b78, Eight-Constitution Medicine)\uc740 \uc0ac\ub78c\uc758 \uccb4\uc9c8\uc744 \uc624\uc7a5\uc721\ubd80\uc758 \uac15\uc57d\ubc30\uc5f4\uc5d0 \ub530\ub77c \ubaa9\uc591(\u967d\u967d), \ubaa9\uc74c(\u6728\u9670), \uc218\uc591(\u6c34\u967d), \uc218\uc74c(\u6c34\u9670), \ud1a0\uc591(\u571f\u967d), \ud1a0\uc74c(\u571f\u9670), \uae08\uc591(\u91d1\u967d), \uae08\uc74c(\u91d1\u9670)\uc758 8\uac00\uc9c0\ub85c \uad6c\ubd84\ud558\ub294 \uccb4\uc9c8\uc758\ud559\ub860\uc774\ub2e4. 1965\ub144 10\uc6d4 24\uc77c \ub300\ud55c\ubbfc\uad6d\uc758 \ud55c\uc758\uc0ac \uad8c\ub3c4\uc6d0(\u6b0a\u5ea6\u676c)\uc774 \uc77c\ubcf8 \ub3c4\ucfc4\uc5d0\uc11c \uac1c\ucd5c\ub41c \uc81c1\ud68c \uad6d\uc81c \uce68\uad6c\ud559\ud68c(The International Congress of Acupuncture)\uc5d0\uc11c \ucd5c\ucd08\ub85c \ubc1c\ud45c\ud558\uc600\ub2e4.[1] \uc804\ud1b5\ud55c\uc758\ud559\uc5d0\uc11c\ub294 \ubb38\uc9c4, \uc124\ubb38\uc870\uc0ac, \uce68, \uc57d \ub4f1\uc758 \ubc29\ubc95\uc73c\ub85c \uc0ac\uc0c1\uccb4\uc9c8\uc744 \ud310\ubcc4\ud55c\ub2e4. \uadf8\ub7ec\ub098 \uad8c\ub3c4\uc6d0\uc5d0 \uc758\ud574 \ucc3d\uc2dc\ub41c 8\uccb4\uc9c8\uc758 \ud310\ubcc4\uc740 \uc624\uc9c1 \uc694\uace8\ub3d9\ub9e5 \uc758 \ub9e5\uc9c4\uc744 \uae30\ubcf8\uc73c\ub85c \ud558\uc5ec \uc774\ub8e8\uc5b4\uc9c4\ub2e4. \ud55c \ub54c \u2018\uc774\uc81c\ub9c8\uc758 \uc0ac\uc0c1\uc758\ud559\uacfc \ub2e4\ub974\ub2e4\u2019\uac70\ub098, \u2018\uacfc\ud559\uc801\uc778 \uadfc\uac70\uac00 \ubd80\uc871\ud558\ub2e4\u2019\ub294 \uc774\uc720\ub85c \ud55c\uc758\ud559\uacc4\uc5d0\uc11c \uc774\ub2e8\uc2dc\ub418\uae30\ub3c4 \ud558\uc600\ub2e4. \uadf8\ub7ec\ub098 \uadf8\uac00 \ub3c4\ucfc4\uc5d0\uc11c \ucd5c\ucd08\ub85c \ubc1c\ud45c\ud55c 8\uccb4\uc9c8 \uc758\ud559\uc740 \uc138\uacc4 \uc758\ud559\uacc4\uc5d0 \ud070 \ubc18\ud5a5\uc744 \uc77c\uc73c\ucf30\ub2e4\uace0 \ud55c\ub2e4. \ubbf8\uad6d \uce98\ub9ac\ud3ec\ub2c8\uc544\uc8fc(\u5dde)\ub294 \uadf8\uc5d0\uac8c \ud658\uc790\ub97c \uce58\ub8cc\ud560 \uc218 \uc788\ub294 \uc815\uc2dd \uba74\ud5c8\ub97c \ud5c8\uac00\ud588\uace0, \uad6d\ub0b4\uc5d0\uc120 \ub3d9\uc591\uc758\ub300(\uacbd\ud76c\ub300 \ud55c\uc758\uacfc\ub300\ud559\uc758 \uc804\uc2e0)\uc5d0\uc11c \uac15\uc758\ub3c4 \uc694\uccad\ud574 \uc654\ub2e4. \uba85\uc9c0\ub300\ub294 \uadf8\uc5d0\uac8c \uba85\uc608\ucca0\ud559\ubc15\uc0ac \ud559\uc704\ub97c \uc218\uc5ec\ud588\ub2e4.[2]", "sentence_answer": "\uadf8\ub7ec\ub098 \uad8c\ub3c4\uc6d0\uc5d0 \uc758\ud574 \ucc3d\uc2dc\ub41c 8\uccb4\uc9c8\uc758 \ud310\ubcc4\uc740 \uc624\uc9c1 \uc694\uace8\ub3d9\ub9e5 \uc758 \ub9e5\uc9c4\uc744 \uae30\ubcf8\uc73c\ub85c \ud558\uc5ec \uc774\ub8e8\uc5b4\uc9c4\ub2e4.", "paragraph_id": "6566751-0-2", "paragraph_question": "question: \uad8c\ub3c4\uc6d0\uc5d0 \uc758\ud574 \ucc3d\uc2dc\ub41c 8\uccb4\uc9c8\uc758 \ud310\ubcc4\uc740 \uc624\uc9c1 \ubb34\uc5c7\uc758 \ub9e5\uc9c4\uc744 \uae30\ubcf8\uc73c\ub85c \uc774\ub8e8\uc5b4\uc9c0\ub294\uac00?, context: 8\uccb4\uc9c8 \uc758\ud559(\ud314\uccb4\uc9c8\uc758\ud559, \u516b\u9ad4\u8cea\u91ab\u5b78, Eight-Constitution Medicine)\uc740 \uc0ac\ub78c\uc758 \uccb4\uc9c8\uc744 \uc624\uc7a5\uc721\ubd80\uc758 \uac15\uc57d\ubc30\uc5f4\uc5d0 \ub530\ub77c \ubaa9\uc591(\u967d\u967d), \ubaa9\uc74c(\u6728\u9670), \uc218\uc591(\u6c34\u967d), \uc218\uc74c(\u6c34\u9670), \ud1a0\uc591(\u571f\u967d), \ud1a0\uc74c(\u571f\u9670), \uae08\uc591(\u91d1\u967d), \uae08\uc74c(\u91d1\u9670)\uc758 8\uac00\uc9c0\ub85c \uad6c\ubd84\ud558\ub294 \uccb4\uc9c8\uc758\ud559\ub860\uc774\ub2e4. 1965\ub144 10\uc6d4 24\uc77c \ub300\ud55c\ubbfc\uad6d\uc758 \ud55c\uc758\uc0ac \uad8c\ub3c4\uc6d0(\u6b0a\u5ea6\u676c)\uc774 \uc77c\ubcf8 \ub3c4\ucfc4\uc5d0\uc11c \uac1c\ucd5c\ub41c \uc81c1\ud68c \uad6d\uc81c \uce68\uad6c\ud559\ud68c(The International Congress of Acupuncture)\uc5d0\uc11c \ucd5c\ucd08\ub85c \ubc1c\ud45c\ud558\uc600\ub2e4.[1] \uc804\ud1b5\ud55c\uc758\ud559\uc5d0\uc11c\ub294 \ubb38\uc9c4, \uc124\ubb38\uc870\uc0ac, \uce68, \uc57d \ub4f1\uc758 \ubc29\ubc95\uc73c\ub85c \uc0ac\uc0c1\uccb4\uc9c8\uc744 \ud310\ubcc4\ud55c\ub2e4. \uadf8\ub7ec\ub098 \uad8c\ub3c4\uc6d0\uc5d0 \uc758\ud574 \ucc3d\uc2dc\ub41c 8\uccb4\uc9c8\uc758 \ud310\ubcc4\uc740 \uc624\uc9c1 \uc694\uace8\ub3d9\ub9e5\uc758 \ub9e5\uc9c4\uc744 \uae30\ubcf8\uc73c\ub85c \ud558\uc5ec \uc774\ub8e8\uc5b4\uc9c4\ub2e4. \ud55c \ub54c \u2018\uc774\uc81c\ub9c8\uc758 \uc0ac\uc0c1\uc758\ud559\uacfc \ub2e4\ub974\ub2e4\u2019\uac70\ub098, \u2018\uacfc\ud559\uc801\uc778 \uadfc\uac70\uac00 \ubd80\uc871\ud558\ub2e4\u2019\ub294 \uc774\uc720\ub85c \ud55c\uc758\ud559\uacc4\uc5d0\uc11c \uc774\ub2e8\uc2dc\ub418\uae30\ub3c4 \ud558\uc600\ub2e4. \uadf8\ub7ec\ub098 \uadf8\uac00 \ub3c4\ucfc4\uc5d0\uc11c \ucd5c\ucd08\ub85c \ubc1c\ud45c\ud55c 8\uccb4\uc9c8 \uc758\ud559\uc740 \uc138\uacc4 \uc758\ud559\uacc4\uc5d0 \ud070 \ubc18\ud5a5\uc744 \uc77c\uc73c\ucf30\ub2e4\uace0 \ud55c\ub2e4. \ubbf8\uad6d \uce98\ub9ac\ud3ec\ub2c8\uc544\uc8fc(\u5dde)\ub294 \uadf8\uc5d0\uac8c \ud658\uc790\ub97c \uce58\ub8cc\ud560 \uc218 \uc788\ub294 \uc815\uc2dd \uba74\ud5c8\ub97c \ud5c8\uac00\ud588\uace0, \uad6d\ub0b4\uc5d0\uc120 \ub3d9\uc591\uc758\ub300(\uacbd\ud76c\ub300 \ud55c\uc758\uacfc\ub300\ud559\uc758 \uc804\uc2e0)\uc5d0\uc11c \uac15\uc758\ub3c4 \uc694\uccad\ud574 \uc654\ub2e4. \uba85\uc9c0\ub300\ub294 \uadf8\uc5d0\uac8c \uba85\uc608\ucca0\ud559\ubc15\uc0ac \ud559\uc704\ub97c \uc218\uc5ec\ud588\ub2e4.[2]"} {"question": "\uacbd\ud5a5\uc2e0\ubb38\uc774 \ub0b8 \uc0c8\ub204\ub9ac \ud6c4\ubcf4 \ubc15\uadfc\ud61c \ub4a4\uc9d1\uc5b4\ubcf4\uae30: \ub3c4\ub355\uc131\uacfc \uacfc\uac70\ub97c \ubb3b\ub294\ub2e4\ub294 \uc81c\ubaa9\uc758 \uae30\uc0ac\ub97c \ub0b8 \uac83\uc740 \uc5b8\uc81c\uc778\uac00?", "paragraph": "\ud55c\ud3b8 2014\ub144 5\uc6d4 15\uc77c \uc815\uc218\uc7a5\ud559\ud68c\uc640\uc758 \uad00\ub828\uc131\uc744 \ud5c8\uc704\ubcf4\ub3c4\ud588\ub2e4\uba70 \ubc15\uadfc\ud61c \ub300\ud1b5\ub839\uc774 \uacbd\ud5a5\uc2e0\ubb38\uacfc \ud574\ub2f9\uae30\uc790\ub97c \uc0c1\ub300\ub85c \ub0b8 \uc190\ud574\ubc30\uc0c1 \ubc0f \uc815\uc815\ubcf4\ub3c4 \uccad\uad6c \uc18c\uc1a1\uc5d0\uc11c \uc11c\uc6b8\ub0a8\ubd80\uc9c0\ubc95\uc740 \uc6d0\uace0 \uc77c\ubd80 \uc2b9\uc18c \ud310\uacb0\uc744 \ud588\ub2e4. \uc7ac\ud310\ubd80\ub294 \"\uae30\uc0ac\uc758 \uc77c\ubd80 \ub0b4\uc6a9\uc774 \ud5c8\uc704\uc5ec\uc11c \ubc15 \ub300\ud1b5\ub839\uc758 \uba85\uc608\uac00 \ud6fc\uc190\ub41c \uc0ac\uc2e4\uc774 \uc778\uc815\ub41c\ub2e4\"\uba70 \"500\ub9cc\uc6d0\uc744 \uc9c0\uae09\ud558\uace0 \uc815\uc815\ubcf4\ub3c4\ubb38\uc744 \uac8c\uc7ac\ud558\ub77c\"\uace0 \ud310\uc2dc\ud588\ub2e4. \uacbd\ud5a5\uc2e0\ubb38\uc740 \uc81c18\ub300 \ub300\uc120 \ub2f9\uc2dc\uc778 2012\ub144 8\uc6d4 28\uc77c '\uc0c8\ub204\ub9ac \ud6c4\ubcf4 \ubc15\uadfc\ud61c \ub4a4\uc9d1\uc5b4\ubcf4\uae30: \ub3c4\ub355\uc131\uacfc \uacfc\uac70\ub97c \ubb3b\ub294\ub2e4'\ub294 \uc81c\ubaa9\uc758 \uae30\uc0ac\uc5d0\uc11c \ubc15 \ub300\ud1b5\ub839\uc774 \uc774\uc0ac\uc7a5\uc73c\ub85c \uc7ac\uc9c1\ud558\uba74\uc11c \uc5f0\uac04 \uc815\uc218\uc7a5\ud559\ud68c \uc7a5\ud559\uae08\uc758 10%\ub97c \ubcf4\uc218\ub85c \ubc1b\uc558\uace0, \uc815\uc218\uc7a5\ud559\ud68c\uc5d0 \ub300\ud55c '\uad6d\uac00\uc815\ubcf4\uc6d0 \uacfc\uac70\uc0ac \uc9c4\uc2e4 \uaddc\uba85\uc744 \uc704\ud55c \ubc1c\uc804\uc704\uc6d0\ud68c'\uc758 \uacb0\uc815\uc774 \ub098\uc624\uc790 \uc774\uc0ac\uc7a5\uc9c1\uc744 \uc0ac\ud1f4\ud588\ub2e4\ub294 \ub0b4\uc6a9 \ub4f1\uc744 \ubcf4\ub3c4\ud588\uc5c8\ub2e4.", "answer": "2012\ub144 8\uc6d4 28\uc77c", "sentence": "\uacbd\ud5a5\uc2e0\ubb38\uc740 \uc81c18\ub300 \ub300\uc120 \ub2f9\uc2dc\uc778 2012\ub144 8\uc6d4 28\uc77c '\uc0c8\ub204\ub9ac \ud6c4\ubcf4 \ubc15\uadfc\ud61c \ub4a4\uc9d1\uc5b4\ubcf4\uae30: \ub3c4\ub355\uc131\uacfc \uacfc\uac70\ub97c \ubb3b\ub294\ub2e4'\ub294 \uc81c\ubaa9\uc758 \uae30\uc0ac\uc5d0\uc11c \ubc15 \ub300\ud1b5\ub839\uc774 \uc774\uc0ac\uc7a5\uc73c\ub85c \uc7ac\uc9c1\ud558\uba74\uc11c \uc5f0\uac04 \uc815\uc218\uc7a5\ud559\ud68c \uc7a5\ud559\uae08\uc758 10%\ub97c \ubcf4\uc218\ub85c \ubc1b\uc558\uace0, \uc815\uc218\uc7a5\ud559\ud68c\uc5d0 \ub300\ud55c '\uad6d\uac00\uc815\ubcf4\uc6d0 \uacfc\uac70\uc0ac \uc9c4\uc2e4 \uaddc\uba85\uc744 \uc704\ud55c \ubc1c\uc804\uc704\uc6d0\ud68c'\uc758 \uacb0\uc815\uc774 \ub098\uc624\uc790 \uc774\uc0ac\uc7a5\uc9c1\uc744 \uc0ac\ud1f4\ud588\ub2e4\ub294 \ub0b4\uc6a9 \ub4f1\uc744 \ubcf4\ub3c4\ud588\uc5c8\ub2e4.", "paragraph_sentence": "\ud55c\ud3b8 2014\ub144 5\uc6d4 15\uc77c \uc815\uc218\uc7a5\ud559\ud68c\uc640\uc758 \uad00\ub828\uc131\uc744 \ud5c8\uc704\ubcf4\ub3c4\ud588\ub2e4\uba70 \ubc15\uadfc\ud61c \ub300\ud1b5\ub839\uc774 \uacbd\ud5a5\uc2e0\ubb38\uacfc \ud574\ub2f9\uae30\uc790\ub97c \uc0c1\ub300\ub85c \ub0b8 \uc190\ud574\ubc30\uc0c1 \ubc0f \uc815\uc815\ubcf4\ub3c4 \uccad\uad6c \uc18c\uc1a1\uc5d0\uc11c \uc11c\uc6b8\ub0a8\ubd80\uc9c0\ubc95\uc740 \uc6d0\uace0 \uc77c\ubd80 \uc2b9\uc18c \ud310\uacb0\uc744 \ud588\ub2e4. \uc7ac\ud310\ubd80\ub294 \"\uae30\uc0ac\uc758 \uc77c\ubd80 \ub0b4\uc6a9\uc774 \ud5c8\uc704\uc5ec\uc11c \ubc15 \ub300\ud1b5\ub839\uc758 \uba85\uc608\uac00 \ud6fc\uc190\ub41c \uc0ac\uc2e4\uc774 \uc778\uc815\ub41c\ub2e4\"\uba70 \"500\ub9cc\uc6d0\uc744 \uc9c0\uae09\ud558\uace0 \uc815\uc815\ubcf4\ub3c4\ubb38\uc744 \uac8c\uc7ac\ud558\ub77c\"\uace0 \ud310\uc2dc\ud588\ub2e4. \uacbd\ud5a5\uc2e0\ubb38\uc740 \uc81c18\ub300 \ub300\uc120 \ub2f9\uc2dc\uc778 2012\ub144 8\uc6d4 28\uc77c '\uc0c8\ub204\ub9ac \ud6c4\ubcf4 \ubc15\uadfc\ud61c \ub4a4\uc9d1\uc5b4\ubcf4\uae30: \ub3c4\ub355\uc131\uacfc \uacfc\uac70\ub97c \ubb3b\ub294\ub2e4'\ub294 \uc81c\ubaa9\uc758 \uae30\uc0ac\uc5d0\uc11c \ubc15 \ub300\ud1b5\ub839\uc774 \uc774\uc0ac\uc7a5\uc73c\ub85c \uc7ac\uc9c1\ud558\uba74\uc11c \uc5f0\uac04 \uc815\uc218\uc7a5\ud559\ud68c \uc7a5\ud559\uae08\uc758 10%\ub97c \ubcf4\uc218\ub85c \ubc1b\uc558\uace0, \uc815\uc218\uc7a5\ud559\ud68c\uc5d0 \ub300\ud55c '\uad6d\uac00\uc815\ubcf4\uc6d0 \uacfc\uac70\uc0ac \uc9c4\uc2e4 \uaddc\uba85\uc744 \uc704\ud55c \ubc1c\uc804\uc704\uc6d0\ud68c'\uc758 \uacb0\uc815\uc774 \ub098\uc624\uc790 \uc774\uc0ac\uc7a5\uc9c1\uc744 \uc0ac\ud1f4\ud588\ub2e4\ub294 \ub0b4\uc6a9 \ub4f1\uc744 \ubcf4\ub3c4\ud588\uc5c8\ub2e4. ", "paragraph_answer": "\ud55c\ud3b8 2014\ub144 5\uc6d4 15\uc77c \uc815\uc218\uc7a5\ud559\ud68c\uc640\uc758 \uad00\ub828\uc131\uc744 \ud5c8\uc704\ubcf4\ub3c4\ud588\ub2e4\uba70 \ubc15\uadfc\ud61c \ub300\ud1b5\ub839\uc774 \uacbd\ud5a5\uc2e0\ubb38\uacfc \ud574\ub2f9\uae30\uc790\ub97c \uc0c1\ub300\ub85c \ub0b8 \uc190\ud574\ubc30\uc0c1 \ubc0f \uc815\uc815\ubcf4\ub3c4 \uccad\uad6c \uc18c\uc1a1\uc5d0\uc11c \uc11c\uc6b8\ub0a8\ubd80\uc9c0\ubc95\uc740 \uc6d0\uace0 \uc77c\ubd80 \uc2b9\uc18c \ud310\uacb0\uc744 \ud588\ub2e4. \uc7ac\ud310\ubd80\ub294 \"\uae30\uc0ac\uc758 \uc77c\ubd80 \ub0b4\uc6a9\uc774 \ud5c8\uc704\uc5ec\uc11c \ubc15 \ub300\ud1b5\ub839\uc758 \uba85\uc608\uac00 \ud6fc\uc190\ub41c \uc0ac\uc2e4\uc774 \uc778\uc815\ub41c\ub2e4\"\uba70 \"500\ub9cc\uc6d0\uc744 \uc9c0\uae09\ud558\uace0 \uc815\uc815\ubcf4\ub3c4\ubb38\uc744 \uac8c\uc7ac\ud558\ub77c\"\uace0 \ud310\uc2dc\ud588\ub2e4. \uacbd\ud5a5\uc2e0\ubb38\uc740 \uc81c18\ub300 \ub300\uc120 \ub2f9\uc2dc\uc778 2012\ub144 8\uc6d4 28\uc77c '\uc0c8\ub204\ub9ac \ud6c4\ubcf4 \ubc15\uadfc\ud61c \ub4a4\uc9d1\uc5b4\ubcf4\uae30: \ub3c4\ub355\uc131\uacfc \uacfc\uac70\ub97c \ubb3b\ub294\ub2e4'\ub294 \uc81c\ubaa9\uc758 \uae30\uc0ac\uc5d0\uc11c \ubc15 \ub300\ud1b5\ub839\uc774 \uc774\uc0ac\uc7a5\uc73c\ub85c \uc7ac\uc9c1\ud558\uba74\uc11c \uc5f0\uac04 \uc815\uc218\uc7a5\ud559\ud68c \uc7a5\ud559\uae08\uc758 10%\ub97c \ubcf4\uc218\ub85c \ubc1b\uc558\uace0, \uc815\uc218\uc7a5\ud559\ud68c\uc5d0 \ub300\ud55c '\uad6d\uac00\uc815\ubcf4\uc6d0 \uacfc\uac70\uc0ac \uc9c4\uc2e4 \uaddc\uba85\uc744 \uc704\ud55c \ubc1c\uc804\uc704\uc6d0\ud68c'\uc758 \uacb0\uc815\uc774 \ub098\uc624\uc790 \uc774\uc0ac\uc7a5\uc9c1\uc744 \uc0ac\ud1f4\ud588\ub2e4\ub294 \ub0b4\uc6a9 \ub4f1\uc744 \ubcf4\ub3c4\ud588\uc5c8\ub2e4.", "sentence_answer": "\uacbd\ud5a5\uc2e0\ubb38\uc740 \uc81c18\ub300 \ub300\uc120 \ub2f9\uc2dc\uc778 2012\ub144 8\uc6d4 28\uc77c '\uc0c8\ub204\ub9ac \ud6c4\ubcf4 \ubc15\uadfc\ud61c \ub4a4\uc9d1\uc5b4\ubcf4\uae30: \ub3c4\ub355\uc131\uacfc \uacfc\uac70\ub97c \ubb3b\ub294\ub2e4'\ub294 \uc81c\ubaa9\uc758 \uae30\uc0ac\uc5d0\uc11c \ubc15 \ub300\ud1b5\ub839\uc774 \uc774\uc0ac\uc7a5\uc73c\ub85c \uc7ac\uc9c1\ud558\uba74\uc11c \uc5f0\uac04 \uc815\uc218\uc7a5\ud559\ud68c \uc7a5\ud559\uae08\uc758 10%\ub97c \ubcf4\uc218\ub85c \ubc1b\uc558\uace0, \uc815\uc218\uc7a5\ud559\ud68c\uc5d0 \ub300\ud55c '\uad6d\uac00\uc815\ubcf4\uc6d0 \uacfc\uac70\uc0ac \uc9c4\uc2e4 \uaddc\uba85\uc744 \uc704\ud55c \ubc1c\uc804\uc704\uc6d0\ud68c'\uc758 \uacb0\uc815\uc774 \ub098\uc624\uc790 \uc774\uc0ac\uc7a5\uc9c1\uc744 \uc0ac\ud1f4\ud588\ub2e4\ub294 \ub0b4\uc6a9 \ub4f1\uc744 \ubcf4\ub3c4\ud588\uc5c8\ub2e4.", "paragraph_id": "6581158-49-2", "paragraph_question": "question: \uacbd\ud5a5\uc2e0\ubb38\uc774 \ub0b8 \uc0c8\ub204\ub9ac \ud6c4\ubcf4 \ubc15\uadfc\ud61c \ub4a4\uc9d1\uc5b4\ubcf4\uae30: \ub3c4\ub355\uc131\uacfc \uacfc\uac70\ub97c \ubb3b\ub294\ub2e4\ub294 \uc81c\ubaa9\uc758 \uae30\uc0ac\ub97c \ub0b8 \uac83\uc740 \uc5b8\uc81c\uc778\uac00?, context: \ud55c\ud3b8 2014\ub144 5\uc6d4 15\uc77c \uc815\uc218\uc7a5\ud559\ud68c\uc640\uc758 \uad00\ub828\uc131\uc744 \ud5c8\uc704\ubcf4\ub3c4\ud588\ub2e4\uba70 \ubc15\uadfc\ud61c \ub300\ud1b5\ub839\uc774 \uacbd\ud5a5\uc2e0\ubb38\uacfc \ud574\ub2f9\uae30\uc790\ub97c \uc0c1\ub300\ub85c \ub0b8 \uc190\ud574\ubc30\uc0c1 \ubc0f \uc815\uc815\ubcf4\ub3c4 \uccad\uad6c \uc18c\uc1a1\uc5d0\uc11c \uc11c\uc6b8\ub0a8\ubd80\uc9c0\ubc95\uc740 \uc6d0\uace0 \uc77c\ubd80 \uc2b9\uc18c \ud310\uacb0\uc744 \ud588\ub2e4. \uc7ac\ud310\ubd80\ub294 \"\uae30\uc0ac\uc758 \uc77c\ubd80 \ub0b4\uc6a9\uc774 \ud5c8\uc704\uc5ec\uc11c \ubc15 \ub300\ud1b5\ub839\uc758 \uba85\uc608\uac00 \ud6fc\uc190\ub41c \uc0ac\uc2e4\uc774 \uc778\uc815\ub41c\ub2e4\"\uba70 \"500\ub9cc\uc6d0\uc744 \uc9c0\uae09\ud558\uace0 \uc815\uc815\ubcf4\ub3c4\ubb38\uc744 \uac8c\uc7ac\ud558\ub77c\"\uace0 \ud310\uc2dc\ud588\ub2e4. \uacbd\ud5a5\uc2e0\ubb38\uc740 \uc81c18\ub300 \ub300\uc120 \ub2f9\uc2dc\uc778 2012\ub144 8\uc6d4 28\uc77c '\uc0c8\ub204\ub9ac \ud6c4\ubcf4 \ubc15\uadfc\ud61c \ub4a4\uc9d1\uc5b4\ubcf4\uae30: \ub3c4\ub355\uc131\uacfc \uacfc\uac70\ub97c \ubb3b\ub294\ub2e4'\ub294 \uc81c\ubaa9\uc758 \uae30\uc0ac\uc5d0\uc11c \ubc15 \ub300\ud1b5\ub839\uc774 \uc774\uc0ac\uc7a5\uc73c\ub85c \uc7ac\uc9c1\ud558\uba74\uc11c \uc5f0\uac04 \uc815\uc218\uc7a5\ud559\ud68c \uc7a5\ud559\uae08\uc758 10%\ub97c \ubcf4\uc218\ub85c \ubc1b\uc558\uace0, \uc815\uc218\uc7a5\ud559\ud68c\uc5d0 \ub300\ud55c '\uad6d\uac00\uc815\ubcf4\uc6d0 \uacfc\uac70\uc0ac \uc9c4\uc2e4 \uaddc\uba85\uc744 \uc704\ud55c \ubc1c\uc804\uc704\uc6d0\ud68c'\uc758 \uacb0\uc815\uc774 \ub098\uc624\uc790 \uc774\uc0ac\uc7a5\uc9c1\uc744 \uc0ac\ud1f4\ud588\ub2e4\ub294 \ub0b4\uc6a9 \ub4f1\uc744 \ubcf4\ub3c4\ud588\uc5c8\ub2e4."} {"question": "\ubd81\uc544\uba54\ub9ac\uce74\uc5d0 \uc720\ub7fd\ucc0c\ub974\ub808\uae30\ub97c \ub3c4\uc785\ud558\ub294 \uc77c\uc740 \uba87 \ubc88\uc758 \uc2e4\ud328\uac00 \uc788\uc5c8\ub294\uac00?", "paragraph": "\ubd81\uc544\uba54\ub9ac\uce74\uc5d0 \uc720\ub7fd\ucc0c\ub974\ub808\uae30\ub97c \ub3c4\uc785\ud558\ub294 \uc77c\uc740 \ub450 \ubc88 \uc2e4\ud328\ud588\ub2e4. \uc774\ud6c4 1890\ub144 \ub274\uc695 \uc13c\ud2b8\ub7f4 \ud30c\ud06c\uc5d0\uc11c 60\ub9c8\ub9ac\uc758 \uc720\ub7fd\ucc0c\ub974\ub808\uae30\uac00 \ubc29\uc0ac\ub418\uc5c8\ub2e4. \uc774\ub97c \ucd94\uc9c4\ud55c \ubbf8\uad6d \uc801\uc751\ud654 \ud611\ud68c \ud611\ud68c\uc7a5 \uc720\uc9c4 \uc170\ud3a0\ub9b0\uc740 \uc70c\ub9ac\uc5c4 \uc170\uc775\uc2a4\ud53c\uc5b4\uc758 \ud76c\uace1\ub4e4\uc5d0 \ub098\uc624\ub294 \uc0c8\ub4e4\uc744 \ubaa8\ub450 \ubd81\ubbf8\uc5d0 \ub3c4\uc785\ud558\ub824\uace0 \ud558\ub358 \uc790\uc600\ub2e4. \uac19\uc740 \ub0a0 \ud3ec\ud2c0\ub79c\ub4dc \uba85\uae08 \uad6c\ub77d\ubd80\uac00 \uc624\ub9ac\uac74 \ud3ec\ud2c0\ub79c\ub4dc\uc5d0\uc11c \uc720\ub7fd\ucc0c\ub974\ub808\uae30 35\uc30d\uc744 \ubc29\uc0ac\ud588\ub2e4. \uc774 \uc0c8\ub4e4\uc740 \uc790\ub9ac\ub294 \uc7a1\uc558\uc73c\ub098 1902\ub144\uc744 \uc804\ud6c4\ud574\uc11c \uc0ac\ub77c\uc84c\ub2e4. \uc774\ud6c4 1940\ub144\ub300 \uc911\ubc18\uc5d0 \ubd81\uc11c\ubd80 \ud0dc\ud3c9\uc591 \uc5f0\uc548\uc5d0 \uc720\ub7fd\ucc0c\ub974\ub808\uae30\ub4e4\uc774 \ub098\ud0c0\ub0ac\ub294\ub370, \uc774\ub4e4\uc740 \uc544\ub9c8 1890\ub144 \uc13c\ud2b8\ub7f4 \ud30c\ud06c\uc5d0\uc11c \ubc29\uc0ac\ud55c \uac1c\uccb4\uad70\uc758 \ud6c4\uc608\ub85c \ucd94\uce21\ub41c\ub2e4. 60\ub9c8\ub9ac\uac00 \uc774\uc81c 1\uc5b5 5000\ub9cc \ub9c8\ub9ac\ub85c \ubd88\uc5b4\ub098\uc11c \uce90\ub098\ub2e4 \ub0a8\ubd80\uc5d0\uc11c \uc54c\ub798\uc2a4\uce74, \uc911\uc559\uc544\uba54\ub9ac\uce74\uc5d0 \uc774\ub974\ub294 \uc11c\uc2dd\uc9c0\ub97c \uc810\uc720\ud558\uace0 \uc788\ub2e4.", "answer": "\ub450 \ubc88", "sentence": "\ubd81\uc544\uba54\ub9ac\uce74\uc5d0 \uc720\ub7fd\ucc0c\ub974\ub808\uae30\ub97c \ub3c4\uc785\ud558\ub294 \uc77c\uc740 \ub450 \ubc88 \uc2e4\ud328\ud588\ub2e4.", "paragraph_sentence": " \ubd81\uc544\uba54\ub9ac\uce74\uc5d0 \uc720\ub7fd\ucc0c\ub974\ub808\uae30\ub97c \ub3c4\uc785\ud558\ub294 \uc77c\uc740 \ub450 \ubc88 \uc2e4\ud328\ud588\ub2e4. \uc774\ud6c4 1890\ub144 \ub274\uc695 \uc13c\ud2b8\ub7f4 \ud30c\ud06c\uc5d0\uc11c 60\ub9c8\ub9ac\uc758 \uc720\ub7fd\ucc0c\ub974\ub808\uae30\uac00 \ubc29\uc0ac\ub418\uc5c8\ub2e4. \uc774\ub97c \ucd94\uc9c4\ud55c \ubbf8\uad6d \uc801\uc751\ud654 \ud611\ud68c \ud611\ud68c\uc7a5 \uc720\uc9c4 \uc170\ud3a0\ub9b0\uc740 \uc70c\ub9ac\uc5c4 \uc170\uc775\uc2a4\ud53c\uc5b4\uc758 \ud76c\uace1\ub4e4\uc5d0 \ub098\uc624\ub294 \uc0c8\ub4e4\uc744 \ubaa8\ub450 \ubd81\ubbf8\uc5d0 \ub3c4\uc785\ud558\ub824\uace0 \ud558\ub358 \uc790\uc600\ub2e4. \uac19\uc740 \ub0a0 \ud3ec\ud2c0\ub79c\ub4dc \uba85\uae08 \uad6c\ub77d\ubd80\uac00 \uc624\ub9ac\uac74 \ud3ec\ud2c0\ub79c\ub4dc\uc5d0\uc11c \uc720\ub7fd\ucc0c\ub974\ub808\uae30 35\uc30d\uc744 \ubc29\uc0ac\ud588\ub2e4. \uc774 \uc0c8\ub4e4\uc740 \uc790\ub9ac\ub294 \uc7a1\uc558\uc73c\ub098 1902\ub144\uc744 \uc804\ud6c4\ud574\uc11c \uc0ac\ub77c\uc84c\ub2e4. \uc774\ud6c4 1940\ub144\ub300 \uc911\ubc18\uc5d0 \ubd81\uc11c\ubd80 \ud0dc\ud3c9\uc591 \uc5f0\uc548\uc5d0 \uc720\ub7fd\ucc0c\ub974\ub808\uae30\ub4e4\uc774 \ub098\ud0c0\ub0ac\ub294\ub370, \uc774\ub4e4\uc740 \uc544\ub9c8 1890\ub144 \uc13c\ud2b8\ub7f4 \ud30c\ud06c\uc5d0\uc11c \ubc29\uc0ac\ud55c \uac1c\uccb4\uad70\uc758 \ud6c4\uc608\ub85c \ucd94\uce21\ub41c\ub2e4. 60\ub9c8\ub9ac\uac00 \uc774\uc81c 1\uc5b5 5000\ub9cc \ub9c8\ub9ac\ub85c \ubd88\uc5b4\ub098\uc11c \uce90\ub098\ub2e4 \ub0a8\ubd80\uc5d0\uc11c \uc54c\ub798\uc2a4\uce74, \uc911\uc559\uc544\uba54\ub9ac\uce74\uc5d0 \uc774\ub974\ub294 \uc11c\uc2dd\uc9c0\ub97c \uc810\uc720\ud558\uace0 \uc788\ub2e4.", "paragraph_answer": "\ubd81\uc544\uba54\ub9ac\uce74\uc5d0 \uc720\ub7fd\ucc0c\ub974\ub808\uae30\ub97c \ub3c4\uc785\ud558\ub294 \uc77c\uc740 \ub450 \ubc88 \uc2e4\ud328\ud588\ub2e4. \uc774\ud6c4 1890\ub144 \ub274\uc695 \uc13c\ud2b8\ub7f4 \ud30c\ud06c\uc5d0\uc11c 60\ub9c8\ub9ac\uc758 \uc720\ub7fd\ucc0c\ub974\ub808\uae30\uac00 \ubc29\uc0ac\ub418\uc5c8\ub2e4. \uc774\ub97c \ucd94\uc9c4\ud55c \ubbf8\uad6d \uc801\uc751\ud654 \ud611\ud68c \ud611\ud68c\uc7a5 \uc720\uc9c4 \uc170\ud3a0\ub9b0\uc740 \uc70c\ub9ac\uc5c4 \uc170\uc775\uc2a4\ud53c\uc5b4\uc758 \ud76c\uace1\ub4e4\uc5d0 \ub098\uc624\ub294 \uc0c8\ub4e4\uc744 \ubaa8\ub450 \ubd81\ubbf8\uc5d0 \ub3c4\uc785\ud558\ub824\uace0 \ud558\ub358 \uc790\uc600\ub2e4. \uac19\uc740 \ub0a0 \ud3ec\ud2c0\ub79c\ub4dc \uba85\uae08 \uad6c\ub77d\ubd80\uac00 \uc624\ub9ac\uac74 \ud3ec\ud2c0\ub79c\ub4dc\uc5d0\uc11c \uc720\ub7fd\ucc0c\ub974\ub808\uae30 35\uc30d\uc744 \ubc29\uc0ac\ud588\ub2e4. \uc774 \uc0c8\ub4e4\uc740 \uc790\ub9ac\ub294 \uc7a1\uc558\uc73c\ub098 1902\ub144\uc744 \uc804\ud6c4\ud574\uc11c \uc0ac\ub77c\uc84c\ub2e4. \uc774\ud6c4 1940\ub144\ub300 \uc911\ubc18\uc5d0 \ubd81\uc11c\ubd80 \ud0dc\ud3c9\uc591 \uc5f0\uc548\uc5d0 \uc720\ub7fd\ucc0c\ub974\ub808\uae30\ub4e4\uc774 \ub098\ud0c0\ub0ac\ub294\ub370, \uc774\ub4e4\uc740 \uc544\ub9c8 1890\ub144 \uc13c\ud2b8\ub7f4 \ud30c\ud06c\uc5d0\uc11c \ubc29\uc0ac\ud55c \uac1c\uccb4\uad70\uc758 \ud6c4\uc608\ub85c \ucd94\uce21\ub41c\ub2e4. 60\ub9c8\ub9ac\uac00 \uc774\uc81c 1\uc5b5 5000\ub9cc \ub9c8\ub9ac\ub85c \ubd88\uc5b4\ub098\uc11c \uce90\ub098\ub2e4 \ub0a8\ubd80\uc5d0\uc11c \uc54c\ub798\uc2a4\uce74, \uc911\uc559\uc544\uba54\ub9ac\uce74\uc5d0 \uc774\ub974\ub294 \uc11c\uc2dd\uc9c0\ub97c \uc810\uc720\ud558\uace0 \uc788\ub2e4.", "sentence_answer": "\ubd81\uc544\uba54\ub9ac\uce74\uc5d0 \uc720\ub7fd\ucc0c\ub974\ub808\uae30\ub97c \ub3c4\uc785\ud558\ub294 \uc77c\uc740 \ub450 \ubc88 \uc2e4\ud328\ud588\ub2e4.", "paragraph_id": "6464640-22-1", "paragraph_question": "question: \ubd81\uc544\uba54\ub9ac\uce74\uc5d0 \uc720\ub7fd\ucc0c\ub974\ub808\uae30\ub97c \ub3c4\uc785\ud558\ub294 \uc77c\uc740 \uba87 \ubc88\uc758 \uc2e4\ud328\uac00 \uc788\uc5c8\ub294\uac00?, context: \ubd81\uc544\uba54\ub9ac\uce74\uc5d0 \uc720\ub7fd\ucc0c\ub974\ub808\uae30\ub97c \ub3c4\uc785\ud558\ub294 \uc77c\uc740 \ub450 \ubc88 \uc2e4\ud328\ud588\ub2e4. \uc774\ud6c4 1890\ub144 \ub274\uc695 \uc13c\ud2b8\ub7f4 \ud30c\ud06c\uc5d0\uc11c 60\ub9c8\ub9ac\uc758 \uc720\ub7fd\ucc0c\ub974\ub808\uae30\uac00 \ubc29\uc0ac\ub418\uc5c8\ub2e4. \uc774\ub97c \ucd94\uc9c4\ud55c \ubbf8\uad6d \uc801\uc751\ud654 \ud611\ud68c \ud611\ud68c\uc7a5 \uc720\uc9c4 \uc170\ud3a0\ub9b0\uc740 \uc70c\ub9ac\uc5c4 \uc170\uc775\uc2a4\ud53c\uc5b4\uc758 \ud76c\uace1\ub4e4\uc5d0 \ub098\uc624\ub294 \uc0c8\ub4e4\uc744 \ubaa8\ub450 \ubd81\ubbf8\uc5d0 \ub3c4\uc785\ud558\ub824\uace0 \ud558\ub358 \uc790\uc600\ub2e4. \uac19\uc740 \ub0a0 \ud3ec\ud2c0\ub79c\ub4dc \uba85\uae08 \uad6c\ub77d\ubd80\uac00 \uc624\ub9ac\uac74 \ud3ec\ud2c0\ub79c\ub4dc\uc5d0\uc11c \uc720\ub7fd\ucc0c\ub974\ub808\uae30 35\uc30d\uc744 \ubc29\uc0ac\ud588\ub2e4. \uc774 \uc0c8\ub4e4\uc740 \uc790\ub9ac\ub294 \uc7a1\uc558\uc73c\ub098 1902\ub144\uc744 \uc804\ud6c4\ud574\uc11c \uc0ac\ub77c\uc84c\ub2e4. \uc774\ud6c4 1940\ub144\ub300 \uc911\ubc18\uc5d0 \ubd81\uc11c\ubd80 \ud0dc\ud3c9\uc591 \uc5f0\uc548\uc5d0 \uc720\ub7fd\ucc0c\ub974\ub808\uae30\ub4e4\uc774 \ub098\ud0c0\ub0ac\ub294\ub370, \uc774\ub4e4\uc740 \uc544\ub9c8 1890\ub144 \uc13c\ud2b8\ub7f4 \ud30c\ud06c\uc5d0\uc11c \ubc29\uc0ac\ud55c \uac1c\uccb4\uad70\uc758 \ud6c4\uc608\ub85c \ucd94\uce21\ub41c\ub2e4. 60\ub9c8\ub9ac\uac00 \uc774\uc81c 1\uc5b5 5000\ub9cc \ub9c8\ub9ac\ub85c \ubd88\uc5b4\ub098\uc11c \uce90\ub098\ub2e4 \ub0a8\ubd80\uc5d0\uc11c \uc54c\ub798\uc2a4\uce74, \uc911\uc559\uc544\uba54\ub9ac\uce74\uc5d0 \uc774\ub974\ub294 \uc11c\uc2dd\uc9c0\ub97c \uc810\uc720\ud558\uace0 \uc788\ub2e4."} {"question": "\uc870\uc120\ubbfc\uc8fc\uc8fc\uc758\uc778\ubbfc\uacf5\ud654\uad6d\uc740 \uc8fc\uccb4\uc0ac\uc0c1\uc758 \uac15\ud654 \uc774\ud6c4\uc5d0\ub294 \uc624\ub298\ub0a0\uc758 \uc5b4\ub514\uac00 \uace0\uc870\uc120\uc758 \ub3c4\uc74d\uc9c0\ub77c\uace0 \uc8fc\uc7a5\ud558\uc600\ub098?", "paragraph": "\uace0\uc870\uc120\uc758 \ub3c4\uc74d\uc9c0\ub294 \uc5ec\ub7ec \ucc28\ub840 \uc774\ub3d9\ud55c \uac83\uc73c\ub85c \uae30\ub85d\ub418\uc5b4 \uc788\ub2e4. \u300a\uc0bc\uad6d\uc720\uc0ac\u300b\ub294 \u300a\uace0\uae30\u300b\ub97c \uc778\uc6a9\ud558\uc5ec, \ub2e8\uad70\uc655\uac80\uc774 \ucc98\uc74c\uc5d0\ub294 \ud3c9\uc591\uc131\uc5d0 \ub3c4\uc74d\uc744 \uc815\ud558\uc600\uc73c\ub098 \uc774\ud6c4 \ubc31\uc545\uc0b0\uc544\uc0ac\ub2ec\ub85c \uc62e\uaca8\uc11c 1\ucc9c 5\ubc31 \ub144\uac04 \ub098\ub77c\ub97c \ub2e4\uc2a4\ub838\uc73c\uba70, \uc774\ud6c4 \uc8fc\ub098\ub77c \ub54c \uae30\uc790\uac00 \uc870\uc120\uc655\uc5d0 \ucc45\ubd09\ub418\uc790, \ub2e8\uad70\uc740 \uc7a5\ub2f9\uacbd\uc73c\ub85c \uc62e\uacbc\ub2e4\uac00 \ub4a4\uc5d0 \uc544\uc0ac\ub2ec\ub85c \ub3cc\uc544\uc654\ub2e4\uace0 \ud558\uc600\ub2e4. \uadf8\ub7ec\ub098 \uc774\ub7ec\ud55c \uc2e0\ud654 \uc18d\uc758 \uc218\ub3c4 \ubcc0\ucc9c\uc740 \uc2e4\uc81c \uc0ac\uc2e4\ub85c \ubcf4\uc9c0 \uc54a\ub294 \uac83\uc774 \ub300\ud55c\ubbfc\uad6d \uc5ed\uc0ac\ud559\uacc4\uc758 \uc77c\ubc18\uc801\uc778 \uacac\ud574\uc774\ub2e4. \uc870\uc120\ubbfc\uc8fc\uc8fc\uc758\uc778\ubbfc\uacf5\ud654\uad6d\uc740 1970\ub144\ub300 \uc774\uc804\uae4c\uc9c0 \uace0\uc870\uc120\uc758 \ub3c4\uc74d\uc9c0\ub97c \ub7b4\uc624\ub2dd \uc131\uc774\ub77c \uc8fc\uc7a5\ud558\uc600\uc73c\ub098, \uc8fc\uccb4\uc0ac\uc0c1\uc758 \uac15\ud654 \uc774\ud6c4\uc5d0\ub294 \uc624\ub298\ub0a0\uc758 \ud3c9\uc591\uc2dc\uac00 \uace0\uc870\uc120\uc758 \ub3c4\uc74d\uc9c0\ub77c\uace0 \uc8fc\uc7a5\ud558\uba74\uc11c \ub2e8\uad70\ub989\uc774 \ud3c9\uc591\uc2dc\uc5d0 \uc788\ub2e4\ub294 \uc810\uc744 \ub0b4\uc138\uc6b0\uace0 \uc788\ub2e4. \uc774\ub7ec\ud55c \uc870\uc120\ubbfc\uc8fc\uc8fc\uc758\uc778\ubbfc\uacf5\ud654\uad6d \uc5ed\uc0ac\ud559\uc758 \uc785\uc7a5 \ubcc0\ud654\ub294 \uc815\uce58\uc801 \uc601\ud5a5\uc5d0 \ub530\ub978 \uac83\uc774\ub77c \ube44\ud310\ub41c\ub2e4. \ud55c\ud3b8 \uc724\ub0b4\ud604\uc740 \uace0\uc870\uc120\uc758 \ub3c4\uc74d\uc9c0 \uc774\ub3d9\uc774 \ucd1d 5\ucc28\ub840\ub77c\uace0 \uc8fc\uc7a5\ud558\uba70 \uadf8 \uc704\uce58\ub97c \ubaa8\ub450 \ube44\uc815\ud558\ub294 \uc5f0\uad6c\ub97c \ud558\uae30\ub3c4 \ud558\uc600\ub2e4.", "answer": "\ud3c9\uc591\uc2dc", "sentence": "\uc8fc\uccb4\uc0ac\uc0c1\uc758 \uac15\ud654 \uc774\ud6c4\uc5d0\ub294 \uc624\ub298\ub0a0\uc758 \ud3c9\uc591\uc2dc \uac00 \uace0\uc870\uc120\uc758 \ub3c4\uc74d\uc9c0\ub77c\uace0 \uc8fc\uc7a5\ud558\uba74\uc11c \ub2e8\uad70\ub989\uc774 \ud3c9\uc591\uc2dc\uc5d0 \uc788\ub2e4\ub294 \uc810\uc744 \ub0b4\uc138\uc6b0\uace0 \uc788\ub2e4.", "paragraph_sentence": "\uace0\uc870\uc120\uc758 \ub3c4\uc74d\uc9c0\ub294 \uc5ec\ub7ec \ucc28\ub840 \uc774\ub3d9\ud55c \uac83\uc73c\ub85c \uae30\ub85d\ub418\uc5b4 \uc788\ub2e4. \u300a\uc0bc\uad6d\uc720\uc0ac\u300b\ub294 \u300a\uace0\uae30\u300b\ub97c \uc778\uc6a9\ud558\uc5ec, \ub2e8\uad70\uc655\uac80\uc774 \ucc98\uc74c\uc5d0\ub294 \ud3c9\uc591\uc131\uc5d0 \ub3c4\uc74d\uc744 \uc815\ud558\uc600\uc73c\ub098 \uc774\ud6c4 \ubc31\uc545\uc0b0\uc544\uc0ac\ub2ec\ub85c \uc62e\uaca8\uc11c 1\ucc9c 5\ubc31 \ub144\uac04 \ub098\ub77c\ub97c \ub2e4\uc2a4\ub838\uc73c\uba70, \uc774\ud6c4 \uc8fc\ub098\ub77c \ub54c \uae30\uc790\uac00 \uc870\uc120\uc655\uc5d0 \ucc45\ubd09\ub418\uc790, \ub2e8\uad70\uc740 \uc7a5\ub2f9\uacbd\uc73c\ub85c \uc62e\uacbc\ub2e4\uac00 \ub4a4\uc5d0 \uc544\uc0ac\ub2ec\ub85c \ub3cc\uc544\uc654\ub2e4\uace0 \ud558\uc600\ub2e4. \uadf8\ub7ec\ub098 \uc774\ub7ec\ud55c \uc2e0\ud654 \uc18d\uc758 \uc218\ub3c4 \ubcc0\ucc9c\uc740 \uc2e4\uc81c \uc0ac\uc2e4\ub85c \ubcf4\uc9c0 \uc54a\ub294 \uac83\uc774 \ub300\ud55c\ubbfc\uad6d \uc5ed\uc0ac\ud559\uacc4\uc758 \uc77c\ubc18\uc801\uc778 \uacac\ud574\uc774\ub2e4. \uc870\uc120\ubbfc\uc8fc\uc8fc\uc758\uc778\ubbfc\uacf5\ud654\uad6d\uc740 1970\ub144\ub300 \uc774\uc804\uae4c\uc9c0 \uace0\uc870\uc120\uc758 \ub3c4\uc74d\uc9c0\ub97c \ub7b4\uc624\ub2dd \uc131\uc774\ub77c \uc8fc\uc7a5\ud558\uc600\uc73c\ub098, \uc8fc\uccb4\uc0ac\uc0c1\uc758 \uac15\ud654 \uc774\ud6c4\uc5d0\ub294 \uc624\ub298\ub0a0\uc758 \ud3c9\uc591\uc2dc \uac00 \uace0\uc870\uc120\uc758 \ub3c4\uc74d\uc9c0\ub77c\uace0 \uc8fc\uc7a5\ud558\uba74\uc11c \ub2e8\uad70\ub989\uc774 \ud3c9\uc591\uc2dc\uc5d0 \uc788\ub2e4\ub294 \uc810\uc744 \ub0b4\uc138\uc6b0\uace0 \uc788\ub2e4. \uc774\ub7ec\ud55c \uc870\uc120\ubbfc\uc8fc\uc8fc\uc758\uc778\ubbfc\uacf5\ud654\uad6d \uc5ed\uc0ac\ud559\uc758 \uc785\uc7a5 \ubcc0\ud654\ub294 \uc815\uce58\uc801 \uc601\ud5a5\uc5d0 \ub530\ub978 \uac83\uc774\ub77c \ube44\ud310\ub41c\ub2e4. \ud55c\ud3b8 \uc724\ub0b4\ud604\uc740 \uace0\uc870\uc120\uc758 \ub3c4\uc74d\uc9c0 \uc774\ub3d9\uc774 \ucd1d 5\ucc28\ub840\ub77c\uace0 \uc8fc\uc7a5\ud558\uba70 \uadf8 \uc704\uce58\ub97c \ubaa8\ub450 \ube44\uc815\ud558\ub294 \uc5f0\uad6c\ub97c \ud558\uae30\ub3c4 \ud558\uc600\ub2e4.", "paragraph_answer": "\uace0\uc870\uc120\uc758 \ub3c4\uc74d\uc9c0\ub294 \uc5ec\ub7ec \ucc28\ub840 \uc774\ub3d9\ud55c \uac83\uc73c\ub85c \uae30\ub85d\ub418\uc5b4 \uc788\ub2e4. \u300a\uc0bc\uad6d\uc720\uc0ac\u300b\ub294 \u300a\uace0\uae30\u300b\ub97c \uc778\uc6a9\ud558\uc5ec, \ub2e8\uad70\uc655\uac80\uc774 \ucc98\uc74c\uc5d0\ub294 \ud3c9\uc591\uc131\uc5d0 \ub3c4\uc74d\uc744 \uc815\ud558\uc600\uc73c\ub098 \uc774\ud6c4 \ubc31\uc545\uc0b0\uc544\uc0ac\ub2ec\ub85c \uc62e\uaca8\uc11c 1\ucc9c 5\ubc31 \ub144\uac04 \ub098\ub77c\ub97c \ub2e4\uc2a4\ub838\uc73c\uba70, \uc774\ud6c4 \uc8fc\ub098\ub77c \ub54c \uae30\uc790\uac00 \uc870\uc120\uc655\uc5d0 \ucc45\ubd09\ub418\uc790, \ub2e8\uad70\uc740 \uc7a5\ub2f9\uacbd\uc73c\ub85c \uc62e\uacbc\ub2e4\uac00 \ub4a4\uc5d0 \uc544\uc0ac\ub2ec\ub85c \ub3cc\uc544\uc654\ub2e4\uace0 \ud558\uc600\ub2e4. \uadf8\ub7ec\ub098 \uc774\ub7ec\ud55c \uc2e0\ud654 \uc18d\uc758 \uc218\ub3c4 \ubcc0\ucc9c\uc740 \uc2e4\uc81c \uc0ac\uc2e4\ub85c \ubcf4\uc9c0 \uc54a\ub294 \uac83\uc774 \ub300\ud55c\ubbfc\uad6d \uc5ed\uc0ac\ud559\uacc4\uc758 \uc77c\ubc18\uc801\uc778 \uacac\ud574\uc774\ub2e4. \uc870\uc120\ubbfc\uc8fc\uc8fc\uc758\uc778\ubbfc\uacf5\ud654\uad6d\uc740 1970\ub144\ub300 \uc774\uc804\uae4c\uc9c0 \uace0\uc870\uc120\uc758 \ub3c4\uc74d\uc9c0\ub97c \ub7b4\uc624\ub2dd \uc131\uc774\ub77c \uc8fc\uc7a5\ud558\uc600\uc73c\ub098, \uc8fc\uccb4\uc0ac\uc0c1\uc758 \uac15\ud654 \uc774\ud6c4\uc5d0\ub294 \uc624\ub298\ub0a0\uc758 \ud3c9\uc591\uc2dc \uac00 \uace0\uc870\uc120\uc758 \ub3c4\uc74d\uc9c0\ub77c\uace0 \uc8fc\uc7a5\ud558\uba74\uc11c \ub2e8\uad70\ub989\uc774 \ud3c9\uc591\uc2dc\uc5d0 \uc788\ub2e4\ub294 \uc810\uc744 \ub0b4\uc138\uc6b0\uace0 \uc788\ub2e4. \uc774\ub7ec\ud55c \uc870\uc120\ubbfc\uc8fc\uc8fc\uc758\uc778\ubbfc\uacf5\ud654\uad6d \uc5ed\uc0ac\ud559\uc758 \uc785\uc7a5 \ubcc0\ud654\ub294 \uc815\uce58\uc801 \uc601\ud5a5\uc5d0 \ub530\ub978 \uac83\uc774\ub77c \ube44\ud310\ub41c\ub2e4. \ud55c\ud3b8 \uc724\ub0b4\ud604\uc740 \uace0\uc870\uc120\uc758 \ub3c4\uc74d\uc9c0 \uc774\ub3d9\uc774 \ucd1d 5\ucc28\ub840\ub77c\uace0 \uc8fc\uc7a5\ud558\uba70 \uadf8 \uc704\uce58\ub97c \ubaa8\ub450 \ube44\uc815\ud558\ub294 \uc5f0\uad6c\ub97c \ud558\uae30\ub3c4 \ud558\uc600\ub2e4.", "sentence_answer": "\uc8fc\uccb4\uc0ac\uc0c1\uc758 \uac15\ud654 \uc774\ud6c4\uc5d0\ub294 \uc624\ub298\ub0a0\uc758 \ud3c9\uc591\uc2dc \uac00 \uace0\uc870\uc120\uc758 \ub3c4\uc74d\uc9c0\ub77c\uace0 \uc8fc\uc7a5\ud558\uba74\uc11c \ub2e8\uad70\ub989\uc774 \ud3c9\uc591\uc2dc\uc5d0 \uc788\ub2e4\ub294 \uc810\uc744 \ub0b4\uc138\uc6b0\uace0 \uc788\ub2e4.", "paragraph_id": "6471479-7-1", "paragraph_question": "question: \uc870\uc120\ubbfc\uc8fc\uc8fc\uc758\uc778\ubbfc\uacf5\ud654\uad6d\uc740 \uc8fc\uccb4\uc0ac\uc0c1\uc758 \uac15\ud654 \uc774\ud6c4\uc5d0\ub294 \uc624\ub298\ub0a0\uc758 \uc5b4\ub514\uac00 \uace0\uc870\uc120\uc758 \ub3c4\uc74d\uc9c0\ub77c\uace0 \uc8fc\uc7a5\ud558\uc600\ub098?, context: \uace0\uc870\uc120\uc758 \ub3c4\uc74d\uc9c0\ub294 \uc5ec\ub7ec \ucc28\ub840 \uc774\ub3d9\ud55c \uac83\uc73c\ub85c \uae30\ub85d\ub418\uc5b4 \uc788\ub2e4. \u300a\uc0bc\uad6d\uc720\uc0ac\u300b\ub294 \u300a\uace0\uae30\u300b\ub97c \uc778\uc6a9\ud558\uc5ec, \ub2e8\uad70\uc655\uac80\uc774 \ucc98\uc74c\uc5d0\ub294 \ud3c9\uc591\uc131\uc5d0 \ub3c4\uc74d\uc744 \uc815\ud558\uc600\uc73c\ub098 \uc774\ud6c4 \ubc31\uc545\uc0b0\uc544\uc0ac\ub2ec\ub85c \uc62e\uaca8\uc11c 1\ucc9c 5\ubc31 \ub144\uac04 \ub098\ub77c\ub97c \ub2e4\uc2a4\ub838\uc73c\uba70, \uc774\ud6c4 \uc8fc\ub098\ub77c \ub54c \uae30\uc790\uac00 \uc870\uc120\uc655\uc5d0 \ucc45\ubd09\ub418\uc790, \ub2e8\uad70\uc740 \uc7a5\ub2f9\uacbd\uc73c\ub85c \uc62e\uacbc\ub2e4\uac00 \ub4a4\uc5d0 \uc544\uc0ac\ub2ec\ub85c \ub3cc\uc544\uc654\ub2e4\uace0 \ud558\uc600\ub2e4. \uadf8\ub7ec\ub098 \uc774\ub7ec\ud55c \uc2e0\ud654 \uc18d\uc758 \uc218\ub3c4 \ubcc0\ucc9c\uc740 \uc2e4\uc81c \uc0ac\uc2e4\ub85c \ubcf4\uc9c0 \uc54a\ub294 \uac83\uc774 \ub300\ud55c\ubbfc\uad6d \uc5ed\uc0ac\ud559\uacc4\uc758 \uc77c\ubc18\uc801\uc778 \uacac\ud574\uc774\ub2e4. \uc870\uc120\ubbfc\uc8fc\uc8fc\uc758\uc778\ubbfc\uacf5\ud654\uad6d\uc740 1970\ub144\ub300 \uc774\uc804\uae4c\uc9c0 \uace0\uc870\uc120\uc758 \ub3c4\uc74d\uc9c0\ub97c \ub7b4\uc624\ub2dd \uc131\uc774\ub77c \uc8fc\uc7a5\ud558\uc600\uc73c\ub098, \uc8fc\uccb4\uc0ac\uc0c1\uc758 \uac15\ud654 \uc774\ud6c4\uc5d0\ub294 \uc624\ub298\ub0a0\uc758 \ud3c9\uc591\uc2dc\uac00 \uace0\uc870\uc120\uc758 \ub3c4\uc74d\uc9c0\ub77c\uace0 \uc8fc\uc7a5\ud558\uba74\uc11c \ub2e8\uad70\ub989\uc774 \ud3c9\uc591\uc2dc\uc5d0 \uc788\ub2e4\ub294 \uc810\uc744 \ub0b4\uc138\uc6b0\uace0 \uc788\ub2e4. \uc774\ub7ec\ud55c \uc870\uc120\ubbfc\uc8fc\uc8fc\uc758\uc778\ubbfc\uacf5\ud654\uad6d \uc5ed\uc0ac\ud559\uc758 \uc785\uc7a5 \ubcc0\ud654\ub294 \uc815\uce58\uc801 \uc601\ud5a5\uc5d0 \ub530\ub978 \uac83\uc774\ub77c \ube44\ud310\ub41c\ub2e4. \ud55c\ud3b8 \uc724\ub0b4\ud604\uc740 \uace0\uc870\uc120\uc758 \ub3c4\uc74d\uc9c0 \uc774\ub3d9\uc774 \ucd1d 5\ucc28\ub840\ub77c\uace0 \uc8fc\uc7a5\ud558\uba70 \uadf8 \uc704\uce58\ub97c \ubaa8\ub450 \ube44\uc815\ud558\ub294 \uc5f0\uad6c\ub97c \ud558\uae30\ub3c4 \ud558\uc600\ub2e4."} {"question": "\ub9c1\ucee8 \uc554\uc0b4 \uc774\ud6c4 \uc544\ud504\ub9ac\uce74\uacc4 \ubbf8\uad6d \uc2dc\ubbfc\ub4e4\uc758 \uc2dc\ubbfc\uad8c\uc744 \ubc15\ud0c8\ud55c \ubc95\uc740 \ubb34\uc5c7\uc778\uac00?", "paragraph": "\ub9c1\ucee8\uc774 \uc554\uc0b4\ub2f9\ud558\uba74\uc11c, \uc0c8\ub85c\uc774 \ud574\ubc29\ub41c \ub178\uc608\ub4e4\uc758 \uad8c\ub9ac\ub97c \ubcf4\uc7a5\ud558\ub294 \ud55c\ud3b8 \ub0a8\ubd80 \uc8fc\ub97c \uc7ac\uac74\ud558\uace0 \uc7ac\ud1b5\ud569\ud558\ub294 \ubaa9\ud45c\ub85c \ud588\ub358 \uacf5\ud654\ub2f9\uc758 \uc7ac\uac74 \uc815\ucc45\uc774 \uae09\uc9c4\ud654\ub418\uc5c8\ub2e4. 1876\ub144 \ub300\uc120 \ub17c\uc7c1\uc774 1877\ub144\uc758 \ud569\uc758\ub85c \ud574\uacb0\ub418\uba74\uc11c \ubbf8\uad6d\uc758 \uc7ac\uac74 \uc815\ucc45\uc774 \uc885\uc2dd\ub418\uc5c8\uc73c\uba70, \uc774\ub0b4 \uc9d0 \ud06c\ub85c \ubc95\uc73c\ub85c \uc218\ub9ce\uc740 \uc544\ud504\ub9ac\uce74\uacc4 \ubbf8\uad6d \uc2dc\ubbfc\ub4e4\uc774 \uc2dc\ubbfc\uad8c\uc744 \ubc15\ud0c8\ub2f9\ud558\uc600\ub2e4. \ubd81\ubd80\uc5d0\uc11c\ub294 \ub3c4\uc2dc\ud654\uac00 \uc774\ub8e8\uc5b4\uc9c0\uace0 \ub0a8\uc720\ub7fd\uacfc \ub3d9\uc720\ub7fd\uc5d0\uc11c \uc804\ub840\uc5c6\uc774 \ub9ce\uc740 \uc774\ubbfc\uc790\ub4e4\uc774 \uc720\uc785\ub418\uba74\uc11c \ubbf8\uad6d\uc758 \uc0b0\uc5c5\ud654\uac00 \ub354\uc6b1 \uc9c4\uc804\ud558\uc600\ub2e4. 1929\ub144\uae4c\uc9c0 \uc9c0\uc18d\ub41c \uc774\ubbfc \ubb3c\uacb0\ub85c \ub178\ub3d9\ub825\uc774 \uc720\uc785\ub418\uace0 \ubbf8\uad6d \ubb38\ud654\uac00 \ubcc0\ud654\ud558\uc600\ub2e4. \uad6d\uac00 \uae30\ubc18 \uc2dc\uc124\uc774 \ubc1c\uc804\ud558\uba74\uc11c \uacbd\uc81c\ub294 \ud638\ud669\uc744 \uc774\ub8e8\uac8c\ub418\uc5c8\ub2e4. 1867\ub144 \ub7ec\uc2dc\uc544\uc5d0\uc11c \uc54c\ub798\uc2a4\uce74\ub97c \ub9e4\uc785\ud558\uba74\uc11c \ubbf8\uad6d \ubcf8\ud1a0\uc758 \uc601\ud1a0 \ud655\uc7a5\uc774 \uc77c\ub2e8\ub77d\ub418\uc5c8\ub2e4. \uc601\ud1a0\ub97c \uc783\uc740 \uc6d0\uc8fc\ubbfc\uc774 1890\ub144 \uc6b4\ub514\ub4dc \ub2c8 \ud559\uc0b4\ub85c \ucd5c\ud6c4\uc758 \ub300\uaddc\ubaa8 \ubb34\uc7a5 \ud22c\uc7c1\uc744 \ubc8c\uc600\uc73c\ub098 \uc2e4\ud328\ud588\ub2e4. 1893\ub144, \ud0dc\ud3c9\uc591 \ud558\uc640\uc774 \uc655\uad6d\uc758 \uc655\uc815\uc774 \ubbf8\uad6d \uac70\ub958\ubbfc\ub4e4\uc774 \uc8fc\ub3d9\ud55c \uc815\ubcc0\uc73c\ub85c \uc804\ubcf5\ub418\uc790, 1898\ub144\uc5d0 \ubbf8\uad6d\uc740 \uc774 \uad70\ub3c4\ub97c \ubcd1\ud569\ud588\ub2e4. \uac19\uc740 \ud574 \ubbf8\uad6d-\uc2a4\ud398\uc778 \uc804\uc7c1\uc5d0\uc11c \uc2b9\ub9ac\ud558\uba74\uc11c \ubbf8\uad6d\uc740 \uac15\ub300\uad6d\uc758 \uba74\ubaa8\ub97c \ubcf4\uc774\uac8c \ub418\uc5c8\uc73c\uba70, \ud478\uc5d0\ub974\ud1a0\ub9ac\ucf54, \uad0c, \ud544\ub9ac\ud540\uc744 \ubcd1\ud569\ud588\ub2e4. (50\uc5ec\ub144 \ub4a4\uc5d0 \ud544\ub9ac\ud540\uc740 \ubbf8\uad6d\uc5d0\uc11c \ub3c5\ub9bd\ud588\uc73c\ub098, \ud478\uc5d0\ub974\ud1a0\ub9ac\ucf54\uc640 \uad0c\uc740 \ubbf8\uad6d \uc601\ud1a0\ub85c \ub0a8\uc544\uc788\ub2e4.)", "answer": "\uc9d0 \ud06c\ub85c \ubc95", "sentence": "\uc774\ub0b4 \uc9d0 \ud06c\ub85c \ubc95 \uc73c\ub85c \uc218\ub9ce\uc740 \uc544\ud504\ub9ac\uce74\uacc4 \ubbf8\uad6d \uc2dc\ubbfc\ub4e4\uc774 \uc2dc\ubbfc\uad8c\uc744 \ubc15\ud0c8\ub2f9\ud558\uc600\ub2e4.", "paragraph_sentence": "\ub9c1\ucee8\uc774 \uc554\uc0b4\ub2f9\ud558\uba74\uc11c, \uc0c8\ub85c\uc774 \ud574\ubc29\ub41c \ub178\uc608\ub4e4\uc758 \uad8c\ub9ac\ub97c \ubcf4\uc7a5\ud558\ub294 \ud55c\ud3b8 \ub0a8\ubd80 \uc8fc\ub97c \uc7ac\uac74\ud558\uace0 \uc7ac\ud1b5\ud569\ud558\ub294 \ubaa9\ud45c\ub85c \ud588\ub358 \uacf5\ud654\ub2f9\uc758 \uc7ac\uac74 \uc815\ucc45\uc774 \uae09\uc9c4\ud654\ub418\uc5c8\ub2e4. 1876\ub144 \ub300\uc120 \ub17c\uc7c1\uc774 1877\ub144\uc758 \ud569\uc758\ub85c \ud574\uacb0\ub418\uba74\uc11c \ubbf8\uad6d\uc758 \uc7ac\uac74 \uc815\ucc45\uc774 \uc885\uc2dd\ub418\uc5c8\uc73c\uba70, \uc774\ub0b4 \uc9d0 \ud06c\ub85c \ubc95 \uc73c\ub85c \uc218\ub9ce\uc740 \uc544\ud504\ub9ac\uce74\uacc4 \ubbf8\uad6d \uc2dc\ubbfc\ub4e4\uc774 \uc2dc\ubbfc\uad8c\uc744 \ubc15\ud0c8\ub2f9\ud558\uc600\ub2e4. \ubd81\ubd80\uc5d0\uc11c\ub294 \ub3c4\uc2dc\ud654\uac00 \uc774\ub8e8\uc5b4\uc9c0\uace0 \ub0a8\uc720\ub7fd\uacfc \ub3d9\uc720\ub7fd\uc5d0\uc11c \uc804\ub840\uc5c6\uc774 \ub9ce\uc740 \uc774\ubbfc\uc790\ub4e4\uc774 \uc720\uc785\ub418\uba74\uc11c \ubbf8\uad6d\uc758 \uc0b0\uc5c5\ud654\uac00 \ub354\uc6b1 \uc9c4\uc804\ud558\uc600\ub2e4. 1929\ub144\uae4c\uc9c0 \uc9c0\uc18d\ub41c \uc774\ubbfc \ubb3c\uacb0\ub85c \ub178\ub3d9\ub825\uc774 \uc720\uc785\ub418\uace0 \ubbf8\uad6d \ubb38\ud654\uac00 \ubcc0\ud654\ud558\uc600\ub2e4. \uad6d\uac00 \uae30\ubc18 \uc2dc\uc124\uc774 \ubc1c\uc804\ud558\uba74\uc11c \uacbd\uc81c\ub294 \ud638\ud669\uc744 \uc774\ub8e8\uac8c\ub418\uc5c8\ub2e4. 1867\ub144 \ub7ec\uc2dc\uc544\uc5d0\uc11c \uc54c\ub798\uc2a4\uce74\ub97c \ub9e4\uc785\ud558\uba74\uc11c \ubbf8\uad6d \ubcf8\ud1a0\uc758 \uc601\ud1a0 \ud655\uc7a5\uc774 \uc77c\ub2e8\ub77d\ub418\uc5c8\ub2e4. \uc601\ud1a0\ub97c \uc783\uc740 \uc6d0\uc8fc\ubbfc\uc774 1890\ub144 \uc6b4\ub514\ub4dc \ub2c8 \ud559\uc0b4\ub85c \ucd5c\ud6c4\uc758 \ub300\uaddc\ubaa8 \ubb34\uc7a5 \ud22c\uc7c1\uc744 \ubc8c\uc600\uc73c\ub098 \uc2e4\ud328\ud588\ub2e4. 1893\ub144, \ud0dc\ud3c9\uc591 \ud558\uc640\uc774 \uc655\uad6d\uc758 \uc655\uc815\uc774 \ubbf8\uad6d \uac70\ub958\ubbfc\ub4e4\uc774 \uc8fc\ub3d9\ud55c \uc815\ubcc0\uc73c\ub85c \uc804\ubcf5\ub418\uc790, 1898\ub144\uc5d0 \ubbf8\uad6d\uc740 \uc774 \uad70\ub3c4\ub97c \ubcd1\ud569\ud588\ub2e4. \uac19\uc740 \ud574 \ubbf8\uad6d-\uc2a4\ud398\uc778 \uc804\uc7c1\uc5d0\uc11c \uc2b9\ub9ac\ud558\uba74\uc11c \ubbf8\uad6d\uc740 \uac15\ub300\uad6d\uc758 \uba74\ubaa8\ub97c \ubcf4\uc774\uac8c \ub418\uc5c8\uc73c\uba70, \ud478\uc5d0\ub974\ud1a0\ub9ac\ucf54, \uad0c, \ud544\ub9ac\ud540\uc744 \ubcd1\ud569\ud588\ub2e4. (50\uc5ec\ub144 \ub4a4\uc5d0 \ud544\ub9ac\ud540\uc740 \ubbf8\uad6d\uc5d0\uc11c \ub3c5\ub9bd\ud588\uc73c\ub098, \ud478\uc5d0\ub974\ud1a0\ub9ac\ucf54\uc640 \uad0c\uc740 \ubbf8\uad6d \uc601\ud1a0\ub85c \ub0a8\uc544\uc788\ub2e4.)", "paragraph_answer": "\ub9c1\ucee8\uc774 \uc554\uc0b4\ub2f9\ud558\uba74\uc11c, \uc0c8\ub85c\uc774 \ud574\ubc29\ub41c \ub178\uc608\ub4e4\uc758 \uad8c\ub9ac\ub97c \ubcf4\uc7a5\ud558\ub294 \ud55c\ud3b8 \ub0a8\ubd80 \uc8fc\ub97c \uc7ac\uac74\ud558\uace0 \uc7ac\ud1b5\ud569\ud558\ub294 \ubaa9\ud45c\ub85c \ud588\ub358 \uacf5\ud654\ub2f9\uc758 \uc7ac\uac74 \uc815\ucc45\uc774 \uae09\uc9c4\ud654\ub418\uc5c8\ub2e4. 1876\ub144 \ub300\uc120 \ub17c\uc7c1\uc774 1877\ub144\uc758 \ud569\uc758\ub85c \ud574\uacb0\ub418\uba74\uc11c \ubbf8\uad6d\uc758 \uc7ac\uac74 \uc815\ucc45\uc774 \uc885\uc2dd\ub418\uc5c8\uc73c\uba70, \uc774\ub0b4 \uc9d0 \ud06c\ub85c \ubc95 \uc73c\ub85c \uc218\ub9ce\uc740 \uc544\ud504\ub9ac\uce74\uacc4 \ubbf8\uad6d \uc2dc\ubbfc\ub4e4\uc774 \uc2dc\ubbfc\uad8c\uc744 \ubc15\ud0c8\ub2f9\ud558\uc600\ub2e4. \ubd81\ubd80\uc5d0\uc11c\ub294 \ub3c4\uc2dc\ud654\uac00 \uc774\ub8e8\uc5b4\uc9c0\uace0 \ub0a8\uc720\ub7fd\uacfc \ub3d9\uc720\ub7fd\uc5d0\uc11c \uc804\ub840\uc5c6\uc774 \ub9ce\uc740 \uc774\ubbfc\uc790\ub4e4\uc774 \uc720\uc785\ub418\uba74\uc11c \ubbf8\uad6d\uc758 \uc0b0\uc5c5\ud654\uac00 \ub354\uc6b1 \uc9c4\uc804\ud558\uc600\ub2e4. 1929\ub144\uae4c\uc9c0 \uc9c0\uc18d\ub41c \uc774\ubbfc \ubb3c\uacb0\ub85c \ub178\ub3d9\ub825\uc774 \uc720\uc785\ub418\uace0 \ubbf8\uad6d \ubb38\ud654\uac00 \ubcc0\ud654\ud558\uc600\ub2e4. \uad6d\uac00 \uae30\ubc18 \uc2dc\uc124\uc774 \ubc1c\uc804\ud558\uba74\uc11c \uacbd\uc81c\ub294 \ud638\ud669\uc744 \uc774\ub8e8\uac8c\ub418\uc5c8\ub2e4. 1867\ub144 \ub7ec\uc2dc\uc544\uc5d0\uc11c \uc54c\ub798\uc2a4\uce74\ub97c \ub9e4\uc785\ud558\uba74\uc11c \ubbf8\uad6d \ubcf8\ud1a0\uc758 \uc601\ud1a0 \ud655\uc7a5\uc774 \uc77c\ub2e8\ub77d\ub418\uc5c8\ub2e4. \uc601\ud1a0\ub97c \uc783\uc740 \uc6d0\uc8fc\ubbfc\uc774 1890\ub144 \uc6b4\ub514\ub4dc \ub2c8 \ud559\uc0b4\ub85c \ucd5c\ud6c4\uc758 \ub300\uaddc\ubaa8 \ubb34\uc7a5 \ud22c\uc7c1\uc744 \ubc8c\uc600\uc73c\ub098 \uc2e4\ud328\ud588\ub2e4. 1893\ub144, \ud0dc\ud3c9\uc591 \ud558\uc640\uc774 \uc655\uad6d\uc758 \uc655\uc815\uc774 \ubbf8\uad6d \uac70\ub958\ubbfc\ub4e4\uc774 \uc8fc\ub3d9\ud55c \uc815\ubcc0\uc73c\ub85c \uc804\ubcf5\ub418\uc790, 1898\ub144\uc5d0 \ubbf8\uad6d\uc740 \uc774 \uad70\ub3c4\ub97c \ubcd1\ud569\ud588\ub2e4. \uac19\uc740 \ud574 \ubbf8\uad6d-\uc2a4\ud398\uc778 \uc804\uc7c1\uc5d0\uc11c \uc2b9\ub9ac\ud558\uba74\uc11c \ubbf8\uad6d\uc740 \uac15\ub300\uad6d\uc758 \uba74\ubaa8\ub97c \ubcf4\uc774\uac8c \ub418\uc5c8\uc73c\uba70, \ud478\uc5d0\ub974\ud1a0\ub9ac\ucf54, \uad0c, \ud544\ub9ac\ud540\uc744 \ubcd1\ud569\ud588\ub2e4. (50\uc5ec\ub144 \ub4a4\uc5d0 \ud544\ub9ac\ud540\uc740 \ubbf8\uad6d\uc5d0\uc11c \ub3c5\ub9bd\ud588\uc73c\ub098, \ud478\uc5d0\ub974\ud1a0\ub9ac\ucf54\uc640 \uad0c\uc740 \ubbf8\uad6d \uc601\ud1a0\ub85c \ub0a8\uc544\uc788\ub2e4.)", "sentence_answer": "\uc774\ub0b4 \uc9d0 \ud06c\ub85c \ubc95 \uc73c\ub85c \uc218\ub9ce\uc740 \uc544\ud504\ub9ac\uce74\uacc4 \ubbf8\uad6d \uc2dc\ubbfc\ub4e4\uc774 \uc2dc\ubbfc\uad8c\uc744 \ubc15\ud0c8\ub2f9\ud558\uc600\ub2e4.", "paragraph_id": "6514865-10-0", "paragraph_question": "question: \ub9c1\ucee8 \uc554\uc0b4 \uc774\ud6c4 \uc544\ud504\ub9ac\uce74\uacc4 \ubbf8\uad6d \uc2dc\ubbfc\ub4e4\uc758 \uc2dc\ubbfc\uad8c\uc744 \ubc15\ud0c8\ud55c \ubc95\uc740 \ubb34\uc5c7\uc778\uac00?, context: \ub9c1\ucee8\uc774 \uc554\uc0b4\ub2f9\ud558\uba74\uc11c, \uc0c8\ub85c\uc774 \ud574\ubc29\ub41c \ub178\uc608\ub4e4\uc758 \uad8c\ub9ac\ub97c \ubcf4\uc7a5\ud558\ub294 \ud55c\ud3b8 \ub0a8\ubd80 \uc8fc\ub97c \uc7ac\uac74\ud558\uace0 \uc7ac\ud1b5\ud569\ud558\ub294 \ubaa9\ud45c\ub85c \ud588\ub358 \uacf5\ud654\ub2f9\uc758 \uc7ac\uac74 \uc815\ucc45\uc774 \uae09\uc9c4\ud654\ub418\uc5c8\ub2e4. 1876\ub144 \ub300\uc120 \ub17c\uc7c1\uc774 1877\ub144\uc758 \ud569\uc758\ub85c \ud574\uacb0\ub418\uba74\uc11c \ubbf8\uad6d\uc758 \uc7ac\uac74 \uc815\ucc45\uc774 \uc885\uc2dd\ub418\uc5c8\uc73c\uba70, \uc774\ub0b4 \uc9d0 \ud06c\ub85c \ubc95\uc73c\ub85c \uc218\ub9ce\uc740 \uc544\ud504\ub9ac\uce74\uacc4 \ubbf8\uad6d \uc2dc\ubbfc\ub4e4\uc774 \uc2dc\ubbfc\uad8c\uc744 \ubc15\ud0c8\ub2f9\ud558\uc600\ub2e4. \ubd81\ubd80\uc5d0\uc11c\ub294 \ub3c4\uc2dc\ud654\uac00 \uc774\ub8e8\uc5b4\uc9c0\uace0 \ub0a8\uc720\ub7fd\uacfc \ub3d9\uc720\ub7fd\uc5d0\uc11c \uc804\ub840\uc5c6\uc774 \ub9ce\uc740 \uc774\ubbfc\uc790\ub4e4\uc774 \uc720\uc785\ub418\uba74\uc11c \ubbf8\uad6d\uc758 \uc0b0\uc5c5\ud654\uac00 \ub354\uc6b1 \uc9c4\uc804\ud558\uc600\ub2e4. 1929\ub144\uae4c\uc9c0 \uc9c0\uc18d\ub41c \uc774\ubbfc \ubb3c\uacb0\ub85c \ub178\ub3d9\ub825\uc774 \uc720\uc785\ub418\uace0 \ubbf8\uad6d \ubb38\ud654\uac00 \ubcc0\ud654\ud558\uc600\ub2e4. \uad6d\uac00 \uae30\ubc18 \uc2dc\uc124\uc774 \ubc1c\uc804\ud558\uba74\uc11c \uacbd\uc81c\ub294 \ud638\ud669\uc744 \uc774\ub8e8\uac8c\ub418\uc5c8\ub2e4. 1867\ub144 \ub7ec\uc2dc\uc544\uc5d0\uc11c \uc54c\ub798\uc2a4\uce74\ub97c \ub9e4\uc785\ud558\uba74\uc11c \ubbf8\uad6d \ubcf8\ud1a0\uc758 \uc601\ud1a0 \ud655\uc7a5\uc774 \uc77c\ub2e8\ub77d\ub418\uc5c8\ub2e4. \uc601\ud1a0\ub97c \uc783\uc740 \uc6d0\uc8fc\ubbfc\uc774 1890\ub144 \uc6b4\ub514\ub4dc \ub2c8 \ud559\uc0b4\ub85c \ucd5c\ud6c4\uc758 \ub300\uaddc\ubaa8 \ubb34\uc7a5 \ud22c\uc7c1\uc744 \ubc8c\uc600\uc73c\ub098 \uc2e4\ud328\ud588\ub2e4. 1893\ub144, \ud0dc\ud3c9\uc591 \ud558\uc640\uc774 \uc655\uad6d\uc758 \uc655\uc815\uc774 \ubbf8\uad6d \uac70\ub958\ubbfc\ub4e4\uc774 \uc8fc\ub3d9\ud55c \uc815\ubcc0\uc73c\ub85c \uc804\ubcf5\ub418\uc790, 1898\ub144\uc5d0 \ubbf8\uad6d\uc740 \uc774 \uad70\ub3c4\ub97c \ubcd1\ud569\ud588\ub2e4. \uac19\uc740 \ud574 \ubbf8\uad6d-\uc2a4\ud398\uc778 \uc804\uc7c1\uc5d0\uc11c \uc2b9\ub9ac\ud558\uba74\uc11c \ubbf8\uad6d\uc740 \uac15\ub300\uad6d\uc758 \uba74\ubaa8\ub97c \ubcf4\uc774\uac8c \ub418\uc5c8\uc73c\uba70, \ud478\uc5d0\ub974\ud1a0\ub9ac\ucf54, \uad0c, \ud544\ub9ac\ud540\uc744 \ubcd1\ud569\ud588\ub2e4. (50\uc5ec\ub144 \ub4a4\uc5d0 \ud544\ub9ac\ud540\uc740 \ubbf8\uad6d\uc5d0\uc11c \ub3c5\ub9bd\ud588\uc73c\ub098, \ud478\uc5d0\ub974\ud1a0\ub9ac\ucf54\uc640 \uad0c\uc740 \ubbf8\uad6d \uc601\ud1a0\ub85c \ub0a8\uc544\uc788\ub2e4.)"} {"question": "\ucf00\ud50c\ub7ec\uac00 \ud560\uc544\ubc84\uc9c0\uc5d0\uac8c \ubb3c\ub824\ubc1b\uc740 \uac83\uc740?", "paragraph": "1595\ub144 12\uc6d4, \ucf00\ud50c\ub7ec\ub294 23\uc0b4\uc758 \ub538 \ub538\ub9b0 \uacfc\ubd80 \ubc14\ubc14\ub77c \ubb90\ub7ec\ub97c \uc18c\uac1c\ubc1b\uace0 \uc5f0\uc560\ud558\uae30 \uc2dc\uc791\ud588\ub2e4. \ubb90\ub7ec\ub294 \uc804\ub0a8\ud3b8\uc758 \uc7ac\uc0b0\ub4e4\uc744 \uc0c1\uc18d\ubc1b\uac8c \ub418\uc5b4 \uc788\uc5c8\uace0, \uacbd\uc81c\uc801\uc73c\ub85c \uc720\ubcf5\ud55c \ubc29\uc557\uac04 \uc8fc\uc778\uc758 \ub538\uc774\uc5c8\ub2e4. \uadf8\ub140\uc758 \uc544\ubc84\uc9c0 \uc6a5\uc2a4\ud2b8\ub294 \ucf00\ud50c\ub7ec\uc758 \uc2e0\ubd84\uc5d0\ub3c4 \ubd88\uad6c\ud558\uace0 \ucc98\uc74c\ubd80\ud130 \uacb0\ud63c\uc744 \ubc18\ub300\ud588\ub2e4. \ucf00\ud50c\ub7ec\ub294 \ud560\uc544\ubc84\uc9c0\uc758 \uc2e0\ubd84\uc744 \ubb3c\ub824\ubc1b\uae34 \ud588\uc9c0\ub9cc \uadf8\uc758 \uac00\ub09c\uc740 \uc2e0\ubd84\uacfc \uc5b4\uc6b8\ub9ac\uc9c0 \uc54a\uc558\ub2e4. \uc6a5\uc2a4\ud2b8\ub294 \ucf00\ud50c\ub7ec\uac00 \u300a\uc6b0\uc8fc\uad6c\uc870\uc758 \uc2e0\ube44\u300b\uc791\uc5c5\uc744 \ub9c8\uce58\uace0 \ub098\uc790 \ub9c8\uc74c\uc774 \ub204\uadf8\ub7ec\uc84c\uc73c\ub098 \ucf00\ud50c\ub7ec\uac00 \ucc45 \ucd9c\ud310\uc758 \uc138\uc138\ud55c \uc77c\uc5d0 \uc2e0\uacbd\uc744 \uc4f0\ub294 \ub3d9\uc548 \uc57d\ud63c\uc740 \uac70\uc758 \ud30c\uae30 \uc9c1\uc804\uae4c\uc9c0 \uac14\ub2e4. \uadf8\ub7ec\ub098 \uc57d\ud63c\uc774 \uc131\uc0ac\ub418\ub3c4\ub85d \ub3c4\uc654\ub358 \uad50\ud68c \uc0ac\ub78c\ub4e4\uc774 \ubb90\ub7ec\uc5d0\uac8c \uc57d\uc18d\uc744 \uc9c0\ud0a4\ub77c\uace0 \uc555\ub825\uc744 \ub123\uc5c8\uace0 \ubb90\ub7ec\uc640 \ucf00\ud50c\ub7ec\ub294 1597\ub144 4\uc6d4 27\uc77c \uacb0\ud63c\ud588\ub2e4.", "answer": "\uc2e0\ubd84", "sentence": "\uadf8\ub140\uc758 \uc544\ubc84\uc9c0 \uc6a5\uc2a4\ud2b8\ub294 \ucf00\ud50c\ub7ec\uc758 \uc2e0\ubd84 \uc5d0\ub3c4 \ubd88\uad6c\ud558\uace0 \ucc98\uc74c\ubd80\ud130 \uacb0\ud63c\uc744 \ubc18\ub300\ud588\ub2e4.", "paragraph_sentence": "1595\ub144 12\uc6d4, \ucf00\ud50c\ub7ec\ub294 23\uc0b4\uc758 \ub538 \ub538\ub9b0 \uacfc\ubd80 \ubc14\ubc14\ub77c \ubb90\ub7ec\ub97c \uc18c\uac1c\ubc1b\uace0 \uc5f0\uc560\ud558\uae30 \uc2dc\uc791\ud588\ub2e4. \ubb90\ub7ec\ub294 \uc804\ub0a8\ud3b8\uc758 \uc7ac\uc0b0\ub4e4\uc744 \uc0c1\uc18d\ubc1b\uac8c \ub418\uc5b4 \uc788\uc5c8\uace0, \uacbd\uc81c\uc801\uc73c\ub85c \uc720\ubcf5\ud55c \ubc29\uc557\uac04 \uc8fc\uc778\uc758 \ub538\uc774\uc5c8\ub2e4. \uadf8\ub140\uc758 \uc544\ubc84\uc9c0 \uc6a5\uc2a4\ud2b8\ub294 \ucf00\ud50c\ub7ec\uc758 \uc2e0\ubd84 \uc5d0\ub3c4 \ubd88\uad6c\ud558\uace0 \ucc98\uc74c\ubd80\ud130 \uacb0\ud63c\uc744 \ubc18\ub300\ud588\ub2e4. \ucf00\ud50c\ub7ec\ub294 \ud560\uc544\ubc84\uc9c0\uc758 \uc2e0\ubd84\uc744 \ubb3c\ub824\ubc1b\uae34 \ud588\uc9c0\ub9cc \uadf8\uc758 \uac00\ub09c\uc740 \uc2e0\ubd84\uacfc \uc5b4\uc6b8\ub9ac\uc9c0 \uc54a\uc558\ub2e4. \uc6a5\uc2a4\ud2b8\ub294 \ucf00\ud50c\ub7ec\uac00 \u300a\uc6b0\uc8fc\uad6c\uc870\uc758 \uc2e0\ube44\u300b\uc791\uc5c5\uc744 \ub9c8\uce58\uace0 \ub098\uc790 \ub9c8\uc74c\uc774 \ub204\uadf8\ub7ec\uc84c\uc73c\ub098 \ucf00\ud50c\ub7ec\uac00 \ucc45 \ucd9c\ud310\uc758 \uc138\uc138\ud55c \uc77c\uc5d0 \uc2e0\uacbd\uc744 \uc4f0\ub294 \ub3d9\uc548 \uc57d\ud63c\uc740 \uac70\uc758 \ud30c\uae30 \uc9c1\uc804\uae4c\uc9c0 \uac14\ub2e4. \uadf8\ub7ec\ub098 \uc57d\ud63c\uc774 \uc131\uc0ac\ub418\ub3c4\ub85d \ub3c4\uc654\ub358 \uad50\ud68c \uc0ac\ub78c\ub4e4\uc774 \ubb90\ub7ec\uc5d0\uac8c \uc57d\uc18d\uc744 \uc9c0\ud0a4\ub77c\uace0 \uc555\ub825\uc744 \ub123\uc5c8\uace0 \ubb90\ub7ec\uc640 \ucf00\ud50c\ub7ec\ub294 1597\ub144 4\uc6d4 27\uc77c \uacb0\ud63c\ud588\ub2e4.", "paragraph_answer": "1595\ub144 12\uc6d4, \ucf00\ud50c\ub7ec\ub294 23\uc0b4\uc758 \ub538 \ub538\ub9b0 \uacfc\ubd80 \ubc14\ubc14\ub77c \ubb90\ub7ec\ub97c \uc18c\uac1c\ubc1b\uace0 \uc5f0\uc560\ud558\uae30 \uc2dc\uc791\ud588\ub2e4. \ubb90\ub7ec\ub294 \uc804\ub0a8\ud3b8\uc758 \uc7ac\uc0b0\ub4e4\uc744 \uc0c1\uc18d\ubc1b\uac8c \ub418\uc5b4 \uc788\uc5c8\uace0, \uacbd\uc81c\uc801\uc73c\ub85c \uc720\ubcf5\ud55c \ubc29\uc557\uac04 \uc8fc\uc778\uc758 \ub538\uc774\uc5c8\ub2e4. \uadf8\ub140\uc758 \uc544\ubc84\uc9c0 \uc6a5\uc2a4\ud2b8\ub294 \ucf00\ud50c\ub7ec\uc758 \uc2e0\ubd84 \uc5d0\ub3c4 \ubd88\uad6c\ud558\uace0 \ucc98\uc74c\ubd80\ud130 \uacb0\ud63c\uc744 \ubc18\ub300\ud588\ub2e4. \ucf00\ud50c\ub7ec\ub294 \ud560\uc544\ubc84\uc9c0\uc758 \uc2e0\ubd84\uc744 \ubb3c\ub824\ubc1b\uae34 \ud588\uc9c0\ub9cc \uadf8\uc758 \uac00\ub09c\uc740 \uc2e0\ubd84\uacfc \uc5b4\uc6b8\ub9ac\uc9c0 \uc54a\uc558\ub2e4. \uc6a5\uc2a4\ud2b8\ub294 \ucf00\ud50c\ub7ec\uac00 \u300a\uc6b0\uc8fc\uad6c\uc870\uc758 \uc2e0\ube44\u300b\uc791\uc5c5\uc744 \ub9c8\uce58\uace0 \ub098\uc790 \ub9c8\uc74c\uc774 \ub204\uadf8\ub7ec\uc84c\uc73c\ub098 \ucf00\ud50c\ub7ec\uac00 \ucc45 \ucd9c\ud310\uc758 \uc138\uc138\ud55c \uc77c\uc5d0 \uc2e0\uacbd\uc744 \uc4f0\ub294 \ub3d9\uc548 \uc57d\ud63c\uc740 \uac70\uc758 \ud30c\uae30 \uc9c1\uc804\uae4c\uc9c0 \uac14\ub2e4. \uadf8\ub7ec\ub098 \uc57d\ud63c\uc774 \uc131\uc0ac\ub418\ub3c4\ub85d \ub3c4\uc654\ub358 \uad50\ud68c \uc0ac\ub78c\ub4e4\uc774 \ubb90\ub7ec\uc5d0\uac8c \uc57d\uc18d\uc744 \uc9c0\ud0a4\ub77c\uace0 \uc555\ub825\uc744 \ub123\uc5c8\uace0 \ubb90\ub7ec\uc640 \ucf00\ud50c\ub7ec\ub294 1597\ub144 4\uc6d4 27\uc77c \uacb0\ud63c\ud588\ub2e4.", "sentence_answer": "\uadf8\ub140\uc758 \uc544\ubc84\uc9c0 \uc6a5\uc2a4\ud2b8\ub294 \ucf00\ud50c\ub7ec\uc758 \uc2e0\ubd84 \uc5d0\ub3c4 \ubd88\uad6c\ud558\uace0 \ucc98\uc74c\ubd80\ud130 \uacb0\ud63c\uc744 \ubc18\ub300\ud588\ub2e4.", "paragraph_id": "6541867-4-2", "paragraph_question": "question: \ucf00\ud50c\ub7ec\uac00 \ud560\uc544\ubc84\uc9c0\uc5d0\uac8c \ubb3c\ub824\ubc1b\uc740 \uac83\uc740?, context: 1595\ub144 12\uc6d4, \ucf00\ud50c\ub7ec\ub294 23\uc0b4\uc758 \ub538 \ub538\ub9b0 \uacfc\ubd80 \ubc14\ubc14\ub77c \ubb90\ub7ec\ub97c \uc18c\uac1c\ubc1b\uace0 \uc5f0\uc560\ud558\uae30 \uc2dc\uc791\ud588\ub2e4. \ubb90\ub7ec\ub294 \uc804\ub0a8\ud3b8\uc758 \uc7ac\uc0b0\ub4e4\uc744 \uc0c1\uc18d\ubc1b\uac8c \ub418\uc5b4 \uc788\uc5c8\uace0, \uacbd\uc81c\uc801\uc73c\ub85c \uc720\ubcf5\ud55c \ubc29\uc557\uac04 \uc8fc\uc778\uc758 \ub538\uc774\uc5c8\ub2e4. \uadf8\ub140\uc758 \uc544\ubc84\uc9c0 \uc6a5\uc2a4\ud2b8\ub294 \ucf00\ud50c\ub7ec\uc758 \uc2e0\ubd84\uc5d0\ub3c4 \ubd88\uad6c\ud558\uace0 \ucc98\uc74c\ubd80\ud130 \uacb0\ud63c\uc744 \ubc18\ub300\ud588\ub2e4. \ucf00\ud50c\ub7ec\ub294 \ud560\uc544\ubc84\uc9c0\uc758 \uc2e0\ubd84\uc744 \ubb3c\ub824\ubc1b\uae34 \ud588\uc9c0\ub9cc \uadf8\uc758 \uac00\ub09c\uc740 \uc2e0\ubd84\uacfc \uc5b4\uc6b8\ub9ac\uc9c0 \uc54a\uc558\ub2e4. \uc6a5\uc2a4\ud2b8\ub294 \ucf00\ud50c\ub7ec\uac00 \u300a\uc6b0\uc8fc\uad6c\uc870\uc758 \uc2e0\ube44\u300b\uc791\uc5c5\uc744 \ub9c8\uce58\uace0 \ub098\uc790 \ub9c8\uc74c\uc774 \ub204\uadf8\ub7ec\uc84c\uc73c\ub098 \ucf00\ud50c\ub7ec\uac00 \ucc45 \ucd9c\ud310\uc758 \uc138\uc138\ud55c \uc77c\uc5d0 \uc2e0\uacbd\uc744 \uc4f0\ub294 \ub3d9\uc548 \uc57d\ud63c\uc740 \uac70\uc758 \ud30c\uae30 \uc9c1\uc804\uae4c\uc9c0 \uac14\ub2e4. \uadf8\ub7ec\ub098 \uc57d\ud63c\uc774 \uc131\uc0ac\ub418\ub3c4\ub85d \ub3c4\uc654\ub358 \uad50\ud68c \uc0ac\ub78c\ub4e4\uc774 \ubb90\ub7ec\uc5d0\uac8c \uc57d\uc18d\uc744 \uc9c0\ud0a4\ub77c\uace0 \uc555\ub825\uc744 \ub123\uc5c8\uace0 \ubb90\ub7ec\uc640 \ucf00\ud50c\ub7ec\ub294 1597\ub144 4\uc6d4 27\uc77c \uacb0\ud63c\ud588\ub2e4."} {"question": "\uace0\ub824\uc2dc\ub300 \ud0c0\uc778\uc758 \ud1a0\uc9c0\ub97c \ube4c\ub824 \uacbd\uc791\ud558\ub294 \uac83\uc744 \ubb34\uc5c7\uc774\ub77c\uace0 \ud558\ub294\uac00?", "paragraph": "\ud53c\uc9c0\ubc30\uce35\uc740 \uacf5\ubbfc(\u516c\u6c11)\uc778 \uc591\uc778(\u826f\u4eba) \uc911\uc758 \ud3c9\ubbfc\uce35, \uacf5\uc801\u00b7\uc0ac\uc801 \uad8c\ub9ac\uc758 \uc77c\ubd80\uac00 \ubc15\ud0c8\ub418\uc5b4 \uc5b4\ub290 \uc815\ub3c4 \ucc9c\ubbfc(\u8ce4\u6c11)\uac19\uc774 \ub300\uc6b0\ubc1b\uc740 \ud5a5\u00b7\ubd80\uace1 \ub4f1\uc758 \uc885\uc18d \uad6c\uc5ed\ubbfc, \uacf5\ubbfc\uc73c\ub85c\uc11c\uc758 \uc790\uaca9\uc774 \uc644\uc804\ud788 \ubc15\ud0c8\ub41c \ucc9c\ubbfc \ub4f1 \uc138 \uac00\uc9c0 \uc2e0\ubd84\uce35\uc73c\ub85c \uad6c\uc131\ub418\uc5c8\ub2e4. \uc790\uc720\ub85c\uc6b4 \uc2e0\ubd84\uc778 \uc591\uc778\uc758 \ub300\ub2e4\uc218\ub294 \ub18d\ubbfc\uc73c\ub85c, \uc774\ub4e4\uc740 \uc9c1\uc5ed\uc774 \uc5c6\ub294 \ubc31\uc815(\u767d\u4e01)\uc774\uc5c8\ub2e4. \uc774\ub4e4\uc740 \uc790\uae30 \uc18c\uc720\uc758 \uc18c\uaddc\ubaa8 \ub18d\uc9c0\ub97c \uacbd\uc791\ud558\uac70\ub098 \ud0c0\uc778\uc758 \ud1a0\uc9c0\ub97c \ube4c\ub824 \ucc28\uacbd(\u501f\u8015)\ud558\uc600\ub2e4. \ucc28\uacbd\uc744 \ud558\ub294 \uacbd\uc6b0 \uc774\ub4e4\uc740 \uc9c0\uc8fc\uc5d0\uac8c \uc218\ud655\ubb3c\uc758 2\ubd84\uc758 1\uc744 \uc9c0\ub300\ub85c \ubb3c\uc5c8\uc73c\uba70, \uc790\uc2e0\uc758 \ud1a0\uc9c0\ub97c \uacbd\uc791\ud558\ub354\ub77c\ub3c4 \uc870(\u79df)\ub85c\uc11c \uc0dd\uc0b0\ubb3c\uc758 10\ubd84\uc758 1\uc744 \uad6d\uac00\uae30\uad00\uc774\ub098 \uc804\uc2dc\uacfc\uc81c\ub3c4\uc5d0 \uc758\ud574 \uc9c0\uc815\ub41c \uc218\uc870\uad8c\uc790(\u6536\u79df\u6b0a\u8005)\uc5d0\uac8c \ubb3c\uc5b4\uc57c \ud588\ub2e4. \ubc31\uc815\uce35\uc740 \uc774\uc678\uc5d0\ub3c4 \uc9c0\ubc29\ubcc4\ub85c \ud560\ub2f9\ub41c \ubb3c\ud488\uc744 \uc9d5\uc218\ud558\ub294 \uacf5\ubd80(\u8ca2\u8ce6)\uc640 \ub178\ub3d9\ub825\uc744 \uc9d5\ubc1c\ud558\ub294 \uc5ed\uc5ed(\u529b\u5f79)\uc744 \ubd80\ub2f4\ud574\uc57c \ud588\ub2e4.", "answer": "\ucc28\uacbd", "sentence": "\uc774\ub4e4\uc740 \uc790\uae30 \uc18c\uc720\uc758 \uc18c\uaddc\ubaa8 \ub18d\uc9c0\ub97c \uacbd\uc791\ud558\uac70\ub098 \ud0c0\uc778\uc758 \ud1a0\uc9c0\ub97c \ube4c\ub824 \ucc28\uacbd (\u501f\u8015)\ud558\uc600\ub2e4.", "paragraph_sentence": "\ud53c\uc9c0\ubc30\uce35\uc740 \uacf5\ubbfc(\u516c\u6c11)\uc778 \uc591\uc778(\u826f\u4eba) \uc911\uc758 \ud3c9\ubbfc\uce35, \uacf5\uc801\u00b7\uc0ac\uc801 \uad8c\ub9ac\uc758 \uc77c\ubd80\uac00 \ubc15\ud0c8\ub418\uc5b4 \uc5b4\ub290 \uc815\ub3c4 \ucc9c\ubbfc(\u8ce4\u6c11)\uac19\uc774 \ub300\uc6b0\ubc1b\uc740 \ud5a5\u00b7\ubd80\uace1 \ub4f1\uc758 \uc885\uc18d \uad6c\uc5ed\ubbfc, \uacf5\ubbfc\uc73c\ub85c\uc11c\uc758 \uc790\uaca9\uc774 \uc644\uc804\ud788 \ubc15\ud0c8\ub41c \ucc9c\ubbfc \ub4f1 \uc138 \uac00\uc9c0 \uc2e0\ubd84\uce35\uc73c\ub85c \uad6c\uc131\ub418\uc5c8\ub2e4. \uc790\uc720\ub85c\uc6b4 \uc2e0\ubd84\uc778 \uc591\uc778\uc758 \ub300\ub2e4\uc218\ub294 \ub18d\ubbfc\uc73c\ub85c, \uc774\ub4e4\uc740 \uc9c1\uc5ed\uc774 \uc5c6\ub294 \ubc31\uc815(\u767d\u4e01)\uc774\uc5c8\ub2e4. \uc774\ub4e4\uc740 \uc790\uae30 \uc18c\uc720\uc758 \uc18c\uaddc\ubaa8 \ub18d\uc9c0\ub97c \uacbd\uc791\ud558\uac70\ub098 \ud0c0\uc778\uc758 \ud1a0\uc9c0\ub97c \ube4c\ub824 \ucc28\uacbd (\u501f\u8015)\ud558\uc600\ub2e4. \ucc28\uacbd\uc744 \ud558\ub294 \uacbd\uc6b0 \uc774\ub4e4\uc740 \uc9c0\uc8fc\uc5d0\uac8c \uc218\ud655\ubb3c\uc758 2\ubd84\uc758 1\uc744 \uc9c0\ub300\ub85c \ubb3c\uc5c8\uc73c\uba70, \uc790\uc2e0\uc758 \ud1a0\uc9c0\ub97c \uacbd\uc791\ud558\ub354\ub77c\ub3c4 \uc870(\u79df)\ub85c\uc11c \uc0dd\uc0b0\ubb3c\uc758 10\ubd84\uc758 1\uc744 \uad6d\uac00\uae30\uad00\uc774\ub098 \uc804\uc2dc\uacfc\uc81c\ub3c4\uc5d0 \uc758\ud574 \uc9c0\uc815\ub41c \uc218\uc870\uad8c\uc790(\u6536\u79df\u6b0a\u8005)\uc5d0\uac8c \ubb3c\uc5b4\uc57c \ud588\ub2e4. \ubc31\uc815\uce35\uc740 \uc774\uc678\uc5d0\ub3c4 \uc9c0\ubc29\ubcc4\ub85c \ud560\ub2f9\ub41c \ubb3c\ud488\uc744 \uc9d5\uc218\ud558\ub294 \uacf5\ubd80(\u8ca2\u8ce6)\uc640 \ub178\ub3d9\ub825\uc744 \uc9d5\ubc1c\ud558\ub294 \uc5ed\uc5ed(\u529b\u5f79)\uc744 \ubd80\ub2f4\ud574\uc57c \ud588\ub2e4.", "paragraph_answer": "\ud53c\uc9c0\ubc30\uce35\uc740 \uacf5\ubbfc(\u516c\u6c11)\uc778 \uc591\uc778(\u826f\u4eba) \uc911\uc758 \ud3c9\ubbfc\uce35, \uacf5\uc801\u00b7\uc0ac\uc801 \uad8c\ub9ac\uc758 \uc77c\ubd80\uac00 \ubc15\ud0c8\ub418\uc5b4 \uc5b4\ub290 \uc815\ub3c4 \ucc9c\ubbfc(\u8ce4\u6c11)\uac19\uc774 \ub300\uc6b0\ubc1b\uc740 \ud5a5\u00b7\ubd80\uace1 \ub4f1\uc758 \uc885\uc18d \uad6c\uc5ed\ubbfc, \uacf5\ubbfc\uc73c\ub85c\uc11c\uc758 \uc790\uaca9\uc774 \uc644\uc804\ud788 \ubc15\ud0c8\ub41c \ucc9c\ubbfc \ub4f1 \uc138 \uac00\uc9c0 \uc2e0\ubd84\uce35\uc73c\ub85c \uad6c\uc131\ub418\uc5c8\ub2e4. \uc790\uc720\ub85c\uc6b4 \uc2e0\ubd84\uc778 \uc591\uc778\uc758 \ub300\ub2e4\uc218\ub294 \ub18d\ubbfc\uc73c\ub85c, \uc774\ub4e4\uc740 \uc9c1\uc5ed\uc774 \uc5c6\ub294 \ubc31\uc815(\u767d\u4e01)\uc774\uc5c8\ub2e4. \uc774\ub4e4\uc740 \uc790\uae30 \uc18c\uc720\uc758 \uc18c\uaddc\ubaa8 \ub18d\uc9c0\ub97c \uacbd\uc791\ud558\uac70\ub098 \ud0c0\uc778\uc758 \ud1a0\uc9c0\ub97c \ube4c\ub824 \ucc28\uacbd (\u501f\u8015)\ud558\uc600\ub2e4. \ucc28\uacbd\uc744 \ud558\ub294 \uacbd\uc6b0 \uc774\ub4e4\uc740 \uc9c0\uc8fc\uc5d0\uac8c \uc218\ud655\ubb3c\uc758 2\ubd84\uc758 1\uc744 \uc9c0\ub300\ub85c \ubb3c\uc5c8\uc73c\uba70, \uc790\uc2e0\uc758 \ud1a0\uc9c0\ub97c \uacbd\uc791\ud558\ub354\ub77c\ub3c4 \uc870(\u79df)\ub85c\uc11c \uc0dd\uc0b0\ubb3c\uc758 10\ubd84\uc758 1\uc744 \uad6d\uac00\uae30\uad00\uc774\ub098 \uc804\uc2dc\uacfc\uc81c\ub3c4\uc5d0 \uc758\ud574 \uc9c0\uc815\ub41c \uc218\uc870\uad8c\uc790(\u6536\u79df\u6b0a\u8005)\uc5d0\uac8c \ubb3c\uc5b4\uc57c \ud588\ub2e4. \ubc31\uc815\uce35\uc740 \uc774\uc678\uc5d0\ub3c4 \uc9c0\ubc29\ubcc4\ub85c \ud560\ub2f9\ub41c \ubb3c\ud488\uc744 \uc9d5\uc218\ud558\ub294 \uacf5\ubd80(\u8ca2\u8ce6)\uc640 \ub178\ub3d9\ub825\uc744 \uc9d5\ubc1c\ud558\ub294 \uc5ed\uc5ed(\u529b\u5f79)\uc744 \ubd80\ub2f4\ud574\uc57c \ud588\ub2e4.", "sentence_answer": "\uc774\ub4e4\uc740 \uc790\uae30 \uc18c\uc720\uc758 \uc18c\uaddc\ubaa8 \ub18d\uc9c0\ub97c \uacbd\uc791\ud558\uac70\ub098 \ud0c0\uc778\uc758 \ud1a0\uc9c0\ub97c \ube4c\ub824 \ucc28\uacbd (\u501f\u8015)\ud558\uc600\ub2e4.", "paragraph_id": "6575700-2-2", "paragraph_question": "question: \uace0\ub824\uc2dc\ub300 \ud0c0\uc778\uc758 \ud1a0\uc9c0\ub97c \ube4c\ub824 \uacbd\uc791\ud558\ub294 \uac83\uc744 \ubb34\uc5c7\uc774\ub77c\uace0 \ud558\ub294\uac00?, context: \ud53c\uc9c0\ubc30\uce35\uc740 \uacf5\ubbfc(\u516c\u6c11)\uc778 \uc591\uc778(\u826f\u4eba) \uc911\uc758 \ud3c9\ubbfc\uce35, \uacf5\uc801\u00b7\uc0ac\uc801 \uad8c\ub9ac\uc758 \uc77c\ubd80\uac00 \ubc15\ud0c8\ub418\uc5b4 \uc5b4\ub290 \uc815\ub3c4 \ucc9c\ubbfc(\u8ce4\u6c11)\uac19\uc774 \ub300\uc6b0\ubc1b\uc740 \ud5a5\u00b7\ubd80\uace1 \ub4f1\uc758 \uc885\uc18d \uad6c\uc5ed\ubbfc, \uacf5\ubbfc\uc73c\ub85c\uc11c\uc758 \uc790\uaca9\uc774 \uc644\uc804\ud788 \ubc15\ud0c8\ub41c \ucc9c\ubbfc \ub4f1 \uc138 \uac00\uc9c0 \uc2e0\ubd84\uce35\uc73c\ub85c \uad6c\uc131\ub418\uc5c8\ub2e4. \uc790\uc720\ub85c\uc6b4 \uc2e0\ubd84\uc778 \uc591\uc778\uc758 \ub300\ub2e4\uc218\ub294 \ub18d\ubbfc\uc73c\ub85c, \uc774\ub4e4\uc740 \uc9c1\uc5ed\uc774 \uc5c6\ub294 \ubc31\uc815(\u767d\u4e01)\uc774\uc5c8\ub2e4. \uc774\ub4e4\uc740 \uc790\uae30 \uc18c\uc720\uc758 \uc18c\uaddc\ubaa8 \ub18d\uc9c0\ub97c \uacbd\uc791\ud558\uac70\ub098 \ud0c0\uc778\uc758 \ud1a0\uc9c0\ub97c \ube4c\ub824 \ucc28\uacbd(\u501f\u8015)\ud558\uc600\ub2e4. \ucc28\uacbd\uc744 \ud558\ub294 \uacbd\uc6b0 \uc774\ub4e4\uc740 \uc9c0\uc8fc\uc5d0\uac8c \uc218\ud655\ubb3c\uc758 2\ubd84\uc758 1\uc744 \uc9c0\ub300\ub85c \ubb3c\uc5c8\uc73c\uba70, \uc790\uc2e0\uc758 \ud1a0\uc9c0\ub97c \uacbd\uc791\ud558\ub354\ub77c\ub3c4 \uc870(\u79df)\ub85c\uc11c \uc0dd\uc0b0\ubb3c\uc758 10\ubd84\uc758 1\uc744 \uad6d\uac00\uae30\uad00\uc774\ub098 \uc804\uc2dc\uacfc\uc81c\ub3c4\uc5d0 \uc758\ud574 \uc9c0\uc815\ub41c \uc218\uc870\uad8c\uc790(\u6536\u79df\u6b0a\u8005)\uc5d0\uac8c \ubb3c\uc5b4\uc57c \ud588\ub2e4. \ubc31\uc815\uce35\uc740 \uc774\uc678\uc5d0\ub3c4 \uc9c0\ubc29\ubcc4\ub85c \ud560\ub2f9\ub41c \ubb3c\ud488\uc744 \uc9d5\uc218\ud558\ub294 \uacf5\ubd80(\u8ca2\u8ce6)\uc640 \ub178\ub3d9\ub825\uc744 \uc9d5\ubc1c\ud558\ub294 \uc5ed\uc5ed(\u529b\u5f79)\uc744 \ubd80\ub2f4\ud574\uc57c \ud588\ub2e4."} {"question": "\uc804\uc7a5 \uc0c1\ud669\uacfc \uc804 \uc720\uc800\uc758 \ud0ac, \ud30c\uad34 \ub4f1\uc758 \uc815\ubcf4\ub97c \ud655\uc778\ud558\ub294\ub370 \uc774\uc6a9\ud558\ub294 \ud0a4\ub294?", "paragraph": "Tab\ud0a4\ub97c \ub204\ub974\uba74 \uc804\uc7a5 \uc0c1\ud669\uc744 \uc54c \uc218 \uc788\uc73c\uba70 \uc801\uad70\uc744 \ud3ec\ud568\ud55c \ubaa8\ub4e0 \uc720\uc800\ub4e4\uc758 \ud0ac, \ud30c\uad34, \ub3c4\uc6c0, \ud14c\ud06c\ub2c9, \ucd1d\uc810\uc744 \ubcfc \uc218 \uc788\ub2e4. \ub610\ud55c \uc544\uc774\ud15c \ud15c\ud2b8\ub9ac\ub97c \ubcf4\ub294 \uae30\ub2a5\uc774 \ucd94\uac00\ub418\uc5b4\uc11c \uc544\uad70\uc758 \uacbd\uc6b0 \uac8c\uc784\uc2dc\uc791\uacfc \ub3d9\uc2dc\uc5d0 \uc544\uad70\uc758 \ud15c\ud2b8\ub9ac\ub97c \ud655\uc778\ud560 \uc218\ub294 \uc788\uc9c0\ub9cc \uc801\uad70\uc758 \uacbd\uc6b0 \uac8c\uc784\uc744 \uc2dc\uc791\ud558\uace0 5\ubd84\ub4a4\uc5d0 \uacf5\uac1c\ub41c\ub2e4. \ub2e4\uc2dc \ub9d0\ud574 34\ubd84 59\ucd08\uac00 \ub418\uba74 \ubaa8\ub4e0 \uc720\uc800\ub4e4\uc758 \uc7a5\ube44\uc544\uc774\ud15c \ud14c\ud06c\ud2b8\ub9ac\ub97c \ubcfc \uc218 \uc788\ub2e4\ub294 \uac83\uc774\ub2e4. \uadf8\ub7ec\ub098 \uad6c\uccb4\uc801\uc73c\ub85c \uc5b4\ub5a4 \uc544\uc774\ud15c\uc744 \ucc29\uc6a9\ud558\uc600\ub294\uc9c0\ub294 \ubcf4\uc5ec\uc8fc\uc9c0 \uc54a\ub294\ub2e4. \uc7a5\uac11, \ubaa8\uc790, \uac00\uc2b4, \ud5c8\ub9ac, \ub2e4\ub9ac, \ubc1c \uc7a5\ube44\uc758 \ud22c\uc790\ub808\ubca8\ub9cc\uc744 \uc54c \uc218 \uc788\uac8c \ub41c\ub2e4. `\ud0a4(Tab\ubc14\ub85c \uc704\uc5d0 \uc788\ub294 \ud0a4)\ub97c \ub204\ub974\uba74 \uc778\uac8c\uc784 \ucc44\ud305\ucc28\ub2e8 \ubc0f \ud074\ub79c\ucc44\ud305, \uadd3\uc18d\ub9d0\uc744 \ucc28\ub2e8\ud560 \uc218 \uc788\ub2e4. \ucd94\uac00\ub41c \uae30\ub2a5\uc73c\ub85c\ub294 \uc801\uad70\uc744 \ud3ec\ud568\ud55c \ubaa8\ub4e0 \uc720\uc800\ub4e4\uc758 \uc18c\ubaa8\uc544\uc774\ud15c\uacfc \ubaa9\uac78\uc774, \uc7a5\uc2e0\uad6c1, \uc7a5\uc2e0\uad6c2, \uc7a5\uc2e0\uad6c3, \uc7a5\uc2e0\uad6c4\uc758 \uc544\uc774\ud15c\uc744 \uc54c \uc218 \uc788\ub2e4. \uc704\uc5d0\uc11c \uc124\uba85\ud55c \uac83\uacfc \ub9c8\ucc2c\uac00\uc9c0\ub85c \uc544\uad70\uc740 \uac8c\uc784 \uc2dc\uc791\uacfc \ub3d9\uc2dc\uc5d0 \uc801\uad70\uc740 34\ubd84 59\ucd08\uac00 \ub418\ub294 \uc21c\uac04 \uacf5\uac1c\ub41c\ub2e4. Tab\ud0a4\uc640\ub294 \ub2e4\ub974\uac8c \uc5b4\ub5a4 \uc18c\ubaa8\uc544\uc774\ud15c\uc744 \ucc29\uc6a9\ud558\uace0 \uc788\ub294\uc9c0 \uad6c\uccb4\uc801\uc73c\ub85c \uc54c \uc218 \uc788\uc744 \ubfd0\ub9cc \uc544\ub2c8\ub77c \ub098\uba38\uc9c0 \uc7a5\ube44 \ubaa9, \uc7a5\uc2e0\uad6c \uc544\uc774\ud15c\uc758 \uad6c\uccb4\uc801\uc778 \uba85\uce6d\ub3c4 \uc54c \uc218 \uc788\ub2e4. \ub2e4\uc2dc \ub9d0\ud574 \uc774 \uc0ac\ub78c\uc774 \uc5d1\uc140\ub808\uc774\uc158\uc740 \uc4f0\ub294\uc9c0, \uce90\ub9ad\ud130 \uc804\uc6a9 \uce58\uba85\ud0b7 \uc774\ud399\ud2b8\ub97c \uc4f0\ub294\uc9c0, \ub610\ud55c \ubaa9 \uc7a5\ube44\ub294 \uc720\ub2c8\ud06c\uc778\uc9c0\ub97c \ubaa8\ub450 \uc54c \uc218 \uc788\ub2e4. \uc774 \uae30\ub2a5\uc5d0 \ub300\ud574\uc11c\ub294 \uc720\uc800\ub4e4\uc758 \ubc18\ubc1c\uc774 \uc0c1\ub2f9\ud788 \uc788\ub294 \ub4ef \ud558\uc9c0\ub9cc \uc810\ucc28 \ub204\uadf8\ub7ec\uc9c4 \uac83 \uac19\ub2e4.", "answer": "Tab\ud0a4", "sentence": "Tab\ud0a4 \ub97c \ub204\ub974\uba74 \uc804\uc7a5 \uc0c1\ud669\uc744 \uc54c \uc218 \uc788\uc73c\uba70 \uc801\uad70\uc744 \ud3ec\ud568\ud55c \ubaa8\ub4e0 \uc720\uc800\ub4e4\uc758 \ud0ac, \ud30c\uad34, \ub3c4\uc6c0, \ud14c\ud06c\ub2c9, \ucd1d\uc810\uc744 \ubcfc \uc218 \uc788\ub2e4.", "paragraph_sentence": " Tab\ud0a4 \ub97c \ub204\ub974\uba74 \uc804\uc7a5 \uc0c1\ud669\uc744 \uc54c \uc218 \uc788\uc73c\uba70 \uc801\uad70\uc744 \ud3ec\ud568\ud55c \ubaa8\ub4e0 \uc720\uc800\ub4e4\uc758 \ud0ac, \ud30c\uad34, \ub3c4\uc6c0, \ud14c\ud06c\ub2c9, \ucd1d\uc810\uc744 \ubcfc \uc218 \uc788\ub2e4. \ub610\ud55c \uc544\uc774\ud15c \ud15c\ud2b8\ub9ac\ub97c \ubcf4\ub294 \uae30\ub2a5\uc774 \ucd94\uac00\ub418\uc5b4\uc11c \uc544\uad70\uc758 \uacbd\uc6b0 \uac8c\uc784\uc2dc\uc791\uacfc \ub3d9\uc2dc\uc5d0 \uc544\uad70\uc758 \ud15c\ud2b8\ub9ac\ub97c \ud655\uc778\ud560 \uc218\ub294 \uc788\uc9c0\ub9cc \uc801\uad70\uc758 \uacbd\uc6b0 \uac8c\uc784\uc744 \uc2dc\uc791\ud558\uace0 5\ubd84\ub4a4\uc5d0 \uacf5\uac1c\ub41c\ub2e4. \ub2e4\uc2dc \ub9d0\ud574 34\ubd84 59\ucd08\uac00 \ub418\uba74 \ubaa8\ub4e0 \uc720\uc800\ub4e4\uc758 \uc7a5\ube44\uc544\uc774\ud15c \ud14c\ud06c\ud2b8\ub9ac\ub97c \ubcfc \uc218 \uc788\ub2e4\ub294 \uac83\uc774\ub2e4. \uadf8\ub7ec\ub098 \uad6c\uccb4\uc801\uc73c\ub85c \uc5b4\ub5a4 \uc544\uc774\ud15c\uc744 \ucc29\uc6a9\ud558\uc600\ub294\uc9c0\ub294 \ubcf4\uc5ec\uc8fc\uc9c0 \uc54a\ub294\ub2e4. \uc7a5\uac11, \ubaa8\uc790, \uac00\uc2b4, \ud5c8\ub9ac, \ub2e4\ub9ac, \ubc1c \uc7a5\ube44\uc758 \ud22c\uc790\ub808\ubca8\ub9cc\uc744 \uc54c \uc218 \uc788\uac8c \ub41c\ub2e4. `\ud0a4(Tab\ubc14\ub85c \uc704\uc5d0 \uc788\ub294 \ud0a4)\ub97c \ub204\ub974\uba74 \uc778\uac8c\uc784 \ucc44\ud305\ucc28\ub2e8 \ubc0f \ud074\ub79c\ucc44\ud305, \uadd3\uc18d\ub9d0\uc744 \ucc28\ub2e8\ud560 \uc218 \uc788\ub2e4. \ucd94\uac00\ub41c \uae30\ub2a5\uc73c\ub85c\ub294 \uc801\uad70\uc744 \ud3ec\ud568\ud55c \ubaa8\ub4e0 \uc720\uc800\ub4e4\uc758 \uc18c\ubaa8\uc544\uc774\ud15c\uacfc \ubaa9\uac78\uc774, \uc7a5\uc2e0\uad6c1, \uc7a5\uc2e0\uad6c2, \uc7a5\uc2e0\uad6c3, \uc7a5\uc2e0\uad6c4\uc758 \uc544\uc774\ud15c\uc744 \uc54c \uc218 \uc788\ub2e4. \uc704\uc5d0\uc11c \uc124\uba85\ud55c \uac83\uacfc \ub9c8\ucc2c\uac00\uc9c0\ub85c \uc544\uad70\uc740 \uac8c\uc784 \uc2dc\uc791\uacfc \ub3d9\uc2dc\uc5d0 \uc801\uad70\uc740 34\ubd84 59\ucd08\uac00 \ub418\ub294 \uc21c\uac04 \uacf5\uac1c\ub41c\ub2e4. Tab\ud0a4\uc640\ub294 \ub2e4\ub974\uac8c \uc5b4\ub5a4 \uc18c\ubaa8\uc544\uc774\ud15c\uc744 \ucc29\uc6a9\ud558\uace0 \uc788\ub294\uc9c0 \uad6c\uccb4\uc801\uc73c\ub85c \uc54c \uc218 \uc788\uc744 \ubfd0\ub9cc \uc544\ub2c8\ub77c \ub098\uba38\uc9c0 \uc7a5\ube44 \ubaa9, \uc7a5\uc2e0\uad6c \uc544\uc774\ud15c\uc758 \uad6c\uccb4\uc801\uc778 \uba85\uce6d\ub3c4 \uc54c \uc218 \uc788\ub2e4. \ub2e4\uc2dc \ub9d0\ud574 \uc774 \uc0ac\ub78c\uc774 \uc5d1\uc140\ub808\uc774\uc158\uc740 \uc4f0\ub294\uc9c0, \uce90\ub9ad\ud130 \uc804\uc6a9 \uce58\uba85\ud0b7 \uc774\ud399\ud2b8\ub97c \uc4f0\ub294\uc9c0, \ub610\ud55c \ubaa9 \uc7a5\ube44\ub294 \uc720\ub2c8\ud06c\uc778\uc9c0\ub97c \ubaa8\ub450 \uc54c \uc218 \uc788\ub2e4. \uc774 \uae30\ub2a5\uc5d0 \ub300\ud574\uc11c\ub294 \uc720\uc800\ub4e4\uc758 \ubc18\ubc1c\uc774 \uc0c1\ub2f9\ud788 \uc788\ub294 \ub4ef \ud558\uc9c0\ub9cc \uc810\ucc28 \ub204\uadf8\ub7ec\uc9c4 \uac83 \uac19\ub2e4.", "paragraph_answer": " Tab\ud0a4 \ub97c \ub204\ub974\uba74 \uc804\uc7a5 \uc0c1\ud669\uc744 \uc54c \uc218 \uc788\uc73c\uba70 \uc801\uad70\uc744 \ud3ec\ud568\ud55c \ubaa8\ub4e0 \uc720\uc800\ub4e4\uc758 \ud0ac, \ud30c\uad34, \ub3c4\uc6c0, \ud14c\ud06c\ub2c9, \ucd1d\uc810\uc744 \ubcfc \uc218 \uc788\ub2e4. \ub610\ud55c \uc544\uc774\ud15c \ud15c\ud2b8\ub9ac\ub97c \ubcf4\ub294 \uae30\ub2a5\uc774 \ucd94\uac00\ub418\uc5b4\uc11c \uc544\uad70\uc758 \uacbd\uc6b0 \uac8c\uc784\uc2dc\uc791\uacfc \ub3d9\uc2dc\uc5d0 \uc544\uad70\uc758 \ud15c\ud2b8\ub9ac\ub97c \ud655\uc778\ud560 \uc218\ub294 \uc788\uc9c0\ub9cc \uc801\uad70\uc758 \uacbd\uc6b0 \uac8c\uc784\uc744 \uc2dc\uc791\ud558\uace0 5\ubd84\ub4a4\uc5d0 \uacf5\uac1c\ub41c\ub2e4. \ub2e4\uc2dc \ub9d0\ud574 34\ubd84 59\ucd08\uac00 \ub418\uba74 \ubaa8\ub4e0 \uc720\uc800\ub4e4\uc758 \uc7a5\ube44\uc544\uc774\ud15c \ud14c\ud06c\ud2b8\ub9ac\ub97c \ubcfc \uc218 \uc788\ub2e4\ub294 \uac83\uc774\ub2e4. \uadf8\ub7ec\ub098 \uad6c\uccb4\uc801\uc73c\ub85c \uc5b4\ub5a4 \uc544\uc774\ud15c\uc744 \ucc29\uc6a9\ud558\uc600\ub294\uc9c0\ub294 \ubcf4\uc5ec\uc8fc\uc9c0 \uc54a\ub294\ub2e4. \uc7a5\uac11, \ubaa8\uc790, \uac00\uc2b4, \ud5c8\ub9ac, \ub2e4\ub9ac, \ubc1c \uc7a5\ube44\uc758 \ud22c\uc790\ub808\ubca8\ub9cc\uc744 \uc54c \uc218 \uc788\uac8c \ub41c\ub2e4. `\ud0a4(Tab\ubc14\ub85c \uc704\uc5d0 \uc788\ub294 \ud0a4)\ub97c \ub204\ub974\uba74 \uc778\uac8c\uc784 \ucc44\ud305\ucc28\ub2e8 \ubc0f \ud074\ub79c\ucc44\ud305, \uadd3\uc18d\ub9d0\uc744 \ucc28\ub2e8\ud560 \uc218 \uc788\ub2e4. \ucd94\uac00\ub41c \uae30\ub2a5\uc73c\ub85c\ub294 \uc801\uad70\uc744 \ud3ec\ud568\ud55c \ubaa8\ub4e0 \uc720\uc800\ub4e4\uc758 \uc18c\ubaa8\uc544\uc774\ud15c\uacfc \ubaa9\uac78\uc774, \uc7a5\uc2e0\uad6c1, \uc7a5\uc2e0\uad6c2, \uc7a5\uc2e0\uad6c3, \uc7a5\uc2e0\uad6c4\uc758 \uc544\uc774\ud15c\uc744 \uc54c \uc218 \uc788\ub2e4. \uc704\uc5d0\uc11c \uc124\uba85\ud55c \uac83\uacfc \ub9c8\ucc2c\uac00\uc9c0\ub85c \uc544\uad70\uc740 \uac8c\uc784 \uc2dc\uc791\uacfc \ub3d9\uc2dc\uc5d0 \uc801\uad70\uc740 34\ubd84 59\ucd08\uac00 \ub418\ub294 \uc21c\uac04 \uacf5\uac1c\ub41c\ub2e4. Tab\ud0a4\uc640\ub294 \ub2e4\ub974\uac8c \uc5b4\ub5a4 \uc18c\ubaa8\uc544\uc774\ud15c\uc744 \ucc29\uc6a9\ud558\uace0 \uc788\ub294\uc9c0 \uad6c\uccb4\uc801\uc73c\ub85c \uc54c \uc218 \uc788\uc744 \ubfd0\ub9cc \uc544\ub2c8\ub77c \ub098\uba38\uc9c0 \uc7a5\ube44 \ubaa9, \uc7a5\uc2e0\uad6c \uc544\uc774\ud15c\uc758 \uad6c\uccb4\uc801\uc778 \uba85\uce6d\ub3c4 \uc54c \uc218 \uc788\ub2e4. \ub2e4\uc2dc \ub9d0\ud574 \uc774 \uc0ac\ub78c\uc774 \uc5d1\uc140\ub808\uc774\uc158\uc740 \uc4f0\ub294\uc9c0, \uce90\ub9ad\ud130 \uc804\uc6a9 \uce58\uba85\ud0b7 \uc774\ud399\ud2b8\ub97c \uc4f0\ub294\uc9c0, \ub610\ud55c \ubaa9 \uc7a5\ube44\ub294 \uc720\ub2c8\ud06c\uc778\uc9c0\ub97c \ubaa8\ub450 \uc54c \uc218 \uc788\ub2e4. \uc774 \uae30\ub2a5\uc5d0 \ub300\ud574\uc11c\ub294 \uc720\uc800\ub4e4\uc758 \ubc18\ubc1c\uc774 \uc0c1\ub2f9\ud788 \uc788\ub294 \ub4ef \ud558\uc9c0\ub9cc \uc810\ucc28 \ub204\uadf8\ub7ec\uc9c4 \uac83 \uac19\ub2e4.", "sentence_answer": " Tab\ud0a4 \ub97c \ub204\ub974\uba74 \uc804\uc7a5 \uc0c1\ud669\uc744 \uc54c \uc218 \uc788\uc73c\uba70 \uc801\uad70\uc744 \ud3ec\ud568\ud55c \ubaa8\ub4e0 \uc720\uc800\ub4e4\uc758 \ud0ac, \ud30c\uad34, \ub3c4\uc6c0, \ud14c\ud06c\ub2c9, \ucd1d\uc810\uc744 \ubcfc \uc218 \uc788\ub2e4.", "paragraph_id": "6657185-2-0", "paragraph_question": "question: \uc804\uc7a5 \uc0c1\ud669\uacfc \uc804 \uc720\uc800\uc758 \ud0ac, \ud30c\uad34 \ub4f1\uc758 \uc815\ubcf4\ub97c \ud655\uc778\ud558\ub294\ub370 \uc774\uc6a9\ud558\ub294 \ud0a4\ub294?, context: Tab\ud0a4\ub97c \ub204\ub974\uba74 \uc804\uc7a5 \uc0c1\ud669\uc744 \uc54c \uc218 \uc788\uc73c\uba70 \uc801\uad70\uc744 \ud3ec\ud568\ud55c \ubaa8\ub4e0 \uc720\uc800\ub4e4\uc758 \ud0ac, \ud30c\uad34, \ub3c4\uc6c0, \ud14c\ud06c\ub2c9, \ucd1d\uc810\uc744 \ubcfc \uc218 \uc788\ub2e4. \ub610\ud55c \uc544\uc774\ud15c \ud15c\ud2b8\ub9ac\ub97c \ubcf4\ub294 \uae30\ub2a5\uc774 \ucd94\uac00\ub418\uc5b4\uc11c \uc544\uad70\uc758 \uacbd\uc6b0 \uac8c\uc784\uc2dc\uc791\uacfc \ub3d9\uc2dc\uc5d0 \uc544\uad70\uc758 \ud15c\ud2b8\ub9ac\ub97c \ud655\uc778\ud560 \uc218\ub294 \uc788\uc9c0\ub9cc \uc801\uad70\uc758 \uacbd\uc6b0 \uac8c\uc784\uc744 \uc2dc\uc791\ud558\uace0 5\ubd84\ub4a4\uc5d0 \uacf5\uac1c\ub41c\ub2e4. \ub2e4\uc2dc \ub9d0\ud574 34\ubd84 59\ucd08\uac00 \ub418\uba74 \ubaa8\ub4e0 \uc720\uc800\ub4e4\uc758 \uc7a5\ube44\uc544\uc774\ud15c \ud14c\ud06c\ud2b8\ub9ac\ub97c \ubcfc \uc218 \uc788\ub2e4\ub294 \uac83\uc774\ub2e4. \uadf8\ub7ec\ub098 \uad6c\uccb4\uc801\uc73c\ub85c \uc5b4\ub5a4 \uc544\uc774\ud15c\uc744 \ucc29\uc6a9\ud558\uc600\ub294\uc9c0\ub294 \ubcf4\uc5ec\uc8fc\uc9c0 \uc54a\ub294\ub2e4. \uc7a5\uac11, \ubaa8\uc790, \uac00\uc2b4, \ud5c8\ub9ac, \ub2e4\ub9ac, \ubc1c \uc7a5\ube44\uc758 \ud22c\uc790\ub808\ubca8\ub9cc\uc744 \uc54c \uc218 \uc788\uac8c \ub41c\ub2e4. `\ud0a4(Tab\ubc14\ub85c \uc704\uc5d0 \uc788\ub294 \ud0a4)\ub97c \ub204\ub974\uba74 \uc778\uac8c\uc784 \ucc44\ud305\ucc28\ub2e8 \ubc0f \ud074\ub79c\ucc44\ud305, \uadd3\uc18d\ub9d0\uc744 \ucc28\ub2e8\ud560 \uc218 \uc788\ub2e4. \ucd94\uac00\ub41c \uae30\ub2a5\uc73c\ub85c\ub294 \uc801\uad70\uc744 \ud3ec\ud568\ud55c \ubaa8\ub4e0 \uc720\uc800\ub4e4\uc758 \uc18c\ubaa8\uc544\uc774\ud15c\uacfc \ubaa9\uac78\uc774, \uc7a5\uc2e0\uad6c1, \uc7a5\uc2e0\uad6c2, \uc7a5\uc2e0\uad6c3, \uc7a5\uc2e0\uad6c4\uc758 \uc544\uc774\ud15c\uc744 \uc54c \uc218 \uc788\ub2e4. \uc704\uc5d0\uc11c \uc124\uba85\ud55c \uac83\uacfc \ub9c8\ucc2c\uac00\uc9c0\ub85c \uc544\uad70\uc740 \uac8c\uc784 \uc2dc\uc791\uacfc \ub3d9\uc2dc\uc5d0 \uc801\uad70\uc740 34\ubd84 59\ucd08\uac00 \ub418\ub294 \uc21c\uac04 \uacf5\uac1c\ub41c\ub2e4. Tab\ud0a4\uc640\ub294 \ub2e4\ub974\uac8c \uc5b4\ub5a4 \uc18c\ubaa8\uc544\uc774\ud15c\uc744 \ucc29\uc6a9\ud558\uace0 \uc788\ub294\uc9c0 \uad6c\uccb4\uc801\uc73c\ub85c \uc54c \uc218 \uc788\uc744 \ubfd0\ub9cc \uc544\ub2c8\ub77c \ub098\uba38\uc9c0 \uc7a5\ube44 \ubaa9, \uc7a5\uc2e0\uad6c \uc544\uc774\ud15c\uc758 \uad6c\uccb4\uc801\uc778 \uba85\uce6d\ub3c4 \uc54c \uc218 \uc788\ub2e4. \ub2e4\uc2dc \ub9d0\ud574 \uc774 \uc0ac\ub78c\uc774 \uc5d1\uc140\ub808\uc774\uc158\uc740 \uc4f0\ub294\uc9c0, \uce90\ub9ad\ud130 \uc804\uc6a9 \uce58\uba85\ud0b7 \uc774\ud399\ud2b8\ub97c \uc4f0\ub294\uc9c0, \ub610\ud55c \ubaa9 \uc7a5\ube44\ub294 \uc720\ub2c8\ud06c\uc778\uc9c0\ub97c \ubaa8\ub450 \uc54c \uc218 \uc788\ub2e4. \uc774 \uae30\ub2a5\uc5d0 \ub300\ud574\uc11c\ub294 \uc720\uc800\ub4e4\uc758 \ubc18\ubc1c\uc774 \uc0c1\ub2f9\ud788 \uc788\ub294 \ub4ef \ud558\uc9c0\ub9cc \uc810\ucc28 \ub204\uadf8\ub7ec\uc9c4 \uac83 \uac19\ub2e4."} {"question": "\uc624\uc2a4\ud2b8\ub808\uc77c\ub9ac\uc544\uae4c\uce58\uc758 \uc544\uc885\uc744 \uad6c\ubd84\ud558\ub294 \uae30\uc900\uc740?", "paragraph": "\uc131\uccb4 \uc624\uc2a4\ud2b8\ub808\uc77c\ub9ac\uc544\uae4c\uce58\ub294 \uc81c\ubc95 \uac15\uc778\ud558\uace0 \ub2e4\ubd80\uc9c4 \uc0c8\ub85c, \uc2e0\uc7a5 37 ~ 43 \uc13c\ud2f0\ubbf8\ud130, \uc775\ud3ed 65 ~ 85 \uc13c\ud2f0\ubbf8\ud130, \uccb4\uc911 220 ~ 350 \ud0ac\ub85c\uadf8\ub7a8\uc774\ub2e4. \ud2bc\ud2bc\ud55c \uc410\uae30 \ubaa8\uc591 \ubd80\ub9ac\ub294 \uc804\uccb4\uc801\uc73c\ub85c \ud478\ub974\uc2a4\ub984\ud55c \ud770\uc0c9\uc5d0, \uac00\uc7a5\uc790\ub9ac\ub294 \uac80\uace0 \ubd80\ub9ac \ub05d\uc5d0 \uc791\uc740 \uac08\uace0\ub9ac\uac00 \uc788\ub2e4. \uac80\uc740 \ub2e4\ub9ac\ub294 \uae38\uace0 \uac15\ud558\ub2e4. \uae43\ud138\uc740 \uc644\ubcbd\ud55c \ud751\uacfc \ubc31\uc73c\ub85c\ub9cc \uc774\ub8e8\uc5b4\uc838 \uc788\uc73c\uba70 \uc724\uae30\uac00 \uc788\ub2e4. \uc554\uc218\ub97c \ub9c9\ub860\ud558\uace0 \ubaa8\ub4e0 \uc544\uc885\uc758 \uba38\ub9ac\ub294 \uac80\uc740\uc0c9\uc774\uace0 \ub0a0\uac1c\uc640 \ubab8\ud1b5\uc774 \uc5f0\uacb0\ub418\ub294 \uc5b4\uae68 \ubd80\uc704\ub294 \ud770\uc0c9\uc774\ub2e4. \uaf2c\ub9ac \ub05d\uc5d0\ub294 \uac80\uc740 \ub760\uac00 \uc788\ub2e4. \uc218\ucef7\uc740 \ubaa9\ub35c\ubbf8\uac00 \ud770\uc0c9\uc774\uc9c0\ub9cc \uc554\ucef7\uc740 \ud68c\uc0c9\ube5b\uc774 \uac10\ub3c4\ub294 \ud770\uc0c9\uc774\ub2e4. \uc624\uc2a4\ud2b8\ub808\uc77c\ub9ac\uc544\uae4c\uce58 \uc131\uccb4\uc758 \ub208\uc740 \ud750\ub9bf\ud55c \ubd89\uc740\uc0c9\uc778\ub370, \ud53c\ub9ac\uae4c\ub9c8\uadc0\ub098 \ub3c4\ub798\uae4c\ub9c8\uadc0, \uae4c\ub9c8\uadc0\ub294 \ub208\uc774 \ub178\ub780 \uc0c9\uc774\ub77c\uc11c \uad6c\ubd84\ud560 \uc218 \uc788\ub2e4. \ubaa9\ub35c\ubbf8 \uc544\ub798 \ub4f1 \ubd80\ubd84\uc758 \uc548\uc7a5 \ud45c\uc2dc\ub85c \uc544\uc885\ub4e4\uc744 \uad6c\ubd84\ud574\ub0bc \uc218 \uc788\ub2e4. \uac80\uc740\ub4f1\uc624\uc2a4\ud2b8\ub808\uc77c\ub9ac\uc544\uae4c\uce58\ub294 \uac80\uc740\uc0c9 \ubaa9\ub35c\ubbf8\uc5d0 \ud770\uc0c9 \uc548\uc7a5\uc744 \uc5b9\uc5c8\uace0, \ud770\ub4f1\uc624\uc2a4\ud2b8\ub808\uc77c\ub9ac\uc544\uae4c\uce58\ub294 \ubaa9\ub35c\ubbf8\uc640 \uc548\uc7a5\uc774 \ubaa8\ub450 \ud770\uc0c9\uc774\ub2e4. \uc11c\ubd80\uc624\uc2a4\ud2b8\ub808\uc77c\ub9ac\uc544\uae4c\uce58(C. t. dorsalis) \uc544\uc885\uc758 \uacbd\uc6b0 \uc218\ucef7\uc740 \ub4f1\uc774 \ud770\uc0c9\uc774\uc9c0\ub9cc \uc554\ucef7\uc740 \uac00\ub9ac\ube44 \ubaa8\uc591\uc758 \uac80\uc740\uc0c9\uc774 \uc788\ub2e4.", "answer": "\uc548\uc7a5 \ud45c\uc2dc", "sentence": "\ubaa9\ub35c\ubbf8 \uc544\ub798 \ub4f1 \ubd80\ubd84\uc758 \uc548\uc7a5 \ud45c\uc2dc \ub85c \uc544\uc885\ub4e4\uc744 \uad6c\ubd84\ud574\ub0bc \uc218 \uc788\ub2e4.", "paragraph_sentence": "\uc131\uccb4 \uc624\uc2a4\ud2b8\ub808\uc77c\ub9ac\uc544\uae4c\uce58\ub294 \uc81c\ubc95 \uac15\uc778\ud558\uace0 \ub2e4\ubd80\uc9c4 \uc0c8\ub85c, \uc2e0\uc7a5 37 ~ 43 \uc13c\ud2f0\ubbf8\ud130, \uc775\ud3ed 65 ~ 85 \uc13c\ud2f0\ubbf8\ud130, \uccb4\uc911 220 ~ 350 \ud0ac\ub85c\uadf8\ub7a8\uc774\ub2e4. \ud2bc\ud2bc\ud55c \uc410\uae30 \ubaa8\uc591 \ubd80\ub9ac\ub294 \uc804\uccb4\uc801\uc73c\ub85c \ud478\ub974\uc2a4\ub984\ud55c \ud770\uc0c9\uc5d0, \uac00\uc7a5\uc790\ub9ac\ub294 \uac80\uace0 \ubd80\ub9ac \ub05d\uc5d0 \uc791\uc740 \uac08\uace0\ub9ac\uac00 \uc788\ub2e4. \uac80\uc740 \ub2e4\ub9ac\ub294 \uae38\uace0 \uac15\ud558\ub2e4. \uae43\ud138\uc740 \uc644\ubcbd\ud55c \ud751\uacfc \ubc31\uc73c\ub85c\ub9cc \uc774\ub8e8\uc5b4\uc838 \uc788\uc73c\uba70 \uc724\uae30\uac00 \uc788\ub2e4. \uc554\uc218\ub97c \ub9c9\ub860\ud558\uace0 \ubaa8\ub4e0 \uc544\uc885\uc758 \uba38\ub9ac\ub294 \uac80\uc740\uc0c9\uc774\uace0 \ub0a0\uac1c\uc640 \ubab8\ud1b5\uc774 \uc5f0\uacb0\ub418\ub294 \uc5b4\uae68 \ubd80\uc704\ub294 \ud770\uc0c9\uc774\ub2e4. \uaf2c\ub9ac \ub05d\uc5d0\ub294 \uac80\uc740 \ub760\uac00 \uc788\ub2e4. \uc218\ucef7\uc740 \ubaa9\ub35c\ubbf8\uac00 \ud770\uc0c9\uc774\uc9c0\ub9cc \uc554\ucef7\uc740 \ud68c\uc0c9\ube5b\uc774 \uac10\ub3c4\ub294 \ud770\uc0c9\uc774\ub2e4. \uc624\uc2a4\ud2b8\ub808\uc77c\ub9ac\uc544\uae4c\uce58 \uc131\uccb4\uc758 \ub208\uc740 \ud750\ub9bf\ud55c \ubd89\uc740\uc0c9\uc778\ub370, \ud53c\ub9ac\uae4c\ub9c8\uadc0\ub098 \ub3c4\ub798\uae4c\ub9c8\uadc0, \uae4c\ub9c8\uadc0\ub294 \ub208\uc774 \ub178\ub780 \uc0c9\uc774\ub77c\uc11c \uad6c\ubd84\ud560 \uc218 \uc788\ub2e4. \ubaa9\ub35c\ubbf8 \uc544\ub798 \ub4f1 \ubd80\ubd84\uc758 \uc548\uc7a5 \ud45c\uc2dc \ub85c \uc544\uc885\ub4e4\uc744 \uad6c\ubd84\ud574\ub0bc \uc218 \uc788\ub2e4. \uac80\uc740\ub4f1\uc624\uc2a4\ud2b8\ub808\uc77c\ub9ac\uc544\uae4c\uce58\ub294 \uac80\uc740\uc0c9 \ubaa9\ub35c\ubbf8\uc5d0 \ud770\uc0c9 \uc548\uc7a5\uc744 \uc5b9\uc5c8\uace0, \ud770\ub4f1\uc624\uc2a4\ud2b8\ub808\uc77c\ub9ac\uc544\uae4c\uce58\ub294 \ubaa9\ub35c\ubbf8\uc640 \uc548\uc7a5\uc774 \ubaa8\ub450 \ud770\uc0c9\uc774\ub2e4. \uc11c\ubd80\uc624\uc2a4\ud2b8\ub808\uc77c\ub9ac\uc544\uae4c\uce58(C. t. dorsalis) \uc544\uc885\uc758 \uacbd\uc6b0 \uc218\ucef7\uc740 \ub4f1\uc774 \ud770\uc0c9\uc774\uc9c0\ub9cc \uc554\ucef7\uc740 \uac00\ub9ac\ube44 \ubaa8\uc591\uc758 \uac80\uc740\uc0c9\uc774 \uc788\ub2e4.", "paragraph_answer": "\uc131\uccb4 \uc624\uc2a4\ud2b8\ub808\uc77c\ub9ac\uc544\uae4c\uce58\ub294 \uc81c\ubc95 \uac15\uc778\ud558\uace0 \ub2e4\ubd80\uc9c4 \uc0c8\ub85c, \uc2e0\uc7a5 37 ~ 43 \uc13c\ud2f0\ubbf8\ud130, \uc775\ud3ed 65 ~ 85 \uc13c\ud2f0\ubbf8\ud130, \uccb4\uc911 220 ~ 350 \ud0ac\ub85c\uadf8\ub7a8\uc774\ub2e4. \ud2bc\ud2bc\ud55c \uc410\uae30 \ubaa8\uc591 \ubd80\ub9ac\ub294 \uc804\uccb4\uc801\uc73c\ub85c \ud478\ub974\uc2a4\ub984\ud55c \ud770\uc0c9\uc5d0, \uac00\uc7a5\uc790\ub9ac\ub294 \uac80\uace0 \ubd80\ub9ac \ub05d\uc5d0 \uc791\uc740 \uac08\uace0\ub9ac\uac00 \uc788\ub2e4. \uac80\uc740 \ub2e4\ub9ac\ub294 \uae38\uace0 \uac15\ud558\ub2e4. \uae43\ud138\uc740 \uc644\ubcbd\ud55c \ud751\uacfc \ubc31\uc73c\ub85c\ub9cc \uc774\ub8e8\uc5b4\uc838 \uc788\uc73c\uba70 \uc724\uae30\uac00 \uc788\ub2e4. \uc554\uc218\ub97c \ub9c9\ub860\ud558\uace0 \ubaa8\ub4e0 \uc544\uc885\uc758 \uba38\ub9ac\ub294 \uac80\uc740\uc0c9\uc774\uace0 \ub0a0\uac1c\uc640 \ubab8\ud1b5\uc774 \uc5f0\uacb0\ub418\ub294 \uc5b4\uae68 \ubd80\uc704\ub294 \ud770\uc0c9\uc774\ub2e4. \uaf2c\ub9ac \ub05d\uc5d0\ub294 \uac80\uc740 \ub760\uac00 \uc788\ub2e4. \uc218\ucef7\uc740 \ubaa9\ub35c\ubbf8\uac00 \ud770\uc0c9\uc774\uc9c0\ub9cc \uc554\ucef7\uc740 \ud68c\uc0c9\ube5b\uc774 \uac10\ub3c4\ub294 \ud770\uc0c9\uc774\ub2e4. \uc624\uc2a4\ud2b8\ub808\uc77c\ub9ac\uc544\uae4c\uce58 \uc131\uccb4\uc758 \ub208\uc740 \ud750\ub9bf\ud55c \ubd89\uc740\uc0c9\uc778\ub370, \ud53c\ub9ac\uae4c\ub9c8\uadc0\ub098 \ub3c4\ub798\uae4c\ub9c8\uadc0, \uae4c\ub9c8\uadc0\ub294 \ub208\uc774 \ub178\ub780 \uc0c9\uc774\ub77c\uc11c \uad6c\ubd84\ud560 \uc218 \uc788\ub2e4. \ubaa9\ub35c\ubbf8 \uc544\ub798 \ub4f1 \ubd80\ubd84\uc758 \uc548\uc7a5 \ud45c\uc2dc \ub85c \uc544\uc885\ub4e4\uc744 \uad6c\ubd84\ud574\ub0bc \uc218 \uc788\ub2e4. \uac80\uc740\ub4f1\uc624\uc2a4\ud2b8\ub808\uc77c\ub9ac\uc544\uae4c\uce58\ub294 \uac80\uc740\uc0c9 \ubaa9\ub35c\ubbf8\uc5d0 \ud770\uc0c9 \uc548\uc7a5\uc744 \uc5b9\uc5c8\uace0, \ud770\ub4f1\uc624\uc2a4\ud2b8\ub808\uc77c\ub9ac\uc544\uae4c\uce58\ub294 \ubaa9\ub35c\ubbf8\uc640 \uc548\uc7a5\uc774 \ubaa8\ub450 \ud770\uc0c9\uc774\ub2e4. \uc11c\ubd80\uc624\uc2a4\ud2b8\ub808\uc77c\ub9ac\uc544\uae4c\uce58(C. t. dorsalis) \uc544\uc885\uc758 \uacbd\uc6b0 \uc218\ucef7\uc740 \ub4f1\uc774 \ud770\uc0c9\uc774\uc9c0\ub9cc \uc554\ucef7\uc740 \uac00\ub9ac\ube44 \ubaa8\uc591\uc758 \uac80\uc740\uc0c9\uc774 \uc788\ub2e4.", "sentence_answer": "\ubaa9\ub35c\ubbf8 \uc544\ub798 \ub4f1 \ubd80\ubd84\uc758 \uc548\uc7a5 \ud45c\uc2dc \ub85c \uc544\uc885\ub4e4\uc744 \uad6c\ubd84\ud574\ub0bc \uc218 \uc788\ub2e4.", "paragraph_id": "6541265-3-2", "paragraph_question": "question: \uc624\uc2a4\ud2b8\ub808\uc77c\ub9ac\uc544\uae4c\uce58\uc758 \uc544\uc885\uc744 \uad6c\ubd84\ud558\ub294 \uae30\uc900\uc740?, context: \uc131\uccb4 \uc624\uc2a4\ud2b8\ub808\uc77c\ub9ac\uc544\uae4c\uce58\ub294 \uc81c\ubc95 \uac15\uc778\ud558\uace0 \ub2e4\ubd80\uc9c4 \uc0c8\ub85c, \uc2e0\uc7a5 37 ~ 43 \uc13c\ud2f0\ubbf8\ud130, \uc775\ud3ed 65 ~ 85 \uc13c\ud2f0\ubbf8\ud130, \uccb4\uc911 220 ~ 350 \ud0ac\ub85c\uadf8\ub7a8\uc774\ub2e4. \ud2bc\ud2bc\ud55c \uc410\uae30 \ubaa8\uc591 \ubd80\ub9ac\ub294 \uc804\uccb4\uc801\uc73c\ub85c \ud478\ub974\uc2a4\ub984\ud55c \ud770\uc0c9\uc5d0, \uac00\uc7a5\uc790\ub9ac\ub294 \uac80\uace0 \ubd80\ub9ac \ub05d\uc5d0 \uc791\uc740 \uac08\uace0\ub9ac\uac00 \uc788\ub2e4. \uac80\uc740 \ub2e4\ub9ac\ub294 \uae38\uace0 \uac15\ud558\ub2e4. \uae43\ud138\uc740 \uc644\ubcbd\ud55c \ud751\uacfc \ubc31\uc73c\ub85c\ub9cc \uc774\ub8e8\uc5b4\uc838 \uc788\uc73c\uba70 \uc724\uae30\uac00 \uc788\ub2e4. \uc554\uc218\ub97c \ub9c9\ub860\ud558\uace0 \ubaa8\ub4e0 \uc544\uc885\uc758 \uba38\ub9ac\ub294 \uac80\uc740\uc0c9\uc774\uace0 \ub0a0\uac1c\uc640 \ubab8\ud1b5\uc774 \uc5f0\uacb0\ub418\ub294 \uc5b4\uae68 \ubd80\uc704\ub294 \ud770\uc0c9\uc774\ub2e4. \uaf2c\ub9ac \ub05d\uc5d0\ub294 \uac80\uc740 \ub760\uac00 \uc788\ub2e4. \uc218\ucef7\uc740 \ubaa9\ub35c\ubbf8\uac00 \ud770\uc0c9\uc774\uc9c0\ub9cc \uc554\ucef7\uc740 \ud68c\uc0c9\ube5b\uc774 \uac10\ub3c4\ub294 \ud770\uc0c9\uc774\ub2e4. \uc624\uc2a4\ud2b8\ub808\uc77c\ub9ac\uc544\uae4c\uce58 \uc131\uccb4\uc758 \ub208\uc740 \ud750\ub9bf\ud55c \ubd89\uc740\uc0c9\uc778\ub370, \ud53c\ub9ac\uae4c\ub9c8\uadc0\ub098 \ub3c4\ub798\uae4c\ub9c8\uadc0, \uae4c\ub9c8\uadc0\ub294 \ub208\uc774 \ub178\ub780 \uc0c9\uc774\ub77c\uc11c \uad6c\ubd84\ud560 \uc218 \uc788\ub2e4. \ubaa9\ub35c\ubbf8 \uc544\ub798 \ub4f1 \ubd80\ubd84\uc758 \uc548\uc7a5 \ud45c\uc2dc\ub85c \uc544\uc885\ub4e4\uc744 \uad6c\ubd84\ud574\ub0bc \uc218 \uc788\ub2e4. \uac80\uc740\ub4f1\uc624\uc2a4\ud2b8\ub808\uc77c\ub9ac\uc544\uae4c\uce58\ub294 \uac80\uc740\uc0c9 \ubaa9\ub35c\ubbf8\uc5d0 \ud770\uc0c9 \uc548\uc7a5\uc744 \uc5b9\uc5c8\uace0, \ud770\ub4f1\uc624\uc2a4\ud2b8\ub808\uc77c\ub9ac\uc544\uae4c\uce58\ub294 \ubaa9\ub35c\ubbf8\uc640 \uc548\uc7a5\uc774 \ubaa8\ub450 \ud770\uc0c9\uc774\ub2e4. \uc11c\ubd80\uc624\uc2a4\ud2b8\ub808\uc77c\ub9ac\uc544\uae4c\uce58(C. t. dorsalis) \uc544\uc885\uc758 \uacbd\uc6b0 \uc218\ucef7\uc740 \ub4f1\uc774 \ud770\uc0c9\uc774\uc9c0\ub9cc \uc554\ucef7\uc740 \uac00\ub9ac\ube44 \ubaa8\uc591\uc758 \uac80\uc740\uc0c9\uc774 \uc788\ub2e4."} {"question": "\ube0c\ub85c\ub354\ub9ad\uc758 \ubc30\uc6b0\uc790\ub294?", "paragraph": "\ube0c\ub85c\ub354\ub9ad\uc758 \ubc30\uc6b0\uc790\ub294 \ubc30\uc6b0 \uc138\ub77c \uc81c\uc2dc\uce74 \ud30c\ucee4\ub85c, \uadf8\ub140\uc758 \uc624\ube60\ub97c \ud1b5\ud574\uc11c \uc11c\ub85c \ub9cc\ub098\uac8c \ub418\uc5c8\ub2e4. \ub458\uc740 1997\ub144 5\uc6d4 19\uc77c \ub77c\uc6cc \uc5d0\uc2a4\ud2b8 \uc0ac\uc774\ub4dc\uc758 \uc2dc\ub098\uace0\uadf8\uc5d0\uc11c \uc815\uc2dd \uacb0\ud63c\uc2dd\uc744 \uce58\ub800\ub2e4. \uc774\uac83\uc740 \ud30c\ucee4\uc640 \ube0c\ub85c\ub354\ub9ad\uc774 \uc2a4\uc2a4\ub85c\ub97c \ubb38\ud654\uc801\uc73c\ub85c \uc720\ub300\uc778\uc774\ub77c\uace0 \uc5ec\uae34 \uac83\uc5d0 \ube44\ub86f\ud558\uc600\ub2e4. \uc7a5\uc18c\uc640 \ub2e4\ub974\uac8c \uacb0\ud63c\uc2dd\uc740 \uc138\uc18d\uc801\uc73c\ub85c \uc774\ub8e8\uc5b4\uc84c\uc73c\uba70, \uadf8\uc758 \ub204\uc774\uc778 \uac10\ub3c5 \ubaa9\uc0ac \uc790\ub137 \ube0c\ub85c\ub354\ub9ad \ud06c\ub798\ud504\ud2b8\uac00 \uac70\ud589\ud558\uc600\ub2e4. \ube0c\ub85c\ub354\ub9ad\uc740 \ud30c\ucee4\uc640\uc758 \uc0ac\uc774\uc5d0\uc11c 2002\ub144 10\uc6d4 28\uc77c \uc544\ub4e4 \uc81c\uc784\uc2a4 \uc70c\ud0a4 \ube0c\ub85c\ub354\ub9ad\uc744 \ub0b3\uc558\ub2e4. \ube0c\ub85c\ub354\ub9ad\uc740 2006\ub144 6\uc6d4 \ubbf8\uad6d\uc758 \uc8fc\uac04\uc9c0 \u3008\ub77c\uc774\ud504 \uc564 \uc2a4\ud0c0\uc77c\u3009\uc774 \uc544\ubc84\uc9c0\uc758 \ub0a0\uc744 \ub9de\uc544 \uc2e4\uc2dc\ud55c \uc124\ubb38 \uc870\uc0ac\uc5d0\uc11c \ucd5c\uace0\uc758 \uc544\ube60 6\uc704\ub85c \uc120\uc815\ub418\uc5c8\ub2e4. 2009\ub144 4\uc6d4 28\uc77c\uc5d0\ub294 \ube0c\ub85c\ub354\ub9ad\uacfc \ud30c\ucee4\uac00 \ub300\ub9ac\ubaa8\ub97c \ud1b5\ud574\uc11c \uadf8 \ud574 \uc5ec\ub984\uc5d0 \uc5ec\uc790 \uc30d\ub465\uc774\ub97c \uc5bb\uc744 \uac83\uc73c\ub85c \ud655\uc778\ub418\uc5c8\ub2e4. \ub300\ub9ac\ubaa8\ub294 2009\ub144 6\uc6d4 22\uc77c \ube0c\ub85c\ub354\ub9ad\uacfc \ud30c\ucee4\uc758 \ub450 \ub538\uc778 \ub9c8\ub9ac\uc628 \ub85c\ub808\ud0c0 \uc5d8\uc6f0\uacfc \ud0c0\ube44\ud0c0 \ud638\uc9c0\ub97c \ucd9c\uc0b0\ud558\uc5ec \uadf8\ub4e4\uc5d0\uac8c \uc778\uacc4\ud558\uc600\ub2e4.", "answer": "\uc138\ub77c \uc81c\uc2dc\uce74 \ud30c\ucee4", "sentence": "\ube0c\ub85c\ub354\ub9ad\uc758 \ubc30\uc6b0\uc790\ub294 \ubc30\uc6b0 \uc138\ub77c \uc81c\uc2dc\uce74 \ud30c\ucee4 \ub85c, \uadf8\ub140\uc758 \uc624\ube60\ub97c \ud1b5\ud574\uc11c \uc11c\ub85c \ub9cc\ub098\uac8c \ub418\uc5c8\ub2e4.", "paragraph_sentence": " \ube0c\ub85c\ub354\ub9ad\uc758 \ubc30\uc6b0\uc790\ub294 \ubc30\uc6b0 \uc138\ub77c \uc81c\uc2dc\uce74 \ud30c\ucee4 \ub85c, \uadf8\ub140\uc758 \uc624\ube60\ub97c \ud1b5\ud574\uc11c \uc11c\ub85c \ub9cc\ub098\uac8c \ub418\uc5c8\ub2e4. \ub458\uc740 1997\ub144 5\uc6d4 19\uc77c \ub77c\uc6cc \uc5d0\uc2a4\ud2b8 \uc0ac\uc774\ub4dc\uc758 \uc2dc\ub098\uace0\uadf8\uc5d0\uc11c \uc815\uc2dd \uacb0\ud63c\uc2dd\uc744 \uce58\ub800\ub2e4. \uc774\uac83\uc740 \ud30c\ucee4\uc640 \ube0c\ub85c\ub354\ub9ad\uc774 \uc2a4\uc2a4\ub85c\ub97c \ubb38\ud654\uc801\uc73c\ub85c \uc720\ub300\uc778\uc774\ub77c\uace0 \uc5ec\uae34 \uac83\uc5d0 \ube44\ub86f\ud558\uc600\ub2e4. \uc7a5\uc18c\uc640 \ub2e4\ub974\uac8c \uacb0\ud63c\uc2dd\uc740 \uc138\uc18d\uc801\uc73c\ub85c \uc774\ub8e8\uc5b4\uc84c\uc73c\uba70, \uadf8\uc758 \ub204\uc774\uc778 \uac10\ub3c5 \ubaa9\uc0ac \uc790\ub137 \ube0c\ub85c\ub354\ub9ad \ud06c\ub798\ud504\ud2b8\uac00 \uac70\ud589\ud558\uc600\ub2e4. \ube0c\ub85c\ub354\ub9ad\uc740 \ud30c\ucee4\uc640\uc758 \uc0ac\uc774\uc5d0\uc11c 2002\ub144 10\uc6d4 28\uc77c \uc544\ub4e4 \uc81c\uc784\uc2a4 \uc70c\ud0a4 \ube0c\ub85c\ub354\ub9ad\uc744 \ub0b3\uc558\ub2e4. \ube0c\ub85c\ub354\ub9ad\uc740 2006\ub144 6\uc6d4 \ubbf8\uad6d\uc758 \uc8fc\uac04\uc9c0 \u3008\ub77c\uc774\ud504 \uc564 \uc2a4\ud0c0\uc77c\u3009\uc774 \uc544\ubc84\uc9c0\uc758 \ub0a0\uc744 \ub9de\uc544 \uc2e4\uc2dc\ud55c \uc124\ubb38 \uc870\uc0ac\uc5d0\uc11c \ucd5c\uace0\uc758 \uc544\ube60 6\uc704\ub85c \uc120\uc815\ub418\uc5c8\ub2e4. 2009\ub144 4\uc6d4 28\uc77c\uc5d0\ub294 \ube0c\ub85c\ub354\ub9ad\uacfc \ud30c\ucee4\uac00 \ub300\ub9ac\ubaa8\ub97c \ud1b5\ud574\uc11c \uadf8 \ud574 \uc5ec\ub984\uc5d0 \uc5ec\uc790 \uc30d\ub465\uc774\ub97c \uc5bb\uc744 \uac83\uc73c\ub85c \ud655\uc778\ub418\uc5c8\ub2e4. \ub300\ub9ac\ubaa8\ub294 2009\ub144 6\uc6d4 22\uc77c \ube0c\ub85c\ub354\ub9ad\uacfc \ud30c\ucee4\uc758 \ub450 \ub538\uc778 \ub9c8\ub9ac\uc628 \ub85c\ub808\ud0c0 \uc5d8\uc6f0\uacfc \ud0c0\ube44\ud0c0 \ud638\uc9c0\ub97c \ucd9c\uc0b0\ud558\uc5ec \uadf8\ub4e4\uc5d0\uac8c \uc778\uacc4\ud558\uc600\ub2e4.", "paragraph_answer": "\ube0c\ub85c\ub354\ub9ad\uc758 \ubc30\uc6b0\uc790\ub294 \ubc30\uc6b0 \uc138\ub77c \uc81c\uc2dc\uce74 \ud30c\ucee4 \ub85c, \uadf8\ub140\uc758 \uc624\ube60\ub97c \ud1b5\ud574\uc11c \uc11c\ub85c \ub9cc\ub098\uac8c \ub418\uc5c8\ub2e4. \ub458\uc740 1997\ub144 5\uc6d4 19\uc77c \ub77c\uc6cc \uc5d0\uc2a4\ud2b8 \uc0ac\uc774\ub4dc\uc758 \uc2dc\ub098\uace0\uadf8\uc5d0\uc11c \uc815\uc2dd \uacb0\ud63c\uc2dd\uc744 \uce58\ub800\ub2e4. \uc774\uac83\uc740 \ud30c\ucee4\uc640 \ube0c\ub85c\ub354\ub9ad\uc774 \uc2a4\uc2a4\ub85c\ub97c \ubb38\ud654\uc801\uc73c\ub85c \uc720\ub300\uc778\uc774\ub77c\uace0 \uc5ec\uae34 \uac83\uc5d0 \ube44\ub86f\ud558\uc600\ub2e4. \uc7a5\uc18c\uc640 \ub2e4\ub974\uac8c \uacb0\ud63c\uc2dd\uc740 \uc138\uc18d\uc801\uc73c\ub85c \uc774\ub8e8\uc5b4\uc84c\uc73c\uba70, \uadf8\uc758 \ub204\uc774\uc778 \uac10\ub3c5 \ubaa9\uc0ac \uc790\ub137 \ube0c\ub85c\ub354\ub9ad \ud06c\ub798\ud504\ud2b8\uac00 \uac70\ud589\ud558\uc600\ub2e4. \ube0c\ub85c\ub354\ub9ad\uc740 \ud30c\ucee4\uc640\uc758 \uc0ac\uc774\uc5d0\uc11c 2002\ub144 10\uc6d4 28\uc77c \uc544\ub4e4 \uc81c\uc784\uc2a4 \uc70c\ud0a4 \ube0c\ub85c\ub354\ub9ad\uc744 \ub0b3\uc558\ub2e4. \ube0c\ub85c\ub354\ub9ad\uc740 2006\ub144 6\uc6d4 \ubbf8\uad6d\uc758 \uc8fc\uac04\uc9c0 \u3008\ub77c\uc774\ud504 \uc564 \uc2a4\ud0c0\uc77c\u3009\uc774 \uc544\ubc84\uc9c0\uc758 \ub0a0\uc744 \ub9de\uc544 \uc2e4\uc2dc\ud55c \uc124\ubb38 \uc870\uc0ac\uc5d0\uc11c \ucd5c\uace0\uc758 \uc544\ube60 6\uc704\ub85c \uc120\uc815\ub418\uc5c8\ub2e4. 2009\ub144 4\uc6d4 28\uc77c\uc5d0\ub294 \ube0c\ub85c\ub354\ub9ad\uacfc \ud30c\ucee4\uac00 \ub300\ub9ac\ubaa8\ub97c \ud1b5\ud574\uc11c \uadf8 \ud574 \uc5ec\ub984\uc5d0 \uc5ec\uc790 \uc30d\ub465\uc774\ub97c \uc5bb\uc744 \uac83\uc73c\ub85c \ud655\uc778\ub418\uc5c8\ub2e4. \ub300\ub9ac\ubaa8\ub294 2009\ub144 6\uc6d4 22\uc77c \ube0c\ub85c\ub354\ub9ad\uacfc \ud30c\ucee4\uc758 \ub450 \ub538\uc778 \ub9c8\ub9ac\uc628 \ub85c\ub808\ud0c0 \uc5d8\uc6f0\uacfc \ud0c0\ube44\ud0c0 \ud638\uc9c0\ub97c \ucd9c\uc0b0\ud558\uc5ec \uadf8\ub4e4\uc5d0\uac8c \uc778\uacc4\ud558\uc600\ub2e4.", "sentence_answer": "\ube0c\ub85c\ub354\ub9ad\uc758 \ubc30\uc6b0\uc790\ub294 \ubc30\uc6b0 \uc138\ub77c \uc81c\uc2dc\uce74 \ud30c\ucee4 \ub85c, \uadf8\ub140\uc758 \uc624\ube60\ub97c \ud1b5\ud574\uc11c \uc11c\ub85c \ub9cc\ub098\uac8c \ub418\uc5c8\ub2e4.", "paragraph_id": "6501901-2-0", "paragraph_question": "question: \ube0c\ub85c\ub354\ub9ad\uc758 \ubc30\uc6b0\uc790\ub294?, context: \ube0c\ub85c\ub354\ub9ad\uc758 \ubc30\uc6b0\uc790\ub294 \ubc30\uc6b0 \uc138\ub77c \uc81c\uc2dc\uce74 \ud30c\ucee4\ub85c, \uadf8\ub140\uc758 \uc624\ube60\ub97c \ud1b5\ud574\uc11c \uc11c\ub85c \ub9cc\ub098\uac8c \ub418\uc5c8\ub2e4. \ub458\uc740 1997\ub144 5\uc6d4 19\uc77c \ub77c\uc6cc \uc5d0\uc2a4\ud2b8 \uc0ac\uc774\ub4dc\uc758 \uc2dc\ub098\uace0\uadf8\uc5d0\uc11c \uc815\uc2dd \uacb0\ud63c\uc2dd\uc744 \uce58\ub800\ub2e4. \uc774\uac83\uc740 \ud30c\ucee4\uc640 \ube0c\ub85c\ub354\ub9ad\uc774 \uc2a4\uc2a4\ub85c\ub97c \ubb38\ud654\uc801\uc73c\ub85c \uc720\ub300\uc778\uc774\ub77c\uace0 \uc5ec\uae34 \uac83\uc5d0 \ube44\ub86f\ud558\uc600\ub2e4. \uc7a5\uc18c\uc640 \ub2e4\ub974\uac8c \uacb0\ud63c\uc2dd\uc740 \uc138\uc18d\uc801\uc73c\ub85c \uc774\ub8e8\uc5b4\uc84c\uc73c\uba70, \uadf8\uc758 \ub204\uc774\uc778 \uac10\ub3c5 \ubaa9\uc0ac \uc790\ub137 \ube0c\ub85c\ub354\ub9ad \ud06c\ub798\ud504\ud2b8\uac00 \uac70\ud589\ud558\uc600\ub2e4. \ube0c\ub85c\ub354\ub9ad\uc740 \ud30c\ucee4\uc640\uc758 \uc0ac\uc774\uc5d0\uc11c 2002\ub144 10\uc6d4 28\uc77c \uc544\ub4e4 \uc81c\uc784\uc2a4 \uc70c\ud0a4 \ube0c\ub85c\ub354\ub9ad\uc744 \ub0b3\uc558\ub2e4. \ube0c\ub85c\ub354\ub9ad\uc740 2006\ub144 6\uc6d4 \ubbf8\uad6d\uc758 \uc8fc\uac04\uc9c0 \u3008\ub77c\uc774\ud504 \uc564 \uc2a4\ud0c0\uc77c\u3009\uc774 \uc544\ubc84\uc9c0\uc758 \ub0a0\uc744 \ub9de\uc544 \uc2e4\uc2dc\ud55c \uc124\ubb38 \uc870\uc0ac\uc5d0\uc11c \ucd5c\uace0\uc758 \uc544\ube60 6\uc704\ub85c \uc120\uc815\ub418\uc5c8\ub2e4. 2009\ub144 4\uc6d4 28\uc77c\uc5d0\ub294 \ube0c\ub85c\ub354\ub9ad\uacfc \ud30c\ucee4\uac00 \ub300\ub9ac\ubaa8\ub97c \ud1b5\ud574\uc11c \uadf8 \ud574 \uc5ec\ub984\uc5d0 \uc5ec\uc790 \uc30d\ub465\uc774\ub97c \uc5bb\uc744 \uac83\uc73c\ub85c \ud655\uc778\ub418\uc5c8\ub2e4. \ub300\ub9ac\ubaa8\ub294 2009\ub144 6\uc6d4 22\uc77c \ube0c\ub85c\ub354\ub9ad\uacfc \ud30c\ucee4\uc758 \ub450 \ub538\uc778 \ub9c8\ub9ac\uc628 \ub85c\ub808\ud0c0 \uc5d8\uc6f0\uacfc \ud0c0\ube44\ud0c0 \ud638\uc9c0\ub97c \ucd9c\uc0b0\ud558\uc5ec \uadf8\ub4e4\uc5d0\uac8c \uc778\uacc4\ud558\uc600\ub2e4."} {"question": "1\ucc28 \uc608\uc1a1\uc5d0\uc11c \uc2b9\ub9ac\ud55c \ubd95\ub2f9\uc740?", "paragraph": "\uc774\ub54c, \ubd95\ub2f9\uc758 \ubcc0\uc9c8\uc744 \uac00\uc838\uc628 \ub450 \uac00\uc9c0 \ubb38\uc81c\uac00 \uc608\uc1a1\uacfc \ud658\uad6d\uc774\ub2e4. \uc608\uc1a1\uc740, \uc0c1\ubcf5\uc744 \uc785\ub294 \uae30\uac04\uc5d0 \uad00\ud55c \ubb38\uc81c\ub85c, \ud6a8\uc885\uacfc \ud6a8\uc885\uc758 \ube44\uc778 \uc778\uc120\uc655\ud6c4\uac00 \uc2b9\ud558\ud588\uc744 \ub54c, \ud6a8\uc885\uc758 \ubaa8\uce5c\uc774\uc790 \uc120\uc655 \uc778\uc870\uc758 \uacc4\ube44\uc600\ub358 \uc790\uc758\ub300\ube44(\uc7a5\ub82c\uc655\ud6c4)\uac00 \uc0c1\ubcf5\uc744 \uc785\ub294 \uae30\uac04\uc5d0 \uad00\ud55c \ub17c\uc7c1\uc774\ub2e4. \uc774\ub7ec\ud55c \ub17c\uc7c1\uc774 \ubc1c\uc0dd\ud55c \uae4c\ub2ed\uc740, \uc778\uc870\uc640 \uadf8\uc758 \uc81c 1\ube44\uc600\ub358 \uc778\uc5f4\uc655\ud6c4\uc758 \uccab\uc9f8 \uc544\ub4e4\uc778 \uc18c\ud604\uc138\uc790\uac00 \uc655\uc774 \ub418\uc9c0 \ubabb\ud558\uace0 \ubcd1\uc0ac\ud558\uc5ec \uc18c\ud604\uc138\uc790\uc758 \uc544\ub4e4\uc774 \uc655\uc774 \ub418\uc5b4\uc57c \ud588\uc73c\ub098, \ub458\uc9f8 \uc544\ub4e4\uc774\uc5c8\ub358 \ubd09\ub9bc\ub300\uad70\uc774 \ud6a8\uc885\uc73c\ub85c \uc989\uc704\ud558\uc5ec \uc655\uc774 \ub418\uc5c8\uae30 \ub54c\ubb38\uc774\ub2e4. 1\ucc28 \uc608\uc1a1\uc5d0\uc11c \ud6a8\uc885\uc774 \uc8fd\uc5c8\uc744 \ub54c, \uc11c\uc778\uc740 \ud6a8\uc885\uc774 \ub458\uc9f8\uc544\ub4e4\uc774\ub77c\ub294 \ub370\uc5d0 \ucc29\uc548\ud558\uc5ec 1\ub144 \uc0c1\uc744 \uc8fc\uc7a5\ud558\uc600\uace0, \ub0a8\uc778\uc740 \uc655\uc774\ub77c\ub294 \ub370\uc5d0 \ucc29\uc548\ud558\uc5ec 3\ub144 \uc0c1\uc744 \uc8fc\uc7a5\ud558\uc600\ub2e4. \uacb0\uad6d \uc11c\uc778\uc774 \uc2b9\ub9ac\ud558\uc5ec 1\ub144 \uc0c1\uc744 \ud558\ub294 \uac83\uc73c\ub85c \ud558\uc600\ub2e4. \ud558\uc9c0\ub9cc 2\ucc28 \uc608\uc1a1\uc5d0\uc11c \uc778\uc120\uc655\ud6c4\uac00 \uc8fd\uc5c8\uc744 \ub54c, \uc11c\uc778\uc740 \ud6a8\uc885\uc774 \ucc28\uc790\uc774\ubbc0\ub85c \uc870\ub300\ube44(\uc790\uc758\ub300\ube44)\uac00 9\uac1c\uc6d4\uac04 \uc0c1\ubcf5\uc744 \uc785\uc5b4\uc57c \ud55c\ub2e4\uace0 \ud588\uace0, \ub0a8\uc778\uc740 \ud6a8\uc885\uc744 \uc7a5\uc790\ub85c \uc778\uc815\ud558\uc5ec 1\ub144\uac04 \uc785\uc5b4\uc57c \ud55c\ub2e4\uace0 \ud588\ub2e4. \uc774\ub54c\ub294 \ub0a8\uc778\uc774 \uc2b9\ub9ac\ud558\uc600\ub2e4. \uadf8\ub85c\uc368, \uc11c\uc778\uc740 \uc815\uacc4\uc5d0\uc11c \ucd95\ucd9c\ub418\uc5c8\ub2e4.", "answer": "\uc11c\uc778", "sentence": "1\ucc28 \uc608\uc1a1\uc5d0\uc11c \ud6a8\uc885\uc774 \uc8fd\uc5c8\uc744 \ub54c, \uc11c\uc778 \uc740 \ud6a8\uc885\uc774 \ub458\uc9f8\uc544\ub4e4\uc774\ub77c\ub294 \ub370\uc5d0 \ucc29\uc548\ud558\uc5ec 1\ub144 \uc0c1\uc744 \uc8fc\uc7a5\ud558\uc600\uace0, \ub0a8\uc778\uc740 \uc655\uc774\ub77c\ub294 \ub370\uc5d0 \ucc29\uc548\ud558\uc5ec 3\ub144 \uc0c1\uc744 \uc8fc\uc7a5\ud558\uc600\ub2e4.", "paragraph_sentence": "\uc774\ub54c, \ubd95\ub2f9\uc758 \ubcc0\uc9c8\uc744 \uac00\uc838\uc628 \ub450 \uac00\uc9c0 \ubb38\uc81c\uac00 \uc608\uc1a1\uacfc \ud658\uad6d\uc774\ub2e4. \uc608\uc1a1\uc740, \uc0c1\ubcf5\uc744 \uc785\ub294 \uae30\uac04\uc5d0 \uad00\ud55c \ubb38\uc81c\ub85c, \ud6a8\uc885\uacfc \ud6a8\uc885\uc758 \ube44\uc778 \uc778\uc120\uc655\ud6c4\uac00 \uc2b9\ud558\ud588\uc744 \ub54c, \ud6a8\uc885\uc758 \ubaa8\uce5c\uc774\uc790 \uc120\uc655 \uc778\uc870\uc758 \uacc4\ube44\uc600\ub358 \uc790\uc758\ub300\ube44(\uc7a5\ub82c\uc655\ud6c4)\uac00 \uc0c1\ubcf5\uc744 \uc785\ub294 \uae30\uac04\uc5d0 \uad00\ud55c \ub17c\uc7c1\uc774\ub2e4. \uc774\ub7ec\ud55c \ub17c\uc7c1\uc774 \ubc1c\uc0dd\ud55c \uae4c\ub2ed\uc740, \uc778\uc870\uc640 \uadf8\uc758 \uc81c 1\ube44\uc600\ub358 \uc778\uc5f4\uc655\ud6c4\uc758 \uccab\uc9f8 \uc544\ub4e4\uc778 \uc18c\ud604\uc138\uc790\uac00 \uc655\uc774 \ub418\uc9c0 \ubabb\ud558\uace0 \ubcd1\uc0ac\ud558\uc5ec \uc18c\ud604\uc138\uc790\uc758 \uc544\ub4e4\uc774 \uc655\uc774 \ub418\uc5b4\uc57c \ud588\uc73c\ub098, \ub458\uc9f8 \uc544\ub4e4\uc774\uc5c8\ub358 \ubd09\ub9bc\ub300\uad70\uc774 \ud6a8\uc885\uc73c\ub85c \uc989\uc704\ud558\uc5ec \uc655\uc774 \ub418\uc5c8\uae30 \ub54c\ubb38\uc774\ub2e4. 1\ucc28 \uc608\uc1a1\uc5d0\uc11c \ud6a8\uc885\uc774 \uc8fd\uc5c8\uc744 \ub54c, \uc11c\uc778 \uc740 \ud6a8\uc885\uc774 \ub458\uc9f8\uc544\ub4e4\uc774\ub77c\ub294 \ub370\uc5d0 \ucc29\uc548\ud558\uc5ec 1\ub144 \uc0c1\uc744 \uc8fc\uc7a5\ud558\uc600\uace0, \ub0a8\uc778\uc740 \uc655\uc774\ub77c\ub294 \ub370\uc5d0 \ucc29\uc548\ud558\uc5ec 3\ub144 \uc0c1\uc744 \uc8fc\uc7a5\ud558\uc600\ub2e4. \uacb0\uad6d \uc11c\uc778\uc774 \uc2b9\ub9ac\ud558\uc5ec 1\ub144 \uc0c1\uc744 \ud558\ub294 \uac83\uc73c\ub85c \ud558\uc600\ub2e4. \ud558\uc9c0\ub9cc 2\ucc28 \uc608\uc1a1\uc5d0\uc11c \uc778\uc120\uc655\ud6c4\uac00 \uc8fd\uc5c8\uc744 \ub54c, \uc11c\uc778\uc740 \ud6a8\uc885\uc774 \ucc28\uc790\uc774\ubbc0\ub85c \uc870\ub300\ube44(\uc790\uc758\ub300\ube44)\uac00 9\uac1c\uc6d4\uac04 \uc0c1\ubcf5\uc744 \uc785\uc5b4\uc57c \ud55c\ub2e4\uace0 \ud588\uace0, \ub0a8\uc778\uc740 \ud6a8\uc885\uc744 \uc7a5\uc790\ub85c \uc778\uc815\ud558\uc5ec 1\ub144\uac04 \uc785\uc5b4\uc57c \ud55c\ub2e4\uace0 \ud588\ub2e4. \uc774\ub54c\ub294 \ub0a8\uc778\uc774 \uc2b9\ub9ac\ud558\uc600\ub2e4. \uadf8\ub85c\uc368, \uc11c\uc778\uc740 \uc815\uacc4\uc5d0\uc11c \ucd95\ucd9c\ub418\uc5c8\ub2e4.", "paragraph_answer": "\uc774\ub54c, \ubd95\ub2f9\uc758 \ubcc0\uc9c8\uc744 \uac00\uc838\uc628 \ub450 \uac00\uc9c0 \ubb38\uc81c\uac00 \uc608\uc1a1\uacfc \ud658\uad6d\uc774\ub2e4. \uc608\uc1a1\uc740, \uc0c1\ubcf5\uc744 \uc785\ub294 \uae30\uac04\uc5d0 \uad00\ud55c \ubb38\uc81c\ub85c, \ud6a8\uc885\uacfc \ud6a8\uc885\uc758 \ube44\uc778 \uc778\uc120\uc655\ud6c4\uac00 \uc2b9\ud558\ud588\uc744 \ub54c, \ud6a8\uc885\uc758 \ubaa8\uce5c\uc774\uc790 \uc120\uc655 \uc778\uc870\uc758 \uacc4\ube44\uc600\ub358 \uc790\uc758\ub300\ube44(\uc7a5\ub82c\uc655\ud6c4)\uac00 \uc0c1\ubcf5\uc744 \uc785\ub294 \uae30\uac04\uc5d0 \uad00\ud55c \ub17c\uc7c1\uc774\ub2e4. \uc774\ub7ec\ud55c \ub17c\uc7c1\uc774 \ubc1c\uc0dd\ud55c \uae4c\ub2ed\uc740, \uc778\uc870\uc640 \uadf8\uc758 \uc81c 1\ube44\uc600\ub358 \uc778\uc5f4\uc655\ud6c4\uc758 \uccab\uc9f8 \uc544\ub4e4\uc778 \uc18c\ud604\uc138\uc790\uac00 \uc655\uc774 \ub418\uc9c0 \ubabb\ud558\uace0 \ubcd1\uc0ac\ud558\uc5ec \uc18c\ud604\uc138\uc790\uc758 \uc544\ub4e4\uc774 \uc655\uc774 \ub418\uc5b4\uc57c \ud588\uc73c\ub098, \ub458\uc9f8 \uc544\ub4e4\uc774\uc5c8\ub358 \ubd09\ub9bc\ub300\uad70\uc774 \ud6a8\uc885\uc73c\ub85c \uc989\uc704\ud558\uc5ec \uc655\uc774 \ub418\uc5c8\uae30 \ub54c\ubb38\uc774\ub2e4. 1\ucc28 \uc608\uc1a1\uc5d0\uc11c \ud6a8\uc885\uc774 \uc8fd\uc5c8\uc744 \ub54c, \uc11c\uc778 \uc740 \ud6a8\uc885\uc774 \ub458\uc9f8\uc544\ub4e4\uc774\ub77c\ub294 \ub370\uc5d0 \ucc29\uc548\ud558\uc5ec 1\ub144 \uc0c1\uc744 \uc8fc\uc7a5\ud558\uc600\uace0, \ub0a8\uc778\uc740 \uc655\uc774\ub77c\ub294 \ub370\uc5d0 \ucc29\uc548\ud558\uc5ec 3\ub144 \uc0c1\uc744 \uc8fc\uc7a5\ud558\uc600\ub2e4. \uacb0\uad6d \uc11c\uc778\uc774 \uc2b9\ub9ac\ud558\uc5ec 1\ub144 \uc0c1\uc744 \ud558\ub294 \uac83\uc73c\ub85c \ud558\uc600\ub2e4. \ud558\uc9c0\ub9cc 2\ucc28 \uc608\uc1a1\uc5d0\uc11c \uc778\uc120\uc655\ud6c4\uac00 \uc8fd\uc5c8\uc744 \ub54c, \uc11c\uc778\uc740 \ud6a8\uc885\uc774 \ucc28\uc790\uc774\ubbc0\ub85c \uc870\ub300\ube44(\uc790\uc758\ub300\ube44)\uac00 9\uac1c\uc6d4\uac04 \uc0c1\ubcf5\uc744 \uc785\uc5b4\uc57c \ud55c\ub2e4\uace0 \ud588\uace0, \ub0a8\uc778\uc740 \ud6a8\uc885\uc744 \uc7a5\uc790\ub85c \uc778\uc815\ud558\uc5ec 1\ub144\uac04 \uc785\uc5b4\uc57c \ud55c\ub2e4\uace0 \ud588\ub2e4. \uc774\ub54c\ub294 \ub0a8\uc778\uc774 \uc2b9\ub9ac\ud558\uc600\ub2e4. \uadf8\ub85c\uc368, \uc11c\uc778\uc740 \uc815\uacc4\uc5d0\uc11c \ucd95\ucd9c\ub418\uc5c8\ub2e4.", "sentence_answer": "1\ucc28 \uc608\uc1a1\uc5d0\uc11c \ud6a8\uc885\uc774 \uc8fd\uc5c8\uc744 \ub54c, \uc11c\uc778 \uc740 \ud6a8\uc885\uc774 \ub458\uc9f8\uc544\ub4e4\uc774\ub77c\ub294 \ub370\uc5d0 \ucc29\uc548\ud558\uc5ec 1\ub144 \uc0c1\uc744 \uc8fc\uc7a5\ud558\uc600\uace0, \ub0a8\uc778\uc740 \uc655\uc774\ub77c\ub294 \ub370\uc5d0 \ucc29\uc548\ud558\uc5ec 3\ub144 \uc0c1\uc744 \uc8fc\uc7a5\ud558\uc600\ub2e4.", "paragraph_id": "6509326-3-0", "paragraph_question": "question: 1\ucc28 \uc608\uc1a1\uc5d0\uc11c \uc2b9\ub9ac\ud55c \ubd95\ub2f9\uc740?, context: \uc774\ub54c, \ubd95\ub2f9\uc758 \ubcc0\uc9c8\uc744 \uac00\uc838\uc628 \ub450 \uac00\uc9c0 \ubb38\uc81c\uac00 \uc608\uc1a1\uacfc \ud658\uad6d\uc774\ub2e4. \uc608\uc1a1\uc740, \uc0c1\ubcf5\uc744 \uc785\ub294 \uae30\uac04\uc5d0 \uad00\ud55c \ubb38\uc81c\ub85c, \ud6a8\uc885\uacfc \ud6a8\uc885\uc758 \ube44\uc778 \uc778\uc120\uc655\ud6c4\uac00 \uc2b9\ud558\ud588\uc744 \ub54c, \ud6a8\uc885\uc758 \ubaa8\uce5c\uc774\uc790 \uc120\uc655 \uc778\uc870\uc758 \uacc4\ube44\uc600\ub358 \uc790\uc758\ub300\ube44(\uc7a5\ub82c\uc655\ud6c4)\uac00 \uc0c1\ubcf5\uc744 \uc785\ub294 \uae30\uac04\uc5d0 \uad00\ud55c \ub17c\uc7c1\uc774\ub2e4. \uc774\ub7ec\ud55c \ub17c\uc7c1\uc774 \ubc1c\uc0dd\ud55c \uae4c\ub2ed\uc740, \uc778\uc870\uc640 \uadf8\uc758 \uc81c 1\ube44\uc600\ub358 \uc778\uc5f4\uc655\ud6c4\uc758 \uccab\uc9f8 \uc544\ub4e4\uc778 \uc18c\ud604\uc138\uc790\uac00 \uc655\uc774 \ub418\uc9c0 \ubabb\ud558\uace0 \ubcd1\uc0ac\ud558\uc5ec \uc18c\ud604\uc138\uc790\uc758 \uc544\ub4e4\uc774 \uc655\uc774 \ub418\uc5b4\uc57c \ud588\uc73c\ub098, \ub458\uc9f8 \uc544\ub4e4\uc774\uc5c8\ub358 \ubd09\ub9bc\ub300\uad70\uc774 \ud6a8\uc885\uc73c\ub85c \uc989\uc704\ud558\uc5ec \uc655\uc774 \ub418\uc5c8\uae30 \ub54c\ubb38\uc774\ub2e4. 1\ucc28 \uc608\uc1a1\uc5d0\uc11c \ud6a8\uc885\uc774 \uc8fd\uc5c8\uc744 \ub54c, \uc11c\uc778\uc740 \ud6a8\uc885\uc774 \ub458\uc9f8\uc544\ub4e4\uc774\ub77c\ub294 \ub370\uc5d0 \ucc29\uc548\ud558\uc5ec 1\ub144 \uc0c1\uc744 \uc8fc\uc7a5\ud558\uc600\uace0, \ub0a8\uc778\uc740 \uc655\uc774\ub77c\ub294 \ub370\uc5d0 \ucc29\uc548\ud558\uc5ec 3\ub144 \uc0c1\uc744 \uc8fc\uc7a5\ud558\uc600\ub2e4. \uacb0\uad6d \uc11c\uc778\uc774 \uc2b9\ub9ac\ud558\uc5ec 1\ub144 \uc0c1\uc744 \ud558\ub294 \uac83\uc73c\ub85c \ud558\uc600\ub2e4. \ud558\uc9c0\ub9cc 2\ucc28 \uc608\uc1a1\uc5d0\uc11c \uc778\uc120\uc655\ud6c4\uac00 \uc8fd\uc5c8\uc744 \ub54c, \uc11c\uc778\uc740 \ud6a8\uc885\uc774 \ucc28\uc790\uc774\ubbc0\ub85c \uc870\ub300\ube44(\uc790\uc758\ub300\ube44)\uac00 9\uac1c\uc6d4\uac04 \uc0c1\ubcf5\uc744 \uc785\uc5b4\uc57c \ud55c\ub2e4\uace0 \ud588\uace0, \ub0a8\uc778\uc740 \ud6a8\uc885\uc744 \uc7a5\uc790\ub85c \uc778\uc815\ud558\uc5ec 1\ub144\uac04 \uc785\uc5b4\uc57c \ud55c\ub2e4\uace0 \ud588\ub2e4. \uc774\ub54c\ub294 \ub0a8\uc778\uc774 \uc2b9\ub9ac\ud558\uc600\ub2e4. \uadf8\ub85c\uc368, \uc11c\uc778\uc740 \uc815\uacc4\uc5d0\uc11c \ucd95\ucd9c\ub418\uc5c8\ub2e4."} {"question": "\uce7c\ub9ac\uc2a4\ud1a0\ub294 \ubb34\uc5c7\uc73c\ub85c \uc778\ud574 \ubc1c\uc0dd\ub41c \uc5f4\uc774 \ud45c\uba74 \uadfc\ucc98\uc5d0\uc11c \uc2dd\ub294\uacfc\uc815\uc744 \uac70\ucce4\ub294\uac00?", "paragraph": "\uac15\ucc29 \uc774\ud6c4 \ud615\uc131\ub41c \uce7c\ub9ac\uc2a4\ud1a0\uc5d0\uc11c\ub294, \ubc29\uc0ac\uc131 \ubd95\uad34\ub85c \uc778\ud574 \ubc1c\uc0dd\ub41c \uc5f4\uc774 \uc5f4\uc804\ub3c4\uc640 \uace0\uccb4/\uc900\uace0\uccb4 \ub300\ub958\ub97c \ud1b5\ud574 \ud45c\uba74 \uadfc\ucc98\uc5d0\uc11c \uc2dd\ub294 \uacfc\uc815\uc744 \uac70\uccd0\uc654\ub2e4. \uc5bc\uc74c \uc18d\uc5d0\uc11c\uc758 \uc900\uace0\uccb4 \ub300\ub958\ub294 \ubaa8\ub4e0 \uc5bc\uc74c \uc704\uc131\ub4e4\uc744 \uae30\uc220\ud55c \uc774\ub860\ub4e4\uc5d0\uc11c \ub098\ud0c0\ub098\ub294 \uc8fc\uc694 \ubd88\ud655\uc815\uc131\uc774\ub2e4. \uc774\ub294 \uc5bc\uc74c\uc758 \uc810\uc131\uc5d0 \ub530\ub978, \uc628\ub3c4\uac00 \ub179\ub294\uc810\uc5d0 \uac00\uae4c\uc6cc\uc84c\uc744 \ub54c \ub098\ud0c0\ub098\ub294 \uac83\uc73c\ub85c \uc54c\ub824\uc838 \uc788\ub2e4. \uc5bc\uc74c \ucc9c\uccb4\ub4e4\uc5d0\uc11c\uc758 \uc900\uace0\uccb4 \ub300\ub958\ub294 \uc5bc\uc74c\uc774 1\ub144\uc5d0 \uc57d 1 cm\uac00\ub7c9 \uc6c0\uc9c1\uc774\ub294 \ub9e4\uc6b0 \ub290\ub9b0 \uacfc\uc815\uc774\uc9c0\ub9cc, \ub9e4\uc6b0 \uae34 \uae30\uac04\uc73c\ub85c \ubc14\ub77c\ubcf4\uac8c \ub418\uba74 \ub9e4\uc6b0 \ud6a8\uacfc\uc801\uc778 \ub0c9\uac01 \ubc29\ubc95\uc774\ub2e4. \uc774\ub294 \uc18c\uc704 \u201cstagnant lid regime\u201d\uc774\ub77c\uace0 \ubd88\ub9ac\ub294 \uacfc\uc815\uc5d0 \uc758\ud574 \uc9c4\ud589\ub41c\ub2e4\uace0 \ucd94\uce21\ub418\uba70, \uc774 \uacfc\uc815\uc740 \ubee3\ubee3\ud558\uace0 \ucc28\uac00\uc6b4 \uce7c\ub9ac\uc2a4\ud1a0\uc758 \uc678\uacfd\uce35\uc774 \ub300\ub958\ub97c \ud1b5\ud558\uc9c0 \uc54a\uace0 \uc5f4\uc744 \uc804\ub3c4\ud558\ub294 \ubc18\uba74 \uadf8 \ubc11\uc758 \uc5bc\uc74c\uce35\uc740 \uc900\uace0\uccb4 \ub300\ub958\ub97c \ud558\ub294 \uac83\uc744 \ub9d0\ud55c\ub2e4. \uce7c\ub9ac\uc2a4\ud1a0\uc5d0\uc11c\ub294, \uc678\uacfd\uc758 \uc804\ub3c4\uce35\uc740 \ucc28\uac11\uace0 \ubee3\ubee3\ud55c, \ub450\uaed8 100 km \uac00\ub7c9\uc758 \uc554\uad8c\uc5d0 \ud574\ub2f9\ub41c\ub2e4. \uc774 \ud604\uc0c1\uc744 \ud1b5\ud574\uc11c \uce7c\ub9ac\uc2a4\ud1a0 \ud45c\uba74\uc5d0\uc11c \ub098\ud0c0\ub098\ub294 \ub0b4\uc778\uc131 \uc9c0\uc9c8\ud559\uc801 \ud65c\ub3d9\uc758 \ud754\uc801\ub4e4\uc774 \ubd80\uc871\ud55c \uc774\uc720\ub97c \uc124\uba85\ud560 \uc218 \uc788\ub2e4. \uce7c\ub9ac\uc2a4\ud1a0 \ub0b4\ubd80\uc758 \ub300\ub958\ub294 \uce35\uc744 \uc774\ub8e8\uace0 \uc788\uc744 \uc218 \uc788\ub294\ub370, \uadf8 \uc774\uc720\ub294 \ub0b4\ubd80\uc758 \uac15\ud55c \uc555\ub825\uc5d0 \uc758\ud574 \uc5bc\uc74c\uc774 \ud45c\uba74\uc758 \uc5bc\uc74c Ih\ubd80\ud130 \uc911\uc2ec\ubd80\uc758 \uc5bc\uc74c VII\uae4c\uc9c0 \ub2e4\uc591\ud55c \ud615\ud0dc\ub85c \uc874\uc7ac\ud558\uae30 \ub54c\ubb38\uc774\ub2e4. \uce7c\ub9ac\uc2a4\ud1a0 \ub0b4\ubd80 \uc900\uace0\uccb4 \ub300\ub958\uac00 \ucd08\uae30\uc5d0 \uc2dc\uc791\ud568\uc5d0 \ub530\ub77c, \ub300\uaddc\ubaa8\uc758 \uc5bc\uc74c \uc735\ud574\uc640, \ud575\uacfc \ub9e8\ud2c0\uc744 \ud615\uc131\ud588\uc744 \ud589\uc131 \ubd84\ud654\ub97c \ub9c9\uc558\ub2e4. \ud558\uc9c0\ub9cc \ub300\ub958 \uacfc\uc815\uc5d0 \uc758\ud574, \ubd80\ubd84\uc801\uc774\uace0 \ub9e4\uc6b0 \ub290\ub9b0 \ubd84\ub9ac/\ubd84\ud654\uac00 \uba87\uc2ed\uc5b5 \ub144 \uaddc\ubaa8\uc5d0\uc11c \uc9c4\ud589\ub418\uc5b4 \uc654\uc73c\uba70, \ud604\uc7ac\ub3c4 \uacc4\uc18d\ud574\uc11c \uc9c4\ud589\ub418\uace0 \uc788\uc744 \uc218\ub3c4 \uc788\ub2e4.", "answer": "\ubc29\uc0ac\uc131 \ubd95\uad34", "sentence": "\uac15\ucc29 \uc774\ud6c4 \ud615\uc131\ub41c \uce7c\ub9ac\uc2a4\ud1a0\uc5d0\uc11c\ub294, \ubc29\uc0ac\uc131 \ubd95\uad34 \ub85c \uc778\ud574 \ubc1c\uc0dd\ub41c \uc5f4\uc774 \uc5f4\uc804\ub3c4\uc640 \uace0\uccb4/\uc900\uace0\uccb4 \ub300\ub958\ub97c \ud1b5\ud574 \ud45c\uba74 \uadfc\ucc98\uc5d0\uc11c \uc2dd\ub294 \uacfc\uc815\uc744 \uac70\uccd0\uc654\ub2e4.", "paragraph_sentence": " \uac15\ucc29 \uc774\ud6c4 \ud615\uc131\ub41c \uce7c\ub9ac\uc2a4\ud1a0\uc5d0\uc11c\ub294, \ubc29\uc0ac\uc131 \ubd95\uad34 \ub85c \uc778\ud574 \ubc1c\uc0dd\ub41c \uc5f4\uc774 \uc5f4\uc804\ub3c4\uc640 \uace0\uccb4/\uc900\uace0\uccb4 \ub300\ub958\ub97c \ud1b5\ud574 \ud45c\uba74 \uadfc\ucc98\uc5d0\uc11c \uc2dd\ub294 \uacfc\uc815\uc744 \uac70\uccd0\uc654\ub2e4. \uc5bc\uc74c \uc18d\uc5d0\uc11c\uc758 \uc900\uace0\uccb4 \ub300\ub958\ub294 \ubaa8\ub4e0 \uc5bc\uc74c \uc704\uc131\ub4e4\uc744 \uae30\uc220\ud55c \uc774\ub860\ub4e4\uc5d0\uc11c \ub098\ud0c0\ub098\ub294 \uc8fc\uc694 \ubd88\ud655\uc815\uc131\uc774\ub2e4. \uc774\ub294 \uc5bc\uc74c\uc758 \uc810\uc131\uc5d0 \ub530\ub978, \uc628\ub3c4\uac00 \ub179\ub294\uc810\uc5d0 \uac00\uae4c\uc6cc\uc84c\uc744 \ub54c \ub098\ud0c0\ub098\ub294 \uac83\uc73c\ub85c \uc54c\ub824\uc838 \uc788\ub2e4. \uc5bc\uc74c \ucc9c\uccb4\ub4e4\uc5d0\uc11c\uc758 \uc900\uace0\uccb4 \ub300\ub958\ub294 \uc5bc\uc74c\uc774 1\ub144\uc5d0 \uc57d 1 cm\uac00\ub7c9 \uc6c0\uc9c1\uc774\ub294 \ub9e4\uc6b0 \ub290\ub9b0 \uacfc\uc815\uc774\uc9c0\ub9cc, \ub9e4\uc6b0 \uae34 \uae30\uac04\uc73c\ub85c \ubc14\ub77c\ubcf4\uac8c \ub418\uba74 \ub9e4\uc6b0 \ud6a8\uacfc\uc801\uc778 \ub0c9\uac01 \ubc29\ubc95\uc774\ub2e4. \uc774\ub294 \uc18c\uc704 \u201cstagnant lid regime\u201d\uc774\ub77c\uace0 \ubd88\ub9ac\ub294 \uacfc\uc815\uc5d0 \uc758\ud574 \uc9c4\ud589\ub41c\ub2e4\uace0 \ucd94\uce21\ub418\uba70, \uc774 \uacfc\uc815\uc740 \ubee3\ubee3\ud558\uace0 \ucc28\uac00\uc6b4 \uce7c\ub9ac\uc2a4\ud1a0\uc758 \uc678\uacfd\uce35\uc774 \ub300\ub958\ub97c \ud1b5\ud558\uc9c0 \uc54a\uace0 \uc5f4\uc744 \uc804\ub3c4\ud558\ub294 \ubc18\uba74 \uadf8 \ubc11\uc758 \uc5bc\uc74c\uce35\uc740 \uc900\uace0\uccb4 \ub300\ub958\ub97c \ud558\ub294 \uac83\uc744 \ub9d0\ud55c\ub2e4. \uce7c\ub9ac\uc2a4\ud1a0\uc5d0\uc11c\ub294, \uc678\uacfd\uc758 \uc804\ub3c4\uce35\uc740 \ucc28\uac11\uace0 \ubee3\ubee3\ud55c, \ub450\uaed8 100 km \uac00\ub7c9\uc758 \uc554\uad8c\uc5d0 \ud574\ub2f9\ub41c\ub2e4. \uc774 \ud604\uc0c1\uc744 \ud1b5\ud574\uc11c \uce7c\ub9ac\uc2a4\ud1a0 \ud45c\uba74\uc5d0\uc11c \ub098\ud0c0\ub098\ub294 \ub0b4\uc778\uc131 \uc9c0\uc9c8\ud559\uc801 \ud65c\ub3d9\uc758 \ud754\uc801\ub4e4\uc774 \ubd80\uc871\ud55c \uc774\uc720\ub97c \uc124\uba85\ud560 \uc218 \uc788\ub2e4. \uce7c\ub9ac\uc2a4\ud1a0 \ub0b4\ubd80\uc758 \ub300\ub958\ub294 \uce35\uc744 \uc774\ub8e8\uace0 \uc788\uc744 \uc218 \uc788\ub294\ub370, \uadf8 \uc774\uc720\ub294 \ub0b4\ubd80\uc758 \uac15\ud55c \uc555\ub825\uc5d0 \uc758\ud574 \uc5bc\uc74c\uc774 \ud45c\uba74\uc758 \uc5bc\uc74c Ih\ubd80\ud130 \uc911\uc2ec\ubd80\uc758 \uc5bc\uc74c VII\uae4c\uc9c0 \ub2e4\uc591\ud55c \ud615\ud0dc\ub85c \uc874\uc7ac\ud558\uae30 \ub54c\ubb38\uc774\ub2e4. \uce7c\ub9ac\uc2a4\ud1a0 \ub0b4\ubd80 \uc900\uace0\uccb4 \ub300\ub958\uac00 \ucd08\uae30\uc5d0 \uc2dc\uc791\ud568\uc5d0 \ub530\ub77c, \ub300\uaddc\ubaa8\uc758 \uc5bc\uc74c \uc735\ud574\uc640, \ud575\uacfc \ub9e8\ud2c0\uc744 \ud615\uc131\ud588\uc744 \ud589\uc131 \ubd84\ud654\ub97c \ub9c9\uc558\ub2e4. \ud558\uc9c0\ub9cc \ub300\ub958 \uacfc\uc815\uc5d0 \uc758\ud574, \ubd80\ubd84\uc801\uc774\uace0 \ub9e4\uc6b0 \ub290\ub9b0 \ubd84\ub9ac/\ubd84\ud654\uac00 \uba87\uc2ed\uc5b5 \ub144 \uaddc\ubaa8\uc5d0\uc11c \uc9c4\ud589\ub418\uc5b4 \uc654\uc73c\uba70, \ud604\uc7ac\ub3c4 \uacc4\uc18d\ud574\uc11c \uc9c4\ud589\ub418\uace0 \uc788\uc744 \uc218\ub3c4 \uc788\ub2e4.", "paragraph_answer": "\uac15\ucc29 \uc774\ud6c4 \ud615\uc131\ub41c \uce7c\ub9ac\uc2a4\ud1a0\uc5d0\uc11c\ub294, \ubc29\uc0ac\uc131 \ubd95\uad34 \ub85c \uc778\ud574 \ubc1c\uc0dd\ub41c \uc5f4\uc774 \uc5f4\uc804\ub3c4\uc640 \uace0\uccb4/\uc900\uace0\uccb4 \ub300\ub958\ub97c \ud1b5\ud574 \ud45c\uba74 \uadfc\ucc98\uc5d0\uc11c \uc2dd\ub294 \uacfc\uc815\uc744 \uac70\uccd0\uc654\ub2e4. \uc5bc\uc74c \uc18d\uc5d0\uc11c\uc758 \uc900\uace0\uccb4 \ub300\ub958\ub294 \ubaa8\ub4e0 \uc5bc\uc74c \uc704\uc131\ub4e4\uc744 \uae30\uc220\ud55c \uc774\ub860\ub4e4\uc5d0\uc11c \ub098\ud0c0\ub098\ub294 \uc8fc\uc694 \ubd88\ud655\uc815\uc131\uc774\ub2e4. \uc774\ub294 \uc5bc\uc74c\uc758 \uc810\uc131\uc5d0 \ub530\ub978, \uc628\ub3c4\uac00 \ub179\ub294\uc810\uc5d0 \uac00\uae4c\uc6cc\uc84c\uc744 \ub54c \ub098\ud0c0\ub098\ub294 \uac83\uc73c\ub85c \uc54c\ub824\uc838 \uc788\ub2e4. \uc5bc\uc74c \ucc9c\uccb4\ub4e4\uc5d0\uc11c\uc758 \uc900\uace0\uccb4 \ub300\ub958\ub294 \uc5bc\uc74c\uc774 1\ub144\uc5d0 \uc57d 1 cm\uac00\ub7c9 \uc6c0\uc9c1\uc774\ub294 \ub9e4\uc6b0 \ub290\ub9b0 \uacfc\uc815\uc774\uc9c0\ub9cc, \ub9e4\uc6b0 \uae34 \uae30\uac04\uc73c\ub85c \ubc14\ub77c\ubcf4\uac8c \ub418\uba74 \ub9e4\uc6b0 \ud6a8\uacfc\uc801\uc778 \ub0c9\uac01 \ubc29\ubc95\uc774\ub2e4. \uc774\ub294 \uc18c\uc704 \u201cstagnant lid regime\u201d\uc774\ub77c\uace0 \ubd88\ub9ac\ub294 \uacfc\uc815\uc5d0 \uc758\ud574 \uc9c4\ud589\ub41c\ub2e4\uace0 \ucd94\uce21\ub418\uba70, \uc774 \uacfc\uc815\uc740 \ubee3\ubee3\ud558\uace0 \ucc28\uac00\uc6b4 \uce7c\ub9ac\uc2a4\ud1a0\uc758 \uc678\uacfd\uce35\uc774 \ub300\ub958\ub97c \ud1b5\ud558\uc9c0 \uc54a\uace0 \uc5f4\uc744 \uc804\ub3c4\ud558\ub294 \ubc18\uba74 \uadf8 \ubc11\uc758 \uc5bc\uc74c\uce35\uc740 \uc900\uace0\uccb4 \ub300\ub958\ub97c \ud558\ub294 \uac83\uc744 \ub9d0\ud55c\ub2e4. \uce7c\ub9ac\uc2a4\ud1a0\uc5d0\uc11c\ub294, \uc678\uacfd\uc758 \uc804\ub3c4\uce35\uc740 \ucc28\uac11\uace0 \ubee3\ubee3\ud55c, \ub450\uaed8 100 km \uac00\ub7c9\uc758 \uc554\uad8c\uc5d0 \ud574\ub2f9\ub41c\ub2e4. \uc774 \ud604\uc0c1\uc744 \ud1b5\ud574\uc11c \uce7c\ub9ac\uc2a4\ud1a0 \ud45c\uba74\uc5d0\uc11c \ub098\ud0c0\ub098\ub294 \ub0b4\uc778\uc131 \uc9c0\uc9c8\ud559\uc801 \ud65c\ub3d9\uc758 \ud754\uc801\ub4e4\uc774 \ubd80\uc871\ud55c \uc774\uc720\ub97c \uc124\uba85\ud560 \uc218 \uc788\ub2e4. \uce7c\ub9ac\uc2a4\ud1a0 \ub0b4\ubd80\uc758 \ub300\ub958\ub294 \uce35\uc744 \uc774\ub8e8\uace0 \uc788\uc744 \uc218 \uc788\ub294\ub370, \uadf8 \uc774\uc720\ub294 \ub0b4\ubd80\uc758 \uac15\ud55c \uc555\ub825\uc5d0 \uc758\ud574 \uc5bc\uc74c\uc774 \ud45c\uba74\uc758 \uc5bc\uc74c Ih\ubd80\ud130 \uc911\uc2ec\ubd80\uc758 \uc5bc\uc74c VII\uae4c\uc9c0 \ub2e4\uc591\ud55c \ud615\ud0dc\ub85c \uc874\uc7ac\ud558\uae30 \ub54c\ubb38\uc774\ub2e4. \uce7c\ub9ac\uc2a4\ud1a0 \ub0b4\ubd80 \uc900\uace0\uccb4 \ub300\ub958\uac00 \ucd08\uae30\uc5d0 \uc2dc\uc791\ud568\uc5d0 \ub530\ub77c, \ub300\uaddc\ubaa8\uc758 \uc5bc\uc74c \uc735\ud574\uc640, \ud575\uacfc \ub9e8\ud2c0\uc744 \ud615\uc131\ud588\uc744 \ud589\uc131 \ubd84\ud654\ub97c \ub9c9\uc558\ub2e4. \ud558\uc9c0\ub9cc \ub300\ub958 \uacfc\uc815\uc5d0 \uc758\ud574, \ubd80\ubd84\uc801\uc774\uace0 \ub9e4\uc6b0 \ub290\ub9b0 \ubd84\ub9ac/\ubd84\ud654\uac00 \uba87\uc2ed\uc5b5 \ub144 \uaddc\ubaa8\uc5d0\uc11c \uc9c4\ud589\ub418\uc5b4 \uc654\uc73c\uba70, \ud604\uc7ac\ub3c4 \uacc4\uc18d\ud574\uc11c \uc9c4\ud589\ub418\uace0 \uc788\uc744 \uc218\ub3c4 \uc788\ub2e4.", "sentence_answer": "\uac15\ucc29 \uc774\ud6c4 \ud615\uc131\ub41c \uce7c\ub9ac\uc2a4\ud1a0\uc5d0\uc11c\ub294, \ubc29\uc0ac\uc131 \ubd95\uad34 \ub85c \uc778\ud574 \ubc1c\uc0dd\ub41c \uc5f4\uc774 \uc5f4\uc804\ub3c4\uc640 \uace0\uccb4/\uc900\uace0\uccb4 \ub300\ub958\ub97c \ud1b5\ud574 \ud45c\uba74 \uadfc\ucc98\uc5d0\uc11c \uc2dd\ub294 \uacfc\uc815\uc744 \uac70\uccd0\uc654\ub2e4.", "paragraph_id": "6208599-9-0", "paragraph_question": "question: \uce7c\ub9ac\uc2a4\ud1a0\ub294 \ubb34\uc5c7\uc73c\ub85c \uc778\ud574 \ubc1c\uc0dd\ub41c \uc5f4\uc774 \ud45c\uba74 \uadfc\ucc98\uc5d0\uc11c \uc2dd\ub294\uacfc\uc815\uc744 \uac70\ucce4\ub294\uac00?, context: \uac15\ucc29 \uc774\ud6c4 \ud615\uc131\ub41c \uce7c\ub9ac\uc2a4\ud1a0\uc5d0\uc11c\ub294, \ubc29\uc0ac\uc131 \ubd95\uad34\ub85c \uc778\ud574 \ubc1c\uc0dd\ub41c \uc5f4\uc774 \uc5f4\uc804\ub3c4\uc640 \uace0\uccb4/\uc900\uace0\uccb4 \ub300\ub958\ub97c \ud1b5\ud574 \ud45c\uba74 \uadfc\ucc98\uc5d0\uc11c \uc2dd\ub294 \uacfc\uc815\uc744 \uac70\uccd0\uc654\ub2e4. \uc5bc\uc74c \uc18d\uc5d0\uc11c\uc758 \uc900\uace0\uccb4 \ub300\ub958\ub294 \ubaa8\ub4e0 \uc5bc\uc74c \uc704\uc131\ub4e4\uc744 \uae30\uc220\ud55c \uc774\ub860\ub4e4\uc5d0\uc11c \ub098\ud0c0\ub098\ub294 \uc8fc\uc694 \ubd88\ud655\uc815\uc131\uc774\ub2e4. \uc774\ub294 \uc5bc\uc74c\uc758 \uc810\uc131\uc5d0 \ub530\ub978, \uc628\ub3c4\uac00 \ub179\ub294\uc810\uc5d0 \uac00\uae4c\uc6cc\uc84c\uc744 \ub54c \ub098\ud0c0\ub098\ub294 \uac83\uc73c\ub85c \uc54c\ub824\uc838 \uc788\ub2e4. \uc5bc\uc74c \ucc9c\uccb4\ub4e4\uc5d0\uc11c\uc758 \uc900\uace0\uccb4 \ub300\ub958\ub294 \uc5bc\uc74c\uc774 1\ub144\uc5d0 \uc57d 1 cm\uac00\ub7c9 \uc6c0\uc9c1\uc774\ub294 \ub9e4\uc6b0 \ub290\ub9b0 \uacfc\uc815\uc774\uc9c0\ub9cc, \ub9e4\uc6b0 \uae34 \uae30\uac04\uc73c\ub85c \ubc14\ub77c\ubcf4\uac8c \ub418\uba74 \ub9e4\uc6b0 \ud6a8\uacfc\uc801\uc778 \ub0c9\uac01 \ubc29\ubc95\uc774\ub2e4. \uc774\ub294 \uc18c\uc704 \u201cstagnant lid regime\u201d\uc774\ub77c\uace0 \ubd88\ub9ac\ub294 \uacfc\uc815\uc5d0 \uc758\ud574 \uc9c4\ud589\ub41c\ub2e4\uace0 \ucd94\uce21\ub418\uba70, \uc774 \uacfc\uc815\uc740 \ubee3\ubee3\ud558\uace0 \ucc28\uac00\uc6b4 \uce7c\ub9ac\uc2a4\ud1a0\uc758 \uc678\uacfd\uce35\uc774 \ub300\ub958\ub97c \ud1b5\ud558\uc9c0 \uc54a\uace0 \uc5f4\uc744 \uc804\ub3c4\ud558\ub294 \ubc18\uba74 \uadf8 \ubc11\uc758 \uc5bc\uc74c\uce35\uc740 \uc900\uace0\uccb4 \ub300\ub958\ub97c \ud558\ub294 \uac83\uc744 \ub9d0\ud55c\ub2e4. \uce7c\ub9ac\uc2a4\ud1a0\uc5d0\uc11c\ub294, \uc678\uacfd\uc758 \uc804\ub3c4\uce35\uc740 \ucc28\uac11\uace0 \ubee3\ubee3\ud55c, \ub450\uaed8 100 km \uac00\ub7c9\uc758 \uc554\uad8c\uc5d0 \ud574\ub2f9\ub41c\ub2e4. \uc774 \ud604\uc0c1\uc744 \ud1b5\ud574\uc11c \uce7c\ub9ac\uc2a4\ud1a0 \ud45c\uba74\uc5d0\uc11c \ub098\ud0c0\ub098\ub294 \ub0b4\uc778\uc131 \uc9c0\uc9c8\ud559\uc801 \ud65c\ub3d9\uc758 \ud754\uc801\ub4e4\uc774 \ubd80\uc871\ud55c \uc774\uc720\ub97c \uc124\uba85\ud560 \uc218 \uc788\ub2e4. \uce7c\ub9ac\uc2a4\ud1a0 \ub0b4\ubd80\uc758 \ub300\ub958\ub294 \uce35\uc744 \uc774\ub8e8\uace0 \uc788\uc744 \uc218 \uc788\ub294\ub370, \uadf8 \uc774\uc720\ub294 \ub0b4\ubd80\uc758 \uac15\ud55c \uc555\ub825\uc5d0 \uc758\ud574 \uc5bc\uc74c\uc774 \ud45c\uba74\uc758 \uc5bc\uc74c Ih\ubd80\ud130 \uc911\uc2ec\ubd80\uc758 \uc5bc\uc74c VII\uae4c\uc9c0 \ub2e4\uc591\ud55c \ud615\ud0dc\ub85c \uc874\uc7ac\ud558\uae30 \ub54c\ubb38\uc774\ub2e4. \uce7c\ub9ac\uc2a4\ud1a0 \ub0b4\ubd80 \uc900\uace0\uccb4 \ub300\ub958\uac00 \ucd08\uae30\uc5d0 \uc2dc\uc791\ud568\uc5d0 \ub530\ub77c, \ub300\uaddc\ubaa8\uc758 \uc5bc\uc74c \uc735\ud574\uc640, \ud575\uacfc \ub9e8\ud2c0\uc744 \ud615\uc131\ud588\uc744 \ud589\uc131 \ubd84\ud654\ub97c \ub9c9\uc558\ub2e4. \ud558\uc9c0\ub9cc \ub300\ub958 \uacfc\uc815\uc5d0 \uc758\ud574, \ubd80\ubd84\uc801\uc774\uace0 \ub9e4\uc6b0 \ub290\ub9b0 \ubd84\ub9ac/\ubd84\ud654\uac00 \uba87\uc2ed\uc5b5 \ub144 \uaddc\ubaa8\uc5d0\uc11c \uc9c4\ud589\ub418\uc5b4 \uc654\uc73c\uba70, \ud604\uc7ac\ub3c4 \uacc4\uc18d\ud574\uc11c \uc9c4\ud589\ub418\uace0 \uc788\uc744 \uc218\ub3c4 \uc788\ub2e4."} {"question": "\uc9c0\uae08 \uacf5\uc758 \uc678\uc99d\uc190\uc740 \ub204\uad6c\uc778\uac00?", "paragraph": "\uace0 \ucc2c\uc131 \ud314\uacc4\uad70 \uc815\uacf5\uc740 \ubb38\ud559\uacfc \ub355\ub9dd\uc73c\ub85c \uc911\uc885\uc5d0\uc11c \uc120\uc870\uc5d0 \uc774\ub974\ub294 \uc0ac\uc870\uc758 \uba85\uc2e0\uc774\uc5c8\ub2e4. \ub2e8\ud558\ub294 \uc77c\ucc0d\uc774 \uc9d1\uc548 \uc0ac\ub78c\uc774\ub098 \uace0\ud5a5 \uc0ac\ub78c\ub4e4\uc744 \ucad3\uc544\ub2e4\ub2c8\uba74\uc11c \uacf5\uc5d0 \ub300\ud574 \uc61b\ubd80\ud130 \ub0b4\ub824\uc624\ub358 \uc774\uc57c\uae30\uc640 \uadf8\uc5d0 \ub300\ud574 \ub0a8\uc544\uc788\ub294 \uc6b4\uce58\ub97c \ub4e3\uace0 \ub098\uc11c \uc5fc\ubaa8\ud558\ub294 \uc0dd\uac01\uc774 \ub2e4\ub978 \uc0ac\ub78c\uacfc \ube44\uad50\ud574\uc11c \uba87 \ubc30\ub098 \ub418\uc5c8\ub2e4. \uadf8\ub7ec\ub2e4 \uc9c0\uae08 \uacf5\uc758 \uc99d\uc190\uc778 \uc815\uc720(\u912d\u936e)\uc640 \uc815\uc11d\uc724(\u912d\u932b\u80e4), \uc678\uc99d\uc190\uc778 \ucd5c\ubb38\uc2dd(\u5d14\u6587\u6e5c)\ub4f1 \uc5ec\ub7ec \uc0ac\ub78c\uc774 \uacc4\uc18d\ud574\uc11c \ub2e8\ud558\ub97c \ucc3e\uc544\uc640 \"\uc870\uc0c1\uc758 \ubb18\ube44\ubb38\uc744 \uc798 \uc9c0\uc5b4 \ube44\uc11d\uc5d0 \uc0c8\uae30\uace0\uc790 \uc77c\ucc0d\uc774 \ub3cc\uc544\uac00\uc2e0 \ub300\uc7ac\uaed8 \uae00\uc744 \uc368\uc8fc\uae30\ub97c \ubd80\ud0c1\ud558\uc600\ub294\ub370 \uc5bb\uc9c0 \ubabb\ud558\uc600\uc2b5\ub2c8\ub2e4. \ub2e4\ud589\ud788 \uad00\uc0ac\uc5d0 \uc5c4\uc5f0\ud574 \uc788\uc73c\ub2c8 \uc774\uc81c \uc120\uc0dd\uaed8\uc11c \uacc4\uc18d\ud574\uc11c \uae00\uc744 \uc9c0\uc5b4\uc57c \ud558\ub294 \ucc45\uc784\uc774 \uc788\uc73c\ub2c8 \uae00\uc744 \ub9e1\uc544 \uc8fc\uc2ed\uc2dc\uc694\"\ub77c\uace0 \ub9d0\ud558\uc600\ub2e4. \uadf8\ub7ec\ub098 \ub2e8\ud558\ub294 \uc5ec\ub7ec\ubd84\ub4e4\uc758 \uccad\uc744 \uc0ac\uc591\ud558\uace0 \uae00\uc744 \uc9d3\uc9c0 \uc54a\uc558\ub2e4. \uc0ac\uc0ac\ub85c\uc6b4 \uac1c\uc778\uc758 \uc758\uacac\uc5d0 \uc758\uc9c0\ud558\uc9c0 \uc54a\uace0\ub294 \ucc28\ub9c8 \uae00\uc744 \ub9c8\uce60 \uc218 \uc5c6\uc5c8\uae30 \ub54c\ubb38\uc774\uc5c8\ub2e4. \uadf8\ub7ec\ub2e4 \ub9c8\uce68\ub0b4 \uccad\ucc9c\uc120\uc0dd(\uc120\uc870\ub54c \uc6b0\uc758\uc815\uc744 \uc9c0\ub0b8 \uc2ec\uc218\uacbd)\uc774 \uc9c0\uc740 \ud589\uc7a5\uacfc \uac00\ubcf4 \uc0ac\uc774 \uc0ac\uc774\uc5d0 \ub07c\uc5b4\uc788\ub294 \uc61b \uae00\ub4e4\uc744 \ucc44\ub85d\ud558\uc5ec \uba85\uc744 \uc11c\uc220\ud558\uac8c \ub418\uc5c8\ub2e4.", "answer": "\ucd5c\ubb38\uc2dd", "sentence": "\uadf8\ub7ec\ub2e4 \uc9c0\uae08 \uacf5\uc758 \uc99d\uc190\uc778 \uc815\uc720(\u912d\u936e)\uc640 \uc815\uc11d\uc724(\u912d\u932b\u80e4), \uc678\uc99d\uc190\uc778 \ucd5c\ubb38\uc2dd (\u5d14\u6587\u6e5c)\ub4f1 \uc5ec\ub7ec \uc0ac\ub78c\uc774 \uacc4\uc18d\ud574\uc11c \ub2e8\ud558\ub97c \ucc3e\uc544\uc640 \"\uc870\uc0c1\uc758 \ubb18\ube44\ubb38\uc744 \uc798 \uc9c0\uc5b4 \ube44\uc11d\uc5d0 \uc0c8\uae30\uace0\uc790 \uc77c\ucc0d\uc774 \ub3cc\uc544\uac00\uc2e0 \ub300\uc7ac\uaed8 \uae00\uc744 \uc368\uc8fc\uae30\ub97c \ubd80\ud0c1\ud558\uc600\ub294\ub370 \uc5bb\uc9c0 \ubabb\ud558\uc600\uc2b5\ub2c8\ub2e4.", "paragraph_sentence": "\uace0 \ucc2c\uc131 \ud314\uacc4\uad70 \uc815\uacf5\uc740 \ubb38\ud559\uacfc \ub355\ub9dd\uc73c\ub85c \uc911\uc885\uc5d0\uc11c \uc120\uc870\uc5d0 \uc774\ub974\ub294 \uc0ac\uc870\uc758 \uba85\uc2e0\uc774\uc5c8\ub2e4. \ub2e8\ud558\ub294 \uc77c\ucc0d\uc774 \uc9d1\uc548 \uc0ac\ub78c\uc774\ub098 \uace0\ud5a5 \uc0ac\ub78c\ub4e4\uc744 \ucad3\uc544\ub2e4\ub2c8\uba74\uc11c \uacf5\uc5d0 \ub300\ud574 \uc61b\ubd80\ud130 \ub0b4\ub824\uc624\ub358 \uc774\uc57c\uae30\uc640 \uadf8\uc5d0 \ub300\ud574 \ub0a8\uc544\uc788\ub294 \uc6b4\uce58\ub97c \ub4e3\uace0 \ub098\uc11c \uc5fc\ubaa8\ud558\ub294 \uc0dd\uac01\uc774 \ub2e4\ub978 \uc0ac\ub78c\uacfc \ube44\uad50\ud574\uc11c \uba87 \ubc30\ub098 \ub418\uc5c8\ub2e4. \uadf8\ub7ec\ub2e4 \uc9c0\uae08 \uacf5\uc758 \uc99d\uc190\uc778 \uc815\uc720(\u912d\u936e)\uc640 \uc815\uc11d\uc724(\u912d\u932b\u80e4), \uc678\uc99d\uc190\uc778 \ucd5c\ubb38\uc2dd (\u5d14\u6587\u6e5c)\ub4f1 \uc5ec\ub7ec \uc0ac\ub78c\uc774 \uacc4\uc18d\ud574\uc11c \ub2e8\ud558\ub97c \ucc3e\uc544\uc640 \"\uc870\uc0c1\uc758 \ubb18\ube44\ubb38\uc744 \uc798 \uc9c0\uc5b4 \ube44\uc11d\uc5d0 \uc0c8\uae30\uace0\uc790 \uc77c\ucc0d\uc774 \ub3cc\uc544\uac00\uc2e0 \ub300\uc7ac\uaed8 \uae00\uc744 \uc368\uc8fc\uae30\ub97c \ubd80\ud0c1\ud558\uc600\ub294\ub370 \uc5bb\uc9c0 \ubabb\ud558\uc600\uc2b5\ub2c8\ub2e4. \ub2e4\ud589\ud788 \uad00\uc0ac\uc5d0 \uc5c4\uc5f0\ud574 \uc788\uc73c\ub2c8 \uc774\uc81c \uc120\uc0dd\uaed8\uc11c \uacc4\uc18d\ud574\uc11c \uae00\uc744 \uc9c0\uc5b4\uc57c \ud558\ub294 \ucc45\uc784\uc774 \uc788\uc73c\ub2c8 \uae00\uc744 \ub9e1\uc544 \uc8fc\uc2ed\uc2dc\uc694\"\ub77c\uace0 \ub9d0\ud558\uc600\ub2e4. \uadf8\ub7ec\ub098 \ub2e8\ud558\ub294 \uc5ec\ub7ec\ubd84\ub4e4\uc758 \uccad\uc744 \uc0ac\uc591\ud558\uace0 \uae00\uc744 \uc9d3\uc9c0 \uc54a\uc558\ub2e4. \uc0ac\uc0ac\ub85c\uc6b4 \uac1c\uc778\uc758 \uc758\uacac\uc5d0 \uc758\uc9c0\ud558\uc9c0 \uc54a\uace0\ub294 \ucc28\ub9c8 \uae00\uc744 \ub9c8\uce60 \uc218 \uc5c6\uc5c8\uae30 \ub54c\ubb38\uc774\uc5c8\ub2e4. \uadf8\ub7ec\ub2e4 \ub9c8\uce68\ub0b4 \uccad\ucc9c\uc120\uc0dd(\uc120\uc870\ub54c \uc6b0\uc758\uc815\uc744 \uc9c0\ub0b8 \uc2ec\uc218\uacbd)\uc774 \uc9c0\uc740 \ud589\uc7a5\uacfc \uac00\ubcf4 \uc0ac\uc774 \uc0ac\uc774\uc5d0 \ub07c\uc5b4\uc788\ub294 \uc61b \uae00\ub4e4\uc744 \ucc44\ub85d\ud558\uc5ec \uba85\uc744 \uc11c\uc220\ud558\uac8c \ub418\uc5c8\ub2e4.", "paragraph_answer": "\uace0 \ucc2c\uc131 \ud314\uacc4\uad70 \uc815\uacf5\uc740 \ubb38\ud559\uacfc \ub355\ub9dd\uc73c\ub85c \uc911\uc885\uc5d0\uc11c \uc120\uc870\uc5d0 \uc774\ub974\ub294 \uc0ac\uc870\uc758 \uba85\uc2e0\uc774\uc5c8\ub2e4. \ub2e8\ud558\ub294 \uc77c\ucc0d\uc774 \uc9d1\uc548 \uc0ac\ub78c\uc774\ub098 \uace0\ud5a5 \uc0ac\ub78c\ub4e4\uc744 \ucad3\uc544\ub2e4\ub2c8\uba74\uc11c \uacf5\uc5d0 \ub300\ud574 \uc61b\ubd80\ud130 \ub0b4\ub824\uc624\ub358 \uc774\uc57c\uae30\uc640 \uadf8\uc5d0 \ub300\ud574 \ub0a8\uc544\uc788\ub294 \uc6b4\uce58\ub97c \ub4e3\uace0 \ub098\uc11c \uc5fc\ubaa8\ud558\ub294 \uc0dd\uac01\uc774 \ub2e4\ub978 \uc0ac\ub78c\uacfc \ube44\uad50\ud574\uc11c \uba87 \ubc30\ub098 \ub418\uc5c8\ub2e4. \uadf8\ub7ec\ub2e4 \uc9c0\uae08 \uacf5\uc758 \uc99d\uc190\uc778 \uc815\uc720(\u912d\u936e)\uc640 \uc815\uc11d\uc724(\u912d\u932b\u80e4), \uc678\uc99d\uc190\uc778 \ucd5c\ubb38\uc2dd (\u5d14\u6587\u6e5c)\ub4f1 \uc5ec\ub7ec \uc0ac\ub78c\uc774 \uacc4\uc18d\ud574\uc11c \ub2e8\ud558\ub97c \ucc3e\uc544\uc640 \"\uc870\uc0c1\uc758 \ubb18\ube44\ubb38\uc744 \uc798 \uc9c0\uc5b4 \ube44\uc11d\uc5d0 \uc0c8\uae30\uace0\uc790 \uc77c\ucc0d\uc774 \ub3cc\uc544\uac00\uc2e0 \ub300\uc7ac\uaed8 \uae00\uc744 \uc368\uc8fc\uae30\ub97c \ubd80\ud0c1\ud558\uc600\ub294\ub370 \uc5bb\uc9c0 \ubabb\ud558\uc600\uc2b5\ub2c8\ub2e4. \ub2e4\ud589\ud788 \uad00\uc0ac\uc5d0 \uc5c4\uc5f0\ud574 \uc788\uc73c\ub2c8 \uc774\uc81c \uc120\uc0dd\uaed8\uc11c \uacc4\uc18d\ud574\uc11c \uae00\uc744 \uc9c0\uc5b4\uc57c \ud558\ub294 \ucc45\uc784\uc774 \uc788\uc73c\ub2c8 \uae00\uc744 \ub9e1\uc544 \uc8fc\uc2ed\uc2dc\uc694\"\ub77c\uace0 \ub9d0\ud558\uc600\ub2e4. \uadf8\ub7ec\ub098 \ub2e8\ud558\ub294 \uc5ec\ub7ec\ubd84\ub4e4\uc758 \uccad\uc744 \uc0ac\uc591\ud558\uace0 \uae00\uc744 \uc9d3\uc9c0 \uc54a\uc558\ub2e4. \uc0ac\uc0ac\ub85c\uc6b4 \uac1c\uc778\uc758 \uc758\uacac\uc5d0 \uc758\uc9c0\ud558\uc9c0 \uc54a\uace0\ub294 \ucc28\ub9c8 \uae00\uc744 \ub9c8\uce60 \uc218 \uc5c6\uc5c8\uae30 \ub54c\ubb38\uc774\uc5c8\ub2e4. \uadf8\ub7ec\ub2e4 \ub9c8\uce68\ub0b4 \uccad\ucc9c\uc120\uc0dd(\uc120\uc870\ub54c \uc6b0\uc758\uc815\uc744 \uc9c0\ub0b8 \uc2ec\uc218\uacbd)\uc774 \uc9c0\uc740 \ud589\uc7a5\uacfc \uac00\ubcf4 \uc0ac\uc774 \uc0ac\uc774\uc5d0 \ub07c\uc5b4\uc788\ub294 \uc61b \uae00\ub4e4\uc744 \ucc44\ub85d\ud558\uc5ec \uba85\uc744 \uc11c\uc220\ud558\uac8c \ub418\uc5c8\ub2e4.", "sentence_answer": "\uadf8\ub7ec\ub2e4 \uc9c0\uae08 \uacf5\uc758 \uc99d\uc190\uc778 \uc815\uc720(\u912d\u936e)\uc640 \uc815\uc11d\uc724(\u912d\u932b\u80e4), \uc678\uc99d\uc190\uc778 \ucd5c\ubb38\uc2dd (\u5d14\u6587\u6e5c)\ub4f1 \uc5ec\ub7ec \uc0ac\ub78c\uc774 \uacc4\uc18d\ud574\uc11c \ub2e8\ud558\ub97c \ucc3e\uc544\uc640 \"\uc870\uc0c1\uc758 \ubb18\ube44\ubb38\uc744 \uc798 \uc9c0\uc5b4 \ube44\uc11d\uc5d0 \uc0c8\uae30\uace0\uc790 \uc77c\ucc0d\uc774 \ub3cc\uc544\uac00\uc2e0 \ub300\uc7ac\uaed8 \uae00\uc744 \uc368\uc8fc\uae30\ub97c \ubd80\ud0c1\ud558\uc600\ub294\ub370 \uc5bb\uc9c0 \ubabb\ud558\uc600\uc2b5\ub2c8\ub2e4.", "paragraph_id": "6594086-1-0", "paragraph_question": "question: \uc9c0\uae08 \uacf5\uc758 \uc678\uc99d\uc190\uc740 \ub204\uad6c\uc778\uac00?, context: \uace0 \ucc2c\uc131 \ud314\uacc4\uad70 \uc815\uacf5\uc740 \ubb38\ud559\uacfc \ub355\ub9dd\uc73c\ub85c \uc911\uc885\uc5d0\uc11c \uc120\uc870\uc5d0 \uc774\ub974\ub294 \uc0ac\uc870\uc758 \uba85\uc2e0\uc774\uc5c8\ub2e4. \ub2e8\ud558\ub294 \uc77c\ucc0d\uc774 \uc9d1\uc548 \uc0ac\ub78c\uc774\ub098 \uace0\ud5a5 \uc0ac\ub78c\ub4e4\uc744 \ucad3\uc544\ub2e4\ub2c8\uba74\uc11c \uacf5\uc5d0 \ub300\ud574 \uc61b\ubd80\ud130 \ub0b4\ub824\uc624\ub358 \uc774\uc57c\uae30\uc640 \uadf8\uc5d0 \ub300\ud574 \ub0a8\uc544\uc788\ub294 \uc6b4\uce58\ub97c \ub4e3\uace0 \ub098\uc11c \uc5fc\ubaa8\ud558\ub294 \uc0dd\uac01\uc774 \ub2e4\ub978 \uc0ac\ub78c\uacfc \ube44\uad50\ud574\uc11c \uba87 \ubc30\ub098 \ub418\uc5c8\ub2e4. \uadf8\ub7ec\ub2e4 \uc9c0\uae08 \uacf5\uc758 \uc99d\uc190\uc778 \uc815\uc720(\u912d\u936e)\uc640 \uc815\uc11d\uc724(\u912d\u932b\u80e4), \uc678\uc99d\uc190\uc778 \ucd5c\ubb38\uc2dd(\u5d14\u6587\u6e5c)\ub4f1 \uc5ec\ub7ec \uc0ac\ub78c\uc774 \uacc4\uc18d\ud574\uc11c \ub2e8\ud558\ub97c \ucc3e\uc544\uc640 \"\uc870\uc0c1\uc758 \ubb18\ube44\ubb38\uc744 \uc798 \uc9c0\uc5b4 \ube44\uc11d\uc5d0 \uc0c8\uae30\uace0\uc790 \uc77c\ucc0d\uc774 \ub3cc\uc544\uac00\uc2e0 \ub300\uc7ac\uaed8 \uae00\uc744 \uc368\uc8fc\uae30\ub97c \ubd80\ud0c1\ud558\uc600\ub294\ub370 \uc5bb\uc9c0 \ubabb\ud558\uc600\uc2b5\ub2c8\ub2e4. \ub2e4\ud589\ud788 \uad00\uc0ac\uc5d0 \uc5c4\uc5f0\ud574 \uc788\uc73c\ub2c8 \uc774\uc81c \uc120\uc0dd\uaed8\uc11c \uacc4\uc18d\ud574\uc11c \uae00\uc744 \uc9c0\uc5b4\uc57c \ud558\ub294 \ucc45\uc784\uc774 \uc788\uc73c\ub2c8 \uae00\uc744 \ub9e1\uc544 \uc8fc\uc2ed\uc2dc\uc694\"\ub77c\uace0 \ub9d0\ud558\uc600\ub2e4. \uadf8\ub7ec\ub098 \ub2e8\ud558\ub294 \uc5ec\ub7ec\ubd84\ub4e4\uc758 \uccad\uc744 \uc0ac\uc591\ud558\uace0 \uae00\uc744 \uc9d3\uc9c0 \uc54a\uc558\ub2e4. \uc0ac\uc0ac\ub85c\uc6b4 \uac1c\uc778\uc758 \uc758\uacac\uc5d0 \uc758\uc9c0\ud558\uc9c0 \uc54a\uace0\ub294 \ucc28\ub9c8 \uae00\uc744 \ub9c8\uce60 \uc218 \uc5c6\uc5c8\uae30 \ub54c\ubb38\uc774\uc5c8\ub2e4. \uadf8\ub7ec\ub2e4 \ub9c8\uce68\ub0b4 \uccad\ucc9c\uc120\uc0dd(\uc120\uc870\ub54c \uc6b0\uc758\uc815\uc744 \uc9c0\ub0b8 \uc2ec\uc218\uacbd)\uc774 \uc9c0\uc740 \ud589\uc7a5\uacfc \uac00\ubcf4 \uc0ac\uc774 \uc0ac\uc774\uc5d0 \ub07c\uc5b4\uc788\ub294 \uc61b \uae00\ub4e4\uc744 \ucc44\ub85d\ud558\uc5ec \uba85\uc744 \uc11c\uc220\ud558\uac8c \ub418\uc5c8\ub2e4."} {"question": "\uc720\uc0c9\uc778\uc885 \uc9c0\uc704\ud5a5\uc0c1\ud611\ud68c\ub294 \uc628\uac74\ud30c\uc778\uac00 \uc911\uac04\ud30c\uc778\uac00?", "paragraph": "\ud751\uc778\ub4e4\uc758 \uc9c0\uc704 \ud5a5\uc0c1\uc744 \uc704\ud55c \ub2e8\uccb4\ub294 \ubbf8\uad6d\uc5d0 \uc218\uc5c6\uc774 \ub9ce\ub2e4. \uc911\uc694\ud55c \uac83\uc744 \ub4e0\ub2e4\uba74 \uc624\ub79c \uc5ed\uc0ac\ub97c \uc9c0\ub2c8\uace0 \uc788\ub294 \uc628\uac74\ud30c \ub2e8\uccb4\ub85c \uc720\uc0c9\uc778\uc885 \uc9c0\uc704\ud5a5\uc0c1\ud611\ud68c(NAACP), \uc804\uad6d \ub3c4\uc2dc\ub3d9\ub9f9(NUL), \ub9c8\ud2f4 \ub8e8\uc11c \ud0b9 \ubaa9\uc0ac\uac00 \uc774\ub04c\ub358 \uc911\uac04\ud30c\ub85c\uc11c \ub0a8\ubd80 \uae30\ub3c5\uad50 \uc9c0\ub3c4\uc790\ud68c\uc758(SCLC), \uadf8\uc678 \uc88c\ud30c\ub85c\uc11c \uc778\uc885\ud3c9\ub4f1\ud68c\uc758(CORE), \ud559\uc0dd \ube44\ud3ed\ub825 \uc870\uc815\uc704\uc6d0\ud68c(SNCC), \ub9ec\ucef4 \uc5d1\uc2a4(1965\ub144 \uc554\uc0b4\ub428)\uc758 \ud751\uc778\ud68c\uad50\ub2e8 \ub4f1\uc774 \uc788\ub2e4. 1966\ub144\uacbd\ubd80\ud130 \ud751\uc778\uc6b4\ub3d9\uc758 \uc8fc\ub3c4\uad8c\uc740 \ube14\ub799 \ud30c\uc6cc\ub97c \uc678\uce58\ub294 SNCC\ub97c \uc911\uc2ec\uc73c\ub85c \ud55c \uae09\uc9c4\ud30c\uc5d0\uac8c \uc62e\uaca8\uc838 \uac14\ub2e4. \uadf8\ub7ec\ub098 \ubc31\uc778\ub4e4\uacfc\uc758 \ud611\ub825\uc744 \uc77c\uccb4 \uac70\ubd80\ud558\ub294 \uae09\uc9c4\ud30c\ub4e4\uc758 \ud0dc\ub3c4\uc5d0 \ub300\ud558\uc5ec NAACP \ub4f1 \uc628\uac74\ud30c\ub4e4\uc740 \uc774\ub97c \ubc18\ub300\ud568\uc73c\ub85c\uc368 \ud751\uc778\uc6b4\ub3d9 \uc9c0\ub3c4\uc790\uac04\uc758 \ubd84\uc5f4\uc740 \ud55c\uce35 \uc2ec\uac01\ud558\uac8c \ub098\ud0c0\ub098\uace0 \uc788\ub2e4.", "answer": "\uc628\uac74\ud30c", "sentence": "\uc911\uc694\ud55c \uac83\uc744 \ub4e0\ub2e4\uba74 \uc624\ub79c \uc5ed\uc0ac\ub97c \uc9c0\ub2c8\uace0 \uc788\ub294 \uc628\uac74\ud30c \ub2e8\uccb4\ub85c \uc720\uc0c9\uc778\uc885 \uc9c0\uc704\ud5a5\uc0c1\ud611\ud68c(NAACP), \uc804\uad6d \ub3c4\uc2dc\ub3d9\ub9f9(NUL), \ub9c8\ud2f4 \ub8e8\uc11c \ud0b9 \ubaa9\uc0ac\uac00 \uc774\ub04c\ub358 \uc911\uac04\ud30c\ub85c\uc11c \ub0a8\ubd80 \uae30\ub3c5\uad50 \uc9c0\ub3c4\uc790\ud68c\uc758(SCLC), \uadf8\uc678 \uc88c\ud30c\ub85c\uc11c \uc778\uc885\ud3c9\ub4f1\ud68c\uc758(CORE), \ud559\uc0dd \ube44\ud3ed\ub825 \uc870\uc815\uc704\uc6d0\ud68c(SNCC), \ub9ec\ucef4 \uc5d1\uc2a4(1965\ub144 \uc554\uc0b4\ub428)\uc758 \ud751\uc778\ud68c\uad50\ub2e8 \ub4f1\uc774 \uc788\ub2e4.", "paragraph_sentence": "\ud751\uc778\ub4e4\uc758 \uc9c0\uc704 \ud5a5\uc0c1\uc744 \uc704\ud55c \ub2e8\uccb4\ub294 \ubbf8\uad6d\uc5d0 \uc218\uc5c6\uc774 \ub9ce\ub2e4. \uc911\uc694\ud55c \uac83\uc744 \ub4e0\ub2e4\uba74 \uc624\ub79c \uc5ed\uc0ac\ub97c \uc9c0\ub2c8\uace0 \uc788\ub294 \uc628\uac74\ud30c \ub2e8\uccb4\ub85c \uc720\uc0c9\uc778\uc885 \uc9c0\uc704\ud5a5\uc0c1\ud611\ud68c(NAACP), \uc804\uad6d \ub3c4\uc2dc\ub3d9\ub9f9(NUL), \ub9c8\ud2f4 \ub8e8\uc11c \ud0b9 \ubaa9\uc0ac\uac00 \uc774\ub04c\ub358 \uc911\uac04\ud30c\ub85c\uc11c \ub0a8\ubd80 \uae30\ub3c5\uad50 \uc9c0\ub3c4\uc790\ud68c\uc758(SCLC), \uadf8\uc678 \uc88c\ud30c\ub85c\uc11c \uc778\uc885\ud3c9\ub4f1\ud68c\uc758(CORE), \ud559\uc0dd \ube44\ud3ed\ub825 \uc870\uc815\uc704\uc6d0\ud68c(SNCC), \ub9ec\ucef4 \uc5d1\uc2a4(1965\ub144 \uc554\uc0b4\ub428)\uc758 \ud751\uc778\ud68c\uad50\ub2e8 \ub4f1\uc774 \uc788\ub2e4. 1966\ub144\uacbd\ubd80\ud130 \ud751\uc778\uc6b4\ub3d9\uc758 \uc8fc\ub3c4\uad8c\uc740 \ube14\ub799 \ud30c\uc6cc\ub97c \uc678\uce58\ub294 SNCC\ub97c \uc911\uc2ec\uc73c\ub85c \ud55c \uae09\uc9c4\ud30c\uc5d0\uac8c \uc62e\uaca8\uc838 \uac14\ub2e4. \uadf8\ub7ec\ub098 \ubc31\uc778\ub4e4\uacfc\uc758 \ud611\ub825\uc744 \uc77c\uccb4 \uac70\ubd80\ud558\ub294 \uae09\uc9c4\ud30c\ub4e4\uc758 \ud0dc\ub3c4\uc5d0 \ub300\ud558\uc5ec NAACP \ub4f1 \uc628\uac74\ud30c\ub4e4\uc740 \uc774\ub97c \ubc18\ub300\ud568\uc73c\ub85c\uc368 \ud751\uc778\uc6b4\ub3d9 \uc9c0\ub3c4\uc790\uac04\uc758 \ubd84\uc5f4\uc740 \ud55c\uce35 \uc2ec\uac01\ud558\uac8c \ub098\ud0c0\ub098\uace0 \uc788\ub2e4.", "paragraph_answer": "\ud751\uc778\ub4e4\uc758 \uc9c0\uc704 \ud5a5\uc0c1\uc744 \uc704\ud55c \ub2e8\uccb4\ub294 \ubbf8\uad6d\uc5d0 \uc218\uc5c6\uc774 \ub9ce\ub2e4. \uc911\uc694\ud55c \uac83\uc744 \ub4e0\ub2e4\uba74 \uc624\ub79c \uc5ed\uc0ac\ub97c \uc9c0\ub2c8\uace0 \uc788\ub294 \uc628\uac74\ud30c \ub2e8\uccb4\ub85c \uc720\uc0c9\uc778\uc885 \uc9c0\uc704\ud5a5\uc0c1\ud611\ud68c(NAACP), \uc804\uad6d \ub3c4\uc2dc\ub3d9\ub9f9(NUL), \ub9c8\ud2f4 \ub8e8\uc11c \ud0b9 \ubaa9\uc0ac\uac00 \uc774\ub04c\ub358 \uc911\uac04\ud30c\ub85c\uc11c \ub0a8\ubd80 \uae30\ub3c5\uad50 \uc9c0\ub3c4\uc790\ud68c\uc758(SCLC), \uadf8\uc678 \uc88c\ud30c\ub85c\uc11c \uc778\uc885\ud3c9\ub4f1\ud68c\uc758(CORE), \ud559\uc0dd \ube44\ud3ed\ub825 \uc870\uc815\uc704\uc6d0\ud68c(SNCC), \ub9ec\ucef4 \uc5d1\uc2a4(1965\ub144 \uc554\uc0b4\ub428)\uc758 \ud751\uc778\ud68c\uad50\ub2e8 \ub4f1\uc774 \uc788\ub2e4. 1966\ub144\uacbd\ubd80\ud130 \ud751\uc778\uc6b4\ub3d9\uc758 \uc8fc\ub3c4\uad8c\uc740 \ube14\ub799 \ud30c\uc6cc\ub97c \uc678\uce58\ub294 SNCC\ub97c \uc911\uc2ec\uc73c\ub85c \ud55c \uae09\uc9c4\ud30c\uc5d0\uac8c \uc62e\uaca8\uc838 \uac14\ub2e4. \uadf8\ub7ec\ub098 \ubc31\uc778\ub4e4\uacfc\uc758 \ud611\ub825\uc744 \uc77c\uccb4 \uac70\ubd80\ud558\ub294 \uae09\uc9c4\ud30c\ub4e4\uc758 \ud0dc\ub3c4\uc5d0 \ub300\ud558\uc5ec NAACP \ub4f1 \uc628\uac74\ud30c\ub4e4\uc740 \uc774\ub97c \ubc18\ub300\ud568\uc73c\ub85c\uc368 \ud751\uc778\uc6b4\ub3d9 \uc9c0\ub3c4\uc790\uac04\uc758 \ubd84\uc5f4\uc740 \ud55c\uce35 \uc2ec\uac01\ud558\uac8c \ub098\ud0c0\ub098\uace0 \uc788\ub2e4.", "sentence_answer": "\uc911\uc694\ud55c \uac83\uc744 \ub4e0\ub2e4\uba74 \uc624\ub79c \uc5ed\uc0ac\ub97c \uc9c0\ub2c8\uace0 \uc788\ub294 \uc628\uac74\ud30c \ub2e8\uccb4\ub85c \uc720\uc0c9\uc778\uc885 \uc9c0\uc704\ud5a5\uc0c1\ud611\ud68c(NAACP), \uc804\uad6d \ub3c4\uc2dc\ub3d9\ub9f9(NUL), \ub9c8\ud2f4 \ub8e8\uc11c \ud0b9 \ubaa9\uc0ac\uac00 \uc774\ub04c\ub358 \uc911\uac04\ud30c\ub85c\uc11c \ub0a8\ubd80 \uae30\ub3c5\uad50 \uc9c0\ub3c4\uc790\ud68c\uc758(SCLC), \uadf8\uc678 \uc88c\ud30c\ub85c\uc11c \uc778\uc885\ud3c9\ub4f1\ud68c\uc758(CORE), \ud559\uc0dd \ube44\ud3ed\ub825 \uc870\uc815\uc704\uc6d0\ud68c(SNCC), \ub9ec\ucef4 \uc5d1\uc2a4(1965\ub144 \uc554\uc0b4\ub428)\uc758 \ud751\uc778\ud68c\uad50\ub2e8 \ub4f1\uc774 \uc788\ub2e4.", "paragraph_id": "6462682-15-0", "paragraph_question": "question: \uc720\uc0c9\uc778\uc885 \uc9c0\uc704\ud5a5\uc0c1\ud611\ud68c\ub294 \uc628\uac74\ud30c\uc778\uac00 \uc911\uac04\ud30c\uc778\uac00?, context: \ud751\uc778\ub4e4\uc758 \uc9c0\uc704 \ud5a5\uc0c1\uc744 \uc704\ud55c \ub2e8\uccb4\ub294 \ubbf8\uad6d\uc5d0 \uc218\uc5c6\uc774 \ub9ce\ub2e4. \uc911\uc694\ud55c \uac83\uc744 \ub4e0\ub2e4\uba74 \uc624\ub79c \uc5ed\uc0ac\ub97c \uc9c0\ub2c8\uace0 \uc788\ub294 \uc628\uac74\ud30c \ub2e8\uccb4\ub85c \uc720\uc0c9\uc778\uc885 \uc9c0\uc704\ud5a5\uc0c1\ud611\ud68c(NAACP), \uc804\uad6d \ub3c4\uc2dc\ub3d9\ub9f9(NUL), \ub9c8\ud2f4 \ub8e8\uc11c \ud0b9 \ubaa9\uc0ac\uac00 \uc774\ub04c\ub358 \uc911\uac04\ud30c\ub85c\uc11c \ub0a8\ubd80 \uae30\ub3c5\uad50 \uc9c0\ub3c4\uc790\ud68c\uc758(SCLC), \uadf8\uc678 \uc88c\ud30c\ub85c\uc11c \uc778\uc885\ud3c9\ub4f1\ud68c\uc758(CORE), \ud559\uc0dd \ube44\ud3ed\ub825 \uc870\uc815\uc704\uc6d0\ud68c(SNCC), \ub9ec\ucef4 \uc5d1\uc2a4(1965\ub144 \uc554\uc0b4\ub428)\uc758 \ud751\uc778\ud68c\uad50\ub2e8 \ub4f1\uc774 \uc788\ub2e4. 1966\ub144\uacbd\ubd80\ud130 \ud751\uc778\uc6b4\ub3d9\uc758 \uc8fc\ub3c4\uad8c\uc740 \ube14\ub799 \ud30c\uc6cc\ub97c \uc678\uce58\ub294 SNCC\ub97c \uc911\uc2ec\uc73c\ub85c \ud55c \uae09\uc9c4\ud30c\uc5d0\uac8c \uc62e\uaca8\uc838 \uac14\ub2e4. \uadf8\ub7ec\ub098 \ubc31\uc778\ub4e4\uacfc\uc758 \ud611\ub825\uc744 \uc77c\uccb4 \uac70\ubd80\ud558\ub294 \uae09\uc9c4\ud30c\ub4e4\uc758 \ud0dc\ub3c4\uc5d0 \ub300\ud558\uc5ec NAACP \ub4f1 \uc628\uac74\ud30c\ub4e4\uc740 \uc774\ub97c \ubc18\ub300\ud568\uc73c\ub85c\uc368 \ud751\uc778\uc6b4\ub3d9 \uc9c0\ub3c4\uc790\uac04\uc758 \ubd84\uc5f4\uc740 \ud55c\uce35 \uc2ec\uac01\ud558\uac8c \ub098\ud0c0\ub098\uace0 \uc788\ub2e4."} {"question": "\uc2a4\ubb34 \uc0b4\uc758 \ub51c\ub7f0\uc774 \ub300\ud559 \uc911\ud1f4 \ud6c4 \uac00\uae30\ub85c \ud55c \ub3c4\uc2dc\uc758 \uc774\ub984\uc740?", "paragraph": "1961\ub144 \ub300\ud559\uc744 \uc911\ud1f4\ud55c \ub51c\ub7f0\uc740 \uadf8\uac00 \uc22d\ubc30\ud55c \uc6b0\ub514 \uac70\uc2a4\ub9ac\uac00 \uc0ac\ub294 \ub274\uc695\uc2dc\ub85c \uac00\uae30\ub85c \uacb0\uc2ec\ud588\uace0, \uc2a4\ubb34 \uc0b4\uc758 \ub51c\ub7f0\uc740 \ub274\uc695 \uadf8\ub9ac\ub2c8\uce58\ube4c\ub9ac\uc9c0\uc5d0 \ub3c4\ucc29\ud588\ub2e4. \ub51c\ub7f0\uc740 \ub274\uc695\uc5d0 \ub3c4\ucc29\ud558\uc790\ub9c8\uc790 \uace7\ubc14\ub85c \ud55c \uc2dc\uac04 \ubc18 \ubc84\uc2a4\ub97c \ud0c0\uace0 \uc790\uc2e0\uc744 \uc74c\uc545\uc758 \uae38\ub85c \uc774\ub048 \uc601\uc6c5 \uc6b0\ub514 \uac70\uc2a4\ub9ac\ub97c \ub9cc\ub098\ub7ec \uac14\ub2e4. \uadf8\ub7ec\ub098 \uc6b0\ub514 \uac70\uc2a4\ub9ac\ub294 \ub299\uc5b4\uc11c \uc2e0\uacbd\ud1f4\ubcf4\uc99d\uc5d0 \uac78\ub824 \ub274\uc800\uc9c0\uc8fc\ub9bd\ubcd1\uc6d0\uc758 \ubcd1\uc0c1\uc5d0 \ub204\uc6cc \uc788\uc5c8\ub2e4. \uc218\uc2ed \uac1c\uc758 \uce68\uc0c1\uc774 \ub098\ub780\ud788 \ub193\uc5ec \uc788\ub294 \ubc29\uc5d0\uc11c \uac70\uc2a4\ub9ac\ub294 \ubb34\uae30\ub825\ud558\uac8c \ub204\uc6cc \uc788\uc5c8\ub2e4. \ubb34\uc11c\uc6b4 \ubcd1\ub9c8\uc5d0 \ud56d\ubcf5\ud55c \uc6b0\ub514 \uac70\uc2a4\ub9ac\uc758 \uc721\uccb4\ub97c \ubcf4\uba74\uc11c \ub51c\ub7f0\uc740 \uae30\uac81\ud588\uc73c\ub098 \uadf8\ub294 \uae30\ud0c0\ub97c \uce58\uba70 \uc790\uc2e0\uc774 \uc88b\uc544\ud55c \uc6b0\ub514 \uac70\uc2a4\ub9ac\uc758 \ub178\ub798\ub97c \ubd80\ub974\uba70 \uc8fd\uc5b4\uac00\ub294 \ub178\uc778\uc744 \uc704\ub85c\ud588\ub2e4. \uadf8\ub9ac\uace0 \ub5a8\ub9ac\ub294 \uc190\uc73c\ub85c \ub178\uc778\uc5d0\uac8c \uce5c\ud544 \uc11c\uba85\uc744 \ubc1b\uc558\ub2e4. \ucc98\uc640 \uc544\ub4e4\uc778 \uc544\ub860 \uac70\uc2a4\ub9ac\uac00 \ub51c\ub7f0\uc758 \ubc29\ubb38\uc5d0 \ub9e4\uc6b0 \uae30\ubed0\ud588\ub2e4\uace0 \uc804\ud574\uc9c4\ub2e4.", "answer": "\ub274\uc695", "sentence": "1961\ub144 \ub300\ud559\uc744 \uc911\ud1f4\ud55c \ub51c\ub7f0\uc740 \uadf8\uac00 \uc22d\ubc30\ud55c \uc6b0\ub514 \uac70\uc2a4\ub9ac\uac00 \uc0ac\ub294 \ub274\uc695 \uc2dc\ub85c \uac00\uae30\ub85c \uacb0\uc2ec\ud588\uace0, \uc2a4\ubb34 \uc0b4\uc758 \ub51c\ub7f0\uc740 \ub274\uc695 \uadf8\ub9ac\ub2c8\uce58\ube4c\ub9ac\uc9c0\uc5d0 \ub3c4\ucc29\ud588\ub2e4.", "paragraph_sentence": " 1961\ub144 \ub300\ud559\uc744 \uc911\ud1f4\ud55c \ub51c\ub7f0\uc740 \uadf8\uac00 \uc22d\ubc30\ud55c \uc6b0\ub514 \uac70\uc2a4\ub9ac\uac00 \uc0ac\ub294 \ub274\uc695 \uc2dc\ub85c \uac00\uae30\ub85c \uacb0\uc2ec\ud588\uace0, \uc2a4\ubb34 \uc0b4\uc758 \ub51c\ub7f0\uc740 \ub274\uc695 \uadf8\ub9ac\ub2c8\uce58\ube4c\ub9ac\uc9c0\uc5d0 \ub3c4\ucc29\ud588\ub2e4. \ub51c\ub7f0\uc740 \ub274\uc695\uc5d0 \ub3c4\ucc29\ud558\uc790\ub9c8\uc790 \uace7\ubc14\ub85c \ud55c \uc2dc\uac04 \ubc18 \ubc84\uc2a4\ub97c \ud0c0\uace0 \uc790\uc2e0\uc744 \uc74c\uc545\uc758 \uae38\ub85c \uc774\ub048 \uc601\uc6c5 \uc6b0\ub514 \uac70\uc2a4\ub9ac\ub97c \ub9cc\ub098\ub7ec \uac14\ub2e4. \uadf8\ub7ec\ub098 \uc6b0\ub514 \uac70\uc2a4\ub9ac\ub294 \ub299\uc5b4\uc11c \uc2e0\uacbd\ud1f4\ubcf4\uc99d\uc5d0 \uac78\ub824 \ub274\uc800\uc9c0\uc8fc\ub9bd\ubcd1\uc6d0\uc758 \ubcd1\uc0c1\uc5d0 \ub204\uc6cc \uc788\uc5c8\ub2e4. \uc218\uc2ed \uac1c\uc758 \uce68\uc0c1\uc774 \ub098\ub780\ud788 \ub193\uc5ec \uc788\ub294 \ubc29\uc5d0\uc11c \uac70\uc2a4\ub9ac\ub294 \ubb34\uae30\ub825\ud558\uac8c \ub204\uc6cc \uc788\uc5c8\ub2e4. \ubb34\uc11c\uc6b4 \ubcd1\ub9c8\uc5d0 \ud56d\ubcf5\ud55c \uc6b0\ub514 \uac70\uc2a4\ub9ac\uc758 \uc721\uccb4\ub97c \ubcf4\uba74\uc11c \ub51c\ub7f0\uc740 \uae30\uac81\ud588\uc73c\ub098 \uadf8\ub294 \uae30\ud0c0\ub97c \uce58\uba70 \uc790\uc2e0\uc774 \uc88b\uc544\ud55c \uc6b0\ub514 \uac70\uc2a4\ub9ac\uc758 \ub178\ub798\ub97c \ubd80\ub974\uba70 \uc8fd\uc5b4\uac00\ub294 \ub178\uc778\uc744 \uc704\ub85c\ud588\ub2e4. \uadf8\ub9ac\uace0 \ub5a8\ub9ac\ub294 \uc190\uc73c\ub85c \ub178\uc778\uc5d0\uac8c \uce5c\ud544 \uc11c\uba85\uc744 \ubc1b\uc558\ub2e4. \ucc98\uc640 \uc544\ub4e4\uc778 \uc544\ub860 \uac70\uc2a4\ub9ac\uac00 \ub51c\ub7f0\uc758 \ubc29\ubb38\uc5d0 \ub9e4\uc6b0 \uae30\ubed0\ud588\ub2e4\uace0 \uc804\ud574\uc9c4\ub2e4.", "paragraph_answer": "1961\ub144 \ub300\ud559\uc744 \uc911\ud1f4\ud55c \ub51c\ub7f0\uc740 \uadf8\uac00 \uc22d\ubc30\ud55c \uc6b0\ub514 \uac70\uc2a4\ub9ac\uac00 \uc0ac\ub294 \ub274\uc695 \uc2dc\ub85c \uac00\uae30\ub85c \uacb0\uc2ec\ud588\uace0, \uc2a4\ubb34 \uc0b4\uc758 \ub51c\ub7f0\uc740 \ub274\uc695 \uadf8\ub9ac\ub2c8\uce58\ube4c\ub9ac\uc9c0\uc5d0 \ub3c4\ucc29\ud588\ub2e4. \ub51c\ub7f0\uc740 \ub274\uc695\uc5d0 \ub3c4\ucc29\ud558\uc790\ub9c8\uc790 \uace7\ubc14\ub85c \ud55c \uc2dc\uac04 \ubc18 \ubc84\uc2a4\ub97c \ud0c0\uace0 \uc790\uc2e0\uc744 \uc74c\uc545\uc758 \uae38\ub85c \uc774\ub048 \uc601\uc6c5 \uc6b0\ub514 \uac70\uc2a4\ub9ac\ub97c \ub9cc\ub098\ub7ec \uac14\ub2e4. \uadf8\ub7ec\ub098 \uc6b0\ub514 \uac70\uc2a4\ub9ac\ub294 \ub299\uc5b4\uc11c \uc2e0\uacbd\ud1f4\ubcf4\uc99d\uc5d0 \uac78\ub824 \ub274\uc800\uc9c0\uc8fc\ub9bd\ubcd1\uc6d0\uc758 \ubcd1\uc0c1\uc5d0 \ub204\uc6cc \uc788\uc5c8\ub2e4. \uc218\uc2ed \uac1c\uc758 \uce68\uc0c1\uc774 \ub098\ub780\ud788 \ub193\uc5ec \uc788\ub294 \ubc29\uc5d0\uc11c \uac70\uc2a4\ub9ac\ub294 \ubb34\uae30\ub825\ud558\uac8c \ub204\uc6cc \uc788\uc5c8\ub2e4. \ubb34\uc11c\uc6b4 \ubcd1\ub9c8\uc5d0 \ud56d\ubcf5\ud55c \uc6b0\ub514 \uac70\uc2a4\ub9ac\uc758 \uc721\uccb4\ub97c \ubcf4\uba74\uc11c \ub51c\ub7f0\uc740 \uae30\uac81\ud588\uc73c\ub098 \uadf8\ub294 \uae30\ud0c0\ub97c \uce58\uba70 \uc790\uc2e0\uc774 \uc88b\uc544\ud55c \uc6b0\ub514 \uac70\uc2a4\ub9ac\uc758 \ub178\ub798\ub97c \ubd80\ub974\uba70 \uc8fd\uc5b4\uac00\ub294 \ub178\uc778\uc744 \uc704\ub85c\ud588\ub2e4. \uadf8\ub9ac\uace0 \ub5a8\ub9ac\ub294 \uc190\uc73c\ub85c \ub178\uc778\uc5d0\uac8c \uce5c\ud544 \uc11c\uba85\uc744 \ubc1b\uc558\ub2e4. \ucc98\uc640 \uc544\ub4e4\uc778 \uc544\ub860 \uac70\uc2a4\ub9ac\uac00 \ub51c\ub7f0\uc758 \ubc29\ubb38\uc5d0 \ub9e4\uc6b0 \uae30\ubed0\ud588\ub2e4\uace0 \uc804\ud574\uc9c4\ub2e4.", "sentence_answer": "1961\ub144 \ub300\ud559\uc744 \uc911\ud1f4\ud55c \ub51c\ub7f0\uc740 \uadf8\uac00 \uc22d\ubc30\ud55c \uc6b0\ub514 \uac70\uc2a4\ub9ac\uac00 \uc0ac\ub294 \ub274\uc695 \uc2dc\ub85c \uac00\uae30\ub85c \uacb0\uc2ec\ud588\uace0, \uc2a4\ubb34 \uc0b4\uc758 \ub51c\ub7f0\uc740 \ub274\uc695 \uadf8\ub9ac\ub2c8\uce58\ube4c\ub9ac\uc9c0\uc5d0 \ub3c4\ucc29\ud588\ub2e4.", "paragraph_id": "6507863-3-1", "paragraph_question": "question: \uc2a4\ubb34 \uc0b4\uc758 \ub51c\ub7f0\uc774 \ub300\ud559 \uc911\ud1f4 \ud6c4 \uac00\uae30\ub85c \ud55c \ub3c4\uc2dc\uc758 \uc774\ub984\uc740?, context: 1961\ub144 \ub300\ud559\uc744 \uc911\ud1f4\ud55c \ub51c\ub7f0\uc740 \uadf8\uac00 \uc22d\ubc30\ud55c \uc6b0\ub514 \uac70\uc2a4\ub9ac\uac00 \uc0ac\ub294 \ub274\uc695\uc2dc\ub85c \uac00\uae30\ub85c \uacb0\uc2ec\ud588\uace0, \uc2a4\ubb34 \uc0b4\uc758 \ub51c\ub7f0\uc740 \ub274\uc695 \uadf8\ub9ac\ub2c8\uce58\ube4c\ub9ac\uc9c0\uc5d0 \ub3c4\ucc29\ud588\ub2e4. \ub51c\ub7f0\uc740 \ub274\uc695\uc5d0 \ub3c4\ucc29\ud558\uc790\ub9c8\uc790 \uace7\ubc14\ub85c \ud55c \uc2dc\uac04 \ubc18 \ubc84\uc2a4\ub97c \ud0c0\uace0 \uc790\uc2e0\uc744 \uc74c\uc545\uc758 \uae38\ub85c \uc774\ub048 \uc601\uc6c5 \uc6b0\ub514 \uac70\uc2a4\ub9ac\ub97c \ub9cc\ub098\ub7ec \uac14\ub2e4. \uadf8\ub7ec\ub098 \uc6b0\ub514 \uac70\uc2a4\ub9ac\ub294 \ub299\uc5b4\uc11c \uc2e0\uacbd\ud1f4\ubcf4\uc99d\uc5d0 \uac78\ub824 \ub274\uc800\uc9c0\uc8fc\ub9bd\ubcd1\uc6d0\uc758 \ubcd1\uc0c1\uc5d0 \ub204\uc6cc \uc788\uc5c8\ub2e4. \uc218\uc2ed \uac1c\uc758 \uce68\uc0c1\uc774 \ub098\ub780\ud788 \ub193\uc5ec \uc788\ub294 \ubc29\uc5d0\uc11c \uac70\uc2a4\ub9ac\ub294 \ubb34\uae30\ub825\ud558\uac8c \ub204\uc6cc \uc788\uc5c8\ub2e4. \ubb34\uc11c\uc6b4 \ubcd1\ub9c8\uc5d0 \ud56d\ubcf5\ud55c \uc6b0\ub514 \uac70\uc2a4\ub9ac\uc758 \uc721\uccb4\ub97c \ubcf4\uba74\uc11c \ub51c\ub7f0\uc740 \uae30\uac81\ud588\uc73c\ub098 \uadf8\ub294 \uae30\ud0c0\ub97c \uce58\uba70 \uc790\uc2e0\uc774 \uc88b\uc544\ud55c \uc6b0\ub514 \uac70\uc2a4\ub9ac\uc758 \ub178\ub798\ub97c \ubd80\ub974\uba70 \uc8fd\uc5b4\uac00\ub294 \ub178\uc778\uc744 \uc704\ub85c\ud588\ub2e4. \uadf8\ub9ac\uace0 \ub5a8\ub9ac\ub294 \uc190\uc73c\ub85c \ub178\uc778\uc5d0\uac8c \uce5c\ud544 \uc11c\uba85\uc744 \ubc1b\uc558\ub2e4. \ucc98\uc640 \uc544\ub4e4\uc778 \uc544\ub860 \uac70\uc2a4\ub9ac\uac00 \ub51c\ub7f0\uc758 \ubc29\ubb38\uc5d0 \ub9e4\uc6b0 \uae30\ubed0\ud588\ub2e4\uace0 \uc804\ud574\uc9c4\ub2e4."} {"question": "\uc54c\ub809\uc2dc\uc6b0\uc2a4 4\uc138\ub97c \ud3d0\uc704\ud558\uace0 \uc2a4\uc2a4\ub85c \uc54c\ub809\uc2dc\uc6b0\uc2a4 5\uc138\ub85c\uc11c \uc81c\uc704\uc5d0 \uc624\ub978 \uac83\uc740 \ub204\uad6c\uc778\uac00?", "paragraph": "\ud55c\ud3b8 \uc54c\ub809\uc2dc\uc6b0\uc2a4 4\uc138\uc5d0\uac8c \ubd88\ub9cc\uc744 \ud488\uc740 \ud55c \ubb34\ub9ac\uc758 \uc2dc\ubbfc\ub4e4\uc774 \uc131 \uc18c\ud53c\uc544 \uc131\ub2f9\uc744 \uc810\uac70\ud558\uace0 \ub2c8\ucf5c\ub77c\uc2a4 \uce74\ub098\ubc14\uc6b0\uc2a4\ub97c \ud669\uc81c\ub85c \ucd94\uac1c\ud558\ub294 \uc0ac\uac74\uc774 \ubc8c\uc5b4\uc84c\ub2e4. \uc774 \uc18c\uc2dd\uc744 \ub4e4\uc740 \uc54c\ub809\uc2dc\uc6b0\uc2a4 4\uc138\ub294, \ub9c8\uce68\ub0b4 \uc2ed\uc790\uad70\uc5d0 \ub300\ud55c \uc800\ud56d\uc744 \ud3ec\uae30\ud558\uace0 \uc2ed\uc790\uad70\uc5d0\uac8c \uad81\uad90 \ub0b4\ub85c \ub4e4\uc5b4\uc640 \uc790\uc2e0\uc744 \uc9c0\ucf1c\ub2ec\ub77c\uace0 \uc694\uccad\ud588\ub2e4. \uadf8\ub7ec\ub098 \uc2ed\uc790\uad70\uc758 \ub3c4\uc6c0\uc744 \ubc1b\uae30\ub3c4 \uc804\uc5d0, \uc54c\ub809\uc2dc\uc6b0\uc2a4 \ubb34\ub974\uc8fc\ud50c\ub8e8\uc2a4\ub294 \uad81\uad90 \ub0b4 \uc778\uc0ac\ub4e4\uc744 \uaddc\ud569\ud55c \ub4a4, \uc54c\ub809\uc2dc\uc6b0\uc2a4 4\uc138\ub97c \ud3d0\uc704\ud558\uace0 \uc790\uc2e0\uc774 \uc54c\ub809\uc2dc\uc6b0\uc2a4 5\uc138\ub85c\uc11c \uc81c\uc704\uc5d0 \uc62c\ub790\ub2e4. \uadf8\ub7f0 \ub4a4, \uc131 \uc18c\ud53c\uc544 \uc131\ub2f9\uc73c\ub85c \uc9c4\uad70\ud558\uc5ec \ud669\uc81c\uc5d0 \ucd94\ub300\ub418\uc5c8\ub358 \ub2c8\ucf5c\ub77c\uc2a4 \uce78\ub098\ubc14\uc6b0\uc2a4\ub97c \uc0b4\ud574\ud588\uace0, \ub3c4\uc2dc \uc678\ubd80\uc758 \uc2ed\uc790\uad70\uc744 \uacf5\uaca9\ud558\uc600\uc73c\ub098 \uc624\ud788\ub824 \ud328\ubc30\ud558\uc600\ub2e4. \uc774 \ud328\ubc30\ub85c \ub0b4\ubd80\uc758 \ubd88\ub9cc\uc774 \uc2ec\ud574\uc9c0\uc790, \uc54c\ub809\uc2dc\uc6b0\uc2a4 5\uc138\ub294 \uc2ed\uc790\uad70\uce21\uc5d0\uac8c \ud68c\ub2f4\uc744 \uc81c\uc758\ud558\uc600\uc73c\ub098, \uc2ed\uc790\uad70\uce21\uc740 \uc54c\ub809\uc2dc\uc6b0\uc2a4 4\uc138\uc758 \ubcf5\uc704\ub97c \uc694\uad6c\ud558\uc600\uace0, \uc774\uc5d0 \uc54c\ub809\uc2dc\uc6b0\uc2a4 5\uc138\ub294 \uc624\ud788\ub824 \uc774\uc0ac\ud0a4\ub8e8\uc2a4\uc640 \uc54c\ub809\uc2dc\uc6b0\uc2a4 4\uc138\ub97c \uc0b4\ud574\ud568\uc73c\ub85c\uc368 \ubcf4\ub2f5\ud558\uc600\ub2e4. \uc2ed\uc790\uad70\uc740 \ub354 \uc774\uc0c1 \uae30\ub2e4\ub9ac\uc9c0 \uc54a\uace0 \ucf58\uc2a4\ud0c4\ud2f0\ub178\ud3f4\ub9ac\uc2a4\uc5d0 \ub300\ud55c \ub450 \ubc88\uc9f8 \uacf5\uaca9\uc744 \uac10\ud589\ud588\ub2e4.", "answer": "\uc54c\ub809\uc2dc\uc6b0\uc2a4 \ubb34\ub974\uc8fc\ud50c\ub8e8\uc2a4", "sentence": "\uadf8\ub7ec\ub098 \uc2ed\uc790\uad70\uc758 \ub3c4\uc6c0\uc744 \ubc1b\uae30\ub3c4 \uc804\uc5d0, \uc54c\ub809\uc2dc\uc6b0\uc2a4 \ubb34\ub974\uc8fc\ud50c\ub8e8\uc2a4 \ub294 \uad81\uad90 \ub0b4 \uc778\uc0ac\ub4e4\uc744 \uaddc\ud569\ud55c \ub4a4, \uc54c\ub809\uc2dc\uc6b0\uc2a4 4\uc138\ub97c \ud3d0\uc704\ud558\uace0 \uc790\uc2e0\uc774 \uc54c\ub809\uc2dc\uc6b0\uc2a4 5\uc138\ub85c\uc11c \uc81c\uc704\uc5d0 \uc62c\ub790\ub2e4.", "paragraph_sentence": "\ud55c\ud3b8 \uc54c\ub809\uc2dc\uc6b0\uc2a4 4\uc138\uc5d0\uac8c \ubd88\ub9cc\uc744 \ud488\uc740 \ud55c \ubb34\ub9ac\uc758 \uc2dc\ubbfc\ub4e4\uc774 \uc131 \uc18c\ud53c\uc544 \uc131\ub2f9\uc744 \uc810\uac70\ud558\uace0 \ub2c8\ucf5c\ub77c\uc2a4 \uce74\ub098\ubc14\uc6b0\uc2a4\ub97c \ud669\uc81c\ub85c \ucd94\uac1c\ud558\ub294 \uc0ac\uac74\uc774 \ubc8c\uc5b4\uc84c\ub2e4. \uc774 \uc18c\uc2dd\uc744 \ub4e4\uc740 \uc54c\ub809\uc2dc\uc6b0\uc2a4 4\uc138\ub294, \ub9c8\uce68\ub0b4 \uc2ed\uc790\uad70\uc5d0 \ub300\ud55c \uc800\ud56d\uc744 \ud3ec\uae30\ud558\uace0 \uc2ed\uc790\uad70\uc5d0\uac8c \uad81\uad90 \ub0b4\ub85c \ub4e4\uc5b4\uc640 \uc790\uc2e0\uc744 \uc9c0\ucf1c\ub2ec\ub77c\uace0 \uc694\uccad\ud588\ub2e4. \uadf8\ub7ec\ub098 \uc2ed\uc790\uad70\uc758 \ub3c4\uc6c0\uc744 \ubc1b\uae30\ub3c4 \uc804\uc5d0, \uc54c\ub809\uc2dc\uc6b0\uc2a4 \ubb34\ub974\uc8fc\ud50c\ub8e8\uc2a4 \ub294 \uad81\uad90 \ub0b4 \uc778\uc0ac\ub4e4\uc744 \uaddc\ud569\ud55c \ub4a4, \uc54c\ub809\uc2dc\uc6b0\uc2a4 4\uc138\ub97c \ud3d0\uc704\ud558\uace0 \uc790\uc2e0\uc774 \uc54c\ub809\uc2dc\uc6b0\uc2a4 5\uc138\ub85c\uc11c \uc81c\uc704\uc5d0 \uc62c\ub790\ub2e4. \uadf8\ub7f0 \ub4a4, \uc131 \uc18c\ud53c\uc544 \uc131\ub2f9\uc73c\ub85c \uc9c4\uad70\ud558\uc5ec \ud669\uc81c\uc5d0 \ucd94\ub300\ub418\uc5c8\ub358 \ub2c8\ucf5c\ub77c\uc2a4 \uce78\ub098\ubc14\uc6b0\uc2a4\ub97c \uc0b4\ud574\ud588\uace0, \ub3c4\uc2dc \uc678\ubd80\uc758 \uc2ed\uc790\uad70\uc744 \uacf5\uaca9\ud558\uc600\uc73c\ub098 \uc624\ud788\ub824 \ud328\ubc30\ud558\uc600\ub2e4. \uc774 \ud328\ubc30\ub85c \ub0b4\ubd80\uc758 \ubd88\ub9cc\uc774 \uc2ec\ud574\uc9c0\uc790, \uc54c\ub809\uc2dc\uc6b0\uc2a4 5\uc138\ub294 \uc2ed\uc790\uad70\uce21\uc5d0\uac8c \ud68c\ub2f4\uc744 \uc81c\uc758\ud558\uc600\uc73c\ub098, \uc2ed\uc790\uad70\uce21\uc740 \uc54c\ub809\uc2dc\uc6b0\uc2a4 4\uc138\uc758 \ubcf5\uc704\ub97c \uc694\uad6c\ud558\uc600\uace0, \uc774\uc5d0 \uc54c\ub809\uc2dc\uc6b0\uc2a4 5\uc138\ub294 \uc624\ud788\ub824 \uc774\uc0ac\ud0a4\ub8e8\uc2a4\uc640 \uc54c\ub809\uc2dc\uc6b0\uc2a4 4\uc138\ub97c \uc0b4\ud574\ud568\uc73c\ub85c\uc368 \ubcf4\ub2f5\ud558\uc600\ub2e4. \uc2ed\uc790\uad70\uc740 \ub354 \uc774\uc0c1 \uae30\ub2e4\ub9ac\uc9c0 \uc54a\uace0 \ucf58\uc2a4\ud0c4\ud2f0\ub178\ud3f4\ub9ac\uc2a4\uc5d0 \ub300\ud55c \ub450 \ubc88\uc9f8 \uacf5\uaca9\uc744 \uac10\ud589\ud588\ub2e4.", "paragraph_answer": "\ud55c\ud3b8 \uc54c\ub809\uc2dc\uc6b0\uc2a4 4\uc138\uc5d0\uac8c \ubd88\ub9cc\uc744 \ud488\uc740 \ud55c \ubb34\ub9ac\uc758 \uc2dc\ubbfc\ub4e4\uc774 \uc131 \uc18c\ud53c\uc544 \uc131\ub2f9\uc744 \uc810\uac70\ud558\uace0 \ub2c8\ucf5c\ub77c\uc2a4 \uce74\ub098\ubc14\uc6b0\uc2a4\ub97c \ud669\uc81c\ub85c \ucd94\uac1c\ud558\ub294 \uc0ac\uac74\uc774 \ubc8c\uc5b4\uc84c\ub2e4. \uc774 \uc18c\uc2dd\uc744 \ub4e4\uc740 \uc54c\ub809\uc2dc\uc6b0\uc2a4 4\uc138\ub294, \ub9c8\uce68\ub0b4 \uc2ed\uc790\uad70\uc5d0 \ub300\ud55c \uc800\ud56d\uc744 \ud3ec\uae30\ud558\uace0 \uc2ed\uc790\uad70\uc5d0\uac8c \uad81\uad90 \ub0b4\ub85c \ub4e4\uc5b4\uc640 \uc790\uc2e0\uc744 \uc9c0\ucf1c\ub2ec\ub77c\uace0 \uc694\uccad\ud588\ub2e4. \uadf8\ub7ec\ub098 \uc2ed\uc790\uad70\uc758 \ub3c4\uc6c0\uc744 \ubc1b\uae30\ub3c4 \uc804\uc5d0, \uc54c\ub809\uc2dc\uc6b0\uc2a4 \ubb34\ub974\uc8fc\ud50c\ub8e8\uc2a4 \ub294 \uad81\uad90 \ub0b4 \uc778\uc0ac\ub4e4\uc744 \uaddc\ud569\ud55c \ub4a4, \uc54c\ub809\uc2dc\uc6b0\uc2a4 4\uc138\ub97c \ud3d0\uc704\ud558\uace0 \uc790\uc2e0\uc774 \uc54c\ub809\uc2dc\uc6b0\uc2a4 5\uc138\ub85c\uc11c \uc81c\uc704\uc5d0 \uc62c\ub790\ub2e4. \uadf8\ub7f0 \ub4a4, \uc131 \uc18c\ud53c\uc544 \uc131\ub2f9\uc73c\ub85c \uc9c4\uad70\ud558\uc5ec \ud669\uc81c\uc5d0 \ucd94\ub300\ub418\uc5c8\ub358 \ub2c8\ucf5c\ub77c\uc2a4 \uce78\ub098\ubc14\uc6b0\uc2a4\ub97c \uc0b4\ud574\ud588\uace0, \ub3c4\uc2dc \uc678\ubd80\uc758 \uc2ed\uc790\uad70\uc744 \uacf5\uaca9\ud558\uc600\uc73c\ub098 \uc624\ud788\ub824 \ud328\ubc30\ud558\uc600\ub2e4. \uc774 \ud328\ubc30\ub85c \ub0b4\ubd80\uc758 \ubd88\ub9cc\uc774 \uc2ec\ud574\uc9c0\uc790, \uc54c\ub809\uc2dc\uc6b0\uc2a4 5\uc138\ub294 \uc2ed\uc790\uad70\uce21\uc5d0\uac8c \ud68c\ub2f4\uc744 \uc81c\uc758\ud558\uc600\uc73c\ub098, \uc2ed\uc790\uad70\uce21\uc740 \uc54c\ub809\uc2dc\uc6b0\uc2a4 4\uc138\uc758 \ubcf5\uc704\ub97c \uc694\uad6c\ud558\uc600\uace0, \uc774\uc5d0 \uc54c\ub809\uc2dc\uc6b0\uc2a4 5\uc138\ub294 \uc624\ud788\ub824 \uc774\uc0ac\ud0a4\ub8e8\uc2a4\uc640 \uc54c\ub809\uc2dc\uc6b0\uc2a4 4\uc138\ub97c \uc0b4\ud574\ud568\uc73c\ub85c\uc368 \ubcf4\ub2f5\ud558\uc600\ub2e4. \uc2ed\uc790\uad70\uc740 \ub354 \uc774\uc0c1 \uae30\ub2e4\ub9ac\uc9c0 \uc54a\uace0 \ucf58\uc2a4\ud0c4\ud2f0\ub178\ud3f4\ub9ac\uc2a4\uc5d0 \ub300\ud55c \ub450 \ubc88\uc9f8 \uacf5\uaca9\uc744 \uac10\ud589\ud588\ub2e4.", "sentence_answer": "\uadf8\ub7ec\ub098 \uc2ed\uc790\uad70\uc758 \ub3c4\uc6c0\uc744 \ubc1b\uae30\ub3c4 \uc804\uc5d0, \uc54c\ub809\uc2dc\uc6b0\uc2a4 \ubb34\ub974\uc8fc\ud50c\ub8e8\uc2a4 \ub294 \uad81\uad90 \ub0b4 \uc778\uc0ac\ub4e4\uc744 \uaddc\ud569\ud55c \ub4a4, \uc54c\ub809\uc2dc\uc6b0\uc2a4 4\uc138\ub97c \ud3d0\uc704\ud558\uace0 \uc790\uc2e0\uc774 \uc54c\ub809\uc2dc\uc6b0\uc2a4 5\uc138\ub85c\uc11c \uc81c\uc704\uc5d0 \uc62c\ub790\ub2e4.", "paragraph_id": "6550839-8-2", "paragraph_question": "question: \uc54c\ub809\uc2dc\uc6b0\uc2a4 4\uc138\ub97c \ud3d0\uc704\ud558\uace0 \uc2a4\uc2a4\ub85c \uc54c\ub809\uc2dc\uc6b0\uc2a4 5\uc138\ub85c\uc11c \uc81c\uc704\uc5d0 \uc624\ub978 \uac83\uc740 \ub204\uad6c\uc778\uac00?, context: \ud55c\ud3b8 \uc54c\ub809\uc2dc\uc6b0\uc2a4 4\uc138\uc5d0\uac8c \ubd88\ub9cc\uc744 \ud488\uc740 \ud55c \ubb34\ub9ac\uc758 \uc2dc\ubbfc\ub4e4\uc774 \uc131 \uc18c\ud53c\uc544 \uc131\ub2f9\uc744 \uc810\uac70\ud558\uace0 \ub2c8\ucf5c\ub77c\uc2a4 \uce74\ub098\ubc14\uc6b0\uc2a4\ub97c \ud669\uc81c\ub85c \ucd94\uac1c\ud558\ub294 \uc0ac\uac74\uc774 \ubc8c\uc5b4\uc84c\ub2e4. \uc774 \uc18c\uc2dd\uc744 \ub4e4\uc740 \uc54c\ub809\uc2dc\uc6b0\uc2a4 4\uc138\ub294, \ub9c8\uce68\ub0b4 \uc2ed\uc790\uad70\uc5d0 \ub300\ud55c \uc800\ud56d\uc744 \ud3ec\uae30\ud558\uace0 \uc2ed\uc790\uad70\uc5d0\uac8c \uad81\uad90 \ub0b4\ub85c \ub4e4\uc5b4\uc640 \uc790\uc2e0\uc744 \uc9c0\ucf1c\ub2ec\ub77c\uace0 \uc694\uccad\ud588\ub2e4. \uadf8\ub7ec\ub098 \uc2ed\uc790\uad70\uc758 \ub3c4\uc6c0\uc744 \ubc1b\uae30\ub3c4 \uc804\uc5d0, \uc54c\ub809\uc2dc\uc6b0\uc2a4 \ubb34\ub974\uc8fc\ud50c\ub8e8\uc2a4\ub294 \uad81\uad90 \ub0b4 \uc778\uc0ac\ub4e4\uc744 \uaddc\ud569\ud55c \ub4a4, \uc54c\ub809\uc2dc\uc6b0\uc2a4 4\uc138\ub97c \ud3d0\uc704\ud558\uace0 \uc790\uc2e0\uc774 \uc54c\ub809\uc2dc\uc6b0\uc2a4 5\uc138\ub85c\uc11c \uc81c\uc704\uc5d0 \uc62c\ub790\ub2e4. \uadf8\ub7f0 \ub4a4, \uc131 \uc18c\ud53c\uc544 \uc131\ub2f9\uc73c\ub85c \uc9c4\uad70\ud558\uc5ec \ud669\uc81c\uc5d0 \ucd94\ub300\ub418\uc5c8\ub358 \ub2c8\ucf5c\ub77c\uc2a4 \uce78\ub098\ubc14\uc6b0\uc2a4\ub97c \uc0b4\ud574\ud588\uace0, \ub3c4\uc2dc \uc678\ubd80\uc758 \uc2ed\uc790\uad70\uc744 \uacf5\uaca9\ud558\uc600\uc73c\ub098 \uc624\ud788\ub824 \ud328\ubc30\ud558\uc600\ub2e4. \uc774 \ud328\ubc30\ub85c \ub0b4\ubd80\uc758 \ubd88\ub9cc\uc774 \uc2ec\ud574\uc9c0\uc790, \uc54c\ub809\uc2dc\uc6b0\uc2a4 5\uc138\ub294 \uc2ed\uc790\uad70\uce21\uc5d0\uac8c \ud68c\ub2f4\uc744 \uc81c\uc758\ud558\uc600\uc73c\ub098, \uc2ed\uc790\uad70\uce21\uc740 \uc54c\ub809\uc2dc\uc6b0\uc2a4 4\uc138\uc758 \ubcf5\uc704\ub97c \uc694\uad6c\ud558\uc600\uace0, \uc774\uc5d0 \uc54c\ub809\uc2dc\uc6b0\uc2a4 5\uc138\ub294 \uc624\ud788\ub824 \uc774\uc0ac\ud0a4\ub8e8\uc2a4\uc640 \uc54c\ub809\uc2dc\uc6b0\uc2a4 4\uc138\ub97c \uc0b4\ud574\ud568\uc73c\ub85c\uc368 \ubcf4\ub2f5\ud558\uc600\ub2e4. \uc2ed\uc790\uad70\uc740 \ub354 \uc774\uc0c1 \uae30\ub2e4\ub9ac\uc9c0 \uc54a\uace0 \ucf58\uc2a4\ud0c4\ud2f0\ub178\ud3f4\ub9ac\uc2a4\uc5d0 \ub300\ud55c \ub450 \ubc88\uc9f8 \uacf5\uaca9\uc744 \uac10\ud589\ud588\ub2e4."} {"question": "\uacbd\uc131\uc77c\ucd9c\uacf5\ub9bd\uc2ec\uc0c1\uc18c\ud559\uad50\ub294 \uc5b4\ub5a4 \uc0ac\ub78c\ub4e4\uc744 \uc704\ud574 \uc138\uc6cc\uc9c4 \ud559\uad50\uc778\uac00?", "paragraph": "1920\ub144 4\uc6d4 1\uc77c\ubd80\ud130 \uad81 \ub0b4\uc5d0\uc11c \uc2ec\uc0c1\uc18c\ud559\uad50 1\ud559\ub144 \uacfc\uc815\uc744 3\uc778\uc758 \ud559\uc6b0\ub4e4\uacfc \uac19\uc774 \uad50\uc721 \ubc1b\ub2e4\uac00 1921\ub144 4\uc6d4 1\uc77c \uacbd\uc131\uc5d0 \uba38\ubb38 \uc77c\ubcf8\uc778\uc744 \uc218\uc6a9\ud558\uae30 \uc704\ud574 \uc124\ub9bd\ub41c \uacbd\uc131\uc77c\ucd9c\uacf5\ub9bd\uc2ec\uc0c1\uc18c\ud559\uad50 2\ud559\ub144\uc73c\ub85c \uc785\ud559\ud558\uc600\ub2e4. \uc77c\ucd9c\uc2ec\uc0c1\uc18c\ud559\uad50\uc5d0\uc11c\ub294 \uc77c\ubcf8\ubcf5\uc2dd\uc744 \uc785\uc5b4\uc57c \ud55c\ub2e4\ub294 \uaddc\uc815\uc774 \uc788\uc5c8\uc73c\ubbc0\ub85c \ud1b5\ud559\ud560 \ub54c\uc5d0\ub294 \uc77c\ubcf8 \ubcf5\uc2dd\uc744 \uac16\ucd94\uc5b4 \uc785\uc5c8\ub294\ub370, \uc639\uc8fc\ub294 \uc8fc\ub85c \ubaac\uc4f0\ud0a4\uc640 \ud558\uce74\ub9c8\ub97c \ucc29\uc6a9\ud558\uc600\ub2e4. \ud559\uad50\uc758 \uc218\uc5c5\uc740 \uc77c\ubcf8\uc5b4\ub85c \uc9c4\ud589\ub418\uc5c8\uace0, \uadf8\ub140\ub294 \uc720\ud559 \uc774\uc804\uc5d0 \uc774\ubbf8 \uc77c\ubcf8\uc5b4\ub97c \ub2a5\uc219\ud558\uac8c \uad6c\uc0ac\ud558\uc600\ub2e4. 1921\ub144 5\uc6d4 4\uc77c\uc5d0 \u2018\ub355\ud61c\u2019\ub77c\ub294 \ud638\ub97c \uc815\uc2dd\uc73c\ub85c \ud558\uc0ac\ubc1b\uc558\uc73c\uba70, \uc77c\ubcf8 \uad81\ub0b4\uc131\uc5d0 \uc0c1\uc2e0\uc744 \uac70\uccd0 \uc639\uc8fc\uc758 \uc874\uce6d\uc744 \uc774\ub54c\ubd80\ud130 \uc0ac\uc6a9\ud558\uac8c \ub418\uc5c8\ub2e4. \ub2f9\uc2dc \ud568\uaed8 \uc7ac\ud559\uc911\uc774\ub358 \uae09\uc6b0\uc758 \ud68c\uace0\ub85d\uc5d0 \ub530\ub974\uba74, \uc639\uc8fc\ub294 \uc655\uc871\ub2e4\uc6b4 \uae30\ud488\uc744 \uac16\ucd94\uace0, \ud0a4\uac00 \ud06c\uace0 \uc5bc\uad74\uc774 \ud76c\uc5c8\uc73c\uba70, \uba38\ub9ac\ub294 \ud55c\uac00\uc6b4\ub370\ub97c \ubc18\uc73c\ub85c \ub098\ub204\uc5b4 \ub4a4\uc5d0\uc11c \uc591\ucabd\uc73c\ub85c \uae38\uac8c \ub54b\uc544 \uc58c\uc804\ud558\uac8c \ub298\uc774\uace0, \uc77c\ubcf8 \uc637\uc5d0 \ud558\uce74\ub9c8\ub97c \uc785\uace0 \ud559\uad50\ub97c \ub2e4\ub154\ub2e4\uace0 \ud55c\ub2e4. \ub610\ud55c \uc0c1\uad81\uc774 \ud56d\uc0c1 \ub9c8\ucc28\ub97c \uac19\uc774 \ud0c0\uace0 \uc640 \uc218\uc5c5 \uc911 \uad50\uc2e4 \ub4a4\uc5d0 \uc11c\uc11c \uc9c0\ucf1c\ubcf4\uace0 \uc788\uc5c8\ub2e4\uace0 \ud55c\ub2e4. \ub2f9\uc2dc \uc639\uc8fc\uc758 \ub610\ub2e4\ub978 \ud559\uc6b0\ub294 \u201c\uc639\uc8fc\uac00 \uacf5\ubd80\ub97c \uc798 \ud558\uc600\uc73c\uba70, \ud2b9\ud788 \uc2b5\uc790\uc5d0 \ub2a5\ud558\uc600\ub2e4\u201d\uace0 \ud68c\uc0c1\ud558\uace0 \uc788\ub2e4. \uc639\uc8fc\ub294 \ub3d9\uc694\ub97c \uc9d3\ub294\ub370\uc5d0\ub3c4 \ub2a5\ud558\uc600\ub294\ub370, \uc77c\ucd9c \uc2ec\uc0c1\uc18c\ud559\uad50 \uc7ac\ud559 \uc2dc\uc808 \uc9c0\uc740 \ub3d9\uc694 \ud55c \ud3b8\uc758 \uac00\uc0ac\ub294 \ub2e4\uc74c\uacfc \uac19\ub2e4.", "answer": "\uc77c\ubcf8\uc778", "sentence": "1920\ub144 4\uc6d4 1\uc77c\ubd80\ud130 \uad81 \ub0b4\uc5d0\uc11c \uc2ec\uc0c1\uc18c\ud559\uad50 1\ud559\ub144 \uacfc\uc815\uc744 3\uc778\uc758 \ud559\uc6b0\ub4e4\uacfc \uac19\uc774 \uad50\uc721 \ubc1b\ub2e4\uac00 1921\ub144 4\uc6d4 1\uc77c \uacbd\uc131\uc5d0 \uba38\ubb38 \uc77c\ubcf8\uc778 \uc744 \uc218\uc6a9\ud558\uae30 \uc704\ud574 \uc124\ub9bd\ub41c \uacbd\uc131\uc77c\ucd9c\uacf5\ub9bd\uc2ec\uc0c1\uc18c\ud559\uad50 2\ud559\ub144\uc73c\ub85c \uc785\ud559\ud558\uc600\ub2e4.", "paragraph_sentence": " 1920\ub144 4\uc6d4 1\uc77c\ubd80\ud130 \uad81 \ub0b4\uc5d0\uc11c \uc2ec\uc0c1\uc18c\ud559\uad50 1\ud559\ub144 \uacfc\uc815\uc744 3\uc778\uc758 \ud559\uc6b0\ub4e4\uacfc \uac19\uc774 \uad50\uc721 \ubc1b\ub2e4\uac00 1921\ub144 4\uc6d4 1\uc77c \uacbd\uc131\uc5d0 \uba38\ubb38 \uc77c\ubcf8\uc778 \uc744 \uc218\uc6a9\ud558\uae30 \uc704\ud574 \uc124\ub9bd\ub41c \uacbd\uc131\uc77c\ucd9c\uacf5\ub9bd\uc2ec\uc0c1\uc18c\ud559\uad50 2\ud559\ub144\uc73c\ub85c \uc785\ud559\ud558\uc600\ub2e4. \uc77c\ucd9c\uc2ec\uc0c1\uc18c\ud559\uad50\uc5d0\uc11c\ub294 \uc77c\ubcf8\ubcf5\uc2dd\uc744 \uc785\uc5b4\uc57c \ud55c\ub2e4\ub294 \uaddc\uc815\uc774 \uc788\uc5c8\uc73c\ubbc0\ub85c \ud1b5\ud559\ud560 \ub54c\uc5d0\ub294 \uc77c\ubcf8 \ubcf5\uc2dd\uc744 \uac16\ucd94\uc5b4 \uc785\uc5c8\ub294\ub370, \uc639\uc8fc\ub294 \uc8fc\ub85c \ubaac\uc4f0\ud0a4\uc640 \ud558\uce74\ub9c8\ub97c \ucc29\uc6a9\ud558\uc600\ub2e4. \ud559\uad50\uc758 \uc218\uc5c5\uc740 \uc77c\ubcf8\uc5b4\ub85c \uc9c4\ud589\ub418\uc5c8\uace0, \uadf8\ub140\ub294 \uc720\ud559 \uc774\uc804\uc5d0 \uc774\ubbf8 \uc77c\ubcf8\uc5b4\ub97c \ub2a5\uc219\ud558\uac8c \uad6c\uc0ac\ud558\uc600\ub2e4. 1921\ub144 5\uc6d4 4\uc77c\uc5d0 \u2018\ub355\ud61c\u2019\ub77c\ub294 \ud638\ub97c \uc815\uc2dd\uc73c\ub85c \ud558\uc0ac\ubc1b\uc558\uc73c\uba70, \uc77c\ubcf8 \uad81\ub0b4\uc131\uc5d0 \uc0c1\uc2e0\uc744 \uac70\uccd0 \uc639\uc8fc\uc758 \uc874\uce6d\uc744 \uc774\ub54c\ubd80\ud130 \uc0ac\uc6a9\ud558\uac8c \ub418\uc5c8\ub2e4. \ub2f9\uc2dc \ud568\uaed8 \uc7ac\ud559\uc911\uc774\ub358 \uae09\uc6b0\uc758 \ud68c\uace0\ub85d\uc5d0 \ub530\ub974\uba74, \uc639\uc8fc\ub294 \uc655\uc871\ub2e4\uc6b4 \uae30\ud488\uc744 \uac16\ucd94\uace0, \ud0a4\uac00 \ud06c\uace0 \uc5bc\uad74\uc774 \ud76c\uc5c8\uc73c\uba70, \uba38\ub9ac\ub294 \ud55c\uac00\uc6b4\ub370\ub97c \ubc18\uc73c\ub85c \ub098\ub204\uc5b4 \ub4a4\uc5d0\uc11c \uc591\ucabd\uc73c\ub85c \uae38\uac8c \ub54b\uc544 \uc58c\uc804\ud558\uac8c \ub298\uc774\uace0, \uc77c\ubcf8 \uc637\uc5d0 \ud558\uce74\ub9c8\ub97c \uc785\uace0 \ud559\uad50\ub97c \ub2e4\ub154\ub2e4\uace0 \ud55c\ub2e4. \ub610\ud55c \uc0c1\uad81\uc774 \ud56d\uc0c1 \ub9c8\ucc28\ub97c \uac19\uc774 \ud0c0\uace0 \uc640 \uc218\uc5c5 \uc911 \uad50\uc2e4 \ub4a4\uc5d0 \uc11c\uc11c \uc9c0\ucf1c\ubcf4\uace0 \uc788\uc5c8\ub2e4\uace0 \ud55c\ub2e4. \ub2f9\uc2dc \uc639\uc8fc\uc758 \ub610\ub2e4\ub978 \ud559\uc6b0\ub294 \u201c\uc639\uc8fc\uac00 \uacf5\ubd80\ub97c \uc798 \ud558\uc600\uc73c\uba70, \ud2b9\ud788 \uc2b5\uc790\uc5d0 \ub2a5\ud558\uc600\ub2e4\u201d\uace0 \ud68c\uc0c1\ud558\uace0 \uc788\ub2e4. \uc639\uc8fc\ub294 \ub3d9\uc694\ub97c \uc9d3\ub294\ub370\uc5d0\ub3c4 \ub2a5\ud558\uc600\ub294\ub370, \uc77c\ucd9c \uc2ec\uc0c1\uc18c\ud559\uad50 \uc7ac\ud559 \uc2dc\uc808 \uc9c0\uc740 \ub3d9\uc694 \ud55c \ud3b8\uc758 \uac00\uc0ac\ub294 \ub2e4\uc74c\uacfc \uac19\ub2e4.", "paragraph_answer": "1920\ub144 4\uc6d4 1\uc77c\ubd80\ud130 \uad81 \ub0b4\uc5d0\uc11c \uc2ec\uc0c1\uc18c\ud559\uad50 1\ud559\ub144 \uacfc\uc815\uc744 3\uc778\uc758 \ud559\uc6b0\ub4e4\uacfc \uac19\uc774 \uad50\uc721 \ubc1b\ub2e4\uac00 1921\ub144 4\uc6d4 1\uc77c \uacbd\uc131\uc5d0 \uba38\ubb38 \uc77c\ubcf8\uc778 \uc744 \uc218\uc6a9\ud558\uae30 \uc704\ud574 \uc124\ub9bd\ub41c \uacbd\uc131\uc77c\ucd9c\uacf5\ub9bd\uc2ec\uc0c1\uc18c\ud559\uad50 2\ud559\ub144\uc73c\ub85c \uc785\ud559\ud558\uc600\ub2e4. \uc77c\ucd9c\uc2ec\uc0c1\uc18c\ud559\uad50\uc5d0\uc11c\ub294 \uc77c\ubcf8\ubcf5\uc2dd\uc744 \uc785\uc5b4\uc57c \ud55c\ub2e4\ub294 \uaddc\uc815\uc774 \uc788\uc5c8\uc73c\ubbc0\ub85c \ud1b5\ud559\ud560 \ub54c\uc5d0\ub294 \uc77c\ubcf8 \ubcf5\uc2dd\uc744 \uac16\ucd94\uc5b4 \uc785\uc5c8\ub294\ub370, \uc639\uc8fc\ub294 \uc8fc\ub85c \ubaac\uc4f0\ud0a4\uc640 \ud558\uce74\ub9c8\ub97c \ucc29\uc6a9\ud558\uc600\ub2e4. \ud559\uad50\uc758 \uc218\uc5c5\uc740 \uc77c\ubcf8\uc5b4\ub85c \uc9c4\ud589\ub418\uc5c8\uace0, \uadf8\ub140\ub294 \uc720\ud559 \uc774\uc804\uc5d0 \uc774\ubbf8 \uc77c\ubcf8\uc5b4\ub97c \ub2a5\uc219\ud558\uac8c \uad6c\uc0ac\ud558\uc600\ub2e4. 1921\ub144 5\uc6d4 4\uc77c\uc5d0 \u2018\ub355\ud61c\u2019\ub77c\ub294 \ud638\ub97c \uc815\uc2dd\uc73c\ub85c \ud558\uc0ac\ubc1b\uc558\uc73c\uba70, \uc77c\ubcf8 \uad81\ub0b4\uc131\uc5d0 \uc0c1\uc2e0\uc744 \uac70\uccd0 \uc639\uc8fc\uc758 \uc874\uce6d\uc744 \uc774\ub54c\ubd80\ud130 \uc0ac\uc6a9\ud558\uac8c \ub418\uc5c8\ub2e4. \ub2f9\uc2dc \ud568\uaed8 \uc7ac\ud559\uc911\uc774\ub358 \uae09\uc6b0\uc758 \ud68c\uace0\ub85d\uc5d0 \ub530\ub974\uba74, \uc639\uc8fc\ub294 \uc655\uc871\ub2e4\uc6b4 \uae30\ud488\uc744 \uac16\ucd94\uace0, \ud0a4\uac00 \ud06c\uace0 \uc5bc\uad74\uc774 \ud76c\uc5c8\uc73c\uba70, \uba38\ub9ac\ub294 \ud55c\uac00\uc6b4\ub370\ub97c \ubc18\uc73c\ub85c \ub098\ub204\uc5b4 \ub4a4\uc5d0\uc11c \uc591\ucabd\uc73c\ub85c \uae38\uac8c \ub54b\uc544 \uc58c\uc804\ud558\uac8c \ub298\uc774\uace0, \uc77c\ubcf8 \uc637\uc5d0 \ud558\uce74\ub9c8\ub97c \uc785\uace0 \ud559\uad50\ub97c \ub2e4\ub154\ub2e4\uace0 \ud55c\ub2e4. \ub610\ud55c \uc0c1\uad81\uc774 \ud56d\uc0c1 \ub9c8\ucc28\ub97c \uac19\uc774 \ud0c0\uace0 \uc640 \uc218\uc5c5 \uc911 \uad50\uc2e4 \ub4a4\uc5d0 \uc11c\uc11c \uc9c0\ucf1c\ubcf4\uace0 \uc788\uc5c8\ub2e4\uace0 \ud55c\ub2e4. \ub2f9\uc2dc \uc639\uc8fc\uc758 \ub610\ub2e4\ub978 \ud559\uc6b0\ub294 \u201c\uc639\uc8fc\uac00 \uacf5\ubd80\ub97c \uc798 \ud558\uc600\uc73c\uba70, \ud2b9\ud788 \uc2b5\uc790\uc5d0 \ub2a5\ud558\uc600\ub2e4\u201d\uace0 \ud68c\uc0c1\ud558\uace0 \uc788\ub2e4. \uc639\uc8fc\ub294 \ub3d9\uc694\ub97c \uc9d3\ub294\ub370\uc5d0\ub3c4 \ub2a5\ud558\uc600\ub294\ub370, \uc77c\ucd9c \uc2ec\uc0c1\uc18c\ud559\uad50 \uc7ac\ud559 \uc2dc\uc808 \uc9c0\uc740 \ub3d9\uc694 \ud55c \ud3b8\uc758 \uac00\uc0ac\ub294 \ub2e4\uc74c\uacfc \uac19\ub2e4.", "sentence_answer": "1920\ub144 4\uc6d4 1\uc77c\ubd80\ud130 \uad81 \ub0b4\uc5d0\uc11c \uc2ec\uc0c1\uc18c\ud559\uad50 1\ud559\ub144 \uacfc\uc815\uc744 3\uc778\uc758 \ud559\uc6b0\ub4e4\uacfc \uac19\uc774 \uad50\uc721 \ubc1b\ub2e4\uac00 1921\ub144 4\uc6d4 1\uc77c \uacbd\uc131\uc5d0 \uba38\ubb38 \uc77c\ubcf8\uc778 \uc744 \uc218\uc6a9\ud558\uae30 \uc704\ud574 \uc124\ub9bd\ub41c \uacbd\uc131\uc77c\ucd9c\uacf5\ub9bd\uc2ec\uc0c1\uc18c\ud559\uad50 2\ud559\ub144\uc73c\ub85c \uc785\ud559\ud558\uc600\ub2e4.", "paragraph_id": "6491328-2-0", "paragraph_question": "question: \uacbd\uc131\uc77c\ucd9c\uacf5\ub9bd\uc2ec\uc0c1\uc18c\ud559\uad50\ub294 \uc5b4\ub5a4 \uc0ac\ub78c\ub4e4\uc744 \uc704\ud574 \uc138\uc6cc\uc9c4 \ud559\uad50\uc778\uac00?, context: 1920\ub144 4\uc6d4 1\uc77c\ubd80\ud130 \uad81 \ub0b4\uc5d0\uc11c \uc2ec\uc0c1\uc18c\ud559\uad50 1\ud559\ub144 \uacfc\uc815\uc744 3\uc778\uc758 \ud559\uc6b0\ub4e4\uacfc \uac19\uc774 \uad50\uc721 \ubc1b\ub2e4\uac00 1921\ub144 4\uc6d4 1\uc77c \uacbd\uc131\uc5d0 \uba38\ubb38 \uc77c\ubcf8\uc778\uc744 \uc218\uc6a9\ud558\uae30 \uc704\ud574 \uc124\ub9bd\ub41c \uacbd\uc131\uc77c\ucd9c\uacf5\ub9bd\uc2ec\uc0c1\uc18c\ud559\uad50 2\ud559\ub144\uc73c\ub85c \uc785\ud559\ud558\uc600\ub2e4. \uc77c\ucd9c\uc2ec\uc0c1\uc18c\ud559\uad50\uc5d0\uc11c\ub294 \uc77c\ubcf8\ubcf5\uc2dd\uc744 \uc785\uc5b4\uc57c \ud55c\ub2e4\ub294 \uaddc\uc815\uc774 \uc788\uc5c8\uc73c\ubbc0\ub85c \ud1b5\ud559\ud560 \ub54c\uc5d0\ub294 \uc77c\ubcf8 \ubcf5\uc2dd\uc744 \uac16\ucd94\uc5b4 \uc785\uc5c8\ub294\ub370, \uc639\uc8fc\ub294 \uc8fc\ub85c \ubaac\uc4f0\ud0a4\uc640 \ud558\uce74\ub9c8\ub97c \ucc29\uc6a9\ud558\uc600\ub2e4. \ud559\uad50\uc758 \uc218\uc5c5\uc740 \uc77c\ubcf8\uc5b4\ub85c \uc9c4\ud589\ub418\uc5c8\uace0, \uadf8\ub140\ub294 \uc720\ud559 \uc774\uc804\uc5d0 \uc774\ubbf8 \uc77c\ubcf8\uc5b4\ub97c \ub2a5\uc219\ud558\uac8c \uad6c\uc0ac\ud558\uc600\ub2e4. 1921\ub144 5\uc6d4 4\uc77c\uc5d0 \u2018\ub355\ud61c\u2019\ub77c\ub294 \ud638\ub97c \uc815\uc2dd\uc73c\ub85c \ud558\uc0ac\ubc1b\uc558\uc73c\uba70, \uc77c\ubcf8 \uad81\ub0b4\uc131\uc5d0 \uc0c1\uc2e0\uc744 \uac70\uccd0 \uc639\uc8fc\uc758 \uc874\uce6d\uc744 \uc774\ub54c\ubd80\ud130 \uc0ac\uc6a9\ud558\uac8c \ub418\uc5c8\ub2e4. \ub2f9\uc2dc \ud568\uaed8 \uc7ac\ud559\uc911\uc774\ub358 \uae09\uc6b0\uc758 \ud68c\uace0\ub85d\uc5d0 \ub530\ub974\uba74, \uc639\uc8fc\ub294 \uc655\uc871\ub2e4\uc6b4 \uae30\ud488\uc744 \uac16\ucd94\uace0, \ud0a4\uac00 \ud06c\uace0 \uc5bc\uad74\uc774 \ud76c\uc5c8\uc73c\uba70, \uba38\ub9ac\ub294 \ud55c\uac00\uc6b4\ub370\ub97c \ubc18\uc73c\ub85c \ub098\ub204\uc5b4 \ub4a4\uc5d0\uc11c \uc591\ucabd\uc73c\ub85c \uae38\uac8c \ub54b\uc544 \uc58c\uc804\ud558\uac8c \ub298\uc774\uace0, \uc77c\ubcf8 \uc637\uc5d0 \ud558\uce74\ub9c8\ub97c \uc785\uace0 \ud559\uad50\ub97c \ub2e4\ub154\ub2e4\uace0 \ud55c\ub2e4. \ub610\ud55c \uc0c1\uad81\uc774 \ud56d\uc0c1 \ub9c8\ucc28\ub97c \uac19\uc774 \ud0c0\uace0 \uc640 \uc218\uc5c5 \uc911 \uad50\uc2e4 \ub4a4\uc5d0 \uc11c\uc11c \uc9c0\ucf1c\ubcf4\uace0 \uc788\uc5c8\ub2e4\uace0 \ud55c\ub2e4. \ub2f9\uc2dc \uc639\uc8fc\uc758 \ub610\ub2e4\ub978 \ud559\uc6b0\ub294 \u201c\uc639\uc8fc\uac00 \uacf5\ubd80\ub97c \uc798 \ud558\uc600\uc73c\uba70, \ud2b9\ud788 \uc2b5\uc790\uc5d0 \ub2a5\ud558\uc600\ub2e4\u201d\uace0 \ud68c\uc0c1\ud558\uace0 \uc788\ub2e4. \uc639\uc8fc\ub294 \ub3d9\uc694\ub97c \uc9d3\ub294\ub370\uc5d0\ub3c4 \ub2a5\ud558\uc600\ub294\ub370, \uc77c\ucd9c \uc2ec\uc0c1\uc18c\ud559\uad50 \uc7ac\ud559 \uc2dc\uc808 \uc9c0\uc740 \ub3d9\uc694 \ud55c \ud3b8\uc758 \uac00\uc0ac\ub294 \ub2e4\uc74c\uacfc \uac19\ub2e4."} {"question": "\uc6d0\uc815\ud654\uc758 \uc758\ubd93\uc544\ubc84\uc9c0 \uc774\ub984\uc740 \ubb34\uc5c7\uc778\uac00?", "paragraph": "\uc6d4\uac04\uc9c0\u2018\uc2e0\ub3d9\uc544\u2019(2014\ub144 4\uc6d4\ud638)\ub294 \"\uc6d0\uc815\ud654\uac00 2014\ub144 2\uc6d4 \ub9d0 \uc790\uc2e0\uc758 \uc758\ubd93\uc544\ubc84\uc9c0\uc778 \uae40\ub3d9\uc21c\uc744 \ub9cc\ub098 \ubd81\ud55c \ubcf4\uc704\ubd80 \uc9c1\ud30c\uac04\ucca9, \ud0c8\ubd81 \uc774\ud6c4 3\ucc28\ub840 \ubd81\ud55c \ubc29\ubb38 \ub4f1 \uc0ac\uac74 \ub2f9\uc2dc\ubd80\ud130 \uc790\uc2e0\uc774 \uc8fc\uc7a5\ud588\ub358 \uc8fc\uc694 \uac04\ucca9 \ud589\uc801\uc744 \ubd80\uc778\ud558\ub294 \ub300\ud654\ub97c \ud588\ub2e4.\"\uace0 \ubcf4\ub3c4\ud588\ub2e4. \ubcf4\ub3c4 \uc9c1\ud6c4 \ubbfc\uc8fc\uc0ac\ud68c\ub97c \uc704\ud55c \ubcc0\ud638\uc0ac \ubaa8\uc784(\ubbfc\ubcc0)\uc740 \u2018\uc2e0\ub3d9\uc544 \ubcf4\ub3c4\uc5d0 \ub300\ud55c \uc785\uc7a5\u2019\uc774\ub780 \uc81c\ubaa9\uc758 \ubcf4\ub3c4\uc790\ub8cc\ub97c \ubc1c\ud45c\ud558\uace0 \uc9c4\uc0c1\uaddc\uba85\uc744 \uc694\uad6c\ud588\ub2e4. \ud55c\uaca8\ub808\ub294 2014\ub144 3\uc6d4 22\uc77c \u201c\uac80\ucc30\ub85c\ubd80\ud130 \ud5c8\uc704\uc9c4\uc220\uc744 \uac15\uc694\ub2f9\ud588\ub2e4. \ub098\ub294 \uac04\ucca9\uc774 \ub9de\uc9c0\ub9cc, \uc544\ubc84\uc9c0\ub294 \uc544\ub2c8\ub2e4\u201d\ub77c\ub294 \uc6d0\uc815\ud654 \uc778\ud130\ubdf0 \ub0b4\uc6a9\uc744 \ubcf4\ub3c4\ud558\uba74\uc11c \uc218\uc0ac \uc870\uc791 \uc758\ud639\uc744 \uc81c\uae30\ud588\ub2e4. \ud558\uc9c0\ub9cc \uc6d0\uc815\ud654\ub294 '\uc2e0\ub3d9\uc544 2014\ub144 4\uc6d4\ud638\u2019\uac00 \ubcf4\ub3c4\ud55c \u201c\ubcf4\uc704\ubd80\uc758 \u2018\ubcf4\u2019\uc790\ub3c4 \ubaa8\ub978\ub2e4. \ub098\ub294 \ubd81\ud55c \ubcf4\uc704\ubd80\uac00 \ub0a8\ud30c\ud55c \uac04\ucca9\uc774 \uc544\ub2c8\ub2e4\u201d\ub77c\ub294 \ucde8\uc9c0\uc758 \uc790\uc2e0\uc758 \ube44\uacf5\uac1c \uc99d\uc5b8\uc5d0 \ub300\ud574 \ubd80\uc778\ud558\uba70 \u201c\uc5b4\ub9b0 \ub098\uc774\ubd80\ud130 \ubcf4\uc704\ubd80 \uc694\uc6d0\uc5d0 \uc120\ubc1c\ub410\ub2e4\uace0 \ub9d0\ud558\uba74 \uc544\ubc84\uc9c0\uac00 \ub180\ub784\uae4c\ubd10 \uadf8\ub807\uac8c \uc124\uba85\ub4dc\ub9b0 \uac83\u201d\uc774\ub77c\uace0 \ud558\uba74\uc11c 2014\ub144 4\uc6d4 8\uc77c\uc5d0\ub294 \u201c\ub098\ub294 \uac04\ucca9\ud589\uc704\ub97c \uc778\uc815\ud558\uace0 \uc8d7\uac12\ub3c4 \uc815\ub2f9\ud558\uac8c \uce58\ub800\ub2e4. \ub0b4 \uc0ac\uac74\uc740 \uc218\uc0ac\uacfc\uc815\uc5d0\uc11c \ud68c\uc720\uc640 \ud611\ubc15\uc73c\ub85c \uc774\ub904\uc9c4 \uac83\uc774 \uc544\ub2c8\ub77c \uc624\ub79c \ub0b4\uc0ac\uc640 \ub9ce\uc740 \uc99d\uac70\ubb3c\uc744 \ub300\ud55c\ubbfc\uad6d \ubc95\uae30\uad00\uc5d0\uc11c \ucda9\ubd84\ud788 \uac80\ud1a0\ud558\uace0 \ud310\uacb0\uc744 \ub0b4\ub824 \ucc98\ubc8c\uc744 \ud55c \uac83\uc774\ub2e4. \ub300\ud55c\ubbfc\uad6d\uc5d0\uc11c \uc870\uc6a9\ud788 \uc0b4\ub824\ub294 \uc800\ub97c \ub354 \uc774\uc0c1 \uad34\ub86d\ud788\uc9c0 \ub9d0\uace0 \ub538\uacfc \ud3b8\uc548\ud788 \uc0b4 \uc218 \uc788\uac8c \ud574 \ub2ec\ub77c.\u201d\ub77c\ub294 \ub0b4\uc6a9\uc744 \ub2f4\uc740 \ubb38\uc11c\ub97c \uc5b8\ub860\uc0ac\uc5d0 \ubc30\ud3ec\ud588\ub2e4.", "answer": "\uae40\ub3d9\uc21c", "sentence": "\uc6d4\uac04\uc9c0\u2018\uc2e0\ub3d9\uc544\u2019(2014\ub144 4\uc6d4\ud638)\ub294 \"\uc6d0\uc815\ud654\uac00 2014\ub144 2\uc6d4 \ub9d0 \uc790\uc2e0\uc758 \uc758\ubd93\uc544\ubc84\uc9c0\uc778 \uae40\ub3d9\uc21c \uc744 \ub9cc\ub098 \ubd81\ud55c \ubcf4\uc704\ubd80 \uc9c1\ud30c\uac04\ucca9, \ud0c8\ubd81 \uc774\ud6c4 3\ucc28\ub840 \ubd81\ud55c \ubc29\ubb38 \ub4f1 \uc0ac\uac74 \ub2f9\uc2dc\ubd80\ud130 \uc790\uc2e0\uc774 \uc8fc\uc7a5\ud588\ub358 \uc8fc\uc694 \uac04\ucca9 \ud589\uc801\uc744 \ubd80\uc778\ud558\ub294 \ub300\ud654\ub97c \ud588\ub2e4.", "paragraph_sentence": " \uc6d4\uac04\uc9c0\u2018\uc2e0\ub3d9\uc544\u2019(2014\ub144 4\uc6d4\ud638)\ub294 \"\uc6d0\uc815\ud654\uac00 2014\ub144 2\uc6d4 \ub9d0 \uc790\uc2e0\uc758 \uc758\ubd93\uc544\ubc84\uc9c0\uc778 \uae40\ub3d9\uc21c \uc744 \ub9cc\ub098 \ubd81\ud55c \ubcf4\uc704\ubd80 \uc9c1\ud30c\uac04\ucca9, \ud0c8\ubd81 \uc774\ud6c4 3\ucc28\ub840 \ubd81\ud55c \ubc29\ubb38 \ub4f1 \uc0ac\uac74 \ub2f9\uc2dc\ubd80\ud130 \uc790\uc2e0\uc774 \uc8fc\uc7a5\ud588\ub358 \uc8fc\uc694 \uac04\ucca9 \ud589\uc801\uc744 \ubd80\uc778\ud558\ub294 \ub300\ud654\ub97c \ud588\ub2e4. \"\uace0 \ubcf4\ub3c4\ud588\ub2e4. \ubcf4\ub3c4 \uc9c1\ud6c4 \ubbfc\uc8fc\uc0ac\ud68c\ub97c \uc704\ud55c \ubcc0\ud638\uc0ac \ubaa8\uc784(\ubbfc\ubcc0)\uc740 \u2018\uc2e0\ub3d9\uc544 \ubcf4\ub3c4\uc5d0 \ub300\ud55c \uc785\uc7a5\u2019\uc774\ub780 \uc81c\ubaa9\uc758 \ubcf4\ub3c4\uc790\ub8cc\ub97c \ubc1c\ud45c\ud558\uace0 \uc9c4\uc0c1\uaddc\uba85\uc744 \uc694\uad6c\ud588\ub2e4. \ud55c\uaca8\ub808\ub294 2014\ub144 3\uc6d4 22\uc77c \u201c\uac80\ucc30\ub85c\ubd80\ud130 \ud5c8\uc704\uc9c4\uc220\uc744 \uac15\uc694\ub2f9\ud588\ub2e4. \ub098\ub294 \uac04\ucca9\uc774 \ub9de\uc9c0\ub9cc, \uc544\ubc84\uc9c0\ub294 \uc544\ub2c8\ub2e4\u201d\ub77c\ub294 \uc6d0\uc815\ud654 \uc778\ud130\ubdf0 \ub0b4\uc6a9\uc744 \ubcf4\ub3c4\ud558\uba74\uc11c \uc218\uc0ac \uc870\uc791 \uc758\ud639\uc744 \uc81c\uae30\ud588\ub2e4. \ud558\uc9c0\ub9cc \uc6d0\uc815\ud654\ub294 '\uc2e0\ub3d9\uc544 2014\ub144 4\uc6d4\ud638\u2019\uac00 \ubcf4\ub3c4\ud55c \u201c\ubcf4\uc704\ubd80\uc758 \u2018\ubcf4\u2019\uc790\ub3c4 \ubaa8\ub978\ub2e4. \ub098\ub294 \ubd81\ud55c \ubcf4\uc704\ubd80\uac00 \ub0a8\ud30c\ud55c \uac04\ucca9\uc774 \uc544\ub2c8\ub2e4\u201d\ub77c\ub294 \ucde8\uc9c0\uc758 \uc790\uc2e0\uc758 \ube44\uacf5\uac1c \uc99d\uc5b8\uc5d0 \ub300\ud574 \ubd80\uc778\ud558\uba70 \u201c\uc5b4\ub9b0 \ub098\uc774\ubd80\ud130 \ubcf4\uc704\ubd80 \uc694\uc6d0\uc5d0 \uc120\ubc1c\ub410\ub2e4\uace0 \ub9d0\ud558\uba74 \uc544\ubc84\uc9c0\uac00 \ub180\ub784\uae4c\ubd10 \uadf8\ub807\uac8c \uc124\uba85\ub4dc\ub9b0 \uac83\u201d\uc774\ub77c\uace0 \ud558\uba74\uc11c 2014\ub144 4\uc6d4 8\uc77c\uc5d0\ub294 \u201c\ub098\ub294 \uac04\ucca9\ud589\uc704\ub97c \uc778\uc815\ud558\uace0 \uc8d7\uac12\ub3c4 \uc815\ub2f9\ud558\uac8c \uce58\ub800\ub2e4. \ub0b4 \uc0ac\uac74\uc740 \uc218\uc0ac\uacfc\uc815\uc5d0\uc11c \ud68c\uc720\uc640 \ud611\ubc15\uc73c\ub85c \uc774\ub904\uc9c4 \uac83\uc774 \uc544\ub2c8\ub77c \uc624\ub79c \ub0b4\uc0ac\uc640 \ub9ce\uc740 \uc99d\uac70\ubb3c\uc744 \ub300\ud55c\ubbfc\uad6d \ubc95\uae30\uad00\uc5d0\uc11c \ucda9\ubd84\ud788 \uac80\ud1a0\ud558\uace0 \ud310\uacb0\uc744 \ub0b4\ub824 \ucc98\ubc8c\uc744 \ud55c \uac83\uc774\ub2e4. \ub300\ud55c\ubbfc\uad6d\uc5d0\uc11c \uc870\uc6a9\ud788 \uc0b4\ub824\ub294 \uc800\ub97c \ub354 \uc774\uc0c1 \uad34\ub86d\ud788\uc9c0 \ub9d0\uace0 \ub538\uacfc \ud3b8\uc548\ud788 \uc0b4 \uc218 \uc788\uac8c \ud574 \ub2ec\ub77c. \u201d\ub77c\ub294 \ub0b4\uc6a9\uc744 \ub2f4\uc740 \ubb38\uc11c\ub97c \uc5b8\ub860\uc0ac\uc5d0 \ubc30\ud3ec\ud588\ub2e4.", "paragraph_answer": "\uc6d4\uac04\uc9c0\u2018\uc2e0\ub3d9\uc544\u2019(2014\ub144 4\uc6d4\ud638)\ub294 \"\uc6d0\uc815\ud654\uac00 2014\ub144 2\uc6d4 \ub9d0 \uc790\uc2e0\uc758 \uc758\ubd93\uc544\ubc84\uc9c0\uc778 \uae40\ub3d9\uc21c \uc744 \ub9cc\ub098 \ubd81\ud55c \ubcf4\uc704\ubd80 \uc9c1\ud30c\uac04\ucca9, \ud0c8\ubd81 \uc774\ud6c4 3\ucc28\ub840 \ubd81\ud55c \ubc29\ubb38 \ub4f1 \uc0ac\uac74 \ub2f9\uc2dc\ubd80\ud130 \uc790\uc2e0\uc774 \uc8fc\uc7a5\ud588\ub358 \uc8fc\uc694 \uac04\ucca9 \ud589\uc801\uc744 \ubd80\uc778\ud558\ub294 \ub300\ud654\ub97c \ud588\ub2e4.\"\uace0 \ubcf4\ub3c4\ud588\ub2e4. \ubcf4\ub3c4 \uc9c1\ud6c4 \ubbfc\uc8fc\uc0ac\ud68c\ub97c \uc704\ud55c \ubcc0\ud638\uc0ac \ubaa8\uc784(\ubbfc\ubcc0)\uc740 \u2018\uc2e0\ub3d9\uc544 \ubcf4\ub3c4\uc5d0 \ub300\ud55c \uc785\uc7a5\u2019\uc774\ub780 \uc81c\ubaa9\uc758 \ubcf4\ub3c4\uc790\ub8cc\ub97c \ubc1c\ud45c\ud558\uace0 \uc9c4\uc0c1\uaddc\uba85\uc744 \uc694\uad6c\ud588\ub2e4. \ud55c\uaca8\ub808\ub294 2014\ub144 3\uc6d4 22\uc77c \u201c\uac80\ucc30\ub85c\ubd80\ud130 \ud5c8\uc704\uc9c4\uc220\uc744 \uac15\uc694\ub2f9\ud588\ub2e4. \ub098\ub294 \uac04\ucca9\uc774 \ub9de\uc9c0\ub9cc, \uc544\ubc84\uc9c0\ub294 \uc544\ub2c8\ub2e4\u201d\ub77c\ub294 \uc6d0\uc815\ud654 \uc778\ud130\ubdf0 \ub0b4\uc6a9\uc744 \ubcf4\ub3c4\ud558\uba74\uc11c \uc218\uc0ac \uc870\uc791 \uc758\ud639\uc744 \uc81c\uae30\ud588\ub2e4. \ud558\uc9c0\ub9cc \uc6d0\uc815\ud654\ub294 '\uc2e0\ub3d9\uc544 2014\ub144 4\uc6d4\ud638\u2019\uac00 \ubcf4\ub3c4\ud55c \u201c\ubcf4\uc704\ubd80\uc758 \u2018\ubcf4\u2019\uc790\ub3c4 \ubaa8\ub978\ub2e4. \ub098\ub294 \ubd81\ud55c \ubcf4\uc704\ubd80\uac00 \ub0a8\ud30c\ud55c \uac04\ucca9\uc774 \uc544\ub2c8\ub2e4\u201d\ub77c\ub294 \ucde8\uc9c0\uc758 \uc790\uc2e0\uc758 \ube44\uacf5\uac1c \uc99d\uc5b8\uc5d0 \ub300\ud574 \ubd80\uc778\ud558\uba70 \u201c\uc5b4\ub9b0 \ub098\uc774\ubd80\ud130 \ubcf4\uc704\ubd80 \uc694\uc6d0\uc5d0 \uc120\ubc1c\ub410\ub2e4\uace0 \ub9d0\ud558\uba74 \uc544\ubc84\uc9c0\uac00 \ub180\ub784\uae4c\ubd10 \uadf8\ub807\uac8c \uc124\uba85\ub4dc\ub9b0 \uac83\u201d\uc774\ub77c\uace0 \ud558\uba74\uc11c 2014\ub144 4\uc6d4 8\uc77c\uc5d0\ub294 \u201c\ub098\ub294 \uac04\ucca9\ud589\uc704\ub97c \uc778\uc815\ud558\uace0 \uc8d7\uac12\ub3c4 \uc815\ub2f9\ud558\uac8c \uce58\ub800\ub2e4. \ub0b4 \uc0ac\uac74\uc740 \uc218\uc0ac\uacfc\uc815\uc5d0\uc11c \ud68c\uc720\uc640 \ud611\ubc15\uc73c\ub85c \uc774\ub904\uc9c4 \uac83\uc774 \uc544\ub2c8\ub77c \uc624\ub79c \ub0b4\uc0ac\uc640 \ub9ce\uc740 \uc99d\uac70\ubb3c\uc744 \ub300\ud55c\ubbfc\uad6d \ubc95\uae30\uad00\uc5d0\uc11c \ucda9\ubd84\ud788 \uac80\ud1a0\ud558\uace0 \ud310\uacb0\uc744 \ub0b4\ub824 \ucc98\ubc8c\uc744 \ud55c \uac83\uc774\ub2e4. \ub300\ud55c\ubbfc\uad6d\uc5d0\uc11c \uc870\uc6a9\ud788 \uc0b4\ub824\ub294 \uc800\ub97c \ub354 \uc774\uc0c1 \uad34\ub86d\ud788\uc9c0 \ub9d0\uace0 \ub538\uacfc \ud3b8\uc548\ud788 \uc0b4 \uc218 \uc788\uac8c \ud574 \ub2ec\ub77c.\u201d\ub77c\ub294 \ub0b4\uc6a9\uc744 \ub2f4\uc740 \ubb38\uc11c\ub97c \uc5b8\ub860\uc0ac\uc5d0 \ubc30\ud3ec\ud588\ub2e4.", "sentence_answer": "\uc6d4\uac04\uc9c0\u2018\uc2e0\ub3d9\uc544\u2019(2014\ub144 4\uc6d4\ud638)\ub294 \"\uc6d0\uc815\ud654\uac00 2014\ub144 2\uc6d4 \ub9d0 \uc790\uc2e0\uc758 \uc758\ubd93\uc544\ubc84\uc9c0\uc778 \uae40\ub3d9\uc21c \uc744 \ub9cc\ub098 \ubd81\ud55c \ubcf4\uc704\ubd80 \uc9c1\ud30c\uac04\ucca9, \ud0c8\ubd81 \uc774\ud6c4 3\ucc28\ub840 \ubd81\ud55c \ubc29\ubb38 \ub4f1 \uc0ac\uac74 \ub2f9\uc2dc\ubd80\ud130 \uc790\uc2e0\uc774 \uc8fc\uc7a5\ud588\ub358 \uc8fc\uc694 \uac04\ucca9 \ud589\uc801\uc744 \ubd80\uc778\ud558\ub294 \ub300\ud654\ub97c \ud588\ub2e4.", "paragraph_id": "6605503-2-0", "paragraph_question": "question: \uc6d0\uc815\ud654\uc758 \uc758\ubd93\uc544\ubc84\uc9c0 \uc774\ub984\uc740 \ubb34\uc5c7\uc778\uac00?, context: \uc6d4\uac04\uc9c0\u2018\uc2e0\ub3d9\uc544\u2019(2014\ub144 4\uc6d4\ud638)\ub294 \"\uc6d0\uc815\ud654\uac00 2014\ub144 2\uc6d4 \ub9d0 \uc790\uc2e0\uc758 \uc758\ubd93\uc544\ubc84\uc9c0\uc778 \uae40\ub3d9\uc21c\uc744 \ub9cc\ub098 \ubd81\ud55c \ubcf4\uc704\ubd80 \uc9c1\ud30c\uac04\ucca9, \ud0c8\ubd81 \uc774\ud6c4 3\ucc28\ub840 \ubd81\ud55c \ubc29\ubb38 \ub4f1 \uc0ac\uac74 \ub2f9\uc2dc\ubd80\ud130 \uc790\uc2e0\uc774 \uc8fc\uc7a5\ud588\ub358 \uc8fc\uc694 \uac04\ucca9 \ud589\uc801\uc744 \ubd80\uc778\ud558\ub294 \ub300\ud654\ub97c \ud588\ub2e4.\"\uace0 \ubcf4\ub3c4\ud588\ub2e4. \ubcf4\ub3c4 \uc9c1\ud6c4 \ubbfc\uc8fc\uc0ac\ud68c\ub97c \uc704\ud55c \ubcc0\ud638\uc0ac \ubaa8\uc784(\ubbfc\ubcc0)\uc740 \u2018\uc2e0\ub3d9\uc544 \ubcf4\ub3c4\uc5d0 \ub300\ud55c \uc785\uc7a5\u2019\uc774\ub780 \uc81c\ubaa9\uc758 \ubcf4\ub3c4\uc790\ub8cc\ub97c \ubc1c\ud45c\ud558\uace0 \uc9c4\uc0c1\uaddc\uba85\uc744 \uc694\uad6c\ud588\ub2e4. \ud55c\uaca8\ub808\ub294 2014\ub144 3\uc6d4 22\uc77c \u201c\uac80\ucc30\ub85c\ubd80\ud130 \ud5c8\uc704\uc9c4\uc220\uc744 \uac15\uc694\ub2f9\ud588\ub2e4. \ub098\ub294 \uac04\ucca9\uc774 \ub9de\uc9c0\ub9cc, \uc544\ubc84\uc9c0\ub294 \uc544\ub2c8\ub2e4\u201d\ub77c\ub294 \uc6d0\uc815\ud654 \uc778\ud130\ubdf0 \ub0b4\uc6a9\uc744 \ubcf4\ub3c4\ud558\uba74\uc11c \uc218\uc0ac \uc870\uc791 \uc758\ud639\uc744 \uc81c\uae30\ud588\ub2e4. \ud558\uc9c0\ub9cc \uc6d0\uc815\ud654\ub294 '\uc2e0\ub3d9\uc544 2014\ub144 4\uc6d4\ud638\u2019\uac00 \ubcf4\ub3c4\ud55c \u201c\ubcf4\uc704\ubd80\uc758 \u2018\ubcf4\u2019\uc790\ub3c4 \ubaa8\ub978\ub2e4. \ub098\ub294 \ubd81\ud55c \ubcf4\uc704\ubd80\uac00 \ub0a8\ud30c\ud55c \uac04\ucca9\uc774 \uc544\ub2c8\ub2e4\u201d\ub77c\ub294 \ucde8\uc9c0\uc758 \uc790\uc2e0\uc758 \ube44\uacf5\uac1c \uc99d\uc5b8\uc5d0 \ub300\ud574 \ubd80\uc778\ud558\uba70 \u201c\uc5b4\ub9b0 \ub098\uc774\ubd80\ud130 \ubcf4\uc704\ubd80 \uc694\uc6d0\uc5d0 \uc120\ubc1c\ub410\ub2e4\uace0 \ub9d0\ud558\uba74 \uc544\ubc84\uc9c0\uac00 \ub180\ub784\uae4c\ubd10 \uadf8\ub807\uac8c \uc124\uba85\ub4dc\ub9b0 \uac83\u201d\uc774\ub77c\uace0 \ud558\uba74\uc11c 2014\ub144 4\uc6d4 8\uc77c\uc5d0\ub294 \u201c\ub098\ub294 \uac04\ucca9\ud589\uc704\ub97c \uc778\uc815\ud558\uace0 \uc8d7\uac12\ub3c4 \uc815\ub2f9\ud558\uac8c \uce58\ub800\ub2e4. \ub0b4 \uc0ac\uac74\uc740 \uc218\uc0ac\uacfc\uc815\uc5d0\uc11c \ud68c\uc720\uc640 \ud611\ubc15\uc73c\ub85c \uc774\ub904\uc9c4 \uac83\uc774 \uc544\ub2c8\ub77c \uc624\ub79c \ub0b4\uc0ac\uc640 \ub9ce\uc740 \uc99d\uac70\ubb3c\uc744 \ub300\ud55c\ubbfc\uad6d \ubc95\uae30\uad00\uc5d0\uc11c \ucda9\ubd84\ud788 \uac80\ud1a0\ud558\uace0 \ud310\uacb0\uc744 \ub0b4\ub824 \ucc98\ubc8c\uc744 \ud55c \uac83\uc774\ub2e4. \ub300\ud55c\ubbfc\uad6d\uc5d0\uc11c \uc870\uc6a9\ud788 \uc0b4\ub824\ub294 \uc800\ub97c \ub354 \uc774\uc0c1 \uad34\ub86d\ud788\uc9c0 \ub9d0\uace0 \ub538\uacfc \ud3b8\uc548\ud788 \uc0b4 \uc218 \uc788\uac8c \ud574 \ub2ec\ub77c.\u201d\ub77c\ub294 \ub0b4\uc6a9\uc744 \ub2f4\uc740 \ubb38\uc11c\ub97c \uc5b8\ub860\uc0ac\uc5d0 \ubc30\ud3ec\ud588\ub2e4."} {"question": "\uc7a5\uc790\uc5f0 \ubb38\uac74\uc744 \ud655\ubcf4\ud55c \ubc29\uc1a1\uad6d\uc740?", "paragraph": "\uc7a5\uc790\uc5f0 \ubb38\uac74\uc740 KBS \ubc29\uc1a1\uad6d\uc5d0 \uc758\ud574 \ud655\ubcf4\ub418\uc5c8\ub2e4. \uc7a5\uc790\uc5f0 \ub4f1\uc5d0\uac8c \uc131\uc0c1\ub0a9\uacfc \uc220\uc811\ub300\ub97c \uac15\uc694\ud588\ub358 \uacf3\uc73c\ub85c \uc9c0\ubaa9\ub41c \uc11c\uc6b8 \uc0bc\uc131\ub3d9 \ud55c \uac74\ubb3c\uc758 CCTV\uac00 \uacbd\ucc30 \ub4f1 \uc218\uc0ac\uae30\uad00\uc5d0 \ud655\ubcf4\ub418\uba74\uc11c CCTV 4\u223c5\ub300\ub97c \ubd84\uc11d\uc911\uc5d0 \uc788\uc73c\uba70 1\uce35\uc774 \uc640\uc778\ubc14, 2\uce35\uc774 \uc0ac\ubb34\uc2e4, 3\uce35\uc774 \uce68\uc2e4\uc6a9\ub3c4\ub85c \uc4f0\uc5ec \uc774\ub978\ubc14 \uc6d0\uc2a4\ud1b1 \ub85c\ube44\uc758 \uc7a5\uc18c\ub85c \uc758\uc2ec\ubc1b\uace0 \uc788\ub2e4. \uacbd\ucc30\uc740 \u201cCCTV\uc758 \ub179\ud654\uae30\ub85d \uc800\uc7a5\uae30\ud55c\uc774 \ud1b5\uc0c1 1\uac1c\uc6d4\uc774\uace0 \uac74\ubb3c 2\uce35 CCTV\ub294 \ub179\ud654\uac00 \uc548\ub418\ub294 \uac78\ub85c \ud604\uc7ac \ud655\uc778\ub418\uace0 \uc788\ub2e4\u201d\uba74\uc11c\ub3c4 \u201cCCTV \ubcf5\uc6d0 \ubc0f \ubd84\uc11d\uc5d0 \uc8fc\ub825\ud558\uace0 \uc788\ub2e4\u201d\uace0 \ubc1c\ud45c\ud558\uc600\ub2e4. \uacbd\ucc30\uc740 \uc55e\uc11c \uc7a5\uc790\uc5f0 \uc804 \ub9e4\ub2c8\uc800 \uc720\ubaa8(30)\uc528\uc758 \uae30\ud68d\uc0ac \uc0ac\ubb34\uc2e4 \ube4c\ub529\uc5d0 \uc124\uce58\ub41c CCTV \uc5d0 \uc7a5\uc790\uc5f0\uc528\uac00 \uc9c0\ub09c\ub2ec 28\uc77c \uc624\ud6c4 5\uc2dc34\ubd84\uc5d0 \uac74\ubb3c\ub85c \uc62c\ub77c\uac00 \uc624\ud6c4 10\uc2dc\uc5d0 \ub0b4\ub824\uc628 \uac83\uc774 \ud655\uc778\ub410\ub2e4\uba70 \uc720 \uc528\uac00 \uc9c4\uc220\ud55c \uc7a5 \uc528\uc758 \ubb38\uc11c\uc791\uc131 \uc2dc\uc810\uacfc \uc77c\uce58\ud55c\ub2e4\uace0 \ubc1d\ud614\ub2e4. \uacbd\ucc30\uc740 KBS\uc758 \uc7a5\uc790\uc5f0 \uad00\ub828 \ubb38\uac74\uc785\uc218 \uacbd\uc704\uc640 \uad00\ub828\ud574 KBS \uad00\uacc4\uc790\uac00 \uc4f0\ub808\uae30\ubd09\ud22c\ub97c \ub4e4\uace0 \uc720 \uc528 \uae30\ud68d\uc0ac \uc0ac\ubb34\uc2e4 \ube4c\ub529\uc758 \uacc4\ub2e8\uc744 \ub0b4\ub824\uc624\ub294 \ubaa8\uc2b5\uc774 CCTV\uc5d0 \ub179\ud654\ub410\ub2e4\uba70 KBS\uc758 \ubcf4\ub3c4\ub0b4\uc6a9\uc774 \uc0ac\uc2e4\uc784\uc744 \uc785\uc99d\ud588\ub2e4. KBS\uc640 \ud568\uaed8 \ubb38\uac74\uc744 \uacf5\uac1c\ud55c \ub178\ucef7\ub274\uc2a4\uc5d0 \ub300\ud574\uc11c\ub3c4 \uacbd\ucc30\uc740 \ub2e4\ub978 \uc5b8\ub860\uc0ac \uc18c\uc18d\uc758 \uae30\uc790\uac00 \ubb38\uac74\uc744 \ud568\uaed8 \ubd10 \uc2e0\ub8b0\uc131\uc774 \uc788\ub2e4\uace0 \uc5b8\ub860\uc5d0 \ubc1c\ud45c\ud558\uc600\uc73c\uba70, \uc720 \uc528 \uae30\ud68d\uc0ac \uc0ac\ubb34\uc2e4\uc5d0\uc11c \ubb38\uac74\uc744 \ubcf8 \uac83\uc73c\ub85c \uc54c\ub824\uc838 CCTV\ub85c \uc774 \uc0ac\uc2e4\uc744 \ud655\uc778\ud560 \uc218 \uc788\uc5c8\ub358 \uac83\uc73c\ub85c \ucd94\uc815\ub418\uace0 \uc788\ub2e4. 3\uc6d4 24\uc77c \uc624\ud6c4 2\uc2dc 30\ubd84 \uacfc\ud559\uc218\uc0ac\ub300(CSI)\ub294 \uc811\ub300 \uc7a5\uc18c\ub85c \uc54c\ub824\uc9c4 \uc11c\uc6b8\ud2b9\ubcc4\uc2dc \uc0bc\uc131\ub3d9 40-9\ubc88\uc9c0\uc5d0 \uc704\uce58\ud55c \uc61b \uc0ac\ubb34\uc2e4 \uac74\ubb3c\uc5d0 \ub300\ud574 \uc815\ubc00\uc870\uc0ac\uc640 \uac10\uc2dd\uc744 \ud558\uc600\uc73c\uba70, \uac74\ubb3c\uc5d0\uc11c \ub2e4\ub7c9\uc758 \uc9c0\ubb38\uc744 \ucc44\ucde8\ud588\uace0, \uba38\ub9ac\uce74\ub77d\uc744 \uc218\uac70\ud558\uc600\ub2e4. \uac74\ubb3c\uc758 3\uce35\uc5d0\uc11c\ub294 \ubc00\uc2e4\uc744 \ud655\uc778\ud558\uace0 \ubc00\uc2e4 \ub0b4\ubd80\uc758 \uc695\uc2e4\uc5d0\uc11c \uba38\ub9ac\uce74\ub77d \ub4f1\uc744 \ubc1c\uacac\ud574 \uc218\uac70\ud588\uace0 \uc9c0\ubb38\uc744 \ucc44\ucde8\ud558\uace0, \uc871\uc801\uc744 \ucc44\ucde8\ud558\uc600\ub2e4. \uacbd\ucc30\uc774 \uc218\uac70\ud55c \uc790\ub8cc\uc5d0\ub294 \uc720\uc804\uc790 \uc870\uac01\uc774 \ub0a8\uc544\uc788\uc744 \uce68\uc2e4 \ubc0f \uac70\uc2e4 \ub3c4\uad6c \ub4f1\ub3c4 \ucc44\ucde8\ub418\uc5c8\ub2e4. \uacbd\ucc30\uc740 201\uc810 \ubb3c\ud488\uc5d0 \uc74c\uc545 CD\ub97c \uac01\uac01 1\uac1c\uc529 \uc13c \uac83\ub3c4 \ud3ec\ud568\ub41c\ub2e4. \uc2e4\uc81c \uc911\uc694\ud55c \ubb3c\ud488\uc740 20\uc810 \uc815\ub3c4\ub85c \ucd94\uc815\ub41c\ub2e4\ub294 \uc758\uacac\uc744 \ub367\ubd99\uc774\uae30\ub3c4 \ud588\ub2e4. 3\uc6d4 24\uc77c\uc5d0\ub294 2008\ub144 5\uc6d4 \ud0dc\uad6d \uace8\ud504\uc678\uc720\uc5d0 \uad00\ub828\ub41c \uc0ac\uc9c4 \uc790\ub8cc\uac00 \uacbd\ucc30\uc5d0 \uc785\uc218\ub418\uae30\ub3c4 \ud558\uc600\ub2e4. 25\uc77c \uc0bc\uc131\ub3d9 \uc0ac\ubb34\uc2e4\uc5d0\uc11c DNA \uc720\uc804\uc790 \uc790\ub8cc \ub4f1 \uc57d 96\uc810\uc744 \ud655\ubcf4\ub2e4\uace0 \ubc1d\ud788\uae30\ub3c4 \ud588\ub2e4.", "answer": "KBS", "sentence": "\uc7a5\uc790\uc5f0 \ubb38\uac74\uc740 KBS \ubc29\uc1a1\uad6d\uc5d0 \uc758\ud574 \ud655\ubcf4\ub418\uc5c8\ub2e4.", "paragraph_sentence": " \uc7a5\uc790\uc5f0 \ubb38\uac74\uc740 KBS \ubc29\uc1a1\uad6d\uc5d0 \uc758\ud574 \ud655\ubcf4\ub418\uc5c8\ub2e4. \uc7a5\uc790\uc5f0 \ub4f1\uc5d0\uac8c \uc131\uc0c1\ub0a9\uacfc \uc220\uc811\ub300\ub97c \uac15\uc694\ud588\ub358 \uacf3\uc73c\ub85c \uc9c0\ubaa9\ub41c \uc11c\uc6b8 \uc0bc\uc131\ub3d9 \ud55c \uac74\ubb3c\uc758 CCTV\uac00 \uacbd\ucc30 \ub4f1 \uc218\uc0ac\uae30\uad00\uc5d0 \ud655\ubcf4\ub418\uba74\uc11c CCTV 4\u223c5\ub300\ub97c \ubd84\uc11d\uc911\uc5d0 \uc788\uc73c\uba70 1\uce35\uc774 \uc640\uc778\ubc14, 2\uce35\uc774 \uc0ac\ubb34\uc2e4, 3\uce35\uc774 \uce68\uc2e4\uc6a9\ub3c4\ub85c \uc4f0\uc5ec \uc774\ub978\ubc14 \uc6d0\uc2a4\ud1b1 \ub85c\ube44\uc758 \uc7a5\uc18c\ub85c \uc758\uc2ec\ubc1b\uace0 \uc788\ub2e4. \uacbd\ucc30\uc740 \u201cCCTV\uc758 \ub179\ud654\uae30\ub85d \uc800\uc7a5\uae30\ud55c\uc774 \ud1b5\uc0c1 1\uac1c\uc6d4\uc774\uace0 \uac74\ubb3c 2\uce35 CCTV\ub294 \ub179\ud654\uac00 \uc548\ub418\ub294 \uac78\ub85c \ud604\uc7ac \ud655\uc778\ub418\uace0 \uc788\ub2e4\u201d\uba74\uc11c\ub3c4 \u201cCCTV \ubcf5\uc6d0 \ubc0f \ubd84\uc11d\uc5d0 \uc8fc\ub825\ud558\uace0 \uc788\ub2e4\u201d\uace0 \ubc1c\ud45c\ud558\uc600\ub2e4. \uacbd\ucc30\uc740 \uc55e\uc11c \uc7a5\uc790\uc5f0 \uc804 \ub9e4\ub2c8\uc800 \uc720\ubaa8(30)\uc528\uc758 \uae30\ud68d\uc0ac \uc0ac\ubb34\uc2e4 \ube4c\ub529\uc5d0 \uc124\uce58\ub41c CCTV \uc5d0 \uc7a5\uc790\uc5f0\uc528\uac00 \uc9c0\ub09c\ub2ec 28\uc77c \uc624\ud6c4 5\uc2dc34\ubd84\uc5d0 \uac74\ubb3c\ub85c \uc62c\ub77c\uac00 \uc624\ud6c4 10\uc2dc\uc5d0 \ub0b4\ub824\uc628 \uac83\uc774 \ud655\uc778\ub410\ub2e4\uba70 \uc720 \uc528\uac00 \uc9c4\uc220\ud55c \uc7a5 \uc528\uc758 \ubb38\uc11c\uc791\uc131 \uc2dc\uc810\uacfc \uc77c\uce58\ud55c\ub2e4\uace0 \ubc1d\ud614\ub2e4. \uacbd\ucc30\uc740 KBS\uc758 \uc7a5\uc790\uc5f0 \uad00\ub828 \ubb38\uac74\uc785\uc218 \uacbd\uc704\uc640 \uad00\ub828\ud574 KBS \uad00\uacc4\uc790\uac00 \uc4f0\ub808\uae30\ubd09\ud22c\ub97c \ub4e4\uace0 \uc720 \uc528 \uae30\ud68d\uc0ac \uc0ac\ubb34\uc2e4 \ube4c\ub529\uc758 \uacc4\ub2e8\uc744 \ub0b4\ub824\uc624\ub294 \ubaa8\uc2b5\uc774 CCTV\uc5d0 \ub179\ud654\ub410\ub2e4\uba70 KBS\uc758 \ubcf4\ub3c4\ub0b4\uc6a9\uc774 \uc0ac\uc2e4\uc784\uc744 \uc785\uc99d\ud588\ub2e4. KBS\uc640 \ud568\uaed8 \ubb38\uac74\uc744 \uacf5\uac1c\ud55c \ub178\ucef7\ub274\uc2a4\uc5d0 \ub300\ud574\uc11c\ub3c4 \uacbd\ucc30\uc740 \ub2e4\ub978 \uc5b8\ub860\uc0ac \uc18c\uc18d\uc758 \uae30\uc790\uac00 \ubb38\uac74\uc744 \ud568\uaed8 \ubd10 \uc2e0\ub8b0\uc131\uc774 \uc788\ub2e4\uace0 \uc5b8\ub860\uc5d0 \ubc1c\ud45c\ud558\uc600\uc73c\uba70, \uc720 \uc528 \uae30\ud68d\uc0ac \uc0ac\ubb34\uc2e4\uc5d0\uc11c \ubb38\uac74\uc744 \ubcf8 \uac83\uc73c\ub85c \uc54c\ub824\uc838 CCTV\ub85c \uc774 \uc0ac\uc2e4\uc744 \ud655\uc778\ud560 \uc218 \uc788\uc5c8\ub358 \uac83\uc73c\ub85c \ucd94\uc815\ub418\uace0 \uc788\ub2e4. 3\uc6d4 24\uc77c \uc624\ud6c4 2\uc2dc 30\ubd84 \uacfc\ud559\uc218\uc0ac\ub300(CSI)\ub294 \uc811\ub300 \uc7a5\uc18c\ub85c \uc54c\ub824\uc9c4 \uc11c\uc6b8\ud2b9\ubcc4\uc2dc \uc0bc\uc131\ub3d9 40-9\ubc88\uc9c0\uc5d0 \uc704\uce58\ud55c \uc61b \uc0ac\ubb34\uc2e4 \uac74\ubb3c\uc5d0 \ub300\ud574 \uc815\ubc00\uc870\uc0ac\uc640 \uac10\uc2dd\uc744 \ud558\uc600\uc73c\uba70, \uac74\ubb3c\uc5d0\uc11c \ub2e4\ub7c9\uc758 \uc9c0\ubb38\uc744 \ucc44\ucde8\ud588\uace0, \uba38\ub9ac\uce74\ub77d\uc744 \uc218\uac70\ud558\uc600\ub2e4. \uac74\ubb3c\uc758 3\uce35\uc5d0\uc11c\ub294 \ubc00\uc2e4\uc744 \ud655\uc778\ud558\uace0 \ubc00\uc2e4 \ub0b4\ubd80\uc758 \uc695\uc2e4\uc5d0\uc11c \uba38\ub9ac\uce74\ub77d \ub4f1\uc744 \ubc1c\uacac\ud574 \uc218\uac70\ud588\uace0 \uc9c0\ubb38\uc744 \ucc44\ucde8\ud558\uace0, \uc871\uc801\uc744 \ucc44\ucde8\ud558\uc600\ub2e4. \uacbd\ucc30\uc774 \uc218\uac70\ud55c \uc790\ub8cc\uc5d0\ub294 \uc720\uc804\uc790 \uc870\uac01\uc774 \ub0a8\uc544\uc788\uc744 \uce68\uc2e4 \ubc0f \uac70\uc2e4 \ub3c4\uad6c \ub4f1\ub3c4 \ucc44\ucde8\ub418\uc5c8\ub2e4. \uacbd\ucc30\uc740 201\uc810 \ubb3c\ud488\uc5d0 \uc74c\uc545 CD\ub97c \uac01\uac01 1\uac1c\uc529 \uc13c \uac83\ub3c4 \ud3ec\ud568\ub41c\ub2e4. \uc2e4\uc81c \uc911\uc694\ud55c \ubb3c\ud488\uc740 20\uc810 \uc815\ub3c4\ub85c \ucd94\uc815\ub41c\ub2e4\ub294 \uc758\uacac\uc744 \ub367\ubd99\uc774\uae30\ub3c4 \ud588\ub2e4. 3\uc6d4 24\uc77c\uc5d0\ub294 2008\ub144 5\uc6d4 \ud0dc\uad6d \uace8\ud504\uc678\uc720\uc5d0 \uad00\ub828\ub41c \uc0ac\uc9c4 \uc790\ub8cc\uac00 \uacbd\ucc30\uc5d0 \uc785\uc218\ub418\uae30\ub3c4 \ud558\uc600\ub2e4. 25\uc77c \uc0bc\uc131\ub3d9 \uc0ac\ubb34\uc2e4\uc5d0\uc11c DNA \uc720\uc804\uc790 \uc790\ub8cc \ub4f1 \uc57d 96\uc810\uc744 \ud655\ubcf4\ub2e4\uace0 \ubc1d\ud788\uae30\ub3c4 \ud588\ub2e4.", "paragraph_answer": "\uc7a5\uc790\uc5f0 \ubb38\uac74\uc740 KBS \ubc29\uc1a1\uad6d\uc5d0 \uc758\ud574 \ud655\ubcf4\ub418\uc5c8\ub2e4. \uc7a5\uc790\uc5f0 \ub4f1\uc5d0\uac8c \uc131\uc0c1\ub0a9\uacfc \uc220\uc811\ub300\ub97c \uac15\uc694\ud588\ub358 \uacf3\uc73c\ub85c \uc9c0\ubaa9\ub41c \uc11c\uc6b8 \uc0bc\uc131\ub3d9 \ud55c \uac74\ubb3c\uc758 CCTV\uac00 \uacbd\ucc30 \ub4f1 \uc218\uc0ac\uae30\uad00\uc5d0 \ud655\ubcf4\ub418\uba74\uc11c CCTV 4\u223c5\ub300\ub97c \ubd84\uc11d\uc911\uc5d0 \uc788\uc73c\uba70 1\uce35\uc774 \uc640\uc778\ubc14, 2\uce35\uc774 \uc0ac\ubb34\uc2e4, 3\uce35\uc774 \uce68\uc2e4\uc6a9\ub3c4\ub85c \uc4f0\uc5ec \uc774\ub978\ubc14 \uc6d0\uc2a4\ud1b1 \ub85c\ube44\uc758 \uc7a5\uc18c\ub85c \uc758\uc2ec\ubc1b\uace0 \uc788\ub2e4. \uacbd\ucc30\uc740 \u201cCCTV\uc758 \ub179\ud654\uae30\ub85d \uc800\uc7a5\uae30\ud55c\uc774 \ud1b5\uc0c1 1\uac1c\uc6d4\uc774\uace0 \uac74\ubb3c 2\uce35 CCTV\ub294 \ub179\ud654\uac00 \uc548\ub418\ub294 \uac78\ub85c \ud604\uc7ac \ud655\uc778\ub418\uace0 \uc788\ub2e4\u201d\uba74\uc11c\ub3c4 \u201cCCTV \ubcf5\uc6d0 \ubc0f \ubd84\uc11d\uc5d0 \uc8fc\ub825\ud558\uace0 \uc788\ub2e4\u201d\uace0 \ubc1c\ud45c\ud558\uc600\ub2e4. \uacbd\ucc30\uc740 \uc55e\uc11c \uc7a5\uc790\uc5f0 \uc804 \ub9e4\ub2c8\uc800 \uc720\ubaa8(30)\uc528\uc758 \uae30\ud68d\uc0ac \uc0ac\ubb34\uc2e4 \ube4c\ub529\uc5d0 \uc124\uce58\ub41c CCTV \uc5d0 \uc7a5\uc790\uc5f0\uc528\uac00 \uc9c0\ub09c\ub2ec 28\uc77c \uc624\ud6c4 5\uc2dc34\ubd84\uc5d0 \uac74\ubb3c\ub85c \uc62c\ub77c\uac00 \uc624\ud6c4 10\uc2dc\uc5d0 \ub0b4\ub824\uc628 \uac83\uc774 \ud655\uc778\ub410\ub2e4\uba70 \uc720 \uc528\uac00 \uc9c4\uc220\ud55c \uc7a5 \uc528\uc758 \ubb38\uc11c\uc791\uc131 \uc2dc\uc810\uacfc \uc77c\uce58\ud55c\ub2e4\uace0 \ubc1d\ud614\ub2e4. \uacbd\ucc30\uc740 KBS\uc758 \uc7a5\uc790\uc5f0 \uad00\ub828 \ubb38\uac74\uc785\uc218 \uacbd\uc704\uc640 \uad00\ub828\ud574 KBS \uad00\uacc4\uc790\uac00 \uc4f0\ub808\uae30\ubd09\ud22c\ub97c \ub4e4\uace0 \uc720 \uc528 \uae30\ud68d\uc0ac \uc0ac\ubb34\uc2e4 \ube4c\ub529\uc758 \uacc4\ub2e8\uc744 \ub0b4\ub824\uc624\ub294 \ubaa8\uc2b5\uc774 CCTV\uc5d0 \ub179\ud654\ub410\ub2e4\uba70 KBS\uc758 \ubcf4\ub3c4\ub0b4\uc6a9\uc774 \uc0ac\uc2e4\uc784\uc744 \uc785\uc99d\ud588\ub2e4. KBS\uc640 \ud568\uaed8 \ubb38\uac74\uc744 \uacf5\uac1c\ud55c \ub178\ucef7\ub274\uc2a4\uc5d0 \ub300\ud574\uc11c\ub3c4 \uacbd\ucc30\uc740 \ub2e4\ub978 \uc5b8\ub860\uc0ac \uc18c\uc18d\uc758 \uae30\uc790\uac00 \ubb38\uac74\uc744 \ud568\uaed8 \ubd10 \uc2e0\ub8b0\uc131\uc774 \uc788\ub2e4\uace0 \uc5b8\ub860\uc5d0 \ubc1c\ud45c\ud558\uc600\uc73c\uba70, \uc720 \uc528 \uae30\ud68d\uc0ac \uc0ac\ubb34\uc2e4\uc5d0\uc11c \ubb38\uac74\uc744 \ubcf8 \uac83\uc73c\ub85c \uc54c\ub824\uc838 CCTV\ub85c \uc774 \uc0ac\uc2e4\uc744 \ud655\uc778\ud560 \uc218 \uc788\uc5c8\ub358 \uac83\uc73c\ub85c \ucd94\uc815\ub418\uace0 \uc788\ub2e4. 3\uc6d4 24\uc77c \uc624\ud6c4 2\uc2dc 30\ubd84 \uacfc\ud559\uc218\uc0ac\ub300(CSI)\ub294 \uc811\ub300 \uc7a5\uc18c\ub85c \uc54c\ub824\uc9c4 \uc11c\uc6b8\ud2b9\ubcc4\uc2dc \uc0bc\uc131\ub3d9 40-9\ubc88\uc9c0\uc5d0 \uc704\uce58\ud55c \uc61b \uc0ac\ubb34\uc2e4 \uac74\ubb3c\uc5d0 \ub300\ud574 \uc815\ubc00\uc870\uc0ac\uc640 \uac10\uc2dd\uc744 \ud558\uc600\uc73c\uba70, \uac74\ubb3c\uc5d0\uc11c \ub2e4\ub7c9\uc758 \uc9c0\ubb38\uc744 \ucc44\ucde8\ud588\uace0, \uba38\ub9ac\uce74\ub77d\uc744 \uc218\uac70\ud558\uc600\ub2e4. \uac74\ubb3c\uc758 3\uce35\uc5d0\uc11c\ub294 \ubc00\uc2e4\uc744 \ud655\uc778\ud558\uace0 \ubc00\uc2e4 \ub0b4\ubd80\uc758 \uc695\uc2e4\uc5d0\uc11c \uba38\ub9ac\uce74\ub77d \ub4f1\uc744 \ubc1c\uacac\ud574 \uc218\uac70\ud588\uace0 \uc9c0\ubb38\uc744 \ucc44\ucde8\ud558\uace0, \uc871\uc801\uc744 \ucc44\ucde8\ud558\uc600\ub2e4. \uacbd\ucc30\uc774 \uc218\uac70\ud55c \uc790\ub8cc\uc5d0\ub294 \uc720\uc804\uc790 \uc870\uac01\uc774 \ub0a8\uc544\uc788\uc744 \uce68\uc2e4 \ubc0f \uac70\uc2e4 \ub3c4\uad6c \ub4f1\ub3c4 \ucc44\ucde8\ub418\uc5c8\ub2e4. \uacbd\ucc30\uc740 201\uc810 \ubb3c\ud488\uc5d0 \uc74c\uc545 CD\ub97c \uac01\uac01 1\uac1c\uc529 \uc13c \uac83\ub3c4 \ud3ec\ud568\ub41c\ub2e4. \uc2e4\uc81c \uc911\uc694\ud55c \ubb3c\ud488\uc740 20\uc810 \uc815\ub3c4\ub85c \ucd94\uc815\ub41c\ub2e4\ub294 \uc758\uacac\uc744 \ub367\ubd99\uc774\uae30\ub3c4 \ud588\ub2e4. 3\uc6d4 24\uc77c\uc5d0\ub294 2008\ub144 5\uc6d4 \ud0dc\uad6d \uace8\ud504\uc678\uc720\uc5d0 \uad00\ub828\ub41c \uc0ac\uc9c4 \uc790\ub8cc\uac00 \uacbd\ucc30\uc5d0 \uc785\uc218\ub418\uae30\ub3c4 \ud558\uc600\ub2e4. 25\uc77c \uc0bc\uc131\ub3d9 \uc0ac\ubb34\uc2e4\uc5d0\uc11c DNA \uc720\uc804\uc790 \uc790\ub8cc \ub4f1 \uc57d 96\uc810\uc744 \ud655\ubcf4\ub2e4\uace0 \ubc1d\ud788\uae30\ub3c4 \ud588\ub2e4.", "sentence_answer": "\uc7a5\uc790\uc5f0 \ubb38\uac74\uc740 KBS \ubc29\uc1a1\uad6d\uc5d0 \uc758\ud574 \ud655\ubcf4\ub418\uc5c8\ub2e4.", "paragraph_id": "6549120-3-0", "paragraph_question": "question: \uc7a5\uc790\uc5f0 \ubb38\uac74\uc744 \ud655\ubcf4\ud55c \ubc29\uc1a1\uad6d\uc740?, context: \uc7a5\uc790\uc5f0 \ubb38\uac74\uc740 KBS \ubc29\uc1a1\uad6d\uc5d0 \uc758\ud574 \ud655\ubcf4\ub418\uc5c8\ub2e4. \uc7a5\uc790\uc5f0 \ub4f1\uc5d0\uac8c \uc131\uc0c1\ub0a9\uacfc \uc220\uc811\ub300\ub97c \uac15\uc694\ud588\ub358 \uacf3\uc73c\ub85c \uc9c0\ubaa9\ub41c \uc11c\uc6b8 \uc0bc\uc131\ub3d9 \ud55c \uac74\ubb3c\uc758 CCTV\uac00 \uacbd\ucc30 \ub4f1 \uc218\uc0ac\uae30\uad00\uc5d0 \ud655\ubcf4\ub418\uba74\uc11c CCTV 4\u223c5\ub300\ub97c \ubd84\uc11d\uc911\uc5d0 \uc788\uc73c\uba70 1\uce35\uc774 \uc640\uc778\ubc14, 2\uce35\uc774 \uc0ac\ubb34\uc2e4, 3\uce35\uc774 \uce68\uc2e4\uc6a9\ub3c4\ub85c \uc4f0\uc5ec \uc774\ub978\ubc14 \uc6d0\uc2a4\ud1b1 \ub85c\ube44\uc758 \uc7a5\uc18c\ub85c \uc758\uc2ec\ubc1b\uace0 \uc788\ub2e4. \uacbd\ucc30\uc740 \u201cCCTV\uc758 \ub179\ud654\uae30\ub85d \uc800\uc7a5\uae30\ud55c\uc774 \ud1b5\uc0c1 1\uac1c\uc6d4\uc774\uace0 \uac74\ubb3c 2\uce35 CCTV\ub294 \ub179\ud654\uac00 \uc548\ub418\ub294 \uac78\ub85c \ud604\uc7ac \ud655\uc778\ub418\uace0 \uc788\ub2e4\u201d\uba74\uc11c\ub3c4 \u201cCCTV \ubcf5\uc6d0 \ubc0f \ubd84\uc11d\uc5d0 \uc8fc\ub825\ud558\uace0 \uc788\ub2e4\u201d\uace0 \ubc1c\ud45c\ud558\uc600\ub2e4. \uacbd\ucc30\uc740 \uc55e\uc11c \uc7a5\uc790\uc5f0 \uc804 \ub9e4\ub2c8\uc800 \uc720\ubaa8(30)\uc528\uc758 \uae30\ud68d\uc0ac \uc0ac\ubb34\uc2e4 \ube4c\ub529\uc5d0 \uc124\uce58\ub41c CCTV \uc5d0 \uc7a5\uc790\uc5f0\uc528\uac00 \uc9c0\ub09c\ub2ec 28\uc77c \uc624\ud6c4 5\uc2dc34\ubd84\uc5d0 \uac74\ubb3c\ub85c \uc62c\ub77c\uac00 \uc624\ud6c4 10\uc2dc\uc5d0 \ub0b4\ub824\uc628 \uac83\uc774 \ud655\uc778\ub410\ub2e4\uba70 \uc720 \uc528\uac00 \uc9c4\uc220\ud55c \uc7a5 \uc528\uc758 \ubb38\uc11c\uc791\uc131 \uc2dc\uc810\uacfc \uc77c\uce58\ud55c\ub2e4\uace0 \ubc1d\ud614\ub2e4. \uacbd\ucc30\uc740 KBS\uc758 \uc7a5\uc790\uc5f0 \uad00\ub828 \ubb38\uac74\uc785\uc218 \uacbd\uc704\uc640 \uad00\ub828\ud574 KBS \uad00\uacc4\uc790\uac00 \uc4f0\ub808\uae30\ubd09\ud22c\ub97c \ub4e4\uace0 \uc720 \uc528 \uae30\ud68d\uc0ac \uc0ac\ubb34\uc2e4 \ube4c\ub529\uc758 \uacc4\ub2e8\uc744 \ub0b4\ub824\uc624\ub294 \ubaa8\uc2b5\uc774 CCTV\uc5d0 \ub179\ud654\ub410\ub2e4\uba70 KBS\uc758 \ubcf4\ub3c4\ub0b4\uc6a9\uc774 \uc0ac\uc2e4\uc784\uc744 \uc785\uc99d\ud588\ub2e4. KBS\uc640 \ud568\uaed8 \ubb38\uac74\uc744 \uacf5\uac1c\ud55c \ub178\ucef7\ub274\uc2a4\uc5d0 \ub300\ud574\uc11c\ub3c4 \uacbd\ucc30\uc740 \ub2e4\ub978 \uc5b8\ub860\uc0ac \uc18c\uc18d\uc758 \uae30\uc790\uac00 \ubb38\uac74\uc744 \ud568\uaed8 \ubd10 \uc2e0\ub8b0\uc131\uc774 \uc788\ub2e4\uace0 \uc5b8\ub860\uc5d0 \ubc1c\ud45c\ud558\uc600\uc73c\uba70, \uc720 \uc528 \uae30\ud68d\uc0ac \uc0ac\ubb34\uc2e4\uc5d0\uc11c \ubb38\uac74\uc744 \ubcf8 \uac83\uc73c\ub85c \uc54c\ub824\uc838 CCTV\ub85c \uc774 \uc0ac\uc2e4\uc744 \ud655\uc778\ud560 \uc218 \uc788\uc5c8\ub358 \uac83\uc73c\ub85c \ucd94\uc815\ub418\uace0 \uc788\ub2e4. 3\uc6d4 24\uc77c \uc624\ud6c4 2\uc2dc 30\ubd84 \uacfc\ud559\uc218\uc0ac\ub300(CSI)\ub294 \uc811\ub300 \uc7a5\uc18c\ub85c \uc54c\ub824\uc9c4 \uc11c\uc6b8\ud2b9\ubcc4\uc2dc \uc0bc\uc131\ub3d9 40-9\ubc88\uc9c0\uc5d0 \uc704\uce58\ud55c \uc61b \uc0ac\ubb34\uc2e4 \uac74\ubb3c\uc5d0 \ub300\ud574 \uc815\ubc00\uc870\uc0ac\uc640 \uac10\uc2dd\uc744 \ud558\uc600\uc73c\uba70, \uac74\ubb3c\uc5d0\uc11c \ub2e4\ub7c9\uc758 \uc9c0\ubb38\uc744 \ucc44\ucde8\ud588\uace0, \uba38\ub9ac\uce74\ub77d\uc744 \uc218\uac70\ud558\uc600\ub2e4. \uac74\ubb3c\uc758 3\uce35\uc5d0\uc11c\ub294 \ubc00\uc2e4\uc744 \ud655\uc778\ud558\uace0 \ubc00\uc2e4 \ub0b4\ubd80\uc758 \uc695\uc2e4\uc5d0\uc11c \uba38\ub9ac\uce74\ub77d \ub4f1\uc744 \ubc1c\uacac\ud574 \uc218\uac70\ud588\uace0 \uc9c0\ubb38\uc744 \ucc44\ucde8\ud558\uace0, \uc871\uc801\uc744 \ucc44\ucde8\ud558\uc600\ub2e4. \uacbd\ucc30\uc774 \uc218\uac70\ud55c \uc790\ub8cc\uc5d0\ub294 \uc720\uc804\uc790 \uc870\uac01\uc774 \ub0a8\uc544\uc788\uc744 \uce68\uc2e4 \ubc0f \uac70\uc2e4 \ub3c4\uad6c \ub4f1\ub3c4 \ucc44\ucde8\ub418\uc5c8\ub2e4. \uacbd\ucc30\uc740 201\uc810 \ubb3c\ud488\uc5d0 \uc74c\uc545 CD\ub97c \uac01\uac01 1\uac1c\uc529 \uc13c \uac83\ub3c4 \ud3ec\ud568\ub41c\ub2e4. \uc2e4\uc81c \uc911\uc694\ud55c \ubb3c\ud488\uc740 20\uc810 \uc815\ub3c4\ub85c \ucd94\uc815\ub41c\ub2e4\ub294 \uc758\uacac\uc744 \ub367\ubd99\uc774\uae30\ub3c4 \ud588\ub2e4. 3\uc6d4 24\uc77c\uc5d0\ub294 2008\ub144 5\uc6d4 \ud0dc\uad6d \uace8\ud504\uc678\uc720\uc5d0 \uad00\ub828\ub41c \uc0ac\uc9c4 \uc790\ub8cc\uac00 \uacbd\ucc30\uc5d0 \uc785\uc218\ub418\uae30\ub3c4 \ud558\uc600\ub2e4. 25\uc77c \uc0bc\uc131\ub3d9 \uc0ac\ubb34\uc2e4\uc5d0\uc11c DNA \uc720\uc804\uc790 \uc790\ub8cc \ub4f1 \uc57d 96\uc810\uc744 \ud655\ubcf4\ub2e4\uace0 \ubc1d\ud788\uae30\ub3c4 \ud588\ub2e4."} {"question": "\uc62c\ub9bc\ud53d \uc804\uc57c\uc81c\ub97c \ud558\uae30\ub85c \ud569\uc758\ud55c \uc9c0\uc5ed\uc740 \uc5b4\ub514\uc778\uac00?", "paragraph": "\ub2e4\ub9cc, \ubaa8\ub4e0 \uacfc\uc815\uc774 \uc21c\uc870\ub86d\uac8c \uc9c4\ud589\ub418\uc9c0\ub9cc\uc740 \uc54a\uc558\ub2e4. 14\uc77c \ubd81\ud55c \uad00\uc601\ub9e4\uccb4\ub4e4\uc740 \uae40\uc815\uc740\uc758 \uc2e0\ub144\uc0ac \uc774\ud6c4 \ucc98\uc74c\uc73c\ub85c \uc815\ubd80\ub97c \ube44\ub09c\ud588\ub294\ub370 \ubb38\uc7ac\uc778\uc758 \uc2e0\ub144 \uae30\uc790\ud68c\uacac\uc5d0 \ub300\ud574 \"\uc6b0\ub9ac\ub294 \uc55e\uc73c\ub85c\ub3c4 \ubd81\ub0a8\uad00\uacc4 \uac1c\uc120\uc744 \uc704\ud558\uc5ec \uc801\uadf9 \ub178\ub825\ud560 \uac83\uc774\uc9c0\ub9cc \uadf8\uc5d0 \ucc2c\ubb3c\uc744 \ub07c\uc5b9\ub294 \ubd88\uc21c\ud55c \ud589\uc704\uc5d0 \ub300\ud574\uc11c\ub294 \uacb0\ucf54 \ubcf4\uace0\ub9cc \uc788\uc9c0 \uc54a\uc744 \uac83\"\uc774\ub77c\uace0 \ubc1d\ud78c \uac83\uc774\ub2e4. \ubb38\uc7ac\uc778\uc744 \ub0a8\uc870\uc120 \ub2f9\uad6d\uc790\ub85c \uc9c0\uce6d\ud558\uba70 '\uc5bc\ube60\uc9c4 \uada4\ubcc0', '\uac00\uc2dc\ub3cb\ud78c \uc74c\ud749\ud55c \uc545\uc124\uc77c\uc0c9'\uc774\ub77c\ub294 \ud45c\ud604\uae4c\uc9c0 \uc0ac\uc6a9\ud558\uba70 \"\uaca8\uc6b8\ucca0\uc62c\ub9bc\ud53d\uacbd\uae30\ub300\ud68c\uc5d0 \ucc38\uac00\ud560 \uc6b0\ub9ac \ub300\ud45c\ub2e8\uc744 \ud0dc\uc6b4 \uc5f4\ucc28\ub098 \ubc84\uc2a4\ub3c4 \uc544\uc9c1 \ud3c9\uc591\uc5d0 \uc788\ub2e4\ub294 \uac83\uc744 \uc54c\uc544\uc57c \ud55c\ub2e4\"\uace0 \uc704\ud611\ud588\ub2e4. \ubd81\ud55c\uc758 \uc18d\ub0b4\ub97c \uc758\uc2ec\ud558\ub294 \ud55c\uad6d \uc5b8\ub860\uc5d0 \ub300\ud55c \ubd88\ub9cc\ub3c4 \ud130\ub728\ub838\ub294\ub370 15\uc77c \u300a\uc870\uc120\uc911\uc559\ud1b5\uc2e0\u300b\uc740 \"\ub0a8\uc870\uc120 \ubcf4\uc218 \uc5b8\ub860\ub4e4 \uc18d\uc5d0\uc11c \ub3d9\uc871\uc758 \uc131\uc758\ub97c \uc6b0\ub871\ud558\uace0 \ubaa8\ub3c5\ud558\ub294 \uace0\uc57d\ud55c \uc545\uc124\ub4e4\uc774 \uc3df\uc544\uc838 \ub098\uc640 \uc138\uc0c1\uc744 \uacbd\uc545\uc2dc\ud0a4\uace0 \uc788\ub2e4\"\uace0 \uc131\ud1a0\ud588\ub2e4. \ubbf8\uad6d\uc5d0 \ub300\ud574\uc11c\ub3c4 '\ubbfc\uc871\uc758 \ub300\uc0ac\ub97c \ub9dd\uccd0\ub193\uc73c\ub824\uace0 \ubc1c\uad11\ud558\ub294 \uc544\uba54\ub9ac\uce74 \uae61\ud328\ub4e4'\uc774\ub77c\uba70 \ube44\ud575\ud654\uc640 \uc81c\uc7ac\ub97c \uc8fc\uc7a5\ud558\ub294 \ubbf8\uad6d\uc744 \ube44\ub09c\ud588\ub2e4. \ud55c\uad6d \ub0b4\uc5d0\uc11c\ub3c4 \ub17c\ub780\uc740 \ub9ce\uc558\ub2e4. \uae08\uac15\uc0b0 \uc62c\ub9bc\ud53d \uc804\uc57c\uc81c \ud569\uc758 \uc18c\uc2dd\uc744 \ub4e4\uc740 \ud3c9\ucc3d \uc8fc\ubbfc\ub4e4\uc740 \"\uc6b0\ub9ac\uac00 20\ub144 \ub3d9\uc548 \ud53c\ub540 \ud758\ub824 \uc720\uce58\ud588\ub294\ub370 \uae08\uac15\uc0b0 \ud589\uc0ac\uac00 \ubb50\ub0d0\"\uace0 \uc5b4\uc774\uc5c6\uc5b4 \ud588\uc73c\uba70 \ub0a8\ubd81 \ub2e8\uc77c\ud300\uacfc \uad00\ub828\ud574\uc11c \uccb4\uc721\uacc4 \ub0b4\uc5d0\uc11c\ub294 \"\uc62c\ub9bc\ud53d\uc5d0\uc11c \ubcf8\uc9c8\uc778 \uc2a4\ud3ec\uce20\uc640 \uc120\uc218\ub294 \uc0ac\ub77c\uc9c0\uace0 \ub0a8\ubd81\uc774\ub780 \ub2e8\uc5b4\ub9cc \uc3df\uc544\uc9c4\ub2e4\"\uace0 \ube44\ud310\ud588\ub2e4. 19\uc77c \ud604\uc1a1\uc6d4 \uad00\ud604\uc545\ub2e8\uc7a5\uc774 \uc608\uc220\ub2e8 \uc0ac\uc804\uc810\uac80 \ucc28\uc6d0\uc5d0\uc11c \ub2e4\uc74c \ub0a0 \ubc29\ub0a8\ud558\uae30\ub85c \ud1b5\uc9c0\ud588\ub2e4\uac00 \uadf8 \ub0a0 \ubc24\uc5d0 \uc774\ub97c \ucde8\uc18c\ud588\ub294\ub370 \ud1b5\uc77c\ubd80\ub294 \"\uacfc\uac70 \ubd81\ud55c\uc740 \uc6b0\ub9ac \uc5b8\ub860 \ubcf4\ub3c4\uc5d0 \ub300\ud574 \ubd88\ud3b8\ud55c \ubc18\uc751\uc744 \ubcf4\uc5ec\uc654\ub2e4\"\uba70 \uc5b8\ub860\uc758 \ucd94\uce21\uc131\u00b7\ube44\ud310\uc131 \ubcf4\ub3c4\ub97c \uc790\uc81c\ud574\uc904 \uac83\uc744 \uc694\uccad\ud558\uae30\ub3c4 \ud588\ub2e4.", "answer": "\uae08\uac15\uc0b0", "sentence": "\uae08\uac15\uc0b0 \uc62c\ub9bc\ud53d \uc804\uc57c\uc81c \ud569\uc758 \uc18c\uc2dd\uc744 \ub4e4\uc740 \ud3c9\ucc3d \uc8fc\ubbfc\ub4e4\uc740 \"\uc6b0\ub9ac\uac00 20\ub144 \ub3d9\uc548 \ud53c\ub540 \ud758\ub824 \uc720\uce58\ud588\ub294\ub370 \uae08\uac15\uc0b0 \ud589\uc0ac\uac00 \ubb50\ub0d0\"\uace0 \uc5b4\uc774\uc5c6\uc5b4 \ud588\uc73c\uba70 \ub0a8\ubd81 \ub2e8\uc77c\ud300\uacfc \uad00\ub828\ud574\uc11c \uccb4\uc721\uacc4 \ub0b4\uc5d0\uc11c\ub294 \"\uc62c\ub9bc\ud53d\uc5d0\uc11c \ubcf8\uc9c8\uc778 \uc2a4\ud3ec\uce20\uc640 \uc120\uc218\ub294 \uc0ac\ub77c\uc9c0\uace0 \ub0a8\ubd81\uc774\ub780 \ub2e8\uc5b4\ub9cc \uc3df\uc544\uc9c4\ub2e4\"\uace0 \ube44\ud310\ud588\ub2e4.", "paragraph_sentence": "\ub2e4\ub9cc, \ubaa8\ub4e0 \uacfc\uc815\uc774 \uc21c\uc870\ub86d\uac8c \uc9c4\ud589\ub418\uc9c0\ub9cc\uc740 \uc54a\uc558\ub2e4. 14\uc77c \ubd81\ud55c \uad00\uc601\ub9e4\uccb4\ub4e4\uc740 \uae40\uc815\uc740\uc758 \uc2e0\ub144\uc0ac \uc774\ud6c4 \ucc98\uc74c\uc73c\ub85c \uc815\ubd80\ub97c \ube44\ub09c\ud588\ub294\ub370 \ubb38\uc7ac\uc778\uc758 \uc2e0\ub144 \uae30\uc790\ud68c\uacac\uc5d0 \ub300\ud574 \"\uc6b0\ub9ac\ub294 \uc55e\uc73c\ub85c\ub3c4 \ubd81\ub0a8\uad00\uacc4 \uac1c\uc120\uc744 \uc704\ud558\uc5ec \uc801\uadf9 \ub178\ub825\ud560 \uac83\uc774\uc9c0\ub9cc \uadf8\uc5d0 \ucc2c\ubb3c\uc744 \ub07c\uc5b9\ub294 \ubd88\uc21c\ud55c \ud589\uc704\uc5d0 \ub300\ud574\uc11c\ub294 \uacb0\ucf54 \ubcf4\uace0\ub9cc \uc788\uc9c0 \uc54a\uc744 \uac83\"\uc774\ub77c\uace0 \ubc1d\ud78c \uac83\uc774\ub2e4. \ubb38\uc7ac\uc778\uc744 \ub0a8\uc870\uc120 \ub2f9\uad6d\uc790\ub85c \uc9c0\uce6d\ud558\uba70 '\uc5bc\ube60\uc9c4 \uada4\ubcc0', '\uac00\uc2dc\ub3cb\ud78c \uc74c\ud749\ud55c \uc545\uc124\uc77c\uc0c9'\uc774\ub77c\ub294 \ud45c\ud604\uae4c\uc9c0 \uc0ac\uc6a9\ud558\uba70 \"\uaca8\uc6b8\ucca0\uc62c\ub9bc\ud53d\uacbd\uae30\ub300\ud68c\uc5d0 \ucc38\uac00\ud560 \uc6b0\ub9ac \ub300\ud45c\ub2e8\uc744 \ud0dc\uc6b4 \uc5f4\ucc28\ub098 \ubc84\uc2a4\ub3c4 \uc544\uc9c1 \ud3c9\uc591\uc5d0 \uc788\ub2e4\ub294 \uac83\uc744 \uc54c\uc544\uc57c \ud55c\ub2e4\"\uace0 \uc704\ud611\ud588\ub2e4. \ubd81\ud55c\uc758 \uc18d\ub0b4\ub97c \uc758\uc2ec\ud558\ub294 \ud55c\uad6d \uc5b8\ub860\uc5d0 \ub300\ud55c \ubd88\ub9cc\ub3c4 \ud130\ub728\ub838\ub294\ub370 15\uc77c \u300a\uc870\uc120\uc911\uc559\ud1b5\uc2e0\u300b\uc740 \"\ub0a8\uc870\uc120 \ubcf4\uc218 \uc5b8\ub860\ub4e4 \uc18d\uc5d0\uc11c \ub3d9\uc871\uc758 \uc131\uc758\ub97c \uc6b0\ub871\ud558\uace0 \ubaa8\ub3c5\ud558\ub294 \uace0\uc57d\ud55c \uc545\uc124\ub4e4\uc774 \uc3df\uc544\uc838 \ub098\uc640 \uc138\uc0c1\uc744 \uacbd\uc545\uc2dc\ud0a4\uace0 \uc788\ub2e4\"\uace0 \uc131\ud1a0\ud588\ub2e4. \ubbf8\uad6d\uc5d0 \ub300\ud574\uc11c\ub3c4 '\ubbfc\uc871\uc758 \ub300\uc0ac\ub97c \ub9dd\uccd0\ub193\uc73c\ub824\uace0 \ubc1c\uad11\ud558\ub294 \uc544\uba54\ub9ac\uce74 \uae61\ud328\ub4e4'\uc774\ub77c\uba70 \ube44\ud575\ud654\uc640 \uc81c\uc7ac\ub97c \uc8fc\uc7a5\ud558\ub294 \ubbf8\uad6d\uc744 \ube44\ub09c\ud588\ub2e4. \ud55c\uad6d \ub0b4\uc5d0\uc11c\ub3c4 \ub17c\ub780\uc740 \ub9ce\uc558\ub2e4. \uae08\uac15\uc0b0 \uc62c\ub9bc\ud53d \uc804\uc57c\uc81c \ud569\uc758 \uc18c\uc2dd\uc744 \ub4e4\uc740 \ud3c9\ucc3d \uc8fc\ubbfc\ub4e4\uc740 \"\uc6b0\ub9ac\uac00 20\ub144 \ub3d9\uc548 \ud53c\ub540 \ud758\ub824 \uc720\uce58\ud588\ub294\ub370 \uae08\uac15\uc0b0 \ud589\uc0ac\uac00 \ubb50\ub0d0\"\uace0 \uc5b4\uc774\uc5c6\uc5b4 \ud588\uc73c\uba70 \ub0a8\ubd81 \ub2e8\uc77c\ud300\uacfc \uad00\ub828\ud574\uc11c \uccb4\uc721\uacc4 \ub0b4\uc5d0\uc11c\ub294 \"\uc62c\ub9bc\ud53d\uc5d0\uc11c \ubcf8\uc9c8\uc778 \uc2a4\ud3ec\uce20\uc640 \uc120\uc218\ub294 \uc0ac\ub77c\uc9c0\uace0 \ub0a8\ubd81\uc774\ub780 \ub2e8\uc5b4\ub9cc \uc3df\uc544\uc9c4\ub2e4\"\uace0 \ube44\ud310\ud588\ub2e4. 19\uc77c \ud604\uc1a1\uc6d4 \uad00\ud604\uc545\ub2e8\uc7a5\uc774 \uc608\uc220\ub2e8 \uc0ac\uc804\uc810\uac80 \ucc28\uc6d0\uc5d0\uc11c \ub2e4\uc74c \ub0a0 \ubc29\ub0a8\ud558\uae30\ub85c \ud1b5\uc9c0\ud588\ub2e4\uac00 \uadf8 \ub0a0 \ubc24\uc5d0 \uc774\ub97c \ucde8\uc18c\ud588\ub294\ub370 \ud1b5\uc77c\ubd80\ub294 \"\uacfc\uac70 \ubd81\ud55c\uc740 \uc6b0\ub9ac \uc5b8\ub860 \ubcf4\ub3c4\uc5d0 \ub300\ud574 \ubd88\ud3b8\ud55c \ubc18\uc751\uc744 \ubcf4\uc5ec\uc654\ub2e4\"\uba70 \uc5b8\ub860\uc758 \ucd94\uce21\uc131\u00b7\ube44\ud310\uc131 \ubcf4\ub3c4\ub97c \uc790\uc81c\ud574\uc904 \uac83\uc744 \uc694\uccad\ud558\uae30\ub3c4 \ud588\ub2e4.", "paragraph_answer": "\ub2e4\ub9cc, \ubaa8\ub4e0 \uacfc\uc815\uc774 \uc21c\uc870\ub86d\uac8c \uc9c4\ud589\ub418\uc9c0\ub9cc\uc740 \uc54a\uc558\ub2e4. 14\uc77c \ubd81\ud55c \uad00\uc601\ub9e4\uccb4\ub4e4\uc740 \uae40\uc815\uc740\uc758 \uc2e0\ub144\uc0ac \uc774\ud6c4 \ucc98\uc74c\uc73c\ub85c \uc815\ubd80\ub97c \ube44\ub09c\ud588\ub294\ub370 \ubb38\uc7ac\uc778\uc758 \uc2e0\ub144 \uae30\uc790\ud68c\uacac\uc5d0 \ub300\ud574 \"\uc6b0\ub9ac\ub294 \uc55e\uc73c\ub85c\ub3c4 \ubd81\ub0a8\uad00\uacc4 \uac1c\uc120\uc744 \uc704\ud558\uc5ec \uc801\uadf9 \ub178\ub825\ud560 \uac83\uc774\uc9c0\ub9cc \uadf8\uc5d0 \ucc2c\ubb3c\uc744 \ub07c\uc5b9\ub294 \ubd88\uc21c\ud55c \ud589\uc704\uc5d0 \ub300\ud574\uc11c\ub294 \uacb0\ucf54 \ubcf4\uace0\ub9cc \uc788\uc9c0 \uc54a\uc744 \uac83\"\uc774\ub77c\uace0 \ubc1d\ud78c \uac83\uc774\ub2e4. \ubb38\uc7ac\uc778\uc744 \ub0a8\uc870\uc120 \ub2f9\uad6d\uc790\ub85c \uc9c0\uce6d\ud558\uba70 '\uc5bc\ube60\uc9c4 \uada4\ubcc0', '\uac00\uc2dc\ub3cb\ud78c \uc74c\ud749\ud55c \uc545\uc124\uc77c\uc0c9'\uc774\ub77c\ub294 \ud45c\ud604\uae4c\uc9c0 \uc0ac\uc6a9\ud558\uba70 \"\uaca8\uc6b8\ucca0\uc62c\ub9bc\ud53d\uacbd\uae30\ub300\ud68c\uc5d0 \ucc38\uac00\ud560 \uc6b0\ub9ac \ub300\ud45c\ub2e8\uc744 \ud0dc\uc6b4 \uc5f4\ucc28\ub098 \ubc84\uc2a4\ub3c4 \uc544\uc9c1 \ud3c9\uc591\uc5d0 \uc788\ub2e4\ub294 \uac83\uc744 \uc54c\uc544\uc57c \ud55c\ub2e4\"\uace0 \uc704\ud611\ud588\ub2e4. \ubd81\ud55c\uc758 \uc18d\ub0b4\ub97c \uc758\uc2ec\ud558\ub294 \ud55c\uad6d \uc5b8\ub860\uc5d0 \ub300\ud55c \ubd88\ub9cc\ub3c4 \ud130\ub728\ub838\ub294\ub370 15\uc77c \u300a\uc870\uc120\uc911\uc559\ud1b5\uc2e0\u300b\uc740 \"\ub0a8\uc870\uc120 \ubcf4\uc218 \uc5b8\ub860\ub4e4 \uc18d\uc5d0\uc11c \ub3d9\uc871\uc758 \uc131\uc758\ub97c \uc6b0\ub871\ud558\uace0 \ubaa8\ub3c5\ud558\ub294 \uace0\uc57d\ud55c \uc545\uc124\ub4e4\uc774 \uc3df\uc544\uc838 \ub098\uc640 \uc138\uc0c1\uc744 \uacbd\uc545\uc2dc\ud0a4\uace0 \uc788\ub2e4\"\uace0 \uc131\ud1a0\ud588\ub2e4. \ubbf8\uad6d\uc5d0 \ub300\ud574\uc11c\ub3c4 '\ubbfc\uc871\uc758 \ub300\uc0ac\ub97c \ub9dd\uccd0\ub193\uc73c\ub824\uace0 \ubc1c\uad11\ud558\ub294 \uc544\uba54\ub9ac\uce74 \uae61\ud328\ub4e4'\uc774\ub77c\uba70 \ube44\ud575\ud654\uc640 \uc81c\uc7ac\ub97c \uc8fc\uc7a5\ud558\ub294 \ubbf8\uad6d\uc744 \ube44\ub09c\ud588\ub2e4. \ud55c\uad6d \ub0b4\uc5d0\uc11c\ub3c4 \ub17c\ub780\uc740 \ub9ce\uc558\ub2e4. \uae08\uac15\uc0b0 \uc62c\ub9bc\ud53d \uc804\uc57c\uc81c \ud569\uc758 \uc18c\uc2dd\uc744 \ub4e4\uc740 \ud3c9\ucc3d \uc8fc\ubbfc\ub4e4\uc740 \"\uc6b0\ub9ac\uac00 20\ub144 \ub3d9\uc548 \ud53c\ub540 \ud758\ub824 \uc720\uce58\ud588\ub294\ub370 \uae08\uac15\uc0b0 \ud589\uc0ac\uac00 \ubb50\ub0d0\"\uace0 \uc5b4\uc774\uc5c6\uc5b4 \ud588\uc73c\uba70 \ub0a8\ubd81 \ub2e8\uc77c\ud300\uacfc \uad00\ub828\ud574\uc11c \uccb4\uc721\uacc4 \ub0b4\uc5d0\uc11c\ub294 \"\uc62c\ub9bc\ud53d\uc5d0\uc11c \ubcf8\uc9c8\uc778 \uc2a4\ud3ec\uce20\uc640 \uc120\uc218\ub294 \uc0ac\ub77c\uc9c0\uace0 \ub0a8\ubd81\uc774\ub780 \ub2e8\uc5b4\ub9cc \uc3df\uc544\uc9c4\ub2e4\"\uace0 \ube44\ud310\ud588\ub2e4. 19\uc77c \ud604\uc1a1\uc6d4 \uad00\ud604\uc545\ub2e8\uc7a5\uc774 \uc608\uc220\ub2e8 \uc0ac\uc804\uc810\uac80 \ucc28\uc6d0\uc5d0\uc11c \ub2e4\uc74c \ub0a0 \ubc29\ub0a8\ud558\uae30\ub85c \ud1b5\uc9c0\ud588\ub2e4\uac00 \uadf8 \ub0a0 \ubc24\uc5d0 \uc774\ub97c \ucde8\uc18c\ud588\ub294\ub370 \ud1b5\uc77c\ubd80\ub294 \"\uacfc\uac70 \ubd81\ud55c\uc740 \uc6b0\ub9ac \uc5b8\ub860 \ubcf4\ub3c4\uc5d0 \ub300\ud574 \ubd88\ud3b8\ud55c \ubc18\uc751\uc744 \ubcf4\uc5ec\uc654\ub2e4\"\uba70 \uc5b8\ub860\uc758 \ucd94\uce21\uc131\u00b7\ube44\ud310\uc131 \ubcf4\ub3c4\ub97c \uc790\uc81c\ud574\uc904 \uac83\uc744 \uc694\uccad\ud558\uae30\ub3c4 \ud588\ub2e4.", "sentence_answer": " \uae08\uac15\uc0b0 \uc62c\ub9bc\ud53d \uc804\uc57c\uc81c \ud569\uc758 \uc18c\uc2dd\uc744 \ub4e4\uc740 \ud3c9\ucc3d \uc8fc\ubbfc\ub4e4\uc740 \"\uc6b0\ub9ac\uac00 20\ub144 \ub3d9\uc548 \ud53c\ub540 \ud758\ub824 \uc720\uce58\ud588\ub294\ub370 \uae08\uac15\uc0b0 \ud589\uc0ac\uac00 \ubb50\ub0d0\"\uace0 \uc5b4\uc774\uc5c6\uc5b4 \ud588\uc73c\uba70 \ub0a8\ubd81 \ub2e8\uc77c\ud300\uacfc \uad00\ub828\ud574\uc11c \uccb4\uc721\uacc4 \ub0b4\uc5d0\uc11c\ub294 \"\uc62c\ub9bc\ud53d\uc5d0\uc11c \ubcf8\uc9c8\uc778 \uc2a4\ud3ec\uce20\uc640 \uc120\uc218\ub294 \uc0ac\ub77c\uc9c0\uace0 \ub0a8\ubd81\uc774\ub780 \ub2e8\uc5b4\ub9cc \uc3df\uc544\uc9c4\ub2e4\"\uace0 \ube44\ud310\ud588\ub2e4.", "paragraph_id": "6546926-108-1", "paragraph_question": "question: \uc62c\ub9bc\ud53d \uc804\uc57c\uc81c\ub97c \ud558\uae30\ub85c \ud569\uc758\ud55c \uc9c0\uc5ed\uc740 \uc5b4\ub514\uc778\uac00?, context: \ub2e4\ub9cc, \ubaa8\ub4e0 \uacfc\uc815\uc774 \uc21c\uc870\ub86d\uac8c \uc9c4\ud589\ub418\uc9c0\ub9cc\uc740 \uc54a\uc558\ub2e4. 14\uc77c \ubd81\ud55c \uad00\uc601\ub9e4\uccb4\ub4e4\uc740 \uae40\uc815\uc740\uc758 \uc2e0\ub144\uc0ac \uc774\ud6c4 \ucc98\uc74c\uc73c\ub85c \uc815\ubd80\ub97c \ube44\ub09c\ud588\ub294\ub370 \ubb38\uc7ac\uc778\uc758 \uc2e0\ub144 \uae30\uc790\ud68c\uacac\uc5d0 \ub300\ud574 \"\uc6b0\ub9ac\ub294 \uc55e\uc73c\ub85c\ub3c4 \ubd81\ub0a8\uad00\uacc4 \uac1c\uc120\uc744 \uc704\ud558\uc5ec \uc801\uadf9 \ub178\ub825\ud560 \uac83\uc774\uc9c0\ub9cc \uadf8\uc5d0 \ucc2c\ubb3c\uc744 \ub07c\uc5b9\ub294 \ubd88\uc21c\ud55c \ud589\uc704\uc5d0 \ub300\ud574\uc11c\ub294 \uacb0\ucf54 \ubcf4\uace0\ub9cc \uc788\uc9c0 \uc54a\uc744 \uac83\"\uc774\ub77c\uace0 \ubc1d\ud78c \uac83\uc774\ub2e4. \ubb38\uc7ac\uc778\uc744 \ub0a8\uc870\uc120 \ub2f9\uad6d\uc790\ub85c \uc9c0\uce6d\ud558\uba70 '\uc5bc\ube60\uc9c4 \uada4\ubcc0', '\uac00\uc2dc\ub3cb\ud78c \uc74c\ud749\ud55c \uc545\uc124\uc77c\uc0c9'\uc774\ub77c\ub294 \ud45c\ud604\uae4c\uc9c0 \uc0ac\uc6a9\ud558\uba70 \"\uaca8\uc6b8\ucca0\uc62c\ub9bc\ud53d\uacbd\uae30\ub300\ud68c\uc5d0 \ucc38\uac00\ud560 \uc6b0\ub9ac \ub300\ud45c\ub2e8\uc744 \ud0dc\uc6b4 \uc5f4\ucc28\ub098 \ubc84\uc2a4\ub3c4 \uc544\uc9c1 \ud3c9\uc591\uc5d0 \uc788\ub2e4\ub294 \uac83\uc744 \uc54c\uc544\uc57c \ud55c\ub2e4\"\uace0 \uc704\ud611\ud588\ub2e4. \ubd81\ud55c\uc758 \uc18d\ub0b4\ub97c \uc758\uc2ec\ud558\ub294 \ud55c\uad6d \uc5b8\ub860\uc5d0 \ub300\ud55c \ubd88\ub9cc\ub3c4 \ud130\ub728\ub838\ub294\ub370 15\uc77c \u300a\uc870\uc120\uc911\uc559\ud1b5\uc2e0\u300b\uc740 \"\ub0a8\uc870\uc120 \ubcf4\uc218 \uc5b8\ub860\ub4e4 \uc18d\uc5d0\uc11c \ub3d9\uc871\uc758 \uc131\uc758\ub97c \uc6b0\ub871\ud558\uace0 \ubaa8\ub3c5\ud558\ub294 \uace0\uc57d\ud55c \uc545\uc124\ub4e4\uc774 \uc3df\uc544\uc838 \ub098\uc640 \uc138\uc0c1\uc744 \uacbd\uc545\uc2dc\ud0a4\uace0 \uc788\ub2e4\"\uace0 \uc131\ud1a0\ud588\ub2e4. \ubbf8\uad6d\uc5d0 \ub300\ud574\uc11c\ub3c4 '\ubbfc\uc871\uc758 \ub300\uc0ac\ub97c \ub9dd\uccd0\ub193\uc73c\ub824\uace0 \ubc1c\uad11\ud558\ub294 \uc544\uba54\ub9ac\uce74 \uae61\ud328\ub4e4'\uc774\ub77c\uba70 \ube44\ud575\ud654\uc640 \uc81c\uc7ac\ub97c \uc8fc\uc7a5\ud558\ub294 \ubbf8\uad6d\uc744 \ube44\ub09c\ud588\ub2e4. \ud55c\uad6d \ub0b4\uc5d0\uc11c\ub3c4 \ub17c\ub780\uc740 \ub9ce\uc558\ub2e4. \uae08\uac15\uc0b0 \uc62c\ub9bc\ud53d \uc804\uc57c\uc81c \ud569\uc758 \uc18c\uc2dd\uc744 \ub4e4\uc740 \ud3c9\ucc3d \uc8fc\ubbfc\ub4e4\uc740 \"\uc6b0\ub9ac\uac00 20\ub144 \ub3d9\uc548 \ud53c\ub540 \ud758\ub824 \uc720\uce58\ud588\ub294\ub370 \uae08\uac15\uc0b0 \ud589\uc0ac\uac00 \ubb50\ub0d0\"\uace0 \uc5b4\uc774\uc5c6\uc5b4 \ud588\uc73c\uba70 \ub0a8\ubd81 \ub2e8\uc77c\ud300\uacfc \uad00\ub828\ud574\uc11c \uccb4\uc721\uacc4 \ub0b4\uc5d0\uc11c\ub294 \"\uc62c\ub9bc\ud53d\uc5d0\uc11c \ubcf8\uc9c8\uc778 \uc2a4\ud3ec\uce20\uc640 \uc120\uc218\ub294 \uc0ac\ub77c\uc9c0\uace0 \ub0a8\ubd81\uc774\ub780 \ub2e8\uc5b4\ub9cc \uc3df\uc544\uc9c4\ub2e4\"\uace0 \ube44\ud310\ud588\ub2e4. 19\uc77c \ud604\uc1a1\uc6d4 \uad00\ud604\uc545\ub2e8\uc7a5\uc774 \uc608\uc220\ub2e8 \uc0ac\uc804\uc810\uac80 \ucc28\uc6d0\uc5d0\uc11c \ub2e4\uc74c \ub0a0 \ubc29\ub0a8\ud558\uae30\ub85c \ud1b5\uc9c0\ud588\ub2e4\uac00 \uadf8 \ub0a0 \ubc24\uc5d0 \uc774\ub97c \ucde8\uc18c\ud588\ub294\ub370 \ud1b5\uc77c\ubd80\ub294 \"\uacfc\uac70 \ubd81\ud55c\uc740 \uc6b0\ub9ac \uc5b8\ub860 \ubcf4\ub3c4\uc5d0 \ub300\ud574 \ubd88\ud3b8\ud55c \ubc18\uc751\uc744 \ubcf4\uc5ec\uc654\ub2e4\"\uba70 \uc5b8\ub860\uc758 \ucd94\uce21\uc131\u00b7\ube44\ud310\uc131 \ubcf4\ub3c4\ub97c \uc790\uc81c\ud574\uc904 \uac83\uc744 \uc694\uccad\ud558\uae30\ub3c4 \ud588\ub2e4."} {"question": "2012\ub144 10\uc6d4 \uc0c8\ub204\ub9ac\ub2f9\uacfc \ud569\ub2f9\ud558\uc5ec \ubcf4\uc218\uc815\ub2f9\uc758 \ubd84\uc5f4\uc744 \uc885\uc2dd\uc2dc\ud0a8 \ub2f9\uc758 \uc774\ub984\uc740?", "paragraph": "2011\ub144 10.26 \uc7ac\ubcf4\uad90\uc120\uac70\uc5d0\uc11c \ud328\ubc30\ud558\uc790 \ud55c\ub098\ub77c\ub2f9\uc774 \ubc15\uadfc\ud61c \ube44\uc0c1\ub300\ucc45\uc704\uc6d0\uc7a5\uc744 \uc911\uc2ec\uc73c\ub85c \uc7ac\ud3b8\ub418\uc5c8\uc73c\uba70, 2012\ub144 2\uc6d4 13\uc77c \uc0c8\ub204\ub9ac\ub2f9\uc73c\ub85c \ub2f9\uba85\uc744 \uac1c\uba85\ud558\uc600\ub2e4. 2012\ub144 \ucd1d\uc120\uc5d0\uc11c \ud328\ubc30\uac00 \uc608\uc0c1\ub418\uc5c8\uc73c\ub098 \uacfc\ubc18\uc758\uc11d\uc744 \uc5bb\uc5c8\ub2e4. 2012\ub144 \ub300\uc120\uc5d0\uc11c \ubc15\uadfc\ud61c\uac00 \ub2f9\uc120\ub418\uc5b4 \uc5ec\ub2f9\uc758 \uc9c0\uc704\ub97c \uacc4\uc18d \uc720\uc9c0\ud558\uc600\ub2e4. \uae30\uc874\uc758 \ud55c\ub098\ub77c\ub2f9\uc5d0\uc11c \uac1c\uce6d\ud558\uc600\uae30 \ub54c\ubb38\uc5d0 \uc815\ub2f9\ub4f1\ub85d\uc77c\uc740 1997\ub144 11\uc6d4 24\uc77c\uc774\ub2e4. 2012\ub144 10\uc6d4 \uc120\uc9c4\ud1b5\uc77c\ub2f9\uacfc \ud569\ub2f9\ud568\uc73c\ub85c\uc368 2000\ub144\ub300 \uc911\ubc18 \uc774\ud6c4 \ubd84\uc5f4\ub418\uc5c8\ub358 \ubcf4\uc218\uc815\ub2f9\ub4e4\uc774 \uc0c8\ub204\ub9ac\ub2f9\uc73c\ub85c \ubaa8\ub450 \ud569\ub2f9\ub418\uc5b4 \ubd84\uc5f4\uc774 \uc885\uc2dd\ub418\uc5c8\ub2e4. \ub300\ud45c\uc801\uc778 \uc778\ubb3c\ub85c\ub294 \ubc15\uadfc\ud61c, \uc774\uba85\ubc15, \uc774\uc7ac\uc624, \uc815\ub450\uc5b8, \uae40\ubb34\uc131, \uc815\ubabd\uc900, \uc11c\uccad\uc6d0, \uc774\ud55c\uad6c, \uae40\ubb38\uc218, \ud64d\uc900\ud45c, \uac15\uc7ac\uc12d, \uc774\uc778\uc81c, \uc774\ud68c\ucc3d, \uc2ec\uc7ac\ucca0, \ucd5c\uacbd\ud658, \ud669\uc6b0\uc5ec \ub4f1\uc774 \uc788\ub2e4.", "answer": "\uc120\uc9c4\ud1b5\uc77c\ub2f9", "sentence": "2012\ub144 10\uc6d4 \uc120\uc9c4\ud1b5\uc77c\ub2f9 \uacfc \ud569\ub2f9\ud568\uc73c\ub85c\uc368 2000\ub144\ub300 \uc911\ubc18 \uc774\ud6c4 \ubd84\uc5f4\ub418\uc5c8\ub358 \ubcf4\uc218\uc815\ub2f9\ub4e4\uc774 \uc0c8\ub204\ub9ac\ub2f9\uc73c\ub85c \ubaa8\ub450 \ud569\ub2f9\ub418\uc5b4 \ubd84\uc5f4\uc774 \uc885\uc2dd\ub418\uc5c8\ub2e4.", "paragraph_sentence": "2011\ub144 10.26 \uc7ac\ubcf4\uad90\uc120\uac70\uc5d0\uc11c \ud328\ubc30\ud558\uc790 \ud55c\ub098\ub77c\ub2f9\uc774 \ubc15\uadfc\ud61c \ube44\uc0c1\ub300\ucc45\uc704\uc6d0\uc7a5\uc744 \uc911\uc2ec\uc73c\ub85c \uc7ac\ud3b8\ub418\uc5c8\uc73c\uba70, 2012\ub144 2\uc6d4 13\uc77c \uc0c8\ub204\ub9ac\ub2f9\uc73c\ub85c \ub2f9\uba85\uc744 \uac1c\uba85\ud558\uc600\ub2e4. 2012\ub144 \ucd1d\uc120\uc5d0\uc11c \ud328\ubc30\uac00 \uc608\uc0c1\ub418\uc5c8\uc73c\ub098 \uacfc\ubc18\uc758\uc11d\uc744 \uc5bb\uc5c8\ub2e4. 2012\ub144 \ub300\uc120\uc5d0\uc11c \ubc15\uadfc\ud61c\uac00 \ub2f9\uc120\ub418\uc5b4 \uc5ec\ub2f9\uc758 \uc9c0\uc704\ub97c \uacc4\uc18d \uc720\uc9c0\ud558\uc600\ub2e4. \uae30\uc874\uc758 \ud55c\ub098\ub77c\ub2f9\uc5d0\uc11c \uac1c\uce6d\ud558\uc600\uae30 \ub54c\ubb38\uc5d0 \uc815\ub2f9\ub4f1\ub85d\uc77c\uc740 1997\ub144 11\uc6d4 24\uc77c\uc774\ub2e4. 2012\ub144 10\uc6d4 \uc120\uc9c4\ud1b5\uc77c\ub2f9 \uacfc \ud569\ub2f9\ud568\uc73c\ub85c\uc368 2000\ub144\ub300 \uc911\ubc18 \uc774\ud6c4 \ubd84\uc5f4\ub418\uc5c8\ub358 \ubcf4\uc218\uc815\ub2f9\ub4e4\uc774 \uc0c8\ub204\ub9ac\ub2f9\uc73c\ub85c \ubaa8\ub450 \ud569\ub2f9\ub418\uc5b4 \ubd84\uc5f4\uc774 \uc885\uc2dd\ub418\uc5c8\ub2e4. \ub300\ud45c\uc801\uc778 \uc778\ubb3c\ub85c\ub294 \ubc15\uadfc\ud61c, \uc774\uba85\ubc15, \uc774\uc7ac\uc624, \uc815\ub450\uc5b8, \uae40\ubb34\uc131, \uc815\ubabd\uc900, \uc11c\uccad\uc6d0, \uc774\ud55c\uad6c, \uae40\ubb38\uc218, \ud64d\uc900\ud45c, \uac15\uc7ac\uc12d, \uc774\uc778\uc81c, \uc774\ud68c\ucc3d, \uc2ec\uc7ac\ucca0, \ucd5c\uacbd\ud658, \ud669\uc6b0\uc5ec \ub4f1\uc774 \uc788\ub2e4.", "paragraph_answer": "2011\ub144 10.26 \uc7ac\ubcf4\uad90\uc120\uac70\uc5d0\uc11c \ud328\ubc30\ud558\uc790 \ud55c\ub098\ub77c\ub2f9\uc774 \ubc15\uadfc\ud61c \ube44\uc0c1\ub300\ucc45\uc704\uc6d0\uc7a5\uc744 \uc911\uc2ec\uc73c\ub85c \uc7ac\ud3b8\ub418\uc5c8\uc73c\uba70, 2012\ub144 2\uc6d4 13\uc77c \uc0c8\ub204\ub9ac\ub2f9\uc73c\ub85c \ub2f9\uba85\uc744 \uac1c\uba85\ud558\uc600\ub2e4. 2012\ub144 \ucd1d\uc120\uc5d0\uc11c \ud328\ubc30\uac00 \uc608\uc0c1\ub418\uc5c8\uc73c\ub098 \uacfc\ubc18\uc758\uc11d\uc744 \uc5bb\uc5c8\ub2e4. 2012\ub144 \ub300\uc120\uc5d0\uc11c \ubc15\uadfc\ud61c\uac00 \ub2f9\uc120\ub418\uc5b4 \uc5ec\ub2f9\uc758 \uc9c0\uc704\ub97c \uacc4\uc18d \uc720\uc9c0\ud558\uc600\ub2e4. \uae30\uc874\uc758 \ud55c\ub098\ub77c\ub2f9\uc5d0\uc11c \uac1c\uce6d\ud558\uc600\uae30 \ub54c\ubb38\uc5d0 \uc815\ub2f9\ub4f1\ub85d\uc77c\uc740 1997\ub144 11\uc6d4 24\uc77c\uc774\ub2e4. 2012\ub144 10\uc6d4 \uc120\uc9c4\ud1b5\uc77c\ub2f9 \uacfc \ud569\ub2f9\ud568\uc73c\ub85c\uc368 2000\ub144\ub300 \uc911\ubc18 \uc774\ud6c4 \ubd84\uc5f4\ub418\uc5c8\ub358 \ubcf4\uc218\uc815\ub2f9\ub4e4\uc774 \uc0c8\ub204\ub9ac\ub2f9\uc73c\ub85c \ubaa8\ub450 \ud569\ub2f9\ub418\uc5b4 \ubd84\uc5f4\uc774 \uc885\uc2dd\ub418\uc5c8\ub2e4. \ub300\ud45c\uc801\uc778 \uc778\ubb3c\ub85c\ub294 \ubc15\uadfc\ud61c, \uc774\uba85\ubc15, \uc774\uc7ac\uc624, \uc815\ub450\uc5b8, \uae40\ubb34\uc131, \uc815\ubabd\uc900, \uc11c\uccad\uc6d0, \uc774\ud55c\uad6c, \uae40\ubb38\uc218, \ud64d\uc900\ud45c, \uac15\uc7ac\uc12d, \uc774\uc778\uc81c, \uc774\ud68c\ucc3d, \uc2ec\uc7ac\ucca0, \ucd5c\uacbd\ud658, \ud669\uc6b0\uc5ec \ub4f1\uc774 \uc788\ub2e4.", "sentence_answer": "2012\ub144 10\uc6d4 \uc120\uc9c4\ud1b5\uc77c\ub2f9 \uacfc \ud569\ub2f9\ud568\uc73c\ub85c\uc368 2000\ub144\ub300 \uc911\ubc18 \uc774\ud6c4 \ubd84\uc5f4\ub418\uc5c8\ub358 \ubcf4\uc218\uc815\ub2f9\ub4e4\uc774 \uc0c8\ub204\ub9ac\ub2f9\uc73c\ub85c \ubaa8\ub450 \ud569\ub2f9\ub418\uc5b4 \ubd84\uc5f4\uc774 \uc885\uc2dd\ub418\uc5c8\ub2e4.", "paragraph_id": "6486249-3-1", "paragraph_question": "question: 2012\ub144 10\uc6d4 \uc0c8\ub204\ub9ac\ub2f9\uacfc \ud569\ub2f9\ud558\uc5ec \ubcf4\uc218\uc815\ub2f9\uc758 \ubd84\uc5f4\uc744 \uc885\uc2dd\uc2dc\ud0a8 \ub2f9\uc758 \uc774\ub984\uc740?, context: 2011\ub144 10.26 \uc7ac\ubcf4\uad90\uc120\uac70\uc5d0\uc11c \ud328\ubc30\ud558\uc790 \ud55c\ub098\ub77c\ub2f9\uc774 \ubc15\uadfc\ud61c \ube44\uc0c1\ub300\ucc45\uc704\uc6d0\uc7a5\uc744 \uc911\uc2ec\uc73c\ub85c \uc7ac\ud3b8\ub418\uc5c8\uc73c\uba70, 2012\ub144 2\uc6d4 13\uc77c \uc0c8\ub204\ub9ac\ub2f9\uc73c\ub85c \ub2f9\uba85\uc744 \uac1c\uba85\ud558\uc600\ub2e4. 2012\ub144 \ucd1d\uc120\uc5d0\uc11c \ud328\ubc30\uac00 \uc608\uc0c1\ub418\uc5c8\uc73c\ub098 \uacfc\ubc18\uc758\uc11d\uc744 \uc5bb\uc5c8\ub2e4. 2012\ub144 \ub300\uc120\uc5d0\uc11c \ubc15\uadfc\ud61c\uac00 \ub2f9\uc120\ub418\uc5b4 \uc5ec\ub2f9\uc758 \uc9c0\uc704\ub97c \uacc4\uc18d \uc720\uc9c0\ud558\uc600\ub2e4. \uae30\uc874\uc758 \ud55c\ub098\ub77c\ub2f9\uc5d0\uc11c \uac1c\uce6d\ud558\uc600\uae30 \ub54c\ubb38\uc5d0 \uc815\ub2f9\ub4f1\ub85d\uc77c\uc740 1997\ub144 11\uc6d4 24\uc77c\uc774\ub2e4. 2012\ub144 10\uc6d4 \uc120\uc9c4\ud1b5\uc77c\ub2f9\uacfc \ud569\ub2f9\ud568\uc73c\ub85c\uc368 2000\ub144\ub300 \uc911\ubc18 \uc774\ud6c4 \ubd84\uc5f4\ub418\uc5c8\ub358 \ubcf4\uc218\uc815\ub2f9\ub4e4\uc774 \uc0c8\ub204\ub9ac\ub2f9\uc73c\ub85c \ubaa8\ub450 \ud569\ub2f9\ub418\uc5b4 \ubd84\uc5f4\uc774 \uc885\uc2dd\ub418\uc5c8\ub2e4. \ub300\ud45c\uc801\uc778 \uc778\ubb3c\ub85c\ub294 \ubc15\uadfc\ud61c, \uc774\uba85\ubc15, \uc774\uc7ac\uc624, \uc815\ub450\uc5b8, \uae40\ubb34\uc131, \uc815\ubabd\uc900, \uc11c\uccad\uc6d0, \uc774\ud55c\uad6c, \uae40\ubb38\uc218, \ud64d\uc900\ud45c, \uac15\uc7ac\uc12d, \uc774\uc778\uc81c, \uc774\ud68c\ucc3d, \uc2ec\uc7ac\ucca0, \ucd5c\uacbd\ud658, \ud669\uc6b0\uc5ec \ub4f1\uc774 \uc788\ub2e4."} {"question": "\ub17c\ubb38\uc744 \uc9c0\ub3c4\ud55c \uad50\uc218\ub294 \ub204\uad6c\uc778\uac00?", "paragraph": "\uadf8\ub7ec\ub098 \ud589\uc815\ub300\ud559\uc6d0\uc740 \uac19\uc740 \ud574 5\uc6d4 27\uc77c \"\uc6d0\uc0dd\uc758 \ud76c\ub9dd\uc5d0 \uc758\ud574 \ud559\uc704\ub97c \ucde8\uc18c\ud558\uae30 \uc704\ud55c, \ud559\uce59\uc0c1 \uadfc\uac70\uac00 \ubd88\ubd84\uba85\ud558\ub2e4\"\uba70 \"\ubcf8\uc870\uc0ac\uc758 (\ud559\uc704 \ucde8\uc18c\ub97c \uc704\ud55c \ud655\uace0\ud55c \ud45c\uc808 \ud310\uc815) \uacb0\uacfc\ub97c \ub2ec\ub77c\"\uace0 \uc5f0\uad6c\uc724\ub9ac\uc704\uc5d0 \ubc18\uc1a1\ud558\uc600\uace0 \uacb0\uacfc\uc801\uc73c\ub85c \ub17c\ubb38\uc740 \uc720\ud6a8\ud55c \uc0c1\ud0dc\uac00 \ub410\ub2e4. \ub17c\ubb38\uc744 \uc9c0\ub3c4\ud55c \uc774\uc601\uade0(\ud589\uc815\ud559\uacfc) \uad50\uc218\ub294 \uc77c\ubd80 \uc778\uc6a9\uacfc \ucd9c\ucc98\ub97c \uba85\uae30\ud558\uc9c0 \uc54a\uc558\ub2e4\ub294 \uc8fc\uc7a5\uc5d0 \ub300\ud574 \"\uc774 \ub17c\ubb38\uc758 \ud575\uc2ec\uc740 \ucd1d\uccb4\uc801\uc73c\ub85c \ubcc0\ud638\uc0ac\uc774\uace0 \uc2dc\ubbfc\ub2e8\uccb4 \ub9ac\ub354\uc600\ub358 \uc791\uc131\uc790\uc758 \uacbd\ud5d8\uacfc \ud604\uc7a5\uc790\ub8cc \uc218\uc9d1\uc73c\ub85c \uc791\uc131\ub41c \uac83\uc73c\ub85c, \ub17c\ubb38\uc2ec\uc0ac \ub2f9\uc2dc\uc758 \uc801\uaca9\ud310\uc815\uc744 \ub4a4\uc9d1\uc744 \uc815\ub3c4\ub294 \uc544\ub2c8\ub2e4\"\ub77c\uba70 \"\ub2f9\uc2dc\uc758 \ud2b9\uc218\ub300\ud559\uc6d0 \uc11d\uc0ac \ud559\uc704 \ub17c\ubb38\uc758 \uc77c\ubc18\uc801\uc778 \uad00\ud589\uacfc \ud559\ubb38\uc801 \uc131\ucde8\ub3c4 \uc218\uc900\uc5d0 \ube44\ucdb0 \ubcf4\ub354\ub77c\ub3c4, \uc190\uc0c9\uc774 \uc5c6\ub294 \ub17c\ubb38\uc73c\ub85c \ud310\ub2e8\ud55c\ub2e4\"\uace0 \ub9d0\ud588\ub2e4.", "answer": "\uc774\uc601\uade0", "sentence": "\ub17c\ubb38\uc744 \uc9c0\ub3c4\ud55c \uc774\uc601\uade0 (\ud589\uc815\ud559\uacfc)", "paragraph_sentence": "\uadf8\ub7ec\ub098 \ud589\uc815\ub300\ud559\uc6d0\uc740 \uac19\uc740 \ud574 5\uc6d4 27\uc77c \"\uc6d0\uc0dd\uc758 \ud76c\ub9dd\uc5d0 \uc758\ud574 \ud559\uc704\ub97c \ucde8\uc18c\ud558\uae30 \uc704\ud55c, \ud559\uce59\uc0c1 \uadfc\uac70\uac00 \ubd88\ubd84\uba85\ud558\ub2e4\"\uba70 \"\ubcf8\uc870\uc0ac\uc758 (\ud559\uc704 \ucde8\uc18c\ub97c \uc704\ud55c \ud655\uace0\ud55c \ud45c\uc808 \ud310\uc815) \uacb0\uacfc\ub97c \ub2ec\ub77c\"\uace0 \uc5f0\uad6c\uc724\ub9ac\uc704\uc5d0 \ubc18\uc1a1\ud558\uc600\uace0 \uacb0\uacfc\uc801\uc73c\ub85c \ub17c\ubb38\uc740 \uc720\ud6a8\ud55c \uc0c1\ud0dc\uac00 \ub410\ub2e4. \ub17c\ubb38\uc744 \uc9c0\ub3c4\ud55c \uc774\uc601\uade0 (\ud589\uc815\ud559\uacfc) \uad50\uc218\ub294 \uc77c\ubd80 \uc778\uc6a9\uacfc \ucd9c\ucc98\ub97c \uba85\uae30\ud558\uc9c0 \uc54a\uc558\ub2e4\ub294 \uc8fc\uc7a5\uc5d0 \ub300\ud574 \"\uc774 \ub17c\ubb38\uc758 \ud575\uc2ec\uc740 \ucd1d\uccb4\uc801\uc73c\ub85c \ubcc0\ud638\uc0ac\uc774\uace0 \uc2dc\ubbfc\ub2e8\uccb4 \ub9ac\ub354\uc600\ub358 \uc791\uc131\uc790\uc758 \uacbd\ud5d8\uacfc \ud604\uc7a5\uc790\ub8cc \uc218\uc9d1\uc73c\ub85c \uc791\uc131\ub41c \uac83\uc73c\ub85c, \ub17c\ubb38\uc2ec\uc0ac \ub2f9\uc2dc\uc758 \uc801\uaca9\ud310\uc815\uc744 \ub4a4\uc9d1\uc744 \uc815\ub3c4\ub294 \uc544\ub2c8\ub2e4\"\ub77c\uba70 \"\ub2f9\uc2dc\uc758 \ud2b9\uc218\ub300\ud559\uc6d0 \uc11d\uc0ac \ud559\uc704 \ub17c\ubb38\uc758 \uc77c\ubc18\uc801\uc778 \uad00\ud589\uacfc \ud559\ubb38\uc801 \uc131\ucde8\ub3c4 \uc218\uc900\uc5d0 \ube44\ucdb0 \ubcf4\ub354\ub77c\ub3c4, \uc190\uc0c9\uc774 \uc5c6\ub294 \ub17c\ubb38\uc73c\ub85c \ud310\ub2e8\ud55c\ub2e4\"\uace0 \ub9d0\ud588\ub2e4.", "paragraph_answer": "\uadf8\ub7ec\ub098 \ud589\uc815\ub300\ud559\uc6d0\uc740 \uac19\uc740 \ud574 5\uc6d4 27\uc77c \"\uc6d0\uc0dd\uc758 \ud76c\ub9dd\uc5d0 \uc758\ud574 \ud559\uc704\ub97c \ucde8\uc18c\ud558\uae30 \uc704\ud55c, \ud559\uce59\uc0c1 \uadfc\uac70\uac00 \ubd88\ubd84\uba85\ud558\ub2e4\"\uba70 \"\ubcf8\uc870\uc0ac\uc758 (\ud559\uc704 \ucde8\uc18c\ub97c \uc704\ud55c \ud655\uace0\ud55c \ud45c\uc808 \ud310\uc815) \uacb0\uacfc\ub97c \ub2ec\ub77c\"\uace0 \uc5f0\uad6c\uc724\ub9ac\uc704\uc5d0 \ubc18\uc1a1\ud558\uc600\uace0 \uacb0\uacfc\uc801\uc73c\ub85c \ub17c\ubb38\uc740 \uc720\ud6a8\ud55c \uc0c1\ud0dc\uac00 \ub410\ub2e4. \ub17c\ubb38\uc744 \uc9c0\ub3c4\ud55c \uc774\uc601\uade0 (\ud589\uc815\ud559\uacfc) \uad50\uc218\ub294 \uc77c\ubd80 \uc778\uc6a9\uacfc \ucd9c\ucc98\ub97c \uba85\uae30\ud558\uc9c0 \uc54a\uc558\ub2e4\ub294 \uc8fc\uc7a5\uc5d0 \ub300\ud574 \"\uc774 \ub17c\ubb38\uc758 \ud575\uc2ec\uc740 \ucd1d\uccb4\uc801\uc73c\ub85c \ubcc0\ud638\uc0ac\uc774\uace0 \uc2dc\ubbfc\ub2e8\uccb4 \ub9ac\ub354\uc600\ub358 \uc791\uc131\uc790\uc758 \uacbd\ud5d8\uacfc \ud604\uc7a5\uc790\ub8cc \uc218\uc9d1\uc73c\ub85c \uc791\uc131\ub41c \uac83\uc73c\ub85c, \ub17c\ubb38\uc2ec\uc0ac \ub2f9\uc2dc\uc758 \uc801\uaca9\ud310\uc815\uc744 \ub4a4\uc9d1\uc744 \uc815\ub3c4\ub294 \uc544\ub2c8\ub2e4\"\ub77c\uba70 \"\ub2f9\uc2dc\uc758 \ud2b9\uc218\ub300\ud559\uc6d0 \uc11d\uc0ac \ud559\uc704 \ub17c\ubb38\uc758 \uc77c\ubc18\uc801\uc778 \uad00\ud589\uacfc \ud559\ubb38\uc801 \uc131\ucde8\ub3c4 \uc218\uc900\uc5d0 \ube44\ucdb0 \ubcf4\ub354\ub77c\ub3c4, \uc190\uc0c9\uc774 \uc5c6\ub294 \ub17c\ubb38\uc73c\ub85c \ud310\ub2e8\ud55c\ub2e4\"\uace0 \ub9d0\ud588\ub2e4.", "sentence_answer": "\ub17c\ubb38\uc744 \uc9c0\ub3c4\ud55c \uc774\uc601\uade0 (\ud589\uc815\ud559\uacfc)", "paragraph_id": "6657517-2-0", "paragraph_question": "question: \ub17c\ubb38\uc744 \uc9c0\ub3c4\ud55c \uad50\uc218\ub294 \ub204\uad6c\uc778\uac00?, context: \uadf8\ub7ec\ub098 \ud589\uc815\ub300\ud559\uc6d0\uc740 \uac19\uc740 \ud574 5\uc6d4 27\uc77c \"\uc6d0\uc0dd\uc758 \ud76c\ub9dd\uc5d0 \uc758\ud574 \ud559\uc704\ub97c \ucde8\uc18c\ud558\uae30 \uc704\ud55c, \ud559\uce59\uc0c1 \uadfc\uac70\uac00 \ubd88\ubd84\uba85\ud558\ub2e4\"\uba70 \"\ubcf8\uc870\uc0ac\uc758 (\ud559\uc704 \ucde8\uc18c\ub97c \uc704\ud55c \ud655\uace0\ud55c \ud45c\uc808 \ud310\uc815) \uacb0\uacfc\ub97c \ub2ec\ub77c\"\uace0 \uc5f0\uad6c\uc724\ub9ac\uc704\uc5d0 \ubc18\uc1a1\ud558\uc600\uace0 \uacb0\uacfc\uc801\uc73c\ub85c \ub17c\ubb38\uc740 \uc720\ud6a8\ud55c \uc0c1\ud0dc\uac00 \ub410\ub2e4. \ub17c\ubb38\uc744 \uc9c0\ub3c4\ud55c \uc774\uc601\uade0(\ud589\uc815\ud559\uacfc) \uad50\uc218\ub294 \uc77c\ubd80 \uc778\uc6a9\uacfc \ucd9c\ucc98\ub97c \uba85\uae30\ud558\uc9c0 \uc54a\uc558\ub2e4\ub294 \uc8fc\uc7a5\uc5d0 \ub300\ud574 \"\uc774 \ub17c\ubb38\uc758 \ud575\uc2ec\uc740 \ucd1d\uccb4\uc801\uc73c\ub85c \ubcc0\ud638\uc0ac\uc774\uace0 \uc2dc\ubbfc\ub2e8\uccb4 \ub9ac\ub354\uc600\ub358 \uc791\uc131\uc790\uc758 \uacbd\ud5d8\uacfc \ud604\uc7a5\uc790\ub8cc \uc218\uc9d1\uc73c\ub85c \uc791\uc131\ub41c \uac83\uc73c\ub85c, \ub17c\ubb38\uc2ec\uc0ac \ub2f9\uc2dc\uc758 \uc801\uaca9\ud310\uc815\uc744 \ub4a4\uc9d1\uc744 \uc815\ub3c4\ub294 \uc544\ub2c8\ub2e4\"\ub77c\uba70 \"\ub2f9\uc2dc\uc758 \ud2b9\uc218\ub300\ud559\uc6d0 \uc11d\uc0ac \ud559\uc704 \ub17c\ubb38\uc758 \uc77c\ubc18\uc801\uc778 \uad00\ud589\uacfc \ud559\ubb38\uc801 \uc131\ucde8\ub3c4 \uc218\uc900\uc5d0 \ube44\ucdb0 \ubcf4\ub354\ub77c\ub3c4, \uc190\uc0c9\uc774 \uc5c6\ub294 \ub17c\ubb38\uc73c\ub85c \ud310\ub2e8\ud55c\ub2e4\"\uace0 \ub9d0\ud588\ub2e4."} {"question": "\uace0\uc591\uc774 \ubcc4\uc790\ub9ac\uc5d0\uc11c \ud2b9\ud788 \uad00\uacc4\ub418\uc5b4 \uc788\ub294 \uac83\uc740 \ubb34\uc5c7\uc778\uac00?", "paragraph": "\"In such signs the exaltations is regarded as especially related to a certain degree; but there are many differences of opinion in this matter, some saying that it extends to some degrees in front or behind the degree in question, while others hold that it extends from the first point of the sign to that degree, and again others that it is present in the whole sign without any special degree\".\u2192\"\uc774\uc640 \uac19\uc740 \ubcc4\uc790\ub9ac\uc5d0\uc11c, \uace0\uc591\uc740 \ud2b9\ud788 \ud2b9\uc815 \ub3c4\uc218\uc640 \uad00\uacc4\ub418\uc5b4 \uc788\ub2e4\uace0 \uc5ec\uaca8\uc9c4\ub2e4. \uadf8\ub7ec\ub098, \uadf8\ub7ec\ud55c \uac83\uc5d0\ub294 \uc758\uacac \ucc28\uc774\uac00 \ub9ce\uc740\ub370, \uc5b4\ub5a4 \uc0ac\ub78c\ub4e4\uc740 \uadf8\uac83\uc744 \ud574\ub2f9 \ub3c4\uc218\uc5d0\uc11c \uc55e\uc774\ub098 \ub4a4\ub85c \ud655\uc7a5\ud558\uace0, \ub2e4\ub978 \uc0ac\ub78c\ub4e4\uc740 \ubcc4\uc790\ub9ac\uc758 \uc2dc\uc791\uc810\uc5d0\uc11c\ubd80\ud130 \ud655\uc7a5\ud558\uba70, \ub610 \ub2e4\ub978 \uc0ac\ub78c\ub4e4\uc740 \ud2b9\uc815 \ub3c4\uc218\ub97c \uc9c0\uc815\ud558\uc9c0 \uc54a\uace0 \uc5b4\ub290 \ud55c \ubcc4\uc790\ub9ac\uc758 \uc804\uccb4\uac00 \uadf8\ub807\ub2e4\uace0 \ud55c\ub2e4.\"", "answer": "\ud2b9\uc815 \ub3c4\uc218", "sentence": "\"In such signs the exaltations is regarded as especially related to a certain degree; but there are many differences of opinion in this matter, some saying that it extends to some degrees in front or behind the degree in question, while others hold that it extends from the first point of the sign to that degree, and again others that it is present in the whole sign without any special degree\".\u2192\"\uc774\uc640 \uac19\uc740 \ubcc4\uc790\ub9ac\uc5d0\uc11c, \uace0\uc591\uc740 \ud2b9\ud788 \ud2b9\uc815 \ub3c4\uc218 \uc640 \uad00\uacc4\ub418\uc5b4 \uc788\ub2e4\uace0 \uc5ec\uaca8\uc9c4\ub2e4.", "paragraph_sentence": " \"In such signs the exaltations is regarded as especially related to a certain degree; but there are many differences of opinion in this matter, some saying that it extends to some degrees in front or behind the degree in question, while others hold that it extends from the first point of the sign to that degree, and again others that it is present in the whole sign without any special degree\".\u2192\"\uc774\uc640 \uac19\uc740 \ubcc4\uc790\ub9ac\uc5d0\uc11c, \uace0\uc591\uc740 \ud2b9\ud788 \ud2b9\uc815 \ub3c4\uc218 \uc640 \uad00\uacc4\ub418\uc5b4 \uc788\ub2e4\uace0 \uc5ec\uaca8\uc9c4\ub2e4. \uadf8\ub7ec\ub098, \uadf8\ub7ec\ud55c \uac83\uc5d0\ub294 \uc758\uacac \ucc28\uc774\uac00 \ub9ce\uc740\ub370, \uc5b4\ub5a4 \uc0ac\ub78c\ub4e4\uc740 \uadf8\uac83\uc744 \ud574\ub2f9 \ub3c4\uc218\uc5d0\uc11c \uc55e\uc774\ub098 \ub4a4\ub85c \ud655\uc7a5\ud558\uace0, \ub2e4\ub978 \uc0ac\ub78c\ub4e4\uc740 \ubcc4\uc790\ub9ac\uc758 \uc2dc\uc791\uc810\uc5d0\uc11c\ubd80\ud130 \ud655\uc7a5\ud558\uba70, \ub610 \ub2e4\ub978 \uc0ac\ub78c\ub4e4\uc740 \ud2b9\uc815 \ub3c4\uc218\ub97c \uc9c0\uc815\ud558\uc9c0 \uc54a\uace0 \uc5b4\ub290 \ud55c \ubcc4\uc790\ub9ac\uc758 \uc804\uccb4\uac00 \uadf8\ub807\ub2e4\uace0 \ud55c\ub2e4. \"", "paragraph_answer": "\"In such signs the exaltations is regarded as especially related to a certain degree; but there are many differences of opinion in this matter, some saying that it extends to some degrees in front or behind the degree in question, while others hold that it extends from the first point of the sign to that degree, and again others that it is present in the whole sign without any special degree\".\u2192\"\uc774\uc640 \uac19\uc740 \ubcc4\uc790\ub9ac\uc5d0\uc11c, \uace0\uc591\uc740 \ud2b9\ud788 \ud2b9\uc815 \ub3c4\uc218 \uc640 \uad00\uacc4\ub418\uc5b4 \uc788\ub2e4\uace0 \uc5ec\uaca8\uc9c4\ub2e4. \uadf8\ub7ec\ub098, \uadf8\ub7ec\ud55c \uac83\uc5d0\ub294 \uc758\uacac \ucc28\uc774\uac00 \ub9ce\uc740\ub370, \uc5b4\ub5a4 \uc0ac\ub78c\ub4e4\uc740 \uadf8\uac83\uc744 \ud574\ub2f9 \ub3c4\uc218\uc5d0\uc11c \uc55e\uc774\ub098 \ub4a4\ub85c \ud655\uc7a5\ud558\uace0, \ub2e4\ub978 \uc0ac\ub78c\ub4e4\uc740 \ubcc4\uc790\ub9ac\uc758 \uc2dc\uc791\uc810\uc5d0\uc11c\ubd80\ud130 \ud655\uc7a5\ud558\uba70, \ub610 \ub2e4\ub978 \uc0ac\ub78c\ub4e4\uc740 \ud2b9\uc815 \ub3c4\uc218\ub97c \uc9c0\uc815\ud558\uc9c0 \uc54a\uace0 \uc5b4\ub290 \ud55c \ubcc4\uc790\ub9ac\uc758 \uc804\uccb4\uac00 \uadf8\ub807\ub2e4\uace0 \ud55c\ub2e4.\"", "sentence_answer": "\"In such signs the exaltations is regarded as especially related to a certain degree; but there are many differences of opinion in this matter, some saying that it extends to some degrees in front or behind the degree in question, while others hold that it extends from the first point of the sign to that degree, and again others that it is present in the whole sign without any special degree\".\u2192\"\uc774\uc640 \uac19\uc740 \ubcc4\uc790\ub9ac\uc5d0\uc11c, \uace0\uc591\uc740 \ud2b9\ud788 \ud2b9\uc815 \ub3c4\uc218 \uc640 \uad00\uacc4\ub418\uc5b4 \uc788\ub2e4\uace0 \uc5ec\uaca8\uc9c4\ub2e4.", "paragraph_id": "6545987-3-0", "paragraph_question": "question: \uace0\uc591\uc774 \ubcc4\uc790\ub9ac\uc5d0\uc11c \ud2b9\ud788 \uad00\uacc4\ub418\uc5b4 \uc788\ub294 \uac83\uc740 \ubb34\uc5c7\uc778\uac00?, context: \"In such signs the exaltations is regarded as especially related to a certain degree; but there are many differences of opinion in this matter, some saying that it extends to some degrees in front or behind the degree in question, while others hold that it extends from the first point of the sign to that degree, and again others that it is present in the whole sign without any special degree\".\u2192\"\uc774\uc640 \uac19\uc740 \ubcc4\uc790\ub9ac\uc5d0\uc11c, \uace0\uc591\uc740 \ud2b9\ud788 \ud2b9\uc815 \ub3c4\uc218\uc640 \uad00\uacc4\ub418\uc5b4 \uc788\ub2e4\uace0 \uc5ec\uaca8\uc9c4\ub2e4. \uadf8\ub7ec\ub098, \uadf8\ub7ec\ud55c \uac83\uc5d0\ub294 \uc758\uacac \ucc28\uc774\uac00 \ub9ce\uc740\ub370, \uc5b4\ub5a4 \uc0ac\ub78c\ub4e4\uc740 \uadf8\uac83\uc744 \ud574\ub2f9 \ub3c4\uc218\uc5d0\uc11c \uc55e\uc774\ub098 \ub4a4\ub85c \ud655\uc7a5\ud558\uace0, \ub2e4\ub978 \uc0ac\ub78c\ub4e4\uc740 \ubcc4\uc790\ub9ac\uc758 \uc2dc\uc791\uc810\uc5d0\uc11c\ubd80\ud130 \ud655\uc7a5\ud558\uba70, \ub610 \ub2e4\ub978 \uc0ac\ub78c\ub4e4\uc740 \ud2b9\uc815 \ub3c4\uc218\ub97c \uc9c0\uc815\ud558\uc9c0 \uc54a\uace0 \uc5b4\ub290 \ud55c \ubcc4\uc790\ub9ac\uc758 \uc804\uccb4\uac00 \uadf8\ub807\ub2e4\uace0 \ud55c\ub2e4.\""} {"question": "1899\ub144 \uc548\uc0b0\uc5d0 \uac1c\uad50\ud55c \ucd5c\ucd08\uc758 \uadfc\ub300\uc2dd \uad50\uc721\uc2dc\uc124\uc740?", "paragraph": "\ucd5c\ucd08\uc758 \uadfc\ub300\uc2dd \uad50\uc721 \uc2dc\uc124\ub85c\uc368\ub294 1899\ub144 9\uc6d4 \ub300\ud55c\uc81c\uad6d\uc2dc\uae30\uc5d0 \uc548\uc0b0\uad70\uacf5\ub9bd\uc18c\ud559\uad50(\ud604 \uc548\uc0b0\ucd08\ub4f1\ud559\uad50)\uac00 \uac1c\uad50\ub418\uc5c8\ub2e4. \uc774 \ud559\uad50\ub294 \ud604\uc7ac\uc758 \uc548\uc0b0\uc2dc\u00b7\uc2dc\ud765\uc2dc\u00b7\uad11\uba85\uc2dc\u00b7\uad70\ud3ec\uc2dc\u00b7\uc758\uc655\uc2dc\u00b7\uacfc\ucc9c\uc2dc\u00b7\uc548\uc591\uc2dc \ub4f1\uc774 \ud3ec\ud568\ub41c \uacbd\uae30\ub3c4 \uc11c\ub0a8\ubd80\uc9c0\uc5ed\uc758 \uacf5\ub9bd\uc18c\ud559\uad50\ub85c\uc368 \uac00\uc7a5 \uba3c\uc800 \uc0dd\uae34 \uadfc\ub300\uc2dd \uad50\uc721\uae30\uad00\uc774\ub2e4. \uc774\ud6c4 \uc548\uc0b0 \uc9c0\uc5ed\uc5d0\ub294 \uc77c\uc81c\uc758 \uc870\uc120 \ucd1d\ub3c5\ubd80\uc758 \uc2dd\ubbfc \uc815\ucc45\uc5d0 \ub530\ub77c\uc11c 1922\ub144\uc5d0\ub294 \ub300\ubd80\uacf5\ub9bd\ubcf4\ud1b5\ud559\uad50\ub97c \uc2dc\uc791\uc73c\ub85c 1923\ub144 5\uc6d4\uc5d0\ub294 \ubc18\uc6d4\uacf5\ub9bd\ubcf4\ud1b5\ud559\uad50, 1923\ub144 7\uc6d4\uc5d0\ub294 \uad70\uc790\uacf5\ub9bd\uc2ec\uc0c1\uc18c\ud559\uad50, 12\uc6d4\uc5d0\ub294 \uad70\uc790\uacf5\ub9bd\ubcf4\ud1b5\ud559\uad50\uac00 \uac01\uac01 \uc138\uc6cc\uc84c\ub2e4. \uad11\ubcf5 \ud6c4\uc5d0\ub294 \ub2f9\uc2dc \uc548\uc0b0 \uc9c0\uc5ed\uc758 \ucd08\ub4f1\ud559\uad50 \uc218\ub294 7\uac1c \ud559\uad50\uc600\uc73c\uba70, \ubbf8\uad70\uc815\uae30\uc5d0\ub294 \uad70\uc11c\uacf5\ub9bd\uad6d\ubbfc\ud559\uad50\uc640 \ud654\uc815\uacf5\ub9bd\uad6d\ubbfc\ud559\uad50\uc758 2\uac1c \ud559\uad50\uac00 \ucd94\uac00\ub85c \uc138\uc6cc\uc84c\ub2e4. \ud604\ub300 \uc2dc\ub300\uc5d0\ub294 \uc548\uc0b0\uc9c0\uc5ed\uc758 \ub3c4\uc2dc \ubc1c\uc804\uacfc \ud568\uaed8 \ub9ce\uc740 \uc218\uc758 \ucd08\uc911\uace0 \ud559\uad50\ub4e4\uc774 \uc124\ub9bd\ub418\uc5c8\ub294\ub370, 2009\ub144 3\uc6d4 \uae30\uc900\uc73c\ub85c \uacbd\uae30\ub3c4 \uc548\uc0b0\uc2dc\uc5d0 \uc788\ub294 \ucd08\ub4f1\ud559\uad50\ub294 \ucd1d 53\uac1c\uad50\uc774\uba70, \uc804\uccb4 1,832\ud559\uae09\uc5d0 \ucd1d 61,302\uba85\uc758 \ud559\uc0dd \uc218\ub97c \uae30\ub85d\ud588\ub2e4.", "answer": "\uc548\uc0b0\uad70\uacf5\ub9bd\uc18c\ud559\uad50", "sentence": "\ucd5c\ucd08\uc758 \uadfc\ub300\uc2dd \uad50\uc721 \uc2dc\uc124\ub85c\uc368\ub294 1899\ub144 9\uc6d4 \ub300\ud55c\uc81c\uad6d\uc2dc\uae30\uc5d0 \uc548\uc0b0\uad70\uacf5\ub9bd\uc18c\ud559\uad50 (\ud604 \uc548\uc0b0\ucd08\ub4f1\ud559\uad50)\uac00 \uac1c\uad50\ub418\uc5c8\ub2e4.", "paragraph_sentence": " \ucd5c\ucd08\uc758 \uadfc\ub300\uc2dd \uad50\uc721 \uc2dc\uc124\ub85c\uc368\ub294 1899\ub144 9\uc6d4 \ub300\ud55c\uc81c\uad6d\uc2dc\uae30\uc5d0 \uc548\uc0b0\uad70\uacf5\ub9bd\uc18c\ud559\uad50 (\ud604 \uc548\uc0b0\ucd08\ub4f1\ud559\uad50)\uac00 \uac1c\uad50\ub418\uc5c8\ub2e4. \uc774 \ud559\uad50\ub294 \ud604\uc7ac\uc758 \uc548\uc0b0\uc2dc\u00b7\uc2dc\ud765\uc2dc\u00b7\uad11\uba85\uc2dc\u00b7\uad70\ud3ec\uc2dc\u00b7\uc758\uc655\uc2dc\u00b7\uacfc\ucc9c\uc2dc\u00b7\uc548\uc591\uc2dc \ub4f1\uc774 \ud3ec\ud568\ub41c \uacbd\uae30\ub3c4 \uc11c\ub0a8\ubd80\uc9c0\uc5ed\uc758 \uacf5\ub9bd\uc18c\ud559\uad50\ub85c\uc368 \uac00\uc7a5 \uba3c\uc800 \uc0dd\uae34 \uadfc\ub300\uc2dd \uad50\uc721\uae30\uad00\uc774\ub2e4. \uc774\ud6c4 \uc548\uc0b0 \uc9c0\uc5ed\uc5d0\ub294 \uc77c\uc81c\uc758 \uc870\uc120 \ucd1d\ub3c5\ubd80\uc758 \uc2dd\ubbfc \uc815\ucc45\uc5d0 \ub530\ub77c\uc11c 1922\ub144\uc5d0\ub294 \ub300\ubd80\uacf5\ub9bd\ubcf4\ud1b5\ud559\uad50\ub97c \uc2dc\uc791\uc73c\ub85c 1923\ub144 5\uc6d4\uc5d0\ub294 \ubc18\uc6d4\uacf5\ub9bd\ubcf4\ud1b5\ud559\uad50, 1923\ub144 7\uc6d4\uc5d0\ub294 \uad70\uc790\uacf5\ub9bd\uc2ec\uc0c1\uc18c\ud559\uad50, 12\uc6d4\uc5d0\ub294 \uad70\uc790\uacf5\ub9bd\ubcf4\ud1b5\ud559\uad50\uac00 \uac01\uac01 \uc138\uc6cc\uc84c\ub2e4. \uad11\ubcf5 \ud6c4\uc5d0\ub294 \ub2f9\uc2dc \uc548\uc0b0 \uc9c0\uc5ed\uc758 \ucd08\ub4f1\ud559\uad50 \uc218\ub294 7\uac1c \ud559\uad50\uc600\uc73c\uba70, \ubbf8\uad70\uc815\uae30\uc5d0\ub294 \uad70\uc11c\uacf5\ub9bd\uad6d\ubbfc\ud559\uad50\uc640 \ud654\uc815\uacf5\ub9bd\uad6d\ubbfc\ud559\uad50\uc758 2\uac1c \ud559\uad50\uac00 \ucd94\uac00\ub85c \uc138\uc6cc\uc84c\ub2e4. \ud604\ub300 \uc2dc\ub300\uc5d0\ub294 \uc548\uc0b0\uc9c0\uc5ed\uc758 \ub3c4\uc2dc \ubc1c\uc804\uacfc \ud568\uaed8 \ub9ce\uc740 \uc218\uc758 \ucd08\uc911\uace0 \ud559\uad50\ub4e4\uc774 \uc124\ub9bd\ub418\uc5c8\ub294\ub370, 2009\ub144 3\uc6d4 \uae30\uc900\uc73c\ub85c \uacbd\uae30\ub3c4 \uc548\uc0b0\uc2dc\uc5d0 \uc788\ub294 \ucd08\ub4f1\ud559\uad50\ub294 \ucd1d 53\uac1c\uad50\uc774\uba70, \uc804\uccb4 1,832\ud559\uae09\uc5d0 \ucd1d 61,302\uba85\uc758 \ud559\uc0dd \uc218\ub97c \uae30\ub85d\ud588\ub2e4.", "paragraph_answer": "\ucd5c\ucd08\uc758 \uadfc\ub300\uc2dd \uad50\uc721 \uc2dc\uc124\ub85c\uc368\ub294 1899\ub144 9\uc6d4 \ub300\ud55c\uc81c\uad6d\uc2dc\uae30\uc5d0 \uc548\uc0b0\uad70\uacf5\ub9bd\uc18c\ud559\uad50 (\ud604 \uc548\uc0b0\ucd08\ub4f1\ud559\uad50)\uac00 \uac1c\uad50\ub418\uc5c8\ub2e4. \uc774 \ud559\uad50\ub294 \ud604\uc7ac\uc758 \uc548\uc0b0\uc2dc\u00b7\uc2dc\ud765\uc2dc\u00b7\uad11\uba85\uc2dc\u00b7\uad70\ud3ec\uc2dc\u00b7\uc758\uc655\uc2dc\u00b7\uacfc\ucc9c\uc2dc\u00b7\uc548\uc591\uc2dc \ub4f1\uc774 \ud3ec\ud568\ub41c \uacbd\uae30\ub3c4 \uc11c\ub0a8\ubd80\uc9c0\uc5ed\uc758 \uacf5\ub9bd\uc18c\ud559\uad50\ub85c\uc368 \uac00\uc7a5 \uba3c\uc800 \uc0dd\uae34 \uadfc\ub300\uc2dd \uad50\uc721\uae30\uad00\uc774\ub2e4. \uc774\ud6c4 \uc548\uc0b0 \uc9c0\uc5ed\uc5d0\ub294 \uc77c\uc81c\uc758 \uc870\uc120 \ucd1d\ub3c5\ubd80\uc758 \uc2dd\ubbfc \uc815\ucc45\uc5d0 \ub530\ub77c\uc11c 1922\ub144\uc5d0\ub294 \ub300\ubd80\uacf5\ub9bd\ubcf4\ud1b5\ud559\uad50\ub97c \uc2dc\uc791\uc73c\ub85c 1923\ub144 5\uc6d4\uc5d0\ub294 \ubc18\uc6d4\uacf5\ub9bd\ubcf4\ud1b5\ud559\uad50, 1923\ub144 7\uc6d4\uc5d0\ub294 \uad70\uc790\uacf5\ub9bd\uc2ec\uc0c1\uc18c\ud559\uad50, 12\uc6d4\uc5d0\ub294 \uad70\uc790\uacf5\ub9bd\ubcf4\ud1b5\ud559\uad50\uac00 \uac01\uac01 \uc138\uc6cc\uc84c\ub2e4. \uad11\ubcf5 \ud6c4\uc5d0\ub294 \ub2f9\uc2dc \uc548\uc0b0 \uc9c0\uc5ed\uc758 \ucd08\ub4f1\ud559\uad50 \uc218\ub294 7\uac1c \ud559\uad50\uc600\uc73c\uba70, \ubbf8\uad70\uc815\uae30\uc5d0\ub294 \uad70\uc11c\uacf5\ub9bd\uad6d\ubbfc\ud559\uad50\uc640 \ud654\uc815\uacf5\ub9bd\uad6d\ubbfc\ud559\uad50\uc758 2\uac1c \ud559\uad50\uac00 \ucd94\uac00\ub85c \uc138\uc6cc\uc84c\ub2e4. \ud604\ub300 \uc2dc\ub300\uc5d0\ub294 \uc548\uc0b0\uc9c0\uc5ed\uc758 \ub3c4\uc2dc \ubc1c\uc804\uacfc \ud568\uaed8 \ub9ce\uc740 \uc218\uc758 \ucd08\uc911\uace0 \ud559\uad50\ub4e4\uc774 \uc124\ub9bd\ub418\uc5c8\ub294\ub370, 2009\ub144 3\uc6d4 \uae30\uc900\uc73c\ub85c \uacbd\uae30\ub3c4 \uc548\uc0b0\uc2dc\uc5d0 \uc788\ub294 \ucd08\ub4f1\ud559\uad50\ub294 \ucd1d 53\uac1c\uad50\uc774\uba70, \uc804\uccb4 1,832\ud559\uae09\uc5d0 \ucd1d 61,302\uba85\uc758 \ud559\uc0dd \uc218\ub97c \uae30\ub85d\ud588\ub2e4.", "sentence_answer": "\ucd5c\ucd08\uc758 \uadfc\ub300\uc2dd \uad50\uc721 \uc2dc\uc124\ub85c\uc368\ub294 1899\ub144 9\uc6d4 \ub300\ud55c\uc81c\uad6d\uc2dc\uae30\uc5d0 \uc548\uc0b0\uad70\uacf5\ub9bd\uc18c\ud559\uad50 (\ud604 \uc548\uc0b0\ucd08\ub4f1\ud559\uad50)\uac00 \uac1c\uad50\ub418\uc5c8\ub2e4.", "paragraph_id": "6538931-11-0", "paragraph_question": "question: 1899\ub144 \uc548\uc0b0\uc5d0 \uac1c\uad50\ud55c \ucd5c\ucd08\uc758 \uadfc\ub300\uc2dd \uad50\uc721\uc2dc\uc124\uc740?, context: \ucd5c\ucd08\uc758 \uadfc\ub300\uc2dd \uad50\uc721 \uc2dc\uc124\ub85c\uc368\ub294 1899\ub144 9\uc6d4 \ub300\ud55c\uc81c\uad6d\uc2dc\uae30\uc5d0 \uc548\uc0b0\uad70\uacf5\ub9bd\uc18c\ud559\uad50(\ud604 \uc548\uc0b0\ucd08\ub4f1\ud559\uad50)\uac00 \uac1c\uad50\ub418\uc5c8\ub2e4. \uc774 \ud559\uad50\ub294 \ud604\uc7ac\uc758 \uc548\uc0b0\uc2dc\u00b7\uc2dc\ud765\uc2dc\u00b7\uad11\uba85\uc2dc\u00b7\uad70\ud3ec\uc2dc\u00b7\uc758\uc655\uc2dc\u00b7\uacfc\ucc9c\uc2dc\u00b7\uc548\uc591\uc2dc \ub4f1\uc774 \ud3ec\ud568\ub41c \uacbd\uae30\ub3c4 \uc11c\ub0a8\ubd80\uc9c0\uc5ed\uc758 \uacf5\ub9bd\uc18c\ud559\uad50\ub85c\uc368 \uac00\uc7a5 \uba3c\uc800 \uc0dd\uae34 \uadfc\ub300\uc2dd \uad50\uc721\uae30\uad00\uc774\ub2e4. \uc774\ud6c4 \uc548\uc0b0 \uc9c0\uc5ed\uc5d0\ub294 \uc77c\uc81c\uc758 \uc870\uc120 \ucd1d\ub3c5\ubd80\uc758 \uc2dd\ubbfc \uc815\ucc45\uc5d0 \ub530\ub77c\uc11c 1922\ub144\uc5d0\ub294 \ub300\ubd80\uacf5\ub9bd\ubcf4\ud1b5\ud559\uad50\ub97c \uc2dc\uc791\uc73c\ub85c 1923\ub144 5\uc6d4\uc5d0\ub294 \ubc18\uc6d4\uacf5\ub9bd\ubcf4\ud1b5\ud559\uad50, 1923\ub144 7\uc6d4\uc5d0\ub294 \uad70\uc790\uacf5\ub9bd\uc2ec\uc0c1\uc18c\ud559\uad50, 12\uc6d4\uc5d0\ub294 \uad70\uc790\uacf5\ub9bd\ubcf4\ud1b5\ud559\uad50\uac00 \uac01\uac01 \uc138\uc6cc\uc84c\ub2e4. \uad11\ubcf5 \ud6c4\uc5d0\ub294 \ub2f9\uc2dc \uc548\uc0b0 \uc9c0\uc5ed\uc758 \ucd08\ub4f1\ud559\uad50 \uc218\ub294 7\uac1c \ud559\uad50\uc600\uc73c\uba70, \ubbf8\uad70\uc815\uae30\uc5d0\ub294 \uad70\uc11c\uacf5\ub9bd\uad6d\ubbfc\ud559\uad50\uc640 \ud654\uc815\uacf5\ub9bd\uad6d\ubbfc\ud559\uad50\uc758 2\uac1c \ud559\uad50\uac00 \ucd94\uac00\ub85c \uc138\uc6cc\uc84c\ub2e4. \ud604\ub300 \uc2dc\ub300\uc5d0\ub294 \uc548\uc0b0\uc9c0\uc5ed\uc758 \ub3c4\uc2dc \ubc1c\uc804\uacfc \ud568\uaed8 \ub9ce\uc740 \uc218\uc758 \ucd08\uc911\uace0 \ud559\uad50\ub4e4\uc774 \uc124\ub9bd\ub418\uc5c8\ub294\ub370, 2009\ub144 3\uc6d4 \uae30\uc900\uc73c\ub85c \uacbd\uae30\ub3c4 \uc548\uc0b0\uc2dc\uc5d0 \uc788\ub294 \ucd08\ub4f1\ud559\uad50\ub294 \ucd1d 53\uac1c\uad50\uc774\uba70, \uc804\uccb4 1,832\ud559\uae09\uc5d0 \ucd1d 61,302\uba85\uc758 \ud559\uc0dd \uc218\ub97c \uae30\ub85d\ud588\ub2e4."} {"question": "2003\ub144 10\uc6d4 27\uc77c \uc5d4\ud1a0\uc774\uc758 \ubcc0\uacbd\ub41c \uc815\ucc45\uc5d0\uc11c \ub9e4\uc77c \uccab\ud68c \ub85c\uadf8\uc778\uc2dc \ubcf4\ub108\uc2a4\ub85c \uc81c\uacf5\ud558\uae30\ub85c \ud55c \uc6b0\uc720\uc758 \uac2f\uc218\ub294?", "paragraph": "\uacb0\uad6d NHN\uc740 2003\ub144 10\uc6d4 \ub9d0 \uc815\ucc45\uc744 \uc804\uba74 \ubb34\ub8cc\ud654\ud558\ub294 \ub300\ucc45\uc744 \ub0b4\ub193\uc558\ub2e4. 10\uc6d4 12\uc77c \u2018\ubb34\ud55c \ubb34\ub8cc \ub300\ubcc0\uc2e0\u2019\uc744 \uc120\uc5b8\ud574 10\uc6d4 15\uc77c\ubd80\ud130 \ube14\ub85c\uadf8 \uc6a9\ub7c9\uc744 \ubb34\uc81c\ud55c\uc73c\ub85c \uc81c\uacf5\ud558\uace0, \uae30\uc874\uc5d0 \uc720\ub8cc\uc11c\ube44\uc2a4\ub85c \uc6b4\uc601\ub410\ub358 \uc6f9\ud3f4\ub354\uc640 \ubba4\ube44\ub9cc\ub4e4\uae30\ub97c \ubb34\ub8cc\ub85c \uc11c\ube44\uc2a4\ud558\uba70, \ube14\ub85c\uadf8 \ubc14\ud0d5\ud654\uba74\uc5d0 \ub4e4\uc5b4\uac00\ub294 \uc544\uc774\ud15c\ub3c4 \ubb34\ub8cc\ub85c \uc81c\uacf5\ud558\ub294 \uc774\ubca4\ud2b8\ub97c \uc2e4\uc2dc\ud55c\ub2e4\uace0 \ubc1d\ud614\ub2e4. 10\uc6d4 27\uc77c\uc5d0\ub294 \uc6b0\uc720 \uc815\ucc45\uc744 \ub300\ud3ed \ubcc0\uacbd\ud574, \ub9e4\uc77c \uccab\ud68c \ub85c\uadf8\uc778\uc2dc \uc6b0\uc720 100cc\ub97c \ubcf4\ub108\uc2a4\ub85c \uc81c\uacf5\ud558\uace0, \uc2a4\ud0c0\uc758 \ube14\ub85c\uadf8\ub97c \ubc29\ubb38\ud560 \ub54c \ud32c\uc774 \ub0b4\ub358 \uc6b0\uc720\ub97c \uc5d4\ud1a0\uc774\uce21\uc774 \ub300\uc2e0 \uc9c0\uae09\ud558\uae30\ub85c \ud558\uc5ec, \uc2a4\ud0c0-\ud32c \uad00\uacc4\uc758 \ud65c\uc131\ud654\ub97c \uaf80\ud588\ub2e4. 2003\ub144 12\uc6d4 9\uc77c \ub124\uc774\ubc84 \ud3ec\ud138 UI \uac1c\ud3b8 \uc2dc\uc5d0\ub294 \ud3ec\ud138 \uba54\uc778\uc5d0 \uc5d4\ud1a0\uc774 \ubc14\ub85c\uac00\uae30\ub97c \ucd94\uac00\ud558\uae30\ub3c4 \ud588\ub2e4.", "answer": "100cc", "sentence": "10\uc6d4 27\uc77c\uc5d0\ub294 \uc6b0\uc720 \uc815\ucc45\uc744 \ub300\ud3ed \ubcc0\uacbd\ud574, \ub9e4\uc77c \uccab\ud68c \ub85c\uadf8\uc778\uc2dc \uc6b0\uc720 100cc \ub97c \ubcf4\ub108\uc2a4\ub85c \uc81c\uacf5\ud558\uace0, \uc2a4\ud0c0\uc758 \ube14\ub85c\uadf8\ub97c \ubc29\ubb38\ud560 \ub54c \ud32c\uc774 \ub0b4\ub358 \uc6b0\uc720\ub97c \uc5d4\ud1a0\uc774\uce21\uc774 \ub300\uc2e0 \uc9c0\uae09\ud558\uae30\ub85c \ud558\uc5ec, \uc2a4\ud0c0-\ud32c \uad00\uacc4\uc758 \ud65c\uc131\ud654\ub97c \uaf80\ud588\ub2e4.", "paragraph_sentence": "\uacb0\uad6d NHN\uc740 2003\ub144 10\uc6d4 \ub9d0 \uc815\ucc45\uc744 \uc804\uba74 \ubb34\ub8cc\ud654\ud558\ub294 \ub300\ucc45\uc744 \ub0b4\ub193\uc558\ub2e4. 10\uc6d4 12\uc77c \u2018\ubb34\ud55c \ubb34\ub8cc \ub300\ubcc0\uc2e0\u2019\uc744 \uc120\uc5b8\ud574 10\uc6d4 15\uc77c\ubd80\ud130 \ube14\ub85c\uadf8 \uc6a9\ub7c9\uc744 \ubb34\uc81c\ud55c\uc73c\ub85c \uc81c\uacf5\ud558\uace0, \uae30\uc874\uc5d0 \uc720\ub8cc\uc11c\ube44\uc2a4\ub85c \uc6b4\uc601\ub410\ub358 \uc6f9\ud3f4\ub354\uc640 \ubba4\ube44\ub9cc\ub4e4\uae30\ub97c \ubb34\ub8cc\ub85c \uc11c\ube44\uc2a4\ud558\uba70, \ube14\ub85c\uadf8 \ubc14\ud0d5\ud654\uba74\uc5d0 \ub4e4\uc5b4\uac00\ub294 \uc544\uc774\ud15c\ub3c4 \ubb34\ub8cc\ub85c \uc81c\uacf5\ud558\ub294 \uc774\ubca4\ud2b8\ub97c \uc2e4\uc2dc\ud55c\ub2e4\uace0 \ubc1d\ud614\ub2e4. 10\uc6d4 27\uc77c\uc5d0\ub294 \uc6b0\uc720 \uc815\ucc45\uc744 \ub300\ud3ed \ubcc0\uacbd\ud574, \ub9e4\uc77c \uccab\ud68c \ub85c\uadf8\uc778\uc2dc \uc6b0\uc720 100cc \ub97c \ubcf4\ub108\uc2a4\ub85c \uc81c\uacf5\ud558\uace0, \uc2a4\ud0c0\uc758 \ube14\ub85c\uadf8\ub97c \ubc29\ubb38\ud560 \ub54c \ud32c\uc774 \ub0b4\ub358 \uc6b0\uc720\ub97c \uc5d4\ud1a0\uc774\uce21\uc774 \ub300\uc2e0 \uc9c0\uae09\ud558\uae30\ub85c \ud558\uc5ec, \uc2a4\ud0c0-\ud32c \uad00\uacc4\uc758 \ud65c\uc131\ud654\ub97c \uaf80\ud588\ub2e4. 2003\ub144 12\uc6d4 9\uc77c \ub124\uc774\ubc84 \ud3ec\ud138 UI \uac1c\ud3b8 \uc2dc\uc5d0\ub294 \ud3ec\ud138 \uba54\uc778\uc5d0 \uc5d4\ud1a0\uc774 \ubc14\ub85c\uac00\uae30\ub97c \ucd94\uac00\ud558\uae30\ub3c4 \ud588\ub2e4.", "paragraph_answer": "\uacb0\uad6d NHN\uc740 2003\ub144 10\uc6d4 \ub9d0 \uc815\ucc45\uc744 \uc804\uba74 \ubb34\ub8cc\ud654\ud558\ub294 \ub300\ucc45\uc744 \ub0b4\ub193\uc558\ub2e4. 10\uc6d4 12\uc77c \u2018\ubb34\ud55c \ubb34\ub8cc \ub300\ubcc0\uc2e0\u2019\uc744 \uc120\uc5b8\ud574 10\uc6d4 15\uc77c\ubd80\ud130 \ube14\ub85c\uadf8 \uc6a9\ub7c9\uc744 \ubb34\uc81c\ud55c\uc73c\ub85c \uc81c\uacf5\ud558\uace0, \uae30\uc874\uc5d0 \uc720\ub8cc\uc11c\ube44\uc2a4\ub85c \uc6b4\uc601\ub410\ub358 \uc6f9\ud3f4\ub354\uc640 \ubba4\ube44\ub9cc\ub4e4\uae30\ub97c \ubb34\ub8cc\ub85c \uc11c\ube44\uc2a4\ud558\uba70, \ube14\ub85c\uadf8 \ubc14\ud0d5\ud654\uba74\uc5d0 \ub4e4\uc5b4\uac00\ub294 \uc544\uc774\ud15c\ub3c4 \ubb34\ub8cc\ub85c \uc81c\uacf5\ud558\ub294 \uc774\ubca4\ud2b8\ub97c \uc2e4\uc2dc\ud55c\ub2e4\uace0 \ubc1d\ud614\ub2e4. 10\uc6d4 27\uc77c\uc5d0\ub294 \uc6b0\uc720 \uc815\ucc45\uc744 \ub300\ud3ed \ubcc0\uacbd\ud574, \ub9e4\uc77c \uccab\ud68c \ub85c\uadf8\uc778\uc2dc \uc6b0\uc720 100cc \ub97c \ubcf4\ub108\uc2a4\ub85c \uc81c\uacf5\ud558\uace0, \uc2a4\ud0c0\uc758 \ube14\ub85c\uadf8\ub97c \ubc29\ubb38\ud560 \ub54c \ud32c\uc774 \ub0b4\ub358 \uc6b0\uc720\ub97c \uc5d4\ud1a0\uc774\uce21\uc774 \ub300\uc2e0 \uc9c0\uae09\ud558\uae30\ub85c \ud558\uc5ec, \uc2a4\ud0c0-\ud32c \uad00\uacc4\uc758 \ud65c\uc131\ud654\ub97c \uaf80\ud588\ub2e4. 2003\ub144 12\uc6d4 9\uc77c \ub124\uc774\ubc84 \ud3ec\ud138 UI \uac1c\ud3b8 \uc2dc\uc5d0\ub294 \ud3ec\ud138 \uba54\uc778\uc5d0 \uc5d4\ud1a0\uc774 \ubc14\ub85c\uac00\uae30\ub97c \ucd94\uac00\ud558\uae30\ub3c4 \ud588\ub2e4.", "sentence_answer": "10\uc6d4 27\uc77c\uc5d0\ub294 \uc6b0\uc720 \uc815\ucc45\uc744 \ub300\ud3ed \ubcc0\uacbd\ud574, \ub9e4\uc77c \uccab\ud68c \ub85c\uadf8\uc778\uc2dc \uc6b0\uc720 100cc \ub97c \ubcf4\ub108\uc2a4\ub85c \uc81c\uacf5\ud558\uace0, \uc2a4\ud0c0\uc758 \ube14\ub85c\uadf8\ub97c \ubc29\ubb38\ud560 \ub54c \ud32c\uc774 \ub0b4\ub358 \uc6b0\uc720\ub97c \uc5d4\ud1a0\uc774\uce21\uc774 \ub300\uc2e0 \uc9c0\uae09\ud558\uae30\ub85c \ud558\uc5ec, \uc2a4\ud0c0-\ud32c \uad00\uacc4\uc758 \ud65c\uc131\ud654\ub97c \uaf80\ud588\ub2e4.", "paragraph_id": "6586998-2-1", "paragraph_question": "question: 2003\ub144 10\uc6d4 27\uc77c \uc5d4\ud1a0\uc774\uc758 \ubcc0\uacbd\ub41c \uc815\ucc45\uc5d0\uc11c \ub9e4\uc77c \uccab\ud68c \ub85c\uadf8\uc778\uc2dc \ubcf4\ub108\uc2a4\ub85c \uc81c\uacf5\ud558\uae30\ub85c \ud55c \uc6b0\uc720\uc758 \uac2f\uc218\ub294?, context: \uacb0\uad6d NHN\uc740 2003\ub144 10\uc6d4 \ub9d0 \uc815\ucc45\uc744 \uc804\uba74 \ubb34\ub8cc\ud654\ud558\ub294 \ub300\ucc45\uc744 \ub0b4\ub193\uc558\ub2e4. 10\uc6d4 12\uc77c \u2018\ubb34\ud55c \ubb34\ub8cc \ub300\ubcc0\uc2e0\u2019\uc744 \uc120\uc5b8\ud574 10\uc6d4 15\uc77c\ubd80\ud130 \ube14\ub85c\uadf8 \uc6a9\ub7c9\uc744 \ubb34\uc81c\ud55c\uc73c\ub85c \uc81c\uacf5\ud558\uace0, \uae30\uc874\uc5d0 \uc720\ub8cc\uc11c\ube44\uc2a4\ub85c \uc6b4\uc601\ub410\ub358 \uc6f9\ud3f4\ub354\uc640 \ubba4\ube44\ub9cc\ub4e4\uae30\ub97c \ubb34\ub8cc\ub85c \uc11c\ube44\uc2a4\ud558\uba70, \ube14\ub85c\uadf8 \ubc14\ud0d5\ud654\uba74\uc5d0 \ub4e4\uc5b4\uac00\ub294 \uc544\uc774\ud15c\ub3c4 \ubb34\ub8cc\ub85c \uc81c\uacf5\ud558\ub294 \uc774\ubca4\ud2b8\ub97c \uc2e4\uc2dc\ud55c\ub2e4\uace0 \ubc1d\ud614\ub2e4. 10\uc6d4 27\uc77c\uc5d0\ub294 \uc6b0\uc720 \uc815\ucc45\uc744 \ub300\ud3ed \ubcc0\uacbd\ud574, \ub9e4\uc77c \uccab\ud68c \ub85c\uadf8\uc778\uc2dc \uc6b0\uc720 100cc\ub97c \ubcf4\ub108\uc2a4\ub85c \uc81c\uacf5\ud558\uace0, \uc2a4\ud0c0\uc758 \ube14\ub85c\uadf8\ub97c \ubc29\ubb38\ud560 \ub54c \ud32c\uc774 \ub0b4\ub358 \uc6b0\uc720\ub97c \uc5d4\ud1a0\uc774\uce21\uc774 \ub300\uc2e0 \uc9c0\uae09\ud558\uae30\ub85c \ud558\uc5ec, \uc2a4\ud0c0-\ud32c \uad00\uacc4\uc758 \ud65c\uc131\ud654\ub97c \uaf80\ud588\ub2e4. 2003\ub144 12\uc6d4 9\uc77c \ub124\uc774\ubc84 \ud3ec\ud138 UI \uac1c\ud3b8 \uc2dc\uc5d0\ub294 \ud3ec\ud138 \uba54\uc778\uc5d0 \uc5d4\ud1a0\uc774 \ubc14\ub85c\uac00\uae30\ub97c \ucd94\uac00\ud558\uae30\ub3c4 \ud588\ub2e4."} {"question": "\ud1a0\uce74\ub9c9 \ub0b4\ubd80 \ud50c\ub77c\uc988\ub9c8\uc5d0 \ud750\ub974\ub294 \uac83\uc740?", "paragraph": "\uc55e\uc5d0\uc11c \uc774\uc57c\uae30\ud558\uc600\ub4ef\uc774 \ud1a0\uce74\ub9c9 \ub0b4\ubd80 \ud50c\ub77c\uc988\ub9c8\uc5d0\ub294 \uc804\ub958\uac00 \ud750\ub978\ub2e4. \ud50c\ub77c\uc988\ub9c8 \uc0c1\ud0dc\ub294 \ub3c4\uccb4 \uc0c1\ud0dc\uc774\ubbc0\ub85c \uc804\ub958\uac00 \ud750\ub974\ub294 \uac83\uc5d0\ub294 \ubb38\uc81c\uac00 \uc5c6\ub2e4. \ud558\uc9c0\ub9cc \ub3c4\uccb4 \uc0c1\ud0dc\uc640 \ub9c8\ucc2c\uac00\uc9c0\ub85c \ud50c\ub77c\uc988\ub9c8\uc5d0\ub294 \uc800\ud56d\uc774 \uc788\ub2e4. \ub9cc\uc57d \uc800\ud56d\uc774 \uc788\ub2e4\uba74 \uadf8\ub85c \uc778\ud574 \uc5f4\uc774 \ubc1c\uc0dd\ud558\uac8c \ub420 \uac83\uc774\uace0 \uc774\uac83\uc774 \ud50c\ub77c\uc988\ub9c8\ub97c \uac00\uc5f4\uc2dc\ud0a4\ub294 \uc5ed\ud560\uc744 \ud558\uac8c \ub41c\ub2e4. \ud50c\ub77c\uc988\ub9c8\ub294 \ub3c4\uccb4\uc774\uae30 \ub54c\ubb38\uc5d0 \uc800\ud56d\uc774 \uc5bc\ub9c8 \uc548 \ub418\uc9c0\ub9cc \uc804\ub958\uac00 \uc0c1\ub2f9\ud788 \ud06c\uae30 \ub54c\ubb38\uc5d0 \uc5f4\uc744 \ubc1c\uc0dd\uc2dc\ucf1c \uc628\ub3c4\ub97c \ub192\uc774\uac8c \ub41c\ub2e4. \ud558\uc9c0\ub9cc \uacc4\uc18d \uc628\ub3c4\uac00 \uc62c\ub77c\uac08\uc218\ub85d \ud50c\ub77c\uc988\ub9c8\uc758 \uc800\ud56d\uc774 \uc904\uc5b4\ub4e4\uac8c \ub418\uace0 \uc774\ub85c \uc778\ud574 \ub192\uc77c \uc218 \uc788\ub294 \uc628\ub3c4\uc5d0 \ud55c\uacc4\uac00 \uc624\uac8c \ub41c\ub2e4. \ud558\uc9c0\ub9cc \ud1a0\uce74\ub9c9\uc740 \ucd08\uace0\uc628 \ud50c\ub77c\uc988\ub9c8\ub97c \ub9cc\ub4e4\uc5b4\uc57c \ud558\uae30 \ub54c\ubb38\uc5d0 \uc800\ud56d\uc5d0 \uc758\ud55c \uac00\uc5f4\ub85c\ub294 \ub9cc\uc871\uc2dc\ud0ac \uc218 \uc5c6\ub2e4. \uadf8\ub7ec\ubbc0\ub85c \uc678\ubd80\uc5d0\uc11c \uac00\uc5f4\uc744 \uc704\ud55c \ub2e4\ub978 \ubc29\ubc95\ub4e4\uc774 \ud544\uc694\ud558\ub2e4.", "answer": "\uc804\ub958", "sentence": "\uc55e\uc5d0\uc11c \uc774\uc57c\uae30\ud558\uc600\ub4ef\uc774 \ud1a0\uce74\ub9c9 \ub0b4\ubd80 \ud50c\ub77c\uc988\ub9c8\uc5d0\ub294 \uc804\ub958 \uac00 \ud750\ub978\ub2e4.", "paragraph_sentence": " \uc55e\uc5d0\uc11c \uc774\uc57c\uae30\ud558\uc600\ub4ef\uc774 \ud1a0\uce74\ub9c9 \ub0b4\ubd80 \ud50c\ub77c\uc988\ub9c8\uc5d0\ub294 \uc804\ub958 \uac00 \ud750\ub978\ub2e4. \ud50c\ub77c\uc988\ub9c8 \uc0c1\ud0dc\ub294 \ub3c4\uccb4 \uc0c1\ud0dc\uc774\ubbc0\ub85c \uc804\ub958\uac00 \ud750\ub974\ub294 \uac83\uc5d0\ub294 \ubb38\uc81c\uac00 \uc5c6\ub2e4. \ud558\uc9c0\ub9cc \ub3c4\uccb4 \uc0c1\ud0dc\uc640 \ub9c8\ucc2c\uac00\uc9c0\ub85c \ud50c\ub77c\uc988\ub9c8\uc5d0\ub294 \uc800\ud56d\uc774 \uc788\ub2e4. \ub9cc\uc57d \uc800\ud56d\uc774 \uc788\ub2e4\uba74 \uadf8\ub85c \uc778\ud574 \uc5f4\uc774 \ubc1c\uc0dd\ud558\uac8c \ub420 \uac83\uc774\uace0 \uc774\uac83\uc774 \ud50c\ub77c\uc988\ub9c8\ub97c \uac00\uc5f4\uc2dc\ud0a4\ub294 \uc5ed\ud560\uc744 \ud558\uac8c \ub41c\ub2e4. \ud50c\ub77c\uc988\ub9c8\ub294 \ub3c4\uccb4\uc774\uae30 \ub54c\ubb38\uc5d0 \uc800\ud56d\uc774 \uc5bc\ub9c8 \uc548 \ub418\uc9c0\ub9cc \uc804\ub958\uac00 \uc0c1\ub2f9\ud788 \ud06c\uae30 \ub54c\ubb38\uc5d0 \uc5f4\uc744 \ubc1c\uc0dd\uc2dc\ucf1c \uc628\ub3c4\ub97c \ub192\uc774\uac8c \ub41c\ub2e4. \ud558\uc9c0\ub9cc \uacc4\uc18d \uc628\ub3c4\uac00 \uc62c\ub77c\uac08\uc218\ub85d \ud50c\ub77c\uc988\ub9c8\uc758 \uc800\ud56d\uc774 \uc904\uc5b4\ub4e4\uac8c \ub418\uace0 \uc774\ub85c \uc778\ud574 \ub192\uc77c \uc218 \uc788\ub294 \uc628\ub3c4\uc5d0 \ud55c\uacc4\uac00 \uc624\uac8c \ub41c\ub2e4. \ud558\uc9c0\ub9cc \ud1a0\uce74\ub9c9\uc740 \ucd08\uace0\uc628 \ud50c\ub77c\uc988\ub9c8\ub97c \ub9cc\ub4e4\uc5b4\uc57c \ud558\uae30 \ub54c\ubb38\uc5d0 \uc800\ud56d\uc5d0 \uc758\ud55c \uac00\uc5f4\ub85c\ub294 \ub9cc\uc871\uc2dc\ud0ac \uc218 \uc5c6\ub2e4. \uadf8\ub7ec\ubbc0\ub85c \uc678\ubd80\uc5d0\uc11c \uac00\uc5f4\uc744 \uc704\ud55c \ub2e4\ub978 \ubc29\ubc95\ub4e4\uc774 \ud544\uc694\ud558\ub2e4.", "paragraph_answer": "\uc55e\uc5d0\uc11c \uc774\uc57c\uae30\ud558\uc600\ub4ef\uc774 \ud1a0\uce74\ub9c9 \ub0b4\ubd80 \ud50c\ub77c\uc988\ub9c8\uc5d0\ub294 \uc804\ub958 \uac00 \ud750\ub978\ub2e4. \ud50c\ub77c\uc988\ub9c8 \uc0c1\ud0dc\ub294 \ub3c4\uccb4 \uc0c1\ud0dc\uc774\ubbc0\ub85c \uc804\ub958\uac00 \ud750\ub974\ub294 \uac83\uc5d0\ub294 \ubb38\uc81c\uac00 \uc5c6\ub2e4. \ud558\uc9c0\ub9cc \ub3c4\uccb4 \uc0c1\ud0dc\uc640 \ub9c8\ucc2c\uac00\uc9c0\ub85c \ud50c\ub77c\uc988\ub9c8\uc5d0\ub294 \uc800\ud56d\uc774 \uc788\ub2e4. \ub9cc\uc57d \uc800\ud56d\uc774 \uc788\ub2e4\uba74 \uadf8\ub85c \uc778\ud574 \uc5f4\uc774 \ubc1c\uc0dd\ud558\uac8c \ub420 \uac83\uc774\uace0 \uc774\uac83\uc774 \ud50c\ub77c\uc988\ub9c8\ub97c \uac00\uc5f4\uc2dc\ud0a4\ub294 \uc5ed\ud560\uc744 \ud558\uac8c \ub41c\ub2e4. \ud50c\ub77c\uc988\ub9c8\ub294 \ub3c4\uccb4\uc774\uae30 \ub54c\ubb38\uc5d0 \uc800\ud56d\uc774 \uc5bc\ub9c8 \uc548 \ub418\uc9c0\ub9cc \uc804\ub958\uac00 \uc0c1\ub2f9\ud788 \ud06c\uae30 \ub54c\ubb38\uc5d0 \uc5f4\uc744 \ubc1c\uc0dd\uc2dc\ucf1c \uc628\ub3c4\ub97c \ub192\uc774\uac8c \ub41c\ub2e4. \ud558\uc9c0\ub9cc \uacc4\uc18d \uc628\ub3c4\uac00 \uc62c\ub77c\uac08\uc218\ub85d \ud50c\ub77c\uc988\ub9c8\uc758 \uc800\ud56d\uc774 \uc904\uc5b4\ub4e4\uac8c \ub418\uace0 \uc774\ub85c \uc778\ud574 \ub192\uc77c \uc218 \uc788\ub294 \uc628\ub3c4\uc5d0 \ud55c\uacc4\uac00 \uc624\uac8c \ub41c\ub2e4. \ud558\uc9c0\ub9cc \ud1a0\uce74\ub9c9\uc740 \ucd08\uace0\uc628 \ud50c\ub77c\uc988\ub9c8\ub97c \ub9cc\ub4e4\uc5b4\uc57c \ud558\uae30 \ub54c\ubb38\uc5d0 \uc800\ud56d\uc5d0 \uc758\ud55c \uac00\uc5f4\ub85c\ub294 \ub9cc\uc871\uc2dc\ud0ac \uc218 \uc5c6\ub2e4. \uadf8\ub7ec\ubbc0\ub85c \uc678\ubd80\uc5d0\uc11c \uac00\uc5f4\uc744 \uc704\ud55c \ub2e4\ub978 \ubc29\ubc95\ub4e4\uc774 \ud544\uc694\ud558\ub2e4.", "sentence_answer": "\uc55e\uc5d0\uc11c \uc774\uc57c\uae30\ud558\uc600\ub4ef\uc774 \ud1a0\uce74\ub9c9 \ub0b4\ubd80 \ud50c\ub77c\uc988\ub9c8\uc5d0\ub294 \uc804\ub958 \uac00 \ud750\ub978\ub2e4.", "paragraph_id": "6566153-3-0", "paragraph_question": "question: \ud1a0\uce74\ub9c9 \ub0b4\ubd80 \ud50c\ub77c\uc988\ub9c8\uc5d0 \ud750\ub974\ub294 \uac83\uc740?, context: \uc55e\uc5d0\uc11c \uc774\uc57c\uae30\ud558\uc600\ub4ef\uc774 \ud1a0\uce74\ub9c9 \ub0b4\ubd80 \ud50c\ub77c\uc988\ub9c8\uc5d0\ub294 \uc804\ub958\uac00 \ud750\ub978\ub2e4. \ud50c\ub77c\uc988\ub9c8 \uc0c1\ud0dc\ub294 \ub3c4\uccb4 \uc0c1\ud0dc\uc774\ubbc0\ub85c \uc804\ub958\uac00 \ud750\ub974\ub294 \uac83\uc5d0\ub294 \ubb38\uc81c\uac00 \uc5c6\ub2e4. \ud558\uc9c0\ub9cc \ub3c4\uccb4 \uc0c1\ud0dc\uc640 \ub9c8\ucc2c\uac00\uc9c0\ub85c \ud50c\ub77c\uc988\ub9c8\uc5d0\ub294 \uc800\ud56d\uc774 \uc788\ub2e4. \ub9cc\uc57d \uc800\ud56d\uc774 \uc788\ub2e4\uba74 \uadf8\ub85c \uc778\ud574 \uc5f4\uc774 \ubc1c\uc0dd\ud558\uac8c \ub420 \uac83\uc774\uace0 \uc774\uac83\uc774 \ud50c\ub77c\uc988\ub9c8\ub97c \uac00\uc5f4\uc2dc\ud0a4\ub294 \uc5ed\ud560\uc744 \ud558\uac8c \ub41c\ub2e4. \ud50c\ub77c\uc988\ub9c8\ub294 \ub3c4\uccb4\uc774\uae30 \ub54c\ubb38\uc5d0 \uc800\ud56d\uc774 \uc5bc\ub9c8 \uc548 \ub418\uc9c0\ub9cc \uc804\ub958\uac00 \uc0c1\ub2f9\ud788 \ud06c\uae30 \ub54c\ubb38\uc5d0 \uc5f4\uc744 \ubc1c\uc0dd\uc2dc\ucf1c \uc628\ub3c4\ub97c \ub192\uc774\uac8c \ub41c\ub2e4. \ud558\uc9c0\ub9cc \uacc4\uc18d \uc628\ub3c4\uac00 \uc62c\ub77c\uac08\uc218\ub85d \ud50c\ub77c\uc988\ub9c8\uc758 \uc800\ud56d\uc774 \uc904\uc5b4\ub4e4\uac8c \ub418\uace0 \uc774\ub85c \uc778\ud574 \ub192\uc77c \uc218 \uc788\ub294 \uc628\ub3c4\uc5d0 \ud55c\uacc4\uac00 \uc624\uac8c \ub41c\ub2e4. \ud558\uc9c0\ub9cc \ud1a0\uce74\ub9c9\uc740 \ucd08\uace0\uc628 \ud50c\ub77c\uc988\ub9c8\ub97c \ub9cc\ub4e4\uc5b4\uc57c \ud558\uae30 \ub54c\ubb38\uc5d0 \uc800\ud56d\uc5d0 \uc758\ud55c \uac00\uc5f4\ub85c\ub294 \ub9cc\uc871\uc2dc\ud0ac \uc218 \uc5c6\ub2e4. \uadf8\ub7ec\ubbc0\ub85c \uc678\ubd80\uc5d0\uc11c \uac00\uc5f4\uc744 \uc704\ud55c \ub2e4\ub978 \ubc29\ubc95\ub4e4\uc774 \ud544\uc694\ud558\ub2e4."} {"question": "\ub450\ud23d\uc0c1\uc5b4\uc758 \uac00\uc2b4\uc9c0\ub290\ub7ec\ubbf8\ub294 \uc544\uac00\ubbf8 \uba87\uc9f8 \uad6c\uba4d\uc704\ub85c \uc704\uce58\ud574\uc788\ub294\uac00?", "paragraph": "\ub450\ud23d\uc0c1\uc5b4\ub294 \uc2e0\uc7a5 50 \uc13c\ud2f0\ubbf8\ud130\uae4c\uc9c0 \uc790\ub77c\uba70, \uadf8 \ubab8\uc740 \uac00\ub298\uba74\uc11c\ub3c4 \uacac\uace0\ud558\ub2e4. \uc881\uc740 \uba38\ub9ac\uac00 \uc804\uccb4 \uc2e0\uc7a5\uc758 6\ubd84\uc9c0 1 \uc0b4\uc9dd \uc774\ud558 \uc815\ub3c4\ub97c \ucc28\uc9c0\ud558\uba70, \uba38\ub9ac\uc758 \ub108\ube44\uac00 \uae38\uc774\uc758 3\ubd84\uc9c0 2\uc815\ub3c4 \ub41c\ub2e4. \uc8fc\ub465\uc774\ub294 \uc9e7\uace0 \ub465\uae00\ub2e4. \ud070 \ube44\uacf5 \uc55e\uc5d0\ub294 \uc791\uc740 \uc0bc\uac01\ud615 \uc0b4\uc810\uc774 \uc788\uc73c\uba70, \uc774 \uc0b4\uc810\uc740 \uc785\uae4c\uc9c0 \ub2ff\uc9c0 \uc54a\ub294\ub2e4. \ub208\uc740 \uc911\uac04 \uc815\ub3c4 \ud06c\uae30\uc774\uba70 \uac00\ub85c\ub85c \ucc22\uc5b4\uc9c4 \ud0c0\uc6d0\ud615\uc774\ub2e4. \ub208\uc758 \uc21c\ub9c9(\uc81c3\uc758 \ub208\uaebc\ud480)\uc740 \ud754\uc801\uae30\uad00 \uc815\ub3c4\ub85c \ub0a8\uc544 \uc788\uc73c\uba70, \ub208 \ub4a4\uc5d0\ub294 \uae30\ubb38\uc774 \uc788\ub2e4. \ube44\uacf5\uacfc \uc785 \uc0ac\uc774\uc5d0 \ud648\uc740 \uc5c6\ub2e4. \uc785 \uad6c\uc11d\uc5d0\uc11c \ud558\uc545 \ucabd\uc73c\ub85c\ub9cc \uace8\uc774 \uc788\uace0 \uc0c1\uc545 \ucabd\uc73c\ub85c\ub294 \uc5c6\ub2e4. \uc774\ube68\uc740 \uc791\uace0, \uadf8 \uc911\uc559\ubd80\uac00 \ubfb0\uc871\ud558\uba70 \uce21\uba74\uc73c\ub85c \uadf8\ubcf4\ub2e4\ub294 \uc791\uc740 \ub3cc\ucd9c\ubd80\uac00 \ud55c \uc30d \uc090\uc8fd\ud558\ub2e4. \uc544\uac00\ubbf8\uad6c\uba4d\uc740 \ub2e4\uc12f \uc30d\uc774\uba70 \uc9e7\uace0, \ub137\uc9f8 \uad6c\uba4d \uc704\ub85c \uac00\uc2b4\uc9c0\ub290\ub7ec\ubbf8\uac00 \ub098 \uc788\ub2e4.", "answer": "\ub137\uc9f8", "sentence": "\uc544\uac00\ubbf8\uad6c\uba4d\uc740 \ub2e4\uc12f \uc30d\uc774\uba70 \uc9e7\uace0, \ub137\uc9f8 \uad6c\uba4d \uc704\ub85c \uac00\uc2b4\uc9c0\ub290\ub7ec\ubbf8\uac00 \ub098 \uc788\ub2e4.", "paragraph_sentence": "\ub450\ud23d\uc0c1\uc5b4\ub294 \uc2e0\uc7a5 50 \uc13c\ud2f0\ubbf8\ud130\uae4c\uc9c0 \uc790\ub77c\uba70, \uadf8 \ubab8\uc740 \uac00\ub298\uba74\uc11c\ub3c4 \uacac\uace0\ud558\ub2e4. \uc881\uc740 \uba38\ub9ac\uac00 \uc804\uccb4 \uc2e0\uc7a5\uc758 6\ubd84\uc9c0 1 \uc0b4\uc9dd \uc774\ud558 \uc815\ub3c4\ub97c \ucc28\uc9c0\ud558\uba70, \uba38\ub9ac\uc758 \ub108\ube44\uac00 \uae38\uc774\uc758 3\ubd84\uc9c0 2\uc815\ub3c4 \ub41c\ub2e4. \uc8fc\ub465\uc774\ub294 \uc9e7\uace0 \ub465\uae00\ub2e4. \ud070 \ube44\uacf5 \uc55e\uc5d0\ub294 \uc791\uc740 \uc0bc\uac01\ud615 \uc0b4\uc810\uc774 \uc788\uc73c\uba70, \uc774 \uc0b4\uc810\uc740 \uc785\uae4c\uc9c0 \ub2ff\uc9c0 \uc54a\ub294\ub2e4. \ub208\uc740 \uc911\uac04 \uc815\ub3c4 \ud06c\uae30\uc774\uba70 \uac00\ub85c\ub85c \ucc22\uc5b4\uc9c4 \ud0c0\uc6d0\ud615\uc774\ub2e4. \ub208\uc758 \uc21c\ub9c9(\uc81c3\uc758 \ub208\uaebc\ud480)\uc740 \ud754\uc801\uae30\uad00 \uc815\ub3c4\ub85c \ub0a8\uc544 \uc788\uc73c\uba70, \ub208 \ub4a4\uc5d0\ub294 \uae30\ubb38\uc774 \uc788\ub2e4. \ube44\uacf5\uacfc \uc785 \uc0ac\uc774\uc5d0 \ud648\uc740 \uc5c6\ub2e4. \uc785 \uad6c\uc11d\uc5d0\uc11c \ud558\uc545 \ucabd\uc73c\ub85c\ub9cc \uace8\uc774 \uc788\uace0 \uc0c1\uc545 \ucabd\uc73c\ub85c\ub294 \uc5c6\ub2e4. \uc774\ube68\uc740 \uc791\uace0, \uadf8 \uc911\uc559\ubd80\uac00 \ubfb0\uc871\ud558\uba70 \uce21\uba74\uc73c\ub85c \uadf8\ubcf4\ub2e4\ub294 \uc791\uc740 \ub3cc\ucd9c\ubd80\uac00 \ud55c \uc30d \uc090\uc8fd\ud558\ub2e4. \uc544\uac00\ubbf8\uad6c\uba4d\uc740 \ub2e4\uc12f \uc30d\uc774\uba70 \uc9e7\uace0, \ub137\uc9f8 \uad6c\uba4d \uc704\ub85c \uac00\uc2b4\uc9c0\ub290\ub7ec\ubbf8\uac00 \ub098 \uc788\ub2e4. ", "paragraph_answer": "\ub450\ud23d\uc0c1\uc5b4\ub294 \uc2e0\uc7a5 50 \uc13c\ud2f0\ubbf8\ud130\uae4c\uc9c0 \uc790\ub77c\uba70, \uadf8 \ubab8\uc740 \uac00\ub298\uba74\uc11c\ub3c4 \uacac\uace0\ud558\ub2e4. \uc881\uc740 \uba38\ub9ac\uac00 \uc804\uccb4 \uc2e0\uc7a5\uc758 6\ubd84\uc9c0 1 \uc0b4\uc9dd \uc774\ud558 \uc815\ub3c4\ub97c \ucc28\uc9c0\ud558\uba70, \uba38\ub9ac\uc758 \ub108\ube44\uac00 \uae38\uc774\uc758 3\ubd84\uc9c0 2\uc815\ub3c4 \ub41c\ub2e4. \uc8fc\ub465\uc774\ub294 \uc9e7\uace0 \ub465\uae00\ub2e4. \ud070 \ube44\uacf5 \uc55e\uc5d0\ub294 \uc791\uc740 \uc0bc\uac01\ud615 \uc0b4\uc810\uc774 \uc788\uc73c\uba70, \uc774 \uc0b4\uc810\uc740 \uc785\uae4c\uc9c0 \ub2ff\uc9c0 \uc54a\ub294\ub2e4. \ub208\uc740 \uc911\uac04 \uc815\ub3c4 \ud06c\uae30\uc774\uba70 \uac00\ub85c\ub85c \ucc22\uc5b4\uc9c4 \ud0c0\uc6d0\ud615\uc774\ub2e4. \ub208\uc758 \uc21c\ub9c9(\uc81c3\uc758 \ub208\uaebc\ud480)\uc740 \ud754\uc801\uae30\uad00 \uc815\ub3c4\ub85c \ub0a8\uc544 \uc788\uc73c\uba70, \ub208 \ub4a4\uc5d0\ub294 \uae30\ubb38\uc774 \uc788\ub2e4. \ube44\uacf5\uacfc \uc785 \uc0ac\uc774\uc5d0 \ud648\uc740 \uc5c6\ub2e4. \uc785 \uad6c\uc11d\uc5d0\uc11c \ud558\uc545 \ucabd\uc73c\ub85c\ub9cc \uace8\uc774 \uc788\uace0 \uc0c1\uc545 \ucabd\uc73c\ub85c\ub294 \uc5c6\ub2e4. \uc774\ube68\uc740 \uc791\uace0, \uadf8 \uc911\uc559\ubd80\uac00 \ubfb0\uc871\ud558\uba70 \uce21\uba74\uc73c\ub85c \uadf8\ubcf4\ub2e4\ub294 \uc791\uc740 \ub3cc\ucd9c\ubd80\uac00 \ud55c \uc30d \uc090\uc8fd\ud558\ub2e4. \uc544\uac00\ubbf8\uad6c\uba4d\uc740 \ub2e4\uc12f \uc30d\uc774\uba70 \uc9e7\uace0, \ub137\uc9f8 \uad6c\uba4d \uc704\ub85c \uac00\uc2b4\uc9c0\ub290\ub7ec\ubbf8\uac00 \ub098 \uc788\ub2e4.", "sentence_answer": "\uc544\uac00\ubbf8\uad6c\uba4d\uc740 \ub2e4\uc12f \uc30d\uc774\uba70 \uc9e7\uace0, \ub137\uc9f8 \uad6c\uba4d \uc704\ub85c \uac00\uc2b4\uc9c0\ub290\ub7ec\ubbf8\uac00 \ub098 \uc788\ub2e4.", "paragraph_id": "6048075-0-2", "paragraph_question": "question: \ub450\ud23d\uc0c1\uc5b4\uc758 \uac00\uc2b4\uc9c0\ub290\ub7ec\ubbf8\ub294 \uc544\uac00\ubbf8 \uba87\uc9f8 \uad6c\uba4d\uc704\ub85c \uc704\uce58\ud574\uc788\ub294\uac00?, context: \ub450\ud23d\uc0c1\uc5b4\ub294 \uc2e0\uc7a5 50 \uc13c\ud2f0\ubbf8\ud130\uae4c\uc9c0 \uc790\ub77c\uba70, \uadf8 \ubab8\uc740 \uac00\ub298\uba74\uc11c\ub3c4 \uacac\uace0\ud558\ub2e4. \uc881\uc740 \uba38\ub9ac\uac00 \uc804\uccb4 \uc2e0\uc7a5\uc758 6\ubd84\uc9c0 1 \uc0b4\uc9dd \uc774\ud558 \uc815\ub3c4\ub97c \ucc28\uc9c0\ud558\uba70, \uba38\ub9ac\uc758 \ub108\ube44\uac00 \uae38\uc774\uc758 3\ubd84\uc9c0 2\uc815\ub3c4 \ub41c\ub2e4. \uc8fc\ub465\uc774\ub294 \uc9e7\uace0 \ub465\uae00\ub2e4. \ud070 \ube44\uacf5 \uc55e\uc5d0\ub294 \uc791\uc740 \uc0bc\uac01\ud615 \uc0b4\uc810\uc774 \uc788\uc73c\uba70, \uc774 \uc0b4\uc810\uc740 \uc785\uae4c\uc9c0 \ub2ff\uc9c0 \uc54a\ub294\ub2e4. \ub208\uc740 \uc911\uac04 \uc815\ub3c4 \ud06c\uae30\uc774\uba70 \uac00\ub85c\ub85c \ucc22\uc5b4\uc9c4 \ud0c0\uc6d0\ud615\uc774\ub2e4. \ub208\uc758 \uc21c\ub9c9(\uc81c3\uc758 \ub208\uaebc\ud480)\uc740 \ud754\uc801\uae30\uad00 \uc815\ub3c4\ub85c \ub0a8\uc544 \uc788\uc73c\uba70, \ub208 \ub4a4\uc5d0\ub294 \uae30\ubb38\uc774 \uc788\ub2e4. \ube44\uacf5\uacfc \uc785 \uc0ac\uc774\uc5d0 \ud648\uc740 \uc5c6\ub2e4. \uc785 \uad6c\uc11d\uc5d0\uc11c \ud558\uc545 \ucabd\uc73c\ub85c\ub9cc \uace8\uc774 \uc788\uace0 \uc0c1\uc545 \ucabd\uc73c\ub85c\ub294 \uc5c6\ub2e4. \uc774\ube68\uc740 \uc791\uace0, \uadf8 \uc911\uc559\ubd80\uac00 \ubfb0\uc871\ud558\uba70 \uce21\uba74\uc73c\ub85c \uadf8\ubcf4\ub2e4\ub294 \uc791\uc740 \ub3cc\ucd9c\ubd80\uac00 \ud55c \uc30d \uc090\uc8fd\ud558\ub2e4. \uc544\uac00\ubbf8\uad6c\uba4d\uc740 \ub2e4\uc12f \uc30d\uc774\uba70 \uc9e7\uace0, \ub137\uc9f8 \uad6c\uba4d \uc704\ub85c \uac00\uc2b4\uc9c0\ub290\ub7ec\ubbf8\uac00 \ub098 \uc788\ub2e4."} {"question": "\ube44\ud2b8\uac90\uc288\ud0c0\uc778\uc758 \ud6c4\uae30 \uc0ac\uc0c1\uc774 \uc9d1\uc57d\ub41c \ucc45\uc740?", "paragraph": "\ube44\ud2b8\uac90\uc288\ud0c0\uc778\uc758 \uc0ac\uc0c1\uc740 \u300a\ub17c\ub9ac \ucca0\ud559 \ub17c\uace0\u300b\ub85c \ub300\ud45c\ub418\ub294 \uc804\uae30\uc640 \u300a\ucca0\ud559 \ud0d0\uad6c\u300b\ub85c \ub300\ud45c\ub418\ub294 \ud6c4\uae30\ub85c \ub098\ub25c\ub2e4. \u300a\ub17c\ub9ac \ucca0\ud559 \ub17c\uace0\u300b\uc5d0 \ub098\ud0c0\ub09c \uc804\uae30 \uc0ac\uc0c1\uc774 \uba85\uc81c\uc5d0 \uc0ac\uc6a9\ub41c \ub0b1\ub9d0\uc758 \uc740\uc720\ub2e4\uc6b4 \uad00\uacc4\ub97c \ubd84\uc11d\ud558\uc5ec \uae30\uc874 \ucca0\ud559\uc5d0\uc11c \uc798\ubabb\ub41c \uac1c\ub150 \ud0d3\uc5d0 \ube5a\uc5b4\uc9c4 \ub17c\ub9ac\uc5d0 \uc0c1\ucda9\ud558\ub294 \uc810\uc744 \uc9c0\ubaa9\ud558\ub294 \ub370 \uc9d1\uc911\ub41c \ubc18\uba74, \ud6c4\uae30 \uc0ac\uc0c1\uc740 \uc5b8\uc5b4-\ub180\uc774\uc5d0\uc11c \uc0c1\ud638 \ubcc0\ud658\ub418\ub294 \uc790\uc5f0 \uc5b8\uc5b4\uac00 \ub17c\ub9ac\uc5d0 \ubd80\ud569\ud55c \uad6c\uc870\ub85c \uc815\ud615\ud654\ud55c \uc5b8\uc5b4\uc640\ub294 \ub2e4\ub978 \uc758\ubbf8\uac00 \uc788\ub2e4\ub294 \uc810\uc744 \uc5ed\uc124\ud558\ub294\ub370 \uc911\uc2ec\uc774 \ub193\uc5ec \uc788\ub2e4. \ube44\ud2b8\uac90\uc288\ud0c0\uc778\uc740 \u201c\ub2e8\uc5b4\uc758 \uc758\ubbf8\ub294 \uc8fc\uc5b4\uc9c4 \uc5b8\uc5b4-\ub180\uc774 \uc548\uc5d0\uc11c \uadf8 \ub2e8\uc5b4\ub4e4\uc774 \uc0ac\uc6a9\uc774 \ub420 \ub54c \uac00\uc7a5 \uc798 \uc774\ud574\ub41c\ub2e4\u201d(the meaning of words is best understood as their use within a given language-game)\ub77c\uace0 \uc8fc\uc7a5\ud558\uc600\ub294\ub370 \uc774\uac83\uc740 \ube44\ud2b8\uac90\uc288\ud0c0\uc778\uc758 \ud6c4\uae30 \uc0ac\uc0c1\uc744 \ub300\ud45c\ud558\ub294 \ub9d0\uc774\ub2e4.", "answer": "\ucca0\ud559 \ud0d0\uad6c", "sentence": "\ube44\ud2b8\uac90\uc288\ud0c0\uc778\uc758 \uc0ac\uc0c1\uc740 \u300a\ub17c\ub9ac \ucca0\ud559 \ub17c\uace0\u300b\ub85c \ub300\ud45c\ub418\ub294 \uc804\uae30\uc640 \u300a \ucca0\ud559 \ud0d0\uad6c \u300b\ub85c \ub300\ud45c\ub418\ub294 \ud6c4\uae30\ub85c \ub098\ub25c\ub2e4.", "paragraph_sentence": " \ube44\ud2b8\uac90\uc288\ud0c0\uc778\uc758 \uc0ac\uc0c1\uc740 \u300a\ub17c\ub9ac \ucca0\ud559 \ub17c\uace0\u300b\ub85c \ub300\ud45c\ub418\ub294 \uc804\uae30\uc640 \u300a \ucca0\ud559 \ud0d0\uad6c \u300b\ub85c \ub300\ud45c\ub418\ub294 \ud6c4\uae30\ub85c \ub098\ub25c\ub2e4. \u300a\ub17c\ub9ac \ucca0\ud559 \ub17c\uace0\u300b\uc5d0 \ub098\ud0c0\ub09c \uc804\uae30 \uc0ac\uc0c1\uc774 \uba85\uc81c\uc5d0 \uc0ac\uc6a9\ub41c \ub0b1\ub9d0\uc758 \uc740\uc720\ub2e4\uc6b4 \uad00\uacc4\ub97c \ubd84\uc11d\ud558\uc5ec \uae30\uc874 \ucca0\ud559\uc5d0\uc11c \uc798\ubabb\ub41c \uac1c\ub150 \ud0d3\uc5d0 \ube5a\uc5b4\uc9c4 \ub17c\ub9ac\uc5d0 \uc0c1\ucda9\ud558\ub294 \uc810\uc744 \uc9c0\ubaa9\ud558\ub294 \ub370 \uc9d1\uc911\ub41c \ubc18\uba74, \ud6c4\uae30 \uc0ac\uc0c1\uc740 \uc5b8\uc5b4-\ub180\uc774\uc5d0\uc11c \uc0c1\ud638 \ubcc0\ud658\ub418\ub294 \uc790\uc5f0 \uc5b8\uc5b4\uac00 \ub17c\ub9ac\uc5d0 \ubd80\ud569\ud55c \uad6c\uc870\ub85c \uc815\ud615\ud654\ud55c \uc5b8\uc5b4\uc640\ub294 \ub2e4\ub978 \uc758\ubbf8\uac00 \uc788\ub2e4\ub294 \uc810\uc744 \uc5ed\uc124\ud558\ub294\ub370 \uc911\uc2ec\uc774 \ub193\uc5ec \uc788\ub2e4. \ube44\ud2b8\uac90\uc288\ud0c0\uc778\uc740 \u201c\ub2e8\uc5b4\uc758 \uc758\ubbf8\ub294 \uc8fc\uc5b4\uc9c4 \uc5b8\uc5b4-\ub180\uc774 \uc548\uc5d0\uc11c \uadf8 \ub2e8\uc5b4\ub4e4\uc774 \uc0ac\uc6a9\uc774 \ub420 \ub54c \uac00\uc7a5 \uc798 \uc774\ud574\ub41c\ub2e4\u201d(the meaning of words is best understood as their use within a given language-game)\ub77c\uace0 \uc8fc\uc7a5\ud558\uc600\ub294\ub370 \uc774\uac83\uc740 \ube44\ud2b8\uac90\uc288\ud0c0\uc778\uc758 \ud6c4\uae30 \uc0ac\uc0c1\uc744 \ub300\ud45c\ud558\ub294 \ub9d0\uc774\ub2e4.", "paragraph_answer": "\ube44\ud2b8\uac90\uc288\ud0c0\uc778\uc758 \uc0ac\uc0c1\uc740 \u300a\ub17c\ub9ac \ucca0\ud559 \ub17c\uace0\u300b\ub85c \ub300\ud45c\ub418\ub294 \uc804\uae30\uc640 \u300a \ucca0\ud559 \ud0d0\uad6c \u300b\ub85c \ub300\ud45c\ub418\ub294 \ud6c4\uae30\ub85c \ub098\ub25c\ub2e4. \u300a\ub17c\ub9ac \ucca0\ud559 \ub17c\uace0\u300b\uc5d0 \ub098\ud0c0\ub09c \uc804\uae30 \uc0ac\uc0c1\uc774 \uba85\uc81c\uc5d0 \uc0ac\uc6a9\ub41c \ub0b1\ub9d0\uc758 \uc740\uc720\ub2e4\uc6b4 \uad00\uacc4\ub97c \ubd84\uc11d\ud558\uc5ec \uae30\uc874 \ucca0\ud559\uc5d0\uc11c \uc798\ubabb\ub41c \uac1c\ub150 \ud0d3\uc5d0 \ube5a\uc5b4\uc9c4 \ub17c\ub9ac\uc5d0 \uc0c1\ucda9\ud558\ub294 \uc810\uc744 \uc9c0\ubaa9\ud558\ub294 \ub370 \uc9d1\uc911\ub41c \ubc18\uba74, \ud6c4\uae30 \uc0ac\uc0c1\uc740 \uc5b8\uc5b4-\ub180\uc774\uc5d0\uc11c \uc0c1\ud638 \ubcc0\ud658\ub418\ub294 \uc790\uc5f0 \uc5b8\uc5b4\uac00 \ub17c\ub9ac\uc5d0 \ubd80\ud569\ud55c \uad6c\uc870\ub85c \uc815\ud615\ud654\ud55c \uc5b8\uc5b4\uc640\ub294 \ub2e4\ub978 \uc758\ubbf8\uac00 \uc788\ub2e4\ub294 \uc810\uc744 \uc5ed\uc124\ud558\ub294\ub370 \uc911\uc2ec\uc774 \ub193\uc5ec \uc788\ub2e4. \ube44\ud2b8\uac90\uc288\ud0c0\uc778\uc740 \u201c\ub2e8\uc5b4\uc758 \uc758\ubbf8\ub294 \uc8fc\uc5b4\uc9c4 \uc5b8\uc5b4-\ub180\uc774 \uc548\uc5d0\uc11c \uadf8 \ub2e8\uc5b4\ub4e4\uc774 \uc0ac\uc6a9\uc774 \ub420 \ub54c \uac00\uc7a5 \uc798 \uc774\ud574\ub41c\ub2e4\u201d(the meaning of words is best understood as their use within a given language-game)\ub77c\uace0 \uc8fc\uc7a5\ud558\uc600\ub294\ub370 \uc774\uac83\uc740 \ube44\ud2b8\uac90\uc288\ud0c0\uc778\uc758 \ud6c4\uae30 \uc0ac\uc0c1\uc744 \ub300\ud45c\ud558\ub294 \ub9d0\uc774\ub2e4.", "sentence_answer": "\ube44\ud2b8\uac90\uc288\ud0c0\uc778\uc758 \uc0ac\uc0c1\uc740 \u300a\ub17c\ub9ac \ucca0\ud559 \ub17c\uace0\u300b\ub85c \ub300\ud45c\ub418\ub294 \uc804\uae30\uc640 \u300a \ucca0\ud559 \ud0d0\uad6c \u300b\ub85c \ub300\ud45c\ub418\ub294 \ud6c4\uae30\ub85c \ub098\ub25c\ub2e4.", "paragraph_id": "6497900-0-1", "paragraph_question": "question: \ube44\ud2b8\uac90\uc288\ud0c0\uc778\uc758 \ud6c4\uae30 \uc0ac\uc0c1\uc774 \uc9d1\uc57d\ub41c \ucc45\uc740?, context: \ube44\ud2b8\uac90\uc288\ud0c0\uc778\uc758 \uc0ac\uc0c1\uc740 \u300a\ub17c\ub9ac \ucca0\ud559 \ub17c\uace0\u300b\ub85c \ub300\ud45c\ub418\ub294 \uc804\uae30\uc640 \u300a\ucca0\ud559 \ud0d0\uad6c\u300b\ub85c \ub300\ud45c\ub418\ub294 \ud6c4\uae30\ub85c \ub098\ub25c\ub2e4. \u300a\ub17c\ub9ac \ucca0\ud559 \ub17c\uace0\u300b\uc5d0 \ub098\ud0c0\ub09c \uc804\uae30 \uc0ac\uc0c1\uc774 \uba85\uc81c\uc5d0 \uc0ac\uc6a9\ub41c \ub0b1\ub9d0\uc758 \uc740\uc720\ub2e4\uc6b4 \uad00\uacc4\ub97c \ubd84\uc11d\ud558\uc5ec \uae30\uc874 \ucca0\ud559\uc5d0\uc11c \uc798\ubabb\ub41c \uac1c\ub150 \ud0d3\uc5d0 \ube5a\uc5b4\uc9c4 \ub17c\ub9ac\uc5d0 \uc0c1\ucda9\ud558\ub294 \uc810\uc744 \uc9c0\ubaa9\ud558\ub294 \ub370 \uc9d1\uc911\ub41c \ubc18\uba74, \ud6c4\uae30 \uc0ac\uc0c1\uc740 \uc5b8\uc5b4-\ub180\uc774\uc5d0\uc11c \uc0c1\ud638 \ubcc0\ud658\ub418\ub294 \uc790\uc5f0 \uc5b8\uc5b4\uac00 \ub17c\ub9ac\uc5d0 \ubd80\ud569\ud55c \uad6c\uc870\ub85c \uc815\ud615\ud654\ud55c \uc5b8\uc5b4\uc640\ub294 \ub2e4\ub978 \uc758\ubbf8\uac00 \uc788\ub2e4\ub294 \uc810\uc744 \uc5ed\uc124\ud558\ub294\ub370 \uc911\uc2ec\uc774 \ub193\uc5ec \uc788\ub2e4. \ube44\ud2b8\uac90\uc288\ud0c0\uc778\uc740 \u201c\ub2e8\uc5b4\uc758 \uc758\ubbf8\ub294 \uc8fc\uc5b4\uc9c4 \uc5b8\uc5b4-\ub180\uc774 \uc548\uc5d0\uc11c \uadf8 \ub2e8\uc5b4\ub4e4\uc774 \uc0ac\uc6a9\uc774 \ub420 \ub54c \uac00\uc7a5 \uc798 \uc774\ud574\ub41c\ub2e4\u201d(the meaning of words is best understood as their use within a given language-game)\ub77c\uace0 \uc8fc\uc7a5\ud558\uc600\ub294\ub370 \uc774\uac83\uc740 \ube44\ud2b8\uac90\uc288\ud0c0\uc778\uc758 \ud6c4\uae30 \uc0ac\uc0c1\uc744 \ub300\ud45c\ud558\ub294 \ub9d0\uc774\ub2e4."} {"question": "\uc804\ubbf8\uad50\uc721\ud611\ud68c\uac00 \uc804\ud1b5\uc801 \uc5ed\uc0ac\uad50\uc721\uc744 \ubc97\uc5b4\ub098 \uacfc\ud559\uc801 \ud504\ub85c\uadf8\ub7a8\uc744 \uc81c\uc2dc\ud558\uae30 \uc704\ud574 \uc870\uc9c1\ud55c \uac83\uc740?", "paragraph": "\uc2dc\ubbfc\uad50\uc721\uc740 \uc778\ub958\uac00 \uc0ac\ud68c\ub97c \ud615\uc131\ud558\uba74\uc11c \uc2dc\uc791\ub418\uc5c8\uc73c\ub098, \uc5ec\ub7ec \uc0ac\ud68c\uacfc\ud559 \ud559\ubb38\uc744 \ud1b5\ud569\ud558\uc5ec \uc0ac\ud68c\uacfc\ub85c \ub4f1\uc7a5\ud55c \uac83\uc740 20\uc138\uae30 \ucd08\ubc18, \ubbf8\uad6d \uc0ac\ud68c\uc600\ub2e4. \ub2f9\uc2dc \ubbf8\uad6d\uc740 \uc774\ubbfc\uc790\ub97c \ubbf8\uad6d\ud654\ud558\uace0, \ubbf8\uad6d \ub0a8\ubd80\uc5d0\uc11c \ubd81\ubd80\ub85c \uc774\uc8fc\ud55c \uc774\uc8fc\ubbfc\uc774 \ub3c4\uc2dc\uc5d0 \uc801\uc751\ud558\ub3c4\ub85d \uad50\uc721\uc744 \uc2e4\uc2dc\ud560 \ud544\uc694\uc131\uc774 \ub300\ub450\ub418\uc5c8\ub2e4. \uc774\ub7ec\ud55c \uc0ac\ud68c\uc801 \uc694\uad6c\ub294 \uc9c0\uc2dd \uc804\ub2ec \uc911\uc2ec\uc758 \uc804\ud1b5\uc801 \uad50\uc721\uc5d0 \ub300\ud55c \uac1c\ud601\uc744 \uc694\uad6c\ud558\ub294 \ud615\ud0dc\ub85c \ub098\ud0c0\ub0ac\uace0, \uc774 \uc694\uad6c\uc5d0 \ubd80\uc751\ud558\uc5ec 1892\ub144 \uc804\ubbf8\uad50\uc721\ud611\ud68c\uc5d0\uc11c 10\uc778 \uc704\uc6d0\ud68c\ub97c \uc870\uc9c1\ud574 \uc9c1\uc5c5\uc0dd\ud65c\uacfc \uc2dc\ubbfc\uad50\uc721\uc744 \ud3ec\ud568\ud55c \uad50\uc721\uacfc\uc815\uc744 \uc81c\uc2dc\ud558\uc600\uc73c\ub098, \uc5ec\uc804\ud788 \uc9c0\uc2dd \uc804\ub2ec \uc911\uc2ec\uc758 \uc804\ud1b5\uc801 \uad50\uc721\uc744 \ubc97\uc5b4\ub098\uc9c0 \ubabb\ud558\uc600\ub2e4. \uc774\ub7ec\ud55c \uc810\uc744 \uac1c\uc120\ud558\uae30 \uc704\ud574 \uc804\ubbf8\uad50\uc721\ud611\ud68c\ub294 7\uc778 \uc704\uc6d0\ud68c\ub97c \uc870\uc9c1\ud558\uc5ec \uc804\ud1b5\uc801\uc778 \uc5ed\uc0ac\uad50\uc721\uc5d0\uc11c \ubc97\uc5b4\ub098 \uacfc\ud559\uc801\uc778 \ud504\ub85c\uadf8\ub7a8\uc744 \uc81c\uc2dc\ud558\uc600\uc73c\ub098, \uc774 \ub610\ud55c \uc2dc\ubbfc\uad50\uc721\uc758 \uc774\uc0c1\uc5d0 \ubbf8\uce58\uc9c0 \ubabb\ud558\uc600\ub2e4. \uc774\uc5d0 \uc804\ubbf8\uad50\uc721\ud611\ud68c\uc5d0\uc11c \uc0ac\ud68c\uacfc\uc704\uc6d0\ud68c\uac00 \ub4f1\uc7a5\ud558\uc5ec 1916\ub144\uc5d0 '\uc911\ub4f1\uad50\uc721\uc758 \uc0ac\ud68c\uacfc' \ub4f1\uc758 \ubcf4\uace0\uc11c\ub97c \ud1b5\ud558\uc5ec \uc0ac\ud68c\uacfc\ub77c\ub294 \uad50\uacfc\ub97c \ucc3d\uc124\ud560 \uac83\uc744 \uc8fc\uc7a5\ud558\uc600\ub2e4.", "answer": "7\uc778 \uc704\uc6d0\ud68c", "sentence": "\uc804\ubbf8\uad50\uc721\ud611\ud68c\ub294 7\uc778 \uc704\uc6d0\ud68c \ub97c \uc870\uc9c1\ud558\uc5ec \uc804\ud1b5\uc801\uc778 \uc5ed\uc0ac\uad50\uc721\uc5d0\uc11c \ubc97\uc5b4\ub098 \uacfc\ud559\uc801\uc778 \ud504\ub85c\uadf8\ub7a8\uc744 \uc81c\uc2dc\ud558\uc600\uc73c\ub098, \uc774 \ub610\ud55c \uc2dc\ubbfc\uad50\uc721\uc758 \uc774\uc0c1\uc5d0 \ubbf8\uce58\uc9c0 \ubabb\ud558\uc600\ub2e4.", "paragraph_sentence": "\uc2dc\ubbfc\uad50\uc721\uc740 \uc778\ub958\uac00 \uc0ac\ud68c\ub97c \ud615\uc131\ud558\uba74\uc11c \uc2dc\uc791\ub418\uc5c8\uc73c\ub098, \uc5ec\ub7ec \uc0ac\ud68c\uacfc\ud559 \ud559\ubb38\uc744 \ud1b5\ud569\ud558\uc5ec \uc0ac\ud68c\uacfc\ub85c \ub4f1\uc7a5\ud55c \uac83\uc740 20\uc138\uae30 \ucd08\ubc18, \ubbf8\uad6d \uc0ac\ud68c\uc600\ub2e4. \ub2f9\uc2dc \ubbf8\uad6d\uc740 \uc774\ubbfc\uc790\ub97c \ubbf8\uad6d\ud654\ud558\uace0, \ubbf8\uad6d \ub0a8\ubd80\uc5d0\uc11c \ubd81\ubd80\ub85c \uc774\uc8fc\ud55c \uc774\uc8fc\ubbfc\uc774 \ub3c4\uc2dc\uc5d0 \uc801\uc751\ud558\ub3c4\ub85d \uad50\uc721\uc744 \uc2e4\uc2dc\ud560 \ud544\uc694\uc131\uc774 \ub300\ub450\ub418\uc5c8\ub2e4. \uc774\ub7ec\ud55c \uc0ac\ud68c\uc801 \uc694\uad6c\ub294 \uc9c0\uc2dd \uc804\ub2ec \uc911\uc2ec\uc758 \uc804\ud1b5\uc801 \uad50\uc721\uc5d0 \ub300\ud55c \uac1c\ud601\uc744 \uc694\uad6c\ud558\ub294 \ud615\ud0dc\ub85c \ub098\ud0c0\ub0ac\uace0, \uc774 \uc694\uad6c\uc5d0 \ubd80\uc751\ud558\uc5ec 1892\ub144 \uc804\ubbf8\uad50\uc721\ud611\ud68c\uc5d0\uc11c 10\uc778 \uc704\uc6d0\ud68c\ub97c \uc870\uc9c1\ud574 \uc9c1\uc5c5\uc0dd\ud65c\uacfc \uc2dc\ubbfc\uad50\uc721\uc744 \ud3ec\ud568\ud55c \uad50\uc721\uacfc\uc815\uc744 \uc81c\uc2dc\ud558\uc600\uc73c\ub098, \uc5ec\uc804\ud788 \uc9c0\uc2dd \uc804\ub2ec \uc911\uc2ec\uc758 \uc804\ud1b5\uc801 \uad50\uc721\uc744 \ubc97\uc5b4\ub098\uc9c0 \ubabb\ud558\uc600\ub2e4. \uc774\ub7ec\ud55c \uc810\uc744 \uac1c\uc120\ud558\uae30 \uc704\ud574 \uc804\ubbf8\uad50\uc721\ud611\ud68c\ub294 7\uc778 \uc704\uc6d0\ud68c \ub97c \uc870\uc9c1\ud558\uc5ec \uc804\ud1b5\uc801\uc778 \uc5ed\uc0ac\uad50\uc721\uc5d0\uc11c \ubc97\uc5b4\ub098 \uacfc\ud559\uc801\uc778 \ud504\ub85c\uadf8\ub7a8\uc744 \uc81c\uc2dc\ud558\uc600\uc73c\ub098, \uc774 \ub610\ud55c \uc2dc\ubbfc\uad50\uc721\uc758 \uc774\uc0c1\uc5d0 \ubbf8\uce58\uc9c0 \ubabb\ud558\uc600\ub2e4. \uc774\uc5d0 \uc804\ubbf8\uad50\uc721\ud611\ud68c\uc5d0\uc11c \uc0ac\ud68c\uacfc\uc704\uc6d0\ud68c\uac00 \ub4f1\uc7a5\ud558\uc5ec 1916\ub144\uc5d0 '\uc911\ub4f1\uad50\uc721\uc758 \uc0ac\ud68c\uacfc' \ub4f1\uc758 \ubcf4\uace0\uc11c\ub97c \ud1b5\ud558\uc5ec \uc0ac\ud68c\uacfc\ub77c\ub294 \uad50\uacfc\ub97c \ucc3d\uc124\ud560 \uac83\uc744 \uc8fc\uc7a5\ud558\uc600\ub2e4.", "paragraph_answer": "\uc2dc\ubbfc\uad50\uc721\uc740 \uc778\ub958\uac00 \uc0ac\ud68c\ub97c \ud615\uc131\ud558\uba74\uc11c \uc2dc\uc791\ub418\uc5c8\uc73c\ub098, \uc5ec\ub7ec \uc0ac\ud68c\uacfc\ud559 \ud559\ubb38\uc744 \ud1b5\ud569\ud558\uc5ec \uc0ac\ud68c\uacfc\ub85c \ub4f1\uc7a5\ud55c \uac83\uc740 20\uc138\uae30 \ucd08\ubc18, \ubbf8\uad6d \uc0ac\ud68c\uc600\ub2e4. \ub2f9\uc2dc \ubbf8\uad6d\uc740 \uc774\ubbfc\uc790\ub97c \ubbf8\uad6d\ud654\ud558\uace0, \ubbf8\uad6d \ub0a8\ubd80\uc5d0\uc11c \ubd81\ubd80\ub85c \uc774\uc8fc\ud55c \uc774\uc8fc\ubbfc\uc774 \ub3c4\uc2dc\uc5d0 \uc801\uc751\ud558\ub3c4\ub85d \uad50\uc721\uc744 \uc2e4\uc2dc\ud560 \ud544\uc694\uc131\uc774 \ub300\ub450\ub418\uc5c8\ub2e4. \uc774\ub7ec\ud55c \uc0ac\ud68c\uc801 \uc694\uad6c\ub294 \uc9c0\uc2dd \uc804\ub2ec \uc911\uc2ec\uc758 \uc804\ud1b5\uc801 \uad50\uc721\uc5d0 \ub300\ud55c \uac1c\ud601\uc744 \uc694\uad6c\ud558\ub294 \ud615\ud0dc\ub85c \ub098\ud0c0\ub0ac\uace0, \uc774 \uc694\uad6c\uc5d0 \ubd80\uc751\ud558\uc5ec 1892\ub144 \uc804\ubbf8\uad50\uc721\ud611\ud68c\uc5d0\uc11c 10\uc778 \uc704\uc6d0\ud68c\ub97c \uc870\uc9c1\ud574 \uc9c1\uc5c5\uc0dd\ud65c\uacfc \uc2dc\ubbfc\uad50\uc721\uc744 \ud3ec\ud568\ud55c \uad50\uc721\uacfc\uc815\uc744 \uc81c\uc2dc\ud558\uc600\uc73c\ub098, \uc5ec\uc804\ud788 \uc9c0\uc2dd \uc804\ub2ec \uc911\uc2ec\uc758 \uc804\ud1b5\uc801 \uad50\uc721\uc744 \ubc97\uc5b4\ub098\uc9c0 \ubabb\ud558\uc600\ub2e4. \uc774\ub7ec\ud55c \uc810\uc744 \uac1c\uc120\ud558\uae30 \uc704\ud574 \uc804\ubbf8\uad50\uc721\ud611\ud68c\ub294 7\uc778 \uc704\uc6d0\ud68c \ub97c \uc870\uc9c1\ud558\uc5ec \uc804\ud1b5\uc801\uc778 \uc5ed\uc0ac\uad50\uc721\uc5d0\uc11c \ubc97\uc5b4\ub098 \uacfc\ud559\uc801\uc778 \ud504\ub85c\uadf8\ub7a8\uc744 \uc81c\uc2dc\ud558\uc600\uc73c\ub098, \uc774 \ub610\ud55c \uc2dc\ubbfc\uad50\uc721\uc758 \uc774\uc0c1\uc5d0 \ubbf8\uce58\uc9c0 \ubabb\ud558\uc600\ub2e4. \uc774\uc5d0 \uc804\ubbf8\uad50\uc721\ud611\ud68c\uc5d0\uc11c \uc0ac\ud68c\uacfc\uc704\uc6d0\ud68c\uac00 \ub4f1\uc7a5\ud558\uc5ec 1916\ub144\uc5d0 '\uc911\ub4f1\uad50\uc721\uc758 \uc0ac\ud68c\uacfc' \ub4f1\uc758 \ubcf4\uace0\uc11c\ub97c \ud1b5\ud558\uc5ec \uc0ac\ud68c\uacfc\ub77c\ub294 \uad50\uacfc\ub97c \ucc3d\uc124\ud560 \uac83\uc744 \uc8fc\uc7a5\ud558\uc600\ub2e4.", "sentence_answer": "\uc804\ubbf8\uad50\uc721\ud611\ud68c\ub294 7\uc778 \uc704\uc6d0\ud68c \ub97c \uc870\uc9c1\ud558\uc5ec \uc804\ud1b5\uc801\uc778 \uc5ed\uc0ac\uad50\uc721\uc5d0\uc11c \ubc97\uc5b4\ub098 \uacfc\ud559\uc801\uc778 \ud504\ub85c\uadf8\ub7a8\uc744 \uc81c\uc2dc\ud558\uc600\uc73c\ub098, \uc774 \ub610\ud55c \uc2dc\ubbfc\uad50\uc721\uc758 \uc774\uc0c1\uc5d0 \ubbf8\uce58\uc9c0 \ubabb\ud558\uc600\ub2e4.", "paragraph_id": "6591052-0-1", "paragraph_question": "question: \uc804\ubbf8\uad50\uc721\ud611\ud68c\uac00 \uc804\ud1b5\uc801 \uc5ed\uc0ac\uad50\uc721\uc744 \ubc97\uc5b4\ub098 \uacfc\ud559\uc801 \ud504\ub85c\uadf8\ub7a8\uc744 \uc81c\uc2dc\ud558\uae30 \uc704\ud574 \uc870\uc9c1\ud55c \uac83\uc740?, context: \uc2dc\ubbfc\uad50\uc721\uc740 \uc778\ub958\uac00 \uc0ac\ud68c\ub97c \ud615\uc131\ud558\uba74\uc11c \uc2dc\uc791\ub418\uc5c8\uc73c\ub098, \uc5ec\ub7ec \uc0ac\ud68c\uacfc\ud559 \ud559\ubb38\uc744 \ud1b5\ud569\ud558\uc5ec \uc0ac\ud68c\uacfc\ub85c \ub4f1\uc7a5\ud55c \uac83\uc740 20\uc138\uae30 \ucd08\ubc18, \ubbf8\uad6d \uc0ac\ud68c\uc600\ub2e4. \ub2f9\uc2dc \ubbf8\uad6d\uc740 \uc774\ubbfc\uc790\ub97c \ubbf8\uad6d\ud654\ud558\uace0, \ubbf8\uad6d \ub0a8\ubd80\uc5d0\uc11c \ubd81\ubd80\ub85c \uc774\uc8fc\ud55c \uc774\uc8fc\ubbfc\uc774 \ub3c4\uc2dc\uc5d0 \uc801\uc751\ud558\ub3c4\ub85d \uad50\uc721\uc744 \uc2e4\uc2dc\ud560 \ud544\uc694\uc131\uc774 \ub300\ub450\ub418\uc5c8\ub2e4. \uc774\ub7ec\ud55c \uc0ac\ud68c\uc801 \uc694\uad6c\ub294 \uc9c0\uc2dd \uc804\ub2ec \uc911\uc2ec\uc758 \uc804\ud1b5\uc801 \uad50\uc721\uc5d0 \ub300\ud55c \uac1c\ud601\uc744 \uc694\uad6c\ud558\ub294 \ud615\ud0dc\ub85c \ub098\ud0c0\ub0ac\uace0, \uc774 \uc694\uad6c\uc5d0 \ubd80\uc751\ud558\uc5ec 1892\ub144 \uc804\ubbf8\uad50\uc721\ud611\ud68c\uc5d0\uc11c 10\uc778 \uc704\uc6d0\ud68c\ub97c \uc870\uc9c1\ud574 \uc9c1\uc5c5\uc0dd\ud65c\uacfc \uc2dc\ubbfc\uad50\uc721\uc744 \ud3ec\ud568\ud55c \uad50\uc721\uacfc\uc815\uc744 \uc81c\uc2dc\ud558\uc600\uc73c\ub098, \uc5ec\uc804\ud788 \uc9c0\uc2dd \uc804\ub2ec \uc911\uc2ec\uc758 \uc804\ud1b5\uc801 \uad50\uc721\uc744 \ubc97\uc5b4\ub098\uc9c0 \ubabb\ud558\uc600\ub2e4. \uc774\ub7ec\ud55c \uc810\uc744 \uac1c\uc120\ud558\uae30 \uc704\ud574 \uc804\ubbf8\uad50\uc721\ud611\ud68c\ub294 7\uc778 \uc704\uc6d0\ud68c\ub97c \uc870\uc9c1\ud558\uc5ec \uc804\ud1b5\uc801\uc778 \uc5ed\uc0ac\uad50\uc721\uc5d0\uc11c \ubc97\uc5b4\ub098 \uacfc\ud559\uc801\uc778 \ud504\ub85c\uadf8\ub7a8\uc744 \uc81c\uc2dc\ud558\uc600\uc73c\ub098, \uc774 \ub610\ud55c \uc2dc\ubbfc\uad50\uc721\uc758 \uc774\uc0c1\uc5d0 \ubbf8\uce58\uc9c0 \ubabb\ud558\uc600\ub2e4. \uc774\uc5d0 \uc804\ubbf8\uad50\uc721\ud611\ud68c\uc5d0\uc11c \uc0ac\ud68c\uacfc\uc704\uc6d0\ud68c\uac00 \ub4f1\uc7a5\ud558\uc5ec 1916\ub144\uc5d0 '\uc911\ub4f1\uad50\uc721\uc758 \uc0ac\ud68c\uacfc' \ub4f1\uc758 \ubcf4\uace0\uc11c\ub97c \ud1b5\ud558\uc5ec \uc0ac\ud68c\uacfc\ub77c\ub294 \uad50\uacfc\ub97c \ucc3d\uc124\ud560 \uac83\uc744 \uc8fc\uc7a5\ud558\uc600\ub2e4."} {"question": "\uc11c\uc77c\ubcf8 \uc5ec\uac1d\ucca0\ub3c4\uc5d0\uc11c \uc9c0\uc5ed \ub2f9\uad6d\ucabd\uc5d0 \uc0c8 \uad50\ub7c9 \uac74\uc124\uc744 \uc81c\uc548\ud55c \ub144\ub3c4\ub294?", "paragraph": "1986\ub144\uc5d0 \uc77c\uc5b4\ub09c \uc5f4\ucc28 \ucd94\ub77d\uc0ac\uace0 \uc774\ud6c4 \uc6b4\ud589 \uaddc\uc81c\uac00 \uac15\ud654\ub418\uba74\uc11c \uc5f4\ucc28 \uc6b4\ud589 \ud734\uc9c0\ub098 \uc9c0\uc5f0\uc774 \uc790\uc8fc \ubc1c\uc0dd\ud558\uc600\uace0 (1994\ub144\ubd80\ud130 2003\ub144 \uc0ac\uc774\uc5d0\ub9cc 120\ubc88\uc758 \uc5f4\ucc28 \uc815\uc9c0\uc640 84\ubc88\uc758 \uc6b4\ud589 \ud734\uc9c0\uac00 \uc77c\uc5b4\ub0ac\ub2e4.), \uc774\ub85c \uc778\ud574 \uc5f0\uc120 \uc9c0\uc5ed \uc8fc\ubbfc\ub4e4\uc758 \ud1b5\uadfc\u00b7\ud1b5\ud559\uc774\ub098 \uacbd\uc81c \ud65c\ub3d9\uc5d0 \ud070 \uc9c0\uc7a5\uc744 \uc8fc\uac8c \ub418\uc790 \uc774\ub7ec\ud55c \ubb38\uc81c\ub4e4\uc758 \ud574\uc18c\ub97c \uc704\ud574 1991\ub144 3\uc6d4 \ub3d7\ud1a0\ub9ac\ud604\uacfc \ud6a8\uace0 \ud604\uc744 \ube44\ub86f\ud55c \uc9c0\ubc29\uc790\uce58\ub2e8\uccb4\ub97c \uc911\uc2ec\uc73c\ub85c '\uc544\ub9c8\ub8e8\ubca0 \ucca0\uad50 \ub300\ucc45 \ud611\uc758\ud68c'\uac00 \uc138\uc6cc\uc9c0\uae30\uc5d0 \uc774\ub974\ub800\ub2e4. \ub300\uc548\uc73c\ub85c \uc0c8 \ucca0\ub3c4\ub97c \uac74\uc124\ud558\ub294 \uac83\uc774 \uac80\ud1a0\ub418\uc5c8\uc73c\ub098 \uae30\uc874 \uc5ed\uc758 \uc774\uc804 \ub4f1\uc73c\ub85c \ub9ce\uc740 \ube44\uc6a9\uc774 \ub4e4\uae30\uc5d0 \ubc18\ub824\ub418\uc5c8\ub2e4. \ub300\uc2e0 \uc61b \uad50\ub7c9\uc5d0 \ubc29\ud48d\ubcbd\uc744 \uc124\uce58\ud558\ub294 \uc548\uc774 1994\ub144 3\uc6d4\ubd80\ud130 \uac80\ud1a0\ub418\uc5c8\ub2e4. \uadf8\ub7ec\ub098 \ud48d\ub3d9(\u98a8\u6d1e) \uc2e4\ud5d8 \ub4f1\uc744 \ud1b5\ud574 \ubcf4\uac15\ud55c \ubc29\ud48d\ubcbd\uc744 \uc124\uce58\ud568\uc73c\ub85c\uc368 \uac15\ud48d\uc5d0 \uc5b4\ub290 \uc815\ub3c4 \ub300\uc751\ud560 \uc218\ub294 \uc788\uc9c0\ub9cc, \uc61b \uad50\ub7c9\uc774 \ub108\ubb34 \uc624\ub798\ub418\uc5c8\uae30 \ub54c\ubb38\uc5d0 \uc7a5\uae30\uc801\uc778 \uc548\uc804\uc131\uc744 \ud655\ubcf4\ud558\uae30 \uc5b4\ub835\ub2e4\ub294 \uc774\uc720\ub85c \uc774 \uc548 \ub610\ud55c \ubc18\ub824\ub418\uc5c8\ub2e4. 2001\ub144 11\uc6d4 22\uc77c \uc11c\uc77c\ubcf8 \uc5ec\uac1d\ucca0\ub3c4\ub294 \uc9c0\uc5ed \ub2f9\uad6d \ucabd\uc5d0 \uc0c8 \uad50\ub7c9\uc744 \uac74\uc124\ud558\ub294 \uac83\uc744 \uc81c\uc758\ud588\ub2e4.", "answer": "2001\ub144", "sentence": "2001\ub144 11\uc6d4 22\uc77c \uc11c\uc77c\ubcf8 \uc5ec\uac1d\ucca0\ub3c4\ub294 \uc9c0\uc5ed \ub2f9\uad6d \ucabd\uc5d0 \uc0c8 \uad50\ub7c9\uc744 \uac74\uc124\ud558\ub294 \uac83\uc744 \uc81c\uc758\ud588\ub2e4.", "paragraph_sentence": "1986\ub144\uc5d0 \uc77c\uc5b4\ub09c \uc5f4\ucc28 \ucd94\ub77d\uc0ac\uace0 \uc774\ud6c4 \uc6b4\ud589 \uaddc\uc81c\uac00 \uac15\ud654\ub418\uba74\uc11c \uc5f4\ucc28 \uc6b4\ud589 \ud734\uc9c0\ub098 \uc9c0\uc5f0\uc774 \uc790\uc8fc \ubc1c\uc0dd\ud558\uc600\uace0 (1994\ub144\ubd80\ud130 2003\ub144 \uc0ac\uc774\uc5d0\ub9cc 120\ubc88\uc758 \uc5f4\ucc28 \uc815\uc9c0\uc640 84\ubc88\uc758 \uc6b4\ud589 \ud734\uc9c0\uac00 \uc77c\uc5b4\ub0ac\ub2e4. ), \uc774\ub85c \uc778\ud574 \uc5f0\uc120 \uc9c0\uc5ed \uc8fc\ubbfc\ub4e4\uc758 \ud1b5\uadfc\u00b7\ud1b5\ud559\uc774\ub098 \uacbd\uc81c \ud65c\ub3d9\uc5d0 \ud070 \uc9c0\uc7a5\uc744 \uc8fc\uac8c \ub418\uc790 \uc774\ub7ec\ud55c \ubb38\uc81c\ub4e4\uc758 \ud574\uc18c\ub97c \uc704\ud574 1991\ub144 3\uc6d4 \ub3d7\ud1a0\ub9ac\ud604\uacfc \ud6a8\uace0 \ud604\uc744 \ube44\ub86f\ud55c \uc9c0\ubc29\uc790\uce58\ub2e8\uccb4\ub97c \uc911\uc2ec\uc73c\ub85c '\uc544\ub9c8\ub8e8\ubca0 \ucca0\uad50 \ub300\ucc45 \ud611\uc758\ud68c'\uac00 \uc138\uc6cc\uc9c0\uae30\uc5d0 \uc774\ub974\ub800\ub2e4. \ub300\uc548\uc73c\ub85c \uc0c8 \ucca0\ub3c4\ub97c \uac74\uc124\ud558\ub294 \uac83\uc774 \uac80\ud1a0\ub418\uc5c8\uc73c\ub098 \uae30\uc874 \uc5ed\uc758 \uc774\uc804 \ub4f1\uc73c\ub85c \ub9ce\uc740 \ube44\uc6a9\uc774 \ub4e4\uae30\uc5d0 \ubc18\ub824\ub418\uc5c8\ub2e4. \ub300\uc2e0 \uc61b \uad50\ub7c9\uc5d0 \ubc29\ud48d\ubcbd\uc744 \uc124\uce58\ud558\ub294 \uc548\uc774 1994\ub144 3\uc6d4\ubd80\ud130 \uac80\ud1a0\ub418\uc5c8\ub2e4. \uadf8\ub7ec\ub098 \ud48d\ub3d9(\u98a8\u6d1e) \uc2e4\ud5d8 \ub4f1\uc744 \ud1b5\ud574 \ubcf4\uac15\ud55c \ubc29\ud48d\ubcbd\uc744 \uc124\uce58\ud568\uc73c\ub85c\uc368 \uac15\ud48d\uc5d0 \uc5b4\ub290 \uc815\ub3c4 \ub300\uc751\ud560 \uc218\ub294 \uc788\uc9c0\ub9cc, \uc61b \uad50\ub7c9\uc774 \ub108\ubb34 \uc624\ub798\ub418\uc5c8\uae30 \ub54c\ubb38\uc5d0 \uc7a5\uae30\uc801\uc778 \uc548\uc804\uc131\uc744 \ud655\ubcf4\ud558\uae30 \uc5b4\ub835\ub2e4\ub294 \uc774\uc720\ub85c \uc774 \uc548 \ub610\ud55c \ubc18\ub824\ub418\uc5c8\ub2e4. 2001\ub144 11\uc6d4 22\uc77c \uc11c\uc77c\ubcf8 \uc5ec\uac1d\ucca0\ub3c4\ub294 \uc9c0\uc5ed \ub2f9\uad6d \ucabd\uc5d0 \uc0c8 \uad50\ub7c9\uc744 \uac74\uc124\ud558\ub294 \uac83\uc744 \uc81c\uc758\ud588\ub2e4. ", "paragraph_answer": "1986\ub144\uc5d0 \uc77c\uc5b4\ub09c \uc5f4\ucc28 \ucd94\ub77d\uc0ac\uace0 \uc774\ud6c4 \uc6b4\ud589 \uaddc\uc81c\uac00 \uac15\ud654\ub418\uba74\uc11c \uc5f4\ucc28 \uc6b4\ud589 \ud734\uc9c0\ub098 \uc9c0\uc5f0\uc774 \uc790\uc8fc \ubc1c\uc0dd\ud558\uc600\uace0 (1994\ub144\ubd80\ud130 2003\ub144 \uc0ac\uc774\uc5d0\ub9cc 120\ubc88\uc758 \uc5f4\ucc28 \uc815\uc9c0\uc640 84\ubc88\uc758 \uc6b4\ud589 \ud734\uc9c0\uac00 \uc77c\uc5b4\ub0ac\ub2e4.), \uc774\ub85c \uc778\ud574 \uc5f0\uc120 \uc9c0\uc5ed \uc8fc\ubbfc\ub4e4\uc758 \ud1b5\uadfc\u00b7\ud1b5\ud559\uc774\ub098 \uacbd\uc81c \ud65c\ub3d9\uc5d0 \ud070 \uc9c0\uc7a5\uc744 \uc8fc\uac8c \ub418\uc790 \uc774\ub7ec\ud55c \ubb38\uc81c\ub4e4\uc758 \ud574\uc18c\ub97c \uc704\ud574 1991\ub144 3\uc6d4 \ub3d7\ud1a0\ub9ac\ud604\uacfc \ud6a8\uace0 \ud604\uc744 \ube44\ub86f\ud55c \uc9c0\ubc29\uc790\uce58\ub2e8\uccb4\ub97c \uc911\uc2ec\uc73c\ub85c '\uc544\ub9c8\ub8e8\ubca0 \ucca0\uad50 \ub300\ucc45 \ud611\uc758\ud68c'\uac00 \uc138\uc6cc\uc9c0\uae30\uc5d0 \uc774\ub974\ub800\ub2e4. \ub300\uc548\uc73c\ub85c \uc0c8 \ucca0\ub3c4\ub97c \uac74\uc124\ud558\ub294 \uac83\uc774 \uac80\ud1a0\ub418\uc5c8\uc73c\ub098 \uae30\uc874 \uc5ed\uc758 \uc774\uc804 \ub4f1\uc73c\ub85c \ub9ce\uc740 \ube44\uc6a9\uc774 \ub4e4\uae30\uc5d0 \ubc18\ub824\ub418\uc5c8\ub2e4. \ub300\uc2e0 \uc61b \uad50\ub7c9\uc5d0 \ubc29\ud48d\ubcbd\uc744 \uc124\uce58\ud558\ub294 \uc548\uc774 1994\ub144 3\uc6d4\ubd80\ud130 \uac80\ud1a0\ub418\uc5c8\ub2e4. \uadf8\ub7ec\ub098 \ud48d\ub3d9(\u98a8\u6d1e) \uc2e4\ud5d8 \ub4f1\uc744 \ud1b5\ud574 \ubcf4\uac15\ud55c \ubc29\ud48d\ubcbd\uc744 \uc124\uce58\ud568\uc73c\ub85c\uc368 \uac15\ud48d\uc5d0 \uc5b4\ub290 \uc815\ub3c4 \ub300\uc751\ud560 \uc218\ub294 \uc788\uc9c0\ub9cc, \uc61b \uad50\ub7c9\uc774 \ub108\ubb34 \uc624\ub798\ub418\uc5c8\uae30 \ub54c\ubb38\uc5d0 \uc7a5\uae30\uc801\uc778 \uc548\uc804\uc131\uc744 \ud655\ubcf4\ud558\uae30 \uc5b4\ub835\ub2e4\ub294 \uc774\uc720\ub85c \uc774 \uc548 \ub610\ud55c \ubc18\ub824\ub418\uc5c8\ub2e4. 2001\ub144 11\uc6d4 22\uc77c \uc11c\uc77c\ubcf8 \uc5ec\uac1d\ucca0\ub3c4\ub294 \uc9c0\uc5ed \ub2f9\uad6d \ucabd\uc5d0 \uc0c8 \uad50\ub7c9\uc744 \uac74\uc124\ud558\ub294 \uac83\uc744 \uc81c\uc758\ud588\ub2e4.", "sentence_answer": " 2001\ub144 11\uc6d4 22\uc77c \uc11c\uc77c\ubcf8 \uc5ec\uac1d\ucca0\ub3c4\ub294 \uc9c0\uc5ed \ub2f9\uad6d \ucabd\uc5d0 \uc0c8 \uad50\ub7c9\uc744 \uac74\uc124\ud558\ub294 \uac83\uc744 \uc81c\uc758\ud588\ub2e4.", "paragraph_id": "6520384-17-1", "paragraph_question": "question: \uc11c\uc77c\ubcf8 \uc5ec\uac1d\ucca0\ub3c4\uc5d0\uc11c \uc9c0\uc5ed \ub2f9\uad6d\ucabd\uc5d0 \uc0c8 \uad50\ub7c9 \uac74\uc124\uc744 \uc81c\uc548\ud55c \ub144\ub3c4\ub294?, context: 1986\ub144\uc5d0 \uc77c\uc5b4\ub09c \uc5f4\ucc28 \ucd94\ub77d\uc0ac\uace0 \uc774\ud6c4 \uc6b4\ud589 \uaddc\uc81c\uac00 \uac15\ud654\ub418\uba74\uc11c \uc5f4\ucc28 \uc6b4\ud589 \ud734\uc9c0\ub098 \uc9c0\uc5f0\uc774 \uc790\uc8fc \ubc1c\uc0dd\ud558\uc600\uace0 (1994\ub144\ubd80\ud130 2003\ub144 \uc0ac\uc774\uc5d0\ub9cc 120\ubc88\uc758 \uc5f4\ucc28 \uc815\uc9c0\uc640 84\ubc88\uc758 \uc6b4\ud589 \ud734\uc9c0\uac00 \uc77c\uc5b4\ub0ac\ub2e4.), \uc774\ub85c \uc778\ud574 \uc5f0\uc120 \uc9c0\uc5ed \uc8fc\ubbfc\ub4e4\uc758 \ud1b5\uadfc\u00b7\ud1b5\ud559\uc774\ub098 \uacbd\uc81c \ud65c\ub3d9\uc5d0 \ud070 \uc9c0\uc7a5\uc744 \uc8fc\uac8c \ub418\uc790 \uc774\ub7ec\ud55c \ubb38\uc81c\ub4e4\uc758 \ud574\uc18c\ub97c \uc704\ud574 1991\ub144 3\uc6d4 \ub3d7\ud1a0\ub9ac\ud604\uacfc \ud6a8\uace0 \ud604\uc744 \ube44\ub86f\ud55c \uc9c0\ubc29\uc790\uce58\ub2e8\uccb4\ub97c \uc911\uc2ec\uc73c\ub85c '\uc544\ub9c8\ub8e8\ubca0 \ucca0\uad50 \ub300\ucc45 \ud611\uc758\ud68c'\uac00 \uc138\uc6cc\uc9c0\uae30\uc5d0 \uc774\ub974\ub800\ub2e4. \ub300\uc548\uc73c\ub85c \uc0c8 \ucca0\ub3c4\ub97c \uac74\uc124\ud558\ub294 \uac83\uc774 \uac80\ud1a0\ub418\uc5c8\uc73c\ub098 \uae30\uc874 \uc5ed\uc758 \uc774\uc804 \ub4f1\uc73c\ub85c \ub9ce\uc740 \ube44\uc6a9\uc774 \ub4e4\uae30\uc5d0 \ubc18\ub824\ub418\uc5c8\ub2e4. \ub300\uc2e0 \uc61b \uad50\ub7c9\uc5d0 \ubc29\ud48d\ubcbd\uc744 \uc124\uce58\ud558\ub294 \uc548\uc774 1994\ub144 3\uc6d4\ubd80\ud130 \uac80\ud1a0\ub418\uc5c8\ub2e4. \uadf8\ub7ec\ub098 \ud48d\ub3d9(\u98a8\u6d1e) \uc2e4\ud5d8 \ub4f1\uc744 \ud1b5\ud574 \ubcf4\uac15\ud55c \ubc29\ud48d\ubcbd\uc744 \uc124\uce58\ud568\uc73c\ub85c\uc368 \uac15\ud48d\uc5d0 \uc5b4\ub290 \uc815\ub3c4 \ub300\uc751\ud560 \uc218\ub294 \uc788\uc9c0\ub9cc, \uc61b \uad50\ub7c9\uc774 \ub108\ubb34 \uc624\ub798\ub418\uc5c8\uae30 \ub54c\ubb38\uc5d0 \uc7a5\uae30\uc801\uc778 \uc548\uc804\uc131\uc744 \ud655\ubcf4\ud558\uae30 \uc5b4\ub835\ub2e4\ub294 \uc774\uc720\ub85c \uc774 \uc548 \ub610\ud55c \ubc18\ub824\ub418\uc5c8\ub2e4. 2001\ub144 11\uc6d4 22\uc77c \uc11c\uc77c\ubcf8 \uc5ec\uac1d\ucca0\ub3c4\ub294 \uc9c0\uc5ed \ub2f9\uad6d \ucabd\uc5d0 \uc0c8 \uad50\ub7c9\uc744 \uac74\uc124\ud558\ub294 \uac83\uc744 \uc81c\uc758\ud588\ub2e4."} {"question": "\uc815\uc131\ud0dd\uc758 \uc219\uccad \uc0ac\uc2e4\uc744 \uc678\ubd80\uc5d0 \uc54c\ub9b0 \uc778\ubb3c\uc740?", "paragraph": "\uc548\ucc3d\uc5f0\uc774 \uc219\uccad\ub2f9\ud55c \uc774\uc720\ub294 \uc644\ubcbd\ud788 \uc54c \uc21c \uc5c6\uc73c\ub098, \ub300\ud55c\ubbfc\uad6d\uc758 \ub300\ubd81 \uc804\ubb38\uac00\ub4e4\uc740 \uc7a5\uc131\ud0dd\uc758 \uc219\uccad \ubc30\uacbd\uc5d0 \ub300\ud574 \ub2e4\uc591\ud55c \uc758\uacac\uc744 \ub0b4\ub193\uc558\ub2e4. 12\uc6d4 3\uc77c \uad6d\uc815\uc6d0\uc774 \uc7a5\uc131\ud0dd\uc758 \uc2e4\uac01 \uac00\ub2a5\uc131\uc744 \ubc1c\ud45c\ud55c \uc774\ud6c4 \ub0a8\ud55c\uc5d0\uc11c\ub294 \uc7a5\uc131\ud0dd\uacfc \uadf8\uc758 \ub77c\uc774\ubc8c\ub85c \uc54c\ub824\uc9c4 \ucd5c\ub8e1\ud574\uc758 \uad00\uacc4\uc5d0 \uc8fc\ubaa9\ud588\ub2e4. \uc7a5\uc131\ud0dd\uc758 \uc219\uccad \uc0ac\uc2e4\uc744 \uad6d\uc815\uc6d0\uc73c\ub85c\ubd80\ud130 \ubcf4\uace0\ubc1b\uace0 \uc774\ub97c \ub300\uc678\uc801\uc73c\ub85c \uc54c\ub9b0 \uc815\uccad\ub798 \ubbfc\uc8fc\ub2f9 \uc758\uc6d0\uc740 \uc7a5\uc131\ud0dd\uc744 \uc81c\uac70\ud55c \uac83\uc774 \ucd5c\ub8e1\ud574 \uad70 \ucd1d\uc815\uce58\uad6d\uc7a5\uc73c\ub85c \ubcf4\uc778\ub2e4\uace0 \ubc1d\ud614\ub2e4. \ucd5c\ub8e1\ud574\ub294 \ucd5c\ud604 \uc804 \uc778\ubbfc\ubb34\ub825\ubd80\uc7a5\uc758 \uc544\ub4e4\uc774\ub2e4. \uc870\uc120\uc77c\ubcf4\ub294 \ucd5c\ub8e1\ud574\ub85c \ub300\ud45c\ub418\ub294 \ud6c8\uc2e0(\u52f3\u81e3) \uc138\ub825\uacfc \uc7a5\uc131\ud0dd \ub4f1 \uae40\uc815\uc740\uc758 \ucc99\uc2e0(\u621a\u81e3)\uc758 \ub300\uacb0\uc5d0\uc11c \ud6c8\uc2e0 \uc138\ub825\uc774 \uc2b9\ub9ac\ud588\ub2e4\ub294 \ubd84\uc11d\uc744 \ub0b4\ub1a8\ub2e4. \ub300\ubd81 \uc804\ubb38\uac00 \uae40\uadfc\uc2dd \uacbd\ub0a8\ub300 \uad50\uc218\ub3c4 \"\ucd5c\ub8e1\ud574\ub97c \uc911\uc2ec\uc73c\ub85c \ud55c \uc2e0\uad70\ubd80\uc138\ub825\uc774 \ubc00\uc5b4\ub0b8 \uac83 \uc544\ub2cc\uac00 \ud574\uc11d\uc774 \uac00\ub2a5\ud558\ub2e4\"\uba70 \ubd81\uc870\uc120 \ub0b4 \ubcf4\uc218\uc138\ub825\uc758 \uc785\uc7a5\uc774 \uad00\ucca0\ub41c \uacb0\uacfc\ub77c\uace0 \ud574\uc11d\ud588\ub2e4. \uadf8 \uc774\uc720\ub85c \uae40\uadfc\uc2dd \uad50\uc218\ub294 \uc7a5\uc131\ud0dd\uc774 \uc911\uad6d, \ub0a8\ud55c \ub4f1\uacfc\uc758 \uacbd\uc81c\uad50\ub958\uc5d0 \uc801\uadf9\uc801\uc774\uc5c8\ub358 \ub370 \ube44\ud574, \uad70\ubd80\uc758 \uc911\uc2ec\uc778 \ucd5c\ub8e1\ud574\ub294 \uc6d0\uce59\uc801\uc774\uace0 \ubcf4\uc218\uc801\uc778 \uc785\uc7a5\uc774\ub77c\ub294 \uc810\uc744 \ub4e4\uc5c8\ub2e4.", "answer": "\uc815\uccad\ub798", "sentence": "\uc7a5\uc131\ud0dd\uc758 \uc219\uccad \uc0ac\uc2e4\uc744 \uad6d\uc815\uc6d0\uc73c\ub85c\ubd80\ud130 \ubcf4\uace0\ubc1b\uace0 \uc774\ub97c \ub300\uc678\uc801\uc73c\ub85c \uc54c\ub9b0 \uc815\uccad\ub798 \ubbfc\uc8fc\ub2f9 \uc758\uc6d0\uc740 \uc7a5\uc131\ud0dd\uc744 \uc81c\uac70\ud55c \uac83\uc774 \ucd5c\ub8e1\ud574 \uad70 \ucd1d\uc815\uce58\uad6d\uc7a5\uc73c\ub85c \ubcf4\uc778\ub2e4\uace0 \ubc1d\ud614\ub2e4.", "paragraph_sentence": "\uc548\ucc3d\uc5f0\uc774 \uc219\uccad\ub2f9\ud55c \uc774\uc720\ub294 \uc644\ubcbd\ud788 \uc54c \uc21c \uc5c6\uc73c\ub098, \ub300\ud55c\ubbfc\uad6d\uc758 \ub300\ubd81 \uc804\ubb38\uac00\ub4e4\uc740 \uc7a5\uc131\ud0dd\uc758 \uc219\uccad \ubc30\uacbd\uc5d0 \ub300\ud574 \ub2e4\uc591\ud55c \uc758\uacac\uc744 \ub0b4\ub193\uc558\ub2e4. 12\uc6d4 3\uc77c \uad6d\uc815\uc6d0\uc774 \uc7a5\uc131\ud0dd\uc758 \uc2e4\uac01 \uac00\ub2a5\uc131\uc744 \ubc1c\ud45c\ud55c \uc774\ud6c4 \ub0a8\ud55c\uc5d0\uc11c\ub294 \uc7a5\uc131\ud0dd\uacfc \uadf8\uc758 \ub77c\uc774\ubc8c\ub85c \uc54c\ub824\uc9c4 \ucd5c\ub8e1\ud574\uc758 \uad00\uacc4\uc5d0 \uc8fc\ubaa9\ud588\ub2e4. \uc7a5\uc131\ud0dd\uc758 \uc219\uccad \uc0ac\uc2e4\uc744 \uad6d\uc815\uc6d0\uc73c\ub85c\ubd80\ud130 \ubcf4\uace0\ubc1b\uace0 \uc774\ub97c \ub300\uc678\uc801\uc73c\ub85c \uc54c\ub9b0 \uc815\uccad\ub798 \ubbfc\uc8fc\ub2f9 \uc758\uc6d0\uc740 \uc7a5\uc131\ud0dd\uc744 \uc81c\uac70\ud55c \uac83\uc774 \ucd5c\ub8e1\ud574 \uad70 \ucd1d\uc815\uce58\uad6d\uc7a5\uc73c\ub85c \ubcf4\uc778\ub2e4\uace0 \ubc1d\ud614\ub2e4. \ucd5c\ub8e1\ud574\ub294 \ucd5c\ud604 \uc804 \uc778\ubbfc\ubb34\ub825\ubd80\uc7a5\uc758 \uc544\ub4e4\uc774\ub2e4. \uc870\uc120\uc77c\ubcf4\ub294 \ucd5c\ub8e1\ud574\ub85c \ub300\ud45c\ub418\ub294 \ud6c8\uc2e0(\u52f3\u81e3) \uc138\ub825\uacfc \uc7a5\uc131\ud0dd \ub4f1 \uae40\uc815\uc740\uc758 \ucc99\uc2e0(\u621a\u81e3)\uc758 \ub300\uacb0\uc5d0\uc11c \ud6c8\uc2e0 \uc138\ub825\uc774 \uc2b9\ub9ac\ud588\ub2e4\ub294 \ubd84\uc11d\uc744 \ub0b4\ub1a8\ub2e4. \ub300\ubd81 \uc804\ubb38\uac00 \uae40\uadfc\uc2dd \uacbd\ub0a8\ub300 \uad50\uc218\ub3c4 \"\ucd5c\ub8e1\ud574\ub97c \uc911\uc2ec\uc73c\ub85c \ud55c \uc2e0\uad70\ubd80\uc138\ub825\uc774 \ubc00\uc5b4\ub0b8 \uac83 \uc544\ub2cc\uac00 \ud574\uc11d\uc774 \uac00\ub2a5\ud558\ub2e4\"\uba70 \ubd81\uc870\uc120 \ub0b4 \ubcf4\uc218\uc138\ub825\uc758 \uc785\uc7a5\uc774 \uad00\ucca0\ub41c \uacb0\uacfc\ub77c\uace0 \ud574\uc11d\ud588\ub2e4. \uadf8 \uc774\uc720\ub85c \uae40\uadfc\uc2dd \uad50\uc218\ub294 \uc7a5\uc131\ud0dd\uc774 \uc911\uad6d, \ub0a8\ud55c \ub4f1\uacfc\uc758 \uacbd\uc81c\uad50\ub958\uc5d0 \uc801\uadf9\uc801\uc774\uc5c8\ub358 \ub370 \ube44\ud574, \uad70\ubd80\uc758 \uc911\uc2ec\uc778 \ucd5c\ub8e1\ud574\ub294 \uc6d0\uce59\uc801\uc774\uace0 \ubcf4\uc218\uc801\uc778 \uc785\uc7a5\uc774\ub77c\ub294 \uc810\uc744 \ub4e4\uc5c8\ub2e4.", "paragraph_answer": "\uc548\ucc3d\uc5f0\uc774 \uc219\uccad\ub2f9\ud55c \uc774\uc720\ub294 \uc644\ubcbd\ud788 \uc54c \uc21c \uc5c6\uc73c\ub098, \ub300\ud55c\ubbfc\uad6d\uc758 \ub300\ubd81 \uc804\ubb38\uac00\ub4e4\uc740 \uc7a5\uc131\ud0dd\uc758 \uc219\uccad \ubc30\uacbd\uc5d0 \ub300\ud574 \ub2e4\uc591\ud55c \uc758\uacac\uc744 \ub0b4\ub193\uc558\ub2e4. 12\uc6d4 3\uc77c \uad6d\uc815\uc6d0\uc774 \uc7a5\uc131\ud0dd\uc758 \uc2e4\uac01 \uac00\ub2a5\uc131\uc744 \ubc1c\ud45c\ud55c \uc774\ud6c4 \ub0a8\ud55c\uc5d0\uc11c\ub294 \uc7a5\uc131\ud0dd\uacfc \uadf8\uc758 \ub77c\uc774\ubc8c\ub85c \uc54c\ub824\uc9c4 \ucd5c\ub8e1\ud574\uc758 \uad00\uacc4\uc5d0 \uc8fc\ubaa9\ud588\ub2e4. \uc7a5\uc131\ud0dd\uc758 \uc219\uccad \uc0ac\uc2e4\uc744 \uad6d\uc815\uc6d0\uc73c\ub85c\ubd80\ud130 \ubcf4\uace0\ubc1b\uace0 \uc774\ub97c \ub300\uc678\uc801\uc73c\ub85c \uc54c\ub9b0 \uc815\uccad\ub798 \ubbfc\uc8fc\ub2f9 \uc758\uc6d0\uc740 \uc7a5\uc131\ud0dd\uc744 \uc81c\uac70\ud55c \uac83\uc774 \ucd5c\ub8e1\ud574 \uad70 \ucd1d\uc815\uce58\uad6d\uc7a5\uc73c\ub85c \ubcf4\uc778\ub2e4\uace0 \ubc1d\ud614\ub2e4. \ucd5c\ub8e1\ud574\ub294 \ucd5c\ud604 \uc804 \uc778\ubbfc\ubb34\ub825\ubd80\uc7a5\uc758 \uc544\ub4e4\uc774\ub2e4. \uc870\uc120\uc77c\ubcf4\ub294 \ucd5c\ub8e1\ud574\ub85c \ub300\ud45c\ub418\ub294 \ud6c8\uc2e0(\u52f3\u81e3) \uc138\ub825\uacfc \uc7a5\uc131\ud0dd \ub4f1 \uae40\uc815\uc740\uc758 \ucc99\uc2e0(\u621a\u81e3)\uc758 \ub300\uacb0\uc5d0\uc11c \ud6c8\uc2e0 \uc138\ub825\uc774 \uc2b9\ub9ac\ud588\ub2e4\ub294 \ubd84\uc11d\uc744 \ub0b4\ub1a8\ub2e4. \ub300\ubd81 \uc804\ubb38\uac00 \uae40\uadfc\uc2dd \uacbd\ub0a8\ub300 \uad50\uc218\ub3c4 \"\ucd5c\ub8e1\ud574\ub97c \uc911\uc2ec\uc73c\ub85c \ud55c \uc2e0\uad70\ubd80\uc138\ub825\uc774 \ubc00\uc5b4\ub0b8 \uac83 \uc544\ub2cc\uac00 \ud574\uc11d\uc774 \uac00\ub2a5\ud558\ub2e4\"\uba70 \ubd81\uc870\uc120 \ub0b4 \ubcf4\uc218\uc138\ub825\uc758 \uc785\uc7a5\uc774 \uad00\ucca0\ub41c \uacb0\uacfc\ub77c\uace0 \ud574\uc11d\ud588\ub2e4. \uadf8 \uc774\uc720\ub85c \uae40\uadfc\uc2dd \uad50\uc218\ub294 \uc7a5\uc131\ud0dd\uc774 \uc911\uad6d, \ub0a8\ud55c \ub4f1\uacfc\uc758 \uacbd\uc81c\uad50\ub958\uc5d0 \uc801\uadf9\uc801\uc774\uc5c8\ub358 \ub370 \ube44\ud574, \uad70\ubd80\uc758 \uc911\uc2ec\uc778 \ucd5c\ub8e1\ud574\ub294 \uc6d0\uce59\uc801\uc774\uace0 \ubcf4\uc218\uc801\uc778 \uc785\uc7a5\uc774\ub77c\ub294 \uc810\uc744 \ub4e4\uc5c8\ub2e4.", "sentence_answer": "\uc7a5\uc131\ud0dd\uc758 \uc219\uccad \uc0ac\uc2e4\uc744 \uad6d\uc815\uc6d0\uc73c\ub85c\ubd80\ud130 \ubcf4\uace0\ubc1b\uace0 \uc774\ub97c \ub300\uc678\uc801\uc73c\ub85c \uc54c\ub9b0 \uc815\uccad\ub798 \ubbfc\uc8fc\ub2f9 \uc758\uc6d0\uc740 \uc7a5\uc131\ud0dd\uc744 \uc81c\uac70\ud55c \uac83\uc774 \ucd5c\ub8e1\ud574 \uad70 \ucd1d\uc815\uce58\uad6d\uc7a5\uc73c\ub85c \ubcf4\uc778\ub2e4\uace0 \ubc1d\ud614\ub2e4.", "paragraph_id": "6511779-4-0", "paragraph_question": "question: \uc815\uc131\ud0dd\uc758 \uc219\uccad \uc0ac\uc2e4\uc744 \uc678\ubd80\uc5d0 \uc54c\ub9b0 \uc778\ubb3c\uc740?, context: \uc548\ucc3d\uc5f0\uc774 \uc219\uccad\ub2f9\ud55c \uc774\uc720\ub294 \uc644\ubcbd\ud788 \uc54c \uc21c \uc5c6\uc73c\ub098, \ub300\ud55c\ubbfc\uad6d\uc758 \ub300\ubd81 \uc804\ubb38\uac00\ub4e4\uc740 \uc7a5\uc131\ud0dd\uc758 \uc219\uccad \ubc30\uacbd\uc5d0 \ub300\ud574 \ub2e4\uc591\ud55c \uc758\uacac\uc744 \ub0b4\ub193\uc558\ub2e4. 12\uc6d4 3\uc77c \uad6d\uc815\uc6d0\uc774 \uc7a5\uc131\ud0dd\uc758 \uc2e4\uac01 \uac00\ub2a5\uc131\uc744 \ubc1c\ud45c\ud55c \uc774\ud6c4 \ub0a8\ud55c\uc5d0\uc11c\ub294 \uc7a5\uc131\ud0dd\uacfc \uadf8\uc758 \ub77c\uc774\ubc8c\ub85c \uc54c\ub824\uc9c4 \ucd5c\ub8e1\ud574\uc758 \uad00\uacc4\uc5d0 \uc8fc\ubaa9\ud588\ub2e4. \uc7a5\uc131\ud0dd\uc758 \uc219\uccad \uc0ac\uc2e4\uc744 \uad6d\uc815\uc6d0\uc73c\ub85c\ubd80\ud130 \ubcf4\uace0\ubc1b\uace0 \uc774\ub97c \ub300\uc678\uc801\uc73c\ub85c \uc54c\ub9b0 \uc815\uccad\ub798 \ubbfc\uc8fc\ub2f9 \uc758\uc6d0\uc740 \uc7a5\uc131\ud0dd\uc744 \uc81c\uac70\ud55c \uac83\uc774 \ucd5c\ub8e1\ud574 \uad70 \ucd1d\uc815\uce58\uad6d\uc7a5\uc73c\ub85c \ubcf4\uc778\ub2e4\uace0 \ubc1d\ud614\ub2e4. \ucd5c\ub8e1\ud574\ub294 \ucd5c\ud604 \uc804 \uc778\ubbfc\ubb34\ub825\ubd80\uc7a5\uc758 \uc544\ub4e4\uc774\ub2e4. \uc870\uc120\uc77c\ubcf4\ub294 \ucd5c\ub8e1\ud574\ub85c \ub300\ud45c\ub418\ub294 \ud6c8\uc2e0(\u52f3\u81e3) \uc138\ub825\uacfc \uc7a5\uc131\ud0dd \ub4f1 \uae40\uc815\uc740\uc758 \ucc99\uc2e0(\u621a\u81e3)\uc758 \ub300\uacb0\uc5d0\uc11c \ud6c8\uc2e0 \uc138\ub825\uc774 \uc2b9\ub9ac\ud588\ub2e4\ub294 \ubd84\uc11d\uc744 \ub0b4\ub1a8\ub2e4. \ub300\ubd81 \uc804\ubb38\uac00 \uae40\uadfc\uc2dd \uacbd\ub0a8\ub300 \uad50\uc218\ub3c4 \"\ucd5c\ub8e1\ud574\ub97c \uc911\uc2ec\uc73c\ub85c \ud55c \uc2e0\uad70\ubd80\uc138\ub825\uc774 \ubc00\uc5b4\ub0b8 \uac83 \uc544\ub2cc\uac00 \ud574\uc11d\uc774 \uac00\ub2a5\ud558\ub2e4\"\uba70 \ubd81\uc870\uc120 \ub0b4 \ubcf4\uc218\uc138\ub825\uc758 \uc785\uc7a5\uc774 \uad00\ucca0\ub41c \uacb0\uacfc\ub77c\uace0 \ud574\uc11d\ud588\ub2e4. \uadf8 \uc774\uc720\ub85c \uae40\uadfc\uc2dd \uad50\uc218\ub294 \uc7a5\uc131\ud0dd\uc774 \uc911\uad6d, \ub0a8\ud55c \ub4f1\uacfc\uc758 \uacbd\uc81c\uad50\ub958\uc5d0 \uc801\uadf9\uc801\uc774\uc5c8\ub358 \ub370 \ube44\ud574, \uad70\ubd80\uc758 \uc911\uc2ec\uc778 \ucd5c\ub8e1\ud574\ub294 \uc6d0\uce59\uc801\uc774\uace0 \ubcf4\uc218\uc801\uc778 \uc785\uc7a5\uc774\ub77c\ub294 \uc810\uc744 \ub4e4\uc5c8\ub2e4."} {"question": "\uc2f8\uc774\uc758 \ub370\ubdd4\uace1 \uc81c\ubaa9\uc740?", "paragraph": "2001\ub144 1\uc6d4, \uc2f8\uc774\ub294 \uadf8\uc758 \uccab \ubc88\uc9f8 \uc815\uaddc \uc568\ubc94 \u300aPSY... From the Psycho World\u300b\uc744 \ubc1c\ub9e4\ud558\uba70 \ub370\ubdd4\ud558\uc600\ub2e4, \ud558\uc9c0\ub9cc \uc774 \uc74c\ubc18\uc758 \uac00\uc0ac \uc18d\uc5d0 \ubd80\uc801\uc808\ud55c \ub0b4\uc6a9\uc774 \ud3ec\ud568\ub418\uc5b4 \uc788\ub2e4\ub294 \uc774\uc720\ub85c \ubc8c\uae08\uc744 \ub0c8\ub2e4. \uc2f8\uc774\ub294 \ud1b5\ud1b5\ud55c \ubab8\ub9e4\uc5d0 \ubc88\uca4d\uc774\ub294 \uc7ac\ud0b7\uc744 \uc785\uace0 \ubb34\ub300\uc5d0 \uc62c\ub790\uace0 \ub2e4\uc18c \uc704\ud611\uc801\uc778 \ub7a9\uacfc \uc5fd\uae30\uc801\uc778 \ucda4\uc73c\ub85c \u201c\uc5fd\uae30 \uac00\uc218\u201d\ub85c \ubd88\ub838\ub2e4. \uadf8\uc758 \ub370\ubdd4\uace1 \u3008\uc0c8\u3009\ub294 \ubc1c\ub9e4\uc640 \ub3d9\uc2dc\uc5d0 \ud070 \uc778\uae30\ub97c \uc5bb\uc73c\uba70 \uc77c\uc57d \uc2a4\ud0c0\ub364\uc5d0 \uc62c\ub790\ub2e4. \uadf8 \ud574 11\uc6d4 15\uc77c, \uc2f8\uc774\ub294 \ub300\ub9c8\ucd08\ub97c \ud53c\uc6b4 \ud610\uc758\ub85c \uac80\uac70\ub418\uc5c8\ub2e4. \uc774\ub85c \uc778\ud574 \ub2f9\uc2dc \uc5f0\ub9d0 \uac00\uc694 \uc2dc\uc0c1\uc2dd\uc5d0\uc11c \uc720\ub825\ud55c \uc2e0\uc778\uc0c1 \ud6c4\ubcf4\uc600\ub358 \uadf8\ub294 \uc218\uc0c1\uc758 \uae30\uc068\uc744 \ub9db\ubcf4\uc9c0 \ubabb\ud558\uace0 \uc138\ud654\uace0\ub4f1\ud559\uad50 2\ub144 \ud6c4\ubc30\uc778 \uc131\uc2dc\uacbd\uc774 \uc2e0\uc778\uc0c1\uc744 \ubc1b\ub294 \ubaa8\uc2b5\uc744 \uc9c0\ucf1c\ubcf4\uc544\uc57c \ud588\ub2e4. \uadf8\uc758 \ub450 \ubc88\uc9f8 \uc815\uaddc \uc568\ubc94 \u300a\uc2f82\u300b\ub294 2002\ub144 1\uc6d4 \ubc1c\ub9e4 \ub418\uc5c8\uace0, \uc774 \uc74c\ubc18 \uc5ed\uc2dc \uc120\uc815\uc801\uc778 \uac00\uc0ac\ub85c \uc778\ud574 \uc5b4\ub9b0\uc774\uc640 \uccad\uc18c\ub144\uc5d0 \ubbf8\uce60 \uc601\ud5a5\uc5d0 \ub300\ud55c \uc2dc\ubbfc \ub2e8\uccb4\uc758 \ud56d\uc758\ub85c 19\uc138 \ubbf8\ub9cc \uccad\ucde8 \uae08\uc9c0 \ud310\uc815\uc744 \ubc1b\uc544 \ud310\ub9e4\uac00 \uae08\uc9c0\ub418\uba74\uc11c \ub2e4\uc2dc \ud55c\ubc88 \ub17c\ub780\uc774 \uc77c\uae30\ub3c4 \ud588\ub2e4. \uac19\uc740 \ud574 9\uc6d4, \uc2f8\uc774\ub294 \uc138 \ubc88\uc9f8 \uc815\uaddc \uc568\ubc94 \u300a3\ub9c8\uc774\u300b\ub97c \ubc1c\ud45c\ud588\ub2e4. \uc774 \uc74c\ubc18\uc758 \ud0c0\uc774\ud2c0 \uace1 \u3008\ucc54\ud53c\uc5b8\u3009\uc740 \ud55c\uc77c \uc6d4\ub4dc\ucef5\uc758 \uad11\uace0 \ud2b9\uc218\ub97c \ub204\ub824 \ud070 \uc131\uacf5\uc744 \uac70\ub480\ub2e4. \uadf8\uc758 \uc74c\uc545\uc744 \ub458\ub7ec\uc2fc \uc0c1\ub2f9\ud55c \ub17c\uc7c1\uc5d0\ub3c4 \ubd88\uad6c\ud558\uace0, \uc2f8\uc774\ub294 \ud55c\uad6d \uc74c\uc545 \uc0b0\uc5c5\uc5d0\uc11c \uc790\uc2e0\ub9cc\uc758 \ub3c5\ud2b9\ud558\uace0 \ub69d\uc2ec\uc788\ub294 \uc74c\uc545\uc73c\ub85c \uc11c\uc6b8\uac00\uc694\ub300\uc0c1\uc5d0\uc11c \uc791\uace1\uac00\uc0c1\uc744 \uc218\uc0c1\ud588\ub2e4.", "answer": "\uc0c8", "sentence": "\uadf8\uc758 \ub370\ubdd4\uace1 \u3008 \uc0c8 \u3009\ub294 \ubc1c\ub9e4\uc640 \ub3d9\uc2dc\uc5d0 \ud070 \uc778\uae30\ub97c \uc5bb\uc73c\uba70 \uc77c\uc57d \uc2a4\ud0c0\ub364\uc5d0 \uc62c\ub790\ub2e4.", "paragraph_sentence": "2001\ub144 1\uc6d4, \uc2f8\uc774\ub294 \uadf8\uc758 \uccab \ubc88\uc9f8 \uc815\uaddc \uc568\ubc94 \u300aPSY... From the Psycho World\u300b\uc744 \ubc1c\ub9e4\ud558\uba70 \ub370\ubdd4\ud558\uc600\ub2e4, \ud558\uc9c0\ub9cc \uc774 \uc74c\ubc18\uc758 \uac00\uc0ac \uc18d\uc5d0 \ubd80\uc801\uc808\ud55c \ub0b4\uc6a9\uc774 \ud3ec\ud568\ub418\uc5b4 \uc788\ub2e4\ub294 \uc774\uc720\ub85c \ubc8c\uae08\uc744 \ub0c8\ub2e4. \uc2f8\uc774\ub294 \ud1b5\ud1b5\ud55c \ubab8\ub9e4\uc5d0 \ubc88\uca4d\uc774\ub294 \uc7ac\ud0b7\uc744 \uc785\uace0 \ubb34\ub300\uc5d0 \uc62c\ub790\uace0 \ub2e4\uc18c \uc704\ud611\uc801\uc778 \ub7a9\uacfc \uc5fd\uae30\uc801\uc778 \ucda4\uc73c\ub85c \u201c\uc5fd\uae30 \uac00\uc218\u201d\ub85c \ubd88\ub838\ub2e4. \uadf8\uc758 \ub370\ubdd4\uace1 \u3008 \uc0c8 \u3009\ub294 \ubc1c\ub9e4\uc640 \ub3d9\uc2dc\uc5d0 \ud070 \uc778\uae30\ub97c \uc5bb\uc73c\uba70 \uc77c\uc57d \uc2a4\ud0c0\ub364\uc5d0 \uc62c\ub790\ub2e4. \uadf8 \ud574 11\uc6d4 15\uc77c, \uc2f8\uc774\ub294 \ub300\ub9c8\ucd08\ub97c \ud53c\uc6b4 \ud610\uc758\ub85c \uac80\uac70\ub418\uc5c8\ub2e4. \uc774\ub85c \uc778\ud574 \ub2f9\uc2dc \uc5f0\ub9d0 \uac00\uc694 \uc2dc\uc0c1\uc2dd\uc5d0\uc11c \uc720\ub825\ud55c \uc2e0\uc778\uc0c1 \ud6c4\ubcf4\uc600\ub358 \uadf8\ub294 \uc218\uc0c1\uc758 \uae30\uc068\uc744 \ub9db\ubcf4\uc9c0 \ubabb\ud558\uace0 \uc138\ud654\uace0\ub4f1\ud559\uad50 2\ub144 \ud6c4\ubc30\uc778 \uc131\uc2dc\uacbd\uc774 \uc2e0\uc778\uc0c1\uc744 \ubc1b\ub294 \ubaa8\uc2b5\uc744 \uc9c0\ucf1c\ubcf4\uc544\uc57c \ud588\ub2e4. \uadf8\uc758 \ub450 \ubc88\uc9f8 \uc815\uaddc \uc568\ubc94 \u300a\uc2f82\u300b\ub294 2002\ub144 1\uc6d4 \ubc1c\ub9e4 \ub418\uc5c8\uace0, \uc774 \uc74c\ubc18 \uc5ed\uc2dc \uc120\uc815\uc801\uc778 \uac00\uc0ac\ub85c \uc778\ud574 \uc5b4\ub9b0\uc774\uc640 \uccad\uc18c\ub144\uc5d0 \ubbf8\uce60 \uc601\ud5a5\uc5d0 \ub300\ud55c \uc2dc\ubbfc \ub2e8\uccb4\uc758 \ud56d\uc758\ub85c 19\uc138 \ubbf8\ub9cc \uccad\ucde8 \uae08\uc9c0 \ud310\uc815\uc744 \ubc1b\uc544 \ud310\ub9e4\uac00 \uae08\uc9c0\ub418\uba74\uc11c \ub2e4\uc2dc \ud55c\ubc88 \ub17c\ub780\uc774 \uc77c\uae30\ub3c4 \ud588\ub2e4. \uac19\uc740 \ud574 9\uc6d4, \uc2f8\uc774\ub294 \uc138 \ubc88\uc9f8 \uc815\uaddc \uc568\ubc94 \u300a3\ub9c8\uc774\u300b\ub97c \ubc1c\ud45c\ud588\ub2e4. \uc774 \uc74c\ubc18\uc758 \ud0c0\uc774\ud2c0 \uace1 \u3008\ucc54\ud53c\uc5b8\u3009\uc740 \ud55c\uc77c \uc6d4\ub4dc\ucef5\uc758 \uad11\uace0 \ud2b9\uc218\ub97c \ub204\ub824 \ud070 \uc131\uacf5\uc744 \uac70\ub480\ub2e4. \uadf8\uc758 \uc74c\uc545\uc744 \ub458\ub7ec\uc2fc \uc0c1\ub2f9\ud55c \ub17c\uc7c1\uc5d0\ub3c4 \ubd88\uad6c\ud558\uace0, \uc2f8\uc774\ub294 \ud55c\uad6d \uc74c\uc545 \uc0b0\uc5c5\uc5d0\uc11c \uc790\uc2e0\ub9cc\uc758 \ub3c5\ud2b9\ud558\uace0 \ub69d\uc2ec\uc788\ub294 \uc74c\uc545\uc73c\ub85c \uc11c\uc6b8\uac00\uc694\ub300\uc0c1\uc5d0\uc11c \uc791\uace1\uac00\uc0c1\uc744 \uc218\uc0c1\ud588\ub2e4.", "paragraph_answer": "2001\ub144 1\uc6d4, \uc2f8\uc774\ub294 \uadf8\uc758 \uccab \ubc88\uc9f8 \uc815\uaddc \uc568\ubc94 \u300aPSY... From the Psycho World\u300b\uc744 \ubc1c\ub9e4\ud558\uba70 \ub370\ubdd4\ud558\uc600\ub2e4, \ud558\uc9c0\ub9cc \uc774 \uc74c\ubc18\uc758 \uac00\uc0ac \uc18d\uc5d0 \ubd80\uc801\uc808\ud55c \ub0b4\uc6a9\uc774 \ud3ec\ud568\ub418\uc5b4 \uc788\ub2e4\ub294 \uc774\uc720\ub85c \ubc8c\uae08\uc744 \ub0c8\ub2e4. \uc2f8\uc774\ub294 \ud1b5\ud1b5\ud55c \ubab8\ub9e4\uc5d0 \ubc88\uca4d\uc774\ub294 \uc7ac\ud0b7\uc744 \uc785\uace0 \ubb34\ub300\uc5d0 \uc62c\ub790\uace0 \ub2e4\uc18c \uc704\ud611\uc801\uc778 \ub7a9\uacfc \uc5fd\uae30\uc801\uc778 \ucda4\uc73c\ub85c \u201c\uc5fd\uae30 \uac00\uc218\u201d\ub85c \ubd88\ub838\ub2e4. \uadf8\uc758 \ub370\ubdd4\uace1 \u3008 \uc0c8 \u3009\ub294 \ubc1c\ub9e4\uc640 \ub3d9\uc2dc\uc5d0 \ud070 \uc778\uae30\ub97c \uc5bb\uc73c\uba70 \uc77c\uc57d \uc2a4\ud0c0\ub364\uc5d0 \uc62c\ub790\ub2e4. \uadf8 \ud574 11\uc6d4 15\uc77c, \uc2f8\uc774\ub294 \ub300\ub9c8\ucd08\ub97c \ud53c\uc6b4 \ud610\uc758\ub85c \uac80\uac70\ub418\uc5c8\ub2e4. \uc774\ub85c \uc778\ud574 \ub2f9\uc2dc \uc5f0\ub9d0 \uac00\uc694 \uc2dc\uc0c1\uc2dd\uc5d0\uc11c \uc720\ub825\ud55c \uc2e0\uc778\uc0c1 \ud6c4\ubcf4\uc600\ub358 \uadf8\ub294 \uc218\uc0c1\uc758 \uae30\uc068\uc744 \ub9db\ubcf4\uc9c0 \ubabb\ud558\uace0 \uc138\ud654\uace0\ub4f1\ud559\uad50 2\ub144 \ud6c4\ubc30\uc778 \uc131\uc2dc\uacbd\uc774 \uc2e0\uc778\uc0c1\uc744 \ubc1b\ub294 \ubaa8\uc2b5\uc744 \uc9c0\ucf1c\ubcf4\uc544\uc57c \ud588\ub2e4. \uadf8\uc758 \ub450 \ubc88\uc9f8 \uc815\uaddc \uc568\ubc94 \u300a\uc2f82\u300b\ub294 2002\ub144 1\uc6d4 \ubc1c\ub9e4 \ub418\uc5c8\uace0, \uc774 \uc74c\ubc18 \uc5ed\uc2dc \uc120\uc815\uc801\uc778 \uac00\uc0ac\ub85c \uc778\ud574 \uc5b4\ub9b0\uc774\uc640 \uccad\uc18c\ub144\uc5d0 \ubbf8\uce60 \uc601\ud5a5\uc5d0 \ub300\ud55c \uc2dc\ubbfc \ub2e8\uccb4\uc758 \ud56d\uc758\ub85c 19\uc138 \ubbf8\ub9cc \uccad\ucde8 \uae08\uc9c0 \ud310\uc815\uc744 \ubc1b\uc544 \ud310\ub9e4\uac00 \uae08\uc9c0\ub418\uba74\uc11c \ub2e4\uc2dc \ud55c\ubc88 \ub17c\ub780\uc774 \uc77c\uae30\ub3c4 \ud588\ub2e4. \uac19\uc740 \ud574 9\uc6d4, \uc2f8\uc774\ub294 \uc138 \ubc88\uc9f8 \uc815\uaddc \uc568\ubc94 \u300a3\ub9c8\uc774\u300b\ub97c \ubc1c\ud45c\ud588\ub2e4. \uc774 \uc74c\ubc18\uc758 \ud0c0\uc774\ud2c0 \uace1 \u3008\ucc54\ud53c\uc5b8\u3009\uc740 \ud55c\uc77c \uc6d4\ub4dc\ucef5\uc758 \uad11\uace0 \ud2b9\uc218\ub97c \ub204\ub824 \ud070 \uc131\uacf5\uc744 \uac70\ub480\ub2e4. \uadf8\uc758 \uc74c\uc545\uc744 \ub458\ub7ec\uc2fc \uc0c1\ub2f9\ud55c \ub17c\uc7c1\uc5d0\ub3c4 \ubd88\uad6c\ud558\uace0, \uc2f8\uc774\ub294 \ud55c\uad6d \uc74c\uc545 \uc0b0\uc5c5\uc5d0\uc11c \uc790\uc2e0\ub9cc\uc758 \ub3c5\ud2b9\ud558\uace0 \ub69d\uc2ec\uc788\ub294 \uc74c\uc545\uc73c\ub85c \uc11c\uc6b8\uac00\uc694\ub300\uc0c1\uc5d0\uc11c \uc791\uace1\uac00\uc0c1\uc744 \uc218\uc0c1\ud588\ub2e4.", "sentence_answer": "\uadf8\uc758 \ub370\ubdd4\uace1 \u3008 \uc0c8 \u3009\ub294 \ubc1c\ub9e4\uc640 \ub3d9\uc2dc\uc5d0 \ud070 \uc778\uae30\ub97c \uc5bb\uc73c\uba70 \uc77c\uc57d \uc2a4\ud0c0\ub364\uc5d0 \uc62c\ub790\ub2e4.", "paragraph_id": "6457532-1-0", "paragraph_question": "question: \uc2f8\uc774\uc758 \ub370\ubdd4\uace1 \uc81c\ubaa9\uc740?, context: 2001\ub144 1\uc6d4, \uc2f8\uc774\ub294 \uadf8\uc758 \uccab \ubc88\uc9f8 \uc815\uaddc \uc568\ubc94 \u300aPSY... From the Psycho World\u300b\uc744 \ubc1c\ub9e4\ud558\uba70 \ub370\ubdd4\ud558\uc600\ub2e4, \ud558\uc9c0\ub9cc \uc774 \uc74c\ubc18\uc758 \uac00\uc0ac \uc18d\uc5d0 \ubd80\uc801\uc808\ud55c \ub0b4\uc6a9\uc774 \ud3ec\ud568\ub418\uc5b4 \uc788\ub2e4\ub294 \uc774\uc720\ub85c \ubc8c\uae08\uc744 \ub0c8\ub2e4. \uc2f8\uc774\ub294 \ud1b5\ud1b5\ud55c \ubab8\ub9e4\uc5d0 \ubc88\uca4d\uc774\ub294 \uc7ac\ud0b7\uc744 \uc785\uace0 \ubb34\ub300\uc5d0 \uc62c\ub790\uace0 \ub2e4\uc18c \uc704\ud611\uc801\uc778 \ub7a9\uacfc \uc5fd\uae30\uc801\uc778 \ucda4\uc73c\ub85c \u201c\uc5fd\uae30 \uac00\uc218\u201d\ub85c \ubd88\ub838\ub2e4. \uadf8\uc758 \ub370\ubdd4\uace1 \u3008\uc0c8\u3009\ub294 \ubc1c\ub9e4\uc640 \ub3d9\uc2dc\uc5d0 \ud070 \uc778\uae30\ub97c \uc5bb\uc73c\uba70 \uc77c\uc57d \uc2a4\ud0c0\ub364\uc5d0 \uc62c\ub790\ub2e4. \uadf8 \ud574 11\uc6d4 15\uc77c, \uc2f8\uc774\ub294 \ub300\ub9c8\ucd08\ub97c \ud53c\uc6b4 \ud610\uc758\ub85c \uac80\uac70\ub418\uc5c8\ub2e4. \uc774\ub85c \uc778\ud574 \ub2f9\uc2dc \uc5f0\ub9d0 \uac00\uc694 \uc2dc\uc0c1\uc2dd\uc5d0\uc11c \uc720\ub825\ud55c \uc2e0\uc778\uc0c1 \ud6c4\ubcf4\uc600\ub358 \uadf8\ub294 \uc218\uc0c1\uc758 \uae30\uc068\uc744 \ub9db\ubcf4\uc9c0 \ubabb\ud558\uace0 \uc138\ud654\uace0\ub4f1\ud559\uad50 2\ub144 \ud6c4\ubc30\uc778 \uc131\uc2dc\uacbd\uc774 \uc2e0\uc778\uc0c1\uc744 \ubc1b\ub294 \ubaa8\uc2b5\uc744 \uc9c0\ucf1c\ubcf4\uc544\uc57c \ud588\ub2e4. \uadf8\uc758 \ub450 \ubc88\uc9f8 \uc815\uaddc \uc568\ubc94 \u300a\uc2f82\u300b\ub294 2002\ub144 1\uc6d4 \ubc1c\ub9e4 \ub418\uc5c8\uace0, \uc774 \uc74c\ubc18 \uc5ed\uc2dc \uc120\uc815\uc801\uc778 \uac00\uc0ac\ub85c \uc778\ud574 \uc5b4\ub9b0\uc774\uc640 \uccad\uc18c\ub144\uc5d0 \ubbf8\uce60 \uc601\ud5a5\uc5d0 \ub300\ud55c \uc2dc\ubbfc \ub2e8\uccb4\uc758 \ud56d\uc758\ub85c 19\uc138 \ubbf8\ub9cc \uccad\ucde8 \uae08\uc9c0 \ud310\uc815\uc744 \ubc1b\uc544 \ud310\ub9e4\uac00 \uae08\uc9c0\ub418\uba74\uc11c \ub2e4\uc2dc \ud55c\ubc88 \ub17c\ub780\uc774 \uc77c\uae30\ub3c4 \ud588\ub2e4. \uac19\uc740 \ud574 9\uc6d4, \uc2f8\uc774\ub294 \uc138 \ubc88\uc9f8 \uc815\uaddc \uc568\ubc94 \u300a3\ub9c8\uc774\u300b\ub97c \ubc1c\ud45c\ud588\ub2e4. \uc774 \uc74c\ubc18\uc758 \ud0c0\uc774\ud2c0 \uace1 \u3008\ucc54\ud53c\uc5b8\u3009\uc740 \ud55c\uc77c \uc6d4\ub4dc\ucef5\uc758 \uad11\uace0 \ud2b9\uc218\ub97c \ub204\ub824 \ud070 \uc131\uacf5\uc744 \uac70\ub480\ub2e4. \uadf8\uc758 \uc74c\uc545\uc744 \ub458\ub7ec\uc2fc \uc0c1\ub2f9\ud55c \ub17c\uc7c1\uc5d0\ub3c4 \ubd88\uad6c\ud558\uace0, \uc2f8\uc774\ub294 \ud55c\uad6d \uc74c\uc545 \uc0b0\uc5c5\uc5d0\uc11c \uc790\uc2e0\ub9cc\uc758 \ub3c5\ud2b9\ud558\uace0 \ub69d\uc2ec\uc788\ub294 \uc74c\uc545\uc73c\ub85c \uc11c\uc6b8\uac00\uc694\ub300\uc0c1\uc5d0\uc11c \uc791\uace1\uac00\uc0c1\uc744 \uc218\uc0c1\ud588\ub2e4."} {"question": "<<\ub098\uc05c \ub0a8\uc790>>\uc758 \uad00\uac1d \ub3d9\uc6d0 \uc218\ub294 \uba87 \uba85\uc778\uac00?", "paragraph": "\uac19\uc740 \ud574\uc778 2001\ub144 \uc11c\uc6d0\uc740 \uae40\uae30\ub355 \uac10\ub3c5\uc758 \uc601\ud654 \u300a\ub098\uc05c \ub0a8\uc790\u300b\uc5d0 \uc8fc\uc5f0\uc73c\ub85c \uce90\uc2a4\ud305\ub418\uc5c8\ub2e4. \uadf8\ub140\uac00 \ub9e1\uc740 \uc5ed\ud560\uc740 \uc0ac\ucc3d\uac00 \uae61\ud328\uc778 \ud55c\uae30(\uc870\uc7ac\ud604 \ubd84)\uc758 \uacc4\ub7b5\uc5d0 \ub9d0\ub824 \uc5ec\ub300\uc0dd\uc5d0\uc11c \ucc3d\ub140\ub85c \ucd94\ub77d\ud558\ub294 \uc120\ud654 \uc5ed\uc774\uc5c8\ub2e4. \uae40\uae30\ub355\uc740 \u201c\u300a\ub098\uc05c \ub0a8\uc790\u300b\uc758 \uc120\ud654 \uc5ed\uc740 \uc11c\uc6d0\uc5d0\uac8c\ub9cc \uc81c\uc548\ud588\ub2e4. \uc120\ud654 \uc5ed\uc744 \ud574\ub0bc \ubc30\uc6b0\ub294 \uac70\uc758 \uc5c6\ub2e4. \uc2dc\ub098\ub9ac\uc624\ub97c \ubcf4\uba74 \uacf3\uacf3\uc5d0 \uce58\uac00 \ub5a8\ub9ac\ub294 \ubb18\uc0ac \ud22c\uc131\uc774\ub2e4\u201d\uace0 \ub9d0\ud558\uc600\ub2e4. \uc11c\uc6d0\uc758 \uc8fc\ubcc0 \uc0ac\ub78c\ub4e4\uc740 \uadf8\ub140\uac00 \uc774 \uc601\ud654\uc5d0 \ucd9c\uc5f0\ud558\ub294 \uac83\uc744 \ub9d0\ub838\uc73c\ub098, \uae40\uae30\ub355 \uac10\ub3c5\uc758 \uc791\ud488\uc5d0 \uad00\uc2ec\uc774 \ub9ce\uc558\ub358 \uc11c\uc6d0\uc740 \uc120\ud654 \uc5ed\uc744 \uc218\ub77d\ud558\uc600\ub2e4. \uae40\uae30\ub355\uc740 \u201c\uc11c\uc6d0\uc740 \uc120\ud654\ub97c \ub108\ubb34 \uc798 \ud574\ub0c8\ub2e4. \uadf8 \uc5b4\ub5a4 \ubc30\uc6b0\ubcf4\ub2e4 \uc2e4\uac10\ub098\uac8c \ud588\uace0, \ucc0d\uc73c\uba74\uc11c\ub3c4 \uc9c4\uc9dc \ucc3d\ub140\uac19\uc558\ub2e4\u201d\uace0 \uadf8\ub140\uc758 \uc5f0\uae30\ub97c \uce6d\ucc2c\ud588\uc73c\ub098, \uc11c\uc6d0\uc740 \u201c\uc601\ud63c\uc744 \ub2e4\ucce4\ub2e4\u201d\ub294 \ub9d0\ub85c \uc774 \uc5ed\uc774 \ubb34\ucc99 \uace0\ud1b5\uc2a4\ub7ec\uc6e0\uc74c\uc744 \ud45c\ud604\ud588\ub2e4. 2002\ub144 1\uc6d4 \uac1c\ubd09\ud55c \u300a\ub098\uc05c \ub0a8\uc790\u300b\ub294 \uc804\uad6d\uad00\uac1d 70\ub9cc \uba85(\ube44\uacf5\uc2dd)\uc744 \uae30\ub85d\ud558\uba70 \uc608\uc0c1\uc678\ub85c \ud765\ud589\ud558\uc600\uace0, \uc11c\uc6d0\uc740 \uc774 \uc791\ud488\uc73c\ub85c 2002\ub144 \ub300\uc885\uc0c1 \uc2e0\uc778\uc5ec\uc6b0\uc0c1, \ucd98\uc0ac\ub098\uc6b4\uaddc\uc601\ud654\uc608\uc220\uc81c \uc62c\ud574\uc758 \uc2e0\uc778\uc5f0\uae30\uc0c1 \ubc0f \ud669\uae08\ucd2c\uc601\uc0c1 \uc2e0\uc778\uc5ec\uc6b0\uc0c1\uc744 \uc218\uc0c1\ud588\ub2e4. \ud55c\ud3b8, \u300a\ub098\uc05c \ub0a8\uc790\u300b\ub294 \u2018\uc5ec\uc131\uc5d0 \ub300\ud55c \ud3ed\ub825\uc131\u2019\uc73c\ub85c \uc5ec\uc131\uc6b4\ub3d9\uacc4\ub85c\ubd80\ud130 \ube44\ud310\uc744 \ubc1b\uc558\ub294\ub370, \uc870\uc7ac\ud604\uacfc \uc11c\uc6d0\uc740 \uac19\uc740 \ud574 \uc5ec\uc131\uad00\uac1d\uc601\ud654\uc0c1\uc5d0\uc11c \ucd5c\uc545\uc758 \ub0a8\ub140 \ubc30\uc6b0\ub85c \uac01\uac01 \uc120\uc815\ub418\uae30\ub3c4 \ud588\ub2e4.", "answer": "70\ub9cc \uba85", "sentence": "2002\ub144 1\uc6d4 \uac1c\ubd09\ud55c \u300a\ub098\uc05c \ub0a8\uc790\u300b\ub294 \uc804\uad6d\uad00\uac1d 70\ub9cc \uba85 (\ube44\uacf5\uc2dd)", "paragraph_sentence": "\uac19\uc740 \ud574\uc778 2001\ub144 \uc11c\uc6d0\uc740 \uae40\uae30\ub355 \uac10\ub3c5\uc758 \uc601\ud654 \u300a\ub098\uc05c \ub0a8\uc790\u300b\uc5d0 \uc8fc\uc5f0\uc73c\ub85c \uce90\uc2a4\ud305\ub418\uc5c8\ub2e4. \uadf8\ub140\uac00 \ub9e1\uc740 \uc5ed\ud560\uc740 \uc0ac\ucc3d\uac00 \uae61\ud328\uc778 \ud55c\uae30(\uc870\uc7ac\ud604 \ubd84)\uc758 \uacc4\ub7b5\uc5d0 \ub9d0\ub824 \uc5ec\ub300\uc0dd\uc5d0\uc11c \ucc3d\ub140\ub85c \ucd94\ub77d\ud558\ub294 \uc120\ud654 \uc5ed\uc774\uc5c8\ub2e4. \uae40\uae30\ub355\uc740 \u201c\u300a\ub098\uc05c \ub0a8\uc790\u300b\uc758 \uc120\ud654 \uc5ed\uc740 \uc11c\uc6d0\uc5d0\uac8c\ub9cc \uc81c\uc548\ud588\ub2e4. \uc120\ud654 \uc5ed\uc744 \ud574\ub0bc \ubc30\uc6b0\ub294 \uac70\uc758 \uc5c6\ub2e4. \uc2dc\ub098\ub9ac\uc624\ub97c \ubcf4\uba74 \uacf3\uacf3\uc5d0 \uce58\uac00 \ub5a8\ub9ac\ub294 \ubb18\uc0ac \ud22c\uc131\uc774\ub2e4\u201d\uace0 \ub9d0\ud558\uc600\ub2e4. \uc11c\uc6d0\uc758 \uc8fc\ubcc0 \uc0ac\ub78c\ub4e4\uc740 \uadf8\ub140\uac00 \uc774 \uc601\ud654\uc5d0 \ucd9c\uc5f0\ud558\ub294 \uac83\uc744 \ub9d0\ub838\uc73c\ub098, \uae40\uae30\ub355 \uac10\ub3c5\uc758 \uc791\ud488\uc5d0 \uad00\uc2ec\uc774 \ub9ce\uc558\ub358 \uc11c\uc6d0\uc740 \uc120\ud654 \uc5ed\uc744 \uc218\ub77d\ud558\uc600\ub2e4. \uae40\uae30\ub355\uc740 \u201c\uc11c\uc6d0\uc740 \uc120\ud654\ub97c \ub108\ubb34 \uc798 \ud574\ub0c8\ub2e4. \uadf8 \uc5b4\ub5a4 \ubc30\uc6b0\ubcf4\ub2e4 \uc2e4\uac10\ub098\uac8c \ud588\uace0, \ucc0d\uc73c\uba74\uc11c\ub3c4 \uc9c4\uc9dc \ucc3d\ub140\uac19\uc558\ub2e4\u201d\uace0 \uadf8\ub140\uc758 \uc5f0\uae30\ub97c \uce6d\ucc2c\ud588\uc73c\ub098, \uc11c\uc6d0\uc740 \u201c\uc601\ud63c\uc744 \ub2e4\ucce4\ub2e4\u201d\ub294 \ub9d0\ub85c \uc774 \uc5ed\uc774 \ubb34\ucc99 \uace0\ud1b5\uc2a4\ub7ec\uc6e0\uc74c\uc744 \ud45c\ud604\ud588\ub2e4. 2002\ub144 1\uc6d4 \uac1c\ubd09\ud55c \u300a\ub098\uc05c \ub0a8\uc790\u300b\ub294 \uc804\uad6d\uad00\uac1d 70\ub9cc \uba85 (\ube44\uacf5\uc2dd) \uc744 \uae30\ub85d\ud558\uba70 \uc608\uc0c1\uc678\ub85c \ud765\ud589\ud558\uc600\uace0, \uc11c\uc6d0\uc740 \uc774 \uc791\ud488\uc73c\ub85c 2002\ub144 \ub300\uc885\uc0c1 \uc2e0\uc778\uc5ec\uc6b0\uc0c1, \ucd98\uc0ac\ub098\uc6b4\uaddc\uc601\ud654\uc608\uc220\uc81c \uc62c\ud574\uc758 \uc2e0\uc778\uc5f0\uae30\uc0c1 \ubc0f \ud669\uae08\ucd2c\uc601\uc0c1 \uc2e0\uc778\uc5ec\uc6b0\uc0c1\uc744 \uc218\uc0c1\ud588\ub2e4. \ud55c\ud3b8, \u300a\ub098\uc05c \ub0a8\uc790\u300b\ub294 \u2018\uc5ec\uc131\uc5d0 \ub300\ud55c \ud3ed\ub825\uc131\u2019\uc73c\ub85c \uc5ec\uc131\uc6b4\ub3d9\uacc4\ub85c\ubd80\ud130 \ube44\ud310\uc744 \ubc1b\uc558\ub294\ub370, \uc870\uc7ac\ud604\uacfc \uc11c\uc6d0\uc740 \uac19\uc740 \ud574 \uc5ec\uc131\uad00\uac1d\uc601\ud654\uc0c1\uc5d0\uc11c \ucd5c\uc545\uc758 \ub0a8\ub140 \ubc30\uc6b0\ub85c \uac01\uac01 \uc120\uc815\ub418\uae30\ub3c4 \ud588\ub2e4.", "paragraph_answer": "\uac19\uc740 \ud574\uc778 2001\ub144 \uc11c\uc6d0\uc740 \uae40\uae30\ub355 \uac10\ub3c5\uc758 \uc601\ud654 \u300a\ub098\uc05c \ub0a8\uc790\u300b\uc5d0 \uc8fc\uc5f0\uc73c\ub85c \uce90\uc2a4\ud305\ub418\uc5c8\ub2e4. \uadf8\ub140\uac00 \ub9e1\uc740 \uc5ed\ud560\uc740 \uc0ac\ucc3d\uac00 \uae61\ud328\uc778 \ud55c\uae30(\uc870\uc7ac\ud604 \ubd84)\uc758 \uacc4\ub7b5\uc5d0 \ub9d0\ub824 \uc5ec\ub300\uc0dd\uc5d0\uc11c \ucc3d\ub140\ub85c \ucd94\ub77d\ud558\ub294 \uc120\ud654 \uc5ed\uc774\uc5c8\ub2e4. \uae40\uae30\ub355\uc740 \u201c\u300a\ub098\uc05c \ub0a8\uc790\u300b\uc758 \uc120\ud654 \uc5ed\uc740 \uc11c\uc6d0\uc5d0\uac8c\ub9cc \uc81c\uc548\ud588\ub2e4. \uc120\ud654 \uc5ed\uc744 \ud574\ub0bc \ubc30\uc6b0\ub294 \uac70\uc758 \uc5c6\ub2e4. \uc2dc\ub098\ub9ac\uc624\ub97c \ubcf4\uba74 \uacf3\uacf3\uc5d0 \uce58\uac00 \ub5a8\ub9ac\ub294 \ubb18\uc0ac \ud22c\uc131\uc774\ub2e4\u201d\uace0 \ub9d0\ud558\uc600\ub2e4. \uc11c\uc6d0\uc758 \uc8fc\ubcc0 \uc0ac\ub78c\ub4e4\uc740 \uadf8\ub140\uac00 \uc774 \uc601\ud654\uc5d0 \ucd9c\uc5f0\ud558\ub294 \uac83\uc744 \ub9d0\ub838\uc73c\ub098, \uae40\uae30\ub355 \uac10\ub3c5\uc758 \uc791\ud488\uc5d0 \uad00\uc2ec\uc774 \ub9ce\uc558\ub358 \uc11c\uc6d0\uc740 \uc120\ud654 \uc5ed\uc744 \uc218\ub77d\ud558\uc600\ub2e4. \uae40\uae30\ub355\uc740 \u201c\uc11c\uc6d0\uc740 \uc120\ud654\ub97c \ub108\ubb34 \uc798 \ud574\ub0c8\ub2e4. \uadf8 \uc5b4\ub5a4 \ubc30\uc6b0\ubcf4\ub2e4 \uc2e4\uac10\ub098\uac8c \ud588\uace0, \ucc0d\uc73c\uba74\uc11c\ub3c4 \uc9c4\uc9dc \ucc3d\ub140\uac19\uc558\ub2e4\u201d\uace0 \uadf8\ub140\uc758 \uc5f0\uae30\ub97c \uce6d\ucc2c\ud588\uc73c\ub098, \uc11c\uc6d0\uc740 \u201c\uc601\ud63c\uc744 \ub2e4\ucce4\ub2e4\u201d\ub294 \ub9d0\ub85c \uc774 \uc5ed\uc774 \ubb34\ucc99 \uace0\ud1b5\uc2a4\ub7ec\uc6e0\uc74c\uc744 \ud45c\ud604\ud588\ub2e4. 2002\ub144 1\uc6d4 \uac1c\ubd09\ud55c \u300a\ub098\uc05c \ub0a8\uc790\u300b\ub294 \uc804\uad6d\uad00\uac1d 70\ub9cc \uba85 (\ube44\uacf5\uc2dd)\uc744 \uae30\ub85d\ud558\uba70 \uc608\uc0c1\uc678\ub85c \ud765\ud589\ud558\uc600\uace0, \uc11c\uc6d0\uc740 \uc774 \uc791\ud488\uc73c\ub85c 2002\ub144 \ub300\uc885\uc0c1 \uc2e0\uc778\uc5ec\uc6b0\uc0c1, \ucd98\uc0ac\ub098\uc6b4\uaddc\uc601\ud654\uc608\uc220\uc81c \uc62c\ud574\uc758 \uc2e0\uc778\uc5f0\uae30\uc0c1 \ubc0f \ud669\uae08\ucd2c\uc601\uc0c1 \uc2e0\uc778\uc5ec\uc6b0\uc0c1\uc744 \uc218\uc0c1\ud588\ub2e4. \ud55c\ud3b8, \u300a\ub098\uc05c \ub0a8\uc790\u300b\ub294 \u2018\uc5ec\uc131\uc5d0 \ub300\ud55c \ud3ed\ub825\uc131\u2019\uc73c\ub85c \uc5ec\uc131\uc6b4\ub3d9\uacc4\ub85c\ubd80\ud130 \ube44\ud310\uc744 \ubc1b\uc558\ub294\ub370, \uc870\uc7ac\ud604\uacfc \uc11c\uc6d0\uc740 \uac19\uc740 \ud574 \uc5ec\uc131\uad00\uac1d\uc601\ud654\uc0c1\uc5d0\uc11c \ucd5c\uc545\uc758 \ub0a8\ub140 \ubc30\uc6b0\ub85c \uac01\uac01 \uc120\uc815\ub418\uae30\ub3c4 \ud588\ub2e4.", "sentence_answer": "2002\ub144 1\uc6d4 \uac1c\ubd09\ud55c \u300a\ub098\uc05c \ub0a8\uc790\u300b\ub294 \uc804\uad6d\uad00\uac1d 70\ub9cc \uba85 (\ube44\uacf5\uc2dd)", "paragraph_id": "6439450-1-2", "paragraph_question": "question: <<\ub098\uc05c \ub0a8\uc790>>\uc758 \uad00\uac1d \ub3d9\uc6d0 \uc218\ub294 \uba87 \uba85\uc778\uac00?, context: \uac19\uc740 \ud574\uc778 2001\ub144 \uc11c\uc6d0\uc740 \uae40\uae30\ub355 \uac10\ub3c5\uc758 \uc601\ud654 \u300a\ub098\uc05c \ub0a8\uc790\u300b\uc5d0 \uc8fc\uc5f0\uc73c\ub85c \uce90\uc2a4\ud305\ub418\uc5c8\ub2e4. \uadf8\ub140\uac00 \ub9e1\uc740 \uc5ed\ud560\uc740 \uc0ac\ucc3d\uac00 \uae61\ud328\uc778 \ud55c\uae30(\uc870\uc7ac\ud604 \ubd84)\uc758 \uacc4\ub7b5\uc5d0 \ub9d0\ub824 \uc5ec\ub300\uc0dd\uc5d0\uc11c \ucc3d\ub140\ub85c \ucd94\ub77d\ud558\ub294 \uc120\ud654 \uc5ed\uc774\uc5c8\ub2e4. \uae40\uae30\ub355\uc740 \u201c\u300a\ub098\uc05c \ub0a8\uc790\u300b\uc758 \uc120\ud654 \uc5ed\uc740 \uc11c\uc6d0\uc5d0\uac8c\ub9cc \uc81c\uc548\ud588\ub2e4. \uc120\ud654 \uc5ed\uc744 \ud574\ub0bc \ubc30\uc6b0\ub294 \uac70\uc758 \uc5c6\ub2e4. \uc2dc\ub098\ub9ac\uc624\ub97c \ubcf4\uba74 \uacf3\uacf3\uc5d0 \uce58\uac00 \ub5a8\ub9ac\ub294 \ubb18\uc0ac \ud22c\uc131\uc774\ub2e4\u201d\uace0 \ub9d0\ud558\uc600\ub2e4. \uc11c\uc6d0\uc758 \uc8fc\ubcc0 \uc0ac\ub78c\ub4e4\uc740 \uadf8\ub140\uac00 \uc774 \uc601\ud654\uc5d0 \ucd9c\uc5f0\ud558\ub294 \uac83\uc744 \ub9d0\ub838\uc73c\ub098, \uae40\uae30\ub355 \uac10\ub3c5\uc758 \uc791\ud488\uc5d0 \uad00\uc2ec\uc774 \ub9ce\uc558\ub358 \uc11c\uc6d0\uc740 \uc120\ud654 \uc5ed\uc744 \uc218\ub77d\ud558\uc600\ub2e4. \uae40\uae30\ub355\uc740 \u201c\uc11c\uc6d0\uc740 \uc120\ud654\ub97c \ub108\ubb34 \uc798 \ud574\ub0c8\ub2e4. \uadf8 \uc5b4\ub5a4 \ubc30\uc6b0\ubcf4\ub2e4 \uc2e4\uac10\ub098\uac8c \ud588\uace0, \ucc0d\uc73c\uba74\uc11c\ub3c4 \uc9c4\uc9dc \ucc3d\ub140\uac19\uc558\ub2e4\u201d\uace0 \uadf8\ub140\uc758 \uc5f0\uae30\ub97c \uce6d\ucc2c\ud588\uc73c\ub098, \uc11c\uc6d0\uc740 \u201c\uc601\ud63c\uc744 \ub2e4\ucce4\ub2e4\u201d\ub294 \ub9d0\ub85c \uc774 \uc5ed\uc774 \ubb34\ucc99 \uace0\ud1b5\uc2a4\ub7ec\uc6e0\uc74c\uc744 \ud45c\ud604\ud588\ub2e4. 2002\ub144 1\uc6d4 \uac1c\ubd09\ud55c \u300a\ub098\uc05c \ub0a8\uc790\u300b\ub294 \uc804\uad6d\uad00\uac1d 70\ub9cc \uba85(\ube44\uacf5\uc2dd)\uc744 \uae30\ub85d\ud558\uba70 \uc608\uc0c1\uc678\ub85c \ud765\ud589\ud558\uc600\uace0, \uc11c\uc6d0\uc740 \uc774 \uc791\ud488\uc73c\ub85c 2002\ub144 \ub300\uc885\uc0c1 \uc2e0\uc778\uc5ec\uc6b0\uc0c1, \ucd98\uc0ac\ub098\uc6b4\uaddc\uc601\ud654\uc608\uc220\uc81c \uc62c\ud574\uc758 \uc2e0\uc778\uc5f0\uae30\uc0c1 \ubc0f \ud669\uae08\ucd2c\uc601\uc0c1 \uc2e0\uc778\uc5ec\uc6b0\uc0c1\uc744 \uc218\uc0c1\ud588\ub2e4. \ud55c\ud3b8, \u300a\ub098\uc05c \ub0a8\uc790\u300b\ub294 \u2018\uc5ec\uc131\uc5d0 \ub300\ud55c \ud3ed\ub825\uc131\u2019\uc73c\ub85c \uc5ec\uc131\uc6b4\ub3d9\uacc4\ub85c\ubd80\ud130 \ube44\ud310\uc744 \ubc1b\uc558\ub294\ub370, \uc870\uc7ac\ud604\uacfc \uc11c\uc6d0\uc740 \uac19\uc740 \ud574 \uc5ec\uc131\uad00\uac1d\uc601\ud654\uc0c1\uc5d0\uc11c \ucd5c\uc545\uc758 \ub0a8\ub140 \ubc30\uc6b0\ub85c \uac01\uac01 \uc120\uc815\ub418\uae30\ub3c4 \ud588\ub2e4."} {"question": "\uc774\ud6a8\ub9ac\uac00 \uc120\uacf5\uac1c\uace1 \ubbf8\uc2a4\ucf54\ub9ac\uc544\ub97c \ubc1c\ub9e4\ud55c \uc2dc\uae30\ub294 \uc5b8\uc81c\uc778\uac00?", "paragraph": "2013\ub144 5\uc6d4 6\uc77c\uc5d0\ub294 \uc0c8\ub85c\uc6b4 \uc568\ubc94 \ubc1c\ub9e4\uc5d0 \uc55e\uc11c \uc120\uacf5\uac1c\uace1 \"\ubbf8\uc2a4\ucf54\ub9ac\uc544\" \ub97c \ubc1c\ub9e4\ud558\uc600\uace0, \uc774\uace1\uc740 \uc774\ud6a8\ub9ac\uac00 \uc9c1\uc811 \uc791\uc0ac.\uc791\uace1\ud55c \uace1\uc73c\ub85c \ub9ce\uc740 \ud654\uc81c\ub97c \ubaa8\uc558\ub2e4. \"\ubbf8\uc2a4\ucf54\ub9ac\uc544\" \ub294 \uac00\uc628 \ub514\uc9c0\ud138 \uc885\ud569 \ucc28\ud2b8\uc5d0\uc11c 1\uc704,, \uba5c\ub860, \uc5e0\ub137, \uc62c\ub808 \ubba4\uc9c1 \ub4f1 \uad6d\ub0b4 \uc8fc\uc694 \uc74c\uc545\uc0ac\uc774\ud2b8\uc5d0\uc11c \uc8fc\uac04\ucc28\ud2b8 1\uc704\ub97c \ucc28\uc9c0\ud558\uc600\ub2e4. 5\uc6d4 21\uc77c\uc5d0\ub294 \uc774\ud6a8\ub9ac\uc758 \ub2e4\uc12f \ubc88\uc9f8 \uc815\uaddc \uc568\ubc94 MONOCHROME\uc774 \ubc1c\ub9e4\ub418\uc5c8\ub2e4. 22\uc77c\uc5d0\ub294 \uacf5\uc911\ud30c\ub85c \ucef4\ubc31\ud558\uae30\uc804 M-net\uc5d0\uc11c \ubc29\uc601\ud55c \ucef4\ubc31\uc1fc \u300a2HYORI SHOW\u300b\ub97c \ud1b5\ud574\uc11c 5\uc9d1\uc758 \ud0c0\uc774\ud2c0\uace1\uacfc \uc218\ub85d\uace1\ub4e4\uc744 \ucd5c\ucd08\ub85c \uacf5\uac1c\ud558\uc600\ub2e4. \ud0c0\uc774\ud2c0\uace1\uc740 \"Bad Girls\"\uc774\uba70 \uc5b4\ucfe0\uc2a4\ud2f1 \ubc34\ub4dc\uc0ac\uc6b4\ub4dc\ub85c\ub9cc \uad6c\uc131\ub41c \ub304\uc2a4\uace1\uc73c\ub85c \uc774\ud6a8\ub9ac\uac00 \uc9c1\uc811 \uc791\uc0ac\ud588\uace0, SBS \u300a\uc778\uae30\uac00\uc694\u300b, KBS \u300a\ubba4\uc9c1\ubc45\ud06c\u300b, M-net \u300a\uc5e0\uce74\uc6b4\ud2b8\ub2e4\uc6b4\u300b\uc5d0\uc11c 1\uc704\ub97c \ud558\uc600\ub2e4. \uadf8\ub9ac\uace0 On Style\uc5d0\uc11c \ubc29\uc601\ub41c 5\ubd80\uc791 \ub9ac\uc5bc\ub9ac\ud2f0 \ud504\ub85c\uadf8\ub7a8\u300a\uc774\ud6a8\ub9ac\uc758 x\uc5b8\ub2c8\u300b\ub97c \ud1b5\ud574\uc11c \uac19\uc740 \uc18c\uc18d\uc0ac \ud6c4\ubc30 \uac78 \uadf8\ub8f9 \uc2a4\ud53c\uce74\ub97c \ud504\ub85c\ub4c0\uc2f1\ud574 \ud654\uc81c\ub97c \ubaa8\uc73c\uae30\ub3c4 \ud558\uc600\uace0, \uc790\uc2e0\uc774 \uc791\uc0ac.\uc791\uace1\ud55c \uace1 \"\ube14\ub799 \uc564 \ud654\uc774\ud2b8\" \ub294 \uac00\uc778\uc758 \uc568\ubc94\uc5d0 \uc218\ub85d\uace1\uc73c\ub85c \uc2e4\ub9ac\uae30\ub3c4 \ud558\uc600\ub2e4. \ud55c\ud3b8, \uc774\ud6a8\ub9ac\ub294 9\uc6d4 1\uc77c \uc81c\uc8fc\ub3c4\uc5d0 \uc704\uce58\ud55c \uc790\uc2e0\uc758 \ubcc4\uc7a5\uc5d0\uc11c \uac00\uc218 \uc774\uc0c1\uc21c\uacfc \uacb0\ud63c\uc2dd\uc744 \uc62c\ub838\ub2e4.", "answer": "2013\ub144", "sentence": "2013\ub144 5\uc6d4 6\uc77c\uc5d0\ub294 \uc0c8\ub85c\uc6b4 \uc568\ubc94 \ubc1c\ub9e4\uc5d0 \uc55e\uc11c \uc120\uacf5\uac1c\uace1 \"\ubbf8\uc2a4\ucf54\ub9ac\uc544\" \ub97c \ubc1c\ub9e4\ud558\uc600\uace0, \uc774\uace1\uc740 \uc774\ud6a8\ub9ac\uac00 \uc9c1\uc811 \uc791\uc0ac.\uc791\uace1\ud55c \uace1\uc73c\ub85c \ub9ce\uc740 \ud654\uc81c\ub97c \ubaa8\uc558\ub2e4.", "paragraph_sentence": " 2013\ub144 5\uc6d4 6\uc77c\uc5d0\ub294 \uc0c8\ub85c\uc6b4 \uc568\ubc94 \ubc1c\ub9e4\uc5d0 \uc55e\uc11c \uc120\uacf5\uac1c\uace1 \"\ubbf8\uc2a4\ucf54\ub9ac\uc544\" \ub97c \ubc1c\ub9e4\ud558\uc600\uace0, \uc774\uace1\uc740 \uc774\ud6a8\ub9ac\uac00 \uc9c1\uc811 \uc791\uc0ac.\uc791\uace1\ud55c \uace1\uc73c\ub85c \ub9ce\uc740 \ud654\uc81c\ub97c \ubaa8\uc558\ub2e4. \"\ubbf8\uc2a4\ucf54\ub9ac\uc544\" \ub294 \uac00\uc628 \ub514\uc9c0\ud138 \uc885\ud569 \ucc28\ud2b8\uc5d0\uc11c 1\uc704,, \uba5c\ub860, \uc5e0\ub137, \uc62c\ub808 \ubba4\uc9c1 \ub4f1 \uad6d\ub0b4 \uc8fc\uc694 \uc74c\uc545\uc0ac\uc774\ud2b8\uc5d0\uc11c \uc8fc\uac04\ucc28\ud2b8 1\uc704\ub97c \ucc28\uc9c0\ud558\uc600\ub2e4. 5\uc6d4 21\uc77c\uc5d0\ub294 \uc774\ud6a8\ub9ac\uc758 \ub2e4\uc12f \ubc88\uc9f8 \uc815\uaddc \uc568\ubc94 MONOCHROME\uc774 \ubc1c\ub9e4\ub418\uc5c8\ub2e4. 22\uc77c\uc5d0\ub294 \uacf5\uc911\ud30c\ub85c \ucef4\ubc31\ud558\uae30\uc804 M-net\uc5d0\uc11c \ubc29\uc601\ud55c \ucef4\ubc31\uc1fc \u300a2HYORI SHOW\u300b\ub97c \ud1b5\ud574\uc11c 5\uc9d1\uc758 \ud0c0\uc774\ud2c0\uace1\uacfc \uc218\ub85d\uace1\ub4e4\uc744 \ucd5c\ucd08\ub85c \uacf5\uac1c\ud558\uc600\ub2e4. \ud0c0\uc774\ud2c0\uace1\uc740 \"Bad Girls\"\uc774\uba70 \uc5b4\ucfe0\uc2a4\ud2f1 \ubc34\ub4dc\uc0ac\uc6b4\ub4dc\ub85c\ub9cc \uad6c\uc131\ub41c \ub304\uc2a4\uace1\uc73c\ub85c \uc774\ud6a8\ub9ac\uac00 \uc9c1\uc811 \uc791\uc0ac\ud588\uace0, SBS \u300a\uc778\uae30\uac00\uc694\u300b, KBS \u300a\ubba4\uc9c1\ubc45\ud06c\u300b, M-net \u300a\uc5e0\uce74\uc6b4\ud2b8\ub2e4\uc6b4\u300b\uc5d0\uc11c 1\uc704\ub97c \ud558\uc600\ub2e4. \uadf8\ub9ac\uace0 On Style\uc5d0\uc11c \ubc29\uc601\ub41c 5\ubd80\uc791 \ub9ac\uc5bc\ub9ac\ud2f0 \ud504\ub85c\uadf8\ub7a8\u300a\uc774\ud6a8\ub9ac\uc758 x\uc5b8\ub2c8\u300b\ub97c \ud1b5\ud574\uc11c \uac19\uc740 \uc18c\uc18d\uc0ac \ud6c4\ubc30 \uac78 \uadf8\ub8f9 \uc2a4\ud53c\uce74\ub97c \ud504\ub85c\ub4c0\uc2f1\ud574 \ud654\uc81c\ub97c \ubaa8\uc73c\uae30\ub3c4 \ud558\uc600\uace0, \uc790\uc2e0\uc774 \uc791\uc0ac.\uc791\uace1\ud55c \uace1 \"\ube14\ub799 \uc564 \ud654\uc774\ud2b8\" \ub294 \uac00\uc778\uc758 \uc568\ubc94\uc5d0 \uc218\ub85d\uace1\uc73c\ub85c \uc2e4\ub9ac\uae30\ub3c4 \ud558\uc600\ub2e4. \ud55c\ud3b8, \uc774\ud6a8\ub9ac\ub294 9\uc6d4 1\uc77c \uc81c\uc8fc\ub3c4\uc5d0 \uc704\uce58\ud55c \uc790\uc2e0\uc758 \ubcc4\uc7a5\uc5d0\uc11c \uac00\uc218 \uc774\uc0c1\uc21c\uacfc \uacb0\ud63c\uc2dd\uc744 \uc62c\ub838\ub2e4.", "paragraph_answer": " 2013\ub144 5\uc6d4 6\uc77c\uc5d0\ub294 \uc0c8\ub85c\uc6b4 \uc568\ubc94 \ubc1c\ub9e4\uc5d0 \uc55e\uc11c \uc120\uacf5\uac1c\uace1 \"\ubbf8\uc2a4\ucf54\ub9ac\uc544\" \ub97c \ubc1c\ub9e4\ud558\uc600\uace0, \uc774\uace1\uc740 \uc774\ud6a8\ub9ac\uac00 \uc9c1\uc811 \uc791\uc0ac.\uc791\uace1\ud55c \uace1\uc73c\ub85c \ub9ce\uc740 \ud654\uc81c\ub97c \ubaa8\uc558\ub2e4. \"\ubbf8\uc2a4\ucf54\ub9ac\uc544\" \ub294 \uac00\uc628 \ub514\uc9c0\ud138 \uc885\ud569 \ucc28\ud2b8\uc5d0\uc11c 1\uc704,, \uba5c\ub860, \uc5e0\ub137, \uc62c\ub808 \ubba4\uc9c1 \ub4f1 \uad6d\ub0b4 \uc8fc\uc694 \uc74c\uc545\uc0ac\uc774\ud2b8\uc5d0\uc11c \uc8fc\uac04\ucc28\ud2b8 1\uc704\ub97c \ucc28\uc9c0\ud558\uc600\ub2e4. 5\uc6d4 21\uc77c\uc5d0\ub294 \uc774\ud6a8\ub9ac\uc758 \ub2e4\uc12f \ubc88\uc9f8 \uc815\uaddc \uc568\ubc94 MONOCHROME\uc774 \ubc1c\ub9e4\ub418\uc5c8\ub2e4. 22\uc77c\uc5d0\ub294 \uacf5\uc911\ud30c\ub85c \ucef4\ubc31\ud558\uae30\uc804 M-net\uc5d0\uc11c \ubc29\uc601\ud55c \ucef4\ubc31\uc1fc \u300a2HYORI SHOW\u300b\ub97c \ud1b5\ud574\uc11c 5\uc9d1\uc758 \ud0c0\uc774\ud2c0\uace1\uacfc \uc218\ub85d\uace1\ub4e4\uc744 \ucd5c\ucd08\ub85c \uacf5\uac1c\ud558\uc600\ub2e4. \ud0c0\uc774\ud2c0\uace1\uc740 \"Bad Girls\"\uc774\uba70 \uc5b4\ucfe0\uc2a4\ud2f1 \ubc34\ub4dc\uc0ac\uc6b4\ub4dc\ub85c\ub9cc \uad6c\uc131\ub41c \ub304\uc2a4\uace1\uc73c\ub85c \uc774\ud6a8\ub9ac\uac00 \uc9c1\uc811 \uc791\uc0ac\ud588\uace0, SBS \u300a\uc778\uae30\uac00\uc694\u300b, KBS \u300a\ubba4\uc9c1\ubc45\ud06c\u300b, M-net \u300a\uc5e0\uce74\uc6b4\ud2b8\ub2e4\uc6b4\u300b\uc5d0\uc11c 1\uc704\ub97c \ud558\uc600\ub2e4. \uadf8\ub9ac\uace0 On Style\uc5d0\uc11c \ubc29\uc601\ub41c 5\ubd80\uc791 \ub9ac\uc5bc\ub9ac\ud2f0 \ud504\ub85c\uadf8\ub7a8\u300a\uc774\ud6a8\ub9ac\uc758 x\uc5b8\ub2c8\u300b\ub97c \ud1b5\ud574\uc11c \uac19\uc740 \uc18c\uc18d\uc0ac \ud6c4\ubc30 \uac78 \uadf8\ub8f9 \uc2a4\ud53c\uce74\ub97c \ud504\ub85c\ub4c0\uc2f1\ud574 \ud654\uc81c\ub97c \ubaa8\uc73c\uae30\ub3c4 \ud558\uc600\uace0, \uc790\uc2e0\uc774 \uc791\uc0ac.\uc791\uace1\ud55c \uace1 \"\ube14\ub799 \uc564 \ud654\uc774\ud2b8\" \ub294 \uac00\uc778\uc758 \uc568\ubc94\uc5d0 \uc218\ub85d\uace1\uc73c\ub85c \uc2e4\ub9ac\uae30\ub3c4 \ud558\uc600\ub2e4. \ud55c\ud3b8, \uc774\ud6a8\ub9ac\ub294 9\uc6d4 1\uc77c \uc81c\uc8fc\ub3c4\uc5d0 \uc704\uce58\ud55c \uc790\uc2e0\uc758 \ubcc4\uc7a5\uc5d0\uc11c \uac00\uc218 \uc774\uc0c1\uc21c\uacfc \uacb0\ud63c\uc2dd\uc744 \uc62c\ub838\ub2e4.", "sentence_answer": " 2013\ub144 5\uc6d4 6\uc77c\uc5d0\ub294 \uc0c8\ub85c\uc6b4 \uc568\ubc94 \ubc1c\ub9e4\uc5d0 \uc55e\uc11c \uc120\uacf5\uac1c\uace1 \"\ubbf8\uc2a4\ucf54\ub9ac\uc544\" \ub97c \ubc1c\ub9e4\ud558\uc600\uace0, \uc774\uace1\uc740 \uc774\ud6a8\ub9ac\uac00 \uc9c1\uc811 \uc791\uc0ac.\uc791\uace1\ud55c \uace1\uc73c\ub85c \ub9ce\uc740 \ud654\uc81c\ub97c \ubaa8\uc558\ub2e4.", "paragraph_id": "6606078-8-0", "paragraph_question": "question: \uc774\ud6a8\ub9ac\uac00 \uc120\uacf5\uac1c\uace1 \ubbf8\uc2a4\ucf54\ub9ac\uc544\ub97c \ubc1c\ub9e4\ud55c \uc2dc\uae30\ub294 \uc5b8\uc81c\uc778\uac00?, context: 2013\ub144 5\uc6d4 6\uc77c\uc5d0\ub294 \uc0c8\ub85c\uc6b4 \uc568\ubc94 \ubc1c\ub9e4\uc5d0 \uc55e\uc11c \uc120\uacf5\uac1c\uace1 \"\ubbf8\uc2a4\ucf54\ub9ac\uc544\" \ub97c \ubc1c\ub9e4\ud558\uc600\uace0, \uc774\uace1\uc740 \uc774\ud6a8\ub9ac\uac00 \uc9c1\uc811 \uc791\uc0ac.\uc791\uace1\ud55c \uace1\uc73c\ub85c \ub9ce\uc740 \ud654\uc81c\ub97c \ubaa8\uc558\ub2e4. \"\ubbf8\uc2a4\ucf54\ub9ac\uc544\" \ub294 \uac00\uc628 \ub514\uc9c0\ud138 \uc885\ud569 \ucc28\ud2b8\uc5d0\uc11c 1\uc704,, \uba5c\ub860, \uc5e0\ub137, \uc62c\ub808 \ubba4\uc9c1 \ub4f1 \uad6d\ub0b4 \uc8fc\uc694 \uc74c\uc545\uc0ac\uc774\ud2b8\uc5d0\uc11c \uc8fc\uac04\ucc28\ud2b8 1\uc704\ub97c \ucc28\uc9c0\ud558\uc600\ub2e4. 5\uc6d4 21\uc77c\uc5d0\ub294 \uc774\ud6a8\ub9ac\uc758 \ub2e4\uc12f \ubc88\uc9f8 \uc815\uaddc \uc568\ubc94 MONOCHROME\uc774 \ubc1c\ub9e4\ub418\uc5c8\ub2e4. 22\uc77c\uc5d0\ub294 \uacf5\uc911\ud30c\ub85c \ucef4\ubc31\ud558\uae30\uc804 M-net\uc5d0\uc11c \ubc29\uc601\ud55c \ucef4\ubc31\uc1fc \u300a2HYORI SHOW\u300b\ub97c \ud1b5\ud574\uc11c 5\uc9d1\uc758 \ud0c0\uc774\ud2c0\uace1\uacfc \uc218\ub85d\uace1\ub4e4\uc744 \ucd5c\ucd08\ub85c \uacf5\uac1c\ud558\uc600\ub2e4. \ud0c0\uc774\ud2c0\uace1\uc740 \"Bad Girls\"\uc774\uba70 \uc5b4\ucfe0\uc2a4\ud2f1 \ubc34\ub4dc\uc0ac\uc6b4\ub4dc\ub85c\ub9cc \uad6c\uc131\ub41c \ub304\uc2a4\uace1\uc73c\ub85c \uc774\ud6a8\ub9ac\uac00 \uc9c1\uc811 \uc791\uc0ac\ud588\uace0, SBS \u300a\uc778\uae30\uac00\uc694\u300b, KBS \u300a\ubba4\uc9c1\ubc45\ud06c\u300b, M-net \u300a\uc5e0\uce74\uc6b4\ud2b8\ub2e4\uc6b4\u300b\uc5d0\uc11c 1\uc704\ub97c \ud558\uc600\ub2e4. \uadf8\ub9ac\uace0 On Style\uc5d0\uc11c \ubc29\uc601\ub41c 5\ubd80\uc791 \ub9ac\uc5bc\ub9ac\ud2f0 \ud504\ub85c\uadf8\ub7a8\u300a\uc774\ud6a8\ub9ac\uc758 x\uc5b8\ub2c8\u300b\ub97c \ud1b5\ud574\uc11c \uac19\uc740 \uc18c\uc18d\uc0ac \ud6c4\ubc30 \uac78 \uadf8\ub8f9 \uc2a4\ud53c\uce74\ub97c \ud504\ub85c\ub4c0\uc2f1\ud574 \ud654\uc81c\ub97c \ubaa8\uc73c\uae30\ub3c4 \ud558\uc600\uace0, \uc790\uc2e0\uc774 \uc791\uc0ac.\uc791\uace1\ud55c \uace1 \"\ube14\ub799 \uc564 \ud654\uc774\ud2b8\" \ub294 \uac00\uc778\uc758 \uc568\ubc94\uc5d0 \uc218\ub85d\uace1\uc73c\ub85c \uc2e4\ub9ac\uae30\ub3c4 \ud558\uc600\ub2e4. \ud55c\ud3b8, \uc774\ud6a8\ub9ac\ub294 9\uc6d4 1\uc77c \uc81c\uc8fc\ub3c4\uc5d0 \uc704\uce58\ud55c \uc790\uc2e0\uc758 \ubcc4\uc7a5\uc5d0\uc11c \uac00\uc218 \uc774\uc0c1\uc21c\uacfc \uacb0\ud63c\uc2dd\uc744 \uc62c\ub838\ub2e4."} {"question": "\uc774\uc885\uc6b1\uc774 FA \uacc4\uc57d\uc73c\ub85c \uc62e\uae34 \ud300\uc740?", "paragraph": "\uc791\ub144 \ucee4\ub9ac\uc5b4 \ub85c\uc6b0\uc758 \uc131\uc801\uc744 \uac70\ub450\uc5c8\uace0, \uc62c \uc2dc\uc98c \ub4a4\uc5d0 \uc790\uc720 \uacc4\uc57d \uc120\uc218 \uc790\uaca9\uc744 \uc5bb\uae30 \ub54c\ubb38\uc5d0 2013\ub144 \uc2dc\uc98c\uc740 \uadf8\uc5d0\uac8c \uc788\uc5b4\uc11c \uc911\uc694\ud55c \ud574\uc600\ub2e4. \uac1c\ub9c9\uc804\ubd80\ud130 19\uacbd\uae30 \uc5f0\uc18d \ucd9c\ub8e8 \ud589\uc9c4\uc744 \uc774\uc5b4\uac14\uc73c\ub098, 4\uc6d4\ub9d0\uc5d0 \uc67c\ucabd \ud584\uc2a4\ud2b8\ub9c1 \ubd80\uc0c1\uc73c\ub85c \uc7a0\uc2dc 2\uad70\uc5d0 \ub2e4\ub140\uc624\uae30\ub3c4 \ud588\ub2e4. \uc774\ud6c4 \uc88b\uc740 \ud0c0\uaca9\uac10\uc744 \uc720\uc9c0\ud558\uba70 6\uc6d4 .388\ub85c \uc6d4\uac04 \ud0c0\uc728 \ub9ac\uadf8 1\uc704\ub97c \uae30\ub85d\ud588\uc73c\uba70, 6\uc6d4 \ud300\ub0b4 \ud0c0\uc790 \ubd80\ubb38 MVP\ub85c\ub3c4 \uc120\uc815\ub418\uc5c8\ub2e4. \uc62c\uc2a4\ud0c0\uc804\uc5d0\ub294 \uac10\ub3c5 \ucd94\ucc9c \uc120\uc218\ub85c \uc120\ubc1c\ub418\uc5c8\ub2e4. 8\uc6d4 \ud558\uc21c\uc5d0 \uc885\uc544\ub9ac \ud1b5\uc99d\uc73c\ub85c \uc7a0\uc2dc 2\uad70\uc5d0 \ub2e4\ub140\uc654\ub2e4\uac00 \ubcf5\uadc0\ud588\ub2e4. \uadf8\ub294 \uc774\ubc88 \uc2dc\uc98c\ub124 \uac1c\uc778 \ud55c \uc2dc\uc98c \ud648\ub7f0, 2\ub8e8\ud0c0 \uae30\ub85d\uc744 \uac31\uc2e0\ud558\uba70 \uc8fc\ub825\uc5d0 \uc7a5\ud0c0\ub825\uae4c\uc9c0 \uacb8\ube44\ud55c \uc644\uc131\ud615 1\ubc88 \ud0c0\uc790\uc758 \ubaa8\uc2b5\uc744 \ubcf4\uc5ec\uc8fc\uc5c8\ub2e4. \ub450\uc0b0 \ubca0\uc5b4\uc2a4\ub294 4\uc704\ub85c \ud3ec\uc2a4\ud2b8\uc2dc\uc98c\uc5d0 \uc9c4\ucd9c\ud558\uc600\ub2e4. \uc900\ud50c\ub808\uc774\uc624\ud504\uc5d0\uc11c \ub125\uc13c \ud788\uc5b4\ub85c\uc988\uc640 \ub9de\ubd99\uc5b4 2\uc5f0\ud328 \ub4a4 3\uc5f0\uc2b9\uc744 \uac70\ub450\uba70 \ud50c\ub808\uc774\uc624\ud504\uc5d0 \uc9c4\ucd9c\ud558\uc600\uc73c\uba70, \ud50c\ub808\uc774\uc624\ud504\uc5d0\uc11c LG \ud2b8\uc708\uc2a4\uc640 \ub9de\ubd99\uc5c8\ub2e4. \uadf8\ub294 1\ucc28\uc804 1\ud68c\ucd08 \uccab \ud0c0\uc11d\uc5d0\uc11c \uc6b0\uc911\uac04 \uc678\uc57c\ub97c \uaff0\ub6ab\ub294 3\ub8e8\ud0c0\ub97c \ud130\ub728\ub838\uc73c\uba70, \ud6c4\uc18d \ud0c0\uc790\uc758 \uc801\uc2dc\ud0c0\uc5d0 \ud648\uc73c\ub85c \ub4e4\uc5b4\uc654\ub2e4. \uc774\ub0a0 \uacbd\uae30\uc5d0\uc11c 2\ub4dd\uc810\uc744 \uc62c\ub838\ub2e4. 4\ucc28\uc804\uc5d0\uc11c\ub294 \ud76c\uc0dd \ud50c\ub77c\uc774\ub85c \uacb0\uc2b9 \ud0c0\uc810\uc744 \uc62c\ub838\uc73c\uba70, \uc774 \uacbd\uae30\uc5d0\uc11c \uc2b9\ub9ac\ud558\uba70 \ud55c\uad6d\uc2dc\ub9ac\uc988\uc5d0 \uc9c4\ucd9c\ud588\ub2e4. \uc0bc\uc131 \ub77c\uc774\uc628\uc988\uc640 \ub9de\ubd99\uc740 \ud55c\uad6d\uc2dc\ub9ac\uc988\uc5d0\uc11c 2\ud560 \ub300\uc758 \ud0c0\uc728\ub85c \ubd80\uc9c4\ud588\uc73c\uba70 \uc2dc\ub9ac\uc988 \uc911 \ud558\uc704 \ud0c0\uc120\uc5d0 \ubc30\uce58\ub418\uae30\ub3c4 \ud588\ub2e4. \uc2dc\uc98c \ud6c4 FA\ub97c \uc120\uc5b8\ud558\uc600\uc73c\ub098 \uc18c\uc18d \uad6c\ub2e8\uacfc\uc758 \uc6b0\uc120 \ud611\uc0c1 \uae30\uac04 \ub3d9\uc548 \uacc4\uc57d \uc870\uac74\uc5d0 \uc774\uacac\uc744 \ubcf4\uc5ec \ud611\uc0c1\uc774 \uacb0\ub82c\ub418\uc5c8\ub2e4. 11\uc6d4 17\uc77c, 4\ub144 \ucd1d\uc561 50\uc5b5\uc6d0(\uacc4\uc57d\uae08 28\uc5b5\uc6d0, \uc5f0\ubd09 5\uc5b5\uc6d0, \uc635\uc158 2\uc5b5\uc6d0)\uc758 \uc870\uac74\uc73c\ub85c NC \ub2e4\uc774\ub178\uc2a4\ub85c \uc774\uc801\ud558\uc600\ub2e4. 11\uc6d4 29\uc77c \uc5f4\ub9b0 ADT \ucea1\uc2a4\ud50c\ub808\uc774 2013\uc5d0\uc11c \ub300\uc0c1\uc744 \uc218\uc0c1\ud558\uc600\ub2e4.", "answer": "NC \ub2e4\uc774\ub178\uc2a4", "sentence": "11\uc6d4 17\uc77c, 4\ub144 \ucd1d\uc561 50\uc5b5\uc6d0(\uacc4\uc57d\uae08 28\uc5b5\uc6d0, \uc5f0\ubd09 5\uc5b5\uc6d0, \uc635\uc158 2\uc5b5\uc6d0)\uc758 \uc870\uac74\uc73c\ub85c NC \ub2e4\uc774\ub178\uc2a4 \ub85c \uc774\uc801\ud558\uc600\ub2e4.", "paragraph_sentence": "\uc791\ub144 \ucee4\ub9ac\uc5b4 \ub85c\uc6b0\uc758 \uc131\uc801\uc744 \uac70\ub450\uc5c8\uace0, \uc62c \uc2dc\uc98c \ub4a4\uc5d0 \uc790\uc720 \uacc4\uc57d \uc120\uc218 \uc790\uaca9\uc744 \uc5bb\uae30 \ub54c\ubb38\uc5d0 2013\ub144 \uc2dc\uc98c\uc740 \uadf8\uc5d0\uac8c \uc788\uc5b4\uc11c \uc911\uc694\ud55c \ud574\uc600\ub2e4. \uac1c\ub9c9\uc804\ubd80\ud130 19\uacbd\uae30 \uc5f0\uc18d \ucd9c\ub8e8 \ud589\uc9c4\uc744 \uc774\uc5b4\uac14\uc73c\ub098, 4\uc6d4\ub9d0\uc5d0 \uc67c\ucabd \ud584\uc2a4\ud2b8\ub9c1 \ubd80\uc0c1\uc73c\ub85c \uc7a0\uc2dc 2\uad70\uc5d0 \ub2e4\ub140\uc624\uae30\ub3c4 \ud588\ub2e4. \uc774\ud6c4 \uc88b\uc740 \ud0c0\uaca9\uac10\uc744 \uc720\uc9c0\ud558\uba70 6\uc6d4 .388\ub85c \uc6d4\uac04 \ud0c0\uc728 \ub9ac\uadf8 1\uc704\ub97c \uae30\ub85d\ud588\uc73c\uba70, 6\uc6d4 \ud300\ub0b4 \ud0c0\uc790 \ubd80\ubb38 MVP\ub85c\ub3c4 \uc120\uc815\ub418\uc5c8\ub2e4. \uc62c\uc2a4\ud0c0\uc804\uc5d0\ub294 \uac10\ub3c5 \ucd94\ucc9c \uc120\uc218\ub85c \uc120\ubc1c\ub418\uc5c8\ub2e4. 8\uc6d4 \ud558\uc21c\uc5d0 \uc885\uc544\ub9ac \ud1b5\uc99d\uc73c\ub85c \uc7a0\uc2dc 2\uad70\uc5d0 \ub2e4\ub140\uc654\ub2e4\uac00 \ubcf5\uadc0\ud588\ub2e4. \uadf8\ub294 \uc774\ubc88 \uc2dc\uc98c\ub124 \uac1c\uc778 \ud55c \uc2dc\uc98c \ud648\ub7f0, 2\ub8e8\ud0c0 \uae30\ub85d\uc744 \uac31\uc2e0\ud558\uba70 \uc8fc\ub825\uc5d0 \uc7a5\ud0c0\ub825\uae4c\uc9c0 \uacb8\ube44\ud55c \uc644\uc131\ud615 1\ubc88 \ud0c0\uc790\uc758 \ubaa8\uc2b5\uc744 \ubcf4\uc5ec\uc8fc\uc5c8\ub2e4. \ub450\uc0b0 \ubca0\uc5b4\uc2a4\ub294 4\uc704\ub85c \ud3ec\uc2a4\ud2b8\uc2dc\uc98c\uc5d0 \uc9c4\ucd9c\ud558\uc600\ub2e4. \uc900\ud50c\ub808\uc774\uc624\ud504\uc5d0\uc11c \ub125\uc13c \ud788\uc5b4\ub85c\uc988\uc640 \ub9de\ubd99\uc5b4 2\uc5f0\ud328 \ub4a4 3\uc5f0\uc2b9\uc744 \uac70\ub450\uba70 \ud50c\ub808\uc774\uc624\ud504\uc5d0 \uc9c4\ucd9c\ud558\uc600\uc73c\uba70, \ud50c\ub808\uc774\uc624\ud504\uc5d0\uc11c LG \ud2b8\uc708\uc2a4\uc640 \ub9de\ubd99\uc5c8\ub2e4. \uadf8\ub294 1\ucc28\uc804 1\ud68c\ucd08 \uccab \ud0c0\uc11d\uc5d0\uc11c \uc6b0\uc911\uac04 \uc678\uc57c\ub97c \uaff0\ub6ab\ub294 3\ub8e8\ud0c0\ub97c \ud130\ub728\ub838\uc73c\uba70, \ud6c4\uc18d \ud0c0\uc790\uc758 \uc801\uc2dc\ud0c0\uc5d0 \ud648\uc73c\ub85c \ub4e4\uc5b4\uc654\ub2e4. \uc774\ub0a0 \uacbd\uae30\uc5d0\uc11c 2\ub4dd\uc810\uc744 \uc62c\ub838\ub2e4. 4\ucc28\uc804\uc5d0\uc11c\ub294 \ud76c\uc0dd \ud50c\ub77c\uc774\ub85c \uacb0\uc2b9 \ud0c0\uc810\uc744 \uc62c\ub838\uc73c\uba70, \uc774 \uacbd\uae30\uc5d0\uc11c \uc2b9\ub9ac\ud558\uba70 \ud55c\uad6d\uc2dc\ub9ac\uc988\uc5d0 \uc9c4\ucd9c\ud588\ub2e4. \uc0bc\uc131 \ub77c\uc774\uc628\uc988\uc640 \ub9de\ubd99\uc740 \ud55c\uad6d\uc2dc\ub9ac\uc988\uc5d0\uc11c 2\ud560 \ub300\uc758 \ud0c0\uc728\ub85c \ubd80\uc9c4\ud588\uc73c\uba70 \uc2dc\ub9ac\uc988 \uc911 \ud558\uc704 \ud0c0\uc120\uc5d0 \ubc30\uce58\ub418\uae30\ub3c4 \ud588\ub2e4. \uc2dc\uc98c \ud6c4 FA\ub97c \uc120\uc5b8\ud558\uc600\uc73c\ub098 \uc18c\uc18d \uad6c\ub2e8\uacfc\uc758 \uc6b0\uc120 \ud611\uc0c1 \uae30\uac04 \ub3d9\uc548 \uacc4\uc57d \uc870\uac74\uc5d0 \uc774\uacac\uc744 \ubcf4\uc5ec \ud611\uc0c1\uc774 \uacb0\ub82c\ub418\uc5c8\ub2e4. 11\uc6d4 17\uc77c, 4\ub144 \ucd1d\uc561 50\uc5b5\uc6d0(\uacc4\uc57d\uae08 28\uc5b5\uc6d0, \uc5f0\ubd09 5\uc5b5\uc6d0, \uc635\uc158 2\uc5b5\uc6d0)\uc758 \uc870\uac74\uc73c\ub85c NC \ub2e4\uc774\ub178\uc2a4 \ub85c \uc774\uc801\ud558\uc600\ub2e4. 11\uc6d4 29\uc77c \uc5f4\ub9b0 ADT \ucea1\uc2a4\ud50c\ub808\uc774 2013\uc5d0\uc11c \ub300\uc0c1\uc744 \uc218\uc0c1\ud558\uc600\ub2e4.", "paragraph_answer": "\uc791\ub144 \ucee4\ub9ac\uc5b4 \ub85c\uc6b0\uc758 \uc131\uc801\uc744 \uac70\ub450\uc5c8\uace0, \uc62c \uc2dc\uc98c \ub4a4\uc5d0 \uc790\uc720 \uacc4\uc57d \uc120\uc218 \uc790\uaca9\uc744 \uc5bb\uae30 \ub54c\ubb38\uc5d0 2013\ub144 \uc2dc\uc98c\uc740 \uadf8\uc5d0\uac8c \uc788\uc5b4\uc11c \uc911\uc694\ud55c \ud574\uc600\ub2e4. \uac1c\ub9c9\uc804\ubd80\ud130 19\uacbd\uae30 \uc5f0\uc18d \ucd9c\ub8e8 \ud589\uc9c4\uc744 \uc774\uc5b4\uac14\uc73c\ub098, 4\uc6d4\ub9d0\uc5d0 \uc67c\ucabd \ud584\uc2a4\ud2b8\ub9c1 \ubd80\uc0c1\uc73c\ub85c \uc7a0\uc2dc 2\uad70\uc5d0 \ub2e4\ub140\uc624\uae30\ub3c4 \ud588\ub2e4. \uc774\ud6c4 \uc88b\uc740 \ud0c0\uaca9\uac10\uc744 \uc720\uc9c0\ud558\uba70 6\uc6d4 .388\ub85c \uc6d4\uac04 \ud0c0\uc728 \ub9ac\uadf8 1\uc704\ub97c \uae30\ub85d\ud588\uc73c\uba70, 6\uc6d4 \ud300\ub0b4 \ud0c0\uc790 \ubd80\ubb38 MVP\ub85c\ub3c4 \uc120\uc815\ub418\uc5c8\ub2e4. \uc62c\uc2a4\ud0c0\uc804\uc5d0\ub294 \uac10\ub3c5 \ucd94\ucc9c \uc120\uc218\ub85c \uc120\ubc1c\ub418\uc5c8\ub2e4. 8\uc6d4 \ud558\uc21c\uc5d0 \uc885\uc544\ub9ac \ud1b5\uc99d\uc73c\ub85c \uc7a0\uc2dc 2\uad70\uc5d0 \ub2e4\ub140\uc654\ub2e4\uac00 \ubcf5\uadc0\ud588\ub2e4. \uadf8\ub294 \uc774\ubc88 \uc2dc\uc98c\ub124 \uac1c\uc778 \ud55c \uc2dc\uc98c \ud648\ub7f0, 2\ub8e8\ud0c0 \uae30\ub85d\uc744 \uac31\uc2e0\ud558\uba70 \uc8fc\ub825\uc5d0 \uc7a5\ud0c0\ub825\uae4c\uc9c0 \uacb8\ube44\ud55c \uc644\uc131\ud615 1\ubc88 \ud0c0\uc790\uc758 \ubaa8\uc2b5\uc744 \ubcf4\uc5ec\uc8fc\uc5c8\ub2e4. \ub450\uc0b0 \ubca0\uc5b4\uc2a4\ub294 4\uc704\ub85c \ud3ec\uc2a4\ud2b8\uc2dc\uc98c\uc5d0 \uc9c4\ucd9c\ud558\uc600\ub2e4. \uc900\ud50c\ub808\uc774\uc624\ud504\uc5d0\uc11c \ub125\uc13c \ud788\uc5b4\ub85c\uc988\uc640 \ub9de\ubd99\uc5b4 2\uc5f0\ud328 \ub4a4 3\uc5f0\uc2b9\uc744 \uac70\ub450\uba70 \ud50c\ub808\uc774\uc624\ud504\uc5d0 \uc9c4\ucd9c\ud558\uc600\uc73c\uba70, \ud50c\ub808\uc774\uc624\ud504\uc5d0\uc11c LG \ud2b8\uc708\uc2a4\uc640 \ub9de\ubd99\uc5c8\ub2e4. \uadf8\ub294 1\ucc28\uc804 1\ud68c\ucd08 \uccab \ud0c0\uc11d\uc5d0\uc11c \uc6b0\uc911\uac04 \uc678\uc57c\ub97c \uaff0\ub6ab\ub294 3\ub8e8\ud0c0\ub97c \ud130\ub728\ub838\uc73c\uba70, \ud6c4\uc18d \ud0c0\uc790\uc758 \uc801\uc2dc\ud0c0\uc5d0 \ud648\uc73c\ub85c \ub4e4\uc5b4\uc654\ub2e4. \uc774\ub0a0 \uacbd\uae30\uc5d0\uc11c 2\ub4dd\uc810\uc744 \uc62c\ub838\ub2e4. 4\ucc28\uc804\uc5d0\uc11c\ub294 \ud76c\uc0dd \ud50c\ub77c\uc774\ub85c \uacb0\uc2b9 \ud0c0\uc810\uc744 \uc62c\ub838\uc73c\uba70, \uc774 \uacbd\uae30\uc5d0\uc11c \uc2b9\ub9ac\ud558\uba70 \ud55c\uad6d\uc2dc\ub9ac\uc988\uc5d0 \uc9c4\ucd9c\ud588\ub2e4. \uc0bc\uc131 \ub77c\uc774\uc628\uc988\uc640 \ub9de\ubd99\uc740 \ud55c\uad6d\uc2dc\ub9ac\uc988\uc5d0\uc11c 2\ud560 \ub300\uc758 \ud0c0\uc728\ub85c \ubd80\uc9c4\ud588\uc73c\uba70 \uc2dc\ub9ac\uc988 \uc911 \ud558\uc704 \ud0c0\uc120\uc5d0 \ubc30\uce58\ub418\uae30\ub3c4 \ud588\ub2e4. \uc2dc\uc98c \ud6c4 FA\ub97c \uc120\uc5b8\ud558\uc600\uc73c\ub098 \uc18c\uc18d \uad6c\ub2e8\uacfc\uc758 \uc6b0\uc120 \ud611\uc0c1 \uae30\uac04 \ub3d9\uc548 \uacc4\uc57d \uc870\uac74\uc5d0 \uc774\uacac\uc744 \ubcf4\uc5ec \ud611\uc0c1\uc774 \uacb0\ub82c\ub418\uc5c8\ub2e4. 11\uc6d4 17\uc77c, 4\ub144 \ucd1d\uc561 50\uc5b5\uc6d0(\uacc4\uc57d\uae08 28\uc5b5\uc6d0, \uc5f0\ubd09 5\uc5b5\uc6d0, \uc635\uc158 2\uc5b5\uc6d0)\uc758 \uc870\uac74\uc73c\ub85c NC \ub2e4\uc774\ub178\uc2a4 \ub85c \uc774\uc801\ud558\uc600\ub2e4. 11\uc6d4 29\uc77c \uc5f4\ub9b0 ADT \ucea1\uc2a4\ud50c\ub808\uc774 2013\uc5d0\uc11c \ub300\uc0c1\uc744 \uc218\uc0c1\ud558\uc600\ub2e4.", "sentence_answer": "11\uc6d4 17\uc77c, 4\ub144 \ucd1d\uc561 50\uc5b5\uc6d0(\uacc4\uc57d\uae08 28\uc5b5\uc6d0, \uc5f0\ubd09 5\uc5b5\uc6d0, \uc635\uc158 2\uc5b5\uc6d0)\uc758 \uc870\uac74\uc73c\ub85c NC \ub2e4\uc774\ub178\uc2a4 \ub85c \uc774\uc801\ud558\uc600\ub2e4.", "paragraph_id": "6547838-8-2", "paragraph_question": "question: \uc774\uc885\uc6b1\uc774 FA \uacc4\uc57d\uc73c\ub85c \uc62e\uae34 \ud300\uc740?, context: \uc791\ub144 \ucee4\ub9ac\uc5b4 \ub85c\uc6b0\uc758 \uc131\uc801\uc744 \uac70\ub450\uc5c8\uace0, \uc62c \uc2dc\uc98c \ub4a4\uc5d0 \uc790\uc720 \uacc4\uc57d \uc120\uc218 \uc790\uaca9\uc744 \uc5bb\uae30 \ub54c\ubb38\uc5d0 2013\ub144 \uc2dc\uc98c\uc740 \uadf8\uc5d0\uac8c \uc788\uc5b4\uc11c \uc911\uc694\ud55c \ud574\uc600\ub2e4. \uac1c\ub9c9\uc804\ubd80\ud130 19\uacbd\uae30 \uc5f0\uc18d \ucd9c\ub8e8 \ud589\uc9c4\uc744 \uc774\uc5b4\uac14\uc73c\ub098, 4\uc6d4\ub9d0\uc5d0 \uc67c\ucabd \ud584\uc2a4\ud2b8\ub9c1 \ubd80\uc0c1\uc73c\ub85c \uc7a0\uc2dc 2\uad70\uc5d0 \ub2e4\ub140\uc624\uae30\ub3c4 \ud588\ub2e4. \uc774\ud6c4 \uc88b\uc740 \ud0c0\uaca9\uac10\uc744 \uc720\uc9c0\ud558\uba70 6\uc6d4 .388\ub85c \uc6d4\uac04 \ud0c0\uc728 \ub9ac\uadf8 1\uc704\ub97c \uae30\ub85d\ud588\uc73c\uba70, 6\uc6d4 \ud300\ub0b4 \ud0c0\uc790 \ubd80\ubb38 MVP\ub85c\ub3c4 \uc120\uc815\ub418\uc5c8\ub2e4. \uc62c\uc2a4\ud0c0\uc804\uc5d0\ub294 \uac10\ub3c5 \ucd94\ucc9c \uc120\uc218\ub85c \uc120\ubc1c\ub418\uc5c8\ub2e4. 8\uc6d4 \ud558\uc21c\uc5d0 \uc885\uc544\ub9ac \ud1b5\uc99d\uc73c\ub85c \uc7a0\uc2dc 2\uad70\uc5d0 \ub2e4\ub140\uc654\ub2e4\uac00 \ubcf5\uadc0\ud588\ub2e4. \uadf8\ub294 \uc774\ubc88 \uc2dc\uc98c\ub124 \uac1c\uc778 \ud55c \uc2dc\uc98c \ud648\ub7f0, 2\ub8e8\ud0c0 \uae30\ub85d\uc744 \uac31\uc2e0\ud558\uba70 \uc8fc\ub825\uc5d0 \uc7a5\ud0c0\ub825\uae4c\uc9c0 \uacb8\ube44\ud55c \uc644\uc131\ud615 1\ubc88 \ud0c0\uc790\uc758 \ubaa8\uc2b5\uc744 \ubcf4\uc5ec\uc8fc\uc5c8\ub2e4. \ub450\uc0b0 \ubca0\uc5b4\uc2a4\ub294 4\uc704\ub85c \ud3ec\uc2a4\ud2b8\uc2dc\uc98c\uc5d0 \uc9c4\ucd9c\ud558\uc600\ub2e4. \uc900\ud50c\ub808\uc774\uc624\ud504\uc5d0\uc11c \ub125\uc13c \ud788\uc5b4\ub85c\uc988\uc640 \ub9de\ubd99\uc5b4 2\uc5f0\ud328 \ub4a4 3\uc5f0\uc2b9\uc744 \uac70\ub450\uba70 \ud50c\ub808\uc774\uc624\ud504\uc5d0 \uc9c4\ucd9c\ud558\uc600\uc73c\uba70, \ud50c\ub808\uc774\uc624\ud504\uc5d0\uc11c LG \ud2b8\uc708\uc2a4\uc640 \ub9de\ubd99\uc5c8\ub2e4. \uadf8\ub294 1\ucc28\uc804 1\ud68c\ucd08 \uccab \ud0c0\uc11d\uc5d0\uc11c \uc6b0\uc911\uac04 \uc678\uc57c\ub97c \uaff0\ub6ab\ub294 3\ub8e8\ud0c0\ub97c \ud130\ub728\ub838\uc73c\uba70, \ud6c4\uc18d \ud0c0\uc790\uc758 \uc801\uc2dc\ud0c0\uc5d0 \ud648\uc73c\ub85c \ub4e4\uc5b4\uc654\ub2e4. \uc774\ub0a0 \uacbd\uae30\uc5d0\uc11c 2\ub4dd\uc810\uc744 \uc62c\ub838\ub2e4. 4\ucc28\uc804\uc5d0\uc11c\ub294 \ud76c\uc0dd \ud50c\ub77c\uc774\ub85c \uacb0\uc2b9 \ud0c0\uc810\uc744 \uc62c\ub838\uc73c\uba70, \uc774 \uacbd\uae30\uc5d0\uc11c \uc2b9\ub9ac\ud558\uba70 \ud55c\uad6d\uc2dc\ub9ac\uc988\uc5d0 \uc9c4\ucd9c\ud588\ub2e4. \uc0bc\uc131 \ub77c\uc774\uc628\uc988\uc640 \ub9de\ubd99\uc740 \ud55c\uad6d\uc2dc\ub9ac\uc988\uc5d0\uc11c 2\ud560 \ub300\uc758 \ud0c0\uc728\ub85c \ubd80\uc9c4\ud588\uc73c\uba70 \uc2dc\ub9ac\uc988 \uc911 \ud558\uc704 \ud0c0\uc120\uc5d0 \ubc30\uce58\ub418\uae30\ub3c4 \ud588\ub2e4. \uc2dc\uc98c \ud6c4 FA\ub97c \uc120\uc5b8\ud558\uc600\uc73c\ub098 \uc18c\uc18d \uad6c\ub2e8\uacfc\uc758 \uc6b0\uc120 \ud611\uc0c1 \uae30\uac04 \ub3d9\uc548 \uacc4\uc57d \uc870\uac74\uc5d0 \uc774\uacac\uc744 \ubcf4\uc5ec \ud611\uc0c1\uc774 \uacb0\ub82c\ub418\uc5c8\ub2e4. 11\uc6d4 17\uc77c, 4\ub144 \ucd1d\uc561 50\uc5b5\uc6d0(\uacc4\uc57d\uae08 28\uc5b5\uc6d0, \uc5f0\ubd09 5\uc5b5\uc6d0, \uc635\uc158 2\uc5b5\uc6d0)\uc758 \uc870\uac74\uc73c\ub85c NC \ub2e4\uc774\ub178\uc2a4\ub85c \uc774\uc801\ud558\uc600\ub2e4. 11\uc6d4 29\uc77c \uc5f4\ub9b0 ADT \ucea1\uc2a4\ud50c\ub808\uc774 2013\uc5d0\uc11c \ub300\uc0c1\uc744 \uc218\uc0c1\ud558\uc600\ub2e4."} {"question": "\uc624\ubc14\ub9c8\uac00 \uc625\uc2dc\ub374\ud0c8 \ub300\ud559\uad50\uc5d0 \uc785\ud559\ud55c \ud574\ub294?", "paragraph": "\uace0\ub4f1\ud559\uad50\ub97c \ub9c8\uce5c \ub4a4 \uc624\ubc14\ub9c8\ub294 1979\ub144 \ub85c\uc2a4\uc564\uc824\ub808\uc2a4\uc758 \uc625\uc2dc\ub374\ud0c8 \ub300\ud559\uad50\uc5d0 \uc785\ud559\ud558\uc600\ub2e4. \uc625\uc2dc\ub374\ud0c8\uc5d0\uc11c \uc624\ubc14\ub9c8\ub294 \uc5ec\uc804\ud788 \ud751\uc778\uc73c\ub85c\uc11c\uc758 \uc790\uc2e0\uc758 \uc815\uccb4\uc131\uc744 \uace0\ubbfc\ud558\uc600\uc73c\uba70, Earl Chew \ub4f1\uacfc \ud568\uaed8 \ud751\uc778 \ud559\uc0dd\ud68c \ud65c\ub3d9\uc744 \ud558\uc600\ub2e4. 2\ub144 \ub4a4\uc778 1981\ub144\uc5d0\ub294 \ub274\uc695 \uc2dc\uc758 \uceec\ub7fc\ube44\uc544 \ub300\ud559\uad50\uc5d0 \ud3b8\uc785\ud558\uc5ec \uad6d\uc81c \uad00\uacc4\ud559\uc744 \uc8fc\uc804\uacf5(specialty)\uc73c\ub85c \uc815\uce58\ud559\uc744 \uc804\uacf5\ud558\uace0, 1983\ub144\uc5d0 \ud559\uc0ac \ud559\uc704\ub97c \ub544\ub2e4. \uc774 \uc2dc\uae30\uc5d0 \uadf8\ub294 \uc7a0\uc2dc \ub274\uc695\uc2dc 339 East 94\ubc88\uac00\uc5d0 \uc788\ub294 \ud5c8\ub984\ud55c \uc544\ud30c\ud2b8\uc5d0\uc11c \ud30c\ud0a4\uc2a4\ud0c4 \ucd9c\uc2e0\uc758 \ub0a8\uc131 \ubd88\ubc95\uccb4\ub958\uc790\uc778 Sohale Siddiqi\uc640 \uac19\uc774 \uc0b4\uc558\ub2e4. \uc7a0\uc2dc \uadf8\uc640 \ud568\uaed8 \uc5ec\uc790\ub4e4\uacfc \uc5b4\uc6b8\ub9ac\uace0 \ub9c8\ub9ac\ud654\ub098\ub97c \ud53c\ub294 \ub4f1 \ubc29\ud0d5\ud55c \uc0dd\ud65c \ud558\uc600\uc73c\ub098 \uace7 \ud559\uc5c5\uc5d0 \uc5f4\uc911\ud558\uc600\uace0 \uadf8\uc640 \ud5e4\uc5b4\uc84c\ub2e4. \uadf8\ub294 1\ub144 \ub3d9\uc548 \uae30\uc5c5 \uad6d\uc81c \ud68c\uc0ac(Business International Corporation) \uc640 \uadf8 \ub2e4\uc74c\uc73c\ub85c \ub274\uc695 \uacf5\uacf5 \uc774\uc775 \uc870\uc0ac\ub2e8(New York Public Interest Research Group) \uc5d0\uc11c \uc77c\ud558\uc600\ub2e4.", "answer": "1979\ub144", "sentence": "\uace0\ub4f1\ud559\uad50\ub97c \ub9c8\uce5c \ub4a4 \uc624\ubc14\ub9c8\ub294 1979\ub144 \ub85c\uc2a4\uc564\uc824\ub808\uc2a4\uc758 \uc625\uc2dc\ub374\ud0c8 \ub300\ud559\uad50\uc5d0 \uc785\ud559\ud558\uc600\ub2e4.", "paragraph_sentence": " \uace0\ub4f1\ud559\uad50\ub97c \ub9c8\uce5c \ub4a4 \uc624\ubc14\ub9c8\ub294 1979\ub144 \ub85c\uc2a4\uc564\uc824\ub808\uc2a4\uc758 \uc625\uc2dc\ub374\ud0c8 \ub300\ud559\uad50\uc5d0 \uc785\ud559\ud558\uc600\ub2e4. \uc625\uc2dc\ub374\ud0c8\uc5d0\uc11c \uc624\ubc14\ub9c8\ub294 \uc5ec\uc804\ud788 \ud751\uc778\uc73c\ub85c\uc11c\uc758 \uc790\uc2e0\uc758 \uc815\uccb4\uc131\uc744 \uace0\ubbfc\ud558\uc600\uc73c\uba70, Earl Chew \ub4f1\uacfc \ud568\uaed8 \ud751\uc778 \ud559\uc0dd\ud68c \ud65c\ub3d9\uc744 \ud558\uc600\ub2e4. 2\ub144 \ub4a4\uc778 1981\ub144\uc5d0\ub294 \ub274\uc695 \uc2dc\uc758 \uceec\ub7fc\ube44\uc544 \ub300\ud559\uad50\uc5d0 \ud3b8\uc785\ud558\uc5ec \uad6d\uc81c \uad00\uacc4\ud559\uc744 \uc8fc\uc804\uacf5(specialty)\uc73c\ub85c \uc815\uce58\ud559\uc744 \uc804\uacf5\ud558\uace0, 1983\ub144\uc5d0 \ud559\uc0ac \ud559\uc704\ub97c \ub544\ub2e4. \uc774 \uc2dc\uae30\uc5d0 \uadf8\ub294 \uc7a0\uc2dc \ub274\uc695\uc2dc 339 East 94\ubc88\uac00\uc5d0 \uc788\ub294 \ud5c8\ub984\ud55c \uc544\ud30c\ud2b8\uc5d0\uc11c \ud30c\ud0a4\uc2a4\ud0c4 \ucd9c\uc2e0\uc758 \ub0a8\uc131 \ubd88\ubc95\uccb4\ub958\uc790\uc778 Sohale Siddiqi\uc640 \uac19\uc774 \uc0b4\uc558\ub2e4. \uc7a0\uc2dc \uadf8\uc640 \ud568\uaed8 \uc5ec\uc790\ub4e4\uacfc \uc5b4\uc6b8\ub9ac\uace0 \ub9c8\ub9ac\ud654\ub098\ub97c \ud53c\ub294 \ub4f1 \ubc29\ud0d5\ud55c \uc0dd\ud65c \ud558\uc600\uc73c\ub098 \uace7 \ud559\uc5c5\uc5d0 \uc5f4\uc911\ud558\uc600\uace0 \uadf8\uc640 \ud5e4\uc5b4\uc84c\ub2e4. \uadf8\ub294 1\ub144 \ub3d9\uc548 \uae30\uc5c5 \uad6d\uc81c \ud68c\uc0ac(Business International Corporation) \uc640 \uadf8 \ub2e4\uc74c\uc73c\ub85c \ub274\uc695 \uacf5\uacf5 \uc774\uc775 \uc870\uc0ac\ub2e8(New York Public Interest Research Group) \uc5d0\uc11c \uc77c\ud558\uc600\ub2e4.", "paragraph_answer": "\uace0\ub4f1\ud559\uad50\ub97c \ub9c8\uce5c \ub4a4 \uc624\ubc14\ub9c8\ub294 1979\ub144 \ub85c\uc2a4\uc564\uc824\ub808\uc2a4\uc758 \uc625\uc2dc\ub374\ud0c8 \ub300\ud559\uad50\uc5d0 \uc785\ud559\ud558\uc600\ub2e4. \uc625\uc2dc\ub374\ud0c8\uc5d0\uc11c \uc624\ubc14\ub9c8\ub294 \uc5ec\uc804\ud788 \ud751\uc778\uc73c\ub85c\uc11c\uc758 \uc790\uc2e0\uc758 \uc815\uccb4\uc131\uc744 \uace0\ubbfc\ud558\uc600\uc73c\uba70, Earl Chew \ub4f1\uacfc \ud568\uaed8 \ud751\uc778 \ud559\uc0dd\ud68c \ud65c\ub3d9\uc744 \ud558\uc600\ub2e4. 2\ub144 \ub4a4\uc778 1981\ub144\uc5d0\ub294 \ub274\uc695 \uc2dc\uc758 \uceec\ub7fc\ube44\uc544 \ub300\ud559\uad50\uc5d0 \ud3b8\uc785\ud558\uc5ec \uad6d\uc81c \uad00\uacc4\ud559\uc744 \uc8fc\uc804\uacf5(specialty)\uc73c\ub85c \uc815\uce58\ud559\uc744 \uc804\uacf5\ud558\uace0, 1983\ub144\uc5d0 \ud559\uc0ac \ud559\uc704\ub97c \ub544\ub2e4. \uc774 \uc2dc\uae30\uc5d0 \uadf8\ub294 \uc7a0\uc2dc \ub274\uc695\uc2dc 339 East 94\ubc88\uac00\uc5d0 \uc788\ub294 \ud5c8\ub984\ud55c \uc544\ud30c\ud2b8\uc5d0\uc11c \ud30c\ud0a4\uc2a4\ud0c4 \ucd9c\uc2e0\uc758 \ub0a8\uc131 \ubd88\ubc95\uccb4\ub958\uc790\uc778 Sohale Siddiqi\uc640 \uac19\uc774 \uc0b4\uc558\ub2e4. \uc7a0\uc2dc \uadf8\uc640 \ud568\uaed8 \uc5ec\uc790\ub4e4\uacfc \uc5b4\uc6b8\ub9ac\uace0 \ub9c8\ub9ac\ud654\ub098\ub97c \ud53c\ub294 \ub4f1 \ubc29\ud0d5\ud55c \uc0dd\ud65c \ud558\uc600\uc73c\ub098 \uace7 \ud559\uc5c5\uc5d0 \uc5f4\uc911\ud558\uc600\uace0 \uadf8\uc640 \ud5e4\uc5b4\uc84c\ub2e4. \uadf8\ub294 1\ub144 \ub3d9\uc548 \uae30\uc5c5 \uad6d\uc81c \ud68c\uc0ac(Business International Corporation) \uc640 \uadf8 \ub2e4\uc74c\uc73c\ub85c \ub274\uc695 \uacf5\uacf5 \uc774\uc775 \uc870\uc0ac\ub2e8(New York Public Interest Research Group) \uc5d0\uc11c \uc77c\ud558\uc600\ub2e4.", "sentence_answer": "\uace0\ub4f1\ud559\uad50\ub97c \ub9c8\uce5c \ub4a4 \uc624\ubc14\ub9c8\ub294 1979\ub144 \ub85c\uc2a4\uc564\uc824\ub808\uc2a4\uc758 \uc625\uc2dc\ub374\ud0c8 \ub300\ud559\uad50\uc5d0 \uc785\ud559\ud558\uc600\ub2e4.", "paragraph_id": "6108839-2-0", "paragraph_question": "question: \uc624\ubc14\ub9c8\uac00 \uc625\uc2dc\ub374\ud0c8 \ub300\ud559\uad50\uc5d0 \uc785\ud559\ud55c \ud574\ub294?, context: \uace0\ub4f1\ud559\uad50\ub97c \ub9c8\uce5c \ub4a4 \uc624\ubc14\ub9c8\ub294 1979\ub144 \ub85c\uc2a4\uc564\uc824\ub808\uc2a4\uc758 \uc625\uc2dc\ub374\ud0c8 \ub300\ud559\uad50\uc5d0 \uc785\ud559\ud558\uc600\ub2e4. \uc625\uc2dc\ub374\ud0c8\uc5d0\uc11c \uc624\ubc14\ub9c8\ub294 \uc5ec\uc804\ud788 \ud751\uc778\uc73c\ub85c\uc11c\uc758 \uc790\uc2e0\uc758 \uc815\uccb4\uc131\uc744 \uace0\ubbfc\ud558\uc600\uc73c\uba70, Earl Chew \ub4f1\uacfc \ud568\uaed8 \ud751\uc778 \ud559\uc0dd\ud68c \ud65c\ub3d9\uc744 \ud558\uc600\ub2e4. 2\ub144 \ub4a4\uc778 1981\ub144\uc5d0\ub294 \ub274\uc695 \uc2dc\uc758 \uceec\ub7fc\ube44\uc544 \ub300\ud559\uad50\uc5d0 \ud3b8\uc785\ud558\uc5ec \uad6d\uc81c \uad00\uacc4\ud559\uc744 \uc8fc\uc804\uacf5(specialty)\uc73c\ub85c \uc815\uce58\ud559\uc744 \uc804\uacf5\ud558\uace0, 1983\ub144\uc5d0 \ud559\uc0ac \ud559\uc704\ub97c \ub544\ub2e4. \uc774 \uc2dc\uae30\uc5d0 \uadf8\ub294 \uc7a0\uc2dc \ub274\uc695\uc2dc 339 East 94\ubc88\uac00\uc5d0 \uc788\ub294 \ud5c8\ub984\ud55c \uc544\ud30c\ud2b8\uc5d0\uc11c \ud30c\ud0a4\uc2a4\ud0c4 \ucd9c\uc2e0\uc758 \ub0a8\uc131 \ubd88\ubc95\uccb4\ub958\uc790\uc778 Sohale Siddiqi\uc640 \uac19\uc774 \uc0b4\uc558\ub2e4. \uc7a0\uc2dc \uadf8\uc640 \ud568\uaed8 \uc5ec\uc790\ub4e4\uacfc \uc5b4\uc6b8\ub9ac\uace0 \ub9c8\ub9ac\ud654\ub098\ub97c \ud53c\ub294 \ub4f1 \ubc29\ud0d5\ud55c \uc0dd\ud65c \ud558\uc600\uc73c\ub098 \uace7 \ud559\uc5c5\uc5d0 \uc5f4\uc911\ud558\uc600\uace0 \uadf8\uc640 \ud5e4\uc5b4\uc84c\ub2e4. \uadf8\ub294 1\ub144 \ub3d9\uc548 \uae30\uc5c5 \uad6d\uc81c \ud68c\uc0ac(Business International Corporation) \uc640 \uadf8 \ub2e4\uc74c\uc73c\ub85c \ub274\uc695 \uacf5\uacf5 \uc774\uc775 \uc870\uc0ac\ub2e8(New York Public Interest Research Group) \uc5d0\uc11c \uc77c\ud558\uc600\ub2e4."} {"question": "\ub77c\uc988\ubca0\ub9ac \ud30c\uc774 3 \ubaa8\ub378B\uc758 \uac00\uaca9\uc740?", "paragraph": "2016\ub144 2\uc6d4 29\uc77c \ub77c\uc988\ubca0\ub9ac \ud30c\uc774\uc758 \uc0c8\ub85c\uc6b4 \ubaa8\ub378 '\ub77c\uc988\ubca0\ub9ac \ud30c\uc774 3 \ubaa8\ub378 B(Raspberry Pi 3 Model B)'\uc758 \ucd9c\uc2dc\ub97c \ubc1c\ud45c\ud588\ub2e4. \uc138 \ubc88\uc9f8 \ubc84\uc804\uc778 \ub77c\uc988\ubca0\ub9ac \ud30c\uc774 3\ub294 \ubb34\uc120\ub79c \ubc0f \ube14\ub8e8\ud22c\uc2a4 \uae30\ub2a5\uc774 \uae30\ubcf8 \ub0b4\uc7a5\ub418\uc5b4 \uc788\uc73c\uba70, \uc774\uc804 \ubaa8\ub378\uc778 \ub77c\uc988\ubca0\ub9ac \ud30c\uc774 2\uc758 900MHz\ubcf4\ub2e4 \uac1c\uc120\ub41c 1.2GHz\uc758 \uc18d\ub3c4\ub97c \uad6c\ud604\ud55c \ube0c\ub85c\ub4dc\ucef4 BCM2837 64bit ARMv8 \ud504\ub85c\uc138\uc11c\ub97c \uae30\ubc18\uc73c\ub85c \ud55c\ub2e4. \ub610\ud55c, \uc804\ub825\uad00\ub9ac \uae30\ub2a5\ub3c4 \uac1c\uc120\ub418\uc5c8\ub2e4. \ucd5c\ub300 2.5 Amps\uc758 \uc804\ub825\ub9cc\uc744 \uc0ac\uc6a9\ud558\ub294 \ub3d9\uc2dc\uc5d0 \uac15\ub825\ud55c \uc678\ubd80 USB\uae30\uae30\ub97c \ub354\uc6b1 \ub9ce\uc774 \uc9c0\uc6d0\ud560 \uc218 \uc788\ub2e4. \uc800\uc804\ub825 \ube14\ub8e8\ud22c\uc2a4(Bluetooth Low Energy)\uc640 \ubb34\uc120 LAN\uc744 \uc9c0\uc6d0\ud558\uac8c \ub41c \ub77c\uc988\ubca0\ub9ac\ud30c\uc774 3\ub294 \uad6c\uc785 \uc989\uc2dc IoT, \ube14\ub8e8\ud22c\uc2a4 \ud5e4\ub4dc\ud3f0 \ub610\ub294 \uc2a4\ud53c\ucee4, \uc640\uc774\ud30c\uc774 \uac8c\uc774\ud2b8\uc6e8\uc774,\ud648 \ud074\ub77c\uc6b0\ub4dc \uc2a4\ud1a0\ub9ac\uc9c0 \ub4f1\uc5d0 \uc0ac\uc6a9\ud560 \uc218 \uc788\ub2e4. \uc81c\ud488\uc740 35\ub2ec\ub7ec\uc5d0 \uad6c\ub9e4 \uac00\ub2a5\ud558\uba70 \uc774\uc804 \ubc84\uc804\uacfc \ud638\ud658\ub41c\ub2e4.", "answer": "35\ub2ec\ub7ec", "sentence": "\uc81c\ud488\uc740 35\ub2ec\ub7ec \uc5d0 \uad6c\ub9e4 \uac00\ub2a5\ud558\uba70 \uc774\uc804 \ubc84\uc804\uacfc \ud638\ud658\ub41c\ub2e4.", "paragraph_sentence": "2016\ub144 2\uc6d4 29\uc77c \ub77c\uc988\ubca0\ub9ac \ud30c\uc774\uc758 \uc0c8\ub85c\uc6b4 \ubaa8\ub378 '\ub77c\uc988\ubca0\ub9ac \ud30c\uc774 3 \ubaa8\ub378 B(Raspberry Pi 3 Model B)'\uc758 \ucd9c\uc2dc\ub97c \ubc1c\ud45c\ud588\ub2e4. \uc138 \ubc88\uc9f8 \ubc84\uc804\uc778 \ub77c\uc988\ubca0\ub9ac \ud30c\uc774 3\ub294 \ubb34\uc120\ub79c \ubc0f \ube14\ub8e8\ud22c\uc2a4 \uae30\ub2a5\uc774 \uae30\ubcf8 \ub0b4\uc7a5\ub418\uc5b4 \uc788\uc73c\uba70, \uc774\uc804 \ubaa8\ub378\uc778 \ub77c\uc988\ubca0\ub9ac \ud30c\uc774 2\uc758 900MHz\ubcf4\ub2e4 \uac1c\uc120\ub41c 1.2GHz\uc758 \uc18d\ub3c4\ub97c \uad6c\ud604\ud55c \ube0c\ub85c\ub4dc\ucef4 BCM2837 64bit ARMv8 \ud504\ub85c\uc138\uc11c\ub97c \uae30\ubc18\uc73c\ub85c \ud55c\ub2e4. \ub610\ud55c, \uc804\ub825\uad00\ub9ac \uae30\ub2a5\ub3c4 \uac1c\uc120\ub418\uc5c8\ub2e4. \ucd5c\ub300 2.5 Amps\uc758 \uc804\ub825\ub9cc\uc744 \uc0ac\uc6a9\ud558\ub294 \ub3d9\uc2dc\uc5d0 \uac15\ub825\ud55c \uc678\ubd80 USB\uae30\uae30\ub97c \ub354\uc6b1 \ub9ce\uc774 \uc9c0\uc6d0\ud560 \uc218 \uc788\ub2e4. \uc800\uc804\ub825 \ube14\ub8e8\ud22c\uc2a4(Bluetooth Low Energy)\uc640 \ubb34\uc120 LAN\uc744 \uc9c0\uc6d0\ud558\uac8c \ub41c \ub77c\uc988\ubca0\ub9ac\ud30c\uc774 3\ub294 \uad6c\uc785 \uc989\uc2dc IoT, \ube14\ub8e8\ud22c\uc2a4 \ud5e4\ub4dc\ud3f0 \ub610\ub294 \uc2a4\ud53c\ucee4, \uc640\uc774\ud30c\uc774 \uac8c\uc774\ud2b8\uc6e8\uc774,\ud648 \ud074\ub77c\uc6b0\ub4dc \uc2a4\ud1a0\ub9ac\uc9c0 \ub4f1\uc5d0 \uc0ac\uc6a9\ud560 \uc218 \uc788\ub2e4. \uc81c\ud488\uc740 35\ub2ec\ub7ec \uc5d0 \uad6c\ub9e4 \uac00\ub2a5\ud558\uba70 \uc774\uc804 \ubc84\uc804\uacfc \ud638\ud658\ub41c\ub2e4. ", "paragraph_answer": "2016\ub144 2\uc6d4 29\uc77c \ub77c\uc988\ubca0\ub9ac \ud30c\uc774\uc758 \uc0c8\ub85c\uc6b4 \ubaa8\ub378 '\ub77c\uc988\ubca0\ub9ac \ud30c\uc774 3 \ubaa8\ub378 B(Raspberry Pi 3 Model B)'\uc758 \ucd9c\uc2dc\ub97c \ubc1c\ud45c\ud588\ub2e4. \uc138 \ubc88\uc9f8 \ubc84\uc804\uc778 \ub77c\uc988\ubca0\ub9ac \ud30c\uc774 3\ub294 \ubb34\uc120\ub79c \ubc0f \ube14\ub8e8\ud22c\uc2a4 \uae30\ub2a5\uc774 \uae30\ubcf8 \ub0b4\uc7a5\ub418\uc5b4 \uc788\uc73c\uba70, \uc774\uc804 \ubaa8\ub378\uc778 \ub77c\uc988\ubca0\ub9ac \ud30c\uc774 2\uc758 900MHz\ubcf4\ub2e4 \uac1c\uc120\ub41c 1.2GHz\uc758 \uc18d\ub3c4\ub97c \uad6c\ud604\ud55c \ube0c\ub85c\ub4dc\ucef4 BCM2837 64bit ARMv8 \ud504\ub85c\uc138\uc11c\ub97c \uae30\ubc18\uc73c\ub85c \ud55c\ub2e4. \ub610\ud55c, \uc804\ub825\uad00\ub9ac \uae30\ub2a5\ub3c4 \uac1c\uc120\ub418\uc5c8\ub2e4. \ucd5c\ub300 2.5 Amps\uc758 \uc804\ub825\ub9cc\uc744 \uc0ac\uc6a9\ud558\ub294 \ub3d9\uc2dc\uc5d0 \uac15\ub825\ud55c \uc678\ubd80 USB\uae30\uae30\ub97c \ub354\uc6b1 \ub9ce\uc774 \uc9c0\uc6d0\ud560 \uc218 \uc788\ub2e4. \uc800\uc804\ub825 \ube14\ub8e8\ud22c\uc2a4(Bluetooth Low Energy)\uc640 \ubb34\uc120 LAN\uc744 \uc9c0\uc6d0\ud558\uac8c \ub41c \ub77c\uc988\ubca0\ub9ac\ud30c\uc774 3\ub294 \uad6c\uc785 \uc989\uc2dc IoT, \ube14\ub8e8\ud22c\uc2a4 \ud5e4\ub4dc\ud3f0 \ub610\ub294 \uc2a4\ud53c\ucee4, \uc640\uc774\ud30c\uc774 \uac8c\uc774\ud2b8\uc6e8\uc774,\ud648 \ud074\ub77c\uc6b0\ub4dc \uc2a4\ud1a0\ub9ac\uc9c0 \ub4f1\uc5d0 \uc0ac\uc6a9\ud560 \uc218 \uc788\ub2e4. \uc81c\ud488\uc740 35\ub2ec\ub7ec \uc5d0 \uad6c\ub9e4 \uac00\ub2a5\ud558\uba70 \uc774\uc804 \ubc84\uc804\uacfc \ud638\ud658\ub41c\ub2e4.", "sentence_answer": "\uc81c\ud488\uc740 35\ub2ec\ub7ec \uc5d0 \uad6c\ub9e4 \uac00\ub2a5\ud558\uba70 \uc774\uc804 \ubc84\uc804\uacfc \ud638\ud658\ub41c\ub2e4.", "paragraph_id": "6494420-2-2", "paragraph_question": "question: \ub77c\uc988\ubca0\ub9ac \ud30c\uc774 3 \ubaa8\ub378B\uc758 \uac00\uaca9\uc740?, context: 2016\ub144 2\uc6d4 29\uc77c \ub77c\uc988\ubca0\ub9ac \ud30c\uc774\uc758 \uc0c8\ub85c\uc6b4 \ubaa8\ub378 '\ub77c\uc988\ubca0\ub9ac \ud30c\uc774 3 \ubaa8\ub378 B(Raspberry Pi 3 Model B)'\uc758 \ucd9c\uc2dc\ub97c \ubc1c\ud45c\ud588\ub2e4. \uc138 \ubc88\uc9f8 \ubc84\uc804\uc778 \ub77c\uc988\ubca0\ub9ac \ud30c\uc774 3\ub294 \ubb34\uc120\ub79c \ubc0f \ube14\ub8e8\ud22c\uc2a4 \uae30\ub2a5\uc774 \uae30\ubcf8 \ub0b4\uc7a5\ub418\uc5b4 \uc788\uc73c\uba70, \uc774\uc804 \ubaa8\ub378\uc778 \ub77c\uc988\ubca0\ub9ac \ud30c\uc774 2\uc758 900MHz\ubcf4\ub2e4 \uac1c\uc120\ub41c 1.2GHz\uc758 \uc18d\ub3c4\ub97c \uad6c\ud604\ud55c \ube0c\ub85c\ub4dc\ucef4 BCM2837 64bit ARMv8 \ud504\ub85c\uc138\uc11c\ub97c \uae30\ubc18\uc73c\ub85c \ud55c\ub2e4. \ub610\ud55c, \uc804\ub825\uad00\ub9ac \uae30\ub2a5\ub3c4 \uac1c\uc120\ub418\uc5c8\ub2e4. \ucd5c\ub300 2.5 Amps\uc758 \uc804\ub825\ub9cc\uc744 \uc0ac\uc6a9\ud558\ub294 \ub3d9\uc2dc\uc5d0 \uac15\ub825\ud55c \uc678\ubd80 USB\uae30\uae30\ub97c \ub354\uc6b1 \ub9ce\uc774 \uc9c0\uc6d0\ud560 \uc218 \uc788\ub2e4. \uc800\uc804\ub825 \ube14\ub8e8\ud22c\uc2a4(Bluetooth Low Energy)\uc640 \ubb34\uc120 LAN\uc744 \uc9c0\uc6d0\ud558\uac8c \ub41c \ub77c\uc988\ubca0\ub9ac\ud30c\uc774 3\ub294 \uad6c\uc785 \uc989\uc2dc IoT, \ube14\ub8e8\ud22c\uc2a4 \ud5e4\ub4dc\ud3f0 \ub610\ub294 \uc2a4\ud53c\ucee4, \uc640\uc774\ud30c\uc774 \uac8c\uc774\ud2b8\uc6e8\uc774,\ud648 \ud074\ub77c\uc6b0\ub4dc \uc2a4\ud1a0\ub9ac\uc9c0 \ub4f1\uc5d0 \uc0ac\uc6a9\ud560 \uc218 \uc788\ub2e4. \uc81c\ud488\uc740 35\ub2ec\ub7ec\uc5d0 \uad6c\ub9e4 \uac00\ub2a5\ud558\uba70 \uc774\uc804 \ubc84\uc804\uacfc \ud638\ud658\ub41c\ub2e4."} {"question": "\ud55c\uad6d\uc774 \ub7ec\uc2dc\uc544\ub85c\ubd80\ud130 \ud575\uc7a0\uc218\ud568 \uc124\uacc4\ub3c4\ub97c \uc785\uc218\ud55c \ud574\ub294?", "paragraph": "\uc778\ub3c4\uac00 \uc544\ub9ac\ud55c\ud2b8\ud568 \uc81c\uc791\uc5d0 \ub7ec\uc2dc\uc544\uc640 \ud504\ub791\uc2a4\uc758 \uae30\uc220\uc744 \ub3c4\uc785\ud558\uc600\ub2e4\uace0 \ud558\ub294\ub370, \ud55c\uad6d\uacfc \ube0c\ub77c\uc9c8\ub3c4 \ucd5c\ucd08\uc758 \ud575\uc7a0\uc218\ud568 \uac74\uc870\ub97c \uc704\ud574 \ub7ec\uc2dc\uc544\uc640 \ud504\ub791\uc2a4\uc758 \uae30\uc220\uc744 \ub3c4\uc785\ud558\ub824 \ud55c\ub2e4\ub294 \ubcf4\ub3c4\uac00 \uc5ec\ub7ec\ucc28\ub840 \uc788\uc5c8\ub2e4. 2009\ub144 \ube0c\ub77c\uc9c8 \ub8f0\ub77c \ub300\ud1b5\ub839\uc740 \ud504\ub791\uc2a4 \uc0ac\ub974\ucf54\uc9c0 \ub300\ud1b5\ub839\uacfc \ud575\uc7a0\uc218\ud568 \uc218\uc785\uacc4\uc57d\uc744 \uc815\uc2dd\uc73c\ub85c \uccb4\uacb0\ud588\ub2e4. \ub7ec\uc2dc\uc544\uc640\ub3c4 \ud611\uc0c1\uc744 \ud588\uc5c8\ub2e4. \ube0c\ub77c\uc9c8\uc740 1970\ub144\ub300 \ubd80\ud130 \ud575\uc7a0\uc218\ud568 \uac1c\ubc1c\uc744 \ud574\uc654\ub2e4. \ud55c\uad6d\uc740 1993\ub144 \ub7ec\uc2dc\uc544\ub85c \ubd80\ud130 \ud575\uc7a0\uc218\ud568 \uc124\uacc4\ub3c4\ub97c \uc785\uc218\ud588\uc73c\uba70, \ub7ec\uc2dc\uc544 \ud575\uc7a0\uc218\ud568 \uc6d0\uc790\ub85c\uc758 \uae30\uc220\uc744 \uc644\uc804\ud788 \uc774\uc804\ubc1b\uc544 \uc2a4\ub9c8\ud2b8 \uc6d0\uc790\ub85c \uac1c\ubc1c\uc744 \uc644\ub8cc\ud588\ub2e4. \ub300\uccb4\uc801\uc73c\ub85c \ubbf8\uad6d\uc740 \ud575\ud655\uc0b0\uc5d0 \ub9e4\uc6b0 \uc18c\uadf9\uc801\uc774\uace0 \uc801\ub300\uc801\uc778 \ubc18\uba74\uc5d0, \ub7ec\uc2dc\uc544\uc640 \ud504\ub791\uc2a4\ub294 \ud575\ud655\uc0b0\uc5d0 \ub9e4\uc6b0 \uc801\uadf9\uc801\uc774\ub2e4. \ud30c\ud0a4\uc2a4\ud0c4, \uc774\ub780, \ubd81\ud55c \ub4f1\uc758 \ud575\uac1c\ubc1c\uc5d0\ub294 \ub7ec\uc2dc\uc544 \ub610\ub294 \ud504\ub791\uc2a4\uc758 \ud575\uae30\uc220\uc774 \uc774\uc804\ub41c \uac83\uc73c\ub85c \uc54c\ub824\uc838 \uc788\ub2e4.", "answer": "1993\ub144", "sentence": "\ud55c\uad6d\uc740 1993\ub144 \ub7ec\uc2dc\uc544\ub85c \ubd80\ud130 \ud575\uc7a0\uc218\ud568 \uc124\uacc4\ub3c4\ub97c \uc785\uc218\ud588\uc73c\uba70, \ub7ec\uc2dc\uc544 \ud575\uc7a0\uc218\ud568 \uc6d0\uc790\ub85c\uc758 \uae30\uc220\uc744 \uc644\uc804\ud788 \uc774\uc804\ubc1b\uc544 \uc2a4\ub9c8\ud2b8 \uc6d0\uc790\ub85c \uac1c\ubc1c\uc744 \uc644\ub8cc\ud588\ub2e4.", "paragraph_sentence": "\uc778\ub3c4\uac00 \uc544\ub9ac\ud55c\ud2b8\ud568 \uc81c\uc791\uc5d0 \ub7ec\uc2dc\uc544\uc640 \ud504\ub791\uc2a4\uc758 \uae30\uc220\uc744 \ub3c4\uc785\ud558\uc600\ub2e4\uace0 \ud558\ub294\ub370, \ud55c\uad6d\uacfc \ube0c\ub77c\uc9c8\ub3c4 \ucd5c\ucd08\uc758 \ud575\uc7a0\uc218\ud568 \uac74\uc870\ub97c \uc704\ud574 \ub7ec\uc2dc\uc544\uc640 \ud504\ub791\uc2a4\uc758 \uae30\uc220\uc744 \ub3c4\uc785\ud558\ub824 \ud55c\ub2e4\ub294 \ubcf4\ub3c4\uac00 \uc5ec\ub7ec\ucc28\ub840 \uc788\uc5c8\ub2e4. 2009\ub144 \ube0c\ub77c\uc9c8 \ub8f0\ub77c \ub300\ud1b5\ub839\uc740 \ud504\ub791\uc2a4 \uc0ac\ub974\ucf54\uc9c0 \ub300\ud1b5\ub839\uacfc \ud575\uc7a0\uc218\ud568 \uc218\uc785\uacc4\uc57d\uc744 \uc815\uc2dd\uc73c\ub85c \uccb4\uacb0\ud588\ub2e4. \ub7ec\uc2dc\uc544\uc640\ub3c4 \ud611\uc0c1\uc744 \ud588\uc5c8\ub2e4. \ube0c\ub77c\uc9c8\uc740 1970\ub144\ub300 \ubd80\ud130 \ud575\uc7a0\uc218\ud568 \uac1c\ubc1c\uc744 \ud574\uc654\ub2e4. \ud55c\uad6d\uc740 1993\ub144 \ub7ec\uc2dc\uc544\ub85c \ubd80\ud130 \ud575\uc7a0\uc218\ud568 \uc124\uacc4\ub3c4\ub97c \uc785\uc218\ud588\uc73c\uba70, \ub7ec\uc2dc\uc544 \ud575\uc7a0\uc218\ud568 \uc6d0\uc790\ub85c\uc758 \uae30\uc220\uc744 \uc644\uc804\ud788 \uc774\uc804\ubc1b\uc544 \uc2a4\ub9c8\ud2b8 \uc6d0\uc790\ub85c \uac1c\ubc1c\uc744 \uc644\ub8cc\ud588\ub2e4. \ub300\uccb4\uc801\uc73c\ub85c \ubbf8\uad6d\uc740 \ud575\ud655\uc0b0\uc5d0 \ub9e4\uc6b0 \uc18c\uadf9\uc801\uc774\uace0 \uc801\ub300\uc801\uc778 \ubc18\uba74\uc5d0, \ub7ec\uc2dc\uc544\uc640 \ud504\ub791\uc2a4\ub294 \ud575\ud655\uc0b0\uc5d0 \ub9e4\uc6b0 \uc801\uadf9\uc801\uc774\ub2e4. \ud30c\ud0a4\uc2a4\ud0c4, \uc774\ub780, \ubd81\ud55c \ub4f1\uc758 \ud575\uac1c\ubc1c\uc5d0\ub294 \ub7ec\uc2dc\uc544 \ub610\ub294 \ud504\ub791\uc2a4\uc758 \ud575\uae30\uc220\uc774 \uc774\uc804\ub41c \uac83\uc73c\ub85c \uc54c\ub824\uc838 \uc788\ub2e4.", "paragraph_answer": "\uc778\ub3c4\uac00 \uc544\ub9ac\ud55c\ud2b8\ud568 \uc81c\uc791\uc5d0 \ub7ec\uc2dc\uc544\uc640 \ud504\ub791\uc2a4\uc758 \uae30\uc220\uc744 \ub3c4\uc785\ud558\uc600\ub2e4\uace0 \ud558\ub294\ub370, \ud55c\uad6d\uacfc \ube0c\ub77c\uc9c8\ub3c4 \ucd5c\ucd08\uc758 \ud575\uc7a0\uc218\ud568 \uac74\uc870\ub97c \uc704\ud574 \ub7ec\uc2dc\uc544\uc640 \ud504\ub791\uc2a4\uc758 \uae30\uc220\uc744 \ub3c4\uc785\ud558\ub824 \ud55c\ub2e4\ub294 \ubcf4\ub3c4\uac00 \uc5ec\ub7ec\ucc28\ub840 \uc788\uc5c8\ub2e4. 2009\ub144 \ube0c\ub77c\uc9c8 \ub8f0\ub77c \ub300\ud1b5\ub839\uc740 \ud504\ub791\uc2a4 \uc0ac\ub974\ucf54\uc9c0 \ub300\ud1b5\ub839\uacfc \ud575\uc7a0\uc218\ud568 \uc218\uc785\uacc4\uc57d\uc744 \uc815\uc2dd\uc73c\ub85c \uccb4\uacb0\ud588\ub2e4. \ub7ec\uc2dc\uc544\uc640\ub3c4 \ud611\uc0c1\uc744 \ud588\uc5c8\ub2e4. \ube0c\ub77c\uc9c8\uc740 1970\ub144\ub300 \ubd80\ud130 \ud575\uc7a0\uc218\ud568 \uac1c\ubc1c\uc744 \ud574\uc654\ub2e4. \ud55c\uad6d\uc740 1993\ub144 \ub7ec\uc2dc\uc544\ub85c \ubd80\ud130 \ud575\uc7a0\uc218\ud568 \uc124\uacc4\ub3c4\ub97c \uc785\uc218\ud588\uc73c\uba70, \ub7ec\uc2dc\uc544 \ud575\uc7a0\uc218\ud568 \uc6d0\uc790\ub85c\uc758 \uae30\uc220\uc744 \uc644\uc804\ud788 \uc774\uc804\ubc1b\uc544 \uc2a4\ub9c8\ud2b8 \uc6d0\uc790\ub85c \uac1c\ubc1c\uc744 \uc644\ub8cc\ud588\ub2e4. \ub300\uccb4\uc801\uc73c\ub85c \ubbf8\uad6d\uc740 \ud575\ud655\uc0b0\uc5d0 \ub9e4\uc6b0 \uc18c\uadf9\uc801\uc774\uace0 \uc801\ub300\uc801\uc778 \ubc18\uba74\uc5d0, \ub7ec\uc2dc\uc544\uc640 \ud504\ub791\uc2a4\ub294 \ud575\ud655\uc0b0\uc5d0 \ub9e4\uc6b0 \uc801\uadf9\uc801\uc774\ub2e4. \ud30c\ud0a4\uc2a4\ud0c4, \uc774\ub780, \ubd81\ud55c \ub4f1\uc758 \ud575\uac1c\ubc1c\uc5d0\ub294 \ub7ec\uc2dc\uc544 \ub610\ub294 \ud504\ub791\uc2a4\uc758 \ud575\uae30\uc220\uc774 \uc774\uc804\ub41c \uac83\uc73c\ub85c \uc54c\ub824\uc838 \uc788\ub2e4.", "sentence_answer": "\ud55c\uad6d\uc740 1993\ub144 \ub7ec\uc2dc\uc544\ub85c \ubd80\ud130 \ud575\uc7a0\uc218\ud568 \uc124\uacc4\ub3c4\ub97c \uc785\uc218\ud588\uc73c\uba70, \ub7ec\uc2dc\uc544 \ud575\uc7a0\uc218\ud568 \uc6d0\uc790\ub85c\uc758 \uae30\uc220\uc744 \uc644\uc804\ud788 \uc774\uc804\ubc1b\uc544 \uc2a4\ub9c8\ud2b8 \uc6d0\uc790\ub85c \uac1c\ubc1c\uc744 \uc644\ub8cc\ud588\ub2e4.", "paragraph_id": "6533540-0-2", "paragraph_question": "question: \ud55c\uad6d\uc774 \ub7ec\uc2dc\uc544\ub85c\ubd80\ud130 \ud575\uc7a0\uc218\ud568 \uc124\uacc4\ub3c4\ub97c \uc785\uc218\ud55c \ud574\ub294?, context: \uc778\ub3c4\uac00 \uc544\ub9ac\ud55c\ud2b8\ud568 \uc81c\uc791\uc5d0 \ub7ec\uc2dc\uc544\uc640 \ud504\ub791\uc2a4\uc758 \uae30\uc220\uc744 \ub3c4\uc785\ud558\uc600\ub2e4\uace0 \ud558\ub294\ub370, \ud55c\uad6d\uacfc \ube0c\ub77c\uc9c8\ub3c4 \ucd5c\ucd08\uc758 \ud575\uc7a0\uc218\ud568 \uac74\uc870\ub97c \uc704\ud574 \ub7ec\uc2dc\uc544\uc640 \ud504\ub791\uc2a4\uc758 \uae30\uc220\uc744 \ub3c4\uc785\ud558\ub824 \ud55c\ub2e4\ub294 \ubcf4\ub3c4\uac00 \uc5ec\ub7ec\ucc28\ub840 \uc788\uc5c8\ub2e4. 2009\ub144 \ube0c\ub77c\uc9c8 \ub8f0\ub77c \ub300\ud1b5\ub839\uc740 \ud504\ub791\uc2a4 \uc0ac\ub974\ucf54\uc9c0 \ub300\ud1b5\ub839\uacfc \ud575\uc7a0\uc218\ud568 \uc218\uc785\uacc4\uc57d\uc744 \uc815\uc2dd\uc73c\ub85c \uccb4\uacb0\ud588\ub2e4. \ub7ec\uc2dc\uc544\uc640\ub3c4 \ud611\uc0c1\uc744 \ud588\uc5c8\ub2e4. \ube0c\ub77c\uc9c8\uc740 1970\ub144\ub300 \ubd80\ud130 \ud575\uc7a0\uc218\ud568 \uac1c\ubc1c\uc744 \ud574\uc654\ub2e4. \ud55c\uad6d\uc740 1993\ub144 \ub7ec\uc2dc\uc544\ub85c \ubd80\ud130 \ud575\uc7a0\uc218\ud568 \uc124\uacc4\ub3c4\ub97c \uc785\uc218\ud588\uc73c\uba70, \ub7ec\uc2dc\uc544 \ud575\uc7a0\uc218\ud568 \uc6d0\uc790\ub85c\uc758 \uae30\uc220\uc744 \uc644\uc804\ud788 \uc774\uc804\ubc1b\uc544 \uc2a4\ub9c8\ud2b8 \uc6d0\uc790\ub85c \uac1c\ubc1c\uc744 \uc644\ub8cc\ud588\ub2e4. \ub300\uccb4\uc801\uc73c\ub85c \ubbf8\uad6d\uc740 \ud575\ud655\uc0b0\uc5d0 \ub9e4\uc6b0 \uc18c\uadf9\uc801\uc774\uace0 \uc801\ub300\uc801\uc778 \ubc18\uba74\uc5d0, \ub7ec\uc2dc\uc544\uc640 \ud504\ub791\uc2a4\ub294 \ud575\ud655\uc0b0\uc5d0 \ub9e4\uc6b0 \uc801\uadf9\uc801\uc774\ub2e4. \ud30c\ud0a4\uc2a4\ud0c4, \uc774\ub780, \ubd81\ud55c \ub4f1\uc758 \ud575\uac1c\ubc1c\uc5d0\ub294 \ub7ec\uc2dc\uc544 \ub610\ub294 \ud504\ub791\uc2a4\uc758 \ud575\uae30\uc220\uc774 \uc774\uc804\ub41c \uac83\uc73c\ub85c \uc54c\ub824\uc838 \uc788\ub2e4."} {"question": "\uc678\uad50\uc815\ucc45\uc744 \uc218\ub9bd\ud558\uace0 \uc2e4\ud589\ud558\ub294 \ub370 \uc788\uc5b4 \uc8fc\ub41c \ucc45\uc784\uc744 \uac00\uc9c0\ub294 \uc9c0\uc704\ub294?", "paragraph": "\uc678\uad50\uc815\ucc45\uc758 \uc218\ub9bd\uacfc \uc774\ud589\uc5d0 \ub300\ud55c \uc8fc\ub41c \ucc45\uc784\uc744 \ub300\ud1b5\ub839\uc774 \uc9c0\uac8c \ub418\uc5b4 \uc788\uc73c\ub098, \uc758\ud68c\ub3c4 \uc774\uc5d0 \ub300\ud574 \uac15\ud55c \ud1b5\uc81c\ub825\uc744 \ubc1c\ud718\ud560 \uc218 \uc788\ub2e4. \uc6b0\uc120 \uc758\ud68c\ub294 \uc804\uc7c1\uc744 \uc120\ud3ec\ud560 \uc218 \uc788\ub294 \uad8c\ud55c\uc774 \uc788\ub2e4. \ud558\uc6d0\uc740 \ub300\ud1b5\ub839\uc758 \uc678\uad50\uc815\ucc45 \uc218\ud589\uc5d0 \ud544\uc694\ud55c \uc608\uc0b0\uc5d0 \ub300\ud574 \uac15\ud55c \ud1b5\uc81c\ub825\uc744 \uac00\uc9c0\uace0 \uc788\uc73c\uba70, \uc0c1\uc6d0\ub3c4 \uc7ac\uc6d0\uc5d0 \ub300\ud55c \ud1b5\uc81c\uac00 \uac00\ub2a5\ud558\ub2e4. \ud2b9\ud788 \uc0c1\uc6d0\uc758 \uacbd\uc6b0\uc5d0\ub294 \uace0\uae09 \uc678\ubb34\uc801 \uad00\ub9ac\uc758 \uc784\uba85\uc5d0 \ub300\ud55c \ud1b5\uc81c\ub825\uc744 \uac00\uc9c0\uace0 \uc788\uc744 \ubfd0\ub9cc \uc544\ub2c8\ub77c, \ubbf8\uad6d\uc774 \uccb4\uacb0\ud558\ub294 \ubaa8\ub4e0 \uc870\uc57d\uc5d0 \ub300\ud55c \ube44\uc900\uad8c\uc744 \uac00\uc9c0\uace0 \uc788\ub2e4. \uc758\ud68c\uc758 \uc774\uc640 \uac19\uc740 \uc81c\uc7ac\uad8c\ud55c\uc740 \ud589\uc815\ubd80\uc640 \uc758\ud68c\uc758 \uac08\ub4f1\uc744 \ucd08\ub798\ud558\ub294 \uc694\uc18c\ub85c \uc791\uc6a9\ud55c\ub2e4. \uc5d0\ub4dc\uc6cc\ub4dc \ucf54\ub974\uc708(Elward S. Corwin) \uad50\uc218\ub294 \ubbf8\uad6d\uc758 \uc815\uce58\uccb4\uacc4\uac00 \uc678\uad50\uc815\ucc45\uc5d0 \uc788\uc5b4\uc11c \ud589\uc815\ubd80\uc640 \uc758\ud68c \uac04\uc758 \ud22c\uc7c1\uc744 \ucd08\ub798\ud55c\ub2e4\uace0 \uc9c0\uc801\ud55c \ubc14 \uc788\ub2e4. \uadf8 \uc88b\uc740 \uc608\ub85c\uc11c\ub294 \uc0c1\uc6d0\uc774 \uad6d\uc81c\uc5f0\ub9f9 \ud5cc\uc7a5\uc758 \ube44\uc900\uc744 \uac70\ubd80\ud55c \uc0ac\uc2e4\uc744 \ub4e4 \uc218 \uc788\ub2e4. \uc758\ud68c\uc640 \ud589\uc815\ubd80\uac04\uc5d0\ub294 \uae34\ubc00\ud55c \ud611\uc870\uac00 \uc774\ub8e8\uc5b4\uc9c0\uace0 \uc788\ub2e4. \uc720\uc5d4\ud5cc\uc7a5 \ube44\uc900 \ub2f9\uc2dc\uc758 \uc758\ud68c\uc640 \ud589\uc815\ubd80\uac04\uc758 \uae34\ubc00\ud55c \ud611\uc870\ub294 \ub108\ubb34\ub098 \uc720\uba85\ud558\ub2e4. \uadf8\ub7ec\ub098 \uc774\ub7ec\ud55c \ud611\ub3d9\uc678\uad50\ub294 \ud55c\uad6d\uc5d0\uc11c \ud754\ud788 \uc0ac\uc6a9\ub418\uace0 \uc788\ub294 '\ucd08\ub2f9\uc678\uad50'\ub77c\ub294 \ub9d0\uacfc\ub294 \uc5c4\uaca9\ud788 \uad6c\ubd84\ub418\uc5b4\uc57c \ud55c\ub2e4. \ubca0\ud2b8\ub0a8 \uc804\uc7c1\uc73c\ub85c \ub9d0\ubbf8\uc554\uc544 \ud30c\uc0dd\ub41c \ud589\uc815\ubd80\uc640 \uc758\ud68c\uac04\uc758 \uac08\ub4f1\uc740 \uc2ec\uac01\ud55c \ud615\ud3b8\uc774\uc5c8\ub2e4. \uadf8\ub7ec\ub098 \uc758\ud68c\uac00 \ub300\ud1b5\ub839\uc5d0 \uac00\ud558\ub294 \uc555\ub825\uc740 \ud589\uc815\ubd80\uc758 \ub3c5\uc8fc\ub97c \uacac\uc81c\ud558\ub294 \ub3d9\uc2dc\uc5d0 \uac74\uc804\ud55c \ubc29\ud5a5\uc758 \ubbf8\uad6d \ub300\uc678\uc815\ucc45\uc744 \ubaa8\uc0c9\ud558\uace0 \uc788\ub2e4. \uc758\ud68c\ub294 \uad6d\ubbfc\uc758 \ub300\ud45c\uae30\uad6c\ub85c\uc11c \uad6d\ubbfc\uc758 \uc758\uc0ac\uc640 \uc9c1\uacb0\ub3fc \uc788\ub2e4. \ub530\ub77c\uc11c \ub300\uc678\uc815\ucc45\uc758 \uc218\ub9bd\uc774\ub098 \uc774\ud589 \uacfc\uc815\uc5d0\ub3c4 \ud56d\uc0c1 \uc758\ud68c\ub97c '\ubb38\uc81c\uc544'\ub85c \ucde8\uae09\ud558\uac70\ub098, \ub610\ub294 \uadf8\ub7ec\ud55c \ubc94\uc8fc\ub0b4\uc5d0 \uc758\ud68c\uc758 \uae30\ub2a5\uc744 \uc774\ud574\ud574\uc11c\ub294 \uc548 \ub41c\ub2e4.", "answer": "\ub300\ud1b5\ub839", "sentence": "\uc678\uad50\uc815\ucc45\uc758 \uc218\ub9bd\uacfc \uc774\ud589\uc5d0 \ub300\ud55c \uc8fc\ub41c \ucc45\uc784\uc744 \ub300\ud1b5\ub839 \uc774 \uc9c0\uac8c \ub418\uc5b4 \uc788\uc73c\ub098, \uc758\ud68c\ub3c4 \uc774\uc5d0 \ub300\ud574 \uac15\ud55c \ud1b5\uc81c\ub825\uc744 \ubc1c\ud718\ud560 \uc218 \uc788\ub2e4.", "paragraph_sentence": " \uc678\uad50\uc815\ucc45\uc758 \uc218\ub9bd\uacfc \uc774\ud589\uc5d0 \ub300\ud55c \uc8fc\ub41c \ucc45\uc784\uc744 \ub300\ud1b5\ub839 \uc774 \uc9c0\uac8c \ub418\uc5b4 \uc788\uc73c\ub098, \uc758\ud68c\ub3c4 \uc774\uc5d0 \ub300\ud574 \uac15\ud55c \ud1b5\uc81c\ub825\uc744 \ubc1c\ud718\ud560 \uc218 \uc788\ub2e4. \uc6b0\uc120 \uc758\ud68c\ub294 \uc804\uc7c1\uc744 \uc120\ud3ec\ud560 \uc218 \uc788\ub294 \uad8c\ud55c\uc774 \uc788\ub2e4. \ud558\uc6d0\uc740 \ub300\ud1b5\ub839\uc758 \uc678\uad50\uc815\ucc45 \uc218\ud589\uc5d0 \ud544\uc694\ud55c \uc608\uc0b0\uc5d0 \ub300\ud574 \uac15\ud55c \ud1b5\uc81c\ub825\uc744 \uac00\uc9c0\uace0 \uc788\uc73c\uba70, \uc0c1\uc6d0\ub3c4 \uc7ac\uc6d0\uc5d0 \ub300\ud55c \ud1b5\uc81c\uac00 \uac00\ub2a5\ud558\ub2e4. \ud2b9\ud788 \uc0c1\uc6d0\uc758 \uacbd\uc6b0\uc5d0\ub294 \uace0\uae09 \uc678\ubb34\uc801 \uad00\ub9ac\uc758 \uc784\uba85\uc5d0 \ub300\ud55c \ud1b5\uc81c\ub825\uc744 \uac00\uc9c0\uace0 \uc788\uc744 \ubfd0\ub9cc \uc544\ub2c8\ub77c, \ubbf8\uad6d\uc774 \uccb4\uacb0\ud558\ub294 \ubaa8\ub4e0 \uc870\uc57d\uc5d0 \ub300\ud55c \ube44\uc900\uad8c\uc744 \uac00\uc9c0\uace0 \uc788\ub2e4. \uc758\ud68c\uc758 \uc774\uc640 \uac19\uc740 \uc81c\uc7ac\uad8c\ud55c\uc740 \ud589\uc815\ubd80\uc640 \uc758\ud68c\uc758 \uac08\ub4f1\uc744 \ucd08\ub798\ud558\ub294 \uc694\uc18c\ub85c \uc791\uc6a9\ud55c\ub2e4. \uc5d0\ub4dc\uc6cc\ub4dc \ucf54\ub974\uc708(Elward S. Corwin) \uad50\uc218\ub294 \ubbf8\uad6d\uc758 \uc815\uce58\uccb4\uacc4\uac00 \uc678\uad50\uc815\ucc45\uc5d0 \uc788\uc5b4\uc11c \ud589\uc815\ubd80\uc640 \uc758\ud68c \uac04\uc758 \ud22c\uc7c1\uc744 \ucd08\ub798\ud55c\ub2e4\uace0 \uc9c0\uc801\ud55c \ubc14 \uc788\ub2e4. \uadf8 \uc88b\uc740 \uc608\ub85c\uc11c\ub294 \uc0c1\uc6d0\uc774 \uad6d\uc81c\uc5f0\ub9f9 \ud5cc\uc7a5\uc758 \ube44\uc900\uc744 \uac70\ubd80\ud55c \uc0ac\uc2e4\uc744 \ub4e4 \uc218 \uc788\ub2e4. \uc758\ud68c\uc640 \ud589\uc815\ubd80\uac04\uc5d0\ub294 \uae34\ubc00\ud55c \ud611\uc870\uac00 \uc774\ub8e8\uc5b4\uc9c0\uace0 \uc788\ub2e4. \uc720\uc5d4\ud5cc\uc7a5 \ube44\uc900 \ub2f9\uc2dc\uc758 \uc758\ud68c\uc640 \ud589\uc815\ubd80\uac04\uc758 \uae34\ubc00\ud55c \ud611\uc870\ub294 \ub108\ubb34\ub098 \uc720\uba85\ud558\ub2e4. \uadf8\ub7ec\ub098 \uc774\ub7ec\ud55c \ud611\ub3d9\uc678\uad50\ub294 \ud55c\uad6d\uc5d0\uc11c \ud754\ud788 \uc0ac\uc6a9\ub418\uace0 \uc788\ub294 '\ucd08\ub2f9\uc678\uad50'\ub77c\ub294 \ub9d0\uacfc\ub294 \uc5c4\uaca9\ud788 \uad6c\ubd84\ub418\uc5b4\uc57c \ud55c\ub2e4. \ubca0\ud2b8\ub0a8 \uc804\uc7c1\uc73c\ub85c \ub9d0\ubbf8\uc554\uc544 \ud30c\uc0dd\ub41c \ud589\uc815\ubd80\uc640 \uc758\ud68c\uac04\uc758 \uac08\ub4f1\uc740 \uc2ec\uac01\ud55c \ud615\ud3b8\uc774\uc5c8\ub2e4. \uadf8\ub7ec\ub098 \uc758\ud68c\uac00 \ub300\ud1b5\ub839\uc5d0 \uac00\ud558\ub294 \uc555\ub825\uc740 \ud589\uc815\ubd80\uc758 \ub3c5\uc8fc\ub97c \uacac\uc81c\ud558\ub294 \ub3d9\uc2dc\uc5d0 \uac74\uc804\ud55c \ubc29\ud5a5\uc758 \ubbf8\uad6d \ub300\uc678\uc815\ucc45\uc744 \ubaa8\uc0c9\ud558\uace0 \uc788\ub2e4. \uc758\ud68c\ub294 \uad6d\ubbfc\uc758 \ub300\ud45c\uae30\uad6c\ub85c\uc11c \uad6d\ubbfc\uc758 \uc758\uc0ac\uc640 \uc9c1\uacb0\ub3fc \uc788\ub2e4. \ub530\ub77c\uc11c \ub300\uc678\uc815\ucc45\uc758 \uc218\ub9bd\uc774\ub098 \uc774\ud589 \uacfc\uc815\uc5d0\ub3c4 \ud56d\uc0c1 \uc758\ud68c\ub97c '\ubb38\uc81c\uc544'\ub85c \ucde8\uae09\ud558\uac70\ub098, \ub610\ub294 \uadf8\ub7ec\ud55c \ubc94\uc8fc\ub0b4\uc5d0 \uc758\ud68c\uc758 \uae30\ub2a5\uc744 \uc774\ud574\ud574\uc11c\ub294 \uc548 \ub41c\ub2e4.", "paragraph_answer": "\uc678\uad50\uc815\ucc45\uc758 \uc218\ub9bd\uacfc \uc774\ud589\uc5d0 \ub300\ud55c \uc8fc\ub41c \ucc45\uc784\uc744 \ub300\ud1b5\ub839 \uc774 \uc9c0\uac8c \ub418\uc5b4 \uc788\uc73c\ub098, \uc758\ud68c\ub3c4 \uc774\uc5d0 \ub300\ud574 \uac15\ud55c \ud1b5\uc81c\ub825\uc744 \ubc1c\ud718\ud560 \uc218 \uc788\ub2e4. \uc6b0\uc120 \uc758\ud68c\ub294 \uc804\uc7c1\uc744 \uc120\ud3ec\ud560 \uc218 \uc788\ub294 \uad8c\ud55c\uc774 \uc788\ub2e4. \ud558\uc6d0\uc740 \ub300\ud1b5\ub839\uc758 \uc678\uad50\uc815\ucc45 \uc218\ud589\uc5d0 \ud544\uc694\ud55c \uc608\uc0b0\uc5d0 \ub300\ud574 \uac15\ud55c \ud1b5\uc81c\ub825\uc744 \uac00\uc9c0\uace0 \uc788\uc73c\uba70, \uc0c1\uc6d0\ub3c4 \uc7ac\uc6d0\uc5d0 \ub300\ud55c \ud1b5\uc81c\uac00 \uac00\ub2a5\ud558\ub2e4. \ud2b9\ud788 \uc0c1\uc6d0\uc758 \uacbd\uc6b0\uc5d0\ub294 \uace0\uae09 \uc678\ubb34\uc801 \uad00\ub9ac\uc758 \uc784\uba85\uc5d0 \ub300\ud55c \ud1b5\uc81c\ub825\uc744 \uac00\uc9c0\uace0 \uc788\uc744 \ubfd0\ub9cc \uc544\ub2c8\ub77c, \ubbf8\uad6d\uc774 \uccb4\uacb0\ud558\ub294 \ubaa8\ub4e0 \uc870\uc57d\uc5d0 \ub300\ud55c \ube44\uc900\uad8c\uc744 \uac00\uc9c0\uace0 \uc788\ub2e4. \uc758\ud68c\uc758 \uc774\uc640 \uac19\uc740 \uc81c\uc7ac\uad8c\ud55c\uc740 \ud589\uc815\ubd80\uc640 \uc758\ud68c\uc758 \uac08\ub4f1\uc744 \ucd08\ub798\ud558\ub294 \uc694\uc18c\ub85c \uc791\uc6a9\ud55c\ub2e4. \uc5d0\ub4dc\uc6cc\ub4dc \ucf54\ub974\uc708(Elward S. Corwin) \uad50\uc218\ub294 \ubbf8\uad6d\uc758 \uc815\uce58\uccb4\uacc4\uac00 \uc678\uad50\uc815\ucc45\uc5d0 \uc788\uc5b4\uc11c \ud589\uc815\ubd80\uc640 \uc758\ud68c \uac04\uc758 \ud22c\uc7c1\uc744 \ucd08\ub798\ud55c\ub2e4\uace0 \uc9c0\uc801\ud55c \ubc14 \uc788\ub2e4. \uadf8 \uc88b\uc740 \uc608\ub85c\uc11c\ub294 \uc0c1\uc6d0\uc774 \uad6d\uc81c\uc5f0\ub9f9 \ud5cc\uc7a5\uc758 \ube44\uc900\uc744 \uac70\ubd80\ud55c \uc0ac\uc2e4\uc744 \ub4e4 \uc218 \uc788\ub2e4. \uc758\ud68c\uc640 \ud589\uc815\ubd80\uac04\uc5d0\ub294 \uae34\ubc00\ud55c \ud611\uc870\uac00 \uc774\ub8e8\uc5b4\uc9c0\uace0 \uc788\ub2e4. \uc720\uc5d4\ud5cc\uc7a5 \ube44\uc900 \ub2f9\uc2dc\uc758 \uc758\ud68c\uc640 \ud589\uc815\ubd80\uac04\uc758 \uae34\ubc00\ud55c \ud611\uc870\ub294 \ub108\ubb34\ub098 \uc720\uba85\ud558\ub2e4. \uadf8\ub7ec\ub098 \uc774\ub7ec\ud55c \ud611\ub3d9\uc678\uad50\ub294 \ud55c\uad6d\uc5d0\uc11c \ud754\ud788 \uc0ac\uc6a9\ub418\uace0 \uc788\ub294 '\ucd08\ub2f9\uc678\uad50'\ub77c\ub294 \ub9d0\uacfc\ub294 \uc5c4\uaca9\ud788 \uad6c\ubd84\ub418\uc5b4\uc57c \ud55c\ub2e4. \ubca0\ud2b8\ub0a8 \uc804\uc7c1\uc73c\ub85c \ub9d0\ubbf8\uc554\uc544 \ud30c\uc0dd\ub41c \ud589\uc815\ubd80\uc640 \uc758\ud68c\uac04\uc758 \uac08\ub4f1\uc740 \uc2ec\uac01\ud55c \ud615\ud3b8\uc774\uc5c8\ub2e4. \uadf8\ub7ec\ub098 \uc758\ud68c\uac00 \ub300\ud1b5\ub839\uc5d0 \uac00\ud558\ub294 \uc555\ub825\uc740 \ud589\uc815\ubd80\uc758 \ub3c5\uc8fc\ub97c \uacac\uc81c\ud558\ub294 \ub3d9\uc2dc\uc5d0 \uac74\uc804\ud55c \ubc29\ud5a5\uc758 \ubbf8\uad6d \ub300\uc678\uc815\ucc45\uc744 \ubaa8\uc0c9\ud558\uace0 \uc788\ub2e4. \uc758\ud68c\ub294 \uad6d\ubbfc\uc758 \ub300\ud45c\uae30\uad6c\ub85c\uc11c \uad6d\ubbfc\uc758 \uc758\uc0ac\uc640 \uc9c1\uacb0\ub3fc \uc788\ub2e4. \ub530\ub77c\uc11c \ub300\uc678\uc815\ucc45\uc758 \uc218\ub9bd\uc774\ub098 \uc774\ud589 \uacfc\uc815\uc5d0\ub3c4 \ud56d\uc0c1 \uc758\ud68c\ub97c '\ubb38\uc81c\uc544'\ub85c \ucde8\uae09\ud558\uac70\ub098, \ub610\ub294 \uadf8\ub7ec\ud55c \ubc94\uc8fc\ub0b4\uc5d0 \uc758\ud68c\uc758 \uae30\ub2a5\uc744 \uc774\ud574\ud574\uc11c\ub294 \uc548 \ub41c\ub2e4.", "sentence_answer": "\uc678\uad50\uc815\ucc45\uc758 \uc218\ub9bd\uacfc \uc774\ud589\uc5d0 \ub300\ud55c \uc8fc\ub41c \ucc45\uc784\uc744 \ub300\ud1b5\ub839 \uc774 \uc9c0\uac8c \ub418\uc5b4 \uc788\uc73c\ub098, \uc758\ud68c\ub3c4 \uc774\uc5d0 \ub300\ud574 \uac15\ud55c \ud1b5\uc81c\ub825\uc744 \ubc1c\ud718\ud560 \uc218 \uc788\ub2e4.", "paragraph_id": "6462682-20-1", "paragraph_question": "question: \uc678\uad50\uc815\ucc45\uc744 \uc218\ub9bd\ud558\uace0 \uc2e4\ud589\ud558\ub294 \ub370 \uc788\uc5b4 \uc8fc\ub41c \ucc45\uc784\uc744 \uac00\uc9c0\ub294 \uc9c0\uc704\ub294?, context: \uc678\uad50\uc815\ucc45\uc758 \uc218\ub9bd\uacfc \uc774\ud589\uc5d0 \ub300\ud55c \uc8fc\ub41c \ucc45\uc784\uc744 \ub300\ud1b5\ub839\uc774 \uc9c0\uac8c \ub418\uc5b4 \uc788\uc73c\ub098, \uc758\ud68c\ub3c4 \uc774\uc5d0 \ub300\ud574 \uac15\ud55c \ud1b5\uc81c\ub825\uc744 \ubc1c\ud718\ud560 \uc218 \uc788\ub2e4. \uc6b0\uc120 \uc758\ud68c\ub294 \uc804\uc7c1\uc744 \uc120\ud3ec\ud560 \uc218 \uc788\ub294 \uad8c\ud55c\uc774 \uc788\ub2e4. \ud558\uc6d0\uc740 \ub300\ud1b5\ub839\uc758 \uc678\uad50\uc815\ucc45 \uc218\ud589\uc5d0 \ud544\uc694\ud55c \uc608\uc0b0\uc5d0 \ub300\ud574 \uac15\ud55c \ud1b5\uc81c\ub825\uc744 \uac00\uc9c0\uace0 \uc788\uc73c\uba70, \uc0c1\uc6d0\ub3c4 \uc7ac\uc6d0\uc5d0 \ub300\ud55c \ud1b5\uc81c\uac00 \uac00\ub2a5\ud558\ub2e4. \ud2b9\ud788 \uc0c1\uc6d0\uc758 \uacbd\uc6b0\uc5d0\ub294 \uace0\uae09 \uc678\ubb34\uc801 \uad00\ub9ac\uc758 \uc784\uba85\uc5d0 \ub300\ud55c \ud1b5\uc81c\ub825\uc744 \uac00\uc9c0\uace0 \uc788\uc744 \ubfd0\ub9cc \uc544\ub2c8\ub77c, \ubbf8\uad6d\uc774 \uccb4\uacb0\ud558\ub294 \ubaa8\ub4e0 \uc870\uc57d\uc5d0 \ub300\ud55c \ube44\uc900\uad8c\uc744 \uac00\uc9c0\uace0 \uc788\ub2e4. \uc758\ud68c\uc758 \uc774\uc640 \uac19\uc740 \uc81c\uc7ac\uad8c\ud55c\uc740 \ud589\uc815\ubd80\uc640 \uc758\ud68c\uc758 \uac08\ub4f1\uc744 \ucd08\ub798\ud558\ub294 \uc694\uc18c\ub85c \uc791\uc6a9\ud55c\ub2e4. \uc5d0\ub4dc\uc6cc\ub4dc \ucf54\ub974\uc708(Elward S. Corwin) \uad50\uc218\ub294 \ubbf8\uad6d\uc758 \uc815\uce58\uccb4\uacc4\uac00 \uc678\uad50\uc815\ucc45\uc5d0 \uc788\uc5b4\uc11c \ud589\uc815\ubd80\uc640 \uc758\ud68c \uac04\uc758 \ud22c\uc7c1\uc744 \ucd08\ub798\ud55c\ub2e4\uace0 \uc9c0\uc801\ud55c \ubc14 \uc788\ub2e4. \uadf8 \uc88b\uc740 \uc608\ub85c\uc11c\ub294 \uc0c1\uc6d0\uc774 \uad6d\uc81c\uc5f0\ub9f9 \ud5cc\uc7a5\uc758 \ube44\uc900\uc744 \uac70\ubd80\ud55c \uc0ac\uc2e4\uc744 \ub4e4 \uc218 \uc788\ub2e4. \uc758\ud68c\uc640 \ud589\uc815\ubd80\uac04\uc5d0\ub294 \uae34\ubc00\ud55c \ud611\uc870\uac00 \uc774\ub8e8\uc5b4\uc9c0\uace0 \uc788\ub2e4. \uc720\uc5d4\ud5cc\uc7a5 \ube44\uc900 \ub2f9\uc2dc\uc758 \uc758\ud68c\uc640 \ud589\uc815\ubd80\uac04\uc758 \uae34\ubc00\ud55c \ud611\uc870\ub294 \ub108\ubb34\ub098 \uc720\uba85\ud558\ub2e4. \uadf8\ub7ec\ub098 \uc774\ub7ec\ud55c \ud611\ub3d9\uc678\uad50\ub294 \ud55c\uad6d\uc5d0\uc11c \ud754\ud788 \uc0ac\uc6a9\ub418\uace0 \uc788\ub294 '\ucd08\ub2f9\uc678\uad50'\ub77c\ub294 \ub9d0\uacfc\ub294 \uc5c4\uaca9\ud788 \uad6c\ubd84\ub418\uc5b4\uc57c \ud55c\ub2e4. \ubca0\ud2b8\ub0a8 \uc804\uc7c1\uc73c\ub85c \ub9d0\ubbf8\uc554\uc544 \ud30c\uc0dd\ub41c \ud589\uc815\ubd80\uc640 \uc758\ud68c\uac04\uc758 \uac08\ub4f1\uc740 \uc2ec\uac01\ud55c \ud615\ud3b8\uc774\uc5c8\ub2e4. \uadf8\ub7ec\ub098 \uc758\ud68c\uac00 \ub300\ud1b5\ub839\uc5d0 \uac00\ud558\ub294 \uc555\ub825\uc740 \ud589\uc815\ubd80\uc758 \ub3c5\uc8fc\ub97c \uacac\uc81c\ud558\ub294 \ub3d9\uc2dc\uc5d0 \uac74\uc804\ud55c \ubc29\ud5a5\uc758 \ubbf8\uad6d \ub300\uc678\uc815\ucc45\uc744 \ubaa8\uc0c9\ud558\uace0 \uc788\ub2e4. \uc758\ud68c\ub294 \uad6d\ubbfc\uc758 \ub300\ud45c\uae30\uad6c\ub85c\uc11c \uad6d\ubbfc\uc758 \uc758\uc0ac\uc640 \uc9c1\uacb0\ub3fc \uc788\ub2e4. \ub530\ub77c\uc11c \ub300\uc678\uc815\ucc45\uc758 \uc218\ub9bd\uc774\ub098 \uc774\ud589 \uacfc\uc815\uc5d0\ub3c4 \ud56d\uc0c1 \uc758\ud68c\ub97c '\ubb38\uc81c\uc544'\ub85c \ucde8\uae09\ud558\uac70\ub098, \ub610\ub294 \uadf8\ub7ec\ud55c \ubc94\uc8fc\ub0b4\uc5d0 \uc758\ud68c\uc758 \uae30\ub2a5\uc744 \uc774\ud574\ud574\uc11c\ub294 \uc548 \ub41c\ub2e4."} {"question": "\uc724\uc6d0\ud615\uc740 \ubb38\uc815\uc655\ud6c4\uac00 \uacbd\uc6d0\ub300\uad70\uc744 \ub0b3\uc790 \uc81c\uac70\ud55c \uac83\uc740 \ub204\uad6c\uc778\uac00?", "paragraph": "\uc911\uc885\uc758 \uc655\ube44\uc778 \ubb38\uc815\uc655\ud6c4\uc758 \ub3d9\uc0dd\uc73c\ub85c \ub204\ub098\uc778 \ubb38\uc815\uc655\ud6c4\uac00 \uc911\uc885\uc758 \uacc4\ube44\ub85c \ub4e4\uc5b4\uac04 \ub4a4, \uc911\uc885 \ub54c \ubb38\uacfc\uc5d0 \uae09\uc81c\ud558\uc600\ub2e4. \uc774\ud6c4 \uc0ac\uad00\uc744 \uac70\uccd0 1538\ub144 \uad50\ub9ac\ub85c \uc2dc\ub3c5\uad00\uc744 \uacb8\uc9c1\ud558\uace0 \uc0ac\ud5cc\ubd80\uc9c0\ud3c9, \ud64d\ubb38\uad00\uc751\uad50 \ub4f1\uc744 \uc9c0\ub0c8\ub2e4. \ubb38\uc815\uc655\ud6c4\uac00 \uacbd\uc6d0\ub300\uad70(\ud6c4\uc758 \uba85\uc885)\uc744 \ub0b3\uc790 \ub2f9\uc2dc\uc758 \uad8c\uc138\uac00\uc778 \uae40\uc548\ub85c\ub97c \uc81c\uac70\ud558\uc600\ub2e4. \ub610\ud55c \uc778\uc885\uc758 \uc678\uc0bc\ucd0c \uc724\uc784\uc774 \uacbd\uc6d0\ub300\uad70\uc744 \ud574\ud558\uace0\uc790 \ud55c\ub2e4\uace0 \ubaa8\ud568\ud558\uc5ec \uc11c\ub85c \ub2e4\ud22c\uc5c8\ub294\ub370, \uc774\ub54c \uc724\uc784 \uc77c\ud30c\ub97c \ub300\uc724\uc774\ub77c \ud558\uace0 \uc724\uc6d0\ud615 \uc77c\ud30c\ub97c \uc18c\uc724\uc774\ub77c\uace0 \ud588\ub2e4. \uc724\uc6d0\ud615\uacfc \uc724\uc784\uc740 \uce5c\ucc99\uc774\uc5c8\ub294\ub370, \uc724\uc6d0\ud615\uc758 5\ub300\uc870 \uc724\ubc88\uc758 \ub538\uc774 \uc138\uc870\ube44 \uc815\ud76c\uc655\ud6c4\ub85c, \uc815\ud76c\uc655\ud6c4\uc758 \ub0a8\ub3d9\uc0dd \uc724\uc0ac\ud754\uc758 4\ub300\uc190\uc774\uc5c8\ub2e4. \uc724\uc784\uc740 \uc724\ubc88\uc758 \ucc28\ub0a8 \uc724\uc0ac\uc724\uc758 \ud6c4\uc190\uc73c\ub85c \uc724\uc6d0\ud615\uc758 \uc544\ubc84\uc9c0 \uc724\uc9c0\uc784\uacfc\ub294 8\ucd0c\uac04\uc774\uc5c8\ub2e4. \uc724\uc6d0\ud615\uc740 \uc724\ubc88\uc758 \uc14b\uc9f8 \uc544\ub4e4 \uc724\uc0ac\ud754\uc758 \uace0\uc190\uc790\uc600\ub2e4.", "answer": "\uae40\uc548\ub85c", "sentence": "\ubb38\uc815\uc655\ud6c4\uac00 \uacbd\uc6d0\ub300\uad70(\ud6c4\uc758 \uba85\uc885)\uc744 \ub0b3\uc790 \ub2f9\uc2dc\uc758 \uad8c\uc138\uac00\uc778 \uae40\uc548\ub85c \ub97c \uc81c\uac70\ud558\uc600\ub2e4.", "paragraph_sentence": "\uc911\uc885\uc758 \uc655\ube44\uc778 \ubb38\uc815\uc655\ud6c4\uc758 \ub3d9\uc0dd\uc73c\ub85c \ub204\ub098\uc778 \ubb38\uc815\uc655\ud6c4\uac00 \uc911\uc885\uc758 \uacc4\ube44\ub85c \ub4e4\uc5b4\uac04 \ub4a4, \uc911\uc885 \ub54c \ubb38\uacfc\uc5d0 \uae09\uc81c\ud558\uc600\ub2e4. \uc774\ud6c4 \uc0ac\uad00\uc744 \uac70\uccd0 1538\ub144 \uad50\ub9ac\ub85c \uc2dc\ub3c5\uad00\uc744 \uacb8\uc9c1\ud558\uace0 \uc0ac\ud5cc\ubd80\uc9c0\ud3c9, \ud64d\ubb38\uad00\uc751\uad50 \ub4f1\uc744 \uc9c0\ub0c8\ub2e4. \ubb38\uc815\uc655\ud6c4\uac00 \uacbd\uc6d0\ub300\uad70(\ud6c4\uc758 \uba85\uc885)\uc744 \ub0b3\uc790 \ub2f9\uc2dc\uc758 \uad8c\uc138\uac00\uc778 \uae40\uc548\ub85c \ub97c \uc81c\uac70\ud558\uc600\ub2e4. \ub610\ud55c \uc778\uc885\uc758 \uc678\uc0bc\ucd0c \uc724\uc784\uc774 \uacbd\uc6d0\ub300\uad70\uc744 \ud574\ud558\uace0\uc790 \ud55c\ub2e4\uace0 \ubaa8\ud568\ud558\uc5ec \uc11c\ub85c \ub2e4\ud22c\uc5c8\ub294\ub370, \uc774\ub54c \uc724\uc784 \uc77c\ud30c\ub97c \ub300\uc724\uc774\ub77c \ud558\uace0 \uc724\uc6d0\ud615 \uc77c\ud30c\ub97c \uc18c\uc724\uc774\ub77c\uace0 \ud588\ub2e4. \uc724\uc6d0\ud615\uacfc \uc724\uc784\uc740 \uce5c\ucc99\uc774\uc5c8\ub294\ub370, \uc724\uc6d0\ud615\uc758 5\ub300\uc870 \uc724\ubc88\uc758 \ub538\uc774 \uc138\uc870\ube44 \uc815\ud76c\uc655\ud6c4\ub85c, \uc815\ud76c\uc655\ud6c4\uc758 \ub0a8\ub3d9\uc0dd \uc724\uc0ac\ud754\uc758 4\ub300\uc190\uc774\uc5c8\ub2e4. \uc724\uc784\uc740 \uc724\ubc88\uc758 \ucc28\ub0a8 \uc724\uc0ac\uc724\uc758 \ud6c4\uc190\uc73c\ub85c \uc724\uc6d0\ud615\uc758 \uc544\ubc84\uc9c0 \uc724\uc9c0\uc784\uacfc\ub294 8\ucd0c\uac04\uc774\uc5c8\ub2e4. \uc724\uc6d0\ud615\uc740 \uc724\ubc88\uc758 \uc14b\uc9f8 \uc544\ub4e4 \uc724\uc0ac\ud754\uc758 \uace0\uc190\uc790\uc600\ub2e4.", "paragraph_answer": "\uc911\uc885\uc758 \uc655\ube44\uc778 \ubb38\uc815\uc655\ud6c4\uc758 \ub3d9\uc0dd\uc73c\ub85c \ub204\ub098\uc778 \ubb38\uc815\uc655\ud6c4\uac00 \uc911\uc885\uc758 \uacc4\ube44\ub85c \ub4e4\uc5b4\uac04 \ub4a4, \uc911\uc885 \ub54c \ubb38\uacfc\uc5d0 \uae09\uc81c\ud558\uc600\ub2e4. \uc774\ud6c4 \uc0ac\uad00\uc744 \uac70\uccd0 1538\ub144 \uad50\ub9ac\ub85c \uc2dc\ub3c5\uad00\uc744 \uacb8\uc9c1\ud558\uace0 \uc0ac\ud5cc\ubd80\uc9c0\ud3c9, \ud64d\ubb38\uad00\uc751\uad50 \ub4f1\uc744 \uc9c0\ub0c8\ub2e4. \ubb38\uc815\uc655\ud6c4\uac00 \uacbd\uc6d0\ub300\uad70(\ud6c4\uc758 \uba85\uc885)\uc744 \ub0b3\uc790 \ub2f9\uc2dc\uc758 \uad8c\uc138\uac00\uc778 \uae40\uc548\ub85c \ub97c \uc81c\uac70\ud558\uc600\ub2e4. \ub610\ud55c \uc778\uc885\uc758 \uc678\uc0bc\ucd0c \uc724\uc784\uc774 \uacbd\uc6d0\ub300\uad70\uc744 \ud574\ud558\uace0\uc790 \ud55c\ub2e4\uace0 \ubaa8\ud568\ud558\uc5ec \uc11c\ub85c \ub2e4\ud22c\uc5c8\ub294\ub370, \uc774\ub54c \uc724\uc784 \uc77c\ud30c\ub97c \ub300\uc724\uc774\ub77c \ud558\uace0 \uc724\uc6d0\ud615 \uc77c\ud30c\ub97c \uc18c\uc724\uc774\ub77c\uace0 \ud588\ub2e4. \uc724\uc6d0\ud615\uacfc \uc724\uc784\uc740 \uce5c\ucc99\uc774\uc5c8\ub294\ub370, \uc724\uc6d0\ud615\uc758 5\ub300\uc870 \uc724\ubc88\uc758 \ub538\uc774 \uc138\uc870\ube44 \uc815\ud76c\uc655\ud6c4\ub85c, \uc815\ud76c\uc655\ud6c4\uc758 \ub0a8\ub3d9\uc0dd \uc724\uc0ac\ud754\uc758 4\ub300\uc190\uc774\uc5c8\ub2e4. \uc724\uc784\uc740 \uc724\ubc88\uc758 \ucc28\ub0a8 \uc724\uc0ac\uc724\uc758 \ud6c4\uc190\uc73c\ub85c \uc724\uc6d0\ud615\uc758 \uc544\ubc84\uc9c0 \uc724\uc9c0\uc784\uacfc\ub294 8\ucd0c\uac04\uc774\uc5c8\ub2e4. \uc724\uc6d0\ud615\uc740 \uc724\ubc88\uc758 \uc14b\uc9f8 \uc544\ub4e4 \uc724\uc0ac\ud754\uc758 \uace0\uc190\uc790\uc600\ub2e4.", "sentence_answer": "\ubb38\uc815\uc655\ud6c4\uac00 \uacbd\uc6d0\ub300\uad70(\ud6c4\uc758 \uba85\uc885)\uc744 \ub0b3\uc790 \ub2f9\uc2dc\uc758 \uad8c\uc138\uac00\uc778 \uae40\uc548\ub85c \ub97c \uc81c\uac70\ud558\uc600\ub2e4.", "paragraph_id": "6544095-1-1", "paragraph_question": "question: \uc724\uc6d0\ud615\uc740 \ubb38\uc815\uc655\ud6c4\uac00 \uacbd\uc6d0\ub300\uad70\uc744 \ub0b3\uc790 \uc81c\uac70\ud55c \uac83\uc740 \ub204\uad6c\uc778\uac00?, context: \uc911\uc885\uc758 \uc655\ube44\uc778 \ubb38\uc815\uc655\ud6c4\uc758 \ub3d9\uc0dd\uc73c\ub85c \ub204\ub098\uc778 \ubb38\uc815\uc655\ud6c4\uac00 \uc911\uc885\uc758 \uacc4\ube44\ub85c \ub4e4\uc5b4\uac04 \ub4a4, \uc911\uc885 \ub54c \ubb38\uacfc\uc5d0 \uae09\uc81c\ud558\uc600\ub2e4. \uc774\ud6c4 \uc0ac\uad00\uc744 \uac70\uccd0 1538\ub144 \uad50\ub9ac\ub85c \uc2dc\ub3c5\uad00\uc744 \uacb8\uc9c1\ud558\uace0 \uc0ac\ud5cc\ubd80\uc9c0\ud3c9, \ud64d\ubb38\uad00\uc751\uad50 \ub4f1\uc744 \uc9c0\ub0c8\ub2e4. \ubb38\uc815\uc655\ud6c4\uac00 \uacbd\uc6d0\ub300\uad70(\ud6c4\uc758 \uba85\uc885)\uc744 \ub0b3\uc790 \ub2f9\uc2dc\uc758 \uad8c\uc138\uac00\uc778 \uae40\uc548\ub85c\ub97c \uc81c\uac70\ud558\uc600\ub2e4. \ub610\ud55c \uc778\uc885\uc758 \uc678\uc0bc\ucd0c \uc724\uc784\uc774 \uacbd\uc6d0\ub300\uad70\uc744 \ud574\ud558\uace0\uc790 \ud55c\ub2e4\uace0 \ubaa8\ud568\ud558\uc5ec \uc11c\ub85c \ub2e4\ud22c\uc5c8\ub294\ub370, \uc774\ub54c \uc724\uc784 \uc77c\ud30c\ub97c \ub300\uc724\uc774\ub77c \ud558\uace0 \uc724\uc6d0\ud615 \uc77c\ud30c\ub97c \uc18c\uc724\uc774\ub77c\uace0 \ud588\ub2e4. \uc724\uc6d0\ud615\uacfc \uc724\uc784\uc740 \uce5c\ucc99\uc774\uc5c8\ub294\ub370, \uc724\uc6d0\ud615\uc758 5\ub300\uc870 \uc724\ubc88\uc758 \ub538\uc774 \uc138\uc870\ube44 \uc815\ud76c\uc655\ud6c4\ub85c, \uc815\ud76c\uc655\ud6c4\uc758 \ub0a8\ub3d9\uc0dd \uc724\uc0ac\ud754\uc758 4\ub300\uc190\uc774\uc5c8\ub2e4. \uc724\uc784\uc740 \uc724\ubc88\uc758 \ucc28\ub0a8 \uc724\uc0ac\uc724\uc758 \ud6c4\uc190\uc73c\ub85c \uc724\uc6d0\ud615\uc758 \uc544\ubc84\uc9c0 \uc724\uc9c0\uc784\uacfc\ub294 8\ucd0c\uac04\uc774\uc5c8\ub2e4. \uc724\uc6d0\ud615\uc740 \uc724\ubc88\uc758 \uc14b\uc9f8 \uc544\ub4e4 \uc724\uc0ac\ud754\uc758 \uace0\uc190\uc790\uc600\ub2e4."} {"question": "\uc77c\ubcf8\uacc4 \ube0c\ub77c\uc9c8\uc778\ub4e4\uc740 \ud2b9\uc720\uc758 \uc774\uac83\uacfc \ub192\uc740 \uad50\uc721\uc5f4 \ub355\ubd84\uc5d0 \uc0ac\ud68c\uc801 \uc9c0\uc704\uac00 \ub192\uc740 \uc9c1\uc5c5\uc744 \uac00\uc9c4 \uc0ac\ub78c\ub4e4\uc774 \ub9ce\ub2e4. \uc774\uac83\uc740 \ubb34\uc5c7\uc778\uac00?", "paragraph": "\uc774\ubbfc \uc2dc\uc791 \uc774\ud6c4 \ube0c\ub77c\uc9c8\ub85c \uc774\uc8fc\ud574 \uac04 \uc77c\ubcf8\uc778\ub4e4\uc758 \uc804\uccb4 \ud569\uacc4\ub294 25\ub9cc \uba85\uc5d0 \uc774\ub974\ub800\uace0, \uadf8 \uc790\uc190\uc778 2\uc138, 3\uc138, 4\uc138\uac00 \ubd88\uc5b4\ub098\uba74\uc11c \ud604\uc7ac 150\ub9cc \uba85 \uc774\uc0c1\uc778 \uac83\uc73c\ub85c \ucd94\uc0b0\ub418\uace0 \uc788\uc73c\uba70, \ube0c\ub77c\uc9c8\uc758 \ub3d9\uc591\uacc4 \uc8fc\ubbfc \uc911 \ub2e4\uc218\ub97c \ucc28\uc9c0\ud558\uace0 \uc788\ub2e4. \uc774\ub4e4 \uc77c\ubcf8\uacc4 \ube0c\ub77c\uc9c8\uc778\ub4e4\uc740 \ud2b9\uc720\uc758 \uadfc\uba74\ud568\uacfc \ub192\uc740 \uad50\uc721\uc5f4\ub85c \uad6d\ud68c\uc758\uc6d0\uc774\ub098 \uc758\uc0ac, \ub18d\uc7a5\uc8fc\ub4f1 \uc0ac\ud68c\uc801 \uc9c0\uc704\uac00 \ub192\uc740 \uc9c1\uc5c5\uc5d0 \uc885\uc0ac\ud558\ub294 \uc0ac\ub78c\uc774 \ub9ce\ub2e4. \uc77c\ubcf8\uacc4 \ube0c\ub77c\uc9c8\uc778\uc740 \uc5ec\ub7ec \uc138\ub300\uac00 \uc9c0\ub098\uba74\uc11c \ud0c0 \uc778\uc885\uacfc\uc758 \ud63c\ud608\ub3c4 \ub298\uc5b4\ub098\uace0 \uc788\uc73c\uba70, \ucd08\uae30\uc5d0\ub294 \uc0c1\ud30c\uc6b8\ub8e8 \uc8fc\ubcc0\uc5d0 \uc9d1\uc911\ub418\uc5b4 \uc788\uc5c8\uc73c\ub098, \uc774\ud6c4 \ud30c\ub77c\ub098 \uc8fc\uc5d0\ub3c4 \ub9ce\uc774 \uc815\ucc29\ud588\uc73c\uba70, \ucd5c\uadfc\uc5d0\ub294 \ube0c\ub77c\uc9c8 \uc804\uad6d \uac01\uc9c0\ub85c\ub3c4 \uc9c4\ucd9c\ud558\uace0 \uc788\ub2e4. 2008\ub144 \uc77c\ubcf8\uc758 \ube0c\ub77c\uc9c8 \uc774\ubbfc \uc5ed\uc0ac\ub294 100\ub144\uc744 \ub9de\uc774\ud558\uc5ec, \ube0c\ub77c\uc9c8 \uc774\ubbfc 100\uc8fc\ub144\uc744 \uae30\ub150\ud558\ub294 \ud589\uc0ac\uac00 \uc131\ub300\ud558\uac8c \uc5f4\ub9ac\uace0 \uc788\ub2e4.", "answer": "\uadfc\uba74\ud568", "sentence": "\uc774\ub4e4 \uc77c\ubcf8\uacc4 \ube0c\ub77c\uc9c8\uc778\ub4e4\uc740 \ud2b9\uc720\uc758 \uadfc\uba74\ud568 \uacfc \ub192\uc740 \uad50\uc721\uc5f4\ub85c \uad6d\ud68c\uc758\uc6d0\uc774\ub098 \uc758\uc0ac, \ub18d\uc7a5\uc8fc\ub4f1 \uc0ac\ud68c\uc801 \uc9c0\uc704\uac00 \ub192\uc740 \uc9c1\uc5c5\uc5d0 \uc885\uc0ac\ud558\ub294 \uc0ac\ub78c\uc774 \ub9ce\ub2e4.", "paragraph_sentence": "\uc774\ubbfc \uc2dc\uc791 \uc774\ud6c4 \ube0c\ub77c\uc9c8\ub85c \uc774\uc8fc\ud574 \uac04 \uc77c\ubcf8\uc778\ub4e4\uc758 \uc804\uccb4 \ud569\uacc4\ub294 25\ub9cc \uba85\uc5d0 \uc774\ub974\ub800\uace0, \uadf8 \uc790\uc190\uc778 2\uc138, 3\uc138, 4\uc138\uac00 \ubd88\uc5b4\ub098\uba74\uc11c \ud604\uc7ac 150\ub9cc \uba85 \uc774\uc0c1\uc778 \uac83\uc73c\ub85c \ucd94\uc0b0\ub418\uace0 \uc788\uc73c\uba70, \ube0c\ub77c\uc9c8\uc758 \ub3d9\uc591\uacc4 \uc8fc\ubbfc \uc911 \ub2e4\uc218\ub97c \ucc28\uc9c0\ud558\uace0 \uc788\ub2e4. \uc774\ub4e4 \uc77c\ubcf8\uacc4 \ube0c\ub77c\uc9c8\uc778\ub4e4\uc740 \ud2b9\uc720\uc758 \uadfc\uba74\ud568 \uacfc \ub192\uc740 \uad50\uc721\uc5f4\ub85c \uad6d\ud68c\uc758\uc6d0\uc774\ub098 \uc758\uc0ac, \ub18d\uc7a5\uc8fc\ub4f1 \uc0ac\ud68c\uc801 \uc9c0\uc704\uac00 \ub192\uc740 \uc9c1\uc5c5\uc5d0 \uc885\uc0ac\ud558\ub294 \uc0ac\ub78c\uc774 \ub9ce\ub2e4. \uc77c\ubcf8\uacc4 \ube0c\ub77c\uc9c8\uc778\uc740 \uc5ec\ub7ec \uc138\ub300\uac00 \uc9c0\ub098\uba74\uc11c \ud0c0 \uc778\uc885\uacfc\uc758 \ud63c\ud608\ub3c4 \ub298\uc5b4\ub098\uace0 \uc788\uc73c\uba70, \ucd08\uae30\uc5d0\ub294 \uc0c1\ud30c\uc6b8\ub8e8 \uc8fc\ubcc0\uc5d0 \uc9d1\uc911\ub418\uc5b4 \uc788\uc5c8\uc73c\ub098, \uc774\ud6c4 \ud30c\ub77c\ub098 \uc8fc\uc5d0\ub3c4 \ub9ce\uc774 \uc815\ucc29\ud588\uc73c\uba70, \ucd5c\uadfc\uc5d0\ub294 \ube0c\ub77c\uc9c8 \uc804\uad6d \uac01\uc9c0\ub85c\ub3c4 \uc9c4\ucd9c\ud558\uace0 \uc788\ub2e4. 2008\ub144 \uc77c\ubcf8\uc758 \ube0c\ub77c\uc9c8 \uc774\ubbfc \uc5ed\uc0ac\ub294 100\ub144\uc744 \ub9de\uc774\ud558\uc5ec, \ube0c\ub77c\uc9c8 \uc774\ubbfc 100\uc8fc\ub144\uc744 \uae30\ub150\ud558\ub294 \ud589\uc0ac\uac00 \uc131\ub300\ud558\uac8c \uc5f4\ub9ac\uace0 \uc788\ub2e4.", "paragraph_answer": "\uc774\ubbfc \uc2dc\uc791 \uc774\ud6c4 \ube0c\ub77c\uc9c8\ub85c \uc774\uc8fc\ud574 \uac04 \uc77c\ubcf8\uc778\ub4e4\uc758 \uc804\uccb4 \ud569\uacc4\ub294 25\ub9cc \uba85\uc5d0 \uc774\ub974\ub800\uace0, \uadf8 \uc790\uc190\uc778 2\uc138, 3\uc138, 4\uc138\uac00 \ubd88\uc5b4\ub098\uba74\uc11c \ud604\uc7ac 150\ub9cc \uba85 \uc774\uc0c1\uc778 \uac83\uc73c\ub85c \ucd94\uc0b0\ub418\uace0 \uc788\uc73c\uba70, \ube0c\ub77c\uc9c8\uc758 \ub3d9\uc591\uacc4 \uc8fc\ubbfc \uc911 \ub2e4\uc218\ub97c \ucc28\uc9c0\ud558\uace0 \uc788\ub2e4. \uc774\ub4e4 \uc77c\ubcf8\uacc4 \ube0c\ub77c\uc9c8\uc778\ub4e4\uc740 \ud2b9\uc720\uc758 \uadfc\uba74\ud568 \uacfc \ub192\uc740 \uad50\uc721\uc5f4\ub85c \uad6d\ud68c\uc758\uc6d0\uc774\ub098 \uc758\uc0ac, \ub18d\uc7a5\uc8fc\ub4f1 \uc0ac\ud68c\uc801 \uc9c0\uc704\uac00 \ub192\uc740 \uc9c1\uc5c5\uc5d0 \uc885\uc0ac\ud558\ub294 \uc0ac\ub78c\uc774 \ub9ce\ub2e4. \uc77c\ubcf8\uacc4 \ube0c\ub77c\uc9c8\uc778\uc740 \uc5ec\ub7ec \uc138\ub300\uac00 \uc9c0\ub098\uba74\uc11c \ud0c0 \uc778\uc885\uacfc\uc758 \ud63c\ud608\ub3c4 \ub298\uc5b4\ub098\uace0 \uc788\uc73c\uba70, \ucd08\uae30\uc5d0\ub294 \uc0c1\ud30c\uc6b8\ub8e8 \uc8fc\ubcc0\uc5d0 \uc9d1\uc911\ub418\uc5b4 \uc788\uc5c8\uc73c\ub098, \uc774\ud6c4 \ud30c\ub77c\ub098 \uc8fc\uc5d0\ub3c4 \ub9ce\uc774 \uc815\ucc29\ud588\uc73c\uba70, \ucd5c\uadfc\uc5d0\ub294 \ube0c\ub77c\uc9c8 \uc804\uad6d \uac01\uc9c0\ub85c\ub3c4 \uc9c4\ucd9c\ud558\uace0 \uc788\ub2e4. 2008\ub144 \uc77c\ubcf8\uc758 \ube0c\ub77c\uc9c8 \uc774\ubbfc \uc5ed\uc0ac\ub294 100\ub144\uc744 \ub9de\uc774\ud558\uc5ec, \ube0c\ub77c\uc9c8 \uc774\ubbfc 100\uc8fc\ub144\uc744 \uae30\ub150\ud558\ub294 \ud589\uc0ac\uac00 \uc131\ub300\ud558\uac8c \uc5f4\ub9ac\uace0 \uc788\ub2e4.", "sentence_answer": "\uc774\ub4e4 \uc77c\ubcf8\uacc4 \ube0c\ub77c\uc9c8\uc778\ub4e4\uc740 \ud2b9\uc720\uc758 \uadfc\uba74\ud568 \uacfc \ub192\uc740 \uad50\uc721\uc5f4\ub85c \uad6d\ud68c\uc758\uc6d0\uc774\ub098 \uc758\uc0ac, \ub18d\uc7a5\uc8fc\ub4f1 \uc0ac\ud68c\uc801 \uc9c0\uc704\uac00 \ub192\uc740 \uc9c1\uc5c5\uc5d0 \uc885\uc0ac\ud558\ub294 \uc0ac\ub78c\uc774 \ub9ce\ub2e4.", "paragraph_id": "5854615-1-1", "paragraph_question": "question: \uc77c\ubcf8\uacc4 \ube0c\ub77c\uc9c8\uc778\ub4e4\uc740 \ud2b9\uc720\uc758 \uc774\uac83\uacfc \ub192\uc740 \uad50\uc721\uc5f4 \ub355\ubd84\uc5d0 \uc0ac\ud68c\uc801 \uc9c0\uc704\uac00 \ub192\uc740 \uc9c1\uc5c5\uc744 \uac00\uc9c4 \uc0ac\ub78c\ub4e4\uc774 \ub9ce\ub2e4. \uc774\uac83\uc740 \ubb34\uc5c7\uc778\uac00?, context: \uc774\ubbfc \uc2dc\uc791 \uc774\ud6c4 \ube0c\ub77c\uc9c8\ub85c \uc774\uc8fc\ud574 \uac04 \uc77c\ubcf8\uc778\ub4e4\uc758 \uc804\uccb4 \ud569\uacc4\ub294 25\ub9cc \uba85\uc5d0 \uc774\ub974\ub800\uace0, \uadf8 \uc790\uc190\uc778 2\uc138, 3\uc138, 4\uc138\uac00 \ubd88\uc5b4\ub098\uba74\uc11c \ud604\uc7ac 150\ub9cc \uba85 \uc774\uc0c1\uc778 \uac83\uc73c\ub85c \ucd94\uc0b0\ub418\uace0 \uc788\uc73c\uba70, \ube0c\ub77c\uc9c8\uc758 \ub3d9\uc591\uacc4 \uc8fc\ubbfc \uc911 \ub2e4\uc218\ub97c \ucc28\uc9c0\ud558\uace0 \uc788\ub2e4. \uc774\ub4e4 \uc77c\ubcf8\uacc4 \ube0c\ub77c\uc9c8\uc778\ub4e4\uc740 \ud2b9\uc720\uc758 \uadfc\uba74\ud568\uacfc \ub192\uc740 \uad50\uc721\uc5f4\ub85c \uad6d\ud68c\uc758\uc6d0\uc774\ub098 \uc758\uc0ac, \ub18d\uc7a5\uc8fc\ub4f1 \uc0ac\ud68c\uc801 \uc9c0\uc704\uac00 \ub192\uc740 \uc9c1\uc5c5\uc5d0 \uc885\uc0ac\ud558\ub294 \uc0ac\ub78c\uc774 \ub9ce\ub2e4. \uc77c\ubcf8\uacc4 \ube0c\ub77c\uc9c8\uc778\uc740 \uc5ec\ub7ec \uc138\ub300\uac00 \uc9c0\ub098\uba74\uc11c \ud0c0 \uc778\uc885\uacfc\uc758 \ud63c\ud608\ub3c4 \ub298\uc5b4\ub098\uace0 \uc788\uc73c\uba70, \ucd08\uae30\uc5d0\ub294 \uc0c1\ud30c\uc6b8\ub8e8 \uc8fc\ubcc0\uc5d0 \uc9d1\uc911\ub418\uc5b4 \uc788\uc5c8\uc73c\ub098, \uc774\ud6c4 \ud30c\ub77c\ub098 \uc8fc\uc5d0\ub3c4 \ub9ce\uc774 \uc815\ucc29\ud588\uc73c\uba70, \ucd5c\uadfc\uc5d0\ub294 \ube0c\ub77c\uc9c8 \uc804\uad6d \uac01\uc9c0\ub85c\ub3c4 \uc9c4\ucd9c\ud558\uace0 \uc788\ub2e4. 2008\ub144 \uc77c\ubcf8\uc758 \ube0c\ub77c\uc9c8 \uc774\ubbfc \uc5ed\uc0ac\ub294 100\ub144\uc744 \ub9de\uc774\ud558\uc5ec, \ube0c\ub77c\uc9c8 \uc774\ubbfc 100\uc8fc\ub144\uc744 \uae30\ub150\ud558\ub294 \ud589\uc0ac\uac00 \uc131\ub300\ud558\uac8c \uc5f4\ub9ac\uace0 \uc788\ub2e4."} {"question": "\uc6b0\uc2b9\ud558\uc9c0 \ubabb\ud558\uc5ec \uae30\ubd84\uc774 \ub098\ube60\uc9c4 \uc544\ub974\ud5e8\ud2f0\ub098 \uc0ac\ub78c\ub4e4\uc774 \ub3cc\uc744 \ub358\uc9c4 \uacf3\uc740?", "paragraph": "\ud648\ud300\uc778 \uc6b0\ub8e8\uacfc\uc774\uac00 \ud30c\ube14\ub85c \ub3c4\ub77c\ub3c4\uac00 \uace8\ub300 \uc624\ub978 \ud3b8\uc5d0\uc11c \ub0ae\uac8c \uae54\uc544 \ucc2c \uc29b\uc73c\ub85c \uae30\ub85d\ud55c \uc120\uc81c\uace8\ub85c \uc55e\uc11c\ub098\uac00\uae30 \uc2dc\uc791\ud588\ub2e4. \uc544\ub974\ud5e8\ud2f0\ub098\ub294 \ub6f0\uc5b4\ub09c \ud328\uc2f1 \uac8c\uc784\uc744 \ud3bc\uce58\uba70 \uc6b0\ub8e8\uacfc\uc774\uc5d0 \uac15\ud558\uac8c \ub9de\uc130\ub2e4. \uadf8\ub9ac\uace4 8\ubd84 \ub9cc\uc5d0 \uce74\ub97c\ub85c\uc2a4 \ud398\uc6b0\uc140\ub808\uac00 \ud398\ub808\uc774\ub77c\uc758 \uc2a4\ub8e8\ud328\uc2a4\ub97c \ubc1b\uc544 \uc218\ube44\uc218\ub97c \ub530\ub3cc\ub9ac\uace0 \ub3d9\uc810\uace8\uc744 \uae30\ub85d\ud588\ub2e4. \uc804\ubc18\uc804\uc774 \ub05d\ub098\uae30 \uc9c1\uc804 \ub2f9\uc2dc \ub300\ud68c \ub4dd\uc810 \uc120\ub450\uc600\ub358 \uae38\ub808\ub974\ubaa8 \uc2a4\ud0c0\ube4c\ub808\uac00 \uc5ed\uc804\uace8\uc744 \ub123\uc5b4 \uc544\ub974\ud5e8\ud2f0\ub098\uac00 2:1\ub85c \uc55e\uc11c\uac00\uae30 \uc2dc\uc791\ud588\ub2e4. \uc6b0\ub8e8\uacfc\uc774\uc758 \uc8fc\uc7a5\uc778 \ud638\uc138 \ub098\uc0ac\uc2dc\uac00 \uc2a4\ud0c0\ube4c\ub808\uac00 \uc624\ud504\uc0ac\uc774\ub4dc \uc704\uce58\uc5d0 \uc788\uc5c8\ub2e4\uba70 \uc2ec\ud310\uc5d0\uac8c \ud56d\uc758\ud588\uc73c\ub098 \ubc1b\uc544\ub4e4\uc5ec\uc9c0\uc9c0 \uc54a\uc558\ub2e4. \ud6c4\ubc18\uc804\uc5d0\ub294 \uc6b0\ub8e8\uacfc\uc758\uc758 \uacbd\uae30\ub825\uc774 \uc810\ucc28 \uc62c\ub77c\uc624\uae30 \uc2dc\uc791\ud588\ub2e4. \uc544\ub974\ud5e8\ud2f0\ub098\uc758 \ubaac\ud2f0\uac00 \ub450 \uace8\ucc28\ub85c \uc55e\uc11c\uac08 \uc808\ud638\uc758 \uae30\ud68c\ub97c \ub193\uce58\uc790 \ub9c8\uc790 \uc6b0\ub8e8\uacfc\uc774\uac00 \uc5ed\uc2b5\uc5d0 \ub098\uc130\uace0, \ud398\ub4dc\ub85c \uc138\uc544\uac00 \ub3d9\uc810\uace8\uc744 \uae30\ub85d\ud574 \uc2b9\ubd80\ub294 \uc6d0\uc810\uc73c\ub85c \ub3cc\uc544\uac14\ub2e4. 10\ubd84 \ud6c4\uc5d0\ub294 \uc6b0\ub8e8\uacfc\uc774\uc758 \uc0b0\ud1a0\uc2a4 \uc774\ub9ac\uc544\ub974\ud14c\uac00 \uc5ed\uc804\uace8\uc744 \ubf51\uc544\ub0c8\uace0, \uacbd\uae30 \uc885\ub8cc \uc9c1\uc804\uc5d0\ub294 \uce74\uc2a4\ud2b8\ub85c\uac00 \uc410\uae30\ub97c \ubc15\ub294 \uace8\uc744 \uc131\uacf5\uc2dc\ucf1c \uacb0\uad6d \uacbd\uae30\ub294 4:2\ub85c \uc885\ub8cc\ub418\uc5c8\ub2e4. \ub791\uac8c\ub258 \uc8fc\uc2ec\uc740 1\ubd84\uc758 \ucd94\uac00\uc2dc\uac04\uc774 \uc9c0\ub09c \ud6c4\uc5d0 \ud718\uc2ac\uc744 \ubd88\uc5c8\uace0, \uc6b0\ub8e8\uacfc\uc774\ub294 \uc62c\ub9bc\ud53d\uc5d0\uc11c\uc758 \uc6b0\uc2b9\uc5d0 \uc774\uc5b4 \uccab \uc6d4\ub4dc\ucef5 \uc6b0\uc2b9 \ud300\uc774\ub77c\ub294 \uc601\uad11\uc744 \uc548\uc558\ub2e4. \uc958 \ub9ac\uba54\ub294 \uc790\uc2e0\uc758 \uc774\ub984\uc744 \ub534 \uc6d4\ub4dc\ucef5 \ud2b8\ub85c\ud53c\uc778 \uc958 \ub9ac\uba54 \ucef5\uc744 \uc6b0\ub8e8\uacfc\uc774 \ucd95\uad6c \ud611\ud68c \ud68c\uc7a5 \ub77c\uc6b8 \ud6c4\ub370\uc5d0\uac8c \uc804\ub2ec\ud588\ub2e4. \uc6b0\ub8e8\uacfc\uc774 \uc815\ubd80\ub294 \uc6b0\uc2b9\uc5d0 \ub300\ud55c \ud658\ud76c\ub85c \uacbd\uae30 \ub2e4\uc74c\ub0a0\uc778 7\uc6d4 31\uc77c\uc744 \uad6d\uac00 \uacf5\ud734\uc77c\ub85c \uc9c0\uc815\ud588\ub2e4. \ud55c\ud3b8 \uc900\uc6b0\uc2b9\uc5d0 \uadf8\uce5c \uc544\ub974\ud5e8\ud2f0\ub098\uc758 \uc218\ub3c4 \ubd80\uc5d0\ub178\uc2a4 \uc544\uc774\ub808\uc2a4\uc5d0\uc11c\ub294 \uc6b0\ub8e8\uacfc\uc774 \uc601\uc0ac\uad00\uc744 \ud5a5\ud574 \uad70\uc911\ub4e4\uc774 \ub3cc\uc744 \ub358\uc84c\ub2e4.", "answer": "\uc6b0\ub8e8\uacfc\uc774 \uc601\uc0ac\uad00", "sentence": "\ud55c\ud3b8 \uc900\uc6b0\uc2b9\uc5d0 \uadf8\uce5c \uc544\ub974\ud5e8\ud2f0\ub098\uc758 \uc218\ub3c4 \ubd80\uc5d0\ub178\uc2a4 \uc544\uc774\ub808\uc2a4\uc5d0\uc11c\ub294 \uc6b0\ub8e8\uacfc\uc774 \uc601\uc0ac\uad00 \uc744 \ud5a5\ud574 \uad70\uc911\ub4e4\uc774 \ub3cc\uc744 \ub358\uc84c\ub2e4.", "paragraph_sentence": "\ud648\ud300\uc778 \uc6b0\ub8e8\uacfc\uc774\uac00 \ud30c\ube14\ub85c \ub3c4\ub77c\ub3c4\uac00 \uace8\ub300 \uc624\ub978 \ud3b8\uc5d0\uc11c \ub0ae\uac8c \uae54\uc544 \ucc2c \uc29b\uc73c\ub85c \uae30\ub85d\ud55c \uc120\uc81c\uace8\ub85c \uc55e\uc11c\ub098\uac00\uae30 \uc2dc\uc791\ud588\ub2e4. \uc544\ub974\ud5e8\ud2f0\ub098\ub294 \ub6f0\uc5b4\ub09c \ud328\uc2f1 \uac8c\uc784\uc744 \ud3bc\uce58\uba70 \uc6b0\ub8e8\uacfc\uc774\uc5d0 \uac15\ud558\uac8c \ub9de\uc130\ub2e4. \uadf8\ub9ac\uace4 8\ubd84 \ub9cc\uc5d0 \uce74\ub97c\ub85c\uc2a4 \ud398\uc6b0\uc140\ub808\uac00 \ud398\ub808\uc774\ub77c\uc758 \uc2a4\ub8e8\ud328\uc2a4\ub97c \ubc1b\uc544 \uc218\ube44\uc218\ub97c \ub530\ub3cc\ub9ac\uace0 \ub3d9\uc810\uace8\uc744 \uae30\ub85d\ud588\ub2e4. \uc804\ubc18\uc804\uc774 \ub05d\ub098\uae30 \uc9c1\uc804 \ub2f9\uc2dc \ub300\ud68c \ub4dd\uc810 \uc120\ub450\uc600\ub358 \uae38\ub808\ub974\ubaa8 \uc2a4\ud0c0\ube4c\ub808\uac00 \uc5ed\uc804\uace8\uc744 \ub123\uc5b4 \uc544\ub974\ud5e8\ud2f0\ub098\uac00 2:1\ub85c \uc55e\uc11c\uac00\uae30 \uc2dc\uc791\ud588\ub2e4. \uc6b0\ub8e8\uacfc\uc774\uc758 \uc8fc\uc7a5\uc778 \ud638\uc138 \ub098\uc0ac\uc2dc\uac00 \uc2a4\ud0c0\ube4c\ub808\uac00 \uc624\ud504\uc0ac\uc774\ub4dc \uc704\uce58\uc5d0 \uc788\uc5c8\ub2e4\uba70 \uc2ec\ud310\uc5d0\uac8c \ud56d\uc758\ud588\uc73c\ub098 \ubc1b\uc544\ub4e4\uc5ec\uc9c0\uc9c0 \uc54a\uc558\ub2e4. \ud6c4\ubc18\uc804\uc5d0\ub294 \uc6b0\ub8e8\uacfc\uc758\uc758 \uacbd\uae30\ub825\uc774 \uc810\ucc28 \uc62c\ub77c\uc624\uae30 \uc2dc\uc791\ud588\ub2e4. \uc544\ub974\ud5e8\ud2f0\ub098\uc758 \ubaac\ud2f0\uac00 \ub450 \uace8\ucc28\ub85c \uc55e\uc11c\uac08 \uc808\ud638\uc758 \uae30\ud68c\ub97c \ub193\uce58\uc790 \ub9c8\uc790 \uc6b0\ub8e8\uacfc\uc774\uac00 \uc5ed\uc2b5\uc5d0 \ub098\uc130\uace0, \ud398\ub4dc\ub85c \uc138\uc544\uac00 \ub3d9\uc810\uace8\uc744 \uae30\ub85d\ud574 \uc2b9\ubd80\ub294 \uc6d0\uc810\uc73c\ub85c \ub3cc\uc544\uac14\ub2e4. 10\ubd84 \ud6c4\uc5d0\ub294 \uc6b0\ub8e8\uacfc\uc774\uc758 \uc0b0\ud1a0\uc2a4 \uc774\ub9ac\uc544\ub974\ud14c\uac00 \uc5ed\uc804\uace8\uc744 \ubf51\uc544\ub0c8\uace0, \uacbd\uae30 \uc885\ub8cc \uc9c1\uc804\uc5d0\ub294 \uce74\uc2a4\ud2b8\ub85c\uac00 \uc410\uae30\ub97c \ubc15\ub294 \uace8\uc744 \uc131\uacf5\uc2dc\ucf1c \uacb0\uad6d \uacbd\uae30\ub294 4:2\ub85c \uc885\ub8cc\ub418\uc5c8\ub2e4. \ub791\uac8c\ub258 \uc8fc\uc2ec\uc740 1\ubd84\uc758 \ucd94\uac00\uc2dc\uac04\uc774 \uc9c0\ub09c \ud6c4\uc5d0 \ud718\uc2ac\uc744 \ubd88\uc5c8\uace0, \uc6b0\ub8e8\uacfc\uc774\ub294 \uc62c\ub9bc\ud53d\uc5d0\uc11c\uc758 \uc6b0\uc2b9\uc5d0 \uc774\uc5b4 \uccab \uc6d4\ub4dc\ucef5 \uc6b0\uc2b9 \ud300\uc774\ub77c\ub294 \uc601\uad11\uc744 \uc548\uc558\ub2e4. \uc958 \ub9ac\uba54\ub294 \uc790\uc2e0\uc758 \uc774\ub984\uc744 \ub534 \uc6d4\ub4dc\ucef5 \ud2b8\ub85c\ud53c\uc778 \uc958 \ub9ac\uba54 \ucef5\uc744 \uc6b0\ub8e8\uacfc\uc774 \ucd95\uad6c \ud611\ud68c \ud68c\uc7a5 \ub77c\uc6b8 \ud6c4\ub370\uc5d0\uac8c \uc804\ub2ec\ud588\ub2e4. \uc6b0\ub8e8\uacfc\uc774 \uc815\ubd80\ub294 \uc6b0\uc2b9\uc5d0 \ub300\ud55c \ud658\ud76c\ub85c \uacbd\uae30 \ub2e4\uc74c\ub0a0\uc778 7\uc6d4 31\uc77c\uc744 \uad6d\uac00 \uacf5\ud734\uc77c\ub85c \uc9c0\uc815\ud588\ub2e4. \ud55c\ud3b8 \uc900\uc6b0\uc2b9\uc5d0 \uadf8\uce5c \uc544\ub974\ud5e8\ud2f0\ub098\uc758 \uc218\ub3c4 \ubd80\uc5d0\ub178\uc2a4 \uc544\uc774\ub808\uc2a4\uc5d0\uc11c\ub294 \uc6b0\ub8e8\uacfc\uc774 \uc601\uc0ac\uad00 \uc744 \ud5a5\ud574 \uad70\uc911\ub4e4\uc774 \ub3cc\uc744 \ub358\uc84c\ub2e4. ", "paragraph_answer": "\ud648\ud300\uc778 \uc6b0\ub8e8\uacfc\uc774\uac00 \ud30c\ube14\ub85c \ub3c4\ub77c\ub3c4\uac00 \uace8\ub300 \uc624\ub978 \ud3b8\uc5d0\uc11c \ub0ae\uac8c \uae54\uc544 \ucc2c \uc29b\uc73c\ub85c \uae30\ub85d\ud55c \uc120\uc81c\uace8\ub85c \uc55e\uc11c\ub098\uac00\uae30 \uc2dc\uc791\ud588\ub2e4. \uc544\ub974\ud5e8\ud2f0\ub098\ub294 \ub6f0\uc5b4\ub09c \ud328\uc2f1 \uac8c\uc784\uc744 \ud3bc\uce58\uba70 \uc6b0\ub8e8\uacfc\uc774\uc5d0 \uac15\ud558\uac8c \ub9de\uc130\ub2e4. \uadf8\ub9ac\uace4 8\ubd84 \ub9cc\uc5d0 \uce74\ub97c\ub85c\uc2a4 \ud398\uc6b0\uc140\ub808\uac00 \ud398\ub808\uc774\ub77c\uc758 \uc2a4\ub8e8\ud328\uc2a4\ub97c \ubc1b\uc544 \uc218\ube44\uc218\ub97c \ub530\ub3cc\ub9ac\uace0 \ub3d9\uc810\uace8\uc744 \uae30\ub85d\ud588\ub2e4. \uc804\ubc18\uc804\uc774 \ub05d\ub098\uae30 \uc9c1\uc804 \ub2f9\uc2dc \ub300\ud68c \ub4dd\uc810 \uc120\ub450\uc600\ub358 \uae38\ub808\ub974\ubaa8 \uc2a4\ud0c0\ube4c\ub808\uac00 \uc5ed\uc804\uace8\uc744 \ub123\uc5b4 \uc544\ub974\ud5e8\ud2f0\ub098\uac00 2:1\ub85c \uc55e\uc11c\uac00\uae30 \uc2dc\uc791\ud588\ub2e4. \uc6b0\ub8e8\uacfc\uc774\uc758 \uc8fc\uc7a5\uc778 \ud638\uc138 \ub098\uc0ac\uc2dc\uac00 \uc2a4\ud0c0\ube4c\ub808\uac00 \uc624\ud504\uc0ac\uc774\ub4dc \uc704\uce58\uc5d0 \uc788\uc5c8\ub2e4\uba70 \uc2ec\ud310\uc5d0\uac8c \ud56d\uc758\ud588\uc73c\ub098 \ubc1b\uc544\ub4e4\uc5ec\uc9c0\uc9c0 \uc54a\uc558\ub2e4. \ud6c4\ubc18\uc804\uc5d0\ub294 \uc6b0\ub8e8\uacfc\uc758\uc758 \uacbd\uae30\ub825\uc774 \uc810\ucc28 \uc62c\ub77c\uc624\uae30 \uc2dc\uc791\ud588\ub2e4. \uc544\ub974\ud5e8\ud2f0\ub098\uc758 \ubaac\ud2f0\uac00 \ub450 \uace8\ucc28\ub85c \uc55e\uc11c\uac08 \uc808\ud638\uc758 \uae30\ud68c\ub97c \ub193\uce58\uc790 \ub9c8\uc790 \uc6b0\ub8e8\uacfc\uc774\uac00 \uc5ed\uc2b5\uc5d0 \ub098\uc130\uace0, \ud398\ub4dc\ub85c \uc138\uc544\uac00 \ub3d9\uc810\uace8\uc744 \uae30\ub85d\ud574 \uc2b9\ubd80\ub294 \uc6d0\uc810\uc73c\ub85c \ub3cc\uc544\uac14\ub2e4. 10\ubd84 \ud6c4\uc5d0\ub294 \uc6b0\ub8e8\uacfc\uc774\uc758 \uc0b0\ud1a0\uc2a4 \uc774\ub9ac\uc544\ub974\ud14c\uac00 \uc5ed\uc804\uace8\uc744 \ubf51\uc544\ub0c8\uace0, \uacbd\uae30 \uc885\ub8cc \uc9c1\uc804\uc5d0\ub294 \uce74\uc2a4\ud2b8\ub85c\uac00 \uc410\uae30\ub97c \ubc15\ub294 \uace8\uc744 \uc131\uacf5\uc2dc\ucf1c \uacb0\uad6d \uacbd\uae30\ub294 4:2\ub85c \uc885\ub8cc\ub418\uc5c8\ub2e4. \ub791\uac8c\ub258 \uc8fc\uc2ec\uc740 1\ubd84\uc758 \ucd94\uac00\uc2dc\uac04\uc774 \uc9c0\ub09c \ud6c4\uc5d0 \ud718\uc2ac\uc744 \ubd88\uc5c8\uace0, \uc6b0\ub8e8\uacfc\uc774\ub294 \uc62c\ub9bc\ud53d\uc5d0\uc11c\uc758 \uc6b0\uc2b9\uc5d0 \uc774\uc5b4 \uccab \uc6d4\ub4dc\ucef5 \uc6b0\uc2b9 \ud300\uc774\ub77c\ub294 \uc601\uad11\uc744 \uc548\uc558\ub2e4. \uc958 \ub9ac\uba54\ub294 \uc790\uc2e0\uc758 \uc774\ub984\uc744 \ub534 \uc6d4\ub4dc\ucef5 \ud2b8\ub85c\ud53c\uc778 \uc958 \ub9ac\uba54 \ucef5\uc744 \uc6b0\ub8e8\uacfc\uc774 \ucd95\uad6c \ud611\ud68c \ud68c\uc7a5 \ub77c\uc6b8 \ud6c4\ub370\uc5d0\uac8c \uc804\ub2ec\ud588\ub2e4. \uc6b0\ub8e8\uacfc\uc774 \uc815\ubd80\ub294 \uc6b0\uc2b9\uc5d0 \ub300\ud55c \ud658\ud76c\ub85c \uacbd\uae30 \ub2e4\uc74c\ub0a0\uc778 7\uc6d4 31\uc77c\uc744 \uad6d\uac00 \uacf5\ud734\uc77c\ub85c \uc9c0\uc815\ud588\ub2e4. \ud55c\ud3b8 \uc900\uc6b0\uc2b9\uc5d0 \uadf8\uce5c \uc544\ub974\ud5e8\ud2f0\ub098\uc758 \uc218\ub3c4 \ubd80\uc5d0\ub178\uc2a4 \uc544\uc774\ub808\uc2a4\uc5d0\uc11c\ub294 \uc6b0\ub8e8\uacfc\uc774 \uc601\uc0ac\uad00 \uc744 \ud5a5\ud574 \uad70\uc911\ub4e4\uc774 \ub3cc\uc744 \ub358\uc84c\ub2e4.", "sentence_answer": "\ud55c\ud3b8 \uc900\uc6b0\uc2b9\uc5d0 \uadf8\uce5c \uc544\ub974\ud5e8\ud2f0\ub098\uc758 \uc218\ub3c4 \ubd80\uc5d0\ub178\uc2a4 \uc544\uc774\ub808\uc2a4\uc5d0\uc11c\ub294 \uc6b0\ub8e8\uacfc\uc774 \uc601\uc0ac\uad00 \uc744 \ud5a5\ud574 \uad70\uc911\ub4e4\uc774 \ub3cc\uc744 \ub358\uc84c\ub2e4.", "paragraph_id": "6508913-12-2", "paragraph_question": "question: \uc6b0\uc2b9\ud558\uc9c0 \ubabb\ud558\uc5ec \uae30\ubd84\uc774 \ub098\ube60\uc9c4 \uc544\ub974\ud5e8\ud2f0\ub098 \uc0ac\ub78c\ub4e4\uc774 \ub3cc\uc744 \ub358\uc9c4 \uacf3\uc740?, context: \ud648\ud300\uc778 \uc6b0\ub8e8\uacfc\uc774\uac00 \ud30c\ube14\ub85c \ub3c4\ub77c\ub3c4\uac00 \uace8\ub300 \uc624\ub978 \ud3b8\uc5d0\uc11c \ub0ae\uac8c \uae54\uc544 \ucc2c \uc29b\uc73c\ub85c \uae30\ub85d\ud55c \uc120\uc81c\uace8\ub85c \uc55e\uc11c\ub098\uac00\uae30 \uc2dc\uc791\ud588\ub2e4. \uc544\ub974\ud5e8\ud2f0\ub098\ub294 \ub6f0\uc5b4\ub09c \ud328\uc2f1 \uac8c\uc784\uc744 \ud3bc\uce58\uba70 \uc6b0\ub8e8\uacfc\uc774\uc5d0 \uac15\ud558\uac8c \ub9de\uc130\ub2e4. \uadf8\ub9ac\uace4 8\ubd84 \ub9cc\uc5d0 \uce74\ub97c\ub85c\uc2a4 \ud398\uc6b0\uc140\ub808\uac00 \ud398\ub808\uc774\ub77c\uc758 \uc2a4\ub8e8\ud328\uc2a4\ub97c \ubc1b\uc544 \uc218\ube44\uc218\ub97c \ub530\ub3cc\ub9ac\uace0 \ub3d9\uc810\uace8\uc744 \uae30\ub85d\ud588\ub2e4. \uc804\ubc18\uc804\uc774 \ub05d\ub098\uae30 \uc9c1\uc804 \ub2f9\uc2dc \ub300\ud68c \ub4dd\uc810 \uc120\ub450\uc600\ub358 \uae38\ub808\ub974\ubaa8 \uc2a4\ud0c0\ube4c\ub808\uac00 \uc5ed\uc804\uace8\uc744 \ub123\uc5b4 \uc544\ub974\ud5e8\ud2f0\ub098\uac00 2:1\ub85c \uc55e\uc11c\uac00\uae30 \uc2dc\uc791\ud588\ub2e4. \uc6b0\ub8e8\uacfc\uc774\uc758 \uc8fc\uc7a5\uc778 \ud638\uc138 \ub098\uc0ac\uc2dc\uac00 \uc2a4\ud0c0\ube4c\ub808\uac00 \uc624\ud504\uc0ac\uc774\ub4dc \uc704\uce58\uc5d0 \uc788\uc5c8\ub2e4\uba70 \uc2ec\ud310\uc5d0\uac8c \ud56d\uc758\ud588\uc73c\ub098 \ubc1b\uc544\ub4e4\uc5ec\uc9c0\uc9c0 \uc54a\uc558\ub2e4. \ud6c4\ubc18\uc804\uc5d0\ub294 \uc6b0\ub8e8\uacfc\uc758\uc758 \uacbd\uae30\ub825\uc774 \uc810\ucc28 \uc62c\ub77c\uc624\uae30 \uc2dc\uc791\ud588\ub2e4. \uc544\ub974\ud5e8\ud2f0\ub098\uc758 \ubaac\ud2f0\uac00 \ub450 \uace8\ucc28\ub85c \uc55e\uc11c\uac08 \uc808\ud638\uc758 \uae30\ud68c\ub97c \ub193\uce58\uc790 \ub9c8\uc790 \uc6b0\ub8e8\uacfc\uc774\uac00 \uc5ed\uc2b5\uc5d0 \ub098\uc130\uace0, \ud398\ub4dc\ub85c \uc138\uc544\uac00 \ub3d9\uc810\uace8\uc744 \uae30\ub85d\ud574 \uc2b9\ubd80\ub294 \uc6d0\uc810\uc73c\ub85c \ub3cc\uc544\uac14\ub2e4. 10\ubd84 \ud6c4\uc5d0\ub294 \uc6b0\ub8e8\uacfc\uc774\uc758 \uc0b0\ud1a0\uc2a4 \uc774\ub9ac\uc544\ub974\ud14c\uac00 \uc5ed\uc804\uace8\uc744 \ubf51\uc544\ub0c8\uace0, \uacbd\uae30 \uc885\ub8cc \uc9c1\uc804\uc5d0\ub294 \uce74\uc2a4\ud2b8\ub85c\uac00 \uc410\uae30\ub97c \ubc15\ub294 \uace8\uc744 \uc131\uacf5\uc2dc\ucf1c \uacb0\uad6d \uacbd\uae30\ub294 4:2\ub85c \uc885\ub8cc\ub418\uc5c8\ub2e4. \ub791\uac8c\ub258 \uc8fc\uc2ec\uc740 1\ubd84\uc758 \ucd94\uac00\uc2dc\uac04\uc774 \uc9c0\ub09c \ud6c4\uc5d0 \ud718\uc2ac\uc744 \ubd88\uc5c8\uace0, \uc6b0\ub8e8\uacfc\uc774\ub294 \uc62c\ub9bc\ud53d\uc5d0\uc11c\uc758 \uc6b0\uc2b9\uc5d0 \uc774\uc5b4 \uccab \uc6d4\ub4dc\ucef5 \uc6b0\uc2b9 \ud300\uc774\ub77c\ub294 \uc601\uad11\uc744 \uc548\uc558\ub2e4. \uc958 \ub9ac\uba54\ub294 \uc790\uc2e0\uc758 \uc774\ub984\uc744 \ub534 \uc6d4\ub4dc\ucef5 \ud2b8\ub85c\ud53c\uc778 \uc958 \ub9ac\uba54 \ucef5\uc744 \uc6b0\ub8e8\uacfc\uc774 \ucd95\uad6c \ud611\ud68c \ud68c\uc7a5 \ub77c\uc6b8 \ud6c4\ub370\uc5d0\uac8c \uc804\ub2ec\ud588\ub2e4. \uc6b0\ub8e8\uacfc\uc774 \uc815\ubd80\ub294 \uc6b0\uc2b9\uc5d0 \ub300\ud55c \ud658\ud76c\ub85c \uacbd\uae30 \ub2e4\uc74c\ub0a0\uc778 7\uc6d4 31\uc77c\uc744 \uad6d\uac00 \uacf5\ud734\uc77c\ub85c \uc9c0\uc815\ud588\ub2e4. \ud55c\ud3b8 \uc900\uc6b0\uc2b9\uc5d0 \uadf8\uce5c \uc544\ub974\ud5e8\ud2f0\ub098\uc758 \uc218\ub3c4 \ubd80\uc5d0\ub178\uc2a4 \uc544\uc774\ub808\uc2a4\uc5d0\uc11c\ub294 \uc6b0\ub8e8\uacfc\uc774 \uc601\uc0ac\uad00\uc744 \ud5a5\ud574 \uad70\uc911\ub4e4\uc774 \ub3cc\uc744 \ub358\uc84c\ub2e4."} {"question": "\uc704\uae30 \ubc1c\uc0dd 84\uc77c\uc9f8 \ub36e\uac1c \ud0d1\uc774 \ubcf4\ub0b4\uc9c4 \uacf3\uc740?", "paragraph": "\ub9c8\uc9c0\ub9c9 \uc791\uc804\uc73c\ub85c BP\ub294 \ub36e\uac1c \ud0d1\uc744 \uc81c\uc791\ud588\ub2e4. \ub36e\uac1c \ud0d1\uc740 BOP \uc0c1\ubd80\ub97c \uc644\uc804\ud788 \ubc00\ubd09\ud560 \uc218 \uc788\ub294 \uac83\uc774\uc5c8\ub2e4. \uc548\ucabd\uc5d0\ub294 3\uac1c\uc758 \ubca8\ube0c\uac00 \uc788\uc5c8\ub2e4. \uc5f4\ub824\uc788\ub294 \ub3d9\uc548 \uae30\ub984\uc774 \ubc29\ucd9c\ub418\uace0, \ubca8\ube0c\uac00 \uc7a0\uae30\uba74 \uc720\uc815\uc740 \ubd09\uc1c4\ub41c\ub2e4. \uc774 \uc791\uc5c5\uc5d0\uc11c \uac00\uc7a5 \uc704\ud5d8\ud55c \uc810\uc740 \uc804\uccb4 \uc2dc\uc2a4\ud15c\uc774 \uc555\ub825\uc744 \uacac\ub514\ub290\ub0d0 \ud558\ub294 \uac83\uc774\uc5c8\ub2e4. \ud0d1\ud0ac \uc774\ud6c4 \uc774\uac83\uc774 \uc720\uc815\uc744 \ucc28\ub2e8\ud560 \uc218 \uc788\uc744\uc9c0 \uc5fc\ub824\uac00 \uc788\uc5c8\ub2e4. \uc704\uae30 \ubc1c\uc0dd 84\uc77c\uc9f8. \ub36e\uac1c \ud0d1\uc740 \ud574\uc800 1,500\ubbf8\ud130\ub85c \ubcf4\ub0b4\uc84c\uace0, \uc800\ub141 7\uc2dc \ub36e\uac1c \ud0d1\uc740 BOP \uc704\uc5d0 \uc790\ub9ac\uc7a1\uc558\ub2e4. \ud558\ub8e8\uac00 \uc9c0\ub098\uace0 \ub36e\uac1c\ud0d1\uc758 \ubca8\ube0c 3\uac1c\uac00 \ucc28\ub840\ub85c \ub2eb\ud614\ub2e4. 7\uc6d4 15\uc77c \uc624\ud6c4 2\uc2dc 25\ubd84, \uc11d\uc720 \uc720\ucd9c\uc774 \ubaa8\ub450 \uba4e\uc5c8\ub2e4. \ud558\uc9c0\ub9cc BP\uc758 \ucc45\uc784\ub860\uc740 \uc5ec\uc804\ud788 \ub300\ub450\ub410\ub2e4. BP\ub294 \ub36e\uac1c\ud0d1 \uc791\uc804\uc744 \uc9c0\uc5f0\uc2dc\ud0a8\uc801\uc774 \uc788\ub2e4\ub294 \uc758\ud639\uc774 \uc81c\uae30\ub410\uae30 \ub54c\ubb38\uc774\ub2e4. \ud558\uc9c0\ub9cc BP\ub294 \uadf8\ub807\uc9c0 \uc54a\ub2e4\uace0 \uc8fc\uc7a5\ud55c\ub2e4. \uc720\uc815\uc744 \ud3ec\ud68d\ud558\ub294 \uc791\uc5c5\uacfc, \ub36e\ub294 \uc791\uc5c5 \uc0ac\uc774\uc5d0 \ub300\ud55c \uade0\ud615 \ub17c\ub780\ub3c4 \uc81c\uae30\ub410\ub2e4. \ub354 \ube68\ub9ac \uc0ac\uace0\ub97c \uc218\uc2b5\ud560 \uc218 \uc788\uc5c8\ub2e4\ub294 \uc9c0\uc801\uc774 \uc788\ub294 \uac83\uc774\ub2e4. \uadf8\ub3d9\uc548 490\ub9cc \ubc30\ub7f4\uc758 \uc6d0\uc720\uac00 \uc720\ucd9c\ub41c\uac83\uc73c\ub85c \ucd94\uc815\ub41c\ub2e4.", "answer": "\ud574\uc800 1,500\ubbf8\ud130", "sentence": "\ud0d1\uc740 \ud574\uc800 1,500\ubbf8\ud130 \ub85c \ubcf4\ub0b4\uc84c\uace0, \uc800\ub141 7\uc2dc \ub36e\uac1c \ud0d1\uc740 BOP \uc704\uc5d0 \uc790\ub9ac\uc7a1\uc558\ub2e4.", "paragraph_sentence": "\ub9c8\uc9c0\ub9c9 \uc791\uc804\uc73c\ub85c BP\ub294 \ub36e\uac1c \ud0d1\uc744 \uc81c\uc791\ud588\ub2e4. \ub36e\uac1c \ud0d1\uc740 BOP \uc0c1\ubd80\ub97c \uc644\uc804\ud788 \ubc00\ubd09\ud560 \uc218 \uc788\ub294 \uac83\uc774\uc5c8\ub2e4. \uc548\ucabd\uc5d0\ub294 3\uac1c\uc758 \ubca8\ube0c\uac00 \uc788\uc5c8\ub2e4. \uc5f4\ub824\uc788\ub294 \ub3d9\uc548 \uae30\ub984\uc774 \ubc29\ucd9c\ub418\uace0, \ubca8\ube0c\uac00 \uc7a0\uae30\uba74 \uc720\uc815\uc740 \ubd09\uc1c4\ub41c\ub2e4. \uc774 \uc791\uc5c5\uc5d0\uc11c \uac00\uc7a5 \uc704\ud5d8\ud55c \uc810\uc740 \uc804\uccb4 \uc2dc\uc2a4\ud15c\uc774 \uc555\ub825\uc744 \uacac\ub514\ub290\ub0d0 \ud558\ub294 \uac83\uc774\uc5c8\ub2e4. \ud0d1\ud0ac \uc774\ud6c4 \uc774\uac83\uc774 \uc720\uc815\uc744 \ucc28\ub2e8\ud560 \uc218 \uc788\uc744\uc9c0 \uc5fc\ub824\uac00 \uc788\uc5c8\ub2e4. \uc704\uae30 \ubc1c\uc0dd 84\uc77c\uc9f8. \ub36e\uac1c \ud0d1\uc740 \ud574\uc800 1,500\ubbf8\ud130 \ub85c \ubcf4\ub0b4\uc84c\uace0, \uc800\ub141 7\uc2dc \ub36e\uac1c \ud0d1\uc740 BOP \uc704\uc5d0 \uc790\ub9ac\uc7a1\uc558\ub2e4. \ud558\ub8e8\uac00 \uc9c0\ub098\uace0 \ub36e\uac1c\ud0d1\uc758 \ubca8\ube0c 3\uac1c\uac00 \ucc28\ub840\ub85c \ub2eb\ud614\ub2e4. 7\uc6d4 15\uc77c \uc624\ud6c4 2\uc2dc 25\ubd84, \uc11d\uc720 \uc720\ucd9c\uc774 \ubaa8\ub450 \uba4e\uc5c8\ub2e4. \ud558\uc9c0\ub9cc BP\uc758 \ucc45\uc784\ub860\uc740 \uc5ec\uc804\ud788 \ub300\ub450\ub410\ub2e4. BP\ub294 \ub36e\uac1c\ud0d1 \uc791\uc804\uc744 \uc9c0\uc5f0\uc2dc\ud0a8\uc801\uc774 \uc788\ub2e4\ub294 \uc758\ud639\uc774 \uc81c\uae30\ub410\uae30 \ub54c\ubb38\uc774\ub2e4. \ud558\uc9c0\ub9cc BP\ub294 \uadf8\ub807\uc9c0 \uc54a\ub2e4\uace0 \uc8fc\uc7a5\ud55c\ub2e4. \uc720\uc815\uc744 \ud3ec\ud68d\ud558\ub294 \uc791\uc5c5\uacfc, \ub36e\ub294 \uc791\uc5c5 \uc0ac\uc774\uc5d0 \ub300\ud55c \uade0\ud615 \ub17c\ub780\ub3c4 \uc81c\uae30\ub410\ub2e4. \ub354 \ube68\ub9ac \uc0ac\uace0\ub97c \uc218\uc2b5\ud560 \uc218 \uc788\uc5c8\ub2e4\ub294 \uc9c0\uc801\uc774 \uc788\ub294 \uac83\uc774\ub2e4. \uadf8\ub3d9\uc548 490\ub9cc \ubc30\ub7f4\uc758 \uc6d0\uc720\uac00 \uc720\ucd9c\ub41c\uac83\uc73c\ub85c \ucd94\uc815\ub41c\ub2e4.", "paragraph_answer": "\ub9c8\uc9c0\ub9c9 \uc791\uc804\uc73c\ub85c BP\ub294 \ub36e\uac1c \ud0d1\uc744 \uc81c\uc791\ud588\ub2e4. \ub36e\uac1c \ud0d1\uc740 BOP \uc0c1\ubd80\ub97c \uc644\uc804\ud788 \ubc00\ubd09\ud560 \uc218 \uc788\ub294 \uac83\uc774\uc5c8\ub2e4. \uc548\ucabd\uc5d0\ub294 3\uac1c\uc758 \ubca8\ube0c\uac00 \uc788\uc5c8\ub2e4. \uc5f4\ub824\uc788\ub294 \ub3d9\uc548 \uae30\ub984\uc774 \ubc29\ucd9c\ub418\uace0, \ubca8\ube0c\uac00 \uc7a0\uae30\uba74 \uc720\uc815\uc740 \ubd09\uc1c4\ub41c\ub2e4. \uc774 \uc791\uc5c5\uc5d0\uc11c \uac00\uc7a5 \uc704\ud5d8\ud55c \uc810\uc740 \uc804\uccb4 \uc2dc\uc2a4\ud15c\uc774 \uc555\ub825\uc744 \uacac\ub514\ub290\ub0d0 \ud558\ub294 \uac83\uc774\uc5c8\ub2e4. \ud0d1\ud0ac \uc774\ud6c4 \uc774\uac83\uc774 \uc720\uc815\uc744 \ucc28\ub2e8\ud560 \uc218 \uc788\uc744\uc9c0 \uc5fc\ub824\uac00 \uc788\uc5c8\ub2e4. \uc704\uae30 \ubc1c\uc0dd 84\uc77c\uc9f8. \ub36e\uac1c \ud0d1\uc740 \ud574\uc800 1,500\ubbf8\ud130 \ub85c \ubcf4\ub0b4\uc84c\uace0, \uc800\ub141 7\uc2dc \ub36e\uac1c \ud0d1\uc740 BOP \uc704\uc5d0 \uc790\ub9ac\uc7a1\uc558\ub2e4. \ud558\ub8e8\uac00 \uc9c0\ub098\uace0 \ub36e\uac1c\ud0d1\uc758 \ubca8\ube0c 3\uac1c\uac00 \ucc28\ub840\ub85c \ub2eb\ud614\ub2e4. 7\uc6d4 15\uc77c \uc624\ud6c4 2\uc2dc 25\ubd84, \uc11d\uc720 \uc720\ucd9c\uc774 \ubaa8\ub450 \uba4e\uc5c8\ub2e4. \ud558\uc9c0\ub9cc BP\uc758 \ucc45\uc784\ub860\uc740 \uc5ec\uc804\ud788 \ub300\ub450\ub410\ub2e4. BP\ub294 \ub36e\uac1c\ud0d1 \uc791\uc804\uc744 \uc9c0\uc5f0\uc2dc\ud0a8\uc801\uc774 \uc788\ub2e4\ub294 \uc758\ud639\uc774 \uc81c\uae30\ub410\uae30 \ub54c\ubb38\uc774\ub2e4. \ud558\uc9c0\ub9cc BP\ub294 \uadf8\ub807\uc9c0 \uc54a\ub2e4\uace0 \uc8fc\uc7a5\ud55c\ub2e4. \uc720\uc815\uc744 \ud3ec\ud68d\ud558\ub294 \uc791\uc5c5\uacfc, \ub36e\ub294 \uc791\uc5c5 \uc0ac\uc774\uc5d0 \ub300\ud55c \uade0\ud615 \ub17c\ub780\ub3c4 \uc81c\uae30\ub410\ub2e4. \ub354 \ube68\ub9ac \uc0ac\uace0\ub97c \uc218\uc2b5\ud560 \uc218 \uc788\uc5c8\ub2e4\ub294 \uc9c0\uc801\uc774 \uc788\ub294 \uac83\uc774\ub2e4. \uadf8\ub3d9\uc548 490\ub9cc \ubc30\ub7f4\uc758 \uc6d0\uc720\uac00 \uc720\ucd9c\ub41c\uac83\uc73c\ub85c \ucd94\uc815\ub41c\ub2e4.", "sentence_answer": "\ud0d1\uc740 \ud574\uc800 1,500\ubbf8\ud130 \ub85c \ubcf4\ub0b4\uc84c\uace0, \uc800\ub141 7\uc2dc \ub36e\uac1c \ud0d1\uc740 BOP \uc704\uc5d0 \uc790\ub9ac\uc7a1\uc558\ub2e4.", "paragraph_id": "6300420-11-2", "paragraph_question": "question: \uc704\uae30 \ubc1c\uc0dd 84\uc77c\uc9f8 \ub36e\uac1c \ud0d1\uc774 \ubcf4\ub0b4\uc9c4 \uacf3\uc740?, context: \ub9c8\uc9c0\ub9c9 \uc791\uc804\uc73c\ub85c BP\ub294 \ub36e\uac1c \ud0d1\uc744 \uc81c\uc791\ud588\ub2e4. \ub36e\uac1c \ud0d1\uc740 BOP \uc0c1\ubd80\ub97c \uc644\uc804\ud788 \ubc00\ubd09\ud560 \uc218 \uc788\ub294 \uac83\uc774\uc5c8\ub2e4. \uc548\ucabd\uc5d0\ub294 3\uac1c\uc758 \ubca8\ube0c\uac00 \uc788\uc5c8\ub2e4. \uc5f4\ub824\uc788\ub294 \ub3d9\uc548 \uae30\ub984\uc774 \ubc29\ucd9c\ub418\uace0, \ubca8\ube0c\uac00 \uc7a0\uae30\uba74 \uc720\uc815\uc740 \ubd09\uc1c4\ub41c\ub2e4. \uc774 \uc791\uc5c5\uc5d0\uc11c \uac00\uc7a5 \uc704\ud5d8\ud55c \uc810\uc740 \uc804\uccb4 \uc2dc\uc2a4\ud15c\uc774 \uc555\ub825\uc744 \uacac\ub514\ub290\ub0d0 \ud558\ub294 \uac83\uc774\uc5c8\ub2e4. \ud0d1\ud0ac \uc774\ud6c4 \uc774\uac83\uc774 \uc720\uc815\uc744 \ucc28\ub2e8\ud560 \uc218 \uc788\uc744\uc9c0 \uc5fc\ub824\uac00 \uc788\uc5c8\ub2e4. \uc704\uae30 \ubc1c\uc0dd 84\uc77c\uc9f8. \ub36e\uac1c \ud0d1\uc740 \ud574\uc800 1,500\ubbf8\ud130\ub85c \ubcf4\ub0b4\uc84c\uace0, \uc800\ub141 7\uc2dc \ub36e\uac1c \ud0d1\uc740 BOP \uc704\uc5d0 \uc790\ub9ac\uc7a1\uc558\ub2e4. \ud558\ub8e8\uac00 \uc9c0\ub098\uace0 \ub36e\uac1c\ud0d1\uc758 \ubca8\ube0c 3\uac1c\uac00 \ucc28\ub840\ub85c \ub2eb\ud614\ub2e4. 7\uc6d4 15\uc77c \uc624\ud6c4 2\uc2dc 25\ubd84, \uc11d\uc720 \uc720\ucd9c\uc774 \ubaa8\ub450 \uba4e\uc5c8\ub2e4. \ud558\uc9c0\ub9cc BP\uc758 \ucc45\uc784\ub860\uc740 \uc5ec\uc804\ud788 \ub300\ub450\ub410\ub2e4. BP\ub294 \ub36e\uac1c\ud0d1 \uc791\uc804\uc744 \uc9c0\uc5f0\uc2dc\ud0a8\uc801\uc774 \uc788\ub2e4\ub294 \uc758\ud639\uc774 \uc81c\uae30\ub410\uae30 \ub54c\ubb38\uc774\ub2e4. \ud558\uc9c0\ub9cc BP\ub294 \uadf8\ub807\uc9c0 \uc54a\ub2e4\uace0 \uc8fc\uc7a5\ud55c\ub2e4. \uc720\uc815\uc744 \ud3ec\ud68d\ud558\ub294 \uc791\uc5c5\uacfc, \ub36e\ub294 \uc791\uc5c5 \uc0ac\uc774\uc5d0 \ub300\ud55c \uade0\ud615 \ub17c\ub780\ub3c4 \uc81c\uae30\ub410\ub2e4. \ub354 \ube68\ub9ac \uc0ac\uace0\ub97c \uc218\uc2b5\ud560 \uc218 \uc788\uc5c8\ub2e4\ub294 \uc9c0\uc801\uc774 \uc788\ub294 \uac83\uc774\ub2e4. \uadf8\ub3d9\uc548 490\ub9cc \ubc30\ub7f4\uc758 \uc6d0\uc720\uac00 \uc720\ucd9c\ub41c\uac83\uc73c\ub85c \ucd94\uc815\ub41c\ub2e4."} {"question": "KBO\ub294 \uba87\uc6d4 \uba87\uc77c\uc5d0 \uc774\uc0ac\ud68c\ub97c \ud1b5\ud574 \ud2b8\ub808\uc774\ub4dc\uc5d0 \ub300\ud55c \uc785\uc7a5\uc744 \ubc1d\ud790 \uc608\uc815\uc774\uc5c8\ub098?", "paragraph": "2010\ub144 \uc544\uc2dc\uc548 \uac8c\uc784\uc774 \uc885\ub8cc\ub418\uc790 \uc77c\ubd80 \uad6c\ub2e8\uc774 \ub178\uace8\uc801\uc778 \uad00\uc2ec\uc744 \ub4dc\ub7ec\ub0b4\uba70 \ub450 \uc120\uc218\uc5d0\uac8c \uc811\uadfc\ud558\uae30 \uc2dc\uc791\ud588\ub2e4. 2010\ub144 \uc544\uc2dc\uc548 \uac8c\uc784 \uc57c\uad6c \ub300\ud45c\ud300\uc73c\ub85c \ucc38\uac00\ud588\ub358 \uac15\uc815\ud638\ub294 \uae08\uba54\ub2ec\uc744 \ubaa9\uc5d0 \uac78\uba70 \ubcd1\uc5ed \ud61c\ud0dd\uc744 \ubc1b\uc544 \ubab8\uac12\uc774 \uae09\ub4f1\ud588\ub2e4. \uc190\uc2b9\ub77d\uacfc \ud568\uaed8 2010\ub144 \ud22c\ud0c0 \uace0\uacfc 1\uc704\uc5d0 \uc62c\ub77c \uc5f0\ubd09 \uc0c1\uc2b9\ub3c4 \uc608\uc57d\ub41c \uac15\uc815\ud638\ub294 \ud300\uc758 \uc0ac\uc2e4 \ubd80\uc778\uc5d0\ub3c4 \ubd88\uad6c\ud558\uace0 \uacc4\uc18d\ud574\uc11c \uace0\ud5a5 \ud300\uc778 KIA \ud0c0\uc774\uac70\uc988\ub85c\uc758 \ud2b8\ub808\uc774\ub4dc \uc124\uc5d0 \ud729\uc2f8\uc600\ub2e4. KIA\uac00 \uac70\uc758 \uacf5\ubc31 \uc0c1\ud0dc\uc778 \uc720\uaca9\uc218 \ubcf4\uac15\uc5d0 \ub098\uc124 \uac83\uc774\ub77c\ub294 \uc124\uacfc \ud568\uaed8 \ub9c8\ubb34\ub9ac \uc190\uc2b9\ub77d\uc740 LG \ud2b8\uc708\uc2a4\uc640 \uad6c\uccb4\uc801\uc778 \ud611\uc0c1\uae4c\uc9c0 \uc624\uace0 \uac04 \uac83\uc73c\ub85c \uc54c\ub824\uc84c\ub2e4. LG\uac00 \ub9c8\ubb34\ub9ac \ubcf4\uac15\uc744 \uc704\ud574 \ub125\uc13c\uc5d0\uac8c \uc811\uadfc\ud588\ub294\ub370 \ub125\uc13c\uc758 \ubb34\ub9ac\ud55c \uc694\uad6c \ud0d3\uc5d0 \ud611\uc0c1\uc774 \uacb0\ub82c\ub418\uc5c8\ub2e4. \uc774 \ud2b8\ub808\uc774\ub4dc\ub294 \ud604\uae08 \ud2b8\ub808\uc774\ub4dc\ub85c \uc54c\ub824\uc84c\ub294\ub370, \ubc18\uba74 \ub125\uc13c \uad6c\ub2e8\uc740 LG\uac00 \uba3c\uc800 \uc811\uadfc\ud574\uc11c \ud611\uc0c1\uc744 \ud588\uace0 \uc804\ub825 \ubcf4\uac15 \uc678\uc5d0 \ud604\uae08 \ud2b8\ub808\uc774\ub4dc\ub294 \uace0\ub824\ud558\uc9c0 \uc54a\uc558\ub2e4\uace0 \ubc18\ubc15\ud588\ub2e4. \ub125\uc13c\uc740 4\uac15\uc744 \ub178\ub9ac\uae30 \uc704\ud55c \uc804\ub825\ubcf4\uac15 \ud2b8\ub808\uc774\ub4dc \uc678\uc5d0 \ud604\uae08 \ud2b8\ub808\uc774\ub4dc\ub294 \ud558\uc9c0 \uc54a\uaca0\ub2e4\uace0 \ubc1d\ud600\uc654\ub294\ub370, LG\uc5d0\uac8c \uc791\uc740 \uc774\ubcd1\uaddc\ub97c \uc694\uad6c\ud558\ub2e4\uac00 LG\uc758 \ud2b8\ub808\uc774\ub4dc \ubd88\uac00 \ubc29\uce68\uc73c\ub85c \ubb34\uc0b0\ub418\uc5c8\ub2e4\uace0 \ubc1d\ud788\uba70 \ud604\uae08 \ud2b8\ub808\uc774\ub4dc \uc2dc\ub3c4 \uc0ac\uc2e4\uc744 \ubd80\uc778\ud588\ub2e4. KBO\ub294 12\uc6d4 14\uc77c \uc774\uc0ac\ud68c\uc5d0\uc11c \ub17c\uc758\ub97c \uac70\uccd0 \ud2b8\ub808\uc774\ub4dc\uc5d0 \uad00\ud55c \uc785\uc7a5\uc744 \ubc1c\ud45c\ud560 \uc608\uc815\uc73c\ub85c \uc54c\ub824\uc84c\ub2e4.", "answer": "12\uc6d4 14\uc77c", "sentence": "KBO\ub294 12\uc6d4 14\uc77c \uc774\uc0ac\ud68c\uc5d0\uc11c \ub17c\uc758\ub97c \uac70\uccd0 \ud2b8\ub808\uc774\ub4dc\uc5d0 \uad00\ud55c \uc785\uc7a5\uc744 \ubc1c\ud45c\ud560 \uc608\uc815\uc73c\ub85c \uc54c\ub824\uc84c\ub2e4.", "paragraph_sentence": "2010\ub144 \uc544\uc2dc\uc548 \uac8c\uc784\uc774 \uc885\ub8cc\ub418\uc790 \uc77c\ubd80 \uad6c\ub2e8\uc774 \ub178\uace8\uc801\uc778 \uad00\uc2ec\uc744 \ub4dc\ub7ec\ub0b4\uba70 \ub450 \uc120\uc218\uc5d0\uac8c \uc811\uadfc\ud558\uae30 \uc2dc\uc791\ud588\ub2e4. 2010\ub144 \uc544\uc2dc\uc548 \uac8c\uc784 \uc57c\uad6c \ub300\ud45c\ud300\uc73c\ub85c \ucc38\uac00\ud588\ub358 \uac15\uc815\ud638\ub294 \uae08\uba54\ub2ec\uc744 \ubaa9\uc5d0 \uac78\uba70 \ubcd1\uc5ed \ud61c\ud0dd\uc744 \ubc1b\uc544 \ubab8\uac12\uc774 \uae09\ub4f1\ud588\ub2e4. \uc190\uc2b9\ub77d\uacfc \ud568\uaed8 2010\ub144 \ud22c\ud0c0 \uace0\uacfc 1\uc704\uc5d0 \uc62c\ub77c \uc5f0\ubd09 \uc0c1\uc2b9\ub3c4 \uc608\uc57d\ub41c \uac15\uc815\ud638\ub294 \ud300\uc758 \uc0ac\uc2e4 \ubd80\uc778\uc5d0\ub3c4 \ubd88\uad6c\ud558\uace0 \uacc4\uc18d\ud574\uc11c \uace0\ud5a5 \ud300\uc778 KIA \ud0c0\uc774\uac70\uc988\ub85c\uc758 \ud2b8\ub808\uc774\ub4dc \uc124\uc5d0 \ud729\uc2f8\uc600\ub2e4. KIA\uac00 \uac70\uc758 \uacf5\ubc31 \uc0c1\ud0dc\uc778 \uc720\uaca9\uc218 \ubcf4\uac15\uc5d0 \ub098\uc124 \uac83\uc774\ub77c\ub294 \uc124\uacfc \ud568\uaed8 \ub9c8\ubb34\ub9ac \uc190\uc2b9\ub77d\uc740 LG \ud2b8\uc708\uc2a4\uc640 \uad6c\uccb4\uc801\uc778 \ud611\uc0c1\uae4c\uc9c0 \uc624\uace0 \uac04 \uac83\uc73c\ub85c \uc54c\ub824\uc84c\ub2e4. LG\uac00 \ub9c8\ubb34\ub9ac \ubcf4\uac15\uc744 \uc704\ud574 \ub125\uc13c\uc5d0\uac8c \uc811\uadfc\ud588\ub294\ub370 \ub125\uc13c\uc758 \ubb34\ub9ac\ud55c \uc694\uad6c \ud0d3\uc5d0 \ud611\uc0c1\uc774 \uacb0\ub82c\ub418\uc5c8\ub2e4. \uc774 \ud2b8\ub808\uc774\ub4dc\ub294 \ud604\uae08 \ud2b8\ub808\uc774\ub4dc\ub85c \uc54c\ub824\uc84c\ub294\ub370, \ubc18\uba74 \ub125\uc13c \uad6c\ub2e8\uc740 LG\uac00 \uba3c\uc800 \uc811\uadfc\ud574\uc11c \ud611\uc0c1\uc744 \ud588\uace0 \uc804\ub825 \ubcf4\uac15 \uc678\uc5d0 \ud604\uae08 \ud2b8\ub808\uc774\ub4dc\ub294 \uace0\ub824\ud558\uc9c0 \uc54a\uc558\ub2e4\uace0 \ubc18\ubc15\ud588\ub2e4. \ub125\uc13c\uc740 4\uac15\uc744 \ub178\ub9ac\uae30 \uc704\ud55c \uc804\ub825\ubcf4\uac15 \ud2b8\ub808\uc774\ub4dc \uc678\uc5d0 \ud604\uae08 \ud2b8\ub808\uc774\ub4dc\ub294 \ud558\uc9c0 \uc54a\uaca0\ub2e4\uace0 \ubc1d\ud600\uc654\ub294\ub370, LG\uc5d0\uac8c \uc791\uc740 \uc774\ubcd1\uaddc\ub97c \uc694\uad6c\ud558\ub2e4\uac00 LG\uc758 \ud2b8\ub808\uc774\ub4dc \ubd88\uac00 \ubc29\uce68\uc73c\ub85c \ubb34\uc0b0\ub418\uc5c8\ub2e4\uace0 \ubc1d\ud788\uba70 \ud604\uae08 \ud2b8\ub808\uc774\ub4dc \uc2dc\ub3c4 \uc0ac\uc2e4\uc744 \ubd80\uc778\ud588\ub2e4. KBO\ub294 12\uc6d4 14\uc77c \uc774\uc0ac\ud68c\uc5d0\uc11c \ub17c\uc758\ub97c \uac70\uccd0 \ud2b8\ub808\uc774\ub4dc\uc5d0 \uad00\ud55c \uc785\uc7a5\uc744 \ubc1c\ud45c\ud560 \uc608\uc815\uc73c\ub85c \uc54c\ub824\uc84c\ub2e4. ", "paragraph_answer": "2010\ub144 \uc544\uc2dc\uc548 \uac8c\uc784\uc774 \uc885\ub8cc\ub418\uc790 \uc77c\ubd80 \uad6c\ub2e8\uc774 \ub178\uace8\uc801\uc778 \uad00\uc2ec\uc744 \ub4dc\ub7ec\ub0b4\uba70 \ub450 \uc120\uc218\uc5d0\uac8c \uc811\uadfc\ud558\uae30 \uc2dc\uc791\ud588\ub2e4. 2010\ub144 \uc544\uc2dc\uc548 \uac8c\uc784 \uc57c\uad6c \ub300\ud45c\ud300\uc73c\ub85c \ucc38\uac00\ud588\ub358 \uac15\uc815\ud638\ub294 \uae08\uba54\ub2ec\uc744 \ubaa9\uc5d0 \uac78\uba70 \ubcd1\uc5ed \ud61c\ud0dd\uc744 \ubc1b\uc544 \ubab8\uac12\uc774 \uae09\ub4f1\ud588\ub2e4. \uc190\uc2b9\ub77d\uacfc \ud568\uaed8 2010\ub144 \ud22c\ud0c0 \uace0\uacfc 1\uc704\uc5d0 \uc62c\ub77c \uc5f0\ubd09 \uc0c1\uc2b9\ub3c4 \uc608\uc57d\ub41c \uac15\uc815\ud638\ub294 \ud300\uc758 \uc0ac\uc2e4 \ubd80\uc778\uc5d0\ub3c4 \ubd88\uad6c\ud558\uace0 \uacc4\uc18d\ud574\uc11c \uace0\ud5a5 \ud300\uc778 KIA \ud0c0\uc774\uac70\uc988\ub85c\uc758 \ud2b8\ub808\uc774\ub4dc \uc124\uc5d0 \ud729\uc2f8\uc600\ub2e4. KIA\uac00 \uac70\uc758 \uacf5\ubc31 \uc0c1\ud0dc\uc778 \uc720\uaca9\uc218 \ubcf4\uac15\uc5d0 \ub098\uc124 \uac83\uc774\ub77c\ub294 \uc124\uacfc \ud568\uaed8 \ub9c8\ubb34\ub9ac \uc190\uc2b9\ub77d\uc740 LG \ud2b8\uc708\uc2a4\uc640 \uad6c\uccb4\uc801\uc778 \ud611\uc0c1\uae4c\uc9c0 \uc624\uace0 \uac04 \uac83\uc73c\ub85c \uc54c\ub824\uc84c\ub2e4. LG\uac00 \ub9c8\ubb34\ub9ac \ubcf4\uac15\uc744 \uc704\ud574 \ub125\uc13c\uc5d0\uac8c \uc811\uadfc\ud588\ub294\ub370 \ub125\uc13c\uc758 \ubb34\ub9ac\ud55c \uc694\uad6c \ud0d3\uc5d0 \ud611\uc0c1\uc774 \uacb0\ub82c\ub418\uc5c8\ub2e4. \uc774 \ud2b8\ub808\uc774\ub4dc\ub294 \ud604\uae08 \ud2b8\ub808\uc774\ub4dc\ub85c \uc54c\ub824\uc84c\ub294\ub370, \ubc18\uba74 \ub125\uc13c \uad6c\ub2e8\uc740 LG\uac00 \uba3c\uc800 \uc811\uadfc\ud574\uc11c \ud611\uc0c1\uc744 \ud588\uace0 \uc804\ub825 \ubcf4\uac15 \uc678\uc5d0 \ud604\uae08 \ud2b8\ub808\uc774\ub4dc\ub294 \uace0\ub824\ud558\uc9c0 \uc54a\uc558\ub2e4\uace0 \ubc18\ubc15\ud588\ub2e4. \ub125\uc13c\uc740 4\uac15\uc744 \ub178\ub9ac\uae30 \uc704\ud55c \uc804\ub825\ubcf4\uac15 \ud2b8\ub808\uc774\ub4dc \uc678\uc5d0 \ud604\uae08 \ud2b8\ub808\uc774\ub4dc\ub294 \ud558\uc9c0 \uc54a\uaca0\ub2e4\uace0 \ubc1d\ud600\uc654\ub294\ub370, LG\uc5d0\uac8c \uc791\uc740 \uc774\ubcd1\uaddc\ub97c \uc694\uad6c\ud558\ub2e4\uac00 LG\uc758 \ud2b8\ub808\uc774\ub4dc \ubd88\uac00 \ubc29\uce68\uc73c\ub85c \ubb34\uc0b0\ub418\uc5c8\ub2e4\uace0 \ubc1d\ud788\uba70 \ud604\uae08 \ud2b8\ub808\uc774\ub4dc \uc2dc\ub3c4 \uc0ac\uc2e4\uc744 \ubd80\uc778\ud588\ub2e4. KBO\ub294 12\uc6d4 14\uc77c \uc774\uc0ac\ud68c\uc5d0\uc11c \ub17c\uc758\ub97c \uac70\uccd0 \ud2b8\ub808\uc774\ub4dc\uc5d0 \uad00\ud55c \uc785\uc7a5\uc744 \ubc1c\ud45c\ud560 \uc608\uc815\uc73c\ub85c \uc54c\ub824\uc84c\ub2e4.", "sentence_answer": "KBO\ub294 12\uc6d4 14\uc77c \uc774\uc0ac\ud68c\uc5d0\uc11c \ub17c\uc758\ub97c \uac70\uccd0 \ud2b8\ub808\uc774\ub4dc\uc5d0 \uad00\ud55c \uc785\uc7a5\uc744 \ubc1c\ud45c\ud560 \uc608\uc815\uc73c\ub85c \uc54c\ub824\uc84c\ub2e4.", "paragraph_id": "6533929-14-2", "paragraph_question": "question: KBO\ub294 \uba87\uc6d4 \uba87\uc77c\uc5d0 \uc774\uc0ac\ud68c\ub97c \ud1b5\ud574 \ud2b8\ub808\uc774\ub4dc\uc5d0 \ub300\ud55c \uc785\uc7a5\uc744 \ubc1d\ud790 \uc608\uc815\uc774\uc5c8\ub098?, context: 2010\ub144 \uc544\uc2dc\uc548 \uac8c\uc784\uc774 \uc885\ub8cc\ub418\uc790 \uc77c\ubd80 \uad6c\ub2e8\uc774 \ub178\uace8\uc801\uc778 \uad00\uc2ec\uc744 \ub4dc\ub7ec\ub0b4\uba70 \ub450 \uc120\uc218\uc5d0\uac8c \uc811\uadfc\ud558\uae30 \uc2dc\uc791\ud588\ub2e4. 2010\ub144 \uc544\uc2dc\uc548 \uac8c\uc784 \uc57c\uad6c \ub300\ud45c\ud300\uc73c\ub85c \ucc38\uac00\ud588\ub358 \uac15\uc815\ud638\ub294 \uae08\uba54\ub2ec\uc744 \ubaa9\uc5d0 \uac78\uba70 \ubcd1\uc5ed \ud61c\ud0dd\uc744 \ubc1b\uc544 \ubab8\uac12\uc774 \uae09\ub4f1\ud588\ub2e4. \uc190\uc2b9\ub77d\uacfc \ud568\uaed8 2010\ub144 \ud22c\ud0c0 \uace0\uacfc 1\uc704\uc5d0 \uc62c\ub77c \uc5f0\ubd09 \uc0c1\uc2b9\ub3c4 \uc608\uc57d\ub41c \uac15\uc815\ud638\ub294 \ud300\uc758 \uc0ac\uc2e4 \ubd80\uc778\uc5d0\ub3c4 \ubd88\uad6c\ud558\uace0 \uacc4\uc18d\ud574\uc11c \uace0\ud5a5 \ud300\uc778 KIA \ud0c0\uc774\uac70\uc988\ub85c\uc758 \ud2b8\ub808\uc774\ub4dc \uc124\uc5d0 \ud729\uc2f8\uc600\ub2e4. KIA\uac00 \uac70\uc758 \uacf5\ubc31 \uc0c1\ud0dc\uc778 \uc720\uaca9\uc218 \ubcf4\uac15\uc5d0 \ub098\uc124 \uac83\uc774\ub77c\ub294 \uc124\uacfc \ud568\uaed8 \ub9c8\ubb34\ub9ac \uc190\uc2b9\ub77d\uc740 LG \ud2b8\uc708\uc2a4\uc640 \uad6c\uccb4\uc801\uc778 \ud611\uc0c1\uae4c\uc9c0 \uc624\uace0 \uac04 \uac83\uc73c\ub85c \uc54c\ub824\uc84c\ub2e4. LG\uac00 \ub9c8\ubb34\ub9ac \ubcf4\uac15\uc744 \uc704\ud574 \ub125\uc13c\uc5d0\uac8c \uc811\uadfc\ud588\ub294\ub370 \ub125\uc13c\uc758 \ubb34\ub9ac\ud55c \uc694\uad6c \ud0d3\uc5d0 \ud611\uc0c1\uc774 \uacb0\ub82c\ub418\uc5c8\ub2e4. \uc774 \ud2b8\ub808\uc774\ub4dc\ub294 \ud604\uae08 \ud2b8\ub808\uc774\ub4dc\ub85c \uc54c\ub824\uc84c\ub294\ub370, \ubc18\uba74 \ub125\uc13c \uad6c\ub2e8\uc740 LG\uac00 \uba3c\uc800 \uc811\uadfc\ud574\uc11c \ud611\uc0c1\uc744 \ud588\uace0 \uc804\ub825 \ubcf4\uac15 \uc678\uc5d0 \ud604\uae08 \ud2b8\ub808\uc774\ub4dc\ub294 \uace0\ub824\ud558\uc9c0 \uc54a\uc558\ub2e4\uace0 \ubc18\ubc15\ud588\ub2e4. \ub125\uc13c\uc740 4\uac15\uc744 \ub178\ub9ac\uae30 \uc704\ud55c \uc804\ub825\ubcf4\uac15 \ud2b8\ub808\uc774\ub4dc \uc678\uc5d0 \ud604\uae08 \ud2b8\ub808\uc774\ub4dc\ub294 \ud558\uc9c0 \uc54a\uaca0\ub2e4\uace0 \ubc1d\ud600\uc654\ub294\ub370, LG\uc5d0\uac8c \uc791\uc740 \uc774\ubcd1\uaddc\ub97c \uc694\uad6c\ud558\ub2e4\uac00 LG\uc758 \ud2b8\ub808\uc774\ub4dc \ubd88\uac00 \ubc29\uce68\uc73c\ub85c \ubb34\uc0b0\ub418\uc5c8\ub2e4\uace0 \ubc1d\ud788\uba70 \ud604\uae08 \ud2b8\ub808\uc774\ub4dc \uc2dc\ub3c4 \uc0ac\uc2e4\uc744 \ubd80\uc778\ud588\ub2e4. KBO\ub294 12\uc6d4 14\uc77c \uc774\uc0ac\ud68c\uc5d0\uc11c \ub17c\uc758\ub97c \uac70\uccd0 \ud2b8\ub808\uc774\ub4dc\uc5d0 \uad00\ud55c \uc785\uc7a5\uc744 \ubc1c\ud45c\ud560 \uc608\uc815\uc73c\ub85c \uc54c\ub824\uc84c\ub2e4."} {"question": "\ud22c\uc0ac\uc580 \ub9c8\uc744 \uadfc\ucc98\uc758 \ucea0\ud504\uc7a5\uc758 \uc774\ub984\uc740?", "paragraph": "\uc0ac\uc6b0\uc2a4\ub9bc \uacf5\uc6d0\uc548\uc5d0\ub294 6\uac1c\uc758 \ud638\ud154/\uc219\uc18c\uac00, \ub178\uc2a4\ub9bc\uc5d0\ub294 \ud55c\uac1c\uc758 \uc219\uc18c\uac00 \uc788\ub2e4. \uc0ac\uc6b0\uc2a4 \ub9bc\uc5d0 \uc788\ub294 \ud638\ud154 \uad00\ub9ac\uc640 \uc608\uc57d\uc740 \uc794\ud14c\ub77c \ud30c\ud06c \uc5d4\ub4dc \ub9ac\uc18c\ud2b8(Xanterra Parks & Resorts)\ud68c\uc0ac\uc5d0\uc11c \ud55c\ub2e4. \ub178\uc2a4\ub9bc\uc5d0 \uc788\ub294 \uc219\uc18c \uc608\uc57d\uc740 \ud3ec\ub808\ubc84\ub9ac\uc18c\ud2b8(Forever Resort)\ub85c \ud574\uc57c \ub41c\ub2e4. \uc5ec\ub984\ucca0\uc758 \uc608\uc57d\uc740 \uc57d 3~6\uac1c\uc6d4 \uc815\ub3c4 \ubbf8\ub9ac \ud574\uc57c \ub420\ub54c\uac00 \ub9ce\ub2e4. \uc0ac\uc6b0\uc2a4\ub9bc\uc5d0\uc11c \uc57d 7\ub9c8\uc77c \ub5a8\uc5b4\uc9c4 \uacf5\uc6d0 \uc785\uad6c\uc5d0 \uc788\ub294 \ud22c\uc0ac\uc580(Tusayan)\uc774\ub780 \ub9c8\uc744\uc5d0 7\uac1c\uc758 \ud638\ud154/\ubaa8\ud154\uc774 \uc788\ub2e4. \uc57d \ud55c\uc2dc\uac04 \uac78\ub9ac\ub294 \uc70c\ub9ac\uc554\uc2a4(Williams)\uc5d0 20\uac1c\uac00 \ub118\ub294 \ud638\ud154\uacfc \ubaa8\ud154\uc774 \uc788\uace0 \uc57d 90\ubd84 \uac78\ub9ac\ub294 \ud50c\ub798\uadf8\uc2a4\ud0dc\ud504(Flagstaff)\uc5d0\ub294 50\uc5ec\uac1c\uc758 \ud638\ud154\uacfc \ubaa8\ud154\uc774 \uc788\ub2e4. \uacf5\uc6d0\uc548\uc5d0\ub294 \ucea0\ud504\uc7a5\uc2dc\uc124\ub3c4 \uc0ac\uc6b0\uc2a4\ub9bc\uc5d0 \ub450 \uacf3, \ub178\uc2a4\ub9bc\uc5d0 \ud558\ub098\uac00 \uc788\ub2e4. \uacf5\uc6d0\ubc16\uc5d0\ub3c4 Ten-X \ub77c\uace0 \ud558\ub294 \ucea0\ud504\uc7a5\uc774 \ud22c\uc0ac\uc580 \ub9c8\uc744 \uadfc\ucc98\uc5d0 \uc788\ub2e4.", "answer": "Ten-X", "sentence": "\uacf5\uc6d0\ubc16\uc5d0\ub3c4 Ten-X \ub77c\uace0 \ud558\ub294 \ucea0\ud504\uc7a5\uc774 \ud22c\uc0ac\uc580 \ub9c8\uc744 \uadfc\ucc98\uc5d0 \uc788\ub2e4.", "paragraph_sentence": "\uc0ac\uc6b0\uc2a4\ub9bc \uacf5\uc6d0\uc548\uc5d0\ub294 6\uac1c\uc758 \ud638\ud154/\uc219\uc18c\uac00, \ub178\uc2a4\ub9bc\uc5d0\ub294 \ud55c\uac1c\uc758 \uc219\uc18c\uac00 \uc788\ub2e4. \uc0ac\uc6b0\uc2a4 \ub9bc\uc5d0 \uc788\ub294 \ud638\ud154 \uad00\ub9ac\uc640 \uc608\uc57d\uc740 \uc794\ud14c\ub77c \ud30c\ud06c \uc5d4\ub4dc \ub9ac\uc18c\ud2b8(Xanterra Parks & Resorts)\ud68c\uc0ac\uc5d0\uc11c \ud55c\ub2e4. \ub178\uc2a4\ub9bc\uc5d0 \uc788\ub294 \uc219\uc18c \uc608\uc57d\uc740 \ud3ec\ub808\ubc84\ub9ac\uc18c\ud2b8(Forever Resort)\ub85c \ud574\uc57c \ub41c\ub2e4. \uc5ec\ub984\ucca0\uc758 \uc608\uc57d\uc740 \uc57d 3~6\uac1c\uc6d4 \uc815\ub3c4 \ubbf8\ub9ac \ud574\uc57c \ub420\ub54c\uac00 \ub9ce\ub2e4. \uc0ac\uc6b0\uc2a4\ub9bc\uc5d0\uc11c \uc57d 7\ub9c8\uc77c \ub5a8\uc5b4\uc9c4 \uacf5\uc6d0 \uc785\uad6c\uc5d0 \uc788\ub294 \ud22c\uc0ac\uc580(Tusayan)\uc774\ub780 \ub9c8\uc744\uc5d0 7\uac1c\uc758 \ud638\ud154/\ubaa8\ud154\uc774 \uc788\ub2e4. \uc57d \ud55c\uc2dc\uac04 \uac78\ub9ac\ub294 \uc70c\ub9ac\uc554\uc2a4(Williams)\uc5d0 20\uac1c\uac00 \ub118\ub294 \ud638\ud154\uacfc \ubaa8\ud154\uc774 \uc788\uace0 \uc57d 90\ubd84 \uac78\ub9ac\ub294 \ud50c\ub798\uadf8\uc2a4\ud0dc\ud504(Flagstaff)\uc5d0\ub294 50\uc5ec\uac1c\uc758 \ud638\ud154\uacfc \ubaa8\ud154\uc774 \uc788\ub2e4. \uacf5\uc6d0\uc548\uc5d0\ub294 \ucea0\ud504\uc7a5\uc2dc\uc124\ub3c4 \uc0ac\uc6b0\uc2a4\ub9bc\uc5d0 \ub450 \uacf3, \ub178\uc2a4\ub9bc\uc5d0 \ud558\ub098\uac00 \uc788\ub2e4. \uacf5\uc6d0\ubc16\uc5d0\ub3c4 Ten-X \ub77c\uace0 \ud558\ub294 \ucea0\ud504\uc7a5\uc774 \ud22c\uc0ac\uc580 \ub9c8\uc744 \uadfc\ucc98\uc5d0 \uc788\ub2e4. ", "paragraph_answer": "\uc0ac\uc6b0\uc2a4\ub9bc \uacf5\uc6d0\uc548\uc5d0\ub294 6\uac1c\uc758 \ud638\ud154/\uc219\uc18c\uac00, \ub178\uc2a4\ub9bc\uc5d0\ub294 \ud55c\uac1c\uc758 \uc219\uc18c\uac00 \uc788\ub2e4. \uc0ac\uc6b0\uc2a4 \ub9bc\uc5d0 \uc788\ub294 \ud638\ud154 \uad00\ub9ac\uc640 \uc608\uc57d\uc740 \uc794\ud14c\ub77c \ud30c\ud06c \uc5d4\ub4dc \ub9ac\uc18c\ud2b8(Xanterra Parks & Resorts)\ud68c\uc0ac\uc5d0\uc11c \ud55c\ub2e4. \ub178\uc2a4\ub9bc\uc5d0 \uc788\ub294 \uc219\uc18c \uc608\uc57d\uc740 \ud3ec\ub808\ubc84\ub9ac\uc18c\ud2b8(Forever Resort)\ub85c \ud574\uc57c \ub41c\ub2e4. \uc5ec\ub984\ucca0\uc758 \uc608\uc57d\uc740 \uc57d 3~6\uac1c\uc6d4 \uc815\ub3c4 \ubbf8\ub9ac \ud574\uc57c \ub420\ub54c\uac00 \ub9ce\ub2e4. \uc0ac\uc6b0\uc2a4\ub9bc\uc5d0\uc11c \uc57d 7\ub9c8\uc77c \ub5a8\uc5b4\uc9c4 \uacf5\uc6d0 \uc785\uad6c\uc5d0 \uc788\ub294 \ud22c\uc0ac\uc580(Tusayan)\uc774\ub780 \ub9c8\uc744\uc5d0 7\uac1c\uc758 \ud638\ud154/\ubaa8\ud154\uc774 \uc788\ub2e4. \uc57d \ud55c\uc2dc\uac04 \uac78\ub9ac\ub294 \uc70c\ub9ac\uc554\uc2a4(Williams)\uc5d0 20\uac1c\uac00 \ub118\ub294 \ud638\ud154\uacfc \ubaa8\ud154\uc774 \uc788\uace0 \uc57d 90\ubd84 \uac78\ub9ac\ub294 \ud50c\ub798\uadf8\uc2a4\ud0dc\ud504(Flagstaff)\uc5d0\ub294 50\uc5ec\uac1c\uc758 \ud638\ud154\uacfc \ubaa8\ud154\uc774 \uc788\ub2e4. \uacf5\uc6d0\uc548\uc5d0\ub294 \ucea0\ud504\uc7a5\uc2dc\uc124\ub3c4 \uc0ac\uc6b0\uc2a4\ub9bc\uc5d0 \ub450 \uacf3, \ub178\uc2a4\ub9bc\uc5d0 \ud558\ub098\uac00 \uc788\ub2e4. \uacf5\uc6d0\ubc16\uc5d0\ub3c4 Ten-X \ub77c\uace0 \ud558\ub294 \ucea0\ud504\uc7a5\uc774 \ud22c\uc0ac\uc580 \ub9c8\uc744 \uadfc\ucc98\uc5d0 \uc788\ub2e4.", "sentence_answer": "\uacf5\uc6d0\ubc16\uc5d0\ub3c4 Ten-X \ub77c\uace0 \ud558\ub294 \ucea0\ud504\uc7a5\uc774 \ud22c\uc0ac\uc580 \ub9c8\uc744 \uadfc\ucc98\uc5d0 \uc788\ub2e4.", "paragraph_id": "6490009-3-1", "paragraph_question": "question: \ud22c\uc0ac\uc580 \ub9c8\uc744 \uadfc\ucc98\uc758 \ucea0\ud504\uc7a5\uc758 \uc774\ub984\uc740?, context: \uc0ac\uc6b0\uc2a4\ub9bc \uacf5\uc6d0\uc548\uc5d0\ub294 6\uac1c\uc758 \ud638\ud154/\uc219\uc18c\uac00, \ub178\uc2a4\ub9bc\uc5d0\ub294 \ud55c\uac1c\uc758 \uc219\uc18c\uac00 \uc788\ub2e4. \uc0ac\uc6b0\uc2a4 \ub9bc\uc5d0 \uc788\ub294 \ud638\ud154 \uad00\ub9ac\uc640 \uc608\uc57d\uc740 \uc794\ud14c\ub77c \ud30c\ud06c \uc5d4\ub4dc \ub9ac\uc18c\ud2b8(Xanterra Parks & Resorts)\ud68c\uc0ac\uc5d0\uc11c \ud55c\ub2e4. \ub178\uc2a4\ub9bc\uc5d0 \uc788\ub294 \uc219\uc18c \uc608\uc57d\uc740 \ud3ec\ub808\ubc84\ub9ac\uc18c\ud2b8(Forever Resort)\ub85c \ud574\uc57c \ub41c\ub2e4. \uc5ec\ub984\ucca0\uc758 \uc608\uc57d\uc740 \uc57d 3~6\uac1c\uc6d4 \uc815\ub3c4 \ubbf8\ub9ac \ud574\uc57c \ub420\ub54c\uac00 \ub9ce\ub2e4. \uc0ac\uc6b0\uc2a4\ub9bc\uc5d0\uc11c \uc57d 7\ub9c8\uc77c \ub5a8\uc5b4\uc9c4 \uacf5\uc6d0 \uc785\uad6c\uc5d0 \uc788\ub294 \ud22c\uc0ac\uc580(Tusayan)\uc774\ub780 \ub9c8\uc744\uc5d0 7\uac1c\uc758 \ud638\ud154/\ubaa8\ud154\uc774 \uc788\ub2e4. \uc57d \ud55c\uc2dc\uac04 \uac78\ub9ac\ub294 \uc70c\ub9ac\uc554\uc2a4(Williams)\uc5d0 20\uac1c\uac00 \ub118\ub294 \ud638\ud154\uacfc \ubaa8\ud154\uc774 \uc788\uace0 \uc57d 90\ubd84 \uac78\ub9ac\ub294 \ud50c\ub798\uadf8\uc2a4\ud0dc\ud504(Flagstaff)\uc5d0\ub294 50\uc5ec\uac1c\uc758 \ud638\ud154\uacfc \ubaa8\ud154\uc774 \uc788\ub2e4. \uacf5\uc6d0\uc548\uc5d0\ub294 \ucea0\ud504\uc7a5\uc2dc\uc124\ub3c4 \uc0ac\uc6b0\uc2a4\ub9bc\uc5d0 \ub450 \uacf3, \ub178\uc2a4\ub9bc\uc5d0 \ud558\ub098\uac00 \uc788\ub2e4. \uacf5\uc6d0\ubc16\uc5d0\ub3c4 Ten-X \ub77c\uace0 \ud558\ub294 \ucea0\ud504\uc7a5\uc774 \ud22c\uc0ac\uc580 \ub9c8\uc744 \uadfc\ucc98\uc5d0 \uc788\ub2e4."} {"question": "\uc8fc\uacc4\uc5f4 \uc704\uc758 \ud56d\uc131\ub4e4\uc758 \uad11\ub3c4\ub4f1\uae09\uc740?", "paragraph": "1943\ub144 W. W. \ubaa8\uac74\uacfc P. C. \ud0a4\ub10c\uc774 \ud56d\uc131\ubd84\ub958\ubc95\uc744 \uac1c\uc815\ud558\uc600\uace0, \uc774\uac78 \ubaa8\uac74\uacfc \ud0a4\ub10c\uc758 \uc774\ub984\uc744 \ub530\uc11c \ubaa8\uac74-\ud0a4\ub10c \ubd84\ub958, \ub610\ub294 MK \ubd84\ub958\ub77c\uace0 \ud55c\ub2e4. MK \ubd84\ub958\ub294 \ud56d\uc131 \uac01\uac01\uc5d0\uac8c \ubd84\uad11\ud615(\ud558\ubc84\ub4dc \ubd84\ub958\ubc95\uc5d0 \uae30\ucd08\ud55c) \ubc0f \uad11\ub3c4\ub4f1\uae09\uc744 \ubc30\uc815\ud588\ub2e4. \ud558\ubc84\ub4dc \ubd84\ub958\ubc95\uc740 \uc218\uc18c\uc120\uc758 \uc138\uae30\uc5d0 \ub530\ub77c \ubcc4\ub4e4\uc5d0\uac8c \uc11c\ub85c \ub2e4\ub978 \uae00\uc790\ub4e4\uc744 \ubc30\uc815\ud588\ub294\ub370(\uadf8\ub54c\ub294 \uc544\uc9c1 \uc2a4\ud399\ud2b8\ub7fc\uacfc \uc628\ub3c4 \uc0ac\uc774\uc758 \uad00\uacc4\uac00 \ubc1d\ud600\uc9c0\uae30 \uc804\uc774\uc5c8\ub2e4), \ud56d\uc131\uc758 \uc628\ub3c4\uc5d0 \ub530\ub978 \uc815\ub82c\uc744 \ud55c \ub4a4 \uc911\ubcf5\ub418\ub294 \uac83\ub4e4\uc744 \uc81c\uac70\ud558\uace0 \ub098\uc790 \ud478\ub978\uc0c9\uc5d0\uc11c \ubd89\uc740\uc0c9\uc73c\ub85c, \uc989 \ud30c\uc7a5\uc774 \uc9e7\uc740 \uc21c\uc11c\ub85c O, B, A, F, G, K, M\uc774\ub77c\ub294 \ubd84\uad11\ud615\ub4e4\uc774 \uc815\ub9bd\ub418\uc5c8\ub2e4. \uad11\ub3c4\ub4f1\uae09\uc740 \uad11\ub3c4\uac00 \ud070 \uac83\uc744 I\ub85c \uc2dc\uc791\ud574\uc11c \uc5b4\ub450\uc6cc\uc9c0\ub294 \uc21c\uc73c\ub85c V\uae4c\uc9c0 \ubd84\ub958\ud588\ub2e4. \uc8fc\uacc4\uc5f4 \uc704\uc758 \ud56d\uc131\ub4e4\uc740 \uad11\ub3c4\ub4f1\uae09 V\uc5d0 \ud574\ub2f9\ud55c\ub2e4.", "answer": "V", "sentence": "\uad11\ub3c4\ub4f1\uae09\uc740 \uad11\ub3c4\uac00 \ud070 \uac83\uc744 I\ub85c \uc2dc\uc791\ud574\uc11c \uc5b4\ub450\uc6cc\uc9c0\ub294 \uc21c\uc73c\ub85c V \uae4c\uc9c0 \ubd84\ub958\ud588\ub2e4.", "paragraph_sentence": "1943\ub144 W. W. \ubaa8\uac74\uacfc P. C. \ud0a4\ub10c\uc774 \ud56d\uc131\ubd84\ub958\ubc95\uc744 \uac1c\uc815\ud558\uc600\uace0, \uc774\uac78 \ubaa8\uac74\uacfc \ud0a4\ub10c\uc758 \uc774\ub984\uc744 \ub530\uc11c \ubaa8\uac74-\ud0a4\ub10c \ubd84\ub958, \ub610\ub294 MK \ubd84\ub958\ub77c\uace0 \ud55c\ub2e4. MK \ubd84\ub958\ub294 \ud56d\uc131 \uac01\uac01\uc5d0\uac8c \ubd84\uad11\ud615(\ud558\ubc84\ub4dc \ubd84\ub958\ubc95\uc5d0 \uae30\ucd08\ud55c) \ubc0f \uad11\ub3c4\ub4f1\uae09\uc744 \ubc30\uc815\ud588\ub2e4. \ud558\ubc84\ub4dc \ubd84\ub958\ubc95\uc740 \uc218\uc18c\uc120\uc758 \uc138\uae30\uc5d0 \ub530\ub77c \ubcc4\ub4e4\uc5d0\uac8c \uc11c\ub85c \ub2e4\ub978 \uae00\uc790\ub4e4\uc744 \ubc30\uc815\ud588\ub294\ub370(\uadf8\ub54c\ub294 \uc544\uc9c1 \uc2a4\ud399\ud2b8\ub7fc\uacfc \uc628\ub3c4 \uc0ac\uc774\uc758 \uad00\uacc4\uac00 \ubc1d\ud600\uc9c0\uae30 \uc804\uc774\uc5c8\ub2e4), \ud56d\uc131\uc758 \uc628\ub3c4\uc5d0 \ub530\ub978 \uc815\ub82c\uc744 \ud55c \ub4a4 \uc911\ubcf5\ub418\ub294 \uac83\ub4e4\uc744 \uc81c\uac70\ud558\uace0 \ub098\uc790 \ud478\ub978\uc0c9\uc5d0\uc11c \ubd89\uc740\uc0c9\uc73c\ub85c, \uc989 \ud30c\uc7a5\uc774 \uc9e7\uc740 \uc21c\uc11c\ub85c O, B, A, F, G, K, M\uc774\ub77c\ub294 \ubd84\uad11\ud615\ub4e4\uc774 \uc815\ub9bd\ub418\uc5c8\ub2e4. \uad11\ub3c4\ub4f1\uae09\uc740 \uad11\ub3c4\uac00 \ud070 \uac83\uc744 I\ub85c \uc2dc\uc791\ud574\uc11c \uc5b4\ub450\uc6cc\uc9c0\ub294 \uc21c\uc73c\ub85c V \uae4c\uc9c0 \ubd84\ub958\ud588\ub2e4. \uc8fc\uacc4\uc5f4 \uc704\uc758 \ud56d\uc131\ub4e4\uc740 \uad11\ub3c4\ub4f1\uae09 V\uc5d0 \ud574\ub2f9\ud55c\ub2e4.", "paragraph_answer": "1943\ub144 W. W. \ubaa8\uac74\uacfc P. C. \ud0a4\ub10c\uc774 \ud56d\uc131\ubd84\ub958\ubc95\uc744 \uac1c\uc815\ud558\uc600\uace0, \uc774\uac78 \ubaa8\uac74\uacfc \ud0a4\ub10c\uc758 \uc774\ub984\uc744 \ub530\uc11c \ubaa8\uac74-\ud0a4\ub10c \ubd84\ub958, \ub610\ub294 MK \ubd84\ub958\ub77c\uace0 \ud55c\ub2e4. MK \ubd84\ub958\ub294 \ud56d\uc131 \uac01\uac01\uc5d0\uac8c \ubd84\uad11\ud615(\ud558\ubc84\ub4dc \ubd84\ub958\ubc95\uc5d0 \uae30\ucd08\ud55c) \ubc0f \uad11\ub3c4\ub4f1\uae09\uc744 \ubc30\uc815\ud588\ub2e4. \ud558\ubc84\ub4dc \ubd84\ub958\ubc95\uc740 \uc218\uc18c\uc120\uc758 \uc138\uae30\uc5d0 \ub530\ub77c \ubcc4\ub4e4\uc5d0\uac8c \uc11c\ub85c \ub2e4\ub978 \uae00\uc790\ub4e4\uc744 \ubc30\uc815\ud588\ub294\ub370(\uadf8\ub54c\ub294 \uc544\uc9c1 \uc2a4\ud399\ud2b8\ub7fc\uacfc \uc628\ub3c4 \uc0ac\uc774\uc758 \uad00\uacc4\uac00 \ubc1d\ud600\uc9c0\uae30 \uc804\uc774\uc5c8\ub2e4), \ud56d\uc131\uc758 \uc628\ub3c4\uc5d0 \ub530\ub978 \uc815\ub82c\uc744 \ud55c \ub4a4 \uc911\ubcf5\ub418\ub294 \uac83\ub4e4\uc744 \uc81c\uac70\ud558\uace0 \ub098\uc790 \ud478\ub978\uc0c9\uc5d0\uc11c \ubd89\uc740\uc0c9\uc73c\ub85c, \uc989 \ud30c\uc7a5\uc774 \uc9e7\uc740 \uc21c\uc11c\ub85c O, B, A, F, G, K, M\uc774\ub77c\ub294 \ubd84\uad11\ud615\ub4e4\uc774 \uc815\ub9bd\ub418\uc5c8\ub2e4. \uad11\ub3c4\ub4f1\uae09\uc740 \uad11\ub3c4\uac00 \ud070 \uac83\uc744 I\ub85c \uc2dc\uc791\ud574\uc11c \uc5b4\ub450\uc6cc\uc9c0\ub294 \uc21c\uc73c\ub85c V \uae4c\uc9c0 \ubd84\ub958\ud588\ub2e4. \uc8fc\uacc4\uc5f4 \uc704\uc758 \ud56d\uc131\ub4e4\uc740 \uad11\ub3c4\ub4f1\uae09 V\uc5d0 \ud574\ub2f9\ud55c\ub2e4.", "sentence_answer": "\uad11\ub3c4\ub4f1\uae09\uc740 \uad11\ub3c4\uac00 \ud070 \uac83\uc744 I\ub85c \uc2dc\uc791\ud574\uc11c \uc5b4\ub450\uc6cc\uc9c0\ub294 \uc21c\uc73c\ub85c V \uae4c\uc9c0 \ubd84\ub958\ud588\ub2e4.", "paragraph_id": "6595843-3-0", "paragraph_question": "question: \uc8fc\uacc4\uc5f4 \uc704\uc758 \ud56d\uc131\ub4e4\uc758 \uad11\ub3c4\ub4f1\uae09\uc740?, context: 1943\ub144 W. W. \ubaa8\uac74\uacfc P. C. \ud0a4\ub10c\uc774 \ud56d\uc131\ubd84\ub958\ubc95\uc744 \uac1c\uc815\ud558\uc600\uace0, \uc774\uac78 \ubaa8\uac74\uacfc \ud0a4\ub10c\uc758 \uc774\ub984\uc744 \ub530\uc11c \ubaa8\uac74-\ud0a4\ub10c \ubd84\ub958, \ub610\ub294 MK \ubd84\ub958\ub77c\uace0 \ud55c\ub2e4. MK \ubd84\ub958\ub294 \ud56d\uc131 \uac01\uac01\uc5d0\uac8c \ubd84\uad11\ud615(\ud558\ubc84\ub4dc \ubd84\ub958\ubc95\uc5d0 \uae30\ucd08\ud55c) \ubc0f \uad11\ub3c4\ub4f1\uae09\uc744 \ubc30\uc815\ud588\ub2e4. \ud558\ubc84\ub4dc \ubd84\ub958\ubc95\uc740 \uc218\uc18c\uc120\uc758 \uc138\uae30\uc5d0 \ub530\ub77c \ubcc4\ub4e4\uc5d0\uac8c \uc11c\ub85c \ub2e4\ub978 \uae00\uc790\ub4e4\uc744 \ubc30\uc815\ud588\ub294\ub370(\uadf8\ub54c\ub294 \uc544\uc9c1 \uc2a4\ud399\ud2b8\ub7fc\uacfc \uc628\ub3c4 \uc0ac\uc774\uc758 \uad00\uacc4\uac00 \ubc1d\ud600\uc9c0\uae30 \uc804\uc774\uc5c8\ub2e4), \ud56d\uc131\uc758 \uc628\ub3c4\uc5d0 \ub530\ub978 \uc815\ub82c\uc744 \ud55c \ub4a4 \uc911\ubcf5\ub418\ub294 \uac83\ub4e4\uc744 \uc81c\uac70\ud558\uace0 \ub098\uc790 \ud478\ub978\uc0c9\uc5d0\uc11c \ubd89\uc740\uc0c9\uc73c\ub85c, \uc989 \ud30c\uc7a5\uc774 \uc9e7\uc740 \uc21c\uc11c\ub85c O, B, A, F, G, K, M\uc774\ub77c\ub294 \ubd84\uad11\ud615\ub4e4\uc774 \uc815\ub9bd\ub418\uc5c8\ub2e4. \uad11\ub3c4\ub4f1\uae09\uc740 \uad11\ub3c4\uac00 \ud070 \uac83\uc744 I\ub85c \uc2dc\uc791\ud574\uc11c \uc5b4\ub450\uc6cc\uc9c0\ub294 \uc21c\uc73c\ub85c V\uae4c\uc9c0 \ubd84\ub958\ud588\ub2e4. \uc8fc\uacc4\uc5f4 \uc704\uc758 \ud56d\uc131\ub4e4\uc740 \uad11\ub3c4\ub4f1\uae09 V\uc5d0 \ud574\ub2f9\ud55c\ub2e4."} {"question": "\uc81c2\uc5f0\ud3c9\ud574\uc804\uc73c\ub85c \uc778\ud574 \uba87 \uba85\uc758 \uc870\uc120\uc778\ubbfc\uad70 \ud574\uad70\uc774 \uc804\uc0ac\ud588\ub294\uac00?", "paragraph": "\uc81c2\uc5f0\ud3c9\ud574\uc804(\u7b2c\u4e8c\u5ef6\u576a\u6d77\u6230)\uc740 2002\ub144 6\uc6d4 29\uc77c \uc5f0\ud3c9\ub3c4 \uadfc\ud574 \ubd81\ubc29\ud55c\uacc4\uc120 \ubd80\uadfc \ud574\uc0c1\uc5d0\uc11c \uc77c\uc5b4\ub09c \ub0a8\ubd81\ud55c \uac04\uc758 \uad70\uc0ac\uc801 \ucda9\ub3cc\uc774\ub2e4. \ub300\ud55c\ubbfc\uad6d \ud574\uad70 \uace0\uc18d\uc815\uc5d0 \ub300\ud55c \ubd81\ud55c \ud574\uad70 \uacbd\ube44\uc815\uc758 \uae30\uc2b5 \uacf5\uaca9\uc73c\ub85c \uc2dc\uc791\ub418\uc5b4 30\ubd84 \uac00\ub7c9 \uc9c4\ud589\ub41c \uc774 \uc804\ud22c\uc5d0\uc11c \uc591\uce21 \ubaa8\ub450 \uc190\uc0c1\uc744 \uc785\uc5c8\ub2e4. \ubd81\ud55c\uad70\uc758 \uc120\uc81c \uacf5\uaca9\uc744 \ub2f9\ud55c \ub300\ud55c\ubbfc\uad6d \ud574\uad70\uc758 \ucc38\uc218\ub9ac 357\ud638\ub294 \uad50\uc804 \ud6c4 \uc608\uc778\ub3c4\uc911 \uce68\ubab0\ud558\uc600\uace0, \uc815\uc7a5(\ucc38\uc218\ub9ac\uae09\uc740 150\ud1a4\uae09\uc73c\ub85c \"\ud568\"\uc774 \uc544\ub2cc\"\uc815\" \uc9c0\ud718\uad00\ub3c4 \"\ud568\uc7a5\"\uc774 \uc544\ub2cc \"\uc815\uc7a5\")\uc744 \ud3ec\ud568\ud55c \uc2b9\ubb34\uc6d0 6\uba85\uc774 \uc804\uc0ac\ub2f9\ud558\uace0 19\uba85\uc774 \ubd80\uc0c1\ub2f9\ud558\ub294 \uc778\uba85\ud53c\ud574\ub97c \uacaa\uc5c8\ub2e4. \uc870\uc120\uc778\ubbfc\uad70 \ud574\uad70\uc18c\uc18d \ub4f1\uc0b0\uacf6 684\ud638\ub3c4 \ub300\ud55c\ubbfc\uad6d \ud574\uad70\uc758 \ubc18\uaca9\uc73c\ub85c \uc804\ud22c\ud6c4 \uc0c1\ub2f9\ud55c \ud53c\ud574\ub97c \uc785\uace0 \uc608\uc778\ub2f9\ud588\ub2e4. \uc81c2\uc5f0\ud3c9\ud574\uc804\uc73c\ub85c \uc870\uc120\uc778\ubbfc\uad70 \ud574\uad70 13\uba85\uc774 \uc804\uc0ac\ud558\uace0, 25\uba85\uc774 \ubd80\uc0c1\ub2f9\ud588\ub2e4.", "answer": "13\uba85", "sentence": "\uc81c2\uc5f0\ud3c9\ud574\uc804\uc73c\ub85c \uc870\uc120\uc778\ubbfc\uad70 \ud574\uad70 13\uba85 \uc774 \uc804\uc0ac\ud558\uace0, 25\uba85\uc774 \ubd80\uc0c1\ub2f9\ud588\ub2e4.", "paragraph_sentence": "\uc81c2\uc5f0\ud3c9\ud574\uc804(\u7b2c\u4e8c\u5ef6\u576a\u6d77\u6230)\uc740 2002\ub144 6\uc6d4 29\uc77c \uc5f0\ud3c9\ub3c4 \uadfc\ud574 \ubd81\ubc29\ud55c\uacc4\uc120 \ubd80\uadfc \ud574\uc0c1\uc5d0\uc11c \uc77c\uc5b4\ub09c \ub0a8\ubd81\ud55c \uac04\uc758 \uad70\uc0ac\uc801 \ucda9\ub3cc\uc774\ub2e4. \ub300\ud55c\ubbfc\uad6d \ud574\uad70 \uace0\uc18d\uc815\uc5d0 \ub300\ud55c \ubd81\ud55c \ud574\uad70 \uacbd\ube44\uc815\uc758 \uae30\uc2b5 \uacf5\uaca9\uc73c\ub85c \uc2dc\uc791\ub418\uc5b4 30\ubd84 \uac00\ub7c9 \uc9c4\ud589\ub41c \uc774 \uc804\ud22c\uc5d0\uc11c \uc591\uce21 \ubaa8\ub450 \uc190\uc0c1\uc744 \uc785\uc5c8\ub2e4. \ubd81\ud55c\uad70\uc758 \uc120\uc81c \uacf5\uaca9\uc744 \ub2f9\ud55c \ub300\ud55c\ubbfc\uad6d \ud574\uad70\uc758 \ucc38\uc218\ub9ac 357\ud638\ub294 \uad50\uc804 \ud6c4 \uc608\uc778\ub3c4\uc911 \uce68\ubab0\ud558\uc600\uace0, \uc815\uc7a5(\ucc38\uc218\ub9ac\uae09\uc740 150\ud1a4\uae09\uc73c\ub85c \"\ud568\"\uc774 \uc544\ub2cc\"\uc815\" \uc9c0\ud718\uad00\ub3c4 \"\ud568\uc7a5\"\uc774 \uc544\ub2cc \"\uc815\uc7a5\")\uc744 \ud3ec\ud568\ud55c \uc2b9\ubb34\uc6d0 6\uba85\uc774 \uc804\uc0ac\ub2f9\ud558\uace0 19\uba85\uc774 \ubd80\uc0c1\ub2f9\ud558\ub294 \uc778\uba85\ud53c\ud574\ub97c \uacaa\uc5c8\ub2e4. \uc870\uc120\uc778\ubbfc\uad70 \ud574\uad70\uc18c\uc18d \ub4f1\uc0b0\uacf6 684\ud638\ub3c4 \ub300\ud55c\ubbfc\uad6d \ud574\uad70\uc758 \ubc18\uaca9\uc73c\ub85c \uc804\ud22c\ud6c4 \uc0c1\ub2f9\ud55c \ud53c\ud574\ub97c \uc785\uace0 \uc608\uc778\ub2f9\ud588\ub2e4. \uc81c2\uc5f0\ud3c9\ud574\uc804\uc73c\ub85c \uc870\uc120\uc778\ubbfc\uad70 \ud574\uad70 13\uba85 \uc774 \uc804\uc0ac\ud558\uace0, 25\uba85\uc774 \ubd80\uc0c1\ub2f9\ud588\ub2e4. ", "paragraph_answer": "\uc81c2\uc5f0\ud3c9\ud574\uc804(\u7b2c\u4e8c\u5ef6\u576a\u6d77\u6230)\uc740 2002\ub144 6\uc6d4 29\uc77c \uc5f0\ud3c9\ub3c4 \uadfc\ud574 \ubd81\ubc29\ud55c\uacc4\uc120 \ubd80\uadfc \ud574\uc0c1\uc5d0\uc11c \uc77c\uc5b4\ub09c \ub0a8\ubd81\ud55c \uac04\uc758 \uad70\uc0ac\uc801 \ucda9\ub3cc\uc774\ub2e4. \ub300\ud55c\ubbfc\uad6d \ud574\uad70 \uace0\uc18d\uc815\uc5d0 \ub300\ud55c \ubd81\ud55c \ud574\uad70 \uacbd\ube44\uc815\uc758 \uae30\uc2b5 \uacf5\uaca9\uc73c\ub85c \uc2dc\uc791\ub418\uc5b4 30\ubd84 \uac00\ub7c9 \uc9c4\ud589\ub41c \uc774 \uc804\ud22c\uc5d0\uc11c \uc591\uce21 \ubaa8\ub450 \uc190\uc0c1\uc744 \uc785\uc5c8\ub2e4. \ubd81\ud55c\uad70\uc758 \uc120\uc81c \uacf5\uaca9\uc744 \ub2f9\ud55c \ub300\ud55c\ubbfc\uad6d \ud574\uad70\uc758 \ucc38\uc218\ub9ac 357\ud638\ub294 \uad50\uc804 \ud6c4 \uc608\uc778\ub3c4\uc911 \uce68\ubab0\ud558\uc600\uace0, \uc815\uc7a5(\ucc38\uc218\ub9ac\uae09\uc740 150\ud1a4\uae09\uc73c\ub85c \"\ud568\"\uc774 \uc544\ub2cc\"\uc815\" \uc9c0\ud718\uad00\ub3c4 \"\ud568\uc7a5\"\uc774 \uc544\ub2cc \"\uc815\uc7a5\")\uc744 \ud3ec\ud568\ud55c \uc2b9\ubb34\uc6d0 6\uba85\uc774 \uc804\uc0ac\ub2f9\ud558\uace0 19\uba85\uc774 \ubd80\uc0c1\ub2f9\ud558\ub294 \uc778\uba85\ud53c\ud574\ub97c \uacaa\uc5c8\ub2e4. \uc870\uc120\uc778\ubbfc\uad70 \ud574\uad70\uc18c\uc18d \ub4f1\uc0b0\uacf6 684\ud638\ub3c4 \ub300\ud55c\ubbfc\uad6d \ud574\uad70\uc758 \ubc18\uaca9\uc73c\ub85c \uc804\ud22c\ud6c4 \uc0c1\ub2f9\ud55c \ud53c\ud574\ub97c \uc785\uace0 \uc608\uc778\ub2f9\ud588\ub2e4. \uc81c2\uc5f0\ud3c9\ud574\uc804\uc73c\ub85c \uc870\uc120\uc778\ubbfc\uad70 \ud574\uad70 13\uba85 \uc774 \uc804\uc0ac\ud558\uace0, 25\uba85\uc774 \ubd80\uc0c1\ub2f9\ud588\ub2e4.", "sentence_answer": "\uc81c2\uc5f0\ud3c9\ud574\uc804\uc73c\ub85c \uc870\uc120\uc778\ubbfc\uad70 \ud574\uad70 13\uba85 \uc774 \uc804\uc0ac\ud558\uace0, 25\uba85\uc774 \ubd80\uc0c1\ub2f9\ud588\ub2e4.", "paragraph_id": "6550788-0-2", "paragraph_question": "question: \uc81c2\uc5f0\ud3c9\ud574\uc804\uc73c\ub85c \uc778\ud574 \uba87 \uba85\uc758 \uc870\uc120\uc778\ubbfc\uad70 \ud574\uad70\uc774 \uc804\uc0ac\ud588\ub294\uac00?, context: \uc81c2\uc5f0\ud3c9\ud574\uc804(\u7b2c\u4e8c\u5ef6\u576a\u6d77\u6230)\uc740 2002\ub144 6\uc6d4 29\uc77c \uc5f0\ud3c9\ub3c4 \uadfc\ud574 \ubd81\ubc29\ud55c\uacc4\uc120 \ubd80\uadfc \ud574\uc0c1\uc5d0\uc11c \uc77c\uc5b4\ub09c \ub0a8\ubd81\ud55c \uac04\uc758 \uad70\uc0ac\uc801 \ucda9\ub3cc\uc774\ub2e4. \ub300\ud55c\ubbfc\uad6d \ud574\uad70 \uace0\uc18d\uc815\uc5d0 \ub300\ud55c \ubd81\ud55c \ud574\uad70 \uacbd\ube44\uc815\uc758 \uae30\uc2b5 \uacf5\uaca9\uc73c\ub85c \uc2dc\uc791\ub418\uc5b4 30\ubd84 \uac00\ub7c9 \uc9c4\ud589\ub41c \uc774 \uc804\ud22c\uc5d0\uc11c \uc591\uce21 \ubaa8\ub450 \uc190\uc0c1\uc744 \uc785\uc5c8\ub2e4. \ubd81\ud55c\uad70\uc758 \uc120\uc81c \uacf5\uaca9\uc744 \ub2f9\ud55c \ub300\ud55c\ubbfc\uad6d \ud574\uad70\uc758 \ucc38\uc218\ub9ac 357\ud638\ub294 \uad50\uc804 \ud6c4 \uc608\uc778\ub3c4\uc911 \uce68\ubab0\ud558\uc600\uace0, \uc815\uc7a5(\ucc38\uc218\ub9ac\uae09\uc740 150\ud1a4\uae09\uc73c\ub85c \"\ud568\"\uc774 \uc544\ub2cc\"\uc815\" \uc9c0\ud718\uad00\ub3c4 \"\ud568\uc7a5\"\uc774 \uc544\ub2cc \"\uc815\uc7a5\")\uc744 \ud3ec\ud568\ud55c \uc2b9\ubb34\uc6d0 6\uba85\uc774 \uc804\uc0ac\ub2f9\ud558\uace0 19\uba85\uc774 \ubd80\uc0c1\ub2f9\ud558\ub294 \uc778\uba85\ud53c\ud574\ub97c \uacaa\uc5c8\ub2e4. \uc870\uc120\uc778\ubbfc\uad70 \ud574\uad70\uc18c\uc18d \ub4f1\uc0b0\uacf6 684\ud638\ub3c4 \ub300\ud55c\ubbfc\uad6d \ud574\uad70\uc758 \ubc18\uaca9\uc73c\ub85c \uc804\ud22c\ud6c4 \uc0c1\ub2f9\ud55c \ud53c\ud574\ub97c \uc785\uace0 \uc608\uc778\ub2f9\ud588\ub2e4. \uc81c2\uc5f0\ud3c9\ud574\uc804\uc73c\ub85c \uc870\uc120\uc778\ubbfc\uad70 \ud574\uad70 13\uba85\uc774 \uc804\uc0ac\ud558\uace0, 25\uba85\uc774 \ubd80\uc0c1\ub2f9\ud588\ub2e4."} {"question": "1379\ub144 \ub450 \ubc88\uc9f8 \uc778\ub450\uc138\ub97c \uac77\uc5b4 \ucc44\uc6b0\uace0\uc790 \ud55c \uae08\uc561\uc758 \uc561\uc218\ub294?", "paragraph": "\uc5d0\ub4dc\uc6cc\ub4dc 3\uc138\uac00 \uc8fd\uae30 \uc9c1\uc804, \uc758\ud68c\ub294 \uc778\ub450\uc138\ub77c \ubd88\ub9ac\ub294 \uc0c8\ub85c\uc6b4 \ud615\ud0dc\uc758 \uc138\uae08\uc744 \ub3c4\uc785\ud588\ub2e4. \uc774\uac83\uc740 14\uc138 \uc774\uc0c1\uc758 \ubaa8\ub4e0 \uc0ac\ub78c \uac01\uac01\uc5d0 \ub300\ud558\uc5ec 4 \ud39c\uc2a4\uc758 \uc138\uae08\uc744 \ubd80\uacfc\ud588\uace0, \uacb0\ud63c\ud55c \ubd80\ubd80\uc5d0\uac8c\ub294 \uc77c\ubd80 \uacf5\uc81c\ub97c \ud558\ub294 \uc138\uae08\uc774\uc5c8\ub2e4. \uc804\uc7c1 \uc804\ube44\uc758 \ubd80\ub2f4\uc744 \uc774\uc804\uc758 \uc138\uae08 \uc81c\ub3c4\ubcf4\ub2e4 \ub113\uc740 \uacbd\uc81c\uc801 \uae30\ubc18\uc5d0 \ud655\uc0b0\uc2dc\ud0a4\uae30 \uc704\ud574 \uace0\uc548\ub41c \uc778\ub450\uc138\ub294 \ub054\ucc0d\ud55c \uc6d0\uc131\uc744 \uc0c0\uc9c0\ub9cc, \uc778\ub450\uc138\ub97c \uac77\uc740 \uacb0\uacfc 22,000 \ud30c\uc6b4\ub4dc\uac00 \uc99d\uc138\ub418\uc5c8\ub2e4. \uc774\ud6c4\ub85c\ub3c4 \uc804\ud669\uc740 \uacc4\uc18d \ubd88\ub9ac\ud558\uac8c \uc9c4\ud589\ub418\uc5c8\ub2e4. \uac15\uc81c \uacf5\ucc44\ub97c \ud1b5\ud574\uc11c \uc77c\ubd80 \ub3c8\uc744 \uac70\ub450\uc5b4\ub0c8\uc9c0\ub9cc, \uc655\uc740 1379\ub144\uc5d0 \uc758\ud68c\uc5d0\uc11c \ucd94\uac00 \uae30\uae08\uc744 \uc694\uad6c\ud574\uc57c \ud588\ub2e4. \ud558\uc6d0\uc740 \uc5b4\ub9b0 \uad6d\uc655\uc744 \uc9c0\uc9c0\ud588\uc9c0\ub9cc, \uac77\uc5b4\uc57c \ud558\ub294 \ub3c8\uc758 \uc561\uc218\uc640 \ub3c8\uc744 \uc0ac\uc6a9\ud560 \uad6d\uc655\uc758 \uc0c1\ub2f4\uc5ed\ub4e4\uc774 \ubd80\ud328 \uc758\ud639\uc744 \ubc1b\uace0 \uc788\ub2e4\ub294 \uc810\uc5d0 \ub300\ud574\uc11c\ub294 \uc6b0\ub824\ub97c \ud45c\ud588\ub2e4. \ub450 \ubc88\uc9f8 \uc778\ub450\uc138\uac00 \uc2b9\uc778\ub418\uc5c8\uace0, \uc774\ubc88\uc5d0\ub294 \uc601\uad6d \uc0ac\ud68c\uc758 \uc77c\uacf1 \uacc4\uae09\uc5d0 \ub530\ub77c \ucc28\ub4f1 \ubd80\uacfc\ub418\uc5b4 \uc0c1\ub958 \uacc4\uce35\uc774 \uc808\ub300 \uc218\uce58\ub85c \ub354 \ub9ce\uc740 \uae08\uc561\uc744 \ub0a9\ubd80\ud588\ub2e4. \ud558\uc9c0\ub9cc \ud0c8\uc138\ub85c \uc778\ud558\uc5ec \ubd88\uacfc 18,600 \ud30c\uc6b4\ub4dc\ubc16\uc5d0 \uc99d\uc138\ub418\uc9c0 \uc54a\uc558\uace0, \uc774\ub294 \ud544\uc694 \uc561\uc218\uc778 50,000 \ud30c\uc6b4\ub4dc\uc5d0 \ud6e8\uc52c \ubabb \ubbf8\uce58\ub294 \uc561\uc218\uc600\ub2e4.", "answer": "50,000 \ud30c\uc6b4\ub4dc", "sentence": "\ud558\uc9c0\ub9cc \ud0c8\uc138\ub85c \uc778\ud558\uc5ec \ubd88\uacfc 18,600 \ud30c\uc6b4\ub4dc\ubc16\uc5d0 \uc99d\uc138\ub418\uc9c0 \uc54a\uc558\uace0, \uc774\ub294 \ud544\uc694 \uc561\uc218\uc778 50,000 \ud30c\uc6b4\ub4dc \uc5d0 \ud6e8\uc52c \ubabb \ubbf8\uce58\ub294 \uc561\uc218\uc600\ub2e4.", "paragraph_sentence": "\uc5d0\ub4dc\uc6cc\ub4dc 3\uc138\uac00 \uc8fd\uae30 \uc9c1\uc804, \uc758\ud68c\ub294 \uc778\ub450\uc138\ub77c \ubd88\ub9ac\ub294 \uc0c8\ub85c\uc6b4 \ud615\ud0dc\uc758 \uc138\uae08\uc744 \ub3c4\uc785\ud588\ub2e4. \uc774\uac83\uc740 14\uc138 \uc774\uc0c1\uc758 \ubaa8\ub4e0 \uc0ac\ub78c \uac01\uac01\uc5d0 \ub300\ud558\uc5ec 4 \ud39c\uc2a4\uc758 \uc138\uae08\uc744 \ubd80\uacfc\ud588\uace0, \uacb0\ud63c\ud55c \ubd80\ubd80\uc5d0\uac8c\ub294 \uc77c\ubd80 \uacf5\uc81c\ub97c \ud558\ub294 \uc138\uae08\uc774\uc5c8\ub2e4. \uc804\uc7c1 \uc804\ube44\uc758 \ubd80\ub2f4\uc744 \uc774\uc804\uc758 \uc138\uae08 \uc81c\ub3c4\ubcf4\ub2e4 \ub113\uc740 \uacbd\uc81c\uc801 \uae30\ubc18\uc5d0 \ud655\uc0b0\uc2dc\ud0a4\uae30 \uc704\ud574 \uace0\uc548\ub41c \uc778\ub450\uc138\ub294 \ub054\ucc0d\ud55c \uc6d0\uc131\uc744 \uc0c0\uc9c0\ub9cc, \uc778\ub450\uc138\ub97c \uac77\uc740 \uacb0\uacfc 22,000 \ud30c\uc6b4\ub4dc\uac00 \uc99d\uc138\ub418\uc5c8\ub2e4. \uc774\ud6c4\ub85c\ub3c4 \uc804\ud669\uc740 \uacc4\uc18d \ubd88\ub9ac\ud558\uac8c \uc9c4\ud589\ub418\uc5c8\ub2e4. \uac15\uc81c \uacf5\ucc44\ub97c \ud1b5\ud574\uc11c \uc77c\ubd80 \ub3c8\uc744 \uac70\ub450\uc5b4\ub0c8\uc9c0\ub9cc, \uc655\uc740 1379\ub144\uc5d0 \uc758\ud68c\uc5d0\uc11c \ucd94\uac00 \uae30\uae08\uc744 \uc694\uad6c\ud574\uc57c \ud588\ub2e4. \ud558\uc6d0\uc740 \uc5b4\ub9b0 \uad6d\uc655\uc744 \uc9c0\uc9c0\ud588\uc9c0\ub9cc, \uac77\uc5b4\uc57c \ud558\ub294 \ub3c8\uc758 \uc561\uc218\uc640 \ub3c8\uc744 \uc0ac\uc6a9\ud560 \uad6d\uc655\uc758 \uc0c1\ub2f4\uc5ed\ub4e4\uc774 \ubd80\ud328 \uc758\ud639\uc744 \ubc1b\uace0 \uc788\ub2e4\ub294 \uc810\uc5d0 \ub300\ud574\uc11c\ub294 \uc6b0\ub824\ub97c \ud45c\ud588\ub2e4. \ub450 \ubc88\uc9f8 \uc778\ub450\uc138\uac00 \uc2b9\uc778\ub418\uc5c8\uace0, \uc774\ubc88\uc5d0\ub294 \uc601\uad6d \uc0ac\ud68c\uc758 \uc77c\uacf1 \uacc4\uae09\uc5d0 \ub530\ub77c \ucc28\ub4f1 \ubd80\uacfc\ub418\uc5b4 \uc0c1\ub958 \uacc4\uce35\uc774 \uc808\ub300 \uc218\uce58\ub85c \ub354 \ub9ce\uc740 \uae08\uc561\uc744 \ub0a9\ubd80\ud588\ub2e4. \ud558\uc9c0\ub9cc \ud0c8\uc138\ub85c \uc778\ud558\uc5ec \ubd88\uacfc 18,600 \ud30c\uc6b4\ub4dc\ubc16\uc5d0 \uc99d\uc138\ub418\uc9c0 \uc54a\uc558\uace0, \uc774\ub294 \ud544\uc694 \uc561\uc218\uc778 50,000 \ud30c\uc6b4\ub4dc \uc5d0 \ud6e8\uc52c \ubabb \ubbf8\uce58\ub294 \uc561\uc218\uc600\ub2e4. ", "paragraph_answer": "\uc5d0\ub4dc\uc6cc\ub4dc 3\uc138\uac00 \uc8fd\uae30 \uc9c1\uc804, \uc758\ud68c\ub294 \uc778\ub450\uc138\ub77c \ubd88\ub9ac\ub294 \uc0c8\ub85c\uc6b4 \ud615\ud0dc\uc758 \uc138\uae08\uc744 \ub3c4\uc785\ud588\ub2e4. \uc774\uac83\uc740 14\uc138 \uc774\uc0c1\uc758 \ubaa8\ub4e0 \uc0ac\ub78c \uac01\uac01\uc5d0 \ub300\ud558\uc5ec 4 \ud39c\uc2a4\uc758 \uc138\uae08\uc744 \ubd80\uacfc\ud588\uace0, \uacb0\ud63c\ud55c \ubd80\ubd80\uc5d0\uac8c\ub294 \uc77c\ubd80 \uacf5\uc81c\ub97c \ud558\ub294 \uc138\uae08\uc774\uc5c8\ub2e4. \uc804\uc7c1 \uc804\ube44\uc758 \ubd80\ub2f4\uc744 \uc774\uc804\uc758 \uc138\uae08 \uc81c\ub3c4\ubcf4\ub2e4 \ub113\uc740 \uacbd\uc81c\uc801 \uae30\ubc18\uc5d0 \ud655\uc0b0\uc2dc\ud0a4\uae30 \uc704\ud574 \uace0\uc548\ub41c \uc778\ub450\uc138\ub294 \ub054\ucc0d\ud55c \uc6d0\uc131\uc744 \uc0c0\uc9c0\ub9cc, \uc778\ub450\uc138\ub97c \uac77\uc740 \uacb0\uacfc 22,000 \ud30c\uc6b4\ub4dc\uac00 \uc99d\uc138\ub418\uc5c8\ub2e4. \uc774\ud6c4\ub85c\ub3c4 \uc804\ud669\uc740 \uacc4\uc18d \ubd88\ub9ac\ud558\uac8c \uc9c4\ud589\ub418\uc5c8\ub2e4. \uac15\uc81c \uacf5\ucc44\ub97c \ud1b5\ud574\uc11c \uc77c\ubd80 \ub3c8\uc744 \uac70\ub450\uc5b4\ub0c8\uc9c0\ub9cc, \uc655\uc740 1379\ub144\uc5d0 \uc758\ud68c\uc5d0\uc11c \ucd94\uac00 \uae30\uae08\uc744 \uc694\uad6c\ud574\uc57c \ud588\ub2e4. \ud558\uc6d0\uc740 \uc5b4\ub9b0 \uad6d\uc655\uc744 \uc9c0\uc9c0\ud588\uc9c0\ub9cc, \uac77\uc5b4\uc57c \ud558\ub294 \ub3c8\uc758 \uc561\uc218\uc640 \ub3c8\uc744 \uc0ac\uc6a9\ud560 \uad6d\uc655\uc758 \uc0c1\ub2f4\uc5ed\ub4e4\uc774 \ubd80\ud328 \uc758\ud639\uc744 \ubc1b\uace0 \uc788\ub2e4\ub294 \uc810\uc5d0 \ub300\ud574\uc11c\ub294 \uc6b0\ub824\ub97c \ud45c\ud588\ub2e4. \ub450 \ubc88\uc9f8 \uc778\ub450\uc138\uac00 \uc2b9\uc778\ub418\uc5c8\uace0, \uc774\ubc88\uc5d0\ub294 \uc601\uad6d \uc0ac\ud68c\uc758 \uc77c\uacf1 \uacc4\uae09\uc5d0 \ub530\ub77c \ucc28\ub4f1 \ubd80\uacfc\ub418\uc5b4 \uc0c1\ub958 \uacc4\uce35\uc774 \uc808\ub300 \uc218\uce58\ub85c \ub354 \ub9ce\uc740 \uae08\uc561\uc744 \ub0a9\ubd80\ud588\ub2e4. \ud558\uc9c0\ub9cc \ud0c8\uc138\ub85c \uc778\ud558\uc5ec \ubd88\uacfc 18,600 \ud30c\uc6b4\ub4dc\ubc16\uc5d0 \uc99d\uc138\ub418\uc9c0 \uc54a\uc558\uace0, \uc774\ub294 \ud544\uc694 \uc561\uc218\uc778 50,000 \ud30c\uc6b4\ub4dc \uc5d0 \ud6e8\uc52c \ubabb \ubbf8\uce58\ub294 \uc561\uc218\uc600\ub2e4.", "sentence_answer": "\ud558\uc9c0\ub9cc \ud0c8\uc138\ub85c \uc778\ud558\uc5ec \ubd88\uacfc 18,600 \ud30c\uc6b4\ub4dc\ubc16\uc5d0 \uc99d\uc138\ub418\uc9c0 \uc54a\uc558\uace0, \uc774\ub294 \ud544\uc694 \uc561\uc218\uc778 50,000 \ud30c\uc6b4\ub4dc \uc5d0 \ud6e8\uc52c \ubabb \ubbf8\uce58\ub294 \uc561\uc218\uc600\ub2e4.", "paragraph_id": "5844060-9-2", "paragraph_question": "question: 1379\ub144 \ub450 \ubc88\uc9f8 \uc778\ub450\uc138\ub97c \uac77\uc5b4 \ucc44\uc6b0\uace0\uc790 \ud55c \uae08\uc561\uc758 \uc561\uc218\ub294?, context: \uc5d0\ub4dc\uc6cc\ub4dc 3\uc138\uac00 \uc8fd\uae30 \uc9c1\uc804, \uc758\ud68c\ub294 \uc778\ub450\uc138\ub77c \ubd88\ub9ac\ub294 \uc0c8\ub85c\uc6b4 \ud615\ud0dc\uc758 \uc138\uae08\uc744 \ub3c4\uc785\ud588\ub2e4. \uc774\uac83\uc740 14\uc138 \uc774\uc0c1\uc758 \ubaa8\ub4e0 \uc0ac\ub78c \uac01\uac01\uc5d0 \ub300\ud558\uc5ec 4 \ud39c\uc2a4\uc758 \uc138\uae08\uc744 \ubd80\uacfc\ud588\uace0, \uacb0\ud63c\ud55c \ubd80\ubd80\uc5d0\uac8c\ub294 \uc77c\ubd80 \uacf5\uc81c\ub97c \ud558\ub294 \uc138\uae08\uc774\uc5c8\ub2e4. \uc804\uc7c1 \uc804\ube44\uc758 \ubd80\ub2f4\uc744 \uc774\uc804\uc758 \uc138\uae08 \uc81c\ub3c4\ubcf4\ub2e4 \ub113\uc740 \uacbd\uc81c\uc801 \uae30\ubc18\uc5d0 \ud655\uc0b0\uc2dc\ud0a4\uae30 \uc704\ud574 \uace0\uc548\ub41c \uc778\ub450\uc138\ub294 \ub054\ucc0d\ud55c \uc6d0\uc131\uc744 \uc0c0\uc9c0\ub9cc, \uc778\ub450\uc138\ub97c \uac77\uc740 \uacb0\uacfc 22,000 \ud30c\uc6b4\ub4dc\uac00 \uc99d\uc138\ub418\uc5c8\ub2e4. \uc774\ud6c4\ub85c\ub3c4 \uc804\ud669\uc740 \uacc4\uc18d \ubd88\ub9ac\ud558\uac8c \uc9c4\ud589\ub418\uc5c8\ub2e4. \uac15\uc81c \uacf5\ucc44\ub97c \ud1b5\ud574\uc11c \uc77c\ubd80 \ub3c8\uc744 \uac70\ub450\uc5b4\ub0c8\uc9c0\ub9cc, \uc655\uc740 1379\ub144\uc5d0 \uc758\ud68c\uc5d0\uc11c \ucd94\uac00 \uae30\uae08\uc744 \uc694\uad6c\ud574\uc57c \ud588\ub2e4. \ud558\uc6d0\uc740 \uc5b4\ub9b0 \uad6d\uc655\uc744 \uc9c0\uc9c0\ud588\uc9c0\ub9cc, \uac77\uc5b4\uc57c \ud558\ub294 \ub3c8\uc758 \uc561\uc218\uc640 \ub3c8\uc744 \uc0ac\uc6a9\ud560 \uad6d\uc655\uc758 \uc0c1\ub2f4\uc5ed\ub4e4\uc774 \ubd80\ud328 \uc758\ud639\uc744 \ubc1b\uace0 \uc788\ub2e4\ub294 \uc810\uc5d0 \ub300\ud574\uc11c\ub294 \uc6b0\ub824\ub97c \ud45c\ud588\ub2e4. \ub450 \ubc88\uc9f8 \uc778\ub450\uc138\uac00 \uc2b9\uc778\ub418\uc5c8\uace0, \uc774\ubc88\uc5d0\ub294 \uc601\uad6d \uc0ac\ud68c\uc758 \uc77c\uacf1 \uacc4\uae09\uc5d0 \ub530\ub77c \ucc28\ub4f1 \ubd80\uacfc\ub418\uc5b4 \uc0c1\ub958 \uacc4\uce35\uc774 \uc808\ub300 \uc218\uce58\ub85c \ub354 \ub9ce\uc740 \uae08\uc561\uc744 \ub0a9\ubd80\ud588\ub2e4. \ud558\uc9c0\ub9cc \ud0c8\uc138\ub85c \uc778\ud558\uc5ec \ubd88\uacfc 18,600 \ud30c\uc6b4\ub4dc\ubc16\uc5d0 \uc99d\uc138\ub418\uc9c0 \uc54a\uc558\uace0, \uc774\ub294 \ud544\uc694 \uc561\uc218\uc778 50,000 \ud30c\uc6b4\ub4dc\uc5d0 \ud6e8\uc52c \ubabb \ubbf8\uce58\ub294 \uc561\uc218\uc600\ub2e4."} {"question": "\uba69\uac94\ub808\ub294 \uc544\uc774\ub4e4\uc758 \ub208 \uc0c9\uae54\uc744 \ubc14\uafb8\uae30 \uc704\ud574 \ub208\uc5d0 \ubb34\uc5c7\uc744 \uc8fc\uc0ac\ud558\uc600\ub294\uac00?", "paragraph": "\uba69\uac94\ub808\ub294 \uc218\uc6a9\uc18c\uc5d0 \ub3c4\ucc29\ud55c \uc218\uac10\uc790\ub4e4 \uc911 \uae30\ud615\uc744 \uac00\uc9c0\uace0 \uc788\ub294 \uc0ac\ub78c\ub4e4\uc5d0\uac8c \uad00\uc2ec\uc774 \ub9ce\uc558\ub294\ub370, \uc774\ub4e4 \uc911\uc5d0\ub294 \uc624\ube44\uce20(Ovitz) \uac00\uc871\ub3c4 \ud3ec\ud568\ub418\uc5b4 \uc788\uc5c8\ub2e4. \ub8e8\ub9c8\ub2c8\uc544 \ucd9c\uc2e0\uc758 \uc608\uc220\uac00 \uac00\uc871\uc778 \uc774\ub4e4\uc740 \uc5f4 \uba85\uc758 \uac00\uc871 \uc911 7\uba85\uc774 \ub09c\uc7c1\uc774\uc600\uc73c\uba70, \ub04c\ub824\uc624\uae30 \uc804\uc5d0\ub294 \ub9b4\ub9ac\ud48b \uace1\uc608\ub2e8(Lilliput Troupe)\uc774\ub77c\ub294 \uc774\ub984\uc73c\ub85c \ub3d9\uc720\ub7fd\uc744 \uc804\uc804\ud558\uba70 \uacf5\uc5f0\uc744 \ud558\uc600\ub2e4. \uba69\uac94\ub808\ub294 \uc774\ub4e4\uc744 \uc885\uc885 '\ub098\uc758 \ub09c\uc7c1\uc774 \uac00\uc871'\uc774\ub77c\uace0 \ubd80\ub974\uba70 \uc544\uaf08\ub2e4. \uba69\uac94\ub808\uc758 \uc2e4\ud5d8\ub4e4\uc740 \uacfc\ud559\uc801 \uac00\uce58\uac00 \ubaa8\ud638\ud55c \uac83\ub4e4\uc774 \ub9ce\uc558\ub294\ub370, \uc544\uc774\ub4e4\uc758 \ub208\uc5d0 \uc5fc\uc0c9\uc57d\uc744 \uc8fc\uc0ac\ud574 \ub208 \uc0c9\uae54\uc744 \ubc14\uafb8\ub294 \uc2e4\ud5d8\uc774\ub098 \ub9c8\ucde8 \uc5c6\ub294 \ub291\uace8 \uc801\ucd9c, \uae30\ud0c0 \uc794\uc778\ud55c \uc678\uacfc \uc2e4\ud5d8\ub4e4\uc774 \ud3ec\ud568\ub418\uc5c8\ub2e4. \uc774\ub7ec\ud55c \uacfc\uc815\uc5d0\uc11c \uc228\uc744 \uac70\ub454 \ud53c\uc2e4\ud5d8\uc790\ub4e4\uc758 \ub208\uc740 \uba69\uac94\ub808\uc758 \uc218\uc9d1\ud488\uc774 \ub418\uc5c8\ub2e4. \ub808\ub098 \uac94\ub9ac\uc13c(Rena Gelissen)\uc774 \uc544\uc6b0\uc288\ube44\uce20\uc5d0 \uc788\uc5c8\ub358 \ub3d9\uc548\uc758 \uc99d\uc5b8\uc5d0 \ub530\ub974\uba74 1943\ub144 10\uc6d4\uacbd \uba69\uac94\ub808\ub294 \uc5ec\uc790 \uc218\uac10\uc790\ub4e4\uc744 \ub300\uc0c1\uc73c\ub85c \uc2e4\ud5d8\uc744 \ud558\uae30\ub3c4 \ud558\uc600\ub2e4\uace0 \ud55c\ub2e4. \uba69\uac94\ub808\ub294 \ud2b9\ubcc4 \uc791\uc5c5\uc744 \ud560 \uc0ac\ub78c\uc744 \uc120\ubc1c\ud55c\ub2e4\ub294 \uba85\ubaa9\uc73c\ub85c \uc5ec\uc790 \uc218\uac10\uc790\ub4e4\uc744 \uc120\ud0dd\ud558\uc600\uc73c\uba70, \uc774\ub97c \uc911\ub178\ub3d9\uc5d0\uc11c \ud574\ubc29\ub420 \uae30\ud68c\ub85c \uc0dd\uac01\ud55c \uc5ec\uc790 \uc218\uac10\uc790\ub4e4\uc740 \uba69\uac94\ub808\uc5d0\uac8c \ub04c\ub824\uac00 \ubd88\uc784 \uc218\uc220\uacfc \ucda9\uaca9 \uc694\ubc95 \ub4f1\uc744 \ubc1b\uc558\uace0, \ub300\ubd80\ubd84 \uc2e4\ud5d8 \ub610\ub294 \uc0ac\ud6c4 \uac10\uc5fc \ub4f1\uc73c\ub85c \uc778\ud558\uc5ec \uc8fd\uc74c\uc744 \ub9de\uc558\ub2e4.", "answer": "\uc5fc\uc0c9\uc57d", "sentence": "\uc544\uc774\ub4e4\uc758 \ub208\uc5d0 \uc5fc\uc0c9\uc57d \uc744 \uc8fc\uc0ac\ud574 \ub208 \uc0c9\uae54\uc744 \ubc14\uafb8\ub294 \uc2e4\ud5d8\uc774\ub098 \ub9c8\ucde8 \uc5c6\ub294 \ub291\uace8 \uc801\ucd9c, \uae30\ud0c0 \uc794\uc778\ud55c \uc678\uacfc \uc2e4\ud5d8\ub4e4\uc774 \ud3ec\ud568\ub418\uc5c8\ub2e4.", "paragraph_sentence": "\uba69\uac94\ub808\ub294 \uc218\uc6a9\uc18c\uc5d0 \ub3c4\ucc29\ud55c \uc218\uac10\uc790\ub4e4 \uc911 \uae30\ud615\uc744 \uac00\uc9c0\uace0 \uc788\ub294 \uc0ac\ub78c\ub4e4\uc5d0\uac8c \uad00\uc2ec\uc774 \ub9ce\uc558\ub294\ub370, \uc774\ub4e4 \uc911\uc5d0\ub294 \uc624\ube44\uce20(Ovitz) \uac00\uc871\ub3c4 \ud3ec\ud568\ub418\uc5b4 \uc788\uc5c8\ub2e4. \ub8e8\ub9c8\ub2c8\uc544 \ucd9c\uc2e0\uc758 \uc608\uc220\uac00 \uac00\uc871\uc778 \uc774\ub4e4\uc740 \uc5f4 \uba85\uc758 \uac00\uc871 \uc911 7\uba85\uc774 \ub09c\uc7c1\uc774\uc600\uc73c\uba70, \ub04c\ub824\uc624\uae30 \uc804\uc5d0\ub294 \ub9b4\ub9ac\ud48b \uace1\uc608\ub2e8(Lilliput Troupe)\uc774\ub77c\ub294 \uc774\ub984\uc73c\ub85c \ub3d9\uc720\ub7fd\uc744 \uc804\uc804\ud558\uba70 \uacf5\uc5f0\uc744 \ud558\uc600\ub2e4. \uba69\uac94\ub808\ub294 \uc774\ub4e4\uc744 \uc885\uc885 '\ub098\uc758 \ub09c\uc7c1\uc774 \uac00\uc871'\uc774\ub77c\uace0 \ubd80\ub974\uba70 \uc544\uaf08\ub2e4. \uba69\uac94\ub808\uc758 \uc2e4\ud5d8\ub4e4\uc740 \uacfc\ud559\uc801 \uac00\uce58\uac00 \ubaa8\ud638\ud55c \uac83\ub4e4\uc774 \ub9ce\uc558\ub294\ub370, \uc544\uc774\ub4e4\uc758 \ub208\uc5d0 \uc5fc\uc0c9\uc57d \uc744 \uc8fc\uc0ac\ud574 \ub208 \uc0c9\uae54\uc744 \ubc14\uafb8\ub294 \uc2e4\ud5d8\uc774\ub098 \ub9c8\ucde8 \uc5c6\ub294 \ub291\uace8 \uc801\ucd9c, \uae30\ud0c0 \uc794\uc778\ud55c \uc678\uacfc \uc2e4\ud5d8\ub4e4\uc774 \ud3ec\ud568\ub418\uc5c8\ub2e4. \uc774\ub7ec\ud55c \uacfc\uc815\uc5d0\uc11c \uc228\uc744 \uac70\ub454 \ud53c\uc2e4\ud5d8\uc790\ub4e4\uc758 \ub208\uc740 \uba69\uac94\ub808\uc758 \uc218\uc9d1\ud488\uc774 \ub418\uc5c8\ub2e4. \ub808\ub098 \uac94\ub9ac\uc13c(Rena Gelissen)\uc774 \uc544\uc6b0\uc288\ube44\uce20\uc5d0 \uc788\uc5c8\ub358 \ub3d9\uc548\uc758 \uc99d\uc5b8\uc5d0 \ub530\ub974\uba74 1943\ub144 10\uc6d4\uacbd \uba69\uac94\ub808\ub294 \uc5ec\uc790 \uc218\uac10\uc790\ub4e4\uc744 \ub300\uc0c1\uc73c\ub85c \uc2e4\ud5d8\uc744 \ud558\uae30\ub3c4 \ud558\uc600\ub2e4\uace0 \ud55c\ub2e4. \uba69\uac94\ub808\ub294 \ud2b9\ubcc4 \uc791\uc5c5\uc744 \ud560 \uc0ac\ub78c\uc744 \uc120\ubc1c\ud55c\ub2e4\ub294 \uba85\ubaa9\uc73c\ub85c \uc5ec\uc790 \uc218\uac10\uc790\ub4e4\uc744 \uc120\ud0dd\ud558\uc600\uc73c\uba70, \uc774\ub97c \uc911\ub178\ub3d9\uc5d0\uc11c \ud574\ubc29\ub420 \uae30\ud68c\ub85c \uc0dd\uac01\ud55c \uc5ec\uc790 \uc218\uac10\uc790\ub4e4\uc740 \uba69\uac94\ub808\uc5d0\uac8c \ub04c\ub824\uac00 \ubd88\uc784 \uc218\uc220\uacfc \ucda9\uaca9 \uc694\ubc95 \ub4f1\uc744 \ubc1b\uc558\uace0, \ub300\ubd80\ubd84 \uc2e4\ud5d8 \ub610\ub294 \uc0ac\ud6c4 \uac10\uc5fc \ub4f1\uc73c\ub85c \uc778\ud558\uc5ec \uc8fd\uc74c\uc744 \ub9de\uc558\ub2e4.", "paragraph_answer": "\uba69\uac94\ub808\ub294 \uc218\uc6a9\uc18c\uc5d0 \ub3c4\ucc29\ud55c \uc218\uac10\uc790\ub4e4 \uc911 \uae30\ud615\uc744 \uac00\uc9c0\uace0 \uc788\ub294 \uc0ac\ub78c\ub4e4\uc5d0\uac8c \uad00\uc2ec\uc774 \ub9ce\uc558\ub294\ub370, \uc774\ub4e4 \uc911\uc5d0\ub294 \uc624\ube44\uce20(Ovitz) \uac00\uc871\ub3c4 \ud3ec\ud568\ub418\uc5b4 \uc788\uc5c8\ub2e4. \ub8e8\ub9c8\ub2c8\uc544 \ucd9c\uc2e0\uc758 \uc608\uc220\uac00 \uac00\uc871\uc778 \uc774\ub4e4\uc740 \uc5f4 \uba85\uc758 \uac00\uc871 \uc911 7\uba85\uc774 \ub09c\uc7c1\uc774\uc600\uc73c\uba70, \ub04c\ub824\uc624\uae30 \uc804\uc5d0\ub294 \ub9b4\ub9ac\ud48b \uace1\uc608\ub2e8(Lilliput Troupe)\uc774\ub77c\ub294 \uc774\ub984\uc73c\ub85c \ub3d9\uc720\ub7fd\uc744 \uc804\uc804\ud558\uba70 \uacf5\uc5f0\uc744 \ud558\uc600\ub2e4. \uba69\uac94\ub808\ub294 \uc774\ub4e4\uc744 \uc885\uc885 '\ub098\uc758 \ub09c\uc7c1\uc774 \uac00\uc871'\uc774\ub77c\uace0 \ubd80\ub974\uba70 \uc544\uaf08\ub2e4. \uba69\uac94\ub808\uc758 \uc2e4\ud5d8\ub4e4\uc740 \uacfc\ud559\uc801 \uac00\uce58\uac00 \ubaa8\ud638\ud55c \uac83\ub4e4\uc774 \ub9ce\uc558\ub294\ub370, \uc544\uc774\ub4e4\uc758 \ub208\uc5d0 \uc5fc\uc0c9\uc57d \uc744 \uc8fc\uc0ac\ud574 \ub208 \uc0c9\uae54\uc744 \ubc14\uafb8\ub294 \uc2e4\ud5d8\uc774\ub098 \ub9c8\ucde8 \uc5c6\ub294 \ub291\uace8 \uc801\ucd9c, \uae30\ud0c0 \uc794\uc778\ud55c \uc678\uacfc \uc2e4\ud5d8\ub4e4\uc774 \ud3ec\ud568\ub418\uc5c8\ub2e4. \uc774\ub7ec\ud55c \uacfc\uc815\uc5d0\uc11c \uc228\uc744 \uac70\ub454 \ud53c\uc2e4\ud5d8\uc790\ub4e4\uc758 \ub208\uc740 \uba69\uac94\ub808\uc758 \uc218\uc9d1\ud488\uc774 \ub418\uc5c8\ub2e4. \ub808\ub098 \uac94\ub9ac\uc13c(Rena Gelissen)\uc774 \uc544\uc6b0\uc288\ube44\uce20\uc5d0 \uc788\uc5c8\ub358 \ub3d9\uc548\uc758 \uc99d\uc5b8\uc5d0 \ub530\ub974\uba74 1943\ub144 10\uc6d4\uacbd \uba69\uac94\ub808\ub294 \uc5ec\uc790 \uc218\uac10\uc790\ub4e4\uc744 \ub300\uc0c1\uc73c\ub85c \uc2e4\ud5d8\uc744 \ud558\uae30\ub3c4 \ud558\uc600\ub2e4\uace0 \ud55c\ub2e4. \uba69\uac94\ub808\ub294 \ud2b9\ubcc4 \uc791\uc5c5\uc744 \ud560 \uc0ac\ub78c\uc744 \uc120\ubc1c\ud55c\ub2e4\ub294 \uba85\ubaa9\uc73c\ub85c \uc5ec\uc790 \uc218\uac10\uc790\ub4e4\uc744 \uc120\ud0dd\ud558\uc600\uc73c\uba70, \uc774\ub97c \uc911\ub178\ub3d9\uc5d0\uc11c \ud574\ubc29\ub420 \uae30\ud68c\ub85c \uc0dd\uac01\ud55c \uc5ec\uc790 \uc218\uac10\uc790\ub4e4\uc740 \uba69\uac94\ub808\uc5d0\uac8c \ub04c\ub824\uac00 \ubd88\uc784 \uc218\uc220\uacfc \ucda9\uaca9 \uc694\ubc95 \ub4f1\uc744 \ubc1b\uc558\uace0, \ub300\ubd80\ubd84 \uc2e4\ud5d8 \ub610\ub294 \uc0ac\ud6c4 \uac10\uc5fc \ub4f1\uc73c\ub85c \uc778\ud558\uc5ec \uc8fd\uc74c\uc744 \ub9de\uc558\ub2e4.", "sentence_answer": "\uc544\uc774\ub4e4\uc758 \ub208\uc5d0 \uc5fc\uc0c9\uc57d \uc744 \uc8fc\uc0ac\ud574 \ub208 \uc0c9\uae54\uc744 \ubc14\uafb8\ub294 \uc2e4\ud5d8\uc774\ub098 \ub9c8\ucde8 \uc5c6\ub294 \ub291\uace8 \uc801\ucd9c, \uae30\ud0c0 \uc794\uc778\ud55c \uc678\uacfc \uc2e4\ud5d8\ub4e4\uc774 \ud3ec\ud568\ub418\uc5c8\ub2e4.", "paragraph_id": "6541836-2-2", "paragraph_question": "question: \uba69\uac94\ub808\ub294 \uc544\uc774\ub4e4\uc758 \ub208 \uc0c9\uae54\uc744 \ubc14\uafb8\uae30 \uc704\ud574 \ub208\uc5d0 \ubb34\uc5c7\uc744 \uc8fc\uc0ac\ud558\uc600\ub294\uac00?, context: \uba69\uac94\ub808\ub294 \uc218\uc6a9\uc18c\uc5d0 \ub3c4\ucc29\ud55c \uc218\uac10\uc790\ub4e4 \uc911 \uae30\ud615\uc744 \uac00\uc9c0\uace0 \uc788\ub294 \uc0ac\ub78c\ub4e4\uc5d0\uac8c \uad00\uc2ec\uc774 \ub9ce\uc558\ub294\ub370, \uc774\ub4e4 \uc911\uc5d0\ub294 \uc624\ube44\uce20(Ovitz) \uac00\uc871\ub3c4 \ud3ec\ud568\ub418\uc5b4 \uc788\uc5c8\ub2e4. \ub8e8\ub9c8\ub2c8\uc544 \ucd9c\uc2e0\uc758 \uc608\uc220\uac00 \uac00\uc871\uc778 \uc774\ub4e4\uc740 \uc5f4 \uba85\uc758 \uac00\uc871 \uc911 7\uba85\uc774 \ub09c\uc7c1\uc774\uc600\uc73c\uba70, \ub04c\ub824\uc624\uae30 \uc804\uc5d0\ub294 \ub9b4\ub9ac\ud48b \uace1\uc608\ub2e8(Lilliput Troupe)\uc774\ub77c\ub294 \uc774\ub984\uc73c\ub85c \ub3d9\uc720\ub7fd\uc744 \uc804\uc804\ud558\uba70 \uacf5\uc5f0\uc744 \ud558\uc600\ub2e4. \uba69\uac94\ub808\ub294 \uc774\ub4e4\uc744 \uc885\uc885 '\ub098\uc758 \ub09c\uc7c1\uc774 \uac00\uc871'\uc774\ub77c\uace0 \ubd80\ub974\uba70 \uc544\uaf08\ub2e4. \uba69\uac94\ub808\uc758 \uc2e4\ud5d8\ub4e4\uc740 \uacfc\ud559\uc801 \uac00\uce58\uac00 \ubaa8\ud638\ud55c \uac83\ub4e4\uc774 \ub9ce\uc558\ub294\ub370, \uc544\uc774\ub4e4\uc758 \ub208\uc5d0 \uc5fc\uc0c9\uc57d\uc744 \uc8fc\uc0ac\ud574 \ub208 \uc0c9\uae54\uc744 \ubc14\uafb8\ub294 \uc2e4\ud5d8\uc774\ub098 \ub9c8\ucde8 \uc5c6\ub294 \ub291\uace8 \uc801\ucd9c, \uae30\ud0c0 \uc794\uc778\ud55c \uc678\uacfc \uc2e4\ud5d8\ub4e4\uc774 \ud3ec\ud568\ub418\uc5c8\ub2e4. \uc774\ub7ec\ud55c \uacfc\uc815\uc5d0\uc11c \uc228\uc744 \uac70\ub454 \ud53c\uc2e4\ud5d8\uc790\ub4e4\uc758 \ub208\uc740 \uba69\uac94\ub808\uc758 \uc218\uc9d1\ud488\uc774 \ub418\uc5c8\ub2e4. \ub808\ub098 \uac94\ub9ac\uc13c(Rena Gelissen)\uc774 \uc544\uc6b0\uc288\ube44\uce20\uc5d0 \uc788\uc5c8\ub358 \ub3d9\uc548\uc758 \uc99d\uc5b8\uc5d0 \ub530\ub974\uba74 1943\ub144 10\uc6d4\uacbd \uba69\uac94\ub808\ub294 \uc5ec\uc790 \uc218\uac10\uc790\ub4e4\uc744 \ub300\uc0c1\uc73c\ub85c \uc2e4\ud5d8\uc744 \ud558\uae30\ub3c4 \ud558\uc600\ub2e4\uace0 \ud55c\ub2e4. \uba69\uac94\ub808\ub294 \ud2b9\ubcc4 \uc791\uc5c5\uc744 \ud560 \uc0ac\ub78c\uc744 \uc120\ubc1c\ud55c\ub2e4\ub294 \uba85\ubaa9\uc73c\ub85c \uc5ec\uc790 \uc218\uac10\uc790\ub4e4\uc744 \uc120\ud0dd\ud558\uc600\uc73c\uba70, \uc774\ub97c \uc911\ub178\ub3d9\uc5d0\uc11c \ud574\ubc29\ub420 \uae30\ud68c\ub85c \uc0dd\uac01\ud55c \uc5ec\uc790 \uc218\uac10\uc790\ub4e4\uc740 \uba69\uac94\ub808\uc5d0\uac8c \ub04c\ub824\uac00 \ubd88\uc784 \uc218\uc220\uacfc \ucda9\uaca9 \uc694\ubc95 \ub4f1\uc744 \ubc1b\uc558\uace0, \ub300\ubd80\ubd84 \uc2e4\ud5d8 \ub610\ub294 \uc0ac\ud6c4 \uac10\uc5fc \ub4f1\uc73c\ub85c \uc778\ud558\uc5ec \uc8fd\uc74c\uc744 \ub9de\uc558\ub2e4."} {"question": "2010\ub144 \uc5ed\ub300 \ub300\ud1b5\ub839 \uc2e0\ub8b0\ub3c4 \uc5ec\ub860\uc870\uc0ac\uc5d0\uc11c 1\uc704\ub97c \ucc28\uc9c0\ud55c \uc0ac\ub78c\uc740 \ub204\uad6c\uc778\uac00?", "paragraph": "2010\ub144 9\uc6d4\uc5d0 \uc2e4\uc2dc\ub41c \uc5ed\ub300 \ub300\ud1b5\ub839 \uc2e0\ub8b0\ub3c4\uc5d0 \uad00\ud55c \uc5ec\ub860\uc870\uc0ac\uc5d0\uc11c \ubc15\uc815\ud76c\ub294 34.2%\ub85c 1\uc704\ub85c \ub098\ud0c0\ub0ac\ub2e4. 2007\ub144\uc5d0 \uc2e4\uc2dc\ub41c \uc870\uc0ac\uc5d0\uc11c\ub294 52.7%, 2009\ub144\uc5d0\ub294 41.8%\ub85c \ub098\ud0c0\ub098 \ub208\uc5d0 \ub744\uac8c \ud558\ub77d\uc138\ub97c \ubcf4\uc600\ub2e4. \uc2dc\uc0acIN\uc740 \uc2e0\ub8b0\ub3c4 \uc870\uc0ac \uacb0\uacfc\uc5d0 \ub300\ud574 \uc774\uba85\ubc15 \ub300\ud1b5\ub839\uc758 \uacbd\uc81c \ubc1c\uc804 \ubc29\uc2dd\uc774 \u2018\ubc15\uc815\ud76c \ud5a5\uc218\u2019\ub97c \uc0c1\ub2f9\ud788 \uc18c\uc9c4\uc2dc\ud0a8 \uac83\uc73c\ub85c \ubd84\uc11d\ud588\ub2e4. \ubc15\uc815\ud76c\uc5d0 \ub300\ud55c \ud3c9\uac00\ub294 \uacc4\uce35\ubcc4\ub85c\ub3c4 \ucc28\uc774\uac00 \ucef8\ub2e4. 40\ub300 \uc774\uc0c1, \ub300\uad6c\uacbd\ubd81, \ud55c\ub098\ub77c\ub2f9, \uc790\uc720\uc120\uc9c4\ub2f9 \uc9c0\uc9c0\uce35\uc5d0\uc11c \ud3c9\uac00\uac00 \ub192\uc558\uc73c\uba70, 20~30\ub300 \uc80a\uc740\uce35, \uc9c4\ubcf4\uc131\ud5a5, \ud654\uc774\ud2b8\uce7c\ub77c, \ub300\ud559\uad50 \uc7ac\ud559 \uc774\uc0c1\uc758 \uace0\ud559\ub825\uce35\uc744 \uc911\uc2ec\uc73c\ub85c \ud3c9\uac00\uac00 \ub0ae\uc558\ub2e4. 2011\ub144 \ub9ac\uc11c\uce58\ubdf0\uac00 \uc870\uc0ac\ud55c \uc5ec\ub860\uc870\uc0ac\uc5d0\uc11c \"\uc804\ud604\uc9c1 \ub300\ud1b5\ub839\ub4e4 \uc911 \uac00\uc7a5 \ud638\uac10\uac00\ub294 \uc0ac\ub78c\uc744 1\uba85 \uc120\ud0dd\ud574\ub2ec\ub77c\"\ub294 \uc9c8\ubb38\uc5d0 \ubc15\uc815\ud76c\ub97c \uc120\ud0dd\ud55c \uc0ac\ub78c\uc740 31.9%\ub85c \ub178\ubb34\ud604 30.3%\ub97c \uc81c\uce58\uace0 1\uc704\ub97c \uae30\ub85d\ud588\uc73c\ub2e4.", "answer": "\ubc15\uc815\ud76c", "sentence": "2010\ub144 9\uc6d4\uc5d0 \uc2e4\uc2dc\ub41c \uc5ed\ub300 \ub300\ud1b5\ub839 \uc2e0\ub8b0\ub3c4\uc5d0 \uad00\ud55c \uc5ec\ub860\uc870\uc0ac\uc5d0\uc11c \ubc15\uc815\ud76c \ub294 34.2%\ub85c 1\uc704\ub85c \ub098\ud0c0\ub0ac\ub2e4.", "paragraph_sentence": " 2010\ub144 9\uc6d4\uc5d0 \uc2e4\uc2dc\ub41c \uc5ed\ub300 \ub300\ud1b5\ub839 \uc2e0\ub8b0\ub3c4\uc5d0 \uad00\ud55c \uc5ec\ub860\uc870\uc0ac\uc5d0\uc11c \ubc15\uc815\ud76c \ub294 34.2%\ub85c 1\uc704\ub85c \ub098\ud0c0\ub0ac\ub2e4. 2007\ub144\uc5d0 \uc2e4\uc2dc\ub41c \uc870\uc0ac\uc5d0\uc11c\ub294 52.7%, 2009\ub144\uc5d0\ub294 41.8%\ub85c \ub098\ud0c0\ub098 \ub208\uc5d0 \ub744\uac8c \ud558\ub77d\uc138\ub97c \ubcf4\uc600\ub2e4. \uc2dc\uc0acIN\uc740 \uc2e0\ub8b0\ub3c4 \uc870\uc0ac \uacb0\uacfc\uc5d0 \ub300\ud574 \uc774\uba85\ubc15 \ub300\ud1b5\ub839\uc758 \uacbd\uc81c \ubc1c\uc804 \ubc29\uc2dd\uc774 \u2018\ubc15\uc815\ud76c \ud5a5\uc218\u2019\ub97c \uc0c1\ub2f9\ud788 \uc18c\uc9c4\uc2dc\ud0a8 \uac83\uc73c\ub85c \ubd84\uc11d\ud588\ub2e4. \ubc15\uc815\ud76c\uc5d0 \ub300\ud55c \ud3c9\uac00\ub294 \uacc4\uce35\ubcc4\ub85c\ub3c4 \ucc28\uc774\uac00 \ucef8\ub2e4. 40\ub300 \uc774\uc0c1, \ub300\uad6c\uacbd\ubd81, \ud55c\ub098\ub77c\ub2f9, \uc790\uc720\uc120\uc9c4\ub2f9 \uc9c0\uc9c0\uce35\uc5d0\uc11c \ud3c9\uac00\uac00 \ub192\uc558\uc73c\uba70, 20~30\ub300 \uc80a\uc740\uce35, \uc9c4\ubcf4\uc131\ud5a5, \ud654\uc774\ud2b8\uce7c\ub77c, \ub300\ud559\uad50 \uc7ac\ud559 \uc774\uc0c1\uc758 \uace0\ud559\ub825\uce35\uc744 \uc911\uc2ec\uc73c\ub85c \ud3c9\uac00\uac00 \ub0ae\uc558\ub2e4. 2011\ub144 \ub9ac\uc11c\uce58\ubdf0\uac00 \uc870\uc0ac\ud55c \uc5ec\ub860\uc870\uc0ac\uc5d0\uc11c \"\uc804\ud604\uc9c1 \ub300\ud1b5\ub839\ub4e4 \uc911 \uac00\uc7a5 \ud638\uac10\uac00\ub294 \uc0ac\ub78c\uc744 1\uba85 \uc120\ud0dd\ud574\ub2ec\ub77c\"\ub294 \uc9c8\ubb38\uc5d0 \ubc15\uc815\ud76c\ub97c \uc120\ud0dd\ud55c \uc0ac\ub78c\uc740 31.9%\ub85c \ub178\ubb34\ud604 30.3%\ub97c \uc81c\uce58\uace0 1\uc704\ub97c \uae30\ub85d\ud588\uc73c\ub2e4.", "paragraph_answer": "2010\ub144 9\uc6d4\uc5d0 \uc2e4\uc2dc\ub41c \uc5ed\ub300 \ub300\ud1b5\ub839 \uc2e0\ub8b0\ub3c4\uc5d0 \uad00\ud55c \uc5ec\ub860\uc870\uc0ac\uc5d0\uc11c \ubc15\uc815\ud76c \ub294 34.2%\ub85c 1\uc704\ub85c \ub098\ud0c0\ub0ac\ub2e4. 2007\ub144\uc5d0 \uc2e4\uc2dc\ub41c \uc870\uc0ac\uc5d0\uc11c\ub294 52.7%, 2009\ub144\uc5d0\ub294 41.8%\ub85c \ub098\ud0c0\ub098 \ub208\uc5d0 \ub744\uac8c \ud558\ub77d\uc138\ub97c \ubcf4\uc600\ub2e4. \uc2dc\uc0acIN\uc740 \uc2e0\ub8b0\ub3c4 \uc870\uc0ac \uacb0\uacfc\uc5d0 \ub300\ud574 \uc774\uba85\ubc15 \ub300\ud1b5\ub839\uc758 \uacbd\uc81c \ubc1c\uc804 \ubc29\uc2dd\uc774 \u2018\ubc15\uc815\ud76c \ud5a5\uc218\u2019\ub97c \uc0c1\ub2f9\ud788 \uc18c\uc9c4\uc2dc\ud0a8 \uac83\uc73c\ub85c \ubd84\uc11d\ud588\ub2e4. \ubc15\uc815\ud76c\uc5d0 \ub300\ud55c \ud3c9\uac00\ub294 \uacc4\uce35\ubcc4\ub85c\ub3c4 \ucc28\uc774\uac00 \ucef8\ub2e4. 40\ub300 \uc774\uc0c1, \ub300\uad6c\uacbd\ubd81, \ud55c\ub098\ub77c\ub2f9, \uc790\uc720\uc120\uc9c4\ub2f9 \uc9c0\uc9c0\uce35\uc5d0\uc11c \ud3c9\uac00\uac00 \ub192\uc558\uc73c\uba70, 20~30\ub300 \uc80a\uc740\uce35, \uc9c4\ubcf4\uc131\ud5a5, \ud654\uc774\ud2b8\uce7c\ub77c, \ub300\ud559\uad50 \uc7ac\ud559 \uc774\uc0c1\uc758 \uace0\ud559\ub825\uce35\uc744 \uc911\uc2ec\uc73c\ub85c \ud3c9\uac00\uac00 \ub0ae\uc558\ub2e4. 2011\ub144 \ub9ac\uc11c\uce58\ubdf0\uac00 \uc870\uc0ac\ud55c \uc5ec\ub860\uc870\uc0ac\uc5d0\uc11c \"\uc804\ud604\uc9c1 \ub300\ud1b5\ub839\ub4e4 \uc911 \uac00\uc7a5 \ud638\uac10\uac00\ub294 \uc0ac\ub78c\uc744 1\uba85 \uc120\ud0dd\ud574\ub2ec\ub77c\"\ub294 \uc9c8\ubb38\uc5d0 \ubc15\uc815\ud76c\ub97c \uc120\ud0dd\ud55c \uc0ac\ub78c\uc740 31.9%\ub85c \ub178\ubb34\ud604 30.3%\ub97c \uc81c\uce58\uace0 1\uc704\ub97c \uae30\ub85d\ud588\uc73c\ub2e4.", "sentence_answer": "2010\ub144 9\uc6d4\uc5d0 \uc2e4\uc2dc\ub41c \uc5ed\ub300 \ub300\ud1b5\ub839 \uc2e0\ub8b0\ub3c4\uc5d0 \uad00\ud55c \uc5ec\ub860\uc870\uc0ac\uc5d0\uc11c \ubc15\uc815\ud76c \ub294 34.2%\ub85c 1\uc704\ub85c \ub098\ud0c0\ub0ac\ub2e4.", "paragraph_id": "6417222-62-0", "paragraph_question": "question: 2010\ub144 \uc5ed\ub300 \ub300\ud1b5\ub839 \uc2e0\ub8b0\ub3c4 \uc5ec\ub860\uc870\uc0ac\uc5d0\uc11c 1\uc704\ub97c \ucc28\uc9c0\ud55c \uc0ac\ub78c\uc740 \ub204\uad6c\uc778\uac00?, context: 2010\ub144 9\uc6d4\uc5d0 \uc2e4\uc2dc\ub41c \uc5ed\ub300 \ub300\ud1b5\ub839 \uc2e0\ub8b0\ub3c4\uc5d0 \uad00\ud55c \uc5ec\ub860\uc870\uc0ac\uc5d0\uc11c \ubc15\uc815\ud76c\ub294 34.2%\ub85c 1\uc704\ub85c \ub098\ud0c0\ub0ac\ub2e4. 2007\ub144\uc5d0 \uc2e4\uc2dc\ub41c \uc870\uc0ac\uc5d0\uc11c\ub294 52.7%, 2009\ub144\uc5d0\ub294 41.8%\ub85c \ub098\ud0c0\ub098 \ub208\uc5d0 \ub744\uac8c \ud558\ub77d\uc138\ub97c \ubcf4\uc600\ub2e4. \uc2dc\uc0acIN\uc740 \uc2e0\ub8b0\ub3c4 \uc870\uc0ac \uacb0\uacfc\uc5d0 \ub300\ud574 \uc774\uba85\ubc15 \ub300\ud1b5\ub839\uc758 \uacbd\uc81c \ubc1c\uc804 \ubc29\uc2dd\uc774 \u2018\ubc15\uc815\ud76c \ud5a5\uc218\u2019\ub97c \uc0c1\ub2f9\ud788 \uc18c\uc9c4\uc2dc\ud0a8 \uac83\uc73c\ub85c \ubd84\uc11d\ud588\ub2e4. \ubc15\uc815\ud76c\uc5d0 \ub300\ud55c \ud3c9\uac00\ub294 \uacc4\uce35\ubcc4\ub85c\ub3c4 \ucc28\uc774\uac00 \ucef8\ub2e4. 40\ub300 \uc774\uc0c1, \ub300\uad6c\uacbd\ubd81, \ud55c\ub098\ub77c\ub2f9, \uc790\uc720\uc120\uc9c4\ub2f9 \uc9c0\uc9c0\uce35\uc5d0\uc11c \ud3c9\uac00\uac00 \ub192\uc558\uc73c\uba70, 20~30\ub300 \uc80a\uc740\uce35, \uc9c4\ubcf4\uc131\ud5a5, \ud654\uc774\ud2b8\uce7c\ub77c, \ub300\ud559\uad50 \uc7ac\ud559 \uc774\uc0c1\uc758 \uace0\ud559\ub825\uce35\uc744 \uc911\uc2ec\uc73c\ub85c \ud3c9\uac00\uac00 \ub0ae\uc558\ub2e4. 2011\ub144 \ub9ac\uc11c\uce58\ubdf0\uac00 \uc870\uc0ac\ud55c \uc5ec\ub860\uc870\uc0ac\uc5d0\uc11c \"\uc804\ud604\uc9c1 \ub300\ud1b5\ub839\ub4e4 \uc911 \uac00\uc7a5 \ud638\uac10\uac00\ub294 \uc0ac\ub78c\uc744 1\uba85 \uc120\ud0dd\ud574\ub2ec\ub77c\"\ub294 \uc9c8\ubb38\uc5d0 \ubc15\uc815\ud76c\ub97c \uc120\ud0dd\ud55c \uc0ac\ub78c\uc740 31.9%\ub85c \ub178\ubb34\ud604 30.3%\ub97c \uc81c\uce58\uace0 1\uc704\ub97c \uae30\ub85d\ud588\uc73c\ub2e4."} {"question": "\ubca8\ub77c\ub8e8\uc2a4 \uccad\uc18c\ub144 \ucf58\uc11c\ud2b8\ub97c \uc8fc\ucd5c\ud55c \ub2e8\uccb4\ub294?", "paragraph": "\ubca8\ub77c\ub8e8\uc2a4 \uacf5\ud654\uad6d \uccad\ub144\ub2e8\uc740 \ubca8\ub77c\ub8e8\uc2a4 \uccad\uc18c\ub144 \ucf58\uc11c\ud2b8\uc640 \uac19\uc740 \uc0ac\ud68c \ud589\uc0ac\ub97c \uc8fc\ucd5c\ud55c\ub2e4. \uadf8\ub7ec\ub098 \ubca8\ub77c\ub8e8\uc2a4 \uacf5\ud654\uad6d \uccad\ub144\ub2e8\uc774 \ud6c4\uc6d0\ud558\ub294 \ucf58\uc11c\ud2b8\uc5d0\uc11c\ub294 \uc2e0\ub098\uce58\uc8fc\uc758\uc790\ub4e4\ub3c4 \ucc38\uc11d\ud560 \ubfd0\ub9cc \uc544\ub2c8\ub77c \uc774\ub4e4\uc758 \uacf5\uc5f0\uc744 \ubcf4\uc5ec\uc8fc\uae30\ub3c4 \ud55c\ub2e4. \uc774\ub85c \ubca8\ub77c\ub8e8\uc2a4 \uacf5\ud654\uad6d \uccad\ub144\ub2e8\uc740 \uc9c0\uc5ed \uc9c0\ub3c4\uc790\ub4e4\uacfc \ud1f4\uc5ed \uad70\uc778\ub4e4\uc5d0\uac8c \ub9ce\uc740 \ube44\ub09c\uc744 \ubc1b\uae30\ub3c4 \ud55c\ub2e4. \ub610\ud55c \ubca8\ub77c\ub8e8\uc2a4 \uacf5\ud654\uad6d \uccad\ub144\ub2e8\uc740 2004\ub144\uc5d0 \ubbf8\uc2a4 \ubca8\ub77c\ub8e8\uc2a4 \uc120\ubc1c \ub300\ud68c\uc758 \uc8fc\ucd5c \uac00\uc6b4\ub370 \ud558\ub098\ub85c \ub098\uc11c\uba74\uc11c \ubbf8\uc2a4 \uc544\uba54\ub9ac\uce74\uc640 \ubbf8\uc2a4 \uc720\ub2c8\ubc84\uc2a4\uc640 \uac19\uc740 \ubc29\uc2dd\uc73c\ub85c \uc9c4\ud589\ud558\uae30\ub3c4 \ud558\uc600\uc73c\uba70 \ubca8\ub77c\ub8e8\uc2a4 \uacf5\ud654\uad6d \uccad\ub144\ub2e8\uc774 \uc815\uce58\uc5d0 \uac1c\uc785\ud558\uc9c0 \uc54a\ub294 \ub3d9\uc548 \uc81c1\uc11c\uae30\ub97c \uc9c0\ub0c8\ub358 \ubbf8\ud558\uc77c \uc624\ub974\ub2e4\ub294 \uc5ec\ub7ec \uacf5\ubb34\uc6d0\ub4e4\uacfc \ud568\uaed8 \ubbf8\uad6d\uc758 2004\ub144 \ubca8\ub77c\ub8e8\uc2a4 \ubbfc\uc8fc\uc8fc\uc758 \ubc95\uc744 \ube44\ub09c\ud558\ub294 \ud3b8\uc9c0\uc5d0 \uc11c\uba85\ud558\uae30\ub3c4 \ud558\uc600\ub2e4. \ubca8\ub77c\ub8e8\uc2a4 \uacf5\ud654\uad6d \uccad\ub144\ub2e8\uc758 \ud559\uc0dd \uac74\uc124 \uc5ec\ub2e8\uc740 \uc18c\ube44\uc5d0\ud2b8 \uc5f0\ubc29\uc758 \uccad\ub144 \ub2e8\uccb4\uc5d0 \uae30\uc6d0\uc744 \ub454\ub2e4.", "answer": "\ubca8\ub77c\ub8e8\uc2a4 \uacf5\ud654\uad6d \uccad\ub144\ub2e8", "sentence": "\ubca8\ub77c\ub8e8\uc2a4 \uacf5\ud654\uad6d \uccad\ub144\ub2e8 \uc740 \ubca8\ub77c\ub8e8\uc2a4 \uccad\uc18c\ub144 \ucf58\uc11c\ud2b8\uc640 \uac19\uc740 \uc0ac\ud68c \ud589\uc0ac\ub97c \uc8fc\ucd5c\ud55c\ub2e4.", "paragraph_sentence": " \ubca8\ub77c\ub8e8\uc2a4 \uacf5\ud654\uad6d \uccad\ub144\ub2e8 \uc740 \ubca8\ub77c\ub8e8\uc2a4 \uccad\uc18c\ub144 \ucf58\uc11c\ud2b8\uc640 \uac19\uc740 \uc0ac\ud68c \ud589\uc0ac\ub97c \uc8fc\ucd5c\ud55c\ub2e4. \uadf8\ub7ec\ub098 \ubca8\ub77c\ub8e8\uc2a4 \uacf5\ud654\uad6d \uccad\ub144\ub2e8\uc774 \ud6c4\uc6d0\ud558\ub294 \ucf58\uc11c\ud2b8\uc5d0\uc11c\ub294 \uc2e0\ub098\uce58\uc8fc\uc758\uc790\ub4e4\ub3c4 \ucc38\uc11d\ud560 \ubfd0\ub9cc \uc544\ub2c8\ub77c \uc774\ub4e4\uc758 \uacf5\uc5f0\uc744 \ubcf4\uc5ec\uc8fc\uae30\ub3c4 \ud55c\ub2e4. \uc774\ub85c \ubca8\ub77c\ub8e8\uc2a4 \uacf5\ud654\uad6d \uccad\ub144\ub2e8\uc740 \uc9c0\uc5ed \uc9c0\ub3c4\uc790\ub4e4\uacfc \ud1f4\uc5ed \uad70\uc778\ub4e4\uc5d0\uac8c \ub9ce\uc740 \ube44\ub09c\uc744 \ubc1b\uae30\ub3c4 \ud55c\ub2e4. \ub610\ud55c \ubca8\ub77c\ub8e8\uc2a4 \uacf5\ud654\uad6d \uccad\ub144\ub2e8\uc740 2004\ub144\uc5d0 \ubbf8\uc2a4 \ubca8\ub77c\ub8e8\uc2a4 \uc120\ubc1c \ub300\ud68c\uc758 \uc8fc\ucd5c \uac00\uc6b4\ub370 \ud558\ub098\ub85c \ub098\uc11c\uba74\uc11c \ubbf8\uc2a4 \uc544\uba54\ub9ac\uce74\uc640 \ubbf8\uc2a4 \uc720\ub2c8\ubc84\uc2a4\uc640 \uac19\uc740 \ubc29\uc2dd\uc73c\ub85c \uc9c4\ud589\ud558\uae30\ub3c4 \ud558\uc600\uc73c\uba70 \ubca8\ub77c\ub8e8\uc2a4 \uacf5\ud654\uad6d \uccad\ub144\ub2e8\uc774 \uc815\uce58\uc5d0 \uac1c\uc785\ud558\uc9c0 \uc54a\ub294 \ub3d9\uc548 \uc81c1\uc11c\uae30\ub97c \uc9c0\ub0c8\ub358 \ubbf8\ud558\uc77c \uc624\ub974\ub2e4\ub294 \uc5ec\ub7ec \uacf5\ubb34\uc6d0\ub4e4\uacfc \ud568\uaed8 \ubbf8\uad6d\uc758 2004\ub144 \ubca8\ub77c\ub8e8\uc2a4 \ubbfc\uc8fc\uc8fc\uc758 \ubc95\uc744 \ube44\ub09c\ud558\ub294 \ud3b8\uc9c0\uc5d0 \uc11c\uba85\ud558\uae30\ub3c4 \ud558\uc600\ub2e4. \ubca8\ub77c\ub8e8\uc2a4 \uacf5\ud654\uad6d \uccad\ub144\ub2e8\uc758 \ud559\uc0dd \uac74\uc124 \uc5ec\ub2e8\uc740 \uc18c\ube44\uc5d0\ud2b8 \uc5f0\ubc29\uc758 \uccad\ub144 \ub2e8\uccb4\uc5d0 \uae30\uc6d0\uc744 \ub454\ub2e4.", "paragraph_answer": " \ubca8\ub77c\ub8e8\uc2a4 \uacf5\ud654\uad6d \uccad\ub144\ub2e8 \uc740 \ubca8\ub77c\ub8e8\uc2a4 \uccad\uc18c\ub144 \ucf58\uc11c\ud2b8\uc640 \uac19\uc740 \uc0ac\ud68c \ud589\uc0ac\ub97c \uc8fc\ucd5c\ud55c\ub2e4. \uadf8\ub7ec\ub098 \ubca8\ub77c\ub8e8\uc2a4 \uacf5\ud654\uad6d \uccad\ub144\ub2e8\uc774 \ud6c4\uc6d0\ud558\ub294 \ucf58\uc11c\ud2b8\uc5d0\uc11c\ub294 \uc2e0\ub098\uce58\uc8fc\uc758\uc790\ub4e4\ub3c4 \ucc38\uc11d\ud560 \ubfd0\ub9cc \uc544\ub2c8\ub77c \uc774\ub4e4\uc758 \uacf5\uc5f0\uc744 \ubcf4\uc5ec\uc8fc\uae30\ub3c4 \ud55c\ub2e4. \uc774\ub85c \ubca8\ub77c\ub8e8\uc2a4 \uacf5\ud654\uad6d \uccad\ub144\ub2e8\uc740 \uc9c0\uc5ed \uc9c0\ub3c4\uc790\ub4e4\uacfc \ud1f4\uc5ed \uad70\uc778\ub4e4\uc5d0\uac8c \ub9ce\uc740 \ube44\ub09c\uc744 \ubc1b\uae30\ub3c4 \ud55c\ub2e4. \ub610\ud55c \ubca8\ub77c\ub8e8\uc2a4 \uacf5\ud654\uad6d \uccad\ub144\ub2e8\uc740 2004\ub144\uc5d0 \ubbf8\uc2a4 \ubca8\ub77c\ub8e8\uc2a4 \uc120\ubc1c \ub300\ud68c\uc758 \uc8fc\ucd5c \uac00\uc6b4\ub370 \ud558\ub098\ub85c \ub098\uc11c\uba74\uc11c \ubbf8\uc2a4 \uc544\uba54\ub9ac\uce74\uc640 \ubbf8\uc2a4 \uc720\ub2c8\ubc84\uc2a4\uc640 \uac19\uc740 \ubc29\uc2dd\uc73c\ub85c \uc9c4\ud589\ud558\uae30\ub3c4 \ud558\uc600\uc73c\uba70 \ubca8\ub77c\ub8e8\uc2a4 \uacf5\ud654\uad6d \uccad\ub144\ub2e8\uc774 \uc815\uce58\uc5d0 \uac1c\uc785\ud558\uc9c0 \uc54a\ub294 \ub3d9\uc548 \uc81c1\uc11c\uae30\ub97c \uc9c0\ub0c8\ub358 \ubbf8\ud558\uc77c \uc624\ub974\ub2e4\ub294 \uc5ec\ub7ec \uacf5\ubb34\uc6d0\ub4e4\uacfc \ud568\uaed8 \ubbf8\uad6d\uc758 2004\ub144 \ubca8\ub77c\ub8e8\uc2a4 \ubbfc\uc8fc\uc8fc\uc758 \ubc95\uc744 \ube44\ub09c\ud558\ub294 \ud3b8\uc9c0\uc5d0 \uc11c\uba85\ud558\uae30\ub3c4 \ud558\uc600\ub2e4. \ubca8\ub77c\ub8e8\uc2a4 \uacf5\ud654\uad6d \uccad\ub144\ub2e8\uc758 \ud559\uc0dd \uac74\uc124 \uc5ec\ub2e8\uc740 \uc18c\ube44\uc5d0\ud2b8 \uc5f0\ubc29\uc758 \uccad\ub144 \ub2e8\uccb4\uc5d0 \uae30\uc6d0\uc744 \ub454\ub2e4.", "sentence_answer": " \ubca8\ub77c\ub8e8\uc2a4 \uacf5\ud654\uad6d \uccad\ub144\ub2e8 \uc740 \ubca8\ub77c\ub8e8\uc2a4 \uccad\uc18c\ub144 \ucf58\uc11c\ud2b8\uc640 \uac19\uc740 \uc0ac\ud68c \ud589\uc0ac\ub97c \uc8fc\ucd5c\ud55c\ub2e4.", "paragraph_id": "6508541-0-0", "paragraph_question": "question: \ubca8\ub77c\ub8e8\uc2a4 \uccad\uc18c\ub144 \ucf58\uc11c\ud2b8\ub97c \uc8fc\ucd5c\ud55c \ub2e8\uccb4\ub294?, context: \ubca8\ub77c\ub8e8\uc2a4 \uacf5\ud654\uad6d \uccad\ub144\ub2e8\uc740 \ubca8\ub77c\ub8e8\uc2a4 \uccad\uc18c\ub144 \ucf58\uc11c\ud2b8\uc640 \uac19\uc740 \uc0ac\ud68c \ud589\uc0ac\ub97c \uc8fc\ucd5c\ud55c\ub2e4. \uadf8\ub7ec\ub098 \ubca8\ub77c\ub8e8\uc2a4 \uacf5\ud654\uad6d \uccad\ub144\ub2e8\uc774 \ud6c4\uc6d0\ud558\ub294 \ucf58\uc11c\ud2b8\uc5d0\uc11c\ub294 \uc2e0\ub098\uce58\uc8fc\uc758\uc790\ub4e4\ub3c4 \ucc38\uc11d\ud560 \ubfd0\ub9cc \uc544\ub2c8\ub77c \uc774\ub4e4\uc758 \uacf5\uc5f0\uc744 \ubcf4\uc5ec\uc8fc\uae30\ub3c4 \ud55c\ub2e4. \uc774\ub85c \ubca8\ub77c\ub8e8\uc2a4 \uacf5\ud654\uad6d \uccad\ub144\ub2e8\uc740 \uc9c0\uc5ed \uc9c0\ub3c4\uc790\ub4e4\uacfc \ud1f4\uc5ed \uad70\uc778\ub4e4\uc5d0\uac8c \ub9ce\uc740 \ube44\ub09c\uc744 \ubc1b\uae30\ub3c4 \ud55c\ub2e4. \ub610\ud55c \ubca8\ub77c\ub8e8\uc2a4 \uacf5\ud654\uad6d \uccad\ub144\ub2e8\uc740 2004\ub144\uc5d0 \ubbf8\uc2a4 \ubca8\ub77c\ub8e8\uc2a4 \uc120\ubc1c \ub300\ud68c\uc758 \uc8fc\ucd5c \uac00\uc6b4\ub370 \ud558\ub098\ub85c \ub098\uc11c\uba74\uc11c \ubbf8\uc2a4 \uc544\uba54\ub9ac\uce74\uc640 \ubbf8\uc2a4 \uc720\ub2c8\ubc84\uc2a4\uc640 \uac19\uc740 \ubc29\uc2dd\uc73c\ub85c \uc9c4\ud589\ud558\uae30\ub3c4 \ud558\uc600\uc73c\uba70 \ubca8\ub77c\ub8e8\uc2a4 \uacf5\ud654\uad6d \uccad\ub144\ub2e8\uc774 \uc815\uce58\uc5d0 \uac1c\uc785\ud558\uc9c0 \uc54a\ub294 \ub3d9\uc548 \uc81c1\uc11c\uae30\ub97c \uc9c0\ub0c8\ub358 \ubbf8\ud558\uc77c \uc624\ub974\ub2e4\ub294 \uc5ec\ub7ec \uacf5\ubb34\uc6d0\ub4e4\uacfc \ud568\uaed8 \ubbf8\uad6d\uc758 2004\ub144 \ubca8\ub77c\ub8e8\uc2a4 \ubbfc\uc8fc\uc8fc\uc758 \ubc95\uc744 \ube44\ub09c\ud558\ub294 \ud3b8\uc9c0\uc5d0 \uc11c\uba85\ud558\uae30\ub3c4 \ud558\uc600\ub2e4. \ubca8\ub77c\ub8e8\uc2a4 \uacf5\ud654\uad6d \uccad\ub144\ub2e8\uc758 \ud559\uc0dd \uac74\uc124 \uc5ec\ub2e8\uc740 \uc18c\ube44\uc5d0\ud2b8 \uc5f0\ubc29\uc758 \uccad\ub144 \ub2e8\uccb4\uc5d0 \uae30\uc6d0\uc744 \ub454\ub2e4."} {"question": "\uc544\ub974\ud0a4\uba54\ub370\uc2a4\uac00 \ubc30\uc560 \ubb34\uac70\uc6b4 \ubb3c\uac74\uc744 \uc62e\uae38 \uc218 \uc788\ub294 \uc7a5\uce58\ub97c \uc5b4\ub5a4 \uac83\uc744 \uc774\uc6a9\ud558\uc5ec \ubc1c\uba85\ud558\uc600\ub294\uac00?", "paragraph": "\uc544\ub974\ud0a4\uba54\ub370\uc2a4\ub294 \u300a\ud3c9\uba74\uc758 \uade0\ud615\u300b\uc5d0\uc11c \uc9c0\ub808\uc758 \uc5ec\ub7ec \uc6d0\ub9ac\ub97c \ubc1d\ud614\ub2e4. \uace0\ub300 \uadf8\ub9ac\uc2a4\uc5d0\uc11c\ub294 \uc544\ub974\ud0a4\uba54\ub370\uc2a4 \uc774\uc678\uc5d0\ub3c4 \uc544\ub9ac\uc2a4\ud1a0\ud154\ub808\uc2a4\uc640 \uadf8\uc758 \ud559\ud30c\uc5d0\uc11c \uc5f0\uad6c\ud558\uc600\ub358 \uc544\ub974\ud0a4\ud0c0\uc2a4 \ub4f1\uc774 \uc9c0\ub808\uc758 \uc6d0\ub9ac\ub97c \uc5f0\uad6c\ud558\uc600\ub2e4. \uc54c\ub809\uc0b0\ub4dc\ub9ac\uc544\uc758 \ud30c\ud478\uc2a4\uac00 \uc4f4 \uae30\ub85d\uc5d0 \ub530\ub974\uba74, \uc544\ub974\ud0a4\uba54\ub370\uc2a4\ub294 \u201c\uc801\ub2f9\ud55c \uc7a5\uc18c\uac00 \uc8fc\uc5b4\uc9c4\ub2e4\uba74 \uc9c0\uad6c\ub77c\ub3c4 \ub4e4\uc5b4\ubcf4\uc774\uaca0\ub2e4\u201d(\uadf8\ub9ac\uc2a4\uc5b4: \u03b4\u1ff6\u03c2 \u03bc\u03bf\u03b9 \u03c0\u1fb6 \u03c3\u03c4\u1ff6 \u03ba\u03b1\u1f76 \u03c4\u1f70\u03bd \u03b3\u1fb6\u03bd \u03ba\u03b9\u03bd\u03ac\u03c3\u03c9)\ub77c\uace0 \ub9d0\ud558\uc600\ub2e4\uace0 \ud55c\ub2e4. \ud50c\ub8e8\ud0c0\ub974\ucf54\uc2a4\ub294 \uc544\ub974\ud0a4\uba54\ub370\uc2a4\uac00 \ubcf5\ud569 \ub3c4\ub974\ub798\uc640 \uc9c0\ub808\ub97c \uc774\uc6a9\ud558\uc5ec \ubc30\uc5d0\uc11c \ubb34\uac70\uc6b4 \ubb3c\uac74\uc744 \uc62e\uae38 \uc218 \uc788\ub294 \uc7a5\uce58\ub97c \ubc1c\uba85\ud558\uc600\ub2e4\uace0 \uc801\uace0 \uc788\ub2e4. \uc774 \uc678\uc5d0\ub3c4 \uc544\ub974\ud0a4\uba54\ub370\uc2a4\ub294 \uc0ac\uac70\ub9ac\uc758 \uc870\uc815\uc774 \uac00\ub2a5\ud55c \ud22c\uc11d\uae30\uc640 \uc8fc\ud589\uac70\ub9ac\uacc4\ub97c \uc81c\uc791\ud558\uc600\ub2e4. \ud1b1\ub2c8\ubc14\ud034\ub97c \uc774\uc6a9\ud55c \uc8fc\ud589\uac70\ub9ac\uacc4\ub294 \uc624\ub298\ub0a0 \uc790\ub3d9\ucc28\uc5d0\uc11c\ub3c4 \uc0ac\uc6a9\ub41c\ub2e4.", "answer": "\ubcf5\ud569 \ub3c4\ub974\ub798\uc640 \uc9c0\ub808", "sentence": "\ud50c\ub8e8\ud0c0\ub974\ucf54\uc2a4\ub294 \uc544\ub974\ud0a4\uba54\ub370\uc2a4\uac00 \ubcf5\ud569 \ub3c4\ub974\ub798\uc640 \uc9c0\ub808 \ub97c \uc774\uc6a9\ud558\uc5ec \ubc30\uc5d0\uc11c \ubb34\uac70\uc6b4 \ubb3c\uac74\uc744 \uc62e\uae38 \uc218 \uc788\ub294 \uc7a5\uce58\ub97c \ubc1c\uba85\ud558\uc600\ub2e4\uace0 \uc801\uace0 \uc788\ub2e4.", "paragraph_sentence": "\uc544\ub974\ud0a4\uba54\ub370\uc2a4\ub294 \u300a\ud3c9\uba74\uc758 \uade0\ud615\u300b\uc5d0\uc11c \uc9c0\ub808\uc758 \uc5ec\ub7ec \uc6d0\ub9ac\ub97c \ubc1d\ud614\ub2e4. \uace0\ub300 \uadf8\ub9ac\uc2a4\uc5d0\uc11c\ub294 \uc544\ub974\ud0a4\uba54\ub370\uc2a4 \uc774\uc678\uc5d0\ub3c4 \uc544\ub9ac\uc2a4\ud1a0\ud154\ub808\uc2a4\uc640 \uadf8\uc758 \ud559\ud30c\uc5d0\uc11c \uc5f0\uad6c\ud558\uc600\ub358 \uc544\ub974\ud0a4\ud0c0\uc2a4 \ub4f1\uc774 \uc9c0\ub808\uc758 \uc6d0\ub9ac\ub97c \uc5f0\uad6c\ud558\uc600\ub2e4. \uc54c\ub809\uc0b0\ub4dc\ub9ac\uc544\uc758 \ud30c\ud478\uc2a4\uac00 \uc4f4 \uae30\ub85d\uc5d0 \ub530\ub974\uba74, \uc544\ub974\ud0a4\uba54\ub370\uc2a4\ub294 \u201c\uc801\ub2f9\ud55c \uc7a5\uc18c\uac00 \uc8fc\uc5b4\uc9c4\ub2e4\uba74 \uc9c0\uad6c\ub77c\ub3c4 \ub4e4\uc5b4\ubcf4\uc774\uaca0\ub2e4\u201d(\uadf8\ub9ac\uc2a4\uc5b4: \u03b4\u1ff6\u03c2 \u03bc\u03bf\u03b9 \u03c0\u1fb6 \u03c3\u03c4\u1ff6 \u03ba\u03b1\u1f76 \u03c4\u1f70\u03bd \u03b3\u1fb6\u03bd \u03ba\u03b9\u03bd\u03ac\u03c3\u03c9)\ub77c\uace0 \ub9d0\ud558\uc600\ub2e4\uace0 \ud55c\ub2e4. \ud50c\ub8e8\ud0c0\ub974\ucf54\uc2a4\ub294 \uc544\ub974\ud0a4\uba54\ub370\uc2a4\uac00 \ubcf5\ud569 \ub3c4\ub974\ub798\uc640 \uc9c0\ub808 \ub97c \uc774\uc6a9\ud558\uc5ec \ubc30\uc5d0\uc11c \ubb34\uac70\uc6b4 \ubb3c\uac74\uc744 \uc62e\uae38 \uc218 \uc788\ub294 \uc7a5\uce58\ub97c \ubc1c\uba85\ud558\uc600\ub2e4\uace0 \uc801\uace0 \uc788\ub2e4. \uc774 \uc678\uc5d0\ub3c4 \uc544\ub974\ud0a4\uba54\ub370\uc2a4\ub294 \uc0ac\uac70\ub9ac\uc758 \uc870\uc815\uc774 \uac00\ub2a5\ud55c \ud22c\uc11d\uae30\uc640 \uc8fc\ud589\uac70\ub9ac\uacc4\ub97c \uc81c\uc791\ud558\uc600\ub2e4. \ud1b1\ub2c8\ubc14\ud034\ub97c \uc774\uc6a9\ud55c \uc8fc\ud589\uac70\ub9ac\uacc4\ub294 \uc624\ub298\ub0a0 \uc790\ub3d9\ucc28\uc5d0\uc11c\ub3c4 \uc0ac\uc6a9\ub41c\ub2e4.", "paragraph_answer": "\uc544\ub974\ud0a4\uba54\ub370\uc2a4\ub294 \u300a\ud3c9\uba74\uc758 \uade0\ud615\u300b\uc5d0\uc11c \uc9c0\ub808\uc758 \uc5ec\ub7ec \uc6d0\ub9ac\ub97c \ubc1d\ud614\ub2e4. \uace0\ub300 \uadf8\ub9ac\uc2a4\uc5d0\uc11c\ub294 \uc544\ub974\ud0a4\uba54\ub370\uc2a4 \uc774\uc678\uc5d0\ub3c4 \uc544\ub9ac\uc2a4\ud1a0\ud154\ub808\uc2a4\uc640 \uadf8\uc758 \ud559\ud30c\uc5d0\uc11c \uc5f0\uad6c\ud558\uc600\ub358 \uc544\ub974\ud0a4\ud0c0\uc2a4 \ub4f1\uc774 \uc9c0\ub808\uc758 \uc6d0\ub9ac\ub97c \uc5f0\uad6c\ud558\uc600\ub2e4. \uc54c\ub809\uc0b0\ub4dc\ub9ac\uc544\uc758 \ud30c\ud478\uc2a4\uac00 \uc4f4 \uae30\ub85d\uc5d0 \ub530\ub974\uba74, \uc544\ub974\ud0a4\uba54\ub370\uc2a4\ub294 \u201c\uc801\ub2f9\ud55c \uc7a5\uc18c\uac00 \uc8fc\uc5b4\uc9c4\ub2e4\uba74 \uc9c0\uad6c\ub77c\ub3c4 \ub4e4\uc5b4\ubcf4\uc774\uaca0\ub2e4\u201d(\uadf8\ub9ac\uc2a4\uc5b4: \u03b4\u1ff6\u03c2 \u03bc\u03bf\u03b9 \u03c0\u1fb6 \u03c3\u03c4\u1ff6 \u03ba\u03b1\u1f76 \u03c4\u1f70\u03bd \u03b3\u1fb6\u03bd \u03ba\u03b9\u03bd\u03ac\u03c3\u03c9)\ub77c\uace0 \ub9d0\ud558\uc600\ub2e4\uace0 \ud55c\ub2e4. \ud50c\ub8e8\ud0c0\ub974\ucf54\uc2a4\ub294 \uc544\ub974\ud0a4\uba54\ub370\uc2a4\uac00 \ubcf5\ud569 \ub3c4\ub974\ub798\uc640 \uc9c0\ub808 \ub97c \uc774\uc6a9\ud558\uc5ec \ubc30\uc5d0\uc11c \ubb34\uac70\uc6b4 \ubb3c\uac74\uc744 \uc62e\uae38 \uc218 \uc788\ub294 \uc7a5\uce58\ub97c \ubc1c\uba85\ud558\uc600\ub2e4\uace0 \uc801\uace0 \uc788\ub2e4. \uc774 \uc678\uc5d0\ub3c4 \uc544\ub974\ud0a4\uba54\ub370\uc2a4\ub294 \uc0ac\uac70\ub9ac\uc758 \uc870\uc815\uc774 \uac00\ub2a5\ud55c \ud22c\uc11d\uae30\uc640 \uc8fc\ud589\uac70\ub9ac\uacc4\ub97c \uc81c\uc791\ud558\uc600\ub2e4. \ud1b1\ub2c8\ubc14\ud034\ub97c \uc774\uc6a9\ud55c \uc8fc\ud589\uac70\ub9ac\uacc4\ub294 \uc624\ub298\ub0a0 \uc790\ub3d9\ucc28\uc5d0\uc11c\ub3c4 \uc0ac\uc6a9\ub41c\ub2e4.", "sentence_answer": "\ud50c\ub8e8\ud0c0\ub974\ucf54\uc2a4\ub294 \uc544\ub974\ud0a4\uba54\ub370\uc2a4\uac00 \ubcf5\ud569 \ub3c4\ub974\ub798\uc640 \uc9c0\ub808 \ub97c \uc774\uc6a9\ud558\uc5ec \ubc30\uc5d0\uc11c \ubb34\uac70\uc6b4 \ubb3c\uac74\uc744 \uc62e\uae38 \uc218 \uc788\ub294 \uc7a5\uce58\ub97c \ubc1c\uba85\ud558\uc600\ub2e4\uace0 \uc801\uace0 \uc788\ub2e4.", "paragraph_id": "6466714-5-1", "paragraph_question": "question: \uc544\ub974\ud0a4\uba54\ub370\uc2a4\uac00 \ubc30\uc560 \ubb34\uac70\uc6b4 \ubb3c\uac74\uc744 \uc62e\uae38 \uc218 \uc788\ub294 \uc7a5\uce58\ub97c \uc5b4\ub5a4 \uac83\uc744 \uc774\uc6a9\ud558\uc5ec \ubc1c\uba85\ud558\uc600\ub294\uac00?, context: \uc544\ub974\ud0a4\uba54\ub370\uc2a4\ub294 \u300a\ud3c9\uba74\uc758 \uade0\ud615\u300b\uc5d0\uc11c \uc9c0\ub808\uc758 \uc5ec\ub7ec \uc6d0\ub9ac\ub97c \ubc1d\ud614\ub2e4. \uace0\ub300 \uadf8\ub9ac\uc2a4\uc5d0\uc11c\ub294 \uc544\ub974\ud0a4\uba54\ub370\uc2a4 \uc774\uc678\uc5d0\ub3c4 \uc544\ub9ac\uc2a4\ud1a0\ud154\ub808\uc2a4\uc640 \uadf8\uc758 \ud559\ud30c\uc5d0\uc11c \uc5f0\uad6c\ud558\uc600\ub358 \uc544\ub974\ud0a4\ud0c0\uc2a4 \ub4f1\uc774 \uc9c0\ub808\uc758 \uc6d0\ub9ac\ub97c \uc5f0\uad6c\ud558\uc600\ub2e4. \uc54c\ub809\uc0b0\ub4dc\ub9ac\uc544\uc758 \ud30c\ud478\uc2a4\uac00 \uc4f4 \uae30\ub85d\uc5d0 \ub530\ub974\uba74, \uc544\ub974\ud0a4\uba54\ub370\uc2a4\ub294 \u201c\uc801\ub2f9\ud55c \uc7a5\uc18c\uac00 \uc8fc\uc5b4\uc9c4\ub2e4\uba74 \uc9c0\uad6c\ub77c\ub3c4 \ub4e4\uc5b4\ubcf4\uc774\uaca0\ub2e4\u201d(\uadf8\ub9ac\uc2a4\uc5b4: \u03b4\u1ff6\u03c2 \u03bc\u03bf\u03b9 \u03c0\u1fb6 \u03c3\u03c4\u1ff6 \u03ba\u03b1\u1f76 \u03c4\u1f70\u03bd \u03b3\u1fb6\u03bd \u03ba\u03b9\u03bd\u03ac\u03c3\u03c9)\ub77c\uace0 \ub9d0\ud558\uc600\ub2e4\uace0 \ud55c\ub2e4. \ud50c\ub8e8\ud0c0\ub974\ucf54\uc2a4\ub294 \uc544\ub974\ud0a4\uba54\ub370\uc2a4\uac00 \ubcf5\ud569 \ub3c4\ub974\ub798\uc640 \uc9c0\ub808\ub97c \uc774\uc6a9\ud558\uc5ec \ubc30\uc5d0\uc11c \ubb34\uac70\uc6b4 \ubb3c\uac74\uc744 \uc62e\uae38 \uc218 \uc788\ub294 \uc7a5\uce58\ub97c \ubc1c\uba85\ud558\uc600\ub2e4\uace0 \uc801\uace0 \uc788\ub2e4. \uc774 \uc678\uc5d0\ub3c4 \uc544\ub974\ud0a4\uba54\ub370\uc2a4\ub294 \uc0ac\uac70\ub9ac\uc758 \uc870\uc815\uc774 \uac00\ub2a5\ud55c \ud22c\uc11d\uae30\uc640 \uc8fc\ud589\uac70\ub9ac\uacc4\ub97c \uc81c\uc791\ud558\uc600\ub2e4. \ud1b1\ub2c8\ubc14\ud034\ub97c \uc774\uc6a9\ud55c \uc8fc\ud589\uac70\ub9ac\uacc4\ub294 \uc624\ub298\ub0a0 \uc790\ub3d9\ucc28\uc5d0\uc11c\ub3c4 \uc0ac\uc6a9\ub41c\ub2e4."} {"question": "\uc640\ud2b8 \ud0c0\uc77c\ub7ec\uc758 \ub09c \ub2f9\uc2dc \uc789\uae00\ub79c\ub4dc \uad6d\uc655\uc740 \ub204\uad6c\uc778\uac00?", "paragraph": "\uae09\uc9c4\uc801 \uc131\uc9c1\uc790 \uc874 \ubcfc\uc758 \uc124\uad50\uac00 \ub09c\ubbfc\ub4e4\uc744 \uace0\ubb34\uc2dc\ucf30\uace0, \ucf04\ud2b8 \uc8fc\uc5d0\uc11c \ubd09\uae30\ud55c \ub18d\ubbfc\uad70\uc758 \uc9c0\ub3c4\uc790\uc778 \uc640\ud2b8 \ud0c0\uc77c\ub7ec\uac00 \uc774\ub044\ub294 \ubd09\uae30\uad70\uc740 \ub7f0\ub358\uc73c\ub85c \uc9c4\uaca9\ud588\ub2e4. \ub18d\ubbfc\uad70\uacfc \uc789\uae00\ub79c\ub4dc \uc655\uad6d \uc815\ubd80 \ub300\ud45c\ub4e4\uc740 \ube14\ub799\ud788\uc2a4\uc5d0\uc11c \ud68c\ub3d9\ud558\uc600\ub2e4. \uc815\ubd80\uce21\uc740 \ub18d\ubbfc\uad70\uc758 \ud574\uc0b0\uacfc \uadc0\uacbd\uc744 \uc885\uc6a9\ud558\uc600\uc73c\ub098 \uc124\ub4dd\uc5d0 \uc2e4\ud328\ud558\uc600\ub2e4. \uc655\uad6d\uad70 \ubcd1\ub825\uc758 \ub300\ubd80\ubd84\uc740 \ud574\uc678\uc640 \uc789\uae00\ub79c\ub4dc\uc758 \ubd81\ubd80\uc5d0 \uc8fc\ub454\ud55c \uc0c1\ud0dc\uc600\uace0, \ub2f9\uc2dc \ub098\uc774 14\uc138\uc600\ub358 \uad6d\uc655 \ub9ac\ucc98\ub4dc 2\uc138\ub294 \ub7f0\ub358\ud0d1\uc73c\ub85c \ud53c\uc2e0\ud558\uc600\ub2e4. 6\uc6d4 13\uc77c, \ub18d\ubbfc\uad70\uc740 \ub7f0\ub358\uc5d0 \uc785\uc131\ud558\uc5ec \ub7f0\ub358 \uc74d\ubbfc\ub4e4\uacfc \ud569\ub958\ud588\ub2e4. \uadf8\ub4e4\uc740 \uac10\uc625\uc744 \uacf5\uaca9\ud558\uace0 \uc0ac\ubcf4\uc774 \uad81\uc804\uacfc \ud15c\ud50c\uc9c0\uad6c\uc758 \ubc95\uc870\uc6d0\uc744 \ud30c\uad34\ud588\uc73c\uba70 \ubc95\ub839\ubb38\uc11c\uc5d0 \ubd88\uc744 \uc9c0\ub974\uace0 \uc655\uad6d \uc815\ubd80\uc640 \uad00\ub828\ub41c \uc790\ub294 \ub204\uad6c\ub4e0 \uc7a1\uc544\ub2e4 \uc8fd\uc600\ub2e4. \ub2e4\uc74c\ub0a0, \ub9ac\ucc98\ub4dc\ub294 \ub9c8\uc77c\uc5d4\ub4dc\uc5d0\uc11c \ub18d\ubbfc\uad70\uacfc \ub9cc\ub098 \ub18d\ub178\uc81c\uc758 \ud3d0\uc9c0\ub97c \ud3ec\ud568\ud55c \uadf8\ub4e4\uc758 \uc694\uad6c \ub300\ubd80\ubd84\uc744 \ub4e4\uc5b4\uc8fc\uae30\ub85c \uc57d\uc18d\ud588\ub2e4. \ub3d9\uc2dc\uc5d0 \ub18d\ubbfc\uad70\uc740 \ub7f0\ub358\ud0d1\uc5d0 \uc9c4\uc785\ud558\uc5ec \ub300\ubc95\uad00 \uc0ac\uc774\uba3c \uc11c\ub4dc\ubc84\ub9ac\uc640 \uc7ac\ubb34\uacbd \ub85c\ubc84\ud2b8 \ud5e4\uc77c\uc2a4\ub97c \ucc3e\uc544\ub0b4 \uc8fd\uc600\ub2e4.", "answer": "\ub9ac\ucc98\ub4dc 2\uc138", "sentence": "\ub2f9\uc2dc \ub098\uc774 14\uc138\uc600\ub358 \uad6d\uc655 \ub9ac\ucc98\ub4dc 2\uc138 \ub294 \ub7f0\ub358\ud0d1\uc73c\ub85c \ud53c\uc2e0\ud558\uc600\ub2e4.", "paragraph_sentence": "\uae09\uc9c4\uc801 \uc131\uc9c1\uc790 \uc874 \ubcfc\uc758 \uc124\uad50\uac00 \ub09c\ubbfc\ub4e4\uc744 \uace0\ubb34\uc2dc\ucf30\uace0, \ucf04\ud2b8 \uc8fc\uc5d0\uc11c \ubd09\uae30\ud55c \ub18d\ubbfc\uad70\uc758 \uc9c0\ub3c4\uc790\uc778 \uc640\ud2b8 \ud0c0\uc77c\ub7ec\uac00 \uc774\ub044\ub294 \ubd09\uae30\uad70\uc740 \ub7f0\ub358\uc73c\ub85c \uc9c4\uaca9\ud588\ub2e4. \ub18d\ubbfc\uad70\uacfc \uc789\uae00\ub79c\ub4dc \uc655\uad6d \uc815\ubd80 \ub300\ud45c\ub4e4\uc740 \ube14\ub799\ud788\uc2a4\uc5d0\uc11c \ud68c\ub3d9\ud558\uc600\ub2e4. \uc815\ubd80\uce21\uc740 \ub18d\ubbfc\uad70\uc758 \ud574\uc0b0\uacfc \uadc0\uacbd\uc744 \uc885\uc6a9\ud558\uc600\uc73c\ub098 \uc124\ub4dd\uc5d0 \uc2e4\ud328\ud558\uc600\ub2e4. \uc655\uad6d\uad70 \ubcd1\ub825\uc758 \ub300\ubd80\ubd84\uc740 \ud574\uc678\uc640 \uc789\uae00\ub79c\ub4dc\uc758 \ubd81\ubd80\uc5d0 \uc8fc\ub454\ud55c \uc0c1\ud0dc\uc600\uace0, \ub2f9\uc2dc \ub098\uc774 14\uc138\uc600\ub358 \uad6d\uc655 \ub9ac\ucc98\ub4dc 2\uc138 \ub294 \ub7f0\ub358\ud0d1\uc73c\ub85c \ud53c\uc2e0\ud558\uc600\ub2e4. 6\uc6d4 13\uc77c, \ub18d\ubbfc\uad70\uc740 \ub7f0\ub358\uc5d0 \uc785\uc131\ud558\uc5ec \ub7f0\ub358 \uc74d\ubbfc\ub4e4\uacfc \ud569\ub958\ud588\ub2e4. \uadf8\ub4e4\uc740 \uac10\uc625\uc744 \uacf5\uaca9\ud558\uace0 \uc0ac\ubcf4\uc774 \uad81\uc804\uacfc \ud15c\ud50c\uc9c0\uad6c\uc758 \ubc95\uc870\uc6d0\uc744 \ud30c\uad34\ud588\uc73c\uba70 \ubc95\ub839\ubb38\uc11c\uc5d0 \ubd88\uc744 \uc9c0\ub974\uace0 \uc655\uad6d \uc815\ubd80\uc640 \uad00\ub828\ub41c \uc790\ub294 \ub204\uad6c\ub4e0 \uc7a1\uc544\ub2e4 \uc8fd\uc600\ub2e4. \ub2e4\uc74c\ub0a0, \ub9ac\ucc98\ub4dc\ub294 \ub9c8\uc77c\uc5d4\ub4dc\uc5d0\uc11c \ub18d\ubbfc\uad70\uacfc \ub9cc\ub098 \ub18d\ub178\uc81c\uc758 \ud3d0\uc9c0\ub97c \ud3ec\ud568\ud55c \uadf8\ub4e4\uc758 \uc694\uad6c \ub300\ubd80\ubd84\uc744 \ub4e4\uc5b4\uc8fc\uae30\ub85c \uc57d\uc18d\ud588\ub2e4. \ub3d9\uc2dc\uc5d0 \ub18d\ubbfc\uad70\uc740 \ub7f0\ub358\ud0d1\uc5d0 \uc9c4\uc785\ud558\uc5ec \ub300\ubc95\uad00 \uc0ac\uc774\uba3c \uc11c\ub4dc\ubc84\ub9ac\uc640 \uc7ac\ubb34\uacbd \ub85c\ubc84\ud2b8 \ud5e4\uc77c\uc2a4\ub97c \ucc3e\uc544\ub0b4 \uc8fd\uc600\ub2e4.", "paragraph_answer": "\uae09\uc9c4\uc801 \uc131\uc9c1\uc790 \uc874 \ubcfc\uc758 \uc124\uad50\uac00 \ub09c\ubbfc\ub4e4\uc744 \uace0\ubb34\uc2dc\ucf30\uace0, \ucf04\ud2b8 \uc8fc\uc5d0\uc11c \ubd09\uae30\ud55c \ub18d\ubbfc\uad70\uc758 \uc9c0\ub3c4\uc790\uc778 \uc640\ud2b8 \ud0c0\uc77c\ub7ec\uac00 \uc774\ub044\ub294 \ubd09\uae30\uad70\uc740 \ub7f0\ub358\uc73c\ub85c \uc9c4\uaca9\ud588\ub2e4. \ub18d\ubbfc\uad70\uacfc \uc789\uae00\ub79c\ub4dc \uc655\uad6d \uc815\ubd80 \ub300\ud45c\ub4e4\uc740 \ube14\ub799\ud788\uc2a4\uc5d0\uc11c \ud68c\ub3d9\ud558\uc600\ub2e4. \uc815\ubd80\uce21\uc740 \ub18d\ubbfc\uad70\uc758 \ud574\uc0b0\uacfc \uadc0\uacbd\uc744 \uc885\uc6a9\ud558\uc600\uc73c\ub098 \uc124\ub4dd\uc5d0 \uc2e4\ud328\ud558\uc600\ub2e4. \uc655\uad6d\uad70 \ubcd1\ub825\uc758 \ub300\ubd80\ubd84\uc740 \ud574\uc678\uc640 \uc789\uae00\ub79c\ub4dc\uc758 \ubd81\ubd80\uc5d0 \uc8fc\ub454\ud55c \uc0c1\ud0dc\uc600\uace0, \ub2f9\uc2dc \ub098\uc774 14\uc138\uc600\ub358 \uad6d\uc655 \ub9ac\ucc98\ub4dc 2\uc138 \ub294 \ub7f0\ub358\ud0d1\uc73c\ub85c \ud53c\uc2e0\ud558\uc600\ub2e4. 6\uc6d4 13\uc77c, \ub18d\ubbfc\uad70\uc740 \ub7f0\ub358\uc5d0 \uc785\uc131\ud558\uc5ec \ub7f0\ub358 \uc74d\ubbfc\ub4e4\uacfc \ud569\ub958\ud588\ub2e4. \uadf8\ub4e4\uc740 \uac10\uc625\uc744 \uacf5\uaca9\ud558\uace0 \uc0ac\ubcf4\uc774 \uad81\uc804\uacfc \ud15c\ud50c\uc9c0\uad6c\uc758 \ubc95\uc870\uc6d0\uc744 \ud30c\uad34\ud588\uc73c\uba70 \ubc95\ub839\ubb38\uc11c\uc5d0 \ubd88\uc744 \uc9c0\ub974\uace0 \uc655\uad6d \uc815\ubd80\uc640 \uad00\ub828\ub41c \uc790\ub294 \ub204\uad6c\ub4e0 \uc7a1\uc544\ub2e4 \uc8fd\uc600\ub2e4. \ub2e4\uc74c\ub0a0, \ub9ac\ucc98\ub4dc\ub294 \ub9c8\uc77c\uc5d4\ub4dc\uc5d0\uc11c \ub18d\ubbfc\uad70\uacfc \ub9cc\ub098 \ub18d\ub178\uc81c\uc758 \ud3d0\uc9c0\ub97c \ud3ec\ud568\ud55c \uadf8\ub4e4\uc758 \uc694\uad6c \ub300\ubd80\ubd84\uc744 \ub4e4\uc5b4\uc8fc\uae30\ub85c \uc57d\uc18d\ud588\ub2e4. \ub3d9\uc2dc\uc5d0 \ub18d\ubbfc\uad70\uc740 \ub7f0\ub358\ud0d1\uc5d0 \uc9c4\uc785\ud558\uc5ec \ub300\ubc95\uad00 \uc0ac\uc774\uba3c \uc11c\ub4dc\ubc84\ub9ac\uc640 \uc7ac\ubb34\uacbd \ub85c\ubc84\ud2b8 \ud5e4\uc77c\uc2a4\ub97c \ucc3e\uc544\ub0b4 \uc8fd\uc600\ub2e4.", "sentence_answer": "\ub2f9\uc2dc \ub098\uc774 14\uc138\uc600\ub358 \uad6d\uc655 \ub9ac\ucc98\ub4dc 2\uc138 \ub294 \ub7f0\ub358\ud0d1\uc73c\ub85c \ud53c\uc2e0\ud558\uc600\ub2e4.", "paragraph_id": "5844060-1-1", "paragraph_question": "question: \uc640\ud2b8 \ud0c0\uc77c\ub7ec\uc758 \ub09c \ub2f9\uc2dc \uc789\uae00\ub79c\ub4dc \uad6d\uc655\uc740 \ub204\uad6c\uc778\uac00?, context: \uae09\uc9c4\uc801 \uc131\uc9c1\uc790 \uc874 \ubcfc\uc758 \uc124\uad50\uac00 \ub09c\ubbfc\ub4e4\uc744 \uace0\ubb34\uc2dc\ucf30\uace0, \ucf04\ud2b8 \uc8fc\uc5d0\uc11c \ubd09\uae30\ud55c \ub18d\ubbfc\uad70\uc758 \uc9c0\ub3c4\uc790\uc778 \uc640\ud2b8 \ud0c0\uc77c\ub7ec\uac00 \uc774\ub044\ub294 \ubd09\uae30\uad70\uc740 \ub7f0\ub358\uc73c\ub85c \uc9c4\uaca9\ud588\ub2e4. \ub18d\ubbfc\uad70\uacfc \uc789\uae00\ub79c\ub4dc \uc655\uad6d \uc815\ubd80 \ub300\ud45c\ub4e4\uc740 \ube14\ub799\ud788\uc2a4\uc5d0\uc11c \ud68c\ub3d9\ud558\uc600\ub2e4. \uc815\ubd80\uce21\uc740 \ub18d\ubbfc\uad70\uc758 \ud574\uc0b0\uacfc \uadc0\uacbd\uc744 \uc885\uc6a9\ud558\uc600\uc73c\ub098 \uc124\ub4dd\uc5d0 \uc2e4\ud328\ud558\uc600\ub2e4. \uc655\uad6d\uad70 \ubcd1\ub825\uc758 \ub300\ubd80\ubd84\uc740 \ud574\uc678\uc640 \uc789\uae00\ub79c\ub4dc\uc758 \ubd81\ubd80\uc5d0 \uc8fc\ub454\ud55c \uc0c1\ud0dc\uc600\uace0, \ub2f9\uc2dc \ub098\uc774 14\uc138\uc600\ub358 \uad6d\uc655 \ub9ac\ucc98\ub4dc 2\uc138\ub294 \ub7f0\ub358\ud0d1\uc73c\ub85c \ud53c\uc2e0\ud558\uc600\ub2e4. 6\uc6d4 13\uc77c, \ub18d\ubbfc\uad70\uc740 \ub7f0\ub358\uc5d0 \uc785\uc131\ud558\uc5ec \ub7f0\ub358 \uc74d\ubbfc\ub4e4\uacfc \ud569\ub958\ud588\ub2e4. \uadf8\ub4e4\uc740 \uac10\uc625\uc744 \uacf5\uaca9\ud558\uace0 \uc0ac\ubcf4\uc774 \uad81\uc804\uacfc \ud15c\ud50c\uc9c0\uad6c\uc758 \ubc95\uc870\uc6d0\uc744 \ud30c\uad34\ud588\uc73c\uba70 \ubc95\ub839\ubb38\uc11c\uc5d0 \ubd88\uc744 \uc9c0\ub974\uace0 \uc655\uad6d \uc815\ubd80\uc640 \uad00\ub828\ub41c \uc790\ub294 \ub204\uad6c\ub4e0 \uc7a1\uc544\ub2e4 \uc8fd\uc600\ub2e4. \ub2e4\uc74c\ub0a0, \ub9ac\ucc98\ub4dc\ub294 \ub9c8\uc77c\uc5d4\ub4dc\uc5d0\uc11c \ub18d\ubbfc\uad70\uacfc \ub9cc\ub098 \ub18d\ub178\uc81c\uc758 \ud3d0\uc9c0\ub97c \ud3ec\ud568\ud55c \uadf8\ub4e4\uc758 \uc694\uad6c \ub300\ubd80\ubd84\uc744 \ub4e4\uc5b4\uc8fc\uae30\ub85c \uc57d\uc18d\ud588\ub2e4. \ub3d9\uc2dc\uc5d0 \ub18d\ubbfc\uad70\uc740 \ub7f0\ub358\ud0d1\uc5d0 \uc9c4\uc785\ud558\uc5ec \ub300\ubc95\uad00 \uc0ac\uc774\uba3c \uc11c\ub4dc\ubc84\ub9ac\uc640 \uc7ac\ubb34\uacbd \ub85c\ubc84\ud2b8 \ud5e4\uc77c\uc2a4\ub97c \ucc3e\uc544\ub0b4 \uc8fd\uc600\ub2e4."} {"question": "1527\ub144 5\uc6d4 6\uc77c \ubc8c\uc5b4\uc9c4 \uc804\ud22c\uc5d0\uc11c \uc0ac\ub9dd\ud55c \uc0e4\ub97c \uacf5\uc791\uc744 \ub300\uc2e0\ud574 \uad70\ub300\uc758 \ucd1d\uc9c0\ud718\uad8c\uc744 \ub9e1\uc740 \uc0ac\ub78c\uc740?", "paragraph": "5\uc6d4 6\uc77c \uc2e0\uc131 \ub85c\ub9c8 \uc81c\uad6d \uad70\ub300\ub294 \uc794\ub2c8\ucf5c\ub85c\uc640 \ubc14\ud2f0\uce78 \uc5b8\ub355 \ucabd\uc5d0 \uc788\ub294 \uc131\ubcbd\ub4e4\uc744 \uacf5\uaca9\ud558\uc600\ub2e4. \uc0e4\ub97c \uacf5\uc791\uc740 \ubca4\ubca0\ub204\ud1a0 \uccbc\ub9ac\ub2c8\uac00 \uc3dc \ucd1d\uc744 \ub9de\uace0 \uce58\uba85\uc0c1\uc744 \uc785\uc5c8\ub2e4. \uc0e4\ub97c \uacf5\uc791\uc740 \ubcd1\uc0ac\ub4e4\uc5d0\uac8c \uc790\uc2e0\uc758 \ubd80\uc0c1\uc744 \uc228\uae30\uae30 \uc704\ud558\uc5ec \ud558\uc580 \ub9dd\ud1a0\ub97c \uac78\ucce4\ub294\ub370, \ubcf8\ub798 \uc758\ub3c4\ud55c \ubc14\uc640\ub294 \ub2e4\ub974\uac8c \uc801\ub4e4\uc5d0\uac8c \uadf8\uc758 \uc2e0\ubd84\uc744 \ub178\ucd9c\uc2dc\ud0a4\ub294 \uaf34\uc774 \ub418\uace0 \ub9d0\uc558\ub2e4. \uc2e0\uc131 \ub85c\ub9c8 \uc81c\uad6d\uc758 \uc9c0\ud718\uad00 \uac00\uc6b4\ub370 \uac00\uc7a5 \uc874\uacbd\ubc1b\uc740 \uc9c0\ud718\uad00\uc774\uc5c8\ub358 \uc0e4\ub97c \uacf5\uc791\uc758 \uc8fd\uc74c\uc740 \uace7 \ubcd1\uc0ac\ub4e4\uc758 \uaddc\uc728\uc774 \ubb34\ub108\uc9c0\ub294 \uacb0\uacfc\ub97c \uc57c\uae30\ud588\uc73c\uba70, \uc774\ub4e4\uc740 \uac19\uc740 \ub0a0 \ub85c\ub9c8\uc758 \uc131\ubcbd\uc744 \ud568\ub77d\uc2dc\ud0a4\ub294\ub370 \uc131\uacf5\ud588\ub2e4. \uc804\uc0ac\ud55c \uc0e4\ub97c \uacf5\uc791\uc744 \ub300\uc2e0\ud574 \ud544\ub9ac\ubca0\ub974 \ub4dc \uc0ec\ub871\uac00 \uad70\ub300\uc758 \ucd1d\uc9c0\ud718\uad8c\uc790\uac00 \ub418\uc5c8\uc9c0\ub9cc, \uc0e4\ub97c \uacf5\uc791\uacfc \uac19\uc740 \uc778\uae30\ub97c \ub204\ub9ac\uc9c0\ub3c4 \ub450\ub824\uc6c0\uc758 \ub300\uc0c1\ub3c4 \ub418\uc9c0 \ubabb\ud588\ub2e4.", "answer": "\ud544\ub9ac\ubca0\ub974 \ub4dc \uc0ec\ub871", "sentence": "\uc804\uc0ac\ud55c \uc0e4\ub97c \uacf5\uc791\uc744 \ub300\uc2e0\ud574 \ud544\ub9ac\ubca0\ub974 \ub4dc \uc0ec\ub871 \uac00 \uad70\ub300\uc758 \ucd1d\uc9c0\ud718\uad8c\uc790\uac00 \ub418\uc5c8\uc9c0\ub9cc, \uc0e4\ub97c \uacf5\uc791\uacfc \uac19\uc740 \uc778\uae30\ub97c \ub204\ub9ac\uc9c0\ub3c4 \ub450\ub824\uc6c0\uc758 \ub300\uc0c1\ub3c4 \ub418\uc9c0 \ubabb\ud588\ub2e4.", "paragraph_sentence": "5\uc6d4 6\uc77c \uc2e0\uc131 \ub85c\ub9c8 \uc81c\uad6d \uad70\ub300\ub294 \uc794\ub2c8\ucf5c\ub85c\uc640 \ubc14\ud2f0\uce78 \uc5b8\ub355 \ucabd\uc5d0 \uc788\ub294 \uc131\ubcbd\ub4e4\uc744 \uacf5\uaca9\ud558\uc600\ub2e4. \uc0e4\ub97c \uacf5\uc791\uc740 \ubca4\ubca0\ub204\ud1a0 \uccbc\ub9ac\ub2c8\uac00 \uc3dc \ucd1d\uc744 \ub9de\uace0 \uce58\uba85\uc0c1\uc744 \uc785\uc5c8\ub2e4. \uc0e4\ub97c \uacf5\uc791\uc740 \ubcd1\uc0ac\ub4e4\uc5d0\uac8c \uc790\uc2e0\uc758 \ubd80\uc0c1\uc744 \uc228\uae30\uae30 \uc704\ud558\uc5ec \ud558\uc580 \ub9dd\ud1a0\ub97c \uac78\ucce4\ub294\ub370, \ubcf8\ub798 \uc758\ub3c4\ud55c \ubc14\uc640\ub294 \ub2e4\ub974\uac8c \uc801\ub4e4\uc5d0\uac8c \uadf8\uc758 \uc2e0\ubd84\uc744 \ub178\ucd9c\uc2dc\ud0a4\ub294 \uaf34\uc774 \ub418\uace0 \ub9d0\uc558\ub2e4. \uc2e0\uc131 \ub85c\ub9c8 \uc81c\uad6d\uc758 \uc9c0\ud718\uad00 \uac00\uc6b4\ub370 \uac00\uc7a5 \uc874\uacbd\ubc1b\uc740 \uc9c0\ud718\uad00\uc774\uc5c8\ub358 \uc0e4\ub97c \uacf5\uc791\uc758 \uc8fd\uc74c\uc740 \uace7 \ubcd1\uc0ac\ub4e4\uc758 \uaddc\uc728\uc774 \ubb34\ub108\uc9c0\ub294 \uacb0\uacfc\ub97c \uc57c\uae30\ud588\uc73c\uba70, \uc774\ub4e4\uc740 \uac19\uc740 \ub0a0 \ub85c\ub9c8\uc758 \uc131\ubcbd\uc744 \ud568\ub77d\uc2dc\ud0a4\ub294\ub370 \uc131\uacf5\ud588\ub2e4. \uc804\uc0ac\ud55c \uc0e4\ub97c \uacf5\uc791\uc744 \ub300\uc2e0\ud574 \ud544\ub9ac\ubca0\ub974 \ub4dc \uc0ec\ub871 \uac00 \uad70\ub300\uc758 \ucd1d\uc9c0\ud718\uad8c\uc790\uac00 \ub418\uc5c8\uc9c0\ub9cc, \uc0e4\ub97c \uacf5\uc791\uacfc \uac19\uc740 \uc778\uae30\ub97c \ub204\ub9ac\uc9c0\ub3c4 \ub450\ub824\uc6c0\uc758 \ub300\uc0c1\ub3c4 \ub418\uc9c0 \ubabb\ud588\ub2e4. ", "paragraph_answer": "5\uc6d4 6\uc77c \uc2e0\uc131 \ub85c\ub9c8 \uc81c\uad6d \uad70\ub300\ub294 \uc794\ub2c8\ucf5c\ub85c\uc640 \ubc14\ud2f0\uce78 \uc5b8\ub355 \ucabd\uc5d0 \uc788\ub294 \uc131\ubcbd\ub4e4\uc744 \uacf5\uaca9\ud558\uc600\ub2e4. \uc0e4\ub97c \uacf5\uc791\uc740 \ubca4\ubca0\ub204\ud1a0 \uccbc\ub9ac\ub2c8\uac00 \uc3dc \ucd1d\uc744 \ub9de\uace0 \uce58\uba85\uc0c1\uc744 \uc785\uc5c8\ub2e4. \uc0e4\ub97c \uacf5\uc791\uc740 \ubcd1\uc0ac\ub4e4\uc5d0\uac8c \uc790\uc2e0\uc758 \ubd80\uc0c1\uc744 \uc228\uae30\uae30 \uc704\ud558\uc5ec \ud558\uc580 \ub9dd\ud1a0\ub97c \uac78\ucce4\ub294\ub370, \ubcf8\ub798 \uc758\ub3c4\ud55c \ubc14\uc640\ub294 \ub2e4\ub974\uac8c \uc801\ub4e4\uc5d0\uac8c \uadf8\uc758 \uc2e0\ubd84\uc744 \ub178\ucd9c\uc2dc\ud0a4\ub294 \uaf34\uc774 \ub418\uace0 \ub9d0\uc558\ub2e4. \uc2e0\uc131 \ub85c\ub9c8 \uc81c\uad6d\uc758 \uc9c0\ud718\uad00 \uac00\uc6b4\ub370 \uac00\uc7a5 \uc874\uacbd\ubc1b\uc740 \uc9c0\ud718\uad00\uc774\uc5c8\ub358 \uc0e4\ub97c \uacf5\uc791\uc758 \uc8fd\uc74c\uc740 \uace7 \ubcd1\uc0ac\ub4e4\uc758 \uaddc\uc728\uc774 \ubb34\ub108\uc9c0\ub294 \uacb0\uacfc\ub97c \uc57c\uae30\ud588\uc73c\uba70, \uc774\ub4e4\uc740 \uac19\uc740 \ub0a0 \ub85c\ub9c8\uc758 \uc131\ubcbd\uc744 \ud568\ub77d\uc2dc\ud0a4\ub294\ub370 \uc131\uacf5\ud588\ub2e4. \uc804\uc0ac\ud55c \uc0e4\ub97c \uacf5\uc791\uc744 \ub300\uc2e0\ud574 \ud544\ub9ac\ubca0\ub974 \ub4dc \uc0ec\ub871 \uac00 \uad70\ub300\uc758 \ucd1d\uc9c0\ud718\uad8c\uc790\uac00 \ub418\uc5c8\uc9c0\ub9cc, \uc0e4\ub97c \uacf5\uc791\uacfc \uac19\uc740 \uc778\uae30\ub97c \ub204\ub9ac\uc9c0\ub3c4 \ub450\ub824\uc6c0\uc758 \ub300\uc0c1\ub3c4 \ub418\uc9c0 \ubabb\ud588\ub2e4.", "sentence_answer": "\uc804\uc0ac\ud55c \uc0e4\ub97c \uacf5\uc791\uc744 \ub300\uc2e0\ud574 \ud544\ub9ac\ubca0\ub974 \ub4dc \uc0ec\ub871 \uac00 \uad70\ub300\uc758 \ucd1d\uc9c0\ud718\uad8c\uc790\uac00 \ub418\uc5c8\uc9c0\ub9cc, \uc0e4\ub97c \uacf5\uc791\uacfc \uac19\uc740 \uc778\uae30\ub97c \ub204\ub9ac\uc9c0\ub3c4 \ub450\ub824\uc6c0\uc758 \ub300\uc0c1\ub3c4 \ub418\uc9c0 \ubabb\ud588\ub2e4.", "paragraph_id": "6513719-1-0", "paragraph_question": "question: 1527\ub144 5\uc6d4 6\uc77c \ubc8c\uc5b4\uc9c4 \uc804\ud22c\uc5d0\uc11c \uc0ac\ub9dd\ud55c \uc0e4\ub97c \uacf5\uc791\uc744 \ub300\uc2e0\ud574 \uad70\ub300\uc758 \ucd1d\uc9c0\ud718\uad8c\uc744 \ub9e1\uc740 \uc0ac\ub78c\uc740?, context: 5\uc6d4 6\uc77c \uc2e0\uc131 \ub85c\ub9c8 \uc81c\uad6d \uad70\ub300\ub294 \uc794\ub2c8\ucf5c\ub85c\uc640 \ubc14\ud2f0\uce78 \uc5b8\ub355 \ucabd\uc5d0 \uc788\ub294 \uc131\ubcbd\ub4e4\uc744 \uacf5\uaca9\ud558\uc600\ub2e4. \uc0e4\ub97c \uacf5\uc791\uc740 \ubca4\ubca0\ub204\ud1a0 \uccbc\ub9ac\ub2c8\uac00 \uc3dc \ucd1d\uc744 \ub9de\uace0 \uce58\uba85\uc0c1\uc744 \uc785\uc5c8\ub2e4. \uc0e4\ub97c \uacf5\uc791\uc740 \ubcd1\uc0ac\ub4e4\uc5d0\uac8c \uc790\uc2e0\uc758 \ubd80\uc0c1\uc744 \uc228\uae30\uae30 \uc704\ud558\uc5ec \ud558\uc580 \ub9dd\ud1a0\ub97c \uac78\ucce4\ub294\ub370, \ubcf8\ub798 \uc758\ub3c4\ud55c \ubc14\uc640\ub294 \ub2e4\ub974\uac8c \uc801\ub4e4\uc5d0\uac8c \uadf8\uc758 \uc2e0\ubd84\uc744 \ub178\ucd9c\uc2dc\ud0a4\ub294 \uaf34\uc774 \ub418\uace0 \ub9d0\uc558\ub2e4. \uc2e0\uc131 \ub85c\ub9c8 \uc81c\uad6d\uc758 \uc9c0\ud718\uad00 \uac00\uc6b4\ub370 \uac00\uc7a5 \uc874\uacbd\ubc1b\uc740 \uc9c0\ud718\uad00\uc774\uc5c8\ub358 \uc0e4\ub97c \uacf5\uc791\uc758 \uc8fd\uc74c\uc740 \uace7 \ubcd1\uc0ac\ub4e4\uc758 \uaddc\uc728\uc774 \ubb34\ub108\uc9c0\ub294 \uacb0\uacfc\ub97c \uc57c\uae30\ud588\uc73c\uba70, \uc774\ub4e4\uc740 \uac19\uc740 \ub0a0 \ub85c\ub9c8\uc758 \uc131\ubcbd\uc744 \ud568\ub77d\uc2dc\ud0a4\ub294\ub370 \uc131\uacf5\ud588\ub2e4. \uc804\uc0ac\ud55c \uc0e4\ub97c \uacf5\uc791\uc744 \ub300\uc2e0\ud574 \ud544\ub9ac\ubca0\ub974 \ub4dc \uc0ec\ub871\uac00 \uad70\ub300\uc758 \ucd1d\uc9c0\ud718\uad8c\uc790\uac00 \ub418\uc5c8\uc9c0\ub9cc, \uc0e4\ub97c \uacf5\uc791\uacfc \uac19\uc740 \uc778\uae30\ub97c \ub204\ub9ac\uc9c0\ub3c4 \ub450\ub824\uc6c0\uc758 \ub300\uc0c1\ub3c4 \ub418\uc9c0 \ubabb\ud588\ub2e4."} {"question": "1901\ub144 8\uc6d4\uc5d0 \ubb34\uacfc\uc758 \uacfc\uac70\uc81c\ub97c \ud3d0\uc9c0\ud558\uace0 \uac01 \uc131\uc5d0 \uba85\ub839\ud558\uc5ec \ubd81\uc591\uacfc \uc591\uac15\uc744 \ubaa8\ubc29\ud574 \uc124\ub9bd\ud55c \uac83\uc740?", "paragraph": "\uccad\uc870\uc5d0\uc11c \uc8fc\ub3c4\ud55c \uad50\uc721\uac1c\ud601\uc740 \uc5ec\ub7ec \uac00\uc9c0 \ud55c\uacc4\uc810\uc774 \ubcf4\uc600\ub294\ub370 \uad70\uc0ac\uc81c\ub3c4 \uac1c\ud601\uc5d0\uc11c\ub294 \uc0c1\ub2f9\ud55c \ubcc0\ud654\uac00 \uc788\uc5c8\ub2e4. \uc81c1\ucc28\uc640 \uc81c2\ucc28 \uc544\ud3b8 \uc804\uc7c1\uc5d0\uc11c \uc2dc\uc791\ud574 \uccad\ubd88 \uc804\uc7c1, \uccad\uc77c \uc804\uc7c1, \uc758\ud654\ub2e8 \uc6b4\ub3d9 \ub4f1 \uc804\uc7c1\ub9c8\ub2e4 \ud328\ubc30\ub97c \uac70\ub4ed\ud574 \uc628 \uccad\uc740 \uc2e0\uc2dd \uad70\ub300\uc758 \ud544\uc694\uc131\uc744 \ud1b5\uac10\ud558\uc5ec \uad70\uc0ac \uac1c\ud601\uc5d0 \ucc29\uc218\ud588\ub2e4. \uc758\ud654\ub2e8\uc6b4\ub3d9 \uc774\ud6c4 \uc8fc\ub825 \uad70\ub300\ub294 \ubb34\uc704\uc6b0\uad70\ub9cc \ub0a8\uace0 \ub098\uba38\uc9c0\ub294 \ud574\uc0b0\ub418\uc5c8\ub2e4. \uc758\ud654\ub2e8\uc6b4\ub3d9\uc744 \uacc4\uae30\ub85c \uae09\ubd80\uc0c1\ud558\uace0 \ucd9c\uc138 \uc9c0\ud5a5 \uc77c\ub85c\ub97c \ub2ec\ub9ac\ub358 \uc704\uc548\uc2a4\uce74\uc774\uac00 \uc9c1\ub840\ucd1d\ub3c5\uc774 \ub418\uc5b4 \ubb34\uc704\uc6b0\uad70\uc744 \uc774\ub04c\uace0 \ud188\uc9c4\uc5d0 \uc8fc\ub454\ud558\uc600\ub2e4. \uccad\uc870\ub3c4 \uae30\uc874 \uad70\ub300\uc758 \ubd80\ud328\uc640 \ubb34\ub2a5\uc744 \uc54c\uace0 1901\ub144 8\uc6d4\uc5d0 \ubb34\uacfc\uc758 \uacfc\uac70\uc81c\ub97c \ud3d0\uc9c0\ud558\uace0 \uac01 \uc131\uc5d0 \uba85\ub839\ud558\uc5ec \ubd81\uc591(\uc704\uc548\uc2a4\uce74\uc774\uc758 \uc2e0\uac74\uc721\uad70, \u65b0\u5efa\u9678\u8ecd)\uacfc \uc591\uac15(\uc7a5\uc9c0\ub3d9\uc758 \uc790\uac15\uad70)\uc744 \ubaa8\ubc29\ud574 \ubb34\ube44\ud559\ub2f9(\u6b66\u5099\u5b78\u5802)\uc744 \uc124\ub9bd\ud558\ub3c4\ub85d \ud558\uc600\uc73c\uba70 \uae30\uc874 \uad70\ub300\ub97c \ud574\uc0b0\uc2dc\ud0a4\uace0 \uc0c1\ube44\uad70\uc744 \uc0c8\ub85c \ud3b8\uc131 \ud6c8\ub828\ud558\ub3c4\ub85d \uc9c0\uc2dc\ud558\uc600\ub2e4.", "answer": "\ubb34\ube44\ud559\ub2f9", "sentence": "\uccad\uc870\ub3c4 \uae30\uc874 \uad70\ub300\uc758 \ubd80\ud328\uc640 \ubb34\ub2a5\uc744 \uc54c\uace0 1901\ub144 8\uc6d4\uc5d0 \ubb34\uacfc\uc758 \uacfc\uac70\uc81c\ub97c \ud3d0\uc9c0\ud558\uace0 \uac01 \uc131\uc5d0 \uba85\ub839\ud558\uc5ec \ubd81\uc591(\uc704\uc548\uc2a4\uce74\uc774\uc758 \uc2e0\uac74\uc721\uad70, \u65b0\u5efa\u9678\u8ecd)\uacfc \uc591\uac15(\uc7a5\uc9c0\ub3d9\uc758 \uc790\uac15\uad70)\uc744 \ubaa8\ubc29\ud574 \ubb34\ube44\ud559\ub2f9 (\u6b66\u5099\u5b78\u5802)\uc744 \uc124\ub9bd\ud558\ub3c4\ub85d \ud558\uc600\uc73c\uba70 \uae30\uc874 \uad70\ub300\ub97c \ud574\uc0b0\uc2dc\ud0a4\uace0 \uc0c1\ube44\uad70\uc744 \uc0c8\ub85c \ud3b8\uc131 \ud6c8\ub828\ud558\ub3c4\ub85d \uc9c0\uc2dc\ud558\uc600\ub2e4.", "paragraph_sentence": "\uccad\uc870\uc5d0\uc11c \uc8fc\ub3c4\ud55c \uad50\uc721\uac1c\ud601\uc740 \uc5ec\ub7ec \uac00\uc9c0 \ud55c\uacc4\uc810\uc774 \ubcf4\uc600\ub294\ub370 \uad70\uc0ac\uc81c\ub3c4 \uac1c\ud601\uc5d0\uc11c\ub294 \uc0c1\ub2f9\ud55c \ubcc0\ud654\uac00 \uc788\uc5c8\ub2e4. \uc81c1\ucc28\uc640 \uc81c2\ucc28 \uc544\ud3b8 \uc804\uc7c1\uc5d0\uc11c \uc2dc\uc791\ud574 \uccad\ubd88 \uc804\uc7c1, \uccad\uc77c \uc804\uc7c1, \uc758\ud654\ub2e8 \uc6b4\ub3d9 \ub4f1 \uc804\uc7c1\ub9c8\ub2e4 \ud328\ubc30\ub97c \uac70\ub4ed\ud574 \uc628 \uccad\uc740 \uc2e0\uc2dd \uad70\ub300\uc758 \ud544\uc694\uc131\uc744 \ud1b5\uac10\ud558\uc5ec \uad70\uc0ac \uac1c\ud601\uc5d0 \ucc29\uc218\ud588\ub2e4. \uc758\ud654\ub2e8\uc6b4\ub3d9 \uc774\ud6c4 \uc8fc\ub825 \uad70\ub300\ub294 \ubb34\uc704\uc6b0\uad70\ub9cc \ub0a8\uace0 \ub098\uba38\uc9c0\ub294 \ud574\uc0b0\ub418\uc5c8\ub2e4. \uc758\ud654\ub2e8\uc6b4\ub3d9\uc744 \uacc4\uae30\ub85c \uae09\ubd80\uc0c1\ud558\uace0 \ucd9c\uc138 \uc9c0\ud5a5 \uc77c\ub85c\ub97c \ub2ec\ub9ac\ub358 \uc704\uc548\uc2a4\uce74\uc774\uac00 \uc9c1\ub840\ucd1d\ub3c5\uc774 \ub418\uc5b4 \ubb34\uc704\uc6b0\uad70\uc744 \uc774\ub04c\uace0 \ud188\uc9c4\uc5d0 \uc8fc\ub454\ud558\uc600\ub2e4. \uccad\uc870\ub3c4 \uae30\uc874 \uad70\ub300\uc758 \ubd80\ud328\uc640 \ubb34\ub2a5\uc744 \uc54c\uace0 1901\ub144 8\uc6d4\uc5d0 \ubb34\uacfc\uc758 \uacfc\uac70\uc81c\ub97c \ud3d0\uc9c0\ud558\uace0 \uac01 \uc131\uc5d0 \uba85\ub839\ud558\uc5ec \ubd81\uc591(\uc704\uc548\uc2a4\uce74\uc774\uc758 \uc2e0\uac74\uc721\uad70, \u65b0\u5efa\u9678\u8ecd)\uacfc \uc591\uac15(\uc7a5\uc9c0\ub3d9\uc758 \uc790\uac15\uad70)\uc744 \ubaa8\ubc29\ud574 \ubb34\ube44\ud559\ub2f9 (\u6b66\u5099\u5b78\u5802)\uc744 \uc124\ub9bd\ud558\ub3c4\ub85d \ud558\uc600\uc73c\uba70 \uae30\uc874 \uad70\ub300\ub97c \ud574\uc0b0\uc2dc\ud0a4\uace0 \uc0c1\ube44\uad70\uc744 \uc0c8\ub85c \ud3b8\uc131 \ud6c8\ub828\ud558\ub3c4\ub85d \uc9c0\uc2dc\ud558\uc600\ub2e4. ", "paragraph_answer": "\uccad\uc870\uc5d0\uc11c \uc8fc\ub3c4\ud55c \uad50\uc721\uac1c\ud601\uc740 \uc5ec\ub7ec \uac00\uc9c0 \ud55c\uacc4\uc810\uc774 \ubcf4\uc600\ub294\ub370 \uad70\uc0ac\uc81c\ub3c4 \uac1c\ud601\uc5d0\uc11c\ub294 \uc0c1\ub2f9\ud55c \ubcc0\ud654\uac00 \uc788\uc5c8\ub2e4. \uc81c1\ucc28\uc640 \uc81c2\ucc28 \uc544\ud3b8 \uc804\uc7c1\uc5d0\uc11c \uc2dc\uc791\ud574 \uccad\ubd88 \uc804\uc7c1, \uccad\uc77c \uc804\uc7c1, \uc758\ud654\ub2e8 \uc6b4\ub3d9 \ub4f1 \uc804\uc7c1\ub9c8\ub2e4 \ud328\ubc30\ub97c \uac70\ub4ed\ud574 \uc628 \uccad\uc740 \uc2e0\uc2dd \uad70\ub300\uc758 \ud544\uc694\uc131\uc744 \ud1b5\uac10\ud558\uc5ec \uad70\uc0ac \uac1c\ud601\uc5d0 \ucc29\uc218\ud588\ub2e4. \uc758\ud654\ub2e8\uc6b4\ub3d9 \uc774\ud6c4 \uc8fc\ub825 \uad70\ub300\ub294 \ubb34\uc704\uc6b0\uad70\ub9cc \ub0a8\uace0 \ub098\uba38\uc9c0\ub294 \ud574\uc0b0\ub418\uc5c8\ub2e4. \uc758\ud654\ub2e8\uc6b4\ub3d9\uc744 \uacc4\uae30\ub85c \uae09\ubd80\uc0c1\ud558\uace0 \ucd9c\uc138 \uc9c0\ud5a5 \uc77c\ub85c\ub97c \ub2ec\ub9ac\ub358 \uc704\uc548\uc2a4\uce74\uc774\uac00 \uc9c1\ub840\ucd1d\ub3c5\uc774 \ub418\uc5b4 \ubb34\uc704\uc6b0\uad70\uc744 \uc774\ub04c\uace0 \ud188\uc9c4\uc5d0 \uc8fc\ub454\ud558\uc600\ub2e4. \uccad\uc870\ub3c4 \uae30\uc874 \uad70\ub300\uc758 \ubd80\ud328\uc640 \ubb34\ub2a5\uc744 \uc54c\uace0 1901\ub144 8\uc6d4\uc5d0 \ubb34\uacfc\uc758 \uacfc\uac70\uc81c\ub97c \ud3d0\uc9c0\ud558\uace0 \uac01 \uc131\uc5d0 \uba85\ub839\ud558\uc5ec \ubd81\uc591(\uc704\uc548\uc2a4\uce74\uc774\uc758 \uc2e0\uac74\uc721\uad70, \u65b0\u5efa\u9678\u8ecd)\uacfc \uc591\uac15(\uc7a5\uc9c0\ub3d9\uc758 \uc790\uac15\uad70)\uc744 \ubaa8\ubc29\ud574 \ubb34\ube44\ud559\ub2f9 (\u6b66\u5099\u5b78\u5802)\uc744 \uc124\ub9bd\ud558\ub3c4\ub85d \ud558\uc600\uc73c\uba70 \uae30\uc874 \uad70\ub300\ub97c \ud574\uc0b0\uc2dc\ud0a4\uace0 \uc0c1\ube44\uad70\uc744 \uc0c8\ub85c \ud3b8\uc131 \ud6c8\ub828\ud558\ub3c4\ub85d \uc9c0\uc2dc\ud558\uc600\ub2e4.", "sentence_answer": "\uccad\uc870\ub3c4 \uae30\uc874 \uad70\ub300\uc758 \ubd80\ud328\uc640 \ubb34\ub2a5\uc744 \uc54c\uace0 1901\ub144 8\uc6d4\uc5d0 \ubb34\uacfc\uc758 \uacfc\uac70\uc81c\ub97c \ud3d0\uc9c0\ud558\uace0 \uac01 \uc131\uc5d0 \uba85\ub839\ud558\uc5ec \ubd81\uc591(\uc704\uc548\uc2a4\uce74\uc774\uc758 \uc2e0\uac74\uc721\uad70, \u65b0\u5efa\u9678\u8ecd)\uacfc \uc591\uac15(\uc7a5\uc9c0\ub3d9\uc758 \uc790\uac15\uad70)\uc744 \ubaa8\ubc29\ud574 \ubb34\ube44\ud559\ub2f9 (\u6b66\u5099\u5b78\u5802)\uc744 \uc124\ub9bd\ud558\ub3c4\ub85d \ud558\uc600\uc73c\uba70 \uae30\uc874 \uad70\ub300\ub97c \ud574\uc0b0\uc2dc\ud0a4\uace0 \uc0c1\ube44\uad70\uc744 \uc0c8\ub85c \ud3b8\uc131 \ud6c8\ub828\ud558\ub3c4\ub85d \uc9c0\uc2dc\ud558\uc600\ub2e4.", "paragraph_id": "6457494-7-1", "paragraph_question": "question: 1901\ub144 8\uc6d4\uc5d0 \ubb34\uacfc\uc758 \uacfc\uac70\uc81c\ub97c \ud3d0\uc9c0\ud558\uace0 \uac01 \uc131\uc5d0 \uba85\ub839\ud558\uc5ec \ubd81\uc591\uacfc \uc591\uac15\uc744 \ubaa8\ubc29\ud574 \uc124\ub9bd\ud55c \uac83\uc740?, context: \uccad\uc870\uc5d0\uc11c \uc8fc\ub3c4\ud55c \uad50\uc721\uac1c\ud601\uc740 \uc5ec\ub7ec \uac00\uc9c0 \ud55c\uacc4\uc810\uc774 \ubcf4\uc600\ub294\ub370 \uad70\uc0ac\uc81c\ub3c4 \uac1c\ud601\uc5d0\uc11c\ub294 \uc0c1\ub2f9\ud55c \ubcc0\ud654\uac00 \uc788\uc5c8\ub2e4. \uc81c1\ucc28\uc640 \uc81c2\ucc28 \uc544\ud3b8 \uc804\uc7c1\uc5d0\uc11c \uc2dc\uc791\ud574 \uccad\ubd88 \uc804\uc7c1, \uccad\uc77c \uc804\uc7c1, \uc758\ud654\ub2e8 \uc6b4\ub3d9 \ub4f1 \uc804\uc7c1\ub9c8\ub2e4 \ud328\ubc30\ub97c \uac70\ub4ed\ud574 \uc628 \uccad\uc740 \uc2e0\uc2dd \uad70\ub300\uc758 \ud544\uc694\uc131\uc744 \ud1b5\uac10\ud558\uc5ec \uad70\uc0ac \uac1c\ud601\uc5d0 \ucc29\uc218\ud588\ub2e4. \uc758\ud654\ub2e8\uc6b4\ub3d9 \uc774\ud6c4 \uc8fc\ub825 \uad70\ub300\ub294 \ubb34\uc704\uc6b0\uad70\ub9cc \ub0a8\uace0 \ub098\uba38\uc9c0\ub294 \ud574\uc0b0\ub418\uc5c8\ub2e4. \uc758\ud654\ub2e8\uc6b4\ub3d9\uc744 \uacc4\uae30\ub85c \uae09\ubd80\uc0c1\ud558\uace0 \ucd9c\uc138 \uc9c0\ud5a5 \uc77c\ub85c\ub97c \ub2ec\ub9ac\ub358 \uc704\uc548\uc2a4\uce74\uc774\uac00 \uc9c1\ub840\ucd1d\ub3c5\uc774 \ub418\uc5b4 \ubb34\uc704\uc6b0\uad70\uc744 \uc774\ub04c\uace0 \ud188\uc9c4\uc5d0 \uc8fc\ub454\ud558\uc600\ub2e4. \uccad\uc870\ub3c4 \uae30\uc874 \uad70\ub300\uc758 \ubd80\ud328\uc640 \ubb34\ub2a5\uc744 \uc54c\uace0 1901\ub144 8\uc6d4\uc5d0 \ubb34\uacfc\uc758 \uacfc\uac70\uc81c\ub97c \ud3d0\uc9c0\ud558\uace0 \uac01 \uc131\uc5d0 \uba85\ub839\ud558\uc5ec \ubd81\uc591(\uc704\uc548\uc2a4\uce74\uc774\uc758 \uc2e0\uac74\uc721\uad70, \u65b0\u5efa\u9678\u8ecd)\uacfc \uc591\uac15(\uc7a5\uc9c0\ub3d9\uc758 \uc790\uac15\uad70)\uc744 \ubaa8\ubc29\ud574 \ubb34\ube44\ud559\ub2f9(\u6b66\u5099\u5b78\u5802)\uc744 \uc124\ub9bd\ud558\ub3c4\ub85d \ud558\uc600\uc73c\uba70 \uae30\uc874 \uad70\ub300\ub97c \ud574\uc0b0\uc2dc\ud0a4\uace0 \uc0c1\ube44\uad70\uc744 \uc0c8\ub85c \ud3b8\uc131 \ud6c8\ub828\ud558\ub3c4\ub85d \uc9c0\uc2dc\ud558\uc600\ub2e4."} {"question": "\uad11\ud574\uad70\uc740 \uc989\uc704 \ud6c4 \uc655\uad8c\uac15\ud654\ub97c \uc704\ud574 \ubb34\uc5c7\uc744 \uc9c0\uc5c8\ub294\uac00?", "paragraph": "\uc784\uae08\uc774 \ub41c \uad11\ud574\uad70\uc740 \uc989\uc704 \ucd08\ubd80\ud130 \uc548\uc73c\ub85c\ub294 \uc655\uad8c\uc744 \uac15\ud654\ud558\uae30 \uc704\ud574 \uad81\uad90\uc744 \uc9c0\uc5b4 \uacbd\uc81c\uac00 \ud30c\ud0c4 \ub0ac\uace0 \uc774\uc810\uc740 \uad11\ud574\uad70\uc744 \uae0d\uc815\uc801\uc73c\ub85c \ud3c9\uac00\ud558\ub294 \ud55c\uba85\uae30 \uad50\uc218 \uc870\ucc28\ub3c4 \"\uad11\ud574\uad70\uc740 \ud0c1\uc6d4\ud55c \uc678\uad50 \uc815\ucc45\uc744 \uad81\uad90 \uacf5\uc0ac\ub85c \ubaa8\ub450 \ub9d0\uc544 \uba39\uc5c8\ub2e4\" \ub77c\uba70 \ube44\ud310\ud55c\ub2e4. 1608\ub144 \uc120\ud61c\uccad\uc744 \ub450\uc5b4 \uacbd\uae30\ub3c4\uc5d0\uc11c \uc300\ub85c \uc870\uc138\ub97c \ub0b4\ub3c4\ub85d \ud568\uc73c\ub85c\uc368 \uc18c\ub4dd\uc5d0 \ub530\ub77c \uc138\uae08\uc744 \ub0b4\ub294 \uc870\uc138\uac1c\ud601\uc778 \ub300\ub3d9\ubc95\uc744 \uc5b5\uc9c0\ub85c \uc2dc\ud589\ud588\uc73c\ub098,\ub450 \ubc88\uc774\ub098 \ud3d0\uc9c0\ud558\ub824 \ud588\ub2e4. 1611\ub144 \uc591\uc804 \uc0ac\uc5c5\uc744 \ubc8c\uc600\ub2e4\uace0 \ud558\ub098 \uc120\uc870 \ub300\uc758 \uacc4\ubb18\uc591\uc804, \uc778\uc870 \ub300\uc758 \uac11\uc220\uc591\uc804 \uc774\uc678\uc5d0\ub294 \uc591\uc804\uc744 \uc2dc\ud589\ud55c \uae30\ub85d\uc774 \uc5c6\ub2e4. \uc774\uc5b4 \uc784\uc9c4\uc65c\ub780 \ub54c \ud654\uc7ac\ub85c \uc18c\uc2e4\ub41c \ucc3d\ub355\uad81, \uacbd\ud76c\uad81, \ucc3d\uacbd\uad81\uc744 \uc7ac\uac74\ud558\uace0 \uc778\uacbd\uad81\uc744 \uac74\uc124\ud588\uc73c\uba70, \uc784\uc9c4\uc65c\ub780 \ub54c \uc18c\uc2e4\ub41c \uc11c\uc801 \uac04\ud589\uc5d0\ub3c4 \ud798\uc368 \u300a\uc2e0\uc99d\ub3d9\uad6d\uc5ec\uc9c0\uc2b9\ub78c\u300b, \u300a\uc6a9\ube44\uc5b4\ucc9c\uac00\u300b ,\u300a\ub3d9\uad6d\uc2e0\uc18d\uc0bc\uac15\ud589\uc2e4\u300b \ub4f1\uc744 \ub2e4\uc2dc \uac04\ud589\ud588\ub2e4. \ud5c8\uade0\uc758 \u300a\ud64d\uae38\ub3d9\uc804\u300b, \ud5c8\uc900\uc758 \ud55c\uc758\ud559\ucc45\uc778\u300a\ub3d9\uc758\ubcf4\uac10\u300b \ub4f1\ub3c4 \uc774 \uc2dc\uae30\uc5d0 \uc644\uc131\ub418\uc5c8\ub2e4.", "answer": "\uad81\uad90", "sentence": "\uc784\uae08\uc774 \ub41c \uad11\ud574\uad70\uc740 \uc989\uc704 \ucd08\ubd80\ud130 \uc548\uc73c\ub85c\ub294 \uc655\uad8c\uc744 \uac15\ud654\ud558\uae30 \uc704\ud574 \uad81\uad90 \uc744 \uc9c0\uc5b4 \uacbd\uc81c\uac00 \ud30c\ud0c4 \ub0ac\uace0 \uc774\uc810\uc740 \uad11\ud574\uad70\uc744 \uae0d\uc815\uc801\uc73c\ub85c \ud3c9\uac00\ud558\ub294 \ud55c\uba85\uae30 \uad50\uc218 \uc870\ucc28\ub3c4 \"\uad11\ud574\uad70\uc740 \ud0c1\uc6d4\ud55c \uc678\uad50 \uc815\ucc45\uc744 \uad81\uad90 \uacf5\uc0ac\ub85c \ubaa8\ub450 \ub9d0\uc544 \uba39\uc5c8\ub2e4\" \ub77c\uba70 \ube44\ud310\ud55c\ub2e4.", "paragraph_sentence": " \uc784\uae08\uc774 \ub41c \uad11\ud574\uad70\uc740 \uc989\uc704 \ucd08\ubd80\ud130 \uc548\uc73c\ub85c\ub294 \uc655\uad8c\uc744 \uac15\ud654\ud558\uae30 \uc704\ud574 \uad81\uad90 \uc744 \uc9c0\uc5b4 \uacbd\uc81c\uac00 \ud30c\ud0c4 \ub0ac\uace0 \uc774\uc810\uc740 \uad11\ud574\uad70\uc744 \uae0d\uc815\uc801\uc73c\ub85c \ud3c9\uac00\ud558\ub294 \ud55c\uba85\uae30 \uad50\uc218 \uc870\ucc28\ub3c4 \"\uad11\ud574\uad70\uc740 \ud0c1\uc6d4\ud55c \uc678\uad50 \uc815\ucc45\uc744 \uad81\uad90 \uacf5\uc0ac\ub85c \ubaa8\ub450 \ub9d0\uc544 \uba39\uc5c8\ub2e4\" \ub77c\uba70 \ube44\ud310\ud55c\ub2e4. 1608\ub144 \uc120\ud61c\uccad\uc744 \ub450\uc5b4 \uacbd\uae30\ub3c4\uc5d0\uc11c \uc300\ub85c \uc870\uc138\ub97c \ub0b4\ub3c4\ub85d \ud568\uc73c\ub85c\uc368 \uc18c\ub4dd\uc5d0 \ub530\ub77c \uc138\uae08\uc744 \ub0b4\ub294 \uc870\uc138\uac1c\ud601\uc778 \ub300\ub3d9\ubc95\uc744 \uc5b5\uc9c0\ub85c \uc2dc\ud589\ud588\uc73c\ub098,\ub450 \ubc88\uc774\ub098 \ud3d0\uc9c0\ud558\ub824 \ud588\ub2e4. 1611\ub144 \uc591\uc804 \uc0ac\uc5c5\uc744 \ubc8c\uc600\ub2e4\uace0 \ud558\ub098 \uc120\uc870 \ub300\uc758 \uacc4\ubb18\uc591\uc804, \uc778\uc870 \ub300\uc758 \uac11\uc220\uc591\uc804 \uc774\uc678\uc5d0\ub294 \uc591\uc804\uc744 \uc2dc\ud589\ud55c \uae30\ub85d\uc774 \uc5c6\ub2e4. \uc774\uc5b4 \uc784\uc9c4\uc65c\ub780 \ub54c \ud654\uc7ac\ub85c \uc18c\uc2e4\ub41c \ucc3d\ub355\uad81, \uacbd\ud76c\uad81, \ucc3d\uacbd\uad81\uc744 \uc7ac\uac74\ud558\uace0 \uc778\uacbd\uad81\uc744 \uac74\uc124\ud588\uc73c\uba70, \uc784\uc9c4\uc65c\ub780 \ub54c \uc18c\uc2e4\ub41c \uc11c\uc801 \uac04\ud589\uc5d0\ub3c4 \ud798\uc368 \u300a\uc2e0\uc99d\ub3d9\uad6d\uc5ec\uc9c0\uc2b9\ub78c\u300b, \u300a\uc6a9\ube44\uc5b4\ucc9c\uac00\u300b ,\u300a\ub3d9\uad6d\uc2e0\uc18d\uc0bc\uac15\ud589\uc2e4\u300b \ub4f1\uc744 \ub2e4\uc2dc \uac04\ud589\ud588\ub2e4. \ud5c8\uade0\uc758 \u300a\ud64d\uae38\ub3d9\uc804\u300b, \ud5c8\uc900\uc758 \ud55c\uc758\ud559\ucc45\uc778\u300a\ub3d9\uc758\ubcf4\uac10\u300b \ub4f1\ub3c4 \uc774 \uc2dc\uae30\uc5d0 \uc644\uc131\ub418\uc5c8\ub2e4.", "paragraph_answer": "\uc784\uae08\uc774 \ub41c \uad11\ud574\uad70\uc740 \uc989\uc704 \ucd08\ubd80\ud130 \uc548\uc73c\ub85c\ub294 \uc655\uad8c\uc744 \uac15\ud654\ud558\uae30 \uc704\ud574 \uad81\uad90 \uc744 \uc9c0\uc5b4 \uacbd\uc81c\uac00 \ud30c\ud0c4 \ub0ac\uace0 \uc774\uc810\uc740 \uad11\ud574\uad70\uc744 \uae0d\uc815\uc801\uc73c\ub85c \ud3c9\uac00\ud558\ub294 \ud55c\uba85\uae30 \uad50\uc218 \uc870\ucc28\ub3c4 \"\uad11\ud574\uad70\uc740 \ud0c1\uc6d4\ud55c \uc678\uad50 \uc815\ucc45\uc744 \uad81\uad90 \uacf5\uc0ac\ub85c \ubaa8\ub450 \ub9d0\uc544 \uba39\uc5c8\ub2e4\" \ub77c\uba70 \ube44\ud310\ud55c\ub2e4. 1608\ub144 \uc120\ud61c\uccad\uc744 \ub450\uc5b4 \uacbd\uae30\ub3c4\uc5d0\uc11c \uc300\ub85c \uc870\uc138\ub97c \ub0b4\ub3c4\ub85d \ud568\uc73c\ub85c\uc368 \uc18c\ub4dd\uc5d0 \ub530\ub77c \uc138\uae08\uc744 \ub0b4\ub294 \uc870\uc138\uac1c\ud601\uc778 \ub300\ub3d9\ubc95\uc744 \uc5b5\uc9c0\ub85c \uc2dc\ud589\ud588\uc73c\ub098,\ub450 \ubc88\uc774\ub098 \ud3d0\uc9c0\ud558\ub824 \ud588\ub2e4. 1611\ub144 \uc591\uc804 \uc0ac\uc5c5\uc744 \ubc8c\uc600\ub2e4\uace0 \ud558\ub098 \uc120\uc870 \ub300\uc758 \uacc4\ubb18\uc591\uc804, \uc778\uc870 \ub300\uc758 \uac11\uc220\uc591\uc804 \uc774\uc678\uc5d0\ub294 \uc591\uc804\uc744 \uc2dc\ud589\ud55c \uae30\ub85d\uc774 \uc5c6\ub2e4. \uc774\uc5b4 \uc784\uc9c4\uc65c\ub780 \ub54c \ud654\uc7ac\ub85c \uc18c\uc2e4\ub41c \ucc3d\ub355\uad81, \uacbd\ud76c\uad81, \ucc3d\uacbd\uad81\uc744 \uc7ac\uac74\ud558\uace0 \uc778\uacbd\uad81\uc744 \uac74\uc124\ud588\uc73c\uba70, \uc784\uc9c4\uc65c\ub780 \ub54c \uc18c\uc2e4\ub41c \uc11c\uc801 \uac04\ud589\uc5d0\ub3c4 \ud798\uc368 \u300a\uc2e0\uc99d\ub3d9\uad6d\uc5ec\uc9c0\uc2b9\ub78c\u300b, \u300a\uc6a9\ube44\uc5b4\ucc9c\uac00\u300b ,\u300a\ub3d9\uad6d\uc2e0\uc18d\uc0bc\uac15\ud589\uc2e4\u300b \ub4f1\uc744 \ub2e4\uc2dc \uac04\ud589\ud588\ub2e4. \ud5c8\uade0\uc758 \u300a\ud64d\uae38\ub3d9\uc804\u300b, \ud5c8\uc900\uc758 \ud55c\uc758\ud559\ucc45\uc778\u300a\ub3d9\uc758\ubcf4\uac10\u300b \ub4f1\ub3c4 \uc774 \uc2dc\uae30\uc5d0 \uc644\uc131\ub418\uc5c8\ub2e4.", "sentence_answer": "\uc784\uae08\uc774 \ub41c \uad11\ud574\uad70\uc740 \uc989\uc704 \ucd08\ubd80\ud130 \uc548\uc73c\ub85c\ub294 \uc655\uad8c\uc744 \uac15\ud654\ud558\uae30 \uc704\ud574 \uad81\uad90 \uc744 \uc9c0\uc5b4 \uacbd\uc81c\uac00 \ud30c\ud0c4 \ub0ac\uace0 \uc774\uc810\uc740 \uad11\ud574\uad70\uc744 \uae0d\uc815\uc801\uc73c\ub85c \ud3c9\uac00\ud558\ub294 \ud55c\uba85\uae30 \uad50\uc218 \uc870\ucc28\ub3c4 \"\uad11\ud574\uad70\uc740 \ud0c1\uc6d4\ud55c \uc678\uad50 \uc815\ucc45\uc744 \uad81\uad90 \uacf5\uc0ac\ub85c \ubaa8\ub450 \ub9d0\uc544 \uba39\uc5c8\ub2e4\" \ub77c\uba70 \ube44\ud310\ud55c\ub2e4.", "paragraph_id": "6551280-2-0", "paragraph_question": "question: \uad11\ud574\uad70\uc740 \uc989\uc704 \ud6c4 \uc655\uad8c\uac15\ud654\ub97c \uc704\ud574 \ubb34\uc5c7\uc744 \uc9c0\uc5c8\ub294\uac00?, context: \uc784\uae08\uc774 \ub41c \uad11\ud574\uad70\uc740 \uc989\uc704 \ucd08\ubd80\ud130 \uc548\uc73c\ub85c\ub294 \uc655\uad8c\uc744 \uac15\ud654\ud558\uae30 \uc704\ud574 \uad81\uad90\uc744 \uc9c0\uc5b4 \uacbd\uc81c\uac00 \ud30c\ud0c4 \ub0ac\uace0 \uc774\uc810\uc740 \uad11\ud574\uad70\uc744 \uae0d\uc815\uc801\uc73c\ub85c \ud3c9\uac00\ud558\ub294 \ud55c\uba85\uae30 \uad50\uc218 \uc870\ucc28\ub3c4 \"\uad11\ud574\uad70\uc740 \ud0c1\uc6d4\ud55c \uc678\uad50 \uc815\ucc45\uc744 \uad81\uad90 \uacf5\uc0ac\ub85c \ubaa8\ub450 \ub9d0\uc544 \uba39\uc5c8\ub2e4\" \ub77c\uba70 \ube44\ud310\ud55c\ub2e4. 1608\ub144 \uc120\ud61c\uccad\uc744 \ub450\uc5b4 \uacbd\uae30\ub3c4\uc5d0\uc11c \uc300\ub85c \uc870\uc138\ub97c \ub0b4\ub3c4\ub85d \ud568\uc73c\ub85c\uc368 \uc18c\ub4dd\uc5d0 \ub530\ub77c \uc138\uae08\uc744 \ub0b4\ub294 \uc870\uc138\uac1c\ud601\uc778 \ub300\ub3d9\ubc95\uc744 \uc5b5\uc9c0\ub85c \uc2dc\ud589\ud588\uc73c\ub098,\ub450 \ubc88\uc774\ub098 \ud3d0\uc9c0\ud558\ub824 \ud588\ub2e4. 1611\ub144 \uc591\uc804 \uc0ac\uc5c5\uc744 \ubc8c\uc600\ub2e4\uace0 \ud558\ub098 \uc120\uc870 \ub300\uc758 \uacc4\ubb18\uc591\uc804, \uc778\uc870 \ub300\uc758 \uac11\uc220\uc591\uc804 \uc774\uc678\uc5d0\ub294 \uc591\uc804\uc744 \uc2dc\ud589\ud55c \uae30\ub85d\uc774 \uc5c6\ub2e4. \uc774\uc5b4 \uc784\uc9c4\uc65c\ub780 \ub54c \ud654\uc7ac\ub85c \uc18c\uc2e4\ub41c \ucc3d\ub355\uad81, \uacbd\ud76c\uad81, \ucc3d\uacbd\uad81\uc744 \uc7ac\uac74\ud558\uace0 \uc778\uacbd\uad81\uc744 \uac74\uc124\ud588\uc73c\uba70, \uc784\uc9c4\uc65c\ub780 \ub54c \uc18c\uc2e4\ub41c \uc11c\uc801 \uac04\ud589\uc5d0\ub3c4 \ud798\uc368 \u300a\uc2e0\uc99d\ub3d9\uad6d\uc5ec\uc9c0\uc2b9\ub78c\u300b, \u300a\uc6a9\ube44\uc5b4\ucc9c\uac00\u300b ,\u300a\ub3d9\uad6d\uc2e0\uc18d\uc0bc\uac15\ud589\uc2e4\u300b \ub4f1\uc744 \ub2e4\uc2dc \uac04\ud589\ud588\ub2e4. \ud5c8\uade0\uc758 \u300a\ud64d\uae38\ub3d9\uc804\u300b, \ud5c8\uc900\uc758 \ud55c\uc758\ud559\ucc45\uc778\u300a\ub3d9\uc758\ubcf4\uac10\u300b \ub4f1\ub3c4 \uc774 \uc2dc\uae30\uc5d0 \uc644\uc131\ub418\uc5c8\ub2e4."} {"question": "\uc545\ud2f0\ub284\uc774 \uc0b0\uc18c \ubc0f \uc218\ubd84\uacfc \uae09\uc18d\ud788 \uacb0\ud569\ud558\uba74\uc11c \ud615\uc131\ub418\ub294 \ubb3c\uc9c8\uc740?", "paragraph": "\ubd80\ub4dc\ub7fd\uace0, \uc740\ubc31\uc0c9\uc778 \uae08\uc18d \uc545\ud2f0\ub284\uc740 \uacf5\uae30 \uc911\uc5d0\uc11c \uc0b0\uc18c \ubc0f \uc218\ubd84\uacfc \uae09\uc18d\ud788 \uacb0\ud569\ud558\uc5ec \ud770\uc0c9 \ub9c9\uc758 \uc0b0\ud654\uc545\ud2f0\ub284\uc744 \ud615\uc131\ud558\uba70, \uc774\ub294 \uc545\ud2f0\ub284\uc774 \ub354 \uc0b0\ud654\ub418\ub294 \uac83\uc744 \ubc29\uc9c0\ud55c\ub2e4. \ub300\ubd80\ubd84\uc758 \ub780\ud0c0\ub118\uc871\uacfc \uc545\ud2f0\ub284\uc871\uacfc \uac19\uc774, \uc545\ud2f0\ub284\uc740 \uac70\uc758 \ub300\ubd80\ubd84\uc758 \ud654\ud569\ubb3c\uc5d0\uc11c \ub300\ubd80\ubd84 +3\uc758 \uc0b0\ud654\uc218\ub97c \uac00\uc9c4\ub2e4. \uc545\ud2f0\ub284\uc740 \uc624\uc9c1 \uc6b0\ub77c\ub284 \uad11\uc11d\uc5d0\uc11c Ac \ub3d9\uc704\uc6d0\uc18c\ub85c\ub9cc \ubc1c\uacac\ub418\uba70, Ac\ub294 21.772\ub144\uc758 \ubc18\uac10\uae30\ub97c \ub764\ub2e4. 1\ud1a4\uc758 \uc6b0\ub77c\ub284 \uad11\uc11d\uc5d0\uc11c\ub294 0.2\ubc00\ub9ac\uadf8\ub7a8 \uc815\ub3c4\uc758 \uc545\ud2f0\ub284\uc774 \ubc1c\uacac\ub41c\ub2e4. \uc545\ud2f0\ub284\uacfc \ub780\ud0c0\ub118\uc740 \ubb3c\ub9ac\uc801\u00b7\ud654\ud559\uc801 \uc131\uc9c8\uc774 \ub9e4\uc6b0 \ube44\uc2b7\ud558\uae30 \ub54c\ubb38\uc5d0, \uc6b0\ub77c\ub284 \uad11\uc11d\uc5d0\uc11c \uc774 \ub458\uc744 \uad6c\ubd84\ud558\ub294 \uac83\uc740 \ubd88\uac00\ub2a5\ud558\ub2e4. \uadf8 \ub300\uc2e0, \uc6d0\uc790\ub85c \uc548\uc5d0\uc11c Ra\uc5d0 \uc911\uc131\uc790 \uc120\uc744 \ucb10\uc600\uc744 \ub54c \ubc00\ub9ac\uadf8\ub7a8\uc758 \uc591\uc73c\ub85c \uc545\ud2f0\ub284\uc744 \ucd94\ucd9c\ud574\ub0bc \uc218 \uc788\ub2e4. \uc545\ud2f0\ub284\uc758 \uc591\uc774 \uc801\uace0, \ube44\uc2f8\uba70 \ubc29\uc0ac\uc131\uc774 \uc788\uae30 \ub54c\ubb38\uc5d0 \uc545\ud2f0\ub284\uc740 \uc0c1\uc5c5\uc801\uc73c\ub85c \uc798 \uc0ac\uc6a9\ub418\uc9c0 \uc54a\ub294\ub2e4. \ud604\uc7ac \uc545\ud2f0\ub284\uc740 \uc911\uc131\uc790\uc6d0\uc744 \ud3ec\ud568\ud558\uac70\ub098 \uc2e0\uccb4 \uc554\uc138\ud3ec\uc758 \ud45c\uc801\uc744 \uc124\uc815\ud558\ub294 \ubc29\uc0ac\uc120 \uc694\ubc95\uc758 \uc57d\ud488\uc73c\ub85c \ud65c\uc6a9\ub41c\ub2e4.", "answer": "\uc0b0\ud654\uc545\ud2f0\ub284", "sentence": "\ubd80\ub4dc\ub7fd\uace0, \uc740\ubc31\uc0c9\uc778 \uae08\uc18d \uc545\ud2f0\ub284\uc740 \uacf5\uae30 \uc911\uc5d0\uc11c \uc0b0\uc18c \ubc0f \uc218\ubd84\uacfc \uae09\uc18d\ud788 \uacb0\ud569\ud558\uc5ec \ud770\uc0c9 \ub9c9\uc758 \uc0b0\ud654\uc545\ud2f0\ub284 \uc744 \ud615\uc131\ud558\uba70, \uc774\ub294 \uc545\ud2f0\ub284\uc774 \ub354 \uc0b0\ud654\ub418\ub294 \uac83\uc744 \ubc29\uc9c0\ud55c\ub2e4.", "paragraph_sentence": " \ubd80\ub4dc\ub7fd\uace0, \uc740\ubc31\uc0c9\uc778 \uae08\uc18d \uc545\ud2f0\ub284\uc740 \uacf5\uae30 \uc911\uc5d0\uc11c \uc0b0\uc18c \ubc0f \uc218\ubd84\uacfc \uae09\uc18d\ud788 \uacb0\ud569\ud558\uc5ec \ud770\uc0c9 \ub9c9\uc758 \uc0b0\ud654\uc545\ud2f0\ub284 \uc744 \ud615\uc131\ud558\uba70, \uc774\ub294 \uc545\ud2f0\ub284\uc774 \ub354 \uc0b0\ud654\ub418\ub294 \uac83\uc744 \ubc29\uc9c0\ud55c\ub2e4. \ub300\ubd80\ubd84\uc758 \ub780\ud0c0\ub118\uc871\uacfc \uc545\ud2f0\ub284\uc871\uacfc \uac19\uc774, \uc545\ud2f0\ub284\uc740 \uac70\uc758 \ub300\ubd80\ubd84\uc758 \ud654\ud569\ubb3c\uc5d0\uc11c \ub300\ubd80\ubd84 +3\uc758 \uc0b0\ud654\uc218\ub97c \uac00\uc9c4\ub2e4. \uc545\ud2f0\ub284\uc740 \uc624\uc9c1 \uc6b0\ub77c\ub284 \uad11\uc11d\uc5d0\uc11c Ac \ub3d9\uc704\uc6d0\uc18c\ub85c\ub9cc \ubc1c\uacac\ub418\uba70, Ac\ub294 21.772\ub144\uc758 \ubc18\uac10\uae30\ub97c \ub764\ub2e4. 1\ud1a4\uc758 \uc6b0\ub77c\ub284 \uad11\uc11d\uc5d0\uc11c\ub294 0.2\ubc00\ub9ac\uadf8\ub7a8 \uc815\ub3c4\uc758 \uc545\ud2f0\ub284\uc774 \ubc1c\uacac\ub41c\ub2e4. \uc545\ud2f0\ub284\uacfc \ub780\ud0c0\ub118\uc740 \ubb3c\ub9ac\uc801\u00b7\ud654\ud559\uc801 \uc131\uc9c8\uc774 \ub9e4\uc6b0 \ube44\uc2b7\ud558\uae30 \ub54c\ubb38\uc5d0, \uc6b0\ub77c\ub284 \uad11\uc11d\uc5d0\uc11c \uc774 \ub458\uc744 \uad6c\ubd84\ud558\ub294 \uac83\uc740 \ubd88\uac00\ub2a5\ud558\ub2e4. \uadf8 \ub300\uc2e0, \uc6d0\uc790\ub85c \uc548\uc5d0\uc11c Ra\uc5d0 \uc911\uc131\uc790 \uc120\uc744 \ucb10\uc600\uc744 \ub54c \ubc00\ub9ac\uadf8\ub7a8\uc758 \uc591\uc73c\ub85c \uc545\ud2f0\ub284\uc744 \ucd94\ucd9c\ud574\ub0bc \uc218 \uc788\ub2e4. \uc545\ud2f0\ub284\uc758 \uc591\uc774 \uc801\uace0, \ube44\uc2f8\uba70 \ubc29\uc0ac\uc131\uc774 \uc788\uae30 \ub54c\ubb38\uc5d0 \uc545\ud2f0\ub284\uc740 \uc0c1\uc5c5\uc801\uc73c\ub85c \uc798 \uc0ac\uc6a9\ub418\uc9c0 \uc54a\ub294\ub2e4. \ud604\uc7ac \uc545\ud2f0\ub284\uc740 \uc911\uc131\uc790\uc6d0\uc744 \ud3ec\ud568\ud558\uac70\ub098 \uc2e0\uccb4 \uc554\uc138\ud3ec\uc758 \ud45c\uc801\uc744 \uc124\uc815\ud558\ub294 \ubc29\uc0ac\uc120 \uc694\ubc95\uc758 \uc57d\ud488\uc73c\ub85c \ud65c\uc6a9\ub41c\ub2e4.", "paragraph_answer": "\ubd80\ub4dc\ub7fd\uace0, \uc740\ubc31\uc0c9\uc778 \uae08\uc18d \uc545\ud2f0\ub284\uc740 \uacf5\uae30 \uc911\uc5d0\uc11c \uc0b0\uc18c \ubc0f \uc218\ubd84\uacfc \uae09\uc18d\ud788 \uacb0\ud569\ud558\uc5ec \ud770\uc0c9 \ub9c9\uc758 \uc0b0\ud654\uc545\ud2f0\ub284 \uc744 \ud615\uc131\ud558\uba70, \uc774\ub294 \uc545\ud2f0\ub284\uc774 \ub354 \uc0b0\ud654\ub418\ub294 \uac83\uc744 \ubc29\uc9c0\ud55c\ub2e4. \ub300\ubd80\ubd84\uc758 \ub780\ud0c0\ub118\uc871\uacfc \uc545\ud2f0\ub284\uc871\uacfc \uac19\uc774, \uc545\ud2f0\ub284\uc740 \uac70\uc758 \ub300\ubd80\ubd84\uc758 \ud654\ud569\ubb3c\uc5d0\uc11c \ub300\ubd80\ubd84 +3\uc758 \uc0b0\ud654\uc218\ub97c \uac00\uc9c4\ub2e4. \uc545\ud2f0\ub284\uc740 \uc624\uc9c1 \uc6b0\ub77c\ub284 \uad11\uc11d\uc5d0\uc11c Ac \ub3d9\uc704\uc6d0\uc18c\ub85c\ub9cc \ubc1c\uacac\ub418\uba70, Ac\ub294 21.772\ub144\uc758 \ubc18\uac10\uae30\ub97c \ub764\ub2e4. 1\ud1a4\uc758 \uc6b0\ub77c\ub284 \uad11\uc11d\uc5d0\uc11c\ub294 0.2\ubc00\ub9ac\uadf8\ub7a8 \uc815\ub3c4\uc758 \uc545\ud2f0\ub284\uc774 \ubc1c\uacac\ub41c\ub2e4. \uc545\ud2f0\ub284\uacfc \ub780\ud0c0\ub118\uc740 \ubb3c\ub9ac\uc801\u00b7\ud654\ud559\uc801 \uc131\uc9c8\uc774 \ub9e4\uc6b0 \ube44\uc2b7\ud558\uae30 \ub54c\ubb38\uc5d0, \uc6b0\ub77c\ub284 \uad11\uc11d\uc5d0\uc11c \uc774 \ub458\uc744 \uad6c\ubd84\ud558\ub294 \uac83\uc740 \ubd88\uac00\ub2a5\ud558\ub2e4. \uadf8 \ub300\uc2e0, \uc6d0\uc790\ub85c \uc548\uc5d0\uc11c Ra\uc5d0 \uc911\uc131\uc790 \uc120\uc744 \ucb10\uc600\uc744 \ub54c \ubc00\ub9ac\uadf8\ub7a8\uc758 \uc591\uc73c\ub85c \uc545\ud2f0\ub284\uc744 \ucd94\ucd9c\ud574\ub0bc \uc218 \uc788\ub2e4. \uc545\ud2f0\ub284\uc758 \uc591\uc774 \uc801\uace0, \ube44\uc2f8\uba70 \ubc29\uc0ac\uc131\uc774 \uc788\uae30 \ub54c\ubb38\uc5d0 \uc545\ud2f0\ub284\uc740 \uc0c1\uc5c5\uc801\uc73c\ub85c \uc798 \uc0ac\uc6a9\ub418\uc9c0 \uc54a\ub294\ub2e4. \ud604\uc7ac \uc545\ud2f0\ub284\uc740 \uc911\uc131\uc790\uc6d0\uc744 \ud3ec\ud568\ud558\uac70\ub098 \uc2e0\uccb4 \uc554\uc138\ud3ec\uc758 \ud45c\uc801\uc744 \uc124\uc815\ud558\ub294 \ubc29\uc0ac\uc120 \uc694\ubc95\uc758 \uc57d\ud488\uc73c\ub85c \ud65c\uc6a9\ub41c\ub2e4.", "sentence_answer": "\ubd80\ub4dc\ub7fd\uace0, \uc740\ubc31\uc0c9\uc778 \uae08\uc18d \uc545\ud2f0\ub284\uc740 \uacf5\uae30 \uc911\uc5d0\uc11c \uc0b0\uc18c \ubc0f \uc218\ubd84\uacfc \uae09\uc18d\ud788 \uacb0\ud569\ud558\uc5ec \ud770\uc0c9 \ub9c9\uc758 \uc0b0\ud654\uc545\ud2f0\ub284 \uc744 \ud615\uc131\ud558\uba70, \uc774\ub294 \uc545\ud2f0\ub284\uc774 \ub354 \uc0b0\ud654\ub418\ub294 \uac83\uc744 \ubc29\uc9c0\ud55c\ub2e4.", "paragraph_id": "6465399-0-0", "paragraph_question": "question: \uc545\ud2f0\ub284\uc774 \uc0b0\uc18c \ubc0f \uc218\ubd84\uacfc \uae09\uc18d\ud788 \uacb0\ud569\ud558\uba74\uc11c \ud615\uc131\ub418\ub294 \ubb3c\uc9c8\uc740?, context: \ubd80\ub4dc\ub7fd\uace0, \uc740\ubc31\uc0c9\uc778 \uae08\uc18d \uc545\ud2f0\ub284\uc740 \uacf5\uae30 \uc911\uc5d0\uc11c \uc0b0\uc18c \ubc0f \uc218\ubd84\uacfc \uae09\uc18d\ud788 \uacb0\ud569\ud558\uc5ec \ud770\uc0c9 \ub9c9\uc758 \uc0b0\ud654\uc545\ud2f0\ub284\uc744 \ud615\uc131\ud558\uba70, \uc774\ub294 \uc545\ud2f0\ub284\uc774 \ub354 \uc0b0\ud654\ub418\ub294 \uac83\uc744 \ubc29\uc9c0\ud55c\ub2e4. \ub300\ubd80\ubd84\uc758 \ub780\ud0c0\ub118\uc871\uacfc \uc545\ud2f0\ub284\uc871\uacfc \uac19\uc774, \uc545\ud2f0\ub284\uc740 \uac70\uc758 \ub300\ubd80\ubd84\uc758 \ud654\ud569\ubb3c\uc5d0\uc11c \ub300\ubd80\ubd84 +3\uc758 \uc0b0\ud654\uc218\ub97c \uac00\uc9c4\ub2e4. \uc545\ud2f0\ub284\uc740 \uc624\uc9c1 \uc6b0\ub77c\ub284 \uad11\uc11d\uc5d0\uc11c Ac \ub3d9\uc704\uc6d0\uc18c\ub85c\ub9cc \ubc1c\uacac\ub418\uba70, Ac\ub294 21.772\ub144\uc758 \ubc18\uac10\uae30\ub97c \ub764\ub2e4. 1\ud1a4\uc758 \uc6b0\ub77c\ub284 \uad11\uc11d\uc5d0\uc11c\ub294 0.2\ubc00\ub9ac\uadf8\ub7a8 \uc815\ub3c4\uc758 \uc545\ud2f0\ub284\uc774 \ubc1c\uacac\ub41c\ub2e4. \uc545\ud2f0\ub284\uacfc \ub780\ud0c0\ub118\uc740 \ubb3c\ub9ac\uc801\u00b7\ud654\ud559\uc801 \uc131\uc9c8\uc774 \ub9e4\uc6b0 \ube44\uc2b7\ud558\uae30 \ub54c\ubb38\uc5d0, \uc6b0\ub77c\ub284 \uad11\uc11d\uc5d0\uc11c \uc774 \ub458\uc744 \uad6c\ubd84\ud558\ub294 \uac83\uc740 \ubd88\uac00\ub2a5\ud558\ub2e4. \uadf8 \ub300\uc2e0, \uc6d0\uc790\ub85c \uc548\uc5d0\uc11c Ra\uc5d0 \uc911\uc131\uc790 \uc120\uc744 \ucb10\uc600\uc744 \ub54c \ubc00\ub9ac\uadf8\ub7a8\uc758 \uc591\uc73c\ub85c \uc545\ud2f0\ub284\uc744 \ucd94\ucd9c\ud574\ub0bc \uc218 \uc788\ub2e4. \uc545\ud2f0\ub284\uc758 \uc591\uc774 \uc801\uace0, \ube44\uc2f8\uba70 \ubc29\uc0ac\uc131\uc774 \uc788\uae30 \ub54c\ubb38\uc5d0 \uc545\ud2f0\ub284\uc740 \uc0c1\uc5c5\uc801\uc73c\ub85c \uc798 \uc0ac\uc6a9\ub418\uc9c0 \uc54a\ub294\ub2e4. \ud604\uc7ac \uc545\ud2f0\ub284\uc740 \uc911\uc131\uc790\uc6d0\uc744 \ud3ec\ud568\ud558\uac70\ub098 \uc2e0\uccb4 \uc554\uc138\ud3ec\uc758 \ud45c\uc801\uc744 \uc124\uc815\ud558\ub294 \ubc29\uc0ac\uc120 \uc694\ubc95\uc758 \uc57d\ud488\uc73c\ub85c \ud65c\uc6a9\ub41c\ub2e4."} {"question": "\ub4c0\uc2a4\uc758 3\uc9d1\uc778 'Force DEUX'\uc5d0 \uae40\uc131\uc7ac\uc758 \uc194\ub85c \ubcf4\uceec\uc774 \uc0bd\uc785\ub41c \uace1\uc740 \ubb34\uc5c7\uc778\uac00?", "paragraph": "2009\ub144 2\uc6d4 \uc0c8\ub86d\uac8c \uc120\ubcf4\uc774\ub294 \uc774\ud0c8\ub9ac\uc544 \ud504\ub9ac\ubbf8\uc5c4\uc9c4 \ube0c\ub79c\ub4dc\uc778 \"\ub9ac\ud50c\ub808\uc774\"\uc758 \ud55c\uad6d\ub860\uce6d\uc758 \uccab \ubaa8\ub378\ub85c \uae40\uc131\uc7ac\uac00 \ubc1c\ud0c1\ub418\uc5b4 \uc231\ud55c \ud654\uc81c\ub97c \ubaa8\uc558\ub2e4. \ub610\ud55c \uadf8\uc758 \uce5c\ud55c \uce5c\uad6c\uc778 \ub4c0\uc2a4\uc758 \uba64\ubc84,\uc774\ud604\ub3c4 \ub610\ud55c \uadf8\uc758 \ucef4\ubc31\uc744 \uae30\ub150\ud558\uc5ec \"\ub9ac\ud50c\ub808\uc774\"\uc758 \ud55c\uad6d\ub860\uce6d\uc758 CF\uc74c\uc545\uc744 \uc120\ubb3c\ud55c\ub2e4. \uc774\ud604\ub3c4\ub294 \uadf8\uc758 RMC\uc18c\uc18d\uc758 \uc74c\uc545\ud30c\ud2b8\ub108\uc778 \"Donnie J(\uc7a5\ub3d9\uade0)\"\uacfc \ud568\uaed8 \"\ub9d0\ud558\uc790\uba74\"\uc744 2009\ub144\uc758 \ud2b8\ub79c\ub4dc\uc5d0 \ub9de\uac8c \uc0c8\ub86d\uac8c \ud3b8\uace1\ud558\uc600\uc73c\uba70, \uc774\uc804\uc5d0 \ubc1c\ud45c\ub418\uc5c8\ub358 \ub4c0\uc2a4\uc758 \ubca0\uc2a4\ud2b8\uc74c\ubc18\uc778 \"DEUX FOREVER\"\uc74c\ubc18\uacfc \uac19\uc774 1\uc808\uc740 \uae40\uc131\uc7ac\uac00 2\uc808\uc740 \uc774\ud604\ub3c4\uac00 \ubd80\ub978\ub2e4. \uadf8\ub9ac\uace0 \ud6c4\ubc18\ubd80\uc5d0\ub294 \ub4c0\uc2a4\uc758 3\uc9d1\uc778 \"Force DEUX\"\uc5d0 \uc218\ub85d\ub41c \"\uc0c1\ucc98\"\uc758 \uae40\uc131\uc7ac \uc194\ub85c \ubcf4\uceec\uc774 \uc0bd\uc785\ub418\uc5b4 \ub354\uc6b1 \uadf8\uc5d0 \ub300\ud55c \uadf8\ub9ac\uc6c0\uc744 \ubc30\uac00\uc2dc\ud0a8\ub2e4.", "answer": "\uc0c1\ucc98", "sentence": "\uadf8\ub9ac\uace0 \ud6c4\ubc18\ubd80\uc5d0\ub294 \ub4c0\uc2a4\uc758 3\uc9d1\uc778 \"Force DEUX\"\uc5d0 \uc218\ub85d\ub41c \" \uc0c1\ucc98 \"\uc758 \uae40\uc131\uc7ac \uc194\ub85c \ubcf4\uceec\uc774 \uc0bd\uc785\ub418\uc5b4 \ub354\uc6b1 \uadf8\uc5d0 \ub300\ud55c \uadf8\ub9ac\uc6c0\uc744 \ubc30\uac00\uc2dc\ud0a8\ub2e4.", "paragraph_sentence": "2009\ub144 2\uc6d4 \uc0c8\ub86d\uac8c \uc120\ubcf4\uc774\ub294 \uc774\ud0c8\ub9ac\uc544 \ud504\ub9ac\ubbf8\uc5c4\uc9c4 \ube0c\ub79c\ub4dc\uc778 \"\ub9ac\ud50c\ub808\uc774\"\uc758 \ud55c\uad6d\ub860\uce6d\uc758 \uccab \ubaa8\ub378\ub85c \uae40\uc131\uc7ac\uac00 \ubc1c\ud0c1\ub418\uc5b4 \uc231\ud55c \ud654\uc81c\ub97c \ubaa8\uc558\ub2e4. \ub610\ud55c \uadf8\uc758 \uce5c\ud55c \uce5c\uad6c\uc778 \ub4c0\uc2a4\uc758 \uba64\ubc84,\uc774\ud604\ub3c4 \ub610\ud55c \uadf8\uc758 \ucef4\ubc31\uc744 \uae30\ub150\ud558\uc5ec \"\ub9ac\ud50c\ub808\uc774\"\uc758 \ud55c\uad6d\ub860\uce6d\uc758 CF\uc74c\uc545\uc744 \uc120\ubb3c\ud55c\ub2e4. \uc774\ud604\ub3c4\ub294 \uadf8\uc758 RMC\uc18c\uc18d\uc758 \uc74c\uc545\ud30c\ud2b8\ub108\uc778 \"Donnie J(\uc7a5\ub3d9\uade0)\"\uacfc \ud568\uaed8 \"\ub9d0\ud558\uc790\uba74\"\uc744 2009\ub144\uc758 \ud2b8\ub79c\ub4dc\uc5d0 \ub9de\uac8c \uc0c8\ub86d\uac8c \ud3b8\uace1\ud558\uc600\uc73c\uba70, \uc774\uc804\uc5d0 \ubc1c\ud45c\ub418\uc5c8\ub358 \ub4c0\uc2a4\uc758 \ubca0\uc2a4\ud2b8\uc74c\ubc18\uc778 \"DEUX FOREVER\"\uc74c\ubc18\uacfc \uac19\uc774 1\uc808\uc740 \uae40\uc131\uc7ac\uac00 2\uc808\uc740 \uc774\ud604\ub3c4\uac00 \ubd80\ub978\ub2e4. \uadf8\ub9ac\uace0 \ud6c4\ubc18\ubd80\uc5d0\ub294 \ub4c0\uc2a4\uc758 3\uc9d1\uc778 \"Force DEUX\"\uc5d0 \uc218\ub85d\ub41c \" \uc0c1\ucc98 \"\uc758 \uae40\uc131\uc7ac \uc194\ub85c \ubcf4\uceec\uc774 \uc0bd\uc785\ub418\uc5b4 \ub354\uc6b1 \uadf8\uc5d0 \ub300\ud55c \uadf8\ub9ac\uc6c0\uc744 \ubc30\uac00\uc2dc\ud0a8\ub2e4. ", "paragraph_answer": "2009\ub144 2\uc6d4 \uc0c8\ub86d\uac8c \uc120\ubcf4\uc774\ub294 \uc774\ud0c8\ub9ac\uc544 \ud504\ub9ac\ubbf8\uc5c4\uc9c4 \ube0c\ub79c\ub4dc\uc778 \"\ub9ac\ud50c\ub808\uc774\"\uc758 \ud55c\uad6d\ub860\uce6d\uc758 \uccab \ubaa8\ub378\ub85c \uae40\uc131\uc7ac\uac00 \ubc1c\ud0c1\ub418\uc5b4 \uc231\ud55c \ud654\uc81c\ub97c \ubaa8\uc558\ub2e4. \ub610\ud55c \uadf8\uc758 \uce5c\ud55c \uce5c\uad6c\uc778 \ub4c0\uc2a4\uc758 \uba64\ubc84,\uc774\ud604\ub3c4 \ub610\ud55c \uadf8\uc758 \ucef4\ubc31\uc744 \uae30\ub150\ud558\uc5ec \"\ub9ac\ud50c\ub808\uc774\"\uc758 \ud55c\uad6d\ub860\uce6d\uc758 CF\uc74c\uc545\uc744 \uc120\ubb3c\ud55c\ub2e4. \uc774\ud604\ub3c4\ub294 \uadf8\uc758 RMC\uc18c\uc18d\uc758 \uc74c\uc545\ud30c\ud2b8\ub108\uc778 \"Donnie J(\uc7a5\ub3d9\uade0)\"\uacfc \ud568\uaed8 \"\ub9d0\ud558\uc790\uba74\"\uc744 2009\ub144\uc758 \ud2b8\ub79c\ub4dc\uc5d0 \ub9de\uac8c \uc0c8\ub86d\uac8c \ud3b8\uace1\ud558\uc600\uc73c\uba70, \uc774\uc804\uc5d0 \ubc1c\ud45c\ub418\uc5c8\ub358 \ub4c0\uc2a4\uc758 \ubca0\uc2a4\ud2b8\uc74c\ubc18\uc778 \"DEUX FOREVER\"\uc74c\ubc18\uacfc \uac19\uc774 1\uc808\uc740 \uae40\uc131\uc7ac\uac00 2\uc808\uc740 \uc774\ud604\ub3c4\uac00 \ubd80\ub978\ub2e4. \uadf8\ub9ac\uace0 \ud6c4\ubc18\ubd80\uc5d0\ub294 \ub4c0\uc2a4\uc758 3\uc9d1\uc778 \"Force DEUX\"\uc5d0 \uc218\ub85d\ub41c \" \uc0c1\ucc98 \"\uc758 \uae40\uc131\uc7ac \uc194\ub85c \ubcf4\uceec\uc774 \uc0bd\uc785\ub418\uc5b4 \ub354\uc6b1 \uadf8\uc5d0 \ub300\ud55c \uadf8\ub9ac\uc6c0\uc744 \ubc30\uac00\uc2dc\ud0a8\ub2e4.", "sentence_answer": "\uadf8\ub9ac\uace0 \ud6c4\ubc18\ubd80\uc5d0\ub294 \ub4c0\uc2a4\uc758 3\uc9d1\uc778 \"Force DEUX\"\uc5d0 \uc218\ub85d\ub41c \" \uc0c1\ucc98 \"\uc758 \uae40\uc131\uc7ac \uc194\ub85c \ubcf4\uceec\uc774 \uc0bd\uc785\ub418\uc5b4 \ub354\uc6b1 \uadf8\uc5d0 \ub300\ud55c \uadf8\ub9ac\uc6c0\uc744 \ubc30\uac00\uc2dc\ud0a8\ub2e4.", "paragraph_id": "6490539-0-2", "paragraph_question": "question: \ub4c0\uc2a4\uc758 3\uc9d1\uc778 'Force DEUX'\uc5d0 \uae40\uc131\uc7ac\uc758 \uc194\ub85c \ubcf4\uceec\uc774 \uc0bd\uc785\ub41c \uace1\uc740 \ubb34\uc5c7\uc778\uac00?, context: 2009\ub144 2\uc6d4 \uc0c8\ub86d\uac8c \uc120\ubcf4\uc774\ub294 \uc774\ud0c8\ub9ac\uc544 \ud504\ub9ac\ubbf8\uc5c4\uc9c4 \ube0c\ub79c\ub4dc\uc778 \"\ub9ac\ud50c\ub808\uc774\"\uc758 \ud55c\uad6d\ub860\uce6d\uc758 \uccab \ubaa8\ub378\ub85c \uae40\uc131\uc7ac\uac00 \ubc1c\ud0c1\ub418\uc5b4 \uc231\ud55c \ud654\uc81c\ub97c \ubaa8\uc558\ub2e4. \ub610\ud55c \uadf8\uc758 \uce5c\ud55c \uce5c\uad6c\uc778 \ub4c0\uc2a4\uc758 \uba64\ubc84,\uc774\ud604\ub3c4 \ub610\ud55c \uadf8\uc758 \ucef4\ubc31\uc744 \uae30\ub150\ud558\uc5ec \"\ub9ac\ud50c\ub808\uc774\"\uc758 \ud55c\uad6d\ub860\uce6d\uc758 CF\uc74c\uc545\uc744 \uc120\ubb3c\ud55c\ub2e4. \uc774\ud604\ub3c4\ub294 \uadf8\uc758 RMC\uc18c\uc18d\uc758 \uc74c\uc545\ud30c\ud2b8\ub108\uc778 \"Donnie J(\uc7a5\ub3d9\uade0)\"\uacfc \ud568\uaed8 \"\ub9d0\ud558\uc790\uba74\"\uc744 2009\ub144\uc758 \ud2b8\ub79c\ub4dc\uc5d0 \ub9de\uac8c \uc0c8\ub86d\uac8c \ud3b8\uace1\ud558\uc600\uc73c\uba70, \uc774\uc804\uc5d0 \ubc1c\ud45c\ub418\uc5c8\ub358 \ub4c0\uc2a4\uc758 \ubca0\uc2a4\ud2b8\uc74c\ubc18\uc778 \"DEUX FOREVER\"\uc74c\ubc18\uacfc \uac19\uc774 1\uc808\uc740 \uae40\uc131\uc7ac\uac00 2\uc808\uc740 \uc774\ud604\ub3c4\uac00 \ubd80\ub978\ub2e4. \uadf8\ub9ac\uace0 \ud6c4\ubc18\ubd80\uc5d0\ub294 \ub4c0\uc2a4\uc758 3\uc9d1\uc778 \"Force DEUX\"\uc5d0 \uc218\ub85d\ub41c \"\uc0c1\ucc98\"\uc758 \uae40\uc131\uc7ac \uc194\ub85c \ubcf4\uceec\uc774 \uc0bd\uc785\ub418\uc5b4 \ub354\uc6b1 \uadf8\uc5d0 \ub300\ud55c \uadf8\ub9ac\uc6c0\uc744 \ubc30\uac00\uc2dc\ud0a8\ub2e4."} {"question": "\uae40\uae30\uc2dd\uc740 2014\ub144\uacfc 2015\ub144 \ub2f9\uc2dc \uc790\uc2e0\uacfc \uce5c\ubd84\uc774 \uc788\ub294 \uc5f0\uad6c\uc18c\ub4e4\uc5d0\uac8c \ubb34\uc2a8 \ubc29\uc2dd\uc73c\ub85c \uc5f0\uad6c\uc6a9\uc5ed\ub4e4\uc744 \uccb4\uacb0\ud558\uc600\ub294\uac00?", "paragraph": "\uae40\uae30\uc2dd\uc774 \uc124\ub9bd\ud55c \ub354\ubbf8\ub798\uc5f0\uad6c\uc18c\uac00 \uad6d\ud68c \uc0c1\uc784\uc704\uc6d0\ud68c\uc5d0\uc11c \ub354\ubd88\uc5b4\ubbfc\uc8fc\ub2f9 \uc18c\uc18d \uc758\uc6d0\uc73c\ub85c\ubd80\ud130 \ub2e4\ub7c9\uc758 \uc5f0\uad6c\uc6a9\uc5ed\uc744 \uc218\uc8fc\ud55c \uac83\uc73c\ub85c \ub098\ud0c0\ub0ac\ub2e4. \uc5f0\uad6c\uc6a9\uc5ed\uc740 \ub354\ubbf8\ub798\uc5f0\uad6c\uc18c \uc18c\uc18d \uc758\uc6d0\uc774 \ubc1c\uc8fc\ud55c \uac83\uc774\uc5b4\uc11c \uc0ac\uc2e4\uc0c1 \uc790\uc2e0\uc774 \uc9c1\uc811\uc801\uc73c\ub85c \uad00\uc5ec\ud558\uace0 \uc788\ub294 \uc2f1\ud06c\ud0f1\ud06c\uc5d0 \uc6a9\uc5ed \uc77c\uac10\uc744 \ubab0\uc544\uc92c\ub2e4\ub294 \uc758\ud639\uc774 \uc81c\uae30\ub41c\ub2e4. \ud1b5\uc0c1 \uad6d\ud68c \uc0c1\uc784\uc704\ub294 \ud615\ud3c9\uc131 \ucc28\uc6d0\uc5d0\uc11c \ud2b9\uc815 \uc5f0\uad6c\uc7ac\ub2e8\uc774\ub098 \ub300\ud559\uc5d0 3~4\uac1c\uc758 \uc6a9\uc5ed\uc744 \ube44\uc2b7\ud55c \uc2dc\uae30\uc5d0 \ud55c\uaebc\ubc88\uc5d0 \uc8fc\uc9c0 \uc54a\ub294\ub2e4. \uad6d\ud68c\uac00 2017\ub144\ubd80\ud130 \uc815\ucc45 \uc5f0\uad6c\uc6a9\uc5ed \uacfc\uc81c \ubc1c\uc8fc \ubc29\uc2dd\uc744 \uacf5\uac1c\uacbd\uc7c1\uc73c\ub85c \ubc14\uafbc \uac83\ub3c4 \uc758\uc6d0\uc758 \uc774\ud574\uad00\uacc4\uac00 \uac1c\uc785\ub418\uc9c0 \uc54a\ub3c4\ub85d \ud558\uae30 \uc704\ud574\uc11c\uc600\ub2e4. \uc815\ubb34\uc704\uc758 \ud55c \uc57c\ub2f9 \uc758\uc6d0\uc740 \"\uc0ac\uc2e4\uc0c1 \uc790\uc2e0\uacfc \uce5c\ubd84\uc774 \uc788\ub294 \uae30\uad00\uc5d0 \uad6d\ubbfc \uc138\uae08\uc744 \uc900 \uac83\"\uc774\ub77c\uace0 \uc9c0\uc801\ud588\ub2e4. \uae40\uae30\uc2dd\uc774 \uad6d\ud68c\uc758\uc6d0 \uc2e0\ubd84\uc774\ub358 2014\ub144\uacfc 2015\ub144 \uc815\ubb34\uc704 \uac04\uc0ac\ub97c \ud558\uba70 \uc790\uc2e0\uacfc \uce5c\ubd84\uc774 \uc788\ub294 \uacbd\uc81c\uac1c\ud601\uc5f0\uad6c\uc18c\uc640 \uc88b\uc740\uae30\uc5c5\uc9c0\ubc30\uad6c\uc870\uc5f0\uad6c\uc18c\uc5d0 \uc218\uc758\uacc4\uc57d \ubc29\uc2dd\uc73c\ub85c 1000\ub9cc\uc6d0\uc529 \uc5f0\uad6c\uc6a9\uc5ed\uc744 \uccb4\uacb0\ud558\uae30\ub3c4 \ud588\ub2e4.", "answer": "\uc218\uc758\uacc4\uc57d \ubc29\uc2dd", "sentence": "\uc790\uc2e0\uacfc \uce5c\ubd84\uc774 \uc788\ub294 \uacbd\uc81c\uac1c\ud601\uc5f0\uad6c\uc18c\uc640 \uc88b\uc740\uae30\uc5c5\uc9c0\ubc30\uad6c\uc870\uc5f0\uad6c\uc18c\uc5d0 \uc218\uc758\uacc4\uc57d \ubc29\uc2dd \uc73c\ub85c 1000\ub9cc\uc6d0\uc529 \uc5f0\uad6c\uc6a9\uc5ed\uc744 \uccb4\uacb0\ud558\uae30\ub3c4 \ud588\ub2e4.", "paragraph_sentence": "\uae40\uae30\uc2dd\uc774 \uc124\ub9bd\ud55c \ub354\ubbf8\ub798\uc5f0\uad6c\uc18c\uac00 \uad6d\ud68c \uc0c1\uc784\uc704\uc6d0\ud68c\uc5d0\uc11c \ub354\ubd88\uc5b4\ubbfc\uc8fc\ub2f9 \uc18c\uc18d \uc758\uc6d0\uc73c\ub85c\ubd80\ud130 \ub2e4\ub7c9\uc758 \uc5f0\uad6c\uc6a9\uc5ed\uc744 \uc218\uc8fc\ud55c \uac83\uc73c\ub85c \ub098\ud0c0\ub0ac\ub2e4. \uc5f0\uad6c\uc6a9\uc5ed\uc740 \ub354\ubbf8\ub798\uc5f0\uad6c\uc18c \uc18c\uc18d \uc758\uc6d0\uc774 \ubc1c\uc8fc\ud55c \uac83\uc774\uc5b4\uc11c \uc0ac\uc2e4\uc0c1 \uc790\uc2e0\uc774 \uc9c1\uc811\uc801\uc73c\ub85c \uad00\uc5ec\ud558\uace0 \uc788\ub294 \uc2f1\ud06c\ud0f1\ud06c\uc5d0 \uc6a9\uc5ed \uc77c\uac10\uc744 \ubab0\uc544\uc92c\ub2e4\ub294 \uc758\ud639\uc774 \uc81c\uae30\ub41c\ub2e4. \ud1b5\uc0c1 \uad6d\ud68c \uc0c1\uc784\uc704\ub294 \ud615\ud3c9\uc131 \ucc28\uc6d0\uc5d0\uc11c \ud2b9\uc815 \uc5f0\uad6c\uc7ac\ub2e8\uc774\ub098 \ub300\ud559\uc5d0 3~4\uac1c\uc758 \uc6a9\uc5ed\uc744 \ube44\uc2b7\ud55c \uc2dc\uae30\uc5d0 \ud55c\uaebc\ubc88\uc5d0 \uc8fc\uc9c0 \uc54a\ub294\ub2e4. \uad6d\ud68c\uac00 2017\ub144\ubd80\ud130 \uc815\ucc45 \uc5f0\uad6c\uc6a9\uc5ed \uacfc\uc81c \ubc1c\uc8fc \ubc29\uc2dd\uc744 \uacf5\uac1c\uacbd\uc7c1\uc73c\ub85c \ubc14\uafbc \uac83\ub3c4 \uc758\uc6d0\uc758 \uc774\ud574\uad00\uacc4\uac00 \uac1c\uc785\ub418\uc9c0 \uc54a\ub3c4\ub85d \ud558\uae30 \uc704\ud574\uc11c\uc600\ub2e4. \uc815\ubb34\uc704\uc758 \ud55c \uc57c\ub2f9 \uc758\uc6d0\uc740 \"\uc0ac\uc2e4\uc0c1 \uc790\uc2e0\uacfc \uce5c\ubd84\uc774 \uc788\ub294 \uae30\uad00\uc5d0 \uad6d\ubbfc \uc138\uae08\uc744 \uc900 \uac83\"\uc774\ub77c\uace0 \uc9c0\uc801\ud588\ub2e4. \uae40\uae30\uc2dd\uc774 \uad6d\ud68c\uc758\uc6d0 \uc2e0\ubd84\uc774\ub358 2014\ub144\uacfc 2015\ub144 \uc815\ubb34\uc704 \uac04\uc0ac\ub97c \ud558\uba70 \uc790\uc2e0\uacfc \uce5c\ubd84\uc774 \uc788\ub294 \uacbd\uc81c\uac1c\ud601\uc5f0\uad6c\uc18c\uc640 \uc88b\uc740\uae30\uc5c5\uc9c0\ubc30\uad6c\uc870\uc5f0\uad6c\uc18c\uc5d0 \uc218\uc758\uacc4\uc57d \ubc29\uc2dd \uc73c\ub85c 1000\ub9cc\uc6d0\uc529 \uc5f0\uad6c\uc6a9\uc5ed\uc744 \uccb4\uacb0\ud558\uae30\ub3c4 \ud588\ub2e4. ", "paragraph_answer": "\uae40\uae30\uc2dd\uc774 \uc124\ub9bd\ud55c \ub354\ubbf8\ub798\uc5f0\uad6c\uc18c\uac00 \uad6d\ud68c \uc0c1\uc784\uc704\uc6d0\ud68c\uc5d0\uc11c \ub354\ubd88\uc5b4\ubbfc\uc8fc\ub2f9 \uc18c\uc18d \uc758\uc6d0\uc73c\ub85c\ubd80\ud130 \ub2e4\ub7c9\uc758 \uc5f0\uad6c\uc6a9\uc5ed\uc744 \uc218\uc8fc\ud55c \uac83\uc73c\ub85c \ub098\ud0c0\ub0ac\ub2e4. \uc5f0\uad6c\uc6a9\uc5ed\uc740 \ub354\ubbf8\ub798\uc5f0\uad6c\uc18c \uc18c\uc18d \uc758\uc6d0\uc774 \ubc1c\uc8fc\ud55c \uac83\uc774\uc5b4\uc11c \uc0ac\uc2e4\uc0c1 \uc790\uc2e0\uc774 \uc9c1\uc811\uc801\uc73c\ub85c \uad00\uc5ec\ud558\uace0 \uc788\ub294 \uc2f1\ud06c\ud0f1\ud06c\uc5d0 \uc6a9\uc5ed \uc77c\uac10\uc744 \ubab0\uc544\uc92c\ub2e4\ub294 \uc758\ud639\uc774 \uc81c\uae30\ub41c\ub2e4. \ud1b5\uc0c1 \uad6d\ud68c \uc0c1\uc784\uc704\ub294 \ud615\ud3c9\uc131 \ucc28\uc6d0\uc5d0\uc11c \ud2b9\uc815 \uc5f0\uad6c\uc7ac\ub2e8\uc774\ub098 \ub300\ud559\uc5d0 3~4\uac1c\uc758 \uc6a9\uc5ed\uc744 \ube44\uc2b7\ud55c \uc2dc\uae30\uc5d0 \ud55c\uaebc\ubc88\uc5d0 \uc8fc\uc9c0 \uc54a\ub294\ub2e4. \uad6d\ud68c\uac00 2017\ub144\ubd80\ud130 \uc815\ucc45 \uc5f0\uad6c\uc6a9\uc5ed \uacfc\uc81c \ubc1c\uc8fc \ubc29\uc2dd\uc744 \uacf5\uac1c\uacbd\uc7c1\uc73c\ub85c \ubc14\uafbc \uac83\ub3c4 \uc758\uc6d0\uc758 \uc774\ud574\uad00\uacc4\uac00 \uac1c\uc785\ub418\uc9c0 \uc54a\ub3c4\ub85d \ud558\uae30 \uc704\ud574\uc11c\uc600\ub2e4. \uc815\ubb34\uc704\uc758 \ud55c \uc57c\ub2f9 \uc758\uc6d0\uc740 \"\uc0ac\uc2e4\uc0c1 \uc790\uc2e0\uacfc \uce5c\ubd84\uc774 \uc788\ub294 \uae30\uad00\uc5d0 \uad6d\ubbfc \uc138\uae08\uc744 \uc900 \uac83\"\uc774\ub77c\uace0 \uc9c0\uc801\ud588\ub2e4. \uae40\uae30\uc2dd\uc774 \uad6d\ud68c\uc758\uc6d0 \uc2e0\ubd84\uc774\ub358 2014\ub144\uacfc 2015\ub144 \uc815\ubb34\uc704 \uac04\uc0ac\ub97c \ud558\uba70 \uc790\uc2e0\uacfc \uce5c\ubd84\uc774 \uc788\ub294 \uacbd\uc81c\uac1c\ud601\uc5f0\uad6c\uc18c\uc640 \uc88b\uc740\uae30\uc5c5\uc9c0\ubc30\uad6c\uc870\uc5f0\uad6c\uc18c\uc5d0 \uc218\uc758\uacc4\uc57d \ubc29\uc2dd \uc73c\ub85c 1000\ub9cc\uc6d0\uc529 \uc5f0\uad6c\uc6a9\uc5ed\uc744 \uccb4\uacb0\ud558\uae30\ub3c4 \ud588\ub2e4.", "sentence_answer": "\uc790\uc2e0\uacfc \uce5c\ubd84\uc774 \uc788\ub294 \uacbd\uc81c\uac1c\ud601\uc5f0\uad6c\uc18c\uc640 \uc88b\uc740\uae30\uc5c5\uc9c0\ubc30\uad6c\uc870\uc5f0\uad6c\uc18c\uc5d0 \uc218\uc758\uacc4\uc57d \ubc29\uc2dd \uc73c\ub85c 1000\ub9cc\uc6d0\uc529 \uc5f0\uad6c\uc6a9\uc5ed\uc744 \uccb4\uacb0\ud558\uae30\ub3c4 \ud588\ub2e4.", "paragraph_id": "6481286-7-1", "paragraph_question": "question: \uae40\uae30\uc2dd\uc740 2014\ub144\uacfc 2015\ub144 \ub2f9\uc2dc \uc790\uc2e0\uacfc \uce5c\ubd84\uc774 \uc788\ub294 \uc5f0\uad6c\uc18c\ub4e4\uc5d0\uac8c \ubb34\uc2a8 \ubc29\uc2dd\uc73c\ub85c \uc5f0\uad6c\uc6a9\uc5ed\ub4e4\uc744 \uccb4\uacb0\ud558\uc600\ub294\uac00?, context: \uae40\uae30\uc2dd\uc774 \uc124\ub9bd\ud55c \ub354\ubbf8\ub798\uc5f0\uad6c\uc18c\uac00 \uad6d\ud68c \uc0c1\uc784\uc704\uc6d0\ud68c\uc5d0\uc11c \ub354\ubd88\uc5b4\ubbfc\uc8fc\ub2f9 \uc18c\uc18d \uc758\uc6d0\uc73c\ub85c\ubd80\ud130 \ub2e4\ub7c9\uc758 \uc5f0\uad6c\uc6a9\uc5ed\uc744 \uc218\uc8fc\ud55c \uac83\uc73c\ub85c \ub098\ud0c0\ub0ac\ub2e4. \uc5f0\uad6c\uc6a9\uc5ed\uc740 \ub354\ubbf8\ub798\uc5f0\uad6c\uc18c \uc18c\uc18d \uc758\uc6d0\uc774 \ubc1c\uc8fc\ud55c \uac83\uc774\uc5b4\uc11c \uc0ac\uc2e4\uc0c1 \uc790\uc2e0\uc774 \uc9c1\uc811\uc801\uc73c\ub85c \uad00\uc5ec\ud558\uace0 \uc788\ub294 \uc2f1\ud06c\ud0f1\ud06c\uc5d0 \uc6a9\uc5ed \uc77c\uac10\uc744 \ubab0\uc544\uc92c\ub2e4\ub294 \uc758\ud639\uc774 \uc81c\uae30\ub41c\ub2e4. \ud1b5\uc0c1 \uad6d\ud68c \uc0c1\uc784\uc704\ub294 \ud615\ud3c9\uc131 \ucc28\uc6d0\uc5d0\uc11c \ud2b9\uc815 \uc5f0\uad6c\uc7ac\ub2e8\uc774\ub098 \ub300\ud559\uc5d0 3~4\uac1c\uc758 \uc6a9\uc5ed\uc744 \ube44\uc2b7\ud55c \uc2dc\uae30\uc5d0 \ud55c\uaebc\ubc88\uc5d0 \uc8fc\uc9c0 \uc54a\ub294\ub2e4. \uad6d\ud68c\uac00 2017\ub144\ubd80\ud130 \uc815\ucc45 \uc5f0\uad6c\uc6a9\uc5ed \uacfc\uc81c \ubc1c\uc8fc \ubc29\uc2dd\uc744 \uacf5\uac1c\uacbd\uc7c1\uc73c\ub85c \ubc14\uafbc \uac83\ub3c4 \uc758\uc6d0\uc758 \uc774\ud574\uad00\uacc4\uac00 \uac1c\uc785\ub418\uc9c0 \uc54a\ub3c4\ub85d \ud558\uae30 \uc704\ud574\uc11c\uc600\ub2e4. \uc815\ubb34\uc704\uc758 \ud55c \uc57c\ub2f9 \uc758\uc6d0\uc740 \"\uc0ac\uc2e4\uc0c1 \uc790\uc2e0\uacfc \uce5c\ubd84\uc774 \uc788\ub294 \uae30\uad00\uc5d0 \uad6d\ubbfc \uc138\uae08\uc744 \uc900 \uac83\"\uc774\ub77c\uace0 \uc9c0\uc801\ud588\ub2e4. \uae40\uae30\uc2dd\uc774 \uad6d\ud68c\uc758\uc6d0 \uc2e0\ubd84\uc774\ub358 2014\ub144\uacfc 2015\ub144 \uc815\ubb34\uc704 \uac04\uc0ac\ub97c \ud558\uba70 \uc790\uc2e0\uacfc \uce5c\ubd84\uc774 \uc788\ub294 \uacbd\uc81c\uac1c\ud601\uc5f0\uad6c\uc18c\uc640 \uc88b\uc740\uae30\uc5c5\uc9c0\ubc30\uad6c\uc870\uc5f0\uad6c\uc18c\uc5d0 \uc218\uc758\uacc4\uc57d \ubc29\uc2dd\uc73c\ub85c 1000\ub9cc\uc6d0\uc529 \uc5f0\uad6c\uc6a9\uc5ed\uc744 \uccb4\uacb0\ud558\uae30\ub3c4 \ud588\ub2e4."} {"question": "\uc724\ubbf8\ub798\uac00 \ud0c0\uc774\uac70 JK\uc640 \ud568\uaed8 \ucc3d\ub9bd\ud55c \uc5d4\ud130\ud14c\uc778\uba3c\ud2b8\ub294 \ubb34\uc5c7\uc778\uac00?", "paragraph": "2006\ub144 \uadf8\ub140\ub294 \ud0c0\uc774\uac70 JK(Tiger JK)\uc640 \ud568\uaed8 \uc815\uae00 \uc5d4\ud130\ud14c\uc778\uba3c\ud2b8(Jungle Ent.)\uc758 \ucc3d\ub9bd \uba64\ubc84\uac00 \ub418\uc5c8\uc73c\uba70 4\ub144 \ub9cc\uc5d0 \uc138 \ubc88\uc9f8 \uc568\ubc94 \"Yoonmirae\"\ub85c \ucef4\ubc31\ud558\uc5ec \uc88b\uc740 \ubc18\uc751\uc744 \uc5bb\uc5c8\ub2e4. \uc774\ud6c4 Tiger JK\uc640 \uacb0\ud63c, 2008\ub144 3\uc6d4 \uc544\ub4e4\uc744 \ub0b3\uc558\uc73c\uba70, \ud55c\ub3d9\uc548 \uc0b0\ud6c4\uc870\ub9ac\ub85c \ud734\uc2dd\uc744 \ucde8\ud558\ub2e4 2009\ub144 1\uc6d4\uc5d0 \ub514\uc9c0\ud138 \uc2f1\uae00\uc744 \ub0b4\uace0 \uc7a0\uc2dc \ud65c\ub3d9\ud558\uc600\ub2e4. 2009\ub144\uc5d0 \ub4e4\uc5b4\uc11c\ub294 \ubb34\ud55c\ub3c4\uc804 \ud504\ub85c\uadf8\ub7a8\uc758 \uc77c\ud658\uc73c\ub85c \ud4e8\uccd0 \ub77c\uc774\uac70 \ud65c\ub3d9 \ubc0f Drunken Tiger 8\uc9d1, Supreme Team 1\uc9d1 \ud53c\uccd0\ub9c1 \ucc38\uc5ec \ub4f1\uc758 \ud65c\ub3d9\uc744 \ud558\uc600\uace0, \uadf8\uc640 \ud568\uaed8 \uc0c8 \uc568\ubc94 \uc791\uc5c5\uc5d0 \ub4e4\uc5b4\uac14\ub2e4. \uc774 \uc568\ubc94\uc740 2010\ub144 8\uc6d4\uc5d0 \ub098\uc62c \uac83\uc774 \uc608\uace0\ub418\uc5c8\uc73c\ub098 \ub098\uc624\uc9c0\ub294 \ubabb\ud558\uc600\ub2e4.", "answer": "\uc815\uae00 \uc5d4\ud130\ud14c\uc778\uba3c\ud2b8", "sentence": "2006\ub144 \uadf8\ub140\ub294 \ud0c0\uc774\uac70 JK(Tiger JK)\uc640 \ud568\uaed8 \uc815\uae00 \uc5d4\ud130\ud14c\uc778\uba3c\ud2b8 (Jungle Ent.)\uc758 \ucc3d\ub9bd \uba64\ubc84\uac00 \ub418\uc5c8\uc73c\uba70 4\ub144 \ub9cc\uc5d0 \uc138 \ubc88\uc9f8 \uc568\ubc94 \"Yoonmirae\"\ub85c \ucef4\ubc31\ud558\uc5ec \uc88b\uc740 \ubc18\uc751\uc744 \uc5bb\uc5c8\ub2e4.", "paragraph_sentence": " 2006\ub144 \uadf8\ub140\ub294 \ud0c0\uc774\uac70 JK(Tiger JK)\uc640 \ud568\uaed8 \uc815\uae00 \uc5d4\ud130\ud14c\uc778\uba3c\ud2b8 (Jungle Ent.)\uc758 \ucc3d\ub9bd \uba64\ubc84\uac00 \ub418\uc5c8\uc73c\uba70 4\ub144 \ub9cc\uc5d0 \uc138 \ubc88\uc9f8 \uc568\ubc94 \"Yoonmirae\"\ub85c \ucef4\ubc31\ud558\uc5ec \uc88b\uc740 \ubc18\uc751\uc744 \uc5bb\uc5c8\ub2e4. \uc774\ud6c4 Tiger JK\uc640 \uacb0\ud63c, 2008\ub144 3\uc6d4 \uc544\ub4e4\uc744 \ub0b3\uc558\uc73c\uba70, \ud55c\ub3d9\uc548 \uc0b0\ud6c4\uc870\ub9ac\ub85c \ud734\uc2dd\uc744 \ucde8\ud558\ub2e4 2009\ub144 1\uc6d4\uc5d0 \ub514\uc9c0\ud138 \uc2f1\uae00\uc744 \ub0b4\uace0 \uc7a0\uc2dc \ud65c\ub3d9\ud558\uc600\ub2e4. 2009\ub144\uc5d0 \ub4e4\uc5b4\uc11c\ub294 \ubb34\ud55c\ub3c4\uc804 \ud504\ub85c\uadf8\ub7a8\uc758 \uc77c\ud658\uc73c\ub85c \ud4e8\uccd0 \ub77c\uc774\uac70 \ud65c\ub3d9 \ubc0f Drunken Tiger 8\uc9d1, Supreme Team 1\uc9d1 \ud53c\uccd0\ub9c1 \ucc38\uc5ec \ub4f1\uc758 \ud65c\ub3d9\uc744 \ud558\uc600\uace0, \uadf8\uc640 \ud568\uaed8 \uc0c8 \uc568\ubc94 \uc791\uc5c5\uc5d0 \ub4e4\uc5b4\uac14\ub2e4. \uc774 \uc568\ubc94\uc740 2010\ub144 8\uc6d4\uc5d0 \ub098\uc62c \uac83\uc774 \uc608\uace0\ub418\uc5c8\uc73c\ub098 \ub098\uc624\uc9c0\ub294 \ubabb\ud558\uc600\ub2e4.", "paragraph_answer": "2006\ub144 \uadf8\ub140\ub294 \ud0c0\uc774\uac70 JK(Tiger JK)\uc640 \ud568\uaed8 \uc815\uae00 \uc5d4\ud130\ud14c\uc778\uba3c\ud2b8 (Jungle Ent.)\uc758 \ucc3d\ub9bd \uba64\ubc84\uac00 \ub418\uc5c8\uc73c\uba70 4\ub144 \ub9cc\uc5d0 \uc138 \ubc88\uc9f8 \uc568\ubc94 \"Yoonmirae\"\ub85c \ucef4\ubc31\ud558\uc5ec \uc88b\uc740 \ubc18\uc751\uc744 \uc5bb\uc5c8\ub2e4. \uc774\ud6c4 Tiger JK\uc640 \uacb0\ud63c, 2008\ub144 3\uc6d4 \uc544\ub4e4\uc744 \ub0b3\uc558\uc73c\uba70, \ud55c\ub3d9\uc548 \uc0b0\ud6c4\uc870\ub9ac\ub85c \ud734\uc2dd\uc744 \ucde8\ud558\ub2e4 2009\ub144 1\uc6d4\uc5d0 \ub514\uc9c0\ud138 \uc2f1\uae00\uc744 \ub0b4\uace0 \uc7a0\uc2dc \ud65c\ub3d9\ud558\uc600\ub2e4. 2009\ub144\uc5d0 \ub4e4\uc5b4\uc11c\ub294 \ubb34\ud55c\ub3c4\uc804 \ud504\ub85c\uadf8\ub7a8\uc758 \uc77c\ud658\uc73c\ub85c \ud4e8\uccd0 \ub77c\uc774\uac70 \ud65c\ub3d9 \ubc0f Drunken Tiger 8\uc9d1, Supreme Team 1\uc9d1 \ud53c\uccd0\ub9c1 \ucc38\uc5ec \ub4f1\uc758 \ud65c\ub3d9\uc744 \ud558\uc600\uace0, \uadf8\uc640 \ud568\uaed8 \uc0c8 \uc568\ubc94 \uc791\uc5c5\uc5d0 \ub4e4\uc5b4\uac14\ub2e4. \uc774 \uc568\ubc94\uc740 2010\ub144 8\uc6d4\uc5d0 \ub098\uc62c \uac83\uc774 \uc608\uace0\ub418\uc5c8\uc73c\ub098 \ub098\uc624\uc9c0\ub294 \ubabb\ud558\uc600\ub2e4.", "sentence_answer": "2006\ub144 \uadf8\ub140\ub294 \ud0c0\uc774\uac70 JK(Tiger JK)\uc640 \ud568\uaed8 \uc815\uae00 \uc5d4\ud130\ud14c\uc778\uba3c\ud2b8 (Jungle Ent.)\uc758 \ucc3d\ub9bd \uba64\ubc84\uac00 \ub418\uc5c8\uc73c\uba70 4\ub144 \ub9cc\uc5d0 \uc138 \ubc88\uc9f8 \uc568\ubc94 \"Yoonmirae\"\ub85c \ucef4\ubc31\ud558\uc5ec \uc88b\uc740 \ubc18\uc751\uc744 \uc5bb\uc5c8\ub2e4.", "paragraph_id": "5834994-1-0", "paragraph_question": "question: \uc724\ubbf8\ub798\uac00 \ud0c0\uc774\uac70 JK\uc640 \ud568\uaed8 \ucc3d\ub9bd\ud55c \uc5d4\ud130\ud14c\uc778\uba3c\ud2b8\ub294 \ubb34\uc5c7\uc778\uac00?, context: 2006\ub144 \uadf8\ub140\ub294 \ud0c0\uc774\uac70 JK(Tiger JK)\uc640 \ud568\uaed8 \uc815\uae00 \uc5d4\ud130\ud14c\uc778\uba3c\ud2b8(Jungle Ent.)\uc758 \ucc3d\ub9bd \uba64\ubc84\uac00 \ub418\uc5c8\uc73c\uba70 4\ub144 \ub9cc\uc5d0 \uc138 \ubc88\uc9f8 \uc568\ubc94 \"Yoonmirae\"\ub85c \ucef4\ubc31\ud558\uc5ec \uc88b\uc740 \ubc18\uc751\uc744 \uc5bb\uc5c8\ub2e4. \uc774\ud6c4 Tiger JK\uc640 \uacb0\ud63c, 2008\ub144 3\uc6d4 \uc544\ub4e4\uc744 \ub0b3\uc558\uc73c\uba70, \ud55c\ub3d9\uc548 \uc0b0\ud6c4\uc870\ub9ac\ub85c \ud734\uc2dd\uc744 \ucde8\ud558\ub2e4 2009\ub144 1\uc6d4\uc5d0 \ub514\uc9c0\ud138 \uc2f1\uae00\uc744 \ub0b4\uace0 \uc7a0\uc2dc \ud65c\ub3d9\ud558\uc600\ub2e4. 2009\ub144\uc5d0 \ub4e4\uc5b4\uc11c\ub294 \ubb34\ud55c\ub3c4\uc804 \ud504\ub85c\uadf8\ub7a8\uc758 \uc77c\ud658\uc73c\ub85c \ud4e8\uccd0 \ub77c\uc774\uac70 \ud65c\ub3d9 \ubc0f Drunken Tiger 8\uc9d1, Supreme Team 1\uc9d1 \ud53c\uccd0\ub9c1 \ucc38\uc5ec \ub4f1\uc758 \ud65c\ub3d9\uc744 \ud558\uc600\uace0, \uadf8\uc640 \ud568\uaed8 \uc0c8 \uc568\ubc94 \uc791\uc5c5\uc5d0 \ub4e4\uc5b4\uac14\ub2e4. \uc774 \uc568\ubc94\uc740 2010\ub144 8\uc6d4\uc5d0 \ub098\uc62c \uac83\uc774 \uc608\uace0\ub418\uc5c8\uc73c\ub098 \ub098\uc624\uc9c0\ub294 \ubabb\ud558\uc600\ub2e4."} {"question": "\ubb38\uc7ac\uc778 \uc815\ubd80\uc758 \uacbd\uc81c\uc815\ucc45\uc740?", "paragraph": "\ubb38\uc7ac\uc778 \uc815\ubd80\uc758 \uacbd\uc81c\uc815\ucc45\uc740 \uc18c\uc704 '\uc81c\uc774\ub178\ubbf9\uc2a4'(Jaein + Economics)\uc774\ub2e4. \ubbfc\uac04\ubcf4\ub2e4 \uc815\ubd80\uc758 \uc5ed\ud560\uc744 \uac15\uc870\ud558\uba70 \uc815\ubd80 \uc8fc\ub3c4\ub85c \ub098\ub78f\ub3c8\uc744 \ud480\uc5b4 \uc77c\uc790\ub9ac\ub97c \ub9cc\ub4e4\uace0 \uac00\uacc4\uc18c\ub4dd\uc744 \ubd88\ub824\uc8fc\ub294 \ubc29\uc2dd\uc73c\ub85c \uacbd\uc81c\uc131\uc7a5\uc744 \uc774\ub8e8\uaca0\ub2e4\ub294 \uc804\ub7b5\uc778\ub370 \uae30\uc5c5\uc5d0 \ub300\ud55c \uaddc\uc81c\ub97c \uc644\ud654\ud558\uace0 \uc138\uae08\uc744 \uc778\ud558\ud574 \uacbd\uc81c\uc131\uc7a5\uc744 \uaf80\ud588\ub358, '\uae30\uc5c5\ud558\uae30 \uc88b\uc740 \ub098\ub77c'\ub85c \ub300\ubcc0\ub418\ub294 \uc774\uc804\uc758 \ubcf4\uc218 \uc815\uad8c\uacfc\ub294 \uada4\ub97c \ub2ec\ub9ac\ud55c\ub2e4. \uc81c\uc774\ub178\ubbf9\uc2a4\ub97c \uc124\uacc4\ud55c \uae40\uad11\ub450 \uc11c\uac15\ub300\ud559\uad50 \uc11d\uc88c\uad50\uc218\ub294 \"\uc0ac\ub78c\uc5d0 \ub300\ud55c \ud22c\uc790\uac00 \ud575\uc2ec\"\uc774\ub77c\uba70 \uc0ac\ub78c \uc911\uc2ec \uacbd\uc81c \uc131\uc7a5\uc744 \ubaa9\ud45c\ub85c \ub0b4\uc138\uc6b0\uba70 \uc77c\uc790\ub9ac \ucc3d\ucd9c\uc744 \uc911\uc694\uc2dc\ud588\ub294\ub370 \ucde8\uc784 \uc9c1\ud6c4 \ub300\ud1b5\ub839 \uc5c5\ubb34\uc9c0\uc2dc 1\ud638\ub85c \ub098\uc628 \uc77c\uc790\ub9ac\uc704\uc6d0\ud68c\ub97c \uad6c\uc131\ud55c \uac83\ub3c4 \uc774 \uac19\uc740 \ub9e5\ub77d\uc5d0\uc11c \uc774\ub8e8\uc5b4\uc9c4 \uac83\uc774\ub2e4. \uc120\uac70 \uae30\uac04\uc5d0\ub3c4 \uc2a4\uc2a4\ub85c\ub97c '\uc77c\uc790\ub9ac \ub300\ud1b5\ub839'\uc774\ub77c\uace0 \ubd88\ub800\ub358 \ubb38\uc7ac\uc778\uc740 \ucde8\uc784 \uc120\uc11c\uc5d0\uc11c\ub3c4 \"\ubb34\uc5c7\ubcf4\ub2e4 \uba3c\uc800 \uc77c\uc790\ub9ac\ub97c \ucc59\uae30\uaca0\ub2e4\"\uba70, 21\uc870 \uc6d0 \uac00\ub7c9\uc744 \ud480\uc5b4\uc11c \uacf5\uacf5\ubd80\ubb38 \uc77c\uc790\ub9ac 81\ub9cc \uac1c\ub97c \ucc3d\ucd9c\ud560 \uac83\uc744 \ubc1d\ud614\ub2e4. \uc774\ub7ec\ud55c \uc815\ucc45\uc740 \uc77c\uc790\ub9ac \ucc3d\ucd9c\ub85c \uac00\uacc4 \uc18c\ub4dd\uc774 \ub298\uc5b4\ub098\uba74 \uc18c\ube44\uac00 \uc99d\uac00\ud574 \uae30\uc5c5\uc758 \uc0dd\uc0b0\uacfc \ud22c\uc790\uac00 \ub369\ub2ec\uc544 \ub298\uc5b4\ub0a0 \uc218 \uc788\ub2e4\ub294 \uc774\ub978\ubc14 \ubd84\uc218 \ud6a8\uacfc\uc5d0 \uc774\ub860\uc801 \ubfcc\ub9ac\ub97c \ub450\uace0 \uc788\ub2e4. \uc774\ub97c \uc704\ud574 \uc7ac\uc815 \uc9c0\ucd9c\ub3c4 \ub9e4\ub144 7%\uc529 \ub298\ub9b4 \uac83\uc744 \uacf5\uc5b8\ud588\ub294\ub370, \uae40\uad11\ub450\ub294 \"\uc591\uadf9\ud654\ub85c \uc778\ud55c \uac08\ub4f1\uc758 \ubfcc\ub9ac\ub3c4 \uacb0\uad6d\uc740 \uc18c\ub4dd\uacfc \ubd80\ub97c \ucc3d\ucd9c\ud560 \uc218 \uc788\ub294 \uc790\uc9c8\uc744 \uae30\ub974\ub294 \uae30\ud68c \uc790\uccb4\uac00 \uacf5\uc815\ud558\uc9c0 \uc54a\ub2e4\ub294 \ub370 \uc788\ub2e4\"\uba74\uc11c \uae30\uc874\uc758 \ub099\uc218 \ud6a8\uacfc\ub97c \ubd80\uc815\ud558\uc600\ub2e4. \ub610 \ud558\ub098 \uc81c\uc774\ub178\ubbf9\uc2a4\uc758 \uc8fc\uc694 \ub0b4\uc6a9\uc73c\ub85c '\uacf5\uc815\ud55c \uc784\uae08'\uc744 \ub0b4\uc138\uc6e0\ub294\ub370 \uc911\uc18c\uae30\uc5c5\uc740 \ub300\uae30\uc5c5\uc5d0 \ube44\ud574 61.4% \uc218\uc900, \ube44\uc815\uaddc\uc9c1\uc740 \uc815\uaddc\uc9c1\uc5d0 \ube44\ud574 53.5%\uc778 \uc784\uae08\uc218\uc900\uc744 80%\uae4c\uc9c0 \uaca9\ucc28\ub97c \uc904\uc774\uaca0\ub2e4\ub294 \uac83\uc774\ub2e4. \uc774\ub294 \uc9c0\ub09c 10\ub144 \uac04 \ub298\uc5b4\ub09c \uc77c\uc790\ub9ac\uc758 92%\ub97c \uc911\uc18c\uae30\uc5c5\uc774 \ub9cc\ub4e4\uc5c8\ub2e8 \uc810\uc744 \uac10\uc548\ud558\uba74 \uc911\uc18c\uae30\uc5c5\uc5d0 \uc9c8 \uc88b\uc740 \uc77c\uc790\ub9ac\ub97c \ud0a4\uc6cc\ub0b4\ub294 \uac83\uc774 \uc911\uc694\ud55c \uacfc\uc81c\uac00 \ub420 \uac83\uc73c\ub85c \ubcf4\uc778\ub2e4. \ucd5c\uc800\uc784\uae08 \uc5ed\uc2dc 3\ub144 \ub0b4\uc778 2020\ub144\uae4c\uc9c0 1\ub9cc \uc6d0\uae4c\uc9c0 \uc62c\ub824\uc11c \uc800\uc18c\ub4dd\uacc4\uce35\uc758 \uc18c\ub4dd\uc744 \ubcf4\uc804\ud560 \uacc4\ud68d\uc778\ub370 \uc774\ub97c \uc704\ud574\uc11c\ub294 \uc5f0\ud3c9\uade0 \uc57d 16%\uc529 \uc778\uc0c1\ud558\uac8c \ub41c\ub2e4. \uc774\uc5d0 \ub300\ud55c \ube44\ud310\ub3c4 \uc874\uc7ac\ud558\ub294\ub370, \uc6b0\uc120 \uc18c\uc704 \uc18c\ub4dd\uc8fc\ub3c4 \uc131\uc7a5\ub860\uc5d0 \ub300\ud55c \ubd84\uba85\ud55c \uccad\uc0ac\uc9c4\uacfc \uc815\ucc45\uc801 \uc2e0\ud638\uac00 \uc874\uc7ac\ud558\uc9c0 \uc54a\ub294\ub2e4\ub294 \uc810\uc774\ub2e4. \uc2e4\uc81c\ub85c \uc120\uac70 \uacfc\uc815\uc5d0\uc11c \uad6c\uccb4\uc801\uc778 \ub85c\ub4dc\ub9f5\uc744 \uc81c\uc2dc\ud558\uc9c0 \uc54a\uc740 \ucc44 \uacf5\uacf5\ubd80\ubb38 \uc77c\uc790\ub9ac \ucc3d\ucd9c\uc774\ub77c\ub294 \uad6c\ud638\ub9cc \ub098\ub3cc\uc558\ub2e4. \ub610\ud55c \uc774\ub860\uc801\uc73c\ub85c \"\uc815\ubd80\uac00 \uc7ac\uc815\uc744 \ud22c\uc785\ud574 \uacbd\uae30\ub97c \uc0b4\ub9ac\ub294 \ucf00\uc778\uc9c0\uc5b8 \uc815\ucc45\uc740 \uacbd\uae30\ub300\uc751 \uc815\ucc45\uc774\uc9c0, \uc131\uc7a5\uc815\ucc45\uc740 \uc544\ub2c8\ub2e4. \uc815\ubd80\uac00 \uc77c\uc790\ub9ac\ub97c \ud1b5\ud574 \uc131\uc7a5\uc815\ucc45\uc73c\ub85c \ubc1c\uc804\uc2dc\ud0a4\ub824\uba74 \uacf5\uacf5 \uc77c\uc790\ub9ac\ubcf4\ub2e4\ub294 \ubd80\uac00\uac00\uce58\ub97c \uc0dd\uc0b0\ud558\uace0 \uc131\uc7a5\uc744 \uc8fc\ub3c4\ud558\ub294 \uc77c\uc790\ub9ac\uac00 \ub9ce\uc774 \ub9cc\ub4e4\uc5b4\uc838\uc57c \ud55c\ub2e4\"\ub294 \uc598\uae30\uac00 \ub098\uc654\ub2e4. \uc989, \uc131\uc7a5 \ub3d9\ub825\uc744 \ud655\ucda9\u00b7\ubcf4\uc644\ud560 \ud544\uc694\uac00 \uc788\ub294\ub370 \uacbd\uc81c\ubbfc\uc8fc\ud654\ub97c \uc911\uc694\uc2dc\ud558\uba74\uc11c \ub300\uae30\uc5c5\uc744 \uac1c\ud601\uc758 \ub300\uc0c1\uc73c\ub85c\ub9cc \ub2e4\ub8e8\ub294 \uac83\uc740 \ubb38\uc81c\uac00 \uc788\ub2e4\ub294 \uac83\uc774\ub2e4. \uadfc\ub85c\uc2dc\uac04 \ub2e8\ucd94, \ucd5c\uc800\uc784\uae08 \uc778\uc0c1, \ube44\uc815\uaddc\uc9c1 \ucc28\ubcc4 \uae08\uc9c0\uc640 \uac19\uc740 \uc815\ucc45\ub4e4\ub3c4 \ub178\ub3d9 \uc2dc\uc7a5\uc744 \uc624\ud788\ub824 \ub354 \ub531\ub531\ud558\uac8c \ub9cc\ub4e4 \uc6b0\ub824\ub3c4 \uc788\ub2e4. \ubb34\uc5c7\ubcf4\ub2e4 \uc911\uc694\ud55c \ubb38\uc81c\ub294 \uc7ac\uc815\uc778\ub370 \ub098\ub78f\ube5a\uc744 \ub9ce\uc774 \ub298\ub9ac\uc9c0 \uc54a\uc73c\uba74\uc11c \uc138\uc785\uc744 \ub298\ub9ac\ub294 \uac83\uc740 \uc26c\uc6b4 \uc77c\uc774 \uc544\ub2c8\ub2e4. \uc815\ubd80\ub294 \ubc29\uc0b0\ube44\ub9ac\u00b7\ud574\uc678\uc790\uc6d0\uac1c\ubc1c \ub4f1 \uad8c\ub825\ud615 \ube44\ub9ac \uc608\uc0b0\uc744 \uc0ad\uac10\ud558\uace0 \uc18c\ube44\uc131\u00b7\uc120\uc2ec\uc131\u00b7\uc911\ubcf5\uc131 \uc608\uc0b0\ub3c4 \uc904\uc774\ub294 \ub4f1 \uc138\ucd9c \uad6c\uc870\uc870\uc815\uc744 \ud1b5\ud55c 5\ub144 \uac04 112\uc870 \uc6d0\uac00\ub7c9\uc744 \uc544\ub084 \uc218 \uc788\ub2e4\ub294 \uc785\uc7a5\uc774\uc9c0\ub9cc \ud55c\uacc4\uac00 \uc788\uae30 \ub54c\ubb38\uc5d0 \uae30\uc5c5\uc774\ub098 \uace0\uc18c\ub4dd\uc790\ub85c\ubd80\ud130 \uc138\uae08\uc744 \ub354 \uac70\ub46c\ub4e4\uc774\ub294 \uc138\uc81c\uac1c\ud601\uc5d0 \ubc29\uc810\uc774 \ucc0d\ud790 \uac83\uc73c\ub85c \ubcf4\uc778\ub2e4.", "answer": "\uc81c\uc774\ub178\ubbf9\uc2a4", "sentence": "\ubb38\uc7ac\uc778 \uc815\ubd80\uc758 \uacbd\uc81c\uc815\ucc45\uc740 \uc18c\uc704 ' \uc81c\uc774\ub178\ubbf9\uc2a4 '(Jaein + Economics)\uc774\ub2e4.", "paragraph_sentence": " \ubb38\uc7ac\uc778 \uc815\ubd80\uc758 \uacbd\uc81c\uc815\ucc45\uc740 \uc18c\uc704 ' \uc81c\uc774\ub178\ubbf9\uc2a4 '(Jaein + Economics)\uc774\ub2e4. \ubbfc\uac04\ubcf4\ub2e4 \uc815\ubd80\uc758 \uc5ed\ud560\uc744 \uac15\uc870\ud558\uba70 \uc815\ubd80 \uc8fc\ub3c4\ub85c \ub098\ub78f\ub3c8\uc744 \ud480\uc5b4 \uc77c\uc790\ub9ac\ub97c \ub9cc\ub4e4\uace0 \uac00\uacc4\uc18c\ub4dd\uc744 \ubd88\ub824\uc8fc\ub294 \ubc29\uc2dd\uc73c\ub85c \uacbd\uc81c\uc131\uc7a5\uc744 \uc774\ub8e8\uaca0\ub2e4\ub294 \uc804\ub7b5\uc778\ub370 \uae30\uc5c5\uc5d0 \ub300\ud55c \uaddc\uc81c\ub97c \uc644\ud654\ud558\uace0 \uc138\uae08\uc744 \uc778\ud558\ud574 \uacbd\uc81c\uc131\uc7a5\uc744 \uaf80\ud588\ub358, '\uae30\uc5c5\ud558\uae30 \uc88b\uc740 \ub098\ub77c'\ub85c \ub300\ubcc0\ub418\ub294 \uc774\uc804\uc758 \ubcf4\uc218 \uc815\uad8c\uacfc\ub294 \uada4\ub97c \ub2ec\ub9ac\ud55c\ub2e4. \uc81c\uc774\ub178\ubbf9\uc2a4\ub97c \uc124\uacc4\ud55c \uae40\uad11\ub450 \uc11c\uac15\ub300\ud559\uad50 \uc11d\uc88c\uad50\uc218\ub294 \"\uc0ac\ub78c\uc5d0 \ub300\ud55c \ud22c\uc790\uac00 \ud575\uc2ec\"\uc774\ub77c\uba70 \uc0ac\ub78c \uc911\uc2ec \uacbd\uc81c \uc131\uc7a5\uc744 \ubaa9\ud45c\ub85c \ub0b4\uc138\uc6b0\uba70 \uc77c\uc790\ub9ac \ucc3d\ucd9c\uc744 \uc911\uc694\uc2dc\ud588\ub294\ub370 \ucde8\uc784 \uc9c1\ud6c4 \ub300\ud1b5\ub839 \uc5c5\ubb34\uc9c0\uc2dc 1\ud638\ub85c \ub098\uc628 \uc77c\uc790\ub9ac\uc704\uc6d0\ud68c\ub97c \uad6c\uc131\ud55c \uac83\ub3c4 \uc774 \uac19\uc740 \ub9e5\ub77d\uc5d0\uc11c \uc774\ub8e8\uc5b4\uc9c4 \uac83\uc774\ub2e4. \uc120\uac70 \uae30\uac04\uc5d0\ub3c4 \uc2a4\uc2a4\ub85c\ub97c '\uc77c\uc790\ub9ac \ub300\ud1b5\ub839'\uc774\ub77c\uace0 \ubd88\ub800\ub358 \ubb38\uc7ac\uc778\uc740 \ucde8\uc784 \uc120\uc11c\uc5d0\uc11c\ub3c4 \"\ubb34\uc5c7\ubcf4\ub2e4 \uba3c\uc800 \uc77c\uc790\ub9ac\ub97c \ucc59\uae30\uaca0\ub2e4\"\uba70, 21\uc870 \uc6d0 \uac00\ub7c9\uc744 \ud480\uc5b4\uc11c \uacf5\uacf5\ubd80\ubb38 \uc77c\uc790\ub9ac 81\ub9cc \uac1c\ub97c \ucc3d\ucd9c\ud560 \uac83\uc744 \ubc1d\ud614\ub2e4. \uc774\ub7ec\ud55c \uc815\ucc45\uc740 \uc77c\uc790\ub9ac \ucc3d\ucd9c\ub85c \uac00\uacc4 \uc18c\ub4dd\uc774 \ub298\uc5b4\ub098\uba74 \uc18c\ube44\uac00 \uc99d\uac00\ud574 \uae30\uc5c5\uc758 \uc0dd\uc0b0\uacfc \ud22c\uc790\uac00 \ub369\ub2ec\uc544 \ub298\uc5b4\ub0a0 \uc218 \uc788\ub2e4\ub294 \uc774\ub978\ubc14 \ubd84\uc218 \ud6a8\uacfc\uc5d0 \uc774\ub860\uc801 \ubfcc\ub9ac\ub97c \ub450\uace0 \uc788\ub2e4. \uc774\ub97c \uc704\ud574 \uc7ac\uc815 \uc9c0\ucd9c\ub3c4 \ub9e4\ub144 7%\uc529 \ub298\ub9b4 \uac83\uc744 \uacf5\uc5b8\ud588\ub294\ub370, \uae40\uad11\ub450\ub294 \"\uc591\uadf9\ud654\ub85c \uc778\ud55c \uac08\ub4f1\uc758 \ubfcc\ub9ac\ub3c4 \uacb0\uad6d\uc740 \uc18c\ub4dd\uacfc \ubd80\ub97c \ucc3d\ucd9c\ud560 \uc218 \uc788\ub294 \uc790\uc9c8\uc744 \uae30\ub974\ub294 \uae30\ud68c \uc790\uccb4\uac00 \uacf5\uc815\ud558\uc9c0 \uc54a\ub2e4\ub294 \ub370 \uc788\ub2e4\"\uba74\uc11c \uae30\uc874\uc758 \ub099\uc218 \ud6a8\uacfc\ub97c \ubd80\uc815\ud558\uc600\ub2e4. \ub610 \ud558\ub098 \uc81c\uc774\ub178\ubbf9\uc2a4\uc758 \uc8fc\uc694 \ub0b4\uc6a9\uc73c\ub85c '\uacf5\uc815\ud55c \uc784\uae08'\uc744 \ub0b4\uc138\uc6e0\ub294\ub370 \uc911\uc18c\uae30\uc5c5\uc740 \ub300\uae30\uc5c5\uc5d0 \ube44\ud574 61.4% \uc218\uc900, \ube44\uc815\uaddc\uc9c1\uc740 \uc815\uaddc\uc9c1\uc5d0 \ube44\ud574 53.5%\uc778 \uc784\uae08\uc218\uc900\uc744 80%\uae4c\uc9c0 \uaca9\ucc28\ub97c \uc904\uc774\uaca0\ub2e4\ub294 \uac83\uc774\ub2e4. \uc774\ub294 \uc9c0\ub09c 10\ub144 \uac04 \ub298\uc5b4\ub09c \uc77c\uc790\ub9ac\uc758 92%\ub97c \uc911\uc18c\uae30\uc5c5\uc774 \ub9cc\ub4e4\uc5c8\ub2e8 \uc810\uc744 \uac10\uc548\ud558\uba74 \uc911\uc18c\uae30\uc5c5\uc5d0 \uc9c8 \uc88b\uc740 \uc77c\uc790\ub9ac\ub97c \ud0a4\uc6cc\ub0b4\ub294 \uac83\uc774 \uc911\uc694\ud55c \uacfc\uc81c\uac00 \ub420 \uac83\uc73c\ub85c \ubcf4\uc778\ub2e4. \ucd5c\uc800\uc784\uae08 \uc5ed\uc2dc 3\ub144 \ub0b4\uc778 2020\ub144\uae4c\uc9c0 1\ub9cc \uc6d0\uae4c\uc9c0 \uc62c\ub824\uc11c \uc800\uc18c\ub4dd\uacc4\uce35\uc758 \uc18c\ub4dd\uc744 \ubcf4\uc804\ud560 \uacc4\ud68d\uc778\ub370 \uc774\ub97c \uc704\ud574\uc11c\ub294 \uc5f0\ud3c9\uade0 \uc57d 16%\uc529 \uc778\uc0c1\ud558\uac8c \ub41c\ub2e4. \uc774\uc5d0 \ub300\ud55c \ube44\ud310\ub3c4 \uc874\uc7ac\ud558\ub294\ub370, \uc6b0\uc120 \uc18c\uc704 \uc18c\ub4dd\uc8fc\ub3c4 \uc131\uc7a5\ub860\uc5d0 \ub300\ud55c \ubd84\uba85\ud55c \uccad\uc0ac\uc9c4\uacfc \uc815\ucc45\uc801 \uc2e0\ud638\uac00 \uc874\uc7ac\ud558\uc9c0 \uc54a\ub294\ub2e4\ub294 \uc810\uc774\ub2e4. \uc2e4\uc81c\ub85c \uc120\uac70 \uacfc\uc815\uc5d0\uc11c \uad6c\uccb4\uc801\uc778 \ub85c\ub4dc\ub9f5\uc744 \uc81c\uc2dc\ud558\uc9c0 \uc54a\uc740 \ucc44 \uacf5\uacf5\ubd80\ubb38 \uc77c\uc790\ub9ac \ucc3d\ucd9c\uc774\ub77c\ub294 \uad6c\ud638\ub9cc \ub098\ub3cc\uc558\ub2e4. \ub610\ud55c \uc774\ub860\uc801\uc73c\ub85c \"\uc815\ubd80\uac00 \uc7ac\uc815\uc744 \ud22c\uc785\ud574 \uacbd\uae30\ub97c \uc0b4\ub9ac\ub294 \ucf00\uc778\uc9c0\uc5b8 \uc815\ucc45\uc740 \uacbd\uae30\ub300\uc751 \uc815\ucc45\uc774\uc9c0, \uc131\uc7a5\uc815\ucc45\uc740 \uc544\ub2c8\ub2e4. \uc815\ubd80\uac00 \uc77c\uc790\ub9ac\ub97c \ud1b5\ud574 \uc131\uc7a5\uc815\ucc45\uc73c\ub85c \ubc1c\uc804\uc2dc\ud0a4\ub824\uba74 \uacf5\uacf5 \uc77c\uc790\ub9ac\ubcf4\ub2e4\ub294 \ubd80\uac00\uac00\uce58\ub97c \uc0dd\uc0b0\ud558\uace0 \uc131\uc7a5\uc744 \uc8fc\ub3c4\ud558\ub294 \uc77c\uc790\ub9ac\uac00 \ub9ce\uc774 \ub9cc\ub4e4\uc5b4\uc838\uc57c \ud55c\ub2e4\"\ub294 \uc598\uae30\uac00 \ub098\uc654\ub2e4. \uc989, \uc131\uc7a5 \ub3d9\ub825\uc744 \ud655\ucda9\u00b7\ubcf4\uc644\ud560 \ud544\uc694\uac00 \uc788\ub294\ub370 \uacbd\uc81c\ubbfc\uc8fc\ud654\ub97c \uc911\uc694\uc2dc\ud558\uba74\uc11c \ub300\uae30\uc5c5\uc744 \uac1c\ud601\uc758 \ub300\uc0c1\uc73c\ub85c\ub9cc \ub2e4\ub8e8\ub294 \uac83\uc740 \ubb38\uc81c\uac00 \uc788\ub2e4\ub294 \uac83\uc774\ub2e4. \uadfc\ub85c\uc2dc\uac04 \ub2e8\ucd94, \ucd5c\uc800\uc784\uae08 \uc778\uc0c1, \ube44\uc815\uaddc\uc9c1 \ucc28\ubcc4 \uae08\uc9c0\uc640 \uac19\uc740 \uc815\ucc45\ub4e4\ub3c4 \ub178\ub3d9 \uc2dc\uc7a5\uc744 \uc624\ud788\ub824 \ub354 \ub531\ub531\ud558\uac8c \ub9cc\ub4e4 \uc6b0\ub824\ub3c4 \uc788\ub2e4. \ubb34\uc5c7\ubcf4\ub2e4 \uc911\uc694\ud55c \ubb38\uc81c\ub294 \uc7ac\uc815\uc778\ub370 \ub098\ub78f\ube5a\uc744 \ub9ce\uc774 \ub298\ub9ac\uc9c0 \uc54a\uc73c\uba74\uc11c \uc138\uc785\uc744 \ub298\ub9ac\ub294 \uac83\uc740 \uc26c\uc6b4 \uc77c\uc774 \uc544\ub2c8\ub2e4. \uc815\ubd80\ub294 \ubc29\uc0b0\ube44\ub9ac\u00b7\ud574\uc678\uc790\uc6d0\uac1c\ubc1c \ub4f1 \uad8c\ub825\ud615 \ube44\ub9ac \uc608\uc0b0\uc744 \uc0ad\uac10\ud558\uace0 \uc18c\ube44\uc131\u00b7\uc120\uc2ec\uc131\u00b7\uc911\ubcf5\uc131 \uc608\uc0b0\ub3c4 \uc904\uc774\ub294 \ub4f1 \uc138\ucd9c \uad6c\uc870\uc870\uc815\uc744 \ud1b5\ud55c 5\ub144 \uac04 112\uc870 \uc6d0\uac00\ub7c9\uc744 \uc544\ub084 \uc218 \uc788\ub2e4\ub294 \uc785\uc7a5\uc774\uc9c0\ub9cc \ud55c\uacc4\uac00 \uc788\uae30 \ub54c\ubb38\uc5d0 \uae30\uc5c5\uc774\ub098 \uace0\uc18c\ub4dd\uc790\ub85c\ubd80\ud130 \uc138\uae08\uc744 \ub354 \uac70\ub46c\ub4e4\uc774\ub294 \uc138\uc81c\uac1c\ud601\uc5d0 \ubc29\uc810\uc774 \ucc0d\ud790 \uac83\uc73c\ub85c \ubcf4\uc778\ub2e4.", "paragraph_answer": "\ubb38\uc7ac\uc778 \uc815\ubd80\uc758 \uacbd\uc81c\uc815\ucc45\uc740 \uc18c\uc704 ' \uc81c\uc774\ub178\ubbf9\uc2a4 '(Jaein + Economics)\uc774\ub2e4. \ubbfc\uac04\ubcf4\ub2e4 \uc815\ubd80\uc758 \uc5ed\ud560\uc744 \uac15\uc870\ud558\uba70 \uc815\ubd80 \uc8fc\ub3c4\ub85c \ub098\ub78f\ub3c8\uc744 \ud480\uc5b4 \uc77c\uc790\ub9ac\ub97c \ub9cc\ub4e4\uace0 \uac00\uacc4\uc18c\ub4dd\uc744 \ubd88\ub824\uc8fc\ub294 \ubc29\uc2dd\uc73c\ub85c \uacbd\uc81c\uc131\uc7a5\uc744 \uc774\ub8e8\uaca0\ub2e4\ub294 \uc804\ub7b5\uc778\ub370 \uae30\uc5c5\uc5d0 \ub300\ud55c \uaddc\uc81c\ub97c \uc644\ud654\ud558\uace0 \uc138\uae08\uc744 \uc778\ud558\ud574 \uacbd\uc81c\uc131\uc7a5\uc744 \uaf80\ud588\ub358, '\uae30\uc5c5\ud558\uae30 \uc88b\uc740 \ub098\ub77c'\ub85c \ub300\ubcc0\ub418\ub294 \uc774\uc804\uc758 \ubcf4\uc218 \uc815\uad8c\uacfc\ub294 \uada4\ub97c \ub2ec\ub9ac\ud55c\ub2e4. \uc81c\uc774\ub178\ubbf9\uc2a4\ub97c \uc124\uacc4\ud55c \uae40\uad11\ub450 \uc11c\uac15\ub300\ud559\uad50 \uc11d\uc88c\uad50\uc218\ub294 \"\uc0ac\ub78c\uc5d0 \ub300\ud55c \ud22c\uc790\uac00 \ud575\uc2ec\"\uc774\ub77c\uba70 \uc0ac\ub78c \uc911\uc2ec \uacbd\uc81c \uc131\uc7a5\uc744 \ubaa9\ud45c\ub85c \ub0b4\uc138\uc6b0\uba70 \uc77c\uc790\ub9ac \ucc3d\ucd9c\uc744 \uc911\uc694\uc2dc\ud588\ub294\ub370 \ucde8\uc784 \uc9c1\ud6c4 \ub300\ud1b5\ub839 \uc5c5\ubb34\uc9c0\uc2dc 1\ud638\ub85c \ub098\uc628 \uc77c\uc790\ub9ac\uc704\uc6d0\ud68c\ub97c \uad6c\uc131\ud55c \uac83\ub3c4 \uc774 \uac19\uc740 \ub9e5\ub77d\uc5d0\uc11c \uc774\ub8e8\uc5b4\uc9c4 \uac83\uc774\ub2e4. \uc120\uac70 \uae30\uac04\uc5d0\ub3c4 \uc2a4\uc2a4\ub85c\ub97c '\uc77c\uc790\ub9ac \ub300\ud1b5\ub839'\uc774\ub77c\uace0 \ubd88\ub800\ub358 \ubb38\uc7ac\uc778\uc740 \ucde8\uc784 \uc120\uc11c\uc5d0\uc11c\ub3c4 \"\ubb34\uc5c7\ubcf4\ub2e4 \uba3c\uc800 \uc77c\uc790\ub9ac\ub97c \ucc59\uae30\uaca0\ub2e4\"\uba70, 21\uc870 \uc6d0 \uac00\ub7c9\uc744 \ud480\uc5b4\uc11c \uacf5\uacf5\ubd80\ubb38 \uc77c\uc790\ub9ac 81\ub9cc \uac1c\ub97c \ucc3d\ucd9c\ud560 \uac83\uc744 \ubc1d\ud614\ub2e4. \uc774\ub7ec\ud55c \uc815\ucc45\uc740 \uc77c\uc790\ub9ac \ucc3d\ucd9c\ub85c \uac00\uacc4 \uc18c\ub4dd\uc774 \ub298\uc5b4\ub098\uba74 \uc18c\ube44\uac00 \uc99d\uac00\ud574 \uae30\uc5c5\uc758 \uc0dd\uc0b0\uacfc \ud22c\uc790\uac00 \ub369\ub2ec\uc544 \ub298\uc5b4\ub0a0 \uc218 \uc788\ub2e4\ub294 \uc774\ub978\ubc14 \ubd84\uc218 \ud6a8\uacfc\uc5d0 \uc774\ub860\uc801 \ubfcc\ub9ac\ub97c \ub450\uace0 \uc788\ub2e4. \uc774\ub97c \uc704\ud574 \uc7ac\uc815 \uc9c0\ucd9c\ub3c4 \ub9e4\ub144 7%\uc529 \ub298\ub9b4 \uac83\uc744 \uacf5\uc5b8\ud588\ub294\ub370, \uae40\uad11\ub450\ub294 \"\uc591\uadf9\ud654\ub85c \uc778\ud55c \uac08\ub4f1\uc758 \ubfcc\ub9ac\ub3c4 \uacb0\uad6d\uc740 \uc18c\ub4dd\uacfc \ubd80\ub97c \ucc3d\ucd9c\ud560 \uc218 \uc788\ub294 \uc790\uc9c8\uc744 \uae30\ub974\ub294 \uae30\ud68c \uc790\uccb4\uac00 \uacf5\uc815\ud558\uc9c0 \uc54a\ub2e4\ub294 \ub370 \uc788\ub2e4\"\uba74\uc11c \uae30\uc874\uc758 \ub099\uc218 \ud6a8\uacfc\ub97c \ubd80\uc815\ud558\uc600\ub2e4. \ub610 \ud558\ub098 \uc81c\uc774\ub178\ubbf9\uc2a4\uc758 \uc8fc\uc694 \ub0b4\uc6a9\uc73c\ub85c '\uacf5\uc815\ud55c \uc784\uae08'\uc744 \ub0b4\uc138\uc6e0\ub294\ub370 \uc911\uc18c\uae30\uc5c5\uc740 \ub300\uae30\uc5c5\uc5d0 \ube44\ud574 61.4% \uc218\uc900, \ube44\uc815\uaddc\uc9c1\uc740 \uc815\uaddc\uc9c1\uc5d0 \ube44\ud574 53.5%\uc778 \uc784\uae08\uc218\uc900\uc744 80%\uae4c\uc9c0 \uaca9\ucc28\ub97c \uc904\uc774\uaca0\ub2e4\ub294 \uac83\uc774\ub2e4. \uc774\ub294 \uc9c0\ub09c 10\ub144 \uac04 \ub298\uc5b4\ub09c \uc77c\uc790\ub9ac\uc758 92%\ub97c \uc911\uc18c\uae30\uc5c5\uc774 \ub9cc\ub4e4\uc5c8\ub2e8 \uc810\uc744 \uac10\uc548\ud558\uba74 \uc911\uc18c\uae30\uc5c5\uc5d0 \uc9c8 \uc88b\uc740 \uc77c\uc790\ub9ac\ub97c \ud0a4\uc6cc\ub0b4\ub294 \uac83\uc774 \uc911\uc694\ud55c \uacfc\uc81c\uac00 \ub420 \uac83\uc73c\ub85c \ubcf4\uc778\ub2e4. \ucd5c\uc800\uc784\uae08 \uc5ed\uc2dc 3\ub144 \ub0b4\uc778 2020\ub144\uae4c\uc9c0 1\ub9cc \uc6d0\uae4c\uc9c0 \uc62c\ub824\uc11c \uc800\uc18c\ub4dd\uacc4\uce35\uc758 \uc18c\ub4dd\uc744 \ubcf4\uc804\ud560 \uacc4\ud68d\uc778\ub370 \uc774\ub97c \uc704\ud574\uc11c\ub294 \uc5f0\ud3c9\uade0 \uc57d 16%\uc529 \uc778\uc0c1\ud558\uac8c \ub41c\ub2e4. \uc774\uc5d0 \ub300\ud55c \ube44\ud310\ub3c4 \uc874\uc7ac\ud558\ub294\ub370, \uc6b0\uc120 \uc18c\uc704 \uc18c\ub4dd\uc8fc\ub3c4 \uc131\uc7a5\ub860\uc5d0 \ub300\ud55c \ubd84\uba85\ud55c \uccad\uc0ac\uc9c4\uacfc \uc815\ucc45\uc801 \uc2e0\ud638\uac00 \uc874\uc7ac\ud558\uc9c0 \uc54a\ub294\ub2e4\ub294 \uc810\uc774\ub2e4. \uc2e4\uc81c\ub85c \uc120\uac70 \uacfc\uc815\uc5d0\uc11c \uad6c\uccb4\uc801\uc778 \ub85c\ub4dc\ub9f5\uc744 \uc81c\uc2dc\ud558\uc9c0 \uc54a\uc740 \ucc44 \uacf5\uacf5\ubd80\ubb38 \uc77c\uc790\ub9ac \ucc3d\ucd9c\uc774\ub77c\ub294 \uad6c\ud638\ub9cc \ub098\ub3cc\uc558\ub2e4. \ub610\ud55c \uc774\ub860\uc801\uc73c\ub85c \"\uc815\ubd80\uac00 \uc7ac\uc815\uc744 \ud22c\uc785\ud574 \uacbd\uae30\ub97c \uc0b4\ub9ac\ub294 \ucf00\uc778\uc9c0\uc5b8 \uc815\ucc45\uc740 \uacbd\uae30\ub300\uc751 \uc815\ucc45\uc774\uc9c0, \uc131\uc7a5\uc815\ucc45\uc740 \uc544\ub2c8\ub2e4. \uc815\ubd80\uac00 \uc77c\uc790\ub9ac\ub97c \ud1b5\ud574 \uc131\uc7a5\uc815\ucc45\uc73c\ub85c \ubc1c\uc804\uc2dc\ud0a4\ub824\uba74 \uacf5\uacf5 \uc77c\uc790\ub9ac\ubcf4\ub2e4\ub294 \ubd80\uac00\uac00\uce58\ub97c \uc0dd\uc0b0\ud558\uace0 \uc131\uc7a5\uc744 \uc8fc\ub3c4\ud558\ub294 \uc77c\uc790\ub9ac\uac00 \ub9ce\uc774 \ub9cc\ub4e4\uc5b4\uc838\uc57c \ud55c\ub2e4\"\ub294 \uc598\uae30\uac00 \ub098\uc654\ub2e4. \uc989, \uc131\uc7a5 \ub3d9\ub825\uc744 \ud655\ucda9\u00b7\ubcf4\uc644\ud560 \ud544\uc694\uac00 \uc788\ub294\ub370 \uacbd\uc81c\ubbfc\uc8fc\ud654\ub97c \uc911\uc694\uc2dc\ud558\uba74\uc11c \ub300\uae30\uc5c5\uc744 \uac1c\ud601\uc758 \ub300\uc0c1\uc73c\ub85c\ub9cc \ub2e4\ub8e8\ub294 \uac83\uc740 \ubb38\uc81c\uac00 \uc788\ub2e4\ub294 \uac83\uc774\ub2e4. \uadfc\ub85c\uc2dc\uac04 \ub2e8\ucd94, \ucd5c\uc800\uc784\uae08 \uc778\uc0c1, \ube44\uc815\uaddc\uc9c1 \ucc28\ubcc4 \uae08\uc9c0\uc640 \uac19\uc740 \uc815\ucc45\ub4e4\ub3c4 \ub178\ub3d9 \uc2dc\uc7a5\uc744 \uc624\ud788\ub824 \ub354 \ub531\ub531\ud558\uac8c \ub9cc\ub4e4 \uc6b0\ub824\ub3c4 \uc788\ub2e4. \ubb34\uc5c7\ubcf4\ub2e4 \uc911\uc694\ud55c \ubb38\uc81c\ub294 \uc7ac\uc815\uc778\ub370 \ub098\ub78f\ube5a\uc744 \ub9ce\uc774 \ub298\ub9ac\uc9c0 \uc54a\uc73c\uba74\uc11c \uc138\uc785\uc744 \ub298\ub9ac\ub294 \uac83\uc740 \uc26c\uc6b4 \uc77c\uc774 \uc544\ub2c8\ub2e4. \uc815\ubd80\ub294 \ubc29\uc0b0\ube44\ub9ac\u00b7\ud574\uc678\uc790\uc6d0\uac1c\ubc1c \ub4f1 \uad8c\ub825\ud615 \ube44\ub9ac \uc608\uc0b0\uc744 \uc0ad\uac10\ud558\uace0 \uc18c\ube44\uc131\u00b7\uc120\uc2ec\uc131\u00b7\uc911\ubcf5\uc131 \uc608\uc0b0\ub3c4 \uc904\uc774\ub294 \ub4f1 \uc138\ucd9c \uad6c\uc870\uc870\uc815\uc744 \ud1b5\ud55c 5\ub144 \uac04 112\uc870 \uc6d0\uac00\ub7c9\uc744 \uc544\ub084 \uc218 \uc788\ub2e4\ub294 \uc785\uc7a5\uc774\uc9c0\ub9cc \ud55c\uacc4\uac00 \uc788\uae30 \ub54c\ubb38\uc5d0 \uae30\uc5c5\uc774\ub098 \uace0\uc18c\ub4dd\uc790\ub85c\ubd80\ud130 \uc138\uae08\uc744 \ub354 \uac70\ub46c\ub4e4\uc774\ub294 \uc138\uc81c\uac1c\ud601\uc5d0 \ubc29\uc810\uc774 \ucc0d\ud790 \uac83\uc73c\ub85c \ubcf4\uc778\ub2e4.", "sentence_answer": "\ubb38\uc7ac\uc778 \uc815\ubd80\uc758 \uacbd\uc81c\uc815\ucc45\uc740 \uc18c\uc704 ' \uc81c\uc774\ub178\ubbf9\uc2a4 '(Jaein + Economics)\uc774\ub2e4.", "paragraph_id": "6580792-74-0", "paragraph_question": "question: \ubb38\uc7ac\uc778 \uc815\ubd80\uc758 \uacbd\uc81c\uc815\ucc45\uc740?, context: \ubb38\uc7ac\uc778 \uc815\ubd80\uc758 \uacbd\uc81c\uc815\ucc45\uc740 \uc18c\uc704 '\uc81c\uc774\ub178\ubbf9\uc2a4'(Jaein + Economics)\uc774\ub2e4. \ubbfc\uac04\ubcf4\ub2e4 \uc815\ubd80\uc758 \uc5ed\ud560\uc744 \uac15\uc870\ud558\uba70 \uc815\ubd80 \uc8fc\ub3c4\ub85c \ub098\ub78f\ub3c8\uc744 \ud480\uc5b4 \uc77c\uc790\ub9ac\ub97c \ub9cc\ub4e4\uace0 \uac00\uacc4\uc18c\ub4dd\uc744 \ubd88\ub824\uc8fc\ub294 \ubc29\uc2dd\uc73c\ub85c \uacbd\uc81c\uc131\uc7a5\uc744 \uc774\ub8e8\uaca0\ub2e4\ub294 \uc804\ub7b5\uc778\ub370 \uae30\uc5c5\uc5d0 \ub300\ud55c \uaddc\uc81c\ub97c \uc644\ud654\ud558\uace0 \uc138\uae08\uc744 \uc778\ud558\ud574 \uacbd\uc81c\uc131\uc7a5\uc744 \uaf80\ud588\ub358, '\uae30\uc5c5\ud558\uae30 \uc88b\uc740 \ub098\ub77c'\ub85c \ub300\ubcc0\ub418\ub294 \uc774\uc804\uc758 \ubcf4\uc218 \uc815\uad8c\uacfc\ub294 \uada4\ub97c \ub2ec\ub9ac\ud55c\ub2e4. \uc81c\uc774\ub178\ubbf9\uc2a4\ub97c \uc124\uacc4\ud55c \uae40\uad11\ub450 \uc11c\uac15\ub300\ud559\uad50 \uc11d\uc88c\uad50\uc218\ub294 \"\uc0ac\ub78c\uc5d0 \ub300\ud55c \ud22c\uc790\uac00 \ud575\uc2ec\"\uc774\ub77c\uba70 \uc0ac\ub78c \uc911\uc2ec \uacbd\uc81c \uc131\uc7a5\uc744 \ubaa9\ud45c\ub85c \ub0b4\uc138\uc6b0\uba70 \uc77c\uc790\ub9ac \ucc3d\ucd9c\uc744 \uc911\uc694\uc2dc\ud588\ub294\ub370 \ucde8\uc784 \uc9c1\ud6c4 \ub300\ud1b5\ub839 \uc5c5\ubb34\uc9c0\uc2dc 1\ud638\ub85c \ub098\uc628 \uc77c\uc790\ub9ac\uc704\uc6d0\ud68c\ub97c \uad6c\uc131\ud55c \uac83\ub3c4 \uc774 \uac19\uc740 \ub9e5\ub77d\uc5d0\uc11c \uc774\ub8e8\uc5b4\uc9c4 \uac83\uc774\ub2e4. \uc120\uac70 \uae30\uac04\uc5d0\ub3c4 \uc2a4\uc2a4\ub85c\ub97c '\uc77c\uc790\ub9ac \ub300\ud1b5\ub839'\uc774\ub77c\uace0 \ubd88\ub800\ub358 \ubb38\uc7ac\uc778\uc740 \ucde8\uc784 \uc120\uc11c\uc5d0\uc11c\ub3c4 \"\ubb34\uc5c7\ubcf4\ub2e4 \uba3c\uc800 \uc77c\uc790\ub9ac\ub97c \ucc59\uae30\uaca0\ub2e4\"\uba70, 21\uc870 \uc6d0 \uac00\ub7c9\uc744 \ud480\uc5b4\uc11c \uacf5\uacf5\ubd80\ubb38 \uc77c\uc790\ub9ac 81\ub9cc \uac1c\ub97c \ucc3d\ucd9c\ud560 \uac83\uc744 \ubc1d\ud614\ub2e4. \uc774\ub7ec\ud55c \uc815\ucc45\uc740 \uc77c\uc790\ub9ac \ucc3d\ucd9c\ub85c \uac00\uacc4 \uc18c\ub4dd\uc774 \ub298\uc5b4\ub098\uba74 \uc18c\ube44\uac00 \uc99d\uac00\ud574 \uae30\uc5c5\uc758 \uc0dd\uc0b0\uacfc \ud22c\uc790\uac00 \ub369\ub2ec\uc544 \ub298\uc5b4\ub0a0 \uc218 \uc788\ub2e4\ub294 \uc774\ub978\ubc14 \ubd84\uc218 \ud6a8\uacfc\uc5d0 \uc774\ub860\uc801 \ubfcc\ub9ac\ub97c \ub450\uace0 \uc788\ub2e4. \uc774\ub97c \uc704\ud574 \uc7ac\uc815 \uc9c0\ucd9c\ub3c4 \ub9e4\ub144 7%\uc529 \ub298\ub9b4 \uac83\uc744 \uacf5\uc5b8\ud588\ub294\ub370, \uae40\uad11\ub450\ub294 \"\uc591\uadf9\ud654\ub85c \uc778\ud55c \uac08\ub4f1\uc758 \ubfcc\ub9ac\ub3c4 \uacb0\uad6d\uc740 \uc18c\ub4dd\uacfc \ubd80\ub97c \ucc3d\ucd9c\ud560 \uc218 \uc788\ub294 \uc790\uc9c8\uc744 \uae30\ub974\ub294 \uae30\ud68c \uc790\uccb4\uac00 \uacf5\uc815\ud558\uc9c0 \uc54a\ub2e4\ub294 \ub370 \uc788\ub2e4\"\uba74\uc11c \uae30\uc874\uc758 \ub099\uc218 \ud6a8\uacfc\ub97c \ubd80\uc815\ud558\uc600\ub2e4. \ub610 \ud558\ub098 \uc81c\uc774\ub178\ubbf9\uc2a4\uc758 \uc8fc\uc694 \ub0b4\uc6a9\uc73c\ub85c '\uacf5\uc815\ud55c \uc784\uae08'\uc744 \ub0b4\uc138\uc6e0\ub294\ub370 \uc911\uc18c\uae30\uc5c5\uc740 \ub300\uae30\uc5c5\uc5d0 \ube44\ud574 61.4% \uc218\uc900, \ube44\uc815\uaddc\uc9c1\uc740 \uc815\uaddc\uc9c1\uc5d0 \ube44\ud574 53.5%\uc778 \uc784\uae08\uc218\uc900\uc744 80%\uae4c\uc9c0 \uaca9\ucc28\ub97c \uc904\uc774\uaca0\ub2e4\ub294 \uac83\uc774\ub2e4. \uc774\ub294 \uc9c0\ub09c 10\ub144 \uac04 \ub298\uc5b4\ub09c \uc77c\uc790\ub9ac\uc758 92%\ub97c \uc911\uc18c\uae30\uc5c5\uc774 \ub9cc\ub4e4\uc5c8\ub2e8 \uc810\uc744 \uac10\uc548\ud558\uba74 \uc911\uc18c\uae30\uc5c5\uc5d0 \uc9c8 \uc88b\uc740 \uc77c\uc790\ub9ac\ub97c \ud0a4\uc6cc\ub0b4\ub294 \uac83\uc774 \uc911\uc694\ud55c \uacfc\uc81c\uac00 \ub420 \uac83\uc73c\ub85c \ubcf4\uc778\ub2e4. \ucd5c\uc800\uc784\uae08 \uc5ed\uc2dc 3\ub144 \ub0b4\uc778 2020\ub144\uae4c\uc9c0 1\ub9cc \uc6d0\uae4c\uc9c0 \uc62c\ub824\uc11c \uc800\uc18c\ub4dd\uacc4\uce35\uc758 \uc18c\ub4dd\uc744 \ubcf4\uc804\ud560 \uacc4\ud68d\uc778\ub370 \uc774\ub97c \uc704\ud574\uc11c\ub294 \uc5f0\ud3c9\uade0 \uc57d 16%\uc529 \uc778\uc0c1\ud558\uac8c \ub41c\ub2e4. \uc774\uc5d0 \ub300\ud55c \ube44\ud310\ub3c4 \uc874\uc7ac\ud558\ub294\ub370, \uc6b0\uc120 \uc18c\uc704 \uc18c\ub4dd\uc8fc\ub3c4 \uc131\uc7a5\ub860\uc5d0 \ub300\ud55c \ubd84\uba85\ud55c \uccad\uc0ac\uc9c4\uacfc \uc815\ucc45\uc801 \uc2e0\ud638\uac00 \uc874\uc7ac\ud558\uc9c0 \uc54a\ub294\ub2e4\ub294 \uc810\uc774\ub2e4. \uc2e4\uc81c\ub85c \uc120\uac70 \uacfc\uc815\uc5d0\uc11c \uad6c\uccb4\uc801\uc778 \ub85c\ub4dc\ub9f5\uc744 \uc81c\uc2dc\ud558\uc9c0 \uc54a\uc740 \ucc44 \uacf5\uacf5\ubd80\ubb38 \uc77c\uc790\ub9ac \ucc3d\ucd9c\uc774\ub77c\ub294 \uad6c\ud638\ub9cc \ub098\ub3cc\uc558\ub2e4. \ub610\ud55c \uc774\ub860\uc801\uc73c\ub85c \"\uc815\ubd80\uac00 \uc7ac\uc815\uc744 \ud22c\uc785\ud574 \uacbd\uae30\ub97c \uc0b4\ub9ac\ub294 \ucf00\uc778\uc9c0\uc5b8 \uc815\ucc45\uc740 \uacbd\uae30\ub300\uc751 \uc815\ucc45\uc774\uc9c0, \uc131\uc7a5\uc815\ucc45\uc740 \uc544\ub2c8\ub2e4. \uc815\ubd80\uac00 \uc77c\uc790\ub9ac\ub97c \ud1b5\ud574 \uc131\uc7a5\uc815\ucc45\uc73c\ub85c \ubc1c\uc804\uc2dc\ud0a4\ub824\uba74 \uacf5\uacf5 \uc77c\uc790\ub9ac\ubcf4\ub2e4\ub294 \ubd80\uac00\uac00\uce58\ub97c \uc0dd\uc0b0\ud558\uace0 \uc131\uc7a5\uc744 \uc8fc\ub3c4\ud558\ub294 \uc77c\uc790\ub9ac\uac00 \ub9ce\uc774 \ub9cc\ub4e4\uc5b4\uc838\uc57c \ud55c\ub2e4\"\ub294 \uc598\uae30\uac00 \ub098\uc654\ub2e4. \uc989, \uc131\uc7a5 \ub3d9\ub825\uc744 \ud655\ucda9\u00b7\ubcf4\uc644\ud560 \ud544\uc694\uac00 \uc788\ub294\ub370 \uacbd\uc81c\ubbfc\uc8fc\ud654\ub97c \uc911\uc694\uc2dc\ud558\uba74\uc11c \ub300\uae30\uc5c5\uc744 \uac1c\ud601\uc758 \ub300\uc0c1\uc73c\ub85c\ub9cc \ub2e4\ub8e8\ub294 \uac83\uc740 \ubb38\uc81c\uac00 \uc788\ub2e4\ub294 \uac83\uc774\ub2e4. \uadfc\ub85c\uc2dc\uac04 \ub2e8\ucd94, \ucd5c\uc800\uc784\uae08 \uc778\uc0c1, \ube44\uc815\uaddc\uc9c1 \ucc28\ubcc4 \uae08\uc9c0\uc640 \uac19\uc740 \uc815\ucc45\ub4e4\ub3c4 \ub178\ub3d9 \uc2dc\uc7a5\uc744 \uc624\ud788\ub824 \ub354 \ub531\ub531\ud558\uac8c \ub9cc\ub4e4 \uc6b0\ub824\ub3c4 \uc788\ub2e4. \ubb34\uc5c7\ubcf4\ub2e4 \uc911\uc694\ud55c \ubb38\uc81c\ub294 \uc7ac\uc815\uc778\ub370 \ub098\ub78f\ube5a\uc744 \ub9ce\uc774 \ub298\ub9ac\uc9c0 \uc54a\uc73c\uba74\uc11c \uc138\uc785\uc744 \ub298\ub9ac\ub294 \uac83\uc740 \uc26c\uc6b4 \uc77c\uc774 \uc544\ub2c8\ub2e4. \uc815\ubd80\ub294 \ubc29\uc0b0\ube44\ub9ac\u00b7\ud574\uc678\uc790\uc6d0\uac1c\ubc1c \ub4f1 \uad8c\ub825\ud615 \ube44\ub9ac \uc608\uc0b0\uc744 \uc0ad\uac10\ud558\uace0 \uc18c\ube44\uc131\u00b7\uc120\uc2ec\uc131\u00b7\uc911\ubcf5\uc131 \uc608\uc0b0\ub3c4 \uc904\uc774\ub294 \ub4f1 \uc138\ucd9c \uad6c\uc870\uc870\uc815\uc744 \ud1b5\ud55c 5\ub144 \uac04 112\uc870 \uc6d0\uac00\ub7c9\uc744 \uc544\ub084 \uc218 \uc788\ub2e4\ub294 \uc785\uc7a5\uc774\uc9c0\ub9cc \ud55c\uacc4\uac00 \uc788\uae30 \ub54c\ubb38\uc5d0 \uae30\uc5c5\uc774\ub098 \uace0\uc18c\ub4dd\uc790\ub85c\ubd80\ud130 \uc138\uae08\uc744 \ub354 \uac70\ub46c\ub4e4\uc774\ub294 \uc138\uc81c\uac1c\ud601\uc5d0 \ubc29\uc810\uc774 \ucc0d\ud790 \uac83\uc73c\ub85c \ubcf4\uc778\ub2e4."} {"question": "14\uc138\uae30\uc758 \uc601\uad6d\uc758 \uad6d\uac00\uc758 \uc77c\uc6d0\uc73c\ub85c\uc11c \uc601\uc8fc\uc758 \uc18c\uc720\ubb3c\ucc98\ub7fc \uc5ec\uaca8\uc9c0\uba70, \uc601\uc8fc\uc758 \ud5c8\uac00\uac00 \uc5c6\uc774\ub294 \ub2e4\ub978 \uc9c0\uc5ed\uc73c\ub85c\uc758 \uc774\ub3d9\uc870\ucc28 \ud560 \uc218 \uc5c6\uc5c8\ub358 \uc774\ub4e4\uc744 \ubb34\uc5c7\uc774\ub77c\uace0 \ud558\ub294\uac00?", "paragraph": "\uc640\ud2b8 \ud0c0\uc77c\ub7ec\uc758 \ub09c\uc774 \ucd09\ubc1c\ub41c \uacc4\uae30\ub294 14\uc138\uae30\uc5d0 \ub098\ud0c0\ub09c \uacbd\uc81c \ubc0f \uc0ac\ud68c\uc758 \ub300\uaca9\ubcc0\uc774\uc5c8\ub2e4. 14\uc138\uae30\uac00 \uc2dc\uc791\ud560 \ubb34\ub835, \uc789\uae00\ub79c\ub4dc \uc0ac\ub78c \ub300\ubd80\ubd84\uc740 \uc804\uc6d0\uc9c0\ub300\uc5d0 \uac70\uc8fc\ud558\uba74\uc11c \ub098\ub77c\uc758 \uc74d\uc131\uacfc \ub3c4\uc2dc\ub4e4\uc744 \uba39\uc5ec\uc0b4\ub838\uc73c\uba70 \ubcf5\uc7a1\ud55c \uacbd\uc81c\uccb4\uc81c\uc758 \uc77c\uc6d0\uc774\uc5c8\ub2e4. \uc789\uae00\ub79c\ub4dc \uc804\uc5ed \ub300\ubd80\ubd84\uc5d0\uc11c \uc0dd\uc0b0\ud65c\ub3d9\uc740 \uc2e0\uc0ac\uce35\u00b7\uad50\ud68c \ub4f1\uc758 \uc9c0\uc5ed \uc601\uc8fc\uac00 \ud1b5\uce58\ud558\ub294 \uc7a5\uc6d0 \uc8fc\uc704\ub85c \uc870\uc9c1\ub418\uc5b4 \uc788\uc5c8\uc73c\uba70 \uc7a5\uc6d0 \uc7ac\ud310\uc18c\uc758 \uc0ac\ubc95\uccb4\uacc4\uc5d0 \ub530\ub77c \ud1b5\uce58\ub418\uc5c8\ub2e4. \uc0dd\uc0b0\uc744 \ub2f4\ub2f9\ud55c \uacbd\uc81c\ud65c\ub3d9\uc778\uad6c \uc911 \uc77c\ubd80\ub294 \ube44\uc790\uc720\uc2e0\ubd84\uc778 \ub18d\ub178\uc600\uc73c\uba70, \ub9e4\ub144 \uc77c\uc815 \uae30\uac04\ub3d9\uc548 \uc601\uc8fc\uc758 \ub545\uc5d0 \uc18d\ubc15\ub418\uc5b4 \uc77c\ud574\uc57c \ud588\ub2e4. \uc789\uae00\ub79c\ub4dc \uc804\uc5ed\uc5d0 \uac78\uccd0 \uc790\uc720\ub18d\ubbfc\uacfc \ub18d\ub178\uc758 \ube44\uc728\uc740 \ub2e4\uc591\ud588\uace0, \uc789\uae00\ub79c\ub4dc \ub0a8\ub3d9 \uc9c0\ubc29\uc740 \uc0c1\ub300\uc801\uc73c\ub85c \ub18d\ub178\uac00 \uc801\uc740 \ud3b8\uc774\uc5c8\ub2e4. \uc77c\ubd80 \ub18d\ub178\ub4e4\uc740 \ud0dc\uc5b4\ub0a0 \ub54c\ubd80\ud130 \ube44\uc790\uc720 \uc2e0\ubd84\uc73c\ub85c \ud0dc\uc5b4\ub098 \uc601\uc8fc\uc758 \ud5c8\ub77d \uc5c6\uc774\ub294 \uc7a5\uc6d0 \ubc14\uae65 \ub2e4\ub978 \uacf3\uc5d0\uc11c \uc77c\ud560 \uc218 \uc5c6\uc5c8\ub2e4. \uc77c\ubd80\ub294 \uc790\uc2e0\ub4e4\uc758 \uc2e0\ubd84\uc758 \ubd80\uc790\uc720\ub97c \ub18d\ud1a0\uc5d0 \ub300\ud55c \uc0ac\uc6a9\uad8c\uc758 \uc77c\ubd80\ub85c\uc11c \ubc1b\uc544\ub4e4\uc774\ub294 \ub18d\ub178\ub3c4 \uc788\uc5c8\ub2e4. \uc778\uad6c \uc99d\uac00\ub294 \ub18d\uacbd \uac00\ub2a5 \ud1a0\uc9c0\uc5d0 \ub300\ud55c \uc555\ubc15\uc73c\ub85c \uc774\uc5b4\uc84c\uace0, \uc9c0\uc5ed \uc9c0\uc8fc\ub4e4\uc758 \uad8c\ub825\uc774 \uc99d\uac00\ud558\uc600\ub2e4.", "answer": "\ub18d\ub178", "sentence": "\uc0dd\uc0b0\uc744 \ub2f4\ub2f9\ud55c \uacbd\uc81c\ud65c\ub3d9\uc778\uad6c \uc911 \uc77c\ubd80\ub294 \ube44\uc790\uc720\uc2e0\ubd84\uc778 \ub18d\ub178 \uc600\uc73c\uba70, \ub9e4\ub144 \uc77c\uc815 \uae30\uac04\ub3d9\uc548 \uc601\uc8fc\uc758 \ub545\uc5d0 \uc18d\ubc15\ub418\uc5b4 \uc77c\ud574\uc57c \ud588\ub2e4.", "paragraph_sentence": "\uc640\ud2b8 \ud0c0\uc77c\ub7ec\uc758 \ub09c\uc774 \ucd09\ubc1c\ub41c \uacc4\uae30\ub294 14\uc138\uae30\uc5d0 \ub098\ud0c0\ub09c \uacbd\uc81c \ubc0f \uc0ac\ud68c\uc758 \ub300\uaca9\ubcc0\uc774\uc5c8\ub2e4. 14\uc138\uae30\uac00 \uc2dc\uc791\ud560 \ubb34\ub835, \uc789\uae00\ub79c\ub4dc \uc0ac\ub78c \ub300\ubd80\ubd84\uc740 \uc804\uc6d0\uc9c0\ub300\uc5d0 \uac70\uc8fc\ud558\uba74\uc11c \ub098\ub77c\uc758 \uc74d\uc131\uacfc \ub3c4\uc2dc\ub4e4\uc744 \uba39\uc5ec\uc0b4\ub838\uc73c\uba70 \ubcf5\uc7a1\ud55c \uacbd\uc81c\uccb4\uc81c\uc758 \uc77c\uc6d0\uc774\uc5c8\ub2e4. \uc789\uae00\ub79c\ub4dc \uc804\uc5ed \ub300\ubd80\ubd84\uc5d0\uc11c \uc0dd\uc0b0\ud65c\ub3d9\uc740 \uc2e0\uc0ac\uce35\u00b7\uad50\ud68c \ub4f1\uc758 \uc9c0\uc5ed \uc601\uc8fc\uac00 \ud1b5\uce58\ud558\ub294 \uc7a5\uc6d0 \uc8fc\uc704\ub85c \uc870\uc9c1\ub418\uc5b4 \uc788\uc5c8\uc73c\uba70 \uc7a5\uc6d0 \uc7ac\ud310\uc18c\uc758 \uc0ac\ubc95\uccb4\uacc4\uc5d0 \ub530\ub77c \ud1b5\uce58\ub418\uc5c8\ub2e4. \uc0dd\uc0b0\uc744 \ub2f4\ub2f9\ud55c \uacbd\uc81c\ud65c\ub3d9\uc778\uad6c \uc911 \uc77c\ubd80\ub294 \ube44\uc790\uc720\uc2e0\ubd84\uc778 \ub18d\ub178 \uc600\uc73c\uba70, \ub9e4\ub144 \uc77c\uc815 \uae30\uac04\ub3d9\uc548 \uc601\uc8fc\uc758 \ub545\uc5d0 \uc18d\ubc15\ub418\uc5b4 \uc77c\ud574\uc57c \ud588\ub2e4. \uc789\uae00\ub79c\ub4dc \uc804\uc5ed\uc5d0 \uac78\uccd0 \uc790\uc720\ub18d\ubbfc\uacfc \ub18d\ub178\uc758 \ube44\uc728\uc740 \ub2e4\uc591\ud588\uace0, \uc789\uae00\ub79c\ub4dc \ub0a8\ub3d9 \uc9c0\ubc29\uc740 \uc0c1\ub300\uc801\uc73c\ub85c \ub18d\ub178\uac00 \uc801\uc740 \ud3b8\uc774\uc5c8\ub2e4. \uc77c\ubd80 \ub18d\ub178\ub4e4\uc740 \ud0dc\uc5b4\ub0a0 \ub54c\ubd80\ud130 \ube44\uc790\uc720 \uc2e0\ubd84\uc73c\ub85c \ud0dc\uc5b4\ub098 \uc601\uc8fc\uc758 \ud5c8\ub77d \uc5c6\uc774\ub294 \uc7a5\uc6d0 \ubc14\uae65 \ub2e4\ub978 \uacf3\uc5d0\uc11c \uc77c\ud560 \uc218 \uc5c6\uc5c8\ub2e4. \uc77c\ubd80\ub294 \uc790\uc2e0\ub4e4\uc758 \uc2e0\ubd84\uc758 \ubd80\uc790\uc720\ub97c \ub18d\ud1a0\uc5d0 \ub300\ud55c \uc0ac\uc6a9\uad8c\uc758 \uc77c\ubd80\ub85c\uc11c \ubc1b\uc544\ub4e4\uc774\ub294 \ub18d\ub178\ub3c4 \uc788\uc5c8\ub2e4. \uc778\uad6c \uc99d\uac00\ub294 \ub18d\uacbd \uac00\ub2a5 \ud1a0\uc9c0\uc5d0 \ub300\ud55c \uc555\ubc15\uc73c\ub85c \uc774\uc5b4\uc84c\uace0, \uc9c0\uc5ed \uc9c0\uc8fc\ub4e4\uc758 \uad8c\ub825\uc774 \uc99d\uac00\ud558\uc600\ub2e4.", "paragraph_answer": "\uc640\ud2b8 \ud0c0\uc77c\ub7ec\uc758 \ub09c\uc774 \ucd09\ubc1c\ub41c \uacc4\uae30\ub294 14\uc138\uae30\uc5d0 \ub098\ud0c0\ub09c \uacbd\uc81c \ubc0f \uc0ac\ud68c\uc758 \ub300\uaca9\ubcc0\uc774\uc5c8\ub2e4. 14\uc138\uae30\uac00 \uc2dc\uc791\ud560 \ubb34\ub835, \uc789\uae00\ub79c\ub4dc \uc0ac\ub78c \ub300\ubd80\ubd84\uc740 \uc804\uc6d0\uc9c0\ub300\uc5d0 \uac70\uc8fc\ud558\uba74\uc11c \ub098\ub77c\uc758 \uc74d\uc131\uacfc \ub3c4\uc2dc\ub4e4\uc744 \uba39\uc5ec\uc0b4\ub838\uc73c\uba70 \ubcf5\uc7a1\ud55c \uacbd\uc81c\uccb4\uc81c\uc758 \uc77c\uc6d0\uc774\uc5c8\ub2e4. \uc789\uae00\ub79c\ub4dc \uc804\uc5ed \ub300\ubd80\ubd84\uc5d0\uc11c \uc0dd\uc0b0\ud65c\ub3d9\uc740 \uc2e0\uc0ac\uce35\u00b7\uad50\ud68c \ub4f1\uc758 \uc9c0\uc5ed \uc601\uc8fc\uac00 \ud1b5\uce58\ud558\ub294 \uc7a5\uc6d0 \uc8fc\uc704\ub85c \uc870\uc9c1\ub418\uc5b4 \uc788\uc5c8\uc73c\uba70 \uc7a5\uc6d0 \uc7ac\ud310\uc18c\uc758 \uc0ac\ubc95\uccb4\uacc4\uc5d0 \ub530\ub77c \ud1b5\uce58\ub418\uc5c8\ub2e4. \uc0dd\uc0b0\uc744 \ub2f4\ub2f9\ud55c \uacbd\uc81c\ud65c\ub3d9\uc778\uad6c \uc911 \uc77c\ubd80\ub294 \ube44\uc790\uc720\uc2e0\ubd84\uc778 \ub18d\ub178 \uc600\uc73c\uba70, \ub9e4\ub144 \uc77c\uc815 \uae30\uac04\ub3d9\uc548 \uc601\uc8fc\uc758 \ub545\uc5d0 \uc18d\ubc15\ub418\uc5b4 \uc77c\ud574\uc57c \ud588\ub2e4. \uc789\uae00\ub79c\ub4dc \uc804\uc5ed\uc5d0 \uac78\uccd0 \uc790\uc720\ub18d\ubbfc\uacfc \ub18d\ub178\uc758 \ube44\uc728\uc740 \ub2e4\uc591\ud588\uace0, \uc789\uae00\ub79c\ub4dc \ub0a8\ub3d9 \uc9c0\ubc29\uc740 \uc0c1\ub300\uc801\uc73c\ub85c \ub18d\ub178\uac00 \uc801\uc740 \ud3b8\uc774\uc5c8\ub2e4. \uc77c\ubd80 \ub18d\ub178\ub4e4\uc740 \ud0dc\uc5b4\ub0a0 \ub54c\ubd80\ud130 \ube44\uc790\uc720 \uc2e0\ubd84\uc73c\ub85c \ud0dc\uc5b4\ub098 \uc601\uc8fc\uc758 \ud5c8\ub77d \uc5c6\uc774\ub294 \uc7a5\uc6d0 \ubc14\uae65 \ub2e4\ub978 \uacf3\uc5d0\uc11c \uc77c\ud560 \uc218 \uc5c6\uc5c8\ub2e4. \uc77c\ubd80\ub294 \uc790\uc2e0\ub4e4\uc758 \uc2e0\ubd84\uc758 \ubd80\uc790\uc720\ub97c \ub18d\ud1a0\uc5d0 \ub300\ud55c \uc0ac\uc6a9\uad8c\uc758 \uc77c\ubd80\ub85c\uc11c \ubc1b\uc544\ub4e4\uc774\ub294 \ub18d\ub178\ub3c4 \uc788\uc5c8\ub2e4. \uc778\uad6c \uc99d\uac00\ub294 \ub18d\uacbd \uac00\ub2a5 \ud1a0\uc9c0\uc5d0 \ub300\ud55c \uc555\ubc15\uc73c\ub85c \uc774\uc5b4\uc84c\uace0, \uc9c0\uc5ed \uc9c0\uc8fc\ub4e4\uc758 \uad8c\ub825\uc774 \uc99d\uac00\ud558\uc600\ub2e4.", "sentence_answer": "\uc0dd\uc0b0\uc744 \ub2f4\ub2f9\ud55c \uacbd\uc81c\ud65c\ub3d9\uc778\uad6c \uc911 \uc77c\ubd80\ub294 \ube44\uc790\uc720\uc2e0\ubd84\uc778 \ub18d\ub178 \uc600\uc73c\uba70, \ub9e4\ub144 \uc77c\uc815 \uae30\uac04\ub3d9\uc548 \uc601\uc8fc\uc758 \ub545\uc5d0 \uc18d\ubc15\ub418\uc5b4 \uc77c\ud574\uc57c \ud588\ub2e4.", "paragraph_id": "6541489-4-2", "paragraph_question": "question: 14\uc138\uae30\uc758 \uc601\uad6d\uc758 \uad6d\uac00\uc758 \uc77c\uc6d0\uc73c\ub85c\uc11c \uc601\uc8fc\uc758 \uc18c\uc720\ubb3c\ucc98\ub7fc \uc5ec\uaca8\uc9c0\uba70, \uc601\uc8fc\uc758 \ud5c8\uac00\uac00 \uc5c6\uc774\ub294 \ub2e4\ub978 \uc9c0\uc5ed\uc73c\ub85c\uc758 \uc774\ub3d9\uc870\ucc28 \ud560 \uc218 \uc5c6\uc5c8\ub358 \uc774\ub4e4\uc744 \ubb34\uc5c7\uc774\ub77c\uace0 \ud558\ub294\uac00?, context: \uc640\ud2b8 \ud0c0\uc77c\ub7ec\uc758 \ub09c\uc774 \ucd09\ubc1c\ub41c \uacc4\uae30\ub294 14\uc138\uae30\uc5d0 \ub098\ud0c0\ub09c \uacbd\uc81c \ubc0f \uc0ac\ud68c\uc758 \ub300\uaca9\ubcc0\uc774\uc5c8\ub2e4. 14\uc138\uae30\uac00 \uc2dc\uc791\ud560 \ubb34\ub835, \uc789\uae00\ub79c\ub4dc \uc0ac\ub78c \ub300\ubd80\ubd84\uc740 \uc804\uc6d0\uc9c0\ub300\uc5d0 \uac70\uc8fc\ud558\uba74\uc11c \ub098\ub77c\uc758 \uc74d\uc131\uacfc \ub3c4\uc2dc\ub4e4\uc744 \uba39\uc5ec\uc0b4\ub838\uc73c\uba70 \ubcf5\uc7a1\ud55c \uacbd\uc81c\uccb4\uc81c\uc758 \uc77c\uc6d0\uc774\uc5c8\ub2e4. \uc789\uae00\ub79c\ub4dc \uc804\uc5ed \ub300\ubd80\ubd84\uc5d0\uc11c \uc0dd\uc0b0\ud65c\ub3d9\uc740 \uc2e0\uc0ac\uce35\u00b7\uad50\ud68c \ub4f1\uc758 \uc9c0\uc5ed \uc601\uc8fc\uac00 \ud1b5\uce58\ud558\ub294 \uc7a5\uc6d0 \uc8fc\uc704\ub85c \uc870\uc9c1\ub418\uc5b4 \uc788\uc5c8\uc73c\uba70 \uc7a5\uc6d0 \uc7ac\ud310\uc18c\uc758 \uc0ac\ubc95\uccb4\uacc4\uc5d0 \ub530\ub77c \ud1b5\uce58\ub418\uc5c8\ub2e4. \uc0dd\uc0b0\uc744 \ub2f4\ub2f9\ud55c \uacbd\uc81c\ud65c\ub3d9\uc778\uad6c \uc911 \uc77c\ubd80\ub294 \ube44\uc790\uc720\uc2e0\ubd84\uc778 \ub18d\ub178\uc600\uc73c\uba70, \ub9e4\ub144 \uc77c\uc815 \uae30\uac04\ub3d9\uc548 \uc601\uc8fc\uc758 \ub545\uc5d0 \uc18d\ubc15\ub418\uc5b4 \uc77c\ud574\uc57c \ud588\ub2e4. \uc789\uae00\ub79c\ub4dc \uc804\uc5ed\uc5d0 \uac78\uccd0 \uc790\uc720\ub18d\ubbfc\uacfc \ub18d\ub178\uc758 \ube44\uc728\uc740 \ub2e4\uc591\ud588\uace0, \uc789\uae00\ub79c\ub4dc \ub0a8\ub3d9 \uc9c0\ubc29\uc740 \uc0c1\ub300\uc801\uc73c\ub85c \ub18d\ub178\uac00 \uc801\uc740 \ud3b8\uc774\uc5c8\ub2e4. \uc77c\ubd80 \ub18d\ub178\ub4e4\uc740 \ud0dc\uc5b4\ub0a0 \ub54c\ubd80\ud130 \ube44\uc790\uc720 \uc2e0\ubd84\uc73c\ub85c \ud0dc\uc5b4\ub098 \uc601\uc8fc\uc758 \ud5c8\ub77d \uc5c6\uc774\ub294 \uc7a5\uc6d0 \ubc14\uae65 \ub2e4\ub978 \uacf3\uc5d0\uc11c \uc77c\ud560 \uc218 \uc5c6\uc5c8\ub2e4. \uc77c\ubd80\ub294 \uc790\uc2e0\ub4e4\uc758 \uc2e0\ubd84\uc758 \ubd80\uc790\uc720\ub97c \ub18d\ud1a0\uc5d0 \ub300\ud55c \uc0ac\uc6a9\uad8c\uc758 \uc77c\ubd80\ub85c\uc11c \ubc1b\uc544\ub4e4\uc774\ub294 \ub18d\ub178\ub3c4 \uc788\uc5c8\ub2e4. \uc778\uad6c \uc99d\uac00\ub294 \ub18d\uacbd \uac00\ub2a5 \ud1a0\uc9c0\uc5d0 \ub300\ud55c \uc555\ubc15\uc73c\ub85c \uc774\uc5b4\uc84c\uace0, \uc9c0\uc5ed \uc9c0\uc8fc\ub4e4\uc758 \uad8c\ub825\uc774 \uc99d\uac00\ud558\uc600\ub2e4."} {"question": "\uc724\uc601\uc774 \ud55c\uc131\uc744 \uc911\uc2ec\uc73c\ub85c \uc0ac\ubc29 \ubc29\uc704\ub97c \uc815\ud588\uc744 \ub54c \ubb18\ubc29\uc5d0 \ud574\ub2f9\ub418\ub294 \uacf3\uc740?", "paragraph": "\ub610\ud55c \uadf8\ub294 \uac01 \uc9c0\uc5ed\uc758 \uac70\ub9ac\uc5d0 \ub300\ud574\uc11c\ub3c4 \uac04\ub7b5\ud558\uac8c \uc11c\uc220\ud558\uc600\ub2e4. '\ud55c\uc131\uc744 \uc911\uc2ec\uc73c\ub85c \ud558\uc5ec \uc0ac\ubc29(\u56db\u65b9) \ubc29\uc704\ub97c \uc815\ud55c\ub2e4\uba74 \uc628\uc131(\u7a69\u57ce)\uc774 \uc790\ubc29(\u5b50\u65b9), \ud574\ub0a8\uc774 \uc624\ubc29(\u5348\u65b9), \ud48d\ucc9c(\u8c4a\u5ddd)\uc774 \uc720\ubc29(\u9149\u65b9), \uac15\ub989\uc774 \ubb18\ubc29(\u536f\u65b9)\uc774 \ub41c\ub2e4. \uadfc\uc138\uc5d0 \uc18c\uc704 \uc9c0\ub3c4\ub77c\ub294 \uac83\uc744 \ubcf4\uba74, \ub300\uac1c\ub294 \uc885\uc774 \ub113\uc774\uc5d0 \ub530\ub77c \uc704\uce58\ub97c \ubc30\uc815\ud558\uace0 \uc704\uce58\uc5d0 \ub530\ub77c\uc11c \uadf8\ub9bc\uc744 \uc791\uc131\ud588\uae30 \ub54c\ubb38\uc5d0 \uae38\uc774\uc640 \ub113\uc774\uac00 \uc11c\ub85c \uc5b4\uae0b\ub098\uace0 \uc881\uace0 \ub113\uc740 \ub370\ub3c4 \uc81c\ub300\ub85c \ub9de\uc9c0 \uc54a\ub294\ub2e4. \ub450\ub9cc\uac15\uc774 \uc628\uc131\uc73c\ub85c \ub4e4\uc5b4\uc624\ub294 \uacf3\ub9cc \ubcf4\uc544\ub3c4 \ub3d9\ucabd\uc73c\ub85c \ud750\ub974\ub2e4\uac00\ub294 \ub2e4\uc2dc \ubd81\ucabd\uc73c\ub85c \ud758\ub7ec\uc11c \ubbf8\uc804\ubcf4(\u7f8e\u9322\u5821)\uc5d0 \uc640\uc11c\ub294 \ub2e4\uc2dc \ub0a8\ucabd\uc73c\ub85c \ud758\ub7ec\uc11c \uc11c\uc218\ub77c(\u897f\u6c34\u7f85)\ub85c \ub4e4\uc5b4\uac00\uace0, \uc555\ub85d\uac15\uc740 \uc0bc\uc218(\u4e09\u6c34)\uc640 \uac11\uc0b0(\u7532\u5c71) \ubc0f \ud3d0\uc0ac\uad70\uc744 \uc9c0\ub098\ub294\ub370, \uc5ec\ub7ec \ubc88 \uad7d\uc774\uccd0 \uac00\uc9c0\uace0 \uac15\uacc4(\u6c5f\u754c)\uc640 \uc704\uc6d0(\u6e2d\u539f)\uc744 \uc2a4\uccd0 \uc11c\ub0a8\ucabd\uc73c\ub85c \ud5a5\ud558\ub2e4\uac00 \ucc3d\uc131(\u660c\u57ce)\uc744 \uc9c0\ub098\uc11c\uc57c \ube44\ub85c\uc18c \ub0a8\ucabd\uc73c\ub85c \uace7\uc7a5 \ube60\uc838 \ub0b4\ub824 \ud1b5\uad70\uc815(\u7d71\u8ecd\u4ead)\uc758 \uc11c\ucabd\uc744 \uc2f8\uace0 \ub3cc\uc544\uc11c \ub300\ucd1d\uac15(\u5927\u7e3d\u6c5f)\uc73c\ub85c \ub4e4\uc5b4\uac04\ub2e4.'\ub294 \uac83\uc774\ub2e4.", "answer": "\uac15\ub989", "sentence": "'\ud55c\uc131\uc744 \uc911\uc2ec\uc73c\ub85c \ud558\uc5ec \uc0ac\ubc29(\u56db\u65b9) \ubc29\uc704\ub97c \uc815\ud55c\ub2e4\uba74 \uc628\uc131(\u7a69\u57ce)\uc774 \uc790\ubc29(\u5b50\u65b9), \ud574\ub0a8\uc774 \uc624\ubc29(\u5348\u65b9), \ud48d\ucc9c(\u8c4a\u5ddd)\uc774 \uc720\ubc29(\u9149\u65b9), \uac15\ub989 \uc774 \ubb18\ubc29(\u536f\u65b9)\uc774 \ub41c\ub2e4.", "paragraph_sentence": "\ub610\ud55c \uadf8\ub294 \uac01 \uc9c0\uc5ed\uc758 \uac70\ub9ac\uc5d0 \ub300\ud574\uc11c\ub3c4 \uac04\ub7b5\ud558\uac8c \uc11c\uc220\ud558\uc600\ub2e4. '\ud55c\uc131\uc744 \uc911\uc2ec\uc73c\ub85c \ud558\uc5ec \uc0ac\ubc29(\u56db\u65b9) \ubc29\uc704\ub97c \uc815\ud55c\ub2e4\uba74 \uc628\uc131(\u7a69\u57ce)\uc774 \uc790\ubc29(\u5b50\u65b9), \ud574\ub0a8\uc774 \uc624\ubc29(\u5348\u65b9), \ud48d\ucc9c(\u8c4a\u5ddd)\uc774 \uc720\ubc29(\u9149\u65b9), \uac15\ub989 \uc774 \ubb18\ubc29(\u536f\u65b9)\uc774 \ub41c\ub2e4. \uadfc\uc138\uc5d0 \uc18c\uc704 \uc9c0\ub3c4\ub77c\ub294 \uac83\uc744 \ubcf4\uba74, \ub300\uac1c\ub294 \uc885\uc774 \ub113\uc774\uc5d0 \ub530\ub77c \uc704\uce58\ub97c \ubc30\uc815\ud558\uace0 \uc704\uce58\uc5d0 \ub530\ub77c\uc11c \uadf8\ub9bc\uc744 \uc791\uc131\ud588\uae30 \ub54c\ubb38\uc5d0 \uae38\uc774\uc640 \ub113\uc774\uac00 \uc11c\ub85c \uc5b4\uae0b\ub098\uace0 \uc881\uace0 \ub113\uc740 \ub370\ub3c4 \uc81c\ub300\ub85c \ub9de\uc9c0 \uc54a\ub294\ub2e4. \ub450\ub9cc\uac15\uc774 \uc628\uc131\uc73c\ub85c \ub4e4\uc5b4\uc624\ub294 \uacf3\ub9cc \ubcf4\uc544\ub3c4 \ub3d9\ucabd\uc73c\ub85c \ud750\ub974\ub2e4\uac00\ub294 \ub2e4\uc2dc \ubd81\ucabd\uc73c\ub85c \ud758\ub7ec\uc11c \ubbf8\uc804\ubcf4(\u7f8e\u9322\u5821)\uc5d0 \uc640\uc11c\ub294 \ub2e4\uc2dc \ub0a8\ucabd\uc73c\ub85c \ud758\ub7ec\uc11c \uc11c\uc218\ub77c(\u897f\u6c34\u7f85)\ub85c \ub4e4\uc5b4\uac00\uace0, \uc555\ub85d\uac15\uc740 \uc0bc\uc218(\u4e09\u6c34)\uc640 \uac11\uc0b0(\u7532\u5c71) \ubc0f \ud3d0\uc0ac\uad70\uc744 \uc9c0\ub098\ub294\ub370, \uc5ec\ub7ec \ubc88 \uad7d\uc774\uccd0 \uac00\uc9c0\uace0 \uac15\uacc4(\u6c5f\u754c)\uc640 \uc704\uc6d0(\u6e2d\u539f)\uc744 \uc2a4\uccd0 \uc11c\ub0a8\ucabd\uc73c\ub85c \ud5a5\ud558\ub2e4\uac00 \ucc3d\uc131(\u660c\u57ce)\uc744 \uc9c0\ub098\uc11c\uc57c \ube44\ub85c\uc18c \ub0a8\ucabd\uc73c\ub85c \uace7\uc7a5 \ube60\uc838 \ub0b4\ub824 \ud1b5\uad70\uc815(\u7d71\u8ecd\u4ead)\uc758 \uc11c\ucabd\uc744 \uc2f8\uace0 \ub3cc\uc544\uc11c \ub300\ucd1d\uac15(\u5927\u7e3d\u6c5f)\uc73c\ub85c \ub4e4\uc5b4\uac04\ub2e4. '\ub294 \uac83\uc774\ub2e4.", "paragraph_answer": "\ub610\ud55c \uadf8\ub294 \uac01 \uc9c0\uc5ed\uc758 \uac70\ub9ac\uc5d0 \ub300\ud574\uc11c\ub3c4 \uac04\ub7b5\ud558\uac8c \uc11c\uc220\ud558\uc600\ub2e4. '\ud55c\uc131\uc744 \uc911\uc2ec\uc73c\ub85c \ud558\uc5ec \uc0ac\ubc29(\u56db\u65b9) \ubc29\uc704\ub97c \uc815\ud55c\ub2e4\uba74 \uc628\uc131(\u7a69\u57ce)\uc774 \uc790\ubc29(\u5b50\u65b9), \ud574\ub0a8\uc774 \uc624\ubc29(\u5348\u65b9), \ud48d\ucc9c(\u8c4a\u5ddd)\uc774 \uc720\ubc29(\u9149\u65b9), \uac15\ub989 \uc774 \ubb18\ubc29(\u536f\u65b9)\uc774 \ub41c\ub2e4. \uadfc\uc138\uc5d0 \uc18c\uc704 \uc9c0\ub3c4\ub77c\ub294 \uac83\uc744 \ubcf4\uba74, \ub300\uac1c\ub294 \uc885\uc774 \ub113\uc774\uc5d0 \ub530\ub77c \uc704\uce58\ub97c \ubc30\uc815\ud558\uace0 \uc704\uce58\uc5d0 \ub530\ub77c\uc11c \uadf8\ub9bc\uc744 \uc791\uc131\ud588\uae30 \ub54c\ubb38\uc5d0 \uae38\uc774\uc640 \ub113\uc774\uac00 \uc11c\ub85c \uc5b4\uae0b\ub098\uace0 \uc881\uace0 \ub113\uc740 \ub370\ub3c4 \uc81c\ub300\ub85c \ub9de\uc9c0 \uc54a\ub294\ub2e4. \ub450\ub9cc\uac15\uc774 \uc628\uc131\uc73c\ub85c \ub4e4\uc5b4\uc624\ub294 \uacf3\ub9cc \ubcf4\uc544\ub3c4 \ub3d9\ucabd\uc73c\ub85c \ud750\ub974\ub2e4\uac00\ub294 \ub2e4\uc2dc \ubd81\ucabd\uc73c\ub85c \ud758\ub7ec\uc11c \ubbf8\uc804\ubcf4(\u7f8e\u9322\u5821)\uc5d0 \uc640\uc11c\ub294 \ub2e4\uc2dc \ub0a8\ucabd\uc73c\ub85c \ud758\ub7ec\uc11c \uc11c\uc218\ub77c(\u897f\u6c34\u7f85)\ub85c \ub4e4\uc5b4\uac00\uace0, \uc555\ub85d\uac15\uc740 \uc0bc\uc218(\u4e09\u6c34)\uc640 \uac11\uc0b0(\u7532\u5c71) \ubc0f \ud3d0\uc0ac\uad70\uc744 \uc9c0\ub098\ub294\ub370, \uc5ec\ub7ec \ubc88 \uad7d\uc774\uccd0 \uac00\uc9c0\uace0 \uac15\uacc4(\u6c5f\u754c)\uc640 \uc704\uc6d0(\u6e2d\u539f)\uc744 \uc2a4\uccd0 \uc11c\ub0a8\ucabd\uc73c\ub85c \ud5a5\ud558\ub2e4\uac00 \ucc3d\uc131(\u660c\u57ce)\uc744 \uc9c0\ub098\uc11c\uc57c \ube44\ub85c\uc18c \ub0a8\ucabd\uc73c\ub85c \uace7\uc7a5 \ube60\uc838 \ub0b4\ub824 \ud1b5\uad70\uc815(\u7d71\u8ecd\u4ead)\uc758 \uc11c\ucabd\uc744 \uc2f8\uace0 \ub3cc\uc544\uc11c \ub300\ucd1d\uac15(\u5927\u7e3d\u6c5f)\uc73c\ub85c \ub4e4\uc5b4\uac04\ub2e4.'\ub294 \uac83\uc774\ub2e4.", "sentence_answer": "'\ud55c\uc131\uc744 \uc911\uc2ec\uc73c\ub85c \ud558\uc5ec \uc0ac\ubc29(\u56db\u65b9) \ubc29\uc704\ub97c \uc815\ud55c\ub2e4\uba74 \uc628\uc131(\u7a69\u57ce)\uc774 \uc790\ubc29(\u5b50\u65b9), \ud574\ub0a8\uc774 \uc624\ubc29(\u5348\u65b9), \ud48d\ucc9c(\u8c4a\u5ddd)\uc774 \uc720\ubc29(\u9149\u65b9), \uac15\ub989 \uc774 \ubb18\ubc29(\u536f\u65b9)\uc774 \ub41c\ub2e4.", "paragraph_id": "6543990-2-1", "paragraph_question": "question: \uc724\uc601\uc774 \ud55c\uc131\uc744 \uc911\uc2ec\uc73c\ub85c \uc0ac\ubc29 \ubc29\uc704\ub97c \uc815\ud588\uc744 \ub54c \ubb18\ubc29\uc5d0 \ud574\ub2f9\ub418\ub294 \uacf3\uc740?, context: \ub610\ud55c \uadf8\ub294 \uac01 \uc9c0\uc5ed\uc758 \uac70\ub9ac\uc5d0 \ub300\ud574\uc11c\ub3c4 \uac04\ub7b5\ud558\uac8c \uc11c\uc220\ud558\uc600\ub2e4. '\ud55c\uc131\uc744 \uc911\uc2ec\uc73c\ub85c \ud558\uc5ec \uc0ac\ubc29(\u56db\u65b9) \ubc29\uc704\ub97c \uc815\ud55c\ub2e4\uba74 \uc628\uc131(\u7a69\u57ce)\uc774 \uc790\ubc29(\u5b50\u65b9), \ud574\ub0a8\uc774 \uc624\ubc29(\u5348\u65b9), \ud48d\ucc9c(\u8c4a\u5ddd)\uc774 \uc720\ubc29(\u9149\u65b9), \uac15\ub989\uc774 \ubb18\ubc29(\u536f\u65b9)\uc774 \ub41c\ub2e4. \uadfc\uc138\uc5d0 \uc18c\uc704 \uc9c0\ub3c4\ub77c\ub294 \uac83\uc744 \ubcf4\uba74, \ub300\uac1c\ub294 \uc885\uc774 \ub113\uc774\uc5d0 \ub530\ub77c \uc704\uce58\ub97c \ubc30\uc815\ud558\uace0 \uc704\uce58\uc5d0 \ub530\ub77c\uc11c \uadf8\ub9bc\uc744 \uc791\uc131\ud588\uae30 \ub54c\ubb38\uc5d0 \uae38\uc774\uc640 \ub113\uc774\uac00 \uc11c\ub85c \uc5b4\uae0b\ub098\uace0 \uc881\uace0 \ub113\uc740 \ub370\ub3c4 \uc81c\ub300\ub85c \ub9de\uc9c0 \uc54a\ub294\ub2e4. \ub450\ub9cc\uac15\uc774 \uc628\uc131\uc73c\ub85c \ub4e4\uc5b4\uc624\ub294 \uacf3\ub9cc \ubcf4\uc544\ub3c4 \ub3d9\ucabd\uc73c\ub85c \ud750\ub974\ub2e4\uac00\ub294 \ub2e4\uc2dc \ubd81\ucabd\uc73c\ub85c \ud758\ub7ec\uc11c \ubbf8\uc804\ubcf4(\u7f8e\u9322\u5821)\uc5d0 \uc640\uc11c\ub294 \ub2e4\uc2dc \ub0a8\ucabd\uc73c\ub85c \ud758\ub7ec\uc11c \uc11c\uc218\ub77c(\u897f\u6c34\u7f85)\ub85c \ub4e4\uc5b4\uac00\uace0, \uc555\ub85d\uac15\uc740 \uc0bc\uc218(\u4e09\u6c34)\uc640 \uac11\uc0b0(\u7532\u5c71) \ubc0f \ud3d0\uc0ac\uad70\uc744 \uc9c0\ub098\ub294\ub370, \uc5ec\ub7ec \ubc88 \uad7d\uc774\uccd0 \uac00\uc9c0\uace0 \uac15\uacc4(\u6c5f\u754c)\uc640 \uc704\uc6d0(\u6e2d\u539f)\uc744 \uc2a4\uccd0 \uc11c\ub0a8\ucabd\uc73c\ub85c \ud5a5\ud558\ub2e4\uac00 \ucc3d\uc131(\u660c\u57ce)\uc744 \uc9c0\ub098\uc11c\uc57c \ube44\ub85c\uc18c \ub0a8\ucabd\uc73c\ub85c \uace7\uc7a5 \ube60\uc838 \ub0b4\ub824 \ud1b5\uad70\uc815(\u7d71\u8ecd\u4ead)\uc758 \uc11c\ucabd\uc744 \uc2f8\uace0 \ub3cc\uc544\uc11c \ub300\ucd1d\uac15(\u5927\u7e3d\u6c5f)\uc73c\ub85c \ub4e4\uc5b4\uac04\ub2e4.'\ub294 \uac83\uc774\ub2e4."} {"question": "2010\ub144\uc5d0 \uba55\uc2dc\ucf54 \ub9cc\uc5d0 \uc704\uce58\ud55c \uc601\uad6d\uc758 \ucd5c\ub300 \uae30\uc5c5\uc778 \uc138\uacc4 2\uc704 \uc11d\uc720\ud68c\uc0ac\ub294?", "paragraph": "2010\ub144 04\uc6d4 20\uc77c, \ubbf8\uad6d \ub8e8\uc774\uc9c0\uc560\ub098 \uc8fc \uba55\uc2dc\ucf54 \ub9cc\uc5d0 \uc788\ub294 \uc5f0 \ub9e4\ucd9c 246\uc870\uc6d0\uc758 \uc601\uad6d \ucd5c\ub300 \uae30\uc5c5\uc774\uc790 \uc138\uacc4 2\uc704 \uc11d\uc720\ud68c\uc0ac\uc778 BP\uc758 \ud604\ub300\uc911\uacf5\uc5c5\uc774 \uc81c\uc870\ud55c \uc2dc\ucd94\uc120\uc778 \ub525\uc6cc\ud130 \ud638\ub77c\uc774\uc98c \uc11d\uc720 \uc2dc\ucd94 \uc2dc\uc124\uc774 \ud3ed\ubc1c\ud588\ub2e4. \uac70\ub300\ud55c \uc2dc\ucd94\uc120\uc740 \ub77c\uc774\uc800\ub77c\uace0 \ubd80\ub974\ub294 \ud30c\uc774\ud504\ub85c \ud574\uc800\uc5d0 \uc5f0\uacb0\ub418\uc5b4 \uc788\uc5c8\ub2e4. 0.5\ubbf8\ud130 \ud3ed\uc5d0 1600\ubbf8\ud130 \uc815\ub3c4 \ub418\ub294 \uae38\uc774\uc600\ub2e4. \ud30c\uc774\ud504\ub294 \uc720\uc0ac\uc2dc \uc720\uc815\uc744 \ucc28\ub2e8\ud558\uae30 \uc704\ud574 \uace0\uc548\ub41c \uc6d0\uc720\ubd84\ucd9c \ubc29\uc9c0 \uc7a5\uce58 BOP(Blowout preventer)\uc5d0 \uc5f0\uacb0\ub418\uc5b4 \uc788\uc5c8\ub2e4. \ub2fb\uc774 \uc5c6\ub294 \uc2dc\ucd94\uc120\uc740 GPS\ub97c \uc774\uc6a9\ud574 \ud574\uc800\ub85c\ubd80\ud130 1500\ubbf8\ud130 \uc704\uc5d0 \uc788\uc5c8\ub2e4. \uc720\uc815\uc740 \uc6d0\uc720\ubd84\ucd9c \ubc29\uc9c0 \uc7a5\uce58 4000\ubbf8\ud130 \uc544\ub798\uc5d0 \uc788\uc5c8\ub2e4. \uac00\uc2a4\uac00 \uc720\uc815\uc5d0 \uc0c8\uc5b4\ub4e4\uc5b4\uac00\uba74\uc11c \ud3ed\ubc1c\uc774 \uc77c\uc5b4\ub0ac\uace0, 5500\ubbf8\ud130 \ub5a8\uc5b4\uc838 \uc788\ub294 \ud574\uc0c1 \uc2dc\ucd94\uc120\uc5d0 \ubd88\uc774 \ubd99\uc5c8\ub2e4. \uc2b9\ubb34\uc6d0\uc740 BOP\ub97c \uc791\ub3d9\uc2dc\ud0a4\ub824 \ud588\uc9c0\ub9cc \uc791\ub3d9\ud558\uc9c0 \uc54a\uc558\ub2e4. \uc6d0\uc720 \uc2dc\ucd94\uac00 \uc9c4\ud589 \uc911\uc774\ub358 \uc2dc\ucd94\uacf5\uc5d0\uc11c \uc6d0\uc720\uac00 \ubd80\uadfc\uc758 \uba55\uc2dc\ucf54\ub9cc\uc73c\ub85c \ud758\ub7ec \ub4e4\uc5b4\uac14\uc73c\uba70, \ubbf8\uad6d \uc5ed\uc0ac\uc0c1 \ucd5c\uc545\uc758 \ud574\uc0c1 \uae30\ub984 \uc720\ucd9c \uc0ac\uace0\ub97c \uc77c\uc73c\ucf30\ub2e4. 2010\ub144 6\uc6d4 \ucd08 \ub610\ub294 5\uc6d4 \ub9d0\uc5d0 \ub098\uc628 \uc608\uc0c1\uce58\uc5d0 \ub530\ub974\uba74 \uc218\uc5b5 \uac24\ub7f0\uc758 \uc6d0\uc720\uac00 \ubc14\ub2e4\ub85c \ud758\ub7ec\ub4e4\uc5b4\uac14\ub2e4\uace0 \ud55c\ub2e4. \uc2dc\ucd94 \uc2dc\uc124\uc774 \ud3ed\ubc1c\ud558\uba70 11\uba85\uc758 \uc2dc\ucd94 \ub178\ub3d9\uc790\uac00 \uc0ac\ub9dd\ud588\uace0 18\uba85\uc774 \ubd80\uc0c1\ub2f9\ud588\ub2e4.", "answer": "BP", "sentence": "2010\ub144 04\uc6d4 20\uc77c, \ubbf8\uad6d \ub8e8\uc774\uc9c0\uc560\ub098 \uc8fc \uba55\uc2dc\ucf54 \ub9cc\uc5d0 \uc788\ub294 \uc5f0 \ub9e4\ucd9c 246\uc870\uc6d0\uc758 \uc601\uad6d \ucd5c\ub300 \uae30\uc5c5\uc774\uc790 \uc138\uacc4 2\uc704 \uc11d\uc720\ud68c\uc0ac\uc778 BP \uc758 \ud604\ub300\uc911\uacf5\uc5c5\uc774 \uc81c\uc870\ud55c \uc2dc\ucd94\uc120\uc778 \ub525\uc6cc\ud130 \ud638\ub77c\uc774\uc98c \uc11d\uc720 \uc2dc\ucd94 \uc2dc\uc124\uc774 \ud3ed\ubc1c\ud588\ub2e4.", "paragraph_sentence": " 2010\ub144 04\uc6d4 20\uc77c, \ubbf8\uad6d \ub8e8\uc774\uc9c0\uc560\ub098 \uc8fc \uba55\uc2dc\ucf54 \ub9cc\uc5d0 \uc788\ub294 \uc5f0 \ub9e4\ucd9c 246\uc870\uc6d0\uc758 \uc601\uad6d \ucd5c\ub300 \uae30\uc5c5\uc774\uc790 \uc138\uacc4 2\uc704 \uc11d\uc720\ud68c\uc0ac\uc778 BP \uc758 \ud604\ub300\uc911\uacf5\uc5c5\uc774 \uc81c\uc870\ud55c \uc2dc\ucd94\uc120\uc778 \ub525\uc6cc\ud130 \ud638\ub77c\uc774\uc98c \uc11d\uc720 \uc2dc\ucd94 \uc2dc\uc124\uc774 \ud3ed\ubc1c\ud588\ub2e4. \uac70\ub300\ud55c \uc2dc\ucd94\uc120\uc740 \ub77c\uc774\uc800\ub77c\uace0 \ubd80\ub974\ub294 \ud30c\uc774\ud504\ub85c \ud574\uc800\uc5d0 \uc5f0\uacb0\ub418\uc5b4 \uc788\uc5c8\ub2e4. 0.5\ubbf8\ud130 \ud3ed\uc5d0 1600\ubbf8\ud130 \uc815\ub3c4 \ub418\ub294 \uae38\uc774\uc600\ub2e4. \ud30c\uc774\ud504\ub294 \uc720\uc0ac\uc2dc \uc720\uc815\uc744 \ucc28\ub2e8\ud558\uae30 \uc704\ud574 \uace0\uc548\ub41c \uc6d0\uc720\ubd84\ucd9c \ubc29\uc9c0 \uc7a5\uce58 BOP(Blowout preventer)\uc5d0 \uc5f0\uacb0\ub418\uc5b4 \uc788\uc5c8\ub2e4. \ub2fb\uc774 \uc5c6\ub294 \uc2dc\ucd94\uc120\uc740 GPS\ub97c \uc774\uc6a9\ud574 \ud574\uc800\ub85c\ubd80\ud130 1500\ubbf8\ud130 \uc704\uc5d0 \uc788\uc5c8\ub2e4. \uc720\uc815\uc740 \uc6d0\uc720\ubd84\ucd9c \ubc29\uc9c0 \uc7a5\uce58 4000\ubbf8\ud130 \uc544\ub798\uc5d0 \uc788\uc5c8\ub2e4. \uac00\uc2a4\uac00 \uc720\uc815\uc5d0 \uc0c8\uc5b4\ub4e4\uc5b4\uac00\uba74\uc11c \ud3ed\ubc1c\uc774 \uc77c\uc5b4\ub0ac\uace0, 5500\ubbf8\ud130 \ub5a8\uc5b4\uc838 \uc788\ub294 \ud574\uc0c1 \uc2dc\ucd94\uc120\uc5d0 \ubd88\uc774 \ubd99\uc5c8\ub2e4. \uc2b9\ubb34\uc6d0\uc740 BOP\ub97c \uc791\ub3d9\uc2dc\ud0a4\ub824 \ud588\uc9c0\ub9cc \uc791\ub3d9\ud558\uc9c0 \uc54a\uc558\ub2e4. \uc6d0\uc720 \uc2dc\ucd94\uac00 \uc9c4\ud589 \uc911\uc774\ub358 \uc2dc\ucd94\uacf5\uc5d0\uc11c \uc6d0\uc720\uac00 \ubd80\uadfc\uc758 \uba55\uc2dc\ucf54\ub9cc\uc73c\ub85c \ud758\ub7ec \ub4e4\uc5b4\uac14\uc73c\uba70, \ubbf8\uad6d \uc5ed\uc0ac\uc0c1 \ucd5c\uc545\uc758 \ud574\uc0c1 \uae30\ub984 \uc720\ucd9c \uc0ac\uace0\ub97c \uc77c\uc73c\ucf30\ub2e4. 2010\ub144 6\uc6d4 \ucd08 \ub610\ub294 5\uc6d4 \ub9d0\uc5d0 \ub098\uc628 \uc608\uc0c1\uce58\uc5d0 \ub530\ub974\uba74 \uc218\uc5b5 \uac24\ub7f0\uc758 \uc6d0\uc720\uac00 \ubc14\ub2e4\ub85c \ud758\ub7ec\ub4e4\uc5b4\uac14\ub2e4\uace0 \ud55c\ub2e4. \uc2dc\ucd94 \uc2dc\uc124\uc774 \ud3ed\ubc1c\ud558\uba70 11\uba85\uc758 \uc2dc\ucd94 \ub178\ub3d9\uc790\uac00 \uc0ac\ub9dd\ud588\uace0 18\uba85\uc774 \ubd80\uc0c1\ub2f9\ud588\ub2e4.", "paragraph_answer": "2010\ub144 04\uc6d4 20\uc77c, \ubbf8\uad6d \ub8e8\uc774\uc9c0\uc560\ub098 \uc8fc \uba55\uc2dc\ucf54 \ub9cc\uc5d0 \uc788\ub294 \uc5f0 \ub9e4\ucd9c 246\uc870\uc6d0\uc758 \uc601\uad6d \ucd5c\ub300 \uae30\uc5c5\uc774\uc790 \uc138\uacc4 2\uc704 \uc11d\uc720\ud68c\uc0ac\uc778 BP \uc758 \ud604\ub300\uc911\uacf5\uc5c5\uc774 \uc81c\uc870\ud55c \uc2dc\ucd94\uc120\uc778 \ub525\uc6cc\ud130 \ud638\ub77c\uc774\uc98c \uc11d\uc720 \uc2dc\ucd94 \uc2dc\uc124\uc774 \ud3ed\ubc1c\ud588\ub2e4. \uac70\ub300\ud55c \uc2dc\ucd94\uc120\uc740 \ub77c\uc774\uc800\ub77c\uace0 \ubd80\ub974\ub294 \ud30c\uc774\ud504\ub85c \ud574\uc800\uc5d0 \uc5f0\uacb0\ub418\uc5b4 \uc788\uc5c8\ub2e4. 0.5\ubbf8\ud130 \ud3ed\uc5d0 1600\ubbf8\ud130 \uc815\ub3c4 \ub418\ub294 \uae38\uc774\uc600\ub2e4. \ud30c\uc774\ud504\ub294 \uc720\uc0ac\uc2dc \uc720\uc815\uc744 \ucc28\ub2e8\ud558\uae30 \uc704\ud574 \uace0\uc548\ub41c \uc6d0\uc720\ubd84\ucd9c \ubc29\uc9c0 \uc7a5\uce58 BOP(Blowout preventer)\uc5d0 \uc5f0\uacb0\ub418\uc5b4 \uc788\uc5c8\ub2e4. \ub2fb\uc774 \uc5c6\ub294 \uc2dc\ucd94\uc120\uc740 GPS\ub97c \uc774\uc6a9\ud574 \ud574\uc800\ub85c\ubd80\ud130 1500\ubbf8\ud130 \uc704\uc5d0 \uc788\uc5c8\ub2e4. \uc720\uc815\uc740 \uc6d0\uc720\ubd84\ucd9c \ubc29\uc9c0 \uc7a5\uce58 4000\ubbf8\ud130 \uc544\ub798\uc5d0 \uc788\uc5c8\ub2e4. \uac00\uc2a4\uac00 \uc720\uc815\uc5d0 \uc0c8\uc5b4\ub4e4\uc5b4\uac00\uba74\uc11c \ud3ed\ubc1c\uc774 \uc77c\uc5b4\ub0ac\uace0, 5500\ubbf8\ud130 \ub5a8\uc5b4\uc838 \uc788\ub294 \ud574\uc0c1 \uc2dc\ucd94\uc120\uc5d0 \ubd88\uc774 \ubd99\uc5c8\ub2e4. \uc2b9\ubb34\uc6d0\uc740 BOP\ub97c \uc791\ub3d9\uc2dc\ud0a4\ub824 \ud588\uc9c0\ub9cc \uc791\ub3d9\ud558\uc9c0 \uc54a\uc558\ub2e4. \uc6d0\uc720 \uc2dc\ucd94\uac00 \uc9c4\ud589 \uc911\uc774\ub358 \uc2dc\ucd94\uacf5\uc5d0\uc11c \uc6d0\uc720\uac00 \ubd80\uadfc\uc758 \uba55\uc2dc\ucf54\ub9cc\uc73c\ub85c \ud758\ub7ec \ub4e4\uc5b4\uac14\uc73c\uba70, \ubbf8\uad6d \uc5ed\uc0ac\uc0c1 \ucd5c\uc545\uc758 \ud574\uc0c1 \uae30\ub984 \uc720\ucd9c \uc0ac\uace0\ub97c \uc77c\uc73c\ucf30\ub2e4. 2010\ub144 6\uc6d4 \ucd08 \ub610\ub294 5\uc6d4 \ub9d0\uc5d0 \ub098\uc628 \uc608\uc0c1\uce58\uc5d0 \ub530\ub974\uba74 \uc218\uc5b5 \uac24\ub7f0\uc758 \uc6d0\uc720\uac00 \ubc14\ub2e4\ub85c \ud758\ub7ec\ub4e4\uc5b4\uac14\ub2e4\uace0 \ud55c\ub2e4. \uc2dc\ucd94 \uc2dc\uc124\uc774 \ud3ed\ubc1c\ud558\uba70 11\uba85\uc758 \uc2dc\ucd94 \ub178\ub3d9\uc790\uac00 \uc0ac\ub9dd\ud588\uace0 18\uba85\uc774 \ubd80\uc0c1\ub2f9\ud588\ub2e4.", "sentence_answer": "2010\ub144 04\uc6d4 20\uc77c, \ubbf8\uad6d \ub8e8\uc774\uc9c0\uc560\ub098 \uc8fc \uba55\uc2dc\ucf54 \ub9cc\uc5d0 \uc788\ub294 \uc5f0 \ub9e4\ucd9c 246\uc870\uc6d0\uc758 \uc601\uad6d \ucd5c\ub300 \uae30\uc5c5\uc774\uc790 \uc138\uacc4 2\uc704 \uc11d\uc720\ud68c\uc0ac\uc778 BP \uc758 \ud604\ub300\uc911\uacf5\uc5c5\uc774 \uc81c\uc870\ud55c \uc2dc\ucd94\uc120\uc778 \ub525\uc6cc\ud130 \ud638\ub77c\uc774\uc98c \uc11d\uc720 \uc2dc\ucd94 \uc2dc\uc124\uc774 \ud3ed\ubc1c\ud588\ub2e4.", "paragraph_id": "6300420-0-0", "paragraph_question": "question: 2010\ub144\uc5d0 \uba55\uc2dc\ucf54 \ub9cc\uc5d0 \uc704\uce58\ud55c \uc601\uad6d\uc758 \ucd5c\ub300 \uae30\uc5c5\uc778 \uc138\uacc4 2\uc704 \uc11d\uc720\ud68c\uc0ac\ub294?, context: 2010\ub144 04\uc6d4 20\uc77c, \ubbf8\uad6d \ub8e8\uc774\uc9c0\uc560\ub098 \uc8fc \uba55\uc2dc\ucf54 \ub9cc\uc5d0 \uc788\ub294 \uc5f0 \ub9e4\ucd9c 246\uc870\uc6d0\uc758 \uc601\uad6d \ucd5c\ub300 \uae30\uc5c5\uc774\uc790 \uc138\uacc4 2\uc704 \uc11d\uc720\ud68c\uc0ac\uc778 BP\uc758 \ud604\ub300\uc911\uacf5\uc5c5\uc774 \uc81c\uc870\ud55c \uc2dc\ucd94\uc120\uc778 \ub525\uc6cc\ud130 \ud638\ub77c\uc774\uc98c \uc11d\uc720 \uc2dc\ucd94 \uc2dc\uc124\uc774 \ud3ed\ubc1c\ud588\ub2e4. \uac70\ub300\ud55c \uc2dc\ucd94\uc120\uc740 \ub77c\uc774\uc800\ub77c\uace0 \ubd80\ub974\ub294 \ud30c\uc774\ud504\ub85c \ud574\uc800\uc5d0 \uc5f0\uacb0\ub418\uc5b4 \uc788\uc5c8\ub2e4. 0.5\ubbf8\ud130 \ud3ed\uc5d0 1600\ubbf8\ud130 \uc815\ub3c4 \ub418\ub294 \uae38\uc774\uc600\ub2e4. \ud30c\uc774\ud504\ub294 \uc720\uc0ac\uc2dc \uc720\uc815\uc744 \ucc28\ub2e8\ud558\uae30 \uc704\ud574 \uace0\uc548\ub41c \uc6d0\uc720\ubd84\ucd9c \ubc29\uc9c0 \uc7a5\uce58 BOP(Blowout preventer)\uc5d0 \uc5f0\uacb0\ub418\uc5b4 \uc788\uc5c8\ub2e4. \ub2fb\uc774 \uc5c6\ub294 \uc2dc\ucd94\uc120\uc740 GPS\ub97c \uc774\uc6a9\ud574 \ud574\uc800\ub85c\ubd80\ud130 1500\ubbf8\ud130 \uc704\uc5d0 \uc788\uc5c8\ub2e4. \uc720\uc815\uc740 \uc6d0\uc720\ubd84\ucd9c \ubc29\uc9c0 \uc7a5\uce58 4000\ubbf8\ud130 \uc544\ub798\uc5d0 \uc788\uc5c8\ub2e4. \uac00\uc2a4\uac00 \uc720\uc815\uc5d0 \uc0c8\uc5b4\ub4e4\uc5b4\uac00\uba74\uc11c \ud3ed\ubc1c\uc774 \uc77c\uc5b4\ub0ac\uace0, 5500\ubbf8\ud130 \ub5a8\uc5b4\uc838 \uc788\ub294 \ud574\uc0c1 \uc2dc\ucd94\uc120\uc5d0 \ubd88\uc774 \ubd99\uc5c8\ub2e4. \uc2b9\ubb34\uc6d0\uc740 BOP\ub97c \uc791\ub3d9\uc2dc\ud0a4\ub824 \ud588\uc9c0\ub9cc \uc791\ub3d9\ud558\uc9c0 \uc54a\uc558\ub2e4. \uc6d0\uc720 \uc2dc\ucd94\uac00 \uc9c4\ud589 \uc911\uc774\ub358 \uc2dc\ucd94\uacf5\uc5d0\uc11c \uc6d0\uc720\uac00 \ubd80\uadfc\uc758 \uba55\uc2dc\ucf54\ub9cc\uc73c\ub85c \ud758\ub7ec \ub4e4\uc5b4\uac14\uc73c\uba70, \ubbf8\uad6d \uc5ed\uc0ac\uc0c1 \ucd5c\uc545\uc758 \ud574\uc0c1 \uae30\ub984 \uc720\ucd9c \uc0ac\uace0\ub97c \uc77c\uc73c\ucf30\ub2e4. 2010\ub144 6\uc6d4 \ucd08 \ub610\ub294 5\uc6d4 \ub9d0\uc5d0 \ub098\uc628 \uc608\uc0c1\uce58\uc5d0 \ub530\ub974\uba74 \uc218\uc5b5 \uac24\ub7f0\uc758 \uc6d0\uc720\uac00 \ubc14\ub2e4\ub85c \ud758\ub7ec\ub4e4\uc5b4\uac14\ub2e4\uace0 \ud55c\ub2e4. \uc2dc\ucd94 \uc2dc\uc124\uc774 \ud3ed\ubc1c\ud558\uba70 11\uba85\uc758 \uc2dc\ucd94 \ub178\ub3d9\uc790\uac00 \uc0ac\ub9dd\ud588\uace0 18\uba85\uc774 \ubd80\uc0c1\ub2f9\ud588\ub2e4."} {"question": "\uc81c19\ub300 \ub300\ud1b5\ub839 \uc120\uac70\uc5d0\uc11c \ubc14\ub978\uc815\ub2f9 \ud6c4\ubcf4\ub85c \uc120\ucd9c\ub41c \uc758\uc6d0\uc740?", "paragraph": "\ubc14\ub978\uc815\ub2f9\uc740 2017\ub144 3\uc6d4 16\uc77c\ubd80\ud130 3\uc6d4 17\uc77c\uae4c\uc9c0 \ub300\ud1b5\ub839 \uc120\uac70 \uc608\ube44\ud6c4\ubcf4 \ub4f1\ub85d\uc744 \uc9c4\ud589\ud558\uc600\ub2e4. \ucd5c\uc885\uc801\uc73c\ub85c \uc720\uc2b9\ubbfc \uc758\uc6d0\uacfc \ub0a8\uacbd\ud544 \uacbd\uae30\ub3c4\uc9c0\uc0ac\uac00 \uc785\ud6c4\ubcf4\ud558\uc5ec 2\ud30c\uc804\uc774 \ub418\uc5c8\ub2e4. \uc774\ud6c4 \ud6c4\ubcf4\ud1a0\ub860\ud68c \uc77c\uc815\uc744 \uac1c\uc2dc\ud558\uc5ec, 3\uc6d4 18\uc77c \uad11\uc8fc\uc5d0\uc11c \ud638\ub0a8\uad8c \ud1a0\ub860\ud68c, 3\uc6d4 21\uc77c\uc5d0\ub294 \ubd80\uc0b0\uc5d0\uc11c \uc601\ub0a8\uad8c \ud1a0\ub860\ud68c\ub97c \uac1c\ucd5c\ud558\uc600\ub2e4. 3\uc6d4 23\uc77c\uc5d0\ub294 \ub300\uc804\uc5d0\uc11c \ucda9\uccad\u00b7\uac15\uc6d0\uad8c \ud1a0\ub860\ud68c\ub97c, 3\uc6d4 25\uc77c\uc5d0\ub294 \uc11c\uc6b8\uc5d0\uc11c \uc218\ub3c4\uad8c \ud1a0\ub860\ud68c\ub97c \uc9c4\ud589\ud558\uc600\ub2e4. \ud638\ub0a8\uad8c \ud1a0\ub860\ud68c\uc640 \uc218\ub3c4\uad8c \ud1a0\ub860\ud68c\ub294 \uac01\uac01 \uad11\uc8fcMBC\uc640 KBS\uac00 \ud154\ub808\ube44\uc804 \ub179\ud654\u00b7\uc0dd\uc911\uacc4\ub97c \uc9c4\ud589\ud558\uc600\ub2e4. \uc774\ud6c4 2017\ub144 3\uc6d4 28\uc77c, \ubc14\ub978\uc815\ub2f9\uc740 \uad6d\ubbfc\uc815\ucc45\ud3c9\uac00\ub2e8 \uc804\ud654\uba74\uc811\ud22c\ud45c 40%, \uc804\ub2f9\uc6d0 \ud22c\ud45c 30%, \uc77c\ubc18\uad6d\ubbfc \uc5ec\ub860\uc870\uc0ac 30%\ub97c \ubc18\uc601\ud55c \uacbd\uc120 \ud22c\ud45c \uacb0\uacfc, \uc720\uc2b9\ubbfc \ubc14\ub978\uc815\ub2f9 \uc0c1\uc784\uace0\ubb38\uc774 36,593\ud45c (62.9%)\ub97c \ucc28\uc9c0\ud574 \ub0a8\uacbd\ud544 \uacbd\uae30\ub3c4\uc9c0\uc0ac (21,625\ud45c, 37.1%)\ub97c \ub204\ub974\uace0 \ub300\ud55c\ubbfc\uad6d \uc81c19\ub300 \ub300\ud1b5\ub839 \uc120\uac70 \ud6c4\ubcf4\uc5d0 \uc120\ucd9c\ub418\uc5c8\ub2e4. \uc720\uc2b9\ubbfc \ud6c4\ubcf4\ub294 \uc218\ub77d\uc5f0\uc124\uc5d0\uc11c \"\uc0c8\ub85c\uc6b4 \ubcf4\uc218\uc758 \ud76c\ub9dd\uc774 \ub418\uaca0\ub2e4\"\uba74\uc11c \"\ubcf4\uc218\uc758 \uc7ac\uac74\uc744 \ubc14\ub77c\ub294 \uad6d\ubbfc\uc758 \uc5ec\ub9dd\uc744 \ubaa8\uc544 \uc774\ubc88 \ub300\ud1b5\ub839\uc120\uac70\uc5d0\uc11c \ub2f9\ub2f9\ud558\uac8c \uad6d\ubbfc\uc758 \uc120\ud0dd\uc744 \ubc1b\uaca0\ub2e4\"\uace0 \ubc1d\ud614\ub2e4.", "answer": "\uc720\uc2b9\ubbfc", "sentence": "\ucd5c\uc885\uc801\uc73c\ub85c \uc720\uc2b9\ubbfc \uc758\uc6d0\uacfc \ub0a8\uacbd\ud544 \uacbd\uae30\ub3c4\uc9c0\uc0ac\uac00 \uc785\ud6c4\ubcf4\ud558\uc5ec 2\ud30c\uc804\uc774 \ub418\uc5c8\ub2e4.", "paragraph_sentence": "\ubc14\ub978\uc815\ub2f9\uc740 2017\ub144 3\uc6d4 16\uc77c\ubd80\ud130 3\uc6d4 17\uc77c\uae4c\uc9c0 \ub300\ud1b5\ub839 \uc120\uac70 \uc608\ube44\ud6c4\ubcf4 \ub4f1\ub85d\uc744 \uc9c4\ud589\ud558\uc600\ub2e4. \ucd5c\uc885\uc801\uc73c\ub85c \uc720\uc2b9\ubbfc \uc758\uc6d0\uacfc \ub0a8\uacbd\ud544 \uacbd\uae30\ub3c4\uc9c0\uc0ac\uac00 \uc785\ud6c4\ubcf4\ud558\uc5ec 2\ud30c\uc804\uc774 \ub418\uc5c8\ub2e4. \uc774\ud6c4 \ud6c4\ubcf4\ud1a0\ub860\ud68c \uc77c\uc815\uc744 \uac1c\uc2dc\ud558\uc5ec, 3\uc6d4 18\uc77c \uad11\uc8fc\uc5d0\uc11c \ud638\ub0a8\uad8c \ud1a0\ub860\ud68c, 3\uc6d4 21\uc77c\uc5d0\ub294 \ubd80\uc0b0\uc5d0\uc11c \uc601\ub0a8\uad8c \ud1a0\ub860\ud68c\ub97c \uac1c\ucd5c\ud558\uc600\ub2e4. 3\uc6d4 23\uc77c\uc5d0\ub294 \ub300\uc804\uc5d0\uc11c \ucda9\uccad\u00b7\uac15\uc6d0\uad8c \ud1a0\ub860\ud68c\ub97c, 3\uc6d4 25\uc77c\uc5d0\ub294 \uc11c\uc6b8\uc5d0\uc11c \uc218\ub3c4\uad8c \ud1a0\ub860\ud68c\ub97c \uc9c4\ud589\ud558\uc600\ub2e4. \ud638\ub0a8\uad8c \ud1a0\ub860\ud68c\uc640 \uc218\ub3c4\uad8c \ud1a0\ub860\ud68c\ub294 \uac01\uac01 \uad11\uc8fcMBC\uc640 KBS\uac00 \ud154\ub808\ube44\uc804 \ub179\ud654\u00b7\uc0dd\uc911\uacc4\ub97c \uc9c4\ud589\ud558\uc600\ub2e4. \uc774\ud6c4 2017\ub144 3\uc6d4 28\uc77c, \ubc14\ub978\uc815\ub2f9\uc740 \uad6d\ubbfc\uc815\ucc45\ud3c9\uac00\ub2e8 \uc804\ud654\uba74\uc811\ud22c\ud45c 40%, \uc804\ub2f9\uc6d0 \ud22c\ud45c 30%, \uc77c\ubc18\uad6d\ubbfc \uc5ec\ub860\uc870\uc0ac 30%\ub97c \ubc18\uc601\ud55c \uacbd\uc120 \ud22c\ud45c \uacb0\uacfc, \uc720\uc2b9\ubbfc \ubc14\ub978\uc815\ub2f9 \uc0c1\uc784\uace0\ubb38\uc774 36,593\ud45c (62.9%)\ub97c \ucc28\uc9c0\ud574 \ub0a8\uacbd\ud544 \uacbd\uae30\ub3c4\uc9c0\uc0ac (21,625\ud45c, 37.1%)\ub97c \ub204\ub974\uace0 \ub300\ud55c\ubbfc\uad6d \uc81c19\ub300 \ub300\ud1b5\ub839 \uc120\uac70 \ud6c4\ubcf4\uc5d0 \uc120\ucd9c\ub418\uc5c8\ub2e4. \uc720\uc2b9\ubbfc \ud6c4\ubcf4\ub294 \uc218\ub77d\uc5f0\uc124\uc5d0\uc11c \"\uc0c8\ub85c\uc6b4 \ubcf4\uc218\uc758 \ud76c\ub9dd\uc774 \ub418\uaca0\ub2e4\"\uba74\uc11c \"\ubcf4\uc218\uc758 \uc7ac\uac74\uc744 \ubc14\ub77c\ub294 \uad6d\ubbfc\uc758 \uc5ec\ub9dd\uc744 \ubaa8\uc544 \uc774\ubc88 \ub300\ud1b5\ub839\uc120\uac70\uc5d0\uc11c \ub2f9\ub2f9\ud558\uac8c \uad6d\ubbfc\uc758 \uc120\ud0dd\uc744 \ubc1b\uaca0\ub2e4\"\uace0 \ubc1d\ud614\ub2e4.", "paragraph_answer": "\ubc14\ub978\uc815\ub2f9\uc740 2017\ub144 3\uc6d4 16\uc77c\ubd80\ud130 3\uc6d4 17\uc77c\uae4c\uc9c0 \ub300\ud1b5\ub839 \uc120\uac70 \uc608\ube44\ud6c4\ubcf4 \ub4f1\ub85d\uc744 \uc9c4\ud589\ud558\uc600\ub2e4. \ucd5c\uc885\uc801\uc73c\ub85c \uc720\uc2b9\ubbfc \uc758\uc6d0\uacfc \ub0a8\uacbd\ud544 \uacbd\uae30\ub3c4\uc9c0\uc0ac\uac00 \uc785\ud6c4\ubcf4\ud558\uc5ec 2\ud30c\uc804\uc774 \ub418\uc5c8\ub2e4. \uc774\ud6c4 \ud6c4\ubcf4\ud1a0\ub860\ud68c \uc77c\uc815\uc744 \uac1c\uc2dc\ud558\uc5ec, 3\uc6d4 18\uc77c \uad11\uc8fc\uc5d0\uc11c \ud638\ub0a8\uad8c \ud1a0\ub860\ud68c, 3\uc6d4 21\uc77c\uc5d0\ub294 \ubd80\uc0b0\uc5d0\uc11c \uc601\ub0a8\uad8c \ud1a0\ub860\ud68c\ub97c \uac1c\ucd5c\ud558\uc600\ub2e4. 3\uc6d4 23\uc77c\uc5d0\ub294 \ub300\uc804\uc5d0\uc11c \ucda9\uccad\u00b7\uac15\uc6d0\uad8c \ud1a0\ub860\ud68c\ub97c, 3\uc6d4 25\uc77c\uc5d0\ub294 \uc11c\uc6b8\uc5d0\uc11c \uc218\ub3c4\uad8c \ud1a0\ub860\ud68c\ub97c \uc9c4\ud589\ud558\uc600\ub2e4. \ud638\ub0a8\uad8c \ud1a0\ub860\ud68c\uc640 \uc218\ub3c4\uad8c \ud1a0\ub860\ud68c\ub294 \uac01\uac01 \uad11\uc8fcMBC\uc640 KBS\uac00 \ud154\ub808\ube44\uc804 \ub179\ud654\u00b7\uc0dd\uc911\uacc4\ub97c \uc9c4\ud589\ud558\uc600\ub2e4. \uc774\ud6c4 2017\ub144 3\uc6d4 28\uc77c, \ubc14\ub978\uc815\ub2f9\uc740 \uad6d\ubbfc\uc815\ucc45\ud3c9\uac00\ub2e8 \uc804\ud654\uba74\uc811\ud22c\ud45c 40%, \uc804\ub2f9\uc6d0 \ud22c\ud45c 30%, \uc77c\ubc18\uad6d\ubbfc \uc5ec\ub860\uc870\uc0ac 30%\ub97c \ubc18\uc601\ud55c \uacbd\uc120 \ud22c\ud45c \uacb0\uacfc, \uc720\uc2b9\ubbfc \ubc14\ub978\uc815\ub2f9 \uc0c1\uc784\uace0\ubb38\uc774 36,593\ud45c (62.9%)\ub97c \ucc28\uc9c0\ud574 \ub0a8\uacbd\ud544 \uacbd\uae30\ub3c4\uc9c0\uc0ac (21,625\ud45c, 37.1%)\ub97c \ub204\ub974\uace0 \ub300\ud55c\ubbfc\uad6d \uc81c19\ub300 \ub300\ud1b5\ub839 \uc120\uac70 \ud6c4\ubcf4\uc5d0 \uc120\ucd9c\ub418\uc5c8\ub2e4. \uc720\uc2b9\ubbfc \ud6c4\ubcf4\ub294 \uc218\ub77d\uc5f0\uc124\uc5d0\uc11c \"\uc0c8\ub85c\uc6b4 \ubcf4\uc218\uc758 \ud76c\ub9dd\uc774 \ub418\uaca0\ub2e4\"\uba74\uc11c \"\ubcf4\uc218\uc758 \uc7ac\uac74\uc744 \ubc14\ub77c\ub294 \uad6d\ubbfc\uc758 \uc5ec\ub9dd\uc744 \ubaa8\uc544 \uc774\ubc88 \ub300\ud1b5\ub839\uc120\uac70\uc5d0\uc11c \ub2f9\ub2f9\ud558\uac8c \uad6d\ubbfc\uc758 \uc120\ud0dd\uc744 \ubc1b\uaca0\ub2e4\"\uace0 \ubc1d\ud614\ub2e4.", "sentence_answer": "\ucd5c\uc885\uc801\uc73c\ub85c \uc720\uc2b9\ubbfc \uc758\uc6d0\uacfc \ub0a8\uacbd\ud544 \uacbd\uae30\ub3c4\uc9c0\uc0ac\uac00 \uc785\ud6c4\ubcf4\ud558\uc5ec 2\ud30c\uc804\uc774 \ub418\uc5c8\ub2e4.", "paragraph_id": "6490942-4-0", "paragraph_question": "question: \uc81c19\ub300 \ub300\ud1b5\ub839 \uc120\uac70\uc5d0\uc11c \ubc14\ub978\uc815\ub2f9 \ud6c4\ubcf4\ub85c \uc120\ucd9c\ub41c \uc758\uc6d0\uc740?, context: \ubc14\ub978\uc815\ub2f9\uc740 2017\ub144 3\uc6d4 16\uc77c\ubd80\ud130 3\uc6d4 17\uc77c\uae4c\uc9c0 \ub300\ud1b5\ub839 \uc120\uac70 \uc608\ube44\ud6c4\ubcf4 \ub4f1\ub85d\uc744 \uc9c4\ud589\ud558\uc600\ub2e4. \ucd5c\uc885\uc801\uc73c\ub85c \uc720\uc2b9\ubbfc \uc758\uc6d0\uacfc \ub0a8\uacbd\ud544 \uacbd\uae30\ub3c4\uc9c0\uc0ac\uac00 \uc785\ud6c4\ubcf4\ud558\uc5ec 2\ud30c\uc804\uc774 \ub418\uc5c8\ub2e4. \uc774\ud6c4 \ud6c4\ubcf4\ud1a0\ub860\ud68c \uc77c\uc815\uc744 \uac1c\uc2dc\ud558\uc5ec, 3\uc6d4 18\uc77c \uad11\uc8fc\uc5d0\uc11c \ud638\ub0a8\uad8c \ud1a0\ub860\ud68c, 3\uc6d4 21\uc77c\uc5d0\ub294 \ubd80\uc0b0\uc5d0\uc11c \uc601\ub0a8\uad8c \ud1a0\ub860\ud68c\ub97c \uac1c\ucd5c\ud558\uc600\ub2e4. 3\uc6d4 23\uc77c\uc5d0\ub294 \ub300\uc804\uc5d0\uc11c \ucda9\uccad\u00b7\uac15\uc6d0\uad8c \ud1a0\ub860\ud68c\ub97c, 3\uc6d4 25\uc77c\uc5d0\ub294 \uc11c\uc6b8\uc5d0\uc11c \uc218\ub3c4\uad8c \ud1a0\ub860\ud68c\ub97c \uc9c4\ud589\ud558\uc600\ub2e4. \ud638\ub0a8\uad8c \ud1a0\ub860\ud68c\uc640 \uc218\ub3c4\uad8c \ud1a0\ub860\ud68c\ub294 \uac01\uac01 \uad11\uc8fcMBC\uc640 KBS\uac00 \ud154\ub808\ube44\uc804 \ub179\ud654\u00b7\uc0dd\uc911\uacc4\ub97c \uc9c4\ud589\ud558\uc600\ub2e4. \uc774\ud6c4 2017\ub144 3\uc6d4 28\uc77c, \ubc14\ub978\uc815\ub2f9\uc740 \uad6d\ubbfc\uc815\ucc45\ud3c9\uac00\ub2e8 \uc804\ud654\uba74\uc811\ud22c\ud45c 40%, \uc804\ub2f9\uc6d0 \ud22c\ud45c 30%, \uc77c\ubc18\uad6d\ubbfc \uc5ec\ub860\uc870\uc0ac 30%\ub97c \ubc18\uc601\ud55c \uacbd\uc120 \ud22c\ud45c \uacb0\uacfc, \uc720\uc2b9\ubbfc \ubc14\ub978\uc815\ub2f9 \uc0c1\uc784\uace0\ubb38\uc774 36,593\ud45c (62.9%)\ub97c \ucc28\uc9c0\ud574 \ub0a8\uacbd\ud544 \uacbd\uae30\ub3c4\uc9c0\uc0ac (21,625\ud45c, 37.1%)\ub97c \ub204\ub974\uace0 \ub300\ud55c\ubbfc\uad6d \uc81c19\ub300 \ub300\ud1b5\ub839 \uc120\uac70 \ud6c4\ubcf4\uc5d0 \uc120\ucd9c\ub418\uc5c8\ub2e4. \uc720\uc2b9\ubbfc \ud6c4\ubcf4\ub294 \uc218\ub77d\uc5f0\uc124\uc5d0\uc11c \"\uc0c8\ub85c\uc6b4 \ubcf4\uc218\uc758 \ud76c\ub9dd\uc774 \ub418\uaca0\ub2e4\"\uba74\uc11c \"\ubcf4\uc218\uc758 \uc7ac\uac74\uc744 \ubc14\ub77c\ub294 \uad6d\ubbfc\uc758 \uc5ec\ub9dd\uc744 \ubaa8\uc544 \uc774\ubc88 \ub300\ud1b5\ub839\uc120\uac70\uc5d0\uc11c \ub2f9\ub2f9\ud558\uac8c \uad6d\ubbfc\uc758 \uc120\ud0dd\uc744 \ubc1b\uaca0\ub2e4\"\uace0 \ubc1d\ud614\ub2e4."} {"question": "\uc5b4\ub5a4 \uc694\uc778\uc740 \ub3d9\uc131\uc560 \ud589\ub3d9\uc5d0 \uc788\uc5b4\uc11c \ub0a8\uc131\uc5d0\uac8c\ub294 \uc804\ud600 \uc0c1\uad00\uad00\uacc4\uac00 \uc5c6\uace0 \uc5ec\uc131\uc5d0\uac8c\ub294 \uc57d\uac04\uc758 \uc0c1\uad00\uad00\uacc4\uac00 \uc788\ub294\uac00?", "paragraph": "\uac1c\uc778\uc774 \ud2b9\uc815\ud55c \uc131\uc801 \uc9c0\ud5a5\uc744 \ubc1c\ub2ec\ud558\uac8c \ub418\ub294 \uc774\uc720\ub294 \uc544\uc9c1 \uacfc\ud559\uc790\ub4e4 \uc0ac\uc774\uc5d0 \ud569\uc758\ub97c \uc774\ub8e8\uc9c0 \ubabb\ud588\ub2e4. \uadf8\ub7ec\ub098 \uc804\ubb38\uac00\ub4e4 \uc0ac\uc774\uc5d0\uc11c\ub294 \uc720\uc804\uc790, \ucd08\uae30 \uc790\uad81 \ub0b4 \ud658\uacbd, \ub610\ub294 \uadf8 \ub458\uc758 \uc870\ud569\uc73c\ub85c \uc124\uba85\ub418\ub294 \uc0dd\ubb3c\ud559\uc801\uc778 \uc774\ub860\ub4e4\uc774 \uc8fc\ub958\ub97c \uc774\ub8ec\ub2e4. \ubd80\ubaa8\uc758 \uc721\uc544 \ubc29\uc2dd\uc774\ub098 \uc720\uc544 \ub54c\uc758 \uacbd\ud5d8\uc774 \uc131\uc801 \uc9c0\ud5a5\uc5d0 \uc601\ud5a5\uc744 \uc900\ub2e4\ub294 \ucda9\ubd84\ud55c \uadfc\uac70\ub294 \uc874\uc7ac\ud558\uc9c0 \uc54a\ub294\ub2e4. \uac00\uc871 \ud658\uacbd\uc740 \ub3d9\uc131\uc560 \ud589\ub3d9\uc5d0 \uc788\uc5b4\uc11c \ub0a8\uc131\uc5d0\uac8c\ub294 \uc804\ud600 \uc601\ud5a5\uc744 \uc8fc\uc9c0 \uc54a\uc558\uace0 \uc5ec\uc131\uc5d0\uac8c\ub294 \ubbf8\uc138\ud55c \uc601\ud5a5\ub9cc\uc744 \uc8fc\uc5c8\ub2e4. \ub3d9\uc131\uc560\uac00 \"\uc790\uc5f0\uc2a4\ub7fd\uc9c0 \uc54a\ub2e4\"\uac70\ub098 \"\ubb38\uc81c\uac00 \uc788\ub2e4\"\ub294 \uacac\ud574\ub97c \uac16\uace0 \uc788\ub294 \uc0ac\ub78c\ub3c4 \uc788\uc9c0\ub9cc, \uc5f0\uad6c \uacb0\uacfc, \ub3d9\uc131\uc560\ub294 \uc815\uc0c1\uc801\uc774\uba70 \uc790\uc5f0\uc2a4\ub7ec\uc6b4 \ucc28\uc774\uc810\uc77c \ubfd0 \ubd80\uc815\uc801\uc778 \uc2ec\ub9ac\uc758 \uc6d0\uc778\uc774 \ub418\uc9c0 \uc54a\ub294\ub2e4\ub294 \uac83\uc774 \ubc1d\ud600\uc84c\ub2e4. \ub610\ud55c \ub3d9\uc131\uc560 \uad00\uacc4\ub294 \uc2ec\ub9ac\ud559\uc801\uc778 \uce21\uba74\uc5d0\uc11c \uc774\uc131\uc560 \uad00\uacc4\uc640 \ub3d9\ub4f1\ud558\ub2e4. \ub300\ubd80\ubd84\uc758 \uc0ac\ub78c\ub4e4\uc740 \uc2a4\uc2a4\ub85c \uc131\uc801 \uc9c0\ud5a5\uc744 '\uc120\ud0dd'\ud558\ub294 \uacbd\ud5d8\uc744 \ud558\uc9c0 \uc54a\uc73c\uba70, \ud0c0\uc778\uc774 \uac1c\uc785\ud574\uc11c \uc131\uc801 \uc9c0\ud5a5\uc744 \ubc14\uafc0 \uc218 \uc788\ub2e4\ub294 \uc99d\uac70\ub294 \ucda9\ubd84\ud558\uc9c0 \uc54a\ub2e4.", "answer": "\uac00\uc871 \ud658\uacbd", "sentence": "\uac00\uc871 \ud658\uacbd \uc740 \ub3d9\uc131\uc560 \ud589\ub3d9\uc5d0 \uc788\uc5b4\uc11c \ub0a8\uc131\uc5d0\uac8c\ub294 \uc804\ud600 \uc601\ud5a5\uc744 \uc8fc\uc9c0 \uc54a\uc558\uace0 \uc5ec\uc131\uc5d0\uac8c\ub294 \ubbf8\uc138\ud55c \uc601\ud5a5\ub9cc\uc744 \uc8fc\uc5c8\ub2e4.", "paragraph_sentence": "\uac1c\uc778\uc774 \ud2b9\uc815\ud55c \uc131\uc801 \uc9c0\ud5a5\uc744 \ubc1c\ub2ec\ud558\uac8c \ub418\ub294 \uc774\uc720\ub294 \uc544\uc9c1 \uacfc\ud559\uc790\ub4e4 \uc0ac\uc774\uc5d0 \ud569\uc758\ub97c \uc774\ub8e8\uc9c0 \ubabb\ud588\ub2e4. \uadf8\ub7ec\ub098 \uc804\ubb38\uac00\ub4e4 \uc0ac\uc774\uc5d0\uc11c\ub294 \uc720\uc804\uc790, \ucd08\uae30 \uc790\uad81 \ub0b4 \ud658\uacbd, \ub610\ub294 \uadf8 \ub458\uc758 \uc870\ud569\uc73c\ub85c \uc124\uba85\ub418\ub294 \uc0dd\ubb3c\ud559\uc801\uc778 \uc774\ub860\ub4e4\uc774 \uc8fc\ub958\ub97c \uc774\ub8ec\ub2e4. \ubd80\ubaa8\uc758 \uc721\uc544 \ubc29\uc2dd\uc774\ub098 \uc720\uc544 \ub54c\uc758 \uacbd\ud5d8\uc774 \uc131\uc801 \uc9c0\ud5a5\uc5d0 \uc601\ud5a5\uc744 \uc900\ub2e4\ub294 \ucda9\ubd84\ud55c \uadfc\uac70\ub294 \uc874\uc7ac\ud558\uc9c0 \uc54a\ub294\ub2e4. \uac00\uc871 \ud658\uacbd \uc740 \ub3d9\uc131\uc560 \ud589\ub3d9\uc5d0 \uc788\uc5b4\uc11c \ub0a8\uc131\uc5d0\uac8c\ub294 \uc804\ud600 \uc601\ud5a5\uc744 \uc8fc\uc9c0 \uc54a\uc558\uace0 \uc5ec\uc131\uc5d0\uac8c\ub294 \ubbf8\uc138\ud55c \uc601\ud5a5\ub9cc\uc744 \uc8fc\uc5c8\ub2e4. \ub3d9\uc131\uc560\uac00 \"\uc790\uc5f0\uc2a4\ub7fd\uc9c0 \uc54a\ub2e4\"\uac70\ub098 \"\ubb38\uc81c\uac00 \uc788\ub2e4\"\ub294 \uacac\ud574\ub97c \uac16\uace0 \uc788\ub294 \uc0ac\ub78c\ub3c4 \uc788\uc9c0\ub9cc, \uc5f0\uad6c \uacb0\uacfc, \ub3d9\uc131\uc560\ub294 \uc815\uc0c1\uc801\uc774\uba70 \uc790\uc5f0\uc2a4\ub7ec\uc6b4 \ucc28\uc774\uc810\uc77c \ubfd0 \ubd80\uc815\uc801\uc778 \uc2ec\ub9ac\uc758 \uc6d0\uc778\uc774 \ub418\uc9c0 \uc54a\ub294\ub2e4\ub294 \uac83\uc774 \ubc1d\ud600\uc84c\ub2e4. \ub610\ud55c \ub3d9\uc131\uc560 \uad00\uacc4\ub294 \uc2ec\ub9ac\ud559\uc801\uc778 \uce21\uba74\uc5d0\uc11c \uc774\uc131\uc560 \uad00\uacc4\uc640 \ub3d9\ub4f1\ud558\ub2e4. \ub300\ubd80\ubd84\uc758 \uc0ac\ub78c\ub4e4\uc740 \uc2a4\uc2a4\ub85c \uc131\uc801 \uc9c0\ud5a5\uc744 '\uc120\ud0dd'\ud558\ub294 \uacbd\ud5d8\uc744 \ud558\uc9c0 \uc54a\uc73c\uba70, \ud0c0\uc778\uc774 \uac1c\uc785\ud574\uc11c \uc131\uc801 \uc9c0\ud5a5\uc744 \ubc14\uafc0 \uc218 \uc788\ub2e4\ub294 \uc99d\uac70\ub294 \ucda9\ubd84\ud558\uc9c0 \uc54a\ub2e4.", "paragraph_answer": "\uac1c\uc778\uc774 \ud2b9\uc815\ud55c \uc131\uc801 \uc9c0\ud5a5\uc744 \ubc1c\ub2ec\ud558\uac8c \ub418\ub294 \uc774\uc720\ub294 \uc544\uc9c1 \uacfc\ud559\uc790\ub4e4 \uc0ac\uc774\uc5d0 \ud569\uc758\ub97c \uc774\ub8e8\uc9c0 \ubabb\ud588\ub2e4. \uadf8\ub7ec\ub098 \uc804\ubb38\uac00\ub4e4 \uc0ac\uc774\uc5d0\uc11c\ub294 \uc720\uc804\uc790, \ucd08\uae30 \uc790\uad81 \ub0b4 \ud658\uacbd, \ub610\ub294 \uadf8 \ub458\uc758 \uc870\ud569\uc73c\ub85c \uc124\uba85\ub418\ub294 \uc0dd\ubb3c\ud559\uc801\uc778 \uc774\ub860\ub4e4\uc774 \uc8fc\ub958\ub97c \uc774\ub8ec\ub2e4. \ubd80\ubaa8\uc758 \uc721\uc544 \ubc29\uc2dd\uc774\ub098 \uc720\uc544 \ub54c\uc758 \uacbd\ud5d8\uc774 \uc131\uc801 \uc9c0\ud5a5\uc5d0 \uc601\ud5a5\uc744 \uc900\ub2e4\ub294 \ucda9\ubd84\ud55c \uadfc\uac70\ub294 \uc874\uc7ac\ud558\uc9c0 \uc54a\ub294\ub2e4. \uac00\uc871 \ud658\uacbd \uc740 \ub3d9\uc131\uc560 \ud589\ub3d9\uc5d0 \uc788\uc5b4\uc11c \ub0a8\uc131\uc5d0\uac8c\ub294 \uc804\ud600 \uc601\ud5a5\uc744 \uc8fc\uc9c0 \uc54a\uc558\uace0 \uc5ec\uc131\uc5d0\uac8c\ub294 \ubbf8\uc138\ud55c \uc601\ud5a5\ub9cc\uc744 \uc8fc\uc5c8\ub2e4. \ub3d9\uc131\uc560\uac00 \"\uc790\uc5f0\uc2a4\ub7fd\uc9c0 \uc54a\ub2e4\"\uac70\ub098 \"\ubb38\uc81c\uac00 \uc788\ub2e4\"\ub294 \uacac\ud574\ub97c \uac16\uace0 \uc788\ub294 \uc0ac\ub78c\ub3c4 \uc788\uc9c0\ub9cc, \uc5f0\uad6c \uacb0\uacfc, \ub3d9\uc131\uc560\ub294 \uc815\uc0c1\uc801\uc774\uba70 \uc790\uc5f0\uc2a4\ub7ec\uc6b4 \ucc28\uc774\uc810\uc77c \ubfd0 \ubd80\uc815\uc801\uc778 \uc2ec\ub9ac\uc758 \uc6d0\uc778\uc774 \ub418\uc9c0 \uc54a\ub294\ub2e4\ub294 \uac83\uc774 \ubc1d\ud600\uc84c\ub2e4. \ub610\ud55c \ub3d9\uc131\uc560 \uad00\uacc4\ub294 \uc2ec\ub9ac\ud559\uc801\uc778 \uce21\uba74\uc5d0\uc11c \uc774\uc131\uc560 \uad00\uacc4\uc640 \ub3d9\ub4f1\ud558\ub2e4. \ub300\ubd80\ubd84\uc758 \uc0ac\ub78c\ub4e4\uc740 \uc2a4\uc2a4\ub85c \uc131\uc801 \uc9c0\ud5a5\uc744 '\uc120\ud0dd'\ud558\ub294 \uacbd\ud5d8\uc744 \ud558\uc9c0 \uc54a\uc73c\uba70, \ud0c0\uc778\uc774 \uac1c\uc785\ud574\uc11c \uc131\uc801 \uc9c0\ud5a5\uc744 \ubc14\uafc0 \uc218 \uc788\ub2e4\ub294 \uc99d\uac70\ub294 \ucda9\ubd84\ud558\uc9c0 \uc54a\ub2e4.", "sentence_answer": " \uac00\uc871 \ud658\uacbd \uc740 \ub3d9\uc131\uc560 \ud589\ub3d9\uc5d0 \uc788\uc5b4\uc11c \ub0a8\uc131\uc5d0\uac8c\ub294 \uc804\ud600 \uc601\ud5a5\uc744 \uc8fc\uc9c0 \uc54a\uc558\uace0 \uc5ec\uc131\uc5d0\uac8c\ub294 \ubbf8\uc138\ud55c \uc601\ud5a5\ub9cc\uc744 \uc8fc\uc5c8\ub2e4.", "paragraph_id": "6492356-0-0", "paragraph_question": "question: \uc5b4\ub5a4 \uc694\uc778\uc740 \ub3d9\uc131\uc560 \ud589\ub3d9\uc5d0 \uc788\uc5b4\uc11c \ub0a8\uc131\uc5d0\uac8c\ub294 \uc804\ud600 \uc0c1\uad00\uad00\uacc4\uac00 \uc5c6\uace0 \uc5ec\uc131\uc5d0\uac8c\ub294 \uc57d\uac04\uc758 \uc0c1\uad00\uad00\uacc4\uac00 \uc788\ub294\uac00?, context: \uac1c\uc778\uc774 \ud2b9\uc815\ud55c \uc131\uc801 \uc9c0\ud5a5\uc744 \ubc1c\ub2ec\ud558\uac8c \ub418\ub294 \uc774\uc720\ub294 \uc544\uc9c1 \uacfc\ud559\uc790\ub4e4 \uc0ac\uc774\uc5d0 \ud569\uc758\ub97c \uc774\ub8e8\uc9c0 \ubabb\ud588\ub2e4. \uadf8\ub7ec\ub098 \uc804\ubb38\uac00\ub4e4 \uc0ac\uc774\uc5d0\uc11c\ub294 \uc720\uc804\uc790, \ucd08\uae30 \uc790\uad81 \ub0b4 \ud658\uacbd, \ub610\ub294 \uadf8 \ub458\uc758 \uc870\ud569\uc73c\ub85c \uc124\uba85\ub418\ub294 \uc0dd\ubb3c\ud559\uc801\uc778 \uc774\ub860\ub4e4\uc774 \uc8fc\ub958\ub97c \uc774\ub8ec\ub2e4. \ubd80\ubaa8\uc758 \uc721\uc544 \ubc29\uc2dd\uc774\ub098 \uc720\uc544 \ub54c\uc758 \uacbd\ud5d8\uc774 \uc131\uc801 \uc9c0\ud5a5\uc5d0 \uc601\ud5a5\uc744 \uc900\ub2e4\ub294 \ucda9\ubd84\ud55c \uadfc\uac70\ub294 \uc874\uc7ac\ud558\uc9c0 \uc54a\ub294\ub2e4. \uac00\uc871 \ud658\uacbd\uc740 \ub3d9\uc131\uc560 \ud589\ub3d9\uc5d0 \uc788\uc5b4\uc11c \ub0a8\uc131\uc5d0\uac8c\ub294 \uc804\ud600 \uc601\ud5a5\uc744 \uc8fc\uc9c0 \uc54a\uc558\uace0 \uc5ec\uc131\uc5d0\uac8c\ub294 \ubbf8\uc138\ud55c \uc601\ud5a5\ub9cc\uc744 \uc8fc\uc5c8\ub2e4. \ub3d9\uc131\uc560\uac00 \"\uc790\uc5f0\uc2a4\ub7fd\uc9c0 \uc54a\ub2e4\"\uac70\ub098 \"\ubb38\uc81c\uac00 \uc788\ub2e4\"\ub294 \uacac\ud574\ub97c \uac16\uace0 \uc788\ub294 \uc0ac\ub78c\ub3c4 \uc788\uc9c0\ub9cc, \uc5f0\uad6c \uacb0\uacfc, \ub3d9\uc131\uc560\ub294 \uc815\uc0c1\uc801\uc774\uba70 \uc790\uc5f0\uc2a4\ub7ec\uc6b4 \ucc28\uc774\uc810\uc77c \ubfd0 \ubd80\uc815\uc801\uc778 \uc2ec\ub9ac\uc758 \uc6d0\uc778\uc774 \ub418\uc9c0 \uc54a\ub294\ub2e4\ub294 \uac83\uc774 \ubc1d\ud600\uc84c\ub2e4. \ub610\ud55c \ub3d9\uc131\uc560 \uad00\uacc4\ub294 \uc2ec\ub9ac\ud559\uc801\uc778 \uce21\uba74\uc5d0\uc11c \uc774\uc131\uc560 \uad00\uacc4\uc640 \ub3d9\ub4f1\ud558\ub2e4. \ub300\ubd80\ubd84\uc758 \uc0ac\ub78c\ub4e4\uc740 \uc2a4\uc2a4\ub85c \uc131\uc801 \uc9c0\ud5a5\uc744 '\uc120\ud0dd'\ud558\ub294 \uacbd\ud5d8\uc744 \ud558\uc9c0 \uc54a\uc73c\uba70, \ud0c0\uc778\uc774 \uac1c\uc785\ud574\uc11c \uc131\uc801 \uc9c0\ud5a5\uc744 \ubc14\uafc0 \uc218 \uc788\ub2e4\ub294 \uc99d\uac70\ub294 \ucda9\ubd84\ud558\uc9c0 \uc54a\ub2e4."} {"question": "2015\ub144 \ub204\uad6c\uc758 \ub4a4\ub97c \uc774\uc5b4 NC \ub2e4\uc774\ub178\uc2a4\uc758 \uc0c8 \uc8fc\uc7a5\uc774 \ub418\uc5c8\ub098?", "paragraph": "2015\ub144 \uc2dc\uc98c\uc5d0 \uc774\ud638\uc900\uc758 \ub4a4\ub97c \uc774\uc5b4 NC \ub2e4\uc774\ub178\uc2a4\uc758 \uc0c8 \uc8fc\uc7a5\uc774 \ub418\uc5c8\ub2e4. \uadf8\ub294 \u201c\uc774\ud638\uc900 \uc120\ubc30\uc758 \ub4a4\ub97c \uc787\ub294\ub2e4\ub294 \uc810\uc5d0\uc11c \ubd80\ub2f4\uc774 \ud06c\uc9c0\ub9cc \uc88b\uc740 \ud2c0\uc744 \ub9cc\ub4e4\uc5b4\ub193\uc740 \ub9cc\ud07c \uc804\ud1b5\uc744 \uc798 \uc787\ub294 \uc8fc\uc7a5\uc774 \ub418\uaca0\ub2e4\u201d\uba70 \u201c\ucc45\uc784\uac10\uc744 \uac00\uc9c0\uace0 \uc5f4\uc2ec\ud788 \ud558\uaca0\ub2e4\u201d\uace0 \uc18c\uac10\uc744 \ubc1d\ud614\ub2e4. 4\uc6d4 1\uc77c \ub125\uc13c \ud788\uc5b4\ub85c\uc988\uc640\uc758 \uacbd\uae30\uc5d0\uc11c \ud53c\uc5b4\ubc34\ub4dc\ub97c \uc0c1\ub300\ub85c 4\ud68c 2\uc0ac\uc5d0\uc11c \uc88c\uc804 \uc548\ud0c0\ub97c \ucce4\ub2e4. \uc774 \uc548\ud0c0\ub85c \uc5ed\ub300 63\ubc88\uc9f8 \ud1b5\uc0b0 1,100\uc548\ud0c0\ub97c \uae30\ub85d\ud588\ub2e4. 4\uc6d4 15\uc77c \ub86f\ub370 \uc790\uc774\uc5b8\uce20\uc640\uc758 \uacbd\uae30\uc5d0\uc11c 2\ub8e8 \ub3c4\ub8e8\uc5d0 \uc131\uacf5\ud558\uba74\uc11c \uc5ed\ub300 9\ubc88\uc9f8 \ud1b5\uc0b0 300\ub3c4\ub8e8\ub97c \uae30\ub85d\ud588\ub2e4. 6\uc6d4 18\uc77c kt \uc704\uc988\uc640\uc758 \uacbd\uae30\uc5d0\uc11c \uc5ed\ub300 88\ubc88\uc9f8 \ud1b5\uc0b0 1100\uacbd\uae30 \ucd9c\uc7a5\uc744 \uae30\ub85d\ud588\ub2e4. 6\uc6d4 21\uc77c \ud55c\ud654 \uc774\uae00\uc2a4\uc640\uc758 \uacbd\uae30\uc5d0\uc11c 2\ub8e8 \ub3c4\ub8e8\ub97c \uc131\uacf5\ud558\uba74\uc11c \ud1b5\uc0b0 9\ubc88\uc9f8 10\ub144 \uc5f0\uc18d \ub450 \uc790\ub9bf\uc218 \ub3c4\ub8e8\ub97c \ub2ec\uc131\ud588\ub2e4. \uc804\ubc18\uae30 81\uacbd\uae30 293\ud0c0\uc218 79\uc548\ud0c0 \ud0c0\uc728 0.270, 45\ub4dd\uc810, 2\ud648\ub7f0, 31\ud0c0\uc810, 14\ub3c4\ub8e8, 37\ubcfc\ub137, 48\ud0c8\uc0bc\uc9c4\uc744 \uae30\ub85d\ud588\ub2e4. 8\uc6d4 22\uc77c SK \uc640\uc774\ubc88\uc2a4\uc640\uc758 \uacbd\uae30\uc5d0\uc11c \uc2e0\uc7ac\uc6c5\uc744 \uc0c1\ub300\ub85c \uc88c\uc775\uc218 \ud0a4\ub97c \ub118\uae30\ub294 2\ub8e8\ud0c0\ub97c \uce58\uba74\uc11c KBO \ub9ac\uadf8 \ud1b5\uc0b0 52\ubc88\uc9f8 1200\uc548\ud0c0\ub97c \uae30\ub85d\ud588\ub2e4. 2015 \uc2dc\uc98c 2\ud560\ub300 \ud0c0\uc728, 125\uacbd\uae30, 440\ud0c0\uc218, 118\uc548\ud0c0(5\ud648\ub7f0), 63\ub4dd\uc810, 52\ud0c0\uc810, 17\ub3c4\ub8e8\ub97c \uae30\ub85d\ud588\ub2e4. \ud50c\ub808\uc774\uc624\ud504 1, 2\ucc28\uc804\uc5d0\uc11c 3\ubc88\ud0c0\uc21c\uc5d0 \uae30\uc6a9\ub410\uc73c\ub098 \ucd9c\ub8e8\uc870\ucc28 \ud558\uc9c0 \ubabb\ud558\uace0 \ubd80\uc9c4\ud588\ub2e4. 3\ucc28\uc804\uc5d0\uc11c\ub294 5\ud0c0\uc218 2\uc548\ud0c0 2\ud0c0\uc810\uc73c\ub85c \ud300\uc758 \uc2b9\ub9ac\ub97c \uc774\ub04c\uc5c8\uace0, 4\ucc28\uc804\uc5d0\uc11c\ub3c4 \uc548\ud0c0\ub97c 1\uac1c \ucce4\ub2e4. \uadf8\ub7ec\ub098 5\ucc28\uc804\uc5d0\uc11c \uc548\ud0c0\ub97c \uce58\uc9c0 \ubabb\ud558\uba74\uc11c \ud50c\ub808\uc774\uc624\ud504 \ucd5c\uc885 \uc131\uc801 18\ud0c0\uc218 3\uc548\ud0c0 \ud0c0\uc728 0.166 2\ud0c0\uc810\uc744 \uae30\ub85d\ud588\ub2e4.", "answer": "\uc774\ud638\uc900", "sentence": "2015\ub144 \uc2dc\uc98c\uc5d0 \uc774\ud638\uc900 \uc758 \ub4a4\ub97c \uc774\uc5b4 NC \ub2e4\uc774\ub178\uc2a4\uc758 \uc0c8 \uc8fc\uc7a5\uc774 \ub418\uc5c8\ub2e4.", "paragraph_sentence": " 2015\ub144 \uc2dc\uc98c\uc5d0 \uc774\ud638\uc900 \uc758 \ub4a4\ub97c \uc774\uc5b4 NC \ub2e4\uc774\ub178\uc2a4\uc758 \uc0c8 \uc8fc\uc7a5\uc774 \ub418\uc5c8\ub2e4. \uadf8\ub294 \u201c\uc774\ud638\uc900 \uc120\ubc30\uc758 \ub4a4\ub97c \uc787\ub294\ub2e4\ub294 \uc810\uc5d0\uc11c \ubd80\ub2f4\uc774 \ud06c\uc9c0\ub9cc \uc88b\uc740 \ud2c0\uc744 \ub9cc\ub4e4\uc5b4\ub193\uc740 \ub9cc\ud07c \uc804\ud1b5\uc744 \uc798 \uc787\ub294 \uc8fc\uc7a5\uc774 \ub418\uaca0\ub2e4\u201d\uba70 \u201c\ucc45\uc784\uac10\uc744 \uac00\uc9c0\uace0 \uc5f4\uc2ec\ud788 \ud558\uaca0\ub2e4\u201d\uace0 \uc18c\uac10\uc744 \ubc1d\ud614\ub2e4. 4\uc6d4 1\uc77c \ub125\uc13c \ud788\uc5b4\ub85c\uc988\uc640\uc758 \uacbd\uae30\uc5d0\uc11c \ud53c\uc5b4\ubc34\ub4dc\ub97c \uc0c1\ub300\ub85c 4\ud68c 2\uc0ac\uc5d0\uc11c \uc88c\uc804 \uc548\ud0c0\ub97c \ucce4\ub2e4. \uc774 \uc548\ud0c0\ub85c \uc5ed\ub300 63\ubc88\uc9f8 \ud1b5\uc0b0 1,100\uc548\ud0c0\ub97c \uae30\ub85d\ud588\ub2e4. 4\uc6d4 15\uc77c \ub86f\ub370 \uc790\uc774\uc5b8\uce20\uc640\uc758 \uacbd\uae30\uc5d0\uc11c 2\ub8e8 \ub3c4\ub8e8\uc5d0 \uc131\uacf5\ud558\uba74\uc11c \uc5ed\ub300 9\ubc88\uc9f8 \ud1b5\uc0b0 300\ub3c4\ub8e8\ub97c \uae30\ub85d\ud588\ub2e4. 6\uc6d4 18\uc77c kt \uc704\uc988\uc640\uc758 \uacbd\uae30\uc5d0\uc11c \uc5ed\ub300 88\ubc88\uc9f8 \ud1b5\uc0b0 1100\uacbd\uae30 \ucd9c\uc7a5\uc744 \uae30\ub85d\ud588\ub2e4. 6\uc6d4 21\uc77c \ud55c\ud654 \uc774\uae00\uc2a4\uc640\uc758 \uacbd\uae30\uc5d0\uc11c 2\ub8e8 \ub3c4\ub8e8\ub97c \uc131\uacf5\ud558\uba74\uc11c \ud1b5\uc0b0 9\ubc88\uc9f8 10\ub144 \uc5f0\uc18d \ub450 \uc790\ub9bf\uc218 \ub3c4\ub8e8\ub97c \ub2ec\uc131\ud588\ub2e4. \uc804\ubc18\uae30 81\uacbd\uae30 293\ud0c0\uc218 79\uc548\ud0c0 \ud0c0\uc728 0.270, 45\ub4dd\uc810, 2\ud648\ub7f0, 31\ud0c0\uc810, 14\ub3c4\ub8e8, 37\ubcfc\ub137, 48\ud0c8\uc0bc\uc9c4\uc744 \uae30\ub85d\ud588\ub2e4. 8\uc6d4 22\uc77c SK \uc640\uc774\ubc88\uc2a4\uc640\uc758 \uacbd\uae30\uc5d0\uc11c \uc2e0\uc7ac\uc6c5\uc744 \uc0c1\ub300\ub85c \uc88c\uc775\uc218 \ud0a4\ub97c \ub118\uae30\ub294 2\ub8e8\ud0c0\ub97c \uce58\uba74\uc11c KBO \ub9ac\uadf8 \ud1b5\uc0b0 52\ubc88\uc9f8 1200\uc548\ud0c0\ub97c \uae30\ub85d\ud588\ub2e4. 2015 \uc2dc\uc98c 2\ud560\ub300 \ud0c0\uc728, 125\uacbd\uae30, 440\ud0c0\uc218, 118\uc548\ud0c0(5\ud648\ub7f0), 63\ub4dd\uc810, 52\ud0c0\uc810, 17\ub3c4\ub8e8\ub97c \uae30\ub85d\ud588\ub2e4. \ud50c\ub808\uc774\uc624\ud504 1, 2\ucc28\uc804\uc5d0\uc11c 3\ubc88\ud0c0\uc21c\uc5d0 \uae30\uc6a9\ub410\uc73c\ub098 \ucd9c\ub8e8\uc870\ucc28 \ud558\uc9c0 \ubabb\ud558\uace0 \ubd80\uc9c4\ud588\ub2e4. 3\ucc28\uc804\uc5d0\uc11c\ub294 5\ud0c0\uc218 2\uc548\ud0c0 2\ud0c0\uc810\uc73c\ub85c \ud300\uc758 \uc2b9\ub9ac\ub97c \uc774\ub04c\uc5c8\uace0, 4\ucc28\uc804\uc5d0\uc11c\ub3c4 \uc548\ud0c0\ub97c 1\uac1c \ucce4\ub2e4. \uadf8\ub7ec\ub098 5\ucc28\uc804\uc5d0\uc11c \uc548\ud0c0\ub97c \uce58\uc9c0 \ubabb\ud558\uba74\uc11c \ud50c\ub808\uc774\uc624\ud504 \ucd5c\uc885 \uc131\uc801 18\ud0c0\uc218 3\uc548\ud0c0 \ud0c0\uc728 0.166 2\ud0c0\uc810\uc744 \uae30\ub85d\ud588\ub2e4.", "paragraph_answer": "2015\ub144 \uc2dc\uc98c\uc5d0 \uc774\ud638\uc900 \uc758 \ub4a4\ub97c \uc774\uc5b4 NC \ub2e4\uc774\ub178\uc2a4\uc758 \uc0c8 \uc8fc\uc7a5\uc774 \ub418\uc5c8\ub2e4. \uadf8\ub294 \u201c\uc774\ud638\uc900 \uc120\ubc30\uc758 \ub4a4\ub97c \uc787\ub294\ub2e4\ub294 \uc810\uc5d0\uc11c \ubd80\ub2f4\uc774 \ud06c\uc9c0\ub9cc \uc88b\uc740 \ud2c0\uc744 \ub9cc\ub4e4\uc5b4\ub193\uc740 \ub9cc\ud07c \uc804\ud1b5\uc744 \uc798 \uc787\ub294 \uc8fc\uc7a5\uc774 \ub418\uaca0\ub2e4\u201d\uba70 \u201c\ucc45\uc784\uac10\uc744 \uac00\uc9c0\uace0 \uc5f4\uc2ec\ud788 \ud558\uaca0\ub2e4\u201d\uace0 \uc18c\uac10\uc744 \ubc1d\ud614\ub2e4. 4\uc6d4 1\uc77c \ub125\uc13c \ud788\uc5b4\ub85c\uc988\uc640\uc758 \uacbd\uae30\uc5d0\uc11c \ud53c\uc5b4\ubc34\ub4dc\ub97c \uc0c1\ub300\ub85c 4\ud68c 2\uc0ac\uc5d0\uc11c \uc88c\uc804 \uc548\ud0c0\ub97c \ucce4\ub2e4. \uc774 \uc548\ud0c0\ub85c \uc5ed\ub300 63\ubc88\uc9f8 \ud1b5\uc0b0 1,100\uc548\ud0c0\ub97c \uae30\ub85d\ud588\ub2e4. 4\uc6d4 15\uc77c \ub86f\ub370 \uc790\uc774\uc5b8\uce20\uc640\uc758 \uacbd\uae30\uc5d0\uc11c 2\ub8e8 \ub3c4\ub8e8\uc5d0 \uc131\uacf5\ud558\uba74\uc11c \uc5ed\ub300 9\ubc88\uc9f8 \ud1b5\uc0b0 300\ub3c4\ub8e8\ub97c \uae30\ub85d\ud588\ub2e4. 6\uc6d4 18\uc77c kt \uc704\uc988\uc640\uc758 \uacbd\uae30\uc5d0\uc11c \uc5ed\ub300 88\ubc88\uc9f8 \ud1b5\uc0b0 1100\uacbd\uae30 \ucd9c\uc7a5\uc744 \uae30\ub85d\ud588\ub2e4. 6\uc6d4 21\uc77c \ud55c\ud654 \uc774\uae00\uc2a4\uc640\uc758 \uacbd\uae30\uc5d0\uc11c 2\ub8e8 \ub3c4\ub8e8\ub97c \uc131\uacf5\ud558\uba74\uc11c \ud1b5\uc0b0 9\ubc88\uc9f8 10\ub144 \uc5f0\uc18d \ub450 \uc790\ub9bf\uc218 \ub3c4\ub8e8\ub97c \ub2ec\uc131\ud588\ub2e4. \uc804\ubc18\uae30 81\uacbd\uae30 293\ud0c0\uc218 79\uc548\ud0c0 \ud0c0\uc728 0.270, 45\ub4dd\uc810, 2\ud648\ub7f0, 31\ud0c0\uc810, 14\ub3c4\ub8e8, 37\ubcfc\ub137, 48\ud0c8\uc0bc\uc9c4\uc744 \uae30\ub85d\ud588\ub2e4. 8\uc6d4 22\uc77c SK \uc640\uc774\ubc88\uc2a4\uc640\uc758 \uacbd\uae30\uc5d0\uc11c \uc2e0\uc7ac\uc6c5\uc744 \uc0c1\ub300\ub85c \uc88c\uc775\uc218 \ud0a4\ub97c \ub118\uae30\ub294 2\ub8e8\ud0c0\ub97c \uce58\uba74\uc11c KBO \ub9ac\uadf8 \ud1b5\uc0b0 52\ubc88\uc9f8 1200\uc548\ud0c0\ub97c \uae30\ub85d\ud588\ub2e4. 2015 \uc2dc\uc98c 2\ud560\ub300 \ud0c0\uc728, 125\uacbd\uae30, 440\ud0c0\uc218, 118\uc548\ud0c0(5\ud648\ub7f0), 63\ub4dd\uc810, 52\ud0c0\uc810, 17\ub3c4\ub8e8\ub97c \uae30\ub85d\ud588\ub2e4. \ud50c\ub808\uc774\uc624\ud504 1, 2\ucc28\uc804\uc5d0\uc11c 3\ubc88\ud0c0\uc21c\uc5d0 \uae30\uc6a9\ub410\uc73c\ub098 \ucd9c\ub8e8\uc870\ucc28 \ud558\uc9c0 \ubabb\ud558\uace0 \ubd80\uc9c4\ud588\ub2e4. 3\ucc28\uc804\uc5d0\uc11c\ub294 5\ud0c0\uc218 2\uc548\ud0c0 2\ud0c0\uc810\uc73c\ub85c \ud300\uc758 \uc2b9\ub9ac\ub97c \uc774\ub04c\uc5c8\uace0, 4\ucc28\uc804\uc5d0\uc11c\ub3c4 \uc548\ud0c0\ub97c 1\uac1c \ucce4\ub2e4. \uadf8\ub7ec\ub098 5\ucc28\uc804\uc5d0\uc11c \uc548\ud0c0\ub97c \uce58\uc9c0 \ubabb\ud558\uba74\uc11c \ud50c\ub808\uc774\uc624\ud504 \ucd5c\uc885 \uc131\uc801 18\ud0c0\uc218 3\uc548\ud0c0 \ud0c0\uc728 0.166 2\ud0c0\uc810\uc744 \uae30\ub85d\ud588\ub2e4.", "sentence_answer": "2015\ub144 \uc2dc\uc98c\uc5d0 \uc774\ud638\uc900 \uc758 \ub4a4\ub97c \uc774\uc5b4 NC \ub2e4\uc774\ub178\uc2a4\uc758 \uc0c8 \uc8fc\uc7a5\uc774 \ub418\uc5c8\ub2e4.", "paragraph_id": "6590288-10-0", "paragraph_question": "question: 2015\ub144 \ub204\uad6c\uc758 \ub4a4\ub97c \uc774\uc5b4 NC \ub2e4\uc774\ub178\uc2a4\uc758 \uc0c8 \uc8fc\uc7a5\uc774 \ub418\uc5c8\ub098?, context: 2015\ub144 \uc2dc\uc98c\uc5d0 \uc774\ud638\uc900\uc758 \ub4a4\ub97c \uc774\uc5b4 NC \ub2e4\uc774\ub178\uc2a4\uc758 \uc0c8 \uc8fc\uc7a5\uc774 \ub418\uc5c8\ub2e4. \uadf8\ub294 \u201c\uc774\ud638\uc900 \uc120\ubc30\uc758 \ub4a4\ub97c \uc787\ub294\ub2e4\ub294 \uc810\uc5d0\uc11c \ubd80\ub2f4\uc774 \ud06c\uc9c0\ub9cc \uc88b\uc740 \ud2c0\uc744 \ub9cc\ub4e4\uc5b4\ub193\uc740 \ub9cc\ud07c \uc804\ud1b5\uc744 \uc798 \uc787\ub294 \uc8fc\uc7a5\uc774 \ub418\uaca0\ub2e4\u201d\uba70 \u201c\ucc45\uc784\uac10\uc744 \uac00\uc9c0\uace0 \uc5f4\uc2ec\ud788 \ud558\uaca0\ub2e4\u201d\uace0 \uc18c\uac10\uc744 \ubc1d\ud614\ub2e4. 4\uc6d4 1\uc77c \ub125\uc13c \ud788\uc5b4\ub85c\uc988\uc640\uc758 \uacbd\uae30\uc5d0\uc11c \ud53c\uc5b4\ubc34\ub4dc\ub97c \uc0c1\ub300\ub85c 4\ud68c 2\uc0ac\uc5d0\uc11c \uc88c\uc804 \uc548\ud0c0\ub97c \ucce4\ub2e4. \uc774 \uc548\ud0c0\ub85c \uc5ed\ub300 63\ubc88\uc9f8 \ud1b5\uc0b0 1,100\uc548\ud0c0\ub97c \uae30\ub85d\ud588\ub2e4. 4\uc6d4 15\uc77c \ub86f\ub370 \uc790\uc774\uc5b8\uce20\uc640\uc758 \uacbd\uae30\uc5d0\uc11c 2\ub8e8 \ub3c4\ub8e8\uc5d0 \uc131\uacf5\ud558\uba74\uc11c \uc5ed\ub300 9\ubc88\uc9f8 \ud1b5\uc0b0 300\ub3c4\ub8e8\ub97c \uae30\ub85d\ud588\ub2e4. 6\uc6d4 18\uc77c kt \uc704\uc988\uc640\uc758 \uacbd\uae30\uc5d0\uc11c \uc5ed\ub300 88\ubc88\uc9f8 \ud1b5\uc0b0 1100\uacbd\uae30 \ucd9c\uc7a5\uc744 \uae30\ub85d\ud588\ub2e4. 6\uc6d4 21\uc77c \ud55c\ud654 \uc774\uae00\uc2a4\uc640\uc758 \uacbd\uae30\uc5d0\uc11c 2\ub8e8 \ub3c4\ub8e8\ub97c \uc131\uacf5\ud558\uba74\uc11c \ud1b5\uc0b0 9\ubc88\uc9f8 10\ub144 \uc5f0\uc18d \ub450 \uc790\ub9bf\uc218 \ub3c4\ub8e8\ub97c \ub2ec\uc131\ud588\ub2e4. \uc804\ubc18\uae30 81\uacbd\uae30 293\ud0c0\uc218 79\uc548\ud0c0 \ud0c0\uc728 0.270, 45\ub4dd\uc810, 2\ud648\ub7f0, 31\ud0c0\uc810, 14\ub3c4\ub8e8, 37\ubcfc\ub137, 48\ud0c8\uc0bc\uc9c4\uc744 \uae30\ub85d\ud588\ub2e4. 8\uc6d4 22\uc77c SK \uc640\uc774\ubc88\uc2a4\uc640\uc758 \uacbd\uae30\uc5d0\uc11c \uc2e0\uc7ac\uc6c5\uc744 \uc0c1\ub300\ub85c \uc88c\uc775\uc218 \ud0a4\ub97c \ub118\uae30\ub294 2\ub8e8\ud0c0\ub97c \uce58\uba74\uc11c KBO \ub9ac\uadf8 \ud1b5\uc0b0 52\ubc88\uc9f8 1200\uc548\ud0c0\ub97c \uae30\ub85d\ud588\ub2e4. 2015 \uc2dc\uc98c 2\ud560\ub300 \ud0c0\uc728, 125\uacbd\uae30, 440\ud0c0\uc218, 118\uc548\ud0c0(5\ud648\ub7f0), 63\ub4dd\uc810, 52\ud0c0\uc810, 17\ub3c4\ub8e8\ub97c \uae30\ub85d\ud588\ub2e4. \ud50c\ub808\uc774\uc624\ud504 1, 2\ucc28\uc804\uc5d0\uc11c 3\ubc88\ud0c0\uc21c\uc5d0 \uae30\uc6a9\ub410\uc73c\ub098 \ucd9c\ub8e8\uc870\ucc28 \ud558\uc9c0 \ubabb\ud558\uace0 \ubd80\uc9c4\ud588\ub2e4. 3\ucc28\uc804\uc5d0\uc11c\ub294 5\ud0c0\uc218 2\uc548\ud0c0 2\ud0c0\uc810\uc73c\ub85c \ud300\uc758 \uc2b9\ub9ac\ub97c \uc774\ub04c\uc5c8\uace0, 4\ucc28\uc804\uc5d0\uc11c\ub3c4 \uc548\ud0c0\ub97c 1\uac1c \ucce4\ub2e4. \uadf8\ub7ec\ub098 5\ucc28\uc804\uc5d0\uc11c \uc548\ud0c0\ub97c \uce58\uc9c0 \ubabb\ud558\uba74\uc11c \ud50c\ub808\uc774\uc624\ud504 \ucd5c\uc885 \uc131\uc801 18\ud0c0\uc218 3\uc548\ud0c0 \ud0c0\uc728 0.166 2\ud0c0\uc810\uc744 \uae30\ub85d\ud588\ub2e4."} {"question": "\uc544\ub2e5\uc2a4 \ubb34\ub9ac\uc5d0\uc11c \uc9c0\uc704\uac00 \uacb0\uc815\ub418\ub294 \uae30\uc900\uc740 \ubb34\uc5c7\uc778\uac00?", "paragraph": "\uc544\ub2e5\uc2a4\ub294 \uc8fc\ub85c \ubc24\uc5d0 \ubd84\uc8fc\ud558\uac8c \ud65c\ub3d9\ud558\ub294 \uc57c\ud589\uc131 \ub3d9\ubb3c\ub85c, \ud2b9\ud788 \uc5ec\ub984\uc5d0 \ub354 \ud65c\ubc1c\ud574\uc9c4\ub2e4. \ub0ae \ub3d9\uc548\uc5d0\ub294 \uadf8\ub298\uc9c4 \uc790\ub9ac\ub97c \uc7a1\uace0 \ubaa8\ub798 \uad6c\ub369\uc774\ub97c \ud30c\uace0 \uc270\ub2e4. \uc774 \uad6c\ub369\uc774\ub294 \ud587\ube5b\ub3c4 \ub9c9\uc544 \uc8fc\uc9c0\ub9cc \uc774\ub530\uae08 \ubaa8\ub798 \ud3ed\ud48d\uc744 \ud53c\ud560 \uc218 \uc788\ub294 \ubc29\uacf5\ud638 \uc5ed\ud560\ub3c4 \ud55c\ub2e4. \ub2e4\uc218\uc758 \uc218\ucef7\uacfc \uc554\ucef7\uc774 \ud3ec\ud568\ub41c \ubb34\ub9ac\ub97c \uc9c0\uc73c\uba70, \uaddc\ubaa8\ub294 5-20\ub9c8\ub9ac \uc815\ub3c4\uc774\ub2e4. \ubb34\ub9ac\uac00 \ub9cc\ub4e4\uc5b4\uc9c0\uba74 \uc815\ucc29 \uc0dd\ud65c\uc744 \ud558\uba70 \uc624\ub85c\uc9c0 \uba39\uc774\uac00 \ub2e4 \ub5a8\uc5b4\uc84c\uc744 \ub54c\ub9cc \uc774\ub3d9\ud55c\ub2e4. \uba39\uc774\ub85c \uc0bc\uc744 \ucd08\ubaa9\uc774 \ud48d\ubd80\ud55c \uacf3\uc744 \ucc3e\uc544\uac08 \ub54c\uc5d0\ub294 \uc8fc\ub85c \ube44\uac00 \uc798 \ub0b4\ub9ac\ub294 \ucabd\uc73c\ub85c \uc774\ub3d9\ud558\uba70, \uc5ec\ub984 \ub3d9\uc548\uc5d0 \uc6b0\uae30\uac00 \ucc3e\uc544\uc624\ub294 \uc9c0\uc5ed\uc73c\ub85c \ubb34\ub9ac\ub97c \ucc3e\uc744 \uc218 \uc788\ub2e4. \ub69c\ub837\ud55c \uc0ac\ud68c \uccb4\uacc4\ub97c \uc774\ub8e8\uace0 \uc788\uc73c\uba70, \ud559\uacc4\uc5d0\uc11c\ub294 \uadf8 \uc9c0\uc704\uac00 \ub098\uc774\uc5d0 \uc758\ud574\uc11c \uacb0\uc815\ub41c\ub2e4\uace0 \ubcf4\uace0 \uc788\ub2e4. \ubb34\ub9ac\uc758 \ub300\uc7a5\uc740 \uac00\uc7a5 \ub098\uc774\ub4e0 \uc554\ucef7\uc73c\ub85c \uc815\ud574\uc9c0\uba70, \uc774\ub294 \ub3d9\ubb3c\uc6d0 \ub4f1 \uc0ac\uc721 \uc0c1\ud0dc\uc5d0\uc11c\ub3c4 \ub098\ud0c0\ub098\ub294 \ud604\uc0c1\uc774\ub2e4. \uc218\ucef7\ub4e4\uc740 \ub300\ubd80\ubd84 \uc138\ub825\uad8c\uc744 \uac00\uc9c0\uace0 \uc788\uc73c\uba70, \uc554\ucef7\ub4e4\uc774 \uc790\uc2e0\uc774 \uc774\ub044\ub294 \ubb34\ub9ac\ub97c \ub9cc\ub4e4\uace0 \uc788\uc744 \ub54c \uc554\ucef7\uc744 \ud638\uc704\ud558\ub294 \uc5ed\ud560\uc744 \ud55c\ub2e4.", "answer": "\ub098\uc774", "sentence": "\ud559\uacc4\uc5d0\uc11c\ub294 \uadf8 \uc9c0\uc704\uac00 \ub098\uc774 \uc5d0", "paragraph_sentence": "\uc544\ub2e5\uc2a4\ub294 \uc8fc\ub85c \ubc24\uc5d0 \ubd84\uc8fc\ud558\uac8c \ud65c\ub3d9\ud558\ub294 \uc57c\ud589\uc131 \ub3d9\ubb3c\ub85c, \ud2b9\ud788 \uc5ec\ub984\uc5d0 \ub354 \ud65c\ubc1c\ud574\uc9c4\ub2e4. \ub0ae \ub3d9\uc548\uc5d0\ub294 \uadf8\ub298\uc9c4 \uc790\ub9ac\ub97c \uc7a1\uace0 \ubaa8\ub798 \uad6c\ub369\uc774\ub97c \ud30c\uace0 \uc270\ub2e4. \uc774 \uad6c\ub369\uc774\ub294 \ud587\ube5b\ub3c4 \ub9c9\uc544 \uc8fc\uc9c0\ub9cc \uc774\ub530\uae08 \ubaa8\ub798 \ud3ed\ud48d\uc744 \ud53c\ud560 \uc218 \uc788\ub294 \ubc29\uacf5\ud638 \uc5ed\ud560\ub3c4 \ud55c\ub2e4. \ub2e4\uc218\uc758 \uc218\ucef7\uacfc \uc554\ucef7\uc774 \ud3ec\ud568\ub41c \ubb34\ub9ac\ub97c \uc9c0\uc73c\uba70, \uaddc\ubaa8\ub294 5-20\ub9c8\ub9ac \uc815\ub3c4\uc774\ub2e4. \ubb34\ub9ac\uac00 \ub9cc\ub4e4\uc5b4\uc9c0\uba74 \uc815\ucc29 \uc0dd\ud65c\uc744 \ud558\uba70 \uc624\ub85c\uc9c0 \uba39\uc774\uac00 \ub2e4 \ub5a8\uc5b4\uc84c\uc744 \ub54c\ub9cc \uc774\ub3d9\ud55c\ub2e4. \uba39\uc774\ub85c \uc0bc\uc744 \ucd08\ubaa9\uc774 \ud48d\ubd80\ud55c \uacf3\uc744 \ucc3e\uc544\uac08 \ub54c\uc5d0\ub294 \uc8fc\ub85c \ube44\uac00 \uc798 \ub0b4\ub9ac\ub294 \ucabd\uc73c\ub85c \uc774\ub3d9\ud558\uba70, \uc5ec\ub984 \ub3d9\uc548\uc5d0 \uc6b0\uae30\uac00 \ucc3e\uc544\uc624\ub294 \uc9c0\uc5ed\uc73c\ub85c \ubb34\ub9ac\ub97c \ucc3e\uc744 \uc218 \uc788\ub2e4. \ub69c\ub837\ud55c \uc0ac\ud68c \uccb4\uacc4\ub97c \uc774\ub8e8\uace0 \uc788\uc73c\uba70, \ud559\uacc4\uc5d0\uc11c\ub294 \uadf8 \uc9c0\uc704\uac00 \ub098\uc774 \uc5d0 \uc758\ud574\uc11c \uacb0\uc815\ub41c\ub2e4\uace0 \ubcf4\uace0 \uc788\ub2e4. \ubb34\ub9ac\uc758 \ub300\uc7a5\uc740 \uac00\uc7a5 \ub098\uc774\ub4e0 \uc554\ucef7\uc73c\ub85c \uc815\ud574\uc9c0\uba70, \uc774\ub294 \ub3d9\ubb3c\uc6d0 \ub4f1 \uc0ac\uc721 \uc0c1\ud0dc\uc5d0\uc11c\ub3c4 \ub098\ud0c0\ub098\ub294 \ud604\uc0c1\uc774\ub2e4. \uc218\ucef7\ub4e4\uc740 \ub300\ubd80\ubd84 \uc138\ub825\uad8c\uc744 \uac00\uc9c0\uace0 \uc788\uc73c\uba70, \uc554\ucef7\ub4e4\uc774 \uc790\uc2e0\uc774 \uc774\ub044\ub294 \ubb34\ub9ac\ub97c \ub9cc\ub4e4\uace0 \uc788\uc744 \ub54c \uc554\ucef7\uc744 \ud638\uc704\ud558\ub294 \uc5ed\ud560\uc744 \ud55c\ub2e4.", "paragraph_answer": "\uc544\ub2e5\uc2a4\ub294 \uc8fc\ub85c \ubc24\uc5d0 \ubd84\uc8fc\ud558\uac8c \ud65c\ub3d9\ud558\ub294 \uc57c\ud589\uc131 \ub3d9\ubb3c\ub85c, \ud2b9\ud788 \uc5ec\ub984\uc5d0 \ub354 \ud65c\ubc1c\ud574\uc9c4\ub2e4. \ub0ae \ub3d9\uc548\uc5d0\ub294 \uadf8\ub298\uc9c4 \uc790\ub9ac\ub97c \uc7a1\uace0 \ubaa8\ub798 \uad6c\ub369\uc774\ub97c \ud30c\uace0 \uc270\ub2e4. \uc774 \uad6c\ub369\uc774\ub294 \ud587\ube5b\ub3c4 \ub9c9\uc544 \uc8fc\uc9c0\ub9cc \uc774\ub530\uae08 \ubaa8\ub798 \ud3ed\ud48d\uc744 \ud53c\ud560 \uc218 \uc788\ub294 \ubc29\uacf5\ud638 \uc5ed\ud560\ub3c4 \ud55c\ub2e4. \ub2e4\uc218\uc758 \uc218\ucef7\uacfc \uc554\ucef7\uc774 \ud3ec\ud568\ub41c \ubb34\ub9ac\ub97c \uc9c0\uc73c\uba70, \uaddc\ubaa8\ub294 5-20\ub9c8\ub9ac \uc815\ub3c4\uc774\ub2e4. \ubb34\ub9ac\uac00 \ub9cc\ub4e4\uc5b4\uc9c0\uba74 \uc815\ucc29 \uc0dd\ud65c\uc744 \ud558\uba70 \uc624\ub85c\uc9c0 \uba39\uc774\uac00 \ub2e4 \ub5a8\uc5b4\uc84c\uc744 \ub54c\ub9cc \uc774\ub3d9\ud55c\ub2e4. \uba39\uc774\ub85c \uc0bc\uc744 \ucd08\ubaa9\uc774 \ud48d\ubd80\ud55c \uacf3\uc744 \ucc3e\uc544\uac08 \ub54c\uc5d0\ub294 \uc8fc\ub85c \ube44\uac00 \uc798 \ub0b4\ub9ac\ub294 \ucabd\uc73c\ub85c \uc774\ub3d9\ud558\uba70, \uc5ec\ub984 \ub3d9\uc548\uc5d0 \uc6b0\uae30\uac00 \ucc3e\uc544\uc624\ub294 \uc9c0\uc5ed\uc73c\ub85c \ubb34\ub9ac\ub97c \ucc3e\uc744 \uc218 \uc788\ub2e4. \ub69c\ub837\ud55c \uc0ac\ud68c \uccb4\uacc4\ub97c \uc774\ub8e8\uace0 \uc788\uc73c\uba70, \ud559\uacc4\uc5d0\uc11c\ub294 \uadf8 \uc9c0\uc704\uac00 \ub098\uc774 \uc5d0 \uc758\ud574\uc11c \uacb0\uc815\ub41c\ub2e4\uace0 \ubcf4\uace0 \uc788\ub2e4. \ubb34\ub9ac\uc758 \ub300\uc7a5\uc740 \uac00\uc7a5 \ub098\uc774\ub4e0 \uc554\ucef7\uc73c\ub85c \uc815\ud574\uc9c0\uba70, \uc774\ub294 \ub3d9\ubb3c\uc6d0 \ub4f1 \uc0ac\uc721 \uc0c1\ud0dc\uc5d0\uc11c\ub3c4 \ub098\ud0c0\ub098\ub294 \ud604\uc0c1\uc774\ub2e4. \uc218\ucef7\ub4e4\uc740 \ub300\ubd80\ubd84 \uc138\ub825\uad8c\uc744 \uac00\uc9c0\uace0 \uc788\uc73c\uba70, \uc554\ucef7\ub4e4\uc774 \uc790\uc2e0\uc774 \uc774\ub044\ub294 \ubb34\ub9ac\ub97c \ub9cc\ub4e4\uace0 \uc788\uc744 \ub54c \uc554\ucef7\uc744 \ud638\uc704\ud558\ub294 \uc5ed\ud560\uc744 \ud55c\ub2e4.", "sentence_answer": "\ud559\uacc4\uc5d0\uc11c\ub294 \uadf8 \uc9c0\uc704\uac00 \ub098\uc774 \uc5d0", "paragraph_id": "6520245-3-1", "paragraph_question": "question: \uc544\ub2e5\uc2a4 \ubb34\ub9ac\uc5d0\uc11c \uc9c0\uc704\uac00 \uacb0\uc815\ub418\ub294 \uae30\uc900\uc740 \ubb34\uc5c7\uc778\uac00?, context: \uc544\ub2e5\uc2a4\ub294 \uc8fc\ub85c \ubc24\uc5d0 \ubd84\uc8fc\ud558\uac8c \ud65c\ub3d9\ud558\ub294 \uc57c\ud589\uc131 \ub3d9\ubb3c\ub85c, \ud2b9\ud788 \uc5ec\ub984\uc5d0 \ub354 \ud65c\ubc1c\ud574\uc9c4\ub2e4. \ub0ae \ub3d9\uc548\uc5d0\ub294 \uadf8\ub298\uc9c4 \uc790\ub9ac\ub97c \uc7a1\uace0 \ubaa8\ub798 \uad6c\ub369\uc774\ub97c \ud30c\uace0 \uc270\ub2e4. \uc774 \uad6c\ub369\uc774\ub294 \ud587\ube5b\ub3c4 \ub9c9\uc544 \uc8fc\uc9c0\ub9cc \uc774\ub530\uae08 \ubaa8\ub798 \ud3ed\ud48d\uc744 \ud53c\ud560 \uc218 \uc788\ub294 \ubc29\uacf5\ud638 \uc5ed\ud560\ub3c4 \ud55c\ub2e4. \ub2e4\uc218\uc758 \uc218\ucef7\uacfc \uc554\ucef7\uc774 \ud3ec\ud568\ub41c \ubb34\ub9ac\ub97c \uc9c0\uc73c\uba70, \uaddc\ubaa8\ub294 5-20\ub9c8\ub9ac \uc815\ub3c4\uc774\ub2e4. \ubb34\ub9ac\uac00 \ub9cc\ub4e4\uc5b4\uc9c0\uba74 \uc815\ucc29 \uc0dd\ud65c\uc744 \ud558\uba70 \uc624\ub85c\uc9c0 \uba39\uc774\uac00 \ub2e4 \ub5a8\uc5b4\uc84c\uc744 \ub54c\ub9cc \uc774\ub3d9\ud55c\ub2e4. \uba39\uc774\ub85c \uc0bc\uc744 \ucd08\ubaa9\uc774 \ud48d\ubd80\ud55c \uacf3\uc744 \ucc3e\uc544\uac08 \ub54c\uc5d0\ub294 \uc8fc\ub85c \ube44\uac00 \uc798 \ub0b4\ub9ac\ub294 \ucabd\uc73c\ub85c \uc774\ub3d9\ud558\uba70, \uc5ec\ub984 \ub3d9\uc548\uc5d0 \uc6b0\uae30\uac00 \ucc3e\uc544\uc624\ub294 \uc9c0\uc5ed\uc73c\ub85c \ubb34\ub9ac\ub97c \ucc3e\uc744 \uc218 \uc788\ub2e4. \ub69c\ub837\ud55c \uc0ac\ud68c \uccb4\uacc4\ub97c \uc774\ub8e8\uace0 \uc788\uc73c\uba70, \ud559\uacc4\uc5d0\uc11c\ub294 \uadf8 \uc9c0\uc704\uac00 \ub098\uc774\uc5d0 \uc758\ud574\uc11c \uacb0\uc815\ub41c\ub2e4\uace0 \ubcf4\uace0 \uc788\ub2e4. \ubb34\ub9ac\uc758 \ub300\uc7a5\uc740 \uac00\uc7a5 \ub098\uc774\ub4e0 \uc554\ucef7\uc73c\ub85c \uc815\ud574\uc9c0\uba70, \uc774\ub294 \ub3d9\ubb3c\uc6d0 \ub4f1 \uc0ac\uc721 \uc0c1\ud0dc\uc5d0\uc11c\ub3c4 \ub098\ud0c0\ub098\ub294 \ud604\uc0c1\uc774\ub2e4. \uc218\ucef7\ub4e4\uc740 \ub300\ubd80\ubd84 \uc138\ub825\uad8c\uc744 \uac00\uc9c0\uace0 \uc788\uc73c\uba70, \uc554\ucef7\ub4e4\uc774 \uc790\uc2e0\uc774 \uc774\ub044\ub294 \ubb34\ub9ac\ub97c \ub9cc\ub4e4\uace0 \uc788\uc744 \ub54c \uc554\ucef7\uc744 \ud638\uc704\ud558\ub294 \uc5ed\ud560\uc744 \ud55c\ub2e4."} {"question": "\ub85c\ud070\ub864\uc758 \ud558\uc704\uc7a5\ub974\ub85c \ub85c\ud070\ub864\uacfc \ud790\ube4c\ub9ac \ucee8\ud2b8\ub9ac \ubba4\uc9c1\uc774 \uacb0\ud569\ud55c \uc7a5\ub974\ub294 \ubb34\uc5c7\uc778\uac00?", "paragraph": "\ub85c\ud070\ub864\uc740 \uc218\ub9ce\uc740 \ud558\uc704\uc7a5\ub974\ub85c \ub098\ub204\uc5b4\uc9c4\ub2e4. '\ub85c\ud070\ub864'\uacfc \ud790\ube4c\ub9ac \ucee8\ud2b8\ub9ac \ubba4\uc9c1'\uc774 \ud569\uccd0\uc838 \ub85c\ucee4\ube4c\ub9ac\uac00 \ub418\uc5c8\uc73c\uba70, \uc774\ub294 1950\ub144\ub300 \uc911\ubc18 \uce7c \ud37c\ud0a8\uc2a4, \uc81c\ub9ac \ub9ac \ub8e8\uc774\uc2a4\ubc84\ub514 \ud640\ub9ac\uc640 \uc0c1\uc5c5\uc801\uc73c\ub85c \ud070 \uc131\uacf5\uc744 \uac70\ub454 '\uc5d8\ube44\uc2a4 \ud504\ub808\uc2ac\ub9ac'\uac00 \ub300\ud45c\uc801\uc774\ub2e4 \uadf8\uc640 \ubc18\ub300\ub85c \ub450\uc651\uc740 \uc5ec\ub7ec \ud30c\ud2b8\uc758 \ubcf4\uceec \uac04\uc758 \uc870\ud654\uc640 \uc758\ubbf8\uc5c6\ub294 \uc74c(\ub098\uc911\uc5d0 \uc7a5\ub974\uac00 \uc0dd\uae34\ub2e4)\ub97c \uc911\uc694\ud558\uac8c \uc0dd\uac01\ud558\uc600\ub2e4.\uc774\ub294 1930~1940\ub144\ub300 \uc0ac\uc774 \ud751\uc778 \ubcf4\uceec\ub4e4\uc774 \uac04\ub2e8\ud55c \uae30\uad6c\ub9cc\uc744 \uc0ac\uc6a9\ud55c \uac83\uc5d0\uc11c \uc720\ub798\ud55c\ub2e4. The Crows, The Penguins, The El Dorados\uc640 The Turbans\uc740 \ubaa8\ub450 \ud070 \uc131\uacf5\uc744 \uac70\ub450\uc5c8\uace0, The Platters\uc758 The Great Pretender(1955)\uc640 Yakety Yak\uacfc \uac19\uc774 \uc720\uba38\ub7ec\uc2a4\ud55c \uc74c\uc545\uc744 \uac00\uc9c4 The Coaster\uc5ed\uc2dc \uadf8 \ub2f9\uc2dc \uac00\uc7a5 \uc131\uacf5\ud55c \ub85c\ud070\ub864 \uadf8\ub8f9\uc774\ub77c \ud3c9\uac00\ubc1b\ub294\ub2e4. \uc774 \uc2dc\ub300\uc5d0\ub294 \uc804\uc790\uae30\ud0c0\uc758 \uc131\uc7a5\uacfc Chuck Berry, Link Wray\ub97c \uc8fc\ubaa9\ud574\uc57c \ud55c\ub2e4. \uadf8\ub9ac\uace0 Scootty Moore\uc640 \uac19\uc774 \ub3c5\ud2b9\ud55c \ubc29\uc2dd\uc758 \ub85c\ud070\ub864 \uc2a4\ud0c0\uc77c\uc758 \ubc1c\ub2ec\ub3c4 \uc788\ub2e4. \ub610\ud55c, \uc8fc\uc694 \uc0c1\uc5c5 \ubaa9\uc801\uc73c\ub85c \uc0ac\uc6a9\ub41c \uc194 \uc74c\uc545 \ub610\ud55c \ud070 \uc758\ubbf8\uac00 \uc788\ub2e4. \uadf8\uac83\uc740 \uc601\uac00\uc801 \ubd84\uc704\uae30\uc758 \uc74c\uc545\uacfc \ud31d \uc74c\uc545\uc774 \ud569\uccd0\uc9c4 \ub9ac\ub4ec \uc564 \ube14\ub8e8\uc2a4\uc758 \ubc1c\ub2ec\uc744 \uc77c\uc73c\ucf30\uc73c\uba70, 1950\ub144\ub300 \uc911\ubc18\uc758 Ray CharlesSam Cooke\uac00 \uc774\uc5d0 \ub300\ud55c \ub300\ud45c\uc801\uc778 \uc120\uad6c\uc790\uc774\ub2e4.\ub9c8\ube48 \uac8c\uc774, \uc81c\uc784\uc2a4 \ube0c\ub77c\uc6b4, \uc544\ub808\uc0ac \ud504\ub7ad\ud074\ub9b0, Curtis Mayfield\uc640 \uc2a4\ud2f0\ube44 \uc6d0\ub354 \uc640 \uac19\uc740 1960\ub144\ub300 \ucd08\ubc18\uc758 \uc778\ubb3c\uc740 \ub9ac\ub4ec \uc564 \ube14\ub8e8\uc2a4\uc758 \ucc28\ud2b8\ub97c \ub3c5\uc810\ud558\uace0 \uc788\uc5c8\uc73c\uba70, \ub354 \ub098\uc544\uac00 \uc8fc\uc694 \ud31d \ucc28\ud2b8\uae4c\uc9c0 \ub4e4\uc5b4\uc624\uace0 \uc788\uc5c8\ub2e4. \uc774\ub294 \ubaa8\ud0c0\uc6b4\uacfc \uc2a4\ud0dd\uc2a4 \ub808\ucf54\ub4dc \uc74c\ubc18\uc0ac\uac00 \ub179\uc74c \uc0b0\uc5c5\uc758 \uc8fc\ub958\ub97c \ucc28\uc9c0\ud558\ub294 \ub3d9\uc548 \uc778\uc885\ucc28\ubcc4 \ucca0\ud3d0\uc5d0 \ud070 \ub3c4\uc6c0\uc744 \uc8fc\uc5c8\ub2e4. \ub450\uc651\uacfc \uac78\uad6c\ub8f9\uc758 \uc870\ud654\ub97c \ud3ec\ud568\ud55c \ubaa8\ub4e0 \uc694\uc18c\ub4e4\uc774 \uc870\ud654\ub97c \uc774\ub8e8\ub294 \ub3d9\uc548 \uc2e0\uc911\ud558\uac8c \ub9cc\ub4e4\uc5b4\uc9c4\ub2e4.", "answer": "\ub85c\ucee4\ube4c\ub9ac", "sentence": "'\ub85c\ud070\ub864'\uacfc \ud790\ube4c\ub9ac \ucee8\ud2b8\ub9ac \ubba4\uc9c1'\uc774 \ud569\uccd0\uc838 \ub85c\ucee4\ube4c\ub9ac \uac00 \ub418\uc5c8\uc73c\uba70, \uc774\ub294 1950\ub144\ub300 \uc911\ubc18 \uce7c \ud37c\ud0a8\uc2a4, \uc81c\ub9ac \ub9ac \ub8e8\uc774\uc2a4\ubc84\ub514 \ud640\ub9ac\uc640 \uc0c1\uc5c5\uc801\uc73c\ub85c \ud070 \uc131\uacf5\uc744 \uac70\ub454 '\uc5d8\ube44\uc2a4 \ud504\ub808\uc2ac\ub9ac'\uac00 \ub300\ud45c\uc801\uc774\ub2e4 \uadf8\uc640 \ubc18\ub300\ub85c \ub450\uc651\uc740 \uc5ec\ub7ec \ud30c\ud2b8\uc758 \ubcf4\uceec \uac04\uc758 \uc870\ud654\uc640 \uc758\ubbf8\uc5c6\ub294 \uc74c(\ub098\uc911\uc5d0 \uc7a5\ub974\uac00 \uc0dd\uae34\ub2e4)\ub97c \uc911\uc694\ud558\uac8c \uc0dd\uac01\ud558\uc600\ub2e4.\uc774\ub294 1930~1940\ub144\ub300 \uc0ac\uc774 \ud751\uc778 \ubcf4\uceec\ub4e4\uc774 \uac04\ub2e8\ud55c \uae30\uad6c\ub9cc\uc744 \uc0ac\uc6a9\ud55c \uac83\uc5d0\uc11c \uc720\ub798\ud55c\ub2e4.", "paragraph_sentence": "\ub85c\ud070\ub864\uc740 \uc218\ub9ce\uc740 \ud558\uc704\uc7a5\ub974\ub85c \ub098\ub204\uc5b4\uc9c4\ub2e4. '\ub85c\ud070\ub864'\uacfc \ud790\ube4c\ub9ac \ucee8\ud2b8\ub9ac \ubba4\uc9c1'\uc774 \ud569\uccd0\uc838 \ub85c\ucee4\ube4c\ub9ac \uac00 \ub418\uc5c8\uc73c\uba70, \uc774\ub294 1950\ub144\ub300 \uc911\ubc18 \uce7c \ud37c\ud0a8\uc2a4, \uc81c\ub9ac \ub9ac \ub8e8\uc774\uc2a4\ubc84\ub514 \ud640\ub9ac\uc640 \uc0c1\uc5c5\uc801\uc73c\ub85c \ud070 \uc131\uacf5\uc744 \uac70\ub454 '\uc5d8\ube44\uc2a4 \ud504\ub808\uc2ac\ub9ac'\uac00 \ub300\ud45c\uc801\uc774\ub2e4 \uadf8\uc640 \ubc18\ub300\ub85c \ub450\uc651\uc740 \uc5ec\ub7ec \ud30c\ud2b8\uc758 \ubcf4\uceec \uac04\uc758 \uc870\ud654\uc640 \uc758\ubbf8\uc5c6\ub294 \uc74c(\ub098\uc911\uc5d0 \uc7a5\ub974\uac00 \uc0dd\uae34\ub2e4)\ub97c \uc911\uc694\ud558\uac8c \uc0dd\uac01\ud558\uc600\ub2e4.\uc774\ub294 1930~1940\ub144\ub300 \uc0ac\uc774 \ud751\uc778 \ubcf4\uceec\ub4e4\uc774 \uac04\ub2e8\ud55c \uae30\uad6c\ub9cc\uc744 \uc0ac\uc6a9\ud55c \uac83\uc5d0\uc11c \uc720\ub798\ud55c\ub2e4. The Crows, The Penguins, The El Dorados\uc640 The Turbans\uc740 \ubaa8\ub450 \ud070 \uc131\uacf5\uc744 \uac70\ub450\uc5c8\uace0, The Platters\uc758 The Great Pretender(1955)\uc640 Yakety Yak\uacfc \uac19\uc774 \uc720\uba38\ub7ec\uc2a4\ud55c \uc74c\uc545\uc744 \uac00\uc9c4 The Coaster\uc5ed\uc2dc \uadf8 \ub2f9\uc2dc \uac00\uc7a5 \uc131\uacf5\ud55c \ub85c\ud070\ub864 \uadf8\ub8f9\uc774\ub77c \ud3c9\uac00\ubc1b\ub294\ub2e4. \uc774 \uc2dc\ub300\uc5d0\ub294 \uc804\uc790\uae30\ud0c0\uc758 \uc131\uc7a5\uacfc Chuck Berry, Link Wray\ub97c \uc8fc\ubaa9\ud574\uc57c \ud55c\ub2e4. \uadf8\ub9ac\uace0 Scootty Moore\uc640 \uac19\uc774 \ub3c5\ud2b9\ud55c \ubc29\uc2dd\uc758 \ub85c\ud070\ub864 \uc2a4\ud0c0\uc77c\uc758 \ubc1c\ub2ec\ub3c4 \uc788\ub2e4. \ub610\ud55c, \uc8fc\uc694 \uc0c1\uc5c5 \ubaa9\uc801\uc73c\ub85c \uc0ac\uc6a9\ub41c \uc194 \uc74c\uc545 \ub610\ud55c \ud070 \uc758\ubbf8\uac00 \uc788\ub2e4. \uadf8\uac83\uc740 \uc601\uac00\uc801 \ubd84\uc704\uae30\uc758 \uc74c\uc545\uacfc \ud31d \uc74c\uc545\uc774 \ud569\uccd0\uc9c4 \ub9ac\ub4ec \uc564 \ube14\ub8e8\uc2a4\uc758 \ubc1c\ub2ec\uc744 \uc77c\uc73c\ucf30\uc73c\uba70, 1950\ub144\ub300 \uc911\ubc18\uc758 Ray CharlesSam Cooke\uac00 \uc774\uc5d0 \ub300\ud55c \ub300\ud45c\uc801\uc778 \uc120\uad6c\uc790\uc774\ub2e4.\ub9c8\ube48 \uac8c\uc774, \uc81c\uc784\uc2a4 \ube0c\ub77c\uc6b4, \uc544\ub808\uc0ac \ud504\ub7ad\ud074\ub9b0, Curtis Mayfield\uc640 \uc2a4\ud2f0\ube44 \uc6d0\ub354 \uc640 \uac19\uc740 1960\ub144\ub300 \ucd08\ubc18\uc758 \uc778\ubb3c\uc740 \ub9ac\ub4ec \uc564 \ube14\ub8e8\uc2a4\uc758 \ucc28\ud2b8\ub97c \ub3c5\uc810\ud558\uace0 \uc788\uc5c8\uc73c\uba70, \ub354 \ub098\uc544\uac00 \uc8fc\uc694 \ud31d \ucc28\ud2b8\uae4c\uc9c0 \ub4e4\uc5b4\uc624\uace0 \uc788\uc5c8\ub2e4. \uc774\ub294 \ubaa8\ud0c0\uc6b4\uacfc \uc2a4\ud0dd\uc2a4 \ub808\ucf54\ub4dc \uc74c\ubc18\uc0ac\uac00 \ub179\uc74c \uc0b0\uc5c5\uc758 \uc8fc\ub958\ub97c \ucc28\uc9c0\ud558\ub294 \ub3d9\uc548 \uc778\uc885\ucc28\ubcc4 \ucca0\ud3d0\uc5d0 \ud070 \ub3c4\uc6c0\uc744 \uc8fc\uc5c8\ub2e4. \ub450\uc651\uacfc \uac78\uad6c\ub8f9\uc758 \uc870\ud654\ub97c \ud3ec\ud568\ud55c \ubaa8\ub4e0 \uc694\uc18c\ub4e4\uc774 \uc870\ud654\ub97c \uc774\ub8e8\ub294 \ub3d9\uc548 \uc2e0\uc911\ud558\uac8c \ub9cc\ub4e4\uc5b4\uc9c4\ub2e4.", "paragraph_answer": "\ub85c\ud070\ub864\uc740 \uc218\ub9ce\uc740 \ud558\uc704\uc7a5\ub974\ub85c \ub098\ub204\uc5b4\uc9c4\ub2e4. '\ub85c\ud070\ub864'\uacfc \ud790\ube4c\ub9ac \ucee8\ud2b8\ub9ac \ubba4\uc9c1'\uc774 \ud569\uccd0\uc838 \ub85c\ucee4\ube4c\ub9ac \uac00 \ub418\uc5c8\uc73c\uba70, \uc774\ub294 1950\ub144\ub300 \uc911\ubc18 \uce7c \ud37c\ud0a8\uc2a4, \uc81c\ub9ac \ub9ac \ub8e8\uc774\uc2a4\ubc84\ub514 \ud640\ub9ac\uc640 \uc0c1\uc5c5\uc801\uc73c\ub85c \ud070 \uc131\uacf5\uc744 \uac70\ub454 '\uc5d8\ube44\uc2a4 \ud504\ub808\uc2ac\ub9ac'\uac00 \ub300\ud45c\uc801\uc774\ub2e4 \uadf8\uc640 \ubc18\ub300\ub85c \ub450\uc651\uc740 \uc5ec\ub7ec \ud30c\ud2b8\uc758 \ubcf4\uceec \uac04\uc758 \uc870\ud654\uc640 \uc758\ubbf8\uc5c6\ub294 \uc74c(\ub098\uc911\uc5d0 \uc7a5\ub974\uac00 \uc0dd\uae34\ub2e4)\ub97c \uc911\uc694\ud558\uac8c \uc0dd\uac01\ud558\uc600\ub2e4.\uc774\ub294 1930~1940\ub144\ub300 \uc0ac\uc774 \ud751\uc778 \ubcf4\uceec\ub4e4\uc774 \uac04\ub2e8\ud55c \uae30\uad6c\ub9cc\uc744 \uc0ac\uc6a9\ud55c \uac83\uc5d0\uc11c \uc720\ub798\ud55c\ub2e4. The Crows, The Penguins, The El Dorados\uc640 The Turbans\uc740 \ubaa8\ub450 \ud070 \uc131\uacf5\uc744 \uac70\ub450\uc5c8\uace0, The Platters\uc758 The Great Pretender(1955)\uc640 Yakety Yak\uacfc \uac19\uc774 \uc720\uba38\ub7ec\uc2a4\ud55c \uc74c\uc545\uc744 \uac00\uc9c4 The Coaster\uc5ed\uc2dc \uadf8 \ub2f9\uc2dc \uac00\uc7a5 \uc131\uacf5\ud55c \ub85c\ud070\ub864 \uadf8\ub8f9\uc774\ub77c \ud3c9\uac00\ubc1b\ub294\ub2e4. \uc774 \uc2dc\ub300\uc5d0\ub294 \uc804\uc790\uae30\ud0c0\uc758 \uc131\uc7a5\uacfc Chuck Berry, Link Wray\ub97c \uc8fc\ubaa9\ud574\uc57c \ud55c\ub2e4. \uadf8\ub9ac\uace0 Scootty Moore\uc640 \uac19\uc774 \ub3c5\ud2b9\ud55c \ubc29\uc2dd\uc758 \ub85c\ud070\ub864 \uc2a4\ud0c0\uc77c\uc758 \ubc1c\ub2ec\ub3c4 \uc788\ub2e4. \ub610\ud55c, \uc8fc\uc694 \uc0c1\uc5c5 \ubaa9\uc801\uc73c\ub85c \uc0ac\uc6a9\ub41c \uc194 \uc74c\uc545 \ub610\ud55c \ud070 \uc758\ubbf8\uac00 \uc788\ub2e4. \uadf8\uac83\uc740 \uc601\uac00\uc801 \ubd84\uc704\uae30\uc758 \uc74c\uc545\uacfc \ud31d \uc74c\uc545\uc774 \ud569\uccd0\uc9c4 \ub9ac\ub4ec \uc564 \ube14\ub8e8\uc2a4\uc758 \ubc1c\ub2ec\uc744 \uc77c\uc73c\ucf30\uc73c\uba70, 1950\ub144\ub300 \uc911\ubc18\uc758 Ray CharlesSam Cooke\uac00 \uc774\uc5d0 \ub300\ud55c \ub300\ud45c\uc801\uc778 \uc120\uad6c\uc790\uc774\ub2e4.\ub9c8\ube48 \uac8c\uc774, \uc81c\uc784\uc2a4 \ube0c\ub77c\uc6b4, \uc544\ub808\uc0ac \ud504\ub7ad\ud074\ub9b0, Curtis Mayfield\uc640 \uc2a4\ud2f0\ube44 \uc6d0\ub354 \uc640 \uac19\uc740 1960\ub144\ub300 \ucd08\ubc18\uc758 \uc778\ubb3c\uc740 \ub9ac\ub4ec \uc564 \ube14\ub8e8\uc2a4\uc758 \ucc28\ud2b8\ub97c \ub3c5\uc810\ud558\uace0 \uc788\uc5c8\uc73c\uba70, \ub354 \ub098\uc544\uac00 \uc8fc\uc694 \ud31d \ucc28\ud2b8\uae4c\uc9c0 \ub4e4\uc5b4\uc624\uace0 \uc788\uc5c8\ub2e4. \uc774\ub294 \ubaa8\ud0c0\uc6b4\uacfc \uc2a4\ud0dd\uc2a4 \ub808\ucf54\ub4dc \uc74c\ubc18\uc0ac\uac00 \ub179\uc74c \uc0b0\uc5c5\uc758 \uc8fc\ub958\ub97c \ucc28\uc9c0\ud558\ub294 \ub3d9\uc548 \uc778\uc885\ucc28\ubcc4 \ucca0\ud3d0\uc5d0 \ud070 \ub3c4\uc6c0\uc744 \uc8fc\uc5c8\ub2e4. \ub450\uc651\uacfc \uac78\uad6c\ub8f9\uc758 \uc870\ud654\ub97c \ud3ec\ud568\ud55c \ubaa8\ub4e0 \uc694\uc18c\ub4e4\uc774 \uc870\ud654\ub97c \uc774\ub8e8\ub294 \ub3d9\uc548 \uc2e0\uc911\ud558\uac8c \ub9cc\ub4e4\uc5b4\uc9c4\ub2e4.", "sentence_answer": "'\ub85c\ud070\ub864'\uacfc \ud790\ube4c\ub9ac \ucee8\ud2b8\ub9ac \ubba4\uc9c1'\uc774 \ud569\uccd0\uc838 \ub85c\ucee4\ube4c\ub9ac \uac00 \ub418\uc5c8\uc73c\uba70, \uc774\ub294 1950\ub144\ub300 \uc911\ubc18 \uce7c \ud37c\ud0a8\uc2a4, \uc81c\ub9ac \ub9ac \ub8e8\uc774\uc2a4\ubc84\ub514 \ud640\ub9ac\uc640 \uc0c1\uc5c5\uc801\uc73c\ub85c \ud070 \uc131\uacf5\uc744 \uac70\ub454 '\uc5d8\ube44\uc2a4 \ud504\ub808\uc2ac\ub9ac'\uac00 \ub300\ud45c\uc801\uc774\ub2e4 \uadf8\uc640 \ubc18\ub300\ub85c \ub450\uc651\uc740 \uc5ec\ub7ec \ud30c\ud2b8\uc758 \ubcf4\uceec \uac04\uc758 \uc870\ud654\uc640 \uc758\ubbf8\uc5c6\ub294 \uc74c(\ub098\uc911\uc5d0 \uc7a5\ub974\uac00 \uc0dd\uae34\ub2e4)\ub97c \uc911\uc694\ud558\uac8c \uc0dd\uac01\ud558\uc600\ub2e4.\uc774\ub294 1930~1940\ub144\ub300 \uc0ac\uc774 \ud751\uc778 \ubcf4\uceec\ub4e4\uc774 \uac04\ub2e8\ud55c \uae30\uad6c\ub9cc\uc744 \uc0ac\uc6a9\ud55c \uac83\uc5d0\uc11c \uc720\ub798\ud55c\ub2e4.", "paragraph_id": "5782615-1-0", "paragraph_question": "question: \ub85c\ud070\ub864\uc758 \ud558\uc704\uc7a5\ub974\ub85c \ub85c\ud070\ub864\uacfc \ud790\ube4c\ub9ac \ucee8\ud2b8\ub9ac \ubba4\uc9c1\uc774 \uacb0\ud569\ud55c \uc7a5\ub974\ub294 \ubb34\uc5c7\uc778\uac00?, context: \ub85c\ud070\ub864\uc740 \uc218\ub9ce\uc740 \ud558\uc704\uc7a5\ub974\ub85c \ub098\ub204\uc5b4\uc9c4\ub2e4. '\ub85c\ud070\ub864'\uacfc \ud790\ube4c\ub9ac \ucee8\ud2b8\ub9ac \ubba4\uc9c1'\uc774 \ud569\uccd0\uc838 \ub85c\ucee4\ube4c\ub9ac\uac00 \ub418\uc5c8\uc73c\uba70, \uc774\ub294 1950\ub144\ub300 \uc911\ubc18 \uce7c \ud37c\ud0a8\uc2a4, \uc81c\ub9ac \ub9ac \ub8e8\uc774\uc2a4\ubc84\ub514 \ud640\ub9ac\uc640 \uc0c1\uc5c5\uc801\uc73c\ub85c \ud070 \uc131\uacf5\uc744 \uac70\ub454 '\uc5d8\ube44\uc2a4 \ud504\ub808\uc2ac\ub9ac'\uac00 \ub300\ud45c\uc801\uc774\ub2e4 \uadf8\uc640 \ubc18\ub300\ub85c \ub450\uc651\uc740 \uc5ec\ub7ec \ud30c\ud2b8\uc758 \ubcf4\uceec \uac04\uc758 \uc870\ud654\uc640 \uc758\ubbf8\uc5c6\ub294 \uc74c(\ub098\uc911\uc5d0 \uc7a5\ub974\uac00 \uc0dd\uae34\ub2e4)\ub97c \uc911\uc694\ud558\uac8c \uc0dd\uac01\ud558\uc600\ub2e4.\uc774\ub294 1930~1940\ub144\ub300 \uc0ac\uc774 \ud751\uc778 \ubcf4\uceec\ub4e4\uc774 \uac04\ub2e8\ud55c \uae30\uad6c\ub9cc\uc744 \uc0ac\uc6a9\ud55c \uac83\uc5d0\uc11c \uc720\ub798\ud55c\ub2e4. The Crows, The Penguins, The El Dorados\uc640 The Turbans\uc740 \ubaa8\ub450 \ud070 \uc131\uacf5\uc744 \uac70\ub450\uc5c8\uace0, The Platters\uc758 The Great Pretender(1955)\uc640 Yakety Yak\uacfc \uac19\uc774 \uc720\uba38\ub7ec\uc2a4\ud55c \uc74c\uc545\uc744 \uac00\uc9c4 The Coaster\uc5ed\uc2dc \uadf8 \ub2f9\uc2dc \uac00\uc7a5 \uc131\uacf5\ud55c \ub85c\ud070\ub864 \uadf8\ub8f9\uc774\ub77c \ud3c9\uac00\ubc1b\ub294\ub2e4. \uc774 \uc2dc\ub300\uc5d0\ub294 \uc804\uc790\uae30\ud0c0\uc758 \uc131\uc7a5\uacfc Chuck Berry, Link Wray\ub97c \uc8fc\ubaa9\ud574\uc57c \ud55c\ub2e4. \uadf8\ub9ac\uace0 Scootty Moore\uc640 \uac19\uc774 \ub3c5\ud2b9\ud55c \ubc29\uc2dd\uc758 \ub85c\ud070\ub864 \uc2a4\ud0c0\uc77c\uc758 \ubc1c\ub2ec\ub3c4 \uc788\ub2e4. \ub610\ud55c, \uc8fc\uc694 \uc0c1\uc5c5 \ubaa9\uc801\uc73c\ub85c \uc0ac\uc6a9\ub41c \uc194 \uc74c\uc545 \ub610\ud55c \ud070 \uc758\ubbf8\uac00 \uc788\ub2e4. \uadf8\uac83\uc740 \uc601\uac00\uc801 \ubd84\uc704\uae30\uc758 \uc74c\uc545\uacfc \ud31d \uc74c\uc545\uc774 \ud569\uccd0\uc9c4 \ub9ac\ub4ec \uc564 \ube14\ub8e8\uc2a4\uc758 \ubc1c\ub2ec\uc744 \uc77c\uc73c\ucf30\uc73c\uba70, 1950\ub144\ub300 \uc911\ubc18\uc758 Ray CharlesSam Cooke\uac00 \uc774\uc5d0 \ub300\ud55c \ub300\ud45c\uc801\uc778 \uc120\uad6c\uc790\uc774\ub2e4.\ub9c8\ube48 \uac8c\uc774, \uc81c\uc784\uc2a4 \ube0c\ub77c\uc6b4, \uc544\ub808\uc0ac \ud504\ub7ad\ud074\ub9b0, Curtis Mayfield\uc640 \uc2a4\ud2f0\ube44 \uc6d0\ub354 \uc640 \uac19\uc740 1960\ub144\ub300 \ucd08\ubc18\uc758 \uc778\ubb3c\uc740 \ub9ac\ub4ec \uc564 \ube14\ub8e8\uc2a4\uc758 \ucc28\ud2b8\ub97c \ub3c5\uc810\ud558\uace0 \uc788\uc5c8\uc73c\uba70, \ub354 \ub098\uc544\uac00 \uc8fc\uc694 \ud31d \ucc28\ud2b8\uae4c\uc9c0 \ub4e4\uc5b4\uc624\uace0 \uc788\uc5c8\ub2e4. \uc774\ub294 \ubaa8\ud0c0\uc6b4\uacfc \uc2a4\ud0dd\uc2a4 \ub808\ucf54\ub4dc \uc74c\ubc18\uc0ac\uac00 \ub179\uc74c \uc0b0\uc5c5\uc758 \uc8fc\ub958\ub97c \ucc28\uc9c0\ud558\ub294 \ub3d9\uc548 \uc778\uc885\ucc28\ubcc4 \ucca0\ud3d0\uc5d0 \ud070 \ub3c4\uc6c0\uc744 \uc8fc\uc5c8\ub2e4. \ub450\uc651\uacfc \uac78\uad6c\ub8f9\uc758 \uc870\ud654\ub97c \ud3ec\ud568\ud55c \ubaa8\ub4e0 \uc694\uc18c\ub4e4\uc774 \uc870\ud654\ub97c \uc774\ub8e8\ub294 \ub3d9\uc548 \uc2e0\uc911\ud558\uac8c \ub9cc\ub4e4\uc5b4\uc9c4\ub2e4."} {"question": "\ud0c4\ud654\uc218\uc18c \uc2ac\ub7ec\uc9c0\ub098 \ud1a8\ub9b0\uc740 \uc720\uae30\ud654\ud569\ubb3c\uc774 \uc5b4\ub5a4 \ube5b\uc744 \ucb10\uba74 \ub9cc\ub4e4\uc5b4\uc9c0\ub294\uac00?", "paragraph": "SMARTS \ub9dd\uc6d0\uacbd\uc744 \ud1b5\ud55c \uad00\uce21 \uacb0\uacfc \uc138\ub4dc\ub098\ub294 \uac00\uc2dc\uad11\uc120 \ub300\uc5ed\uc5d0\uc11c \ud0dc\uc591\uacc4\uc758 \ucc9c\uccb4\ub4e4 \uc911 \uac00\uc7a5 \ubd89\uc740 \ucd95\uc5d0 \uc18d\ud55c\ub2e4\ub294 \uac83\uc774 \ubc1d\ud600\uc84c\ub2e4. \uac70\uc758 \ud654\uc131\ub9cc\ud07c \ubd89\ub2e4. \ucc44\ub4dc \ud2b8\ub8e8\uc9c8\ub85c\uc640 \uadf8 \ub3d9\ub8cc\ub4e4\uc740 \uc138\ub4dc\ub098\uc758 \uc554\uc801\uc0c9\uc774 \ud45c\uba74\uc774 \ud0c4\ud654\uc218\uc18c \uc2ac\ub7ec\uc9c0\ub098 \ud1a8\ub9b0\uc73c\ub85c \ub36e\uc5ec \uc788\uae30 \ub54c\ubb38\uc774\ub77c\ub294 \uc124\uc744 \uc81c\uae30\ud588\ub2e4. \uc774\uac83\ub4e4\uc740 \ubcf4\ub2e4 \ub2e8\uc21c\ud55c \uc720\uae30\ud654\ud569\ubb3c\uc774 \uc790\uc678\uc120 \ubcf5\uc0ac\uc5d0 \uc624\ub798 \ub178\ucd9c\ub428\uc73c\ub85c\uc368 \ub9cc\ub4e4\uc5b4\uc9c4\ub2e4. \ud45c\uba74\uc740 \uc0c9\uae54 \uba74\uc5d0\uc11c\ub098 \ubd84\uad11 \uba74\uc5d0\uc11c\ub098 \uade0\uc9c8\ud558\ub2e4. \uc774\ub294 \uc138\ub4dc\ub098\uac00 \ud0dc\uc591\uc5d0 \uac00\uae4c\uc6b4 \ucc9c\uccb4\ub4e4\uacfc \ub2ec\ub9ac \ub2e4\ub978 \ucc9c\uccb4\uc640 \ucda9\ub3cc\ud55c \uc801\uc774 \uac70\uc758 \uc5c6\uae30 \ub54c\ubb38\uc77c \uac83\uc774\ub2e4. 8405 \uc544\uc2a4\ubcfc\ub8e8\uc2a4\uc758 \uc0ac\ub840\ucc98\ub7fc \uadf8\ub7f0 \ucda9\ub3cc\uc774 \uc77c\uc5b4\ub098\uc57c \uc544\ub798\ucabd\uc758 \ubc1d\uc740 \uc5bc\uc74c\uc9c8 \ubb3c\uc9c8\uc774 \ub4dc\ub7ec\ub098\uc11c \ubc1d\uae30\uc758 \ucc28\uc774\uac00 \uc0dd\uae30\uac8c \ub41c\ub2e4. \uc138\ub4dc\ub098\uc640 \ub610\ub2e4\ub978 \ub9e4\uc6b0 \uba3c \ucc9c\uccb4 \ub450 \uac1c(2006 SQ372 \ubc0f (87269) 2000 OO67)\ub294 \uace0\uc804\uc801 \uce74\uc774\ud37c\ub300 \ucc9c\uccb4 \ubc0f \uc13c\ud0c0\uc6b0\ub8e8\uc2a4\uad70 \uc18c\ud589\uc131 5145 \ud3f4\ub8e8\uc2a4\uc640 \uac19\uc740 \uc0c9\uae54\uc744 \uacf5\uc720\ud558\ub294\ub370, \uc5b4\uca4c\uba74 \uadf8 \uae30\uc6d0\uc774 \uc720\uc0ac\ud560 \uc218\ub3c4 \uc788\ub2e4.", "answer": "\uc790\uc678\uc120 \ubcf5\uc0ac", "sentence": "\uc774\uac83\ub4e4\uc740 \ubcf4\ub2e4 \ub2e8\uc21c\ud55c \uc720\uae30\ud654\ud569\ubb3c\uc774 \uc790\uc678\uc120 \ubcf5\uc0ac \uc5d0 \uc624\ub798 \ub178\ucd9c\ub428\uc73c\ub85c\uc368 \ub9cc\ub4e4\uc5b4\uc9c4\ub2e4.", "paragraph_sentence": "SMARTS \ub9dd\uc6d0\uacbd\uc744 \ud1b5\ud55c \uad00\uce21 \uacb0\uacfc \uc138\ub4dc\ub098\ub294 \uac00\uc2dc\uad11\uc120 \ub300\uc5ed\uc5d0\uc11c \ud0dc\uc591\uacc4\uc758 \ucc9c\uccb4\ub4e4 \uc911 \uac00\uc7a5 \ubd89\uc740 \ucd95\uc5d0 \uc18d\ud55c\ub2e4\ub294 \uac83\uc774 \ubc1d\ud600\uc84c\ub2e4. \uac70\uc758 \ud654\uc131\ub9cc\ud07c \ubd89\ub2e4. \ucc44\ub4dc \ud2b8\ub8e8\uc9c8\ub85c\uc640 \uadf8 \ub3d9\ub8cc\ub4e4\uc740 \uc138\ub4dc\ub098\uc758 \uc554\uc801\uc0c9\uc774 \ud45c\uba74\uc774 \ud0c4\ud654\uc218\uc18c \uc2ac\ub7ec\uc9c0\ub098 \ud1a8\ub9b0\uc73c\ub85c \ub36e\uc5ec \uc788\uae30 \ub54c\ubb38\uc774\ub77c\ub294 \uc124\uc744 \uc81c\uae30\ud588\ub2e4. \uc774\uac83\ub4e4\uc740 \ubcf4\ub2e4 \ub2e8\uc21c\ud55c \uc720\uae30\ud654\ud569\ubb3c\uc774 \uc790\uc678\uc120 \ubcf5\uc0ac \uc5d0 \uc624\ub798 \ub178\ucd9c\ub428\uc73c\ub85c\uc368 \ub9cc\ub4e4\uc5b4\uc9c4\ub2e4. \ud45c\uba74\uc740 \uc0c9\uae54 \uba74\uc5d0\uc11c\ub098 \ubd84\uad11 \uba74\uc5d0\uc11c\ub098 \uade0\uc9c8\ud558\ub2e4. \uc774\ub294 \uc138\ub4dc\ub098\uac00 \ud0dc\uc591\uc5d0 \uac00\uae4c\uc6b4 \ucc9c\uccb4\ub4e4\uacfc \ub2ec\ub9ac \ub2e4\ub978 \ucc9c\uccb4\uc640 \ucda9\ub3cc\ud55c \uc801\uc774 \uac70\uc758 \uc5c6\uae30 \ub54c\ubb38\uc77c \uac83\uc774\ub2e4. 8405 \uc544\uc2a4\ubcfc\ub8e8\uc2a4\uc758 \uc0ac\ub840\ucc98\ub7fc \uadf8\ub7f0 \ucda9\ub3cc\uc774 \uc77c\uc5b4\ub098\uc57c \uc544\ub798\ucabd\uc758 \ubc1d\uc740 \uc5bc\uc74c\uc9c8 \ubb3c\uc9c8\uc774 \ub4dc\ub7ec\ub098\uc11c \ubc1d\uae30\uc758 \ucc28\uc774\uac00 \uc0dd\uae30\uac8c \ub41c\ub2e4. \uc138\ub4dc\ub098\uc640 \ub610\ub2e4\ub978 \ub9e4\uc6b0 \uba3c \ucc9c\uccb4 \ub450 \uac1c(2006 SQ372 \ubc0f (87269) 2000 OO67)\ub294 \uace0\uc804\uc801 \uce74\uc774\ud37c\ub300 \ucc9c\uccb4 \ubc0f \uc13c\ud0c0\uc6b0\ub8e8\uc2a4\uad70 \uc18c\ud589\uc131 5145 \ud3f4\ub8e8\uc2a4\uc640 \uac19\uc740 \uc0c9\uae54\uc744 \uacf5\uc720\ud558\ub294\ub370, \uc5b4\uca4c\uba74 \uadf8 \uae30\uc6d0\uc774 \uc720\uc0ac\ud560 \uc218\ub3c4 \uc788\ub2e4.", "paragraph_answer": "SMARTS \ub9dd\uc6d0\uacbd\uc744 \ud1b5\ud55c \uad00\uce21 \uacb0\uacfc \uc138\ub4dc\ub098\ub294 \uac00\uc2dc\uad11\uc120 \ub300\uc5ed\uc5d0\uc11c \ud0dc\uc591\uacc4\uc758 \ucc9c\uccb4\ub4e4 \uc911 \uac00\uc7a5 \ubd89\uc740 \ucd95\uc5d0 \uc18d\ud55c\ub2e4\ub294 \uac83\uc774 \ubc1d\ud600\uc84c\ub2e4. \uac70\uc758 \ud654\uc131\ub9cc\ud07c \ubd89\ub2e4. \ucc44\ub4dc \ud2b8\ub8e8\uc9c8\ub85c\uc640 \uadf8 \ub3d9\ub8cc\ub4e4\uc740 \uc138\ub4dc\ub098\uc758 \uc554\uc801\uc0c9\uc774 \ud45c\uba74\uc774 \ud0c4\ud654\uc218\uc18c \uc2ac\ub7ec\uc9c0\ub098 \ud1a8\ub9b0\uc73c\ub85c \ub36e\uc5ec \uc788\uae30 \ub54c\ubb38\uc774\ub77c\ub294 \uc124\uc744 \uc81c\uae30\ud588\ub2e4. \uc774\uac83\ub4e4\uc740 \ubcf4\ub2e4 \ub2e8\uc21c\ud55c \uc720\uae30\ud654\ud569\ubb3c\uc774 \uc790\uc678\uc120 \ubcf5\uc0ac \uc5d0 \uc624\ub798 \ub178\ucd9c\ub428\uc73c\ub85c\uc368 \ub9cc\ub4e4\uc5b4\uc9c4\ub2e4. \ud45c\uba74\uc740 \uc0c9\uae54 \uba74\uc5d0\uc11c\ub098 \ubd84\uad11 \uba74\uc5d0\uc11c\ub098 \uade0\uc9c8\ud558\ub2e4. \uc774\ub294 \uc138\ub4dc\ub098\uac00 \ud0dc\uc591\uc5d0 \uac00\uae4c\uc6b4 \ucc9c\uccb4\ub4e4\uacfc \ub2ec\ub9ac \ub2e4\ub978 \ucc9c\uccb4\uc640 \ucda9\ub3cc\ud55c \uc801\uc774 \uac70\uc758 \uc5c6\uae30 \ub54c\ubb38\uc77c \uac83\uc774\ub2e4. 8405 \uc544\uc2a4\ubcfc\ub8e8\uc2a4\uc758 \uc0ac\ub840\ucc98\ub7fc \uadf8\ub7f0 \ucda9\ub3cc\uc774 \uc77c\uc5b4\ub098\uc57c \uc544\ub798\ucabd\uc758 \ubc1d\uc740 \uc5bc\uc74c\uc9c8 \ubb3c\uc9c8\uc774 \ub4dc\ub7ec\ub098\uc11c \ubc1d\uae30\uc758 \ucc28\uc774\uac00 \uc0dd\uae30\uac8c \ub41c\ub2e4. \uc138\ub4dc\ub098\uc640 \ub610\ub2e4\ub978 \ub9e4\uc6b0 \uba3c \ucc9c\uccb4 \ub450 \uac1c(2006 SQ372 \ubc0f (87269) 2000 OO67)\ub294 \uace0\uc804\uc801 \uce74\uc774\ud37c\ub300 \ucc9c\uccb4 \ubc0f \uc13c\ud0c0\uc6b0\ub8e8\uc2a4\uad70 \uc18c\ud589\uc131 5145 \ud3f4\ub8e8\uc2a4\uc640 \uac19\uc740 \uc0c9\uae54\uc744 \uacf5\uc720\ud558\ub294\ub370, \uc5b4\uca4c\uba74 \uadf8 \uae30\uc6d0\uc774 \uc720\uc0ac\ud560 \uc218\ub3c4 \uc788\ub2e4.", "sentence_answer": "\uc774\uac83\ub4e4\uc740 \ubcf4\ub2e4 \ub2e8\uc21c\ud55c \uc720\uae30\ud654\ud569\ubb3c\uc774 \uc790\uc678\uc120 \ubcf5\uc0ac \uc5d0 \uc624\ub798 \ub178\ucd9c\ub428\uc73c\ub85c\uc368 \ub9cc\ub4e4\uc5b4\uc9c4\ub2e4.", "paragraph_id": "6568339-4-1", "paragraph_question": "question: \ud0c4\ud654\uc218\uc18c \uc2ac\ub7ec\uc9c0\ub098 \ud1a8\ub9b0\uc740 \uc720\uae30\ud654\ud569\ubb3c\uc774 \uc5b4\ub5a4 \ube5b\uc744 \ucb10\uba74 \ub9cc\ub4e4\uc5b4\uc9c0\ub294\uac00?, context: SMARTS \ub9dd\uc6d0\uacbd\uc744 \ud1b5\ud55c \uad00\uce21 \uacb0\uacfc \uc138\ub4dc\ub098\ub294 \uac00\uc2dc\uad11\uc120 \ub300\uc5ed\uc5d0\uc11c \ud0dc\uc591\uacc4\uc758 \ucc9c\uccb4\ub4e4 \uc911 \uac00\uc7a5 \ubd89\uc740 \ucd95\uc5d0 \uc18d\ud55c\ub2e4\ub294 \uac83\uc774 \ubc1d\ud600\uc84c\ub2e4. \uac70\uc758 \ud654\uc131\ub9cc\ud07c \ubd89\ub2e4. \ucc44\ub4dc \ud2b8\ub8e8\uc9c8\ub85c\uc640 \uadf8 \ub3d9\ub8cc\ub4e4\uc740 \uc138\ub4dc\ub098\uc758 \uc554\uc801\uc0c9\uc774 \ud45c\uba74\uc774 \ud0c4\ud654\uc218\uc18c \uc2ac\ub7ec\uc9c0\ub098 \ud1a8\ub9b0\uc73c\ub85c \ub36e\uc5ec \uc788\uae30 \ub54c\ubb38\uc774\ub77c\ub294 \uc124\uc744 \uc81c\uae30\ud588\ub2e4. \uc774\uac83\ub4e4\uc740 \ubcf4\ub2e4 \ub2e8\uc21c\ud55c \uc720\uae30\ud654\ud569\ubb3c\uc774 \uc790\uc678\uc120 \ubcf5\uc0ac\uc5d0 \uc624\ub798 \ub178\ucd9c\ub428\uc73c\ub85c\uc368 \ub9cc\ub4e4\uc5b4\uc9c4\ub2e4. \ud45c\uba74\uc740 \uc0c9\uae54 \uba74\uc5d0\uc11c\ub098 \ubd84\uad11 \uba74\uc5d0\uc11c\ub098 \uade0\uc9c8\ud558\ub2e4. \uc774\ub294 \uc138\ub4dc\ub098\uac00 \ud0dc\uc591\uc5d0 \uac00\uae4c\uc6b4 \ucc9c\uccb4\ub4e4\uacfc \ub2ec\ub9ac \ub2e4\ub978 \ucc9c\uccb4\uc640 \ucda9\ub3cc\ud55c \uc801\uc774 \uac70\uc758 \uc5c6\uae30 \ub54c\ubb38\uc77c \uac83\uc774\ub2e4. 8405 \uc544\uc2a4\ubcfc\ub8e8\uc2a4\uc758 \uc0ac\ub840\ucc98\ub7fc \uadf8\ub7f0 \ucda9\ub3cc\uc774 \uc77c\uc5b4\ub098\uc57c \uc544\ub798\ucabd\uc758 \ubc1d\uc740 \uc5bc\uc74c\uc9c8 \ubb3c\uc9c8\uc774 \ub4dc\ub7ec\ub098\uc11c \ubc1d\uae30\uc758 \ucc28\uc774\uac00 \uc0dd\uae30\uac8c \ub41c\ub2e4. \uc138\ub4dc\ub098\uc640 \ub610\ub2e4\ub978 \ub9e4\uc6b0 \uba3c \ucc9c\uccb4 \ub450 \uac1c(2006 SQ372 \ubc0f (87269) 2000 OO67)\ub294 \uace0\uc804\uc801 \uce74\uc774\ud37c\ub300 \ucc9c\uccb4 \ubc0f \uc13c\ud0c0\uc6b0\ub8e8\uc2a4\uad70 \uc18c\ud589\uc131 5145 \ud3f4\ub8e8\uc2a4\uc640 \uac19\uc740 \uc0c9\uae54\uc744 \uacf5\uc720\ud558\ub294\ub370, \uc5b4\uca4c\uba74 \uadf8 \uae30\uc6d0\uc774 \uc720\uc0ac\ud560 \uc218\ub3c4 \uc788\ub2e4."} {"question": "\ucd5c\ucd08\ub85c \ube4c\ubcf4\ub4dc \uc0c1\uc704\uad8c\uc5d0 \uc704\uce58\ud55c \ub85c\ud070\ub864 \uc74c\uc545\uc774 \ub41c Rock Around the Clock\uc740 \ub204\uad6c\uc758 \ub178\ub798\uc778\uac00?", "paragraph": "\ucd5c\ucd08\uc758 \ub85c\ud070\ub864 \uc74c\ubc18\uc774 \ubb34\uc5c7\uc778\uac00?\uc5d0 \ub300\ud574\uc11c \ub9ce\uc740 \ub17c\uc7c1\uc774 \uc788\uc5c8\ub2e4. \ud55c \uc608\uce21\uc740 \uc7ad\ud0a4 \ube0c\ub79c\uc2a4\ud134\uacfc \uadf8\uc758 \ub378\ud0c0 \ucea3\uc758 Rocket 88 \uc774\ub2e4(\uc544\uc774\ud06c \ud130\ub108\uc640 \uadf8\uc758 \ubc34\ub4dc The king of Rythmn) \uc774\ub294 1951\ub144 \uba64\ud53c\uc2a4\uc5d0 \uc788\ub294 \uc120 \ub808\ucf54\ub4dc\uc5d0\uc11c \uc0d8 \ud544\ub9bd\uc2a4\uc5d0\uac8c \ub179\uc74c\ub418\uc5c8\ub2e4. \uc5d8\ube44\uc2a4 \ud504\ub808\uc2ac\ub9ac\uac00 Sun Record\uc5d0\uc11c \ub179\uc74c\ud55c \uccab\ubc88\uc9f8 \uc2f1\uae00 That's All Right(1954)\uac00 \uccab\ubc88\uc9f8 \ub85c\ud070\ub864\uc774\ub77c \uc0dd\uac01\ub418\uc5c8\ub2e4. \ud558\uc9c0\ub9cc \ub3d9\uc2dc\uc5d0 \ud6c4\ub300\uc5d0 \ube4c \ud5e4\uc77c\ub9ac\uac00 \ub2e4\ub8ec Big Joe Turner\uc758 Shake, Rattle&Roll\uac00 \uc774\ubbf8 R&B \ube4c\ubcf4\ub4dc \ucc28\ud2b8\uc758 \ub9e8 \uc704\uc5d0 \uc788\uc5c8\ub2e4.\ucd08\uae30 \ub85c\ud070\ub864 \uc21c\uc704\uc5d0\ub294 \ucc99 \ubca0\ub9ac, \ubcf4 \ub514\ub4e4\ub9ac, \ud328\uce20 \ub3c4\ubbf8\ub178 \ub9ac\ud2c0 \ub9ac\ucc98\ub4dc \uc81c\ub9ac \ub9ac \ub8e8\uc774\uc2a4 \uc9c4 \ube48\uc13c\ud2b8\uac00 \uc788\ub2e4. \ube4c \ud5e4\uc77c\ub9ac\uc758 Rock Around the Clock(1955)\ub294 \ucd5c\ucd08\ub85c \ube4c\ubcf4\ub4dc \uc0c1\uc704\uad8c\uc5d0 \uc704\uce58\ud55c \ub85c\ud070\ub864 \uc74c\uc545\uc774 \ub418\uc5c8\uc73c\uba70, \ub77c\ub514\uc624 \ucc28\ud2b8\uac00 \ub418\uc5c8\ub2e4. \uadf8\ub9ac\uace0 \uc774\ub294 \uc0c8\ub86d\uac8c \uc720\ud5a5\ud558\ub294 \ubb38\ud654\ub97c \ub9cc\ub4e4\uc5c8\ub2e4. \uace7 \ub85c\ud070\ub864\uc740 \uc9c0\ub09c 10\ub144\uac04 \ub300\uc911 \uc74c\uc545\uc744 \uc9c0\ubc30\ud574 \uc628 \uc5d0\ub514 \ud53c\uc154, \ud398\ub9ac \ucf54\ubaa8 \ud328\ud2f0 \ud398\uc774\uc9c0\uc640 \uac19\uc740 \ubbf8\uad6d \uc74c\ubc18 \ud310\ub9e4\uc758 \uc8fc\uc694 \uc694\uc18c\uac00 \ub418\uc5c8\ub2e4.", "answer": "\ube4c \ud5e4\uc77c\ub9ac", "sentence": "\ud6c4\ub300\uc5d0 \ube4c \ud5e4\uc77c\ub9ac \uac00 \ub2e4\ub8ec Big Joe Turner\uc758 Shake, Rattle&Roll\uac00 \uc774\ubbf8 R&B \ube4c\ubcf4\ub4dc \ucc28\ud2b8\uc758 \ub9e8 \uc704\uc5d0 \uc788\uc5c8\ub2e4.\ucd08\uae30 \ub85c\ud070\ub864 \uc21c\uc704\uc5d0\ub294 \ucc99 \ubca0\ub9ac, \ubcf4 \ub514\ub4e4\ub9ac, \ud328\uce20 \ub3c4\ubbf8\ub178 \ub9ac\ud2c0 \ub9ac\ucc98\ub4dc \uc81c\ub9ac \ub9ac \ub8e8\uc774\uc2a4 \uc9c4 \ube48\uc13c\ud2b8\uac00 \uc788\ub2e4.", "paragraph_sentence": "\ucd5c\ucd08\uc758 \ub85c\ud070\ub864 \uc74c\ubc18\uc774 \ubb34\uc5c7\uc778\uac00?\uc5d0 \ub300\ud574\uc11c \ub9ce\uc740 \ub17c\uc7c1\uc774 \uc788\uc5c8\ub2e4. \ud55c \uc608\uce21\uc740 \uc7ad\ud0a4 \ube0c\ub79c\uc2a4\ud134\uacfc \uadf8\uc758 \ub378\ud0c0 \ucea3\uc758 Rocket 88 \uc774\ub2e4(\uc544\uc774\ud06c \ud130\ub108\uc640 \uadf8\uc758 \ubc34\ub4dc The king of Rythmn) \uc774\ub294 1951\ub144 \uba64\ud53c\uc2a4\uc5d0 \uc788\ub294 \uc120 \ub808\ucf54\ub4dc\uc5d0\uc11c \uc0d8 \ud544\ub9bd\uc2a4\uc5d0\uac8c \ub179\uc74c\ub418\uc5c8\ub2e4. \uc5d8\ube44\uc2a4 \ud504\ub808\uc2ac\ub9ac\uac00 Sun Record\uc5d0\uc11c \ub179\uc74c\ud55c \uccab\ubc88\uc9f8 \uc2f1\uae00 That's All Right(1954)\uac00 \uccab\ubc88\uc9f8 \ub85c\ud070\ub864\uc774\ub77c \uc0dd\uac01\ub418\uc5c8\ub2e4. \ud558\uc9c0\ub9cc \ub3d9\uc2dc\uc5d0 \ud6c4\ub300\uc5d0 \ube4c \ud5e4\uc77c\ub9ac \uac00 \ub2e4\ub8ec Big Joe Turner\uc758 Shake, Rattle&Roll\uac00 \uc774\ubbf8 R&B \ube4c\ubcf4\ub4dc \ucc28\ud2b8\uc758 \ub9e8 \uc704\uc5d0 \uc788\uc5c8\ub2e4.\ucd08\uae30 \ub85c\ud070\ub864 \uc21c\uc704\uc5d0\ub294 \ucc99 \ubca0\ub9ac, \ubcf4 \ub514\ub4e4\ub9ac, \ud328\uce20 \ub3c4\ubbf8\ub178 \ub9ac\ud2c0 \ub9ac\ucc98\ub4dc \uc81c\ub9ac \ub9ac \ub8e8\uc774\uc2a4 \uc9c4 \ube48\uc13c\ud2b8\uac00 \uc788\ub2e4. \ube4c \ud5e4\uc77c\ub9ac\uc758 Rock Around the Clock(1955)\ub294 \ucd5c\ucd08\ub85c \ube4c\ubcf4\ub4dc \uc0c1\uc704\uad8c\uc5d0 \uc704\uce58\ud55c \ub85c\ud070\ub864 \uc74c\uc545\uc774 \ub418\uc5c8\uc73c\uba70, \ub77c\ub514\uc624 \ucc28\ud2b8\uac00 \ub418\uc5c8\ub2e4. \uadf8\ub9ac\uace0 \uc774\ub294 \uc0c8\ub86d\uac8c \uc720\ud5a5\ud558\ub294 \ubb38\ud654\ub97c \ub9cc\ub4e4\uc5c8\ub2e4. \uace7 \ub85c\ud070\ub864\uc740 \uc9c0\ub09c 10\ub144\uac04 \ub300\uc911 \uc74c\uc545\uc744 \uc9c0\ubc30\ud574 \uc628 \uc5d0\ub514 \ud53c\uc154, \ud398\ub9ac \ucf54\ubaa8 \ud328\ud2f0 \ud398\uc774\uc9c0\uc640 \uac19\uc740 \ubbf8\uad6d \uc74c\ubc18 \ud310\ub9e4\uc758 \uc8fc\uc694 \uc694\uc18c\uac00 \ub418\uc5c8\ub2e4.", "paragraph_answer": "\ucd5c\ucd08\uc758 \ub85c\ud070\ub864 \uc74c\ubc18\uc774 \ubb34\uc5c7\uc778\uac00?\uc5d0 \ub300\ud574\uc11c \ub9ce\uc740 \ub17c\uc7c1\uc774 \uc788\uc5c8\ub2e4. \ud55c \uc608\uce21\uc740 \uc7ad\ud0a4 \ube0c\ub79c\uc2a4\ud134\uacfc \uadf8\uc758 \ub378\ud0c0 \ucea3\uc758 Rocket 88 \uc774\ub2e4(\uc544\uc774\ud06c \ud130\ub108\uc640 \uadf8\uc758 \ubc34\ub4dc The king of Rythmn) \uc774\ub294 1951\ub144 \uba64\ud53c\uc2a4\uc5d0 \uc788\ub294 \uc120 \ub808\ucf54\ub4dc\uc5d0\uc11c \uc0d8 \ud544\ub9bd\uc2a4\uc5d0\uac8c \ub179\uc74c\ub418\uc5c8\ub2e4. \uc5d8\ube44\uc2a4 \ud504\ub808\uc2ac\ub9ac\uac00 Sun Record\uc5d0\uc11c \ub179\uc74c\ud55c \uccab\ubc88\uc9f8 \uc2f1\uae00 That's All Right(1954)\uac00 \uccab\ubc88\uc9f8 \ub85c\ud070\ub864\uc774\ub77c \uc0dd\uac01\ub418\uc5c8\ub2e4. \ud558\uc9c0\ub9cc \ub3d9\uc2dc\uc5d0 \ud6c4\ub300\uc5d0 \ube4c \ud5e4\uc77c\ub9ac \uac00 \ub2e4\ub8ec Big Joe Turner\uc758 Shake, Rattle&Roll\uac00 \uc774\ubbf8 R&B \ube4c\ubcf4\ub4dc \ucc28\ud2b8\uc758 \ub9e8 \uc704\uc5d0 \uc788\uc5c8\ub2e4.\ucd08\uae30 \ub85c\ud070\ub864 \uc21c\uc704\uc5d0\ub294 \ucc99 \ubca0\ub9ac, \ubcf4 \ub514\ub4e4\ub9ac, \ud328\uce20 \ub3c4\ubbf8\ub178 \ub9ac\ud2c0 \ub9ac\ucc98\ub4dc \uc81c\ub9ac \ub9ac \ub8e8\uc774\uc2a4 \uc9c4 \ube48\uc13c\ud2b8\uac00 \uc788\ub2e4. \ube4c \ud5e4\uc77c\ub9ac\uc758 Rock Around the Clock(1955)\ub294 \ucd5c\ucd08\ub85c \ube4c\ubcf4\ub4dc \uc0c1\uc704\uad8c\uc5d0 \uc704\uce58\ud55c \ub85c\ud070\ub864 \uc74c\uc545\uc774 \ub418\uc5c8\uc73c\uba70, \ub77c\ub514\uc624 \ucc28\ud2b8\uac00 \ub418\uc5c8\ub2e4. \uadf8\ub9ac\uace0 \uc774\ub294 \uc0c8\ub86d\uac8c \uc720\ud5a5\ud558\ub294 \ubb38\ud654\ub97c \ub9cc\ub4e4\uc5c8\ub2e4. \uace7 \ub85c\ud070\ub864\uc740 \uc9c0\ub09c 10\ub144\uac04 \ub300\uc911 \uc74c\uc545\uc744 \uc9c0\ubc30\ud574 \uc628 \uc5d0\ub514 \ud53c\uc154, \ud398\ub9ac \ucf54\ubaa8 \ud328\ud2f0 \ud398\uc774\uc9c0\uc640 \uac19\uc740 \ubbf8\uad6d \uc74c\ubc18 \ud310\ub9e4\uc758 \uc8fc\uc694 \uc694\uc18c\uac00 \ub418\uc5c8\ub2e4.", "sentence_answer": "\ud6c4\ub300\uc5d0 \ube4c \ud5e4\uc77c\ub9ac \uac00 \ub2e4\ub8ec Big Joe Turner\uc758 Shake, Rattle&Roll\uac00 \uc774\ubbf8 R&B \ube4c\ubcf4\ub4dc \ucc28\ud2b8\uc758 \ub9e8 \uc704\uc5d0 \uc788\uc5c8\ub2e4.\ucd08\uae30 \ub85c\ud070\ub864 \uc21c\uc704\uc5d0\ub294 \ucc99 \ubca0\ub9ac, \ubcf4 \ub514\ub4e4\ub9ac, \ud328\uce20 \ub3c4\ubbf8\ub178 \ub9ac\ud2c0 \ub9ac\ucc98\ub4dc \uc81c\ub9ac \ub9ac \ub8e8\uc774\uc2a4 \uc9c4 \ube48\uc13c\ud2b8\uac00 \uc788\ub2e4.", "paragraph_id": "6476543-0-1", "paragraph_question": "question: \ucd5c\ucd08\ub85c \ube4c\ubcf4\ub4dc \uc0c1\uc704\uad8c\uc5d0 \uc704\uce58\ud55c \ub85c\ud070\ub864 \uc74c\uc545\uc774 \ub41c Rock Around the Clock\uc740 \ub204\uad6c\uc758 \ub178\ub798\uc778\uac00?, context: \ucd5c\ucd08\uc758 \ub85c\ud070\ub864 \uc74c\ubc18\uc774 \ubb34\uc5c7\uc778\uac00?\uc5d0 \ub300\ud574\uc11c \ub9ce\uc740 \ub17c\uc7c1\uc774 \uc788\uc5c8\ub2e4. \ud55c \uc608\uce21\uc740 \uc7ad\ud0a4 \ube0c\ub79c\uc2a4\ud134\uacfc \uadf8\uc758 \ub378\ud0c0 \ucea3\uc758 Rocket 88 \uc774\ub2e4(\uc544\uc774\ud06c \ud130\ub108\uc640 \uadf8\uc758 \ubc34\ub4dc The king of Rythmn) \uc774\ub294 1951\ub144 \uba64\ud53c\uc2a4\uc5d0 \uc788\ub294 \uc120 \ub808\ucf54\ub4dc\uc5d0\uc11c \uc0d8 \ud544\ub9bd\uc2a4\uc5d0\uac8c \ub179\uc74c\ub418\uc5c8\ub2e4. \uc5d8\ube44\uc2a4 \ud504\ub808\uc2ac\ub9ac\uac00 Sun Record\uc5d0\uc11c \ub179\uc74c\ud55c \uccab\ubc88\uc9f8 \uc2f1\uae00 That's All Right(1954)\uac00 \uccab\ubc88\uc9f8 \ub85c\ud070\ub864\uc774\ub77c \uc0dd\uac01\ub418\uc5c8\ub2e4. \ud558\uc9c0\ub9cc \ub3d9\uc2dc\uc5d0 \ud6c4\ub300\uc5d0 \ube4c \ud5e4\uc77c\ub9ac\uac00 \ub2e4\ub8ec Big Joe Turner\uc758 Shake, Rattle&Roll\uac00 \uc774\ubbf8 R&B \ube4c\ubcf4\ub4dc \ucc28\ud2b8\uc758 \ub9e8 \uc704\uc5d0 \uc788\uc5c8\ub2e4.\ucd08\uae30 \ub85c\ud070\ub864 \uc21c\uc704\uc5d0\ub294 \ucc99 \ubca0\ub9ac, \ubcf4 \ub514\ub4e4\ub9ac, \ud328\uce20 \ub3c4\ubbf8\ub178 \ub9ac\ud2c0 \ub9ac\ucc98\ub4dc \uc81c\ub9ac \ub9ac \ub8e8\uc774\uc2a4 \uc9c4 \ube48\uc13c\ud2b8\uac00 \uc788\ub2e4. \ube4c \ud5e4\uc77c\ub9ac\uc758 Rock Around the Clock(1955)\ub294 \ucd5c\ucd08\ub85c \ube4c\ubcf4\ub4dc \uc0c1\uc704\uad8c\uc5d0 \uc704\uce58\ud55c \ub85c\ud070\ub864 \uc74c\uc545\uc774 \ub418\uc5c8\uc73c\uba70, \ub77c\ub514\uc624 \ucc28\ud2b8\uac00 \ub418\uc5c8\ub2e4. \uadf8\ub9ac\uace0 \uc774\ub294 \uc0c8\ub86d\uac8c \uc720\ud5a5\ud558\ub294 \ubb38\ud654\ub97c \ub9cc\ub4e4\uc5c8\ub2e4. \uace7 \ub85c\ud070\ub864\uc740 \uc9c0\ub09c 10\ub144\uac04 \ub300\uc911 \uc74c\uc545\uc744 \uc9c0\ubc30\ud574 \uc628 \uc5d0\ub514 \ud53c\uc154, \ud398\ub9ac \ucf54\ubaa8 \ud328\ud2f0 \ud398\uc774\uc9c0\uc640 \uac19\uc740 \ubbf8\uad6d \uc74c\ubc18 \ud310\ub9e4\uc758 \uc8fc\uc694 \uc694\uc18c\uac00 \ub418\uc5c8\ub2e4."} {"question": "2010\ub144 \uc870\uc0c1\uc5f4 \ubc15\uc0ac\uac00 \ucd9c\ud310\ud55c \ucc45\uc758 \uc774\ub984\uc740?", "paragraph": "\ud53c\uc5b4\uc120\uc5d0 \uad00\ud55c \uc120\uad6c\uc801\uc778 \uc5f0\uad6c\uac00\ub85c\ub294 \ubbf8\uad6d \ubcf4\uc2a4\ud1a4\ub300\ud559\uad50\uc758 \ub370\uc774\ub098 L. \ub85c\ubc84\ud2b8(Dana L. Robert)\uad50\uc218\uac00 \uc788\uc73c\uba70 \ud3c9\ud0dd\ub300\ud559\uad50 \uad50\uc218\uc778 \uae40\ubb38\uae30, \ub958\uc6d0\ub82c, \uc2e0\ud604\uc218, \uc548\uba85\uc900, \uc720\uc724\uc885, \uc774\uad11\ud76c, \uc870\uc0c1\uc5f4, \uc591\uc720\uc131, \ud55c\ub3d9\uad6c, \ub958\uc6d0\ub82c, \uae40\ud0dc\uc12d \ub4f1\uc774 \ud53c\uc5b4\uc120\uc5d0 \uad00\ud55c \uc5f0\uad6c\ub17c\ubb38\ub4e4\uc744 \ud65c\ubc1c\ud558\uac8c \ubc1c\ud45c\ud558\uace0 \uc788\ub2e4. 2009\ub144\uc5d0\ub294 \uace0\uad11\uad6d\ubc15\uc0ac\uac00 \"\ud53c\uc5b4\uc120\uc758 \uc120\uad50\uc804\ub7b5\uacfc \uc2e4\uc81c\uc5d0 \uad00\ud55c \uc5f0\uad6c\"\ub85c, 2011\ub144\uc5d0\ub294 \uc804\ub300\uacbd\ubc15\uc0ac\uac00 \"\uc544\ub354 \ud53c\uc5b4\uc120\uc758 \ubcc0\uc99d\uc801 \ubc29\ubc95\ub860\uc73c\ub85c\uc11c\uc758 \uc131\uc11c \ud574\uc11d \uc6d0\ub9ac\"\ub85c \uac01\uac01 \ud3c9\ud0dd\ub300\ud559\uad50 \ud53c\uc5b4\uc120\uc2e0\ud559\uc804\ubb38\ub300\ud559\uc6d0\uc5d0\uc11c \ubc15\uc0ac\ud559\uc704\ub97c \ubc1b\uc558\ub2e4. 2010\ub144\uc5d0\ub294 \uc870\uc0c1\uc5f4\ubc15\uc0ac\uac00 \u300c\ud53c\uc5b4\uc120\uae30\ub150\uc131\uacbd\ud559\uc6d0\u300d(\uac1c\uc815\ud310, 2011)\uc744 \ucd9c\ud310\ud558\uc600\ub2e4. \ud53c\uc5b4\uc120 \uad00\ub828 \uc5f0\uad6c \ubc1c\ud45c\ub294 \ub9e4\ub144 \ud53c\uc5b4\uc120\uae30\ub150\uc131\uacbd\uc5f0\uad6c\uc6d0\uc5d0\uc11c 2 \ucc28\ub840\uac00 \uc788\uc73c\uba70, \ud53c\uc5b4\uc120\uc2e0\ud559\ub17c\ub2e8\uc744 \ud1b5\ud574 \ucd9c\ud310\ub418\uace0 \uc788\ub2e4.", "answer": "\ud53c\uc5b4\uc120\uae30\ub150\uc131\uacbd\ud559\uc6d0", "sentence": "2010\ub144\uc5d0\ub294 \uc870\uc0c1\uc5f4\ubc15\uc0ac\uac00 \u300c \ud53c\uc5b4\uc120\uae30\ub150\uc131\uacbd\ud559\uc6d0 \u300d(\uac1c\uc815\ud310, 2011)\uc744 \ucd9c\ud310\ud558\uc600\ub2e4.", "paragraph_sentence": "\ud53c\uc5b4\uc120\uc5d0 \uad00\ud55c \uc120\uad6c\uc801\uc778 \uc5f0\uad6c\uac00\ub85c\ub294 \ubbf8\uad6d \ubcf4\uc2a4\ud1a4\ub300\ud559\uad50\uc758 \ub370\uc774\ub098 L. \ub85c\ubc84\ud2b8(Dana L. Robert)\uad50\uc218\uac00 \uc788\uc73c\uba70 \ud3c9\ud0dd\ub300\ud559\uad50 \uad50\uc218\uc778 \uae40\ubb38\uae30, \ub958\uc6d0\ub82c, \uc2e0\ud604\uc218, \uc548\uba85\uc900, \uc720\uc724\uc885, \uc774\uad11\ud76c, \uc870\uc0c1\uc5f4, \uc591\uc720\uc131, \ud55c\ub3d9\uad6c, \ub958\uc6d0\ub82c, \uae40\ud0dc\uc12d \ub4f1\uc774 \ud53c\uc5b4\uc120\uc5d0 \uad00\ud55c \uc5f0\uad6c\ub17c\ubb38\ub4e4\uc744 \ud65c\ubc1c\ud558\uac8c \ubc1c\ud45c\ud558\uace0 \uc788\ub2e4. 2009\ub144\uc5d0\ub294 \uace0\uad11\uad6d\ubc15\uc0ac\uac00 \"\ud53c\uc5b4\uc120\uc758 \uc120\uad50\uc804\ub7b5\uacfc \uc2e4\uc81c\uc5d0 \uad00\ud55c \uc5f0\uad6c\"\ub85c, 2011\ub144\uc5d0\ub294 \uc804\ub300\uacbd\ubc15\uc0ac\uac00 \"\uc544\ub354 \ud53c\uc5b4\uc120\uc758 \ubcc0\uc99d\uc801 \ubc29\ubc95\ub860\uc73c\ub85c\uc11c\uc758 \uc131\uc11c \ud574\uc11d \uc6d0\ub9ac\"\ub85c \uac01\uac01 \ud3c9\ud0dd\ub300\ud559\uad50 \ud53c\uc5b4\uc120\uc2e0\ud559\uc804\ubb38\ub300\ud559\uc6d0\uc5d0\uc11c \ubc15\uc0ac\ud559\uc704\ub97c \ubc1b\uc558\ub2e4. 2010\ub144\uc5d0\ub294 \uc870\uc0c1\uc5f4\ubc15\uc0ac\uac00 \u300c \ud53c\uc5b4\uc120\uae30\ub150\uc131\uacbd\ud559\uc6d0 \u300d(\uac1c\uc815\ud310, 2011)\uc744 \ucd9c\ud310\ud558\uc600\ub2e4. \ud53c\uc5b4\uc120 \uad00\ub828 \uc5f0\uad6c \ubc1c\ud45c\ub294 \ub9e4\ub144 \ud53c\uc5b4\uc120\uae30\ub150\uc131\uacbd\uc5f0\uad6c\uc6d0\uc5d0\uc11c 2 \ucc28\ub840\uac00 \uc788\uc73c\uba70, \ud53c\uc5b4\uc120\uc2e0\ud559\ub17c\ub2e8\uc744 \ud1b5\ud574 \ucd9c\ud310\ub418\uace0 \uc788\ub2e4.", "paragraph_answer": "\ud53c\uc5b4\uc120\uc5d0 \uad00\ud55c \uc120\uad6c\uc801\uc778 \uc5f0\uad6c\uac00\ub85c\ub294 \ubbf8\uad6d \ubcf4\uc2a4\ud1a4\ub300\ud559\uad50\uc758 \ub370\uc774\ub098 L. \ub85c\ubc84\ud2b8(Dana L. Robert)\uad50\uc218\uac00 \uc788\uc73c\uba70 \ud3c9\ud0dd\ub300\ud559\uad50 \uad50\uc218\uc778 \uae40\ubb38\uae30, \ub958\uc6d0\ub82c, \uc2e0\ud604\uc218, \uc548\uba85\uc900, \uc720\uc724\uc885, \uc774\uad11\ud76c, \uc870\uc0c1\uc5f4, \uc591\uc720\uc131, \ud55c\ub3d9\uad6c, \ub958\uc6d0\ub82c, \uae40\ud0dc\uc12d \ub4f1\uc774 \ud53c\uc5b4\uc120\uc5d0 \uad00\ud55c \uc5f0\uad6c\ub17c\ubb38\ub4e4\uc744 \ud65c\ubc1c\ud558\uac8c \ubc1c\ud45c\ud558\uace0 \uc788\ub2e4. 2009\ub144\uc5d0\ub294 \uace0\uad11\uad6d\ubc15\uc0ac\uac00 \"\ud53c\uc5b4\uc120\uc758 \uc120\uad50\uc804\ub7b5\uacfc \uc2e4\uc81c\uc5d0 \uad00\ud55c \uc5f0\uad6c\"\ub85c, 2011\ub144\uc5d0\ub294 \uc804\ub300\uacbd\ubc15\uc0ac\uac00 \"\uc544\ub354 \ud53c\uc5b4\uc120\uc758 \ubcc0\uc99d\uc801 \ubc29\ubc95\ub860\uc73c\ub85c\uc11c\uc758 \uc131\uc11c \ud574\uc11d \uc6d0\ub9ac\"\ub85c \uac01\uac01 \ud3c9\ud0dd\ub300\ud559\uad50 \ud53c\uc5b4\uc120\uc2e0\ud559\uc804\ubb38\ub300\ud559\uc6d0\uc5d0\uc11c \ubc15\uc0ac\ud559\uc704\ub97c \ubc1b\uc558\ub2e4. 2010\ub144\uc5d0\ub294 \uc870\uc0c1\uc5f4\ubc15\uc0ac\uac00 \u300c \ud53c\uc5b4\uc120\uae30\ub150\uc131\uacbd\ud559\uc6d0 \u300d(\uac1c\uc815\ud310, 2011)\uc744 \ucd9c\ud310\ud558\uc600\ub2e4. \ud53c\uc5b4\uc120 \uad00\ub828 \uc5f0\uad6c \ubc1c\ud45c\ub294 \ub9e4\ub144 \ud53c\uc5b4\uc120\uae30\ub150\uc131\uacbd\uc5f0\uad6c\uc6d0\uc5d0\uc11c 2 \ucc28\ub840\uac00 \uc788\uc73c\uba70, \ud53c\uc5b4\uc120\uc2e0\ud559\ub17c\ub2e8\uc744 \ud1b5\ud574 \ucd9c\ud310\ub418\uace0 \uc788\ub2e4.", "sentence_answer": "2010\ub144\uc5d0\ub294 \uc870\uc0c1\uc5f4\ubc15\uc0ac\uac00 \u300c \ud53c\uc5b4\uc120\uae30\ub150\uc131\uacbd\ud559\uc6d0 \u300d(\uac1c\uc815\ud310, 2011)\uc744 \ucd9c\ud310\ud558\uc600\ub2e4.", "paragraph_id": "6533524-1-2", "paragraph_question": "question: 2010\ub144 \uc870\uc0c1\uc5f4 \ubc15\uc0ac\uac00 \ucd9c\ud310\ud55c \ucc45\uc758 \uc774\ub984\uc740?, context: \ud53c\uc5b4\uc120\uc5d0 \uad00\ud55c \uc120\uad6c\uc801\uc778 \uc5f0\uad6c\uac00\ub85c\ub294 \ubbf8\uad6d \ubcf4\uc2a4\ud1a4\ub300\ud559\uad50\uc758 \ub370\uc774\ub098 L. \ub85c\ubc84\ud2b8(Dana L. Robert)\uad50\uc218\uac00 \uc788\uc73c\uba70 \ud3c9\ud0dd\ub300\ud559\uad50 \uad50\uc218\uc778 \uae40\ubb38\uae30, \ub958\uc6d0\ub82c, \uc2e0\ud604\uc218, \uc548\uba85\uc900, \uc720\uc724\uc885, \uc774\uad11\ud76c, \uc870\uc0c1\uc5f4, \uc591\uc720\uc131, \ud55c\ub3d9\uad6c, \ub958\uc6d0\ub82c, \uae40\ud0dc\uc12d \ub4f1\uc774 \ud53c\uc5b4\uc120\uc5d0 \uad00\ud55c \uc5f0\uad6c\ub17c\ubb38\ub4e4\uc744 \ud65c\ubc1c\ud558\uac8c \ubc1c\ud45c\ud558\uace0 \uc788\ub2e4. 2009\ub144\uc5d0\ub294 \uace0\uad11\uad6d\ubc15\uc0ac\uac00 \"\ud53c\uc5b4\uc120\uc758 \uc120\uad50\uc804\ub7b5\uacfc \uc2e4\uc81c\uc5d0 \uad00\ud55c \uc5f0\uad6c\"\ub85c, 2011\ub144\uc5d0\ub294 \uc804\ub300\uacbd\ubc15\uc0ac\uac00 \"\uc544\ub354 \ud53c\uc5b4\uc120\uc758 \ubcc0\uc99d\uc801 \ubc29\ubc95\ub860\uc73c\ub85c\uc11c\uc758 \uc131\uc11c \ud574\uc11d \uc6d0\ub9ac\"\ub85c \uac01\uac01 \ud3c9\ud0dd\ub300\ud559\uad50 \ud53c\uc5b4\uc120\uc2e0\ud559\uc804\ubb38\ub300\ud559\uc6d0\uc5d0\uc11c \ubc15\uc0ac\ud559\uc704\ub97c \ubc1b\uc558\ub2e4. 2010\ub144\uc5d0\ub294 \uc870\uc0c1\uc5f4\ubc15\uc0ac\uac00 \u300c\ud53c\uc5b4\uc120\uae30\ub150\uc131\uacbd\ud559\uc6d0\u300d(\uac1c\uc815\ud310, 2011)\uc744 \ucd9c\ud310\ud558\uc600\ub2e4. \ud53c\uc5b4\uc120 \uad00\ub828 \uc5f0\uad6c \ubc1c\ud45c\ub294 \ub9e4\ub144 \ud53c\uc5b4\uc120\uae30\ub150\uc131\uacbd\uc5f0\uad6c\uc6d0\uc5d0\uc11c 2 \ucc28\ub840\uac00 \uc788\uc73c\uba70, \ud53c\uc5b4\uc120\uc2e0\ud559\ub17c\ub2e8\uc744 \ud1b5\ud574 \ucd9c\ud310\ub418\uace0 \uc788\ub2e4."} {"question": "2014\ub144 5\uc138 \ubbf8\ub9cc \uc218\ub2e8 \uc544\ub3d9\uc758 \ud5c8\uc57d \uc218\uc900\uc740?", "paragraph": "\uc218\ub2e8\uc758 \uae30\uc544\ubb38\uc81c\ub294 \uc9c0\uc18d\ub418\ub294 \uc0ac\ud68c\ubb38\uc81c \uc911\uc5d0 \ud558\ub098\uc774\ub2e4. \ub9e4\ub144 \uc544\uc77c\ub79c\ub4dc\uc758 NGO\uc778 \ucee8\uc120\uc6d4\ub4dc\uc640\uc774\ub4dc(Concern Worldwide), \ub3c5\uc77c\uc758 NGO\uc778 \uc138\uacc4\uae30\uc544\uc6d0\uc870(Welthungerhilfe) \uadf8\ub9ac\uace0 \ubbf8\uad6d\uc758 \uc5f0\uad6c\uae30\uad00\uc778 \uad6d\uc81c\uc2dd\ub7c9\uc815\ucc45\uc5f0\uad6c\uc18c(IFPRI)\uac00 \ud611\ub825\ud558\uc5ec \ubc1c\ud45c\ud558\ub294 \uc138\uacc4\uae30\uc544\uc9c0\uc218(GHI)\uc5d0 \ub530\ub974\uba74 2016\ub144 \uc218\ub2e8\uc740 \ubd88\uc548\ud55c \uc815\uce58\uc801 \uc0c1\ud669\ub54c\ubb38\uc5d0 \uc790\ub8cc\ub97c \uc0b0\ucd9c\ud560 \uc218 \uc5c6\uc9c0\ub9cc \ub208\uc5ec\uaca8\ubcf4\uc544\uc57c\ud560 \ub098\ub77c\ub4e4 \uc911\uc5d0 \ud558\ub098\uc774\ub2e4. 2014\ub144 \uc870\uc0ac \uc790\ub8cc\ub97c \ubcf4\uba74 \uc218\ub2e8\uc758 \uc544\ub3d9\uc601\uc591\ubd80\uc871\uc740 \uc6b0\ub824\ud560\ub9cc\ud55c \uc218\uc900\uc774\ub2e4. 5\uc138 \ubbf8\ub9cc \uc544\ub3d9\uc758 \ubc1c\uc721 \ubd80\uc9c4\uacfc \ud5c8\uc57d \uc218\uc900\uc740 \uac01\uac01 38.2\ud37c\uc13c\ud2b8\uc640 16.3\ud37c\uc13c\ud2b8\uc774\ub2e4. \uc218\ub2e8\uc758 \uae30\uc544\uc640 \uc601\uc591\ubd80\uc871\ubb38\uc81c\ub294 \ube48\uace4\uc774 \ub9cc\uc5f0\ud55c \uac83, 2015-2016\ub144 \uc5d8\ub2c8\ub1e8\ub85c \uc778\ud55c \uac00\ubb44\uc744 \ube44\ub86f\ud55c \ub18d\uc5c5\uacfc \uc0dd\ud0dc \ud658\uacbd \ud30c\uad34, \ud3ed\ub825\uc801 \ucda9\ub3cc\uacfc \uc815\uce58\ubd88\uc548\uacfc \uad00\ub828\uc774 \uc788\ub2e4. \uc218\ub2e8 \uad6d\ub0b4 \uac08\ub4f1\uc73c\ub85c \ubc1c\uc0dd\ud55c \ub0a8\uc218\ub2e8 \ub09c\ubbfc \uc720\uc785\uacfc \ub300\uaddc\ubaa8 \uad6d\ub0b4 \uc774\uc7ac\ubbfc\uc73c\ub85c \uae30\uc544\uc640 \uc601\uc591\ubd80\uc871\uc740 \uc545\ud654\ub418\uc5c8\ub2e4. \uc2dd\ub7c9 \ubd88\uc548 \ud53c\ud574\uac00 \uac00\uc7a5 \uadf9\uc2ec\ud55c \uc9c0\uc5ed\uc740 \ubd84\uc7c1\uc758 \uc601\ud5a5\uc744 \ubc1b\uc740 \ubd88\ub8e8\ub098\uc77c, \ub2e4\ub974\ud478\ub974, \uc6e8\uc2a4\ud2b8\ucf54\ub974\ub3c4\ud310, \uc0ac\uc6b0\uc2a4\ucf54\ub974\ub3c4\ud310 \uc8fc\uc774\ub2e4.", "answer": "16.3\ud37c\uc13c\ud2b8", "sentence": "5\uc138 \ubbf8\ub9cc \uc544\ub3d9\uc758 \ubc1c\uc721 \ubd80\uc9c4\uacfc \ud5c8\uc57d \uc218\uc900\uc740 \uac01\uac01 38.2\ud37c\uc13c\ud2b8\uc640 16.3\ud37c\uc13c\ud2b8 \uc774\ub2e4.", "paragraph_sentence": "\uc218\ub2e8\uc758 \uae30\uc544\ubb38\uc81c\ub294 \uc9c0\uc18d\ub418\ub294 \uc0ac\ud68c\ubb38\uc81c \uc911\uc5d0 \ud558\ub098\uc774\ub2e4. \ub9e4\ub144 \uc544\uc77c\ub79c\ub4dc\uc758 NGO\uc778 \ucee8\uc120\uc6d4\ub4dc\uc640\uc774\ub4dc(Concern Worldwide), \ub3c5\uc77c\uc758 NGO\uc778 \uc138\uacc4\uae30\uc544\uc6d0\uc870(Welthungerhilfe) \uadf8\ub9ac\uace0 \ubbf8\uad6d\uc758 \uc5f0\uad6c\uae30\uad00\uc778 \uad6d\uc81c\uc2dd\ub7c9\uc815\ucc45\uc5f0\uad6c\uc18c(IFPRI)\uac00 \ud611\ub825\ud558\uc5ec \ubc1c\ud45c\ud558\ub294 \uc138\uacc4\uae30\uc544\uc9c0\uc218(GHI)\uc5d0 \ub530\ub974\uba74 2016\ub144 \uc218\ub2e8\uc740 \ubd88\uc548\ud55c \uc815\uce58\uc801 \uc0c1\ud669\ub54c\ubb38\uc5d0 \uc790\ub8cc\ub97c \uc0b0\ucd9c\ud560 \uc218 \uc5c6\uc9c0\ub9cc \ub208\uc5ec\uaca8\ubcf4\uc544\uc57c\ud560 \ub098\ub77c\ub4e4 \uc911\uc5d0 \ud558\ub098\uc774\ub2e4. 2014\ub144 \uc870\uc0ac \uc790\ub8cc\ub97c \ubcf4\uba74 \uc218\ub2e8\uc758 \uc544\ub3d9\uc601\uc591\ubd80\uc871\uc740 \uc6b0\ub824\ud560\ub9cc\ud55c \uc218\uc900\uc774\ub2e4. 5\uc138 \ubbf8\ub9cc \uc544\ub3d9\uc758 \ubc1c\uc721 \ubd80\uc9c4\uacfc \ud5c8\uc57d \uc218\uc900\uc740 \uac01\uac01 38.2\ud37c\uc13c\ud2b8\uc640 16.3\ud37c\uc13c\ud2b8 \uc774\ub2e4. \uc218\ub2e8\uc758 \uae30\uc544\uc640 \uc601\uc591\ubd80\uc871\ubb38\uc81c\ub294 \ube48\uace4\uc774 \ub9cc\uc5f0\ud55c \uac83, 2015-2016\ub144 \uc5d8\ub2c8\ub1e8\ub85c \uc778\ud55c \uac00\ubb44\uc744 \ube44\ub86f\ud55c \ub18d\uc5c5\uacfc \uc0dd\ud0dc \ud658\uacbd \ud30c\uad34, \ud3ed\ub825\uc801 \ucda9\ub3cc\uacfc \uc815\uce58\ubd88\uc548\uacfc \uad00\ub828\uc774 \uc788\ub2e4. \uc218\ub2e8 \uad6d\ub0b4 \uac08\ub4f1\uc73c\ub85c \ubc1c\uc0dd\ud55c \ub0a8\uc218\ub2e8 \ub09c\ubbfc \uc720\uc785\uacfc \ub300\uaddc\ubaa8 \uad6d\ub0b4 \uc774\uc7ac\ubbfc\uc73c\ub85c \uae30\uc544\uc640 \uc601\uc591\ubd80\uc871\uc740 \uc545\ud654\ub418\uc5c8\ub2e4. \uc2dd\ub7c9 \ubd88\uc548 \ud53c\ud574\uac00 \uac00\uc7a5 \uadf9\uc2ec\ud55c \uc9c0\uc5ed\uc740 \ubd84\uc7c1\uc758 \uc601\ud5a5\uc744 \ubc1b\uc740 \ubd88\ub8e8\ub098\uc77c, \ub2e4\ub974\ud478\ub974, \uc6e8\uc2a4\ud2b8\ucf54\ub974\ub3c4\ud310, \uc0ac\uc6b0\uc2a4\ucf54\ub974\ub3c4\ud310 \uc8fc\uc774\ub2e4.", "paragraph_answer": "\uc218\ub2e8\uc758 \uae30\uc544\ubb38\uc81c\ub294 \uc9c0\uc18d\ub418\ub294 \uc0ac\ud68c\ubb38\uc81c \uc911\uc5d0 \ud558\ub098\uc774\ub2e4. \ub9e4\ub144 \uc544\uc77c\ub79c\ub4dc\uc758 NGO\uc778 \ucee8\uc120\uc6d4\ub4dc\uc640\uc774\ub4dc(Concern Worldwide), \ub3c5\uc77c\uc758 NGO\uc778 \uc138\uacc4\uae30\uc544\uc6d0\uc870(Welthungerhilfe) \uadf8\ub9ac\uace0 \ubbf8\uad6d\uc758 \uc5f0\uad6c\uae30\uad00\uc778 \uad6d\uc81c\uc2dd\ub7c9\uc815\ucc45\uc5f0\uad6c\uc18c(IFPRI)\uac00 \ud611\ub825\ud558\uc5ec \ubc1c\ud45c\ud558\ub294 \uc138\uacc4\uae30\uc544\uc9c0\uc218(GHI)\uc5d0 \ub530\ub974\uba74 2016\ub144 \uc218\ub2e8\uc740 \ubd88\uc548\ud55c \uc815\uce58\uc801 \uc0c1\ud669\ub54c\ubb38\uc5d0 \uc790\ub8cc\ub97c \uc0b0\ucd9c\ud560 \uc218 \uc5c6\uc9c0\ub9cc \ub208\uc5ec\uaca8\ubcf4\uc544\uc57c\ud560 \ub098\ub77c\ub4e4 \uc911\uc5d0 \ud558\ub098\uc774\ub2e4. 2014\ub144 \uc870\uc0ac \uc790\ub8cc\ub97c \ubcf4\uba74 \uc218\ub2e8\uc758 \uc544\ub3d9\uc601\uc591\ubd80\uc871\uc740 \uc6b0\ub824\ud560\ub9cc\ud55c \uc218\uc900\uc774\ub2e4. 5\uc138 \ubbf8\ub9cc \uc544\ub3d9\uc758 \ubc1c\uc721 \ubd80\uc9c4\uacfc \ud5c8\uc57d \uc218\uc900\uc740 \uac01\uac01 38.2\ud37c\uc13c\ud2b8\uc640 16.3\ud37c\uc13c\ud2b8 \uc774\ub2e4. \uc218\ub2e8\uc758 \uae30\uc544\uc640 \uc601\uc591\ubd80\uc871\ubb38\uc81c\ub294 \ube48\uace4\uc774 \ub9cc\uc5f0\ud55c \uac83, 2015-2016\ub144 \uc5d8\ub2c8\ub1e8\ub85c \uc778\ud55c \uac00\ubb44\uc744 \ube44\ub86f\ud55c \ub18d\uc5c5\uacfc \uc0dd\ud0dc \ud658\uacbd \ud30c\uad34, \ud3ed\ub825\uc801 \ucda9\ub3cc\uacfc \uc815\uce58\ubd88\uc548\uacfc \uad00\ub828\uc774 \uc788\ub2e4. \uc218\ub2e8 \uad6d\ub0b4 \uac08\ub4f1\uc73c\ub85c \ubc1c\uc0dd\ud55c \ub0a8\uc218\ub2e8 \ub09c\ubbfc \uc720\uc785\uacfc \ub300\uaddc\ubaa8 \uad6d\ub0b4 \uc774\uc7ac\ubbfc\uc73c\ub85c \uae30\uc544\uc640 \uc601\uc591\ubd80\uc871\uc740 \uc545\ud654\ub418\uc5c8\ub2e4. \uc2dd\ub7c9 \ubd88\uc548 \ud53c\ud574\uac00 \uac00\uc7a5 \uadf9\uc2ec\ud55c \uc9c0\uc5ed\uc740 \ubd84\uc7c1\uc758 \uc601\ud5a5\uc744 \ubc1b\uc740 \ubd88\ub8e8\ub098\uc77c, \ub2e4\ub974\ud478\ub974, \uc6e8\uc2a4\ud2b8\ucf54\ub974\ub3c4\ud310, \uc0ac\uc6b0\uc2a4\ucf54\ub974\ub3c4\ud310 \uc8fc\uc774\ub2e4.", "sentence_answer": "5\uc138 \ubbf8\ub9cc \uc544\ub3d9\uc758 \ubc1c\uc721 \ubd80\uc9c4\uacfc \ud5c8\uc57d \uc218\uc900\uc740 \uac01\uac01 38.2\ud37c\uc13c\ud2b8\uc640 16.3\ud37c\uc13c\ud2b8 \uc774\ub2e4.", "paragraph_id": "6532488-2-1", "paragraph_question": "question: 2014\ub144 5\uc138 \ubbf8\ub9cc \uc218\ub2e8 \uc544\ub3d9\uc758 \ud5c8\uc57d \uc218\uc900\uc740?, context: \uc218\ub2e8\uc758 \uae30\uc544\ubb38\uc81c\ub294 \uc9c0\uc18d\ub418\ub294 \uc0ac\ud68c\ubb38\uc81c \uc911\uc5d0 \ud558\ub098\uc774\ub2e4. \ub9e4\ub144 \uc544\uc77c\ub79c\ub4dc\uc758 NGO\uc778 \ucee8\uc120\uc6d4\ub4dc\uc640\uc774\ub4dc(Concern Worldwide), \ub3c5\uc77c\uc758 NGO\uc778 \uc138\uacc4\uae30\uc544\uc6d0\uc870(Welthungerhilfe) \uadf8\ub9ac\uace0 \ubbf8\uad6d\uc758 \uc5f0\uad6c\uae30\uad00\uc778 \uad6d\uc81c\uc2dd\ub7c9\uc815\ucc45\uc5f0\uad6c\uc18c(IFPRI)\uac00 \ud611\ub825\ud558\uc5ec \ubc1c\ud45c\ud558\ub294 \uc138\uacc4\uae30\uc544\uc9c0\uc218(GHI)\uc5d0 \ub530\ub974\uba74 2016\ub144 \uc218\ub2e8\uc740 \ubd88\uc548\ud55c \uc815\uce58\uc801 \uc0c1\ud669\ub54c\ubb38\uc5d0 \uc790\ub8cc\ub97c \uc0b0\ucd9c\ud560 \uc218 \uc5c6\uc9c0\ub9cc \ub208\uc5ec\uaca8\ubcf4\uc544\uc57c\ud560 \ub098\ub77c\ub4e4 \uc911\uc5d0 \ud558\ub098\uc774\ub2e4. 2014\ub144 \uc870\uc0ac \uc790\ub8cc\ub97c \ubcf4\uba74 \uc218\ub2e8\uc758 \uc544\ub3d9\uc601\uc591\ubd80\uc871\uc740 \uc6b0\ub824\ud560\ub9cc\ud55c \uc218\uc900\uc774\ub2e4. 5\uc138 \ubbf8\ub9cc \uc544\ub3d9\uc758 \ubc1c\uc721 \ubd80\uc9c4\uacfc \ud5c8\uc57d \uc218\uc900\uc740 \uac01\uac01 38.2\ud37c\uc13c\ud2b8\uc640 16.3\ud37c\uc13c\ud2b8\uc774\ub2e4. \uc218\ub2e8\uc758 \uae30\uc544\uc640 \uc601\uc591\ubd80\uc871\ubb38\uc81c\ub294 \ube48\uace4\uc774 \ub9cc\uc5f0\ud55c \uac83, 2015-2016\ub144 \uc5d8\ub2c8\ub1e8\ub85c \uc778\ud55c \uac00\ubb44\uc744 \ube44\ub86f\ud55c \ub18d\uc5c5\uacfc \uc0dd\ud0dc \ud658\uacbd \ud30c\uad34, \ud3ed\ub825\uc801 \ucda9\ub3cc\uacfc \uc815\uce58\ubd88\uc548\uacfc \uad00\ub828\uc774 \uc788\ub2e4. \uc218\ub2e8 \uad6d\ub0b4 \uac08\ub4f1\uc73c\ub85c \ubc1c\uc0dd\ud55c \ub0a8\uc218\ub2e8 \ub09c\ubbfc \uc720\uc785\uacfc \ub300\uaddc\ubaa8 \uad6d\ub0b4 \uc774\uc7ac\ubbfc\uc73c\ub85c \uae30\uc544\uc640 \uc601\uc591\ubd80\uc871\uc740 \uc545\ud654\ub418\uc5c8\ub2e4. \uc2dd\ub7c9 \ubd88\uc548 \ud53c\ud574\uac00 \uac00\uc7a5 \uadf9\uc2ec\ud55c \uc9c0\uc5ed\uc740 \ubd84\uc7c1\uc758 \uc601\ud5a5\uc744 \ubc1b\uc740 \ubd88\ub8e8\ub098\uc77c, \ub2e4\ub974\ud478\ub974, \uc6e8\uc2a4\ud2b8\ucf54\ub974\ub3c4\ud310, \uc0ac\uc6b0\uc2a4\ucf54\ub974\ub3c4\ud310 \uc8fc\uc774\ub2e4."} {"question": "\uc62c\ud574 \uc138\uc0c1\uc744 \ub5a0\ub0ac\uc9c0\ub9cc \uc78a\ud600\uc9c0\uc9c0 \uc54a\uc744 \uc800\uba85\uc778\uc0ac \uc911 \ud558\ub098\ub85c \uae40\ub300\uc911\uc744 \uc120\uc815\ud55c \uc2e0\ubb38\uc0ac\ub294?", "paragraph": "2009\ub144 12\uc6d4 19\uc77c, \ub274\uc2a4\uc704\ud06c \uc778\ud130\ub137\ud310\uc774 '\uc62c\ud574 \uc138\uc0c1\uc744 \ub5a0\ub0ac\uc9c0\ub9cc \uc78a\ud600\uc9c0\uc9c0 \uc54a\uc744 \uc800\uba85\uc778\uc0ac'\ub85c \uae40\ub300\uc911 \uc804 \ub300\ud1b5\ub839\uc744 \ud3ec\ud568\ud574 \uc804 \uc138\uacc4 36\uba85\uc758 \uba85\uc0ac\ub97c \uc120\uc815\ud574 \uc18c\uac1c\ud588\ub2e4. \ub274\uc2a4\uc704\ud06c\ub294 \"\uc8fd\uc74c\uc744 \ub450\ub824\uc6cc\ud558\ub294 \uc790\ub4e4\uc740 \uc0b6\uc744 \uc2dc\uc791\ud560 \uc218 \uc5c6\ub2e4\"\ub294 \ubbf8\uad6d\uc758 \uc131\uc9c1\uc790 \ud5e8\ub9ac \ubc18 \ub2e4\uc774\ud06c\uc758 \ub9d0\uc744 \uc778\uc6a9\ud558\uba74\uc11c \uae40\ub300\uc911 \uc804 \ub300\ud1b5\ub839\uc744 \ube44\ub86f\ud574 \uc815.\uc7ac\uacc4, \uc5f0\uc608\uacc4, \ud559\uacc4\uc5d0\uc11c \ud0c1\uc6d4\ud55c \uc5c5\uc801\uc744 \ub0a8\uae34 \uc800\uba85\uc778\uc0ac 36\uc778\uc740 \uc62c\ud574 \uc138\uc0c1\uc744 \ub5a0\ub0ac\uc9c0\ub9cc \uc0ac\ub78c\ub4e4\uc758 \ub1cc\ub9ac \uc18d\uc5d0 \uc601\uc6d0\ud788 \uae30\uc5b5\ub420 \uac83\uc774\ub77c\uace0 \ud3c9\uac00\ud588\ub2e4. \ud2b9\ud788 \ub274\uc2a4\uc704\ud06c\ub294 \"\uae40 \uc804 \ub300\ud1b5\ub839\uc740 1997\ub144 \ud55c\uad6d\uc758 \ub300\ud1b5\ub839\uc5d0 \ub2f9\uc120\ub420 \ub54c\uae4c\uc9c0 \uad70\ubd80 \ub3c5\uc7ac\uc5d0 \ud56d\uac70\ud558\uace0 \ub9dd\uba85\uacfc \ud22c\uc625, \ub0a9\uce58, \uc554\uc0b4 \uc704\ud611 \ub4f1 \uace0\ub09c\uc758 \uc138\uc6d4\uc744 \uc774\uaca8\ub0b8 \uc57c\uad8c \uc9c0\ub3c4\uc790\"\uc600\ub2e4\uace0 \uc18c\uac1c\ud588\ub2e4. \ub610 \"\uae40 \uc804 \ub300\ud1b5\ub839\uc740 \uc7ac\uc784\uc2dc \ub0a8\ubd81 \ud654\ud574\ub97c \uc99d\uc9c4\uc2dc\ud0a8 \uacf5\ub85c\ub85c \ub178\ubca8 \ud3c9\ud654\uc0c1\uc744 \ubc1b\uc558\ub2e4\"\ub77c\uace0 \ub367\ubd99\uc600\ub2e4.", "answer": "\ub274\uc2a4\uc704\ud06c", "sentence": "2009\ub144 12\uc6d4 19\uc77c, \ub274\uc2a4\uc704\ud06c \uc778\ud130\ub137\ud310\uc774 '\uc62c\ud574 \uc138\uc0c1\uc744 \ub5a0\ub0ac\uc9c0\ub9cc \uc78a\ud600\uc9c0\uc9c0 \uc54a\uc744 \uc800\uba85\uc778\uc0ac'\ub85c \uae40\ub300\uc911 \uc804 \ub300\ud1b5\ub839\uc744 \ud3ec\ud568\ud574 \uc804 \uc138\uacc4 36\uba85\uc758 \uba85\uc0ac\ub97c \uc120\uc815\ud574 \uc18c\uac1c\ud588\ub2e4.", "paragraph_sentence": " 2009\ub144 12\uc6d4 19\uc77c, \ub274\uc2a4\uc704\ud06c \uc778\ud130\ub137\ud310\uc774 '\uc62c\ud574 \uc138\uc0c1\uc744 \ub5a0\ub0ac\uc9c0\ub9cc \uc78a\ud600\uc9c0\uc9c0 \uc54a\uc744 \uc800\uba85\uc778\uc0ac'\ub85c \uae40\ub300\uc911 \uc804 \ub300\ud1b5\ub839\uc744 \ud3ec\ud568\ud574 \uc804 \uc138\uacc4 36\uba85\uc758 \uba85\uc0ac\ub97c \uc120\uc815\ud574 \uc18c\uac1c\ud588\ub2e4. \ub274\uc2a4\uc704\ud06c\ub294 \"\uc8fd\uc74c\uc744 \ub450\ub824\uc6cc\ud558\ub294 \uc790\ub4e4\uc740 \uc0b6\uc744 \uc2dc\uc791\ud560 \uc218 \uc5c6\ub2e4\"\ub294 \ubbf8\uad6d\uc758 \uc131\uc9c1\uc790 \ud5e8\ub9ac \ubc18 \ub2e4\uc774\ud06c\uc758 \ub9d0\uc744 \uc778\uc6a9\ud558\uba74\uc11c \uae40\ub300\uc911 \uc804 \ub300\ud1b5\ub839\uc744 \ube44\ub86f\ud574 \uc815.\uc7ac\uacc4, \uc5f0\uc608\uacc4, \ud559\uacc4\uc5d0\uc11c \ud0c1\uc6d4\ud55c \uc5c5\uc801\uc744 \ub0a8\uae34 \uc800\uba85\uc778\uc0ac 36\uc778\uc740 \uc62c\ud574 \uc138\uc0c1\uc744 \ub5a0\ub0ac\uc9c0\ub9cc \uc0ac\ub78c\ub4e4\uc758 \ub1cc\ub9ac \uc18d\uc5d0 \uc601\uc6d0\ud788 \uae30\uc5b5\ub420 \uac83\uc774\ub77c\uace0 \ud3c9\uac00\ud588\ub2e4. \ud2b9\ud788 \ub274\uc2a4\uc704\ud06c\ub294 \"\uae40 \uc804 \ub300\ud1b5\ub839\uc740 1997\ub144 \ud55c\uad6d\uc758 \ub300\ud1b5\ub839\uc5d0 \ub2f9\uc120\ub420 \ub54c\uae4c\uc9c0 \uad70\ubd80 \ub3c5\uc7ac\uc5d0 \ud56d\uac70\ud558\uace0 \ub9dd\uba85\uacfc \ud22c\uc625, \ub0a9\uce58, \uc554\uc0b4 \uc704\ud611 \ub4f1 \uace0\ub09c\uc758 \uc138\uc6d4\uc744 \uc774\uaca8\ub0b8 \uc57c\uad8c \uc9c0\ub3c4\uc790\"\uc600\ub2e4\uace0 \uc18c\uac1c\ud588\ub2e4. \ub610 \"\uae40 \uc804 \ub300\ud1b5\ub839\uc740 \uc7ac\uc784\uc2dc \ub0a8\ubd81 \ud654\ud574\ub97c \uc99d\uc9c4\uc2dc\ud0a8 \uacf5\ub85c\ub85c \ub178\ubca8 \ud3c9\ud654\uc0c1\uc744 \ubc1b\uc558\ub2e4\"\ub77c\uace0 \ub367\ubd99\uc600\ub2e4.", "paragraph_answer": "2009\ub144 12\uc6d4 19\uc77c, \ub274\uc2a4\uc704\ud06c \uc778\ud130\ub137\ud310\uc774 '\uc62c\ud574 \uc138\uc0c1\uc744 \ub5a0\ub0ac\uc9c0\ub9cc \uc78a\ud600\uc9c0\uc9c0 \uc54a\uc744 \uc800\uba85\uc778\uc0ac'\ub85c \uae40\ub300\uc911 \uc804 \ub300\ud1b5\ub839\uc744 \ud3ec\ud568\ud574 \uc804 \uc138\uacc4 36\uba85\uc758 \uba85\uc0ac\ub97c \uc120\uc815\ud574 \uc18c\uac1c\ud588\ub2e4. \ub274\uc2a4\uc704\ud06c\ub294 \"\uc8fd\uc74c\uc744 \ub450\ub824\uc6cc\ud558\ub294 \uc790\ub4e4\uc740 \uc0b6\uc744 \uc2dc\uc791\ud560 \uc218 \uc5c6\ub2e4\"\ub294 \ubbf8\uad6d\uc758 \uc131\uc9c1\uc790 \ud5e8\ub9ac \ubc18 \ub2e4\uc774\ud06c\uc758 \ub9d0\uc744 \uc778\uc6a9\ud558\uba74\uc11c \uae40\ub300\uc911 \uc804 \ub300\ud1b5\ub839\uc744 \ube44\ub86f\ud574 \uc815.\uc7ac\uacc4, \uc5f0\uc608\uacc4, \ud559\uacc4\uc5d0\uc11c \ud0c1\uc6d4\ud55c \uc5c5\uc801\uc744 \ub0a8\uae34 \uc800\uba85\uc778\uc0ac 36\uc778\uc740 \uc62c\ud574 \uc138\uc0c1\uc744 \ub5a0\ub0ac\uc9c0\ub9cc \uc0ac\ub78c\ub4e4\uc758 \ub1cc\ub9ac \uc18d\uc5d0 \uc601\uc6d0\ud788 \uae30\uc5b5\ub420 \uac83\uc774\ub77c\uace0 \ud3c9\uac00\ud588\ub2e4. \ud2b9\ud788 \ub274\uc2a4\uc704\ud06c\ub294 \"\uae40 \uc804 \ub300\ud1b5\ub839\uc740 1997\ub144 \ud55c\uad6d\uc758 \ub300\ud1b5\ub839\uc5d0 \ub2f9\uc120\ub420 \ub54c\uae4c\uc9c0 \uad70\ubd80 \ub3c5\uc7ac\uc5d0 \ud56d\uac70\ud558\uace0 \ub9dd\uba85\uacfc \ud22c\uc625, \ub0a9\uce58, \uc554\uc0b4 \uc704\ud611 \ub4f1 \uace0\ub09c\uc758 \uc138\uc6d4\uc744 \uc774\uaca8\ub0b8 \uc57c\uad8c \uc9c0\ub3c4\uc790\"\uc600\ub2e4\uace0 \uc18c\uac1c\ud588\ub2e4. \ub610 \"\uae40 \uc804 \ub300\ud1b5\ub839\uc740 \uc7ac\uc784\uc2dc \ub0a8\ubd81 \ud654\ud574\ub97c \uc99d\uc9c4\uc2dc\ud0a8 \uacf5\ub85c\ub85c \ub178\ubca8 \ud3c9\ud654\uc0c1\uc744 \ubc1b\uc558\ub2e4\"\ub77c\uace0 \ub367\ubd99\uc600\ub2e4.", "sentence_answer": "2009\ub144 12\uc6d4 19\uc77c, \ub274\uc2a4\uc704\ud06c \uc778\ud130\ub137\ud310\uc774 '\uc62c\ud574 \uc138\uc0c1\uc744 \ub5a0\ub0ac\uc9c0\ub9cc \uc78a\ud600\uc9c0\uc9c0 \uc54a\uc744 \uc800\uba85\uc778\uc0ac'\ub85c \uae40\ub300\uc911 \uc804 \ub300\ud1b5\ub839\uc744 \ud3ec\ud568\ud574 \uc804 \uc138\uacc4 36\uba85\uc758 \uba85\uc0ac\ub97c \uc120\uc815\ud574 \uc18c\uac1c\ud588\ub2e4.", "paragraph_id": "6656766-41-1", "paragraph_question": "question: \uc62c\ud574 \uc138\uc0c1\uc744 \ub5a0\ub0ac\uc9c0\ub9cc \uc78a\ud600\uc9c0\uc9c0 \uc54a\uc744 \uc800\uba85\uc778\uc0ac \uc911 \ud558\ub098\ub85c \uae40\ub300\uc911\uc744 \uc120\uc815\ud55c \uc2e0\ubb38\uc0ac\ub294?, context: 2009\ub144 12\uc6d4 19\uc77c, \ub274\uc2a4\uc704\ud06c \uc778\ud130\ub137\ud310\uc774 '\uc62c\ud574 \uc138\uc0c1\uc744 \ub5a0\ub0ac\uc9c0\ub9cc \uc78a\ud600\uc9c0\uc9c0 \uc54a\uc744 \uc800\uba85\uc778\uc0ac'\ub85c \uae40\ub300\uc911 \uc804 \ub300\ud1b5\ub839\uc744 \ud3ec\ud568\ud574 \uc804 \uc138\uacc4 36\uba85\uc758 \uba85\uc0ac\ub97c \uc120\uc815\ud574 \uc18c\uac1c\ud588\ub2e4. \ub274\uc2a4\uc704\ud06c\ub294 \"\uc8fd\uc74c\uc744 \ub450\ub824\uc6cc\ud558\ub294 \uc790\ub4e4\uc740 \uc0b6\uc744 \uc2dc\uc791\ud560 \uc218 \uc5c6\ub2e4\"\ub294 \ubbf8\uad6d\uc758 \uc131\uc9c1\uc790 \ud5e8\ub9ac \ubc18 \ub2e4\uc774\ud06c\uc758 \ub9d0\uc744 \uc778\uc6a9\ud558\uba74\uc11c \uae40\ub300\uc911 \uc804 \ub300\ud1b5\ub839\uc744 \ube44\ub86f\ud574 \uc815.\uc7ac\uacc4, \uc5f0\uc608\uacc4, \ud559\uacc4\uc5d0\uc11c \ud0c1\uc6d4\ud55c \uc5c5\uc801\uc744 \ub0a8\uae34 \uc800\uba85\uc778\uc0ac 36\uc778\uc740 \uc62c\ud574 \uc138\uc0c1\uc744 \ub5a0\ub0ac\uc9c0\ub9cc \uc0ac\ub78c\ub4e4\uc758 \ub1cc\ub9ac \uc18d\uc5d0 \uc601\uc6d0\ud788 \uae30\uc5b5\ub420 \uac83\uc774\ub77c\uace0 \ud3c9\uac00\ud588\ub2e4. \ud2b9\ud788 \ub274\uc2a4\uc704\ud06c\ub294 \"\uae40 \uc804 \ub300\ud1b5\ub839\uc740 1997\ub144 \ud55c\uad6d\uc758 \ub300\ud1b5\ub839\uc5d0 \ub2f9\uc120\ub420 \ub54c\uae4c\uc9c0 \uad70\ubd80 \ub3c5\uc7ac\uc5d0 \ud56d\uac70\ud558\uace0 \ub9dd\uba85\uacfc \ud22c\uc625, \ub0a9\uce58, \uc554\uc0b4 \uc704\ud611 \ub4f1 \uace0\ub09c\uc758 \uc138\uc6d4\uc744 \uc774\uaca8\ub0b8 \uc57c\uad8c \uc9c0\ub3c4\uc790\"\uc600\ub2e4\uace0 \uc18c\uac1c\ud588\ub2e4. \ub610 \"\uae40 \uc804 \ub300\ud1b5\ub839\uc740 \uc7ac\uc784\uc2dc \ub0a8\ubd81 \ud654\ud574\ub97c \uc99d\uc9c4\uc2dc\ud0a8 \uacf5\ub85c\ub85c \ub178\ubca8 \ud3c9\ud654\uc0c1\uc744 \ubc1b\uc558\ub2e4\"\ub77c\uace0 \ub367\ubd99\uc600\ub2e4."} {"question": "\uc77c\ubcf8\uc0ac\ud68c\ub2f9\uc774 \uc0ac\ud68c\uc8fc\uc758\uc758 \uc870\uad6d\uc774\ub77c \uc0dd\uac01\ud55c \ub098\ub77c\ub294?", "paragraph": "1970\ub144\ub300\uc5d0 \ub4e4\uc5b4\uc11c\uba74\uc11c \uc0ac\ud68c\ub2f9\uc740 \uc77c\ubcf8\uacf5\uc0b0\ub2f9\uae4c\uc9c0 \ud3ec\ud568\ud55c \ubaa8\ub4e0 \uc57c\ub2f9\uc758 \uacf5\ub3d9\ud22c\uc7c1 \ub178\uc120\uc744 \ucde8\ud574, \uc9c0\uc790\uccb4\uc5d0\uc11c \uacf5\uc0b0\ub2f9\uacfc \ud568\uaed8 \uc120\uac70\ub97c \uc9c4\ud589\uc2dc\ucf30\ub2e4. \uadf8 \uacb0\uacfc \ub3c4\ucfc4\ub3c4, \uc624\uc0ac\uce74\ubd80 \ub4f1 \uac01\uc9c0\uc5d0\uc11c \ud601\uc2e0 \uc9c0\uc790\uccb4\uc758 \uc218\uc7a5\uc774 \ud0c4\uc0dd\ud588\ub2e4. \uc774\ub7f0 \ud601\uc2e0 \uc9c0\uc790\uccb4\ub4e4\uc740 \uc0ac\ud68c \ubcf5\uc9c0\uc758 \uac15\ud654 \ub4f1 \uc77c\uc815\ud55c \uacb0\uacfc\ub97c \ub0a8\uacbc\uc9c0\ub9cc, \uc7ac\uc815\uc758 \uc545\ud654\ub97c \ubd88\ub800\ub2e4\ub294 \ube44\ud310\ub3c4 \ubc1b\uace0\uc788\ub2e4. \uc774\ub7f0 \uac00\uc6b4\ub370 \uc77c\ubcf8\uc0ac\ud68c\uc8fc\uc758\uccad\ub144\ub3d9\ub9f9\uc5d0 \uc138\ub825\uc744 \uac00\uc9c0\uace0 \uc788\ub358 \uadf9\uc88c\ud30c\ub97c \ubc30\uc81c\ud558\uba74\uc11c \uc0ac\ud68c\uc8fc\uc758\ud611\ud68c\uc758 \uc601\ud5a5\ub825\uc774 \uc870\uc9c1\uc5d0\uc11c\ub3c4 \uac15\ud574\uc9c0\uae30 \uc2dc\uc791\ud588\ub2e4. \uc0ac\ud0a4\uc0ac\uce74 \uc774\uce20\ub85c\ub97c \uc9c0\ub3c4\uc790\ub85c \ud55c \ub2f9\uc2dc\uc758 \uc0ac\ud68c\uc8fc\uc758\ud611\ud68c\ub294 \ub9c8\ub974\ud06c\uc2a4\ub098 \ub808\ub2cc \ub4f1\uc758 \uc800\uc791\uc744 \uc808\ub300\uc2dc\ud558\uba74\uc11c, \uc18c\ub828\uc744 \u2018\uc0ac\ud68c\uc8fc\uc758\uc758 \uc870\uad6d\u2019\uc73c\ub85c \uc0dd\uac01\ud558\uba70 \uccb4\ucf54 \uce68\uacf5\uc744 \uacf5\uacf5\uc5f0\ud558\uac8c \uc9c0\uc9c0\ud558\ub294 \ub4f1, \uc0ac\ud68c\ub2f9\uc774 \uc81c\uc2dc\ud55c \ub2f9\uc2dc\uc758 \uc911\ub9bd\uc815\ucc45\uc774 \uc6a9\ub0a9\ud560 \uc218 \uc5c6\ub294 \uc9c0\uacbd\uc5d0\uae4c\uc9c0 \uc774\ub974\ub800\ub2e4. \uce5c\uc18c \uacbd\ud5a5\uc758 \uc0ac\ud68c\uc8fc\uc758\ud611\ud68c \uc138\ub825\uc774 \ud655\ub300\ub418\uba74\uc11c, \uc0ac\uc0ac\ud0a4 \uace0\uc870 \ub4f1\uc740 \uc608\uc804\uc5d0 \uc801\ub300\uc2dc\ud558\ub358 \uc5d0\ub2e4 \uc0ac\ubd80\ub85c\uc640 \uc5f0\ud569\ud558\uba74\uc11c, \uc774\ud6c4 \uc0ac\ud68c\uc8fc\uc758\ud611\ud68c \uc138\ub825\uacfc \ubc18\ud611\ud68c \uc138\ub825\uc758 \ub300\ub9bd\uc740 \uaca9\ud654\ub418\uc5b4 \uac14\ub2e4.", "answer": "\uc18c\ub828", "sentence": "\uc0ac\ud0a4\uc0ac\uce74 \uc774\uce20\ub85c\ub97c \uc9c0\ub3c4\uc790\ub85c \ud55c \ub2f9\uc2dc\uc758 \uc0ac\ud68c\uc8fc\uc758\ud611\ud68c\ub294 \ub9c8\ub974\ud06c\uc2a4\ub098 \ub808\ub2cc \ub4f1\uc758 \uc800\uc791\uc744 \uc808\ub300\uc2dc\ud558\uba74\uc11c, \uc18c\ub828 \uc744 \u2018\uc0ac\ud68c\uc8fc\uc758\uc758 \uc870\uad6d\u2019\uc73c\ub85c \uc0dd\uac01\ud558\uba70 \uccb4\ucf54 \uce68\uacf5\uc744 \uacf5\uacf5\uc5f0\ud558\uac8c \uc9c0\uc9c0\ud558\ub294 \ub4f1, \uc0ac\ud68c\ub2f9\uc774 \uc81c\uc2dc\ud55c \ub2f9\uc2dc\uc758 \uc911\ub9bd\uc815\ucc45\uc774 \uc6a9\ub0a9\ud560 \uc218 \uc5c6\ub294 \uc9c0\uacbd\uc5d0\uae4c\uc9c0 \uc774\ub974\ub800\ub2e4.", "paragraph_sentence": "1970\ub144\ub300\uc5d0 \ub4e4\uc5b4\uc11c\uba74\uc11c \uc0ac\ud68c\ub2f9\uc740 \uc77c\ubcf8\uacf5\uc0b0\ub2f9\uae4c\uc9c0 \ud3ec\ud568\ud55c \ubaa8\ub4e0 \uc57c\ub2f9\uc758 \uacf5\ub3d9\ud22c\uc7c1 \ub178\uc120\uc744 \ucde8\ud574, \uc9c0\uc790\uccb4\uc5d0\uc11c \uacf5\uc0b0\ub2f9\uacfc \ud568\uaed8 \uc120\uac70\ub97c \uc9c4\ud589\uc2dc\ucf30\ub2e4. \uadf8 \uacb0\uacfc \ub3c4\ucfc4\ub3c4, \uc624\uc0ac\uce74\ubd80 \ub4f1 \uac01\uc9c0\uc5d0\uc11c \ud601\uc2e0 \uc9c0\uc790\uccb4\uc758 \uc218\uc7a5\uc774 \ud0c4\uc0dd\ud588\ub2e4. \uc774\ub7f0 \ud601\uc2e0 \uc9c0\uc790\uccb4\ub4e4\uc740 \uc0ac\ud68c \ubcf5\uc9c0\uc758 \uac15\ud654 \ub4f1 \uc77c\uc815\ud55c \uacb0\uacfc\ub97c \ub0a8\uacbc\uc9c0\ub9cc, \uc7ac\uc815\uc758 \uc545\ud654\ub97c \ubd88\ub800\ub2e4\ub294 \ube44\ud310\ub3c4 \ubc1b\uace0\uc788\ub2e4. \uc774\ub7f0 \uac00\uc6b4\ub370 \uc77c\ubcf8\uc0ac\ud68c\uc8fc\uc758\uccad\ub144\ub3d9\ub9f9\uc5d0 \uc138\ub825\uc744 \uac00\uc9c0\uace0 \uc788\ub358 \uadf9\uc88c\ud30c\ub97c \ubc30\uc81c\ud558\uba74\uc11c \uc0ac\ud68c\uc8fc\uc758\ud611\ud68c\uc758 \uc601\ud5a5\ub825\uc774 \uc870\uc9c1\uc5d0\uc11c\ub3c4 \uac15\ud574\uc9c0\uae30 \uc2dc\uc791\ud588\ub2e4. \uc0ac\ud0a4\uc0ac\uce74 \uc774\uce20\ub85c\ub97c \uc9c0\ub3c4\uc790\ub85c \ud55c \ub2f9\uc2dc\uc758 \uc0ac\ud68c\uc8fc\uc758\ud611\ud68c\ub294 \ub9c8\ub974\ud06c\uc2a4\ub098 \ub808\ub2cc \ub4f1\uc758 \uc800\uc791\uc744 \uc808\ub300\uc2dc\ud558\uba74\uc11c, \uc18c\ub828 \uc744 \u2018\uc0ac\ud68c\uc8fc\uc758\uc758 \uc870\uad6d\u2019\uc73c\ub85c \uc0dd\uac01\ud558\uba70 \uccb4\ucf54 \uce68\uacf5\uc744 \uacf5\uacf5\uc5f0\ud558\uac8c \uc9c0\uc9c0\ud558\ub294 \ub4f1, \uc0ac\ud68c\ub2f9\uc774 \uc81c\uc2dc\ud55c \ub2f9\uc2dc\uc758 \uc911\ub9bd\uc815\ucc45\uc774 \uc6a9\ub0a9\ud560 \uc218 \uc5c6\ub294 \uc9c0\uacbd\uc5d0\uae4c\uc9c0 \uc774\ub974\ub800\ub2e4. \uce5c\uc18c \uacbd\ud5a5\uc758 \uc0ac\ud68c\uc8fc\uc758\ud611\ud68c \uc138\ub825\uc774 \ud655\ub300\ub418\uba74\uc11c, \uc0ac\uc0ac\ud0a4 \uace0\uc870 \ub4f1\uc740 \uc608\uc804\uc5d0 \uc801\ub300\uc2dc\ud558\ub358 \uc5d0\ub2e4 \uc0ac\ubd80\ub85c\uc640 \uc5f0\ud569\ud558\uba74\uc11c, \uc774\ud6c4 \uc0ac\ud68c\uc8fc\uc758\ud611\ud68c \uc138\ub825\uacfc \ubc18\ud611\ud68c \uc138\ub825\uc758 \ub300\ub9bd\uc740 \uaca9\ud654\ub418\uc5b4 \uac14\ub2e4.", "paragraph_answer": "1970\ub144\ub300\uc5d0 \ub4e4\uc5b4\uc11c\uba74\uc11c \uc0ac\ud68c\ub2f9\uc740 \uc77c\ubcf8\uacf5\uc0b0\ub2f9\uae4c\uc9c0 \ud3ec\ud568\ud55c \ubaa8\ub4e0 \uc57c\ub2f9\uc758 \uacf5\ub3d9\ud22c\uc7c1 \ub178\uc120\uc744 \ucde8\ud574, \uc9c0\uc790\uccb4\uc5d0\uc11c \uacf5\uc0b0\ub2f9\uacfc \ud568\uaed8 \uc120\uac70\ub97c \uc9c4\ud589\uc2dc\ucf30\ub2e4. \uadf8 \uacb0\uacfc \ub3c4\ucfc4\ub3c4, \uc624\uc0ac\uce74\ubd80 \ub4f1 \uac01\uc9c0\uc5d0\uc11c \ud601\uc2e0 \uc9c0\uc790\uccb4\uc758 \uc218\uc7a5\uc774 \ud0c4\uc0dd\ud588\ub2e4. \uc774\ub7f0 \ud601\uc2e0 \uc9c0\uc790\uccb4\ub4e4\uc740 \uc0ac\ud68c \ubcf5\uc9c0\uc758 \uac15\ud654 \ub4f1 \uc77c\uc815\ud55c \uacb0\uacfc\ub97c \ub0a8\uacbc\uc9c0\ub9cc, \uc7ac\uc815\uc758 \uc545\ud654\ub97c \ubd88\ub800\ub2e4\ub294 \ube44\ud310\ub3c4 \ubc1b\uace0\uc788\ub2e4. \uc774\ub7f0 \uac00\uc6b4\ub370 \uc77c\ubcf8\uc0ac\ud68c\uc8fc\uc758\uccad\ub144\ub3d9\ub9f9\uc5d0 \uc138\ub825\uc744 \uac00\uc9c0\uace0 \uc788\ub358 \uadf9\uc88c\ud30c\ub97c \ubc30\uc81c\ud558\uba74\uc11c \uc0ac\ud68c\uc8fc\uc758\ud611\ud68c\uc758 \uc601\ud5a5\ub825\uc774 \uc870\uc9c1\uc5d0\uc11c\ub3c4 \uac15\ud574\uc9c0\uae30 \uc2dc\uc791\ud588\ub2e4. \uc0ac\ud0a4\uc0ac\uce74 \uc774\uce20\ub85c\ub97c \uc9c0\ub3c4\uc790\ub85c \ud55c \ub2f9\uc2dc\uc758 \uc0ac\ud68c\uc8fc\uc758\ud611\ud68c\ub294 \ub9c8\ub974\ud06c\uc2a4\ub098 \ub808\ub2cc \ub4f1\uc758 \uc800\uc791\uc744 \uc808\ub300\uc2dc\ud558\uba74\uc11c, \uc18c\ub828 \uc744 \u2018\uc0ac\ud68c\uc8fc\uc758\uc758 \uc870\uad6d\u2019\uc73c\ub85c \uc0dd\uac01\ud558\uba70 \uccb4\ucf54 \uce68\uacf5\uc744 \uacf5\uacf5\uc5f0\ud558\uac8c \uc9c0\uc9c0\ud558\ub294 \ub4f1, \uc0ac\ud68c\ub2f9\uc774 \uc81c\uc2dc\ud55c \ub2f9\uc2dc\uc758 \uc911\ub9bd\uc815\ucc45\uc774 \uc6a9\ub0a9\ud560 \uc218 \uc5c6\ub294 \uc9c0\uacbd\uc5d0\uae4c\uc9c0 \uc774\ub974\ub800\ub2e4. \uce5c\uc18c \uacbd\ud5a5\uc758 \uc0ac\ud68c\uc8fc\uc758\ud611\ud68c \uc138\ub825\uc774 \ud655\ub300\ub418\uba74\uc11c, \uc0ac\uc0ac\ud0a4 \uace0\uc870 \ub4f1\uc740 \uc608\uc804\uc5d0 \uc801\ub300\uc2dc\ud558\ub358 \uc5d0\ub2e4 \uc0ac\ubd80\ub85c\uc640 \uc5f0\ud569\ud558\uba74\uc11c, \uc774\ud6c4 \uc0ac\ud68c\uc8fc\uc758\ud611\ud68c \uc138\ub825\uacfc \ubc18\ud611\ud68c \uc138\ub825\uc758 \ub300\ub9bd\uc740 \uaca9\ud654\ub418\uc5b4 \uac14\ub2e4.", "sentence_answer": "\uc0ac\ud0a4\uc0ac\uce74 \uc774\uce20\ub85c\ub97c \uc9c0\ub3c4\uc790\ub85c \ud55c \ub2f9\uc2dc\uc758 \uc0ac\ud68c\uc8fc\uc758\ud611\ud68c\ub294 \ub9c8\ub974\ud06c\uc2a4\ub098 \ub808\ub2cc \ub4f1\uc758 \uc800\uc791\uc744 \uc808\ub300\uc2dc\ud558\uba74\uc11c, \uc18c\ub828 \uc744 \u2018\uc0ac\ud68c\uc8fc\uc758\uc758 \uc870\uad6d\u2019\uc73c\ub85c \uc0dd\uac01\ud558\uba70 \uccb4\ucf54 \uce68\uacf5\uc744 \uacf5\uacf5\uc5f0\ud558\uac8c \uc9c0\uc9c0\ud558\ub294 \ub4f1, \uc0ac\ud68c\ub2f9\uc774 \uc81c\uc2dc\ud55c \ub2f9\uc2dc\uc758 \uc911\ub9bd\uc815\ucc45\uc774 \uc6a9\ub0a9\ud560 \uc218 \uc5c6\ub294 \uc9c0\uacbd\uc5d0\uae4c\uc9c0 \uc774\ub974\ub800\ub2e4.", "paragraph_id": "6591960-3-1", "paragraph_question": "question: \uc77c\ubcf8\uc0ac\ud68c\ub2f9\uc774 \uc0ac\ud68c\uc8fc\uc758\uc758 \uc870\uad6d\uc774\ub77c \uc0dd\uac01\ud55c \ub098\ub77c\ub294?, context: 1970\ub144\ub300\uc5d0 \ub4e4\uc5b4\uc11c\uba74\uc11c \uc0ac\ud68c\ub2f9\uc740 \uc77c\ubcf8\uacf5\uc0b0\ub2f9\uae4c\uc9c0 \ud3ec\ud568\ud55c \ubaa8\ub4e0 \uc57c\ub2f9\uc758 \uacf5\ub3d9\ud22c\uc7c1 \ub178\uc120\uc744 \ucde8\ud574, \uc9c0\uc790\uccb4\uc5d0\uc11c \uacf5\uc0b0\ub2f9\uacfc \ud568\uaed8 \uc120\uac70\ub97c \uc9c4\ud589\uc2dc\ucf30\ub2e4. \uadf8 \uacb0\uacfc \ub3c4\ucfc4\ub3c4, \uc624\uc0ac\uce74\ubd80 \ub4f1 \uac01\uc9c0\uc5d0\uc11c \ud601\uc2e0 \uc9c0\uc790\uccb4\uc758 \uc218\uc7a5\uc774 \ud0c4\uc0dd\ud588\ub2e4. \uc774\ub7f0 \ud601\uc2e0 \uc9c0\uc790\uccb4\ub4e4\uc740 \uc0ac\ud68c \ubcf5\uc9c0\uc758 \uac15\ud654 \ub4f1 \uc77c\uc815\ud55c \uacb0\uacfc\ub97c \ub0a8\uacbc\uc9c0\ub9cc, \uc7ac\uc815\uc758 \uc545\ud654\ub97c \ubd88\ub800\ub2e4\ub294 \ube44\ud310\ub3c4 \ubc1b\uace0\uc788\ub2e4. \uc774\ub7f0 \uac00\uc6b4\ub370 \uc77c\ubcf8\uc0ac\ud68c\uc8fc\uc758\uccad\ub144\ub3d9\ub9f9\uc5d0 \uc138\ub825\uc744 \uac00\uc9c0\uace0 \uc788\ub358 \uadf9\uc88c\ud30c\ub97c \ubc30\uc81c\ud558\uba74\uc11c \uc0ac\ud68c\uc8fc\uc758\ud611\ud68c\uc758 \uc601\ud5a5\ub825\uc774 \uc870\uc9c1\uc5d0\uc11c\ub3c4 \uac15\ud574\uc9c0\uae30 \uc2dc\uc791\ud588\ub2e4. \uc0ac\ud0a4\uc0ac\uce74 \uc774\uce20\ub85c\ub97c \uc9c0\ub3c4\uc790\ub85c \ud55c \ub2f9\uc2dc\uc758 \uc0ac\ud68c\uc8fc\uc758\ud611\ud68c\ub294 \ub9c8\ub974\ud06c\uc2a4\ub098 \ub808\ub2cc \ub4f1\uc758 \uc800\uc791\uc744 \uc808\ub300\uc2dc\ud558\uba74\uc11c, \uc18c\ub828\uc744 \u2018\uc0ac\ud68c\uc8fc\uc758\uc758 \uc870\uad6d\u2019\uc73c\ub85c \uc0dd\uac01\ud558\uba70 \uccb4\ucf54 \uce68\uacf5\uc744 \uacf5\uacf5\uc5f0\ud558\uac8c \uc9c0\uc9c0\ud558\ub294 \ub4f1, \uc0ac\ud68c\ub2f9\uc774 \uc81c\uc2dc\ud55c \ub2f9\uc2dc\uc758 \uc911\ub9bd\uc815\ucc45\uc774 \uc6a9\ub0a9\ud560 \uc218 \uc5c6\ub294 \uc9c0\uacbd\uc5d0\uae4c\uc9c0 \uc774\ub974\ub800\ub2e4. \uce5c\uc18c \uacbd\ud5a5\uc758 \uc0ac\ud68c\uc8fc\uc758\ud611\ud68c \uc138\ub825\uc774 \ud655\ub300\ub418\uba74\uc11c, \uc0ac\uc0ac\ud0a4 \uace0\uc870 \ub4f1\uc740 \uc608\uc804\uc5d0 \uc801\ub300\uc2dc\ud558\ub358 \uc5d0\ub2e4 \uc0ac\ubd80\ub85c\uc640 \uc5f0\ud569\ud558\uba74\uc11c, \uc774\ud6c4 \uc0ac\ud68c\uc8fc\uc758\ud611\ud68c \uc138\ub825\uacfc \ubc18\ud611\ud68c \uc138\ub825\uc758 \ub300\ub9bd\uc740 \uaca9\ud654\ub418\uc5b4 \uac14\ub2e4."} {"question": "\uc804\ub450\ud658\uc774 1996\ub144 9\uc6d4 \uc11c\uc6b8\uc9c0\ubc29\ubc95\uc6d0\uc5d0\uc11c \ubc1b\uc740 \ud310\uacb0\uc740?", "paragraph": "1995\ub144 12.12 5.18 \uc7ac\ud310\uacfc\uc815 \uc911 \uadf8\ub294 5.18\uc758 \uc9c4\uc555\ub2f9\uc704\uc131\uc744 \uc8fc\uc7a5\ud558\uba70 \uc88c\ud30c\uc138\ub825\uc758 \uacf5\uc138\ub97c \uc8fc\uc7a5\ud588\ub2e4. \uadf8\uc758 \uad11\uc8fc \uc88c\ud30c \ubc1c\uc5b8\uc774 \uc54c\ub824\uc9c0\uc790 \uc2dc\uc778 \uacf5\uc120\uc625 \ub4f1\uc740 \uadf8\uc5d0\uac8c \uc778\uac04\uc774 \ub418\ub77c\uba70 \ud56d\uc758\ud558\uae30\ub3c4 \ud588\ub2e4. \uacf5\ud310\uc740 1996\ub144 3\uc6d4\ubd80\ud130 \uc9c4\ud589\uc774 \ub410\ub294\ub370, 8\uc6d4 26\uc77c \uc11c\uc6b8\uc9c0\ubc29\ubc95\uc6d0\uc5d0\uc11c \uc0ac\ud615\uc744, 12\uc6d4 16\uc77c \uc11c\uc6b8\uace0\ub4f1\ubc95\uc6d0\uc5d0\uc11c \ubb34\uae30\uc9d5\uc5ed\uacfc \ucd94\uc9d5\uae08 2,205\uc5b5 \uc6d0\uc744(96\ub1781892) \uac01 \uc120\uace0\ubc1b\uc558\uc73c\uba70, 1997\ub144 4\uc6d4 17\uc77c\uc5d0 \ub300\ubc95\uc6d0\uc5d0\uc11c \ud655\uc815\ub410\ub2e4. \ub2f9\uc2dc \ub300\ubc95\uc6d0\uc740 \uc804\ub450\ud658 \ub4f1\uc774 \"\ubc18\ub780\uc218\uad34\u00b7\ubc18\ub780\ubaa8\uc758\ucc38\uc5ec\u00b7\ubc18\ub780\uc911\uc694\uc784\ubb34\uc885\uc0ac\u00b7\ubd88\ubc95\uc9c4\ud1f4\u00b7\uc9c0\ud718\uad00\uacc4\uc5c4\uc9c0\uc5ed\uc218\uc18c\uc774\ud0c8\u00b7\uc0c1\uad00\uc0b4\ud574\u00b7\uc0c1\uad00\uc0b4\ud574\ubbf8\uc218\u00b7\ucd08\ubcd1\uc0b4\ud574\u00b7\ub0b4\ub780\uc218\uad34\u00b7\ub0b4\ub780\ubaa8\uc758\ucc38\uc5ec\u00b7\ub0b4\ub780\uc911\uc694\uc784\ubb34\uc885\uc0ac\u00b7\ub0b4\ub780\ubaa9\uc801\uc0b4\uc778\u00b7\ud2b9\uc815\ubc94\uc8c4\uac00\uc911\ucc98\ubc8c \ub4f1\uc5d0 \uad00\ud55c \ubc95\ub960\uc704\ubc18(\ub1cc\ubb3c)\"\uacfc \uac19\uc740 \ubc94\uc8c4\ub97c \uc800\uc9c0\ub978 \uac83\uc73c\ub85c \ud310\uacb0\ud588\ub2e4. \ub2f9\uc2dc \uadf8\uc758 \uc218\uc778\ubc88\ud638\ub294 3124\ubc88\uc774\uc5c8\ub2e4. \uadf8\ub7ec\ub098 \uc218\uc2dc\ub85c \uc548\uc591\uad50\ub3c4\uc18c\uc5d0\ub294 \uba74\ud68c\uac1d\uc774 \uc904\uc744 \uc774\uc5c8\uace0 \uad50\ub3c4\uad00\ub4e4\ub3c4 \uc804\ub450\ud658\uc5d0\uac8c\ub294 \ub2e4\ub978 \uc0ac\ub78c\ub4e4\ucc98\ub7fc \uc218\uc778\ubc88\ud638\ub85c \ud638\ucd9c\ud558\uc9c0 \ubabb\ud588\ub2e4. \uc804\ub450\ud658, \ub178\ud0dc\uc6b0\uc758 \uad6c\uc18d \uc774\ud6c4 \uae40\uc601\uc0bc\uc5d0 \ub300\ud55c \ubcf4\uc218\uc8fc\uc758 \uc138\ub825\uc758 \ubc18\ubc1c\uacfc \ubc18\uac10\uc740 \uacc4\uc18d\ub410\uace0, \uc0ac\ud68c\uc801 \uac08\ub4f1\uc744 \uc720\ubc1c\ud588\ub2e4. \uc774\ud6c4 \uce5c\uc804\ub450\ud658 \uc778\uc0ac\uc640 \uc81c5\uacf5\ud654\uad6d \uac00\uc2e0 \uc138\ub825\uc5d0 \uc758\ud55c 5\uacf5\uc2e0\ub2f9 \ucc3d\ub2f9\uc774 \uc2dc\ub3c4\ub410\uc73c\ub098 \uc2dc\ubbfc, \uc0ac\ud68c\ub2e8\uccb4\ub4e4\uc758 \ubc18\ubc1c\uacfc \uc815\ubd80\uc758 \uc555\ub825\uc73c\ub85c \ubb34\uc0b0\ub418\uae30\ub3c4 \ud588\ub2e4.", "answer": "\uc0ac\ud615", "sentence": "\uacf5\ud310\uc740 1996\ub144 3\uc6d4\ubd80\ud130 \uc9c4\ud589\uc774 \ub410\ub294\ub370, 8\uc6d4 26\uc77c \uc11c\uc6b8\uc9c0\ubc29\ubc95\uc6d0\uc5d0\uc11c \uc0ac\ud615 \uc744, 12\uc6d4 16\uc77c \uc11c\uc6b8\uace0\ub4f1\ubc95\uc6d0\uc5d0\uc11c \ubb34\uae30\uc9d5\uc5ed\uacfc \ucd94\uc9d5\uae08 2,205\uc5b5 \uc6d0\uc744(96\ub1781892) \uac01 \uc120\uace0\ubc1b\uc558\uc73c\uba70, 1997\ub144 4\uc6d4 17\uc77c\uc5d0 \ub300\ubc95\uc6d0\uc5d0\uc11c \ud655\uc815\ub410\ub2e4.", "paragraph_sentence": "1995\ub144 12.12 5.18 \uc7ac\ud310\uacfc\uc815 \uc911 \uadf8\ub294 5.18\uc758 \uc9c4\uc555\ub2f9\uc704\uc131\uc744 \uc8fc\uc7a5\ud558\uba70 \uc88c\ud30c\uc138\ub825\uc758 \uacf5\uc138\ub97c \uc8fc\uc7a5\ud588\ub2e4. \uadf8\uc758 \uad11\uc8fc \uc88c\ud30c \ubc1c\uc5b8\uc774 \uc54c\ub824\uc9c0\uc790 \uc2dc\uc778 \uacf5\uc120\uc625 \ub4f1\uc740 \uadf8\uc5d0\uac8c \uc778\uac04\uc774 \ub418\ub77c\uba70 \ud56d\uc758\ud558\uae30\ub3c4 \ud588\ub2e4. \uacf5\ud310\uc740 1996\ub144 3\uc6d4\ubd80\ud130 \uc9c4\ud589\uc774 \ub410\ub294\ub370, 8\uc6d4 26\uc77c \uc11c\uc6b8\uc9c0\ubc29\ubc95\uc6d0\uc5d0\uc11c \uc0ac\ud615 \uc744, 12\uc6d4 16\uc77c \uc11c\uc6b8\uace0\ub4f1\ubc95\uc6d0\uc5d0\uc11c \ubb34\uae30\uc9d5\uc5ed\uacfc \ucd94\uc9d5\uae08 2,205\uc5b5 \uc6d0\uc744(96\ub1781892) \uac01 \uc120\uace0\ubc1b\uc558\uc73c\uba70, 1997\ub144 4\uc6d4 17\uc77c\uc5d0 \ub300\ubc95\uc6d0\uc5d0\uc11c \ud655\uc815\ub410\ub2e4. \ub2f9\uc2dc \ub300\ubc95\uc6d0\uc740 \uc804\ub450\ud658 \ub4f1\uc774 \"\ubc18\ub780\uc218\uad34\u00b7\ubc18\ub780\ubaa8\uc758\ucc38\uc5ec\u00b7\ubc18\ub780\uc911\uc694\uc784\ubb34\uc885\uc0ac\u00b7\ubd88\ubc95\uc9c4\ud1f4\u00b7\uc9c0\ud718\uad00\uacc4\uc5c4\uc9c0\uc5ed\uc218\uc18c\uc774\ud0c8\u00b7\uc0c1\uad00\uc0b4\ud574\u00b7\uc0c1\uad00\uc0b4\ud574\ubbf8\uc218\u00b7\ucd08\ubcd1\uc0b4\ud574\u00b7\ub0b4\ub780\uc218\uad34\u00b7\ub0b4\ub780\ubaa8\uc758\ucc38\uc5ec\u00b7\ub0b4\ub780\uc911\uc694\uc784\ubb34\uc885\uc0ac\u00b7\ub0b4\ub780\ubaa9\uc801\uc0b4\uc778\u00b7\ud2b9\uc815\ubc94\uc8c4\uac00\uc911\ucc98\ubc8c \ub4f1\uc5d0 \uad00\ud55c \ubc95\ub960\uc704\ubc18(\ub1cc\ubb3c)\"\uacfc \uac19\uc740 \ubc94\uc8c4\ub97c \uc800\uc9c0\ub978 \uac83\uc73c\ub85c \ud310\uacb0\ud588\ub2e4. \ub2f9\uc2dc \uadf8\uc758 \uc218\uc778\ubc88\ud638\ub294 3124\ubc88\uc774\uc5c8\ub2e4. \uadf8\ub7ec\ub098 \uc218\uc2dc\ub85c \uc548\uc591\uad50\ub3c4\uc18c\uc5d0\ub294 \uba74\ud68c\uac1d\uc774 \uc904\uc744 \uc774\uc5c8\uace0 \uad50\ub3c4\uad00\ub4e4\ub3c4 \uc804\ub450\ud658\uc5d0\uac8c\ub294 \ub2e4\ub978 \uc0ac\ub78c\ub4e4\ucc98\ub7fc \uc218\uc778\ubc88\ud638\ub85c \ud638\ucd9c\ud558\uc9c0 \ubabb\ud588\ub2e4. \uc804\ub450\ud658, \ub178\ud0dc\uc6b0\uc758 \uad6c\uc18d \uc774\ud6c4 \uae40\uc601\uc0bc\uc5d0 \ub300\ud55c \ubcf4\uc218\uc8fc\uc758 \uc138\ub825\uc758 \ubc18\ubc1c\uacfc \ubc18\uac10\uc740 \uacc4\uc18d\ub410\uace0, \uc0ac\ud68c\uc801 \uac08\ub4f1\uc744 \uc720\ubc1c\ud588\ub2e4. \uc774\ud6c4 \uce5c\uc804\ub450\ud658 \uc778\uc0ac\uc640 \uc81c5\uacf5\ud654\uad6d \uac00\uc2e0 \uc138\ub825\uc5d0 \uc758\ud55c 5\uacf5\uc2e0\ub2f9 \ucc3d\ub2f9\uc774 \uc2dc\ub3c4\ub410\uc73c\ub098 \uc2dc\ubbfc, \uc0ac\ud68c\ub2e8\uccb4\ub4e4\uc758 \ubc18\ubc1c\uacfc \uc815\ubd80\uc758 \uc555\ub825\uc73c\ub85c \ubb34\uc0b0\ub418\uae30\ub3c4 \ud588\ub2e4.", "paragraph_answer": "1995\ub144 12.12 5.18 \uc7ac\ud310\uacfc\uc815 \uc911 \uadf8\ub294 5.18\uc758 \uc9c4\uc555\ub2f9\uc704\uc131\uc744 \uc8fc\uc7a5\ud558\uba70 \uc88c\ud30c\uc138\ub825\uc758 \uacf5\uc138\ub97c \uc8fc\uc7a5\ud588\ub2e4. \uadf8\uc758 \uad11\uc8fc \uc88c\ud30c \ubc1c\uc5b8\uc774 \uc54c\ub824\uc9c0\uc790 \uc2dc\uc778 \uacf5\uc120\uc625 \ub4f1\uc740 \uadf8\uc5d0\uac8c \uc778\uac04\uc774 \ub418\ub77c\uba70 \ud56d\uc758\ud558\uae30\ub3c4 \ud588\ub2e4. \uacf5\ud310\uc740 1996\ub144 3\uc6d4\ubd80\ud130 \uc9c4\ud589\uc774 \ub410\ub294\ub370, 8\uc6d4 26\uc77c \uc11c\uc6b8\uc9c0\ubc29\ubc95\uc6d0\uc5d0\uc11c \uc0ac\ud615 \uc744, 12\uc6d4 16\uc77c \uc11c\uc6b8\uace0\ub4f1\ubc95\uc6d0\uc5d0\uc11c \ubb34\uae30\uc9d5\uc5ed\uacfc \ucd94\uc9d5\uae08 2,205\uc5b5 \uc6d0\uc744(96\ub1781892) \uac01 \uc120\uace0\ubc1b\uc558\uc73c\uba70, 1997\ub144 4\uc6d4 17\uc77c\uc5d0 \ub300\ubc95\uc6d0\uc5d0\uc11c \ud655\uc815\ub410\ub2e4. \ub2f9\uc2dc \ub300\ubc95\uc6d0\uc740 \uc804\ub450\ud658 \ub4f1\uc774 \"\ubc18\ub780\uc218\uad34\u00b7\ubc18\ub780\ubaa8\uc758\ucc38\uc5ec\u00b7\ubc18\ub780\uc911\uc694\uc784\ubb34\uc885\uc0ac\u00b7\ubd88\ubc95\uc9c4\ud1f4\u00b7\uc9c0\ud718\uad00\uacc4\uc5c4\uc9c0\uc5ed\uc218\uc18c\uc774\ud0c8\u00b7\uc0c1\uad00\uc0b4\ud574\u00b7\uc0c1\uad00\uc0b4\ud574\ubbf8\uc218\u00b7\ucd08\ubcd1\uc0b4\ud574\u00b7\ub0b4\ub780\uc218\uad34\u00b7\ub0b4\ub780\ubaa8\uc758\ucc38\uc5ec\u00b7\ub0b4\ub780\uc911\uc694\uc784\ubb34\uc885\uc0ac\u00b7\ub0b4\ub780\ubaa9\uc801\uc0b4\uc778\u00b7\ud2b9\uc815\ubc94\uc8c4\uac00\uc911\ucc98\ubc8c \ub4f1\uc5d0 \uad00\ud55c \ubc95\ub960\uc704\ubc18(\ub1cc\ubb3c)\"\uacfc \uac19\uc740 \ubc94\uc8c4\ub97c \uc800\uc9c0\ub978 \uac83\uc73c\ub85c \ud310\uacb0\ud588\ub2e4. \ub2f9\uc2dc \uadf8\uc758 \uc218\uc778\ubc88\ud638\ub294 3124\ubc88\uc774\uc5c8\ub2e4. \uadf8\ub7ec\ub098 \uc218\uc2dc\ub85c \uc548\uc591\uad50\ub3c4\uc18c\uc5d0\ub294 \uba74\ud68c\uac1d\uc774 \uc904\uc744 \uc774\uc5c8\uace0 \uad50\ub3c4\uad00\ub4e4\ub3c4 \uc804\ub450\ud658\uc5d0\uac8c\ub294 \ub2e4\ub978 \uc0ac\ub78c\ub4e4\ucc98\ub7fc \uc218\uc778\ubc88\ud638\ub85c \ud638\ucd9c\ud558\uc9c0 \ubabb\ud588\ub2e4. \uc804\ub450\ud658, \ub178\ud0dc\uc6b0\uc758 \uad6c\uc18d \uc774\ud6c4 \uae40\uc601\uc0bc\uc5d0 \ub300\ud55c \ubcf4\uc218\uc8fc\uc758 \uc138\ub825\uc758 \ubc18\ubc1c\uacfc \ubc18\uac10\uc740 \uacc4\uc18d\ub410\uace0, \uc0ac\ud68c\uc801 \uac08\ub4f1\uc744 \uc720\ubc1c\ud588\ub2e4. \uc774\ud6c4 \uce5c\uc804\ub450\ud658 \uc778\uc0ac\uc640 \uc81c5\uacf5\ud654\uad6d \uac00\uc2e0 \uc138\ub825\uc5d0 \uc758\ud55c 5\uacf5\uc2e0\ub2f9 \ucc3d\ub2f9\uc774 \uc2dc\ub3c4\ub410\uc73c\ub098 \uc2dc\ubbfc, \uc0ac\ud68c\ub2e8\uccb4\ub4e4\uc758 \ubc18\ubc1c\uacfc \uc815\ubd80\uc758 \uc555\ub825\uc73c\ub85c \ubb34\uc0b0\ub418\uae30\ub3c4 \ud588\ub2e4.", "sentence_answer": "\uacf5\ud310\uc740 1996\ub144 3\uc6d4\ubd80\ud130 \uc9c4\ud589\uc774 \ub410\ub294\ub370, 8\uc6d4 26\uc77c \uc11c\uc6b8\uc9c0\ubc29\ubc95\uc6d0\uc5d0\uc11c \uc0ac\ud615 \uc744, 12\uc6d4 16\uc77c \uc11c\uc6b8\uace0\ub4f1\ubc95\uc6d0\uc5d0\uc11c \ubb34\uae30\uc9d5\uc5ed\uacfc \ucd94\uc9d5\uae08 2,205\uc5b5 \uc6d0\uc744(96\ub1781892) \uac01 \uc120\uace0\ubc1b\uc558\uc73c\uba70, 1997\ub144 4\uc6d4 17\uc77c\uc5d0 \ub300\ubc95\uc6d0\uc5d0\uc11c \ud655\uc815\ub410\ub2e4.", "paragraph_id": "6390080-31-0", "paragraph_question": "question: \uc804\ub450\ud658\uc774 1996\ub144 9\uc6d4 \uc11c\uc6b8\uc9c0\ubc29\ubc95\uc6d0\uc5d0\uc11c \ubc1b\uc740 \ud310\uacb0\uc740?, context: 1995\ub144 12.12 5.18 \uc7ac\ud310\uacfc\uc815 \uc911 \uadf8\ub294 5.18\uc758 \uc9c4\uc555\ub2f9\uc704\uc131\uc744 \uc8fc\uc7a5\ud558\uba70 \uc88c\ud30c\uc138\ub825\uc758 \uacf5\uc138\ub97c \uc8fc\uc7a5\ud588\ub2e4. \uadf8\uc758 \uad11\uc8fc \uc88c\ud30c \ubc1c\uc5b8\uc774 \uc54c\ub824\uc9c0\uc790 \uc2dc\uc778 \uacf5\uc120\uc625 \ub4f1\uc740 \uadf8\uc5d0\uac8c \uc778\uac04\uc774 \ub418\ub77c\uba70 \ud56d\uc758\ud558\uae30\ub3c4 \ud588\ub2e4. \uacf5\ud310\uc740 1996\ub144 3\uc6d4\ubd80\ud130 \uc9c4\ud589\uc774 \ub410\ub294\ub370, 8\uc6d4 26\uc77c \uc11c\uc6b8\uc9c0\ubc29\ubc95\uc6d0\uc5d0\uc11c \uc0ac\ud615\uc744, 12\uc6d4 16\uc77c \uc11c\uc6b8\uace0\ub4f1\ubc95\uc6d0\uc5d0\uc11c \ubb34\uae30\uc9d5\uc5ed\uacfc \ucd94\uc9d5\uae08 2,205\uc5b5 \uc6d0\uc744(96\ub1781892) \uac01 \uc120\uace0\ubc1b\uc558\uc73c\uba70, 1997\ub144 4\uc6d4 17\uc77c\uc5d0 \ub300\ubc95\uc6d0\uc5d0\uc11c \ud655\uc815\ub410\ub2e4. \ub2f9\uc2dc \ub300\ubc95\uc6d0\uc740 \uc804\ub450\ud658 \ub4f1\uc774 \"\ubc18\ub780\uc218\uad34\u00b7\ubc18\ub780\ubaa8\uc758\ucc38\uc5ec\u00b7\ubc18\ub780\uc911\uc694\uc784\ubb34\uc885\uc0ac\u00b7\ubd88\ubc95\uc9c4\ud1f4\u00b7\uc9c0\ud718\uad00\uacc4\uc5c4\uc9c0\uc5ed\uc218\uc18c\uc774\ud0c8\u00b7\uc0c1\uad00\uc0b4\ud574\u00b7\uc0c1\uad00\uc0b4\ud574\ubbf8\uc218\u00b7\ucd08\ubcd1\uc0b4\ud574\u00b7\ub0b4\ub780\uc218\uad34\u00b7\ub0b4\ub780\ubaa8\uc758\ucc38\uc5ec\u00b7\ub0b4\ub780\uc911\uc694\uc784\ubb34\uc885\uc0ac\u00b7\ub0b4\ub780\ubaa9\uc801\uc0b4\uc778\u00b7\ud2b9\uc815\ubc94\uc8c4\uac00\uc911\ucc98\ubc8c \ub4f1\uc5d0 \uad00\ud55c \ubc95\ub960\uc704\ubc18(\ub1cc\ubb3c)\"\uacfc \uac19\uc740 \ubc94\uc8c4\ub97c \uc800\uc9c0\ub978 \uac83\uc73c\ub85c \ud310\uacb0\ud588\ub2e4. \ub2f9\uc2dc \uadf8\uc758 \uc218\uc778\ubc88\ud638\ub294 3124\ubc88\uc774\uc5c8\ub2e4. \uadf8\ub7ec\ub098 \uc218\uc2dc\ub85c \uc548\uc591\uad50\ub3c4\uc18c\uc5d0\ub294 \uba74\ud68c\uac1d\uc774 \uc904\uc744 \uc774\uc5c8\uace0 \uad50\ub3c4\uad00\ub4e4\ub3c4 \uc804\ub450\ud658\uc5d0\uac8c\ub294 \ub2e4\ub978 \uc0ac\ub78c\ub4e4\ucc98\ub7fc \uc218\uc778\ubc88\ud638\ub85c \ud638\ucd9c\ud558\uc9c0 \ubabb\ud588\ub2e4. \uc804\ub450\ud658, \ub178\ud0dc\uc6b0\uc758 \uad6c\uc18d \uc774\ud6c4 \uae40\uc601\uc0bc\uc5d0 \ub300\ud55c \ubcf4\uc218\uc8fc\uc758 \uc138\ub825\uc758 \ubc18\ubc1c\uacfc \ubc18\uac10\uc740 \uacc4\uc18d\ub410\uace0, \uc0ac\ud68c\uc801 \uac08\ub4f1\uc744 \uc720\ubc1c\ud588\ub2e4. \uc774\ud6c4 \uce5c\uc804\ub450\ud658 \uc778\uc0ac\uc640 \uc81c5\uacf5\ud654\uad6d \uac00\uc2e0 \uc138\ub825\uc5d0 \uc758\ud55c 5\uacf5\uc2e0\ub2f9 \ucc3d\ub2f9\uc774 \uc2dc\ub3c4\ub410\uc73c\ub098 \uc2dc\ubbfc, \uc0ac\ud68c\ub2e8\uccb4\ub4e4\uc758 \ubc18\ubc1c\uacfc \uc815\ubd80\uc758 \uc555\ub825\uc73c\ub85c \ubb34\uc0b0\ub418\uae30\ub3c4 \ud588\ub2e4."} {"question": "\uae40\uc815\uc740\uc758 \uc131\ud615 \uc218\uc220\uc5d0 \ub300\ud574 \ubcf4\ub3c4\ud55c \uc911\uad6d\uc758 \ubc29\uc1a1\uc740?", "paragraph": "\uc911\uad6d \uc120\uc804 \uc704\uc131TV\ub294 2013\ub144 1\uc6d4 19\uc77c \uae40\uc815\uc740 \uc81c1\uc704\uc6d0\uc7a5\uc774 \ud560\uc544\ubc84\uc9c0 \uae40\uc77c\uc131 \uc8fc\uc11d\uacfc \ub2ee\uc544 \ubcf4\uc774\ub3c4\ub85d \uc131\ud615\uc218\uc220\uc744 \ud588\ub2e4\ub294 \uc0ac\uc2e4\uc744 \ud655\uc778\ud588\ub2e4\uace0 \ubcf4\ub3c4\ud55c \ubc14 \uc788\ub2e4. \uc911\uad6d \uc5b8\ub860\uc778 \uad00\uc57c\uc624(\u7ba1\u59da)\ub294 \uc774 \ubc29\uc1a1\uc5d0\uc11c \ubd81\ud55c\uc744 \ubc29\ubb38\ud588\ub358 \uc911\uad6d \uc678\uad50\uad00\uc744 \uc5b8\uae09\ud558\uba70 \"\uc6b0\ub9ac \uc678\uad50\uad00\uc774 \ubd81\ud55c\uc744 \ubc29\ubb38\ud588\uc744 \ub54c \uc0ac\uc801\uc778 \ub300\ud654\uc5d0\uc11c \ubd81\ud55c \uc678\uad50\uad00\uc5d0\uac8c `\uae40\uc815\uc740\uc774 \uc870\ubd80 \uae40\uc77c\uc131\uacfc \uaf2d \ub2ee\uc558\ub2e4`\uace0 \ud558\uc790 `\uc131\ud615\uc218\uc220\uc744 \ud588\ub2e4`\uace0 \ub2f5\ud588\ub2e4\"\uace0 \uc804\ud588\ub2e4. \uc2e0\ud654\ud1b5\uc2e0\uc740 \uc11c\uc6b8 \uc8fc\uc7ac \ud2b9\ud30c\uc6d0\uacfc \ud3c9\uc591 \uc8fc\uc7ac \ud2b9\ud30c\uc6d0\uc744 \ubaa8\ub450 \uc5f0\uacb0\ud574 \ubcf4\ub3c4 \uacbd\uc704\uc640 \uadfc\uac70\ub97c \ucd94\uc801\ud55c \uac83\uc73c\ub85c \uc54c\ub824\uc84c\ub2e4. \ud3c9\uc591 \uc8fc\uc7ac \ud2b9\ud30c\uc6d0\uc740 \"\ubd81\ud55c\uc5d0\uc120 \uae40\uc815\uc740 \uc131\ud615\uacfc \uad00\ub828\ub41c \ubcf4\ub3c4\uac00 \ud55c \ubc88\ub3c4 \uc5c6\uc5c8\ub2e4\"\uba70 \"\uadfc\uac70 \uc5c6\ub294 \uc18c\ubb38\"\uc774\ub77c\uace0 \ub9d0\ud588\ub2e4\uace0 \uc2e0\ud654\ud1b5\uc2e0\uc774 \uc804\ud588\ub2e4.", "answer": "\uc120\uc804 \uc704\uc131TV", "sentence": "\uc911\uad6d \uc120\uc804 \uc704\uc131TV \ub294 2013\ub144 1\uc6d4 19\uc77c \uae40\uc815\uc740 \uc81c1\uc704\uc6d0\uc7a5\uc774 \ud560\uc544\ubc84\uc9c0 \uae40\uc77c\uc131 \uc8fc\uc11d\uacfc \ub2ee\uc544 \ubcf4\uc774\ub3c4\ub85d \uc131\ud615\uc218\uc220\uc744 \ud588\ub2e4\ub294 \uc0ac\uc2e4\uc744 \ud655\uc778\ud588\ub2e4\uace0 \ubcf4\ub3c4\ud55c \ubc14 \uc788\ub2e4.", "paragraph_sentence": " \uc911\uad6d \uc120\uc804 \uc704\uc131TV \ub294 2013\ub144 1\uc6d4 19\uc77c \uae40\uc815\uc740 \uc81c1\uc704\uc6d0\uc7a5\uc774 \ud560\uc544\ubc84\uc9c0 \uae40\uc77c\uc131 \uc8fc\uc11d\uacfc \ub2ee\uc544 \ubcf4\uc774\ub3c4\ub85d \uc131\ud615\uc218\uc220\uc744 \ud588\ub2e4\ub294 \uc0ac\uc2e4\uc744 \ud655\uc778\ud588\ub2e4\uace0 \ubcf4\ub3c4\ud55c \ubc14 \uc788\ub2e4. \uc911\uad6d \uc5b8\ub860\uc778 \uad00\uc57c\uc624(\u7ba1\u59da)\ub294 \uc774 \ubc29\uc1a1\uc5d0\uc11c \ubd81\ud55c\uc744 \ubc29\ubb38\ud588\ub358 \uc911\uad6d \uc678\uad50\uad00\uc744 \uc5b8\uae09\ud558\uba70 \"\uc6b0\ub9ac \uc678\uad50\uad00\uc774 \ubd81\ud55c\uc744 \ubc29\ubb38\ud588\uc744 \ub54c \uc0ac\uc801\uc778 \ub300\ud654\uc5d0\uc11c \ubd81\ud55c \uc678\uad50\uad00\uc5d0\uac8c `\uae40\uc815\uc740\uc774 \uc870\ubd80 \uae40\uc77c\uc131\uacfc \uaf2d \ub2ee\uc558\ub2e4`\uace0 \ud558\uc790 `\uc131\ud615\uc218\uc220\uc744 \ud588\ub2e4`\uace0 \ub2f5\ud588\ub2e4\"\uace0 \uc804\ud588\ub2e4. \uc2e0\ud654\ud1b5\uc2e0\uc740 \uc11c\uc6b8 \uc8fc\uc7ac \ud2b9\ud30c\uc6d0\uacfc \ud3c9\uc591 \uc8fc\uc7ac \ud2b9\ud30c\uc6d0\uc744 \ubaa8\ub450 \uc5f0\uacb0\ud574 \ubcf4\ub3c4 \uacbd\uc704\uc640 \uadfc\uac70\ub97c \ucd94\uc801\ud55c \uac83\uc73c\ub85c \uc54c\ub824\uc84c\ub2e4. \ud3c9\uc591 \uc8fc\uc7ac \ud2b9\ud30c\uc6d0\uc740 \"\ubd81\ud55c\uc5d0\uc120 \uae40\uc815\uc740 \uc131\ud615\uacfc \uad00\ub828\ub41c \ubcf4\ub3c4\uac00 \ud55c \ubc88\ub3c4 \uc5c6\uc5c8\ub2e4\"\uba70 \"\uadfc\uac70 \uc5c6\ub294 \uc18c\ubb38\"\uc774\ub77c\uace0 \ub9d0\ud588\ub2e4\uace0 \uc2e0\ud654\ud1b5\uc2e0\uc774 \uc804\ud588\ub2e4.", "paragraph_answer": "\uc911\uad6d \uc120\uc804 \uc704\uc131TV \ub294 2013\ub144 1\uc6d4 19\uc77c \uae40\uc815\uc740 \uc81c1\uc704\uc6d0\uc7a5\uc774 \ud560\uc544\ubc84\uc9c0 \uae40\uc77c\uc131 \uc8fc\uc11d\uacfc \ub2ee\uc544 \ubcf4\uc774\ub3c4\ub85d \uc131\ud615\uc218\uc220\uc744 \ud588\ub2e4\ub294 \uc0ac\uc2e4\uc744 \ud655\uc778\ud588\ub2e4\uace0 \ubcf4\ub3c4\ud55c \ubc14 \uc788\ub2e4. \uc911\uad6d \uc5b8\ub860\uc778 \uad00\uc57c\uc624(\u7ba1\u59da)\ub294 \uc774 \ubc29\uc1a1\uc5d0\uc11c \ubd81\ud55c\uc744 \ubc29\ubb38\ud588\ub358 \uc911\uad6d \uc678\uad50\uad00\uc744 \uc5b8\uae09\ud558\uba70 \"\uc6b0\ub9ac \uc678\uad50\uad00\uc774 \ubd81\ud55c\uc744 \ubc29\ubb38\ud588\uc744 \ub54c \uc0ac\uc801\uc778 \ub300\ud654\uc5d0\uc11c \ubd81\ud55c \uc678\uad50\uad00\uc5d0\uac8c `\uae40\uc815\uc740\uc774 \uc870\ubd80 \uae40\uc77c\uc131\uacfc \uaf2d \ub2ee\uc558\ub2e4`\uace0 \ud558\uc790 `\uc131\ud615\uc218\uc220\uc744 \ud588\ub2e4`\uace0 \ub2f5\ud588\ub2e4\"\uace0 \uc804\ud588\ub2e4. \uc2e0\ud654\ud1b5\uc2e0\uc740 \uc11c\uc6b8 \uc8fc\uc7ac \ud2b9\ud30c\uc6d0\uacfc \ud3c9\uc591 \uc8fc\uc7ac \ud2b9\ud30c\uc6d0\uc744 \ubaa8\ub450 \uc5f0\uacb0\ud574 \ubcf4\ub3c4 \uacbd\uc704\uc640 \uadfc\uac70\ub97c \ucd94\uc801\ud55c \uac83\uc73c\ub85c \uc54c\ub824\uc84c\ub2e4. \ud3c9\uc591 \uc8fc\uc7ac \ud2b9\ud30c\uc6d0\uc740 \"\ubd81\ud55c\uc5d0\uc120 \uae40\uc815\uc740 \uc131\ud615\uacfc \uad00\ub828\ub41c \ubcf4\ub3c4\uac00 \ud55c \ubc88\ub3c4 \uc5c6\uc5c8\ub2e4\"\uba70 \"\uadfc\uac70 \uc5c6\ub294 \uc18c\ubb38\"\uc774\ub77c\uace0 \ub9d0\ud588\ub2e4\uace0 \uc2e0\ud654\ud1b5\uc2e0\uc774 \uc804\ud588\ub2e4.", "sentence_answer": "\uc911\uad6d \uc120\uc804 \uc704\uc131TV \ub294 2013\ub144 1\uc6d4 19\uc77c \uae40\uc815\uc740 \uc81c1\uc704\uc6d0\uc7a5\uc774 \ud560\uc544\ubc84\uc9c0 \uae40\uc77c\uc131 \uc8fc\uc11d\uacfc \ub2ee\uc544 \ubcf4\uc774\ub3c4\ub85d \uc131\ud615\uc218\uc220\uc744 \ud588\ub2e4\ub294 \uc0ac\uc2e4\uc744 \ud655\uc778\ud588\ub2e4\uace0 \ubcf4\ub3c4\ud55c \ubc14 \uc788\ub2e4.", "paragraph_id": "6482362-9-1", "paragraph_question": "question: \uae40\uc815\uc740\uc758 \uc131\ud615 \uc218\uc220\uc5d0 \ub300\ud574 \ubcf4\ub3c4\ud55c \uc911\uad6d\uc758 \ubc29\uc1a1\uc740?, context: \uc911\uad6d \uc120\uc804 \uc704\uc131TV\ub294 2013\ub144 1\uc6d4 19\uc77c \uae40\uc815\uc740 \uc81c1\uc704\uc6d0\uc7a5\uc774 \ud560\uc544\ubc84\uc9c0 \uae40\uc77c\uc131 \uc8fc\uc11d\uacfc \ub2ee\uc544 \ubcf4\uc774\ub3c4\ub85d \uc131\ud615\uc218\uc220\uc744 \ud588\ub2e4\ub294 \uc0ac\uc2e4\uc744 \ud655\uc778\ud588\ub2e4\uace0 \ubcf4\ub3c4\ud55c \ubc14 \uc788\ub2e4. \uc911\uad6d \uc5b8\ub860\uc778 \uad00\uc57c\uc624(\u7ba1\u59da)\ub294 \uc774 \ubc29\uc1a1\uc5d0\uc11c \ubd81\ud55c\uc744 \ubc29\ubb38\ud588\ub358 \uc911\uad6d \uc678\uad50\uad00\uc744 \uc5b8\uae09\ud558\uba70 \"\uc6b0\ub9ac \uc678\uad50\uad00\uc774 \ubd81\ud55c\uc744 \ubc29\ubb38\ud588\uc744 \ub54c \uc0ac\uc801\uc778 \ub300\ud654\uc5d0\uc11c \ubd81\ud55c \uc678\uad50\uad00\uc5d0\uac8c `\uae40\uc815\uc740\uc774 \uc870\ubd80 \uae40\uc77c\uc131\uacfc \uaf2d \ub2ee\uc558\ub2e4`\uace0 \ud558\uc790 `\uc131\ud615\uc218\uc220\uc744 \ud588\ub2e4`\uace0 \ub2f5\ud588\ub2e4\"\uace0 \uc804\ud588\ub2e4. \uc2e0\ud654\ud1b5\uc2e0\uc740 \uc11c\uc6b8 \uc8fc\uc7ac \ud2b9\ud30c\uc6d0\uacfc \ud3c9\uc591 \uc8fc\uc7ac \ud2b9\ud30c\uc6d0\uc744 \ubaa8\ub450 \uc5f0\uacb0\ud574 \ubcf4\ub3c4 \uacbd\uc704\uc640 \uadfc\uac70\ub97c \ucd94\uc801\ud55c \uac83\uc73c\ub85c \uc54c\ub824\uc84c\ub2e4. \ud3c9\uc591 \uc8fc\uc7ac \ud2b9\ud30c\uc6d0\uc740 \"\ubd81\ud55c\uc5d0\uc120 \uae40\uc815\uc740 \uc131\ud615\uacfc \uad00\ub828\ub41c \ubcf4\ub3c4\uac00 \ud55c \ubc88\ub3c4 \uc5c6\uc5c8\ub2e4\"\uba70 \"\uadfc\uac70 \uc5c6\ub294 \uc18c\ubb38\"\uc774\ub77c\uace0 \ub9d0\ud588\ub2e4\uace0 \uc2e0\ud654\ud1b5\uc2e0\uc774 \uc804\ud588\ub2e4."} {"question": "\uc624\uc2a4\ud2b8\ub808\uc77c\ub9ac\uc544\uccad\uac1c\uad6c\ub9ac\uc758 \ud53c\ubd80 \ubd84\ube44\ubb3c\uc5d0\ub294 \ud56d\ubc14\uc774\ub7ec\uc2a4\uc640 \ud56d\ubc15\ud14c\ub9ac\uc544 \uc131\uc9c8\uc774\uc788\ub294 \ud3a8\ud2f0\ub4dc\uac00 \uba87\uc885 \ud568\uc720\ub418\uc5b4\uc788\ub098?", "paragraph": "\uac1c\uad6c\ub9ac\ub4e4\uc740 \ud5c8\ud30c\ub97c \uac00\uc9c0\uace0 \uc788\uc9c0\ub9cc \ud53c\ubd80\ub97c \ud1b5\ud574 \uc0b0\uc18c\ub97c \ud761\uc218\ud55c\ub2e4. \uc774 \uc0b0\uc18c \ud761\uc218 \uacfc\uc815\uc774 \uc6d0\ud65c\ud788 \uc774\ub8e8\uc5b4\uc9c0\uae30 \uc704\ud574\uc11c\ub294 \ud53c\ubd80\uc758 \uc2b5\ub3c4\uac00 \ucd95\ucd95\ud558\uac8c \uc720\uc9c0\ub418\uc5b4\uc57c \ud55c\ub2e4. \ucd95\ucd95\ud55c \ud53c\ubd80\uc758 \uac00\uc7a5 \ud070 \ub2e8\uc810\uc740 \uacf0\ud321\uc774\ub098 \ubc15\ud14c\ub9ac\uc544 \uac19\uc740 \ubcd1\uc6d0\uccb4\uac00 \uadf8 \uc2b5\uae30 \uc18d\uc5d0 \ubc88\uc2dd\ud558\uc5ec \ubcd1\uc6d0\uccb4 \uac10\uc5fc\uc758 \uac00\ub2a5\uc131\uc774 \ub192\uc544\uc9c4\ub2e4\ub294 \uc810\uc774\ub2e4. \uc774\uc5d0 \ub300\uc751\ud558\uc5ec \uac1c\uad6c\ub9ac\ub4e4\uc740 \uc774\ub7ec\ud55c \ubcd1\uc6d0\uccb4\ub97c \ud30c\uad34\ud560 \uc218 \uc788\ub294 \ud3a9\ud2f0\ub4dc\ub97c \ubd84\ube44\ud55c\ub2e4. \uc624\uc2a4\ud2b8\ub808\uc774\ub9ac\uc544\uccad\uac1c\uad6c\ub9ac\uc758 \ud53c\ubd80 \ubd84\ube44\ubb3c\uc5d0\ub294 \ud56d\ubc15\ud14c\ub9ac\uc544 \ubc0f \ud56d\ubc14\uc774\ub7ec\uc2a4 \uc131\uc9c8\uc774 \uc788\ub294 \ud3a9\ud2f0\ub4dc 25\uc885\uc774 \ud568\uc720\ub418\uc5b4 \uc788\ub2e4. \ub610\ud55c \uc774 \ubd84\ube44\ubb3c\uc5d0\ub294 \uc778\uac04 \uc2ed\uc774\uc9c0\uc7a5 \uc810\ub9c9\uc5d0\uc11c \ubd84\ube44\ub418\ub294 \uc18c\ud654 \ud638\ub974\ubaac\uc778 \ucf5c\ub808\ud0a4\uc2a4\ud1a0\ud0a4\ub2cc\uacfc \uac70\uc758 \uac19\uc740 \uc0dd\ub9ac\uc801 \uc791\uc6a9\uc744 \ud558\ub294 \uc138\ub8f0\ub808\uc778(cerulein)\ub3c4 \ud568\uc720\ub418\uc5b4 \uc788\ub2e4. \uc774 \uc138\ub8f0\ub808\uc778\uc740 \ub2e4\uc591\ud55c \uc758\ub8cc \ubaa9\uc801\uc73c\ub85c \uc751\uc6a9\ub418\uc5b4 \uc0ac\uc6a9\ub418\uace0 \uc788\ub2e4. \uc624\uc2a4\ud2b8\ub808\uc77c\ub9ac\uc544\uccad\uac1c\uad6c\ub9ac\uc758 \ud53c\ubd80 \ubd84\ube44\ubb3c\uc5d0 \ud568\uc720\ub41c \uc5ec\ub7ec \ud3a9\ud2f0\ub4dc\uac00 \uac74\uac15\ud55c T \uc138\ud3ec\ub294 \uc190\uc0c1\uc2dc\ud0a4\uc9c0 \uc54a\uace0 HIV\ub97c \ud30c\uad34\ud560 \uc218 \uc788\ub2e4\ub294 \uc810\uc774 \ubc1d\ud600\uc838 \uc788\ub2e4.", "answer": "25\uc885", "sentence": "\uc624\uc2a4\ud2b8\ub808\uc774\ub9ac\uc544\uccad\uac1c\uad6c\ub9ac\uc758 \ud53c\ubd80 \ubd84\ube44\ubb3c\uc5d0\ub294 \ud56d\ubc15\ud14c\ub9ac\uc544 \ubc0f \ud56d\ubc14\uc774\ub7ec\uc2a4 \uc131\uc9c8\uc774 \uc788\ub294 \ud3a9\ud2f0\ub4dc 25\uc885 \uc774 \ud568\uc720\ub418\uc5b4 \uc788\ub2e4.", "paragraph_sentence": "\uac1c\uad6c\ub9ac\ub4e4\uc740 \ud5c8\ud30c\ub97c \uac00\uc9c0\uace0 \uc788\uc9c0\ub9cc \ud53c\ubd80\ub97c \ud1b5\ud574 \uc0b0\uc18c\ub97c \ud761\uc218\ud55c\ub2e4. \uc774 \uc0b0\uc18c \ud761\uc218 \uacfc\uc815\uc774 \uc6d0\ud65c\ud788 \uc774\ub8e8\uc5b4\uc9c0\uae30 \uc704\ud574\uc11c\ub294 \ud53c\ubd80\uc758 \uc2b5\ub3c4\uac00 \ucd95\ucd95\ud558\uac8c \uc720\uc9c0\ub418\uc5b4\uc57c \ud55c\ub2e4. \ucd95\ucd95\ud55c \ud53c\ubd80\uc758 \uac00\uc7a5 \ud070 \ub2e8\uc810\uc740 \uacf0\ud321\uc774\ub098 \ubc15\ud14c\ub9ac\uc544 \uac19\uc740 \ubcd1\uc6d0\uccb4\uac00 \uadf8 \uc2b5\uae30 \uc18d\uc5d0 \ubc88\uc2dd\ud558\uc5ec \ubcd1\uc6d0\uccb4 \uac10\uc5fc\uc758 \uac00\ub2a5\uc131\uc774 \ub192\uc544\uc9c4\ub2e4\ub294 \uc810\uc774\ub2e4. \uc774\uc5d0 \ub300\uc751\ud558\uc5ec \uac1c\uad6c\ub9ac\ub4e4\uc740 \uc774\ub7ec\ud55c \ubcd1\uc6d0\uccb4\ub97c \ud30c\uad34\ud560 \uc218 \uc788\ub294 \ud3a9\ud2f0\ub4dc\ub97c \ubd84\ube44\ud55c\ub2e4. \uc624\uc2a4\ud2b8\ub808\uc774\ub9ac\uc544\uccad\uac1c\uad6c\ub9ac\uc758 \ud53c\ubd80 \ubd84\ube44\ubb3c\uc5d0\ub294 \ud56d\ubc15\ud14c\ub9ac\uc544 \ubc0f \ud56d\ubc14\uc774\ub7ec\uc2a4 \uc131\uc9c8\uc774 \uc788\ub294 \ud3a9\ud2f0\ub4dc 25\uc885 \uc774 \ud568\uc720\ub418\uc5b4 \uc788\ub2e4. \ub610\ud55c \uc774 \ubd84\ube44\ubb3c\uc5d0\ub294 \uc778\uac04 \uc2ed\uc774\uc9c0\uc7a5 \uc810\ub9c9\uc5d0\uc11c \ubd84\ube44\ub418\ub294 \uc18c\ud654 \ud638\ub974\ubaac\uc778 \ucf5c\ub808\ud0a4\uc2a4\ud1a0\ud0a4\ub2cc\uacfc \uac70\uc758 \uac19\uc740 \uc0dd\ub9ac\uc801 \uc791\uc6a9\uc744 \ud558\ub294 \uc138\ub8f0\ub808\uc778(cerulein)\ub3c4 \ud568\uc720\ub418\uc5b4 \uc788\ub2e4. \uc774 \uc138\ub8f0\ub808\uc778\uc740 \ub2e4\uc591\ud55c \uc758\ub8cc \ubaa9\uc801\uc73c\ub85c \uc751\uc6a9\ub418\uc5b4 \uc0ac\uc6a9\ub418\uace0 \uc788\ub2e4. \uc624\uc2a4\ud2b8\ub808\uc77c\ub9ac\uc544\uccad\uac1c\uad6c\ub9ac\uc758 \ud53c\ubd80 \ubd84\ube44\ubb3c\uc5d0 \ud568\uc720\ub41c \uc5ec\ub7ec \ud3a9\ud2f0\ub4dc\uac00 \uac74\uac15\ud55c T \uc138\ud3ec\ub294 \uc190\uc0c1\uc2dc\ud0a4\uc9c0 \uc54a\uace0 HIV\ub97c \ud30c\uad34\ud560 \uc218 \uc788\ub2e4\ub294 \uc810\uc774 \ubc1d\ud600\uc838 \uc788\ub2e4.", "paragraph_answer": "\uac1c\uad6c\ub9ac\ub4e4\uc740 \ud5c8\ud30c\ub97c \uac00\uc9c0\uace0 \uc788\uc9c0\ub9cc \ud53c\ubd80\ub97c \ud1b5\ud574 \uc0b0\uc18c\ub97c \ud761\uc218\ud55c\ub2e4. \uc774 \uc0b0\uc18c \ud761\uc218 \uacfc\uc815\uc774 \uc6d0\ud65c\ud788 \uc774\ub8e8\uc5b4\uc9c0\uae30 \uc704\ud574\uc11c\ub294 \ud53c\ubd80\uc758 \uc2b5\ub3c4\uac00 \ucd95\ucd95\ud558\uac8c \uc720\uc9c0\ub418\uc5b4\uc57c \ud55c\ub2e4. \ucd95\ucd95\ud55c \ud53c\ubd80\uc758 \uac00\uc7a5 \ud070 \ub2e8\uc810\uc740 \uacf0\ud321\uc774\ub098 \ubc15\ud14c\ub9ac\uc544 \uac19\uc740 \ubcd1\uc6d0\uccb4\uac00 \uadf8 \uc2b5\uae30 \uc18d\uc5d0 \ubc88\uc2dd\ud558\uc5ec \ubcd1\uc6d0\uccb4 \uac10\uc5fc\uc758 \uac00\ub2a5\uc131\uc774 \ub192\uc544\uc9c4\ub2e4\ub294 \uc810\uc774\ub2e4. \uc774\uc5d0 \ub300\uc751\ud558\uc5ec \uac1c\uad6c\ub9ac\ub4e4\uc740 \uc774\ub7ec\ud55c \ubcd1\uc6d0\uccb4\ub97c \ud30c\uad34\ud560 \uc218 \uc788\ub294 \ud3a9\ud2f0\ub4dc\ub97c \ubd84\ube44\ud55c\ub2e4. \uc624\uc2a4\ud2b8\ub808\uc774\ub9ac\uc544\uccad\uac1c\uad6c\ub9ac\uc758 \ud53c\ubd80 \ubd84\ube44\ubb3c\uc5d0\ub294 \ud56d\ubc15\ud14c\ub9ac\uc544 \ubc0f \ud56d\ubc14\uc774\ub7ec\uc2a4 \uc131\uc9c8\uc774 \uc788\ub294 \ud3a9\ud2f0\ub4dc 25\uc885 \uc774 \ud568\uc720\ub418\uc5b4 \uc788\ub2e4. \ub610\ud55c \uc774 \ubd84\ube44\ubb3c\uc5d0\ub294 \uc778\uac04 \uc2ed\uc774\uc9c0\uc7a5 \uc810\ub9c9\uc5d0\uc11c \ubd84\ube44\ub418\ub294 \uc18c\ud654 \ud638\ub974\ubaac\uc778 \ucf5c\ub808\ud0a4\uc2a4\ud1a0\ud0a4\ub2cc\uacfc \uac70\uc758 \uac19\uc740 \uc0dd\ub9ac\uc801 \uc791\uc6a9\uc744 \ud558\ub294 \uc138\ub8f0\ub808\uc778(cerulein)\ub3c4 \ud568\uc720\ub418\uc5b4 \uc788\ub2e4. \uc774 \uc138\ub8f0\ub808\uc778\uc740 \ub2e4\uc591\ud55c \uc758\ub8cc \ubaa9\uc801\uc73c\ub85c \uc751\uc6a9\ub418\uc5b4 \uc0ac\uc6a9\ub418\uace0 \uc788\ub2e4. \uc624\uc2a4\ud2b8\ub808\uc77c\ub9ac\uc544\uccad\uac1c\uad6c\ub9ac\uc758 \ud53c\ubd80 \ubd84\ube44\ubb3c\uc5d0 \ud568\uc720\ub41c \uc5ec\ub7ec \ud3a9\ud2f0\ub4dc\uac00 \uac74\uac15\ud55c T \uc138\ud3ec\ub294 \uc190\uc0c1\uc2dc\ud0a4\uc9c0 \uc54a\uace0 HIV\ub97c \ud30c\uad34\ud560 \uc218 \uc788\ub2e4\ub294 \uc810\uc774 \ubc1d\ud600\uc838 \uc788\ub2e4.", "sentence_answer": "\uc624\uc2a4\ud2b8\ub808\uc774\ub9ac\uc544\uccad\uac1c\uad6c\ub9ac\uc758 \ud53c\ubd80 \ubd84\ube44\ubb3c\uc5d0\ub294 \ud56d\ubc15\ud14c\ub9ac\uc544 \ubc0f \ud56d\ubc14\uc774\ub7ec\uc2a4 \uc131\uc9c8\uc774 \uc788\ub294 \ud3a9\ud2f0\ub4dc 25\uc885 \uc774 \ud568\uc720\ub418\uc5b4 \uc788\ub2e4.", "paragraph_id": "6524997-6-1", "paragraph_question": "question: \uc624\uc2a4\ud2b8\ub808\uc77c\ub9ac\uc544\uccad\uac1c\uad6c\ub9ac\uc758 \ud53c\ubd80 \ubd84\ube44\ubb3c\uc5d0\ub294 \ud56d\ubc14\uc774\ub7ec\uc2a4\uc640 \ud56d\ubc15\ud14c\ub9ac\uc544 \uc131\uc9c8\uc774\uc788\ub294 \ud3a8\ud2f0\ub4dc\uac00 \uba87\uc885 \ud568\uc720\ub418\uc5b4\uc788\ub098?, context: \uac1c\uad6c\ub9ac\ub4e4\uc740 \ud5c8\ud30c\ub97c \uac00\uc9c0\uace0 \uc788\uc9c0\ub9cc \ud53c\ubd80\ub97c \ud1b5\ud574 \uc0b0\uc18c\ub97c \ud761\uc218\ud55c\ub2e4. \uc774 \uc0b0\uc18c \ud761\uc218 \uacfc\uc815\uc774 \uc6d0\ud65c\ud788 \uc774\ub8e8\uc5b4\uc9c0\uae30 \uc704\ud574\uc11c\ub294 \ud53c\ubd80\uc758 \uc2b5\ub3c4\uac00 \ucd95\ucd95\ud558\uac8c \uc720\uc9c0\ub418\uc5b4\uc57c \ud55c\ub2e4. \ucd95\ucd95\ud55c \ud53c\ubd80\uc758 \uac00\uc7a5 \ud070 \ub2e8\uc810\uc740 \uacf0\ud321\uc774\ub098 \ubc15\ud14c\ub9ac\uc544 \uac19\uc740 \ubcd1\uc6d0\uccb4\uac00 \uadf8 \uc2b5\uae30 \uc18d\uc5d0 \ubc88\uc2dd\ud558\uc5ec \ubcd1\uc6d0\uccb4 \uac10\uc5fc\uc758 \uac00\ub2a5\uc131\uc774 \ub192\uc544\uc9c4\ub2e4\ub294 \uc810\uc774\ub2e4. \uc774\uc5d0 \ub300\uc751\ud558\uc5ec \uac1c\uad6c\ub9ac\ub4e4\uc740 \uc774\ub7ec\ud55c \ubcd1\uc6d0\uccb4\ub97c \ud30c\uad34\ud560 \uc218 \uc788\ub294 \ud3a9\ud2f0\ub4dc\ub97c \ubd84\ube44\ud55c\ub2e4. \uc624\uc2a4\ud2b8\ub808\uc774\ub9ac\uc544\uccad\uac1c\uad6c\ub9ac\uc758 \ud53c\ubd80 \ubd84\ube44\ubb3c\uc5d0\ub294 \ud56d\ubc15\ud14c\ub9ac\uc544 \ubc0f \ud56d\ubc14\uc774\ub7ec\uc2a4 \uc131\uc9c8\uc774 \uc788\ub294 \ud3a9\ud2f0\ub4dc 25\uc885\uc774 \ud568\uc720\ub418\uc5b4 \uc788\ub2e4. \ub610\ud55c \uc774 \ubd84\ube44\ubb3c\uc5d0\ub294 \uc778\uac04 \uc2ed\uc774\uc9c0\uc7a5 \uc810\ub9c9\uc5d0\uc11c \ubd84\ube44\ub418\ub294 \uc18c\ud654 \ud638\ub974\ubaac\uc778 \ucf5c\ub808\ud0a4\uc2a4\ud1a0\ud0a4\ub2cc\uacfc \uac70\uc758 \uac19\uc740 \uc0dd\ub9ac\uc801 \uc791\uc6a9\uc744 \ud558\ub294 \uc138\ub8f0\ub808\uc778(cerulein)\ub3c4 \ud568\uc720\ub418\uc5b4 \uc788\ub2e4. \uc774 \uc138\ub8f0\ub808\uc778\uc740 \ub2e4\uc591\ud55c \uc758\ub8cc \ubaa9\uc801\uc73c\ub85c \uc751\uc6a9\ub418\uc5b4 \uc0ac\uc6a9\ub418\uace0 \uc788\ub2e4. \uc624\uc2a4\ud2b8\ub808\uc77c\ub9ac\uc544\uccad\uac1c\uad6c\ub9ac\uc758 \ud53c\ubd80 \ubd84\ube44\ubb3c\uc5d0 \ud568\uc720\ub41c \uc5ec\ub7ec \ud3a9\ud2f0\ub4dc\uac00 \uac74\uac15\ud55c T \uc138\ud3ec\ub294 \uc190\uc0c1\uc2dc\ud0a4\uc9c0 \uc54a\uace0 HIV\ub97c \ud30c\uad34\ud560 \uc218 \uc788\ub2e4\ub294 \uc810\uc774 \ubc1d\ud600\uc838 \uc788\ub2e4."} {"question": "\uc544\ub3d9\uc2ec\ub9ac\ud559\uc744 \ucd5c\ucd08\ub85c \uc2dc\ub3c4\ud55c \uc0ac\ub78c\uc740?", "paragraph": "\uad50\uc721\uc2ec\ub9ac\ud559\uc740 \uad50\uc721\uc744 \ud6a8\uacfc\uc801\uc73c\ub85c \uc774\ub8e9\ud558\uae30 \uc704\ud55c \uc2dd\uacac\uc774\ub098 \uae30\uc220\uc744 \uc81c\uacf5\ud558\ub294 \ud559\ubb38\uc774\ub77c\uace0 \uadf8 \uc131\uaca9\uc744 \uaddc\uc815\ud55c\ub2e4\uba74, '\ub2e8\uacc4\uc2dd \uad50\uc721\uccb4\uacc4'\ub97c \uc81c\ucc3d\ud558\uc600\uace0, \uadf8\ub9bc\uacfc \uc5b8\uc5b4\ub97c \uacb0\ud569\ud55c \ucd5c\ucd08\uc758 \uad50\uacfc\uc11c\uc778 <\uc138\uacc4\ub3c4\ud68c(\u4e16\u754c\u5716\u7e6a)>\uc5d0 \uc758\ud574\uc11c \uc5b4\ub9b0\uc774\uc758 \uc5b8\uc5b4\uad50\uc721 \ubc0f \uc2dc\uccad\uac01\uad50\uc721\uc758 \uae30\ucd08\ub97c \uc81c\uacf5\ud55c \ucf54\uba54\ub2c8\uc6b0\uc2a4(J. A. Comenuis), <\uc5d0\ubc00>\uc758 \uc800\uc790 \ub8e8\uc18c(J. J.Rousseau),\ud398\uc2a4\ud0c8\ub85c\uce58(J. H. Pestalozzi) \ub4f1\uc744 \uc774 \ubd84\uc57c\uc758 \uc120\uad6c\uc801\uc778 \uc778\ubb3c\ub85c \uc81c\uc2dc\ud560 \uc218 \uc788\uc73c\ub098, \uad50\uc721\uc758 \ubc29\ubc95\uc5d0\uc11c \uc694\uad6c\ub418\ub294 \uc5ec\ub7ec \uac00\uc9c0 \uac1c\ub150\uc744 \uc2ec\ub9ac\ud559\uc801\uc73c\ub85c \ucde8\uae09\ud55c \uac83\uc740 \ud5e4\ub974\ubc14\ub974\ud2b8(J. F. Herbart)\uc5d0 \uc774\ub974\ub7ec\uc11c \ucd5c\ucd08\uc758 \uc2dc\ub3c4\uac00 \uc774\ub8e8\uc5b4\uc84c\ub2e4\uace0 \ud560 \uc218 \uc788\ub2e4. \uadf8\ub7ec\ub098 \uc2ec\ub9ac\uacfc\ud559\uc801\uc778 \uc785\uc7a5\uc5d0\uc11c \uad50\uc721\uc2ec\ub9ac\ud559\uc758 \uc6d0\ud615\uc744 \uc774\ub8e9\ud55c \uac83\uc740 \uc5ed\uc2dc \ud604\ub300\uc2ec\ub9ac\ud559\uc758 \uc2dc\uc870\uc778 \ubd84\ud2b8(W. Wundt)\uc774\ub2e4. \uad50\uc721\uc2ec\ub9ac\ud559\uc740 \uc544\ub3d9\uc2ec\ub9ac\ud559\uc758 \ubc1c\ub2ec\uc5d0 \ud798\uc744 \uc785\uc740 \ubc14 \uc801\uc9c0 \uc54a\ub2e4. \uc544\ub3d9\uc758 \uc815\uc2e0\ubc1c\ub2ec\uc744 \uae30\uc220\ud558\uace0, \uac70\uae30\uc5d0\uc11c \ubc1c\uacac\ub418\ub294 \ubc1c\ub2ec\uc6d0\ub9ac\ub97c \ucd94\uad6c\ud558\ub294 \uc0c8\ub85c\uc6b4 \uc2ec\ub9ac\ud559\uc758 \ubd84\uc57c\uc778 \uc544\ub3d9\uc2ec\ub9ac\ud559\uc740 \ubbf8\uad6d\uc758 \ud640(S. Hall)\uc5d0 \uc758\ud574 \ucd5c\ucd08\ub85c \uc2dc\ub3c4\ub418\uc5c8\ub2e4. \uadf8\ub294 \uc0dd\ubb3c\ud559\uc801\u00b7\uc778\ub958\ud559\uc801\uc778 \uc601\uc5ed\uc744 \ud3ec\ud568\ud558\ub294 \ubc1c\uc0dd\uc2ec\ub9ac\ud559\uc801\uc778 \uc785\uc7a5\uc5d0\uc11c <\uc544\ub3d9\uc758 \ub3c4\ub355\uc801\u00b7\uc885\uad50\uc801 \uad50\uc721>(1883) \ub4f1\uc744 \ubc1c\ud45c\ud558\uc600\ub2e4. \uc774\uc640 \uac19\uc740 \uc544\ub3d9\uc2ec\ub9ac\ud559\uc5d0 \ub300\ud55c \uad00\uc2ec\uacfc \ud559\ubb38\uc801\uc778 \ud65c\ub3d9\uc740 \uad50\uc721\uc2ec\ub9ac\ud559\uc758 \uae30\ucd08\ub97c \uc774\ub8e8\uac8c \ub418\uc5c8\ub2e4. \uc774\uc758 \uc77c\ud658\uc73c\ub85c \ub3c5\uc77c\uc758 \ubaa8\uc774\ub9cc(E. Meumann)\uc740 \uc2ec\ub9ac\ud559\uacfc \uad50\uc721\ud559\uc758 \uacb0\ud569\uc744 \uc2dc\ub3c4\ud558\ub294 \uc5f0\uad6c\uc81c\ub3c4\ub85c\uc11c, \u2460 \uc544\ub3d9\uc758 \uc815\uc2e0\uc801\u00b7\uc2e0\uccb4\uc801 \ubc1c\ub2ec, \u2461 \uc544\ub3d9 \uac1c\uc778\uc758 \uc815\uc2e0\ub2a5\ub825\uc758 \ubc1c\ub2ec, \u2462 \uc544\ub3d9\uc758 \uac1c\uc131, \u2463 \ucc9c\ubd80\uc758 \uac1c\uc778\ucc28, \u2464 \uc544\ub3d9\uc758 \ud559\uad50\uc0dd\ud65c, \u2465 \uac01 \uad50\uacfc\uc5d0 \uc788\uc5b4\uc11c\uc758 \ud559\uc2b5\ud65c\ub3d9, \u2466 \uad50\uc0ac\uc758 \ud65c\ub3d9\u00b7\ud559\uc2b5\uc9c0\ub3c4\u00b7\ud559\uad50\uc81c\ub3c4\u00b7\uad50\uc218\ubc29\ubc95 \ub4f1\uc73c\ub85c \ubd84\ub958\ud558\uc5ec \uc81c\uc2dc\ub97c \ud558\uc600\uc73c\uba70, \uc774\uac83\uc740 \uad50\uc721\uc2ec\ub9ac\ud559\uc758 \ud559\ubb38\uc801 \uace8\uaca9\uc744 \uc774\ub8e8\uac8c \ub41c \uac83\uc774\ub2e4. \uad50\uc721\uc2ec\ub9ac\ud559\uc758 \ub610 \ud558\ub098\uc758 \uc120\uad6c\uc801 \uc5c5\uc801\uc740 \uac1c\uc778\ucc28\uc5d0 \uad00\ud55c \uc5f0\uad6c\uc774\ub2e4. \uc774\ub294 \uad6c\ubbf8\uc81c\uad6d\uc5d0\uc11c 19\uc138\uae30 \ud6c4\ubc18\uc5d0\uc11c 20\uc138\uae30\uc5d0 \uac78\uccd0 \uc774\ub8e8\uc5b4\uc84c\ub294\ub370, \uac1c\uc778\ucc28\uc5d0 \uad00\ud55c \uc5f0\uad6c\uc758 \uc120\uad6c\uc790\ub294 \uace8\ud134(F. Galton)\uc73c\ub85c <\uc720\uc804\uc801 \ucc9c\uc7ac>(1869) \ubc0f <\uc778\uac04\ub2a5\ub825\uc758 \uc5f0\uad6c>(1883) \ub4f1\uc744 \ubc1c\ud45c\ud588\uace0, \ubbf8\uad6d\uc758 \uce74\ub378(G. M. Cattell)\uc740 \uac1d\uad00\uc801 \ubc29\ubc95\uc5d0 \uc758\ud574\uc11c \uc778\uac04\uc758 \ub2a5\ub825\uc744 \uce21\uc815\ud558\uc5ec \uc774\uac83\uc744 \uc2e4\uc81c\uba74\uc5d0 \uc801\uc6a9\ud558\ub294 \ub370 \ub178\ub825\ud558\uc600\uc73c\uba70, \ud2b9\ud788 1893\ub144 \uc774\ud6c4 \ub9e4\ub144 \uce7c\ub7fc\ube44\uc544 \ub300\ud559\uc758 \uc2e0\uc785\uc0dd\ub4e4\uc5d0\uac8c \uc2e4\uc2dc\ud55c \ud14c\uc2a4\ud2b8\ub294 \uba58\ud0c8 \ud14c\uc2a4\ud2b8(mental test)\uc758 \uc2dc\ucd08\uac00 \ub418\uc5c8\ub2e4. \uadf8\ub7ec\ub098 \uace8\ud134\uacfc \uce74\ud154\uc758 \uac1c\uc778\ucc28 \uce21\uc815\uc740 \uc9c0\ub2a5\uac80\uc0ac\ub85c\ub294 \uacb0\uc2e4\uc744 \ubcf4\uc9c0 \ubabb\ud558\uace0, \ud504\ub791\uc2a4\uc758 \ube44\ub124(A. Binet)\uc5d0 \uc758\ud558\uc5ec 1905\ub144\uc5d0 \ud30c\ub9ac \ucd08\ub4f1\ud559\uad50 \uc5b4\ub9b0\uc774\uc758 \uc800\ub2a5\uc544 \uac10\ubcc4\uc774\ub77c\ub294 \ub2f9\uc2dc\uc758 \uad50\uc721\uacc4\uc758 \uc694\uccad\uc5d0 \ubd80\uc751\ud574\uc11c \uc774\ub8e8\uc5b4\uc84c\ub2e4. \ubfd0\ub9cc \uc544\ub2c8\ub77c \uc720\uc544\uc5d0 \uad00\ud55c \uc5f0\uad6c\ub294 19\uc138\uae30\uc5d0\uc11c\ubd80\ud130 20\uc138\uae30\ucd08\uae4c\uc9c0 \ubc1c\ub2ec\ud558\uc5ec \uc654\ub294\ub370, \uc720\uce58\uc6d0 \uad50\uc721\uc758 \uae30\ucd08\ub294 \ud504\ub8b0\ubca8(F. Fr bel)\uc5d0 \uc758\ud574 \ud655\ub9bd\ub418\uc5c8\uace0, \uc544\ub3d9\uc2ec\ub9ac\ud559\uc758 \uc2dc\uc870\ub77c\uace0 \ubd88\ub9ac\ub294 \ub3c5\uc77c\uc758 \uc0dd\ub9ac\ud559\uc790 \ud504\ub77c\uc774\uc5b4(W. Preyer) \uc774\ud6c4 \uc544\ub3d9\uc5d0 \uad00\ud55c \ub9ce\uc740 \uc5f0\uad6c\uac00 \uc774\ub8e8\uc5b4\uc84c\ub2e4. \uc774\uc0c1\uc5d0\uc11c \ubcf8 \ubc14\uc640 \uac19\uc774 \uad50\uc721\uc2ec\ub9ac\ud559\uc758 \uc131\ub9bd\ub3d9\uae30\ub294 \ub2e8\uc21c\ud788 \uc2ec\ub9ac\ud559\uacfc \uad50\uc721\uc774 \uc811\ucd09\ud558\ub294 \uc810\uc5d0\uc11c \ubc1c\uc0dd\ud55c \uac83\uc774 \uc544\ub2c8\ub2e4.", "answer": "\ud640", "sentence": "\ubbf8\uad6d\uc758 \ud640 (S. Hall)", "paragraph_sentence": "\uad50\uc721\uc2ec\ub9ac\ud559\uc740 \uad50\uc721\uc744 \ud6a8\uacfc\uc801\uc73c\ub85c \uc774\ub8e9\ud558\uae30 \uc704\ud55c \uc2dd\uacac\uc774\ub098 \uae30\uc220\uc744 \uc81c\uacf5\ud558\ub294 \ud559\ubb38\uc774\ub77c\uace0 \uadf8 \uc131\uaca9\uc744 \uaddc\uc815\ud55c\ub2e4\uba74, '\ub2e8\uacc4\uc2dd \uad50\uc721\uccb4\uacc4'\ub97c \uc81c\ucc3d\ud558\uc600\uace0, \uadf8\ub9bc\uacfc \uc5b8\uc5b4\ub97c \uacb0\ud569\ud55c \ucd5c\ucd08\uc758 \uad50\uacfc\uc11c\uc778 <\uc138\uacc4\ub3c4\ud68c(\u4e16\u754c\u5716\u7e6a)>\uc5d0 \uc758\ud574\uc11c \uc5b4\ub9b0\uc774\uc758 \uc5b8\uc5b4\uad50\uc721 \ubc0f \uc2dc\uccad\uac01\uad50\uc721\uc758 \uae30\ucd08\ub97c \uc81c\uacf5\ud55c \ucf54\uba54\ub2c8\uc6b0\uc2a4(J. A. Comenuis), <\uc5d0\ubc00>\uc758 \uc800\uc790 \ub8e8\uc18c(J. J.Rousseau),\ud398\uc2a4\ud0c8\ub85c\uce58(J. H. Pestalozzi) \ub4f1\uc744 \uc774 \ubd84\uc57c\uc758 \uc120\uad6c\uc801\uc778 \uc778\ubb3c\ub85c \uc81c\uc2dc\ud560 \uc218 \uc788\uc73c\ub098, \uad50\uc721\uc758 \ubc29\ubc95\uc5d0\uc11c \uc694\uad6c\ub418\ub294 \uc5ec\ub7ec \uac00\uc9c0 \uac1c\ub150\uc744 \uc2ec\ub9ac\ud559\uc801\uc73c\ub85c \ucde8\uae09\ud55c \uac83\uc740 \ud5e4\ub974\ubc14\ub974\ud2b8(J. F. Herbart)\uc5d0 \uc774\ub974\ub7ec\uc11c \ucd5c\ucd08\uc758 \uc2dc\ub3c4\uac00 \uc774\ub8e8\uc5b4\uc84c\ub2e4\uace0 \ud560 \uc218 \uc788\ub2e4. \uadf8\ub7ec\ub098 \uc2ec\ub9ac\uacfc\ud559\uc801\uc778 \uc785\uc7a5\uc5d0\uc11c \uad50\uc721\uc2ec\ub9ac\ud559\uc758 \uc6d0\ud615\uc744 \uc774\ub8e9\ud55c \uac83\uc740 \uc5ed\uc2dc \ud604\ub300\uc2ec\ub9ac\ud559\uc758 \uc2dc\uc870\uc778 \ubd84\ud2b8(W. Wundt)\uc774\ub2e4. \uad50\uc721\uc2ec\ub9ac\ud559\uc740 \uc544\ub3d9\uc2ec\ub9ac\ud559\uc758 \ubc1c\ub2ec\uc5d0 \ud798\uc744 \uc785\uc740 \ubc14 \uc801\uc9c0 \uc54a\ub2e4. \uc544\ub3d9\uc758 \uc815\uc2e0\ubc1c\ub2ec\uc744 \uae30\uc220\ud558\uace0, \uac70\uae30\uc5d0\uc11c \ubc1c\uacac\ub418\ub294 \ubc1c\ub2ec\uc6d0\ub9ac\ub97c \ucd94\uad6c\ud558\ub294 \uc0c8\ub85c\uc6b4 \uc2ec\ub9ac\ud559\uc758 \ubd84\uc57c\uc778 \uc544\ub3d9\uc2ec\ub9ac\ud559\uc740 \ubbf8\uad6d\uc758 \ud640 (S. Hall) \uc5d0 \uc758\ud574 \ucd5c\ucd08\ub85c \uc2dc\ub3c4\ub418\uc5c8\ub2e4. \uadf8\ub294 \uc0dd\ubb3c\ud559\uc801\u00b7\uc778\ub958\ud559\uc801\uc778 \uc601\uc5ed\uc744 \ud3ec\ud568\ud558\ub294 \ubc1c\uc0dd\uc2ec\ub9ac\ud559\uc801\uc778 \uc785\uc7a5\uc5d0\uc11c <\uc544\ub3d9\uc758 \ub3c4\ub355\uc801\u00b7\uc885\uad50\uc801 \uad50\uc721>(1883) \ub4f1\uc744 \ubc1c\ud45c\ud558\uc600\ub2e4. \uc774\uc640 \uac19\uc740 \uc544\ub3d9\uc2ec\ub9ac\ud559\uc5d0 \ub300\ud55c \uad00\uc2ec\uacfc \ud559\ubb38\uc801\uc778 \ud65c\ub3d9\uc740 \uad50\uc721\uc2ec\ub9ac\ud559\uc758 \uae30\ucd08\ub97c \uc774\ub8e8\uac8c \ub418\uc5c8\ub2e4. \uc774\uc758 \uc77c\ud658\uc73c\ub85c \ub3c5\uc77c\uc758 \ubaa8\uc774\ub9cc(E. Meumann)\uc740 \uc2ec\ub9ac\ud559\uacfc \uad50\uc721\ud559\uc758 \uacb0\ud569\uc744 \uc2dc\ub3c4\ud558\ub294 \uc5f0\uad6c\uc81c\ub3c4\ub85c\uc11c, \u2460 \uc544\ub3d9\uc758 \uc815\uc2e0\uc801\u00b7\uc2e0\uccb4\uc801 \ubc1c\ub2ec, \u2461 \uc544\ub3d9 \uac1c\uc778\uc758 \uc815\uc2e0\ub2a5\ub825\uc758 \ubc1c\ub2ec, \u2462 \uc544\ub3d9\uc758 \uac1c\uc131, \u2463 \ucc9c\ubd80\uc758 \uac1c\uc778\ucc28, \u2464 \uc544\ub3d9\uc758 \ud559\uad50\uc0dd\ud65c, \u2465 \uac01 \uad50\uacfc\uc5d0 \uc788\uc5b4\uc11c\uc758 \ud559\uc2b5\ud65c\ub3d9, \u2466 \uad50\uc0ac\uc758 \ud65c\ub3d9\u00b7\ud559\uc2b5\uc9c0\ub3c4\u00b7\ud559\uad50\uc81c\ub3c4\u00b7\uad50\uc218\ubc29\ubc95 \ub4f1\uc73c\ub85c \ubd84\ub958\ud558\uc5ec \uc81c\uc2dc\ub97c \ud558\uc600\uc73c\uba70, \uc774\uac83\uc740 \uad50\uc721\uc2ec\ub9ac\ud559\uc758 \ud559\ubb38\uc801 \uace8\uaca9\uc744 \uc774\ub8e8\uac8c \ub41c \uac83\uc774\ub2e4. \uad50\uc721\uc2ec\ub9ac\ud559\uc758 \ub610 \ud558\ub098\uc758 \uc120\uad6c\uc801 \uc5c5\uc801\uc740 \uac1c\uc778\ucc28\uc5d0 \uad00\ud55c \uc5f0\uad6c\uc774\ub2e4. \uc774\ub294 \uad6c\ubbf8\uc81c\uad6d\uc5d0\uc11c 19\uc138\uae30 \ud6c4\ubc18\uc5d0\uc11c 20\uc138\uae30\uc5d0 \uac78\uccd0 \uc774\ub8e8\uc5b4\uc84c\ub294\ub370, \uac1c\uc778\ucc28\uc5d0 \uad00\ud55c \uc5f0\uad6c\uc758 \uc120\uad6c\uc790\ub294 \uace8\ud134(F. Galton)\uc73c\ub85c <\uc720\uc804\uc801 \ucc9c\uc7ac>(1869) \ubc0f <\uc778\uac04\ub2a5\ub825\uc758 \uc5f0\uad6c>(1883) \ub4f1\uc744 \ubc1c\ud45c\ud588\uace0, \ubbf8\uad6d\uc758 \uce74\ub378(G. M. Cattell)\uc740 \uac1d\uad00\uc801 \ubc29\ubc95\uc5d0 \uc758\ud574\uc11c \uc778\uac04\uc758 \ub2a5\ub825\uc744 \uce21\uc815\ud558\uc5ec \uc774\uac83\uc744 \uc2e4\uc81c\uba74\uc5d0 \uc801\uc6a9\ud558\ub294 \ub370 \ub178\ub825\ud558\uc600\uc73c\uba70, \ud2b9\ud788 1893\ub144 \uc774\ud6c4 \ub9e4\ub144 \uce7c\ub7fc\ube44\uc544 \ub300\ud559\uc758 \uc2e0\uc785\uc0dd\ub4e4\uc5d0\uac8c \uc2e4\uc2dc\ud55c \ud14c\uc2a4\ud2b8\ub294 \uba58\ud0c8 \ud14c\uc2a4\ud2b8(mental test)\uc758 \uc2dc\ucd08\uac00 \ub418\uc5c8\ub2e4. \uadf8\ub7ec\ub098 \uace8\ud134\uacfc \uce74\ud154\uc758 \uac1c\uc778\ucc28 \uce21\uc815\uc740 \uc9c0\ub2a5\uac80\uc0ac\ub85c\ub294 \uacb0\uc2e4\uc744 \ubcf4\uc9c0 \ubabb\ud558\uace0, \ud504\ub791\uc2a4\uc758 \ube44\ub124(A. Binet)\uc5d0 \uc758\ud558\uc5ec 1905\ub144\uc5d0 \ud30c\ub9ac \ucd08\ub4f1\ud559\uad50 \uc5b4\ub9b0\uc774\uc758 \uc800\ub2a5\uc544 \uac10\ubcc4\uc774\ub77c\ub294 \ub2f9\uc2dc\uc758 \uad50\uc721\uacc4\uc758 \uc694\uccad\uc5d0 \ubd80\uc751\ud574\uc11c \uc774\ub8e8\uc5b4\uc84c\ub2e4. \ubfd0\ub9cc \uc544\ub2c8\ub77c \uc720\uc544\uc5d0 \uad00\ud55c \uc5f0\uad6c\ub294 19\uc138\uae30\uc5d0\uc11c\ubd80\ud130 20\uc138\uae30\ucd08\uae4c\uc9c0 \ubc1c\ub2ec\ud558\uc5ec \uc654\ub294\ub370, \uc720\uce58\uc6d0 \uad50\uc721\uc758 \uae30\ucd08\ub294 \ud504\ub8b0\ubca8(F. Fr bel)\uc5d0 \uc758\ud574 \ud655\ub9bd\ub418\uc5c8\uace0, \uc544\ub3d9\uc2ec\ub9ac\ud559\uc758 \uc2dc\uc870\ub77c\uace0 \ubd88\ub9ac\ub294 \ub3c5\uc77c\uc758 \uc0dd\ub9ac\ud559\uc790 \ud504\ub77c\uc774\uc5b4(W. Preyer) \uc774\ud6c4 \uc544\ub3d9\uc5d0 \uad00\ud55c \ub9ce\uc740 \uc5f0\uad6c\uac00 \uc774\ub8e8\uc5b4\uc84c\ub2e4. \uc774\uc0c1\uc5d0\uc11c \ubcf8 \ubc14\uc640 \uac19\uc774 \uad50\uc721\uc2ec\ub9ac\ud559\uc758 \uc131\ub9bd\ub3d9\uae30\ub294 \ub2e8\uc21c\ud788 \uc2ec\ub9ac\ud559\uacfc \uad50\uc721\uc774 \uc811\ucd09\ud558\ub294 \uc810\uc5d0\uc11c \ubc1c\uc0dd\ud55c \uac83\uc774 \uc544\ub2c8\ub2e4.", "paragraph_answer": "\uad50\uc721\uc2ec\ub9ac\ud559\uc740 \uad50\uc721\uc744 \ud6a8\uacfc\uc801\uc73c\ub85c \uc774\ub8e9\ud558\uae30 \uc704\ud55c \uc2dd\uacac\uc774\ub098 \uae30\uc220\uc744 \uc81c\uacf5\ud558\ub294 \ud559\ubb38\uc774\ub77c\uace0 \uadf8 \uc131\uaca9\uc744 \uaddc\uc815\ud55c\ub2e4\uba74, '\ub2e8\uacc4\uc2dd \uad50\uc721\uccb4\uacc4'\ub97c \uc81c\ucc3d\ud558\uc600\uace0, \uadf8\ub9bc\uacfc \uc5b8\uc5b4\ub97c \uacb0\ud569\ud55c \ucd5c\ucd08\uc758 \uad50\uacfc\uc11c\uc778 <\uc138\uacc4\ub3c4\ud68c(\u4e16\u754c\u5716\u7e6a)>\uc5d0 \uc758\ud574\uc11c \uc5b4\ub9b0\uc774\uc758 \uc5b8\uc5b4\uad50\uc721 \ubc0f \uc2dc\uccad\uac01\uad50\uc721\uc758 \uae30\ucd08\ub97c \uc81c\uacf5\ud55c \ucf54\uba54\ub2c8\uc6b0\uc2a4(J. A. Comenuis), <\uc5d0\ubc00>\uc758 \uc800\uc790 \ub8e8\uc18c(J. J.Rousseau),\ud398\uc2a4\ud0c8\ub85c\uce58(J. H. Pestalozzi) \ub4f1\uc744 \uc774 \ubd84\uc57c\uc758 \uc120\uad6c\uc801\uc778 \uc778\ubb3c\ub85c \uc81c\uc2dc\ud560 \uc218 \uc788\uc73c\ub098, \uad50\uc721\uc758 \ubc29\ubc95\uc5d0\uc11c \uc694\uad6c\ub418\ub294 \uc5ec\ub7ec \uac00\uc9c0 \uac1c\ub150\uc744 \uc2ec\ub9ac\ud559\uc801\uc73c\ub85c \ucde8\uae09\ud55c \uac83\uc740 \ud5e4\ub974\ubc14\ub974\ud2b8(J. F. Herbart)\uc5d0 \uc774\ub974\ub7ec\uc11c \ucd5c\ucd08\uc758 \uc2dc\ub3c4\uac00 \uc774\ub8e8\uc5b4\uc84c\ub2e4\uace0 \ud560 \uc218 \uc788\ub2e4. \uadf8\ub7ec\ub098 \uc2ec\ub9ac\uacfc\ud559\uc801\uc778 \uc785\uc7a5\uc5d0\uc11c \uad50\uc721\uc2ec\ub9ac\ud559\uc758 \uc6d0\ud615\uc744 \uc774\ub8e9\ud55c \uac83\uc740 \uc5ed\uc2dc \ud604\ub300\uc2ec\ub9ac\ud559\uc758 \uc2dc\uc870\uc778 \ubd84\ud2b8(W. Wundt)\uc774\ub2e4. \uad50\uc721\uc2ec\ub9ac\ud559\uc740 \uc544\ub3d9\uc2ec\ub9ac\ud559\uc758 \ubc1c\ub2ec\uc5d0 \ud798\uc744 \uc785\uc740 \ubc14 \uc801\uc9c0 \uc54a\ub2e4. \uc544\ub3d9\uc758 \uc815\uc2e0\ubc1c\ub2ec\uc744 \uae30\uc220\ud558\uace0, \uac70\uae30\uc5d0\uc11c \ubc1c\uacac\ub418\ub294 \ubc1c\ub2ec\uc6d0\ub9ac\ub97c \ucd94\uad6c\ud558\ub294 \uc0c8\ub85c\uc6b4 \uc2ec\ub9ac\ud559\uc758 \ubd84\uc57c\uc778 \uc544\ub3d9\uc2ec\ub9ac\ud559\uc740 \ubbf8\uad6d\uc758 \ud640 (S. Hall)\uc5d0 \uc758\ud574 \ucd5c\ucd08\ub85c \uc2dc\ub3c4\ub418\uc5c8\ub2e4. \uadf8\ub294 \uc0dd\ubb3c\ud559\uc801\u00b7\uc778\ub958\ud559\uc801\uc778 \uc601\uc5ed\uc744 \ud3ec\ud568\ud558\ub294 \ubc1c\uc0dd\uc2ec\ub9ac\ud559\uc801\uc778 \uc785\uc7a5\uc5d0\uc11c <\uc544\ub3d9\uc758 \ub3c4\ub355\uc801\u00b7\uc885\uad50\uc801 \uad50\uc721>(1883) \ub4f1\uc744 \ubc1c\ud45c\ud558\uc600\ub2e4. \uc774\uc640 \uac19\uc740 \uc544\ub3d9\uc2ec\ub9ac\ud559\uc5d0 \ub300\ud55c \uad00\uc2ec\uacfc \ud559\ubb38\uc801\uc778 \ud65c\ub3d9\uc740 \uad50\uc721\uc2ec\ub9ac\ud559\uc758 \uae30\ucd08\ub97c \uc774\ub8e8\uac8c \ub418\uc5c8\ub2e4. \uc774\uc758 \uc77c\ud658\uc73c\ub85c \ub3c5\uc77c\uc758 \ubaa8\uc774\ub9cc(E. Meumann)\uc740 \uc2ec\ub9ac\ud559\uacfc \uad50\uc721\ud559\uc758 \uacb0\ud569\uc744 \uc2dc\ub3c4\ud558\ub294 \uc5f0\uad6c\uc81c\ub3c4\ub85c\uc11c, \u2460 \uc544\ub3d9\uc758 \uc815\uc2e0\uc801\u00b7\uc2e0\uccb4\uc801 \ubc1c\ub2ec, \u2461 \uc544\ub3d9 \uac1c\uc778\uc758 \uc815\uc2e0\ub2a5\ub825\uc758 \ubc1c\ub2ec, \u2462 \uc544\ub3d9\uc758 \uac1c\uc131, \u2463 \ucc9c\ubd80\uc758 \uac1c\uc778\ucc28, \u2464 \uc544\ub3d9\uc758 \ud559\uad50\uc0dd\ud65c, \u2465 \uac01 \uad50\uacfc\uc5d0 \uc788\uc5b4\uc11c\uc758 \ud559\uc2b5\ud65c\ub3d9, \u2466 \uad50\uc0ac\uc758 \ud65c\ub3d9\u00b7\ud559\uc2b5\uc9c0\ub3c4\u00b7\ud559\uad50\uc81c\ub3c4\u00b7\uad50\uc218\ubc29\ubc95 \ub4f1\uc73c\ub85c \ubd84\ub958\ud558\uc5ec \uc81c\uc2dc\ub97c \ud558\uc600\uc73c\uba70, \uc774\uac83\uc740 \uad50\uc721\uc2ec\ub9ac\ud559\uc758 \ud559\ubb38\uc801 \uace8\uaca9\uc744 \uc774\ub8e8\uac8c \ub41c \uac83\uc774\ub2e4. \uad50\uc721\uc2ec\ub9ac\ud559\uc758 \ub610 \ud558\ub098\uc758 \uc120\uad6c\uc801 \uc5c5\uc801\uc740 \uac1c\uc778\ucc28\uc5d0 \uad00\ud55c \uc5f0\uad6c\uc774\ub2e4. \uc774\ub294 \uad6c\ubbf8\uc81c\uad6d\uc5d0\uc11c 19\uc138\uae30 \ud6c4\ubc18\uc5d0\uc11c 20\uc138\uae30\uc5d0 \uac78\uccd0 \uc774\ub8e8\uc5b4\uc84c\ub294\ub370, \uac1c\uc778\ucc28\uc5d0 \uad00\ud55c \uc5f0\uad6c\uc758 \uc120\uad6c\uc790\ub294 \uace8\ud134(F. Galton)\uc73c\ub85c <\uc720\uc804\uc801 \ucc9c\uc7ac>(1869) \ubc0f <\uc778\uac04\ub2a5\ub825\uc758 \uc5f0\uad6c>(1883) \ub4f1\uc744 \ubc1c\ud45c\ud588\uace0, \ubbf8\uad6d\uc758 \uce74\ub378(G. M. Cattell)\uc740 \uac1d\uad00\uc801 \ubc29\ubc95\uc5d0 \uc758\ud574\uc11c \uc778\uac04\uc758 \ub2a5\ub825\uc744 \uce21\uc815\ud558\uc5ec \uc774\uac83\uc744 \uc2e4\uc81c\uba74\uc5d0 \uc801\uc6a9\ud558\ub294 \ub370 \ub178\ub825\ud558\uc600\uc73c\uba70, \ud2b9\ud788 1893\ub144 \uc774\ud6c4 \ub9e4\ub144 \uce7c\ub7fc\ube44\uc544 \ub300\ud559\uc758 \uc2e0\uc785\uc0dd\ub4e4\uc5d0\uac8c \uc2e4\uc2dc\ud55c \ud14c\uc2a4\ud2b8\ub294 \uba58\ud0c8 \ud14c\uc2a4\ud2b8(mental test)\uc758 \uc2dc\ucd08\uac00 \ub418\uc5c8\ub2e4. \uadf8\ub7ec\ub098 \uace8\ud134\uacfc \uce74\ud154\uc758 \uac1c\uc778\ucc28 \uce21\uc815\uc740 \uc9c0\ub2a5\uac80\uc0ac\ub85c\ub294 \uacb0\uc2e4\uc744 \ubcf4\uc9c0 \ubabb\ud558\uace0, \ud504\ub791\uc2a4\uc758 \ube44\ub124(A. Binet)\uc5d0 \uc758\ud558\uc5ec 1905\ub144\uc5d0 \ud30c\ub9ac \ucd08\ub4f1\ud559\uad50 \uc5b4\ub9b0\uc774\uc758 \uc800\ub2a5\uc544 \uac10\ubcc4\uc774\ub77c\ub294 \ub2f9\uc2dc\uc758 \uad50\uc721\uacc4\uc758 \uc694\uccad\uc5d0 \ubd80\uc751\ud574\uc11c \uc774\ub8e8\uc5b4\uc84c\ub2e4. \ubfd0\ub9cc \uc544\ub2c8\ub77c \uc720\uc544\uc5d0 \uad00\ud55c \uc5f0\uad6c\ub294 19\uc138\uae30\uc5d0\uc11c\ubd80\ud130 20\uc138\uae30\ucd08\uae4c\uc9c0 \ubc1c\ub2ec\ud558\uc5ec \uc654\ub294\ub370, \uc720\uce58\uc6d0 \uad50\uc721\uc758 \uae30\ucd08\ub294 \ud504\ub8b0\ubca8(F. Fr bel)\uc5d0 \uc758\ud574 \ud655\ub9bd\ub418\uc5c8\uace0, \uc544\ub3d9\uc2ec\ub9ac\ud559\uc758 \uc2dc\uc870\ub77c\uace0 \ubd88\ub9ac\ub294 \ub3c5\uc77c\uc758 \uc0dd\ub9ac\ud559\uc790 \ud504\ub77c\uc774\uc5b4(W. Preyer) \uc774\ud6c4 \uc544\ub3d9\uc5d0 \uad00\ud55c \ub9ce\uc740 \uc5f0\uad6c\uac00 \uc774\ub8e8\uc5b4\uc84c\ub2e4. \uc774\uc0c1\uc5d0\uc11c \ubcf8 \ubc14\uc640 \uac19\uc774 \uad50\uc721\uc2ec\ub9ac\ud559\uc758 \uc131\ub9bd\ub3d9\uae30\ub294 \ub2e8\uc21c\ud788 \uc2ec\ub9ac\ud559\uacfc \uad50\uc721\uc774 \uc811\ucd09\ud558\ub294 \uc810\uc5d0\uc11c \ubc1c\uc0dd\ud55c \uac83\uc774 \uc544\ub2c8\ub2e4.", "sentence_answer": "\ubbf8\uad6d\uc758 \ud640 (S. Hall)", "paragraph_id": "6252846-1-2", "paragraph_question": "question: \uc544\ub3d9\uc2ec\ub9ac\ud559\uc744 \ucd5c\ucd08\ub85c \uc2dc\ub3c4\ud55c \uc0ac\ub78c\uc740?, context: \uad50\uc721\uc2ec\ub9ac\ud559\uc740 \uad50\uc721\uc744 \ud6a8\uacfc\uc801\uc73c\ub85c \uc774\ub8e9\ud558\uae30 \uc704\ud55c \uc2dd\uacac\uc774\ub098 \uae30\uc220\uc744 \uc81c\uacf5\ud558\ub294 \ud559\ubb38\uc774\ub77c\uace0 \uadf8 \uc131\uaca9\uc744 \uaddc\uc815\ud55c\ub2e4\uba74, '\ub2e8\uacc4\uc2dd \uad50\uc721\uccb4\uacc4'\ub97c \uc81c\ucc3d\ud558\uc600\uace0, \uadf8\ub9bc\uacfc \uc5b8\uc5b4\ub97c \uacb0\ud569\ud55c \ucd5c\ucd08\uc758 \uad50\uacfc\uc11c\uc778 <\uc138\uacc4\ub3c4\ud68c(\u4e16\u754c\u5716\u7e6a)>\uc5d0 \uc758\ud574\uc11c \uc5b4\ub9b0\uc774\uc758 \uc5b8\uc5b4\uad50\uc721 \ubc0f \uc2dc\uccad\uac01\uad50\uc721\uc758 \uae30\ucd08\ub97c \uc81c\uacf5\ud55c \ucf54\uba54\ub2c8\uc6b0\uc2a4(J. A. Comenuis), <\uc5d0\ubc00>\uc758 \uc800\uc790 \ub8e8\uc18c(J. J.Rousseau),\ud398\uc2a4\ud0c8\ub85c\uce58(J. H. Pestalozzi) \ub4f1\uc744 \uc774 \ubd84\uc57c\uc758 \uc120\uad6c\uc801\uc778 \uc778\ubb3c\ub85c \uc81c\uc2dc\ud560 \uc218 \uc788\uc73c\ub098, \uad50\uc721\uc758 \ubc29\ubc95\uc5d0\uc11c \uc694\uad6c\ub418\ub294 \uc5ec\ub7ec \uac00\uc9c0 \uac1c\ub150\uc744 \uc2ec\ub9ac\ud559\uc801\uc73c\ub85c \ucde8\uae09\ud55c \uac83\uc740 \ud5e4\ub974\ubc14\ub974\ud2b8(J. F. Herbart)\uc5d0 \uc774\ub974\ub7ec\uc11c \ucd5c\ucd08\uc758 \uc2dc\ub3c4\uac00 \uc774\ub8e8\uc5b4\uc84c\ub2e4\uace0 \ud560 \uc218 \uc788\ub2e4. \uadf8\ub7ec\ub098 \uc2ec\ub9ac\uacfc\ud559\uc801\uc778 \uc785\uc7a5\uc5d0\uc11c \uad50\uc721\uc2ec\ub9ac\ud559\uc758 \uc6d0\ud615\uc744 \uc774\ub8e9\ud55c \uac83\uc740 \uc5ed\uc2dc \ud604\ub300\uc2ec\ub9ac\ud559\uc758 \uc2dc\uc870\uc778 \ubd84\ud2b8(W. Wundt)\uc774\ub2e4. \uad50\uc721\uc2ec\ub9ac\ud559\uc740 \uc544\ub3d9\uc2ec\ub9ac\ud559\uc758 \ubc1c\ub2ec\uc5d0 \ud798\uc744 \uc785\uc740 \ubc14 \uc801\uc9c0 \uc54a\ub2e4. \uc544\ub3d9\uc758 \uc815\uc2e0\ubc1c\ub2ec\uc744 \uae30\uc220\ud558\uace0, \uac70\uae30\uc5d0\uc11c \ubc1c\uacac\ub418\ub294 \ubc1c\ub2ec\uc6d0\ub9ac\ub97c \ucd94\uad6c\ud558\ub294 \uc0c8\ub85c\uc6b4 \uc2ec\ub9ac\ud559\uc758 \ubd84\uc57c\uc778 \uc544\ub3d9\uc2ec\ub9ac\ud559\uc740 \ubbf8\uad6d\uc758 \ud640(S. Hall)\uc5d0 \uc758\ud574 \ucd5c\ucd08\ub85c \uc2dc\ub3c4\ub418\uc5c8\ub2e4. \uadf8\ub294 \uc0dd\ubb3c\ud559\uc801\u00b7\uc778\ub958\ud559\uc801\uc778 \uc601\uc5ed\uc744 \ud3ec\ud568\ud558\ub294 \ubc1c\uc0dd\uc2ec\ub9ac\ud559\uc801\uc778 \uc785\uc7a5\uc5d0\uc11c <\uc544\ub3d9\uc758 \ub3c4\ub355\uc801\u00b7\uc885\uad50\uc801 \uad50\uc721>(1883) \ub4f1\uc744 \ubc1c\ud45c\ud558\uc600\ub2e4. \uc774\uc640 \uac19\uc740 \uc544\ub3d9\uc2ec\ub9ac\ud559\uc5d0 \ub300\ud55c \uad00\uc2ec\uacfc \ud559\ubb38\uc801\uc778 \ud65c\ub3d9\uc740 \uad50\uc721\uc2ec\ub9ac\ud559\uc758 \uae30\ucd08\ub97c \uc774\ub8e8\uac8c \ub418\uc5c8\ub2e4. \uc774\uc758 \uc77c\ud658\uc73c\ub85c \ub3c5\uc77c\uc758 \ubaa8\uc774\ub9cc(E. Meumann)\uc740 \uc2ec\ub9ac\ud559\uacfc \uad50\uc721\ud559\uc758 \uacb0\ud569\uc744 \uc2dc\ub3c4\ud558\ub294 \uc5f0\uad6c\uc81c\ub3c4\ub85c\uc11c, \u2460 \uc544\ub3d9\uc758 \uc815\uc2e0\uc801\u00b7\uc2e0\uccb4\uc801 \ubc1c\ub2ec, \u2461 \uc544\ub3d9 \uac1c\uc778\uc758 \uc815\uc2e0\ub2a5\ub825\uc758 \ubc1c\ub2ec, \u2462 \uc544\ub3d9\uc758 \uac1c\uc131, \u2463 \ucc9c\ubd80\uc758 \uac1c\uc778\ucc28, \u2464 \uc544\ub3d9\uc758 \ud559\uad50\uc0dd\ud65c, \u2465 \uac01 \uad50\uacfc\uc5d0 \uc788\uc5b4\uc11c\uc758 \ud559\uc2b5\ud65c\ub3d9, \u2466 \uad50\uc0ac\uc758 \ud65c\ub3d9\u00b7\ud559\uc2b5\uc9c0\ub3c4\u00b7\ud559\uad50\uc81c\ub3c4\u00b7\uad50\uc218\ubc29\ubc95 \ub4f1\uc73c\ub85c \ubd84\ub958\ud558\uc5ec \uc81c\uc2dc\ub97c \ud558\uc600\uc73c\uba70, \uc774\uac83\uc740 \uad50\uc721\uc2ec\ub9ac\ud559\uc758 \ud559\ubb38\uc801 \uace8\uaca9\uc744 \uc774\ub8e8\uac8c \ub41c \uac83\uc774\ub2e4. \uad50\uc721\uc2ec\ub9ac\ud559\uc758 \ub610 \ud558\ub098\uc758 \uc120\uad6c\uc801 \uc5c5\uc801\uc740 \uac1c\uc778\ucc28\uc5d0 \uad00\ud55c \uc5f0\uad6c\uc774\ub2e4. \uc774\ub294 \uad6c\ubbf8\uc81c\uad6d\uc5d0\uc11c 19\uc138\uae30 \ud6c4\ubc18\uc5d0\uc11c 20\uc138\uae30\uc5d0 \uac78\uccd0 \uc774\ub8e8\uc5b4\uc84c\ub294\ub370, \uac1c\uc778\ucc28\uc5d0 \uad00\ud55c \uc5f0\uad6c\uc758 \uc120\uad6c\uc790\ub294 \uace8\ud134(F. Galton)\uc73c\ub85c <\uc720\uc804\uc801 \ucc9c\uc7ac>(1869) \ubc0f <\uc778\uac04\ub2a5\ub825\uc758 \uc5f0\uad6c>(1883) \ub4f1\uc744 \ubc1c\ud45c\ud588\uace0, \ubbf8\uad6d\uc758 \uce74\ub378(G. M. Cattell)\uc740 \uac1d\uad00\uc801 \ubc29\ubc95\uc5d0 \uc758\ud574\uc11c \uc778\uac04\uc758 \ub2a5\ub825\uc744 \uce21\uc815\ud558\uc5ec \uc774\uac83\uc744 \uc2e4\uc81c\uba74\uc5d0 \uc801\uc6a9\ud558\ub294 \ub370 \ub178\ub825\ud558\uc600\uc73c\uba70, \ud2b9\ud788 1893\ub144 \uc774\ud6c4 \ub9e4\ub144 \uce7c\ub7fc\ube44\uc544 \ub300\ud559\uc758 \uc2e0\uc785\uc0dd\ub4e4\uc5d0\uac8c \uc2e4\uc2dc\ud55c \ud14c\uc2a4\ud2b8\ub294 \uba58\ud0c8 \ud14c\uc2a4\ud2b8(mental test)\uc758 \uc2dc\ucd08\uac00 \ub418\uc5c8\ub2e4. \uadf8\ub7ec\ub098 \uace8\ud134\uacfc \uce74\ud154\uc758 \uac1c\uc778\ucc28 \uce21\uc815\uc740 \uc9c0\ub2a5\uac80\uc0ac\ub85c\ub294 \uacb0\uc2e4\uc744 \ubcf4\uc9c0 \ubabb\ud558\uace0, \ud504\ub791\uc2a4\uc758 \ube44\ub124(A. Binet)\uc5d0 \uc758\ud558\uc5ec 1905\ub144\uc5d0 \ud30c\ub9ac \ucd08\ub4f1\ud559\uad50 \uc5b4\ub9b0\uc774\uc758 \uc800\ub2a5\uc544 \uac10\ubcc4\uc774\ub77c\ub294 \ub2f9\uc2dc\uc758 \uad50\uc721\uacc4\uc758 \uc694\uccad\uc5d0 \ubd80\uc751\ud574\uc11c \uc774\ub8e8\uc5b4\uc84c\ub2e4. \ubfd0\ub9cc \uc544\ub2c8\ub77c \uc720\uc544\uc5d0 \uad00\ud55c \uc5f0\uad6c\ub294 19\uc138\uae30\uc5d0\uc11c\ubd80\ud130 20\uc138\uae30\ucd08\uae4c\uc9c0 \ubc1c\ub2ec\ud558\uc5ec \uc654\ub294\ub370, \uc720\uce58\uc6d0 \uad50\uc721\uc758 \uae30\ucd08\ub294 \ud504\ub8b0\ubca8(F. Fr bel)\uc5d0 \uc758\ud574 \ud655\ub9bd\ub418\uc5c8\uace0, \uc544\ub3d9\uc2ec\ub9ac\ud559\uc758 \uc2dc\uc870\ub77c\uace0 \ubd88\ub9ac\ub294 \ub3c5\uc77c\uc758 \uc0dd\ub9ac\ud559\uc790 \ud504\ub77c\uc774\uc5b4(W. Preyer) \uc774\ud6c4 \uc544\ub3d9\uc5d0 \uad00\ud55c \ub9ce\uc740 \uc5f0\uad6c\uac00 \uc774\ub8e8\uc5b4\uc84c\ub2e4. \uc774\uc0c1\uc5d0\uc11c \ubcf8 \ubc14\uc640 \uac19\uc774 \uad50\uc721\uc2ec\ub9ac\ud559\uc758 \uc131\ub9bd\ub3d9\uae30\ub294 \ub2e8\uc21c\ud788 \uc2ec\ub9ac\ud559\uacfc \uad50\uc721\uc774 \uc811\ucd09\ud558\ub294 \uc810\uc5d0\uc11c \ubc1c\uc0dd\ud55c \uac83\uc774 \uc544\ub2c8\ub2e4."} {"question": "\uc724\uc601\uc774 \uc190\ubb34, \uc624\uae30\uc758 \ubcd1\ubc95\uc744 \uc775\ud600 \uce58\ub824\uace0 \ud588\ub358 \ub098\ub77c\ub294?", "paragraph": "\uac15\ud55c\uc9d1(\u6c5f\u6f22\u96c6)\uc5d0 \uc758\ud558\uba74 \uc724\uc601\uc740 \uc77c\ucc0d\ubd80\ud130 '\uae30\ub7b5\uc744 \uc88b\uc544\ud558\uace0 \uc5f4\uc0ac\uc758 \ud48d\uc774 \uc788\uc5c8\ub2e4.'\uace0 \ud55c\ub2e4. \uacfc\uac70\ub97c \ub2e8\ub150\ud558\uace0 \uacbd\uc138\uc81c\ubbfc\ud558\ub294 \ud559\ubb38\uc744 \uc88b\uc544\ud558\uc600\ub2e4. \uadf8\ub294 \ub0a8\uc778\uc7ac\uc0c1 \uc774\uc6d0\uc775\uc758 \uc11c\ub140\uc640 \uacb0\ud63c\ud588\ub294\ub370, \uc774\uc6d0\uc775\uc740 \uadf8\uc758 \uc7ac\uc8fc\ub97c \uc544\uae5d\uac8c \uc5ec\uaca8 \uc11c\ub140\uc758 \ub0a8\ud3b8\uc774\uace0 \uc11c\uc5bc\uc774\uc5c8\uc9c0\ub9cc \uc0ac\uc704\ub85c \ub300\uc811\ud558\uc600\ub2e4. \uc774\uc6d0\uc775\uc740 \uadf8\ub97c \ub450\uace0 \"\uc6b0\ub9ac \uc0ac\uc704\ub294 \uc7ac\uc0c1\uac10\uc774\ub2e4\"\ub77c\uace0 \ud558\uc600\ub2e4. \uc18c\ub144\uae30\uc5d0 \uc774\ub974\ub7ec \uadf8\ub294 \ud559\ubb38\uc740 \ub418\uc5c8\uc9c0\ub9cc \uacfc\uac70\uc5d0 \uc751\uc2dc\ud558\uc9c0 \uc54a\uc558\uace0 \uba85\ub098\ub77c\uac00 \uccad\ub098\ub77c\uc5d0 \ub9dd\ud558\ub294 \uac83\uc744 \ubd84\ud558\uac8c \uc5ec\uacbc\ub2e4. \uc774\uc5d0 \uc724\uc601\uc740 \ubab0\ub798 \uc190\ubb34\u22c5\uc624\uae30\uc758 \ubcd1\ubc95\uc744 \uc775\ud600\uc11c \uccad\ub098\ub77c\ub97c \uce58\ub824\ub294 \ubc29\ucc45\uc744 \ub9cc\ub4e4\uc5b4 \uc2a4\uc2a4\ub85c \ub098\ud0c0\ub0c8\ub2e4. \uc801\uc81c\uc600\ub358 \uc724\ud734 \uc80a\uc5b4\uc11c \uc11c\ud615\uc778 \uc724\uc601\uc744 \ub530\ub77c \uadf8 \uc124\uc744 \uc5bb\uc5c8\ub2e4 \ud55c\ub2e4. \uadf8\ub294 \uc7a5\uc778 \uc774\uc6d0\uc775\uc758 \ucd1d\uc560\ub97c \ud2b9\ubcc4\ud788 \ubc1b\uc558\uace0 \uc724\ud734 \uc5ed\uc2dc \u201c\ud615\uc758 \ubb38\uc7a5\uc740 \ub098\ubcf4\ub2e4 \ub0ab\ub2e4.\u201d \ud558\uc600\ub2e4.", "answer": "\uccad\ub098\ub77c", "sentence": "\uba85\ub098\ub77c\uac00 \uccad\ub098\ub77c \uc5d0 \ub9dd\ud558\ub294 \uac83\uc744 \ubd84\ud558\uac8c \uc5ec\uacbc\ub2e4.", "paragraph_sentence": "\uac15\ud55c\uc9d1(\u6c5f\u6f22\u96c6)\uc5d0 \uc758\ud558\uba74 \uc724\uc601\uc740 \uc77c\ucc0d\ubd80\ud130 '\uae30\ub7b5\uc744 \uc88b\uc544\ud558\uace0 \uc5f4\uc0ac\uc758 \ud48d\uc774 \uc788\uc5c8\ub2e4. '\uace0 \ud55c\ub2e4. \uacfc\uac70\ub97c \ub2e8\ub150\ud558\uace0 \uacbd\uc138\uc81c\ubbfc\ud558\ub294 \ud559\ubb38\uc744 \uc88b\uc544\ud558\uc600\ub2e4. \uadf8\ub294 \ub0a8\uc778\uc7ac\uc0c1 \uc774\uc6d0\uc775\uc758 \uc11c\ub140\uc640 \uacb0\ud63c\ud588\ub294\ub370, \uc774\uc6d0\uc775\uc740 \uadf8\uc758 \uc7ac\uc8fc\ub97c \uc544\uae5d\uac8c \uc5ec\uaca8 \uc11c\ub140\uc758 \ub0a8\ud3b8\uc774\uace0 \uc11c\uc5bc\uc774\uc5c8\uc9c0\ub9cc \uc0ac\uc704\ub85c \ub300\uc811\ud558\uc600\ub2e4. \uc774\uc6d0\uc775\uc740 \uadf8\ub97c \ub450\uace0 \"\uc6b0\ub9ac \uc0ac\uc704\ub294 \uc7ac\uc0c1\uac10\uc774\ub2e4\"\ub77c\uace0 \ud558\uc600\ub2e4. \uc18c\ub144\uae30\uc5d0 \uc774\ub974\ub7ec \uadf8\ub294 \ud559\ubb38\uc740 \ub418\uc5c8\uc9c0\ub9cc \uacfc\uac70\uc5d0 \uc751\uc2dc\ud558\uc9c0 \uc54a\uc558\uace0 \uba85\ub098\ub77c\uac00 \uccad\ub098\ub77c \uc5d0 \ub9dd\ud558\ub294 \uac83\uc744 \ubd84\ud558\uac8c \uc5ec\uacbc\ub2e4. \uc774\uc5d0 \uc724\uc601\uc740 \ubab0\ub798 \uc190\ubb34\u22c5\uc624\uae30\uc758 \ubcd1\ubc95\uc744 \uc775\ud600\uc11c \uccad\ub098\ub77c\ub97c \uce58\ub824\ub294 \ubc29\ucc45\uc744 \ub9cc\ub4e4\uc5b4 \uc2a4\uc2a4\ub85c \ub098\ud0c0\ub0c8\ub2e4. \uc801\uc81c\uc600\ub358 \uc724\ud734 \uc80a\uc5b4\uc11c \uc11c\ud615\uc778 \uc724\uc601\uc744 \ub530\ub77c \uadf8 \uc124\uc744 \uc5bb\uc5c8\ub2e4 \ud55c\ub2e4. \uadf8\ub294 \uc7a5\uc778 \uc774\uc6d0\uc775\uc758 \ucd1d\uc560\ub97c \ud2b9\ubcc4\ud788 \ubc1b\uc558\uace0 \uc724\ud734 \uc5ed\uc2dc \u201c\ud615\uc758 \ubb38\uc7a5\uc740 \ub098\ubcf4\ub2e4 \ub0ab\ub2e4. \u201d \ud558\uc600\ub2e4.", "paragraph_answer": "\uac15\ud55c\uc9d1(\u6c5f\u6f22\u96c6)\uc5d0 \uc758\ud558\uba74 \uc724\uc601\uc740 \uc77c\ucc0d\ubd80\ud130 '\uae30\ub7b5\uc744 \uc88b\uc544\ud558\uace0 \uc5f4\uc0ac\uc758 \ud48d\uc774 \uc788\uc5c8\ub2e4.'\uace0 \ud55c\ub2e4. \uacfc\uac70\ub97c \ub2e8\ub150\ud558\uace0 \uacbd\uc138\uc81c\ubbfc\ud558\ub294 \ud559\ubb38\uc744 \uc88b\uc544\ud558\uc600\ub2e4. \uadf8\ub294 \ub0a8\uc778\uc7ac\uc0c1 \uc774\uc6d0\uc775\uc758 \uc11c\ub140\uc640 \uacb0\ud63c\ud588\ub294\ub370, \uc774\uc6d0\uc775\uc740 \uadf8\uc758 \uc7ac\uc8fc\ub97c \uc544\uae5d\uac8c \uc5ec\uaca8 \uc11c\ub140\uc758 \ub0a8\ud3b8\uc774\uace0 \uc11c\uc5bc\uc774\uc5c8\uc9c0\ub9cc \uc0ac\uc704\ub85c \ub300\uc811\ud558\uc600\ub2e4. \uc774\uc6d0\uc775\uc740 \uadf8\ub97c \ub450\uace0 \"\uc6b0\ub9ac \uc0ac\uc704\ub294 \uc7ac\uc0c1\uac10\uc774\ub2e4\"\ub77c\uace0 \ud558\uc600\ub2e4. \uc18c\ub144\uae30\uc5d0 \uc774\ub974\ub7ec \uadf8\ub294 \ud559\ubb38\uc740 \ub418\uc5c8\uc9c0\ub9cc \uacfc\uac70\uc5d0 \uc751\uc2dc\ud558\uc9c0 \uc54a\uc558\uace0 \uba85\ub098\ub77c\uac00 \uccad\ub098\ub77c \uc5d0 \ub9dd\ud558\ub294 \uac83\uc744 \ubd84\ud558\uac8c \uc5ec\uacbc\ub2e4. \uc774\uc5d0 \uc724\uc601\uc740 \ubab0\ub798 \uc190\ubb34\u22c5\uc624\uae30\uc758 \ubcd1\ubc95\uc744 \uc775\ud600\uc11c \uccad\ub098\ub77c\ub97c \uce58\ub824\ub294 \ubc29\ucc45\uc744 \ub9cc\ub4e4\uc5b4 \uc2a4\uc2a4\ub85c \ub098\ud0c0\ub0c8\ub2e4. \uc801\uc81c\uc600\ub358 \uc724\ud734 \uc80a\uc5b4\uc11c \uc11c\ud615\uc778 \uc724\uc601\uc744 \ub530\ub77c \uadf8 \uc124\uc744 \uc5bb\uc5c8\ub2e4 \ud55c\ub2e4. \uadf8\ub294 \uc7a5\uc778 \uc774\uc6d0\uc775\uc758 \ucd1d\uc560\ub97c \ud2b9\ubcc4\ud788 \ubc1b\uc558\uace0 \uc724\ud734 \uc5ed\uc2dc \u201c\ud615\uc758 \ubb38\uc7a5\uc740 \ub098\ubcf4\ub2e4 \ub0ab\ub2e4.\u201d \ud558\uc600\ub2e4.", "sentence_answer": "\uba85\ub098\ub77c\uac00 \uccad\ub098\ub77c \uc5d0 \ub9dd\ud558\ub294 \uac83\uc744 \ubd84\ud558\uac8c \uc5ec\uacbc\ub2e4.", "paragraph_id": "6589645-0-1", "paragraph_question": "question: \uc724\uc601\uc774 \uc190\ubb34, \uc624\uae30\uc758 \ubcd1\ubc95\uc744 \uc775\ud600 \uce58\ub824\uace0 \ud588\ub358 \ub098\ub77c\ub294?, context: \uac15\ud55c\uc9d1(\u6c5f\u6f22\u96c6)\uc5d0 \uc758\ud558\uba74 \uc724\uc601\uc740 \uc77c\ucc0d\ubd80\ud130 '\uae30\ub7b5\uc744 \uc88b\uc544\ud558\uace0 \uc5f4\uc0ac\uc758 \ud48d\uc774 \uc788\uc5c8\ub2e4.'\uace0 \ud55c\ub2e4. \uacfc\uac70\ub97c \ub2e8\ub150\ud558\uace0 \uacbd\uc138\uc81c\ubbfc\ud558\ub294 \ud559\ubb38\uc744 \uc88b\uc544\ud558\uc600\ub2e4. \uadf8\ub294 \ub0a8\uc778\uc7ac\uc0c1 \uc774\uc6d0\uc775\uc758 \uc11c\ub140\uc640 \uacb0\ud63c\ud588\ub294\ub370, \uc774\uc6d0\uc775\uc740 \uadf8\uc758 \uc7ac\uc8fc\ub97c \uc544\uae5d\uac8c \uc5ec\uaca8 \uc11c\ub140\uc758 \ub0a8\ud3b8\uc774\uace0 \uc11c\uc5bc\uc774\uc5c8\uc9c0\ub9cc \uc0ac\uc704\ub85c \ub300\uc811\ud558\uc600\ub2e4. \uc774\uc6d0\uc775\uc740 \uadf8\ub97c \ub450\uace0 \"\uc6b0\ub9ac \uc0ac\uc704\ub294 \uc7ac\uc0c1\uac10\uc774\ub2e4\"\ub77c\uace0 \ud558\uc600\ub2e4. \uc18c\ub144\uae30\uc5d0 \uc774\ub974\ub7ec \uadf8\ub294 \ud559\ubb38\uc740 \ub418\uc5c8\uc9c0\ub9cc \uacfc\uac70\uc5d0 \uc751\uc2dc\ud558\uc9c0 \uc54a\uc558\uace0 \uba85\ub098\ub77c\uac00 \uccad\ub098\ub77c\uc5d0 \ub9dd\ud558\ub294 \uac83\uc744 \ubd84\ud558\uac8c \uc5ec\uacbc\ub2e4. \uc774\uc5d0 \uc724\uc601\uc740 \ubab0\ub798 \uc190\ubb34\u22c5\uc624\uae30\uc758 \ubcd1\ubc95\uc744 \uc775\ud600\uc11c \uccad\ub098\ub77c\ub97c \uce58\ub824\ub294 \ubc29\ucc45\uc744 \ub9cc\ub4e4\uc5b4 \uc2a4\uc2a4\ub85c \ub098\ud0c0\ub0c8\ub2e4. \uc801\uc81c\uc600\ub358 \uc724\ud734 \uc80a\uc5b4\uc11c \uc11c\ud615\uc778 \uc724\uc601\uc744 \ub530\ub77c \uadf8 \uc124\uc744 \uc5bb\uc5c8\ub2e4 \ud55c\ub2e4. \uadf8\ub294 \uc7a5\uc778 \uc774\uc6d0\uc775\uc758 \ucd1d\uc560\ub97c \ud2b9\ubcc4\ud788 \ubc1b\uc558\uace0 \uc724\ud734 \uc5ed\uc2dc \u201c\ud615\uc758 \ubb38\uc7a5\uc740 \ub098\ubcf4\ub2e4 \ub0ab\ub2e4.\u201d \ud558\uc600\ub2e4."} {"question": "\ucf54\ud398\ub974\ub2c8\ucfe0\uc2a4 \ucc9c\ubb38\ud559 \uac1c\uc694\ub294 \ubb34\uc5c7\uc5d0 \ub300\ud55c \uacb0\ub860\uc744 \uc124\uba85\ud558\uc9c0 \ubabb\ud558\uc600\ub294\uac00?", "paragraph": "\u300a\uc2e0\ucc9c\ubb38\ud559\u300b\uc758 \uc644\uc131 \uc774\ud6c4\ub85c, \ucf00\ud50c\ub7ec\ub294 \ucc9c\ubb38\ud559 \ucc45\uc744 \ub9cc\ub4e4\ub824\ub294 \ub9c8\uc74c\uc744 \uba39\uace0 \uc788\uc5c8\ub2e4. 1615\ub144, \uadf8\ub294 \u300a\ucf54\ud398\ub974\ub2c8\ucfe0\uc2a4 \ucc9c\ubb38\ud559 \uac1c\uc694\u300b\uc758 \ucc98\uc74c \uc138 \uad8c\uc744 \uc644\uc131\ud588\uace0, \uc774\ud6c4 4\uad8c\uc740 1620\ub144\uc5d0, \uadf8\ub9ac\uace0 5 ~ 7\uad8c\uc740 1621\ub144\uc5d0 \ucd9c\ud310\ud558\uc600\ub2e4. \ucf00\ud50c\ub7ec\uc758 \uc774 \ucc45\uc740 \ud0c0\uc6d0\uada4\ub3c4\uc758 \ubc95\uce59\uc758 \uacb0\uc815\ud310\uc73c\ub85c, \uc9c0\ub3d9\uc124\uc744 \uc5f0\uc0c1\uc2dc\ud0a4\ub294 \uc81c\ubaa9\uc5d0\ub3c4 \ubd88\uad6c\ud558\uace0 \ucf00\ud50c\ub7ec\uc758 \uac00\uc7a5 \uc601\ud5a5\ub825 \uc788\ub294 \uc791\uc5c5\uc774 \ub418\uc5c8\ub2e4. \uc774 \ucc45\uc740 \ud589\uc131\uc6b4\ub3d9\uc758 \uc138 \ubc95\uce59\uc758 \ubaa8\ub4e0 \uac83\uc744 \ud488\uace0 \uc788\uc5c8\uc73c\uba70 \ucc9c\uccb4\uc6b4\ub3d9\uc744 \ubb3c\ub9ac\ud559\uc801\uc73c\ub85c \uc124\uba85\ud558\ub824\ub294 \uc2dc\ub3c4\uc600\ub2e4. \uadf8\ub7ec\ub098 \uc774 \ucc45\uc740 \uc55e\uc758 \ub450 \ud589\uc131\uc6b4\ub3d9\ubc95\uce59\uc744 \ub178\uace8\uc801\uc73c\ub85c \ub2ec\uacfc \ubaa9\uc131\uc758 \uc8fc\uc694 \uc704\uc131\ub4e4\uc5d0\uae4c\uc9c0 \ud655\ub300 \ud574\uc11d\ud558\uace0 \uc788\uae30\ub294 \ud558\uc9c0\ub9cc, \uc5b4\ub5bb\uac8c \uad00\uce21 \uc790\ub8cc\ub85c\ubd80\ud130 \ud0c0\uc6d0\ud615 \uada4\ub3c4\ub97c \uacb0\ub860\uc9c0\uc5c8\ub294\uc9c0\uc5d0 \ub300\ud574 \uc124\uba85\ud558\uc9c0\ub294 \uc54a\uc558\ub2e4.", "answer": "\ud0c0\uc6d0\ud615 \uada4\ub3c4", "sentence": "\uc5b4\ub5bb\uac8c \uad00\uce21 \uc790\ub8cc\ub85c\ubd80\ud130 \ud0c0\uc6d0\ud615 \uada4\ub3c4 \ub97c \uacb0\ub860\uc9c0\uc5c8\ub294\uc9c0\uc5d0 \ub300\ud574 \uc124\uba85\ud558\uc9c0\ub294 \uc54a\uc558\ub2e4.", "paragraph_sentence": "\u300a\uc2e0\ucc9c\ubb38\ud559\u300b\uc758 \uc644\uc131 \uc774\ud6c4\ub85c, \ucf00\ud50c\ub7ec\ub294 \ucc9c\ubb38\ud559 \ucc45\uc744 \ub9cc\ub4e4\ub824\ub294 \ub9c8\uc74c\uc744 \uba39\uace0 \uc788\uc5c8\ub2e4. 1615\ub144, \uadf8\ub294 \u300a\ucf54\ud398\ub974\ub2c8\ucfe0\uc2a4 \ucc9c\ubb38\ud559 \uac1c\uc694\u300b\uc758 \ucc98\uc74c \uc138 \uad8c\uc744 \uc644\uc131\ud588\uace0, \uc774\ud6c4 4\uad8c\uc740 1620\ub144\uc5d0, \uadf8\ub9ac\uace0 5 ~ 7\uad8c\uc740 1621\ub144\uc5d0 \ucd9c\ud310\ud558\uc600\ub2e4. \ucf00\ud50c\ub7ec\uc758 \uc774 \ucc45\uc740 \ud0c0\uc6d0\uada4\ub3c4\uc758 \ubc95\uce59\uc758 \uacb0\uc815\ud310\uc73c\ub85c, \uc9c0\ub3d9\uc124\uc744 \uc5f0\uc0c1\uc2dc\ud0a4\ub294 \uc81c\ubaa9\uc5d0\ub3c4 \ubd88\uad6c\ud558\uace0 \ucf00\ud50c\ub7ec\uc758 \uac00\uc7a5 \uc601\ud5a5\ub825 \uc788\ub294 \uc791\uc5c5\uc774 \ub418\uc5c8\ub2e4. \uc774 \ucc45\uc740 \ud589\uc131\uc6b4\ub3d9\uc758 \uc138 \ubc95\uce59\uc758 \ubaa8\ub4e0 \uac83\uc744 \ud488\uace0 \uc788\uc5c8\uc73c\uba70 \ucc9c\uccb4\uc6b4\ub3d9\uc744 \ubb3c\ub9ac\ud559\uc801\uc73c\ub85c \uc124\uba85\ud558\ub824\ub294 \uc2dc\ub3c4\uc600\ub2e4. \uadf8\ub7ec\ub098 \uc774 \ucc45\uc740 \uc55e\uc758 \ub450 \ud589\uc131\uc6b4\ub3d9\ubc95\uce59\uc744 \ub178\uace8\uc801\uc73c\ub85c \ub2ec\uacfc \ubaa9\uc131\uc758 \uc8fc\uc694 \uc704\uc131\ub4e4\uc5d0\uae4c\uc9c0 \ud655\ub300 \ud574\uc11d\ud558\uace0 \uc788\uae30\ub294 \ud558\uc9c0\ub9cc, \uc5b4\ub5bb\uac8c \uad00\uce21 \uc790\ub8cc\ub85c\ubd80\ud130 \ud0c0\uc6d0\ud615 \uada4\ub3c4 \ub97c \uacb0\ub860\uc9c0\uc5c8\ub294\uc9c0\uc5d0 \ub300\ud574 \uc124\uba85\ud558\uc9c0\ub294 \uc54a\uc558\ub2e4. ", "paragraph_answer": "\u300a\uc2e0\ucc9c\ubb38\ud559\u300b\uc758 \uc644\uc131 \uc774\ud6c4\ub85c, \ucf00\ud50c\ub7ec\ub294 \ucc9c\ubb38\ud559 \ucc45\uc744 \ub9cc\ub4e4\ub824\ub294 \ub9c8\uc74c\uc744 \uba39\uace0 \uc788\uc5c8\ub2e4. 1615\ub144, \uadf8\ub294 \u300a\ucf54\ud398\ub974\ub2c8\ucfe0\uc2a4 \ucc9c\ubb38\ud559 \uac1c\uc694\u300b\uc758 \ucc98\uc74c \uc138 \uad8c\uc744 \uc644\uc131\ud588\uace0, \uc774\ud6c4 4\uad8c\uc740 1620\ub144\uc5d0, \uadf8\ub9ac\uace0 5 ~ 7\uad8c\uc740 1621\ub144\uc5d0 \ucd9c\ud310\ud558\uc600\ub2e4. \ucf00\ud50c\ub7ec\uc758 \uc774 \ucc45\uc740 \ud0c0\uc6d0\uada4\ub3c4\uc758 \ubc95\uce59\uc758 \uacb0\uc815\ud310\uc73c\ub85c, \uc9c0\ub3d9\uc124\uc744 \uc5f0\uc0c1\uc2dc\ud0a4\ub294 \uc81c\ubaa9\uc5d0\ub3c4 \ubd88\uad6c\ud558\uace0 \ucf00\ud50c\ub7ec\uc758 \uac00\uc7a5 \uc601\ud5a5\ub825 \uc788\ub294 \uc791\uc5c5\uc774 \ub418\uc5c8\ub2e4. \uc774 \ucc45\uc740 \ud589\uc131\uc6b4\ub3d9\uc758 \uc138 \ubc95\uce59\uc758 \ubaa8\ub4e0 \uac83\uc744 \ud488\uace0 \uc788\uc5c8\uc73c\uba70 \ucc9c\uccb4\uc6b4\ub3d9\uc744 \ubb3c\ub9ac\ud559\uc801\uc73c\ub85c \uc124\uba85\ud558\ub824\ub294 \uc2dc\ub3c4\uc600\ub2e4. \uadf8\ub7ec\ub098 \uc774 \ucc45\uc740 \uc55e\uc758 \ub450 \ud589\uc131\uc6b4\ub3d9\ubc95\uce59\uc744 \ub178\uace8\uc801\uc73c\ub85c \ub2ec\uacfc \ubaa9\uc131\uc758 \uc8fc\uc694 \uc704\uc131\ub4e4\uc5d0\uae4c\uc9c0 \ud655\ub300 \ud574\uc11d\ud558\uace0 \uc788\uae30\ub294 \ud558\uc9c0\ub9cc, \uc5b4\ub5bb\uac8c \uad00\uce21 \uc790\ub8cc\ub85c\ubd80\ud130 \ud0c0\uc6d0\ud615 \uada4\ub3c4 \ub97c \uacb0\ub860\uc9c0\uc5c8\ub294\uc9c0\uc5d0 \ub300\ud574 \uc124\uba85\ud558\uc9c0\ub294 \uc54a\uc558\ub2e4.", "sentence_answer": "\uc5b4\ub5bb\uac8c \uad00\uce21 \uc790\ub8cc\ub85c\ubd80\ud130 \ud0c0\uc6d0\ud615 \uada4\ub3c4 \ub97c \uacb0\ub860\uc9c0\uc5c8\ub294\uc9c0\uc5d0 \ub300\ud574 \uc124\uba85\ud558\uc9c0\ub294 \uc54a\uc558\ub2e4.", "paragraph_id": "6384301-25-1", "paragraph_question": "question: \ucf54\ud398\ub974\ub2c8\ucfe0\uc2a4 \ucc9c\ubb38\ud559 \uac1c\uc694\ub294 \ubb34\uc5c7\uc5d0 \ub300\ud55c \uacb0\ub860\uc744 \uc124\uba85\ud558\uc9c0 \ubabb\ud558\uc600\ub294\uac00?, context: \u300a\uc2e0\ucc9c\ubb38\ud559\u300b\uc758 \uc644\uc131 \uc774\ud6c4\ub85c, \ucf00\ud50c\ub7ec\ub294 \ucc9c\ubb38\ud559 \ucc45\uc744 \ub9cc\ub4e4\ub824\ub294 \ub9c8\uc74c\uc744 \uba39\uace0 \uc788\uc5c8\ub2e4. 1615\ub144, \uadf8\ub294 \u300a\ucf54\ud398\ub974\ub2c8\ucfe0\uc2a4 \ucc9c\ubb38\ud559 \uac1c\uc694\u300b\uc758 \ucc98\uc74c \uc138 \uad8c\uc744 \uc644\uc131\ud588\uace0, \uc774\ud6c4 4\uad8c\uc740 1620\ub144\uc5d0, \uadf8\ub9ac\uace0 5 ~ 7\uad8c\uc740 1621\ub144\uc5d0 \ucd9c\ud310\ud558\uc600\ub2e4. \ucf00\ud50c\ub7ec\uc758 \uc774 \ucc45\uc740 \ud0c0\uc6d0\uada4\ub3c4\uc758 \ubc95\uce59\uc758 \uacb0\uc815\ud310\uc73c\ub85c, \uc9c0\ub3d9\uc124\uc744 \uc5f0\uc0c1\uc2dc\ud0a4\ub294 \uc81c\ubaa9\uc5d0\ub3c4 \ubd88\uad6c\ud558\uace0 \ucf00\ud50c\ub7ec\uc758 \uac00\uc7a5 \uc601\ud5a5\ub825 \uc788\ub294 \uc791\uc5c5\uc774 \ub418\uc5c8\ub2e4. \uc774 \ucc45\uc740 \ud589\uc131\uc6b4\ub3d9\uc758 \uc138 \ubc95\uce59\uc758 \ubaa8\ub4e0 \uac83\uc744 \ud488\uace0 \uc788\uc5c8\uc73c\uba70 \ucc9c\uccb4\uc6b4\ub3d9\uc744 \ubb3c\ub9ac\ud559\uc801\uc73c\ub85c \uc124\uba85\ud558\ub824\ub294 \uc2dc\ub3c4\uc600\ub2e4. \uadf8\ub7ec\ub098 \uc774 \ucc45\uc740 \uc55e\uc758 \ub450 \ud589\uc131\uc6b4\ub3d9\ubc95\uce59\uc744 \ub178\uace8\uc801\uc73c\ub85c \ub2ec\uacfc \ubaa9\uc131\uc758 \uc8fc\uc694 \uc704\uc131\ub4e4\uc5d0\uae4c\uc9c0 \ud655\ub300 \ud574\uc11d\ud558\uace0 \uc788\uae30\ub294 \ud558\uc9c0\ub9cc, \uc5b4\ub5bb\uac8c \uad00\uce21 \uc790\ub8cc\ub85c\ubd80\ud130 \ud0c0\uc6d0\ud615 \uada4\ub3c4\ub97c \uacb0\ub860\uc9c0\uc5c8\ub294\uc9c0\uc5d0 \ub300\ud574 \uc124\uba85\ud558\uc9c0\ub294 \uc54a\uc558\ub2e4."} {"question": "1939\ub144 \uad11\uc0b0\uacfc\ub97c \ubd84\ub9ac\ud558\uc5ec \uc124\ub9bd\ud558\uac8c \ub41c \ud559\uad50\uba85\uc740?", "paragraph": "\uacbd\uc131\uace0\ub4f1\uacf5\uc5c5\ud559\uad50\uc758 \uad50\uc721\uc5f0\ud55c\uc740 3\ub144\uc774\uc5c8\uace0, \ud559\uacfc\ub85c\ub294 \ubc29\uc9c1\uacfc, \ud1a0\ubaa9\uacfc, \uac74\ucd95\uacfc, \uad11\uc0b0\uacfc, \uc751\uc6a9\ud654\ud559\uacfc\uac00 \uc124\uce58\ub418\uc5b4 \uc788\uc5c8\uc73c\uba70, \uc751\uc6a9\ud654\ud559\uacfc\uc5d0\ub294 \uc694\uc5c5, \uc5fc\uc0c9, \uc751\uc6a9\ud654\ud559\uc758 \uc138\uacfc(\u7d30\u79d1)\uac00 \uc788\uc5c8\ub2e4. 1937\ub144\uc5d0\ub294 \uc804\uae30\uacf5\ud559\uacfc\uc640 \uae30\uacc4\uacfc\uac00 \uac00\uc124\ub418\uc5c8\ub2e4. 1930\ub144\ub300 \uc911\ubc18 \uc911\uad6d\uacfc \ub9cc\uc8fc\ub85c\uc758 \ub300\ub959\uce68\ub7b5\uc774 \uc774\ub8e8\uc5b4\uc9c0\uba74\uc11c \uc870\uc120\uc758 \ubcd1\ucc38\uae30\uc9c0\uc801 \uacf5\uc5c5\ud654\uac00 \uac00\uc18d\ud654\ub418\uc790 \uae30\uacc4\uacfc, \ud1a0\ubaa9\uacfc, \uac74\ucd95\uacfc\uac00 \ud06c\uac8c \uc778\uae30\ub97c \ub04c\uc5c8\uc73c\uba70, \uc870\uc120 \uac01\uc9c0\uc640 \ub9cc\uc8fc\uad6d\uc758 \uad11\uc0b0\uc774 \uac1c\ubc1c\ub418\uba74\uc11c \uad11\uc0b0 \uacbd\uc601\uc790 \uc218\uc694\uac00 \uae09\uc99d\ud558\uc790 \uad11\uc0b0\uacfc\uc758 \uc778\uae30\ub3c4 \ub192\uc544\uc84c\ub2e4. \uad11\uc5c5\ubd84\uc57c \uc218\uc694\uac00 \ud3ed\uc99d\ud558\uc5ec \uacbd\uc131\uace0\ub4f1\uacf5\uc5c5\ud559\uad50\uc758 \uad11\uc0b0\uacfc\ub85c\ub294 \uc218\uc694\ub97c \uac10\ub2f9\ud560 \uc218 \uc5c6\uac8c \ub418\uc790, 1939\ub144\uc5d0\ub294 \uad11\uc0b0\uacfc\ub97c \ubd84\ub9ac\ud558\uc5ec \uacbd\uc131\uad11\uc0b0\uc804\ubb38\ud559\uad50\ub97c \uc124\ub9bd\ud558\uc600\ub2e4. 1941\ub144\uc5d0\ub294 \uc774\uacfc\uc591\uc131\uc18c(\u7406\u79d1\u990a\u6210\u6240)\uac00 \ubd80\uc124\ub418\uc5b4 \uc218\ud559, \ubb3c\uc0c1, \ud654\ud559, \uc0dd\ubb3c \uc911\ub4f1\uad50\uc0ac\ub97c \uc591\uc131\ud558\uae30\ub3c4 \ud588\ub2e4. 1944\ub144\uc5d0\ub294 \uad50\uba85\uc774 \uacbd\uc131\uacf5\uc5c5\uc804\ubb38\ud559\uad50\ub85c \ubc14\ub00c\uc5c8\uc73c\uba70, \uc6d0\ub3d9\uae30\uacfc\uc640 \uc804\uae30\ud654\ud559\uacfc, \uc804\uae30\ud1b5\uc2e0\uacfc\uac00 \ucd94\uac00 \uc124\uce58\ub418\uc5c8\ub2e4.", "answer": "\uacbd\uc131\uad11\uc0b0\uc804\ubb38\ud559\uad50", "sentence": "\uad11\uc5c5\ubd84\uc57c \uc218\uc694\uac00 \ud3ed\uc99d\ud558\uc5ec \uacbd\uc131\uace0\ub4f1\uacf5\uc5c5\ud559\uad50\uc758 \uad11\uc0b0\uacfc\ub85c\ub294 \uc218\uc694\ub97c \uac10\ub2f9\ud560 \uc218 \uc5c6\uac8c \ub418\uc790, 1939\ub144\uc5d0\ub294 \uad11\uc0b0\uacfc\ub97c \ubd84\ub9ac\ud558\uc5ec \uacbd\uc131\uad11\uc0b0\uc804\ubb38\ud559\uad50 \ub97c \uc124\ub9bd\ud558\uc600\ub2e4.", "paragraph_sentence": "\uacbd\uc131\uace0\ub4f1\uacf5\uc5c5\ud559\uad50\uc758 \uad50\uc721\uc5f0\ud55c\uc740 3\ub144\uc774\uc5c8\uace0, \ud559\uacfc\ub85c\ub294 \ubc29\uc9c1\uacfc, \ud1a0\ubaa9\uacfc, \uac74\ucd95\uacfc, \uad11\uc0b0\uacfc, \uc751\uc6a9\ud654\ud559\uacfc\uac00 \uc124\uce58\ub418\uc5b4 \uc788\uc5c8\uc73c\uba70, \uc751\uc6a9\ud654\ud559\uacfc\uc5d0\ub294 \uc694\uc5c5, \uc5fc\uc0c9, \uc751\uc6a9\ud654\ud559\uc758 \uc138\uacfc(\u7d30\u79d1)\uac00 \uc788\uc5c8\ub2e4. 1937\ub144\uc5d0\ub294 \uc804\uae30\uacf5\ud559\uacfc\uc640 \uae30\uacc4\uacfc\uac00 \uac00\uc124\ub418\uc5c8\ub2e4. 1930\ub144\ub300 \uc911\ubc18 \uc911\uad6d\uacfc \ub9cc\uc8fc\ub85c\uc758 \ub300\ub959\uce68\ub7b5\uc774 \uc774\ub8e8\uc5b4\uc9c0\uba74\uc11c \uc870\uc120\uc758 \ubcd1\ucc38\uae30\uc9c0\uc801 \uacf5\uc5c5\ud654\uac00 \uac00\uc18d\ud654\ub418\uc790 \uae30\uacc4\uacfc, \ud1a0\ubaa9\uacfc, \uac74\ucd95\uacfc\uac00 \ud06c\uac8c \uc778\uae30\ub97c \ub04c\uc5c8\uc73c\uba70, \uc870\uc120 \uac01\uc9c0\uc640 \ub9cc\uc8fc\uad6d\uc758 \uad11\uc0b0\uc774 \uac1c\ubc1c\ub418\uba74\uc11c \uad11\uc0b0 \uacbd\uc601\uc790 \uc218\uc694\uac00 \uae09\uc99d\ud558\uc790 \uad11\uc0b0\uacfc\uc758 \uc778\uae30\ub3c4 \ub192\uc544\uc84c\ub2e4. \uad11\uc5c5\ubd84\uc57c \uc218\uc694\uac00 \ud3ed\uc99d\ud558\uc5ec \uacbd\uc131\uace0\ub4f1\uacf5\uc5c5\ud559\uad50\uc758 \uad11\uc0b0\uacfc\ub85c\ub294 \uc218\uc694\ub97c \uac10\ub2f9\ud560 \uc218 \uc5c6\uac8c \ub418\uc790, 1939\ub144\uc5d0\ub294 \uad11\uc0b0\uacfc\ub97c \ubd84\ub9ac\ud558\uc5ec \uacbd\uc131\uad11\uc0b0\uc804\ubb38\ud559\uad50 \ub97c \uc124\ub9bd\ud558\uc600\ub2e4. 1941\ub144\uc5d0\ub294 \uc774\uacfc\uc591\uc131\uc18c(\u7406\u79d1\u990a\u6210\u6240)\uac00 \ubd80\uc124\ub418\uc5b4 \uc218\ud559, \ubb3c\uc0c1, \ud654\ud559, \uc0dd\ubb3c \uc911\ub4f1\uad50\uc0ac\ub97c \uc591\uc131\ud558\uae30\ub3c4 \ud588\ub2e4. 1944\ub144\uc5d0\ub294 \uad50\uba85\uc774 \uacbd\uc131\uacf5\uc5c5\uc804\ubb38\ud559\uad50\ub85c \ubc14\ub00c\uc5c8\uc73c\uba70, \uc6d0\ub3d9\uae30\uacfc\uc640 \uc804\uae30\ud654\ud559\uacfc, \uc804\uae30\ud1b5\uc2e0\uacfc\uac00 \ucd94\uac00 \uc124\uce58\ub418\uc5c8\ub2e4.", "paragraph_answer": "\uacbd\uc131\uace0\ub4f1\uacf5\uc5c5\ud559\uad50\uc758 \uad50\uc721\uc5f0\ud55c\uc740 3\ub144\uc774\uc5c8\uace0, \ud559\uacfc\ub85c\ub294 \ubc29\uc9c1\uacfc, \ud1a0\ubaa9\uacfc, \uac74\ucd95\uacfc, \uad11\uc0b0\uacfc, \uc751\uc6a9\ud654\ud559\uacfc\uac00 \uc124\uce58\ub418\uc5b4 \uc788\uc5c8\uc73c\uba70, \uc751\uc6a9\ud654\ud559\uacfc\uc5d0\ub294 \uc694\uc5c5, \uc5fc\uc0c9, \uc751\uc6a9\ud654\ud559\uc758 \uc138\uacfc(\u7d30\u79d1)\uac00 \uc788\uc5c8\ub2e4. 1937\ub144\uc5d0\ub294 \uc804\uae30\uacf5\ud559\uacfc\uc640 \uae30\uacc4\uacfc\uac00 \uac00\uc124\ub418\uc5c8\ub2e4. 1930\ub144\ub300 \uc911\ubc18 \uc911\uad6d\uacfc \ub9cc\uc8fc\ub85c\uc758 \ub300\ub959\uce68\ub7b5\uc774 \uc774\ub8e8\uc5b4\uc9c0\uba74\uc11c \uc870\uc120\uc758 \ubcd1\ucc38\uae30\uc9c0\uc801 \uacf5\uc5c5\ud654\uac00 \uac00\uc18d\ud654\ub418\uc790 \uae30\uacc4\uacfc, \ud1a0\ubaa9\uacfc, \uac74\ucd95\uacfc\uac00 \ud06c\uac8c \uc778\uae30\ub97c \ub04c\uc5c8\uc73c\uba70, \uc870\uc120 \uac01\uc9c0\uc640 \ub9cc\uc8fc\uad6d\uc758 \uad11\uc0b0\uc774 \uac1c\ubc1c\ub418\uba74\uc11c \uad11\uc0b0 \uacbd\uc601\uc790 \uc218\uc694\uac00 \uae09\uc99d\ud558\uc790 \uad11\uc0b0\uacfc\uc758 \uc778\uae30\ub3c4 \ub192\uc544\uc84c\ub2e4. \uad11\uc5c5\ubd84\uc57c \uc218\uc694\uac00 \ud3ed\uc99d\ud558\uc5ec \uacbd\uc131\uace0\ub4f1\uacf5\uc5c5\ud559\uad50\uc758 \uad11\uc0b0\uacfc\ub85c\ub294 \uc218\uc694\ub97c \uac10\ub2f9\ud560 \uc218 \uc5c6\uac8c \ub418\uc790, 1939\ub144\uc5d0\ub294 \uad11\uc0b0\uacfc\ub97c \ubd84\ub9ac\ud558\uc5ec \uacbd\uc131\uad11\uc0b0\uc804\ubb38\ud559\uad50 \ub97c \uc124\ub9bd\ud558\uc600\ub2e4. 1941\ub144\uc5d0\ub294 \uc774\uacfc\uc591\uc131\uc18c(\u7406\u79d1\u990a\u6210\u6240)\uac00 \ubd80\uc124\ub418\uc5b4 \uc218\ud559, \ubb3c\uc0c1, \ud654\ud559, \uc0dd\ubb3c \uc911\ub4f1\uad50\uc0ac\ub97c \uc591\uc131\ud558\uae30\ub3c4 \ud588\ub2e4. 1944\ub144\uc5d0\ub294 \uad50\uba85\uc774 \uacbd\uc131\uacf5\uc5c5\uc804\ubb38\ud559\uad50\ub85c \ubc14\ub00c\uc5c8\uc73c\uba70, \uc6d0\ub3d9\uae30\uacfc\uc640 \uc804\uae30\ud654\ud559\uacfc, \uc804\uae30\ud1b5\uc2e0\uacfc\uac00 \ucd94\uac00 \uc124\uce58\ub418\uc5c8\ub2e4.", "sentence_answer": "\uad11\uc5c5\ubd84\uc57c \uc218\uc694\uac00 \ud3ed\uc99d\ud558\uc5ec \uacbd\uc131\uace0\ub4f1\uacf5\uc5c5\ud559\uad50\uc758 \uad11\uc0b0\uacfc\ub85c\ub294 \uc218\uc694\ub97c \uac10\ub2f9\ud560 \uc218 \uc5c6\uac8c \ub418\uc790, 1939\ub144\uc5d0\ub294 \uad11\uc0b0\uacfc\ub97c \ubd84\ub9ac\ud558\uc5ec \uacbd\uc131\uad11\uc0b0\uc804\ubb38\ud559\uad50 \ub97c \uc124\ub9bd\ud558\uc600\ub2e4.", "paragraph_id": "6473065-3-0", "paragraph_question": "question: 1939\ub144 \uad11\uc0b0\uacfc\ub97c \ubd84\ub9ac\ud558\uc5ec \uc124\ub9bd\ud558\uac8c \ub41c \ud559\uad50\uba85\uc740?, context: \uacbd\uc131\uace0\ub4f1\uacf5\uc5c5\ud559\uad50\uc758 \uad50\uc721\uc5f0\ud55c\uc740 3\ub144\uc774\uc5c8\uace0, \ud559\uacfc\ub85c\ub294 \ubc29\uc9c1\uacfc, \ud1a0\ubaa9\uacfc, \uac74\ucd95\uacfc, \uad11\uc0b0\uacfc, \uc751\uc6a9\ud654\ud559\uacfc\uac00 \uc124\uce58\ub418\uc5b4 \uc788\uc5c8\uc73c\uba70, \uc751\uc6a9\ud654\ud559\uacfc\uc5d0\ub294 \uc694\uc5c5, \uc5fc\uc0c9, \uc751\uc6a9\ud654\ud559\uc758 \uc138\uacfc(\u7d30\u79d1)\uac00 \uc788\uc5c8\ub2e4. 1937\ub144\uc5d0\ub294 \uc804\uae30\uacf5\ud559\uacfc\uc640 \uae30\uacc4\uacfc\uac00 \uac00\uc124\ub418\uc5c8\ub2e4. 1930\ub144\ub300 \uc911\ubc18 \uc911\uad6d\uacfc \ub9cc\uc8fc\ub85c\uc758 \ub300\ub959\uce68\ub7b5\uc774 \uc774\ub8e8\uc5b4\uc9c0\uba74\uc11c \uc870\uc120\uc758 \ubcd1\ucc38\uae30\uc9c0\uc801 \uacf5\uc5c5\ud654\uac00 \uac00\uc18d\ud654\ub418\uc790 \uae30\uacc4\uacfc, \ud1a0\ubaa9\uacfc, \uac74\ucd95\uacfc\uac00 \ud06c\uac8c \uc778\uae30\ub97c \ub04c\uc5c8\uc73c\uba70, \uc870\uc120 \uac01\uc9c0\uc640 \ub9cc\uc8fc\uad6d\uc758 \uad11\uc0b0\uc774 \uac1c\ubc1c\ub418\uba74\uc11c \uad11\uc0b0 \uacbd\uc601\uc790 \uc218\uc694\uac00 \uae09\uc99d\ud558\uc790 \uad11\uc0b0\uacfc\uc758 \uc778\uae30\ub3c4 \ub192\uc544\uc84c\ub2e4. \uad11\uc5c5\ubd84\uc57c \uc218\uc694\uac00 \ud3ed\uc99d\ud558\uc5ec \uacbd\uc131\uace0\ub4f1\uacf5\uc5c5\ud559\uad50\uc758 \uad11\uc0b0\uacfc\ub85c\ub294 \uc218\uc694\ub97c \uac10\ub2f9\ud560 \uc218 \uc5c6\uac8c \ub418\uc790, 1939\ub144\uc5d0\ub294 \uad11\uc0b0\uacfc\ub97c \ubd84\ub9ac\ud558\uc5ec \uacbd\uc131\uad11\uc0b0\uc804\ubb38\ud559\uad50\ub97c \uc124\ub9bd\ud558\uc600\ub2e4. 1941\ub144\uc5d0\ub294 \uc774\uacfc\uc591\uc131\uc18c(\u7406\u79d1\u990a\u6210\u6240)\uac00 \ubd80\uc124\ub418\uc5b4 \uc218\ud559, \ubb3c\uc0c1, \ud654\ud559, \uc0dd\ubb3c \uc911\ub4f1\uad50\uc0ac\ub97c \uc591\uc131\ud558\uae30\ub3c4 \ud588\ub2e4. 1944\ub144\uc5d0\ub294 \uad50\uba85\uc774 \uacbd\uc131\uacf5\uc5c5\uc804\ubb38\ud559\uad50\ub85c \ubc14\ub00c\uc5c8\uc73c\uba70, \uc6d0\ub3d9\uae30\uacfc\uc640 \uc804\uae30\ud654\ud559\uacfc, \uc804\uae30\ud1b5\uc2e0\uacfc\uac00 \ucd94\uac00 \uc124\uce58\ub418\uc5c8\ub2e4."} {"question": "Just the way you are \uc568\ubc94\uc740 \uc5bc\ub9c8\ub098 \ub9ce\uc774 \ud314\ub838\ub098?", "paragraph": "\ube0c\ub8e8\ub178\ub294 \ubaa8\ud0c0\uc6b4 \ub808\ucf54\ub4dc\uc640\uc758 \uacc4\uc57d\uc5d0 \uc2e4\ud328\ud558\uc600\uc9c0\ub9cc, 2009\ub144 \uc560\ud2c0\ub79c\ud2f1 \ub808\ucf54\ub4dc\uc640 \uacc4\uc57d\uc744 \uccb4\uacb0\ud558\uc600\ub2e4. B.o.B\uc758 \ub178\ub798 \"Nothin' on You\"\uc640 \ud2b8\ub798\ube44 \ub9e4\ucf54\uc774\uc758 \ub178\ub798 \"Billionaire\" \ud53c\ucc98\ub9c1\uacfc \uc791\uc5c5\uc744 \uac19\uc774\ud558\uc600\ub2e4. \ub610\ud55c, \ube0c\ub8e8\ub178\ub294 \ucf00\uc0e4\uac00 \ud53c\ucc98\ub9c1\ud55c \ud50c\ub85c \ub77c\uc774\ub354\uc758 \ub178\ub798 \"Right Round\"\uc640 \ucf00\uc774\ub09c\uc758 \ub178\ub798 \"Wavin' Flag\"\uc5d0\ub3c4 \uacf5\ub3d9\uc73c\ub85c \uc791\uc5c5\uc5d0 \ucc38\uc5ec\ud558\uc600\ub2e4. 2010\ub144 \ub370\ubdd4 \uc568\ubc94 Doo-Wops & Hooligans\uac00 \ubc1c\ub9e4\ub418\uc5c8\uc73c\uba70, \ube4c\ubcf4\ub4dc 200 3\uc704\uc5d0 \uc62c\ub790\ub2e4. \ucc28\ub840\ub85c \ubc1c\ub9e4\ub41c \uc2f1\uae00 \"Just the Way You Are\"\uc640 \"Grenade\"\ub294 \ube4c\ubcf4\ub4dc \ud56b 100\uc5d0\uc11c 1\uc704\ub97c \ud558\uc600\uc73c\uba70, \"The Lazy Song\" \ub610\ud55c \ud06c\uac8c \ud788\ud2b8\ub97c \ucce4\ub2e4. \"Just the Way You Are\"\ub294 1,250\ub9cc\uc7a5\uc758 \ud310\ub9e4\uace0\ub97c \uc62c\ub838\uc73c\uba70, 2011\ub144\uc5d0 \uac00\uc7a5 \ub9ce\uc774 \ud314\ub9b0 \ub514\uc9c0\ud138 \uc544\ud2f0\uc2a4\ud2b8\uac00 \ub418\uc5c8\ub2e4. 2\uc9d1 \uc74c\ubc18 Unorthodox Jukebox\ub294 2012\ub144\uc5d0 \ubc1c\ub9e4\ub418\uc5c8\uc73c\uba70, \uc774 \uc74c\ubc18 \ub610\ud55c \ubbf8\uad6d\uc5d0\uc11c 1\uc704\ub97c \ud558\uc600\ub2e4. \uc774 \uc74c\ubc18\uc5d0\uc11c \ubc1c\ub9e4\ub41c \uc2f1\uae00 \"Locked Out of Heaven\"\uacfc \"When I Was Your Man\"\uc740 \ube4c\ubcf4\ub4dc \ud56b 100\uc5d0\uc11c 1\uc704\ub97c \ud558\uc600\uc73c\uba70, \uad6d\uc81c\uc801\uc73c\ub85c\ub3c4 \ud06c\uac8c \ud788\ud2b8\ub97c \ucce4\ub2e4. \uac8c\ub2e4\uac00, \ube0c\ub8e8\ub178\uac00 \ucc38\uc5ec\ud55c \ub178\ub798\uc5d0\uc11c\ub3c4 \ud06c\uac8c \ud788\ud2b8\uce5c \ub178\ub798\uac00 \ub9ce\uc740 \ub370, \uadf8 \uc608\ub85c\ub294 \ubc30\ub4dc \ubbf8\uce20 \uc774\ube14\uc758 \"Lighters\", \uc2a4\ub215 \ub3c5 \ubc0f \uc704\uc988 \uce7c\ub9ac\ud30c\uc758 \"Young, Wild & Free\" \ub4f1\uc774 \uc788\ub2e4.", "answer": "1,250\ub9cc\uc7a5", "sentence": "\"Just the Way You Are\"\ub294 1,250\ub9cc\uc7a5 \uc758 \ud310\ub9e4\uace0\ub97c \uc62c\ub838\uc73c\uba70, 2011\ub144\uc5d0 \uac00\uc7a5 \ub9ce\uc774 \ud314\ub9b0 \ub514\uc9c0\ud138 \uc544\ud2f0\uc2a4\ud2b8\uac00 \ub418\uc5c8\ub2e4.", "paragraph_sentence": "\ube0c\ub8e8\ub178\ub294 \ubaa8\ud0c0\uc6b4 \ub808\ucf54\ub4dc\uc640\uc758 \uacc4\uc57d\uc5d0 \uc2e4\ud328\ud558\uc600\uc9c0\ub9cc, 2009\ub144 \uc560\ud2c0\ub79c\ud2f1 \ub808\ucf54\ub4dc\uc640 \uacc4\uc57d\uc744 \uccb4\uacb0\ud558\uc600\ub2e4. B.o. B\uc758 \ub178\ub798 \"Nothin' on You\"\uc640 \ud2b8\ub798\ube44 \ub9e4\ucf54\uc774\uc758 \ub178\ub798 \"Billionaire\" \ud53c\ucc98\ub9c1\uacfc \uc791\uc5c5\uc744 \uac19\uc774\ud558\uc600\ub2e4. \ub610\ud55c, \ube0c\ub8e8\ub178\ub294 \ucf00\uc0e4\uac00 \ud53c\ucc98\ub9c1\ud55c \ud50c\ub85c \ub77c\uc774\ub354\uc758 \ub178\ub798 \"Right Round\"\uc640 \ucf00\uc774\ub09c\uc758 \ub178\ub798 \"Wavin' Flag\"\uc5d0\ub3c4 \uacf5\ub3d9\uc73c\ub85c \uc791\uc5c5\uc5d0 \ucc38\uc5ec\ud558\uc600\ub2e4. 2010\ub144 \ub370\ubdd4 \uc568\ubc94 Doo-Wops & Hooligans\uac00 \ubc1c\ub9e4\ub418\uc5c8\uc73c\uba70, \ube4c\ubcf4\ub4dc 200 3\uc704\uc5d0 \uc62c\ub790\ub2e4. \ucc28\ub840\ub85c \ubc1c\ub9e4\ub41c \uc2f1\uae00 \"Just the Way You Are\"\uc640 \"Grenade\"\ub294 \ube4c\ubcf4\ub4dc \ud56b 100\uc5d0\uc11c 1\uc704\ub97c \ud558\uc600\uc73c\uba70, \"The Lazy Song\" \ub610\ud55c \ud06c\uac8c \ud788\ud2b8\ub97c \ucce4\ub2e4. \"Just the Way You Are\"\ub294 1,250\ub9cc\uc7a5 \uc758 \ud310\ub9e4\uace0\ub97c \uc62c\ub838\uc73c\uba70, 2011\ub144\uc5d0 \uac00\uc7a5 \ub9ce\uc774 \ud314\ub9b0 \ub514\uc9c0\ud138 \uc544\ud2f0\uc2a4\ud2b8\uac00 \ub418\uc5c8\ub2e4. 2\uc9d1 \uc74c\ubc18 Unorthodox Jukebox\ub294 2012\ub144\uc5d0 \ubc1c\ub9e4\ub418\uc5c8\uc73c\uba70, \uc774 \uc74c\ubc18 \ub610\ud55c \ubbf8\uad6d\uc5d0\uc11c 1\uc704\ub97c \ud558\uc600\ub2e4. \uc774 \uc74c\ubc18\uc5d0\uc11c \ubc1c\ub9e4\ub41c \uc2f1\uae00 \"Locked Out of Heaven\"\uacfc \"When I Was Your Man\"\uc740 \ube4c\ubcf4\ub4dc \ud56b 100\uc5d0\uc11c 1\uc704\ub97c \ud558\uc600\uc73c\uba70, \uad6d\uc81c\uc801\uc73c\ub85c\ub3c4 \ud06c\uac8c \ud788\ud2b8\ub97c \ucce4\ub2e4. \uac8c\ub2e4\uac00, \ube0c\ub8e8\ub178\uac00 \ucc38\uc5ec\ud55c \ub178\ub798\uc5d0\uc11c\ub3c4 \ud06c\uac8c \ud788\ud2b8\uce5c \ub178\ub798\uac00 \ub9ce\uc740 \ub370, \uadf8 \uc608\ub85c\ub294 \ubc30\ub4dc \ubbf8\uce20 \uc774\ube14\uc758 \"Lighters\", \uc2a4\ub215 \ub3c5 \ubc0f \uc704\uc988 \uce7c\ub9ac\ud30c\uc758 \"Young, Wild & Free\" \ub4f1\uc774 \uc788\ub2e4.", "paragraph_answer": "\ube0c\ub8e8\ub178\ub294 \ubaa8\ud0c0\uc6b4 \ub808\ucf54\ub4dc\uc640\uc758 \uacc4\uc57d\uc5d0 \uc2e4\ud328\ud558\uc600\uc9c0\ub9cc, 2009\ub144 \uc560\ud2c0\ub79c\ud2f1 \ub808\ucf54\ub4dc\uc640 \uacc4\uc57d\uc744 \uccb4\uacb0\ud558\uc600\ub2e4. B.o.B\uc758 \ub178\ub798 \"Nothin' on You\"\uc640 \ud2b8\ub798\ube44 \ub9e4\ucf54\uc774\uc758 \ub178\ub798 \"Billionaire\" \ud53c\ucc98\ub9c1\uacfc \uc791\uc5c5\uc744 \uac19\uc774\ud558\uc600\ub2e4. \ub610\ud55c, \ube0c\ub8e8\ub178\ub294 \ucf00\uc0e4\uac00 \ud53c\ucc98\ub9c1\ud55c \ud50c\ub85c \ub77c\uc774\ub354\uc758 \ub178\ub798 \"Right Round\"\uc640 \ucf00\uc774\ub09c\uc758 \ub178\ub798 \"Wavin' Flag\"\uc5d0\ub3c4 \uacf5\ub3d9\uc73c\ub85c \uc791\uc5c5\uc5d0 \ucc38\uc5ec\ud558\uc600\ub2e4. 2010\ub144 \ub370\ubdd4 \uc568\ubc94 Doo-Wops & Hooligans\uac00 \ubc1c\ub9e4\ub418\uc5c8\uc73c\uba70, \ube4c\ubcf4\ub4dc 200 3\uc704\uc5d0 \uc62c\ub790\ub2e4. \ucc28\ub840\ub85c \ubc1c\ub9e4\ub41c \uc2f1\uae00 \"Just the Way You Are\"\uc640 \"Grenade\"\ub294 \ube4c\ubcf4\ub4dc \ud56b 100\uc5d0\uc11c 1\uc704\ub97c \ud558\uc600\uc73c\uba70, \"The Lazy Song\" \ub610\ud55c \ud06c\uac8c \ud788\ud2b8\ub97c \ucce4\ub2e4. \"Just the Way You Are\"\ub294 1,250\ub9cc\uc7a5 \uc758 \ud310\ub9e4\uace0\ub97c \uc62c\ub838\uc73c\uba70, 2011\ub144\uc5d0 \uac00\uc7a5 \ub9ce\uc774 \ud314\ub9b0 \ub514\uc9c0\ud138 \uc544\ud2f0\uc2a4\ud2b8\uac00 \ub418\uc5c8\ub2e4. 2\uc9d1 \uc74c\ubc18 Unorthodox Jukebox\ub294 2012\ub144\uc5d0 \ubc1c\ub9e4\ub418\uc5c8\uc73c\uba70, \uc774 \uc74c\ubc18 \ub610\ud55c \ubbf8\uad6d\uc5d0\uc11c 1\uc704\ub97c \ud558\uc600\ub2e4. \uc774 \uc74c\ubc18\uc5d0\uc11c \ubc1c\ub9e4\ub41c \uc2f1\uae00 \"Locked Out of Heaven\"\uacfc \"When I Was Your Man\"\uc740 \ube4c\ubcf4\ub4dc \ud56b 100\uc5d0\uc11c 1\uc704\ub97c \ud558\uc600\uc73c\uba70, \uad6d\uc81c\uc801\uc73c\ub85c\ub3c4 \ud06c\uac8c \ud788\ud2b8\ub97c \ucce4\ub2e4. \uac8c\ub2e4\uac00, \ube0c\ub8e8\ub178\uac00 \ucc38\uc5ec\ud55c \ub178\ub798\uc5d0\uc11c\ub3c4 \ud06c\uac8c \ud788\ud2b8\uce5c \ub178\ub798\uac00 \ub9ce\uc740 \ub370, \uadf8 \uc608\ub85c\ub294 \ubc30\ub4dc \ubbf8\uce20 \uc774\ube14\uc758 \"Lighters\", \uc2a4\ub215 \ub3c5 \ubc0f \uc704\uc988 \uce7c\ub9ac\ud30c\uc758 \"Young, Wild & Free\" \ub4f1\uc774 \uc788\ub2e4.", "sentence_answer": "\"Just the Way You Are\"\ub294 1,250\ub9cc\uc7a5 \uc758 \ud310\ub9e4\uace0\ub97c \uc62c\ub838\uc73c\uba70, 2011\ub144\uc5d0 \uac00\uc7a5 \ub9ce\uc774 \ud314\ub9b0 \ub514\uc9c0\ud138 \uc544\ud2f0\uc2a4\ud2b8\uac00 \ub418\uc5c8\ub2e4.", "paragraph_id": "6535231-0-2", "paragraph_question": "question: Just the way you are \uc568\ubc94\uc740 \uc5bc\ub9c8\ub098 \ub9ce\uc774 \ud314\ub838\ub098?, context: \ube0c\ub8e8\ub178\ub294 \ubaa8\ud0c0\uc6b4 \ub808\ucf54\ub4dc\uc640\uc758 \uacc4\uc57d\uc5d0 \uc2e4\ud328\ud558\uc600\uc9c0\ub9cc, 2009\ub144 \uc560\ud2c0\ub79c\ud2f1 \ub808\ucf54\ub4dc\uc640 \uacc4\uc57d\uc744 \uccb4\uacb0\ud558\uc600\ub2e4. B.o.B\uc758 \ub178\ub798 \"Nothin' on You\"\uc640 \ud2b8\ub798\ube44 \ub9e4\ucf54\uc774\uc758 \ub178\ub798 \"Billionaire\" \ud53c\ucc98\ub9c1\uacfc \uc791\uc5c5\uc744 \uac19\uc774\ud558\uc600\ub2e4. \ub610\ud55c, \ube0c\ub8e8\ub178\ub294 \ucf00\uc0e4\uac00 \ud53c\ucc98\ub9c1\ud55c \ud50c\ub85c \ub77c\uc774\ub354\uc758 \ub178\ub798 \"Right Round\"\uc640 \ucf00\uc774\ub09c\uc758 \ub178\ub798 \"Wavin' Flag\"\uc5d0\ub3c4 \uacf5\ub3d9\uc73c\ub85c \uc791\uc5c5\uc5d0 \ucc38\uc5ec\ud558\uc600\ub2e4. 2010\ub144 \ub370\ubdd4 \uc568\ubc94 Doo-Wops & Hooligans\uac00 \ubc1c\ub9e4\ub418\uc5c8\uc73c\uba70, \ube4c\ubcf4\ub4dc 200 3\uc704\uc5d0 \uc62c\ub790\ub2e4. \ucc28\ub840\ub85c \ubc1c\ub9e4\ub41c \uc2f1\uae00 \"Just the Way You Are\"\uc640 \"Grenade\"\ub294 \ube4c\ubcf4\ub4dc \ud56b 100\uc5d0\uc11c 1\uc704\ub97c \ud558\uc600\uc73c\uba70, \"The Lazy Song\" \ub610\ud55c \ud06c\uac8c \ud788\ud2b8\ub97c \ucce4\ub2e4. \"Just the Way You Are\"\ub294 1,250\ub9cc\uc7a5\uc758 \ud310\ub9e4\uace0\ub97c \uc62c\ub838\uc73c\uba70, 2011\ub144\uc5d0 \uac00\uc7a5 \ub9ce\uc774 \ud314\ub9b0 \ub514\uc9c0\ud138 \uc544\ud2f0\uc2a4\ud2b8\uac00 \ub418\uc5c8\ub2e4. 2\uc9d1 \uc74c\ubc18 Unorthodox Jukebox\ub294 2012\ub144\uc5d0 \ubc1c\ub9e4\ub418\uc5c8\uc73c\uba70, \uc774 \uc74c\ubc18 \ub610\ud55c \ubbf8\uad6d\uc5d0\uc11c 1\uc704\ub97c \ud558\uc600\ub2e4. \uc774 \uc74c\ubc18\uc5d0\uc11c \ubc1c\ub9e4\ub41c \uc2f1\uae00 \"Locked Out of Heaven\"\uacfc \"When I Was Your Man\"\uc740 \ube4c\ubcf4\ub4dc \ud56b 100\uc5d0\uc11c 1\uc704\ub97c \ud558\uc600\uc73c\uba70, \uad6d\uc81c\uc801\uc73c\ub85c\ub3c4 \ud06c\uac8c \ud788\ud2b8\ub97c \ucce4\ub2e4. \uac8c\ub2e4\uac00, \ube0c\ub8e8\ub178\uac00 \ucc38\uc5ec\ud55c \ub178\ub798\uc5d0\uc11c\ub3c4 \ud06c\uac8c \ud788\ud2b8\uce5c \ub178\ub798\uac00 \ub9ce\uc740 \ub370, \uadf8 \uc608\ub85c\ub294 \ubc30\ub4dc \ubbf8\uce20 \uc774\ube14\uc758 \"Lighters\", \uc2a4\ub215 \ub3c5 \ubc0f \uc704\uc988 \uce7c\ub9ac\ud30c\uc758 \"Young, Wild & Free\" \ub4f1\uc774 \uc788\ub2e4."} {"question": "\ube44\ub8e8\uc2a4\ub294 \ub204\uad6c\uc640 \uc30d\ub465\uc774\uc778\uac00?", "paragraph": "\ud504\ub9ac\uc800\uc640\uc758 \uc7ac\ub300\uacb0 \uc774\ud6c4, \uc624\uacf5\uacfc \ubc30\uc9c0\ud130\ub294 \ube44\ub8e8\uc2a4 \ud589\uc131\uc5d0\uc11c \uc218\ud589\uc744 \uacc4\uc18d\ud55c\ub2e4. \uadf8\ub7ec\ub358 \uc911 \uc774\ubbf8 \ud55c\ucc28\ub840 \ub5a1\ubc25 \ub4f1\uc7a5\ud558\uc600\ub358 \uc0f4\ud30c\uc640 \ubc14\ub85c\uc2a4\uac00 \uc774 \uacf3\uc744 \ubc29\ubb38\ud55c\ub2e4. \uc0ac\uc2e4 \ube44\ub8e8\uc2a4\uc640 \uc0f4\ud30c\ub294 \uc30d\ub465\uc774\uace0, \uc6b0\uc774\uc2a4\uc640 \ubc14\ub85c\uc2a4\ub3c4 \uc624\ub204\uc774\uc600\ub2e4. \ucd1d 12\uac1c\uc758 \uc6b0\uc8fc\uac00 \uc788\ub294\ub370, \uc11c\ub85c \ub300\uce6d\ub418\ub294 \uc6b0\uc8fc\ub07c\ub9ac \uc11c\ub85c \uc720\uc0ac\ud55c \uc874\uc7ac\ub97c \uac00\uc9c0\uace0 \uc788\uace0, \ube44\ub8e8\uc2a4\ub294 7\uc6b0\uc8fc, \uc0f4\ud30c\ub294 6\uc6b0\uc8fc\ub97c \uad00\uc7a5\ud558\ub294 \ud30c\uad34\uc2e0\uc774\ub780 \uc124\uc815\uc774\ub2e4. \uc0f4\ud30c\ub294 \ube44\ub8e8\uc2a4\uac00 \uc790\ub791\uce5c \uc9c0\uad6c\uc5d0\uc11c \uac00\uc838\uc628 \uc74c\uc2dd\ub4e4\uc744 \ubd80\ub7ec\uc6cc\ud588\ub294\ub370, \uc790\uae30 \ucabd \uc6b0\uc8fc \uc9c0\uad6c\uc5d0 \ud574\ub2f9\ud558\ub294 \ud589\uc131\uc5d0\uc120 \uc778\ub958 \ubb38\uba85\uc774 \uc774\ubbf8 \uba78\ub9dd\ud588\ub2e4\ub294 \uc0ac\uc2e4\uc744 \uc54c\uc790, 7\uc6b0\uc8fc \ucabd \uc9c0\uad6c\ub97c \uac00\uc9c0\uace0 \ub0b4\uae30\ub97c \ud558\uae30\ub85c \ud55c\ub2e4. \uc0f4\ud30c\ub294 \uc790\uc2e0\uc774 \uc288\ud37c \ub4dc\ub798\uace4\ubcfc 6\uac1c\ub97c \uc774\ubbf8 \ubaa8\uc558\ub2e4\uba74\uc11c, \ube44\ub8e8\uc2a4 \ub124\uac00 \uc774\uae30\uba74 \uc774\uac78 \uc591\ubcf4\ud560\uaebc\uace0, \uc790\uc2e0\uc774 \uc774\uae30\uba74 \uc774 \uc288\ud37c \ub4dc\ub798\uace4\ubcfc\uc758 \ud798\uc73c\ub85c \uc11c\ub85c\uc758 \uc9c0\uad6c\ub97c \ub9de\ubc14\uafb8\uaca0\ub2e8 \ub0b4\uc6a9\uc758 \ub0b4\uae30\uc600\ub2e4. \ub450 \uc0ac\ub78c\uc774 \uc9c1\uc811 \uc2f8\uc6b0\uba74 \uc6b0\uc8fc\uac00 \ud30c\uad34\ub420\uc9c0\ub3c4 \ubaa8\ub974\ub294 \uc0c1\ud669\uc774\uc5c8\uae30\uc5d0 \uac01\uc790 5\uba85\uc758 \ub300\ub9ac\uc778\uc744 \ub0b4\uc138\uc6cc \uaca9\ud22c \ub300\ud68c\ub97c \ubc8c\uc774\uae30\ub85c \ud558\uc600\ub2e4. \ube44\ub8e8\uc2a4\ub294 \ub531\ud788 \uc6d0\ud558\ub358 \uc18c\uc6d0\uc774 \uc5c6\uc5c8\uc9c0\ub9cc \uc624\uacf5\uc774 \uc6d0\ud558\uc600\uace0, \uacbd\uc7c1\uc2ec\uc5d0 \uc7ac\ubbf8\uc0bc\uc544 \uc81c\uc548\uc744 \ubc1b\uc544\ub4e4\uc774\uae30\ub85c \ud55c\ub2e4. \uc774\ud6c4 5\uba85\uc758 \ub300\ub9ac\uc778\uc740 \uc790\uc2e0\uc774 \uc18d\ud55c \uc6b0\uc8fc\ub97c \uc9c0\ud0a4\uae30 \uc704\ud574 \uaca9\ud22c \ub300\ud68c\uc5d0\uc11c \ubd84\uc804\ud55c\ub2e4. \uc774\ud6c4 \uaca9\ud22c \ub300\ud68c\uc758 \ub0b4\uc6a9\uc5d0 \ub9cc\uc871\ud55c \uacc4\uc655\uc2e0 \uc77c\ud589\uc740 \ub450 \uc6b0\uc8fc\ub97c \uc874\uc18d\uc2dc\ud0a4\uae30\ub85c \ud569\uc758\ud55c\ub2e4.", "answer": "\uc0f4\ud30c", "sentence": "\uadf8\ub7ec\ub358 \uc911 \uc774\ubbf8 \ud55c\ucc28\ub840 \ub5a1\ubc25 \ub4f1\uc7a5\ud558\uc600\ub358 \uc0f4\ud30c \uc640 \ubc14\ub85c\uc2a4\uac00 \uc774 \uacf3\uc744 \ubc29\ubb38\ud55c\ub2e4.", "paragraph_sentence": "\ud504\ub9ac\uc800\uc640\uc758 \uc7ac\ub300\uacb0 \uc774\ud6c4, \uc624\uacf5\uacfc \ubc30\uc9c0\ud130\ub294 \ube44\ub8e8\uc2a4 \ud589\uc131\uc5d0\uc11c \uc218\ud589\uc744 \uacc4\uc18d\ud55c\ub2e4. \uadf8\ub7ec\ub358 \uc911 \uc774\ubbf8 \ud55c\ucc28\ub840 \ub5a1\ubc25 \ub4f1\uc7a5\ud558\uc600\ub358 \uc0f4\ud30c \uc640 \ubc14\ub85c\uc2a4\uac00 \uc774 \uacf3\uc744 \ubc29\ubb38\ud55c\ub2e4. \uc0ac\uc2e4 \ube44\ub8e8\uc2a4\uc640 \uc0f4\ud30c\ub294 \uc30d\ub465\uc774\uace0, \uc6b0\uc774\uc2a4\uc640 \ubc14\ub85c\uc2a4\ub3c4 \uc624\ub204\uc774\uc600\ub2e4. \ucd1d 12\uac1c\uc758 \uc6b0\uc8fc\uac00 \uc788\ub294\ub370, \uc11c\ub85c \ub300\uce6d\ub418\ub294 \uc6b0\uc8fc\ub07c\ub9ac \uc11c\ub85c \uc720\uc0ac\ud55c \uc874\uc7ac\ub97c \uac00\uc9c0\uace0 \uc788\uace0, \ube44\ub8e8\uc2a4\ub294 7\uc6b0\uc8fc, \uc0f4\ud30c\ub294 6\uc6b0\uc8fc\ub97c \uad00\uc7a5\ud558\ub294 \ud30c\uad34\uc2e0\uc774\ub780 \uc124\uc815\uc774\ub2e4. \uc0f4\ud30c\ub294 \ube44\ub8e8\uc2a4\uac00 \uc790\ub791\uce5c \uc9c0\uad6c\uc5d0\uc11c \uac00\uc838\uc628 \uc74c\uc2dd\ub4e4\uc744 \ubd80\ub7ec\uc6cc\ud588\ub294\ub370, \uc790\uae30 \ucabd \uc6b0\uc8fc \uc9c0\uad6c\uc5d0 \ud574\ub2f9\ud558\ub294 \ud589\uc131\uc5d0\uc120 \uc778\ub958 \ubb38\uba85\uc774 \uc774\ubbf8 \uba78\ub9dd\ud588\ub2e4\ub294 \uc0ac\uc2e4\uc744 \uc54c\uc790, 7\uc6b0\uc8fc \ucabd \uc9c0\uad6c\ub97c \uac00\uc9c0\uace0 \ub0b4\uae30\ub97c \ud558\uae30\ub85c \ud55c\ub2e4. \uc0f4\ud30c\ub294 \uc790\uc2e0\uc774 \uc288\ud37c \ub4dc\ub798\uace4\ubcfc 6\uac1c\ub97c \uc774\ubbf8 \ubaa8\uc558\ub2e4\uba74\uc11c, \ube44\ub8e8\uc2a4 \ub124\uac00 \uc774\uae30\uba74 \uc774\uac78 \uc591\ubcf4\ud560\uaebc\uace0, \uc790\uc2e0\uc774 \uc774\uae30\uba74 \uc774 \uc288\ud37c \ub4dc\ub798\uace4\ubcfc\uc758 \ud798\uc73c\ub85c \uc11c\ub85c\uc758 \uc9c0\uad6c\ub97c \ub9de\ubc14\uafb8\uaca0\ub2e8 \ub0b4\uc6a9\uc758 \ub0b4\uae30\uc600\ub2e4. \ub450 \uc0ac\ub78c\uc774 \uc9c1\uc811 \uc2f8\uc6b0\uba74 \uc6b0\uc8fc\uac00 \ud30c\uad34\ub420\uc9c0\ub3c4 \ubaa8\ub974\ub294 \uc0c1\ud669\uc774\uc5c8\uae30\uc5d0 \uac01\uc790 5\uba85\uc758 \ub300\ub9ac\uc778\uc744 \ub0b4\uc138\uc6cc \uaca9\ud22c \ub300\ud68c\ub97c \ubc8c\uc774\uae30\ub85c \ud558\uc600\ub2e4. \ube44\ub8e8\uc2a4\ub294 \ub531\ud788 \uc6d0\ud558\ub358 \uc18c\uc6d0\uc774 \uc5c6\uc5c8\uc9c0\ub9cc \uc624\uacf5\uc774 \uc6d0\ud558\uc600\uace0, \uacbd\uc7c1\uc2ec\uc5d0 \uc7ac\ubbf8\uc0bc\uc544 \uc81c\uc548\uc744 \ubc1b\uc544\ub4e4\uc774\uae30\ub85c \ud55c\ub2e4. \uc774\ud6c4 5\uba85\uc758 \ub300\ub9ac\uc778\uc740 \uc790\uc2e0\uc774 \uc18d\ud55c \uc6b0\uc8fc\ub97c \uc9c0\ud0a4\uae30 \uc704\ud574 \uaca9\ud22c \ub300\ud68c\uc5d0\uc11c \ubd84\uc804\ud55c\ub2e4. \uc774\ud6c4 \uaca9\ud22c \ub300\ud68c\uc758 \ub0b4\uc6a9\uc5d0 \ub9cc\uc871\ud55c \uacc4\uc655\uc2e0 \uc77c\ud589\uc740 \ub450 \uc6b0\uc8fc\ub97c \uc874\uc18d\uc2dc\ud0a4\uae30\ub85c \ud569\uc758\ud55c\ub2e4.", "paragraph_answer": "\ud504\ub9ac\uc800\uc640\uc758 \uc7ac\ub300\uacb0 \uc774\ud6c4, \uc624\uacf5\uacfc \ubc30\uc9c0\ud130\ub294 \ube44\ub8e8\uc2a4 \ud589\uc131\uc5d0\uc11c \uc218\ud589\uc744 \uacc4\uc18d\ud55c\ub2e4. \uadf8\ub7ec\ub358 \uc911 \uc774\ubbf8 \ud55c\ucc28\ub840 \ub5a1\ubc25 \ub4f1\uc7a5\ud558\uc600\ub358 \uc0f4\ud30c \uc640 \ubc14\ub85c\uc2a4\uac00 \uc774 \uacf3\uc744 \ubc29\ubb38\ud55c\ub2e4. \uc0ac\uc2e4 \ube44\ub8e8\uc2a4\uc640 \uc0f4\ud30c\ub294 \uc30d\ub465\uc774\uace0, \uc6b0\uc774\uc2a4\uc640 \ubc14\ub85c\uc2a4\ub3c4 \uc624\ub204\uc774\uc600\ub2e4. \ucd1d 12\uac1c\uc758 \uc6b0\uc8fc\uac00 \uc788\ub294\ub370, \uc11c\ub85c \ub300\uce6d\ub418\ub294 \uc6b0\uc8fc\ub07c\ub9ac \uc11c\ub85c \uc720\uc0ac\ud55c \uc874\uc7ac\ub97c \uac00\uc9c0\uace0 \uc788\uace0, \ube44\ub8e8\uc2a4\ub294 7\uc6b0\uc8fc, \uc0f4\ud30c\ub294 6\uc6b0\uc8fc\ub97c \uad00\uc7a5\ud558\ub294 \ud30c\uad34\uc2e0\uc774\ub780 \uc124\uc815\uc774\ub2e4. \uc0f4\ud30c\ub294 \ube44\ub8e8\uc2a4\uac00 \uc790\ub791\uce5c \uc9c0\uad6c\uc5d0\uc11c \uac00\uc838\uc628 \uc74c\uc2dd\ub4e4\uc744 \ubd80\ub7ec\uc6cc\ud588\ub294\ub370, \uc790\uae30 \ucabd \uc6b0\uc8fc \uc9c0\uad6c\uc5d0 \ud574\ub2f9\ud558\ub294 \ud589\uc131\uc5d0\uc120 \uc778\ub958 \ubb38\uba85\uc774 \uc774\ubbf8 \uba78\ub9dd\ud588\ub2e4\ub294 \uc0ac\uc2e4\uc744 \uc54c\uc790, 7\uc6b0\uc8fc \ucabd \uc9c0\uad6c\ub97c \uac00\uc9c0\uace0 \ub0b4\uae30\ub97c \ud558\uae30\ub85c \ud55c\ub2e4. \uc0f4\ud30c\ub294 \uc790\uc2e0\uc774 \uc288\ud37c \ub4dc\ub798\uace4\ubcfc 6\uac1c\ub97c \uc774\ubbf8 \ubaa8\uc558\ub2e4\uba74\uc11c, \ube44\ub8e8\uc2a4 \ub124\uac00 \uc774\uae30\uba74 \uc774\uac78 \uc591\ubcf4\ud560\uaebc\uace0, \uc790\uc2e0\uc774 \uc774\uae30\uba74 \uc774 \uc288\ud37c \ub4dc\ub798\uace4\ubcfc\uc758 \ud798\uc73c\ub85c \uc11c\ub85c\uc758 \uc9c0\uad6c\ub97c \ub9de\ubc14\uafb8\uaca0\ub2e8 \ub0b4\uc6a9\uc758 \ub0b4\uae30\uc600\ub2e4. \ub450 \uc0ac\ub78c\uc774 \uc9c1\uc811 \uc2f8\uc6b0\uba74 \uc6b0\uc8fc\uac00 \ud30c\uad34\ub420\uc9c0\ub3c4 \ubaa8\ub974\ub294 \uc0c1\ud669\uc774\uc5c8\uae30\uc5d0 \uac01\uc790 5\uba85\uc758 \ub300\ub9ac\uc778\uc744 \ub0b4\uc138\uc6cc \uaca9\ud22c \ub300\ud68c\ub97c \ubc8c\uc774\uae30\ub85c \ud558\uc600\ub2e4. \ube44\ub8e8\uc2a4\ub294 \ub531\ud788 \uc6d0\ud558\ub358 \uc18c\uc6d0\uc774 \uc5c6\uc5c8\uc9c0\ub9cc \uc624\uacf5\uc774 \uc6d0\ud558\uc600\uace0, \uacbd\uc7c1\uc2ec\uc5d0 \uc7ac\ubbf8\uc0bc\uc544 \uc81c\uc548\uc744 \ubc1b\uc544\ub4e4\uc774\uae30\ub85c \ud55c\ub2e4. \uc774\ud6c4 5\uba85\uc758 \ub300\ub9ac\uc778\uc740 \uc790\uc2e0\uc774 \uc18d\ud55c \uc6b0\uc8fc\ub97c \uc9c0\ud0a4\uae30 \uc704\ud574 \uaca9\ud22c \ub300\ud68c\uc5d0\uc11c \ubd84\uc804\ud55c\ub2e4. \uc774\ud6c4 \uaca9\ud22c \ub300\ud68c\uc758 \ub0b4\uc6a9\uc5d0 \ub9cc\uc871\ud55c \uacc4\uc655\uc2e0 \uc77c\ud589\uc740 \ub450 \uc6b0\uc8fc\ub97c \uc874\uc18d\uc2dc\ud0a4\uae30\ub85c \ud569\uc758\ud55c\ub2e4.", "sentence_answer": "\uadf8\ub7ec\ub358 \uc911 \uc774\ubbf8 \ud55c\ucc28\ub840 \ub5a1\ubc25 \ub4f1\uc7a5\ud558\uc600\ub358 \uc0f4\ud30c \uc640 \ubc14\ub85c\uc2a4\uac00 \uc774 \uacf3\uc744 \ubc29\ubb38\ud55c\ub2e4.", "paragraph_id": "6513084-4-0", "paragraph_question": "question: \ube44\ub8e8\uc2a4\ub294 \ub204\uad6c\uc640 \uc30d\ub465\uc774\uc778\uac00?, context: \ud504\ub9ac\uc800\uc640\uc758 \uc7ac\ub300\uacb0 \uc774\ud6c4, \uc624\uacf5\uacfc \ubc30\uc9c0\ud130\ub294 \ube44\ub8e8\uc2a4 \ud589\uc131\uc5d0\uc11c \uc218\ud589\uc744 \uacc4\uc18d\ud55c\ub2e4. \uadf8\ub7ec\ub358 \uc911 \uc774\ubbf8 \ud55c\ucc28\ub840 \ub5a1\ubc25 \ub4f1\uc7a5\ud558\uc600\ub358 \uc0f4\ud30c\uc640 \ubc14\ub85c\uc2a4\uac00 \uc774 \uacf3\uc744 \ubc29\ubb38\ud55c\ub2e4. \uc0ac\uc2e4 \ube44\ub8e8\uc2a4\uc640 \uc0f4\ud30c\ub294 \uc30d\ub465\uc774\uace0, \uc6b0\uc774\uc2a4\uc640 \ubc14\ub85c\uc2a4\ub3c4 \uc624\ub204\uc774\uc600\ub2e4. \ucd1d 12\uac1c\uc758 \uc6b0\uc8fc\uac00 \uc788\ub294\ub370, \uc11c\ub85c \ub300\uce6d\ub418\ub294 \uc6b0\uc8fc\ub07c\ub9ac \uc11c\ub85c \uc720\uc0ac\ud55c \uc874\uc7ac\ub97c \uac00\uc9c0\uace0 \uc788\uace0, \ube44\ub8e8\uc2a4\ub294 7\uc6b0\uc8fc, \uc0f4\ud30c\ub294 6\uc6b0\uc8fc\ub97c \uad00\uc7a5\ud558\ub294 \ud30c\uad34\uc2e0\uc774\ub780 \uc124\uc815\uc774\ub2e4. \uc0f4\ud30c\ub294 \ube44\ub8e8\uc2a4\uac00 \uc790\ub791\uce5c \uc9c0\uad6c\uc5d0\uc11c \uac00\uc838\uc628 \uc74c\uc2dd\ub4e4\uc744 \ubd80\ub7ec\uc6cc\ud588\ub294\ub370, \uc790\uae30 \ucabd \uc6b0\uc8fc \uc9c0\uad6c\uc5d0 \ud574\ub2f9\ud558\ub294 \ud589\uc131\uc5d0\uc120 \uc778\ub958 \ubb38\uba85\uc774 \uc774\ubbf8 \uba78\ub9dd\ud588\ub2e4\ub294 \uc0ac\uc2e4\uc744 \uc54c\uc790, 7\uc6b0\uc8fc \ucabd \uc9c0\uad6c\ub97c \uac00\uc9c0\uace0 \ub0b4\uae30\ub97c \ud558\uae30\ub85c \ud55c\ub2e4. \uc0f4\ud30c\ub294 \uc790\uc2e0\uc774 \uc288\ud37c \ub4dc\ub798\uace4\ubcfc 6\uac1c\ub97c \uc774\ubbf8 \ubaa8\uc558\ub2e4\uba74\uc11c, \ube44\ub8e8\uc2a4 \ub124\uac00 \uc774\uae30\uba74 \uc774\uac78 \uc591\ubcf4\ud560\uaebc\uace0, \uc790\uc2e0\uc774 \uc774\uae30\uba74 \uc774 \uc288\ud37c \ub4dc\ub798\uace4\ubcfc\uc758 \ud798\uc73c\ub85c \uc11c\ub85c\uc758 \uc9c0\uad6c\ub97c \ub9de\ubc14\uafb8\uaca0\ub2e8 \ub0b4\uc6a9\uc758 \ub0b4\uae30\uc600\ub2e4. \ub450 \uc0ac\ub78c\uc774 \uc9c1\uc811 \uc2f8\uc6b0\uba74 \uc6b0\uc8fc\uac00 \ud30c\uad34\ub420\uc9c0\ub3c4 \ubaa8\ub974\ub294 \uc0c1\ud669\uc774\uc5c8\uae30\uc5d0 \uac01\uc790 5\uba85\uc758 \ub300\ub9ac\uc778\uc744 \ub0b4\uc138\uc6cc \uaca9\ud22c \ub300\ud68c\ub97c \ubc8c\uc774\uae30\ub85c \ud558\uc600\ub2e4. \ube44\ub8e8\uc2a4\ub294 \ub531\ud788 \uc6d0\ud558\ub358 \uc18c\uc6d0\uc774 \uc5c6\uc5c8\uc9c0\ub9cc \uc624\uacf5\uc774 \uc6d0\ud558\uc600\uace0, \uacbd\uc7c1\uc2ec\uc5d0 \uc7ac\ubbf8\uc0bc\uc544 \uc81c\uc548\uc744 \ubc1b\uc544\ub4e4\uc774\uae30\ub85c \ud55c\ub2e4. \uc774\ud6c4 5\uba85\uc758 \ub300\ub9ac\uc778\uc740 \uc790\uc2e0\uc774 \uc18d\ud55c \uc6b0\uc8fc\ub97c \uc9c0\ud0a4\uae30 \uc704\ud574 \uaca9\ud22c \ub300\ud68c\uc5d0\uc11c \ubd84\uc804\ud55c\ub2e4. \uc774\ud6c4 \uaca9\ud22c \ub300\ud68c\uc758 \ub0b4\uc6a9\uc5d0 \ub9cc\uc871\ud55c \uacc4\uc655\uc2e0 \uc77c\ud589\uc740 \ub450 \uc6b0\uc8fc\ub97c \uc874\uc18d\uc2dc\ud0a4\uae30\ub85c \ud569\uc758\ud55c\ub2e4."} {"question": "\ubbf8\uc801\ubd84\ud559\uc744 \ub3c5\uc790\uc801\uc73c\ub85c \ubc1c\uacac\ud55c \uc0ac\ub78c\ub4e4\uc740 \ub204\uad6c\uc778\uac00?", "paragraph": "\uacfc\ud559 \ubd84\uc57c\uc5d0\uc11c\ub294 \uc9c1\uc811\u00b7 \uac04\uc811\uc801 \uad50\ub958\uac00 \uc5c6\ub294 \ud559\uc790\ub4e4\uc774 \ub3d9\uc77c\ud55c \uc5f0\uad6c\ub97c \ub3c5\ub9bd\uc801\uc73c\ub85c \uc218\ud589\ud558\ub294 \ud604\uc0c1\uc774 \uc790\uc8fc \ubc1c\uc0dd\ud55c\ub2e4. \uc774\ub7ec\ud55c \ud604\uc0c1\uc740 \uc911\ubcf5 \ubc1c\uacac\uc774\ub77c \ubd88\ub9ac\uba70, \ub9ce\uc740 \uc5ed\uc0ac\ud559\uc790\ub4e4\uacfc \uc0ac\ud68c\ud559\uc790\ub4e4\uc758 \uad00\uc2ec\ub300\uc0c1\uc774\uc5c8\ub2e4. \uc608\ub97c \ub4e4\uc5b4 19\uc138\uae30 \uc601\uad6d\uc758 \uc5ed\uc0ac\ud559\uc790 \ud1a0\ub9c8\uc2a4 \ub9e4\ucf5c\ub9ac\ub294 \ubbf8\uc801\ubd84\ud559\uc744 \ub3c5\uc790\uc801\uc73c\ub85c \ubc1c\uacac\ud55c \uc601\uad6d\uc758 \ub274\ud134\uacfc \ub3c5\uc77c\uc758 \ub77c\uc774\ud504\ub2c8\uce20\ub97c \uc5f0\uad6c\ud55c \ud6c4, \ub2f9\uc2dc\uc758 \uc0ac\ud68c\uc801 \uc0c1\ud669\uc774 \ubbf8\uc801\ubd84\ud559\uc744 \uac1c\ubc1c\ud560 \uc218 \ubc16\uc5d0 \uc5c6\ub294 \ud658\uacbd\uc744 \ub9cc\ub4e4\uc5c8\ub2e4\uace0 \uc0dd\uac01\ud558\uc600\ub2e4. \uc989, \ub274\ud134\uc774\ub098 \ub77c\uc774\ud504\ub2c8\uce20\uac00 \uc544\ub2c8\ub77c\ub3c4 \ub204\uad70\uac00\uac00 \ubbf8\uc801\ubd84\ud559\uc744 \ube44\uc2b7\ud55c \uc2dc\uae30\uc5d0 \uac1c\ubc1c\ud588\uc73c\ub9ac\ub77c\ub294 \uac83\uc774\ub2e4. \uc774\uc640 \ube44\uc2b7\ud558\uac8c 20\uc138\uae30 \ubbf8\uad6d\uc758 \uc0ac\ud68c\ud559\uc790 \uc70c\ub9ac\uc5c4 \uc624\uadf8\ubc88\uacfc \ub3c4\ub85c\uc2dc \ud1a0\ub9c8\uc2a4\ub294 \uc9c0\uc2dd\uc774 \ucd95\uc801\ub418\uace0 \uc0ac\ud68c\uac00 \ubc1c\uc804\ud568\uc5d0 \ub530\ub77c \uc5f0\uad6c\uc790\ub4e4\uc740 \uc5b4\uca54 \uc218 \uc5c6\uc774 \ud2b9\uc815\ud55c \ubb38\uc81c\ub4e4\uc5d0 \uc8fc\ubaa9\ud558\uac8c \ub41c\ub2e4\uace0 \uc5b8\uae09\ud558\uc600\ub2e4. \uba38\ud134\uc740 \uc880 \ub354 \ub098\uc544\uac00\uc11c \"\uacfc\ud559\uc801 \uc131\uacfc\ub294 \uc6d0\uce59\uc801\uc73c\ub85c \uc911\ubcf5 \ubc1c\uacac\uc774\uba70, \ud45c\uba74\uc801\uc73c\ub85c\ub294 \ub2e8\ub3c5 \ubc1c\uacac\ucc98\ub7fc \uc0dd\uac01\ub418\ub294 \uac83\ub3c4 \uc608\uc678\uac00 \uc544\ub2c8\ub2e4\"\ub77c\uace0 \uc774\uc57c\uae30\ud558\uba74\uc11c \ub2e4\uc591\ud55c \uc5ed\uc0ac\uc801 \uc0ac\ub840\ub97c \ub17c\uac70\ub85c \uc81c\uc2dc\ud55c\ub2e4. \uadf8\ub294 \uc911\ubcf5 \ubc1c\uacac\uc744 \uc5ec\ub7ec\uac00\uc9c0 \ud615\ud0dc\ub85c \ubd84\ub958\ud558\uae30\ub3c4 \ud558\uc600\ub294\ub370, \ubbf8\ubc1c\ud45c \uc5f0\uad6c\ub97c \ub098\uc911\uc5d0 \ub2e4\ub978 \ud559\uc790\uac00 \ub3c5\ub9bd\uc801\uc73c\ub85c \uc218\ud589\ud558\ub294 \uacbd\uc6b0\u00b7 \ub17c\ubb38\uc774 \ubc1c\ud45c\ub418\uae30\ub294 \ud558\uc600\uc9c0\ub9cc \ubb3b\ud600\uc788\ub2e4\uac00 \ub2e4\ub978 \uc0ac\ub78c\uc758 \uc5f0\uad6c\ub85c \ube5b\uc744 \ubcf4\ub294 \uacbd\uc6b0\u00b7 \uac70\uc758 \uac19\uc740 \uc2dc\uae30\uc5d0 \uc911\ubcf5 \ubc1c\uacac\uc774 \uc774\ub904\uc838\uc11c \ubaa8\ub450\uac00 \ub17c\ubb38\uc744 \ubc1c\ud45c\ud558\uac70\ub098 \ud55c \ucabd\uc774 \uc18c\uc2dd\uc744 \ubbf8\ub9ac \ub4e3\uace0 \ud3ec\uae30\ud558\ub294 \uacbd\uc6b0 \ub4f1 \ub2e4\uc591\ud558\ub2e4.", "answer": "\uc601\uad6d\uc758 \ub274\ud134\uacfc \ub3c5\uc77c\uc758 \ub77c\uc774\ud504\ub2c8\uce20", "sentence": "\uc608\ub97c \ub4e4\uc5b4 19\uc138\uae30 \uc601\uad6d\uc758 \uc5ed\uc0ac\ud559\uc790 \ud1a0\ub9c8\uc2a4 \ub9e4\ucf5c\ub9ac\ub294 \ubbf8\uc801\ubd84\ud559\uc744 \ub3c5\uc790\uc801\uc73c\ub85c \ubc1c\uacac\ud55c \uc601\uad6d\uc758 \ub274\ud134\uacfc \ub3c5\uc77c\uc758 \ub77c\uc774\ud504\ub2c8\uce20 \ub97c \uc5f0\uad6c\ud55c \ud6c4, \ub2f9\uc2dc\uc758 \uc0ac\ud68c\uc801 \uc0c1\ud669\uc774 \ubbf8\uc801\ubd84\ud559\uc744 \uac1c\ubc1c\ud560 \uc218 \ubc16\uc5d0 \uc5c6\ub294 \ud658\uacbd\uc744 \ub9cc\ub4e4\uc5c8\ub2e4\uace0 \uc0dd\uac01\ud558\uc600\ub2e4.", "paragraph_sentence": "\uacfc\ud559 \ubd84\uc57c\uc5d0\uc11c\ub294 \uc9c1\uc811\u00b7 \uac04\uc811\uc801 \uad50\ub958\uac00 \uc5c6\ub294 \ud559\uc790\ub4e4\uc774 \ub3d9\uc77c\ud55c \uc5f0\uad6c\ub97c \ub3c5\ub9bd\uc801\uc73c\ub85c \uc218\ud589\ud558\ub294 \ud604\uc0c1\uc774 \uc790\uc8fc \ubc1c\uc0dd\ud55c\ub2e4. \uc774\ub7ec\ud55c \ud604\uc0c1\uc740 \uc911\ubcf5 \ubc1c\uacac\uc774\ub77c \ubd88\ub9ac\uba70, \ub9ce\uc740 \uc5ed\uc0ac\ud559\uc790\ub4e4\uacfc \uc0ac\ud68c\ud559\uc790\ub4e4\uc758 \uad00\uc2ec\ub300\uc0c1\uc774\uc5c8\ub2e4. \uc608\ub97c \ub4e4\uc5b4 19\uc138\uae30 \uc601\uad6d\uc758 \uc5ed\uc0ac\ud559\uc790 \ud1a0\ub9c8\uc2a4 \ub9e4\ucf5c\ub9ac\ub294 \ubbf8\uc801\ubd84\ud559\uc744 \ub3c5\uc790\uc801\uc73c\ub85c \ubc1c\uacac\ud55c \uc601\uad6d\uc758 \ub274\ud134\uacfc \ub3c5\uc77c\uc758 \ub77c\uc774\ud504\ub2c8\uce20 \ub97c \uc5f0\uad6c\ud55c \ud6c4, \ub2f9\uc2dc\uc758 \uc0ac\ud68c\uc801 \uc0c1\ud669\uc774 \ubbf8\uc801\ubd84\ud559\uc744 \uac1c\ubc1c\ud560 \uc218 \ubc16\uc5d0 \uc5c6\ub294 \ud658\uacbd\uc744 \ub9cc\ub4e4\uc5c8\ub2e4\uace0 \uc0dd\uac01\ud558\uc600\ub2e4. \uc989, \ub274\ud134\uc774\ub098 \ub77c\uc774\ud504\ub2c8\uce20\uac00 \uc544\ub2c8\ub77c\ub3c4 \ub204\uad70\uac00\uac00 \ubbf8\uc801\ubd84\ud559\uc744 \ube44\uc2b7\ud55c \uc2dc\uae30\uc5d0 \uac1c\ubc1c\ud588\uc73c\ub9ac\ub77c\ub294 \uac83\uc774\ub2e4. \uc774\uc640 \ube44\uc2b7\ud558\uac8c 20\uc138\uae30 \ubbf8\uad6d\uc758 \uc0ac\ud68c\ud559\uc790 \uc70c\ub9ac\uc5c4 \uc624\uadf8\ubc88\uacfc \ub3c4\ub85c\uc2dc \ud1a0\ub9c8\uc2a4\ub294 \uc9c0\uc2dd\uc774 \ucd95\uc801\ub418\uace0 \uc0ac\ud68c\uac00 \ubc1c\uc804\ud568\uc5d0 \ub530\ub77c \uc5f0\uad6c\uc790\ub4e4\uc740 \uc5b4\uca54 \uc218 \uc5c6\uc774 \ud2b9\uc815\ud55c \ubb38\uc81c\ub4e4\uc5d0 \uc8fc\ubaa9\ud558\uac8c \ub41c\ub2e4\uace0 \uc5b8\uae09\ud558\uc600\ub2e4. \uba38\ud134\uc740 \uc880 \ub354 \ub098\uc544\uac00\uc11c \"\uacfc\ud559\uc801 \uc131\uacfc\ub294 \uc6d0\uce59\uc801\uc73c\ub85c \uc911\ubcf5 \ubc1c\uacac\uc774\uba70, \ud45c\uba74\uc801\uc73c\ub85c\ub294 \ub2e8\ub3c5 \ubc1c\uacac\ucc98\ub7fc \uc0dd\uac01\ub418\ub294 \uac83\ub3c4 \uc608\uc678\uac00 \uc544\ub2c8\ub2e4\"\ub77c\uace0 \uc774\uc57c\uae30\ud558\uba74\uc11c \ub2e4\uc591\ud55c \uc5ed\uc0ac\uc801 \uc0ac\ub840\ub97c \ub17c\uac70\ub85c \uc81c\uc2dc\ud55c\ub2e4. \uadf8\ub294 \uc911\ubcf5 \ubc1c\uacac\uc744 \uc5ec\ub7ec\uac00\uc9c0 \ud615\ud0dc\ub85c \ubd84\ub958\ud558\uae30\ub3c4 \ud558\uc600\ub294\ub370, \ubbf8\ubc1c\ud45c \uc5f0\uad6c\ub97c \ub098\uc911\uc5d0 \ub2e4\ub978 \ud559\uc790\uac00 \ub3c5\ub9bd\uc801\uc73c\ub85c \uc218\ud589\ud558\ub294 \uacbd\uc6b0\u00b7 \ub17c\ubb38\uc774 \ubc1c\ud45c\ub418\uae30\ub294 \ud558\uc600\uc9c0\ub9cc \ubb3b\ud600\uc788\ub2e4\uac00 \ub2e4\ub978 \uc0ac\ub78c\uc758 \uc5f0\uad6c\ub85c \ube5b\uc744 \ubcf4\ub294 \uacbd\uc6b0\u00b7 \uac70\uc758 \uac19\uc740 \uc2dc\uae30\uc5d0 \uc911\ubcf5 \ubc1c\uacac\uc774 \uc774\ub904\uc838\uc11c \ubaa8\ub450\uac00 \ub17c\ubb38\uc744 \ubc1c\ud45c\ud558\uac70\ub098 \ud55c \ucabd\uc774 \uc18c\uc2dd\uc744 \ubbf8\ub9ac \ub4e3\uace0 \ud3ec\uae30\ud558\ub294 \uacbd\uc6b0 \ub4f1 \ub2e4\uc591\ud558\ub2e4.", "paragraph_answer": "\uacfc\ud559 \ubd84\uc57c\uc5d0\uc11c\ub294 \uc9c1\uc811\u00b7 \uac04\uc811\uc801 \uad50\ub958\uac00 \uc5c6\ub294 \ud559\uc790\ub4e4\uc774 \ub3d9\uc77c\ud55c \uc5f0\uad6c\ub97c \ub3c5\ub9bd\uc801\uc73c\ub85c \uc218\ud589\ud558\ub294 \ud604\uc0c1\uc774 \uc790\uc8fc \ubc1c\uc0dd\ud55c\ub2e4. \uc774\ub7ec\ud55c \ud604\uc0c1\uc740 \uc911\ubcf5 \ubc1c\uacac\uc774\ub77c \ubd88\ub9ac\uba70, \ub9ce\uc740 \uc5ed\uc0ac\ud559\uc790\ub4e4\uacfc \uc0ac\ud68c\ud559\uc790\ub4e4\uc758 \uad00\uc2ec\ub300\uc0c1\uc774\uc5c8\ub2e4. \uc608\ub97c \ub4e4\uc5b4 19\uc138\uae30 \uc601\uad6d\uc758 \uc5ed\uc0ac\ud559\uc790 \ud1a0\ub9c8\uc2a4 \ub9e4\ucf5c\ub9ac\ub294 \ubbf8\uc801\ubd84\ud559\uc744 \ub3c5\uc790\uc801\uc73c\ub85c \ubc1c\uacac\ud55c \uc601\uad6d\uc758 \ub274\ud134\uacfc \ub3c5\uc77c\uc758 \ub77c\uc774\ud504\ub2c8\uce20 \ub97c \uc5f0\uad6c\ud55c \ud6c4, \ub2f9\uc2dc\uc758 \uc0ac\ud68c\uc801 \uc0c1\ud669\uc774 \ubbf8\uc801\ubd84\ud559\uc744 \uac1c\ubc1c\ud560 \uc218 \ubc16\uc5d0 \uc5c6\ub294 \ud658\uacbd\uc744 \ub9cc\ub4e4\uc5c8\ub2e4\uace0 \uc0dd\uac01\ud558\uc600\ub2e4. \uc989, \ub274\ud134\uc774\ub098 \ub77c\uc774\ud504\ub2c8\uce20\uac00 \uc544\ub2c8\ub77c\ub3c4 \ub204\uad70\uac00\uac00 \ubbf8\uc801\ubd84\ud559\uc744 \ube44\uc2b7\ud55c \uc2dc\uae30\uc5d0 \uac1c\ubc1c\ud588\uc73c\ub9ac\ub77c\ub294 \uac83\uc774\ub2e4. \uc774\uc640 \ube44\uc2b7\ud558\uac8c 20\uc138\uae30 \ubbf8\uad6d\uc758 \uc0ac\ud68c\ud559\uc790 \uc70c\ub9ac\uc5c4 \uc624\uadf8\ubc88\uacfc \ub3c4\ub85c\uc2dc \ud1a0\ub9c8\uc2a4\ub294 \uc9c0\uc2dd\uc774 \ucd95\uc801\ub418\uace0 \uc0ac\ud68c\uac00 \ubc1c\uc804\ud568\uc5d0 \ub530\ub77c \uc5f0\uad6c\uc790\ub4e4\uc740 \uc5b4\uca54 \uc218 \uc5c6\uc774 \ud2b9\uc815\ud55c \ubb38\uc81c\ub4e4\uc5d0 \uc8fc\ubaa9\ud558\uac8c \ub41c\ub2e4\uace0 \uc5b8\uae09\ud558\uc600\ub2e4. \uba38\ud134\uc740 \uc880 \ub354 \ub098\uc544\uac00\uc11c \"\uacfc\ud559\uc801 \uc131\uacfc\ub294 \uc6d0\uce59\uc801\uc73c\ub85c \uc911\ubcf5 \ubc1c\uacac\uc774\uba70, \ud45c\uba74\uc801\uc73c\ub85c\ub294 \ub2e8\ub3c5 \ubc1c\uacac\ucc98\ub7fc \uc0dd\uac01\ub418\ub294 \uac83\ub3c4 \uc608\uc678\uac00 \uc544\ub2c8\ub2e4\"\ub77c\uace0 \uc774\uc57c\uae30\ud558\uba74\uc11c \ub2e4\uc591\ud55c \uc5ed\uc0ac\uc801 \uc0ac\ub840\ub97c \ub17c\uac70\ub85c \uc81c\uc2dc\ud55c\ub2e4. \uadf8\ub294 \uc911\ubcf5 \ubc1c\uacac\uc744 \uc5ec\ub7ec\uac00\uc9c0 \ud615\ud0dc\ub85c \ubd84\ub958\ud558\uae30\ub3c4 \ud558\uc600\ub294\ub370, \ubbf8\ubc1c\ud45c \uc5f0\uad6c\ub97c \ub098\uc911\uc5d0 \ub2e4\ub978 \ud559\uc790\uac00 \ub3c5\ub9bd\uc801\uc73c\ub85c \uc218\ud589\ud558\ub294 \uacbd\uc6b0\u00b7 \ub17c\ubb38\uc774 \ubc1c\ud45c\ub418\uae30\ub294 \ud558\uc600\uc9c0\ub9cc \ubb3b\ud600\uc788\ub2e4\uac00 \ub2e4\ub978 \uc0ac\ub78c\uc758 \uc5f0\uad6c\ub85c \ube5b\uc744 \ubcf4\ub294 \uacbd\uc6b0\u00b7 \uac70\uc758 \uac19\uc740 \uc2dc\uae30\uc5d0 \uc911\ubcf5 \ubc1c\uacac\uc774 \uc774\ub904\uc838\uc11c \ubaa8\ub450\uac00 \ub17c\ubb38\uc744 \ubc1c\ud45c\ud558\uac70\ub098 \ud55c \ucabd\uc774 \uc18c\uc2dd\uc744 \ubbf8\ub9ac \ub4e3\uace0 \ud3ec\uae30\ud558\ub294 \uacbd\uc6b0 \ub4f1 \ub2e4\uc591\ud558\ub2e4.", "sentence_answer": "\uc608\ub97c \ub4e4\uc5b4 19\uc138\uae30 \uc601\uad6d\uc758 \uc5ed\uc0ac\ud559\uc790 \ud1a0\ub9c8\uc2a4 \ub9e4\ucf5c\ub9ac\ub294 \ubbf8\uc801\ubd84\ud559\uc744 \ub3c5\uc790\uc801\uc73c\ub85c \ubc1c\uacac\ud55c \uc601\uad6d\uc758 \ub274\ud134\uacfc \ub3c5\uc77c\uc758 \ub77c\uc774\ud504\ub2c8\uce20 \ub97c \uc5f0\uad6c\ud55c \ud6c4, \ub2f9\uc2dc\uc758 \uc0ac\ud68c\uc801 \uc0c1\ud669\uc774 \ubbf8\uc801\ubd84\ud559\uc744 \uac1c\ubc1c\ud560 \uc218 \ubc16\uc5d0 \uc5c6\ub294 \ud658\uacbd\uc744 \ub9cc\ub4e4\uc5c8\ub2e4\uace0 \uc0dd\uac01\ud558\uc600\ub2e4.", "paragraph_id": "5959284-10-0", "paragraph_question": "question: \ubbf8\uc801\ubd84\ud559\uc744 \ub3c5\uc790\uc801\uc73c\ub85c \ubc1c\uacac\ud55c \uc0ac\ub78c\ub4e4\uc740 \ub204\uad6c\uc778\uac00?, context: \uacfc\ud559 \ubd84\uc57c\uc5d0\uc11c\ub294 \uc9c1\uc811\u00b7 \uac04\uc811\uc801 \uad50\ub958\uac00 \uc5c6\ub294 \ud559\uc790\ub4e4\uc774 \ub3d9\uc77c\ud55c \uc5f0\uad6c\ub97c \ub3c5\ub9bd\uc801\uc73c\ub85c \uc218\ud589\ud558\ub294 \ud604\uc0c1\uc774 \uc790\uc8fc \ubc1c\uc0dd\ud55c\ub2e4. \uc774\ub7ec\ud55c \ud604\uc0c1\uc740 \uc911\ubcf5 \ubc1c\uacac\uc774\ub77c \ubd88\ub9ac\uba70, \ub9ce\uc740 \uc5ed\uc0ac\ud559\uc790\ub4e4\uacfc \uc0ac\ud68c\ud559\uc790\ub4e4\uc758 \uad00\uc2ec\ub300\uc0c1\uc774\uc5c8\ub2e4. \uc608\ub97c \ub4e4\uc5b4 19\uc138\uae30 \uc601\uad6d\uc758 \uc5ed\uc0ac\ud559\uc790 \ud1a0\ub9c8\uc2a4 \ub9e4\ucf5c\ub9ac\ub294 \ubbf8\uc801\ubd84\ud559\uc744 \ub3c5\uc790\uc801\uc73c\ub85c \ubc1c\uacac\ud55c \uc601\uad6d\uc758 \ub274\ud134\uacfc \ub3c5\uc77c\uc758 \ub77c\uc774\ud504\ub2c8\uce20\ub97c \uc5f0\uad6c\ud55c \ud6c4, \ub2f9\uc2dc\uc758 \uc0ac\ud68c\uc801 \uc0c1\ud669\uc774 \ubbf8\uc801\ubd84\ud559\uc744 \uac1c\ubc1c\ud560 \uc218 \ubc16\uc5d0 \uc5c6\ub294 \ud658\uacbd\uc744 \ub9cc\ub4e4\uc5c8\ub2e4\uace0 \uc0dd\uac01\ud558\uc600\ub2e4. \uc989, \ub274\ud134\uc774\ub098 \ub77c\uc774\ud504\ub2c8\uce20\uac00 \uc544\ub2c8\ub77c\ub3c4 \ub204\uad70\uac00\uac00 \ubbf8\uc801\ubd84\ud559\uc744 \ube44\uc2b7\ud55c \uc2dc\uae30\uc5d0 \uac1c\ubc1c\ud588\uc73c\ub9ac\ub77c\ub294 \uac83\uc774\ub2e4. \uc774\uc640 \ube44\uc2b7\ud558\uac8c 20\uc138\uae30 \ubbf8\uad6d\uc758 \uc0ac\ud68c\ud559\uc790 \uc70c\ub9ac\uc5c4 \uc624\uadf8\ubc88\uacfc \ub3c4\ub85c\uc2dc \ud1a0\ub9c8\uc2a4\ub294 \uc9c0\uc2dd\uc774 \ucd95\uc801\ub418\uace0 \uc0ac\ud68c\uac00 \ubc1c\uc804\ud568\uc5d0 \ub530\ub77c \uc5f0\uad6c\uc790\ub4e4\uc740 \uc5b4\uca54 \uc218 \uc5c6\uc774 \ud2b9\uc815\ud55c \ubb38\uc81c\ub4e4\uc5d0 \uc8fc\ubaa9\ud558\uac8c \ub41c\ub2e4\uace0 \uc5b8\uae09\ud558\uc600\ub2e4. \uba38\ud134\uc740 \uc880 \ub354 \ub098\uc544\uac00\uc11c \"\uacfc\ud559\uc801 \uc131\uacfc\ub294 \uc6d0\uce59\uc801\uc73c\ub85c \uc911\ubcf5 \ubc1c\uacac\uc774\uba70, \ud45c\uba74\uc801\uc73c\ub85c\ub294 \ub2e8\ub3c5 \ubc1c\uacac\ucc98\ub7fc \uc0dd\uac01\ub418\ub294 \uac83\ub3c4 \uc608\uc678\uac00 \uc544\ub2c8\ub2e4\"\ub77c\uace0 \uc774\uc57c\uae30\ud558\uba74\uc11c \ub2e4\uc591\ud55c \uc5ed\uc0ac\uc801 \uc0ac\ub840\ub97c \ub17c\uac70\ub85c \uc81c\uc2dc\ud55c\ub2e4. \uadf8\ub294 \uc911\ubcf5 \ubc1c\uacac\uc744 \uc5ec\ub7ec\uac00\uc9c0 \ud615\ud0dc\ub85c \ubd84\ub958\ud558\uae30\ub3c4 \ud558\uc600\ub294\ub370, \ubbf8\ubc1c\ud45c \uc5f0\uad6c\ub97c \ub098\uc911\uc5d0 \ub2e4\ub978 \ud559\uc790\uac00 \ub3c5\ub9bd\uc801\uc73c\ub85c \uc218\ud589\ud558\ub294 \uacbd\uc6b0\u00b7 \ub17c\ubb38\uc774 \ubc1c\ud45c\ub418\uae30\ub294 \ud558\uc600\uc9c0\ub9cc \ubb3b\ud600\uc788\ub2e4\uac00 \ub2e4\ub978 \uc0ac\ub78c\uc758 \uc5f0\uad6c\ub85c \ube5b\uc744 \ubcf4\ub294 \uacbd\uc6b0\u00b7 \uac70\uc758 \uac19\uc740 \uc2dc\uae30\uc5d0 \uc911\ubcf5 \ubc1c\uacac\uc774 \uc774\ub904\uc838\uc11c \ubaa8\ub450\uac00 \ub17c\ubb38\uc744 \ubc1c\ud45c\ud558\uac70\ub098 \ud55c \ucabd\uc774 \uc18c\uc2dd\uc744 \ubbf8\ub9ac \ub4e3\uace0 \ud3ec\uae30\ud558\ub294 \uacbd\uc6b0 \ub4f1 \ub2e4\uc591\ud558\ub2e4."} {"question": "\ub77c\uc790\ub7ec\uc2a4\uac00 1976\ub144 \ud074\ub9ac\uc624 \uc5b4\uc6cc\ub4dc\uc5d0\uc11c \uc218\uc0c1\ud55c \uc0c1\uc758 \uba85\uce6d\uc740?", "paragraph": "\ube4c\ub9ac \ud734\uc988\ub294 \u2018\ub77c\uc790\ub7ec\uc2a4(Lazarus)\u2019\ub77c\ub294 \uadf8\ub8f9\uc758 \ub9ac\ub354\ub85c \uadf8\uc758 \uc74c\uc545 \uacbd\ub825\uc744 \uc2dc\uc791\ud588\ub2e4. \u2018\ud53c\ud130, \ud3f4 \uc564\ub4dc \uba54\ub9ac(Peter, Paul & Mary)\u2019 \uba64\ubc84\uc778 \ud53c\ud130 \uc608\ub85c\uc6b0(Peter Yarrow)\uc758 \ub3c4\uc6c0\uc73c\ub85c \ub274\uc695\uc8fc \uc6b0\ub4dc\uc2a4\ud0c1(Woodstock)\uc73c\ub85c \ubcf8\uac70\uc9c0\ub97c \uc62e\uae34 \ub4a4, \ub77c\uc790\ub7ec\uc2a4\ub294 \uc0c8\ub85c \uc124\ub9bd\ub41c \uc54c\ubc84\ud2b8 \uadf8\ub85c\uc2a4\uba3c(Albert Grossman)\uc758 \u2018\ubca0\uc5b4\uc2a4\ube4c \ub808\ucf54\ub4dc(Bearsville Records)\u2019 (\uc6cc\ub108\ube0c\ub77c\ub354\uc2a4(Warner Bros.)\uac00 \ubc30\uae09)\uc640 \uacc4\uc57d\uc744 \ud558\uac8c \ub41c\ub2e4. \uadf8 \uc774\ud6c4 \ub77c\uc790\ub7ec\uc2a4\ub294 \ubca0\uc5b4\uc2a4\ube4c \ub808\ucf54\ub4dc\uc5d0\uc11c \ud53c\ud130 \uc608\ub85c\uc6b0\uc640 \ud544 \ub77c\ubaac(Phil Ramone)\uc758 \ud504\ub85c\ub4c0\uc2f1\uc73c\ub85c \ub450 \uc7a5\uc758 \uc568\ubc94\uc744 \ub0b4\uac8c \ub41c\ub2e4. \uc774\ud6c4 4\ub144 \ub3d9\uc548 \ubbf8\uad6d\uacfc \uce90\ub098\ub2e4 \uc804\uc5ed\uc5d0\uc11c \uacf5\uc5f0\uc744 \ud558\uac8c \ub41c \ub77c\uc790\ub7ec\uc2a4\ub294, \uc774\ud6c4 9\ub144 \ub3d9\uc548 \ubbf8 \uc804\uc5ed\uc5d0 \ubc29\uc1a1\ub41c \u2018\ub77c\uc774\ud504\uc138\uc774\ubc84\uc2a4(Lifesavers)\u2019 \uce94\ub514 \uad11\uace0\uc5d0 \uc0bd\uc785\ub41c \uc74c\uc545\uc73c\ub85c 1976\ub144\uc5d0 \uad11\uace0\uacc4\uc758 \uc544\uce74\ub370\ubbf8\uc0c1\uc5d0 \ube44\uacac\ub418\ub294 \u2018\ud074\ub9ac\uc624 \uc5b4\uc6cc\ub4dc(Clio Award)\u2019\uc5d0\uc11c \u2018\uc62c\ud574\uc758 \uad11\uace0\uc0c1(Best Commercial of the Year)\u2019 \ubd80\ubb38\uc744 \uc218\uc0c1\ud558\uac8c \ub41c\ub2e4.", "answer": "\uc62c\ud574\uc758 \uad11\uace0\uc0c1", "sentence": "\uc774\ud6c4 4\ub144 \ub3d9\uc548 \ubbf8\uad6d\uacfc \uce90\ub098\ub2e4 \uc804\uc5ed\uc5d0\uc11c \uacf5\uc5f0\uc744 \ud558\uac8c \ub41c \ub77c\uc790\ub7ec\uc2a4\ub294, \uc774\ud6c4 9\ub144 \ub3d9\uc548 \ubbf8 \uc804\uc5ed\uc5d0 \ubc29\uc1a1\ub41c \u2018\ub77c\uc774\ud504\uc138\uc774\ubc84\uc2a4(Lifesavers)\u2019 \uce94\ub514 \uad11\uace0\uc5d0 \uc0bd\uc785\ub41c \uc74c\uc545\uc73c\ub85c 1976\ub144\uc5d0 \uad11\uace0\uacc4\uc758 \uc544\uce74\ub370\ubbf8\uc0c1\uc5d0 \ube44\uacac\ub418\ub294 \u2018\ud074\ub9ac\uc624 \uc5b4\uc6cc\ub4dc(Clio Award)\u2019\uc5d0\uc11c \u2018 \uc62c\ud574\uc758 \uad11\uace0\uc0c1 (Best Commercial of the Year)\u2019 \ubd80\ubb38\uc744 \uc218\uc0c1\ud558\uac8c \ub41c\ub2e4.", "paragraph_sentence": "\ube4c\ub9ac \ud734\uc988\ub294 \u2018\ub77c\uc790\ub7ec\uc2a4(Lazarus)\u2019\ub77c\ub294 \uadf8\ub8f9\uc758 \ub9ac\ub354\ub85c \uadf8\uc758 \uc74c\uc545 \uacbd\ub825\uc744 \uc2dc\uc791\ud588\ub2e4. \u2018\ud53c\ud130, \ud3f4 \uc564\ub4dc \uba54\ub9ac(Peter, Paul & Mary)\u2019 \uba64\ubc84\uc778 \ud53c\ud130 \uc608\ub85c\uc6b0(Peter Yarrow)\uc758 \ub3c4\uc6c0\uc73c\ub85c \ub274\uc695\uc8fc \uc6b0\ub4dc\uc2a4\ud0c1(Woodstock)\uc73c\ub85c \ubcf8\uac70\uc9c0\ub97c \uc62e\uae34 \ub4a4, \ub77c\uc790\ub7ec\uc2a4\ub294 \uc0c8\ub85c \uc124\ub9bd\ub41c \uc54c\ubc84\ud2b8 \uadf8\ub85c\uc2a4\uba3c(Albert Grossman)\uc758 \u2018\ubca0\uc5b4\uc2a4\ube4c \ub808\ucf54\ub4dc(Bearsville Records)\u2019 (\uc6cc\ub108\ube0c\ub77c\ub354\uc2a4(Warner Bros.)\uac00 \ubc30\uae09)\uc640 \uacc4\uc57d\uc744 \ud558\uac8c \ub41c\ub2e4. \uadf8 \uc774\ud6c4 \ub77c\uc790\ub7ec\uc2a4\ub294 \ubca0\uc5b4\uc2a4\ube4c \ub808\ucf54\ub4dc\uc5d0\uc11c \ud53c\ud130 \uc608\ub85c\uc6b0\uc640 \ud544 \ub77c\ubaac(Phil Ramone)\uc758 \ud504\ub85c\ub4c0\uc2f1\uc73c\ub85c \ub450 \uc7a5\uc758 \uc568\ubc94\uc744 \ub0b4\uac8c \ub41c\ub2e4. \uc774\ud6c4 4\ub144 \ub3d9\uc548 \ubbf8\uad6d\uacfc \uce90\ub098\ub2e4 \uc804\uc5ed\uc5d0\uc11c \uacf5\uc5f0\uc744 \ud558\uac8c \ub41c \ub77c\uc790\ub7ec\uc2a4\ub294, \uc774\ud6c4 9\ub144 \ub3d9\uc548 \ubbf8 \uc804\uc5ed\uc5d0 \ubc29\uc1a1\ub41c \u2018\ub77c\uc774\ud504\uc138\uc774\ubc84\uc2a4(Lifesavers)\u2019 \uce94\ub514 \uad11\uace0\uc5d0 \uc0bd\uc785\ub41c \uc74c\uc545\uc73c\ub85c 1976\ub144\uc5d0 \uad11\uace0\uacc4\uc758 \uc544\uce74\ub370\ubbf8\uc0c1\uc5d0 \ube44\uacac\ub418\ub294 \u2018\ud074\ub9ac\uc624 \uc5b4\uc6cc\ub4dc(Clio Award)\u2019\uc5d0\uc11c \u2018 \uc62c\ud574\uc758 \uad11\uace0\uc0c1 (Best Commercial of the Year)\u2019 \ubd80\ubb38\uc744 \uc218\uc0c1\ud558\uac8c \ub41c\ub2e4. ", "paragraph_answer": "\ube4c\ub9ac \ud734\uc988\ub294 \u2018\ub77c\uc790\ub7ec\uc2a4(Lazarus)\u2019\ub77c\ub294 \uadf8\ub8f9\uc758 \ub9ac\ub354\ub85c \uadf8\uc758 \uc74c\uc545 \uacbd\ub825\uc744 \uc2dc\uc791\ud588\ub2e4. \u2018\ud53c\ud130, \ud3f4 \uc564\ub4dc \uba54\ub9ac(Peter, Paul & Mary)\u2019 \uba64\ubc84\uc778 \ud53c\ud130 \uc608\ub85c\uc6b0(Peter Yarrow)\uc758 \ub3c4\uc6c0\uc73c\ub85c \ub274\uc695\uc8fc \uc6b0\ub4dc\uc2a4\ud0c1(Woodstock)\uc73c\ub85c \ubcf8\uac70\uc9c0\ub97c \uc62e\uae34 \ub4a4, \ub77c\uc790\ub7ec\uc2a4\ub294 \uc0c8\ub85c \uc124\ub9bd\ub41c \uc54c\ubc84\ud2b8 \uadf8\ub85c\uc2a4\uba3c(Albert Grossman)\uc758 \u2018\ubca0\uc5b4\uc2a4\ube4c \ub808\ucf54\ub4dc(Bearsville Records)\u2019 (\uc6cc\ub108\ube0c\ub77c\ub354\uc2a4(Warner Bros.)\uac00 \ubc30\uae09)\uc640 \uacc4\uc57d\uc744 \ud558\uac8c \ub41c\ub2e4. \uadf8 \uc774\ud6c4 \ub77c\uc790\ub7ec\uc2a4\ub294 \ubca0\uc5b4\uc2a4\ube4c \ub808\ucf54\ub4dc\uc5d0\uc11c \ud53c\ud130 \uc608\ub85c\uc6b0\uc640 \ud544 \ub77c\ubaac(Phil Ramone)\uc758 \ud504\ub85c\ub4c0\uc2f1\uc73c\ub85c \ub450 \uc7a5\uc758 \uc568\ubc94\uc744 \ub0b4\uac8c \ub41c\ub2e4. \uc774\ud6c4 4\ub144 \ub3d9\uc548 \ubbf8\uad6d\uacfc \uce90\ub098\ub2e4 \uc804\uc5ed\uc5d0\uc11c \uacf5\uc5f0\uc744 \ud558\uac8c \ub41c \ub77c\uc790\ub7ec\uc2a4\ub294, \uc774\ud6c4 9\ub144 \ub3d9\uc548 \ubbf8 \uc804\uc5ed\uc5d0 \ubc29\uc1a1\ub41c \u2018\ub77c\uc774\ud504\uc138\uc774\ubc84\uc2a4(Lifesavers)\u2019 \uce94\ub514 \uad11\uace0\uc5d0 \uc0bd\uc785\ub41c \uc74c\uc545\uc73c\ub85c 1976\ub144\uc5d0 \uad11\uace0\uacc4\uc758 \uc544\uce74\ub370\ubbf8\uc0c1\uc5d0 \ube44\uacac\ub418\ub294 \u2018\ud074\ub9ac\uc624 \uc5b4\uc6cc\ub4dc(Clio Award)\u2019\uc5d0\uc11c \u2018 \uc62c\ud574\uc758 \uad11\uace0\uc0c1 (Best Commercial of the Year)\u2019 \ubd80\ubb38\uc744 \uc218\uc0c1\ud558\uac8c \ub41c\ub2e4.", "sentence_answer": "\uc774\ud6c4 4\ub144 \ub3d9\uc548 \ubbf8\uad6d\uacfc \uce90\ub098\ub2e4 \uc804\uc5ed\uc5d0\uc11c \uacf5\uc5f0\uc744 \ud558\uac8c \ub41c \ub77c\uc790\ub7ec\uc2a4\ub294, \uc774\ud6c4 9\ub144 \ub3d9\uc548 \ubbf8 \uc804\uc5ed\uc5d0 \ubc29\uc1a1\ub41c \u2018\ub77c\uc774\ud504\uc138\uc774\ubc84\uc2a4(Lifesavers)\u2019 \uce94\ub514 \uad11\uace0\uc5d0 \uc0bd\uc785\ub41c \uc74c\uc545\uc73c\ub85c 1976\ub144\uc5d0 \uad11\uace0\uacc4\uc758 \uc544\uce74\ub370\ubbf8\uc0c1\uc5d0 \ube44\uacac\ub418\ub294 \u2018\ud074\ub9ac\uc624 \uc5b4\uc6cc\ub4dc(Clio Award)\u2019\uc5d0\uc11c \u2018 \uc62c\ud574\uc758 \uad11\uace0\uc0c1 (Best Commercial of the Year)\u2019 \ubd80\ubb38\uc744 \uc218\uc0c1\ud558\uac8c \ub41c\ub2e4.", "paragraph_id": "6531221-0-2", "paragraph_question": "question: \ub77c\uc790\ub7ec\uc2a4\uac00 1976\ub144 \ud074\ub9ac\uc624 \uc5b4\uc6cc\ub4dc\uc5d0\uc11c \uc218\uc0c1\ud55c \uc0c1\uc758 \uba85\uce6d\uc740?, context: \ube4c\ub9ac \ud734\uc988\ub294 \u2018\ub77c\uc790\ub7ec\uc2a4(Lazarus)\u2019\ub77c\ub294 \uadf8\ub8f9\uc758 \ub9ac\ub354\ub85c \uadf8\uc758 \uc74c\uc545 \uacbd\ub825\uc744 \uc2dc\uc791\ud588\ub2e4. \u2018\ud53c\ud130, \ud3f4 \uc564\ub4dc \uba54\ub9ac(Peter, Paul & Mary)\u2019 \uba64\ubc84\uc778 \ud53c\ud130 \uc608\ub85c\uc6b0(Peter Yarrow)\uc758 \ub3c4\uc6c0\uc73c\ub85c \ub274\uc695\uc8fc \uc6b0\ub4dc\uc2a4\ud0c1(Woodstock)\uc73c\ub85c \ubcf8\uac70\uc9c0\ub97c \uc62e\uae34 \ub4a4, \ub77c\uc790\ub7ec\uc2a4\ub294 \uc0c8\ub85c \uc124\ub9bd\ub41c \uc54c\ubc84\ud2b8 \uadf8\ub85c\uc2a4\uba3c(Albert Grossman)\uc758 \u2018\ubca0\uc5b4\uc2a4\ube4c \ub808\ucf54\ub4dc(Bearsville Records)\u2019 (\uc6cc\ub108\ube0c\ub77c\ub354\uc2a4(Warner Bros.)\uac00 \ubc30\uae09)\uc640 \uacc4\uc57d\uc744 \ud558\uac8c \ub41c\ub2e4. \uadf8 \uc774\ud6c4 \ub77c\uc790\ub7ec\uc2a4\ub294 \ubca0\uc5b4\uc2a4\ube4c \ub808\ucf54\ub4dc\uc5d0\uc11c \ud53c\ud130 \uc608\ub85c\uc6b0\uc640 \ud544 \ub77c\ubaac(Phil Ramone)\uc758 \ud504\ub85c\ub4c0\uc2f1\uc73c\ub85c \ub450 \uc7a5\uc758 \uc568\ubc94\uc744 \ub0b4\uac8c \ub41c\ub2e4. \uc774\ud6c4 4\ub144 \ub3d9\uc548 \ubbf8\uad6d\uacfc \uce90\ub098\ub2e4 \uc804\uc5ed\uc5d0\uc11c \uacf5\uc5f0\uc744 \ud558\uac8c \ub41c \ub77c\uc790\ub7ec\uc2a4\ub294, \uc774\ud6c4 9\ub144 \ub3d9\uc548 \ubbf8 \uc804\uc5ed\uc5d0 \ubc29\uc1a1\ub41c \u2018\ub77c\uc774\ud504\uc138\uc774\ubc84\uc2a4(Lifesavers)\u2019 \uce94\ub514 \uad11\uace0\uc5d0 \uc0bd\uc785\ub41c \uc74c\uc545\uc73c\ub85c 1976\ub144\uc5d0 \uad11\uace0\uacc4\uc758 \uc544\uce74\ub370\ubbf8\uc0c1\uc5d0 \ube44\uacac\ub418\ub294 \u2018\ud074\ub9ac\uc624 \uc5b4\uc6cc\ub4dc(Clio Award)\u2019\uc5d0\uc11c \u2018\uc62c\ud574\uc758 \uad11\uace0\uc0c1(Best Commercial of the Year)\u2019 \ubd80\ubb38\uc744 \uc218\uc0c1\ud558\uac8c \ub41c\ub2e4."} {"question": "\ub178\ub974\ubca0\ub974\ud1a0\uac00 \uc774\ub2e8\uc801 \uc0ac\uc0c1\uacfc\uc758 \uc601\uc801 \uc804\ud22c\uc5d0\uc11c \uc2b9\ub9ac\ub97c \uac70\ub450\uac8c \ub418\uba74\uc11c \uc5bb\uac8c \ub41c \ubcc4\uba85\uc740?", "paragraph": "\ub178\ub974\ubca0\ub974\ud1a0\ub294 \ub3c5\uc77c, \ud504\ub791\uc2a4, \ubca8\uae30\uc5d0, \ud2b8\ub780\uc2e4\ubc14\ub2c8\uc544 \ub4f1\uc9c0\uc5d0\uc11c \uc9c0\uc9c0\uc790\ub4e4\uc744 \uc5bb\uc5b4 \ud50c\ub85c\ub974\ud504, \ube44\ube44\uc5d0\ub974, \uc0dd\uc870\uc2a4, \uc544\ub974\ub374, \ud050\uc2dc, \ub77c\uc628, \ub9ac\uc5d0\uc8fc, \uc564\ud2b8\uc6cc\ud504, \uce74\ud39c\ubca0\ub974\ud06c, \uadf8\ub85c\uc2a4\ubc14\ub2e4\uc778(\uc624\ub298\ub0a0\uc758 \uc624\ub77c\ub370\uc544) \ub4f1\uc9c0\uc5d0 \uc218\ub3c4\uc6d0 \ubd84\uc6d0\uc744 \uc138\uc6e0\ub2e4. \uc0f9\ud30c\ub274\uc758 \ud14c\uc624\ubc1c\ub4dc 2\uc138 \ubc31\uc791 \uc5ed\uc2dc \ud504\ub808\ubabd\ud2b8\ub808\ud68c\uc5d0 \uc785\ud68c\ud558\uae30\ub97c \ud76c\ub9dd\ud588\uc9c0\ub9cc, \ub178\ub974\ubca0\ub974\ud1a0\ub294 \uadf8\uc5d0\uac8c \ud3c9\uc2e0\ub3c4\ub85c \ub0a8\uc544 \uacb0\ud63c\ud560 \uac83\uc744 \uad8c\uace0\ud558\uc600\ub2e4. \ub178\ub974\ubca0\ub974\ud1a0\ub294 \uba87 \uac00\uc9c0 \uaddc\uce59\uc744 \uc138\uc6b0\uace0, \uc218\ub3c4\ud68c\uc758 \ud558\uc580\uc0c9 \uc218\ub3c4\ubcf5\uc744 \ud14c\uc624\ubc1c\ub4dc \ubc31\uc791\uc5d0\uac8c \uc218\uc5ec\ud558\uc600\ub2e4. \uadf8\ub9ac\ud558\uc5ec 1122\ub144 \ud504\ub808\ubabd\ud2b8\ub808\ud68c \uc81c3\ud68c\uac00 \uc0dd\uae30\uac8c \ub418\uc5c8\ub2e4. \ub178\ub974\ubca0\ub974\ud1a0\ub294 \ud504\ub791\uc2a4, \ubca8\uae30\uc5d0, \ub3c5\uc77c \uc804\uc5ed\uc744 \ub3cc\uc544\ub2e4\ub2c8\uba74\uc11c \uc0ac\ub78c\ub4e4\uc744 \uac00\ub974\ucce4\uc73c\uba70, \ud0c4\ucf08\ub984\uc758 \uc694\uccad\uc5d0 \ub530\ub77c \uc564\ud2b8\uc6cc\ud504\uc5d0 \ud37c\uc9c4 \uc131\uccb4\uc131\uc0ac\uc5d0 \ub300\ud55c \uc774\ub2e8\uc801 \uc0ac\uc0c1\uacfc\uc758 \uc601\uc801 \uc804\ud22c\uc5d0 \ucc38\uc5ec\ud574 \uc2b9\ub9ac\ub97c \uac70\ub450\uc5c8\ub2e4. \uc774\ub7ec\ud55c \uc5f0\uc720\ub85c \ub178\ub974\ubca0\ub974\ud1a0\ub294 \u2018\uc564\ud2b8\uc6cc\ud504\uc758 \uc0ac\ub3c4\u2019\ub77c\ub294 \ubcc4\uba85\uc774 \ubd99\uac8c \ub418\uc5c8\ub2e4.", "answer": "\uc564\ud2b8\uc6cc\ud504\uc758 \uc0ac\ub3c4", "sentence": "\uc774\ub7ec\ud55c \uc5f0\uc720\ub85c \ub178\ub974\ubca0\ub974\ud1a0\ub294 \u2018 \uc564\ud2b8\uc6cc\ud504\uc758 \uc0ac\ub3c4 \u2019\ub77c\ub294 \ubcc4\uba85\uc774 \ubd99\uac8c \ub418\uc5c8\ub2e4.", "paragraph_sentence": "\ub178\ub974\ubca0\ub974\ud1a0\ub294 \ub3c5\uc77c, \ud504\ub791\uc2a4, \ubca8\uae30\uc5d0, \ud2b8\ub780\uc2e4\ubc14\ub2c8\uc544 \ub4f1\uc9c0\uc5d0\uc11c \uc9c0\uc9c0\uc790\ub4e4\uc744 \uc5bb\uc5b4 \ud50c\ub85c\ub974\ud504, \ube44\ube44\uc5d0\ub974, \uc0dd\uc870\uc2a4, \uc544\ub974\ub374, \ud050\uc2dc, \ub77c\uc628, \ub9ac\uc5d0\uc8fc, \uc564\ud2b8\uc6cc\ud504, \uce74\ud39c\ubca0\ub974\ud06c, \uadf8\ub85c\uc2a4\ubc14\ub2e4\uc778(\uc624\ub298\ub0a0\uc758 \uc624\ub77c\ub370\uc544) \ub4f1\uc9c0\uc5d0 \uc218\ub3c4\uc6d0 \ubd84\uc6d0\uc744 \uc138\uc6e0\ub2e4. \uc0f9\ud30c\ub274\uc758 \ud14c\uc624\ubc1c\ub4dc 2\uc138 \ubc31\uc791 \uc5ed\uc2dc \ud504\ub808\ubabd\ud2b8\ub808\ud68c\uc5d0 \uc785\ud68c\ud558\uae30\ub97c \ud76c\ub9dd\ud588\uc9c0\ub9cc, \ub178\ub974\ubca0\ub974\ud1a0\ub294 \uadf8\uc5d0\uac8c \ud3c9\uc2e0\ub3c4\ub85c \ub0a8\uc544 \uacb0\ud63c\ud560 \uac83\uc744 \uad8c\uace0\ud558\uc600\ub2e4. \ub178\ub974\ubca0\ub974\ud1a0\ub294 \uba87 \uac00\uc9c0 \uaddc\uce59\uc744 \uc138\uc6b0\uace0, \uc218\ub3c4\ud68c\uc758 \ud558\uc580\uc0c9 \uc218\ub3c4\ubcf5\uc744 \ud14c\uc624\ubc1c\ub4dc \ubc31\uc791\uc5d0\uac8c \uc218\uc5ec\ud558\uc600\ub2e4. \uadf8\ub9ac\ud558\uc5ec 1122\ub144 \ud504\ub808\ubabd\ud2b8\ub808\ud68c \uc81c3\ud68c\uac00 \uc0dd\uae30\uac8c \ub418\uc5c8\ub2e4. \ub178\ub974\ubca0\ub974\ud1a0\ub294 \ud504\ub791\uc2a4, \ubca8\uae30\uc5d0, \ub3c5\uc77c \uc804\uc5ed\uc744 \ub3cc\uc544\ub2e4\ub2c8\uba74\uc11c \uc0ac\ub78c\ub4e4\uc744 \uac00\ub974\ucce4\uc73c\uba70, \ud0c4\ucf08\ub984\uc758 \uc694\uccad\uc5d0 \ub530\ub77c \uc564\ud2b8\uc6cc\ud504\uc5d0 \ud37c\uc9c4 \uc131\uccb4\uc131\uc0ac\uc5d0 \ub300\ud55c \uc774\ub2e8\uc801 \uc0ac\uc0c1\uacfc\uc758 \uc601\uc801 \uc804\ud22c\uc5d0 \ucc38\uc5ec\ud574 \uc2b9\ub9ac\ub97c \uac70\ub450\uc5c8\ub2e4. \uc774\ub7ec\ud55c \uc5f0\uc720\ub85c \ub178\ub974\ubca0\ub974\ud1a0\ub294 \u2018 \uc564\ud2b8\uc6cc\ud504\uc758 \uc0ac\ub3c4 \u2019\ub77c\ub294 \ubcc4\uba85\uc774 \ubd99\uac8c \ub418\uc5c8\ub2e4. ", "paragraph_answer": "\ub178\ub974\ubca0\ub974\ud1a0\ub294 \ub3c5\uc77c, \ud504\ub791\uc2a4, \ubca8\uae30\uc5d0, \ud2b8\ub780\uc2e4\ubc14\ub2c8\uc544 \ub4f1\uc9c0\uc5d0\uc11c \uc9c0\uc9c0\uc790\ub4e4\uc744 \uc5bb\uc5b4 \ud50c\ub85c\ub974\ud504, \ube44\ube44\uc5d0\ub974, \uc0dd\uc870\uc2a4, \uc544\ub974\ub374, \ud050\uc2dc, \ub77c\uc628, \ub9ac\uc5d0\uc8fc, \uc564\ud2b8\uc6cc\ud504, \uce74\ud39c\ubca0\ub974\ud06c, \uadf8\ub85c\uc2a4\ubc14\ub2e4\uc778(\uc624\ub298\ub0a0\uc758 \uc624\ub77c\ub370\uc544) \ub4f1\uc9c0\uc5d0 \uc218\ub3c4\uc6d0 \ubd84\uc6d0\uc744 \uc138\uc6e0\ub2e4. \uc0f9\ud30c\ub274\uc758 \ud14c\uc624\ubc1c\ub4dc 2\uc138 \ubc31\uc791 \uc5ed\uc2dc \ud504\ub808\ubabd\ud2b8\ub808\ud68c\uc5d0 \uc785\ud68c\ud558\uae30\ub97c \ud76c\ub9dd\ud588\uc9c0\ub9cc, \ub178\ub974\ubca0\ub974\ud1a0\ub294 \uadf8\uc5d0\uac8c \ud3c9\uc2e0\ub3c4\ub85c \ub0a8\uc544 \uacb0\ud63c\ud560 \uac83\uc744 \uad8c\uace0\ud558\uc600\ub2e4. \ub178\ub974\ubca0\ub974\ud1a0\ub294 \uba87 \uac00\uc9c0 \uaddc\uce59\uc744 \uc138\uc6b0\uace0, \uc218\ub3c4\ud68c\uc758 \ud558\uc580\uc0c9 \uc218\ub3c4\ubcf5\uc744 \ud14c\uc624\ubc1c\ub4dc \ubc31\uc791\uc5d0\uac8c \uc218\uc5ec\ud558\uc600\ub2e4. \uadf8\ub9ac\ud558\uc5ec 1122\ub144 \ud504\ub808\ubabd\ud2b8\ub808\ud68c \uc81c3\ud68c\uac00 \uc0dd\uae30\uac8c \ub418\uc5c8\ub2e4. \ub178\ub974\ubca0\ub974\ud1a0\ub294 \ud504\ub791\uc2a4, \ubca8\uae30\uc5d0, \ub3c5\uc77c \uc804\uc5ed\uc744 \ub3cc\uc544\ub2e4\ub2c8\uba74\uc11c \uc0ac\ub78c\ub4e4\uc744 \uac00\ub974\ucce4\uc73c\uba70, \ud0c4\ucf08\ub984\uc758 \uc694\uccad\uc5d0 \ub530\ub77c \uc564\ud2b8\uc6cc\ud504\uc5d0 \ud37c\uc9c4 \uc131\uccb4\uc131\uc0ac\uc5d0 \ub300\ud55c \uc774\ub2e8\uc801 \uc0ac\uc0c1\uacfc\uc758 \uc601\uc801 \uc804\ud22c\uc5d0 \ucc38\uc5ec\ud574 \uc2b9\ub9ac\ub97c \uac70\ub450\uc5c8\ub2e4. \uc774\ub7ec\ud55c \uc5f0\uc720\ub85c \ub178\ub974\ubca0\ub974\ud1a0\ub294 \u2018 \uc564\ud2b8\uc6cc\ud504\uc758 \uc0ac\ub3c4 \u2019\ub77c\ub294 \ubcc4\uba85\uc774 \ubd99\uac8c \ub418\uc5c8\ub2e4.", "sentence_answer": "\uc774\ub7ec\ud55c \uc5f0\uc720\ub85c \ub178\ub974\ubca0\ub974\ud1a0\ub294 \u2018 \uc564\ud2b8\uc6cc\ud504\uc758 \uc0ac\ub3c4 \u2019\ub77c\ub294 \ubcc4\uba85\uc774 \ubd99\uac8c \ub418\uc5c8\ub2e4.", "paragraph_id": "5909378-3-2", "paragraph_question": "question: \ub178\ub974\ubca0\ub974\ud1a0\uac00 \uc774\ub2e8\uc801 \uc0ac\uc0c1\uacfc\uc758 \uc601\uc801 \uc804\ud22c\uc5d0\uc11c \uc2b9\ub9ac\ub97c \uac70\ub450\uac8c \ub418\uba74\uc11c \uc5bb\uac8c \ub41c \ubcc4\uba85\uc740?, context: \ub178\ub974\ubca0\ub974\ud1a0\ub294 \ub3c5\uc77c, \ud504\ub791\uc2a4, \ubca8\uae30\uc5d0, \ud2b8\ub780\uc2e4\ubc14\ub2c8\uc544 \ub4f1\uc9c0\uc5d0\uc11c \uc9c0\uc9c0\uc790\ub4e4\uc744 \uc5bb\uc5b4 \ud50c\ub85c\ub974\ud504, \ube44\ube44\uc5d0\ub974, \uc0dd\uc870\uc2a4, \uc544\ub974\ub374, \ud050\uc2dc, \ub77c\uc628, \ub9ac\uc5d0\uc8fc, \uc564\ud2b8\uc6cc\ud504, \uce74\ud39c\ubca0\ub974\ud06c, \uadf8\ub85c\uc2a4\ubc14\ub2e4\uc778(\uc624\ub298\ub0a0\uc758 \uc624\ub77c\ub370\uc544) \ub4f1\uc9c0\uc5d0 \uc218\ub3c4\uc6d0 \ubd84\uc6d0\uc744 \uc138\uc6e0\ub2e4. \uc0f9\ud30c\ub274\uc758 \ud14c\uc624\ubc1c\ub4dc 2\uc138 \ubc31\uc791 \uc5ed\uc2dc \ud504\ub808\ubabd\ud2b8\ub808\ud68c\uc5d0 \uc785\ud68c\ud558\uae30\ub97c \ud76c\ub9dd\ud588\uc9c0\ub9cc, \ub178\ub974\ubca0\ub974\ud1a0\ub294 \uadf8\uc5d0\uac8c \ud3c9\uc2e0\ub3c4\ub85c \ub0a8\uc544 \uacb0\ud63c\ud560 \uac83\uc744 \uad8c\uace0\ud558\uc600\ub2e4. \ub178\ub974\ubca0\ub974\ud1a0\ub294 \uba87 \uac00\uc9c0 \uaddc\uce59\uc744 \uc138\uc6b0\uace0, \uc218\ub3c4\ud68c\uc758 \ud558\uc580\uc0c9 \uc218\ub3c4\ubcf5\uc744 \ud14c\uc624\ubc1c\ub4dc \ubc31\uc791\uc5d0\uac8c \uc218\uc5ec\ud558\uc600\ub2e4. \uadf8\ub9ac\ud558\uc5ec 1122\ub144 \ud504\ub808\ubabd\ud2b8\ub808\ud68c \uc81c3\ud68c\uac00 \uc0dd\uae30\uac8c \ub418\uc5c8\ub2e4. \ub178\ub974\ubca0\ub974\ud1a0\ub294 \ud504\ub791\uc2a4, \ubca8\uae30\uc5d0, \ub3c5\uc77c \uc804\uc5ed\uc744 \ub3cc\uc544\ub2e4\ub2c8\uba74\uc11c \uc0ac\ub78c\ub4e4\uc744 \uac00\ub974\ucce4\uc73c\uba70, \ud0c4\ucf08\ub984\uc758 \uc694\uccad\uc5d0 \ub530\ub77c \uc564\ud2b8\uc6cc\ud504\uc5d0 \ud37c\uc9c4 \uc131\uccb4\uc131\uc0ac\uc5d0 \ub300\ud55c \uc774\ub2e8\uc801 \uc0ac\uc0c1\uacfc\uc758 \uc601\uc801 \uc804\ud22c\uc5d0 \ucc38\uc5ec\ud574 \uc2b9\ub9ac\ub97c \uac70\ub450\uc5c8\ub2e4. \uc774\ub7ec\ud55c \uc5f0\uc720\ub85c \ub178\ub974\ubca0\ub974\ud1a0\ub294 \u2018\uc564\ud2b8\uc6cc\ud504\uc758 \uc0ac\ub3c4\u2019\ub77c\ub294 \ubcc4\uba85\uc774 \ubd99\uac8c \ub418\uc5c8\ub2e4."} {"question": "\uce74\ub77c\uac00 \ub370\ubdd4 \ud6c4 \ucc98\uc74c\uc73c\ub85c 1\uc704\ub97c \ud55c \uace1\uc740?", "paragraph": "11\uc6d4 29\uc77c, \ub450 \ubc88\uc9f8 \ubbf8\ub2c8\uc568\ubc94 Pretty Girl\uc758 \ud0c0\uc774\ud2c0\uace1 \"Pretty Girl\"\uc758 \ud2f0\uc800\uc601\uc0c1\uc774 \uacf5\uac1c\ub418\uc5c8\ub2e4. \uc774 \ud2f0\uc800\uc601\uc0c1\uc740 12\uc2dc\uac04 \ub9cc\uc5d0 \uc870\ud68c\uc218 5\ub9cc\uc744 \ub118\ub294 \ub4f1 \ub300\ubc15\uc744 \ud130\ub728\ub9b4 \ub9cc\ud55c \uc870\uc9d0\uc744 \ubcf4\uc774\uae30 \uc2dc\uc791\ud588\ub2e4. 12\uc6d4 4\uc77c \uc568\ubc94 \ubc1c\ub9e4\uc640 \ud568\uaed8 M.net \u300a\uc5e0\uce74\uc6b4\ud2b8\ub2e4\uc6b4\u300b\uc5d0\uc11c \ucef4\ubc31\ubb34\ub300\ub97c \uac00\uc9c0\uac8c \ub418\uc5c8\ub2e4. \"Pretty Girl\"\uc740 \uce74\ub77c \ub370\ubdd4 \uc774\ub798 \ucc98\uc74c\uc73c\ub85c \ucc28\ud2b8\uc5d0\uc11c 2\uc704\uae4c\uc9c0 \uc624\ub974\ub294 \uc704\ub825\uc744 \ubcf4\uc600\uc73c\uba70, \ub354\ubd88\uc5b4 \uc548\ubb34\uc778 '\ud654\uc7a5\ucda4', '\ub2f9\ub2f9\ud558\uac8c \uac77\uae30 \ucda4', '\uc608\uc608 \ucda4' \ub4f1\uc758 \ucda4\ub4e4\uc774 \uc778\uae30\ub97c \uc5bb\uc5c8\ub2e4. \ud2b9\ud788 '\uc608\uc608 \ucda4'\uc740 \ud32c\ub4e4\uc774 \uace0\ubb34\uc7a5\uac11\uc744 \ucc29\uc6a9\ud558\uace0 \uadf8 \ucda4\uc744 \ud568\uaed8 \ucd94\ub294 \uc751\uc6d0\uc744 \uc120\ubcf4\uc600\uace0, \uc774\uac83 \ub610\ud55c \uc5ed\uc2dc \ub9ce\uc740 \uad00\uc2ec\uc758 \ub300\uc0c1\uc774 \ub418\uc5c8\ub2e4. \"Pretty Girl\"\uc740 \ud0dc\uad6d\uc5d0\uc11c\ub294 \ud65c\ub3d9 \uc5c6\uc774 Pop Chart 1\uc704\ub97c \ud558\ub294 \uc601\uad11\uc744 \uc548\uac8c \ub418\uc5c8\ub2e4. \"Pretty Girl\"\uc740 \ub610\ud55c \ud574\uc678 \ud65c\ub3d9\uc744 \uc804\ud600 \ud558\uc9c0 \uc54a\uc558\uc9c0\ub9cc 2009\ub144\uc5d0 \ud0dc\uad6d AS TV \uc544\uc2dc\uc544 \ud31d \ucc28\ud2b8\uc5d0\uc11c 1\uc704\ub97c \ud588\ub2e4. \uadf8 \ud6c4, \ub3d9\ub0a8\uc544\uc2dc\uc544\uc640 \uc77c\ubcf8, \uc911\uad6d\uc5d0\uc11c \ub9ce\uc740 \ub7ec\ube0c\ucf5c\uc744 \ubc1b\uc558\ub2e4. 2009\ub144 9\uc6d4 17\uc77c\uc5d0\ub294 \ud0dc\uad6d\uc5d0\uc11c Thai-Korean Concert\uc5d0 \ucd9c\uc5f0\ud574\uc11c \ub9ce\uc740 \ud574\uc678\ud32c\ub4e4\uc744 \uc990\uac81\uac8c \ud558\uc600\uc73c\uba70, 2009\ub144 \ud0dc\uad6d Channel V Asia TOP 50\uc5d0\uc11c \"Wanna\"\uc640 \"Honey\"\uac00 \uac01\uac01 31\uc704, 46\uc704\uc5d0 \ub7ad\ud06c\ub418\uae30\ub3c4 \ud558\uc600\ub2e4. 2009\ub144 2\uc6d4 10\uc77c, \"Pretty Girl\" \ud65c\ub3d9 \ud6c4 \uce74\ub77c\ub294 \ud22c\ud45c\ub97c \ud1b5\ud574 \ud6c4\uc18d\uace1\uc73c\ub85c \ubf51\ud78c \"Honey\"\uc758 \uc548\ubb34 \ub3d9\uc601\uc0c1\uc744 \uacf5\uac1c\ud558\uc600\uc73c\uba70 2\uc6d4 13\uc77c \ud2f0\uc800\uc601\uc0c1\uc744 \uacf5\uac1c\ud558\uc600\ub2e4. \"Honey\"\ub294 \ub450 \ubc88\uc9f8 \ubbf8\ub2c8\uc568\ubc94\uc5d0 \uc218\ub85d\ub41c \uace1 \"\ud558\ub2c8\"\uc758 \uc7ac\ud3b8\uc9d1\uace1\uc774\uae30\ub3c4 \ud558\ub2e4. 2\uc6d4 14\uc77c \uc1fc! \uc74c\uc545\uc911\uc2ec\uc5d0\uc11c \uccab \ubb34\ub300\ub97c \uac00\uc9c0\uace0, 2\uc6d4 16\uc77c \ubba4\uc9c1 \ube44\ub514\uc624\ub97c \uacf5\uac1c\ud558\uc600\ub2e4. \uc774\uc5b4 2\uc6d4 18\uc77c \uccab \uc2a4\ud398\uc15c \uc568\ubc94\uc744 \ubc1c\ub9e4\ud558\uc600\ub2e4. \"Honey\"\ub294 \uc548\ubb34 \uc911 '\uafc0 \ucc0d\uc5b4\uba39\uae30 \ucda4'\uc774 \ud070 \uc778\uae30\ub97c \ub04c\uc5c8\uace0, \uc74c\uc6d0 \ucc28\ud2b8\uc5d0\uc11c\ub3c4 \uc0c1\uc704\uad8c\uc5d0 \ub7ad\ud06c \ub418\uc5b4 3\uc6d4 5\uc77c\uc5d0\ub294 Mnet \u300a\uc5e0\uce74\uc6b4\ud2b8\ub2e4\uc6b4\u300b\uc5d0\uc11c \ub9c8\uce68\ub0b4 \ub370\ubdd4 \ud6c4 \uc57d 2\ub144 \ub9cc\uc5d0 \ucc98\uc74c\uc73c\ub85c \uc74c\uc545\ud504\ub85c\uadf8\ub7a8 1\uc704\uc758 \uac10\uaca9\uc744 \ub9db\ubcf4\uc558\ub2e4. \ub610\ud55c 3\uc6d4 8\uc77c \uc778\uae30\uac00\uc694\uc5d0\uc11c\ub294 \ubba4\ud2f0\uc98c\uc1a1\uc744 \uc218\uc0c1\ud574 \ucc98\uc74c\uc73c\ub85c \uacf5\uc911\ud30c 1\uc704\ub97c \ucc28\uc9c0\ud588\uace0, 4\uc6d4 \ub9d0 \ub9c8\uc9c0\ub9c9 \ubb34\ub300\ub85c\uc368 \ud65c\ub3d9\uc744 \ub9c8\ubb34\ub9ac\ud588\ub2e4.", "answer": "Honey", "sentence": "2009\ub144 9\uc6d4 17\uc77c\uc5d0\ub294 \ud0dc\uad6d\uc5d0\uc11c Thai-Korean Concert\uc5d0 \ucd9c\uc5f0\ud574\uc11c \ub9ce\uc740 \ud574\uc678\ud32c\ub4e4\uc744 \uc990\uac81\uac8c \ud558\uc600\uc73c\uba70, 2009\ub144 \ud0dc\uad6d Channel V Asia TOP 50\uc5d0\uc11c \"Wanna\"\uc640 \" Honey \"\uac00 \uac01\uac01 31\uc704, 46\uc704\uc5d0 \ub7ad\ud06c\ub418\uae30\ub3c4 \ud558\uc600\ub2e4.", "paragraph_sentence": "11\uc6d4 29\uc77c, \ub450 \ubc88\uc9f8 \ubbf8\ub2c8\uc568\ubc94 Pretty Girl\uc758 \ud0c0\uc774\ud2c0\uace1 \"Pretty Girl\"\uc758 \ud2f0\uc800\uc601\uc0c1\uc774 \uacf5\uac1c\ub418\uc5c8\ub2e4. \uc774 \ud2f0\uc800\uc601\uc0c1\uc740 12\uc2dc\uac04 \ub9cc\uc5d0 \uc870\ud68c\uc218 5\ub9cc\uc744 \ub118\ub294 \ub4f1 \ub300\ubc15\uc744 \ud130\ub728\ub9b4 \ub9cc\ud55c \uc870\uc9d0\uc744 \ubcf4\uc774\uae30 \uc2dc\uc791\ud588\ub2e4. 12\uc6d4 4\uc77c \uc568\ubc94 \ubc1c\ub9e4\uc640 \ud568\uaed8 M.net \u300a\uc5e0\uce74\uc6b4\ud2b8\ub2e4\uc6b4\u300b\uc5d0\uc11c \ucef4\ubc31\ubb34\ub300\ub97c \uac00\uc9c0\uac8c \ub418\uc5c8\ub2e4. \"Pretty Girl\"\uc740 \uce74\ub77c \ub370\ubdd4 \uc774\ub798 \ucc98\uc74c\uc73c\ub85c \ucc28\ud2b8\uc5d0\uc11c 2\uc704\uae4c\uc9c0 \uc624\ub974\ub294 \uc704\ub825\uc744 \ubcf4\uc600\uc73c\uba70, \ub354\ubd88\uc5b4 \uc548\ubb34\uc778 '\ud654\uc7a5\ucda4', '\ub2f9\ub2f9\ud558\uac8c \uac77\uae30 \ucda4', '\uc608\uc608 \ucda4' \ub4f1\uc758 \ucda4\ub4e4\uc774 \uc778\uae30\ub97c \uc5bb\uc5c8\ub2e4. \ud2b9\ud788 '\uc608\uc608 \ucda4'\uc740 \ud32c\ub4e4\uc774 \uace0\ubb34\uc7a5\uac11\uc744 \ucc29\uc6a9\ud558\uace0 \uadf8 \ucda4\uc744 \ud568\uaed8 \ucd94\ub294 \uc751\uc6d0\uc744 \uc120\ubcf4\uc600\uace0, \uc774\uac83 \ub610\ud55c \uc5ed\uc2dc \ub9ce\uc740 \uad00\uc2ec\uc758 \ub300\uc0c1\uc774 \ub418\uc5c8\ub2e4. \"Pretty Girl\"\uc740 \ud0dc\uad6d\uc5d0\uc11c\ub294 \ud65c\ub3d9 \uc5c6\uc774 Pop Chart 1\uc704\ub97c \ud558\ub294 \uc601\uad11\uc744 \uc548\uac8c \ub418\uc5c8\ub2e4. \"Pretty Girl\"\uc740 \ub610\ud55c \ud574\uc678 \ud65c\ub3d9\uc744 \uc804\ud600 \ud558\uc9c0 \uc54a\uc558\uc9c0\ub9cc 2009\ub144\uc5d0 \ud0dc\uad6d AS TV \uc544\uc2dc\uc544 \ud31d \ucc28\ud2b8\uc5d0\uc11c 1\uc704\ub97c \ud588\ub2e4. \uadf8 \ud6c4, \ub3d9\ub0a8\uc544\uc2dc\uc544\uc640 \uc77c\ubcf8, \uc911\uad6d\uc5d0\uc11c \ub9ce\uc740 \ub7ec\ube0c\ucf5c\uc744 \ubc1b\uc558\ub2e4. 2009\ub144 9\uc6d4 17\uc77c\uc5d0\ub294 \ud0dc\uad6d\uc5d0\uc11c Thai-Korean Concert\uc5d0 \ucd9c\uc5f0\ud574\uc11c \ub9ce\uc740 \ud574\uc678\ud32c\ub4e4\uc744 \uc990\uac81\uac8c \ud558\uc600\uc73c\uba70, 2009\ub144 \ud0dc\uad6d Channel V Asia TOP 50\uc5d0\uc11c \"Wanna\"\uc640 \" Honey \"\uac00 \uac01\uac01 31\uc704, 46\uc704\uc5d0 \ub7ad\ud06c\ub418\uae30\ub3c4 \ud558\uc600\ub2e4. 2009\ub144 2\uc6d4 10\uc77c, \"Pretty Girl\" \ud65c\ub3d9 \ud6c4 \uce74\ub77c\ub294 \ud22c\ud45c\ub97c \ud1b5\ud574 \ud6c4\uc18d\uace1\uc73c\ub85c \ubf51\ud78c \"Honey\"\uc758 \uc548\ubb34 \ub3d9\uc601\uc0c1\uc744 \uacf5\uac1c\ud558\uc600\uc73c\uba70 2\uc6d4 13\uc77c \ud2f0\uc800\uc601\uc0c1\uc744 \uacf5\uac1c\ud558\uc600\ub2e4. \"Honey\"\ub294 \ub450 \ubc88\uc9f8 \ubbf8\ub2c8\uc568\ubc94\uc5d0 \uc218\ub85d\ub41c \uace1 \"\ud558\ub2c8\"\uc758 \uc7ac\ud3b8\uc9d1\uace1\uc774\uae30\ub3c4 \ud558\ub2e4. 2\uc6d4 14\uc77c \uc1fc! \uc74c\uc545\uc911\uc2ec\uc5d0\uc11c \uccab \ubb34\ub300\ub97c \uac00\uc9c0\uace0, 2\uc6d4 16\uc77c \ubba4\uc9c1 \ube44\ub514\uc624\ub97c \uacf5\uac1c\ud558\uc600\ub2e4. \uc774\uc5b4 2\uc6d4 18\uc77c \uccab \uc2a4\ud398\uc15c \uc568\ubc94\uc744 \ubc1c\ub9e4\ud558\uc600\ub2e4. \"Honey\"\ub294 \uc548\ubb34 \uc911 '\uafc0 \ucc0d\uc5b4\uba39\uae30 \ucda4'\uc774 \ud070 \uc778\uae30\ub97c \ub04c\uc5c8\uace0, \uc74c\uc6d0 \ucc28\ud2b8\uc5d0\uc11c\ub3c4 \uc0c1\uc704\uad8c\uc5d0 \ub7ad\ud06c \ub418\uc5b4 3\uc6d4 5\uc77c\uc5d0\ub294 Mnet \u300a\uc5e0\uce74\uc6b4\ud2b8\ub2e4\uc6b4\u300b\uc5d0\uc11c \ub9c8\uce68\ub0b4 \ub370\ubdd4 \ud6c4 \uc57d 2\ub144 \ub9cc\uc5d0 \ucc98\uc74c\uc73c\ub85c \uc74c\uc545\ud504\ub85c\uadf8\ub7a8 1\uc704\uc758 \uac10\uaca9\uc744 \ub9db\ubcf4\uc558\ub2e4. \ub610\ud55c 3\uc6d4 8\uc77c \uc778\uae30\uac00\uc694\uc5d0\uc11c\ub294 \ubba4\ud2f0\uc98c\uc1a1\uc744 \uc218\uc0c1\ud574 \ucc98\uc74c\uc73c\ub85c \uacf5\uc911\ud30c 1\uc704\ub97c \ucc28\uc9c0\ud588\uace0, 4\uc6d4 \ub9d0 \ub9c8\uc9c0\ub9c9 \ubb34\ub300\ub85c\uc368 \ud65c\ub3d9\uc744 \ub9c8\ubb34\ub9ac\ud588\ub2e4.", "paragraph_answer": "11\uc6d4 29\uc77c, \ub450 \ubc88\uc9f8 \ubbf8\ub2c8\uc568\ubc94 Pretty Girl\uc758 \ud0c0\uc774\ud2c0\uace1 \"Pretty Girl\"\uc758 \ud2f0\uc800\uc601\uc0c1\uc774 \uacf5\uac1c\ub418\uc5c8\ub2e4. \uc774 \ud2f0\uc800\uc601\uc0c1\uc740 12\uc2dc\uac04 \ub9cc\uc5d0 \uc870\ud68c\uc218 5\ub9cc\uc744 \ub118\ub294 \ub4f1 \ub300\ubc15\uc744 \ud130\ub728\ub9b4 \ub9cc\ud55c \uc870\uc9d0\uc744 \ubcf4\uc774\uae30 \uc2dc\uc791\ud588\ub2e4. 12\uc6d4 4\uc77c \uc568\ubc94 \ubc1c\ub9e4\uc640 \ud568\uaed8 M.net \u300a\uc5e0\uce74\uc6b4\ud2b8\ub2e4\uc6b4\u300b\uc5d0\uc11c \ucef4\ubc31\ubb34\ub300\ub97c \uac00\uc9c0\uac8c \ub418\uc5c8\ub2e4. \"Pretty Girl\"\uc740 \uce74\ub77c \ub370\ubdd4 \uc774\ub798 \ucc98\uc74c\uc73c\ub85c \ucc28\ud2b8\uc5d0\uc11c 2\uc704\uae4c\uc9c0 \uc624\ub974\ub294 \uc704\ub825\uc744 \ubcf4\uc600\uc73c\uba70, \ub354\ubd88\uc5b4 \uc548\ubb34\uc778 '\ud654\uc7a5\ucda4', '\ub2f9\ub2f9\ud558\uac8c \uac77\uae30 \ucda4', '\uc608\uc608 \ucda4' \ub4f1\uc758 \ucda4\ub4e4\uc774 \uc778\uae30\ub97c \uc5bb\uc5c8\ub2e4. \ud2b9\ud788 '\uc608\uc608 \ucda4'\uc740 \ud32c\ub4e4\uc774 \uace0\ubb34\uc7a5\uac11\uc744 \ucc29\uc6a9\ud558\uace0 \uadf8 \ucda4\uc744 \ud568\uaed8 \ucd94\ub294 \uc751\uc6d0\uc744 \uc120\ubcf4\uc600\uace0, \uc774\uac83 \ub610\ud55c \uc5ed\uc2dc \ub9ce\uc740 \uad00\uc2ec\uc758 \ub300\uc0c1\uc774 \ub418\uc5c8\ub2e4. \"Pretty Girl\"\uc740 \ud0dc\uad6d\uc5d0\uc11c\ub294 \ud65c\ub3d9 \uc5c6\uc774 Pop Chart 1\uc704\ub97c \ud558\ub294 \uc601\uad11\uc744 \uc548\uac8c \ub418\uc5c8\ub2e4. \"Pretty Girl\"\uc740 \ub610\ud55c \ud574\uc678 \ud65c\ub3d9\uc744 \uc804\ud600 \ud558\uc9c0 \uc54a\uc558\uc9c0\ub9cc 2009\ub144\uc5d0 \ud0dc\uad6d AS TV \uc544\uc2dc\uc544 \ud31d \ucc28\ud2b8\uc5d0\uc11c 1\uc704\ub97c \ud588\ub2e4. \uadf8 \ud6c4, \ub3d9\ub0a8\uc544\uc2dc\uc544\uc640 \uc77c\ubcf8, \uc911\uad6d\uc5d0\uc11c \ub9ce\uc740 \ub7ec\ube0c\ucf5c\uc744 \ubc1b\uc558\ub2e4. 2009\ub144 9\uc6d4 17\uc77c\uc5d0\ub294 \ud0dc\uad6d\uc5d0\uc11c Thai-Korean Concert\uc5d0 \ucd9c\uc5f0\ud574\uc11c \ub9ce\uc740 \ud574\uc678\ud32c\ub4e4\uc744 \uc990\uac81\uac8c \ud558\uc600\uc73c\uba70, 2009\ub144 \ud0dc\uad6d Channel V Asia TOP 50\uc5d0\uc11c \"Wanna\"\uc640 \" Honey \"\uac00 \uac01\uac01 31\uc704, 46\uc704\uc5d0 \ub7ad\ud06c\ub418\uae30\ub3c4 \ud558\uc600\ub2e4. 2009\ub144 2\uc6d4 10\uc77c, \"Pretty Girl\" \ud65c\ub3d9 \ud6c4 \uce74\ub77c\ub294 \ud22c\ud45c\ub97c \ud1b5\ud574 \ud6c4\uc18d\uace1\uc73c\ub85c \ubf51\ud78c \"Honey\"\uc758 \uc548\ubb34 \ub3d9\uc601\uc0c1\uc744 \uacf5\uac1c\ud558\uc600\uc73c\uba70 2\uc6d4 13\uc77c \ud2f0\uc800\uc601\uc0c1\uc744 \uacf5\uac1c\ud558\uc600\ub2e4. \"Honey\"\ub294 \ub450 \ubc88\uc9f8 \ubbf8\ub2c8\uc568\ubc94\uc5d0 \uc218\ub85d\ub41c \uace1 \"\ud558\ub2c8\"\uc758 \uc7ac\ud3b8\uc9d1\uace1\uc774\uae30\ub3c4 \ud558\ub2e4. 2\uc6d4 14\uc77c \uc1fc! \uc74c\uc545\uc911\uc2ec\uc5d0\uc11c \uccab \ubb34\ub300\ub97c \uac00\uc9c0\uace0, 2\uc6d4 16\uc77c \ubba4\uc9c1 \ube44\ub514\uc624\ub97c \uacf5\uac1c\ud558\uc600\ub2e4. \uc774\uc5b4 2\uc6d4 18\uc77c \uccab \uc2a4\ud398\uc15c \uc568\ubc94\uc744 \ubc1c\ub9e4\ud558\uc600\ub2e4. \"Honey\"\ub294 \uc548\ubb34 \uc911 '\uafc0 \ucc0d\uc5b4\uba39\uae30 \ucda4'\uc774 \ud070 \uc778\uae30\ub97c \ub04c\uc5c8\uace0, \uc74c\uc6d0 \ucc28\ud2b8\uc5d0\uc11c\ub3c4 \uc0c1\uc704\uad8c\uc5d0 \ub7ad\ud06c \ub418\uc5b4 3\uc6d4 5\uc77c\uc5d0\ub294 Mnet \u300a\uc5e0\uce74\uc6b4\ud2b8\ub2e4\uc6b4\u300b\uc5d0\uc11c \ub9c8\uce68\ub0b4 \ub370\ubdd4 \ud6c4 \uc57d 2\ub144 \ub9cc\uc5d0 \ucc98\uc74c\uc73c\ub85c \uc74c\uc545\ud504\ub85c\uadf8\ub7a8 1\uc704\uc758 \uac10\uaca9\uc744 \ub9db\ubcf4\uc558\ub2e4. \ub610\ud55c 3\uc6d4 8\uc77c \uc778\uae30\uac00\uc694\uc5d0\uc11c\ub294 \ubba4\ud2f0\uc98c\uc1a1\uc744 \uc218\uc0c1\ud574 \ucc98\uc74c\uc73c\ub85c \uacf5\uc911\ud30c 1\uc704\ub97c \ucc28\uc9c0\ud588\uace0, 4\uc6d4 \ub9d0 \ub9c8\uc9c0\ub9c9 \ubb34\ub300\ub85c\uc368 \ud65c\ub3d9\uc744 \ub9c8\ubb34\ub9ac\ud588\ub2e4.", "sentence_answer": "2009\ub144 9\uc6d4 17\uc77c\uc5d0\ub294 \ud0dc\uad6d\uc5d0\uc11c Thai-Korean Concert\uc5d0 \ucd9c\uc5f0\ud574\uc11c \ub9ce\uc740 \ud574\uc678\ud32c\ub4e4\uc744 \uc990\uac81\uac8c \ud558\uc600\uc73c\uba70, 2009\ub144 \ud0dc\uad6d Channel V Asia TOP 50\uc5d0\uc11c \"Wanna\"\uc640 \" Honey \"\uac00 \uac01\uac01 31\uc704, 46\uc704\uc5d0 \ub7ad\ud06c\ub418\uae30\ub3c4 \ud558\uc600\ub2e4.", "paragraph_id": "5901372-1-1", "paragraph_question": "question: \uce74\ub77c\uac00 \ub370\ubdd4 \ud6c4 \ucc98\uc74c\uc73c\ub85c 1\uc704\ub97c \ud55c \uace1\uc740?, context: 11\uc6d4 29\uc77c, \ub450 \ubc88\uc9f8 \ubbf8\ub2c8\uc568\ubc94 Pretty Girl\uc758 \ud0c0\uc774\ud2c0\uace1 \"Pretty Girl\"\uc758 \ud2f0\uc800\uc601\uc0c1\uc774 \uacf5\uac1c\ub418\uc5c8\ub2e4. \uc774 \ud2f0\uc800\uc601\uc0c1\uc740 12\uc2dc\uac04 \ub9cc\uc5d0 \uc870\ud68c\uc218 5\ub9cc\uc744 \ub118\ub294 \ub4f1 \ub300\ubc15\uc744 \ud130\ub728\ub9b4 \ub9cc\ud55c \uc870\uc9d0\uc744 \ubcf4\uc774\uae30 \uc2dc\uc791\ud588\ub2e4. 12\uc6d4 4\uc77c \uc568\ubc94 \ubc1c\ub9e4\uc640 \ud568\uaed8 M.net \u300a\uc5e0\uce74\uc6b4\ud2b8\ub2e4\uc6b4\u300b\uc5d0\uc11c \ucef4\ubc31\ubb34\ub300\ub97c \uac00\uc9c0\uac8c \ub418\uc5c8\ub2e4. \"Pretty Girl\"\uc740 \uce74\ub77c \ub370\ubdd4 \uc774\ub798 \ucc98\uc74c\uc73c\ub85c \ucc28\ud2b8\uc5d0\uc11c 2\uc704\uae4c\uc9c0 \uc624\ub974\ub294 \uc704\ub825\uc744 \ubcf4\uc600\uc73c\uba70, \ub354\ubd88\uc5b4 \uc548\ubb34\uc778 '\ud654\uc7a5\ucda4', '\ub2f9\ub2f9\ud558\uac8c \uac77\uae30 \ucda4', '\uc608\uc608 \ucda4' \ub4f1\uc758 \ucda4\ub4e4\uc774 \uc778\uae30\ub97c \uc5bb\uc5c8\ub2e4. \ud2b9\ud788 '\uc608\uc608 \ucda4'\uc740 \ud32c\ub4e4\uc774 \uace0\ubb34\uc7a5\uac11\uc744 \ucc29\uc6a9\ud558\uace0 \uadf8 \ucda4\uc744 \ud568\uaed8 \ucd94\ub294 \uc751\uc6d0\uc744 \uc120\ubcf4\uc600\uace0, \uc774\uac83 \ub610\ud55c \uc5ed\uc2dc \ub9ce\uc740 \uad00\uc2ec\uc758 \ub300\uc0c1\uc774 \ub418\uc5c8\ub2e4. \"Pretty Girl\"\uc740 \ud0dc\uad6d\uc5d0\uc11c\ub294 \ud65c\ub3d9 \uc5c6\uc774 Pop Chart 1\uc704\ub97c \ud558\ub294 \uc601\uad11\uc744 \uc548\uac8c \ub418\uc5c8\ub2e4. \"Pretty Girl\"\uc740 \ub610\ud55c \ud574\uc678 \ud65c\ub3d9\uc744 \uc804\ud600 \ud558\uc9c0 \uc54a\uc558\uc9c0\ub9cc 2009\ub144\uc5d0 \ud0dc\uad6d AS TV \uc544\uc2dc\uc544 \ud31d \ucc28\ud2b8\uc5d0\uc11c 1\uc704\ub97c \ud588\ub2e4. \uadf8 \ud6c4, \ub3d9\ub0a8\uc544\uc2dc\uc544\uc640 \uc77c\ubcf8, \uc911\uad6d\uc5d0\uc11c \ub9ce\uc740 \ub7ec\ube0c\ucf5c\uc744 \ubc1b\uc558\ub2e4. 2009\ub144 9\uc6d4 17\uc77c\uc5d0\ub294 \ud0dc\uad6d\uc5d0\uc11c Thai-Korean Concert\uc5d0 \ucd9c\uc5f0\ud574\uc11c \ub9ce\uc740 \ud574\uc678\ud32c\ub4e4\uc744 \uc990\uac81\uac8c \ud558\uc600\uc73c\uba70, 2009\ub144 \ud0dc\uad6d Channel V Asia TOP 50\uc5d0\uc11c \"Wanna\"\uc640 \"Honey\"\uac00 \uac01\uac01 31\uc704, 46\uc704\uc5d0 \ub7ad\ud06c\ub418\uae30\ub3c4 \ud558\uc600\ub2e4. 2009\ub144 2\uc6d4 10\uc77c, \"Pretty Girl\" \ud65c\ub3d9 \ud6c4 \uce74\ub77c\ub294 \ud22c\ud45c\ub97c \ud1b5\ud574 \ud6c4\uc18d\uace1\uc73c\ub85c \ubf51\ud78c \"Honey\"\uc758 \uc548\ubb34 \ub3d9\uc601\uc0c1\uc744 \uacf5\uac1c\ud558\uc600\uc73c\uba70 2\uc6d4 13\uc77c \ud2f0\uc800\uc601\uc0c1\uc744 \uacf5\uac1c\ud558\uc600\ub2e4. \"Honey\"\ub294 \ub450 \ubc88\uc9f8 \ubbf8\ub2c8\uc568\ubc94\uc5d0 \uc218\ub85d\ub41c \uace1 \"\ud558\ub2c8\"\uc758 \uc7ac\ud3b8\uc9d1\uace1\uc774\uae30\ub3c4 \ud558\ub2e4. 2\uc6d4 14\uc77c \uc1fc! \uc74c\uc545\uc911\uc2ec\uc5d0\uc11c \uccab \ubb34\ub300\ub97c \uac00\uc9c0\uace0, 2\uc6d4 16\uc77c \ubba4\uc9c1 \ube44\ub514\uc624\ub97c \uacf5\uac1c\ud558\uc600\ub2e4. \uc774\uc5b4 2\uc6d4 18\uc77c \uccab \uc2a4\ud398\uc15c \uc568\ubc94\uc744 \ubc1c\ub9e4\ud558\uc600\ub2e4. \"Honey\"\ub294 \uc548\ubb34 \uc911 '\uafc0 \ucc0d\uc5b4\uba39\uae30 \ucda4'\uc774 \ud070 \uc778\uae30\ub97c \ub04c\uc5c8\uace0, \uc74c\uc6d0 \ucc28\ud2b8\uc5d0\uc11c\ub3c4 \uc0c1\uc704\uad8c\uc5d0 \ub7ad\ud06c \ub418\uc5b4 3\uc6d4 5\uc77c\uc5d0\ub294 Mnet \u300a\uc5e0\uce74\uc6b4\ud2b8\ub2e4\uc6b4\u300b\uc5d0\uc11c \ub9c8\uce68\ub0b4 \ub370\ubdd4 \ud6c4 \uc57d 2\ub144 \ub9cc\uc5d0 \ucc98\uc74c\uc73c\ub85c \uc74c\uc545\ud504\ub85c\uadf8\ub7a8 1\uc704\uc758 \uac10\uaca9\uc744 \ub9db\ubcf4\uc558\ub2e4. \ub610\ud55c 3\uc6d4 8\uc77c \uc778\uae30\uac00\uc694\uc5d0\uc11c\ub294 \ubba4\ud2f0\uc98c\uc1a1\uc744 \uc218\uc0c1\ud574 \ucc98\uc74c\uc73c\ub85c \uacf5\uc911\ud30c 1\uc704\ub97c \ucc28\uc9c0\ud588\uace0, 4\uc6d4 \ub9d0 \ub9c8\uc9c0\ub9c9 \ubb34\ub300\ub85c\uc368 \ud65c\ub3d9\uc744 \ub9c8\ubb34\ub9ac\ud588\ub2e4."} {"question": "\ubb34\ub77c\ud0c0\ub294 1976 \ubb34\uc5c7\uc744 \uc2b5\ub4dd\ud558\uc5ec \uc608\ub9ac\ud558\uac8c \ub0a0\uce74\ub85c\uc6b4 \ubcc0\ud654\ub97c \uc774\ub8e8\uc5c8\ub098?", "paragraph": "1976\ub144\uc5d0\ub294 \ud3ec\ud06c\ubcfc\uc744 \uc2b5\ub4dd\ud588\ub294\ub370 \ubcf4\ud1b5 \ud3c9\ubc94\ud558\uc9c0 \uc54a\uc740 \uae34 \uc190\uac00\ub77d\uc774 \ub0b3\ub294 \uc608\ub9ac\ud560 \uc815\ub3c4\uc758 \ub0a0\uce74\ub85c\uc6b4 \ubcc0\ud654\ub294 \ub9e4\ubc88 \ud0c0\uc790\uc758 \ubc29\ub9dd\uc774\uac00 \ud5c8\uacf5\uc744 \uac00\ub974\uac8c \ud588\ub2e4. \uadf8 \ud574\uc758 \uc2dc\uc98c\uc5d0\ub294 257\uc774\ub2dd\uc744 \ub358\uc838 21\uc2b9\uc744 \uc62c\ub9ac\ub294 \uac83\uacfc \ub3d9\uc2dc\uc5d0 \ud3c9\uade0 \uc790\ucc45\uc810 1.82\ub97c \uae30\ub85d\ud558\uc5ec \ucd5c\uc6b0\uc218 \ud3c9\uade0 \uc790\ucc45\uc810 \ud0c0\uc774\ud2c0\uc744 \ud68d\ub4dd\ud588\uace0 202\uac1c\uc758 \ub9ac\uadf8 \ucd5c\ub2e4 \ud0c8\uc0bc\uc9c4\uc744 \uae30\ub85d\ud588\ub2e4. \ud3ec\uc218\uc5d0\uac8c \uc758\uc874\ud558\uc9c0 \uc54a\uace0 \u2018\ub178 \uc0ac\uc778\u2019\uc73c\ub85c \uc2a4\uc2a4\ub85c \ud22c\uad6c\ub97c \uc870\uc808\ud588\uae30 \ub54c\ubb38\uc5d0 1976\ub144\ubd80\ud130 1979\ub144\uae4c\uc9c0 4\ub144 \uc5f0\uc18d\uc73c\ub85c \ub450 \uc790\ub9bf\uc218 \ud3ed\ud22c\ub97c \uae30\ub85d\ud588\ub2e4. 1979\ub144\uc5d0\ub294 32\uacbd\uae30\uc5d0 \uc120\ubc1c\ub85c \ub098\uc640 \uac1c\uc778 \ucd5c\ub2e4\uc778 21\ucc28\ub840 \uc644\ud22c\uc640 230\uac1c\uc758 \ud0c8\uc0bc\uc9c4\uc744 \uae30\ub85d\ud588\ub2e4. 1981\ub144\uc5d0\ub294 \uac1c\ub9c9 11\uc5f0\uc2b9\uc744 \uc7a5\uc2dd\ud558\uc5ec \uc2dc\uc98c 19\uc2b9\uc744 \uae30\ub85d\ud558\ub294 \ub4f1 \ub2e4\uc2b9\uc655 \ud0c0\uc774\ud2c0\ub3c4 \ud68d\ub4dd\ud574 \uc2a4\uc988\ud0a4 \uac8c\uc774\uc2dc, \uc57c\ub9c8\ub2e4 \ud788\uc0ac\uc2dc, \ud788\uac00\uc2dc\uc624 \uc624\uc0ac\ubb34 \ub4f1\uacfc \ud568\uaed8 1970\ub144\ub300\ubd80\ud130 1980\ub144\ub300\uae4c\uc9c0 \ud37c\uc2dc\ud53d \ub9ac\uadf8\ub97c \ub300\ud45c\ud558\ub294 \uac04\ud310 \ud22c\uc218\uac00 \ub410\ub2e4.", "answer": "\ud3ec\ud06c\ubcfc", "sentence": "1976\ub144\uc5d0\ub294 \ud3ec\ud06c\ubcfc \uc744 \uc2b5\ub4dd\ud588\ub294\ub370 \ubcf4\ud1b5 \ud3c9\ubc94\ud558\uc9c0 \uc54a\uc740 \uae34 \uc190\uac00\ub77d\uc774 \ub0b3\ub294 \uc608\ub9ac\ud560 \uc815\ub3c4\uc758 \ub0a0\uce74\ub85c\uc6b4 \ubcc0\ud654\ub294 \ub9e4\ubc88 \ud0c0\uc790\uc758 \ubc29\ub9dd\uc774\uac00 \ud5c8\uacf5\uc744 \uac00\ub974\uac8c \ud588\ub2e4.", "paragraph_sentence": " 1976\ub144\uc5d0\ub294 \ud3ec\ud06c\ubcfc \uc744 \uc2b5\ub4dd\ud588\ub294\ub370 \ubcf4\ud1b5 \ud3c9\ubc94\ud558\uc9c0 \uc54a\uc740 \uae34 \uc190\uac00\ub77d\uc774 \ub0b3\ub294 \uc608\ub9ac\ud560 \uc815\ub3c4\uc758 \ub0a0\uce74\ub85c\uc6b4 \ubcc0\ud654\ub294 \ub9e4\ubc88 \ud0c0\uc790\uc758 \ubc29\ub9dd\uc774\uac00 \ud5c8\uacf5\uc744 \uac00\ub974\uac8c \ud588\ub2e4. \uadf8 \ud574\uc758 \uc2dc\uc98c\uc5d0\ub294 257\uc774\ub2dd\uc744 \ub358\uc838 21\uc2b9\uc744 \uc62c\ub9ac\ub294 \uac83\uacfc \ub3d9\uc2dc\uc5d0 \ud3c9\uade0 \uc790\ucc45\uc810 1.82\ub97c \uae30\ub85d\ud558\uc5ec \ucd5c\uc6b0\uc218 \ud3c9\uade0 \uc790\ucc45\uc810 \ud0c0\uc774\ud2c0\uc744 \ud68d\ub4dd\ud588\uace0 202\uac1c\uc758 \ub9ac\uadf8 \ucd5c\ub2e4 \ud0c8\uc0bc\uc9c4\uc744 \uae30\ub85d\ud588\ub2e4. \ud3ec\uc218\uc5d0\uac8c \uc758\uc874\ud558\uc9c0 \uc54a\uace0 \u2018\ub178 \uc0ac\uc778\u2019\uc73c\ub85c \uc2a4\uc2a4\ub85c \ud22c\uad6c\ub97c \uc870\uc808\ud588\uae30 \ub54c\ubb38\uc5d0 1976\ub144\ubd80\ud130 1979\ub144\uae4c\uc9c0 4\ub144 \uc5f0\uc18d\uc73c\ub85c \ub450 \uc790\ub9bf\uc218 \ud3ed\ud22c\ub97c \uae30\ub85d\ud588\ub2e4. 1979\ub144\uc5d0\ub294 32\uacbd\uae30\uc5d0 \uc120\ubc1c\ub85c \ub098\uc640 \uac1c\uc778 \ucd5c\ub2e4\uc778 21\ucc28\ub840 \uc644\ud22c\uc640 230\uac1c\uc758 \ud0c8\uc0bc\uc9c4\uc744 \uae30\ub85d\ud588\ub2e4. 1981\ub144\uc5d0\ub294 \uac1c\ub9c9 11\uc5f0\uc2b9\uc744 \uc7a5\uc2dd\ud558\uc5ec \uc2dc\uc98c 19\uc2b9\uc744 \uae30\ub85d\ud558\ub294 \ub4f1 \ub2e4\uc2b9\uc655 \ud0c0\uc774\ud2c0\ub3c4 \ud68d\ub4dd\ud574 \uc2a4\uc988\ud0a4 \uac8c\uc774\uc2dc, \uc57c\ub9c8\ub2e4 \ud788\uc0ac\uc2dc, \ud788\uac00\uc2dc\uc624 \uc624\uc0ac\ubb34 \ub4f1\uacfc \ud568\uaed8 1970\ub144\ub300\ubd80\ud130 1980\ub144\ub300\uae4c\uc9c0 \ud37c\uc2dc\ud53d \ub9ac\uadf8\ub97c \ub300\ud45c\ud558\ub294 \uac04\ud310 \ud22c\uc218\uac00 \ub410\ub2e4.", "paragraph_answer": "1976\ub144\uc5d0\ub294 \ud3ec\ud06c\ubcfc \uc744 \uc2b5\ub4dd\ud588\ub294\ub370 \ubcf4\ud1b5 \ud3c9\ubc94\ud558\uc9c0 \uc54a\uc740 \uae34 \uc190\uac00\ub77d\uc774 \ub0b3\ub294 \uc608\ub9ac\ud560 \uc815\ub3c4\uc758 \ub0a0\uce74\ub85c\uc6b4 \ubcc0\ud654\ub294 \ub9e4\ubc88 \ud0c0\uc790\uc758 \ubc29\ub9dd\uc774\uac00 \ud5c8\uacf5\uc744 \uac00\ub974\uac8c \ud588\ub2e4. \uadf8 \ud574\uc758 \uc2dc\uc98c\uc5d0\ub294 257\uc774\ub2dd\uc744 \ub358\uc838 21\uc2b9\uc744 \uc62c\ub9ac\ub294 \uac83\uacfc \ub3d9\uc2dc\uc5d0 \ud3c9\uade0 \uc790\ucc45\uc810 1.82\ub97c \uae30\ub85d\ud558\uc5ec \ucd5c\uc6b0\uc218 \ud3c9\uade0 \uc790\ucc45\uc810 \ud0c0\uc774\ud2c0\uc744 \ud68d\ub4dd\ud588\uace0 202\uac1c\uc758 \ub9ac\uadf8 \ucd5c\ub2e4 \ud0c8\uc0bc\uc9c4\uc744 \uae30\ub85d\ud588\ub2e4. \ud3ec\uc218\uc5d0\uac8c \uc758\uc874\ud558\uc9c0 \uc54a\uace0 \u2018\ub178 \uc0ac\uc778\u2019\uc73c\ub85c \uc2a4\uc2a4\ub85c \ud22c\uad6c\ub97c \uc870\uc808\ud588\uae30 \ub54c\ubb38\uc5d0 1976\ub144\ubd80\ud130 1979\ub144\uae4c\uc9c0 4\ub144 \uc5f0\uc18d\uc73c\ub85c \ub450 \uc790\ub9bf\uc218 \ud3ed\ud22c\ub97c \uae30\ub85d\ud588\ub2e4. 1979\ub144\uc5d0\ub294 32\uacbd\uae30\uc5d0 \uc120\ubc1c\ub85c \ub098\uc640 \uac1c\uc778 \ucd5c\ub2e4\uc778 21\ucc28\ub840 \uc644\ud22c\uc640 230\uac1c\uc758 \ud0c8\uc0bc\uc9c4\uc744 \uae30\ub85d\ud588\ub2e4. 1981\ub144\uc5d0\ub294 \uac1c\ub9c9 11\uc5f0\uc2b9\uc744 \uc7a5\uc2dd\ud558\uc5ec \uc2dc\uc98c 19\uc2b9\uc744 \uae30\ub85d\ud558\ub294 \ub4f1 \ub2e4\uc2b9\uc655 \ud0c0\uc774\ud2c0\ub3c4 \ud68d\ub4dd\ud574 \uc2a4\uc988\ud0a4 \uac8c\uc774\uc2dc, \uc57c\ub9c8\ub2e4 \ud788\uc0ac\uc2dc, \ud788\uac00\uc2dc\uc624 \uc624\uc0ac\ubb34 \ub4f1\uacfc \ud568\uaed8 1970\ub144\ub300\ubd80\ud130 1980\ub144\ub300\uae4c\uc9c0 \ud37c\uc2dc\ud53d \ub9ac\uadf8\ub97c \ub300\ud45c\ud558\ub294 \uac04\ud310 \ud22c\uc218\uac00 \ub410\ub2e4.", "sentence_answer": "1976\ub144\uc5d0\ub294 \ud3ec\ud06c\ubcfc \uc744 \uc2b5\ub4dd\ud588\ub294\ub370 \ubcf4\ud1b5 \ud3c9\ubc94\ud558\uc9c0 \uc54a\uc740 \uae34 \uc190\uac00\ub77d\uc774 \ub0b3\ub294 \uc608\ub9ac\ud560 \uc815\ub3c4\uc758 \ub0a0\uce74\ub85c\uc6b4 \ubcc0\ud654\ub294 \ub9e4\ubc88 \ud0c0\uc790\uc758 \ubc29\ub9dd\uc774\uac00 \ud5c8\uacf5\uc744 \uac00\ub974\uac8c \ud588\ub2e4.", "paragraph_id": "6410644-3-0", "paragraph_question": "question: \ubb34\ub77c\ud0c0\ub294 1976 \ubb34\uc5c7\uc744 \uc2b5\ub4dd\ud558\uc5ec \uc608\ub9ac\ud558\uac8c \ub0a0\uce74\ub85c\uc6b4 \ubcc0\ud654\ub97c \uc774\ub8e8\uc5c8\ub098?, context: 1976\ub144\uc5d0\ub294 \ud3ec\ud06c\ubcfc\uc744 \uc2b5\ub4dd\ud588\ub294\ub370 \ubcf4\ud1b5 \ud3c9\ubc94\ud558\uc9c0 \uc54a\uc740 \uae34 \uc190\uac00\ub77d\uc774 \ub0b3\ub294 \uc608\ub9ac\ud560 \uc815\ub3c4\uc758 \ub0a0\uce74\ub85c\uc6b4 \ubcc0\ud654\ub294 \ub9e4\ubc88 \ud0c0\uc790\uc758 \ubc29\ub9dd\uc774\uac00 \ud5c8\uacf5\uc744 \uac00\ub974\uac8c \ud588\ub2e4. \uadf8 \ud574\uc758 \uc2dc\uc98c\uc5d0\ub294 257\uc774\ub2dd\uc744 \ub358\uc838 21\uc2b9\uc744 \uc62c\ub9ac\ub294 \uac83\uacfc \ub3d9\uc2dc\uc5d0 \ud3c9\uade0 \uc790\ucc45\uc810 1.82\ub97c \uae30\ub85d\ud558\uc5ec \ucd5c\uc6b0\uc218 \ud3c9\uade0 \uc790\ucc45\uc810 \ud0c0\uc774\ud2c0\uc744 \ud68d\ub4dd\ud588\uace0 202\uac1c\uc758 \ub9ac\uadf8 \ucd5c\ub2e4 \ud0c8\uc0bc\uc9c4\uc744 \uae30\ub85d\ud588\ub2e4. \ud3ec\uc218\uc5d0\uac8c \uc758\uc874\ud558\uc9c0 \uc54a\uace0 \u2018\ub178 \uc0ac\uc778\u2019\uc73c\ub85c \uc2a4\uc2a4\ub85c \ud22c\uad6c\ub97c \uc870\uc808\ud588\uae30 \ub54c\ubb38\uc5d0 1976\ub144\ubd80\ud130 1979\ub144\uae4c\uc9c0 4\ub144 \uc5f0\uc18d\uc73c\ub85c \ub450 \uc790\ub9bf\uc218 \ud3ed\ud22c\ub97c \uae30\ub85d\ud588\ub2e4. 1979\ub144\uc5d0\ub294 32\uacbd\uae30\uc5d0 \uc120\ubc1c\ub85c \ub098\uc640 \uac1c\uc778 \ucd5c\ub2e4\uc778 21\ucc28\ub840 \uc644\ud22c\uc640 230\uac1c\uc758 \ud0c8\uc0bc\uc9c4\uc744 \uae30\ub85d\ud588\ub2e4. 1981\ub144\uc5d0\ub294 \uac1c\ub9c9 11\uc5f0\uc2b9\uc744 \uc7a5\uc2dd\ud558\uc5ec \uc2dc\uc98c 19\uc2b9\uc744 \uae30\ub85d\ud558\ub294 \ub4f1 \ub2e4\uc2b9\uc655 \ud0c0\uc774\ud2c0\ub3c4 \ud68d\ub4dd\ud574 \uc2a4\uc988\ud0a4 \uac8c\uc774\uc2dc, \uc57c\ub9c8\ub2e4 \ud788\uc0ac\uc2dc, \ud788\uac00\uc2dc\uc624 \uc624\uc0ac\ubb34 \ub4f1\uacfc \ud568\uaed8 1970\ub144\ub300\ubd80\ud130 1980\ub144\ub300\uae4c\uc9c0 \ud37c\uc2dc\ud53d \ub9ac\uadf8\ub97c \ub300\ud45c\ud558\ub294 \uac04\ud310 \ud22c\uc218\uac00 \ub410\ub2e4."} {"question": "\uc624\ubc14\ub9c8\ub294 \uc2ed\ub300\uc2dc\uc808 \uc5b4\ub5a4 \ub9c8\uc57d\uc744 \ubcf5\uc6a9\ud588\ub098?", "paragraph": "\uc624\ubc14\ub9c8\ub294 \uc5b4\ub9b0 \uc2dc\uc808\uc5d0 \ub300\ud558\uc5ec \"\uc544\ubc84\uc9c0\ub294 \ub0b4 \uc8fc\ubcc0 \uc0ac\ub78c\ub4e4\uacfc \uc804\ud600 \ub2e4\ub974\uac8c \uc0dd\uacbc\ub2e4\ub294 \uc810 - \uc544\ubc84\uc9c0\ub294 \ud53c\uce58\ucc98\ub7fc \uc2dc\uaebc\uba53\uace0, \uc5b4\uba38\ub2c8\ub294 \uc6b0\uc720\ucc98\ub7fc \ud558\uc597\ub2e4 - \uc744 \ub098\ub294 \uac1c\uc758\uce58 \uc54a\uc558\ub2e4\"\ub77c\uace0 \ud68c\uc0c1\ud558\uc600\ub2e4. \uadf8\ub294 \uadf8\uc758 \ud22c\uc7c1\uc744 \uc790\uc2e0\uc758 \ub2e4\ubbfc\uc871 \ud608\ud1b5\uacfc \uc0ac\ud68c\uc801 \uc778\uc2dd\uc744 \ud654\ud574\uc2dc\ud0a4\uae30 \uc704\ud55c \uc5b4\ub9b0 \uc131\ub144\uc774\ub77c\uace0 \ub9d0\ud558\uc600\ub2e4. \uc624\ubc14\ub9c8\ub294 \ud638\ub180\ub8f0\ub8e8\uc5d0\uc11c \uc9c0\ub0b8 \uc790\uc2e0\uc758 \uc131\uc7a5\uae30\ub97c \ubc18\ucd94\ud558\uba70, \"\ud558\uc640\uc774\uc5d0\uc11c \uc5bb\ub294 \uae30\ud68c - \uc0c1\ud638 \uc874\uc911\uc758 \ubd84\uc704\uae30 \uc18d\uc5d0\uc11c \ub2e4\uc591\ud55c \ubb38\ud654\ub97c \uacbd\ud5d8\ud55c \uac83 - \ub294 \ub0b4 \uc138\uacc4\uad00\uc5d0\uc11c \uc911\uc694\ud55c \ubd80\ubd84\uc774 \ub418\uc5c8\uc73c\uba70, \ub0b4\uac00 \uac00\uc7a5 \uc544\ub07c\ub294 \uac00\uce58\uc758 \uadfc\uac04\uc774 \ub418\uc5c8\ub2e4\"\ub77c\uace0 \uc37c\ub2e4. \uc624\ubc14\ub9c8\ub294 \ub610 \uc2ed\ub300 \uc2dc\uc808 \uc54c\ucf54\uc62c, \ub9c8\ub9ac\ud654\ub098, \ucf54\uce74\uc778\uc744 \ubcf5\uc6a9\ud55c \uc0ac\uc2e4\uc5d0 \ub300\ud574 \"\ub098\ub294 \ub204\uad6c\uc778\uac00 \ud558\ub294 \uc9c8\ubb38\uc744 \uba38\ub9ac \uc18d\uc5d0\uc11c \uc78a\uc73c\ub824\" \ud588\ub358 \uac83\uc774\ub77c\uace0 \ubc1d\ud614\ub2e4. 2008\ub144 \ub300\ud1b5\ub839 \ud6c4\ubcf4 \uacf5\uac1c \ud1a0\ub860(Civil Forum on the Presidency)\uc5d0\uc11c \uc624\ubc14\ub9c8\ub294 \uc790\uc2e0\uc758 \uace0\ub4f1\ud559\uad50 \uc2dc\uc808 \ub9c8\uc57d\uc5d0 \uc190 \ub304 \uc77c\uc774 \uc790\uc2e0\uc758 \"\ucd5c\ub300\uc758 \ub3c4\ub355\uc801 \uacfc\uc624\"\ub77c\uace0 \ub9d0\ud558\uc600\ub2e4.", "answer": "\ub9c8\ub9ac\ud654\ub098, \ucf54\uce74\uc778", "sentence": "\uc624\ubc14\ub9c8\ub294 \ub610 \uc2ed\ub300 \uc2dc\uc808 \uc54c\ucf54\uc62c, \ub9c8\ub9ac\ud654\ub098, \ucf54\uce74\uc778 \uc744 \ubcf5\uc6a9\ud55c \uc0ac\uc2e4\uc5d0 \ub300\ud574 \"\ub098\ub294 \ub204\uad6c\uc778\uac00 \ud558\ub294 \uc9c8\ubb38\uc744 \uba38\ub9ac \uc18d\uc5d0\uc11c \uc78a\uc73c\ub824\" \ud588\ub358 \uac83\uc774\ub77c\uace0 \ubc1d\ud614\ub2e4.", "paragraph_sentence": "\uc624\ubc14\ub9c8\ub294 \uc5b4\ub9b0 \uc2dc\uc808\uc5d0 \ub300\ud558\uc5ec \"\uc544\ubc84\uc9c0\ub294 \ub0b4 \uc8fc\ubcc0 \uc0ac\ub78c\ub4e4\uacfc \uc804\ud600 \ub2e4\ub974\uac8c \uc0dd\uacbc\ub2e4\ub294 \uc810 - \uc544\ubc84\uc9c0\ub294 \ud53c\uce58\ucc98\ub7fc \uc2dc\uaebc\uba53\uace0, \uc5b4\uba38\ub2c8\ub294 \uc6b0\uc720\ucc98\ub7fc \ud558\uc597\ub2e4 - \uc744 \ub098\ub294 \uac1c\uc758\uce58 \uc54a\uc558\ub2e4\"\ub77c\uace0 \ud68c\uc0c1\ud558\uc600\ub2e4. \uadf8\ub294 \uadf8\uc758 \ud22c\uc7c1\uc744 \uc790\uc2e0\uc758 \ub2e4\ubbfc\uc871 \ud608\ud1b5\uacfc \uc0ac\ud68c\uc801 \uc778\uc2dd\uc744 \ud654\ud574\uc2dc\ud0a4\uae30 \uc704\ud55c \uc5b4\ub9b0 \uc131\ub144\uc774\ub77c\uace0 \ub9d0\ud558\uc600\ub2e4. \uc624\ubc14\ub9c8\ub294 \ud638\ub180\ub8f0\ub8e8\uc5d0\uc11c \uc9c0\ub0b8 \uc790\uc2e0\uc758 \uc131\uc7a5\uae30\ub97c \ubc18\ucd94\ud558\uba70, \"\ud558\uc640\uc774\uc5d0\uc11c \uc5bb\ub294 \uae30\ud68c - \uc0c1\ud638 \uc874\uc911\uc758 \ubd84\uc704\uae30 \uc18d\uc5d0\uc11c \ub2e4\uc591\ud55c \ubb38\ud654\ub97c \uacbd\ud5d8\ud55c \uac83 - \ub294 \ub0b4 \uc138\uacc4\uad00\uc5d0\uc11c \uc911\uc694\ud55c \ubd80\ubd84\uc774 \ub418\uc5c8\uc73c\uba70, \ub0b4\uac00 \uac00\uc7a5 \uc544\ub07c\ub294 \uac00\uce58\uc758 \uadfc\uac04\uc774 \ub418\uc5c8\ub2e4\"\ub77c\uace0 \uc37c\ub2e4. \uc624\ubc14\ub9c8\ub294 \ub610 \uc2ed\ub300 \uc2dc\uc808 \uc54c\ucf54\uc62c, \ub9c8\ub9ac\ud654\ub098, \ucf54\uce74\uc778 \uc744 \ubcf5\uc6a9\ud55c \uc0ac\uc2e4\uc5d0 \ub300\ud574 \"\ub098\ub294 \ub204\uad6c\uc778\uac00 \ud558\ub294 \uc9c8\ubb38\uc744 \uba38\ub9ac \uc18d\uc5d0\uc11c \uc78a\uc73c\ub824\" \ud588\ub358 \uac83\uc774\ub77c\uace0 \ubc1d\ud614\ub2e4. 2008\ub144 \ub300\ud1b5\ub839 \ud6c4\ubcf4 \uacf5\uac1c \ud1a0\ub860(Civil Forum on the Presidency)\uc5d0\uc11c \uc624\ubc14\ub9c8\ub294 \uc790\uc2e0\uc758 \uace0\ub4f1\ud559\uad50 \uc2dc\uc808 \ub9c8\uc57d\uc5d0 \uc190 \ub304 \uc77c\uc774 \uc790\uc2e0\uc758 \"\ucd5c\ub300\uc758 \ub3c4\ub355\uc801 \uacfc\uc624\"\ub77c\uace0 \ub9d0\ud558\uc600\ub2e4.", "paragraph_answer": "\uc624\ubc14\ub9c8\ub294 \uc5b4\ub9b0 \uc2dc\uc808\uc5d0 \ub300\ud558\uc5ec \"\uc544\ubc84\uc9c0\ub294 \ub0b4 \uc8fc\ubcc0 \uc0ac\ub78c\ub4e4\uacfc \uc804\ud600 \ub2e4\ub974\uac8c \uc0dd\uacbc\ub2e4\ub294 \uc810 - \uc544\ubc84\uc9c0\ub294 \ud53c\uce58\ucc98\ub7fc \uc2dc\uaebc\uba53\uace0, \uc5b4\uba38\ub2c8\ub294 \uc6b0\uc720\ucc98\ub7fc \ud558\uc597\ub2e4 - \uc744 \ub098\ub294 \uac1c\uc758\uce58 \uc54a\uc558\ub2e4\"\ub77c\uace0 \ud68c\uc0c1\ud558\uc600\ub2e4. \uadf8\ub294 \uadf8\uc758 \ud22c\uc7c1\uc744 \uc790\uc2e0\uc758 \ub2e4\ubbfc\uc871 \ud608\ud1b5\uacfc \uc0ac\ud68c\uc801 \uc778\uc2dd\uc744 \ud654\ud574\uc2dc\ud0a4\uae30 \uc704\ud55c \uc5b4\ub9b0 \uc131\ub144\uc774\ub77c\uace0 \ub9d0\ud558\uc600\ub2e4. \uc624\ubc14\ub9c8\ub294 \ud638\ub180\ub8f0\ub8e8\uc5d0\uc11c \uc9c0\ub0b8 \uc790\uc2e0\uc758 \uc131\uc7a5\uae30\ub97c \ubc18\ucd94\ud558\uba70, \"\ud558\uc640\uc774\uc5d0\uc11c \uc5bb\ub294 \uae30\ud68c - \uc0c1\ud638 \uc874\uc911\uc758 \ubd84\uc704\uae30 \uc18d\uc5d0\uc11c \ub2e4\uc591\ud55c \ubb38\ud654\ub97c \uacbd\ud5d8\ud55c \uac83 - \ub294 \ub0b4 \uc138\uacc4\uad00\uc5d0\uc11c \uc911\uc694\ud55c \ubd80\ubd84\uc774 \ub418\uc5c8\uc73c\uba70, \ub0b4\uac00 \uac00\uc7a5 \uc544\ub07c\ub294 \uac00\uce58\uc758 \uadfc\uac04\uc774 \ub418\uc5c8\ub2e4\"\ub77c\uace0 \uc37c\ub2e4. \uc624\ubc14\ub9c8\ub294 \ub610 \uc2ed\ub300 \uc2dc\uc808 \uc54c\ucf54\uc62c, \ub9c8\ub9ac\ud654\ub098, \ucf54\uce74\uc778 \uc744 \ubcf5\uc6a9\ud55c \uc0ac\uc2e4\uc5d0 \ub300\ud574 \"\ub098\ub294 \ub204\uad6c\uc778\uac00 \ud558\ub294 \uc9c8\ubb38\uc744 \uba38\ub9ac \uc18d\uc5d0\uc11c \uc78a\uc73c\ub824\" \ud588\ub358 \uac83\uc774\ub77c\uace0 \ubc1d\ud614\ub2e4. 2008\ub144 \ub300\ud1b5\ub839 \ud6c4\ubcf4 \uacf5\uac1c \ud1a0\ub860(Civil Forum on the Presidency)\uc5d0\uc11c \uc624\ubc14\ub9c8\ub294 \uc790\uc2e0\uc758 \uace0\ub4f1\ud559\uad50 \uc2dc\uc808 \ub9c8\uc57d\uc5d0 \uc190 \ub304 \uc77c\uc774 \uc790\uc2e0\uc758 \"\ucd5c\ub300\uc758 \ub3c4\ub355\uc801 \uacfc\uc624\"\ub77c\uace0 \ub9d0\ud558\uc600\ub2e4.", "sentence_answer": "\uc624\ubc14\ub9c8\ub294 \ub610 \uc2ed\ub300 \uc2dc\uc808 \uc54c\ucf54\uc62c, \ub9c8\ub9ac\ud654\ub098, \ucf54\uce74\uc778 \uc744 \ubcf5\uc6a9\ud55c \uc0ac\uc2e4\uc5d0 \ub300\ud574 \"\ub098\ub294 \ub204\uad6c\uc778\uac00 \ud558\ub294 \uc9c8\ubb38\uc744 \uba38\ub9ac \uc18d\uc5d0\uc11c \uc78a\uc73c\ub824\" \ud588\ub358 \uac83\uc774\ub77c\uace0 \ubc1d\ud614\ub2e4.", "paragraph_id": "6581697-1-1", "paragraph_question": "question: \uc624\ubc14\ub9c8\ub294 \uc2ed\ub300\uc2dc\uc808 \uc5b4\ub5a4 \ub9c8\uc57d\uc744 \ubcf5\uc6a9\ud588\ub098?, context: \uc624\ubc14\ub9c8\ub294 \uc5b4\ub9b0 \uc2dc\uc808\uc5d0 \ub300\ud558\uc5ec \"\uc544\ubc84\uc9c0\ub294 \ub0b4 \uc8fc\ubcc0 \uc0ac\ub78c\ub4e4\uacfc \uc804\ud600 \ub2e4\ub974\uac8c \uc0dd\uacbc\ub2e4\ub294 \uc810 - \uc544\ubc84\uc9c0\ub294 \ud53c\uce58\ucc98\ub7fc \uc2dc\uaebc\uba53\uace0, \uc5b4\uba38\ub2c8\ub294 \uc6b0\uc720\ucc98\ub7fc \ud558\uc597\ub2e4 - \uc744 \ub098\ub294 \uac1c\uc758\uce58 \uc54a\uc558\ub2e4\"\ub77c\uace0 \ud68c\uc0c1\ud558\uc600\ub2e4. \uadf8\ub294 \uadf8\uc758 \ud22c\uc7c1\uc744 \uc790\uc2e0\uc758 \ub2e4\ubbfc\uc871 \ud608\ud1b5\uacfc \uc0ac\ud68c\uc801 \uc778\uc2dd\uc744 \ud654\ud574\uc2dc\ud0a4\uae30 \uc704\ud55c \uc5b4\ub9b0 \uc131\ub144\uc774\ub77c\uace0 \ub9d0\ud558\uc600\ub2e4. \uc624\ubc14\ub9c8\ub294 \ud638\ub180\ub8f0\ub8e8\uc5d0\uc11c \uc9c0\ub0b8 \uc790\uc2e0\uc758 \uc131\uc7a5\uae30\ub97c \ubc18\ucd94\ud558\uba70, \"\ud558\uc640\uc774\uc5d0\uc11c \uc5bb\ub294 \uae30\ud68c - \uc0c1\ud638 \uc874\uc911\uc758 \ubd84\uc704\uae30 \uc18d\uc5d0\uc11c \ub2e4\uc591\ud55c \ubb38\ud654\ub97c \uacbd\ud5d8\ud55c \uac83 - \ub294 \ub0b4 \uc138\uacc4\uad00\uc5d0\uc11c \uc911\uc694\ud55c \ubd80\ubd84\uc774 \ub418\uc5c8\uc73c\uba70, \ub0b4\uac00 \uac00\uc7a5 \uc544\ub07c\ub294 \uac00\uce58\uc758 \uadfc\uac04\uc774 \ub418\uc5c8\ub2e4\"\ub77c\uace0 \uc37c\ub2e4. \uc624\ubc14\ub9c8\ub294 \ub610 \uc2ed\ub300 \uc2dc\uc808 \uc54c\ucf54\uc62c, \ub9c8\ub9ac\ud654\ub098, \ucf54\uce74\uc778\uc744 \ubcf5\uc6a9\ud55c \uc0ac\uc2e4\uc5d0 \ub300\ud574 \"\ub098\ub294 \ub204\uad6c\uc778\uac00 \ud558\ub294 \uc9c8\ubb38\uc744 \uba38\ub9ac \uc18d\uc5d0\uc11c \uc78a\uc73c\ub824\" \ud588\ub358 \uac83\uc774\ub77c\uace0 \ubc1d\ud614\ub2e4. 2008\ub144 \ub300\ud1b5\ub839 \ud6c4\ubcf4 \uacf5\uac1c \ud1a0\ub860(Civil Forum on the Presidency)\uc5d0\uc11c \uc624\ubc14\ub9c8\ub294 \uc790\uc2e0\uc758 \uace0\ub4f1\ud559\uad50 \uc2dc\uc808 \ub9c8\uc57d\uc5d0 \uc190 \ub304 \uc77c\uc774 \uc790\uc2e0\uc758 \"\ucd5c\ub300\uc758 \ub3c4\ub355\uc801 \uacfc\uc624\"\ub77c\uace0 \ub9d0\ud558\uc600\ub2e4."} {"question": "\uc870\uc120\uc5d0 \ub300\ud55c \uc18c\uadf9\uc801 \uc815\ucc45\uc744 \uc801\uadf9\uc801 \uc815\ucc45\uc73c\ub85c \ubc14\uafb8\uc5b4 \ub3c5\ub9bd\ud611\ud68c\uc758 \ubc18\uc678\uc138\uc6b4\ub3d9\uc744 \uc774\ub048 \ub098\ub77c\ub294?", "paragraph": "\ud55c\ud3b8 \uc774\uad8c\uce68\ud0c8\uc5d0 \ub300\ud574\uc11c\ub3c4, \uadf8\uac83\uc744 \uc790\uc6d0 \uac1c\ubc1c\ub85c \uc774\ud574\ud558\uc5ec, \ud56d\uc0c1 \ubc18\ub300\ud558\ub294 \ubd80\uc815\uc801 \uc785\uc7a5\uc744 \uac00\uc9c0\uc9c0 \uc54a\uc558\ub2e4. \uadf8\ub4e4\uc740 \uc5f4\uac15\uc758 \uc2dd\ubbfc\uc9c0 \uce68\ud0c8\uc744 \uc0ac\ud68c\uc9c4\ud654\ub860\uc5d0 \ub530\ub77c \uc138\uacc4 \uc9c8\uc11c\ub97c \uc0dd\uc874\uacbd\uc7c1\uacfc \uc801\uc790\uc0dd\uc874\uc758 \uc6d0\ub9ac\ub85c \uc774\ud574\ud558\uace0 \uae0d\uc815\ud558\uc600\uace0, \uc870\uc120\uc5d0 \ub300\ud55c \uc774\uad8c \uce68\ud0c8\ub3c4 \uac19\uc740 \ub9e5\ub77d\uc5d0\uc11c \ubc14\ub77c\ubcf4\uc558\ub2e4. \uadf8\ub4e4\uc774 \uac00\uc9c4 \uc678\uc138\uc5d0 \ub300\ud55c \uc704\uae30\uc758\uc2dd\uc740 \uc815\uce58\u00b7\uad70\uc0ac\uc801 \uce68\ud0c8\uc5d0 \uace0\uc815\ub418\uc5b4 \uc788\uc5b4 \uacbd\uc81c\u00b7\ubb38\ud654\uc801 \uce68\ud0c8\uc740 \uc624\ud788\ub824 \uc870\uc120\uc758 \uadfc\ub300\ud654\ub97c \uc55e\ub2f9\uae34\ub2e4\uace0 \ubc1b\uc544\ub4e4\uc600\ub2e4. \uc774\uc5d0 \ub530\ub77c \ub7ec\uc2dc\uc544\uac00 \uc870\uc120\uc5d0 \ub300\ud574 \uc18c\uadf9\uc801 \uc815\ucc45\uc744 \ud3bc\uce58\ub2e4\uac00 \uc801\uadf9\uc801 \uc815\ucc45\uc73c\ub85c \ubc14\uafb8\uc790 \ubc18\ubc1c\ud558\uc5ec \ubc18\ub7ec\uc6b4\ub3d9\uc744 \uc804\uc7ac\ud55c \uc77c\ub3c4 \uac19\uc740 \ub9e5\ub77d\uc774\ub2e4. \uc77c\ubd80 \uc5f0\uad6c\uc790\ub4e4\uacfc \uad50\uacfc\uc11c\uac00 \uc8fc\uc7a5 \ud574 \uc654\ub358 \ub3c5\ub9bd\ud611\ud68c\uc758 \ubc18\uc678\uc138\uc6b4\ub3d9\uc774 \ub7ec\uc2dc\uc544\uc640 \uadf8 \ub3d9\ub9f9\uad6d\uc774\uc5c8\ub358 \ud504\ub791\uc2a4 \ub4f1\uc5d0 \uad6d\ud55c\ud568\uc744 \uba85\ud655\ud788\ud574\uc57c \ud55c\ub2e4 \ub294 \ube44\ud310\ub3c4 \uc788\ub2e4.", "answer": "\ub7ec\uc2dc\uc544", "sentence": "\uc774\uc5d0 \ub530\ub77c \ub7ec\uc2dc\uc544 \uac00 \uc870\uc120\uc5d0 \ub300\ud574 \uc18c\uadf9\uc801 \uc815\ucc45\uc744 \ud3bc\uce58\ub2e4\uac00 \uc801\uadf9\uc801 \uc815\ucc45\uc73c\ub85c \ubc14\uafb8\uc790 \ubc18\ubc1c\ud558\uc5ec \ubc18\ub7ec\uc6b4\ub3d9\uc744 \uc804\uc7ac\ud55c \uc77c\ub3c4 \uac19\uc740 \ub9e5\ub77d\uc774\ub2e4.", "paragraph_sentence": "\ud55c\ud3b8 \uc774\uad8c\uce68\ud0c8\uc5d0 \ub300\ud574\uc11c\ub3c4, \uadf8\uac83\uc744 \uc790\uc6d0 \uac1c\ubc1c\ub85c \uc774\ud574\ud558\uc5ec, \ud56d\uc0c1 \ubc18\ub300\ud558\ub294 \ubd80\uc815\uc801 \uc785\uc7a5\uc744 \uac00\uc9c0\uc9c0 \uc54a\uc558\ub2e4. \uadf8\ub4e4\uc740 \uc5f4\uac15\uc758 \uc2dd\ubbfc\uc9c0 \uce68\ud0c8\uc744 \uc0ac\ud68c\uc9c4\ud654\ub860\uc5d0 \ub530\ub77c \uc138\uacc4 \uc9c8\uc11c\ub97c \uc0dd\uc874\uacbd\uc7c1\uacfc \uc801\uc790\uc0dd\uc874\uc758 \uc6d0\ub9ac\ub85c \uc774\ud574\ud558\uace0 \uae0d\uc815\ud558\uc600\uace0, \uc870\uc120\uc5d0 \ub300\ud55c \uc774\uad8c \uce68\ud0c8\ub3c4 \uac19\uc740 \ub9e5\ub77d\uc5d0\uc11c \ubc14\ub77c\ubcf4\uc558\ub2e4. \uadf8\ub4e4\uc774 \uac00\uc9c4 \uc678\uc138\uc5d0 \ub300\ud55c \uc704\uae30\uc758\uc2dd\uc740 \uc815\uce58\u00b7\uad70\uc0ac\uc801 \uce68\ud0c8\uc5d0 \uace0\uc815\ub418\uc5b4 \uc788\uc5b4 \uacbd\uc81c\u00b7\ubb38\ud654\uc801 \uce68\ud0c8\uc740 \uc624\ud788\ub824 \uc870\uc120\uc758 \uadfc\ub300\ud654\ub97c \uc55e\ub2f9\uae34\ub2e4\uace0 \ubc1b\uc544\ub4e4\uc600\ub2e4. \uc774\uc5d0 \ub530\ub77c \ub7ec\uc2dc\uc544 \uac00 \uc870\uc120\uc5d0 \ub300\ud574 \uc18c\uadf9\uc801 \uc815\ucc45\uc744 \ud3bc\uce58\ub2e4\uac00 \uc801\uadf9\uc801 \uc815\ucc45\uc73c\ub85c \ubc14\uafb8\uc790 \ubc18\ubc1c\ud558\uc5ec \ubc18\ub7ec\uc6b4\ub3d9\uc744 \uc804\uc7ac\ud55c \uc77c\ub3c4 \uac19\uc740 \ub9e5\ub77d\uc774\ub2e4. \uc77c\ubd80 \uc5f0\uad6c\uc790\ub4e4\uacfc \uad50\uacfc\uc11c\uac00 \uc8fc\uc7a5 \ud574 \uc654\ub358 \ub3c5\ub9bd\ud611\ud68c\uc758 \ubc18\uc678\uc138\uc6b4\ub3d9\uc774 \ub7ec\uc2dc\uc544\uc640 \uadf8 \ub3d9\ub9f9\uad6d\uc774\uc5c8\ub358 \ud504\ub791\uc2a4 \ub4f1\uc5d0 \uad6d\ud55c\ud568\uc744 \uba85\ud655\ud788\ud574\uc57c \ud55c\ub2e4 \ub294 \ube44\ud310\ub3c4 \uc788\ub2e4.", "paragraph_answer": "\ud55c\ud3b8 \uc774\uad8c\uce68\ud0c8\uc5d0 \ub300\ud574\uc11c\ub3c4, \uadf8\uac83\uc744 \uc790\uc6d0 \uac1c\ubc1c\ub85c \uc774\ud574\ud558\uc5ec, \ud56d\uc0c1 \ubc18\ub300\ud558\ub294 \ubd80\uc815\uc801 \uc785\uc7a5\uc744 \uac00\uc9c0\uc9c0 \uc54a\uc558\ub2e4. \uadf8\ub4e4\uc740 \uc5f4\uac15\uc758 \uc2dd\ubbfc\uc9c0 \uce68\ud0c8\uc744 \uc0ac\ud68c\uc9c4\ud654\ub860\uc5d0 \ub530\ub77c \uc138\uacc4 \uc9c8\uc11c\ub97c \uc0dd\uc874\uacbd\uc7c1\uacfc \uc801\uc790\uc0dd\uc874\uc758 \uc6d0\ub9ac\ub85c \uc774\ud574\ud558\uace0 \uae0d\uc815\ud558\uc600\uace0, \uc870\uc120\uc5d0 \ub300\ud55c \uc774\uad8c \uce68\ud0c8\ub3c4 \uac19\uc740 \ub9e5\ub77d\uc5d0\uc11c \ubc14\ub77c\ubcf4\uc558\ub2e4. \uadf8\ub4e4\uc774 \uac00\uc9c4 \uc678\uc138\uc5d0 \ub300\ud55c \uc704\uae30\uc758\uc2dd\uc740 \uc815\uce58\u00b7\uad70\uc0ac\uc801 \uce68\ud0c8\uc5d0 \uace0\uc815\ub418\uc5b4 \uc788\uc5b4 \uacbd\uc81c\u00b7\ubb38\ud654\uc801 \uce68\ud0c8\uc740 \uc624\ud788\ub824 \uc870\uc120\uc758 \uadfc\ub300\ud654\ub97c \uc55e\ub2f9\uae34\ub2e4\uace0 \ubc1b\uc544\ub4e4\uc600\ub2e4. \uc774\uc5d0 \ub530\ub77c \ub7ec\uc2dc\uc544 \uac00 \uc870\uc120\uc5d0 \ub300\ud574 \uc18c\uadf9\uc801 \uc815\ucc45\uc744 \ud3bc\uce58\ub2e4\uac00 \uc801\uadf9\uc801 \uc815\ucc45\uc73c\ub85c \ubc14\uafb8\uc790 \ubc18\ubc1c\ud558\uc5ec \ubc18\ub7ec\uc6b4\ub3d9\uc744 \uc804\uc7ac\ud55c \uc77c\ub3c4 \uac19\uc740 \ub9e5\ub77d\uc774\ub2e4. \uc77c\ubd80 \uc5f0\uad6c\uc790\ub4e4\uacfc \uad50\uacfc\uc11c\uac00 \uc8fc\uc7a5 \ud574 \uc654\ub358 \ub3c5\ub9bd\ud611\ud68c\uc758 \ubc18\uc678\uc138\uc6b4\ub3d9\uc774 \ub7ec\uc2dc\uc544\uc640 \uadf8 \ub3d9\ub9f9\uad6d\uc774\uc5c8\ub358 \ud504\ub791\uc2a4 \ub4f1\uc5d0 \uad6d\ud55c\ud568\uc744 \uba85\ud655\ud788\ud574\uc57c \ud55c\ub2e4 \ub294 \ube44\ud310\ub3c4 \uc788\ub2e4.", "sentence_answer": "\uc774\uc5d0 \ub530\ub77c \ub7ec\uc2dc\uc544 \uac00 \uc870\uc120\uc5d0 \ub300\ud574 \uc18c\uadf9\uc801 \uc815\ucc45\uc744 \ud3bc\uce58\ub2e4\uac00 \uc801\uadf9\uc801 \uc815\ucc45\uc73c\ub85c \ubc14\uafb8\uc790 \ubc18\ubc1c\ud558\uc5ec \ubc18\ub7ec\uc6b4\ub3d9\uc744 \uc804\uc7ac\ud55c \uc77c\ub3c4 \uac19\uc740 \ub9e5\ub77d\uc774\ub2e4.", "paragraph_id": "6298883-9-2", "paragraph_question": "question: \uc870\uc120\uc5d0 \ub300\ud55c \uc18c\uadf9\uc801 \uc815\ucc45\uc744 \uc801\uadf9\uc801 \uc815\ucc45\uc73c\ub85c \ubc14\uafb8\uc5b4 \ub3c5\ub9bd\ud611\ud68c\uc758 \ubc18\uc678\uc138\uc6b4\ub3d9\uc744 \uc774\ub048 \ub098\ub77c\ub294?, context: \ud55c\ud3b8 \uc774\uad8c\uce68\ud0c8\uc5d0 \ub300\ud574\uc11c\ub3c4, \uadf8\uac83\uc744 \uc790\uc6d0 \uac1c\ubc1c\ub85c \uc774\ud574\ud558\uc5ec, \ud56d\uc0c1 \ubc18\ub300\ud558\ub294 \ubd80\uc815\uc801 \uc785\uc7a5\uc744 \uac00\uc9c0\uc9c0 \uc54a\uc558\ub2e4. \uadf8\ub4e4\uc740 \uc5f4\uac15\uc758 \uc2dd\ubbfc\uc9c0 \uce68\ud0c8\uc744 \uc0ac\ud68c\uc9c4\ud654\ub860\uc5d0 \ub530\ub77c \uc138\uacc4 \uc9c8\uc11c\ub97c \uc0dd\uc874\uacbd\uc7c1\uacfc \uc801\uc790\uc0dd\uc874\uc758 \uc6d0\ub9ac\ub85c \uc774\ud574\ud558\uace0 \uae0d\uc815\ud558\uc600\uace0, \uc870\uc120\uc5d0 \ub300\ud55c \uc774\uad8c \uce68\ud0c8\ub3c4 \uac19\uc740 \ub9e5\ub77d\uc5d0\uc11c \ubc14\ub77c\ubcf4\uc558\ub2e4. \uadf8\ub4e4\uc774 \uac00\uc9c4 \uc678\uc138\uc5d0 \ub300\ud55c \uc704\uae30\uc758\uc2dd\uc740 \uc815\uce58\u00b7\uad70\uc0ac\uc801 \uce68\ud0c8\uc5d0 \uace0\uc815\ub418\uc5b4 \uc788\uc5b4 \uacbd\uc81c\u00b7\ubb38\ud654\uc801 \uce68\ud0c8\uc740 \uc624\ud788\ub824 \uc870\uc120\uc758 \uadfc\ub300\ud654\ub97c \uc55e\ub2f9\uae34\ub2e4\uace0 \ubc1b\uc544\ub4e4\uc600\ub2e4. \uc774\uc5d0 \ub530\ub77c \ub7ec\uc2dc\uc544\uac00 \uc870\uc120\uc5d0 \ub300\ud574 \uc18c\uadf9\uc801 \uc815\ucc45\uc744 \ud3bc\uce58\ub2e4\uac00 \uc801\uadf9\uc801 \uc815\ucc45\uc73c\ub85c \ubc14\uafb8\uc790 \ubc18\ubc1c\ud558\uc5ec \ubc18\ub7ec\uc6b4\ub3d9\uc744 \uc804\uc7ac\ud55c \uc77c\ub3c4 \uac19\uc740 \ub9e5\ub77d\uc774\ub2e4. \uc77c\ubd80 \uc5f0\uad6c\uc790\ub4e4\uacfc \uad50\uacfc\uc11c\uac00 \uc8fc\uc7a5 \ud574 \uc654\ub358 \ub3c5\ub9bd\ud611\ud68c\uc758 \ubc18\uc678\uc138\uc6b4\ub3d9\uc774 \ub7ec\uc2dc\uc544\uc640 \uadf8 \ub3d9\ub9f9\uad6d\uc774\uc5c8\ub358 \ud504\ub791\uc2a4 \ub4f1\uc5d0 \uad6d\ud55c\ud568\uc744 \uba85\ud655\ud788\ud574\uc57c \ud55c\ub2e4 \ub294 \ube44\ud310\ub3c4 \uc788\ub2e4."} {"question": "\ud604\ub300\uc758 \uadf8\ub9ac\uc2a4 \uc2e0\ud654\uc758 \uae30\uc6d0\uc5d0 \ub300\ud55c \uc774\ub860\uc911, \ubaa8\ub4e0 \uace0\ub300 \uc2e0\ud654\uac00 \uc0c1\uc9d5\uc801\uc778 \uc758\ubbf8\ub97c \uac00\uc84c\ub2e4\uace0 \ucd94\uc815\ud558\ub294 \uc774\ub860\uc740?", "paragraph": "\ud604\ub300\uc5d0\ub294 \uadf8\ub9ac\uc2a4 \uc2e0\ud654\uc758 \uae30\uc6d0\uc5d0 \ub300\ud55c \ub2e4\uc591\ud55c \uc774\ub860\uc774 \uc788\ub2e4. \uc131\uc11c\uc801 \uc774\ub860\uc5d0 \ub530\ub974\uba74, \uc2e0\ud654\uc801 \uc804\uc124\uc740 \uc2e4\uc81c \uc0ac\uc2e4\uc5d0\uc11c \uac00\uc7a5\ub418\uace0 \ubc14\ub010 \ubd80\ubd84\uc740 \uc788\uc73c\ub098 \ubaa8\ub450 \uc131\uc11c \uc18d \uc774\uc57c\uae30\uc5d0\uc11c \ube44\ub86f\ub41c \uac83\uc774\ub2e4. \uc5ed\uc0ac\uc801 \uc774\ub860\uc5d0\uc11c\ub294 \uc2e0\ud654\uc5d0\uc11c \uc5b8\uae09\ub418\ub294 \ubaa8\ub4e0 \uc778\ubb3c\uc740 \uc2e4\uc874 \uc778\ubb3c\uc774\uba70, \uadf8\ub4e4\uacfc \uad00\ub828\ub41c \uc804\uc124\uc740 \ub2e8\uc9c0 \ud6c4\ub300\uc5d0 \ub367\ubd99\uc5ec\uc9c4 \uac83\uc774\ub77c\uace0 \ubcf8\ub2e4. \uc774 \uc774\ub860\uc5d0\uc11c\ub294 \uc544\uc774\uc62c\ub85c\uc2a4\uc758 \uc774\uc57c\uae30\ub97c \uc544\uc774\uc62c\ub85c\uc2a4\uac00 \ud2f0\ub808\ub2c8\uc544 \ud574\uc5d0 \uc704\uce58\ud55c \uc5b4\ub5a4 \uc12c\uc758 \uc9c0\ubc30\uc790\uc600\ub2e4\ub294 \uc0ac\uc2e4\uc5d0\uc11c \uc0dd\uaca8\ub09c \uac83\uc73c\ub85c \ucd94\uc815\ud55c\ub2e4. \uc6b0\uc758\uc801 \uc774\ub860\uc740 \ubaa8\ub4e0 \uace0\ub300 \uc2e0\ud654\uac00 \uc6b0\uc758\uc801\uc774\uace0 \uc0c1\uc9d5\uc801\uc778 \uc758\ubbf8\ub97c \uac00\uc84c\ub2e4\uace0 \ucd94\uc815\ud55c\ub2e4. \uc774\uc640\ub294 \ub2e4\ub974\uac8c \ubb3c\ub9ac\uc801 \uc774\ub860\uc5d0\uc11c\ub294 \uacf5\uae30\uc640 \ubd88, \ubb3c\uc758 \uc6d0\uc18c\uac00 \ubcf8\ub798 \uc885\uad50\uc801 \uc22d\ubc30\uc758 \ub300\uc0c1\uc774\uc5c8\uc73c\uba70, \uc8fc\uc694\ud55c \uc2e0\ub4e4\uc774 \uc774\ub7ec\ud55c \uc790\uc5f0\uc758 \ud798\uc744 \uc2e0\uaca9\ud654\ud55c \uac83\uc774\ub77c\ub294 \uc0dd\uac01\uc744 \ub530\ub974\uace0 \uc788\ub2e4. \ub9c9\uc2a4 \ubb90\ub7ec\ub294 \uc778\ub3c4\uc720\ub7fd \uc885\uad50\uc758 \ud615\ud0dc\ub97c \"\ubcf8\ub798\" \uba85\uc2dc\ud558\ub358 \uc544\ub9ac\uc544\uc778\uc758 \ud754\uc801\uc744 \ucc3e\uc544\uac00\ub294 \ubc29\uc2dd\uc744 \ud1b5\ud574\uc11c \uc774\ud574\ud558\uace0\uc790 \ud588\ub2e4. 1891\ub144, \uadf8\ub294 \ub2e4\uc74c\uacfc \uac19\uc774 \uc8fc\uc7a5\ud588\ub2e4. \"19\uc138\uae30\uc5d0 \uc774\ub974\ub7ec \uc644\uc131\ub41c \uc778\ub958 \uace0\ub300\uc0ac\uc5d0 \uad00\ud55c \ucd5c\uace0\ub85c \uc911\uc694\ud55c \ubc1c\uacac\uc740 ... \ub2e4\uc74c\uc758 \ud45c\ubcf8 \ub4f1\uc2dd\uc774\uc5c8\ub2e4. \uc0b0\uc2a4\ud06c\ub9ac\ud2b8\uc5b4 \ub514\uc544\uc6b0\uc2a4 \ud53c\ud2b8\ub974 = \uadf8\ub9ac\uc2a4\uc5b4 \uc81c\uc6b0\uc2a4 = \ub77c\ud2f4\uc5b4 \uc8fc\ud53c\ud130 = \uace0\ub300 \ub178\ub974\ub4dc\uc5b4 \ud2f0\ub974\" \uc11c\ub85c \ub2e4\ub978 \uc9c0\uc5ed\uc758 \uc2e0\ud654\uc5d0\uc11c \ub4dc\ub7ec\ub098\ub294 \ud2b9\uc131\uacfc \uae30\ub2a5\uc740 \ubc00\uc811\ud55c \ud3c9\ud589\uc131\uc744 \ubcf4\uc774\uace0 \uc788\uc73c\uba70, \uc774\uac83\uc740 \uc804\uc2b9\uc774 \uacf5\uc720\ub418\uc5c8\uc74c\uc744 \uc554\uc2dc\ud55c\ub2e4. \ud558\uc9c0\ub9cc, \uc6b0\ub77c\ub204\uc2a4\uc640 \uc0b0\uc2a4\ud06c\ub9ac\ud2b8 \ubc14\ub8e8\ub098 \ub610\ub294 \uadf8\ub9ac\uc2a4\uc758 \ubaa8\uc774\ub77c\uc640 \ub178\ub974\ub4dc \uc2e0\ud654\uc758 \ub178\ub978\uc758 \uacbd\uc6b0\ucc98\ub7fc \uc5b8\uc5b4\uc758 \uc720\uc0ac\uc131\uc744 \ubcf4\uc774\ub294 \uc99d\uac70\uac00 \ubd80\uc871\ud558\uc5ec \ud655\uc2e4\ud55c \uc785\uc99d\uc740 \uc5b4\ub835\ub2e4.", "answer": "\uc6b0\uc758\uc801 \uc774\ub860", "sentence": "\uc6b0\uc758\uc801 \uc774\ub860 \uc740 \ubaa8\ub4e0 \uace0\ub300 \uc2e0\ud654\uac00 \uc6b0\uc758\uc801\uc774\uace0 \uc0c1\uc9d5\uc801\uc778 \uc758\ubbf8\ub97c \uac00\uc84c\ub2e4\uace0 \ucd94\uc815\ud55c\ub2e4.", "paragraph_sentence": "\ud604\ub300\uc5d0\ub294 \uadf8\ub9ac\uc2a4 \uc2e0\ud654\uc758 \uae30\uc6d0\uc5d0 \ub300\ud55c \ub2e4\uc591\ud55c \uc774\ub860\uc774 \uc788\ub2e4. \uc131\uc11c\uc801 \uc774\ub860\uc5d0 \ub530\ub974\uba74, \uc2e0\ud654\uc801 \uc804\uc124\uc740 \uc2e4\uc81c \uc0ac\uc2e4\uc5d0\uc11c \uac00\uc7a5\ub418\uace0 \ubc14\ub010 \ubd80\ubd84\uc740 \uc788\uc73c\ub098 \ubaa8\ub450 \uc131\uc11c \uc18d \uc774\uc57c\uae30\uc5d0\uc11c \ube44\ub86f\ub41c \uac83\uc774\ub2e4. \uc5ed\uc0ac\uc801 \uc774\ub860\uc5d0\uc11c\ub294 \uc2e0\ud654\uc5d0\uc11c \uc5b8\uae09\ub418\ub294 \ubaa8\ub4e0 \uc778\ubb3c\uc740 \uc2e4\uc874 \uc778\ubb3c\uc774\uba70, \uadf8\ub4e4\uacfc \uad00\ub828\ub41c \uc804\uc124\uc740 \ub2e8\uc9c0 \ud6c4\ub300\uc5d0 \ub367\ubd99\uc5ec\uc9c4 \uac83\uc774\ub77c\uace0 \ubcf8\ub2e4. \uc774 \uc774\ub860\uc5d0\uc11c\ub294 \uc544\uc774\uc62c\ub85c\uc2a4\uc758 \uc774\uc57c\uae30\ub97c \uc544\uc774\uc62c\ub85c\uc2a4\uac00 \ud2f0\ub808\ub2c8\uc544 \ud574\uc5d0 \uc704\uce58\ud55c \uc5b4\ub5a4 \uc12c\uc758 \uc9c0\ubc30\uc790\uc600\ub2e4\ub294 \uc0ac\uc2e4\uc5d0\uc11c \uc0dd\uaca8\ub09c \uac83\uc73c\ub85c \ucd94\uc815\ud55c\ub2e4. \uc6b0\uc758\uc801 \uc774\ub860 \uc740 \ubaa8\ub4e0 \uace0\ub300 \uc2e0\ud654\uac00 \uc6b0\uc758\uc801\uc774\uace0 \uc0c1\uc9d5\uc801\uc778 \uc758\ubbf8\ub97c \uac00\uc84c\ub2e4\uace0 \ucd94\uc815\ud55c\ub2e4. \uc774\uc640\ub294 \ub2e4\ub974\uac8c \ubb3c\ub9ac\uc801 \uc774\ub860\uc5d0\uc11c\ub294 \uacf5\uae30\uc640 \ubd88, \ubb3c\uc758 \uc6d0\uc18c\uac00 \ubcf8\ub798 \uc885\uad50\uc801 \uc22d\ubc30\uc758 \ub300\uc0c1\uc774\uc5c8\uc73c\uba70, \uc8fc\uc694\ud55c \uc2e0\ub4e4\uc774 \uc774\ub7ec\ud55c \uc790\uc5f0\uc758 \ud798\uc744 \uc2e0\uaca9\ud654\ud55c \uac83\uc774\ub77c\ub294 \uc0dd\uac01\uc744 \ub530\ub974\uace0 \uc788\ub2e4. \ub9c9\uc2a4 \ubb90\ub7ec\ub294 \uc778\ub3c4\uc720\ub7fd \uc885\uad50\uc758 \ud615\ud0dc\ub97c \"\ubcf8\ub798\" \uba85\uc2dc\ud558\ub358 \uc544\ub9ac\uc544\uc778\uc758 \ud754\uc801\uc744 \ucc3e\uc544\uac00\ub294 \ubc29\uc2dd\uc744 \ud1b5\ud574\uc11c \uc774\ud574\ud558\uace0\uc790 \ud588\ub2e4. 1891\ub144, \uadf8\ub294 \ub2e4\uc74c\uacfc \uac19\uc774 \uc8fc\uc7a5\ud588\ub2e4. \"19\uc138\uae30\uc5d0 \uc774\ub974\ub7ec \uc644\uc131\ub41c \uc778\ub958 \uace0\ub300\uc0ac\uc5d0 \uad00\ud55c \ucd5c\uace0\ub85c \uc911\uc694\ud55c \ubc1c\uacac\uc740 ... \ub2e4\uc74c\uc758 \ud45c\ubcf8 \ub4f1\uc2dd\uc774\uc5c8\ub2e4. \uc0b0\uc2a4\ud06c\ub9ac\ud2b8\uc5b4 \ub514\uc544\uc6b0\uc2a4 \ud53c\ud2b8\ub974 = \uadf8\ub9ac\uc2a4\uc5b4 \uc81c\uc6b0\uc2a4 = \ub77c\ud2f4\uc5b4 \uc8fc\ud53c\ud130 = \uace0\ub300 \ub178\ub974\ub4dc\uc5b4 \ud2f0\ub974\" \uc11c\ub85c \ub2e4\ub978 \uc9c0\uc5ed\uc758 \uc2e0\ud654\uc5d0\uc11c \ub4dc\ub7ec\ub098\ub294 \ud2b9\uc131\uacfc \uae30\ub2a5\uc740 \ubc00\uc811\ud55c \ud3c9\ud589\uc131\uc744 \ubcf4\uc774\uace0 \uc788\uc73c\uba70, \uc774\uac83\uc740 \uc804\uc2b9\uc774 \uacf5\uc720\ub418\uc5c8\uc74c\uc744 \uc554\uc2dc\ud55c\ub2e4. \ud558\uc9c0\ub9cc, \uc6b0\ub77c\ub204\uc2a4\uc640 \uc0b0\uc2a4\ud06c\ub9ac\ud2b8 \ubc14\ub8e8\ub098 \ub610\ub294 \uadf8\ub9ac\uc2a4\uc758 \ubaa8\uc774\ub77c\uc640 \ub178\ub974\ub4dc \uc2e0\ud654\uc758 \ub178\ub978\uc758 \uacbd\uc6b0\ucc98\ub7fc \uc5b8\uc5b4\uc758 \uc720\uc0ac\uc131\uc744 \ubcf4\uc774\ub294 \uc99d\uac70\uac00 \ubd80\uc871\ud558\uc5ec \ud655\uc2e4\ud55c \uc785\uc99d\uc740 \uc5b4\ub835\ub2e4.", "paragraph_answer": "\ud604\ub300\uc5d0\ub294 \uadf8\ub9ac\uc2a4 \uc2e0\ud654\uc758 \uae30\uc6d0\uc5d0 \ub300\ud55c \ub2e4\uc591\ud55c \uc774\ub860\uc774 \uc788\ub2e4. \uc131\uc11c\uc801 \uc774\ub860\uc5d0 \ub530\ub974\uba74, \uc2e0\ud654\uc801 \uc804\uc124\uc740 \uc2e4\uc81c \uc0ac\uc2e4\uc5d0\uc11c \uac00\uc7a5\ub418\uace0 \ubc14\ub010 \ubd80\ubd84\uc740 \uc788\uc73c\ub098 \ubaa8\ub450 \uc131\uc11c \uc18d \uc774\uc57c\uae30\uc5d0\uc11c \ube44\ub86f\ub41c \uac83\uc774\ub2e4. \uc5ed\uc0ac\uc801 \uc774\ub860\uc5d0\uc11c\ub294 \uc2e0\ud654\uc5d0\uc11c \uc5b8\uae09\ub418\ub294 \ubaa8\ub4e0 \uc778\ubb3c\uc740 \uc2e4\uc874 \uc778\ubb3c\uc774\uba70, \uadf8\ub4e4\uacfc \uad00\ub828\ub41c \uc804\uc124\uc740 \ub2e8\uc9c0 \ud6c4\ub300\uc5d0 \ub367\ubd99\uc5ec\uc9c4 \uac83\uc774\ub77c\uace0 \ubcf8\ub2e4. \uc774 \uc774\ub860\uc5d0\uc11c\ub294 \uc544\uc774\uc62c\ub85c\uc2a4\uc758 \uc774\uc57c\uae30\ub97c \uc544\uc774\uc62c\ub85c\uc2a4\uac00 \ud2f0\ub808\ub2c8\uc544 \ud574\uc5d0 \uc704\uce58\ud55c \uc5b4\ub5a4 \uc12c\uc758 \uc9c0\ubc30\uc790\uc600\ub2e4\ub294 \uc0ac\uc2e4\uc5d0\uc11c \uc0dd\uaca8\ub09c \uac83\uc73c\ub85c \ucd94\uc815\ud55c\ub2e4. \uc6b0\uc758\uc801 \uc774\ub860 \uc740 \ubaa8\ub4e0 \uace0\ub300 \uc2e0\ud654\uac00 \uc6b0\uc758\uc801\uc774\uace0 \uc0c1\uc9d5\uc801\uc778 \uc758\ubbf8\ub97c \uac00\uc84c\ub2e4\uace0 \ucd94\uc815\ud55c\ub2e4. \uc774\uc640\ub294 \ub2e4\ub974\uac8c \ubb3c\ub9ac\uc801 \uc774\ub860\uc5d0\uc11c\ub294 \uacf5\uae30\uc640 \ubd88, \ubb3c\uc758 \uc6d0\uc18c\uac00 \ubcf8\ub798 \uc885\uad50\uc801 \uc22d\ubc30\uc758 \ub300\uc0c1\uc774\uc5c8\uc73c\uba70, \uc8fc\uc694\ud55c \uc2e0\ub4e4\uc774 \uc774\ub7ec\ud55c \uc790\uc5f0\uc758 \ud798\uc744 \uc2e0\uaca9\ud654\ud55c \uac83\uc774\ub77c\ub294 \uc0dd\uac01\uc744 \ub530\ub974\uace0 \uc788\ub2e4. \ub9c9\uc2a4 \ubb90\ub7ec\ub294 \uc778\ub3c4\uc720\ub7fd \uc885\uad50\uc758 \ud615\ud0dc\ub97c \"\ubcf8\ub798\" \uba85\uc2dc\ud558\ub358 \uc544\ub9ac\uc544\uc778\uc758 \ud754\uc801\uc744 \ucc3e\uc544\uac00\ub294 \ubc29\uc2dd\uc744 \ud1b5\ud574\uc11c \uc774\ud574\ud558\uace0\uc790 \ud588\ub2e4. 1891\ub144, \uadf8\ub294 \ub2e4\uc74c\uacfc \uac19\uc774 \uc8fc\uc7a5\ud588\ub2e4. \"19\uc138\uae30\uc5d0 \uc774\ub974\ub7ec \uc644\uc131\ub41c \uc778\ub958 \uace0\ub300\uc0ac\uc5d0 \uad00\ud55c \ucd5c\uace0\ub85c \uc911\uc694\ud55c \ubc1c\uacac\uc740 ... \ub2e4\uc74c\uc758 \ud45c\ubcf8 \ub4f1\uc2dd\uc774\uc5c8\ub2e4. \uc0b0\uc2a4\ud06c\ub9ac\ud2b8\uc5b4 \ub514\uc544\uc6b0\uc2a4 \ud53c\ud2b8\ub974 = \uadf8\ub9ac\uc2a4\uc5b4 \uc81c\uc6b0\uc2a4 = \ub77c\ud2f4\uc5b4 \uc8fc\ud53c\ud130 = \uace0\ub300 \ub178\ub974\ub4dc\uc5b4 \ud2f0\ub974\" \uc11c\ub85c \ub2e4\ub978 \uc9c0\uc5ed\uc758 \uc2e0\ud654\uc5d0\uc11c \ub4dc\ub7ec\ub098\ub294 \ud2b9\uc131\uacfc \uae30\ub2a5\uc740 \ubc00\uc811\ud55c \ud3c9\ud589\uc131\uc744 \ubcf4\uc774\uace0 \uc788\uc73c\uba70, \uc774\uac83\uc740 \uc804\uc2b9\uc774 \uacf5\uc720\ub418\uc5c8\uc74c\uc744 \uc554\uc2dc\ud55c\ub2e4. \ud558\uc9c0\ub9cc, \uc6b0\ub77c\ub204\uc2a4\uc640 \uc0b0\uc2a4\ud06c\ub9ac\ud2b8 \ubc14\ub8e8\ub098 \ub610\ub294 \uadf8\ub9ac\uc2a4\uc758 \ubaa8\uc774\ub77c\uc640 \ub178\ub974\ub4dc \uc2e0\ud654\uc758 \ub178\ub978\uc758 \uacbd\uc6b0\ucc98\ub7fc \uc5b8\uc5b4\uc758 \uc720\uc0ac\uc131\uc744 \ubcf4\uc774\ub294 \uc99d\uac70\uac00 \ubd80\uc871\ud558\uc5ec \ud655\uc2e4\ud55c \uc785\uc99d\uc740 \uc5b4\ub835\ub2e4.", "sentence_answer": " \uc6b0\uc758\uc801 \uc774\ub860 \uc740 \ubaa8\ub4e0 \uace0\ub300 \uc2e0\ud654\uac00 \uc6b0\uc758\uc801\uc774\uace0 \uc0c1\uc9d5\uc801\uc778 \uc758\ubbf8\ub97c \uac00\uc84c\ub2e4\uace0 \ucd94\uc815\ud55c\ub2e4.", "paragraph_id": "6477962-31-0", "paragraph_question": "question: \ud604\ub300\uc758 \uadf8\ub9ac\uc2a4 \uc2e0\ud654\uc758 \uae30\uc6d0\uc5d0 \ub300\ud55c \uc774\ub860\uc911, \ubaa8\ub4e0 \uace0\ub300 \uc2e0\ud654\uac00 \uc0c1\uc9d5\uc801\uc778 \uc758\ubbf8\ub97c \uac00\uc84c\ub2e4\uace0 \ucd94\uc815\ud558\ub294 \uc774\ub860\uc740?, context: \ud604\ub300\uc5d0\ub294 \uadf8\ub9ac\uc2a4 \uc2e0\ud654\uc758 \uae30\uc6d0\uc5d0 \ub300\ud55c \ub2e4\uc591\ud55c \uc774\ub860\uc774 \uc788\ub2e4. \uc131\uc11c\uc801 \uc774\ub860\uc5d0 \ub530\ub974\uba74, \uc2e0\ud654\uc801 \uc804\uc124\uc740 \uc2e4\uc81c \uc0ac\uc2e4\uc5d0\uc11c \uac00\uc7a5\ub418\uace0 \ubc14\ub010 \ubd80\ubd84\uc740 \uc788\uc73c\ub098 \ubaa8\ub450 \uc131\uc11c \uc18d \uc774\uc57c\uae30\uc5d0\uc11c \ube44\ub86f\ub41c \uac83\uc774\ub2e4. \uc5ed\uc0ac\uc801 \uc774\ub860\uc5d0\uc11c\ub294 \uc2e0\ud654\uc5d0\uc11c \uc5b8\uae09\ub418\ub294 \ubaa8\ub4e0 \uc778\ubb3c\uc740 \uc2e4\uc874 \uc778\ubb3c\uc774\uba70, \uadf8\ub4e4\uacfc \uad00\ub828\ub41c \uc804\uc124\uc740 \ub2e8\uc9c0 \ud6c4\ub300\uc5d0 \ub367\ubd99\uc5ec\uc9c4 \uac83\uc774\ub77c\uace0 \ubcf8\ub2e4. \uc774 \uc774\ub860\uc5d0\uc11c\ub294 \uc544\uc774\uc62c\ub85c\uc2a4\uc758 \uc774\uc57c\uae30\ub97c \uc544\uc774\uc62c\ub85c\uc2a4\uac00 \ud2f0\ub808\ub2c8\uc544 \ud574\uc5d0 \uc704\uce58\ud55c \uc5b4\ub5a4 \uc12c\uc758 \uc9c0\ubc30\uc790\uc600\ub2e4\ub294 \uc0ac\uc2e4\uc5d0\uc11c \uc0dd\uaca8\ub09c \uac83\uc73c\ub85c \ucd94\uc815\ud55c\ub2e4. \uc6b0\uc758\uc801 \uc774\ub860\uc740 \ubaa8\ub4e0 \uace0\ub300 \uc2e0\ud654\uac00 \uc6b0\uc758\uc801\uc774\uace0 \uc0c1\uc9d5\uc801\uc778 \uc758\ubbf8\ub97c \uac00\uc84c\ub2e4\uace0 \ucd94\uc815\ud55c\ub2e4. \uc774\uc640\ub294 \ub2e4\ub974\uac8c \ubb3c\ub9ac\uc801 \uc774\ub860\uc5d0\uc11c\ub294 \uacf5\uae30\uc640 \ubd88, \ubb3c\uc758 \uc6d0\uc18c\uac00 \ubcf8\ub798 \uc885\uad50\uc801 \uc22d\ubc30\uc758 \ub300\uc0c1\uc774\uc5c8\uc73c\uba70, \uc8fc\uc694\ud55c \uc2e0\ub4e4\uc774 \uc774\ub7ec\ud55c \uc790\uc5f0\uc758 \ud798\uc744 \uc2e0\uaca9\ud654\ud55c \uac83\uc774\ub77c\ub294 \uc0dd\uac01\uc744 \ub530\ub974\uace0 \uc788\ub2e4. \ub9c9\uc2a4 \ubb90\ub7ec\ub294 \uc778\ub3c4\uc720\ub7fd \uc885\uad50\uc758 \ud615\ud0dc\ub97c \"\ubcf8\ub798\" \uba85\uc2dc\ud558\ub358 \uc544\ub9ac\uc544\uc778\uc758 \ud754\uc801\uc744 \ucc3e\uc544\uac00\ub294 \ubc29\uc2dd\uc744 \ud1b5\ud574\uc11c \uc774\ud574\ud558\uace0\uc790 \ud588\ub2e4. 1891\ub144, \uadf8\ub294 \ub2e4\uc74c\uacfc \uac19\uc774 \uc8fc\uc7a5\ud588\ub2e4. \"19\uc138\uae30\uc5d0 \uc774\ub974\ub7ec \uc644\uc131\ub41c \uc778\ub958 \uace0\ub300\uc0ac\uc5d0 \uad00\ud55c \ucd5c\uace0\ub85c \uc911\uc694\ud55c \ubc1c\uacac\uc740 ... \ub2e4\uc74c\uc758 \ud45c\ubcf8 \ub4f1\uc2dd\uc774\uc5c8\ub2e4. \uc0b0\uc2a4\ud06c\ub9ac\ud2b8\uc5b4 \ub514\uc544\uc6b0\uc2a4 \ud53c\ud2b8\ub974 = \uadf8\ub9ac\uc2a4\uc5b4 \uc81c\uc6b0\uc2a4 = \ub77c\ud2f4\uc5b4 \uc8fc\ud53c\ud130 = \uace0\ub300 \ub178\ub974\ub4dc\uc5b4 \ud2f0\ub974\" \uc11c\ub85c \ub2e4\ub978 \uc9c0\uc5ed\uc758 \uc2e0\ud654\uc5d0\uc11c \ub4dc\ub7ec\ub098\ub294 \ud2b9\uc131\uacfc \uae30\ub2a5\uc740 \ubc00\uc811\ud55c \ud3c9\ud589\uc131\uc744 \ubcf4\uc774\uace0 \uc788\uc73c\uba70, \uc774\uac83\uc740 \uc804\uc2b9\uc774 \uacf5\uc720\ub418\uc5c8\uc74c\uc744 \uc554\uc2dc\ud55c\ub2e4. \ud558\uc9c0\ub9cc, \uc6b0\ub77c\ub204\uc2a4\uc640 \uc0b0\uc2a4\ud06c\ub9ac\ud2b8 \ubc14\ub8e8\ub098 \ub610\ub294 \uadf8\ub9ac\uc2a4\uc758 \ubaa8\uc774\ub77c\uc640 \ub178\ub974\ub4dc \uc2e0\ud654\uc758 \ub178\ub978\uc758 \uacbd\uc6b0\ucc98\ub7fc \uc5b8\uc5b4\uc758 \uc720\uc0ac\uc131\uc744 \ubcf4\uc774\ub294 \uc99d\uac70\uac00 \ubd80\uc871\ud558\uc5ec \ud655\uc2e4\ud55c \uc785\uc99d\uc740 \uc5b4\ub835\ub2e4."} {"question": "\uc6d0\uc815\ud654\uac00 \uad6c\uc18d \uae30\uc18c\ub41c \ud610\uc758\ub294 \ubb34\uc5c7\uc778\uac00?", "paragraph": "\uc6d0\uc815\ud654\ub294 \uc218\uc6d0\uc9c0\ubc29\ubc95\uc6d0\uc5d0\uc11c \uc5f4\ub9b0 \uc7ac\ud310\uc5d0\uc11c \"\uac04\ucca9\uc73c\ub85c \ud65c\ub3d9\ud558\uc5ec \ud6c4\ud68c\ud55c\ub2e4\"\ub294 \ub0b4\uc6a9\uc758 \uc804\ud5a5\uc11c\ub97c \uc81c\ucd9c\ud55c \uac00\uc6b4\ub370 \uc218\uc6d0\uc9c0\ubc29\ubc95\uc6d0 \uc81c11\ud615\uc0ac\ubd80(\uc7ac\ud310\uc7a5 \uc2e0\uc6a9\uc11d \ubd80\uc7a5\ud310\uc0ac)\ub294 2008\ub144 10\uc6d4 15\uc77c \uad6d\uac00\ubcf4\uc548\ubc95 \uc704\ubc18 \ud610\uc758\ub85c \uad6c\uc18d \uae30\uc18c\ub41c \uc6d0\uc815\ud654\uc5d0\uac8c \"\uad6d\uac00\ubcf4\uc548\ubc95\uc0c1 \uac04\ucca9, \ubaa9\uc801\uc218\ud589, \uc790\uc9c4\uc9c0\uc6d0, \uae08\ud488\uc218\uc218, \uc7a0\uc785\uacfc \ud0c8\ucd9c, \ucc2c\uc591\uacfc \uace0\ubb34, \ud68c\ud569\uacfc \ud1b5\uc2e0 \ub4f1\uc5d0 \uad00\ud55c \uacf5\uc18c\uc0ac\uc2e4\uc774 \ubaa8\ub450 \uc720\uc8c4\ub85c \uc778\uc815\ub41c\ub2e4 \ud53c\uace0\uc778\uc774 \uc131\uc744 \ub9e4\uac1c\ub85c \uad70\uc778\uacfc \uc815\ubcf4\uae30\uad00 \uc694\uc6d0\uc5d0\uac8c \uc811\uadfc\ud574 \uc57d\ucde8, \uc720\uc778\uc744 \uc2dc\ub3c4\ud558\uace0 \uae30\uc874\uc5d0 \uc5c6\ub358 \ud0c8\ubd81\uc790 \uc2e0\ubd84\uc744 \uc774\uc6a9\ud574 \uae30\ubc00 \ud0d0\uc9c0\ud65c\ub3d9\uc744 \uc7a5\uae30\uc801\uc73c\ub85c \uc218\ud589\ud588\uc73c\uba70 \uc911\uad6d\uc5d0\uc11c \ub0a9\uce58\ud55c \ud55c\uad6d\uc778 \uc0ac\uc5c5\uac00\uac00 \uc0ac\ub9dd\ud588\uc744 \uac00\ub2a5\uc131\uc774 \ub192\uc740 \ubc18\uc778\ub95c\uc801 \ubc94\uc8c4\"\ub77c\uace0 \ud558\uba74\uc11c\ub3c4 \"\ucde8\ub4dd\ud55c \uad70\uc0ac\uae30\ubc00\uc774 \uc5b8\ub860\ub9e4\uccb4\uc640 \uc815\ubcf4\ud1b5\uc2e0\uc758 \ubc1c\ub2ec\ub85c \uc77c\ubc18\uc778\uc758 \uc811\uadfc\uc774 \uac00\ub2a5\ud574 \uad6d\uac00\uc548\ubcf4\uc5d0 \uc2ec\uac01\ud55c \uc704\ud574\ub97c \uc8fc\uc9c0\uc54a\uc740 \uc810, \ubd81\ud55c\uc5d0\uc11c \ud0dc\uc5b4\ub098 \ud589\uc704 \uc120\ud0dd\uc758 \ud3ed\uc774 \ub2e4\uc591\ud558\uc9c0 \ubabb\ud588\ub358 \uc810, \uc218\uc0ac\uc5d0 \ud611\uc870\ud558\uace0 \uc804\ud5a5\uc11c\ub97c \uc81c\ucd9c\ud558\uace0 \ubc18\uc131\ud558\uace0 \uc788\ub294 \uc810 \ub4f1\uc744 \ucc38\uc791\ud574 \ud615\ub7c9\uc744 \uc815\ud588\ub2e4\"\uace0 \ud558\uba74\uc11c \uac80\ucc30\uc774 \uad6c\ud615\ud55c\ub300\ub85c \uc9d5\uc5ed 5\ub144\uc744 \uc120\uace0\ud588\ub2e4. \ud53c\uace0\uc778 \uc6d0\uc815\ud654\uc758 \uad6d\uc120\ubcc0\ud638\uc778\uc744 \ub9e1\uc740 \ub300\ud55c\ubc95\ub960\uad6c\uc870\uacf5\ub2e8 \uc218\uc6d0\uc9c0\ubd80\ub294 \"\ucd5c\uadfc \ud56d\uc18c\uc2ec \uacbd\ud5a5\uc774 1\uc2ec \ud310\uacb0\uc744 \uc874\uc911\ud558\ub294 \ucd94\uc138\uc778\ub370\ub2e4 \uc7ac\ud310\uc774 \uacc4\uc18d \uc9c4\ud589\ub420 \uacbd\uc6b0 \uc9d1\uc911\ub420 \uc5b8\ub860\uc758 \uad00\uc2ec\uc5d0 \ub300\ud55c \ubd80\ub2f4\uac10\uc774 \ucee4 \ud56d\uc18c\ud558\uc9c0 \uc54a\uae30\ub85c \ud55c \uac83\uc73c\ub85c \uc54c\ub824\uc84c\ub2e4.\"\uace0 \ubc1d\ud78c \uac00\uc6b4\ub370 \uac80\ucc30\ub3c4 1\uc2ec \uc7ac\ud310\ubd80\uac00 \uad6c\ud615\ub7c9\ub300\ub85c \uc120\uace0\ub97c \ud588\uae30 \ub54c\ubb38\uc5d0 \ud56d\uc18c\ud558\uc9c0 \uc54a\uc544 1\uc2ec \ud310\uacb0\uc774 \ucd5c\uc885 \ud655\uc815\ub418\uc5c8\ub2e4. \uac80\ucc30\uc740 \uc5ec\ub7ec \uc9c4\uc220\uc744 \ud655\ubcf4\ud588\uc73c\ub098 \"\ud0c8\ubd81\uc790 100\uc5ec \uba85\uacfc \ud55c\uad6d\uc778 \uc0ac\uc5c5\uac00 7\uc5ec \uba85\uc744 \ubd81\ud55c\uc73c\ub85c \ub0a9\uce58 \uc1a1\ud658\ud588\ub2e4\"\ub294 \uac83\uc740 \uc785\uc99d\ud558\uc9c0 \ubabb\ud588\ub2e4.", "answer": "\uad6d\uac00\ubcf4\uc548\ubc95 \uc704\ubc18 \ud610\uc758", "sentence": "\uc6d0\uc815\ud654\ub294 \uc218\uc6d0\uc9c0\ubc29\ubc95\uc6d0\uc5d0\uc11c \uc5f4\ub9b0 \uc7ac\ud310\uc5d0\uc11c \"\uac04\ucca9\uc73c\ub85c \ud65c\ub3d9\ud558\uc5ec \ud6c4\ud68c\ud55c\ub2e4\"\ub294 \ub0b4\uc6a9\uc758 \uc804\ud5a5\uc11c\ub97c \uc81c\ucd9c\ud55c \uac00\uc6b4\ub370 \uc218\uc6d0\uc9c0\ubc29\ubc95\uc6d0 \uc81c11\ud615\uc0ac\ubd80(\uc7ac\ud310\uc7a5 \uc2e0\uc6a9\uc11d \ubd80\uc7a5\ud310\uc0ac)\ub294 2008\ub144 10\uc6d4 15\uc77c \uad6d\uac00\ubcf4\uc548\ubc95 \uc704\ubc18 \ud610\uc758 \ub85c \uad6c\uc18d \uae30\uc18c\ub41c \uc6d0\uc815\ud654\uc5d0\uac8c \"\uad6d\uac00\ubcf4\uc548\ubc95\uc0c1 \uac04\ucca9, \ubaa9\uc801\uc218\ud589, \uc790\uc9c4\uc9c0\uc6d0, \uae08\ud488\uc218\uc218, \uc7a0\uc785\uacfc \ud0c8\ucd9c, \ucc2c\uc591\uacfc \uace0\ubb34, \ud68c\ud569\uacfc \ud1b5\uc2e0 \ub4f1\uc5d0 \uad00\ud55c \uacf5\uc18c\uc0ac\uc2e4\uc774 \ubaa8\ub450 \uc720\uc8c4\ub85c \uc778\uc815\ub41c\ub2e4 \ud53c\uace0\uc778\uc774 \uc131\uc744 \ub9e4\uac1c\ub85c \uad70\uc778\uacfc \uc815\ubcf4\uae30\uad00 \uc694\uc6d0\uc5d0\uac8c \uc811\uadfc\ud574 \uc57d\ucde8, \uc720\uc778\uc744 \uc2dc\ub3c4\ud558\uace0 \uae30\uc874\uc5d0 \uc5c6\ub358 \ud0c8\ubd81\uc790 \uc2e0\ubd84\uc744 \uc774\uc6a9\ud574 \uae30\ubc00 \ud0d0\uc9c0\ud65c\ub3d9\uc744 \uc7a5\uae30\uc801\uc73c\ub85c \uc218\ud589\ud588\uc73c\uba70 \uc911\uad6d\uc5d0\uc11c \ub0a9\uce58\ud55c \ud55c\uad6d\uc778 \uc0ac\uc5c5\uac00\uac00 \uc0ac\ub9dd\ud588\uc744 \uac00\ub2a5\uc131\uc774 \ub192\uc740 \ubc18\uc778\ub95c\uc801 \ubc94\uc8c4\"\ub77c\uace0 \ud558\uba74\uc11c\ub3c4 \"\ucde8\ub4dd\ud55c \uad70\uc0ac\uae30\ubc00\uc774 \uc5b8\ub860\ub9e4\uccb4\uc640 \uc815\ubcf4\ud1b5\uc2e0\uc758 \ubc1c\ub2ec\ub85c \uc77c\ubc18\uc778\uc758 \uc811\uadfc\uc774 \uac00\ub2a5\ud574 \uad6d\uac00\uc548\ubcf4\uc5d0 \uc2ec\uac01\ud55c \uc704\ud574\ub97c \uc8fc\uc9c0\uc54a\uc740 \uc810, \ubd81\ud55c\uc5d0\uc11c \ud0dc\uc5b4\ub098 \ud589\uc704 \uc120\ud0dd\uc758 \ud3ed\uc774 \ub2e4\uc591\ud558\uc9c0 \ubabb\ud588\ub358 \uc810, \uc218\uc0ac\uc5d0 \ud611\uc870\ud558\uace0 \uc804\ud5a5\uc11c\ub97c \uc81c\ucd9c\ud558\uace0 \ubc18\uc131\ud558\uace0 \uc788\ub294 \uc810 \ub4f1\uc744 \ucc38\uc791\ud574 \ud615\ub7c9\uc744 \uc815\ud588\ub2e4\"\uace0 \ud558\uba74\uc11c \uac80\ucc30\uc774 \uad6c\ud615\ud55c\ub300\ub85c \uc9d5\uc5ed 5\ub144\uc744 \uc120\uace0\ud588\ub2e4.", "paragraph_sentence": " \uc6d0\uc815\ud654\ub294 \uc218\uc6d0\uc9c0\ubc29\ubc95\uc6d0\uc5d0\uc11c \uc5f4\ub9b0 \uc7ac\ud310\uc5d0\uc11c \"\uac04\ucca9\uc73c\ub85c \ud65c\ub3d9\ud558\uc5ec \ud6c4\ud68c\ud55c\ub2e4\"\ub294 \ub0b4\uc6a9\uc758 \uc804\ud5a5\uc11c\ub97c \uc81c\ucd9c\ud55c \uac00\uc6b4\ub370 \uc218\uc6d0\uc9c0\ubc29\ubc95\uc6d0 \uc81c11\ud615\uc0ac\ubd80(\uc7ac\ud310\uc7a5 \uc2e0\uc6a9\uc11d \ubd80\uc7a5\ud310\uc0ac)\ub294 2008\ub144 10\uc6d4 15\uc77c \uad6d\uac00\ubcf4\uc548\ubc95 \uc704\ubc18 \ud610\uc758 \ub85c \uad6c\uc18d \uae30\uc18c\ub41c \uc6d0\uc815\ud654\uc5d0\uac8c \"\uad6d\uac00\ubcf4\uc548\ubc95\uc0c1 \uac04\ucca9, \ubaa9\uc801\uc218\ud589, \uc790\uc9c4\uc9c0\uc6d0, \uae08\ud488\uc218\uc218, \uc7a0\uc785\uacfc \ud0c8\ucd9c, \ucc2c\uc591\uacfc \uace0\ubb34, \ud68c\ud569\uacfc \ud1b5\uc2e0 \ub4f1\uc5d0 \uad00\ud55c \uacf5\uc18c\uc0ac\uc2e4\uc774 \ubaa8\ub450 \uc720\uc8c4\ub85c \uc778\uc815\ub41c\ub2e4 \ud53c\uace0\uc778\uc774 \uc131\uc744 \ub9e4\uac1c\ub85c \uad70\uc778\uacfc \uc815\ubcf4\uae30\uad00 \uc694\uc6d0\uc5d0\uac8c \uc811\uadfc\ud574 \uc57d\ucde8, \uc720\uc778\uc744 \uc2dc\ub3c4\ud558\uace0 \uae30\uc874\uc5d0 \uc5c6\ub358 \ud0c8\ubd81\uc790 \uc2e0\ubd84\uc744 \uc774\uc6a9\ud574 \uae30\ubc00 \ud0d0\uc9c0\ud65c\ub3d9\uc744 \uc7a5\uae30\uc801\uc73c\ub85c \uc218\ud589\ud588\uc73c\uba70 \uc911\uad6d\uc5d0\uc11c \ub0a9\uce58\ud55c \ud55c\uad6d\uc778 \uc0ac\uc5c5\uac00\uac00 \uc0ac\ub9dd\ud588\uc744 \uac00\ub2a5\uc131\uc774 \ub192\uc740 \ubc18\uc778\ub95c\uc801 \ubc94\uc8c4\"\ub77c\uace0 \ud558\uba74\uc11c\ub3c4 \"\ucde8\ub4dd\ud55c \uad70\uc0ac\uae30\ubc00\uc774 \uc5b8\ub860\ub9e4\uccb4\uc640 \uc815\ubcf4\ud1b5\uc2e0\uc758 \ubc1c\ub2ec\ub85c \uc77c\ubc18\uc778\uc758 \uc811\uadfc\uc774 \uac00\ub2a5\ud574 \uad6d\uac00\uc548\ubcf4\uc5d0 \uc2ec\uac01\ud55c \uc704\ud574\ub97c \uc8fc\uc9c0\uc54a\uc740 \uc810, \ubd81\ud55c\uc5d0\uc11c \ud0dc\uc5b4\ub098 \ud589\uc704 \uc120\ud0dd\uc758 \ud3ed\uc774 \ub2e4\uc591\ud558\uc9c0 \ubabb\ud588\ub358 \uc810, \uc218\uc0ac\uc5d0 \ud611\uc870\ud558\uace0 \uc804\ud5a5\uc11c\ub97c \uc81c\ucd9c\ud558\uace0 \ubc18\uc131\ud558\uace0 \uc788\ub294 \uc810 \ub4f1\uc744 \ucc38\uc791\ud574 \ud615\ub7c9\uc744 \uc815\ud588\ub2e4\"\uace0 \ud558\uba74\uc11c \uac80\ucc30\uc774 \uad6c\ud615\ud55c\ub300\ub85c \uc9d5\uc5ed 5\ub144\uc744 \uc120\uace0\ud588\ub2e4. \ud53c\uace0\uc778 \uc6d0\uc815\ud654\uc758 \uad6d\uc120\ubcc0\ud638\uc778\uc744 \ub9e1\uc740 \ub300\ud55c\ubc95\ub960\uad6c\uc870\uacf5\ub2e8 \uc218\uc6d0\uc9c0\ubd80\ub294 \"\ucd5c\uadfc \ud56d\uc18c\uc2ec \uacbd\ud5a5\uc774 1\uc2ec \ud310\uacb0\uc744 \uc874\uc911\ud558\ub294 \ucd94\uc138\uc778\ub370\ub2e4 \uc7ac\ud310\uc774 \uacc4\uc18d \uc9c4\ud589\ub420 \uacbd\uc6b0 \uc9d1\uc911\ub420 \uc5b8\ub860\uc758 \uad00\uc2ec\uc5d0 \ub300\ud55c \ubd80\ub2f4\uac10\uc774 \ucee4 \ud56d\uc18c\ud558\uc9c0 \uc54a\uae30\ub85c \ud55c \uac83\uc73c\ub85c \uc54c\ub824\uc84c\ub2e4. \"\uace0 \ubc1d\ud78c \uac00\uc6b4\ub370 \uac80\ucc30\ub3c4 1\uc2ec \uc7ac\ud310\ubd80\uac00 \uad6c\ud615\ub7c9\ub300\ub85c \uc120\uace0\ub97c \ud588\uae30 \ub54c\ubb38\uc5d0 \ud56d\uc18c\ud558\uc9c0 \uc54a\uc544 1\uc2ec \ud310\uacb0\uc774 \ucd5c\uc885 \ud655\uc815\ub418\uc5c8\ub2e4. \uac80\ucc30\uc740 \uc5ec\ub7ec \uc9c4\uc220\uc744 \ud655\ubcf4\ud588\uc73c\ub098 \"\ud0c8\ubd81\uc790 100\uc5ec \uba85\uacfc \ud55c\uad6d\uc778 \uc0ac\uc5c5\uac00 7\uc5ec \uba85\uc744 \ubd81\ud55c\uc73c\ub85c \ub0a9\uce58 \uc1a1\ud658\ud588\ub2e4\"\ub294 \uac83\uc740 \uc785\uc99d\ud558\uc9c0 \ubabb\ud588\ub2e4.", "paragraph_answer": "\uc6d0\uc815\ud654\ub294 \uc218\uc6d0\uc9c0\ubc29\ubc95\uc6d0\uc5d0\uc11c \uc5f4\ub9b0 \uc7ac\ud310\uc5d0\uc11c \"\uac04\ucca9\uc73c\ub85c \ud65c\ub3d9\ud558\uc5ec \ud6c4\ud68c\ud55c\ub2e4\"\ub294 \ub0b4\uc6a9\uc758 \uc804\ud5a5\uc11c\ub97c \uc81c\ucd9c\ud55c \uac00\uc6b4\ub370 \uc218\uc6d0\uc9c0\ubc29\ubc95\uc6d0 \uc81c11\ud615\uc0ac\ubd80(\uc7ac\ud310\uc7a5 \uc2e0\uc6a9\uc11d \ubd80\uc7a5\ud310\uc0ac)\ub294 2008\ub144 10\uc6d4 15\uc77c \uad6d\uac00\ubcf4\uc548\ubc95 \uc704\ubc18 \ud610\uc758 \ub85c \uad6c\uc18d \uae30\uc18c\ub41c \uc6d0\uc815\ud654\uc5d0\uac8c \"\uad6d\uac00\ubcf4\uc548\ubc95\uc0c1 \uac04\ucca9, \ubaa9\uc801\uc218\ud589, \uc790\uc9c4\uc9c0\uc6d0, \uae08\ud488\uc218\uc218, \uc7a0\uc785\uacfc \ud0c8\ucd9c, \ucc2c\uc591\uacfc \uace0\ubb34, \ud68c\ud569\uacfc \ud1b5\uc2e0 \ub4f1\uc5d0 \uad00\ud55c \uacf5\uc18c\uc0ac\uc2e4\uc774 \ubaa8\ub450 \uc720\uc8c4\ub85c \uc778\uc815\ub41c\ub2e4 \ud53c\uace0\uc778\uc774 \uc131\uc744 \ub9e4\uac1c\ub85c \uad70\uc778\uacfc \uc815\ubcf4\uae30\uad00 \uc694\uc6d0\uc5d0\uac8c \uc811\uadfc\ud574 \uc57d\ucde8, \uc720\uc778\uc744 \uc2dc\ub3c4\ud558\uace0 \uae30\uc874\uc5d0 \uc5c6\ub358 \ud0c8\ubd81\uc790 \uc2e0\ubd84\uc744 \uc774\uc6a9\ud574 \uae30\ubc00 \ud0d0\uc9c0\ud65c\ub3d9\uc744 \uc7a5\uae30\uc801\uc73c\ub85c \uc218\ud589\ud588\uc73c\uba70 \uc911\uad6d\uc5d0\uc11c \ub0a9\uce58\ud55c \ud55c\uad6d\uc778 \uc0ac\uc5c5\uac00\uac00 \uc0ac\ub9dd\ud588\uc744 \uac00\ub2a5\uc131\uc774 \ub192\uc740 \ubc18\uc778\ub95c\uc801 \ubc94\uc8c4\"\ub77c\uace0 \ud558\uba74\uc11c\ub3c4 \"\ucde8\ub4dd\ud55c \uad70\uc0ac\uae30\ubc00\uc774 \uc5b8\ub860\ub9e4\uccb4\uc640 \uc815\ubcf4\ud1b5\uc2e0\uc758 \ubc1c\ub2ec\ub85c \uc77c\ubc18\uc778\uc758 \uc811\uadfc\uc774 \uac00\ub2a5\ud574 \uad6d\uac00\uc548\ubcf4\uc5d0 \uc2ec\uac01\ud55c \uc704\ud574\ub97c \uc8fc\uc9c0\uc54a\uc740 \uc810, \ubd81\ud55c\uc5d0\uc11c \ud0dc\uc5b4\ub098 \ud589\uc704 \uc120\ud0dd\uc758 \ud3ed\uc774 \ub2e4\uc591\ud558\uc9c0 \ubabb\ud588\ub358 \uc810, \uc218\uc0ac\uc5d0 \ud611\uc870\ud558\uace0 \uc804\ud5a5\uc11c\ub97c \uc81c\ucd9c\ud558\uace0 \ubc18\uc131\ud558\uace0 \uc788\ub294 \uc810 \ub4f1\uc744 \ucc38\uc791\ud574 \ud615\ub7c9\uc744 \uc815\ud588\ub2e4\"\uace0 \ud558\uba74\uc11c \uac80\ucc30\uc774 \uad6c\ud615\ud55c\ub300\ub85c \uc9d5\uc5ed 5\ub144\uc744 \uc120\uace0\ud588\ub2e4. \ud53c\uace0\uc778 \uc6d0\uc815\ud654\uc758 \uad6d\uc120\ubcc0\ud638\uc778\uc744 \ub9e1\uc740 \ub300\ud55c\ubc95\ub960\uad6c\uc870\uacf5\ub2e8 \uc218\uc6d0\uc9c0\ubd80\ub294 \"\ucd5c\uadfc \ud56d\uc18c\uc2ec \uacbd\ud5a5\uc774 1\uc2ec \ud310\uacb0\uc744 \uc874\uc911\ud558\ub294 \ucd94\uc138\uc778\ub370\ub2e4 \uc7ac\ud310\uc774 \uacc4\uc18d \uc9c4\ud589\ub420 \uacbd\uc6b0 \uc9d1\uc911\ub420 \uc5b8\ub860\uc758 \uad00\uc2ec\uc5d0 \ub300\ud55c \ubd80\ub2f4\uac10\uc774 \ucee4 \ud56d\uc18c\ud558\uc9c0 \uc54a\uae30\ub85c \ud55c \uac83\uc73c\ub85c \uc54c\ub824\uc84c\ub2e4.\"\uace0 \ubc1d\ud78c \uac00\uc6b4\ub370 \uac80\ucc30\ub3c4 1\uc2ec \uc7ac\ud310\ubd80\uac00 \uad6c\ud615\ub7c9\ub300\ub85c \uc120\uace0\ub97c \ud588\uae30 \ub54c\ubb38\uc5d0 \ud56d\uc18c\ud558\uc9c0 \uc54a\uc544 1\uc2ec \ud310\uacb0\uc774 \ucd5c\uc885 \ud655\uc815\ub418\uc5c8\ub2e4. \uac80\ucc30\uc740 \uc5ec\ub7ec \uc9c4\uc220\uc744 \ud655\ubcf4\ud588\uc73c\ub098 \"\ud0c8\ubd81\uc790 100\uc5ec \uba85\uacfc \ud55c\uad6d\uc778 \uc0ac\uc5c5\uac00 7\uc5ec \uba85\uc744 \ubd81\ud55c\uc73c\ub85c \ub0a9\uce58 \uc1a1\ud658\ud588\ub2e4\"\ub294 \uac83\uc740 \uc785\uc99d\ud558\uc9c0 \ubabb\ud588\ub2e4.", "sentence_answer": "\uc6d0\uc815\ud654\ub294 \uc218\uc6d0\uc9c0\ubc29\ubc95\uc6d0\uc5d0\uc11c \uc5f4\ub9b0 \uc7ac\ud310\uc5d0\uc11c \"\uac04\ucca9\uc73c\ub85c \ud65c\ub3d9\ud558\uc5ec \ud6c4\ud68c\ud55c\ub2e4\"\ub294 \ub0b4\uc6a9\uc758 \uc804\ud5a5\uc11c\ub97c \uc81c\ucd9c\ud55c \uac00\uc6b4\ub370 \uc218\uc6d0\uc9c0\ubc29\ubc95\uc6d0 \uc81c11\ud615\uc0ac\ubd80(\uc7ac\ud310\uc7a5 \uc2e0\uc6a9\uc11d \ubd80\uc7a5\ud310\uc0ac)\ub294 2008\ub144 10\uc6d4 15\uc77c \uad6d\uac00\ubcf4\uc548\ubc95 \uc704\ubc18 \ud610\uc758 \ub85c \uad6c\uc18d \uae30\uc18c\ub41c \uc6d0\uc815\ud654\uc5d0\uac8c \"\uad6d\uac00\ubcf4\uc548\ubc95\uc0c1 \uac04\ucca9, \ubaa9\uc801\uc218\ud589, \uc790\uc9c4\uc9c0\uc6d0, \uae08\ud488\uc218\uc218, \uc7a0\uc785\uacfc \ud0c8\ucd9c, \ucc2c\uc591\uacfc \uace0\ubb34, \ud68c\ud569\uacfc \ud1b5\uc2e0 \ub4f1\uc5d0 \uad00\ud55c \uacf5\uc18c\uc0ac\uc2e4\uc774 \ubaa8\ub450 \uc720\uc8c4\ub85c \uc778\uc815\ub41c\ub2e4 \ud53c\uace0\uc778\uc774 \uc131\uc744 \ub9e4\uac1c\ub85c \uad70\uc778\uacfc \uc815\ubcf4\uae30\uad00 \uc694\uc6d0\uc5d0\uac8c \uc811\uadfc\ud574 \uc57d\ucde8, \uc720\uc778\uc744 \uc2dc\ub3c4\ud558\uace0 \uae30\uc874\uc5d0 \uc5c6\ub358 \ud0c8\ubd81\uc790 \uc2e0\ubd84\uc744 \uc774\uc6a9\ud574 \uae30\ubc00 \ud0d0\uc9c0\ud65c\ub3d9\uc744 \uc7a5\uae30\uc801\uc73c\ub85c \uc218\ud589\ud588\uc73c\uba70 \uc911\uad6d\uc5d0\uc11c \ub0a9\uce58\ud55c \ud55c\uad6d\uc778 \uc0ac\uc5c5\uac00\uac00 \uc0ac\ub9dd\ud588\uc744 \uac00\ub2a5\uc131\uc774 \ub192\uc740 \ubc18\uc778\ub95c\uc801 \ubc94\uc8c4\"\ub77c\uace0 \ud558\uba74\uc11c\ub3c4 \"\ucde8\ub4dd\ud55c \uad70\uc0ac\uae30\ubc00\uc774 \uc5b8\ub860\ub9e4\uccb4\uc640 \uc815\ubcf4\ud1b5\uc2e0\uc758 \ubc1c\ub2ec\ub85c \uc77c\ubc18\uc778\uc758 \uc811\uadfc\uc774 \uac00\ub2a5\ud574 \uad6d\uac00\uc548\ubcf4\uc5d0 \uc2ec\uac01\ud55c \uc704\ud574\ub97c \uc8fc\uc9c0\uc54a\uc740 \uc810, \ubd81\ud55c\uc5d0\uc11c \ud0dc\uc5b4\ub098 \ud589\uc704 \uc120\ud0dd\uc758 \ud3ed\uc774 \ub2e4\uc591\ud558\uc9c0 \ubabb\ud588\ub358 \uc810, \uc218\uc0ac\uc5d0 \ud611\uc870\ud558\uace0 \uc804\ud5a5\uc11c\ub97c \uc81c\ucd9c\ud558\uace0 \ubc18\uc131\ud558\uace0 \uc788\ub294 \uc810 \ub4f1\uc744 \ucc38\uc791\ud574 \ud615\ub7c9\uc744 \uc815\ud588\ub2e4\"\uace0 \ud558\uba74\uc11c \uac80\ucc30\uc774 \uad6c\ud615\ud55c\ub300\ub85c \uc9d5\uc5ed 5\ub144\uc744 \uc120\uace0\ud588\ub2e4.", "paragraph_id": "6605503-1-2", "paragraph_question": "question: \uc6d0\uc815\ud654\uac00 \uad6c\uc18d \uae30\uc18c\ub41c \ud610\uc758\ub294 \ubb34\uc5c7\uc778\uac00?, context: \uc6d0\uc815\ud654\ub294 \uc218\uc6d0\uc9c0\ubc29\ubc95\uc6d0\uc5d0\uc11c \uc5f4\ub9b0 \uc7ac\ud310\uc5d0\uc11c \"\uac04\ucca9\uc73c\ub85c \ud65c\ub3d9\ud558\uc5ec \ud6c4\ud68c\ud55c\ub2e4\"\ub294 \ub0b4\uc6a9\uc758 \uc804\ud5a5\uc11c\ub97c \uc81c\ucd9c\ud55c \uac00\uc6b4\ub370 \uc218\uc6d0\uc9c0\ubc29\ubc95\uc6d0 \uc81c11\ud615\uc0ac\ubd80(\uc7ac\ud310\uc7a5 \uc2e0\uc6a9\uc11d \ubd80\uc7a5\ud310\uc0ac)\ub294 2008\ub144 10\uc6d4 15\uc77c \uad6d\uac00\ubcf4\uc548\ubc95 \uc704\ubc18 \ud610\uc758\ub85c \uad6c\uc18d \uae30\uc18c\ub41c \uc6d0\uc815\ud654\uc5d0\uac8c \"\uad6d\uac00\ubcf4\uc548\ubc95\uc0c1 \uac04\ucca9, \ubaa9\uc801\uc218\ud589, \uc790\uc9c4\uc9c0\uc6d0, \uae08\ud488\uc218\uc218, \uc7a0\uc785\uacfc \ud0c8\ucd9c, \ucc2c\uc591\uacfc \uace0\ubb34, \ud68c\ud569\uacfc \ud1b5\uc2e0 \ub4f1\uc5d0 \uad00\ud55c \uacf5\uc18c\uc0ac\uc2e4\uc774 \ubaa8\ub450 \uc720\uc8c4\ub85c \uc778\uc815\ub41c\ub2e4 \ud53c\uace0\uc778\uc774 \uc131\uc744 \ub9e4\uac1c\ub85c \uad70\uc778\uacfc \uc815\ubcf4\uae30\uad00 \uc694\uc6d0\uc5d0\uac8c \uc811\uadfc\ud574 \uc57d\ucde8, \uc720\uc778\uc744 \uc2dc\ub3c4\ud558\uace0 \uae30\uc874\uc5d0 \uc5c6\ub358 \ud0c8\ubd81\uc790 \uc2e0\ubd84\uc744 \uc774\uc6a9\ud574 \uae30\ubc00 \ud0d0\uc9c0\ud65c\ub3d9\uc744 \uc7a5\uae30\uc801\uc73c\ub85c \uc218\ud589\ud588\uc73c\uba70 \uc911\uad6d\uc5d0\uc11c \ub0a9\uce58\ud55c \ud55c\uad6d\uc778 \uc0ac\uc5c5\uac00\uac00 \uc0ac\ub9dd\ud588\uc744 \uac00\ub2a5\uc131\uc774 \ub192\uc740 \ubc18\uc778\ub95c\uc801 \ubc94\uc8c4\"\ub77c\uace0 \ud558\uba74\uc11c\ub3c4 \"\ucde8\ub4dd\ud55c \uad70\uc0ac\uae30\ubc00\uc774 \uc5b8\ub860\ub9e4\uccb4\uc640 \uc815\ubcf4\ud1b5\uc2e0\uc758 \ubc1c\ub2ec\ub85c \uc77c\ubc18\uc778\uc758 \uc811\uadfc\uc774 \uac00\ub2a5\ud574 \uad6d\uac00\uc548\ubcf4\uc5d0 \uc2ec\uac01\ud55c \uc704\ud574\ub97c \uc8fc\uc9c0\uc54a\uc740 \uc810, \ubd81\ud55c\uc5d0\uc11c \ud0dc\uc5b4\ub098 \ud589\uc704 \uc120\ud0dd\uc758 \ud3ed\uc774 \ub2e4\uc591\ud558\uc9c0 \ubabb\ud588\ub358 \uc810, \uc218\uc0ac\uc5d0 \ud611\uc870\ud558\uace0 \uc804\ud5a5\uc11c\ub97c \uc81c\ucd9c\ud558\uace0 \ubc18\uc131\ud558\uace0 \uc788\ub294 \uc810 \ub4f1\uc744 \ucc38\uc791\ud574 \ud615\ub7c9\uc744 \uc815\ud588\ub2e4\"\uace0 \ud558\uba74\uc11c \uac80\ucc30\uc774 \uad6c\ud615\ud55c\ub300\ub85c \uc9d5\uc5ed 5\ub144\uc744 \uc120\uace0\ud588\ub2e4. \ud53c\uace0\uc778 \uc6d0\uc815\ud654\uc758 \uad6d\uc120\ubcc0\ud638\uc778\uc744 \ub9e1\uc740 \ub300\ud55c\ubc95\ub960\uad6c\uc870\uacf5\ub2e8 \uc218\uc6d0\uc9c0\ubd80\ub294 \"\ucd5c\uadfc \ud56d\uc18c\uc2ec \uacbd\ud5a5\uc774 1\uc2ec \ud310\uacb0\uc744 \uc874\uc911\ud558\ub294 \ucd94\uc138\uc778\ub370\ub2e4 \uc7ac\ud310\uc774 \uacc4\uc18d \uc9c4\ud589\ub420 \uacbd\uc6b0 \uc9d1\uc911\ub420 \uc5b8\ub860\uc758 \uad00\uc2ec\uc5d0 \ub300\ud55c \ubd80\ub2f4\uac10\uc774 \ucee4 \ud56d\uc18c\ud558\uc9c0 \uc54a\uae30\ub85c \ud55c \uac83\uc73c\ub85c \uc54c\ub824\uc84c\ub2e4.\"\uace0 \ubc1d\ud78c \uac00\uc6b4\ub370 \uac80\ucc30\ub3c4 1\uc2ec \uc7ac\ud310\ubd80\uac00 \uad6c\ud615\ub7c9\ub300\ub85c \uc120\uace0\ub97c \ud588\uae30 \ub54c\ubb38\uc5d0 \ud56d\uc18c\ud558\uc9c0 \uc54a\uc544 1\uc2ec \ud310\uacb0\uc774 \ucd5c\uc885 \ud655\uc815\ub418\uc5c8\ub2e4. \uac80\ucc30\uc740 \uc5ec\ub7ec \uc9c4\uc220\uc744 \ud655\ubcf4\ud588\uc73c\ub098 \"\ud0c8\ubd81\uc790 100\uc5ec \uba85\uacfc \ud55c\uad6d\uc778 \uc0ac\uc5c5\uac00 7\uc5ec \uba85\uc744 \ubd81\ud55c\uc73c\ub85c \ub0a9\uce58 \uc1a1\ud658\ud588\ub2e4\"\ub294 \uac83\uc740 \uc785\uc99d\ud558\uc9c0 \ubabb\ud588\ub2e4."} {"question": "2011\ub144\uc758 \uc5f0\uad6c\uc5d0 \ub530\ub77c \uc2a4\ud30c\ub2e4\uc774\ub098 \uc9c0\ud558\ucca0\uc744 \uc694\ud06c \ub300\ud559\uad50\uc5d0\uc11c \uc5b4\ub514\uae4c\uc9c0 \uc5f0\uc7a5\ud560 \uac83\uc744 \uc81c\uc548\ud558\uc600\ub098?", "paragraph": "\ud558\uc9c0\ub9cc 2001\ub144, \ud1a0\ub860\ud1a0 \uad50\ud1b5\uad6d\uc740 \uc601 \uc9c0\ud558\ucca0\uc758 \ub3c4\uc2ec \uad6c\uac04\uc5d0 \uc2b9\uac1d\uc774 \uae09\uc99d\ud558\ub294 \uac83\uc744 \uc8fc\ubaa9\ud558\uac8c \ub418\uace0, \ud1a0\ub860\ud1a0 \uad50\ud1b5\uad6d\uc740 \ub300\uc911\uad50\ud1b5 \uc5f0\uc7a5\uc5d0 \uad00\ud574 \uc5b4\ub5a4 \uc9c0\ud558\ucca0 \ub178\uc120\uc774 \uc5f0\uc7a5 \ubc0f \uc2e0\uc124\ub418\uc5b4\uc57c \ud558\ub294\uc9c0, \uc5b4\ub5a4 \ub178\uc120\uc774 \uc8fc \uc815\ubd80\uc640 \uc5f0\ubc29 \uc815\ubd80\uc758 \uc9c0\uc6d0\uc744 \ubc1b\uc544\uc57c \ud558\ub294\uc9c0\uc5d0 \ub300\ud55c \uc5f0\uad6c\ub97c \uc9c4\ud589\ud558\uc600\ub2e4. \uc774 \ubcf4\uace0\uc11c\uc5d0 \ub530\ub974\uba74 \uc170\ud37c\ub4dc \uc9c0\ud558\ucca0\uc744 \ub3c8 \ubc00\uc2a4 \ub85c\ub4dc\uc5d0\uc11c \ube45\ud1a0\ub9ac\uc544 \ud30c\ud06c \uc560\ube44\ub274\uae4c\uc9c0 \uc5f0\uc7a5\ud558\uace0 \uc2a4\ud30c\ub2e4\uc774\ub098 \uc9c0\ud558\ucca0\uc744 \uc694\ud06c \ub300\ud559\uad50\uc5d0\uc11c \uc2a4\ud2f8\uc988 \uc560\ube44\ub274\uae4c\uc9c0 \uc5f0\uc7a5\ud560 \uac83\uc744 \uc81c\uc548\ud558\uc600\ub2e4. \ub610\ud55c \uc601 \uc9c0\ud558\ucca0\uc744 \ud540\uce58 \uc560\ube44\ub274\uc5d0\uc11c \ud074\ub77c\ud06c \ube14\ub8e8\ubc84\ub4dc\uae4c\uc9c0 \uc5f0\uc7a5\ud558\ub294 \uac83\uc740 \ub2e4\ub978 \ub450 \ubc29\uc548\ubcf4\ub2e4 \uc2b9\uac1d\uc744 \ub354 \ub9ce\uc774 \uc774\ub04c \uc218 \uc788\uaca0\uc73c\ub098 \uc601 \uc9c0\ud558\ucca0\uc740 \ub354 \ud63c\uc7a1\ud574\uc9c8 \uac83\uc774\uba70 \uc2a4\ud30c\ub2e4\uc774\ub098 \uc120\uc744 \uc5f0\uc7a5\ud558\uac8c \ub418\uba74 \uc601 \uc9c0\ud558\ucca0\uc758 \uc2b9\uac1d\uc744 \uc5b4\ub290\uc815\ub3c4 \ubd84\uc0b0\uc2dc\ud0ac \uc218 \uc788\uc744 \uac83\uc73c\ub85c \ubcf4\uc778\ub2e4\uace0 \ubc1d\ud614\ub2e4.", "answer": "\uc2a4\ud2f8\uc988 \uc560\ube44\ub274", "sentence": "\uc774 \ubcf4\uace0\uc11c\uc5d0 \ub530\ub974\uba74 \uc170\ud37c\ub4dc \uc9c0\ud558\ucca0\uc744 \ub3c8 \ubc00\uc2a4 \ub85c\ub4dc\uc5d0\uc11c \ube45\ud1a0\ub9ac\uc544 \ud30c\ud06c \uc560\ube44\ub274\uae4c\uc9c0 \uc5f0\uc7a5\ud558\uace0 \uc2a4\ud30c\ub2e4\uc774\ub098 \uc9c0\ud558\ucca0\uc744 \uc694\ud06c \ub300\ud559\uad50\uc5d0\uc11c \uc2a4\ud2f8\uc988 \uc560\ube44\ub274 \uae4c\uc9c0 \uc5f0\uc7a5\ud560 \uac83\uc744 \uc81c\uc548\ud558\uc600\ub2e4.", "paragraph_sentence": "\ud558\uc9c0\ub9cc 2001\ub144, \ud1a0\ub860\ud1a0 \uad50\ud1b5\uad6d\uc740 \uc601 \uc9c0\ud558\ucca0\uc758 \ub3c4\uc2ec \uad6c\uac04\uc5d0 \uc2b9\uac1d\uc774 \uae09\uc99d\ud558\ub294 \uac83\uc744 \uc8fc\ubaa9\ud558\uac8c \ub418\uace0, \ud1a0\ub860\ud1a0 \uad50\ud1b5\uad6d\uc740 \ub300\uc911\uad50\ud1b5 \uc5f0\uc7a5\uc5d0 \uad00\ud574 \uc5b4\ub5a4 \uc9c0\ud558\ucca0 \ub178\uc120\uc774 \uc5f0\uc7a5 \ubc0f \uc2e0\uc124\ub418\uc5b4\uc57c \ud558\ub294\uc9c0, \uc5b4\ub5a4 \ub178\uc120\uc774 \uc8fc \uc815\ubd80\uc640 \uc5f0\ubc29 \uc815\ubd80\uc758 \uc9c0\uc6d0\uc744 \ubc1b\uc544\uc57c \ud558\ub294\uc9c0\uc5d0 \ub300\ud55c \uc5f0\uad6c\ub97c \uc9c4\ud589\ud558\uc600\ub2e4. \uc774 \ubcf4\uace0\uc11c\uc5d0 \ub530\ub974\uba74 \uc170\ud37c\ub4dc \uc9c0\ud558\ucca0\uc744 \ub3c8 \ubc00\uc2a4 \ub85c\ub4dc\uc5d0\uc11c \ube45\ud1a0\ub9ac\uc544 \ud30c\ud06c \uc560\ube44\ub274\uae4c\uc9c0 \uc5f0\uc7a5\ud558\uace0 \uc2a4\ud30c\ub2e4\uc774\ub098 \uc9c0\ud558\ucca0\uc744 \uc694\ud06c \ub300\ud559\uad50\uc5d0\uc11c \uc2a4\ud2f8\uc988 \uc560\ube44\ub274 \uae4c\uc9c0 \uc5f0\uc7a5\ud560 \uac83\uc744 \uc81c\uc548\ud558\uc600\ub2e4. \ub610\ud55c \uc601 \uc9c0\ud558\ucca0\uc744 \ud540\uce58 \uc560\ube44\ub274\uc5d0\uc11c \ud074\ub77c\ud06c \ube14\ub8e8\ubc84\ub4dc\uae4c\uc9c0 \uc5f0\uc7a5\ud558\ub294 \uac83\uc740 \ub2e4\ub978 \ub450 \ubc29\uc548\ubcf4\ub2e4 \uc2b9\uac1d\uc744 \ub354 \ub9ce\uc774 \uc774\ub04c \uc218 \uc788\uaca0\uc73c\ub098 \uc601 \uc9c0\ud558\ucca0\uc740 \ub354 \ud63c\uc7a1\ud574\uc9c8 \uac83\uc774\uba70 \uc2a4\ud30c\ub2e4\uc774\ub098 \uc120\uc744 \uc5f0\uc7a5\ud558\uac8c \ub418\uba74 \uc601 \uc9c0\ud558\ucca0\uc758 \uc2b9\uac1d\uc744 \uc5b4\ub290\uc815\ub3c4 \ubd84\uc0b0\uc2dc\ud0ac \uc218 \uc788\uc744 \uac83\uc73c\ub85c \ubcf4\uc778\ub2e4\uace0 \ubc1d\ud614\ub2e4.", "paragraph_answer": "\ud558\uc9c0\ub9cc 2001\ub144, \ud1a0\ub860\ud1a0 \uad50\ud1b5\uad6d\uc740 \uc601 \uc9c0\ud558\ucca0\uc758 \ub3c4\uc2ec \uad6c\uac04\uc5d0 \uc2b9\uac1d\uc774 \uae09\uc99d\ud558\ub294 \uac83\uc744 \uc8fc\ubaa9\ud558\uac8c \ub418\uace0, \ud1a0\ub860\ud1a0 \uad50\ud1b5\uad6d\uc740 \ub300\uc911\uad50\ud1b5 \uc5f0\uc7a5\uc5d0 \uad00\ud574 \uc5b4\ub5a4 \uc9c0\ud558\ucca0 \ub178\uc120\uc774 \uc5f0\uc7a5 \ubc0f \uc2e0\uc124\ub418\uc5b4\uc57c \ud558\ub294\uc9c0, \uc5b4\ub5a4 \ub178\uc120\uc774 \uc8fc \uc815\ubd80\uc640 \uc5f0\ubc29 \uc815\ubd80\uc758 \uc9c0\uc6d0\uc744 \ubc1b\uc544\uc57c \ud558\ub294\uc9c0\uc5d0 \ub300\ud55c \uc5f0\uad6c\ub97c \uc9c4\ud589\ud558\uc600\ub2e4. \uc774 \ubcf4\uace0\uc11c\uc5d0 \ub530\ub974\uba74 \uc170\ud37c\ub4dc \uc9c0\ud558\ucca0\uc744 \ub3c8 \ubc00\uc2a4 \ub85c\ub4dc\uc5d0\uc11c \ube45\ud1a0\ub9ac\uc544 \ud30c\ud06c \uc560\ube44\ub274\uae4c\uc9c0 \uc5f0\uc7a5\ud558\uace0 \uc2a4\ud30c\ub2e4\uc774\ub098 \uc9c0\ud558\ucca0\uc744 \uc694\ud06c \ub300\ud559\uad50\uc5d0\uc11c \uc2a4\ud2f8\uc988 \uc560\ube44\ub274 \uae4c\uc9c0 \uc5f0\uc7a5\ud560 \uac83\uc744 \uc81c\uc548\ud558\uc600\ub2e4. \ub610\ud55c \uc601 \uc9c0\ud558\ucca0\uc744 \ud540\uce58 \uc560\ube44\ub274\uc5d0\uc11c \ud074\ub77c\ud06c \ube14\ub8e8\ubc84\ub4dc\uae4c\uc9c0 \uc5f0\uc7a5\ud558\ub294 \uac83\uc740 \ub2e4\ub978 \ub450 \ubc29\uc548\ubcf4\ub2e4 \uc2b9\uac1d\uc744 \ub354 \ub9ce\uc774 \uc774\ub04c \uc218 \uc788\uaca0\uc73c\ub098 \uc601 \uc9c0\ud558\ucca0\uc740 \ub354 \ud63c\uc7a1\ud574\uc9c8 \uac83\uc774\uba70 \uc2a4\ud30c\ub2e4\uc774\ub098 \uc120\uc744 \uc5f0\uc7a5\ud558\uac8c \ub418\uba74 \uc601 \uc9c0\ud558\ucca0\uc758 \uc2b9\uac1d\uc744 \uc5b4\ub290\uc815\ub3c4 \ubd84\uc0b0\uc2dc\ud0ac \uc218 \uc788\uc744 \uac83\uc73c\ub85c \ubcf4\uc778\ub2e4\uace0 \ubc1d\ud614\ub2e4.", "sentence_answer": "\uc774 \ubcf4\uace0\uc11c\uc5d0 \ub530\ub974\uba74 \uc170\ud37c\ub4dc \uc9c0\ud558\ucca0\uc744 \ub3c8 \ubc00\uc2a4 \ub85c\ub4dc\uc5d0\uc11c \ube45\ud1a0\ub9ac\uc544 \ud30c\ud06c \uc560\ube44\ub274\uae4c\uc9c0 \uc5f0\uc7a5\ud558\uace0 \uc2a4\ud30c\ub2e4\uc774\ub098 \uc9c0\ud558\ucca0\uc744 \uc694\ud06c \ub300\ud559\uad50\uc5d0\uc11c \uc2a4\ud2f8\uc988 \uc560\ube44\ub274 \uae4c\uc9c0 \uc5f0\uc7a5\ud560 \uac83\uc744 \uc81c\uc548\ud558\uc600\ub2e4.", "paragraph_id": "6599092-18-2", "paragraph_question": "question: 2011\ub144\uc758 \uc5f0\uad6c\uc5d0 \ub530\ub77c \uc2a4\ud30c\ub2e4\uc774\ub098 \uc9c0\ud558\ucca0\uc744 \uc694\ud06c \ub300\ud559\uad50\uc5d0\uc11c \uc5b4\ub514\uae4c\uc9c0 \uc5f0\uc7a5\ud560 \uac83\uc744 \uc81c\uc548\ud558\uc600\ub098?, context: \ud558\uc9c0\ub9cc 2001\ub144, \ud1a0\ub860\ud1a0 \uad50\ud1b5\uad6d\uc740 \uc601 \uc9c0\ud558\ucca0\uc758 \ub3c4\uc2ec \uad6c\uac04\uc5d0 \uc2b9\uac1d\uc774 \uae09\uc99d\ud558\ub294 \uac83\uc744 \uc8fc\ubaa9\ud558\uac8c \ub418\uace0, \ud1a0\ub860\ud1a0 \uad50\ud1b5\uad6d\uc740 \ub300\uc911\uad50\ud1b5 \uc5f0\uc7a5\uc5d0 \uad00\ud574 \uc5b4\ub5a4 \uc9c0\ud558\ucca0 \ub178\uc120\uc774 \uc5f0\uc7a5 \ubc0f \uc2e0\uc124\ub418\uc5b4\uc57c \ud558\ub294\uc9c0, \uc5b4\ub5a4 \ub178\uc120\uc774 \uc8fc \uc815\ubd80\uc640 \uc5f0\ubc29 \uc815\ubd80\uc758 \uc9c0\uc6d0\uc744 \ubc1b\uc544\uc57c \ud558\ub294\uc9c0\uc5d0 \ub300\ud55c \uc5f0\uad6c\ub97c \uc9c4\ud589\ud558\uc600\ub2e4. \uc774 \ubcf4\uace0\uc11c\uc5d0 \ub530\ub974\uba74 \uc170\ud37c\ub4dc \uc9c0\ud558\ucca0\uc744 \ub3c8 \ubc00\uc2a4 \ub85c\ub4dc\uc5d0\uc11c \ube45\ud1a0\ub9ac\uc544 \ud30c\ud06c \uc560\ube44\ub274\uae4c\uc9c0 \uc5f0\uc7a5\ud558\uace0 \uc2a4\ud30c\ub2e4\uc774\ub098 \uc9c0\ud558\ucca0\uc744 \uc694\ud06c \ub300\ud559\uad50\uc5d0\uc11c \uc2a4\ud2f8\uc988 \uc560\ube44\ub274\uae4c\uc9c0 \uc5f0\uc7a5\ud560 \uac83\uc744 \uc81c\uc548\ud558\uc600\ub2e4. \ub610\ud55c \uc601 \uc9c0\ud558\ucca0\uc744 \ud540\uce58 \uc560\ube44\ub274\uc5d0\uc11c \ud074\ub77c\ud06c \ube14\ub8e8\ubc84\ub4dc\uae4c\uc9c0 \uc5f0\uc7a5\ud558\ub294 \uac83\uc740 \ub2e4\ub978 \ub450 \ubc29\uc548\ubcf4\ub2e4 \uc2b9\uac1d\uc744 \ub354 \ub9ce\uc774 \uc774\ub04c \uc218 \uc788\uaca0\uc73c\ub098 \uc601 \uc9c0\ud558\ucca0\uc740 \ub354 \ud63c\uc7a1\ud574\uc9c8 \uac83\uc774\uba70 \uc2a4\ud30c\ub2e4\uc774\ub098 \uc120\uc744 \uc5f0\uc7a5\ud558\uac8c \ub418\uba74 \uc601 \uc9c0\ud558\ucca0\uc758 \uc2b9\uac1d\uc744 \uc5b4\ub290\uc815\ub3c4 \ubd84\uc0b0\uc2dc\ud0ac \uc218 \uc788\uc744 \uac83\uc73c\ub85c \ubcf4\uc778\ub2e4\uace0 \ubc1d\ud614\ub2e4."} {"question": "\ud544\ub9ac\ud540 \uc2dd\ubbfc\uc9c0\ud654\uc758 \ubc1c\ud310\uc744 \ub9c8\ub828\ud55c \uc815\ubcf5\uc790\ub294?", "paragraph": "1565\ub144 4\uc6d4 27\uc77c, \uc815\ubcf5\uc790 \ubbf8\uac94 \ub85c\ud398\uc2a4 \ub370 \ub808\uac00\uc2a4\ud53c\ub294 \ubcd1\uc0ac\ub4e4\uacfc \uc131 \uc544\uc6b0\uad6c\uc2a4\uce58\ub178 \uc218\ub3c4\ud68c \ubc0f \ud504\ub780\uc2dc\uc2a4\ucf54 \ud68c\ub85c\ubd80\ud130\uc758 \uc218\ub3c4\uc0ac\uc640 \ud568\uaed8 \uc0c1\ub959\ud574, \ub77c\uc790 \ud6c4\ub9c8\ubcf8 \uc655\uc758 \uc544\ub4e4, \ub77c\uc790 \ud22c \ud30c\uc2a4\uc655\uc758 \ub9c8\uc744\uc744 \uacf5\uaca9\ud558\uc5ec \uc810\ub839\ud558\uc5ec \ud544\ub9ac\ud540 \uc2dd\ubbfc\uc9c0\ud654\uc758 \ucd5c\ucd08\uc758 \ubc1c\ud310\uc744 \uc313\uc544 \uc62c\ub838\ub2e4. \ub9c8\uc824\ub780\uc774 \uc0b0 \ubbf8\uad6c\uc5d8\ub85c \uac1c\uba85\ud55c \uc138\ubd80\uc758 \ub9c8\uc744\uc740 \ub808\uac00\uc2a4\ud53c\uc5d0 \uc758\ud574\uc11c \ube44\uc57c \ub378 \uc0b0\ud2f0\uc2dc\ubaa8 \ub17c\ube0c\ub808 \ub370 \ud5e4\uc2a4\uc2a4(Villa del Santisimo Nombre de Jess)\ub77c\uace0 \ub2e4\uc2dc \uac1c\uba85\ub418\uc5b4 \ub9c8\ub2d0\ub77c\uac00 \uc2dd\ubbfc\uc9c0\ud654 \ub420 \ub54c\uae4c\uc9c0\uc758 6\ub144\uac04, \uc138\ubd80 \uc2dc\ub294 \uc0c8\ub85c\uc6b4 \uc2a4\ud398\uc778 \uc2dd\ubbfc\uc9c0\uc758 \uc218\ub3c4\uac00 \ub418\uc5c8\ub2e4. \uadf8\ub4e4\uc740 \uc774 \ud574\uc548\uc5d0 \uba55\uc2dc\ucf54\uc640\uc758 \ubb34\uc5ed\uc774\ub098 \ud604\uc9c0\uc778\uacfc\uc758 \uc804\uc7c1\uc5d0 \ub300\ube44\ud55c \ud56d\uad6c \uc694\uc0c8\uc778 \uc0b0 \ud398\ub4dc\ub85c \uc694\uc0c8\ub97c \uac74\uc124\ud588\uace0, \uc774\uac83\uc740 \uc9c0\uae08\ub3c4 \uc138\ubd80 \uc2dc\uc758 \ud574\uc548\uc5d0 \ub0a8\uc544 \uc788\ub2e4.", "answer": "\ubbf8\uac94 \ub85c\ud398\uc2a4 \ub370 \ub808\uac00\uc2a4\ud53c", "sentence": "1565\ub144 4\uc6d4 27\uc77c, \uc815\ubcf5\uc790 \ubbf8\uac94 \ub85c\ud398\uc2a4 \ub370 \ub808\uac00\uc2a4\ud53c \ub294 \ubcd1\uc0ac\ub4e4\uacfc \uc131 \uc544\uc6b0\uad6c\uc2a4\uce58\ub178 \uc218\ub3c4\ud68c \ubc0f \ud504\ub780\uc2dc\uc2a4\ucf54 \ud68c\ub85c\ubd80\ud130\uc758 \uc218\ub3c4\uc0ac\uc640 \ud568\uaed8 \uc0c1\ub959\ud574, \ub77c\uc790 \ud6c4\ub9c8\ubcf8 \uc655\uc758 \uc544\ub4e4, \ub77c\uc790 \ud22c \ud30c\uc2a4\uc655\uc758 \ub9c8\uc744\uc744 \uacf5\uaca9\ud558\uc5ec \uc810\ub839\ud558\uc5ec \ud544\ub9ac\ud540 \uc2dd\ubbfc\uc9c0\ud654\uc758 \ucd5c\ucd08\uc758 \ubc1c\ud310\uc744 \uc313\uc544 \uc62c\ub838\ub2e4.", "paragraph_sentence": " 1565\ub144 4\uc6d4 27\uc77c, \uc815\ubcf5\uc790 \ubbf8\uac94 \ub85c\ud398\uc2a4 \ub370 \ub808\uac00\uc2a4\ud53c \ub294 \ubcd1\uc0ac\ub4e4\uacfc \uc131 \uc544\uc6b0\uad6c\uc2a4\uce58\ub178 \uc218\ub3c4\ud68c \ubc0f \ud504\ub780\uc2dc\uc2a4\ucf54 \ud68c\ub85c\ubd80\ud130\uc758 \uc218\ub3c4\uc0ac\uc640 \ud568\uaed8 \uc0c1\ub959\ud574, \ub77c\uc790 \ud6c4\ub9c8\ubcf8 \uc655\uc758 \uc544\ub4e4, \ub77c\uc790 \ud22c \ud30c\uc2a4\uc655\uc758 \ub9c8\uc744\uc744 \uacf5\uaca9\ud558\uc5ec \uc810\ub839\ud558\uc5ec \ud544\ub9ac\ud540 \uc2dd\ubbfc\uc9c0\ud654\uc758 \ucd5c\ucd08\uc758 \ubc1c\ud310\uc744 \uc313\uc544 \uc62c\ub838\ub2e4. \ub9c8\uc824\ub780\uc774 \uc0b0 \ubbf8\uad6c\uc5d8\ub85c \uac1c\uba85\ud55c \uc138\ubd80\uc758 \ub9c8\uc744\uc740 \ub808\uac00\uc2a4\ud53c\uc5d0 \uc758\ud574\uc11c \ube44\uc57c \ub378 \uc0b0\ud2f0\uc2dc\ubaa8 \ub17c\ube0c\ub808 \ub370 \ud5e4\uc2a4\uc2a4(Villa del Santisimo Nombre de Jess)\ub77c\uace0 \ub2e4\uc2dc \uac1c\uba85\ub418\uc5b4 \ub9c8\ub2d0\ub77c\uac00 \uc2dd\ubbfc\uc9c0\ud654 \ub420 \ub54c\uae4c\uc9c0\uc758 6\ub144\uac04, \uc138\ubd80 \uc2dc\ub294 \uc0c8\ub85c\uc6b4 \uc2a4\ud398\uc778 \uc2dd\ubbfc\uc9c0\uc758 \uc218\ub3c4\uac00 \ub418\uc5c8\ub2e4. \uadf8\ub4e4\uc740 \uc774 \ud574\uc548\uc5d0 \uba55\uc2dc\ucf54\uc640\uc758 \ubb34\uc5ed\uc774\ub098 \ud604\uc9c0\uc778\uacfc\uc758 \uc804\uc7c1\uc5d0 \ub300\ube44\ud55c \ud56d\uad6c \uc694\uc0c8\uc778 \uc0b0 \ud398\ub4dc\ub85c \uc694\uc0c8\ub97c \uac74\uc124\ud588\uace0, \uc774\uac83\uc740 \uc9c0\uae08\ub3c4 \uc138\ubd80 \uc2dc\uc758 \ud574\uc548\uc5d0 \ub0a8\uc544 \uc788\ub2e4.", "paragraph_answer": "1565\ub144 4\uc6d4 27\uc77c, \uc815\ubcf5\uc790 \ubbf8\uac94 \ub85c\ud398\uc2a4 \ub370 \ub808\uac00\uc2a4\ud53c \ub294 \ubcd1\uc0ac\ub4e4\uacfc \uc131 \uc544\uc6b0\uad6c\uc2a4\uce58\ub178 \uc218\ub3c4\ud68c \ubc0f \ud504\ub780\uc2dc\uc2a4\ucf54 \ud68c\ub85c\ubd80\ud130\uc758 \uc218\ub3c4\uc0ac\uc640 \ud568\uaed8 \uc0c1\ub959\ud574, \ub77c\uc790 \ud6c4\ub9c8\ubcf8 \uc655\uc758 \uc544\ub4e4, \ub77c\uc790 \ud22c \ud30c\uc2a4\uc655\uc758 \ub9c8\uc744\uc744 \uacf5\uaca9\ud558\uc5ec \uc810\ub839\ud558\uc5ec \ud544\ub9ac\ud540 \uc2dd\ubbfc\uc9c0\ud654\uc758 \ucd5c\ucd08\uc758 \ubc1c\ud310\uc744 \uc313\uc544 \uc62c\ub838\ub2e4. \ub9c8\uc824\ub780\uc774 \uc0b0 \ubbf8\uad6c\uc5d8\ub85c \uac1c\uba85\ud55c \uc138\ubd80\uc758 \ub9c8\uc744\uc740 \ub808\uac00\uc2a4\ud53c\uc5d0 \uc758\ud574\uc11c \ube44\uc57c \ub378 \uc0b0\ud2f0\uc2dc\ubaa8 \ub17c\ube0c\ub808 \ub370 \ud5e4\uc2a4\uc2a4(Villa del Santisimo Nombre de Jess)\ub77c\uace0 \ub2e4\uc2dc \uac1c\uba85\ub418\uc5b4 \ub9c8\ub2d0\ub77c\uac00 \uc2dd\ubbfc\uc9c0\ud654 \ub420 \ub54c\uae4c\uc9c0\uc758 6\ub144\uac04, \uc138\ubd80 \uc2dc\ub294 \uc0c8\ub85c\uc6b4 \uc2a4\ud398\uc778 \uc2dd\ubbfc\uc9c0\uc758 \uc218\ub3c4\uac00 \ub418\uc5c8\ub2e4. \uadf8\ub4e4\uc740 \uc774 \ud574\uc548\uc5d0 \uba55\uc2dc\ucf54\uc640\uc758 \ubb34\uc5ed\uc774\ub098 \ud604\uc9c0\uc778\uacfc\uc758 \uc804\uc7c1\uc5d0 \ub300\ube44\ud55c \ud56d\uad6c \uc694\uc0c8\uc778 \uc0b0 \ud398\ub4dc\ub85c \uc694\uc0c8\ub97c \uac74\uc124\ud588\uace0, \uc774\uac83\uc740 \uc9c0\uae08\ub3c4 \uc138\ubd80 \uc2dc\uc758 \ud574\uc548\uc5d0 \ub0a8\uc544 \uc788\ub2e4.", "sentence_answer": "1565\ub144 4\uc6d4 27\uc77c, \uc815\ubcf5\uc790 \ubbf8\uac94 \ub85c\ud398\uc2a4 \ub370 \ub808\uac00\uc2a4\ud53c \ub294 \ubcd1\uc0ac\ub4e4\uacfc \uc131 \uc544\uc6b0\uad6c\uc2a4\uce58\ub178 \uc218\ub3c4\ud68c \ubc0f \ud504\ub780\uc2dc\uc2a4\ucf54 \ud68c\ub85c\ubd80\ud130\uc758 \uc218\ub3c4\uc0ac\uc640 \ud568\uaed8 \uc0c1\ub959\ud574, \ub77c\uc790 \ud6c4\ub9c8\ubcf8 \uc655\uc758 \uc544\ub4e4, \ub77c\uc790 \ud22c \ud30c\uc2a4\uc655\uc758 \ub9c8\uc744\uc744 \uacf5\uaca9\ud558\uc5ec \uc810\ub839\ud558\uc5ec \ud544\ub9ac\ud540 \uc2dd\ubbfc\uc9c0\ud654\uc758 \ucd5c\ucd08\uc758 \ubc1c\ud310\uc744 \uc313\uc544 \uc62c\ub838\ub2e4.", "paragraph_id": "6532123-1-0", "paragraph_question": "question: \ud544\ub9ac\ud540 \uc2dd\ubbfc\uc9c0\ud654\uc758 \ubc1c\ud310\uc744 \ub9c8\ub828\ud55c \uc815\ubcf5\uc790\ub294?, context: 1565\ub144 4\uc6d4 27\uc77c, \uc815\ubcf5\uc790 \ubbf8\uac94 \ub85c\ud398\uc2a4 \ub370 \ub808\uac00\uc2a4\ud53c\ub294 \ubcd1\uc0ac\ub4e4\uacfc \uc131 \uc544\uc6b0\uad6c\uc2a4\uce58\ub178 \uc218\ub3c4\ud68c \ubc0f \ud504\ub780\uc2dc\uc2a4\ucf54 \ud68c\ub85c\ubd80\ud130\uc758 \uc218\ub3c4\uc0ac\uc640 \ud568\uaed8 \uc0c1\ub959\ud574, \ub77c\uc790 \ud6c4\ub9c8\ubcf8 \uc655\uc758 \uc544\ub4e4, \ub77c\uc790 \ud22c \ud30c\uc2a4\uc655\uc758 \ub9c8\uc744\uc744 \uacf5\uaca9\ud558\uc5ec \uc810\ub839\ud558\uc5ec \ud544\ub9ac\ud540 \uc2dd\ubbfc\uc9c0\ud654\uc758 \ucd5c\ucd08\uc758 \ubc1c\ud310\uc744 \uc313\uc544 \uc62c\ub838\ub2e4. \ub9c8\uc824\ub780\uc774 \uc0b0 \ubbf8\uad6c\uc5d8\ub85c \uac1c\uba85\ud55c \uc138\ubd80\uc758 \ub9c8\uc744\uc740 \ub808\uac00\uc2a4\ud53c\uc5d0 \uc758\ud574\uc11c \ube44\uc57c \ub378 \uc0b0\ud2f0\uc2dc\ubaa8 \ub17c\ube0c\ub808 \ub370 \ud5e4\uc2a4\uc2a4(Villa del Santisimo Nombre de Jess)\ub77c\uace0 \ub2e4\uc2dc \uac1c\uba85\ub418\uc5b4 \ub9c8\ub2d0\ub77c\uac00 \uc2dd\ubbfc\uc9c0\ud654 \ub420 \ub54c\uae4c\uc9c0\uc758 6\ub144\uac04, \uc138\ubd80 \uc2dc\ub294 \uc0c8\ub85c\uc6b4 \uc2a4\ud398\uc778 \uc2dd\ubbfc\uc9c0\uc758 \uc218\ub3c4\uac00 \ub418\uc5c8\ub2e4. \uadf8\ub4e4\uc740 \uc774 \ud574\uc548\uc5d0 \uba55\uc2dc\ucf54\uc640\uc758 \ubb34\uc5ed\uc774\ub098 \ud604\uc9c0\uc778\uacfc\uc758 \uc804\uc7c1\uc5d0 \ub300\ube44\ud55c \ud56d\uad6c \uc694\uc0c8\uc778 \uc0b0 \ud398\ub4dc\ub85c \uc694\uc0c8\ub97c \uac74\uc124\ud588\uace0, \uc774\uac83\uc740 \uc9c0\uae08\ub3c4 \uc138\ubd80 \uc2dc\uc758 \ud574\uc548\uc5d0 \ub0a8\uc544 \uc788\ub2e4."} {"question": "\ud669\uac74\uc801 \uc11c\ud654\uc640 \ud568\uaed8 \uc81c\ub0a8\uacfc \ub099\uc548\uc744 \uacf5\uaca9\ud55c \uc0ac\ub78c\uc740?", "paragraph": "\uc77c\ucc0d\uc774 \uace0\ud5a5\uc5d0 \uc788\uc744 \ub54c \uc870\uc870\uac00 \uc800\uc9c0\ub978 \uc8c4\ub97c \ub300\uc2e0 \uc9ca\uc5b4\uc9c0\ub824 \ud588\ub294\ub370 \uc870\uc870\uac00 \uc6a9\uc11c\ub97c \ube4c\uc5b4 \uba74\ud558\uc600\ub2e4. \uc870\uc870\uac00 \uac70\ubcd1\ud558\uc790 \ubcc4\ubd80\uc0ac\ub9c8(\u5225\u90e8\u53f8\u99ac)\uc640 \uae30\ub3c4\uc704\uac00 \ub418\uc5b4 \ub530\ub974\uace0 \uc9c4\ub958\ud0dc\uc218\u00b7\uc601\ucc9c\ud0dc\uc218\ub97c \uc9c0\ub0c8\ub2e4. \uad00\ub3c4 \uc804\ud22c\uc5d0\uc11c\ub294 \ub3c5\uad70\uad50\uc704(\u7763\u8ecd\u6821\u5c09)\ub85c\uc11c \uc5f0\uc8fc\u00b7\uc608\uc8fc\u00b7\uc11c\uc8fc\ub85c\ubd80\ud130 \uad70\ub7c9\uc744 \uc870\ub2ec\ud558\uc600\ub2e4. 201\ub144(\uac74\uc548 6\ub144) \uc7a5\ub8cc\uc640 \ud568\uaed8 \ub3d9\ud574\uad70\uc758 \ucc3d\ud76c\ub97c \ud3ec\uc704 \uacf5\uaca9\ud588\uc73c\ub098 \uba87 \ub2ec\uac04 \ud568\ub77d\uc2dc\ud0a4\uc9c0 \ubabb\ud558\uace0 \uad70\ub7c9\uc774 \ub2e4 \ub5a8\uc5b4\uc84c\ub2e4. \ud68c\uad70\ud558\uae30 \uc804\uc5d0 \uc7a5\ub8cc\uac00 \ucc3d\ud76c\uc640 \ub2e8\ub458\uc774 \ub9cc\ub098 \uc124\ub4dd\ud558\ub2c8 \ud22c\ud56d\ud558\uc600\ub2e4. 206\ub144 \ucc3d\ud76c\uac00 \ub2e4\uc2dc \ubc18\uae30\ub97c \ub4e4\uc5b4\uc11c \uc6b0\uae08\uacfc \uac19\uc774 \uc9c4\uc555\ud558\uace0 \uc804\uad70\uad50\uc704(\u5178\u8ecd\u2015)\uac00 \ub418\uc5c8\ub2e4. \ud669\uac74\uc801 \uc11c\ud654(\u5f90\u548c)\uc640 \uc0ac\ub9c8\uad6c(\u53f8\u99ac\u4ff1)\uac00 \uc81c\ub0a8\uacfc \ub099\uc548\uc744 \uce58\ubbc0\ub85c \ud0dc\uc0b0\u00b7\uc81c\uad70\u00b7\ud3c9\uc6d0\uc758 \uad70\uc0ac\ub97c \ub3d9\uc6d0\ud558\uc5ec \ud1a0\ubc8c\ud558\uace0 \uadf8 \uc2dd\ub7c9\uc744 \ud655\ubcf4\ud558\uc600\ub2e4. 209\ub144 \ud589\ub839\uad70(\u884c\u9818\u8ecd)\uc5d0 \uc784\uba85\ub418\uc5b4 \uc5ec\uac15\uc758 \ub1cc\uc11c\ub97c \uaca9\ud1f4\ud588\ub2e4.", "answer": "\uc0ac\ub9c8\uad6c", "sentence": "\ud669\uac74\uc801 \uc11c\ud654(\u5f90\u548c)\uc640 \uc0ac\ub9c8\uad6c (\u53f8\u99ac\u4ff1)", "paragraph_sentence": "\uc77c\ucc0d\uc774 \uace0\ud5a5\uc5d0 \uc788\uc744 \ub54c \uc870\uc870\uac00 \uc800\uc9c0\ub978 \uc8c4\ub97c \ub300\uc2e0 \uc9ca\uc5b4\uc9c0\ub824 \ud588\ub294\ub370 \uc870\uc870\uac00 \uc6a9\uc11c\ub97c \ube4c\uc5b4 \uba74\ud558\uc600\ub2e4. \uc870\uc870\uac00 \uac70\ubcd1\ud558\uc790 \ubcc4\ubd80\uc0ac\ub9c8(\u5225\u90e8\u53f8\u99ac)\uc640 \uae30\ub3c4\uc704\uac00 \ub418\uc5b4 \ub530\ub974\uace0 \uc9c4\ub958\ud0dc\uc218\u00b7\uc601\ucc9c\ud0dc\uc218\ub97c \uc9c0\ub0c8\ub2e4. \uad00\ub3c4 \uc804\ud22c\uc5d0\uc11c\ub294 \ub3c5\uad70\uad50\uc704(\u7763\u8ecd\u6821\u5c09)\ub85c\uc11c \uc5f0\uc8fc\u00b7\uc608\uc8fc\u00b7\uc11c\uc8fc\ub85c\ubd80\ud130 \uad70\ub7c9\uc744 \uc870\ub2ec\ud558\uc600\ub2e4. 201\ub144(\uac74\uc548 6\ub144) \uc7a5\ub8cc\uc640 \ud568\uaed8 \ub3d9\ud574\uad70\uc758 \ucc3d\ud76c\ub97c \ud3ec\uc704 \uacf5\uaca9\ud588\uc73c\ub098 \uba87 \ub2ec\uac04 \ud568\ub77d\uc2dc\ud0a4\uc9c0 \ubabb\ud558\uace0 \uad70\ub7c9\uc774 \ub2e4 \ub5a8\uc5b4\uc84c\ub2e4. \ud68c\uad70\ud558\uae30 \uc804\uc5d0 \uc7a5\ub8cc\uac00 \ucc3d\ud76c\uc640 \ub2e8\ub458\uc774 \ub9cc\ub098 \uc124\ub4dd\ud558\ub2c8 \ud22c\ud56d\ud558\uc600\ub2e4. 206\ub144 \ucc3d\ud76c\uac00 \ub2e4\uc2dc \ubc18\uae30\ub97c \ub4e4\uc5b4\uc11c \uc6b0\uae08\uacfc \uac19\uc774 \uc9c4\uc555\ud558\uace0 \uc804\uad70\uad50\uc704(\u5178\u8ecd\u2015)\uac00 \ub418\uc5c8\ub2e4. \ud669\uac74\uc801 \uc11c\ud654(\u5f90\u548c)\uc640 \uc0ac\ub9c8\uad6c (\u53f8\u99ac\u4ff1) \uac00 \uc81c\ub0a8\uacfc \ub099\uc548\uc744 \uce58\ubbc0\ub85c \ud0dc\uc0b0\u00b7\uc81c\uad70\u00b7\ud3c9\uc6d0\uc758 \uad70\uc0ac\ub97c \ub3d9\uc6d0\ud558\uc5ec \ud1a0\ubc8c\ud558\uace0 \uadf8 \uc2dd\ub7c9\uc744 \ud655\ubcf4\ud558\uc600\ub2e4. 209\ub144 \ud589\ub839\uad70(\u884c\u9818\u8ecd)\uc5d0 \uc784\uba85\ub418\uc5b4 \uc5ec\uac15\uc758 \ub1cc\uc11c\ub97c \uaca9\ud1f4\ud588\ub2e4.", "paragraph_answer": "\uc77c\ucc0d\uc774 \uace0\ud5a5\uc5d0 \uc788\uc744 \ub54c \uc870\uc870\uac00 \uc800\uc9c0\ub978 \uc8c4\ub97c \ub300\uc2e0 \uc9ca\uc5b4\uc9c0\ub824 \ud588\ub294\ub370 \uc870\uc870\uac00 \uc6a9\uc11c\ub97c \ube4c\uc5b4 \uba74\ud558\uc600\ub2e4. \uc870\uc870\uac00 \uac70\ubcd1\ud558\uc790 \ubcc4\ubd80\uc0ac\ub9c8(\u5225\u90e8\u53f8\u99ac)\uc640 \uae30\ub3c4\uc704\uac00 \ub418\uc5b4 \ub530\ub974\uace0 \uc9c4\ub958\ud0dc\uc218\u00b7\uc601\ucc9c\ud0dc\uc218\ub97c \uc9c0\ub0c8\ub2e4. \uad00\ub3c4 \uc804\ud22c\uc5d0\uc11c\ub294 \ub3c5\uad70\uad50\uc704(\u7763\u8ecd\u6821\u5c09)\ub85c\uc11c \uc5f0\uc8fc\u00b7\uc608\uc8fc\u00b7\uc11c\uc8fc\ub85c\ubd80\ud130 \uad70\ub7c9\uc744 \uc870\ub2ec\ud558\uc600\ub2e4. 201\ub144(\uac74\uc548 6\ub144) \uc7a5\ub8cc\uc640 \ud568\uaed8 \ub3d9\ud574\uad70\uc758 \ucc3d\ud76c\ub97c \ud3ec\uc704 \uacf5\uaca9\ud588\uc73c\ub098 \uba87 \ub2ec\uac04 \ud568\ub77d\uc2dc\ud0a4\uc9c0 \ubabb\ud558\uace0 \uad70\ub7c9\uc774 \ub2e4 \ub5a8\uc5b4\uc84c\ub2e4. \ud68c\uad70\ud558\uae30 \uc804\uc5d0 \uc7a5\ub8cc\uac00 \ucc3d\ud76c\uc640 \ub2e8\ub458\uc774 \ub9cc\ub098 \uc124\ub4dd\ud558\ub2c8 \ud22c\ud56d\ud558\uc600\ub2e4. 206\ub144 \ucc3d\ud76c\uac00 \ub2e4\uc2dc \ubc18\uae30\ub97c \ub4e4\uc5b4\uc11c \uc6b0\uae08\uacfc \uac19\uc774 \uc9c4\uc555\ud558\uace0 \uc804\uad70\uad50\uc704(\u5178\u8ecd\u2015)\uac00 \ub418\uc5c8\ub2e4. \ud669\uac74\uc801 \uc11c\ud654(\u5f90\u548c)\uc640 \uc0ac\ub9c8\uad6c (\u53f8\u99ac\u4ff1)\uac00 \uc81c\ub0a8\uacfc \ub099\uc548\uc744 \uce58\ubbc0\ub85c \ud0dc\uc0b0\u00b7\uc81c\uad70\u00b7\ud3c9\uc6d0\uc758 \uad70\uc0ac\ub97c \ub3d9\uc6d0\ud558\uc5ec \ud1a0\ubc8c\ud558\uace0 \uadf8 \uc2dd\ub7c9\uc744 \ud655\ubcf4\ud558\uc600\ub2e4. 209\ub144 \ud589\ub839\uad70(\u884c\u9818\u8ecd)\uc5d0 \uc784\uba85\ub418\uc5b4 \uc5ec\uac15\uc758 \ub1cc\uc11c\ub97c \uaca9\ud1f4\ud588\ub2e4.", "sentence_answer": "\ud669\uac74\uc801 \uc11c\ud654(\u5f90\u548c)\uc640 \uc0ac\ub9c8\uad6c (\u53f8\u99ac\u4ff1)", "paragraph_id": "6569851-0-1", "paragraph_question": "question: \ud669\uac74\uc801 \uc11c\ud654\uc640 \ud568\uaed8 \uc81c\ub0a8\uacfc \ub099\uc548\uc744 \uacf5\uaca9\ud55c \uc0ac\ub78c\uc740?, context: \uc77c\ucc0d\uc774 \uace0\ud5a5\uc5d0 \uc788\uc744 \ub54c \uc870\uc870\uac00 \uc800\uc9c0\ub978 \uc8c4\ub97c \ub300\uc2e0 \uc9ca\uc5b4\uc9c0\ub824 \ud588\ub294\ub370 \uc870\uc870\uac00 \uc6a9\uc11c\ub97c \ube4c\uc5b4 \uba74\ud558\uc600\ub2e4. \uc870\uc870\uac00 \uac70\ubcd1\ud558\uc790 \ubcc4\ubd80\uc0ac\ub9c8(\u5225\u90e8\u53f8\u99ac)\uc640 \uae30\ub3c4\uc704\uac00 \ub418\uc5b4 \ub530\ub974\uace0 \uc9c4\ub958\ud0dc\uc218\u00b7\uc601\ucc9c\ud0dc\uc218\ub97c \uc9c0\ub0c8\ub2e4. \uad00\ub3c4 \uc804\ud22c\uc5d0\uc11c\ub294 \ub3c5\uad70\uad50\uc704(\u7763\u8ecd\u6821\u5c09)\ub85c\uc11c \uc5f0\uc8fc\u00b7\uc608\uc8fc\u00b7\uc11c\uc8fc\ub85c\ubd80\ud130 \uad70\ub7c9\uc744 \uc870\ub2ec\ud558\uc600\ub2e4. 201\ub144(\uac74\uc548 6\ub144) \uc7a5\ub8cc\uc640 \ud568\uaed8 \ub3d9\ud574\uad70\uc758 \ucc3d\ud76c\ub97c \ud3ec\uc704 \uacf5\uaca9\ud588\uc73c\ub098 \uba87 \ub2ec\uac04 \ud568\ub77d\uc2dc\ud0a4\uc9c0 \ubabb\ud558\uace0 \uad70\ub7c9\uc774 \ub2e4 \ub5a8\uc5b4\uc84c\ub2e4. \ud68c\uad70\ud558\uae30 \uc804\uc5d0 \uc7a5\ub8cc\uac00 \ucc3d\ud76c\uc640 \ub2e8\ub458\uc774 \ub9cc\ub098 \uc124\ub4dd\ud558\ub2c8 \ud22c\ud56d\ud558\uc600\ub2e4. 206\ub144 \ucc3d\ud76c\uac00 \ub2e4\uc2dc \ubc18\uae30\ub97c \ub4e4\uc5b4\uc11c \uc6b0\uae08\uacfc \uac19\uc774 \uc9c4\uc555\ud558\uace0 \uc804\uad70\uad50\uc704(\u5178\u8ecd\u2015)\uac00 \ub418\uc5c8\ub2e4. \ud669\uac74\uc801 \uc11c\ud654(\u5f90\u548c)\uc640 \uc0ac\ub9c8\uad6c(\u53f8\u99ac\u4ff1)\uac00 \uc81c\ub0a8\uacfc \ub099\uc548\uc744 \uce58\ubbc0\ub85c \ud0dc\uc0b0\u00b7\uc81c\uad70\u00b7\ud3c9\uc6d0\uc758 \uad70\uc0ac\ub97c \ub3d9\uc6d0\ud558\uc5ec \ud1a0\ubc8c\ud558\uace0 \uadf8 \uc2dd\ub7c9\uc744 \ud655\ubcf4\ud558\uc600\ub2e4. 209\ub144 \ud589\ub839\uad70(\u884c\u9818\u8ecd)\uc5d0 \uc784\uba85\ub418\uc5b4 \uc5ec\uac15\uc758 \ub1cc\uc11c\ub97c \uaca9\ud1f4\ud588\ub2e4."} {"question": "\uc18c\ub828\uc758 \ub2c8\ucf5c\ub77c\uc774 \ud45c\ub3c4\ub85c\ube44\uce58 \ubc14\ud22c\ud2f4\uc774 \uc9c0\ud718\ud558\ub294 \uc81c1\uc6b0\ud06c\ub77c\uc774\ub098 \uc804\uc120\uad70\uc774 \ud0a4\uc608\ud504 \uadfc\uad50\uc5d0\uc11c \ub9de\ub2e5\ub728\ub9b0 \ub3c5\uc77c\uc758 \uad70\ub2e8\uc740?", "paragraph": "1943\ub144 9\uc6d4 \ub9cc\uc288\ud0c0\uc778\uc740 \ub4dc\ub124\ud504\ub974 \uac15 \uc11c\uc548\uae4c\uc9c0 \ud6c4\ud1f4\ud588\uace0, \ud6c4\ud1f4\ub294 \ub300\ubd80\ubd84 \uc9c8\uc11c\uc815\uc5f0\ud558\uac8c \uc9c4\ud589\ub418\uace0 \uc788\uc5c8\ub2e4. \uadf8\ub7ec\ub098 \uc77c\ubd80 \uacbd\uc6b0 \ubb34\uc9c8\uc11c\ud558\uac8c \uada4\uc8fc\ud558\ub4ef \ud6c4\ud1f4\ud588\ub2e4. \uc9c0\uce5c \ubcd1\uc0ac\ub4e4\uc774 \"\uc774\uc131\uc744 \uc783\uc5b4\uac00\uace0\" \uc788\uc5c8\uae30 \ub54c\ubb38\uc774\ub2e4. \uc18c\ub828 \ubbfc\uac04\uc778 \uc218\uc2ed\ub9cc \uba85\uc774 \ub9cc\uc288\ud0c0\uc778\uc758 \ubd80\ub300\uc640 \ud568\uaed8 \uc11c\ucabd\uc73c\ub85c \uc6c0\uc9c1\uc600\ub2e4. \uadf8\ub4e4\uc740 \ub300\ubd80\ubd84 \uac00\ucd95\uc774\uba70 \uac1c\uc778\uc7ac\uc0b0\uc744 \uc9c0\ub2c8\uace0 \uc788\uc5c8\ub2e4. \ub9cc\uc288\ud0c0\uc778\uc740 \uc18c\ub828\uad70\uc758 \ub2e4\uc74c \uacf5\uaca9\uc774 \ud0a4\uc608\ud504 \ubc29\ud5a5\uc77c \uac83\uc774\ub77c\uace0 \uc815\ud655\ud788 \uc608\uce21\ud588\uc9c0\ub9cc, \uc18c\ub828\uce21\uc740 \uc804\uc5ed \ub0b4\ub0b4 \ub9c8\uc2a4\ud0a4\ub86d\uce74\ub77c\ub294 \uae30\ub9cc\uc791\uc804\uc73c\ub85c \uacf5\uc138 \uc2dc\uc791\uc2dc\uae30\uc640 \uc815\ud655\ud55c \uc704\uce58\ub97c \uc54c\uc9c0 \ubabb\ud558\uac8c \ud588\ub2e4. \uc5ed\uc0ac\ud559\uc790 \uc70c\ub9ac\uc5c4\uc2a8 \uba38\ub808\uc774\uc640 \uc568\ub7f0 \ub9ac\ub4dc \ubc00\ub808(Allan Reed Millett)\uc5d0 \ub530\ub974\uba74, \ub3c5\uc77c \uc7a5\uc131\ub4e4\uc740 \ub098\uce58\uc758 \uc778\uc885\uc8fc\uc758\ub97c \ubbff\uc5c8\uace0, \uc5f4\ub4f1\ud55c \u201c\uc2ac\ub77c\ube0c\uac00 \ub3c5\uc77c\uc758 \uc815\ubcf4\uccb4\uacc4\ub97c \uadf8\ub807\uac8c \uc9c0\uc18d\uc801\uc73c\ub85c \uad50\ub780\ud560 \uc218 \uc788\ub2e4\ub294 \uc0dd\uac01\uc740 \uac10\ud788 \ud560 \uc218\uc870\ucc28 \uc5c6\uc5c8\ub2e4.\u201d. \ub2c8\ucf5c\ub77c\uc774 \ud45c\ub3c4\ub85c\ube44\uce58 \ubc14\ud22c\ud2f4\uc774 \uc9c0\ud718\ud558\ub294 \uc81c1\uc6b0\ud06c\ub77c\uc774\ub098 \uc804\uc120\uad70\uc740 \ud0a4\uc608\ud504 \uadfc\uad50\uc5d0\uc11c \ub3c5\uc77c \uc81c4\uae30\uac11\uad70\uacfc \uc870\uc6b0\ud588\ub2e4. \ubcd1\ub825\uba74\uc5d0\uc11c \uc18c\ub828\uad70\uc774 \ub3c5\uc77c\uad70\uc744 \uc555\ub3c4\ud588\ub2e4. \ubc14\ud22c\ud2f4\uc740 \uc6b0\uc120 \ud0a4\uc608\ud504 \ubc14\ub85c \ubd81\ucabd \ub9ac\uc6b0\ud14c\uc9c0\ub97c \ucc14\ub7ec\ub4e4\uc5b4\uac14\uace0, \uadf8 \ub4a4 \ub0a8\ucabd\uc758 \ubd80\ud06c\ub9b0 \uadfc\uad50\ub97c \uacf5\uaca9\ud588\ub2e4. \uc774\uac8c 11\uc6d4 1\uc77c\uc774\uc5c8\ub2e4. \ub3c5\uc77c\uad70\uc740 \ubd80\ud06c\ub9b0\uc774 \uc8fc\uacf5(\u4e3b\u653b)\uc758 \ub300\uc0c1\uc77c \uac83\uc774\ub77c\uace0 \uc0dd\uac01\ud558\uc5ec \ubc14\ud22c\ud2f4\uc774 \ub9ac\uc6b0\ud14c\uc9c0\uc5d0 \uad50\ub450\ubcf4\ub97c \ub9c8\ub828\ud574 \ub4dc\ub124\ud504\ub974 \uac15\uc744 \ub3c4\ud558\ud560 \uc218 \uc788\uac8c \ub418\uc790 \uae5c\uc9dd \ub180\ub790\ub2e4. \ud0a4\uc608\ud504\ub294 11\uc6d4 6\uc77c \uc18c\ub828\uad70\uc5d0\uac8c \ud0c8\ud658\ub418\uc5c8\ub2e4. \ub3c5\uc77c \uc81c17\uad70\uc740 10\uc6d4 28\uc77c \uc81c4\uc6b0\ud06c\ub77c\uc774\ub098 \uc804\uc120\uad70\uc744 \uacf5\uaca9\ud558\ub2e4\uac00 \ud06c\ub9bc \ubc18\ub3c4\uc5d0 \uace0\ub9bd\ub2f9\ud588\ub2e4.", "answer": "\uc81c4\uae30\uac11\uad70", "sentence": "\ub3c5\uc77c \uc81c4\uae30\uac11\uad70 \uacfc \uc870\uc6b0\ud588\ub2e4.", "paragraph_sentence": "1943\ub144 9\uc6d4 \ub9cc\uc288\ud0c0\uc778\uc740 \ub4dc\ub124\ud504\ub974 \uac15 \uc11c\uc548\uae4c\uc9c0 \ud6c4\ud1f4\ud588\uace0, \ud6c4\ud1f4\ub294 \ub300\ubd80\ubd84 \uc9c8\uc11c\uc815\uc5f0\ud558\uac8c \uc9c4\ud589\ub418\uace0 \uc788\uc5c8\ub2e4. \uadf8\ub7ec\ub098 \uc77c\ubd80 \uacbd\uc6b0 \ubb34\uc9c8\uc11c\ud558\uac8c \uada4\uc8fc\ud558\ub4ef \ud6c4\ud1f4\ud588\ub2e4. \uc9c0\uce5c \ubcd1\uc0ac\ub4e4\uc774 \"\uc774\uc131\uc744 \uc783\uc5b4\uac00\uace0\" \uc788\uc5c8\uae30 \ub54c\ubb38\uc774\ub2e4. \uc18c\ub828 \ubbfc\uac04\uc778 \uc218\uc2ed\ub9cc \uba85\uc774 \ub9cc\uc288\ud0c0\uc778\uc758 \ubd80\ub300\uc640 \ud568\uaed8 \uc11c\ucabd\uc73c\ub85c \uc6c0\uc9c1\uc600\ub2e4. \uadf8\ub4e4\uc740 \ub300\ubd80\ubd84 \uac00\ucd95\uc774\uba70 \uac1c\uc778\uc7ac\uc0b0\uc744 \uc9c0\ub2c8\uace0 \uc788\uc5c8\ub2e4. \ub9cc\uc288\ud0c0\uc778\uc740 \uc18c\ub828\uad70\uc758 \ub2e4\uc74c \uacf5\uaca9\uc774 \ud0a4\uc608\ud504 \ubc29\ud5a5\uc77c \uac83\uc774\ub77c\uace0 \uc815\ud655\ud788 \uc608\uce21\ud588\uc9c0\ub9cc, \uc18c\ub828\uce21\uc740 \uc804\uc5ed \ub0b4\ub0b4 \ub9c8\uc2a4\ud0a4\ub86d\uce74\ub77c\ub294 \uae30\ub9cc\uc791\uc804\uc73c\ub85c \uacf5\uc138 \uc2dc\uc791\uc2dc\uae30\uc640 \uc815\ud655\ud55c \uc704\uce58\ub97c \uc54c\uc9c0 \ubabb\ud558\uac8c \ud588\ub2e4. \uc5ed\uc0ac\ud559\uc790 \uc70c\ub9ac\uc5c4\uc2a8 \uba38\ub808\uc774\uc640 \uc568\ub7f0 \ub9ac\ub4dc \ubc00\ub808(Allan Reed Millett)\uc5d0 \ub530\ub974\uba74, \ub3c5\uc77c \uc7a5\uc131\ub4e4\uc740 \ub098\uce58\uc758 \uc778\uc885\uc8fc\uc758\ub97c \ubbff\uc5c8\uace0, \uc5f4\ub4f1\ud55c \u201c\uc2ac\ub77c\ube0c\uac00 \ub3c5\uc77c\uc758 \uc815\ubcf4\uccb4\uacc4\ub97c \uadf8\ub807\uac8c \uc9c0\uc18d\uc801\uc73c\ub85c \uad50\ub780\ud560 \uc218 \uc788\ub2e4\ub294 \uc0dd\uac01\uc740 \uac10\ud788 \ud560 \uc218\uc870\ucc28 \uc5c6\uc5c8\ub2e4. \u201d. \ub2c8\ucf5c\ub77c\uc774 \ud45c\ub3c4\ub85c\ube44\uce58 \ubc14\ud22c\ud2f4\uc774 \uc9c0\ud718\ud558\ub294 \uc81c1\uc6b0\ud06c\ub77c\uc774\ub098 \uc804\uc120\uad70\uc740 \ud0a4\uc608\ud504 \uadfc\uad50\uc5d0\uc11c \ub3c5\uc77c \uc81c4\uae30\uac11\uad70 \uacfc \uc870\uc6b0\ud588\ub2e4. \ubcd1\ub825\uba74\uc5d0\uc11c \uc18c\ub828\uad70\uc774 \ub3c5\uc77c\uad70\uc744 \uc555\ub3c4\ud588\ub2e4. \ubc14\ud22c\ud2f4\uc740 \uc6b0\uc120 \ud0a4\uc608\ud504 \ubc14\ub85c \ubd81\ucabd \ub9ac\uc6b0\ud14c\uc9c0\ub97c \ucc14\ub7ec\ub4e4\uc5b4\uac14\uace0, \uadf8 \ub4a4 \ub0a8\ucabd\uc758 \ubd80\ud06c\ub9b0 \uadfc\uad50\ub97c \uacf5\uaca9\ud588\ub2e4. \uc774\uac8c 11\uc6d4 1\uc77c\uc774\uc5c8\ub2e4. \ub3c5\uc77c\uad70\uc740 \ubd80\ud06c\ub9b0\uc774 \uc8fc\uacf5(\u4e3b\u653b)\uc758 \ub300\uc0c1\uc77c \uac83\uc774\ub77c\uace0 \uc0dd\uac01\ud558\uc5ec \ubc14\ud22c\ud2f4\uc774 \ub9ac\uc6b0\ud14c\uc9c0\uc5d0 \uad50\ub450\ubcf4\ub97c \ub9c8\ub828\ud574 \ub4dc\ub124\ud504\ub974 \uac15\uc744 \ub3c4\ud558\ud560 \uc218 \uc788\uac8c \ub418\uc790 \uae5c\uc9dd \ub180\ub790\ub2e4. \ud0a4\uc608\ud504\ub294 11\uc6d4 6\uc77c \uc18c\ub828\uad70\uc5d0\uac8c \ud0c8\ud658\ub418\uc5c8\ub2e4. \ub3c5\uc77c \uc81c17\uad70\uc740 10\uc6d4 28\uc77c \uc81c4\uc6b0\ud06c\ub77c\uc774\ub098 \uc804\uc120\uad70\uc744 \uacf5\uaca9\ud558\ub2e4\uac00 \ud06c\ub9bc \ubc18\ub3c4\uc5d0 \uace0\ub9bd\ub2f9\ud588\ub2e4.", "paragraph_answer": "1943\ub144 9\uc6d4 \ub9cc\uc288\ud0c0\uc778\uc740 \ub4dc\ub124\ud504\ub974 \uac15 \uc11c\uc548\uae4c\uc9c0 \ud6c4\ud1f4\ud588\uace0, \ud6c4\ud1f4\ub294 \ub300\ubd80\ubd84 \uc9c8\uc11c\uc815\uc5f0\ud558\uac8c \uc9c4\ud589\ub418\uace0 \uc788\uc5c8\ub2e4. \uadf8\ub7ec\ub098 \uc77c\ubd80 \uacbd\uc6b0 \ubb34\uc9c8\uc11c\ud558\uac8c \uada4\uc8fc\ud558\ub4ef \ud6c4\ud1f4\ud588\ub2e4. \uc9c0\uce5c \ubcd1\uc0ac\ub4e4\uc774 \"\uc774\uc131\uc744 \uc783\uc5b4\uac00\uace0\" \uc788\uc5c8\uae30 \ub54c\ubb38\uc774\ub2e4. \uc18c\ub828 \ubbfc\uac04\uc778 \uc218\uc2ed\ub9cc \uba85\uc774 \ub9cc\uc288\ud0c0\uc778\uc758 \ubd80\ub300\uc640 \ud568\uaed8 \uc11c\ucabd\uc73c\ub85c \uc6c0\uc9c1\uc600\ub2e4. \uadf8\ub4e4\uc740 \ub300\ubd80\ubd84 \uac00\ucd95\uc774\uba70 \uac1c\uc778\uc7ac\uc0b0\uc744 \uc9c0\ub2c8\uace0 \uc788\uc5c8\ub2e4. \ub9cc\uc288\ud0c0\uc778\uc740 \uc18c\ub828\uad70\uc758 \ub2e4\uc74c \uacf5\uaca9\uc774 \ud0a4\uc608\ud504 \ubc29\ud5a5\uc77c \uac83\uc774\ub77c\uace0 \uc815\ud655\ud788 \uc608\uce21\ud588\uc9c0\ub9cc, \uc18c\ub828\uce21\uc740 \uc804\uc5ed \ub0b4\ub0b4 \ub9c8\uc2a4\ud0a4\ub86d\uce74\ub77c\ub294 \uae30\ub9cc\uc791\uc804\uc73c\ub85c \uacf5\uc138 \uc2dc\uc791\uc2dc\uae30\uc640 \uc815\ud655\ud55c \uc704\uce58\ub97c \uc54c\uc9c0 \ubabb\ud558\uac8c \ud588\ub2e4. \uc5ed\uc0ac\ud559\uc790 \uc70c\ub9ac\uc5c4\uc2a8 \uba38\ub808\uc774\uc640 \uc568\ub7f0 \ub9ac\ub4dc \ubc00\ub808(Allan Reed Millett)\uc5d0 \ub530\ub974\uba74, \ub3c5\uc77c \uc7a5\uc131\ub4e4\uc740 \ub098\uce58\uc758 \uc778\uc885\uc8fc\uc758\ub97c \ubbff\uc5c8\uace0, \uc5f4\ub4f1\ud55c \u201c\uc2ac\ub77c\ube0c\uac00 \ub3c5\uc77c\uc758 \uc815\ubcf4\uccb4\uacc4\ub97c \uadf8\ub807\uac8c \uc9c0\uc18d\uc801\uc73c\ub85c \uad50\ub780\ud560 \uc218 \uc788\ub2e4\ub294 \uc0dd\uac01\uc740 \uac10\ud788 \ud560 \uc218\uc870\ucc28 \uc5c6\uc5c8\ub2e4.\u201d. \ub2c8\ucf5c\ub77c\uc774 \ud45c\ub3c4\ub85c\ube44\uce58 \ubc14\ud22c\ud2f4\uc774 \uc9c0\ud718\ud558\ub294 \uc81c1\uc6b0\ud06c\ub77c\uc774\ub098 \uc804\uc120\uad70\uc740 \ud0a4\uc608\ud504 \uadfc\uad50\uc5d0\uc11c \ub3c5\uc77c \uc81c4\uae30\uac11\uad70 \uacfc \uc870\uc6b0\ud588\ub2e4. \ubcd1\ub825\uba74\uc5d0\uc11c \uc18c\ub828\uad70\uc774 \ub3c5\uc77c\uad70\uc744 \uc555\ub3c4\ud588\ub2e4. \ubc14\ud22c\ud2f4\uc740 \uc6b0\uc120 \ud0a4\uc608\ud504 \ubc14\ub85c \ubd81\ucabd \ub9ac\uc6b0\ud14c\uc9c0\ub97c \ucc14\ub7ec\ub4e4\uc5b4\uac14\uace0, \uadf8 \ub4a4 \ub0a8\ucabd\uc758 \ubd80\ud06c\ub9b0 \uadfc\uad50\ub97c \uacf5\uaca9\ud588\ub2e4. \uc774\uac8c 11\uc6d4 1\uc77c\uc774\uc5c8\ub2e4. \ub3c5\uc77c\uad70\uc740 \ubd80\ud06c\ub9b0\uc774 \uc8fc\uacf5(\u4e3b\u653b)\uc758 \ub300\uc0c1\uc77c \uac83\uc774\ub77c\uace0 \uc0dd\uac01\ud558\uc5ec \ubc14\ud22c\ud2f4\uc774 \ub9ac\uc6b0\ud14c\uc9c0\uc5d0 \uad50\ub450\ubcf4\ub97c \ub9c8\ub828\ud574 \ub4dc\ub124\ud504\ub974 \uac15\uc744 \ub3c4\ud558\ud560 \uc218 \uc788\uac8c \ub418\uc790 \uae5c\uc9dd \ub180\ub790\ub2e4. \ud0a4\uc608\ud504\ub294 11\uc6d4 6\uc77c \uc18c\ub828\uad70\uc5d0\uac8c \ud0c8\ud658\ub418\uc5c8\ub2e4. \ub3c5\uc77c \uc81c17\uad70\uc740 10\uc6d4 28\uc77c \uc81c4\uc6b0\ud06c\ub77c\uc774\ub098 \uc804\uc120\uad70\uc744 \uacf5\uaca9\ud558\ub2e4\uac00 \ud06c\ub9bc \ubc18\ub3c4\uc5d0 \uace0\ub9bd\ub2f9\ud588\ub2e4.", "sentence_answer": "\ub3c5\uc77c \uc81c4\uae30\uac11\uad70 \uacfc \uc870\uc6b0\ud588\ub2e4.", "paragraph_id": "6540644-28-1", "paragraph_question": "question: \uc18c\ub828\uc758 \ub2c8\ucf5c\ub77c\uc774 \ud45c\ub3c4\ub85c\ube44\uce58 \ubc14\ud22c\ud2f4\uc774 \uc9c0\ud718\ud558\ub294 \uc81c1\uc6b0\ud06c\ub77c\uc774\ub098 \uc804\uc120\uad70\uc774 \ud0a4\uc608\ud504 \uadfc\uad50\uc5d0\uc11c \ub9de\ub2e5\ub728\ub9b0 \ub3c5\uc77c\uc758 \uad70\ub2e8\uc740?, context: 1943\ub144 9\uc6d4 \ub9cc\uc288\ud0c0\uc778\uc740 \ub4dc\ub124\ud504\ub974 \uac15 \uc11c\uc548\uae4c\uc9c0 \ud6c4\ud1f4\ud588\uace0, \ud6c4\ud1f4\ub294 \ub300\ubd80\ubd84 \uc9c8\uc11c\uc815\uc5f0\ud558\uac8c \uc9c4\ud589\ub418\uace0 \uc788\uc5c8\ub2e4. \uadf8\ub7ec\ub098 \uc77c\ubd80 \uacbd\uc6b0 \ubb34\uc9c8\uc11c\ud558\uac8c \uada4\uc8fc\ud558\ub4ef \ud6c4\ud1f4\ud588\ub2e4. \uc9c0\uce5c \ubcd1\uc0ac\ub4e4\uc774 \"\uc774\uc131\uc744 \uc783\uc5b4\uac00\uace0\" \uc788\uc5c8\uae30 \ub54c\ubb38\uc774\ub2e4. \uc18c\ub828 \ubbfc\uac04\uc778 \uc218\uc2ed\ub9cc \uba85\uc774 \ub9cc\uc288\ud0c0\uc778\uc758 \ubd80\ub300\uc640 \ud568\uaed8 \uc11c\ucabd\uc73c\ub85c \uc6c0\uc9c1\uc600\ub2e4. \uadf8\ub4e4\uc740 \ub300\ubd80\ubd84 \uac00\ucd95\uc774\uba70 \uac1c\uc778\uc7ac\uc0b0\uc744 \uc9c0\ub2c8\uace0 \uc788\uc5c8\ub2e4. \ub9cc\uc288\ud0c0\uc778\uc740 \uc18c\ub828\uad70\uc758 \ub2e4\uc74c \uacf5\uaca9\uc774 \ud0a4\uc608\ud504 \ubc29\ud5a5\uc77c \uac83\uc774\ub77c\uace0 \uc815\ud655\ud788 \uc608\uce21\ud588\uc9c0\ub9cc, \uc18c\ub828\uce21\uc740 \uc804\uc5ed \ub0b4\ub0b4 \ub9c8\uc2a4\ud0a4\ub86d\uce74\ub77c\ub294 \uae30\ub9cc\uc791\uc804\uc73c\ub85c \uacf5\uc138 \uc2dc\uc791\uc2dc\uae30\uc640 \uc815\ud655\ud55c \uc704\uce58\ub97c \uc54c\uc9c0 \ubabb\ud558\uac8c \ud588\ub2e4. \uc5ed\uc0ac\ud559\uc790 \uc70c\ub9ac\uc5c4\uc2a8 \uba38\ub808\uc774\uc640 \uc568\ub7f0 \ub9ac\ub4dc \ubc00\ub808(Allan Reed Millett)\uc5d0 \ub530\ub974\uba74, \ub3c5\uc77c \uc7a5\uc131\ub4e4\uc740 \ub098\uce58\uc758 \uc778\uc885\uc8fc\uc758\ub97c \ubbff\uc5c8\uace0, \uc5f4\ub4f1\ud55c \u201c\uc2ac\ub77c\ube0c\uac00 \ub3c5\uc77c\uc758 \uc815\ubcf4\uccb4\uacc4\ub97c \uadf8\ub807\uac8c \uc9c0\uc18d\uc801\uc73c\ub85c \uad50\ub780\ud560 \uc218 \uc788\ub2e4\ub294 \uc0dd\uac01\uc740 \uac10\ud788 \ud560 \uc218\uc870\ucc28 \uc5c6\uc5c8\ub2e4.\u201d. \ub2c8\ucf5c\ub77c\uc774 \ud45c\ub3c4\ub85c\ube44\uce58 \ubc14\ud22c\ud2f4\uc774 \uc9c0\ud718\ud558\ub294 \uc81c1\uc6b0\ud06c\ub77c\uc774\ub098 \uc804\uc120\uad70\uc740 \ud0a4\uc608\ud504 \uadfc\uad50\uc5d0\uc11c \ub3c5\uc77c \uc81c4\uae30\uac11\uad70\uacfc \uc870\uc6b0\ud588\ub2e4. \ubcd1\ub825\uba74\uc5d0\uc11c \uc18c\ub828\uad70\uc774 \ub3c5\uc77c\uad70\uc744 \uc555\ub3c4\ud588\ub2e4. \ubc14\ud22c\ud2f4\uc740 \uc6b0\uc120 \ud0a4\uc608\ud504 \ubc14\ub85c \ubd81\ucabd \ub9ac\uc6b0\ud14c\uc9c0\ub97c \ucc14\ub7ec\ub4e4\uc5b4\uac14\uace0, \uadf8 \ub4a4 \ub0a8\ucabd\uc758 \ubd80\ud06c\ub9b0 \uadfc\uad50\ub97c \uacf5\uaca9\ud588\ub2e4. \uc774\uac8c 11\uc6d4 1\uc77c\uc774\uc5c8\ub2e4. \ub3c5\uc77c\uad70\uc740 \ubd80\ud06c\ub9b0\uc774 \uc8fc\uacf5(\u4e3b\u653b)\uc758 \ub300\uc0c1\uc77c \uac83\uc774\ub77c\uace0 \uc0dd\uac01\ud558\uc5ec \ubc14\ud22c\ud2f4\uc774 \ub9ac\uc6b0\ud14c\uc9c0\uc5d0 \uad50\ub450\ubcf4\ub97c \ub9c8\ub828\ud574 \ub4dc\ub124\ud504\ub974 \uac15\uc744 \ub3c4\ud558\ud560 \uc218 \uc788\uac8c \ub418\uc790 \uae5c\uc9dd \ub180\ub790\ub2e4. \ud0a4\uc608\ud504\ub294 11\uc6d4 6\uc77c \uc18c\ub828\uad70\uc5d0\uac8c \ud0c8\ud658\ub418\uc5c8\ub2e4. \ub3c5\uc77c \uc81c17\uad70\uc740 10\uc6d4 28\uc77c \uc81c4\uc6b0\ud06c\ub77c\uc774\ub098 \uc804\uc120\uad70\uc744 \uacf5\uaca9\ud558\ub2e4\uac00 \ud06c\ub9bc \ubc18\ub3c4\uc5d0 \uace0\ub9bd\ub2f9\ud588\ub2e4."} {"question": "\uc57c\ub178 \uc544\ud0a4\ud788\ub85c\uac00 \ud55c\uc2e0 \ud0c0\uc774\uac70\uc2a4\uc5d0\uc11c \ub9e1\uc740 \ud3ec\uc9c0\uc158\uc740?", "paragraph": "\ud55c\uc2e0 \ud0c0\uc774\uac70\uc2a4\uc5d0\uc11c\ub294 \ub2f9\uc2dc \ud55c\uc2e0\uc758 \uac10\ub3c5\uc778 \uc694\uc2dc\ub2e4 \uc694\uc2dc\uc624\uc5d0\uac8c \ub192\uc740 \ud3c9\uac00\ub97c \ubc1b\uc73c\uba70, \uc57c\ub9c8\ub2e4 \uac00\uc4f0\ud788\ucf54\ub97c \ubc00\uc5b4\ub0b4\uace0 \uc8fc\uc804 \ud3ec\uc218 \uc790\ub9ac\ub97c \ucc28\uc9c0\ud558\uc600\ub2e4. 1998\ub144 5\uc6d4 26\uc77c\uc5d0 \uc5f4\ub9b0 \uc774\uc804 \uc18c\uc18d\ud300\uc778 \uc8fc\ub2c8\uce58\uc640\uc758 \uacbd\uae30\uc5d0\uc11c \uac00\uc640\uc9c0\ub9ac \ub370\uc4f0\ub85c\uc640 \ud568\uaed8 \ub450 \ubc88\uc9f8 \ub178\ud788\ud2b8 \ub178\ub7f0\uc744 \uae30\ub85d\ud588\uace0, 1999\ub144\uc5d0\ub294 \ucc98\uc74c\uc73c\ub85c 3\ud560 \ub300\uc758 \ud0c0\uc728\uc744 \uae30\ub85d\ud588\ub2e4. 2001\ub144\uc5d0 \uac10\ub3c5\uc9c1\uc744 \uc0ac\uc784\ud55c \ub178\ubb34\ub77c \uac00\uc4f0\uc57c\uc758 \ud6c4\uc784\uc73c\ub85c \uc8fc\ub2c8\uce58 \uc18c\uc18d \uc2dc\uc808\uc5d0 \uc790\uc2e0\uc744 \ud2b8\ub808\uc774\ub4dc\ub85c \ubc29\ucd9c\ud55c \ud638\uc2dc\ub178 \uc13c\uc774\uce58\uac00 \ud55c\uc2e0\uc758 \ucc28\uae30 \uac10\ub3c5\uc73c\ub85c \ucde8\uc784\ud558\uc790 \ub2e4\uc2dc \uc911\uc6a9\ub418\uc9c0 \uc54a\uace0 \ubc29\ucd9c\ub420\uae4c\ubd10 \ubd88\uc548\ud574 \ud558\uba70 \uc2e4\ub9dd\uac10\uc744 \ub4dc\ub7ec\ub0b4\uae30\ub3c4 \ud588\ub2e4. 2002\ub144\uc5d0\ub294 \ub450 \ucc28\ub840\uc758 \uace8\uc808\uc0c1\uc744 \uc785\ub294 \ub4f1 \ubd80\uc0c1\uc774 \uc7a6\uc544\uc9c0\uba74\uc11c \ud300\ub3c4 \uc57c\ub178\uc758 \uc804\ub825 \uc774\ud0c8\uacfc \ud568\uaed8 \ucd5c\ud558\uc704(4\uc704)\ub97c \uae30\ub85d\ud588\ub2e4.", "answer": "\ud3ec\uc218", "sentence": "\ud55c\uc2e0 \ud0c0\uc774\uac70\uc2a4\uc5d0\uc11c\ub294 \ub2f9\uc2dc \ud55c\uc2e0\uc758 \uac10\ub3c5\uc778 \uc694\uc2dc\ub2e4 \uc694\uc2dc\uc624\uc5d0\uac8c \ub192\uc740 \ud3c9\uac00\ub97c \ubc1b\uc73c\uba70, \uc57c\ub9c8\ub2e4 \uac00\uc4f0\ud788\ucf54\ub97c \ubc00\uc5b4\ub0b4\uace0 \uc8fc\uc804 \ud3ec\uc218 \uc790\ub9ac\ub97c \ucc28\uc9c0\ud558\uc600\ub2e4.", "paragraph_sentence": " \ud55c\uc2e0 \ud0c0\uc774\uac70\uc2a4\uc5d0\uc11c\ub294 \ub2f9\uc2dc \ud55c\uc2e0\uc758 \uac10\ub3c5\uc778 \uc694\uc2dc\ub2e4 \uc694\uc2dc\uc624\uc5d0\uac8c \ub192\uc740 \ud3c9\uac00\ub97c \ubc1b\uc73c\uba70, \uc57c\ub9c8\ub2e4 \uac00\uc4f0\ud788\ucf54\ub97c \ubc00\uc5b4\ub0b4\uace0 \uc8fc\uc804 \ud3ec\uc218 \uc790\ub9ac\ub97c \ucc28\uc9c0\ud558\uc600\ub2e4. 1998\ub144 5\uc6d4 26\uc77c\uc5d0 \uc5f4\ub9b0 \uc774\uc804 \uc18c\uc18d\ud300\uc778 \uc8fc\ub2c8\uce58\uc640\uc758 \uacbd\uae30\uc5d0\uc11c \uac00\uc640\uc9c0\ub9ac \ub370\uc4f0\ub85c\uc640 \ud568\uaed8 \ub450 \ubc88\uc9f8 \ub178\ud788\ud2b8 \ub178\ub7f0\uc744 \uae30\ub85d\ud588\uace0, 1999\ub144\uc5d0\ub294 \ucc98\uc74c\uc73c\ub85c 3\ud560 \ub300\uc758 \ud0c0\uc728\uc744 \uae30\ub85d\ud588\ub2e4. 2001\ub144\uc5d0 \uac10\ub3c5\uc9c1\uc744 \uc0ac\uc784\ud55c \ub178\ubb34\ub77c \uac00\uc4f0\uc57c\uc758 \ud6c4\uc784\uc73c\ub85c \uc8fc\ub2c8\uce58 \uc18c\uc18d \uc2dc\uc808\uc5d0 \uc790\uc2e0\uc744 \ud2b8\ub808\uc774\ub4dc\ub85c \ubc29\ucd9c\ud55c \ud638\uc2dc\ub178 \uc13c\uc774\uce58\uac00 \ud55c\uc2e0\uc758 \ucc28\uae30 \uac10\ub3c5\uc73c\ub85c \ucde8\uc784\ud558\uc790 \ub2e4\uc2dc \uc911\uc6a9\ub418\uc9c0 \uc54a\uace0 \ubc29\ucd9c\ub420\uae4c\ubd10 \ubd88\uc548\ud574 \ud558\uba70 \uc2e4\ub9dd\uac10\uc744 \ub4dc\ub7ec\ub0b4\uae30\ub3c4 \ud588\ub2e4. 2002\ub144\uc5d0\ub294 \ub450 \ucc28\ub840\uc758 \uace8\uc808\uc0c1\uc744 \uc785\ub294 \ub4f1 \ubd80\uc0c1\uc774 \uc7a6\uc544\uc9c0\uba74\uc11c \ud300\ub3c4 \uc57c\ub178\uc758 \uc804\ub825 \uc774\ud0c8\uacfc \ud568\uaed8 \ucd5c\ud558\uc704(4\uc704)\ub97c \uae30\ub85d\ud588\ub2e4.", "paragraph_answer": "\ud55c\uc2e0 \ud0c0\uc774\uac70\uc2a4\uc5d0\uc11c\ub294 \ub2f9\uc2dc \ud55c\uc2e0\uc758 \uac10\ub3c5\uc778 \uc694\uc2dc\ub2e4 \uc694\uc2dc\uc624\uc5d0\uac8c \ub192\uc740 \ud3c9\uac00\ub97c \ubc1b\uc73c\uba70, \uc57c\ub9c8\ub2e4 \uac00\uc4f0\ud788\ucf54\ub97c \ubc00\uc5b4\ub0b4\uace0 \uc8fc\uc804 \ud3ec\uc218 \uc790\ub9ac\ub97c \ucc28\uc9c0\ud558\uc600\ub2e4. 1998\ub144 5\uc6d4 26\uc77c\uc5d0 \uc5f4\ub9b0 \uc774\uc804 \uc18c\uc18d\ud300\uc778 \uc8fc\ub2c8\uce58\uc640\uc758 \uacbd\uae30\uc5d0\uc11c \uac00\uc640\uc9c0\ub9ac \ub370\uc4f0\ub85c\uc640 \ud568\uaed8 \ub450 \ubc88\uc9f8 \ub178\ud788\ud2b8 \ub178\ub7f0\uc744 \uae30\ub85d\ud588\uace0, 1999\ub144\uc5d0\ub294 \ucc98\uc74c\uc73c\ub85c 3\ud560 \ub300\uc758 \ud0c0\uc728\uc744 \uae30\ub85d\ud588\ub2e4. 2001\ub144\uc5d0 \uac10\ub3c5\uc9c1\uc744 \uc0ac\uc784\ud55c \ub178\ubb34\ub77c \uac00\uc4f0\uc57c\uc758 \ud6c4\uc784\uc73c\ub85c \uc8fc\ub2c8\uce58 \uc18c\uc18d \uc2dc\uc808\uc5d0 \uc790\uc2e0\uc744 \ud2b8\ub808\uc774\ub4dc\ub85c \ubc29\ucd9c\ud55c \ud638\uc2dc\ub178 \uc13c\uc774\uce58\uac00 \ud55c\uc2e0\uc758 \ucc28\uae30 \uac10\ub3c5\uc73c\ub85c \ucde8\uc784\ud558\uc790 \ub2e4\uc2dc \uc911\uc6a9\ub418\uc9c0 \uc54a\uace0 \ubc29\ucd9c\ub420\uae4c\ubd10 \ubd88\uc548\ud574 \ud558\uba70 \uc2e4\ub9dd\uac10\uc744 \ub4dc\ub7ec\ub0b4\uae30\ub3c4 \ud588\ub2e4. 2002\ub144\uc5d0\ub294 \ub450 \ucc28\ub840\uc758 \uace8\uc808\uc0c1\uc744 \uc785\ub294 \ub4f1 \ubd80\uc0c1\uc774 \uc7a6\uc544\uc9c0\uba74\uc11c \ud300\ub3c4 \uc57c\ub178\uc758 \uc804\ub825 \uc774\ud0c8\uacfc \ud568\uaed8 \ucd5c\ud558\uc704(4\uc704)\ub97c \uae30\ub85d\ud588\ub2e4.", "sentence_answer": "\ud55c\uc2e0 \ud0c0\uc774\uac70\uc2a4\uc5d0\uc11c\ub294 \ub2f9\uc2dc \ud55c\uc2e0\uc758 \uac10\ub3c5\uc778 \uc694\uc2dc\ub2e4 \uc694\uc2dc\uc624\uc5d0\uac8c \ub192\uc740 \ud3c9\uac00\ub97c \ubc1b\uc73c\uba70, \uc57c\ub9c8\ub2e4 \uac00\uc4f0\ud788\ucf54\ub97c \ubc00\uc5b4\ub0b4\uace0 \uc8fc\uc804 \ud3ec\uc218 \uc790\ub9ac\ub97c \ucc28\uc9c0\ud558\uc600\ub2e4.", "paragraph_id": "6522583-0-1", "paragraph_question": "question: \uc57c\ub178 \uc544\ud0a4\ud788\ub85c\uac00 \ud55c\uc2e0 \ud0c0\uc774\uac70\uc2a4\uc5d0\uc11c \ub9e1\uc740 \ud3ec\uc9c0\uc158\uc740?, context: \ud55c\uc2e0 \ud0c0\uc774\uac70\uc2a4\uc5d0\uc11c\ub294 \ub2f9\uc2dc \ud55c\uc2e0\uc758 \uac10\ub3c5\uc778 \uc694\uc2dc\ub2e4 \uc694\uc2dc\uc624\uc5d0\uac8c \ub192\uc740 \ud3c9\uac00\ub97c \ubc1b\uc73c\uba70, \uc57c\ub9c8\ub2e4 \uac00\uc4f0\ud788\ucf54\ub97c \ubc00\uc5b4\ub0b4\uace0 \uc8fc\uc804 \ud3ec\uc218 \uc790\ub9ac\ub97c \ucc28\uc9c0\ud558\uc600\ub2e4. 1998\ub144 5\uc6d4 26\uc77c\uc5d0 \uc5f4\ub9b0 \uc774\uc804 \uc18c\uc18d\ud300\uc778 \uc8fc\ub2c8\uce58\uc640\uc758 \uacbd\uae30\uc5d0\uc11c \uac00\uc640\uc9c0\ub9ac \ub370\uc4f0\ub85c\uc640 \ud568\uaed8 \ub450 \ubc88\uc9f8 \ub178\ud788\ud2b8 \ub178\ub7f0\uc744 \uae30\ub85d\ud588\uace0, 1999\ub144\uc5d0\ub294 \ucc98\uc74c\uc73c\ub85c 3\ud560 \ub300\uc758 \ud0c0\uc728\uc744 \uae30\ub85d\ud588\ub2e4. 2001\ub144\uc5d0 \uac10\ub3c5\uc9c1\uc744 \uc0ac\uc784\ud55c \ub178\ubb34\ub77c \uac00\uc4f0\uc57c\uc758 \ud6c4\uc784\uc73c\ub85c \uc8fc\ub2c8\uce58 \uc18c\uc18d \uc2dc\uc808\uc5d0 \uc790\uc2e0\uc744 \ud2b8\ub808\uc774\ub4dc\ub85c \ubc29\ucd9c\ud55c \ud638\uc2dc\ub178 \uc13c\uc774\uce58\uac00 \ud55c\uc2e0\uc758 \ucc28\uae30 \uac10\ub3c5\uc73c\ub85c \ucde8\uc784\ud558\uc790 \ub2e4\uc2dc \uc911\uc6a9\ub418\uc9c0 \uc54a\uace0 \ubc29\ucd9c\ub420\uae4c\ubd10 \ubd88\uc548\ud574 \ud558\uba70 \uc2e4\ub9dd\uac10\uc744 \ub4dc\ub7ec\ub0b4\uae30\ub3c4 \ud588\ub2e4. 2002\ub144\uc5d0\ub294 \ub450 \ucc28\ub840\uc758 \uace8\uc808\uc0c1\uc744 \uc785\ub294 \ub4f1 \ubd80\uc0c1\uc774 \uc7a6\uc544\uc9c0\uba74\uc11c \ud300\ub3c4 \uc57c\ub178\uc758 \uc804\ub825 \uc774\ud0c8\uacfc \ud568\uaed8 \ucd5c\ud558\uc704(4\uc704)\ub97c \uae30\ub85d\ud588\ub2e4."} {"question": "\uc694\ud55c 10\uc138\uac00 \uc5b4\ub5bb\uac8c \uc0ac\ub9dd\ud588\ub294\uc9c0\uc5d0 \ub300\ud55c \uc18c\ubb38\uc774 \uba87 \uac00\uc9c0\uc778\uac00\uc694?", "paragraph": "\uc694\ud55c 10\uc138\uc640 \ub9c8\ub85c\uce58\uc544\uc640 \uadf8\uc758 \ub0a8\ud3b8 \uae30 \uc0ac\uc774\uc758 \uad8c\ub825 \ud22c\uc7c1\uc740 928\ub144\uc5d0 \uc885\uc9c0\ubd80\ub97c \ucc0d\uc5c8\ub2e4. \uae30\ub294 \ube44\ubc00\ub9ac\uc5d0 \ubcd1\uc0ac\ub4e4\uc744 \ubaa8\uc544 \ub77c\ud14c\ub77c\ub178 \uad81\uc804\uc744 \uae09\uc2b5\ud558\uc5ec \ub2f9\uc2dc \uc18c\uc218\uc758 \uc815\uc608\ubcd1\ub825 \ubc16\uc5d0 \uc5c6\uc5c8\ub358 \uc2a4\ud3f4\ub808\ud1a0 \uacf5\uc791 \ud398\ud2b8\ub8e8\uc2a4\uc640 \ub9de\ubd99\uc5c8\ub2e4. \uae30\uc758 \ubcd1\uc0ac\ub4e4\uc5d0\uac8c \uc0ac\ub85c\uc7a1\ud78c \ud398\ud2b8\ub8e8\uc2a4\ub294 \uc790\uc2e0\uc758 \ud615 \uc694\ud55c 10\uc138\uac00 \ubcf4\ub294 \uc55e\uc5d0\uc11c \ubab8\uc774 \uc5ec\ub7ec \uc870\uac01\uc73c\ub85c \ub09c\ub3c4\uc9c8\ub2f9\ud574 \ucc98\ucc38\ud558\uac8c \uc8fd\uc5c8\uc73c\uba70, \uc694\ud55c 10\uc138\ub294 \uc9c0\ud558 \uac10\uc625\uc5d0 \ub358\uc838\uc838 \uadf8\uacf3\uc5d0\uc11c \ube44\ucc38\ud558\uac8c \uc0dd\uc744 \ub9c8\uac10\ud588\ub2e4. \uc694\ud55c 10\uc138\uac00 \uc5b4\ub5bb\uac8c \uc8fd\uc74c\uc744 \ub9de\uc774\ud588\ub294\uc9c0\uc5d0 \ub300\ud574\uc11c\ub294 \ub450 \uac00\uc9c0\uc758 \ub2e4\ub978 \uc774\uc57c\uae30\uac00 \uc804\ud574\uc838\uc624\uace0 \uc788\ub2e4. \uccab \ubc88\uc9f8\ub85c\ub294 \uadf8\uac00 \uad50\ud669\uc9c1\uc5d0\uc11c \ud3d0\uc704\ub2f9\ud55c \ud6c4 \uba87\ub2ec \ub9cc\uc5d0 \uc9c0\ud558 \uac10\uc625\uc5d0\uc11c \uc9c8\uc2dd\uc0ac\ub2f9\ud574 \uc0b4\ud574\ub2f9\ud588\ub2e4\ub294 \uc18c\ubb38\uc774\ub2e4. \ub610 \ub2e4\ub978 \uc774\uc57c\uae30\ub294 \uac10\uc625\uc5d0 \uac10\uae08\ub2f9\ud55c \ucc44 \uc6b0\uc6b8\uc99d\uc5d0 \uc2dc\ub2ec\ub9ac\ub2e4\uac00 929\ub144 \uc5b4\ub290 \ub0a0 \uae09\uc0ac\ud588\ub2e4\ub294 \uac83\uc774\ub2e4.", "answer": "\ub450 \uac00\uc9c0", "sentence": "\uc694\ud55c 10\uc138\uac00 \uc5b4\ub5bb\uac8c \uc8fd\uc74c\uc744 \ub9de\uc774\ud588\ub294\uc9c0\uc5d0 \ub300\ud574\uc11c\ub294 \ub450 \uac00\uc9c0 \uc758 \ub2e4\ub978 \uc774\uc57c\uae30\uac00 \uc804\ud574\uc838\uc624\uace0 \uc788\ub2e4.", "paragraph_sentence": "\uc694\ud55c 10\uc138\uc640 \ub9c8\ub85c\uce58\uc544\uc640 \uadf8\uc758 \ub0a8\ud3b8 \uae30 \uc0ac\uc774\uc758 \uad8c\ub825 \ud22c\uc7c1\uc740 928\ub144\uc5d0 \uc885\uc9c0\ubd80\ub97c \ucc0d\uc5c8\ub2e4. \uae30\ub294 \ube44\ubc00\ub9ac\uc5d0 \ubcd1\uc0ac\ub4e4\uc744 \ubaa8\uc544 \ub77c\ud14c\ub77c\ub178 \uad81\uc804\uc744 \uae09\uc2b5\ud558\uc5ec \ub2f9\uc2dc \uc18c\uc218\uc758 \uc815\uc608\ubcd1\ub825 \ubc16\uc5d0 \uc5c6\uc5c8\ub358 \uc2a4\ud3f4\ub808\ud1a0 \uacf5\uc791 \ud398\ud2b8\ub8e8\uc2a4\uc640 \ub9de\ubd99\uc5c8\ub2e4. \uae30\uc758 \ubcd1\uc0ac\ub4e4\uc5d0\uac8c \uc0ac\ub85c\uc7a1\ud78c \ud398\ud2b8\ub8e8\uc2a4\ub294 \uc790\uc2e0\uc758 \ud615 \uc694\ud55c 10\uc138\uac00 \ubcf4\ub294 \uc55e\uc5d0\uc11c \ubab8\uc774 \uc5ec\ub7ec \uc870\uac01\uc73c\ub85c \ub09c\ub3c4\uc9c8\ub2f9\ud574 \ucc98\ucc38\ud558\uac8c \uc8fd\uc5c8\uc73c\uba70, \uc694\ud55c 10\uc138\ub294 \uc9c0\ud558 \uac10\uc625\uc5d0 \ub358\uc838\uc838 \uadf8\uacf3\uc5d0\uc11c \ube44\ucc38\ud558\uac8c \uc0dd\uc744 \ub9c8\uac10\ud588\ub2e4. \uc694\ud55c 10\uc138\uac00 \uc5b4\ub5bb\uac8c \uc8fd\uc74c\uc744 \ub9de\uc774\ud588\ub294\uc9c0\uc5d0 \ub300\ud574\uc11c\ub294 \ub450 \uac00\uc9c0 \uc758 \ub2e4\ub978 \uc774\uc57c\uae30\uac00 \uc804\ud574\uc838\uc624\uace0 \uc788\ub2e4. \uccab \ubc88\uc9f8\ub85c\ub294 \uadf8\uac00 \uad50\ud669\uc9c1\uc5d0\uc11c \ud3d0\uc704\ub2f9\ud55c \ud6c4 \uba87\ub2ec \ub9cc\uc5d0 \uc9c0\ud558 \uac10\uc625\uc5d0\uc11c \uc9c8\uc2dd\uc0ac\ub2f9\ud574 \uc0b4\ud574\ub2f9\ud588\ub2e4\ub294 \uc18c\ubb38\uc774\ub2e4. \ub610 \ub2e4\ub978 \uc774\uc57c\uae30\ub294 \uac10\uc625\uc5d0 \uac10\uae08\ub2f9\ud55c \ucc44 \uc6b0\uc6b8\uc99d\uc5d0 \uc2dc\ub2ec\ub9ac\ub2e4\uac00 929\ub144 \uc5b4\ub290 \ub0a0 \uae09\uc0ac\ud588\ub2e4\ub294 \uac83\uc774\ub2e4.", "paragraph_answer": "\uc694\ud55c 10\uc138\uc640 \ub9c8\ub85c\uce58\uc544\uc640 \uadf8\uc758 \ub0a8\ud3b8 \uae30 \uc0ac\uc774\uc758 \uad8c\ub825 \ud22c\uc7c1\uc740 928\ub144\uc5d0 \uc885\uc9c0\ubd80\ub97c \ucc0d\uc5c8\ub2e4. \uae30\ub294 \ube44\ubc00\ub9ac\uc5d0 \ubcd1\uc0ac\ub4e4\uc744 \ubaa8\uc544 \ub77c\ud14c\ub77c\ub178 \uad81\uc804\uc744 \uae09\uc2b5\ud558\uc5ec \ub2f9\uc2dc \uc18c\uc218\uc758 \uc815\uc608\ubcd1\ub825 \ubc16\uc5d0 \uc5c6\uc5c8\ub358 \uc2a4\ud3f4\ub808\ud1a0 \uacf5\uc791 \ud398\ud2b8\ub8e8\uc2a4\uc640 \ub9de\ubd99\uc5c8\ub2e4. \uae30\uc758 \ubcd1\uc0ac\ub4e4\uc5d0\uac8c \uc0ac\ub85c\uc7a1\ud78c \ud398\ud2b8\ub8e8\uc2a4\ub294 \uc790\uc2e0\uc758 \ud615 \uc694\ud55c 10\uc138\uac00 \ubcf4\ub294 \uc55e\uc5d0\uc11c \ubab8\uc774 \uc5ec\ub7ec \uc870\uac01\uc73c\ub85c \ub09c\ub3c4\uc9c8\ub2f9\ud574 \ucc98\ucc38\ud558\uac8c \uc8fd\uc5c8\uc73c\uba70, \uc694\ud55c 10\uc138\ub294 \uc9c0\ud558 \uac10\uc625\uc5d0 \ub358\uc838\uc838 \uadf8\uacf3\uc5d0\uc11c \ube44\ucc38\ud558\uac8c \uc0dd\uc744 \ub9c8\uac10\ud588\ub2e4. \uc694\ud55c 10\uc138\uac00 \uc5b4\ub5bb\uac8c \uc8fd\uc74c\uc744 \ub9de\uc774\ud588\ub294\uc9c0\uc5d0 \ub300\ud574\uc11c\ub294 \ub450 \uac00\uc9c0 \uc758 \ub2e4\ub978 \uc774\uc57c\uae30\uac00 \uc804\ud574\uc838\uc624\uace0 \uc788\ub2e4. \uccab \ubc88\uc9f8\ub85c\ub294 \uadf8\uac00 \uad50\ud669\uc9c1\uc5d0\uc11c \ud3d0\uc704\ub2f9\ud55c \ud6c4 \uba87\ub2ec \ub9cc\uc5d0 \uc9c0\ud558 \uac10\uc625\uc5d0\uc11c \uc9c8\uc2dd\uc0ac\ub2f9\ud574 \uc0b4\ud574\ub2f9\ud588\ub2e4\ub294 \uc18c\ubb38\uc774\ub2e4. \ub610 \ub2e4\ub978 \uc774\uc57c\uae30\ub294 \uac10\uc625\uc5d0 \uac10\uae08\ub2f9\ud55c \ucc44 \uc6b0\uc6b8\uc99d\uc5d0 \uc2dc\ub2ec\ub9ac\ub2e4\uac00 929\ub144 \uc5b4\ub290 \ub0a0 \uae09\uc0ac\ud588\ub2e4\ub294 \uac83\uc774\ub2e4.", "sentence_answer": "\uc694\ud55c 10\uc138\uac00 \uc5b4\ub5bb\uac8c \uc8fd\uc74c\uc744 \ub9de\uc774\ud588\ub294\uc9c0\uc5d0 \ub300\ud574\uc11c\ub294 \ub450 \uac00\uc9c0 \uc758 \ub2e4\ub978 \uc774\uc57c\uae30\uac00 \uc804\ud574\uc838\uc624\uace0 \uc788\ub2e4.", "paragraph_id": "6476387-8-2", "paragraph_question": "question: \uc694\ud55c 10\uc138\uac00 \uc5b4\ub5bb\uac8c \uc0ac\ub9dd\ud588\ub294\uc9c0\uc5d0 \ub300\ud55c \uc18c\ubb38\uc774 \uba87 \uac00\uc9c0\uc778\uac00\uc694?, context: \uc694\ud55c 10\uc138\uc640 \ub9c8\ub85c\uce58\uc544\uc640 \uadf8\uc758 \ub0a8\ud3b8 \uae30 \uc0ac\uc774\uc758 \uad8c\ub825 \ud22c\uc7c1\uc740 928\ub144\uc5d0 \uc885\uc9c0\ubd80\ub97c \ucc0d\uc5c8\ub2e4. \uae30\ub294 \ube44\ubc00\ub9ac\uc5d0 \ubcd1\uc0ac\ub4e4\uc744 \ubaa8\uc544 \ub77c\ud14c\ub77c\ub178 \uad81\uc804\uc744 \uae09\uc2b5\ud558\uc5ec \ub2f9\uc2dc \uc18c\uc218\uc758 \uc815\uc608\ubcd1\ub825 \ubc16\uc5d0 \uc5c6\uc5c8\ub358 \uc2a4\ud3f4\ub808\ud1a0 \uacf5\uc791 \ud398\ud2b8\ub8e8\uc2a4\uc640 \ub9de\ubd99\uc5c8\ub2e4. \uae30\uc758 \ubcd1\uc0ac\ub4e4\uc5d0\uac8c \uc0ac\ub85c\uc7a1\ud78c \ud398\ud2b8\ub8e8\uc2a4\ub294 \uc790\uc2e0\uc758 \ud615 \uc694\ud55c 10\uc138\uac00 \ubcf4\ub294 \uc55e\uc5d0\uc11c \ubab8\uc774 \uc5ec\ub7ec \uc870\uac01\uc73c\ub85c \ub09c\ub3c4\uc9c8\ub2f9\ud574 \ucc98\ucc38\ud558\uac8c \uc8fd\uc5c8\uc73c\uba70, \uc694\ud55c 10\uc138\ub294 \uc9c0\ud558 \uac10\uc625\uc5d0 \ub358\uc838\uc838 \uadf8\uacf3\uc5d0\uc11c \ube44\ucc38\ud558\uac8c \uc0dd\uc744 \ub9c8\uac10\ud588\ub2e4. \uc694\ud55c 10\uc138\uac00 \uc5b4\ub5bb\uac8c \uc8fd\uc74c\uc744 \ub9de\uc774\ud588\ub294\uc9c0\uc5d0 \ub300\ud574\uc11c\ub294 \ub450 \uac00\uc9c0\uc758 \ub2e4\ub978 \uc774\uc57c\uae30\uac00 \uc804\ud574\uc838\uc624\uace0 \uc788\ub2e4. \uccab \ubc88\uc9f8\ub85c\ub294 \uadf8\uac00 \uad50\ud669\uc9c1\uc5d0\uc11c \ud3d0\uc704\ub2f9\ud55c \ud6c4 \uba87\ub2ec \ub9cc\uc5d0 \uc9c0\ud558 \uac10\uc625\uc5d0\uc11c \uc9c8\uc2dd\uc0ac\ub2f9\ud574 \uc0b4\ud574\ub2f9\ud588\ub2e4\ub294 \uc18c\ubb38\uc774\ub2e4. \ub610 \ub2e4\ub978 \uc774\uc57c\uae30\ub294 \uac10\uc625\uc5d0 \uac10\uae08\ub2f9\ud55c \ucc44 \uc6b0\uc6b8\uc99d\uc5d0 \uc2dc\ub2ec\ub9ac\ub2e4\uac00 929\ub144 \uc5b4\ub290 \ub0a0 \uae09\uc0ac\ud588\ub2e4\ub294 \uac83\uc774\ub2e4."} {"question": "\ucd5c\uc218\uc885\uc758 \ud559\ub825 \uc704\uc870 \uc758\ud639 \ub17c\ub780\uc758 \uc6d0\uc778\uc774 \ub418\uc5c8\ub358 \ub300\ud559\uad50\ub294?", "paragraph": "2007\ub144 8\uc6d4\uc5d0 \ub4dc\ub77c\ub9c8 \u300a\ub300\uc870\uc601\u300b\uc744 \ucd2c\uc601\uc911\uc774\ub358 \ucd5c\uc218\uc885\uc740 \ub54c \uc544\ub2cc \ud559\ub825 \uc704\uc870 \uc758\ud639 \ub17c\ub780\uc774 \ub4a4\ub2a6\uac8c \ubc1d\ud600\uc9c0\uba74\uc11c \ub124\ud2f0\uc98c\ub4e4\uc744 \ubd84\ub178\ud558\uac8c \ud588\ub2e4. 1981\ub144 \ud55c\uad6d\uc678\ub300 \ubb34\uc5ed\ud559\uacfc\ub97c \ud569\uaca9\ud558\uc5ec \uc774\ud6c4 \uc878\uc5c5\ud55c \uac83\uc73c\ub85c \uc54c\ub824\uc84c\uc9c0\ub9cc \ud559\uad50 \uce21\uc5d0 \ud655\uc778 \uacb0\uacfc \uc0ac\uc2e4\uc774 \uc544\ub2cc \uac83\uc73c\ub85c \ubc1d\ud600\uc84c\ub2e4. \uc0ac\ud0dc\uac00 \uc2ec\uac01\ud574\uc9c0\uc790 2007\ub144 8\uc6d4 22\uc77c\uc744 \uae30\ud558\uc5ec \uc18c\uc18d\uc0ac \u2018\uc18c\ud504\ud2b8\ub79c\ub4dc\u2019\ub294 \ud574\uba85\ud55c\uc989 \u201c\ucd5c\uc218\uc885\uc740 \uc11c\uc6b8 \ubc30\uc7ac\uc911\ud559\uad50\uc640 \uc11c\uc6b8 \ubc30\uba85\uace0\ub4f1\ud559\uad50\ub97c \uc878\uc5c5\ud55c \ub4a4 \ud55c\uad6d\uc678\ub300 \ubb34\uc5ed\ud559\uacfc\uc5d0 \uc9c0\uc6d0\ud558\uc5ec \ud569\uaca9\uc744 \ud558\uc600\uc9c0\ub9cc, \uc9d1\uc548 \uc0ac\uc815\uc73c\ub85c \uc778\ud574 \ub4f1\ub85d\ud558\uc9c0 \ubabb\ud558\uc600\ub2e4\u201d\ub294 \ud574\uba85\uacfc \uc544\uc6b8\ub7ec, \u201c\ucd5c\uc218\uc885\uc740 \uc9c0\uae08\uae4c\uc9c0 \ud559\ub825 \ud504\ub9ac\ubbf8\uc5c4\uc744 \uc5bb\uc5b4 \ubcf8 \uc801\ub3c4 \uc5c6\uc73c\uba70, \uc678\ub300\ub97c \uc878\uc5c5\ud558\uc600\ub2e4\uace0 \uc9c1\uc811 \uc5b8\uae09\uc744 \ud55c \uc801\ub3c4 \uc5c6\ub2e4\u201d\uace0 \ud574\uba85\ud588\ub2e4. \uc774\uc5b4 \uc18c\uc18d\uc0ac \uce21\uc5d0\uc11c\ub294 \ud574\uba85\ud55c\uc989 \u201c\ucd5c\uc218\uc885\uc740 \uc678\ub300 \ub4f1\ub85d\uc744 \ud558\uc9c0 \ubabb\ud558\uace0 \ubbf8\uad6d\uc73c\ub85c \uac74\ub108\uac00 \ucf5c\ub85c\ub77c\ub3c4\uc5d0 \uc704\uce58\ud55c \ud3ec\ud2b8\ubaa8\uac74 \uceec\ub9ac\uc9c0 \uacbd\uc601\ud559\uacfc \ube44\ud559\uc704\uc18d\uc131 \uc218\ub8cc \uacfc\uc815\uc744 1\ub144 \uc815\ub3c4 \ub2e4\ub2cc \uac83\uc774 \uc804\ubd80\uc774\uba70, \uadf8\ub9c8\uc800 \ubd80\uce5c\uc0c1\uc744 \ub2f9\ud558\uc5ec \uadc0\uad6d\ud558\uac8c \ub410\ub2e4\u201d\uba70 \ub2f9\uc2dc \uc0c1\ud669\uc744 \ud574\uba85\ud558\uc600\uace0, \u201c\ucd5c\uc218\uc885\uc758 \ud559\ub825\uc744 \uc81c\ub300\ub85c \ud655\uc778\ud558\uc9c0 \uc54a\uace0 \uac01 \ud3ec\ud138 \uc0ac\uc774\ud2b8\uc5d0 \uc77c\ubc29\uc801 \uc624\ub4f1\ub85d \uae30\uc7ac\ub41c \uac83\uc5d0 \ub300\uc751\ud558\uc9c0 \uc54a\uc558\ub358 \uac83\uc774 \uc2e4\uc218\u201d\ub77c\uace0 \ud574\uba85 \uc5b8\uae09\ud558\uba70 \"\uac01 \ud3ec\ud138 \uc0ac\uc774\ud2b8\uc5d0 \uae30\uc7ac\ub41c \uac83\uc740 \uc624\ub4f1\ub85d\uc774\uc790 \uc2e4\uc218\u201d\ub77c\uace0 \uc8fc\uc7a5\ud558\uc600\ub2e4.", "answer": "\ud55c\uad6d\uc678\ub300", "sentence": "1981\ub144 \ud55c\uad6d\uc678\ub300 \ubb34\uc5ed\ud559\uacfc\ub97c \ud569\uaca9\ud558\uc5ec \uc774\ud6c4 \uc878\uc5c5\ud55c \uac83\uc73c\ub85c \uc54c\ub824\uc84c\uc9c0\ub9cc \ud559\uad50 \uce21\uc5d0 \ud655\uc778 \uacb0\uacfc \uc0ac\uc2e4\uc774 \uc544\ub2cc \uac83\uc73c\ub85c \ubc1d\ud600\uc84c\ub2e4.", "paragraph_sentence": "2007\ub144 8\uc6d4\uc5d0 \ub4dc\ub77c\ub9c8 \u300a\ub300\uc870\uc601\u300b\uc744 \ucd2c\uc601\uc911\uc774\ub358 \ucd5c\uc218\uc885\uc740 \ub54c \uc544\ub2cc \ud559\ub825 \uc704\uc870 \uc758\ud639 \ub17c\ub780\uc774 \ub4a4\ub2a6\uac8c \ubc1d\ud600\uc9c0\uba74\uc11c \ub124\ud2f0\uc98c\ub4e4\uc744 \ubd84\ub178\ud558\uac8c \ud588\ub2e4. 1981\ub144 \ud55c\uad6d\uc678\ub300 \ubb34\uc5ed\ud559\uacfc\ub97c \ud569\uaca9\ud558\uc5ec \uc774\ud6c4 \uc878\uc5c5\ud55c \uac83\uc73c\ub85c \uc54c\ub824\uc84c\uc9c0\ub9cc \ud559\uad50 \uce21\uc5d0 \ud655\uc778 \uacb0\uacfc \uc0ac\uc2e4\uc774 \uc544\ub2cc \uac83\uc73c\ub85c \ubc1d\ud600\uc84c\ub2e4. \uc0ac\ud0dc\uac00 \uc2ec\uac01\ud574\uc9c0\uc790 2007\ub144 8\uc6d4 22\uc77c\uc744 \uae30\ud558\uc5ec \uc18c\uc18d\uc0ac \u2018\uc18c\ud504\ud2b8\ub79c\ub4dc\u2019\ub294 \ud574\uba85\ud55c\uc989 \u201c\ucd5c\uc218\uc885\uc740 \uc11c\uc6b8 \ubc30\uc7ac\uc911\ud559\uad50\uc640 \uc11c\uc6b8 \ubc30\uba85\uace0\ub4f1\ud559\uad50\ub97c \uc878\uc5c5\ud55c \ub4a4 \ud55c\uad6d\uc678\ub300 \ubb34\uc5ed\ud559\uacfc\uc5d0 \uc9c0\uc6d0\ud558\uc5ec \ud569\uaca9\uc744 \ud558\uc600\uc9c0\ub9cc, \uc9d1\uc548 \uc0ac\uc815\uc73c\ub85c \uc778\ud574 \ub4f1\ub85d\ud558\uc9c0 \ubabb\ud558\uc600\ub2e4\u201d\ub294 \ud574\uba85\uacfc \uc544\uc6b8\ub7ec, \u201c\ucd5c\uc218\uc885\uc740 \uc9c0\uae08\uae4c\uc9c0 \ud559\ub825 \ud504\ub9ac\ubbf8\uc5c4\uc744 \uc5bb\uc5b4 \ubcf8 \uc801\ub3c4 \uc5c6\uc73c\uba70, \uc678\ub300\ub97c \uc878\uc5c5\ud558\uc600\ub2e4\uace0 \uc9c1\uc811 \uc5b8\uae09\uc744 \ud55c \uc801\ub3c4 \uc5c6\ub2e4\u201d\uace0 \ud574\uba85\ud588\ub2e4. \uc774\uc5b4 \uc18c\uc18d\uc0ac \uce21\uc5d0\uc11c\ub294 \ud574\uba85\ud55c\uc989 \u201c\ucd5c\uc218\uc885\uc740 \uc678\ub300 \ub4f1\ub85d\uc744 \ud558\uc9c0 \ubabb\ud558\uace0 \ubbf8\uad6d\uc73c\ub85c \uac74\ub108\uac00 \ucf5c\ub85c\ub77c\ub3c4\uc5d0 \uc704\uce58\ud55c \ud3ec\ud2b8\ubaa8\uac74 \uceec\ub9ac\uc9c0 \uacbd\uc601\ud559\uacfc \ube44\ud559\uc704\uc18d\uc131 \uc218\ub8cc \uacfc\uc815\uc744 1\ub144 \uc815\ub3c4 \ub2e4\ub2cc \uac83\uc774 \uc804\ubd80\uc774\uba70, \uadf8\ub9c8\uc800 \ubd80\uce5c\uc0c1\uc744 \ub2f9\ud558\uc5ec \uadc0\uad6d\ud558\uac8c \ub410\ub2e4\u201d\uba70 \ub2f9\uc2dc \uc0c1\ud669\uc744 \ud574\uba85\ud558\uc600\uace0, \u201c\ucd5c\uc218\uc885\uc758 \ud559\ub825\uc744 \uc81c\ub300\ub85c \ud655\uc778\ud558\uc9c0 \uc54a\uace0 \uac01 \ud3ec\ud138 \uc0ac\uc774\ud2b8\uc5d0 \uc77c\ubc29\uc801 \uc624\ub4f1\ub85d \uae30\uc7ac\ub41c \uac83\uc5d0 \ub300\uc751\ud558\uc9c0 \uc54a\uc558\ub358 \uac83\uc774 \uc2e4\uc218\u201d\ub77c\uace0 \ud574\uba85 \uc5b8\uae09\ud558\uba70 \"\uac01 \ud3ec\ud138 \uc0ac\uc774\ud2b8\uc5d0 \uae30\uc7ac\ub41c \uac83\uc740 \uc624\ub4f1\ub85d\uc774\uc790 \uc2e4\uc218\u201d\ub77c\uace0 \uc8fc\uc7a5\ud558\uc600\ub2e4.", "paragraph_answer": "2007\ub144 8\uc6d4\uc5d0 \ub4dc\ub77c\ub9c8 \u300a\ub300\uc870\uc601\u300b\uc744 \ucd2c\uc601\uc911\uc774\ub358 \ucd5c\uc218\uc885\uc740 \ub54c \uc544\ub2cc \ud559\ub825 \uc704\uc870 \uc758\ud639 \ub17c\ub780\uc774 \ub4a4\ub2a6\uac8c \ubc1d\ud600\uc9c0\uba74\uc11c \ub124\ud2f0\uc98c\ub4e4\uc744 \ubd84\ub178\ud558\uac8c \ud588\ub2e4. 1981\ub144 \ud55c\uad6d\uc678\ub300 \ubb34\uc5ed\ud559\uacfc\ub97c \ud569\uaca9\ud558\uc5ec \uc774\ud6c4 \uc878\uc5c5\ud55c \uac83\uc73c\ub85c \uc54c\ub824\uc84c\uc9c0\ub9cc \ud559\uad50 \uce21\uc5d0 \ud655\uc778 \uacb0\uacfc \uc0ac\uc2e4\uc774 \uc544\ub2cc \uac83\uc73c\ub85c \ubc1d\ud600\uc84c\ub2e4. \uc0ac\ud0dc\uac00 \uc2ec\uac01\ud574\uc9c0\uc790 2007\ub144 8\uc6d4 22\uc77c\uc744 \uae30\ud558\uc5ec \uc18c\uc18d\uc0ac \u2018\uc18c\ud504\ud2b8\ub79c\ub4dc\u2019\ub294 \ud574\uba85\ud55c\uc989 \u201c\ucd5c\uc218\uc885\uc740 \uc11c\uc6b8 \ubc30\uc7ac\uc911\ud559\uad50\uc640 \uc11c\uc6b8 \ubc30\uba85\uace0\ub4f1\ud559\uad50\ub97c \uc878\uc5c5\ud55c \ub4a4 \ud55c\uad6d\uc678\ub300 \ubb34\uc5ed\ud559\uacfc\uc5d0 \uc9c0\uc6d0\ud558\uc5ec \ud569\uaca9\uc744 \ud558\uc600\uc9c0\ub9cc, \uc9d1\uc548 \uc0ac\uc815\uc73c\ub85c \uc778\ud574 \ub4f1\ub85d\ud558\uc9c0 \ubabb\ud558\uc600\ub2e4\u201d\ub294 \ud574\uba85\uacfc \uc544\uc6b8\ub7ec, \u201c\ucd5c\uc218\uc885\uc740 \uc9c0\uae08\uae4c\uc9c0 \ud559\ub825 \ud504\ub9ac\ubbf8\uc5c4\uc744 \uc5bb\uc5b4 \ubcf8 \uc801\ub3c4 \uc5c6\uc73c\uba70, \uc678\ub300\ub97c \uc878\uc5c5\ud558\uc600\ub2e4\uace0 \uc9c1\uc811 \uc5b8\uae09\uc744 \ud55c \uc801\ub3c4 \uc5c6\ub2e4\u201d\uace0 \ud574\uba85\ud588\ub2e4. \uc774\uc5b4 \uc18c\uc18d\uc0ac \uce21\uc5d0\uc11c\ub294 \ud574\uba85\ud55c\uc989 \u201c\ucd5c\uc218\uc885\uc740 \uc678\ub300 \ub4f1\ub85d\uc744 \ud558\uc9c0 \ubabb\ud558\uace0 \ubbf8\uad6d\uc73c\ub85c \uac74\ub108\uac00 \ucf5c\ub85c\ub77c\ub3c4\uc5d0 \uc704\uce58\ud55c \ud3ec\ud2b8\ubaa8\uac74 \uceec\ub9ac\uc9c0 \uacbd\uc601\ud559\uacfc \ube44\ud559\uc704\uc18d\uc131 \uc218\ub8cc \uacfc\uc815\uc744 1\ub144 \uc815\ub3c4 \ub2e4\ub2cc \uac83\uc774 \uc804\ubd80\uc774\uba70, \uadf8\ub9c8\uc800 \ubd80\uce5c\uc0c1\uc744 \ub2f9\ud558\uc5ec \uadc0\uad6d\ud558\uac8c \ub410\ub2e4\u201d\uba70 \ub2f9\uc2dc \uc0c1\ud669\uc744 \ud574\uba85\ud558\uc600\uace0, \u201c\ucd5c\uc218\uc885\uc758 \ud559\ub825\uc744 \uc81c\ub300\ub85c \ud655\uc778\ud558\uc9c0 \uc54a\uace0 \uac01 \ud3ec\ud138 \uc0ac\uc774\ud2b8\uc5d0 \uc77c\ubc29\uc801 \uc624\ub4f1\ub85d \uae30\uc7ac\ub41c \uac83\uc5d0 \ub300\uc751\ud558\uc9c0 \uc54a\uc558\ub358 \uac83\uc774 \uc2e4\uc218\u201d\ub77c\uace0 \ud574\uba85 \uc5b8\uae09\ud558\uba70 \"\uac01 \ud3ec\ud138 \uc0ac\uc774\ud2b8\uc5d0 \uae30\uc7ac\ub41c \uac83\uc740 \uc624\ub4f1\ub85d\uc774\uc790 \uc2e4\uc218\u201d\ub77c\uace0 \uc8fc\uc7a5\ud558\uc600\ub2e4.", "sentence_answer": "1981\ub144 \ud55c\uad6d\uc678\ub300 \ubb34\uc5ed\ud559\uacfc\ub97c \ud569\uaca9\ud558\uc5ec \uc774\ud6c4 \uc878\uc5c5\ud55c \uac83\uc73c\ub85c \uc54c\ub824\uc84c\uc9c0\ub9cc \ud559\uad50 \uce21\uc5d0 \ud655\uc778 \uacb0\uacfc \uc0ac\uc2e4\uc774 \uc544\ub2cc \uac83\uc73c\ub85c \ubc1d\ud600\uc84c\ub2e4.", "paragraph_id": "6560132-0-0", "paragraph_question": "question: \ucd5c\uc218\uc885\uc758 \ud559\ub825 \uc704\uc870 \uc758\ud639 \ub17c\ub780\uc758 \uc6d0\uc778\uc774 \ub418\uc5c8\ub358 \ub300\ud559\uad50\ub294?, context: 2007\ub144 8\uc6d4\uc5d0 \ub4dc\ub77c\ub9c8 \u300a\ub300\uc870\uc601\u300b\uc744 \ucd2c\uc601\uc911\uc774\ub358 \ucd5c\uc218\uc885\uc740 \ub54c \uc544\ub2cc \ud559\ub825 \uc704\uc870 \uc758\ud639 \ub17c\ub780\uc774 \ub4a4\ub2a6\uac8c \ubc1d\ud600\uc9c0\uba74\uc11c \ub124\ud2f0\uc98c\ub4e4\uc744 \ubd84\ub178\ud558\uac8c \ud588\ub2e4. 1981\ub144 \ud55c\uad6d\uc678\ub300 \ubb34\uc5ed\ud559\uacfc\ub97c \ud569\uaca9\ud558\uc5ec \uc774\ud6c4 \uc878\uc5c5\ud55c \uac83\uc73c\ub85c \uc54c\ub824\uc84c\uc9c0\ub9cc \ud559\uad50 \uce21\uc5d0 \ud655\uc778 \uacb0\uacfc \uc0ac\uc2e4\uc774 \uc544\ub2cc \uac83\uc73c\ub85c \ubc1d\ud600\uc84c\ub2e4. \uc0ac\ud0dc\uac00 \uc2ec\uac01\ud574\uc9c0\uc790 2007\ub144 8\uc6d4 22\uc77c\uc744 \uae30\ud558\uc5ec \uc18c\uc18d\uc0ac \u2018\uc18c\ud504\ud2b8\ub79c\ub4dc\u2019\ub294 \ud574\uba85\ud55c\uc989 \u201c\ucd5c\uc218\uc885\uc740 \uc11c\uc6b8 \ubc30\uc7ac\uc911\ud559\uad50\uc640 \uc11c\uc6b8 \ubc30\uba85\uace0\ub4f1\ud559\uad50\ub97c \uc878\uc5c5\ud55c \ub4a4 \ud55c\uad6d\uc678\ub300 \ubb34\uc5ed\ud559\uacfc\uc5d0 \uc9c0\uc6d0\ud558\uc5ec \ud569\uaca9\uc744 \ud558\uc600\uc9c0\ub9cc, \uc9d1\uc548 \uc0ac\uc815\uc73c\ub85c \uc778\ud574 \ub4f1\ub85d\ud558\uc9c0 \ubabb\ud558\uc600\ub2e4\u201d\ub294 \ud574\uba85\uacfc \uc544\uc6b8\ub7ec, \u201c\ucd5c\uc218\uc885\uc740 \uc9c0\uae08\uae4c\uc9c0 \ud559\ub825 \ud504\ub9ac\ubbf8\uc5c4\uc744 \uc5bb\uc5b4 \ubcf8 \uc801\ub3c4 \uc5c6\uc73c\uba70, \uc678\ub300\ub97c \uc878\uc5c5\ud558\uc600\ub2e4\uace0 \uc9c1\uc811 \uc5b8\uae09\uc744 \ud55c \uc801\ub3c4 \uc5c6\ub2e4\u201d\uace0 \ud574\uba85\ud588\ub2e4. \uc774\uc5b4 \uc18c\uc18d\uc0ac \uce21\uc5d0\uc11c\ub294 \ud574\uba85\ud55c\uc989 \u201c\ucd5c\uc218\uc885\uc740 \uc678\ub300 \ub4f1\ub85d\uc744 \ud558\uc9c0 \ubabb\ud558\uace0 \ubbf8\uad6d\uc73c\ub85c \uac74\ub108\uac00 \ucf5c\ub85c\ub77c\ub3c4\uc5d0 \uc704\uce58\ud55c \ud3ec\ud2b8\ubaa8\uac74 \uceec\ub9ac\uc9c0 \uacbd\uc601\ud559\uacfc \ube44\ud559\uc704\uc18d\uc131 \uc218\ub8cc \uacfc\uc815\uc744 1\ub144 \uc815\ub3c4 \ub2e4\ub2cc \uac83\uc774 \uc804\ubd80\uc774\uba70, \uadf8\ub9c8\uc800 \ubd80\uce5c\uc0c1\uc744 \ub2f9\ud558\uc5ec \uadc0\uad6d\ud558\uac8c \ub410\ub2e4\u201d\uba70 \ub2f9\uc2dc \uc0c1\ud669\uc744 \ud574\uba85\ud558\uc600\uace0, \u201c\ucd5c\uc218\uc885\uc758 \ud559\ub825\uc744 \uc81c\ub300\ub85c \ud655\uc778\ud558\uc9c0 \uc54a\uace0 \uac01 \ud3ec\ud138 \uc0ac\uc774\ud2b8\uc5d0 \uc77c\ubc29\uc801 \uc624\ub4f1\ub85d \uae30\uc7ac\ub41c \uac83\uc5d0 \ub300\uc751\ud558\uc9c0 \uc54a\uc558\ub358 \uac83\uc774 \uc2e4\uc218\u201d\ub77c\uace0 \ud574\uba85 \uc5b8\uae09\ud558\uba70 \"\uac01 \ud3ec\ud138 \uc0ac\uc774\ud2b8\uc5d0 \uae30\uc7ac\ub41c \uac83\uc740 \uc624\ub4f1\ub85d\uc774\uc790 \uc2e4\uc218\u201d\ub77c\uace0 \uc8fc\uc7a5\ud558\uc600\ub2e4."} {"question": "\ud398\ub2c8\uc290\ub77c \ub77c\ucf13 \ud074\ub7fd\uc5d0\uc11c \uc0d8\ud504\ub7ec\uc2a4\uc758 \uc7ac\ub2a5\uc744 \uc54c\uc544\ubcf8 \uc0ac\ub78c\uc758 \uc774\ub984\uc740?", "paragraph": "\uc0d8\ud504\ub7ec\uc2a4\ub294 \uc77c\ucc0d\uc774 \uc5b4\ub9b4 \ub54c\ubd80\ud130 \ub6f0\uc5b4\ub09c \uc6b4\ub3d9\uc120\uc218\uac00 \ub420 \uc790\uc9c8\uc744 \ubcf4\uc600\ub2e4. \uadf8\ub294 \uc5b4\ub9b0 \uc2dc\uc808 \uc9d1\uc5d0 \uc788\ub294 \uc9c0\ud558\uc2e4\uc5d0\uc11c \ub77c\ucf13\uc744 \ubc1c\uacac\ud558\uace0\ub294 \uba87 \uc2dc\uac04\ub3d9\uc548\uc774\uace0 \ubcbd\uc5d0 \uacf5\uc744 \uce58\uba70 \ub180\uc558\ub2e4. 1978\ub144 \uadf8\uc758 \uac00\uc871\uc740 \uce98\ub9ac\ud3ec\ub2c8\uc544\uc758 \ud314\ub85c\uc2a4 \ubc84\ub2e4\uc2a4(Palos Verdes)\ub85c \uc774\uc8fc\ud588\ub294\ub370, \ub530\ub73b\ud55c \uc774 \uc9c0\uc5ed\uc758 \uae30\ud6c4 \ub355\ubd84\uc5d0 \ub2f9\uc2dc 7\uc0b4\uc774\uc5c8\ub358 \uc0d8\ud504\ub7ec\uc2a4\ub294 \ud14c\ub2c8\uc2a4\ub97c \ub354 \ub9c8\uc74c\uaecf \uce60 \uc218 \uc788\uac8c \ub418\uc5c8\ub2e4. \uc5b4\ub9b0\uc2dc\uc808 \uadf8\uc758 \uc6b0\uc0c1\uc740 \ub85c\ub4dc \ub808\uc774\ubc84\uc600\uc73c\uba70, 11\uc0b4 \ub54c \uc2e4\uc81c\ub85c \ub85c\ub4dc \ub808\uc774\ubc84\ub97c \ub9cc\ub098 \ud568\uaed8 \ud14c\ub2c8\uc2a4\ub97c \uce58\uae30\ub3c4 \ud588\ub2e4. \uc0d8\ud504\ub7ec\uc2a4\uc758 \uac00\uc871\uc740 \ud398\ub2c8\uc290\ub77c \ub77c\ucf13 \ud074\ub7fd(Peninsula Racket Club)\uc5d0 \uac00\uc785\ud588\ub294\ub370, \uc774\uac83\uc774 \uadf8\uc758 \uc7ac\ub2a5\uc744 \ubc1c\uacac\ud558\ub294 \uacc4\uae30\uac00 \ub418\uc5c8\ub2e4. \uc18c\uc544\uacfc \uc758\uc0ac\uc774\uc790 \ud14c\ub2c8\uc2a4\uad11\uc774\uc5c8\ub358 \ud53c\ud130 \ud53c\uc154(Peter Fischer)\uac00 \ud074\ub7fd\uc5d0 \uc628 \uc0d8\ud504\ub7ec\uc2a4\uc758 \uc7ac\ub2a5\uc744 \uc54c\uc544\ubcf4\uace0 \uadf8\ub54c\ubd80\ud130 1989\ub144\uae4c\uc9c0 \uc0d8\ud504\ub7ec\uc2a4\uc758 \ucf54\uce58 \uc5ed\ud560\uc744 \ub9e1\uc558\ub2e4 \uc0d8\ud504\ub7ec\uc2a4\uac00 \uc714\ube14\ub358\uc5d0\uc11c \uc6b0\uc2b9\ud558\uac8c \ub9cc\ub4dc\ub294 \uac83\uc744 \ubaa9\ud45c\ub85c \uc0bc\uc740 \ud53c\uc154\ub294 \uc714\ube14\ub358\uc758 \uc794\ub514 \ucf54\ud2b8\ub97c \uace0\ub824\ud558\uc5ec \uc0d8\ud504\ub7ec\uc2a4\uc758 \ubc31\ud578\ub4dc\ub97c \ud22c\ud578\ub4dc\uc5d0\uc11c \uc6d0\ud578\ub4dc\ub85c \ubc14\uafb8\uac8c \ud588\ub2e4.", "answer": "\ud53c\ud130 \ud53c\uc154", "sentence": "\uc18c\uc544\uacfc \uc758\uc0ac\uc774\uc790 \ud14c\ub2c8\uc2a4\uad11\uc774\uc5c8\ub358 \ud53c\ud130 \ud53c\uc154 (Peter Fischer)", "paragraph_sentence": "\uc0d8\ud504\ub7ec\uc2a4\ub294 \uc77c\ucc0d\uc774 \uc5b4\ub9b4 \ub54c\ubd80\ud130 \ub6f0\uc5b4\ub09c \uc6b4\ub3d9\uc120\uc218\uac00 \ub420 \uc790\uc9c8\uc744 \ubcf4\uc600\ub2e4. \uadf8\ub294 \uc5b4\ub9b0 \uc2dc\uc808 \uc9d1\uc5d0 \uc788\ub294 \uc9c0\ud558\uc2e4\uc5d0\uc11c \ub77c\ucf13\uc744 \ubc1c\uacac\ud558\uace0\ub294 \uba87 \uc2dc\uac04\ub3d9\uc548\uc774\uace0 \ubcbd\uc5d0 \uacf5\uc744 \uce58\uba70 \ub180\uc558\ub2e4. 1978\ub144 \uadf8\uc758 \uac00\uc871\uc740 \uce98\ub9ac\ud3ec\ub2c8\uc544\uc758 \ud314\ub85c\uc2a4 \ubc84\ub2e4\uc2a4(Palos Verdes)\ub85c \uc774\uc8fc\ud588\ub294\ub370, \ub530\ub73b\ud55c \uc774 \uc9c0\uc5ed\uc758 \uae30\ud6c4 \ub355\ubd84\uc5d0 \ub2f9\uc2dc 7\uc0b4\uc774\uc5c8\ub358 \uc0d8\ud504\ub7ec\uc2a4\ub294 \ud14c\ub2c8\uc2a4\ub97c \ub354 \ub9c8\uc74c\uaecf \uce60 \uc218 \uc788\uac8c \ub418\uc5c8\ub2e4. \uc5b4\ub9b0\uc2dc\uc808 \uadf8\uc758 \uc6b0\uc0c1\uc740 \ub85c\ub4dc \ub808\uc774\ubc84\uc600\uc73c\uba70, 11\uc0b4 \ub54c \uc2e4\uc81c\ub85c \ub85c\ub4dc \ub808\uc774\ubc84\ub97c \ub9cc\ub098 \ud568\uaed8 \ud14c\ub2c8\uc2a4\ub97c \uce58\uae30\ub3c4 \ud588\ub2e4. \uc0d8\ud504\ub7ec\uc2a4\uc758 \uac00\uc871\uc740 \ud398\ub2c8\uc290\ub77c \ub77c\ucf13 \ud074\ub7fd(Peninsula Racket Club)\uc5d0 \uac00\uc785\ud588\ub294\ub370, \uc774\uac83\uc774 \uadf8\uc758 \uc7ac\ub2a5\uc744 \ubc1c\uacac\ud558\ub294 \uacc4\uae30\uac00 \ub418\uc5c8\ub2e4. \uc18c\uc544\uacfc \uc758\uc0ac\uc774\uc790 \ud14c\ub2c8\uc2a4\uad11\uc774\uc5c8\ub358 \ud53c\ud130 \ud53c\uc154 (Peter Fischer) \uac00 \ud074\ub7fd\uc5d0 \uc628 \uc0d8\ud504\ub7ec\uc2a4\uc758 \uc7ac\ub2a5\uc744 \uc54c\uc544\ubcf4\uace0 \uadf8\ub54c\ubd80\ud130 1989\ub144\uae4c\uc9c0 \uc0d8\ud504\ub7ec\uc2a4\uc758 \ucf54\uce58 \uc5ed\ud560\uc744 \ub9e1\uc558\ub2e4 \uc0d8\ud504\ub7ec\uc2a4\uac00 \uc714\ube14\ub358\uc5d0\uc11c \uc6b0\uc2b9\ud558\uac8c \ub9cc\ub4dc\ub294 \uac83\uc744 \ubaa9\ud45c\ub85c \uc0bc\uc740 \ud53c\uc154\ub294 \uc714\ube14\ub358\uc758 \uc794\ub514 \ucf54\ud2b8\ub97c \uace0\ub824\ud558\uc5ec \uc0d8\ud504\ub7ec\uc2a4\uc758 \ubc31\ud578\ub4dc\ub97c \ud22c\ud578\ub4dc\uc5d0\uc11c \uc6d0\ud578\ub4dc\ub85c \ubc14\uafb8\uac8c \ud588\ub2e4.", "paragraph_answer": "\uc0d8\ud504\ub7ec\uc2a4\ub294 \uc77c\ucc0d\uc774 \uc5b4\ub9b4 \ub54c\ubd80\ud130 \ub6f0\uc5b4\ub09c \uc6b4\ub3d9\uc120\uc218\uac00 \ub420 \uc790\uc9c8\uc744 \ubcf4\uc600\ub2e4. \uadf8\ub294 \uc5b4\ub9b0 \uc2dc\uc808 \uc9d1\uc5d0 \uc788\ub294 \uc9c0\ud558\uc2e4\uc5d0\uc11c \ub77c\ucf13\uc744 \ubc1c\uacac\ud558\uace0\ub294 \uba87 \uc2dc\uac04\ub3d9\uc548\uc774\uace0 \ubcbd\uc5d0 \uacf5\uc744 \uce58\uba70 \ub180\uc558\ub2e4. 1978\ub144 \uadf8\uc758 \uac00\uc871\uc740 \uce98\ub9ac\ud3ec\ub2c8\uc544\uc758 \ud314\ub85c\uc2a4 \ubc84\ub2e4\uc2a4(Palos Verdes)\ub85c \uc774\uc8fc\ud588\ub294\ub370, \ub530\ub73b\ud55c \uc774 \uc9c0\uc5ed\uc758 \uae30\ud6c4 \ub355\ubd84\uc5d0 \ub2f9\uc2dc 7\uc0b4\uc774\uc5c8\ub358 \uc0d8\ud504\ub7ec\uc2a4\ub294 \ud14c\ub2c8\uc2a4\ub97c \ub354 \ub9c8\uc74c\uaecf \uce60 \uc218 \uc788\uac8c \ub418\uc5c8\ub2e4. \uc5b4\ub9b0\uc2dc\uc808 \uadf8\uc758 \uc6b0\uc0c1\uc740 \ub85c\ub4dc \ub808\uc774\ubc84\uc600\uc73c\uba70, 11\uc0b4 \ub54c \uc2e4\uc81c\ub85c \ub85c\ub4dc \ub808\uc774\ubc84\ub97c \ub9cc\ub098 \ud568\uaed8 \ud14c\ub2c8\uc2a4\ub97c \uce58\uae30\ub3c4 \ud588\ub2e4. \uc0d8\ud504\ub7ec\uc2a4\uc758 \uac00\uc871\uc740 \ud398\ub2c8\uc290\ub77c \ub77c\ucf13 \ud074\ub7fd(Peninsula Racket Club)\uc5d0 \uac00\uc785\ud588\ub294\ub370, \uc774\uac83\uc774 \uadf8\uc758 \uc7ac\ub2a5\uc744 \ubc1c\uacac\ud558\ub294 \uacc4\uae30\uac00 \ub418\uc5c8\ub2e4. \uc18c\uc544\uacfc \uc758\uc0ac\uc774\uc790 \ud14c\ub2c8\uc2a4\uad11\uc774\uc5c8\ub358 \ud53c\ud130 \ud53c\uc154 (Peter Fischer)\uac00 \ud074\ub7fd\uc5d0 \uc628 \uc0d8\ud504\ub7ec\uc2a4\uc758 \uc7ac\ub2a5\uc744 \uc54c\uc544\ubcf4\uace0 \uadf8\ub54c\ubd80\ud130 1989\ub144\uae4c\uc9c0 \uc0d8\ud504\ub7ec\uc2a4\uc758 \ucf54\uce58 \uc5ed\ud560\uc744 \ub9e1\uc558\ub2e4 \uc0d8\ud504\ub7ec\uc2a4\uac00 \uc714\ube14\ub358\uc5d0\uc11c \uc6b0\uc2b9\ud558\uac8c \ub9cc\ub4dc\ub294 \uac83\uc744 \ubaa9\ud45c\ub85c \uc0bc\uc740 \ud53c\uc154\ub294 \uc714\ube14\ub358\uc758 \uc794\ub514 \ucf54\ud2b8\ub97c \uace0\ub824\ud558\uc5ec \uc0d8\ud504\ub7ec\uc2a4\uc758 \ubc31\ud578\ub4dc\ub97c \ud22c\ud578\ub4dc\uc5d0\uc11c \uc6d0\ud578\ub4dc\ub85c \ubc14\uafb8\uac8c \ud588\ub2e4.", "sentence_answer": "\uc18c\uc544\uacfc \uc758\uc0ac\uc774\uc790 \ud14c\ub2c8\uc2a4\uad11\uc774\uc5c8\ub358 \ud53c\ud130 \ud53c\uc154 (Peter Fischer)", "paragraph_id": "6568928-1-2", "paragraph_question": "question: \ud398\ub2c8\uc290\ub77c \ub77c\ucf13 \ud074\ub7fd\uc5d0\uc11c \uc0d8\ud504\ub7ec\uc2a4\uc758 \uc7ac\ub2a5\uc744 \uc54c\uc544\ubcf8 \uc0ac\ub78c\uc758 \uc774\ub984\uc740?, context: \uc0d8\ud504\ub7ec\uc2a4\ub294 \uc77c\ucc0d\uc774 \uc5b4\ub9b4 \ub54c\ubd80\ud130 \ub6f0\uc5b4\ub09c \uc6b4\ub3d9\uc120\uc218\uac00 \ub420 \uc790\uc9c8\uc744 \ubcf4\uc600\ub2e4. \uadf8\ub294 \uc5b4\ub9b0 \uc2dc\uc808 \uc9d1\uc5d0 \uc788\ub294 \uc9c0\ud558\uc2e4\uc5d0\uc11c \ub77c\ucf13\uc744 \ubc1c\uacac\ud558\uace0\ub294 \uba87 \uc2dc\uac04\ub3d9\uc548\uc774\uace0 \ubcbd\uc5d0 \uacf5\uc744 \uce58\uba70 \ub180\uc558\ub2e4. 1978\ub144 \uadf8\uc758 \uac00\uc871\uc740 \uce98\ub9ac\ud3ec\ub2c8\uc544\uc758 \ud314\ub85c\uc2a4 \ubc84\ub2e4\uc2a4(Palos Verdes)\ub85c \uc774\uc8fc\ud588\ub294\ub370, \ub530\ub73b\ud55c \uc774 \uc9c0\uc5ed\uc758 \uae30\ud6c4 \ub355\ubd84\uc5d0 \ub2f9\uc2dc 7\uc0b4\uc774\uc5c8\ub358 \uc0d8\ud504\ub7ec\uc2a4\ub294 \ud14c\ub2c8\uc2a4\ub97c \ub354 \ub9c8\uc74c\uaecf \uce60 \uc218 \uc788\uac8c \ub418\uc5c8\ub2e4. \uc5b4\ub9b0\uc2dc\uc808 \uadf8\uc758 \uc6b0\uc0c1\uc740 \ub85c\ub4dc \ub808\uc774\ubc84\uc600\uc73c\uba70, 11\uc0b4 \ub54c \uc2e4\uc81c\ub85c \ub85c\ub4dc \ub808\uc774\ubc84\ub97c \ub9cc\ub098 \ud568\uaed8 \ud14c\ub2c8\uc2a4\ub97c \uce58\uae30\ub3c4 \ud588\ub2e4. \uc0d8\ud504\ub7ec\uc2a4\uc758 \uac00\uc871\uc740 \ud398\ub2c8\uc290\ub77c \ub77c\ucf13 \ud074\ub7fd(Peninsula Racket Club)\uc5d0 \uac00\uc785\ud588\ub294\ub370, \uc774\uac83\uc774 \uadf8\uc758 \uc7ac\ub2a5\uc744 \ubc1c\uacac\ud558\ub294 \uacc4\uae30\uac00 \ub418\uc5c8\ub2e4. \uc18c\uc544\uacfc \uc758\uc0ac\uc774\uc790 \ud14c\ub2c8\uc2a4\uad11\uc774\uc5c8\ub358 \ud53c\ud130 \ud53c\uc154(Peter Fischer)\uac00 \ud074\ub7fd\uc5d0 \uc628 \uc0d8\ud504\ub7ec\uc2a4\uc758 \uc7ac\ub2a5\uc744 \uc54c\uc544\ubcf4\uace0 \uadf8\ub54c\ubd80\ud130 1989\ub144\uae4c\uc9c0 \uc0d8\ud504\ub7ec\uc2a4\uc758 \ucf54\uce58 \uc5ed\ud560\uc744 \ub9e1\uc558\ub2e4 \uc0d8\ud504\ub7ec\uc2a4\uac00 \uc714\ube14\ub358\uc5d0\uc11c \uc6b0\uc2b9\ud558\uac8c \ub9cc\ub4dc\ub294 \uac83\uc744 \ubaa9\ud45c\ub85c \uc0bc\uc740 \ud53c\uc154\ub294 \uc714\ube14\ub358\uc758 \uc794\ub514 \ucf54\ud2b8\ub97c \uace0\ub824\ud558\uc5ec \uc0d8\ud504\ub7ec\uc2a4\uc758 \ubc31\ud578\ub4dc\ub97c \ud22c\ud578\ub4dc\uc5d0\uc11c \uc6d0\ud578\ub4dc\ub85c \ubc14\uafb8\uac8c \ud588\ub2e4."} {"question": "\uc911\uad6d\uc774 \ud575\ubbf8\uc0ac\uc77c \uac1c\ubc1c\uc5d0 \ud544\uc694\ud55c \ubb3c\uc790\ub97c \uc218\ucd9c\ud55c \ubd81\ud55c\uc758 \ud56d\uad6c\ub294?", "paragraph": "\uad6d\uc81c\uc0ac\ud68c\uc758 \uac15\ub825\ud55c \uc81c\uc7ac\ub97c \ubc1b\uace0 \uc788\ub294 \ubd81\ud55c\uc774 \uc5b4\ub5bb\uac8c \ud575\uacfc \ubbf8\uc0ac\uc77c \uac1c\ubc1c\uc5d0 \ud544\uc694\ud55c \ubb3c\uc790\ub97c \uc870\ub2ec \ubc1b\uc558\uc744\uae4c \ud558\ub294 \uc758\ubb38\uc774 \uc788\uc5b4\uc654\ub294\ub370, \ubd81\ud55c\uc5d0 \uc720\uc785\ub3fc\uc120 \uc548 \ub420 \ud575\uc2ec \ubb3c\uc790\ub4e4\uc744 \uafb8\uc900\ud788 \uacf5\uae09\ud574\uc628 \uc911\uad6d\uc758 \uad6d\uc720\uae30\uc5c5\uc774 \uc801\ubc1c\ub418\uc5c8\ub2e4. \uc911\uad6d\uc740 \ubd81\ud55c \ucd5c\ub300 \ubb34\uc5ed\ud56d\uc778 \ub0a8\ud3ec\ud56d\uc744 \ud1b5\ud574 2011\ub144\ubd80\ud130 \ubd81\ud55c\uc758 4\ucc28 \ud575\uc2e4\ud5d8 \ub109 \ub2ec \uc804\uc778 2015\ub144 9\uc6d4\uae4c\uc9c0 \ud575\ubbf8\uc0ac\uc77c \uac1c\ubc1c\uc5d0 \ud544\uc694\ud55c \ubb3c\uc790\ub97c \uc218\ucd9c\ud574\uc654\ub2e4. \uadf8 \uc911\uc5d4 \uace0\uc21c\ub3c4\uc758 \uc54c\ub8e8\ubbf8\ub284\uad34\uc640 \uc0b0\ud654\uc54c\ub8e8\ubbf8\ub284, \ud145\uc2a4\ud150 \ub4f1 \ud575\uacfc \ubbf8\uc0ac\uc77c \uac1c\ubc1c\uc5d0 \ud544\uc694\ud55c \ud575\uc2ec \uc7ac\ub8cc 4\uc885\ub958\uac00 \ud3ec\ud568\ub418\uc5b4\uc788\ub2e4. \uc911\uad6d\uc740 \ubd81\ud55c\uc758 \ud575\ubbf8\uc0ac\uc77c \uac1c\ubc1c\uc740 \uc9c0\uc6d0\ud574\uc92c\uc73c\uba74\uc11c \ud55c\uad6d\uc5d0\uac8c\ub294 \ubc29\uc5b4\uc801\uc778 \ubb34\uae30\uc870\ucc28\ub3c4 \ubc30\uce58\ud558\uc9c0 \ub9d0\ub77c\uba70 \uacbd\uc81c\uc801 \ubcf4\ubcf5\uc744 \ud558\uace0 \uc788\ub2e4. \ud575 \uac1c\ubc1c \ubb3c\uc790\ub97c \ubd81\ud55c\uc73c\ub85c \uc218\ucd9c\ud55c \uc911\uad6d\uc740 \ubd81\ud55c\uc758 \ud55c\ubc18\ub3c4 \ud575 \uc704\ud611\uc5d0 \ub300\ud55c \ucc45\uc784\uc73c\ub85c\ubd80\ud130 \uc790\uc720\ub86d\uc9c0 \ubabb\ud558\uace0 \ubd81\ud55c\uc758 \ud575 \uc704\ud611\uc5d0 \ub300\ud56d\ud558\uae30 \uc704\ud574 \ub3c4\uc785\ub418\ub294 \uc0ac\ub4dc\ub97c \uc911\uad6d\uc774 \ubc18\ub300\ud560 \uba85\ubd84\uc774 \uc5c6\ub2e4.", "answer": "\ub0a8\ud3ec\ud56d", "sentence": "\uc911\uad6d\uc740 \ubd81\ud55c \ucd5c\ub300 \ubb34\uc5ed\ud56d\uc778 \ub0a8\ud3ec\ud56d \uc744 \ud1b5\ud574 2011\ub144\ubd80\ud130 \ubd81\ud55c\uc758 4\ucc28 \ud575\uc2e4\ud5d8 \ub109 \ub2ec \uc804\uc778 2015\ub144 9\uc6d4\uae4c\uc9c0 \ud575\ubbf8\uc0ac\uc77c \uac1c\ubc1c\uc5d0 \ud544\uc694\ud55c \ubb3c\uc790\ub97c \uc218\ucd9c\ud574\uc654\ub2e4.", "paragraph_sentence": "\uad6d\uc81c\uc0ac\ud68c\uc758 \uac15\ub825\ud55c \uc81c\uc7ac\ub97c \ubc1b\uace0 \uc788\ub294 \ubd81\ud55c\uc774 \uc5b4\ub5bb\uac8c \ud575\uacfc \ubbf8\uc0ac\uc77c \uac1c\ubc1c\uc5d0 \ud544\uc694\ud55c \ubb3c\uc790\ub97c \uc870\ub2ec \ubc1b\uc558\uc744\uae4c \ud558\ub294 \uc758\ubb38\uc774 \uc788\uc5b4\uc654\ub294\ub370, \ubd81\ud55c\uc5d0 \uc720\uc785\ub3fc\uc120 \uc548 \ub420 \ud575\uc2ec \ubb3c\uc790\ub4e4\uc744 \uafb8\uc900\ud788 \uacf5\uae09\ud574\uc628 \uc911\uad6d\uc758 \uad6d\uc720\uae30\uc5c5\uc774 \uc801\ubc1c\ub418\uc5c8\ub2e4. \uc911\uad6d\uc740 \ubd81\ud55c \ucd5c\ub300 \ubb34\uc5ed\ud56d\uc778 \ub0a8\ud3ec\ud56d \uc744 \ud1b5\ud574 2011\ub144\ubd80\ud130 \ubd81\ud55c\uc758 4\ucc28 \ud575\uc2e4\ud5d8 \ub109 \ub2ec \uc804\uc778 2015\ub144 9\uc6d4\uae4c\uc9c0 \ud575\ubbf8\uc0ac\uc77c \uac1c\ubc1c\uc5d0 \ud544\uc694\ud55c \ubb3c\uc790\ub97c \uc218\ucd9c\ud574\uc654\ub2e4. \uadf8 \uc911\uc5d4 \uace0\uc21c\ub3c4\uc758 \uc54c\ub8e8\ubbf8\ub284\uad34\uc640 \uc0b0\ud654\uc54c\ub8e8\ubbf8\ub284, \ud145\uc2a4\ud150 \ub4f1 \ud575\uacfc \ubbf8\uc0ac\uc77c \uac1c\ubc1c\uc5d0 \ud544\uc694\ud55c \ud575\uc2ec \uc7ac\ub8cc 4\uc885\ub958\uac00 \ud3ec\ud568\ub418\uc5b4\uc788\ub2e4. \uc911\uad6d\uc740 \ubd81\ud55c\uc758 \ud575\ubbf8\uc0ac\uc77c \uac1c\ubc1c\uc740 \uc9c0\uc6d0\ud574\uc92c\uc73c\uba74\uc11c \ud55c\uad6d\uc5d0\uac8c\ub294 \ubc29\uc5b4\uc801\uc778 \ubb34\uae30\uc870\ucc28\ub3c4 \ubc30\uce58\ud558\uc9c0 \ub9d0\ub77c\uba70 \uacbd\uc81c\uc801 \ubcf4\ubcf5\uc744 \ud558\uace0 \uc788\ub2e4. \ud575 \uac1c\ubc1c \ubb3c\uc790\ub97c \ubd81\ud55c\uc73c\ub85c \uc218\ucd9c\ud55c \uc911\uad6d\uc740 \ubd81\ud55c\uc758 \ud55c\ubc18\ub3c4 \ud575 \uc704\ud611\uc5d0 \ub300\ud55c \ucc45\uc784\uc73c\ub85c\ubd80\ud130 \uc790\uc720\ub86d\uc9c0 \ubabb\ud558\uace0 \ubd81\ud55c\uc758 \ud575 \uc704\ud611\uc5d0 \ub300\ud56d\ud558\uae30 \uc704\ud574 \ub3c4\uc785\ub418\ub294 \uc0ac\ub4dc\ub97c \uc911\uad6d\uc774 \ubc18\ub300\ud560 \uba85\ubd84\uc774 \uc5c6\ub2e4.", "paragraph_answer": "\uad6d\uc81c\uc0ac\ud68c\uc758 \uac15\ub825\ud55c \uc81c\uc7ac\ub97c \ubc1b\uace0 \uc788\ub294 \ubd81\ud55c\uc774 \uc5b4\ub5bb\uac8c \ud575\uacfc \ubbf8\uc0ac\uc77c \uac1c\ubc1c\uc5d0 \ud544\uc694\ud55c \ubb3c\uc790\ub97c \uc870\ub2ec \ubc1b\uc558\uc744\uae4c \ud558\ub294 \uc758\ubb38\uc774 \uc788\uc5b4\uc654\ub294\ub370, \ubd81\ud55c\uc5d0 \uc720\uc785\ub3fc\uc120 \uc548 \ub420 \ud575\uc2ec \ubb3c\uc790\ub4e4\uc744 \uafb8\uc900\ud788 \uacf5\uae09\ud574\uc628 \uc911\uad6d\uc758 \uad6d\uc720\uae30\uc5c5\uc774 \uc801\ubc1c\ub418\uc5c8\ub2e4. \uc911\uad6d\uc740 \ubd81\ud55c \ucd5c\ub300 \ubb34\uc5ed\ud56d\uc778 \ub0a8\ud3ec\ud56d \uc744 \ud1b5\ud574 2011\ub144\ubd80\ud130 \ubd81\ud55c\uc758 4\ucc28 \ud575\uc2e4\ud5d8 \ub109 \ub2ec \uc804\uc778 2015\ub144 9\uc6d4\uae4c\uc9c0 \ud575\ubbf8\uc0ac\uc77c \uac1c\ubc1c\uc5d0 \ud544\uc694\ud55c \ubb3c\uc790\ub97c \uc218\ucd9c\ud574\uc654\ub2e4. \uadf8 \uc911\uc5d4 \uace0\uc21c\ub3c4\uc758 \uc54c\ub8e8\ubbf8\ub284\uad34\uc640 \uc0b0\ud654\uc54c\ub8e8\ubbf8\ub284, \ud145\uc2a4\ud150 \ub4f1 \ud575\uacfc \ubbf8\uc0ac\uc77c \uac1c\ubc1c\uc5d0 \ud544\uc694\ud55c \ud575\uc2ec \uc7ac\ub8cc 4\uc885\ub958\uac00 \ud3ec\ud568\ub418\uc5b4\uc788\ub2e4. \uc911\uad6d\uc740 \ubd81\ud55c\uc758 \ud575\ubbf8\uc0ac\uc77c \uac1c\ubc1c\uc740 \uc9c0\uc6d0\ud574\uc92c\uc73c\uba74\uc11c \ud55c\uad6d\uc5d0\uac8c\ub294 \ubc29\uc5b4\uc801\uc778 \ubb34\uae30\uc870\ucc28\ub3c4 \ubc30\uce58\ud558\uc9c0 \ub9d0\ub77c\uba70 \uacbd\uc81c\uc801 \ubcf4\ubcf5\uc744 \ud558\uace0 \uc788\ub2e4. \ud575 \uac1c\ubc1c \ubb3c\uc790\ub97c \ubd81\ud55c\uc73c\ub85c \uc218\ucd9c\ud55c \uc911\uad6d\uc740 \ubd81\ud55c\uc758 \ud55c\ubc18\ub3c4 \ud575 \uc704\ud611\uc5d0 \ub300\ud55c \ucc45\uc784\uc73c\ub85c\ubd80\ud130 \uc790\uc720\ub86d\uc9c0 \ubabb\ud558\uace0 \ubd81\ud55c\uc758 \ud575 \uc704\ud611\uc5d0 \ub300\ud56d\ud558\uae30 \uc704\ud574 \ub3c4\uc785\ub418\ub294 \uc0ac\ub4dc\ub97c \uc911\uad6d\uc774 \ubc18\ub300\ud560 \uba85\ubd84\uc774 \uc5c6\ub2e4.", "sentence_answer": "\uc911\uad6d\uc740 \ubd81\ud55c \ucd5c\ub300 \ubb34\uc5ed\ud56d\uc778 \ub0a8\ud3ec\ud56d \uc744 \ud1b5\ud574 2011\ub144\ubd80\ud130 \ubd81\ud55c\uc758 4\ucc28 \ud575\uc2e4\ud5d8 \ub109 \ub2ec \uc804\uc778 2015\ub144 9\uc6d4\uae4c\uc9c0 \ud575\ubbf8\uc0ac\uc77c \uac1c\ubc1c\uc5d0 \ud544\uc694\ud55c \ubb3c\uc790\ub97c \uc218\ucd9c\ud574\uc654\ub2e4.", "paragraph_id": "6486360-14-1", "paragraph_question": "question: \uc911\uad6d\uc774 \ud575\ubbf8\uc0ac\uc77c \uac1c\ubc1c\uc5d0 \ud544\uc694\ud55c \ubb3c\uc790\ub97c \uc218\ucd9c\ud55c \ubd81\ud55c\uc758 \ud56d\uad6c\ub294?, context: \uad6d\uc81c\uc0ac\ud68c\uc758 \uac15\ub825\ud55c \uc81c\uc7ac\ub97c \ubc1b\uace0 \uc788\ub294 \ubd81\ud55c\uc774 \uc5b4\ub5bb\uac8c \ud575\uacfc \ubbf8\uc0ac\uc77c \uac1c\ubc1c\uc5d0 \ud544\uc694\ud55c \ubb3c\uc790\ub97c \uc870\ub2ec \ubc1b\uc558\uc744\uae4c \ud558\ub294 \uc758\ubb38\uc774 \uc788\uc5b4\uc654\ub294\ub370, \ubd81\ud55c\uc5d0 \uc720\uc785\ub3fc\uc120 \uc548 \ub420 \ud575\uc2ec \ubb3c\uc790\ub4e4\uc744 \uafb8\uc900\ud788 \uacf5\uae09\ud574\uc628 \uc911\uad6d\uc758 \uad6d\uc720\uae30\uc5c5\uc774 \uc801\ubc1c\ub418\uc5c8\ub2e4. \uc911\uad6d\uc740 \ubd81\ud55c \ucd5c\ub300 \ubb34\uc5ed\ud56d\uc778 \ub0a8\ud3ec\ud56d\uc744 \ud1b5\ud574 2011\ub144\ubd80\ud130 \ubd81\ud55c\uc758 4\ucc28 \ud575\uc2e4\ud5d8 \ub109 \ub2ec \uc804\uc778 2015\ub144 9\uc6d4\uae4c\uc9c0 \ud575\ubbf8\uc0ac\uc77c \uac1c\ubc1c\uc5d0 \ud544\uc694\ud55c \ubb3c\uc790\ub97c \uc218\ucd9c\ud574\uc654\ub2e4. \uadf8 \uc911\uc5d4 \uace0\uc21c\ub3c4\uc758 \uc54c\ub8e8\ubbf8\ub284\uad34\uc640 \uc0b0\ud654\uc54c\ub8e8\ubbf8\ub284, \ud145\uc2a4\ud150 \ub4f1 \ud575\uacfc \ubbf8\uc0ac\uc77c \uac1c\ubc1c\uc5d0 \ud544\uc694\ud55c \ud575\uc2ec \uc7ac\ub8cc 4\uc885\ub958\uac00 \ud3ec\ud568\ub418\uc5b4\uc788\ub2e4. \uc911\uad6d\uc740 \ubd81\ud55c\uc758 \ud575\ubbf8\uc0ac\uc77c \uac1c\ubc1c\uc740 \uc9c0\uc6d0\ud574\uc92c\uc73c\uba74\uc11c \ud55c\uad6d\uc5d0\uac8c\ub294 \ubc29\uc5b4\uc801\uc778 \ubb34\uae30\uc870\ucc28\ub3c4 \ubc30\uce58\ud558\uc9c0 \ub9d0\ub77c\uba70 \uacbd\uc81c\uc801 \ubcf4\ubcf5\uc744 \ud558\uace0 \uc788\ub2e4. \ud575 \uac1c\ubc1c \ubb3c\uc790\ub97c \ubd81\ud55c\uc73c\ub85c \uc218\ucd9c\ud55c \uc911\uad6d\uc740 \ubd81\ud55c\uc758 \ud55c\ubc18\ub3c4 \ud575 \uc704\ud611\uc5d0 \ub300\ud55c \ucc45\uc784\uc73c\ub85c\ubd80\ud130 \uc790\uc720\ub86d\uc9c0 \ubabb\ud558\uace0 \ubd81\ud55c\uc758 \ud575 \uc704\ud611\uc5d0 \ub300\ud56d\ud558\uae30 \uc704\ud574 \ub3c4\uc785\ub418\ub294 \uc0ac\ub4dc\ub97c \uc911\uad6d\uc774 \ubc18\ub300\ud560 \uba85\ubd84\uc774 \uc5c6\ub2e4."} {"question": "\ud50c\ub808\uc774\uc624\ud504 1\ub77c\uc6b4\ub4dc\uc5d0\uc11c \uc5c4\uccad\ub09c \ud65c\uc57d\uc744 \ud3bc\uccd0 \uace8\ub4e0\uc2a4\ud14c\uc774\ud2b8 \uc6cc\ub9ac\uc5b4\uc2a4\uc758 2\ub77c\uc6b4\ub4dc \uc9c4\ucd9c\uc744 \uacac\uc778\ud55c \uc120\uc218\ub294?", "paragraph": "\ucee4\ub9ac\uc758 \uc131\uacf5\uc801\uc778 \ubcf5\uadc0\uc640 \ud568\uaed8 2\ub144\ucc28 \ub274\ube44 \uac10\ub3c5 \ub9c8\ud06c \uc7ad\uc2a8, \ubcf8\uc778 \uc2a4\ud0ef\ub9cc \uc88b\uace0 \ud300\uc740 \ub9e8\ub0a0 \uc9c4\ub2e4\ub294 \ube44\uc544\ub0e5\uc744 \ubc1b\ub358 PF \ub370\uc774\ube44\ub4dc \ub9ac, \uc800\ub2c8\ub9e8\uc774\uc5c8\ub358 \uc7ac\ub7ff \uc7ad, \uc2e0\uc778 \uc288\ud130 \ud074\ub808\uc774 \ud1b0\uc2a8 \ub4f1\uc758 \uc2dc\ub108\uc9c0\uc5d0 \ud798\uc785\uc5b4 1994\ub144 \uc774\ub798 \ud50c\ub808\uc774\uc624\ud504 \uc9c4\ucd9c\uc744 \ub531 \ud55c\ubc88\ubc16\uc5d0 \ud574\ubcf8 \uc801\uc774 \uc5c6\ub294(2006\ub144) \uc548\uc2b5\uc758 \ud300 \uace8\ub4e0\uc2a4\ud14c\uc774\ud2b8 \uc6cc\ub9ac\uc5b4\uc2a4\ub294 47\uc2b9 35\ud328\ub85c \ud50c\ub808\uc774\uc624\ud504\uc5d0 \uc9c4\ucd9c\ud558\ub294 \ucf8c\uac70\ub97c \uc774\ub8ec\ub2e4. \ub374\ubc84 \ub108\uac8c\uce20\uc640\uc758 \ud50c\ub808\uc774\uc624\ud504 1\ub77c\uc6b4\ub4dc \ucd1d 6\uacbd\uae30\uc5d0\uc11c \ucee4\ub9ac\ub294 \ud3c9\uade0 24.3\uc810 9.3\uc5b4\uc2dc\uc2a4\ud2b8 4.3\ub9ac\ubc14\uc6b4\ub4dc 2.2\uc2a4\ud2f8, FG 46.8% 3P 43.4% FT(\uc790\uc720\ud22c) 100%(...)\ub77c\ub294 \uc5c4\uccad\ub09c \ud65c\uc57d\uc744 \ud3bc\uce58\uba70 4\uc2b9 2\ud328\ub85c \ud300\uc744 2\ub77c\uc6b4\ub4dc\uc5d0 \uc9c4\ucd9c\uc2dc\ud0a4\ub294 \uc77c\ub4f1\uacf5\uc2e0\uc774 \ub418\uc5c8\ub2e4. 2\ub77c\uc6b4\ub4dc\uc758 \uc0c1\ub300\ub294 \uc804\ud1b5\uc758 \uac15\ud300 \uc0cc\uc548\ud1a0\ub2c8\uc624 \uc2a4\ud37c\uc2a4. \uace8\ub4e0\uc2a4\ud14c\uc774\ud2b8\ub294 2\uc2b9 2\ud328\uae4c\uc9c0 \ub530\ub77c\uac00\uba70 \uc120\uc804\ud588\uc73c\ub098, 5, 6 \uacbd\uae30\ub97c \uc5f0\uc774\uc5b4 \ub0b4\uc8fc\uba70 2\uc2b9 4\ud328\ub85c \uc544\uc26c\uc6b4 \ud0c8\ub77d\uc744 \ub9db\ubcf8\ub2e4. \ucee4\ub9ac\ub294 1\uacbd\uae30\uc5d0\uc11c 44\uc810 11\uc5b4\uc2dc\uc2a4\ud2b8(3\uc810\uc29b 42.9%)\ub97c \uae30\ub85d\ud558\uba70 \uc13c\uc138\uc774\uc158\uc744 \uc774\uc5b4\uac08 \uac83\ucc98\ub7fc \ubcf4\uc600\uc73c\ub098, 2\uacbd\uae30\ubd80\ud130 \uc2a4\ud37c\uc2a4\uc758 \uc218\ube44\uc5d0 \ucca0\uc800\ud788 \ubb36\uc774\uba70 \uae30\ub300\ub9cc\ud07c\uc758 \ud65c\uc57d\uc744 \ubcf4\uc5ec\uc8fc\uc9c0 \ubabb\ud588\ub2e4. 2\ub77c\uc6b4\ub4dc\uc5d0\uc11c\uc758 \ubd80\uc9c4\uc73c\ub85c \ud50c\ub808\uc774\uc624\ud504 \uc804\uccb4 \uc131\uc801\uc740 23.4\uc810 8.1\uc5b4\uc2dc\uc2a4\ud2b8 3.8\ub9ac\ubc14\uc6b4\ub4dc, FG 43.4% 3P 39.6%, FT 92.1%\ub85c \ub5a8\uc5b4\uc84c\ub2e4. \ucee4\ub9ac\uc5b4 \uccab \ud50c\ub808\uc774\uc624\ud504\uc600\uace0 \uc778\uc0ac\uc774\ub4dc\uc758 \ud575\uc2ec \ub370\uc774\ube57 \ub9ac\uac00 \ubd80\uc0c1 \ub54c\ubb38\uc5d0 \uc815\uc0c1 \ucd9c\uc804\uc744 \ubabb\ud55c \uc810\uc744 \uac10\uc548\ud558\uba74 \ub098\uc058\uc9c0 \uc54a\uc740 \uc131\uc801\uc774\uae34 \ud558\uc9c0\ub9cc.", "answer": "\ucee4\ub9ac", "sentence": "\ucee4\ub9ac \uc758 \uc131\uacf5\uc801\uc778 \ubcf5\uadc0\uc640 \ud568\uaed8 2\ub144\ucc28 \ub274\ube44 \uac10\ub3c5 \ub9c8\ud06c \uc7ad\uc2a8, \ubcf8\uc778 \uc2a4\ud0ef\ub9cc \uc88b\uace0 \ud300\uc740", "paragraph_sentence": " \ucee4\ub9ac \uc758 \uc131\uacf5\uc801\uc778 \ubcf5\uadc0\uc640 \ud568\uaed8 2\ub144\ucc28 \ub274\ube44 \uac10\ub3c5 \ub9c8\ud06c \uc7ad\uc2a8, \ubcf8\uc778 \uc2a4\ud0ef\ub9cc \uc88b\uace0 \ud300\uc740 \ub9e8\ub0a0 \uc9c4\ub2e4\ub294 \ube44\uc544\ub0e5\uc744 \ubc1b\ub358 PF \ub370\uc774\ube44\ub4dc \ub9ac, \uc800\ub2c8\ub9e8\uc774\uc5c8\ub358 \uc7ac\ub7ff \uc7ad, \uc2e0\uc778 \uc288\ud130 \ud074\ub808\uc774 \ud1b0\uc2a8 \ub4f1\uc758 \uc2dc\ub108\uc9c0\uc5d0 \ud798\uc785\uc5b4 1994\ub144 \uc774\ub798 \ud50c\ub808\uc774\uc624\ud504 \uc9c4\ucd9c\uc744 \ub531 \ud55c\ubc88\ubc16\uc5d0 \ud574\ubcf8 \uc801\uc774 \uc5c6\ub294(2006\ub144) \uc548\uc2b5\uc758 \ud300 \uace8\ub4e0\uc2a4\ud14c\uc774\ud2b8 \uc6cc\ub9ac\uc5b4\uc2a4\ub294 47\uc2b9 35\ud328\ub85c \ud50c\ub808\uc774\uc624\ud504\uc5d0 \uc9c4\ucd9c\ud558\ub294 \ucf8c\uac70\ub97c \uc774\ub8ec\ub2e4. \ub374\ubc84 \ub108\uac8c\uce20\uc640\uc758 \ud50c\ub808\uc774\uc624\ud504 1\ub77c\uc6b4\ub4dc \ucd1d 6\uacbd\uae30\uc5d0\uc11c \ucee4\ub9ac\ub294 \ud3c9\uade0 24.3\uc810 9.3\uc5b4\uc2dc\uc2a4\ud2b8 4.3\ub9ac\ubc14\uc6b4\ub4dc 2.2\uc2a4\ud2f8, FG 46.8% 3P 43.4% FT(\uc790\uc720\ud22c) 100%(...)\ub77c\ub294 \uc5c4\uccad\ub09c \ud65c\uc57d\uc744 \ud3bc\uce58\uba70 4\uc2b9 2\ud328\ub85c \ud300\uc744 2\ub77c\uc6b4\ub4dc\uc5d0 \uc9c4\ucd9c\uc2dc\ud0a4\ub294 \uc77c\ub4f1\uacf5\uc2e0\uc774 \ub418\uc5c8\ub2e4. 2\ub77c\uc6b4\ub4dc\uc758 \uc0c1\ub300\ub294 \uc804\ud1b5\uc758 \uac15\ud300 \uc0cc\uc548\ud1a0\ub2c8\uc624 \uc2a4\ud37c\uc2a4. \uace8\ub4e0\uc2a4\ud14c\uc774\ud2b8\ub294 2\uc2b9 2\ud328\uae4c\uc9c0 \ub530\ub77c\uac00\uba70 \uc120\uc804\ud588\uc73c\ub098, 5, 6 \uacbd\uae30\ub97c \uc5f0\uc774\uc5b4 \ub0b4\uc8fc\uba70 2\uc2b9 4\ud328\ub85c \uc544\uc26c\uc6b4 \ud0c8\ub77d\uc744 \ub9db\ubcf8\ub2e4. \ucee4\ub9ac\ub294 1\uacbd\uae30\uc5d0\uc11c 44\uc810 11\uc5b4\uc2dc\uc2a4\ud2b8(3\uc810\uc29b 42.9%)\ub97c \uae30\ub85d\ud558\uba70 \uc13c\uc138\uc774\uc158\uc744 \uc774\uc5b4\uac08 \uac83\ucc98\ub7fc \ubcf4\uc600\uc73c\ub098, 2\uacbd\uae30\ubd80\ud130 \uc2a4\ud37c\uc2a4\uc758 \uc218\ube44\uc5d0 \ucca0\uc800\ud788 \ubb36\uc774\uba70 \uae30\ub300\ub9cc\ud07c\uc758 \ud65c\uc57d\uc744 \ubcf4\uc5ec\uc8fc\uc9c0 \ubabb\ud588\ub2e4. 2\ub77c\uc6b4\ub4dc\uc5d0\uc11c\uc758 \ubd80\uc9c4\uc73c\ub85c \ud50c\ub808\uc774\uc624\ud504 \uc804\uccb4 \uc131\uc801\uc740 23.4\uc810 8.1\uc5b4\uc2dc\uc2a4\ud2b8 3.8\ub9ac\ubc14\uc6b4\ub4dc, FG 43.4% 3P 39.6%, FT 92.1%\ub85c \ub5a8\uc5b4\uc84c\ub2e4. \ucee4\ub9ac\uc5b4 \uccab \ud50c\ub808\uc774\uc624\ud504\uc600\uace0 \uc778\uc0ac\uc774\ub4dc\uc758 \ud575\uc2ec \ub370\uc774\ube57 \ub9ac\uac00 \ubd80\uc0c1 \ub54c\ubb38\uc5d0 \uc815\uc0c1 \ucd9c\uc804\uc744 \ubabb\ud55c \uc810\uc744 \uac10\uc548\ud558\uba74 \ub098\uc058\uc9c0 \uc54a\uc740 \uc131\uc801\uc774\uae34 \ud558\uc9c0\ub9cc.", "paragraph_answer": " \ucee4\ub9ac \uc758 \uc131\uacf5\uc801\uc778 \ubcf5\uadc0\uc640 \ud568\uaed8 2\ub144\ucc28 \ub274\ube44 \uac10\ub3c5 \ub9c8\ud06c \uc7ad\uc2a8, \ubcf8\uc778 \uc2a4\ud0ef\ub9cc \uc88b\uace0 \ud300\uc740 \ub9e8\ub0a0 \uc9c4\ub2e4\ub294 \ube44\uc544\ub0e5\uc744 \ubc1b\ub358 PF \ub370\uc774\ube44\ub4dc \ub9ac, \uc800\ub2c8\ub9e8\uc774\uc5c8\ub358 \uc7ac\ub7ff \uc7ad, \uc2e0\uc778 \uc288\ud130 \ud074\ub808\uc774 \ud1b0\uc2a8 \ub4f1\uc758 \uc2dc\ub108\uc9c0\uc5d0 \ud798\uc785\uc5b4 1994\ub144 \uc774\ub798 \ud50c\ub808\uc774\uc624\ud504 \uc9c4\ucd9c\uc744 \ub531 \ud55c\ubc88\ubc16\uc5d0 \ud574\ubcf8 \uc801\uc774 \uc5c6\ub294(2006\ub144) \uc548\uc2b5\uc758 \ud300 \uace8\ub4e0\uc2a4\ud14c\uc774\ud2b8 \uc6cc\ub9ac\uc5b4\uc2a4\ub294 47\uc2b9 35\ud328\ub85c \ud50c\ub808\uc774\uc624\ud504\uc5d0 \uc9c4\ucd9c\ud558\ub294 \ucf8c\uac70\ub97c \uc774\ub8ec\ub2e4. \ub374\ubc84 \ub108\uac8c\uce20\uc640\uc758 \ud50c\ub808\uc774\uc624\ud504 1\ub77c\uc6b4\ub4dc \ucd1d 6\uacbd\uae30\uc5d0\uc11c \ucee4\ub9ac\ub294 \ud3c9\uade0 24.3\uc810 9.3\uc5b4\uc2dc\uc2a4\ud2b8 4.3\ub9ac\ubc14\uc6b4\ub4dc 2.2\uc2a4\ud2f8, FG 46.8% 3P 43.4% FT(\uc790\uc720\ud22c) 100%(...)\ub77c\ub294 \uc5c4\uccad\ub09c \ud65c\uc57d\uc744 \ud3bc\uce58\uba70 4\uc2b9 2\ud328\ub85c \ud300\uc744 2\ub77c\uc6b4\ub4dc\uc5d0 \uc9c4\ucd9c\uc2dc\ud0a4\ub294 \uc77c\ub4f1\uacf5\uc2e0\uc774 \ub418\uc5c8\ub2e4. 2\ub77c\uc6b4\ub4dc\uc758 \uc0c1\ub300\ub294 \uc804\ud1b5\uc758 \uac15\ud300 \uc0cc\uc548\ud1a0\ub2c8\uc624 \uc2a4\ud37c\uc2a4. \uace8\ub4e0\uc2a4\ud14c\uc774\ud2b8\ub294 2\uc2b9 2\ud328\uae4c\uc9c0 \ub530\ub77c\uac00\uba70 \uc120\uc804\ud588\uc73c\ub098, 5, 6 \uacbd\uae30\ub97c \uc5f0\uc774\uc5b4 \ub0b4\uc8fc\uba70 2\uc2b9 4\ud328\ub85c \uc544\uc26c\uc6b4 \ud0c8\ub77d\uc744 \ub9db\ubcf8\ub2e4. \ucee4\ub9ac\ub294 1\uacbd\uae30\uc5d0\uc11c 44\uc810 11\uc5b4\uc2dc\uc2a4\ud2b8(3\uc810\uc29b 42.9%)\ub97c \uae30\ub85d\ud558\uba70 \uc13c\uc138\uc774\uc158\uc744 \uc774\uc5b4\uac08 \uac83\ucc98\ub7fc \ubcf4\uc600\uc73c\ub098, 2\uacbd\uae30\ubd80\ud130 \uc2a4\ud37c\uc2a4\uc758 \uc218\ube44\uc5d0 \ucca0\uc800\ud788 \ubb36\uc774\uba70 \uae30\ub300\ub9cc\ud07c\uc758 \ud65c\uc57d\uc744 \ubcf4\uc5ec\uc8fc\uc9c0 \ubabb\ud588\ub2e4. 2\ub77c\uc6b4\ub4dc\uc5d0\uc11c\uc758 \ubd80\uc9c4\uc73c\ub85c \ud50c\ub808\uc774\uc624\ud504 \uc804\uccb4 \uc131\uc801\uc740 23.4\uc810 8.1\uc5b4\uc2dc\uc2a4\ud2b8 3.8\ub9ac\ubc14\uc6b4\ub4dc, FG 43.4% 3P 39.6%, FT 92.1%\ub85c \ub5a8\uc5b4\uc84c\ub2e4. \ucee4\ub9ac\uc5b4 \uccab \ud50c\ub808\uc774\uc624\ud504\uc600\uace0 \uc778\uc0ac\uc774\ub4dc\uc758 \ud575\uc2ec \ub370\uc774\ube57 \ub9ac\uac00 \ubd80\uc0c1 \ub54c\ubb38\uc5d0 \uc815\uc0c1 \ucd9c\uc804\uc744 \ubabb\ud55c \uc810\uc744 \uac10\uc548\ud558\uba74 \ub098\uc058\uc9c0 \uc54a\uc740 \uc131\uc801\uc774\uae34 \ud558\uc9c0\ub9cc.", "sentence_answer": " \ucee4\ub9ac \uc758 \uc131\uacf5\uc801\uc778 \ubcf5\uadc0\uc640 \ud568\uaed8 2\ub144\ucc28 \ub274\ube44 \uac10\ub3c5 \ub9c8\ud06c \uc7ad\uc2a8, \ubcf8\uc778 \uc2a4\ud0ef\ub9cc \uc88b\uace0 \ud300\uc740", "paragraph_id": "6532884-2-2", "paragraph_question": "question: \ud50c\ub808\uc774\uc624\ud504 1\ub77c\uc6b4\ub4dc\uc5d0\uc11c \uc5c4\uccad\ub09c \ud65c\uc57d\uc744 \ud3bc\uccd0 \uace8\ub4e0\uc2a4\ud14c\uc774\ud2b8 \uc6cc\ub9ac\uc5b4\uc2a4\uc758 2\ub77c\uc6b4\ub4dc \uc9c4\ucd9c\uc744 \uacac\uc778\ud55c \uc120\uc218\ub294?, context: \ucee4\ub9ac\uc758 \uc131\uacf5\uc801\uc778 \ubcf5\uadc0\uc640 \ud568\uaed8 2\ub144\ucc28 \ub274\ube44 \uac10\ub3c5 \ub9c8\ud06c \uc7ad\uc2a8, \ubcf8\uc778 \uc2a4\ud0ef\ub9cc \uc88b\uace0 \ud300\uc740 \ub9e8\ub0a0 \uc9c4\ub2e4\ub294 \ube44\uc544\ub0e5\uc744 \ubc1b\ub358 PF \ub370\uc774\ube44\ub4dc \ub9ac, \uc800\ub2c8\ub9e8\uc774\uc5c8\ub358 \uc7ac\ub7ff \uc7ad, \uc2e0\uc778 \uc288\ud130 \ud074\ub808\uc774 \ud1b0\uc2a8 \ub4f1\uc758 \uc2dc\ub108\uc9c0\uc5d0 \ud798\uc785\uc5b4 1994\ub144 \uc774\ub798 \ud50c\ub808\uc774\uc624\ud504 \uc9c4\ucd9c\uc744 \ub531 \ud55c\ubc88\ubc16\uc5d0 \ud574\ubcf8 \uc801\uc774 \uc5c6\ub294(2006\ub144) \uc548\uc2b5\uc758 \ud300 \uace8\ub4e0\uc2a4\ud14c\uc774\ud2b8 \uc6cc\ub9ac\uc5b4\uc2a4\ub294 47\uc2b9 35\ud328\ub85c \ud50c\ub808\uc774\uc624\ud504\uc5d0 \uc9c4\ucd9c\ud558\ub294 \ucf8c\uac70\ub97c \uc774\ub8ec\ub2e4. \ub374\ubc84 \ub108\uac8c\uce20\uc640\uc758 \ud50c\ub808\uc774\uc624\ud504 1\ub77c\uc6b4\ub4dc \ucd1d 6\uacbd\uae30\uc5d0\uc11c \ucee4\ub9ac\ub294 \ud3c9\uade0 24.3\uc810 9.3\uc5b4\uc2dc\uc2a4\ud2b8 4.3\ub9ac\ubc14\uc6b4\ub4dc 2.2\uc2a4\ud2f8, FG 46.8% 3P 43.4% FT(\uc790\uc720\ud22c) 100%(...)\ub77c\ub294 \uc5c4\uccad\ub09c \ud65c\uc57d\uc744 \ud3bc\uce58\uba70 4\uc2b9 2\ud328\ub85c \ud300\uc744 2\ub77c\uc6b4\ub4dc\uc5d0 \uc9c4\ucd9c\uc2dc\ud0a4\ub294 \uc77c\ub4f1\uacf5\uc2e0\uc774 \ub418\uc5c8\ub2e4. 2\ub77c\uc6b4\ub4dc\uc758 \uc0c1\ub300\ub294 \uc804\ud1b5\uc758 \uac15\ud300 \uc0cc\uc548\ud1a0\ub2c8\uc624 \uc2a4\ud37c\uc2a4. \uace8\ub4e0\uc2a4\ud14c\uc774\ud2b8\ub294 2\uc2b9 2\ud328\uae4c\uc9c0 \ub530\ub77c\uac00\uba70 \uc120\uc804\ud588\uc73c\ub098, 5, 6 \uacbd\uae30\ub97c \uc5f0\uc774\uc5b4 \ub0b4\uc8fc\uba70 2\uc2b9 4\ud328\ub85c \uc544\uc26c\uc6b4 \ud0c8\ub77d\uc744 \ub9db\ubcf8\ub2e4. \ucee4\ub9ac\ub294 1\uacbd\uae30\uc5d0\uc11c 44\uc810 11\uc5b4\uc2dc\uc2a4\ud2b8(3\uc810\uc29b 42.9%)\ub97c \uae30\ub85d\ud558\uba70 \uc13c\uc138\uc774\uc158\uc744 \uc774\uc5b4\uac08 \uac83\ucc98\ub7fc \ubcf4\uc600\uc73c\ub098, 2\uacbd\uae30\ubd80\ud130 \uc2a4\ud37c\uc2a4\uc758 \uc218\ube44\uc5d0 \ucca0\uc800\ud788 \ubb36\uc774\uba70 \uae30\ub300\ub9cc\ud07c\uc758 \ud65c\uc57d\uc744 \ubcf4\uc5ec\uc8fc\uc9c0 \ubabb\ud588\ub2e4. 2\ub77c\uc6b4\ub4dc\uc5d0\uc11c\uc758 \ubd80\uc9c4\uc73c\ub85c \ud50c\ub808\uc774\uc624\ud504 \uc804\uccb4 \uc131\uc801\uc740 23.4\uc810 8.1\uc5b4\uc2dc\uc2a4\ud2b8 3.8\ub9ac\ubc14\uc6b4\ub4dc, FG 43.4% 3P 39.6%, FT 92.1%\ub85c \ub5a8\uc5b4\uc84c\ub2e4. \ucee4\ub9ac\uc5b4 \uccab \ud50c\ub808\uc774\uc624\ud504\uc600\uace0 \uc778\uc0ac\uc774\ub4dc\uc758 \ud575\uc2ec \ub370\uc774\ube57 \ub9ac\uac00 \ubd80\uc0c1 \ub54c\ubb38\uc5d0 \uc815\uc0c1 \ucd9c\uc804\uc744 \ubabb\ud55c \uc810\uc744 \uac10\uc548\ud558\uba74 \ub098\uc058\uc9c0 \uc54a\uc740 \uc131\uc801\uc774\uae34 \ud558\uc9c0\ub9cc."} {"question": "2003\ub144 \uc0ac\uc5c5 \uc2dc\uc791\uc2dc \ub18d\ub9bc\uc218\uc0b0\uc2dd\ud488\ubd80\uc758 \uc57d\uc815\uba74\uc801\uc740 \uba87ha\uc778\uac00?", "paragraph": "\ub9cc\uc57d 3\ub144\uac04 \ubcbc\ub098 \uae30\ud0c0 \uc0c1\uc5c5\uc801 \uc791\ubb3c\uc744 \uc7ac\ubc30\ud558\uc9c0 \ub9d0\uc544\uc57c \ud55c\ub2e4\ub294 \uc57d\uc815\uc744 \uc704\ubc18\ud558\uc600\uc744 \ub54c\uc5d0\ub294 \uc81c\uc7ac\uc870\uce58\uac00 \ucde8\ud558\uc5ec\uc84c\ub2e4. \ubcbc\ub97c \uc7ac\ubc30\ud560 \uacbd\uc6b0\uc5d0\ub294 \ud574\ub2f9\uba74\uc801\ub9cc\ud07c \ubcf4\uc870\uae08\uc744 \uc9c0\uae09\ud558\uc9c0 \uc54a\uc73c\uba70 \ud5a5\ud6c4 \uc0dd\uc0b0\uc870\uc815\uc81c \ub300\uc0c1\uc5d0\uc11c \uc81c\uc678\ub418\uace0 \ub2f9\ucd08 \uc57d\uc815\uc794\uc5ec\uae30\uac04 \ub3d9\uc548 \uc300\uc18c\ub4dd\ubcf4\uc804\uc9c1\ubd88\uc81c \ub300\uc0c1\uc5d0\uc11c \uc81c\uc678\ub418\uc5c8\ub2e4. \uadf8\ub9ac\uace0 \uc0c1\uc5c5\uc801 \uc791\ubb3c\uc744 \uc7ac\ubc30\ud560 \uacbd\uc6b0\uc5d0\ub294 \ud574\ub2f9\uba74\uc801\ub9cc\ud07c \ubcf4\uc870\uae08\uc774 \uc9c0\uae09\ub418\uc9c0 \uc54a\uc73c\uba70 \uc794\uc5ec\uae30\uac04 \ub3d9\uc548 \uc7ac\ucc28 \uc0c1\uc5c5\uc801 \uc791\ubb3c\uc744 \uc7ac\ubc30\ud558\uba74 \uc0dd\uc0b0\uc870\uc815\uc81c \ub300\uc0c1\uc5d0\uc11c \uc81c\uc678\ub418\uc5c8\ub2e4. \ub18d\ub9bc\uc218\uc0b0\uc2dd\ud488\ubd80\ub294 \uc0dd\uc0b0\uc870\uc815 \uba74\uc801\uc744 \ub2f9\ucd08 \uc804\uccb4 \ubcbc\uc7ac\ubc30\uba74\uc801\uc758 2.6%\uc778 27,500ha\ub85c \uacc4\ud68d\ud558\uc600\ub294\ub370, 2003\ub144 \uc2e4\uc81c \uc57d\uc815\uba74\uc801\uc740 27,52900ha, \uc57d\uc815\ub18d\uac00\ub294 76,565\ud638\ub85c \uc0ac\uc5c5\uc774 \uc2dc\uc791\ub418\uc5c8\ub294\ub370 \uc57d\uc815\uc704\ubc18 \ub4f1\uc73c\ub85c \uc57d\uc815\uc774 \ud574\uc9c0\ub418\uac70\ub098 \ud558\uc5ec \uc0ac\uc5c5 \ucd5c\uc885\uc5f0\ub3c4\uc778 2005\ub144 \uc57d\uc815\uba74\uc801\uc740 23,429ha, \uc57d\uc815\ub18d\uac00 67,910\ud638\ub85c \uc904\uc5b4\ub4e4\uc5c8\ub2e4.", "answer": "27,52900ha", "sentence": "\ub18d\ub9bc\uc218\uc0b0\uc2dd\ud488\ubd80\ub294 \uc0dd\uc0b0\uc870\uc815 \uba74\uc801\uc744 \ub2f9\ucd08 \uc804\uccb4 \ubcbc\uc7ac\ubc30\uba74\uc801\uc758 2.6%\uc778 27,500ha\ub85c \uacc4\ud68d\ud558\uc600\ub294\ub370, 2003\ub144 \uc2e4\uc81c \uc57d\uc815\uba74\uc801\uc740 27,52900ha ,", "paragraph_sentence": "\ub9cc\uc57d 3\ub144\uac04 \ubcbc\ub098 \uae30\ud0c0 \uc0c1\uc5c5\uc801 \uc791\ubb3c\uc744 \uc7ac\ubc30\ud558\uc9c0 \ub9d0\uc544\uc57c \ud55c\ub2e4\ub294 \uc57d\uc815\uc744 \uc704\ubc18\ud558\uc600\uc744 \ub54c\uc5d0\ub294 \uc81c\uc7ac\uc870\uce58\uac00 \ucde8\ud558\uc5ec\uc84c\ub2e4. \ubcbc\ub97c \uc7ac\ubc30\ud560 \uacbd\uc6b0\uc5d0\ub294 \ud574\ub2f9\uba74\uc801\ub9cc\ud07c \ubcf4\uc870\uae08\uc744 \uc9c0\uae09\ud558\uc9c0 \uc54a\uc73c\uba70 \ud5a5\ud6c4 \uc0dd\uc0b0\uc870\uc815\uc81c \ub300\uc0c1\uc5d0\uc11c \uc81c\uc678\ub418\uace0 \ub2f9\ucd08 \uc57d\uc815\uc794\uc5ec\uae30\uac04 \ub3d9\uc548 \uc300\uc18c\ub4dd\ubcf4\uc804\uc9c1\ubd88\uc81c \ub300\uc0c1\uc5d0\uc11c \uc81c\uc678\ub418\uc5c8\ub2e4. \uadf8\ub9ac\uace0 \uc0c1\uc5c5\uc801 \uc791\ubb3c\uc744 \uc7ac\ubc30\ud560 \uacbd\uc6b0\uc5d0\ub294 \ud574\ub2f9\uba74\uc801\ub9cc\ud07c \ubcf4\uc870\uae08\uc774 \uc9c0\uae09\ub418\uc9c0 \uc54a\uc73c\uba70 \uc794\uc5ec\uae30\uac04 \ub3d9\uc548 \uc7ac\ucc28 \uc0c1\uc5c5\uc801 \uc791\ubb3c\uc744 \uc7ac\ubc30\ud558\uba74 \uc0dd\uc0b0\uc870\uc815\uc81c \ub300\uc0c1\uc5d0\uc11c \uc81c\uc678\ub418\uc5c8\ub2e4. \ub18d\ub9bc\uc218\uc0b0\uc2dd\ud488\ubd80\ub294 \uc0dd\uc0b0\uc870\uc815 \uba74\uc801\uc744 \ub2f9\ucd08 \uc804\uccb4 \ubcbc\uc7ac\ubc30\uba74\uc801\uc758 2.6%\uc778 27,500ha\ub85c \uacc4\ud68d\ud558\uc600\ub294\ub370, 2003\ub144 \uc2e4\uc81c \uc57d\uc815\uba74\uc801\uc740 27,52900ha , \uc57d\uc815\ub18d\uac00\ub294 76,565\ud638\ub85c \uc0ac\uc5c5\uc774 \uc2dc\uc791\ub418\uc5c8\ub294\ub370 \uc57d\uc815\uc704\ubc18 \ub4f1\uc73c\ub85c \uc57d\uc815\uc774 \ud574\uc9c0\ub418\uac70\ub098 \ud558\uc5ec \uc0ac\uc5c5 \ucd5c\uc885\uc5f0\ub3c4\uc778 2005\ub144 \uc57d\uc815\uba74\uc801\uc740 23,429ha, \uc57d\uc815\ub18d\uac00 67,910\ud638\ub85c \uc904\uc5b4\ub4e4\uc5c8\ub2e4.", "paragraph_answer": "\ub9cc\uc57d 3\ub144\uac04 \ubcbc\ub098 \uae30\ud0c0 \uc0c1\uc5c5\uc801 \uc791\ubb3c\uc744 \uc7ac\ubc30\ud558\uc9c0 \ub9d0\uc544\uc57c \ud55c\ub2e4\ub294 \uc57d\uc815\uc744 \uc704\ubc18\ud558\uc600\uc744 \ub54c\uc5d0\ub294 \uc81c\uc7ac\uc870\uce58\uac00 \ucde8\ud558\uc5ec\uc84c\ub2e4. \ubcbc\ub97c \uc7ac\ubc30\ud560 \uacbd\uc6b0\uc5d0\ub294 \ud574\ub2f9\uba74\uc801\ub9cc\ud07c \ubcf4\uc870\uae08\uc744 \uc9c0\uae09\ud558\uc9c0 \uc54a\uc73c\uba70 \ud5a5\ud6c4 \uc0dd\uc0b0\uc870\uc815\uc81c \ub300\uc0c1\uc5d0\uc11c \uc81c\uc678\ub418\uace0 \ub2f9\ucd08 \uc57d\uc815\uc794\uc5ec\uae30\uac04 \ub3d9\uc548 \uc300\uc18c\ub4dd\ubcf4\uc804\uc9c1\ubd88\uc81c \ub300\uc0c1\uc5d0\uc11c \uc81c\uc678\ub418\uc5c8\ub2e4. \uadf8\ub9ac\uace0 \uc0c1\uc5c5\uc801 \uc791\ubb3c\uc744 \uc7ac\ubc30\ud560 \uacbd\uc6b0\uc5d0\ub294 \ud574\ub2f9\uba74\uc801\ub9cc\ud07c \ubcf4\uc870\uae08\uc774 \uc9c0\uae09\ub418\uc9c0 \uc54a\uc73c\uba70 \uc794\uc5ec\uae30\uac04 \ub3d9\uc548 \uc7ac\ucc28 \uc0c1\uc5c5\uc801 \uc791\ubb3c\uc744 \uc7ac\ubc30\ud558\uba74 \uc0dd\uc0b0\uc870\uc815\uc81c \ub300\uc0c1\uc5d0\uc11c \uc81c\uc678\ub418\uc5c8\ub2e4. \ub18d\ub9bc\uc218\uc0b0\uc2dd\ud488\ubd80\ub294 \uc0dd\uc0b0\uc870\uc815 \uba74\uc801\uc744 \ub2f9\ucd08 \uc804\uccb4 \ubcbc\uc7ac\ubc30\uba74\uc801\uc758 2.6%\uc778 27,500ha\ub85c \uacc4\ud68d\ud558\uc600\ub294\ub370, 2003\ub144 \uc2e4\uc81c \uc57d\uc815\uba74\uc801\uc740 27,52900ha , \uc57d\uc815\ub18d\uac00\ub294 76,565\ud638\ub85c \uc0ac\uc5c5\uc774 \uc2dc\uc791\ub418\uc5c8\ub294\ub370 \uc57d\uc815\uc704\ubc18 \ub4f1\uc73c\ub85c \uc57d\uc815\uc774 \ud574\uc9c0\ub418\uac70\ub098 \ud558\uc5ec \uc0ac\uc5c5 \ucd5c\uc885\uc5f0\ub3c4\uc778 2005\ub144 \uc57d\uc815\uba74\uc801\uc740 23,429ha, \uc57d\uc815\ub18d\uac00 67,910\ud638\ub85c \uc904\uc5b4\ub4e4\uc5c8\ub2e4.", "sentence_answer": "\ub18d\ub9bc\uc218\uc0b0\uc2dd\ud488\ubd80\ub294 \uc0dd\uc0b0\uc870\uc815 \uba74\uc801\uc744 \ub2f9\ucd08 \uc804\uccb4 \ubcbc\uc7ac\ubc30\uba74\uc801\uc758 2.6%\uc778 27,500ha\ub85c \uacc4\ud68d\ud558\uc600\ub294\ub370, 2003\ub144 \uc2e4\uc81c \uc57d\uc815\uba74\uc801\uc740 27,52900ha ,", "paragraph_id": "6538576-0-0", "paragraph_question": "question: 2003\ub144 \uc0ac\uc5c5 \uc2dc\uc791\uc2dc \ub18d\ub9bc\uc218\uc0b0\uc2dd\ud488\ubd80\uc758 \uc57d\uc815\uba74\uc801\uc740 \uba87ha\uc778\uac00?, context: \ub9cc\uc57d 3\ub144\uac04 \ubcbc\ub098 \uae30\ud0c0 \uc0c1\uc5c5\uc801 \uc791\ubb3c\uc744 \uc7ac\ubc30\ud558\uc9c0 \ub9d0\uc544\uc57c \ud55c\ub2e4\ub294 \uc57d\uc815\uc744 \uc704\ubc18\ud558\uc600\uc744 \ub54c\uc5d0\ub294 \uc81c\uc7ac\uc870\uce58\uac00 \ucde8\ud558\uc5ec\uc84c\ub2e4. \ubcbc\ub97c \uc7ac\ubc30\ud560 \uacbd\uc6b0\uc5d0\ub294 \ud574\ub2f9\uba74\uc801\ub9cc\ud07c \ubcf4\uc870\uae08\uc744 \uc9c0\uae09\ud558\uc9c0 \uc54a\uc73c\uba70 \ud5a5\ud6c4 \uc0dd\uc0b0\uc870\uc815\uc81c \ub300\uc0c1\uc5d0\uc11c \uc81c\uc678\ub418\uace0 \ub2f9\ucd08 \uc57d\uc815\uc794\uc5ec\uae30\uac04 \ub3d9\uc548 \uc300\uc18c\ub4dd\ubcf4\uc804\uc9c1\ubd88\uc81c \ub300\uc0c1\uc5d0\uc11c \uc81c\uc678\ub418\uc5c8\ub2e4. \uadf8\ub9ac\uace0 \uc0c1\uc5c5\uc801 \uc791\ubb3c\uc744 \uc7ac\ubc30\ud560 \uacbd\uc6b0\uc5d0\ub294 \ud574\ub2f9\uba74\uc801\ub9cc\ud07c \ubcf4\uc870\uae08\uc774 \uc9c0\uae09\ub418\uc9c0 \uc54a\uc73c\uba70 \uc794\uc5ec\uae30\uac04 \ub3d9\uc548 \uc7ac\ucc28 \uc0c1\uc5c5\uc801 \uc791\ubb3c\uc744 \uc7ac\ubc30\ud558\uba74 \uc0dd\uc0b0\uc870\uc815\uc81c \ub300\uc0c1\uc5d0\uc11c \uc81c\uc678\ub418\uc5c8\ub2e4. \ub18d\ub9bc\uc218\uc0b0\uc2dd\ud488\ubd80\ub294 \uc0dd\uc0b0\uc870\uc815 \uba74\uc801\uc744 \ub2f9\ucd08 \uc804\uccb4 \ubcbc\uc7ac\ubc30\uba74\uc801\uc758 2.6%\uc778 27,500ha\ub85c \uacc4\ud68d\ud558\uc600\ub294\ub370, 2003\ub144 \uc2e4\uc81c \uc57d\uc815\uba74\uc801\uc740 27,52900ha, \uc57d\uc815\ub18d\uac00\ub294 76,565\ud638\ub85c \uc0ac\uc5c5\uc774 \uc2dc\uc791\ub418\uc5c8\ub294\ub370 \uc57d\uc815\uc704\ubc18 \ub4f1\uc73c\ub85c \uc57d\uc815\uc774 \ud574\uc9c0\ub418\uac70\ub098 \ud558\uc5ec \uc0ac\uc5c5 \ucd5c\uc885\uc5f0\ub3c4\uc778 2005\ub144 \uc57d\uc815\uba74\uc801\uc740 23,429ha, \uc57d\uc815\ub18d\uac00 67,910\ud638\ub85c \uc904\uc5b4\ub4e4\uc5c8\ub2e4."} {"question": "1995\ub144 12\uc6d4 2\uc77c \ubd80\ud130 22\uc77c\uae4c\uc9c0 \ub78c\uc288\ud0c0\uc778\uc774 \ub3c5\uc77c\uc5d0\uc11c \ucd1d \uba87\ubc88\uc758 \uacf5\uc5f0\uc744 \ud558\uc600\ub294\uac00?", "paragraph": "1995\ub144 3\uc6d4, \ud504\ub85c\ub4c0\uc11c \uc81c\uc774\ucf65 \ud5ec\ub108\uc640 \ud568\uaed8 \ub78c\uc288\ud0c0\uc778\uc740 \uadf8\ub4e4\uc758 \uccab \ubc88\uc9f8 \uc815\uc2dd \uc2a4\ud29c\ub514\uc624 \uc568\ubc94\uc778 Herzeleid (\uc601\uc5b4: Heartbreak) \uc758 \uc81c\uc791\uc5d0 \ucc29\uc218\ud55c\ub2e4. \ub78c\uc288\ud0c0\uc778\uc758 \uccab \ubc88\uc9f8 \uc2f1\uae00 Du Riechst So Gut (\uc601\uc5b4: You Smell So Good) \uc740 8\uc6d4 17\uc77c \ubc1c\ub9e4\ub418\uc5c8\uc73c\uba70 Herzeleid \uc568\ubc94\uc740 1995\ub144 9\uc6d4 24\uc77c \ubc1c\ub9e4\ub418\uc5c8\ub2e4. \uac19\uc740\ud574 \ub9d0 \ub78c\uc288\ud0c0\uc778\uc740 Clawfinger\uc640 \ud568\uaed8 \ubc14\ub974\uc0e4\ubc14\uc640 \ud504\ub77c\ud558\ub85c \uacf5\uc5f0 \ud22c\uc5b4\ub97c \ub5a0\ub09c\ub2e4. 12\uc6d4 2\uc77c \ubd80\ud130 22\uc77c\uae4c\uc9c0 \ub78c\uc288\ud0c0\uc778\uc740 \ub3c5\uc77c \ub0b4\uc5d0\uc11c 17\ubc88\uc758 \uacf5\uc5f0\uc744 \ud558\uc600\uc73c\uba70, \uc774\ub294 \ub3c5\uc77c\ub0b4 \ub78c\uc288\ud0c0\uc778\uc758 \uc778\uc9c0\ub3c4\uac00 \uce58\uc19f\ub294 \uacc4\uae30\uac00 \ub41c\ub2e4. 1996\ub144 \ucd08 \ub78c\uc288\ud0c0\uc778\uc740 \uba87 \ubc88 \uc815\ub3c4\uc758 \uacf5\uc5f0 \ud22c\uc5b4\ub97c \ub354 \uac00\uc84c\uc73c\uba70, 1996\ub144 1\uc6d4 8\uc77c \uadf8\ub4e4\uc758 \ub450 \ubc88\uc9f8 \uc2f1\uae00 Seemann (\uc601\uc5b4: Sailor) \ub97c \ub0b4\ub193\uac8c \ub41c\ub2e4.", "answer": "17\ubc88", "sentence": "12\uc6d4 2\uc77c \ubd80\ud130 22\uc77c\uae4c\uc9c0 \ub78c\uc288\ud0c0\uc778\uc740 \ub3c5\uc77c \ub0b4\uc5d0\uc11c 17\ubc88 \uc758 \uacf5\uc5f0\uc744 \ud558\uc600\uc73c\uba70, \uc774\ub294 \ub3c5\uc77c\ub0b4 \ub78c\uc288\ud0c0\uc778\uc758 \uc778\uc9c0\ub3c4\uac00 \uce58\uc19f\ub294 \uacc4\uae30\uac00 \ub41c\ub2e4.", "paragraph_sentence": "1995\ub144 3\uc6d4, \ud504\ub85c\ub4c0\uc11c \uc81c\uc774\ucf65 \ud5ec\ub108\uc640 \ud568\uaed8 \ub78c\uc288\ud0c0\uc778\uc740 \uadf8\ub4e4\uc758 \uccab \ubc88\uc9f8 \uc815\uc2dd \uc2a4\ud29c\ub514\uc624 \uc568\ubc94\uc778 Herzeleid (\uc601\uc5b4: Heartbreak) \uc758 \uc81c\uc791\uc5d0 \ucc29\uc218\ud55c\ub2e4. \ub78c\uc288\ud0c0\uc778\uc758 \uccab \ubc88\uc9f8 \uc2f1\uae00 Du Riechst So Gut (\uc601\uc5b4: You Smell So Good) \uc740 8\uc6d4 17\uc77c \ubc1c\ub9e4\ub418\uc5c8\uc73c\uba70 Herzeleid \uc568\ubc94\uc740 1995\ub144 9\uc6d4 24\uc77c \ubc1c\ub9e4\ub418\uc5c8\ub2e4. \uac19\uc740\ud574 \ub9d0 \ub78c\uc288\ud0c0\uc778\uc740 Clawfinger\uc640 \ud568\uaed8 \ubc14\ub974\uc0e4\ubc14\uc640 \ud504\ub77c\ud558\ub85c \uacf5\uc5f0 \ud22c\uc5b4\ub97c \ub5a0\ub09c\ub2e4. 12\uc6d4 2\uc77c \ubd80\ud130 22\uc77c\uae4c\uc9c0 \ub78c\uc288\ud0c0\uc778\uc740 \ub3c5\uc77c \ub0b4\uc5d0\uc11c 17\ubc88 \uc758 \uacf5\uc5f0\uc744 \ud558\uc600\uc73c\uba70, \uc774\ub294 \ub3c5\uc77c\ub0b4 \ub78c\uc288\ud0c0\uc778\uc758 \uc778\uc9c0\ub3c4\uac00 \uce58\uc19f\ub294 \uacc4\uae30\uac00 \ub41c\ub2e4. 1996\ub144 \ucd08 \ub78c\uc288\ud0c0\uc778\uc740 \uba87 \ubc88 \uc815\ub3c4\uc758 \uacf5\uc5f0 \ud22c\uc5b4\ub97c \ub354 \uac00\uc84c\uc73c\uba70, 1996\ub144 1\uc6d4 8\uc77c \uadf8\ub4e4\uc758 \ub450 \ubc88\uc9f8 \uc2f1\uae00 Seemann (\uc601\uc5b4: Sailor) \ub97c \ub0b4\ub193\uac8c \ub41c\ub2e4.", "paragraph_answer": "1995\ub144 3\uc6d4, \ud504\ub85c\ub4c0\uc11c \uc81c\uc774\ucf65 \ud5ec\ub108\uc640 \ud568\uaed8 \ub78c\uc288\ud0c0\uc778\uc740 \uadf8\ub4e4\uc758 \uccab \ubc88\uc9f8 \uc815\uc2dd \uc2a4\ud29c\ub514\uc624 \uc568\ubc94\uc778 Herzeleid (\uc601\uc5b4: Heartbreak) \uc758 \uc81c\uc791\uc5d0 \ucc29\uc218\ud55c\ub2e4. \ub78c\uc288\ud0c0\uc778\uc758 \uccab \ubc88\uc9f8 \uc2f1\uae00 Du Riechst So Gut (\uc601\uc5b4: You Smell So Good) \uc740 8\uc6d4 17\uc77c \ubc1c\ub9e4\ub418\uc5c8\uc73c\uba70 Herzeleid \uc568\ubc94\uc740 1995\ub144 9\uc6d4 24\uc77c \ubc1c\ub9e4\ub418\uc5c8\ub2e4. \uac19\uc740\ud574 \ub9d0 \ub78c\uc288\ud0c0\uc778\uc740 Clawfinger\uc640 \ud568\uaed8 \ubc14\ub974\uc0e4\ubc14\uc640 \ud504\ub77c\ud558\ub85c \uacf5\uc5f0 \ud22c\uc5b4\ub97c \ub5a0\ub09c\ub2e4. 12\uc6d4 2\uc77c \ubd80\ud130 22\uc77c\uae4c\uc9c0 \ub78c\uc288\ud0c0\uc778\uc740 \ub3c5\uc77c \ub0b4\uc5d0\uc11c 17\ubc88 \uc758 \uacf5\uc5f0\uc744 \ud558\uc600\uc73c\uba70, \uc774\ub294 \ub3c5\uc77c\ub0b4 \ub78c\uc288\ud0c0\uc778\uc758 \uc778\uc9c0\ub3c4\uac00 \uce58\uc19f\ub294 \uacc4\uae30\uac00 \ub41c\ub2e4. 1996\ub144 \ucd08 \ub78c\uc288\ud0c0\uc778\uc740 \uba87 \ubc88 \uc815\ub3c4\uc758 \uacf5\uc5f0 \ud22c\uc5b4\ub97c \ub354 \uac00\uc84c\uc73c\uba70, 1996\ub144 1\uc6d4 8\uc77c \uadf8\ub4e4\uc758 \ub450 \ubc88\uc9f8 \uc2f1\uae00 Seemann (\uc601\uc5b4: Sailor) \ub97c \ub0b4\ub193\uac8c \ub41c\ub2e4.", "sentence_answer": "12\uc6d4 2\uc77c \ubd80\ud130 22\uc77c\uae4c\uc9c0 \ub78c\uc288\ud0c0\uc778\uc740 \ub3c5\uc77c \ub0b4\uc5d0\uc11c 17\ubc88 \uc758 \uacf5\uc5f0\uc744 \ud558\uc600\uc73c\uba70, \uc774\ub294 \ub3c5\uc77c\ub0b4 \ub78c\uc288\ud0c0\uc778\uc758 \uc778\uc9c0\ub3c4\uac00 \uce58\uc19f\ub294 \uacc4\uae30\uac00 \ub41c\ub2e4.", "paragraph_id": "6494900-2-1", "paragraph_question": "question: 1995\ub144 12\uc6d4 2\uc77c \ubd80\ud130 22\uc77c\uae4c\uc9c0 \ub78c\uc288\ud0c0\uc778\uc774 \ub3c5\uc77c\uc5d0\uc11c \ucd1d \uba87\ubc88\uc758 \uacf5\uc5f0\uc744 \ud558\uc600\ub294\uac00?, context: 1995\ub144 3\uc6d4, \ud504\ub85c\ub4c0\uc11c \uc81c\uc774\ucf65 \ud5ec\ub108\uc640 \ud568\uaed8 \ub78c\uc288\ud0c0\uc778\uc740 \uadf8\ub4e4\uc758 \uccab \ubc88\uc9f8 \uc815\uc2dd \uc2a4\ud29c\ub514\uc624 \uc568\ubc94\uc778 Herzeleid (\uc601\uc5b4: Heartbreak) \uc758 \uc81c\uc791\uc5d0 \ucc29\uc218\ud55c\ub2e4. \ub78c\uc288\ud0c0\uc778\uc758 \uccab \ubc88\uc9f8 \uc2f1\uae00 Du Riechst So Gut (\uc601\uc5b4: You Smell So Good) \uc740 8\uc6d4 17\uc77c \ubc1c\ub9e4\ub418\uc5c8\uc73c\uba70 Herzeleid \uc568\ubc94\uc740 1995\ub144 9\uc6d4 24\uc77c \ubc1c\ub9e4\ub418\uc5c8\ub2e4. \uac19\uc740\ud574 \ub9d0 \ub78c\uc288\ud0c0\uc778\uc740 Clawfinger\uc640 \ud568\uaed8 \ubc14\ub974\uc0e4\ubc14\uc640 \ud504\ub77c\ud558\ub85c \uacf5\uc5f0 \ud22c\uc5b4\ub97c \ub5a0\ub09c\ub2e4. 12\uc6d4 2\uc77c \ubd80\ud130 22\uc77c\uae4c\uc9c0 \ub78c\uc288\ud0c0\uc778\uc740 \ub3c5\uc77c \ub0b4\uc5d0\uc11c 17\ubc88\uc758 \uacf5\uc5f0\uc744 \ud558\uc600\uc73c\uba70, \uc774\ub294 \ub3c5\uc77c\ub0b4 \ub78c\uc288\ud0c0\uc778\uc758 \uc778\uc9c0\ub3c4\uac00 \uce58\uc19f\ub294 \uacc4\uae30\uac00 \ub41c\ub2e4. 1996\ub144 \ucd08 \ub78c\uc288\ud0c0\uc778\uc740 \uba87 \ubc88 \uc815\ub3c4\uc758 \uacf5\uc5f0 \ud22c\uc5b4\ub97c \ub354 \uac00\uc84c\uc73c\uba70, 1996\ub144 1\uc6d4 8\uc77c \uadf8\ub4e4\uc758 \ub450 \ubc88\uc9f8 \uc2f1\uae00 Seemann (\uc601\uc5b4: Sailor) \ub97c \ub0b4\ub193\uac8c \ub41c\ub2e4."} {"question": "2003\ub144 12\uc6d4 12\uc77c \uc5f4\ub9b0 \uc785\uc218\ud601 \ub3d5\uae30 \ud589\uc0ac \uc774\ub984\uc740?", "paragraph": "\ubc95\uc6d0 \uac15\uc81c\uc870\uc815\uc5d0 \ub300\ud574 LG \ud2b8\uc708\uc2a4, \uc784\uc218\ud601 \uac00\uc871\ub4e4\uc758 \uc774\uc758 \uc2e0\uccad\uc740 2003\ub144\uc744 \uc9c0\ub098\uac00, 2004\ub144\uc5d0 \ubcf4\uc0c1 \ud569\uc758\ub85c \ub05d\ub0ac\ub2e4. \ubcf4\uc0c1 \ud569\uc758\uae08\uc740 \ub86f\ub370 \uc790\uc774\uc5b8\uce20, LG \ud2b8\uc708\uc2a4\uc5d0\uc11c \uc11c\uc6b8\ub3d9\ubd80\uc9c0\ubc29\ubc95\uc6d0\uc774 \uc81c\uc2dc\ud55c \uac01\uac01 2\uc5b5 2\ucc9c\ub9cc \uc6d0\uacfc 1\uc5b5 \ucc9c\ub9cc \uc6d0\uc758 \ubcf4\uc0c1\uae08\uc548\uc744 \uc784\uc218\ud601 \uac00\uc871\uc774 \uc218\uc6a9\ud568\uc73c\ub85c\uc368 \ub05d\ub098\uac8c \ub418\uc5c8\ub2e4. 8\uc5b5\uacfc, 4\uc5b5 2\ucc9c 6\ubc31\ub9cc \uc6d0\uc5d0\ub294 \ud06c\uac8c \ubabb \ubbf8\uce58\uc9c0\ub9cc, \uc784\uc218\ud601\uc758 \uc544\ubc84\uc9c0\ub294 \uc774 \ud310\uacb0\uc5d0 \ub300\ud574\uc11c '\uc218\ud601\uc774 \ubb38\uc81c\uac00 \ubc95\uc815\uc5d0\uc11c \ub2e4\ub904\uc9c4 \ud6c4 \ud504\ub85c\uc57c\uad6c \uacbd\uae30\uc7a5\uc5d0 \uad6c\uae09\ucc28\uac00 \uc9c1\uc811 \ub4e4\uc5b4\uac00\uace0 \uc751\uae09\uad6c\uc870\uc0ac\uac00 \ubc30\uce58\ub418\ub294 \ub4f1 \uad6c\ub2e8\ub4e4\uc774 \uc120\uc218 \uc548\uc804\uc5d0 \ub354\uc6b1 \uad00\uc2ec\uc744 \uae30\uc6b8\uc774\uac8c \ub41c \uac83 \uac19\ub2e4'\ub77c\uace0 \ub9d0\ud588\uace0, \uc120\uc218\ud611\ub3c4 '\uc774\ubc88 \uc0ac\uac74\uc744 \ud1b5\ud574 \uac01 \uad6c\ub2e8\ub4e4\uc740 \uc120\uc218 \ubc0f \uad00\uc911\ub4e4\uc758 \uc548\uc804\uacfc \uc0dd\uba85 \uc874\uc911\uc744 \uc704\ud574 \ub354\uc6b1 \ub9cc\uc804\uc744 \uae30\ud558\uace0 \uc120\uc218 \uc0c1\ud574\uc640 \uad00\ub828\ud55c \uc81c\ub3c4 \uac1c\uc120\uc774 \ub4a4\ub530\ub77c\uc57c \ub420 \uac83'\uc774\ub77c\uace0 \uacac\ud574\ub97c \ubc1d\ud614\ub2e4. \uc784\uc218\ud601 \uc120\uc218\uc5d0 \ub300\ud55c \ubcf4\uc0c1 \ud569\uc758\uac00 \ub05d\ub09c \ud6c4\uc5d0, \ub86f\ub370 \uc790\uc774\uc5b8\uce20 \uc120\uc218\ub4e4\uc740 5\uc77c \ub4a4\uc5d0, \uc784\uc218\ud601\uc758 \ubcd1\uc0c1\uc5d0 \ucc3e\uc544\uac00, \uc784\uc218\ud601\uc758 \ubd80\uc778\uc744 \uc704\ub85c\ud558\uace0, \uc784\uc218\ud601\uc774 \ubcd1\uc0c1\uc5d0\uc11c \uc77c\uc5b4\ub098\uae30\ub97c \uae30\uc6d0\ud588\ub2e4.\ub86f\ub370 \uc790\uc774\uc5b8\uce20\uc5d0 \ub2f9\uc2dc FA\ub85c \uc785\ub2e8\ud55c \uc9c0 \uc5bc\ub9c8 \uc548 \ub41c \uc815\uc218\uadfc\uc740 \uc784\uc218\ud601\uc744 \uc704\ud574\uc11c 1\ucc9c\ub9cc \uc6d0\uc744 \uce58\ub8cc\ube44\ub85c \ubcf4\ub0c8\uace0, \uba87 \ub2ec \ud6c4, '\uc784\uc218\ud601\uc744 \uc704\ud55c \uce58\ub8cc\ube44 \ub9c8\ub828'\uc774 \uc5b4\ub290\ub367 \ub2e4\uc12f \ubc88\uc9f8\ub97c \ub9de\uac8c \ub418\uc5c8\ub2e4. \ub86f\ub370 \uc790\uc774\uc5b8\uce20 \uc120\uc218\ub2e8 \uc0c1\uc870\ud68c\ub294 \uc784\uc218\ud601\uc744 \uc704\ud574\uc11c \uc77c\uc77c \uace0\uae30\uc9d1 \ud589\uc0ac\ub97c \uc5f4\uc5b4, \uc9c1\uc811 \uc55e\uce58\ub9c8\ub97c \ub450\ub974\uace0 \uc190\ub2d8\ub4e4\uc5d0\uac8c \uc74c\uc2dd\uc744 \uc81c\uacf5\ud558\uba70 \ud604\uc7a5\uc5d0\uc11c \uc989\uc11d \uc0ac\uc778\ubcfc\ub3c4 \ud310\ub9e4\ud558\ub294 \uc2dd\uc73c\ub85c, \uc5bb\uc740 \uae08\uc561\uc744 \uc804\uc561 \uc784\uc218\ud601\uc758 \uce58\ub8cc\ube44\ub85c \uc804\ub2ec\ud588\ub2e4.2004\ub144\uc774 \ub05d\ub098\uae30 \uc804\uc778, 12\uc6d4 12\uc77c\uc5d0\ub294 '\uace0\ub824\ub300 \uc57c\uad6c\uc778\uc758 \ubc24'\uc774 \uc5f4\ub824, \ud22c\ubcd1 \uc911\uc778 \uc804 \ub86f\ub370 \uc784\uc218\ud601 \ub3d5\uae30 \ud589\uc0ac\ub3c4 \ud568\uaed8 \uc5f4\ub9ac\uac8c \ub418\uc5c8\ub2e4.", "answer": "\uace0\ub824\ub300 \uc57c\uad6c\uc778\uc758 \ubc24", "sentence": "\ub86f\ub370 \uc790\uc774\uc5b8\uce20 \uc120\uc218\ub2e8 \uc0c1\uc870\ud68c\ub294 \uc784\uc218\ud601\uc744 \uc704\ud574\uc11c \uc77c\uc77c \uace0\uae30\uc9d1 \ud589\uc0ac\ub97c \uc5f4\uc5b4, \uc9c1\uc811 \uc55e\uce58\ub9c8\ub97c \ub450\ub974\uace0 \uc190\ub2d8\ub4e4\uc5d0\uac8c \uc74c\uc2dd\uc744 \uc81c\uacf5\ud558\uba70 \ud604\uc7a5\uc5d0\uc11c \uc989\uc11d \uc0ac\uc778\ubcfc\ub3c4 \ud310\ub9e4\ud558\ub294 \uc2dd\uc73c\ub85c, \uc5bb\uc740 \uae08\uc561\uc744 \uc804\uc561 \uc784\uc218\ud601\uc758 \uce58\ub8cc\ube44\ub85c \uc804\ub2ec\ud588\ub2e4.2004\ub144\uc774 \ub05d\ub098\uae30 \uc804\uc778, 12\uc6d4 12\uc77c\uc5d0\ub294 ' \uace0\ub824\ub300 \uc57c\uad6c\uc778\uc758 \ubc24 '\uc774 \uc5f4\ub824, \ud22c\ubcd1 \uc911\uc778 \uc804 \ub86f\ub370 \uc784\uc218\ud601 \ub3d5\uae30 \ud589\uc0ac\ub3c4 \ud568\uaed8 \uc5f4\ub9ac\uac8c \ub418\uc5c8\ub2e4.", "paragraph_sentence": "\ubc95\uc6d0 \uac15\uc81c\uc870\uc815\uc5d0 \ub300\ud574 LG \ud2b8\uc708\uc2a4, \uc784\uc218\ud601 \uac00\uc871\ub4e4\uc758 \uc774\uc758 \uc2e0\uccad\uc740 2003\ub144\uc744 \uc9c0\ub098\uac00, 2004\ub144\uc5d0 \ubcf4\uc0c1 \ud569\uc758\ub85c \ub05d\ub0ac\ub2e4. \ubcf4\uc0c1 \ud569\uc758\uae08\uc740 \ub86f\ub370 \uc790\uc774\uc5b8\uce20, LG \ud2b8\uc708\uc2a4\uc5d0\uc11c \uc11c\uc6b8\ub3d9\ubd80\uc9c0\ubc29\ubc95\uc6d0\uc774 \uc81c\uc2dc\ud55c \uac01\uac01 2\uc5b5 2\ucc9c\ub9cc \uc6d0\uacfc 1\uc5b5 \ucc9c\ub9cc \uc6d0\uc758 \ubcf4\uc0c1\uae08\uc548\uc744 \uc784\uc218\ud601 \uac00\uc871\uc774 \uc218\uc6a9\ud568\uc73c\ub85c\uc368 \ub05d\ub098\uac8c \ub418\uc5c8\ub2e4. 8\uc5b5\uacfc, 4\uc5b5 2\ucc9c 6\ubc31\ub9cc \uc6d0\uc5d0\ub294 \ud06c\uac8c \ubabb \ubbf8\uce58\uc9c0\ub9cc, \uc784\uc218\ud601\uc758 \uc544\ubc84\uc9c0\ub294 \uc774 \ud310\uacb0\uc5d0 \ub300\ud574\uc11c '\uc218\ud601\uc774 \ubb38\uc81c\uac00 \ubc95\uc815\uc5d0\uc11c \ub2e4\ub904\uc9c4 \ud6c4 \ud504\ub85c\uc57c\uad6c \uacbd\uae30\uc7a5\uc5d0 \uad6c\uae09\ucc28\uac00 \uc9c1\uc811 \ub4e4\uc5b4\uac00\uace0 \uc751\uae09\uad6c\uc870\uc0ac\uac00 \ubc30\uce58\ub418\ub294 \ub4f1 \uad6c\ub2e8\ub4e4\uc774 \uc120\uc218 \uc548\uc804\uc5d0 \ub354\uc6b1 \uad00\uc2ec\uc744 \uae30\uc6b8\uc774\uac8c \ub41c \uac83 \uac19\ub2e4'\ub77c\uace0 \ub9d0\ud588\uace0, \uc120\uc218\ud611\ub3c4 '\uc774\ubc88 \uc0ac\uac74\uc744 \ud1b5\ud574 \uac01 \uad6c\ub2e8\ub4e4\uc740 \uc120\uc218 \ubc0f \uad00\uc911\ub4e4\uc758 \uc548\uc804\uacfc \uc0dd\uba85 \uc874\uc911\uc744 \uc704\ud574 \ub354\uc6b1 \ub9cc\uc804\uc744 \uae30\ud558\uace0 \uc120\uc218 \uc0c1\ud574\uc640 \uad00\ub828\ud55c \uc81c\ub3c4 \uac1c\uc120\uc774 \ub4a4\ub530\ub77c\uc57c \ub420 \uac83'\uc774\ub77c\uace0 \uacac\ud574\ub97c \ubc1d\ud614\ub2e4. \uc784\uc218\ud601 \uc120\uc218\uc5d0 \ub300\ud55c \ubcf4\uc0c1 \ud569\uc758\uac00 \ub05d\ub09c \ud6c4\uc5d0, \ub86f\ub370 \uc790\uc774\uc5b8\uce20 \uc120\uc218\ub4e4\uc740 5\uc77c \ub4a4\uc5d0, \uc784\uc218\ud601\uc758 \ubcd1\uc0c1\uc5d0 \ucc3e\uc544\uac00, \uc784\uc218\ud601\uc758 \ubd80\uc778\uc744 \uc704\ub85c\ud558\uace0, \uc784\uc218\ud601\uc774 \ubcd1\uc0c1\uc5d0\uc11c \uc77c\uc5b4\ub098\uae30\ub97c \uae30\uc6d0\ud588\ub2e4.\ub86f\ub370 \uc790\uc774\uc5b8\uce20\uc5d0 \ub2f9\uc2dc FA\ub85c \uc785\ub2e8\ud55c \uc9c0 \uc5bc\ub9c8 \uc548 \ub41c \uc815\uc218\uadfc\uc740 \uc784\uc218\ud601\uc744 \uc704\ud574\uc11c 1\ucc9c\ub9cc \uc6d0\uc744 \uce58\ub8cc\ube44\ub85c \ubcf4\ub0c8\uace0, \uba87 \ub2ec \ud6c4, '\uc784\uc218\ud601\uc744 \uc704\ud55c \uce58\ub8cc\ube44 \ub9c8\ub828'\uc774 \uc5b4\ub290\ub367 \ub2e4\uc12f \ubc88\uc9f8\ub97c \ub9de\uac8c \ub418\uc5c8\ub2e4. \ub86f\ub370 \uc790\uc774\uc5b8\uce20 \uc120\uc218\ub2e8 \uc0c1\uc870\ud68c\ub294 \uc784\uc218\ud601\uc744 \uc704\ud574\uc11c \uc77c\uc77c \uace0\uae30\uc9d1 \ud589\uc0ac\ub97c \uc5f4\uc5b4, \uc9c1\uc811 \uc55e\uce58\ub9c8\ub97c \ub450\ub974\uace0 \uc190\ub2d8\ub4e4\uc5d0\uac8c \uc74c\uc2dd\uc744 \uc81c\uacf5\ud558\uba70 \ud604\uc7a5\uc5d0\uc11c \uc989\uc11d \uc0ac\uc778\ubcfc\ub3c4 \ud310\ub9e4\ud558\ub294 \uc2dd\uc73c\ub85c, \uc5bb\uc740 \uae08\uc561\uc744 \uc804\uc561 \uc784\uc218\ud601\uc758 \uce58\ub8cc\ube44\ub85c \uc804\ub2ec\ud588\ub2e4.2004\ub144\uc774 \ub05d\ub098\uae30 \uc804\uc778, 12\uc6d4 12\uc77c\uc5d0\ub294 ' \uace0\ub824\ub300 \uc57c\uad6c\uc778\uc758 \ubc24 '\uc774 \uc5f4\ub824, \ud22c\ubcd1 \uc911\uc778 \uc804 \ub86f\ub370 \uc784\uc218\ud601 \ub3d5\uae30 \ud589\uc0ac\ub3c4 \ud568\uaed8 \uc5f4\ub9ac\uac8c \ub418\uc5c8\ub2e4. ", "paragraph_answer": "\ubc95\uc6d0 \uac15\uc81c\uc870\uc815\uc5d0 \ub300\ud574 LG \ud2b8\uc708\uc2a4, \uc784\uc218\ud601 \uac00\uc871\ub4e4\uc758 \uc774\uc758 \uc2e0\uccad\uc740 2003\ub144\uc744 \uc9c0\ub098\uac00, 2004\ub144\uc5d0 \ubcf4\uc0c1 \ud569\uc758\ub85c \ub05d\ub0ac\ub2e4. \ubcf4\uc0c1 \ud569\uc758\uae08\uc740 \ub86f\ub370 \uc790\uc774\uc5b8\uce20, LG \ud2b8\uc708\uc2a4\uc5d0\uc11c \uc11c\uc6b8\ub3d9\ubd80\uc9c0\ubc29\ubc95\uc6d0\uc774 \uc81c\uc2dc\ud55c \uac01\uac01 2\uc5b5 2\ucc9c\ub9cc \uc6d0\uacfc 1\uc5b5 \ucc9c\ub9cc \uc6d0\uc758 \ubcf4\uc0c1\uae08\uc548\uc744 \uc784\uc218\ud601 \uac00\uc871\uc774 \uc218\uc6a9\ud568\uc73c\ub85c\uc368 \ub05d\ub098\uac8c \ub418\uc5c8\ub2e4. 8\uc5b5\uacfc, 4\uc5b5 2\ucc9c 6\ubc31\ub9cc \uc6d0\uc5d0\ub294 \ud06c\uac8c \ubabb \ubbf8\uce58\uc9c0\ub9cc, \uc784\uc218\ud601\uc758 \uc544\ubc84\uc9c0\ub294 \uc774 \ud310\uacb0\uc5d0 \ub300\ud574\uc11c '\uc218\ud601\uc774 \ubb38\uc81c\uac00 \ubc95\uc815\uc5d0\uc11c \ub2e4\ub904\uc9c4 \ud6c4 \ud504\ub85c\uc57c\uad6c \uacbd\uae30\uc7a5\uc5d0 \uad6c\uae09\ucc28\uac00 \uc9c1\uc811 \ub4e4\uc5b4\uac00\uace0 \uc751\uae09\uad6c\uc870\uc0ac\uac00 \ubc30\uce58\ub418\ub294 \ub4f1 \uad6c\ub2e8\ub4e4\uc774 \uc120\uc218 \uc548\uc804\uc5d0 \ub354\uc6b1 \uad00\uc2ec\uc744 \uae30\uc6b8\uc774\uac8c \ub41c \uac83 \uac19\ub2e4'\ub77c\uace0 \ub9d0\ud588\uace0, \uc120\uc218\ud611\ub3c4 '\uc774\ubc88 \uc0ac\uac74\uc744 \ud1b5\ud574 \uac01 \uad6c\ub2e8\ub4e4\uc740 \uc120\uc218 \ubc0f \uad00\uc911\ub4e4\uc758 \uc548\uc804\uacfc \uc0dd\uba85 \uc874\uc911\uc744 \uc704\ud574 \ub354\uc6b1 \ub9cc\uc804\uc744 \uae30\ud558\uace0 \uc120\uc218 \uc0c1\ud574\uc640 \uad00\ub828\ud55c \uc81c\ub3c4 \uac1c\uc120\uc774 \ub4a4\ub530\ub77c\uc57c \ub420 \uac83'\uc774\ub77c\uace0 \uacac\ud574\ub97c \ubc1d\ud614\ub2e4. \uc784\uc218\ud601 \uc120\uc218\uc5d0 \ub300\ud55c \ubcf4\uc0c1 \ud569\uc758\uac00 \ub05d\ub09c \ud6c4\uc5d0, \ub86f\ub370 \uc790\uc774\uc5b8\uce20 \uc120\uc218\ub4e4\uc740 5\uc77c \ub4a4\uc5d0, \uc784\uc218\ud601\uc758 \ubcd1\uc0c1\uc5d0 \ucc3e\uc544\uac00, \uc784\uc218\ud601\uc758 \ubd80\uc778\uc744 \uc704\ub85c\ud558\uace0, \uc784\uc218\ud601\uc774 \ubcd1\uc0c1\uc5d0\uc11c \uc77c\uc5b4\ub098\uae30\ub97c \uae30\uc6d0\ud588\ub2e4.\ub86f\ub370 \uc790\uc774\uc5b8\uce20\uc5d0 \ub2f9\uc2dc FA\ub85c \uc785\ub2e8\ud55c \uc9c0 \uc5bc\ub9c8 \uc548 \ub41c \uc815\uc218\uadfc\uc740 \uc784\uc218\ud601\uc744 \uc704\ud574\uc11c 1\ucc9c\ub9cc \uc6d0\uc744 \uce58\ub8cc\ube44\ub85c \ubcf4\ub0c8\uace0, \uba87 \ub2ec \ud6c4, '\uc784\uc218\ud601\uc744 \uc704\ud55c \uce58\ub8cc\ube44 \ub9c8\ub828'\uc774 \uc5b4\ub290\ub367 \ub2e4\uc12f \ubc88\uc9f8\ub97c \ub9de\uac8c \ub418\uc5c8\ub2e4. \ub86f\ub370 \uc790\uc774\uc5b8\uce20 \uc120\uc218\ub2e8 \uc0c1\uc870\ud68c\ub294 \uc784\uc218\ud601\uc744 \uc704\ud574\uc11c \uc77c\uc77c \uace0\uae30\uc9d1 \ud589\uc0ac\ub97c \uc5f4\uc5b4, \uc9c1\uc811 \uc55e\uce58\ub9c8\ub97c \ub450\ub974\uace0 \uc190\ub2d8\ub4e4\uc5d0\uac8c \uc74c\uc2dd\uc744 \uc81c\uacf5\ud558\uba70 \ud604\uc7a5\uc5d0\uc11c \uc989\uc11d \uc0ac\uc778\ubcfc\ub3c4 \ud310\ub9e4\ud558\ub294 \uc2dd\uc73c\ub85c, \uc5bb\uc740 \uae08\uc561\uc744 \uc804\uc561 \uc784\uc218\ud601\uc758 \uce58\ub8cc\ube44\ub85c \uc804\ub2ec\ud588\ub2e4.2004\ub144\uc774 \ub05d\ub098\uae30 \uc804\uc778, 12\uc6d4 12\uc77c\uc5d0\ub294 ' \uace0\ub824\ub300 \uc57c\uad6c\uc778\uc758 \ubc24 '\uc774 \uc5f4\ub824, \ud22c\ubcd1 \uc911\uc778 \uc804 \ub86f\ub370 \uc784\uc218\ud601 \ub3d5\uae30 \ud589\uc0ac\ub3c4 \ud568\uaed8 \uc5f4\ub9ac\uac8c \ub418\uc5c8\ub2e4.", "sentence_answer": "\ub86f\ub370 \uc790\uc774\uc5b8\uce20 \uc120\uc218\ub2e8 \uc0c1\uc870\ud68c\ub294 \uc784\uc218\ud601\uc744 \uc704\ud574\uc11c \uc77c\uc77c \uace0\uae30\uc9d1 \ud589\uc0ac\ub97c \uc5f4\uc5b4, \uc9c1\uc811 \uc55e\uce58\ub9c8\ub97c \ub450\ub974\uace0 \uc190\ub2d8\ub4e4\uc5d0\uac8c \uc74c\uc2dd\uc744 \uc81c\uacf5\ud558\uba70 \ud604\uc7a5\uc5d0\uc11c \uc989\uc11d \uc0ac\uc778\ubcfc\ub3c4 \ud310\ub9e4\ud558\ub294 \uc2dd\uc73c\ub85c, \uc5bb\uc740 \uae08\uc561\uc744 \uc804\uc561 \uc784\uc218\ud601\uc758 \uce58\ub8cc\ube44\ub85c \uc804\ub2ec\ud588\ub2e4.2004\ub144\uc774 \ub05d\ub098\uae30 \uc804\uc778, 12\uc6d4 12\uc77c\uc5d0\ub294 ' \uace0\ub824\ub300 \uc57c\uad6c\uc778\uc758 \ubc24 '\uc774 \uc5f4\ub824, \ud22c\ubcd1 \uc911\uc778 \uc804 \ub86f\ub370 \uc784\uc218\ud601 \ub3d5\uae30 \ud589\uc0ac\ub3c4 \ud568\uaed8 \uc5f4\ub9ac\uac8c \ub418\uc5c8\ub2e4.", "paragraph_id": "6176094-6-0", "paragraph_question": "question: 2003\ub144 12\uc6d4 12\uc77c \uc5f4\ub9b0 \uc785\uc218\ud601 \ub3d5\uae30 \ud589\uc0ac \uc774\ub984\uc740?, context: \ubc95\uc6d0 \uac15\uc81c\uc870\uc815\uc5d0 \ub300\ud574 LG \ud2b8\uc708\uc2a4, \uc784\uc218\ud601 \uac00\uc871\ub4e4\uc758 \uc774\uc758 \uc2e0\uccad\uc740 2003\ub144\uc744 \uc9c0\ub098\uac00, 2004\ub144\uc5d0 \ubcf4\uc0c1 \ud569\uc758\ub85c \ub05d\ub0ac\ub2e4. \ubcf4\uc0c1 \ud569\uc758\uae08\uc740 \ub86f\ub370 \uc790\uc774\uc5b8\uce20, LG \ud2b8\uc708\uc2a4\uc5d0\uc11c \uc11c\uc6b8\ub3d9\ubd80\uc9c0\ubc29\ubc95\uc6d0\uc774 \uc81c\uc2dc\ud55c \uac01\uac01 2\uc5b5 2\ucc9c\ub9cc \uc6d0\uacfc 1\uc5b5 \ucc9c\ub9cc \uc6d0\uc758 \ubcf4\uc0c1\uae08\uc548\uc744 \uc784\uc218\ud601 \uac00\uc871\uc774 \uc218\uc6a9\ud568\uc73c\ub85c\uc368 \ub05d\ub098\uac8c \ub418\uc5c8\ub2e4. 8\uc5b5\uacfc, 4\uc5b5 2\ucc9c 6\ubc31\ub9cc \uc6d0\uc5d0\ub294 \ud06c\uac8c \ubabb \ubbf8\uce58\uc9c0\ub9cc, \uc784\uc218\ud601\uc758 \uc544\ubc84\uc9c0\ub294 \uc774 \ud310\uacb0\uc5d0 \ub300\ud574\uc11c '\uc218\ud601\uc774 \ubb38\uc81c\uac00 \ubc95\uc815\uc5d0\uc11c \ub2e4\ub904\uc9c4 \ud6c4 \ud504\ub85c\uc57c\uad6c \uacbd\uae30\uc7a5\uc5d0 \uad6c\uae09\ucc28\uac00 \uc9c1\uc811 \ub4e4\uc5b4\uac00\uace0 \uc751\uae09\uad6c\uc870\uc0ac\uac00 \ubc30\uce58\ub418\ub294 \ub4f1 \uad6c\ub2e8\ub4e4\uc774 \uc120\uc218 \uc548\uc804\uc5d0 \ub354\uc6b1 \uad00\uc2ec\uc744 \uae30\uc6b8\uc774\uac8c \ub41c \uac83 \uac19\ub2e4'\ub77c\uace0 \ub9d0\ud588\uace0, \uc120\uc218\ud611\ub3c4 '\uc774\ubc88 \uc0ac\uac74\uc744 \ud1b5\ud574 \uac01 \uad6c\ub2e8\ub4e4\uc740 \uc120\uc218 \ubc0f \uad00\uc911\ub4e4\uc758 \uc548\uc804\uacfc \uc0dd\uba85 \uc874\uc911\uc744 \uc704\ud574 \ub354\uc6b1 \ub9cc\uc804\uc744 \uae30\ud558\uace0 \uc120\uc218 \uc0c1\ud574\uc640 \uad00\ub828\ud55c \uc81c\ub3c4 \uac1c\uc120\uc774 \ub4a4\ub530\ub77c\uc57c \ub420 \uac83'\uc774\ub77c\uace0 \uacac\ud574\ub97c \ubc1d\ud614\ub2e4. \uc784\uc218\ud601 \uc120\uc218\uc5d0 \ub300\ud55c \ubcf4\uc0c1 \ud569\uc758\uac00 \ub05d\ub09c \ud6c4\uc5d0, \ub86f\ub370 \uc790\uc774\uc5b8\uce20 \uc120\uc218\ub4e4\uc740 5\uc77c \ub4a4\uc5d0, \uc784\uc218\ud601\uc758 \ubcd1\uc0c1\uc5d0 \ucc3e\uc544\uac00, \uc784\uc218\ud601\uc758 \ubd80\uc778\uc744 \uc704\ub85c\ud558\uace0, \uc784\uc218\ud601\uc774 \ubcd1\uc0c1\uc5d0\uc11c \uc77c\uc5b4\ub098\uae30\ub97c \uae30\uc6d0\ud588\ub2e4.\ub86f\ub370 \uc790\uc774\uc5b8\uce20\uc5d0 \ub2f9\uc2dc FA\ub85c \uc785\ub2e8\ud55c \uc9c0 \uc5bc\ub9c8 \uc548 \ub41c \uc815\uc218\uadfc\uc740 \uc784\uc218\ud601\uc744 \uc704\ud574\uc11c 1\ucc9c\ub9cc \uc6d0\uc744 \uce58\ub8cc\ube44\ub85c \ubcf4\ub0c8\uace0, \uba87 \ub2ec \ud6c4, '\uc784\uc218\ud601\uc744 \uc704\ud55c \uce58\ub8cc\ube44 \ub9c8\ub828'\uc774 \uc5b4\ub290\ub367 \ub2e4\uc12f \ubc88\uc9f8\ub97c \ub9de\uac8c \ub418\uc5c8\ub2e4. \ub86f\ub370 \uc790\uc774\uc5b8\uce20 \uc120\uc218\ub2e8 \uc0c1\uc870\ud68c\ub294 \uc784\uc218\ud601\uc744 \uc704\ud574\uc11c \uc77c\uc77c \uace0\uae30\uc9d1 \ud589\uc0ac\ub97c \uc5f4\uc5b4, \uc9c1\uc811 \uc55e\uce58\ub9c8\ub97c \ub450\ub974\uace0 \uc190\ub2d8\ub4e4\uc5d0\uac8c \uc74c\uc2dd\uc744 \uc81c\uacf5\ud558\uba70 \ud604\uc7a5\uc5d0\uc11c \uc989\uc11d \uc0ac\uc778\ubcfc\ub3c4 \ud310\ub9e4\ud558\ub294 \uc2dd\uc73c\ub85c, \uc5bb\uc740 \uae08\uc561\uc744 \uc804\uc561 \uc784\uc218\ud601\uc758 \uce58\ub8cc\ube44\ub85c \uc804\ub2ec\ud588\ub2e4.2004\ub144\uc774 \ub05d\ub098\uae30 \uc804\uc778, 12\uc6d4 12\uc77c\uc5d0\ub294 '\uace0\ub824\ub300 \uc57c\uad6c\uc778\uc758 \ubc24'\uc774 \uc5f4\ub824, \ud22c\ubcd1 \uc911\uc778 \uc804 \ub86f\ub370 \uc784\uc218\ud601 \ub3d5\uae30 \ud589\uc0ac\ub3c4 \ud568\uaed8 \uc5f4\ub9ac\uac8c \ub418\uc5c8\ub2e4."} {"question": "\ubb34\ub77c\ud0c0 \uc870\uc9c0\uac00 \uc740\ud1f4\ud55c \ud574\ub294?", "paragraph": "1989\ub144 5\uc6d4 13\uc77c, \ub2db\ud3f0\ud584 \ud30c\uc774\ud130\uc2a4\uc640\uc758 \uacbd\uae30(\uc57c\ub9c8\uac00\ud0c0 \ud604 \uc57c\uad6c\uc7a5)\uc5d0\uc11c \uac1c\uc778 \ud1b5\uc0b0 200\uc2b9\uc744 \ub2ec\uc131\ud588\ub2e4. 200\uc2b9\uc5d0 \uacb0\uc815\uc801\uc778 \uc218\ub2e8\uc744 \ub3d9\uc6d0\ud588\ub358 4\uc6d4 16\uc77c \uae34\ud14c\uc4f0 \ubc84\ud384\ub85c\uc2a4\uc640\uc758 \uacbd\uae30(\uac00\uc640\uc0ac\ud0a4 \uad6c\uc7a5)\uc5d0\uc11c\ub294 \uc5f0\uc7a5 11\ud68c\ub97c \ub358\uc838\ub0b4\uba74\uc11c \ud328\uc804 \ud22c\uc218\uac00 \ub410\uc9c0\ub9cc \uadf8 \uacbd\uae30\ub97c \ub2db\ud3f0 TV\uac00 \uacbd\uae30 \uc885\ub8cc\ud560 \ub54c\uae4c\uc9c0 \uc911\uacc4\ud558\uc600\uace0 \uc6d0\ub798 \u300a\uc1fc\ud150\u300b\uc774 \ubc29\uc1a1\ub418\ub294 17\uc2dc 20\ubd84\ubd80\ud130 17\uc2dc 39\ubd84\uae4c\uc9c0 \uc2dc\uac04\ub300\uc758 \uc2dc\uccad\ub960\uc740 \uadf8 \ub0a0 \ubc24 \uc694\ubbf8\uc6b0\ub9ac-\ub2e4\uc774\uc694\uc804\uc744 \uc6c3\ub3c4\ub294 22.4%(\uac04\ud1a0 \uc9c0\uad6c)\ub97c \uae30\ub85d\ud588\ub2e4(\uc804\uccb4\uc801\uc73c\ub85c\ub294 10.9%). \uac19\uc740 \ud574 39\uc138\uc758 \ub098\uc774\ub85c \ud1b5\uc0b0 3\ubc88\uc9f8\uc758 \ucd5c\uc6b0\uc218 \ud3c9\uade0 \uc790\ucc45\uc810 \ud0c0\uc774\ud2c0\uc744 \uc11d\uad8c\ud588\ub2e4. \uadf8 \ub2e4\uc74c\ud574\uc778 1990\ub144, 10\uc2b9\uc744 \ub0a8\uae30\uba74\uc11c \uc790\uc2e0\uc73c\ub85c\uc11c\ub294 10\ubc88\uc9f8\uc758 \ub450 \uc790\ub9bf\uc218 \uc2b9\ub9ac\ub97c \uac70\ub450\uc5c8\ub2e4. \uc640\uce74\ubc14\uc57c\uc2dc \ub2e4\ub2e4\uc2dc \uc774\ud6c4 \uc5ed\ub300 2\ubc88\uc9f8\uac00 \ub418\ub294 40\ub300\ub85c\uc11c\uc758 \ub450 \uc790\ub9bf\uc218 \uc2b9\ub9ac\ub97c \uae30\ub85d\ud588\uc9c0\ub9cc \uadf8 \ud574\ub97c \ub9c8\uc9c0\ub9c9\uc73c\ub85c \ud604\uc5ed\uc5d0\uc11c \uc740\ud1f4\ud588\ub2e4. \ub610\ud55c \uc774 \uc740\ud1f4\uc5d0 \uc758\ud574\uc11c 1960\ub144\ub300\uc5d0 \ud504\ub85c\uc5d0 \uc785\ubb38\ud588\ub358 \uc120\uc218\ub4e4 \ubaa8\ub450 \ud604\uc5ed\uc5d0\uc11c \uc740\ud1f4\ud588\ub2e4.", "answer": "1990\ub144", "sentence": "\uadf8 \ub2e4\uc74c\ud574\uc778 1990\ub144 ,", "paragraph_sentence": "1989\ub144 5\uc6d4 13\uc77c, \ub2db\ud3f0\ud584 \ud30c\uc774\ud130\uc2a4\uc640\uc758 \uacbd\uae30(\uc57c\ub9c8\uac00\ud0c0 \ud604 \uc57c\uad6c\uc7a5)\uc5d0\uc11c \uac1c\uc778 \ud1b5\uc0b0 200\uc2b9\uc744 \ub2ec\uc131\ud588\ub2e4. 200\uc2b9\uc5d0 \uacb0\uc815\uc801\uc778 \uc218\ub2e8\uc744 \ub3d9\uc6d0\ud588\ub358 4\uc6d4 16\uc77c \uae34\ud14c\uc4f0 \ubc84\ud384\ub85c\uc2a4\uc640\uc758 \uacbd\uae30(\uac00\uc640\uc0ac\ud0a4 \uad6c\uc7a5)\uc5d0\uc11c\ub294 \uc5f0\uc7a5 11\ud68c\ub97c \ub358\uc838\ub0b4\uba74\uc11c \ud328\uc804 \ud22c\uc218\uac00 \ub410\uc9c0\ub9cc \uadf8 \uacbd\uae30\ub97c \ub2db\ud3f0 TV\uac00 \uacbd\uae30 \uc885\ub8cc\ud560 \ub54c\uae4c\uc9c0 \uc911\uacc4\ud558\uc600\uace0 \uc6d0\ub798 \u300a\uc1fc\ud150\u300b\uc774 \ubc29\uc1a1\ub418\ub294 17\uc2dc 20\ubd84\ubd80\ud130 17\uc2dc 39\ubd84\uae4c\uc9c0 \uc2dc\uac04\ub300\uc758 \uc2dc\uccad\ub960\uc740 \uadf8 \ub0a0 \ubc24 \uc694\ubbf8\uc6b0\ub9ac-\ub2e4\uc774\uc694\uc804\uc744 \uc6c3\ub3c4\ub294 22.4%(\uac04\ud1a0 \uc9c0\uad6c)\ub97c \uae30\ub85d\ud588\ub2e4(\uc804\uccb4\uc801\uc73c\ub85c\ub294 10.9%). \uac19\uc740 \ud574 39\uc138\uc758 \ub098\uc774\ub85c \ud1b5\uc0b0 3\ubc88\uc9f8\uc758 \ucd5c\uc6b0\uc218 \ud3c9\uade0 \uc790\ucc45\uc810 \ud0c0\uc774\ud2c0\uc744 \uc11d\uad8c\ud588\ub2e4. \uadf8 \ub2e4\uc74c\ud574\uc778 1990\ub144 , 10\uc2b9\uc744 \ub0a8\uae30\uba74\uc11c \uc790\uc2e0\uc73c\ub85c\uc11c\ub294 10\ubc88\uc9f8\uc758 \ub450 \uc790\ub9bf\uc218 \uc2b9\ub9ac\ub97c \uac70\ub450\uc5c8\ub2e4. \uc640\uce74\ubc14\uc57c\uc2dc \ub2e4\ub2e4\uc2dc \uc774\ud6c4 \uc5ed\ub300 2\ubc88\uc9f8\uac00 \ub418\ub294 40\ub300\ub85c\uc11c\uc758 \ub450 \uc790\ub9bf\uc218 \uc2b9\ub9ac\ub97c \uae30\ub85d\ud588\uc9c0\ub9cc \uadf8 \ud574\ub97c \ub9c8\uc9c0\ub9c9\uc73c\ub85c \ud604\uc5ed\uc5d0\uc11c \uc740\ud1f4\ud588\ub2e4. \ub610\ud55c \uc774 \uc740\ud1f4\uc5d0 \uc758\ud574\uc11c 1960\ub144\ub300\uc5d0 \ud504\ub85c\uc5d0 \uc785\ubb38\ud588\ub358 \uc120\uc218\ub4e4 \ubaa8\ub450 \ud604\uc5ed\uc5d0\uc11c \uc740\ud1f4\ud588\ub2e4.", "paragraph_answer": "1989\ub144 5\uc6d4 13\uc77c, \ub2db\ud3f0\ud584 \ud30c\uc774\ud130\uc2a4\uc640\uc758 \uacbd\uae30(\uc57c\ub9c8\uac00\ud0c0 \ud604 \uc57c\uad6c\uc7a5)\uc5d0\uc11c \uac1c\uc778 \ud1b5\uc0b0 200\uc2b9\uc744 \ub2ec\uc131\ud588\ub2e4. 200\uc2b9\uc5d0 \uacb0\uc815\uc801\uc778 \uc218\ub2e8\uc744 \ub3d9\uc6d0\ud588\ub358 4\uc6d4 16\uc77c \uae34\ud14c\uc4f0 \ubc84\ud384\ub85c\uc2a4\uc640\uc758 \uacbd\uae30(\uac00\uc640\uc0ac\ud0a4 \uad6c\uc7a5)\uc5d0\uc11c\ub294 \uc5f0\uc7a5 11\ud68c\ub97c \ub358\uc838\ub0b4\uba74\uc11c \ud328\uc804 \ud22c\uc218\uac00 \ub410\uc9c0\ub9cc \uadf8 \uacbd\uae30\ub97c \ub2db\ud3f0 TV\uac00 \uacbd\uae30 \uc885\ub8cc\ud560 \ub54c\uae4c\uc9c0 \uc911\uacc4\ud558\uc600\uace0 \uc6d0\ub798 \u300a\uc1fc\ud150\u300b\uc774 \ubc29\uc1a1\ub418\ub294 17\uc2dc 20\ubd84\ubd80\ud130 17\uc2dc 39\ubd84\uae4c\uc9c0 \uc2dc\uac04\ub300\uc758 \uc2dc\uccad\ub960\uc740 \uadf8 \ub0a0 \ubc24 \uc694\ubbf8\uc6b0\ub9ac-\ub2e4\uc774\uc694\uc804\uc744 \uc6c3\ub3c4\ub294 22.4%(\uac04\ud1a0 \uc9c0\uad6c)\ub97c \uae30\ub85d\ud588\ub2e4(\uc804\uccb4\uc801\uc73c\ub85c\ub294 10.9%). \uac19\uc740 \ud574 39\uc138\uc758 \ub098\uc774\ub85c \ud1b5\uc0b0 3\ubc88\uc9f8\uc758 \ucd5c\uc6b0\uc218 \ud3c9\uade0 \uc790\ucc45\uc810 \ud0c0\uc774\ud2c0\uc744 \uc11d\uad8c\ud588\ub2e4. \uadf8 \ub2e4\uc74c\ud574\uc778 1990\ub144 , 10\uc2b9\uc744 \ub0a8\uae30\uba74\uc11c \uc790\uc2e0\uc73c\ub85c\uc11c\ub294 10\ubc88\uc9f8\uc758 \ub450 \uc790\ub9bf\uc218 \uc2b9\ub9ac\ub97c \uac70\ub450\uc5c8\ub2e4. \uc640\uce74\ubc14\uc57c\uc2dc \ub2e4\ub2e4\uc2dc \uc774\ud6c4 \uc5ed\ub300 2\ubc88\uc9f8\uac00 \ub418\ub294 40\ub300\ub85c\uc11c\uc758 \ub450 \uc790\ub9bf\uc218 \uc2b9\ub9ac\ub97c \uae30\ub85d\ud588\uc9c0\ub9cc \uadf8 \ud574\ub97c \ub9c8\uc9c0\ub9c9\uc73c\ub85c \ud604\uc5ed\uc5d0\uc11c \uc740\ud1f4\ud588\ub2e4. \ub610\ud55c \uc774 \uc740\ud1f4\uc5d0 \uc758\ud574\uc11c 1960\ub144\ub300\uc5d0 \ud504\ub85c\uc5d0 \uc785\ubb38\ud588\ub358 \uc120\uc218\ub4e4 \ubaa8\ub450 \ud604\uc5ed\uc5d0\uc11c \uc740\ud1f4\ud588\ub2e4.", "sentence_answer": "\uadf8 \ub2e4\uc74c\ud574\uc778 1990\ub144 ,", "paragraph_id": "6504766-5-2", "paragraph_question": "question: \ubb34\ub77c\ud0c0 \uc870\uc9c0\uac00 \uc740\ud1f4\ud55c \ud574\ub294?, context: 1989\ub144 5\uc6d4 13\uc77c, \ub2db\ud3f0\ud584 \ud30c\uc774\ud130\uc2a4\uc640\uc758 \uacbd\uae30(\uc57c\ub9c8\uac00\ud0c0 \ud604 \uc57c\uad6c\uc7a5)\uc5d0\uc11c \uac1c\uc778 \ud1b5\uc0b0 200\uc2b9\uc744 \ub2ec\uc131\ud588\ub2e4. 200\uc2b9\uc5d0 \uacb0\uc815\uc801\uc778 \uc218\ub2e8\uc744 \ub3d9\uc6d0\ud588\ub358 4\uc6d4 16\uc77c \uae34\ud14c\uc4f0 \ubc84\ud384\ub85c\uc2a4\uc640\uc758 \uacbd\uae30(\uac00\uc640\uc0ac\ud0a4 \uad6c\uc7a5)\uc5d0\uc11c\ub294 \uc5f0\uc7a5 11\ud68c\ub97c \ub358\uc838\ub0b4\uba74\uc11c \ud328\uc804 \ud22c\uc218\uac00 \ub410\uc9c0\ub9cc \uadf8 \uacbd\uae30\ub97c \ub2db\ud3f0 TV\uac00 \uacbd\uae30 \uc885\ub8cc\ud560 \ub54c\uae4c\uc9c0 \uc911\uacc4\ud558\uc600\uace0 \uc6d0\ub798 \u300a\uc1fc\ud150\u300b\uc774 \ubc29\uc1a1\ub418\ub294 17\uc2dc 20\ubd84\ubd80\ud130 17\uc2dc 39\ubd84\uae4c\uc9c0 \uc2dc\uac04\ub300\uc758 \uc2dc\uccad\ub960\uc740 \uadf8 \ub0a0 \ubc24 \uc694\ubbf8\uc6b0\ub9ac-\ub2e4\uc774\uc694\uc804\uc744 \uc6c3\ub3c4\ub294 22.4%(\uac04\ud1a0 \uc9c0\uad6c)\ub97c \uae30\ub85d\ud588\ub2e4(\uc804\uccb4\uc801\uc73c\ub85c\ub294 10.9%). \uac19\uc740 \ud574 39\uc138\uc758 \ub098\uc774\ub85c \ud1b5\uc0b0 3\ubc88\uc9f8\uc758 \ucd5c\uc6b0\uc218 \ud3c9\uade0 \uc790\ucc45\uc810 \ud0c0\uc774\ud2c0\uc744 \uc11d\uad8c\ud588\ub2e4. \uadf8 \ub2e4\uc74c\ud574\uc778 1990\ub144, 10\uc2b9\uc744 \ub0a8\uae30\uba74\uc11c \uc790\uc2e0\uc73c\ub85c\uc11c\ub294 10\ubc88\uc9f8\uc758 \ub450 \uc790\ub9bf\uc218 \uc2b9\ub9ac\ub97c \uac70\ub450\uc5c8\ub2e4. \uc640\uce74\ubc14\uc57c\uc2dc \ub2e4\ub2e4\uc2dc \uc774\ud6c4 \uc5ed\ub300 2\ubc88\uc9f8\uac00 \ub418\ub294 40\ub300\ub85c\uc11c\uc758 \ub450 \uc790\ub9bf\uc218 \uc2b9\ub9ac\ub97c \uae30\ub85d\ud588\uc9c0\ub9cc \uadf8 \ud574\ub97c \ub9c8\uc9c0\ub9c9\uc73c\ub85c \ud604\uc5ed\uc5d0\uc11c \uc740\ud1f4\ud588\ub2e4. \ub610\ud55c \uc774 \uc740\ud1f4\uc5d0 \uc758\ud574\uc11c 1960\ub144\ub300\uc5d0 \ud504\ub85c\uc5d0 \uc785\ubb38\ud588\ub358 \uc120\uc218\ub4e4 \ubaa8\ub450 \ud604\uc5ed\uc5d0\uc11c \uc740\ud1f4\ud588\ub2e4."} {"question": "\ub370\ub77c\uc6b0\uce58 \ub9c8\uc0ac\ud0c0\ucf00\uc640 \uc774\uc644\uc6a9\uc774 \uad00\uc800 2\uce35\uc5d0\uc11c \uccb4\uacb0\ud55c \uc870\uc57d\uc740?", "paragraph": "1907\ub144 10\uc6d4 16\uc77c\uc5d0 \ub300\ud55c\uc81c\uad6d\uc744 \ubc29\ubb38\ud55c \uc694\uc2dc\ud788\ud1a0 \ud669\ud0dc\uc790\uac00 \uad00\uc800\ub97c 4\uc77c\uac04 \uc219\uc18c\ub85c \uc774\uc6a9\ud558\uc600\ub2e4. 1908\ub144\uc5d0 \ub2e4\uc4f0\ub178 \uae34\uace0(\u8fb0\u91ce\u91d1\u543e)\uac00 \uc0c8\ub85c\uc6b4 \ud1b5\uac10 \uad00\uc800\ub97c \uc124\uacc4\ud558\uc600\uc73c\ub098 \uac19\uc740 \ud574 \uc6a9\uc0b0\uc5d0 \uac00\ud0c0\uc57c\ub9c8\uc758 \ud1b5\uac10 \uad00\uc800\uac00 \ub4e4\uc5b4\uc11c\uba74\uc11c \uacc4\ud68d\uc73c\ub85c \ub05d\ub0ac\ub2e4. 1910\ub144 8\uc6d4 22\uc77c \uc624\ud6c4 4\uc2dc\uc5d0 \uc81c3\ub300 \ud55c\uad6d \ud1b5\uac10 \ub370\ub77c\uc6b0\uce58 \ub9c8\uc0ac\ud0c0\ucf00\uc640 \ub300\ud55c\uc81c\uad6d \ucd1d\ub9ac\ub300\uc2e0 \uc774\uc644\uc6a9\uc774 \uad00\uc800 2\uce35\uc5d0\uc11c \uc744\uc0ac\uc870\uc57d\uc744 \uc870\uc778\ud558\uc600\ub2e4. \uc870\uc120\ucd1d\ub3c5\ubd80 \ucd1d\ub3c5 \uad00\uc800\uac00 \uacbd\ubcf5\uad81 \ub4a4\ud3b8\uc758 \uacbd\ubb34\ub300\ub85c \uc62e\uaca8\uac04 \uc774\ud6c4 \uc65c\uc131\ub300 \ucd1d\ub3c5 \uad00\uc800\ub294 \uc5ed\ub300 \ud1b5\uac10\uacfc \ucd1d\ub3c5\uc758 \ucd08\uc0c1\uacfc \uad00\ub828 \uc720\ubb3c\uc744 \uc804\uc2dc\ud558\ub294 \uc2dc\uc815\uae30\ub150\uad00(\u59cb\u653f\u8a18\u5ff5\u9928)\uc73c\ub85c \uac1c\ud3b8\ub418\uc5b4 1940\ub144 11\uc6d4 22\uc77c\uc5d0 \uac1c\uad00\uc2dd\uc744 \uac70\ud589\ud558\uc600\ub2e4. \uc2dc\uc815\uae30\ub150\uad00\uc73c\ub85c \uac1c\uad00\ud558\uba74\uc11c \uc744\uc0ac\uc870\uc57d\uc744 \uccb4\uacb0\ud55c \uacf5\uac04\uc744 \uae30\ub150\ud558\ub294 \uc804\uc2dc \uacf5\uac04\uc778 \ubcd1\ud569\uc870\uc778\uc2e4\uc774 \uc124\uce58\ub418\uc5c8\ub2e4.", "answer": "\uc744\uc0ac\uc870\uc57d", "sentence": "1910\ub144 8\uc6d4 22\uc77c \uc624\ud6c4 4\uc2dc\uc5d0 \uc81c3\ub300 \ud55c\uad6d \ud1b5\uac10 \ub370\ub77c\uc6b0\uce58 \ub9c8\uc0ac\ud0c0\ucf00\uc640 \ub300\ud55c\uc81c\uad6d \ucd1d\ub9ac\ub300\uc2e0 \uc774\uc644\uc6a9\uc774 \uad00\uc800 2\uce35\uc5d0\uc11c \uc744\uc0ac\uc870\uc57d \uc744 \uc870\uc778\ud558\uc600\ub2e4.", "paragraph_sentence": "1907\ub144 10\uc6d4 16\uc77c\uc5d0 \ub300\ud55c\uc81c\uad6d\uc744 \ubc29\ubb38\ud55c \uc694\uc2dc\ud788\ud1a0 \ud669\ud0dc\uc790\uac00 \uad00\uc800\ub97c 4\uc77c\uac04 \uc219\uc18c\ub85c \uc774\uc6a9\ud558\uc600\ub2e4. 1908\ub144\uc5d0 \ub2e4\uc4f0\ub178 \uae34\uace0(\u8fb0\u91ce\u91d1\u543e)\uac00 \uc0c8\ub85c\uc6b4 \ud1b5\uac10 \uad00\uc800\ub97c \uc124\uacc4\ud558\uc600\uc73c\ub098 \uac19\uc740 \ud574 \uc6a9\uc0b0\uc5d0 \uac00\ud0c0\uc57c\ub9c8\uc758 \ud1b5\uac10 \uad00\uc800\uac00 \ub4e4\uc5b4\uc11c\uba74\uc11c \uacc4\ud68d\uc73c\ub85c \ub05d\ub0ac\ub2e4. 1910\ub144 8\uc6d4 22\uc77c \uc624\ud6c4 4\uc2dc\uc5d0 \uc81c3\ub300 \ud55c\uad6d \ud1b5\uac10 \ub370\ub77c\uc6b0\uce58 \ub9c8\uc0ac\ud0c0\ucf00\uc640 \ub300\ud55c\uc81c\uad6d \ucd1d\ub9ac\ub300\uc2e0 \uc774\uc644\uc6a9\uc774 \uad00\uc800 2\uce35\uc5d0\uc11c \uc744\uc0ac\uc870\uc57d \uc744 \uc870\uc778\ud558\uc600\ub2e4. \uc870\uc120\ucd1d\ub3c5\ubd80 \ucd1d\ub3c5 \uad00\uc800\uac00 \uacbd\ubcf5\uad81 \ub4a4\ud3b8\uc758 \uacbd\ubb34\ub300\ub85c \uc62e\uaca8\uac04 \uc774\ud6c4 \uc65c\uc131\ub300 \ucd1d\ub3c5 \uad00\uc800\ub294 \uc5ed\ub300 \ud1b5\uac10\uacfc \ucd1d\ub3c5\uc758 \ucd08\uc0c1\uacfc \uad00\ub828 \uc720\ubb3c\uc744 \uc804\uc2dc\ud558\ub294 \uc2dc\uc815\uae30\ub150\uad00(\u59cb\u653f\u8a18\u5ff5\u9928)\uc73c\ub85c \uac1c\ud3b8\ub418\uc5b4 1940\ub144 11\uc6d4 22\uc77c\uc5d0 \uac1c\uad00\uc2dd\uc744 \uac70\ud589\ud558\uc600\ub2e4. \uc2dc\uc815\uae30\ub150\uad00\uc73c\ub85c \uac1c\uad00\ud558\uba74\uc11c \uc744\uc0ac\uc870\uc57d\uc744 \uccb4\uacb0\ud55c \uacf5\uac04\uc744 \uae30\ub150\ud558\ub294 \uc804\uc2dc \uacf5\uac04\uc778 \ubcd1\ud569\uc870\uc778\uc2e4\uc774 \uc124\uce58\ub418\uc5c8\ub2e4.", "paragraph_answer": "1907\ub144 10\uc6d4 16\uc77c\uc5d0 \ub300\ud55c\uc81c\uad6d\uc744 \ubc29\ubb38\ud55c \uc694\uc2dc\ud788\ud1a0 \ud669\ud0dc\uc790\uac00 \uad00\uc800\ub97c 4\uc77c\uac04 \uc219\uc18c\ub85c \uc774\uc6a9\ud558\uc600\ub2e4. 1908\ub144\uc5d0 \ub2e4\uc4f0\ub178 \uae34\uace0(\u8fb0\u91ce\u91d1\u543e)\uac00 \uc0c8\ub85c\uc6b4 \ud1b5\uac10 \uad00\uc800\ub97c \uc124\uacc4\ud558\uc600\uc73c\ub098 \uac19\uc740 \ud574 \uc6a9\uc0b0\uc5d0 \uac00\ud0c0\uc57c\ub9c8\uc758 \ud1b5\uac10 \uad00\uc800\uac00 \ub4e4\uc5b4\uc11c\uba74\uc11c \uacc4\ud68d\uc73c\ub85c \ub05d\ub0ac\ub2e4. 1910\ub144 8\uc6d4 22\uc77c \uc624\ud6c4 4\uc2dc\uc5d0 \uc81c3\ub300 \ud55c\uad6d \ud1b5\uac10 \ub370\ub77c\uc6b0\uce58 \ub9c8\uc0ac\ud0c0\ucf00\uc640 \ub300\ud55c\uc81c\uad6d \ucd1d\ub9ac\ub300\uc2e0 \uc774\uc644\uc6a9\uc774 \uad00\uc800 2\uce35\uc5d0\uc11c \uc744\uc0ac\uc870\uc57d \uc744 \uc870\uc778\ud558\uc600\ub2e4. \uc870\uc120\ucd1d\ub3c5\ubd80 \ucd1d\ub3c5 \uad00\uc800\uac00 \uacbd\ubcf5\uad81 \ub4a4\ud3b8\uc758 \uacbd\ubb34\ub300\ub85c \uc62e\uaca8\uac04 \uc774\ud6c4 \uc65c\uc131\ub300 \ucd1d\ub3c5 \uad00\uc800\ub294 \uc5ed\ub300 \ud1b5\uac10\uacfc \ucd1d\ub3c5\uc758 \ucd08\uc0c1\uacfc \uad00\ub828 \uc720\ubb3c\uc744 \uc804\uc2dc\ud558\ub294 \uc2dc\uc815\uae30\ub150\uad00(\u59cb\u653f\u8a18\u5ff5\u9928)\uc73c\ub85c \uac1c\ud3b8\ub418\uc5b4 1940\ub144 11\uc6d4 22\uc77c\uc5d0 \uac1c\uad00\uc2dd\uc744 \uac70\ud589\ud558\uc600\ub2e4. \uc2dc\uc815\uae30\ub150\uad00\uc73c\ub85c \uac1c\uad00\ud558\uba74\uc11c \uc744\uc0ac\uc870\uc57d\uc744 \uccb4\uacb0\ud55c \uacf5\uac04\uc744 \uae30\ub150\ud558\ub294 \uc804\uc2dc \uacf5\uac04\uc778 \ubcd1\ud569\uc870\uc778\uc2e4\uc774 \uc124\uce58\ub418\uc5c8\ub2e4.", "sentence_answer": "1910\ub144 8\uc6d4 22\uc77c \uc624\ud6c4 4\uc2dc\uc5d0 \uc81c3\ub300 \ud55c\uad6d \ud1b5\uac10 \ub370\ub77c\uc6b0\uce58 \ub9c8\uc0ac\ud0c0\ucf00\uc640 \ub300\ud55c\uc81c\uad6d \ucd1d\ub9ac\ub300\uc2e0 \uc774\uc644\uc6a9\uc774 \uad00\uc800 2\uce35\uc5d0\uc11c \uc744\uc0ac\uc870\uc57d \uc744 \uc870\uc778\ud558\uc600\ub2e4.", "paragraph_id": "6552448-6-1", "paragraph_question": "question: \ub370\ub77c\uc6b0\uce58 \ub9c8\uc0ac\ud0c0\ucf00\uc640 \uc774\uc644\uc6a9\uc774 \uad00\uc800 2\uce35\uc5d0\uc11c \uccb4\uacb0\ud55c \uc870\uc57d\uc740?, context: 1907\ub144 10\uc6d4 16\uc77c\uc5d0 \ub300\ud55c\uc81c\uad6d\uc744 \ubc29\ubb38\ud55c \uc694\uc2dc\ud788\ud1a0 \ud669\ud0dc\uc790\uac00 \uad00\uc800\ub97c 4\uc77c\uac04 \uc219\uc18c\ub85c \uc774\uc6a9\ud558\uc600\ub2e4. 1908\ub144\uc5d0 \ub2e4\uc4f0\ub178 \uae34\uace0(\u8fb0\u91ce\u91d1\u543e)\uac00 \uc0c8\ub85c\uc6b4 \ud1b5\uac10 \uad00\uc800\ub97c \uc124\uacc4\ud558\uc600\uc73c\ub098 \uac19\uc740 \ud574 \uc6a9\uc0b0\uc5d0 \uac00\ud0c0\uc57c\ub9c8\uc758 \ud1b5\uac10 \uad00\uc800\uac00 \ub4e4\uc5b4\uc11c\uba74\uc11c \uacc4\ud68d\uc73c\ub85c \ub05d\ub0ac\ub2e4. 1910\ub144 8\uc6d4 22\uc77c \uc624\ud6c4 4\uc2dc\uc5d0 \uc81c3\ub300 \ud55c\uad6d \ud1b5\uac10 \ub370\ub77c\uc6b0\uce58 \ub9c8\uc0ac\ud0c0\ucf00\uc640 \ub300\ud55c\uc81c\uad6d \ucd1d\ub9ac\ub300\uc2e0 \uc774\uc644\uc6a9\uc774 \uad00\uc800 2\uce35\uc5d0\uc11c \uc744\uc0ac\uc870\uc57d\uc744 \uc870\uc778\ud558\uc600\ub2e4. \uc870\uc120\ucd1d\ub3c5\ubd80 \ucd1d\ub3c5 \uad00\uc800\uac00 \uacbd\ubcf5\uad81 \ub4a4\ud3b8\uc758 \uacbd\ubb34\ub300\ub85c \uc62e\uaca8\uac04 \uc774\ud6c4 \uc65c\uc131\ub300 \ucd1d\ub3c5 \uad00\uc800\ub294 \uc5ed\ub300 \ud1b5\uac10\uacfc \ucd1d\ub3c5\uc758 \ucd08\uc0c1\uacfc \uad00\ub828 \uc720\ubb3c\uc744 \uc804\uc2dc\ud558\ub294 \uc2dc\uc815\uae30\ub150\uad00(\u59cb\u653f\u8a18\u5ff5\u9928)\uc73c\ub85c \uac1c\ud3b8\ub418\uc5b4 1940\ub144 11\uc6d4 22\uc77c\uc5d0 \uac1c\uad00\uc2dd\uc744 \uac70\ud589\ud558\uc600\ub2e4. \uc2dc\uc815\uae30\ub150\uad00\uc73c\ub85c \uac1c\uad00\ud558\uba74\uc11c \uc744\uc0ac\uc870\uc57d\uc744 \uccb4\uacb0\ud55c \uacf5\uac04\uc744 \uae30\ub150\ud558\ub294 \uc804\uc2dc \uacf5\uac04\uc778 \ubcd1\ud569\uc870\uc778\uc2e4\uc774 \uc124\uce58\ub418\uc5c8\ub2e4."} {"question": "\uac00\uc7a5 \uc624\ub798\ub41c \ubd80\ucf00\ub85c\ub294 \uae30\uc6d0\uc804 \uba87 \uc138\uae30\uacbd\uc5d0 \uc81c\uc791\ub418\uc5c8\ub098?", "paragraph": "\uc5d0\ud2b8\ub8e8\ub9ac\uc544\uc758 \uacf5\uc608\ub294 \uccad\ub3d9 \ubc0f \uae08\uacf5\ud488(\u91d1\u5de5\u54c1)\uc5d0 \ub3c5\ucc3d\uc801\uc774\uc5b4\uc11c \ub9e4\uc6b0 \ub6f0\uc5b4\ub09c \uae30\ub2a5\uc744 \ubcf4\uc600\ub2e4. \uc774 \uae30\ubc95\uc740 \uc624\ub9ac\uc5d4\ud2b8\u00b7\uadf8\ub9ac\uc2a4\uc5d0\uc11c \ubc30\uc6b0\uace0, \ud6c4\uc5d0\ub294 \uc790\uc2e0\ub4e4\uc758 \uc190\uc73c\ub85c \ud6cc\ub96d\ud55c \uc791\ud488\uc744 \ub9cc\ub4e4\uc5b4\ub0c8\ub2e4. \ud0c0\ub974\ud034\ub2c8\uc544\u00b7\uccb4\ub974\ubca0\ud2b8\ub9ac\uc5d0\uc11c\ub294 \uae30\uc6d0\uc804 7\uc138\uae30\uacbd\ubd80\ud130 \uae30\uc6d0\uc804 6\uc138\uae30\uc5d0 \uac78\uccd0\uc11c \uccad\ub3d9\uc81c\ud488\uc758 \uc81c\uc791\uc774 \uc131\ud589\ud558\uace0, \uae30\uc6d0\uc804 6\uc138\uae30 \ud6c4\ubc18, \ubd88\uce58\uc5d0\uc11c\ub294 \uc7a5\uc2dd\uc801 \uc778\ubb3c\uc744 \uc5b9\uac70\ub098 \ubc1c \ubd80\ubd84\uc5d0 \ub3d9\ubb3c\uc758 \ubc1c \ubaa8\uc591\uc744 \ud55c \uc0bc\uac01\ub300(\u4e09\u811a\u81fa)\u00b7\ucd09\ub300\u00b7\ud5a5\ub85c \ub4f1\uc774 \ub300\ub7c9\uc73c\ub85c \uc81c\uc791\ub418\uc5c8\ub2e4. \uccad\ub3d9 \uac70\uc6b8\uacfc \ud0a4\uc2a4\ud0c0\ub77c\uace0 \ubd88\ub9ac\ub294 \uc9c1\uc0ac\uac01\ud615, \ud639\uc740 \uc6d0\ud1b5\ud615\uc758 \ud654\uc7a5\uc0c1\uc790(\u5316\u7ca7\u7bb1\u5b50)\uc5d0 \uc0c8\uae34 \uc120\uac01(\u7dda\u523b)\uc740 \uc5d0\ud2b8\ub8e8\ub9ac\uc544 \uae08\uacf5(\u91d1\u5de5) \uc911\uc5d0\uc11c\ub3c4 \ud2b9\ud788 \ub6f0\uc5b4\ub098\uac8c \uc12c\uc138\ud558\uba70, \uc815\uce58(\u7cbe\u7dfb)\ud55c \uc120\uc758 \ud130\uce58\ub294 \uc720\ub824\ud558\uc5ec \uace0\ub300\uc5d0 \uc788\uc5b4\uc11c \uc720\ub840\ub97c \ucc3e\uc544\ubcfc \uc218 \uc5c6\uc744 \uc815\ub3c4\uc774\ub2e4. \uc120\uac01\uc758 \uc8fc\uc81c\ub294 \ubbf8(\u7f8e)\uc758 \uc5ec\uc2e0\uc774\ub098 \uc2e0\ud654\uc758 \uc0ac\ub791\uc744 \ub2e4\ub8ec \uac83\uc774 \ub9ce\uace0, \uc81c\uc791\uc758 \uc911\uc2ec\uc9c0\ub294 \ud314\ub808\uc2a4\ud2b8\ub9ac\ub098\uc640 \ubd88\uce58\ub77c\uace0 \uc5ec\uaca8\uc9c4\ub2e4. \ubc14\ud2f0\uce78\uc5d0 \uc788\ub294 <\uc5d0\ub85c\uc2a4\uc640 \ucf00\ud30c\ub85c\uc2a4>\uc758 \uc120\uac01\uc774 \uc788\ub294 \uc190\uac70\uc6b8\uc774\ub098 <\ud53c\ucf54\ub85c\ub2c8\uc758 \ud0a4\uc2a4\ud0c0>\ub294 \uccad\ub3d9\uc81c\ud488\uc758 \uc808\ud488(\u7d76\u54c1)\uc774\ub2e4. \uadf8\ub9ac\uace0 \uae30\uc6d0\uc804 5\uc138\uae30\uacbd\uc758 \ucf54\ub974\ud1a0\ub098\uc5d0\uc11c \ucd9c\ud1a0\ub41c \ubd80\uc870(\u6d6e\u5f6b)\uac00 \ubd99\uc740 \ub7a8\ud504\ub3c4 \ubaa8\uc591\uacfc \uc7a5\uc2dd\uc774 \ud638\uc0ac\uc2a4\ub7fd\uace0 \ub610\ud55c \ub3c5\ud2b9\ud55c \uac83\uc73c\ub85c\uc11c, \ub2f9\uc2dc\uc758 \uc5d0\ud2b8\ub8e8\ub9ac\uc544\uc758 \uace0\ub3c4\ud55c \uccad\ub3d9 \uae30\uc220\uc744 \uc804\ud558\ub294 \uc218\uc791(\u79c0\u4f5c)\uc774\ub2e4. \uae08\uc81c\ud488\uc73c\ub85c\ub294 \ube0c\ub85c\uce58\u00b7\uadc0\uace0\ub9ac \ub4f1 \ud2b9\ud788 \uc5ec\uc131\uc758 \uc7a5\uc2e0\uad6c\uc5d0 \ub180\ub784 \ub9cc\ud55c \uae30\ub2a5\uc774 \ubcf4\uc778\ub2e4. \uadf8 \ud55c \uac00\uc9c0\ub85c \uc608\ub85c\uc11c \uccb4\ub974\ubca0\ud2b8\ub9ac\uc758 \uc655\uacfc \uc655\ube44\uc758 \ubb18\uc5d0\uc11c \ucd9c\ud1a0\ub41c \ube0c\ub85c\uce58(\ubc14\ud2f0\uce78 \ubbf8\uc220\uad00)\ub294, \uae30\uc6d0\uc804 7\uc138\uae30 \uc911\uc5fd\uc5d0 \ub9cc\ub4e0 \uac83\uc774\ub77c\uace0 \ud55c\ub2e4. \uc774 \ube0c\ub85c\uce58\ub294 \uc704\ub294 \ub098\ubb47\uc78e\uc758 \ubaa8\uc591\uc744 \ud55c \ubc29\ud328\ub85c\uc11c, \uadf8 \ub458\ub808\uc5d0\ub294 \ub450\uc904\uc758 \ud314\ubbf8\ud2b8\ub85c \uafb8\uba70\uc9c0\uace0, \ub2e4\uc12f\ub9c8\ub9ac\uc758 \uc0ac\uc790\uac00 \uc911\uc559\uc73c\ub85c \ud798\ucc28\uac8c \uac78\uc5b4\ub098\uac00\uba70, \ub610\ud55c \uc544\ub798\uc758 \ub09c\ud615(\u5375\u5f62) \ubc29\ud328\uc5d0\ub294 \uc9d1\uc624\ub9ac\uac00 \uc77c\uacf1\uc904\ub85c \ub098\ub780\ud788 \uc788\ub294 \ub4f1, \uc815\uad50\ud55c \uae30\uc220\uc740 \ub180\ub784 \ub9cc\ud55c \uac83\uc774\ub2e4. \uc5d0\ud2b8\ub8e8\ub9ac\uc544\uc758 \ub3c4\uc790\uae30\ub294 \ud751\uc0c9\uc758 \ubd80\ucf00\ub85c\ub97c \uc81c\uc678\ud558\uace0\ub294 \uadf8 \ubaa8\uc591\ub3c4, \uc7a5\uc2dd\uc758 \uc8fc\uc81c\ub3c4 \ubaa8\ub450 \uadf8\ub9ac\uc2a4\uc758 \ud56d\uc544\ub9ac\uc758 \ubaa8\ubc29\uc774\ub2e4. \ubc29\ub300\ud55c \uadf8\ub9ac\uc2a4 \ub3c4\uc790\uae30\uc758 \uc218\uc785\uc73c\ub85c \uc5d0\ud2b8\ub8e8\ub9ac\uc544\uc758 \uc694\uacf5(\u7aaf\u5de5)\uc740 \ud070 \uc601\ud5a5\uc744 \ubc1b\uc544, \uadf8\ub9ac\uc2a4\uc758 \ubaa8\ubc29\uc73c\ub85c \uc2dc\uc885\ud588\ub2e4. \uc774\uc5d0 \ub300\ud574 \uc5d0\ud2b8\ub8e8\ub9ac\uc544 \ub3c5\ucc3d\uc801\uc778 \ubd80\ucf00\ub85c\ub294 \ub3c4\ud1a0(\u9676\u571f)\ub97c \uadf8\uc744\ub824\uc11c \ub9cc\ub4e0 \uad11\ud0dd\uc774 \ub098\ub294 \ud751\ub3c4(\u9ed1\u9676)\ub85c\uc11c, \uadf8 \uc6d0\ud615\uc740 \ud1a0\ucc29\uc758 \ube4c\ub77c \ub178\ubc14 \ubb38\ud654\uc758 \uc601\ud5a5\uc5d0\uc11c \ubc1c\uc804\ud558\uace0, \uac00\uc7a5 \uc624\ub79c \ubd80\ucf00\ub85c\ub294 \uae30\uc6d0\uc804 8\uc138\uae30\uacbd\uc73c\ub85c \ub3cc\uc544\uac00\uba70, \uae30\uc6d0\uc804 6\uc138\uae30\u00b75\uc138\uae30\uc5d0 \uc81c\uc791\ub41c \ubd80\ucf00\ub85c\ub294 \ub300\ubd80\ubd84\uc774 \ud0a4\uc6b0\uc2dc\uc5d0\uc11c \uc81c\uc791\ub418\uc5c8\uc73c\uba70, \ud615\uc131\uc740 \ubc14\ud0d5\uc774 \ub450\uaecd\uace0 \ud55c\uce35 \uad11\ud0dd\uc744 \ub0b8\ub2e4. \uc77c\ubc18\uc801\uc73c\ub85c \uc5d0\ud2b8\ub8e8\ub9ac\uc544\uc758 \ubd80\ucf00\ub85c\ub294 \uc18c\ubc15\ud558\uc5ec \ud2bc\ud2bc\ud55c \uac83\uc774\ub2e4.", "answer": "\uae30\uc6d0\uc804 8\uc138\uae30\uacbd", "sentence": "\uc774\uc5d0 \ub300\ud574 \uc5d0\ud2b8\ub8e8\ub9ac\uc544 \ub3c5\ucc3d\uc801\uc778 \ubd80\ucf00\ub85c\ub294 \ub3c4\ud1a0(\u9676\u571f)\ub97c \uadf8\uc744\ub824\uc11c \ub9cc\ub4e0 \uad11\ud0dd\uc774 \ub098\ub294 \ud751\ub3c4(\u9ed1\u9676)\ub85c\uc11c, \uadf8 \uc6d0\ud615\uc740 \ud1a0\ucc29\uc758 \ube4c\ub77c \ub178\ubc14 \ubb38\ud654\uc758 \uc601\ud5a5\uc5d0\uc11c \ubc1c\uc804\ud558\uace0, \uac00\uc7a5 \uc624\ub79c \ubd80\ucf00\ub85c\ub294 \uae30\uc6d0\uc804 8\uc138\uae30\uacbd \uc73c\ub85c \ub3cc\uc544\uac00\uba70, \uae30\uc6d0\uc804 6\uc138\uae30\u00b75\uc138\uae30\uc5d0 \uc81c\uc791\ub41c \ubd80\ucf00\ub85c\ub294 \ub300\ubd80\ubd84\uc774 \ud0a4\uc6b0\uc2dc\uc5d0\uc11c \uc81c\uc791\ub418\uc5c8\uc73c\uba70, \ud615\uc131\uc740 \ubc14\ud0d5\uc774 \ub450\uaecd\uace0 \ud55c\uce35 \uad11\ud0dd\uc744 \ub0b8\ub2e4.", "paragraph_sentence": "\uc5d0\ud2b8\ub8e8\ub9ac\uc544\uc758 \uacf5\uc608\ub294 \uccad\ub3d9 \ubc0f \uae08\uacf5\ud488(\u91d1\u5de5\u54c1)\uc5d0 \ub3c5\ucc3d\uc801\uc774\uc5b4\uc11c \ub9e4\uc6b0 \ub6f0\uc5b4\ub09c \uae30\ub2a5\uc744 \ubcf4\uc600\ub2e4. \uc774 \uae30\ubc95\uc740 \uc624\ub9ac\uc5d4\ud2b8\u00b7\uadf8\ub9ac\uc2a4\uc5d0\uc11c \ubc30\uc6b0\uace0, \ud6c4\uc5d0\ub294 \uc790\uc2e0\ub4e4\uc758 \uc190\uc73c\ub85c \ud6cc\ub96d\ud55c \uc791\ud488\uc744 \ub9cc\ub4e4\uc5b4\ub0c8\ub2e4. \ud0c0\ub974\ud034\ub2c8\uc544\u00b7\uccb4\ub974\ubca0\ud2b8\ub9ac\uc5d0\uc11c\ub294 \uae30\uc6d0\uc804 7\uc138\uae30\uacbd\ubd80\ud130 \uae30\uc6d0\uc804 6\uc138\uae30\uc5d0 \uac78\uccd0\uc11c \uccad\ub3d9\uc81c\ud488\uc758 \uc81c\uc791\uc774 \uc131\ud589\ud558\uace0, \uae30\uc6d0\uc804 6\uc138\uae30 \ud6c4\ubc18, \ubd88\uce58\uc5d0\uc11c\ub294 \uc7a5\uc2dd\uc801 \uc778\ubb3c\uc744 \uc5b9\uac70\ub098 \ubc1c \ubd80\ubd84\uc5d0 \ub3d9\ubb3c\uc758 \ubc1c \ubaa8\uc591\uc744 \ud55c \uc0bc\uac01\ub300(\u4e09\u811a\u81fa)\u00b7\ucd09\ub300\u00b7\ud5a5\ub85c \ub4f1\uc774 \ub300\ub7c9\uc73c\ub85c \uc81c\uc791\ub418\uc5c8\ub2e4. \uccad\ub3d9 \uac70\uc6b8\uacfc \ud0a4\uc2a4\ud0c0\ub77c\uace0 \ubd88\ub9ac\ub294 \uc9c1\uc0ac\uac01\ud615, \ud639\uc740 \uc6d0\ud1b5\ud615\uc758 \ud654\uc7a5\uc0c1\uc790(\u5316\u7ca7\u7bb1\u5b50)\uc5d0 \uc0c8\uae34 \uc120\uac01(\u7dda\u523b)\uc740 \uc5d0\ud2b8\ub8e8\ub9ac\uc544 \uae08\uacf5(\u91d1\u5de5) \uc911\uc5d0\uc11c\ub3c4 \ud2b9\ud788 \ub6f0\uc5b4\ub098\uac8c \uc12c\uc138\ud558\uba70, \uc815\uce58(\u7cbe\u7dfb)\ud55c \uc120\uc758 \ud130\uce58\ub294 \uc720\ub824\ud558\uc5ec \uace0\ub300\uc5d0 \uc788\uc5b4\uc11c \uc720\ub840\ub97c \ucc3e\uc544\ubcfc \uc218 \uc5c6\uc744 \uc815\ub3c4\uc774\ub2e4. \uc120\uac01\uc758 \uc8fc\uc81c\ub294 \ubbf8(\u7f8e)\uc758 \uc5ec\uc2e0\uc774\ub098 \uc2e0\ud654\uc758 \uc0ac\ub791\uc744 \ub2e4\ub8ec \uac83\uc774 \ub9ce\uace0, \uc81c\uc791\uc758 \uc911\uc2ec\uc9c0\ub294 \ud314\ub808\uc2a4\ud2b8\ub9ac\ub098\uc640 \ubd88\uce58\ub77c\uace0 \uc5ec\uaca8\uc9c4\ub2e4. \ubc14\ud2f0\uce78\uc5d0 \uc788\ub294 <\uc5d0\ub85c\uc2a4\uc640 \ucf00\ud30c\ub85c\uc2a4>\uc758 \uc120\uac01\uc774 \uc788\ub294 \uc190\uac70\uc6b8\uc774\ub098 <\ud53c\ucf54\ub85c\ub2c8\uc758 \ud0a4\uc2a4\ud0c0>\ub294 \uccad\ub3d9\uc81c\ud488\uc758 \uc808\ud488(\u7d76\u54c1)\uc774\ub2e4. \uadf8\ub9ac\uace0 \uae30\uc6d0\uc804 5\uc138\uae30\uacbd\uc758 \ucf54\ub974\ud1a0\ub098\uc5d0\uc11c \ucd9c\ud1a0\ub41c \ubd80\uc870(\u6d6e\u5f6b)\uac00 \ubd99\uc740 \ub7a8\ud504\ub3c4 \ubaa8\uc591\uacfc \uc7a5\uc2dd\uc774 \ud638\uc0ac\uc2a4\ub7fd\uace0 \ub610\ud55c \ub3c5\ud2b9\ud55c \uac83\uc73c\ub85c\uc11c, \ub2f9\uc2dc\uc758 \uc5d0\ud2b8\ub8e8\ub9ac\uc544\uc758 \uace0\ub3c4\ud55c \uccad\ub3d9 \uae30\uc220\uc744 \uc804\ud558\ub294 \uc218\uc791(\u79c0\u4f5c)\uc774\ub2e4. \uae08\uc81c\ud488\uc73c\ub85c\ub294 \ube0c\ub85c\uce58\u00b7\uadc0\uace0\ub9ac \ub4f1 \ud2b9\ud788 \uc5ec\uc131\uc758 \uc7a5\uc2e0\uad6c\uc5d0 \ub180\ub784 \ub9cc\ud55c \uae30\ub2a5\uc774 \ubcf4\uc778\ub2e4. \uadf8 \ud55c \uac00\uc9c0\ub85c \uc608\ub85c\uc11c \uccb4\ub974\ubca0\ud2b8\ub9ac\uc758 \uc655\uacfc \uc655\ube44\uc758 \ubb18\uc5d0\uc11c \ucd9c\ud1a0\ub41c \ube0c\ub85c\uce58(\ubc14\ud2f0\uce78 \ubbf8\uc220\uad00)\ub294, \uae30\uc6d0\uc804 7\uc138\uae30 \uc911\uc5fd\uc5d0 \ub9cc\ub4e0 \uac83\uc774\ub77c\uace0 \ud55c\ub2e4. \uc774 \ube0c\ub85c\uce58\ub294 \uc704\ub294 \ub098\ubb47\uc78e\uc758 \ubaa8\uc591\uc744 \ud55c \ubc29\ud328\ub85c\uc11c, \uadf8 \ub458\ub808\uc5d0\ub294 \ub450\uc904\uc758 \ud314\ubbf8\ud2b8\ub85c \uafb8\uba70\uc9c0\uace0, \ub2e4\uc12f\ub9c8\ub9ac\uc758 \uc0ac\uc790\uac00 \uc911\uc559\uc73c\ub85c \ud798\ucc28\uac8c \uac78\uc5b4\ub098\uac00\uba70, \ub610\ud55c \uc544\ub798\uc758 \ub09c\ud615(\u5375\u5f62) \ubc29\ud328\uc5d0\ub294 \uc9d1\uc624\ub9ac\uac00 \uc77c\uacf1\uc904\ub85c \ub098\ub780\ud788 \uc788\ub294 \ub4f1, \uc815\uad50\ud55c \uae30\uc220\uc740 \ub180\ub784 \ub9cc\ud55c \uac83\uc774\ub2e4. \uc5d0\ud2b8\ub8e8\ub9ac\uc544\uc758 \ub3c4\uc790\uae30\ub294 \ud751\uc0c9\uc758 \ubd80\ucf00\ub85c\ub97c \uc81c\uc678\ud558\uace0\ub294 \uadf8 \ubaa8\uc591\ub3c4, \uc7a5\uc2dd\uc758 \uc8fc\uc81c\ub3c4 \ubaa8\ub450 \uadf8\ub9ac\uc2a4\uc758 \ud56d\uc544\ub9ac\uc758 \ubaa8\ubc29\uc774\ub2e4. \ubc29\ub300\ud55c \uadf8\ub9ac\uc2a4 \ub3c4\uc790\uae30\uc758 \uc218\uc785\uc73c\ub85c \uc5d0\ud2b8\ub8e8\ub9ac\uc544\uc758 \uc694\uacf5(\u7aaf\u5de5)\uc740 \ud070 \uc601\ud5a5\uc744 \ubc1b\uc544, \uadf8\ub9ac\uc2a4\uc758 \ubaa8\ubc29\uc73c\ub85c \uc2dc\uc885\ud588\ub2e4. \uc774\uc5d0 \ub300\ud574 \uc5d0\ud2b8\ub8e8\ub9ac\uc544 \ub3c5\ucc3d\uc801\uc778 \ubd80\ucf00\ub85c\ub294 \ub3c4\ud1a0(\u9676\u571f)\ub97c \uadf8\uc744\ub824\uc11c \ub9cc\ub4e0 \uad11\ud0dd\uc774 \ub098\ub294 \ud751\ub3c4(\u9ed1\u9676)\ub85c\uc11c, \uadf8 \uc6d0\ud615\uc740 \ud1a0\ucc29\uc758 \ube4c\ub77c \ub178\ubc14 \ubb38\ud654\uc758 \uc601\ud5a5\uc5d0\uc11c \ubc1c\uc804\ud558\uace0, \uac00\uc7a5 \uc624\ub79c \ubd80\ucf00\ub85c\ub294 \uae30\uc6d0\uc804 8\uc138\uae30\uacbd \uc73c\ub85c \ub3cc\uc544\uac00\uba70, \uae30\uc6d0\uc804 6\uc138\uae30\u00b75\uc138\uae30\uc5d0 \uc81c\uc791\ub41c \ubd80\ucf00\ub85c\ub294 \ub300\ubd80\ubd84\uc774 \ud0a4\uc6b0\uc2dc\uc5d0\uc11c \uc81c\uc791\ub418\uc5c8\uc73c\uba70, \ud615\uc131\uc740 \ubc14\ud0d5\uc774 \ub450\uaecd\uace0 \ud55c\uce35 \uad11\ud0dd\uc744 \ub0b8\ub2e4. \uc77c\ubc18\uc801\uc73c\ub85c \uc5d0\ud2b8\ub8e8\ub9ac\uc544\uc758 \ubd80\ucf00\ub85c\ub294 \uc18c\ubc15\ud558\uc5ec \ud2bc\ud2bc\ud55c \uac83\uc774\ub2e4.", "paragraph_answer": "\uc5d0\ud2b8\ub8e8\ub9ac\uc544\uc758 \uacf5\uc608\ub294 \uccad\ub3d9 \ubc0f \uae08\uacf5\ud488(\u91d1\u5de5\u54c1)\uc5d0 \ub3c5\ucc3d\uc801\uc774\uc5b4\uc11c \ub9e4\uc6b0 \ub6f0\uc5b4\ub09c \uae30\ub2a5\uc744 \ubcf4\uc600\ub2e4. \uc774 \uae30\ubc95\uc740 \uc624\ub9ac\uc5d4\ud2b8\u00b7\uadf8\ub9ac\uc2a4\uc5d0\uc11c \ubc30\uc6b0\uace0, \ud6c4\uc5d0\ub294 \uc790\uc2e0\ub4e4\uc758 \uc190\uc73c\ub85c \ud6cc\ub96d\ud55c \uc791\ud488\uc744 \ub9cc\ub4e4\uc5b4\ub0c8\ub2e4. \ud0c0\ub974\ud034\ub2c8\uc544\u00b7\uccb4\ub974\ubca0\ud2b8\ub9ac\uc5d0\uc11c\ub294 \uae30\uc6d0\uc804 7\uc138\uae30\uacbd\ubd80\ud130 \uae30\uc6d0\uc804 6\uc138\uae30\uc5d0 \uac78\uccd0\uc11c \uccad\ub3d9\uc81c\ud488\uc758 \uc81c\uc791\uc774 \uc131\ud589\ud558\uace0, \uae30\uc6d0\uc804 6\uc138\uae30 \ud6c4\ubc18, \ubd88\uce58\uc5d0\uc11c\ub294 \uc7a5\uc2dd\uc801 \uc778\ubb3c\uc744 \uc5b9\uac70\ub098 \ubc1c \ubd80\ubd84\uc5d0 \ub3d9\ubb3c\uc758 \ubc1c \ubaa8\uc591\uc744 \ud55c \uc0bc\uac01\ub300(\u4e09\u811a\u81fa)\u00b7\ucd09\ub300\u00b7\ud5a5\ub85c \ub4f1\uc774 \ub300\ub7c9\uc73c\ub85c \uc81c\uc791\ub418\uc5c8\ub2e4. \uccad\ub3d9 \uac70\uc6b8\uacfc \ud0a4\uc2a4\ud0c0\ub77c\uace0 \ubd88\ub9ac\ub294 \uc9c1\uc0ac\uac01\ud615, \ud639\uc740 \uc6d0\ud1b5\ud615\uc758 \ud654\uc7a5\uc0c1\uc790(\u5316\u7ca7\u7bb1\u5b50)\uc5d0 \uc0c8\uae34 \uc120\uac01(\u7dda\u523b)\uc740 \uc5d0\ud2b8\ub8e8\ub9ac\uc544 \uae08\uacf5(\u91d1\u5de5) \uc911\uc5d0\uc11c\ub3c4 \ud2b9\ud788 \ub6f0\uc5b4\ub098\uac8c \uc12c\uc138\ud558\uba70, \uc815\uce58(\u7cbe\u7dfb)\ud55c \uc120\uc758 \ud130\uce58\ub294 \uc720\ub824\ud558\uc5ec \uace0\ub300\uc5d0 \uc788\uc5b4\uc11c \uc720\ub840\ub97c \ucc3e\uc544\ubcfc \uc218 \uc5c6\uc744 \uc815\ub3c4\uc774\ub2e4. \uc120\uac01\uc758 \uc8fc\uc81c\ub294 \ubbf8(\u7f8e)\uc758 \uc5ec\uc2e0\uc774\ub098 \uc2e0\ud654\uc758 \uc0ac\ub791\uc744 \ub2e4\ub8ec \uac83\uc774 \ub9ce\uace0, \uc81c\uc791\uc758 \uc911\uc2ec\uc9c0\ub294 \ud314\ub808\uc2a4\ud2b8\ub9ac\ub098\uc640 \ubd88\uce58\ub77c\uace0 \uc5ec\uaca8\uc9c4\ub2e4. \ubc14\ud2f0\uce78\uc5d0 \uc788\ub294 <\uc5d0\ub85c\uc2a4\uc640 \ucf00\ud30c\ub85c\uc2a4>\uc758 \uc120\uac01\uc774 \uc788\ub294 \uc190\uac70\uc6b8\uc774\ub098 <\ud53c\ucf54\ub85c\ub2c8\uc758 \ud0a4\uc2a4\ud0c0>\ub294 \uccad\ub3d9\uc81c\ud488\uc758 \uc808\ud488(\u7d76\u54c1)\uc774\ub2e4. \uadf8\ub9ac\uace0 \uae30\uc6d0\uc804 5\uc138\uae30\uacbd\uc758 \ucf54\ub974\ud1a0\ub098\uc5d0\uc11c \ucd9c\ud1a0\ub41c \ubd80\uc870(\u6d6e\u5f6b)\uac00 \ubd99\uc740 \ub7a8\ud504\ub3c4 \ubaa8\uc591\uacfc \uc7a5\uc2dd\uc774 \ud638\uc0ac\uc2a4\ub7fd\uace0 \ub610\ud55c \ub3c5\ud2b9\ud55c \uac83\uc73c\ub85c\uc11c, \ub2f9\uc2dc\uc758 \uc5d0\ud2b8\ub8e8\ub9ac\uc544\uc758 \uace0\ub3c4\ud55c \uccad\ub3d9 \uae30\uc220\uc744 \uc804\ud558\ub294 \uc218\uc791(\u79c0\u4f5c)\uc774\ub2e4. \uae08\uc81c\ud488\uc73c\ub85c\ub294 \ube0c\ub85c\uce58\u00b7\uadc0\uace0\ub9ac \ub4f1 \ud2b9\ud788 \uc5ec\uc131\uc758 \uc7a5\uc2e0\uad6c\uc5d0 \ub180\ub784 \ub9cc\ud55c \uae30\ub2a5\uc774 \ubcf4\uc778\ub2e4. \uadf8 \ud55c \uac00\uc9c0\ub85c \uc608\ub85c\uc11c \uccb4\ub974\ubca0\ud2b8\ub9ac\uc758 \uc655\uacfc \uc655\ube44\uc758 \ubb18\uc5d0\uc11c \ucd9c\ud1a0\ub41c \ube0c\ub85c\uce58(\ubc14\ud2f0\uce78 \ubbf8\uc220\uad00)\ub294, \uae30\uc6d0\uc804 7\uc138\uae30 \uc911\uc5fd\uc5d0 \ub9cc\ub4e0 \uac83\uc774\ub77c\uace0 \ud55c\ub2e4. \uc774 \ube0c\ub85c\uce58\ub294 \uc704\ub294 \ub098\ubb47\uc78e\uc758 \ubaa8\uc591\uc744 \ud55c \ubc29\ud328\ub85c\uc11c, \uadf8 \ub458\ub808\uc5d0\ub294 \ub450\uc904\uc758 \ud314\ubbf8\ud2b8\ub85c \uafb8\uba70\uc9c0\uace0, \ub2e4\uc12f\ub9c8\ub9ac\uc758 \uc0ac\uc790\uac00 \uc911\uc559\uc73c\ub85c \ud798\ucc28\uac8c \uac78\uc5b4\ub098\uac00\uba70, \ub610\ud55c \uc544\ub798\uc758 \ub09c\ud615(\u5375\u5f62) \ubc29\ud328\uc5d0\ub294 \uc9d1\uc624\ub9ac\uac00 \uc77c\uacf1\uc904\ub85c \ub098\ub780\ud788 \uc788\ub294 \ub4f1, \uc815\uad50\ud55c \uae30\uc220\uc740 \ub180\ub784 \ub9cc\ud55c \uac83\uc774\ub2e4. \uc5d0\ud2b8\ub8e8\ub9ac\uc544\uc758 \ub3c4\uc790\uae30\ub294 \ud751\uc0c9\uc758 \ubd80\ucf00\ub85c\ub97c \uc81c\uc678\ud558\uace0\ub294 \uadf8 \ubaa8\uc591\ub3c4, \uc7a5\uc2dd\uc758 \uc8fc\uc81c\ub3c4 \ubaa8\ub450 \uadf8\ub9ac\uc2a4\uc758 \ud56d\uc544\ub9ac\uc758 \ubaa8\ubc29\uc774\ub2e4. \ubc29\ub300\ud55c \uadf8\ub9ac\uc2a4 \ub3c4\uc790\uae30\uc758 \uc218\uc785\uc73c\ub85c \uc5d0\ud2b8\ub8e8\ub9ac\uc544\uc758 \uc694\uacf5(\u7aaf\u5de5)\uc740 \ud070 \uc601\ud5a5\uc744 \ubc1b\uc544, \uadf8\ub9ac\uc2a4\uc758 \ubaa8\ubc29\uc73c\ub85c \uc2dc\uc885\ud588\ub2e4. \uc774\uc5d0 \ub300\ud574 \uc5d0\ud2b8\ub8e8\ub9ac\uc544 \ub3c5\ucc3d\uc801\uc778 \ubd80\ucf00\ub85c\ub294 \ub3c4\ud1a0(\u9676\u571f)\ub97c \uadf8\uc744\ub824\uc11c \ub9cc\ub4e0 \uad11\ud0dd\uc774 \ub098\ub294 \ud751\ub3c4(\u9ed1\u9676)\ub85c\uc11c, \uadf8 \uc6d0\ud615\uc740 \ud1a0\ucc29\uc758 \ube4c\ub77c \ub178\ubc14 \ubb38\ud654\uc758 \uc601\ud5a5\uc5d0\uc11c \ubc1c\uc804\ud558\uace0, \uac00\uc7a5 \uc624\ub79c \ubd80\ucf00\ub85c\ub294 \uae30\uc6d0\uc804 8\uc138\uae30\uacbd \uc73c\ub85c \ub3cc\uc544\uac00\uba70, \uae30\uc6d0\uc804 6\uc138\uae30\u00b75\uc138\uae30\uc5d0 \uc81c\uc791\ub41c \ubd80\ucf00\ub85c\ub294 \ub300\ubd80\ubd84\uc774 \ud0a4\uc6b0\uc2dc\uc5d0\uc11c \uc81c\uc791\ub418\uc5c8\uc73c\uba70, \ud615\uc131\uc740 \ubc14\ud0d5\uc774 \ub450\uaecd\uace0 \ud55c\uce35 \uad11\ud0dd\uc744 \ub0b8\ub2e4. \uc77c\ubc18\uc801\uc73c\ub85c \uc5d0\ud2b8\ub8e8\ub9ac\uc544\uc758 \ubd80\ucf00\ub85c\ub294 \uc18c\ubc15\ud558\uc5ec \ud2bc\ud2bc\ud55c \uac83\uc774\ub2e4.", "sentence_answer": "\uc774\uc5d0 \ub300\ud574 \uc5d0\ud2b8\ub8e8\ub9ac\uc544 \ub3c5\ucc3d\uc801\uc778 \ubd80\ucf00\ub85c\ub294 \ub3c4\ud1a0(\u9676\u571f)\ub97c \uadf8\uc744\ub824\uc11c \ub9cc\ub4e0 \uad11\ud0dd\uc774 \ub098\ub294 \ud751\ub3c4(\u9ed1\u9676)\ub85c\uc11c, \uadf8 \uc6d0\ud615\uc740 \ud1a0\ucc29\uc758 \ube4c\ub77c \ub178\ubc14 \ubb38\ud654\uc758 \uc601\ud5a5\uc5d0\uc11c \ubc1c\uc804\ud558\uace0, \uac00\uc7a5 \uc624\ub79c \ubd80\ucf00\ub85c\ub294 \uae30\uc6d0\uc804 8\uc138\uae30\uacbd \uc73c\ub85c \ub3cc\uc544\uac00\uba70, \uae30\uc6d0\uc804 6\uc138\uae30\u00b75\uc138\uae30\uc5d0 \uc81c\uc791\ub41c \ubd80\ucf00\ub85c\ub294 \ub300\ubd80\ubd84\uc774 \ud0a4\uc6b0\uc2dc\uc5d0\uc11c \uc81c\uc791\ub418\uc5c8\uc73c\uba70, \ud615\uc131\uc740 \ubc14\ud0d5\uc774 \ub450\uaecd\uace0 \ud55c\uce35 \uad11\ud0dd\uc744 \ub0b8\ub2e4.", "paragraph_id": "6540745-3-1", "paragraph_question": "question: \uac00\uc7a5 \uc624\ub798\ub41c \ubd80\ucf00\ub85c\ub294 \uae30\uc6d0\uc804 \uba87 \uc138\uae30\uacbd\uc5d0 \uc81c\uc791\ub418\uc5c8\ub098?, context: \uc5d0\ud2b8\ub8e8\ub9ac\uc544\uc758 \uacf5\uc608\ub294 \uccad\ub3d9 \ubc0f \uae08\uacf5\ud488(\u91d1\u5de5\u54c1)\uc5d0 \ub3c5\ucc3d\uc801\uc774\uc5b4\uc11c \ub9e4\uc6b0 \ub6f0\uc5b4\ub09c \uae30\ub2a5\uc744 \ubcf4\uc600\ub2e4. \uc774 \uae30\ubc95\uc740 \uc624\ub9ac\uc5d4\ud2b8\u00b7\uadf8\ub9ac\uc2a4\uc5d0\uc11c \ubc30\uc6b0\uace0, \ud6c4\uc5d0\ub294 \uc790\uc2e0\ub4e4\uc758 \uc190\uc73c\ub85c \ud6cc\ub96d\ud55c \uc791\ud488\uc744 \ub9cc\ub4e4\uc5b4\ub0c8\ub2e4. \ud0c0\ub974\ud034\ub2c8\uc544\u00b7\uccb4\ub974\ubca0\ud2b8\ub9ac\uc5d0\uc11c\ub294 \uae30\uc6d0\uc804 7\uc138\uae30\uacbd\ubd80\ud130 \uae30\uc6d0\uc804 6\uc138\uae30\uc5d0 \uac78\uccd0\uc11c \uccad\ub3d9\uc81c\ud488\uc758 \uc81c\uc791\uc774 \uc131\ud589\ud558\uace0, \uae30\uc6d0\uc804 6\uc138\uae30 \ud6c4\ubc18, \ubd88\uce58\uc5d0\uc11c\ub294 \uc7a5\uc2dd\uc801 \uc778\ubb3c\uc744 \uc5b9\uac70\ub098 \ubc1c \ubd80\ubd84\uc5d0 \ub3d9\ubb3c\uc758 \ubc1c \ubaa8\uc591\uc744 \ud55c \uc0bc\uac01\ub300(\u4e09\u811a\u81fa)\u00b7\ucd09\ub300\u00b7\ud5a5\ub85c \ub4f1\uc774 \ub300\ub7c9\uc73c\ub85c \uc81c\uc791\ub418\uc5c8\ub2e4. \uccad\ub3d9 \uac70\uc6b8\uacfc \ud0a4\uc2a4\ud0c0\ub77c\uace0 \ubd88\ub9ac\ub294 \uc9c1\uc0ac\uac01\ud615, \ud639\uc740 \uc6d0\ud1b5\ud615\uc758 \ud654\uc7a5\uc0c1\uc790(\u5316\u7ca7\u7bb1\u5b50)\uc5d0 \uc0c8\uae34 \uc120\uac01(\u7dda\u523b)\uc740 \uc5d0\ud2b8\ub8e8\ub9ac\uc544 \uae08\uacf5(\u91d1\u5de5) \uc911\uc5d0\uc11c\ub3c4 \ud2b9\ud788 \ub6f0\uc5b4\ub098\uac8c \uc12c\uc138\ud558\uba70, \uc815\uce58(\u7cbe\u7dfb)\ud55c \uc120\uc758 \ud130\uce58\ub294 \uc720\ub824\ud558\uc5ec \uace0\ub300\uc5d0 \uc788\uc5b4\uc11c \uc720\ub840\ub97c \ucc3e\uc544\ubcfc \uc218 \uc5c6\uc744 \uc815\ub3c4\uc774\ub2e4. \uc120\uac01\uc758 \uc8fc\uc81c\ub294 \ubbf8(\u7f8e)\uc758 \uc5ec\uc2e0\uc774\ub098 \uc2e0\ud654\uc758 \uc0ac\ub791\uc744 \ub2e4\ub8ec \uac83\uc774 \ub9ce\uace0, \uc81c\uc791\uc758 \uc911\uc2ec\uc9c0\ub294 \ud314\ub808\uc2a4\ud2b8\ub9ac\ub098\uc640 \ubd88\uce58\ub77c\uace0 \uc5ec\uaca8\uc9c4\ub2e4. \ubc14\ud2f0\uce78\uc5d0 \uc788\ub294 <\uc5d0\ub85c\uc2a4\uc640 \ucf00\ud30c\ub85c\uc2a4>\uc758 \uc120\uac01\uc774 \uc788\ub294 \uc190\uac70\uc6b8\uc774\ub098 <\ud53c\ucf54\ub85c\ub2c8\uc758 \ud0a4\uc2a4\ud0c0>\ub294 \uccad\ub3d9\uc81c\ud488\uc758 \uc808\ud488(\u7d76\u54c1)\uc774\ub2e4. \uadf8\ub9ac\uace0 \uae30\uc6d0\uc804 5\uc138\uae30\uacbd\uc758 \ucf54\ub974\ud1a0\ub098\uc5d0\uc11c \ucd9c\ud1a0\ub41c \ubd80\uc870(\u6d6e\u5f6b)\uac00 \ubd99\uc740 \ub7a8\ud504\ub3c4 \ubaa8\uc591\uacfc \uc7a5\uc2dd\uc774 \ud638\uc0ac\uc2a4\ub7fd\uace0 \ub610\ud55c \ub3c5\ud2b9\ud55c \uac83\uc73c\ub85c\uc11c, \ub2f9\uc2dc\uc758 \uc5d0\ud2b8\ub8e8\ub9ac\uc544\uc758 \uace0\ub3c4\ud55c \uccad\ub3d9 \uae30\uc220\uc744 \uc804\ud558\ub294 \uc218\uc791(\u79c0\u4f5c)\uc774\ub2e4. \uae08\uc81c\ud488\uc73c\ub85c\ub294 \ube0c\ub85c\uce58\u00b7\uadc0\uace0\ub9ac \ub4f1 \ud2b9\ud788 \uc5ec\uc131\uc758 \uc7a5\uc2e0\uad6c\uc5d0 \ub180\ub784 \ub9cc\ud55c \uae30\ub2a5\uc774 \ubcf4\uc778\ub2e4. \uadf8 \ud55c \uac00\uc9c0\ub85c \uc608\ub85c\uc11c \uccb4\ub974\ubca0\ud2b8\ub9ac\uc758 \uc655\uacfc \uc655\ube44\uc758 \ubb18\uc5d0\uc11c \ucd9c\ud1a0\ub41c \ube0c\ub85c\uce58(\ubc14\ud2f0\uce78 \ubbf8\uc220\uad00)\ub294, \uae30\uc6d0\uc804 7\uc138\uae30 \uc911\uc5fd\uc5d0 \ub9cc\ub4e0 \uac83\uc774\ub77c\uace0 \ud55c\ub2e4. \uc774 \ube0c\ub85c\uce58\ub294 \uc704\ub294 \ub098\ubb47\uc78e\uc758 \ubaa8\uc591\uc744 \ud55c \ubc29\ud328\ub85c\uc11c, \uadf8 \ub458\ub808\uc5d0\ub294 \ub450\uc904\uc758 \ud314\ubbf8\ud2b8\ub85c \uafb8\uba70\uc9c0\uace0, \ub2e4\uc12f\ub9c8\ub9ac\uc758 \uc0ac\uc790\uac00 \uc911\uc559\uc73c\ub85c \ud798\ucc28\uac8c \uac78\uc5b4\ub098\uac00\uba70, \ub610\ud55c \uc544\ub798\uc758 \ub09c\ud615(\u5375\u5f62) \ubc29\ud328\uc5d0\ub294 \uc9d1\uc624\ub9ac\uac00 \uc77c\uacf1\uc904\ub85c \ub098\ub780\ud788 \uc788\ub294 \ub4f1, \uc815\uad50\ud55c \uae30\uc220\uc740 \ub180\ub784 \ub9cc\ud55c \uac83\uc774\ub2e4. \uc5d0\ud2b8\ub8e8\ub9ac\uc544\uc758 \ub3c4\uc790\uae30\ub294 \ud751\uc0c9\uc758 \ubd80\ucf00\ub85c\ub97c \uc81c\uc678\ud558\uace0\ub294 \uadf8 \ubaa8\uc591\ub3c4, \uc7a5\uc2dd\uc758 \uc8fc\uc81c\ub3c4 \ubaa8\ub450 \uadf8\ub9ac\uc2a4\uc758 \ud56d\uc544\ub9ac\uc758 \ubaa8\ubc29\uc774\ub2e4. \ubc29\ub300\ud55c \uadf8\ub9ac\uc2a4 \ub3c4\uc790\uae30\uc758 \uc218\uc785\uc73c\ub85c \uc5d0\ud2b8\ub8e8\ub9ac\uc544\uc758 \uc694\uacf5(\u7aaf\u5de5)\uc740 \ud070 \uc601\ud5a5\uc744 \ubc1b\uc544, \uadf8\ub9ac\uc2a4\uc758 \ubaa8\ubc29\uc73c\ub85c \uc2dc\uc885\ud588\ub2e4. \uc774\uc5d0 \ub300\ud574 \uc5d0\ud2b8\ub8e8\ub9ac\uc544 \ub3c5\ucc3d\uc801\uc778 \ubd80\ucf00\ub85c\ub294 \ub3c4\ud1a0(\u9676\u571f)\ub97c \uadf8\uc744\ub824\uc11c \ub9cc\ub4e0 \uad11\ud0dd\uc774 \ub098\ub294 \ud751\ub3c4(\u9ed1\u9676)\ub85c\uc11c, \uadf8 \uc6d0\ud615\uc740 \ud1a0\ucc29\uc758 \ube4c\ub77c \ub178\ubc14 \ubb38\ud654\uc758 \uc601\ud5a5\uc5d0\uc11c \ubc1c\uc804\ud558\uace0, \uac00\uc7a5 \uc624\ub79c \ubd80\ucf00\ub85c\ub294 \uae30\uc6d0\uc804 8\uc138\uae30\uacbd\uc73c\ub85c \ub3cc\uc544\uac00\uba70, \uae30\uc6d0\uc804 6\uc138\uae30\u00b75\uc138\uae30\uc5d0 \uc81c\uc791\ub41c \ubd80\ucf00\ub85c\ub294 \ub300\ubd80\ubd84\uc774 \ud0a4\uc6b0\uc2dc\uc5d0\uc11c \uc81c\uc791\ub418\uc5c8\uc73c\uba70, \ud615\uc131\uc740 \ubc14\ud0d5\uc774 \ub450\uaecd\uace0 \ud55c\uce35 \uad11\ud0dd\uc744 \ub0b8\ub2e4. \uc77c\ubc18\uc801\uc73c\ub85c \uc5d0\ud2b8\ub8e8\ub9ac\uc544\uc758 \ubd80\ucf00\ub85c\ub294 \uc18c\ubc15\ud558\uc5ec \ud2bc\ud2bc\ud55c \uac83\uc774\ub2e4."} {"question": "2014\ub144 \ubd80\uc5ec\uad70 \uc0b0\uc5c5\uc758 \ucd1d\uc885\uc0ac\uc790 \uc218\ub294?", "paragraph": "2014\ub144 \ubd80\uc5ec\uad70 \uc0b0\uc5c5\uc758 \ucd1d\uc885\uc0ac\uc790 \uc218\ub294 19,482\uba85\uc73c\ub85c \ucda9\uccad\ub0a8\ub3c4 \ucd1d \uc885\uc0ac\uc790 \uc218\uc758 2.4%\ub97c \ucc28\uc9c0\ud558\uace0 \uc788\ub2e4. \uc774\uc911 \ub18d\ub9bc\uc5b4\uc5c5(1\ucc28 \uc0b0\uc5c5)\uc740 349\uba85\uc73c\ub85c \ube44\uc911\uc774 \ub0ae\uace0 1.8%\uc758 \ube44\uc911\uc744 \ucc28\uc9c0\ud558\uace0, \uad11\uc5c5 \ubc0f \uc81c\uc870\uc5c5(2\ucc28 \uc0b0\uc5c5)\uc740 3,342\uba85\uc73c\ub85c 17.2%\uc758 \ube44\uc911\uc73c\ub85c \ucc28\uc9c0\ud558\uace0 \uc0c1\uc5c5 \ubc0f \uc11c\ube44\uc2a4\uc5c5(3\ucc28\uc0b0\uc5c5)\uc740 15,779\uba85\uc73c\ub85c 81.0%\uc758 \ube44\uc911\uc744 \ucc28\uc9c0\ud55c\ub2e4. 2\ucc28 \uc0b0\uc5c5\uc740 \ucda9\uccad\ub0a8\ub3c4 \uc804\uccb4\uc758 \ube44\uc911(32.8%)\ubcf4\ub2e4 \ub0ae\uace0 3\ucc28 \uc0b0\uc5c5\uc740 \ucda9\uccad\ub0a8\ub3c4 \uc804\uccb4\ube44\uc911(66.7%)\ubcf4\ub2e4 \ub192\ub2e4. 3\ucc28 \uc0b0\uc5c5 \ubd80\ubb38\uc5d0\uc11c\ub294 \uc219\ubc15 \ubc0f \uc74c\uc2dd\uc810\uc5c5(10.8%), \ub3c4\uc18c\ub9e4\uc5c5(16.7%)\uacfc \uad50\uc721\uc11c\ube44\uc2a4\uc5c5(8.3%) \ubcf4\uac74\uc5c5 \ubc0f \uc0ac\ud68c\ubcf5\uc9c0\uc11c\ube44\uc2a4\uc5c5(9.4%), \ubb38\ud654 \ubc0f \uae30\ud0c0\uc11c\ube44\uc2a4\uc5c5(8.6%)\uc774 \ud070 \ube44\uc911\uc744 \ucc28\uc9c0\ud558\uace0 \uc788\ub2e4.", "answer": "19,482\uba85", "sentence": "2014\ub144 \ubd80\uc5ec\uad70 \uc0b0\uc5c5\uc758 \ucd1d\uc885\uc0ac\uc790 \uc218\ub294 19,482\uba85 \uc73c\ub85c \ucda9\uccad\ub0a8\ub3c4 \ucd1d \uc885\uc0ac\uc790 \uc218\uc758 2.4%\ub97c \ucc28\uc9c0\ud558\uace0 \uc788\ub2e4.", "paragraph_sentence": " 2014\ub144 \ubd80\uc5ec\uad70 \uc0b0\uc5c5\uc758 \ucd1d\uc885\uc0ac\uc790 \uc218\ub294 19,482\uba85 \uc73c\ub85c \ucda9\uccad\ub0a8\ub3c4 \ucd1d \uc885\uc0ac\uc790 \uc218\uc758 2.4%\ub97c \ucc28\uc9c0\ud558\uace0 \uc788\ub2e4. \uc774\uc911 \ub18d\ub9bc\uc5b4\uc5c5(1\ucc28 \uc0b0\uc5c5)\uc740 349\uba85\uc73c\ub85c \ube44\uc911\uc774 \ub0ae\uace0 1.8%\uc758 \ube44\uc911\uc744 \ucc28\uc9c0\ud558\uace0, \uad11\uc5c5 \ubc0f \uc81c\uc870\uc5c5(2\ucc28 \uc0b0\uc5c5)\uc740 3,342\uba85\uc73c\ub85c 17.2%\uc758 \ube44\uc911\uc73c\ub85c \ucc28\uc9c0\ud558\uace0 \uc0c1\uc5c5 \ubc0f \uc11c\ube44\uc2a4\uc5c5(3\ucc28\uc0b0\uc5c5)\uc740 15,779\uba85\uc73c\ub85c 81.0%\uc758 \ube44\uc911\uc744 \ucc28\uc9c0\ud55c\ub2e4. 2\ucc28 \uc0b0\uc5c5\uc740 \ucda9\uccad\ub0a8\ub3c4 \uc804\uccb4\uc758 \ube44\uc911(32.8%)\ubcf4\ub2e4 \ub0ae\uace0 3\ucc28 \uc0b0\uc5c5\uc740 \ucda9\uccad\ub0a8\ub3c4 \uc804\uccb4\ube44\uc911(66.7%)\ubcf4\ub2e4 \ub192\ub2e4. 3\ucc28 \uc0b0\uc5c5 \ubd80\ubb38\uc5d0\uc11c\ub294 \uc219\ubc15 \ubc0f \uc74c\uc2dd\uc810\uc5c5(10.8%), \ub3c4\uc18c\ub9e4\uc5c5(16.7%)\uacfc \uad50\uc721\uc11c\ube44\uc2a4\uc5c5(8.3%) \ubcf4\uac74\uc5c5 \ubc0f \uc0ac\ud68c\ubcf5\uc9c0\uc11c\ube44\uc2a4\uc5c5(9.4%), \ubb38\ud654 \ubc0f \uae30\ud0c0\uc11c\ube44\uc2a4\uc5c5(8.6%)\uc774 \ud070 \ube44\uc911\uc744 \ucc28\uc9c0\ud558\uace0 \uc788\ub2e4.", "paragraph_answer": "2014\ub144 \ubd80\uc5ec\uad70 \uc0b0\uc5c5\uc758 \ucd1d\uc885\uc0ac\uc790 \uc218\ub294 19,482\uba85 \uc73c\ub85c \ucda9\uccad\ub0a8\ub3c4 \ucd1d \uc885\uc0ac\uc790 \uc218\uc758 2.4%\ub97c \ucc28\uc9c0\ud558\uace0 \uc788\ub2e4. \uc774\uc911 \ub18d\ub9bc\uc5b4\uc5c5(1\ucc28 \uc0b0\uc5c5)\uc740 349\uba85\uc73c\ub85c \ube44\uc911\uc774 \ub0ae\uace0 1.8%\uc758 \ube44\uc911\uc744 \ucc28\uc9c0\ud558\uace0, \uad11\uc5c5 \ubc0f \uc81c\uc870\uc5c5(2\ucc28 \uc0b0\uc5c5)\uc740 3,342\uba85\uc73c\ub85c 17.2%\uc758 \ube44\uc911\uc73c\ub85c \ucc28\uc9c0\ud558\uace0 \uc0c1\uc5c5 \ubc0f \uc11c\ube44\uc2a4\uc5c5(3\ucc28\uc0b0\uc5c5)\uc740 15,779\uba85\uc73c\ub85c 81.0%\uc758 \ube44\uc911\uc744 \ucc28\uc9c0\ud55c\ub2e4. 2\ucc28 \uc0b0\uc5c5\uc740 \ucda9\uccad\ub0a8\ub3c4 \uc804\uccb4\uc758 \ube44\uc911(32.8%)\ubcf4\ub2e4 \ub0ae\uace0 3\ucc28 \uc0b0\uc5c5\uc740 \ucda9\uccad\ub0a8\ub3c4 \uc804\uccb4\ube44\uc911(66.7%)\ubcf4\ub2e4 \ub192\ub2e4. 3\ucc28 \uc0b0\uc5c5 \ubd80\ubb38\uc5d0\uc11c\ub294 \uc219\ubc15 \ubc0f \uc74c\uc2dd\uc810\uc5c5(10.8%), \ub3c4\uc18c\ub9e4\uc5c5(16.7%)\uacfc \uad50\uc721\uc11c\ube44\uc2a4\uc5c5(8.3%) \ubcf4\uac74\uc5c5 \ubc0f \uc0ac\ud68c\ubcf5\uc9c0\uc11c\ube44\uc2a4\uc5c5(9.4%), \ubb38\ud654 \ubc0f \uae30\ud0c0\uc11c\ube44\uc2a4\uc5c5(8.6%)\uc774 \ud070 \ube44\uc911\uc744 \ucc28\uc9c0\ud558\uace0 \uc788\ub2e4.", "sentence_answer": "2014\ub144 \ubd80\uc5ec\uad70 \uc0b0\uc5c5\uc758 \ucd1d\uc885\uc0ac\uc790 \uc218\ub294 19,482\uba85 \uc73c\ub85c \ucda9\uccad\ub0a8\ub3c4 \ucd1d \uc885\uc0ac\uc790 \uc218\uc758 2.4%\ub97c \ucc28\uc9c0\ud558\uace0 \uc788\ub2e4.", "paragraph_id": "6117753-1-0", "paragraph_question": "question: 2014\ub144 \ubd80\uc5ec\uad70 \uc0b0\uc5c5\uc758 \ucd1d\uc885\uc0ac\uc790 \uc218\ub294?, context: 2014\ub144 \ubd80\uc5ec\uad70 \uc0b0\uc5c5\uc758 \ucd1d\uc885\uc0ac\uc790 \uc218\ub294 19,482\uba85\uc73c\ub85c \ucda9\uccad\ub0a8\ub3c4 \ucd1d \uc885\uc0ac\uc790 \uc218\uc758 2.4%\ub97c \ucc28\uc9c0\ud558\uace0 \uc788\ub2e4. \uc774\uc911 \ub18d\ub9bc\uc5b4\uc5c5(1\ucc28 \uc0b0\uc5c5)\uc740 349\uba85\uc73c\ub85c \ube44\uc911\uc774 \ub0ae\uace0 1.8%\uc758 \ube44\uc911\uc744 \ucc28\uc9c0\ud558\uace0, \uad11\uc5c5 \ubc0f \uc81c\uc870\uc5c5(2\ucc28 \uc0b0\uc5c5)\uc740 3,342\uba85\uc73c\ub85c 17.2%\uc758 \ube44\uc911\uc73c\ub85c \ucc28\uc9c0\ud558\uace0 \uc0c1\uc5c5 \ubc0f \uc11c\ube44\uc2a4\uc5c5(3\ucc28\uc0b0\uc5c5)\uc740 15,779\uba85\uc73c\ub85c 81.0%\uc758 \ube44\uc911\uc744 \ucc28\uc9c0\ud55c\ub2e4. 2\ucc28 \uc0b0\uc5c5\uc740 \ucda9\uccad\ub0a8\ub3c4 \uc804\uccb4\uc758 \ube44\uc911(32.8%)\ubcf4\ub2e4 \ub0ae\uace0 3\ucc28 \uc0b0\uc5c5\uc740 \ucda9\uccad\ub0a8\ub3c4 \uc804\uccb4\ube44\uc911(66.7%)\ubcf4\ub2e4 \ub192\ub2e4. 3\ucc28 \uc0b0\uc5c5 \ubd80\ubb38\uc5d0\uc11c\ub294 \uc219\ubc15 \ubc0f \uc74c\uc2dd\uc810\uc5c5(10.8%), \ub3c4\uc18c\ub9e4\uc5c5(16.7%)\uacfc \uad50\uc721\uc11c\ube44\uc2a4\uc5c5(8.3%) \ubcf4\uac74\uc5c5 \ubc0f \uc0ac\ud68c\ubcf5\uc9c0\uc11c\ube44\uc2a4\uc5c5(9.4%), \ubb38\ud654 \ubc0f \uae30\ud0c0\uc11c\ube44\uc2a4\uc5c5(8.6%)\uc774 \ud070 \ube44\uc911\uc744 \ucc28\uc9c0\ud558\uace0 \uc788\ub2e4."} {"question": "2011\ub144\uc5d0 \uc5b4\ub5a4 \uc0c1\uc758 \ub6f0\uc5b4\ub09c \ub2e8\ud3b8 \uc560\ub2c8\uba54\uc774\uc158 \ud504\ub85c\uadf8\ub7a8\uc758 \ud6c4\ubcf4\uc5d0 \uccab \uc5d0\ud53c\uc18c\ub4dc\uac00 \uc62c\ub790\ub294\uac00?", "paragraph": "\uccab \uc5d0\ud53c\uc18c\ub4dc\uc778 \u3008It Came from the Nightosphere\u3009\ub294 \ub300\uccb4\ub85c \ud638\ud3c9\uc744 \ubc1b\uc558\ub2e4. \u300aDVD \ud1a0\ud06c\u300b(DVD Talk)\uc758 \ud0c0\uc77c\ub7ec \ud3ec\uc2a4\ud130(Tyler Foster)\ub294 \ub9c8\ub974\uc140\ub9b0\uacfc \uadf8\uc758 \uc544\ube60 \uc0ac\uc774\uc758 \uac08\ub4f1\uc774 \ub2e4\ub8e8\uc5b4\uc9c0\ub294 \ubc29\uc2dd\uacfc \uadf8 \ub3c4\uc911\uc5d0 \ub098\uc628 \ud3ad\uadc4\uc5d0 \uad00\ud55c \uac1c\uadf8\uac00 \uc88b\uc558\ub2e4\uace0 \uc758\uacac\uc744 \ubc1d\ud614\ub2e4. \ub610\ud55c \u201c\ub0b4 \uc758\uacac\uc73c\ub85c\ub294 \uc5b4\ub5a4 \uc5d0\ud53c\uc18c\ub4dc\uc5d0 \ub178\ub798\uac00 \ub4e4\uc5b4\uac00\ub358 \ud50c\ub7ec\uc2a4 \uc694\uc18c\uac00 \ub420 \uac83\uc774\ub2e4.\u201d\ub77c\uace0 \ub9d0\ud588\ub2e4. \u300aMTV Geek\u300b\uc758 \ucc30\uc2a4 \uc6e8\ube0c(Charles Webb)\ub294 \ub3d9\uba85\uc758 DVD\uc5d0 \uc218\ub85d\ub41c \uc5d0\ud53c\uc18c\ub4dc \uc911\uc5d0\uc11c \uc774 \ud3b8\uc774 \u2018\ucd5c\uace0\uc758 \ud558\uc774\ub77c\uc774\ud2b8\u2019\ub77c\uace0 \uce6d\ud588\ub2e4. IGN\uc758 \uc791\uac00 \ub9e4\ud2b8 \ud30c\uc6b8\ub7ec(Matt Fowler)\ub294 \uc774 \uc5d0\ud53c\uc18c\ub4dc\ub97c \uac00\ub9ac\ucf1c \u2018\uba85\uc791\u2019(classic)\uc774\ub77c\ub294 \ud45c\ud604\uc744 \uc0ac\uc6a9\ud588\ub2e4. \ub610\ud55c 2011\ub144 \ud504\ub77c\uc784\ud0c0\uc784 \uc5d0\ubbf8\uc0c1 \u2018\ub6f0\uc5b4\ub09c \ub2e8\ud3b8 \uc560\ub2c8\uba54\uc774\uc158 \ud504\ub85c\uadf8\ub7a8\u2019\uc758 \ud6c4\ubcf4\uc5d0 \uc624\ub974\uae30\ub3c4 \ud558\uc600\ub2e4.", "answer": "\ud504\ub77c\uc784\ud0c0\uc784 \uc5d0\ubbf8\uc0c1", "sentence": "\ub610\ud55c 2011\ub144 \ud504\ub77c\uc784\ud0c0\uc784 \uc5d0\ubbf8\uc0c1 \u2018\ub6f0\uc5b4\ub09c \ub2e8\ud3b8 \uc560\ub2c8\uba54\uc774\uc158 \ud504\ub85c\uadf8\ub7a8\u2019\uc758 \ud6c4\ubcf4\uc5d0 \uc624\ub974\uae30\ub3c4 \ud558\uc600\ub2e4.", "paragraph_sentence": "\uccab \uc5d0\ud53c\uc18c\ub4dc\uc778 \u3008It Came from the Nightosphere\u3009\ub294 \ub300\uccb4\ub85c \ud638\ud3c9\uc744 \ubc1b\uc558\ub2e4. \u300aDVD \ud1a0\ud06c\u300b(DVD Talk)\uc758 \ud0c0\uc77c\ub7ec \ud3ec\uc2a4\ud130(Tyler Foster)\ub294 \ub9c8\ub974\uc140\ub9b0\uacfc \uadf8\uc758 \uc544\ube60 \uc0ac\uc774\uc758 \uac08\ub4f1\uc774 \ub2e4\ub8e8\uc5b4\uc9c0\ub294 \ubc29\uc2dd\uacfc \uadf8 \ub3c4\uc911\uc5d0 \ub098\uc628 \ud3ad\uadc4\uc5d0 \uad00\ud55c \uac1c\uadf8\uac00 \uc88b\uc558\ub2e4\uace0 \uc758\uacac\uc744 \ubc1d\ud614\ub2e4. \ub610\ud55c \u201c\ub0b4 \uc758\uacac\uc73c\ub85c\ub294 \uc5b4\ub5a4 \uc5d0\ud53c\uc18c\ub4dc\uc5d0 \ub178\ub798\uac00 \ub4e4\uc5b4\uac00\ub358 \ud50c\ub7ec\uc2a4 \uc694\uc18c\uac00 \ub420 \uac83\uc774\ub2e4. \u201d\ub77c\uace0 \ub9d0\ud588\ub2e4. \u300aMTV Geek\u300b\uc758 \ucc30\uc2a4 \uc6e8\ube0c(Charles Webb)\ub294 \ub3d9\uba85\uc758 DVD\uc5d0 \uc218\ub85d\ub41c \uc5d0\ud53c\uc18c\ub4dc \uc911\uc5d0\uc11c \uc774 \ud3b8\uc774 \u2018\ucd5c\uace0\uc758 \ud558\uc774\ub77c\uc774\ud2b8\u2019\ub77c\uace0 \uce6d\ud588\ub2e4. IGN\uc758 \uc791\uac00 \ub9e4\ud2b8 \ud30c\uc6b8\ub7ec(Matt Fowler)\ub294 \uc774 \uc5d0\ud53c\uc18c\ub4dc\ub97c \uac00\ub9ac\ucf1c \u2018\uba85\uc791\u2019(classic)\uc774\ub77c\ub294 \ud45c\ud604\uc744 \uc0ac\uc6a9\ud588\ub2e4. \ub610\ud55c 2011\ub144 \ud504\ub77c\uc784\ud0c0\uc784 \uc5d0\ubbf8\uc0c1 \u2018\ub6f0\uc5b4\ub09c \ub2e8\ud3b8 \uc560\ub2c8\uba54\uc774\uc158 \ud504\ub85c\uadf8\ub7a8\u2019\uc758 \ud6c4\ubcf4\uc5d0 \uc624\ub974\uae30\ub3c4 \ud558\uc600\ub2e4. ", "paragraph_answer": "\uccab \uc5d0\ud53c\uc18c\ub4dc\uc778 \u3008It Came from the Nightosphere\u3009\ub294 \ub300\uccb4\ub85c \ud638\ud3c9\uc744 \ubc1b\uc558\ub2e4. \u300aDVD \ud1a0\ud06c\u300b(DVD Talk)\uc758 \ud0c0\uc77c\ub7ec \ud3ec\uc2a4\ud130(Tyler Foster)\ub294 \ub9c8\ub974\uc140\ub9b0\uacfc \uadf8\uc758 \uc544\ube60 \uc0ac\uc774\uc758 \uac08\ub4f1\uc774 \ub2e4\ub8e8\uc5b4\uc9c0\ub294 \ubc29\uc2dd\uacfc \uadf8 \ub3c4\uc911\uc5d0 \ub098\uc628 \ud3ad\uadc4\uc5d0 \uad00\ud55c \uac1c\uadf8\uac00 \uc88b\uc558\ub2e4\uace0 \uc758\uacac\uc744 \ubc1d\ud614\ub2e4. \ub610\ud55c \u201c\ub0b4 \uc758\uacac\uc73c\ub85c\ub294 \uc5b4\ub5a4 \uc5d0\ud53c\uc18c\ub4dc\uc5d0 \ub178\ub798\uac00 \ub4e4\uc5b4\uac00\ub358 \ud50c\ub7ec\uc2a4 \uc694\uc18c\uac00 \ub420 \uac83\uc774\ub2e4.\u201d\ub77c\uace0 \ub9d0\ud588\ub2e4. \u300aMTV Geek\u300b\uc758 \ucc30\uc2a4 \uc6e8\ube0c(Charles Webb)\ub294 \ub3d9\uba85\uc758 DVD\uc5d0 \uc218\ub85d\ub41c \uc5d0\ud53c\uc18c\ub4dc \uc911\uc5d0\uc11c \uc774 \ud3b8\uc774 \u2018\ucd5c\uace0\uc758 \ud558\uc774\ub77c\uc774\ud2b8\u2019\ub77c\uace0 \uce6d\ud588\ub2e4. IGN\uc758 \uc791\uac00 \ub9e4\ud2b8 \ud30c\uc6b8\ub7ec(Matt Fowler)\ub294 \uc774 \uc5d0\ud53c\uc18c\ub4dc\ub97c \uac00\ub9ac\ucf1c \u2018\uba85\uc791\u2019(classic)\uc774\ub77c\ub294 \ud45c\ud604\uc744 \uc0ac\uc6a9\ud588\ub2e4. \ub610\ud55c 2011\ub144 \ud504\ub77c\uc784\ud0c0\uc784 \uc5d0\ubbf8\uc0c1 \u2018\ub6f0\uc5b4\ub09c \ub2e8\ud3b8 \uc560\ub2c8\uba54\uc774\uc158 \ud504\ub85c\uadf8\ub7a8\u2019\uc758 \ud6c4\ubcf4\uc5d0 \uc624\ub974\uae30\ub3c4 \ud558\uc600\ub2e4.", "sentence_answer": "\ub610\ud55c 2011\ub144 \ud504\ub77c\uc784\ud0c0\uc784 \uc5d0\ubbf8\uc0c1 \u2018\ub6f0\uc5b4\ub09c \ub2e8\ud3b8 \uc560\ub2c8\uba54\uc774\uc158 \ud504\ub85c\uadf8\ub7a8\u2019\uc758 \ud6c4\ubcf4\uc5d0 \uc624\ub974\uae30\ub3c4 \ud558\uc600\ub2e4.", "paragraph_id": "6569379-7-2", "paragraph_question": "question: 2011\ub144\uc5d0 \uc5b4\ub5a4 \uc0c1\uc758 \ub6f0\uc5b4\ub09c \ub2e8\ud3b8 \uc560\ub2c8\uba54\uc774\uc158 \ud504\ub85c\uadf8\ub7a8\uc758 \ud6c4\ubcf4\uc5d0 \uccab \uc5d0\ud53c\uc18c\ub4dc\uac00 \uc62c\ub790\ub294\uac00?, context: \uccab \uc5d0\ud53c\uc18c\ub4dc\uc778 \u3008It Came from the Nightosphere\u3009\ub294 \ub300\uccb4\ub85c \ud638\ud3c9\uc744 \ubc1b\uc558\ub2e4. \u300aDVD \ud1a0\ud06c\u300b(DVD Talk)\uc758 \ud0c0\uc77c\ub7ec \ud3ec\uc2a4\ud130(Tyler Foster)\ub294 \ub9c8\ub974\uc140\ub9b0\uacfc \uadf8\uc758 \uc544\ube60 \uc0ac\uc774\uc758 \uac08\ub4f1\uc774 \ub2e4\ub8e8\uc5b4\uc9c0\ub294 \ubc29\uc2dd\uacfc \uadf8 \ub3c4\uc911\uc5d0 \ub098\uc628 \ud3ad\uadc4\uc5d0 \uad00\ud55c \uac1c\uadf8\uac00 \uc88b\uc558\ub2e4\uace0 \uc758\uacac\uc744 \ubc1d\ud614\ub2e4. \ub610\ud55c \u201c\ub0b4 \uc758\uacac\uc73c\ub85c\ub294 \uc5b4\ub5a4 \uc5d0\ud53c\uc18c\ub4dc\uc5d0 \ub178\ub798\uac00 \ub4e4\uc5b4\uac00\ub358 \ud50c\ub7ec\uc2a4 \uc694\uc18c\uac00 \ub420 \uac83\uc774\ub2e4.\u201d\ub77c\uace0 \ub9d0\ud588\ub2e4. \u300aMTV Geek\u300b\uc758 \ucc30\uc2a4 \uc6e8\ube0c(Charles Webb)\ub294 \ub3d9\uba85\uc758 DVD\uc5d0 \uc218\ub85d\ub41c \uc5d0\ud53c\uc18c\ub4dc \uc911\uc5d0\uc11c \uc774 \ud3b8\uc774 \u2018\ucd5c\uace0\uc758 \ud558\uc774\ub77c\uc774\ud2b8\u2019\ub77c\uace0 \uce6d\ud588\ub2e4. IGN\uc758 \uc791\uac00 \ub9e4\ud2b8 \ud30c\uc6b8\ub7ec(Matt Fowler)\ub294 \uc774 \uc5d0\ud53c\uc18c\ub4dc\ub97c \uac00\ub9ac\ucf1c \u2018\uba85\uc791\u2019(classic)\uc774\ub77c\ub294 \ud45c\ud604\uc744 \uc0ac\uc6a9\ud588\ub2e4. \ub610\ud55c 2011\ub144 \ud504\ub77c\uc784\ud0c0\uc784 \uc5d0\ubbf8\uc0c1 \u2018\ub6f0\uc5b4\ub09c \ub2e8\ud3b8 \uc560\ub2c8\uba54\uc774\uc158 \ud504\ub85c\uadf8\ub7a8\u2019\uc758 \ud6c4\ubcf4\uc5d0 \uc624\ub974\uae30\ub3c4 \ud558\uc600\ub2e4."} {"question": "\uc624\ub85c\ub77c\uac00 \uac00\uc7a5 \uc790\uc8fc\ubcf4\uc774\ub294 \uc9c0\uc5ed\uc744 \uc77c\uceeb\ub294 \ub9d0\uc740?", "paragraph": "\uc624\ub85c\ub77c\uac00 \uac00\uc7a5 \uc790\uc8fc \ubcf4\uc774\ub294 \uacf3\uc740 \ub0a8\ubd81 \uc591\uadf9\uc9c0\ubc29\uc758 \uc9c0\uad6c\uc790\uae30\uc704\ub3c4 65\u223c70\ub3c4\uc758 \ubc94\uc704\ub85c\uc11c \uc774 \uc9c0\uc5ed\uc744 \uc624\ub85c\ub77c\ub300(auroral zone)\ub77c\uace0 \ud55c\ub2e4. \uc624\ub85c\ub77c\ub300\ubcf4\ub2e4 \uace0\uc704\ub3c4(\uadf9\uad00\uc9c0\uc5ed)\ub098 \uc800\uc704\ub3c4\uc5d0\uc11c\uc758 \ucd9c\ud604\ube48\ub3c4\ub294 \uac10\uc18c\ud55c\ub2e4. \ucd9c\ud604\ud558\ub294 \uc704\ub3c4\ub294 \uc9c0\ubc29\uc2dc(\u5730\u65b9\u6642)\uc5d0 \ub530\ub77c \ub2e4\ub974\uba70, \uc57c\uac04\uc5d0\ub294 65\u223c70\ub3c4\uc5d0 \ub9ce\uc73c\uba70, \uc8fc\uac04\uc5d0\ub294 75\u223c80\ub3c4\ub85c \uc704\ub3c4\uac00 \ub192\uc544\uc9c4\ub2e4. \uc774\ub807\uac8c \uc624\ub85c\ub77c\uac00 \ucd9c\ud604\ud558\ub294 \uc704\ub3c4\uac00 \uc9c0\ubc29\uc2dc\uc5d0 \ub530\ub77c \ubcc0\ud654\ub97c \ubcf4\uc774\uae30 \ub54c\ubb38\uc5d0 \uc624\ub85c\ub77c\ucd9c\ud604\ub300(\uc9c0\uad6c\ub97c \uadf9\uc9c0\uc758 \uc0c1\uacf5\uc5d0\uc11c \ub0b4\ub824\ub2e4 \ubcf4\uc558\uc744 \ub54c \ub3d9\uc2dc\uc5d0 \uc624\ub85c\ub77c\uac00 \ubcf4\uc774\ub294 \uc601\uc5ed)\ub97c \uc624\ub85c\ub77c\ub300\uc640 \uad6c\ubcc4\ud574\uc11c \uc624\ub85c\ub77c \uc624\ubc8c(aurora oval)\uc774\ub77c\uace0 \ud55c\ub2e4. \uc77c\ub828\uc758 \uc624\ub85c\ub77c\uc624\ubc8c\uc740 \ub300\uac1c 2\uc885\ub958\uc758 \uc624\ub85c\ub77c\ub85c \uad6c\uc131\ub418\ub294\ub370, \ub0ae\uc5d0\uc11c \uc800\ub141\uc744 \uac70\uccd0 \uc2ec\uc57c\uc5d0 \uc774\ub974\ub294 \uc2dc\uac04\uc5d0\ub294 \ucee4\ud2bc\ud615\uc624\ub85c\ub77c\uc774\uace0, \uadf8 \uc774\ud6c4 \uc544\uce68\uae4c\uc9c0\uc758 \ubc18(\u534a)\uc740 \uc8fc\ub85c \ub9e5\ub3d9\uc131\uc624\ub85c\ub77c\ub85c\uc11c \uc77c\ubc18\uc801\uc73c\ub85c \uc5f7\uc740 \ubc30\uacbd\uc73c\ub85c \ub3d9\ubc18\ud55c\ub2e4. \uacfc\uac70\uc5d0\ub294 \uad00\uce21\uc7a5\uce58\uc758 \uac10\ub3c4\ubd80\uc871\uc73c\ub85c \uc778\ud574\uc11c \ub9e5\ub3d9\uc131\uc624\ub85c\ub77c\ub97c \ucda9\ubd84\ud788 \uad00\uce21\ud560 \uc218 \uc5c6\uae30 \ub54c\ubb38\uc5d0 \uc774 \ubd80\ubd84\uc744 \ud76c\ubbf8\ud55c \ubd80\uc815\ud615\uc624\ub85c\ub77c\ub77c\uace0 \ud588\uc5c8\ub2e4.", "answer": "\uc624\ub85c\ub77c\ub300", "sentence": "\uc774 \uc9c0\uc5ed\uc744 \uc624\ub85c\ub77c\ub300 (auroral zone)\ub77c\uace0 \ud55c\ub2e4.", "paragraph_sentence": "\uc624\ub85c\ub77c\uac00 \uac00\uc7a5 \uc790\uc8fc \ubcf4\uc774\ub294 \uacf3\uc740 \ub0a8\ubd81 \uc591\uadf9\uc9c0\ubc29\uc758 \uc9c0\uad6c\uc790\uae30\uc704\ub3c4 65\u223c70\ub3c4\uc758 \ubc94\uc704\ub85c\uc11c \uc774 \uc9c0\uc5ed\uc744 \uc624\ub85c\ub77c\ub300 (auroral zone)\ub77c\uace0 \ud55c\ub2e4. \uc624\ub85c\ub77c\ub300\ubcf4\ub2e4 \uace0\uc704\ub3c4(\uadf9\uad00\uc9c0\uc5ed)\ub098 \uc800\uc704\ub3c4\uc5d0\uc11c\uc758 \ucd9c\ud604\ube48\ub3c4\ub294 \uac10\uc18c\ud55c\ub2e4. \ucd9c\ud604\ud558\ub294 \uc704\ub3c4\ub294 \uc9c0\ubc29\uc2dc(\u5730\u65b9\u6642)\uc5d0 \ub530\ub77c \ub2e4\ub974\uba70, \uc57c\uac04\uc5d0\ub294 65\u223c70\ub3c4\uc5d0 \ub9ce\uc73c\uba70, \uc8fc\uac04\uc5d0\ub294 75\u223c80\ub3c4\ub85c \uc704\ub3c4\uac00 \ub192\uc544\uc9c4\ub2e4. \uc774\ub807\uac8c \uc624\ub85c\ub77c\uac00 \ucd9c\ud604\ud558\ub294 \uc704\ub3c4\uac00 \uc9c0\ubc29\uc2dc\uc5d0 \ub530\ub77c \ubcc0\ud654\ub97c \ubcf4\uc774\uae30 \ub54c\ubb38\uc5d0 \uc624\ub85c\ub77c\ucd9c\ud604\ub300(\uc9c0\uad6c\ub97c \uadf9\uc9c0\uc758 \uc0c1\uacf5\uc5d0\uc11c \ub0b4\ub824\ub2e4 \ubcf4\uc558\uc744 \ub54c \ub3d9\uc2dc\uc5d0 \uc624\ub85c\ub77c\uac00 \ubcf4\uc774\ub294 \uc601\uc5ed)\ub97c \uc624\ub85c\ub77c\ub300\uc640 \uad6c\ubcc4\ud574\uc11c \uc624\ub85c\ub77c \uc624\ubc8c(aurora oval)\uc774\ub77c\uace0 \ud55c\ub2e4. \uc77c\ub828\uc758 \uc624\ub85c\ub77c\uc624\ubc8c\uc740 \ub300\uac1c 2\uc885\ub958\uc758 \uc624\ub85c\ub77c\ub85c \uad6c\uc131\ub418\ub294\ub370, \ub0ae\uc5d0\uc11c \uc800\ub141\uc744 \uac70\uccd0 \uc2ec\uc57c\uc5d0 \uc774\ub974\ub294 \uc2dc\uac04\uc5d0\ub294 \ucee4\ud2bc\ud615\uc624\ub85c\ub77c\uc774\uace0, \uadf8 \uc774\ud6c4 \uc544\uce68\uae4c\uc9c0\uc758 \ubc18(\u534a)\uc740 \uc8fc\ub85c \ub9e5\ub3d9\uc131\uc624\ub85c\ub77c\ub85c\uc11c \uc77c\ubc18\uc801\uc73c\ub85c \uc5f7\uc740 \ubc30\uacbd\uc73c\ub85c \ub3d9\ubc18\ud55c\ub2e4. \uacfc\uac70\uc5d0\ub294 \uad00\uce21\uc7a5\uce58\uc758 \uac10\ub3c4\ubd80\uc871\uc73c\ub85c \uc778\ud574\uc11c \ub9e5\ub3d9\uc131\uc624\ub85c\ub77c\ub97c \ucda9\ubd84\ud788 \uad00\uce21\ud560 \uc218 \uc5c6\uae30 \ub54c\ubb38\uc5d0 \uc774 \ubd80\ubd84\uc744 \ud76c\ubbf8\ud55c \ubd80\uc815\ud615\uc624\ub85c\ub77c\ub77c\uace0 \ud588\uc5c8\ub2e4.", "paragraph_answer": "\uc624\ub85c\ub77c\uac00 \uac00\uc7a5 \uc790\uc8fc \ubcf4\uc774\ub294 \uacf3\uc740 \ub0a8\ubd81 \uc591\uadf9\uc9c0\ubc29\uc758 \uc9c0\uad6c\uc790\uae30\uc704\ub3c4 65\u223c70\ub3c4\uc758 \ubc94\uc704\ub85c\uc11c \uc774 \uc9c0\uc5ed\uc744 \uc624\ub85c\ub77c\ub300 (auroral zone)\ub77c\uace0 \ud55c\ub2e4. \uc624\ub85c\ub77c\ub300\ubcf4\ub2e4 \uace0\uc704\ub3c4(\uadf9\uad00\uc9c0\uc5ed)\ub098 \uc800\uc704\ub3c4\uc5d0\uc11c\uc758 \ucd9c\ud604\ube48\ub3c4\ub294 \uac10\uc18c\ud55c\ub2e4. \ucd9c\ud604\ud558\ub294 \uc704\ub3c4\ub294 \uc9c0\ubc29\uc2dc(\u5730\u65b9\u6642)\uc5d0 \ub530\ub77c \ub2e4\ub974\uba70, \uc57c\uac04\uc5d0\ub294 65\u223c70\ub3c4\uc5d0 \ub9ce\uc73c\uba70, \uc8fc\uac04\uc5d0\ub294 75\u223c80\ub3c4\ub85c \uc704\ub3c4\uac00 \ub192\uc544\uc9c4\ub2e4. \uc774\ub807\uac8c \uc624\ub85c\ub77c\uac00 \ucd9c\ud604\ud558\ub294 \uc704\ub3c4\uac00 \uc9c0\ubc29\uc2dc\uc5d0 \ub530\ub77c \ubcc0\ud654\ub97c \ubcf4\uc774\uae30 \ub54c\ubb38\uc5d0 \uc624\ub85c\ub77c\ucd9c\ud604\ub300(\uc9c0\uad6c\ub97c \uadf9\uc9c0\uc758 \uc0c1\uacf5\uc5d0\uc11c \ub0b4\ub824\ub2e4 \ubcf4\uc558\uc744 \ub54c \ub3d9\uc2dc\uc5d0 \uc624\ub85c\ub77c\uac00 \ubcf4\uc774\ub294 \uc601\uc5ed)\ub97c \uc624\ub85c\ub77c\ub300\uc640 \uad6c\ubcc4\ud574\uc11c \uc624\ub85c\ub77c \uc624\ubc8c(aurora oval)\uc774\ub77c\uace0 \ud55c\ub2e4. \uc77c\ub828\uc758 \uc624\ub85c\ub77c\uc624\ubc8c\uc740 \ub300\uac1c 2\uc885\ub958\uc758 \uc624\ub85c\ub77c\ub85c \uad6c\uc131\ub418\ub294\ub370, \ub0ae\uc5d0\uc11c \uc800\ub141\uc744 \uac70\uccd0 \uc2ec\uc57c\uc5d0 \uc774\ub974\ub294 \uc2dc\uac04\uc5d0\ub294 \ucee4\ud2bc\ud615\uc624\ub85c\ub77c\uc774\uace0, \uadf8 \uc774\ud6c4 \uc544\uce68\uae4c\uc9c0\uc758 \ubc18(\u534a)\uc740 \uc8fc\ub85c \ub9e5\ub3d9\uc131\uc624\ub85c\ub77c\ub85c\uc11c \uc77c\ubc18\uc801\uc73c\ub85c \uc5f7\uc740 \ubc30\uacbd\uc73c\ub85c \ub3d9\ubc18\ud55c\ub2e4. \uacfc\uac70\uc5d0\ub294 \uad00\uce21\uc7a5\uce58\uc758 \uac10\ub3c4\ubd80\uc871\uc73c\ub85c \uc778\ud574\uc11c \ub9e5\ub3d9\uc131\uc624\ub85c\ub77c\ub97c \ucda9\ubd84\ud788 \uad00\uce21\ud560 \uc218 \uc5c6\uae30 \ub54c\ubb38\uc5d0 \uc774 \ubd80\ubd84\uc744 \ud76c\ubbf8\ud55c \ubd80\uc815\ud615\uc624\ub85c\ub77c\ub77c\uace0 \ud588\uc5c8\ub2e4.", "sentence_answer": "\uc774 \uc9c0\uc5ed\uc744 \uc624\ub85c\ub77c\ub300 (auroral zone)\ub77c\uace0 \ud55c\ub2e4.", "paragraph_id": "6657423-0-0", "paragraph_question": "question: \uc624\ub85c\ub77c\uac00 \uac00\uc7a5 \uc790\uc8fc\ubcf4\uc774\ub294 \uc9c0\uc5ed\uc744 \uc77c\uceeb\ub294 \ub9d0\uc740?, context: \uc624\ub85c\ub77c\uac00 \uac00\uc7a5 \uc790\uc8fc \ubcf4\uc774\ub294 \uacf3\uc740 \ub0a8\ubd81 \uc591\uadf9\uc9c0\ubc29\uc758 \uc9c0\uad6c\uc790\uae30\uc704\ub3c4 65\u223c70\ub3c4\uc758 \ubc94\uc704\ub85c\uc11c \uc774 \uc9c0\uc5ed\uc744 \uc624\ub85c\ub77c\ub300(auroral zone)\ub77c\uace0 \ud55c\ub2e4. \uc624\ub85c\ub77c\ub300\ubcf4\ub2e4 \uace0\uc704\ub3c4(\uadf9\uad00\uc9c0\uc5ed)\ub098 \uc800\uc704\ub3c4\uc5d0\uc11c\uc758 \ucd9c\ud604\ube48\ub3c4\ub294 \uac10\uc18c\ud55c\ub2e4. \ucd9c\ud604\ud558\ub294 \uc704\ub3c4\ub294 \uc9c0\ubc29\uc2dc(\u5730\u65b9\u6642)\uc5d0 \ub530\ub77c \ub2e4\ub974\uba70, \uc57c\uac04\uc5d0\ub294 65\u223c70\ub3c4\uc5d0 \ub9ce\uc73c\uba70, \uc8fc\uac04\uc5d0\ub294 75\u223c80\ub3c4\ub85c \uc704\ub3c4\uac00 \ub192\uc544\uc9c4\ub2e4. \uc774\ub807\uac8c \uc624\ub85c\ub77c\uac00 \ucd9c\ud604\ud558\ub294 \uc704\ub3c4\uac00 \uc9c0\ubc29\uc2dc\uc5d0 \ub530\ub77c \ubcc0\ud654\ub97c \ubcf4\uc774\uae30 \ub54c\ubb38\uc5d0 \uc624\ub85c\ub77c\ucd9c\ud604\ub300(\uc9c0\uad6c\ub97c \uadf9\uc9c0\uc758 \uc0c1\uacf5\uc5d0\uc11c \ub0b4\ub824\ub2e4 \ubcf4\uc558\uc744 \ub54c \ub3d9\uc2dc\uc5d0 \uc624\ub85c\ub77c\uac00 \ubcf4\uc774\ub294 \uc601\uc5ed)\ub97c \uc624\ub85c\ub77c\ub300\uc640 \uad6c\ubcc4\ud574\uc11c \uc624\ub85c\ub77c \uc624\ubc8c(aurora oval)\uc774\ub77c\uace0 \ud55c\ub2e4. \uc77c\ub828\uc758 \uc624\ub85c\ub77c\uc624\ubc8c\uc740 \ub300\uac1c 2\uc885\ub958\uc758 \uc624\ub85c\ub77c\ub85c \uad6c\uc131\ub418\ub294\ub370, \ub0ae\uc5d0\uc11c \uc800\ub141\uc744 \uac70\uccd0 \uc2ec\uc57c\uc5d0 \uc774\ub974\ub294 \uc2dc\uac04\uc5d0\ub294 \ucee4\ud2bc\ud615\uc624\ub85c\ub77c\uc774\uace0, \uadf8 \uc774\ud6c4 \uc544\uce68\uae4c\uc9c0\uc758 \ubc18(\u534a)\uc740 \uc8fc\ub85c \ub9e5\ub3d9\uc131\uc624\ub85c\ub77c\ub85c\uc11c \uc77c\ubc18\uc801\uc73c\ub85c \uc5f7\uc740 \ubc30\uacbd\uc73c\ub85c \ub3d9\ubc18\ud55c\ub2e4. \uacfc\uac70\uc5d0\ub294 \uad00\uce21\uc7a5\uce58\uc758 \uac10\ub3c4\ubd80\uc871\uc73c\ub85c \uc778\ud574\uc11c \ub9e5\ub3d9\uc131\uc624\ub85c\ub77c\ub97c \ucda9\ubd84\ud788 \uad00\uce21\ud560 \uc218 \uc5c6\uae30 \ub54c\ubb38\uc5d0 \uc774 \ubd80\ubd84\uc744 \ud76c\ubbf8\ud55c \ubd80\uc815\ud615\uc624\ub85c\ub77c\ub77c\uace0 \ud588\uc5c8\ub2e4."} {"question": "\uc804\ub450\ud658\uc774 \ubcf4\uc548\uc0ac\ub839\uad00\uc5d0\uc11c \uc0ac\uc784\ud558\uace0 \uadf8\uc758 \ud6c4\uc784\uc73c\ub85c \uc784\uba85\ub41c \uc778\ubb3c\uc740?", "paragraph": "1980\ub144 8\uc6d4 6\uc77c \uc721\uad70 \ub300\uc7a5(\u5927\u5c07)\uc73c\ub85c \uc9c4\uae09\ud55c \uc804\ub450\ud658\uc740 \uace7 \ub300\ud1b5\ub839 \uc120\uac70 \ucd9c\ub9c8\ub97c \uc120\uc5b8\ud588\ub2e4. \ubcf4\uc548\uc0ac\uc5d0 \uc758\ud574 \uac80\uc5f4\uacfc \uc870\uc885\uc744 \ubc1b\uc558\ub358 \ud55c\uad6d \uc5b8\ub860\uc740 \ubbf8\uad6d\uc758 \ub85c\uc2a4\uc564\uc824\ub808\uc2a4 \ud0c0\uc784\uc2a4\uac00 \uc804\ub450\ud658 \uc7a5\uad70\uc744 \uc9c0\uc9c0\ud55c\ub2e4\uace0 \ubcf4\ub3c4\ud588\ub2e4. \ud558\uc9c0\ub9cc \uc774\ub294 \ubbf8\uad6d \uc778\uc0ac\ub4e4\uc758 \ubc1c\uc5b8\uc758 \uc9c4\uc758\ub97c \ud6fc\uc190\ud55c \uc624\ubcf4\uc600\ub2e4. 8\uc6d4 16\uc77c \ucd5c\uaddc\ud558 \ub300\ud1b5\ub839\uc774 \uc0ac\uc784\ud588\uace0 8\uc6d4 22\uc77c \uc804\ub450\ud658\uc774 \uad70\ub300\uc5d0\uc11c \uc804\uc5ed\ud588\ub2e4. 8\uc6d4 23\uc77c \uc804\ub450\ud658\uc740 \ubcf4\uc548\uc0ac\ub839\uad00\uc5d0\uc11c \uc0ac\uc784\ud558\uace0 \ub2f9\uc2dc \uc218\ub3c4\uacbd\ube44\uc0ac\ub839\uad00\uc774\ub358 \ub178\ud0dc\uc6b0\uac00 \ud6c4\uc784\uc73c\ub85c \uc784\uba85\ub418\uc5c8\ub2e4. 8\uc6d4 25\uc77c \ud544\ub9ac\ud540\uc758 \uc774\uba5c\ub2e4 \ub9c8\ub974\ucf54\uc2a4\ub294 \ud3c9\ud654\ub97c \uc0ac\ub791\ud558\ub294 \uc0ac\ub78c\uc740 \uc804\ub450\ud658\uc744 \uc9c0\uc9c0\ud574\uc57c \ud55c\ub2e4\uace0 \ud588\ub2e4. 8\uc6d4 27\uc77c \uc7a5\ucda9\uccb4\uc721\uad00\uc5d0\uc11c \ud1b5\uc77c\uc8fc\uccb4\uad6d\ubbfc\ud68c\uc758 \ub300\uc758\uc6d0\uc5d0 \uc758\ud574 \uc81c11\ub300 \ub300\ud1b5\ub839\uc73c\ub85c \uc120\ucd9c\ub41c \ud6c4 9\uc6d4 1\uc77c \uc7a0\uc2e4\uccb4\uc721\uad00\uc5d0\uc11c \uc81c11\ub300 \ub300\ud1b5\ub839\uc5d0 \ucde8\uc784\ud588\ub2e4.", "answer": "\ub178\ud0dc\uc6b0", "sentence": "8\uc6d4 23\uc77c \uc804\ub450\ud658\uc740 \ubcf4\uc548\uc0ac\ub839\uad00\uc5d0\uc11c \uc0ac\uc784\ud558\uace0 \ub2f9\uc2dc \uc218\ub3c4\uacbd\ube44\uc0ac\ub839\uad00\uc774\ub358 \ub178\ud0dc\uc6b0 \uac00 \ud6c4\uc784\uc73c\ub85c \uc784\uba85\ub418\uc5c8\ub2e4.", "paragraph_sentence": "1980\ub144 8\uc6d4 6\uc77c \uc721\uad70 \ub300\uc7a5(\u5927\u5c07)\uc73c\ub85c \uc9c4\uae09\ud55c \uc804\ub450\ud658\uc740 \uace7 \ub300\ud1b5\ub839 \uc120\uac70 \ucd9c\ub9c8\ub97c \uc120\uc5b8\ud588\ub2e4. \ubcf4\uc548\uc0ac\uc5d0 \uc758\ud574 \uac80\uc5f4\uacfc \uc870\uc885\uc744 \ubc1b\uc558\ub358 \ud55c\uad6d \uc5b8\ub860\uc740 \ubbf8\uad6d\uc758 \ub85c\uc2a4\uc564\uc824\ub808\uc2a4 \ud0c0\uc784\uc2a4\uac00 \uc804\ub450\ud658 \uc7a5\uad70\uc744 \uc9c0\uc9c0\ud55c\ub2e4\uace0 \ubcf4\ub3c4\ud588\ub2e4. \ud558\uc9c0\ub9cc \uc774\ub294 \ubbf8\uad6d \uc778\uc0ac\ub4e4\uc758 \ubc1c\uc5b8\uc758 \uc9c4\uc758\ub97c \ud6fc\uc190\ud55c \uc624\ubcf4\uc600\ub2e4. 8\uc6d4 16\uc77c \ucd5c\uaddc\ud558 \ub300\ud1b5\ub839\uc774 \uc0ac\uc784\ud588\uace0 8\uc6d4 22\uc77c \uc804\ub450\ud658\uc774 \uad70\ub300\uc5d0\uc11c \uc804\uc5ed\ud588\ub2e4. 8\uc6d4 23\uc77c \uc804\ub450\ud658\uc740 \ubcf4\uc548\uc0ac\ub839\uad00\uc5d0\uc11c \uc0ac\uc784\ud558\uace0 \ub2f9\uc2dc \uc218\ub3c4\uacbd\ube44\uc0ac\ub839\uad00\uc774\ub358 \ub178\ud0dc\uc6b0 \uac00 \ud6c4\uc784\uc73c\ub85c \uc784\uba85\ub418\uc5c8\ub2e4. 8\uc6d4 25\uc77c \ud544\ub9ac\ud540\uc758 \uc774\uba5c\ub2e4 \ub9c8\ub974\ucf54\uc2a4\ub294 \ud3c9\ud654\ub97c \uc0ac\ub791\ud558\ub294 \uc0ac\ub78c\uc740 \uc804\ub450\ud658\uc744 \uc9c0\uc9c0\ud574\uc57c \ud55c\ub2e4\uace0 \ud588\ub2e4. 8\uc6d4 27\uc77c \uc7a5\ucda9\uccb4\uc721\uad00\uc5d0\uc11c \ud1b5\uc77c\uc8fc\uccb4\uad6d\ubbfc\ud68c\uc758 \ub300\uc758\uc6d0\uc5d0 \uc758\ud574 \uc81c11\ub300 \ub300\ud1b5\ub839\uc73c\ub85c \uc120\ucd9c\ub41c \ud6c4 9\uc6d4 1\uc77c \uc7a0\uc2e4\uccb4\uc721\uad00\uc5d0\uc11c \uc81c11\ub300 \ub300\ud1b5\ub839\uc5d0 \ucde8\uc784\ud588\ub2e4.", "paragraph_answer": "1980\ub144 8\uc6d4 6\uc77c \uc721\uad70 \ub300\uc7a5(\u5927\u5c07)\uc73c\ub85c \uc9c4\uae09\ud55c \uc804\ub450\ud658\uc740 \uace7 \ub300\ud1b5\ub839 \uc120\uac70 \ucd9c\ub9c8\ub97c \uc120\uc5b8\ud588\ub2e4. \ubcf4\uc548\uc0ac\uc5d0 \uc758\ud574 \uac80\uc5f4\uacfc \uc870\uc885\uc744 \ubc1b\uc558\ub358 \ud55c\uad6d \uc5b8\ub860\uc740 \ubbf8\uad6d\uc758 \ub85c\uc2a4\uc564\uc824\ub808\uc2a4 \ud0c0\uc784\uc2a4\uac00 \uc804\ub450\ud658 \uc7a5\uad70\uc744 \uc9c0\uc9c0\ud55c\ub2e4\uace0 \ubcf4\ub3c4\ud588\ub2e4. \ud558\uc9c0\ub9cc \uc774\ub294 \ubbf8\uad6d \uc778\uc0ac\ub4e4\uc758 \ubc1c\uc5b8\uc758 \uc9c4\uc758\ub97c \ud6fc\uc190\ud55c \uc624\ubcf4\uc600\ub2e4. 8\uc6d4 16\uc77c \ucd5c\uaddc\ud558 \ub300\ud1b5\ub839\uc774 \uc0ac\uc784\ud588\uace0 8\uc6d4 22\uc77c \uc804\ub450\ud658\uc774 \uad70\ub300\uc5d0\uc11c \uc804\uc5ed\ud588\ub2e4. 8\uc6d4 23\uc77c \uc804\ub450\ud658\uc740 \ubcf4\uc548\uc0ac\ub839\uad00\uc5d0\uc11c \uc0ac\uc784\ud558\uace0 \ub2f9\uc2dc \uc218\ub3c4\uacbd\ube44\uc0ac\ub839\uad00\uc774\ub358 \ub178\ud0dc\uc6b0 \uac00 \ud6c4\uc784\uc73c\ub85c \uc784\uba85\ub418\uc5c8\ub2e4. 8\uc6d4 25\uc77c \ud544\ub9ac\ud540\uc758 \uc774\uba5c\ub2e4 \ub9c8\ub974\ucf54\uc2a4\ub294 \ud3c9\ud654\ub97c \uc0ac\ub791\ud558\ub294 \uc0ac\ub78c\uc740 \uc804\ub450\ud658\uc744 \uc9c0\uc9c0\ud574\uc57c \ud55c\ub2e4\uace0 \ud588\ub2e4. 8\uc6d4 27\uc77c \uc7a5\ucda9\uccb4\uc721\uad00\uc5d0\uc11c \ud1b5\uc77c\uc8fc\uccb4\uad6d\ubbfc\ud68c\uc758 \ub300\uc758\uc6d0\uc5d0 \uc758\ud574 \uc81c11\ub300 \ub300\ud1b5\ub839\uc73c\ub85c \uc120\ucd9c\ub41c \ud6c4 9\uc6d4 1\uc77c \uc7a0\uc2e4\uccb4\uc721\uad00\uc5d0\uc11c \uc81c11\ub300 \ub300\ud1b5\ub839\uc5d0 \ucde8\uc784\ud588\ub2e4.", "sentence_answer": "8\uc6d4 23\uc77c \uc804\ub450\ud658\uc740 \ubcf4\uc548\uc0ac\ub839\uad00\uc5d0\uc11c \uc0ac\uc784\ud558\uace0 \ub2f9\uc2dc \uc218\ub3c4\uacbd\ube44\uc0ac\ub839\uad00\uc774\ub358 \ub178\ud0dc\uc6b0 \uac00 \ud6c4\uc784\uc73c\ub85c \uc784\uba85\ub418\uc5c8\ub2e4.", "paragraph_id": "6593076-15-1", "paragraph_question": "question: \uc804\ub450\ud658\uc774 \ubcf4\uc548\uc0ac\ub839\uad00\uc5d0\uc11c \uc0ac\uc784\ud558\uace0 \uadf8\uc758 \ud6c4\uc784\uc73c\ub85c \uc784\uba85\ub41c \uc778\ubb3c\uc740?, context: 1980\ub144 8\uc6d4 6\uc77c \uc721\uad70 \ub300\uc7a5(\u5927\u5c07)\uc73c\ub85c \uc9c4\uae09\ud55c \uc804\ub450\ud658\uc740 \uace7 \ub300\ud1b5\ub839 \uc120\uac70 \ucd9c\ub9c8\ub97c \uc120\uc5b8\ud588\ub2e4. \ubcf4\uc548\uc0ac\uc5d0 \uc758\ud574 \uac80\uc5f4\uacfc \uc870\uc885\uc744 \ubc1b\uc558\ub358 \ud55c\uad6d \uc5b8\ub860\uc740 \ubbf8\uad6d\uc758 \ub85c\uc2a4\uc564\uc824\ub808\uc2a4 \ud0c0\uc784\uc2a4\uac00 \uc804\ub450\ud658 \uc7a5\uad70\uc744 \uc9c0\uc9c0\ud55c\ub2e4\uace0 \ubcf4\ub3c4\ud588\ub2e4. \ud558\uc9c0\ub9cc \uc774\ub294 \ubbf8\uad6d \uc778\uc0ac\ub4e4\uc758 \ubc1c\uc5b8\uc758 \uc9c4\uc758\ub97c \ud6fc\uc190\ud55c \uc624\ubcf4\uc600\ub2e4. 8\uc6d4 16\uc77c \ucd5c\uaddc\ud558 \ub300\ud1b5\ub839\uc774 \uc0ac\uc784\ud588\uace0 8\uc6d4 22\uc77c \uc804\ub450\ud658\uc774 \uad70\ub300\uc5d0\uc11c \uc804\uc5ed\ud588\ub2e4. 8\uc6d4 23\uc77c \uc804\ub450\ud658\uc740 \ubcf4\uc548\uc0ac\ub839\uad00\uc5d0\uc11c \uc0ac\uc784\ud558\uace0 \ub2f9\uc2dc \uc218\ub3c4\uacbd\ube44\uc0ac\ub839\uad00\uc774\ub358 \ub178\ud0dc\uc6b0\uac00 \ud6c4\uc784\uc73c\ub85c \uc784\uba85\ub418\uc5c8\ub2e4. 8\uc6d4 25\uc77c \ud544\ub9ac\ud540\uc758 \uc774\uba5c\ub2e4 \ub9c8\ub974\ucf54\uc2a4\ub294 \ud3c9\ud654\ub97c \uc0ac\ub791\ud558\ub294 \uc0ac\ub78c\uc740 \uc804\ub450\ud658\uc744 \uc9c0\uc9c0\ud574\uc57c \ud55c\ub2e4\uace0 \ud588\ub2e4. 8\uc6d4 27\uc77c \uc7a5\ucda9\uccb4\uc721\uad00\uc5d0\uc11c \ud1b5\uc77c\uc8fc\uccb4\uad6d\ubbfc\ud68c\uc758 \ub300\uc758\uc6d0\uc5d0 \uc758\ud574 \uc81c11\ub300 \ub300\ud1b5\ub839\uc73c\ub85c \uc120\ucd9c\ub41c \ud6c4 9\uc6d4 1\uc77c \uc7a0\uc2e4\uccb4\uc721\uad00\uc5d0\uc11c \uc81c11\ub300 \ub300\ud1b5\ub839\uc5d0 \ucde8\uc784\ud588\ub2e4."} {"question": "\ub85c\uc800 \ud398\ub354\ub7ec\uac00 2005\ub144\ubd80\ud130 2008\ub144\uae4c\uc9c0 4\ub144 \uc5f0\uc18d \ubc1b\uc740 \uc0c1\uc740?", "paragraph": "\ud398\ub354\ub7ec\ub294 \ub0a8\uc790 \ud14c\ub2c8\uc2a4\uc640 \uad00\ub828\ud574 \ub9ce\uc740 \uc138\uacc4 \uae30\ub85d\uc744 \ubcf4\uc720\ud558\uace0 \uc788\ub2e4. \uc5ed\ub300 \ub0a8\uc790 \uc120\uc218\ub4e4 \uc911 \uac00\uc7a5 \ub9ce\uc740 \ucd1d 20\uac1c\uc758 \uadf8\ub79c\ub4dc \uc2ac\ub7a8 \ub2e8\uc2dd \ud0c0\uc774\ud2c0\uc744 \ud68d\ub4dd\ud588\ub2e4. \ub610\ud55c \uadf8\ub294 2009\ub144 \ud504\ub791\uc2a4 \uc624\ud508\uc5d0\uc11c \uc6b0\uc2b9\ud558\uba74\uc11c \ucee4\ub9ac\uc5b4 \uadf8\ub79c\ub4dc \uc2ac\ub7a8\uc744 \ub2ec\uc131\ud55c \uc5ed\ub300 7\ubc88\uc9f8 \ub0a8\uc790 \uc120\uc218\uac00 \ub418\uc5c8\ub2e4. \uadf8\ub294 \uc5ed\ub300 \ub0a8\uc790 \uc120\uc218\ub4e4 \uc911 \uac00\uc7a5 \ub9ce\uc740 \ucd1d 30\ud68c\uc758 \uadf8\ub79c\ub4dc \uc2ac\ub7a8 \uacb0\uc2b9\uc5d0 \uc9c4\ucd9c\ud588\ub2e4. \ub610\ud55c 2004\ub144 \uc714\ube14\ub358 \uc6b0\uc2b9\uc744 \uc2dc\uc791\uc73c\ub85c 2010\ub144 1\uc6d4 \ud638\uc8fc \uc624\ud508\uc5d0\uc11c \uc6b0\uc2b9\ud558\uba74\uc11c \uc57d 6\ub144\uc5d0 \uac78\uccd0 \uadf8\ub79c\ub4dc \uc2ac\ub7a8 \uc900\uacb0\uc2b9 23\ud68c \uc5f0\uc18d \uc9c4\ucd9c\uc744 \uae30\ub85d, \uc774 \ubd80\ubb38 \ucd5c\uace0 \uae30\ub85d\uc744 \uc138\uc6e0\uc73c\uba70 \uc774\ub294 \uc885\uc804 \ucd5c\uace0 \uae30\ub85d\ubcf4\ub2e4 \ub450 \ubc30 \uc774\uc0c1 \ub9ce\uc740 \uac83\uc774\uc5c8\ub2e4. \uadf8\uac00 \uc120\uc218 \uc0dd\ud65c\ub3d9\uc548 \uc774\ub904\ub0b8 \ub6f0\uc5b4\ub09c \uc131\uc801\ub4e4\uacfc \uae30\ub85d\ub4e4\uc744 \uc778\uc815\ubc1b\uc544 \uadf8\ub294 2005\ub144\ubd80\ud130 2008\ub144\uae4c\uc9c0 4\ub144 \uc5f0\uc18d \ub85c\ub808\uc6b0\uc2a4 \uc62c\ud574\uc758 \uc138\uacc4 \uc2a4\ud3ec\uce20\uc778\uc0c1\uc744 \uc218\uc0c1\ud558\uc600\ub2e4.", "answer": "\ub85c\ub808\uc6b0\uc2a4 \uc62c\ud574\uc758 \uc138\uacc4 \uc2a4\ud3ec\uce20\uc778\uc0c1", "sentence": "\uadf8\ub294 2005\ub144\ubd80\ud130 2008\ub144\uae4c\uc9c0 4\ub144 \uc5f0\uc18d \ub85c\ub808\uc6b0\uc2a4 \uc62c\ud574\uc758 \uc138\uacc4 \uc2a4\ud3ec\uce20\uc778\uc0c1 \uc744 \uc218\uc0c1\ud558\uc600\ub2e4.", "paragraph_sentence": "\ud398\ub354\ub7ec\ub294 \ub0a8\uc790 \ud14c\ub2c8\uc2a4\uc640 \uad00\ub828\ud574 \ub9ce\uc740 \uc138\uacc4 \uae30\ub85d\uc744 \ubcf4\uc720\ud558\uace0 \uc788\ub2e4. \uc5ed\ub300 \ub0a8\uc790 \uc120\uc218\ub4e4 \uc911 \uac00\uc7a5 \ub9ce\uc740 \ucd1d 20\uac1c\uc758 \uadf8\ub79c\ub4dc \uc2ac\ub7a8 \ub2e8\uc2dd \ud0c0\uc774\ud2c0\uc744 \ud68d\ub4dd\ud588\ub2e4. \ub610\ud55c \uadf8\ub294 2009\ub144 \ud504\ub791\uc2a4 \uc624\ud508\uc5d0\uc11c \uc6b0\uc2b9\ud558\uba74\uc11c \ucee4\ub9ac\uc5b4 \uadf8\ub79c\ub4dc \uc2ac\ub7a8\uc744 \ub2ec\uc131\ud55c \uc5ed\ub300 7\ubc88\uc9f8 \ub0a8\uc790 \uc120\uc218\uac00 \ub418\uc5c8\ub2e4. \uadf8\ub294 \uc5ed\ub300 \ub0a8\uc790 \uc120\uc218\ub4e4 \uc911 \uac00\uc7a5 \ub9ce\uc740 \ucd1d 30\ud68c\uc758 \uadf8\ub79c\ub4dc \uc2ac\ub7a8 \uacb0\uc2b9\uc5d0 \uc9c4\ucd9c\ud588\ub2e4. \ub610\ud55c 2004\ub144 \uc714\ube14\ub358 \uc6b0\uc2b9\uc744 \uc2dc\uc791\uc73c\ub85c 2010\ub144 1\uc6d4 \ud638\uc8fc \uc624\ud508\uc5d0\uc11c \uc6b0\uc2b9\ud558\uba74\uc11c \uc57d 6\ub144\uc5d0 \uac78\uccd0 \uadf8\ub79c\ub4dc \uc2ac\ub7a8 \uc900\uacb0\uc2b9 23\ud68c \uc5f0\uc18d \uc9c4\ucd9c\uc744 \uae30\ub85d, \uc774 \ubd80\ubb38 \ucd5c\uace0 \uae30\ub85d\uc744 \uc138\uc6e0\uc73c\uba70 \uc774\ub294 \uc885\uc804 \ucd5c\uace0 \uae30\ub85d\ubcf4\ub2e4 \ub450 \ubc30 \uc774\uc0c1 \ub9ce\uc740 \uac83\uc774\uc5c8\ub2e4. \uadf8\uac00 \uc120\uc218 \uc0dd\ud65c\ub3d9\uc548 \uc774\ub904\ub0b8 \ub6f0\uc5b4\ub09c \uc131\uc801\ub4e4\uacfc \uae30\ub85d\ub4e4\uc744 \uc778\uc815\ubc1b\uc544 \uadf8\ub294 2005\ub144\ubd80\ud130 2008\ub144\uae4c\uc9c0 4\ub144 \uc5f0\uc18d \ub85c\ub808\uc6b0\uc2a4 \uc62c\ud574\uc758 \uc138\uacc4 \uc2a4\ud3ec\uce20\uc778\uc0c1 \uc744 \uc218\uc0c1\ud558\uc600\ub2e4. ", "paragraph_answer": "\ud398\ub354\ub7ec\ub294 \ub0a8\uc790 \ud14c\ub2c8\uc2a4\uc640 \uad00\ub828\ud574 \ub9ce\uc740 \uc138\uacc4 \uae30\ub85d\uc744 \ubcf4\uc720\ud558\uace0 \uc788\ub2e4. \uc5ed\ub300 \ub0a8\uc790 \uc120\uc218\ub4e4 \uc911 \uac00\uc7a5 \ub9ce\uc740 \ucd1d 20\uac1c\uc758 \uadf8\ub79c\ub4dc \uc2ac\ub7a8 \ub2e8\uc2dd \ud0c0\uc774\ud2c0\uc744 \ud68d\ub4dd\ud588\ub2e4. \ub610\ud55c \uadf8\ub294 2009\ub144 \ud504\ub791\uc2a4 \uc624\ud508\uc5d0\uc11c \uc6b0\uc2b9\ud558\uba74\uc11c \ucee4\ub9ac\uc5b4 \uadf8\ub79c\ub4dc \uc2ac\ub7a8\uc744 \ub2ec\uc131\ud55c \uc5ed\ub300 7\ubc88\uc9f8 \ub0a8\uc790 \uc120\uc218\uac00 \ub418\uc5c8\ub2e4. \uadf8\ub294 \uc5ed\ub300 \ub0a8\uc790 \uc120\uc218\ub4e4 \uc911 \uac00\uc7a5 \ub9ce\uc740 \ucd1d 30\ud68c\uc758 \uadf8\ub79c\ub4dc \uc2ac\ub7a8 \uacb0\uc2b9\uc5d0 \uc9c4\ucd9c\ud588\ub2e4. \ub610\ud55c 2004\ub144 \uc714\ube14\ub358 \uc6b0\uc2b9\uc744 \uc2dc\uc791\uc73c\ub85c 2010\ub144 1\uc6d4 \ud638\uc8fc \uc624\ud508\uc5d0\uc11c \uc6b0\uc2b9\ud558\uba74\uc11c \uc57d 6\ub144\uc5d0 \uac78\uccd0 \uadf8\ub79c\ub4dc \uc2ac\ub7a8 \uc900\uacb0\uc2b9 23\ud68c \uc5f0\uc18d \uc9c4\ucd9c\uc744 \uae30\ub85d, \uc774 \ubd80\ubb38 \ucd5c\uace0 \uae30\ub85d\uc744 \uc138\uc6e0\uc73c\uba70 \uc774\ub294 \uc885\uc804 \ucd5c\uace0 \uae30\ub85d\ubcf4\ub2e4 \ub450 \ubc30 \uc774\uc0c1 \ub9ce\uc740 \uac83\uc774\uc5c8\ub2e4. \uadf8\uac00 \uc120\uc218 \uc0dd\ud65c\ub3d9\uc548 \uc774\ub904\ub0b8 \ub6f0\uc5b4\ub09c \uc131\uc801\ub4e4\uacfc \uae30\ub85d\ub4e4\uc744 \uc778\uc815\ubc1b\uc544 \uadf8\ub294 2005\ub144\ubd80\ud130 2008\ub144\uae4c\uc9c0 4\ub144 \uc5f0\uc18d \ub85c\ub808\uc6b0\uc2a4 \uc62c\ud574\uc758 \uc138\uacc4 \uc2a4\ud3ec\uce20\uc778\uc0c1 \uc744 \uc218\uc0c1\ud558\uc600\ub2e4.", "sentence_answer": "\uadf8\ub294 2005\ub144\ubd80\ud130 2008\ub144\uae4c\uc9c0 4\ub144 \uc5f0\uc18d \ub85c\ub808\uc6b0\uc2a4 \uc62c\ud574\uc758 \uc138\uacc4 \uc2a4\ud3ec\uce20\uc778\uc0c1 \uc744 \uc218\uc0c1\ud558\uc600\ub2e4.", "paragraph_id": "6496547-0-2", "paragraph_question": "question: \ub85c\uc800 \ud398\ub354\ub7ec\uac00 2005\ub144\ubd80\ud130 2008\ub144\uae4c\uc9c0 4\ub144 \uc5f0\uc18d \ubc1b\uc740 \uc0c1\uc740?, context: \ud398\ub354\ub7ec\ub294 \ub0a8\uc790 \ud14c\ub2c8\uc2a4\uc640 \uad00\ub828\ud574 \ub9ce\uc740 \uc138\uacc4 \uae30\ub85d\uc744 \ubcf4\uc720\ud558\uace0 \uc788\ub2e4. \uc5ed\ub300 \ub0a8\uc790 \uc120\uc218\ub4e4 \uc911 \uac00\uc7a5 \ub9ce\uc740 \ucd1d 20\uac1c\uc758 \uadf8\ub79c\ub4dc \uc2ac\ub7a8 \ub2e8\uc2dd \ud0c0\uc774\ud2c0\uc744 \ud68d\ub4dd\ud588\ub2e4. \ub610\ud55c \uadf8\ub294 2009\ub144 \ud504\ub791\uc2a4 \uc624\ud508\uc5d0\uc11c \uc6b0\uc2b9\ud558\uba74\uc11c \ucee4\ub9ac\uc5b4 \uadf8\ub79c\ub4dc \uc2ac\ub7a8\uc744 \ub2ec\uc131\ud55c \uc5ed\ub300 7\ubc88\uc9f8 \ub0a8\uc790 \uc120\uc218\uac00 \ub418\uc5c8\ub2e4. \uadf8\ub294 \uc5ed\ub300 \ub0a8\uc790 \uc120\uc218\ub4e4 \uc911 \uac00\uc7a5 \ub9ce\uc740 \ucd1d 30\ud68c\uc758 \uadf8\ub79c\ub4dc \uc2ac\ub7a8 \uacb0\uc2b9\uc5d0 \uc9c4\ucd9c\ud588\ub2e4. \ub610\ud55c 2004\ub144 \uc714\ube14\ub358 \uc6b0\uc2b9\uc744 \uc2dc\uc791\uc73c\ub85c 2010\ub144 1\uc6d4 \ud638\uc8fc \uc624\ud508\uc5d0\uc11c \uc6b0\uc2b9\ud558\uba74\uc11c \uc57d 6\ub144\uc5d0 \uac78\uccd0 \uadf8\ub79c\ub4dc \uc2ac\ub7a8 \uc900\uacb0\uc2b9 23\ud68c \uc5f0\uc18d \uc9c4\ucd9c\uc744 \uae30\ub85d, \uc774 \ubd80\ubb38 \ucd5c\uace0 \uae30\ub85d\uc744 \uc138\uc6e0\uc73c\uba70 \uc774\ub294 \uc885\uc804 \ucd5c\uace0 \uae30\ub85d\ubcf4\ub2e4 \ub450 \ubc30 \uc774\uc0c1 \ub9ce\uc740 \uac83\uc774\uc5c8\ub2e4. \uadf8\uac00 \uc120\uc218 \uc0dd\ud65c\ub3d9\uc548 \uc774\ub904\ub0b8 \ub6f0\uc5b4\ub09c \uc131\uc801\ub4e4\uacfc \uae30\ub85d\ub4e4\uc744 \uc778\uc815\ubc1b\uc544 \uadf8\ub294 2005\ub144\ubd80\ud130 2008\ub144\uae4c\uc9c0 4\ub144 \uc5f0\uc18d \ub85c\ub808\uc6b0\uc2a4 \uc62c\ud574\uc758 \uc138\uacc4 \uc2a4\ud3ec\uce20\uc778\uc0c1\uc744 \uc218\uc0c1\ud558\uc600\ub2e4."} {"question": "\uc560\ud314\ub798\uce58\uc544 \uc0b0\ub9e5 \uc77c\ubd80\uc640 \uc624\ub300\ud638 \uc9c0\uc5ed \ubd81\uc11c\ubd80 \uc9c0\uc5ed\uc744 \uc81c\uc678\ud558\uace0 \ubbf8\uad6d \ub3d9\ubd80\uc5d0\uc11c \ub291\ub300\uac00 \uba78\uc885\ud55c \uac83\uc740 \uc5b8\uc81c\uc778\uac00?", "paragraph": "\ubd81\ubbf8\uc758 \ub291\ub300\ub294 \uc778\uac04 \uc778\uad6c\uc758 \uc99d\uac00\uc640 \ub18d\uc5c5\uc758 \ud655\ub300\uc640 \ub3d9\uc2dc\uc5d0 \uac1c\uccb4\uc218\uac00 \uac10\uc18c\ud558\uae30 \uc2dc\uc791\ud588\ub2e4. 20\uc138\uae30 \ucd08\uc5d0 \ub291\ub300\ub294 \uc560\ud314\ub798\uce58\uc544 \uc0b0\ub9e5 \uc77c\ubd80\uc640 \uc624\ub300\ud638 \uc9c0\uc5ed \ubd81\uc11c\ubd80 \uc9c0\uc5ed\uc744 \uc81c\uc678\ud558\uace0 \ubbf8\uad6d \ub3d9\ubd80\uc5d0\uc11c \uba78\uc885\ud588\ub2e4. \uce90\ub098\ub2e4\uc5d0\uc11c\ub294 1870\ub144\uc5d0\uc11c 1921\ub144 \uc0ac\uc774 \ub274\ube0c\ub7f0\uc988\uc705 \uc8fc\uc640 \ub178\ubc14\uc2a4\ucf54\uc0e4 \uc8fc\uc5d0 \ub291\ub300\uac00 \uba78\uc885\ud588\uc73c\uba70 1911\ub144\uc5d0\ub294 \ub274\ud380\ub4e4\ub79c\ub4dc \uc8fc\uc5d0\uc11c \uba78\uc885\ud588\ub2e4. \ubd81\ubd80 \ub85c\ud0a4\uc0b0\ub9e5\uc5d0\uc11c \ub291\ub300\ub294 1920\ub144\ub300\uc5d0 \uc0ac\ub77c\uc84c\ub2e4. 1850\ub144\uc5d0\uc11c 1900\ub144 \uc0ac\uc774 \ud018\ubca1 \uc8fc\uc640 \uc628\ud0c0\ub9ac\uc624 \uc8fc \ub0a8\ubd80\uc5d0 \ub291\ub300\uac00 \uba78\uc885\ud588\ub2e4. \ub300\ud3c9\uc6d0\uc5d0\uc11c \ub291\ub300 \uac1c\uccb4\uc218 \uac10\uc18c\ub294 1860\ub144-70\ub144 \uc0ac\uc774 \uc544\uba54\ub9ac\uce74\ub4e4\uc18c \ub4f1 \ub2e4\ub978 \uba39\uc774\uc758 \uac1c\uccb4\uc218 \uac10\uc18c\uc640 \uc2dc\uc791\ub418\uc5c8\ub2e4. 1900\ub144\uc5d0\uc11c 1930\ub144 \uc0ac\uc774 \uc885\uc744 \uba78\uc885\ud558\uae30 \uc704\ud55c \uc9d1\uc911\uc801 \uc721\uc2dd \ub3d9\ubb3c \uad00\ub9ac \ud504\ub85c\uadf8\ub7a8\uc73c\ub85c \uc778\ud574 \uc0ac\uc2e4\uc0c1 \ubbf8\uad6d \uc11c\ubd80\uc640 \uce90\ub098\ub2e4 \uad6d\uacbd \uc778\uadfc \ubd80\ubd84\uc5d0\uc11c \uba78\uc885\ud588\ub2e4. \ud68c\uc0c9\ub291\ub300\ub294 \uc54c\ub798\uc2a4\uce74\uc640 \ubbf8\ub124\uc18c\ud0c0 \uc8fc \ubd81\ubd80\ub97c \uc81c\uc678\ud55c \ubaa8\ub4e0 \uc5f0\ubc29 \ubc0f \uc8fc\uc5d0\uc11c \uba78\uc885\ub418\uc5c8\ub2e4. \ubd81\ubbf8 \ub291\ub300\uc758 \uac1c\uccb4\uc218 \uac10\uc18c\ud604\uc0c1\uc740 1930\ub144\uc5d0\uc11c 1950\ub144 \uc0ac\uc774 \ub0a8\uc11c\ubd80 \uce90\ub098\ub2e4\uc5d0\uc11c \ub300\ud615 \uc0ac\ub0e5 \uac8c\uc784\uc758 \uaddc\uc81c\uc640 \ub300\ud615 \uc720\uc81c\ub958\uc758 \ubcf4\ud638\ub85c \uc778\ud574 \ub2e4\uc2dc \uc99d\uac00\ud588\ub2e4. \uc774 \uc99d\uac00\ub294 \uc11c\ubd80\uc640 \ubd81\ubd80 \uce90\ub098\ub2e4\uc758 \ub291\ub300 \uac1c\uccb4\uc218 \uc99d\uac00\uc5d0 \ub3c4\uc6c0\uc744 \uc8fc\uc5c8\ub2e4. 1950\ub144\ub300 \ucd08\ubd80\ud130 1960\ub144\ub300 \ucd08\uae4c\uc9c0 \ub291\ub300 \uc218\ucc9c \ub9c8\ub9ac\uac00 \uc8fc\ub85c \ub3c5\uc5d0 \uc758\ud574 \uc8fd\uc5c8\ub2e4. \uc774 \ucea0\ud328\uc778\uc740 \uc911\ub2e8\ub418\uace0 1970\ub144\ub300 \uc911\ubc18 \ub291\ub300 \uac1c\uccb4\uc218\ub294 \uc99d\uac00\uac00 \uc2dc\uc791\ub418\uc5c8\ub2e4.", "answer": "20\uc138\uae30 \ucd08", "sentence": "20\uc138\uae30 \ucd08 \uc5d0 \ub291\ub300\ub294 \uc560\ud314\ub798\uce58\uc544 \uc0b0\ub9e5 \uc77c\ubd80\uc640 \uc624\ub300\ud638 \uc9c0\uc5ed \ubd81\uc11c\ubd80 \uc9c0\uc5ed\uc744 \uc81c\uc678\ud558\uace0 \ubbf8\uad6d \ub3d9\ubd80\uc5d0\uc11c \uba78\uc885\ud588\ub2e4.", "paragraph_sentence": "\ubd81\ubbf8\uc758 \ub291\ub300\ub294 \uc778\uac04 \uc778\uad6c\uc758 \uc99d\uac00\uc640 \ub18d\uc5c5\uc758 \ud655\ub300\uc640 \ub3d9\uc2dc\uc5d0 \uac1c\uccb4\uc218\uac00 \uac10\uc18c\ud558\uae30 \uc2dc\uc791\ud588\ub2e4. 20\uc138\uae30 \ucd08 \uc5d0 \ub291\ub300\ub294 \uc560\ud314\ub798\uce58\uc544 \uc0b0\ub9e5 \uc77c\ubd80\uc640 \uc624\ub300\ud638 \uc9c0\uc5ed \ubd81\uc11c\ubd80 \uc9c0\uc5ed\uc744 \uc81c\uc678\ud558\uace0 \ubbf8\uad6d \ub3d9\ubd80\uc5d0\uc11c \uba78\uc885\ud588\ub2e4. \uce90\ub098\ub2e4\uc5d0\uc11c\ub294 1870\ub144\uc5d0\uc11c 1921\ub144 \uc0ac\uc774 \ub274\ube0c\ub7f0\uc988\uc705 \uc8fc\uc640 \ub178\ubc14\uc2a4\ucf54\uc0e4 \uc8fc\uc5d0 \ub291\ub300\uac00 \uba78\uc885\ud588\uc73c\uba70 1911\ub144\uc5d0\ub294 \ub274\ud380\ub4e4\ub79c\ub4dc \uc8fc\uc5d0\uc11c \uba78\uc885\ud588\ub2e4. \ubd81\ubd80 \ub85c\ud0a4\uc0b0\ub9e5\uc5d0\uc11c \ub291\ub300\ub294 1920\ub144\ub300\uc5d0 \uc0ac\ub77c\uc84c\ub2e4. 1850\ub144\uc5d0\uc11c 1900\ub144 \uc0ac\uc774 \ud018\ubca1 \uc8fc\uc640 \uc628\ud0c0\ub9ac\uc624 \uc8fc \ub0a8\ubd80\uc5d0 \ub291\ub300\uac00 \uba78\uc885\ud588\ub2e4. \ub300\ud3c9\uc6d0\uc5d0\uc11c \ub291\ub300 \uac1c\uccb4\uc218 \uac10\uc18c\ub294 1860\ub144-70\ub144 \uc0ac\uc774 \uc544\uba54\ub9ac\uce74\ub4e4\uc18c \ub4f1 \ub2e4\ub978 \uba39\uc774\uc758 \uac1c\uccb4\uc218 \uac10\uc18c\uc640 \uc2dc\uc791\ub418\uc5c8\ub2e4. 1900\ub144\uc5d0\uc11c 1930\ub144 \uc0ac\uc774 \uc885\uc744 \uba78\uc885\ud558\uae30 \uc704\ud55c \uc9d1\uc911\uc801 \uc721\uc2dd \ub3d9\ubb3c \uad00\ub9ac \ud504\ub85c\uadf8\ub7a8\uc73c\ub85c \uc778\ud574 \uc0ac\uc2e4\uc0c1 \ubbf8\uad6d \uc11c\ubd80\uc640 \uce90\ub098\ub2e4 \uad6d\uacbd \uc778\uadfc \ubd80\ubd84\uc5d0\uc11c \uba78\uc885\ud588\ub2e4. \ud68c\uc0c9\ub291\ub300\ub294 \uc54c\ub798\uc2a4\uce74\uc640 \ubbf8\ub124\uc18c\ud0c0 \uc8fc \ubd81\ubd80\ub97c \uc81c\uc678\ud55c \ubaa8\ub4e0 \uc5f0\ubc29 \ubc0f \uc8fc\uc5d0\uc11c \uba78\uc885\ub418\uc5c8\ub2e4. \ubd81\ubbf8 \ub291\ub300\uc758 \uac1c\uccb4\uc218 \uac10\uc18c\ud604\uc0c1\uc740 1930\ub144\uc5d0\uc11c 1950\ub144 \uc0ac\uc774 \ub0a8\uc11c\ubd80 \uce90\ub098\ub2e4\uc5d0\uc11c \ub300\ud615 \uc0ac\ub0e5 \uac8c\uc784\uc758 \uaddc\uc81c\uc640 \ub300\ud615 \uc720\uc81c\ub958\uc758 \ubcf4\ud638\ub85c \uc778\ud574 \ub2e4\uc2dc \uc99d\uac00\ud588\ub2e4. \uc774 \uc99d\uac00\ub294 \uc11c\ubd80\uc640 \ubd81\ubd80 \uce90\ub098\ub2e4\uc758 \ub291\ub300 \uac1c\uccb4\uc218 \uc99d\uac00\uc5d0 \ub3c4\uc6c0\uc744 \uc8fc\uc5c8\ub2e4. 1950\ub144\ub300 \ucd08\ubd80\ud130 1960\ub144\ub300 \ucd08\uae4c\uc9c0 \ub291\ub300 \uc218\ucc9c \ub9c8\ub9ac\uac00 \uc8fc\ub85c \ub3c5\uc5d0 \uc758\ud574 \uc8fd\uc5c8\ub2e4. \uc774 \ucea0\ud328\uc778\uc740 \uc911\ub2e8\ub418\uace0 1970\ub144\ub300 \uc911\ubc18 \ub291\ub300 \uac1c\uccb4\uc218\ub294 \uc99d\uac00\uac00 \uc2dc\uc791\ub418\uc5c8\ub2e4.", "paragraph_answer": "\ubd81\ubbf8\uc758 \ub291\ub300\ub294 \uc778\uac04 \uc778\uad6c\uc758 \uc99d\uac00\uc640 \ub18d\uc5c5\uc758 \ud655\ub300\uc640 \ub3d9\uc2dc\uc5d0 \uac1c\uccb4\uc218\uac00 \uac10\uc18c\ud558\uae30 \uc2dc\uc791\ud588\ub2e4. 20\uc138\uae30 \ucd08 \uc5d0 \ub291\ub300\ub294 \uc560\ud314\ub798\uce58\uc544 \uc0b0\ub9e5 \uc77c\ubd80\uc640 \uc624\ub300\ud638 \uc9c0\uc5ed \ubd81\uc11c\ubd80 \uc9c0\uc5ed\uc744 \uc81c\uc678\ud558\uace0 \ubbf8\uad6d \ub3d9\ubd80\uc5d0\uc11c \uba78\uc885\ud588\ub2e4. \uce90\ub098\ub2e4\uc5d0\uc11c\ub294 1870\ub144\uc5d0\uc11c 1921\ub144 \uc0ac\uc774 \ub274\ube0c\ub7f0\uc988\uc705 \uc8fc\uc640 \ub178\ubc14\uc2a4\ucf54\uc0e4 \uc8fc\uc5d0 \ub291\ub300\uac00 \uba78\uc885\ud588\uc73c\uba70 1911\ub144\uc5d0\ub294 \ub274\ud380\ub4e4\ub79c\ub4dc \uc8fc\uc5d0\uc11c \uba78\uc885\ud588\ub2e4. \ubd81\ubd80 \ub85c\ud0a4\uc0b0\ub9e5\uc5d0\uc11c \ub291\ub300\ub294 1920\ub144\ub300\uc5d0 \uc0ac\ub77c\uc84c\ub2e4. 1850\ub144\uc5d0\uc11c 1900\ub144 \uc0ac\uc774 \ud018\ubca1 \uc8fc\uc640 \uc628\ud0c0\ub9ac\uc624 \uc8fc \ub0a8\ubd80\uc5d0 \ub291\ub300\uac00 \uba78\uc885\ud588\ub2e4. \ub300\ud3c9\uc6d0\uc5d0\uc11c \ub291\ub300 \uac1c\uccb4\uc218 \uac10\uc18c\ub294 1860\ub144-70\ub144 \uc0ac\uc774 \uc544\uba54\ub9ac\uce74\ub4e4\uc18c \ub4f1 \ub2e4\ub978 \uba39\uc774\uc758 \uac1c\uccb4\uc218 \uac10\uc18c\uc640 \uc2dc\uc791\ub418\uc5c8\ub2e4. 1900\ub144\uc5d0\uc11c 1930\ub144 \uc0ac\uc774 \uc885\uc744 \uba78\uc885\ud558\uae30 \uc704\ud55c \uc9d1\uc911\uc801 \uc721\uc2dd \ub3d9\ubb3c \uad00\ub9ac \ud504\ub85c\uadf8\ub7a8\uc73c\ub85c \uc778\ud574 \uc0ac\uc2e4\uc0c1 \ubbf8\uad6d \uc11c\ubd80\uc640 \uce90\ub098\ub2e4 \uad6d\uacbd \uc778\uadfc \ubd80\ubd84\uc5d0\uc11c \uba78\uc885\ud588\ub2e4. \ud68c\uc0c9\ub291\ub300\ub294 \uc54c\ub798\uc2a4\uce74\uc640 \ubbf8\ub124\uc18c\ud0c0 \uc8fc \ubd81\ubd80\ub97c \uc81c\uc678\ud55c \ubaa8\ub4e0 \uc5f0\ubc29 \ubc0f \uc8fc\uc5d0\uc11c \uba78\uc885\ub418\uc5c8\ub2e4. \ubd81\ubbf8 \ub291\ub300\uc758 \uac1c\uccb4\uc218 \uac10\uc18c\ud604\uc0c1\uc740 1930\ub144\uc5d0\uc11c 1950\ub144 \uc0ac\uc774 \ub0a8\uc11c\ubd80 \uce90\ub098\ub2e4\uc5d0\uc11c \ub300\ud615 \uc0ac\ub0e5 \uac8c\uc784\uc758 \uaddc\uc81c\uc640 \ub300\ud615 \uc720\uc81c\ub958\uc758 \ubcf4\ud638\ub85c \uc778\ud574 \ub2e4\uc2dc \uc99d\uac00\ud588\ub2e4. \uc774 \uc99d\uac00\ub294 \uc11c\ubd80\uc640 \ubd81\ubd80 \uce90\ub098\ub2e4\uc758 \ub291\ub300 \uac1c\uccb4\uc218 \uc99d\uac00\uc5d0 \ub3c4\uc6c0\uc744 \uc8fc\uc5c8\ub2e4. 1950\ub144\ub300 \ucd08\ubd80\ud130 1960\ub144\ub300 \ucd08\uae4c\uc9c0 \ub291\ub300 \uc218\ucc9c \ub9c8\ub9ac\uac00 \uc8fc\ub85c \ub3c5\uc5d0 \uc758\ud574 \uc8fd\uc5c8\ub2e4. \uc774 \ucea0\ud328\uc778\uc740 \uc911\ub2e8\ub418\uace0 1970\ub144\ub300 \uc911\ubc18 \ub291\ub300 \uac1c\uccb4\uc218\ub294 \uc99d\uac00\uac00 \uc2dc\uc791\ub418\uc5c8\ub2e4.", "sentence_answer": " 20\uc138\uae30 \ucd08 \uc5d0 \ub291\ub300\ub294 \uc560\ud314\ub798\uce58\uc544 \uc0b0\ub9e5 \uc77c\ubd80\uc640 \uc624\ub300\ud638 \uc9c0\uc5ed \ubd81\uc11c\ubd80 \uc9c0\uc5ed\uc744 \uc81c\uc678\ud558\uace0 \ubbf8\uad6d \ub3d9\ubd80\uc5d0\uc11c \uba78\uc885\ud588\ub2e4.", "paragraph_id": "6572454-38-1", "paragraph_question": "question: \uc560\ud314\ub798\uce58\uc544 \uc0b0\ub9e5 \uc77c\ubd80\uc640 \uc624\ub300\ud638 \uc9c0\uc5ed \ubd81\uc11c\ubd80 \uc9c0\uc5ed\uc744 \uc81c\uc678\ud558\uace0 \ubbf8\uad6d \ub3d9\ubd80\uc5d0\uc11c \ub291\ub300\uac00 \uba78\uc885\ud55c \uac83\uc740 \uc5b8\uc81c\uc778\uac00?, context: \ubd81\ubbf8\uc758 \ub291\ub300\ub294 \uc778\uac04 \uc778\uad6c\uc758 \uc99d\uac00\uc640 \ub18d\uc5c5\uc758 \ud655\ub300\uc640 \ub3d9\uc2dc\uc5d0 \uac1c\uccb4\uc218\uac00 \uac10\uc18c\ud558\uae30 \uc2dc\uc791\ud588\ub2e4. 20\uc138\uae30 \ucd08\uc5d0 \ub291\ub300\ub294 \uc560\ud314\ub798\uce58\uc544 \uc0b0\ub9e5 \uc77c\ubd80\uc640 \uc624\ub300\ud638 \uc9c0\uc5ed \ubd81\uc11c\ubd80 \uc9c0\uc5ed\uc744 \uc81c\uc678\ud558\uace0 \ubbf8\uad6d \ub3d9\ubd80\uc5d0\uc11c \uba78\uc885\ud588\ub2e4. \uce90\ub098\ub2e4\uc5d0\uc11c\ub294 1870\ub144\uc5d0\uc11c 1921\ub144 \uc0ac\uc774 \ub274\ube0c\ub7f0\uc988\uc705 \uc8fc\uc640 \ub178\ubc14\uc2a4\ucf54\uc0e4 \uc8fc\uc5d0 \ub291\ub300\uac00 \uba78\uc885\ud588\uc73c\uba70 1911\ub144\uc5d0\ub294 \ub274\ud380\ub4e4\ub79c\ub4dc \uc8fc\uc5d0\uc11c \uba78\uc885\ud588\ub2e4. \ubd81\ubd80 \ub85c\ud0a4\uc0b0\ub9e5\uc5d0\uc11c \ub291\ub300\ub294 1920\ub144\ub300\uc5d0 \uc0ac\ub77c\uc84c\ub2e4. 1850\ub144\uc5d0\uc11c 1900\ub144 \uc0ac\uc774 \ud018\ubca1 \uc8fc\uc640 \uc628\ud0c0\ub9ac\uc624 \uc8fc \ub0a8\ubd80\uc5d0 \ub291\ub300\uac00 \uba78\uc885\ud588\ub2e4. \ub300\ud3c9\uc6d0\uc5d0\uc11c \ub291\ub300 \uac1c\uccb4\uc218 \uac10\uc18c\ub294 1860\ub144-70\ub144 \uc0ac\uc774 \uc544\uba54\ub9ac\uce74\ub4e4\uc18c \ub4f1 \ub2e4\ub978 \uba39\uc774\uc758 \uac1c\uccb4\uc218 \uac10\uc18c\uc640 \uc2dc\uc791\ub418\uc5c8\ub2e4. 1900\ub144\uc5d0\uc11c 1930\ub144 \uc0ac\uc774 \uc885\uc744 \uba78\uc885\ud558\uae30 \uc704\ud55c \uc9d1\uc911\uc801 \uc721\uc2dd \ub3d9\ubb3c \uad00\ub9ac \ud504\ub85c\uadf8\ub7a8\uc73c\ub85c \uc778\ud574 \uc0ac\uc2e4\uc0c1 \ubbf8\uad6d \uc11c\ubd80\uc640 \uce90\ub098\ub2e4 \uad6d\uacbd \uc778\uadfc \ubd80\ubd84\uc5d0\uc11c \uba78\uc885\ud588\ub2e4. \ud68c\uc0c9\ub291\ub300\ub294 \uc54c\ub798\uc2a4\uce74\uc640 \ubbf8\ub124\uc18c\ud0c0 \uc8fc \ubd81\ubd80\ub97c \uc81c\uc678\ud55c \ubaa8\ub4e0 \uc5f0\ubc29 \ubc0f \uc8fc\uc5d0\uc11c \uba78\uc885\ub418\uc5c8\ub2e4. \ubd81\ubbf8 \ub291\ub300\uc758 \uac1c\uccb4\uc218 \uac10\uc18c\ud604\uc0c1\uc740 1930\ub144\uc5d0\uc11c 1950\ub144 \uc0ac\uc774 \ub0a8\uc11c\ubd80 \uce90\ub098\ub2e4\uc5d0\uc11c \ub300\ud615 \uc0ac\ub0e5 \uac8c\uc784\uc758 \uaddc\uc81c\uc640 \ub300\ud615 \uc720\uc81c\ub958\uc758 \ubcf4\ud638\ub85c \uc778\ud574 \ub2e4\uc2dc \uc99d\uac00\ud588\ub2e4. \uc774 \uc99d\uac00\ub294 \uc11c\ubd80\uc640 \ubd81\ubd80 \uce90\ub098\ub2e4\uc758 \ub291\ub300 \uac1c\uccb4\uc218 \uc99d\uac00\uc5d0 \ub3c4\uc6c0\uc744 \uc8fc\uc5c8\ub2e4. 1950\ub144\ub300 \ucd08\ubd80\ud130 1960\ub144\ub300 \ucd08\uae4c\uc9c0 \ub291\ub300 \uc218\ucc9c \ub9c8\ub9ac\uac00 \uc8fc\ub85c \ub3c5\uc5d0 \uc758\ud574 \uc8fd\uc5c8\ub2e4. \uc774 \ucea0\ud328\uc778\uc740 \uc911\ub2e8\ub418\uace0 1970\ub144\ub300 \uc911\ubc18 \ub291\ub300 \uac1c\uccb4\uc218\ub294 \uc99d\uac00\uac00 \uc2dc\uc791\ub418\uc5c8\ub2e4."} {"question": "\ubbf8\uad6d\uc774 \ud734\uc804\ud611\uc815\uc744 \uc81c\uc548\ud55c \uac83\uc740 \uc5b4\ub290 \ub098\ub77c\ub97c \ubd80\ub2f4\uc2a4\ub7ec\uc6cc\ud588\uae30 \ub54c\ubb38\uc778\uac00?", "paragraph": "\ud55c\uad6d \uc804\uc7c1 \uc911 2\ub144 \ub3d9\uc548 \uc774\ub807\ub2e4 \ud55c \uc131\uacfc\ub97c \uc62c\ub9ac\uc9c0 \ubabb\ud558\uace0 \uc911\ud654\uc778\ubbfc\uacf5\ud654\uad6d\uc758 \uac1c\uc785\uc73c\ub85c \uc804\uc120\uc774 \uace0\ucc29\ud654\ud558\uc790, \uc911\ud654\uc778\ubbfc\uacf5\ud654\uad6d\uc744 \ubd80\ub2f4\uc2a4\ub7ec\uc6cc\ud55c \ubbf8\uad6d\uc740 \ud604\uc0c1\uc720\uc9c0 \ucc28\uc6d0\uc5d0\uc11c \ud734\uc804\ud611\uc815\uc744 \ucd94\uc9c4\ud558\uc600\ub2e4. \uc18c\ub828\uc774 \ubd80\ubd84\uc801\uc73c\ub85c \ucc38\uc804\ud558\uc790, \uc138\uacc4 \uc5ec\ub7ec\ub098\ub77c\ub4e4\uc740 \ud55c\uad6d \uc804\uc7c1\uc774 \uba38\uc9c0\uc54a\uc544 \ub0c9\uc804\uccb4\uc81c\uc640 \ud55c\uad6d\uc758 \ud1b5\uc77c\uc744 \uc704\ud55c \uc81c3\ucc28 \uc138\uacc4\ub300\uc804\uc774 \uc77c\uc5b4\ub0a0 \uac70\ub77c\ub294 \uac00\ub2a5\uc131\uc774 \ucee4\uc9c8 \uac70\ub77c\uace0 \uc0dd\uac01\ud588\ub2e4. \uadf8\ub798\uc11c \uc720\uc5d4\uad70\uacfc \uc911\uad6d \uc778\ubbfc\ud574\ubc29\uad70\uc740 \uc5bc\ub978 \uc774 \ud55c\uad6d\uc804\uc7c1\uc744 \uc911\ub2e8\ud558\uace0, \ud734\uc804\uc744 \ud558\uae30\ub85c \ud588\uc5c8\ub2e4. \ubbf8\uad6d\uc740 \uc18c\ube44\uc5d0\ud2b8 \uc5f0\ubc29 \ubc0f \uc911\ud654\uc778\ubbfc\uacf5\ud654\uad6d \uce21\uacfc \ud734\uc804\uc744 \ub17c\uc758\ud55c\ub2e4. 1952\ub144 3\uc6d4 \uc774\uc2b9\ub9cc\uc740 \ubd84\ub2e8 \uc0c1\ud0dc\uc5d0\uc11c\uc758 \ud734\uc804\uc740 \ud55c\uad6d\uc5d0 \ub300\ud55c \u201c\uc0ac\ud615\uc120\uace0\u201d\ub098 \ub2e4\ub984\uc5c6\ub2e4\uace0 \uc8fc\uc7a5\ud558\uba70, \u201c\ubbfc\uc871\uad6d\uac00\ub85c \uc0dd\uc874\ud558\uae30 \uc704\ud574 \ub2e8\ub3c5\uc73c\ub85c\ub77c\ub3c4 \uacc4\uc18d\ud574 \uc2f8\uc6cc \ub098\uac08 \uac83\u201d\uc774\ub77c\uace0 \uac15\uc870\ud588\ub2e4.", "answer": "\uc911\ud654\uc778\ubbfc\uacf5\ud654\uad6d", "sentence": "\ud55c\uad6d \uc804\uc7c1 \uc911 2\ub144 \ub3d9\uc548 \uc774\ub807\ub2e4 \ud55c \uc131\uacfc\ub97c \uc62c\ub9ac\uc9c0 \ubabb\ud558\uace0 \uc911\ud654\uc778\ubbfc\uacf5\ud654\uad6d \uc758 \uac1c\uc785\uc73c\ub85c \uc804\uc120\uc774 \uace0\ucc29\ud654\ud558\uc790, \uc911\ud654\uc778\ubbfc\uacf5\ud654\uad6d\uc744 \ubd80\ub2f4\uc2a4\ub7ec\uc6cc\ud55c \ubbf8\uad6d\uc740 \ud604\uc0c1\uc720\uc9c0 \ucc28\uc6d0\uc5d0\uc11c \ud734\uc804\ud611\uc815\uc744 \ucd94\uc9c4\ud558\uc600\ub2e4.", "paragraph_sentence": " \ud55c\uad6d \uc804\uc7c1 \uc911 2\ub144 \ub3d9\uc548 \uc774\ub807\ub2e4 \ud55c \uc131\uacfc\ub97c \uc62c\ub9ac\uc9c0 \ubabb\ud558\uace0 \uc911\ud654\uc778\ubbfc\uacf5\ud654\uad6d \uc758 \uac1c\uc785\uc73c\ub85c \uc804\uc120\uc774 \uace0\ucc29\ud654\ud558\uc790, \uc911\ud654\uc778\ubbfc\uacf5\ud654\uad6d\uc744 \ubd80\ub2f4\uc2a4\ub7ec\uc6cc\ud55c \ubbf8\uad6d\uc740 \ud604\uc0c1\uc720\uc9c0 \ucc28\uc6d0\uc5d0\uc11c \ud734\uc804\ud611\uc815\uc744 \ucd94\uc9c4\ud558\uc600\ub2e4. \uc18c\ub828\uc774 \ubd80\ubd84\uc801\uc73c\ub85c \ucc38\uc804\ud558\uc790, \uc138\uacc4 \uc5ec\ub7ec\ub098\ub77c\ub4e4\uc740 \ud55c\uad6d \uc804\uc7c1\uc774 \uba38\uc9c0\uc54a\uc544 \ub0c9\uc804\uccb4\uc81c\uc640 \ud55c\uad6d\uc758 \ud1b5\uc77c\uc744 \uc704\ud55c \uc81c3\ucc28 \uc138\uacc4\ub300\uc804\uc774 \uc77c\uc5b4\ub0a0 \uac70\ub77c\ub294 \uac00\ub2a5\uc131\uc774 \ucee4\uc9c8 \uac70\ub77c\uace0 \uc0dd\uac01\ud588\ub2e4. \uadf8\ub798\uc11c \uc720\uc5d4\uad70\uacfc \uc911\uad6d \uc778\ubbfc\ud574\ubc29\uad70\uc740 \uc5bc\ub978 \uc774 \ud55c\uad6d\uc804\uc7c1\uc744 \uc911\ub2e8\ud558\uace0, \ud734\uc804\uc744 \ud558\uae30\ub85c \ud588\uc5c8\ub2e4. \ubbf8\uad6d\uc740 \uc18c\ube44\uc5d0\ud2b8 \uc5f0\ubc29 \ubc0f \uc911\ud654\uc778\ubbfc\uacf5\ud654\uad6d \uce21\uacfc \ud734\uc804\uc744 \ub17c\uc758\ud55c\ub2e4. 1952\ub144 3\uc6d4 \uc774\uc2b9\ub9cc\uc740 \ubd84\ub2e8 \uc0c1\ud0dc\uc5d0\uc11c\uc758 \ud734\uc804\uc740 \ud55c\uad6d\uc5d0 \ub300\ud55c \u201c\uc0ac\ud615\uc120\uace0\u201d\ub098 \ub2e4\ub984\uc5c6\ub2e4\uace0 \uc8fc\uc7a5\ud558\uba70, \u201c\ubbfc\uc871\uad6d\uac00\ub85c \uc0dd\uc874\ud558\uae30 \uc704\ud574 \ub2e8\ub3c5\uc73c\ub85c\ub77c\ub3c4 \uacc4\uc18d\ud574 \uc2f8\uc6cc \ub098\uac08 \uac83\u201d\uc774\ub77c\uace0 \uac15\uc870\ud588\ub2e4.", "paragraph_answer": "\ud55c\uad6d \uc804\uc7c1 \uc911 2\ub144 \ub3d9\uc548 \uc774\ub807\ub2e4 \ud55c \uc131\uacfc\ub97c \uc62c\ub9ac\uc9c0 \ubabb\ud558\uace0 \uc911\ud654\uc778\ubbfc\uacf5\ud654\uad6d \uc758 \uac1c\uc785\uc73c\ub85c \uc804\uc120\uc774 \uace0\ucc29\ud654\ud558\uc790, \uc911\ud654\uc778\ubbfc\uacf5\ud654\uad6d\uc744 \ubd80\ub2f4\uc2a4\ub7ec\uc6cc\ud55c \ubbf8\uad6d\uc740 \ud604\uc0c1\uc720\uc9c0 \ucc28\uc6d0\uc5d0\uc11c \ud734\uc804\ud611\uc815\uc744 \ucd94\uc9c4\ud558\uc600\ub2e4. \uc18c\ub828\uc774 \ubd80\ubd84\uc801\uc73c\ub85c \ucc38\uc804\ud558\uc790, \uc138\uacc4 \uc5ec\ub7ec\ub098\ub77c\ub4e4\uc740 \ud55c\uad6d \uc804\uc7c1\uc774 \uba38\uc9c0\uc54a\uc544 \ub0c9\uc804\uccb4\uc81c\uc640 \ud55c\uad6d\uc758 \ud1b5\uc77c\uc744 \uc704\ud55c \uc81c3\ucc28 \uc138\uacc4\ub300\uc804\uc774 \uc77c\uc5b4\ub0a0 \uac70\ub77c\ub294 \uac00\ub2a5\uc131\uc774 \ucee4\uc9c8 \uac70\ub77c\uace0 \uc0dd\uac01\ud588\ub2e4. \uadf8\ub798\uc11c \uc720\uc5d4\uad70\uacfc \uc911\uad6d \uc778\ubbfc\ud574\ubc29\uad70\uc740 \uc5bc\ub978 \uc774 \ud55c\uad6d\uc804\uc7c1\uc744 \uc911\ub2e8\ud558\uace0, \ud734\uc804\uc744 \ud558\uae30\ub85c \ud588\uc5c8\ub2e4. \ubbf8\uad6d\uc740 \uc18c\ube44\uc5d0\ud2b8 \uc5f0\ubc29 \ubc0f \uc911\ud654\uc778\ubbfc\uacf5\ud654\uad6d \uce21\uacfc \ud734\uc804\uc744 \ub17c\uc758\ud55c\ub2e4. 1952\ub144 3\uc6d4 \uc774\uc2b9\ub9cc\uc740 \ubd84\ub2e8 \uc0c1\ud0dc\uc5d0\uc11c\uc758 \ud734\uc804\uc740 \ud55c\uad6d\uc5d0 \ub300\ud55c \u201c\uc0ac\ud615\uc120\uace0\u201d\ub098 \ub2e4\ub984\uc5c6\ub2e4\uace0 \uc8fc\uc7a5\ud558\uba70, \u201c\ubbfc\uc871\uad6d\uac00\ub85c \uc0dd\uc874\ud558\uae30 \uc704\ud574 \ub2e8\ub3c5\uc73c\ub85c\ub77c\ub3c4 \uacc4\uc18d\ud574 \uc2f8\uc6cc \ub098\uac08 \uac83\u201d\uc774\ub77c\uace0 \uac15\uc870\ud588\ub2e4.", "sentence_answer": "\ud55c\uad6d \uc804\uc7c1 \uc911 2\ub144 \ub3d9\uc548 \uc774\ub807\ub2e4 \ud55c \uc131\uacfc\ub97c \uc62c\ub9ac\uc9c0 \ubabb\ud558\uace0 \uc911\ud654\uc778\ubbfc\uacf5\ud654\uad6d \uc758 \uac1c\uc785\uc73c\ub85c \uc804\uc120\uc774 \uace0\ucc29\ud654\ud558\uc790, \uc911\ud654\uc778\ubbfc\uacf5\ud654\uad6d\uc744 \ubd80\ub2f4\uc2a4\ub7ec\uc6cc\ud55c \ubbf8\uad6d\uc740 \ud604\uc0c1\uc720\uc9c0 \ucc28\uc6d0\uc5d0\uc11c \ud734\uc804\ud611\uc815\uc744 \ucd94\uc9c4\ud558\uc600\ub2e4.", "paragraph_id": "6466237-50-0", "paragraph_question": "question: \ubbf8\uad6d\uc774 \ud734\uc804\ud611\uc815\uc744 \uc81c\uc548\ud55c \uac83\uc740 \uc5b4\ub290 \ub098\ub77c\ub97c \ubd80\ub2f4\uc2a4\ub7ec\uc6cc\ud588\uae30 \ub54c\ubb38\uc778\uac00?, context: \ud55c\uad6d \uc804\uc7c1 \uc911 2\ub144 \ub3d9\uc548 \uc774\ub807\ub2e4 \ud55c \uc131\uacfc\ub97c \uc62c\ub9ac\uc9c0 \ubabb\ud558\uace0 \uc911\ud654\uc778\ubbfc\uacf5\ud654\uad6d\uc758 \uac1c\uc785\uc73c\ub85c \uc804\uc120\uc774 \uace0\ucc29\ud654\ud558\uc790, \uc911\ud654\uc778\ubbfc\uacf5\ud654\uad6d\uc744 \ubd80\ub2f4\uc2a4\ub7ec\uc6cc\ud55c \ubbf8\uad6d\uc740 \ud604\uc0c1\uc720\uc9c0 \ucc28\uc6d0\uc5d0\uc11c \ud734\uc804\ud611\uc815\uc744 \ucd94\uc9c4\ud558\uc600\ub2e4. \uc18c\ub828\uc774 \ubd80\ubd84\uc801\uc73c\ub85c \ucc38\uc804\ud558\uc790, \uc138\uacc4 \uc5ec\ub7ec\ub098\ub77c\ub4e4\uc740 \ud55c\uad6d \uc804\uc7c1\uc774 \uba38\uc9c0\uc54a\uc544 \ub0c9\uc804\uccb4\uc81c\uc640 \ud55c\uad6d\uc758 \ud1b5\uc77c\uc744 \uc704\ud55c \uc81c3\ucc28 \uc138\uacc4\ub300\uc804\uc774 \uc77c\uc5b4\ub0a0 \uac70\ub77c\ub294 \uac00\ub2a5\uc131\uc774 \ucee4\uc9c8 \uac70\ub77c\uace0 \uc0dd\uac01\ud588\ub2e4. \uadf8\ub798\uc11c \uc720\uc5d4\uad70\uacfc \uc911\uad6d \uc778\ubbfc\ud574\ubc29\uad70\uc740 \uc5bc\ub978 \uc774 \ud55c\uad6d\uc804\uc7c1\uc744 \uc911\ub2e8\ud558\uace0, \ud734\uc804\uc744 \ud558\uae30\ub85c \ud588\uc5c8\ub2e4. \ubbf8\uad6d\uc740 \uc18c\ube44\uc5d0\ud2b8 \uc5f0\ubc29 \ubc0f \uc911\ud654\uc778\ubbfc\uacf5\ud654\uad6d \uce21\uacfc \ud734\uc804\uc744 \ub17c\uc758\ud55c\ub2e4. 1952\ub144 3\uc6d4 \uc774\uc2b9\ub9cc\uc740 \ubd84\ub2e8 \uc0c1\ud0dc\uc5d0\uc11c\uc758 \ud734\uc804\uc740 \ud55c\uad6d\uc5d0 \ub300\ud55c \u201c\uc0ac\ud615\uc120\uace0\u201d\ub098 \ub2e4\ub984\uc5c6\ub2e4\uace0 \uc8fc\uc7a5\ud558\uba70, \u201c\ubbfc\uc871\uad6d\uac00\ub85c \uc0dd\uc874\ud558\uae30 \uc704\ud574 \ub2e8\ub3c5\uc73c\ub85c\ub77c\ub3c4 \uacc4\uc18d\ud574 \uc2f8\uc6cc \ub098\uac08 \uac83\u201d\uc774\ub77c\uace0 \uac15\uc870\ud588\ub2e4."} {"question": "\uc7a5\ud6a8\uc870\uac00 1983\ub144 \uc2dc\uc98c\uc744 \ub9c8\uac10\ud560 \ub54c\uc758 \ud0c0\uc728\uc740?", "paragraph": "\uc2e4\uc5c5 \ud300 \ud3ec\ud56d\uc81c\ucca0\uc5d0 \uc785\ub2e8\ud558\uc600\ub2e4\uac00 \uacbd\ub9ac\ub2e8\uc5d0\uc11c \ubcd1\uc5ed\uc744 \ub9c8\uce5c \ud6c4 \uc0bc\uc131 \ub77c\uc774\uc628\uc988\uc5d0 \uc785\ub2e8\ud574 \uccab \ud574\ubd80\ud130 \uc13c\uc138\uc774\uc158\uc744 \uc77c\uc73c\ucf30\ub2e4. \ud504\ub85c \uc801\uc751\uae30\uac00 \ud544\uc694\ud558\ub2e4\ub294 \uc804\ub9dd\uc744 \ube44\uc6c3\ub4ef 1\uad70 92\uacbd\uae30\uc5d0\uc11c \ud0c0\uc728 3\ud560 6\ud47c 9\ub9ac(1\uc704), 18\ud648\ub7f0(\uacf5\uc704), 22\ub3c4\ub8e8(4\uc704), \uc7a5\ud0c0\uc728 6\ud560 1\ud47c 8\ub9ac(1\uc704), \ucd9c\ub8e8\uc728 4\ud560 7\ud47c 5\ub9ac(1\uc704)\ub85c \ucc9c\uc7ac\uc131\uc744 \uc785\uc99d\ud588\ub2e4. 1983\ub144 5\uc6d4\uc5d0 8\uc5f0\ud0c0\uc11d \uc548\ud0c0\uc5d0 3\uc5f0\ud0c0\uc11d \ud648\ub7f0\uc744 \ub54c\ub824 \ub0b4\uba70 \ud0c0\uc728\uc774 0.414\uae4c\uc9c0 \uce58\uc19f\uc558\ub2e4. \uc774\ub294 KBO \ub9ac\uadf8 \uc5ed\uc0ac\uc0c1 \ucd5c\ucd08\uc774\uba70, 1983\ub144 \uc2dc\uc98c \ub0b4\ub0b4 0.380 \uc774\uc0c1\uc758 \uace0\ud0c0\uc728\uc744 \uae30\ub85d\ud558\uc600\uc73c\ub098, \ub9c9\ud310\uc5d0 \uc57d\uac04 \ubd80\uc9c4\ud558\uc5ec 0.369\uc758 \ud0c0\uc728\ub85c \uc2dc\uc98c\uc744 \ub9c8\uac10\ud588\ub2e4. \uc0bc\uc131 \ub77c\uc774\uc628\uc988\uc758 \ub300\ud45c\uc801 \ud504\ub79c\ucc28\uc774\uc988 \uc2a4\ud0c0\ub85c \ud65c\uc57d\ud558\uba70 '\ud0c0\uaca9\uc758 \ub2ec\uc778', '\uc548\ud0c0 \uc81c\uc870\uae30'\ub4f1\uc758 \uba85\uc131\uc744 \uc5bb\uc5c8\ub2e4. 1985 \uc2dc\uc98c \uc0bc\uc131 \ub77c\uc774\uc628\uc988\uc758 \uc804\u00b7\ud6c4\uae30 \ud1b5\ud569 \uc6b0\uc2b9\uc744 \uc774\ub048 \uc8fc\uc5ed\uc774\uae30\ub3c4 \ud558\ub2e4.", "answer": "0.369", "sentence": "\ub9c9\ud310\uc5d0 \uc57d\uac04 \ubd80\uc9c4\ud558\uc5ec 0.369 \uc758 \ud0c0\uc728\ub85c \uc2dc\uc98c\uc744 \ub9c8\uac10\ud588\ub2e4.", "paragraph_sentence": "\uc2e4\uc5c5 \ud300 \ud3ec\ud56d\uc81c\ucca0\uc5d0 \uc785\ub2e8\ud558\uc600\ub2e4\uac00 \uacbd\ub9ac\ub2e8\uc5d0\uc11c \ubcd1\uc5ed\uc744 \ub9c8\uce5c \ud6c4 \uc0bc\uc131 \ub77c\uc774\uc628\uc988\uc5d0 \uc785\ub2e8\ud574 \uccab \ud574\ubd80\ud130 \uc13c\uc138\uc774\uc158\uc744 \uc77c\uc73c\ucf30\ub2e4. \ud504\ub85c \uc801\uc751\uae30\uac00 \ud544\uc694\ud558\ub2e4\ub294 \uc804\ub9dd\uc744 \ube44\uc6c3\ub4ef 1\uad70 92\uacbd\uae30\uc5d0\uc11c \ud0c0\uc728 3\ud560 6\ud47c 9\ub9ac(1\uc704), 18\ud648\ub7f0(\uacf5\uc704), 22\ub3c4\ub8e8(4\uc704), \uc7a5\ud0c0\uc728 6\ud560 1\ud47c 8\ub9ac(1\uc704), \ucd9c\ub8e8\uc728 4\ud560 7\ud47c 5\ub9ac(1\uc704)\ub85c \ucc9c\uc7ac\uc131\uc744 \uc785\uc99d\ud588\ub2e4. 1983\ub144 5\uc6d4\uc5d0 8\uc5f0\ud0c0\uc11d \uc548\ud0c0\uc5d0 3\uc5f0\ud0c0\uc11d \ud648\ub7f0\uc744 \ub54c\ub824 \ub0b4\uba70 \ud0c0\uc728\uc774 0.414\uae4c\uc9c0 \uce58\uc19f\uc558\ub2e4. \uc774\ub294 KBO \ub9ac\uadf8 \uc5ed\uc0ac\uc0c1 \ucd5c\ucd08\uc774\uba70, 1983\ub144 \uc2dc\uc98c \ub0b4\ub0b4 0.380 \uc774\uc0c1\uc758 \uace0\ud0c0\uc728\uc744 \uae30\ub85d\ud558\uc600\uc73c\ub098, \ub9c9\ud310\uc5d0 \uc57d\uac04 \ubd80\uc9c4\ud558\uc5ec 0.369 \uc758 \ud0c0\uc728\ub85c \uc2dc\uc98c\uc744 \ub9c8\uac10\ud588\ub2e4. \uc0bc\uc131 \ub77c\uc774\uc628\uc988\uc758 \ub300\ud45c\uc801 \ud504\ub79c\ucc28\uc774\uc988 \uc2a4\ud0c0\ub85c \ud65c\uc57d\ud558\uba70 '\ud0c0\uaca9\uc758 \ub2ec\uc778', '\uc548\ud0c0 \uc81c\uc870\uae30'\ub4f1\uc758 \uba85\uc131\uc744 \uc5bb\uc5c8\ub2e4. 1985 \uc2dc\uc98c \uc0bc\uc131 \ub77c\uc774\uc628\uc988\uc758 \uc804\u00b7\ud6c4\uae30 \ud1b5\ud569 \uc6b0\uc2b9\uc744 \uc774\ub048 \uc8fc\uc5ed\uc774\uae30\ub3c4 \ud558\ub2e4.", "paragraph_answer": "\uc2e4\uc5c5 \ud300 \ud3ec\ud56d\uc81c\ucca0\uc5d0 \uc785\ub2e8\ud558\uc600\ub2e4\uac00 \uacbd\ub9ac\ub2e8\uc5d0\uc11c \ubcd1\uc5ed\uc744 \ub9c8\uce5c \ud6c4 \uc0bc\uc131 \ub77c\uc774\uc628\uc988\uc5d0 \uc785\ub2e8\ud574 \uccab \ud574\ubd80\ud130 \uc13c\uc138\uc774\uc158\uc744 \uc77c\uc73c\ucf30\ub2e4. \ud504\ub85c \uc801\uc751\uae30\uac00 \ud544\uc694\ud558\ub2e4\ub294 \uc804\ub9dd\uc744 \ube44\uc6c3\ub4ef 1\uad70 92\uacbd\uae30\uc5d0\uc11c \ud0c0\uc728 3\ud560 6\ud47c 9\ub9ac(1\uc704), 18\ud648\ub7f0(\uacf5\uc704), 22\ub3c4\ub8e8(4\uc704), \uc7a5\ud0c0\uc728 6\ud560 1\ud47c 8\ub9ac(1\uc704), \ucd9c\ub8e8\uc728 4\ud560 7\ud47c 5\ub9ac(1\uc704)\ub85c \ucc9c\uc7ac\uc131\uc744 \uc785\uc99d\ud588\ub2e4. 1983\ub144 5\uc6d4\uc5d0 8\uc5f0\ud0c0\uc11d \uc548\ud0c0\uc5d0 3\uc5f0\ud0c0\uc11d \ud648\ub7f0\uc744 \ub54c\ub824 \ub0b4\uba70 \ud0c0\uc728\uc774 0.414\uae4c\uc9c0 \uce58\uc19f\uc558\ub2e4. \uc774\ub294 KBO \ub9ac\uadf8 \uc5ed\uc0ac\uc0c1 \ucd5c\ucd08\uc774\uba70, 1983\ub144 \uc2dc\uc98c \ub0b4\ub0b4 0.380 \uc774\uc0c1\uc758 \uace0\ud0c0\uc728\uc744 \uae30\ub85d\ud558\uc600\uc73c\ub098, \ub9c9\ud310\uc5d0 \uc57d\uac04 \ubd80\uc9c4\ud558\uc5ec 0.369 \uc758 \ud0c0\uc728\ub85c \uc2dc\uc98c\uc744 \ub9c8\uac10\ud588\ub2e4. \uc0bc\uc131 \ub77c\uc774\uc628\uc988\uc758 \ub300\ud45c\uc801 \ud504\ub79c\ucc28\uc774\uc988 \uc2a4\ud0c0\ub85c \ud65c\uc57d\ud558\uba70 '\ud0c0\uaca9\uc758 \ub2ec\uc778', '\uc548\ud0c0 \uc81c\uc870\uae30'\ub4f1\uc758 \uba85\uc131\uc744 \uc5bb\uc5c8\ub2e4. 1985 \uc2dc\uc98c \uc0bc\uc131 \ub77c\uc774\uc628\uc988\uc758 \uc804\u00b7\ud6c4\uae30 \ud1b5\ud569 \uc6b0\uc2b9\uc744 \uc774\ub048 \uc8fc\uc5ed\uc774\uae30\ub3c4 \ud558\ub2e4.", "sentence_answer": "\ub9c9\ud310\uc5d0 \uc57d\uac04 \ubd80\uc9c4\ud558\uc5ec 0.369 \uc758 \ud0c0\uc728\ub85c \uc2dc\uc98c\uc744 \ub9c8\uac10\ud588\ub2e4.", "paragraph_id": "6592760-1-1", "paragraph_question": "question: \uc7a5\ud6a8\uc870\uac00 1983\ub144 \uc2dc\uc98c\uc744 \ub9c8\uac10\ud560 \ub54c\uc758 \ud0c0\uc728\uc740?, context: \uc2e4\uc5c5 \ud300 \ud3ec\ud56d\uc81c\ucca0\uc5d0 \uc785\ub2e8\ud558\uc600\ub2e4\uac00 \uacbd\ub9ac\ub2e8\uc5d0\uc11c \ubcd1\uc5ed\uc744 \ub9c8\uce5c \ud6c4 \uc0bc\uc131 \ub77c\uc774\uc628\uc988\uc5d0 \uc785\ub2e8\ud574 \uccab \ud574\ubd80\ud130 \uc13c\uc138\uc774\uc158\uc744 \uc77c\uc73c\ucf30\ub2e4. \ud504\ub85c \uc801\uc751\uae30\uac00 \ud544\uc694\ud558\ub2e4\ub294 \uc804\ub9dd\uc744 \ube44\uc6c3\ub4ef 1\uad70 92\uacbd\uae30\uc5d0\uc11c \ud0c0\uc728 3\ud560 6\ud47c 9\ub9ac(1\uc704), 18\ud648\ub7f0(\uacf5\uc704), 22\ub3c4\ub8e8(4\uc704), \uc7a5\ud0c0\uc728 6\ud560 1\ud47c 8\ub9ac(1\uc704), \ucd9c\ub8e8\uc728 4\ud560 7\ud47c 5\ub9ac(1\uc704)\ub85c \ucc9c\uc7ac\uc131\uc744 \uc785\uc99d\ud588\ub2e4. 1983\ub144 5\uc6d4\uc5d0 8\uc5f0\ud0c0\uc11d \uc548\ud0c0\uc5d0 3\uc5f0\ud0c0\uc11d \ud648\ub7f0\uc744 \ub54c\ub824 \ub0b4\uba70 \ud0c0\uc728\uc774 0.414\uae4c\uc9c0 \uce58\uc19f\uc558\ub2e4. \uc774\ub294 KBO \ub9ac\uadf8 \uc5ed\uc0ac\uc0c1 \ucd5c\ucd08\uc774\uba70, 1983\ub144 \uc2dc\uc98c \ub0b4\ub0b4 0.380 \uc774\uc0c1\uc758 \uace0\ud0c0\uc728\uc744 \uae30\ub85d\ud558\uc600\uc73c\ub098, \ub9c9\ud310\uc5d0 \uc57d\uac04 \ubd80\uc9c4\ud558\uc5ec 0.369\uc758 \ud0c0\uc728\ub85c \uc2dc\uc98c\uc744 \ub9c8\uac10\ud588\ub2e4. \uc0bc\uc131 \ub77c\uc774\uc628\uc988\uc758 \ub300\ud45c\uc801 \ud504\ub79c\ucc28\uc774\uc988 \uc2a4\ud0c0\ub85c \ud65c\uc57d\ud558\uba70 '\ud0c0\uaca9\uc758 \ub2ec\uc778', '\uc548\ud0c0 \uc81c\uc870\uae30'\ub4f1\uc758 \uba85\uc131\uc744 \uc5bb\uc5c8\ub2e4. 1985 \uc2dc\uc98c \uc0bc\uc131 \ub77c\uc774\uc628\uc988\uc758 \uc804\u00b7\ud6c4\uae30 \ud1b5\ud569 \uc6b0\uc2b9\uc744 \uc774\ub048 \uc8fc\uc5ed\uc774\uae30\ub3c4 \ud558\ub2e4."} {"question": "1911\ub144 \uc6d4\ub4dc \uc2dc\ub9ac\uc988\uc5d0\uc11c \ud544\ub77c\ub378\ud53c\uc544 \uc560\uc2ac\ub808\ud2f1\uc2a4\uc640 \ubd99\uc740 \ud300\uc740?", "paragraph": "\uc774 \ud574 \ud544\ub77c\ub378\ud53c\uc544 \uc560\uc2ac\ub808\ud2f1\uc2a4\ub294 101\uc2b9 50\ud328\ub97c \uac70\ub46c 2\ub144 \uc5f0\uc18d \uc544\uba54\ub9ac\uce78 \ub9ac\uadf8 \uc6b0\uc2b9\uc5d0 \ub3c4\ub2ec\ud588\ub2e4. \ub0b4\uc154\ub110 \ub9ac\uadf8\uc758 \ub274\uc695 \uc790\uc774\uc5b8\uce20\uc640 \ub9de\ubd99\uc740 1911\ub144 \uc6d4\ub4dc \uc2dc\ub9ac\uc988\uc5d0\uc11c \uc560\uc2ac\ub808\ud2f1\uc2a4\uc758 \ud22c\uc218\uc9c4\uc740 \uce58\ud504 \ubca4\ub354\uc640 \uc7ad \ucff0\uc2a4, \uadf8\ub9ac\uace0 \uc5d0\ub514 \ud50c\ub7ad\ud06c\ub85c \uad6c\uc131\ub418\uc5c8\ub2e4. \ucff0\uc2a4\ub294 3\ucc28\uc804\uacfc 5\ucc28\uc804\uc5d0 \uac01\uac01 \uc120\ubc1c \ub4f1\ud310\ud588\ub2e4. 3\ucc28\uc804\uc5d0\uc11c\ub294 \ud06c\ub9ac\uc2a4\ud2f0 \ub9e4\uc288\uc2a8\uacfc \ub9de\ub300\uacb0\uc744 \ubc8c\uc600\ub294\ub370, \uc5f0\uc7a5 11\ud68c\uae4c\uc9c0 \uac00\ub294 \uc2b9\ubd80 \ub05d\uc5d0 \ucff0\uc2a4\uac00 \uc2b9\ub9ac\ub97c \uac70\ub450\uba70 \uc560\uc2ac\ub808\ud2f1\uc2a4\ub97c \ub3c4\uc654\ub2e4. 5\ucc28\uc804\uc5d0\uc11c\ub294 \ub8e8\ube0c \ub9c8\ucffc\ub4dc\uc640 \ub9de\ub300\uacb0\uc744 \ubc8c\uc600\ub2e4. \ucff0\uc2a4\ub294 6\ud68c \uc0ac\ud0c0\uad6c\ub2c8 \ubd80\uc0c1\uc744 \ub2f9\ud588\uc9c0\ub9cc \ub9c8\uc6b4\ub4dc\uc5d0\uc11c \ub0b4\ub824\uc624\uc9c0 \uc54a\uace0 \ubc84\ud17c\ub2e4. \uacb0\uad6d 9\uc774\ub2dd 8\ud53c\uc548\ud0c0 9\ud0c8\uc0bc\uc9c4 3\uc2e4\uc810\uc744 \ud588\uace0 \uc2b9\ubd80\ub294 \ub098\uc9c0 \uc54a\uc558\ub2e4. \uacbd\uae30\ub294 \uc5f0\uc7a5\uc73c\ub85c \ud758\ub7ec\uac14\uace0, \uc790\uc774\uc5b8\uce20\uac00 \uc2b9\ub9ac\ub97c \uac00\uc838\uac14\ub2e4. 6\ucc28\uc804\uae4c\uc9c0 \uac00\ub294 \uc2b9\ubd80 \ub05d\uc5d0 \uc560\uc2ac\ub808\ud2f1\uc2a4\ub294 4\uc2b9 2\ud328\ub85c \uc2dc\ub9ac\uc988 \uc6b0\uc2b9\uc744 \ucc28\uc9c0\ud588\ub294\ub370, \uc560\uc2ac\ub808\ud2f1\uc2a4\uc758 \ud22c\uc218\uc9c4\ub4e4\uc774 6\uacbd\uae30\uc5d0\uc11c 8\uc2e4\uc810\uc73c\ub85c \uc798 \ud2c0\uc5b4\ub9c9\uc558\ub2e4.", "answer": "\ub274\uc695 \uc790\uc774\uc5b8\uce20", "sentence": "\ub0b4\uc154\ub110 \ub9ac\uadf8\uc758 \ub274\uc695 \uc790\uc774\uc5b8\uce20 \uc640 \ub9de\ubd99\uc740 1911\ub144 \uc6d4\ub4dc \uc2dc\ub9ac\uc988\uc5d0\uc11c \uc560\uc2ac\ub808\ud2f1\uc2a4\uc758 \ud22c\uc218\uc9c4\uc740 \uce58\ud504 \ubca4\ub354\uc640 \uc7ad \ucff0\uc2a4, \uadf8\ub9ac\uace0 \uc5d0\ub514 \ud50c\ub7ad\ud06c\ub85c \uad6c\uc131\ub418\uc5c8\ub2e4.", "paragraph_sentence": "\uc774 \ud574 \ud544\ub77c\ub378\ud53c\uc544 \uc560\uc2ac\ub808\ud2f1\uc2a4\ub294 101\uc2b9 50\ud328\ub97c \uac70\ub46c 2\ub144 \uc5f0\uc18d \uc544\uba54\ub9ac\uce78 \ub9ac\uadf8 \uc6b0\uc2b9\uc5d0 \ub3c4\ub2ec\ud588\ub2e4. \ub0b4\uc154\ub110 \ub9ac\uadf8\uc758 \ub274\uc695 \uc790\uc774\uc5b8\uce20 \uc640 \ub9de\ubd99\uc740 1911\ub144 \uc6d4\ub4dc \uc2dc\ub9ac\uc988\uc5d0\uc11c \uc560\uc2ac\ub808\ud2f1\uc2a4\uc758 \ud22c\uc218\uc9c4\uc740 \uce58\ud504 \ubca4\ub354\uc640 \uc7ad \ucff0\uc2a4, \uadf8\ub9ac\uace0 \uc5d0\ub514 \ud50c\ub7ad\ud06c\ub85c \uad6c\uc131\ub418\uc5c8\ub2e4. \ucff0\uc2a4\ub294 3\ucc28\uc804\uacfc 5\ucc28\uc804\uc5d0 \uac01\uac01 \uc120\ubc1c \ub4f1\ud310\ud588\ub2e4. 3\ucc28\uc804\uc5d0\uc11c\ub294 \ud06c\ub9ac\uc2a4\ud2f0 \ub9e4\uc288\uc2a8\uacfc \ub9de\ub300\uacb0\uc744 \ubc8c\uc600\ub294\ub370, \uc5f0\uc7a5 11\ud68c\uae4c\uc9c0 \uac00\ub294 \uc2b9\ubd80 \ub05d\uc5d0 \ucff0\uc2a4\uac00 \uc2b9\ub9ac\ub97c \uac70\ub450\uba70 \uc560\uc2ac\ub808\ud2f1\uc2a4\ub97c \ub3c4\uc654\ub2e4. 5\ucc28\uc804\uc5d0\uc11c\ub294 \ub8e8\ube0c \ub9c8\ucffc\ub4dc\uc640 \ub9de\ub300\uacb0\uc744 \ubc8c\uc600\ub2e4. \ucff0\uc2a4\ub294 6\ud68c \uc0ac\ud0c0\uad6c\ub2c8 \ubd80\uc0c1\uc744 \ub2f9\ud588\uc9c0\ub9cc \ub9c8\uc6b4\ub4dc\uc5d0\uc11c \ub0b4\ub824\uc624\uc9c0 \uc54a\uace0 \ubc84\ud17c\ub2e4. \uacb0\uad6d 9\uc774\ub2dd 8\ud53c\uc548\ud0c0 9\ud0c8\uc0bc\uc9c4 3\uc2e4\uc810\uc744 \ud588\uace0 \uc2b9\ubd80\ub294 \ub098\uc9c0 \uc54a\uc558\ub2e4. \uacbd\uae30\ub294 \uc5f0\uc7a5\uc73c\ub85c \ud758\ub7ec\uac14\uace0, \uc790\uc774\uc5b8\uce20\uac00 \uc2b9\ub9ac\ub97c \uac00\uc838\uac14\ub2e4. 6\ucc28\uc804\uae4c\uc9c0 \uac00\ub294 \uc2b9\ubd80 \ub05d\uc5d0 \uc560\uc2ac\ub808\ud2f1\uc2a4\ub294 4\uc2b9 2\ud328\ub85c \uc2dc\ub9ac\uc988 \uc6b0\uc2b9\uc744 \ucc28\uc9c0\ud588\ub294\ub370, \uc560\uc2ac\ub808\ud2f1\uc2a4\uc758 \ud22c\uc218\uc9c4\ub4e4\uc774 6\uacbd\uae30\uc5d0\uc11c 8\uc2e4\uc810\uc73c\ub85c \uc798 \ud2c0\uc5b4\ub9c9\uc558\ub2e4.", "paragraph_answer": "\uc774 \ud574 \ud544\ub77c\ub378\ud53c\uc544 \uc560\uc2ac\ub808\ud2f1\uc2a4\ub294 101\uc2b9 50\ud328\ub97c \uac70\ub46c 2\ub144 \uc5f0\uc18d \uc544\uba54\ub9ac\uce78 \ub9ac\uadf8 \uc6b0\uc2b9\uc5d0 \ub3c4\ub2ec\ud588\ub2e4. \ub0b4\uc154\ub110 \ub9ac\uadf8\uc758 \ub274\uc695 \uc790\uc774\uc5b8\uce20 \uc640 \ub9de\ubd99\uc740 1911\ub144 \uc6d4\ub4dc \uc2dc\ub9ac\uc988\uc5d0\uc11c \uc560\uc2ac\ub808\ud2f1\uc2a4\uc758 \ud22c\uc218\uc9c4\uc740 \uce58\ud504 \ubca4\ub354\uc640 \uc7ad \ucff0\uc2a4, \uadf8\ub9ac\uace0 \uc5d0\ub514 \ud50c\ub7ad\ud06c\ub85c \uad6c\uc131\ub418\uc5c8\ub2e4. \ucff0\uc2a4\ub294 3\ucc28\uc804\uacfc 5\ucc28\uc804\uc5d0 \uac01\uac01 \uc120\ubc1c \ub4f1\ud310\ud588\ub2e4. 3\ucc28\uc804\uc5d0\uc11c\ub294 \ud06c\ub9ac\uc2a4\ud2f0 \ub9e4\uc288\uc2a8\uacfc \ub9de\ub300\uacb0\uc744 \ubc8c\uc600\ub294\ub370, \uc5f0\uc7a5 11\ud68c\uae4c\uc9c0 \uac00\ub294 \uc2b9\ubd80 \ub05d\uc5d0 \ucff0\uc2a4\uac00 \uc2b9\ub9ac\ub97c \uac70\ub450\uba70 \uc560\uc2ac\ub808\ud2f1\uc2a4\ub97c \ub3c4\uc654\ub2e4. 5\ucc28\uc804\uc5d0\uc11c\ub294 \ub8e8\ube0c \ub9c8\ucffc\ub4dc\uc640 \ub9de\ub300\uacb0\uc744 \ubc8c\uc600\ub2e4. \ucff0\uc2a4\ub294 6\ud68c \uc0ac\ud0c0\uad6c\ub2c8 \ubd80\uc0c1\uc744 \ub2f9\ud588\uc9c0\ub9cc \ub9c8\uc6b4\ub4dc\uc5d0\uc11c \ub0b4\ub824\uc624\uc9c0 \uc54a\uace0 \ubc84\ud17c\ub2e4. \uacb0\uad6d 9\uc774\ub2dd 8\ud53c\uc548\ud0c0 9\ud0c8\uc0bc\uc9c4 3\uc2e4\uc810\uc744 \ud588\uace0 \uc2b9\ubd80\ub294 \ub098\uc9c0 \uc54a\uc558\ub2e4. \uacbd\uae30\ub294 \uc5f0\uc7a5\uc73c\ub85c \ud758\ub7ec\uac14\uace0, \uc790\uc774\uc5b8\uce20\uac00 \uc2b9\ub9ac\ub97c \uac00\uc838\uac14\ub2e4. 6\ucc28\uc804\uae4c\uc9c0 \uac00\ub294 \uc2b9\ubd80 \ub05d\uc5d0 \uc560\uc2ac\ub808\ud2f1\uc2a4\ub294 4\uc2b9 2\ud328\ub85c \uc2dc\ub9ac\uc988 \uc6b0\uc2b9\uc744 \ucc28\uc9c0\ud588\ub294\ub370, \uc560\uc2ac\ub808\ud2f1\uc2a4\uc758 \ud22c\uc218\uc9c4\ub4e4\uc774 6\uacbd\uae30\uc5d0\uc11c 8\uc2e4\uc810\uc73c\ub85c \uc798 \ud2c0\uc5b4\ub9c9\uc558\ub2e4.", "sentence_answer": "\ub0b4\uc154\ub110 \ub9ac\uadf8\uc758 \ub274\uc695 \uc790\uc774\uc5b8\uce20 \uc640 \ub9de\ubd99\uc740 1911\ub144 \uc6d4\ub4dc \uc2dc\ub9ac\uc988\uc5d0\uc11c \uc560\uc2ac\ub808\ud2f1\uc2a4\uc758 \ud22c\uc218\uc9c4\uc740 \uce58\ud504 \ubca4\ub354\uc640 \uc7ad \ucff0\uc2a4, \uadf8\ub9ac\uace0 \uc5d0\ub514 \ud50c\ub7ad\ud06c\ub85c \uad6c\uc131\ub418\uc5c8\ub2e4.", "paragraph_id": "6549325-7-0", "paragraph_question": "question: 1911\ub144 \uc6d4\ub4dc \uc2dc\ub9ac\uc988\uc5d0\uc11c \ud544\ub77c\ub378\ud53c\uc544 \uc560\uc2ac\ub808\ud2f1\uc2a4\uc640 \ubd99\uc740 \ud300\uc740?, context: \uc774 \ud574 \ud544\ub77c\ub378\ud53c\uc544 \uc560\uc2ac\ub808\ud2f1\uc2a4\ub294 101\uc2b9 50\ud328\ub97c \uac70\ub46c 2\ub144 \uc5f0\uc18d \uc544\uba54\ub9ac\uce78 \ub9ac\uadf8 \uc6b0\uc2b9\uc5d0 \ub3c4\ub2ec\ud588\ub2e4. \ub0b4\uc154\ub110 \ub9ac\uadf8\uc758 \ub274\uc695 \uc790\uc774\uc5b8\uce20\uc640 \ub9de\ubd99\uc740 1911\ub144 \uc6d4\ub4dc \uc2dc\ub9ac\uc988\uc5d0\uc11c \uc560\uc2ac\ub808\ud2f1\uc2a4\uc758 \ud22c\uc218\uc9c4\uc740 \uce58\ud504 \ubca4\ub354\uc640 \uc7ad \ucff0\uc2a4, \uadf8\ub9ac\uace0 \uc5d0\ub514 \ud50c\ub7ad\ud06c\ub85c \uad6c\uc131\ub418\uc5c8\ub2e4. \ucff0\uc2a4\ub294 3\ucc28\uc804\uacfc 5\ucc28\uc804\uc5d0 \uac01\uac01 \uc120\ubc1c \ub4f1\ud310\ud588\ub2e4. 3\ucc28\uc804\uc5d0\uc11c\ub294 \ud06c\ub9ac\uc2a4\ud2f0 \ub9e4\uc288\uc2a8\uacfc \ub9de\ub300\uacb0\uc744 \ubc8c\uc600\ub294\ub370, \uc5f0\uc7a5 11\ud68c\uae4c\uc9c0 \uac00\ub294 \uc2b9\ubd80 \ub05d\uc5d0 \ucff0\uc2a4\uac00 \uc2b9\ub9ac\ub97c \uac70\ub450\uba70 \uc560\uc2ac\ub808\ud2f1\uc2a4\ub97c \ub3c4\uc654\ub2e4. 5\ucc28\uc804\uc5d0\uc11c\ub294 \ub8e8\ube0c \ub9c8\ucffc\ub4dc\uc640 \ub9de\ub300\uacb0\uc744 \ubc8c\uc600\ub2e4. \ucff0\uc2a4\ub294 6\ud68c \uc0ac\ud0c0\uad6c\ub2c8 \ubd80\uc0c1\uc744 \ub2f9\ud588\uc9c0\ub9cc \ub9c8\uc6b4\ub4dc\uc5d0\uc11c \ub0b4\ub824\uc624\uc9c0 \uc54a\uace0 \ubc84\ud17c\ub2e4. \uacb0\uad6d 9\uc774\ub2dd 8\ud53c\uc548\ud0c0 9\ud0c8\uc0bc\uc9c4 3\uc2e4\uc810\uc744 \ud588\uace0 \uc2b9\ubd80\ub294 \ub098\uc9c0 \uc54a\uc558\ub2e4. \uacbd\uae30\ub294 \uc5f0\uc7a5\uc73c\ub85c \ud758\ub7ec\uac14\uace0, \uc790\uc774\uc5b8\uce20\uac00 \uc2b9\ub9ac\ub97c \uac00\uc838\uac14\ub2e4. 6\ucc28\uc804\uae4c\uc9c0 \uac00\ub294 \uc2b9\ubd80 \ub05d\uc5d0 \uc560\uc2ac\ub808\ud2f1\uc2a4\ub294 4\uc2b9 2\ud328\ub85c \uc2dc\ub9ac\uc988 \uc6b0\uc2b9\uc744 \ucc28\uc9c0\ud588\ub294\ub370, \uc560\uc2ac\ub808\ud2f1\uc2a4\uc758 \ud22c\uc218\uc9c4\ub4e4\uc774 6\uacbd\uae30\uc5d0\uc11c 8\uc2e4\uc810\uc73c\ub85c \uc798 \ud2c0\uc5b4\ub9c9\uc558\ub2e4."} {"question": "1\uc138\uae30 \uacbd \ub3d9\ub0a8\uc544\uc2dc\uc544\uc5d0\uc11c \uac00\uc7a5 \ud070 \ud56d\uad6c\ub294 \ubb34\uc5c7\uc778\uac00?", "paragraph": "\ud638\uc774\uc548\uc758 \ucd08\uae30 \uc5ed\uc0ac\ub294 \ucc38 \uc871\uc758 \uc5ed\uc0ac\uc774\ub2e4. \ub9d0\ub808\uc774\uacc4\uc778 \ucc38 \uc871\uc740 \uae30\uc6d0\uc804 200\ub144\uacbd\uacfc \uae30\uc6d0\ud6c4 200\ub144 \uacbd\uc5d0 \uc790\ubc14\uc5d0\uc11c \uc774 \uacf3\uc73c\ub85c \uac74\ub108\uc654\ub2e4. \uc774\ub4e4\uc740 \ucc38\ud30c \uc655\uad6d\uc744 \uc774\ub8e8\uc5c8\uace0 \ud6fc\uc640 \ub0d0\uc9f1\uae4c\uc9c0 \uc774 \uc655\uad6d\uc758 \uc138\ub825\ud558\uc5d0 \uc788\uc5c8\ub2e4. 1\uc138\uae30 \uacbd\uc5d0\ub294 \ub3d9\ub0a8\uc544\uc2dc\uc544\uc5d0\uc11c \uac00\uc7a5 \uaddc\ubaa8\uac00 \ud070 \ud56d\uad6c\uac00 \uc5ec\uae30\uc5d0 \uc788\uc5c8\uace0, \ub78c\uc555\ud3ec(L\u00e2m \u1ea4p Ph\u1ed1)\ub77c\uace0 \uc54c\ub824\uc838 \uc788\ub2e4. \ud22c\ubcf8(Thu Bon)\uac15\uc758 \uc5b4\uadc0\uc5d0\ub294 \ucc38\ud30c \uc0ac\ub78c\ub4e4\uc758 \uc608\uc804 \ud56d\uad6c\uac00 \uc788\uc5c8\ub2e4. \uc774 \ud56d\uad6c\ub294 \uc9c0\ub9ac\uc801 \uc5ec\uac74\uc73c\ub85c \uc778\ud558\uc5ec 16\uc138\uae30 \ub9d0\ubd80\ud130 17\uc138\uae30 \ucd08\uae4c\uc9c0(\ub113\uac8c \ubcf4\uba74 15\uc138\uae30\ubd80\ud130 19\uc138\uae30 \ubb34\ub835\uae4c\uc9c0) \ubca0\ud2b8\ub0a8\uc758 \"\ubc14\ub2e4\uc758 \uc2e4\ud06c\ub85c\ub4dc\"\ub77c\uace0 \ubd88\ub9ac\ub358 \uc911\uc694\ud55c \uad6d\uc81c \ubb34\uc5ed \ud56d\uad6c\uc600\uace0, \uc5ec\ub7ec \uc131(\u7701) \ucd9c\uc2e0\uc758 \ud654\uad50\uc640 \uc77c\ubcf8\uc778, \ub124\ub35c\ub780\ub4dc\uc778 \ub4f1 \uc11c\uad6c \uc0c1\uc778 \uadf8\ub9ac\uace0 \uc778\ub3c4\uc778\ub4e4\uc774 \ub4dc\ub098\ub4e4\uc5c8\uace0 \ub9c8\uc744\uc744 \ud615\uc131\ud558\uc5ec \uc815\ucc29\ud558\uc600\ub358 \uacf3\uc774\ub2e4. \uadf8\ub798\uc11c \uc774 \ub9c8\uc744\uc5d0\ub294 \uc11c\uad6c\uc801\uc774\uba74\uc11c\ub3c4 \ub3d9\uc591\uc801\uc778 \ud48d\uacbd\uc774 \uc790\ub9ac\uc7a1\uae30 \uc2dc\uc791\ud588\ub2e4. \uac70\uae30\uc5d0 \ud654\uad50\ub4e4\uc774 \ud638\uc774\uc548\uc5d0 \uc815\ucc29\ud574 \uc0b4\uae30 \uc2dc\uc791\ud558\uba74\uc11c '\ub3d9\ud654\uc801'\uc778 \uc0c9\ub2e4\ub978 \ubd84\uc704\uae30\uac00 \ud615\uc131\ub410\ub2e4. \ud654\uad50\ub97c \uc911\uc2ec\uc73c\ub85c \ud55c \ubb34\uc5ed\uc774 \ubc88\uc131\ud558\ub358 \ub2f9\uc2dc\uc5d0 \uc774 \ub9c8\uc744\uc740 \ubca0\ud2b8\ub0a8\uc5b4\ub85c \ud558\uc774\ud3ec(Hai Pho)\ub77c\uace0 \ubd88\ub800\uace0 \uc774\ub294 \"\ubc14\ub2f7\uac00 \ub9c8\uc744\"\uc774\ub77c\ub294 \ub73b\uc774\uc5c8\ub2e4. \uc6d0\ub798 \ud558\uc774\ud3ec\ub294 \"\uc77c\ubcf8 \ub2e4\ub9ac\"\ub97c \uac00\ub85c\uc9c8\ub7ec \ub9c8\uc744\uc774 \ub098\ub258\uc5c8\ub2e4. \ud558\ub098\ub294 \uc77c\ubcf8\uc778\uc774 \uac70\uc8fc\ud558\ub358 \uacf3\uc774\uc5c8\ub2e4. 16~17\uc138\uae30, \ubb34\uc5ed\uc774 \ubc88\uc131\ud588\uc744 \ub2f9\uc2dc \ud638\uc774\uc548\uc5d0\ub294 \uc77c\ubcf8\uc778\ub4e4\uc774 \ud2b9\ud788 \ub9ce\uc774 \ub4dc\ub098\ub4e4\uc5c8\uace0 \uadf8\ub798\uc11c \uc77c\ubcf8\uc778 \ub9c8\uc744\uae4c\uc9c0 \ub530\ub85c \uc0dd\uaca8\ub0ac\ub2e4. \uc804\uc131\uae30\uc5d0\ub294 1000\uba85\uc774 \ub118\ub294 \uc77c\ubcf8\uc778\uc774 \uac70\uc8fc\ud588\uc744 \uc815\ub3c4\ub85c \ub9c8\uc744 \uaddc\ubaa8\uac00 \ucee4\uc84c\ub2e4. \ud558\uc9c0\ub9cc \uc5d0\ub3c4 \uc2dc\ub300 \uc1c4\uad6d\uc815\ucc45\uc774 \uc2dc\ud589\ub418\uba74\uc11c \uc77c\ubcf8\uc778 \uc218\uac00 \uc810\ucc28 \uc904\uc5b4\ub4e4\uc5c8\uace0 \uc77c\ubcf8\uc778 \ub9c8\uc744 \uc5ed\uc2dc \uc790\uc5f0\uc2a4\ub7fd\uac8c \uc0ac\ub77c\uc84c\ub2e4. \uadf8 \uc77c\ubcf8 \ub2e4\ub9ac(Ch\u00f9a c\u1ea7u)\ub294 \uc77c\ubcf8\uc778\uc5d0 \uc758\ud558\uc5ec \uc9c0\uc5b4\uc84c\ub294\ub370 \ub3c5\ud2b9\ud558\uac8c\ub3c4 \uad6c\uc870\ubb3c\uc774 \ub2e4\ub9ac\ub97c \ub36e\uace0 \uc788\uace0, \ub2e4\ub9ac\uc758 \ud55c\ucabd\uc73c\ub85c\ub294 \uc0ac\ucc30\uc774 \uc5f0\uacb0\ub418\ub294 \uad6c\uc870\ubb3c\uc744 \uac00\uc9c4 \ub2e4\ub9ac\uc774\ub2e4.", "answer": "\ub78c\uc555\ud3ec", "sentence": "1\uc138\uae30 \uacbd\uc5d0\ub294 \ub3d9\ub0a8\uc544\uc2dc\uc544\uc5d0\uc11c \uac00\uc7a5 \uaddc\ubaa8\uac00 \ud070 \ud56d\uad6c\uac00 \uc5ec\uae30\uc5d0 \uc788\uc5c8\uace0, \ub78c\uc555\ud3ec (L\u00e2m \u1ea4p Ph\u1ed1)", "paragraph_sentence": "\ud638\uc774\uc548\uc758 \ucd08\uae30 \uc5ed\uc0ac\ub294 \ucc38 \uc871\uc758 \uc5ed\uc0ac\uc774\ub2e4. \ub9d0\ub808\uc774\uacc4\uc778 \ucc38 \uc871\uc740 \uae30\uc6d0\uc804 200\ub144\uacbd\uacfc \uae30\uc6d0\ud6c4 200\ub144 \uacbd\uc5d0 \uc790\ubc14\uc5d0\uc11c \uc774 \uacf3\uc73c\ub85c \uac74\ub108\uc654\ub2e4. \uc774\ub4e4\uc740 \ucc38\ud30c \uc655\uad6d\uc744 \uc774\ub8e8\uc5c8\uace0 \ud6fc\uc640 \ub0d0\uc9f1\uae4c\uc9c0 \uc774 \uc655\uad6d\uc758 \uc138\ub825\ud558\uc5d0 \uc788\uc5c8\ub2e4. 1\uc138\uae30 \uacbd\uc5d0\ub294 \ub3d9\ub0a8\uc544\uc2dc\uc544\uc5d0\uc11c \uac00\uc7a5 \uaddc\ubaa8\uac00 \ud070 \ud56d\uad6c\uac00 \uc5ec\uae30\uc5d0 \uc788\uc5c8\uace0, \ub78c\uc555\ud3ec (L\u00e2m \u1ea4p Ph\u1ed1) \ub77c\uace0 \uc54c\ub824\uc838 \uc788\ub2e4. \ud22c\ubcf8(Thu Bon)\uac15\uc758 \uc5b4\uadc0\uc5d0\ub294 \ucc38\ud30c \uc0ac\ub78c\ub4e4\uc758 \uc608\uc804 \ud56d\uad6c\uac00 \uc788\uc5c8\ub2e4. \uc774 \ud56d\uad6c\ub294 \uc9c0\ub9ac\uc801 \uc5ec\uac74\uc73c\ub85c \uc778\ud558\uc5ec 16\uc138\uae30 \ub9d0\ubd80\ud130 17\uc138\uae30 \ucd08\uae4c\uc9c0(\ub113\uac8c \ubcf4\uba74 15\uc138\uae30\ubd80\ud130 19\uc138\uae30 \ubb34\ub835\uae4c\uc9c0) \ubca0\ud2b8\ub0a8\uc758 \"\ubc14\ub2e4\uc758 \uc2e4\ud06c\ub85c\ub4dc\"\ub77c\uace0 \ubd88\ub9ac\ub358 \uc911\uc694\ud55c \uad6d\uc81c \ubb34\uc5ed \ud56d\uad6c\uc600\uace0, \uc5ec\ub7ec \uc131(\u7701) \ucd9c\uc2e0\uc758 \ud654\uad50\uc640 \uc77c\ubcf8\uc778, \ub124\ub35c\ub780\ub4dc\uc778 \ub4f1 \uc11c\uad6c \uc0c1\uc778 \uadf8\ub9ac\uace0 \uc778\ub3c4\uc778\ub4e4\uc774 \ub4dc\ub098\ub4e4\uc5c8\uace0 \ub9c8\uc744\uc744 \ud615\uc131\ud558\uc5ec \uc815\ucc29\ud558\uc600\ub358 \uacf3\uc774\ub2e4. \uadf8\ub798\uc11c \uc774 \ub9c8\uc744\uc5d0\ub294 \uc11c\uad6c\uc801\uc774\uba74\uc11c\ub3c4 \ub3d9\uc591\uc801\uc778 \ud48d\uacbd\uc774 \uc790\ub9ac\uc7a1\uae30 \uc2dc\uc791\ud588\ub2e4. \uac70\uae30\uc5d0 \ud654\uad50\ub4e4\uc774 \ud638\uc774\uc548\uc5d0 \uc815\ucc29\ud574 \uc0b4\uae30 \uc2dc\uc791\ud558\uba74\uc11c '\ub3d9\ud654\uc801'\uc778 \uc0c9\ub2e4\ub978 \ubd84\uc704\uae30\uac00 \ud615\uc131\ub410\ub2e4. \ud654\uad50\ub97c \uc911\uc2ec\uc73c\ub85c \ud55c \ubb34\uc5ed\uc774 \ubc88\uc131\ud558\ub358 \ub2f9\uc2dc\uc5d0 \uc774 \ub9c8\uc744\uc740 \ubca0\ud2b8\ub0a8\uc5b4\ub85c \ud558\uc774\ud3ec(Hai Pho)\ub77c\uace0 \ubd88\ub800\uace0 \uc774\ub294 \"\ubc14\ub2f7\uac00 \ub9c8\uc744\"\uc774\ub77c\ub294 \ub73b\uc774\uc5c8\ub2e4. \uc6d0\ub798 \ud558\uc774\ud3ec\ub294 \"\uc77c\ubcf8 \ub2e4\ub9ac\"\ub97c \uac00\ub85c\uc9c8\ub7ec \ub9c8\uc744\uc774 \ub098\ub258\uc5c8\ub2e4. \ud558\ub098\ub294 \uc77c\ubcf8\uc778\uc774 \uac70\uc8fc\ud558\ub358 \uacf3\uc774\uc5c8\ub2e4. 16~17\uc138\uae30, \ubb34\uc5ed\uc774 \ubc88\uc131\ud588\uc744 \ub2f9\uc2dc \ud638\uc774\uc548\uc5d0\ub294 \uc77c\ubcf8\uc778\ub4e4\uc774 \ud2b9\ud788 \ub9ce\uc774 \ub4dc\ub098\ub4e4\uc5c8\uace0 \uadf8\ub798\uc11c \uc77c\ubcf8\uc778 \ub9c8\uc744\uae4c\uc9c0 \ub530\ub85c \uc0dd\uaca8\ub0ac\ub2e4. \uc804\uc131\uae30\uc5d0\ub294 1000\uba85\uc774 \ub118\ub294 \uc77c\ubcf8\uc778\uc774 \uac70\uc8fc\ud588\uc744 \uc815\ub3c4\ub85c \ub9c8\uc744 \uaddc\ubaa8\uac00 \ucee4\uc84c\ub2e4. \ud558\uc9c0\ub9cc \uc5d0\ub3c4 \uc2dc\ub300 \uc1c4\uad6d\uc815\ucc45\uc774 \uc2dc\ud589\ub418\uba74\uc11c \uc77c\ubcf8\uc778 \uc218\uac00 \uc810\ucc28 \uc904\uc5b4\ub4e4\uc5c8\uace0 \uc77c\ubcf8\uc778 \ub9c8\uc744 \uc5ed\uc2dc \uc790\uc5f0\uc2a4\ub7fd\uac8c \uc0ac\ub77c\uc84c\ub2e4. \uadf8 \uc77c\ubcf8 \ub2e4\ub9ac(Ch\u00f9a c\u1ea7u)\ub294 \uc77c\ubcf8\uc778\uc5d0 \uc758\ud558\uc5ec \uc9c0\uc5b4\uc84c\ub294\ub370 \ub3c5\ud2b9\ud558\uac8c\ub3c4 \uad6c\uc870\ubb3c\uc774 \ub2e4\ub9ac\ub97c \ub36e\uace0 \uc788\uace0, \ub2e4\ub9ac\uc758 \ud55c\ucabd\uc73c\ub85c\ub294 \uc0ac\ucc30\uc774 \uc5f0\uacb0\ub418\ub294 \uad6c\uc870\ubb3c\uc744 \uac00\uc9c4 \ub2e4\ub9ac\uc774\ub2e4.", "paragraph_answer": "\ud638\uc774\uc548\uc758 \ucd08\uae30 \uc5ed\uc0ac\ub294 \ucc38 \uc871\uc758 \uc5ed\uc0ac\uc774\ub2e4. \ub9d0\ub808\uc774\uacc4\uc778 \ucc38 \uc871\uc740 \uae30\uc6d0\uc804 200\ub144\uacbd\uacfc \uae30\uc6d0\ud6c4 200\ub144 \uacbd\uc5d0 \uc790\ubc14\uc5d0\uc11c \uc774 \uacf3\uc73c\ub85c \uac74\ub108\uc654\ub2e4. \uc774\ub4e4\uc740 \ucc38\ud30c \uc655\uad6d\uc744 \uc774\ub8e8\uc5c8\uace0 \ud6fc\uc640 \ub0d0\uc9f1\uae4c\uc9c0 \uc774 \uc655\uad6d\uc758 \uc138\ub825\ud558\uc5d0 \uc788\uc5c8\ub2e4. 1\uc138\uae30 \uacbd\uc5d0\ub294 \ub3d9\ub0a8\uc544\uc2dc\uc544\uc5d0\uc11c \uac00\uc7a5 \uaddc\ubaa8\uac00 \ud070 \ud56d\uad6c\uac00 \uc5ec\uae30\uc5d0 \uc788\uc5c8\uace0, \ub78c\uc555\ud3ec (L\u00e2m \u1ea4p Ph\u1ed1)\ub77c\uace0 \uc54c\ub824\uc838 \uc788\ub2e4. \ud22c\ubcf8(Thu Bon)\uac15\uc758 \uc5b4\uadc0\uc5d0\ub294 \ucc38\ud30c \uc0ac\ub78c\ub4e4\uc758 \uc608\uc804 \ud56d\uad6c\uac00 \uc788\uc5c8\ub2e4. \uc774 \ud56d\uad6c\ub294 \uc9c0\ub9ac\uc801 \uc5ec\uac74\uc73c\ub85c \uc778\ud558\uc5ec 16\uc138\uae30 \ub9d0\ubd80\ud130 17\uc138\uae30 \ucd08\uae4c\uc9c0(\ub113\uac8c \ubcf4\uba74 15\uc138\uae30\ubd80\ud130 19\uc138\uae30 \ubb34\ub835\uae4c\uc9c0) \ubca0\ud2b8\ub0a8\uc758 \"\ubc14\ub2e4\uc758 \uc2e4\ud06c\ub85c\ub4dc\"\ub77c\uace0 \ubd88\ub9ac\ub358 \uc911\uc694\ud55c \uad6d\uc81c \ubb34\uc5ed \ud56d\uad6c\uc600\uace0, \uc5ec\ub7ec \uc131(\u7701) \ucd9c\uc2e0\uc758 \ud654\uad50\uc640 \uc77c\ubcf8\uc778, \ub124\ub35c\ub780\ub4dc\uc778 \ub4f1 \uc11c\uad6c \uc0c1\uc778 \uadf8\ub9ac\uace0 \uc778\ub3c4\uc778\ub4e4\uc774 \ub4dc\ub098\ub4e4\uc5c8\uace0 \ub9c8\uc744\uc744 \ud615\uc131\ud558\uc5ec \uc815\ucc29\ud558\uc600\ub358 \uacf3\uc774\ub2e4. \uadf8\ub798\uc11c \uc774 \ub9c8\uc744\uc5d0\ub294 \uc11c\uad6c\uc801\uc774\uba74\uc11c\ub3c4 \ub3d9\uc591\uc801\uc778 \ud48d\uacbd\uc774 \uc790\ub9ac\uc7a1\uae30 \uc2dc\uc791\ud588\ub2e4. \uac70\uae30\uc5d0 \ud654\uad50\ub4e4\uc774 \ud638\uc774\uc548\uc5d0 \uc815\ucc29\ud574 \uc0b4\uae30 \uc2dc\uc791\ud558\uba74\uc11c '\ub3d9\ud654\uc801'\uc778 \uc0c9\ub2e4\ub978 \ubd84\uc704\uae30\uac00 \ud615\uc131\ub410\ub2e4. \ud654\uad50\ub97c \uc911\uc2ec\uc73c\ub85c \ud55c \ubb34\uc5ed\uc774 \ubc88\uc131\ud558\ub358 \ub2f9\uc2dc\uc5d0 \uc774 \ub9c8\uc744\uc740 \ubca0\ud2b8\ub0a8\uc5b4\ub85c \ud558\uc774\ud3ec(Hai Pho)\ub77c\uace0 \ubd88\ub800\uace0 \uc774\ub294 \"\ubc14\ub2f7\uac00 \ub9c8\uc744\"\uc774\ub77c\ub294 \ub73b\uc774\uc5c8\ub2e4. \uc6d0\ub798 \ud558\uc774\ud3ec\ub294 \"\uc77c\ubcf8 \ub2e4\ub9ac\"\ub97c \uac00\ub85c\uc9c8\ub7ec \ub9c8\uc744\uc774 \ub098\ub258\uc5c8\ub2e4. \ud558\ub098\ub294 \uc77c\ubcf8\uc778\uc774 \uac70\uc8fc\ud558\ub358 \uacf3\uc774\uc5c8\ub2e4. 16~17\uc138\uae30, \ubb34\uc5ed\uc774 \ubc88\uc131\ud588\uc744 \ub2f9\uc2dc \ud638\uc774\uc548\uc5d0\ub294 \uc77c\ubcf8\uc778\ub4e4\uc774 \ud2b9\ud788 \ub9ce\uc774 \ub4dc\ub098\ub4e4\uc5c8\uace0 \uadf8\ub798\uc11c \uc77c\ubcf8\uc778 \ub9c8\uc744\uae4c\uc9c0 \ub530\ub85c \uc0dd\uaca8\ub0ac\ub2e4. \uc804\uc131\uae30\uc5d0\ub294 1000\uba85\uc774 \ub118\ub294 \uc77c\ubcf8\uc778\uc774 \uac70\uc8fc\ud588\uc744 \uc815\ub3c4\ub85c \ub9c8\uc744 \uaddc\ubaa8\uac00 \ucee4\uc84c\ub2e4. \ud558\uc9c0\ub9cc \uc5d0\ub3c4 \uc2dc\ub300 \uc1c4\uad6d\uc815\ucc45\uc774 \uc2dc\ud589\ub418\uba74\uc11c \uc77c\ubcf8\uc778 \uc218\uac00 \uc810\ucc28 \uc904\uc5b4\ub4e4\uc5c8\uace0 \uc77c\ubcf8\uc778 \ub9c8\uc744 \uc5ed\uc2dc \uc790\uc5f0\uc2a4\ub7fd\uac8c \uc0ac\ub77c\uc84c\ub2e4. \uadf8 \uc77c\ubcf8 \ub2e4\ub9ac(Ch\u00f9a c\u1ea7u)\ub294 \uc77c\ubcf8\uc778\uc5d0 \uc758\ud558\uc5ec \uc9c0\uc5b4\uc84c\ub294\ub370 \ub3c5\ud2b9\ud558\uac8c\ub3c4 \uad6c\uc870\ubb3c\uc774 \ub2e4\ub9ac\ub97c \ub36e\uace0 \uc788\uace0, \ub2e4\ub9ac\uc758 \ud55c\ucabd\uc73c\ub85c\ub294 \uc0ac\ucc30\uc774 \uc5f0\uacb0\ub418\ub294 \uad6c\uc870\ubb3c\uc744 \uac00\uc9c4 \ub2e4\ub9ac\uc774\ub2e4.", "sentence_answer": "1\uc138\uae30 \uacbd\uc5d0\ub294 \ub3d9\ub0a8\uc544\uc2dc\uc544\uc5d0\uc11c \uac00\uc7a5 \uaddc\ubaa8\uac00 \ud070 \ud56d\uad6c\uac00 \uc5ec\uae30\uc5d0 \uc788\uc5c8\uace0, \ub78c\uc555\ud3ec (L\u00e2m \u1ea4p Ph\u1ed1)", "paragraph_id": "6571906-0-0", "paragraph_question": "question: 1\uc138\uae30 \uacbd \ub3d9\ub0a8\uc544\uc2dc\uc544\uc5d0\uc11c \uac00\uc7a5 \ud070 \ud56d\uad6c\ub294 \ubb34\uc5c7\uc778\uac00?, context: \ud638\uc774\uc548\uc758 \ucd08\uae30 \uc5ed\uc0ac\ub294 \ucc38 \uc871\uc758 \uc5ed\uc0ac\uc774\ub2e4. \ub9d0\ub808\uc774\uacc4\uc778 \ucc38 \uc871\uc740 \uae30\uc6d0\uc804 200\ub144\uacbd\uacfc \uae30\uc6d0\ud6c4 200\ub144 \uacbd\uc5d0 \uc790\ubc14\uc5d0\uc11c \uc774 \uacf3\uc73c\ub85c \uac74\ub108\uc654\ub2e4. \uc774\ub4e4\uc740 \ucc38\ud30c \uc655\uad6d\uc744 \uc774\ub8e8\uc5c8\uace0 \ud6fc\uc640 \ub0d0\uc9f1\uae4c\uc9c0 \uc774 \uc655\uad6d\uc758 \uc138\ub825\ud558\uc5d0 \uc788\uc5c8\ub2e4. 1\uc138\uae30 \uacbd\uc5d0\ub294 \ub3d9\ub0a8\uc544\uc2dc\uc544\uc5d0\uc11c \uac00\uc7a5 \uaddc\ubaa8\uac00 \ud070 \ud56d\uad6c\uac00 \uc5ec\uae30\uc5d0 \uc788\uc5c8\uace0, \ub78c\uc555\ud3ec(L\u00e2m \u1ea4p Ph\u1ed1)\ub77c\uace0 \uc54c\ub824\uc838 \uc788\ub2e4. \ud22c\ubcf8(Thu Bon)\uac15\uc758 \uc5b4\uadc0\uc5d0\ub294 \ucc38\ud30c \uc0ac\ub78c\ub4e4\uc758 \uc608\uc804 \ud56d\uad6c\uac00 \uc788\uc5c8\ub2e4. \uc774 \ud56d\uad6c\ub294 \uc9c0\ub9ac\uc801 \uc5ec\uac74\uc73c\ub85c \uc778\ud558\uc5ec 16\uc138\uae30 \ub9d0\ubd80\ud130 17\uc138\uae30 \ucd08\uae4c\uc9c0(\ub113\uac8c \ubcf4\uba74 15\uc138\uae30\ubd80\ud130 19\uc138\uae30 \ubb34\ub835\uae4c\uc9c0) \ubca0\ud2b8\ub0a8\uc758 \"\ubc14\ub2e4\uc758 \uc2e4\ud06c\ub85c\ub4dc\"\ub77c\uace0 \ubd88\ub9ac\ub358 \uc911\uc694\ud55c \uad6d\uc81c \ubb34\uc5ed \ud56d\uad6c\uc600\uace0, \uc5ec\ub7ec \uc131(\u7701) \ucd9c\uc2e0\uc758 \ud654\uad50\uc640 \uc77c\ubcf8\uc778, \ub124\ub35c\ub780\ub4dc\uc778 \ub4f1 \uc11c\uad6c \uc0c1\uc778 \uadf8\ub9ac\uace0 \uc778\ub3c4\uc778\ub4e4\uc774 \ub4dc\ub098\ub4e4\uc5c8\uace0 \ub9c8\uc744\uc744 \ud615\uc131\ud558\uc5ec \uc815\ucc29\ud558\uc600\ub358 \uacf3\uc774\ub2e4. \uadf8\ub798\uc11c \uc774 \ub9c8\uc744\uc5d0\ub294 \uc11c\uad6c\uc801\uc774\uba74\uc11c\ub3c4 \ub3d9\uc591\uc801\uc778 \ud48d\uacbd\uc774 \uc790\ub9ac\uc7a1\uae30 \uc2dc\uc791\ud588\ub2e4. \uac70\uae30\uc5d0 \ud654\uad50\ub4e4\uc774 \ud638\uc774\uc548\uc5d0 \uc815\ucc29\ud574 \uc0b4\uae30 \uc2dc\uc791\ud558\uba74\uc11c '\ub3d9\ud654\uc801'\uc778 \uc0c9\ub2e4\ub978 \ubd84\uc704\uae30\uac00 \ud615\uc131\ub410\ub2e4. \ud654\uad50\ub97c \uc911\uc2ec\uc73c\ub85c \ud55c \ubb34\uc5ed\uc774 \ubc88\uc131\ud558\ub358 \ub2f9\uc2dc\uc5d0 \uc774 \ub9c8\uc744\uc740 \ubca0\ud2b8\ub0a8\uc5b4\ub85c \ud558\uc774\ud3ec(Hai Pho)\ub77c\uace0 \ubd88\ub800\uace0 \uc774\ub294 \"\ubc14\ub2f7\uac00 \ub9c8\uc744\"\uc774\ub77c\ub294 \ub73b\uc774\uc5c8\ub2e4. \uc6d0\ub798 \ud558\uc774\ud3ec\ub294 \"\uc77c\ubcf8 \ub2e4\ub9ac\"\ub97c \uac00\ub85c\uc9c8\ub7ec \ub9c8\uc744\uc774 \ub098\ub258\uc5c8\ub2e4. \ud558\ub098\ub294 \uc77c\ubcf8\uc778\uc774 \uac70\uc8fc\ud558\ub358 \uacf3\uc774\uc5c8\ub2e4. 16~17\uc138\uae30, \ubb34\uc5ed\uc774 \ubc88\uc131\ud588\uc744 \ub2f9\uc2dc \ud638\uc774\uc548\uc5d0\ub294 \uc77c\ubcf8\uc778\ub4e4\uc774 \ud2b9\ud788 \ub9ce\uc774 \ub4dc\ub098\ub4e4\uc5c8\uace0 \uadf8\ub798\uc11c \uc77c\ubcf8\uc778 \ub9c8\uc744\uae4c\uc9c0 \ub530\ub85c \uc0dd\uaca8\ub0ac\ub2e4. \uc804\uc131\uae30\uc5d0\ub294 1000\uba85\uc774 \ub118\ub294 \uc77c\ubcf8\uc778\uc774 \uac70\uc8fc\ud588\uc744 \uc815\ub3c4\ub85c \ub9c8\uc744 \uaddc\ubaa8\uac00 \ucee4\uc84c\ub2e4. \ud558\uc9c0\ub9cc \uc5d0\ub3c4 \uc2dc\ub300 \uc1c4\uad6d\uc815\ucc45\uc774 \uc2dc\ud589\ub418\uba74\uc11c \uc77c\ubcf8\uc778 \uc218\uac00 \uc810\ucc28 \uc904\uc5b4\ub4e4\uc5c8\uace0 \uc77c\ubcf8\uc778 \ub9c8\uc744 \uc5ed\uc2dc \uc790\uc5f0\uc2a4\ub7fd\uac8c \uc0ac\ub77c\uc84c\ub2e4. \uadf8 \uc77c\ubcf8 \ub2e4\ub9ac(Ch\u00f9a c\u1ea7u)\ub294 \uc77c\ubcf8\uc778\uc5d0 \uc758\ud558\uc5ec \uc9c0\uc5b4\uc84c\ub294\ub370 \ub3c5\ud2b9\ud558\uac8c\ub3c4 \uad6c\uc870\ubb3c\uc774 \ub2e4\ub9ac\ub97c \ub36e\uace0 \uc788\uace0, \ub2e4\ub9ac\uc758 \ud55c\ucabd\uc73c\ub85c\ub294 \uc0ac\ucc30\uc774 \uc5f0\uacb0\ub418\ub294 \uad6c\uc870\ubb3c\uc744 \uac00\uc9c4 \ub2e4\ub9ac\uc774\ub2e4."} {"question": "\uc804\ud55c\uc744 \ucc3d\uac74\ud55c \uc655\uc758 \uc774\ub984\uc740?", "paragraph": "\ud6c4\ub300\uc5d0 \uc774\ub974\uba74\uc11c \uc870(\u7956)\ub97c \uc0ac\uc6a9\ud558\ub294 \ubb18\ud638\uc758 \uc218\ub3c4 \uc810\uc810 \ub298\uc5b4\ub098\uae30 \uc2dc\uc791\ud588\ub2e4. \uc911\uad6d\uc758 \uacbd\uc6b0, \u2018\uc870\u2019\ub294 \ub300\uac1c \uac1c\uad6d \uad70\uc8fc\uc758 \uc870\uc0c1\uc5d0\uac8c\ub9cc \uc62c\ub838\uace0 \uc2e4\uc81c\ub85c \uc7ac\uc704\ud55c \uad70\uc8fc\uc5d0\uac8c \uc8fc\ub294 \uc0ac\ub840\ub294 \uac74\uad6d \uad70\uc8fc\ub098 \ud1b5\uc77c\uac19\uc740 \uac74\uad6d\uacfc \ub9de\uba39\ub294 \uacf5\uc744 \uc138\uc6b4 \uad70\uc8fc\uc5d0\uac8c\ub9cc \uc62c\ub838\ub2e4. \uadf8\ub9ac\ud558\uc5ec \uc804\ud55c\uc758 \uace0\uc81c\uc5d0 \uc774\uc5b4 \uc0c8\ub85c\uc774 \ud55c\ub098\ub77c\ub97c \ucc3d\uac74\ud55c \ud6c4\ud55c\uc758 \uad11\ubb34\uc81c\ub294 \uc138\uc870\uc758 \ubb18\ud638\ub97c \ubc1b\uc740 \uac83\uc774 \ub300\ud45c\uc801\uc778 \uc0ac\ub840 \uc911 \ud558\ub098\uc774\ub2e4. \ud6c4\ub300\uc5d0 \ub4e4\uc5b4\uc624\uba74\uc11c \uc911\uad6d\uc5d0\uc11c\ub3c4 \uc870\ub97c \uc790\uc8fc \uc62c\ub9ac\uae30 \uc2dc\uc791\ud588\ub294\ub370, \uac00\uc815\uc81c\ub294 \uc601\ub77d\uc81c\uc758 \ubb18\ud638\uc778 \ud0dc\uc885(\u592a\u5b97)\uc744 \uace0\uccd0 \uc131\uc870(\u6210\u7956)\ub85c \ub192\uc600\uace0, \uc911\uad6d\uc758 \ub9c8\uc9c0\ub9c9 \ubd09\uac74 \uc655\uc870\uc778 \uccad\ub098\ub77c\uc758 \uacbd\uc6b0, \uac74\uad6d \uad70\uc8fc\uc778 \ucc9c\uba85\uc81c(\ud0dc\uc870)\ub97c \uc81c\uc678\ud55c \u2018\uc870\u2019\ub97c \ubc1b\uc740 \uad70\uc8fc\ub294 \uba85\ub098\ub77c\ub97c \uba78\ub9dd\uc2dc\ud0a4\uace0 \ub300\ub959\uc744 \ucc28\uc9c0\ud55c \uc21c\uce58\uc81c(\uc138\uc870)\uc640 \ub0b4\uc6b0\uc678\ud658\uc744 \ubb3c\ub9ac\uce58\uace0 \ud0dc\ud3c9\uc131\ub300\ub97c \uad6c\uac00\ud55c \uac15\ud76c\uc81c(\uc131\uc870) \ub2e8 \ub450 \uba85\ubfd0\uc774\uc5c8\ub2e4.", "answer": "\uace0\uc81c", "sentence": "\uadf8\ub9ac\ud558\uc5ec \uc804\ud55c\uc758 \uace0\uc81c \uc5d0", "paragraph_sentence": "\ud6c4\ub300\uc5d0 \uc774\ub974\uba74\uc11c \uc870(\u7956)\ub97c \uc0ac\uc6a9\ud558\ub294 \ubb18\ud638\uc758 \uc218\ub3c4 \uc810\uc810 \ub298\uc5b4\ub098\uae30 \uc2dc\uc791\ud588\ub2e4. \uc911\uad6d\uc758 \uacbd\uc6b0, \u2018\uc870\u2019\ub294 \ub300\uac1c \uac1c\uad6d \uad70\uc8fc\uc758 \uc870\uc0c1\uc5d0\uac8c\ub9cc \uc62c\ub838\uace0 \uc2e4\uc81c\ub85c \uc7ac\uc704\ud55c \uad70\uc8fc\uc5d0\uac8c \uc8fc\ub294 \uc0ac\ub840\ub294 \uac74\uad6d \uad70\uc8fc\ub098 \ud1b5\uc77c\uac19\uc740 \uac74\uad6d\uacfc \ub9de\uba39\ub294 \uacf5\uc744 \uc138\uc6b4 \uad70\uc8fc\uc5d0\uac8c\ub9cc \uc62c\ub838\ub2e4. \uadf8\ub9ac\ud558\uc5ec \uc804\ud55c\uc758 \uace0\uc81c \uc5d0 \uc774\uc5b4 \uc0c8\ub85c\uc774 \ud55c\ub098\ub77c\ub97c \ucc3d\uac74\ud55c \ud6c4\ud55c\uc758 \uad11\ubb34\uc81c\ub294 \uc138\uc870\uc758 \ubb18\ud638\ub97c \ubc1b\uc740 \uac83\uc774 \ub300\ud45c\uc801\uc778 \uc0ac\ub840 \uc911 \ud558\ub098\uc774\ub2e4. \ud6c4\ub300\uc5d0 \ub4e4\uc5b4\uc624\uba74\uc11c \uc911\uad6d\uc5d0\uc11c\ub3c4 \uc870\ub97c \uc790\uc8fc \uc62c\ub9ac\uae30 \uc2dc\uc791\ud588\ub294\ub370, \uac00\uc815\uc81c\ub294 \uc601\ub77d\uc81c\uc758 \ubb18\ud638\uc778 \ud0dc\uc885(\u592a\u5b97)\uc744 \uace0\uccd0 \uc131\uc870(\u6210\u7956)\ub85c \ub192\uc600\uace0, \uc911\uad6d\uc758 \ub9c8\uc9c0\ub9c9 \ubd09\uac74 \uc655\uc870\uc778 \uccad\ub098\ub77c\uc758 \uacbd\uc6b0, \uac74\uad6d \uad70\uc8fc\uc778 \ucc9c\uba85\uc81c(\ud0dc\uc870)\ub97c \uc81c\uc678\ud55c \u2018\uc870\u2019\ub97c \ubc1b\uc740 \uad70\uc8fc\ub294 \uba85\ub098\ub77c\ub97c \uba78\ub9dd\uc2dc\ud0a4\uace0 \ub300\ub959\uc744 \ucc28\uc9c0\ud55c \uc21c\uce58\uc81c(\uc138\uc870)\uc640 \ub0b4\uc6b0\uc678\ud658\uc744 \ubb3c\ub9ac\uce58\uace0 \ud0dc\ud3c9\uc131\ub300\ub97c \uad6c\uac00\ud55c \uac15\ud76c\uc81c(\uc131\uc870) \ub2e8 \ub450 \uba85\ubfd0\uc774\uc5c8\ub2e4.", "paragraph_answer": "\ud6c4\ub300\uc5d0 \uc774\ub974\uba74\uc11c \uc870(\u7956)\ub97c \uc0ac\uc6a9\ud558\ub294 \ubb18\ud638\uc758 \uc218\ub3c4 \uc810\uc810 \ub298\uc5b4\ub098\uae30 \uc2dc\uc791\ud588\ub2e4. \uc911\uad6d\uc758 \uacbd\uc6b0, \u2018\uc870\u2019\ub294 \ub300\uac1c \uac1c\uad6d \uad70\uc8fc\uc758 \uc870\uc0c1\uc5d0\uac8c\ub9cc \uc62c\ub838\uace0 \uc2e4\uc81c\ub85c \uc7ac\uc704\ud55c \uad70\uc8fc\uc5d0\uac8c \uc8fc\ub294 \uc0ac\ub840\ub294 \uac74\uad6d \uad70\uc8fc\ub098 \ud1b5\uc77c\uac19\uc740 \uac74\uad6d\uacfc \ub9de\uba39\ub294 \uacf5\uc744 \uc138\uc6b4 \uad70\uc8fc\uc5d0\uac8c\ub9cc \uc62c\ub838\ub2e4. \uadf8\ub9ac\ud558\uc5ec \uc804\ud55c\uc758 \uace0\uc81c \uc5d0 \uc774\uc5b4 \uc0c8\ub85c\uc774 \ud55c\ub098\ub77c\ub97c \ucc3d\uac74\ud55c \ud6c4\ud55c\uc758 \uad11\ubb34\uc81c\ub294 \uc138\uc870\uc758 \ubb18\ud638\ub97c \ubc1b\uc740 \uac83\uc774 \ub300\ud45c\uc801\uc778 \uc0ac\ub840 \uc911 \ud558\ub098\uc774\ub2e4. \ud6c4\ub300\uc5d0 \ub4e4\uc5b4\uc624\uba74\uc11c \uc911\uad6d\uc5d0\uc11c\ub3c4 \uc870\ub97c \uc790\uc8fc \uc62c\ub9ac\uae30 \uc2dc\uc791\ud588\ub294\ub370, \uac00\uc815\uc81c\ub294 \uc601\ub77d\uc81c\uc758 \ubb18\ud638\uc778 \ud0dc\uc885(\u592a\u5b97)\uc744 \uace0\uccd0 \uc131\uc870(\u6210\u7956)\ub85c \ub192\uc600\uace0, \uc911\uad6d\uc758 \ub9c8\uc9c0\ub9c9 \ubd09\uac74 \uc655\uc870\uc778 \uccad\ub098\ub77c\uc758 \uacbd\uc6b0, \uac74\uad6d \uad70\uc8fc\uc778 \ucc9c\uba85\uc81c(\ud0dc\uc870)\ub97c \uc81c\uc678\ud55c \u2018\uc870\u2019\ub97c \ubc1b\uc740 \uad70\uc8fc\ub294 \uba85\ub098\ub77c\ub97c \uba78\ub9dd\uc2dc\ud0a4\uace0 \ub300\ub959\uc744 \ucc28\uc9c0\ud55c \uc21c\uce58\uc81c(\uc138\uc870)\uc640 \ub0b4\uc6b0\uc678\ud658\uc744 \ubb3c\ub9ac\uce58\uace0 \ud0dc\ud3c9\uc131\ub300\ub97c \uad6c\uac00\ud55c \uac15\ud76c\uc81c(\uc131\uc870) \ub2e8 \ub450 \uba85\ubfd0\uc774\uc5c8\ub2e4.", "sentence_answer": "\uadf8\ub9ac\ud558\uc5ec \uc804\ud55c\uc758 \uace0\uc81c \uc5d0", "paragraph_id": "6504691-16-0", "paragraph_question": "question: \uc804\ud55c\uc744 \ucc3d\uac74\ud55c \uc655\uc758 \uc774\ub984\uc740?, context: \ud6c4\ub300\uc5d0 \uc774\ub974\uba74\uc11c \uc870(\u7956)\ub97c \uc0ac\uc6a9\ud558\ub294 \ubb18\ud638\uc758 \uc218\ub3c4 \uc810\uc810 \ub298\uc5b4\ub098\uae30 \uc2dc\uc791\ud588\ub2e4. \uc911\uad6d\uc758 \uacbd\uc6b0, \u2018\uc870\u2019\ub294 \ub300\uac1c \uac1c\uad6d \uad70\uc8fc\uc758 \uc870\uc0c1\uc5d0\uac8c\ub9cc \uc62c\ub838\uace0 \uc2e4\uc81c\ub85c \uc7ac\uc704\ud55c \uad70\uc8fc\uc5d0\uac8c \uc8fc\ub294 \uc0ac\ub840\ub294 \uac74\uad6d \uad70\uc8fc\ub098 \ud1b5\uc77c\uac19\uc740 \uac74\uad6d\uacfc \ub9de\uba39\ub294 \uacf5\uc744 \uc138\uc6b4 \uad70\uc8fc\uc5d0\uac8c\ub9cc \uc62c\ub838\ub2e4. \uadf8\ub9ac\ud558\uc5ec \uc804\ud55c\uc758 \uace0\uc81c\uc5d0 \uc774\uc5b4 \uc0c8\ub85c\uc774 \ud55c\ub098\ub77c\ub97c \ucc3d\uac74\ud55c \ud6c4\ud55c\uc758 \uad11\ubb34\uc81c\ub294 \uc138\uc870\uc758 \ubb18\ud638\ub97c \ubc1b\uc740 \uac83\uc774 \ub300\ud45c\uc801\uc778 \uc0ac\ub840 \uc911 \ud558\ub098\uc774\ub2e4. \ud6c4\ub300\uc5d0 \ub4e4\uc5b4\uc624\uba74\uc11c \uc911\uad6d\uc5d0\uc11c\ub3c4 \uc870\ub97c \uc790\uc8fc \uc62c\ub9ac\uae30 \uc2dc\uc791\ud588\ub294\ub370, \uac00\uc815\uc81c\ub294 \uc601\ub77d\uc81c\uc758 \ubb18\ud638\uc778 \ud0dc\uc885(\u592a\u5b97)\uc744 \uace0\uccd0 \uc131\uc870(\u6210\u7956)\ub85c \ub192\uc600\uace0, \uc911\uad6d\uc758 \ub9c8\uc9c0\ub9c9 \ubd09\uac74 \uc655\uc870\uc778 \uccad\ub098\ub77c\uc758 \uacbd\uc6b0, \uac74\uad6d \uad70\uc8fc\uc778 \ucc9c\uba85\uc81c(\ud0dc\uc870)\ub97c \uc81c\uc678\ud55c \u2018\uc870\u2019\ub97c \ubc1b\uc740 \uad70\uc8fc\ub294 \uba85\ub098\ub77c\ub97c \uba78\ub9dd\uc2dc\ud0a4\uace0 \ub300\ub959\uc744 \ucc28\uc9c0\ud55c \uc21c\uce58\uc81c(\uc138\uc870)\uc640 \ub0b4\uc6b0\uc678\ud658\uc744 \ubb3c\ub9ac\uce58\uace0 \ud0dc\ud3c9\uc131\ub300\ub97c \uad6c\uac00\ud55c \uac15\ud76c\uc81c(\uc131\uc870) \ub2e8 \ub450 \uba85\ubfd0\uc774\uc5c8\ub2e4."} {"question": "\ud574\uc801\ub4e4\uc774 \uc18c\ub9d0\ub9ac\uc544\ub85c \uadc0\ud658\ud558\ub294 \uacfc\uc815\uc5d0\uc11c \uc774\ub8e8\uc5b4\uc9c4 1\ucc28 \uad6c\ucd9c \uc791\uc804\uc740 \uc5b8\uc81c \uc9c4\ud589\ub418\uc5c8\ub294\uac00?", "paragraph": "1\ucc28 \uad6c\ucd9c \uc791\uc804\uc740 2011\ub144 1\uc6d4 18\uc77c 20\uc2dc 9\ubd84(KST)\uc5d0 \ud574\uc801\ub4e4\uc774 \uc18c\ub9d0\ub9ac\uc544\ub85c \uadc0\ud658\ud558\ub294 \uacfc\uc815\uc5d0\uc11c \uc774\ub8e8\uc5b4\uc84c\ub2e4. \ucd5c\uc601\ud568\uc774 \ud574\uc801\ub4e4\uc744 \ub4a4\ucad3\ub294 \ub3c4\uc911 \ud574\uc801\ub4e4\uc774 \ubabd\uace8 \uc120\ubc15\uc744 \ucd94\uac00\ub85c \ud53c\ub78d\ud558\uae30 \uc704\ud574 \uc790\uc120(\u5b50\u8239)\uc744 \ub0b4\ub9ac\ub294\uac83\uc744 \ud3ec\ucc29\ud558\uace0, \uc774 \ub54c \ud574\uc801\ub4e4\uc774 \ubd84\uc5f4\ub41c \ud2c8\uc744 \ud0c0 \uc791\uc804\uc744 \uc2dc\ud589\ud588\ub2e4. \ub9c1\ud06c\uc2a4 \ud5ec\uae30\ub85c \uc790\uc120\uc5d0 \uc704\ud611\uc0ac\uaca9\uc744 \uac00\ud55c \ub4a4 \uad00\uc2ec\uc774 \uc3e0\ub9b0 \uc0ac\uc774 \uccad\ud574\ubd80\ub300 \uc18c\uc18d \ud574\uad70\ud2b9\uc218\uc804\uc5ec\ub2e8 \uc694\uc6d0\ub4e4\uc774 \uace0\uc18d\uc815\uc73c\ub85c \uc0bc\ud638 \uc8fc\uc5bc\ub9ac\ud638\uc5d0 \uc811\uadfc\ud588\uace0 \ud574\uc801\uacfc\uc758 \uad50\uc804\uc774 \ubc8c\uc5b4\uc84c\ub2e4. \uadf8\ub7ec\ub098 \uc774\uac83\uc740 \uc18d\uc784\uc218 \uc791\uc804\uc73c\ub85c \uc801\ub4e4\uc740 \ud770\uae30\ub97c \ub4e4\uace0 \ud56d\ubcf5 \ud558\ub294 \ucc99 \ud558\ub2e4\uac00 \ubc14\ub85c AK \uc18c\ucd1d\uc744 \ubc1c\ud3ec\ud558\uc600\uace0 \uc774 \uacfc\uc815\uc5d0\uc11c \ud2b9\uc218\ubd80\ub300\uc6d0 3\uba85\uc774 \ucd1d\uc0c1\uacfc \ucc30\uacfc\uc0c1 \ub4f1\uc758 \ubd80\uc0c1\uc744 \uc785\uace0 \uc791\uc804\uc744 \uc911\uc9c0, \ucd5c\uc601\ud568\uc73c\ub85c \ubcf5\uadc0\ud588\uc73c\uba70 \ud574\uc801\uc758 \uc790\uc120\uc5d0\uc11c AK-47 3\uc815\uacfc \uc0ac\ub2e4\ub9ac, \ud574\uc801 \uc790\uc120(\u5b50\u8239)\uc744 \ud655\ubcf4\ud588\ub2e4. \ub610\ud55c \uacbd\uace0 \uc0ac\uaca9 \uacfc\uc815\uc5d0\uc11c \ud574\uc801 \uc218 \uba85\uc774 \ubc14\ub2e4\uc5d0 \ube60\uc838 \uc2e4\uc885\ub418\uc5c8\ub2e4.", "answer": "2011\ub144 1\uc6d4 18\uc77c", "sentence": "1\ucc28 \uad6c\ucd9c \uc791\uc804\uc740 2011\ub144 1\uc6d4 18\uc77c 20\uc2dc 9\ubd84(KST)\uc5d0 \ud574\uc801\ub4e4\uc774 \uc18c\ub9d0\ub9ac\uc544\ub85c \uadc0\ud658\ud558\ub294 \uacfc\uc815\uc5d0\uc11c \uc774\ub8e8\uc5b4\uc84c\ub2e4.", "paragraph_sentence": " 1\ucc28 \uad6c\ucd9c \uc791\uc804\uc740 2011\ub144 1\uc6d4 18\uc77c 20\uc2dc 9\ubd84(KST)\uc5d0 \ud574\uc801\ub4e4\uc774 \uc18c\ub9d0\ub9ac\uc544\ub85c \uadc0\ud658\ud558\ub294 \uacfc\uc815\uc5d0\uc11c \uc774\ub8e8\uc5b4\uc84c\ub2e4. \ucd5c\uc601\ud568\uc774 \ud574\uc801\ub4e4\uc744 \ub4a4\ucad3\ub294 \ub3c4\uc911 \ud574\uc801\ub4e4\uc774 \ubabd\uace8 \uc120\ubc15\uc744 \ucd94\uac00\ub85c \ud53c\ub78d\ud558\uae30 \uc704\ud574 \uc790\uc120(\u5b50\u8239)\uc744 \ub0b4\ub9ac\ub294\uac83\uc744 \ud3ec\ucc29\ud558\uace0, \uc774 \ub54c \ud574\uc801\ub4e4\uc774 \ubd84\uc5f4\ub41c \ud2c8\uc744 \ud0c0 \uc791\uc804\uc744 \uc2dc\ud589\ud588\ub2e4. \ub9c1\ud06c\uc2a4 \ud5ec\uae30\ub85c \uc790\uc120\uc5d0 \uc704\ud611\uc0ac\uaca9\uc744 \uac00\ud55c \ub4a4 \uad00\uc2ec\uc774 \uc3e0\ub9b0 \uc0ac\uc774 \uccad\ud574\ubd80\ub300 \uc18c\uc18d \ud574\uad70\ud2b9\uc218\uc804\uc5ec\ub2e8 \uc694\uc6d0\ub4e4\uc774 \uace0\uc18d\uc815\uc73c\ub85c \uc0bc\ud638 \uc8fc\uc5bc\ub9ac\ud638\uc5d0 \uc811\uadfc\ud588\uace0 \ud574\uc801\uacfc\uc758 \uad50\uc804\uc774 \ubc8c\uc5b4\uc84c\ub2e4. \uadf8\ub7ec\ub098 \uc774\uac83\uc740 \uc18d\uc784\uc218 \uc791\uc804\uc73c\ub85c \uc801\ub4e4\uc740 \ud770\uae30\ub97c \ub4e4\uace0 \ud56d\ubcf5 \ud558\ub294 \ucc99 \ud558\ub2e4\uac00 \ubc14\ub85c AK \uc18c\ucd1d\uc744 \ubc1c\ud3ec\ud558\uc600\uace0 \uc774 \uacfc\uc815\uc5d0\uc11c \ud2b9\uc218\ubd80\ub300\uc6d0 3\uba85\uc774 \ucd1d\uc0c1\uacfc \ucc30\uacfc\uc0c1 \ub4f1\uc758 \ubd80\uc0c1\uc744 \uc785\uace0 \uc791\uc804\uc744 \uc911\uc9c0, \ucd5c\uc601\ud568\uc73c\ub85c \ubcf5\uadc0\ud588\uc73c\uba70 \ud574\uc801\uc758 \uc790\uc120\uc5d0\uc11c AK-47 3\uc815\uacfc \uc0ac\ub2e4\ub9ac, \ud574\uc801 \uc790\uc120(\u5b50\u8239)\uc744 \ud655\ubcf4\ud588\ub2e4. \ub610\ud55c \uacbd\uace0 \uc0ac\uaca9 \uacfc\uc815\uc5d0\uc11c \ud574\uc801 \uc218 \uba85\uc774 \ubc14\ub2e4\uc5d0 \ube60\uc838 \uc2e4\uc885\ub418\uc5c8\ub2e4.", "paragraph_answer": "1\ucc28 \uad6c\ucd9c \uc791\uc804\uc740 2011\ub144 1\uc6d4 18\uc77c 20\uc2dc 9\ubd84(KST)\uc5d0 \ud574\uc801\ub4e4\uc774 \uc18c\ub9d0\ub9ac\uc544\ub85c \uadc0\ud658\ud558\ub294 \uacfc\uc815\uc5d0\uc11c \uc774\ub8e8\uc5b4\uc84c\ub2e4. \ucd5c\uc601\ud568\uc774 \ud574\uc801\ub4e4\uc744 \ub4a4\ucad3\ub294 \ub3c4\uc911 \ud574\uc801\ub4e4\uc774 \ubabd\uace8 \uc120\ubc15\uc744 \ucd94\uac00\ub85c \ud53c\ub78d\ud558\uae30 \uc704\ud574 \uc790\uc120(\u5b50\u8239)\uc744 \ub0b4\ub9ac\ub294\uac83\uc744 \ud3ec\ucc29\ud558\uace0, \uc774 \ub54c \ud574\uc801\ub4e4\uc774 \ubd84\uc5f4\ub41c \ud2c8\uc744 \ud0c0 \uc791\uc804\uc744 \uc2dc\ud589\ud588\ub2e4. \ub9c1\ud06c\uc2a4 \ud5ec\uae30\ub85c \uc790\uc120\uc5d0 \uc704\ud611\uc0ac\uaca9\uc744 \uac00\ud55c \ub4a4 \uad00\uc2ec\uc774 \uc3e0\ub9b0 \uc0ac\uc774 \uccad\ud574\ubd80\ub300 \uc18c\uc18d \ud574\uad70\ud2b9\uc218\uc804\uc5ec\ub2e8 \uc694\uc6d0\ub4e4\uc774 \uace0\uc18d\uc815\uc73c\ub85c \uc0bc\ud638 \uc8fc\uc5bc\ub9ac\ud638\uc5d0 \uc811\uadfc\ud588\uace0 \ud574\uc801\uacfc\uc758 \uad50\uc804\uc774 \ubc8c\uc5b4\uc84c\ub2e4. \uadf8\ub7ec\ub098 \uc774\uac83\uc740 \uc18d\uc784\uc218 \uc791\uc804\uc73c\ub85c \uc801\ub4e4\uc740 \ud770\uae30\ub97c \ub4e4\uace0 \ud56d\ubcf5 \ud558\ub294 \ucc99 \ud558\ub2e4\uac00 \ubc14\ub85c AK \uc18c\ucd1d\uc744 \ubc1c\ud3ec\ud558\uc600\uace0 \uc774 \uacfc\uc815\uc5d0\uc11c \ud2b9\uc218\ubd80\ub300\uc6d0 3\uba85\uc774 \ucd1d\uc0c1\uacfc \ucc30\uacfc\uc0c1 \ub4f1\uc758 \ubd80\uc0c1\uc744 \uc785\uace0 \uc791\uc804\uc744 \uc911\uc9c0, \ucd5c\uc601\ud568\uc73c\ub85c \ubcf5\uadc0\ud588\uc73c\uba70 \ud574\uc801\uc758 \uc790\uc120\uc5d0\uc11c AK-47 3\uc815\uacfc \uc0ac\ub2e4\ub9ac, \ud574\uc801 \uc790\uc120(\u5b50\u8239)\uc744 \ud655\ubcf4\ud588\ub2e4. \ub610\ud55c \uacbd\uace0 \uc0ac\uaca9 \uacfc\uc815\uc5d0\uc11c \ud574\uc801 \uc218 \uba85\uc774 \ubc14\ub2e4\uc5d0 \ube60\uc838 \uc2e4\uc885\ub418\uc5c8\ub2e4.", "sentence_answer": "1\ucc28 \uad6c\ucd9c \uc791\uc804\uc740 2011\ub144 1\uc6d4 18\uc77c 20\uc2dc 9\ubd84(KST)\uc5d0 \ud574\uc801\ub4e4\uc774 \uc18c\ub9d0\ub9ac\uc544\ub85c \uadc0\ud658\ud558\ub294 \uacfc\uc815\uc5d0\uc11c \uc774\ub8e8\uc5b4\uc84c\ub2e4.", "paragraph_id": "6520267-0-0", "paragraph_question": "question: \ud574\uc801\ub4e4\uc774 \uc18c\ub9d0\ub9ac\uc544\ub85c \uadc0\ud658\ud558\ub294 \uacfc\uc815\uc5d0\uc11c \uc774\ub8e8\uc5b4\uc9c4 1\ucc28 \uad6c\ucd9c \uc791\uc804\uc740 \uc5b8\uc81c \uc9c4\ud589\ub418\uc5c8\ub294\uac00?, context: 1\ucc28 \uad6c\ucd9c \uc791\uc804\uc740 2011\ub144 1\uc6d4 18\uc77c 20\uc2dc 9\ubd84(KST)\uc5d0 \ud574\uc801\ub4e4\uc774 \uc18c\ub9d0\ub9ac\uc544\ub85c \uadc0\ud658\ud558\ub294 \uacfc\uc815\uc5d0\uc11c \uc774\ub8e8\uc5b4\uc84c\ub2e4. \ucd5c\uc601\ud568\uc774 \ud574\uc801\ub4e4\uc744 \ub4a4\ucad3\ub294 \ub3c4\uc911 \ud574\uc801\ub4e4\uc774 \ubabd\uace8 \uc120\ubc15\uc744 \ucd94\uac00\ub85c \ud53c\ub78d\ud558\uae30 \uc704\ud574 \uc790\uc120(\u5b50\u8239)\uc744 \ub0b4\ub9ac\ub294\uac83\uc744 \ud3ec\ucc29\ud558\uace0, \uc774 \ub54c \ud574\uc801\ub4e4\uc774 \ubd84\uc5f4\ub41c \ud2c8\uc744 \ud0c0 \uc791\uc804\uc744 \uc2dc\ud589\ud588\ub2e4. \ub9c1\ud06c\uc2a4 \ud5ec\uae30\ub85c \uc790\uc120\uc5d0 \uc704\ud611\uc0ac\uaca9\uc744 \uac00\ud55c \ub4a4 \uad00\uc2ec\uc774 \uc3e0\ub9b0 \uc0ac\uc774 \uccad\ud574\ubd80\ub300 \uc18c\uc18d \ud574\uad70\ud2b9\uc218\uc804\uc5ec\ub2e8 \uc694\uc6d0\ub4e4\uc774 \uace0\uc18d\uc815\uc73c\ub85c \uc0bc\ud638 \uc8fc\uc5bc\ub9ac\ud638\uc5d0 \uc811\uadfc\ud588\uace0 \ud574\uc801\uacfc\uc758 \uad50\uc804\uc774 \ubc8c\uc5b4\uc84c\ub2e4. \uadf8\ub7ec\ub098 \uc774\uac83\uc740 \uc18d\uc784\uc218 \uc791\uc804\uc73c\ub85c \uc801\ub4e4\uc740 \ud770\uae30\ub97c \ub4e4\uace0 \ud56d\ubcf5 \ud558\ub294 \ucc99 \ud558\ub2e4\uac00 \ubc14\ub85c AK \uc18c\ucd1d\uc744 \ubc1c\ud3ec\ud558\uc600\uace0 \uc774 \uacfc\uc815\uc5d0\uc11c \ud2b9\uc218\ubd80\ub300\uc6d0 3\uba85\uc774 \ucd1d\uc0c1\uacfc \ucc30\uacfc\uc0c1 \ub4f1\uc758 \ubd80\uc0c1\uc744 \uc785\uace0 \uc791\uc804\uc744 \uc911\uc9c0, \ucd5c\uc601\ud568\uc73c\ub85c \ubcf5\uadc0\ud588\uc73c\uba70 \ud574\uc801\uc758 \uc790\uc120\uc5d0\uc11c AK-47 3\uc815\uacfc \uc0ac\ub2e4\ub9ac, \ud574\uc801 \uc790\uc120(\u5b50\u8239)\uc744 \ud655\ubcf4\ud588\ub2e4. \ub610\ud55c \uacbd\uace0 \uc0ac\uaca9 \uacfc\uc815\uc5d0\uc11c \ud574\uc801 \uc218 \uba85\uc774 \ubc14\ub2e4\uc5d0 \ube60\uc838 \uc2e4\uc885\ub418\uc5c8\ub2e4."} {"question": "\ub178\ubb34\ud604 \ub300\ud1b5\ub839\uc774 \ucc98\uc74c \ub2f9\uc120\ub418 \uad6d\ud68c\uc5d0 \uc785\ubb38\ud558\uac8c\ub41c \ucd1d\uc120\uc740 \uc5b8\uc81c\uc778\uac00?", "paragraph": "\uc601\ud654\ub294 1988\ub144 \ub178\ubb34\ud604 \ubcc0\ud638\uc0ac\uac00 \uadf8\ud574 \uc81c13\ub300 \ucd1d\uc120\uc5d0\uc11c \ub2f9\uc120\ub418\uc5b4 \uad6d\ud68c\uc5d0 \uc785\ubb38\ud588\ub358 \ubaa8\uc2b5\ubd80\ud130 \uc2dc\uc791\ub41c\ub2e4. \uc774\ud6c4\ub85c\ub294 \ub178\ub798\ub97c \ucc3e\ub294 \uc0ac\ub78c\ub4e4\uc758 \u300a\uc0ac\uacc4\u300b\uac00 \ud750\ub974\ub294 \uac00\uc6b4\ub370, \ud0c0\uc784\ub77c\uc778\uc774 \ube68\ub77c\uc9c0\uba74\uc11c \ub178\ubb34\ud604\uc758 \uc815\uce58\uacbd\ub825\uc744 \ud6d1\uace0 \uc9c0\ub098\uac04\ub2e4. \ub178\ubb34\ud604 \uc758\uc6d0\uc740 1990\ub144 3\ub2f9 \ud569\ub2f9\uc744 \uacaa\uace0 \ub2e4\uc74c \ucd1d\uc120\uc778 1992\ub144 \uc81c14\ub300 \ucd1d\uc120\uc5d0\uc11c \ub099\uc120\ud55c\ub2e4. \uc774\ud6c4 1995\ub144 \uc81c1\ud68c \uc9c0\ubc29\uc120\uac70\uc5d0\uc11c \ubd80\uc0b0 \uc2dc\uc7a5 \uc120\uac70\uc5d0 \ub098\uc11c\ub098 \uc774 \uc5ed\uc2dc \ub099\uc120\ud558\uace0 \uc774\ub4ec\ud574 1996\ub144 \uc81c15\ub300 \ucd1d\uc120\uc5d0\uc11c \uc885\ub85c\uad6c\uc5d0 \ucd9c\ub9c8\ud558\ub098 \ub099\uc120\ud55c\ub2e4. \uadf8\ub7ec\ub098 1998\ub144 \uc7ac\ubcf4\uad90 \uc120\uac70\uc5d0\uc11c \uc885\ub85c\uad6c \uc120\uac70\uc5d0 \ub2e4\uc2dc \ucd9c\ub9c8\ud558\uc5ec \ub2f9\uc120\ub418\uace0, \"\ub2e4\uc2dc \uc77c\ud558\uac8c \ub420 \uc218 \uc788\uc5b4 \uae30\uc058\ub2e4\"\ub294 \uadf8\uc758 \ub2f9\uc120\uc18c\uac10\uacfc \ud568\uaed8 \ud0c0\uc784\ub77c\uc778\uc774 \uba48\ucd94\uba70 \uc601\ud654 \uc81c\ubaa9\uc774 \uc2a4\ud06c\ub9b0\uc5d0 \ub5a0\uc624\ub978\ub2e4.", "answer": "\uc81c13\ub300 \ucd1d\uc120", "sentence": "\uadf8\ud574 \uc81c13\ub300 \ucd1d\uc120 \uc5d0\uc11c \ub2f9\uc120\ub418\uc5b4 \uad6d\ud68c\uc5d0 \uc785\ubb38\ud588\ub358 \ubaa8\uc2b5\ubd80\ud130 \uc2dc\uc791\ub41c\ub2e4.", "paragraph_sentence": "\uc601\ud654\ub294 1988\ub144 \ub178\ubb34\ud604 \ubcc0\ud638\uc0ac\uac00 \uadf8\ud574 \uc81c13\ub300 \ucd1d\uc120 \uc5d0\uc11c \ub2f9\uc120\ub418\uc5b4 \uad6d\ud68c\uc5d0 \uc785\ubb38\ud588\ub358 \ubaa8\uc2b5\ubd80\ud130 \uc2dc\uc791\ub41c\ub2e4. \uc774\ud6c4\ub85c\ub294 \ub178\ub798\ub97c \ucc3e\ub294 \uc0ac\ub78c\ub4e4\uc758 \u300a\uc0ac\uacc4\u300b\uac00 \ud750\ub974\ub294 \uac00\uc6b4\ub370, \ud0c0\uc784\ub77c\uc778\uc774 \ube68\ub77c\uc9c0\uba74\uc11c \ub178\ubb34\ud604\uc758 \uc815\uce58\uacbd\ub825\uc744 \ud6d1\uace0 \uc9c0\ub098\uac04\ub2e4. \ub178\ubb34\ud604 \uc758\uc6d0\uc740 1990\ub144 3\ub2f9 \ud569\ub2f9\uc744 \uacaa\uace0 \ub2e4\uc74c \ucd1d\uc120\uc778 1992\ub144 \uc81c14\ub300 \ucd1d\uc120\uc5d0\uc11c \ub099\uc120\ud55c\ub2e4. \uc774\ud6c4 1995\ub144 \uc81c1\ud68c \uc9c0\ubc29\uc120\uac70\uc5d0\uc11c \ubd80\uc0b0 \uc2dc\uc7a5 \uc120\uac70\uc5d0 \ub098\uc11c\ub098 \uc774 \uc5ed\uc2dc \ub099\uc120\ud558\uace0 \uc774\ub4ec\ud574 1996\ub144 \uc81c15\ub300 \ucd1d\uc120\uc5d0\uc11c \uc885\ub85c\uad6c\uc5d0 \ucd9c\ub9c8\ud558\ub098 \ub099\uc120\ud55c\ub2e4. \uadf8\ub7ec\ub098 1998\ub144 \uc7ac\ubcf4\uad90 \uc120\uac70\uc5d0\uc11c \uc885\ub85c\uad6c \uc120\uac70\uc5d0 \ub2e4\uc2dc \ucd9c\ub9c8\ud558\uc5ec \ub2f9\uc120\ub418\uace0, \"\ub2e4\uc2dc \uc77c\ud558\uac8c \ub420 \uc218 \uc788\uc5b4 \uae30\uc058\ub2e4\"\ub294 \uadf8\uc758 \ub2f9\uc120\uc18c\uac10\uacfc \ud568\uaed8 \ud0c0\uc784\ub77c\uc778\uc774 \uba48\ucd94\uba70 \uc601\ud654 \uc81c\ubaa9\uc774 \uc2a4\ud06c\ub9b0\uc5d0 \ub5a0\uc624\ub978\ub2e4.", "paragraph_answer": "\uc601\ud654\ub294 1988\ub144 \ub178\ubb34\ud604 \ubcc0\ud638\uc0ac\uac00 \uadf8\ud574 \uc81c13\ub300 \ucd1d\uc120 \uc5d0\uc11c \ub2f9\uc120\ub418\uc5b4 \uad6d\ud68c\uc5d0 \uc785\ubb38\ud588\ub358 \ubaa8\uc2b5\ubd80\ud130 \uc2dc\uc791\ub41c\ub2e4. \uc774\ud6c4\ub85c\ub294 \ub178\ub798\ub97c \ucc3e\ub294 \uc0ac\ub78c\ub4e4\uc758 \u300a\uc0ac\uacc4\u300b\uac00 \ud750\ub974\ub294 \uac00\uc6b4\ub370, \ud0c0\uc784\ub77c\uc778\uc774 \ube68\ub77c\uc9c0\uba74\uc11c \ub178\ubb34\ud604\uc758 \uc815\uce58\uacbd\ub825\uc744 \ud6d1\uace0 \uc9c0\ub098\uac04\ub2e4. \ub178\ubb34\ud604 \uc758\uc6d0\uc740 1990\ub144 3\ub2f9 \ud569\ub2f9\uc744 \uacaa\uace0 \ub2e4\uc74c \ucd1d\uc120\uc778 1992\ub144 \uc81c14\ub300 \ucd1d\uc120\uc5d0\uc11c \ub099\uc120\ud55c\ub2e4. \uc774\ud6c4 1995\ub144 \uc81c1\ud68c \uc9c0\ubc29\uc120\uac70\uc5d0\uc11c \ubd80\uc0b0 \uc2dc\uc7a5 \uc120\uac70\uc5d0 \ub098\uc11c\ub098 \uc774 \uc5ed\uc2dc \ub099\uc120\ud558\uace0 \uc774\ub4ec\ud574 1996\ub144 \uc81c15\ub300 \ucd1d\uc120\uc5d0\uc11c \uc885\ub85c\uad6c\uc5d0 \ucd9c\ub9c8\ud558\ub098 \ub099\uc120\ud55c\ub2e4. \uadf8\ub7ec\ub098 1998\ub144 \uc7ac\ubcf4\uad90 \uc120\uac70\uc5d0\uc11c \uc885\ub85c\uad6c \uc120\uac70\uc5d0 \ub2e4\uc2dc \ucd9c\ub9c8\ud558\uc5ec \ub2f9\uc120\ub418\uace0, \"\ub2e4\uc2dc \uc77c\ud558\uac8c \ub420 \uc218 \uc788\uc5b4 \uae30\uc058\ub2e4\"\ub294 \uadf8\uc758 \ub2f9\uc120\uc18c\uac10\uacfc \ud568\uaed8 \ud0c0\uc784\ub77c\uc778\uc774 \uba48\ucd94\uba70 \uc601\ud654 \uc81c\ubaa9\uc774 \uc2a4\ud06c\ub9b0\uc5d0 \ub5a0\uc624\ub978\ub2e4.", "sentence_answer": "\uadf8\ud574 \uc81c13\ub300 \ucd1d\uc120 \uc5d0\uc11c \ub2f9\uc120\ub418\uc5b4 \uad6d\ud68c\uc5d0 \uc785\ubb38\ud588\ub358 \ubaa8\uc2b5\ubd80\ud130 \uc2dc\uc791\ub41c\ub2e4.", "paragraph_id": "6484011-0-1", "paragraph_question": "question: \ub178\ubb34\ud604 \ub300\ud1b5\ub839\uc774 \ucc98\uc74c \ub2f9\uc120\ub418 \uad6d\ud68c\uc5d0 \uc785\ubb38\ud558\uac8c\ub41c \ucd1d\uc120\uc740 \uc5b8\uc81c\uc778\uac00?, context: \uc601\ud654\ub294 1988\ub144 \ub178\ubb34\ud604 \ubcc0\ud638\uc0ac\uac00 \uadf8\ud574 \uc81c13\ub300 \ucd1d\uc120\uc5d0\uc11c \ub2f9\uc120\ub418\uc5b4 \uad6d\ud68c\uc5d0 \uc785\ubb38\ud588\ub358 \ubaa8\uc2b5\ubd80\ud130 \uc2dc\uc791\ub41c\ub2e4. \uc774\ud6c4\ub85c\ub294 \ub178\ub798\ub97c \ucc3e\ub294 \uc0ac\ub78c\ub4e4\uc758 \u300a\uc0ac\uacc4\u300b\uac00 \ud750\ub974\ub294 \uac00\uc6b4\ub370, \ud0c0\uc784\ub77c\uc778\uc774 \ube68\ub77c\uc9c0\uba74\uc11c \ub178\ubb34\ud604\uc758 \uc815\uce58\uacbd\ub825\uc744 \ud6d1\uace0 \uc9c0\ub098\uac04\ub2e4. \ub178\ubb34\ud604 \uc758\uc6d0\uc740 1990\ub144 3\ub2f9 \ud569\ub2f9\uc744 \uacaa\uace0 \ub2e4\uc74c \ucd1d\uc120\uc778 1992\ub144 \uc81c14\ub300 \ucd1d\uc120\uc5d0\uc11c \ub099\uc120\ud55c\ub2e4. \uc774\ud6c4 1995\ub144 \uc81c1\ud68c \uc9c0\ubc29\uc120\uac70\uc5d0\uc11c \ubd80\uc0b0 \uc2dc\uc7a5 \uc120\uac70\uc5d0 \ub098\uc11c\ub098 \uc774 \uc5ed\uc2dc \ub099\uc120\ud558\uace0 \uc774\ub4ec\ud574 1996\ub144 \uc81c15\ub300 \ucd1d\uc120\uc5d0\uc11c \uc885\ub85c\uad6c\uc5d0 \ucd9c\ub9c8\ud558\ub098 \ub099\uc120\ud55c\ub2e4. \uadf8\ub7ec\ub098 1998\ub144 \uc7ac\ubcf4\uad90 \uc120\uac70\uc5d0\uc11c \uc885\ub85c\uad6c \uc120\uac70\uc5d0 \ub2e4\uc2dc \ucd9c\ub9c8\ud558\uc5ec \ub2f9\uc120\ub418\uace0, \"\ub2e4\uc2dc \uc77c\ud558\uac8c \ub420 \uc218 \uc788\uc5b4 \uae30\uc058\ub2e4\"\ub294 \uadf8\uc758 \ub2f9\uc120\uc18c\uac10\uacfc \ud568\uaed8 \ud0c0\uc784\ub77c\uc778\uc774 \uba48\ucd94\uba70 \uc601\ud654 \uc81c\ubaa9\uc774 \uc2a4\ud06c\ub9b0\uc5d0 \ub5a0\uc624\ub978\ub2e4."} {"question": "\ub9c8\uc778 \ubd80\uc6b0\ub97c \uc18c\uba78\uc2dc\ud0a8 \uc778\ubb3c\uc740?", "paragraph": "\uc140 \uac8c\uc784\uc73c\ub85c\ubd80\ud130 7\ub144 \ud6c4, \uace0\ub4f1\ud559\uc0dd\uc774 \ub41c \uc624\ubc18\uc774 \ucc9c\ud558\uc81c\uc77c\ubb34\ub3c4\ud68c\uc5d0 \ucd9c\uc804\ud558\ub294 \uac83\uc744 \uc54c\uace0, \uc790\uc2e0\ub3c4 \ucd9c\uc804\ud558\uae30 \uc704\ud574 \uc624\uacf5\uc740 \ud558\ub8e8\ub9cc \uc774\uc2b9\uc73c\ub85c \uc624\uac8c \ub41c\ub2e4. \ucc9c\ud558\uc81c\uc77c\ubb34\ub3c4\ud68c \ub3c4\uc911, \uc804\uc6b0\uc8fc\uc758 \uc2e0\uc778 \uacc4\uc655\uc2e0\uc758 \ubd80\ud0c1\uc73c\ub85c \uc624\uacf5 \uc77c\ud589\uc740 \uc0ac\uc545\ud55c \ub9c8\ub3c4\uc0ac \ubc14\ube44\ub514\uc5d0 \uc758\ud55c \ub9c8\uc778 \ubd80\uc6b0\uc758 \ubd80\ud65c\uc744 \uc800\uc9c0\ud558\ub824\uace0 \ud558\uc9c0\ub9cc \uc2e4\ud328\ud558\uace0, \ub9c8\uc778 \ubd80\uc6b0\ub294 \ubd80\ud65c\ud574\ubc84\ub9b0\ub2e4. \uc624\uacf5\uc740 \uc774\uc2b9\uc5d0\uc11c \uccb4\ub958 \uac00\ub2a5\ud55c \uc2dc\uac04\uc774 \uc784\ubc15\ud574\uc11c \uc800\uc2b9\uc73c\ub85c \ub3cc\uc544\uac00\uac8c \ub418\uace0, \uadf8\ub97c \ub300\uc2e0\ud574 \uc624\uacf5 \uc0ac\ud6c4\uc5d0 \ud0dc\uc5b4\ub09c \ucc28\ub0a8 \uc190\uc624\ucc9c\uacfc \ud2b8\ub7ad\ud06c\uc2a4, \uc624\ubc18\uc774 \ud30c\uc6cc\uc5c5\ud574\uc11c \ub9c8\uc778 \ubd80\uc6b0\uc640 \uc2f8\uc6b0\uac8c \ub41c\ub2e4. \uadf8\ub7ec\ub098 \uc120\uc804\ud55c \ubcf4\ub78c\ub3c4 \uc5c6\uc774 \uc704\uae30\uc5d0 \ube60\uc9c0\uac8c \ub41c \uadf8\ub4e4\uc744 \uad6c\ud558\uae30 \uc704\ud574 \ub178\uacc4\uc655\uc2e0\uc758 \ubaa9\uc228\uc744 \ubc1b\uc544 \ubd80\ud65c\ud55c \uc624\uacf5\uc740 \ubca0\uc9c0\ud130\uc640 \ud568\uaed8 \uacc4\uc655\uc2e0\uacc4\uc5d0\uc11c \uc9c0\uad6c\uc778\uacfc \ub098\uba54\ud06c\uc131\uc778, \uc800\uc2b9 \uc0ac\ub78c\ub4e4\uc758 \uae30\ub97c \ubaa8\uc544 \ub9cc\ub4e4\uc5b4\uc9c4 \uc6d0\uae30\uc625\uc73c\ub85c \ub9c8\uc778 \ubd80\uc6b0\ub97c \uc18c\uba78\uc2dc\ud0a8\ub2e4.", "answer": "\uc624\uacf5", "sentence": "\uc140 \uac8c\uc784\uc73c\ub85c\ubd80\ud130 7\ub144 \ud6c4, \uace0\ub4f1\ud559\uc0dd\uc774 \ub41c \uc624\ubc18\uc774 \ucc9c\ud558\uc81c\uc77c\ubb34\ub3c4\ud68c\uc5d0 \ucd9c\uc804\ud558\ub294 \uac83\uc744 \uc54c\uace0, \uc790\uc2e0\ub3c4 \ucd9c\uc804\ud558\uae30 \uc704\ud574 \uc624\uacf5 \uc740 \ud558\ub8e8\ub9cc \uc774\uc2b9\uc73c\ub85c \uc624\uac8c \ub41c\ub2e4.", "paragraph_sentence": " \uc140 \uac8c\uc784\uc73c\ub85c\ubd80\ud130 7\ub144 \ud6c4, \uace0\ub4f1\ud559\uc0dd\uc774 \ub41c \uc624\ubc18\uc774 \ucc9c\ud558\uc81c\uc77c\ubb34\ub3c4\ud68c\uc5d0 \ucd9c\uc804\ud558\ub294 \uac83\uc744 \uc54c\uace0, \uc790\uc2e0\ub3c4 \ucd9c\uc804\ud558\uae30 \uc704\ud574 \uc624\uacf5 \uc740 \ud558\ub8e8\ub9cc \uc774\uc2b9\uc73c\ub85c \uc624\uac8c \ub41c\ub2e4. \ucc9c\ud558\uc81c\uc77c\ubb34\ub3c4\ud68c \ub3c4\uc911, \uc804\uc6b0\uc8fc\uc758 \uc2e0\uc778 \uacc4\uc655\uc2e0\uc758 \ubd80\ud0c1\uc73c\ub85c \uc624\uacf5 \uc77c\ud589\uc740 \uc0ac\uc545\ud55c \ub9c8\ub3c4\uc0ac \ubc14\ube44\ub514\uc5d0 \uc758\ud55c \ub9c8\uc778 \ubd80\uc6b0\uc758 \ubd80\ud65c\uc744 \uc800\uc9c0\ud558\ub824\uace0 \ud558\uc9c0\ub9cc \uc2e4\ud328\ud558\uace0, \ub9c8\uc778 \ubd80\uc6b0\ub294 \ubd80\ud65c\ud574\ubc84\ub9b0\ub2e4. \uc624\uacf5\uc740 \uc774\uc2b9\uc5d0\uc11c \uccb4\ub958 \uac00\ub2a5\ud55c \uc2dc\uac04\uc774 \uc784\ubc15\ud574\uc11c \uc800\uc2b9\uc73c\ub85c \ub3cc\uc544\uac00\uac8c \ub418\uace0, \uadf8\ub97c \ub300\uc2e0\ud574 \uc624\uacf5 \uc0ac\ud6c4\uc5d0 \ud0dc\uc5b4\ub09c \ucc28\ub0a8 \uc190\uc624\ucc9c\uacfc \ud2b8\ub7ad\ud06c\uc2a4, \uc624\ubc18\uc774 \ud30c\uc6cc\uc5c5\ud574\uc11c \ub9c8\uc778 \ubd80\uc6b0\uc640 \uc2f8\uc6b0\uac8c \ub41c\ub2e4. \uadf8\ub7ec\ub098 \uc120\uc804\ud55c \ubcf4\ub78c\ub3c4 \uc5c6\uc774 \uc704\uae30\uc5d0 \ube60\uc9c0\uac8c \ub41c \uadf8\ub4e4\uc744 \uad6c\ud558\uae30 \uc704\ud574 \ub178\uacc4\uc655\uc2e0\uc758 \ubaa9\uc228\uc744 \ubc1b\uc544 \ubd80\ud65c\ud55c \uc624\uacf5\uc740 \ubca0\uc9c0\ud130\uc640 \ud568\uaed8 \uacc4\uc655\uc2e0\uacc4\uc5d0\uc11c \uc9c0\uad6c\uc778\uacfc \ub098\uba54\ud06c\uc131\uc778, \uc800\uc2b9 \uc0ac\ub78c\ub4e4\uc758 \uae30\ub97c \ubaa8\uc544 \ub9cc\ub4e4\uc5b4\uc9c4 \uc6d0\uae30\uc625\uc73c\ub85c \ub9c8\uc778 \ubd80\uc6b0\ub97c \uc18c\uba78\uc2dc\ud0a8\ub2e4.", "paragraph_answer": "\uc140 \uac8c\uc784\uc73c\ub85c\ubd80\ud130 7\ub144 \ud6c4, \uace0\ub4f1\ud559\uc0dd\uc774 \ub41c \uc624\ubc18\uc774 \ucc9c\ud558\uc81c\uc77c\ubb34\ub3c4\ud68c\uc5d0 \ucd9c\uc804\ud558\ub294 \uac83\uc744 \uc54c\uace0, \uc790\uc2e0\ub3c4 \ucd9c\uc804\ud558\uae30 \uc704\ud574 \uc624\uacf5 \uc740 \ud558\ub8e8\ub9cc \uc774\uc2b9\uc73c\ub85c \uc624\uac8c \ub41c\ub2e4. \ucc9c\ud558\uc81c\uc77c\ubb34\ub3c4\ud68c \ub3c4\uc911, \uc804\uc6b0\uc8fc\uc758 \uc2e0\uc778 \uacc4\uc655\uc2e0\uc758 \ubd80\ud0c1\uc73c\ub85c \uc624\uacf5 \uc77c\ud589\uc740 \uc0ac\uc545\ud55c \ub9c8\ub3c4\uc0ac \ubc14\ube44\ub514\uc5d0 \uc758\ud55c \ub9c8\uc778 \ubd80\uc6b0\uc758 \ubd80\ud65c\uc744 \uc800\uc9c0\ud558\ub824\uace0 \ud558\uc9c0\ub9cc \uc2e4\ud328\ud558\uace0, \ub9c8\uc778 \ubd80\uc6b0\ub294 \ubd80\ud65c\ud574\ubc84\ub9b0\ub2e4. \uc624\uacf5\uc740 \uc774\uc2b9\uc5d0\uc11c \uccb4\ub958 \uac00\ub2a5\ud55c \uc2dc\uac04\uc774 \uc784\ubc15\ud574\uc11c \uc800\uc2b9\uc73c\ub85c \ub3cc\uc544\uac00\uac8c \ub418\uace0, \uadf8\ub97c \ub300\uc2e0\ud574 \uc624\uacf5 \uc0ac\ud6c4\uc5d0 \ud0dc\uc5b4\ub09c \ucc28\ub0a8 \uc190\uc624\ucc9c\uacfc \ud2b8\ub7ad\ud06c\uc2a4, \uc624\ubc18\uc774 \ud30c\uc6cc\uc5c5\ud574\uc11c \ub9c8\uc778 \ubd80\uc6b0\uc640 \uc2f8\uc6b0\uac8c \ub41c\ub2e4. \uadf8\ub7ec\ub098 \uc120\uc804\ud55c \ubcf4\ub78c\ub3c4 \uc5c6\uc774 \uc704\uae30\uc5d0 \ube60\uc9c0\uac8c \ub41c \uadf8\ub4e4\uc744 \uad6c\ud558\uae30 \uc704\ud574 \ub178\uacc4\uc655\uc2e0\uc758 \ubaa9\uc228\uc744 \ubc1b\uc544 \ubd80\ud65c\ud55c \uc624\uacf5\uc740 \ubca0\uc9c0\ud130\uc640 \ud568\uaed8 \uacc4\uc655\uc2e0\uacc4\uc5d0\uc11c \uc9c0\uad6c\uc778\uacfc \ub098\uba54\ud06c\uc131\uc778, \uc800\uc2b9 \uc0ac\ub78c\ub4e4\uc758 \uae30\ub97c \ubaa8\uc544 \ub9cc\ub4e4\uc5b4\uc9c4 \uc6d0\uae30\uc625\uc73c\ub85c \ub9c8\uc778 \ubd80\uc6b0\ub97c \uc18c\uba78\uc2dc\ud0a8\ub2e4.", "sentence_answer": "\uc140 \uac8c\uc784\uc73c\ub85c\ubd80\ud130 7\ub144 \ud6c4, \uace0\ub4f1\ud559\uc0dd\uc774 \ub41c \uc624\ubc18\uc774 \ucc9c\ud558\uc81c\uc77c\ubb34\ub3c4\ud68c\uc5d0 \ucd9c\uc804\ud558\ub294 \uac83\uc744 \uc54c\uace0, \uc790\uc2e0\ub3c4 \ucd9c\uc804\ud558\uae30 \uc704\ud574 \uc624\uacf5 \uc740 \ud558\ub8e8\ub9cc \uc774\uc2b9\uc73c\ub85c \uc624\uac8c \ub41c\ub2e4.", "paragraph_id": "6048697-2-2", "paragraph_question": "question: \ub9c8\uc778 \ubd80\uc6b0\ub97c \uc18c\uba78\uc2dc\ud0a8 \uc778\ubb3c\uc740?, context: \uc140 \uac8c\uc784\uc73c\ub85c\ubd80\ud130 7\ub144 \ud6c4, \uace0\ub4f1\ud559\uc0dd\uc774 \ub41c \uc624\ubc18\uc774 \ucc9c\ud558\uc81c\uc77c\ubb34\ub3c4\ud68c\uc5d0 \ucd9c\uc804\ud558\ub294 \uac83\uc744 \uc54c\uace0, \uc790\uc2e0\ub3c4 \ucd9c\uc804\ud558\uae30 \uc704\ud574 \uc624\uacf5\uc740 \ud558\ub8e8\ub9cc \uc774\uc2b9\uc73c\ub85c \uc624\uac8c \ub41c\ub2e4. \ucc9c\ud558\uc81c\uc77c\ubb34\ub3c4\ud68c \ub3c4\uc911, \uc804\uc6b0\uc8fc\uc758 \uc2e0\uc778 \uacc4\uc655\uc2e0\uc758 \ubd80\ud0c1\uc73c\ub85c \uc624\uacf5 \uc77c\ud589\uc740 \uc0ac\uc545\ud55c \ub9c8\ub3c4\uc0ac \ubc14\ube44\ub514\uc5d0 \uc758\ud55c \ub9c8\uc778 \ubd80\uc6b0\uc758 \ubd80\ud65c\uc744 \uc800\uc9c0\ud558\ub824\uace0 \ud558\uc9c0\ub9cc \uc2e4\ud328\ud558\uace0, \ub9c8\uc778 \ubd80\uc6b0\ub294 \ubd80\ud65c\ud574\ubc84\ub9b0\ub2e4. \uc624\uacf5\uc740 \uc774\uc2b9\uc5d0\uc11c \uccb4\ub958 \uac00\ub2a5\ud55c \uc2dc\uac04\uc774 \uc784\ubc15\ud574\uc11c \uc800\uc2b9\uc73c\ub85c \ub3cc\uc544\uac00\uac8c \ub418\uace0, \uadf8\ub97c \ub300\uc2e0\ud574 \uc624\uacf5 \uc0ac\ud6c4\uc5d0 \ud0dc\uc5b4\ub09c \ucc28\ub0a8 \uc190\uc624\ucc9c\uacfc \ud2b8\ub7ad\ud06c\uc2a4, \uc624\ubc18\uc774 \ud30c\uc6cc\uc5c5\ud574\uc11c \ub9c8\uc778 \ubd80\uc6b0\uc640 \uc2f8\uc6b0\uac8c \ub41c\ub2e4. \uadf8\ub7ec\ub098 \uc120\uc804\ud55c \ubcf4\ub78c\ub3c4 \uc5c6\uc774 \uc704\uae30\uc5d0 \ube60\uc9c0\uac8c \ub41c \uadf8\ub4e4\uc744 \uad6c\ud558\uae30 \uc704\ud574 \ub178\uacc4\uc655\uc2e0\uc758 \ubaa9\uc228\uc744 \ubc1b\uc544 \ubd80\ud65c\ud55c \uc624\uacf5\uc740 \ubca0\uc9c0\ud130\uc640 \ud568\uaed8 \uacc4\uc655\uc2e0\uacc4\uc5d0\uc11c \uc9c0\uad6c\uc778\uacfc \ub098\uba54\ud06c\uc131\uc778, \uc800\uc2b9 \uc0ac\ub78c\ub4e4\uc758 \uae30\ub97c \ubaa8\uc544 \ub9cc\ub4e4\uc5b4\uc9c4 \uc6d0\uae30\uc625\uc73c\ub85c \ub9c8\uc778 \ubd80\uc6b0\ub97c \uc18c\uba78\uc2dc\ud0a8\ub2e4."} {"question": "\uc0e4\uc774\ub2c8\uc758 \uc568\ubc94 <1 of 1>\uc740 \uc815\uaddc \uba87 \uc9d1\uc778\uac00?", "paragraph": "2008\ub144 5\uc6d4 25\uc77c \uccab \ubc88\uc9f8 \ubbf8\ub2c8 \uc568\ubc94 \u300a\ub204\ub09c \ub108\ubb34 \uc608\ubed0 (Replay)\u300b\ub97c \ubc1c\ub9e4\ud574 \ub370\ubdd4\ud558\uc600\uace0, \uadf8\ud574 8\uc6d4 28\uc77c \uc815\uaddc \uc568\ubc94 \u300aThe SHINee World\u300b\ub97c \ubc1c\ub9e4\ud574 \uccab 1\uc704\ub97c \ud558\uc600\ub2e4. 2009\ub144\uc5d0\ub294 \ub450 \uc7a5\uc758 \ubbf8\ub2c8 \uc568\ubc94 \u300aROMEO\u300b, \u300a2009, Year Of Us\u300b\ub97c \ubc1c\ub9e4\ud588\uace0 2010\ub144 7\uc6d4 19\uc77c \ub450 \ubc88\uc9f8 \uc815\uaddc \uc568\ubc94 \u300aLucifer\u300b\uc640 \uac19\uc740\ud574 9\uc6d4 20\uc77c \u300aHello\u300b\ub97c \ubc1c\ub9e4\ud558\uba70 \ud65c\ub3d9\uc744 \uc774\uc5b4\uac14\ub2e4. \uc774\ud6c4 2011\ub144 6\uc6d4 22\uc77c \uc77c\ubcf8 \ub370\ubdd4 \uc2f1\uae00 \u3008Replay\u3009\ub97c \ubc1c\ub9e4\ud574 \uc77c\ubcf8 \ub370\ubdd4\ub97c \ud558\uc600\uace0, \uac19\uc740 \ud574 12\uc6d4 7\uc77c \uc77c\ubcf8 \uccab \uc815\uaddc \uc568\ubc94 \u300aTHE FIRST\u300b\ub97c \ubc1c\ub9e4\ud588\ub2e4. 2012\ub144 3\uc6d4 19\uc77c \ub300\ud55c\ubbfc\uad6d\uc5d0\uc11c\uc758 \uc624\ub79c \uacf5\ubc31\uc744 \uae68\uace0 \ub124 \ubc88\uc9f8 \ubbf8\ub2c8 \uc568\ubc94 \u300aSherlock\u300b\uc744 \ubc1c\ub9e4\ud558\uba70 \ucef4\ubc31\ud588\uace0, \uc774\ub4ec\ud574 \uc138 \ubc88\uc9f8 \uc815\uaddc \uc568\ubc94\uc744 \ub450 \ucc55\ud130\ub85c \ub098\ub208 \u300aChapter 1. Dream Girl \u2013 The Misconceptions of You\u300b, \u300aChapter 2. Why So Serious? \u2013 The Misconceptions of Me\u300b\uc640 \ub450 \uc568\ubc94\uc758 \ud569\ubcf8 \u300aThe Misconceptions of Us\u300b\ub97c \ubc1c\ub9e4\ud588\ub2e4. \uadf8\ud574 7\uc6d4\uc5d0\ub294 \ub450 \ubc88\uc9f8 \uc77c\ubcf8 \uc815\uaddc \uc568\ubc94 \u300aBoys Meet U\u300b\ub97c \ubc1c\ub9e4\ud588\uace0, 2013\ub144 10\uc6d4 5\ubc88\uc9f8 \ubbf8\ub2c8 \uc568\ubc94 \u300aEverybody\u300b\ub97c \ubc1c\ub9e4\ud558\uba70 \ud558\ubc18\uae30 \ud65c\ub3d9\uc744 \uc774\uc5b4\uac14\ub2e4. 2015\ub144 5\uc6d4 \uc815\uaddc \uc568\ubc94 4\uc9d1 \u300aOdd\u300b\ub97c \ubc1c\ub9e4\ud558\uc5ec 1\ub144 7\uac1c\uc6d4 \ub9cc\uc5d0 \uad6d\ub0b4 \ud65c\ub3d9\uc744 \uc7ac\uac1c\ud558\uc600\uace0, 8\uc6d4 3\uc77c \ub9ac\ud328\ud0a4\uc9c0 \uc568\ubc94 \u300aMarried To The Music\u300b\uc744 \ubc1c\ub9e4\ud558\uc600\ub2e4. 2016\ub144 10\uc6d4 5\uc77c \uc815\uaddc 5\uc9d1 \u300a1 of 1\u300b\uc73c\ub85c \ucef4\ubc31\ud558\uc600\uace0, \uac19\uc740 \ud574 11\uc6d4 16\uc77c \ub9ac\ud328\ud0a4\uc9c0 \uc568\ubc94 \u300a1 and 1\u300b\uc744 \ubc1c\ub9e4\ud558\uc5ec \ud65c\ub3d9\uc744 \uc774\uc5b4\ub098\uac14\ub2e4. 2018\ub144 5\uc6d4 28\uc77c \ub370\ubdd4 10\uc8fc\ub144 \uc568\ubc94\uc815\uaddc 6\uc9d1 \u300aThe Story of Light EP.1\u300b\uc73c\ub85c \uccab 4\uc778\uccb4\uc81c\ub85c \uad6d\ub0b4 \ucef4\ubc31 \ud558\uc600\ub2e4. 2018\ub144 6\uc6d4 11\uc77c \u300aThe Story of Light EP.2\u300b\uac00 \uacf5\uac1c \ub418\uc5c8\ub2e4. 2018\ub144 6\uc6d4 25\uc77c, \u300aThe Story of Light EP.3\u300b\uac00 \uacf5\uac1c\ub420 \uc608\uc815\uc774\ub2e4.", "answer": "5\uc9d1", "sentence": "2016\ub144 10\uc6d4 5\uc77c \uc815\uaddc 5\uc9d1 \u300a1 of 1\u300b\uc73c\ub85c \ucef4\ubc31\ud558\uc600\uace0, \uac19\uc740 \ud574 11\uc6d4 16\uc77c \ub9ac\ud328\ud0a4\uc9c0 \uc568\ubc94 \u300a1 and 1\u300b\uc744 \ubc1c\ub9e4\ud558\uc5ec \ud65c\ub3d9\uc744 \uc774\uc5b4\ub098\uac14\ub2e4.", "paragraph_sentence": "2008\ub144 5\uc6d4 25\uc77c \uccab \ubc88\uc9f8 \ubbf8\ub2c8 \uc568\ubc94 \u300a\ub204\ub09c \ub108\ubb34 \uc608\ubed0 (Replay)\u300b\ub97c \ubc1c\ub9e4\ud574 \ub370\ubdd4\ud558\uc600\uace0, \uadf8\ud574 8\uc6d4 28\uc77c \uc815\uaddc \uc568\ubc94 \u300aThe SHINee World\u300b\ub97c \ubc1c\ub9e4\ud574 \uccab 1\uc704\ub97c \ud558\uc600\ub2e4. 2009\ub144\uc5d0\ub294 \ub450 \uc7a5\uc758 \ubbf8\ub2c8 \uc568\ubc94 \u300aROMEO\u300b, \u300a2009, Year Of Us\u300b\ub97c \ubc1c\ub9e4\ud588\uace0 2010\ub144 7\uc6d4 19\uc77c \ub450 \ubc88\uc9f8 \uc815\uaddc \uc568\ubc94 \u300aLucifer\u300b\uc640 \uac19\uc740\ud574 9\uc6d4 20\uc77c \u300aHello\u300b\ub97c \ubc1c\ub9e4\ud558\uba70 \ud65c\ub3d9\uc744 \uc774\uc5b4\uac14\ub2e4. \uc774\ud6c4 2011\ub144 6\uc6d4 22\uc77c \uc77c\ubcf8 \ub370\ubdd4 \uc2f1\uae00 \u3008Replay\u3009\ub97c \ubc1c\ub9e4\ud574 \uc77c\ubcf8 \ub370\ubdd4\ub97c \ud558\uc600\uace0, \uac19\uc740 \ud574 12\uc6d4 7\uc77c \uc77c\ubcf8 \uccab \uc815\uaddc \uc568\ubc94 \u300aTHE FIRST\u300b\ub97c \ubc1c\ub9e4\ud588\ub2e4. 2012\ub144 3\uc6d4 19\uc77c \ub300\ud55c\ubbfc\uad6d\uc5d0\uc11c\uc758 \uc624\ub79c \uacf5\ubc31\uc744 \uae68\uace0 \ub124 \ubc88\uc9f8 \ubbf8\ub2c8 \uc568\ubc94 \u300aSherlock\u300b\uc744 \ubc1c\ub9e4\ud558\uba70 \ucef4\ubc31\ud588\uace0, \uc774\ub4ec\ud574 \uc138 \ubc88\uc9f8 \uc815\uaddc \uc568\ubc94\uc744 \ub450 \ucc55\ud130\ub85c \ub098\ub208 \u300aChapter 1. Dream Girl \u2013 The Misconceptions of You\u300b, \u300aChapter 2. Why So Serious? \u2013 The Misconceptions of Me\u300b\uc640 \ub450 \uc568\ubc94\uc758 \ud569\ubcf8 \u300aThe Misconceptions of Us\u300b\ub97c \ubc1c\ub9e4\ud588\ub2e4. \uadf8\ud574 7\uc6d4\uc5d0\ub294 \ub450 \ubc88\uc9f8 \uc77c\ubcf8 \uc815\uaddc \uc568\ubc94 \u300aBoys Meet U\u300b\ub97c \ubc1c\ub9e4\ud588\uace0, 2013\ub144 10\uc6d4 5\ubc88\uc9f8 \ubbf8\ub2c8 \uc568\ubc94 \u300aEverybody\u300b\ub97c \ubc1c\ub9e4\ud558\uba70 \ud558\ubc18\uae30 \ud65c\ub3d9\uc744 \uc774\uc5b4\uac14\ub2e4. 2015\ub144 5\uc6d4 \uc815\uaddc \uc568\ubc94 4\uc9d1 \u300aOdd\u300b\ub97c \ubc1c\ub9e4\ud558\uc5ec 1\ub144 7\uac1c\uc6d4 \ub9cc\uc5d0 \uad6d\ub0b4 \ud65c\ub3d9\uc744 \uc7ac\uac1c\ud558\uc600\uace0, 8\uc6d4 3\uc77c \ub9ac\ud328\ud0a4\uc9c0 \uc568\ubc94 \u300aMarried To The Music\u300b\uc744 \ubc1c\ub9e4\ud558\uc600\ub2e4. 2016\ub144 10\uc6d4 5\uc77c \uc815\uaddc 5\uc9d1 \u300a1 of 1\u300b\uc73c\ub85c \ucef4\ubc31\ud558\uc600\uace0, \uac19\uc740 \ud574 11\uc6d4 16\uc77c \ub9ac\ud328\ud0a4\uc9c0 \uc568\ubc94 \u300a1 and 1\u300b\uc744 \ubc1c\ub9e4\ud558\uc5ec \ud65c\ub3d9\uc744 \uc774\uc5b4\ub098\uac14\ub2e4. 2018\ub144 5\uc6d4 28\uc77c \ub370\ubdd4 10\uc8fc\ub144 \uc568\ubc94\uc815\uaddc 6\uc9d1 \u300aThe Story of Light EP.1\u300b\uc73c\ub85c \uccab 4\uc778\uccb4\uc81c\ub85c \uad6d\ub0b4 \ucef4\ubc31 \ud558\uc600\ub2e4. 2018\ub144 6\uc6d4 11\uc77c \u300aThe Story of Light EP.2\u300b\uac00 \uacf5\uac1c \ub418\uc5c8\ub2e4. 2018\ub144 6\uc6d4 25\uc77c, \u300aThe Story of Light EP.3\u300b\uac00 \uacf5\uac1c\ub420 \uc608\uc815\uc774\ub2e4.", "paragraph_answer": "2008\ub144 5\uc6d4 25\uc77c \uccab \ubc88\uc9f8 \ubbf8\ub2c8 \uc568\ubc94 \u300a\ub204\ub09c \ub108\ubb34 \uc608\ubed0 (Replay)\u300b\ub97c \ubc1c\ub9e4\ud574 \ub370\ubdd4\ud558\uc600\uace0, \uadf8\ud574 8\uc6d4 28\uc77c \uc815\uaddc \uc568\ubc94 \u300aThe SHINee World\u300b\ub97c \ubc1c\ub9e4\ud574 \uccab 1\uc704\ub97c \ud558\uc600\ub2e4. 2009\ub144\uc5d0\ub294 \ub450 \uc7a5\uc758 \ubbf8\ub2c8 \uc568\ubc94 \u300aROMEO\u300b, \u300a2009, Year Of Us\u300b\ub97c \ubc1c\ub9e4\ud588\uace0 2010\ub144 7\uc6d4 19\uc77c \ub450 \ubc88\uc9f8 \uc815\uaddc \uc568\ubc94 \u300aLucifer\u300b\uc640 \uac19\uc740\ud574 9\uc6d4 20\uc77c \u300aHello\u300b\ub97c \ubc1c\ub9e4\ud558\uba70 \ud65c\ub3d9\uc744 \uc774\uc5b4\uac14\ub2e4. \uc774\ud6c4 2011\ub144 6\uc6d4 22\uc77c \uc77c\ubcf8 \ub370\ubdd4 \uc2f1\uae00 \u3008Replay\u3009\ub97c \ubc1c\ub9e4\ud574 \uc77c\ubcf8 \ub370\ubdd4\ub97c \ud558\uc600\uace0, \uac19\uc740 \ud574 12\uc6d4 7\uc77c \uc77c\ubcf8 \uccab \uc815\uaddc \uc568\ubc94 \u300aTHE FIRST\u300b\ub97c \ubc1c\ub9e4\ud588\ub2e4. 2012\ub144 3\uc6d4 19\uc77c \ub300\ud55c\ubbfc\uad6d\uc5d0\uc11c\uc758 \uc624\ub79c \uacf5\ubc31\uc744 \uae68\uace0 \ub124 \ubc88\uc9f8 \ubbf8\ub2c8 \uc568\ubc94 \u300aSherlock\u300b\uc744 \ubc1c\ub9e4\ud558\uba70 \ucef4\ubc31\ud588\uace0, \uc774\ub4ec\ud574 \uc138 \ubc88\uc9f8 \uc815\uaddc \uc568\ubc94\uc744 \ub450 \ucc55\ud130\ub85c \ub098\ub208 \u300aChapter 1. Dream Girl \u2013 The Misconceptions of You\u300b, \u300aChapter 2. Why So Serious? \u2013 The Misconceptions of Me\u300b\uc640 \ub450 \uc568\ubc94\uc758 \ud569\ubcf8 \u300aThe Misconceptions of Us\u300b\ub97c \ubc1c\ub9e4\ud588\ub2e4. \uadf8\ud574 7\uc6d4\uc5d0\ub294 \ub450 \ubc88\uc9f8 \uc77c\ubcf8 \uc815\uaddc \uc568\ubc94 \u300aBoys Meet U\u300b\ub97c \ubc1c\ub9e4\ud588\uace0, 2013\ub144 10\uc6d4 5\ubc88\uc9f8 \ubbf8\ub2c8 \uc568\ubc94 \u300aEverybody\u300b\ub97c \ubc1c\ub9e4\ud558\uba70 \ud558\ubc18\uae30 \ud65c\ub3d9\uc744 \uc774\uc5b4\uac14\ub2e4. 2015\ub144 5\uc6d4 \uc815\uaddc \uc568\ubc94 4\uc9d1 \u300aOdd\u300b\ub97c \ubc1c\ub9e4\ud558\uc5ec 1\ub144 7\uac1c\uc6d4 \ub9cc\uc5d0 \uad6d\ub0b4 \ud65c\ub3d9\uc744 \uc7ac\uac1c\ud558\uc600\uace0, 8\uc6d4 3\uc77c \ub9ac\ud328\ud0a4\uc9c0 \uc568\ubc94 \u300aMarried To The Music\u300b\uc744 \ubc1c\ub9e4\ud558\uc600\ub2e4. 2016\ub144 10\uc6d4 5\uc77c \uc815\uaddc 5\uc9d1 \u300a1 of 1\u300b\uc73c\ub85c \ucef4\ubc31\ud558\uc600\uace0, \uac19\uc740 \ud574 11\uc6d4 16\uc77c \ub9ac\ud328\ud0a4\uc9c0 \uc568\ubc94 \u300a1 and 1\u300b\uc744 \ubc1c\ub9e4\ud558\uc5ec \ud65c\ub3d9\uc744 \uc774\uc5b4\ub098\uac14\ub2e4. 2018\ub144 5\uc6d4 28\uc77c \ub370\ubdd4 10\uc8fc\ub144 \uc568\ubc94\uc815\uaddc 6\uc9d1 \u300aThe Story of Light EP.1\u300b\uc73c\ub85c \uccab 4\uc778\uccb4\uc81c\ub85c \uad6d\ub0b4 \ucef4\ubc31 \ud558\uc600\ub2e4. 2018\ub144 6\uc6d4 11\uc77c \u300aThe Story of Light EP.2\u300b\uac00 \uacf5\uac1c \ub418\uc5c8\ub2e4. 2018\ub144 6\uc6d4 25\uc77c, \u300aThe Story of Light EP.3\u300b\uac00 \uacf5\uac1c\ub420 \uc608\uc815\uc774\ub2e4.", "sentence_answer": "2016\ub144 10\uc6d4 5\uc77c \uc815\uaddc 5\uc9d1 \u300a1 of 1\u300b\uc73c\ub85c \ucef4\ubc31\ud558\uc600\uace0, \uac19\uc740 \ud574 11\uc6d4 16\uc77c \ub9ac\ud328\ud0a4\uc9c0 \uc568\ubc94 \u300a1 and 1\u300b\uc744 \ubc1c\ub9e4\ud558\uc5ec \ud65c\ub3d9\uc744 \uc774\uc5b4\ub098\uac14\ub2e4.", "paragraph_id": "6511459-0-2", "paragraph_question": "question: \uc0e4\uc774\ub2c8\uc758 \uc568\ubc94 <1 of 1>\uc740 \uc815\uaddc \uba87 \uc9d1\uc778\uac00?, context: 2008\ub144 5\uc6d4 25\uc77c \uccab \ubc88\uc9f8 \ubbf8\ub2c8 \uc568\ubc94 \u300a\ub204\ub09c \ub108\ubb34 \uc608\ubed0 (Replay)\u300b\ub97c \ubc1c\ub9e4\ud574 \ub370\ubdd4\ud558\uc600\uace0, \uadf8\ud574 8\uc6d4 28\uc77c \uc815\uaddc \uc568\ubc94 \u300aThe SHINee World\u300b\ub97c \ubc1c\ub9e4\ud574 \uccab 1\uc704\ub97c \ud558\uc600\ub2e4. 2009\ub144\uc5d0\ub294 \ub450 \uc7a5\uc758 \ubbf8\ub2c8 \uc568\ubc94 \u300aROMEO\u300b, \u300a2009, Year Of Us\u300b\ub97c \ubc1c\ub9e4\ud588\uace0 2010\ub144 7\uc6d4 19\uc77c \ub450 \ubc88\uc9f8 \uc815\uaddc \uc568\ubc94 \u300aLucifer\u300b\uc640 \uac19\uc740\ud574 9\uc6d4 20\uc77c \u300aHello\u300b\ub97c \ubc1c\ub9e4\ud558\uba70 \ud65c\ub3d9\uc744 \uc774\uc5b4\uac14\ub2e4. \uc774\ud6c4 2011\ub144 6\uc6d4 22\uc77c \uc77c\ubcf8 \ub370\ubdd4 \uc2f1\uae00 \u3008Replay\u3009\ub97c \ubc1c\ub9e4\ud574 \uc77c\ubcf8 \ub370\ubdd4\ub97c \ud558\uc600\uace0, \uac19\uc740 \ud574 12\uc6d4 7\uc77c \uc77c\ubcf8 \uccab \uc815\uaddc \uc568\ubc94 \u300aTHE FIRST\u300b\ub97c \ubc1c\ub9e4\ud588\ub2e4. 2012\ub144 3\uc6d4 19\uc77c \ub300\ud55c\ubbfc\uad6d\uc5d0\uc11c\uc758 \uc624\ub79c \uacf5\ubc31\uc744 \uae68\uace0 \ub124 \ubc88\uc9f8 \ubbf8\ub2c8 \uc568\ubc94 \u300aSherlock\u300b\uc744 \ubc1c\ub9e4\ud558\uba70 \ucef4\ubc31\ud588\uace0, \uc774\ub4ec\ud574 \uc138 \ubc88\uc9f8 \uc815\uaddc \uc568\ubc94\uc744 \ub450 \ucc55\ud130\ub85c \ub098\ub208 \u300aChapter 1. Dream Girl \u2013 The Misconceptions of You\u300b, \u300aChapter 2. Why So Serious? \u2013 The Misconceptions of Me\u300b\uc640 \ub450 \uc568\ubc94\uc758 \ud569\ubcf8 \u300aThe Misconceptions of Us\u300b\ub97c \ubc1c\ub9e4\ud588\ub2e4. \uadf8\ud574 7\uc6d4\uc5d0\ub294 \ub450 \ubc88\uc9f8 \uc77c\ubcf8 \uc815\uaddc \uc568\ubc94 \u300aBoys Meet U\u300b\ub97c \ubc1c\ub9e4\ud588\uace0, 2013\ub144 10\uc6d4 5\ubc88\uc9f8 \ubbf8\ub2c8 \uc568\ubc94 \u300aEverybody\u300b\ub97c \ubc1c\ub9e4\ud558\uba70 \ud558\ubc18\uae30 \ud65c\ub3d9\uc744 \uc774\uc5b4\uac14\ub2e4. 2015\ub144 5\uc6d4 \uc815\uaddc \uc568\ubc94 4\uc9d1 \u300aOdd\u300b\ub97c \ubc1c\ub9e4\ud558\uc5ec 1\ub144 7\uac1c\uc6d4 \ub9cc\uc5d0 \uad6d\ub0b4 \ud65c\ub3d9\uc744 \uc7ac\uac1c\ud558\uc600\uace0, 8\uc6d4 3\uc77c \ub9ac\ud328\ud0a4\uc9c0 \uc568\ubc94 \u300aMarried To The Music\u300b\uc744 \ubc1c\ub9e4\ud558\uc600\ub2e4. 2016\ub144 10\uc6d4 5\uc77c \uc815\uaddc 5\uc9d1 \u300a1 of 1\u300b\uc73c\ub85c \ucef4\ubc31\ud558\uc600\uace0, \uac19\uc740 \ud574 11\uc6d4 16\uc77c \ub9ac\ud328\ud0a4\uc9c0 \uc568\ubc94 \u300a1 and 1\u300b\uc744 \ubc1c\ub9e4\ud558\uc5ec \ud65c\ub3d9\uc744 \uc774\uc5b4\ub098\uac14\ub2e4. 2018\ub144 5\uc6d4 28\uc77c \ub370\ubdd4 10\uc8fc\ub144 \uc568\ubc94\uc815\uaddc 6\uc9d1 \u300aThe Story of Light EP.1\u300b\uc73c\ub85c \uccab 4\uc778\uccb4\uc81c\ub85c \uad6d\ub0b4 \ucef4\ubc31 \ud558\uc600\ub2e4. 2018\ub144 6\uc6d4 11\uc77c \u300aThe Story of Light EP.2\u300b\uac00 \uacf5\uac1c \ub418\uc5c8\ub2e4. 2018\ub144 6\uc6d4 25\uc77c, \u300aThe Story of Light EP.3\u300b\uac00 \uacf5\uac1c\ub420 \uc608\uc815\uc774\ub2e4."} {"question": "\ub124\ud314\uc758 \uad6c\ub974\uce74 \ubd80\uc871\uc744 \ubab0\uc544\ub0b4\uae30 \uc704\ud574 \uac74\ub96d\uc81c\uac00 \uc0ac\ub839\uad00\uc5d0 \uc784\uba85\ud55c \uc778\ubb3c\uc740?", "paragraph": "\ub124\ud314\uc744 \uc810\ub839\ud560 \ub54c\ub294 \uc774\ubbf8 \ub124\ud314\uc758 \uad6c\ub974\uce74 \ubd80\uc871\uc774 \uccd0\ub4e4\uc5b4\uc640 1788\ub144(\uac74\ub96d 53\ub144) \ud2f0\ubca0\ud2b8 \ub0a8\ubd80\ub97c \uc810\ub839\ud558\uace0 \ub77c\uc0ac\ub97c \ud5a5\ud574 \uc9c4\uaca9\ud574\uc624\uace0 \uc788\uc5c8\ub2e4. \ud2f0\ubca0\ud2b8 \uc8fc\uc7ac \uccad\uad70\uc740 \ud310\uccb8 \ub77c\ub9c8 \ud150\ud398\uc774 \ub2c8\ub9c8\ub97c \ub0a9\uce58\ud558\uc5ec \ub2e4\ub978 \uacf3\uc5d0\uc11c \uc694\uaca9\ud560 \ud0dc\uc138\ub97c \uac16\ucd94\uc5c8\ub2e4. \ud55c\ud3b8 \uac74\ub96d\uc81c\ub294 \uc0ac\ucc9c\uc131\uc5d0 \uc8fc\uc7ac\ud558\ub358 \uad70\uc0ac\ub4e4\uc5d0\uac8c \ud2f0\ubca0\ud2b8 \ub0a8\ubd80\uc5d0 \uc788\ub294 \uad6c\ub974\uce74 \uad70\uc0ac\ub4e4\uc744 \ubab0\uc544\ub0b4\ub77c \uba85\ub839\ud558\uc600\uc73c\ub098 \uc774\ubbf8 \uad6c\ub974\uce74 \ubd80\uc871\uc740 \ud1f4\uac01\ud55c \uc0c1\ud0dc\uc600\ub2e4. \uadf8\ub7ec\ub098 1791\ub144(\uac74\ub96d 56\ub144) \uaca8\uc6b8, \uad6c\ub974\uce74\ub294 \ub2e4\uc2dc \ud2f0\ubca0\ud2b8\ub85c \uc9c4\uaca9\ud574 \ub4e4\uc5b4\uc654\ub2e4. \uc774\uc5d0 \ub9de\uc11c \uac74\ub96d\uc81c\ub294 \ud314\uae30\uad70\uacfc \ub179\uc601\uc758 \uad70\uc0ac\ub4e4\uc744 \ud30c\ubcd1\ud558\uace0 \uadf8 \uc0ac\ub839\uad00\uc5d0 \uc790\uc2e0\uc758 \ucc98\uc870\uce74\uc778 \ubcf5\uac15\uc548(\u798f\u5eb7\u5b89)\uc744 \uc784\uba85\ud558\uc5ec \uad6c\ub974\uce74\uad70\uc744 \ubab0\uc544\ub0b4\ub3c4\ub85d \ud558\uc600\ub2e4. \ubcf5\uac15\uc548\uc758 \uad70\ub300\ub294 \uccad\ud574\uc131\uc73c\ub85c \uc9c4\uaca9\ud574 \ud2f0\ubca0\ud2b8 \ubd84\uc9c0\ub85c \ub4e4\uc5b4\uac00\ub824 \ud558\uc600\uc73c\ub098 \ub0a0\uc528\uac00 \ucda5\uace0 \uace0\uc9c0 \ubd84\ub300\ub77c\uc11c \ub208\uc774 \ub179\uc9c0 \uc54a\uc544 \ub0a0\uc774 \ud480\ub9b4 \ub54c\uae4c\uc9c0 \uae30\ub2e4\ub9ac\ub2e4\uac00 \ubd04\uc774 \ub418\uc5b4 \ub2e4\uc2dc \uc9c4\uaca9\ud55c \ud6c4 1792\ub144(\uac74\ub96d 57\ub144) \uc5ec\ub984\uc5d0 \uad6c\ub974\uce74\uad70\uc744 \uc12c\uba78\ud558\uace0 \uadf8\ub4e4\uc744 \ud788\ub9d0\ub77c\uc57c \uc0b0\ub9e5\uc758 \uce74\ud2b8\ub9cc\ub450 \uacc4\uace1\uc73c\ub85c\uae4c\uc9c0 \uc555\ubc15\ud558\uc600\ub2e4. 1793\ub144(\uac74\ub96d 58\ub144) \ubcf5\uac15\uc548\uc740 \uad6c\ub974\uce74\uc871\uc758 \ud56d\ubcf5\uc744 \ub9cc\uc8fc\uc2dd\uc73c\ub85c \ubc1b\uace0 \ucca0\uad70\ud558\uc5ec \ubd81\uacbd\uc73c\ub85c \ub3cc\uc544\uac14\ub2e4.", "answer": "\ubcf5\uac15\uc548", "sentence": "\uc774\uc5d0 \ub9de\uc11c \uac74\ub96d\uc81c\ub294 \ud314\uae30\uad70\uacfc \ub179\uc601\uc758 \uad70\uc0ac\ub4e4\uc744 \ud30c\ubcd1\ud558\uace0 \uadf8 \uc0ac\ub839\uad00\uc5d0 \uc790\uc2e0\uc758 \ucc98\uc870\uce74\uc778 \ubcf5\uac15\uc548 (\u798f\u5eb7\u5b89)\uc744 \uc784\uba85\ud558\uc5ec \uad6c\ub974\uce74\uad70\uc744 \ubab0\uc544\ub0b4\ub3c4\ub85d \ud558\uc600\ub2e4.", "paragraph_sentence": "\ub124\ud314\uc744 \uc810\ub839\ud560 \ub54c\ub294 \uc774\ubbf8 \ub124\ud314\uc758 \uad6c\ub974\uce74 \ubd80\uc871\uc774 \uccd0\ub4e4\uc5b4\uc640 1788\ub144(\uac74\ub96d 53\ub144) \ud2f0\ubca0\ud2b8 \ub0a8\ubd80\ub97c \uc810\ub839\ud558\uace0 \ub77c\uc0ac\ub97c \ud5a5\ud574 \uc9c4\uaca9\ud574\uc624\uace0 \uc788\uc5c8\ub2e4. \ud2f0\ubca0\ud2b8 \uc8fc\uc7ac \uccad\uad70\uc740 \ud310\uccb8 \ub77c\ub9c8 \ud150\ud398\uc774 \ub2c8\ub9c8\ub97c \ub0a9\uce58\ud558\uc5ec \ub2e4\ub978 \uacf3\uc5d0\uc11c \uc694\uaca9\ud560 \ud0dc\uc138\ub97c \uac16\ucd94\uc5c8\ub2e4. \ud55c\ud3b8 \uac74\ub96d\uc81c\ub294 \uc0ac\ucc9c\uc131\uc5d0 \uc8fc\uc7ac\ud558\ub358 \uad70\uc0ac\ub4e4\uc5d0\uac8c \ud2f0\ubca0\ud2b8 \ub0a8\ubd80\uc5d0 \uc788\ub294 \uad6c\ub974\uce74 \uad70\uc0ac\ub4e4\uc744 \ubab0\uc544\ub0b4\ub77c \uba85\ub839\ud558\uc600\uc73c\ub098 \uc774\ubbf8 \uad6c\ub974\uce74 \ubd80\uc871\uc740 \ud1f4\uac01\ud55c \uc0c1\ud0dc\uc600\ub2e4. \uadf8\ub7ec\ub098 1791\ub144(\uac74\ub96d 56\ub144) \uaca8\uc6b8, \uad6c\ub974\uce74\ub294 \ub2e4\uc2dc \ud2f0\ubca0\ud2b8\ub85c \uc9c4\uaca9\ud574 \ub4e4\uc5b4\uc654\ub2e4. \uc774\uc5d0 \ub9de\uc11c \uac74\ub96d\uc81c\ub294 \ud314\uae30\uad70\uacfc \ub179\uc601\uc758 \uad70\uc0ac\ub4e4\uc744 \ud30c\ubcd1\ud558\uace0 \uadf8 \uc0ac\ub839\uad00\uc5d0 \uc790\uc2e0\uc758 \ucc98\uc870\uce74\uc778 \ubcf5\uac15\uc548 (\u798f\u5eb7\u5b89)\uc744 \uc784\uba85\ud558\uc5ec \uad6c\ub974\uce74\uad70\uc744 \ubab0\uc544\ub0b4\ub3c4\ub85d \ud558\uc600\ub2e4. \ubcf5\uac15\uc548\uc758 \uad70\ub300\ub294 \uccad\ud574\uc131\uc73c\ub85c \uc9c4\uaca9\ud574 \ud2f0\ubca0\ud2b8 \ubd84\uc9c0\ub85c \ub4e4\uc5b4\uac00\ub824 \ud558\uc600\uc73c\ub098 \ub0a0\uc528\uac00 \ucda5\uace0 \uace0\uc9c0 \ubd84\ub300\ub77c\uc11c \ub208\uc774 \ub179\uc9c0 \uc54a\uc544 \ub0a0\uc774 \ud480\ub9b4 \ub54c\uae4c\uc9c0 \uae30\ub2e4\ub9ac\ub2e4\uac00 \ubd04\uc774 \ub418\uc5b4 \ub2e4\uc2dc \uc9c4\uaca9\ud55c \ud6c4 1792\ub144(\uac74\ub96d 57\ub144) \uc5ec\ub984\uc5d0 \uad6c\ub974\uce74\uad70\uc744 \uc12c\uba78\ud558\uace0 \uadf8\ub4e4\uc744 \ud788\ub9d0\ub77c\uc57c \uc0b0\ub9e5\uc758 \uce74\ud2b8\ub9cc\ub450 \uacc4\uace1\uc73c\ub85c\uae4c\uc9c0 \uc555\ubc15\ud558\uc600\ub2e4. 1793\ub144(\uac74\ub96d 58\ub144) \ubcf5\uac15\uc548\uc740 \uad6c\ub974\uce74\uc871\uc758 \ud56d\ubcf5\uc744 \ub9cc\uc8fc\uc2dd\uc73c\ub85c \ubc1b\uace0 \ucca0\uad70\ud558\uc5ec \ubd81\uacbd\uc73c\ub85c \ub3cc\uc544\uac14\ub2e4.", "paragraph_answer": "\ub124\ud314\uc744 \uc810\ub839\ud560 \ub54c\ub294 \uc774\ubbf8 \ub124\ud314\uc758 \uad6c\ub974\uce74 \ubd80\uc871\uc774 \uccd0\ub4e4\uc5b4\uc640 1788\ub144(\uac74\ub96d 53\ub144) \ud2f0\ubca0\ud2b8 \ub0a8\ubd80\ub97c \uc810\ub839\ud558\uace0 \ub77c\uc0ac\ub97c \ud5a5\ud574 \uc9c4\uaca9\ud574\uc624\uace0 \uc788\uc5c8\ub2e4. \ud2f0\ubca0\ud2b8 \uc8fc\uc7ac \uccad\uad70\uc740 \ud310\uccb8 \ub77c\ub9c8 \ud150\ud398\uc774 \ub2c8\ub9c8\ub97c \ub0a9\uce58\ud558\uc5ec \ub2e4\ub978 \uacf3\uc5d0\uc11c \uc694\uaca9\ud560 \ud0dc\uc138\ub97c \uac16\ucd94\uc5c8\ub2e4. \ud55c\ud3b8 \uac74\ub96d\uc81c\ub294 \uc0ac\ucc9c\uc131\uc5d0 \uc8fc\uc7ac\ud558\ub358 \uad70\uc0ac\ub4e4\uc5d0\uac8c \ud2f0\ubca0\ud2b8 \ub0a8\ubd80\uc5d0 \uc788\ub294 \uad6c\ub974\uce74 \uad70\uc0ac\ub4e4\uc744 \ubab0\uc544\ub0b4\ub77c \uba85\ub839\ud558\uc600\uc73c\ub098 \uc774\ubbf8 \uad6c\ub974\uce74 \ubd80\uc871\uc740 \ud1f4\uac01\ud55c \uc0c1\ud0dc\uc600\ub2e4. \uadf8\ub7ec\ub098 1791\ub144(\uac74\ub96d 56\ub144) \uaca8\uc6b8, \uad6c\ub974\uce74\ub294 \ub2e4\uc2dc \ud2f0\ubca0\ud2b8\ub85c \uc9c4\uaca9\ud574 \ub4e4\uc5b4\uc654\ub2e4. \uc774\uc5d0 \ub9de\uc11c \uac74\ub96d\uc81c\ub294 \ud314\uae30\uad70\uacfc \ub179\uc601\uc758 \uad70\uc0ac\ub4e4\uc744 \ud30c\ubcd1\ud558\uace0 \uadf8 \uc0ac\ub839\uad00\uc5d0 \uc790\uc2e0\uc758 \ucc98\uc870\uce74\uc778 \ubcf5\uac15\uc548 (\u798f\u5eb7\u5b89)\uc744 \uc784\uba85\ud558\uc5ec \uad6c\ub974\uce74\uad70\uc744 \ubab0\uc544\ub0b4\ub3c4\ub85d \ud558\uc600\ub2e4. \ubcf5\uac15\uc548\uc758 \uad70\ub300\ub294 \uccad\ud574\uc131\uc73c\ub85c \uc9c4\uaca9\ud574 \ud2f0\ubca0\ud2b8 \ubd84\uc9c0\ub85c \ub4e4\uc5b4\uac00\ub824 \ud558\uc600\uc73c\ub098 \ub0a0\uc528\uac00 \ucda5\uace0 \uace0\uc9c0 \ubd84\ub300\ub77c\uc11c \ub208\uc774 \ub179\uc9c0 \uc54a\uc544 \ub0a0\uc774 \ud480\ub9b4 \ub54c\uae4c\uc9c0 \uae30\ub2e4\ub9ac\ub2e4\uac00 \ubd04\uc774 \ub418\uc5b4 \ub2e4\uc2dc \uc9c4\uaca9\ud55c \ud6c4 1792\ub144(\uac74\ub96d 57\ub144) \uc5ec\ub984\uc5d0 \uad6c\ub974\uce74\uad70\uc744 \uc12c\uba78\ud558\uace0 \uadf8\ub4e4\uc744 \ud788\ub9d0\ub77c\uc57c \uc0b0\ub9e5\uc758 \uce74\ud2b8\ub9cc\ub450 \uacc4\uace1\uc73c\ub85c\uae4c\uc9c0 \uc555\ubc15\ud558\uc600\ub2e4. 1793\ub144(\uac74\ub96d 58\ub144) \ubcf5\uac15\uc548\uc740 \uad6c\ub974\uce74\uc871\uc758 \ud56d\ubcf5\uc744 \ub9cc\uc8fc\uc2dd\uc73c\ub85c \ubc1b\uace0 \ucca0\uad70\ud558\uc5ec \ubd81\uacbd\uc73c\ub85c \ub3cc\uc544\uac14\ub2e4.", "sentence_answer": "\uc774\uc5d0 \ub9de\uc11c \uac74\ub96d\uc81c\ub294 \ud314\uae30\uad70\uacfc \ub179\uc601\uc758 \uad70\uc0ac\ub4e4\uc744 \ud30c\ubcd1\ud558\uace0 \uadf8 \uc0ac\ub839\uad00\uc5d0 \uc790\uc2e0\uc758 \ucc98\uc870\uce74\uc778 \ubcf5\uac15\uc548 (\u798f\u5eb7\u5b89)\uc744 \uc784\uba85\ud558\uc5ec \uad6c\ub974\uce74\uad70\uc744 \ubab0\uc544\ub0b4\ub3c4\ub85d \ud558\uc600\ub2e4.", "paragraph_id": "6486904-12-1", "paragraph_question": "question: \ub124\ud314\uc758 \uad6c\ub974\uce74 \ubd80\uc871\uc744 \ubab0\uc544\ub0b4\uae30 \uc704\ud574 \uac74\ub96d\uc81c\uac00 \uc0ac\ub839\uad00\uc5d0 \uc784\uba85\ud55c \uc778\ubb3c\uc740?, context: \ub124\ud314\uc744 \uc810\ub839\ud560 \ub54c\ub294 \uc774\ubbf8 \ub124\ud314\uc758 \uad6c\ub974\uce74 \ubd80\uc871\uc774 \uccd0\ub4e4\uc5b4\uc640 1788\ub144(\uac74\ub96d 53\ub144) \ud2f0\ubca0\ud2b8 \ub0a8\ubd80\ub97c \uc810\ub839\ud558\uace0 \ub77c\uc0ac\ub97c \ud5a5\ud574 \uc9c4\uaca9\ud574\uc624\uace0 \uc788\uc5c8\ub2e4. \ud2f0\ubca0\ud2b8 \uc8fc\uc7ac \uccad\uad70\uc740 \ud310\uccb8 \ub77c\ub9c8 \ud150\ud398\uc774 \ub2c8\ub9c8\ub97c \ub0a9\uce58\ud558\uc5ec \ub2e4\ub978 \uacf3\uc5d0\uc11c \uc694\uaca9\ud560 \ud0dc\uc138\ub97c \uac16\ucd94\uc5c8\ub2e4. \ud55c\ud3b8 \uac74\ub96d\uc81c\ub294 \uc0ac\ucc9c\uc131\uc5d0 \uc8fc\uc7ac\ud558\ub358 \uad70\uc0ac\ub4e4\uc5d0\uac8c \ud2f0\ubca0\ud2b8 \ub0a8\ubd80\uc5d0 \uc788\ub294 \uad6c\ub974\uce74 \uad70\uc0ac\ub4e4\uc744 \ubab0\uc544\ub0b4\ub77c \uba85\ub839\ud558\uc600\uc73c\ub098 \uc774\ubbf8 \uad6c\ub974\uce74 \ubd80\uc871\uc740 \ud1f4\uac01\ud55c \uc0c1\ud0dc\uc600\ub2e4. \uadf8\ub7ec\ub098 1791\ub144(\uac74\ub96d 56\ub144) \uaca8\uc6b8, \uad6c\ub974\uce74\ub294 \ub2e4\uc2dc \ud2f0\ubca0\ud2b8\ub85c \uc9c4\uaca9\ud574 \ub4e4\uc5b4\uc654\ub2e4. \uc774\uc5d0 \ub9de\uc11c \uac74\ub96d\uc81c\ub294 \ud314\uae30\uad70\uacfc \ub179\uc601\uc758 \uad70\uc0ac\ub4e4\uc744 \ud30c\ubcd1\ud558\uace0 \uadf8 \uc0ac\ub839\uad00\uc5d0 \uc790\uc2e0\uc758 \ucc98\uc870\uce74\uc778 \ubcf5\uac15\uc548(\u798f\u5eb7\u5b89)\uc744 \uc784\uba85\ud558\uc5ec \uad6c\ub974\uce74\uad70\uc744 \ubab0\uc544\ub0b4\ub3c4\ub85d \ud558\uc600\ub2e4. \ubcf5\uac15\uc548\uc758 \uad70\ub300\ub294 \uccad\ud574\uc131\uc73c\ub85c \uc9c4\uaca9\ud574 \ud2f0\ubca0\ud2b8 \ubd84\uc9c0\ub85c \ub4e4\uc5b4\uac00\ub824 \ud558\uc600\uc73c\ub098 \ub0a0\uc528\uac00 \ucda5\uace0 \uace0\uc9c0 \ubd84\ub300\ub77c\uc11c \ub208\uc774 \ub179\uc9c0 \uc54a\uc544 \ub0a0\uc774 \ud480\ub9b4 \ub54c\uae4c\uc9c0 \uae30\ub2e4\ub9ac\ub2e4\uac00 \ubd04\uc774 \ub418\uc5b4 \ub2e4\uc2dc \uc9c4\uaca9\ud55c \ud6c4 1792\ub144(\uac74\ub96d 57\ub144) \uc5ec\ub984\uc5d0 \uad6c\ub974\uce74\uad70\uc744 \uc12c\uba78\ud558\uace0 \uadf8\ub4e4\uc744 \ud788\ub9d0\ub77c\uc57c \uc0b0\ub9e5\uc758 \uce74\ud2b8\ub9cc\ub450 \uacc4\uace1\uc73c\ub85c\uae4c\uc9c0 \uc555\ubc15\ud558\uc600\ub2e4. 1793\ub144(\uac74\ub96d 58\ub144) \ubcf5\uac15\uc548\uc740 \uad6c\ub974\uce74\uc871\uc758 \ud56d\ubcf5\uc744 \ub9cc\uc8fc\uc2dd\uc73c\ub85c \ubc1b\uace0 \ucca0\uad70\ud558\uc5ec \ubd81\uacbd\uc73c\ub85c \ub3cc\uc544\uac14\ub2e4."} {"question": "1993\ub144 3\uc6d4 2\uc77c \uc5d0\ub098\uc4f0 \uc720\ud0c0\uce74\ub294 \ubb34\uc2a8 \ud610\uc758\ub85c \uc778\ud574 \ud604\ud589\ubc94\uc73c\ub85c \uccb4\ud3ec\ub418\uc5c8\ub098?", "paragraph": "\ud558\uc9c0\ub9cc 1993\ub144 3\uc6d4 2\uc77c, \uac01\uc131\uc81c\ub97c \uc18c\uc9c0\ud558\ub2e4\uac00 \uc801\ubc1c\ub418\uba74\uc11c \uac01\uc131\uc81c \ub2e8\uc18d\ubc95 \uc704\ubc18 \ud610\uc758\ub85c \ud604\ud589\ubc94\uc73c\ub85c \uccb4\ud3ec\ub410\ub2e4. \uccb4\ud3ec\ub418\uae30 \uba70\uce60 \uc804\uae4c\uc9c0 \ub2db\ud3f0\ud584 \ud30c\uc774\ud130\uc2a4\uc758 \uc784\uc2dc \ucf54\uce58\ub97c \uc5ed\uc784\ud558\uace0 \uc788\uc5c8\ub2e4. \uadf8 \ud574\uc5d0 \uc5f4\ub9b0 \uc7ac\ud310\uc5d0\uc11c\ub294 \ub178\ubb34\ub77c \uac00\uc4f0\uc57c, \uc5d0\ubaa8\ud1a0 \ub2e4\ucf00\ub178\ub9ac \ub4f1\uc774 \uc99d\uc778\uc73c\ub85c \ucd9c\uc11d\ud588\ub2e4. \uae30\uc18c\ub41c \ubc94\uc8c4 \uc0ac\uc2e4\uc740 \uac01\uc131\uc81c \uc218\uc6a9\uc561 \uc57d 0.25ml\uc744 \uc67c\ud314\uc5d0 \uc8fc\uc0ac\ud55c \uac01\uc131\uc81c\uc758 \uc0ac\uc6a9 \uac74\uacfc, \uac01\uc131\uc81c \ucd1d 52.117g \ubc0f \uac01\uc131\uc81c \uc218\uc6a9\uc561 \uc57d 0.5ml\uc758 \uc18c\uc9c0\uc774\ub2e4. \ub610\ud55c \ud310\uacb0\uc5d0\uc11c\ub294 \u201c\ub2e4\ub7c9\uc758 \uac01\uc131\uc81c(\uc57d 100g)\uc744 \uc785\uc218\ud55c \ub370\ub2e4 \ubcf8\uac74\uc5d0\uc11c \uac80\uac70\ub418\uae30\uae4c\uc9c0 \uc218 \ub144\uac04\uc5d0 \uac78\uccd0 \uc0ac\uc6a9\ud558\uc5ec \uc9c0\ub09c\ud574 9\uc6d4 \ubb34\ub835\ubd80\ud130 \ub3d9\uac70\ud558\ub358 \uc5ec\uc131\uc5d0\uac8c\ub3c4 \uad8c\ud558\uc5ec \ud568\uaed8 \uc0ac\uc6a9\ud558\ub294 \ub4f1 \uadf8\ub807\uac8c\u201d \ubc94\ud589\uc5d0 \uc774\ub974\ub800\ub358 \uac83\uc774 \ubc1d\ud600\uc84c\ub2e4. \uadf8\ub9ac\uace0 \u201c\uac01\uc131\uc81c \uc18c\uc9c0\ub7c9\uc740 52g \uc815\ub3c4\uc640 \uc790\uc2e0\uc774 \uc0ac\uc6a9\ud558\uae30 \uc704\ud55c \uac83\uc73c\ub85c\ub294 \ub4dc\ubb3c\uac8c \ubcf4\uc77c \ub9cc\ud07c \ub300\ub7c9\u201d \ubc0f \u201c\ub3d9\uac70\ud55c \uc5ec\uc131\uc740 \uac01\uc131\uc81c \uc0ac\uc6a9 \ud610\uc758\ub85c \uc720\uc8c4 \ud310\uacb0\uc744 \ubc1b\uc558\uc73c\uba70, \uadf8 \uacc4\uae30\ub97c \ub9cc\ub4e4\uc5c8\ub2e4\u201d\ub77c\uace0 \uc9c0\uc801\ud558\uc5ec \u201c\ud615 \uc9d1\ud589\uc720\uc608\uc5d0 \uc0c1\ub2f9\ud558\ub294 \uc0ac\uc548\uc774\ub77c\uace0\ub294 \uc778\uc815\ud560 \uc218 \uc5c6\ub2e4\u201d\uba70 \uc9d5\uc5ed 2\ub144 4\uac1c\uc6d4\uc758 \uc2e4\ud615\uc774 \uc120\uace0\ub410\ub2e4. \uc774 \uc0ac\uac74\uc73c\ub85c \uc778\ud574 \uba85\uad6c\ud68c\uc5d0\uc11c\ub3c4 \uc790\uc9c4 \ud0c8\ud1f4\ud588\uace0(\uc774\uac83\uc740 \ub2f9\uc2dc \uba85\uad6c\ud68c \ud68c\uc7a5\uc774\ub358 \uac00\ub124\ub2e4 \ub9c8\uc0ac\uc774\uce58\uc5d0 \uc758\ud55c \uc81c\uba85 \uc870\uce58\uc778 \uac83\uc73c\ub85c \uc54c\ub824\uc838 \uc788\uc73c\uba70 \ud604\uc7ac\ub294 \ubcf5\uadc0) \uadf8 \ud6c4 1995\ub144 4\uc6d4\uc5d0 \uac00\uc11d\ubc29 \ub410\ub2e4.", "answer": "\uac01\uc131\uc81c \ub2e8\uc18d\ubc95 \uc704\ubc18 \ud610\uc758", "sentence": "\ud558\uc9c0\ub9cc 1993\ub144 3\uc6d4 2\uc77c, \uac01\uc131\uc81c\ub97c \uc18c\uc9c0\ud558\ub2e4\uac00 \uc801\ubc1c\ub418\uba74\uc11c \uac01\uc131\uc81c \ub2e8\uc18d\ubc95 \uc704\ubc18 \ud610\uc758 \ub85c \ud604\ud589\ubc94\uc73c\ub85c \uccb4\ud3ec\ub410\ub2e4.", "paragraph_sentence": " \ud558\uc9c0\ub9cc 1993\ub144 3\uc6d4 2\uc77c, \uac01\uc131\uc81c\ub97c \uc18c\uc9c0\ud558\ub2e4\uac00 \uc801\ubc1c\ub418\uba74\uc11c \uac01\uc131\uc81c \ub2e8\uc18d\ubc95 \uc704\ubc18 \ud610\uc758 \ub85c \ud604\ud589\ubc94\uc73c\ub85c \uccb4\ud3ec\ub410\ub2e4. \uccb4\ud3ec\ub418\uae30 \uba70\uce60 \uc804\uae4c\uc9c0 \ub2db\ud3f0\ud584 \ud30c\uc774\ud130\uc2a4\uc758 \uc784\uc2dc \ucf54\uce58\ub97c \uc5ed\uc784\ud558\uace0 \uc788\uc5c8\ub2e4. \uadf8 \ud574\uc5d0 \uc5f4\ub9b0 \uc7ac\ud310\uc5d0\uc11c\ub294 \ub178\ubb34\ub77c \uac00\uc4f0\uc57c, \uc5d0\ubaa8\ud1a0 \ub2e4\ucf00\ub178\ub9ac \ub4f1\uc774 \uc99d\uc778\uc73c\ub85c \ucd9c\uc11d\ud588\ub2e4. \uae30\uc18c\ub41c \ubc94\uc8c4 \uc0ac\uc2e4\uc740 \uac01\uc131\uc81c \uc218\uc6a9\uc561 \uc57d 0.25ml\uc744 \uc67c\ud314\uc5d0 \uc8fc\uc0ac\ud55c \uac01\uc131\uc81c\uc758 \uc0ac\uc6a9 \uac74\uacfc, \uac01\uc131\uc81c \ucd1d 52.117g \ubc0f \uac01\uc131\uc81c \uc218\uc6a9\uc561 \uc57d 0.5ml\uc758 \uc18c\uc9c0\uc774\ub2e4. \ub610\ud55c \ud310\uacb0\uc5d0\uc11c\ub294 \u201c\ub2e4\ub7c9\uc758 \uac01\uc131\uc81c(\uc57d 100g)\uc744 \uc785\uc218\ud55c \ub370\ub2e4 \ubcf8\uac74\uc5d0\uc11c \uac80\uac70\ub418\uae30\uae4c\uc9c0 \uc218 \ub144\uac04\uc5d0 \uac78\uccd0 \uc0ac\uc6a9\ud558\uc5ec \uc9c0\ub09c\ud574 9\uc6d4 \ubb34\ub835\ubd80\ud130 \ub3d9\uac70\ud558\ub358 \uc5ec\uc131\uc5d0\uac8c\ub3c4 \uad8c\ud558\uc5ec \ud568\uaed8 \uc0ac\uc6a9\ud558\ub294 \ub4f1 \uadf8\ub807\uac8c\u201d \ubc94\ud589\uc5d0 \uc774\ub974\ub800\ub358 \uac83\uc774 \ubc1d\ud600\uc84c\ub2e4. \uadf8\ub9ac\uace0 \u201c\uac01\uc131\uc81c \uc18c\uc9c0\ub7c9\uc740 52g \uc815\ub3c4\uc640 \uc790\uc2e0\uc774 \uc0ac\uc6a9\ud558\uae30 \uc704\ud55c \uac83\uc73c\ub85c\ub294 \ub4dc\ubb3c\uac8c \ubcf4\uc77c \ub9cc\ud07c \ub300\ub7c9\u201d \ubc0f \u201c\ub3d9\uac70\ud55c \uc5ec\uc131\uc740 \uac01\uc131\uc81c \uc0ac\uc6a9 \ud610\uc758\ub85c \uc720\uc8c4 \ud310\uacb0\uc744 \ubc1b\uc558\uc73c\uba70, \uadf8 \uacc4\uae30\ub97c \ub9cc\ub4e4\uc5c8\ub2e4\u201d\ub77c\uace0 \uc9c0\uc801\ud558\uc5ec \u201c\ud615 \uc9d1\ud589\uc720\uc608\uc5d0 \uc0c1\ub2f9\ud558\ub294 \uc0ac\uc548\uc774\ub77c\uace0\ub294 \uc778\uc815\ud560 \uc218 \uc5c6\ub2e4\u201d\uba70 \uc9d5\uc5ed 2\ub144 4\uac1c\uc6d4\uc758 \uc2e4\ud615\uc774 \uc120\uace0\ub410\ub2e4. \uc774 \uc0ac\uac74\uc73c\ub85c \uc778\ud574 \uba85\uad6c\ud68c\uc5d0\uc11c\ub3c4 \uc790\uc9c4 \ud0c8\ud1f4\ud588\uace0(\uc774\uac83\uc740 \ub2f9\uc2dc \uba85\uad6c\ud68c \ud68c\uc7a5\uc774\ub358 \uac00\ub124\ub2e4 \ub9c8\uc0ac\uc774\uce58\uc5d0 \uc758\ud55c \uc81c\uba85 \uc870\uce58\uc778 \uac83\uc73c\ub85c \uc54c\ub824\uc838 \uc788\uc73c\uba70 \ud604\uc7ac\ub294 \ubcf5\uadc0) \uadf8 \ud6c4 1995\ub144 4\uc6d4\uc5d0 \uac00\uc11d\ubc29 \ub410\ub2e4.", "paragraph_answer": "\ud558\uc9c0\ub9cc 1993\ub144 3\uc6d4 2\uc77c, \uac01\uc131\uc81c\ub97c \uc18c\uc9c0\ud558\ub2e4\uac00 \uc801\ubc1c\ub418\uba74\uc11c \uac01\uc131\uc81c \ub2e8\uc18d\ubc95 \uc704\ubc18 \ud610\uc758 \ub85c \ud604\ud589\ubc94\uc73c\ub85c \uccb4\ud3ec\ub410\ub2e4. \uccb4\ud3ec\ub418\uae30 \uba70\uce60 \uc804\uae4c\uc9c0 \ub2db\ud3f0\ud584 \ud30c\uc774\ud130\uc2a4\uc758 \uc784\uc2dc \ucf54\uce58\ub97c \uc5ed\uc784\ud558\uace0 \uc788\uc5c8\ub2e4. \uadf8 \ud574\uc5d0 \uc5f4\ub9b0 \uc7ac\ud310\uc5d0\uc11c\ub294 \ub178\ubb34\ub77c \uac00\uc4f0\uc57c, \uc5d0\ubaa8\ud1a0 \ub2e4\ucf00\ub178\ub9ac \ub4f1\uc774 \uc99d\uc778\uc73c\ub85c \ucd9c\uc11d\ud588\ub2e4. \uae30\uc18c\ub41c \ubc94\uc8c4 \uc0ac\uc2e4\uc740 \uac01\uc131\uc81c \uc218\uc6a9\uc561 \uc57d 0.25ml\uc744 \uc67c\ud314\uc5d0 \uc8fc\uc0ac\ud55c \uac01\uc131\uc81c\uc758 \uc0ac\uc6a9 \uac74\uacfc, \uac01\uc131\uc81c \ucd1d 52.117g \ubc0f \uac01\uc131\uc81c \uc218\uc6a9\uc561 \uc57d 0.5ml\uc758 \uc18c\uc9c0\uc774\ub2e4. \ub610\ud55c \ud310\uacb0\uc5d0\uc11c\ub294 \u201c\ub2e4\ub7c9\uc758 \uac01\uc131\uc81c(\uc57d 100g)\uc744 \uc785\uc218\ud55c \ub370\ub2e4 \ubcf8\uac74\uc5d0\uc11c \uac80\uac70\ub418\uae30\uae4c\uc9c0 \uc218 \ub144\uac04\uc5d0 \uac78\uccd0 \uc0ac\uc6a9\ud558\uc5ec \uc9c0\ub09c\ud574 9\uc6d4 \ubb34\ub835\ubd80\ud130 \ub3d9\uac70\ud558\ub358 \uc5ec\uc131\uc5d0\uac8c\ub3c4 \uad8c\ud558\uc5ec \ud568\uaed8 \uc0ac\uc6a9\ud558\ub294 \ub4f1 \uadf8\ub807\uac8c\u201d \ubc94\ud589\uc5d0 \uc774\ub974\ub800\ub358 \uac83\uc774 \ubc1d\ud600\uc84c\ub2e4. \uadf8\ub9ac\uace0 \u201c\uac01\uc131\uc81c \uc18c\uc9c0\ub7c9\uc740 52g \uc815\ub3c4\uc640 \uc790\uc2e0\uc774 \uc0ac\uc6a9\ud558\uae30 \uc704\ud55c \uac83\uc73c\ub85c\ub294 \ub4dc\ubb3c\uac8c \ubcf4\uc77c \ub9cc\ud07c \ub300\ub7c9\u201d \ubc0f \u201c\ub3d9\uac70\ud55c \uc5ec\uc131\uc740 \uac01\uc131\uc81c \uc0ac\uc6a9 \ud610\uc758\ub85c \uc720\uc8c4 \ud310\uacb0\uc744 \ubc1b\uc558\uc73c\uba70, \uadf8 \uacc4\uae30\ub97c \ub9cc\ub4e4\uc5c8\ub2e4\u201d\ub77c\uace0 \uc9c0\uc801\ud558\uc5ec \u201c\ud615 \uc9d1\ud589\uc720\uc608\uc5d0 \uc0c1\ub2f9\ud558\ub294 \uc0ac\uc548\uc774\ub77c\uace0\ub294 \uc778\uc815\ud560 \uc218 \uc5c6\ub2e4\u201d\uba70 \uc9d5\uc5ed 2\ub144 4\uac1c\uc6d4\uc758 \uc2e4\ud615\uc774 \uc120\uace0\ub410\ub2e4. \uc774 \uc0ac\uac74\uc73c\ub85c \uc778\ud574 \uba85\uad6c\ud68c\uc5d0\uc11c\ub3c4 \uc790\uc9c4 \ud0c8\ud1f4\ud588\uace0(\uc774\uac83\uc740 \ub2f9\uc2dc \uba85\uad6c\ud68c \ud68c\uc7a5\uc774\ub358 \uac00\ub124\ub2e4 \ub9c8\uc0ac\uc774\uce58\uc5d0 \uc758\ud55c \uc81c\uba85 \uc870\uce58\uc778 \uac83\uc73c\ub85c \uc54c\ub824\uc838 \uc788\uc73c\uba70 \ud604\uc7ac\ub294 \ubcf5\uadc0) \uadf8 \ud6c4 1995\ub144 4\uc6d4\uc5d0 \uac00\uc11d\ubc29 \ub410\ub2e4.", "sentence_answer": "\ud558\uc9c0\ub9cc 1993\ub144 3\uc6d4 2\uc77c, \uac01\uc131\uc81c\ub97c \uc18c\uc9c0\ud558\ub2e4\uac00 \uc801\ubc1c\ub418\uba74\uc11c \uac01\uc131\uc81c \ub2e8\uc18d\ubc95 \uc704\ubc18 \ud610\uc758 \ub85c \ud604\ud589\ubc94\uc73c\ub85c \uccb4\ud3ec\ub410\ub2e4.", "paragraph_id": "6523006-18-0", "paragraph_question": "question: 1993\ub144 3\uc6d4 2\uc77c \uc5d0\ub098\uc4f0 \uc720\ud0c0\uce74\ub294 \ubb34\uc2a8 \ud610\uc758\ub85c \uc778\ud574 \ud604\ud589\ubc94\uc73c\ub85c \uccb4\ud3ec\ub418\uc5c8\ub098?, context: \ud558\uc9c0\ub9cc 1993\ub144 3\uc6d4 2\uc77c, \uac01\uc131\uc81c\ub97c \uc18c\uc9c0\ud558\ub2e4\uac00 \uc801\ubc1c\ub418\uba74\uc11c \uac01\uc131\uc81c \ub2e8\uc18d\ubc95 \uc704\ubc18 \ud610\uc758\ub85c \ud604\ud589\ubc94\uc73c\ub85c \uccb4\ud3ec\ub410\ub2e4. \uccb4\ud3ec\ub418\uae30 \uba70\uce60 \uc804\uae4c\uc9c0 \ub2db\ud3f0\ud584 \ud30c\uc774\ud130\uc2a4\uc758 \uc784\uc2dc \ucf54\uce58\ub97c \uc5ed\uc784\ud558\uace0 \uc788\uc5c8\ub2e4. \uadf8 \ud574\uc5d0 \uc5f4\ub9b0 \uc7ac\ud310\uc5d0\uc11c\ub294 \ub178\ubb34\ub77c \uac00\uc4f0\uc57c, \uc5d0\ubaa8\ud1a0 \ub2e4\ucf00\ub178\ub9ac \ub4f1\uc774 \uc99d\uc778\uc73c\ub85c \ucd9c\uc11d\ud588\ub2e4. \uae30\uc18c\ub41c \ubc94\uc8c4 \uc0ac\uc2e4\uc740 \uac01\uc131\uc81c \uc218\uc6a9\uc561 \uc57d 0.25ml\uc744 \uc67c\ud314\uc5d0 \uc8fc\uc0ac\ud55c \uac01\uc131\uc81c\uc758 \uc0ac\uc6a9 \uac74\uacfc, \uac01\uc131\uc81c \ucd1d 52.117g \ubc0f \uac01\uc131\uc81c \uc218\uc6a9\uc561 \uc57d 0.5ml\uc758 \uc18c\uc9c0\uc774\ub2e4. \ub610\ud55c \ud310\uacb0\uc5d0\uc11c\ub294 \u201c\ub2e4\ub7c9\uc758 \uac01\uc131\uc81c(\uc57d 100g)\uc744 \uc785\uc218\ud55c \ub370\ub2e4 \ubcf8\uac74\uc5d0\uc11c \uac80\uac70\ub418\uae30\uae4c\uc9c0 \uc218 \ub144\uac04\uc5d0 \uac78\uccd0 \uc0ac\uc6a9\ud558\uc5ec \uc9c0\ub09c\ud574 9\uc6d4 \ubb34\ub835\ubd80\ud130 \ub3d9\uac70\ud558\ub358 \uc5ec\uc131\uc5d0\uac8c\ub3c4 \uad8c\ud558\uc5ec \ud568\uaed8 \uc0ac\uc6a9\ud558\ub294 \ub4f1 \uadf8\ub807\uac8c\u201d \ubc94\ud589\uc5d0 \uc774\ub974\ub800\ub358 \uac83\uc774 \ubc1d\ud600\uc84c\ub2e4. \uadf8\ub9ac\uace0 \u201c\uac01\uc131\uc81c \uc18c\uc9c0\ub7c9\uc740 52g \uc815\ub3c4\uc640 \uc790\uc2e0\uc774 \uc0ac\uc6a9\ud558\uae30 \uc704\ud55c \uac83\uc73c\ub85c\ub294 \ub4dc\ubb3c\uac8c \ubcf4\uc77c \ub9cc\ud07c \ub300\ub7c9\u201d \ubc0f \u201c\ub3d9\uac70\ud55c \uc5ec\uc131\uc740 \uac01\uc131\uc81c \uc0ac\uc6a9 \ud610\uc758\ub85c \uc720\uc8c4 \ud310\uacb0\uc744 \ubc1b\uc558\uc73c\uba70, \uadf8 \uacc4\uae30\ub97c \ub9cc\ub4e4\uc5c8\ub2e4\u201d\ub77c\uace0 \uc9c0\uc801\ud558\uc5ec \u201c\ud615 \uc9d1\ud589\uc720\uc608\uc5d0 \uc0c1\ub2f9\ud558\ub294 \uc0ac\uc548\uc774\ub77c\uace0\ub294 \uc778\uc815\ud560 \uc218 \uc5c6\ub2e4\u201d\uba70 \uc9d5\uc5ed 2\ub144 4\uac1c\uc6d4\uc758 \uc2e4\ud615\uc774 \uc120\uace0\ub410\ub2e4. \uc774 \uc0ac\uac74\uc73c\ub85c \uc778\ud574 \uba85\uad6c\ud68c\uc5d0\uc11c\ub3c4 \uc790\uc9c4 \ud0c8\ud1f4\ud588\uace0(\uc774\uac83\uc740 \ub2f9\uc2dc \uba85\uad6c\ud68c \ud68c\uc7a5\uc774\ub358 \uac00\ub124\ub2e4 \ub9c8\uc0ac\uc774\uce58\uc5d0 \uc758\ud55c \uc81c\uba85 \uc870\uce58\uc778 \uac83\uc73c\ub85c \uc54c\ub824\uc838 \uc788\uc73c\uba70 \ud604\uc7ac\ub294 \ubcf5\uadc0) \uadf8 \ud6c4 1995\ub144 4\uc6d4\uc5d0 \uac00\uc11d\ubc29 \ub410\ub2e4."} {"question": "\uc5ec\ub300\uc0dd\ucee4\ub9ac\uc5b4\uac1c\ubc1c\uc13c\ud130\uac00 \uc124\ub9bd\ub41c \uac83\uc740 \uc5b8\uc81c\uc778\uac00?", "paragraph": "\uc5ec\ub300\uc0dd\ucee4\ub9ac\uc5b4\uac1c\ubc1c\uc13c\ud130\ub294 \uc5ec\ub300\uc0dd\uc774 \uc9c0\uc18d\uc801\uc73c\ub85c \uacbd\uc81c\ud65c\ub3d9\uc744 \ud560 \uc218 \uc788\ub3c4\ub85d \ud3c9\ub4f1\ud55c \uc820\ub354\uc758\uc2dd\uc744 \uac15\ud654\ud558\uace0 \uc9c1\ubb34\ub2a5\ub825\uc744 \ud5a5\uc0c1\uc2dc\ud0a4\uba70 \ucee4\ub9ac\uc5b4\ub97c \uac1c\ubc1c\ud558\uc790\ub294 \ucde8\uc9c0\ub85c 2007\ub144 \uc124\ub9bd\ub418\uc5c8\ub2e4. \uccad\ub144\uc5ec\uc131\uc774 \uc9c0\uc5ed\uc0ac\ud68c\uc758 \ud575\uc2ec \uacbd\uc81c\ud65c\ub3d9 \uc778\uc7ac\ub85c\uc11c \uc9c0\uc18d\uc801\uc73c\ub85c \uc131\uc7a5\ud574\uac08 \uc218 \uc788\ub3c4\ub85d \ub2e4\uc591\ud55c \ud6c8\ub828\uacfc \uad50\uc721 \ud504\ub85c\uadf8\ub7a8\uc744 \uc81c\uacf5\ud558\uace0 \uc0ac\ud68c\uc758 \uc778\uc7ac\ub85c\uc11c \uc5ec\uc131\uc758 \ud559\ub825\uacfc \uc5ed\ub7c9\uc740 \uc810\ucc28 \ucee4\uc9c0\uace0 \uc788\uc9c0\ub9cc \ubfcc\ub9ac\uae4a\uc740 \uc131\ucc28\ubcc4\uc801 \ubb38\ud654\ub85c \ub2e4\uc591\ud55c \ubd84\uc57c\uc5d0\uc11c \ub9ac\ub354\uc2ed\uc744 \ubc1c\ud718\ud558\uae30 \uc27d\uc9c0 \uc54a\uace0 \uc9c0\uc18d\uc801\uc73c\ub85c \ucde8\uc5c5\uc744 \ud558\ub294\ub370 \uc5b4\ub824\uc6c0\uc774 \uc788\ub294 \uc5ec\uc131\ub4e4\uc758 \ucde8\uc5c5\uc744 \ub3d5\ub294 \uc77c\uc744 \ud55c\ub2e4. \uc804\ub0a8 \uc9c0\uc5ed \uc720\uc77c\ud55c \uccad\ub144\uc5ec\uc131\uacbd\ub825\uac1c\ubc1c\uc9c0\uc6d0\uc0ac\uc5c5 \uc218\ud589\uae30\uad00\uc73c\ub85c \uc9c0\uc815\ub41c \uc804\uad6d 13\uac1c \ub300\ud559 \uc911 \ud558\ub098\ub85c \uc5ec\uc131\uac00\uc871\ubd80 \uc8fc\uad00 '\uc5ec\ub300\uc0dd\uacbd\ub825\uac1c\ubc1c\uc9c0\uc6d0\uc0ac\uc5c5(\uc804 \uc5ec\ub300\uc0dd\ucee4\ub9ac\uc5b4\uac1c\ubc1c\uc9c0\uc6d0\uc0ac\uc5c5)'\uc5d0 \uc120\uc815\ub418\uc5b4 \uc5ec\uc131\uac00\uc871\ubd80\uc640 \uc804\ub77c\ub0a8\ub3c4, \uc21c\ucc9c\uc2dc\uc758 \ub4e0\ub4e0\ud55c \uc9c0\uc6d0\uc744 \ubc1b\uace0 \uc5ec\ud559\uc0dd\ub4e4\uc774 \uc9c1\uc5c5\uc801 \uacbd\ub825\uc744 \uc9c0\uc18d\uc801\uc73c\ub85c \uc313\uc544 \uc0ac\ud68c\uc5d0 \uae30\uc5ec\ud560 \uc218 \uc788\ub3c4\ub85d \ub2e4\uc591\ud55c \ud504\ub85c\uadf8\ub7a8\uc744 \uc81c\uacf5\ud55c\ub2e4.", "answer": "2007\ub144", "sentence": "\uc5ec\ub300\uc0dd\ucee4\ub9ac\uc5b4\uac1c\ubc1c\uc13c\ud130\ub294 \uc5ec\ub300\uc0dd\uc774 \uc9c0\uc18d\uc801\uc73c\ub85c \uacbd\uc81c\ud65c\ub3d9\uc744 \ud560 \uc218 \uc788\ub3c4\ub85d \ud3c9\ub4f1\ud55c \uc820\ub354\uc758\uc2dd\uc744 \uac15\ud654\ud558\uace0 \uc9c1\ubb34\ub2a5\ub825\uc744 \ud5a5\uc0c1\uc2dc\ud0a4\uba70 \ucee4\ub9ac\uc5b4\ub97c \uac1c\ubc1c\ud558\uc790\ub294 \ucde8\uc9c0\ub85c 2007\ub144 \uc124\ub9bd\ub418\uc5c8\ub2e4.", "paragraph_sentence": " \uc5ec\ub300\uc0dd\ucee4\ub9ac\uc5b4\uac1c\ubc1c\uc13c\ud130\ub294 \uc5ec\ub300\uc0dd\uc774 \uc9c0\uc18d\uc801\uc73c\ub85c \uacbd\uc81c\ud65c\ub3d9\uc744 \ud560 \uc218 \uc788\ub3c4\ub85d \ud3c9\ub4f1\ud55c \uc820\ub354\uc758\uc2dd\uc744 \uac15\ud654\ud558\uace0 \uc9c1\ubb34\ub2a5\ub825\uc744 \ud5a5\uc0c1\uc2dc\ud0a4\uba70 \ucee4\ub9ac\uc5b4\ub97c \uac1c\ubc1c\ud558\uc790\ub294 \ucde8\uc9c0\ub85c 2007\ub144 \uc124\ub9bd\ub418\uc5c8\ub2e4. \uccad\ub144\uc5ec\uc131\uc774 \uc9c0\uc5ed\uc0ac\ud68c\uc758 \ud575\uc2ec \uacbd\uc81c\ud65c\ub3d9 \uc778\uc7ac\ub85c\uc11c \uc9c0\uc18d\uc801\uc73c\ub85c \uc131\uc7a5\ud574\uac08 \uc218 \uc788\ub3c4\ub85d \ub2e4\uc591\ud55c \ud6c8\ub828\uacfc \uad50\uc721 \ud504\ub85c\uadf8\ub7a8\uc744 \uc81c\uacf5\ud558\uace0 \uc0ac\ud68c\uc758 \uc778\uc7ac\ub85c\uc11c \uc5ec\uc131\uc758 \ud559\ub825\uacfc \uc5ed\ub7c9\uc740 \uc810\ucc28 \ucee4\uc9c0\uace0 \uc788\uc9c0\ub9cc \ubfcc\ub9ac\uae4a\uc740 \uc131\ucc28\ubcc4\uc801 \ubb38\ud654\ub85c \ub2e4\uc591\ud55c \ubd84\uc57c\uc5d0\uc11c \ub9ac\ub354\uc2ed\uc744 \ubc1c\ud718\ud558\uae30 \uc27d\uc9c0 \uc54a\uace0 \uc9c0\uc18d\uc801\uc73c\ub85c \ucde8\uc5c5\uc744 \ud558\ub294\ub370 \uc5b4\ub824\uc6c0\uc774 \uc788\ub294 \uc5ec\uc131\ub4e4\uc758 \ucde8\uc5c5\uc744 \ub3d5\ub294 \uc77c\uc744 \ud55c\ub2e4. \uc804\ub0a8 \uc9c0\uc5ed \uc720\uc77c\ud55c \uccad\ub144\uc5ec\uc131\uacbd\ub825\uac1c\ubc1c\uc9c0\uc6d0\uc0ac\uc5c5 \uc218\ud589\uae30\uad00\uc73c\ub85c \uc9c0\uc815\ub41c \uc804\uad6d 13\uac1c \ub300\ud559 \uc911 \ud558\ub098\ub85c \uc5ec\uc131\uac00\uc871\ubd80 \uc8fc\uad00 '\uc5ec\ub300\uc0dd\uacbd\ub825\uac1c\ubc1c\uc9c0\uc6d0\uc0ac\uc5c5(\uc804 \uc5ec\ub300\uc0dd\ucee4\ub9ac\uc5b4\uac1c\ubc1c\uc9c0\uc6d0\uc0ac\uc5c5)'\uc5d0 \uc120\uc815\ub418\uc5b4 \uc5ec\uc131\uac00\uc871\ubd80\uc640 \uc804\ub77c\ub0a8\ub3c4, \uc21c\ucc9c\uc2dc\uc758 \ub4e0\ub4e0\ud55c \uc9c0\uc6d0\uc744 \ubc1b\uace0 \uc5ec\ud559\uc0dd\ub4e4\uc774 \uc9c1\uc5c5\uc801 \uacbd\ub825\uc744 \uc9c0\uc18d\uc801\uc73c\ub85c \uc313\uc544 \uc0ac\ud68c\uc5d0 \uae30\uc5ec\ud560 \uc218 \uc788\ub3c4\ub85d \ub2e4\uc591\ud55c \ud504\ub85c\uadf8\ub7a8\uc744 \uc81c\uacf5\ud55c\ub2e4.", "paragraph_answer": "\uc5ec\ub300\uc0dd\ucee4\ub9ac\uc5b4\uac1c\ubc1c\uc13c\ud130\ub294 \uc5ec\ub300\uc0dd\uc774 \uc9c0\uc18d\uc801\uc73c\ub85c \uacbd\uc81c\ud65c\ub3d9\uc744 \ud560 \uc218 \uc788\ub3c4\ub85d \ud3c9\ub4f1\ud55c \uc820\ub354\uc758\uc2dd\uc744 \uac15\ud654\ud558\uace0 \uc9c1\ubb34\ub2a5\ub825\uc744 \ud5a5\uc0c1\uc2dc\ud0a4\uba70 \ucee4\ub9ac\uc5b4\ub97c \uac1c\ubc1c\ud558\uc790\ub294 \ucde8\uc9c0\ub85c 2007\ub144 \uc124\ub9bd\ub418\uc5c8\ub2e4. \uccad\ub144\uc5ec\uc131\uc774 \uc9c0\uc5ed\uc0ac\ud68c\uc758 \ud575\uc2ec \uacbd\uc81c\ud65c\ub3d9 \uc778\uc7ac\ub85c\uc11c \uc9c0\uc18d\uc801\uc73c\ub85c \uc131\uc7a5\ud574\uac08 \uc218 \uc788\ub3c4\ub85d \ub2e4\uc591\ud55c \ud6c8\ub828\uacfc \uad50\uc721 \ud504\ub85c\uadf8\ub7a8\uc744 \uc81c\uacf5\ud558\uace0 \uc0ac\ud68c\uc758 \uc778\uc7ac\ub85c\uc11c \uc5ec\uc131\uc758 \ud559\ub825\uacfc \uc5ed\ub7c9\uc740 \uc810\ucc28 \ucee4\uc9c0\uace0 \uc788\uc9c0\ub9cc \ubfcc\ub9ac\uae4a\uc740 \uc131\ucc28\ubcc4\uc801 \ubb38\ud654\ub85c \ub2e4\uc591\ud55c \ubd84\uc57c\uc5d0\uc11c \ub9ac\ub354\uc2ed\uc744 \ubc1c\ud718\ud558\uae30 \uc27d\uc9c0 \uc54a\uace0 \uc9c0\uc18d\uc801\uc73c\ub85c \ucde8\uc5c5\uc744 \ud558\ub294\ub370 \uc5b4\ub824\uc6c0\uc774 \uc788\ub294 \uc5ec\uc131\ub4e4\uc758 \ucde8\uc5c5\uc744 \ub3d5\ub294 \uc77c\uc744 \ud55c\ub2e4. \uc804\ub0a8 \uc9c0\uc5ed \uc720\uc77c\ud55c \uccad\ub144\uc5ec\uc131\uacbd\ub825\uac1c\ubc1c\uc9c0\uc6d0\uc0ac\uc5c5 \uc218\ud589\uae30\uad00\uc73c\ub85c \uc9c0\uc815\ub41c \uc804\uad6d 13\uac1c \ub300\ud559 \uc911 \ud558\ub098\ub85c \uc5ec\uc131\uac00\uc871\ubd80 \uc8fc\uad00 '\uc5ec\ub300\uc0dd\uacbd\ub825\uac1c\ubc1c\uc9c0\uc6d0\uc0ac\uc5c5(\uc804 \uc5ec\ub300\uc0dd\ucee4\ub9ac\uc5b4\uac1c\ubc1c\uc9c0\uc6d0\uc0ac\uc5c5)'\uc5d0 \uc120\uc815\ub418\uc5b4 \uc5ec\uc131\uac00\uc871\ubd80\uc640 \uc804\ub77c\ub0a8\ub3c4, \uc21c\ucc9c\uc2dc\uc758 \ub4e0\ub4e0\ud55c \uc9c0\uc6d0\uc744 \ubc1b\uace0 \uc5ec\ud559\uc0dd\ub4e4\uc774 \uc9c1\uc5c5\uc801 \uacbd\ub825\uc744 \uc9c0\uc18d\uc801\uc73c\ub85c \uc313\uc544 \uc0ac\ud68c\uc5d0 \uae30\uc5ec\ud560 \uc218 \uc788\ub3c4\ub85d \ub2e4\uc591\ud55c \ud504\ub85c\uadf8\ub7a8\uc744 \uc81c\uacf5\ud55c\ub2e4.", "sentence_answer": "\uc5ec\ub300\uc0dd\ucee4\ub9ac\uc5b4\uac1c\ubc1c\uc13c\ud130\ub294 \uc5ec\ub300\uc0dd\uc774 \uc9c0\uc18d\uc801\uc73c\ub85c \uacbd\uc81c\ud65c\ub3d9\uc744 \ud560 \uc218 \uc788\ub3c4\ub85d \ud3c9\ub4f1\ud55c \uc820\ub354\uc758\uc2dd\uc744 \uac15\ud654\ud558\uace0 \uc9c1\ubb34\ub2a5\ub825\uc744 \ud5a5\uc0c1\uc2dc\ud0a4\uba70 \ucee4\ub9ac\uc5b4\ub97c \uac1c\ubc1c\ud558\uc790\ub294 \ucde8\uc9c0\ub85c 2007\ub144 \uc124\ub9bd\ub418\uc5c8\ub2e4.", "paragraph_id": "6532634-10-0", "paragraph_question": "question: \uc5ec\ub300\uc0dd\ucee4\ub9ac\uc5b4\uac1c\ubc1c\uc13c\ud130\uac00 \uc124\ub9bd\ub41c \uac83\uc740 \uc5b8\uc81c\uc778\uac00?, context: \uc5ec\ub300\uc0dd\ucee4\ub9ac\uc5b4\uac1c\ubc1c\uc13c\ud130\ub294 \uc5ec\ub300\uc0dd\uc774 \uc9c0\uc18d\uc801\uc73c\ub85c \uacbd\uc81c\ud65c\ub3d9\uc744 \ud560 \uc218 \uc788\ub3c4\ub85d \ud3c9\ub4f1\ud55c \uc820\ub354\uc758\uc2dd\uc744 \uac15\ud654\ud558\uace0 \uc9c1\ubb34\ub2a5\ub825\uc744 \ud5a5\uc0c1\uc2dc\ud0a4\uba70 \ucee4\ub9ac\uc5b4\ub97c \uac1c\ubc1c\ud558\uc790\ub294 \ucde8\uc9c0\ub85c 2007\ub144 \uc124\ub9bd\ub418\uc5c8\ub2e4. \uccad\ub144\uc5ec\uc131\uc774 \uc9c0\uc5ed\uc0ac\ud68c\uc758 \ud575\uc2ec \uacbd\uc81c\ud65c\ub3d9 \uc778\uc7ac\ub85c\uc11c \uc9c0\uc18d\uc801\uc73c\ub85c \uc131\uc7a5\ud574\uac08 \uc218 \uc788\ub3c4\ub85d \ub2e4\uc591\ud55c \ud6c8\ub828\uacfc \uad50\uc721 \ud504\ub85c\uadf8\ub7a8\uc744 \uc81c\uacf5\ud558\uace0 \uc0ac\ud68c\uc758 \uc778\uc7ac\ub85c\uc11c \uc5ec\uc131\uc758 \ud559\ub825\uacfc \uc5ed\ub7c9\uc740 \uc810\ucc28 \ucee4\uc9c0\uace0 \uc788\uc9c0\ub9cc \ubfcc\ub9ac\uae4a\uc740 \uc131\ucc28\ubcc4\uc801 \ubb38\ud654\ub85c \ub2e4\uc591\ud55c \ubd84\uc57c\uc5d0\uc11c \ub9ac\ub354\uc2ed\uc744 \ubc1c\ud718\ud558\uae30 \uc27d\uc9c0 \uc54a\uace0 \uc9c0\uc18d\uc801\uc73c\ub85c \ucde8\uc5c5\uc744 \ud558\ub294\ub370 \uc5b4\ub824\uc6c0\uc774 \uc788\ub294 \uc5ec\uc131\ub4e4\uc758 \ucde8\uc5c5\uc744 \ub3d5\ub294 \uc77c\uc744 \ud55c\ub2e4. \uc804\ub0a8 \uc9c0\uc5ed \uc720\uc77c\ud55c \uccad\ub144\uc5ec\uc131\uacbd\ub825\uac1c\ubc1c\uc9c0\uc6d0\uc0ac\uc5c5 \uc218\ud589\uae30\uad00\uc73c\ub85c \uc9c0\uc815\ub41c \uc804\uad6d 13\uac1c \ub300\ud559 \uc911 \ud558\ub098\ub85c \uc5ec\uc131\uac00\uc871\ubd80 \uc8fc\uad00 '\uc5ec\ub300\uc0dd\uacbd\ub825\uac1c\ubc1c\uc9c0\uc6d0\uc0ac\uc5c5(\uc804 \uc5ec\ub300\uc0dd\ucee4\ub9ac\uc5b4\uac1c\ubc1c\uc9c0\uc6d0\uc0ac\uc5c5)'\uc5d0 \uc120\uc815\ub418\uc5b4 \uc5ec\uc131\uac00\uc871\ubd80\uc640 \uc804\ub77c\ub0a8\ub3c4, \uc21c\ucc9c\uc2dc\uc758 \ub4e0\ub4e0\ud55c \uc9c0\uc6d0\uc744 \ubc1b\uace0 \uc5ec\ud559\uc0dd\ub4e4\uc774 \uc9c1\uc5c5\uc801 \uacbd\ub825\uc744 \uc9c0\uc18d\uc801\uc73c\ub85c \uc313\uc544 \uc0ac\ud68c\uc5d0 \uae30\uc5ec\ud560 \uc218 \uc788\ub3c4\ub85d \ub2e4\uc591\ud55c \ud504\ub85c\uadf8\ub7a8\uc744 \uc81c\uacf5\ud55c\ub2e4."} {"question": "1895\ub144 \uae08\uace1\ub9ac\uc5d0\uc11c \ubd84\ud65c \ub41c \uacf3\uc740?", "paragraph": "\uae08\ud559\ub9ac\ub294 \uc870\uc120\uc2dc\ub300\uc5d0 \ub178\uc9c0\uba74 \uae08\uace1\ub9ac(\u91d1\u8c37\u91cc)\uc640 \ub9c8\uc804\ub9ac(\u9ebb\u7530\u91cc)\uc758 2\uac1c\ub9ac\uac00 \uc788\uc5c8\uc73c\ub098, 1895\ub144(\uace0\uc88532)\ud589\uc815\uad6c\uc5ed \uac1c\ud3b8\uc73c\ub85c \ub178\uc9c0\uba74\uc5d0\uc11c \uc601\ud48d\ucc3d\uba74\uc73c\ub85c \uc18c\uc18d\uc774 \ubc14\ub00c\uba74\uc11c \uc774\uc804\uc758 \ub9c8\uc804\ub9ac\uc5d0\uc11c \ucd94\ud559\ub9ac(\u79cb\u9db4\u91cc)\uac00 \ubd84\ud65c\ub418\uace0, \ub610 \ub2e4\ub978 \ub9ac\uc600\ub358 \uae08\uace1\ub9ac\uc5d0\uc11c \uc18c\uae38\ub9ac(\u5c0f\u5409\u91cc)\ub85c \ubd84\ud65c\ub418\uc5b4 \ucd1d 4\uac1c \ub9c8\uc744\uc774 \ub418\uc5c8\ub2e4. 1914\ub144\uc758 \ud589\uc815\uad6c\uc5ed \uac1c\ud3b8 \ub54c\uc5d0 \uc778\uadfc\uc758 \uc5ec\ub7ec \ub9ac(\u91cc)\ub97c \ud569\uccd0 \ud558\ub098\uc758 \ub3d9\ub9ac\ub85c \ud1b5\ud569\ud558\uc600\ub294\ub370 \uc601\ud48d\ucc3d\uba74\uc758 \ub9c8\uc804\ub9ac, \ucd94\ud559\ub9ac, \uae08\uace1\ub9ac, \uc18c\uae38\ub9ac \uadf8\ub9ac\uace0 \ubb38\ud604\uba74\uc758 \uc6b0\uae38\ub9ac\ub97c \ud569\uccd0\uc11c \ud55c \ub3d9\ub9ac\ub85c \ub9cc\ub4e4\uc5c8\ub2e4. \uae08\ud559\ub9ac\ub780 \uc774\ub984\uc740 \uae08\uace1\ub9ac(\u91d1\u8c37\u91cc)\uc758 '\uae08(\u91d1)'\uc790\ub97c \ucde8\ud558\uace0 \ucd94\ud559\ub9ac(\u79cb\u9db4\u91cc)\uc5d0\uc11c '\ud559(\u9db4)'\uc790\ub97c \ucde8\ud574\uc11c \uae08\ud559\ub9ac(\u91d1\u9db4\u91cc)\ub85c \uc815\ud574 \ud604\uc7ac\uc758 \uae08\ud559\ub9ac\uac00 \ub418\uc5c8\ub2e4. \uae08\ud559\ub9ac\uc758 \uc790\uc5f0\ud658\uacbd\uc740 \ub17c\ubcf4\ub2e4 \ubc2d\uc774 \ub9ce\uace0 \ube44\uc625\ud558\uc5ec \uc77c\ucc0d\ubd80\ud130 \uc0bc\uacfc \uc57d\ucd08\uc758 \uc7ac\ubc30\uac00 \ubc1c\ub2ec\ud558\uc600\ub2e4.", "answer": "\uc18c\uae38\ub9ac", "sentence": "\uae08\uace1\ub9ac\uc5d0\uc11c \uc18c\uae38\ub9ac (\u5c0f\u5409\u91cc)\ub85c \ubd84\ud65c\ub418\uc5b4 \ucd1d 4\uac1c \ub9c8\uc744\uc774 \ub418\uc5c8\ub2e4.", "paragraph_sentence": "\uae08\ud559\ub9ac\ub294 \uc870\uc120\uc2dc\ub300\uc5d0 \ub178\uc9c0\uba74 \uae08\uace1\ub9ac(\u91d1\u8c37\u91cc)\uc640 \ub9c8\uc804\ub9ac(\u9ebb\u7530\u91cc)\uc758 2\uac1c\ub9ac\uac00 \uc788\uc5c8\uc73c\ub098, 1895\ub144(\uace0\uc88532)\ud589\uc815\uad6c\uc5ed \uac1c\ud3b8\uc73c\ub85c \ub178\uc9c0\uba74\uc5d0\uc11c \uc601\ud48d\ucc3d\uba74\uc73c\ub85c \uc18c\uc18d\uc774 \ubc14\ub00c\uba74\uc11c \uc774\uc804\uc758 \ub9c8\uc804\ub9ac\uc5d0\uc11c \ucd94\ud559\ub9ac(\u79cb\u9db4\u91cc)\uac00 \ubd84\ud65c\ub418\uace0, \ub610 \ub2e4\ub978 \ub9ac\uc600\ub358 \uae08\uace1\ub9ac\uc5d0\uc11c \uc18c\uae38\ub9ac (\u5c0f\u5409\u91cc)\ub85c \ubd84\ud65c\ub418\uc5b4 \ucd1d 4\uac1c \ub9c8\uc744\uc774 \ub418\uc5c8\ub2e4. 1914\ub144\uc758 \ud589\uc815\uad6c\uc5ed \uac1c\ud3b8 \ub54c\uc5d0 \uc778\uadfc\uc758 \uc5ec\ub7ec \ub9ac(\u91cc)\ub97c \ud569\uccd0 \ud558\ub098\uc758 \ub3d9\ub9ac\ub85c \ud1b5\ud569\ud558\uc600\ub294\ub370 \uc601\ud48d\ucc3d\uba74\uc758 \ub9c8\uc804\ub9ac, \ucd94\ud559\ub9ac, \uae08\uace1\ub9ac, \uc18c\uae38\ub9ac \uadf8\ub9ac\uace0 \ubb38\ud604\uba74\uc758 \uc6b0\uae38\ub9ac\ub97c \ud569\uccd0\uc11c \ud55c \ub3d9\ub9ac\ub85c \ub9cc\ub4e4\uc5c8\ub2e4. \uae08\ud559\ub9ac\ub780 \uc774\ub984\uc740 \uae08\uace1\ub9ac(\u91d1\u8c37\u91cc)\uc758 '\uae08(\u91d1)'\uc790\ub97c \ucde8\ud558\uace0 \ucd94\ud559\ub9ac(\u79cb\u9db4\u91cc)\uc5d0\uc11c '\ud559(\u9db4)'\uc790\ub97c \ucde8\ud574\uc11c \uae08\ud559\ub9ac(\u91d1\u9db4\u91cc)\ub85c \uc815\ud574 \ud604\uc7ac\uc758 \uae08\ud559\ub9ac\uac00 \ub418\uc5c8\ub2e4. \uae08\ud559\ub9ac\uc758 \uc790\uc5f0\ud658\uacbd\uc740 \ub17c\ubcf4\ub2e4 \ubc2d\uc774 \ub9ce\uace0 \ube44\uc625\ud558\uc5ec \uc77c\ucc0d\ubd80\ud130 \uc0bc\uacfc \uc57d\ucd08\uc758 \uc7ac\ubc30\uac00 \ubc1c\ub2ec\ud558\uc600\ub2e4.", "paragraph_answer": "\uae08\ud559\ub9ac\ub294 \uc870\uc120\uc2dc\ub300\uc5d0 \ub178\uc9c0\uba74 \uae08\uace1\ub9ac(\u91d1\u8c37\u91cc)\uc640 \ub9c8\uc804\ub9ac(\u9ebb\u7530\u91cc)\uc758 2\uac1c\ub9ac\uac00 \uc788\uc5c8\uc73c\ub098, 1895\ub144(\uace0\uc88532)\ud589\uc815\uad6c\uc5ed \uac1c\ud3b8\uc73c\ub85c \ub178\uc9c0\uba74\uc5d0\uc11c \uc601\ud48d\ucc3d\uba74\uc73c\ub85c \uc18c\uc18d\uc774 \ubc14\ub00c\uba74\uc11c \uc774\uc804\uc758 \ub9c8\uc804\ub9ac\uc5d0\uc11c \ucd94\ud559\ub9ac(\u79cb\u9db4\u91cc)\uac00 \ubd84\ud65c\ub418\uace0, \ub610 \ub2e4\ub978 \ub9ac\uc600\ub358 \uae08\uace1\ub9ac\uc5d0\uc11c \uc18c\uae38\ub9ac (\u5c0f\u5409\u91cc)\ub85c \ubd84\ud65c\ub418\uc5b4 \ucd1d 4\uac1c \ub9c8\uc744\uc774 \ub418\uc5c8\ub2e4. 1914\ub144\uc758 \ud589\uc815\uad6c\uc5ed \uac1c\ud3b8 \ub54c\uc5d0 \uc778\uadfc\uc758 \uc5ec\ub7ec \ub9ac(\u91cc)\ub97c \ud569\uccd0 \ud558\ub098\uc758 \ub3d9\ub9ac\ub85c \ud1b5\ud569\ud558\uc600\ub294\ub370 \uc601\ud48d\ucc3d\uba74\uc758 \ub9c8\uc804\ub9ac, \ucd94\ud559\ub9ac, \uae08\uace1\ub9ac, \uc18c\uae38\ub9ac \uadf8\ub9ac\uace0 \ubb38\ud604\uba74\uc758 \uc6b0\uae38\ub9ac\ub97c \ud569\uccd0\uc11c \ud55c \ub3d9\ub9ac\ub85c \ub9cc\ub4e4\uc5c8\ub2e4. \uae08\ud559\ub9ac\ub780 \uc774\ub984\uc740 \uae08\uace1\ub9ac(\u91d1\u8c37\u91cc)\uc758 '\uae08(\u91d1)'\uc790\ub97c \ucde8\ud558\uace0 \ucd94\ud559\ub9ac(\u79cb\u9db4\u91cc)\uc5d0\uc11c '\ud559(\u9db4)'\uc790\ub97c \ucde8\ud574\uc11c \uae08\ud559\ub9ac(\u91d1\u9db4\u91cc)\ub85c \uc815\ud574 \ud604\uc7ac\uc758 \uae08\ud559\ub9ac\uac00 \ub418\uc5c8\ub2e4. \uae08\ud559\ub9ac\uc758 \uc790\uc5f0\ud658\uacbd\uc740 \ub17c\ubcf4\ub2e4 \ubc2d\uc774 \ub9ce\uace0 \ube44\uc625\ud558\uc5ec \uc77c\ucc0d\ubd80\ud130 \uc0bc\uacfc \uc57d\ucd08\uc758 \uc7ac\ubc30\uac00 \ubc1c\ub2ec\ud558\uc600\ub2e4.", "sentence_answer": "\uae08\uace1\ub9ac\uc5d0\uc11c \uc18c\uae38\ub9ac (\u5c0f\u5409\u91cc)\ub85c \ubd84\ud65c\ub418\uc5b4 \ucd1d 4\uac1c \ub9c8\uc744\uc774 \ub418\uc5c8\ub2e4.", "paragraph_id": "6566707-1-1", "paragraph_question": "question: 1895\ub144 \uae08\uace1\ub9ac\uc5d0\uc11c \ubd84\ud65c \ub41c \uacf3\uc740?, context: \uae08\ud559\ub9ac\ub294 \uc870\uc120\uc2dc\ub300\uc5d0 \ub178\uc9c0\uba74 \uae08\uace1\ub9ac(\u91d1\u8c37\u91cc)\uc640 \ub9c8\uc804\ub9ac(\u9ebb\u7530\u91cc)\uc758 2\uac1c\ub9ac\uac00 \uc788\uc5c8\uc73c\ub098, 1895\ub144(\uace0\uc88532)\ud589\uc815\uad6c\uc5ed \uac1c\ud3b8\uc73c\ub85c \ub178\uc9c0\uba74\uc5d0\uc11c \uc601\ud48d\ucc3d\uba74\uc73c\ub85c \uc18c\uc18d\uc774 \ubc14\ub00c\uba74\uc11c \uc774\uc804\uc758 \ub9c8\uc804\ub9ac\uc5d0\uc11c \ucd94\ud559\ub9ac(\u79cb\u9db4\u91cc)\uac00 \ubd84\ud65c\ub418\uace0, \ub610 \ub2e4\ub978 \ub9ac\uc600\ub358 \uae08\uace1\ub9ac\uc5d0\uc11c \uc18c\uae38\ub9ac(\u5c0f\u5409\u91cc)\ub85c \ubd84\ud65c\ub418\uc5b4 \ucd1d 4\uac1c \ub9c8\uc744\uc774 \ub418\uc5c8\ub2e4. 1914\ub144\uc758 \ud589\uc815\uad6c\uc5ed \uac1c\ud3b8 \ub54c\uc5d0 \uc778\uadfc\uc758 \uc5ec\ub7ec \ub9ac(\u91cc)\ub97c \ud569\uccd0 \ud558\ub098\uc758 \ub3d9\ub9ac\ub85c \ud1b5\ud569\ud558\uc600\ub294\ub370 \uc601\ud48d\ucc3d\uba74\uc758 \ub9c8\uc804\ub9ac, \ucd94\ud559\ub9ac, \uae08\uace1\ub9ac, \uc18c\uae38\ub9ac \uadf8\ub9ac\uace0 \ubb38\ud604\uba74\uc758 \uc6b0\uae38\ub9ac\ub97c \ud569\uccd0\uc11c \ud55c \ub3d9\ub9ac\ub85c \ub9cc\ub4e4\uc5c8\ub2e4. \uae08\ud559\ub9ac\ub780 \uc774\ub984\uc740 \uae08\uace1\ub9ac(\u91d1\u8c37\u91cc)\uc758 '\uae08(\u91d1)'\uc790\ub97c \ucde8\ud558\uace0 \ucd94\ud559\ub9ac(\u79cb\u9db4\u91cc)\uc5d0\uc11c '\ud559(\u9db4)'\uc790\ub97c \ucde8\ud574\uc11c \uae08\ud559\ub9ac(\u91d1\u9db4\u91cc)\ub85c \uc815\ud574 \ud604\uc7ac\uc758 \uae08\ud559\ub9ac\uac00 \ub418\uc5c8\ub2e4. \uae08\ud559\ub9ac\uc758 \uc790\uc5f0\ud658\uacbd\uc740 \ub17c\ubcf4\ub2e4 \ubc2d\uc774 \ub9ce\uace0 \ube44\uc625\ud558\uc5ec \uc77c\ucc0d\ubd80\ud130 \uc0bc\uacfc \uc57d\ucd08\uc758 \uc7ac\ubc30\uac00 \ubc1c\ub2ec\ud558\uc600\ub2e4."} {"question": "\ubc31\ub828\uc740 \ub204\uad6c\uc640 \ub9e4\uc6b0 \uac00\uae4c\uc6b4 \uc0ac\uc774\uc778\uac00?", "paragraph": "+ \uc544\ub9ac\uc5d0 \ud63c(5\uc704), \uc5d4 \uc790\ud558\ub4dc(\ubd09\uc778\ub418\uae30 \uc8047\uc704) \uc544\ub3c4\ub9ac \uc790\ud558\ub4dc(\uc5d4 \uc790\ud558\ub4dc\uac00 \ubd09\uc778 \ub418\uc5b4 7\uc704) \uc790\ud558\ub4dc(3\uc704), \ubc31\ub828(9\uc704, \uc6d4\ud558\uc775\uc1a1\uc758 \uc218\uc7a5\uc73c\ub85c \uc6b0\ub809 \ub9c8\uc9c0\ub178\uc640 \ub9e4\uc6b0 \uac00\uae4c\uc6b4 \uc0ac\uc774) \ud558 \uc720\ub9b0(\ud558 \uac00\ubb38\uc758 \uac00\uc8fc, 10\uc704) \ud22c \ud398\ub9ac \ud2b8\ud398\ub9ac(\uc624\ud398\ub77c\uc758 \uc81c\uc791\uc790, 11\uc704) \ubab0\ub9ad \uc6d0 P. GR(12\uc704) \uc720\ub77c\uc2dc\uc544 \ube14\ub85c\uc12c(13\uc704) \uadf8\ub808\uc774\uc2a4 \ubbf8\ub974\uce58\uc544 \ub8e8\uc2ac\ub809(FUG\uc758 \uc218\uc7a5, 15\uc704 \ud55c\ub54c 10\uac00\ubb38\uc7a5\ub4e4\uacfc \ud568\uaed8 \ud0d1\uc744 \uc624\ub978 \uc870\ub825\uc790) \ud5e8\ub3c4 \ub85d \ube14\ub7ec\ub4dc\uba54\ub354(16\uc704) \ub85c\ubc84\ud2b8 \uc544\uc774\uc0cc\ub4dc(\ub7ad\ud0b9\uc740 \uc874\uc7ac\ud558\uc9c0 \uc54a\uace0 '\ub7ad\ucee4 \uad00\ub9ac\uad6d'\uc758 \ubd80\uad6d\uc7a5,\uc77c\ubc18\uc778 \uc218\uc900\uc758 \uc804\ud22c\ub2a5\ub825\uc744 \uac00\uc9c4\uac83\uc73c\ub85c \ucd94\uc815, \ud39c\ud0c0\ubbf8\ub118\ubcf4\ub2e4 \ub0ae\uc740 \ub7ad\ud0b9\uc5d0 \ubd88\ub9cc\uc744 \uac00\uc9c4 \uc6b0\ub809 \ub9c8\uc9c0\ub178\uac00 \ud611\ubc15\ud55c \uc801\uc774 \uc788\ub294\ub370 \ud55c\uce58\uc758 \ub9dd\uc124\uc784\ub3c4 \uc5c6\uc774 '\ub2f9\uc2e0\uc740 \ud328\ubc30\uc790\uac00 \ub420 \uac83\uc774\ub2e4'\ub77c\uace0 \ub9d0\ud574 \ucda9\uaca9\uc744 \ubc1b\uc740 \uc6b0\ub809 \ub9c8\uc9c0\ub178\uac00 \uc4f8\uc4f8\ud788 \uc9d1\uc73c\ub85c \ub3cc\uc544\uac14\ub2e4\ub294 \uc0ac\uc5f0\uc774 \uc788\ub2e4.) \uc5f0 \ud55c\uc544(17\uc704) \ubca0\uc774\ub85c\ub4dc \uc57c\ub9c8(FUG\uc2ac\ub808\uc774\uc5b4) \ub4f1\ub4f1", "answer": "\uc6b0\ub809 \ub9c8\uc9c0\ub178", "sentence": "+ \uc544\ub9ac\uc5d0 \ud63c(5\uc704), \uc5d4 \uc790\ud558\ub4dc(\ubd09\uc778\ub418\uae30 \uc8047\uc704) \uc544\ub3c4\ub9ac \uc790\ud558\ub4dc(\uc5d4 \uc790\ud558\ub4dc\uac00 \ubd09\uc778 \ub418\uc5b4 7\uc704) \uc790\ud558\ub4dc(3\uc704), \ubc31\ub828(9\uc704, \uc6d4\ud558\uc775\uc1a1\uc758 \uc218\uc7a5\uc73c\ub85c \uc6b0\ub809 \ub9c8\uc9c0\ub178 \uc640 \ub9e4\uc6b0 \uac00\uae4c\uc6b4 \uc0ac\uc774) \ud558 \uc720\ub9b0(\ud558 \uac00\ubb38\uc758 \uac00\uc8fc, 10\uc704) \ud22c \ud398\ub9ac \ud2b8\ud398\ub9ac(\uc624\ud398\ub77c\uc758 \uc81c\uc791\uc790, 11\uc704) \ubab0\ub9ad \uc6d0 P. GR(12\uc704) \uc720\ub77c\uc2dc\uc544 \ube14\ub85c\uc12c(13\uc704) \uadf8\ub808\uc774\uc2a4 \ubbf8\ub974\uce58\uc544 \ub8e8\uc2ac\ub809(FUG\uc758 \uc218\uc7a5, 15\uc704 \ud55c\ub54c 10\uac00\ubb38\uc7a5\ub4e4\uacfc \ud568\uaed8 \ud0d1\uc744 \uc624\ub978 \uc870\ub825\uc790) \ud5e8\ub3c4 \ub85d \ube14\ub7ec\ub4dc\uba54\ub354(16\uc704) \ub85c\ubc84\ud2b8 \uc544\uc774\uc0cc\ub4dc(\ub7ad\ud0b9\uc740 \uc874\uc7ac\ud558\uc9c0 \uc54a\uace0 '\ub7ad\ucee4 \uad00\ub9ac\uad6d'\uc758 \ubd80\uad6d\uc7a5,\uc77c\ubc18\uc778 \uc218\uc900\uc758 \uc804\ud22c\ub2a5\ub825\uc744 \uac00\uc9c4\uac83\uc73c\ub85c \ucd94\uc815, \ud39c\ud0c0\ubbf8\ub118\ubcf4\ub2e4 \ub0ae\uc740 \ub7ad\ud0b9\uc5d0 \ubd88\ub9cc\uc744 \uac00\uc9c4 \uc6b0\ub809 \ub9c8\uc9c0\ub178\uac00 \ud611\ubc15\ud55c \uc801\uc774 \uc788\ub294\ub370 \ud55c\uce58\uc758 \ub9dd\uc124\uc784\ub3c4 \uc5c6\uc774 '\ub2f9\uc2e0\uc740 \ud328\ubc30\uc790\uac00 \ub420 \uac83\uc774\ub2e4'\ub77c\uace0 \ub9d0\ud574 \ucda9\uaca9\uc744 \ubc1b\uc740 \uc6b0\ub809 \ub9c8\uc9c0\ub178\uac00 \uc4f8\uc4f8\ud788 \uc9d1\uc73c\ub85c \ub3cc\uc544\uac14\ub2e4\ub294 \uc0ac\uc5f0\uc774 \uc788\ub2e4.", "paragraph_sentence": " + \uc544\ub9ac\uc5d0 \ud63c(5\uc704), \uc5d4 \uc790\ud558\ub4dc(\ubd09\uc778\ub418\uae30 \uc8047\uc704) \uc544\ub3c4\ub9ac \uc790\ud558\ub4dc(\uc5d4 \uc790\ud558\ub4dc\uac00 \ubd09\uc778 \ub418\uc5b4 7\uc704) \uc790\ud558\ub4dc(3\uc704), \ubc31\ub828(9\uc704, \uc6d4\ud558\uc775\uc1a1\uc758 \uc218\uc7a5\uc73c\ub85c \uc6b0\ub809 \ub9c8\uc9c0\ub178 \uc640 \ub9e4\uc6b0 \uac00\uae4c\uc6b4 \uc0ac\uc774) \ud558 \uc720\ub9b0(\ud558 \uac00\ubb38\uc758 \uac00\uc8fc, 10\uc704) \ud22c \ud398\ub9ac \ud2b8\ud398\ub9ac(\uc624\ud398\ub77c\uc758 \uc81c\uc791\uc790, 11\uc704) \ubab0\ub9ad \uc6d0 P. GR(12\uc704) \uc720\ub77c\uc2dc\uc544 \ube14\ub85c\uc12c(13\uc704) \uadf8\ub808\uc774\uc2a4 \ubbf8\ub974\uce58\uc544 \ub8e8\uc2ac\ub809(FUG\uc758 \uc218\uc7a5, 15\uc704 \ud55c\ub54c 10\uac00\ubb38\uc7a5\ub4e4\uacfc \ud568\uaed8 \ud0d1\uc744 \uc624\ub978 \uc870\ub825\uc790) \ud5e8\ub3c4 \ub85d \ube14\ub7ec\ub4dc\uba54\ub354(16\uc704) \ub85c\ubc84\ud2b8 \uc544\uc774\uc0cc\ub4dc(\ub7ad\ud0b9\uc740 \uc874\uc7ac\ud558\uc9c0 \uc54a\uace0 '\ub7ad\ucee4 \uad00\ub9ac\uad6d'\uc758 \ubd80\uad6d\uc7a5,\uc77c\ubc18\uc778 \uc218\uc900\uc758 \uc804\ud22c\ub2a5\ub825\uc744 \uac00\uc9c4\uac83\uc73c\ub85c \ucd94\uc815, \ud39c\ud0c0\ubbf8\ub118\ubcf4\ub2e4 \ub0ae\uc740 \ub7ad\ud0b9\uc5d0 \ubd88\ub9cc\uc744 \uac00\uc9c4 \uc6b0\ub809 \ub9c8\uc9c0\ub178\uac00 \ud611\ubc15\ud55c \uc801\uc774 \uc788\ub294\ub370 \ud55c\uce58\uc758 \ub9dd\uc124\uc784\ub3c4 \uc5c6\uc774 '\ub2f9\uc2e0\uc740 \ud328\ubc30\uc790\uac00 \ub420 \uac83\uc774\ub2e4'\ub77c\uace0 \ub9d0\ud574 \ucda9\uaca9\uc744 \ubc1b\uc740 \uc6b0\ub809 \ub9c8\uc9c0\ub178\uac00 \uc4f8\uc4f8\ud788 \uc9d1\uc73c\ub85c \ub3cc\uc544\uac14\ub2e4\ub294 \uc0ac\uc5f0\uc774 \uc788\ub2e4. ) \uc5f0 \ud55c\uc544(17\uc704) \ubca0\uc774\ub85c\ub4dc \uc57c\ub9c8(FUG\uc2ac\ub808\uc774\uc5b4) \ub4f1\ub4f1", "paragraph_answer": "+ \uc544\ub9ac\uc5d0 \ud63c(5\uc704), \uc5d4 \uc790\ud558\ub4dc(\ubd09\uc778\ub418\uae30 \uc8047\uc704) \uc544\ub3c4\ub9ac \uc790\ud558\ub4dc(\uc5d4 \uc790\ud558\ub4dc\uac00 \ubd09\uc778 \ub418\uc5b4 7\uc704) \uc790\ud558\ub4dc(3\uc704), \ubc31\ub828(9\uc704, \uc6d4\ud558\uc775\uc1a1\uc758 \uc218\uc7a5\uc73c\ub85c \uc6b0\ub809 \ub9c8\uc9c0\ub178 \uc640 \ub9e4\uc6b0 \uac00\uae4c\uc6b4 \uc0ac\uc774) \ud558 \uc720\ub9b0(\ud558 \uac00\ubb38\uc758 \uac00\uc8fc, 10\uc704) \ud22c \ud398\ub9ac \ud2b8\ud398\ub9ac(\uc624\ud398\ub77c\uc758 \uc81c\uc791\uc790, 11\uc704) \ubab0\ub9ad \uc6d0 P. GR(12\uc704) \uc720\ub77c\uc2dc\uc544 \ube14\ub85c\uc12c(13\uc704) \uadf8\ub808\uc774\uc2a4 \ubbf8\ub974\uce58\uc544 \ub8e8\uc2ac\ub809(FUG\uc758 \uc218\uc7a5, 15\uc704 \ud55c\ub54c 10\uac00\ubb38\uc7a5\ub4e4\uacfc \ud568\uaed8 \ud0d1\uc744 \uc624\ub978 \uc870\ub825\uc790) \ud5e8\ub3c4 \ub85d \ube14\ub7ec\ub4dc\uba54\ub354(16\uc704) \ub85c\ubc84\ud2b8 \uc544\uc774\uc0cc\ub4dc(\ub7ad\ud0b9\uc740 \uc874\uc7ac\ud558\uc9c0 \uc54a\uace0 '\ub7ad\ucee4 \uad00\ub9ac\uad6d'\uc758 \ubd80\uad6d\uc7a5,\uc77c\ubc18\uc778 \uc218\uc900\uc758 \uc804\ud22c\ub2a5\ub825\uc744 \uac00\uc9c4\uac83\uc73c\ub85c \ucd94\uc815, \ud39c\ud0c0\ubbf8\ub118\ubcf4\ub2e4 \ub0ae\uc740 \ub7ad\ud0b9\uc5d0 \ubd88\ub9cc\uc744 \uac00\uc9c4 \uc6b0\ub809 \ub9c8\uc9c0\ub178\uac00 \ud611\ubc15\ud55c \uc801\uc774 \uc788\ub294\ub370 \ud55c\uce58\uc758 \ub9dd\uc124\uc784\ub3c4 \uc5c6\uc774 '\ub2f9\uc2e0\uc740 \ud328\ubc30\uc790\uac00 \ub420 \uac83\uc774\ub2e4'\ub77c\uace0 \ub9d0\ud574 \ucda9\uaca9\uc744 \ubc1b\uc740 \uc6b0\ub809 \ub9c8\uc9c0\ub178\uac00 \uc4f8\uc4f8\ud788 \uc9d1\uc73c\ub85c \ub3cc\uc544\uac14\ub2e4\ub294 \uc0ac\uc5f0\uc774 \uc788\ub2e4.) \uc5f0 \ud55c\uc544(17\uc704) \ubca0\uc774\ub85c\ub4dc \uc57c\ub9c8(FUG\uc2ac\ub808\uc774\uc5b4) \ub4f1\ub4f1", "sentence_answer": "+ \uc544\ub9ac\uc5d0 \ud63c(5\uc704), \uc5d4 \uc790\ud558\ub4dc(\ubd09\uc778\ub418\uae30 \uc8047\uc704) \uc544\ub3c4\ub9ac \uc790\ud558\ub4dc(\uc5d4 \uc790\ud558\ub4dc\uac00 \ubd09\uc778 \ub418\uc5b4 7\uc704) \uc790\ud558\ub4dc(3\uc704), \ubc31\ub828(9\uc704, \uc6d4\ud558\uc775\uc1a1\uc758 \uc218\uc7a5\uc73c\ub85c \uc6b0\ub809 \ub9c8\uc9c0\ub178 \uc640 \ub9e4\uc6b0 \uac00\uae4c\uc6b4 \uc0ac\uc774) \ud558 \uc720\ub9b0(\ud558 \uac00\ubb38\uc758 \uac00\uc8fc, 10\uc704) \ud22c \ud398\ub9ac \ud2b8\ud398\ub9ac(\uc624\ud398\ub77c\uc758 \uc81c\uc791\uc790, 11\uc704) \ubab0\ub9ad \uc6d0 P. GR(12\uc704) \uc720\ub77c\uc2dc\uc544 \ube14\ub85c\uc12c(13\uc704) \uadf8\ub808\uc774\uc2a4 \ubbf8\ub974\uce58\uc544 \ub8e8\uc2ac\ub809(FUG\uc758 \uc218\uc7a5, 15\uc704 \ud55c\ub54c 10\uac00\ubb38\uc7a5\ub4e4\uacfc \ud568\uaed8 \ud0d1\uc744 \uc624\ub978 \uc870\ub825\uc790) \ud5e8\ub3c4 \ub85d \ube14\ub7ec\ub4dc\uba54\ub354(16\uc704) \ub85c\ubc84\ud2b8 \uc544\uc774\uc0cc\ub4dc(\ub7ad\ud0b9\uc740 \uc874\uc7ac\ud558\uc9c0 \uc54a\uace0 '\ub7ad\ucee4 \uad00\ub9ac\uad6d'\uc758 \ubd80\uad6d\uc7a5,\uc77c\ubc18\uc778 \uc218\uc900\uc758 \uc804\ud22c\ub2a5\ub825\uc744 \uac00\uc9c4\uac83\uc73c\ub85c \ucd94\uc815, \ud39c\ud0c0\ubbf8\ub118\ubcf4\ub2e4 \ub0ae\uc740 \ub7ad\ud0b9\uc5d0 \ubd88\ub9cc\uc744 \uac00\uc9c4 \uc6b0\ub809 \ub9c8\uc9c0\ub178\uac00 \ud611\ubc15\ud55c \uc801\uc774 \uc788\ub294\ub370 \ud55c\uce58\uc758 \ub9dd\uc124\uc784\ub3c4 \uc5c6\uc774 '\ub2f9\uc2e0\uc740 \ud328\ubc30\uc790\uac00 \ub420 \uac83\uc774\ub2e4'\ub77c\uace0 \ub9d0\ud574 \ucda9\uaca9\uc744 \ubc1b\uc740 \uc6b0\ub809 \ub9c8\uc9c0\ub178\uac00 \uc4f8\uc4f8\ud788 \uc9d1\uc73c\ub85c \ub3cc\uc544\uac14\ub2e4\ub294 \uc0ac\uc5f0\uc774 \uc788\ub2e4.", "paragraph_id": "6584812-1-0", "paragraph_question": "question: \ubc31\ub828\uc740 \ub204\uad6c\uc640 \ub9e4\uc6b0 \uac00\uae4c\uc6b4 \uc0ac\uc774\uc778\uac00?, context: + \uc544\ub9ac\uc5d0 \ud63c(5\uc704), \uc5d4 \uc790\ud558\ub4dc(\ubd09\uc778\ub418\uae30 \uc8047\uc704) \uc544\ub3c4\ub9ac \uc790\ud558\ub4dc(\uc5d4 \uc790\ud558\ub4dc\uac00 \ubd09\uc778 \ub418\uc5b4 7\uc704) \uc790\ud558\ub4dc(3\uc704), \ubc31\ub828(9\uc704, \uc6d4\ud558\uc775\uc1a1\uc758 \uc218\uc7a5\uc73c\ub85c \uc6b0\ub809 \ub9c8\uc9c0\ub178\uc640 \ub9e4\uc6b0 \uac00\uae4c\uc6b4 \uc0ac\uc774) \ud558 \uc720\ub9b0(\ud558 \uac00\ubb38\uc758 \uac00\uc8fc, 10\uc704) \ud22c \ud398\ub9ac \ud2b8\ud398\ub9ac(\uc624\ud398\ub77c\uc758 \uc81c\uc791\uc790, 11\uc704) \ubab0\ub9ad \uc6d0 P. GR(12\uc704) \uc720\ub77c\uc2dc\uc544 \ube14\ub85c\uc12c(13\uc704) \uadf8\ub808\uc774\uc2a4 \ubbf8\ub974\uce58\uc544 \ub8e8\uc2ac\ub809(FUG\uc758 \uc218\uc7a5, 15\uc704 \ud55c\ub54c 10\uac00\ubb38\uc7a5\ub4e4\uacfc \ud568\uaed8 \ud0d1\uc744 \uc624\ub978 \uc870\ub825\uc790) \ud5e8\ub3c4 \ub85d \ube14\ub7ec\ub4dc\uba54\ub354(16\uc704) \ub85c\ubc84\ud2b8 \uc544\uc774\uc0cc\ub4dc(\ub7ad\ud0b9\uc740 \uc874\uc7ac\ud558\uc9c0 \uc54a\uace0 '\ub7ad\ucee4 \uad00\ub9ac\uad6d'\uc758 \ubd80\uad6d\uc7a5,\uc77c\ubc18\uc778 \uc218\uc900\uc758 \uc804\ud22c\ub2a5\ub825\uc744 \uac00\uc9c4\uac83\uc73c\ub85c \ucd94\uc815, \ud39c\ud0c0\ubbf8\ub118\ubcf4\ub2e4 \ub0ae\uc740 \ub7ad\ud0b9\uc5d0 \ubd88\ub9cc\uc744 \uac00\uc9c4 \uc6b0\ub809 \ub9c8\uc9c0\ub178\uac00 \ud611\ubc15\ud55c \uc801\uc774 \uc788\ub294\ub370 \ud55c\uce58\uc758 \ub9dd\uc124\uc784\ub3c4 \uc5c6\uc774 '\ub2f9\uc2e0\uc740 \ud328\ubc30\uc790\uac00 \ub420 \uac83\uc774\ub2e4'\ub77c\uace0 \ub9d0\ud574 \ucda9\uaca9\uc744 \ubc1b\uc740 \uc6b0\ub809 \ub9c8\uc9c0\ub178\uac00 \uc4f8\uc4f8\ud788 \uc9d1\uc73c\ub85c \ub3cc\uc544\uac14\ub2e4\ub294 \uc0ac\uc5f0\uc774 \uc788\ub2e4.) \uc5f0 \ud55c\uc544(17\uc704) \ubca0\uc774\ub85c\ub4dc \uc57c\ub9c8(FUG\uc2ac\ub808\uc774\uc5b4) \ub4f1\ub4f1"} {"question": "\ub524\ubbf8\uc778 \uc0ac\ub78c\uc774 \ubb34\uc5c7\uc73c\ub85c \uac1c\uc885\ud558\ub294 \uac83\uc774 \uc544\uc8fc \uc26c\uc6b4 \uc77c\ub85c \uc5ec\uaca8\uc84c\ub294\uac00?", "paragraph": "\uc6d0\ub798 \ub524\ubbf8\uc758 \uc9c0\uc704\ub97c \ub204\ub9b4 \uc218 \uc788\uc5c8\ub358 \uc0ac\ub78c\ub4e4\uc740 \uafb8\ub780\uc5d0 '\ucc45\uc758 \uc0ac\ub78c\ub4e4'\ub85c \uc9c0\uce6d\ub41c \uc720\ub300\uc778\uacfc \uae30\ub3c5\uad50\uc778\ubfd0\uc774\uc5c8\uc73c\ub098 \ud6c4\uc5d0 \uc2dc\ud06c\uad50\ub3c4, \uc870\ub85c\uc544\uc2a4\ud130\uad50, \ud78c\ub450\uad50 \uad50\uc778\ub3c4 \ud3ec\ud568\ud558\uac8c \ub410\ub2e4. \ubd88\uad50\ub3c4\ub3c4 \uc774\uc5d0 \ud574\ub2f9\ud558\ub294 \uad6d\ubbfc\uc774\ub2e4. \ub524\ubbf8\uc5d0 \ud574\ub2f9\ud558\ub294 \uad6d\ubbfc\uc740 \ubc95\uc801\u00b7\uc0ac\ud68c\uc801 \uc9c0\uc704\uac00 \ubb34\uc2ac\ub9bc\ubcf4\ub2e4 \uc0c1\ub300\uc801\uc73c\ub85c \ub0ae\uc9c0\ub9cc \ub2e4\ub978 \uc774\uad50\ub3c4\ubcf4\ub2e4\ub294 \ub354 \ub9ce\uc740 \uad8c\ub9ac\ub97c \ub204\ub9ac\uac8c \ub41c\ub2e4. 7\uc138\uae30\ubd80\ud130 \uadfc\ub300\uc5d0 \uc774\ub974\uae30\uae4c\uc9c0 \ub300\uc11c\uc591 \uc5f0\uc548\uc744 \uac70\uccd0 \uc778\ub3c4\uc5d0 \uc774\ub974\ub294 \uc218\ubc31\ub9cc \uba85\uc758 \uc0ac\ub78c\ub4e4\uc774 \ub524\ubbf8\uc5d0 \ud574\ub2f9\ub410\uc744 \uac83\uc73c\ub85c \ucd94\uc815\ub418\uace0 \uc788\ub2e4. \ub524\ubbf8\uc778 \uc0ac\ub78c\uc774 \uc774\uc2ac\ub78c\uad50\ub85c \uac1c\uc885\ud558\ub294 \uac83\uc740 \uc544\uc8fc \uc26c\uc6b4 \uc77c\uc774\uc5c8\uc73c\uba70, \ud2b9\ubcc4\ud55c \ubb38\uc81c\uac00 \uc5c6\ub2e4\uba74 \uc18d\ubc15\uc73c\ub85c\ubd80\ud130 \uc790\uc720\ub85c\uc6cc\uc9c8 \uc218 \uc788\uc5c8\ub2e4. \uc774\uc2ac\ub78c \uc138\uacc4\uc5d0\uc11c\ub294 \uc77c\ubc29\uc801\uc73c\ub85c \uac1c\uc885\uc744 \uac15\uc694\ud558\uac70\ub098 \uc0ac\ub78c\uc744 \uc8fd\uc774\ub294 \uc77c\uc774 \uac70\uc758 \uc5c6\uc5c8\uc73c\ub098 \uc2ed\uc790\uad70 \uc804\uc7c1\uc774 \uc2dc\uc791\ub41c 12\uc138\uae30 \uc774\ud6c4\ubd80\ud130, \uc54c\uc548\ub2ec\ub8e8\uc2a4\uc640 \ubb34\uc640\ud788\ub4dc \uc655\uc870\uc5d0 \uc774\ub974\ub7ec\uc11c\ub294 '\uc774\uc2ac\ub78c \uc544\ub2c8\uba74 \uc8fd\uc74c'\uc744 \uac15\uc694\ud588\ub2e4.", "answer": "\uc774\uc2ac\ub78c\uad50", "sentence": "\ub524\ubbf8\uc778 \uc0ac\ub78c\uc774 \uc774\uc2ac\ub78c\uad50 \ub85c \uac1c\uc885\ud558\ub294 \uac83\uc740 \uc544\uc8fc \uc26c\uc6b4 \uc77c\uc774\uc5c8\uc73c\uba70, \ud2b9\ubcc4\ud55c \ubb38\uc81c\uac00 \uc5c6\ub2e4\uba74 \uc18d\ubc15\uc73c\ub85c\ubd80\ud130 \uc790\uc720\ub85c\uc6cc\uc9c8 \uc218 \uc788\uc5c8\ub2e4.", "paragraph_sentence": "\uc6d0\ub798 \ub524\ubbf8\uc758 \uc9c0\uc704\ub97c \ub204\ub9b4 \uc218 \uc788\uc5c8\ub358 \uc0ac\ub78c\ub4e4\uc740 \uafb8\ub780\uc5d0 '\ucc45\uc758 \uc0ac\ub78c\ub4e4'\ub85c \uc9c0\uce6d\ub41c \uc720\ub300\uc778\uacfc \uae30\ub3c5\uad50\uc778\ubfd0\uc774\uc5c8\uc73c\ub098 \ud6c4\uc5d0 \uc2dc\ud06c\uad50\ub3c4, \uc870\ub85c\uc544\uc2a4\ud130\uad50, \ud78c\ub450\uad50 \uad50\uc778\ub3c4 \ud3ec\ud568\ud558\uac8c \ub410\ub2e4. \ubd88\uad50\ub3c4\ub3c4 \uc774\uc5d0 \ud574\ub2f9\ud558\ub294 \uad6d\ubbfc\uc774\ub2e4. \ub524\ubbf8\uc5d0 \ud574\ub2f9\ud558\ub294 \uad6d\ubbfc\uc740 \ubc95\uc801\u00b7\uc0ac\ud68c\uc801 \uc9c0\uc704\uac00 \ubb34\uc2ac\ub9bc\ubcf4\ub2e4 \uc0c1\ub300\uc801\uc73c\ub85c \ub0ae\uc9c0\ub9cc \ub2e4\ub978 \uc774\uad50\ub3c4\ubcf4\ub2e4\ub294 \ub354 \ub9ce\uc740 \uad8c\ub9ac\ub97c \ub204\ub9ac\uac8c \ub41c\ub2e4. 7\uc138\uae30\ubd80\ud130 \uadfc\ub300\uc5d0 \uc774\ub974\uae30\uae4c\uc9c0 \ub300\uc11c\uc591 \uc5f0\uc548\uc744 \uac70\uccd0 \uc778\ub3c4\uc5d0 \uc774\ub974\ub294 \uc218\ubc31\ub9cc \uba85\uc758 \uc0ac\ub78c\ub4e4\uc774 \ub524\ubbf8\uc5d0 \ud574\ub2f9\ub410\uc744 \uac83\uc73c\ub85c \ucd94\uc815\ub418\uace0 \uc788\ub2e4. \ub524\ubbf8\uc778 \uc0ac\ub78c\uc774 \uc774\uc2ac\ub78c\uad50 \ub85c \uac1c\uc885\ud558\ub294 \uac83\uc740 \uc544\uc8fc \uc26c\uc6b4 \uc77c\uc774\uc5c8\uc73c\uba70, \ud2b9\ubcc4\ud55c \ubb38\uc81c\uac00 \uc5c6\ub2e4\uba74 \uc18d\ubc15\uc73c\ub85c\ubd80\ud130 \uc790\uc720\ub85c\uc6cc\uc9c8 \uc218 \uc788\uc5c8\ub2e4. \uc774\uc2ac\ub78c \uc138\uacc4\uc5d0\uc11c\ub294 \uc77c\ubc29\uc801\uc73c\ub85c \uac1c\uc885\uc744 \uac15\uc694\ud558\uac70\ub098 \uc0ac\ub78c\uc744 \uc8fd\uc774\ub294 \uc77c\uc774 \uac70\uc758 \uc5c6\uc5c8\uc73c\ub098 \uc2ed\uc790\uad70 \uc804\uc7c1\uc774 \uc2dc\uc791\ub41c 12\uc138\uae30 \uc774\ud6c4\ubd80\ud130, \uc54c\uc548\ub2ec\ub8e8\uc2a4\uc640 \ubb34\uc640\ud788\ub4dc \uc655\uc870\uc5d0 \uc774\ub974\ub7ec\uc11c\ub294 '\uc774\uc2ac\ub78c \uc544\ub2c8\uba74 \uc8fd\uc74c'\uc744 \uac15\uc694\ud588\ub2e4.", "paragraph_answer": "\uc6d0\ub798 \ub524\ubbf8\uc758 \uc9c0\uc704\ub97c \ub204\ub9b4 \uc218 \uc788\uc5c8\ub358 \uc0ac\ub78c\ub4e4\uc740 \uafb8\ub780\uc5d0 '\ucc45\uc758 \uc0ac\ub78c\ub4e4'\ub85c \uc9c0\uce6d\ub41c \uc720\ub300\uc778\uacfc \uae30\ub3c5\uad50\uc778\ubfd0\uc774\uc5c8\uc73c\ub098 \ud6c4\uc5d0 \uc2dc\ud06c\uad50\ub3c4, \uc870\ub85c\uc544\uc2a4\ud130\uad50, \ud78c\ub450\uad50 \uad50\uc778\ub3c4 \ud3ec\ud568\ud558\uac8c \ub410\ub2e4. \ubd88\uad50\ub3c4\ub3c4 \uc774\uc5d0 \ud574\ub2f9\ud558\ub294 \uad6d\ubbfc\uc774\ub2e4. \ub524\ubbf8\uc5d0 \ud574\ub2f9\ud558\ub294 \uad6d\ubbfc\uc740 \ubc95\uc801\u00b7\uc0ac\ud68c\uc801 \uc9c0\uc704\uac00 \ubb34\uc2ac\ub9bc\ubcf4\ub2e4 \uc0c1\ub300\uc801\uc73c\ub85c \ub0ae\uc9c0\ub9cc \ub2e4\ub978 \uc774\uad50\ub3c4\ubcf4\ub2e4\ub294 \ub354 \ub9ce\uc740 \uad8c\ub9ac\ub97c \ub204\ub9ac\uac8c \ub41c\ub2e4. 7\uc138\uae30\ubd80\ud130 \uadfc\ub300\uc5d0 \uc774\ub974\uae30\uae4c\uc9c0 \ub300\uc11c\uc591 \uc5f0\uc548\uc744 \uac70\uccd0 \uc778\ub3c4\uc5d0 \uc774\ub974\ub294 \uc218\ubc31\ub9cc \uba85\uc758 \uc0ac\ub78c\ub4e4\uc774 \ub524\ubbf8\uc5d0 \ud574\ub2f9\ub410\uc744 \uac83\uc73c\ub85c \ucd94\uc815\ub418\uace0 \uc788\ub2e4. \ub524\ubbf8\uc778 \uc0ac\ub78c\uc774 \uc774\uc2ac\ub78c\uad50 \ub85c \uac1c\uc885\ud558\ub294 \uac83\uc740 \uc544\uc8fc \uc26c\uc6b4 \uc77c\uc774\uc5c8\uc73c\uba70, \ud2b9\ubcc4\ud55c \ubb38\uc81c\uac00 \uc5c6\ub2e4\uba74 \uc18d\ubc15\uc73c\ub85c\ubd80\ud130 \uc790\uc720\ub85c\uc6cc\uc9c8 \uc218 \uc788\uc5c8\ub2e4. \uc774\uc2ac\ub78c \uc138\uacc4\uc5d0\uc11c\ub294 \uc77c\ubc29\uc801\uc73c\ub85c \uac1c\uc885\uc744 \uac15\uc694\ud558\uac70\ub098 \uc0ac\ub78c\uc744 \uc8fd\uc774\ub294 \uc77c\uc774 \uac70\uc758 \uc5c6\uc5c8\uc73c\ub098 \uc2ed\uc790\uad70 \uc804\uc7c1\uc774 \uc2dc\uc791\ub41c 12\uc138\uae30 \uc774\ud6c4\ubd80\ud130, \uc54c\uc548\ub2ec\ub8e8\uc2a4\uc640 \ubb34\uc640\ud788\ub4dc \uc655\uc870\uc5d0 \uc774\ub974\ub7ec\uc11c\ub294 '\uc774\uc2ac\ub78c \uc544\ub2c8\uba74 \uc8fd\uc74c'\uc744 \uac15\uc694\ud588\ub2e4.", "sentence_answer": "\ub524\ubbf8\uc778 \uc0ac\ub78c\uc774 \uc774\uc2ac\ub78c\uad50 \ub85c \uac1c\uc885\ud558\ub294 \uac83\uc740 \uc544\uc8fc \uc26c\uc6b4 \uc77c\uc774\uc5c8\uc73c\uba70, \ud2b9\ubcc4\ud55c \ubb38\uc81c\uac00 \uc5c6\ub2e4\uba74 \uc18d\ubc15\uc73c\ub85c\ubd80\ud130 \uc790\uc720\ub85c\uc6cc\uc9c8 \uc218 \uc788\uc5c8\ub2e4.", "paragraph_id": "6528850-0-2", "paragraph_question": "question: \ub524\ubbf8\uc778 \uc0ac\ub78c\uc774 \ubb34\uc5c7\uc73c\ub85c \uac1c\uc885\ud558\ub294 \uac83\uc774 \uc544\uc8fc \uc26c\uc6b4 \uc77c\ub85c \uc5ec\uaca8\uc84c\ub294\uac00?, context: \uc6d0\ub798 \ub524\ubbf8\uc758 \uc9c0\uc704\ub97c \ub204\ub9b4 \uc218 \uc788\uc5c8\ub358 \uc0ac\ub78c\ub4e4\uc740 \uafb8\ub780\uc5d0 '\ucc45\uc758 \uc0ac\ub78c\ub4e4'\ub85c \uc9c0\uce6d\ub41c \uc720\ub300\uc778\uacfc \uae30\ub3c5\uad50\uc778\ubfd0\uc774\uc5c8\uc73c\ub098 \ud6c4\uc5d0 \uc2dc\ud06c\uad50\ub3c4, \uc870\ub85c\uc544\uc2a4\ud130\uad50, \ud78c\ub450\uad50 \uad50\uc778\ub3c4 \ud3ec\ud568\ud558\uac8c \ub410\ub2e4. \ubd88\uad50\ub3c4\ub3c4 \uc774\uc5d0 \ud574\ub2f9\ud558\ub294 \uad6d\ubbfc\uc774\ub2e4. \ub524\ubbf8\uc5d0 \ud574\ub2f9\ud558\ub294 \uad6d\ubbfc\uc740 \ubc95\uc801\u00b7\uc0ac\ud68c\uc801 \uc9c0\uc704\uac00 \ubb34\uc2ac\ub9bc\ubcf4\ub2e4 \uc0c1\ub300\uc801\uc73c\ub85c \ub0ae\uc9c0\ub9cc \ub2e4\ub978 \uc774\uad50\ub3c4\ubcf4\ub2e4\ub294 \ub354 \ub9ce\uc740 \uad8c\ub9ac\ub97c \ub204\ub9ac\uac8c \ub41c\ub2e4. 7\uc138\uae30\ubd80\ud130 \uadfc\ub300\uc5d0 \uc774\ub974\uae30\uae4c\uc9c0 \ub300\uc11c\uc591 \uc5f0\uc548\uc744 \uac70\uccd0 \uc778\ub3c4\uc5d0 \uc774\ub974\ub294 \uc218\ubc31\ub9cc \uba85\uc758 \uc0ac\ub78c\ub4e4\uc774 \ub524\ubbf8\uc5d0 \ud574\ub2f9\ub410\uc744 \uac83\uc73c\ub85c \ucd94\uc815\ub418\uace0 \uc788\ub2e4. \ub524\ubbf8\uc778 \uc0ac\ub78c\uc774 \uc774\uc2ac\ub78c\uad50\ub85c \uac1c\uc885\ud558\ub294 \uac83\uc740 \uc544\uc8fc \uc26c\uc6b4 \uc77c\uc774\uc5c8\uc73c\uba70, \ud2b9\ubcc4\ud55c \ubb38\uc81c\uac00 \uc5c6\ub2e4\uba74 \uc18d\ubc15\uc73c\ub85c\ubd80\ud130 \uc790\uc720\ub85c\uc6cc\uc9c8 \uc218 \uc788\uc5c8\ub2e4. \uc774\uc2ac\ub78c \uc138\uacc4\uc5d0\uc11c\ub294 \uc77c\ubc29\uc801\uc73c\ub85c \uac1c\uc885\uc744 \uac15\uc694\ud558\uac70\ub098 \uc0ac\ub78c\uc744 \uc8fd\uc774\ub294 \uc77c\uc774 \uac70\uc758 \uc5c6\uc5c8\uc73c\ub098 \uc2ed\uc790\uad70 \uc804\uc7c1\uc774 \uc2dc\uc791\ub41c 12\uc138\uae30 \uc774\ud6c4\ubd80\ud130, \uc54c\uc548\ub2ec\ub8e8\uc2a4\uc640 \ubb34\uc640\ud788\ub4dc \uc655\uc870\uc5d0 \uc774\ub974\ub7ec\uc11c\ub294 '\uc774\uc2ac\ub78c \uc544\ub2c8\uba74 \uc8fd\uc74c'\uc744 \uac15\uc694\ud588\ub2e4."} {"question": "\ub098\ud3f4\ub9ac \ud1a0\ubc15\uc774 \ucd9c\uc2e0\uc758 \uc624\ud398\ub77c \uac00\uc218\ub294 \ub204\uad6c\uc778\uac00?", "paragraph": "\uc774 \ud654\ud48d\uc73c\ub85c\ubd80\ud130 \uc9c1\uc811\uc801\uc73c\ub85c \uc601\ud5a5\uc744 \ubc1b\uc740 \ud654\uac00\ub4e4\ub85c\ub294 \uc0b4\ubc14\ud1a0\ub974 \ub85c\uc0ac, \ud53c\uc5d0\ub974 \uc790\ud06c \ubcfc\ub808\ub974\uc640 \uadf8\uc758 \uc0b6 \ub9c8\uc9c0\ub9c9 \uba87\ub144\uc744 \ub098\ud3f4\ub9ac\uc5d0\uc11c \ubcf4\ub0b8 \uc548\ud1a0\ub2c8 \uc2a4\ubc0d\ud06c \ud53c\ud2c0\ub85c\uac00 \uc788\ub2e4. \uc624\ud398\ub77c \uac00\uc218 \uc5d4\ub9ac\ucf54 \uce74\ub8e8\uc18c\ub3c4 \ub098\ud3f4\ub9ac \ud1a0\ubc15\uc774 \ucd9c\uc2e0\uc774\ub2e4. \ub7ec\uc2dc\uc544 \ud601\uba85 \uc9c0\ub3c4\uc790 \ube14\ub77c\ub514\ubbf8\ub974 \ub808\ub2cc\uc740 \uce74\ud504\ub9ac\uc5d0\uc11c \uba87\ub144\uc744 \uc0b4\uc558\ub2e4. 20\uc138\uae30\uc5d0 \ub098\ud3f4\ub9ac \ubbfc\uc694\ub77c\uace0 \ubd88\ub9ac\ub294 \uc7a5\ub974\uac00 \uc804 \uc138\uacc4\uc801\uc73c\ub85c \uc720\uba85\ud574\uc84c\ub2e4. \uc720\uba85\ud55c \uace1\ub4e4\ub85c\ub294 \"\uc624\uc194\ub808\ubbf8\uc624(O sole mio)\", \"\ud478\ub2c8\ucfe8\ub9ac, \ud478\ub2c8\ucfe8\ub77c(Funicul\u00ec, funicul\u00e0)\", \"\uc624 \uc218\ub974\ub2e4\ud1a0 \ub098\ubb34\ub77c\ud1a0(O surdato nnamurato)\", \"\ud1a0\ub974\ub098 \uc544 \uc218\ub9ac\uc5d4\ud1a0(Torna a Surriento)\", \"\uad6c\uc544\ud30c\ub9ac\uc544(Guapparia)\", \"\uc0b0\ud0c0 \ub8e8\uce58\uc544(Santa Lucia)\", \"\ub808\uc9c0\ub12c\ub77c(Reginella)\", \"\ub9c8\ub808\ud0a4\uc544\ub85c(Marechiaro)\", \"\uc2a4\ud540\uad74\ub808 \ud504\ub780\uccb4\uc138(Spingule Francese)\"\ub4f1\uc774 \uc788\ub2e4.", "answer": "\uc5d4\ub9ac\ucf54 \uce74\ub8e8\uc18c", "sentence": "\uc624\ud398\ub77c \uac00\uc218 \uc5d4\ub9ac\ucf54 \uce74\ub8e8\uc18c \ub3c4 \ub098\ud3f4\ub9ac \ud1a0\ubc15\uc774 \ucd9c\uc2e0\uc774\ub2e4.", "paragraph_sentence": "\uc774 \ud654\ud48d\uc73c\ub85c\ubd80\ud130 \uc9c1\uc811\uc801\uc73c\ub85c \uc601\ud5a5\uc744 \ubc1b\uc740 \ud654\uac00\ub4e4\ub85c\ub294 \uc0b4\ubc14\ud1a0\ub974 \ub85c\uc0ac, \ud53c\uc5d0\ub974 \uc790\ud06c \ubcfc\ub808\ub974\uc640 \uadf8\uc758 \uc0b6 \ub9c8\uc9c0\ub9c9 \uba87\ub144\uc744 \ub098\ud3f4\ub9ac\uc5d0\uc11c \ubcf4\ub0b8 \uc548\ud1a0\ub2c8 \uc2a4\ubc0d\ud06c \ud53c\ud2c0\ub85c\uac00 \uc788\ub2e4. \uc624\ud398\ub77c \uac00\uc218 \uc5d4\ub9ac\ucf54 \uce74\ub8e8\uc18c \ub3c4 \ub098\ud3f4\ub9ac \ud1a0\ubc15\uc774 \ucd9c\uc2e0\uc774\ub2e4. \ub7ec\uc2dc\uc544 \ud601\uba85 \uc9c0\ub3c4\uc790 \ube14\ub77c\ub514\ubbf8\ub974 \ub808\ub2cc\uc740 \uce74\ud504\ub9ac\uc5d0\uc11c \uba87\ub144\uc744 \uc0b4\uc558\ub2e4. 20\uc138\uae30\uc5d0 \ub098\ud3f4\ub9ac \ubbfc\uc694\ub77c\uace0 \ubd88\ub9ac\ub294 \uc7a5\ub974\uac00 \uc804 \uc138\uacc4\uc801\uc73c\ub85c \uc720\uba85\ud574\uc84c\ub2e4. \uc720\uba85\ud55c \uace1\ub4e4\ub85c\ub294 \"\uc624\uc194\ub808\ubbf8\uc624(O sole mio)\", \"\ud478\ub2c8\ucfe8\ub9ac, \ud478\ub2c8\ucfe8\ub77c(Funicul\u00ec, funicul\u00e0)\", \"\uc624 \uc218\ub974\ub2e4\ud1a0 \ub098\ubb34\ub77c\ud1a0(O surdato nnamurato)\", \"\ud1a0\ub974\ub098 \uc544 \uc218\ub9ac\uc5d4\ud1a0(Torna a Surriento)\", \"\uad6c\uc544\ud30c\ub9ac\uc544(Guapparia)\", \"\uc0b0\ud0c0 \ub8e8\uce58\uc544(Santa Lucia)\", \"\ub808\uc9c0\ub12c\ub77c(Reginella)\", \"\ub9c8\ub808\ud0a4\uc544\ub85c(Marechiaro)\", \"\uc2a4\ud540\uad74\ub808 \ud504\ub780\uccb4\uc138(Spingule Francese)\"\ub4f1\uc774 \uc788\ub2e4.", "paragraph_answer": "\uc774 \ud654\ud48d\uc73c\ub85c\ubd80\ud130 \uc9c1\uc811\uc801\uc73c\ub85c \uc601\ud5a5\uc744 \ubc1b\uc740 \ud654\uac00\ub4e4\ub85c\ub294 \uc0b4\ubc14\ud1a0\ub974 \ub85c\uc0ac, \ud53c\uc5d0\ub974 \uc790\ud06c \ubcfc\ub808\ub974\uc640 \uadf8\uc758 \uc0b6 \ub9c8\uc9c0\ub9c9 \uba87\ub144\uc744 \ub098\ud3f4\ub9ac\uc5d0\uc11c \ubcf4\ub0b8 \uc548\ud1a0\ub2c8 \uc2a4\ubc0d\ud06c \ud53c\ud2c0\ub85c\uac00 \uc788\ub2e4. \uc624\ud398\ub77c \uac00\uc218 \uc5d4\ub9ac\ucf54 \uce74\ub8e8\uc18c \ub3c4 \ub098\ud3f4\ub9ac \ud1a0\ubc15\uc774 \ucd9c\uc2e0\uc774\ub2e4. \ub7ec\uc2dc\uc544 \ud601\uba85 \uc9c0\ub3c4\uc790 \ube14\ub77c\ub514\ubbf8\ub974 \ub808\ub2cc\uc740 \uce74\ud504\ub9ac\uc5d0\uc11c \uba87\ub144\uc744 \uc0b4\uc558\ub2e4. 20\uc138\uae30\uc5d0 \ub098\ud3f4\ub9ac \ubbfc\uc694\ub77c\uace0 \ubd88\ub9ac\ub294 \uc7a5\ub974\uac00 \uc804 \uc138\uacc4\uc801\uc73c\ub85c \uc720\uba85\ud574\uc84c\ub2e4. \uc720\uba85\ud55c \uace1\ub4e4\ub85c\ub294 \"\uc624\uc194\ub808\ubbf8\uc624(O sole mio)\", \"\ud478\ub2c8\ucfe8\ub9ac, \ud478\ub2c8\ucfe8\ub77c(Funicul\u00ec, funicul\u00e0)\", \"\uc624 \uc218\ub974\ub2e4\ud1a0 \ub098\ubb34\ub77c\ud1a0(O surdato nnamurato)\", \"\ud1a0\ub974\ub098 \uc544 \uc218\ub9ac\uc5d4\ud1a0(Torna a Surriento)\", \"\uad6c\uc544\ud30c\ub9ac\uc544(Guapparia)\", \"\uc0b0\ud0c0 \ub8e8\uce58\uc544(Santa Lucia)\", \"\ub808\uc9c0\ub12c\ub77c(Reginella)\", \"\ub9c8\ub808\ud0a4\uc544\ub85c(Marechiaro)\", \"\uc2a4\ud540\uad74\ub808 \ud504\ub780\uccb4\uc138(Spingule Francese)\"\ub4f1\uc774 \uc788\ub2e4.", "sentence_answer": "\uc624\ud398\ub77c \uac00\uc218 \uc5d4\ub9ac\ucf54 \uce74\ub8e8\uc18c \ub3c4 \ub098\ud3f4\ub9ac \ud1a0\ubc15\uc774 \ucd9c\uc2e0\uc774\ub2e4.", "paragraph_id": "6561350-11-0", "paragraph_question": "question: \ub098\ud3f4\ub9ac \ud1a0\ubc15\uc774 \ucd9c\uc2e0\uc758 \uc624\ud398\ub77c \uac00\uc218\ub294 \ub204\uad6c\uc778\uac00?, context: \uc774 \ud654\ud48d\uc73c\ub85c\ubd80\ud130 \uc9c1\uc811\uc801\uc73c\ub85c \uc601\ud5a5\uc744 \ubc1b\uc740 \ud654\uac00\ub4e4\ub85c\ub294 \uc0b4\ubc14\ud1a0\ub974 \ub85c\uc0ac, \ud53c\uc5d0\ub974 \uc790\ud06c \ubcfc\ub808\ub974\uc640 \uadf8\uc758 \uc0b6 \ub9c8\uc9c0\ub9c9 \uba87\ub144\uc744 \ub098\ud3f4\ub9ac\uc5d0\uc11c \ubcf4\ub0b8 \uc548\ud1a0\ub2c8 \uc2a4\ubc0d\ud06c \ud53c\ud2c0\ub85c\uac00 \uc788\ub2e4. \uc624\ud398\ub77c \uac00\uc218 \uc5d4\ub9ac\ucf54 \uce74\ub8e8\uc18c\ub3c4 \ub098\ud3f4\ub9ac \ud1a0\ubc15\uc774 \ucd9c\uc2e0\uc774\ub2e4. \ub7ec\uc2dc\uc544 \ud601\uba85 \uc9c0\ub3c4\uc790 \ube14\ub77c\ub514\ubbf8\ub974 \ub808\ub2cc\uc740 \uce74\ud504\ub9ac\uc5d0\uc11c \uba87\ub144\uc744 \uc0b4\uc558\ub2e4. 20\uc138\uae30\uc5d0 \ub098\ud3f4\ub9ac \ubbfc\uc694\ub77c\uace0 \ubd88\ub9ac\ub294 \uc7a5\ub974\uac00 \uc804 \uc138\uacc4\uc801\uc73c\ub85c \uc720\uba85\ud574\uc84c\ub2e4. \uc720\uba85\ud55c \uace1\ub4e4\ub85c\ub294 \"\uc624\uc194\ub808\ubbf8\uc624(O sole mio)\", \"\ud478\ub2c8\ucfe8\ub9ac, \ud478\ub2c8\ucfe8\ub77c(Funicul\u00ec, funicul\u00e0)\", \"\uc624 \uc218\ub974\ub2e4\ud1a0 \ub098\ubb34\ub77c\ud1a0(O surdato nnamurato)\", \"\ud1a0\ub974\ub098 \uc544 \uc218\ub9ac\uc5d4\ud1a0(Torna a Surriento)\", \"\uad6c\uc544\ud30c\ub9ac\uc544(Guapparia)\", \"\uc0b0\ud0c0 \ub8e8\uce58\uc544(Santa Lucia)\", \"\ub808\uc9c0\ub12c\ub77c(Reginella)\", \"\ub9c8\ub808\ud0a4\uc544\ub85c(Marechiaro)\", \"\uc2a4\ud540\uad74\ub808 \ud504\ub780\uccb4\uc138(Spingule Francese)\"\ub4f1\uc774 \uc788\ub2e4."} {"question": "\ubaa8\ub85c\uc758 \uc791\uc804\uc740 \ubb34\uc5c7\uc758 \uacb0\uacfc\uc5d0 \ub530\ub77c \uc800\uc9c0\ub418\uc5c8\ub294\uac00?", "paragraph": "1797\ub144 \ubaa8\ub85c\ub294 \uc774\ud6c4 \uc790\uae08\uacfc \ubb3c\uc790\uc758 \ubd80\uc871\uc774\ub780 \uc774\uc720\uc5d0\ub3c4 \ubd88\uad6c\ud558\uace0 \ub77c\uc778\uc744 \uac00\ub85c\uc9c8\ub7ec \ub098\uc544\uac14\uc73c\ub098 \ubaa8\ub85c\uc758 \uc791\uc804\uc740 \ubcf4\ub098\ud30c\ub974\ud2b8\uc640 \uc624\uc2a4\ud2b8\ub9ac\uc544 \uc0ac\uc774\uc5d0\uc11c \ub808\uc624\ubcf8 \ud3c9\ud654(Peace of Leoben)\uc758 \uc0ac\uc804 \uc791\uc5c5\uc758 \uacb0\uacfc\uc5d0 \ub530\ub77c \uc800\uc9c0\ub418\uc5c8\ub2e4. \uc774 \uc2dc\uae30 \ubaa8\ub85c\ub294 \uc624\ub79c \ub3d9\ub8cc\uc774\uc790 \uc0ac\ub839\uad00\uc778 \uc0e4\ub97c \ud53c\uc288\uadf8\ub93c\uc640 \ucf69\ub370 \uacf5(Prince de Cond\u00e9) \uc0ac\uc774\uc5d0\uc11c \uc655\ub798\ub41c \ubc18\uc5ed\uc5d0 \uad00\ub828\ub41c \uc11c\uc2e0\uc744 \ubc1c\uacac\ud558\uc600\ub2e4. \ubaa8\ub85c\ub294 \uc774\ubbf8 \ubd88\ucda9\ud55c \ud589\uc704\uc5d0 \ub300\ud55c \ube44\ub09c\uc744 \ubc1b\ub358 \ud53c\uc288\uadf8\ub93c\uc758 \uc639\ud638\uc790\uc600\uc5c8\ub2e4. \ubaa8\ub85c\ub294 \uc5b4\ub9ac\uc11d\uac8c\ub3c4 \ubc1c\uacac\ud55c \uc11c\uc2e0\uc5d0 \ub300\ud574 \ub0b4\uc0c9\ud558\uc9c0 \uc54a\uc558\uace0, \uadf8 \uacb0\uacfc \uac00\uc7a5 \uc791\uc9c0\ub9cc \uc77c\ubd80 \uc5f0\ub8e8\ub41c \uc758\uc2ec\uc744 \ubc1b\uac8c \ub418\uc5c8\ub2e4. \ub4a4\ub2a6\uac8c \ubaa8\ub85c\ub294 \ubc18\uc5ed\uc790\ub85c\uc368 \ud53c\uc288\uadf8\ub93c\ub97c \uace0\ubc1c\ud558\ub294 \uc131\uba85\uc11c\ub97c \uad70\uc5d0 \ubc1c\ud589\ud588\uace0, \uacb0\ubc31\uc744 \uc8fc\uc7a5\ud558\ub294 \uc11c\uc2e0\uc744 \ud30c\ub9ac\uc5d0 \ubcf4\ub0c8\ub2e4.", "answer": "\ub808\uc624\ubcf8 \ud3c9\ud654(Peace of Leoben)\uc758 \uc0ac\uc804 \uc791\uc5c5", "sentence": "\ub098\uc544\uac14\uc73c\ub098 \ubaa8\ub85c\uc758 \uc791\uc804\uc740 \ubcf4\ub098\ud30c\ub974\ud2b8\uc640 \uc624\uc2a4\ud2b8\ub9ac\uc544 \uc0ac\uc774\uc5d0\uc11c \ub808\uc624\ubcf8 \ud3c9\ud654(Peace of Leoben)\uc758 \uc0ac\uc804 \uc791\uc5c5 \uc758 \uacb0\uacfc\uc5d0 \ub530\ub77c \uc800\uc9c0\ub418\uc5c8\ub2e4.", "paragraph_sentence": "1797\ub144 \ubaa8\ub85c\ub294 \uc774\ud6c4 \uc790\uae08\uacfc \ubb3c\uc790\uc758 \ubd80\uc871\uc774\ub780 \uc774\uc720\uc5d0\ub3c4 \ubd88\uad6c\ud558\uace0 \ub77c\uc778\uc744 \uac00\ub85c\uc9c8\ub7ec \ub098\uc544\uac14\uc73c\ub098 \ubaa8\ub85c\uc758 \uc791\uc804\uc740 \ubcf4\ub098\ud30c\ub974\ud2b8\uc640 \uc624\uc2a4\ud2b8\ub9ac\uc544 \uc0ac\uc774\uc5d0\uc11c \ub808\uc624\ubcf8 \ud3c9\ud654(Peace of Leoben)\uc758 \uc0ac\uc804 \uc791\uc5c5 \uc758 \uacb0\uacfc\uc5d0 \ub530\ub77c \uc800\uc9c0\ub418\uc5c8\ub2e4. \uc774 \uc2dc\uae30 \ubaa8\ub85c\ub294 \uc624\ub79c \ub3d9\ub8cc\uc774\uc790 \uc0ac\ub839\uad00\uc778 \uc0e4\ub97c \ud53c\uc288\uadf8\ub93c\uc640 \ucf69\ub370 \uacf5(Prince de Cond\u00e9) \uc0ac\uc774\uc5d0\uc11c \uc655\ub798\ub41c \ubc18\uc5ed\uc5d0 \uad00\ub828\ub41c \uc11c\uc2e0\uc744 \ubc1c\uacac\ud558\uc600\ub2e4. \ubaa8\ub85c\ub294 \uc774\ubbf8 \ubd88\ucda9\ud55c \ud589\uc704\uc5d0 \ub300\ud55c \ube44\ub09c\uc744 \ubc1b\ub358 \ud53c\uc288\uadf8\ub93c\uc758 \uc639\ud638\uc790\uc600\uc5c8\ub2e4. \ubaa8\ub85c\ub294 \uc5b4\ub9ac\uc11d\uac8c\ub3c4 \ubc1c\uacac\ud55c \uc11c\uc2e0\uc5d0 \ub300\ud574 \ub0b4\uc0c9\ud558\uc9c0 \uc54a\uc558\uace0, \uadf8 \uacb0\uacfc \uac00\uc7a5 \uc791\uc9c0\ub9cc \uc77c\ubd80 \uc5f0\ub8e8\ub41c \uc758\uc2ec\uc744 \ubc1b\uac8c \ub418\uc5c8\ub2e4. \ub4a4\ub2a6\uac8c \ubaa8\ub85c\ub294 \ubc18\uc5ed\uc790\ub85c\uc368 \ud53c\uc288\uadf8\ub93c\ub97c \uace0\ubc1c\ud558\ub294 \uc131\uba85\uc11c\ub97c \uad70\uc5d0 \ubc1c\ud589\ud588\uace0, \uacb0\ubc31\uc744 \uc8fc\uc7a5\ud558\ub294 \uc11c\uc2e0\uc744 \ud30c\ub9ac\uc5d0 \ubcf4\ub0c8\ub2e4.", "paragraph_answer": "1797\ub144 \ubaa8\ub85c\ub294 \uc774\ud6c4 \uc790\uae08\uacfc \ubb3c\uc790\uc758 \ubd80\uc871\uc774\ub780 \uc774\uc720\uc5d0\ub3c4 \ubd88\uad6c\ud558\uace0 \ub77c\uc778\uc744 \uac00\ub85c\uc9c8\ub7ec \ub098\uc544\uac14\uc73c\ub098 \ubaa8\ub85c\uc758 \uc791\uc804\uc740 \ubcf4\ub098\ud30c\ub974\ud2b8\uc640 \uc624\uc2a4\ud2b8\ub9ac\uc544 \uc0ac\uc774\uc5d0\uc11c \ub808\uc624\ubcf8 \ud3c9\ud654(Peace of Leoben)\uc758 \uc0ac\uc804 \uc791\uc5c5 \uc758 \uacb0\uacfc\uc5d0 \ub530\ub77c \uc800\uc9c0\ub418\uc5c8\ub2e4. \uc774 \uc2dc\uae30 \ubaa8\ub85c\ub294 \uc624\ub79c \ub3d9\ub8cc\uc774\uc790 \uc0ac\ub839\uad00\uc778 \uc0e4\ub97c \ud53c\uc288\uadf8\ub93c\uc640 \ucf69\ub370 \uacf5(Prince de Cond\u00e9) \uc0ac\uc774\uc5d0\uc11c \uc655\ub798\ub41c \ubc18\uc5ed\uc5d0 \uad00\ub828\ub41c \uc11c\uc2e0\uc744 \ubc1c\uacac\ud558\uc600\ub2e4. \ubaa8\ub85c\ub294 \uc774\ubbf8 \ubd88\ucda9\ud55c \ud589\uc704\uc5d0 \ub300\ud55c \ube44\ub09c\uc744 \ubc1b\ub358 \ud53c\uc288\uadf8\ub93c\uc758 \uc639\ud638\uc790\uc600\uc5c8\ub2e4. \ubaa8\ub85c\ub294 \uc5b4\ub9ac\uc11d\uac8c\ub3c4 \ubc1c\uacac\ud55c \uc11c\uc2e0\uc5d0 \ub300\ud574 \ub0b4\uc0c9\ud558\uc9c0 \uc54a\uc558\uace0, \uadf8 \uacb0\uacfc \uac00\uc7a5 \uc791\uc9c0\ub9cc \uc77c\ubd80 \uc5f0\ub8e8\ub41c \uc758\uc2ec\uc744 \ubc1b\uac8c \ub418\uc5c8\ub2e4. \ub4a4\ub2a6\uac8c \ubaa8\ub85c\ub294 \ubc18\uc5ed\uc790\ub85c\uc368 \ud53c\uc288\uadf8\ub93c\ub97c \uace0\ubc1c\ud558\ub294 \uc131\uba85\uc11c\ub97c \uad70\uc5d0 \ubc1c\ud589\ud588\uace0, \uacb0\ubc31\uc744 \uc8fc\uc7a5\ud558\ub294 \uc11c\uc2e0\uc744 \ud30c\ub9ac\uc5d0 \ubcf4\ub0c8\ub2e4.", "sentence_answer": "\ub098\uc544\uac14\uc73c\ub098 \ubaa8\ub85c\uc758 \uc791\uc804\uc740 \ubcf4\ub098\ud30c\ub974\ud2b8\uc640 \uc624\uc2a4\ud2b8\ub9ac\uc544 \uc0ac\uc774\uc5d0\uc11c \ub808\uc624\ubcf8 \ud3c9\ud654(Peace of Leoben)\uc758 \uc0ac\uc804 \uc791\uc5c5 \uc758 \uacb0\uacfc\uc5d0 \ub530\ub77c \uc800\uc9c0\ub418\uc5c8\ub2e4.", "paragraph_id": "6548931-0-2", "paragraph_question": "question: \ubaa8\ub85c\uc758 \uc791\uc804\uc740 \ubb34\uc5c7\uc758 \uacb0\uacfc\uc5d0 \ub530\ub77c \uc800\uc9c0\ub418\uc5c8\ub294\uac00?, context: 1797\ub144 \ubaa8\ub85c\ub294 \uc774\ud6c4 \uc790\uae08\uacfc \ubb3c\uc790\uc758 \ubd80\uc871\uc774\ub780 \uc774\uc720\uc5d0\ub3c4 \ubd88\uad6c\ud558\uace0 \ub77c\uc778\uc744 \uac00\ub85c\uc9c8\ub7ec \ub098\uc544\uac14\uc73c\ub098 \ubaa8\ub85c\uc758 \uc791\uc804\uc740 \ubcf4\ub098\ud30c\ub974\ud2b8\uc640 \uc624\uc2a4\ud2b8\ub9ac\uc544 \uc0ac\uc774\uc5d0\uc11c \ub808\uc624\ubcf8 \ud3c9\ud654(Peace of Leoben)\uc758 \uc0ac\uc804 \uc791\uc5c5\uc758 \uacb0\uacfc\uc5d0 \ub530\ub77c \uc800\uc9c0\ub418\uc5c8\ub2e4. \uc774 \uc2dc\uae30 \ubaa8\ub85c\ub294 \uc624\ub79c \ub3d9\ub8cc\uc774\uc790 \uc0ac\ub839\uad00\uc778 \uc0e4\ub97c \ud53c\uc288\uadf8\ub93c\uc640 \ucf69\ub370 \uacf5(Prince de Cond\u00e9) \uc0ac\uc774\uc5d0\uc11c \uc655\ub798\ub41c \ubc18\uc5ed\uc5d0 \uad00\ub828\ub41c \uc11c\uc2e0\uc744 \ubc1c\uacac\ud558\uc600\ub2e4. \ubaa8\ub85c\ub294 \uc774\ubbf8 \ubd88\ucda9\ud55c \ud589\uc704\uc5d0 \ub300\ud55c \ube44\ub09c\uc744 \ubc1b\ub358 \ud53c\uc288\uadf8\ub93c\uc758 \uc639\ud638\uc790\uc600\uc5c8\ub2e4. \ubaa8\ub85c\ub294 \uc5b4\ub9ac\uc11d\uac8c\ub3c4 \ubc1c\uacac\ud55c \uc11c\uc2e0\uc5d0 \ub300\ud574 \ub0b4\uc0c9\ud558\uc9c0 \uc54a\uc558\uace0, \uadf8 \uacb0\uacfc \uac00\uc7a5 \uc791\uc9c0\ub9cc \uc77c\ubd80 \uc5f0\ub8e8\ub41c \uc758\uc2ec\uc744 \ubc1b\uac8c \ub418\uc5c8\ub2e4. \ub4a4\ub2a6\uac8c \ubaa8\ub85c\ub294 \ubc18\uc5ed\uc790\ub85c\uc368 \ud53c\uc288\uadf8\ub93c\ub97c \uace0\ubc1c\ud558\ub294 \uc131\uba85\uc11c\ub97c \uad70\uc5d0 \ubc1c\ud589\ud588\uace0, \uacb0\ubc31\uc744 \uc8fc\uc7a5\ud558\ub294 \uc11c\uc2e0\uc744 \ud30c\ub9ac\uc5d0 \ubcf4\ub0c8\ub2e4."} {"question": "1998\ub144 \ub3d9\uacc4 \uc62c\ub9bc\ud53d \uac1c\ucd5c\uad6d\uc740?", "paragraph": "1998\ub144 \ub3d9\uacc4 \uc62c\ub9bc\ud53d\uc740 \uc77c\ubcf8 \ub098\uac00\ub178 \ud604\uc5d0\uc11c \uc5f4\ub838\uc73c\uba70 \ucc98\uc74c\uc73c\ub85c 2,000\uba85\uc774 \ub118\ub294 \uc120\uc218\uac00 \ucd9c\uc804\ud558\uc600\ub2e4. \uadf8\ub9ac\uace0 \uc544\uc774\uc2a4\ud558\ud0a4 \uacbd\uae30\uc5d0\uc11c \ucc98\uc74c\uc73c\ub85c \ud504\ub85c\uc120\uc218\ub4e4\ub3c4 \ucd9c\uc804\ud560 \uc218 \uc788\uac8c \ub418\uc5c8\ub2e4. \uc774\uc5d0 NHL \ucd9c\uc2e0\uc758 \uc7c1\uc7c1\ud55c \uc120\uc218\ub4e4\uc774 \ubab0\ub824\uc788\ub294 \uce90\ub098\ub2e4\uc640 \ubbf8\uad6d \ud300\uc774 \uc720\ub825 \uc6b0\uc2b9\ud6c4\ubcf4\ub85c \ub5a0\uc62c\ub790\uc73c\ub098, \ub450 \uad6d\uac00 \ubaa8\ub450 \uba54\ub2ec \ud68d\ub4dd\uc5d0 \uc2e4\ud328\ud558\uace0 \uccb4\ucf54 \ud300\uc774 \uc6b0\uc2b9\ud558\uc600\ub2e4. \ud55c\ud3b8 \uc774\ubc88 \ub300\ud68c\uc5d0\uc11c \ucc98\uc74c\uc73c\ub85c \ub3c4\uc785\ub41c \uc5ec\uc790 \uc544\uc774\uc2a4\ud558\ud0a4\uc5d0\uc11c\ub294 \ubbf8\uad6d \ud300\uc774 \uccab \uae08\uba54\ub2ec\uc744 \ud68d\ub4dd\ud558\uc600\ub2e4. \ub178\ub974\uc6e8\uc774\uc758 \ube44\uc678\ub978 \ub370\ub9ac\uc5d0 \uc120\uc218\ub294 \uc774\ubc88 \ub300\ud68c \ub178\ub974\ub515\ubcf5\ud569\uc5d0\uc11c 3\uad00\uc655\uc744 \ucc28\uc9c0\ud558\uba74\uc11c \ud1b5\uc0b0 \uae08\uba54\ub2ec 9\uac1c, \uc804\uccb4\uba54\ub2ec 12\uac1c\ub85c \uac00\uc7a5 \uc131\uacf5\ud55c \ub3d9\uacc4\uc62c\ub9bc\ud53d \uc120\uc218\ub85c \uae30\ub85d\ub418\uc5c8\ub2e4. \ub610 \uc624\uc2a4\ud2b8\ub9ac\uc544\uc758 \ud5e4\ub974\ub9cc \ub9c8\uc774\uc5b4\ub294 \ub2e4\uc6b4\ud790 \uacbd\uae30\uc5d0\uc11c \ucda9\ub3cc \uc0ac\uace0\ub97c \uacaa\uc5c8\uc73c\ub098 \uac00\uae4c\uc4f0\ub85c \uc0b4\uc544\ub0a8\uace0 \uc288\ud37c\ub300\ud68c\uc804\uacfc \ub300\ud68c\uc804 \uc885\ubaa9\uc5d0\uc11c \uac01\uac01 \uc6b0\uc2b9\ud558\uc600\ub2e4. \ubbf8\uad6d\uc758 \ud0c0\ub77c \ub9ac\ud540\uc2a4\ud0a4\ub294 \ub9cc 15\uc138\uc758 \ub098\uc774\ub85c \ud53c\uaca8\uc2a4\ucf00\uc774\ud305 \uc5ec\uc790\uc885\ubaa9\uc5d0 \ucd9c\uc804\ud574 1\uc704\ub97c \ucc28\uc9c0\ud558\uba74\uc11c, \uac1c\uc778\uc885\ubaa9\uc73c\ub85c\ub294 \uc5ed\ub300 \ucd5c\uc5f0\uc18c \uae08\uba54\ub2ec\ub9ac\uc2a4\ud2b8\uac00 \ub418\uc5c8\ub2e4. \ub610 \uc2a4\ud53c\ub4dc\uc2a4\ucf00\uc774\ud305\uc5d0\uc11c\ub294 \ud074\ub7a9 \uc2a4\ucf00\uc774\ud2b8 (\ubd84\ub9ac\ud615 \uc2a4\ucf00\uc774\ud2b8)\uac00 \ucc98\uc74c \ub3c4\uc785\ub418\uba74\uc11c \uc138\uacc4\uc2e0\uae30\ub85d\uc774 \uc3df\uc544\uc838\ub098\uc624\uae30\ub3c4 \ud558\uc600\ub2e4.", "answer": "\uc77c\ubcf8", "sentence": "1998\ub144 \ub3d9\uacc4 \uc62c\ub9bc\ud53d\uc740 \uc77c\ubcf8 \ub098\uac00\ub178 \ud604\uc5d0\uc11c \uc5f4\ub838\uc73c\uba70 \ucc98\uc74c\uc73c\ub85c 2,000\uba85\uc774 \ub118\ub294 \uc120\uc218\uac00 \ucd9c\uc804\ud558\uc600\ub2e4.", "paragraph_sentence": " 1998\ub144 \ub3d9\uacc4 \uc62c\ub9bc\ud53d\uc740 \uc77c\ubcf8 \ub098\uac00\ub178 \ud604\uc5d0\uc11c \uc5f4\ub838\uc73c\uba70 \ucc98\uc74c\uc73c\ub85c 2,000\uba85\uc774 \ub118\ub294 \uc120\uc218\uac00 \ucd9c\uc804\ud558\uc600\ub2e4. \uadf8\ub9ac\uace0 \uc544\uc774\uc2a4\ud558\ud0a4 \uacbd\uae30\uc5d0\uc11c \ucc98\uc74c\uc73c\ub85c \ud504\ub85c\uc120\uc218\ub4e4\ub3c4 \ucd9c\uc804\ud560 \uc218 \uc788\uac8c \ub418\uc5c8\ub2e4. \uc774\uc5d0 NHL \ucd9c\uc2e0\uc758 \uc7c1\uc7c1\ud55c \uc120\uc218\ub4e4\uc774 \ubab0\ub824\uc788\ub294 \uce90\ub098\ub2e4\uc640 \ubbf8\uad6d \ud300\uc774 \uc720\ub825 \uc6b0\uc2b9\ud6c4\ubcf4\ub85c \ub5a0\uc62c\ub790\uc73c\ub098, \ub450 \uad6d\uac00 \ubaa8\ub450 \uba54\ub2ec \ud68d\ub4dd\uc5d0 \uc2e4\ud328\ud558\uace0 \uccb4\ucf54 \ud300\uc774 \uc6b0\uc2b9\ud558\uc600\ub2e4. \ud55c\ud3b8 \uc774\ubc88 \ub300\ud68c\uc5d0\uc11c \ucc98\uc74c\uc73c\ub85c \ub3c4\uc785\ub41c \uc5ec\uc790 \uc544\uc774\uc2a4\ud558\ud0a4\uc5d0\uc11c\ub294 \ubbf8\uad6d \ud300\uc774 \uccab \uae08\uba54\ub2ec\uc744 \ud68d\ub4dd\ud558\uc600\ub2e4. \ub178\ub974\uc6e8\uc774\uc758 \ube44\uc678\ub978 \ub370\ub9ac\uc5d0 \uc120\uc218\ub294 \uc774\ubc88 \ub300\ud68c \ub178\ub974\ub515\ubcf5\ud569\uc5d0\uc11c 3\uad00\uc655\uc744 \ucc28\uc9c0\ud558\uba74\uc11c \ud1b5\uc0b0 \uae08\uba54\ub2ec 9\uac1c, \uc804\uccb4\uba54\ub2ec 12\uac1c\ub85c \uac00\uc7a5 \uc131\uacf5\ud55c \ub3d9\uacc4\uc62c\ub9bc\ud53d \uc120\uc218\ub85c \uae30\ub85d\ub418\uc5c8\ub2e4. \ub610 \uc624\uc2a4\ud2b8\ub9ac\uc544\uc758 \ud5e4\ub974\ub9cc \ub9c8\uc774\uc5b4\ub294 \ub2e4\uc6b4\ud790 \uacbd\uae30\uc5d0\uc11c \ucda9\ub3cc \uc0ac\uace0\ub97c \uacaa\uc5c8\uc73c\ub098 \uac00\uae4c\uc4f0\ub85c \uc0b4\uc544\ub0a8\uace0 \uc288\ud37c\ub300\ud68c\uc804\uacfc \ub300\ud68c\uc804 \uc885\ubaa9\uc5d0\uc11c \uac01\uac01 \uc6b0\uc2b9\ud558\uc600\ub2e4. \ubbf8\uad6d\uc758 \ud0c0\ub77c \ub9ac\ud540\uc2a4\ud0a4\ub294 \ub9cc 15\uc138\uc758 \ub098\uc774\ub85c \ud53c\uaca8\uc2a4\ucf00\uc774\ud305 \uc5ec\uc790\uc885\ubaa9\uc5d0 \ucd9c\uc804\ud574 1\uc704\ub97c \ucc28\uc9c0\ud558\uba74\uc11c, \uac1c\uc778\uc885\ubaa9\uc73c\ub85c\ub294 \uc5ed\ub300 \ucd5c\uc5f0\uc18c \uae08\uba54\ub2ec\ub9ac\uc2a4\ud2b8\uac00 \ub418\uc5c8\ub2e4. \ub610 \uc2a4\ud53c\ub4dc\uc2a4\ucf00\uc774\ud305\uc5d0\uc11c\ub294 \ud074\ub7a9 \uc2a4\ucf00\uc774\ud2b8 (\ubd84\ub9ac\ud615 \uc2a4\ucf00\uc774\ud2b8)\uac00 \ucc98\uc74c \ub3c4\uc785\ub418\uba74\uc11c \uc138\uacc4\uc2e0\uae30\ub85d\uc774 \uc3df\uc544\uc838\ub098\uc624\uae30\ub3c4 \ud558\uc600\ub2e4.", "paragraph_answer": "1998\ub144 \ub3d9\uacc4 \uc62c\ub9bc\ud53d\uc740 \uc77c\ubcf8 \ub098\uac00\ub178 \ud604\uc5d0\uc11c \uc5f4\ub838\uc73c\uba70 \ucc98\uc74c\uc73c\ub85c 2,000\uba85\uc774 \ub118\ub294 \uc120\uc218\uac00 \ucd9c\uc804\ud558\uc600\ub2e4. \uadf8\ub9ac\uace0 \uc544\uc774\uc2a4\ud558\ud0a4 \uacbd\uae30\uc5d0\uc11c \ucc98\uc74c\uc73c\ub85c \ud504\ub85c\uc120\uc218\ub4e4\ub3c4 \ucd9c\uc804\ud560 \uc218 \uc788\uac8c \ub418\uc5c8\ub2e4. \uc774\uc5d0 NHL \ucd9c\uc2e0\uc758 \uc7c1\uc7c1\ud55c \uc120\uc218\ub4e4\uc774 \ubab0\ub824\uc788\ub294 \uce90\ub098\ub2e4\uc640 \ubbf8\uad6d \ud300\uc774 \uc720\ub825 \uc6b0\uc2b9\ud6c4\ubcf4\ub85c \ub5a0\uc62c\ub790\uc73c\ub098, \ub450 \uad6d\uac00 \ubaa8\ub450 \uba54\ub2ec \ud68d\ub4dd\uc5d0 \uc2e4\ud328\ud558\uace0 \uccb4\ucf54 \ud300\uc774 \uc6b0\uc2b9\ud558\uc600\ub2e4. \ud55c\ud3b8 \uc774\ubc88 \ub300\ud68c\uc5d0\uc11c \ucc98\uc74c\uc73c\ub85c \ub3c4\uc785\ub41c \uc5ec\uc790 \uc544\uc774\uc2a4\ud558\ud0a4\uc5d0\uc11c\ub294 \ubbf8\uad6d \ud300\uc774 \uccab \uae08\uba54\ub2ec\uc744 \ud68d\ub4dd\ud558\uc600\ub2e4. \ub178\ub974\uc6e8\uc774\uc758 \ube44\uc678\ub978 \ub370\ub9ac\uc5d0 \uc120\uc218\ub294 \uc774\ubc88 \ub300\ud68c \ub178\ub974\ub515\ubcf5\ud569\uc5d0\uc11c 3\uad00\uc655\uc744 \ucc28\uc9c0\ud558\uba74\uc11c \ud1b5\uc0b0 \uae08\uba54\ub2ec 9\uac1c, \uc804\uccb4\uba54\ub2ec 12\uac1c\ub85c \uac00\uc7a5 \uc131\uacf5\ud55c \ub3d9\uacc4\uc62c\ub9bc\ud53d \uc120\uc218\ub85c \uae30\ub85d\ub418\uc5c8\ub2e4. \ub610 \uc624\uc2a4\ud2b8\ub9ac\uc544\uc758 \ud5e4\ub974\ub9cc \ub9c8\uc774\uc5b4\ub294 \ub2e4\uc6b4\ud790 \uacbd\uae30\uc5d0\uc11c \ucda9\ub3cc \uc0ac\uace0\ub97c \uacaa\uc5c8\uc73c\ub098 \uac00\uae4c\uc4f0\ub85c \uc0b4\uc544\ub0a8\uace0 \uc288\ud37c\ub300\ud68c\uc804\uacfc \ub300\ud68c\uc804 \uc885\ubaa9\uc5d0\uc11c \uac01\uac01 \uc6b0\uc2b9\ud558\uc600\ub2e4. \ubbf8\uad6d\uc758 \ud0c0\ub77c \ub9ac\ud540\uc2a4\ud0a4\ub294 \ub9cc 15\uc138\uc758 \ub098\uc774\ub85c \ud53c\uaca8\uc2a4\ucf00\uc774\ud305 \uc5ec\uc790\uc885\ubaa9\uc5d0 \ucd9c\uc804\ud574 1\uc704\ub97c \ucc28\uc9c0\ud558\uba74\uc11c, \uac1c\uc778\uc885\ubaa9\uc73c\ub85c\ub294 \uc5ed\ub300 \ucd5c\uc5f0\uc18c \uae08\uba54\ub2ec\ub9ac\uc2a4\ud2b8\uac00 \ub418\uc5c8\ub2e4. \ub610 \uc2a4\ud53c\ub4dc\uc2a4\ucf00\uc774\ud305\uc5d0\uc11c\ub294 \ud074\ub7a9 \uc2a4\ucf00\uc774\ud2b8 (\ubd84\ub9ac\ud615 \uc2a4\ucf00\uc774\ud2b8)\uac00 \ucc98\uc74c \ub3c4\uc785\ub418\uba74\uc11c \uc138\uacc4\uc2e0\uae30\ub85d\uc774 \uc3df\uc544\uc838\ub098\uc624\uae30\ub3c4 \ud558\uc600\ub2e4.", "sentence_answer": "1998\ub144 \ub3d9\uacc4 \uc62c\ub9bc\ud53d\uc740 \uc77c\ubcf8 \ub098\uac00\ub178 \ud604\uc5d0\uc11c \uc5f4\ub838\uc73c\uba70 \ucc98\uc74c\uc73c\ub85c 2,000\uba85\uc774 \ub118\ub294 \uc120\uc218\uac00 \ucd9c\uc804\ud558\uc600\ub2e4.", "paragraph_id": "6656930-13-0", "paragraph_question": "question: 1998\ub144 \ub3d9\uacc4 \uc62c\ub9bc\ud53d \uac1c\ucd5c\uad6d\uc740?, context: 1998\ub144 \ub3d9\uacc4 \uc62c\ub9bc\ud53d\uc740 \uc77c\ubcf8 \ub098\uac00\ub178 \ud604\uc5d0\uc11c \uc5f4\ub838\uc73c\uba70 \ucc98\uc74c\uc73c\ub85c 2,000\uba85\uc774 \ub118\ub294 \uc120\uc218\uac00 \ucd9c\uc804\ud558\uc600\ub2e4. \uadf8\ub9ac\uace0 \uc544\uc774\uc2a4\ud558\ud0a4 \uacbd\uae30\uc5d0\uc11c \ucc98\uc74c\uc73c\ub85c \ud504\ub85c\uc120\uc218\ub4e4\ub3c4 \ucd9c\uc804\ud560 \uc218 \uc788\uac8c \ub418\uc5c8\ub2e4. \uc774\uc5d0 NHL \ucd9c\uc2e0\uc758 \uc7c1\uc7c1\ud55c \uc120\uc218\ub4e4\uc774 \ubab0\ub824\uc788\ub294 \uce90\ub098\ub2e4\uc640 \ubbf8\uad6d \ud300\uc774 \uc720\ub825 \uc6b0\uc2b9\ud6c4\ubcf4\ub85c \ub5a0\uc62c\ub790\uc73c\ub098, \ub450 \uad6d\uac00 \ubaa8\ub450 \uba54\ub2ec \ud68d\ub4dd\uc5d0 \uc2e4\ud328\ud558\uace0 \uccb4\ucf54 \ud300\uc774 \uc6b0\uc2b9\ud558\uc600\ub2e4. \ud55c\ud3b8 \uc774\ubc88 \ub300\ud68c\uc5d0\uc11c \ucc98\uc74c\uc73c\ub85c \ub3c4\uc785\ub41c \uc5ec\uc790 \uc544\uc774\uc2a4\ud558\ud0a4\uc5d0\uc11c\ub294 \ubbf8\uad6d \ud300\uc774 \uccab \uae08\uba54\ub2ec\uc744 \ud68d\ub4dd\ud558\uc600\ub2e4. \ub178\ub974\uc6e8\uc774\uc758 \ube44\uc678\ub978 \ub370\ub9ac\uc5d0 \uc120\uc218\ub294 \uc774\ubc88 \ub300\ud68c \ub178\ub974\ub515\ubcf5\ud569\uc5d0\uc11c 3\uad00\uc655\uc744 \ucc28\uc9c0\ud558\uba74\uc11c \ud1b5\uc0b0 \uae08\uba54\ub2ec 9\uac1c, \uc804\uccb4\uba54\ub2ec 12\uac1c\ub85c \uac00\uc7a5 \uc131\uacf5\ud55c \ub3d9\uacc4\uc62c\ub9bc\ud53d \uc120\uc218\ub85c \uae30\ub85d\ub418\uc5c8\ub2e4. \ub610 \uc624\uc2a4\ud2b8\ub9ac\uc544\uc758 \ud5e4\ub974\ub9cc \ub9c8\uc774\uc5b4\ub294 \ub2e4\uc6b4\ud790 \uacbd\uae30\uc5d0\uc11c \ucda9\ub3cc \uc0ac\uace0\ub97c \uacaa\uc5c8\uc73c\ub098 \uac00\uae4c\uc4f0\ub85c \uc0b4\uc544\ub0a8\uace0 \uc288\ud37c\ub300\ud68c\uc804\uacfc \ub300\ud68c\uc804 \uc885\ubaa9\uc5d0\uc11c \uac01\uac01 \uc6b0\uc2b9\ud558\uc600\ub2e4. \ubbf8\uad6d\uc758 \ud0c0\ub77c \ub9ac\ud540\uc2a4\ud0a4\ub294 \ub9cc 15\uc138\uc758 \ub098\uc774\ub85c \ud53c\uaca8\uc2a4\ucf00\uc774\ud305 \uc5ec\uc790\uc885\ubaa9\uc5d0 \ucd9c\uc804\ud574 1\uc704\ub97c \ucc28\uc9c0\ud558\uba74\uc11c, \uac1c\uc778\uc885\ubaa9\uc73c\ub85c\ub294 \uc5ed\ub300 \ucd5c\uc5f0\uc18c \uae08\uba54\ub2ec\ub9ac\uc2a4\ud2b8\uac00 \ub418\uc5c8\ub2e4. \ub610 \uc2a4\ud53c\ub4dc\uc2a4\ucf00\uc774\ud305\uc5d0\uc11c\ub294 \ud074\ub7a9 \uc2a4\ucf00\uc774\ud2b8 (\ubd84\ub9ac\ud615 \uc2a4\ucf00\uc774\ud2b8)\uac00 \ucc98\uc74c \ub3c4\uc785\ub418\uba74\uc11c \uc138\uacc4\uc2e0\uae30\ub85d\uc774 \uc3df\uc544\uc838\ub098\uc624\uae30\ub3c4 \ud558\uc600\ub2e4."} {"question": "\uc720\ub7fd\ucc0c\ub974\ub808\uae30\uc758 \uc18c\ub9ac\ub97c \ubaa8\ubc29\ud574\uc11c \ub9cc\ub4e4\uc5c8\ub2e4\ub294 \uc804\uc124\uc758 \ubaa8\ucc28\ub974\ud2b8 \uc74c\uc545\uc740?", "paragraph": "\ubcfc\ud504\uac15 \uc544\ub9c8\ub370\uc6b0\uc2a4 \ubaa8\ucc28\ub974\ud2b8\ub294 \uc720\ub7fd\ucc0c\ub974\ub808\uae30 \ud55c \ub9c8\ub9ac\ub97c \uc560\uc644\ub3d9\ubb3c\ub85c \uae38\ub800\ub294\ub370, \uc774 \ucc0c\ub974\ub808\uae30\ub294 G\uc7a5\uc870 \ud53c\uc544\ub178 \ud611\uc8fc\uace1\uc758 \ud55c \ubd80\ubd84\uc744 \ub530\ub77c \ubd80\ub97c \uc218 \uc788\uc5c8\ub2e4\uace0 \ud55c\ub2e4. \ubaa8\ucc28\ub974\ud2b8\ub294 \ud55c \uac00\uac8c\uc5d0\uc11c \uc0c8\uac00 6\uc8fc \uc804\uc5d0 \uc790\uc2e0\uc774 \uc37c\uc9c0\ub9cc \uc544\uc9c1 \uacf5\uc911\uc5d0 \ubc1c\ud45c\ub294 \ud558\uc9c0 \ubabb\ud55c \ub178\ub798 \uad6c\uc808\uc744 \ub530\ub77c \ubd80\ub974\ub294 \uac83\uc744 \ub4e3\uace0 \uc0c8\ub97c \uc0c0\ub2e4. \uadf8\ub294 \uc0c8\ub97c \ub9e4\uc6b0 \uc544\uaf08\uc73c\uba70 3\ub144 \ub4a4 \uc8fd\uc790 \uc815\uc131\uaecf \uc7a5\uc0ac\uc9c0\ub0b4 \uc8fc\uc5c8\ub2e4. \ubaa8\ucc28\ub974\ud2b8\uc758 \u300a\uc74c\uc545\uc801 \ub18d\ub2f4\u300b \uc774 \ucc0c\ub974\ub808\uae30\uc758 \uc6b0\uc2a4\uaf5d\uc2a4\ub7ec\uc6b4 \uc18c\ub9ac\ub97c \ubaa8\ubc29\ud55c \uac83\uc774\ub77c\ub294 \uc124\ub3c4 \uc788\ub2e4. \uadf8 \uc678\uc5d0 \uc720\ub7fd\ucc0c\ub974\ub808\uae30\ub97c \uc18c\uc720\ud55c \uc0ac\ub78c\ub4e4\ub3c4 \uc0c8\uac00 \ub2e4\uc591\ud55c \uad6c\ubb38\uc774\ub098 \ud45c\ud604\uc744 \uace8\ub77c\ub0b4\ub294 \ud559\uc2b5\uc744 \ud588\uc74c\uc744 \ubc1d\ud78c \ubc14 \uc788\ub2e4. \ucc0c\ub974\ub808\uae30\ub294 \ub0b1\ub9d0\uc758 \uc758\ubbf8\ub97c \uc774\ud574\ud560 \uc218 \uc5c6\uae30 \ub54c\ubb38\uc5d0 \uc544\ubb34\ub807\uac8c\ub098 \uc11e\uc5b4 \ub9d0\ud558\uac70\ub098 \uc0c8\uc18c\ub9ac\ub97c \ub0b4\ub2e4\uac00 \uc911\uac04\uc5d0 \ub72c\uae08\uc5c6\uc774 \uc0ac\ub78c \ub9d0\uc18c\ub9ac\ub97c \ud749\ub0b4\ub0b4\ub294 \ub4f1\uc758 \ubaa8\uc2b5\uc744 \ubcf4\uc5ec\uc900\ub2e4. \uc720\ub7fd\ucc0c\ub974\ub808\uae30\uc758 \ubaa8\ubc29 \ub2a5\ub825\uc740 \ub108\ubb34 \ub300\ub2e8\ud574\uc11c, \uc0ac\uc815\uc744 \ubaa8\ub974\ub294 \uc0ac\ub78c\uc740 \uadf8 \uc18c\ub9ac\uc5d0 \uae5c\ube61 \uc18d\uc544 \ub9d0\ud55c \uc0ac\ub78c\uc744 \ud5db\ub418\uc774 \ucc3e\uc544\ubcf4\uae30\ub3c4 \ud55c\ub2e4.", "answer": "\uc74c\uc545\uc801 \ub18d\ub2f4", "sentence": "\ubaa8\ucc28\ub974\ud2b8\uc758 \u300a \uc74c\uc545\uc801 \ub18d\ub2f4 \u300b \uc774 \ucc0c\ub974\ub808\uae30\uc758 \uc6b0\uc2a4\uaf5d\uc2a4\ub7ec\uc6b4 \uc18c\ub9ac\ub97c \ubaa8\ubc29\ud55c \uac83\uc774\ub77c\ub294 \uc124\ub3c4 \uc788\ub2e4.", "paragraph_sentence": "\ubcfc\ud504\uac15 \uc544\ub9c8\ub370\uc6b0\uc2a4 \ubaa8\ucc28\ub974\ud2b8\ub294 \uc720\ub7fd\ucc0c\ub974\ub808\uae30 \ud55c \ub9c8\ub9ac\ub97c \uc560\uc644\ub3d9\ubb3c\ub85c \uae38\ub800\ub294\ub370, \uc774 \ucc0c\ub974\ub808\uae30\ub294 G\uc7a5\uc870 \ud53c\uc544\ub178 \ud611\uc8fc\uace1\uc758 \ud55c \ubd80\ubd84\uc744 \ub530\ub77c \ubd80\ub97c \uc218 \uc788\uc5c8\ub2e4\uace0 \ud55c\ub2e4. \ubaa8\ucc28\ub974\ud2b8\ub294 \ud55c \uac00\uac8c\uc5d0\uc11c \uc0c8\uac00 6\uc8fc \uc804\uc5d0 \uc790\uc2e0\uc774 \uc37c\uc9c0\ub9cc \uc544\uc9c1 \uacf5\uc911\uc5d0 \ubc1c\ud45c\ub294 \ud558\uc9c0 \ubabb\ud55c \ub178\ub798 \uad6c\uc808\uc744 \ub530\ub77c \ubd80\ub974\ub294 \uac83\uc744 \ub4e3\uace0 \uc0c8\ub97c \uc0c0\ub2e4. \uadf8\ub294 \uc0c8\ub97c \ub9e4\uc6b0 \uc544\uaf08\uc73c\uba70 3\ub144 \ub4a4 \uc8fd\uc790 \uc815\uc131\uaecf \uc7a5\uc0ac\uc9c0\ub0b4 \uc8fc\uc5c8\ub2e4. \ubaa8\ucc28\ub974\ud2b8\uc758 \u300a \uc74c\uc545\uc801 \ub18d\ub2f4 \u300b \uc774 \ucc0c\ub974\ub808\uae30\uc758 \uc6b0\uc2a4\uaf5d\uc2a4\ub7ec\uc6b4 \uc18c\ub9ac\ub97c \ubaa8\ubc29\ud55c \uac83\uc774\ub77c\ub294 \uc124\ub3c4 \uc788\ub2e4. \uadf8 \uc678\uc5d0 \uc720\ub7fd\ucc0c\ub974\ub808\uae30\ub97c \uc18c\uc720\ud55c \uc0ac\ub78c\ub4e4\ub3c4 \uc0c8\uac00 \ub2e4\uc591\ud55c \uad6c\ubb38\uc774\ub098 \ud45c\ud604\uc744 \uace8\ub77c\ub0b4\ub294 \ud559\uc2b5\uc744 \ud588\uc74c\uc744 \ubc1d\ud78c \ubc14 \uc788\ub2e4. \ucc0c\ub974\ub808\uae30\ub294 \ub0b1\ub9d0\uc758 \uc758\ubbf8\ub97c \uc774\ud574\ud560 \uc218 \uc5c6\uae30 \ub54c\ubb38\uc5d0 \uc544\ubb34\ub807\uac8c\ub098 \uc11e\uc5b4 \ub9d0\ud558\uac70\ub098 \uc0c8\uc18c\ub9ac\ub97c \ub0b4\ub2e4\uac00 \uc911\uac04\uc5d0 \ub72c\uae08\uc5c6\uc774 \uc0ac\ub78c \ub9d0\uc18c\ub9ac\ub97c \ud749\ub0b4\ub0b4\ub294 \ub4f1\uc758 \ubaa8\uc2b5\uc744 \ubcf4\uc5ec\uc900\ub2e4. \uc720\ub7fd\ucc0c\ub974\ub808\uae30\uc758 \ubaa8\ubc29 \ub2a5\ub825\uc740 \ub108\ubb34 \ub300\ub2e8\ud574\uc11c, \uc0ac\uc815\uc744 \ubaa8\ub974\ub294 \uc0ac\ub78c\uc740 \uadf8 \uc18c\ub9ac\uc5d0 \uae5c\ube61 \uc18d\uc544 \ub9d0\ud55c \uc0ac\ub78c\uc744 \ud5db\ub418\uc774 \ucc3e\uc544\ubcf4\uae30\ub3c4 \ud55c\ub2e4.", "paragraph_answer": "\ubcfc\ud504\uac15 \uc544\ub9c8\ub370\uc6b0\uc2a4 \ubaa8\ucc28\ub974\ud2b8\ub294 \uc720\ub7fd\ucc0c\ub974\ub808\uae30 \ud55c \ub9c8\ub9ac\ub97c \uc560\uc644\ub3d9\ubb3c\ub85c \uae38\ub800\ub294\ub370, \uc774 \ucc0c\ub974\ub808\uae30\ub294 G\uc7a5\uc870 \ud53c\uc544\ub178 \ud611\uc8fc\uace1\uc758 \ud55c \ubd80\ubd84\uc744 \ub530\ub77c \ubd80\ub97c \uc218 \uc788\uc5c8\ub2e4\uace0 \ud55c\ub2e4. \ubaa8\ucc28\ub974\ud2b8\ub294 \ud55c \uac00\uac8c\uc5d0\uc11c \uc0c8\uac00 6\uc8fc \uc804\uc5d0 \uc790\uc2e0\uc774 \uc37c\uc9c0\ub9cc \uc544\uc9c1 \uacf5\uc911\uc5d0 \ubc1c\ud45c\ub294 \ud558\uc9c0 \ubabb\ud55c \ub178\ub798 \uad6c\uc808\uc744 \ub530\ub77c \ubd80\ub974\ub294 \uac83\uc744 \ub4e3\uace0 \uc0c8\ub97c \uc0c0\ub2e4. \uadf8\ub294 \uc0c8\ub97c \ub9e4\uc6b0 \uc544\uaf08\uc73c\uba70 3\ub144 \ub4a4 \uc8fd\uc790 \uc815\uc131\uaecf \uc7a5\uc0ac\uc9c0\ub0b4 \uc8fc\uc5c8\ub2e4. \ubaa8\ucc28\ub974\ud2b8\uc758 \u300a \uc74c\uc545\uc801 \ub18d\ub2f4 \u300b \uc774 \ucc0c\ub974\ub808\uae30\uc758 \uc6b0\uc2a4\uaf5d\uc2a4\ub7ec\uc6b4 \uc18c\ub9ac\ub97c \ubaa8\ubc29\ud55c \uac83\uc774\ub77c\ub294 \uc124\ub3c4 \uc788\ub2e4. \uadf8 \uc678\uc5d0 \uc720\ub7fd\ucc0c\ub974\ub808\uae30\ub97c \uc18c\uc720\ud55c \uc0ac\ub78c\ub4e4\ub3c4 \uc0c8\uac00 \ub2e4\uc591\ud55c \uad6c\ubb38\uc774\ub098 \ud45c\ud604\uc744 \uace8\ub77c\ub0b4\ub294 \ud559\uc2b5\uc744 \ud588\uc74c\uc744 \ubc1d\ud78c \ubc14 \uc788\ub2e4. \ucc0c\ub974\ub808\uae30\ub294 \ub0b1\ub9d0\uc758 \uc758\ubbf8\ub97c \uc774\ud574\ud560 \uc218 \uc5c6\uae30 \ub54c\ubb38\uc5d0 \uc544\ubb34\ub807\uac8c\ub098 \uc11e\uc5b4 \ub9d0\ud558\uac70\ub098 \uc0c8\uc18c\ub9ac\ub97c \ub0b4\ub2e4\uac00 \uc911\uac04\uc5d0 \ub72c\uae08\uc5c6\uc774 \uc0ac\ub78c \ub9d0\uc18c\ub9ac\ub97c \ud749\ub0b4\ub0b4\ub294 \ub4f1\uc758 \ubaa8\uc2b5\uc744 \ubcf4\uc5ec\uc900\ub2e4. \uc720\ub7fd\ucc0c\ub974\ub808\uae30\uc758 \ubaa8\ubc29 \ub2a5\ub825\uc740 \ub108\ubb34 \ub300\ub2e8\ud574\uc11c, \uc0ac\uc815\uc744 \ubaa8\ub974\ub294 \uc0ac\ub78c\uc740 \uadf8 \uc18c\ub9ac\uc5d0 \uae5c\ube61 \uc18d\uc544 \ub9d0\ud55c \uc0ac\ub78c\uc744 \ud5db\ub418\uc774 \ucc3e\uc544\ubcf4\uae30\ub3c4 \ud55c\ub2e4.", "sentence_answer": "\ubaa8\ucc28\ub974\ud2b8\uc758 \u300a \uc74c\uc545\uc801 \ub18d\ub2f4 \u300b \uc774 \ucc0c\ub974\ub808\uae30\uc758 \uc6b0\uc2a4\uaf5d\uc2a4\ub7ec\uc6b4 \uc18c\ub9ac\ub97c \ubaa8\ubc29\ud55c \uac83\uc774\ub77c\ub294 \uc124\ub3c4 \uc788\ub2e4.", "paragraph_id": "6542858-31-1", "paragraph_question": "question: \uc720\ub7fd\ucc0c\ub974\ub808\uae30\uc758 \uc18c\ub9ac\ub97c \ubaa8\ubc29\ud574\uc11c \ub9cc\ub4e4\uc5c8\ub2e4\ub294 \uc804\uc124\uc758 \ubaa8\ucc28\ub974\ud2b8 \uc74c\uc545\uc740?, context: \ubcfc\ud504\uac15 \uc544\ub9c8\ub370\uc6b0\uc2a4 \ubaa8\ucc28\ub974\ud2b8\ub294 \uc720\ub7fd\ucc0c\ub974\ub808\uae30 \ud55c \ub9c8\ub9ac\ub97c \uc560\uc644\ub3d9\ubb3c\ub85c \uae38\ub800\ub294\ub370, \uc774 \ucc0c\ub974\ub808\uae30\ub294 G\uc7a5\uc870 \ud53c\uc544\ub178 \ud611\uc8fc\uace1\uc758 \ud55c \ubd80\ubd84\uc744 \ub530\ub77c \ubd80\ub97c \uc218 \uc788\uc5c8\ub2e4\uace0 \ud55c\ub2e4. \ubaa8\ucc28\ub974\ud2b8\ub294 \ud55c \uac00\uac8c\uc5d0\uc11c \uc0c8\uac00 6\uc8fc \uc804\uc5d0 \uc790\uc2e0\uc774 \uc37c\uc9c0\ub9cc \uc544\uc9c1 \uacf5\uc911\uc5d0 \ubc1c\ud45c\ub294 \ud558\uc9c0 \ubabb\ud55c \ub178\ub798 \uad6c\uc808\uc744 \ub530\ub77c \ubd80\ub974\ub294 \uac83\uc744 \ub4e3\uace0 \uc0c8\ub97c \uc0c0\ub2e4. \uadf8\ub294 \uc0c8\ub97c \ub9e4\uc6b0 \uc544\uaf08\uc73c\uba70 3\ub144 \ub4a4 \uc8fd\uc790 \uc815\uc131\uaecf \uc7a5\uc0ac\uc9c0\ub0b4 \uc8fc\uc5c8\ub2e4. \ubaa8\ucc28\ub974\ud2b8\uc758 \u300a\uc74c\uc545\uc801 \ub18d\ub2f4\u300b \uc774 \ucc0c\ub974\ub808\uae30\uc758 \uc6b0\uc2a4\uaf5d\uc2a4\ub7ec\uc6b4 \uc18c\ub9ac\ub97c \ubaa8\ubc29\ud55c \uac83\uc774\ub77c\ub294 \uc124\ub3c4 \uc788\ub2e4. \uadf8 \uc678\uc5d0 \uc720\ub7fd\ucc0c\ub974\ub808\uae30\ub97c \uc18c\uc720\ud55c \uc0ac\ub78c\ub4e4\ub3c4 \uc0c8\uac00 \ub2e4\uc591\ud55c \uad6c\ubb38\uc774\ub098 \ud45c\ud604\uc744 \uace8\ub77c\ub0b4\ub294 \ud559\uc2b5\uc744 \ud588\uc74c\uc744 \ubc1d\ud78c \ubc14 \uc788\ub2e4. \ucc0c\ub974\ub808\uae30\ub294 \ub0b1\ub9d0\uc758 \uc758\ubbf8\ub97c \uc774\ud574\ud560 \uc218 \uc5c6\uae30 \ub54c\ubb38\uc5d0 \uc544\ubb34\ub807\uac8c\ub098 \uc11e\uc5b4 \ub9d0\ud558\uac70\ub098 \uc0c8\uc18c\ub9ac\ub97c \ub0b4\ub2e4\uac00 \uc911\uac04\uc5d0 \ub72c\uae08\uc5c6\uc774 \uc0ac\ub78c \ub9d0\uc18c\ub9ac\ub97c \ud749\ub0b4\ub0b4\ub294 \ub4f1\uc758 \ubaa8\uc2b5\uc744 \ubcf4\uc5ec\uc900\ub2e4. \uc720\ub7fd\ucc0c\ub974\ub808\uae30\uc758 \ubaa8\ubc29 \ub2a5\ub825\uc740 \ub108\ubb34 \ub300\ub2e8\ud574\uc11c, \uc0ac\uc815\uc744 \ubaa8\ub974\ub294 \uc0ac\ub78c\uc740 \uadf8 \uc18c\ub9ac\uc5d0 \uae5c\ube61 \uc18d\uc544 \ub9d0\ud55c \uc0ac\ub78c\uc744 \ud5db\ub418\uc774 \ucc3e\uc544\ubcf4\uae30\ub3c4 \ud55c\ub2e4."} {"question": "4\uc138\uae30\uc5d0 \ud65c\ub3d9\ud55c \ubc95\ub960\uac00 \uc774\uc790 \ubc00\ub77c\ub178\uc758 \uc8fc\uad50\uc778 \uc11c\ubc29 \uad50\ud68c\uc758 4\ub300 \uad50\ubd80 \uc911 \ud55c \uc0ac\ub78c\uc740?", "paragraph": "\uc554\ube0c\ub85c\uc2dc\uc6b0\uc2a4(\ub77c\ud2f4\uc5b4: Sanctus Ambrosius, 340\ub144?~397\ub144 4\uc6d4 4\uc77c) \ub610\ub294 \uc131 \uc554\ube0c\ub85c\uc2a4(Ambrose)\ub294 4\uc138\uae30\uc5d0 \ud65c\ub3d9\ud55c \uc11c\ubc29 \uad50\ud68c\uc758 4\ub300 \uad50\ubd80 \uc911 \ud55c \uc0ac\ub78c\uc73c\ub85c\uc11c \ubc95\ub960\uac00\uc774\uc790 \ubc00\ub77c\ub178\uc758 \uc8fc\uad50\uc774\ub2e4. \uc544\ub9ac\uc6b0\uc2a4\ud30c\uc5d0 \ub9de\uc11c \uc815\ud1b5 \uae30\ub3c5\uad50\uc758 \uc804\ub840\uc640 \uc131\uc9c1\uc5d0 \ub300\ud55c \uac1c\ud601\uc744 \uc774\ub8e9\ud55c \uc0ac\ub78c\uc73c\ub85c \uc798 \uc54c\ub824\uc838 \uc788\ub2e4. \uae30\ub3c5\uad50\uc758 \uc131\uc778\uc774\uba70 \uad50\ud68c\ubc15\uc0ac \uac00\uc6b4\ub370 \ud55c \uc0ac\ub78c\uc774\ub2e4. \ub85c\ub9c8 \uac00\ud1a8\ub9ad\uc5d0\uc11c\uc758 \ucd95\uc77c\uc740 12\uc6d4 7\uc77c\uc774\uba70, \uc131\uacf5\ud68c\uc5d0\uc11c\uc758 \ucd95\uc77c\uc740 4\uc6d4 4\uc77c\uc774\ub2e4. \uc554\ube0c\ub85c\uc2dc\uc6b0\uc2a4\ub294 \uadf8\ub9ac\uc2a4\uc5b4\uc5d0\uc11c \uc720\ub798\ud55c \uc774\ub984\uc73c\ub85c \u2018\ubd88\uba78\u2019\uc744 \ub73b\ud55c\ub2e4. \ubbf8\uc220\uc791\ud488\uc5d0\uc11c \uadf8\ub294 \ud754\ud788 \ubaa9\uc7a5\uc744 \ub4e0 \uc8fc\uad50\uc758 \ubaa8\uc2b5\uc73c\ub85c \ud45c\ud604\ub418\ub294\ub370, \ub54c\ub54c\ub85c \uc2b9\ub9c8\uc6a9 \ucc44\ucc0d\uc744 \ud718\ub450\ub974\uba74\uc11c \ub9d0\uc744 \ud0c0\uace0 \uc788\uac70\ub098 \uafc0\ubc8c\uc744 \uac70\ub290\ub9ac\uace0 \uc788\uae30\ub3c4 \ud55c\ub2e4.", "answer": "\uc554\ube0c\ub85c\uc2dc\uc6b0\uc2a4", "sentence": "\uc554\ube0c\ub85c\uc2dc\uc6b0\uc2a4 (\ub77c\ud2f4\uc5b4: Sanctus Ambrosius, 340\ub144?~397\ub144 4\uc6d4 4\uc77c)", "paragraph_sentence": " \uc554\ube0c\ub85c\uc2dc\uc6b0\uc2a4 (\ub77c\ud2f4\uc5b4: Sanctus Ambrosius, 340\ub144?~397\ub144 4\uc6d4 4\uc77c) \ub610\ub294 \uc131 \uc554\ube0c\ub85c\uc2a4(Ambrose)\ub294 4\uc138\uae30\uc5d0 \ud65c\ub3d9\ud55c \uc11c\ubc29 \uad50\ud68c\uc758 4\ub300 \uad50\ubd80 \uc911 \ud55c \uc0ac\ub78c\uc73c\ub85c\uc11c \ubc95\ub960\uac00\uc774\uc790 \ubc00\ub77c\ub178\uc758 \uc8fc\uad50\uc774\ub2e4. \uc544\ub9ac\uc6b0\uc2a4\ud30c\uc5d0 \ub9de\uc11c \uc815\ud1b5 \uae30\ub3c5\uad50\uc758 \uc804\ub840\uc640 \uc131\uc9c1\uc5d0 \ub300\ud55c \uac1c\ud601\uc744 \uc774\ub8e9\ud55c \uc0ac\ub78c\uc73c\ub85c \uc798 \uc54c\ub824\uc838 \uc788\ub2e4. \uae30\ub3c5\uad50\uc758 \uc131\uc778\uc774\uba70 \uad50\ud68c\ubc15\uc0ac \uac00\uc6b4\ub370 \ud55c \uc0ac\ub78c\uc774\ub2e4. \ub85c\ub9c8 \uac00\ud1a8\ub9ad\uc5d0\uc11c\uc758 \ucd95\uc77c\uc740 12\uc6d4 7\uc77c\uc774\uba70, \uc131\uacf5\ud68c\uc5d0\uc11c\uc758 \ucd95\uc77c\uc740 4\uc6d4 4\uc77c\uc774\ub2e4. \uc554\ube0c\ub85c\uc2dc\uc6b0\uc2a4\ub294 \uadf8\ub9ac\uc2a4\uc5b4\uc5d0\uc11c \uc720\ub798\ud55c \uc774\ub984\uc73c\ub85c \u2018\ubd88\uba78\u2019\uc744 \ub73b\ud55c\ub2e4. \ubbf8\uc220\uc791\ud488\uc5d0\uc11c \uadf8\ub294 \ud754\ud788 \ubaa9\uc7a5\uc744 \ub4e0 \uc8fc\uad50\uc758 \ubaa8\uc2b5\uc73c\ub85c \ud45c\ud604\ub418\ub294\ub370, \ub54c\ub54c\ub85c \uc2b9\ub9c8\uc6a9 \ucc44\ucc0d\uc744 \ud718\ub450\ub974\uba74\uc11c \ub9d0\uc744 \ud0c0\uace0 \uc788\uac70\ub098 \uafc0\ubc8c\uc744 \uac70\ub290\ub9ac\uace0 \uc788\uae30\ub3c4 \ud55c\ub2e4.", "paragraph_answer": " \uc554\ube0c\ub85c\uc2dc\uc6b0\uc2a4 (\ub77c\ud2f4\uc5b4: Sanctus Ambrosius, 340\ub144?~397\ub144 4\uc6d4 4\uc77c) \ub610\ub294 \uc131 \uc554\ube0c\ub85c\uc2a4(Ambrose)\ub294 4\uc138\uae30\uc5d0 \ud65c\ub3d9\ud55c \uc11c\ubc29 \uad50\ud68c\uc758 4\ub300 \uad50\ubd80 \uc911 \ud55c \uc0ac\ub78c\uc73c\ub85c\uc11c \ubc95\ub960\uac00\uc774\uc790 \ubc00\ub77c\ub178\uc758 \uc8fc\uad50\uc774\ub2e4. \uc544\ub9ac\uc6b0\uc2a4\ud30c\uc5d0 \ub9de\uc11c \uc815\ud1b5 \uae30\ub3c5\uad50\uc758 \uc804\ub840\uc640 \uc131\uc9c1\uc5d0 \ub300\ud55c \uac1c\ud601\uc744 \uc774\ub8e9\ud55c \uc0ac\ub78c\uc73c\ub85c \uc798 \uc54c\ub824\uc838 \uc788\ub2e4. \uae30\ub3c5\uad50\uc758 \uc131\uc778\uc774\uba70 \uad50\ud68c\ubc15\uc0ac \uac00\uc6b4\ub370 \ud55c \uc0ac\ub78c\uc774\ub2e4. \ub85c\ub9c8 \uac00\ud1a8\ub9ad\uc5d0\uc11c\uc758 \ucd95\uc77c\uc740 12\uc6d4 7\uc77c\uc774\uba70, \uc131\uacf5\ud68c\uc5d0\uc11c\uc758 \ucd95\uc77c\uc740 4\uc6d4 4\uc77c\uc774\ub2e4. \uc554\ube0c\ub85c\uc2dc\uc6b0\uc2a4\ub294 \uadf8\ub9ac\uc2a4\uc5b4\uc5d0\uc11c \uc720\ub798\ud55c \uc774\ub984\uc73c\ub85c \u2018\ubd88\uba78\u2019\uc744 \ub73b\ud55c\ub2e4. \ubbf8\uc220\uc791\ud488\uc5d0\uc11c \uadf8\ub294 \ud754\ud788 \ubaa9\uc7a5\uc744 \ub4e0 \uc8fc\uad50\uc758 \ubaa8\uc2b5\uc73c\ub85c \ud45c\ud604\ub418\ub294\ub370, \ub54c\ub54c\ub85c \uc2b9\ub9c8\uc6a9 \ucc44\ucc0d\uc744 \ud718\ub450\ub974\uba74\uc11c \ub9d0\uc744 \ud0c0\uace0 \uc788\uac70\ub098 \uafc0\ubc8c\uc744 \uac70\ub290\ub9ac\uace0 \uc788\uae30\ub3c4 \ud55c\ub2e4.", "sentence_answer": " \uc554\ube0c\ub85c\uc2dc\uc6b0\uc2a4 (\ub77c\ud2f4\uc5b4: Sanctus Ambrosius, 340\ub144?~397\ub144 4\uc6d4 4\uc77c)", "paragraph_id": "6522268-0-0", "paragraph_question": "question: 4\uc138\uae30\uc5d0 \ud65c\ub3d9\ud55c \ubc95\ub960\uac00 \uc774\uc790 \ubc00\ub77c\ub178\uc758 \uc8fc\uad50\uc778 \uc11c\ubc29 \uad50\ud68c\uc758 4\ub300 \uad50\ubd80 \uc911 \ud55c \uc0ac\ub78c\uc740?, context: \uc554\ube0c\ub85c\uc2dc\uc6b0\uc2a4(\ub77c\ud2f4\uc5b4: Sanctus Ambrosius, 340\ub144?~397\ub144 4\uc6d4 4\uc77c) \ub610\ub294 \uc131 \uc554\ube0c\ub85c\uc2a4(Ambrose)\ub294 4\uc138\uae30\uc5d0 \ud65c\ub3d9\ud55c \uc11c\ubc29 \uad50\ud68c\uc758 4\ub300 \uad50\ubd80 \uc911 \ud55c \uc0ac\ub78c\uc73c\ub85c\uc11c \ubc95\ub960\uac00\uc774\uc790 \ubc00\ub77c\ub178\uc758 \uc8fc\uad50\uc774\ub2e4. \uc544\ub9ac\uc6b0\uc2a4\ud30c\uc5d0 \ub9de\uc11c \uc815\ud1b5 \uae30\ub3c5\uad50\uc758 \uc804\ub840\uc640 \uc131\uc9c1\uc5d0 \ub300\ud55c \uac1c\ud601\uc744 \uc774\ub8e9\ud55c \uc0ac\ub78c\uc73c\ub85c \uc798 \uc54c\ub824\uc838 \uc788\ub2e4. \uae30\ub3c5\uad50\uc758 \uc131\uc778\uc774\uba70 \uad50\ud68c\ubc15\uc0ac \uac00\uc6b4\ub370 \ud55c \uc0ac\ub78c\uc774\ub2e4. \ub85c\ub9c8 \uac00\ud1a8\ub9ad\uc5d0\uc11c\uc758 \ucd95\uc77c\uc740 12\uc6d4 7\uc77c\uc774\uba70, \uc131\uacf5\ud68c\uc5d0\uc11c\uc758 \ucd95\uc77c\uc740 4\uc6d4 4\uc77c\uc774\ub2e4. \uc554\ube0c\ub85c\uc2dc\uc6b0\uc2a4\ub294 \uadf8\ub9ac\uc2a4\uc5b4\uc5d0\uc11c \uc720\ub798\ud55c \uc774\ub984\uc73c\ub85c \u2018\ubd88\uba78\u2019\uc744 \ub73b\ud55c\ub2e4. \ubbf8\uc220\uc791\ud488\uc5d0\uc11c \uadf8\ub294 \ud754\ud788 \ubaa9\uc7a5\uc744 \ub4e0 \uc8fc\uad50\uc758 \ubaa8\uc2b5\uc73c\ub85c \ud45c\ud604\ub418\ub294\ub370, \ub54c\ub54c\ub85c \uc2b9\ub9c8\uc6a9 \ucc44\ucc0d\uc744 \ud718\ub450\ub974\uba74\uc11c \ub9d0\uc744 \ud0c0\uace0 \uc788\uac70\ub098 \uafc0\ubc8c\uc744 \uac70\ub290\ub9ac\uace0 \uc788\uae30\ub3c4 \ud55c\ub2e4."} {"question": "\uad6d\ubbfc\ucd1d\uc18c\ub4dd \ub300\ube44 \uac00\uacc4\uc18c\ub4dd \ube44\uc911\uc744 70%\ub85c \ub192\uc774\ub294 \ub4f1\uc758 \ub0b4\uc6a9\uc744 \ub2f4\uc740 \uacc4\ud68d\uc740?", "paragraph": "\uae40\uc885\uc778 \ub354\ubbfc\uc8fc \ub300\ud45c\uac00 \ucd1d\uc120 \uacf5\uc57d\uc744 2017\ub144 \ub300\uc120\uae4c\uc9c0 \uc5fc\ub450\uc5d0 \ub450\uace0 \ub0b4\ub193\uaca0\ub2e4\uba70, \uc218\uad8c \ud6c4 \ucc28\uae30 \ub300\ud1b5\ub839 \uc9c1\uc18d\uc73c\ub85c '\ubd88\ud3c9\ub4f1 \ud574\uc18c\uc704\uc6d0\ud68c'\ub97c \uc124\uce58\ud574 '777 \ud50c\ub79c'\uc758 \uad6c\uccb4\uc801\uc778 \uc2e4\ucc9c \ubc29\uc548\uc744 \uc911\u00b7\uc7a5\uae30 5\uac1c\ub144 \uacc4\ud68d\uc73c\ub85c \uc218\ub9bd\ud55c\ub2e4\uace0 \ubc1d\ud614\ub2e4. 777\ud50c\ub79c\uc740 61.9%\uc5d0 \uba38\ubb34\ub294 \uad6d\ubbfc\ucd1d\uc18c\ub4dd(GNI) \ub300\ube44 \uac00\uacc4\uc18c\ub4dd \ube44\uc911(2014\ub144 \uae30\uc900)\uc744 2020\ub144\uae4c\uc9c0 70%\ub300\ub85c \ub04c\uc5b4\uc62c\ub9ac\uace0, \uc544\uc6b8\ub7ec \uc804\uccb4 \uc18c\ub4dd\ubd84 \uc911 \uadfc\ub85c\uc790(\uc790\uc601\uc5c5\uc790 \ud3ec\ud568)\uc5d0\uac8c \ubc30\ubd84\ub418\ub294 \ube44\uc728\uc778 \ub178\ub3d9\uc18c\ub4dd \ubd84\ubc30\uc728\uc744 2012\ub144 \uae30\uc900 68.1%\uc5d0\uc11c 70%\ub300\ub85c, \uc911\uc0b0\uce35(\uc911\uc704 \uc18c\ub4dd\uc758 50~150%) \ube44\uc911\ub3c4 1997\ub144 \uc678\ud658\uc704\uae30 \uc774\uc804 \uc218\uc900\uc778 70%\ub300\ub85c \ubcf5\uc6d0\ud55c\ub2e4\ub294 \uacc4\ud68d\uc774\ub2e4. \uacf5\uc57d \ud574\uacb0 \ubc29\uc548\uc73c\ub85c \ub300\u00b7\uc911\uc18c\uae30\uc5c5 \uc131\uacfc\uacf5\uc720\uc81c\ub97c \uc2dc\ud589\ud558\ub294 \uae30\uc5c5\uc5d0 \uc138\uc81c \uc9c0\uc6d0\uc744 \ud558\uace0 \uc784\uae08\uc744 \uc62c\ub9b0 \uc218\uc900\uc5d0 \ub530\ub77c \uae30\uc5c5\uc758 \uc0ac\ub0b4\uc720\ubcf4\uae08 \uacfc\uc138 \uc2dc \uc774\uc775\uc744 \uc8fc\ub294 \ubc29\uc548\uc744 \ub0b4\ub193\uc558\ub2e4. \uc774\uc640 \ud568\uaed8 \ucd5c\uc800\uc784\uae08\uc744 2020\ub144\uae4c\uc9c0 1\ub9cc\uc6d0 \uc218\uc900\uc73c\ub85c \uc62c\ub9ac\uace0 \ucd5c\uc800\uc784\uae08\uc744 \ubcf4\uc7a5\ud558\ub294 \uc0dd\ud65c\uc784\uae08\uc81c\ub97c \uc804\uad6d\uc801\uc73c\ub85c \ud655\uc2e0\ud55c\ub2e4\ub294 \ubc29\uce68\uc774\ub2e4.", "answer": "777\ud50c\ub79c", "sentence": "777\ud50c\ub79c \uc740 61.9%\uc5d0 \uba38\ubb34\ub294 \uad6d\ubbfc\ucd1d\uc18c\ub4dd(GNI) \ub300\ube44 \uac00\uacc4\uc18c\ub4dd \ube44\uc911(2014\ub144 \uae30\uc900)\uc744 2020\ub144\uae4c\uc9c0 70%\ub300\ub85c \ub04c\uc5b4\uc62c\ub9ac\uace0, \uc544\uc6b8\ub7ec \uc804\uccb4 \uc18c\ub4dd\ubd84 \uc911 \uadfc\ub85c\uc790(\uc790\uc601\uc5c5\uc790 \ud3ec\ud568)\uc5d0\uac8c \ubc30\ubd84\ub418\ub294 \ube44\uc728\uc778 \ub178\ub3d9\uc18c\ub4dd \ubd84\ubc30\uc728\uc744 2012\ub144 \uae30\uc900 68.1%\uc5d0\uc11c 70%\ub300\ub85c, \uc911\uc0b0\uce35(\uc911\uc704 \uc18c\ub4dd\uc758 50~150%) \ube44\uc911\ub3c4 1997\ub144 \uc678\ud658\uc704\uae30 \uc774\uc804 \uc218\uc900\uc778 70%\ub300\ub85c \ubcf5\uc6d0\ud55c\ub2e4\ub294 \uacc4\ud68d\uc774\ub2e4.", "paragraph_sentence": "\uae40\uc885\uc778 \ub354\ubbfc\uc8fc \ub300\ud45c\uac00 \ucd1d\uc120 \uacf5\uc57d\uc744 2017\ub144 \ub300\uc120\uae4c\uc9c0 \uc5fc\ub450\uc5d0 \ub450\uace0 \ub0b4\ub193\uaca0\ub2e4\uba70, \uc218\uad8c \ud6c4 \ucc28\uae30 \ub300\ud1b5\ub839 \uc9c1\uc18d\uc73c\ub85c '\ubd88\ud3c9\ub4f1 \ud574\uc18c\uc704\uc6d0\ud68c'\ub97c \uc124\uce58\ud574 '777 \ud50c\ub79c'\uc758 \uad6c\uccb4\uc801\uc778 \uc2e4\ucc9c \ubc29\uc548\uc744 \uc911\u00b7\uc7a5\uae30 5\uac1c\ub144 \uacc4\ud68d\uc73c\ub85c \uc218\ub9bd\ud55c\ub2e4\uace0 \ubc1d\ud614\ub2e4. 777\ud50c\ub79c \uc740 61.9%\uc5d0 \uba38\ubb34\ub294 \uad6d\ubbfc\ucd1d\uc18c\ub4dd(GNI) \ub300\ube44 \uac00\uacc4\uc18c\ub4dd \ube44\uc911(2014\ub144 \uae30\uc900)\uc744 2020\ub144\uae4c\uc9c0 70%\ub300\ub85c \ub04c\uc5b4\uc62c\ub9ac\uace0, \uc544\uc6b8\ub7ec \uc804\uccb4 \uc18c\ub4dd\ubd84 \uc911 \uadfc\ub85c\uc790(\uc790\uc601\uc5c5\uc790 \ud3ec\ud568)\uc5d0\uac8c \ubc30\ubd84\ub418\ub294 \ube44\uc728\uc778 \ub178\ub3d9\uc18c\ub4dd \ubd84\ubc30\uc728\uc744 2012\ub144 \uae30\uc900 68.1%\uc5d0\uc11c 70%\ub300\ub85c, \uc911\uc0b0\uce35(\uc911\uc704 \uc18c\ub4dd\uc758 50~150%) \ube44\uc911\ub3c4 1997\ub144 \uc678\ud658\uc704\uae30 \uc774\uc804 \uc218\uc900\uc778 70%\ub300\ub85c \ubcf5\uc6d0\ud55c\ub2e4\ub294 \uacc4\ud68d\uc774\ub2e4. \uacf5\uc57d \ud574\uacb0 \ubc29\uc548\uc73c\ub85c \ub300\u00b7\uc911\uc18c\uae30\uc5c5 \uc131\uacfc\uacf5\uc720\uc81c\ub97c \uc2dc\ud589\ud558\ub294 \uae30\uc5c5\uc5d0 \uc138\uc81c \uc9c0\uc6d0\uc744 \ud558\uace0 \uc784\uae08\uc744 \uc62c\ub9b0 \uc218\uc900\uc5d0 \ub530\ub77c \uae30\uc5c5\uc758 \uc0ac\ub0b4\uc720\ubcf4\uae08 \uacfc\uc138 \uc2dc \uc774\uc775\uc744 \uc8fc\ub294 \ubc29\uc548\uc744 \ub0b4\ub193\uc558\ub2e4. \uc774\uc640 \ud568\uaed8 \ucd5c\uc800\uc784\uae08\uc744 2020\ub144\uae4c\uc9c0 1\ub9cc\uc6d0 \uc218\uc900\uc73c\ub85c \uc62c\ub9ac\uace0 \ucd5c\uc800\uc784\uae08\uc744 \ubcf4\uc7a5\ud558\ub294 \uc0dd\ud65c\uc784\uae08\uc81c\ub97c \uc804\uad6d\uc801\uc73c\ub85c \ud655\uc2e0\ud55c\ub2e4\ub294 \ubc29\uce68\uc774\ub2e4.", "paragraph_answer": "\uae40\uc885\uc778 \ub354\ubbfc\uc8fc \ub300\ud45c\uac00 \ucd1d\uc120 \uacf5\uc57d\uc744 2017\ub144 \ub300\uc120\uae4c\uc9c0 \uc5fc\ub450\uc5d0 \ub450\uace0 \ub0b4\ub193\uaca0\ub2e4\uba70, \uc218\uad8c \ud6c4 \ucc28\uae30 \ub300\ud1b5\ub839 \uc9c1\uc18d\uc73c\ub85c '\ubd88\ud3c9\ub4f1 \ud574\uc18c\uc704\uc6d0\ud68c'\ub97c \uc124\uce58\ud574 '777 \ud50c\ub79c'\uc758 \uad6c\uccb4\uc801\uc778 \uc2e4\ucc9c \ubc29\uc548\uc744 \uc911\u00b7\uc7a5\uae30 5\uac1c\ub144 \uacc4\ud68d\uc73c\ub85c \uc218\ub9bd\ud55c\ub2e4\uace0 \ubc1d\ud614\ub2e4. 777\ud50c\ub79c \uc740 61.9%\uc5d0 \uba38\ubb34\ub294 \uad6d\ubbfc\ucd1d\uc18c\ub4dd(GNI) \ub300\ube44 \uac00\uacc4\uc18c\ub4dd \ube44\uc911(2014\ub144 \uae30\uc900)\uc744 2020\ub144\uae4c\uc9c0 70%\ub300\ub85c \ub04c\uc5b4\uc62c\ub9ac\uace0, \uc544\uc6b8\ub7ec \uc804\uccb4 \uc18c\ub4dd\ubd84 \uc911 \uadfc\ub85c\uc790(\uc790\uc601\uc5c5\uc790 \ud3ec\ud568)\uc5d0\uac8c \ubc30\ubd84\ub418\ub294 \ube44\uc728\uc778 \ub178\ub3d9\uc18c\ub4dd \ubd84\ubc30\uc728\uc744 2012\ub144 \uae30\uc900 68.1%\uc5d0\uc11c 70%\ub300\ub85c, \uc911\uc0b0\uce35(\uc911\uc704 \uc18c\ub4dd\uc758 50~150%) \ube44\uc911\ub3c4 1997\ub144 \uc678\ud658\uc704\uae30 \uc774\uc804 \uc218\uc900\uc778 70%\ub300\ub85c \ubcf5\uc6d0\ud55c\ub2e4\ub294 \uacc4\ud68d\uc774\ub2e4. \uacf5\uc57d \ud574\uacb0 \ubc29\uc548\uc73c\ub85c \ub300\u00b7\uc911\uc18c\uae30\uc5c5 \uc131\uacfc\uacf5\uc720\uc81c\ub97c \uc2dc\ud589\ud558\ub294 \uae30\uc5c5\uc5d0 \uc138\uc81c \uc9c0\uc6d0\uc744 \ud558\uace0 \uc784\uae08\uc744 \uc62c\ub9b0 \uc218\uc900\uc5d0 \ub530\ub77c \uae30\uc5c5\uc758 \uc0ac\ub0b4\uc720\ubcf4\uae08 \uacfc\uc138 \uc2dc \uc774\uc775\uc744 \uc8fc\ub294 \ubc29\uc548\uc744 \ub0b4\ub193\uc558\ub2e4. \uc774\uc640 \ud568\uaed8 \ucd5c\uc800\uc784\uae08\uc744 2020\ub144\uae4c\uc9c0 1\ub9cc\uc6d0 \uc218\uc900\uc73c\ub85c \uc62c\ub9ac\uace0 \ucd5c\uc800\uc784\uae08\uc744 \ubcf4\uc7a5\ud558\ub294 \uc0dd\ud65c\uc784\uae08\uc81c\ub97c \uc804\uad6d\uc801\uc73c\ub85c \ud655\uc2e0\ud55c\ub2e4\ub294 \ubc29\uce68\uc774\ub2e4.", "sentence_answer": " 777\ud50c\ub79c \uc740 61.9%\uc5d0 \uba38\ubb34\ub294 \uad6d\ubbfc\ucd1d\uc18c\ub4dd(GNI) \ub300\ube44 \uac00\uacc4\uc18c\ub4dd \ube44\uc911(2014\ub144 \uae30\uc900)\uc744 2020\ub144\uae4c\uc9c0 70%\ub300\ub85c \ub04c\uc5b4\uc62c\ub9ac\uace0, \uc544\uc6b8\ub7ec \uc804\uccb4 \uc18c\ub4dd\ubd84 \uc911 \uadfc\ub85c\uc790(\uc790\uc601\uc5c5\uc790 \ud3ec\ud568)\uc5d0\uac8c \ubc30\ubd84\ub418\ub294 \ube44\uc728\uc778 \ub178\ub3d9\uc18c\ub4dd \ubd84\ubc30\uc728\uc744 2012\ub144 \uae30\uc900 68.1%\uc5d0\uc11c 70%\ub300\ub85c, \uc911\uc0b0\uce35(\uc911\uc704 \uc18c\ub4dd\uc758 50~150%) \ube44\uc911\ub3c4 1997\ub144 \uc678\ud658\uc704\uae30 \uc774\uc804 \uc218\uc900\uc778 70%\ub300\ub85c \ubcf5\uc6d0\ud55c\ub2e4\ub294 \uacc4\ud68d\uc774\ub2e4.", "paragraph_id": "6488514-5-0", "paragraph_question": "question: \uad6d\ubbfc\ucd1d\uc18c\ub4dd \ub300\ube44 \uac00\uacc4\uc18c\ub4dd \ube44\uc911\uc744 70%\ub85c \ub192\uc774\ub294 \ub4f1\uc758 \ub0b4\uc6a9\uc744 \ub2f4\uc740 \uacc4\ud68d\uc740?, context: \uae40\uc885\uc778 \ub354\ubbfc\uc8fc \ub300\ud45c\uac00 \ucd1d\uc120 \uacf5\uc57d\uc744 2017\ub144 \ub300\uc120\uae4c\uc9c0 \uc5fc\ub450\uc5d0 \ub450\uace0 \ub0b4\ub193\uaca0\ub2e4\uba70, \uc218\uad8c \ud6c4 \ucc28\uae30 \ub300\ud1b5\ub839 \uc9c1\uc18d\uc73c\ub85c '\ubd88\ud3c9\ub4f1 \ud574\uc18c\uc704\uc6d0\ud68c'\ub97c \uc124\uce58\ud574 '777 \ud50c\ub79c'\uc758 \uad6c\uccb4\uc801\uc778 \uc2e4\ucc9c \ubc29\uc548\uc744 \uc911\u00b7\uc7a5\uae30 5\uac1c\ub144 \uacc4\ud68d\uc73c\ub85c \uc218\ub9bd\ud55c\ub2e4\uace0 \ubc1d\ud614\ub2e4. 777\ud50c\ub79c\uc740 61.9%\uc5d0 \uba38\ubb34\ub294 \uad6d\ubbfc\ucd1d\uc18c\ub4dd(GNI) \ub300\ube44 \uac00\uacc4\uc18c\ub4dd \ube44\uc911(2014\ub144 \uae30\uc900)\uc744 2020\ub144\uae4c\uc9c0 70%\ub300\ub85c \ub04c\uc5b4\uc62c\ub9ac\uace0, \uc544\uc6b8\ub7ec \uc804\uccb4 \uc18c\ub4dd\ubd84 \uc911 \uadfc\ub85c\uc790(\uc790\uc601\uc5c5\uc790 \ud3ec\ud568)\uc5d0\uac8c \ubc30\ubd84\ub418\ub294 \ube44\uc728\uc778 \ub178\ub3d9\uc18c\ub4dd \ubd84\ubc30\uc728\uc744 2012\ub144 \uae30\uc900 68.1%\uc5d0\uc11c 70%\ub300\ub85c, \uc911\uc0b0\uce35(\uc911\uc704 \uc18c\ub4dd\uc758 50~150%) \ube44\uc911\ub3c4 1997\ub144 \uc678\ud658\uc704\uae30 \uc774\uc804 \uc218\uc900\uc778 70%\ub300\ub85c \ubcf5\uc6d0\ud55c\ub2e4\ub294 \uacc4\ud68d\uc774\ub2e4. \uacf5\uc57d \ud574\uacb0 \ubc29\uc548\uc73c\ub85c \ub300\u00b7\uc911\uc18c\uae30\uc5c5 \uc131\uacfc\uacf5\uc720\uc81c\ub97c \uc2dc\ud589\ud558\ub294 \uae30\uc5c5\uc5d0 \uc138\uc81c \uc9c0\uc6d0\uc744 \ud558\uace0 \uc784\uae08\uc744 \uc62c\ub9b0 \uc218\uc900\uc5d0 \ub530\ub77c \uae30\uc5c5\uc758 \uc0ac\ub0b4\uc720\ubcf4\uae08 \uacfc\uc138 \uc2dc \uc774\uc775\uc744 \uc8fc\ub294 \ubc29\uc548\uc744 \ub0b4\ub193\uc558\ub2e4. \uc774\uc640 \ud568\uaed8 \ucd5c\uc800\uc784\uae08\uc744 2020\ub144\uae4c\uc9c0 1\ub9cc\uc6d0 \uc218\uc900\uc73c\ub85c \uc62c\ub9ac\uace0 \ucd5c\uc800\uc784\uae08\uc744 \ubcf4\uc7a5\ud558\ub294 \uc0dd\ud65c\uc784\uae08\uc81c\ub97c \uc804\uad6d\uc801\uc73c\ub85c \ud655\uc2e0\ud55c\ub2e4\ub294 \ubc29\uce68\uc774\ub2e4."} {"question": "\ud2b8\ub7a8\uc774 \uc7ac\uc124\uce58\ub418\uae30 \uc2dc\uc791\ud55c \ub54c\ub294?", "paragraph": "\uadf8\ub7ec\ub358\uc911 1980\ub144\ub300 \uc120\uc9c4\uad6d\uc744 \uc2dc\uc791\uc73c\ub85c \ud2b8\ub7a8\uc774 \uc7ac\uc124\uce58\ub41c \uc774\ud6c4\ub85c \uc138\uacc4 \ub3c4\uc2dc \uacf3\uacf3\uc5d0\uc11c \ud2b8\ub7a8\uc774 \ub3cc\uc544\uc624\uace0 \uc788\ub2e4. \ud2b8\ub7a8\uc740 \uc9c0\ud558\ucca0\uacfc \ub2ec\ub9ac \uacbd\uce58\ub97c \ubcfc \uc218 \uc788\uace0 \uc2dc\ub044\ub7fd\uc9c0\ub3c4 \uc54a\uc73c\uba70 \ud3b8\ub9ac\ud558\ub2e4. \ud2b8\ub7a8\uc740 \ub808\uc77c\uc774 \uc124\uce58\ub418\uc5b4 \uc788\uc5b4 \ub2e4\ub978 \uc790\ub3d9\ucc28\ub4e4\ubcf4\ub2e4 \ud1b5\ud589 \uc6b0\uc120\uad8c\uc744 \uac00\uc9c0\uac8c \ub418\uc5b4 \uc77c\ubc18\uc801\uc73c\ub85c \ub2e4\uc74c \uc815\ucc28\uc5ed\uae4c\uc9c0 \uba48\ucd94\uc9c0 \uc54a\uace0 \uacc4\uc18d \ub2ec\ub9b0\ub2e4. \uad50\ucc28\ub85c\uc5d0\uc11c\ub294 \ud2b8\ub7a8\uc774 \uad50\ucc28\ub85c\uc5d0 \uc9c4\uc785\ud558\uba74 \uc2e0\ud638\uac00 \ubc14\ub00c\uace0 \ud2b8\ub7a8\uc774 \uc6b0\uc120\uc801\uc73c\ub85c \ud1b5\uacfc\ud55c\ub2e4. \ud2b8\ub7a8\uc758 \uc18d\ub3c4\ub294 \uc870\uc885\uc0ac\uac00 \uc6b4\uc804\ud558\uc9c0\ub9cc \ucd5c\ub300 \uc2dc\uc18d\uc740 \uc57d 60km\uc815\ub3c4\uc774\ub2e4. \ucee4\ube0c\uc5d0\uc11c\ub294 \uc18d\ub3c4\ub97c \uc904\uc5ec\uc57c \ud558\ub294\ub370 \uc774\ub7ec\ud55c \ubcc0\uc218\ub97c \ub2e4 \ud3ec\ud568\ud574\uc11c \ud2b8\ub7a8\uc758 \ud3c9\uade0 \uc18d\ub3c4\ub294 \uc2dc\uc18d 15~20 km \uc815\ub3c4\uac00 \ub41c\ub2e4. \ud2b8\ub7a8\uc740 \ub610\ud55c \ud658\uacbd\uc624\uc5fc\ub3c4 \uc801\uac8c \ub0b8\ub2e4. \ucd9c\ud1f4\uadfc\uc2dc\uac04\uc5d0 200\uba85\uc774 \uc774\ub3d9\ud558\uae30 \uc704\ud574\uc11c\ub294 \ud2b8\ub7a8 \ud55c\ub300, \ub610\ub294 65\uc778\uc2b9 \ubc84\uc2a4 3\ub300 \ub610\ub294 \uc2b9\uc6a9\uc790 170\ub300(\uc2b9\uc6a9\ucc28 \ud55c\ub300\ub2f9 \ud3c9\uade0 1.2\ud3c9\uc774 \ud0c4\ub2e4\uace0 \uac00\uc815)\uac00 \ud544\uc694\ud558\ub2e4. \ud2b8\ub7a8\uc740 \uc804\uae30\ub85c \uc6c0\uc9c1\uc774\uace0 \uc804\uae30\ub294 \uc120\uc9c4\uad6d\uc744 \uc911\uc2ec\uc73c\ub85c \uc6d0\uc790\ub825 \ubc1c\uc804\uc18c\uc5d0\uc11c \uc0dd\uc0b0\ub41c\ub2e4. \uacb0\uad6d \ud2b8\ub7a8\uc740 \uc774\uc0b0\ud654\ud0c4\uc18c\ub97c \uac70\uc758 \ubc1c\uc0dd\uc2dc\ud0a4\uc9c0 \uc54a\ub294\ub2e4. \ud2b8\ub7a8\uc740 \uc0ac\ub78c \ud55c \uba85\uc744 1 km \uc774\ub3d9\uc2dc\ud0a4\ub294\ub370 0.8\uadf8\ub78c\uc758 \uc774\uc0b0\ud654\ud0c4\uc18c\uac00 \ubc1c\uc0dd\ud558\uba70, \ubc84\uc2a4\uc640 \uc2b9\uc6a9\ucc28\ub294 \uac01\uac01 21\uadf8\ub78c\uacfc, 239\uadf8\ub78c \uc815\ub3c4\uac00 \ubc30\ucd9c\ub41c\ub2e4. \ud2b8\ub7a8\uc740 \uc591\ucabd\uc5d0 \ub808\uc77c\uc774 \uc788\ub294 \ud615\ud0dc\ub9cc \uc788\ub294 \uac83\uc774 \uc544\ub2c8\ub2e4. \uac00\uc6b4\ub370 \ub808\uc77c\uc774 \uc788\uace0 \ubc14\ud034\ub85c \uc6c0\uc9c1\uc774\ub294 \ud2b8\ub7a8\ub3c4 \uc788\ub294\ub370 \uc774\ub7ec\ud55c \ud615\ud0dc\uc758 \ud2b8\ub7a8\uc740 \ub354 \uae09\ucee4\ube0c\ub098, \uae09\uacbd\uc0ac\ub97c \uc624\ub97c \uc218 \uc788\ub2e4. \ubcf4\ub974\ub3c4\uc5d0\uc11c\ub294 \ub545\uc18d\uc5d0 \uc804\uae30\uacf5\uae09\uc120\uc774 \uc124\uce58\ub418\uc5b4 \uc788\ub294\ub370 \ud2b8\ub7a8\uc774 \uc9c0\ub098\uac08\ub54c\ub9cc \uc804\ub825\uc774 \uacf5\uae09\ub418\ubbc0\ub85c \ubcf4\ud589\uc790\ub4e4\uc774 \uc6c0\uc9c1\uc774\ub294\ub370 \uc548\uc804\ubb38\uc81c\uac00 \uc5c6\ub2e4. \ud2b8\ub7a8\uc740 \ud22c\uc790\ube44\uac00 \ube44\uc2f8\uae30 \ub54c\ubb38\uc5d0 \uc0ac\uc6a9\uc790\uac00 \ub9ce\uc740 \uacf3\uc744 \uc911\uc2ec\uc73c\ub85c \uc124\uce58\ud558\ub294 \uacbd\uc6b0\uac00 \ub9ce\ub2e4.", "answer": "1980\ub144\ub300", "sentence": "\uadf8\ub7ec\ub358\uc911 1980\ub144\ub300 \uc120\uc9c4\uad6d\uc744 \uc2dc\uc791\uc73c\ub85c \ud2b8\ub7a8\uc774 \uc7ac\uc124\uce58\ub41c \uc774\ud6c4\ub85c \uc138\uacc4 \ub3c4\uc2dc \uacf3\uacf3\uc5d0\uc11c \ud2b8\ub7a8\uc774 \ub3cc\uc544\uc624\uace0 \uc788\ub2e4.", "paragraph_sentence": " \uadf8\ub7ec\ub358\uc911 1980\ub144\ub300 \uc120\uc9c4\uad6d\uc744 \uc2dc\uc791\uc73c\ub85c \ud2b8\ub7a8\uc774 \uc7ac\uc124\uce58\ub41c \uc774\ud6c4\ub85c \uc138\uacc4 \ub3c4\uc2dc \uacf3\uacf3\uc5d0\uc11c \ud2b8\ub7a8\uc774 \ub3cc\uc544\uc624\uace0 \uc788\ub2e4. \ud2b8\ub7a8\uc740 \uc9c0\ud558\ucca0\uacfc \ub2ec\ub9ac \uacbd\uce58\ub97c \ubcfc \uc218 \uc788\uace0 \uc2dc\ub044\ub7fd\uc9c0\ub3c4 \uc54a\uc73c\uba70 \ud3b8\ub9ac\ud558\ub2e4. \ud2b8\ub7a8\uc740 \ub808\uc77c\uc774 \uc124\uce58\ub418\uc5b4 \uc788\uc5b4 \ub2e4\ub978 \uc790\ub3d9\ucc28\ub4e4\ubcf4\ub2e4 \ud1b5\ud589 \uc6b0\uc120\uad8c\uc744 \uac00\uc9c0\uac8c \ub418\uc5b4 \uc77c\ubc18\uc801\uc73c\ub85c \ub2e4\uc74c \uc815\ucc28\uc5ed\uae4c\uc9c0 \uba48\ucd94\uc9c0 \uc54a\uace0 \uacc4\uc18d \ub2ec\ub9b0\ub2e4. \uad50\ucc28\ub85c\uc5d0\uc11c\ub294 \ud2b8\ub7a8\uc774 \uad50\ucc28\ub85c\uc5d0 \uc9c4\uc785\ud558\uba74 \uc2e0\ud638\uac00 \ubc14\ub00c\uace0 \ud2b8\ub7a8\uc774 \uc6b0\uc120\uc801\uc73c\ub85c \ud1b5\uacfc\ud55c\ub2e4. \ud2b8\ub7a8\uc758 \uc18d\ub3c4\ub294 \uc870\uc885\uc0ac\uac00 \uc6b4\uc804\ud558\uc9c0\ub9cc \ucd5c\ub300 \uc2dc\uc18d\uc740 \uc57d 60km\uc815\ub3c4\uc774\ub2e4. \ucee4\ube0c\uc5d0\uc11c\ub294 \uc18d\ub3c4\ub97c \uc904\uc5ec\uc57c \ud558\ub294\ub370 \uc774\ub7ec\ud55c \ubcc0\uc218\ub97c \ub2e4 \ud3ec\ud568\ud574\uc11c \ud2b8\ub7a8\uc758 \ud3c9\uade0 \uc18d\ub3c4\ub294 \uc2dc\uc18d 15~20 km \uc815\ub3c4\uac00 \ub41c\ub2e4. \ud2b8\ub7a8\uc740 \ub610\ud55c \ud658\uacbd\uc624\uc5fc\ub3c4 \uc801\uac8c \ub0b8\ub2e4. \ucd9c\ud1f4\uadfc\uc2dc\uac04\uc5d0 200\uba85\uc774 \uc774\ub3d9\ud558\uae30 \uc704\ud574\uc11c\ub294 \ud2b8\ub7a8 \ud55c\ub300, \ub610\ub294 65\uc778\uc2b9 \ubc84\uc2a4 3\ub300 \ub610\ub294 \uc2b9\uc6a9\uc790 170\ub300(\uc2b9\uc6a9\ucc28 \ud55c\ub300\ub2f9 \ud3c9\uade0 1.2\ud3c9\uc774 \ud0c4\ub2e4\uace0 \uac00\uc815)\uac00 \ud544\uc694\ud558\ub2e4. \ud2b8\ub7a8\uc740 \uc804\uae30\ub85c \uc6c0\uc9c1\uc774\uace0 \uc804\uae30\ub294 \uc120\uc9c4\uad6d\uc744 \uc911\uc2ec\uc73c\ub85c \uc6d0\uc790\ub825 \ubc1c\uc804\uc18c\uc5d0\uc11c \uc0dd\uc0b0\ub41c\ub2e4. \uacb0\uad6d \ud2b8\ub7a8\uc740 \uc774\uc0b0\ud654\ud0c4\uc18c\ub97c \uac70\uc758 \ubc1c\uc0dd\uc2dc\ud0a4\uc9c0 \uc54a\ub294\ub2e4. \ud2b8\ub7a8\uc740 \uc0ac\ub78c \ud55c \uba85\uc744 1 km \uc774\ub3d9\uc2dc\ud0a4\ub294\ub370 0.8\uadf8\ub78c\uc758 \uc774\uc0b0\ud654\ud0c4\uc18c\uac00 \ubc1c\uc0dd\ud558\uba70, \ubc84\uc2a4\uc640 \uc2b9\uc6a9\ucc28\ub294 \uac01\uac01 21\uadf8\ub78c\uacfc, 239\uadf8\ub78c \uc815\ub3c4\uac00 \ubc30\ucd9c\ub41c\ub2e4. \ud2b8\ub7a8\uc740 \uc591\ucabd\uc5d0 \ub808\uc77c\uc774 \uc788\ub294 \ud615\ud0dc\ub9cc \uc788\ub294 \uac83\uc774 \uc544\ub2c8\ub2e4. \uac00\uc6b4\ub370 \ub808\uc77c\uc774 \uc788\uace0 \ubc14\ud034\ub85c \uc6c0\uc9c1\uc774\ub294 \ud2b8\ub7a8\ub3c4 \uc788\ub294\ub370 \uc774\ub7ec\ud55c \ud615\ud0dc\uc758 \ud2b8\ub7a8\uc740 \ub354 \uae09\ucee4\ube0c\ub098, \uae09\uacbd\uc0ac\ub97c \uc624\ub97c \uc218 \uc788\ub2e4. \ubcf4\ub974\ub3c4\uc5d0\uc11c\ub294 \ub545\uc18d\uc5d0 \uc804\uae30\uacf5\uae09\uc120\uc774 \uc124\uce58\ub418\uc5b4 \uc788\ub294\ub370 \ud2b8\ub7a8\uc774 \uc9c0\ub098\uac08\ub54c\ub9cc \uc804\ub825\uc774 \uacf5\uae09\ub418\ubbc0\ub85c \ubcf4\ud589\uc790\ub4e4\uc774 \uc6c0\uc9c1\uc774\ub294\ub370 \uc548\uc804\ubb38\uc81c\uac00 \uc5c6\ub2e4. \ud2b8\ub7a8\uc740 \ud22c\uc790\ube44\uac00 \ube44\uc2f8\uae30 \ub54c\ubb38\uc5d0 \uc0ac\uc6a9\uc790\uac00 \ub9ce\uc740 \uacf3\uc744 \uc911\uc2ec\uc73c\ub85c \uc124\uce58\ud558\ub294 \uacbd\uc6b0\uac00 \ub9ce\ub2e4.", "paragraph_answer": "\uadf8\ub7ec\ub358\uc911 1980\ub144\ub300 \uc120\uc9c4\uad6d\uc744 \uc2dc\uc791\uc73c\ub85c \ud2b8\ub7a8\uc774 \uc7ac\uc124\uce58\ub41c \uc774\ud6c4\ub85c \uc138\uacc4 \ub3c4\uc2dc \uacf3\uacf3\uc5d0\uc11c \ud2b8\ub7a8\uc774 \ub3cc\uc544\uc624\uace0 \uc788\ub2e4. \ud2b8\ub7a8\uc740 \uc9c0\ud558\ucca0\uacfc \ub2ec\ub9ac \uacbd\uce58\ub97c \ubcfc \uc218 \uc788\uace0 \uc2dc\ub044\ub7fd\uc9c0\ub3c4 \uc54a\uc73c\uba70 \ud3b8\ub9ac\ud558\ub2e4. \ud2b8\ub7a8\uc740 \ub808\uc77c\uc774 \uc124\uce58\ub418\uc5b4 \uc788\uc5b4 \ub2e4\ub978 \uc790\ub3d9\ucc28\ub4e4\ubcf4\ub2e4 \ud1b5\ud589 \uc6b0\uc120\uad8c\uc744 \uac00\uc9c0\uac8c \ub418\uc5b4 \uc77c\ubc18\uc801\uc73c\ub85c \ub2e4\uc74c \uc815\ucc28\uc5ed\uae4c\uc9c0 \uba48\ucd94\uc9c0 \uc54a\uace0 \uacc4\uc18d \ub2ec\ub9b0\ub2e4. \uad50\ucc28\ub85c\uc5d0\uc11c\ub294 \ud2b8\ub7a8\uc774 \uad50\ucc28\ub85c\uc5d0 \uc9c4\uc785\ud558\uba74 \uc2e0\ud638\uac00 \ubc14\ub00c\uace0 \ud2b8\ub7a8\uc774 \uc6b0\uc120\uc801\uc73c\ub85c \ud1b5\uacfc\ud55c\ub2e4. \ud2b8\ub7a8\uc758 \uc18d\ub3c4\ub294 \uc870\uc885\uc0ac\uac00 \uc6b4\uc804\ud558\uc9c0\ub9cc \ucd5c\ub300 \uc2dc\uc18d\uc740 \uc57d 60km\uc815\ub3c4\uc774\ub2e4. \ucee4\ube0c\uc5d0\uc11c\ub294 \uc18d\ub3c4\ub97c \uc904\uc5ec\uc57c \ud558\ub294\ub370 \uc774\ub7ec\ud55c \ubcc0\uc218\ub97c \ub2e4 \ud3ec\ud568\ud574\uc11c \ud2b8\ub7a8\uc758 \ud3c9\uade0 \uc18d\ub3c4\ub294 \uc2dc\uc18d 15~20 km \uc815\ub3c4\uac00 \ub41c\ub2e4. \ud2b8\ub7a8\uc740 \ub610\ud55c \ud658\uacbd\uc624\uc5fc\ub3c4 \uc801\uac8c \ub0b8\ub2e4. \ucd9c\ud1f4\uadfc\uc2dc\uac04\uc5d0 200\uba85\uc774 \uc774\ub3d9\ud558\uae30 \uc704\ud574\uc11c\ub294 \ud2b8\ub7a8 \ud55c\ub300, \ub610\ub294 65\uc778\uc2b9 \ubc84\uc2a4 3\ub300 \ub610\ub294 \uc2b9\uc6a9\uc790 170\ub300(\uc2b9\uc6a9\ucc28 \ud55c\ub300\ub2f9 \ud3c9\uade0 1.2\ud3c9\uc774 \ud0c4\ub2e4\uace0 \uac00\uc815)\uac00 \ud544\uc694\ud558\ub2e4. \ud2b8\ub7a8\uc740 \uc804\uae30\ub85c \uc6c0\uc9c1\uc774\uace0 \uc804\uae30\ub294 \uc120\uc9c4\uad6d\uc744 \uc911\uc2ec\uc73c\ub85c \uc6d0\uc790\ub825 \ubc1c\uc804\uc18c\uc5d0\uc11c \uc0dd\uc0b0\ub41c\ub2e4. \uacb0\uad6d \ud2b8\ub7a8\uc740 \uc774\uc0b0\ud654\ud0c4\uc18c\ub97c \uac70\uc758 \ubc1c\uc0dd\uc2dc\ud0a4\uc9c0 \uc54a\ub294\ub2e4. \ud2b8\ub7a8\uc740 \uc0ac\ub78c \ud55c \uba85\uc744 1 km \uc774\ub3d9\uc2dc\ud0a4\ub294\ub370 0.8\uadf8\ub78c\uc758 \uc774\uc0b0\ud654\ud0c4\uc18c\uac00 \ubc1c\uc0dd\ud558\uba70, \ubc84\uc2a4\uc640 \uc2b9\uc6a9\ucc28\ub294 \uac01\uac01 21\uadf8\ub78c\uacfc, 239\uadf8\ub78c \uc815\ub3c4\uac00 \ubc30\ucd9c\ub41c\ub2e4. \ud2b8\ub7a8\uc740 \uc591\ucabd\uc5d0 \ub808\uc77c\uc774 \uc788\ub294 \ud615\ud0dc\ub9cc \uc788\ub294 \uac83\uc774 \uc544\ub2c8\ub2e4. \uac00\uc6b4\ub370 \ub808\uc77c\uc774 \uc788\uace0 \ubc14\ud034\ub85c \uc6c0\uc9c1\uc774\ub294 \ud2b8\ub7a8\ub3c4 \uc788\ub294\ub370 \uc774\ub7ec\ud55c \ud615\ud0dc\uc758 \ud2b8\ub7a8\uc740 \ub354 \uae09\ucee4\ube0c\ub098, \uae09\uacbd\uc0ac\ub97c \uc624\ub97c \uc218 \uc788\ub2e4. \ubcf4\ub974\ub3c4\uc5d0\uc11c\ub294 \ub545\uc18d\uc5d0 \uc804\uae30\uacf5\uae09\uc120\uc774 \uc124\uce58\ub418\uc5b4 \uc788\ub294\ub370 \ud2b8\ub7a8\uc774 \uc9c0\ub098\uac08\ub54c\ub9cc \uc804\ub825\uc774 \uacf5\uae09\ub418\ubbc0\ub85c \ubcf4\ud589\uc790\ub4e4\uc774 \uc6c0\uc9c1\uc774\ub294\ub370 \uc548\uc804\ubb38\uc81c\uac00 \uc5c6\ub2e4. \ud2b8\ub7a8\uc740 \ud22c\uc790\ube44\uac00 \ube44\uc2f8\uae30 \ub54c\ubb38\uc5d0 \uc0ac\uc6a9\uc790\uac00 \ub9ce\uc740 \uacf3\uc744 \uc911\uc2ec\uc73c\ub85c \uc124\uce58\ud558\ub294 \uacbd\uc6b0\uac00 \ub9ce\ub2e4.", "sentence_answer": "\uadf8\ub7ec\ub358\uc911 1980\ub144\ub300 \uc120\uc9c4\uad6d\uc744 \uc2dc\uc791\uc73c\ub85c \ud2b8\ub7a8\uc774 \uc7ac\uc124\uce58\ub41c \uc774\ud6c4\ub85c \uc138\uacc4 \ub3c4\uc2dc \uacf3\uacf3\uc5d0\uc11c \ud2b8\ub7a8\uc774 \ub3cc\uc544\uc624\uace0 \uc788\ub2e4.", "paragraph_id": "6489416-2-0", "paragraph_question": "question: \ud2b8\ub7a8\uc774 \uc7ac\uc124\uce58\ub418\uae30 \uc2dc\uc791\ud55c \ub54c\ub294?, context: \uadf8\ub7ec\ub358\uc911 1980\ub144\ub300 \uc120\uc9c4\uad6d\uc744 \uc2dc\uc791\uc73c\ub85c \ud2b8\ub7a8\uc774 \uc7ac\uc124\uce58\ub41c \uc774\ud6c4\ub85c \uc138\uacc4 \ub3c4\uc2dc \uacf3\uacf3\uc5d0\uc11c \ud2b8\ub7a8\uc774 \ub3cc\uc544\uc624\uace0 \uc788\ub2e4. \ud2b8\ub7a8\uc740 \uc9c0\ud558\ucca0\uacfc \ub2ec\ub9ac \uacbd\uce58\ub97c \ubcfc \uc218 \uc788\uace0 \uc2dc\ub044\ub7fd\uc9c0\ub3c4 \uc54a\uc73c\uba70 \ud3b8\ub9ac\ud558\ub2e4. \ud2b8\ub7a8\uc740 \ub808\uc77c\uc774 \uc124\uce58\ub418\uc5b4 \uc788\uc5b4 \ub2e4\ub978 \uc790\ub3d9\ucc28\ub4e4\ubcf4\ub2e4 \ud1b5\ud589 \uc6b0\uc120\uad8c\uc744 \uac00\uc9c0\uac8c \ub418\uc5b4 \uc77c\ubc18\uc801\uc73c\ub85c \ub2e4\uc74c \uc815\ucc28\uc5ed\uae4c\uc9c0 \uba48\ucd94\uc9c0 \uc54a\uace0 \uacc4\uc18d \ub2ec\ub9b0\ub2e4. \uad50\ucc28\ub85c\uc5d0\uc11c\ub294 \ud2b8\ub7a8\uc774 \uad50\ucc28\ub85c\uc5d0 \uc9c4\uc785\ud558\uba74 \uc2e0\ud638\uac00 \ubc14\ub00c\uace0 \ud2b8\ub7a8\uc774 \uc6b0\uc120\uc801\uc73c\ub85c \ud1b5\uacfc\ud55c\ub2e4. \ud2b8\ub7a8\uc758 \uc18d\ub3c4\ub294 \uc870\uc885\uc0ac\uac00 \uc6b4\uc804\ud558\uc9c0\ub9cc \ucd5c\ub300 \uc2dc\uc18d\uc740 \uc57d 60km\uc815\ub3c4\uc774\ub2e4. \ucee4\ube0c\uc5d0\uc11c\ub294 \uc18d\ub3c4\ub97c \uc904\uc5ec\uc57c \ud558\ub294\ub370 \uc774\ub7ec\ud55c \ubcc0\uc218\ub97c \ub2e4 \ud3ec\ud568\ud574\uc11c \ud2b8\ub7a8\uc758 \ud3c9\uade0 \uc18d\ub3c4\ub294 \uc2dc\uc18d 15~20 km \uc815\ub3c4\uac00 \ub41c\ub2e4. \ud2b8\ub7a8\uc740 \ub610\ud55c \ud658\uacbd\uc624\uc5fc\ub3c4 \uc801\uac8c \ub0b8\ub2e4. \ucd9c\ud1f4\uadfc\uc2dc\uac04\uc5d0 200\uba85\uc774 \uc774\ub3d9\ud558\uae30 \uc704\ud574\uc11c\ub294 \ud2b8\ub7a8 \ud55c\ub300, \ub610\ub294 65\uc778\uc2b9 \ubc84\uc2a4 3\ub300 \ub610\ub294 \uc2b9\uc6a9\uc790 170\ub300(\uc2b9\uc6a9\ucc28 \ud55c\ub300\ub2f9 \ud3c9\uade0 1.2\ud3c9\uc774 \ud0c4\ub2e4\uace0 \uac00\uc815)\uac00 \ud544\uc694\ud558\ub2e4. \ud2b8\ub7a8\uc740 \uc804\uae30\ub85c \uc6c0\uc9c1\uc774\uace0 \uc804\uae30\ub294 \uc120\uc9c4\uad6d\uc744 \uc911\uc2ec\uc73c\ub85c \uc6d0\uc790\ub825 \ubc1c\uc804\uc18c\uc5d0\uc11c \uc0dd\uc0b0\ub41c\ub2e4. \uacb0\uad6d \ud2b8\ub7a8\uc740 \uc774\uc0b0\ud654\ud0c4\uc18c\ub97c \uac70\uc758 \ubc1c\uc0dd\uc2dc\ud0a4\uc9c0 \uc54a\ub294\ub2e4. \ud2b8\ub7a8\uc740 \uc0ac\ub78c \ud55c \uba85\uc744 1 km \uc774\ub3d9\uc2dc\ud0a4\ub294\ub370 0.8\uadf8\ub78c\uc758 \uc774\uc0b0\ud654\ud0c4\uc18c\uac00 \ubc1c\uc0dd\ud558\uba70, \ubc84\uc2a4\uc640 \uc2b9\uc6a9\ucc28\ub294 \uac01\uac01 21\uadf8\ub78c\uacfc, 239\uadf8\ub78c \uc815\ub3c4\uac00 \ubc30\ucd9c\ub41c\ub2e4. \ud2b8\ub7a8\uc740 \uc591\ucabd\uc5d0 \ub808\uc77c\uc774 \uc788\ub294 \ud615\ud0dc\ub9cc \uc788\ub294 \uac83\uc774 \uc544\ub2c8\ub2e4. \uac00\uc6b4\ub370 \ub808\uc77c\uc774 \uc788\uace0 \ubc14\ud034\ub85c \uc6c0\uc9c1\uc774\ub294 \ud2b8\ub7a8\ub3c4 \uc788\ub294\ub370 \uc774\ub7ec\ud55c \ud615\ud0dc\uc758 \ud2b8\ub7a8\uc740 \ub354 \uae09\ucee4\ube0c\ub098, \uae09\uacbd\uc0ac\ub97c \uc624\ub97c \uc218 \uc788\ub2e4. \ubcf4\ub974\ub3c4\uc5d0\uc11c\ub294 \ub545\uc18d\uc5d0 \uc804\uae30\uacf5\uae09\uc120\uc774 \uc124\uce58\ub418\uc5b4 \uc788\ub294\ub370 \ud2b8\ub7a8\uc774 \uc9c0\ub098\uac08\ub54c\ub9cc \uc804\ub825\uc774 \uacf5\uae09\ub418\ubbc0\ub85c \ubcf4\ud589\uc790\ub4e4\uc774 \uc6c0\uc9c1\uc774\ub294\ub370 \uc548\uc804\ubb38\uc81c\uac00 \uc5c6\ub2e4. \ud2b8\ub7a8\uc740 \ud22c\uc790\ube44\uac00 \ube44\uc2f8\uae30 \ub54c\ubb38\uc5d0 \uc0ac\uc6a9\uc790\uac00 \ub9ce\uc740 \uacf3\uc744 \uc911\uc2ec\uc73c\ub85c \uc124\uce58\ud558\ub294 \uacbd\uc6b0\uac00 \ub9ce\ub2e4."} {"question": "\uc77c\ubcf8\uad70\uc774 \ucd1d \uc9d1\uacb0\ud558\uc5ec \uc218\ube44\ub97c \uac15\ud654\ud55c \uc9c0\uc5ed\uc740 \uc5b4\ub514\uc778\uac00?", "paragraph": "\uad50\ucc29\ud654\ud55c \uc804\uc138\uc5d0\uc11c \ucd08\uae30\uc758 \uc2b9\uc804\ubcf4 \uc774\ud6c4 \ubcc4\ub2e4\ub978 \uc2b9\ub9ac\uac00 \uc5c6\uc790 \uc120\uc870\ub97c \ube44\ub86f\ud55c \uc870\uc120 \uc870\uc815\uc5d0\uc11c\ub294 \uc774\uc21c\uc2e0\uc758 \uc804\ub7b5\uc744 \ubd88\uc2e0\ud558\uae30 \uc2dc\uc791\ud588\uc73c\uba70 \uc774\uc21c\uc2e0\uc5d0\uac8c \uc65c\uad70\uc5d0 \ub300\ud55c \uc801\uadf9\uc801\uc778 \uacf5\uaca9\uc744 \uac15\uc694\ud558\uac8c \ub418\uc5c8\ub2e4. \ub2f9\uc2dc \uc77c\ubcf8\uad70\uc740 \ub0a8\ud574\uc548 \uc77c\ub300\uc5d0 \ucd1d\uc9d1\uacb0\ud558\uc5ec \uc65c\uc131\uc744 \uc313\ub294 \ub4f1 \uc218\ube44\ub97c \uac15\ud654\ud558\uc600\uc73c\uba70 \uac15\ud654 \ud68c\ub2f4\uc758 \uc9c4\ud589 \uacfc\uc815\uc744 \uc9c0\ucf1c\ubcf4\uace0 \uc788\uc5c8\ub2e4. \ud55c\ud3b8 \uba85\ub098\ub77c\ub294 \uc804\uba74\uc801 \ub300\uacb0\ubcf4\ub2e4\ub294 \uac15\ud654 \ud68c\ub2f4\uc5d0 \uae30\ub300\ud558\uace0 \uc788\uc5c8\uc73c\uba70, \uc870\uc120\uc740 \uc790\uccb4\uc758 \uad70\uc0ac\ub825\uc73c\ub85c \uc77c\ubcf8\uad70\uacfc \uc721\uc804\uc5d0\uc11c \ub300\ub4f1\ud55c \uc804\ud22c\ub97c \uc218\ud589\ud560 \ub2a5\ub825\uc774 \ubd80\uc871\ud588\ub2e4. \uadf8\ub7ec\ub098 \uc870\uc815\uc758 \uc694\uccad\uacfc\ub294 \ub2ec\ub9ac \uc774\uc21c\uc2e0\uc740 \uc77c\ubcf8\uad70\uc758 \uc720\uc778\uc791\uc804\uc5d0 \uac78\ub824\ub4e4 \uc704\ud5d8\uc774 \uc788\ub2e4\ub294 \uc774\uc720\uc5d0\uc11c \uacac\ub0b4\ub7c9 \uc804\uc120\uc744 \uc720\uc9c0\ud558\uace0 \uacf5\uaca9\uc5d0 \uc2e0\uc911\ud558\uac8c \uc784\ud558\uace0\uc790 \ud558\uc600\ub2e4. \uc774\uc5d0 \uc870\uc815\uc5d0\uc11c\ub294 \uc774\uc21c\uc2e0\uc774 \uc9c0\ub098\uce58\uac8c \uc18c\uadf9\uc801\uc774\ub77c\ub294 \ube44\ub09c\uc774 \uc77c\uc5c8\ub2e4.", "answer": "\ub0a8\ud574\uc548 \uc77c\ub300", "sentence": "\ub2f9\uc2dc \uc77c\ubcf8\uad70\uc740 \ub0a8\ud574\uc548 \uc77c\ub300 \uc5d0 \ucd1d\uc9d1\uacb0\ud558\uc5ec \uc65c\uc131\uc744 \uc313\ub294 \ub4f1 \uc218\ube44\ub97c \uac15\ud654\ud558\uc600\uc73c\uba70 \uac15\ud654 \ud68c\ub2f4\uc758 \uc9c4\ud589 \uacfc\uc815\uc744 \uc9c0\ucf1c\ubcf4\uace0 \uc788\uc5c8\ub2e4.", "paragraph_sentence": "\uad50\ucc29\ud654\ud55c \uc804\uc138\uc5d0\uc11c \ucd08\uae30\uc758 \uc2b9\uc804\ubcf4 \uc774\ud6c4 \ubcc4\ub2e4\ub978 \uc2b9\ub9ac\uac00 \uc5c6\uc790 \uc120\uc870\ub97c \ube44\ub86f\ud55c \uc870\uc120 \uc870\uc815\uc5d0\uc11c\ub294 \uc774\uc21c\uc2e0\uc758 \uc804\ub7b5\uc744 \ubd88\uc2e0\ud558\uae30 \uc2dc\uc791\ud588\uc73c\uba70 \uc774\uc21c\uc2e0\uc5d0\uac8c \uc65c\uad70\uc5d0 \ub300\ud55c \uc801\uadf9\uc801\uc778 \uacf5\uaca9\uc744 \uac15\uc694\ud558\uac8c \ub418\uc5c8\ub2e4. \ub2f9\uc2dc \uc77c\ubcf8\uad70\uc740 \ub0a8\ud574\uc548 \uc77c\ub300 \uc5d0 \ucd1d\uc9d1\uacb0\ud558\uc5ec \uc65c\uc131\uc744 \uc313\ub294 \ub4f1 \uc218\ube44\ub97c \uac15\ud654\ud558\uc600\uc73c\uba70 \uac15\ud654 \ud68c\ub2f4\uc758 \uc9c4\ud589 \uacfc\uc815\uc744 \uc9c0\ucf1c\ubcf4\uace0 \uc788\uc5c8\ub2e4. \ud55c\ud3b8 \uba85\ub098\ub77c\ub294 \uc804\uba74\uc801 \ub300\uacb0\ubcf4\ub2e4\ub294 \uac15\ud654 \ud68c\ub2f4\uc5d0 \uae30\ub300\ud558\uace0 \uc788\uc5c8\uc73c\uba70, \uc870\uc120\uc740 \uc790\uccb4\uc758 \uad70\uc0ac\ub825\uc73c\ub85c \uc77c\ubcf8\uad70\uacfc \uc721\uc804\uc5d0\uc11c \ub300\ub4f1\ud55c \uc804\ud22c\ub97c \uc218\ud589\ud560 \ub2a5\ub825\uc774 \ubd80\uc871\ud588\ub2e4. \uadf8\ub7ec\ub098 \uc870\uc815\uc758 \uc694\uccad\uacfc\ub294 \ub2ec\ub9ac \uc774\uc21c\uc2e0\uc740 \uc77c\ubcf8\uad70\uc758 \uc720\uc778\uc791\uc804\uc5d0 \uac78\ub824\ub4e4 \uc704\ud5d8\uc774 \uc788\ub2e4\ub294 \uc774\uc720\uc5d0\uc11c \uacac\ub0b4\ub7c9 \uc804\uc120\uc744 \uc720\uc9c0\ud558\uace0 \uacf5\uaca9\uc5d0 \uc2e0\uc911\ud558\uac8c \uc784\ud558\uace0\uc790 \ud558\uc600\ub2e4. \uc774\uc5d0 \uc870\uc815\uc5d0\uc11c\ub294 \uc774\uc21c\uc2e0\uc774 \uc9c0\ub098\uce58\uac8c \uc18c\uadf9\uc801\uc774\ub77c\ub294 \ube44\ub09c\uc774 \uc77c\uc5c8\ub2e4.", "paragraph_answer": "\uad50\ucc29\ud654\ud55c \uc804\uc138\uc5d0\uc11c \ucd08\uae30\uc758 \uc2b9\uc804\ubcf4 \uc774\ud6c4 \ubcc4\ub2e4\ub978 \uc2b9\ub9ac\uac00 \uc5c6\uc790 \uc120\uc870\ub97c \ube44\ub86f\ud55c \uc870\uc120 \uc870\uc815\uc5d0\uc11c\ub294 \uc774\uc21c\uc2e0\uc758 \uc804\ub7b5\uc744 \ubd88\uc2e0\ud558\uae30 \uc2dc\uc791\ud588\uc73c\uba70 \uc774\uc21c\uc2e0\uc5d0\uac8c \uc65c\uad70\uc5d0 \ub300\ud55c \uc801\uadf9\uc801\uc778 \uacf5\uaca9\uc744 \uac15\uc694\ud558\uac8c \ub418\uc5c8\ub2e4. \ub2f9\uc2dc \uc77c\ubcf8\uad70\uc740 \ub0a8\ud574\uc548 \uc77c\ub300 \uc5d0 \ucd1d\uc9d1\uacb0\ud558\uc5ec \uc65c\uc131\uc744 \uc313\ub294 \ub4f1 \uc218\ube44\ub97c \uac15\ud654\ud558\uc600\uc73c\uba70 \uac15\ud654 \ud68c\ub2f4\uc758 \uc9c4\ud589 \uacfc\uc815\uc744 \uc9c0\ucf1c\ubcf4\uace0 \uc788\uc5c8\ub2e4. \ud55c\ud3b8 \uba85\ub098\ub77c\ub294 \uc804\uba74\uc801 \ub300\uacb0\ubcf4\ub2e4\ub294 \uac15\ud654 \ud68c\ub2f4\uc5d0 \uae30\ub300\ud558\uace0 \uc788\uc5c8\uc73c\uba70, \uc870\uc120\uc740 \uc790\uccb4\uc758 \uad70\uc0ac\ub825\uc73c\ub85c \uc77c\ubcf8\uad70\uacfc \uc721\uc804\uc5d0\uc11c \ub300\ub4f1\ud55c \uc804\ud22c\ub97c \uc218\ud589\ud560 \ub2a5\ub825\uc774 \ubd80\uc871\ud588\ub2e4. \uadf8\ub7ec\ub098 \uc870\uc815\uc758 \uc694\uccad\uacfc\ub294 \ub2ec\ub9ac \uc774\uc21c\uc2e0\uc740 \uc77c\ubcf8\uad70\uc758 \uc720\uc778\uc791\uc804\uc5d0 \uac78\ub824\ub4e4 \uc704\ud5d8\uc774 \uc788\ub2e4\ub294 \uc774\uc720\uc5d0\uc11c \uacac\ub0b4\ub7c9 \uc804\uc120\uc744 \uc720\uc9c0\ud558\uace0 \uacf5\uaca9\uc5d0 \uc2e0\uc911\ud558\uac8c \uc784\ud558\uace0\uc790 \ud558\uc600\ub2e4. \uc774\uc5d0 \uc870\uc815\uc5d0\uc11c\ub294 \uc774\uc21c\uc2e0\uc774 \uc9c0\ub098\uce58\uac8c \uc18c\uadf9\uc801\uc774\ub77c\ub294 \ube44\ub09c\uc774 \uc77c\uc5c8\ub2e4.", "sentence_answer": "\ub2f9\uc2dc \uc77c\ubcf8\uad70\uc740 \ub0a8\ud574\uc548 \uc77c\ub300 \uc5d0 \ucd1d\uc9d1\uacb0\ud558\uc5ec \uc65c\uc131\uc744 \uc313\ub294 \ub4f1 \uc218\ube44\ub97c \uac15\ud654\ud558\uc600\uc73c\uba70 \uac15\ud654 \ud68c\ub2f4\uc758 \uc9c4\ud589 \uacfc\uc815\uc744 \uc9c0\ucf1c\ubcf4\uace0 \uc788\uc5c8\ub2e4.", "paragraph_id": "6545629-9-0", "paragraph_question": "question: \uc77c\ubcf8\uad70\uc774 \ucd1d \uc9d1\uacb0\ud558\uc5ec \uc218\ube44\ub97c \uac15\ud654\ud55c \uc9c0\uc5ed\uc740 \uc5b4\ub514\uc778\uac00?, context: \uad50\ucc29\ud654\ud55c \uc804\uc138\uc5d0\uc11c \ucd08\uae30\uc758 \uc2b9\uc804\ubcf4 \uc774\ud6c4 \ubcc4\ub2e4\ub978 \uc2b9\ub9ac\uac00 \uc5c6\uc790 \uc120\uc870\ub97c \ube44\ub86f\ud55c \uc870\uc120 \uc870\uc815\uc5d0\uc11c\ub294 \uc774\uc21c\uc2e0\uc758 \uc804\ub7b5\uc744 \ubd88\uc2e0\ud558\uae30 \uc2dc\uc791\ud588\uc73c\uba70 \uc774\uc21c\uc2e0\uc5d0\uac8c \uc65c\uad70\uc5d0 \ub300\ud55c \uc801\uadf9\uc801\uc778 \uacf5\uaca9\uc744 \uac15\uc694\ud558\uac8c \ub418\uc5c8\ub2e4. \ub2f9\uc2dc \uc77c\ubcf8\uad70\uc740 \ub0a8\ud574\uc548 \uc77c\ub300\uc5d0 \ucd1d\uc9d1\uacb0\ud558\uc5ec \uc65c\uc131\uc744 \uc313\ub294 \ub4f1 \uc218\ube44\ub97c \uac15\ud654\ud558\uc600\uc73c\uba70 \uac15\ud654 \ud68c\ub2f4\uc758 \uc9c4\ud589 \uacfc\uc815\uc744 \uc9c0\ucf1c\ubcf4\uace0 \uc788\uc5c8\ub2e4. \ud55c\ud3b8 \uba85\ub098\ub77c\ub294 \uc804\uba74\uc801 \ub300\uacb0\ubcf4\ub2e4\ub294 \uac15\ud654 \ud68c\ub2f4\uc5d0 \uae30\ub300\ud558\uace0 \uc788\uc5c8\uc73c\uba70, \uc870\uc120\uc740 \uc790\uccb4\uc758 \uad70\uc0ac\ub825\uc73c\ub85c \uc77c\ubcf8\uad70\uacfc \uc721\uc804\uc5d0\uc11c \ub300\ub4f1\ud55c \uc804\ud22c\ub97c \uc218\ud589\ud560 \ub2a5\ub825\uc774 \ubd80\uc871\ud588\ub2e4. \uadf8\ub7ec\ub098 \uc870\uc815\uc758 \uc694\uccad\uacfc\ub294 \ub2ec\ub9ac \uc774\uc21c\uc2e0\uc740 \uc77c\ubcf8\uad70\uc758 \uc720\uc778\uc791\uc804\uc5d0 \uac78\ub824\ub4e4 \uc704\ud5d8\uc774 \uc788\ub2e4\ub294 \uc774\uc720\uc5d0\uc11c \uacac\ub0b4\ub7c9 \uc804\uc120\uc744 \uc720\uc9c0\ud558\uace0 \uacf5\uaca9\uc5d0 \uc2e0\uc911\ud558\uac8c \uc784\ud558\uace0\uc790 \ud558\uc600\ub2e4. \uc774\uc5d0 \uc870\uc815\uc5d0\uc11c\ub294 \uc774\uc21c\uc2e0\uc774 \uc9c0\ub098\uce58\uac8c \uc18c\uadf9\uc801\uc774\ub77c\ub294 \ube44\ub09c\uc774 \uc77c\uc5c8\ub2e4."} {"question": "\uc0c1\ud558\uc774\ub300\ud55c\uc778\uac70\ub958\ubbfc\ub2e8\uc758 \ucd1d\ubb34\ub294 \ub204\uad6c\uc778\uac00?", "paragraph": "\uc2e0\ud55c\uccad\ub144\ub2f9\uc740 1919\ub144 12\uc6d4 1\uc77c\ubd80\ud130 \uae30\uad00\uc7a1\uc9c0\ub85c\uc11c \u300a\uc2e0\ud55c\uccad\ub144\u300b\uc744 \ucc3d\uac04, \ubc1c\ud589\ud558\uc600\ub2e4. \ub610\ud55c \uc2e0\ud55c\uccad\ub144\ub2f9\uc740 \uc885\ub798\uc758 '\uc0c1\ud574\uace0\ub824\uad50\ubbfc\uce5c\ubaa9\ud68c'\ub97c \uac1c\ud3b8 \uac15\ud654\ud574\uc11c '\uc0c1\ud558\uc774\ub300\ud55c\uc778\uac70\ub958\ubbfc\ub2e8'\uc744 \ucc3d\ub9bd\ud558\uc5ec \uc5ec\uc6b4\ud615\uc774 \ub2e8\uc7a5, \uc120\uc6b0\ud601\uc774 \ucd1d\ubb34\ub97c \ub9e1\uc544\uc11c \ud55c\uad6d\ub3c5\ub9bd\uacfc \uad50\ubbfc\uad8c\uc775\uc744 \uc704\ud574 \ud65c\ub3d9\ud558\uc600\ub2e4. \uc2e0\ud55c\uccad\ub144\ub2f9\uc740 \ub610\ud55c 1920\ub144 8\uc6d4 \ubbf8\uad6d \uc704\uc6d0\ub2e8\uc774 \ubca0\uc774\uc9d5\uc5d0 \uc624\uc790, \uc784\uc2dc\uc815\ubd80 \uc694\uc778\uacfc \ud568\uaed8 \uadf8\ub4e4\uc744 \ub9cc\ub098 \ud55c\uad6d\ub3c5\ub9bd\uc5d0\uc758 \uc9c0\uc6d0\uc744 \uc694\uccad\ud558\ub294 \uc678\uad50\ud65c\ub3d9\uc744 \uc804\uac1c\ud558\uc600\ub2e4. \ub610\ud55c 1922\ub144 1\uc6d4 \uc18c\ub828\uc758 \ubaa8\uc2a4\ud06c\ubc14\uc5d0\uc11c '\uc81c3\uc778\ud130\ub0b4\uc154\ub110' \uc8fc\ucd5c\uc758 '\ub3d9\ubc29\ud53c\uc555\ubc15\ubbfc\uc871\ub300\ud68c'(\uc77c\uba85 \uadf9\ub3d9\uc778\ubbfc\ub300\ud45c\uc790\ub300\ud68c)\uac00 \uc5f4\ub9ac\uac8c \ub418\uc790 \uae40\uaddc\uc2dd\uacfc \uc5ec\uc6b4\ud615\uc744 \uc2e0\ud55c\uccad\ub144\ub2f9 \ub300\ud45c\ub85c \ud30c\uacac\ud558\uc5ec \ud65c\ub3d9\ucf00 \ud558\uc600\ub2e4. \uc2e0\ud55c\uccad\ub144\ub2f9\uc740 \ub610\ud55c 1922\ub144 11\uc6d4 \ub2f9\uc6d0 \uae40\uad6c\uc758 \uc81c\uc758\uc5d0 \uc758\ud574 \ud55c\uad6d\ub178\ubcd1\ud68c\ub97c \ucc3d\ub9bd\ud558\uace0 \ub3c5\ub9bd\uc804\uc7c1\uc744 \uc704\ud55c \uc2e4\ub825\uc900\ube44\uc0ac\uc5c5\uc744 \ucd94\uc9c4\ud558\uae30\ub85c \ud558\uc600\ub2e4.", "answer": "\uc120\uc6b0\ud601", "sentence": "\ub610\ud55c \uc2e0\ud55c\uccad\ub144\ub2f9\uc740 \uc885\ub798\uc758 '\uc0c1\ud574\uace0\ub824\uad50\ubbfc\uce5c\ubaa9\ud68c'\ub97c \uac1c\ud3b8 \uac15\ud654\ud574\uc11c '\uc0c1\ud558\uc774\ub300\ud55c\uc778\uac70\ub958\ubbfc\ub2e8'\uc744 \ucc3d\ub9bd\ud558\uc5ec \uc5ec\uc6b4\ud615\uc774 \ub2e8\uc7a5, \uc120\uc6b0\ud601 \uc774 \ucd1d\ubb34\ub97c \ub9e1\uc544\uc11c \ud55c\uad6d\ub3c5\ub9bd\uacfc \uad50\ubbfc\uad8c\uc775\uc744 \uc704\ud574 \ud65c\ub3d9\ud558\uc600\ub2e4.", "paragraph_sentence": "\uc2e0\ud55c\uccad\ub144\ub2f9\uc740 1919\ub144 12\uc6d4 1\uc77c\ubd80\ud130 \uae30\uad00\uc7a1\uc9c0\ub85c\uc11c \u300a\uc2e0\ud55c\uccad\ub144\u300b\uc744 \ucc3d\uac04, \ubc1c\ud589\ud558\uc600\ub2e4. \ub610\ud55c \uc2e0\ud55c\uccad\ub144\ub2f9\uc740 \uc885\ub798\uc758 '\uc0c1\ud574\uace0\ub824\uad50\ubbfc\uce5c\ubaa9\ud68c'\ub97c \uac1c\ud3b8 \uac15\ud654\ud574\uc11c '\uc0c1\ud558\uc774\ub300\ud55c\uc778\uac70\ub958\ubbfc\ub2e8'\uc744 \ucc3d\ub9bd\ud558\uc5ec \uc5ec\uc6b4\ud615\uc774 \ub2e8\uc7a5, \uc120\uc6b0\ud601 \uc774 \ucd1d\ubb34\ub97c \ub9e1\uc544\uc11c \ud55c\uad6d\ub3c5\ub9bd\uacfc \uad50\ubbfc\uad8c\uc775\uc744 \uc704\ud574 \ud65c\ub3d9\ud558\uc600\ub2e4. \uc2e0\ud55c\uccad\ub144\ub2f9\uc740 \ub610\ud55c 1920\ub144 8\uc6d4 \ubbf8\uad6d \uc704\uc6d0\ub2e8\uc774 \ubca0\uc774\uc9d5\uc5d0 \uc624\uc790, \uc784\uc2dc\uc815\ubd80 \uc694\uc778\uacfc \ud568\uaed8 \uadf8\ub4e4\uc744 \ub9cc\ub098 \ud55c\uad6d\ub3c5\ub9bd\uc5d0\uc758 \uc9c0\uc6d0\uc744 \uc694\uccad\ud558\ub294 \uc678\uad50\ud65c\ub3d9\uc744 \uc804\uac1c\ud558\uc600\ub2e4. \ub610\ud55c 1922\ub144 1\uc6d4 \uc18c\ub828\uc758 \ubaa8\uc2a4\ud06c\ubc14\uc5d0\uc11c '\uc81c3\uc778\ud130\ub0b4\uc154\ub110' \uc8fc\ucd5c\uc758 '\ub3d9\ubc29\ud53c\uc555\ubc15\ubbfc\uc871\ub300\ud68c'(\uc77c\uba85 \uadf9\ub3d9\uc778\ubbfc\ub300\ud45c\uc790\ub300\ud68c)\uac00 \uc5f4\ub9ac\uac8c \ub418\uc790 \uae40\uaddc\uc2dd\uacfc \uc5ec\uc6b4\ud615\uc744 \uc2e0\ud55c\uccad\ub144\ub2f9 \ub300\ud45c\ub85c \ud30c\uacac\ud558\uc5ec \ud65c\ub3d9\ucf00 \ud558\uc600\ub2e4. \uc2e0\ud55c\uccad\ub144\ub2f9\uc740 \ub610\ud55c 1922\ub144 11\uc6d4 \ub2f9\uc6d0 \uae40\uad6c\uc758 \uc81c\uc758\uc5d0 \uc758\ud574 \ud55c\uad6d\ub178\ubcd1\ud68c\ub97c \ucc3d\ub9bd\ud558\uace0 \ub3c5\ub9bd\uc804\uc7c1\uc744 \uc704\ud55c \uc2e4\ub825\uc900\ube44\uc0ac\uc5c5\uc744 \ucd94\uc9c4\ud558\uae30\ub85c \ud558\uc600\ub2e4.", "paragraph_answer": "\uc2e0\ud55c\uccad\ub144\ub2f9\uc740 1919\ub144 12\uc6d4 1\uc77c\ubd80\ud130 \uae30\uad00\uc7a1\uc9c0\ub85c\uc11c \u300a\uc2e0\ud55c\uccad\ub144\u300b\uc744 \ucc3d\uac04, \ubc1c\ud589\ud558\uc600\ub2e4. \ub610\ud55c \uc2e0\ud55c\uccad\ub144\ub2f9\uc740 \uc885\ub798\uc758 '\uc0c1\ud574\uace0\ub824\uad50\ubbfc\uce5c\ubaa9\ud68c'\ub97c \uac1c\ud3b8 \uac15\ud654\ud574\uc11c '\uc0c1\ud558\uc774\ub300\ud55c\uc778\uac70\ub958\ubbfc\ub2e8'\uc744 \ucc3d\ub9bd\ud558\uc5ec \uc5ec\uc6b4\ud615\uc774 \ub2e8\uc7a5, \uc120\uc6b0\ud601 \uc774 \ucd1d\ubb34\ub97c \ub9e1\uc544\uc11c \ud55c\uad6d\ub3c5\ub9bd\uacfc \uad50\ubbfc\uad8c\uc775\uc744 \uc704\ud574 \ud65c\ub3d9\ud558\uc600\ub2e4. \uc2e0\ud55c\uccad\ub144\ub2f9\uc740 \ub610\ud55c 1920\ub144 8\uc6d4 \ubbf8\uad6d \uc704\uc6d0\ub2e8\uc774 \ubca0\uc774\uc9d5\uc5d0 \uc624\uc790, \uc784\uc2dc\uc815\ubd80 \uc694\uc778\uacfc \ud568\uaed8 \uadf8\ub4e4\uc744 \ub9cc\ub098 \ud55c\uad6d\ub3c5\ub9bd\uc5d0\uc758 \uc9c0\uc6d0\uc744 \uc694\uccad\ud558\ub294 \uc678\uad50\ud65c\ub3d9\uc744 \uc804\uac1c\ud558\uc600\ub2e4. \ub610\ud55c 1922\ub144 1\uc6d4 \uc18c\ub828\uc758 \ubaa8\uc2a4\ud06c\ubc14\uc5d0\uc11c '\uc81c3\uc778\ud130\ub0b4\uc154\ub110' \uc8fc\ucd5c\uc758 '\ub3d9\ubc29\ud53c\uc555\ubc15\ubbfc\uc871\ub300\ud68c'(\uc77c\uba85 \uadf9\ub3d9\uc778\ubbfc\ub300\ud45c\uc790\ub300\ud68c)\uac00 \uc5f4\ub9ac\uac8c \ub418\uc790 \uae40\uaddc\uc2dd\uacfc \uc5ec\uc6b4\ud615\uc744 \uc2e0\ud55c\uccad\ub144\ub2f9 \ub300\ud45c\ub85c \ud30c\uacac\ud558\uc5ec \ud65c\ub3d9\ucf00 \ud558\uc600\ub2e4. \uc2e0\ud55c\uccad\ub144\ub2f9\uc740 \ub610\ud55c 1922\ub144 11\uc6d4 \ub2f9\uc6d0 \uae40\uad6c\uc758 \uc81c\uc758\uc5d0 \uc758\ud574 \ud55c\uad6d\ub178\ubcd1\ud68c\ub97c \ucc3d\ub9bd\ud558\uace0 \ub3c5\ub9bd\uc804\uc7c1\uc744 \uc704\ud55c \uc2e4\ub825\uc900\ube44\uc0ac\uc5c5\uc744 \ucd94\uc9c4\ud558\uae30\ub85c \ud558\uc600\ub2e4.", "sentence_answer": "\ub610\ud55c \uc2e0\ud55c\uccad\ub144\ub2f9\uc740 \uc885\ub798\uc758 '\uc0c1\ud574\uace0\ub824\uad50\ubbfc\uce5c\ubaa9\ud68c'\ub97c \uac1c\ud3b8 \uac15\ud654\ud574\uc11c '\uc0c1\ud558\uc774\ub300\ud55c\uc778\uac70\ub958\ubbfc\ub2e8'\uc744 \ucc3d\ub9bd\ud558\uc5ec \uc5ec\uc6b4\ud615\uc774 \ub2e8\uc7a5, \uc120\uc6b0\ud601 \uc774 \ucd1d\ubb34\ub97c \ub9e1\uc544\uc11c \ud55c\uad6d\ub3c5\ub9bd\uacfc \uad50\ubbfc\uad8c\uc775\uc744 \uc704\ud574 \ud65c\ub3d9\ud558\uc600\ub2e4.", "paragraph_id": "6538512-8-1", "paragraph_question": "question: \uc0c1\ud558\uc774\ub300\ud55c\uc778\uac70\ub958\ubbfc\ub2e8\uc758 \ucd1d\ubb34\ub294 \ub204\uad6c\uc778\uac00?, context: \uc2e0\ud55c\uccad\ub144\ub2f9\uc740 1919\ub144 12\uc6d4 1\uc77c\ubd80\ud130 \uae30\uad00\uc7a1\uc9c0\ub85c\uc11c \u300a\uc2e0\ud55c\uccad\ub144\u300b\uc744 \ucc3d\uac04, \ubc1c\ud589\ud558\uc600\ub2e4. \ub610\ud55c \uc2e0\ud55c\uccad\ub144\ub2f9\uc740 \uc885\ub798\uc758 '\uc0c1\ud574\uace0\ub824\uad50\ubbfc\uce5c\ubaa9\ud68c'\ub97c \uac1c\ud3b8 \uac15\ud654\ud574\uc11c '\uc0c1\ud558\uc774\ub300\ud55c\uc778\uac70\ub958\ubbfc\ub2e8'\uc744 \ucc3d\ub9bd\ud558\uc5ec \uc5ec\uc6b4\ud615\uc774 \ub2e8\uc7a5, \uc120\uc6b0\ud601\uc774 \ucd1d\ubb34\ub97c \ub9e1\uc544\uc11c \ud55c\uad6d\ub3c5\ub9bd\uacfc \uad50\ubbfc\uad8c\uc775\uc744 \uc704\ud574 \ud65c\ub3d9\ud558\uc600\ub2e4. \uc2e0\ud55c\uccad\ub144\ub2f9\uc740 \ub610\ud55c 1920\ub144 8\uc6d4 \ubbf8\uad6d \uc704\uc6d0\ub2e8\uc774 \ubca0\uc774\uc9d5\uc5d0 \uc624\uc790, \uc784\uc2dc\uc815\ubd80 \uc694\uc778\uacfc \ud568\uaed8 \uadf8\ub4e4\uc744 \ub9cc\ub098 \ud55c\uad6d\ub3c5\ub9bd\uc5d0\uc758 \uc9c0\uc6d0\uc744 \uc694\uccad\ud558\ub294 \uc678\uad50\ud65c\ub3d9\uc744 \uc804\uac1c\ud558\uc600\ub2e4. \ub610\ud55c 1922\ub144 1\uc6d4 \uc18c\ub828\uc758 \ubaa8\uc2a4\ud06c\ubc14\uc5d0\uc11c '\uc81c3\uc778\ud130\ub0b4\uc154\ub110' \uc8fc\ucd5c\uc758 '\ub3d9\ubc29\ud53c\uc555\ubc15\ubbfc\uc871\ub300\ud68c'(\uc77c\uba85 \uadf9\ub3d9\uc778\ubbfc\ub300\ud45c\uc790\ub300\ud68c)\uac00 \uc5f4\ub9ac\uac8c \ub418\uc790 \uae40\uaddc\uc2dd\uacfc \uc5ec\uc6b4\ud615\uc744 \uc2e0\ud55c\uccad\ub144\ub2f9 \ub300\ud45c\ub85c \ud30c\uacac\ud558\uc5ec \ud65c\ub3d9\ucf00 \ud558\uc600\ub2e4. \uc2e0\ud55c\uccad\ub144\ub2f9\uc740 \ub610\ud55c 1922\ub144 11\uc6d4 \ub2f9\uc6d0 \uae40\uad6c\uc758 \uc81c\uc758\uc5d0 \uc758\ud574 \ud55c\uad6d\ub178\ubcd1\ud68c\ub97c \ucc3d\ub9bd\ud558\uace0 \ub3c5\ub9bd\uc804\uc7c1\uc744 \uc704\ud55c \uc2e4\ub825\uc900\ube44\uc0ac\uc5c5\uc744 \ucd94\uc9c4\ud558\uae30\ub85c \ud558\uc600\ub2e4."} {"question": "\uc804 \uc138\uacc4 \ube48\uace4\uad6d\uac00 \uc544\uc774\ub4e4\uc5d0\uac8c \uad50\uc721\uc801, \uc815\uc11c\uc801 \uc9c0\uc6d0\uc744 \ud558\uae30 \uc704\ud55c \ud504\ub85c\uc81d\ud2b8\ub294?", "paragraph": "\ubc15\uc2e0\ud61c\ub294 \ub610\ud55c 2011\ub144\ubd80\ud130 \uae30\uc544\ub300\ucc45 \ud64d\ubcf4\ub300\uc0ac\ub85c \ud65c\ub3d9, \uc11c\ubd80 \uc544\ud504\ub9ac\uce74 \uac00\ub098\uc5d0 \uc788\ub294 \uacb0\uc5f0\uc544\ub3d9 \uc544\ubc18\ub124\ub97c \uacb0\uc5f0\ud6c4\uc6d0 \ud574\uc624\uace0 \uc788\uc73c\uba70 \ud32c\ub4e4\uacfc \ud574\uc678 \ube48\uace4\uad6d\uc5d0 \uc13c\ud130 \uc138\uc6b0\ub294 \uc77c\uc5d0 \ub3d9\ucc38\ud588\ub2e4. \uad6d\ub0b4\uc678 \ud32c\ub4e4\uacfc \uc804 \uc138\uacc4 \ube48\uace4\uad6d\uac00 \uc544\uc774\ub4e4\uc758 \uad50\uc721\uc801, \uc815\uc11c\uc801 \uc9c0\uc6d0\uc744 \uc704\ud55c \u2018\ubcc4\ube5b\ucc9c\uc0ac\ub9c8\uc744\u2019 \ub9cc\ub4e4\uae30 \ud504\ub85c\uc81d\ud2b8\ub97c \uc9c4\ud589\ud574\uc654\ub2e4. 1\ud638 \uc13c\ud130\ub85c \uc11c\ubd80 \uc544\ud504\ub9ac\uce74 \uac00\ub098 \u2018\uc2e0\ud61c\uc13c\ud130(Shin Hye Center)\u2019\uac00 2013\ub144 11\uc6d4 \uc644\uacf5\ub418\uc5b4 5\ubc31\uc5ec \uba85\uc758 \ud559\uc0dd\ub4e4\uc774 \uc774\uc6a9\ud558\uace0 \uc788\ub2e4. 2014\ub144\uc5d0\ub294 \uc138\uc6d4\ud638 \ud76c\uc0dd\uc790 \uac00\uc871\uc744 \uc704\ud574 \ud76c\ub9dd\ube0c\ub9ac\uc9c0 \uc804\uad6d\uc7ac\ud574\uad6c\ud638\ud611\ud68c\ub97c \ud1b5\ud574 5,000\ub9cc\uc6d0\uc744 \uae30\ubd80\ud588\ub2e4. \ub610\ud55c \uc790\uc2e0\uc758 SNS \ud2b8\uc704\ud130 \ud504\ub85c\ud544 \uc0ac\uc9c4\uc744 \ub178\ub780\ub9ac\ubcf8 \uc0ac\uc9c4\uc73c\ub85c \ubcc0\uacbd, \ub178\ub780\ub9ac\ubcf8 \ucea0\ud398\uc778\uc5d0\ub3c4 \ub3d9\ucc38\ud588\ub2e4. 2015\ub144, \ubc15\uc2e0\ud61c\ub294 \u20182015 Dream of Angel\u2019 \ud0c0\uc774\ud2c0\ub85c \uc544\uc2dc\uc544\ud22c\uc5b4\uc5d0 \ub098\uc130\uace0, \uc0c1\ud574 \ud32c\ubbf8\ud305 \uc218\uc775\uc744 NGO \ub2e8\uccb4 \u2019\uae30\uc544\ub300\ucc45\u2019\uc758 \ubcc4\ube5b\ucc9c\uc0ac\ub9c8\uc744 \ub9cc\ub4e4\uae30\uc640 \uc911\uad6d\uc758 \uc720\uae30\ub3d9\ubb3c \ubcf4\ud638\ub2e8\uccb4 \u2018\ud0c0\uc18c\uc6d0\u2019\uc5d0 \uae30\ubd80\ud588\ub2e4. 4\uc6d4, \ubc15\uc2e0\ud61c\ub294 \ud070 \ud53c\ud574\ub97c \uc785\uc740 \ub124\ud314 \uad6d\ubbfc\ub4e4\uc744 \uc704\ud574 \uad6d\uc81c\uad6c\ud638\ub2e8\uccb4 \uae30\uc544\ub300\ucc45\ub97c \ud1b5\ud574 3,000\ub9cc\uc6d0\uc744 \uae30\ubd80\ud588\ub2e4. 2016\ub144 7\uc6d4, \ubc15\uc2e0\ud61c\ub294 \uc544\ud504\ub9ac\uce74 \uac00\ub098\uc758 \uc2e0\ud61c\uc13c\ud130\uc5d0 \uc774\uc5b4 \ud544\ub9ac\ud540 \ub9c8\ub2d0\ub77c\uc5d0 \uc2e0\ud61c\uc13c\ud130\uc758 \uac74\ub9bd\uc744 \uc9c4\ud589\ud588\ub2e4. \uc774\uc5d0 \ubc15\uc2e0\ud61c\uc640 \uc18c\uc18d\uc0ac \uc194\ud2b8 \uc5d4\ud130\ud14c\uc778\uba3c\ud2b8 \uc9c1\uc6d0\ub4e4\uc740 \uc13c\ud130\uac74\ub9bd\uc5d0 \uc55e\uc11c \ud544\ub9ac\ud540 \uc308\ube68\ub85d \uc9c0\uc5ed\uc758 \uc544\uc774\ub4e4\uacfc 1:1 \uacb0\uc5f0\uc744 \ub9fa\uc5c8\uace0, \uc9c0\ub09c 3\uc6d4 \ubc15\uc2e0\ud61c\ub294 \ud544\ub9ac\ud540\uc758 \uacb0\uc5f0\uc544\ub3d9\uc758 \uc9d1\uc744 \uc9c1\uc811 \ubc29\ubb38\ud558\uace0 \uc544\ub3d9\uacb0\uc5f0\uc13c\ud130 \uc544\uc774\ub4e4\uacfc \ud568\uaed8 \uc990\uac70\uc6b4 \uc2dc\uac04\uc744 \uac00\uc9c0\uba70 \uc13c\ud130 \uac74\ub9bd\uc5d0 \uc55e\uc11c \uac74\ucd95\uae30\ub150\uc2dd\uc744 \uac00\uc84c\ub2e4.", "answer": "\ubcc4\ube5b\ucc9c\uc0ac\ub9c8\uc744 \ub9cc\ub4e4\uae30", "sentence": "2015\ub144, \ubc15\uc2e0\ud61c\ub294 \u20182015 Dream of Angel\u2019 \ud0c0\uc774\ud2c0\ub85c \uc544\uc2dc\uc544\ud22c\uc5b4\uc5d0 \ub098\uc130\uace0, \uc0c1\ud574 \ud32c\ubbf8\ud305 \uc218\uc775\uc744 NGO \ub2e8\uccb4 \u2019\uae30\uc544\ub300\ucc45\u2019\uc758 \ubcc4\ube5b\ucc9c\uc0ac\ub9c8\uc744 \ub9cc\ub4e4\uae30 \uc640 \uc911\uad6d\uc758 \uc720\uae30\ub3d9\ubb3c \ubcf4\ud638\ub2e8\uccb4 \u2018\ud0c0\uc18c\uc6d0\u2019\uc5d0 \uae30\ubd80\ud588\ub2e4.", "paragraph_sentence": "\ubc15\uc2e0\ud61c\ub294 \ub610\ud55c 2011\ub144\ubd80\ud130 \uae30\uc544\ub300\ucc45 \ud64d\ubcf4\ub300\uc0ac\ub85c \ud65c\ub3d9, \uc11c\ubd80 \uc544\ud504\ub9ac\uce74 \uac00\ub098\uc5d0 \uc788\ub294 \uacb0\uc5f0\uc544\ub3d9 \uc544\ubc18\ub124\ub97c \uacb0\uc5f0\ud6c4\uc6d0 \ud574\uc624\uace0 \uc788\uc73c\uba70 \ud32c\ub4e4\uacfc \ud574\uc678 \ube48\uace4\uad6d\uc5d0 \uc13c\ud130 \uc138\uc6b0\ub294 \uc77c\uc5d0 \ub3d9\ucc38\ud588\ub2e4. \uad6d\ub0b4\uc678 \ud32c\ub4e4\uacfc \uc804 \uc138\uacc4 \ube48\uace4\uad6d\uac00 \uc544\uc774\ub4e4\uc758 \uad50\uc721\uc801, \uc815\uc11c\uc801 \uc9c0\uc6d0\uc744 \uc704\ud55c \u2018\ubcc4\ube5b\ucc9c\uc0ac\ub9c8\uc744\u2019 \ub9cc\ub4e4\uae30 \ud504\ub85c\uc81d\ud2b8\ub97c \uc9c4\ud589\ud574\uc654\ub2e4. 1\ud638 \uc13c\ud130\ub85c \uc11c\ubd80 \uc544\ud504\ub9ac\uce74 \uac00\ub098 \u2018\uc2e0\ud61c\uc13c\ud130(Shin Hye Center)\u2019\uac00 2013\ub144 11\uc6d4 \uc644\uacf5\ub418\uc5b4 5\ubc31\uc5ec \uba85\uc758 \ud559\uc0dd\ub4e4\uc774 \uc774\uc6a9\ud558\uace0 \uc788\ub2e4. 2014\ub144\uc5d0\ub294 \uc138\uc6d4\ud638 \ud76c\uc0dd\uc790 \uac00\uc871\uc744 \uc704\ud574 \ud76c\ub9dd\ube0c\ub9ac\uc9c0 \uc804\uad6d\uc7ac\ud574\uad6c\ud638\ud611\ud68c\ub97c \ud1b5\ud574 5,000\ub9cc\uc6d0\uc744 \uae30\ubd80\ud588\ub2e4. \ub610\ud55c \uc790\uc2e0\uc758 SNS \ud2b8\uc704\ud130 \ud504\ub85c\ud544 \uc0ac\uc9c4\uc744 \ub178\ub780\ub9ac\ubcf8 \uc0ac\uc9c4\uc73c\ub85c \ubcc0\uacbd, \ub178\ub780\ub9ac\ubcf8 \ucea0\ud398\uc778\uc5d0\ub3c4 \ub3d9\ucc38\ud588\ub2e4. 2015\ub144, \ubc15\uc2e0\ud61c\ub294 \u20182015 Dream of Angel\u2019 \ud0c0\uc774\ud2c0\ub85c \uc544\uc2dc\uc544\ud22c\uc5b4\uc5d0 \ub098\uc130\uace0, \uc0c1\ud574 \ud32c\ubbf8\ud305 \uc218\uc775\uc744 NGO \ub2e8\uccb4 \u2019\uae30\uc544\ub300\ucc45\u2019\uc758 \ubcc4\ube5b\ucc9c\uc0ac\ub9c8\uc744 \ub9cc\ub4e4\uae30 \uc640 \uc911\uad6d\uc758 \uc720\uae30\ub3d9\ubb3c \ubcf4\ud638\ub2e8\uccb4 \u2018\ud0c0\uc18c\uc6d0\u2019\uc5d0 \uae30\ubd80\ud588\ub2e4. 4\uc6d4, \ubc15\uc2e0\ud61c\ub294 \ud070 \ud53c\ud574\ub97c \uc785\uc740 \ub124\ud314 \uad6d\ubbfc\ub4e4\uc744 \uc704\ud574 \uad6d\uc81c\uad6c\ud638\ub2e8\uccb4 \uae30\uc544\ub300\ucc45\ub97c \ud1b5\ud574 3,000\ub9cc\uc6d0\uc744 \uae30\ubd80\ud588\ub2e4. 2016\ub144 7\uc6d4, \ubc15\uc2e0\ud61c\ub294 \uc544\ud504\ub9ac\uce74 \uac00\ub098\uc758 \uc2e0\ud61c\uc13c\ud130\uc5d0 \uc774\uc5b4 \ud544\ub9ac\ud540 \ub9c8\ub2d0\ub77c\uc5d0 \uc2e0\ud61c\uc13c\ud130\uc758 \uac74\ub9bd\uc744 \uc9c4\ud589\ud588\ub2e4. \uc774\uc5d0 \ubc15\uc2e0\ud61c\uc640 \uc18c\uc18d\uc0ac \uc194\ud2b8 \uc5d4\ud130\ud14c\uc778\uba3c\ud2b8 \uc9c1\uc6d0\ub4e4\uc740 \uc13c\ud130\uac74\ub9bd\uc5d0 \uc55e\uc11c \ud544\ub9ac\ud540 \uc308\ube68\ub85d \uc9c0\uc5ed\uc758 \uc544\uc774\ub4e4\uacfc 1:1 \uacb0\uc5f0\uc744 \ub9fa\uc5c8\uace0, \uc9c0\ub09c 3\uc6d4 \ubc15\uc2e0\ud61c\ub294 \ud544\ub9ac\ud540\uc758 \uacb0\uc5f0\uc544\ub3d9\uc758 \uc9d1\uc744 \uc9c1\uc811 \ubc29\ubb38\ud558\uace0 \uc544\ub3d9\uacb0\uc5f0\uc13c\ud130 \uc544\uc774\ub4e4\uacfc \ud568\uaed8 \uc990\uac70\uc6b4 \uc2dc\uac04\uc744 \uac00\uc9c0\uba70 \uc13c\ud130 \uac74\ub9bd\uc5d0 \uc55e\uc11c \uac74\ucd95\uae30\ub150\uc2dd\uc744 \uac00\uc84c\ub2e4.", "paragraph_answer": "\ubc15\uc2e0\ud61c\ub294 \ub610\ud55c 2011\ub144\ubd80\ud130 \uae30\uc544\ub300\ucc45 \ud64d\ubcf4\ub300\uc0ac\ub85c \ud65c\ub3d9, \uc11c\ubd80 \uc544\ud504\ub9ac\uce74 \uac00\ub098\uc5d0 \uc788\ub294 \uacb0\uc5f0\uc544\ub3d9 \uc544\ubc18\ub124\ub97c \uacb0\uc5f0\ud6c4\uc6d0 \ud574\uc624\uace0 \uc788\uc73c\uba70 \ud32c\ub4e4\uacfc \ud574\uc678 \ube48\uace4\uad6d\uc5d0 \uc13c\ud130 \uc138\uc6b0\ub294 \uc77c\uc5d0 \ub3d9\ucc38\ud588\ub2e4. \uad6d\ub0b4\uc678 \ud32c\ub4e4\uacfc \uc804 \uc138\uacc4 \ube48\uace4\uad6d\uac00 \uc544\uc774\ub4e4\uc758 \uad50\uc721\uc801, \uc815\uc11c\uc801 \uc9c0\uc6d0\uc744 \uc704\ud55c \u2018\ubcc4\ube5b\ucc9c\uc0ac\ub9c8\uc744\u2019 \ub9cc\ub4e4\uae30 \ud504\ub85c\uc81d\ud2b8\ub97c \uc9c4\ud589\ud574\uc654\ub2e4. 1\ud638 \uc13c\ud130\ub85c \uc11c\ubd80 \uc544\ud504\ub9ac\uce74 \uac00\ub098 \u2018\uc2e0\ud61c\uc13c\ud130(Shin Hye Center)\u2019\uac00 2013\ub144 11\uc6d4 \uc644\uacf5\ub418\uc5b4 5\ubc31\uc5ec \uba85\uc758 \ud559\uc0dd\ub4e4\uc774 \uc774\uc6a9\ud558\uace0 \uc788\ub2e4. 2014\ub144\uc5d0\ub294 \uc138\uc6d4\ud638 \ud76c\uc0dd\uc790 \uac00\uc871\uc744 \uc704\ud574 \ud76c\ub9dd\ube0c\ub9ac\uc9c0 \uc804\uad6d\uc7ac\ud574\uad6c\ud638\ud611\ud68c\ub97c \ud1b5\ud574 5,000\ub9cc\uc6d0\uc744 \uae30\ubd80\ud588\ub2e4. \ub610\ud55c \uc790\uc2e0\uc758 SNS \ud2b8\uc704\ud130 \ud504\ub85c\ud544 \uc0ac\uc9c4\uc744 \ub178\ub780\ub9ac\ubcf8 \uc0ac\uc9c4\uc73c\ub85c \ubcc0\uacbd, \ub178\ub780\ub9ac\ubcf8 \ucea0\ud398\uc778\uc5d0\ub3c4 \ub3d9\ucc38\ud588\ub2e4. 2015\ub144, \ubc15\uc2e0\ud61c\ub294 \u20182015 Dream of Angel\u2019 \ud0c0\uc774\ud2c0\ub85c \uc544\uc2dc\uc544\ud22c\uc5b4\uc5d0 \ub098\uc130\uace0, \uc0c1\ud574 \ud32c\ubbf8\ud305 \uc218\uc775\uc744 NGO \ub2e8\uccb4 \u2019\uae30\uc544\ub300\ucc45\u2019\uc758 \ubcc4\ube5b\ucc9c\uc0ac\ub9c8\uc744 \ub9cc\ub4e4\uae30 \uc640 \uc911\uad6d\uc758 \uc720\uae30\ub3d9\ubb3c \ubcf4\ud638\ub2e8\uccb4 \u2018\ud0c0\uc18c\uc6d0\u2019\uc5d0 \uae30\ubd80\ud588\ub2e4. 4\uc6d4, \ubc15\uc2e0\ud61c\ub294 \ud070 \ud53c\ud574\ub97c \uc785\uc740 \ub124\ud314 \uad6d\ubbfc\ub4e4\uc744 \uc704\ud574 \uad6d\uc81c\uad6c\ud638\ub2e8\uccb4 \uae30\uc544\ub300\ucc45\ub97c \ud1b5\ud574 3,000\ub9cc\uc6d0\uc744 \uae30\ubd80\ud588\ub2e4. 2016\ub144 7\uc6d4, \ubc15\uc2e0\ud61c\ub294 \uc544\ud504\ub9ac\uce74 \uac00\ub098\uc758 \uc2e0\ud61c\uc13c\ud130\uc5d0 \uc774\uc5b4 \ud544\ub9ac\ud540 \ub9c8\ub2d0\ub77c\uc5d0 \uc2e0\ud61c\uc13c\ud130\uc758 \uac74\ub9bd\uc744 \uc9c4\ud589\ud588\ub2e4. \uc774\uc5d0 \ubc15\uc2e0\ud61c\uc640 \uc18c\uc18d\uc0ac \uc194\ud2b8 \uc5d4\ud130\ud14c\uc778\uba3c\ud2b8 \uc9c1\uc6d0\ub4e4\uc740 \uc13c\ud130\uac74\ub9bd\uc5d0 \uc55e\uc11c \ud544\ub9ac\ud540 \uc308\ube68\ub85d \uc9c0\uc5ed\uc758 \uc544\uc774\ub4e4\uacfc 1:1 \uacb0\uc5f0\uc744 \ub9fa\uc5c8\uace0, \uc9c0\ub09c 3\uc6d4 \ubc15\uc2e0\ud61c\ub294 \ud544\ub9ac\ud540\uc758 \uacb0\uc5f0\uc544\ub3d9\uc758 \uc9d1\uc744 \uc9c1\uc811 \ubc29\ubb38\ud558\uace0 \uc544\ub3d9\uacb0\uc5f0\uc13c\ud130 \uc544\uc774\ub4e4\uacfc \ud568\uaed8 \uc990\uac70\uc6b4 \uc2dc\uac04\uc744 \uac00\uc9c0\uba70 \uc13c\ud130 \uac74\ub9bd\uc5d0 \uc55e\uc11c \uac74\ucd95\uae30\ub150\uc2dd\uc744 \uac00\uc84c\ub2e4.", "sentence_answer": "2015\ub144, \ubc15\uc2e0\ud61c\ub294 \u20182015 Dream of Angel\u2019 \ud0c0\uc774\ud2c0\ub85c \uc544\uc2dc\uc544\ud22c\uc5b4\uc5d0 \ub098\uc130\uace0, \uc0c1\ud574 \ud32c\ubbf8\ud305 \uc218\uc775\uc744 NGO \ub2e8\uccb4 \u2019\uae30\uc544\ub300\ucc45\u2019\uc758 \ubcc4\ube5b\ucc9c\uc0ac\ub9c8\uc744 \ub9cc\ub4e4\uae30 \uc640 \uc911\uad6d\uc758 \uc720\uae30\ub3d9\ubb3c \ubcf4\ud638\ub2e8\uccb4 \u2018\ud0c0\uc18c\uc6d0\u2019\uc5d0 \uae30\ubd80\ud588\ub2e4.", "paragraph_id": "6581305-7-1", "paragraph_question": "question: \uc804 \uc138\uacc4 \ube48\uace4\uad6d\uac00 \uc544\uc774\ub4e4\uc5d0\uac8c \uad50\uc721\uc801, \uc815\uc11c\uc801 \uc9c0\uc6d0\uc744 \ud558\uae30 \uc704\ud55c \ud504\ub85c\uc81d\ud2b8\ub294?, context: \ubc15\uc2e0\ud61c\ub294 \ub610\ud55c 2011\ub144\ubd80\ud130 \uae30\uc544\ub300\ucc45 \ud64d\ubcf4\ub300\uc0ac\ub85c \ud65c\ub3d9, \uc11c\ubd80 \uc544\ud504\ub9ac\uce74 \uac00\ub098\uc5d0 \uc788\ub294 \uacb0\uc5f0\uc544\ub3d9 \uc544\ubc18\ub124\ub97c \uacb0\uc5f0\ud6c4\uc6d0 \ud574\uc624\uace0 \uc788\uc73c\uba70 \ud32c\ub4e4\uacfc \ud574\uc678 \ube48\uace4\uad6d\uc5d0 \uc13c\ud130 \uc138\uc6b0\ub294 \uc77c\uc5d0 \ub3d9\ucc38\ud588\ub2e4. \uad6d\ub0b4\uc678 \ud32c\ub4e4\uacfc \uc804 \uc138\uacc4 \ube48\uace4\uad6d\uac00 \uc544\uc774\ub4e4\uc758 \uad50\uc721\uc801, \uc815\uc11c\uc801 \uc9c0\uc6d0\uc744 \uc704\ud55c \u2018\ubcc4\ube5b\ucc9c\uc0ac\ub9c8\uc744\u2019 \ub9cc\ub4e4\uae30 \ud504\ub85c\uc81d\ud2b8\ub97c \uc9c4\ud589\ud574\uc654\ub2e4. 1\ud638 \uc13c\ud130\ub85c \uc11c\ubd80 \uc544\ud504\ub9ac\uce74 \uac00\ub098 \u2018\uc2e0\ud61c\uc13c\ud130(Shin Hye Center)\u2019\uac00 2013\ub144 11\uc6d4 \uc644\uacf5\ub418\uc5b4 5\ubc31\uc5ec \uba85\uc758 \ud559\uc0dd\ub4e4\uc774 \uc774\uc6a9\ud558\uace0 \uc788\ub2e4. 2014\ub144\uc5d0\ub294 \uc138\uc6d4\ud638 \ud76c\uc0dd\uc790 \uac00\uc871\uc744 \uc704\ud574 \ud76c\ub9dd\ube0c\ub9ac\uc9c0 \uc804\uad6d\uc7ac\ud574\uad6c\ud638\ud611\ud68c\ub97c \ud1b5\ud574 5,000\ub9cc\uc6d0\uc744 \uae30\ubd80\ud588\ub2e4. \ub610\ud55c \uc790\uc2e0\uc758 SNS \ud2b8\uc704\ud130 \ud504\ub85c\ud544 \uc0ac\uc9c4\uc744 \ub178\ub780\ub9ac\ubcf8 \uc0ac\uc9c4\uc73c\ub85c \ubcc0\uacbd, \ub178\ub780\ub9ac\ubcf8 \ucea0\ud398\uc778\uc5d0\ub3c4 \ub3d9\ucc38\ud588\ub2e4. 2015\ub144, \ubc15\uc2e0\ud61c\ub294 \u20182015 Dream of Angel\u2019 \ud0c0\uc774\ud2c0\ub85c \uc544\uc2dc\uc544\ud22c\uc5b4\uc5d0 \ub098\uc130\uace0, \uc0c1\ud574 \ud32c\ubbf8\ud305 \uc218\uc775\uc744 NGO \ub2e8\uccb4 \u2019\uae30\uc544\ub300\ucc45\u2019\uc758 \ubcc4\ube5b\ucc9c\uc0ac\ub9c8\uc744 \ub9cc\ub4e4\uae30\uc640 \uc911\uad6d\uc758 \uc720\uae30\ub3d9\ubb3c \ubcf4\ud638\ub2e8\uccb4 \u2018\ud0c0\uc18c\uc6d0\u2019\uc5d0 \uae30\ubd80\ud588\ub2e4. 4\uc6d4, \ubc15\uc2e0\ud61c\ub294 \ud070 \ud53c\ud574\ub97c \uc785\uc740 \ub124\ud314 \uad6d\ubbfc\ub4e4\uc744 \uc704\ud574 \uad6d\uc81c\uad6c\ud638\ub2e8\uccb4 \uae30\uc544\ub300\ucc45\ub97c \ud1b5\ud574 3,000\ub9cc\uc6d0\uc744 \uae30\ubd80\ud588\ub2e4. 2016\ub144 7\uc6d4, \ubc15\uc2e0\ud61c\ub294 \uc544\ud504\ub9ac\uce74 \uac00\ub098\uc758 \uc2e0\ud61c\uc13c\ud130\uc5d0 \uc774\uc5b4 \ud544\ub9ac\ud540 \ub9c8\ub2d0\ub77c\uc5d0 \uc2e0\ud61c\uc13c\ud130\uc758 \uac74\ub9bd\uc744 \uc9c4\ud589\ud588\ub2e4. \uc774\uc5d0 \ubc15\uc2e0\ud61c\uc640 \uc18c\uc18d\uc0ac \uc194\ud2b8 \uc5d4\ud130\ud14c\uc778\uba3c\ud2b8 \uc9c1\uc6d0\ub4e4\uc740 \uc13c\ud130\uac74\ub9bd\uc5d0 \uc55e\uc11c \ud544\ub9ac\ud540 \uc308\ube68\ub85d \uc9c0\uc5ed\uc758 \uc544\uc774\ub4e4\uacfc 1:1 \uacb0\uc5f0\uc744 \ub9fa\uc5c8\uace0, \uc9c0\ub09c 3\uc6d4 \ubc15\uc2e0\ud61c\ub294 \ud544\ub9ac\ud540\uc758 \uacb0\uc5f0\uc544\ub3d9\uc758 \uc9d1\uc744 \uc9c1\uc811 \ubc29\ubb38\ud558\uace0 \uc544\ub3d9\uacb0\uc5f0\uc13c\ud130 \uc544\uc774\ub4e4\uacfc \ud568\uaed8 \uc990\uac70\uc6b4 \uc2dc\uac04\uc744 \uac00\uc9c0\uba70 \uc13c\ud130 \uac74\ub9bd\uc5d0 \uc55e\uc11c \uac74\ucd95\uae30\ub150\uc2dd\uc744 \uac00\uc84c\ub2e4."} {"question": "\ud559\uad50 \uc5ed\uc0ac\uc0c1 \ucc98\uc74c\uc73c\ub85c \uc804\ubbf8 1\uc704\uc5d0 \ub7ad\ud06c\ub418\uc5c8\ub358 \ud3f4\uc758 \ub300\ud559\uad50 \uc774\ub984\uc740?", "paragraph": "\ud3f4\uc758 \ub300\ud559\uad50 2\ud559\ub144 \uc2dc\uc98c\uc774 \uc2dc\uc791\ub418\uae30 2\uc8fc \uc804, \uc6e8\uc774\ud06c \ud3ec\ub808\uc2a4\ud2b8\ub294 \ud559\uad50 \uc5ed\uc0ac\uc0c1 \ucc98\uc74c\uc73c\ub85c \uc804\ubbf8 1\uc704\uc5d0 \ub7ad\ud06c\ub418\uc5c8\ub2e4. \uadf8 \ud574 \ub9c8\uc9c0\ub9c9 \uac8c\uc784\uc5d0\uc11c \ud3f4\uc740 \uc0c1\ub300\ud300\uc774\uc5c8\ub358 \uc904\ub9ac\uc5b4\uc2a4 \ud558\uc9c0\uc758 \uae09\uc18c\uc5d0 \uc8fc\uba39\uc744 \ub0a0\ub824 ACC \ud1a0\ub108\uba3c\ud2b8 \ud55c \uac8c\uc784 \ucd9c\uc7a5 \uc815\uc9c0\ub97c \ubc1b\uc558\ub2e4. \uc774 \uc0ac\uac74\uc740 \ub2f9\uc2dc \ud3f4\uc758 \ud3c9\ud310\uc5d0 \uc77c\uc2dc\uc801\uc73c\ub85c \ub098\uc05c \uc601\ud5a5\uc744 \uc8fc\uc5c8\ub2e4. \ub370\ubaac \ub514\ucee8\uc2a4\ub294 \ub610 \ub2e4\uc2dc NCAA \ud1a0\ub108\uba3c\ud2b8\uc5d0 \ucc38\uc5ec\ud558\uc600\uc9c0\ub9cc, \ub450\ubc88\uc9f8 \ub77c\uc6b4\ub4dc\uc5d0\uc11c \uc6e8\uc2a4\ud2b8 \ubc84\uc9c0\ub2c8\uc544\uc5d0\uac8c \ud328\ubc30\ud558\uc5ec \ud0c8\ub77d\ud558\uc600\ub2e4. \ud3c9\uade0 15.3\ub4dd\uc810 4.5\ub9ac\ubc14\uc6b4\ub4dc 6.6\uc5b4\uc2dc\uc2a4\ud2b8 2.4\uc2a4\ud2f8\uc758 \uae30\ub85d\uc73c\ub85c \ub9c8\ubb34\ub9ac\ud55c \ud3f4\uc740 \ub9c8\uce68\ub0b4 NCAA \uc62c-\uc544\uba54\ub9ac\uce78 \ud37c\uc2a4\ud2b8 \ud300\uc5d0 \ubf51\ud788\uac8c \ub418\uba70, 3.21 GPA\uc758 \ud559\uc810\uc73c\ub85c ESPN\uc758 \uc544\uce74\ub370\ubbf9 \uc62c-\uc544\uba54\ub9ac\uce74 \ud300\uc5d0\ub3c4 \ubf51\ud614\ub2e4. 2005\ub144 4\uc6d4 15\uc77c, \uadf8\ub294 \uc5d0\uc774\uc804\ud2b8\ub97c \uace0\uc6a9\ud558\uc5ec \ud504\ub85c \ub18d\uad6c \uc120\uc218\uc758 \uae38\uc744 \uac77\uaca0\ub2e4\uace0 \uc120\uc5b8\ud588\ub2e4. 2011\ub144 3\uc6d4 2\uc77c, \uc6e8\uc774\ud06c \ud3ec\ub808\uc2a4\ud2b8 \ub300\ud559\uc740 \uadf8\uc758 \ub4f1\ubc88\ud638\ub97c \uc601\uad6c \uacb0\ubc88\ud558\uae30\ub85c \uacb0\uc815\ud558\uc600\ub2e4.", "answer": "\uc6e8\uc774\ud06c \ud3ec\ub808\uc2a4\ud2b8", "sentence": "\ud3f4\uc758 \ub300\ud559\uad50 2\ud559\ub144 \uc2dc\uc98c\uc774 \uc2dc\uc791\ub418\uae30 2\uc8fc \uc804, \uc6e8\uc774\ud06c \ud3ec\ub808\uc2a4\ud2b8 \ub294 \ud559\uad50 \uc5ed\uc0ac\uc0c1 \ucc98\uc74c\uc73c\ub85c \uc804\ubbf8 1\uc704\uc5d0 \ub7ad\ud06c\ub418\uc5c8\ub2e4.", "paragraph_sentence": " \ud3f4\uc758 \ub300\ud559\uad50 2\ud559\ub144 \uc2dc\uc98c\uc774 \uc2dc\uc791\ub418\uae30 2\uc8fc \uc804, \uc6e8\uc774\ud06c \ud3ec\ub808\uc2a4\ud2b8 \ub294 \ud559\uad50 \uc5ed\uc0ac\uc0c1 \ucc98\uc74c\uc73c\ub85c \uc804\ubbf8 1\uc704\uc5d0 \ub7ad\ud06c\ub418\uc5c8\ub2e4. \uadf8 \ud574 \ub9c8\uc9c0\ub9c9 \uac8c\uc784\uc5d0\uc11c \ud3f4\uc740 \uc0c1\ub300\ud300\uc774\uc5c8\ub358 \uc904\ub9ac\uc5b4\uc2a4 \ud558\uc9c0\uc758 \uae09\uc18c\uc5d0 \uc8fc\uba39\uc744 \ub0a0\ub824 ACC \ud1a0\ub108\uba3c\ud2b8 \ud55c \uac8c\uc784 \ucd9c\uc7a5 \uc815\uc9c0\ub97c \ubc1b\uc558\ub2e4. \uc774 \uc0ac\uac74\uc740 \ub2f9\uc2dc \ud3f4\uc758 \ud3c9\ud310\uc5d0 \uc77c\uc2dc\uc801\uc73c\ub85c \ub098\uc05c \uc601\ud5a5\uc744 \uc8fc\uc5c8\ub2e4. \ub370\ubaac \ub514\ucee8\uc2a4\ub294 \ub610 \ub2e4\uc2dc NCAA \ud1a0\ub108\uba3c\ud2b8\uc5d0 \ucc38\uc5ec\ud558\uc600\uc9c0\ub9cc, \ub450\ubc88\uc9f8 \ub77c\uc6b4\ub4dc\uc5d0\uc11c \uc6e8\uc2a4\ud2b8 \ubc84\uc9c0\ub2c8\uc544\uc5d0\uac8c \ud328\ubc30\ud558\uc5ec \ud0c8\ub77d\ud558\uc600\ub2e4. \ud3c9\uade0 15.3\ub4dd\uc810 4.5\ub9ac\ubc14\uc6b4\ub4dc 6.6\uc5b4\uc2dc\uc2a4\ud2b8 2.4\uc2a4\ud2f8\uc758 \uae30\ub85d\uc73c\ub85c \ub9c8\ubb34\ub9ac\ud55c \ud3f4\uc740 \ub9c8\uce68\ub0b4 NCAA \uc62c-\uc544\uba54\ub9ac\uce78 \ud37c\uc2a4\ud2b8 \ud300\uc5d0 \ubf51\ud788\uac8c \ub418\uba70, 3.21 GPA\uc758 \ud559\uc810\uc73c\ub85c ESPN\uc758 \uc544\uce74\ub370\ubbf9 \uc62c-\uc544\uba54\ub9ac\uce74 \ud300\uc5d0\ub3c4 \ubf51\ud614\ub2e4. 2005\ub144 4\uc6d4 15\uc77c, \uadf8\ub294 \uc5d0\uc774\uc804\ud2b8\ub97c \uace0\uc6a9\ud558\uc5ec \ud504\ub85c \ub18d\uad6c \uc120\uc218\uc758 \uae38\uc744 \uac77\uaca0\ub2e4\uace0 \uc120\uc5b8\ud588\ub2e4. 2011\ub144 3\uc6d4 2\uc77c, \uc6e8\uc774\ud06c \ud3ec\ub808\uc2a4\ud2b8 \ub300\ud559\uc740 \uadf8\uc758 \ub4f1\ubc88\ud638\ub97c \uc601\uad6c \uacb0\ubc88\ud558\uae30\ub85c \uacb0\uc815\ud558\uc600\ub2e4.", "paragraph_answer": "\ud3f4\uc758 \ub300\ud559\uad50 2\ud559\ub144 \uc2dc\uc98c\uc774 \uc2dc\uc791\ub418\uae30 2\uc8fc \uc804, \uc6e8\uc774\ud06c \ud3ec\ub808\uc2a4\ud2b8 \ub294 \ud559\uad50 \uc5ed\uc0ac\uc0c1 \ucc98\uc74c\uc73c\ub85c \uc804\ubbf8 1\uc704\uc5d0 \ub7ad\ud06c\ub418\uc5c8\ub2e4. \uadf8 \ud574 \ub9c8\uc9c0\ub9c9 \uac8c\uc784\uc5d0\uc11c \ud3f4\uc740 \uc0c1\ub300\ud300\uc774\uc5c8\ub358 \uc904\ub9ac\uc5b4\uc2a4 \ud558\uc9c0\uc758 \uae09\uc18c\uc5d0 \uc8fc\uba39\uc744 \ub0a0\ub824 ACC \ud1a0\ub108\uba3c\ud2b8 \ud55c \uac8c\uc784 \ucd9c\uc7a5 \uc815\uc9c0\ub97c \ubc1b\uc558\ub2e4. \uc774 \uc0ac\uac74\uc740 \ub2f9\uc2dc \ud3f4\uc758 \ud3c9\ud310\uc5d0 \uc77c\uc2dc\uc801\uc73c\ub85c \ub098\uc05c \uc601\ud5a5\uc744 \uc8fc\uc5c8\ub2e4. \ub370\ubaac \ub514\ucee8\uc2a4\ub294 \ub610 \ub2e4\uc2dc NCAA \ud1a0\ub108\uba3c\ud2b8\uc5d0 \ucc38\uc5ec\ud558\uc600\uc9c0\ub9cc, \ub450\ubc88\uc9f8 \ub77c\uc6b4\ub4dc\uc5d0\uc11c \uc6e8\uc2a4\ud2b8 \ubc84\uc9c0\ub2c8\uc544\uc5d0\uac8c \ud328\ubc30\ud558\uc5ec \ud0c8\ub77d\ud558\uc600\ub2e4. \ud3c9\uade0 15.3\ub4dd\uc810 4.5\ub9ac\ubc14\uc6b4\ub4dc 6.6\uc5b4\uc2dc\uc2a4\ud2b8 2.4\uc2a4\ud2f8\uc758 \uae30\ub85d\uc73c\ub85c \ub9c8\ubb34\ub9ac\ud55c \ud3f4\uc740 \ub9c8\uce68\ub0b4 NCAA \uc62c-\uc544\uba54\ub9ac\uce78 \ud37c\uc2a4\ud2b8 \ud300\uc5d0 \ubf51\ud788\uac8c \ub418\uba70, 3.21 GPA\uc758 \ud559\uc810\uc73c\ub85c ESPN\uc758 \uc544\uce74\ub370\ubbf9 \uc62c-\uc544\uba54\ub9ac\uce74 \ud300\uc5d0\ub3c4 \ubf51\ud614\ub2e4. 2005\ub144 4\uc6d4 15\uc77c, \uadf8\ub294 \uc5d0\uc774\uc804\ud2b8\ub97c \uace0\uc6a9\ud558\uc5ec \ud504\ub85c \ub18d\uad6c \uc120\uc218\uc758 \uae38\uc744 \uac77\uaca0\ub2e4\uace0 \uc120\uc5b8\ud588\ub2e4. 2011\ub144 3\uc6d4 2\uc77c, \uc6e8\uc774\ud06c \ud3ec\ub808\uc2a4\ud2b8 \ub300\ud559\uc740 \uadf8\uc758 \ub4f1\ubc88\ud638\ub97c \uc601\uad6c \uacb0\ubc88\ud558\uae30\ub85c \uacb0\uc815\ud558\uc600\ub2e4.", "sentence_answer": "\ud3f4\uc758 \ub300\ud559\uad50 2\ud559\ub144 \uc2dc\uc98c\uc774 \uc2dc\uc791\ub418\uae30 2\uc8fc \uc804, \uc6e8\uc774\ud06c \ud3ec\ub808\uc2a4\ud2b8 \ub294 \ud559\uad50 \uc5ed\uc0ac\uc0c1 \ucc98\uc74c\uc73c\ub85c \uc804\ubbf8 1\uc704\uc5d0 \ub7ad\ud06c\ub418\uc5c8\ub2e4.", "paragraph_id": "6212000-1-0", "paragraph_question": "question: \ud559\uad50 \uc5ed\uc0ac\uc0c1 \ucc98\uc74c\uc73c\ub85c \uc804\ubbf8 1\uc704\uc5d0 \ub7ad\ud06c\ub418\uc5c8\ub358 \ud3f4\uc758 \ub300\ud559\uad50 \uc774\ub984\uc740?, context: \ud3f4\uc758 \ub300\ud559\uad50 2\ud559\ub144 \uc2dc\uc98c\uc774 \uc2dc\uc791\ub418\uae30 2\uc8fc \uc804, \uc6e8\uc774\ud06c \ud3ec\ub808\uc2a4\ud2b8\ub294 \ud559\uad50 \uc5ed\uc0ac\uc0c1 \ucc98\uc74c\uc73c\ub85c \uc804\ubbf8 1\uc704\uc5d0 \ub7ad\ud06c\ub418\uc5c8\ub2e4. \uadf8 \ud574 \ub9c8\uc9c0\ub9c9 \uac8c\uc784\uc5d0\uc11c \ud3f4\uc740 \uc0c1\ub300\ud300\uc774\uc5c8\ub358 \uc904\ub9ac\uc5b4\uc2a4 \ud558\uc9c0\uc758 \uae09\uc18c\uc5d0 \uc8fc\uba39\uc744 \ub0a0\ub824 ACC \ud1a0\ub108\uba3c\ud2b8 \ud55c \uac8c\uc784 \ucd9c\uc7a5 \uc815\uc9c0\ub97c \ubc1b\uc558\ub2e4. \uc774 \uc0ac\uac74\uc740 \ub2f9\uc2dc \ud3f4\uc758 \ud3c9\ud310\uc5d0 \uc77c\uc2dc\uc801\uc73c\ub85c \ub098\uc05c \uc601\ud5a5\uc744 \uc8fc\uc5c8\ub2e4. \ub370\ubaac \ub514\ucee8\uc2a4\ub294 \ub610 \ub2e4\uc2dc NCAA \ud1a0\ub108\uba3c\ud2b8\uc5d0 \ucc38\uc5ec\ud558\uc600\uc9c0\ub9cc, \ub450\ubc88\uc9f8 \ub77c\uc6b4\ub4dc\uc5d0\uc11c \uc6e8\uc2a4\ud2b8 \ubc84\uc9c0\ub2c8\uc544\uc5d0\uac8c \ud328\ubc30\ud558\uc5ec \ud0c8\ub77d\ud558\uc600\ub2e4. \ud3c9\uade0 15.3\ub4dd\uc810 4.5\ub9ac\ubc14\uc6b4\ub4dc 6.6\uc5b4\uc2dc\uc2a4\ud2b8 2.4\uc2a4\ud2f8\uc758 \uae30\ub85d\uc73c\ub85c \ub9c8\ubb34\ub9ac\ud55c \ud3f4\uc740 \ub9c8\uce68\ub0b4 NCAA \uc62c-\uc544\uba54\ub9ac\uce78 \ud37c\uc2a4\ud2b8 \ud300\uc5d0 \ubf51\ud788\uac8c \ub418\uba70, 3.21 GPA\uc758 \ud559\uc810\uc73c\ub85c ESPN\uc758 \uc544\uce74\ub370\ubbf9 \uc62c-\uc544\uba54\ub9ac\uce74 \ud300\uc5d0\ub3c4 \ubf51\ud614\ub2e4. 2005\ub144 4\uc6d4 15\uc77c, \uadf8\ub294 \uc5d0\uc774\uc804\ud2b8\ub97c \uace0\uc6a9\ud558\uc5ec \ud504\ub85c \ub18d\uad6c \uc120\uc218\uc758 \uae38\uc744 \uac77\uaca0\ub2e4\uace0 \uc120\uc5b8\ud588\ub2e4. 2011\ub144 3\uc6d4 2\uc77c, \uc6e8\uc774\ud06c \ud3ec\ub808\uc2a4\ud2b8 \ub300\ud559\uc740 \uadf8\uc758 \ub4f1\ubc88\ud638\ub97c \uc601\uad6c \uacb0\ubc88\ud558\uae30\ub85c \uacb0\uc815\ud558\uc600\ub2e4."} {"question": "\uc81c2\ucc28 \ub2e8\uc2dd\ud22c\uc7c1\uc5d0\uc11c \uc8fd\uc5b4\uac04 10\uc778\uc744 \uae30\ub9ac\ub294 \ub178\ub798\uc758 \uc81c\ubaa9\uc740 \ubb34\uc5c7\uc778\uac00?", "paragraph": "\uc624\uc2a4\ud2b8\ub808\uc77c\ub9ac\uc544 \uc2dc\ub4dc\ub2c8\uc758 \uc6e8\uc774\ubc8c\ub9ac \uacf5\ub3d9\ubb18\uc9c0\uc5d0 \uc5f0\ud569 \uc544\uc77c\ub79c\ub4dc\uc778\ud68c \ubc18\ub780, \ubd80\ud65c\uc808 \ubd09\uae30, \uadf8\ub9ac\uace0 \ub2e8\uc2dd\ud22c\uc7c1 \uc0ac\ub9dd\uc790\ub4e4\uc744 \uae30\ub9ac\ub294 \uae30\ub150\ubb3c\uc774 \uc788\ub2e4. \uc774 \ubb18\uc9c0\ub294 \uc5f0\ud569 \uc544\uc77c\ub79c\ub4dc\ud68c\uc758 \ub9c8\uc774\ud074 \ub4dc\uc774\uc6cc\uc758 \ub9e4\uc7a5\uc9c0\uc774\uae30\ub3c4 \ud558\ub2e4. 1997\ub144 \uc544\uc77c\ub79c\ub4dc\ubd81\ubd80\uc6d0\uc870\uc704\uc6d0\ud68c\uc758 \ud558\ud2b8\ud37c\ub4dc \uc9c0\ud68c\ub294 \ubc14\ube44 \uc0cc\uc988\uc640 \ub2e4\ub978 \ub2e8\uc2dd\ud22c\uc7c1 \ucc38\uc5ec\uc790\ub4e4\uc5d0\uac8c \ubc14\uce58\ub294 \uae30\ub150\ubb3c\uc744 \uc81c\uc791\ud588\ub2e4. \uc774 \uae30\ub150\ubb3c\uc740 \uad7f\uc708 \uacf5\uc6d0 \uadfc\ucc98 \uba54\uc774\ud50c \ub300\ub85c \ub05d\uc790\ub77d\uc758 \"\ubc14\ube44 \uc0cc\uc988 \uc11c\ud074\"\uc774\ub77c\uace0 \ubd88\ub9b0 \ub85c\ud0c0\ub9ac \uac00\uc6b4\ub370 \uc138\uc6cc\uc838 \uc788\ub2e4. 2001\ub144 3\uc6d4 20\uc77c \uc2e0\ud398\uc778 \ucd1d\uc7ac \ubbf8\uccbc \ub9e5\ub798\ud50c\ub9b0\uc740 \ubca8\ud30c\uc2a4\ud2b8\uc758 \uc720\ub85c\ud30c \ud638\ud154\uc5d0\uc11c \ub2e8\uc2dd\ud22c\uc7c1 \uae30\ub150\uc704\uc6d0\ud68c \uc804\uc2dc\ub97c \uc5f4\uc5c8\ub2e4. \uc5ec\uae30\uc5d0\ub294 \ubca8\ud30c\uc2a4\ud2b8 \uc9c0\uc5ed \uc608\uc220\uac00\ub4e4\uc758 \uc624\ub9ac\uc9c0\ub110 \uc791\ud488 \uc138 \uc810\ub3c4 \uc804\uc2dc\ub418\uc5c8\ub2e4. \uadf8\ub2e4\uc74c \ub2ec\uc5d0\ub294 \ub370\ub9ac\uc5d0\uc11c\ub3c4 \ub530\ub85c \uc804\uc2dc\ud68c\uac00 \uc5f4\ub838\ub2e4. \ub2e8\uc2dd\ud22c\uc7c1\uc744 \ub2e4\ub8ec \uc601\ud654\uac00 \uc138 \ud3b8 \ub9cc\ub4e4\uc5b4\uc838 \uc788\ub294\ub370, \ud5ec\ub80c \ubbf8\ub80c\uc774 \ucd9c\uc5f0\ud55c \u300a\uc5b4\ub5a4 \uc5b4\uba38\ub2c8\uc758 \uc544\ub4e4\u300b, \ub2e8\uc2dd\ud22c\uc7c1 \ucc38\uc5ec\uc790 \ub85c\ub80c\uc2a4 \ub9e5\ucf00\uc628\uc774 \uac01\ubcf8\uc5d0 \ucc38\uc5ec\ud55c \u300aH3\u300b, \uc2a4\ud2f0\ube0c \ub9e4\ud038\uc758 \u300a\ud5dd\uac70\u300b\uc774\ub2e4. \uadf8\ub9ac\uace0 \uc81c2\ucc28 \ub2e8\uc2dd\ud22c\uc7c1\uc5d0\uc11c \uc0ac\ub9dd\ud55c 10\uc778\uc744 \uae30\ub9ac\ub294 \ub178\ub798\uc778 \u3008Roll of Honour\u3009\uac00 \uc791\uace1\ub418\uc5c8\ub2e4.", "answer": "Roll of Honour", "sentence": "\uadf8\ub9ac\uace0 \uc81c2\ucc28 \ub2e8\uc2dd\ud22c\uc7c1\uc5d0\uc11c \uc0ac\ub9dd\ud55c 10\uc778\uc744 \uae30\ub9ac\ub294 \ub178\ub798\uc778 \u3008 Roll of Honour \u3009\uac00 \uc791\uace1\ub418\uc5c8\ub2e4.", "paragraph_sentence": "\uc624\uc2a4\ud2b8\ub808\uc77c\ub9ac\uc544 \uc2dc\ub4dc\ub2c8\uc758 \uc6e8\uc774\ubc8c\ub9ac \uacf5\ub3d9\ubb18\uc9c0\uc5d0 \uc5f0\ud569 \uc544\uc77c\ub79c\ub4dc\uc778\ud68c \ubc18\ub780, \ubd80\ud65c\uc808 \ubd09\uae30, \uadf8\ub9ac\uace0 \ub2e8\uc2dd\ud22c\uc7c1 \uc0ac\ub9dd\uc790\ub4e4\uc744 \uae30\ub9ac\ub294 \uae30\ub150\ubb3c\uc774 \uc788\ub2e4. \uc774 \ubb18\uc9c0\ub294 \uc5f0\ud569 \uc544\uc77c\ub79c\ub4dc\ud68c\uc758 \ub9c8\uc774\ud074 \ub4dc\uc774\uc6cc\uc758 \ub9e4\uc7a5\uc9c0\uc774\uae30\ub3c4 \ud558\ub2e4. 1997\ub144 \uc544\uc77c\ub79c\ub4dc\ubd81\ubd80\uc6d0\uc870\uc704\uc6d0\ud68c\uc758 \ud558\ud2b8\ud37c\ub4dc \uc9c0\ud68c\ub294 \ubc14\ube44 \uc0cc\uc988\uc640 \ub2e4\ub978 \ub2e8\uc2dd\ud22c\uc7c1 \ucc38\uc5ec\uc790\ub4e4\uc5d0\uac8c \ubc14\uce58\ub294 \uae30\ub150\ubb3c\uc744 \uc81c\uc791\ud588\ub2e4. \uc774 \uae30\ub150\ubb3c\uc740 \uad7f\uc708 \uacf5\uc6d0 \uadfc\ucc98 \uba54\uc774\ud50c \ub300\ub85c \ub05d\uc790\ub77d\uc758 \"\ubc14\ube44 \uc0cc\uc988 \uc11c\ud074\"\uc774\ub77c\uace0 \ubd88\ub9b0 \ub85c\ud0c0\ub9ac \uac00\uc6b4\ub370 \uc138\uc6cc\uc838 \uc788\ub2e4. 2001\ub144 3\uc6d4 20\uc77c \uc2e0\ud398\uc778 \ucd1d\uc7ac \ubbf8\uccbc \ub9e5\ub798\ud50c\ub9b0\uc740 \ubca8\ud30c\uc2a4\ud2b8\uc758 \uc720\ub85c\ud30c \ud638\ud154\uc5d0\uc11c \ub2e8\uc2dd\ud22c\uc7c1 \uae30\ub150\uc704\uc6d0\ud68c \uc804\uc2dc\ub97c \uc5f4\uc5c8\ub2e4. \uc5ec\uae30\uc5d0\ub294 \ubca8\ud30c\uc2a4\ud2b8 \uc9c0\uc5ed \uc608\uc220\uac00\ub4e4\uc758 \uc624\ub9ac\uc9c0\ub110 \uc791\ud488 \uc138 \uc810\ub3c4 \uc804\uc2dc\ub418\uc5c8\ub2e4. \uadf8\ub2e4\uc74c \ub2ec\uc5d0\ub294 \ub370\ub9ac\uc5d0\uc11c\ub3c4 \ub530\ub85c \uc804\uc2dc\ud68c\uac00 \uc5f4\ub838\ub2e4. \ub2e8\uc2dd\ud22c\uc7c1\uc744 \ub2e4\ub8ec \uc601\ud654\uac00 \uc138 \ud3b8 \ub9cc\ub4e4\uc5b4\uc838 \uc788\ub294\ub370, \ud5ec\ub80c \ubbf8\ub80c\uc774 \ucd9c\uc5f0\ud55c \u300a\uc5b4\ub5a4 \uc5b4\uba38\ub2c8\uc758 \uc544\ub4e4\u300b, \ub2e8\uc2dd\ud22c\uc7c1 \ucc38\uc5ec\uc790 \ub85c\ub80c\uc2a4 \ub9e5\ucf00\uc628\uc774 \uac01\ubcf8\uc5d0 \ucc38\uc5ec\ud55c \u300aH3\u300b, \uc2a4\ud2f0\ube0c \ub9e4\ud038\uc758 \u300a\ud5dd\uac70\u300b\uc774\ub2e4. \uadf8\ub9ac\uace0 \uc81c2\ucc28 \ub2e8\uc2dd\ud22c\uc7c1\uc5d0\uc11c \uc0ac\ub9dd\ud55c 10\uc778\uc744 \uae30\ub9ac\ub294 \ub178\ub798\uc778 \u3008 Roll of Honour \u3009\uac00 \uc791\uace1\ub418\uc5c8\ub2e4. ", "paragraph_answer": "\uc624\uc2a4\ud2b8\ub808\uc77c\ub9ac\uc544 \uc2dc\ub4dc\ub2c8\uc758 \uc6e8\uc774\ubc8c\ub9ac \uacf5\ub3d9\ubb18\uc9c0\uc5d0 \uc5f0\ud569 \uc544\uc77c\ub79c\ub4dc\uc778\ud68c \ubc18\ub780, \ubd80\ud65c\uc808 \ubd09\uae30, \uadf8\ub9ac\uace0 \ub2e8\uc2dd\ud22c\uc7c1 \uc0ac\ub9dd\uc790\ub4e4\uc744 \uae30\ub9ac\ub294 \uae30\ub150\ubb3c\uc774 \uc788\ub2e4. \uc774 \ubb18\uc9c0\ub294 \uc5f0\ud569 \uc544\uc77c\ub79c\ub4dc\ud68c\uc758 \ub9c8\uc774\ud074 \ub4dc\uc774\uc6cc\uc758 \ub9e4\uc7a5\uc9c0\uc774\uae30\ub3c4 \ud558\ub2e4. 1997\ub144 \uc544\uc77c\ub79c\ub4dc\ubd81\ubd80\uc6d0\uc870\uc704\uc6d0\ud68c\uc758 \ud558\ud2b8\ud37c\ub4dc \uc9c0\ud68c\ub294 \ubc14\ube44 \uc0cc\uc988\uc640 \ub2e4\ub978 \ub2e8\uc2dd\ud22c\uc7c1 \ucc38\uc5ec\uc790\ub4e4\uc5d0\uac8c \ubc14\uce58\ub294 \uae30\ub150\ubb3c\uc744 \uc81c\uc791\ud588\ub2e4. \uc774 \uae30\ub150\ubb3c\uc740 \uad7f\uc708 \uacf5\uc6d0 \uadfc\ucc98 \uba54\uc774\ud50c \ub300\ub85c \ub05d\uc790\ub77d\uc758 \"\ubc14\ube44 \uc0cc\uc988 \uc11c\ud074\"\uc774\ub77c\uace0 \ubd88\ub9b0 \ub85c\ud0c0\ub9ac \uac00\uc6b4\ub370 \uc138\uc6cc\uc838 \uc788\ub2e4. 2001\ub144 3\uc6d4 20\uc77c \uc2e0\ud398\uc778 \ucd1d\uc7ac \ubbf8\uccbc \ub9e5\ub798\ud50c\ub9b0\uc740 \ubca8\ud30c\uc2a4\ud2b8\uc758 \uc720\ub85c\ud30c \ud638\ud154\uc5d0\uc11c \ub2e8\uc2dd\ud22c\uc7c1 \uae30\ub150\uc704\uc6d0\ud68c \uc804\uc2dc\ub97c \uc5f4\uc5c8\ub2e4. \uc5ec\uae30\uc5d0\ub294 \ubca8\ud30c\uc2a4\ud2b8 \uc9c0\uc5ed \uc608\uc220\uac00\ub4e4\uc758 \uc624\ub9ac\uc9c0\ub110 \uc791\ud488 \uc138 \uc810\ub3c4 \uc804\uc2dc\ub418\uc5c8\ub2e4. \uadf8\ub2e4\uc74c \ub2ec\uc5d0\ub294 \ub370\ub9ac\uc5d0\uc11c\ub3c4 \ub530\ub85c \uc804\uc2dc\ud68c\uac00 \uc5f4\ub838\ub2e4. \ub2e8\uc2dd\ud22c\uc7c1\uc744 \ub2e4\ub8ec \uc601\ud654\uac00 \uc138 \ud3b8 \ub9cc\ub4e4\uc5b4\uc838 \uc788\ub294\ub370, \ud5ec\ub80c \ubbf8\ub80c\uc774 \ucd9c\uc5f0\ud55c \u300a\uc5b4\ub5a4 \uc5b4\uba38\ub2c8\uc758 \uc544\ub4e4\u300b, \ub2e8\uc2dd\ud22c\uc7c1 \ucc38\uc5ec\uc790 \ub85c\ub80c\uc2a4 \ub9e5\ucf00\uc628\uc774 \uac01\ubcf8\uc5d0 \ucc38\uc5ec\ud55c \u300aH3\u300b, \uc2a4\ud2f0\ube0c \ub9e4\ud038\uc758 \u300a\ud5dd\uac70\u300b\uc774\ub2e4. \uadf8\ub9ac\uace0 \uc81c2\ucc28 \ub2e8\uc2dd\ud22c\uc7c1\uc5d0\uc11c \uc0ac\ub9dd\ud55c 10\uc778\uc744 \uae30\ub9ac\ub294 \ub178\ub798\uc778 \u3008 Roll of Honour \u3009\uac00 \uc791\uace1\ub418\uc5c8\ub2e4.", "sentence_answer": "\uadf8\ub9ac\uace0 \uc81c2\ucc28 \ub2e8\uc2dd\ud22c\uc7c1\uc5d0\uc11c \uc0ac\ub9dd\ud55c 10\uc778\uc744 \uae30\ub9ac\ub294 \ub178\ub798\uc778 \u3008 Roll of Honour \u3009\uac00 \uc791\uace1\ub418\uc5c8\ub2e4.", "paragraph_id": "6521117-14-2", "paragraph_question": "question: \uc81c2\ucc28 \ub2e8\uc2dd\ud22c\uc7c1\uc5d0\uc11c \uc8fd\uc5b4\uac04 10\uc778\uc744 \uae30\ub9ac\ub294 \ub178\ub798\uc758 \uc81c\ubaa9\uc740 \ubb34\uc5c7\uc778\uac00?, context: \uc624\uc2a4\ud2b8\ub808\uc77c\ub9ac\uc544 \uc2dc\ub4dc\ub2c8\uc758 \uc6e8\uc774\ubc8c\ub9ac \uacf5\ub3d9\ubb18\uc9c0\uc5d0 \uc5f0\ud569 \uc544\uc77c\ub79c\ub4dc\uc778\ud68c \ubc18\ub780, \ubd80\ud65c\uc808 \ubd09\uae30, \uadf8\ub9ac\uace0 \ub2e8\uc2dd\ud22c\uc7c1 \uc0ac\ub9dd\uc790\ub4e4\uc744 \uae30\ub9ac\ub294 \uae30\ub150\ubb3c\uc774 \uc788\ub2e4. \uc774 \ubb18\uc9c0\ub294 \uc5f0\ud569 \uc544\uc77c\ub79c\ub4dc\ud68c\uc758 \ub9c8\uc774\ud074 \ub4dc\uc774\uc6cc\uc758 \ub9e4\uc7a5\uc9c0\uc774\uae30\ub3c4 \ud558\ub2e4. 1997\ub144 \uc544\uc77c\ub79c\ub4dc\ubd81\ubd80\uc6d0\uc870\uc704\uc6d0\ud68c\uc758 \ud558\ud2b8\ud37c\ub4dc \uc9c0\ud68c\ub294 \ubc14\ube44 \uc0cc\uc988\uc640 \ub2e4\ub978 \ub2e8\uc2dd\ud22c\uc7c1 \ucc38\uc5ec\uc790\ub4e4\uc5d0\uac8c \ubc14\uce58\ub294 \uae30\ub150\ubb3c\uc744 \uc81c\uc791\ud588\ub2e4. \uc774 \uae30\ub150\ubb3c\uc740 \uad7f\uc708 \uacf5\uc6d0 \uadfc\ucc98 \uba54\uc774\ud50c \ub300\ub85c \ub05d\uc790\ub77d\uc758 \"\ubc14\ube44 \uc0cc\uc988 \uc11c\ud074\"\uc774\ub77c\uace0 \ubd88\ub9b0 \ub85c\ud0c0\ub9ac \uac00\uc6b4\ub370 \uc138\uc6cc\uc838 \uc788\ub2e4. 2001\ub144 3\uc6d4 20\uc77c \uc2e0\ud398\uc778 \ucd1d\uc7ac \ubbf8\uccbc \ub9e5\ub798\ud50c\ub9b0\uc740 \ubca8\ud30c\uc2a4\ud2b8\uc758 \uc720\ub85c\ud30c \ud638\ud154\uc5d0\uc11c \ub2e8\uc2dd\ud22c\uc7c1 \uae30\ub150\uc704\uc6d0\ud68c \uc804\uc2dc\ub97c \uc5f4\uc5c8\ub2e4. \uc5ec\uae30\uc5d0\ub294 \ubca8\ud30c\uc2a4\ud2b8 \uc9c0\uc5ed \uc608\uc220\uac00\ub4e4\uc758 \uc624\ub9ac\uc9c0\ub110 \uc791\ud488 \uc138 \uc810\ub3c4 \uc804\uc2dc\ub418\uc5c8\ub2e4. \uadf8\ub2e4\uc74c \ub2ec\uc5d0\ub294 \ub370\ub9ac\uc5d0\uc11c\ub3c4 \ub530\ub85c \uc804\uc2dc\ud68c\uac00 \uc5f4\ub838\ub2e4. \ub2e8\uc2dd\ud22c\uc7c1\uc744 \ub2e4\ub8ec \uc601\ud654\uac00 \uc138 \ud3b8 \ub9cc\ub4e4\uc5b4\uc838 \uc788\ub294\ub370, \ud5ec\ub80c \ubbf8\ub80c\uc774 \ucd9c\uc5f0\ud55c \u300a\uc5b4\ub5a4 \uc5b4\uba38\ub2c8\uc758 \uc544\ub4e4\u300b, \ub2e8\uc2dd\ud22c\uc7c1 \ucc38\uc5ec\uc790 \ub85c\ub80c\uc2a4 \ub9e5\ucf00\uc628\uc774 \uac01\ubcf8\uc5d0 \ucc38\uc5ec\ud55c \u300aH3\u300b, \uc2a4\ud2f0\ube0c \ub9e4\ud038\uc758 \u300a\ud5dd\uac70\u300b\uc774\ub2e4. \uadf8\ub9ac\uace0 \uc81c2\ucc28 \ub2e8\uc2dd\ud22c\uc7c1\uc5d0\uc11c \uc0ac\ub9dd\ud55c 10\uc778\uc744 \uae30\ub9ac\ub294 \ub178\ub798\uc778 \u3008Roll of Honour\u3009\uac00 \uc791\uace1\ub418\uc5c8\ub2e4."} {"question": "\uc790\ub3d9\ucc28\uc5d0 \uc4f0\uc774\ub294 \ube14\ub85d \ub2e4\uc774\uc5b4\uadf8\ub7a8\uc5d0\uc11c\ub294 \ubb34\uc5c7\uc774 \ube14\ub85d\uc774 \ub418\ub294\uac00?", "paragraph": "\ubcf4\ud1b5 \uc2dc\uc2a4\ud15c \uad6c\uc870 \ubc0f \ub3d9\uc791\uc744 \uc81c\uc5b4\ud558\uace0 \ubd84\uc11d\ud558\ub294\ub370 \ub9ce\uc774 \uc4f0\uc778\ub2e4. \uac01 \uc7a5\uce58, \uc608\ub97c \ub4e4\uba74 \uc790\ub3d9\ucc28\uc5d0\uc11c\ub294 \ubaa8\ud130 \ubc0f \uae30\uc5b4 \ub4f1\uc774 \ube14\ub85d\uc774 \ub418\uace0 \uc544\ud30c\ud2b8\uc5d0\uc11c\ub294 \uac01 \uce35\uacfc \uae30\ub465 \ubc0f \uad6c\uc870\ubb3c\ub4e4\uc774 \ube14\ub85d\uc774 \ub418\ub294 \uac83\uc774\ub2e4. \uc774\ub54c \uc7a5\uce58 \uc0ac\uc774\uc5d0 \uc801\uc6a9\ub418\ub294 \ubb3c\ub9ac\uc801\uc778 \uad00\uacc4\uc2dd\ub4e4\uc774 \ube14\ub85d\uc744 \uc787\ub294 \uc120\uc774 \ub41c\ub2e4. \uc774\ub7ec\ud55c \ube14\ub85d\ub4e4\uc740 \uacb0\uad6d \uc785\ub825\uc5d0\uc11c \ucd9c\ub825\uae4c\uc9c0 \uc774\uc5b4\uc9c0\uac8c \ub418\ub294\ub370 \uc774\ub97c \ud1b5\ud574 \uc2dc\uc2a4\ud15c\uc758 \uad6c\uc870\ub97c \uac04\ub2e8\ud558\uac8c \uc815\ub9ac\ud560 \uc218 \uc788\ub2e4. .\uc989 \ube14\ub85d \ub2e4\uc774\uc5b4\uadf8\ub7a8\uc744 \uc0ac\uc6a9\ud558\uba74 \ubcf5\uc7a1\ud55c \ubb3c\ub9ac\uc801\uc778 \uc778\uacfc\uad00\uacc4\ub4e4\uc774 \uc5b4\ub5bb\uac8c \uc804\ub2ec\ub418\ub294\uc9c0 \uc27d\uac8c \ud30c\uc545\ud560 \uc218 \uc788\ub2e4. \ub3d9\uc2dc\uc5d0 \uc774\ub97c \uc81c\uc5b4\ud558\ub824\uba74 \uc5b4\ub5a4 \uc694\uc18c\ub97c \uc870\uc815\ud574\uc57c \ud558\ub294\uc9c0\uae4c\uc9c0\ub3c4 \uc27d\uac8c \ucc3e\uc744 \uc218 \uc788\uc73c\ubbc0\ub85c \uac04\ub2e8\ud55c \uc219\uc81c\ubb38\uc81c\uc5d0\uc11c\ubd80\ud130 \uad8c\uc704\uc788\ub294 \uc5f0\uad6c \ub17c\ubb38\uae4c\uc9c0 \uad11\ubc94\uc704\ud558\uac8c \uc4f0\uc774\ub294 \uc2dc\uc2a4\ud15c \ud45c\ud604 \ubc29\ubc95 \uc911 \ud558\ub098\uc774\ub2e4.", "answer": "\ubaa8\ud130 \ubc0f \uae30\uc5b4 \ub4f1", "sentence": "\uc790\ub3d9\ucc28\uc5d0\uc11c\ub294 \ubaa8\ud130 \ubc0f \uae30\uc5b4 \ub4f1 \uc774 \ube14\ub85d\uc774 \ub418\uace0 \uc544\ud30c\ud2b8\uc5d0\uc11c\ub294 \uac01 \uce35\uacfc \uae30\ub465 \ubc0f \uad6c\uc870\ubb3c\ub4e4\uc774 \ube14\ub85d\uc774 \ub418\ub294 \uac83\uc774\ub2e4.", "paragraph_sentence": "\ubcf4\ud1b5 \uc2dc\uc2a4\ud15c \uad6c\uc870 \ubc0f \ub3d9\uc791\uc744 \uc81c\uc5b4\ud558\uace0 \ubd84\uc11d\ud558\ub294\ub370 \ub9ce\uc774 \uc4f0\uc778\ub2e4. \uac01 \uc7a5\uce58, \uc608\ub97c \ub4e4\uba74 \uc790\ub3d9\ucc28\uc5d0\uc11c\ub294 \ubaa8\ud130 \ubc0f \uae30\uc5b4 \ub4f1 \uc774 \ube14\ub85d\uc774 \ub418\uace0 \uc544\ud30c\ud2b8\uc5d0\uc11c\ub294 \uac01 \uce35\uacfc \uae30\ub465 \ubc0f \uad6c\uc870\ubb3c\ub4e4\uc774 \ube14\ub85d\uc774 \ub418\ub294 \uac83\uc774\ub2e4. \uc774\ub54c \uc7a5\uce58 \uc0ac\uc774\uc5d0 \uc801\uc6a9\ub418\ub294 \ubb3c\ub9ac\uc801\uc778 \uad00\uacc4\uc2dd\ub4e4\uc774 \ube14\ub85d\uc744 \uc787\ub294 \uc120\uc774 \ub41c\ub2e4. \uc774\ub7ec\ud55c \ube14\ub85d\ub4e4\uc740 \uacb0\uad6d \uc785\ub825\uc5d0\uc11c \ucd9c\ub825\uae4c\uc9c0 \uc774\uc5b4\uc9c0\uac8c \ub418\ub294\ub370 \uc774\ub97c \ud1b5\ud574 \uc2dc\uc2a4\ud15c\uc758 \uad6c\uc870\ub97c \uac04\ub2e8\ud558\uac8c \uc815\ub9ac\ud560 \uc218 \uc788\ub2e4. .\uc989 \ube14\ub85d \ub2e4\uc774\uc5b4\uadf8\ub7a8\uc744 \uc0ac\uc6a9\ud558\uba74 \ubcf5\uc7a1\ud55c \ubb3c\ub9ac\uc801\uc778 \uc778\uacfc\uad00\uacc4\ub4e4\uc774 \uc5b4\ub5bb\uac8c \uc804\ub2ec\ub418\ub294\uc9c0 \uc27d\uac8c \ud30c\uc545\ud560 \uc218 \uc788\ub2e4. \ub3d9\uc2dc\uc5d0 \uc774\ub97c \uc81c\uc5b4\ud558\ub824\uba74 \uc5b4\ub5a4 \uc694\uc18c\ub97c \uc870\uc815\ud574\uc57c \ud558\ub294\uc9c0\uae4c\uc9c0\ub3c4 \uc27d\uac8c \ucc3e\uc744 \uc218 \uc788\uc73c\ubbc0\ub85c \uac04\ub2e8\ud55c \uc219\uc81c\ubb38\uc81c\uc5d0\uc11c\ubd80\ud130 \uad8c\uc704\uc788\ub294 \uc5f0\uad6c \ub17c\ubb38\uae4c\uc9c0 \uad11\ubc94\uc704\ud558\uac8c \uc4f0\uc774\ub294 \uc2dc\uc2a4\ud15c \ud45c\ud604 \ubc29\ubc95 \uc911 \ud558\ub098\uc774\ub2e4.", "paragraph_answer": "\ubcf4\ud1b5 \uc2dc\uc2a4\ud15c \uad6c\uc870 \ubc0f \ub3d9\uc791\uc744 \uc81c\uc5b4\ud558\uace0 \ubd84\uc11d\ud558\ub294\ub370 \ub9ce\uc774 \uc4f0\uc778\ub2e4. \uac01 \uc7a5\uce58, \uc608\ub97c \ub4e4\uba74 \uc790\ub3d9\ucc28\uc5d0\uc11c\ub294 \ubaa8\ud130 \ubc0f \uae30\uc5b4 \ub4f1 \uc774 \ube14\ub85d\uc774 \ub418\uace0 \uc544\ud30c\ud2b8\uc5d0\uc11c\ub294 \uac01 \uce35\uacfc \uae30\ub465 \ubc0f \uad6c\uc870\ubb3c\ub4e4\uc774 \ube14\ub85d\uc774 \ub418\ub294 \uac83\uc774\ub2e4. \uc774\ub54c \uc7a5\uce58 \uc0ac\uc774\uc5d0 \uc801\uc6a9\ub418\ub294 \ubb3c\ub9ac\uc801\uc778 \uad00\uacc4\uc2dd\ub4e4\uc774 \ube14\ub85d\uc744 \uc787\ub294 \uc120\uc774 \ub41c\ub2e4. \uc774\ub7ec\ud55c \ube14\ub85d\ub4e4\uc740 \uacb0\uad6d \uc785\ub825\uc5d0\uc11c \ucd9c\ub825\uae4c\uc9c0 \uc774\uc5b4\uc9c0\uac8c \ub418\ub294\ub370 \uc774\ub97c \ud1b5\ud574 \uc2dc\uc2a4\ud15c\uc758 \uad6c\uc870\ub97c \uac04\ub2e8\ud558\uac8c \uc815\ub9ac\ud560 \uc218 \uc788\ub2e4. .\uc989 \ube14\ub85d \ub2e4\uc774\uc5b4\uadf8\ub7a8\uc744 \uc0ac\uc6a9\ud558\uba74 \ubcf5\uc7a1\ud55c \ubb3c\ub9ac\uc801\uc778 \uc778\uacfc\uad00\uacc4\ub4e4\uc774 \uc5b4\ub5bb\uac8c \uc804\ub2ec\ub418\ub294\uc9c0 \uc27d\uac8c \ud30c\uc545\ud560 \uc218 \uc788\ub2e4. \ub3d9\uc2dc\uc5d0 \uc774\ub97c \uc81c\uc5b4\ud558\ub824\uba74 \uc5b4\ub5a4 \uc694\uc18c\ub97c \uc870\uc815\ud574\uc57c \ud558\ub294\uc9c0\uae4c\uc9c0\ub3c4 \uc27d\uac8c \ucc3e\uc744 \uc218 \uc788\uc73c\ubbc0\ub85c \uac04\ub2e8\ud55c \uc219\uc81c\ubb38\uc81c\uc5d0\uc11c\ubd80\ud130 \uad8c\uc704\uc788\ub294 \uc5f0\uad6c \ub17c\ubb38\uae4c\uc9c0 \uad11\ubc94\uc704\ud558\uac8c \uc4f0\uc774\ub294 \uc2dc\uc2a4\ud15c \ud45c\ud604 \ubc29\ubc95 \uc911 \ud558\ub098\uc774\ub2e4.", "sentence_answer": "\uc790\ub3d9\ucc28\uc5d0\uc11c\ub294 \ubaa8\ud130 \ubc0f \uae30\uc5b4 \ub4f1 \uc774 \ube14\ub85d\uc774 \ub418\uace0 \uc544\ud30c\ud2b8\uc5d0\uc11c\ub294 \uac01 \uce35\uacfc \uae30\ub465 \ubc0f \uad6c\uc870\ubb3c\ub4e4\uc774 \ube14\ub85d\uc774 \ub418\ub294 \uac83\uc774\ub2e4.", "paragraph_id": "6531044-1-0", "paragraph_question": "question: \uc790\ub3d9\ucc28\uc5d0 \uc4f0\uc774\ub294 \ube14\ub85d \ub2e4\uc774\uc5b4\uadf8\ub7a8\uc5d0\uc11c\ub294 \ubb34\uc5c7\uc774 \ube14\ub85d\uc774 \ub418\ub294\uac00?, context: \ubcf4\ud1b5 \uc2dc\uc2a4\ud15c \uad6c\uc870 \ubc0f \ub3d9\uc791\uc744 \uc81c\uc5b4\ud558\uace0 \ubd84\uc11d\ud558\ub294\ub370 \ub9ce\uc774 \uc4f0\uc778\ub2e4. \uac01 \uc7a5\uce58, \uc608\ub97c \ub4e4\uba74 \uc790\ub3d9\ucc28\uc5d0\uc11c\ub294 \ubaa8\ud130 \ubc0f \uae30\uc5b4 \ub4f1\uc774 \ube14\ub85d\uc774 \ub418\uace0 \uc544\ud30c\ud2b8\uc5d0\uc11c\ub294 \uac01 \uce35\uacfc \uae30\ub465 \ubc0f \uad6c\uc870\ubb3c\ub4e4\uc774 \ube14\ub85d\uc774 \ub418\ub294 \uac83\uc774\ub2e4. \uc774\ub54c \uc7a5\uce58 \uc0ac\uc774\uc5d0 \uc801\uc6a9\ub418\ub294 \ubb3c\ub9ac\uc801\uc778 \uad00\uacc4\uc2dd\ub4e4\uc774 \ube14\ub85d\uc744 \uc787\ub294 \uc120\uc774 \ub41c\ub2e4. \uc774\ub7ec\ud55c \ube14\ub85d\ub4e4\uc740 \uacb0\uad6d \uc785\ub825\uc5d0\uc11c \ucd9c\ub825\uae4c\uc9c0 \uc774\uc5b4\uc9c0\uac8c \ub418\ub294\ub370 \uc774\ub97c \ud1b5\ud574 \uc2dc\uc2a4\ud15c\uc758 \uad6c\uc870\ub97c \uac04\ub2e8\ud558\uac8c \uc815\ub9ac\ud560 \uc218 \uc788\ub2e4. .\uc989 \ube14\ub85d \ub2e4\uc774\uc5b4\uadf8\ub7a8\uc744 \uc0ac\uc6a9\ud558\uba74 \ubcf5\uc7a1\ud55c \ubb3c\ub9ac\uc801\uc778 \uc778\uacfc\uad00\uacc4\ub4e4\uc774 \uc5b4\ub5bb\uac8c \uc804\ub2ec\ub418\ub294\uc9c0 \uc27d\uac8c \ud30c\uc545\ud560 \uc218 \uc788\ub2e4. \ub3d9\uc2dc\uc5d0 \uc774\ub97c \uc81c\uc5b4\ud558\ub824\uba74 \uc5b4\ub5a4 \uc694\uc18c\ub97c \uc870\uc815\ud574\uc57c \ud558\ub294\uc9c0\uae4c\uc9c0\ub3c4 \uc27d\uac8c \ucc3e\uc744 \uc218 \uc788\uc73c\ubbc0\ub85c \uac04\ub2e8\ud55c \uc219\uc81c\ubb38\uc81c\uc5d0\uc11c\ubd80\ud130 \uad8c\uc704\uc788\ub294 \uc5f0\uad6c \ub17c\ubb38\uae4c\uc9c0 \uad11\ubc94\uc704\ud558\uac8c \uc4f0\uc774\ub294 \uc2dc\uc2a4\ud15c \ud45c\ud604 \ubc29\ubc95 \uc911 \ud558\ub098\uc774\ub2e4."} {"question": "\ub178\ubb34\ud604 \ubcc0\ud638\uc0ac\ub294 \uba87\ub300 \ucd1d\uc120\uc5d0\uc11c \uad6d\ud68c\uc5d0 \uc785\ubb38\ud558\uc600\ub294\uac00?", "paragraph": "\uc601\ud654\ub294 1988\ub144 \ub178\ubb34\ud604 \ubcc0\ud638\uc0ac\uac00 \uadf8\ud574 \uc81c13\ub300 \ucd1d\uc120\uc5d0\uc11c \ub2f9\uc120\ub418\uc5b4 \uad6d\ud68c\uc5d0 \uc785\ubb38\ud588\ub358 \ubaa8\uc2b5\ubd80\ud130 \uc2dc\uc791\ub41c\ub2e4. \uc774\ud6c4\ub85c\ub294 \ub178\ub798\ub97c \ucc3e\ub294 \uc0ac\ub78c\ub4e4\uc758 \u300a\uc0ac\uacc4\u300b\uac00 \ud750\ub974\ub294 \uac00\uc6b4\ub370, \ud0c0\uc784\ub77c\uc778\uc774 \ube68\ub77c\uc9c0\uba74\uc11c \ub178\ubb34\ud604\uc758 \uc815\uce58\uacbd\ub825\uc744 \ud6d1\uace0 \uc9c0\ub098\uac04\ub2e4. \ub178\ubb34\ud604 \uc758\uc6d0\uc740 1990\ub144 3\ub2f9 \ud569\ub2f9\uc744 \uacaa\uace0 \ub2e4\uc74c \ucd1d\uc120\uc778 1992\ub144 \uc81c14\ub300 \ucd1d\uc120\uc5d0\uc11c \ub099\uc120\ud55c\ub2e4. \uc774\ud6c4 1995\ub144 \uc81c1\ud68c \uc9c0\ubc29\uc120\uac70\uc5d0\uc11c \ubd80\uc0b0 \uc2dc\uc7a5 \uc120\uac70\uc5d0 \ub098\uc11c\ub098 \uc774 \uc5ed\uc2dc \ub099\uc120\ud558\uace0 \uc774\ub4ec\ud574 1996\ub144 \uc81c15\ub300 \ucd1d\uc120\uc5d0\uc11c \uc885\ub85c\uad6c\uc5d0 \ucd9c\ub9c8\ud558\ub098 \ub099\uc120\ud55c\ub2e4. \uadf8\ub7ec\ub098 1998\ub144 \uc7ac\ubcf4\uad90 \uc120\uac70\uc5d0\uc11c \uc885\ub85c\uad6c \uc120\uac70\uc5d0 \ub2e4\uc2dc \ucd9c\ub9c8\ud558\uc5ec \ub2f9\uc120\ub418\uace0, \"\ub2e4\uc2dc \uc77c\ud558\uac8c \ub420 \uc218 \uc788\uc5b4 \uae30\uc058\ub2e4\"\ub294 \uadf8\uc758 \ub2f9\uc120\uc18c\uac10\uacfc \ud568\uaed8 \ud0c0\uc784\ub77c\uc778\uc774 \uba48\ucd94\uba70 \uc601\ud654 \uc81c\ubaa9\uc774 \uc2a4\ud06c\ub9b0\uc5d0 \ub5a0\uc624\ub978\ub2e4.", "answer": "\uc81c13\ub300 \ucd1d\uc120", "sentence": "\uadf8\ud574 \uc81c13\ub300 \ucd1d\uc120 \uc5d0\uc11c \ub2f9\uc120\ub418\uc5b4 \uad6d\ud68c\uc5d0 \uc785\ubb38\ud588\ub358 \ubaa8\uc2b5\ubd80\ud130 \uc2dc\uc791\ub41c\ub2e4.", "paragraph_sentence": "\uc601\ud654\ub294 1988\ub144 \ub178\ubb34\ud604 \ubcc0\ud638\uc0ac\uac00 \uadf8\ud574 \uc81c13\ub300 \ucd1d\uc120 \uc5d0\uc11c \ub2f9\uc120\ub418\uc5b4 \uad6d\ud68c\uc5d0 \uc785\ubb38\ud588\ub358 \ubaa8\uc2b5\ubd80\ud130 \uc2dc\uc791\ub41c\ub2e4. \uc774\ud6c4\ub85c\ub294 \ub178\ub798\ub97c \ucc3e\ub294 \uc0ac\ub78c\ub4e4\uc758 \u300a\uc0ac\uacc4\u300b\uac00 \ud750\ub974\ub294 \uac00\uc6b4\ub370, \ud0c0\uc784\ub77c\uc778\uc774 \ube68\ub77c\uc9c0\uba74\uc11c \ub178\ubb34\ud604\uc758 \uc815\uce58\uacbd\ub825\uc744 \ud6d1\uace0 \uc9c0\ub098\uac04\ub2e4. \ub178\ubb34\ud604 \uc758\uc6d0\uc740 1990\ub144 3\ub2f9 \ud569\ub2f9\uc744 \uacaa\uace0 \ub2e4\uc74c \ucd1d\uc120\uc778 1992\ub144 \uc81c14\ub300 \ucd1d\uc120\uc5d0\uc11c \ub099\uc120\ud55c\ub2e4. \uc774\ud6c4 1995\ub144 \uc81c1\ud68c \uc9c0\ubc29\uc120\uac70\uc5d0\uc11c \ubd80\uc0b0 \uc2dc\uc7a5 \uc120\uac70\uc5d0 \ub098\uc11c\ub098 \uc774 \uc5ed\uc2dc \ub099\uc120\ud558\uace0 \uc774\ub4ec\ud574 1996\ub144 \uc81c15\ub300 \ucd1d\uc120\uc5d0\uc11c \uc885\ub85c\uad6c\uc5d0 \ucd9c\ub9c8\ud558\ub098 \ub099\uc120\ud55c\ub2e4. \uadf8\ub7ec\ub098 1998\ub144 \uc7ac\ubcf4\uad90 \uc120\uac70\uc5d0\uc11c \uc885\ub85c\uad6c \uc120\uac70\uc5d0 \ub2e4\uc2dc \ucd9c\ub9c8\ud558\uc5ec \ub2f9\uc120\ub418\uace0, \"\ub2e4\uc2dc \uc77c\ud558\uac8c \ub420 \uc218 \uc788\uc5b4 \uae30\uc058\ub2e4\"\ub294 \uadf8\uc758 \ub2f9\uc120\uc18c\uac10\uacfc \ud568\uaed8 \ud0c0\uc784\ub77c\uc778\uc774 \uba48\ucd94\uba70 \uc601\ud654 \uc81c\ubaa9\uc774 \uc2a4\ud06c\ub9b0\uc5d0 \ub5a0\uc624\ub978\ub2e4.", "paragraph_answer": "\uc601\ud654\ub294 1988\ub144 \ub178\ubb34\ud604 \ubcc0\ud638\uc0ac\uac00 \uadf8\ud574 \uc81c13\ub300 \ucd1d\uc120 \uc5d0\uc11c \ub2f9\uc120\ub418\uc5b4 \uad6d\ud68c\uc5d0 \uc785\ubb38\ud588\ub358 \ubaa8\uc2b5\ubd80\ud130 \uc2dc\uc791\ub41c\ub2e4. \uc774\ud6c4\ub85c\ub294 \ub178\ub798\ub97c \ucc3e\ub294 \uc0ac\ub78c\ub4e4\uc758 \u300a\uc0ac\uacc4\u300b\uac00 \ud750\ub974\ub294 \uac00\uc6b4\ub370, \ud0c0\uc784\ub77c\uc778\uc774 \ube68\ub77c\uc9c0\uba74\uc11c \ub178\ubb34\ud604\uc758 \uc815\uce58\uacbd\ub825\uc744 \ud6d1\uace0 \uc9c0\ub098\uac04\ub2e4. \ub178\ubb34\ud604 \uc758\uc6d0\uc740 1990\ub144 3\ub2f9 \ud569\ub2f9\uc744 \uacaa\uace0 \ub2e4\uc74c \ucd1d\uc120\uc778 1992\ub144 \uc81c14\ub300 \ucd1d\uc120\uc5d0\uc11c \ub099\uc120\ud55c\ub2e4. \uc774\ud6c4 1995\ub144 \uc81c1\ud68c \uc9c0\ubc29\uc120\uac70\uc5d0\uc11c \ubd80\uc0b0 \uc2dc\uc7a5 \uc120\uac70\uc5d0 \ub098\uc11c\ub098 \uc774 \uc5ed\uc2dc \ub099\uc120\ud558\uace0 \uc774\ub4ec\ud574 1996\ub144 \uc81c15\ub300 \ucd1d\uc120\uc5d0\uc11c \uc885\ub85c\uad6c\uc5d0 \ucd9c\ub9c8\ud558\ub098 \ub099\uc120\ud55c\ub2e4. \uadf8\ub7ec\ub098 1998\ub144 \uc7ac\ubcf4\uad90 \uc120\uac70\uc5d0\uc11c \uc885\ub85c\uad6c \uc120\uac70\uc5d0 \ub2e4\uc2dc \ucd9c\ub9c8\ud558\uc5ec \ub2f9\uc120\ub418\uace0, \"\ub2e4\uc2dc \uc77c\ud558\uac8c \ub420 \uc218 \uc788\uc5b4 \uae30\uc058\ub2e4\"\ub294 \uadf8\uc758 \ub2f9\uc120\uc18c\uac10\uacfc \ud568\uaed8 \ud0c0\uc784\ub77c\uc778\uc774 \uba48\ucd94\uba70 \uc601\ud654 \uc81c\ubaa9\uc774 \uc2a4\ud06c\ub9b0\uc5d0 \ub5a0\uc624\ub978\ub2e4.", "sentence_answer": "\uadf8\ud574 \uc81c13\ub300 \ucd1d\uc120 \uc5d0\uc11c \ub2f9\uc120\ub418\uc5b4 \uad6d\ud68c\uc5d0 \uc785\ubb38\ud588\ub358 \ubaa8\uc2b5\ubd80\ud130 \uc2dc\uc791\ub41c\ub2e4.", "paragraph_id": "6484027-0-0", "paragraph_question": "question: \ub178\ubb34\ud604 \ubcc0\ud638\uc0ac\ub294 \uba87\ub300 \ucd1d\uc120\uc5d0\uc11c \uad6d\ud68c\uc5d0 \uc785\ubb38\ud558\uc600\ub294\uac00?, context: \uc601\ud654\ub294 1988\ub144 \ub178\ubb34\ud604 \ubcc0\ud638\uc0ac\uac00 \uadf8\ud574 \uc81c13\ub300 \ucd1d\uc120\uc5d0\uc11c \ub2f9\uc120\ub418\uc5b4 \uad6d\ud68c\uc5d0 \uc785\ubb38\ud588\ub358 \ubaa8\uc2b5\ubd80\ud130 \uc2dc\uc791\ub41c\ub2e4. \uc774\ud6c4\ub85c\ub294 \ub178\ub798\ub97c \ucc3e\ub294 \uc0ac\ub78c\ub4e4\uc758 \u300a\uc0ac\uacc4\u300b\uac00 \ud750\ub974\ub294 \uac00\uc6b4\ub370, \ud0c0\uc784\ub77c\uc778\uc774 \ube68\ub77c\uc9c0\uba74\uc11c \ub178\ubb34\ud604\uc758 \uc815\uce58\uacbd\ub825\uc744 \ud6d1\uace0 \uc9c0\ub098\uac04\ub2e4. \ub178\ubb34\ud604 \uc758\uc6d0\uc740 1990\ub144 3\ub2f9 \ud569\ub2f9\uc744 \uacaa\uace0 \ub2e4\uc74c \ucd1d\uc120\uc778 1992\ub144 \uc81c14\ub300 \ucd1d\uc120\uc5d0\uc11c \ub099\uc120\ud55c\ub2e4. \uc774\ud6c4 1995\ub144 \uc81c1\ud68c \uc9c0\ubc29\uc120\uac70\uc5d0\uc11c \ubd80\uc0b0 \uc2dc\uc7a5 \uc120\uac70\uc5d0 \ub098\uc11c\ub098 \uc774 \uc5ed\uc2dc \ub099\uc120\ud558\uace0 \uc774\ub4ec\ud574 1996\ub144 \uc81c15\ub300 \ucd1d\uc120\uc5d0\uc11c \uc885\ub85c\uad6c\uc5d0 \ucd9c\ub9c8\ud558\ub098 \ub099\uc120\ud55c\ub2e4. \uadf8\ub7ec\ub098 1998\ub144 \uc7ac\ubcf4\uad90 \uc120\uac70\uc5d0\uc11c \uc885\ub85c\uad6c \uc120\uac70\uc5d0 \ub2e4\uc2dc \ucd9c\ub9c8\ud558\uc5ec \ub2f9\uc120\ub418\uace0, \"\ub2e4\uc2dc \uc77c\ud558\uac8c \ub420 \uc218 \uc788\uc5b4 \uae30\uc058\ub2e4\"\ub294 \uadf8\uc758 \ub2f9\uc120\uc18c\uac10\uacfc \ud568\uaed8 \ud0c0\uc784\ub77c\uc778\uc774 \uba48\ucd94\uba70 \uc601\ud654 \uc81c\ubaa9\uc774 \uc2a4\ud06c\ub9b0\uc5d0 \ub5a0\uc624\ub978\ub2e4."} {"question": "\ubd81\ud55c\uacf5\uc791\uc6d0 \uc5ec\uac1d\uae30 \ud3ed\ud30c\uc0ac\uac74\uc774 \uc788\uae30\uc804\uc5d0 \uae40\uc601\uc0bc \ud6c4\ubcf4\ub294 \uba87%\ub300\uc758 \uc9c0\uc9c0\ub97c \ubc1b\uace0 \uc788\uc5c8\ub294\uac00?", "paragraph": "1987\ub144 11\uc6d4 29\uc77c \uc774\ub77c\ud06c \ubc14\uadf8\ub2e4\ub4dc\uc5d0\uc11c \ucd9c\ubc1c\ud55c KAL 858\uae30\uac00 \uc778\ub3c4\uc591 \uc0c1\uacf5\uc5d0\uc11c \ubd81\ud55c\uacf5\uc791\uc6d0\uc5d0 \uc758\ud558\uc5ec \uacf5\uc911 \ud3ed\ud30c\ub418\ub294 \uc0ac\uac74\uc774 \uc77c\uc5b4\ub0ac\ub2e4. \uc774 \uc0ac\uac74\uc740 1987\ub144 10\uc6d4 7\uc77c \uae40\uc815\uc77c\uc758 \uba85\ub839\uc5d0 \uc758\ud574 88 \uc11c\uc6b8\uc62c\ub9bc\ud53d\ub300\ud68c\ub97c \ubc29\ud574\ud558\uace0, \ub300\ud55c\ubbfc\uad6d \ub0b4 \ub300\uc815\ubd80 \ubd88\uc2e0\uc744 \uc870\uc7a5\uc744 \ubaa9\uc801\uc73c\ub85c \ud558\uc600\ub2e4. \uc5ec\uac1d\uae30 \ud3ed\ud30c\ud6c4 \ub450 \ud3ed\ud30c\ubc94\uc740 1987\ub144 12\uc6d4 1\uc77c \ubc14\ub808\uc778 \uacf5\ud56d\uc5d0\uc11c \uc870\uc0ac\ub97c \ubc1b\ub358 \uc911 \uae40\uc2b9\uc77c\uc740 \ub3c5\uc57d\uc744 \uba39\uace0 \uc790\uc0b4\ud558\uc600\uc73c\uba70, \uae40\ud604\ud76c\ub294 \ub300\ud55c\ubbfc\uad6d\uc73c\ub85c \uc555\uc1a1\ub418\uc5c8\ub2e4. \uc774 \uc0ac\uac74\uc740 \uc5ec\ub7ec\uac00\uc9c0\ub85c \ub17c\ub780\uc744 \ubd88\ub7ec\uc654\ub2e4. \ubfd0\ub9cc \uc544\ub2c8\ub77c \uace7 \uce58\ub7ec\uc9c8 \ub300\uc120\uc5d0\ub3c4 \uc601\ud5a5\uc744 \ubbf8\ucce4\ub294\ub370, \ud2b9\ud788 \ub2f9\uc2dc 40%\ub300\uc758 \uc9c0\uc9c0\uc728\uc744 \uae30\ub85d\ud558\ub358 \ud1b5\uc77c\ubbfc\uc8fc\ub2f9 \uae40\uc601\uc0bc \ud6c4\ubcf4\uc5d0\uac8c\ub294 \uce58\uba85\uc801\uc778 \uc544\ud0ac\ub808\uc2a4\uac74\uc774\ub098 \ub2e4\ub984 \uc5c6\uc5c8\uace0, \uacb0\uad6d \uae40\uc601\uc0bc\uc740 \ub300\uc120\uc5d0\uc11c \uc5ec\ub2f9\uc758 \ub178\ud0dc\uc6b0\uc5d0\uac8c \ud328\ud558\uc600\ub2e4.", "answer": "40%\ub300", "sentence": "\ud2b9\ud788 \ub2f9\uc2dc 40%\ub300 \uc758 \uc9c0\uc9c0\uc728\uc744 \uae30\ub85d\ud558\ub358 \ud1b5\uc77c\ubbfc\uc8fc\ub2f9 \uae40\uc601\uc0bc \ud6c4\ubcf4\uc5d0\uac8c\ub294 \uce58\uba85\uc801\uc778 \uc544\ud0ac\ub808\uc2a4\uac74\uc774\ub098 \ub2e4\ub984 \uc5c6\uc5c8\uace0, \uacb0\uad6d \uae40\uc601\uc0bc\uc740 \ub300\uc120\uc5d0\uc11c \uc5ec\ub2f9\uc758 \ub178\ud0dc\uc6b0\uc5d0\uac8c \ud328\ud558\uc600\ub2e4.", "paragraph_sentence": "1987\ub144 11\uc6d4 29\uc77c \uc774\ub77c\ud06c \ubc14\uadf8\ub2e4\ub4dc\uc5d0\uc11c \ucd9c\ubc1c\ud55c KAL 858\uae30\uac00 \uc778\ub3c4\uc591 \uc0c1\uacf5\uc5d0\uc11c \ubd81\ud55c\uacf5\uc791\uc6d0\uc5d0 \uc758\ud558\uc5ec \uacf5\uc911 \ud3ed\ud30c\ub418\ub294 \uc0ac\uac74\uc774 \uc77c\uc5b4\ub0ac\ub2e4. \uc774 \uc0ac\uac74\uc740 1987\ub144 10\uc6d4 7\uc77c \uae40\uc815\uc77c\uc758 \uba85\ub839\uc5d0 \uc758\ud574 88 \uc11c\uc6b8\uc62c\ub9bc\ud53d\ub300\ud68c\ub97c \ubc29\ud574\ud558\uace0, \ub300\ud55c\ubbfc\uad6d \ub0b4 \ub300\uc815\ubd80 \ubd88\uc2e0\uc744 \uc870\uc7a5\uc744 \ubaa9\uc801\uc73c\ub85c \ud558\uc600\ub2e4. \uc5ec\uac1d\uae30 \ud3ed\ud30c\ud6c4 \ub450 \ud3ed\ud30c\ubc94\uc740 1987\ub144 12\uc6d4 1\uc77c \ubc14\ub808\uc778 \uacf5\ud56d\uc5d0\uc11c \uc870\uc0ac\ub97c \ubc1b\ub358 \uc911 \uae40\uc2b9\uc77c\uc740 \ub3c5\uc57d\uc744 \uba39\uace0 \uc790\uc0b4\ud558\uc600\uc73c\uba70, \uae40\ud604\ud76c\ub294 \ub300\ud55c\ubbfc\uad6d\uc73c\ub85c \uc555\uc1a1\ub418\uc5c8\ub2e4. \uc774 \uc0ac\uac74\uc740 \uc5ec\ub7ec\uac00\uc9c0\ub85c \ub17c\ub780\uc744 \ubd88\ub7ec\uc654\ub2e4. \ubfd0\ub9cc \uc544\ub2c8\ub77c \uace7 \uce58\ub7ec\uc9c8 \ub300\uc120\uc5d0\ub3c4 \uc601\ud5a5\uc744 \ubbf8\ucce4\ub294\ub370, \ud2b9\ud788 \ub2f9\uc2dc 40%\ub300 \uc758 \uc9c0\uc9c0\uc728\uc744 \uae30\ub85d\ud558\ub358 \ud1b5\uc77c\ubbfc\uc8fc\ub2f9 \uae40\uc601\uc0bc \ud6c4\ubcf4\uc5d0\uac8c\ub294 \uce58\uba85\uc801\uc778 \uc544\ud0ac\ub808\uc2a4\uac74\uc774\ub098 \ub2e4\ub984 \uc5c6\uc5c8\uace0, \uacb0\uad6d \uae40\uc601\uc0bc\uc740 \ub300\uc120\uc5d0\uc11c \uc5ec\ub2f9\uc758 \ub178\ud0dc\uc6b0\uc5d0\uac8c \ud328\ud558\uc600\ub2e4. ", "paragraph_answer": "1987\ub144 11\uc6d4 29\uc77c \uc774\ub77c\ud06c \ubc14\uadf8\ub2e4\ub4dc\uc5d0\uc11c \ucd9c\ubc1c\ud55c KAL 858\uae30\uac00 \uc778\ub3c4\uc591 \uc0c1\uacf5\uc5d0\uc11c \ubd81\ud55c\uacf5\uc791\uc6d0\uc5d0 \uc758\ud558\uc5ec \uacf5\uc911 \ud3ed\ud30c\ub418\ub294 \uc0ac\uac74\uc774 \uc77c\uc5b4\ub0ac\ub2e4. \uc774 \uc0ac\uac74\uc740 1987\ub144 10\uc6d4 7\uc77c \uae40\uc815\uc77c\uc758 \uba85\ub839\uc5d0 \uc758\ud574 88 \uc11c\uc6b8\uc62c\ub9bc\ud53d\ub300\ud68c\ub97c \ubc29\ud574\ud558\uace0, \ub300\ud55c\ubbfc\uad6d \ub0b4 \ub300\uc815\ubd80 \ubd88\uc2e0\uc744 \uc870\uc7a5\uc744 \ubaa9\uc801\uc73c\ub85c \ud558\uc600\ub2e4. \uc5ec\uac1d\uae30 \ud3ed\ud30c\ud6c4 \ub450 \ud3ed\ud30c\ubc94\uc740 1987\ub144 12\uc6d4 1\uc77c \ubc14\ub808\uc778 \uacf5\ud56d\uc5d0\uc11c \uc870\uc0ac\ub97c \ubc1b\ub358 \uc911 \uae40\uc2b9\uc77c\uc740 \ub3c5\uc57d\uc744 \uba39\uace0 \uc790\uc0b4\ud558\uc600\uc73c\uba70, \uae40\ud604\ud76c\ub294 \ub300\ud55c\ubbfc\uad6d\uc73c\ub85c \uc555\uc1a1\ub418\uc5c8\ub2e4. \uc774 \uc0ac\uac74\uc740 \uc5ec\ub7ec\uac00\uc9c0\ub85c \ub17c\ub780\uc744 \ubd88\ub7ec\uc654\ub2e4. \ubfd0\ub9cc \uc544\ub2c8\ub77c \uace7 \uce58\ub7ec\uc9c8 \ub300\uc120\uc5d0\ub3c4 \uc601\ud5a5\uc744 \ubbf8\ucce4\ub294\ub370, \ud2b9\ud788 \ub2f9\uc2dc 40%\ub300 \uc758 \uc9c0\uc9c0\uc728\uc744 \uae30\ub85d\ud558\ub358 \ud1b5\uc77c\ubbfc\uc8fc\ub2f9 \uae40\uc601\uc0bc \ud6c4\ubcf4\uc5d0\uac8c\ub294 \uce58\uba85\uc801\uc778 \uc544\ud0ac\ub808\uc2a4\uac74\uc774\ub098 \ub2e4\ub984 \uc5c6\uc5c8\uace0, \uacb0\uad6d \uae40\uc601\uc0bc\uc740 \ub300\uc120\uc5d0\uc11c \uc5ec\ub2f9\uc758 \ub178\ud0dc\uc6b0\uc5d0\uac8c \ud328\ud558\uc600\ub2e4.", "sentence_answer": "\ud2b9\ud788 \ub2f9\uc2dc 40%\ub300 \uc758 \uc9c0\uc9c0\uc728\uc744 \uae30\ub85d\ud558\ub358 \ud1b5\uc77c\ubbfc\uc8fc\ub2f9 \uae40\uc601\uc0bc \ud6c4\ubcf4\uc5d0\uac8c\ub294 \uce58\uba85\uc801\uc778 \uc544\ud0ac\ub808\uc2a4\uac74\uc774\ub098 \ub2e4\ub984 \uc5c6\uc5c8\uace0, \uacb0\uad6d \uae40\uc601\uc0bc\uc740 \ub300\uc120\uc5d0\uc11c \uc5ec\ub2f9\uc758 \ub178\ud0dc\uc6b0\uc5d0\uac8c \ud328\ud558\uc600\ub2e4.", "paragraph_id": "6657628-27-2", "paragraph_question": "question: \ubd81\ud55c\uacf5\uc791\uc6d0 \uc5ec\uac1d\uae30 \ud3ed\ud30c\uc0ac\uac74\uc774 \uc788\uae30\uc804\uc5d0 \uae40\uc601\uc0bc \ud6c4\ubcf4\ub294 \uba87%\ub300\uc758 \uc9c0\uc9c0\ub97c \ubc1b\uace0 \uc788\uc5c8\ub294\uac00?, context: 1987\ub144 11\uc6d4 29\uc77c \uc774\ub77c\ud06c \ubc14\uadf8\ub2e4\ub4dc\uc5d0\uc11c \ucd9c\ubc1c\ud55c KAL 858\uae30\uac00 \uc778\ub3c4\uc591 \uc0c1\uacf5\uc5d0\uc11c \ubd81\ud55c\uacf5\uc791\uc6d0\uc5d0 \uc758\ud558\uc5ec \uacf5\uc911 \ud3ed\ud30c\ub418\ub294 \uc0ac\uac74\uc774 \uc77c\uc5b4\ub0ac\ub2e4. \uc774 \uc0ac\uac74\uc740 1987\ub144 10\uc6d4 7\uc77c \uae40\uc815\uc77c\uc758 \uba85\ub839\uc5d0 \uc758\ud574 88 \uc11c\uc6b8\uc62c\ub9bc\ud53d\ub300\ud68c\ub97c \ubc29\ud574\ud558\uace0, \ub300\ud55c\ubbfc\uad6d \ub0b4 \ub300\uc815\ubd80 \ubd88\uc2e0\uc744 \uc870\uc7a5\uc744 \ubaa9\uc801\uc73c\ub85c \ud558\uc600\ub2e4. \uc5ec\uac1d\uae30 \ud3ed\ud30c\ud6c4 \ub450 \ud3ed\ud30c\ubc94\uc740 1987\ub144 12\uc6d4 1\uc77c \ubc14\ub808\uc778 \uacf5\ud56d\uc5d0\uc11c \uc870\uc0ac\ub97c \ubc1b\ub358 \uc911 \uae40\uc2b9\uc77c\uc740 \ub3c5\uc57d\uc744 \uba39\uace0 \uc790\uc0b4\ud558\uc600\uc73c\uba70, \uae40\ud604\ud76c\ub294 \ub300\ud55c\ubbfc\uad6d\uc73c\ub85c \uc555\uc1a1\ub418\uc5c8\ub2e4. \uc774 \uc0ac\uac74\uc740 \uc5ec\ub7ec\uac00\uc9c0\ub85c \ub17c\ub780\uc744 \ubd88\ub7ec\uc654\ub2e4. \ubfd0\ub9cc \uc544\ub2c8\ub77c \uace7 \uce58\ub7ec\uc9c8 \ub300\uc120\uc5d0\ub3c4 \uc601\ud5a5\uc744 \ubbf8\ucce4\ub294\ub370, \ud2b9\ud788 \ub2f9\uc2dc 40%\ub300\uc758 \uc9c0\uc9c0\uc728\uc744 \uae30\ub85d\ud558\ub358 \ud1b5\uc77c\ubbfc\uc8fc\ub2f9 \uae40\uc601\uc0bc \ud6c4\ubcf4\uc5d0\uac8c\ub294 \uce58\uba85\uc801\uc778 \uc544\ud0ac\ub808\uc2a4\uac74\uc774\ub098 \ub2e4\ub984 \uc5c6\uc5c8\uace0, \uacb0\uad6d \uae40\uc601\uc0bc\uc740 \ub300\uc120\uc5d0\uc11c \uc5ec\ub2f9\uc758 \ub178\ud0dc\uc6b0\uc5d0\uac8c \ud328\ud558\uc600\ub2e4."} {"question": "\uc2dc\ud2f0\uc988 \uc2a4\uce74\uc774\ub77c\uc778\uc758 2014\ub144 \ub2f9\uc2dc \uc9c1\uc6d0 \uc218\ub294?", "paragraph": "\uac1c\ubc1c\uc0ac\uc778 \ucf5c\ub85c\uc124 \uc624\ub354\ub294 2009\ub144\uc5d4 5\uba85\ubc16\uc5d0 \uc548\ub418\uc5c8\uc73c\uba70, \ud68c\uc0ac \ucc3d\ub9bd\ubd80\ud130 \ub3c4\uc2dc \uc2dc\ubbac\ub808\uc774\uc158 \uac1c\ubc1c\uc744 \uc0dd\uac01\ud558\uace0 \uc788\uc5c8\uc9c0\ub9cc, 5\uba85\ubc16\uc5d0 \uc548\ub418\ub294 \uc9c0\ub77c '\uc774\uac8c \uacfc\uc5f0 \ub420\uae4c?' \ub780 \uc0dd\uac01\uc744 \uac16\uace0 \uc788\uc5c8\ub2e4. \ub610\ud55c \uadf8 \ub2f9\uc2dc \ucf5c\ub85c\uc124 \uc624\ub354\uc758 \uac1c\ubc1c\uc790\ub4e4\uc740 \ubaa8\ubc14\uc77c \uac8c\uc784\uc744 \uac1c\ubc1c\ud558\ub2e4 \ub098\uc654\uae30 \ub54c\ubb38\uc5d0 PC \ud50c\ub7ab\ud3fc\uc5d0 \ub300\ud574 \uc804\ud600 \ubaa8\ub974\ub358 \uc0c1\ud0dc\uc600\ub2e4. \uadf8\ub85c \uc778\ud574\uc11c \ucc98\uc74c \uc544\uc774\ub514\uc5b4\ub294 \uc9c4\uc9dc \ud074\ub798\uc2dd\ud55c \ub3c4\uc2dc \uac74\uc124 \uac8c\uc784\uc774\uc5c8\uc73c\ub098, \ud68c\uc0ac\uc5d0\uc11c \uafc8\uafb8\ub358 \ub3c4\uc2dc \uac74\uc124 \uc2dc\ubbac\ub808\uc774\uc158\uc744 \uac1c\ubc1c\ud558\uae30\uc5d4 \ubaa8\ubc14\uc77c \ud658\uacbd\uc774 \ub108\ubb34\ub098 \ub5a8\uc5b4\uc9c0\uc790 PC\ucabd\uc73c\ub85c \uc62e\uaca8\uac00\uac8c \ub418\uc5c8\ub2e4. \ucf5c\ub85c\uc124 \uc624\ub354 \uc9c1\uc6d0\uc740 2014\ub144 \ub2f9\uc2dc 9\uba85\uc774\uc5c8\uc73c\uba70, 2015\ub144 13\uba85\uc73c\ub85c \uc778\uc6d0\uc744 \ub298\ub838\ub2e4. \uadf8\ub9ac\uace0 \ucf5c\ub85c\uc124 \uc624\ub354\ub294 \uc2dc\ud2f0\uc988 \uc778 \ubaa8\uc158 \uc2dc\ub9ac\uc988\ub97c \uac1c\ubc1c\ud558\uba74\uc11c, \ub3c4\uc2dc \uac74\uc124 \uc2dc\ubbac\ub808\uc774\uc158\uc5d0 \ub300\ud55c \uc9c0\uc2dd\ub4e4\uc744 \uc313\uc544\uac14\ub2e4. \uc608\ub97c \ub4e4\uc5b4\uc11c \ud070 \uc9c0\ub3c4\uc640 \uac74\uc124 \ub3c4\uad6c\ub294 \uc2dc\ud2f0\uc988 \uc778 \ubaa8\uc158 2\uc5d0\uc11c \ube44\ub86f\ub41c \uac83\uc774\ub2e4. \ucf5c\ub85c\uc124 \uc624\ub354 \uce21\uc740 \uc2ec\uc2dc\ud2f0\ub97c \ubd24\uc744\ub54c \uc88c\uc808\ud588\uc9c0\ub9cc, \uc2ec\uc2dc\ud2f0\uac00 \uc704\uae30\ub97c \uacaa\uc73c\uba74\uc11c \ud76c\ub9dd\uc774 \uc788\ub2e4\uace0 \ud310\ub2e8\ud558\uc5ec \uac1c\ubc1c\uc744 \uc9c4\ud589\ud558\uc600\ub2e4.\uadf8\ub9ac\uace0 \ucf5c\ub85c\uc124 \uc624\ub354\ub294 \ud2b9\uc815 \uacc4\uce35\ub9cc \uc990\uae30\ub358 \uc2dc\ud2f0\uc988 \uc778 \ubaa8\uc158\uacfc \ub2e4\ub974\uac8c, \uc2dc\ud2f0\uc988: \uc2a4\uce74\uc774\ub77c\uc778\uc744 \ub300\uc911\ub4e4\uc774 \ub354 \ub9ce\uc774 \uc811\ud560 \uc218 \uc788\uac8c \ub178\ub825\ud558\uc600\ub2e4.", "answer": "9\uba85", "sentence": "\ucf5c\ub85c\uc124 \uc624\ub354 \uc9c1\uc6d0\uc740 2014\ub144 \ub2f9\uc2dc 9\uba85 \uc774\uc5c8\uc73c\uba70, 2015\ub144 13\uba85\uc73c\ub85c \uc778\uc6d0\uc744 \ub298\ub838\ub2e4.", "paragraph_sentence": "\uac1c\ubc1c\uc0ac\uc778 \ucf5c\ub85c\uc124 \uc624\ub354\ub294 2009\ub144\uc5d4 5\uba85\ubc16\uc5d0 \uc548\ub418\uc5c8\uc73c\uba70, \ud68c\uc0ac \ucc3d\ub9bd\ubd80\ud130 \ub3c4\uc2dc \uc2dc\ubbac\ub808\uc774\uc158 \uac1c\ubc1c\uc744 \uc0dd\uac01\ud558\uace0 \uc788\uc5c8\uc9c0\ub9cc, 5\uba85\ubc16\uc5d0 \uc548\ub418\ub294 \uc9c0\ub77c '\uc774\uac8c \uacfc\uc5f0 \ub420\uae4c?' \ub780 \uc0dd\uac01\uc744 \uac16\uace0 \uc788\uc5c8\ub2e4. \ub610\ud55c \uadf8 \ub2f9\uc2dc \ucf5c\ub85c\uc124 \uc624\ub354\uc758 \uac1c\ubc1c\uc790\ub4e4\uc740 \ubaa8\ubc14\uc77c \uac8c\uc784\uc744 \uac1c\ubc1c\ud558\ub2e4 \ub098\uc654\uae30 \ub54c\ubb38\uc5d0 PC \ud50c\ub7ab\ud3fc\uc5d0 \ub300\ud574 \uc804\ud600 \ubaa8\ub974\ub358 \uc0c1\ud0dc\uc600\ub2e4. \uadf8\ub85c \uc778\ud574\uc11c \ucc98\uc74c \uc544\uc774\ub514\uc5b4\ub294 \uc9c4\uc9dc \ud074\ub798\uc2dd\ud55c \ub3c4\uc2dc \uac74\uc124 \uac8c\uc784\uc774\uc5c8\uc73c\ub098, \ud68c\uc0ac\uc5d0\uc11c \uafc8\uafb8\ub358 \ub3c4\uc2dc \uac74\uc124 \uc2dc\ubbac\ub808\uc774\uc158\uc744 \uac1c\ubc1c\ud558\uae30\uc5d4 \ubaa8\ubc14\uc77c \ud658\uacbd\uc774 \ub108\ubb34\ub098 \ub5a8\uc5b4\uc9c0\uc790 PC\ucabd\uc73c\ub85c \uc62e\uaca8\uac00\uac8c \ub418\uc5c8\ub2e4. \ucf5c\ub85c\uc124 \uc624\ub354 \uc9c1\uc6d0\uc740 2014\ub144 \ub2f9\uc2dc 9\uba85 \uc774\uc5c8\uc73c\uba70, 2015\ub144 13\uba85\uc73c\ub85c \uc778\uc6d0\uc744 \ub298\ub838\ub2e4. \uadf8\ub9ac\uace0 \ucf5c\ub85c\uc124 \uc624\ub354\ub294 \uc2dc\ud2f0\uc988 \uc778 \ubaa8\uc158 \uc2dc\ub9ac\uc988\ub97c \uac1c\ubc1c\ud558\uba74\uc11c, \ub3c4\uc2dc \uac74\uc124 \uc2dc\ubbac\ub808\uc774\uc158\uc5d0 \ub300\ud55c \uc9c0\uc2dd\ub4e4\uc744 \uc313\uc544\uac14\ub2e4. \uc608\ub97c \ub4e4\uc5b4\uc11c \ud070 \uc9c0\ub3c4\uc640 \uac74\uc124 \ub3c4\uad6c\ub294 \uc2dc\ud2f0\uc988 \uc778 \ubaa8\uc158 2\uc5d0\uc11c \ube44\ub86f\ub41c \uac83\uc774\ub2e4. \ucf5c\ub85c\uc124 \uc624\ub354 \uce21\uc740 \uc2ec\uc2dc\ud2f0\ub97c \ubd24\uc744\ub54c \uc88c\uc808\ud588\uc9c0\ub9cc, \uc2ec\uc2dc\ud2f0\uac00 \uc704\uae30\ub97c \uacaa\uc73c\uba74\uc11c \ud76c\ub9dd\uc774 \uc788\ub2e4\uace0 \ud310\ub2e8\ud558\uc5ec \uac1c\ubc1c\uc744 \uc9c4\ud589\ud558\uc600\ub2e4.\uadf8\ub9ac\uace0 \ucf5c\ub85c\uc124 \uc624\ub354\ub294 \ud2b9\uc815 \uacc4\uce35\ub9cc \uc990\uae30\ub358 \uc2dc\ud2f0\uc988 \uc778 \ubaa8\uc158\uacfc \ub2e4\ub974\uac8c, \uc2dc\ud2f0\uc988: \uc2a4\uce74\uc774\ub77c\uc778\uc744 \ub300\uc911\ub4e4\uc774 \ub354 \ub9ce\uc774 \uc811\ud560 \uc218 \uc788\uac8c \ub178\ub825\ud558\uc600\ub2e4.", "paragraph_answer": "\uac1c\ubc1c\uc0ac\uc778 \ucf5c\ub85c\uc124 \uc624\ub354\ub294 2009\ub144\uc5d4 5\uba85\ubc16\uc5d0 \uc548\ub418\uc5c8\uc73c\uba70, \ud68c\uc0ac \ucc3d\ub9bd\ubd80\ud130 \ub3c4\uc2dc \uc2dc\ubbac\ub808\uc774\uc158 \uac1c\ubc1c\uc744 \uc0dd\uac01\ud558\uace0 \uc788\uc5c8\uc9c0\ub9cc, 5\uba85\ubc16\uc5d0 \uc548\ub418\ub294 \uc9c0\ub77c '\uc774\uac8c \uacfc\uc5f0 \ub420\uae4c?' \ub780 \uc0dd\uac01\uc744 \uac16\uace0 \uc788\uc5c8\ub2e4. \ub610\ud55c \uadf8 \ub2f9\uc2dc \ucf5c\ub85c\uc124 \uc624\ub354\uc758 \uac1c\ubc1c\uc790\ub4e4\uc740 \ubaa8\ubc14\uc77c \uac8c\uc784\uc744 \uac1c\ubc1c\ud558\ub2e4 \ub098\uc654\uae30 \ub54c\ubb38\uc5d0 PC \ud50c\ub7ab\ud3fc\uc5d0 \ub300\ud574 \uc804\ud600 \ubaa8\ub974\ub358 \uc0c1\ud0dc\uc600\ub2e4. \uadf8\ub85c \uc778\ud574\uc11c \ucc98\uc74c \uc544\uc774\ub514\uc5b4\ub294 \uc9c4\uc9dc \ud074\ub798\uc2dd\ud55c \ub3c4\uc2dc \uac74\uc124 \uac8c\uc784\uc774\uc5c8\uc73c\ub098, \ud68c\uc0ac\uc5d0\uc11c \uafc8\uafb8\ub358 \ub3c4\uc2dc \uac74\uc124 \uc2dc\ubbac\ub808\uc774\uc158\uc744 \uac1c\ubc1c\ud558\uae30\uc5d4 \ubaa8\ubc14\uc77c \ud658\uacbd\uc774 \ub108\ubb34\ub098 \ub5a8\uc5b4\uc9c0\uc790 PC\ucabd\uc73c\ub85c \uc62e\uaca8\uac00\uac8c \ub418\uc5c8\ub2e4. \ucf5c\ub85c\uc124 \uc624\ub354 \uc9c1\uc6d0\uc740 2014\ub144 \ub2f9\uc2dc 9\uba85 \uc774\uc5c8\uc73c\uba70, 2015\ub144 13\uba85\uc73c\ub85c \uc778\uc6d0\uc744 \ub298\ub838\ub2e4. \uadf8\ub9ac\uace0 \ucf5c\ub85c\uc124 \uc624\ub354\ub294 \uc2dc\ud2f0\uc988 \uc778 \ubaa8\uc158 \uc2dc\ub9ac\uc988\ub97c \uac1c\ubc1c\ud558\uba74\uc11c, \ub3c4\uc2dc \uac74\uc124 \uc2dc\ubbac\ub808\uc774\uc158\uc5d0 \ub300\ud55c \uc9c0\uc2dd\ub4e4\uc744 \uc313\uc544\uac14\ub2e4. \uc608\ub97c \ub4e4\uc5b4\uc11c \ud070 \uc9c0\ub3c4\uc640 \uac74\uc124 \ub3c4\uad6c\ub294 \uc2dc\ud2f0\uc988 \uc778 \ubaa8\uc158 2\uc5d0\uc11c \ube44\ub86f\ub41c \uac83\uc774\ub2e4. \ucf5c\ub85c\uc124 \uc624\ub354 \uce21\uc740 \uc2ec\uc2dc\ud2f0\ub97c \ubd24\uc744\ub54c \uc88c\uc808\ud588\uc9c0\ub9cc, \uc2ec\uc2dc\ud2f0\uac00 \uc704\uae30\ub97c \uacaa\uc73c\uba74\uc11c \ud76c\ub9dd\uc774 \uc788\ub2e4\uace0 \ud310\ub2e8\ud558\uc5ec \uac1c\ubc1c\uc744 \uc9c4\ud589\ud558\uc600\ub2e4.\uadf8\ub9ac\uace0 \ucf5c\ub85c\uc124 \uc624\ub354\ub294 \ud2b9\uc815 \uacc4\uce35\ub9cc \uc990\uae30\ub358 \uc2dc\ud2f0\uc988 \uc778 \ubaa8\uc158\uacfc \ub2e4\ub974\uac8c, \uc2dc\ud2f0\uc988: \uc2a4\uce74\uc774\ub77c\uc778\uc744 \ub300\uc911\ub4e4\uc774 \ub354 \ub9ce\uc774 \uc811\ud560 \uc218 \uc788\uac8c \ub178\ub825\ud558\uc600\ub2e4.", "sentence_answer": "\ucf5c\ub85c\uc124 \uc624\ub354 \uc9c1\uc6d0\uc740 2014\ub144 \ub2f9\uc2dc 9\uba85 \uc774\uc5c8\uc73c\uba70, 2015\ub144 13\uba85\uc73c\ub85c \uc778\uc6d0\uc744 \ub298\ub838\ub2e4.", "paragraph_id": "6519573-6-1", "paragraph_question": "question: \uc2dc\ud2f0\uc988 \uc2a4\uce74\uc774\ub77c\uc778\uc758 2014\ub144 \ub2f9\uc2dc \uc9c1\uc6d0 \uc218\ub294?, context: \uac1c\ubc1c\uc0ac\uc778 \ucf5c\ub85c\uc124 \uc624\ub354\ub294 2009\ub144\uc5d4 5\uba85\ubc16\uc5d0 \uc548\ub418\uc5c8\uc73c\uba70, \ud68c\uc0ac \ucc3d\ub9bd\ubd80\ud130 \ub3c4\uc2dc \uc2dc\ubbac\ub808\uc774\uc158 \uac1c\ubc1c\uc744 \uc0dd\uac01\ud558\uace0 \uc788\uc5c8\uc9c0\ub9cc, 5\uba85\ubc16\uc5d0 \uc548\ub418\ub294 \uc9c0\ub77c '\uc774\uac8c \uacfc\uc5f0 \ub420\uae4c?' \ub780 \uc0dd\uac01\uc744 \uac16\uace0 \uc788\uc5c8\ub2e4. \ub610\ud55c \uadf8 \ub2f9\uc2dc \ucf5c\ub85c\uc124 \uc624\ub354\uc758 \uac1c\ubc1c\uc790\ub4e4\uc740 \ubaa8\ubc14\uc77c \uac8c\uc784\uc744 \uac1c\ubc1c\ud558\ub2e4 \ub098\uc654\uae30 \ub54c\ubb38\uc5d0 PC \ud50c\ub7ab\ud3fc\uc5d0 \ub300\ud574 \uc804\ud600 \ubaa8\ub974\ub358 \uc0c1\ud0dc\uc600\ub2e4. \uadf8\ub85c \uc778\ud574\uc11c \ucc98\uc74c \uc544\uc774\ub514\uc5b4\ub294 \uc9c4\uc9dc \ud074\ub798\uc2dd\ud55c \ub3c4\uc2dc \uac74\uc124 \uac8c\uc784\uc774\uc5c8\uc73c\ub098, \ud68c\uc0ac\uc5d0\uc11c \uafc8\uafb8\ub358 \ub3c4\uc2dc \uac74\uc124 \uc2dc\ubbac\ub808\uc774\uc158\uc744 \uac1c\ubc1c\ud558\uae30\uc5d4 \ubaa8\ubc14\uc77c \ud658\uacbd\uc774 \ub108\ubb34\ub098 \ub5a8\uc5b4\uc9c0\uc790 PC\ucabd\uc73c\ub85c \uc62e\uaca8\uac00\uac8c \ub418\uc5c8\ub2e4. \ucf5c\ub85c\uc124 \uc624\ub354 \uc9c1\uc6d0\uc740 2014\ub144 \ub2f9\uc2dc 9\uba85\uc774\uc5c8\uc73c\uba70, 2015\ub144 13\uba85\uc73c\ub85c \uc778\uc6d0\uc744 \ub298\ub838\ub2e4. \uadf8\ub9ac\uace0 \ucf5c\ub85c\uc124 \uc624\ub354\ub294 \uc2dc\ud2f0\uc988 \uc778 \ubaa8\uc158 \uc2dc\ub9ac\uc988\ub97c \uac1c\ubc1c\ud558\uba74\uc11c, \ub3c4\uc2dc \uac74\uc124 \uc2dc\ubbac\ub808\uc774\uc158\uc5d0 \ub300\ud55c \uc9c0\uc2dd\ub4e4\uc744 \uc313\uc544\uac14\ub2e4. \uc608\ub97c \ub4e4\uc5b4\uc11c \ud070 \uc9c0\ub3c4\uc640 \uac74\uc124 \ub3c4\uad6c\ub294 \uc2dc\ud2f0\uc988 \uc778 \ubaa8\uc158 2\uc5d0\uc11c \ube44\ub86f\ub41c \uac83\uc774\ub2e4. \ucf5c\ub85c\uc124 \uc624\ub354 \uce21\uc740 \uc2ec\uc2dc\ud2f0\ub97c \ubd24\uc744\ub54c \uc88c\uc808\ud588\uc9c0\ub9cc, \uc2ec\uc2dc\ud2f0\uac00 \uc704\uae30\ub97c \uacaa\uc73c\uba74\uc11c \ud76c\ub9dd\uc774 \uc788\ub2e4\uace0 \ud310\ub2e8\ud558\uc5ec \uac1c\ubc1c\uc744 \uc9c4\ud589\ud558\uc600\ub2e4.\uadf8\ub9ac\uace0 \ucf5c\ub85c\uc124 \uc624\ub354\ub294 \ud2b9\uc815 \uacc4\uce35\ub9cc \uc990\uae30\ub358 \uc2dc\ud2f0\uc988 \uc778 \ubaa8\uc158\uacfc \ub2e4\ub974\uac8c, \uc2dc\ud2f0\uc988: \uc2a4\uce74\uc774\ub77c\uc778\uc744 \ub300\uc911\ub4e4\uc774 \ub354 \ub9ce\uc774 \uc811\ud560 \uc218 \uc788\uac8c \ub178\ub825\ud558\uc600\ub2e4."} {"question": "2013\ub144\uc5d0 \ucee4\ubc8c \uc2a4\ud398\uc774\uc2a4 \ud504\ub85c\uadf8\ub7a8\uc774 \ub4f1\ub85d\ub41c \uacf3\uc740 \uc5b4\ub514\uc778\uac00?", "paragraph": "\ucee4\ubc8c \uc2a4\ud398\uc774\uc2a4 \ud504\ub85c\uadf8\ub7a8(Kerbal Space Program, \uc904\uc5ec\uc11c KSP)\uc740 \uba55\uc2dc\ucf54\uc758 Squad \uc0ac\uc5d0\uc11c \uac1c\ubc1c\ud558\uace0 \uc788\ub294 \uc0cc\ub4dc\ubc15\uc2a4 \ud615\uc2dd\uc758 \uc6b0\uc8fc\ube44\ud589 \uc2dc\ubbac\ub808\uc774\uc158 \uac8c\uc784\uc774\ub2e4. \ud50c\ub808\uc774\uc5b4\ub294 \ucee4\ube48\uc774\ub77c\ub294 \ud589\uc131\uc5d0 \uc704\uce58\ud55c \uc6b0\uc8fc \uae30\uc9c0\uc5d0\uc11c \ub85c\ucf13 \ub514\uc790\uc778, \ub85c\ucf13 \ubc1c\uc0ac, \uc6b0\uc8fc \ube44\ud589\uacfc \uc870\uc885 \ub4f1\uc744 \ub2f4\ub2f9\ud558\uac8c \ub41c\ub2e4.(\uc2e4\uc81c \uc6b0\uc8fc\ucc98\ub7fc \ub9e4\uc6b0 \ud06c\ub2e4) \ucee4\ubc8c \uc2a4\ud398\uc774\uc2a4 \ud504\ub85c\uadf8\ub7a8\uc740 \ub610\ud55c \ubaa8\ub4dc\ub97c \uc9c0\uc6d0\ud558\uace0 \uc788\uc73c\uba70, \ubaa8\ub4dc\uc5d0\uc11c \ub9cc\ub4e4\uc5b4\uc9c4 \uc5ec\ub7ec \uc694\uc18c\ub4e4\uc774 \uac8c\uc784\uc0c1\uc5d0 \uad6c\ud604\ub418\uae30\ub3c4 \ud55c\ub2e4. \ucee4\ubc8c \uc2a4\ud398\uc774\uc2a4 \ud504\ub85c\uadf8\ub7a8\uc740 \uc2a4\ud300\uc5d0 \ub4f1\ub85d\ub418\uae30 \uc804\uae4c\uc9c4, \uc624\uc9c1 \uc785 \uc18c\ubb38\uc5d0 \uc758\uc874\ud558\uc5ec \ud310\ub9e4\ud558\uc600\uc73c\uba70. 2013\ub144 \uc2a4\ud300\uc5d0 \ub4f1\ub85d\ub418\uc5c8\uc744 \ub54c, \ucd5c\uace0 \uc778\uae30 \uc81c\ud488 5\uc704\uc5d0 \uc62c\ub77c\uac00\uae30\ub3c4 \ud558\uc600\ub2e4. \uccab \uacf5\uac1c \ubc84\uc804\uc774 0.7.3\uc774 2011\ub144 6\uc6d4 24\uc77c\uc5d0 \ub098\uc654\uace0, \uc9c0\uae08\uae4c\uc9c0 \uafb8\uc900\ud788 \uc5c5\ub370\uc774\ud2b8\ub418\uc5b4 \uc624\uace0 \uc788\ub2e4.", "answer": "\uc2a4\ud300", "sentence": "\ucee4\ubc8c \uc2a4\ud398\uc774\uc2a4 \ud504\ub85c\uadf8\ub7a8\uc740 \uc2a4\ud300 \uc5d0 \ub4f1\ub85d\ub418\uae30 \uc804\uae4c\uc9c4, \uc624\uc9c1 \uc785 \uc18c\ubb38\uc5d0 \uc758\uc874\ud558\uc5ec \ud310\ub9e4\ud558\uc600\uc73c\uba70.", "paragraph_sentence": "\ucee4\ubc8c \uc2a4\ud398\uc774\uc2a4 \ud504\ub85c\uadf8\ub7a8(Kerbal Space Program, \uc904\uc5ec\uc11c KSP)\uc740 \uba55\uc2dc\ucf54\uc758 Squad \uc0ac\uc5d0\uc11c \uac1c\ubc1c\ud558\uace0 \uc788\ub294 \uc0cc\ub4dc\ubc15\uc2a4 \ud615\uc2dd\uc758 \uc6b0\uc8fc\ube44\ud589 \uc2dc\ubbac\ub808\uc774\uc158 \uac8c\uc784\uc774\ub2e4. \ud50c\ub808\uc774\uc5b4\ub294 \ucee4\ube48\uc774\ub77c\ub294 \ud589\uc131\uc5d0 \uc704\uce58\ud55c \uc6b0\uc8fc \uae30\uc9c0\uc5d0\uc11c \ub85c\ucf13 \ub514\uc790\uc778, \ub85c\ucf13 \ubc1c\uc0ac, \uc6b0\uc8fc \ube44\ud589\uacfc \uc870\uc885 \ub4f1\uc744 \ub2f4\ub2f9\ud558\uac8c \ub41c\ub2e4.(\uc2e4\uc81c \uc6b0\uc8fc\ucc98\ub7fc \ub9e4\uc6b0 \ud06c\ub2e4) \ucee4\ubc8c \uc2a4\ud398\uc774\uc2a4 \ud504\ub85c\uadf8\ub7a8\uc740 \ub610\ud55c \ubaa8\ub4dc\ub97c \uc9c0\uc6d0\ud558\uace0 \uc788\uc73c\uba70, \ubaa8\ub4dc\uc5d0\uc11c \ub9cc\ub4e4\uc5b4\uc9c4 \uc5ec\ub7ec \uc694\uc18c\ub4e4\uc774 \uac8c\uc784\uc0c1\uc5d0 \uad6c\ud604\ub418\uae30\ub3c4 \ud55c\ub2e4. \ucee4\ubc8c \uc2a4\ud398\uc774\uc2a4 \ud504\ub85c\uadf8\ub7a8\uc740 \uc2a4\ud300 \uc5d0 \ub4f1\ub85d\ub418\uae30 \uc804\uae4c\uc9c4, \uc624\uc9c1 \uc785 \uc18c\ubb38\uc5d0 \uc758\uc874\ud558\uc5ec \ud310\ub9e4\ud558\uc600\uc73c\uba70. 2013\ub144 \uc2a4\ud300\uc5d0 \ub4f1\ub85d\ub418\uc5c8\uc744 \ub54c, \ucd5c\uace0 \uc778\uae30 \uc81c\ud488 5\uc704\uc5d0 \uc62c\ub77c\uac00\uae30\ub3c4 \ud558\uc600\ub2e4. \uccab \uacf5\uac1c \ubc84\uc804\uc774 0.7.3\uc774 2011\ub144 6\uc6d4 24\uc77c\uc5d0 \ub098\uc654\uace0, \uc9c0\uae08\uae4c\uc9c0 \uafb8\uc900\ud788 \uc5c5\ub370\uc774\ud2b8\ub418\uc5b4 \uc624\uace0 \uc788\ub2e4.", "paragraph_answer": "\ucee4\ubc8c \uc2a4\ud398\uc774\uc2a4 \ud504\ub85c\uadf8\ub7a8(Kerbal Space Program, \uc904\uc5ec\uc11c KSP)\uc740 \uba55\uc2dc\ucf54\uc758 Squad \uc0ac\uc5d0\uc11c \uac1c\ubc1c\ud558\uace0 \uc788\ub294 \uc0cc\ub4dc\ubc15\uc2a4 \ud615\uc2dd\uc758 \uc6b0\uc8fc\ube44\ud589 \uc2dc\ubbac\ub808\uc774\uc158 \uac8c\uc784\uc774\ub2e4. \ud50c\ub808\uc774\uc5b4\ub294 \ucee4\ube48\uc774\ub77c\ub294 \ud589\uc131\uc5d0 \uc704\uce58\ud55c \uc6b0\uc8fc \uae30\uc9c0\uc5d0\uc11c \ub85c\ucf13 \ub514\uc790\uc778, \ub85c\ucf13 \ubc1c\uc0ac, \uc6b0\uc8fc \ube44\ud589\uacfc \uc870\uc885 \ub4f1\uc744 \ub2f4\ub2f9\ud558\uac8c \ub41c\ub2e4.(\uc2e4\uc81c \uc6b0\uc8fc\ucc98\ub7fc \ub9e4\uc6b0 \ud06c\ub2e4) \ucee4\ubc8c \uc2a4\ud398\uc774\uc2a4 \ud504\ub85c\uadf8\ub7a8\uc740 \ub610\ud55c \ubaa8\ub4dc\ub97c \uc9c0\uc6d0\ud558\uace0 \uc788\uc73c\uba70, \ubaa8\ub4dc\uc5d0\uc11c \ub9cc\ub4e4\uc5b4\uc9c4 \uc5ec\ub7ec \uc694\uc18c\ub4e4\uc774 \uac8c\uc784\uc0c1\uc5d0 \uad6c\ud604\ub418\uae30\ub3c4 \ud55c\ub2e4. \ucee4\ubc8c \uc2a4\ud398\uc774\uc2a4 \ud504\ub85c\uadf8\ub7a8\uc740 \uc2a4\ud300 \uc5d0 \ub4f1\ub85d\ub418\uae30 \uc804\uae4c\uc9c4, \uc624\uc9c1 \uc785 \uc18c\ubb38\uc5d0 \uc758\uc874\ud558\uc5ec \ud310\ub9e4\ud558\uc600\uc73c\uba70. 2013\ub144 \uc2a4\ud300\uc5d0 \ub4f1\ub85d\ub418\uc5c8\uc744 \ub54c, \ucd5c\uace0 \uc778\uae30 \uc81c\ud488 5\uc704\uc5d0 \uc62c\ub77c\uac00\uae30\ub3c4 \ud558\uc600\ub2e4. \uccab \uacf5\uac1c \ubc84\uc804\uc774 0.7.3\uc774 2011\ub144 6\uc6d4 24\uc77c\uc5d0 \ub098\uc654\uace0, \uc9c0\uae08\uae4c\uc9c0 \uafb8\uc900\ud788 \uc5c5\ub370\uc774\ud2b8\ub418\uc5b4 \uc624\uace0 \uc788\ub2e4.", "sentence_answer": "\ucee4\ubc8c \uc2a4\ud398\uc774\uc2a4 \ud504\ub85c\uadf8\ub7a8\uc740 \uc2a4\ud300 \uc5d0 \ub4f1\ub85d\ub418\uae30 \uc804\uae4c\uc9c4, \uc624\uc9c1 \uc785 \uc18c\ubb38\uc5d0 \uc758\uc874\ud558\uc5ec \ud310\ub9e4\ud558\uc600\uc73c\uba70.", "paragraph_id": "6561656-0-1", "paragraph_question": "question: 2013\ub144\uc5d0 \ucee4\ubc8c \uc2a4\ud398\uc774\uc2a4 \ud504\ub85c\uadf8\ub7a8\uc774 \ub4f1\ub85d\ub41c \uacf3\uc740 \uc5b4\ub514\uc778\uac00?, context: \ucee4\ubc8c \uc2a4\ud398\uc774\uc2a4 \ud504\ub85c\uadf8\ub7a8(Kerbal Space Program, \uc904\uc5ec\uc11c KSP)\uc740 \uba55\uc2dc\ucf54\uc758 Squad \uc0ac\uc5d0\uc11c \uac1c\ubc1c\ud558\uace0 \uc788\ub294 \uc0cc\ub4dc\ubc15\uc2a4 \ud615\uc2dd\uc758 \uc6b0\uc8fc\ube44\ud589 \uc2dc\ubbac\ub808\uc774\uc158 \uac8c\uc784\uc774\ub2e4. \ud50c\ub808\uc774\uc5b4\ub294 \ucee4\ube48\uc774\ub77c\ub294 \ud589\uc131\uc5d0 \uc704\uce58\ud55c \uc6b0\uc8fc \uae30\uc9c0\uc5d0\uc11c \ub85c\ucf13 \ub514\uc790\uc778, \ub85c\ucf13 \ubc1c\uc0ac, \uc6b0\uc8fc \ube44\ud589\uacfc \uc870\uc885 \ub4f1\uc744 \ub2f4\ub2f9\ud558\uac8c \ub41c\ub2e4.(\uc2e4\uc81c \uc6b0\uc8fc\ucc98\ub7fc \ub9e4\uc6b0 \ud06c\ub2e4) \ucee4\ubc8c \uc2a4\ud398\uc774\uc2a4 \ud504\ub85c\uadf8\ub7a8\uc740 \ub610\ud55c \ubaa8\ub4dc\ub97c \uc9c0\uc6d0\ud558\uace0 \uc788\uc73c\uba70, \ubaa8\ub4dc\uc5d0\uc11c \ub9cc\ub4e4\uc5b4\uc9c4 \uc5ec\ub7ec \uc694\uc18c\ub4e4\uc774 \uac8c\uc784\uc0c1\uc5d0 \uad6c\ud604\ub418\uae30\ub3c4 \ud55c\ub2e4. \ucee4\ubc8c \uc2a4\ud398\uc774\uc2a4 \ud504\ub85c\uadf8\ub7a8\uc740 \uc2a4\ud300\uc5d0 \ub4f1\ub85d\ub418\uae30 \uc804\uae4c\uc9c4, \uc624\uc9c1 \uc785 \uc18c\ubb38\uc5d0 \uc758\uc874\ud558\uc5ec \ud310\ub9e4\ud558\uc600\uc73c\uba70. 2013\ub144 \uc2a4\ud300\uc5d0 \ub4f1\ub85d\ub418\uc5c8\uc744 \ub54c, \ucd5c\uace0 \uc778\uae30 \uc81c\ud488 5\uc704\uc5d0 \uc62c\ub77c\uac00\uae30\ub3c4 \ud558\uc600\ub2e4. \uccab \uacf5\uac1c \ubc84\uc804\uc774 0.7.3\uc774 2011\ub144 6\uc6d4 24\uc77c\uc5d0 \ub098\uc654\uace0, \uc9c0\uae08\uae4c\uc9c0 \uafb8\uc900\ud788 \uc5c5\ub370\uc774\ud2b8\ub418\uc5b4 \uc624\uace0 \uc788\ub2e4."} {"question": "\ub77c\uc988\ubca0\ub9ac \ud30c\uc774\uc758 \ud504\ub85c\uc138\uc11c\ub294 \uc624\ubc84\ud074\ub7ed\uc2dc \uc131\ub2a5\uc744 \uc5b4\ub290\uc815\ub3c4\uae4c\uc9c0 \ub0bc \uc218 \uc788\ub294\uac00?", "paragraph": "\ub77c\uc988\ubca0\ub9ac \ud30c\uc774 2\ub97c \uc81c\uc678\ud55c \ub77c\uc988\ubca0\ub9ac \ud30c\uc774 \ubaa8\ub378\ub4e4\uc740 \ube0c\ub85c\ub4dc\ucef4\uc758 BCM2835 \ub2e8\uc77c \uce69 \uc2dc\uc2a4\ud15c\uc744 \uc0ac\uc6a9\ud558\uba70, \uc774 \uce69\uc5d0\ub294 ARM1176JZF-S 700 MHz \uc2f1\uae00\ucf54\uc5b4\ud504\ub85c\uc138\uc11c(\uc77c\ubc18 \ub370\uc2a4\ud06c\ud0d1\uc740 \ubcf4\ud1b5 2500 MHz~3500 MHz), \ube44\ub514\uc624\ucf54\uc5b4 IV VGA \uc640 512 \uba54\uac00\ubc14\uc774\ud2b8 RAM\uc774 \ub4e4\uc5b4 \uc788\ub2e4. \uadf8\ub9ac\uace0 \ub77c\uc988\ubca0\ub9ac \ud30c\uc774\uc758 \ud504\ub85c\uc138\uc11c\ub294 \uc624\ubc84\ud074\ub7ed\uc2dc \ucd5c\ub300 1000 MHz\uae4c\uc9c0\uc758 \uc131\ub2a5\uc744 \ubc1c\ud718 \ud560 \uc218 \uc788\uc73c\uba70, \ud558\ub4dc \ub514\uc2a4\ud06c \ub4dc\ub77c\uc774\ube0c\ub098 \uc194\ub9ac\ub4dc \uc2a4\ud14c\uc774\ud2b8 \ub4dc\ub77c\uc774\ube0c\ub97c \ub0b4\uc7a5\ud558\uace0 \uc788\uc9c0 \uc54a\uc73c\uba70, SD \uce74\ub4dc(B+,2 B+ \ubaa8\ub378\uc740 Micro SD Card\ub97c \uc0ac\uc6a9)\ub97c \uc678\ubd80 \uae30\uc5b5\uc7a5\uce58\ub85c \uc0ac\uc6a9\ud55c\ub2e4. \uc0c8\ub85c \ucd9c\uc2dc\ud55c 2 \ubaa8\ub378 B\ub294 ARM Cortex-A7 0.9GHz\ud504\ub85c\uc138\uc11c\uc640 \ub7a8\uc6a9\ub7c9\uc774 1GB\ub85c \uc131\ub2a5\uc774 \uc5c5\uadf8\ub808\uc774\ub4dc\ub418\uc5b4 \ucd9c\uc2dc \ub418\uc5c8\ub2e4. \ub77c\uc988\ubca0\ub9ac \uc7ac\ub2e8\uc740 \ucef4\ud4e8\ud130 \uad50\uc721 \uc99d\uc9c4\uc744 \uc704\ud574 2\uac00\uc9c0 \ubaa8\ub378\uc744 \ub0b4\ub193\uc558\uc73c\uba70, \uac01\uac01 25\ub2ec\ub7ec\uc640 35\ub2ec\ub7ec\ub85c \ucc45\uc815\ub418\uc5c8\ub2e4. 2012\ub144 2\uc6d4 29\uc77c \uc7ac\ub2e8\uc5d0\uc11c 35\ub2ec\ub7ec\uc9dc\ub9ac \ubaa8\ub378\uc758 \uc8fc\ubb38\uc744 \ubc1b\uae30 \uc2dc\uc791\ud558\uc600\ub2e4.", "answer": "1000 MHz", "sentence": "\ucd5c\ub300 1000 MHz \uae4c\uc9c0\uc758 \uc131\ub2a5\uc744 \ubc1c\ud718 \ud560 \uc218 \uc788\uc73c\uba70, \ud558\ub4dc \ub514\uc2a4\ud06c \ub4dc\ub77c\uc774\ube0c\ub098 \uc194\ub9ac\ub4dc \uc2a4\ud14c\uc774\ud2b8 \ub4dc\ub77c\uc774\ube0c\ub97c \ub0b4\uc7a5\ud558\uace0 \uc788\uc9c0 \uc54a\uc73c\uba70, SD \uce74\ub4dc(B+,2 B+ \ubaa8\ub378\uc740 Micro SD Card\ub97c \uc0ac\uc6a9)\ub97c \uc678\ubd80 \uae30\uc5b5\uc7a5\uce58\ub85c \uc0ac\uc6a9\ud55c\ub2e4.", "paragraph_sentence": "\ub77c\uc988\ubca0\ub9ac \ud30c\uc774 2\ub97c \uc81c\uc678\ud55c \ub77c\uc988\ubca0\ub9ac \ud30c\uc774 \ubaa8\ub378\ub4e4\uc740 \ube0c\ub85c\ub4dc\ucef4\uc758 BCM2835 \ub2e8\uc77c \uce69 \uc2dc\uc2a4\ud15c\uc744 \uc0ac\uc6a9\ud558\uba70, \uc774 \uce69\uc5d0\ub294 ARM1176JZF-S 700 MHz \uc2f1\uae00\ucf54\uc5b4\ud504\ub85c\uc138\uc11c(\uc77c\ubc18 \ub370\uc2a4\ud06c\ud0d1\uc740 \ubcf4\ud1b5 2500 MHz~3500 MHz), \ube44\ub514\uc624\ucf54\uc5b4 IV VGA \uc640 512 \uba54\uac00\ubc14\uc774\ud2b8 RAM\uc774 \ub4e4\uc5b4 \uc788\ub2e4. \uadf8\ub9ac\uace0 \ub77c\uc988\ubca0\ub9ac \ud30c\uc774\uc758 \ud504\ub85c\uc138\uc11c\ub294 \uc624\ubc84\ud074\ub7ed\uc2dc \ucd5c\ub300 1000 MHz \uae4c\uc9c0\uc758 \uc131\ub2a5\uc744 \ubc1c\ud718 \ud560 \uc218 \uc788\uc73c\uba70, \ud558\ub4dc \ub514\uc2a4\ud06c \ub4dc\ub77c\uc774\ube0c\ub098 \uc194\ub9ac\ub4dc \uc2a4\ud14c\uc774\ud2b8 \ub4dc\ub77c\uc774\ube0c\ub97c \ub0b4\uc7a5\ud558\uace0 \uc788\uc9c0 \uc54a\uc73c\uba70, SD \uce74\ub4dc(B+,2 B+ \ubaa8\ub378\uc740 Micro SD Card\ub97c \uc0ac\uc6a9)\ub97c \uc678\ubd80 \uae30\uc5b5\uc7a5\uce58\ub85c \uc0ac\uc6a9\ud55c\ub2e4. \uc0c8\ub85c \ucd9c\uc2dc\ud55c 2 \ubaa8\ub378 B\ub294 ARM Cortex-A7 0.9GHz\ud504\ub85c\uc138\uc11c\uc640 \ub7a8\uc6a9\ub7c9\uc774 1GB\ub85c \uc131\ub2a5\uc774 \uc5c5\uadf8\ub808\uc774\ub4dc\ub418\uc5b4 \ucd9c\uc2dc \ub418\uc5c8\ub2e4. \ub77c\uc988\ubca0\ub9ac \uc7ac\ub2e8\uc740 \ucef4\ud4e8\ud130 \uad50\uc721 \uc99d\uc9c4\uc744 \uc704\ud574 2\uac00\uc9c0 \ubaa8\ub378\uc744 \ub0b4\ub193\uc558\uc73c\uba70, \uac01\uac01 25\ub2ec\ub7ec\uc640 35\ub2ec\ub7ec\ub85c \ucc45\uc815\ub418\uc5c8\ub2e4. 2012\ub144 2\uc6d4 29\uc77c \uc7ac\ub2e8\uc5d0\uc11c 35\ub2ec\ub7ec\uc9dc\ub9ac \ubaa8\ub378\uc758 \uc8fc\ubb38\uc744 \ubc1b\uae30 \uc2dc\uc791\ud558\uc600\ub2e4.", "paragraph_answer": "\ub77c\uc988\ubca0\ub9ac \ud30c\uc774 2\ub97c \uc81c\uc678\ud55c \ub77c\uc988\ubca0\ub9ac \ud30c\uc774 \ubaa8\ub378\ub4e4\uc740 \ube0c\ub85c\ub4dc\ucef4\uc758 BCM2835 \ub2e8\uc77c \uce69 \uc2dc\uc2a4\ud15c\uc744 \uc0ac\uc6a9\ud558\uba70, \uc774 \uce69\uc5d0\ub294 ARM1176JZF-S 700 MHz \uc2f1\uae00\ucf54\uc5b4\ud504\ub85c\uc138\uc11c(\uc77c\ubc18 \ub370\uc2a4\ud06c\ud0d1\uc740 \ubcf4\ud1b5 2500 MHz~3500 MHz), \ube44\ub514\uc624\ucf54\uc5b4 IV VGA \uc640 512 \uba54\uac00\ubc14\uc774\ud2b8 RAM\uc774 \ub4e4\uc5b4 \uc788\ub2e4. \uadf8\ub9ac\uace0 \ub77c\uc988\ubca0\ub9ac \ud30c\uc774\uc758 \ud504\ub85c\uc138\uc11c\ub294 \uc624\ubc84\ud074\ub7ed\uc2dc \ucd5c\ub300 1000 MHz \uae4c\uc9c0\uc758 \uc131\ub2a5\uc744 \ubc1c\ud718 \ud560 \uc218 \uc788\uc73c\uba70, \ud558\ub4dc \ub514\uc2a4\ud06c \ub4dc\ub77c\uc774\ube0c\ub098 \uc194\ub9ac\ub4dc \uc2a4\ud14c\uc774\ud2b8 \ub4dc\ub77c\uc774\ube0c\ub97c \ub0b4\uc7a5\ud558\uace0 \uc788\uc9c0 \uc54a\uc73c\uba70, SD \uce74\ub4dc(B+,2 B+ \ubaa8\ub378\uc740 Micro SD Card\ub97c \uc0ac\uc6a9)\ub97c \uc678\ubd80 \uae30\uc5b5\uc7a5\uce58\ub85c \uc0ac\uc6a9\ud55c\ub2e4. \uc0c8\ub85c \ucd9c\uc2dc\ud55c 2 \ubaa8\ub378 B\ub294 ARM Cortex-A7 0.9GHz\ud504\ub85c\uc138\uc11c\uc640 \ub7a8\uc6a9\ub7c9\uc774 1GB\ub85c \uc131\ub2a5\uc774 \uc5c5\uadf8\ub808\uc774\ub4dc\ub418\uc5b4 \ucd9c\uc2dc \ub418\uc5c8\ub2e4. \ub77c\uc988\ubca0\ub9ac \uc7ac\ub2e8\uc740 \ucef4\ud4e8\ud130 \uad50\uc721 \uc99d\uc9c4\uc744 \uc704\ud574 2\uac00\uc9c0 \ubaa8\ub378\uc744 \ub0b4\ub193\uc558\uc73c\uba70, \uac01\uac01 25\ub2ec\ub7ec\uc640 35\ub2ec\ub7ec\ub85c \ucc45\uc815\ub418\uc5c8\ub2e4. 2012\ub144 2\uc6d4 29\uc77c \uc7ac\ub2e8\uc5d0\uc11c 35\ub2ec\ub7ec\uc9dc\ub9ac \ubaa8\ub378\uc758 \uc8fc\ubb38\uc744 \ubc1b\uae30 \uc2dc\uc791\ud558\uc600\ub2e4.", "sentence_answer": "\ucd5c\ub300 1000 MHz \uae4c\uc9c0\uc758 \uc131\ub2a5\uc744 \ubc1c\ud718 \ud560 \uc218 \uc788\uc73c\uba70, \ud558\ub4dc \ub514\uc2a4\ud06c \ub4dc\ub77c\uc774\ube0c\ub098 \uc194\ub9ac\ub4dc \uc2a4\ud14c\uc774\ud2b8 \ub4dc\ub77c\uc774\ube0c\ub97c \ub0b4\uc7a5\ud558\uace0 \uc788\uc9c0 \uc54a\uc73c\uba70, SD \uce74\ub4dc(B+,2 B+ \ubaa8\ub378\uc740 Micro SD Card\ub97c \uc0ac\uc6a9)\ub97c \uc678\ubd80 \uae30\uc5b5\uc7a5\uce58\ub85c \uc0ac\uc6a9\ud55c\ub2e4.", "paragraph_id": "6494420-0-0", "paragraph_question": "question: \ub77c\uc988\ubca0\ub9ac \ud30c\uc774\uc758 \ud504\ub85c\uc138\uc11c\ub294 \uc624\ubc84\ud074\ub7ed\uc2dc \uc131\ub2a5\uc744 \uc5b4\ub290\uc815\ub3c4\uae4c\uc9c0 \ub0bc \uc218 \uc788\ub294\uac00?, context: \ub77c\uc988\ubca0\ub9ac \ud30c\uc774 2\ub97c \uc81c\uc678\ud55c \ub77c\uc988\ubca0\ub9ac \ud30c\uc774 \ubaa8\ub378\ub4e4\uc740 \ube0c\ub85c\ub4dc\ucef4\uc758 BCM2835 \ub2e8\uc77c \uce69 \uc2dc\uc2a4\ud15c\uc744 \uc0ac\uc6a9\ud558\uba70, \uc774 \uce69\uc5d0\ub294 ARM1176JZF-S 700 MHz \uc2f1\uae00\ucf54\uc5b4\ud504\ub85c\uc138\uc11c(\uc77c\ubc18 \ub370\uc2a4\ud06c\ud0d1\uc740 \ubcf4\ud1b5 2500 MHz~3500 MHz), \ube44\ub514\uc624\ucf54\uc5b4 IV VGA \uc640 512 \uba54\uac00\ubc14\uc774\ud2b8 RAM\uc774 \ub4e4\uc5b4 \uc788\ub2e4. \uadf8\ub9ac\uace0 \ub77c\uc988\ubca0\ub9ac \ud30c\uc774\uc758 \ud504\ub85c\uc138\uc11c\ub294 \uc624\ubc84\ud074\ub7ed\uc2dc \ucd5c\ub300 1000 MHz\uae4c\uc9c0\uc758 \uc131\ub2a5\uc744 \ubc1c\ud718 \ud560 \uc218 \uc788\uc73c\uba70, \ud558\ub4dc \ub514\uc2a4\ud06c \ub4dc\ub77c\uc774\ube0c\ub098 \uc194\ub9ac\ub4dc \uc2a4\ud14c\uc774\ud2b8 \ub4dc\ub77c\uc774\ube0c\ub97c \ub0b4\uc7a5\ud558\uace0 \uc788\uc9c0 \uc54a\uc73c\uba70, SD \uce74\ub4dc(B+,2 B+ \ubaa8\ub378\uc740 Micro SD Card\ub97c \uc0ac\uc6a9)\ub97c \uc678\ubd80 \uae30\uc5b5\uc7a5\uce58\ub85c \uc0ac\uc6a9\ud55c\ub2e4. \uc0c8\ub85c \ucd9c\uc2dc\ud55c 2 \ubaa8\ub378 B\ub294 ARM Cortex-A7 0.9GHz\ud504\ub85c\uc138\uc11c\uc640 \ub7a8\uc6a9\ub7c9\uc774 1GB\ub85c \uc131\ub2a5\uc774 \uc5c5\uadf8\ub808\uc774\ub4dc\ub418\uc5b4 \ucd9c\uc2dc \ub418\uc5c8\ub2e4. \ub77c\uc988\ubca0\ub9ac \uc7ac\ub2e8\uc740 \ucef4\ud4e8\ud130 \uad50\uc721 \uc99d\uc9c4\uc744 \uc704\ud574 2\uac00\uc9c0 \ubaa8\ub378\uc744 \ub0b4\ub193\uc558\uc73c\uba70, \uac01\uac01 25\ub2ec\ub7ec\uc640 35\ub2ec\ub7ec\ub85c \ucc45\uc815\ub418\uc5c8\ub2e4. 2012\ub144 2\uc6d4 29\uc77c \uc7ac\ub2e8\uc5d0\uc11c 35\ub2ec\ub7ec\uc9dc\ub9ac \ubaa8\ub378\uc758 \uc8fc\ubb38\uc744 \ubc1b\uae30 \uc2dc\uc791\ud558\uc600\ub2e4."} {"question": "\ubcf4\uc218 \uc9c4\uc601\uc758 \ud45c\uc2ec \uc57d\ud654 \uc6d0\uc778\uc774 \ub41c \uac83\uc740 \ubc18\uae30\ubb38\uacfc \ub204\uad6c\uc758 \ubd88\ucd9c\ub9c8 \uc120\uc5b8\uc778\uac00?", "paragraph": "\ubcf4\uc218 \uc9c4\uc601\uc758 \uc720\ub825 \ub300\uad8c\ud6c4\ubcf4\ub85c \ub4f1\uadf9\ud55c \ubc18\uae30\ubb38 \uc804 \uc720\uc5d4 \uc0ac\ubb34\ucd1d\uc7a5\uc774 1\uc6d4 \ubd88\ucd9c\ub9c8\ub97c \uc120\uc5b8\ud558\uace0, \uc774\uc5b4\uc11c \uc720\ub825 \ud6c4\ubcf4\uc600\ub358 \ud669\uad50\uc548 \ub300\ud1b5\ub839 \uad8c\ud55c\ub300\ud589\ub3c4 \ubd88\ucd9c\ub9c8 \uc120\uc5b8\uc744 \ud568\uc5d0 \ub530\ub77c \ubcf4\uc218 \ud45c\uc2ec\uc774 \ud06c\uac8c \uc57d\ud654\ub418\uc5c8\ub2e4. \ub54c\ubb38\uc5d0 \uc120\uac70 \uadf9\ucd08\ubc18\uc5d0\ub294 \ubb38\uc7ac\uc778 \ub354\ubd88\uc5b4\ubbfc\uc8fc\ub2f9 \ud6c4\ubcf4\uac00 30 ~ 40%\uc5d0 \ub2ec\ud558\ub294 \uc9c0\uc9c0\uc728\ub85c \ub300\uc138\ub860\uc774 \uad73\ud600\uc9c0\uba74\uc11c, \uc774\ub97c \uacac\uc81c\ud558\ub824\ub294 \uc138\ub825 \uac04\uc5d0 \uc774\ub978\ubc14 '\ubb38\uc7ac\uc778 \ub300 \ube44\ubb38 \uc5f0\ub300' \uad6c\ub3c4\uac00 \ud615\uc131\ub418\uc5c8\ub2e4. \uc774\uc5d0 \ub530\ub77c \ube44\ubb38 \uc5f0\ub300\ub97c \ud1b5\ud55c \ub2e8\uc77c\ud654 \ubaa9\uc18c\ub9ac\uac00 \uc9c0\uc18d\uc801\uc73c\ub85c \uc81c\uae30\ub418\uc5c8\uc73c\ub098, \uac01\ub2f9 \ud6c4\ubcf4 \ub2f9\uc0ac\uc790\ub4e4\uc774 \ub2e8\uc77c\ud654 \uac70\ubd80 \ubc0f \ub300\uc120 \uc644\uc8fc\ub97c \uc120\uc5b8\ud558\uba74\uc11c \uc2e4\uc81c \ub2e8\uc77c\ud654\ub85c \uc774\uc5b4\uc9c0\uc9c0\ub294 \uc54a\uc558\ub2e4. \ub530\ub77c\uc11c \ubcf8\uaca9\uc801\uc778 \uc120\uac70\uc804\uc740 \ubb38\uc7ac\uc778\u00b7\ud64d\uc900\ud45c\u00b7\uc548\ucca0\uc218\u00b7\uc720\uc2b9\ubbfc\u00b7\uc2ec\uc0c1\uc815\uc758 \uc6d0\ub0b4 5\ub300 \uc8fc\uc694 \uc815\ub2f9 \ud6c4\ubcf4\ub4e4 \uac04\uc758 5\uc790 \ub300\uacb0 \uad6c\ub3c4\ub85c \uc9c4\ud589\ub418\uc5c8\ub2e4.", "answer": "\ud669\uad50\uc548", "sentence": "\uc774\uc5b4\uc11c \uc720\ub825 \ud6c4\ubcf4\uc600\ub358 \ud669\uad50\uc548 \ub300\ud1b5\ub839 \uad8c\ud55c\ub300\ud589\ub3c4 \ubd88\ucd9c\ub9c8 \uc120\uc5b8\uc744 \ud568\uc5d0 \ub530\ub77c \ubcf4\uc218 \ud45c\uc2ec\uc774 \ud06c\uac8c \uc57d\ud654\ub418\uc5c8\ub2e4.", "paragraph_sentence": "\ubcf4\uc218 \uc9c4\uc601\uc758 \uc720\ub825 \ub300\uad8c\ud6c4\ubcf4\ub85c \ub4f1\uadf9\ud55c \ubc18\uae30\ubb38 \uc804 \uc720\uc5d4 \uc0ac\ubb34\ucd1d\uc7a5\uc774 1\uc6d4 \ubd88\ucd9c\ub9c8\ub97c \uc120\uc5b8\ud558\uace0, \uc774\uc5b4\uc11c \uc720\ub825 \ud6c4\ubcf4\uc600\ub358 \ud669\uad50\uc548 \ub300\ud1b5\ub839 \uad8c\ud55c\ub300\ud589\ub3c4 \ubd88\ucd9c\ub9c8 \uc120\uc5b8\uc744 \ud568\uc5d0 \ub530\ub77c \ubcf4\uc218 \ud45c\uc2ec\uc774 \ud06c\uac8c \uc57d\ud654\ub418\uc5c8\ub2e4. \ub54c\ubb38\uc5d0 \uc120\uac70 \uadf9\ucd08\ubc18\uc5d0\ub294 \ubb38\uc7ac\uc778 \ub354\ubd88\uc5b4\ubbfc\uc8fc\ub2f9 \ud6c4\ubcf4\uac00 30 ~ 40%\uc5d0 \ub2ec\ud558\ub294 \uc9c0\uc9c0\uc728\ub85c \ub300\uc138\ub860\uc774 \uad73\ud600\uc9c0\uba74\uc11c, \uc774\ub97c \uacac\uc81c\ud558\ub824\ub294 \uc138\ub825 \uac04\uc5d0 \uc774\ub978\ubc14 '\ubb38\uc7ac\uc778 \ub300 \ube44\ubb38 \uc5f0\ub300' \uad6c\ub3c4\uac00 \ud615\uc131\ub418\uc5c8\ub2e4. \uc774\uc5d0 \ub530\ub77c \ube44\ubb38 \uc5f0\ub300\ub97c \ud1b5\ud55c \ub2e8\uc77c\ud654 \ubaa9\uc18c\ub9ac\uac00 \uc9c0\uc18d\uc801\uc73c\ub85c \uc81c\uae30\ub418\uc5c8\uc73c\ub098, \uac01\ub2f9 \ud6c4\ubcf4 \ub2f9\uc0ac\uc790\ub4e4\uc774 \ub2e8\uc77c\ud654 \uac70\ubd80 \ubc0f \ub300\uc120 \uc644\uc8fc\ub97c \uc120\uc5b8\ud558\uba74\uc11c \uc2e4\uc81c \ub2e8\uc77c\ud654\ub85c \uc774\uc5b4\uc9c0\uc9c0\ub294 \uc54a\uc558\ub2e4. \ub530\ub77c\uc11c \ubcf8\uaca9\uc801\uc778 \uc120\uac70\uc804\uc740 \ubb38\uc7ac\uc778\u00b7\ud64d\uc900\ud45c\u00b7\uc548\ucca0\uc218\u00b7\uc720\uc2b9\ubbfc\u00b7\uc2ec\uc0c1\uc815\uc758 \uc6d0\ub0b4 5\ub300 \uc8fc\uc694 \uc815\ub2f9 \ud6c4\ubcf4\ub4e4 \uac04\uc758 5\uc790 \ub300\uacb0 \uad6c\ub3c4\ub85c \uc9c4\ud589\ub418\uc5c8\ub2e4.", "paragraph_answer": "\ubcf4\uc218 \uc9c4\uc601\uc758 \uc720\ub825 \ub300\uad8c\ud6c4\ubcf4\ub85c \ub4f1\uadf9\ud55c \ubc18\uae30\ubb38 \uc804 \uc720\uc5d4 \uc0ac\ubb34\ucd1d\uc7a5\uc774 1\uc6d4 \ubd88\ucd9c\ub9c8\ub97c \uc120\uc5b8\ud558\uace0, \uc774\uc5b4\uc11c \uc720\ub825 \ud6c4\ubcf4\uc600\ub358 \ud669\uad50\uc548 \ub300\ud1b5\ub839 \uad8c\ud55c\ub300\ud589\ub3c4 \ubd88\ucd9c\ub9c8 \uc120\uc5b8\uc744 \ud568\uc5d0 \ub530\ub77c \ubcf4\uc218 \ud45c\uc2ec\uc774 \ud06c\uac8c \uc57d\ud654\ub418\uc5c8\ub2e4. \ub54c\ubb38\uc5d0 \uc120\uac70 \uadf9\ucd08\ubc18\uc5d0\ub294 \ubb38\uc7ac\uc778 \ub354\ubd88\uc5b4\ubbfc\uc8fc\ub2f9 \ud6c4\ubcf4\uac00 30 ~ 40%\uc5d0 \ub2ec\ud558\ub294 \uc9c0\uc9c0\uc728\ub85c \ub300\uc138\ub860\uc774 \uad73\ud600\uc9c0\uba74\uc11c, \uc774\ub97c \uacac\uc81c\ud558\ub824\ub294 \uc138\ub825 \uac04\uc5d0 \uc774\ub978\ubc14 '\ubb38\uc7ac\uc778 \ub300 \ube44\ubb38 \uc5f0\ub300' \uad6c\ub3c4\uac00 \ud615\uc131\ub418\uc5c8\ub2e4. \uc774\uc5d0 \ub530\ub77c \ube44\ubb38 \uc5f0\ub300\ub97c \ud1b5\ud55c \ub2e8\uc77c\ud654 \ubaa9\uc18c\ub9ac\uac00 \uc9c0\uc18d\uc801\uc73c\ub85c \uc81c\uae30\ub418\uc5c8\uc73c\ub098, \uac01\ub2f9 \ud6c4\ubcf4 \ub2f9\uc0ac\uc790\ub4e4\uc774 \ub2e8\uc77c\ud654 \uac70\ubd80 \ubc0f \ub300\uc120 \uc644\uc8fc\ub97c \uc120\uc5b8\ud558\uba74\uc11c \uc2e4\uc81c \ub2e8\uc77c\ud654\ub85c \uc774\uc5b4\uc9c0\uc9c0\ub294 \uc54a\uc558\ub2e4. \ub530\ub77c\uc11c \ubcf8\uaca9\uc801\uc778 \uc120\uac70\uc804\uc740 \ubb38\uc7ac\uc778\u00b7\ud64d\uc900\ud45c\u00b7\uc548\ucca0\uc218\u00b7\uc720\uc2b9\ubbfc\u00b7\uc2ec\uc0c1\uc815\uc758 \uc6d0\ub0b4 5\ub300 \uc8fc\uc694 \uc815\ub2f9 \ud6c4\ubcf4\ub4e4 \uac04\uc758 5\uc790 \ub300\uacb0 \uad6c\ub3c4\ub85c \uc9c4\ud589\ub418\uc5c8\ub2e4.", "sentence_answer": "\uc774\uc5b4\uc11c \uc720\ub825 \ud6c4\ubcf4\uc600\ub358 \ud669\uad50\uc548 \ub300\ud1b5\ub839 \uad8c\ud55c\ub300\ud589\ub3c4 \ubd88\ucd9c\ub9c8 \uc120\uc5b8\uc744 \ud568\uc5d0 \ub530\ub77c \ubcf4\uc218 \ud45c\uc2ec\uc774 \ud06c\uac8c \uc57d\ud654\ub418\uc5c8\ub2e4.", "paragraph_id": "6035071-0-1", "paragraph_question": "question: \ubcf4\uc218 \uc9c4\uc601\uc758 \ud45c\uc2ec \uc57d\ud654 \uc6d0\uc778\uc774 \ub41c \uac83\uc740 \ubc18\uae30\ubb38\uacfc \ub204\uad6c\uc758 \ubd88\ucd9c\ub9c8 \uc120\uc5b8\uc778\uac00?, context: \ubcf4\uc218 \uc9c4\uc601\uc758 \uc720\ub825 \ub300\uad8c\ud6c4\ubcf4\ub85c \ub4f1\uadf9\ud55c \ubc18\uae30\ubb38 \uc804 \uc720\uc5d4 \uc0ac\ubb34\ucd1d\uc7a5\uc774 1\uc6d4 \ubd88\ucd9c\ub9c8\ub97c \uc120\uc5b8\ud558\uace0, \uc774\uc5b4\uc11c \uc720\ub825 \ud6c4\ubcf4\uc600\ub358 \ud669\uad50\uc548 \ub300\ud1b5\ub839 \uad8c\ud55c\ub300\ud589\ub3c4 \ubd88\ucd9c\ub9c8 \uc120\uc5b8\uc744 \ud568\uc5d0 \ub530\ub77c \ubcf4\uc218 \ud45c\uc2ec\uc774 \ud06c\uac8c \uc57d\ud654\ub418\uc5c8\ub2e4. \ub54c\ubb38\uc5d0 \uc120\uac70 \uadf9\ucd08\ubc18\uc5d0\ub294 \ubb38\uc7ac\uc778 \ub354\ubd88\uc5b4\ubbfc\uc8fc\ub2f9 \ud6c4\ubcf4\uac00 30 ~ 40%\uc5d0 \ub2ec\ud558\ub294 \uc9c0\uc9c0\uc728\ub85c \ub300\uc138\ub860\uc774 \uad73\ud600\uc9c0\uba74\uc11c, \uc774\ub97c \uacac\uc81c\ud558\ub824\ub294 \uc138\ub825 \uac04\uc5d0 \uc774\ub978\ubc14 '\ubb38\uc7ac\uc778 \ub300 \ube44\ubb38 \uc5f0\ub300' \uad6c\ub3c4\uac00 \ud615\uc131\ub418\uc5c8\ub2e4. \uc774\uc5d0 \ub530\ub77c \ube44\ubb38 \uc5f0\ub300\ub97c \ud1b5\ud55c \ub2e8\uc77c\ud654 \ubaa9\uc18c\ub9ac\uac00 \uc9c0\uc18d\uc801\uc73c\ub85c \uc81c\uae30\ub418\uc5c8\uc73c\ub098, \uac01\ub2f9 \ud6c4\ubcf4 \ub2f9\uc0ac\uc790\ub4e4\uc774 \ub2e8\uc77c\ud654 \uac70\ubd80 \ubc0f \ub300\uc120 \uc644\uc8fc\ub97c \uc120\uc5b8\ud558\uba74\uc11c \uc2e4\uc81c \ub2e8\uc77c\ud654\ub85c \uc774\uc5b4\uc9c0\uc9c0\ub294 \uc54a\uc558\ub2e4. \ub530\ub77c\uc11c \ubcf8\uaca9\uc801\uc778 \uc120\uac70\uc804\uc740 \ubb38\uc7ac\uc778\u00b7\ud64d\uc900\ud45c\u00b7\uc548\ucca0\uc218\u00b7\uc720\uc2b9\ubbfc\u00b7\uc2ec\uc0c1\uc815\uc758 \uc6d0\ub0b4 5\ub300 \uc8fc\uc694 \uc815\ub2f9 \ud6c4\ubcf4\ub4e4 \uac04\uc758 5\uc790 \ub300\uacb0 \uad6c\ub3c4\ub85c \uc9c4\ud589\ub418\uc5c8\ub2e4."} {"question": "\ubc15\uadfc\ud61c \ub300\ud1b5\ub839\uc774 \ud558\uc57c \ud574\uc57c \uad6d\ubbfc\ub4e4\uc774 \ud798\uc744 \uc5bb\ub294\ub2e4\uace0 \ubc1c\uc5b8\ud55c \uc815\uc758\ub2f9 \uc6d0\ub0b4\ub300\ud45c\ub294 \ub204\uad6c\uc778\uac00?", "paragraph": "\uc774\ud6c4 \uc815\uce58\uc778 \ub300\ud45c\ub85c\uc11c \ub178\ud68c\ucc2c \uc815\uc758\ub2f9 \uc6d0\ub0b4\ub300\ud45c, \uae40\uc885\ud6c8 \ubb34\uc18c\uc18d \uc758\uc6d0, \uc774\uc7ac\uba85 \uc131\ub0a8\uc2dc\uc7a5\ub3c4 \ubc1c\uc5b8\uc5d0 \ub098\uc130\ub2e4. \ub178\ud68c\ucc2c \ub300\ud45c\ub294 \"\uc9c0\ub09c 3\ub144 8\uac1c\uc6d4\uac04 \ubd80\uc815\uc744 \uc800\uc9c0\ub978 \ubc15 \ub300\ud1b5\ub839\uc740 \uc5b4\ub5bb\uac8c \ud574\uc57c \ud558\ub290\ub0d0\"\uba70 \"\uc218\uc0ac\uac00 \ubc15 \ub300\ud1b5\ub839\uacfc \ucd5c\uc21c\uc2e4\uc5d0\uac8c \uacf5\uc720\ub418\uace0 \uc788\ub294\ub370 \ub9d0\uc774 \ub418\ub290\ub0d0, \ubc15 \ub300\ud1b5\ub839\uc774 \uc788\ub294 \ud55c \uc9c4\uc2e4\uaddc\uba85\uc774 \uc548 \ub41c\ub2e4. \ubc15 \ub300\ud1b5\ub839\uc774 \ub5a0\ub098\uc57c \uc6b0\ub9ac \uad6d\ubbfc\ub4e4\uc774 \ub2e4\uc2dc \ud798\uc744 \uc5bb\uc744 \uc218 \uc788\ub2e4\u201c\uace0 \uc8fc\uc7a5\ud558\uc600\ub2e4. \uae40\uc885\ud6c8 \ubb34\uc18c\uc18d \uc758\uc6d0\uc740 \"\uc9c0\uc5ed\uad6c\uc778 \uc6b8\uc0b0\uc5d0\uc11c \uc8fc\ubbfc\ub4e4\uc774 \uc11c\uc6b8\uc5d0 \uc62c\ub77c\uac00\ub294 \uc800\uc5d0\uac8c '\ub300\ud55c\ubbfc\uad6d\uc774 \ubd80\ub044\ub7fd\ub2e4', '\uad6d\ubbfc\ub4e4\uc744 \ubd80\ub044\ub7fd\uac8c \ub9cc\ub4e4\uace0 \uc788\ub2e4'\ub77c\ub294 \ub9d0\uc744 \uc804\ud574\ub2ec\ub77c\uace0 \ud588\ub2e4\"\uba74\uc11c, \"\uad6d\ubbfc\uc758 \ub9c8\uc74c\uc740 \ub300\ud1b5\ub839 \ud558\uc57c\ub2e4\", \"\ub300\ud1b5\ub839\uc774 \ud558\uc57c\ud560 \ub54c\uae4c\uc9c0 \ucd1b\ubd88\uc744 \ub4e4\uc790\"\ub77c\uace0 \uc678\ucce4\ub2e4. \uc774\uc7ac\uba85 \uc131\ub0a8\uc2dc\uc7a5\uc740 \"\ub300\ud1b5\ub839\uc740 \ub098\ub77c\uc758 \uc9c0\ubc30\uc790\uac00 \uc544\ub2c8\ub77c \uad6d\ubbfc\uc758 \uba38\uc2b4\uc774\uace0 \ub300\ub9ac\uc778\uc77c \ubfd0\"\uc774\ub77c\uba70 \"\uadf8\ub7f0 \uadf8\uac00 \ub9c8\uce58 \uc9c0\ubc30\uc790\uc778 \uc591, \uc5ec\uc655\uc778 \uc591 \uc0c1\ud669 \ucd5c\uc21c\uc2e4\uc744 \ub07c\uace0 \ub300\ud55c\ubbfc\uad6d\uc744, \ubbfc\uc8fc\uacf5\ud654\uad6d\uc744 \uc6b0\ub871\ud558\uace0 \uc788\ub2e4\"\uace0 \ubc1c\uc5b8\ud558\uc600\ub2e4. \uc138 \uc0ac\ub78c \uc678\uc5d0\ub3c4 \ub354\ubd88\uc5b4\ubbfc\uc8fc\ub2f9\uc758 \ud45c\ucc3d\uc6d0, \uc1a1\uc601\uae38, \ubc15\uc8fc\ubbfc, \uc815\ucd98\uc219 \uc758\uc6d0, \uc815\uc758\ub2f9\uc758 \uc774\uc815\ubbf8, \uae40\uc885\ub300 \uc758\uc6d0 \ub4f1 \uc57c\ub2f9 \uc815\uce58\uc778\ub4e4\uc774 \uc9d1\ud68c\uc5d0 \ucc38\uc5ec\ud558\uc600\ub2e4. \uc774\ub0a0 \uccad\uacc4\ucc9c \ucd1b\ubd88\uc9d1\ud68c\ub294 \ud329\ud2b8TV (\ud648\ud398\uc774\uc9c0, \uc720\ud29c\ube0c, \uc544\ud504\ub9ac\uce74TV)\uc640 \uc624\ub9c8\uc774TV (\uc720\ud29c\ube0c, \ud398\uc774\uc2a4\ubd81)\uc5d0\uc11c \uc0dd\uc911\uacc4\ud558\uc600\ub2e4.", "answer": "\ub178\ud68c\ucc2c", "sentence": "\uc774\ud6c4 \uc815\uce58\uc778 \ub300\ud45c\ub85c\uc11c \ub178\ud68c\ucc2c \uc815\uc758\ub2f9 \uc6d0\ub0b4\ub300\ud45c, \uae40\uc885\ud6c8 \ubb34\uc18c\uc18d \uc758\uc6d0, \uc774\uc7ac\uba85 \uc131\ub0a8\uc2dc\uc7a5\ub3c4 \ubc1c\uc5b8\uc5d0 \ub098\uc130\ub2e4.", "paragraph_sentence": " \uc774\ud6c4 \uc815\uce58\uc778 \ub300\ud45c\ub85c\uc11c \ub178\ud68c\ucc2c \uc815\uc758\ub2f9 \uc6d0\ub0b4\ub300\ud45c, \uae40\uc885\ud6c8 \ubb34\uc18c\uc18d \uc758\uc6d0, \uc774\uc7ac\uba85 \uc131\ub0a8\uc2dc\uc7a5\ub3c4 \ubc1c\uc5b8\uc5d0 \ub098\uc130\ub2e4. \ub178\ud68c\ucc2c \ub300\ud45c\ub294 \"\uc9c0\ub09c 3\ub144 8\uac1c\uc6d4\uac04 \ubd80\uc815\uc744 \uc800\uc9c0\ub978 \ubc15 \ub300\ud1b5\ub839\uc740 \uc5b4\ub5bb\uac8c \ud574\uc57c \ud558\ub290\ub0d0\"\uba70 \"\uc218\uc0ac\uac00 \ubc15 \ub300\ud1b5\ub839\uacfc \ucd5c\uc21c\uc2e4\uc5d0\uac8c \uacf5\uc720\ub418\uace0 \uc788\ub294\ub370 \ub9d0\uc774 \ub418\ub290\ub0d0, \ubc15 \ub300\ud1b5\ub839\uc774 \uc788\ub294 \ud55c \uc9c4\uc2e4\uaddc\uba85\uc774 \uc548 \ub41c\ub2e4. \ubc15 \ub300\ud1b5\ub839\uc774 \ub5a0\ub098\uc57c \uc6b0\ub9ac \uad6d\ubbfc\ub4e4\uc774 \ub2e4\uc2dc \ud798\uc744 \uc5bb\uc744 \uc218 \uc788\ub2e4\u201c\uace0 \uc8fc\uc7a5\ud558\uc600\ub2e4. \uae40\uc885\ud6c8 \ubb34\uc18c\uc18d \uc758\uc6d0\uc740 \"\uc9c0\uc5ed\uad6c\uc778 \uc6b8\uc0b0\uc5d0\uc11c \uc8fc\ubbfc\ub4e4\uc774 \uc11c\uc6b8\uc5d0 \uc62c\ub77c\uac00\ub294 \uc800\uc5d0\uac8c '\ub300\ud55c\ubbfc\uad6d\uc774 \ubd80\ub044\ub7fd\ub2e4', '\uad6d\ubbfc\ub4e4\uc744 \ubd80\ub044\ub7fd\uac8c \ub9cc\ub4e4\uace0 \uc788\ub2e4'\ub77c\ub294 \ub9d0\uc744 \uc804\ud574\ub2ec\ub77c\uace0 \ud588\ub2e4\"\uba74\uc11c, \"\uad6d\ubbfc\uc758 \ub9c8\uc74c\uc740 \ub300\ud1b5\ub839 \ud558\uc57c\ub2e4\", \"\ub300\ud1b5\ub839\uc774 \ud558\uc57c\ud560 \ub54c\uae4c\uc9c0 \ucd1b\ubd88\uc744 \ub4e4\uc790\"\ub77c\uace0 \uc678\ucce4\ub2e4. \uc774\uc7ac\uba85 \uc131\ub0a8\uc2dc\uc7a5\uc740 \"\ub300\ud1b5\ub839\uc740 \ub098\ub77c\uc758 \uc9c0\ubc30\uc790\uac00 \uc544\ub2c8\ub77c \uad6d\ubbfc\uc758 \uba38\uc2b4\uc774\uace0 \ub300\ub9ac\uc778\uc77c \ubfd0\"\uc774\ub77c\uba70 \"\uadf8\ub7f0 \uadf8\uac00 \ub9c8\uce58 \uc9c0\ubc30\uc790\uc778 \uc591, \uc5ec\uc655\uc778 \uc591 \uc0c1\ud669 \ucd5c\uc21c\uc2e4\uc744 \ub07c\uace0 \ub300\ud55c\ubbfc\uad6d\uc744, \ubbfc\uc8fc\uacf5\ud654\uad6d\uc744 \uc6b0\ub871\ud558\uace0 \uc788\ub2e4\"\uace0 \ubc1c\uc5b8\ud558\uc600\ub2e4. \uc138 \uc0ac\ub78c \uc678\uc5d0\ub3c4 \ub354\ubd88\uc5b4\ubbfc\uc8fc\ub2f9\uc758 \ud45c\ucc3d\uc6d0, \uc1a1\uc601\uae38, \ubc15\uc8fc\ubbfc, \uc815\ucd98\uc219 \uc758\uc6d0, \uc815\uc758\ub2f9\uc758 \uc774\uc815\ubbf8, \uae40\uc885\ub300 \uc758\uc6d0 \ub4f1 \uc57c\ub2f9 \uc815\uce58\uc778\ub4e4\uc774 \uc9d1\ud68c\uc5d0 \ucc38\uc5ec\ud558\uc600\ub2e4. \uc774\ub0a0 \uccad\uacc4\ucc9c \ucd1b\ubd88\uc9d1\ud68c\ub294 \ud329\ud2b8TV (\ud648\ud398\uc774\uc9c0, \uc720\ud29c\ube0c, \uc544\ud504\ub9ac\uce74TV)\uc640 \uc624\ub9c8\uc774TV (\uc720\ud29c\ube0c, \ud398\uc774\uc2a4\ubd81)\uc5d0\uc11c \uc0dd\uc911\uacc4\ud558\uc600\ub2e4.", "paragraph_answer": "\uc774\ud6c4 \uc815\uce58\uc778 \ub300\ud45c\ub85c\uc11c \ub178\ud68c\ucc2c \uc815\uc758\ub2f9 \uc6d0\ub0b4\ub300\ud45c, \uae40\uc885\ud6c8 \ubb34\uc18c\uc18d \uc758\uc6d0, \uc774\uc7ac\uba85 \uc131\ub0a8\uc2dc\uc7a5\ub3c4 \ubc1c\uc5b8\uc5d0 \ub098\uc130\ub2e4. \ub178\ud68c\ucc2c \ub300\ud45c\ub294 \"\uc9c0\ub09c 3\ub144 8\uac1c\uc6d4\uac04 \ubd80\uc815\uc744 \uc800\uc9c0\ub978 \ubc15 \ub300\ud1b5\ub839\uc740 \uc5b4\ub5bb\uac8c \ud574\uc57c \ud558\ub290\ub0d0\"\uba70 \"\uc218\uc0ac\uac00 \ubc15 \ub300\ud1b5\ub839\uacfc \ucd5c\uc21c\uc2e4\uc5d0\uac8c \uacf5\uc720\ub418\uace0 \uc788\ub294\ub370 \ub9d0\uc774 \ub418\ub290\ub0d0, \ubc15 \ub300\ud1b5\ub839\uc774 \uc788\ub294 \ud55c \uc9c4\uc2e4\uaddc\uba85\uc774 \uc548 \ub41c\ub2e4. \ubc15 \ub300\ud1b5\ub839\uc774 \ub5a0\ub098\uc57c \uc6b0\ub9ac \uad6d\ubbfc\ub4e4\uc774 \ub2e4\uc2dc \ud798\uc744 \uc5bb\uc744 \uc218 \uc788\ub2e4\u201c\uace0 \uc8fc\uc7a5\ud558\uc600\ub2e4. \uae40\uc885\ud6c8 \ubb34\uc18c\uc18d \uc758\uc6d0\uc740 \"\uc9c0\uc5ed\uad6c\uc778 \uc6b8\uc0b0\uc5d0\uc11c \uc8fc\ubbfc\ub4e4\uc774 \uc11c\uc6b8\uc5d0 \uc62c\ub77c\uac00\ub294 \uc800\uc5d0\uac8c '\ub300\ud55c\ubbfc\uad6d\uc774 \ubd80\ub044\ub7fd\ub2e4', '\uad6d\ubbfc\ub4e4\uc744 \ubd80\ub044\ub7fd\uac8c \ub9cc\ub4e4\uace0 \uc788\ub2e4'\ub77c\ub294 \ub9d0\uc744 \uc804\ud574\ub2ec\ub77c\uace0 \ud588\ub2e4\"\uba74\uc11c, \"\uad6d\ubbfc\uc758 \ub9c8\uc74c\uc740 \ub300\ud1b5\ub839 \ud558\uc57c\ub2e4\", \"\ub300\ud1b5\ub839\uc774 \ud558\uc57c\ud560 \ub54c\uae4c\uc9c0 \ucd1b\ubd88\uc744 \ub4e4\uc790\"\ub77c\uace0 \uc678\ucce4\ub2e4. \uc774\uc7ac\uba85 \uc131\ub0a8\uc2dc\uc7a5\uc740 \"\ub300\ud1b5\ub839\uc740 \ub098\ub77c\uc758 \uc9c0\ubc30\uc790\uac00 \uc544\ub2c8\ub77c \uad6d\ubbfc\uc758 \uba38\uc2b4\uc774\uace0 \ub300\ub9ac\uc778\uc77c \ubfd0\"\uc774\ub77c\uba70 \"\uadf8\ub7f0 \uadf8\uac00 \ub9c8\uce58 \uc9c0\ubc30\uc790\uc778 \uc591, \uc5ec\uc655\uc778 \uc591 \uc0c1\ud669 \ucd5c\uc21c\uc2e4\uc744 \ub07c\uace0 \ub300\ud55c\ubbfc\uad6d\uc744, \ubbfc\uc8fc\uacf5\ud654\uad6d\uc744 \uc6b0\ub871\ud558\uace0 \uc788\ub2e4\"\uace0 \ubc1c\uc5b8\ud558\uc600\ub2e4. \uc138 \uc0ac\ub78c \uc678\uc5d0\ub3c4 \ub354\ubd88\uc5b4\ubbfc\uc8fc\ub2f9\uc758 \ud45c\ucc3d\uc6d0, \uc1a1\uc601\uae38, \ubc15\uc8fc\ubbfc, \uc815\ucd98\uc219 \uc758\uc6d0, \uc815\uc758\ub2f9\uc758 \uc774\uc815\ubbf8, \uae40\uc885\ub300 \uc758\uc6d0 \ub4f1 \uc57c\ub2f9 \uc815\uce58\uc778\ub4e4\uc774 \uc9d1\ud68c\uc5d0 \ucc38\uc5ec\ud558\uc600\ub2e4. \uc774\ub0a0 \uccad\uacc4\ucc9c \ucd1b\ubd88\uc9d1\ud68c\ub294 \ud329\ud2b8TV (\ud648\ud398\uc774\uc9c0, \uc720\ud29c\ube0c, \uc544\ud504\ub9ac\uce74TV)\uc640 \uc624\ub9c8\uc774TV (\uc720\ud29c\ube0c, \ud398\uc774\uc2a4\ubd81)\uc5d0\uc11c \uc0dd\uc911\uacc4\ud558\uc600\ub2e4.", "sentence_answer": "\uc774\ud6c4 \uc815\uce58\uc778 \ub300\ud45c\ub85c\uc11c \ub178\ud68c\ucc2c \uc815\uc758\ub2f9 \uc6d0\ub0b4\ub300\ud45c, \uae40\uc885\ud6c8 \ubb34\uc18c\uc18d \uc758\uc6d0, \uc774\uc7ac\uba85 \uc131\ub0a8\uc2dc\uc7a5\ub3c4 \ubc1c\uc5b8\uc5d0 \ub098\uc130\ub2e4.", "paragraph_id": "6546893-11-0", "paragraph_question": "question: \ubc15\uadfc\ud61c \ub300\ud1b5\ub839\uc774 \ud558\uc57c \ud574\uc57c \uad6d\ubbfc\ub4e4\uc774 \ud798\uc744 \uc5bb\ub294\ub2e4\uace0 \ubc1c\uc5b8\ud55c \uc815\uc758\ub2f9 \uc6d0\ub0b4\ub300\ud45c\ub294 \ub204\uad6c\uc778\uac00?, context: \uc774\ud6c4 \uc815\uce58\uc778 \ub300\ud45c\ub85c\uc11c \ub178\ud68c\ucc2c \uc815\uc758\ub2f9 \uc6d0\ub0b4\ub300\ud45c, \uae40\uc885\ud6c8 \ubb34\uc18c\uc18d \uc758\uc6d0, \uc774\uc7ac\uba85 \uc131\ub0a8\uc2dc\uc7a5\ub3c4 \ubc1c\uc5b8\uc5d0 \ub098\uc130\ub2e4. \ub178\ud68c\ucc2c \ub300\ud45c\ub294 \"\uc9c0\ub09c 3\ub144 8\uac1c\uc6d4\uac04 \ubd80\uc815\uc744 \uc800\uc9c0\ub978 \ubc15 \ub300\ud1b5\ub839\uc740 \uc5b4\ub5bb\uac8c \ud574\uc57c \ud558\ub290\ub0d0\"\uba70 \"\uc218\uc0ac\uac00 \ubc15 \ub300\ud1b5\ub839\uacfc \ucd5c\uc21c\uc2e4\uc5d0\uac8c \uacf5\uc720\ub418\uace0 \uc788\ub294\ub370 \ub9d0\uc774 \ub418\ub290\ub0d0, \ubc15 \ub300\ud1b5\ub839\uc774 \uc788\ub294 \ud55c \uc9c4\uc2e4\uaddc\uba85\uc774 \uc548 \ub41c\ub2e4. \ubc15 \ub300\ud1b5\ub839\uc774 \ub5a0\ub098\uc57c \uc6b0\ub9ac \uad6d\ubbfc\ub4e4\uc774 \ub2e4\uc2dc \ud798\uc744 \uc5bb\uc744 \uc218 \uc788\ub2e4\u201c\uace0 \uc8fc\uc7a5\ud558\uc600\ub2e4. \uae40\uc885\ud6c8 \ubb34\uc18c\uc18d \uc758\uc6d0\uc740 \"\uc9c0\uc5ed\uad6c\uc778 \uc6b8\uc0b0\uc5d0\uc11c \uc8fc\ubbfc\ub4e4\uc774 \uc11c\uc6b8\uc5d0 \uc62c\ub77c\uac00\ub294 \uc800\uc5d0\uac8c '\ub300\ud55c\ubbfc\uad6d\uc774 \ubd80\ub044\ub7fd\ub2e4', '\uad6d\ubbfc\ub4e4\uc744 \ubd80\ub044\ub7fd\uac8c \ub9cc\ub4e4\uace0 \uc788\ub2e4'\ub77c\ub294 \ub9d0\uc744 \uc804\ud574\ub2ec\ub77c\uace0 \ud588\ub2e4\"\uba74\uc11c, \"\uad6d\ubbfc\uc758 \ub9c8\uc74c\uc740 \ub300\ud1b5\ub839 \ud558\uc57c\ub2e4\", \"\ub300\ud1b5\ub839\uc774 \ud558\uc57c\ud560 \ub54c\uae4c\uc9c0 \ucd1b\ubd88\uc744 \ub4e4\uc790\"\ub77c\uace0 \uc678\ucce4\ub2e4. \uc774\uc7ac\uba85 \uc131\ub0a8\uc2dc\uc7a5\uc740 \"\ub300\ud1b5\ub839\uc740 \ub098\ub77c\uc758 \uc9c0\ubc30\uc790\uac00 \uc544\ub2c8\ub77c \uad6d\ubbfc\uc758 \uba38\uc2b4\uc774\uace0 \ub300\ub9ac\uc778\uc77c \ubfd0\"\uc774\ub77c\uba70 \"\uadf8\ub7f0 \uadf8\uac00 \ub9c8\uce58 \uc9c0\ubc30\uc790\uc778 \uc591, \uc5ec\uc655\uc778 \uc591 \uc0c1\ud669 \ucd5c\uc21c\uc2e4\uc744 \ub07c\uace0 \ub300\ud55c\ubbfc\uad6d\uc744, \ubbfc\uc8fc\uacf5\ud654\uad6d\uc744 \uc6b0\ub871\ud558\uace0 \uc788\ub2e4\"\uace0 \ubc1c\uc5b8\ud558\uc600\ub2e4. \uc138 \uc0ac\ub78c \uc678\uc5d0\ub3c4 \ub354\ubd88\uc5b4\ubbfc\uc8fc\ub2f9\uc758 \ud45c\ucc3d\uc6d0, \uc1a1\uc601\uae38, \ubc15\uc8fc\ubbfc, \uc815\ucd98\uc219 \uc758\uc6d0, \uc815\uc758\ub2f9\uc758 \uc774\uc815\ubbf8, \uae40\uc885\ub300 \uc758\uc6d0 \ub4f1 \uc57c\ub2f9 \uc815\uce58\uc778\ub4e4\uc774 \uc9d1\ud68c\uc5d0 \ucc38\uc5ec\ud558\uc600\ub2e4. \uc774\ub0a0 \uccad\uacc4\ucc9c \ucd1b\ubd88\uc9d1\ud68c\ub294 \ud329\ud2b8TV (\ud648\ud398\uc774\uc9c0, \uc720\ud29c\ube0c, \uc544\ud504\ub9ac\uce74TV)\uc640 \uc624\ub9c8\uc774TV (\uc720\ud29c\ube0c, \ud398\uc774\uc2a4\ubd81)\uc5d0\uc11c \uc0dd\uc911\uacc4\ud558\uc600\ub2e4."} {"question": "\uc9c0\ubbf8 \ud504\ub9ac\uc824 \ub2e4\uc74c\uc73c\ub85c \ub9e8\uccb4\uc2a4\ud130 \uc2dc\ud2f0\uc758 \uac10\ub3c5\uc744 \ub9e1\uc740 \uac83\uc740 \ub204\uad6c\uc778\uac00?", "paragraph": "2\ub144\ub4a4, \ub9e8\uccb4\uc2a4\ud130 \uc2dc\ud2f0\ub294 \uc911\uc704\uad8c\uc744 \ub2ec\ub9ac\uace0 \uc788\uc5c8\uace0 \uac10\ub3c5\uc790\ub9ac\uac00 \uc548\uc804\ud560\uac83 \uac19\uc544 \ubcf4\uc600\uc9c0\ub9cc (\ub2f9\uc2dc \uac15\ub4f1\uc758 \uc704\ud5d8\uc870\ucc28 \ubcf4\uc774\uc9c0 \uc54a\uc558\ub2e4.) \ub73b\ubc16\uc5d0\ub3c4 \uc6b4\uc601\uc9c4\ub4e4\uacfc \ub17c\ub780\uc744 \uc77c\uc73c\ucf1c \uc2dc\uc98c\ub3c4\uc911\uc5d0 \ud574\uace0 \ub2f9\ud558\uac8c \ub418\uba70 \ucf54\uce58\uc9c1\uc5d0 \uc788\uc5c8\ub358 \uc874 \ubca4\uc2a8\uc774 \ud6c4\uc784\uc5d0 \uc624\ub978\ub2e4. \ud558\uc9c0\ub9cc \uc720\uac10\uc2a4\ub7fd\uac8c\ub3c4 \ubca4\uc2a8\uc774 \uc9c0\ud718\ud55c 19\uacbd\uae30\uc5d0\uc11c 11\uc810 \ubc16\uc5d0 \uc62c\ub9ac\uc9c0 \ubabb\ud588\uace0 \uc2dc\uc98c \ub9c8\uc9c0\ub9c9 \ub0a0 \ub8e8\ud134 \ud0c0\uc6b4\uc5d0\uac8c \ud328\ud558\uac8c \ub418\uc5b4 2\ubd80\ub9ac\uadf8\ub85c \uac15\ub4f1\ub418\ub294 \uc7ac\uc559\uc744 \uacaa\uac8c \ub41c\ub2e4. \ube4c\ub9ac \ub9e5\ub2d0\uc774 \ub2e4\uc74c \uac10\ub3c5\uc774 \ub418\uc5c8\uace0 \uc704\uae30\uc758 \ud55c \uc2dc\uc98c\uc744 \ubcf4\ub0b8\ub4a4, 1985\ub144, 1\ubd80\ub9ac\uadf8\ub85c \ub2e4\uc2dc \uc2b9\uaca9\uc774 \ub41c\ub2e4. \ud558\uc9c0\ub9cc \uc774\ubc88\uc5d0\ub3c4 \uc624\ub798\uac00\uc9c0 \ubabb\ud588\ub294\ub370 1986\ub144, \uac04\uc2e0\ud788 \uc794\ub958 \ud55c\ub4a4 \ube4c\ub9ac \ub9e5\ub2d0\uc740 \ub2e4\uc74c \uc2dc\uc98c \uc2dc\uc791\ud6c4 \uc5bc\ub9c8 \ubabb\uac00 \uc560\uc2a4\ud134 \ube4c\ub77c\ub85c \ub5a0\ub098\uac8c \ub41c\ub2e4. \ub0a8\uc740 \uc2dc\uc98c\ub3d9\uc548\uc740 \uc9c0\ubbf8 \ud504\ub9ac\uc824\uc774 \uafb8\ub824\ub098\uac00\uc9c0\ub9cc \uac15\ub4f1\uc744 \ub9c9\uc9c0 \ubabb\ud558\uace0 \uc560\uc2a4\ud134 \ube4c\ub77c\uc640 \ub808\uc2a4\ud130 \uc2dc\ud2f0\uc640 \ud568\uaed8 \uac15\ub4f1 \ub2f9\ud558\uac8c \ub41c\ub2e4. \ud504\ub9ac\uc824\uc740 \uc0ac\uc784\uc744 \ud558\uc600\uace0 \uba5c \ub9c8\uce5c\uc774 \ud6c4\uc784\uc5d0 \uc624\ub978\ub2e4.", "answer": "\uba5c \ub9c8\uce5c", "sentence": "\ud504\ub9ac\uc824\uc740 \uc0ac\uc784\uc744 \ud558\uc600\uace0 \uba5c \ub9c8\uce5c \uc774 \ud6c4\uc784\uc5d0 \uc624\ub978\ub2e4.", "paragraph_sentence": "2\ub144\ub4a4, \ub9e8\uccb4\uc2a4\ud130 \uc2dc\ud2f0\ub294 \uc911\uc704\uad8c\uc744 \ub2ec\ub9ac\uace0 \uc788\uc5c8\uace0 \uac10\ub3c5\uc790\ub9ac\uac00 \uc548\uc804\ud560\uac83 \uac19\uc544 \ubcf4\uc600\uc9c0\ub9cc (\ub2f9\uc2dc \uac15\ub4f1\uc758 \uc704\ud5d8\uc870\ucc28 \ubcf4\uc774\uc9c0 \uc54a\uc558\ub2e4. ) \ub73b\ubc16\uc5d0\ub3c4 \uc6b4\uc601\uc9c4\ub4e4\uacfc \ub17c\ub780\uc744 \uc77c\uc73c\ucf1c \uc2dc\uc98c\ub3c4\uc911\uc5d0 \ud574\uace0 \ub2f9\ud558\uac8c \ub418\uba70 \ucf54\uce58\uc9c1\uc5d0 \uc788\uc5c8\ub358 \uc874 \ubca4\uc2a8\uc774 \ud6c4\uc784\uc5d0 \uc624\ub978\ub2e4. \ud558\uc9c0\ub9cc \uc720\uac10\uc2a4\ub7fd\uac8c\ub3c4 \ubca4\uc2a8\uc774 \uc9c0\ud718\ud55c 19\uacbd\uae30\uc5d0\uc11c 11\uc810 \ubc16\uc5d0 \uc62c\ub9ac\uc9c0 \ubabb\ud588\uace0 \uc2dc\uc98c \ub9c8\uc9c0\ub9c9 \ub0a0 \ub8e8\ud134 \ud0c0\uc6b4\uc5d0\uac8c \ud328\ud558\uac8c \ub418\uc5b4 2\ubd80\ub9ac\uadf8\ub85c \uac15\ub4f1\ub418\ub294 \uc7ac\uc559\uc744 \uacaa\uac8c \ub41c\ub2e4. \ube4c\ub9ac \ub9e5\ub2d0\uc774 \ub2e4\uc74c \uac10\ub3c5\uc774 \ub418\uc5c8\uace0 \uc704\uae30\uc758 \ud55c \uc2dc\uc98c\uc744 \ubcf4\ub0b8\ub4a4, 1985\ub144, 1\ubd80\ub9ac\uadf8\ub85c \ub2e4\uc2dc \uc2b9\uaca9\uc774 \ub41c\ub2e4. \ud558\uc9c0\ub9cc \uc774\ubc88\uc5d0\ub3c4 \uc624\ub798\uac00\uc9c0 \ubabb\ud588\ub294\ub370 1986\ub144, \uac04\uc2e0\ud788 \uc794\ub958 \ud55c\ub4a4 \ube4c\ub9ac \ub9e5\ub2d0\uc740 \ub2e4\uc74c \uc2dc\uc98c \uc2dc\uc791\ud6c4 \uc5bc\ub9c8 \ubabb\uac00 \uc560\uc2a4\ud134 \ube4c\ub77c\ub85c \ub5a0\ub098\uac8c \ub41c\ub2e4. \ub0a8\uc740 \uc2dc\uc98c\ub3d9\uc548\uc740 \uc9c0\ubbf8 \ud504\ub9ac\uc824\uc774 \uafb8\ub824\ub098\uac00\uc9c0\ub9cc \uac15\ub4f1\uc744 \ub9c9\uc9c0 \ubabb\ud558\uace0 \uc560\uc2a4\ud134 \ube4c\ub77c\uc640 \ub808\uc2a4\ud130 \uc2dc\ud2f0\uc640 \ud568\uaed8 \uac15\ub4f1 \ub2f9\ud558\uac8c \ub41c\ub2e4. \ud504\ub9ac\uc824\uc740 \uc0ac\uc784\uc744 \ud558\uc600\uace0 \uba5c \ub9c8\uce5c \uc774 \ud6c4\uc784\uc5d0 \uc624\ub978\ub2e4. ", "paragraph_answer": "2\ub144\ub4a4, \ub9e8\uccb4\uc2a4\ud130 \uc2dc\ud2f0\ub294 \uc911\uc704\uad8c\uc744 \ub2ec\ub9ac\uace0 \uc788\uc5c8\uace0 \uac10\ub3c5\uc790\ub9ac\uac00 \uc548\uc804\ud560\uac83 \uac19\uc544 \ubcf4\uc600\uc9c0\ub9cc (\ub2f9\uc2dc \uac15\ub4f1\uc758 \uc704\ud5d8\uc870\ucc28 \ubcf4\uc774\uc9c0 \uc54a\uc558\ub2e4.) \ub73b\ubc16\uc5d0\ub3c4 \uc6b4\uc601\uc9c4\ub4e4\uacfc \ub17c\ub780\uc744 \uc77c\uc73c\ucf1c \uc2dc\uc98c\ub3c4\uc911\uc5d0 \ud574\uace0 \ub2f9\ud558\uac8c \ub418\uba70 \ucf54\uce58\uc9c1\uc5d0 \uc788\uc5c8\ub358 \uc874 \ubca4\uc2a8\uc774 \ud6c4\uc784\uc5d0 \uc624\ub978\ub2e4. \ud558\uc9c0\ub9cc \uc720\uac10\uc2a4\ub7fd\uac8c\ub3c4 \ubca4\uc2a8\uc774 \uc9c0\ud718\ud55c 19\uacbd\uae30\uc5d0\uc11c 11\uc810 \ubc16\uc5d0 \uc62c\ub9ac\uc9c0 \ubabb\ud588\uace0 \uc2dc\uc98c \ub9c8\uc9c0\ub9c9 \ub0a0 \ub8e8\ud134 \ud0c0\uc6b4\uc5d0\uac8c \ud328\ud558\uac8c \ub418\uc5b4 2\ubd80\ub9ac\uadf8\ub85c \uac15\ub4f1\ub418\ub294 \uc7ac\uc559\uc744 \uacaa\uac8c \ub41c\ub2e4. \ube4c\ub9ac \ub9e5\ub2d0\uc774 \ub2e4\uc74c \uac10\ub3c5\uc774 \ub418\uc5c8\uace0 \uc704\uae30\uc758 \ud55c \uc2dc\uc98c\uc744 \ubcf4\ub0b8\ub4a4, 1985\ub144, 1\ubd80\ub9ac\uadf8\ub85c \ub2e4\uc2dc \uc2b9\uaca9\uc774 \ub41c\ub2e4. \ud558\uc9c0\ub9cc \uc774\ubc88\uc5d0\ub3c4 \uc624\ub798\uac00\uc9c0 \ubabb\ud588\ub294\ub370 1986\ub144, \uac04\uc2e0\ud788 \uc794\ub958 \ud55c\ub4a4 \ube4c\ub9ac \ub9e5\ub2d0\uc740 \ub2e4\uc74c \uc2dc\uc98c \uc2dc\uc791\ud6c4 \uc5bc\ub9c8 \ubabb\uac00 \uc560\uc2a4\ud134 \ube4c\ub77c\ub85c \ub5a0\ub098\uac8c \ub41c\ub2e4. \ub0a8\uc740 \uc2dc\uc98c\ub3d9\uc548\uc740 \uc9c0\ubbf8 \ud504\ub9ac\uc824\uc774 \uafb8\ub824\ub098\uac00\uc9c0\ub9cc \uac15\ub4f1\uc744 \ub9c9\uc9c0 \ubabb\ud558\uace0 \uc560\uc2a4\ud134 \ube4c\ub77c\uc640 \ub808\uc2a4\ud130 \uc2dc\ud2f0\uc640 \ud568\uaed8 \uac15\ub4f1 \ub2f9\ud558\uac8c \ub41c\ub2e4. \ud504\ub9ac\uc824\uc740 \uc0ac\uc784\uc744 \ud558\uc600\uace0 \uba5c \ub9c8\uce5c \uc774 \ud6c4\uc784\uc5d0 \uc624\ub978\ub2e4.", "sentence_answer": "\ud504\ub9ac\uc824\uc740 \uc0ac\uc784\uc744 \ud558\uc600\uace0 \uba5c \ub9c8\uce5c \uc774 \ud6c4\uc784\uc5d0 \uc624\ub978\ub2e4.", "paragraph_id": "6529318-9-2", "paragraph_question": "question: \uc9c0\ubbf8 \ud504\ub9ac\uc824 \ub2e4\uc74c\uc73c\ub85c \ub9e8\uccb4\uc2a4\ud130 \uc2dc\ud2f0\uc758 \uac10\ub3c5\uc744 \ub9e1\uc740 \uac83\uc740 \ub204\uad6c\uc778\uac00?, context: 2\ub144\ub4a4, \ub9e8\uccb4\uc2a4\ud130 \uc2dc\ud2f0\ub294 \uc911\uc704\uad8c\uc744 \ub2ec\ub9ac\uace0 \uc788\uc5c8\uace0 \uac10\ub3c5\uc790\ub9ac\uac00 \uc548\uc804\ud560\uac83 \uac19\uc544 \ubcf4\uc600\uc9c0\ub9cc (\ub2f9\uc2dc \uac15\ub4f1\uc758 \uc704\ud5d8\uc870\ucc28 \ubcf4\uc774\uc9c0 \uc54a\uc558\ub2e4.) \ub73b\ubc16\uc5d0\ub3c4 \uc6b4\uc601\uc9c4\ub4e4\uacfc \ub17c\ub780\uc744 \uc77c\uc73c\ucf1c \uc2dc\uc98c\ub3c4\uc911\uc5d0 \ud574\uace0 \ub2f9\ud558\uac8c \ub418\uba70 \ucf54\uce58\uc9c1\uc5d0 \uc788\uc5c8\ub358 \uc874 \ubca4\uc2a8\uc774 \ud6c4\uc784\uc5d0 \uc624\ub978\ub2e4. \ud558\uc9c0\ub9cc \uc720\uac10\uc2a4\ub7fd\uac8c\ub3c4 \ubca4\uc2a8\uc774 \uc9c0\ud718\ud55c 19\uacbd\uae30\uc5d0\uc11c 11\uc810 \ubc16\uc5d0 \uc62c\ub9ac\uc9c0 \ubabb\ud588\uace0 \uc2dc\uc98c \ub9c8\uc9c0\ub9c9 \ub0a0 \ub8e8\ud134 \ud0c0\uc6b4\uc5d0\uac8c \ud328\ud558\uac8c \ub418\uc5b4 2\ubd80\ub9ac\uadf8\ub85c \uac15\ub4f1\ub418\ub294 \uc7ac\uc559\uc744 \uacaa\uac8c \ub41c\ub2e4. \ube4c\ub9ac \ub9e5\ub2d0\uc774 \ub2e4\uc74c \uac10\ub3c5\uc774 \ub418\uc5c8\uace0 \uc704\uae30\uc758 \ud55c \uc2dc\uc98c\uc744 \ubcf4\ub0b8\ub4a4, 1985\ub144, 1\ubd80\ub9ac\uadf8\ub85c \ub2e4\uc2dc \uc2b9\uaca9\uc774 \ub41c\ub2e4. \ud558\uc9c0\ub9cc \uc774\ubc88\uc5d0\ub3c4 \uc624\ub798\uac00\uc9c0 \ubabb\ud588\ub294\ub370 1986\ub144, \uac04\uc2e0\ud788 \uc794\ub958 \ud55c\ub4a4 \ube4c\ub9ac \ub9e5\ub2d0\uc740 \ub2e4\uc74c \uc2dc\uc98c \uc2dc\uc791\ud6c4 \uc5bc\ub9c8 \ubabb\uac00 \uc560\uc2a4\ud134 \ube4c\ub77c\ub85c \ub5a0\ub098\uac8c \ub41c\ub2e4. \ub0a8\uc740 \uc2dc\uc98c\ub3d9\uc548\uc740 \uc9c0\ubbf8 \ud504\ub9ac\uc824\uc774 \uafb8\ub824\ub098\uac00\uc9c0\ub9cc \uac15\ub4f1\uc744 \ub9c9\uc9c0 \ubabb\ud558\uace0 \uc560\uc2a4\ud134 \ube4c\ub77c\uc640 \ub808\uc2a4\ud130 \uc2dc\ud2f0\uc640 \ud568\uaed8 \uac15\ub4f1 \ub2f9\ud558\uac8c \ub41c\ub2e4. \ud504\ub9ac\uc824\uc740 \uc0ac\uc784\uc744 \ud558\uc600\uace0 \uba5c \ub9c8\uce5c\uc774 \ud6c4\uc784\uc5d0 \uc624\ub978\ub2e4."} {"question": "\uce90\uc11c\ub9b0\ub85c\uc774\ud130\uc758 1000m \uacb0\uc2b9\uae30\ub85d\uc740?", "paragraph": "3000m \uacc4\uc8fc\uc5d0\uc11c \uc568\ub9ac\uc2a8 \ub450\ub371, \uc568\ub9ac\uc2a8 \ubca0\uc774\ubc84, \ud0b4\ubc8c\ub9ac \ub370\ub9ad\uacfc \ud568\uaed8 \ucd9c\uc804\ud588\ub2e4. \ubbf8\uad6d \ud300\uc740 \uc900\uacb0\uc2b9\uc5d0\uc11c 4\ubd84 15.376\ucd08\ub97c \uae30\ub85d\ud558\uba70 \ub300\ud55c\ubbfc\uad6d\uc5d0 \ub4a4\uc774\uc5b4 2\uc704\ub97c \ucc28\uc9c0, \uacb0\uc2b9\uc5d0 \uc9c4\ucd9c\ud588\ub2e4. 1500m \uacb0\uc2b9\uc5d0\uc11c\ub294 \uc911\uad6d\uc758 \uc800\uc6b0\uc591, \ub300\ud55c\ubbfc\uad6d\uc758 \uc774\uc740\ubcc4\uacfc \ubc15\uc2b9\ud76c\uc5d0 \ubc00\ub824 4\uc704\ub97c \ucc28\uc9c0\ud588\ub2e4. \uce90\uc11c\ub9b0 \ub85c\uc774\ud130\ub294 \uc900\uacb0\uc2b9\uc5d0\uc11c \uc655\uba4d\uacfc \ubab8\uc2f8\uc6c0\uc744 \ubc8c\uc774\ub2e4 \uc5c9\ucf1c \ub118\uc5b4\uc84c\uace0, \ub300\ud55c\ubbfc\uad6d\uc758 \uc870\ud574\ub9ac\ub3c4 \uac19\uc774 \ub118\uc5b4\uc84c\ub2e4. \uacb0\uad6d \uc655\uba4d\uc740 \uc2e4\uaca9 \ucc98\ub9ac\ub97c \ubc1b\uc558\uace0 \uce90\uc11c\ub9b0 \ub85c\uc774\ud130\uc640 \ubc15\uc2b9\ud76c\ub294 \uc5b4\ub4dc\ubc34\ud2f0\uc9c0\ub85c \uacb0\uc2b9\uc5d0 \uc9c4\ucd9c\ud588\ub2e4. \uacb0\uc2b9\uc804\uc5d0\uc11c\ub3c4 \ubc15\uc2b9\ud76c\uc640 \ubd80\ub52a\ud788\uba70 \uac19\uc740 \ud589\ub3d9\uc744 \ubc18\ubcf5\ud588\uace0, \ub098\uc911\uc5d0 \ub450 \ubc88\uc758 \uc2e4\uc218\uc5d0 \ub300\ud574 \ub2e4\uc74c\uacfc \uac19\uc740 \uc0ac\uacfc\uc758 \ub9d0\uc744 \ud588\ub2e4. \u201c\ub0b4 \uc0dd\uac01\uc5d4 \uc5ec\uc720 \uacf5\uac04\uc774 \uc788\uc5c8\ub358 \uac83 \uac19\uc558\uc9c0\ub9cc \uadf8\ub807\uc9c0 \uc54a\uc558\ub2e4. \uc798 \ubaa8\ub974\uaca0\ub2e4. \uadf8\ub7ec\ub098 \ub0b4 \uc8fc\ubcc0\uc5d0 \uc788\ub294 \uc120\uc218\ub4e4\uc5d0\uac8c \ud3d0\ub97c \ub07c\uce5c \uc810\uc5d0 \ub300\ud574\uc11c\ub294 \ubbf8\uc548\ud568\uc744 \ub290\ub080\ub2e4.\u201d 1000m \uc608\uc120\uc5d0\uc11c \uc62c\ub9bc\ud53d \uae30\ub85d\uc744 \uc138\uc6b0\uba70 \uc870 1\uc704\ub97c \ucc28\uc9c0\ud588\ub2e4. \uadf8\ub85c\ubd80\ud130 \uc57d \ud55c \uc2dc\uac04 \ubc18 \ub4a4, 3000m \uacc4\uc8fc\uc5d0 \uc568\ub9ac\uc2a8 \ubca0\uc774\ubc84, \uc568\ub9ac\uc2a8 \ub450\ub371, \ub77c\ub098 \uac8c\ub9c1\uacfc \ud568\uaed8 \ucd9c\uc804\ud574 \ub300\ud55c\ubbfc\uad6d\uc758 \uc2e4\uaca9 \ub355\ud0dd\uc73c\ub85c \ub3d9\uba54\ub2ec\uc744 \ubaa9\uc5d0 \uac78\uc5c8\ub2e4. 1000m \uacb0\uc2b9\uc5d0\uc11c 1\ubd84 29.324\ucd08\ub97c \ub2ec\ub824, 1\ubd84 29.213\ucd08\ub97c \uae30\ub85d\ud55c \uc655\uba4d\uc5d0\uac8c \uadfc\uc18c\ud55c \ucc28\uc774\ub85c \uae08\uba54\ub2ec\uc744 \ub0b4\uc8fc\uc5c8\ub2e4.", "answer": "1\ubd84 29.324\ucd08", "sentence": "1000m \uacb0\uc2b9\uc5d0\uc11c 1\ubd84 29.324\ucd08 \ub97c \ub2ec\ub824, 1\ubd84 29.213\ucd08\ub97c \uae30\ub85d\ud55c \uc655\uba4d\uc5d0\uac8c \uadfc\uc18c\ud55c \ucc28\uc774\ub85c \uae08\uba54\ub2ec\uc744 \ub0b4\uc8fc\uc5c8\ub2e4.", "paragraph_sentence": "3000m \uacc4\uc8fc\uc5d0\uc11c \uc568\ub9ac\uc2a8 \ub450\ub371, \uc568\ub9ac\uc2a8 \ubca0\uc774\ubc84, \ud0b4\ubc8c\ub9ac \ub370\ub9ad\uacfc \ud568\uaed8 \ucd9c\uc804\ud588\ub2e4. \ubbf8\uad6d \ud300\uc740 \uc900\uacb0\uc2b9\uc5d0\uc11c 4\ubd84 15.376\ucd08\ub97c \uae30\ub85d\ud558\uba70 \ub300\ud55c\ubbfc\uad6d\uc5d0 \ub4a4\uc774\uc5b4 2\uc704\ub97c \ucc28\uc9c0, \uacb0\uc2b9\uc5d0 \uc9c4\ucd9c\ud588\ub2e4. 1500m \uacb0\uc2b9\uc5d0\uc11c\ub294 \uc911\uad6d\uc758 \uc800\uc6b0\uc591, \ub300\ud55c\ubbfc\uad6d\uc758 \uc774\uc740\ubcc4\uacfc \ubc15\uc2b9\ud76c\uc5d0 \ubc00\ub824 4\uc704\ub97c \ucc28\uc9c0\ud588\ub2e4. \uce90\uc11c\ub9b0 \ub85c\uc774\ud130\ub294 \uc900\uacb0\uc2b9\uc5d0\uc11c \uc655\uba4d\uacfc \ubab8\uc2f8\uc6c0\uc744 \ubc8c\uc774\ub2e4 \uc5c9\ucf1c \ub118\uc5b4\uc84c\uace0, \ub300\ud55c\ubbfc\uad6d\uc758 \uc870\ud574\ub9ac\ub3c4 \uac19\uc774 \ub118\uc5b4\uc84c\ub2e4. \uacb0\uad6d \uc655\uba4d\uc740 \uc2e4\uaca9 \ucc98\ub9ac\ub97c \ubc1b\uc558\uace0 \uce90\uc11c\ub9b0 \ub85c\uc774\ud130\uc640 \ubc15\uc2b9\ud76c\ub294 \uc5b4\ub4dc\ubc34\ud2f0\uc9c0\ub85c \uacb0\uc2b9\uc5d0 \uc9c4\ucd9c\ud588\ub2e4. \uacb0\uc2b9\uc804\uc5d0\uc11c\ub3c4 \ubc15\uc2b9\ud76c\uc640 \ubd80\ub52a\ud788\uba70 \uac19\uc740 \ud589\ub3d9\uc744 \ubc18\ubcf5\ud588\uace0, \ub098\uc911\uc5d0 \ub450 \ubc88\uc758 \uc2e4\uc218\uc5d0 \ub300\ud574 \ub2e4\uc74c\uacfc \uac19\uc740 \uc0ac\uacfc\uc758 \ub9d0\uc744 \ud588\ub2e4. \u201c\ub0b4 \uc0dd\uac01\uc5d4 \uc5ec\uc720 \uacf5\uac04\uc774 \uc788\uc5c8\ub358 \uac83 \uac19\uc558\uc9c0\ub9cc \uadf8\ub807\uc9c0 \uc54a\uc558\ub2e4. \uc798 \ubaa8\ub974\uaca0\ub2e4. \uadf8\ub7ec\ub098 \ub0b4 \uc8fc\ubcc0\uc5d0 \uc788\ub294 \uc120\uc218\ub4e4\uc5d0\uac8c \ud3d0\ub97c \ub07c\uce5c \uc810\uc5d0 \ub300\ud574\uc11c\ub294 \ubbf8\uc548\ud568\uc744 \ub290\ub080\ub2e4. \u201d 1000m \uc608\uc120\uc5d0\uc11c \uc62c\ub9bc\ud53d \uae30\ub85d\uc744 \uc138\uc6b0\uba70 \uc870 1\uc704\ub97c \ucc28\uc9c0\ud588\ub2e4. \uadf8\ub85c\ubd80\ud130 \uc57d \ud55c \uc2dc\uac04 \ubc18 \ub4a4, 3000m \uacc4\uc8fc\uc5d0 \uc568\ub9ac\uc2a8 \ubca0\uc774\ubc84, \uc568\ub9ac\uc2a8 \ub450\ub371, \ub77c\ub098 \uac8c\ub9c1\uacfc \ud568\uaed8 \ucd9c\uc804\ud574 \ub300\ud55c\ubbfc\uad6d\uc758 \uc2e4\uaca9 \ub355\ud0dd\uc73c\ub85c \ub3d9\uba54\ub2ec\uc744 \ubaa9\uc5d0 \uac78\uc5c8\ub2e4. 1000m \uacb0\uc2b9\uc5d0\uc11c 1\ubd84 29.324\ucd08 \ub97c \ub2ec\ub824, 1\ubd84 29.213\ucd08\ub97c \uae30\ub85d\ud55c \uc655\uba4d\uc5d0\uac8c \uadfc\uc18c\ud55c \ucc28\uc774\ub85c \uae08\uba54\ub2ec\uc744 \ub0b4\uc8fc\uc5c8\ub2e4. ", "paragraph_answer": "3000m \uacc4\uc8fc\uc5d0\uc11c \uc568\ub9ac\uc2a8 \ub450\ub371, \uc568\ub9ac\uc2a8 \ubca0\uc774\ubc84, \ud0b4\ubc8c\ub9ac \ub370\ub9ad\uacfc \ud568\uaed8 \ucd9c\uc804\ud588\ub2e4. \ubbf8\uad6d \ud300\uc740 \uc900\uacb0\uc2b9\uc5d0\uc11c 4\ubd84 15.376\ucd08\ub97c \uae30\ub85d\ud558\uba70 \ub300\ud55c\ubbfc\uad6d\uc5d0 \ub4a4\uc774\uc5b4 2\uc704\ub97c \ucc28\uc9c0, \uacb0\uc2b9\uc5d0 \uc9c4\ucd9c\ud588\ub2e4. 1500m \uacb0\uc2b9\uc5d0\uc11c\ub294 \uc911\uad6d\uc758 \uc800\uc6b0\uc591, \ub300\ud55c\ubbfc\uad6d\uc758 \uc774\uc740\ubcc4\uacfc \ubc15\uc2b9\ud76c\uc5d0 \ubc00\ub824 4\uc704\ub97c \ucc28\uc9c0\ud588\ub2e4. \uce90\uc11c\ub9b0 \ub85c\uc774\ud130\ub294 \uc900\uacb0\uc2b9\uc5d0\uc11c \uc655\uba4d\uacfc \ubab8\uc2f8\uc6c0\uc744 \ubc8c\uc774\ub2e4 \uc5c9\ucf1c \ub118\uc5b4\uc84c\uace0, \ub300\ud55c\ubbfc\uad6d\uc758 \uc870\ud574\ub9ac\ub3c4 \uac19\uc774 \ub118\uc5b4\uc84c\ub2e4. \uacb0\uad6d \uc655\uba4d\uc740 \uc2e4\uaca9 \ucc98\ub9ac\ub97c \ubc1b\uc558\uace0 \uce90\uc11c\ub9b0 \ub85c\uc774\ud130\uc640 \ubc15\uc2b9\ud76c\ub294 \uc5b4\ub4dc\ubc34\ud2f0\uc9c0\ub85c \uacb0\uc2b9\uc5d0 \uc9c4\ucd9c\ud588\ub2e4. \uacb0\uc2b9\uc804\uc5d0\uc11c\ub3c4 \ubc15\uc2b9\ud76c\uc640 \ubd80\ub52a\ud788\uba70 \uac19\uc740 \ud589\ub3d9\uc744 \ubc18\ubcf5\ud588\uace0, \ub098\uc911\uc5d0 \ub450 \ubc88\uc758 \uc2e4\uc218\uc5d0 \ub300\ud574 \ub2e4\uc74c\uacfc \uac19\uc740 \uc0ac\uacfc\uc758 \ub9d0\uc744 \ud588\ub2e4. \u201c\ub0b4 \uc0dd\uac01\uc5d4 \uc5ec\uc720 \uacf5\uac04\uc774 \uc788\uc5c8\ub358 \uac83 \uac19\uc558\uc9c0\ub9cc \uadf8\ub807\uc9c0 \uc54a\uc558\ub2e4. \uc798 \ubaa8\ub974\uaca0\ub2e4. \uadf8\ub7ec\ub098 \ub0b4 \uc8fc\ubcc0\uc5d0 \uc788\ub294 \uc120\uc218\ub4e4\uc5d0\uac8c \ud3d0\ub97c \ub07c\uce5c \uc810\uc5d0 \ub300\ud574\uc11c\ub294 \ubbf8\uc548\ud568\uc744 \ub290\ub080\ub2e4.\u201d 1000m \uc608\uc120\uc5d0\uc11c \uc62c\ub9bc\ud53d \uae30\ub85d\uc744 \uc138\uc6b0\uba70 \uc870 1\uc704\ub97c \ucc28\uc9c0\ud588\ub2e4. \uadf8\ub85c\ubd80\ud130 \uc57d \ud55c \uc2dc\uac04 \ubc18 \ub4a4, 3000m \uacc4\uc8fc\uc5d0 \uc568\ub9ac\uc2a8 \ubca0\uc774\ubc84, \uc568\ub9ac\uc2a8 \ub450\ub371, \ub77c\ub098 \uac8c\ub9c1\uacfc \ud568\uaed8 \ucd9c\uc804\ud574 \ub300\ud55c\ubbfc\uad6d\uc758 \uc2e4\uaca9 \ub355\ud0dd\uc73c\ub85c \ub3d9\uba54\ub2ec\uc744 \ubaa9\uc5d0 \uac78\uc5c8\ub2e4. 1000m \uacb0\uc2b9\uc5d0\uc11c 1\ubd84 29.324\ucd08 \ub97c \ub2ec\ub824, 1\ubd84 29.213\ucd08\ub97c \uae30\ub85d\ud55c \uc655\uba4d\uc5d0\uac8c \uadfc\uc18c\ud55c \ucc28\uc774\ub85c \uae08\uba54\ub2ec\uc744 \ub0b4\uc8fc\uc5c8\ub2e4.", "sentence_answer": "1000m \uacb0\uc2b9\uc5d0\uc11c 1\ubd84 29.324\ucd08 \ub97c \ub2ec\ub824, 1\ubd84 29.213\ucd08\ub97c \uae30\ub85d\ud55c \uc655\uba4d\uc5d0\uac8c \uadfc\uc18c\ud55c \ucc28\uc774\ub85c \uae08\uba54\ub2ec\uc744 \ub0b4\uc8fc\uc5c8\ub2e4.", "paragraph_id": "6346415-0-2", "paragraph_question": "question: \uce90\uc11c\ub9b0\ub85c\uc774\ud130\uc758 1000m \uacb0\uc2b9\uae30\ub85d\uc740?, context: 3000m \uacc4\uc8fc\uc5d0\uc11c \uc568\ub9ac\uc2a8 \ub450\ub371, \uc568\ub9ac\uc2a8 \ubca0\uc774\ubc84, \ud0b4\ubc8c\ub9ac \ub370\ub9ad\uacfc \ud568\uaed8 \ucd9c\uc804\ud588\ub2e4. \ubbf8\uad6d \ud300\uc740 \uc900\uacb0\uc2b9\uc5d0\uc11c 4\ubd84 15.376\ucd08\ub97c \uae30\ub85d\ud558\uba70 \ub300\ud55c\ubbfc\uad6d\uc5d0 \ub4a4\uc774\uc5b4 2\uc704\ub97c \ucc28\uc9c0, \uacb0\uc2b9\uc5d0 \uc9c4\ucd9c\ud588\ub2e4. 1500m \uacb0\uc2b9\uc5d0\uc11c\ub294 \uc911\uad6d\uc758 \uc800\uc6b0\uc591, \ub300\ud55c\ubbfc\uad6d\uc758 \uc774\uc740\ubcc4\uacfc \ubc15\uc2b9\ud76c\uc5d0 \ubc00\ub824 4\uc704\ub97c \ucc28\uc9c0\ud588\ub2e4. \uce90\uc11c\ub9b0 \ub85c\uc774\ud130\ub294 \uc900\uacb0\uc2b9\uc5d0\uc11c \uc655\uba4d\uacfc \ubab8\uc2f8\uc6c0\uc744 \ubc8c\uc774\ub2e4 \uc5c9\ucf1c \ub118\uc5b4\uc84c\uace0, \ub300\ud55c\ubbfc\uad6d\uc758 \uc870\ud574\ub9ac\ub3c4 \uac19\uc774 \ub118\uc5b4\uc84c\ub2e4. \uacb0\uad6d \uc655\uba4d\uc740 \uc2e4\uaca9 \ucc98\ub9ac\ub97c \ubc1b\uc558\uace0 \uce90\uc11c\ub9b0 \ub85c\uc774\ud130\uc640 \ubc15\uc2b9\ud76c\ub294 \uc5b4\ub4dc\ubc34\ud2f0\uc9c0\ub85c \uacb0\uc2b9\uc5d0 \uc9c4\ucd9c\ud588\ub2e4. \uacb0\uc2b9\uc804\uc5d0\uc11c\ub3c4 \ubc15\uc2b9\ud76c\uc640 \ubd80\ub52a\ud788\uba70 \uac19\uc740 \ud589\ub3d9\uc744 \ubc18\ubcf5\ud588\uace0, \ub098\uc911\uc5d0 \ub450 \ubc88\uc758 \uc2e4\uc218\uc5d0 \ub300\ud574 \ub2e4\uc74c\uacfc \uac19\uc740 \uc0ac\uacfc\uc758 \ub9d0\uc744 \ud588\ub2e4. \u201c\ub0b4 \uc0dd\uac01\uc5d4 \uc5ec\uc720 \uacf5\uac04\uc774 \uc788\uc5c8\ub358 \uac83 \uac19\uc558\uc9c0\ub9cc \uadf8\ub807\uc9c0 \uc54a\uc558\ub2e4. \uc798 \ubaa8\ub974\uaca0\ub2e4. \uadf8\ub7ec\ub098 \ub0b4 \uc8fc\ubcc0\uc5d0 \uc788\ub294 \uc120\uc218\ub4e4\uc5d0\uac8c \ud3d0\ub97c \ub07c\uce5c \uc810\uc5d0 \ub300\ud574\uc11c\ub294 \ubbf8\uc548\ud568\uc744 \ub290\ub080\ub2e4.\u201d 1000m \uc608\uc120\uc5d0\uc11c \uc62c\ub9bc\ud53d \uae30\ub85d\uc744 \uc138\uc6b0\uba70 \uc870 1\uc704\ub97c \ucc28\uc9c0\ud588\ub2e4. \uadf8\ub85c\ubd80\ud130 \uc57d \ud55c \uc2dc\uac04 \ubc18 \ub4a4, 3000m \uacc4\uc8fc\uc5d0 \uc568\ub9ac\uc2a8 \ubca0\uc774\ubc84, \uc568\ub9ac\uc2a8 \ub450\ub371, \ub77c\ub098 \uac8c\ub9c1\uacfc \ud568\uaed8 \ucd9c\uc804\ud574 \ub300\ud55c\ubbfc\uad6d\uc758 \uc2e4\uaca9 \ub355\ud0dd\uc73c\ub85c \ub3d9\uba54\ub2ec\uc744 \ubaa9\uc5d0 \uac78\uc5c8\ub2e4. 1000m \uacb0\uc2b9\uc5d0\uc11c 1\ubd84 29.324\ucd08\ub97c \ub2ec\ub824, 1\ubd84 29.213\ucd08\ub97c \uae30\ub85d\ud55c \uc655\uba4d\uc5d0\uac8c \uadfc\uc18c\ud55c \ucc28\uc774\ub85c \uae08\uba54\ub2ec\uc744 \ub0b4\uc8fc\uc5c8\ub2e4."} {"question": "\ub9ac\uc554 \ub2c8\uc2a8\uc740 \uc5b4\ub290\ub098\ub77c \ubc30\uc6b0\uc778\uac00?", "paragraph": "\ub9ac\uc554 \uc874 \ub2c8\uc2a8(\uc601\uc5b4: Liam John Neeson, OBE, 1952\ub144 6\uc6d4 7\uc77c ~ )\uc740 \ubd81\uc544\uc77c\ub79c\ub4dc\uc758 \ubc30\uc6b0\uc774\ub2e4. 1976\ub144 \uadf8\ub294 2\ub144\uac04 \ubca8\ud30c\uc2a4\ud2b8\uc758 \ub9ac\ub9ad \ud50c\ub808\uc774\uc5b4\uc2a4 \uadf9\uc7a5\uc5d0\uc11c \ud65c\ub3d9\ud588\ub2e4. \uadf8\ub294 \uc544\ub354\uc655\uc758 \uc601\ud654 \u300a\uc5d1\uc2a4\uce7c\ub9ac\ubc84\u300b(1981)\uc5d0 \ucd9c\uc5f0\ud588\ub2e4. 1982\ub144\ubd80\ud130 1987\ub144\uae4c\uc9c0 \ub2c8\uc2a8\uc740 \u300a\ubc14\uc6b4\ud2f0 \ud638\uc758 \ubc18\ub780\u300b(1984)\uc5d0\uc11c \uba5c \uae41\uc2a8, \uc548\uc18c\ub2c8 \ud649\ud0a8\uc2a4, \u300a\ubbf8\uc158\u300b(1986)\uc5d0\uc11c \ub85c\ubc84\ud2b8 \ub4dc \ub2c8\ub85c, \uc81c\ub808\ubbf8 \uc544\uc774\uc5b8\uc2a4\uc640 \ud568\uaed8 5\ud3b8\uc758 \uc601\ud654\uc5d0 \ucd9c\uc5f0\ud588\ub2e4. \uadf8\ub294 \u300a\ubd88\ud0c0\ub294 \ubcf5\uc218\u300b(1989)\uc5d0\uc11c \ud328\ud2b8\ub9ad \uc2a4\uc6e8\uc774\uc9c0\uc640 \ud568\uaed8 \uc8fc\uc5f0 \uc5ed\ud560\uc744 \ub9e1\uc558\ub2e4. \ub2c8\uc2a8\uc740 \ubbf8\uad6d \uc544\uce74\ub370\ubbf8\uc0c1 \uc791\ud488\uc0c1\uc744 \uc218\uc0c1\ud55c \uc2a4\ud2f0\ube10 \uc2a4\ud544\ubc84\uadf8 \uac10\ub3c5\uc758 \uc601\ud654 \u300a\uc270\ub4e4\ub7ec \ub9ac\uc2a4\ud2b8\u300b(1993)\uc5d0\uc11c\uc758 \uc8fc\uc5f0 \uc5ed\ud560\ub85c \uc138\uacc4\uc801\uc778 \uba85\uc131\uc744 \uc5bb\uc5c8\ub2e4.", "answer": "\ubd81\uc544\uc77c\ub79c\ub4dc", "sentence": "\ub9ac\uc554 \uc874 \ub2c8\uc2a8(\uc601\uc5b4: Liam John Neeson, OBE, 1952\ub144 6\uc6d4 7\uc77c ~ )\uc740 \ubd81\uc544\uc77c\ub79c\ub4dc \uc758 \ubc30\uc6b0\uc774\ub2e4.", "paragraph_sentence": " \ub9ac\uc554 \uc874 \ub2c8\uc2a8(\uc601\uc5b4: Liam John Neeson, OBE, 1952\ub144 6\uc6d4 7\uc77c ~ )\uc740 \ubd81\uc544\uc77c\ub79c\ub4dc \uc758 \ubc30\uc6b0\uc774\ub2e4. 1976\ub144 \uadf8\ub294 2\ub144\uac04 \ubca8\ud30c\uc2a4\ud2b8\uc758 \ub9ac\ub9ad \ud50c\ub808\uc774\uc5b4\uc2a4 \uadf9\uc7a5\uc5d0\uc11c \ud65c\ub3d9\ud588\ub2e4. \uadf8\ub294 \uc544\ub354\uc655\uc758 \uc601\ud654 \u300a\uc5d1\uc2a4\uce7c\ub9ac\ubc84\u300b(1981)\uc5d0 \ucd9c\uc5f0\ud588\ub2e4. 1982\ub144\ubd80\ud130 1987\ub144\uae4c\uc9c0 \ub2c8\uc2a8\uc740 \u300a\ubc14\uc6b4\ud2f0 \ud638\uc758 \ubc18\ub780\u300b(1984)\uc5d0\uc11c \uba5c \uae41\uc2a8, \uc548\uc18c\ub2c8 \ud649\ud0a8\uc2a4, \u300a\ubbf8\uc158\u300b(1986)\uc5d0\uc11c \ub85c\ubc84\ud2b8 \ub4dc \ub2c8\ub85c, \uc81c\ub808\ubbf8 \uc544\uc774\uc5b8\uc2a4\uc640 \ud568\uaed8 5\ud3b8\uc758 \uc601\ud654\uc5d0 \ucd9c\uc5f0\ud588\ub2e4. \uadf8\ub294 \u300a\ubd88\ud0c0\ub294 \ubcf5\uc218\u300b(1989)\uc5d0\uc11c \ud328\ud2b8\ub9ad \uc2a4\uc6e8\uc774\uc9c0\uc640 \ud568\uaed8 \uc8fc\uc5f0 \uc5ed\ud560\uc744 \ub9e1\uc558\ub2e4. \ub2c8\uc2a8\uc740 \ubbf8\uad6d \uc544\uce74\ub370\ubbf8\uc0c1 \uc791\ud488\uc0c1\uc744 \uc218\uc0c1\ud55c \uc2a4\ud2f0\ube10 \uc2a4\ud544\ubc84\uadf8 \uac10\ub3c5\uc758 \uc601\ud654 \u300a\uc270\ub4e4\ub7ec \ub9ac\uc2a4\ud2b8\u300b(1993)\uc5d0\uc11c\uc758 \uc8fc\uc5f0 \uc5ed\ud560\ub85c \uc138\uacc4\uc801\uc778 \uba85\uc131\uc744 \uc5bb\uc5c8\ub2e4.", "paragraph_answer": "\ub9ac\uc554 \uc874 \ub2c8\uc2a8(\uc601\uc5b4: Liam John Neeson, OBE, 1952\ub144 6\uc6d4 7\uc77c ~ )\uc740 \ubd81\uc544\uc77c\ub79c\ub4dc \uc758 \ubc30\uc6b0\uc774\ub2e4. 1976\ub144 \uadf8\ub294 2\ub144\uac04 \ubca8\ud30c\uc2a4\ud2b8\uc758 \ub9ac\ub9ad \ud50c\ub808\uc774\uc5b4\uc2a4 \uadf9\uc7a5\uc5d0\uc11c \ud65c\ub3d9\ud588\ub2e4. \uadf8\ub294 \uc544\ub354\uc655\uc758 \uc601\ud654 \u300a\uc5d1\uc2a4\uce7c\ub9ac\ubc84\u300b(1981)\uc5d0 \ucd9c\uc5f0\ud588\ub2e4. 1982\ub144\ubd80\ud130 1987\ub144\uae4c\uc9c0 \ub2c8\uc2a8\uc740 \u300a\ubc14\uc6b4\ud2f0 \ud638\uc758 \ubc18\ub780\u300b(1984)\uc5d0\uc11c \uba5c \uae41\uc2a8, \uc548\uc18c\ub2c8 \ud649\ud0a8\uc2a4, \u300a\ubbf8\uc158\u300b(1986)\uc5d0\uc11c \ub85c\ubc84\ud2b8 \ub4dc \ub2c8\ub85c, \uc81c\ub808\ubbf8 \uc544\uc774\uc5b8\uc2a4\uc640 \ud568\uaed8 5\ud3b8\uc758 \uc601\ud654\uc5d0 \ucd9c\uc5f0\ud588\ub2e4. \uadf8\ub294 \u300a\ubd88\ud0c0\ub294 \ubcf5\uc218\u300b(1989)\uc5d0\uc11c \ud328\ud2b8\ub9ad \uc2a4\uc6e8\uc774\uc9c0\uc640 \ud568\uaed8 \uc8fc\uc5f0 \uc5ed\ud560\uc744 \ub9e1\uc558\ub2e4. \ub2c8\uc2a8\uc740 \ubbf8\uad6d \uc544\uce74\ub370\ubbf8\uc0c1 \uc791\ud488\uc0c1\uc744 \uc218\uc0c1\ud55c \uc2a4\ud2f0\ube10 \uc2a4\ud544\ubc84\uadf8 \uac10\ub3c5\uc758 \uc601\ud654 \u300a\uc270\ub4e4\ub7ec \ub9ac\uc2a4\ud2b8\u300b(1993)\uc5d0\uc11c\uc758 \uc8fc\uc5f0 \uc5ed\ud560\ub85c \uc138\uacc4\uc801\uc778 \uba85\uc131\uc744 \uc5bb\uc5c8\ub2e4.", "sentence_answer": "\ub9ac\uc554 \uc874 \ub2c8\uc2a8(\uc601\uc5b4: Liam John Neeson, OBE, 1952\ub144 6\uc6d4 7\uc77c ~ )\uc740 \ubd81\uc544\uc77c\ub79c\ub4dc \uc758 \ubc30\uc6b0\uc774\ub2e4.", "paragraph_id": "6498397-0-0", "paragraph_question": "question: \ub9ac\uc554 \ub2c8\uc2a8\uc740 \uc5b4\ub290\ub098\ub77c \ubc30\uc6b0\uc778\uac00?, context: \ub9ac\uc554 \uc874 \ub2c8\uc2a8(\uc601\uc5b4: Liam John Neeson, OBE, 1952\ub144 6\uc6d4 7\uc77c ~ )\uc740 \ubd81\uc544\uc77c\ub79c\ub4dc\uc758 \ubc30\uc6b0\uc774\ub2e4. 1976\ub144 \uadf8\ub294 2\ub144\uac04 \ubca8\ud30c\uc2a4\ud2b8\uc758 \ub9ac\ub9ad \ud50c\ub808\uc774\uc5b4\uc2a4 \uadf9\uc7a5\uc5d0\uc11c \ud65c\ub3d9\ud588\ub2e4. \uadf8\ub294 \uc544\ub354\uc655\uc758 \uc601\ud654 \u300a\uc5d1\uc2a4\uce7c\ub9ac\ubc84\u300b(1981)\uc5d0 \ucd9c\uc5f0\ud588\ub2e4. 1982\ub144\ubd80\ud130 1987\ub144\uae4c\uc9c0 \ub2c8\uc2a8\uc740 \u300a\ubc14\uc6b4\ud2f0 \ud638\uc758 \ubc18\ub780\u300b(1984)\uc5d0\uc11c \uba5c \uae41\uc2a8, \uc548\uc18c\ub2c8 \ud649\ud0a8\uc2a4, \u300a\ubbf8\uc158\u300b(1986)\uc5d0\uc11c \ub85c\ubc84\ud2b8 \ub4dc \ub2c8\ub85c, \uc81c\ub808\ubbf8 \uc544\uc774\uc5b8\uc2a4\uc640 \ud568\uaed8 5\ud3b8\uc758 \uc601\ud654\uc5d0 \ucd9c\uc5f0\ud588\ub2e4. \uadf8\ub294 \u300a\ubd88\ud0c0\ub294 \ubcf5\uc218\u300b(1989)\uc5d0\uc11c \ud328\ud2b8\ub9ad \uc2a4\uc6e8\uc774\uc9c0\uc640 \ud568\uaed8 \uc8fc\uc5f0 \uc5ed\ud560\uc744 \ub9e1\uc558\ub2e4. \ub2c8\uc2a8\uc740 \ubbf8\uad6d \uc544\uce74\ub370\ubbf8\uc0c1 \uc791\ud488\uc0c1\uc744 \uc218\uc0c1\ud55c \uc2a4\ud2f0\ube10 \uc2a4\ud544\ubc84\uadf8 \uac10\ub3c5\uc758 \uc601\ud654 \u300a\uc270\ub4e4\ub7ec \ub9ac\uc2a4\ud2b8\u300b(1993)\uc5d0\uc11c\uc758 \uc8fc\uc5f0 \uc5ed\ud560\ub85c \uc138\uacc4\uc801\uc778 \uba85\uc131\uc744 \uc5bb\uc5c8\ub2e4."} {"question": "\uc18c\ub144\uc57c\uad6c\ub300\ud68c \uac1c\ucd5c \uc9c0\uc6d0\uacfc \uc57c\uad6c\uc6a9\ud488\uc774 \ubd80\uc871\ud55c \uc544\uc774\ub4e4\uc5d0\uac8c \uc6a9\ud488\uae30\uc99d\uc744 \uc5b4\ub5a4 \uc774\ub984\ud558\uc5d0 \uc9c4\ud589\ud558\uc600\ub294\uac00?", "paragraph": "\uc77c\ubcf8 \uc804\uad6d \uac01\uc9c0\uc5d0\uc11c \uc9c0\uc790\uccb4, \uad50\uc721\uc704\uc6d0\ud68c, \uc18c\ub144 \uc57c\uad6c\ud300, \uae30\uc5c5 \ub4f1\uc5d0\uc11c\uc758 \uc758\ub8b0\uc5d0 \ub530\ub974\uace0 \uc6b0\uc120 \uc57c\uad6c\uc5d0 \ud765\ubbf8\ub97c \uac16\uac8c\ud558\uae30 \uc704\ud574 \uc57c\uad6c\uc5d0 \ub300\ud55c \uce5c\uadfc\uac10\uc744 \ub290\ub07c\ub3c4\ub85d \uccb4\ud5d8\ud558\ub294 \u2018\uba85\uad6c\ud68c \ubca0\uc774\uc2a4\ubcfc \ud398\uc2a4\ud2f0\ubc8c\u2019\uc744 \uac1c\ucd5c\ud558\uace0 \uc788\ub2e4. \uac70\uae30\uc5d0\uc120 \ud68c\uc6d0\uc774 \uccb4\ud5d8 \uc9c0\ub3c4\ud558\ub294 \ud22c\uad6c, \ud0c0\uaca9, \uc218\ube44\uc5d0 \ub300\ud574 \uc9c4\uc9c0\ud558\uac8c \ub9c8\uc8fc\ud558\ub294 \uc5b4\ub9b0\uc774\uc640 \ub354\ubd88\uc5b4 \uc751\uc6d0\ud558\ub7ec \uc628 \ud615\uc81c, \ubd80\ubaa8, \uc870\ubd80\ubaa8\uac00 \uac01\uc790 \uc2dc\ub300\uc758 \ud788\uc5b4\ub85c\uc640\uc758 \ub9cc\ub0a8\uc744 \ud1b5\ud574 \uc990\uae30\uac8c\ub054 \ub9cc\ub4e4\uace0 \uc788\ub2e4. \uadf8\ub9ac\uace0 \uc57c\uad6c\ub97c \uc798\ud558\uace0 \uc2f6\uc5b4\ud558\ub294 \uc5b4\ub9b0\uc774\uc5d0\uac8c\ub294 \ud68c\uc6d0\uc774 \uc9c1\uc811 \uc9c0\ub3c4\ud558\ub294 \u2018\uba85\uad6c\ud68c \uc57c\uad6c \uad50\uc2e4\u2019\ub3c4 \uc2e4\uc2dc\ud558\uace0 \uc788\ub2e4. \uc5ec\uae30\uc11c \ud22c\uad6c, \ud0c0\uaca9, \uc218\ube44 \uc804\ubb38\uac00\uac00 \uc5b4\ub9b0\uc774 \ud55c \uc0ac\ub78c \ud55c \uc0ac\ub78c\uc758 \uc18c\ub9ac\ub97c \ub4e3\uace0 \ucee4\ubba4\ub2c8\ucf00\uc774\uc158\uc744 \ucde8\ud558\uba74\uc11c \uc9c0\ub3c4\ud558\uace0 \uc788\ub2e4. \ucd5c\uadfc\uc5d0\ub294 \uae00\ub85c\ubc8c\ud654 \ub418\uba74\uc11c \ud574\uc678\uc5d0\uc11c\ub3c4 \uc57c\uad6c\ub97c \uc990\uae30\uba70 \ubcf4\uae09 \ud65c\ub3d9\uc744 \ud558\uace0 \uc788\ub294 \uc0ac\ub78c\ub4e4\uc758 \uc758\ub8b0\ub3c4 \uc788\uc5b4 \ud604\uc7ac \uc544\uc2dc\uc544 \uc5ec\ub7ec \ub098\ub77c(\ub300\ud55c\ubbfc\uad6d, \uc911\ud654\ubbfc\uad6d, \ud544\ub9ac\ud540)\uc758 \ud68c\uc6d0\ub4e4\uc744 \uc704\ud574 \ud604\uc9c0\uc758 \uc5b4\ub9b0\uc774\ub4e4\uc744 \uc9c0\ub3c4\ud558\uace0 \uc9c0\uc6d0 \ud589\uc0ac\ub3c4 \ud558\uace0 \uc788\ub2e4. \uc774 \uc678\uc5d0\ub3c4 \u2018\uba85\uad6c\ud68c\ubc30\u2019\ub77c\ub294 \uc774\ub984\uc774 \ub4e4\uc5b4\uac04 \uc18c\ub144\uc57c\uad6c\ub300\ud68c\uc758 \uac1c\ucd5c \uc9c0\uc6d0\uacfc \uc57c\uad6c\uc6a9\ud488\uc774 \ubd80\uc871\ud55c \uc9c0\uc5ed\uc758 \uc5b4\ub9b0\uc774\ub4e4\uc5d0\uac8c \uc6a9\ud488\uc744 \uae30\uc99d\ud558\uace0 \uc788\uc73c\uba70 \ub610\ud55c \uadf8\ub77c\uc6b4\ub4dc \uc774\uc678\uc5d0\uc11c\ub3c4 \uc5b4\ub9b0\uc774 \ud559\uc0dd, \uc18c\ub144 \uc57c\uad6c\ud300, \uc18c\ub144 \uad50\ub3c4\uc18c \uc7ac\uc18c\uc790\ub4e4\uc5d0\uac8c \ud68c\uc6d0\uc774 \uc9c1\uc811 \uc790\uc2e0\uc758 \uacbd\ud5d8\uc744 \uac15\uc5f0\ud558\ub294 \ub4f1\uc758 \ud65c\ub3d9\ub3c4 \ud558\uace0 \uc788\ub2e4.", "answer": "\uba85\uad6c\ud68c\ubc30", "sentence": "\uc774 \uc678\uc5d0\ub3c4 \u2018 \uba85\uad6c\ud68c\ubc30 \u2019\ub77c\ub294 \uc774\ub984\uc774 \ub4e4\uc5b4\uac04 \uc18c\ub144\uc57c\uad6c\ub300\ud68c\uc758 \uac1c\ucd5c \uc9c0\uc6d0\uacfc \uc57c\uad6c\uc6a9\ud488\uc774 \ubd80\uc871\ud55c \uc9c0\uc5ed\uc758 \uc5b4\ub9b0\uc774\ub4e4\uc5d0\uac8c \uc6a9\ud488\uc744 \uae30\uc99d\ud558\uace0 \uc788\uc73c\uba70 \ub610\ud55c \uadf8\ub77c\uc6b4\ub4dc \uc774\uc678\uc5d0\uc11c\ub3c4 \uc5b4\ub9b0\uc774 \ud559\uc0dd, \uc18c\ub144 \uc57c\uad6c\ud300, \uc18c\ub144 \uad50\ub3c4\uc18c \uc7ac\uc18c\uc790\ub4e4\uc5d0\uac8c \ud68c\uc6d0\uc774 \uc9c1\uc811 \uc790\uc2e0\uc758 \uacbd\ud5d8\uc744 \uac15\uc5f0\ud558\ub294 \ub4f1\uc758 \ud65c\ub3d9\ub3c4 \ud558\uace0 \uc788\ub2e4.", "paragraph_sentence": "\uc77c\ubcf8 \uc804\uad6d \uac01\uc9c0\uc5d0\uc11c \uc9c0\uc790\uccb4, \uad50\uc721\uc704\uc6d0\ud68c, \uc18c\ub144 \uc57c\uad6c\ud300, \uae30\uc5c5 \ub4f1\uc5d0\uc11c\uc758 \uc758\ub8b0\uc5d0 \ub530\ub974\uace0 \uc6b0\uc120 \uc57c\uad6c\uc5d0 \ud765\ubbf8\ub97c \uac16\uac8c\ud558\uae30 \uc704\ud574 \uc57c\uad6c\uc5d0 \ub300\ud55c \uce5c\uadfc\uac10\uc744 \ub290\ub07c\ub3c4\ub85d \uccb4\ud5d8\ud558\ub294 \u2018\uba85\uad6c\ud68c \ubca0\uc774\uc2a4\ubcfc \ud398\uc2a4\ud2f0\ubc8c\u2019\uc744 \uac1c\ucd5c\ud558\uace0 \uc788\ub2e4. \uac70\uae30\uc5d0\uc120 \ud68c\uc6d0\uc774 \uccb4\ud5d8 \uc9c0\ub3c4\ud558\ub294 \ud22c\uad6c, \ud0c0\uaca9, \uc218\ube44\uc5d0 \ub300\ud574 \uc9c4\uc9c0\ud558\uac8c \ub9c8\uc8fc\ud558\ub294 \uc5b4\ub9b0\uc774\uc640 \ub354\ubd88\uc5b4 \uc751\uc6d0\ud558\ub7ec \uc628 \ud615\uc81c, \ubd80\ubaa8, \uc870\ubd80\ubaa8\uac00 \uac01\uc790 \uc2dc\ub300\uc758 \ud788\uc5b4\ub85c\uc640\uc758 \ub9cc\ub0a8\uc744 \ud1b5\ud574 \uc990\uae30\uac8c\ub054 \ub9cc\ub4e4\uace0 \uc788\ub2e4. \uadf8\ub9ac\uace0 \uc57c\uad6c\ub97c \uc798\ud558\uace0 \uc2f6\uc5b4\ud558\ub294 \uc5b4\ub9b0\uc774\uc5d0\uac8c\ub294 \ud68c\uc6d0\uc774 \uc9c1\uc811 \uc9c0\ub3c4\ud558\ub294 \u2018\uba85\uad6c\ud68c \uc57c\uad6c \uad50\uc2e4\u2019\ub3c4 \uc2e4\uc2dc\ud558\uace0 \uc788\ub2e4. \uc5ec\uae30\uc11c \ud22c\uad6c, \ud0c0\uaca9, \uc218\ube44 \uc804\ubb38\uac00\uac00 \uc5b4\ub9b0\uc774 \ud55c \uc0ac\ub78c \ud55c \uc0ac\ub78c\uc758 \uc18c\ub9ac\ub97c \ub4e3\uace0 \ucee4\ubba4\ub2c8\ucf00\uc774\uc158\uc744 \ucde8\ud558\uba74\uc11c \uc9c0\ub3c4\ud558\uace0 \uc788\ub2e4. \ucd5c\uadfc\uc5d0\ub294 \uae00\ub85c\ubc8c\ud654 \ub418\uba74\uc11c \ud574\uc678\uc5d0\uc11c\ub3c4 \uc57c\uad6c\ub97c \uc990\uae30\uba70 \ubcf4\uae09 \ud65c\ub3d9\uc744 \ud558\uace0 \uc788\ub294 \uc0ac\ub78c\ub4e4\uc758 \uc758\ub8b0\ub3c4 \uc788\uc5b4 \ud604\uc7ac \uc544\uc2dc\uc544 \uc5ec\ub7ec \ub098\ub77c(\ub300\ud55c\ubbfc\uad6d, \uc911\ud654\ubbfc\uad6d, \ud544\ub9ac\ud540)\uc758 \ud68c\uc6d0\ub4e4\uc744 \uc704\ud574 \ud604\uc9c0\uc758 \uc5b4\ub9b0\uc774\ub4e4\uc744 \uc9c0\ub3c4\ud558\uace0 \uc9c0\uc6d0 \ud589\uc0ac\ub3c4 \ud558\uace0 \uc788\ub2e4. \uc774 \uc678\uc5d0\ub3c4 \u2018 \uba85\uad6c\ud68c\ubc30 \u2019\ub77c\ub294 \uc774\ub984\uc774 \ub4e4\uc5b4\uac04 \uc18c\ub144\uc57c\uad6c\ub300\ud68c\uc758 \uac1c\ucd5c \uc9c0\uc6d0\uacfc \uc57c\uad6c\uc6a9\ud488\uc774 \ubd80\uc871\ud55c \uc9c0\uc5ed\uc758 \uc5b4\ub9b0\uc774\ub4e4\uc5d0\uac8c \uc6a9\ud488\uc744 \uae30\uc99d\ud558\uace0 \uc788\uc73c\uba70 \ub610\ud55c \uadf8\ub77c\uc6b4\ub4dc \uc774\uc678\uc5d0\uc11c\ub3c4 \uc5b4\ub9b0\uc774 \ud559\uc0dd, \uc18c\ub144 \uc57c\uad6c\ud300, \uc18c\ub144 \uad50\ub3c4\uc18c \uc7ac\uc18c\uc790\ub4e4\uc5d0\uac8c \ud68c\uc6d0\uc774 \uc9c1\uc811 \uc790\uc2e0\uc758 \uacbd\ud5d8\uc744 \uac15\uc5f0\ud558\ub294 \ub4f1\uc758 \ud65c\ub3d9\ub3c4 \ud558\uace0 \uc788\ub2e4. ", "paragraph_answer": "\uc77c\ubcf8 \uc804\uad6d \uac01\uc9c0\uc5d0\uc11c \uc9c0\uc790\uccb4, \uad50\uc721\uc704\uc6d0\ud68c, \uc18c\ub144 \uc57c\uad6c\ud300, \uae30\uc5c5 \ub4f1\uc5d0\uc11c\uc758 \uc758\ub8b0\uc5d0 \ub530\ub974\uace0 \uc6b0\uc120 \uc57c\uad6c\uc5d0 \ud765\ubbf8\ub97c \uac16\uac8c\ud558\uae30 \uc704\ud574 \uc57c\uad6c\uc5d0 \ub300\ud55c \uce5c\uadfc\uac10\uc744 \ub290\ub07c\ub3c4\ub85d \uccb4\ud5d8\ud558\ub294 \u2018\uba85\uad6c\ud68c \ubca0\uc774\uc2a4\ubcfc \ud398\uc2a4\ud2f0\ubc8c\u2019\uc744 \uac1c\ucd5c\ud558\uace0 \uc788\ub2e4. \uac70\uae30\uc5d0\uc120 \ud68c\uc6d0\uc774 \uccb4\ud5d8 \uc9c0\ub3c4\ud558\ub294 \ud22c\uad6c, \ud0c0\uaca9, \uc218\ube44\uc5d0 \ub300\ud574 \uc9c4\uc9c0\ud558\uac8c \ub9c8\uc8fc\ud558\ub294 \uc5b4\ub9b0\uc774\uc640 \ub354\ubd88\uc5b4 \uc751\uc6d0\ud558\ub7ec \uc628 \ud615\uc81c, \ubd80\ubaa8, \uc870\ubd80\ubaa8\uac00 \uac01\uc790 \uc2dc\ub300\uc758 \ud788\uc5b4\ub85c\uc640\uc758 \ub9cc\ub0a8\uc744 \ud1b5\ud574 \uc990\uae30\uac8c\ub054 \ub9cc\ub4e4\uace0 \uc788\ub2e4. \uadf8\ub9ac\uace0 \uc57c\uad6c\ub97c \uc798\ud558\uace0 \uc2f6\uc5b4\ud558\ub294 \uc5b4\ub9b0\uc774\uc5d0\uac8c\ub294 \ud68c\uc6d0\uc774 \uc9c1\uc811 \uc9c0\ub3c4\ud558\ub294 \u2018\uba85\uad6c\ud68c \uc57c\uad6c \uad50\uc2e4\u2019\ub3c4 \uc2e4\uc2dc\ud558\uace0 \uc788\ub2e4. \uc5ec\uae30\uc11c \ud22c\uad6c, \ud0c0\uaca9, \uc218\ube44 \uc804\ubb38\uac00\uac00 \uc5b4\ub9b0\uc774 \ud55c \uc0ac\ub78c \ud55c \uc0ac\ub78c\uc758 \uc18c\ub9ac\ub97c \ub4e3\uace0 \ucee4\ubba4\ub2c8\ucf00\uc774\uc158\uc744 \ucde8\ud558\uba74\uc11c \uc9c0\ub3c4\ud558\uace0 \uc788\ub2e4. \ucd5c\uadfc\uc5d0\ub294 \uae00\ub85c\ubc8c\ud654 \ub418\uba74\uc11c \ud574\uc678\uc5d0\uc11c\ub3c4 \uc57c\uad6c\ub97c \uc990\uae30\uba70 \ubcf4\uae09 \ud65c\ub3d9\uc744 \ud558\uace0 \uc788\ub294 \uc0ac\ub78c\ub4e4\uc758 \uc758\ub8b0\ub3c4 \uc788\uc5b4 \ud604\uc7ac \uc544\uc2dc\uc544 \uc5ec\ub7ec \ub098\ub77c(\ub300\ud55c\ubbfc\uad6d, \uc911\ud654\ubbfc\uad6d, \ud544\ub9ac\ud540)\uc758 \ud68c\uc6d0\ub4e4\uc744 \uc704\ud574 \ud604\uc9c0\uc758 \uc5b4\ub9b0\uc774\ub4e4\uc744 \uc9c0\ub3c4\ud558\uace0 \uc9c0\uc6d0 \ud589\uc0ac\ub3c4 \ud558\uace0 \uc788\ub2e4. \uc774 \uc678\uc5d0\ub3c4 \u2018 \uba85\uad6c\ud68c\ubc30 \u2019\ub77c\ub294 \uc774\ub984\uc774 \ub4e4\uc5b4\uac04 \uc18c\ub144\uc57c\uad6c\ub300\ud68c\uc758 \uac1c\ucd5c \uc9c0\uc6d0\uacfc \uc57c\uad6c\uc6a9\ud488\uc774 \ubd80\uc871\ud55c \uc9c0\uc5ed\uc758 \uc5b4\ub9b0\uc774\ub4e4\uc5d0\uac8c \uc6a9\ud488\uc744 \uae30\uc99d\ud558\uace0 \uc788\uc73c\uba70 \ub610\ud55c \uadf8\ub77c\uc6b4\ub4dc \uc774\uc678\uc5d0\uc11c\ub3c4 \uc5b4\ub9b0\uc774 \ud559\uc0dd, \uc18c\ub144 \uc57c\uad6c\ud300, \uc18c\ub144 \uad50\ub3c4\uc18c \uc7ac\uc18c\uc790\ub4e4\uc5d0\uac8c \ud68c\uc6d0\uc774 \uc9c1\uc811 \uc790\uc2e0\uc758 \uacbd\ud5d8\uc744 \uac15\uc5f0\ud558\ub294 \ub4f1\uc758 \ud65c\ub3d9\ub3c4 \ud558\uace0 \uc788\ub2e4.", "sentence_answer": "\uc774 \uc678\uc5d0\ub3c4 \u2018 \uba85\uad6c\ud68c\ubc30 \u2019\ub77c\ub294 \uc774\ub984\uc774 \ub4e4\uc5b4\uac04 \uc18c\ub144\uc57c\uad6c\ub300\ud68c\uc758 \uac1c\ucd5c \uc9c0\uc6d0\uacfc \uc57c\uad6c\uc6a9\ud488\uc774 \ubd80\uc871\ud55c \uc9c0\uc5ed\uc758 \uc5b4\ub9b0\uc774\ub4e4\uc5d0\uac8c \uc6a9\ud488\uc744 \uae30\uc99d\ud558\uace0 \uc788\uc73c\uba70 \ub610\ud55c \uadf8\ub77c\uc6b4\ub4dc \uc774\uc678\uc5d0\uc11c\ub3c4 \uc5b4\ub9b0\uc774 \ud559\uc0dd, \uc18c\ub144 \uc57c\uad6c\ud300, \uc18c\ub144 \uad50\ub3c4\uc18c \uc7ac\uc18c\uc790\ub4e4\uc5d0\uac8c \ud68c\uc6d0\uc774 \uc9c1\uc811 \uc790\uc2e0\uc758 \uacbd\ud5d8\uc744 \uac15\uc5f0\ud558\ub294 \ub4f1\uc758 \ud65c\ub3d9\ub3c4 \ud558\uace0 \uc788\ub2e4.", "paragraph_id": "6591628-1-1", "paragraph_question": "question: \uc18c\ub144\uc57c\uad6c\ub300\ud68c \uac1c\ucd5c \uc9c0\uc6d0\uacfc \uc57c\uad6c\uc6a9\ud488\uc774 \ubd80\uc871\ud55c \uc544\uc774\ub4e4\uc5d0\uac8c \uc6a9\ud488\uae30\uc99d\uc744 \uc5b4\ub5a4 \uc774\ub984\ud558\uc5d0 \uc9c4\ud589\ud558\uc600\ub294\uac00?, context: \uc77c\ubcf8 \uc804\uad6d \uac01\uc9c0\uc5d0\uc11c \uc9c0\uc790\uccb4, \uad50\uc721\uc704\uc6d0\ud68c, \uc18c\ub144 \uc57c\uad6c\ud300, \uae30\uc5c5 \ub4f1\uc5d0\uc11c\uc758 \uc758\ub8b0\uc5d0 \ub530\ub974\uace0 \uc6b0\uc120 \uc57c\uad6c\uc5d0 \ud765\ubbf8\ub97c \uac16\uac8c\ud558\uae30 \uc704\ud574 \uc57c\uad6c\uc5d0 \ub300\ud55c \uce5c\uadfc\uac10\uc744 \ub290\ub07c\ub3c4\ub85d \uccb4\ud5d8\ud558\ub294 \u2018\uba85\uad6c\ud68c \ubca0\uc774\uc2a4\ubcfc \ud398\uc2a4\ud2f0\ubc8c\u2019\uc744 \uac1c\ucd5c\ud558\uace0 \uc788\ub2e4. \uac70\uae30\uc5d0\uc120 \ud68c\uc6d0\uc774 \uccb4\ud5d8 \uc9c0\ub3c4\ud558\ub294 \ud22c\uad6c, \ud0c0\uaca9, \uc218\ube44\uc5d0 \ub300\ud574 \uc9c4\uc9c0\ud558\uac8c \ub9c8\uc8fc\ud558\ub294 \uc5b4\ub9b0\uc774\uc640 \ub354\ubd88\uc5b4 \uc751\uc6d0\ud558\ub7ec \uc628 \ud615\uc81c, \ubd80\ubaa8, \uc870\ubd80\ubaa8\uac00 \uac01\uc790 \uc2dc\ub300\uc758 \ud788\uc5b4\ub85c\uc640\uc758 \ub9cc\ub0a8\uc744 \ud1b5\ud574 \uc990\uae30\uac8c\ub054 \ub9cc\ub4e4\uace0 \uc788\ub2e4. \uadf8\ub9ac\uace0 \uc57c\uad6c\ub97c \uc798\ud558\uace0 \uc2f6\uc5b4\ud558\ub294 \uc5b4\ub9b0\uc774\uc5d0\uac8c\ub294 \ud68c\uc6d0\uc774 \uc9c1\uc811 \uc9c0\ub3c4\ud558\ub294 \u2018\uba85\uad6c\ud68c \uc57c\uad6c \uad50\uc2e4\u2019\ub3c4 \uc2e4\uc2dc\ud558\uace0 \uc788\ub2e4. \uc5ec\uae30\uc11c \ud22c\uad6c, \ud0c0\uaca9, \uc218\ube44 \uc804\ubb38\uac00\uac00 \uc5b4\ub9b0\uc774 \ud55c \uc0ac\ub78c \ud55c \uc0ac\ub78c\uc758 \uc18c\ub9ac\ub97c \ub4e3\uace0 \ucee4\ubba4\ub2c8\ucf00\uc774\uc158\uc744 \ucde8\ud558\uba74\uc11c \uc9c0\ub3c4\ud558\uace0 \uc788\ub2e4. \ucd5c\uadfc\uc5d0\ub294 \uae00\ub85c\ubc8c\ud654 \ub418\uba74\uc11c \ud574\uc678\uc5d0\uc11c\ub3c4 \uc57c\uad6c\ub97c \uc990\uae30\uba70 \ubcf4\uae09 \ud65c\ub3d9\uc744 \ud558\uace0 \uc788\ub294 \uc0ac\ub78c\ub4e4\uc758 \uc758\ub8b0\ub3c4 \uc788\uc5b4 \ud604\uc7ac \uc544\uc2dc\uc544 \uc5ec\ub7ec \ub098\ub77c(\ub300\ud55c\ubbfc\uad6d, \uc911\ud654\ubbfc\uad6d, \ud544\ub9ac\ud540)\uc758 \ud68c\uc6d0\ub4e4\uc744 \uc704\ud574 \ud604\uc9c0\uc758 \uc5b4\ub9b0\uc774\ub4e4\uc744 \uc9c0\ub3c4\ud558\uace0 \uc9c0\uc6d0 \ud589\uc0ac\ub3c4 \ud558\uace0 \uc788\ub2e4. \uc774 \uc678\uc5d0\ub3c4 \u2018\uba85\uad6c\ud68c\ubc30\u2019\ub77c\ub294 \uc774\ub984\uc774 \ub4e4\uc5b4\uac04 \uc18c\ub144\uc57c\uad6c\ub300\ud68c\uc758 \uac1c\ucd5c \uc9c0\uc6d0\uacfc \uc57c\uad6c\uc6a9\ud488\uc774 \ubd80\uc871\ud55c \uc9c0\uc5ed\uc758 \uc5b4\ub9b0\uc774\ub4e4\uc5d0\uac8c \uc6a9\ud488\uc744 \uae30\uc99d\ud558\uace0 \uc788\uc73c\uba70 \ub610\ud55c \uadf8\ub77c\uc6b4\ub4dc \uc774\uc678\uc5d0\uc11c\ub3c4 \uc5b4\ub9b0\uc774 \ud559\uc0dd, \uc18c\ub144 \uc57c\uad6c\ud300, \uc18c\ub144 \uad50\ub3c4\uc18c \uc7ac\uc18c\uc790\ub4e4\uc5d0\uac8c \ud68c\uc6d0\uc774 \uc9c1\uc811 \uc790\uc2e0\uc758 \uacbd\ud5d8\uc744 \uac15\uc5f0\ud558\ub294 \ub4f1\uc758 \ud65c\ub3d9\ub3c4 \ud558\uace0 \uc788\ub2e4."} {"question": "\uac00\ub124\ucf54\uc758 2008\ub144\ubd80\ud130 \uacc4\uc18d\ub418\uace0 \uc788\ub358 \uc5f0\uc18d \ub450 \uc790\ub9bf\uc218 \uc2b9\ub9ac\ub294 \uba87 \ub144 \ub9cc\uc5d0 \uba48\ucdc4\ub294\uac00?", "paragraph": "\uad50\ub958\uc804\uc778 \uc694\ubbf8\uc6b0\ub9ac \uc790\uc774\uc5b8\uce20\uc804\uc5d0\uc11c\ub294 2\uacbd\uae30\uc5d0 \ub4f1\ud310\ud558\uba74\uc11c \ud328\uc804 \ud22c\uc218\uac00 \ub418\uc5c8\uc9c0\ub9cc \ud55c\uc2e0 \ud0c0\uc774\uac70\uc2a4\uc804\uc5d0\uc11c\ub294 \uc644\ubd09 \uc2b9\ub9ac\ub97c \ud3ec\ud568\ud55c 2\uc804 2\uc2b9\uc744 \uae30\ub85d\ud558\ub294 \ub4f1 \ub300\uc870\uc801\uc778 \ubaa8\uc2b5\uc744 \ubcf4\uc600\ub2e4. \uadf8\ub7ec\ub098 6\uc6d4 22\uc77c \uc5f0\uc2b5 \ub3c4\uc911\uc5d0 \uc624\ub978\ucabd \ud314\uafc8\uce58 \ud1b5\uc99d\uc73c\ub85c \ub2e4\uc74c\ub0a0 \uc138\uc774\ubd80\uc804\uc5d0\uc11c\uc758 \uc120\ubc1c \ub4f1\ud310\uc744 \ud560 \uc218 \uc5c6\uc5c8\uace0 \ubcd1\uc6d0\uc5d0\uc11c \uc815\ubc00 \uac80\uc0ac\ub97c \ubc1b\uc740 \uacb0\uacfc \ud314\uafc8\uce58\uc5d0 \uc5fc\uc99d\uc774 \uc788\ub294 \uac83\uc73c\ub85c \ubc1d\ud600\uc838 \uc7a5\uae30 \uc774\ud0c8\uc744 \ud53c\ud560 \uc218 \uc5c6\uac8c \ub418\uc5c8\ub2e4. \uadf8 \ud6c4 2\uad70 \uacbd\uae30\uc5d0\uc11c\ub294 2\ucc28\ub840\uc758 \ub4f1\ud310\uc744 \uac70\uccd0 \uc2dc\uc98c \uc885\ub8cc \uc9c1\uc804\uc778 10\uc6d4 5\uc77c\uc5d0\ub294 1\uad70\uc5d0 \ubcf5\uadc0, \uac19\uc740 \ub0a0 \uc18c\ud504\ud2b8\ubc45\ud06c\uc804\uc5d0\uc11c\ub294 7\uc774\ub2dd \ubb34\uc2e4\uc810\uc758 \ud638\ud22c\ub85c \uc2b9\ub9ac \ud22c\uc218\uac00 \ub418\uc5c8\ub2e4. \uc2dc\uc98c\uc5d0\uc11c\ub294 \uac70\ub4ed\ub418\ub294 \ubd80\uc0c1\uc758 \uc601\ud5a5\uc73c\ub85c 9\uacbd\uae30\uc5d0 \ub4f1\ud310\ud574 4\uc2b9 3\ud328\uc758 \uc131\uc801\uc73c\ub85c \ub05d\ub098 2008\ub144\ubd80\ud130 \uacc4\uc18d\ub418\uace0 \uc788\ub358 \uc5f0\uc18d \ub450 \uc790\ub9bf\uc218 \uc2b9\ub9ac\ub294 4\ub144 \ub9cc\uc5d0 \uba48\ucdc4\uace0 \ud300\uc740 \uac00\ub124\ucf54 \uc678\uc5d0\ub3c4 \uc8fc\ub825 \uc120\uc218\ub4e4\uc758 \uc787\ub2e8 \ubd80\uc0c1\uacfc \ubd80\uc9c4\uc73c\ub85c \uc545\uc7ac\uac00 \uacb9\uce58\uba74\uc11c \uacb0\uad6d \ucd5c\ud558\uc704\ub85c \ub5a8\uc5b4\uc9c0\ub294 \uc218\ubaa8\ub97c \uacaa\uc5b4\uc57c\ub9cc \ud588\ub2e4.", "answer": "4\ub144", "sentence": "\uc2dc\uc98c\uc5d0\uc11c\ub294 \uac70\ub4ed\ub418\ub294 \ubd80\uc0c1\uc758 \uc601\ud5a5\uc73c\ub85c 9\uacbd\uae30\uc5d0 \ub4f1\ud310\ud574 4\uc2b9 3\ud328\uc758 \uc131\uc801\uc73c\ub85c \ub05d\ub098 2008\ub144\ubd80\ud130 \uacc4\uc18d\ub418\uace0 \uc788\ub358 \uc5f0\uc18d \ub450 \uc790\ub9bf\uc218 \uc2b9\ub9ac\ub294 4\ub144 \ub9cc\uc5d0 \uba48\ucdc4\uace0 \ud300\uc740 \uac00\ub124\ucf54 \uc678\uc5d0\ub3c4 \uc8fc\ub825 \uc120\uc218\ub4e4\uc758 \uc787\ub2e8 \ubd80\uc0c1\uacfc \ubd80\uc9c4\uc73c\ub85c \uc545\uc7ac\uac00 \uacb9\uce58\uba74\uc11c \uacb0\uad6d \ucd5c\ud558\uc704\ub85c \ub5a8\uc5b4\uc9c0\ub294 \uc218\ubaa8\ub97c \uacaa\uc5b4\uc57c\ub9cc \ud588\ub2e4.", "paragraph_sentence": "\uad50\ub958\uc804\uc778 \uc694\ubbf8\uc6b0\ub9ac \uc790\uc774\uc5b8\uce20\uc804\uc5d0\uc11c\ub294 2\uacbd\uae30\uc5d0 \ub4f1\ud310\ud558\uba74\uc11c \ud328\uc804 \ud22c\uc218\uac00 \ub418\uc5c8\uc9c0\ub9cc \ud55c\uc2e0 \ud0c0\uc774\uac70\uc2a4\uc804\uc5d0\uc11c\ub294 \uc644\ubd09 \uc2b9\ub9ac\ub97c \ud3ec\ud568\ud55c 2\uc804 2\uc2b9\uc744 \uae30\ub85d\ud558\ub294 \ub4f1 \ub300\uc870\uc801\uc778 \ubaa8\uc2b5\uc744 \ubcf4\uc600\ub2e4. \uadf8\ub7ec\ub098 6\uc6d4 22\uc77c \uc5f0\uc2b5 \ub3c4\uc911\uc5d0 \uc624\ub978\ucabd \ud314\uafc8\uce58 \ud1b5\uc99d\uc73c\ub85c \ub2e4\uc74c\ub0a0 \uc138\uc774\ubd80\uc804\uc5d0\uc11c\uc758 \uc120\ubc1c \ub4f1\ud310\uc744 \ud560 \uc218 \uc5c6\uc5c8\uace0 \ubcd1\uc6d0\uc5d0\uc11c \uc815\ubc00 \uac80\uc0ac\ub97c \ubc1b\uc740 \uacb0\uacfc \ud314\uafc8\uce58\uc5d0 \uc5fc\uc99d\uc774 \uc788\ub294 \uac83\uc73c\ub85c \ubc1d\ud600\uc838 \uc7a5\uae30 \uc774\ud0c8\uc744 \ud53c\ud560 \uc218 \uc5c6\uac8c \ub418\uc5c8\ub2e4. \uadf8 \ud6c4 2\uad70 \uacbd\uae30\uc5d0\uc11c\ub294 2\ucc28\ub840\uc758 \ub4f1\ud310\uc744 \uac70\uccd0 \uc2dc\uc98c \uc885\ub8cc \uc9c1\uc804\uc778 10\uc6d4 5\uc77c\uc5d0\ub294 1\uad70\uc5d0 \ubcf5\uadc0, \uac19\uc740 \ub0a0 \uc18c\ud504\ud2b8\ubc45\ud06c\uc804\uc5d0\uc11c\ub294 7\uc774\ub2dd \ubb34\uc2e4\uc810\uc758 \ud638\ud22c\ub85c \uc2b9\ub9ac \ud22c\uc218\uac00 \ub418\uc5c8\ub2e4. \uc2dc\uc98c\uc5d0\uc11c\ub294 \uac70\ub4ed\ub418\ub294 \ubd80\uc0c1\uc758 \uc601\ud5a5\uc73c\ub85c 9\uacbd\uae30\uc5d0 \ub4f1\ud310\ud574 4\uc2b9 3\ud328\uc758 \uc131\uc801\uc73c\ub85c \ub05d\ub098 2008\ub144\ubd80\ud130 \uacc4\uc18d\ub418\uace0 \uc788\ub358 \uc5f0\uc18d \ub450 \uc790\ub9bf\uc218 \uc2b9\ub9ac\ub294 4\ub144 \ub9cc\uc5d0 \uba48\ucdc4\uace0 \ud300\uc740 \uac00\ub124\ucf54 \uc678\uc5d0\ub3c4 \uc8fc\ub825 \uc120\uc218\ub4e4\uc758 \uc787\ub2e8 \ubd80\uc0c1\uacfc \ubd80\uc9c4\uc73c\ub85c \uc545\uc7ac\uac00 \uacb9\uce58\uba74\uc11c \uacb0\uad6d \ucd5c\ud558\uc704\ub85c \ub5a8\uc5b4\uc9c0\ub294 \uc218\ubaa8\ub97c \uacaa\uc5b4\uc57c\ub9cc \ud588\ub2e4. ", "paragraph_answer": "\uad50\ub958\uc804\uc778 \uc694\ubbf8\uc6b0\ub9ac \uc790\uc774\uc5b8\uce20\uc804\uc5d0\uc11c\ub294 2\uacbd\uae30\uc5d0 \ub4f1\ud310\ud558\uba74\uc11c \ud328\uc804 \ud22c\uc218\uac00 \ub418\uc5c8\uc9c0\ub9cc \ud55c\uc2e0 \ud0c0\uc774\uac70\uc2a4\uc804\uc5d0\uc11c\ub294 \uc644\ubd09 \uc2b9\ub9ac\ub97c \ud3ec\ud568\ud55c 2\uc804 2\uc2b9\uc744 \uae30\ub85d\ud558\ub294 \ub4f1 \ub300\uc870\uc801\uc778 \ubaa8\uc2b5\uc744 \ubcf4\uc600\ub2e4. \uadf8\ub7ec\ub098 6\uc6d4 22\uc77c \uc5f0\uc2b5 \ub3c4\uc911\uc5d0 \uc624\ub978\ucabd \ud314\uafc8\uce58 \ud1b5\uc99d\uc73c\ub85c \ub2e4\uc74c\ub0a0 \uc138\uc774\ubd80\uc804\uc5d0\uc11c\uc758 \uc120\ubc1c \ub4f1\ud310\uc744 \ud560 \uc218 \uc5c6\uc5c8\uace0 \ubcd1\uc6d0\uc5d0\uc11c \uc815\ubc00 \uac80\uc0ac\ub97c \ubc1b\uc740 \uacb0\uacfc \ud314\uafc8\uce58\uc5d0 \uc5fc\uc99d\uc774 \uc788\ub294 \uac83\uc73c\ub85c \ubc1d\ud600\uc838 \uc7a5\uae30 \uc774\ud0c8\uc744 \ud53c\ud560 \uc218 \uc5c6\uac8c \ub418\uc5c8\ub2e4. \uadf8 \ud6c4 2\uad70 \uacbd\uae30\uc5d0\uc11c\ub294 2\ucc28\ub840\uc758 \ub4f1\ud310\uc744 \uac70\uccd0 \uc2dc\uc98c \uc885\ub8cc \uc9c1\uc804\uc778 10\uc6d4 5\uc77c\uc5d0\ub294 1\uad70\uc5d0 \ubcf5\uadc0, \uac19\uc740 \ub0a0 \uc18c\ud504\ud2b8\ubc45\ud06c\uc804\uc5d0\uc11c\ub294 7\uc774\ub2dd \ubb34\uc2e4\uc810\uc758 \ud638\ud22c\ub85c \uc2b9\ub9ac \ud22c\uc218\uac00 \ub418\uc5c8\ub2e4. \uc2dc\uc98c\uc5d0\uc11c\ub294 \uac70\ub4ed\ub418\ub294 \ubd80\uc0c1\uc758 \uc601\ud5a5\uc73c\ub85c 9\uacbd\uae30\uc5d0 \ub4f1\ud310\ud574 4\uc2b9 3\ud328\uc758 \uc131\uc801\uc73c\ub85c \ub05d\ub098 2008\ub144\ubd80\ud130 \uacc4\uc18d\ub418\uace0 \uc788\ub358 \uc5f0\uc18d \ub450 \uc790\ub9bf\uc218 \uc2b9\ub9ac\ub294 4\ub144 \ub9cc\uc5d0 \uba48\ucdc4\uace0 \ud300\uc740 \uac00\ub124\ucf54 \uc678\uc5d0\ub3c4 \uc8fc\ub825 \uc120\uc218\ub4e4\uc758 \uc787\ub2e8 \ubd80\uc0c1\uacfc \ubd80\uc9c4\uc73c\ub85c \uc545\uc7ac\uac00 \uacb9\uce58\uba74\uc11c \uacb0\uad6d \ucd5c\ud558\uc704\ub85c \ub5a8\uc5b4\uc9c0\ub294 \uc218\ubaa8\ub97c \uacaa\uc5b4\uc57c\ub9cc \ud588\ub2e4.", "sentence_answer": "\uc2dc\uc98c\uc5d0\uc11c\ub294 \uac70\ub4ed\ub418\ub294 \ubd80\uc0c1\uc758 \uc601\ud5a5\uc73c\ub85c 9\uacbd\uae30\uc5d0 \ub4f1\ud310\ud574 4\uc2b9 3\ud328\uc758 \uc131\uc801\uc73c\ub85c \ub05d\ub098 2008\ub144\ubd80\ud130 \uacc4\uc18d\ub418\uace0 \uc788\ub358 \uc5f0\uc18d \ub450 \uc790\ub9bf\uc218 \uc2b9\ub9ac\ub294 4\ub144 \ub9cc\uc5d0 \uba48\ucdc4\uace0 \ud300\uc740 \uac00\ub124\ucf54 \uc678\uc5d0\ub3c4 \uc8fc\ub825 \uc120\uc218\ub4e4\uc758 \uc787\ub2e8 \ubd80\uc0c1\uacfc \ubd80\uc9c4\uc73c\ub85c \uc545\uc7ac\uac00 \uacb9\uce58\uba74\uc11c \uacb0\uad6d \ucd5c\ud558\uc704\ub85c \ub5a8\uc5b4\uc9c0\ub294 \uc218\ubaa8\ub97c \uacaa\uc5b4\uc57c\ub9cc \ud588\ub2e4.", "paragraph_id": "6554213-3-1", "paragraph_question": "question: \uac00\ub124\ucf54\uc758 2008\ub144\ubd80\ud130 \uacc4\uc18d\ub418\uace0 \uc788\ub358 \uc5f0\uc18d \ub450 \uc790\ub9bf\uc218 \uc2b9\ub9ac\ub294 \uba87 \ub144 \ub9cc\uc5d0 \uba48\ucdc4\ub294\uac00?, context: \uad50\ub958\uc804\uc778 \uc694\ubbf8\uc6b0\ub9ac \uc790\uc774\uc5b8\uce20\uc804\uc5d0\uc11c\ub294 2\uacbd\uae30\uc5d0 \ub4f1\ud310\ud558\uba74\uc11c \ud328\uc804 \ud22c\uc218\uac00 \ub418\uc5c8\uc9c0\ub9cc \ud55c\uc2e0 \ud0c0\uc774\uac70\uc2a4\uc804\uc5d0\uc11c\ub294 \uc644\ubd09 \uc2b9\ub9ac\ub97c \ud3ec\ud568\ud55c 2\uc804 2\uc2b9\uc744 \uae30\ub85d\ud558\ub294 \ub4f1 \ub300\uc870\uc801\uc778 \ubaa8\uc2b5\uc744 \ubcf4\uc600\ub2e4. \uadf8\ub7ec\ub098 6\uc6d4 22\uc77c \uc5f0\uc2b5 \ub3c4\uc911\uc5d0 \uc624\ub978\ucabd \ud314\uafc8\uce58 \ud1b5\uc99d\uc73c\ub85c \ub2e4\uc74c\ub0a0 \uc138\uc774\ubd80\uc804\uc5d0\uc11c\uc758 \uc120\ubc1c \ub4f1\ud310\uc744 \ud560 \uc218 \uc5c6\uc5c8\uace0 \ubcd1\uc6d0\uc5d0\uc11c \uc815\ubc00 \uac80\uc0ac\ub97c \ubc1b\uc740 \uacb0\uacfc \ud314\uafc8\uce58\uc5d0 \uc5fc\uc99d\uc774 \uc788\ub294 \uac83\uc73c\ub85c \ubc1d\ud600\uc838 \uc7a5\uae30 \uc774\ud0c8\uc744 \ud53c\ud560 \uc218 \uc5c6\uac8c \ub418\uc5c8\ub2e4. \uadf8 \ud6c4 2\uad70 \uacbd\uae30\uc5d0\uc11c\ub294 2\ucc28\ub840\uc758 \ub4f1\ud310\uc744 \uac70\uccd0 \uc2dc\uc98c \uc885\ub8cc \uc9c1\uc804\uc778 10\uc6d4 5\uc77c\uc5d0\ub294 1\uad70\uc5d0 \ubcf5\uadc0, \uac19\uc740 \ub0a0 \uc18c\ud504\ud2b8\ubc45\ud06c\uc804\uc5d0\uc11c\ub294 7\uc774\ub2dd \ubb34\uc2e4\uc810\uc758 \ud638\ud22c\ub85c \uc2b9\ub9ac \ud22c\uc218\uac00 \ub418\uc5c8\ub2e4. \uc2dc\uc98c\uc5d0\uc11c\ub294 \uac70\ub4ed\ub418\ub294 \ubd80\uc0c1\uc758 \uc601\ud5a5\uc73c\ub85c 9\uacbd\uae30\uc5d0 \ub4f1\ud310\ud574 4\uc2b9 3\ud328\uc758 \uc131\uc801\uc73c\ub85c \ub05d\ub098 2008\ub144\ubd80\ud130 \uacc4\uc18d\ub418\uace0 \uc788\ub358 \uc5f0\uc18d \ub450 \uc790\ub9bf\uc218 \uc2b9\ub9ac\ub294 4\ub144 \ub9cc\uc5d0 \uba48\ucdc4\uace0 \ud300\uc740 \uac00\ub124\ucf54 \uc678\uc5d0\ub3c4 \uc8fc\ub825 \uc120\uc218\ub4e4\uc758 \uc787\ub2e8 \ubd80\uc0c1\uacfc \ubd80\uc9c4\uc73c\ub85c \uc545\uc7ac\uac00 \uacb9\uce58\uba74\uc11c \uacb0\uad6d \ucd5c\ud558\uc704\ub85c \ub5a8\uc5b4\uc9c0\ub294 \uc218\ubaa8\ub97c \uacaa\uc5b4\uc57c\ub9cc \ud588\ub2e4."} {"question": "1929\ub144 \ubca4\ucfe0\ubc84 \uc12c \uc5b4\ub514\uc5d0\uc11c \ucd5c\ud6c4\uc758 \ud574\ub2ec\uc774 \uc7a1\ud614\uc5c8\ub294\uac00?", "paragraph": "\uc54c\ub798\uc2a4\uce74 \ub0a8\ubd80\uc5d0\uc11c \ubd81\uc544\uba54\ub9ac\uce74 \ud574\uc548\uc120\uc744 \ub530\ub77c \ubd84\ud3ec\ud558\ub294 \ud574\ub2ec \uc11c\uc2dd\uc9c0\ub294 \ubd88\uc5f0\uc18d\uc801\uc774\ub2e4. \ubc34\ucfe0\ubc84 \uc12c\uc5d0\uc11c\ub294 \uc794\ub958 \uac1c\uccb4\uad70\uc774 20\uc138\uae30\uae4c\uc9c0 \uc874\uc18d\ud588\uc73c\ub098, 1911\ub144 \uad6d\uc81c\ubcf4\ud638\uc870\uc57d\uc5d0\ub3c4 \ubd88\uad6c\ud558\uace0 \uacb0\uad6d 1929\ub144\uc5d0 \ud0a4\uc720\ucfe0\ud2b8\uc5d0\uc11c \ucd5c\ud6c4\uc758 \ud574\ub2ec\uc774 \uc7a1\ud78c \uc774\ud6c4 \uc790\ucde8\ub97c \uac10\ucd94\uc5c8\ub2e4. 1969\ub144\uc5d0\uc11c 1972\ub144 \uc0ac\uc774, 89 \ub9c8\ub9ac\uc758 \ud574\ub2ec\uc774 \uc54c\ub798\uc2a4\uce74\uc5d0\uc11c \ubc34\ucfe0\ubc84 \uc12c \uc11c\uc548\uc73c\ub85c \uacf5\uc218 \ub610\ub294 \uc218\uc1a1\ub418\uc5c8\ub2e4. \uc774 89\ub9c8\ub9ac\ub294 2004\ub144\uc5d0\ub294 3,000 \ub9c8\ub9ac \uc774\uc0c1\uc73c\ub85c \ubd88\uc5b4\ub0ac\uc73c\uba70, \uc11c\uc2dd\uc9c0\ub3c4 \ubc45\ucfe0\ubc84 \uc11c\uc548\uc5d0\uc11c \ubd81\uc73c\ub85c\ub294 \uc2a4\ucf67 \uacf6, \ub0a8\uc73c\ub85c\ub294 \ubc14\ud074\ub9ac \ud574\ud611\uae4c\uc9c0 \ud655\uc7a5\ub418\uc5c8\ub2e4. 1989\ub144\uc5d0\ub294 \ube0c\ub9ac\ud2f0\uc2dc\uceec\ub7fc\ube44\uc544 \uc911\ubd80 \ud574\uc548\uc5d0\uc11c \uc678\ub530\ub85c \ub5a8\uc5b4\uc838 \uc788\ub358 \uad70\uc9d1\uc774 \ubc1c\uacac\ub418\uc5c8\ub2e4. 2004\ub144 \ud604\uc7ac 300 \ub9c8\ub9ac\uc5d0 \uc774\ub974\ub294 \uc774 \uad70\uc9d1\uc774 \uc54c\ub798\uc2a4\uce74\uc5d0\uc11c \uacf5\uc218\ub41c \ud574\ub2ec\uc778\uc9c0, \ub610\ub294 \ubaa8\ud53c\uc7a1\uc774\uc5d0\uc11c \uc0b4\uc544\ub0a8\uc740 \ud574\ub2ec\uc774 \uc874\uc18d\ud55c \uac83\uc778\uc9c0\ub294 \ubc1d\ud600\uc9c0\uc9c0 \uc54a\uc558\ub2e4.", "answer": "\ud0a4\uc720\ucfe0\ud2b8", "sentence": "\uacb0\uad6d 1929\ub144\uc5d0 \ud0a4\uc720\ucfe0\ud2b8 \uc5d0\uc11c \ucd5c\ud6c4\uc758 \ud574\ub2ec\uc774 \uc7a1\ud78c \uc774\ud6c4 \uc790\ucde8\ub97c \uac10\ucd94\uc5c8\ub2e4.", "paragraph_sentence": "\uc54c\ub798\uc2a4\uce74 \ub0a8\ubd80\uc5d0\uc11c \ubd81\uc544\uba54\ub9ac\uce74 \ud574\uc548\uc120\uc744 \ub530\ub77c \ubd84\ud3ec\ud558\ub294 \ud574\ub2ec \uc11c\uc2dd\uc9c0\ub294 \ubd88\uc5f0\uc18d\uc801\uc774\ub2e4. \ubc34\ucfe0\ubc84 \uc12c\uc5d0\uc11c\ub294 \uc794\ub958 \uac1c\uccb4\uad70\uc774 20\uc138\uae30\uae4c\uc9c0 \uc874\uc18d\ud588\uc73c\ub098, 1911\ub144 \uad6d\uc81c\ubcf4\ud638\uc870\uc57d\uc5d0\ub3c4 \ubd88\uad6c\ud558\uace0 \uacb0\uad6d 1929\ub144\uc5d0 \ud0a4\uc720\ucfe0\ud2b8 \uc5d0\uc11c \ucd5c\ud6c4\uc758 \ud574\ub2ec\uc774 \uc7a1\ud78c \uc774\ud6c4 \uc790\ucde8\ub97c \uac10\ucd94\uc5c8\ub2e4. 1969\ub144\uc5d0\uc11c 1972\ub144 \uc0ac\uc774, 89 \ub9c8\ub9ac\uc758 \ud574\ub2ec\uc774 \uc54c\ub798\uc2a4\uce74\uc5d0\uc11c \ubc34\ucfe0\ubc84 \uc12c \uc11c\uc548\uc73c\ub85c \uacf5\uc218 \ub610\ub294 \uc218\uc1a1\ub418\uc5c8\ub2e4. \uc774 89\ub9c8\ub9ac\ub294 2004\ub144\uc5d0\ub294 3,000 \ub9c8\ub9ac \uc774\uc0c1\uc73c\ub85c \ubd88\uc5b4\ub0ac\uc73c\uba70, \uc11c\uc2dd\uc9c0\ub3c4 \ubc45\ucfe0\ubc84 \uc11c\uc548\uc5d0\uc11c \ubd81\uc73c\ub85c\ub294 \uc2a4\ucf67 \uacf6, \ub0a8\uc73c\ub85c\ub294 \ubc14\ud074\ub9ac \ud574\ud611\uae4c\uc9c0 \ud655\uc7a5\ub418\uc5c8\ub2e4. 1989\ub144\uc5d0\ub294 \ube0c\ub9ac\ud2f0\uc2dc\uceec\ub7fc\ube44\uc544 \uc911\ubd80 \ud574\uc548\uc5d0\uc11c \uc678\ub530\ub85c \ub5a8\uc5b4\uc838 \uc788\ub358 \uad70\uc9d1\uc774 \ubc1c\uacac\ub418\uc5c8\ub2e4. 2004\ub144 \ud604\uc7ac 300 \ub9c8\ub9ac\uc5d0 \uc774\ub974\ub294 \uc774 \uad70\uc9d1\uc774 \uc54c\ub798\uc2a4\uce74\uc5d0\uc11c \uacf5\uc218\ub41c \ud574\ub2ec\uc778\uc9c0, \ub610\ub294 \ubaa8\ud53c\uc7a1\uc774\uc5d0\uc11c \uc0b4\uc544\ub0a8\uc740 \ud574\ub2ec\uc774 \uc874\uc18d\ud55c \uac83\uc778\uc9c0\ub294 \ubc1d\ud600\uc9c0\uc9c0 \uc54a\uc558\ub2e4.", "paragraph_answer": "\uc54c\ub798\uc2a4\uce74 \ub0a8\ubd80\uc5d0\uc11c \ubd81\uc544\uba54\ub9ac\uce74 \ud574\uc548\uc120\uc744 \ub530\ub77c \ubd84\ud3ec\ud558\ub294 \ud574\ub2ec \uc11c\uc2dd\uc9c0\ub294 \ubd88\uc5f0\uc18d\uc801\uc774\ub2e4. \ubc34\ucfe0\ubc84 \uc12c\uc5d0\uc11c\ub294 \uc794\ub958 \uac1c\uccb4\uad70\uc774 20\uc138\uae30\uae4c\uc9c0 \uc874\uc18d\ud588\uc73c\ub098, 1911\ub144 \uad6d\uc81c\ubcf4\ud638\uc870\uc57d\uc5d0\ub3c4 \ubd88\uad6c\ud558\uace0 \uacb0\uad6d 1929\ub144\uc5d0 \ud0a4\uc720\ucfe0\ud2b8 \uc5d0\uc11c \ucd5c\ud6c4\uc758 \ud574\ub2ec\uc774 \uc7a1\ud78c \uc774\ud6c4 \uc790\ucde8\ub97c \uac10\ucd94\uc5c8\ub2e4. 1969\ub144\uc5d0\uc11c 1972\ub144 \uc0ac\uc774, 89 \ub9c8\ub9ac\uc758 \ud574\ub2ec\uc774 \uc54c\ub798\uc2a4\uce74\uc5d0\uc11c \ubc34\ucfe0\ubc84 \uc12c \uc11c\uc548\uc73c\ub85c \uacf5\uc218 \ub610\ub294 \uc218\uc1a1\ub418\uc5c8\ub2e4. \uc774 89\ub9c8\ub9ac\ub294 2004\ub144\uc5d0\ub294 3,000 \ub9c8\ub9ac \uc774\uc0c1\uc73c\ub85c \ubd88\uc5b4\ub0ac\uc73c\uba70, \uc11c\uc2dd\uc9c0\ub3c4 \ubc45\ucfe0\ubc84 \uc11c\uc548\uc5d0\uc11c \ubd81\uc73c\ub85c\ub294 \uc2a4\ucf67 \uacf6, \ub0a8\uc73c\ub85c\ub294 \ubc14\ud074\ub9ac \ud574\ud611\uae4c\uc9c0 \ud655\uc7a5\ub418\uc5c8\ub2e4. 1989\ub144\uc5d0\ub294 \ube0c\ub9ac\ud2f0\uc2dc\uceec\ub7fc\ube44\uc544 \uc911\ubd80 \ud574\uc548\uc5d0\uc11c \uc678\ub530\ub85c \ub5a8\uc5b4\uc838 \uc788\ub358 \uad70\uc9d1\uc774 \ubc1c\uacac\ub418\uc5c8\ub2e4. 2004\ub144 \ud604\uc7ac 300 \ub9c8\ub9ac\uc5d0 \uc774\ub974\ub294 \uc774 \uad70\uc9d1\uc774 \uc54c\ub798\uc2a4\uce74\uc5d0\uc11c \uacf5\uc218\ub41c \ud574\ub2ec\uc778\uc9c0, \ub610\ub294 \ubaa8\ud53c\uc7a1\uc774\uc5d0\uc11c \uc0b4\uc544\ub0a8\uc740 \ud574\ub2ec\uc774 \uc874\uc18d\ud55c \uac83\uc778\uc9c0\ub294 \ubc1d\ud600\uc9c0\uc9c0 \uc54a\uc558\ub2e4.", "sentence_answer": "\uacb0\uad6d 1929\ub144\uc5d0 \ud0a4\uc720\ucfe0\ud2b8 \uc5d0\uc11c \ucd5c\ud6c4\uc758 \ud574\ub2ec\uc774 \uc7a1\ud78c \uc774\ud6c4 \uc790\ucde8\ub97c \uac10\ucd94\uc5c8\ub2e4.", "paragraph_id": "6570443-9-0", "paragraph_question": "question: 1929\ub144 \ubca4\ucfe0\ubc84 \uc12c \uc5b4\ub514\uc5d0\uc11c \ucd5c\ud6c4\uc758 \ud574\ub2ec\uc774 \uc7a1\ud614\uc5c8\ub294\uac00?, context: \uc54c\ub798\uc2a4\uce74 \ub0a8\ubd80\uc5d0\uc11c \ubd81\uc544\uba54\ub9ac\uce74 \ud574\uc548\uc120\uc744 \ub530\ub77c \ubd84\ud3ec\ud558\ub294 \ud574\ub2ec \uc11c\uc2dd\uc9c0\ub294 \ubd88\uc5f0\uc18d\uc801\uc774\ub2e4. \ubc34\ucfe0\ubc84 \uc12c\uc5d0\uc11c\ub294 \uc794\ub958 \uac1c\uccb4\uad70\uc774 20\uc138\uae30\uae4c\uc9c0 \uc874\uc18d\ud588\uc73c\ub098, 1911\ub144 \uad6d\uc81c\ubcf4\ud638\uc870\uc57d\uc5d0\ub3c4 \ubd88\uad6c\ud558\uace0 \uacb0\uad6d 1929\ub144\uc5d0 \ud0a4\uc720\ucfe0\ud2b8\uc5d0\uc11c \ucd5c\ud6c4\uc758 \ud574\ub2ec\uc774 \uc7a1\ud78c \uc774\ud6c4 \uc790\ucde8\ub97c \uac10\ucd94\uc5c8\ub2e4. 1969\ub144\uc5d0\uc11c 1972\ub144 \uc0ac\uc774, 89 \ub9c8\ub9ac\uc758 \ud574\ub2ec\uc774 \uc54c\ub798\uc2a4\uce74\uc5d0\uc11c \ubc34\ucfe0\ubc84 \uc12c \uc11c\uc548\uc73c\ub85c \uacf5\uc218 \ub610\ub294 \uc218\uc1a1\ub418\uc5c8\ub2e4. \uc774 89\ub9c8\ub9ac\ub294 2004\ub144\uc5d0\ub294 3,000 \ub9c8\ub9ac \uc774\uc0c1\uc73c\ub85c \ubd88\uc5b4\ub0ac\uc73c\uba70, \uc11c\uc2dd\uc9c0\ub3c4 \ubc45\ucfe0\ubc84 \uc11c\uc548\uc5d0\uc11c \ubd81\uc73c\ub85c\ub294 \uc2a4\ucf67 \uacf6, \ub0a8\uc73c\ub85c\ub294 \ubc14\ud074\ub9ac \ud574\ud611\uae4c\uc9c0 \ud655\uc7a5\ub418\uc5c8\ub2e4. 1989\ub144\uc5d0\ub294 \ube0c\ub9ac\ud2f0\uc2dc\uceec\ub7fc\ube44\uc544 \uc911\ubd80 \ud574\uc548\uc5d0\uc11c \uc678\ub530\ub85c \ub5a8\uc5b4\uc838 \uc788\ub358 \uad70\uc9d1\uc774 \ubc1c\uacac\ub418\uc5c8\ub2e4. 2004\ub144 \ud604\uc7ac 300 \ub9c8\ub9ac\uc5d0 \uc774\ub974\ub294 \uc774 \uad70\uc9d1\uc774 \uc54c\ub798\uc2a4\uce74\uc5d0\uc11c \uacf5\uc218\ub41c \ud574\ub2ec\uc778\uc9c0, \ub610\ub294 \ubaa8\ud53c\uc7a1\uc774\uc5d0\uc11c \uc0b4\uc544\ub0a8\uc740 \ud574\ub2ec\uc774 \uc874\uc18d\ud55c \uac83\uc778\uc9c0\ub294 \ubc1d\ud600\uc9c0\uc9c0 \uc54a\uc558\ub2e4."} {"question": "\uc694\ud558\ub124\uc2a4 \ud305\ud1a0\ub9ac\uc2a4\uac00 \uc554\ubb35\uc801\uc73c\ub85c \uac70\uc808\ud55c \uc774\ub860\uc740?", "paragraph": "\uace0\ub300 \uadf8\ub9ac\uc2a4\uc758 \uc544\ub9ac\uc2a4\ud1a0\ud154\ub808\uc2a4, \ud53c\ud0c0\uace0\ub77c\uc2a4 \ub4f1\uc740 \ub6f0\uc5b4\ub09c \uc74c\uc545\uc0ac\uc0c1\uac00\uc774\uae30\ub3c4 \ud588\ub2e4. \ud504\ud1a8\ub808\ub9c8\uc774\uc624\uc2a4\uc640 \uc544\ub9ac\uc2a4\ud1a0\ud06c\uc138\ub204\uc2a4, \uc544\ub9ac\uc2a4\ud2f0\ub370\uc2a4 \ud038\ud2f8\ub9ac\uc544\ub204\uc2a4 \ub4f1\uc740 \uace0\ub300 \uadf8\ub9ac\uc2a4\uc74c\uc545\uc758 \uc74c\uc545\uc774\ub860\uc744 \uad6c\ucd95\ud574\ub0b8 \ud6cc\ub96d\ud55c \uc800\uc11c\ub4e4\uc744 \ubc30\ucd9c\ud574\ub0c8\ub2e4. \uc911\uc138\uc2dc\ub300\uc5d0\ub294 \uc790\uc720 \uc778\ubb38 \ud559\ubb38\uc911 \ud558\ub098\ub85c \ub300\ud559\uc5d0\uc11c \uad50\uc721 \ubc0f \uc5f0\uad6c\ub418\uc5c8\uace0, \uae30\ubcf4\ubc95\uc774 \ucc98\uc74c \ubc1c\uc804\ud588\uc73c\uba70, \ubcf4\uc5d0\ud2f0\uc6b0\uc2a4\uc640 \ub9c8\ub974\uce58\uc544\ub204\uc2a4 \uce74\ud3a0\ub77c, \uadc0\ub3c4 \ub2e4\ub808\ucd08 \ub4f1\uc774 \ub6f0\uc5b4\ub09c \uc74c\uc545\uc774\ub860\uac00\uc600\ub2e4. \ub974\ub124\uc0c1\uc2a4\uc2dc\ub300\uc758 \ub6f0\uc5b4\ub09c \uc74c\uc545\uc774\ub860\uac00\uc600\ub358 \uc694\ud558\ub124\uc2a4 \ud305\ud1a0\ub9ac\uc2a4\ub294 \uc74c\uc545\uc758 \uc5ed\uc0ac\uc801 \uae30\ub85d\uc5d0 \uc788\uc5b4 \uae30\ubc95\uc774\ub098 \uc591\uc2dd\uc801 \ud3c9\uac00\uc5d0 \uae30\uc900\uc744 \ub450\uc9c0 \uc54a\uace0 \uc61b \uc74c\uc545\uacfc \uad50\ud68c \ub0b4\uc758 \ud604\ud559\uc801 \uc774\ub860\uc744 \uc554\ubb35\uc801\uc73c\ub85c \uac70\ubd80\ud558\uba74\uc11c \uc694\ud558\ub124\uc2a4 \uc624\ucf00\uac9c \ub4f1 \uc791\uace1\uac00\ub4e4\uc758 \uac1c\ubcc4\uc801 \uc2a4\ud0c0\uc77c\uacfc \uae30\uad50\ub97c \uc790\uc804\uc801 \ubc30\uacbd\uacfc \ud568\uaed8 \uae30\ub85d\ud558\uc600\ub2e4. \uadf8\uc758 \uc778\ubcf8\uc8fc\uc758\uc801 \uc5f0\uad6c\uc640 \uc804\uae30(biography)\ub97c \ud1b5\ud55c \uc811\uadfc\ubc29\ubc95\uc740 \uacb0\uad6d \uc74c\uc545\uc0ac\ud559\uc744 \ubc1c\uc804\uc2dc\ud0a4\ub294 \uae30\ucd08\uac00 \ub418\uc5c8\uc73c\uba70 \ub610\ud55c \uc810\uc9c4\uc801\uc778 \ubcc0\ud654\ub97c \uc774\ub048 \ucd08\uae30 \ub2e8\uacc4\ub97c \ud615\uc131\ud558\uc600\ub2e4.", "answer": "\ud604\ud559\uc801 \uc774\ub860", "sentence": "\uc61b \uc74c\uc545\uacfc \uad50\ud68c \ub0b4\uc758 \ud604\ud559\uc801 \uc774\ub860 \uc744 \uc554\ubb35\uc801\uc73c\ub85c \uac70\ubd80\ud558\uba74\uc11c \uc694\ud558\ub124\uc2a4 \uc624\ucf00\uac9c \ub4f1 \uc791\uace1\uac00\ub4e4\uc758 \uac1c\ubcc4\uc801 \uc2a4\ud0c0\uc77c\uacfc \uae30\uad50\ub97c \uc790\uc804\uc801 \ubc30\uacbd\uacfc \ud568\uaed8 \uae30\ub85d\ud558\uc600\ub2e4.", "paragraph_sentence": "\uace0\ub300 \uadf8\ub9ac\uc2a4\uc758 \uc544\ub9ac\uc2a4\ud1a0\ud154\ub808\uc2a4, \ud53c\ud0c0\uace0\ub77c\uc2a4 \ub4f1\uc740 \ub6f0\uc5b4\ub09c \uc74c\uc545\uc0ac\uc0c1\uac00\uc774\uae30\ub3c4 \ud588\ub2e4. \ud504\ud1a8\ub808\ub9c8\uc774\uc624\uc2a4\uc640 \uc544\ub9ac\uc2a4\ud1a0\ud06c\uc138\ub204\uc2a4, \uc544\ub9ac\uc2a4\ud2f0\ub370\uc2a4 \ud038\ud2f8\ub9ac\uc544\ub204\uc2a4 \ub4f1\uc740 \uace0\ub300 \uadf8\ub9ac\uc2a4\uc74c\uc545\uc758 \uc74c\uc545\uc774\ub860\uc744 \uad6c\ucd95\ud574\ub0b8 \ud6cc\ub96d\ud55c \uc800\uc11c\ub4e4\uc744 \ubc30\ucd9c\ud574\ub0c8\ub2e4. \uc911\uc138\uc2dc\ub300\uc5d0\ub294 \uc790\uc720 \uc778\ubb38 \ud559\ubb38\uc911 \ud558\ub098\ub85c \ub300\ud559\uc5d0\uc11c \uad50\uc721 \ubc0f \uc5f0\uad6c\ub418\uc5c8\uace0, \uae30\ubcf4\ubc95\uc774 \ucc98\uc74c \ubc1c\uc804\ud588\uc73c\uba70, \ubcf4\uc5d0\ud2f0\uc6b0\uc2a4\uc640 \ub9c8\ub974\uce58\uc544\ub204\uc2a4 \uce74\ud3a0\ub77c, \uadc0\ub3c4 \ub2e4\ub808\ucd08 \ub4f1\uc774 \ub6f0\uc5b4\ub09c \uc74c\uc545\uc774\ub860\uac00\uc600\ub2e4. \ub974\ub124\uc0c1\uc2a4\uc2dc\ub300\uc758 \ub6f0\uc5b4\ub09c \uc74c\uc545\uc774\ub860\uac00\uc600\ub358 \uc694\ud558\ub124\uc2a4 \ud305\ud1a0\ub9ac\uc2a4\ub294 \uc74c\uc545\uc758 \uc5ed\uc0ac\uc801 \uae30\ub85d\uc5d0 \uc788\uc5b4 \uae30\ubc95\uc774\ub098 \uc591\uc2dd\uc801 \ud3c9\uac00\uc5d0 \uae30\uc900\uc744 \ub450\uc9c0 \uc54a\uace0 \uc61b \uc74c\uc545\uacfc \uad50\ud68c \ub0b4\uc758 \ud604\ud559\uc801 \uc774\ub860 \uc744 \uc554\ubb35\uc801\uc73c\ub85c \uac70\ubd80\ud558\uba74\uc11c \uc694\ud558\ub124\uc2a4 \uc624\ucf00\uac9c \ub4f1 \uc791\uace1\uac00\ub4e4\uc758 \uac1c\ubcc4\uc801 \uc2a4\ud0c0\uc77c\uacfc \uae30\uad50\ub97c \uc790\uc804\uc801 \ubc30\uacbd\uacfc \ud568\uaed8 \uae30\ub85d\ud558\uc600\ub2e4. \uadf8\uc758 \uc778\ubcf8\uc8fc\uc758\uc801 \uc5f0\uad6c\uc640 \uc804\uae30(biography)\ub97c \ud1b5\ud55c \uc811\uadfc\ubc29\ubc95\uc740 \uacb0\uad6d \uc74c\uc545\uc0ac\ud559\uc744 \ubc1c\uc804\uc2dc\ud0a4\ub294 \uae30\ucd08\uac00 \ub418\uc5c8\uc73c\uba70 \ub610\ud55c \uc810\uc9c4\uc801\uc778 \ubcc0\ud654\ub97c \uc774\ub048 \ucd08\uae30 \ub2e8\uacc4\ub97c \ud615\uc131\ud558\uc600\ub2e4.", "paragraph_answer": "\uace0\ub300 \uadf8\ub9ac\uc2a4\uc758 \uc544\ub9ac\uc2a4\ud1a0\ud154\ub808\uc2a4, \ud53c\ud0c0\uace0\ub77c\uc2a4 \ub4f1\uc740 \ub6f0\uc5b4\ub09c \uc74c\uc545\uc0ac\uc0c1\uac00\uc774\uae30\ub3c4 \ud588\ub2e4. \ud504\ud1a8\ub808\ub9c8\uc774\uc624\uc2a4\uc640 \uc544\ub9ac\uc2a4\ud1a0\ud06c\uc138\ub204\uc2a4, \uc544\ub9ac\uc2a4\ud2f0\ub370\uc2a4 \ud038\ud2f8\ub9ac\uc544\ub204\uc2a4 \ub4f1\uc740 \uace0\ub300 \uadf8\ub9ac\uc2a4\uc74c\uc545\uc758 \uc74c\uc545\uc774\ub860\uc744 \uad6c\ucd95\ud574\ub0b8 \ud6cc\ub96d\ud55c \uc800\uc11c\ub4e4\uc744 \ubc30\ucd9c\ud574\ub0c8\ub2e4. \uc911\uc138\uc2dc\ub300\uc5d0\ub294 \uc790\uc720 \uc778\ubb38 \ud559\ubb38\uc911 \ud558\ub098\ub85c \ub300\ud559\uc5d0\uc11c \uad50\uc721 \ubc0f \uc5f0\uad6c\ub418\uc5c8\uace0, \uae30\ubcf4\ubc95\uc774 \ucc98\uc74c \ubc1c\uc804\ud588\uc73c\uba70, \ubcf4\uc5d0\ud2f0\uc6b0\uc2a4\uc640 \ub9c8\ub974\uce58\uc544\ub204\uc2a4 \uce74\ud3a0\ub77c, \uadc0\ub3c4 \ub2e4\ub808\ucd08 \ub4f1\uc774 \ub6f0\uc5b4\ub09c \uc74c\uc545\uc774\ub860\uac00\uc600\ub2e4. \ub974\ub124\uc0c1\uc2a4\uc2dc\ub300\uc758 \ub6f0\uc5b4\ub09c \uc74c\uc545\uc774\ub860\uac00\uc600\ub358 \uc694\ud558\ub124\uc2a4 \ud305\ud1a0\ub9ac\uc2a4\ub294 \uc74c\uc545\uc758 \uc5ed\uc0ac\uc801 \uae30\ub85d\uc5d0 \uc788\uc5b4 \uae30\ubc95\uc774\ub098 \uc591\uc2dd\uc801 \ud3c9\uac00\uc5d0 \uae30\uc900\uc744 \ub450\uc9c0 \uc54a\uace0 \uc61b \uc74c\uc545\uacfc \uad50\ud68c \ub0b4\uc758 \ud604\ud559\uc801 \uc774\ub860 \uc744 \uc554\ubb35\uc801\uc73c\ub85c \uac70\ubd80\ud558\uba74\uc11c \uc694\ud558\ub124\uc2a4 \uc624\ucf00\uac9c \ub4f1 \uc791\uace1\uac00\ub4e4\uc758 \uac1c\ubcc4\uc801 \uc2a4\ud0c0\uc77c\uacfc \uae30\uad50\ub97c \uc790\uc804\uc801 \ubc30\uacbd\uacfc \ud568\uaed8 \uae30\ub85d\ud558\uc600\ub2e4. \uadf8\uc758 \uc778\ubcf8\uc8fc\uc758\uc801 \uc5f0\uad6c\uc640 \uc804\uae30(biography)\ub97c \ud1b5\ud55c \uc811\uadfc\ubc29\ubc95\uc740 \uacb0\uad6d \uc74c\uc545\uc0ac\ud559\uc744 \ubc1c\uc804\uc2dc\ud0a4\ub294 \uae30\ucd08\uac00 \ub418\uc5c8\uc73c\uba70 \ub610\ud55c \uc810\uc9c4\uc801\uc778 \ubcc0\ud654\ub97c \uc774\ub048 \ucd08\uae30 \ub2e8\uacc4\ub97c \ud615\uc131\ud558\uc600\ub2e4.", "sentence_answer": "\uc61b \uc74c\uc545\uacfc \uad50\ud68c \ub0b4\uc758 \ud604\ud559\uc801 \uc774\ub860 \uc744 \uc554\ubb35\uc801\uc73c\ub85c \uac70\ubd80\ud558\uba74\uc11c \uc694\ud558\ub124\uc2a4 \uc624\ucf00\uac9c \ub4f1 \uc791\uace1\uac00\ub4e4\uc758 \uac1c\ubcc4\uc801 \uc2a4\ud0c0\uc77c\uacfc \uae30\uad50\ub97c \uc790\uc804\uc801 \ubc30\uacbd\uacfc \ud568\uaed8 \uae30\ub85d\ud558\uc600\ub2e4.", "paragraph_id": "6544218-0-1", "paragraph_question": "question: \uc694\ud558\ub124\uc2a4 \ud305\ud1a0\ub9ac\uc2a4\uac00 \uc554\ubb35\uc801\uc73c\ub85c \uac70\uc808\ud55c \uc774\ub860\uc740?, context: \uace0\ub300 \uadf8\ub9ac\uc2a4\uc758 \uc544\ub9ac\uc2a4\ud1a0\ud154\ub808\uc2a4, \ud53c\ud0c0\uace0\ub77c\uc2a4 \ub4f1\uc740 \ub6f0\uc5b4\ub09c \uc74c\uc545\uc0ac\uc0c1\uac00\uc774\uae30\ub3c4 \ud588\ub2e4. \ud504\ud1a8\ub808\ub9c8\uc774\uc624\uc2a4\uc640 \uc544\ub9ac\uc2a4\ud1a0\ud06c\uc138\ub204\uc2a4, \uc544\ub9ac\uc2a4\ud2f0\ub370\uc2a4 \ud038\ud2f8\ub9ac\uc544\ub204\uc2a4 \ub4f1\uc740 \uace0\ub300 \uadf8\ub9ac\uc2a4\uc74c\uc545\uc758 \uc74c\uc545\uc774\ub860\uc744 \uad6c\ucd95\ud574\ub0b8 \ud6cc\ub96d\ud55c \uc800\uc11c\ub4e4\uc744 \ubc30\ucd9c\ud574\ub0c8\ub2e4. \uc911\uc138\uc2dc\ub300\uc5d0\ub294 \uc790\uc720 \uc778\ubb38 \ud559\ubb38\uc911 \ud558\ub098\ub85c \ub300\ud559\uc5d0\uc11c \uad50\uc721 \ubc0f \uc5f0\uad6c\ub418\uc5c8\uace0, \uae30\ubcf4\ubc95\uc774 \ucc98\uc74c \ubc1c\uc804\ud588\uc73c\uba70, \ubcf4\uc5d0\ud2f0\uc6b0\uc2a4\uc640 \ub9c8\ub974\uce58\uc544\ub204\uc2a4 \uce74\ud3a0\ub77c, \uadc0\ub3c4 \ub2e4\ub808\ucd08 \ub4f1\uc774 \ub6f0\uc5b4\ub09c \uc74c\uc545\uc774\ub860\uac00\uc600\ub2e4. \ub974\ub124\uc0c1\uc2a4\uc2dc\ub300\uc758 \ub6f0\uc5b4\ub09c \uc74c\uc545\uc774\ub860\uac00\uc600\ub358 \uc694\ud558\ub124\uc2a4 \ud305\ud1a0\ub9ac\uc2a4\ub294 \uc74c\uc545\uc758 \uc5ed\uc0ac\uc801 \uae30\ub85d\uc5d0 \uc788\uc5b4 \uae30\ubc95\uc774\ub098 \uc591\uc2dd\uc801 \ud3c9\uac00\uc5d0 \uae30\uc900\uc744 \ub450\uc9c0 \uc54a\uace0 \uc61b \uc74c\uc545\uacfc \uad50\ud68c \ub0b4\uc758 \ud604\ud559\uc801 \uc774\ub860\uc744 \uc554\ubb35\uc801\uc73c\ub85c \uac70\ubd80\ud558\uba74\uc11c \uc694\ud558\ub124\uc2a4 \uc624\ucf00\uac9c \ub4f1 \uc791\uace1\uac00\ub4e4\uc758 \uac1c\ubcc4\uc801 \uc2a4\ud0c0\uc77c\uacfc \uae30\uad50\ub97c \uc790\uc804\uc801 \ubc30\uacbd\uacfc \ud568\uaed8 \uae30\ub85d\ud558\uc600\ub2e4. \uadf8\uc758 \uc778\ubcf8\uc8fc\uc758\uc801 \uc5f0\uad6c\uc640 \uc804\uae30(biography)\ub97c \ud1b5\ud55c \uc811\uadfc\ubc29\ubc95\uc740 \uacb0\uad6d \uc74c\uc545\uc0ac\ud559\uc744 \ubc1c\uc804\uc2dc\ud0a4\ub294 \uae30\ucd08\uac00 \ub418\uc5c8\uc73c\uba70 \ub610\ud55c \uc810\uc9c4\uc801\uc778 \ubcc0\ud654\ub97c \uc774\ub048 \ucd08\uae30 \ub2e8\uacc4\ub97c \ud615\uc131\ud558\uc600\ub2e4."} {"question": "\uc11c\ub0a8\uc9c0\uc5ed\uacfc \ub3d9\ubd80\uc804\uc120\ub4f1\uc5d0 \uc9d1\uacb0\ud558\uc5ec \uc720\uc5d4\uad70\uc744 \uacf5\uaca9\ud55c \uac83\uc740 \uc5b8\uc81c\uc778\uac00?", "paragraph": "\uc774 \ub0a0 \uc815\ubd80\ub294 \ub300\uad6c\uc5d0\uc11c \ubd80\uc0b0\uc73c\ub85c \ud6c4\ud1f4\ud558\uace0 \ub300\uad6c \uc2dc\ubbfc\uc5d0\uac8c\ub294 \uc18c\uac1c\ub839(\u758f\u958b\u4ee4)\uc744 \ub0b4\ub838\ub2e4. \ucd5c\ud6c4\uc758 \uacb0\uc804\uc744 \uac01\uc624\ud55c \ud55c\uad6d\uad70 \uc81c1\uc0ac\ub2e8\uc740 \uc601\uad6d\uad70 \ubcf4\ubcd1 \uc81c27\uc5ec\ub2e8\uc758 \uc9c0\uc6d0\uc744 \ubc1b\uc544\uac00\uba70 \ubd81\ubc29\uc73c\ub85c 4Km\ub97c \ubc18\uaca9, \ub4a4\uc774\uc5b4 \uc774 \ub0a0 \ubbf8 \uc81c 24\uc0ac\ub2e8\uacfc \ud574\ubcd1\uc0ac\ub2e8\uc758 \uc751\uc6d0 \ucd9c\ub3d9\uc774 \uc788\uc5b4 23\uc77c\uae4c\uc9c0\ub294 \ub300\uad6c \ubd81\ubc29\uc758 \uc804\uc138\ub97c \uc644\uc804\ud788 \uc5ed\uc804\uc2dc\ucf1c \ub099\ub3d9\uac15 \uc774\ub0a8 \uc9c0\uc5ed\uc744 \ud655\ubcf4\ud588\ub2e4. \uc774 \ud6c4 \ubd81\ud55c\uad70\uc740 \ub300\uad6c\ub97c \ud53c\ud558\uc5ec \ub3d9\u00b7\uc11c\ub85c \ubd80\ub300\ub97c \uc774\ub3d9\ud558\ub354\ub2c8, \ucd5c\ud6c4\uc758 \uacb0\uc804\uc778 \uc81c2\ucc28 \ud611\uacf5(\u633e\u653b)\uc791\uc804\uc744 \uae30\ub3c4\ud588\ub2e4. \uc11c\ub0a8\uc9c0\uc5ed \uc9c4\uc8fc(\u664b\u5dde) \ubc29\uba74\uc5d0 \ubd81\ud55c\uad70\uc758 4\uac1c \ubcf4\ubcd1\uc0ac\ub2e8(4\u00b76\u00b77\u00b79)\uacfc 1\uac1c \uc804\ucc28\uc5f0\ub300(105\uc0ac\ub2e8 16\uc5f0\ub300)\uac00 \uc9d1\uacb0\ud558\uace0, \ub3d9\ubd80\uc804\uc120 \ud3ec\ud56d \ubd81\ubc29\uc5d0 \uc5ed\uc2dc 4\uac1c \ubcf4\ubcd1\uc0ac\ub2e8(5\u00b78\u00b712\u00b715)\uacfc 1\uac1c \uc804\ucc28\uc5f0\ub300(105)\uac00 \uc9d1\uacb0\ud558\uc5ec 9\uc6d4 3\uc77c 12\uc2dc\ub97c \uae30\ud558\uc5ec \ub3d9\uc2dc\uc5d0 \uc720\uc5d4\uad70\uc744 \uacf5\uaca9\ud574 \uc654\ub2e4. \uadf8\ub7ec\ub098 \ubd81\ud55c\uad70\uc740 \uc774\ubbf8 \uc0ac\uae30\uac00 \uadf9\ub3c4\ub85c \uc800\ud558\ub418\uace0 \uc804\ud22c\ub825\uc774 \uae30\uc9c4\ud558\uc600\uc73c\uba70, \ud6c8\ub828\ub3c4 \ub418\uc9c0 \uc54a\uc740 \uc2e0\ubcd1\ub4e4\uc744 \ubcf4\ucda9 \ubcd1\ub825\uc73c\ub85c \ud3b8\uc131\ud55c \uc0c1\ud0dc\uc600\ub2e4.", "answer": "9\uc6d4 3\uc77c 12\uc2dc", "sentence": "\uc11c\ub0a8\uc9c0\uc5ed \uc9c4\uc8fc(\u664b\u5dde) \ubc29\uba74\uc5d0 \ubd81\ud55c\uad70\uc758 4\uac1c \ubcf4\ubcd1\uc0ac\ub2e8(4\u00b76\u00b77\u00b79)\uacfc 1\uac1c \uc804\ucc28\uc5f0\ub300(105\uc0ac\ub2e8 16\uc5f0\ub300)\uac00 \uc9d1\uacb0\ud558\uace0, \ub3d9\ubd80\uc804\uc120 \ud3ec\ud56d \ubd81\ubc29\uc5d0 \uc5ed\uc2dc 4\uac1c \ubcf4\ubcd1\uc0ac\ub2e8(5\u00b78\u00b712\u00b715)\uacfc 1\uac1c \uc804\ucc28\uc5f0\ub300(105)\uac00 \uc9d1\uacb0\ud558\uc5ec 9\uc6d4 3\uc77c 12\uc2dc \ub97c \uae30\ud558\uc5ec \ub3d9\uc2dc\uc5d0 \uc720\uc5d4\uad70\uc744 \uacf5\uaca9\ud574 \uc654\ub2e4.", "paragraph_sentence": "\uc774 \ub0a0 \uc815\ubd80\ub294 \ub300\uad6c\uc5d0\uc11c \ubd80\uc0b0\uc73c\ub85c \ud6c4\ud1f4\ud558\uace0 \ub300\uad6c \uc2dc\ubbfc\uc5d0\uac8c\ub294 \uc18c\uac1c\ub839(\u758f\u958b\u4ee4)\uc744 \ub0b4\ub838\ub2e4. \ucd5c\ud6c4\uc758 \uacb0\uc804\uc744 \uac01\uc624\ud55c \ud55c\uad6d\uad70 \uc81c1\uc0ac\ub2e8\uc740 \uc601\uad6d\uad70 \ubcf4\ubcd1 \uc81c27\uc5ec\ub2e8\uc758 \uc9c0\uc6d0\uc744 \ubc1b\uc544\uac00\uba70 \ubd81\ubc29\uc73c\ub85c 4Km\ub97c \ubc18\uaca9, \ub4a4\uc774\uc5b4 \uc774 \ub0a0 \ubbf8 \uc81c 24\uc0ac\ub2e8\uacfc \ud574\ubcd1\uc0ac\ub2e8\uc758 \uc751\uc6d0 \ucd9c\ub3d9\uc774 \uc788\uc5b4 23\uc77c\uae4c\uc9c0\ub294 \ub300\uad6c \ubd81\ubc29\uc758 \uc804\uc138\ub97c \uc644\uc804\ud788 \uc5ed\uc804\uc2dc\ucf1c \ub099\ub3d9\uac15 \uc774\ub0a8 \uc9c0\uc5ed\uc744 \ud655\ubcf4\ud588\ub2e4. \uc774 \ud6c4 \ubd81\ud55c\uad70\uc740 \ub300\uad6c\ub97c \ud53c\ud558\uc5ec \ub3d9\u00b7\uc11c\ub85c \ubd80\ub300\ub97c \uc774\ub3d9\ud558\ub354\ub2c8, \ucd5c\ud6c4\uc758 \uacb0\uc804\uc778 \uc81c2\ucc28 \ud611\uacf5(\u633e\u653b)\uc791\uc804\uc744 \uae30\ub3c4\ud588\ub2e4. \uc11c\ub0a8\uc9c0\uc5ed \uc9c4\uc8fc(\u664b\u5dde) \ubc29\uba74\uc5d0 \ubd81\ud55c\uad70\uc758 4\uac1c \ubcf4\ubcd1\uc0ac\ub2e8(4\u00b76\u00b77\u00b79)\uacfc 1\uac1c \uc804\ucc28\uc5f0\ub300(105\uc0ac\ub2e8 16\uc5f0\ub300)\uac00 \uc9d1\uacb0\ud558\uace0, \ub3d9\ubd80\uc804\uc120 \ud3ec\ud56d \ubd81\ubc29\uc5d0 \uc5ed\uc2dc 4\uac1c \ubcf4\ubcd1\uc0ac\ub2e8(5\u00b78\u00b712\u00b715)\uacfc 1\uac1c \uc804\ucc28\uc5f0\ub300(105)\uac00 \uc9d1\uacb0\ud558\uc5ec 9\uc6d4 3\uc77c 12\uc2dc \ub97c \uae30\ud558\uc5ec \ub3d9\uc2dc\uc5d0 \uc720\uc5d4\uad70\uc744 \uacf5\uaca9\ud574 \uc654\ub2e4. \uadf8\ub7ec\ub098 \ubd81\ud55c\uad70\uc740 \uc774\ubbf8 \uc0ac\uae30\uac00 \uadf9\ub3c4\ub85c \uc800\ud558\ub418\uace0 \uc804\ud22c\ub825\uc774 \uae30\uc9c4\ud558\uc600\uc73c\uba70, \ud6c8\ub828\ub3c4 \ub418\uc9c0 \uc54a\uc740 \uc2e0\ubcd1\ub4e4\uc744 \ubcf4\ucda9 \ubcd1\ub825\uc73c\ub85c \ud3b8\uc131\ud55c \uc0c1\ud0dc\uc600\ub2e4.", "paragraph_answer": "\uc774 \ub0a0 \uc815\ubd80\ub294 \ub300\uad6c\uc5d0\uc11c \ubd80\uc0b0\uc73c\ub85c \ud6c4\ud1f4\ud558\uace0 \ub300\uad6c \uc2dc\ubbfc\uc5d0\uac8c\ub294 \uc18c\uac1c\ub839(\u758f\u958b\u4ee4)\uc744 \ub0b4\ub838\ub2e4. \ucd5c\ud6c4\uc758 \uacb0\uc804\uc744 \uac01\uc624\ud55c \ud55c\uad6d\uad70 \uc81c1\uc0ac\ub2e8\uc740 \uc601\uad6d\uad70 \ubcf4\ubcd1 \uc81c27\uc5ec\ub2e8\uc758 \uc9c0\uc6d0\uc744 \ubc1b\uc544\uac00\uba70 \ubd81\ubc29\uc73c\ub85c 4Km\ub97c \ubc18\uaca9, \ub4a4\uc774\uc5b4 \uc774 \ub0a0 \ubbf8 \uc81c 24\uc0ac\ub2e8\uacfc \ud574\ubcd1\uc0ac\ub2e8\uc758 \uc751\uc6d0 \ucd9c\ub3d9\uc774 \uc788\uc5b4 23\uc77c\uae4c\uc9c0\ub294 \ub300\uad6c \ubd81\ubc29\uc758 \uc804\uc138\ub97c \uc644\uc804\ud788 \uc5ed\uc804\uc2dc\ucf1c \ub099\ub3d9\uac15 \uc774\ub0a8 \uc9c0\uc5ed\uc744 \ud655\ubcf4\ud588\ub2e4. \uc774 \ud6c4 \ubd81\ud55c\uad70\uc740 \ub300\uad6c\ub97c \ud53c\ud558\uc5ec \ub3d9\u00b7\uc11c\ub85c \ubd80\ub300\ub97c \uc774\ub3d9\ud558\ub354\ub2c8, \ucd5c\ud6c4\uc758 \uacb0\uc804\uc778 \uc81c2\ucc28 \ud611\uacf5(\u633e\u653b)\uc791\uc804\uc744 \uae30\ub3c4\ud588\ub2e4. \uc11c\ub0a8\uc9c0\uc5ed \uc9c4\uc8fc(\u664b\u5dde) \ubc29\uba74\uc5d0 \ubd81\ud55c\uad70\uc758 4\uac1c \ubcf4\ubcd1\uc0ac\ub2e8(4\u00b76\u00b77\u00b79)\uacfc 1\uac1c \uc804\ucc28\uc5f0\ub300(105\uc0ac\ub2e8 16\uc5f0\ub300)\uac00 \uc9d1\uacb0\ud558\uace0, \ub3d9\ubd80\uc804\uc120 \ud3ec\ud56d \ubd81\ubc29\uc5d0 \uc5ed\uc2dc 4\uac1c \ubcf4\ubcd1\uc0ac\ub2e8(5\u00b78\u00b712\u00b715)\uacfc 1\uac1c \uc804\ucc28\uc5f0\ub300(105)\uac00 \uc9d1\uacb0\ud558\uc5ec 9\uc6d4 3\uc77c 12\uc2dc \ub97c \uae30\ud558\uc5ec \ub3d9\uc2dc\uc5d0 \uc720\uc5d4\uad70\uc744 \uacf5\uaca9\ud574 \uc654\ub2e4. \uadf8\ub7ec\ub098 \ubd81\ud55c\uad70\uc740 \uc774\ubbf8 \uc0ac\uae30\uac00 \uadf9\ub3c4\ub85c \uc800\ud558\ub418\uace0 \uc804\ud22c\ub825\uc774 \uae30\uc9c4\ud558\uc600\uc73c\uba70, \ud6c8\ub828\ub3c4 \ub418\uc9c0 \uc54a\uc740 \uc2e0\ubcd1\ub4e4\uc744 \ubcf4\ucda9 \ubcd1\ub825\uc73c\ub85c \ud3b8\uc131\ud55c \uc0c1\ud0dc\uc600\ub2e4.", "sentence_answer": "\uc11c\ub0a8\uc9c0\uc5ed \uc9c4\uc8fc(\u664b\u5dde) \ubc29\uba74\uc5d0 \ubd81\ud55c\uad70\uc758 4\uac1c \ubcf4\ubcd1\uc0ac\ub2e8(4\u00b76\u00b77\u00b79)\uacfc 1\uac1c \uc804\ucc28\uc5f0\ub300(105\uc0ac\ub2e8 16\uc5f0\ub300)\uac00 \uc9d1\uacb0\ud558\uace0, \ub3d9\ubd80\uc804\uc120 \ud3ec\ud56d \ubd81\ubc29\uc5d0 \uc5ed\uc2dc 4\uac1c \ubcf4\ubcd1\uc0ac\ub2e8(5\u00b78\u00b712\u00b715)\uacfc 1\uac1c \uc804\ucc28\uc5f0\ub300(105)\uac00 \uc9d1\uacb0\ud558\uc5ec 9\uc6d4 3\uc77c 12\uc2dc \ub97c \uae30\ud558\uc5ec \ub3d9\uc2dc\uc5d0 \uc720\uc5d4\uad70\uc744 \uacf5\uaca9\ud574 \uc654\ub2e4.", "paragraph_id": "6569867-40-1", "paragraph_question": "question: \uc11c\ub0a8\uc9c0\uc5ed\uacfc \ub3d9\ubd80\uc804\uc120\ub4f1\uc5d0 \uc9d1\uacb0\ud558\uc5ec \uc720\uc5d4\uad70\uc744 \uacf5\uaca9\ud55c \uac83\uc740 \uc5b8\uc81c\uc778\uac00?, context: \uc774 \ub0a0 \uc815\ubd80\ub294 \ub300\uad6c\uc5d0\uc11c \ubd80\uc0b0\uc73c\ub85c \ud6c4\ud1f4\ud558\uace0 \ub300\uad6c \uc2dc\ubbfc\uc5d0\uac8c\ub294 \uc18c\uac1c\ub839(\u758f\u958b\u4ee4)\uc744 \ub0b4\ub838\ub2e4. \ucd5c\ud6c4\uc758 \uacb0\uc804\uc744 \uac01\uc624\ud55c \ud55c\uad6d\uad70 \uc81c1\uc0ac\ub2e8\uc740 \uc601\uad6d\uad70 \ubcf4\ubcd1 \uc81c27\uc5ec\ub2e8\uc758 \uc9c0\uc6d0\uc744 \ubc1b\uc544\uac00\uba70 \ubd81\ubc29\uc73c\ub85c 4Km\ub97c \ubc18\uaca9, \ub4a4\uc774\uc5b4 \uc774 \ub0a0 \ubbf8 \uc81c 24\uc0ac\ub2e8\uacfc \ud574\ubcd1\uc0ac\ub2e8\uc758 \uc751\uc6d0 \ucd9c\ub3d9\uc774 \uc788\uc5b4 23\uc77c\uae4c\uc9c0\ub294 \ub300\uad6c \ubd81\ubc29\uc758 \uc804\uc138\ub97c \uc644\uc804\ud788 \uc5ed\uc804\uc2dc\ucf1c \ub099\ub3d9\uac15 \uc774\ub0a8 \uc9c0\uc5ed\uc744 \ud655\ubcf4\ud588\ub2e4. \uc774 \ud6c4 \ubd81\ud55c\uad70\uc740 \ub300\uad6c\ub97c \ud53c\ud558\uc5ec \ub3d9\u00b7\uc11c\ub85c \ubd80\ub300\ub97c \uc774\ub3d9\ud558\ub354\ub2c8, \ucd5c\ud6c4\uc758 \uacb0\uc804\uc778 \uc81c2\ucc28 \ud611\uacf5(\u633e\u653b)\uc791\uc804\uc744 \uae30\ub3c4\ud588\ub2e4. \uc11c\ub0a8\uc9c0\uc5ed \uc9c4\uc8fc(\u664b\u5dde) \ubc29\uba74\uc5d0 \ubd81\ud55c\uad70\uc758 4\uac1c \ubcf4\ubcd1\uc0ac\ub2e8(4\u00b76\u00b77\u00b79)\uacfc 1\uac1c \uc804\ucc28\uc5f0\ub300(105\uc0ac\ub2e8 16\uc5f0\ub300)\uac00 \uc9d1\uacb0\ud558\uace0, \ub3d9\ubd80\uc804\uc120 \ud3ec\ud56d \ubd81\ubc29\uc5d0 \uc5ed\uc2dc 4\uac1c \ubcf4\ubcd1\uc0ac\ub2e8(5\u00b78\u00b712\u00b715)\uacfc 1\uac1c \uc804\ucc28\uc5f0\ub300(105)\uac00 \uc9d1\uacb0\ud558\uc5ec 9\uc6d4 3\uc77c 12\uc2dc\ub97c \uae30\ud558\uc5ec \ub3d9\uc2dc\uc5d0 \uc720\uc5d4\uad70\uc744 \uacf5\uaca9\ud574 \uc654\ub2e4. \uadf8\ub7ec\ub098 \ubd81\ud55c\uad70\uc740 \uc774\ubbf8 \uc0ac\uae30\uac00 \uadf9\ub3c4\ub85c \uc800\ud558\ub418\uace0 \uc804\ud22c\ub825\uc774 \uae30\uc9c4\ud558\uc600\uc73c\uba70, \ud6c8\ub828\ub3c4 \ub418\uc9c0 \uc54a\uc740 \uc2e0\ubcd1\ub4e4\uc744 \ubcf4\ucda9 \ubcd1\ub825\uc73c\ub85c \ud3b8\uc131\ud55c \uc0c1\ud0dc\uc600\ub2e4."} {"question": "\uc1fc\ud2b8\ud504\ub85c\uadf8\ub7a8\uc758 \uc57d\uc790\ub294?", "paragraph": "2007\ub144 5\uc6d4\ubd80\ud130 \uc548\uc815\ub418\uace0 \uccb4\uacc4\uc801\uc778 \ud6c8\ub828\uc744 \ud558\uae30 \uc704\ud574 \uc5f0\uc2b5\uac70\uc810\uc744 \uce90\ub098\ub2e4 \ud1a0\ub860\ud1a0\ub85c \uc62e\uae30\uace0 \ud1a0\ub860\ud1a0 \ud06c\ub9ac\ucf13 \uc2a4\ucf00\uc774\ud305 & \uceec\ub9c1 \ud074\ub7fd \uc5d0\uc11c \ucf54\uce58\uc778 \ube0c\ub77c\uc774\uc5b8 \uc624\uc11c\uc640 \ud568\uaed8 \ud6c8\ub828\ud558\uae30 \uc2dc\uc791\ud588\ub2e4. 2007\ub144 11\uc6d4 8\uc77c\uc5d0 \uc5f4\ub9b0 \ucef5 \uc624\ube0c \ucc28\uc774\ub098\uc5d0\uc11c \uc1fc\ud2b8\ud504\ub85c\uadf8\ub7a8(SP)\uacfc \ud504\ub9ac\uc2a4\ucf00\uc774\ud305(FS)\uc5d0\uc11c \ud55c \ubc88\uc529\uc758 \uc810\ud504 \uc2e4\uc218\ub97c \ud558\uc600\uc74c\uc5d0\ub3c4 \ubd88\uad6c\ud558\uace0 \uc804\uccb4\uc801\uc73c\ub85c \uc548\uc815\ub418\uace0 \uc9c8\ub192\uc740 \uc5f0\uae30\ub97c \uc120\ubcf4\uc5ec \ud504\ub9ac\uc2a4\ucf00\uc774\ud305\uc5d0\uc11c 122.36\uc810\uc73c\ub85c \uc790\uc2e0\uc758 \ucd5c\uace0\uae30\ub85d\uc744 \uac31\uc2e0\ud558\uba70 \uc6b0\uc2b9\uc744 \ucc28\uc9c0\ud588\ub2e4. 11\uc6d4 24\uc77c\uc5d0\ub294 \ub7ec\uc2dc\uc544 \ubaa8\uc2a4\ud06c\ubc14\uc5d0\uc11c \uc5f4\ub9b0 \ucef5 \uc624\ube0c \ub7ec\uc2dc\uc544\uc5d0 \ucc38\uac00\ud558\uc5ec \ud504\ub9ac\uc640 \ucd1d\uc810\uc5d0\uc11c \uac1c\uc778\ucd5c\uace0\uae30\ub85d\uc744 \uac31\uc2e0\ud558\uba74\uc11c \uc6b0\uc2b9\uc744 \ucc28\uc9c0\ud558\uc600\ub2e4. \ud2b9\ud788 \uc774 \ub300\ud68c \ud504\ub9ac\uc2a4\ucf00\uc774\ud305\uc5d0\uc11c \uae30\ub85d\ud55c 133.70\uc810\uc740 \uc774\uc804\uae4c\uc9c0 \uc138\uacc4 \ucd5c\uace0 \uae30\ub85d\uc774\uc5c8\ub358 \uc544\uc0ac\ub2e4 \ub9c8\uc624\uc758 133.13\uc810\uc744 \ub118\uc5b4\uc11c\ub294 \uc138\uacc4\uc2e0\uae30\ub85d\uc774\uc5c8\ub2e4.", "answer": "SP", "sentence": "2007\ub144 11\uc6d4 8\uc77c\uc5d0 \uc5f4\ub9b0 \ucef5 \uc624\ube0c \ucc28\uc774\ub098\uc5d0\uc11c \uc1fc\ud2b8\ud504\ub85c\uadf8\ub7a8( SP )", "paragraph_sentence": "2007\ub144 5\uc6d4\ubd80\ud130 \uc548\uc815\ub418\uace0 \uccb4\uacc4\uc801\uc778 \ud6c8\ub828\uc744 \ud558\uae30 \uc704\ud574 \uc5f0\uc2b5\uac70\uc810\uc744 \uce90\ub098\ub2e4 \ud1a0\ub860\ud1a0\ub85c \uc62e\uae30\uace0 \ud1a0\ub860\ud1a0 \ud06c\ub9ac\ucf13 \uc2a4\ucf00\uc774\ud305 & \uceec\ub9c1 \ud074\ub7fd \uc5d0\uc11c \ucf54\uce58\uc778 \ube0c\ub77c\uc774\uc5b8 \uc624\uc11c\uc640 \ud568\uaed8 \ud6c8\ub828\ud558\uae30 \uc2dc\uc791\ud588\ub2e4. 2007\ub144 11\uc6d4 8\uc77c\uc5d0 \uc5f4\ub9b0 \ucef5 \uc624\ube0c \ucc28\uc774\ub098\uc5d0\uc11c \uc1fc\ud2b8\ud504\ub85c\uadf8\ub7a8( SP ) \uacfc \ud504\ub9ac\uc2a4\ucf00\uc774\ud305(FS)\uc5d0\uc11c \ud55c \ubc88\uc529\uc758 \uc810\ud504 \uc2e4\uc218\ub97c \ud558\uc600\uc74c\uc5d0\ub3c4 \ubd88\uad6c\ud558\uace0 \uc804\uccb4\uc801\uc73c\ub85c \uc548\uc815\ub418\uace0 \uc9c8\ub192\uc740 \uc5f0\uae30\ub97c \uc120\ubcf4\uc5ec \ud504\ub9ac\uc2a4\ucf00\uc774\ud305\uc5d0\uc11c 122.36\uc810\uc73c\ub85c \uc790\uc2e0\uc758 \ucd5c\uace0\uae30\ub85d\uc744 \uac31\uc2e0\ud558\uba70 \uc6b0\uc2b9\uc744 \ucc28\uc9c0\ud588\ub2e4. 11\uc6d4 24\uc77c\uc5d0\ub294 \ub7ec\uc2dc\uc544 \ubaa8\uc2a4\ud06c\ubc14\uc5d0\uc11c \uc5f4\ub9b0 \ucef5 \uc624\ube0c \ub7ec\uc2dc\uc544\uc5d0 \ucc38\uac00\ud558\uc5ec \ud504\ub9ac\uc640 \ucd1d\uc810\uc5d0\uc11c \uac1c\uc778\ucd5c\uace0\uae30\ub85d\uc744 \uac31\uc2e0\ud558\uba74\uc11c \uc6b0\uc2b9\uc744 \ucc28\uc9c0\ud558\uc600\ub2e4. \ud2b9\ud788 \uc774 \ub300\ud68c \ud504\ub9ac\uc2a4\ucf00\uc774\ud305\uc5d0\uc11c \uae30\ub85d\ud55c 133.70\uc810\uc740 \uc774\uc804\uae4c\uc9c0 \uc138\uacc4 \ucd5c\uace0 \uae30\ub85d\uc774\uc5c8\ub358 \uc544\uc0ac\ub2e4 \ub9c8\uc624\uc758 133.13\uc810\uc744 \ub118\uc5b4\uc11c\ub294 \uc138\uacc4\uc2e0\uae30\ub85d\uc774\uc5c8\ub2e4.", "paragraph_answer": "2007\ub144 5\uc6d4\ubd80\ud130 \uc548\uc815\ub418\uace0 \uccb4\uacc4\uc801\uc778 \ud6c8\ub828\uc744 \ud558\uae30 \uc704\ud574 \uc5f0\uc2b5\uac70\uc810\uc744 \uce90\ub098\ub2e4 \ud1a0\ub860\ud1a0\ub85c \uc62e\uae30\uace0 \ud1a0\ub860\ud1a0 \ud06c\ub9ac\ucf13 \uc2a4\ucf00\uc774\ud305 & \uceec\ub9c1 \ud074\ub7fd \uc5d0\uc11c \ucf54\uce58\uc778 \ube0c\ub77c\uc774\uc5b8 \uc624\uc11c\uc640 \ud568\uaed8 \ud6c8\ub828\ud558\uae30 \uc2dc\uc791\ud588\ub2e4. 2007\ub144 11\uc6d4 8\uc77c\uc5d0 \uc5f4\ub9b0 \ucef5 \uc624\ube0c \ucc28\uc774\ub098\uc5d0\uc11c \uc1fc\ud2b8\ud504\ub85c\uadf8\ub7a8( SP )\uacfc \ud504\ub9ac\uc2a4\ucf00\uc774\ud305(FS)\uc5d0\uc11c \ud55c \ubc88\uc529\uc758 \uc810\ud504 \uc2e4\uc218\ub97c \ud558\uc600\uc74c\uc5d0\ub3c4 \ubd88\uad6c\ud558\uace0 \uc804\uccb4\uc801\uc73c\ub85c \uc548\uc815\ub418\uace0 \uc9c8\ub192\uc740 \uc5f0\uae30\ub97c \uc120\ubcf4\uc5ec \ud504\ub9ac\uc2a4\ucf00\uc774\ud305\uc5d0\uc11c 122.36\uc810\uc73c\ub85c \uc790\uc2e0\uc758 \ucd5c\uace0\uae30\ub85d\uc744 \uac31\uc2e0\ud558\uba70 \uc6b0\uc2b9\uc744 \ucc28\uc9c0\ud588\ub2e4. 11\uc6d4 24\uc77c\uc5d0\ub294 \ub7ec\uc2dc\uc544 \ubaa8\uc2a4\ud06c\ubc14\uc5d0\uc11c \uc5f4\ub9b0 \ucef5 \uc624\ube0c \ub7ec\uc2dc\uc544\uc5d0 \ucc38\uac00\ud558\uc5ec \ud504\ub9ac\uc640 \ucd1d\uc810\uc5d0\uc11c \uac1c\uc778\ucd5c\uace0\uae30\ub85d\uc744 \uac31\uc2e0\ud558\uba74\uc11c \uc6b0\uc2b9\uc744 \ucc28\uc9c0\ud558\uc600\ub2e4. \ud2b9\ud788 \uc774 \ub300\ud68c \ud504\ub9ac\uc2a4\ucf00\uc774\ud305\uc5d0\uc11c \uae30\ub85d\ud55c 133.70\uc810\uc740 \uc774\uc804\uae4c\uc9c0 \uc138\uacc4 \ucd5c\uace0 \uae30\ub85d\uc774\uc5c8\ub358 \uc544\uc0ac\ub2e4 \ub9c8\uc624\uc758 133.13\uc810\uc744 \ub118\uc5b4\uc11c\ub294 \uc138\uacc4\uc2e0\uae30\ub85d\uc774\uc5c8\ub2e4.", "sentence_answer": "2007\ub144 11\uc6d4 8\uc77c\uc5d0 \uc5f4\ub9b0 \ucef5 \uc624\ube0c \ucc28\uc774\ub098\uc5d0\uc11c \uc1fc\ud2b8\ud504\ub85c\uadf8\ub7a8( SP )", "paragraph_id": "6482022-4-0", "paragraph_question": "question: \uc1fc\ud2b8\ud504\ub85c\uadf8\ub7a8\uc758 \uc57d\uc790\ub294?, context: 2007\ub144 5\uc6d4\ubd80\ud130 \uc548\uc815\ub418\uace0 \uccb4\uacc4\uc801\uc778 \ud6c8\ub828\uc744 \ud558\uae30 \uc704\ud574 \uc5f0\uc2b5\uac70\uc810\uc744 \uce90\ub098\ub2e4 \ud1a0\ub860\ud1a0\ub85c \uc62e\uae30\uace0 \ud1a0\ub860\ud1a0 \ud06c\ub9ac\ucf13 \uc2a4\ucf00\uc774\ud305 & \uceec\ub9c1 \ud074\ub7fd \uc5d0\uc11c \ucf54\uce58\uc778 \ube0c\ub77c\uc774\uc5b8 \uc624\uc11c\uc640 \ud568\uaed8 \ud6c8\ub828\ud558\uae30 \uc2dc\uc791\ud588\ub2e4. 2007\ub144 11\uc6d4 8\uc77c\uc5d0 \uc5f4\ub9b0 \ucef5 \uc624\ube0c \ucc28\uc774\ub098\uc5d0\uc11c \uc1fc\ud2b8\ud504\ub85c\uadf8\ub7a8(SP)\uacfc \ud504\ub9ac\uc2a4\ucf00\uc774\ud305(FS)\uc5d0\uc11c \ud55c \ubc88\uc529\uc758 \uc810\ud504 \uc2e4\uc218\ub97c \ud558\uc600\uc74c\uc5d0\ub3c4 \ubd88\uad6c\ud558\uace0 \uc804\uccb4\uc801\uc73c\ub85c \uc548\uc815\ub418\uace0 \uc9c8\ub192\uc740 \uc5f0\uae30\ub97c \uc120\ubcf4\uc5ec \ud504\ub9ac\uc2a4\ucf00\uc774\ud305\uc5d0\uc11c 122.36\uc810\uc73c\ub85c \uc790\uc2e0\uc758 \ucd5c\uace0\uae30\ub85d\uc744 \uac31\uc2e0\ud558\uba70 \uc6b0\uc2b9\uc744 \ucc28\uc9c0\ud588\ub2e4. 11\uc6d4 24\uc77c\uc5d0\ub294 \ub7ec\uc2dc\uc544 \ubaa8\uc2a4\ud06c\ubc14\uc5d0\uc11c \uc5f4\ub9b0 \ucef5 \uc624\ube0c \ub7ec\uc2dc\uc544\uc5d0 \ucc38\uac00\ud558\uc5ec \ud504\ub9ac\uc640 \ucd1d\uc810\uc5d0\uc11c \uac1c\uc778\ucd5c\uace0\uae30\ub85d\uc744 \uac31\uc2e0\ud558\uba74\uc11c \uc6b0\uc2b9\uc744 \ucc28\uc9c0\ud558\uc600\ub2e4. \ud2b9\ud788 \uc774 \ub300\ud68c \ud504\ub9ac\uc2a4\ucf00\uc774\ud305\uc5d0\uc11c \uae30\ub85d\ud55c 133.70\uc810\uc740 \uc774\uc804\uae4c\uc9c0 \uc138\uacc4 \ucd5c\uace0 \uae30\ub85d\uc774\uc5c8\ub358 \uc544\uc0ac\ub2e4 \ub9c8\uc624\uc758 133.13\uc810\uc744 \ub118\uc5b4\uc11c\ub294 \uc138\uacc4\uc2e0\uae30\ub85d\uc774\uc5c8\ub2e4."} {"question": "\uc2e0\ud765\ud559\uad50 \ud559\uc0dd\ub4e4\uc774 \ud559\uc0dd\uc6b4\ub3d9\uc744 \ubc8c\uc778 \ub0a0\uc9dc\ub294?", "paragraph": "\"\uad11\uc8fc\ud559\uc0dd\ud56d\uc77c\uc6b4\ub3d9\"\uc774 \uc2dc\uc791\ub41c\uc9c0 3\ub2ec \ud6c4\uc778 1930\ub144 1\uc6d4 25\uc77c \uc2e0\ud765\ud559\uad50 \ud559\uc0dd\ub4e4\uc740 \ud559\uc0dd\uc6b4\ub3d9\uc744 \ubc8c\uc600\ub2e4. \uad50\uc6b0\ud68c \ubb38\uc608\ubd80\uc7a5 \ubc15\ubb38\uc218(\ud6c4\uc758 \uc774\ub984\uc740 \ubc15\ucca0\uc6c5), \uc6b4\ub3d9\ubd80\uc7a5 \uc774\uc7ac\uc601(\ud6c4\uc758 \uc774\ub984\uc740 \uc774\ucca0), \uadf8\ub9ac\uace0 \ubb38\uc608\ubd80\uc758 \ubb38\uc608\uacc4\uc640 \uc11c\ubb34\ubd80\uc758 \uc0ac\uad50\uacc4\ub97c \ub9e1\uc558\ub358 \ud568\uc218\ub9cc, \ubb38\uc608\ubd80 \ub3c4\uc11c\uacc4\uc640 \uc11c\ubb34\ubd80 \uae30\ub85d\uacc4\ub97c \ub9e1\uc558\ub358 \uc6d0\uc6a9\ub355 \ub4f1\uc774 \uc911\uc2ec\uc774 \ub418\uc5b4 12\uc6d4 2\uc77c \uac1c\uad50 \uae30\ub150 \ud589\uc0ac \ud6c4 \uac70\uc0ac\ud560 \uc608\uc815\uc774\uc5c8\uc73c\ub098 \uc5f0\uae30\ub418\uc5c8\ub2e4. \uc774 \ud6c4 12\uc6d4 12\uc77c\uc5d0 \ub2e4\uc2dc \uac70\uc0ac\ud558\ub824 \ud558\uc600\uc73c\ub098 \ud559\uad50 \ub2f9\uad6d\uc774 \uc0ac\ud0dc\ub97c \ub208\uce58 \ucc44\uace0 \uc870\uae30 \ubc29\ud559\uc5d0 \ub4e4\uc5b4\uac10\uc73c\ub85c \ubb34\uc0b0\ub418\uc5c8\ub2e4. \uac1c\ud559(1\uc6d4 25\uc77c) \uc774 \ud6c4 \uc7ac\ucc28 \uac70\uc0ac\ub97c \ud560 \uacc4\ud68d\uc774\uc5c8\uc9c0\ub9cc \ud559\uad50\uc5d0\uc11c\ub294 \uac1c\ud559 \uc774\ud6c4\uc5d0\ub3c4 \ud559\uc0dd\ub4e4\uc5d0\uac8c \ub4f1\uad50\ud558\uc9c0 \ub9d0\ub77c\uace0 \ud588\ub2e4. \uc774\uc5d0 \uac04\ubd80\ub4e4\uc740 \uae30\uc219\uc0ac\uc5d0 \ubaa8\uc5ec \uc788\ub358 \ud559\uc0dd\ub4e4\uc744 \uaddc\ud569\ud558\uc5ec 1\uc6d4 25\uc77c \uac70\uc0ac \ud558\uae30\ub85c \uacb0\uc815\ud558\uc600\ub2e4. \ud559\uc0dd\ub4e4\uc740 \ube5b\uc774 \uc0c8\uc5b4\ub098\uac00\uc9c0 \ubabb\ud558\ub3c4\ub85d \ud574\ub193\uace0 \ud0dc\uadf9\uae30\uc640 \uc804\ub2e8\uc744 \ubd93\uc73c\ub85c \uadf8\ub824\uac00\uba70, \uac70\uc0ac\ub97c \uc900\ube44\ud588\ub2e4. 1\uc6d4 25\uc77c \uc544\uce68 8\uc2dc 30\ubd84 \uac04\ubd80\ub4e4\uc740 \uae30\uc219\uc0ac\uc5d0 \uc788\ub358 \ud559\uc0dd\ub4e4\uc744 \ubaa8\uc544 \ud0dc\uadf9\uae30\uc640 \uc804\ub2e8\uc9c0\ub97c \ub098\ub204\uc5b4 \uc8fc\uace0 \ub2e4\uac00\uad50 \ucabd\uc5d0\uc11c \ub4f1\uad50\ud558\ub294 \ud559\uc0dd\ub4e4\uc744 \uaddc\ud569\ud558\uc5ec 80\uc5ec\uba85\uc744 \ubaa8\uc558\ub2e4. 9\uc2dc 30\ubd84 \uacb0\uc758\ubb38\uc744 \ub0ad\ub3c5\ud558\uace0 \uc804\ub2e8\uc744 \uc0b4\ud3ec\ud558\uba74\uc11c \"\uc790\uc8fc\ub3c5\ub9bd\ub9cc\uc138! \ud559\uc0dd\ub9cc\uc138! \u540c\u611f\ub9cc\uc138! (\uad11\uc8fc\ud559\uc0dd\ub4e4\uc758 \uc6b4\ub3d9\uc5d0 \u540c\u611f\ud55c\ub2e4\ub294 \ub73b)\" \ub4f1\uc744 \uc678\uce58\uba70 \ub2e4\uac00\uad50\ub97c \uac74\ub108 \uc2dc\ub0b4\ub85c \uc9c4\ucd9c\ud588\ub2e4. \uc2dc\uc704\ud559\uc0dd\ub4e4\uc740 \uc911\uc559\ub3d9\uc73c\ub85c \uc9c4\ucd9c\ud558\uc600\uace0, \ucd9c\ub3d9\ud55c \uc77c\ubcf8\uacbd\ucc30\ub4e4\uc740 \ucd1d\uc744 \uc3d8\uba70 \ud559\uc0dd\ub4e4\uc744 \uc5f0\ud589\ud588\ub2e4. \uc774 \ub0a0 \uacbd\ucc30\uc5d0 \uc5f0\ud589\ub41c \ud559\uc0dd \ucd1d\uc218\ub294 36\uba85\uc774\uc5c8\ub2e4. \uac80\uac70\ub41c \ud559\uc0dd\uc5d0\uac8c \ucd5c\uace0 29\uc77c, \ucd5c\ud558 15\uc77c \uad6c\ub958\ucc98\ubd84\uc744 \uc989\uacb0\ud558\uc5ec \ub3d9\uc77c \uc624\ud6c4 4\uc2dc\uacbd\uc5d0 \uc804\uc8fc \ud615\ubb34\uc18c\ub85c \ub118\uaca8\uc84c\ub2e4. \uc8fc\ub3d9\uc790\uc758 \ud55c\uc0ac\ub78c\uc778 \ud568\uc218\ub9cc\uc740 \uac10\uc625\uc744 \ub098\uc628 \ud6c4 \ud6c4\uc720\uc99d\uc73c\ub85c \uc608\uc218\ubcd1\uc6d0\uc5d0 \uc785\uc6d0\uc744 \ud558\uc600\uace0, 1930\ub144 5\uc6d4 19\uc77c \uc608\uc218\ubcd1\uc6d0\uc5d0\uc11c \ubd84\uc0ac\ud558\uace0 \ub9d0\uc558\ub2e4. \ud574\ubc29 \ud6c4 \ud568\uc218\ub9cc\uc5d0\uac8c\ub294 \uacf5\uc801\uc774 \uc778\uc815\ub418\uc5b4 1991\ub144\uc5d0 \uac74\uad6d\ud6c8\uc7a5 \uc560\uad6d\uc7a5(1980\ub144 \ub300\ud1b5\ub839 \ud45c\ucc3d)\uc774 \uc218\uc5ec\ub418\uc5c8\ub2e4.", "answer": "1930\ub144 1\uc6d4 25\uc77c", "sentence": "\"\uad11\uc8fc\ud559\uc0dd\ud56d\uc77c\uc6b4\ub3d9\"\uc774 \uc2dc\uc791\ub41c\uc9c0 3\ub2ec \ud6c4\uc778 1930\ub144 1\uc6d4 25\uc77c \uc2e0\ud765\ud559\uad50 \ud559\uc0dd\ub4e4\uc740 \ud559\uc0dd\uc6b4\ub3d9\uc744 \ubc8c\uc600\ub2e4.", "paragraph_sentence": " \"\uad11\uc8fc\ud559\uc0dd\ud56d\uc77c\uc6b4\ub3d9\"\uc774 \uc2dc\uc791\ub41c\uc9c0 3\ub2ec \ud6c4\uc778 1930\ub144 1\uc6d4 25\uc77c \uc2e0\ud765\ud559\uad50 \ud559\uc0dd\ub4e4\uc740 \ud559\uc0dd\uc6b4\ub3d9\uc744 \ubc8c\uc600\ub2e4. \uad50\uc6b0\ud68c \ubb38\uc608\ubd80\uc7a5 \ubc15\ubb38\uc218(\ud6c4\uc758 \uc774\ub984\uc740 \ubc15\ucca0\uc6c5), \uc6b4\ub3d9\ubd80\uc7a5 \uc774\uc7ac\uc601(\ud6c4\uc758 \uc774\ub984\uc740 \uc774\ucca0), \uadf8\ub9ac\uace0 \ubb38\uc608\ubd80\uc758 \ubb38\uc608\uacc4\uc640 \uc11c\ubb34\ubd80\uc758 \uc0ac\uad50\uacc4\ub97c \ub9e1\uc558\ub358 \ud568\uc218\ub9cc, \ubb38\uc608\ubd80 \ub3c4\uc11c\uacc4\uc640 \uc11c\ubb34\ubd80 \uae30\ub85d\uacc4\ub97c \ub9e1\uc558\ub358 \uc6d0\uc6a9\ub355 \ub4f1\uc774 \uc911\uc2ec\uc774 \ub418\uc5b4 12\uc6d4 2\uc77c \uac1c\uad50 \uae30\ub150 \ud589\uc0ac \ud6c4 \uac70\uc0ac\ud560 \uc608\uc815\uc774\uc5c8\uc73c\ub098 \uc5f0\uae30\ub418\uc5c8\ub2e4. \uc774 \ud6c4 12\uc6d4 12\uc77c\uc5d0 \ub2e4\uc2dc \uac70\uc0ac\ud558\ub824 \ud558\uc600\uc73c\ub098 \ud559\uad50 \ub2f9\uad6d\uc774 \uc0ac\ud0dc\ub97c \ub208\uce58 \ucc44\uace0 \uc870\uae30 \ubc29\ud559\uc5d0 \ub4e4\uc5b4\uac10\uc73c\ub85c \ubb34\uc0b0\ub418\uc5c8\ub2e4. \uac1c\ud559(1\uc6d4 25\uc77c) \uc774 \ud6c4 \uc7ac\ucc28 \uac70\uc0ac\ub97c \ud560 \uacc4\ud68d\uc774\uc5c8\uc9c0\ub9cc \ud559\uad50\uc5d0\uc11c\ub294 \uac1c\ud559 \uc774\ud6c4\uc5d0\ub3c4 \ud559\uc0dd\ub4e4\uc5d0\uac8c \ub4f1\uad50\ud558\uc9c0 \ub9d0\ub77c\uace0 \ud588\ub2e4. \uc774\uc5d0 \uac04\ubd80\ub4e4\uc740 \uae30\uc219\uc0ac\uc5d0 \ubaa8\uc5ec \uc788\ub358 \ud559\uc0dd\ub4e4\uc744 \uaddc\ud569\ud558\uc5ec 1\uc6d4 25\uc77c \uac70\uc0ac \ud558\uae30\ub85c \uacb0\uc815\ud558\uc600\ub2e4. \ud559\uc0dd\ub4e4\uc740 \ube5b\uc774 \uc0c8\uc5b4\ub098\uac00\uc9c0 \ubabb\ud558\ub3c4\ub85d \ud574\ub193\uace0 \ud0dc\uadf9\uae30\uc640 \uc804\ub2e8\uc744 \ubd93\uc73c\ub85c \uadf8\ub824\uac00\uba70, \uac70\uc0ac\ub97c \uc900\ube44\ud588\ub2e4. 1\uc6d4 25\uc77c \uc544\uce68 8\uc2dc 30\ubd84 \uac04\ubd80\ub4e4\uc740 \uae30\uc219\uc0ac\uc5d0 \uc788\ub358 \ud559\uc0dd\ub4e4\uc744 \ubaa8\uc544 \ud0dc\uadf9\uae30\uc640 \uc804\ub2e8\uc9c0\ub97c \ub098\ub204\uc5b4 \uc8fc\uace0 \ub2e4\uac00\uad50 \ucabd\uc5d0\uc11c \ub4f1\uad50\ud558\ub294 \ud559\uc0dd\ub4e4\uc744 \uaddc\ud569\ud558\uc5ec 80\uc5ec\uba85\uc744 \ubaa8\uc558\ub2e4. 9\uc2dc 30\ubd84 \uacb0\uc758\ubb38\uc744 \ub0ad\ub3c5\ud558\uace0 \uc804\ub2e8\uc744 \uc0b4\ud3ec\ud558\uba74\uc11c \"\uc790\uc8fc\ub3c5\ub9bd\ub9cc\uc138! \ud559\uc0dd\ub9cc\uc138! \u540c\u611f\ub9cc\uc138! (\uad11\uc8fc\ud559\uc0dd\ub4e4\uc758 \uc6b4\ub3d9\uc5d0 \u540c\u611f\ud55c\ub2e4\ub294 \ub73b)\" \ub4f1\uc744 \uc678\uce58\uba70 \ub2e4\uac00\uad50\ub97c \uac74\ub108 \uc2dc\ub0b4\ub85c \uc9c4\ucd9c\ud588\ub2e4. \uc2dc\uc704\ud559\uc0dd\ub4e4\uc740 \uc911\uc559\ub3d9\uc73c\ub85c \uc9c4\ucd9c\ud558\uc600\uace0, \ucd9c\ub3d9\ud55c \uc77c\ubcf8\uacbd\ucc30\ub4e4\uc740 \ucd1d\uc744 \uc3d8\uba70 \ud559\uc0dd\ub4e4\uc744 \uc5f0\ud589\ud588\ub2e4. \uc774 \ub0a0 \uacbd\ucc30\uc5d0 \uc5f0\ud589\ub41c \ud559\uc0dd \ucd1d\uc218\ub294 36\uba85\uc774\uc5c8\ub2e4. \uac80\uac70\ub41c \ud559\uc0dd\uc5d0\uac8c \ucd5c\uace0 29\uc77c, \ucd5c\ud558 15\uc77c \uad6c\ub958\ucc98\ubd84\uc744 \uc989\uacb0\ud558\uc5ec \ub3d9\uc77c \uc624\ud6c4 4\uc2dc\uacbd\uc5d0 \uc804\uc8fc \ud615\ubb34\uc18c\ub85c \ub118\uaca8\uc84c\ub2e4. \uc8fc\ub3d9\uc790\uc758 \ud55c\uc0ac\ub78c\uc778 \ud568\uc218\ub9cc\uc740 \uac10\uc625\uc744 \ub098\uc628 \ud6c4 \ud6c4\uc720\uc99d\uc73c\ub85c \uc608\uc218\ubcd1\uc6d0\uc5d0 \uc785\uc6d0\uc744 \ud558\uc600\uace0, 1930\ub144 5\uc6d4 19\uc77c \uc608\uc218\ubcd1\uc6d0\uc5d0\uc11c \ubd84\uc0ac\ud558\uace0 \ub9d0\uc558\ub2e4. \ud574\ubc29 \ud6c4 \ud568\uc218\ub9cc\uc5d0\uac8c\ub294 \uacf5\uc801\uc774 \uc778\uc815\ub418\uc5b4 1991\ub144\uc5d0 \uac74\uad6d\ud6c8\uc7a5 \uc560\uad6d\uc7a5(1980\ub144 \ub300\ud1b5\ub839 \ud45c\ucc3d)\uc774 \uc218\uc5ec\ub418\uc5c8\ub2e4.", "paragraph_answer": "\"\uad11\uc8fc\ud559\uc0dd\ud56d\uc77c\uc6b4\ub3d9\"\uc774 \uc2dc\uc791\ub41c\uc9c0 3\ub2ec \ud6c4\uc778 1930\ub144 1\uc6d4 25\uc77c \uc2e0\ud765\ud559\uad50 \ud559\uc0dd\ub4e4\uc740 \ud559\uc0dd\uc6b4\ub3d9\uc744 \ubc8c\uc600\ub2e4. \uad50\uc6b0\ud68c \ubb38\uc608\ubd80\uc7a5 \ubc15\ubb38\uc218(\ud6c4\uc758 \uc774\ub984\uc740 \ubc15\ucca0\uc6c5), \uc6b4\ub3d9\ubd80\uc7a5 \uc774\uc7ac\uc601(\ud6c4\uc758 \uc774\ub984\uc740 \uc774\ucca0), \uadf8\ub9ac\uace0 \ubb38\uc608\ubd80\uc758 \ubb38\uc608\uacc4\uc640 \uc11c\ubb34\ubd80\uc758 \uc0ac\uad50\uacc4\ub97c \ub9e1\uc558\ub358 \ud568\uc218\ub9cc, \ubb38\uc608\ubd80 \ub3c4\uc11c\uacc4\uc640 \uc11c\ubb34\ubd80 \uae30\ub85d\uacc4\ub97c \ub9e1\uc558\ub358 \uc6d0\uc6a9\ub355 \ub4f1\uc774 \uc911\uc2ec\uc774 \ub418\uc5b4 12\uc6d4 2\uc77c \uac1c\uad50 \uae30\ub150 \ud589\uc0ac \ud6c4 \uac70\uc0ac\ud560 \uc608\uc815\uc774\uc5c8\uc73c\ub098 \uc5f0\uae30\ub418\uc5c8\ub2e4. \uc774 \ud6c4 12\uc6d4 12\uc77c\uc5d0 \ub2e4\uc2dc \uac70\uc0ac\ud558\ub824 \ud558\uc600\uc73c\ub098 \ud559\uad50 \ub2f9\uad6d\uc774 \uc0ac\ud0dc\ub97c \ub208\uce58 \ucc44\uace0 \uc870\uae30 \ubc29\ud559\uc5d0 \ub4e4\uc5b4\uac10\uc73c\ub85c \ubb34\uc0b0\ub418\uc5c8\ub2e4. \uac1c\ud559(1\uc6d4 25\uc77c) \uc774 \ud6c4 \uc7ac\ucc28 \uac70\uc0ac\ub97c \ud560 \uacc4\ud68d\uc774\uc5c8\uc9c0\ub9cc \ud559\uad50\uc5d0\uc11c\ub294 \uac1c\ud559 \uc774\ud6c4\uc5d0\ub3c4 \ud559\uc0dd\ub4e4\uc5d0\uac8c \ub4f1\uad50\ud558\uc9c0 \ub9d0\ub77c\uace0 \ud588\ub2e4. \uc774\uc5d0 \uac04\ubd80\ub4e4\uc740 \uae30\uc219\uc0ac\uc5d0 \ubaa8\uc5ec \uc788\ub358 \ud559\uc0dd\ub4e4\uc744 \uaddc\ud569\ud558\uc5ec 1\uc6d4 25\uc77c \uac70\uc0ac \ud558\uae30\ub85c \uacb0\uc815\ud558\uc600\ub2e4. \ud559\uc0dd\ub4e4\uc740 \ube5b\uc774 \uc0c8\uc5b4\ub098\uac00\uc9c0 \ubabb\ud558\ub3c4\ub85d \ud574\ub193\uace0 \ud0dc\uadf9\uae30\uc640 \uc804\ub2e8\uc744 \ubd93\uc73c\ub85c \uadf8\ub824\uac00\uba70, \uac70\uc0ac\ub97c \uc900\ube44\ud588\ub2e4. 1\uc6d4 25\uc77c \uc544\uce68 8\uc2dc 30\ubd84 \uac04\ubd80\ub4e4\uc740 \uae30\uc219\uc0ac\uc5d0 \uc788\ub358 \ud559\uc0dd\ub4e4\uc744 \ubaa8\uc544 \ud0dc\uadf9\uae30\uc640 \uc804\ub2e8\uc9c0\ub97c \ub098\ub204\uc5b4 \uc8fc\uace0 \ub2e4\uac00\uad50 \ucabd\uc5d0\uc11c \ub4f1\uad50\ud558\ub294 \ud559\uc0dd\ub4e4\uc744 \uaddc\ud569\ud558\uc5ec 80\uc5ec\uba85\uc744 \ubaa8\uc558\ub2e4. 9\uc2dc 30\ubd84 \uacb0\uc758\ubb38\uc744 \ub0ad\ub3c5\ud558\uace0 \uc804\ub2e8\uc744 \uc0b4\ud3ec\ud558\uba74\uc11c \"\uc790\uc8fc\ub3c5\ub9bd\ub9cc\uc138! \ud559\uc0dd\ub9cc\uc138! \u540c\u611f\ub9cc\uc138! (\uad11\uc8fc\ud559\uc0dd\ub4e4\uc758 \uc6b4\ub3d9\uc5d0 \u540c\u611f\ud55c\ub2e4\ub294 \ub73b)\" \ub4f1\uc744 \uc678\uce58\uba70 \ub2e4\uac00\uad50\ub97c \uac74\ub108 \uc2dc\ub0b4\ub85c \uc9c4\ucd9c\ud588\ub2e4. \uc2dc\uc704\ud559\uc0dd\ub4e4\uc740 \uc911\uc559\ub3d9\uc73c\ub85c \uc9c4\ucd9c\ud558\uc600\uace0, \ucd9c\ub3d9\ud55c \uc77c\ubcf8\uacbd\ucc30\ub4e4\uc740 \ucd1d\uc744 \uc3d8\uba70 \ud559\uc0dd\ub4e4\uc744 \uc5f0\ud589\ud588\ub2e4. \uc774 \ub0a0 \uacbd\ucc30\uc5d0 \uc5f0\ud589\ub41c \ud559\uc0dd \ucd1d\uc218\ub294 36\uba85\uc774\uc5c8\ub2e4. \uac80\uac70\ub41c \ud559\uc0dd\uc5d0\uac8c \ucd5c\uace0 29\uc77c, \ucd5c\ud558 15\uc77c \uad6c\ub958\ucc98\ubd84\uc744 \uc989\uacb0\ud558\uc5ec \ub3d9\uc77c \uc624\ud6c4 4\uc2dc\uacbd\uc5d0 \uc804\uc8fc \ud615\ubb34\uc18c\ub85c \ub118\uaca8\uc84c\ub2e4. \uc8fc\ub3d9\uc790\uc758 \ud55c\uc0ac\ub78c\uc778 \ud568\uc218\ub9cc\uc740 \uac10\uc625\uc744 \ub098\uc628 \ud6c4 \ud6c4\uc720\uc99d\uc73c\ub85c \uc608\uc218\ubcd1\uc6d0\uc5d0 \uc785\uc6d0\uc744 \ud558\uc600\uace0, 1930\ub144 5\uc6d4 19\uc77c \uc608\uc218\ubcd1\uc6d0\uc5d0\uc11c \ubd84\uc0ac\ud558\uace0 \ub9d0\uc558\ub2e4. \ud574\ubc29 \ud6c4 \ud568\uc218\ub9cc\uc5d0\uac8c\ub294 \uacf5\uc801\uc774 \uc778\uc815\ub418\uc5b4 1991\ub144\uc5d0 \uac74\uad6d\ud6c8\uc7a5 \uc560\uad6d\uc7a5(1980\ub144 \ub300\ud1b5\ub839 \ud45c\ucc3d)\uc774 \uc218\uc5ec\ub418\uc5c8\ub2e4.", "sentence_answer": "\"\uad11\uc8fc\ud559\uc0dd\ud56d\uc77c\uc6b4\ub3d9\"\uc774 \uc2dc\uc791\ub41c\uc9c0 3\ub2ec \ud6c4\uc778 1930\ub144 1\uc6d4 25\uc77c \uc2e0\ud765\ud559\uad50 \ud559\uc0dd\ub4e4\uc740 \ud559\uc0dd\uc6b4\ub3d9\uc744 \ubc8c\uc600\ub2e4.", "paragraph_id": "5865150-1-0", "paragraph_question": "question: \uc2e0\ud765\ud559\uad50 \ud559\uc0dd\ub4e4\uc774 \ud559\uc0dd\uc6b4\ub3d9\uc744 \ubc8c\uc778 \ub0a0\uc9dc\ub294?, context: \"\uad11\uc8fc\ud559\uc0dd\ud56d\uc77c\uc6b4\ub3d9\"\uc774 \uc2dc\uc791\ub41c\uc9c0 3\ub2ec \ud6c4\uc778 1930\ub144 1\uc6d4 25\uc77c \uc2e0\ud765\ud559\uad50 \ud559\uc0dd\ub4e4\uc740 \ud559\uc0dd\uc6b4\ub3d9\uc744 \ubc8c\uc600\ub2e4. \uad50\uc6b0\ud68c \ubb38\uc608\ubd80\uc7a5 \ubc15\ubb38\uc218(\ud6c4\uc758 \uc774\ub984\uc740 \ubc15\ucca0\uc6c5), \uc6b4\ub3d9\ubd80\uc7a5 \uc774\uc7ac\uc601(\ud6c4\uc758 \uc774\ub984\uc740 \uc774\ucca0), \uadf8\ub9ac\uace0 \ubb38\uc608\ubd80\uc758 \ubb38\uc608\uacc4\uc640 \uc11c\ubb34\ubd80\uc758 \uc0ac\uad50\uacc4\ub97c \ub9e1\uc558\ub358 \ud568\uc218\ub9cc, \ubb38\uc608\ubd80 \ub3c4\uc11c\uacc4\uc640 \uc11c\ubb34\ubd80 \uae30\ub85d\uacc4\ub97c \ub9e1\uc558\ub358 \uc6d0\uc6a9\ub355 \ub4f1\uc774 \uc911\uc2ec\uc774 \ub418\uc5b4 12\uc6d4 2\uc77c \uac1c\uad50 \uae30\ub150 \ud589\uc0ac \ud6c4 \uac70\uc0ac\ud560 \uc608\uc815\uc774\uc5c8\uc73c\ub098 \uc5f0\uae30\ub418\uc5c8\ub2e4. \uc774 \ud6c4 12\uc6d4 12\uc77c\uc5d0 \ub2e4\uc2dc \uac70\uc0ac\ud558\ub824 \ud558\uc600\uc73c\ub098 \ud559\uad50 \ub2f9\uad6d\uc774 \uc0ac\ud0dc\ub97c \ub208\uce58 \ucc44\uace0 \uc870\uae30 \ubc29\ud559\uc5d0 \ub4e4\uc5b4\uac10\uc73c\ub85c \ubb34\uc0b0\ub418\uc5c8\ub2e4. \uac1c\ud559(1\uc6d4 25\uc77c) \uc774 \ud6c4 \uc7ac\ucc28 \uac70\uc0ac\ub97c \ud560 \uacc4\ud68d\uc774\uc5c8\uc9c0\ub9cc \ud559\uad50\uc5d0\uc11c\ub294 \uac1c\ud559 \uc774\ud6c4\uc5d0\ub3c4 \ud559\uc0dd\ub4e4\uc5d0\uac8c \ub4f1\uad50\ud558\uc9c0 \ub9d0\ub77c\uace0 \ud588\ub2e4. \uc774\uc5d0 \uac04\ubd80\ub4e4\uc740 \uae30\uc219\uc0ac\uc5d0 \ubaa8\uc5ec \uc788\ub358 \ud559\uc0dd\ub4e4\uc744 \uaddc\ud569\ud558\uc5ec 1\uc6d4 25\uc77c \uac70\uc0ac \ud558\uae30\ub85c \uacb0\uc815\ud558\uc600\ub2e4. \ud559\uc0dd\ub4e4\uc740 \ube5b\uc774 \uc0c8\uc5b4\ub098\uac00\uc9c0 \ubabb\ud558\ub3c4\ub85d \ud574\ub193\uace0 \ud0dc\uadf9\uae30\uc640 \uc804\ub2e8\uc744 \ubd93\uc73c\ub85c \uadf8\ub824\uac00\uba70, \uac70\uc0ac\ub97c \uc900\ube44\ud588\ub2e4. 1\uc6d4 25\uc77c \uc544\uce68 8\uc2dc 30\ubd84 \uac04\ubd80\ub4e4\uc740 \uae30\uc219\uc0ac\uc5d0 \uc788\ub358 \ud559\uc0dd\ub4e4\uc744 \ubaa8\uc544 \ud0dc\uadf9\uae30\uc640 \uc804\ub2e8\uc9c0\ub97c \ub098\ub204\uc5b4 \uc8fc\uace0 \ub2e4\uac00\uad50 \ucabd\uc5d0\uc11c \ub4f1\uad50\ud558\ub294 \ud559\uc0dd\ub4e4\uc744 \uaddc\ud569\ud558\uc5ec 80\uc5ec\uba85\uc744 \ubaa8\uc558\ub2e4. 9\uc2dc 30\ubd84 \uacb0\uc758\ubb38\uc744 \ub0ad\ub3c5\ud558\uace0 \uc804\ub2e8\uc744 \uc0b4\ud3ec\ud558\uba74\uc11c \"\uc790\uc8fc\ub3c5\ub9bd\ub9cc\uc138! \ud559\uc0dd\ub9cc\uc138! \u540c\u611f\ub9cc\uc138! (\uad11\uc8fc\ud559\uc0dd\ub4e4\uc758 \uc6b4\ub3d9\uc5d0 \u540c\u611f\ud55c\ub2e4\ub294 \ub73b)\" \ub4f1\uc744 \uc678\uce58\uba70 \ub2e4\uac00\uad50\ub97c \uac74\ub108 \uc2dc\ub0b4\ub85c \uc9c4\ucd9c\ud588\ub2e4. \uc2dc\uc704\ud559\uc0dd\ub4e4\uc740 \uc911\uc559\ub3d9\uc73c\ub85c \uc9c4\ucd9c\ud558\uc600\uace0, \ucd9c\ub3d9\ud55c \uc77c\ubcf8\uacbd\ucc30\ub4e4\uc740 \ucd1d\uc744 \uc3d8\uba70 \ud559\uc0dd\ub4e4\uc744 \uc5f0\ud589\ud588\ub2e4. \uc774 \ub0a0 \uacbd\ucc30\uc5d0 \uc5f0\ud589\ub41c \ud559\uc0dd \ucd1d\uc218\ub294 36\uba85\uc774\uc5c8\ub2e4. \uac80\uac70\ub41c \ud559\uc0dd\uc5d0\uac8c \ucd5c\uace0 29\uc77c, \ucd5c\ud558 15\uc77c \uad6c\ub958\ucc98\ubd84\uc744 \uc989\uacb0\ud558\uc5ec \ub3d9\uc77c \uc624\ud6c4 4\uc2dc\uacbd\uc5d0 \uc804\uc8fc \ud615\ubb34\uc18c\ub85c \ub118\uaca8\uc84c\ub2e4. \uc8fc\ub3d9\uc790\uc758 \ud55c\uc0ac\ub78c\uc778 \ud568\uc218\ub9cc\uc740 \uac10\uc625\uc744 \ub098\uc628 \ud6c4 \ud6c4\uc720\uc99d\uc73c\ub85c \uc608\uc218\ubcd1\uc6d0\uc5d0 \uc785\uc6d0\uc744 \ud558\uc600\uace0, 1930\ub144 5\uc6d4 19\uc77c \uc608\uc218\ubcd1\uc6d0\uc5d0\uc11c \ubd84\uc0ac\ud558\uace0 \ub9d0\uc558\ub2e4. \ud574\ubc29 \ud6c4 \ud568\uc218\ub9cc\uc5d0\uac8c\ub294 \uacf5\uc801\uc774 \uc778\uc815\ub418\uc5b4 1991\ub144\uc5d0 \uac74\uad6d\ud6c8\uc7a5 \uc560\uad6d\uc7a5(1980\ub144 \ub300\ud1b5\ub839 \ud45c\ucc3d)\uc774 \uc218\uc5ec\ub418\uc5c8\ub2e4."} {"question": "\ud55c\uc131\uc804\uae30\ud68c\uc0ac\ub294 \uad11\ubb34 8\ub144 \ubbf8\uad6d\uc758 \uc5b4\ub290 \ud68c\uc0ac\uc5d0 \ud569\uc790\ub418\uc5c8\ub098?", "paragraph": "\uc774\ud6c4 \uc804\ucc28\uc0ac\uc5c5\uc740 \ucc28\ub7c9\uc774 \uac1c\ubc29\ud615\uc774\uc5b4\uc11c \ucd94\uc704\uc640 \ube44\ubc14\ub78c\uc744 \uc81c\ub300\ub85c \ub9c9\uc544\uc8fc\uc9c0 \ubabb\ud558\uace0 \uc758\uc790 \uc5ed\uc2dc \ub098\ubb34\uc758\uc790\ub85c \ubd88\ud3b8\ud588\uc73c\ub098, \ube60\ub974\uace0 \ud3b8\ub9ac\ud588\uae30 \ub54c\ubb38\uc5d0 \uc0c1\ub2f9\ud788 \uc778\uae30\ub97c \ub04c\uc5b4\uc11c \uc120\ub85c \uc5f0\uc7a5 \uc694\uad6c\uac00 \uc81c\uae30\ub418\uae30\uc5d0 \uc774\ub974\ub800\ub2e4. \uac1c\ud1b5 \ub2f9\ud574\uc5d0 \uc6b4\ud589 \uacc4\ud1b5\uc744 \ub0a8\ub300\ubb38\uae4c\uc9c0 \uc5f0\uc7a5\ud558\uc600\uace0, \uadf8 \uc774\ub4ec\ud574(1900\ub144)\uc5d0 \ub0a8\ub300\ubb38\uc5d0\uc11c \uad6c \uc6a9\uc0b0\uae4c\uc9c0 \uc5f0\uc7a5\ud560 \uac83\uc744 \uacb0\uc815\ud558\uc5ec, \uacf5\uc0ac\ub97c \uc2e4\uc2dc, \uc6b4\ud589\uc744 \uac1c\uc2dc\ud558\uc600\ub2e4. \ud2b9\ud788 \uad6c \uc6a9\uc0b0\uae4c\uc9c0 \uc5f0\uc7a5\ud558\uba74\uc11c, \uc6a9\uc0b0\uc758 \ud3ec\uad6c\ub85c\ubd80\ud130 \uc131\ub0b4\uae4c\uc9c0 \ud654\ubb3c \uc218\uc1a1\uc744 \uc2e4\uc2dc\ud55c \ubc14 \uc788\ub2e4. \uc774\ud6c4 \uc11c\ub300\ubb38\uae4c\uc9c0\uc758 \uc5f0\uc7a5, \uadf8\ub9ac\uace0 \ub0a8\ub300\ubb38\uc5d0\uc11c \uc11c\uc18c\ubb38 \uac04\uc744 \uc758\uc8fc\ub85c\ub97c \ub530\ub77c \uc6b4\ud589\ud558\ub294 \ub8e8\ud2b8\uac00 \ucd94\uac00\ub418\uc5c8\uc73c\ub098, \uc2b9\uac1d\uc774 \ub9ce\uc9c0 \uc54a\uc544 2~3\ub144 \ub4a4\uc5d0\ub294 \ud3d0\uc9c0\ub418\uc5c8\ub2e4. \uc774\ub7ec\ud55c \ub178\uc120 \uc5f0\uc7a5 \uacfc\uc815\uc911 \ud2b9\ud788 \uc11c\ub300\ubb38 \uc5f0\uc7a5 \uc2dc\uc5d0\ub294 \ub2f9\uc2dc \ubd80\uc124\ub41c \uacbd\uc778\ucca0\ub3c4\uc8fc\uc2dd\ud68c\uc0ac\uc758 \uacbd\uc778\ucca0\ub3c4\uc640\uc758 \ud3c9\uba74 \uad50\ucc28 \ubb38\uc81c\uac00 \ubc1c\uc0dd\ud558\uc5ec, \uc77c\ubcf8\uce21\uacfc \uc0c1\ub2f9\ud55c \uc2e0\uacbd\uc804\uc744 \ubc8c\uc774\uae30\ub3c4 \ud588\uc73c\ub098, \uc678\uad50\uc801 \ub178\ub825\uc5d0 \uc758\ud574\uc11c \ubb34\ub9c8\ub420 \uc218 \uc788\uc5c8\ub2e4\uace0 \ud55c\ub2e4. \uc774\ud6c4, \ud55c\uc131\uc804\uae30\ud68c\uc0ac\ub294 \uc790\uae08\ub09c\uc5d0 \ube60\uc838 \ubbf8\uad6d\uc758 \uc5e0\ud30c\uc774\uc5b4 \ud2b8\ub7ec\uc2a4\ud2b8\uc0ac\uc5d0 \ud569\uc790, \uad11\ubb348\ub144(1904\ub144)\uc5d0 \uadf8 \uba85\uce6d\uc744 \ud55c\ubbf8\uc804\uae30\ud68c\uc0ac\ub85c \ubc14\uafb8\uac8c \ub418\uc5c8\ub2e4.", "answer": "\uc5e0\ud30c\uc774\uc5b4 \ud2b8\ub7ec\uc2a4\ud2b8\uc0ac", "sentence": "\ubbf8\uad6d\uc758 \uc5e0\ud30c\uc774\uc5b4 \ud2b8\ub7ec\uc2a4\ud2b8\uc0ac \uc5d0 \ud569\uc790, \uad11\ubb348\ub144(1904\ub144)\uc5d0 \uadf8 \uba85\uce6d\uc744 \ud55c\ubbf8\uc804\uae30\ud68c\uc0ac\ub85c \ubc14\uafb8\uac8c \ub418\uc5c8\ub2e4.", "paragraph_sentence": "\uc774\ud6c4 \uc804\ucc28\uc0ac\uc5c5\uc740 \ucc28\ub7c9\uc774 \uac1c\ubc29\ud615\uc774\uc5b4\uc11c \ucd94\uc704\uc640 \ube44\ubc14\ub78c\uc744 \uc81c\ub300\ub85c \ub9c9\uc544\uc8fc\uc9c0 \ubabb\ud558\uace0 \uc758\uc790 \uc5ed\uc2dc \ub098\ubb34\uc758\uc790\ub85c \ubd88\ud3b8\ud588\uc73c\ub098, \ube60\ub974\uace0 \ud3b8\ub9ac\ud588\uae30 \ub54c\ubb38\uc5d0 \uc0c1\ub2f9\ud788 \uc778\uae30\ub97c \ub04c\uc5b4\uc11c \uc120\ub85c \uc5f0\uc7a5 \uc694\uad6c\uac00 \uc81c\uae30\ub418\uae30\uc5d0 \uc774\ub974\ub800\ub2e4. \uac1c\ud1b5 \ub2f9\ud574\uc5d0 \uc6b4\ud589 \uacc4\ud1b5\uc744 \ub0a8\ub300\ubb38\uae4c\uc9c0 \uc5f0\uc7a5\ud558\uc600\uace0, \uadf8 \uc774\ub4ec\ud574(1900\ub144)\uc5d0 \ub0a8\ub300\ubb38\uc5d0\uc11c \uad6c \uc6a9\uc0b0\uae4c\uc9c0 \uc5f0\uc7a5\ud560 \uac83\uc744 \uacb0\uc815\ud558\uc5ec, \uacf5\uc0ac\ub97c \uc2e4\uc2dc, \uc6b4\ud589\uc744 \uac1c\uc2dc\ud558\uc600\ub2e4. \ud2b9\ud788 \uad6c \uc6a9\uc0b0\uae4c\uc9c0 \uc5f0\uc7a5\ud558\uba74\uc11c, \uc6a9\uc0b0\uc758 \ud3ec\uad6c\ub85c\ubd80\ud130 \uc131\ub0b4\uae4c\uc9c0 \ud654\ubb3c \uc218\uc1a1\uc744 \uc2e4\uc2dc\ud55c \ubc14 \uc788\ub2e4. \uc774\ud6c4 \uc11c\ub300\ubb38\uae4c\uc9c0\uc758 \uc5f0\uc7a5, \uadf8\ub9ac\uace0 \ub0a8\ub300\ubb38\uc5d0\uc11c \uc11c\uc18c\ubb38 \uac04\uc744 \uc758\uc8fc\ub85c\ub97c \ub530\ub77c \uc6b4\ud589\ud558\ub294 \ub8e8\ud2b8\uac00 \ucd94\uac00\ub418\uc5c8\uc73c\ub098, \uc2b9\uac1d\uc774 \ub9ce\uc9c0 \uc54a\uc544 2~3\ub144 \ub4a4\uc5d0\ub294 \ud3d0\uc9c0\ub418\uc5c8\ub2e4. \uc774\ub7ec\ud55c \ub178\uc120 \uc5f0\uc7a5 \uacfc\uc815\uc911 \ud2b9\ud788 \uc11c\ub300\ubb38 \uc5f0\uc7a5 \uc2dc\uc5d0\ub294 \ub2f9\uc2dc \ubd80\uc124\ub41c \uacbd\uc778\ucca0\ub3c4\uc8fc\uc2dd\ud68c\uc0ac\uc758 \uacbd\uc778\ucca0\ub3c4\uc640\uc758 \ud3c9\uba74 \uad50\ucc28 \ubb38\uc81c\uac00 \ubc1c\uc0dd\ud558\uc5ec, \uc77c\ubcf8\uce21\uacfc \uc0c1\ub2f9\ud55c \uc2e0\uacbd\uc804\uc744 \ubc8c\uc774\uae30\ub3c4 \ud588\uc73c\ub098, \uc678\uad50\uc801 \ub178\ub825\uc5d0 \uc758\ud574\uc11c \ubb34\ub9c8\ub420 \uc218 \uc788\uc5c8\ub2e4\uace0 \ud55c\ub2e4. \uc774\ud6c4, \ud55c\uc131\uc804\uae30\ud68c\uc0ac\ub294 \uc790\uae08\ub09c\uc5d0 \ube60\uc838 \ubbf8\uad6d\uc758 \uc5e0\ud30c\uc774\uc5b4 \ud2b8\ub7ec\uc2a4\ud2b8\uc0ac \uc5d0 \ud569\uc790, \uad11\ubb348\ub144(1904\ub144)\uc5d0 \uadf8 \uba85\uce6d\uc744 \ud55c\ubbf8\uc804\uae30\ud68c\uc0ac\ub85c \ubc14\uafb8\uac8c \ub418\uc5c8\ub2e4. ", "paragraph_answer": "\uc774\ud6c4 \uc804\ucc28\uc0ac\uc5c5\uc740 \ucc28\ub7c9\uc774 \uac1c\ubc29\ud615\uc774\uc5b4\uc11c \ucd94\uc704\uc640 \ube44\ubc14\ub78c\uc744 \uc81c\ub300\ub85c \ub9c9\uc544\uc8fc\uc9c0 \ubabb\ud558\uace0 \uc758\uc790 \uc5ed\uc2dc \ub098\ubb34\uc758\uc790\ub85c \ubd88\ud3b8\ud588\uc73c\ub098, \ube60\ub974\uace0 \ud3b8\ub9ac\ud588\uae30 \ub54c\ubb38\uc5d0 \uc0c1\ub2f9\ud788 \uc778\uae30\ub97c \ub04c\uc5b4\uc11c \uc120\ub85c \uc5f0\uc7a5 \uc694\uad6c\uac00 \uc81c\uae30\ub418\uae30\uc5d0 \uc774\ub974\ub800\ub2e4. \uac1c\ud1b5 \ub2f9\ud574\uc5d0 \uc6b4\ud589 \uacc4\ud1b5\uc744 \ub0a8\ub300\ubb38\uae4c\uc9c0 \uc5f0\uc7a5\ud558\uc600\uace0, \uadf8 \uc774\ub4ec\ud574(1900\ub144)\uc5d0 \ub0a8\ub300\ubb38\uc5d0\uc11c \uad6c \uc6a9\uc0b0\uae4c\uc9c0 \uc5f0\uc7a5\ud560 \uac83\uc744 \uacb0\uc815\ud558\uc5ec, \uacf5\uc0ac\ub97c \uc2e4\uc2dc, \uc6b4\ud589\uc744 \uac1c\uc2dc\ud558\uc600\ub2e4. \ud2b9\ud788 \uad6c \uc6a9\uc0b0\uae4c\uc9c0 \uc5f0\uc7a5\ud558\uba74\uc11c, \uc6a9\uc0b0\uc758 \ud3ec\uad6c\ub85c\ubd80\ud130 \uc131\ub0b4\uae4c\uc9c0 \ud654\ubb3c \uc218\uc1a1\uc744 \uc2e4\uc2dc\ud55c \ubc14 \uc788\ub2e4. \uc774\ud6c4 \uc11c\ub300\ubb38\uae4c\uc9c0\uc758 \uc5f0\uc7a5, \uadf8\ub9ac\uace0 \ub0a8\ub300\ubb38\uc5d0\uc11c \uc11c\uc18c\ubb38 \uac04\uc744 \uc758\uc8fc\ub85c\ub97c \ub530\ub77c \uc6b4\ud589\ud558\ub294 \ub8e8\ud2b8\uac00 \ucd94\uac00\ub418\uc5c8\uc73c\ub098, \uc2b9\uac1d\uc774 \ub9ce\uc9c0 \uc54a\uc544 2~3\ub144 \ub4a4\uc5d0\ub294 \ud3d0\uc9c0\ub418\uc5c8\ub2e4. \uc774\ub7ec\ud55c \ub178\uc120 \uc5f0\uc7a5 \uacfc\uc815\uc911 \ud2b9\ud788 \uc11c\ub300\ubb38 \uc5f0\uc7a5 \uc2dc\uc5d0\ub294 \ub2f9\uc2dc \ubd80\uc124\ub41c \uacbd\uc778\ucca0\ub3c4\uc8fc\uc2dd\ud68c\uc0ac\uc758 \uacbd\uc778\ucca0\ub3c4\uc640\uc758 \ud3c9\uba74 \uad50\ucc28 \ubb38\uc81c\uac00 \ubc1c\uc0dd\ud558\uc5ec, \uc77c\ubcf8\uce21\uacfc \uc0c1\ub2f9\ud55c \uc2e0\uacbd\uc804\uc744 \ubc8c\uc774\uae30\ub3c4 \ud588\uc73c\ub098, \uc678\uad50\uc801 \ub178\ub825\uc5d0 \uc758\ud574\uc11c \ubb34\ub9c8\ub420 \uc218 \uc788\uc5c8\ub2e4\uace0 \ud55c\ub2e4. \uc774\ud6c4, \ud55c\uc131\uc804\uae30\ud68c\uc0ac\ub294 \uc790\uae08\ub09c\uc5d0 \ube60\uc838 \ubbf8\uad6d\uc758 \uc5e0\ud30c\uc774\uc5b4 \ud2b8\ub7ec\uc2a4\ud2b8\uc0ac \uc5d0 \ud569\uc790, \uad11\ubb348\ub144(1904\ub144)\uc5d0 \uadf8 \uba85\uce6d\uc744 \ud55c\ubbf8\uc804\uae30\ud68c\uc0ac\ub85c \ubc14\uafb8\uac8c \ub418\uc5c8\ub2e4.", "sentence_answer": "\ubbf8\uad6d\uc758 \uc5e0\ud30c\uc774\uc5b4 \ud2b8\ub7ec\uc2a4\ud2b8\uc0ac \uc5d0 \ud569\uc790, \uad11\ubb348\ub144(1904\ub144)\uc5d0 \uadf8 \uba85\uce6d\uc744 \ud55c\ubbf8\uc804\uae30\ud68c\uc0ac\ub85c \ubc14\uafb8\uac8c \ub418\uc5c8\ub2e4.", "paragraph_id": "6536026-2-1", "paragraph_question": "question: \ud55c\uc131\uc804\uae30\ud68c\uc0ac\ub294 \uad11\ubb34 8\ub144 \ubbf8\uad6d\uc758 \uc5b4\ub290 \ud68c\uc0ac\uc5d0 \ud569\uc790\ub418\uc5c8\ub098?, context: \uc774\ud6c4 \uc804\ucc28\uc0ac\uc5c5\uc740 \ucc28\ub7c9\uc774 \uac1c\ubc29\ud615\uc774\uc5b4\uc11c \ucd94\uc704\uc640 \ube44\ubc14\ub78c\uc744 \uc81c\ub300\ub85c \ub9c9\uc544\uc8fc\uc9c0 \ubabb\ud558\uace0 \uc758\uc790 \uc5ed\uc2dc \ub098\ubb34\uc758\uc790\ub85c \ubd88\ud3b8\ud588\uc73c\ub098, \ube60\ub974\uace0 \ud3b8\ub9ac\ud588\uae30 \ub54c\ubb38\uc5d0 \uc0c1\ub2f9\ud788 \uc778\uae30\ub97c \ub04c\uc5b4\uc11c \uc120\ub85c \uc5f0\uc7a5 \uc694\uad6c\uac00 \uc81c\uae30\ub418\uae30\uc5d0 \uc774\ub974\ub800\ub2e4. \uac1c\ud1b5 \ub2f9\ud574\uc5d0 \uc6b4\ud589 \uacc4\ud1b5\uc744 \ub0a8\ub300\ubb38\uae4c\uc9c0 \uc5f0\uc7a5\ud558\uc600\uace0, \uadf8 \uc774\ub4ec\ud574(1900\ub144)\uc5d0 \ub0a8\ub300\ubb38\uc5d0\uc11c \uad6c \uc6a9\uc0b0\uae4c\uc9c0 \uc5f0\uc7a5\ud560 \uac83\uc744 \uacb0\uc815\ud558\uc5ec, \uacf5\uc0ac\ub97c \uc2e4\uc2dc, \uc6b4\ud589\uc744 \uac1c\uc2dc\ud558\uc600\ub2e4. \ud2b9\ud788 \uad6c \uc6a9\uc0b0\uae4c\uc9c0 \uc5f0\uc7a5\ud558\uba74\uc11c, \uc6a9\uc0b0\uc758 \ud3ec\uad6c\ub85c\ubd80\ud130 \uc131\ub0b4\uae4c\uc9c0 \ud654\ubb3c \uc218\uc1a1\uc744 \uc2e4\uc2dc\ud55c \ubc14 \uc788\ub2e4. \uc774\ud6c4 \uc11c\ub300\ubb38\uae4c\uc9c0\uc758 \uc5f0\uc7a5, \uadf8\ub9ac\uace0 \ub0a8\ub300\ubb38\uc5d0\uc11c \uc11c\uc18c\ubb38 \uac04\uc744 \uc758\uc8fc\ub85c\ub97c \ub530\ub77c \uc6b4\ud589\ud558\ub294 \ub8e8\ud2b8\uac00 \ucd94\uac00\ub418\uc5c8\uc73c\ub098, \uc2b9\uac1d\uc774 \ub9ce\uc9c0 \uc54a\uc544 2~3\ub144 \ub4a4\uc5d0\ub294 \ud3d0\uc9c0\ub418\uc5c8\ub2e4. \uc774\ub7ec\ud55c \ub178\uc120 \uc5f0\uc7a5 \uacfc\uc815\uc911 \ud2b9\ud788 \uc11c\ub300\ubb38 \uc5f0\uc7a5 \uc2dc\uc5d0\ub294 \ub2f9\uc2dc \ubd80\uc124\ub41c \uacbd\uc778\ucca0\ub3c4\uc8fc\uc2dd\ud68c\uc0ac\uc758 \uacbd\uc778\ucca0\ub3c4\uc640\uc758 \ud3c9\uba74 \uad50\ucc28 \ubb38\uc81c\uac00 \ubc1c\uc0dd\ud558\uc5ec, \uc77c\ubcf8\uce21\uacfc \uc0c1\ub2f9\ud55c \uc2e0\uacbd\uc804\uc744 \ubc8c\uc774\uae30\ub3c4 \ud588\uc73c\ub098, \uc678\uad50\uc801 \ub178\ub825\uc5d0 \uc758\ud574\uc11c \ubb34\ub9c8\ub420 \uc218 \uc788\uc5c8\ub2e4\uace0 \ud55c\ub2e4. \uc774\ud6c4, \ud55c\uc131\uc804\uae30\ud68c\uc0ac\ub294 \uc790\uae08\ub09c\uc5d0 \ube60\uc838 \ubbf8\uad6d\uc758 \uc5e0\ud30c\uc774\uc5b4 \ud2b8\ub7ec\uc2a4\ud2b8\uc0ac\uc5d0 \ud569\uc790, \uad11\ubb348\ub144(1904\ub144)\uc5d0 \uadf8 \uba85\uce6d\uc744 \ud55c\ubbf8\uc804\uae30\ud68c\uc0ac\ub85c \ubc14\uafb8\uac8c \ub418\uc5c8\ub2e4."} {"question": "\uc0ac\ub78c\uc758 \ub9c8\uc74c \uc568\ubc94\uc5d0 \uc218\ub85d\ub41c \ubcc4 \uc77c \uc5c6\uc5c8\ub2c8\ub294 \uc5b4\ub5a4 \ubc84\uc804\uc5d0 \uc788\ub294 \uace1\uc778\uac00?", "paragraph": "\uc774 \uc568\ubc94\uc5d0\ub294 2013\ub144 3\uc6d4 29\uc77c\uc5d0 \uc6b0\uc120 \uacf5\uac1c\ud588\ub358 \u3008\uc88b\ub2e4 \ub9d0\uc558\ub124\u3009\uac00 \ud3ec\ud568\ub418\uc5b4 \uc788\ub2e4. \uc774\uc804 \uc74c\ubc18 \u300a\uc7a5\uae30\ud558\uc640 \uc5bc\uad74\ub4e4\u300b\uc758 \ubc1c\ub9e4\uc77c\ub85c\ubd80\ud130 3\ub144 4\uac1c\uc6d4 \ub9cc\uc778 2014\ub144 10\uc6d4 15\uc77c\uc5d0 \uacf5\uac1c\ub418\uc5c8\uc73c\uba70, \uc218\ub85d\uace1 \uc911 \ud558\ub098\uc778 \u3008\ub0b4 \uc0ac\ub78c\u3009\uc740 10\uc6d4 7\uc77c\uc5d0 \uc6b0\uc120 \uacf5\uac1c\ub418\uc5c8\ub2e4. \u300a\uc0ac\ub78c\uc758 \ub9c8\uc74c\u300b\uc740 \uc628\ub77c\uc778 \ubc84\uc804\uacfc \uc624\ud504\ub77c\uc778 \ubc84\uc804 \ub4f1 \ub450 \uac00\uc9c0 \ubc84\uc804\uc73c\ub85c \uc81c\uc791\ub410\ub2e4. \uc628\ub77c\uc778 \ubc84\uc804\uc5d0\ub294 \ud0c0\uc774\ud2c0 \uace1 \u3008\uc0ac\ub78c\uc758 \ub9c8\uc74c\u3009\uc744 \ud3ec\ud568\ud574 \ucd1d 12\uac1c \ud2b8\ub799\uc774 \ub2f4\uae30\uba70, \uc624\ud504\ub77c\uc778 \ubc84\uc804 \uc568\ubc94\uc5d0\ub294 \u3008\ubcc4 \uc77c \uc5c6\uc5c8\ub2c8\u3009\uac00 \ubcf4\ub108\uc2a4 \ud2b8\ub799\uc73c\ub85c \uc218\ub85d\ub418\uc5c8\ub2e4. \uc74c\ubc18\uc740 \ubc1c\ud45c\uc640 \ub3d9\uc2dc\uc5d0 \uc88b\uc740 \uc0c1\uc5c5\uc801 \uc131\uacfc\ub97c \uac70\ub450\uc5c8\uace0, \ud3c9\ub860\uac00\uc640 \ub300\uc911\uc758 \ubc18\uc751 \ubaa8\ub450 \ud638\uc758\uc801\uc774\uc5c8\ub2e4. \ubc34\ub4dc\ub294 \uc0c8 \uc568\ubc94 \ubc1c\ub9e4\ub97c \uae30\ub150\ud574 10\uc6d4 10\uc6d4 23\uc77c\ubd80\ud130 \uc11c\uc6b8\uc744 \uc2dc\uc791\uc73c\ub85c \ub300\uc804, \ub300\uad6c, \uc804\uc8fc, \ubd80\uc0b0\uc5d0\uc11c \uc804\uad6d\ud22c\uc5b4\ub97c \uc2e4\uc2dc\ud588\ub2e4.", "answer": "\uc624\ud504\ub77c\uc778 \ubc84\uc804", "sentence": "\u300a\uc0ac\ub78c\uc758 \ub9c8\uc74c\u300b\uc740 \uc628\ub77c\uc778 \ubc84\uc804\uacfc \uc624\ud504\ub77c\uc778 \ubc84\uc804 \ub4f1 \ub450 \uac00\uc9c0 \ubc84\uc804\uc73c\ub85c \uc81c\uc791\ub410\ub2e4.", "paragraph_sentence": "\uc774 \uc568\ubc94\uc5d0\ub294 2013\ub144 3\uc6d4 29\uc77c\uc5d0 \uc6b0\uc120 \uacf5\uac1c\ud588\ub358 \u3008\uc88b\ub2e4 \ub9d0\uc558\ub124\u3009\uac00 \ud3ec\ud568\ub418\uc5b4 \uc788\ub2e4. \uc774\uc804 \uc74c\ubc18 \u300a\uc7a5\uae30\ud558\uc640 \uc5bc\uad74\ub4e4\u300b\uc758 \ubc1c\ub9e4\uc77c\ub85c\ubd80\ud130 3\ub144 4\uac1c\uc6d4 \ub9cc\uc778 2014\ub144 10\uc6d4 15\uc77c\uc5d0 \uacf5\uac1c\ub418\uc5c8\uc73c\uba70, \uc218\ub85d\uace1 \uc911 \ud558\ub098\uc778 \u3008\ub0b4 \uc0ac\ub78c\u3009\uc740 10\uc6d4 7\uc77c\uc5d0 \uc6b0\uc120 \uacf5\uac1c\ub418\uc5c8\ub2e4. \u300a\uc0ac\ub78c\uc758 \ub9c8\uc74c\u300b\uc740 \uc628\ub77c\uc778 \ubc84\uc804\uacfc \uc624\ud504\ub77c\uc778 \ubc84\uc804 \ub4f1 \ub450 \uac00\uc9c0 \ubc84\uc804\uc73c\ub85c \uc81c\uc791\ub410\ub2e4. \uc628\ub77c\uc778 \ubc84\uc804\uc5d0\ub294 \ud0c0\uc774\ud2c0 \uace1 \u3008\uc0ac\ub78c\uc758 \ub9c8\uc74c\u3009\uc744 \ud3ec\ud568\ud574 \ucd1d 12\uac1c \ud2b8\ub799\uc774 \ub2f4\uae30\uba70, \uc624\ud504\ub77c\uc778 \ubc84\uc804 \uc568\ubc94\uc5d0\ub294 \u3008\ubcc4 \uc77c \uc5c6\uc5c8\ub2c8\u3009\uac00 \ubcf4\ub108\uc2a4 \ud2b8\ub799\uc73c\ub85c \uc218\ub85d\ub418\uc5c8\ub2e4. \uc74c\ubc18\uc740 \ubc1c\ud45c\uc640 \ub3d9\uc2dc\uc5d0 \uc88b\uc740 \uc0c1\uc5c5\uc801 \uc131\uacfc\ub97c \uac70\ub450\uc5c8\uace0, \ud3c9\ub860\uac00\uc640 \ub300\uc911\uc758 \ubc18\uc751 \ubaa8\ub450 \ud638\uc758\uc801\uc774\uc5c8\ub2e4. \ubc34\ub4dc\ub294 \uc0c8 \uc568\ubc94 \ubc1c\ub9e4\ub97c \uae30\ub150\ud574 10\uc6d4 10\uc6d4 23\uc77c\ubd80\ud130 \uc11c\uc6b8\uc744 \uc2dc\uc791\uc73c\ub85c \ub300\uc804, \ub300\uad6c, \uc804\uc8fc, \ubd80\uc0b0\uc5d0\uc11c \uc804\uad6d\ud22c\uc5b4\ub97c \uc2e4\uc2dc\ud588\ub2e4.", "paragraph_answer": "\uc774 \uc568\ubc94\uc5d0\ub294 2013\ub144 3\uc6d4 29\uc77c\uc5d0 \uc6b0\uc120 \uacf5\uac1c\ud588\ub358 \u3008\uc88b\ub2e4 \ub9d0\uc558\ub124\u3009\uac00 \ud3ec\ud568\ub418\uc5b4 \uc788\ub2e4. \uc774\uc804 \uc74c\ubc18 \u300a\uc7a5\uae30\ud558\uc640 \uc5bc\uad74\ub4e4\u300b\uc758 \ubc1c\ub9e4\uc77c\ub85c\ubd80\ud130 3\ub144 4\uac1c\uc6d4 \ub9cc\uc778 2014\ub144 10\uc6d4 15\uc77c\uc5d0 \uacf5\uac1c\ub418\uc5c8\uc73c\uba70, \uc218\ub85d\uace1 \uc911 \ud558\ub098\uc778 \u3008\ub0b4 \uc0ac\ub78c\u3009\uc740 10\uc6d4 7\uc77c\uc5d0 \uc6b0\uc120 \uacf5\uac1c\ub418\uc5c8\ub2e4. \u300a\uc0ac\ub78c\uc758 \ub9c8\uc74c\u300b\uc740 \uc628\ub77c\uc778 \ubc84\uc804\uacfc \uc624\ud504\ub77c\uc778 \ubc84\uc804 \ub4f1 \ub450 \uac00\uc9c0 \ubc84\uc804\uc73c\ub85c \uc81c\uc791\ub410\ub2e4. \uc628\ub77c\uc778 \ubc84\uc804\uc5d0\ub294 \ud0c0\uc774\ud2c0 \uace1 \u3008\uc0ac\ub78c\uc758 \ub9c8\uc74c\u3009\uc744 \ud3ec\ud568\ud574 \ucd1d 12\uac1c \ud2b8\ub799\uc774 \ub2f4\uae30\uba70, \uc624\ud504\ub77c\uc778 \ubc84\uc804 \uc568\ubc94\uc5d0\ub294 \u3008\ubcc4 \uc77c \uc5c6\uc5c8\ub2c8\u3009\uac00 \ubcf4\ub108\uc2a4 \ud2b8\ub799\uc73c\ub85c \uc218\ub85d\ub418\uc5c8\ub2e4. \uc74c\ubc18\uc740 \ubc1c\ud45c\uc640 \ub3d9\uc2dc\uc5d0 \uc88b\uc740 \uc0c1\uc5c5\uc801 \uc131\uacfc\ub97c \uac70\ub450\uc5c8\uace0, \ud3c9\ub860\uac00\uc640 \ub300\uc911\uc758 \ubc18\uc751 \ubaa8\ub450 \ud638\uc758\uc801\uc774\uc5c8\ub2e4. \ubc34\ub4dc\ub294 \uc0c8 \uc568\ubc94 \ubc1c\ub9e4\ub97c \uae30\ub150\ud574 10\uc6d4 10\uc6d4 23\uc77c\ubd80\ud130 \uc11c\uc6b8\uc744 \uc2dc\uc791\uc73c\ub85c \ub300\uc804, \ub300\uad6c, \uc804\uc8fc, \ubd80\uc0b0\uc5d0\uc11c \uc804\uad6d\ud22c\uc5b4\ub97c \uc2e4\uc2dc\ud588\ub2e4.", "sentence_answer": "\u300a\uc0ac\ub78c\uc758 \ub9c8\uc74c\u300b\uc740 \uc628\ub77c\uc778 \ubc84\uc804\uacfc \uc624\ud504\ub77c\uc778 \ubc84\uc804 \ub4f1 \ub450 \uac00\uc9c0 \ubc84\uc804\uc73c\ub85c \uc81c\uc791\ub410\ub2e4.", "paragraph_id": "6535321-0-2", "paragraph_question": "question: \uc0ac\ub78c\uc758 \ub9c8\uc74c \uc568\ubc94\uc5d0 \uc218\ub85d\ub41c \ubcc4 \uc77c \uc5c6\uc5c8\ub2c8\ub294 \uc5b4\ub5a4 \ubc84\uc804\uc5d0 \uc788\ub294 \uace1\uc778\uac00?, context: \uc774 \uc568\ubc94\uc5d0\ub294 2013\ub144 3\uc6d4 29\uc77c\uc5d0 \uc6b0\uc120 \uacf5\uac1c\ud588\ub358 \u3008\uc88b\ub2e4 \ub9d0\uc558\ub124\u3009\uac00 \ud3ec\ud568\ub418\uc5b4 \uc788\ub2e4. \uc774\uc804 \uc74c\ubc18 \u300a\uc7a5\uae30\ud558\uc640 \uc5bc\uad74\ub4e4\u300b\uc758 \ubc1c\ub9e4\uc77c\ub85c\ubd80\ud130 3\ub144 4\uac1c\uc6d4 \ub9cc\uc778 2014\ub144 10\uc6d4 15\uc77c\uc5d0 \uacf5\uac1c\ub418\uc5c8\uc73c\uba70, \uc218\ub85d\uace1 \uc911 \ud558\ub098\uc778 \u3008\ub0b4 \uc0ac\ub78c\u3009\uc740 10\uc6d4 7\uc77c\uc5d0 \uc6b0\uc120 \uacf5\uac1c\ub418\uc5c8\ub2e4. \u300a\uc0ac\ub78c\uc758 \ub9c8\uc74c\u300b\uc740 \uc628\ub77c\uc778 \ubc84\uc804\uacfc \uc624\ud504\ub77c\uc778 \ubc84\uc804 \ub4f1 \ub450 \uac00\uc9c0 \ubc84\uc804\uc73c\ub85c \uc81c\uc791\ub410\ub2e4. \uc628\ub77c\uc778 \ubc84\uc804\uc5d0\ub294 \ud0c0\uc774\ud2c0 \uace1 \u3008\uc0ac\ub78c\uc758 \ub9c8\uc74c\u3009\uc744 \ud3ec\ud568\ud574 \ucd1d 12\uac1c \ud2b8\ub799\uc774 \ub2f4\uae30\uba70, \uc624\ud504\ub77c\uc778 \ubc84\uc804 \uc568\ubc94\uc5d0\ub294 \u3008\ubcc4 \uc77c \uc5c6\uc5c8\ub2c8\u3009\uac00 \ubcf4\ub108\uc2a4 \ud2b8\ub799\uc73c\ub85c \uc218\ub85d\ub418\uc5c8\ub2e4. \uc74c\ubc18\uc740 \ubc1c\ud45c\uc640 \ub3d9\uc2dc\uc5d0 \uc88b\uc740 \uc0c1\uc5c5\uc801 \uc131\uacfc\ub97c \uac70\ub450\uc5c8\uace0, \ud3c9\ub860\uac00\uc640 \ub300\uc911\uc758 \ubc18\uc751 \ubaa8\ub450 \ud638\uc758\uc801\uc774\uc5c8\ub2e4. \ubc34\ub4dc\ub294 \uc0c8 \uc568\ubc94 \ubc1c\ub9e4\ub97c \uae30\ub150\ud574 10\uc6d4 10\uc6d4 23\uc77c\ubd80\ud130 \uc11c\uc6b8\uc744 \uc2dc\uc791\uc73c\ub85c \ub300\uc804, \ub300\uad6c, \uc804\uc8fc, \ubd80\uc0b0\uc5d0\uc11c \uc804\uad6d\ud22c\uc5b4\ub97c \uc2e4\uc2dc\ud588\ub2e4."} {"question": "\uba71\uc0b4\uc744 \uc7a1\ub2e4 \ubab8\ubd80\ub9bc\uc73c\ub85c \uc778\ud574 \uba38\ub9ac\uc5d0 \ubd80\ub52a\ud600 \uc5bc\uad74\uc5d0 \ubd80\uc0c1\uc744 \uc785\uc740 \uc9c1\uc6d0\uc758 \uc18c\uc18d\uc744 \ubc1d\ud788\uc9c0 \uc54a\uc558\ub2e4\uac00 \uc548\uba74 \uac00\uaca9\uc744 \ub2f9\ud588\ub2e4\uba70 \uac15\uae30\uc815 \uc758\uc6d0 \ud3ed\ud589\uc0ac\uac74 \ubcf4\ub3c4\ucc38\uace0\uc790\ub8cc\uc5d0\uc11c \ud53c\ud574\uc790\ub85c \uac70\ub860\ud55c \uc21c\uacbd\uc758 \uc18c\uc18d\uc740?", "paragraph": "\uc774\ub97c \ubcf8 \ubbfc\uc8fc\ub2f9 \uc778\uc0ac\ub4e4\uacfc \uccad\uc640\ub300 \uacbd\ud638\uc2e4 \uc9c1\uc6d0 30\uc5ec \uba85\uc774 \ub4a4\uc5c9\ucf1c \uace0\uc131\uc744 \uc9c0\ub974\uba70 \ubab8\uc2f8\uc6c0\uc774 \ubc8c\uc5b4\uc84c\ub2e4. \uc774 \ub54c \uac15\uae30\uc815 \uc758\uc6d0\uc774 \ubab8\ubd80\ub9bc \uce58\uba70 \ubc97\uc5b4\ub098\ub294 \uacfc\uc815\uc5d0\uc11c, \uacbd\ud638\uc2e4 \uc9c1\uc6d0\uc774 \uac15\uae30\uc815 \uc758\uc6d0\uc758 \uba38\ub9ac\uc5d0 \uc5bc\uad74\uc744 \ubd80\ub52a\ud600 \ubd80\uc0c1\ub2f9\ud55c \uac83\uc73c\ub85c \uc54c\ub824\uc84c\ub2e4. \ubbfc\uc8fc\ub2f9 \uc758\uc6d0\ub4e4\uc774 \"\uc65c \uad6d\ud68c\uc758\uc6d0 \uba71\uc0b4\uc744 \uc7a1\ub290\ub0d0\"\uba70 \ud56d\uc758\ud588\uace0, \uc774 \uc9c1\uc6d0\uc5d0\uac8c \uc18c\uc18d\uc744 \ubc1d\ud790 \uac83\uc744 \uc694\uad6c\ud588\uc73c\ub098 \uadf8\ub294 \ub300\ub2f5\ud558\uc9c0 \uc54a\uc740 \ucc44 \uad6d\ud68c \ubcf8\uad00 \uc548\uc73c\ub85c \ub4e4\uc5b4\uac14\ub2e4. \uccad\uc640\ub300 \uacbd\ud638\uc2e4\uc740 \ubc15\uadfc\ud61c \ub300\ud1b5\ub839\uc758 \uad6d\ud68c \uc2dc\uc815\uc5f0\uc124 \uc9c1\ud6c4 \uad6d\ud68c \ubcf8\uad00 \uc55e\uc5d0\uc11c \uacbd\ud638 \uc694\uc6d0\uc778 \uacbd\ucc30\uad00\uc774 \uac15\uae30\uc815 \ubbfc\uc8fc\ub2f9 \uc758\uc6d0\uc73c\ub85c\ubd80\ud130 \ud3ed\ud589\uc744 \ub2f9\ud588\ub2e4\uace0 \uc8fc\uc7a5\ud588\ub2e4. \uacbd\ud638\uc2e4\uc740 \uc5b8\ub860\uc5d0 \ubc30\ud3ec\ud55c '\uac15\uae30\uc815 \uc758\uc6d0 \ud3ed\ud589\uc0ac\uac74 \ubcf4\ub3c4\ucc38\uace0\uc790\ub8cc'\uc5d0\uc11c \"22\uacbd\ucc30\uacbd\ud638\ub300 \uc6b4\uc804\ub2f4\ub2f9 \ud604\ubaa8 \uc21c\uacbd\uc774 \uac15 \uc758\uc6d0\uc73c\ub85c\ubd80\ud130 \uc548\uba74\uc744 \uac00\uaca9\ub2f9\ud574 \uc11c\uc6b8 \uc2dc\ub0b4 \ud55c \ubcd1\uc6d0\uc73c\ub85c \uc751\uae09\ud6c4\uc1a1\ub3fc \ubd09\ud569 \uce58\ub8cc\ub97c \ubc1b\uace0 \uc788\ub2e4\"\uba70 \"\uac15 \uc758\uc6d0\uc758 \ud3ed\ub825\ud589\uc0ac\uc5d0 \ub300\ud574 \ubc95\uc801 \uc870\uce58 \uc5ec\ubd80\ub97c \uac80\ud1a0\ud558\uace0 \uc788\ub2e4\"\uace0 \ubc1d\ud614\ub2e4.", "answer": "22\uacbd\ucc30\uacbd\ud638\ub300", "sentence": "\uacbd\ud638\uc2e4\uc740 \uc5b8\ub860\uc5d0 \ubc30\ud3ec\ud55c '\uac15\uae30\uc815 \uc758\uc6d0 \ud3ed\ud589\uc0ac\uac74 \ubcf4\ub3c4\ucc38\uace0\uc790\ub8cc'\uc5d0\uc11c \" 22\uacbd\ucc30\uacbd\ud638\ub300 \uc6b4\uc804\ub2f4\ub2f9 \ud604\ubaa8 \uc21c\uacbd\uc774 \uac15 \uc758\uc6d0\uc73c\ub85c\ubd80\ud130 \uc548\uba74\uc744 \uac00\uaca9\ub2f9\ud574 \uc11c\uc6b8 \uc2dc\ub0b4 \ud55c \ubcd1\uc6d0\uc73c\ub85c \uc751\uae09\ud6c4\uc1a1\ub3fc \ubd09\ud569 \uce58\ub8cc\ub97c \ubc1b\uace0 \uc788\ub2e4\"\uba70 \"\uac15 \uc758\uc6d0\uc758 \ud3ed\ub825\ud589\uc0ac\uc5d0 \ub300\ud574 \ubc95\uc801 \uc870\uce58 \uc5ec\ubd80\ub97c \uac80\ud1a0\ud558\uace0 \uc788\ub2e4\"\uace0 \ubc1d\ud614\ub2e4.", "paragraph_sentence": "\uc774\ub97c \ubcf8 \ubbfc\uc8fc\ub2f9 \uc778\uc0ac\ub4e4\uacfc \uccad\uc640\ub300 \uacbd\ud638\uc2e4 \uc9c1\uc6d0 30\uc5ec \uba85\uc774 \ub4a4\uc5c9\ucf1c \uace0\uc131\uc744 \uc9c0\ub974\uba70 \ubab8\uc2f8\uc6c0\uc774 \ubc8c\uc5b4\uc84c\ub2e4. \uc774 \ub54c \uac15\uae30\uc815 \uc758\uc6d0\uc774 \ubab8\ubd80\ub9bc \uce58\uba70 \ubc97\uc5b4\ub098\ub294 \uacfc\uc815\uc5d0\uc11c, \uacbd\ud638\uc2e4 \uc9c1\uc6d0\uc774 \uac15\uae30\uc815 \uc758\uc6d0\uc758 \uba38\ub9ac\uc5d0 \uc5bc\uad74\uc744 \ubd80\ub52a\ud600 \ubd80\uc0c1\ub2f9\ud55c \uac83\uc73c\ub85c \uc54c\ub824\uc84c\ub2e4. \ubbfc\uc8fc\ub2f9 \uc758\uc6d0\ub4e4\uc774 \"\uc65c \uad6d\ud68c\uc758\uc6d0 \uba71\uc0b4\uc744 \uc7a1\ub290\ub0d0\"\uba70 \ud56d\uc758\ud588\uace0, \uc774 \uc9c1\uc6d0\uc5d0\uac8c \uc18c\uc18d\uc744 \ubc1d\ud790 \uac83\uc744 \uc694\uad6c\ud588\uc73c\ub098 \uadf8\ub294 \ub300\ub2f5\ud558\uc9c0 \uc54a\uc740 \ucc44 \uad6d\ud68c \ubcf8\uad00 \uc548\uc73c\ub85c \ub4e4\uc5b4\uac14\ub2e4. \uccad\uc640\ub300 \uacbd\ud638\uc2e4\uc740 \ubc15\uadfc\ud61c \ub300\ud1b5\ub839\uc758 \uad6d\ud68c \uc2dc\uc815\uc5f0\uc124 \uc9c1\ud6c4 \uad6d\ud68c \ubcf8\uad00 \uc55e\uc5d0\uc11c \uacbd\ud638 \uc694\uc6d0\uc778 \uacbd\ucc30\uad00\uc774 \uac15\uae30\uc815 \ubbfc\uc8fc\ub2f9 \uc758\uc6d0\uc73c\ub85c\ubd80\ud130 \ud3ed\ud589\uc744 \ub2f9\ud588\ub2e4\uace0 \uc8fc\uc7a5\ud588\ub2e4. \uacbd\ud638\uc2e4\uc740 \uc5b8\ub860\uc5d0 \ubc30\ud3ec\ud55c '\uac15\uae30\uc815 \uc758\uc6d0 \ud3ed\ud589\uc0ac\uac74 \ubcf4\ub3c4\ucc38\uace0\uc790\ub8cc'\uc5d0\uc11c \" 22\uacbd\ucc30\uacbd\ud638\ub300 \uc6b4\uc804\ub2f4\ub2f9 \ud604\ubaa8 \uc21c\uacbd\uc774 \uac15 \uc758\uc6d0\uc73c\ub85c\ubd80\ud130 \uc548\uba74\uc744 \uac00\uaca9\ub2f9\ud574 \uc11c\uc6b8 \uc2dc\ub0b4 \ud55c \ubcd1\uc6d0\uc73c\ub85c \uc751\uae09\ud6c4\uc1a1\ub3fc \ubd09\ud569 \uce58\ub8cc\ub97c \ubc1b\uace0 \uc788\ub2e4\"\uba70 \"\uac15 \uc758\uc6d0\uc758 \ud3ed\ub825\ud589\uc0ac\uc5d0 \ub300\ud574 \ubc95\uc801 \uc870\uce58 \uc5ec\ubd80\ub97c \uac80\ud1a0\ud558\uace0 \uc788\ub2e4\"\uace0 \ubc1d\ud614\ub2e4. ", "paragraph_answer": "\uc774\ub97c \ubcf8 \ubbfc\uc8fc\ub2f9 \uc778\uc0ac\ub4e4\uacfc \uccad\uc640\ub300 \uacbd\ud638\uc2e4 \uc9c1\uc6d0 30\uc5ec \uba85\uc774 \ub4a4\uc5c9\ucf1c \uace0\uc131\uc744 \uc9c0\ub974\uba70 \ubab8\uc2f8\uc6c0\uc774 \ubc8c\uc5b4\uc84c\ub2e4. \uc774 \ub54c \uac15\uae30\uc815 \uc758\uc6d0\uc774 \ubab8\ubd80\ub9bc \uce58\uba70 \ubc97\uc5b4\ub098\ub294 \uacfc\uc815\uc5d0\uc11c, \uacbd\ud638\uc2e4 \uc9c1\uc6d0\uc774 \uac15\uae30\uc815 \uc758\uc6d0\uc758 \uba38\ub9ac\uc5d0 \uc5bc\uad74\uc744 \ubd80\ub52a\ud600 \ubd80\uc0c1\ub2f9\ud55c \uac83\uc73c\ub85c \uc54c\ub824\uc84c\ub2e4. \ubbfc\uc8fc\ub2f9 \uc758\uc6d0\ub4e4\uc774 \"\uc65c \uad6d\ud68c\uc758\uc6d0 \uba71\uc0b4\uc744 \uc7a1\ub290\ub0d0\"\uba70 \ud56d\uc758\ud588\uace0, \uc774 \uc9c1\uc6d0\uc5d0\uac8c \uc18c\uc18d\uc744 \ubc1d\ud790 \uac83\uc744 \uc694\uad6c\ud588\uc73c\ub098 \uadf8\ub294 \ub300\ub2f5\ud558\uc9c0 \uc54a\uc740 \ucc44 \uad6d\ud68c \ubcf8\uad00 \uc548\uc73c\ub85c \ub4e4\uc5b4\uac14\ub2e4. \uccad\uc640\ub300 \uacbd\ud638\uc2e4\uc740 \ubc15\uadfc\ud61c \ub300\ud1b5\ub839\uc758 \uad6d\ud68c \uc2dc\uc815\uc5f0\uc124 \uc9c1\ud6c4 \uad6d\ud68c \ubcf8\uad00 \uc55e\uc5d0\uc11c \uacbd\ud638 \uc694\uc6d0\uc778 \uacbd\ucc30\uad00\uc774 \uac15\uae30\uc815 \ubbfc\uc8fc\ub2f9 \uc758\uc6d0\uc73c\ub85c\ubd80\ud130 \ud3ed\ud589\uc744 \ub2f9\ud588\ub2e4\uace0 \uc8fc\uc7a5\ud588\ub2e4. \uacbd\ud638\uc2e4\uc740 \uc5b8\ub860\uc5d0 \ubc30\ud3ec\ud55c '\uac15\uae30\uc815 \uc758\uc6d0 \ud3ed\ud589\uc0ac\uac74 \ubcf4\ub3c4\ucc38\uace0\uc790\ub8cc'\uc5d0\uc11c \" 22\uacbd\ucc30\uacbd\ud638\ub300 \uc6b4\uc804\ub2f4\ub2f9 \ud604\ubaa8 \uc21c\uacbd\uc774 \uac15 \uc758\uc6d0\uc73c\ub85c\ubd80\ud130 \uc548\uba74\uc744 \uac00\uaca9\ub2f9\ud574 \uc11c\uc6b8 \uc2dc\ub0b4 \ud55c \ubcd1\uc6d0\uc73c\ub85c \uc751\uae09\ud6c4\uc1a1\ub3fc \ubd09\ud569 \uce58\ub8cc\ub97c \ubc1b\uace0 \uc788\ub2e4\"\uba70 \"\uac15 \uc758\uc6d0\uc758 \ud3ed\ub825\ud589\uc0ac\uc5d0 \ub300\ud574 \ubc95\uc801 \uc870\uce58 \uc5ec\ubd80\ub97c \uac80\ud1a0\ud558\uace0 \uc788\ub2e4\"\uace0 \ubc1d\ud614\ub2e4.", "sentence_answer": "\uacbd\ud638\uc2e4\uc740 \uc5b8\ub860\uc5d0 \ubc30\ud3ec\ud55c '\uac15\uae30\uc815 \uc758\uc6d0 \ud3ed\ud589\uc0ac\uac74 \ubcf4\ub3c4\ucc38\uace0\uc790\ub8cc'\uc5d0\uc11c \" 22\uacbd\ucc30\uacbd\ud638\ub300 \uc6b4\uc804\ub2f4\ub2f9 \ud604\ubaa8 \uc21c\uacbd\uc774 \uac15 \uc758\uc6d0\uc73c\ub85c\ubd80\ud130 \uc548\uba74\uc744 \uac00\uaca9\ub2f9\ud574 \uc11c\uc6b8 \uc2dc\ub0b4 \ud55c \ubcd1\uc6d0\uc73c\ub85c \uc751\uae09\ud6c4\uc1a1\ub3fc \ubd09\ud569 \uce58\ub8cc\ub97c \ubc1b\uace0 \uc788\ub2e4\"\uba70 \"\uac15 \uc758\uc6d0\uc758 \ud3ed\ub825\ud589\uc0ac\uc5d0 \ub300\ud574 \ubc95\uc801 \uc870\uce58 \uc5ec\ubd80\ub97c \uac80\ud1a0\ud558\uace0 \uc788\ub2e4\"\uace0 \ubc1d\ud614\ub2e4.", "paragraph_id": "5831863-0-2", "paragraph_question": "question: \uba71\uc0b4\uc744 \uc7a1\ub2e4 \ubab8\ubd80\ub9bc\uc73c\ub85c \uc778\ud574 \uba38\ub9ac\uc5d0 \ubd80\ub52a\ud600 \uc5bc\uad74\uc5d0 \ubd80\uc0c1\uc744 \uc785\uc740 \uc9c1\uc6d0\uc758 \uc18c\uc18d\uc744 \ubc1d\ud788\uc9c0 \uc54a\uc558\ub2e4\uac00 \uc548\uba74 \uac00\uaca9\uc744 \ub2f9\ud588\ub2e4\uba70 \uac15\uae30\uc815 \uc758\uc6d0 \ud3ed\ud589\uc0ac\uac74 \ubcf4\ub3c4\ucc38\uace0\uc790\ub8cc\uc5d0\uc11c \ud53c\ud574\uc790\ub85c \uac70\ub860\ud55c \uc21c\uacbd\uc758 \uc18c\uc18d\uc740?, context: \uc774\ub97c \ubcf8 \ubbfc\uc8fc\ub2f9 \uc778\uc0ac\ub4e4\uacfc \uccad\uc640\ub300 \uacbd\ud638\uc2e4 \uc9c1\uc6d0 30\uc5ec \uba85\uc774 \ub4a4\uc5c9\ucf1c \uace0\uc131\uc744 \uc9c0\ub974\uba70 \ubab8\uc2f8\uc6c0\uc774 \ubc8c\uc5b4\uc84c\ub2e4. \uc774 \ub54c \uac15\uae30\uc815 \uc758\uc6d0\uc774 \ubab8\ubd80\ub9bc \uce58\uba70 \ubc97\uc5b4\ub098\ub294 \uacfc\uc815\uc5d0\uc11c, \uacbd\ud638\uc2e4 \uc9c1\uc6d0\uc774 \uac15\uae30\uc815 \uc758\uc6d0\uc758 \uba38\ub9ac\uc5d0 \uc5bc\uad74\uc744 \ubd80\ub52a\ud600 \ubd80\uc0c1\ub2f9\ud55c \uac83\uc73c\ub85c \uc54c\ub824\uc84c\ub2e4. \ubbfc\uc8fc\ub2f9 \uc758\uc6d0\ub4e4\uc774 \"\uc65c \uad6d\ud68c\uc758\uc6d0 \uba71\uc0b4\uc744 \uc7a1\ub290\ub0d0\"\uba70 \ud56d\uc758\ud588\uace0, \uc774 \uc9c1\uc6d0\uc5d0\uac8c \uc18c\uc18d\uc744 \ubc1d\ud790 \uac83\uc744 \uc694\uad6c\ud588\uc73c\ub098 \uadf8\ub294 \ub300\ub2f5\ud558\uc9c0 \uc54a\uc740 \ucc44 \uad6d\ud68c \ubcf8\uad00 \uc548\uc73c\ub85c \ub4e4\uc5b4\uac14\ub2e4. \uccad\uc640\ub300 \uacbd\ud638\uc2e4\uc740 \ubc15\uadfc\ud61c \ub300\ud1b5\ub839\uc758 \uad6d\ud68c \uc2dc\uc815\uc5f0\uc124 \uc9c1\ud6c4 \uad6d\ud68c \ubcf8\uad00 \uc55e\uc5d0\uc11c \uacbd\ud638 \uc694\uc6d0\uc778 \uacbd\ucc30\uad00\uc774 \uac15\uae30\uc815 \ubbfc\uc8fc\ub2f9 \uc758\uc6d0\uc73c\ub85c\ubd80\ud130 \ud3ed\ud589\uc744 \ub2f9\ud588\ub2e4\uace0 \uc8fc\uc7a5\ud588\ub2e4. \uacbd\ud638\uc2e4\uc740 \uc5b8\ub860\uc5d0 \ubc30\ud3ec\ud55c '\uac15\uae30\uc815 \uc758\uc6d0 \ud3ed\ud589\uc0ac\uac74 \ubcf4\ub3c4\ucc38\uace0\uc790\ub8cc'\uc5d0\uc11c \"22\uacbd\ucc30\uacbd\ud638\ub300 \uc6b4\uc804\ub2f4\ub2f9 \ud604\ubaa8 \uc21c\uacbd\uc774 \uac15 \uc758\uc6d0\uc73c\ub85c\ubd80\ud130 \uc548\uba74\uc744 \uac00\uaca9\ub2f9\ud574 \uc11c\uc6b8 \uc2dc\ub0b4 \ud55c \ubcd1\uc6d0\uc73c\ub85c \uc751\uae09\ud6c4\uc1a1\ub3fc \ubd09\ud569 \uce58\ub8cc\ub97c \ubc1b\uace0 \uc788\ub2e4\"\uba70 \"\uac15 \uc758\uc6d0\uc758 \ud3ed\ub825\ud589\uc0ac\uc5d0 \ub300\ud574 \ubc95\uc801 \uc870\uce58 \uc5ec\ubd80\ub97c \uac80\ud1a0\ud558\uace0 \uc788\ub2e4\"\uace0 \ubc1d\ud614\ub2e4."} {"question": "\ud5c8\ub9ac\ucf00\uc778 \ub9b0\ub2e4\uc758 \ucd5c\uc800 \uc911\uc2ec\uae30\uc555\uc744 \uc5bc\ub9c8\ub85c \uae30\ub85d\ub418\uc5c8\ub098?", "paragraph": "\uc5f4\ub300\uc800\uae30\uc555\uc774 \ub418\uba74\uc11c \uc774 \uc800\uae30\uc555\uc740 \ubd81\uc11c\ucabd\uc73c\ub85c 9.7~19 km/h (6~12 mph)\uc758 \uc18d\ub825\uc73c\ub85c \uc804\uc9c4\ud574\ub098\uac14\ub2e4. \uc774 \ub54c \ubc14\ud558\uce7c\ub9ac\ud3ec\ub974\ub2c8\uc544\uc218\ub974 \uc8fc \ub0a8\ub2e8\uc5d0 \uc704\uce58\ud55c \uc800\uae30\uc555\ub300\uc758 \uc601\ud5a5\uc744 \ubc1b\uc544 \ubd81\uc11c\uc9c4\ud588\ub2e4. \uc774 \ub0a0 \ubd80\ud130 \ube60\ub974\uac8c \uac15\ud574\uc838 9\uc6d4 10\uc77c\uc5d0 \uc5f4\ub300 \ud3ed\ud48d\uc73c\ub85c \ubc1c\ub2ec\ud558\uc600\uace0 \uc5f4\ub300\ud3ed\ud48d\uc774 \ub418\uba74\uc11c \ubbf8\uad6d \uad6d\ub9bd \ud5c8\ub9ac\ucf00\uc778 \uc13c\ud130(NHC)\uac00 '\ub9b0\ub2e4(Linda)'\ub85c \uba85\uba85\ud588\ub2e4. \ub04a\uc784\uc5c6\uc774 \uac15\ub825\ud574\uc838 9\uc6d4 11\uc77c\ubd80\ud130 \ud5c8\ub9ac\ucf00\uc778\uc758 \ub208\uc774 \ubd88\uaddc\uce59\ud558\uac8c \ub098\ud0c0\ub0ac\ub2e4\uac00 \uc18c\uba78\ub418\ub294 \ud604\uc0c1\uc744 \uacaa\uc5c8\uace0 \uace7 NHC\uac00 \ud5c8\ub9ac\ucf00\uc778\uc758 \uc9c0\uc704\ub85c \uc2b9\uaca9\uc2dc\ucf30\ub2e4. \ud5c8\ub9ac\ucf00\uc778 \ub9b0\ub2e4\ub294 \uae09\uaca9\ud558\uac8c \uac15\ub825\ud574\uc838 24\uc2dc\uac04\ub9cc\uc5d0 81 hPa\uac00 \ub5a8\uc5b4\uc84c\ub2e4. \uc774 \uc218\uce58\ub294 1\uc2dc\uac04\ub9c8\ub2e4 \ud3c9\uade0\uc801\uc73c\ub85c 3.38 hPa\uac00 \ub0ae\uc544\uc9c4 \uac83\uacfc \uac19\ub2e4. 9\uc6d4 12\uc77c\uae4c\uc9c0 \uc774\ub7f0 \ud604\uc0c1\uc774 \ub098\ud0c0\ub098\ub2e4\uac00 \uc0ac\ud53c\uc5b4-\uc2ec\uc2a8 \ud5c8\ub9ac\ucf00\uc778 \ub4f1\uae09 \uc81c5\ub4f1\uae09\uc5d0 \ub4e4\uc5b4\uc130\uace0 \uc774\uc5b4\uc11c \uc18c\ucf54\ub85c \uc12c \ub0a8\ub3d9\ucabd \uc57d 235 km (145 mi) \ubd80\uadfc \ud574\uc0c1\uc5d0\uc11c \ud48d\uc18d 295 km/h (185 mph)\uc5d0 \ub3c4\ub2ec\ud558\uac8c \ub418\uc5b4 \ucd5c\uc131\uae30\ub97c \ub9de\uc774\ud558\uc600\ub2e4. \ub9b0\ub2e4\uc758 \ucd5c\ub300\ud48d\uc18d\uc740 \ub4dc\ubcf4\ub77d \ubd84\ub958\ubc95\uc5d0 \uc758\ud574 290~314 km/h (180~195 mph)\uc73c\ub85c \ucd94\uc815\ub418\uace0 \uc788\uace0 \ucd5c\ub300\uc21c\uac04\ud48d\uc18d\uc740 351 km/h (218 mph)\uae4c\uc9c0 \uce58\uc19f\uc558\uc744 \uac83\uc774\ub77c\uace0 \uc608\uc0c1\ud558\uace0 \uc788\ub2e4. \ub9b0\ub2e4\uc758 \ucd5c\uc800\uc911\uc2ec\uae30\uc555\uc740 902 hPa\ub85c \uae30\ub85d\ub418\uc5b4 \uc788\uc73c\ub098, \ud5c8\ub9ac\ucf00\uc778\uc774 \uc2e4\uc874\ud574 \uc788\uc5c8\uc744 \ub54c\uc758 \uc911\uc2ec\uae30\uc555\uc740 902 hPa\ubcf4\ub2e4 \uc57d\uac04 \ub0ae\uc740 900 hPa \uc815\ub3c4\ub97c \uae30\ub85d\ud588\uc744 \uac83\uc774\ub77c\uace0 \uc608\uc0c1\ud558\uace0 \uc788\ub2e4. \ud55c\ud3b8 \uc911\uc2ec\uae30\uc555 902 hPa\ub294 \ub3d9\ud0dc\ud3c9\uc591\uc5d0\uc11c \uad00\uce21\ub41c \ubaa8\ub4e0 \ud5c8\ub9ac\ucf00\uc778 \uc911\uc5d0\uc11c \uac00\uc7a5 \uac15\ud55c \uc218\uce58\uc774\ub2e4.", "answer": "902 hPa", "sentence": "\ub9b0\ub2e4\uc758 \ucd5c\uc800\uc911\uc2ec\uae30\uc555\uc740 902 hPa \ub85c \uae30\ub85d\ub418\uc5b4 \uc788\uc73c\ub098, \ud5c8\ub9ac\ucf00\uc778\uc774 \uc2e4\uc874\ud574 \uc788\uc5c8\uc744 \ub54c\uc758 \uc911\uc2ec\uae30\uc555\uc740 902 hPa\ubcf4\ub2e4 \uc57d\uac04 \ub0ae\uc740 900 hPa \uc815\ub3c4\ub97c \uae30\ub85d\ud588\uc744 \uac83\uc774\ub77c\uace0 \uc608\uc0c1\ud558\uace0 \uc788\ub2e4.", "paragraph_sentence": "\uc5f4\ub300\uc800\uae30\uc555\uc774 \ub418\uba74\uc11c \uc774 \uc800\uae30\uc555\uc740 \ubd81\uc11c\ucabd\uc73c\ub85c 9.7~19 km/h (6~12 mph)\uc758 \uc18d\ub825\uc73c\ub85c \uc804\uc9c4\ud574\ub098\uac14\ub2e4. \uc774 \ub54c \ubc14\ud558\uce7c\ub9ac\ud3ec\ub974\ub2c8\uc544\uc218\ub974 \uc8fc \ub0a8\ub2e8\uc5d0 \uc704\uce58\ud55c \uc800\uae30\uc555\ub300\uc758 \uc601\ud5a5\uc744 \ubc1b\uc544 \ubd81\uc11c\uc9c4\ud588\ub2e4. \uc774 \ub0a0 \ubd80\ud130 \ube60\ub974\uac8c \uac15\ud574\uc838 9\uc6d4 10\uc77c\uc5d0 \uc5f4\ub300 \ud3ed\ud48d\uc73c\ub85c \ubc1c\ub2ec\ud558\uc600\uace0 \uc5f4\ub300\ud3ed\ud48d\uc774 \ub418\uba74\uc11c \ubbf8\uad6d \uad6d\ub9bd \ud5c8\ub9ac\ucf00\uc778 \uc13c\ud130(NHC)\uac00 '\ub9b0\ub2e4(Linda)'\ub85c \uba85\uba85\ud588\ub2e4. \ub04a\uc784\uc5c6\uc774 \uac15\ub825\ud574\uc838 9\uc6d4 11\uc77c\ubd80\ud130 \ud5c8\ub9ac\ucf00\uc778\uc758 \ub208\uc774 \ubd88\uaddc\uce59\ud558\uac8c \ub098\ud0c0\ub0ac\ub2e4\uac00 \uc18c\uba78\ub418\ub294 \ud604\uc0c1\uc744 \uacaa\uc5c8\uace0 \uace7 NHC\uac00 \ud5c8\ub9ac\ucf00\uc778\uc758 \uc9c0\uc704\ub85c \uc2b9\uaca9\uc2dc\ucf30\ub2e4. \ud5c8\ub9ac\ucf00\uc778 \ub9b0\ub2e4\ub294 \uae09\uaca9\ud558\uac8c \uac15\ub825\ud574\uc838 24\uc2dc\uac04\ub9cc\uc5d0 81 hPa\uac00 \ub5a8\uc5b4\uc84c\ub2e4. \uc774 \uc218\uce58\ub294 1\uc2dc\uac04\ub9c8\ub2e4 \ud3c9\uade0\uc801\uc73c\ub85c 3.38 hPa\uac00 \ub0ae\uc544\uc9c4 \uac83\uacfc \uac19\ub2e4. 9\uc6d4 12\uc77c\uae4c\uc9c0 \uc774\ub7f0 \ud604\uc0c1\uc774 \ub098\ud0c0\ub098\ub2e4\uac00 \uc0ac\ud53c\uc5b4-\uc2ec\uc2a8 \ud5c8\ub9ac\ucf00\uc778 \ub4f1\uae09 \uc81c5\ub4f1\uae09\uc5d0 \ub4e4\uc5b4\uc130\uace0 \uc774\uc5b4\uc11c \uc18c\ucf54\ub85c \uc12c \ub0a8\ub3d9\ucabd \uc57d 235 km (145 mi) \ubd80\uadfc \ud574\uc0c1\uc5d0\uc11c \ud48d\uc18d 295 km/h (185 mph)\uc5d0 \ub3c4\ub2ec\ud558\uac8c \ub418\uc5b4 \ucd5c\uc131\uae30\ub97c \ub9de\uc774\ud558\uc600\ub2e4. \ub9b0\ub2e4\uc758 \ucd5c\ub300\ud48d\uc18d\uc740 \ub4dc\ubcf4\ub77d \ubd84\ub958\ubc95\uc5d0 \uc758\ud574 290~314 km/h (180~195 mph)\uc73c\ub85c \ucd94\uc815\ub418\uace0 \uc788\uace0 \ucd5c\ub300\uc21c\uac04\ud48d\uc18d\uc740 351 km/h (218 mph)\uae4c\uc9c0 \uce58\uc19f\uc558\uc744 \uac83\uc774\ub77c\uace0 \uc608\uc0c1\ud558\uace0 \uc788\ub2e4. \ub9b0\ub2e4\uc758 \ucd5c\uc800\uc911\uc2ec\uae30\uc555\uc740 902 hPa \ub85c \uae30\ub85d\ub418\uc5b4 \uc788\uc73c\ub098, \ud5c8\ub9ac\ucf00\uc778\uc774 \uc2e4\uc874\ud574 \uc788\uc5c8\uc744 \ub54c\uc758 \uc911\uc2ec\uae30\uc555\uc740 902 hPa\ubcf4\ub2e4 \uc57d\uac04 \ub0ae\uc740 900 hPa \uc815\ub3c4\ub97c \uae30\ub85d\ud588\uc744 \uac83\uc774\ub77c\uace0 \uc608\uc0c1\ud558\uace0 \uc788\ub2e4. \ud55c\ud3b8 \uc911\uc2ec\uae30\uc555 902 hPa\ub294 \ub3d9\ud0dc\ud3c9\uc591\uc5d0\uc11c \uad00\uce21\ub41c \ubaa8\ub4e0 \ud5c8\ub9ac\ucf00\uc778 \uc911\uc5d0\uc11c \uac00\uc7a5 \uac15\ud55c \uc218\uce58\uc774\ub2e4.", "paragraph_answer": "\uc5f4\ub300\uc800\uae30\uc555\uc774 \ub418\uba74\uc11c \uc774 \uc800\uae30\uc555\uc740 \ubd81\uc11c\ucabd\uc73c\ub85c 9.7~19 km/h (6~12 mph)\uc758 \uc18d\ub825\uc73c\ub85c \uc804\uc9c4\ud574\ub098\uac14\ub2e4. \uc774 \ub54c \ubc14\ud558\uce7c\ub9ac\ud3ec\ub974\ub2c8\uc544\uc218\ub974 \uc8fc \ub0a8\ub2e8\uc5d0 \uc704\uce58\ud55c \uc800\uae30\uc555\ub300\uc758 \uc601\ud5a5\uc744 \ubc1b\uc544 \ubd81\uc11c\uc9c4\ud588\ub2e4. \uc774 \ub0a0 \ubd80\ud130 \ube60\ub974\uac8c \uac15\ud574\uc838 9\uc6d4 10\uc77c\uc5d0 \uc5f4\ub300 \ud3ed\ud48d\uc73c\ub85c \ubc1c\ub2ec\ud558\uc600\uace0 \uc5f4\ub300\ud3ed\ud48d\uc774 \ub418\uba74\uc11c \ubbf8\uad6d \uad6d\ub9bd \ud5c8\ub9ac\ucf00\uc778 \uc13c\ud130(NHC)\uac00 '\ub9b0\ub2e4(Linda)'\ub85c \uba85\uba85\ud588\ub2e4. \ub04a\uc784\uc5c6\uc774 \uac15\ub825\ud574\uc838 9\uc6d4 11\uc77c\ubd80\ud130 \ud5c8\ub9ac\ucf00\uc778\uc758 \ub208\uc774 \ubd88\uaddc\uce59\ud558\uac8c \ub098\ud0c0\ub0ac\ub2e4\uac00 \uc18c\uba78\ub418\ub294 \ud604\uc0c1\uc744 \uacaa\uc5c8\uace0 \uace7 NHC\uac00 \ud5c8\ub9ac\ucf00\uc778\uc758 \uc9c0\uc704\ub85c \uc2b9\uaca9\uc2dc\ucf30\ub2e4. \ud5c8\ub9ac\ucf00\uc778 \ub9b0\ub2e4\ub294 \uae09\uaca9\ud558\uac8c \uac15\ub825\ud574\uc838 24\uc2dc\uac04\ub9cc\uc5d0 81 hPa\uac00 \ub5a8\uc5b4\uc84c\ub2e4. \uc774 \uc218\uce58\ub294 1\uc2dc\uac04\ub9c8\ub2e4 \ud3c9\uade0\uc801\uc73c\ub85c 3.38 hPa\uac00 \ub0ae\uc544\uc9c4 \uac83\uacfc \uac19\ub2e4. 9\uc6d4 12\uc77c\uae4c\uc9c0 \uc774\ub7f0 \ud604\uc0c1\uc774 \ub098\ud0c0\ub098\ub2e4\uac00 \uc0ac\ud53c\uc5b4-\uc2ec\uc2a8 \ud5c8\ub9ac\ucf00\uc778 \ub4f1\uae09 \uc81c5\ub4f1\uae09\uc5d0 \ub4e4\uc5b4\uc130\uace0 \uc774\uc5b4\uc11c \uc18c\ucf54\ub85c \uc12c \ub0a8\ub3d9\ucabd \uc57d 235 km (145 mi) \ubd80\uadfc \ud574\uc0c1\uc5d0\uc11c \ud48d\uc18d 295 km/h (185 mph)\uc5d0 \ub3c4\ub2ec\ud558\uac8c \ub418\uc5b4 \ucd5c\uc131\uae30\ub97c \ub9de\uc774\ud558\uc600\ub2e4. \ub9b0\ub2e4\uc758 \ucd5c\ub300\ud48d\uc18d\uc740 \ub4dc\ubcf4\ub77d \ubd84\ub958\ubc95\uc5d0 \uc758\ud574 290~314 km/h (180~195 mph)\uc73c\ub85c \ucd94\uc815\ub418\uace0 \uc788\uace0 \ucd5c\ub300\uc21c\uac04\ud48d\uc18d\uc740 351 km/h (218 mph)\uae4c\uc9c0 \uce58\uc19f\uc558\uc744 \uac83\uc774\ub77c\uace0 \uc608\uc0c1\ud558\uace0 \uc788\ub2e4. \ub9b0\ub2e4\uc758 \ucd5c\uc800\uc911\uc2ec\uae30\uc555\uc740 902 hPa \ub85c \uae30\ub85d\ub418\uc5b4 \uc788\uc73c\ub098, \ud5c8\ub9ac\ucf00\uc778\uc774 \uc2e4\uc874\ud574 \uc788\uc5c8\uc744 \ub54c\uc758 \uc911\uc2ec\uae30\uc555\uc740 902 hPa\ubcf4\ub2e4 \uc57d\uac04 \ub0ae\uc740 900 hPa \uc815\ub3c4\ub97c \uae30\ub85d\ud588\uc744 \uac83\uc774\ub77c\uace0 \uc608\uc0c1\ud558\uace0 \uc788\ub2e4. \ud55c\ud3b8 \uc911\uc2ec\uae30\uc555 902 hPa\ub294 \ub3d9\ud0dc\ud3c9\uc591\uc5d0\uc11c \uad00\uce21\ub41c \ubaa8\ub4e0 \ud5c8\ub9ac\ucf00\uc778 \uc911\uc5d0\uc11c \uac00\uc7a5 \uac15\ud55c \uc218\uce58\uc774\ub2e4.", "sentence_answer": "\ub9b0\ub2e4\uc758 \ucd5c\uc800\uc911\uc2ec\uae30\uc555\uc740 902 hPa \ub85c \uae30\ub85d\ub418\uc5b4 \uc788\uc73c\ub098, \ud5c8\ub9ac\ucf00\uc778\uc774 \uc2e4\uc874\ud574 \uc788\uc5c8\uc744 \ub54c\uc758 \uc911\uc2ec\uae30\uc555\uc740 902 hPa\ubcf4\ub2e4 \uc57d\uac04 \ub0ae\uc740 900 hPa \uc815\ub3c4\ub97c \uae30\ub85d\ud588\uc744 \uac83\uc774\ub77c\uace0 \uc608\uc0c1\ud558\uace0 \uc788\ub2e4.", "paragraph_id": "6571170-0-1", "paragraph_question": "question: \ud5c8\ub9ac\ucf00\uc778 \ub9b0\ub2e4\uc758 \ucd5c\uc800 \uc911\uc2ec\uae30\uc555\uc744 \uc5bc\ub9c8\ub85c \uae30\ub85d\ub418\uc5c8\ub098?, context: \uc5f4\ub300\uc800\uae30\uc555\uc774 \ub418\uba74\uc11c \uc774 \uc800\uae30\uc555\uc740 \ubd81\uc11c\ucabd\uc73c\ub85c 9.7~19 km/h (6~12 mph)\uc758 \uc18d\ub825\uc73c\ub85c \uc804\uc9c4\ud574\ub098\uac14\ub2e4. \uc774 \ub54c \ubc14\ud558\uce7c\ub9ac\ud3ec\ub974\ub2c8\uc544\uc218\ub974 \uc8fc \ub0a8\ub2e8\uc5d0 \uc704\uce58\ud55c \uc800\uae30\uc555\ub300\uc758 \uc601\ud5a5\uc744 \ubc1b\uc544 \ubd81\uc11c\uc9c4\ud588\ub2e4. \uc774 \ub0a0 \ubd80\ud130 \ube60\ub974\uac8c \uac15\ud574\uc838 9\uc6d4 10\uc77c\uc5d0 \uc5f4\ub300 \ud3ed\ud48d\uc73c\ub85c \ubc1c\ub2ec\ud558\uc600\uace0 \uc5f4\ub300\ud3ed\ud48d\uc774 \ub418\uba74\uc11c \ubbf8\uad6d \uad6d\ub9bd \ud5c8\ub9ac\ucf00\uc778 \uc13c\ud130(NHC)\uac00 '\ub9b0\ub2e4(Linda)'\ub85c \uba85\uba85\ud588\ub2e4. \ub04a\uc784\uc5c6\uc774 \uac15\ub825\ud574\uc838 9\uc6d4 11\uc77c\ubd80\ud130 \ud5c8\ub9ac\ucf00\uc778\uc758 \ub208\uc774 \ubd88\uaddc\uce59\ud558\uac8c \ub098\ud0c0\ub0ac\ub2e4\uac00 \uc18c\uba78\ub418\ub294 \ud604\uc0c1\uc744 \uacaa\uc5c8\uace0 \uace7 NHC\uac00 \ud5c8\ub9ac\ucf00\uc778\uc758 \uc9c0\uc704\ub85c \uc2b9\uaca9\uc2dc\ucf30\ub2e4. \ud5c8\ub9ac\ucf00\uc778 \ub9b0\ub2e4\ub294 \uae09\uaca9\ud558\uac8c \uac15\ub825\ud574\uc838 24\uc2dc\uac04\ub9cc\uc5d0 81 hPa\uac00 \ub5a8\uc5b4\uc84c\ub2e4. \uc774 \uc218\uce58\ub294 1\uc2dc\uac04\ub9c8\ub2e4 \ud3c9\uade0\uc801\uc73c\ub85c 3.38 hPa\uac00 \ub0ae\uc544\uc9c4 \uac83\uacfc \uac19\ub2e4. 9\uc6d4 12\uc77c\uae4c\uc9c0 \uc774\ub7f0 \ud604\uc0c1\uc774 \ub098\ud0c0\ub098\ub2e4\uac00 \uc0ac\ud53c\uc5b4-\uc2ec\uc2a8 \ud5c8\ub9ac\ucf00\uc778 \ub4f1\uae09 \uc81c5\ub4f1\uae09\uc5d0 \ub4e4\uc5b4\uc130\uace0 \uc774\uc5b4\uc11c \uc18c\ucf54\ub85c \uc12c \ub0a8\ub3d9\ucabd \uc57d 235 km (145 mi) \ubd80\uadfc \ud574\uc0c1\uc5d0\uc11c \ud48d\uc18d 295 km/h (185 mph)\uc5d0 \ub3c4\ub2ec\ud558\uac8c \ub418\uc5b4 \ucd5c\uc131\uae30\ub97c \ub9de\uc774\ud558\uc600\ub2e4. \ub9b0\ub2e4\uc758 \ucd5c\ub300\ud48d\uc18d\uc740 \ub4dc\ubcf4\ub77d \ubd84\ub958\ubc95\uc5d0 \uc758\ud574 290~314 km/h (180~195 mph)\uc73c\ub85c \ucd94\uc815\ub418\uace0 \uc788\uace0 \ucd5c\ub300\uc21c\uac04\ud48d\uc18d\uc740 351 km/h (218 mph)\uae4c\uc9c0 \uce58\uc19f\uc558\uc744 \uac83\uc774\ub77c\uace0 \uc608\uc0c1\ud558\uace0 \uc788\ub2e4. \ub9b0\ub2e4\uc758 \ucd5c\uc800\uc911\uc2ec\uae30\uc555\uc740 902 hPa\ub85c \uae30\ub85d\ub418\uc5b4 \uc788\uc73c\ub098, \ud5c8\ub9ac\ucf00\uc778\uc774 \uc2e4\uc874\ud574 \uc788\uc5c8\uc744 \ub54c\uc758 \uc911\uc2ec\uae30\uc555\uc740 902 hPa\ubcf4\ub2e4 \uc57d\uac04 \ub0ae\uc740 900 hPa \uc815\ub3c4\ub97c \uae30\ub85d\ud588\uc744 \uac83\uc774\ub77c\uace0 \uc608\uc0c1\ud558\uace0 \uc788\ub2e4. \ud55c\ud3b8 \uc911\uc2ec\uae30\uc555 902 hPa\ub294 \ub3d9\ud0dc\ud3c9\uc591\uc5d0\uc11c \uad00\uce21\ub41c \ubaa8\ub4e0 \ud5c8\ub9ac\ucf00\uc778 \uc911\uc5d0\uc11c \uac00\uc7a5 \uac15\ud55c \uc218\uce58\uc774\ub2e4."} {"question": "\ubbf8\uad6d\uc774 \uc774\ub780\uc5d0\uac8c \ubcf4\ubcf5\uc801 \uc870\ucde8\ub85c \ucde8\ud55c \uc678\uad50\uc801 \ud589\ub3d9\uc740 \ubb34\uc5c7\uc778\uac00?", "paragraph": "\uc774 \uc0ac\uac74\uc740 \ubbf8\u00b7\uc774\ub780 \uc778\uc9c8 \uc0ac\ud0dc\uc5d0 \uc545\uc7ac\ub85c \uc791\uc6a9\ud558\uc600\uc744 \ubfd0\ub9cc \uc544\ub2c8\ub77c \uac19\uc740 \uc774\uc2ac\ub78c\uad8c\uc758 \uc0ac\uc6b0\ub514\uc544\ub77c\ube44\uc544 \ub4f1 \uc628\uac74\u00b7\uce5c\ubbf8\u00b7\uad70\uc8fc\uad6d\ub4e4\uacfc \uc774\ub780\uc758 \uc815\uce58\uc801 \ub300\ub9bd\uc744 \uc57c\uae30\uc2dc\ucf30\ub2e4. \uc774\ub780\uc758 \uc694\uad6c\ub294 \ud314\ub808\ube44\uc758 \uc2e0\ubcd1 \uc778\ub3c4\uc5d0 \uadf8\uce58\uc9c0 \uc54a\uace0 \ubbf8\uad6d\uc758 \uc774\ub780\uc5d0 \ub300\ud55c \uc8c4\uc0c1(\u7f6a\u72c0)\uc870\uc0ac\uc640 \uadf8\uc5d0 \ub300\ud55c \ubbf8\uad6d\uc758 \uc0ac\uacfc\uae4c\uc9c0\ub85c \ud655\ub300\ub418\uc5c8\uace0, \ubbf8\uad6d\uc73c\ub85c\uc11c\ub294 \uad6d\uc81c\ubc95 \uc9c8\uc11c\uc0c1 \ub3c4\uc800\ud788 \uc788\uc744 \uc218 \uc5c6\ub294 \uacf5\uad00(\u516c\u9928)\uacfc \uc678\uad50\uad00\uc5d0 \ub300\ud55c \ud14c\ub7ec\uc640 \uc774\ub780\uc758 \uc694\uad6c\ub294 \ubbf8\uad6d\uc758 \uba85\uc608\uc5d0 \uc9c1\uacb0\ub41c \ubc1b\uc544\ub4e4\uc77c \uc218 \uc5c6\ub294 \uac83\uc774\uc5c8\ub2e4. \uc774\ub780\uce21\uc5d0\uc11c\ub3c4 \uc77c\uc2dc \uc628\uac74\ud30c\uac00 \uc9d1\uad8c, \ud611\uc0c1\uc758 \uae30\ubbf8\uac00 \uc5ff\ubcf4\uc600\uc73c\ub098 \ud638\uba54\uc774\ub2c8\uc758 \uac15\uacbd \uc120\uc5b8\uc73c\ub85c \ub2e4\uc2dc \uacbd\ud654\ub418\uc5c8\uace0, \uc774\uc5d0 \ubbf8\uad6d\ub3c4 \ubcf4\ubcf5\uc744 \uc120\uc5b8, \ubbf8\uad6d\uacfc \uc11c\ubc29\uce21\uc758 \ub300\uc774\ub780 \uad6d\uad50 \ub2e8\uc808\uacfc \uacbd\uc81c \uc81c\uc7ac \uc870\uce58\ub85c \uc774\uc5b4\uc84c\ub2e4. \uc720\uc5d4\uc744 \uc911\uc2ec\uc73c\ub85c \ud55c \uc911\uc7ac \ub178\ub825\ub3c4 \ubb34\uc0b0\ub418\ub294 \uac00\uc6b4\ub370 1980\ub144 4\uc6d4 \ubbf8\uad6d\uc740 \ud2b9\uacf5\ub300\ub97c \ud22c\uc785, \ubb34\ub825\uc5d0 \uc758\ud55c \uc778\uc9c8\uad6c\ucd9c \uc791\uc804\uc744 \uc2dc\ub3c4\ud588\uc73c\ub098 \uacc4\ud68d \ucc28\uc9c8\uacfc \ud56d\uacf5\uae30 \ucda9\ub3cc\uc0ac\uace0\ub85c 8\uba85\uc758 \ud2b9\uacf5\ub300\uc6d0\uc774 \uc0ac\ub9dd, \uc2e4\ud328\ub85c \ub05d\ub0ac\ub2e4.", "answer": "\ub300\uc774\ub780 \uad6d\uad50 \ub2e8\uc808", "sentence": "\uc774\uc5d0 \ubbf8\uad6d\ub3c4 \ubcf4\ubcf5\uc744 \uc120\uc5b8, \ubbf8\uad6d\uacfc \uc11c\ubc29\uce21\uc758 \ub300\uc774\ub780 \uad6d\uad50 \ub2e8\uc808 \uacfc \uacbd\uc81c \uc81c\uc7ac \uc870\uce58\ub85c \uc774\uc5b4\uc84c\ub2e4.", "paragraph_sentence": "\uc774 \uc0ac\uac74\uc740 \ubbf8\u00b7\uc774\ub780 \uc778\uc9c8 \uc0ac\ud0dc\uc5d0 \uc545\uc7ac\ub85c \uc791\uc6a9\ud558\uc600\uc744 \ubfd0\ub9cc \uc544\ub2c8\ub77c \uac19\uc740 \uc774\uc2ac\ub78c\uad8c\uc758 \uc0ac\uc6b0\ub514\uc544\ub77c\ube44\uc544 \ub4f1 \uc628\uac74\u00b7\uce5c\ubbf8\u00b7\uad70\uc8fc\uad6d\ub4e4\uacfc \uc774\ub780\uc758 \uc815\uce58\uc801 \ub300\ub9bd\uc744 \uc57c\uae30\uc2dc\ucf30\ub2e4. \uc774\ub780\uc758 \uc694\uad6c\ub294 \ud314\ub808\ube44\uc758 \uc2e0\ubcd1 \uc778\ub3c4\uc5d0 \uadf8\uce58\uc9c0 \uc54a\uace0 \ubbf8\uad6d\uc758 \uc774\ub780\uc5d0 \ub300\ud55c \uc8c4\uc0c1(\u7f6a\u72c0)\uc870\uc0ac\uc640 \uadf8\uc5d0 \ub300\ud55c \ubbf8\uad6d\uc758 \uc0ac\uacfc\uae4c\uc9c0\ub85c \ud655\ub300\ub418\uc5c8\uace0, \ubbf8\uad6d\uc73c\ub85c\uc11c\ub294 \uad6d\uc81c\ubc95 \uc9c8\uc11c\uc0c1 \ub3c4\uc800\ud788 \uc788\uc744 \uc218 \uc5c6\ub294 \uacf5\uad00(\u516c\u9928)\uacfc \uc678\uad50\uad00\uc5d0 \ub300\ud55c \ud14c\ub7ec\uc640 \uc774\ub780\uc758 \uc694\uad6c\ub294 \ubbf8\uad6d\uc758 \uba85\uc608\uc5d0 \uc9c1\uacb0\ub41c \ubc1b\uc544\ub4e4\uc77c \uc218 \uc5c6\ub294 \uac83\uc774\uc5c8\ub2e4. \uc774\ub780\uce21\uc5d0\uc11c\ub3c4 \uc77c\uc2dc \uc628\uac74\ud30c\uac00 \uc9d1\uad8c, \ud611\uc0c1\uc758 \uae30\ubbf8\uac00 \uc5ff\ubcf4\uc600\uc73c\ub098 \ud638\uba54\uc774\ub2c8\uc758 \uac15\uacbd \uc120\uc5b8\uc73c\ub85c \ub2e4\uc2dc \uacbd\ud654\ub418\uc5c8\uace0, \uc774\uc5d0 \ubbf8\uad6d\ub3c4 \ubcf4\ubcf5\uc744 \uc120\uc5b8, \ubbf8\uad6d\uacfc \uc11c\ubc29\uce21\uc758 \ub300\uc774\ub780 \uad6d\uad50 \ub2e8\uc808 \uacfc \uacbd\uc81c \uc81c\uc7ac \uc870\uce58\ub85c \uc774\uc5b4\uc84c\ub2e4. \uc720\uc5d4\uc744 \uc911\uc2ec\uc73c\ub85c \ud55c \uc911\uc7ac \ub178\ub825\ub3c4 \ubb34\uc0b0\ub418\ub294 \uac00\uc6b4\ub370 1980\ub144 4\uc6d4 \ubbf8\uad6d\uc740 \ud2b9\uacf5\ub300\ub97c \ud22c\uc785, \ubb34\ub825\uc5d0 \uc758\ud55c \uc778\uc9c8\uad6c\ucd9c \uc791\uc804\uc744 \uc2dc\ub3c4\ud588\uc73c\ub098 \uacc4\ud68d \ucc28\uc9c8\uacfc \ud56d\uacf5\uae30 \ucda9\ub3cc\uc0ac\uace0\ub85c 8\uba85\uc758 \ud2b9\uacf5\ub300\uc6d0\uc774 \uc0ac\ub9dd, \uc2e4\ud328\ub85c \ub05d\ub0ac\ub2e4.", "paragraph_answer": "\uc774 \uc0ac\uac74\uc740 \ubbf8\u00b7\uc774\ub780 \uc778\uc9c8 \uc0ac\ud0dc\uc5d0 \uc545\uc7ac\ub85c \uc791\uc6a9\ud558\uc600\uc744 \ubfd0\ub9cc \uc544\ub2c8\ub77c \uac19\uc740 \uc774\uc2ac\ub78c\uad8c\uc758 \uc0ac\uc6b0\ub514\uc544\ub77c\ube44\uc544 \ub4f1 \uc628\uac74\u00b7\uce5c\ubbf8\u00b7\uad70\uc8fc\uad6d\ub4e4\uacfc \uc774\ub780\uc758 \uc815\uce58\uc801 \ub300\ub9bd\uc744 \uc57c\uae30\uc2dc\ucf30\ub2e4. \uc774\ub780\uc758 \uc694\uad6c\ub294 \ud314\ub808\ube44\uc758 \uc2e0\ubcd1 \uc778\ub3c4\uc5d0 \uadf8\uce58\uc9c0 \uc54a\uace0 \ubbf8\uad6d\uc758 \uc774\ub780\uc5d0 \ub300\ud55c \uc8c4\uc0c1(\u7f6a\u72c0)\uc870\uc0ac\uc640 \uadf8\uc5d0 \ub300\ud55c \ubbf8\uad6d\uc758 \uc0ac\uacfc\uae4c\uc9c0\ub85c \ud655\ub300\ub418\uc5c8\uace0, \ubbf8\uad6d\uc73c\ub85c\uc11c\ub294 \uad6d\uc81c\ubc95 \uc9c8\uc11c\uc0c1 \ub3c4\uc800\ud788 \uc788\uc744 \uc218 \uc5c6\ub294 \uacf5\uad00(\u516c\u9928)\uacfc \uc678\uad50\uad00\uc5d0 \ub300\ud55c \ud14c\ub7ec\uc640 \uc774\ub780\uc758 \uc694\uad6c\ub294 \ubbf8\uad6d\uc758 \uba85\uc608\uc5d0 \uc9c1\uacb0\ub41c \ubc1b\uc544\ub4e4\uc77c \uc218 \uc5c6\ub294 \uac83\uc774\uc5c8\ub2e4. \uc774\ub780\uce21\uc5d0\uc11c\ub3c4 \uc77c\uc2dc \uc628\uac74\ud30c\uac00 \uc9d1\uad8c, \ud611\uc0c1\uc758 \uae30\ubbf8\uac00 \uc5ff\ubcf4\uc600\uc73c\ub098 \ud638\uba54\uc774\ub2c8\uc758 \uac15\uacbd \uc120\uc5b8\uc73c\ub85c \ub2e4\uc2dc \uacbd\ud654\ub418\uc5c8\uace0, \uc774\uc5d0 \ubbf8\uad6d\ub3c4 \ubcf4\ubcf5\uc744 \uc120\uc5b8, \ubbf8\uad6d\uacfc \uc11c\ubc29\uce21\uc758 \ub300\uc774\ub780 \uad6d\uad50 \ub2e8\uc808 \uacfc \uacbd\uc81c \uc81c\uc7ac \uc870\uce58\ub85c \uc774\uc5b4\uc84c\ub2e4. \uc720\uc5d4\uc744 \uc911\uc2ec\uc73c\ub85c \ud55c \uc911\uc7ac \ub178\ub825\ub3c4 \ubb34\uc0b0\ub418\ub294 \uac00\uc6b4\ub370 1980\ub144 4\uc6d4 \ubbf8\uad6d\uc740 \ud2b9\uacf5\ub300\ub97c \ud22c\uc785, \ubb34\ub825\uc5d0 \uc758\ud55c \uc778\uc9c8\uad6c\ucd9c \uc791\uc804\uc744 \uc2dc\ub3c4\ud588\uc73c\ub098 \uacc4\ud68d \ucc28\uc9c8\uacfc \ud56d\uacf5\uae30 \ucda9\ub3cc\uc0ac\uace0\ub85c 8\uba85\uc758 \ud2b9\uacf5\ub300\uc6d0\uc774 \uc0ac\ub9dd, \uc2e4\ud328\ub85c \ub05d\ub0ac\ub2e4.", "sentence_answer": "\uc774\uc5d0 \ubbf8\uad6d\ub3c4 \ubcf4\ubcf5\uc744 \uc120\uc5b8, \ubbf8\uad6d\uacfc \uc11c\ubc29\uce21\uc758 \ub300\uc774\ub780 \uad6d\uad50 \ub2e8\uc808 \uacfc \uacbd\uc81c \uc81c\uc7ac \uc870\uce58\ub85c \uc774\uc5b4\uc84c\ub2e4.", "paragraph_id": "6556343-2-1", "paragraph_question": "question: \ubbf8\uad6d\uc774 \uc774\ub780\uc5d0\uac8c \ubcf4\ubcf5\uc801 \uc870\ucde8\ub85c \ucde8\ud55c \uc678\uad50\uc801 \ud589\ub3d9\uc740 \ubb34\uc5c7\uc778\uac00?, context: \uc774 \uc0ac\uac74\uc740 \ubbf8\u00b7\uc774\ub780 \uc778\uc9c8 \uc0ac\ud0dc\uc5d0 \uc545\uc7ac\ub85c \uc791\uc6a9\ud558\uc600\uc744 \ubfd0\ub9cc \uc544\ub2c8\ub77c \uac19\uc740 \uc774\uc2ac\ub78c\uad8c\uc758 \uc0ac\uc6b0\ub514\uc544\ub77c\ube44\uc544 \ub4f1 \uc628\uac74\u00b7\uce5c\ubbf8\u00b7\uad70\uc8fc\uad6d\ub4e4\uacfc \uc774\ub780\uc758 \uc815\uce58\uc801 \ub300\ub9bd\uc744 \uc57c\uae30\uc2dc\ucf30\ub2e4. \uc774\ub780\uc758 \uc694\uad6c\ub294 \ud314\ub808\ube44\uc758 \uc2e0\ubcd1 \uc778\ub3c4\uc5d0 \uadf8\uce58\uc9c0 \uc54a\uace0 \ubbf8\uad6d\uc758 \uc774\ub780\uc5d0 \ub300\ud55c \uc8c4\uc0c1(\u7f6a\u72c0)\uc870\uc0ac\uc640 \uadf8\uc5d0 \ub300\ud55c \ubbf8\uad6d\uc758 \uc0ac\uacfc\uae4c\uc9c0\ub85c \ud655\ub300\ub418\uc5c8\uace0, \ubbf8\uad6d\uc73c\ub85c\uc11c\ub294 \uad6d\uc81c\ubc95 \uc9c8\uc11c\uc0c1 \ub3c4\uc800\ud788 \uc788\uc744 \uc218 \uc5c6\ub294 \uacf5\uad00(\u516c\u9928)\uacfc \uc678\uad50\uad00\uc5d0 \ub300\ud55c \ud14c\ub7ec\uc640 \uc774\ub780\uc758 \uc694\uad6c\ub294 \ubbf8\uad6d\uc758 \uba85\uc608\uc5d0 \uc9c1\uacb0\ub41c \ubc1b\uc544\ub4e4\uc77c \uc218 \uc5c6\ub294 \uac83\uc774\uc5c8\ub2e4. \uc774\ub780\uce21\uc5d0\uc11c\ub3c4 \uc77c\uc2dc \uc628\uac74\ud30c\uac00 \uc9d1\uad8c, \ud611\uc0c1\uc758 \uae30\ubbf8\uac00 \uc5ff\ubcf4\uc600\uc73c\ub098 \ud638\uba54\uc774\ub2c8\uc758 \uac15\uacbd \uc120\uc5b8\uc73c\ub85c \ub2e4\uc2dc \uacbd\ud654\ub418\uc5c8\uace0, \uc774\uc5d0 \ubbf8\uad6d\ub3c4 \ubcf4\ubcf5\uc744 \uc120\uc5b8, \ubbf8\uad6d\uacfc \uc11c\ubc29\uce21\uc758 \ub300\uc774\ub780 \uad6d\uad50 \ub2e8\uc808\uacfc \uacbd\uc81c \uc81c\uc7ac \uc870\uce58\ub85c \uc774\uc5b4\uc84c\ub2e4. \uc720\uc5d4\uc744 \uc911\uc2ec\uc73c\ub85c \ud55c \uc911\uc7ac \ub178\ub825\ub3c4 \ubb34\uc0b0\ub418\ub294 \uac00\uc6b4\ub370 1980\ub144 4\uc6d4 \ubbf8\uad6d\uc740 \ud2b9\uacf5\ub300\ub97c \ud22c\uc785, \ubb34\ub825\uc5d0 \uc758\ud55c \uc778\uc9c8\uad6c\ucd9c \uc791\uc804\uc744 \uc2dc\ub3c4\ud588\uc73c\ub098 \uacc4\ud68d \ucc28\uc9c8\uacfc \ud56d\uacf5\uae30 \ucda9\ub3cc\uc0ac\uace0\ub85c 8\uba85\uc758 \ud2b9\uacf5\ub300\uc6d0\uc774 \uc0ac\ub9dd, \uc2e4\ud328\ub85c \ub05d\ub0ac\ub2e4."} {"question": "\ud45c\uc900\ud654\ub41c \uc81c\ud488 \ubc0f \uc11c\ube44\uc2a4\uc758 \ub300\ub7c9 \uc0dd\uc0b0, \uc800\uac00 \uc11c\ube44\uc2a4\uc5d0 \uc801\ud569\ud55c \uad6c\uc870\ub294?", "paragraph": "\uae30\ub2a5\uc801 \uc870\uc9c1\uc740 \uc77c\ubc18\uc801\uc73c\ub85c \uc218\ud3c9\uc801 \uc870\uc815\uc758 \ud544\uc694\uac00 \uc801\uc740 \uacbd\uc6b0, \ubaa9\ud45c\ub2ec\uc131\uc5d0 \uc788\uc5b4 \uc804\ubb38\uc9c0\uc2dd\uc774 \ud544\uc694\ud55c \uacbd\uc6b0, \ud655\uc2e4\ud55c \ud658\uacbd \uc18d\uc5d0\uc11c \uc77c\uc0c1\uc801\uc778 \uae30\uc220\uc744 \ub2e4\ub8e8\ub294 \uacbd\uc6b0\uc5d0 \uc0ac\uc6a9\ub41c\ub2e4. \uc989, \ud45c\uc900\ud654\ub41c \uc81c\ud488 \ubc0f \uc11c\ube44\uc2a4\uc758 \ub300\ub7c9 \uc0dd\uc0b0\uacfc \uc800\uac00 \uc11c\ube44\uc2a4\uc5d0 \uc801\ud569\ud55c \uad6c\uc870\uc774\ub2e4. \uadf8\ub807\uae30 \ub54c\ubb38\uc5d0 \uc870\uc9c1\uc758 \uc804\uccb4 \uc5c5\ubb34\ub97c \ub3d9\uc77c\ud55c \uae30\ub2a5\ubcc4\ub85c \ubd80\uc11c\ud654\ud55c \uc870\uc9c1\uc5d0\uc11c \uc218\ud3c9\uc801 \uc870\uc815\uc758 \ud544\uc694\uc131\uc774 \ub0ae\uc744 \ub54c \ud6a8\uacfc\uc801\uc774\ub2e4. \uc5c5\ubb34\uc758 \uc870\uc815 \ubc0f \uc804\ubb38\ud654\ub294 \uae30\ub2a5\uc801 \uad6c\uc870\uc5d0 \uc9d1\uc911\ub418\uc5b4 \uc788\uc5b4 \ud55c\uc815\ub41c \uc591\uc758 \uc81c\ud488\uacfc \uc11c\ube44\uc2a4\ub97c \ud6a8\uc728\uc801\uc774\uace0 \uc608\uce21 \uac00\ub2a5\ud558\uac8c \ud558\uc5ec \ubb3c\uac74\uc744 \ub0ae\uc740 \uac00\uaca9\uc73c\ub85c \ube60\ub974\uac8c \ubc30\ubd84\ud558\uace0 \ud310\ub9e4\ud560 \uc218 \uc788\uac8c \ud55c\ub2e4. \ub610\ud55c \ud2b9\uc815 \uae30\ub2a5\uc5d0 \uad00\ub828\ub41c \uad6c\uc131\uc6d0\ub4e4\uc758 \uc9c0\uc2dd\uacfc \uae30\uc220\uc774 \ud1b5\ud569\uc801\uc73c\ub85c \ud65c\uc6a9\ub418\uc5b4 \uae30\ub2a5\ubcc4 \uc911\ubcf5\uc744 \ubc29\uc9c0\ud560 \uc218 \uc788\uace0, \uadf8\ub4e4\uc758 \ud65c\ub3d9\uc744 \uc218\uc9c1\uc801\uc73c\ub85c \ud1b5\ud569\uc2dc\ucf1c \ud6a8\uc728\uc131\uc744 \ub192\uc5ec\uc900\ub2e4. \uae30\ub2a5\uc801 \uad6c\uc870\ub294 \uc804\ubb38\uc131\uc744 \uc9c0\ub2cc \uc804\ubb38\uac00 \ud65c\uc6a9\uc758 \uc720\uc6a9\uc131\uc774 \ub192\ub2e4. \ub610\ud55c \uc804\uc0ac\uc870\uc9c1\uccb4\uacc4\uc5d0 \ubcc0\uacbd\uc774 \uc5c6\uace0 \ubd80\uc11c \ub0b4\uc5d0 \uba85\ud655\ud558\uac8c \uc815\uc758\ub41c \ucc45\uc784\uacfc \uc5ed\ud560\uc774 \uc788\uae30 \ub54c\ubb38\uc5d0 \ud6a8\uc728\uc801\uc774\ub2e4. \ud558\uc9c0\ub9cc \ubd80\uc11c \uac04\uc758 \ucc45\uc784 \ubd84\uc0b0\uc73c\ub85c \ud1b5\ud569 \uae30\ub2a5\uc774 \ubd80\uc7ac\ub418\uace0 \uac08\ub4f1\uc774 \ubc1c\uc0dd\ud560 \uac00\ub2a5\uc131\uc774 \ub192\uc73c\uba70 \uc694\uad6c\uc0ac\ud56d\uc5d0 \ub300\ud55c \ub300\uc751\uc774 \ub290\ub9ac\ub2e4\ub294 \ub2e8\uc810\uc774 \uc788\ub2e4.", "answer": "\uae30\ub2a5\uc801 \uc870\uc9c1", "sentence": "\uae30\ub2a5\uc801 \uc870\uc9c1 \uc740 \uc77c\ubc18\uc801\uc73c\ub85c \uc218\ud3c9\uc801 \uc870\uc815\uc758 \ud544\uc694\uac00 \uc801\uc740 \uacbd\uc6b0, \ubaa9\ud45c\ub2ec\uc131\uc5d0 \uc788\uc5b4 \uc804\ubb38\uc9c0\uc2dd\uc774 \ud544\uc694\ud55c \uacbd\uc6b0, \ud655\uc2e4\ud55c \ud658\uacbd \uc18d\uc5d0\uc11c \uc77c\uc0c1\uc801\uc778 \uae30\uc220\uc744 \ub2e4\ub8e8\ub294 \uacbd\uc6b0\uc5d0 \uc0ac\uc6a9\ub41c\ub2e4.", "paragraph_sentence": " \uae30\ub2a5\uc801 \uc870\uc9c1 \uc740 \uc77c\ubc18\uc801\uc73c\ub85c \uc218\ud3c9\uc801 \uc870\uc815\uc758 \ud544\uc694\uac00 \uc801\uc740 \uacbd\uc6b0, \ubaa9\ud45c\ub2ec\uc131\uc5d0 \uc788\uc5b4 \uc804\ubb38\uc9c0\uc2dd\uc774 \ud544\uc694\ud55c \uacbd\uc6b0, \ud655\uc2e4\ud55c \ud658\uacbd \uc18d\uc5d0\uc11c \uc77c\uc0c1\uc801\uc778 \uae30\uc220\uc744 \ub2e4\ub8e8\ub294 \uacbd\uc6b0\uc5d0 \uc0ac\uc6a9\ub41c\ub2e4. \uc989, \ud45c\uc900\ud654\ub41c \uc81c\ud488 \ubc0f \uc11c\ube44\uc2a4\uc758 \ub300\ub7c9 \uc0dd\uc0b0\uacfc \uc800\uac00 \uc11c\ube44\uc2a4\uc5d0 \uc801\ud569\ud55c \uad6c\uc870\uc774\ub2e4. \uadf8\ub807\uae30 \ub54c\ubb38\uc5d0 \uc870\uc9c1\uc758 \uc804\uccb4 \uc5c5\ubb34\ub97c \ub3d9\uc77c\ud55c \uae30\ub2a5\ubcc4\ub85c \ubd80\uc11c\ud654\ud55c \uc870\uc9c1\uc5d0\uc11c \uc218\ud3c9\uc801 \uc870\uc815\uc758 \ud544\uc694\uc131\uc774 \ub0ae\uc744 \ub54c \ud6a8\uacfc\uc801\uc774\ub2e4. \uc5c5\ubb34\uc758 \uc870\uc815 \ubc0f \uc804\ubb38\ud654\ub294 \uae30\ub2a5\uc801 \uad6c\uc870\uc5d0 \uc9d1\uc911\ub418\uc5b4 \uc788\uc5b4 \ud55c\uc815\ub41c \uc591\uc758 \uc81c\ud488\uacfc \uc11c\ube44\uc2a4\ub97c \ud6a8\uc728\uc801\uc774\uace0 \uc608\uce21 \uac00\ub2a5\ud558\uac8c \ud558\uc5ec \ubb3c\uac74\uc744 \ub0ae\uc740 \uac00\uaca9\uc73c\ub85c \ube60\ub974\uac8c \ubc30\ubd84\ud558\uace0 \ud310\ub9e4\ud560 \uc218 \uc788\uac8c \ud55c\ub2e4. \ub610\ud55c \ud2b9\uc815 \uae30\ub2a5\uc5d0 \uad00\ub828\ub41c \uad6c\uc131\uc6d0\ub4e4\uc758 \uc9c0\uc2dd\uacfc \uae30\uc220\uc774 \ud1b5\ud569\uc801\uc73c\ub85c \ud65c\uc6a9\ub418\uc5b4 \uae30\ub2a5\ubcc4 \uc911\ubcf5\uc744 \ubc29\uc9c0\ud560 \uc218 \uc788\uace0, \uadf8\ub4e4\uc758 \ud65c\ub3d9\uc744 \uc218\uc9c1\uc801\uc73c\ub85c \ud1b5\ud569\uc2dc\ucf1c \ud6a8\uc728\uc131\uc744 \ub192\uc5ec\uc900\ub2e4. \uae30\ub2a5\uc801 \uad6c\uc870\ub294 \uc804\ubb38\uc131\uc744 \uc9c0\ub2cc \uc804\ubb38\uac00 \ud65c\uc6a9\uc758 \uc720\uc6a9\uc131\uc774 \ub192\ub2e4. \ub610\ud55c \uc804\uc0ac\uc870\uc9c1\uccb4\uacc4\uc5d0 \ubcc0\uacbd\uc774 \uc5c6\uace0 \ubd80\uc11c \ub0b4\uc5d0 \uba85\ud655\ud558\uac8c \uc815\uc758\ub41c \ucc45\uc784\uacfc \uc5ed\ud560\uc774 \uc788\uae30 \ub54c\ubb38\uc5d0 \ud6a8\uc728\uc801\uc774\ub2e4. \ud558\uc9c0\ub9cc \ubd80\uc11c \uac04\uc758 \ucc45\uc784 \ubd84\uc0b0\uc73c\ub85c \ud1b5\ud569 \uae30\ub2a5\uc774 \ubd80\uc7ac\ub418\uace0 \uac08\ub4f1\uc774 \ubc1c\uc0dd\ud560 \uac00\ub2a5\uc131\uc774 \ub192\uc73c\uba70 \uc694\uad6c\uc0ac\ud56d\uc5d0 \ub300\ud55c \ub300\uc751\uc774 \ub290\ub9ac\ub2e4\ub294 \ub2e8\uc810\uc774 \uc788\ub2e4.", "paragraph_answer": " \uae30\ub2a5\uc801 \uc870\uc9c1 \uc740 \uc77c\ubc18\uc801\uc73c\ub85c \uc218\ud3c9\uc801 \uc870\uc815\uc758 \ud544\uc694\uac00 \uc801\uc740 \uacbd\uc6b0, \ubaa9\ud45c\ub2ec\uc131\uc5d0 \uc788\uc5b4 \uc804\ubb38\uc9c0\uc2dd\uc774 \ud544\uc694\ud55c \uacbd\uc6b0, \ud655\uc2e4\ud55c \ud658\uacbd \uc18d\uc5d0\uc11c \uc77c\uc0c1\uc801\uc778 \uae30\uc220\uc744 \ub2e4\ub8e8\ub294 \uacbd\uc6b0\uc5d0 \uc0ac\uc6a9\ub41c\ub2e4. \uc989, \ud45c\uc900\ud654\ub41c \uc81c\ud488 \ubc0f \uc11c\ube44\uc2a4\uc758 \ub300\ub7c9 \uc0dd\uc0b0\uacfc \uc800\uac00 \uc11c\ube44\uc2a4\uc5d0 \uc801\ud569\ud55c \uad6c\uc870\uc774\ub2e4. \uadf8\ub807\uae30 \ub54c\ubb38\uc5d0 \uc870\uc9c1\uc758 \uc804\uccb4 \uc5c5\ubb34\ub97c \ub3d9\uc77c\ud55c \uae30\ub2a5\ubcc4\ub85c \ubd80\uc11c\ud654\ud55c \uc870\uc9c1\uc5d0\uc11c \uc218\ud3c9\uc801 \uc870\uc815\uc758 \ud544\uc694\uc131\uc774 \ub0ae\uc744 \ub54c \ud6a8\uacfc\uc801\uc774\ub2e4. \uc5c5\ubb34\uc758 \uc870\uc815 \ubc0f \uc804\ubb38\ud654\ub294 \uae30\ub2a5\uc801 \uad6c\uc870\uc5d0 \uc9d1\uc911\ub418\uc5b4 \uc788\uc5b4 \ud55c\uc815\ub41c \uc591\uc758 \uc81c\ud488\uacfc \uc11c\ube44\uc2a4\ub97c \ud6a8\uc728\uc801\uc774\uace0 \uc608\uce21 \uac00\ub2a5\ud558\uac8c \ud558\uc5ec \ubb3c\uac74\uc744 \ub0ae\uc740 \uac00\uaca9\uc73c\ub85c \ube60\ub974\uac8c \ubc30\ubd84\ud558\uace0 \ud310\ub9e4\ud560 \uc218 \uc788\uac8c \ud55c\ub2e4. \ub610\ud55c \ud2b9\uc815 \uae30\ub2a5\uc5d0 \uad00\ub828\ub41c \uad6c\uc131\uc6d0\ub4e4\uc758 \uc9c0\uc2dd\uacfc \uae30\uc220\uc774 \ud1b5\ud569\uc801\uc73c\ub85c \ud65c\uc6a9\ub418\uc5b4 \uae30\ub2a5\ubcc4 \uc911\ubcf5\uc744 \ubc29\uc9c0\ud560 \uc218 \uc788\uace0, \uadf8\ub4e4\uc758 \ud65c\ub3d9\uc744 \uc218\uc9c1\uc801\uc73c\ub85c \ud1b5\ud569\uc2dc\ucf1c \ud6a8\uc728\uc131\uc744 \ub192\uc5ec\uc900\ub2e4. \uae30\ub2a5\uc801 \uad6c\uc870\ub294 \uc804\ubb38\uc131\uc744 \uc9c0\ub2cc \uc804\ubb38\uac00 \ud65c\uc6a9\uc758 \uc720\uc6a9\uc131\uc774 \ub192\ub2e4. \ub610\ud55c \uc804\uc0ac\uc870\uc9c1\uccb4\uacc4\uc5d0 \ubcc0\uacbd\uc774 \uc5c6\uace0 \ubd80\uc11c \ub0b4\uc5d0 \uba85\ud655\ud558\uac8c \uc815\uc758\ub41c \ucc45\uc784\uacfc \uc5ed\ud560\uc774 \uc788\uae30 \ub54c\ubb38\uc5d0 \ud6a8\uc728\uc801\uc774\ub2e4. \ud558\uc9c0\ub9cc \ubd80\uc11c \uac04\uc758 \ucc45\uc784 \ubd84\uc0b0\uc73c\ub85c \ud1b5\ud569 \uae30\ub2a5\uc774 \ubd80\uc7ac\ub418\uace0 \uac08\ub4f1\uc774 \ubc1c\uc0dd\ud560 \uac00\ub2a5\uc131\uc774 \ub192\uc73c\uba70 \uc694\uad6c\uc0ac\ud56d\uc5d0 \ub300\ud55c \ub300\uc751\uc774 \ub290\ub9ac\ub2e4\ub294 \ub2e8\uc810\uc774 \uc788\ub2e4.", "sentence_answer": " \uae30\ub2a5\uc801 \uc870\uc9c1 \uc740 \uc77c\ubc18\uc801\uc73c\ub85c \uc218\ud3c9\uc801 \uc870\uc815\uc758 \ud544\uc694\uac00 \uc801\uc740 \uacbd\uc6b0, \ubaa9\ud45c\ub2ec\uc131\uc5d0 \uc788\uc5b4 \uc804\ubb38\uc9c0\uc2dd\uc774 \ud544\uc694\ud55c \uacbd\uc6b0, \ud655\uc2e4\ud55c \ud658\uacbd \uc18d\uc5d0\uc11c \uc77c\uc0c1\uc801\uc778 \uae30\uc220\uc744 \ub2e4\ub8e8\ub294 \uacbd\uc6b0\uc5d0 \uc0ac\uc6a9\ub41c\ub2e4.", "paragraph_id": "5886327-11-0", "paragraph_question": "question: \ud45c\uc900\ud654\ub41c \uc81c\ud488 \ubc0f \uc11c\ube44\uc2a4\uc758 \ub300\ub7c9 \uc0dd\uc0b0, \uc800\uac00 \uc11c\ube44\uc2a4\uc5d0 \uc801\ud569\ud55c \uad6c\uc870\ub294?, context: \uae30\ub2a5\uc801 \uc870\uc9c1\uc740 \uc77c\ubc18\uc801\uc73c\ub85c \uc218\ud3c9\uc801 \uc870\uc815\uc758 \ud544\uc694\uac00 \uc801\uc740 \uacbd\uc6b0, \ubaa9\ud45c\ub2ec\uc131\uc5d0 \uc788\uc5b4 \uc804\ubb38\uc9c0\uc2dd\uc774 \ud544\uc694\ud55c \uacbd\uc6b0, \ud655\uc2e4\ud55c \ud658\uacbd \uc18d\uc5d0\uc11c \uc77c\uc0c1\uc801\uc778 \uae30\uc220\uc744 \ub2e4\ub8e8\ub294 \uacbd\uc6b0\uc5d0 \uc0ac\uc6a9\ub41c\ub2e4. \uc989, \ud45c\uc900\ud654\ub41c \uc81c\ud488 \ubc0f \uc11c\ube44\uc2a4\uc758 \ub300\ub7c9 \uc0dd\uc0b0\uacfc \uc800\uac00 \uc11c\ube44\uc2a4\uc5d0 \uc801\ud569\ud55c \uad6c\uc870\uc774\ub2e4. \uadf8\ub807\uae30 \ub54c\ubb38\uc5d0 \uc870\uc9c1\uc758 \uc804\uccb4 \uc5c5\ubb34\ub97c \ub3d9\uc77c\ud55c \uae30\ub2a5\ubcc4\ub85c \ubd80\uc11c\ud654\ud55c \uc870\uc9c1\uc5d0\uc11c \uc218\ud3c9\uc801 \uc870\uc815\uc758 \ud544\uc694\uc131\uc774 \ub0ae\uc744 \ub54c \ud6a8\uacfc\uc801\uc774\ub2e4. \uc5c5\ubb34\uc758 \uc870\uc815 \ubc0f \uc804\ubb38\ud654\ub294 \uae30\ub2a5\uc801 \uad6c\uc870\uc5d0 \uc9d1\uc911\ub418\uc5b4 \uc788\uc5b4 \ud55c\uc815\ub41c \uc591\uc758 \uc81c\ud488\uacfc \uc11c\ube44\uc2a4\ub97c \ud6a8\uc728\uc801\uc774\uace0 \uc608\uce21 \uac00\ub2a5\ud558\uac8c \ud558\uc5ec \ubb3c\uac74\uc744 \ub0ae\uc740 \uac00\uaca9\uc73c\ub85c \ube60\ub974\uac8c \ubc30\ubd84\ud558\uace0 \ud310\ub9e4\ud560 \uc218 \uc788\uac8c \ud55c\ub2e4. \ub610\ud55c \ud2b9\uc815 \uae30\ub2a5\uc5d0 \uad00\ub828\ub41c \uad6c\uc131\uc6d0\ub4e4\uc758 \uc9c0\uc2dd\uacfc \uae30\uc220\uc774 \ud1b5\ud569\uc801\uc73c\ub85c \ud65c\uc6a9\ub418\uc5b4 \uae30\ub2a5\ubcc4 \uc911\ubcf5\uc744 \ubc29\uc9c0\ud560 \uc218 \uc788\uace0, \uadf8\ub4e4\uc758 \ud65c\ub3d9\uc744 \uc218\uc9c1\uc801\uc73c\ub85c \ud1b5\ud569\uc2dc\ucf1c \ud6a8\uc728\uc131\uc744 \ub192\uc5ec\uc900\ub2e4. \uae30\ub2a5\uc801 \uad6c\uc870\ub294 \uc804\ubb38\uc131\uc744 \uc9c0\ub2cc \uc804\ubb38\uac00 \ud65c\uc6a9\uc758 \uc720\uc6a9\uc131\uc774 \ub192\ub2e4. \ub610\ud55c \uc804\uc0ac\uc870\uc9c1\uccb4\uacc4\uc5d0 \ubcc0\uacbd\uc774 \uc5c6\uace0 \ubd80\uc11c \ub0b4\uc5d0 \uba85\ud655\ud558\uac8c \uc815\uc758\ub41c \ucc45\uc784\uacfc \uc5ed\ud560\uc774 \uc788\uae30 \ub54c\ubb38\uc5d0 \ud6a8\uc728\uc801\uc774\ub2e4. \ud558\uc9c0\ub9cc \ubd80\uc11c \uac04\uc758 \ucc45\uc784 \ubd84\uc0b0\uc73c\ub85c \ud1b5\ud569 \uae30\ub2a5\uc774 \ubd80\uc7ac\ub418\uace0 \uac08\ub4f1\uc774 \ubc1c\uc0dd\ud560 \uac00\ub2a5\uc131\uc774 \ub192\uc73c\uba70 \uc694\uad6c\uc0ac\ud56d\uc5d0 \ub300\ud55c \ub300\uc751\uc774 \ub290\ub9ac\ub2e4\ub294 \ub2e8\uc810\uc774 \uc788\ub2e4."} {"question": "\ud50c\ub8e8\ud1a0\ub284-240\uc740 \ud575\uc5f0\uc1c4 \ubc18\uc751 \uc18d\ub3c4\uac00 \ub108\ubb34 \ube68\ub77c \ubb34\uc5c7\uc744 \uc720\uc9c0\ud558\ub294 \uac83\uc774 \ubd88\uac00\ub2a5 \ud558\uc600\ub294\uac00?", "paragraph": "1943\ub144, \ud50c\ub8e8\ud1a0\ub284 \ud575\ud3ed\ud0c4 \uac1c\ubc1c\uc758 \ud575\uc2ec \uacfc\uc81c\ub294 \ud2f4 \ub9e8\uc774\ub77c \ubd88\ub9b0 \ud3ec\uc2e0\ud615 \ud3ed\ud0c4\uc758 \uac1c\ubc1c\uc774\uc5c8\ub2e4. \ucd08\uae30 \uc2e4\ud5d8 \ub2e8\uacc4\uc5d0\uc11c \uc0ac\uc774\ud074\ub85c\ud2b8\ub860\uc744 \uc774\uc6a9\ud558\uc5ec \uc21c\ub3c4 \ub192\uc740 \uace0\ub18d\ucd95 \ud50c\ub8e8\ud1a0\ub284-239\ub97c \ud655\ubcf4\ud558\uc600\uc73c\ub098 \uadf8 \uc591\uc740 \ub9e4\uc6b0 \uc801\uc5c8\ub2e4. \ub85c\uc2a4\uc568\ub7ec\ubaa8\uc2a4\ub294 1944\ub144 4\uc6d4 \ud074\ub9b0\ud134\uc758 X-10 \ubc18\uc751\ub85c\uc5d0\uc11c \uc0dd\uc131\ub41c \ud50c\ub8e8\ud1a0\ub284\uc744 \uc778\uc218\ud558\uc600\ub2e4. \uadf8\ub7ec\ub098, \ud50c\ub8e8\ud1a0\ub284\uc744 \ubc18\uc751\ub85c\uc5d0\uc11c \uaebc\ub0b8 \ub4a4\uc5d0 \uc77c\uc5b4\ub098\ub294 \uc790\uc5f0\ubc1c\uc0dd\uc801\uc778 \ud575\ubd84\uc5f4\uacfc\uc815\uc73c\ub85c \uc778\ud574 \uc778\uc218 \ubc1b\uc740 \ud50c\ub8e8\ud1a0\ub284-239\uc758 \uc0c1\ub2f9\ub7c9\uc774 \uba70\uce60 \uc0ac\uc774\uc5d0 \ud50c\ub8e8\ud1a0\ub284-240\uc73c\ub85c \ubcc0\ud658\ub418\uc5c8\ub2e4. \uc2dc\ubcf4\uadf8\ub294 1943\uc5d0 \uc774\ubbf8 \uc5ec\ubd84\uc758 \uc911\uc131\uc790\uac00 \ub0a8\uc544\uc788\uc744 \uacbd\uc6b0 \ud50c\ub8e8\ud1a0\ub284-239\uac00 \ud50c\ub8e8\ud1a0\ub284-240\uc73c\ub85c \ubcc0\ud658\ub420 \uc218 \uc788\ub2e4\ub294 \uc810\uc744 \uc608\uacac\ud55c \ubc14 \uc788\ub2e4. \uc774\ub7ec\ud55c \ubb38\uc81c\ub85c \uc778\ud574 \ubc18\uc751\ub85c\uc5d0\uc11c \uc0dd\uc131\ub41c \ud50c\ub8e8\ud1a0\ub284\uc740 \ud3ec\uc2e0\ud615 \ubb34\uae30 \ud615\ud0dc\ub85c \uc81c\uc791\ud558\ub294\ub370 \uc801\ud569\ud558\uc9c0 \uc54a\uc558\ub2e4. \ud50c\ub8e8\ud1a0\ub284-240\uc740 \ud575 \uc5f0\uc1c4 \ubc18\uc751 \uc18d\ub3c4\uac00 \ub108\ubb34 \ube68\ub77c \ud575\ud3ed\ud0c4\uc758 \ud3ed\ubc1c \uc2dc\uc810\uc5d0 \uc784\uacc4 \uc9c8\ub7c9\uc744 \uc720\uc9c0\ud558\ub294 \uac83\uc774 \ubd88\uac00\ub2a5\ud558\uc600\uae30 \ub54c\ubb38\uc774\ub2e4.", "answer": "\ud575\ud3ed\ud0c4\uc758 \ud3ed\ubc1c \uc2dc\uc810\uc5d0 \uc784\uacc4 \uc9c8\ub7c9\uc744 \uc720\uc9c0\ud558\ub294 \uac83", "sentence": "\ud50c\ub8e8\ud1a0\ub284-240\uc740 \ud575 \uc5f0\uc1c4 \ubc18\uc751 \uc18d\ub3c4\uac00 \ub108\ubb34 \ube68\ub77c \ud575\ud3ed\ud0c4\uc758 \ud3ed\ubc1c \uc2dc\uc810\uc5d0 \uc784\uacc4 \uc9c8\ub7c9\uc744 \uc720\uc9c0\ud558\ub294 \uac83 \uc774 \ubd88\uac00\ub2a5\ud558\uc600\uae30 \ub54c\ubb38\uc774\ub2e4.", "paragraph_sentence": "1943\ub144, \ud50c\ub8e8\ud1a0\ub284 \ud575\ud3ed\ud0c4 \uac1c\ubc1c\uc758 \ud575\uc2ec \uacfc\uc81c\ub294 \ud2f4 \ub9e8\uc774\ub77c \ubd88\ub9b0 \ud3ec\uc2e0\ud615 \ud3ed\ud0c4\uc758 \uac1c\ubc1c\uc774\uc5c8\ub2e4. \ucd08\uae30 \uc2e4\ud5d8 \ub2e8\uacc4\uc5d0\uc11c \uc0ac\uc774\ud074\ub85c\ud2b8\ub860\uc744 \uc774\uc6a9\ud558\uc5ec \uc21c\ub3c4 \ub192\uc740 \uace0\ub18d\ucd95 \ud50c\ub8e8\ud1a0\ub284-239\ub97c \ud655\ubcf4\ud558\uc600\uc73c\ub098 \uadf8 \uc591\uc740 \ub9e4\uc6b0 \uc801\uc5c8\ub2e4. \ub85c\uc2a4\uc568\ub7ec\ubaa8\uc2a4\ub294 1944\ub144 4\uc6d4 \ud074\ub9b0\ud134\uc758 X-10 \ubc18\uc751\ub85c\uc5d0\uc11c \uc0dd\uc131\ub41c \ud50c\ub8e8\ud1a0\ub284\uc744 \uc778\uc218\ud558\uc600\ub2e4. \uadf8\ub7ec\ub098, \ud50c\ub8e8\ud1a0\ub284\uc744 \ubc18\uc751\ub85c\uc5d0\uc11c \uaebc\ub0b8 \ub4a4\uc5d0 \uc77c\uc5b4\ub098\ub294 \uc790\uc5f0\ubc1c\uc0dd\uc801\uc778 \ud575\ubd84\uc5f4\uacfc\uc815\uc73c\ub85c \uc778\ud574 \uc778\uc218 \ubc1b\uc740 \ud50c\ub8e8\ud1a0\ub284-239\uc758 \uc0c1\ub2f9\ub7c9\uc774 \uba70\uce60 \uc0ac\uc774\uc5d0 \ud50c\ub8e8\ud1a0\ub284-240\uc73c\ub85c \ubcc0\ud658\ub418\uc5c8\ub2e4. \uc2dc\ubcf4\uadf8\ub294 1943\uc5d0 \uc774\ubbf8 \uc5ec\ubd84\uc758 \uc911\uc131\uc790\uac00 \ub0a8\uc544\uc788\uc744 \uacbd\uc6b0 \ud50c\ub8e8\ud1a0\ub284-239\uac00 \ud50c\ub8e8\ud1a0\ub284-240\uc73c\ub85c \ubcc0\ud658\ub420 \uc218 \uc788\ub2e4\ub294 \uc810\uc744 \uc608\uacac\ud55c \ubc14 \uc788\ub2e4. \uc774\ub7ec\ud55c \ubb38\uc81c\ub85c \uc778\ud574 \ubc18\uc751\ub85c\uc5d0\uc11c \uc0dd\uc131\ub41c \ud50c\ub8e8\ud1a0\ub284\uc740 \ud3ec\uc2e0\ud615 \ubb34\uae30 \ud615\ud0dc\ub85c \uc81c\uc791\ud558\ub294\ub370 \uc801\ud569\ud558\uc9c0 \uc54a\uc558\ub2e4. \ud50c\ub8e8\ud1a0\ub284-240\uc740 \ud575 \uc5f0\uc1c4 \ubc18\uc751 \uc18d\ub3c4\uac00 \ub108\ubb34 \ube68\ub77c \ud575\ud3ed\ud0c4\uc758 \ud3ed\ubc1c \uc2dc\uc810\uc5d0 \uc784\uacc4 \uc9c8\ub7c9\uc744 \uc720\uc9c0\ud558\ub294 \uac83 \uc774 \ubd88\uac00\ub2a5\ud558\uc600\uae30 \ub54c\ubb38\uc774\ub2e4. ", "paragraph_answer": "1943\ub144, \ud50c\ub8e8\ud1a0\ub284 \ud575\ud3ed\ud0c4 \uac1c\ubc1c\uc758 \ud575\uc2ec \uacfc\uc81c\ub294 \ud2f4 \ub9e8\uc774\ub77c \ubd88\ub9b0 \ud3ec\uc2e0\ud615 \ud3ed\ud0c4\uc758 \uac1c\ubc1c\uc774\uc5c8\ub2e4. \ucd08\uae30 \uc2e4\ud5d8 \ub2e8\uacc4\uc5d0\uc11c \uc0ac\uc774\ud074\ub85c\ud2b8\ub860\uc744 \uc774\uc6a9\ud558\uc5ec \uc21c\ub3c4 \ub192\uc740 \uace0\ub18d\ucd95 \ud50c\ub8e8\ud1a0\ub284-239\ub97c \ud655\ubcf4\ud558\uc600\uc73c\ub098 \uadf8 \uc591\uc740 \ub9e4\uc6b0 \uc801\uc5c8\ub2e4. \ub85c\uc2a4\uc568\ub7ec\ubaa8\uc2a4\ub294 1944\ub144 4\uc6d4 \ud074\ub9b0\ud134\uc758 X-10 \ubc18\uc751\ub85c\uc5d0\uc11c \uc0dd\uc131\ub41c \ud50c\ub8e8\ud1a0\ub284\uc744 \uc778\uc218\ud558\uc600\ub2e4. \uadf8\ub7ec\ub098, \ud50c\ub8e8\ud1a0\ub284\uc744 \ubc18\uc751\ub85c\uc5d0\uc11c \uaebc\ub0b8 \ub4a4\uc5d0 \uc77c\uc5b4\ub098\ub294 \uc790\uc5f0\ubc1c\uc0dd\uc801\uc778 \ud575\ubd84\uc5f4\uacfc\uc815\uc73c\ub85c \uc778\ud574 \uc778\uc218 \ubc1b\uc740 \ud50c\ub8e8\ud1a0\ub284-239\uc758 \uc0c1\ub2f9\ub7c9\uc774 \uba70\uce60 \uc0ac\uc774\uc5d0 \ud50c\ub8e8\ud1a0\ub284-240\uc73c\ub85c \ubcc0\ud658\ub418\uc5c8\ub2e4. \uc2dc\ubcf4\uadf8\ub294 1943\uc5d0 \uc774\ubbf8 \uc5ec\ubd84\uc758 \uc911\uc131\uc790\uac00 \ub0a8\uc544\uc788\uc744 \uacbd\uc6b0 \ud50c\ub8e8\ud1a0\ub284-239\uac00 \ud50c\ub8e8\ud1a0\ub284-240\uc73c\ub85c \ubcc0\ud658\ub420 \uc218 \uc788\ub2e4\ub294 \uc810\uc744 \uc608\uacac\ud55c \ubc14 \uc788\ub2e4. \uc774\ub7ec\ud55c \ubb38\uc81c\ub85c \uc778\ud574 \ubc18\uc751\ub85c\uc5d0\uc11c \uc0dd\uc131\ub41c \ud50c\ub8e8\ud1a0\ub284\uc740 \ud3ec\uc2e0\ud615 \ubb34\uae30 \ud615\ud0dc\ub85c \uc81c\uc791\ud558\ub294\ub370 \uc801\ud569\ud558\uc9c0 \uc54a\uc558\ub2e4. \ud50c\ub8e8\ud1a0\ub284-240\uc740 \ud575 \uc5f0\uc1c4 \ubc18\uc751 \uc18d\ub3c4\uac00 \ub108\ubb34 \ube68\ub77c \ud575\ud3ed\ud0c4\uc758 \ud3ed\ubc1c \uc2dc\uc810\uc5d0 \uc784\uacc4 \uc9c8\ub7c9\uc744 \uc720\uc9c0\ud558\ub294 \uac83 \uc774 \ubd88\uac00\ub2a5\ud558\uc600\uae30 \ub54c\ubb38\uc774\ub2e4.", "sentence_answer": "\ud50c\ub8e8\ud1a0\ub284-240\uc740 \ud575 \uc5f0\uc1c4 \ubc18\uc751 \uc18d\ub3c4\uac00 \ub108\ubb34 \ube68\ub77c \ud575\ud3ed\ud0c4\uc758 \ud3ed\ubc1c \uc2dc\uc810\uc5d0 \uc784\uacc4 \uc9c8\ub7c9\uc744 \uc720\uc9c0\ud558\ub294 \uac83 \uc774 \ubd88\uac00\ub2a5\ud558\uc600\uae30 \ub54c\ubb38\uc774\ub2e4.", "paragraph_id": "6502177-18-2", "paragraph_question": "question: \ud50c\ub8e8\ud1a0\ub284-240\uc740 \ud575\uc5f0\uc1c4 \ubc18\uc751 \uc18d\ub3c4\uac00 \ub108\ubb34 \ube68\ub77c \ubb34\uc5c7\uc744 \uc720\uc9c0\ud558\ub294 \uac83\uc774 \ubd88\uac00\ub2a5 \ud558\uc600\ub294\uac00?, context: 1943\ub144, \ud50c\ub8e8\ud1a0\ub284 \ud575\ud3ed\ud0c4 \uac1c\ubc1c\uc758 \ud575\uc2ec \uacfc\uc81c\ub294 \ud2f4 \ub9e8\uc774\ub77c \ubd88\ub9b0 \ud3ec\uc2e0\ud615 \ud3ed\ud0c4\uc758 \uac1c\ubc1c\uc774\uc5c8\ub2e4. \ucd08\uae30 \uc2e4\ud5d8 \ub2e8\uacc4\uc5d0\uc11c \uc0ac\uc774\ud074\ub85c\ud2b8\ub860\uc744 \uc774\uc6a9\ud558\uc5ec \uc21c\ub3c4 \ub192\uc740 \uace0\ub18d\ucd95 \ud50c\ub8e8\ud1a0\ub284-239\ub97c \ud655\ubcf4\ud558\uc600\uc73c\ub098 \uadf8 \uc591\uc740 \ub9e4\uc6b0 \uc801\uc5c8\ub2e4. \ub85c\uc2a4\uc568\ub7ec\ubaa8\uc2a4\ub294 1944\ub144 4\uc6d4 \ud074\ub9b0\ud134\uc758 X-10 \ubc18\uc751\ub85c\uc5d0\uc11c \uc0dd\uc131\ub41c \ud50c\ub8e8\ud1a0\ub284\uc744 \uc778\uc218\ud558\uc600\ub2e4. \uadf8\ub7ec\ub098, \ud50c\ub8e8\ud1a0\ub284\uc744 \ubc18\uc751\ub85c\uc5d0\uc11c \uaebc\ub0b8 \ub4a4\uc5d0 \uc77c\uc5b4\ub098\ub294 \uc790\uc5f0\ubc1c\uc0dd\uc801\uc778 \ud575\ubd84\uc5f4\uacfc\uc815\uc73c\ub85c \uc778\ud574 \uc778\uc218 \ubc1b\uc740 \ud50c\ub8e8\ud1a0\ub284-239\uc758 \uc0c1\ub2f9\ub7c9\uc774 \uba70\uce60 \uc0ac\uc774\uc5d0 \ud50c\ub8e8\ud1a0\ub284-240\uc73c\ub85c \ubcc0\ud658\ub418\uc5c8\ub2e4. \uc2dc\ubcf4\uadf8\ub294 1943\uc5d0 \uc774\ubbf8 \uc5ec\ubd84\uc758 \uc911\uc131\uc790\uac00 \ub0a8\uc544\uc788\uc744 \uacbd\uc6b0 \ud50c\ub8e8\ud1a0\ub284-239\uac00 \ud50c\ub8e8\ud1a0\ub284-240\uc73c\ub85c \ubcc0\ud658\ub420 \uc218 \uc788\ub2e4\ub294 \uc810\uc744 \uc608\uacac\ud55c \ubc14 \uc788\ub2e4. \uc774\ub7ec\ud55c \ubb38\uc81c\ub85c \uc778\ud574 \ubc18\uc751\ub85c\uc5d0\uc11c \uc0dd\uc131\ub41c \ud50c\ub8e8\ud1a0\ub284\uc740 \ud3ec\uc2e0\ud615 \ubb34\uae30 \ud615\ud0dc\ub85c \uc81c\uc791\ud558\ub294\ub370 \uc801\ud569\ud558\uc9c0 \uc54a\uc558\ub2e4. \ud50c\ub8e8\ud1a0\ub284-240\uc740 \ud575 \uc5f0\uc1c4 \ubc18\uc751 \uc18d\ub3c4\uac00 \ub108\ubb34 \ube68\ub77c \ud575\ud3ed\ud0c4\uc758 \ud3ed\ubc1c \uc2dc\uc810\uc5d0 \uc784\uacc4 \uc9c8\ub7c9\uc744 \uc720\uc9c0\ud558\ub294 \uac83\uc774 \ubd88\uac00\ub2a5\ud558\uc600\uae30 \ub54c\ubb38\uc774\ub2e4."} {"question": "\uc0dd\ud0dc\ud559\uc774\ub780 \uc6a9\uc5b4\ub294 \ub204\uac00 \uc0ac\uc6a9\ud558\uba74\uc11c \ub110\ub9ac \uc54c\ub824\uc9c0\uac8c \ub418\uc5c8\ub294\uac00?", "paragraph": "\uc0dd\ud0dc\ud559\uc758 \uae30\uc6d0\uc740 \uc77c\ubc18\uc801\uc73c\ub85c \ub2e4\uc708\uc758 \uc9c4\ud654\ub860\uacfc 19\uc138\uae30\uc5d0\uc11c 20\uc138\uae30 \ucd08 \uc0ac\uc774\uc758 \uadfc\ub300 \uc0dd\ubb3c\uc9c0\ub9ac\ud559\uc774 \uc5b8\uae09\ub41c\ub2e4. \uadf8\ub7ec\ub098 \uc0dd\ud0dc\ud559\uc758 \ub300\ub450 \ub9cc\ud07c\uc774\ub098 \ud30c\uc2a4\ud1f4\ub974, \ud398\ub974\ub514\ub09c\ud2b8 \ucf58(Ferdinand Cohn)\uc774 \uc81c\uc2dc\ud55c \ubbf8\uc0dd\ubb3c\ud559, \ud1a0\uc591\ud559\uc5d0 \uae30\ucd08\ud55c \uc0dd\ubb3c\uc9c0\uad6c\ud654\ud559\uc801 \uc21c\ud658\uc758 \uc911\uc694\uc131\ub3c4 \uac15\uc870\ub418\uace0 \uc788\ub2e4. \uc0dd\ud0dc\ud559\uc774\ub77c\ub294 \uc6a9\uc5b4\ub294 \uc5d0\ub978\uc2a4\ud2b8 \ud5e4\ucf08\uc774 \uc0ac\uc6a9\ud558\uba74\uc11c \ub110\ub9ac \uc54c\ub824\uc9c0\uac8c \ub418\uc5c8\ub294\ub370, \ud5e4\ucf08\uc740 \uc790\uc5f0\uc758 \uc77c\ubc18\uc801 \ud604\uc0c1\uc5d0 \ub300\ud55c \uac70\uc2dc\uc801 \uc870\ub9dd\uc774 \uc0dd\ud0dc\ud559\uc801 \uc0ac\uace0\ub97c \ud655\uc0b0\uc2dc\ud0a4\ub294\ub370 \uc911\uc694\ud558\ub2e4\uace0 \uc8fc\uc7a5\ud558\uc600\ub2e4. 1930\ub144\ub300\uc5d0\ub294 \uc544\uc11c \ud0e0\uc990\ub9ac(Arthur Tansley)\uc640 \uadf8\uc758 \uc870\uad50\uac00 \ud798\uc5d0 \ub300\ud55c \ubb3c\ub9ac\uc801 \uac1c\ub150\uacfc \uc790\uc5f0\uc0ac\uc801 \uae30\ubc95\uc744 \uacb0\ud569\ud558\uc5ec \uc2e4\ud5d8\uc801 \ud1a0\uc591\ud559\uc744 \uad6c\uc131\ud558\ub294 \u2018\uc0dd\ud0dc\uacc4 \uc0dd\ud0dc\ud559(Ecosystem ecology)\u2019\uc5f0\uad6c\ub97c \uc2dc\uc791\ud558\uc600\ub2e4. 20\uc138\uae30\uc758 \uc0dd\ud0dc\ud559\uc740 1960\ub144\ub300\uc5d0 \ub4f1\uc7a5\ud558\uc5ec 1970\ub144\ub300\uc5d0 \ub110\ub9ac \ud37c\uc9c4 \uac00\uc774\uc544 \uc774\ub860\uc744 \uace8\uc790\ub85c \ud558\ub294 \uc0dd\ud0dc\uc8fc\uc758\uc5d0 \uae30\ubc18\ud558\uace0 \uc788\uc5c8\ub2e4. \ucd5c\uadfc\uc758 \uc0dd\ud0dc\ud559\uc740 \uc0dd\ud0dc\uc8fc\uc758\uc5d0\uc11c \u2018\ub514\ud504\uc5d0\ucf5c\ub85c\uc9c0(Deep-Ecology) \uc6b4\ub3d9\u2019\uc73c\ub85c \uc120\ud68c\ud558\uace0 \uc788\ub2e4.", "answer": "\uc5d0\ub978\uc2a4\ud2b8 \ud5e4\ucf08", "sentence": "\uc0dd\ud0dc\ud559\uc774\ub77c\ub294 \uc6a9\uc5b4\ub294 \uc5d0\ub978\uc2a4\ud2b8 \ud5e4\ucf08 \uc774 \uc0ac\uc6a9\ud558\uba74\uc11c \ub110\ub9ac \uc54c\ub824\uc9c0\uac8c \ub418\uc5c8\ub294\ub370, \ud5e4\ucf08\uc740 \uc790\uc5f0\uc758 \uc77c\ubc18\uc801 \ud604\uc0c1\uc5d0 \ub300\ud55c \uac70\uc2dc\uc801 \uc870\ub9dd\uc774 \uc0dd\ud0dc\ud559\uc801 \uc0ac\uace0\ub97c \ud655\uc0b0\uc2dc\ud0a4\ub294\ub370 \uc911\uc694\ud558\ub2e4\uace0 \uc8fc\uc7a5\ud558\uc600\ub2e4.", "paragraph_sentence": "\uc0dd\ud0dc\ud559\uc758 \uae30\uc6d0\uc740 \uc77c\ubc18\uc801\uc73c\ub85c \ub2e4\uc708\uc758 \uc9c4\ud654\ub860\uacfc 19\uc138\uae30\uc5d0\uc11c 20\uc138\uae30 \ucd08 \uc0ac\uc774\uc758 \uadfc\ub300 \uc0dd\ubb3c\uc9c0\ub9ac\ud559\uc774 \uc5b8\uae09\ub41c\ub2e4. \uadf8\ub7ec\ub098 \uc0dd\ud0dc\ud559\uc758 \ub300\ub450 \ub9cc\ud07c\uc774\ub098 \ud30c\uc2a4\ud1f4\ub974, \ud398\ub974\ub514\ub09c\ud2b8 \ucf58(Ferdinand Cohn)\uc774 \uc81c\uc2dc\ud55c \ubbf8\uc0dd\ubb3c\ud559, \ud1a0\uc591\ud559\uc5d0 \uae30\ucd08\ud55c \uc0dd\ubb3c\uc9c0\uad6c\ud654\ud559\uc801 \uc21c\ud658\uc758 \uc911\uc694\uc131\ub3c4 \uac15\uc870\ub418\uace0 \uc788\ub2e4. \uc0dd\ud0dc\ud559\uc774\ub77c\ub294 \uc6a9\uc5b4\ub294 \uc5d0\ub978\uc2a4\ud2b8 \ud5e4\ucf08 \uc774 \uc0ac\uc6a9\ud558\uba74\uc11c \ub110\ub9ac \uc54c\ub824\uc9c0\uac8c \ub418\uc5c8\ub294\ub370, \ud5e4\ucf08\uc740 \uc790\uc5f0\uc758 \uc77c\ubc18\uc801 \ud604\uc0c1\uc5d0 \ub300\ud55c \uac70\uc2dc\uc801 \uc870\ub9dd\uc774 \uc0dd\ud0dc\ud559\uc801 \uc0ac\uace0\ub97c \ud655\uc0b0\uc2dc\ud0a4\ub294\ub370 \uc911\uc694\ud558\ub2e4\uace0 \uc8fc\uc7a5\ud558\uc600\ub2e4. 1930\ub144\ub300\uc5d0\ub294 \uc544\uc11c \ud0e0\uc990\ub9ac(Arthur Tansley)\uc640 \uadf8\uc758 \uc870\uad50\uac00 \ud798\uc5d0 \ub300\ud55c \ubb3c\ub9ac\uc801 \uac1c\ub150\uacfc \uc790\uc5f0\uc0ac\uc801 \uae30\ubc95\uc744 \uacb0\ud569\ud558\uc5ec \uc2e4\ud5d8\uc801 \ud1a0\uc591\ud559\uc744 \uad6c\uc131\ud558\ub294 \u2018\uc0dd\ud0dc\uacc4 \uc0dd\ud0dc\ud559(Ecosystem ecology)\u2019\uc5f0\uad6c\ub97c \uc2dc\uc791\ud558\uc600\ub2e4. 20\uc138\uae30\uc758 \uc0dd\ud0dc\ud559\uc740 1960\ub144\ub300\uc5d0 \ub4f1\uc7a5\ud558\uc5ec 1970\ub144\ub300\uc5d0 \ub110\ub9ac \ud37c\uc9c4 \uac00\uc774\uc544 \uc774\ub860\uc744 \uace8\uc790\ub85c \ud558\ub294 \uc0dd\ud0dc\uc8fc\uc758\uc5d0 \uae30\ubc18\ud558\uace0 \uc788\uc5c8\ub2e4. \ucd5c\uadfc\uc758 \uc0dd\ud0dc\ud559\uc740 \uc0dd\ud0dc\uc8fc\uc758\uc5d0\uc11c \u2018\ub514\ud504\uc5d0\ucf5c\ub85c\uc9c0(Deep-Ecology) \uc6b4\ub3d9\u2019\uc73c\ub85c \uc120\ud68c\ud558\uace0 \uc788\ub2e4.", "paragraph_answer": "\uc0dd\ud0dc\ud559\uc758 \uae30\uc6d0\uc740 \uc77c\ubc18\uc801\uc73c\ub85c \ub2e4\uc708\uc758 \uc9c4\ud654\ub860\uacfc 19\uc138\uae30\uc5d0\uc11c 20\uc138\uae30 \ucd08 \uc0ac\uc774\uc758 \uadfc\ub300 \uc0dd\ubb3c\uc9c0\ub9ac\ud559\uc774 \uc5b8\uae09\ub41c\ub2e4. \uadf8\ub7ec\ub098 \uc0dd\ud0dc\ud559\uc758 \ub300\ub450 \ub9cc\ud07c\uc774\ub098 \ud30c\uc2a4\ud1f4\ub974, \ud398\ub974\ub514\ub09c\ud2b8 \ucf58(Ferdinand Cohn)\uc774 \uc81c\uc2dc\ud55c \ubbf8\uc0dd\ubb3c\ud559, \ud1a0\uc591\ud559\uc5d0 \uae30\ucd08\ud55c \uc0dd\ubb3c\uc9c0\uad6c\ud654\ud559\uc801 \uc21c\ud658\uc758 \uc911\uc694\uc131\ub3c4 \uac15\uc870\ub418\uace0 \uc788\ub2e4. \uc0dd\ud0dc\ud559\uc774\ub77c\ub294 \uc6a9\uc5b4\ub294 \uc5d0\ub978\uc2a4\ud2b8 \ud5e4\ucf08 \uc774 \uc0ac\uc6a9\ud558\uba74\uc11c \ub110\ub9ac \uc54c\ub824\uc9c0\uac8c \ub418\uc5c8\ub294\ub370, \ud5e4\ucf08\uc740 \uc790\uc5f0\uc758 \uc77c\ubc18\uc801 \ud604\uc0c1\uc5d0 \ub300\ud55c \uac70\uc2dc\uc801 \uc870\ub9dd\uc774 \uc0dd\ud0dc\ud559\uc801 \uc0ac\uace0\ub97c \ud655\uc0b0\uc2dc\ud0a4\ub294\ub370 \uc911\uc694\ud558\ub2e4\uace0 \uc8fc\uc7a5\ud558\uc600\ub2e4. 1930\ub144\ub300\uc5d0\ub294 \uc544\uc11c \ud0e0\uc990\ub9ac(Arthur Tansley)\uc640 \uadf8\uc758 \uc870\uad50\uac00 \ud798\uc5d0 \ub300\ud55c \ubb3c\ub9ac\uc801 \uac1c\ub150\uacfc \uc790\uc5f0\uc0ac\uc801 \uae30\ubc95\uc744 \uacb0\ud569\ud558\uc5ec \uc2e4\ud5d8\uc801 \ud1a0\uc591\ud559\uc744 \uad6c\uc131\ud558\ub294 \u2018\uc0dd\ud0dc\uacc4 \uc0dd\ud0dc\ud559(Ecosystem ecology)\u2019\uc5f0\uad6c\ub97c \uc2dc\uc791\ud558\uc600\ub2e4. 20\uc138\uae30\uc758 \uc0dd\ud0dc\ud559\uc740 1960\ub144\ub300\uc5d0 \ub4f1\uc7a5\ud558\uc5ec 1970\ub144\ub300\uc5d0 \ub110\ub9ac \ud37c\uc9c4 \uac00\uc774\uc544 \uc774\ub860\uc744 \uace8\uc790\ub85c \ud558\ub294 \uc0dd\ud0dc\uc8fc\uc758\uc5d0 \uae30\ubc18\ud558\uace0 \uc788\uc5c8\ub2e4. \ucd5c\uadfc\uc758 \uc0dd\ud0dc\ud559\uc740 \uc0dd\ud0dc\uc8fc\uc758\uc5d0\uc11c \u2018\ub514\ud504\uc5d0\ucf5c\ub85c\uc9c0(Deep-Ecology) \uc6b4\ub3d9\u2019\uc73c\ub85c \uc120\ud68c\ud558\uace0 \uc788\ub2e4.", "sentence_answer": "\uc0dd\ud0dc\ud559\uc774\ub77c\ub294 \uc6a9\uc5b4\ub294 \uc5d0\ub978\uc2a4\ud2b8 \ud5e4\ucf08 \uc774 \uc0ac\uc6a9\ud558\uba74\uc11c \ub110\ub9ac \uc54c\ub824\uc9c0\uac8c \ub418\uc5c8\ub294\ub370, \ud5e4\ucf08\uc740 \uc790\uc5f0\uc758 \uc77c\ubc18\uc801 \ud604\uc0c1\uc5d0 \ub300\ud55c \uac70\uc2dc\uc801 \uc870\ub9dd\uc774 \uc0dd\ud0dc\ud559\uc801 \uc0ac\uace0\ub97c \ud655\uc0b0\uc2dc\ud0a4\ub294\ub370 \uc911\uc694\ud558\ub2e4\uace0 \uc8fc\uc7a5\ud558\uc600\ub2e4.", "paragraph_id": "6489929-27-0", "paragraph_question": "question: \uc0dd\ud0dc\ud559\uc774\ub780 \uc6a9\uc5b4\ub294 \ub204\uac00 \uc0ac\uc6a9\ud558\uba74\uc11c \ub110\ub9ac \uc54c\ub824\uc9c0\uac8c \ub418\uc5c8\ub294\uac00?, context: \uc0dd\ud0dc\ud559\uc758 \uae30\uc6d0\uc740 \uc77c\ubc18\uc801\uc73c\ub85c \ub2e4\uc708\uc758 \uc9c4\ud654\ub860\uacfc 19\uc138\uae30\uc5d0\uc11c 20\uc138\uae30 \ucd08 \uc0ac\uc774\uc758 \uadfc\ub300 \uc0dd\ubb3c\uc9c0\ub9ac\ud559\uc774 \uc5b8\uae09\ub41c\ub2e4. \uadf8\ub7ec\ub098 \uc0dd\ud0dc\ud559\uc758 \ub300\ub450 \ub9cc\ud07c\uc774\ub098 \ud30c\uc2a4\ud1f4\ub974, \ud398\ub974\ub514\ub09c\ud2b8 \ucf58(Ferdinand Cohn)\uc774 \uc81c\uc2dc\ud55c \ubbf8\uc0dd\ubb3c\ud559, \ud1a0\uc591\ud559\uc5d0 \uae30\ucd08\ud55c \uc0dd\ubb3c\uc9c0\uad6c\ud654\ud559\uc801 \uc21c\ud658\uc758 \uc911\uc694\uc131\ub3c4 \uac15\uc870\ub418\uace0 \uc788\ub2e4. \uc0dd\ud0dc\ud559\uc774\ub77c\ub294 \uc6a9\uc5b4\ub294 \uc5d0\ub978\uc2a4\ud2b8 \ud5e4\ucf08\uc774 \uc0ac\uc6a9\ud558\uba74\uc11c \ub110\ub9ac \uc54c\ub824\uc9c0\uac8c \ub418\uc5c8\ub294\ub370, \ud5e4\ucf08\uc740 \uc790\uc5f0\uc758 \uc77c\ubc18\uc801 \ud604\uc0c1\uc5d0 \ub300\ud55c \uac70\uc2dc\uc801 \uc870\ub9dd\uc774 \uc0dd\ud0dc\ud559\uc801 \uc0ac\uace0\ub97c \ud655\uc0b0\uc2dc\ud0a4\ub294\ub370 \uc911\uc694\ud558\ub2e4\uace0 \uc8fc\uc7a5\ud558\uc600\ub2e4. 1930\ub144\ub300\uc5d0\ub294 \uc544\uc11c \ud0e0\uc990\ub9ac(Arthur Tansley)\uc640 \uadf8\uc758 \uc870\uad50\uac00 \ud798\uc5d0 \ub300\ud55c \ubb3c\ub9ac\uc801 \uac1c\ub150\uacfc \uc790\uc5f0\uc0ac\uc801 \uae30\ubc95\uc744 \uacb0\ud569\ud558\uc5ec \uc2e4\ud5d8\uc801 \ud1a0\uc591\ud559\uc744 \uad6c\uc131\ud558\ub294 \u2018\uc0dd\ud0dc\uacc4 \uc0dd\ud0dc\ud559(Ecosystem ecology)\u2019\uc5f0\uad6c\ub97c \uc2dc\uc791\ud558\uc600\ub2e4. 20\uc138\uae30\uc758 \uc0dd\ud0dc\ud559\uc740 1960\ub144\ub300\uc5d0 \ub4f1\uc7a5\ud558\uc5ec 1970\ub144\ub300\uc5d0 \ub110\ub9ac \ud37c\uc9c4 \uac00\uc774\uc544 \uc774\ub860\uc744 \uace8\uc790\ub85c \ud558\ub294 \uc0dd\ud0dc\uc8fc\uc758\uc5d0 \uae30\ubc18\ud558\uace0 \uc788\uc5c8\ub2e4. \ucd5c\uadfc\uc758 \uc0dd\ud0dc\ud559\uc740 \uc0dd\ud0dc\uc8fc\uc758\uc5d0\uc11c \u2018\ub514\ud504\uc5d0\ucf5c\ub85c\uc9c0(Deep-Ecology) \uc6b4\ub3d9\u2019\uc73c\ub85c \uc120\ud68c\ud558\uace0 \uc788\ub2e4."} {"question": "\uc5d0\ub77c\uc2a4\ubb34\uc2a4\uc758 \uace0\ud5a5\uc740 \uc5b4\ub514\uc778\uac00?", "paragraph": "\uc5d0\ub77c\uc2a4\ubb34\uc2a4\uc758 \ub124\ub35c\ub780\ub4dc \uc774\ub984\uc740 \ud5e4\ub9ac\ud2b8 \ud5e4\ub9ac\ucd0c(Gerrit Gerritszoon)\uc774\uc5c8\ub2e4. 1466\ub144 \ub610\ub294 1469\ub144 10\uc6d4 27\uc77c \ub85c\ud14c\ub974\ub2f4\uc5d0\uc11c \ud0dc\uc5b4\ub0ac\ub2e4. \uc815\ud655\ud55c \uc0dd\ub144\uc740 \uc54c\ub824\uc9c0\uc9c0 \uc54a\uc558\ub2e4. \ub124\ub35c\ub780\ub4dc\uc758 \uc5ed\uc0ac\uac00 \uc694\ud55c \ud638\uc774\uc9d5\ud558(Johan Huizinga)\uc5d0 \ub530\ub974\uba74 \uadf8\ub294 27\uc77c \ub610\ub294 28\uc77c\uc758 \ubc24\uc5d0 \ud0dc\uc5b4\ub0ac\uace0 10\uc6d4 28\uc77c\uc774 \uc0dd\uc77c\ub85c \uae30\ub150\ub410\ub2e4. \"\uc5d0\ub77c\uc2a4\ubb34\uc2a4\"\ub77c\ub294 \uc774\ub984\uc740 \uc131\uc778\uc758 \uc774\ub984\uc744 \ub530\uc11c \uc138\ub840\uba85\uc73c\ub85c \ubc1b\uc558\uc744 \uac00\ub2a5\uc131\uc774 \uc788\ub2e4. \ub85c\ud14c\ub974\ub2f4\uc5d0\uc11c \ud0dc\uc5b4\ub0ac\uc9c0\ub9cc \uadf8\ub294 4\ub144 \ub3d9\uc548\ub9cc \uadf8\uacf3\uc5d0\uc11c \uc0b4\uc558\uace0 \ub85c\ud14c\ub974\ub2f4\uc744 \ub5a0\ub09c \ud6c4\uc5d0 \ub2e4\uc2dc \ub3cc\uc544\uc624\uc9c0 \uc54a\uc558\ub2e4. \uadf8\uc758 \uac00\uc871\uacfc \uc5b4\ub9b0 \uc2dc\uc808\uc5d0 \ub300\ud55c \uc815\ubcf4\ub294 \uadf8\uc758 \uc800\uc791\uc5d0\uc11c \ucc3e\uc744 \uc218 \uc788\uc9c0\ub9cc \ubaa8\ud638\ud55c \uac83\uc774 \ub9ce\ub2e4. \uadf8\uac00 \uc0ac\uc0dd\uc544\uc600\ub2e4\ub294 \uac83\uc740 \uac70\uc758 \ud655\uc2e4\ud558\ub2e4. \uadf8\uc758 \uc544\ubc84\uc9c0\ub294 \ub098\uc911\uc5d0 \ub85c\ud5e4 \ud5e4\ub77c\ud2b8(Roger Gerard)\ub77c\uace0 \ubd88\ub9ac\ub294 \ub85c\ub9c8 \uac00\ud1a8\ub9ad \uc2e0\ubd80\uac00 \ub410\ub2e4. \uc5b4\uba38\ub2c8\uc5d0 \ub300\ud574\uc11c\ub294 \uc774\ub984\uc774 \ub9c8\ud558\ub808\ud2b8(Margaret)\ub77c\ub294 \uac83\uacfc \uc678\uacfc\uc758\uc0ac\uc758 \ub538\uc774\uc5c8\ub2e4\ub294 \uac83 \ubc16\uc5d0\ub294 \uc54c\ub824\uc9c4 \uac83\uc774 \uc5c6\ub2e4. \uadf8\ub294 \uc0ac\uc0dd\uc544\uc600\uc9c0\ub9cc \ubd80\ubaa8\ub85c\ubd80\ud130 \ubc84\ub824\uc9c4 \uac83\uc740 \uc544\ub2c8\uc5c8\ub2e4. \uadf8\uc758 \ubd80\ubaa8\ub294 1483\ub144\uc5d0 \ud751\uc0ac\ubcd1\uc73c\ub85c \uc77c\ucc0d \uc8fd\uc744 \ub54c\uae4c\uc9c0 \uadf8\ub97c \ud0a4\uc6e0\ub2e4. \uc774 \uc2dc\uc808\uc5d0 \uadf8\ub294 \uc5ec\ub7ec \uc218\ub3c4\uc0ac \ud559\uad50\uc5d0\uc11c \uc218\ud559\ud588\ub2e4. \ud2b9\ud788 \uacf5\ub3d9\uc0dd\ud65c \ud615\uc81c\ud68c(Brethren of the Common Life)\uac00 \uc6b4\uc601\ud55c \ud559\uad50\uc5d0\uc11c \uadf8\ub294 \u300a\uadf8\ub9ac\uc2a4\ub3c4\uc758 \ubaa8\ubc29\u300b(The Imitation of Christ)\ub77c\ub294 \ucc45\uc744 \ud1b5\ud574 \uc2e0\uacfc\uc758 \uac1c\uc778\uc801 \ub9cc\ub0a8\uc758 \uc911\uc694\uc131\uc5d0 \ub300\ud574 \uc5f0\uad6c\ud558\uace0, \uc218\ub3c4\uc6d0\uacfc \ud559\uad50\uc758 \uc5c4\uaca9\ud55c \uaddc\uc728\uc744 \ud53c\ud588\ub2e4.", "answer": "\ub85c\ud14c\ub974\ub2f4", "sentence": "1466\ub144 \ub610\ub294 1469\ub144 10\uc6d4 27\uc77c \ub85c\ud14c\ub974\ub2f4 \uc5d0\uc11c \ud0dc\uc5b4\ub0ac\ub2e4.", "paragraph_sentence": "\uc5d0\ub77c\uc2a4\ubb34\uc2a4\uc758 \ub124\ub35c\ub780\ub4dc \uc774\ub984\uc740 \ud5e4\ub9ac\ud2b8 \ud5e4\ub9ac\ucd0c(Gerrit Gerritszoon)\uc774\uc5c8\ub2e4. 1466\ub144 \ub610\ub294 1469\ub144 10\uc6d4 27\uc77c \ub85c\ud14c\ub974\ub2f4 \uc5d0\uc11c \ud0dc\uc5b4\ub0ac\ub2e4. \uc815\ud655\ud55c \uc0dd\ub144\uc740 \uc54c\ub824\uc9c0\uc9c0 \uc54a\uc558\ub2e4. \ub124\ub35c\ub780\ub4dc\uc758 \uc5ed\uc0ac\uac00 \uc694\ud55c \ud638\uc774\uc9d5\ud558(Johan Huizinga)\uc5d0 \ub530\ub974\uba74 \uadf8\ub294 27\uc77c \ub610\ub294 28\uc77c\uc758 \ubc24\uc5d0 \ud0dc\uc5b4\ub0ac\uace0 10\uc6d4 28\uc77c\uc774 \uc0dd\uc77c\ub85c \uae30\ub150\ub410\ub2e4. \"\uc5d0\ub77c\uc2a4\ubb34\uc2a4\"\ub77c\ub294 \uc774\ub984\uc740 \uc131\uc778\uc758 \uc774\ub984\uc744 \ub530\uc11c \uc138\ub840\uba85\uc73c\ub85c \ubc1b\uc558\uc744 \uac00\ub2a5\uc131\uc774 \uc788\ub2e4. \ub85c\ud14c\ub974\ub2f4\uc5d0\uc11c \ud0dc\uc5b4\ub0ac\uc9c0\ub9cc \uadf8\ub294 4\ub144 \ub3d9\uc548\ub9cc \uadf8\uacf3\uc5d0\uc11c \uc0b4\uc558\uace0 \ub85c\ud14c\ub974\ub2f4\uc744 \ub5a0\ub09c \ud6c4\uc5d0 \ub2e4\uc2dc \ub3cc\uc544\uc624\uc9c0 \uc54a\uc558\ub2e4. \uadf8\uc758 \uac00\uc871\uacfc \uc5b4\ub9b0 \uc2dc\uc808\uc5d0 \ub300\ud55c \uc815\ubcf4\ub294 \uadf8\uc758 \uc800\uc791\uc5d0\uc11c \ucc3e\uc744 \uc218 \uc788\uc9c0\ub9cc \ubaa8\ud638\ud55c \uac83\uc774 \ub9ce\ub2e4. \uadf8\uac00 \uc0ac\uc0dd\uc544\uc600\ub2e4\ub294 \uac83\uc740 \uac70\uc758 \ud655\uc2e4\ud558\ub2e4. \uadf8\uc758 \uc544\ubc84\uc9c0\ub294 \ub098\uc911\uc5d0 \ub85c\ud5e4 \ud5e4\ub77c\ud2b8(Roger Gerard)\ub77c\uace0 \ubd88\ub9ac\ub294 \ub85c\ub9c8 \uac00\ud1a8\ub9ad \uc2e0\ubd80\uac00 \ub410\ub2e4. \uc5b4\uba38\ub2c8\uc5d0 \ub300\ud574\uc11c\ub294 \uc774\ub984\uc774 \ub9c8\ud558\ub808\ud2b8(Margaret)\ub77c\ub294 \uac83\uacfc \uc678\uacfc\uc758\uc0ac\uc758 \ub538\uc774\uc5c8\ub2e4\ub294 \uac83 \ubc16\uc5d0\ub294 \uc54c\ub824\uc9c4 \uac83\uc774 \uc5c6\ub2e4. \uadf8\ub294 \uc0ac\uc0dd\uc544\uc600\uc9c0\ub9cc \ubd80\ubaa8\ub85c\ubd80\ud130 \ubc84\ub824\uc9c4 \uac83\uc740 \uc544\ub2c8\uc5c8\ub2e4. \uadf8\uc758 \ubd80\ubaa8\ub294 1483\ub144\uc5d0 \ud751\uc0ac\ubcd1\uc73c\ub85c \uc77c\ucc0d \uc8fd\uc744 \ub54c\uae4c\uc9c0 \uadf8\ub97c \ud0a4\uc6e0\ub2e4. \uc774 \uc2dc\uc808\uc5d0 \uadf8\ub294 \uc5ec\ub7ec \uc218\ub3c4\uc0ac \ud559\uad50\uc5d0\uc11c \uc218\ud559\ud588\ub2e4. \ud2b9\ud788 \uacf5\ub3d9\uc0dd\ud65c \ud615\uc81c\ud68c(Brethren of the Common Life)\uac00 \uc6b4\uc601\ud55c \ud559\uad50\uc5d0\uc11c \uadf8\ub294 \u300a\uadf8\ub9ac\uc2a4\ub3c4\uc758 \ubaa8\ubc29\u300b(The Imitation of Christ)\ub77c\ub294 \ucc45\uc744 \ud1b5\ud574 \uc2e0\uacfc\uc758 \uac1c\uc778\uc801 \ub9cc\ub0a8\uc758 \uc911\uc694\uc131\uc5d0 \ub300\ud574 \uc5f0\uad6c\ud558\uace0, \uc218\ub3c4\uc6d0\uacfc \ud559\uad50\uc758 \uc5c4\uaca9\ud55c \uaddc\uc728\uc744 \ud53c\ud588\ub2e4.", "paragraph_answer": "\uc5d0\ub77c\uc2a4\ubb34\uc2a4\uc758 \ub124\ub35c\ub780\ub4dc \uc774\ub984\uc740 \ud5e4\ub9ac\ud2b8 \ud5e4\ub9ac\ucd0c(Gerrit Gerritszoon)\uc774\uc5c8\ub2e4. 1466\ub144 \ub610\ub294 1469\ub144 10\uc6d4 27\uc77c \ub85c\ud14c\ub974\ub2f4 \uc5d0\uc11c \ud0dc\uc5b4\ub0ac\ub2e4. \uc815\ud655\ud55c \uc0dd\ub144\uc740 \uc54c\ub824\uc9c0\uc9c0 \uc54a\uc558\ub2e4. \ub124\ub35c\ub780\ub4dc\uc758 \uc5ed\uc0ac\uac00 \uc694\ud55c \ud638\uc774\uc9d5\ud558(Johan Huizinga)\uc5d0 \ub530\ub974\uba74 \uadf8\ub294 27\uc77c \ub610\ub294 28\uc77c\uc758 \ubc24\uc5d0 \ud0dc\uc5b4\ub0ac\uace0 10\uc6d4 28\uc77c\uc774 \uc0dd\uc77c\ub85c \uae30\ub150\ub410\ub2e4. \"\uc5d0\ub77c\uc2a4\ubb34\uc2a4\"\ub77c\ub294 \uc774\ub984\uc740 \uc131\uc778\uc758 \uc774\ub984\uc744 \ub530\uc11c \uc138\ub840\uba85\uc73c\ub85c \ubc1b\uc558\uc744 \uac00\ub2a5\uc131\uc774 \uc788\ub2e4. \ub85c\ud14c\ub974\ub2f4\uc5d0\uc11c \ud0dc\uc5b4\ub0ac\uc9c0\ub9cc \uadf8\ub294 4\ub144 \ub3d9\uc548\ub9cc \uadf8\uacf3\uc5d0\uc11c \uc0b4\uc558\uace0 \ub85c\ud14c\ub974\ub2f4\uc744 \ub5a0\ub09c \ud6c4\uc5d0 \ub2e4\uc2dc \ub3cc\uc544\uc624\uc9c0 \uc54a\uc558\ub2e4. \uadf8\uc758 \uac00\uc871\uacfc \uc5b4\ub9b0 \uc2dc\uc808\uc5d0 \ub300\ud55c \uc815\ubcf4\ub294 \uadf8\uc758 \uc800\uc791\uc5d0\uc11c \ucc3e\uc744 \uc218 \uc788\uc9c0\ub9cc \ubaa8\ud638\ud55c \uac83\uc774 \ub9ce\ub2e4. \uadf8\uac00 \uc0ac\uc0dd\uc544\uc600\ub2e4\ub294 \uac83\uc740 \uac70\uc758 \ud655\uc2e4\ud558\ub2e4. \uadf8\uc758 \uc544\ubc84\uc9c0\ub294 \ub098\uc911\uc5d0 \ub85c\ud5e4 \ud5e4\ub77c\ud2b8(Roger Gerard)\ub77c\uace0 \ubd88\ub9ac\ub294 \ub85c\ub9c8 \uac00\ud1a8\ub9ad \uc2e0\ubd80\uac00 \ub410\ub2e4. \uc5b4\uba38\ub2c8\uc5d0 \ub300\ud574\uc11c\ub294 \uc774\ub984\uc774 \ub9c8\ud558\ub808\ud2b8(Margaret)\ub77c\ub294 \uac83\uacfc \uc678\uacfc\uc758\uc0ac\uc758 \ub538\uc774\uc5c8\ub2e4\ub294 \uac83 \ubc16\uc5d0\ub294 \uc54c\ub824\uc9c4 \uac83\uc774 \uc5c6\ub2e4. \uadf8\ub294 \uc0ac\uc0dd\uc544\uc600\uc9c0\ub9cc \ubd80\ubaa8\ub85c\ubd80\ud130 \ubc84\ub824\uc9c4 \uac83\uc740 \uc544\ub2c8\uc5c8\ub2e4. \uadf8\uc758 \ubd80\ubaa8\ub294 1483\ub144\uc5d0 \ud751\uc0ac\ubcd1\uc73c\ub85c \uc77c\ucc0d \uc8fd\uc744 \ub54c\uae4c\uc9c0 \uadf8\ub97c \ud0a4\uc6e0\ub2e4. \uc774 \uc2dc\uc808\uc5d0 \uadf8\ub294 \uc5ec\ub7ec \uc218\ub3c4\uc0ac \ud559\uad50\uc5d0\uc11c \uc218\ud559\ud588\ub2e4. \ud2b9\ud788 \uacf5\ub3d9\uc0dd\ud65c \ud615\uc81c\ud68c(Brethren of the Common Life)\uac00 \uc6b4\uc601\ud55c \ud559\uad50\uc5d0\uc11c \uadf8\ub294 \u300a\uadf8\ub9ac\uc2a4\ub3c4\uc758 \ubaa8\ubc29\u300b(The Imitation of Christ)\ub77c\ub294 \ucc45\uc744 \ud1b5\ud574 \uc2e0\uacfc\uc758 \uac1c\uc778\uc801 \ub9cc\ub0a8\uc758 \uc911\uc694\uc131\uc5d0 \ub300\ud574 \uc5f0\uad6c\ud558\uace0, \uc218\ub3c4\uc6d0\uacfc \ud559\uad50\uc758 \uc5c4\uaca9\ud55c \uaddc\uc728\uc744 \ud53c\ud588\ub2e4.", "sentence_answer": "1466\ub144 \ub610\ub294 1469\ub144 10\uc6d4 27\uc77c \ub85c\ud14c\ub974\ub2f4 \uc5d0\uc11c \ud0dc\uc5b4\ub0ac\ub2e4.", "paragraph_id": "6533997-1-1", "paragraph_question": "question: \uc5d0\ub77c\uc2a4\ubb34\uc2a4\uc758 \uace0\ud5a5\uc740 \uc5b4\ub514\uc778\uac00?, context: \uc5d0\ub77c\uc2a4\ubb34\uc2a4\uc758 \ub124\ub35c\ub780\ub4dc \uc774\ub984\uc740 \ud5e4\ub9ac\ud2b8 \ud5e4\ub9ac\ucd0c(Gerrit Gerritszoon)\uc774\uc5c8\ub2e4. 1466\ub144 \ub610\ub294 1469\ub144 10\uc6d4 27\uc77c \ub85c\ud14c\ub974\ub2f4\uc5d0\uc11c \ud0dc\uc5b4\ub0ac\ub2e4. \uc815\ud655\ud55c \uc0dd\ub144\uc740 \uc54c\ub824\uc9c0\uc9c0 \uc54a\uc558\ub2e4. \ub124\ub35c\ub780\ub4dc\uc758 \uc5ed\uc0ac\uac00 \uc694\ud55c \ud638\uc774\uc9d5\ud558(Johan Huizinga)\uc5d0 \ub530\ub974\uba74 \uadf8\ub294 27\uc77c \ub610\ub294 28\uc77c\uc758 \ubc24\uc5d0 \ud0dc\uc5b4\ub0ac\uace0 10\uc6d4 28\uc77c\uc774 \uc0dd\uc77c\ub85c \uae30\ub150\ub410\ub2e4. \"\uc5d0\ub77c\uc2a4\ubb34\uc2a4\"\ub77c\ub294 \uc774\ub984\uc740 \uc131\uc778\uc758 \uc774\ub984\uc744 \ub530\uc11c \uc138\ub840\uba85\uc73c\ub85c \ubc1b\uc558\uc744 \uac00\ub2a5\uc131\uc774 \uc788\ub2e4. \ub85c\ud14c\ub974\ub2f4\uc5d0\uc11c \ud0dc\uc5b4\ub0ac\uc9c0\ub9cc \uadf8\ub294 4\ub144 \ub3d9\uc548\ub9cc \uadf8\uacf3\uc5d0\uc11c \uc0b4\uc558\uace0 \ub85c\ud14c\ub974\ub2f4\uc744 \ub5a0\ub09c \ud6c4\uc5d0 \ub2e4\uc2dc \ub3cc\uc544\uc624\uc9c0 \uc54a\uc558\ub2e4. \uadf8\uc758 \uac00\uc871\uacfc \uc5b4\ub9b0 \uc2dc\uc808\uc5d0 \ub300\ud55c \uc815\ubcf4\ub294 \uadf8\uc758 \uc800\uc791\uc5d0\uc11c \ucc3e\uc744 \uc218 \uc788\uc9c0\ub9cc \ubaa8\ud638\ud55c \uac83\uc774 \ub9ce\ub2e4. \uadf8\uac00 \uc0ac\uc0dd\uc544\uc600\ub2e4\ub294 \uac83\uc740 \uac70\uc758 \ud655\uc2e4\ud558\ub2e4. \uadf8\uc758 \uc544\ubc84\uc9c0\ub294 \ub098\uc911\uc5d0 \ub85c\ud5e4 \ud5e4\ub77c\ud2b8(Roger Gerard)\ub77c\uace0 \ubd88\ub9ac\ub294 \ub85c\ub9c8 \uac00\ud1a8\ub9ad \uc2e0\ubd80\uac00 \ub410\ub2e4. \uc5b4\uba38\ub2c8\uc5d0 \ub300\ud574\uc11c\ub294 \uc774\ub984\uc774 \ub9c8\ud558\ub808\ud2b8(Margaret)\ub77c\ub294 \uac83\uacfc \uc678\uacfc\uc758\uc0ac\uc758 \ub538\uc774\uc5c8\ub2e4\ub294 \uac83 \ubc16\uc5d0\ub294 \uc54c\ub824\uc9c4 \uac83\uc774 \uc5c6\ub2e4. \uadf8\ub294 \uc0ac\uc0dd\uc544\uc600\uc9c0\ub9cc \ubd80\ubaa8\ub85c\ubd80\ud130 \ubc84\ub824\uc9c4 \uac83\uc740 \uc544\ub2c8\uc5c8\ub2e4. \uadf8\uc758 \ubd80\ubaa8\ub294 1483\ub144\uc5d0 \ud751\uc0ac\ubcd1\uc73c\ub85c \uc77c\ucc0d \uc8fd\uc744 \ub54c\uae4c\uc9c0 \uadf8\ub97c \ud0a4\uc6e0\ub2e4. \uc774 \uc2dc\uc808\uc5d0 \uadf8\ub294 \uc5ec\ub7ec \uc218\ub3c4\uc0ac \ud559\uad50\uc5d0\uc11c \uc218\ud559\ud588\ub2e4. \ud2b9\ud788 \uacf5\ub3d9\uc0dd\ud65c \ud615\uc81c\ud68c(Brethren of the Common Life)\uac00 \uc6b4\uc601\ud55c \ud559\uad50\uc5d0\uc11c \uadf8\ub294 \u300a\uadf8\ub9ac\uc2a4\ub3c4\uc758 \ubaa8\ubc29\u300b(The Imitation of Christ)\ub77c\ub294 \ucc45\uc744 \ud1b5\ud574 \uc2e0\uacfc\uc758 \uac1c\uc778\uc801 \ub9cc\ub0a8\uc758 \uc911\uc694\uc131\uc5d0 \ub300\ud574 \uc5f0\uad6c\ud558\uace0, \uc218\ub3c4\uc6d0\uacfc \ud559\uad50\uc758 \uc5c4\uaca9\ud55c \uaddc\uc728\uc744 \ud53c\ud588\ub2e4."} {"question": "\uc54c\ub809\uc0b0\ub370\ub974 7\uc138\ub294 \ub3c8\uacfc \uba85\uc608, \uad8c\ub825\uc744 \ud610\uc624\ud558\uba70 \ubb34\uc5c7\uc744 \uc774\ubd88 \uc0bc\uc544 \uc7a0\uc744 \uc7a4\ub294\uac00?", "paragraph": "\uad50\ud669\uc9c1\uc5d0 \uc624\ub978 \ucc98\uc74c \uba87 \uac1c\uc6d4 \ub3d9\uc548 \uadf8\ub294 \ub9e4\uc6b0 \ubcf5\uc74c\uc801\uc778 \uc0b6\uc744 \uc0b4\uc558\ub2e4. \uadf8\ub294 \uc7ac\ub97c \uc774\ubd88 \uc0bc\uc544 \ub531\ub531\ud55c \uce68\ub300 \uc704\uc5d0\uc11c \uc7a0\uc744 \uccad\ud588\uc73c\uba70, \ub3c8\uacfc \uba85\uc608, \uad8c\ub825\uc744 \ud610\uc624\ud588\ub2e4. \uac01\uad6d \ub300\uc0ac\ub4e4\uc758 \uc54c\ud604\uc744 \ubc1b\uc744 \ub54c \uadf8\ub294 \ud56d\uc0c1 \uc790\uc2e0\uc774 \uc5b8\uc820\uac00 \uc774 \uc138\uc0c1\uc744 \ub5a0\ub09c\ub2e4\ub294 \uc0ac\uc2e4\uc744 \uc0c1\uae30\ud558\uae30 \uc704\ud574 \uc54c\ud604\uc2e4 \ub0b4\ubd80\ub97c \ud574\uace8\ub4e4\ub85c \uc7a5\uc2dd\ud558\uace0 \ub208\uc55e\uc5d0 \uc790\uc2e0\uc774 \ub4e4\uc5b4\uac08 \uad00\uc744 \ub193\uace4 \ud558\uc600\ub2e4. \uadf8\ub294 \ud6cc\ub96d\ud55c \uc2e0\uc2ec\uacfc \uac70\ub8e9\ud55c \uc0b6\uc740 \ub3c4\ucc98\uc5d0 \ud37c\uc838 \ub9cc\uc778\uc758 \uc874\uacbd\uc744 \ubc1b\uc558\ub2e4. \ud558\uc9c0\ub9cc \uc624\ub798 \uc9c0\ub098\uc9c0 \uc54a\uc544 \uadf8\uac00 \uc790\uc2e0\uc758 \uc77c\uc871\ub4e4\uc744 \ubd88\ub7ec\ub4e4\uc774\uba74\uc11c \uadf8\uc758 \uc131\uaca9\uc774 \ubcc0\ud558\uac8c \ub418\uc5c8\ub2e4. \uacb8\uc190\ud588\ub358 \uadf8\ub294 \ud5c8\uc601\uc2ec \ub9ce\uc740 \uc778\ubb3c\uc774 \ub418\uc5c8\ub2e4. \ub354 \uc774\uc0c1 \uae08\uc695\uc801\uc778 \uc0b6\uc744 \uc0b4\uc9c0 \uc54a\uc558\uc73c\uba70, \uadf8\uc758 \ub531\ub531\ud55c \uce68\ub300\ub294 \ubd80\ub4dc\ub7ec\uc6b4 \uae43\ud138 \uce68\ub300\ub85c \ubc14\ub00c\uc5c8\uace0, \ud574\uace8\ub85c \uac00\ub4dd \ucc3c\ub358 \uc54c\ud604\uc2e4\uc740 \ubcf4\uc11d\uc73c\ub85c \uac00\ub4dd \ucc3c\ub2e4. \uc8fd\uc74c\uc744 \uc0c1\uae30\ud588\ub358 \uadf8\uc758 \ub9c8\uc74c\uc740 \uc57c\ub9dd\uc73c\ub85c \ubc14\ub00c\uc5c8\uc73c\uba70, \ub9c8\uce58 \uc8fd\uc74c\uc744 \ub354\ub7fd\ud788\uace0 \ub3c8\uc73c\ub85c \ubaa9\uc228\uc744 \uc0ac\ub824\ub294 \ub4ef\uc774 \ube44\uc5b4\uc788\ub358 \uadf8\uc758 \uad00\uc740 \ub3c8\uc73c\ub85c \ucc44\uc6cc\uc84c\ub2e4.", "answer": "\uc7ac", "sentence": "\uadf8\ub294 \uc7ac \ub97c \uc774\ubd88 \uc0bc\uc544 \ub531\ub531\ud55c \uce68\ub300 \uc704\uc5d0\uc11c \uc7a0\uc744 \uccad\ud588\uc73c\uba70, \ub3c8\uacfc \uba85\uc608, \uad8c\ub825\uc744 \ud610\uc624\ud588\ub2e4.", "paragraph_sentence": "\uad50\ud669\uc9c1\uc5d0 \uc624\ub978 \ucc98\uc74c \uba87 \uac1c\uc6d4 \ub3d9\uc548 \uadf8\ub294 \ub9e4\uc6b0 \ubcf5\uc74c\uc801\uc778 \uc0b6\uc744 \uc0b4\uc558\ub2e4. \uadf8\ub294 \uc7ac \ub97c \uc774\ubd88 \uc0bc\uc544 \ub531\ub531\ud55c \uce68\ub300 \uc704\uc5d0\uc11c \uc7a0\uc744 \uccad\ud588\uc73c\uba70, \ub3c8\uacfc \uba85\uc608, \uad8c\ub825\uc744 \ud610\uc624\ud588\ub2e4. \uac01\uad6d \ub300\uc0ac\ub4e4\uc758 \uc54c\ud604\uc744 \ubc1b\uc744 \ub54c \uadf8\ub294 \ud56d\uc0c1 \uc790\uc2e0\uc774 \uc5b8\uc820\uac00 \uc774 \uc138\uc0c1\uc744 \ub5a0\ub09c\ub2e4\ub294 \uc0ac\uc2e4\uc744 \uc0c1\uae30\ud558\uae30 \uc704\ud574 \uc54c\ud604\uc2e4 \ub0b4\ubd80\ub97c \ud574\uace8\ub4e4\ub85c \uc7a5\uc2dd\ud558\uace0 \ub208\uc55e\uc5d0 \uc790\uc2e0\uc774 \ub4e4\uc5b4\uac08 \uad00\uc744 \ub193\uace4 \ud558\uc600\ub2e4. \uadf8\ub294 \ud6cc\ub96d\ud55c \uc2e0\uc2ec\uacfc \uac70\ub8e9\ud55c \uc0b6\uc740 \ub3c4\ucc98\uc5d0 \ud37c\uc838 \ub9cc\uc778\uc758 \uc874\uacbd\uc744 \ubc1b\uc558\ub2e4. \ud558\uc9c0\ub9cc \uc624\ub798 \uc9c0\ub098\uc9c0 \uc54a\uc544 \uadf8\uac00 \uc790\uc2e0\uc758 \uc77c\uc871\ub4e4\uc744 \ubd88\ub7ec\ub4e4\uc774\uba74\uc11c \uadf8\uc758 \uc131\uaca9\uc774 \ubcc0\ud558\uac8c \ub418\uc5c8\ub2e4. \uacb8\uc190\ud588\ub358 \uadf8\ub294 \ud5c8\uc601\uc2ec \ub9ce\uc740 \uc778\ubb3c\uc774 \ub418\uc5c8\ub2e4. \ub354 \uc774\uc0c1 \uae08\uc695\uc801\uc778 \uc0b6\uc744 \uc0b4\uc9c0 \uc54a\uc558\uc73c\uba70, \uadf8\uc758 \ub531\ub531\ud55c \uce68\ub300\ub294 \ubd80\ub4dc\ub7ec\uc6b4 \uae43\ud138 \uce68\ub300\ub85c \ubc14\ub00c\uc5c8\uace0, \ud574\uace8\ub85c \uac00\ub4dd \ucc3c\ub358 \uc54c\ud604\uc2e4\uc740 \ubcf4\uc11d\uc73c\ub85c \uac00\ub4dd \ucc3c\ub2e4. \uc8fd\uc74c\uc744 \uc0c1\uae30\ud588\ub358 \uadf8\uc758 \ub9c8\uc74c\uc740 \uc57c\ub9dd\uc73c\ub85c \ubc14\ub00c\uc5c8\uc73c\uba70, \ub9c8\uce58 \uc8fd\uc74c\uc744 \ub354\ub7fd\ud788\uace0 \ub3c8\uc73c\ub85c \ubaa9\uc228\uc744 \uc0ac\ub824\ub294 \ub4ef\uc774 \ube44\uc5b4\uc788\ub358 \uadf8\uc758 \uad00\uc740 \ub3c8\uc73c\ub85c \ucc44\uc6cc\uc84c\ub2e4.", "paragraph_answer": "\uad50\ud669\uc9c1\uc5d0 \uc624\ub978 \ucc98\uc74c \uba87 \uac1c\uc6d4 \ub3d9\uc548 \uadf8\ub294 \ub9e4\uc6b0 \ubcf5\uc74c\uc801\uc778 \uc0b6\uc744 \uc0b4\uc558\ub2e4. \uadf8\ub294 \uc7ac \ub97c \uc774\ubd88 \uc0bc\uc544 \ub531\ub531\ud55c \uce68\ub300 \uc704\uc5d0\uc11c \uc7a0\uc744 \uccad\ud588\uc73c\uba70, \ub3c8\uacfc \uba85\uc608, \uad8c\ub825\uc744 \ud610\uc624\ud588\ub2e4. \uac01\uad6d \ub300\uc0ac\ub4e4\uc758 \uc54c\ud604\uc744 \ubc1b\uc744 \ub54c \uadf8\ub294 \ud56d\uc0c1 \uc790\uc2e0\uc774 \uc5b8\uc820\uac00 \uc774 \uc138\uc0c1\uc744 \ub5a0\ub09c\ub2e4\ub294 \uc0ac\uc2e4\uc744 \uc0c1\uae30\ud558\uae30 \uc704\ud574 \uc54c\ud604\uc2e4 \ub0b4\ubd80\ub97c \ud574\uace8\ub4e4\ub85c \uc7a5\uc2dd\ud558\uace0 \ub208\uc55e\uc5d0 \uc790\uc2e0\uc774 \ub4e4\uc5b4\uac08 \uad00\uc744 \ub193\uace4 \ud558\uc600\ub2e4. \uadf8\ub294 \ud6cc\ub96d\ud55c \uc2e0\uc2ec\uacfc \uac70\ub8e9\ud55c \uc0b6\uc740 \ub3c4\ucc98\uc5d0 \ud37c\uc838 \ub9cc\uc778\uc758 \uc874\uacbd\uc744 \ubc1b\uc558\ub2e4. \ud558\uc9c0\ub9cc \uc624\ub798 \uc9c0\ub098\uc9c0 \uc54a\uc544 \uadf8\uac00 \uc790\uc2e0\uc758 \uc77c\uc871\ub4e4\uc744 \ubd88\ub7ec\ub4e4\uc774\uba74\uc11c \uadf8\uc758 \uc131\uaca9\uc774 \ubcc0\ud558\uac8c \ub418\uc5c8\ub2e4. \uacb8\uc190\ud588\ub358 \uadf8\ub294 \ud5c8\uc601\uc2ec \ub9ce\uc740 \uc778\ubb3c\uc774 \ub418\uc5c8\ub2e4. \ub354 \uc774\uc0c1 \uae08\uc695\uc801\uc778 \uc0b6\uc744 \uc0b4\uc9c0 \uc54a\uc558\uc73c\uba70, \uadf8\uc758 \ub531\ub531\ud55c \uce68\ub300\ub294 \ubd80\ub4dc\ub7ec\uc6b4 \uae43\ud138 \uce68\ub300\ub85c \ubc14\ub00c\uc5c8\uace0, \ud574\uace8\ub85c \uac00\ub4dd \ucc3c\ub358 \uc54c\ud604\uc2e4\uc740 \ubcf4\uc11d\uc73c\ub85c \uac00\ub4dd \ucc3c\ub2e4. \uc8fd\uc74c\uc744 \uc0c1\uae30\ud588\ub358 \uadf8\uc758 \ub9c8\uc74c\uc740 \uc57c\ub9dd\uc73c\ub85c \ubc14\ub00c\uc5c8\uc73c\uba70, \ub9c8\uce58 \uc8fd\uc74c\uc744 \ub354\ub7fd\ud788\uace0 \ub3c8\uc73c\ub85c \ubaa9\uc228\uc744 \uc0ac\ub824\ub294 \ub4ef\uc774 \ube44\uc5b4\uc788\ub358 \uadf8\uc758 \uad00\uc740 \ub3c8\uc73c\ub85c \ucc44\uc6cc\uc84c\ub2e4.", "sentence_answer": "\uadf8\ub294 \uc7ac \ub97c \uc774\ubd88 \uc0bc\uc544 \ub531\ub531\ud55c \uce68\ub300 \uc704\uc5d0\uc11c \uc7a0\uc744 \uccad\ud588\uc73c\uba70, \ub3c8\uacfc \uba85\uc608, \uad8c\ub825\uc744 \ud610\uc624\ud588\ub2e4.", "paragraph_id": "6656721-2-1", "paragraph_question": "question: \uc54c\ub809\uc0b0\ub370\ub974 7\uc138\ub294 \ub3c8\uacfc \uba85\uc608, \uad8c\ub825\uc744 \ud610\uc624\ud558\uba70 \ubb34\uc5c7\uc744 \uc774\ubd88 \uc0bc\uc544 \uc7a0\uc744 \uc7a4\ub294\uac00?, context: \uad50\ud669\uc9c1\uc5d0 \uc624\ub978 \ucc98\uc74c \uba87 \uac1c\uc6d4 \ub3d9\uc548 \uadf8\ub294 \ub9e4\uc6b0 \ubcf5\uc74c\uc801\uc778 \uc0b6\uc744 \uc0b4\uc558\ub2e4. \uadf8\ub294 \uc7ac\ub97c \uc774\ubd88 \uc0bc\uc544 \ub531\ub531\ud55c \uce68\ub300 \uc704\uc5d0\uc11c \uc7a0\uc744 \uccad\ud588\uc73c\uba70, \ub3c8\uacfc \uba85\uc608, \uad8c\ub825\uc744 \ud610\uc624\ud588\ub2e4. \uac01\uad6d \ub300\uc0ac\ub4e4\uc758 \uc54c\ud604\uc744 \ubc1b\uc744 \ub54c \uadf8\ub294 \ud56d\uc0c1 \uc790\uc2e0\uc774 \uc5b8\uc820\uac00 \uc774 \uc138\uc0c1\uc744 \ub5a0\ub09c\ub2e4\ub294 \uc0ac\uc2e4\uc744 \uc0c1\uae30\ud558\uae30 \uc704\ud574 \uc54c\ud604\uc2e4 \ub0b4\ubd80\ub97c \ud574\uace8\ub4e4\ub85c \uc7a5\uc2dd\ud558\uace0 \ub208\uc55e\uc5d0 \uc790\uc2e0\uc774 \ub4e4\uc5b4\uac08 \uad00\uc744 \ub193\uace4 \ud558\uc600\ub2e4. \uadf8\ub294 \ud6cc\ub96d\ud55c \uc2e0\uc2ec\uacfc \uac70\ub8e9\ud55c \uc0b6\uc740 \ub3c4\ucc98\uc5d0 \ud37c\uc838 \ub9cc\uc778\uc758 \uc874\uacbd\uc744 \ubc1b\uc558\ub2e4. \ud558\uc9c0\ub9cc \uc624\ub798 \uc9c0\ub098\uc9c0 \uc54a\uc544 \uadf8\uac00 \uc790\uc2e0\uc758 \uc77c\uc871\ub4e4\uc744 \ubd88\ub7ec\ub4e4\uc774\uba74\uc11c \uadf8\uc758 \uc131\uaca9\uc774 \ubcc0\ud558\uac8c \ub418\uc5c8\ub2e4. \uacb8\uc190\ud588\ub358 \uadf8\ub294 \ud5c8\uc601\uc2ec \ub9ce\uc740 \uc778\ubb3c\uc774 \ub418\uc5c8\ub2e4. \ub354 \uc774\uc0c1 \uae08\uc695\uc801\uc778 \uc0b6\uc744 \uc0b4\uc9c0 \uc54a\uc558\uc73c\uba70, \uadf8\uc758 \ub531\ub531\ud55c \uce68\ub300\ub294 \ubd80\ub4dc\ub7ec\uc6b4 \uae43\ud138 \uce68\ub300\ub85c \ubc14\ub00c\uc5c8\uace0, \ud574\uace8\ub85c \uac00\ub4dd \ucc3c\ub358 \uc54c\ud604\uc2e4\uc740 \ubcf4\uc11d\uc73c\ub85c \uac00\ub4dd \ucc3c\ub2e4. \uc8fd\uc74c\uc744 \uc0c1\uae30\ud588\ub358 \uadf8\uc758 \ub9c8\uc74c\uc740 \uc57c\ub9dd\uc73c\ub85c \ubc14\ub00c\uc5c8\uc73c\uba70, \ub9c8\uce58 \uc8fd\uc74c\uc744 \ub354\ub7fd\ud788\uace0 \ub3c8\uc73c\ub85c \ubaa9\uc228\uc744 \uc0ac\ub824\ub294 \ub4ef\uc774 \ube44\uc5b4\uc788\ub358 \uadf8\uc758 \uad00\uc740 \ub3c8\uc73c\ub85c \ucc44\uc6cc\uc84c\ub2e4."} {"question": "1987\ub144 \uc900\uacf5\ud55c \ucda9\uc8fc\ucea0\ud37c\uc2a4 \ub3c4\uc11c\uad00\uc758 \uc774\ub984\uc740?", "paragraph": "1980\ub144 \ud604 \uae00\ub85c\uceec\ucea0\ud37c\uc2a4\uc758 \ubaa8\uccb4\uc778 \uac74\uad6d\ub300\ud559\uad50 \ucda9\uc8fc\ubd84\uad50\ub97c \ubb38\uad50\ub85c\ubd80\ub85c\ubd80\ud130 \uc870\uac74\ubd80\ub85c \uc778\uac00\ub97c \ubc1b\uc544 \uc124\ub9bd\ud558\uac8c \ub418\uc5c8\uc73c\uba70, 1980\ub144 \uac1c\uad50\ud558\uc600\ub2e4. \uc6d0\ub798\ub294 \uc77c\ubc18 \ub300\ud559\uad50\uc758 \ub300\ud559\ucc98\ub7fc \uac74\uad6d\ub300\ud559\uad50 \ucda9\uc8fc\ub300\ud559\uc758 \uc778\uc2dd\uc774\uc5c8\ub2e4. \ud5c8\ub098 \uc778\uc6d0 \uc99d\ub300, \ub9ce\uc740 \uacfc\ub4e4\uc774 \uc124\ub9bd\ub418\uace0 \ub09c \uc774\ud6c4 1984\ub144 \ucda9\uc8fc\ub300\ud559\uc744 \uc778\ubb38\uacfc\ud559\ub300\ud559, \uc0ac\ud68c\uacfc\ud559\ub300\ud559\uc73c\ub85c \ubd84\ub9ac\ud558\uc600\ub2e4. \uadf8 \ub4a4 1986\ub144 \uac74\uad6d\ub300\ud559\uad50 \ucda9\uc8fc\ucea0\ud37c\uc2a4 \uc758\uacfc\ub300\ud559\uc744 \uc124\ub9bd\uc778\uac00(40\uba85\uc815\uc6d0)\uc744 \ubc1b\uc558\ub2e4. 1987\ub144 \ucda9\uc8fc\ucea0\ud37c\uc2a4\uc758 \ub3c4\uc11c\uad00\uc778 \uc911\uc6d0\ub3c4\uc11c\uad00\uc744 \uc900\uacf5\ud558\uc600\ub2e4. \uc774\ub54c \uac74\uad6d\ub300\ud559\uad50\ub294 \ucda9\uc8fc\ucea0\ud37c\uc2a4(\ud604 \uae00\ub85c\uceec\ucea0\ud37c\uc2a4)\uc758 \uac1c\uad50\uc640 \ud568\uaed8 \ucda9\uc8fc\ucea0\ud37c\uc2a4 \uc758\uacfc\ub300\ud559\uc720\uce58\ub77c\ub294 \uac00\uc7a5 \ud070 \uc131\uacfc\ub97c \ub9db\ubcf4\uac8c \ub418\uc5c8\uace0, \uc544\uc2dc\uc544\uc5d0\uc11c \uc81c\uc77c \ud070 \ub3c4\uc11c\uad00\uc778 \uc0c1\ud5c8\uae30\ub150\ub3c4\uc11c\uad00\uc744 \uc900\uacf5\ud558\uc5ec \ub300\ub300\uc801\uc778 \ubc1c\uc804\uc744 \ud558\uac8c \ub418\uc5c8\ub2e4.", "answer": "\uc911\uc6d0\ub3c4\uc11c\uad00", "sentence": "1987\ub144 \ucda9\uc8fc\ucea0\ud37c\uc2a4\uc758 \ub3c4\uc11c\uad00\uc778 \uc911\uc6d0\ub3c4\uc11c\uad00 \uc744 \uc900\uacf5\ud558\uc600\ub2e4.", "paragraph_sentence": "1980\ub144 \ud604 \uae00\ub85c\uceec\ucea0\ud37c\uc2a4\uc758 \ubaa8\uccb4\uc778 \uac74\uad6d\ub300\ud559\uad50 \ucda9\uc8fc\ubd84\uad50\ub97c \ubb38\uad50\ub85c\ubd80\ub85c\ubd80\ud130 \uc870\uac74\ubd80\ub85c \uc778\uac00\ub97c \ubc1b\uc544 \uc124\ub9bd\ud558\uac8c \ub418\uc5c8\uc73c\uba70, 1980\ub144 \uac1c\uad50\ud558\uc600\ub2e4. \uc6d0\ub798\ub294 \uc77c\ubc18 \ub300\ud559\uad50\uc758 \ub300\ud559\ucc98\ub7fc \uac74\uad6d\ub300\ud559\uad50 \ucda9\uc8fc\ub300\ud559\uc758 \uc778\uc2dd\uc774\uc5c8\ub2e4. \ud5c8\ub098 \uc778\uc6d0 \uc99d\ub300, \ub9ce\uc740 \uacfc\ub4e4\uc774 \uc124\ub9bd\ub418\uace0 \ub09c \uc774\ud6c4 1984\ub144 \ucda9\uc8fc\ub300\ud559\uc744 \uc778\ubb38\uacfc\ud559\ub300\ud559, \uc0ac\ud68c\uacfc\ud559\ub300\ud559\uc73c\ub85c \ubd84\ub9ac\ud558\uc600\ub2e4. \uadf8 \ub4a4 1986\ub144 \uac74\uad6d\ub300\ud559\uad50 \ucda9\uc8fc\ucea0\ud37c\uc2a4 \uc758\uacfc\ub300\ud559\uc744 \uc124\ub9bd\uc778\uac00(40\uba85\uc815\uc6d0)\uc744 \ubc1b\uc558\ub2e4. 1987\ub144 \ucda9\uc8fc\ucea0\ud37c\uc2a4\uc758 \ub3c4\uc11c\uad00\uc778 \uc911\uc6d0\ub3c4\uc11c\uad00 \uc744 \uc900\uacf5\ud558\uc600\ub2e4. \uc774\ub54c \uac74\uad6d\ub300\ud559\uad50\ub294 \ucda9\uc8fc\ucea0\ud37c\uc2a4(\ud604 \uae00\ub85c\uceec\ucea0\ud37c\uc2a4)\uc758 \uac1c\uad50\uc640 \ud568\uaed8 \ucda9\uc8fc\ucea0\ud37c\uc2a4 \uc758\uacfc\ub300\ud559\uc720\uce58\ub77c\ub294 \uac00\uc7a5 \ud070 \uc131\uacfc\ub97c \ub9db\ubcf4\uac8c \ub418\uc5c8\uace0, \uc544\uc2dc\uc544\uc5d0\uc11c \uc81c\uc77c \ud070 \ub3c4\uc11c\uad00\uc778 \uc0c1\ud5c8\uae30\ub150\ub3c4\uc11c\uad00\uc744 \uc900\uacf5\ud558\uc5ec \ub300\ub300\uc801\uc778 \ubc1c\uc804\uc744 \ud558\uac8c \ub418\uc5c8\ub2e4.", "paragraph_answer": "1980\ub144 \ud604 \uae00\ub85c\uceec\ucea0\ud37c\uc2a4\uc758 \ubaa8\uccb4\uc778 \uac74\uad6d\ub300\ud559\uad50 \ucda9\uc8fc\ubd84\uad50\ub97c \ubb38\uad50\ub85c\ubd80\ub85c\ubd80\ud130 \uc870\uac74\ubd80\ub85c \uc778\uac00\ub97c \ubc1b\uc544 \uc124\ub9bd\ud558\uac8c \ub418\uc5c8\uc73c\uba70, 1980\ub144 \uac1c\uad50\ud558\uc600\ub2e4. \uc6d0\ub798\ub294 \uc77c\ubc18 \ub300\ud559\uad50\uc758 \ub300\ud559\ucc98\ub7fc \uac74\uad6d\ub300\ud559\uad50 \ucda9\uc8fc\ub300\ud559\uc758 \uc778\uc2dd\uc774\uc5c8\ub2e4. \ud5c8\ub098 \uc778\uc6d0 \uc99d\ub300, \ub9ce\uc740 \uacfc\ub4e4\uc774 \uc124\ub9bd\ub418\uace0 \ub09c \uc774\ud6c4 1984\ub144 \ucda9\uc8fc\ub300\ud559\uc744 \uc778\ubb38\uacfc\ud559\ub300\ud559, \uc0ac\ud68c\uacfc\ud559\ub300\ud559\uc73c\ub85c \ubd84\ub9ac\ud558\uc600\ub2e4. \uadf8 \ub4a4 1986\ub144 \uac74\uad6d\ub300\ud559\uad50 \ucda9\uc8fc\ucea0\ud37c\uc2a4 \uc758\uacfc\ub300\ud559\uc744 \uc124\ub9bd\uc778\uac00(40\uba85\uc815\uc6d0)\uc744 \ubc1b\uc558\ub2e4. 1987\ub144 \ucda9\uc8fc\ucea0\ud37c\uc2a4\uc758 \ub3c4\uc11c\uad00\uc778 \uc911\uc6d0\ub3c4\uc11c\uad00 \uc744 \uc900\uacf5\ud558\uc600\ub2e4. \uc774\ub54c \uac74\uad6d\ub300\ud559\uad50\ub294 \ucda9\uc8fc\ucea0\ud37c\uc2a4(\ud604 \uae00\ub85c\uceec\ucea0\ud37c\uc2a4)\uc758 \uac1c\uad50\uc640 \ud568\uaed8 \ucda9\uc8fc\ucea0\ud37c\uc2a4 \uc758\uacfc\ub300\ud559\uc720\uce58\ub77c\ub294 \uac00\uc7a5 \ud070 \uc131\uacfc\ub97c \ub9db\ubcf4\uac8c \ub418\uc5c8\uace0, \uc544\uc2dc\uc544\uc5d0\uc11c \uc81c\uc77c \ud070 \ub3c4\uc11c\uad00\uc778 \uc0c1\ud5c8\uae30\ub150\ub3c4\uc11c\uad00\uc744 \uc900\uacf5\ud558\uc5ec \ub300\ub300\uc801\uc778 \ubc1c\uc804\uc744 \ud558\uac8c \ub418\uc5c8\ub2e4.", "sentence_answer": "1987\ub144 \ucda9\uc8fc\ucea0\ud37c\uc2a4\uc758 \ub3c4\uc11c\uad00\uc778 \uc911\uc6d0\ub3c4\uc11c\uad00 \uc744 \uc900\uacf5\ud558\uc600\ub2e4.", "paragraph_id": "6470332-2-2", "paragraph_question": "question: 1987\ub144 \uc900\uacf5\ud55c \ucda9\uc8fc\ucea0\ud37c\uc2a4 \ub3c4\uc11c\uad00\uc758 \uc774\ub984\uc740?, context: 1980\ub144 \ud604 \uae00\ub85c\uceec\ucea0\ud37c\uc2a4\uc758 \ubaa8\uccb4\uc778 \uac74\uad6d\ub300\ud559\uad50 \ucda9\uc8fc\ubd84\uad50\ub97c \ubb38\uad50\ub85c\ubd80\ub85c\ubd80\ud130 \uc870\uac74\ubd80\ub85c \uc778\uac00\ub97c \ubc1b\uc544 \uc124\ub9bd\ud558\uac8c \ub418\uc5c8\uc73c\uba70, 1980\ub144 \uac1c\uad50\ud558\uc600\ub2e4. \uc6d0\ub798\ub294 \uc77c\ubc18 \ub300\ud559\uad50\uc758 \ub300\ud559\ucc98\ub7fc \uac74\uad6d\ub300\ud559\uad50 \ucda9\uc8fc\ub300\ud559\uc758 \uc778\uc2dd\uc774\uc5c8\ub2e4. \ud5c8\ub098 \uc778\uc6d0 \uc99d\ub300, \ub9ce\uc740 \uacfc\ub4e4\uc774 \uc124\ub9bd\ub418\uace0 \ub09c \uc774\ud6c4 1984\ub144 \ucda9\uc8fc\ub300\ud559\uc744 \uc778\ubb38\uacfc\ud559\ub300\ud559, \uc0ac\ud68c\uacfc\ud559\ub300\ud559\uc73c\ub85c \ubd84\ub9ac\ud558\uc600\ub2e4. \uadf8 \ub4a4 1986\ub144 \uac74\uad6d\ub300\ud559\uad50 \ucda9\uc8fc\ucea0\ud37c\uc2a4 \uc758\uacfc\ub300\ud559\uc744 \uc124\ub9bd\uc778\uac00(40\uba85\uc815\uc6d0)\uc744 \ubc1b\uc558\ub2e4. 1987\ub144 \ucda9\uc8fc\ucea0\ud37c\uc2a4\uc758 \ub3c4\uc11c\uad00\uc778 \uc911\uc6d0\ub3c4\uc11c\uad00\uc744 \uc900\uacf5\ud558\uc600\ub2e4. \uc774\ub54c \uac74\uad6d\ub300\ud559\uad50\ub294 \ucda9\uc8fc\ucea0\ud37c\uc2a4(\ud604 \uae00\ub85c\uceec\ucea0\ud37c\uc2a4)\uc758 \uac1c\uad50\uc640 \ud568\uaed8 \ucda9\uc8fc\ucea0\ud37c\uc2a4 \uc758\uacfc\ub300\ud559\uc720\uce58\ub77c\ub294 \uac00\uc7a5 \ud070 \uc131\uacfc\ub97c \ub9db\ubcf4\uac8c \ub418\uc5c8\uace0, \uc544\uc2dc\uc544\uc5d0\uc11c \uc81c\uc77c \ud070 \ub3c4\uc11c\uad00\uc778 \uc0c1\ud5c8\uae30\ub150\ub3c4\uc11c\uad00\uc744 \uc900\uacf5\ud558\uc5ec \ub300\ub300\uc801\uc778 \ubc1c\uc804\uc744 \ud558\uac8c \ub418\uc5c8\ub2e4."} {"question": "\uc601\ub77d\uc81c\uac00 \ub3d9\ubd81\uc9c0\ubc29\uc5d0 \uc131\uc744 \uc313\uc544 \uacf5\ub7b5\ud558\ub824 \ud588\ub358 \uacf3\uc740?", "paragraph": "\ub610\ud55c \ub3d9\ubd81\uc9c0\ubc29\uc5d0 \uc131\uc744 \uc313\uace0 \uc5ec\uc9c4\uc871\uc744 \uacf5\ub7b5\ud558\ub294 \ud55c\ud3b8 \uc5ec\uc9c4\uc871\uc744 \ud1b5\uc81c\ud558\ub294 \ud1b5\uc81c\uae30\uad00\uc758 \uc124\uce58\ub97c \ucd94\uc9c4\ud55c\ub2e4. 1403\ub144 \ub9cc\uc8fc\uc758 \uc5ec\uc9c4\uc871\uc744 \ud1b5\ud560\ud558\uae30 \uc704\ud558\uc5ec \ubc31\ub450\uc0b0 \ubd81\ucabd\uc5d0 \uac74\uc8fc\uc704(\u5efa\u5dde\u885b)\ub97c \uc124\uce58\ud588\ub294\ub370, \uac74\uc8fc\uc704\uc758 \uc124\uce58\uc7a5\uc18c\ub294 \uac74\uc8fc \uc9c0\ub9b0 \uc131 \ubd80\uadfc\uc758 \ud718\ubc1c\ucc9c(\u8f1d\u767c\u5ddd) \uc0c1\ub958\uc5d0 \uc788\ub294 \ubd81\uc0b0\uc131\uc790(\u5317\u5c71\u57ce\u5b50)\uc600\ub2e4. \uadf8\ub7ec\ub098 \uc5ec\uc9c4\uc871\uc758 \ubd80\ub77d\uc740 \ub2e4\uc591\ud588\uace0, 1411\ub144\uc5d0\ub294 \ud5e4\uc774\ub8fd\uac15(\u9ed1\u9f8d\u6c5f) \ud558\ub958\uc5d0 \ub204\ub974\uac04 \ub3c4\uc9c0\ud718\uc0ac\uc0ac(\u5974\u5152\u5e72\u90fd\u6307\u63ee\u4f7f\u53f8)\ub97c \ub450\uc5c8\ub2e4. \uac74\uc8fc\uc704\u00b7\uc6b0\uc790\uc704(\u5140\u8005\u885b)\u00b7\ub204\ub974\uac04\uc704(\u5974\u5152\u5e72\u885b)\ub97c \uc77c\uad04 \ud1b5\uc81c\ud558\uae30 \uc704\ud558\uc5ec \uc601\ub77d\uc81c\ub294 1411\ub144 \ud0dc\uac10(\u592a\u76e3) \uc774\uc2dc\ud558(\u8d64\u5931\u54c8) \ub4f1\uc5d0\uac8c \uba85\ud558\uc5ec \uad70\ubcd1 \uc57d 1,000\uc744 \uc778\uc194\ud558\uace0 25\ucc99\uc758 \uc120\ubc15\uc73c\ub85c \uc479\ud654 \uac15(\u677e\u82b1\u6c5f)\u00b7\ud5e4\uc774\ub8fd\uac15\uc774 \ub9cc\ub098\ub294 \ud558\ub958\uc9c0\uc810\uc5d0 \ud589\uc815\uad00\uccad\uc778 \ub3c4\uc0ac(\u90fd\u53f8)\ub97c \uc124\uce58\ud558\uc5ec 3\uac1c\uc758 \uc5ec\uc9c4 \ubd80\ub77d\uc744 \uac10\uc2dc, \ud1b5\uc81c\ud558\uc600\ub2e4.", "answer": "\uc5ec\uc9c4\uc871", "sentence": "\ub610\ud55c \ub3d9\ubd81\uc9c0\ubc29\uc5d0 \uc131\uc744 \uc313\uace0 \uc5ec\uc9c4\uc871 \uc744 \uacf5\ub7b5\ud558\ub294 \ud55c\ud3b8 \uc5ec\uc9c4\uc871\uc744 \ud1b5\uc81c\ud558\ub294 \ud1b5\uc81c\uae30\uad00\uc758 \uc124\uce58\ub97c \ucd94\uc9c4\ud55c\ub2e4.", "paragraph_sentence": " \ub610\ud55c \ub3d9\ubd81\uc9c0\ubc29\uc5d0 \uc131\uc744 \uc313\uace0 \uc5ec\uc9c4\uc871 \uc744 \uacf5\ub7b5\ud558\ub294 \ud55c\ud3b8 \uc5ec\uc9c4\uc871\uc744 \ud1b5\uc81c\ud558\ub294 \ud1b5\uc81c\uae30\uad00\uc758 \uc124\uce58\ub97c \ucd94\uc9c4\ud55c\ub2e4. 1403\ub144 \ub9cc\uc8fc\uc758 \uc5ec\uc9c4\uc871\uc744 \ud1b5\ud560\ud558\uae30 \uc704\ud558\uc5ec \ubc31\ub450\uc0b0 \ubd81\ucabd\uc5d0 \uac74\uc8fc\uc704(\u5efa\u5dde\u885b)\ub97c \uc124\uce58\ud588\ub294\ub370, \uac74\uc8fc\uc704\uc758 \uc124\uce58\uc7a5\uc18c\ub294 \uac74\uc8fc \uc9c0\ub9b0 \uc131 \ubd80\uadfc\uc758 \ud718\ubc1c\ucc9c(\u8f1d\u767c\u5ddd) \uc0c1\ub958\uc5d0 \uc788\ub294 \ubd81\uc0b0\uc131\uc790(\u5317\u5c71\u57ce\u5b50)\uc600\ub2e4. \uadf8\ub7ec\ub098 \uc5ec\uc9c4\uc871\uc758 \ubd80\ub77d\uc740 \ub2e4\uc591\ud588\uace0, 1411\ub144\uc5d0\ub294 \ud5e4\uc774\ub8fd\uac15(\u9ed1\u9f8d\u6c5f) \ud558\ub958\uc5d0 \ub204\ub974\uac04 \ub3c4\uc9c0\ud718\uc0ac\uc0ac(\u5974\u5152\u5e72\u90fd\u6307\u63ee\u4f7f\u53f8)\ub97c \ub450\uc5c8\ub2e4. \uac74\uc8fc\uc704\u00b7\uc6b0\uc790\uc704(\u5140\u8005\u885b)\u00b7\ub204\ub974\uac04\uc704(\u5974\u5152\u5e72\u885b)\ub97c \uc77c\uad04 \ud1b5\uc81c\ud558\uae30 \uc704\ud558\uc5ec \uc601\ub77d\uc81c\ub294 1411\ub144 \ud0dc\uac10(\u592a\u76e3) \uc774\uc2dc\ud558(\u8d64\u5931\u54c8) \ub4f1\uc5d0\uac8c \uba85\ud558\uc5ec \uad70\ubcd1 \uc57d 1,000\uc744 \uc778\uc194\ud558\uace0 25\ucc99\uc758 \uc120\ubc15\uc73c\ub85c \uc479\ud654 \uac15(\u677e\u82b1\u6c5f)\u00b7\ud5e4\uc774\ub8fd\uac15\uc774 \ub9cc\ub098\ub294 \ud558\ub958\uc9c0\uc810\uc5d0 \ud589\uc815\uad00\uccad\uc778 \ub3c4\uc0ac(\u90fd\u53f8)\ub97c \uc124\uce58\ud558\uc5ec 3\uac1c\uc758 \uc5ec\uc9c4 \ubd80\ub77d\uc744 \uac10\uc2dc, \ud1b5\uc81c\ud558\uc600\ub2e4.", "paragraph_answer": "\ub610\ud55c \ub3d9\ubd81\uc9c0\ubc29\uc5d0 \uc131\uc744 \uc313\uace0 \uc5ec\uc9c4\uc871 \uc744 \uacf5\ub7b5\ud558\ub294 \ud55c\ud3b8 \uc5ec\uc9c4\uc871\uc744 \ud1b5\uc81c\ud558\ub294 \ud1b5\uc81c\uae30\uad00\uc758 \uc124\uce58\ub97c \ucd94\uc9c4\ud55c\ub2e4. 1403\ub144 \ub9cc\uc8fc\uc758 \uc5ec\uc9c4\uc871\uc744 \ud1b5\ud560\ud558\uae30 \uc704\ud558\uc5ec \ubc31\ub450\uc0b0 \ubd81\ucabd\uc5d0 \uac74\uc8fc\uc704(\u5efa\u5dde\u885b)\ub97c \uc124\uce58\ud588\ub294\ub370, \uac74\uc8fc\uc704\uc758 \uc124\uce58\uc7a5\uc18c\ub294 \uac74\uc8fc \uc9c0\ub9b0 \uc131 \ubd80\uadfc\uc758 \ud718\ubc1c\ucc9c(\u8f1d\u767c\u5ddd) \uc0c1\ub958\uc5d0 \uc788\ub294 \ubd81\uc0b0\uc131\uc790(\u5317\u5c71\u57ce\u5b50)\uc600\ub2e4. \uadf8\ub7ec\ub098 \uc5ec\uc9c4\uc871\uc758 \ubd80\ub77d\uc740 \ub2e4\uc591\ud588\uace0, 1411\ub144\uc5d0\ub294 \ud5e4\uc774\ub8fd\uac15(\u9ed1\u9f8d\u6c5f) \ud558\ub958\uc5d0 \ub204\ub974\uac04 \ub3c4\uc9c0\ud718\uc0ac\uc0ac(\u5974\u5152\u5e72\u90fd\u6307\u63ee\u4f7f\u53f8)\ub97c \ub450\uc5c8\ub2e4. \uac74\uc8fc\uc704\u00b7\uc6b0\uc790\uc704(\u5140\u8005\u885b)\u00b7\ub204\ub974\uac04\uc704(\u5974\u5152\u5e72\u885b)\ub97c \uc77c\uad04 \ud1b5\uc81c\ud558\uae30 \uc704\ud558\uc5ec \uc601\ub77d\uc81c\ub294 1411\ub144 \ud0dc\uac10(\u592a\u76e3) \uc774\uc2dc\ud558(\u8d64\u5931\u54c8) \ub4f1\uc5d0\uac8c \uba85\ud558\uc5ec \uad70\ubcd1 \uc57d 1,000\uc744 \uc778\uc194\ud558\uace0 25\ucc99\uc758 \uc120\ubc15\uc73c\ub85c \uc479\ud654 \uac15(\u677e\u82b1\u6c5f)\u00b7\ud5e4\uc774\ub8fd\uac15\uc774 \ub9cc\ub098\ub294 \ud558\ub958\uc9c0\uc810\uc5d0 \ud589\uc815\uad00\uccad\uc778 \ub3c4\uc0ac(\u90fd\u53f8)\ub97c \uc124\uce58\ud558\uc5ec 3\uac1c\uc758 \uc5ec\uc9c4 \ubd80\ub77d\uc744 \uac10\uc2dc, \ud1b5\uc81c\ud558\uc600\ub2e4.", "sentence_answer": "\ub610\ud55c \ub3d9\ubd81\uc9c0\ubc29\uc5d0 \uc131\uc744 \uc313\uace0 \uc5ec\uc9c4\uc871 \uc744 \uacf5\ub7b5\ud558\ub294 \ud55c\ud3b8 \uc5ec\uc9c4\uc871\uc744 \ud1b5\uc81c\ud558\ub294 \ud1b5\uc81c\uae30\uad00\uc758 \uc124\uce58\ub97c \ucd94\uc9c4\ud55c\ub2e4.", "paragraph_id": "6587570-4-0", "paragraph_question": "question: \uc601\ub77d\uc81c\uac00 \ub3d9\ubd81\uc9c0\ubc29\uc5d0 \uc131\uc744 \uc313\uc544 \uacf5\ub7b5\ud558\ub824 \ud588\ub358 \uacf3\uc740?, context: \ub610\ud55c \ub3d9\ubd81\uc9c0\ubc29\uc5d0 \uc131\uc744 \uc313\uace0 \uc5ec\uc9c4\uc871\uc744 \uacf5\ub7b5\ud558\ub294 \ud55c\ud3b8 \uc5ec\uc9c4\uc871\uc744 \ud1b5\uc81c\ud558\ub294 \ud1b5\uc81c\uae30\uad00\uc758 \uc124\uce58\ub97c \ucd94\uc9c4\ud55c\ub2e4. 1403\ub144 \ub9cc\uc8fc\uc758 \uc5ec\uc9c4\uc871\uc744 \ud1b5\ud560\ud558\uae30 \uc704\ud558\uc5ec \ubc31\ub450\uc0b0 \ubd81\ucabd\uc5d0 \uac74\uc8fc\uc704(\u5efa\u5dde\u885b)\ub97c \uc124\uce58\ud588\ub294\ub370, \uac74\uc8fc\uc704\uc758 \uc124\uce58\uc7a5\uc18c\ub294 \uac74\uc8fc \uc9c0\ub9b0 \uc131 \ubd80\uadfc\uc758 \ud718\ubc1c\ucc9c(\u8f1d\u767c\u5ddd) \uc0c1\ub958\uc5d0 \uc788\ub294 \ubd81\uc0b0\uc131\uc790(\u5317\u5c71\u57ce\u5b50)\uc600\ub2e4. \uadf8\ub7ec\ub098 \uc5ec\uc9c4\uc871\uc758 \ubd80\ub77d\uc740 \ub2e4\uc591\ud588\uace0, 1411\ub144\uc5d0\ub294 \ud5e4\uc774\ub8fd\uac15(\u9ed1\u9f8d\u6c5f) \ud558\ub958\uc5d0 \ub204\ub974\uac04 \ub3c4\uc9c0\ud718\uc0ac\uc0ac(\u5974\u5152\u5e72\u90fd\u6307\u63ee\u4f7f\u53f8)\ub97c \ub450\uc5c8\ub2e4. \uac74\uc8fc\uc704\u00b7\uc6b0\uc790\uc704(\u5140\u8005\u885b)\u00b7\ub204\ub974\uac04\uc704(\u5974\u5152\u5e72\u885b)\ub97c \uc77c\uad04 \ud1b5\uc81c\ud558\uae30 \uc704\ud558\uc5ec \uc601\ub77d\uc81c\ub294 1411\ub144 \ud0dc\uac10(\u592a\u76e3) \uc774\uc2dc\ud558(\u8d64\u5931\u54c8) \ub4f1\uc5d0\uac8c \uba85\ud558\uc5ec \uad70\ubcd1 \uc57d 1,000\uc744 \uc778\uc194\ud558\uace0 25\ucc99\uc758 \uc120\ubc15\uc73c\ub85c \uc479\ud654 \uac15(\u677e\u82b1\u6c5f)\u00b7\ud5e4\uc774\ub8fd\uac15\uc774 \ub9cc\ub098\ub294 \ud558\ub958\uc9c0\uc810\uc5d0 \ud589\uc815\uad00\uccad\uc778 \ub3c4\uc0ac(\u90fd\u53f8)\ub97c \uc124\uce58\ud558\uc5ec 3\uac1c\uc758 \uc5ec\uc9c4 \ubd80\ub77d\uc744 \uac10\uc2dc, \ud1b5\uc81c\ud558\uc600\ub2e4."} {"question": "H.O.T.\uac00 \ucd9c\uc5f0\ud55c SF\uc601\ud654\uc758 \uac10\ub3c5 \uc774\ub984\uc740?", "paragraph": "\ub9c8\uc774\ub124\ud2b8 \ucf54\ub9ac\uc544\uc640 \uc77c\ubcf8\uc758 \uc2dc\ub85c\uad6c\ubbf8\uac00 \uacf5\ub3d9\uc73c\ub85c \uc81c\uc791\ud558\uc600\uace0, \uac10\ub3c5\uc740 \uc77c\ubcf8\uc601\ud654\uc544\uce74\ub370\ubbf8 1\uae30\uc0dd \ucd9c\uc2e0\uc778 \uc7ac\uc77c\ub3d9\ud3ec \ub85c\ub80c\uc2a4 \ub9ac(\uc774\uc6d0\uc11d)\uac00 \ub9e1\uc558\ub2e4. \uc2dc\ub85c\uad6c\ubbf8\uac00 \uc560\ub2c8\uba54\uc774\uc158\uc744 \uc774\uc6a9\ud55c \ud2b9\uc218\ud6a8\uacfc\ub97c, \ub9c8\uc774\ub124\ud2b8 \ucf54\ub9ac\uc544\uac00 \uae30\ud68d \ubc0f \uc5f0\ucd9c\uc744 \ucc45\uc784\uc84c\ub2e4. \uc904\uac70\ub9ac\ub294 2200\ub144 \uc9c0\uad6c\ub300\ud45c H.O.T. 5\uc778\uc774 \uc6b0\uc8fc\uc815\ubcf5\uc758 \uc57c\uc695\uc5d0 \ub9de\uc11c \uc678\uacc4\uc778\uacfc \uc2f8\uc6b0\uba70 \uac70\ub300\ud55c \uc6b0\uc8fc\uc120 \ubaa8\uc591\uc758 \uadf8\ub77c\uc6b4\ub4dc\ub97c \uc9c8\uc8fc\ud558\ub294 \ucd95\uad6c\ud300\uc744 \uadf8\ub9b0 \uac83\uc73c\ub85c \u2018\ud3c9\ud654\u2019\uc758 \uba54\uc2dc\uc9c0\ub97c \uc804\ud558\ub294 \uacf5\uc0c1\uacfc\ud559\uc7a5\ub974\ub2e4.10\ub144\uc774 \uc9c0\ub09c \ud604\uc7ac\uae4c\uc9c0\ub3c4 \ud3ec\ud138 \uc0ac\uc774\ud2b8 \ub124\uc774\ubc84\uc640 \ub2e4\uc74c\uc758 \uad00\uac1d \ud3c9\uac00\uc5d0\uc11c \u300a\ud074\ub808\uba58\ud0c0\uc778\u300b, \u300a\uaf43\ubbf8\ub0a8 \uc5f0\uc1c4 \ud14c\ub7ec\uc0ac\uac74\u300b, \u300a\uae34\uae09\uc870\uce58 19\ud638\u300b \ub4f1\uacfc \ud568\uaed8 \uc5ed\ub300 \uc778\uae30\uac00\uc218 \ud639\uc740 \uc678\uad6d\ubc30\uc6b0\ub97c \uc8fc\uc5f0\uc73c\ub85c \ub300\uc911\uc758 \ud638\uae30\uc2ec\uc740 \ub04c\uc5c8\uc73c\ub098 \ud765\ud589\uc5d0 \ucc38\ud328\ud55c \uc601\ud654\ub4e4\uc758 \ud639\ud3c9\uacfc \ub098\ub780\ud788 \ud68c\uc790\ub418\uace0 \uc788\uc73c\uba70,\uc2e4\ud5d8\uc801 \ub2e8\ud3b8 \uac00\uc871\uc624\ub77d \uc785\uccb4\uc601\ud654\ub85c \uac10\ub3c5\uc744 \ub9e1\uc558\ub358 \ub85c\ub80c\uc2a4 \ub9ac\ub294 \uc774 \uc791\ud488 \ud6c4 \uc740\ud1f4\ud588\ub2e4\uace0 \ud558\ub098, \uc774\ub294 \uc6f9\uc0c1\uc5d0\uc11c \uc774\ud6c4\uc758 \uae30\ub85d\uc744 \ucc3e\uc744 \uc218 \uc5c6\uc5b4 \uc0ac\uc2e4\uc5ec\ubd80\ub97c \uac00\ub2a0\ud558\uae30 \uc5b4\ub824\uc6b4 \ubd80\ubd84\uc774\ub2e4. 70\uc5b5\uc774\ub77c \uc54c\ub824\uc9c4(\ud639\uc740 \ucd1d \ud22c\uc785\ube44 120\uc5b5) \uc81c\uc791\ube44\uc5d0 \ube44\ud574\uc11c \uc9c0\ub098\uce58\uac8c \ucd9c\uc5f0\uc9c4\uc758 \uc778\uc9c0\ub3c4\uc5d0\ub9cc \uc758\uc874\ud55c \ub2e8\uc21c\ud558\uace0 \uac1c\uc5f0\uc131\uc774 \ub5a8\uc5b4\uc9c0\ub294 \uc804\uac1c\uc5d0 \ube44\ud310\uc744 \ubc1b\uc558\uc73c\uba70, 2000\ub144\ub2f9\uc2dc \ucd5c\uace0\uc758 \uc778\uae30\ub97c \ub204\ub838\uc73c\ub098 \ubc30\uc6b0\ub85c\uc11c \uc2e0\uc778\uc778 H.O.T.\uc5d0\uac8c 10\uc5b5\uc774\ub77c\ub294 \ud30c\uaca9\uc801 \ucd9c\uc5f0\ub8cc\ub97c \uc9c0\uae09\ud55c \uac83\uacfc 28\ubd84\uc774\ub77c\ub294 \uc9e7\uc740 \ub7ec\ub2dd\ud0c0\uc784\uc5d0\ub3c4 \ubd88\uad6c\ud558\uace0 \uc601\ud654\uad00\ub78c\ub8cc\uac00 \uc870\uc808\ub418\uc9c0 \uc54a\uc544 \ube44\uc2f8\ub2e4\ub294 \uc9c0\uc801\ub3c4 \uc5ed\uc2dc \uad6c\uc124\uc218\uc5d0 \uc62c\ub790\ub2e4.", "answer": "\ub85c\ub80c\uc2a4 \ub9ac", "sentence": "\ub9c8\uc774\ub124\ud2b8 \ucf54\ub9ac\uc544\uc640 \uc77c\ubcf8\uc758 \uc2dc\ub85c\uad6c\ubbf8\uac00 \uacf5\ub3d9\uc73c\ub85c \uc81c\uc791\ud558\uc600\uace0, \uac10\ub3c5\uc740 \uc77c\ubcf8\uc601\ud654\uc544\uce74\ub370\ubbf8 1\uae30\uc0dd \ucd9c\uc2e0\uc778 \uc7ac\uc77c\ub3d9\ud3ec \ub85c\ub80c\uc2a4 \ub9ac (\uc774\uc6d0\uc11d)\uac00 \ub9e1\uc558\ub2e4.", "paragraph_sentence": " \ub9c8\uc774\ub124\ud2b8 \ucf54\ub9ac\uc544\uc640 \uc77c\ubcf8\uc758 \uc2dc\ub85c\uad6c\ubbf8\uac00 \uacf5\ub3d9\uc73c\ub85c \uc81c\uc791\ud558\uc600\uace0, \uac10\ub3c5\uc740 \uc77c\ubcf8\uc601\ud654\uc544\uce74\ub370\ubbf8 1\uae30\uc0dd \ucd9c\uc2e0\uc778 \uc7ac\uc77c\ub3d9\ud3ec \ub85c\ub80c\uc2a4 \ub9ac (\uc774\uc6d0\uc11d)\uac00 \ub9e1\uc558\ub2e4. \uc2dc\ub85c\uad6c\ubbf8\uac00 \uc560\ub2c8\uba54\uc774\uc158\uc744 \uc774\uc6a9\ud55c \ud2b9\uc218\ud6a8\uacfc\ub97c, \ub9c8\uc774\ub124\ud2b8 \ucf54\ub9ac\uc544\uac00 \uae30\ud68d \ubc0f \uc5f0\ucd9c\uc744 \ucc45\uc784\uc84c\ub2e4. \uc904\uac70\ub9ac\ub294 2200\ub144 \uc9c0\uad6c\ub300\ud45c H.O.T. 5\uc778\uc774 \uc6b0\uc8fc\uc815\ubcf5\uc758 \uc57c\uc695\uc5d0 \ub9de\uc11c \uc678\uacc4\uc778\uacfc \uc2f8\uc6b0\uba70 \uac70\ub300\ud55c \uc6b0\uc8fc\uc120 \ubaa8\uc591\uc758 \uadf8\ub77c\uc6b4\ub4dc\ub97c \uc9c8\uc8fc\ud558\ub294 \ucd95\uad6c\ud300\uc744 \uadf8\ub9b0 \uac83\uc73c\ub85c \u2018\ud3c9\ud654\u2019\uc758 \uba54\uc2dc\uc9c0\ub97c \uc804\ud558\ub294 \uacf5\uc0c1\uacfc\ud559\uc7a5\ub974\ub2e4.10\ub144\uc774 \uc9c0\ub09c \ud604\uc7ac\uae4c\uc9c0\ub3c4 \ud3ec\ud138 \uc0ac\uc774\ud2b8 \ub124\uc774\ubc84\uc640 \ub2e4\uc74c\uc758 \uad00\uac1d \ud3c9\uac00\uc5d0\uc11c \u300a\ud074\ub808\uba58\ud0c0\uc778\u300b, \u300a\uaf43\ubbf8\ub0a8 \uc5f0\uc1c4 \ud14c\ub7ec\uc0ac\uac74\u300b, \u300a\uae34\uae09\uc870\uce58 19\ud638\u300b \ub4f1\uacfc \ud568\uaed8 \uc5ed\ub300 \uc778\uae30\uac00\uc218 \ud639\uc740 \uc678\uad6d\ubc30\uc6b0\ub97c \uc8fc\uc5f0\uc73c\ub85c \ub300\uc911\uc758 \ud638\uae30\uc2ec\uc740 \ub04c\uc5c8\uc73c\ub098 \ud765\ud589\uc5d0 \ucc38\ud328\ud55c \uc601\ud654\ub4e4\uc758 \ud639\ud3c9\uacfc \ub098\ub780\ud788 \ud68c\uc790\ub418\uace0 \uc788\uc73c\uba70,\uc2e4\ud5d8\uc801 \ub2e8\ud3b8 \uac00\uc871\uc624\ub77d \uc785\uccb4\uc601\ud654\ub85c \uac10\ub3c5\uc744 \ub9e1\uc558\ub358 \ub85c\ub80c\uc2a4 \ub9ac\ub294 \uc774 \uc791\ud488 \ud6c4 \uc740\ud1f4\ud588\ub2e4\uace0 \ud558\ub098, \uc774\ub294 \uc6f9\uc0c1\uc5d0\uc11c \uc774\ud6c4\uc758 \uae30\ub85d\uc744 \ucc3e\uc744 \uc218 \uc5c6\uc5b4 \uc0ac\uc2e4\uc5ec\ubd80\ub97c \uac00\ub2a0\ud558\uae30 \uc5b4\ub824\uc6b4 \ubd80\ubd84\uc774\ub2e4. 70\uc5b5\uc774\ub77c \uc54c\ub824\uc9c4(\ud639\uc740 \ucd1d \ud22c\uc785\ube44 120\uc5b5) \uc81c\uc791\ube44\uc5d0 \ube44\ud574\uc11c \uc9c0\ub098\uce58\uac8c \ucd9c\uc5f0\uc9c4\uc758 \uc778\uc9c0\ub3c4\uc5d0\ub9cc \uc758\uc874\ud55c \ub2e8\uc21c\ud558\uace0 \uac1c\uc5f0\uc131\uc774 \ub5a8\uc5b4\uc9c0\ub294 \uc804\uac1c\uc5d0 \ube44\ud310\uc744 \ubc1b\uc558\uc73c\uba70, 2000\ub144\ub2f9\uc2dc \ucd5c\uace0\uc758 \uc778\uae30\ub97c \ub204\ub838\uc73c\ub098 \ubc30\uc6b0\ub85c\uc11c \uc2e0\uc778\uc778 H.O.T.\uc5d0\uac8c 10\uc5b5\uc774\ub77c\ub294 \ud30c\uaca9\uc801 \ucd9c\uc5f0\ub8cc\ub97c \uc9c0\uae09\ud55c \uac83\uacfc 28\ubd84\uc774\ub77c\ub294 \uc9e7\uc740 \ub7ec\ub2dd\ud0c0\uc784\uc5d0\ub3c4 \ubd88\uad6c\ud558\uace0 \uc601\ud654\uad00\ub78c\ub8cc\uac00 \uc870\uc808\ub418\uc9c0 \uc54a\uc544 \ube44\uc2f8\ub2e4\ub294 \uc9c0\uc801\ub3c4 \uc5ed\uc2dc \uad6c\uc124\uc218\uc5d0 \uc62c\ub790\ub2e4.", "paragraph_answer": "\ub9c8\uc774\ub124\ud2b8 \ucf54\ub9ac\uc544\uc640 \uc77c\ubcf8\uc758 \uc2dc\ub85c\uad6c\ubbf8\uac00 \uacf5\ub3d9\uc73c\ub85c \uc81c\uc791\ud558\uc600\uace0, \uac10\ub3c5\uc740 \uc77c\ubcf8\uc601\ud654\uc544\uce74\ub370\ubbf8 1\uae30\uc0dd \ucd9c\uc2e0\uc778 \uc7ac\uc77c\ub3d9\ud3ec \ub85c\ub80c\uc2a4 \ub9ac (\uc774\uc6d0\uc11d)\uac00 \ub9e1\uc558\ub2e4. \uc2dc\ub85c\uad6c\ubbf8\uac00 \uc560\ub2c8\uba54\uc774\uc158\uc744 \uc774\uc6a9\ud55c \ud2b9\uc218\ud6a8\uacfc\ub97c, \ub9c8\uc774\ub124\ud2b8 \ucf54\ub9ac\uc544\uac00 \uae30\ud68d \ubc0f \uc5f0\ucd9c\uc744 \ucc45\uc784\uc84c\ub2e4. \uc904\uac70\ub9ac\ub294 2200\ub144 \uc9c0\uad6c\ub300\ud45c H.O.T. 5\uc778\uc774 \uc6b0\uc8fc\uc815\ubcf5\uc758 \uc57c\uc695\uc5d0 \ub9de\uc11c \uc678\uacc4\uc778\uacfc \uc2f8\uc6b0\uba70 \uac70\ub300\ud55c \uc6b0\uc8fc\uc120 \ubaa8\uc591\uc758 \uadf8\ub77c\uc6b4\ub4dc\ub97c \uc9c8\uc8fc\ud558\ub294 \ucd95\uad6c\ud300\uc744 \uadf8\ub9b0 \uac83\uc73c\ub85c \u2018\ud3c9\ud654\u2019\uc758 \uba54\uc2dc\uc9c0\ub97c \uc804\ud558\ub294 \uacf5\uc0c1\uacfc\ud559\uc7a5\ub974\ub2e4.10\ub144\uc774 \uc9c0\ub09c \ud604\uc7ac\uae4c\uc9c0\ub3c4 \ud3ec\ud138 \uc0ac\uc774\ud2b8 \ub124\uc774\ubc84\uc640 \ub2e4\uc74c\uc758 \uad00\uac1d \ud3c9\uac00\uc5d0\uc11c \u300a\ud074\ub808\uba58\ud0c0\uc778\u300b, \u300a\uaf43\ubbf8\ub0a8 \uc5f0\uc1c4 \ud14c\ub7ec\uc0ac\uac74\u300b, \u300a\uae34\uae09\uc870\uce58 19\ud638\u300b \ub4f1\uacfc \ud568\uaed8 \uc5ed\ub300 \uc778\uae30\uac00\uc218 \ud639\uc740 \uc678\uad6d\ubc30\uc6b0\ub97c \uc8fc\uc5f0\uc73c\ub85c \ub300\uc911\uc758 \ud638\uae30\uc2ec\uc740 \ub04c\uc5c8\uc73c\ub098 \ud765\ud589\uc5d0 \ucc38\ud328\ud55c \uc601\ud654\ub4e4\uc758 \ud639\ud3c9\uacfc \ub098\ub780\ud788 \ud68c\uc790\ub418\uace0 \uc788\uc73c\uba70,\uc2e4\ud5d8\uc801 \ub2e8\ud3b8 \uac00\uc871\uc624\ub77d \uc785\uccb4\uc601\ud654\ub85c \uac10\ub3c5\uc744 \ub9e1\uc558\ub358 \ub85c\ub80c\uc2a4 \ub9ac\ub294 \uc774 \uc791\ud488 \ud6c4 \uc740\ud1f4\ud588\ub2e4\uace0 \ud558\ub098, \uc774\ub294 \uc6f9\uc0c1\uc5d0\uc11c \uc774\ud6c4\uc758 \uae30\ub85d\uc744 \ucc3e\uc744 \uc218 \uc5c6\uc5b4 \uc0ac\uc2e4\uc5ec\ubd80\ub97c \uac00\ub2a0\ud558\uae30 \uc5b4\ub824\uc6b4 \ubd80\ubd84\uc774\ub2e4. 70\uc5b5\uc774\ub77c \uc54c\ub824\uc9c4(\ud639\uc740 \ucd1d \ud22c\uc785\ube44 120\uc5b5) \uc81c\uc791\ube44\uc5d0 \ube44\ud574\uc11c \uc9c0\ub098\uce58\uac8c \ucd9c\uc5f0\uc9c4\uc758 \uc778\uc9c0\ub3c4\uc5d0\ub9cc \uc758\uc874\ud55c \ub2e8\uc21c\ud558\uace0 \uac1c\uc5f0\uc131\uc774 \ub5a8\uc5b4\uc9c0\ub294 \uc804\uac1c\uc5d0 \ube44\ud310\uc744 \ubc1b\uc558\uc73c\uba70, 2000\ub144\ub2f9\uc2dc \ucd5c\uace0\uc758 \uc778\uae30\ub97c \ub204\ub838\uc73c\ub098 \ubc30\uc6b0\ub85c\uc11c \uc2e0\uc778\uc778 H.O.T.\uc5d0\uac8c 10\uc5b5\uc774\ub77c\ub294 \ud30c\uaca9\uc801 \ucd9c\uc5f0\ub8cc\ub97c \uc9c0\uae09\ud55c \uac83\uacfc 28\ubd84\uc774\ub77c\ub294 \uc9e7\uc740 \ub7ec\ub2dd\ud0c0\uc784\uc5d0\ub3c4 \ubd88\uad6c\ud558\uace0 \uc601\ud654\uad00\ub78c\ub8cc\uac00 \uc870\uc808\ub418\uc9c0 \uc54a\uc544 \ube44\uc2f8\ub2e4\ub294 \uc9c0\uc801\ub3c4 \uc5ed\uc2dc \uad6c\uc124\uc218\uc5d0 \uc62c\ub790\ub2e4.", "sentence_answer": "\ub9c8\uc774\ub124\ud2b8 \ucf54\ub9ac\uc544\uc640 \uc77c\ubcf8\uc758 \uc2dc\ub85c\uad6c\ubbf8\uac00 \uacf5\ub3d9\uc73c\ub85c \uc81c\uc791\ud558\uc600\uace0, \uac10\ub3c5\uc740 \uc77c\ubcf8\uc601\ud654\uc544\uce74\ub370\ubbf8 1\uae30\uc0dd \ucd9c\uc2e0\uc778 \uc7ac\uc77c\ub3d9\ud3ec \ub85c\ub80c\uc2a4 \ub9ac (\uc774\uc6d0\uc11d)\uac00 \ub9e1\uc558\ub2e4.", "paragraph_id": "6470348-2-0", "paragraph_question": "question: H.O.T.\uac00 \ucd9c\uc5f0\ud55c SF\uc601\ud654\uc758 \uac10\ub3c5 \uc774\ub984\uc740?, context: \ub9c8\uc774\ub124\ud2b8 \ucf54\ub9ac\uc544\uc640 \uc77c\ubcf8\uc758 \uc2dc\ub85c\uad6c\ubbf8\uac00 \uacf5\ub3d9\uc73c\ub85c \uc81c\uc791\ud558\uc600\uace0, \uac10\ub3c5\uc740 \uc77c\ubcf8\uc601\ud654\uc544\uce74\ub370\ubbf8 1\uae30\uc0dd \ucd9c\uc2e0\uc778 \uc7ac\uc77c\ub3d9\ud3ec \ub85c\ub80c\uc2a4 \ub9ac(\uc774\uc6d0\uc11d)\uac00 \ub9e1\uc558\ub2e4. \uc2dc\ub85c\uad6c\ubbf8\uac00 \uc560\ub2c8\uba54\uc774\uc158\uc744 \uc774\uc6a9\ud55c \ud2b9\uc218\ud6a8\uacfc\ub97c, \ub9c8\uc774\ub124\ud2b8 \ucf54\ub9ac\uc544\uac00 \uae30\ud68d \ubc0f \uc5f0\ucd9c\uc744 \ucc45\uc784\uc84c\ub2e4. \uc904\uac70\ub9ac\ub294 2200\ub144 \uc9c0\uad6c\ub300\ud45c H.O.T. 5\uc778\uc774 \uc6b0\uc8fc\uc815\ubcf5\uc758 \uc57c\uc695\uc5d0 \ub9de\uc11c \uc678\uacc4\uc778\uacfc \uc2f8\uc6b0\uba70 \uac70\ub300\ud55c \uc6b0\uc8fc\uc120 \ubaa8\uc591\uc758 \uadf8\ub77c\uc6b4\ub4dc\ub97c \uc9c8\uc8fc\ud558\ub294 \ucd95\uad6c\ud300\uc744 \uadf8\ub9b0 \uac83\uc73c\ub85c \u2018\ud3c9\ud654\u2019\uc758 \uba54\uc2dc\uc9c0\ub97c \uc804\ud558\ub294 \uacf5\uc0c1\uacfc\ud559\uc7a5\ub974\ub2e4.10\ub144\uc774 \uc9c0\ub09c \ud604\uc7ac\uae4c\uc9c0\ub3c4 \ud3ec\ud138 \uc0ac\uc774\ud2b8 \ub124\uc774\ubc84\uc640 \ub2e4\uc74c\uc758 \uad00\uac1d \ud3c9\uac00\uc5d0\uc11c \u300a\ud074\ub808\uba58\ud0c0\uc778\u300b, \u300a\uaf43\ubbf8\ub0a8 \uc5f0\uc1c4 \ud14c\ub7ec\uc0ac\uac74\u300b, \u300a\uae34\uae09\uc870\uce58 19\ud638\u300b \ub4f1\uacfc \ud568\uaed8 \uc5ed\ub300 \uc778\uae30\uac00\uc218 \ud639\uc740 \uc678\uad6d\ubc30\uc6b0\ub97c \uc8fc\uc5f0\uc73c\ub85c \ub300\uc911\uc758 \ud638\uae30\uc2ec\uc740 \ub04c\uc5c8\uc73c\ub098 \ud765\ud589\uc5d0 \ucc38\ud328\ud55c \uc601\ud654\ub4e4\uc758 \ud639\ud3c9\uacfc \ub098\ub780\ud788 \ud68c\uc790\ub418\uace0 \uc788\uc73c\uba70,\uc2e4\ud5d8\uc801 \ub2e8\ud3b8 \uac00\uc871\uc624\ub77d \uc785\uccb4\uc601\ud654\ub85c \uac10\ub3c5\uc744 \ub9e1\uc558\ub358 \ub85c\ub80c\uc2a4 \ub9ac\ub294 \uc774 \uc791\ud488 \ud6c4 \uc740\ud1f4\ud588\ub2e4\uace0 \ud558\ub098, \uc774\ub294 \uc6f9\uc0c1\uc5d0\uc11c \uc774\ud6c4\uc758 \uae30\ub85d\uc744 \ucc3e\uc744 \uc218 \uc5c6\uc5b4 \uc0ac\uc2e4\uc5ec\ubd80\ub97c \uac00\ub2a0\ud558\uae30 \uc5b4\ub824\uc6b4 \ubd80\ubd84\uc774\ub2e4. 70\uc5b5\uc774\ub77c \uc54c\ub824\uc9c4(\ud639\uc740 \ucd1d \ud22c\uc785\ube44 120\uc5b5) \uc81c\uc791\ube44\uc5d0 \ube44\ud574\uc11c \uc9c0\ub098\uce58\uac8c \ucd9c\uc5f0\uc9c4\uc758 \uc778\uc9c0\ub3c4\uc5d0\ub9cc \uc758\uc874\ud55c \ub2e8\uc21c\ud558\uace0 \uac1c\uc5f0\uc131\uc774 \ub5a8\uc5b4\uc9c0\ub294 \uc804\uac1c\uc5d0 \ube44\ud310\uc744 \ubc1b\uc558\uc73c\uba70, 2000\ub144\ub2f9\uc2dc \ucd5c\uace0\uc758 \uc778\uae30\ub97c \ub204\ub838\uc73c\ub098 \ubc30\uc6b0\ub85c\uc11c \uc2e0\uc778\uc778 H.O.T.\uc5d0\uac8c 10\uc5b5\uc774\ub77c\ub294 \ud30c\uaca9\uc801 \ucd9c\uc5f0\ub8cc\ub97c \uc9c0\uae09\ud55c \uac83\uacfc 28\ubd84\uc774\ub77c\ub294 \uc9e7\uc740 \ub7ec\ub2dd\ud0c0\uc784\uc5d0\ub3c4 \ubd88\uad6c\ud558\uace0 \uc601\ud654\uad00\ub78c\ub8cc\uac00 \uc870\uc808\ub418\uc9c0 \uc54a\uc544 \ube44\uc2f8\ub2e4\ub294 \uc9c0\uc801\ub3c4 \uc5ed\uc2dc \uad6c\uc124\uc218\uc5d0 \uc62c\ub790\ub2e4."}